diff --git a/IMO/md/en-IMO2006SL.md b/IMO/md/en-IMO2006SL.md new file mode 100644 index 0000000000000000000000000000000000000000..ab894f26c5b9927f333f01be25c9df216d4591c1 --- /dev/null +++ b/IMO/md/en-IMO2006SL.md @@ -0,0 +1,1898 @@ +![](https://cdn.mathpix.com/cropped/2024_04_17_e55ccc62f43b046023cag-01.jpg?height=556&width=1748&top_left_y=156&top_left_x=154) + +Shortlisted problems with solutions +$47^{\text {th }}$ International Mathematical Olympiad Slovenia 2006 + +## Shortlisted Problems with Solutions + +## Contents + +Contributing Countries \& Problem Selection Committee ..... 5 +Algebra ..... 7 +Problem A1 ..... 7 +Problem A2 ..... 9 +Problem A3 ..... 10 +Problem A4 ..... 13 +Problem A5 ..... 15 +Problem A6 ..... 17 +Combinatorics ..... 19 +Problem C1 ..... 19 +Problem C2 ..... 21 +Problem C3 ..... 23 +Problem C4 ..... 25 +Problem C5 ..... 27 +Problem C6 ..... 29 +Problem C7 ..... 31 +Geometry ..... 35 +Problem G1 ..... 35 +Problem G2 ..... 36 +Problem G3 ..... 38 +Problem G4 ..... 39 +Problem G5 ..... 40 +Problem G6 ..... 42 +Problem G7 ..... 45 +Problem G8 ..... 46 +Problem G9 ..... 48 +Problem G10 ..... 51 +Number Theory ..... 55 +Problem N1 ..... 55 +Problem N2 ..... 56 +Problem N3 ..... 57 +Problem N4 ..... 58 +Problem N5 ..... 59 +Problem N6 ..... 60 +Problem N7 ..... 63 + +# Contributing Countries + +Argentina, Australia, Brazil, Bulgaria, Canada, Colombia,
Czech Republic, Estonia, Finland, France, Georgia, Greece,
Hong Kong, India, Indonesia, Iran, Ireland, Italy, Japan,
Republic of Korea, Luxembourg, Netherlands, Poland, Peru,
Romania, Russia, Serbia and Montenegro, Singapore, Slovakia,
South Africa, Sweden, Taiwan, Ukraine, United Kingdom,
United States of America, Venezuela + +## Problem Selection Committee + +Andrej Bauer
Robert Geretschläger
Géza Kós
Marcin Kuczma
Svetoslav Savchev + +## Algebra + +A1. A sequence of real numbers $a_{0}, a_{1}, a_{2}, \ldots$ is defined by the formula + +$$ +a_{i+1}=\left\lfloor a_{i}\right\rfloor \cdot\left\langle a_{i}\right\rangle \quad \text { for } \quad i \geq 0 \text {; } +$$ + +here $a_{0}$ is an arbitrary real number, $\left\lfloor a_{i}\right\rfloor$ denotes the greatest integer not exceeding $a_{i}$, and $\left\langle a_{i}\right\rangle=a_{i}-\left\lfloor a_{i}\right\rfloor$. Prove that $a_{i}=a_{i+2}$ for $i$ sufficiently large. + +(Estonia) + +Solution. First note that if $a_{0} \geq 0$, then all $a_{i} \geq 0$. For $a_{i} \geq 1$ we have (in view of $\left\langle a_{i}\right\rangle<1$ and $\left\lfloor a_{i}\right\rfloor>0$ ) + +$$ +\left\lfloor a_{i+1}\right\rfloor \leq a_{i+1}=\left\lfloor a_{i}\right\rfloor \cdot\left\langle a_{i}\right\rangle<\left\lfloor a_{i}\right\rfloor +$$ + +the sequence $\left\lfloor a_{i}\right\rfloor$ is strictly decreasing as long as its terms are in $[1, \infty)$. Eventually there appears a number from the interval $[0,1)$ and all subsequent terms are 0 . + +Now pass to the more interesting situation where $a_{0}<0$; then all $a_{i} \leq 0$. Suppose the sequence never hits 0 . Then we have $\left\lfloor a_{i}\right\rfloor \leq-1$ for all $i$, and so + +$$ +1+\left\lfloor a_{i+1}\right\rfloor>a_{i+1}=\left\lfloor a_{i}\right\rfloor \cdot\left\langle a_{i}\right\rangle>\left\lfloor a_{i}\right\rfloor +$$ + +this means that the sequence $\left\lfloor a_{i}\right\rfloor$ is nondecreasing. And since all its terms are integers from $(-\infty,-1]$, this sequence must be constant from some term on: + +$$ +\left\lfloor a_{i}\right\rfloor=c \quad \text { for } \quad i \geq i_{0} ; \quad c \text { a negative integer. } +$$ + +The defining formula becomes + +$$ +a_{i+1}=c \cdot\left\langle a_{i}\right\rangle=c\left(a_{i}-c\right)=c a_{i}-c^{2} . +$$ + +Consider the sequence + +$$ +b_{i}=a_{i}-\frac{c^{2}}{c-1} +$$ + +It satisfies the recursion rule + +$$ +b_{i+1}=a_{i+1}-\frac{c^{2}}{c-1}=c a_{i}-c^{2}-\frac{c^{2}}{c-1}=c b_{i} +$$ + +implying + +$$ +b_{i}=c^{i-i_{0}} b_{i_{0}} \quad \text { for } \quad i \geq i_{0} . +$$ + +Since all the numbers $a_{i}$ (for $i \geq i_{0}$ ) lie in $\left[c, c+1\right.$ ), the sequence $\left(b_{i}\right)$ is bounded. The equation (2) can be satisfied only if either $b_{i_{0}}=0$ or $|c|=1$, i.e., $c=-1$. + +In the first case, $b_{i}=0$ for all $i \geq i_{0}$, so that + +$$ +a_{i}=\frac{c^{2}}{c-1} \quad \text { for } \quad i \geq i_{0} +$$ + +In the second case, $c=-1$, equations (1) and (2) say that + +$$ +a_{i}=-\frac{1}{2}+(-1)^{i-i_{0}} b_{i_{0}}= \begin{cases}a_{i_{0}} & \text { for } i=i_{0}, i_{0}+2, i_{0}+4, \ldots \\ 1-a_{i_{0}} & \text { for } i=i_{0}+1, i_{0}+3, i_{0}+5, \ldots\end{cases} +$$ + +Summarising, we see that (from some point on) the sequence $\left(a_{i}\right)$ either is constant or takes alternately two values from the interval $(-1,0)$. The result follows. + +Comment. There is nothing mysterious in introducing the sequence $\left(b_{i}\right)$. The sequence $\left(a_{i}\right)$ arises by iterating the function $x \mapsto c x-c^{2}$ whose unique fixed point is $c^{2} /(c-1)$. + +A2. The sequence of real numbers $a_{0}, a_{1}, a_{2}, \ldots$ is defined recursively by + +$$ +a_{0}=-1, \quad \sum_{k=0}^{n} \frac{a_{n-k}}{k+1}=0 \quad \text { for } \quad n \geq 1 +$$ + +Show that $a_{n}>0$ for $n \geq 1$. + +(Poland) + +Solution. The proof goes by induction. For $n=1$ the formula yields $a_{1}=1 / 2$. Take $n \geq 1$, assume $a_{1}, \ldots, a_{n}>0$ and write the recurrence formula for $n$ and $n+1$, respectively as + +$$ +\sum_{k=0}^{n} \frac{a_{k}}{n-k+1}=0 \quad \text { and } \quad \sum_{k=0}^{n+1} \frac{a_{k}}{n-k+2}=0 +$$ + +Subtraction yields + +$$ +\begin{aligned} +0=(n+2) \sum_{k=0}^{n+1} & \frac{a_{k}}{n-k+2}-(n+1) \sum_{k=0}^{n} \frac{a_{k}}{n-k+1} \\ += & (n+2) a_{n+1}+\sum_{k=0}^{n}\left(\frac{n+2}{n-k+2}-\frac{n+1}{n-k+1}\right) a_{k} . +\end{aligned} +$$ + +The coefficient of $a_{0}$ vanishes, so + +$$ +a_{n+1}=\frac{1}{n+2} \sum_{k=1}^{n}\left(\frac{n+1}{n-k+1}-\frac{n+2}{n-k+2}\right) a_{k}=\frac{1}{n+2} \sum_{k=1}^{n} \frac{k}{(n-k+1)(n-k+2)} a_{k} . +$$ + +The coefficients of $a_{1}, \ldots, a_{n}$ are all positive. Therefore, $a_{1}, \ldots, a_{n}>0$ implies $a_{n+1}>0$. + +Comment. Students familiar with the technique of generating functions will immediately recognise $\sum a_{n} x^{n}$ as the power series expansion of $x / \ln (1-x)$ (with value -1 at 0 ). But this can be a trap; attempts along these lines lead to unpleasant differential equations and integrals hard to handle. Using only tools from real analysis (e.g. computing the coefficients from the derivatives) seems very difficult. + +On the other hand, the coefficients can be approached applying complex contour integrals and some other techniques from complex analysis and an attractive formula can be obtained for the coefficients: + +$$ +a_{n}=\int_{1}^{\infty} \frac{\mathrm{d} x}{x^{n}\left(\pi^{2}+\log ^{2}(x-1)\right)} \quad(n \geq 1) +$$ + +which is evidently positive. + +A3. The sequence $c_{0}, c_{1}, \ldots, c_{n}, \ldots$ is defined by $c_{0}=1, c_{1}=0$ and $c_{n+2}=c_{n+1}+c_{n}$ for $n \geq 0$. Consider the set $S$ of ordered pairs $(x, y)$ for which there is a finite set $J$ of positive integers such that $x=\sum_{j \in J} c_{j}, y=\sum_{j \in J} c_{j-1}$. Prove that there exist real numbers $\alpha, \beta$ and $m, M$ with the following property: An ordered pair of nonnegative integers $(x, y)$ satisfies the inequality + +$$ +m<\alpha x+\beta y1$ and $-1<\psi<0$, this implies $\alpha \varphi+\beta=0$. + +To satisfy $\alpha \varphi+\beta=0$, one can set for instance $\alpha=\psi, \beta=1$. We now find the required $m$ and $M$ for this choice of $\alpha$ and $\beta$. + +Note first that the above displayed equation gives $c_{n} \psi+c_{n-1}=\psi^{n-1}, n \geq 1$. In the sequel, we denote the pairs in $S$ by $\left(a_{J}, b_{J}\right)$, where $J$ is a finite subset of the set $\mathbb{N}$ of positive integers and $a_{J}=\sum_{j \in J} c_{j}, b_{J}=\sum_{j \in J} c_{j-1}$. Since $\psi a_{J}+b_{J}=\sum_{j \in J}\left(c_{j} \psi+c_{j-1}\right)$, we obtain + +$$ +\psi a_{J}+b_{J}=\sum_{j \in J} \psi^{j-1} \quad \text { for each }\left(a_{J}, b_{J}\right) \in S +$$ + +On the other hand, in view of $-1<\psi<0$, + +$$ +-1=\frac{\psi}{1-\psi^{2}}=\sum_{j=0}^{\infty} \psi^{2 j+1}<\sum_{j \in J} \psi^{j-1}<\sum_{j=0}^{\infty} \psi^{2 j}=\frac{1}{1-\psi^{2}}=1-\psi=\varphi +$$ + +Therefore, according to (1), + +$$ +-1<\psi a_{J}+b_{J}<\varphi \quad \text { for each }\left(a_{J}, b_{J}\right) \in S +$$ + +Thus $m=-1$ and $M=\varphi$ is an appropriate choice. + +Conversely, we prove that if an ordered pair of nonnegative integers $(x, y)$ satisfies the inequality $-1<\psi x+y<\varphi$ then $(x, y) \in S$. + +Lemma. Let $x, y$ be nonnegative integers such that $-1<\psi x+y<\varphi$. Then there exists a subset $J$ of $\mathbb{N}$ such that + +$$ +\psi x+y=\sum_{j \in J} \psi^{j-1} +$$ + +Proof. For $x=y=0$ it suffices to choose the empty subset of $\mathbb{N}$ as $J$, so let at least one of $x, y$ be nonzero. There exist representations of $\psi x+y$ of the form + +$$ +\psi x+y=\psi^{i_{1}}+\cdots+\psi^{i_{k}} +$$ + +where $i_{1} \leq \cdots \leq i_{k}$ is a sequence of nonnegative integers, not necessarily distinct. For instance, we can take $x$ summands $\psi^{1}=\psi$ and $y$ summands $\psi^{0}=1$. Consider all such representations of minimum length $k$ and focus on the ones for which $i_{1}$ has the minimum possible value $j_{1}$. Among them, consider the representations where $i_{2}$ has the minimum possible value $j_{2}$. Upon choosing $j_{3}, \ldots, j_{k}$ analogously, we obtain a sequence $j_{1} \leq \cdots \leq j_{k}$ which clearly satisfies $\psi x+y=\sum_{r=1}^{k} \psi^{j_{r}}$. To prove the lemma, it suffices to show that $j_{1}, \ldots, j_{k}$ are pairwise distinct. + +Suppose on the contrary that $j_{r}=j_{r+1}$ for some $r=1, \ldots, k-1$. Let us consider the case $j_{r} \geq 2$ first. Observing that $2 \psi^{2}=1+\psi^{3}$, we replace $j_{r}$ and $j_{r+1}$ by $j_{r}-2$ and $j_{r}+1$, respectively. Since + +$$ +\psi^{j_{r}}+\psi^{j_{r+1}}=2 \psi^{j_{r}}=\psi^{j_{r}-2}\left(1+\psi^{3}\right)=\psi^{j_{r}-2}+\psi^{j_{r}+1} +$$ + +the new sequence also represents $\psi x+y$ as needed, and the value of $i_{r}$ in it contradicts the minimum choice of $j_{r}$. + +Let $j_{r}=j_{r+1}=0$. Then the sum $\psi x+y=\sum_{r=1}^{k} \psi^{j_{r}}$ contains at least two summands equal to $\psi^{0}=1$. On the other hand $j_{s} \neq 1$ for all $s$, because the equality $1+\psi=\psi^{2}$ implies that a representation of minimum length cannot contain consecutive $i_{r}$ 's. It follows that + +$$ +\psi x+y=\sum_{r=1}^{k} \psi^{j_{r}}>2+\psi^{3}+\psi^{5}+\psi^{7}+\cdots=2-\psi^{2}=\varphi +$$ + +contradicting the condition of the lemma. + +Let $j_{r}=j_{r+1}=1$; then $\sum_{r=1}^{k} \psi^{j_{r}}$ contains at least two summands equal to $\psi^{1}=\psi$. Like in the case $j_{r}=j_{r+1}=0$, we also infer that $j_{s} \neq 0$ and $j_{s} \neq 2$ for all $s$. Therefore + +$$ +\psi x+y=\sum_{r=1}^{k} \psi^{j_{r}}<2 \psi+\psi^{4}+\psi^{6}+\psi^{8}+\cdots=2 \psi-\psi^{3}=-1 +$$ + +which is a contradiction again. The conclusion follows. + +Now let the ordered pair $(x, y)$ satisfy $-1<\psi x+y<\varphi$; hence the lemma applies to $(x, y)$. Let $J \subset \mathbb{N}$ be such that (2) holds. Comparing (1) and (2), we conclude that $\psi x+y=\psi a_{J}+b_{J}$. Now, $x, y, a_{J}$ and $b_{J}$ are integers, and $\psi$ is irrational. So the last equality implies $x=a_{J}$ and $y=b_{J}$. This shows that the numbers $\alpha=\psi, \beta=1, m=-1, M=\varphi$ meet the requirements. + +Comment. We present another way to prove the lemma, constructing the set $J$ inductively. For $x=y=0$, choose $J=\emptyset$. We induct on $n=3 x+2 y$. Suppose that an appropriate set $J$ exists when $3 x+2 y0$. The current set $J$ should be + +$$ +\text { either } 1 \leq j_{1}\psi>-1$; therefore $x^{\prime}, y^{\prime} \geq 0$. Moreover, we have $3 x^{\prime}+2 y^{\prime}=2 x+y \leq \frac{2}{3} n$; therefore, if (3) holds then the induction applies: the numbers $x^{\prime}, y^{\prime}$ are represented in the form as needed, hence $x, y$ also. + +Now consider $\frac{\psi x+y-1}{\psi}$. Since + +$$ +\frac{\psi x+y-1}{\psi}=x+(\psi-1)(y-1)=\psi(y-1)+(x-y+1) +$$ + +we set $x^{\prime}=y-1$ and $y^{\prime}=x-y+1$. Again we require that $\frac{\psi x+y-1}{\psi} \in(-1, \varphi)$, i.e. + +$$ +\psi x+y \in(\varphi \cdot \psi+1,(-1) \cdot \psi+1)=(0, \varphi) . +$$ + +If (4) holds then $y-1 \geq \psi x+y-1>-1$ and $x-y+1 \geq-\psi x-y+1>-\varphi+1>-1$, therefore $x^{\prime}, y^{\prime} \geq 0$. Moreover, $3 x^{\prime}+2 y^{\prime}=2 x+y-1<\frac{2}{3} n$ and the induction works. + +Finally, $(-1,-\psi) \cup(0, \varphi)=(-1, \varphi)$ so at least one of (3) and (4) holds and the induction step is justified. + +A4. Prove the inequality + +$$ +\sum_{i0$ (which can jointly occur as values of these symmetric forms). Suppose that among the numbers $a_{i}$ there are some three, say $a_{k}, a_{l}, a_{m}$ such that $a_{k}\sqrt{a+b}>\sqrt{c}$. + +Let $x=\sqrt{b}+\sqrt{c}-\sqrt{a}, y=\sqrt{c}+\sqrt{a}-\sqrt{b}$ and $z=\sqrt{a}+\sqrt{b}-\sqrt{c}$. Then + +$b+c-a=\left(\frac{z+x}{2}\right)^{2}+\left(\frac{x+y}{2}\right)^{2}-\left(\frac{y+z}{2}\right)^{2}=\frac{x^{2}+x y+x z-y z}{2}=x^{2}-\frac{1}{2}(x-y)(x-z)$ + +and + +$$ +\frac{\sqrt{b+c-a}}{\sqrt{b}+\sqrt{c}-\sqrt{a}}=\sqrt{1-\frac{(x-y)(x-z)}{2 x^{2}}} \leq 1-\frac{(x-y)(x-z)}{4 x^{2}} +$$ + +applying $\sqrt{1+2 u} \leq 1+u$ in the last step. Similarly we obtain + +$$ +\frac{\sqrt{c+a-b}}{\sqrt{c}+\sqrt{a}-\sqrt{b}} \leq 1-\frac{(z-x)(z-y)}{4 z^{2}} \quad \text { and } \quad \frac{\sqrt{a+b-c}}{\sqrt{a}+\sqrt{b}-\sqrt{c}} \leq 1-\frac{(y-z)(y-x)}{4 y^{2}} +$$ + +Substituting these quantities into the statement, it is sufficient to prove that + +$$ +\frac{(x-y)(x-z)}{x^{2}}+\frac{(y-z)(y-x)}{y^{2}}+\frac{(z-x)(z-y)}{z^{2}} \geq 0 . +$$ + +By symmetry we can assume $x \leq y \leq z$. Then + +$$ +\begin{gathered} +\frac{(x-y)(x-z)}{x^{2}}=\frac{(y-x)(z-x)}{x^{2}} \geq \frac{(y-x)(z-y)}{y^{2}}=-\frac{(y-z)(y-x)}{y^{2}} \\ +\frac{(z-x)(z-y)}{z^{2}} \geq 0 +\end{gathered} +$$ + +and (1) follows. + +Comment 1. Inequality (1) is a special case of the well-known inequality + +$$ +x^{t}(x-y)(x-z)+y^{t}(y-z)(y-x)+z^{t}(z-x)(z-y) \geq 0 +$$ + +which holds for all positive numbers $x, y, z$ and real $t$; in our case $t=-2$. Case $t>0$ is called Schur's inequality. More generally, if $x \leq y \leq z$ are real numbers and $p, q, r$ are nonnegative numbers such that $q \leq p$ or $q \leq r$ then + +$$ +p(x-y)(x-z)+q(y-z)(y-x)+r(z-x)(z-y) \geq 0 \text {. } +$$ + +Comment 2. One might also start using Cauchy-Schwarz' inequality (or the root mean square vs. arithmetic mean inequality) to the effect that + +$$ +\left(\sum \frac{\sqrt{b+c-a}}{\sqrt{b}+\sqrt{c}-\sqrt{a}}\right)^{2} \leq 3 \cdot \sum \frac{b+c-a}{(\sqrt{b}+\sqrt{c}-\sqrt{a})^{2}} +$$ + +in cyclic sum notation. There are several ways to prove that the right-hand side of (2) never exceeds 9 (and this is just what we need). One of them is to introduce new variables $x, y, z$, as in Solution 1, which upon some manipulation brings the problem again to inequality (1). + +Alternatively, the claim that right-hand side of (2) is not greater than 9 can be expressed in terms of the symmetric forms $\sigma_{1}=\sum x, \sigma_{2}=\sum x y, \sigma_{3}=x y z$ equivalently as + +$$ +4 \sigma_{1} \sigma_{2} \sigma_{3} \leq \sigma_{2}^{3}+9 \sigma_{3}^{2} +$$ + +which is a known inequality. A yet different method to deal with the right-hand expression in (2) is to consider $\sqrt{a}, \sqrt{b}, \sqrt{c}$ as sides of a triangle. Through standard trigonometric formulas the problem comes down to showing that + +$$ +p^{2} \leq 4 R^{2}+4 R r+3 r^{2} +$$ + +$p, R$ and $r$ standing for the semiperimeter, the circumradius and the inradius of that triangle. Again, (4) is another known inequality. Note that the inequalities (1), (3), (4) are equivalent statements about the same mathematical situation. + +Solution 2. Due to the symmetry of variables, it can be assumed that $a \geq b \geq c$. We claim that + +$$ +\frac{\sqrt{a+b-c}}{\sqrt{a}+\sqrt{b}-\sqrt{c}} \leq 1 \quad \text { and } \quad \frac{\sqrt{b+c-a}}{\sqrt{b}+\sqrt{c}-\sqrt{a}}+\frac{\sqrt{c+a-b}}{\sqrt{c}+\sqrt{a}-\sqrt{b}} \leq 2 . +$$ + +The first inequality follows from + +$$ +\sqrt{a+b-c}-\sqrt{a}=\frac{(a+b-c)-a}{\sqrt{a+b-c}+\sqrt{a}} \leq \frac{b-c}{\sqrt{b}+\sqrt{c}}=\sqrt{b}-\sqrt{c} . +$$ + +For proving the second inequality, let $p=\sqrt{a}+\sqrt{b}$ and $q=\sqrt{a}-\sqrt{b}$. Then $a-b=p q$ and the inequality becomes + +$$ +\frac{\sqrt{c-p q}}{\sqrt{c}-q}+\frac{\sqrt{c+p q}}{\sqrt{c}+q} \leq 2 +$$ + +From $a \geq b \geq c$ we have $p \geq 2 \sqrt{c}$. Applying the Cauchy-Schwarz inequality, + +$$ +\begin{gathered} +\left(\frac{\sqrt{c-p q}}{\sqrt{c}-q}+\frac{\sqrt{c+p q}}{\sqrt{c}+q}\right)^{2} \leq\left(\frac{c-p q}{\sqrt{c}-q}+\frac{c+p q}{\sqrt{c}+q}\right)\left(\frac{1}{\sqrt{c}-q}+\frac{1}{\sqrt{c}+q}\right) \\ +=\frac{2\left(c \sqrt{c}-p q^{2}\right)}{c-q^{2}} \cdot \frac{2 \sqrt{c}}{c-q^{2}}=4 \cdot \frac{c^{2}-\sqrt{c} p q^{2}}{\left(c-q^{2}\right)^{2}} \leq 4 \cdot \frac{c^{2}-2 c q^{2}}{\left(c-q^{2}\right)^{2}} \leq 4 . +\end{gathered} +$$ + +A6. Determine the smallest number $M$ such that the inequality + +$$ +\left|a b\left(a^{2}-b^{2}\right)+b c\left(b^{2}-c^{2}\right)+c a\left(c^{2}-a^{2}\right)\right| \leq M\left(a^{2}+b^{2}+c^{2}\right)^{2} +$$ + +holds for all real numbers $a, b, c$. + +(Ireland) + +Solution. We first consider the cubic polynomial + +$$ +P(t)=t b\left(t^{2}-b^{2}\right)+b c\left(b^{2}-c^{2}\right)+c t\left(c^{2}-t^{2}\right) . +$$ + +It is easy to check that $P(b)=P(c)=P(-b-c)=0$, and therefore + +$$ +P(t)=(b-c)(t-b)(t-c)(t+b+c), +$$ + +since the cubic coefficient is $b-c$. The left-hand side of the proposed inequality can therefore be written in the form + +$$ +\left|a b\left(a^{2}-b^{2}\right)+b c\left(b^{2}-c^{2}\right)+c a\left(c^{2}-a^{2}\right)\right|=|P(a)|=|(b-c)(a-b)(a-c)(a+b+c)| . +$$ + +The problem comes down to finding the smallest number $M$ that satisfies the inequality + +$$ +|(b-c)(a-b)(a-c)(a+b+c)| \leq M \cdot\left(a^{2}+b^{2}+c^{2}\right)^{2} \text {. } +$$ + +Note that this expression is symmetric, and we can therefore assume $a \leq b \leq c$ without loss of generality. With this assumption, + +$$ +|(a-b)(b-c)|=(b-a)(c-b) \leq\left(\frac{(b-a)+(c-b)}{2}\right)^{2}=\frac{(c-a)^{2}}{4} +$$ + +with equality if and only if $b-a=c-b$, i.e. $2 b=a+c$. Also + +$$ +\left(\frac{(c-b)+(b-a)}{2}\right)^{2} \leq \frac{(c-b)^{2}+(b-a)^{2}}{2} +$$ + +or equivalently, + +$$ +3(c-a)^{2} \leq 2 \cdot\left[(b-a)^{2}+(c-b)^{2}+(c-a)^{2}\right], +$$ + +again with equality only for $2 b=a+c$. From (2) and (3) we get + +$$ +\begin{aligned} +& |(b-c)(a-b)(a-c)(a+b+c)| \\ +\leq & \frac{1}{4} \cdot\left|(c-a)^{3}(a+b+c)\right| \\ += & \frac{1}{4} \cdot \sqrt{(c-a)^{6}(a+b+c)^{2}} \\ +\leq & \frac{1}{4} \cdot \sqrt{\left(\frac{2 \cdot\left[(b-a)^{2}+(c-b)^{2}+(c-a)^{2}\right]}{3}\right)^{3} \cdot(a+b+c)^{2}} \\ += & \frac{\sqrt{2}}{2} \cdot\left(\sqrt[4]{\left(\frac{(b-a)^{2}+(c-b)^{2}+(c-a)^{2}}{3}\right)^{3} \cdot(a+b+c)^{2}}\right)^{2} . +\end{aligned} +$$ + +By the weighted AM-GM inequality this estimate continues as follows: + +$$ +\begin{aligned} +& |(b-c)(a-b)(a-c)(a+b+c)| \\ +\leq & \frac{\sqrt{2}}{2} \cdot\left(\frac{(b-a)^{2}+(c-b)^{2}+(c-a)^{2}+(a+b+c)^{2}}{4}\right)^{2} \\ += & \frac{9 \sqrt{2}}{32} \cdot\left(a^{2}+b^{2}+c^{2}\right)^{2} . +\end{aligned} +$$ + +We see that the inequality (1) is satisfied for $M=\frac{9}{32} \sqrt{2}$, with equality if and only if $2 b=a+c$ and + +$$ +\frac{(b-a)^{2}+(c-b)^{2}+(c-a)^{2}}{3}=(a+b+c)^{2} +$$ + +Plugging $b=(a+c) / 2$ into the last equation, we bring it to the equivalent form + +$$ +2(c-a)^{2}=9(a+c)^{2} . +$$ + +The conditions for equality can now be restated as + +$$ +2 b=a+c \quad \text { and } \quad(c-a)^{2}=18 b^{2} \text {. } +$$ + +Setting $b=1$ yields $a=1-\frac{3}{2} \sqrt{2}$ and $c=1+\frac{3}{2} \sqrt{2}$. We see that $M=\frac{9}{32} \sqrt{2}$ is indeed the smallest constant satisfying the inequality, with equality for any triple $(a, b, c)$ proportional to $\left(1-\frac{3}{2} \sqrt{2}, 1,1+\frac{3}{2} \sqrt{2}\right)$, up to permutation. + +Comment. With the notation $x=b-a, y=c-b, z=a-c, s=a+b+c$ and $r^{2}=a^{2}+b^{2}+c^{2}$, the inequality (1) becomes just $|s x y z| \leq M r^{4}$ (with suitable constraints on $s$ and $r$ ). The original asymmetric inequality turns into a standard symmetric one; from this point on the solution can be completed in many ways. One can e.g. use the fact that, for fixed values of $\sum x$ and $\sum x^{2}$, the product $x y z$ is a maximum/minimum only if some of $x, y, z$ are equal, thus reducing one degree of freedom, etc. + +As observed by the proposer, a specific attraction of the problem is that the maximum is attained at a point $(a, b, c)$ with all coordinates distinct. + +## Combinatorics + +C1. We have $n \geq 2$ lamps $L_{1}, \ldots, L_{n}$ in a row, each of them being either on or off. Every second we simultaneously modify the state of each lamp as follows: + +- if the lamp $L_{i}$ and its neighbours (only one neighbour for $i=1$ or $i=n$, two neighbours for other $i$ ) are in the same state, then $L_{i}$ is switched off; +- otherwise, $L_{i}$ is switched on. + +Initially all the lamps are off except the leftmost one which is on. + +(a) Prove that there are infinitely many integers $n$ for which all the lamps will eventually be off. + +(b) Prove that there are infinitely many integers $n$ for which the lamps will never be all off. + +(France) + +Solution. (a) Experiments with small $n$ lead to the guess that every $n$ of the form $2^{k}$ should be good. This is indeed the case, and more precisely: let $A_{k}$ be the $2^{k} \times 2^{k}$ matrix whose rows represent the evolution of the system, with entries 0,1 (for off and on respectively). The top row shows the initial state $[1,0,0, \ldots, 0]$; the bottom row shows the state after $2^{k}-1$ steps. The claim is that: + +$$ +\text { The bottom row of } A_{k} \text { is }[1,1,1, \ldots, 1] \text {. } +$$ + +This will of course suffice because one more move then produces $[0,0,0, \ldots, 0]$, as required. + +The proof is by induction on $k$. The base $k=1$ is obvious. Assume the claim to be true for a $k \geq 1$ and write the matrix $A_{k+1}$ in the block form $\left(\begin{array}{ll}A_{k} & O_{k} \\ B_{k} & C_{k}\end{array}\right)$ with four $2^{k} \times 2^{k}$ matrices. After $m$ steps, the last 1 in a row is at position $m+1$. Therefore $O_{k}$ is the zero matrix. According to the induction hypothesis, the bottom row of $\left[A_{k} O_{k}\right]$ is $[1, \ldots, 1,0, \ldots, 0]$, with $2^{k}$ ones and $2^{k}$ zeros. The next row is thus + +$$ +[\underbrace{0, \ldots, 0}_{2^{k}-1}, 1,1, \underbrace{0, \ldots, 0}_{2^{k}-1}] +$$ + +It is symmetric about its midpoint, and this symmetry is preserved in all subsequent rows because the procedure described in the problem statement is left/right symmetric. Thus $B_{k}$ is the mirror image of $C_{k}$. In particular, the rightmost column of $B_{k}$ is identical with the leftmost column of $C_{k}$. + +Imagine the matrix $C_{k}$ in isolation from the rest of $A_{k+1}$. Suppose it is subject to evolution as defined in the problem: the first (leftmost) term in a row depends only on the two first terms in the preceding row, according as they are equal or not. Now embed $C_{k}$ again in $A_{k}$. The 'leftmost' terms in the rows of $C_{k}$ now have neighbours on their left side- but these neighbours are their exact copies. Consequently the actual evolution within $C_{k}$ is the same, whether or not $C_{k}$ is considered as a piece of $A_{k+1}$ or in isolation. And since the top row of $C_{k}$ is $[1,0, \ldots, 0]$, it follows that $C_{k}$ is identical with $A_{k}$. + +The bottom row of $A_{k}$ is $[1,1, \ldots, 1]$; the same is the bottom row of $C_{k}$, hence also of $B_{k}$, which mirrors $C_{k}$. So the bottom row of $A_{k+1}$ consists of ones only and the induction is complete. + +(b) There are many ways to produce an infinite sequence of those $n$ for which the state $[0,0, \ldots, 0]$ will never be achieved. As an example, consider $n=2^{k}+1$ (for $k \geq 1$ ). The evolution of the system can be represented by a matrix $\mathcal{A}$ of width $2^{k}+1$ with infinitely many rows. The top $2^{k}$ rows form the matrix $A_{k}$ discussed above, with one column of zeros attached at its right. + +In the next row we then have the vector $[0,0, \ldots, 0,1,1]$. But this is just the second row of $\mathcal{A}$ reversed. Subsequent rows will be mirror copies of the foregoing ones, starting from the second one. So the configuration $[1,1,0, \ldots, 0,0]$, i.e. the second row of $\mathcal{A}$, will reappear. Further rows will periodically repeat this pattern and there will be no row of zeros. + +C2. A diagonal of a regular 2006-gon is called odd if its endpoints divide the boundary into two parts, each composed of an odd number of sides. Sides are also regarded as odd diagonals. + +Suppose the 2006-gon has been dissected into triangles by 2003 nonintersecting diagonals. Find the maximum possible number of isosceles triangles with two odd sides. + +(Serbia) + +Solution 1. Call an isosceles triangle odd if it has two odd sides. Suppose we are given a dissection as in the problem statement. A triangle in the dissection which is odd and isosceles will be called iso-odd for brevity. + +Lemma. Let $A B$ be one of dissecting diagonals and let $\mathcal{L}$ be the shorter part of the boundary of the 2006-gon with endpoints $A, B$. Suppose that $\mathcal{L}$ consists of $n$ segments. Then the number of iso-odd triangles with vertices on $\mathcal{L}$ does not exceed $n / 2$. + +Proof. This is obvious for $n=2$. Take $n$ with $20$ and assume the statement for less than $n$ points. Take a set $S$ of $n$ points. + +Let $C$ be the set of vertices of the convex hull of $S$, let $m=|C|$. + +Let $X \subset C$ be an arbitrary nonempty set. For any convex polygon $P$ with vertices in the set $S \backslash X$, we have $b(P)$ points of $S$ outside $P$. Excluding the points of $X-$ all outside $P$ — the set $S \backslash X$ contains exactly $b(P)-|X|$ of them. Writing $1-x=y$, by the induction hypothesis + +$$ +\sum_{P \subset S \backslash X} x^{a(P)} y^{b(P)-|X|}=1 +$$ + +(where $P \subset S \backslash X$ means that the vertices of $P$ belong to the set $S \backslash X$ ). Therefore + +$$ +\sum_{P \subset S \backslash X} x^{a(P)} y^{b(P)}=y^{|X|} +$$ + +All convex polygons appear at least once, except the convex hull $C$ itself. The convex hull adds $x^{m}$. We can use the inclusion-exclusion principle to compute the sum of the other terms: + +$$ +\begin{gathered} +\sum_{P \neq C} x^{a(P)} y^{b(P)}=\sum_{k=1}^{m}(-1)^{k-1} \sum_{|X|=k} \sum_{P \subset S \backslash X} x^{a(P)} y^{b(P)}=\sum_{k=1}^{m}(-1)^{k-1} \sum_{|X|=k} y^{k} \\ +=\sum_{k=1}^{m}(-1)^{k-1}\left(\begin{array}{c} +m \\ +k +\end{array}\right) y^{k}=-\left((1-y)^{m}-1\right)=1-x^{m} +\end{gathered} +$$ + +and then + +$$ +\sum_{P} x^{a(P)} y^{b(P)}=\sum_{P=C}+\sum_{P \neq C}=x^{m}+\left(1-x^{m}\right)=1 +$$ + +C4. A cake has the form of an $n \times n$ square composed of $n^{2}$ unit squares. Strawberries lie on some of the unit squares so that each row or column contains exactly one strawberry; call this arrangement $\mathcal{A}$. + +Let $\mathcal{B}$ be another such arrangement. Suppose that every grid rectangle with one vertex at the top left corner of the cake contains no fewer strawberries of arrangement $\mathcal{B}$ than of arrangement $\mathcal{A}$. Prove that arrangement $\mathcal{B}$ can be obtained from $\mathcal{A}$ by performing a number of switches, defined as follows: + +A switch consists in selecting a grid rectangle with only two strawberries, situated at its top right corner and bottom left corner, and moving these two strawberries to the other two corners of that rectangle. + +(Taiwan) + +Solution. We use capital letters to denote unit squares; $O$ is the top left corner square. For any two squares $X$ and $Y$ let $[X Y]$ be the smallest grid rectangle containing these two squares. Strawberries lie on some squares in arrangement $\mathcal{A}$. Put a plum on each square of the target configuration $\mathcal{B}$. For a square $X$ denote by $a(X)$ and $b(X)$ respectively the number of strawberries and the number of plums in $[O X]$. By hypothesis $a(X) \leq b(X)$ for each $X$, with strict inequality for some $X$ (otherwise the two arrangements coincide and there is nothing to prove). + +The idea is to show that by a legitimate switch one can obtain an arrangement $\mathcal{A}^{\prime}$ such that + +$$ +a(X) \leq a^{\prime}(X) \leq b(X) \quad \text { for each } X ; \quad \sum_{X} a(X)<\sum_{X} a^{\prime}(X) +$$ + +(with $a^{\prime}(X)$ defined analogously to $a(X)$; the sums range over all unit squares $X$ ). This will be enough because the same reasoning then applies to $\mathcal{A}^{\prime}$, giving rise to a new arrangement $\mathcal{A}^{\prime \prime}$, and so on (induction). Since $\sum a(X)<\sum a^{\prime}(X)<\sum a^{\prime \prime}(X)<\ldots$ and all these sums do not exceed $\sum b(X)$, we eventually obtain a sum with all summands equal to the respective $b(X) \mathrm{s}$; all strawberries will meet with plums. + +Consider the uppermost row in which the plum and the strawberry lie on different squares $P$ and $S$ (respectively); clearly $P$ must be situated left to $S$. In the column passing through $P$, let $T$ be the top square and $B$ the bottom square. The strawberry in that column lies below the plum (because there is no plum in that column above $P$, and the positions of strawberries and plums coincide everywhere above the row of $P$ ). Hence there is at least one strawberry in the region $[B S]$ below $[P S]$. Let $V$ be the position of the uppermost strawberry in that region. + +![](https://cdn.mathpix.com/cropped/2024_04_17_e55ccc62f43b046023cag-26.jpg?height=708&width=711&top_left_y=2010&top_left_x=678) + +Denote by $W$ the square at the intersection of the row through $V$ with the column through $S$ and let $R$ be the square vertex-adjacent to $W$ up-left. We claim that + +$$ +a(X)C D$. Points $K$ and $L$ lie on the line segments $A B$ and $C D$, respectively, so that $A K / K B=D L / L C$. Suppose that there are points $P$ and $Q$ on the line segment $K L$ satisfying + +$$ +\angle A P B=\angle B C D \quad \text { and } \quad \angle C Q D=\angle A B C \text {. } +$$ + +Prove that the points $P, Q, B$ and $C$ are concyclic. + +(Ukraine) + +Solution 1. Because $A B \| C D$, the relation $A K / K B=D L / L C$ readily implies that the lines $A D, B C$ and $K L$ have a common point $S$. + +![](https://cdn.mathpix.com/cropped/2024_04_17_e55ccc62f43b046023cag-37.jpg?height=1136&width=1073&top_left_y=851&top_left_x=500) + +Consider the second intersection points $X$ and $Y$ of the line $S K$ with the circles $(A B P)$ and $(C D Q)$, respectively. Since $A P B X$ is a cyclic quadrilateral and $A B \| C D$, one has + +$$ +\angle A X B=180^{\circ}-\angle A P B=180^{\circ}-\angle B C D=\angle A B C . +$$ + +This shows that $B C$ is tangent to the circle $(A B P)$ at $B$. Likewise, $B C$ is tangent to the circle $(C D Q)$ at $C$. Therefore $S P \cdot S X=S B^{2}$ and $S Q \cdot S Y=S C^{2}$. + +Let $h$ be the homothety with centre $S$ and ratio $S C / S B$. Since $h(B)=C$, the above conclusion about tangency implies that $h$ takes circle $(A B P)$ to circle $(C D Q)$. Also, $h$ takes $A B$ to $C D$, and it easily follows that $h(P)=Y, h(X)=Q$, yielding $S P / S Y=S B / S C=S X / S Q$. + +Equalities $S P \cdot S X=S B^{2}$ and $S Q / S X=S C / S B$ imply $S P \cdot S Q=S B \cdot S C$, which is equivalent to $P, Q, B$ and $C$ being concyclic. + +Solution 2. The case where $P=Q$ is trivial. Thus assume that $P$ and $Q$ are two distinct points. As in the first solution, notice that the lines $A D, B C$ and $K L$ concur at a point $S$. + +![](https://cdn.mathpix.com/cropped/2024_04_17_e55ccc62f43b046023cag-38.jpg?height=762&width=799&top_left_y=430&top_left_x=640) + +Let the lines $A P$ and $D Q$ meet at $E$, and let $B P$ and $C Q$ meet at $F$. Then $\angle E P F=\angle B C D$ and $\angle F Q E=\angle A B C$ by the condition of the problem. Since the angles $B C D$ and $A B C$ add up to $180^{\circ}$, it follows that $P E Q F$ is a cyclic quadrilateral. + +Applying Menelaus' theorem, first to triangle $A S P$ and line $D Q$ and then to triangle $B S P$ and line $C Q$, we have + +$$ +\frac{A D}{D S} \cdot \frac{S Q}{Q P} \cdot \frac{P E}{E A}=1 \text { and } \frac{B C}{C S} \cdot \frac{S Q}{Q P} \cdot \frac{P F}{F B}=1 +$$ + +The first factors in these equations are equal, as $A B \| C D$. Thus the last factors are also equal, which implies that $E F$ is parallel to $A B$ and $C D$. Using this and the cyclicity of $P E Q F$, we obtain + +$$ +\angle B C D=\angle B C F+\angle F C D=\angle B C Q+\angle E F Q=\angle B C Q+\angle E P Q +$$ + +On the other hand, + +$$ +\angle B C D=\angle A P B=\angle E P F=\angle E P Q+\angle Q P F, +$$ + +and consequently $\angle B C Q=\angle Q P F$. The latter angle either coincides with $\angle Q P B$ or is supplementary to $\angle Q P B$, depending on whether $Q$ lies between $K$ and $P$ or not. In either case it follows that $P, Q, B$ and $C$ are concyclic. + +G3. Let $A B C D E$ be a convex pentagon such that + +$$ +\angle B A C=\angle C A D=\angle D A E \quad \text { and } \quad \angle A B C=\angle A C D=\angle A D E \text {. } +$$ + +The diagonals $B D$ and $C E$ meet at $P$. Prove that the line $A P$ bisects the side $C D$. + +Solution. Let the diagonals $A C$ and $B D$ meet at $Q$, the diagonals $A D$ and $C E$ meet at $R$, and let the ray $A P$ meet the side $C D$ at $M$. We want to prove that $C M=M D$ holds. + +![](https://cdn.mathpix.com/cropped/2024_04_17_e55ccc62f43b046023cag-39.jpg?height=457&width=669&top_left_y=751&top_left_x=702) + +The idea is to show that $Q$ and $R$ divide $A C$ and $A D$ in the same ratio, or more precisely + +$$ +\frac{A Q}{Q C}=\frac{A R}{R D} +$$ + +(which is equivalent to $Q R \| C D$ ). The given angle equalities imply that the triangles $A B C$, $A C D$ and $A D E$ are similar. We therefore have + +$$ +\frac{A B}{A C}=\frac{A C}{A D}=\frac{A D}{A E} +$$ + +Since $\angle B A D=\angle B A C+\angle C A D=\angle C A D+\angle D A E=\angle C A E$, it follows from $A B / A C=$ $A D / A E$ that the triangles $A B D$ and $A C E$ are also similar. Their angle bisectors in $A$ are $A Q$ and $A R$, respectively, so that + +$$ +\frac{A B}{A C}=\frac{A Q}{A R} +$$ + +Because $A B / A C=A C / A D$, we obtain $A Q / A R=A C / A D$, which is equivalent to (1). Now Ceva's theorem for the triangle $A C D$ yields + +$$ +\frac{A Q}{Q C} \cdot \frac{C M}{M D} \cdot \frac{D R}{R A}=1 +$$ + +In view of (1), this reduces to $C M=M D$, which completes the proof. + +Comment. Relation (1) immediately follows from the fact that quadrilaterals $A B C D$ and $A C D E$ are similar. + +G4. A point $D$ is chosen on the side $A C$ of a triangle $A B C$ with $\angle C<\angle A<90^{\circ}$ in such a way that $B D=B A$. The incircle of $A B C$ is tangent to $A B$ and $A C$ at points $K$ and $L$, respectively. Let $J$ be the incentre of triangle $B C D$. Prove that the line $K L$ intersects the line segment $A J$ at its midpoint. + +(Russia) + +Solution. Denote by $P$ be the common point of $A J$ and $K L$. Let the parallel to $K L$ through $J$ meet $A C$ at $M$. Then $P$ is the midpoint of $A J$ if and only if $A M=2 \cdot A L$, which we are about to show. + +![](https://cdn.mathpix.com/cropped/2024_04_17_e55ccc62f43b046023cag-40.jpg?height=574&width=937&top_left_y=775&top_left_x=568) + +Denoting $\angle B A C=2 \alpha$, the equalities $B A=B D$ and $A K=A L$ imply $\angle A D B=2 \alpha$ and $\angle A L K=90^{\circ}-\alpha$. Since $D J$ bisects $\angle B D C$, we obtain $\angle C D J=\frac{1}{2} \cdot\left(180^{\circ}-\angle A D B\right)=90^{\circ}-\alpha$. Also $\angle D M J=\angle A L K=90^{\circ}-\alpha$ since $J M \| K L$. It follows that $J D=J M$. + +Let the incircle of triangle $B C D$ touch its side $C D$ at $T$. Then $J T \perp C D$, meaning that $J T$ is the altitude to the base $D M$ of the isosceles triangle $D M J$. It now follows that $D T=M T$, and we have + +$$ +D M=2 \cdot D T=B D+C D-B C \text {. } +$$ + +Therefore + +$$ +\begin{aligned} +A M & =A D+(B D+C D-B C) \\ +& =A D+A B+D C-B C \\ +& =A C+A B-B C \\ +& =2 \cdot A L, +\end{aligned} +$$ + +which completes the proof. + +G5. In triangle $A B C$, let $J$ be the centre of the excircle tangent to side $B C$ at $A_{1}$ and to the extensions of sides $A C$ and $A B$ at $B_{1}$ and $C_{1}$, respectively. Suppose that the lines $A_{1} B_{1}$ and $A B$ are perpendicular and intersect at $D$. Let $E$ be the foot of the perpendicular from $C_{1}$ to line $D J$. Determine the angles $\angle B E A_{1}$ and $\angle A E B_{1}$. + +(Greece) + +Solution 1. Let $K$ be the intersection point of lines $J C$ and $A_{1} B_{1}$. Obviously $J C \perp A_{1} B_{1}$ and since $A_{1} B_{1} \perp A B$, the lines $J K$ and $C_{1} D$ are parallel and equal. From the right triangle $B_{1} C J$ we obtain $J C_{1}^{2}=J B_{1}^{2}=J C \cdot J K=J C \cdot C_{1} D$ from which we infer that $D C_{1} / C_{1} J=C_{1} J / J C$ and the right triangles $D C_{1} J$ and $C_{1} J C$ are similar. Hence $\angle C_{1} D J=\angle J C_{1} C$, which implies that the lines $D J$ and $C_{1} C$ are perpendicular, i.e. the points $C_{1}, E, C$ are collinear. + +![](https://cdn.mathpix.com/cropped/2024_04_17_e55ccc62f43b046023cag-41.jpg?height=625&width=874&top_left_y=924&top_left_x=594) + +Since $\angle C A_{1} J=\angle C B_{1} J=\angle C E J=90^{\circ}$, points $A_{1}, B_{1}$ and $E$ lie on the circle of diameter $C J$. Then $\angle D B A_{1}=\angle A_{1} C J=\angle D E A_{1}$, which implies that quadrilateral $B E A_{1} D$ is cyclic; therefore $\angle A_{1} E B=90^{\circ}$. + +Quadrilateral $A D E B_{1}$ is also cyclic because $\angle E B_{1} A=\angle E J C=\angle E D C_{1}$, therefore we obtain $\angle A E B_{1}=\angle A D B=90^{\circ}$. + +![](https://cdn.mathpix.com/cropped/2024_04_17_e55ccc62f43b046023cag-41.jpg?height=691&width=943&top_left_y=1962&top_left_x=565) + +Solution 2. Consider the circles $\omega_{1}, \omega_{2}$ and $\omega_{3}$ of diameters $C_{1} D, A_{1} B$ and $A B_{1}$, respectively. Line segments $J C_{1}, J B_{1}$ and $J A_{1}$ are tangents to those circles and, due to the right angle at $D$, $\omega_{2}$ and $\omega_{3}$ pass through point $D$. Since $\angle C_{1} E D$ is a right angle, point $E$ lies on circle $\omega_{1}$, therefore + +$$ +J C_{1}^{2}=J D \cdot J E . +$$ + +Since $J A_{1}=J B_{1}=J C_{1}$ are all radii of the excircle, we also have + +$$ +J A_{1}^{2}=J D \cdot J E \quad \text { and } \quad J B_{1}^{2}=J D \cdot J E \text {. } +$$ + +These equalities show that $E$ lies on circles $\omega_{2}$ and $\omega_{3}$ as well, so $\angle B E A_{1}=\angle A E B_{1}=90^{\circ}$. + +Solution 3. First note that $A_{1} B_{1}$ is perpendicular to the external angle bisector $C J$ of $\angle B C A$ and parallel to the internal angle bisector of that angle. Therefore, $A_{1} B_{1}$ is perpendicular to $A B$ if and only if triangle $A B C$ is isosceles, $A C=B C$. In that case the external bisector $C J$ is parallel to $A B$. + +Triangles $A B C$ and $B_{1} A_{1} J$ are similar, as their corresponding sides are perpendicular. In particular, we have $\angle D A_{1} J=\angle C_{1} B A_{1}$; moreover, from cyclic deltoid $J A_{1} B C_{1}$, + +$$ +\angle C_{1} A_{1} J=\angle C_{1} B J=\frac{1}{2} \angle C_{1} B A_{1}=\frac{1}{2} \angle D A_{1} J +$$ + +Therefore, $A_{1} C_{1}$ bisects angle $\angle D A_{1} J$. + +![](https://cdn.mathpix.com/cropped/2024_04_17_e55ccc62f43b046023cag-42.jpg?height=637&width=870&top_left_y=1229&top_left_x=593) + +In triangle $B_{1} A_{1} J$, line $J C_{1}$ is the external bisector at vertex $J$. The point $C_{1}$ is the intersection of two external angle bisectors (at $A_{1}$ and $J$ ) so $C_{1}$ is the centre of the excircle $\omega$, tangent to side $A_{1} J$, and to the extension of $B_{1} A_{1}$ at point $D$. + +Now consider the similarity transform $\varphi$ which moves $B_{1}$ to $A, A_{1}$ to $B$ and $J$ to $C$. This similarity can be decomposed into a rotation by $90^{\circ}$ around a certain point $O$ and a homothety from the same centre. This similarity moves point $C_{1}$ (the centre of excircle $\omega$ ) to $J$ and moves $D$ (the point of tangency) to $C_{1}$. + +Since the rotation angle is $90^{\circ}$, we have $\angle X O \varphi(X)=90^{\circ}$ for an arbitrary point $X \neq O$. For $X=D$ and $X=C_{1}$ we obtain $\angle D O C_{1}=\angle C_{1} O J=90^{\circ}$. Therefore $O$ lies on line segment $D J$ and $C_{1} O$ is perpendicular to $D J$. This means that $O=E$. + +For $X=A_{1}$ and $X=B_{1}$ we obtain $\angle A_{1} O B=\angle B_{1} O A=90^{\circ}$, i.e. + +$$ +\angle B E A_{1}=\angle A E B_{1}=90^{\circ} . +$$ + +Comment. Choosing $X=J$, it also follows that $\angle J E C=90^{\circ}$ which proves that lines $D J$ and $C C_{1}$ intersect at point $E$. However, this is true more generally, without the assumption that $A_{1} B_{1}$ and $A B$ are perpendicular, because points $C$ and $D$ are conjugates with respect to the excircle. The last observation could replace the first paragraph of Solution 1. + +G6. Circles $\omega_{1}$ and $\omega_{2}$ with centres $O_{1}$ and $O_{2}$ are externally tangent at point $D$ and internally tangent to a circle $\omega$ at points $E$ and $F$, respectively. Line $t$ is the common tangent of $\omega_{1}$ and $\omega_{2}$ at $D$. Let $A B$ be the diameter of $\omega$ perpendicular to $t$, so that $A, E$ and $O_{1}$ are on the same side of $t$. Prove that lines $A O_{1}, B O_{2}, E F$ and $t$ are concurrent. + +(Brasil) + +Solution 1. Point $E$ is the centre of a homothety $h$ which takes circle $\omega_{1}$ to circle $\omega$. The radii $O_{1} D$ and $O B$ of these circles are parallel as both are perpendicular to line t. Also, $O_{1} D$ and $O B$ are on the same side of line $E O$, hence $h$ takes $O_{1} D$ to $O B$. Consequently, points $E$, $D$ and $B$ are collinear. Likewise, points $F, D$ and $A$ are collinear as well. + +Let lines $A E$ and $B F$ intersect at $C$. Since $A F$ and $B E$ are altitudes in triangle $A B C$, their common point $D$ is the orthocentre of this triangle. So $C D$ is perpendicular to $A B$, implying that $C$ lies on line $t$. Note that triangle $A B C$ is acute-angled. We mention the well-known fact that triangles $F E C$ and $A B C$ are similar in ratio $\cos \gamma$, where $\gamma=\angle A C B$. In addition, points $C, E, D$ and $F$ lie on the circle with diameter $C D$. + +![](https://cdn.mathpix.com/cropped/2024_04_17_e55ccc62f43b046023cag-43.jpg?height=771&width=748&top_left_y=1071&top_left_x=654) + +Let $P$ be the common point of lines $E F$ and $t$. We are going to prove that $P$ lies on line $A O_{1}$. Denote by $N$ the second common point of circle $\omega_{1}$ and $A C$; this is the point of $\omega_{1}$ diametrically opposite to $D$. By Menelaus' theorem for triangle $D C N$, points $A, O_{1}$ and $P$ are collinear if and only if + +$$ +\frac{C A}{A N} \cdot \frac{N O_{1}}{O_{1} D} \cdot \frac{D P}{P C}=1 +$$ + +Because $N O_{1}=O_{1} D$, this reduces to $C A / A N=C P / P D$. Let line $t$ meet $A B$ at $K$. Then $C A / A N=C K / K D$, so it suffices to show that + +$$ +\frac{C P}{P D}=\frac{C K}{K D} +$$ + +To verify (1), consider the circumcircle $\Omega$ of triangle $A B C$. Draw its diameter $C U$ through $C$, and let $C U$ meet $A B$ at $V$. Extend $C K$ to meet $\Omega$ at $L$. Since $A B$ is parallel to $U L$, we have $\angle A C U=\angle B C L$. On the other hand $\angle E F C=\angle B A C, \angle F E C=\angle A B C$ and $E F / A B=\cos \gamma$, as stated above. So reflection in the bisector of $\angle A C B$ followed by a homothety with centre $C$ and ratio $1 / \cos \gamma$ takes triangle $F E C$ to triangle $A B C$. Consequently, this transformation +takes $C D$ to $C U$, which implies $C P / P D=C V / V U$. Next, we have $K L=K D$, because $D$ is the orthocentre of triangle $A B C$. Hence $C K / K D=C K / K L$. Finally, $C V / V U=C K / K L$ because $A B$ is parallel to $U L$. Relation (1) follows, proving that $P$ lies on line $A O_{1}$. By symmetry, $P$ also lies on line $A O_{2}$ which completes the solution. + +Solution 2. We proceed as in the first solution to define a triangle $A B C$ with orthocentre $D$, in which $A F$ and $B E$ are altitudes. + +Denote by $M$ the midpoint of $C D$. The quadrilateral $C E D F$ is inscribed in a circle with centre $M$, hence $M C=M E=M D=M F$. + +![](https://cdn.mathpix.com/cropped/2024_04_17_e55ccc62f43b046023cag-44.jpg?height=723&width=849&top_left_y=752&top_left_x=612) + +Consider triangles $A B C$ and $O_{1} O_{2} M$. Lines $O_{1} O_{2}$ and $A B$ are parallel, both of them being perpendicular to line $t$. Next, $M O_{1}$ is the line of centres of circles $(C E F)$ and $\omega_{1}$ whose common chord is $D E$. Hence $M O_{1}$ bisects $\angle D M E$ which is the external angle at $M$ in the isosceles triangle $C E M$. It follows that $\angle D M O_{1}=\angle D C A$, so that $M O_{1}$ is parallel to $A C$. Likewise, $M O_{2}$ is parallel to $B C$. + +Thus the respective sides of triangles $A B C$ and $O_{1} O_{2} M$ are parallel; in addition, these triangles are not congruent. Hence there is a homothety taking $A B C$ to $O_{1} O_{2} M$. The lines $A O_{1}$, $B O_{2}$ and $C M=t$ are concurrent at the centre $Q$ of this homothety. + +Finally, apply Pappus' theorem to the triples of collinear points $A, O, B$ and $O_{2}, D, O_{1}$. The theorem implies that the points $A D \cap O O_{2}=F, A O_{1} \cap B O_{2}=Q$ and $O O_{1} \cap B D=E$ are collinear. In other words, line $E F$ passes through the common point $Q$ of $A O_{1}, B O_{2}$ and $t$. + +Comment. Relation (1) from Solution 1 expresses the well-known fact that points $P$ and $K$ are harmonic conjugates with respect to points $C$ and $D$. It is also easy to justify it by direct computation. Denoting $\angle C A B=\alpha, \angle A B C=\beta$, it is straightforward to obtain $C P / P D=C K / K D=\tan \alpha \tan \beta$. + +G7. In a triangle $A B C$, let $M_{a}, M_{b}, M_{c}$ be respectively the midpoints of the sides $B C, C A$, $A B$ and $T_{a}, T_{b}, T_{c}$ be the midpoints of the $\operatorname{arcs} B C, C A, A B$ of the circumcircle of $A B C$, not containing the opposite vertices. For $i \in\{a, b, c\}$, let $\omega_{i}$ be the circle with $M_{i} T_{i}$ as diameter. Let $p_{i}$ be the common external tangent to $\omega_{j}, \omega_{k}(\{i, j, k\}=\{a, b, c\})$ such that $\omega_{i}$ lies on the opposite side of $p_{i}$ than $\omega_{j}, \omega_{k}$ do. Prove that the lines $p_{a}, p_{b}, p_{c}$ form a triangle similar to $A B C$ and find the ratio of similitude. + +(Slovakia) + +Solution. Let $T_{a} T_{b}$ intersect circle $\omega_{b}$ at $T_{b}$ and $U$, and let $T_{a} T_{c}$ intersect circle $\omega_{c}$ at $T_{c}$ and $V$. Further, let $U X$ be the tangent to $\omega_{b}$ at $U$, with $X$ on $A C$, and let $V Y$ be the tangent to $\omega_{c}$ at $V$, with $Y$ on $A B$. The homothety with centre $T_{b}$ and ratio $T_{b} T_{a} / T_{b} U$ maps the circle $\omega_{b}$ onto the circumcircle of $A B C$ and the line $U X$ onto the line tangent to the circumcircle at $T_{a}$, which is parallel to $B C$; thus $U X \| B C$. The same is true of $V Y$, so that $U X\|B C\| V Y$. + +Let $T_{a} T_{b}$ cut $A C$ at $P$ and let $T_{a} T_{c}$ cut $A B$ at $Q$. The point $X$ lies on the hypotenuse $P M_{b}$ of the right triangle $P U M_{b}$ and is equidistant from $U$ and $M_{b}$. So $X$ is the midpoint of $M_{b} P$. Similarly $Y$ is the midpoint of $M_{c} Q$. + +Denote the incentre of triangle $A B C$ as usual by $I$. It is a known fact that $T_{a} I=T_{a} B$ and $T_{c} I=T_{c} B$. Therefore the points $B$ and $I$ are symmetric across $T_{a} T_{c}$, and consequently $\angle Q I B=\angle Q B I=\angle I B C$. This implies that $B C$ is parallel to the line $I Q$, and likewise, to $I P$. In other words, $P Q$ is the line parallel to $B C$ passing through $I$. + +![](https://cdn.mathpix.com/cropped/2024_04_17_e55ccc62f43b046023cag-46.jpg?height=868&width=874&top_left_y=1328&top_left_x=594) + +Clearly $M_{b} M_{c} \| B C$. So $P M_{b} M_{c} Q$ is a trapezoid and the segment $X Y$ connects the midpoints of its nonparallel sides; hence $X Y \| B C$. This combined with the previously established relations $U X\|B C\| V Y$ shows that all the four points $U, X, Y, V$ lie on a line which is the common tangent to circles $\omega_{b}, \omega_{c}$. Since it leaves these two circles on one side and the circle $\omega_{a}$ on the other, this line is just the line $p_{a}$ from the problem statement. + +Line $p_{a}$ runs midway between $I$ and $M_{b} M_{c}$. Analogous conclusions hold for the lines $p_{b}$ and $p_{c}$. So these three lines form a triangle homothetic from centre $I$ to triangle $M_{a} M_{b} M_{c}$ in ratio $1 / 2$, hence similar to $A B C$ in ratio $1 / 4$. + +G8. Let $A B C D$ be a convex quadrilateral. A circle passing through the points $A$ and $D$ and a circle passing through the points $B$ and $C$ are externally tangent at a point $P$ inside the quadrilateral. Suppose that + +$$ +\angle P A B+\angle P D C \leq 90^{\circ} \quad \text { and } \quad \angle P B A+\angle P C D \leq 90^{\circ} \text {. } +$$ + +Prove that $A B+C D \geq B C+A D$. + +(Poland) + +Solution. We start with a preliminary observation. Let $T$ be a point inside the quadrilateral $A B C D$. Then: + +$$ +\begin{aligned} +& \text { Circles }(B C T) \text { and }(D A T) \text { are tangent at } T \\ +& \text { if and only if } \quad \angle A D T+\angle B C T=\angle A T B \text {. } +\end{aligned} +$$ + +Indeed, if the two circles touch each other then their common tangent at $T$ intersects the segment $A B$ at a point $Z$, and so $\angle A D T=\angle A T Z, \angle B C T=\angle B T Z$, by the tangent-chord theorem. Thus $\angle A D T+\angle B C T=\angle A T Z+\angle B T Z=\angle A T B$. + +And conversely, if $\angle A D T+\angle B C T=\angle A T B$ then one can draw from $T$ a ray $T Z$ with $Z$ on $A B$ so that $\angle A D T=\angle A T Z, \angle B C T=\angle B T Z$. The first of these equalities implies that $T Z$ is tangent to the circle $(D A T)$; by the second equality, $T Z$ is tangent to the circle $(B C T)$, so the two circles are tangent at $T$. + +![](https://cdn.mathpix.com/cropped/2024_04_17_e55ccc62f43b046023cag-47.jpg?height=480&width=805&top_left_y=1319&top_left_x=634) + +So the equivalence (1) is settled. It will be used later on. Now pass to the actual solution. Its key idea is to introduce the circumcircles of triangles $A B P$ and $C D P$ and to consider their second intersection $Q$ (assume for the moment that they indeed meet at two distinct points $P$ and $Q$ ). + +Since the point $A$ lies outside the circle $(B C P)$, we have $\angle B C P+\angle B A P<180^{\circ}$. Therefore the point $C$ lies outside the circle $(A B P)$. Analogously, $D$ also lies outside that circle. It follows that $P$ and $Q$ lie on the same $\operatorname{arc} C D$ of the circle $(B C P)$. + +![](https://cdn.mathpix.com/cropped/2024_04_17_e55ccc62f43b046023cag-47.jpg?height=466&width=606&top_left_y=2257&top_left_x=728) + +By symmetry, $P$ and $Q$ lie on the same arc $A B$ of the circle $(A B P)$. Thus the point $Q$ lies either inside the angle $B P C$ or inside the angle $A P D$. Without loss of generality assume that $Q$ lies inside the angle $B P C$. Then + +$$ +\angle A Q D=\angle P Q A+\angle P Q D=\angle P B A+\angle P C D \leq 90^{\circ} \text {, } +$$ + +by the condition of the problem. + +In the cyclic quadrilaterals $A P Q B$ and $D P Q C$, the angles at vertices $A$ and $D$ are acute. So their angles at $Q$ are obtuse. This implies that $Q$ lies not only inside the angle $B P C$ but in fact inside the triangle $B P C$, hence also inside the quadrilateral $A B C D$. + +Now an argument similar to that used in deriving (2) shows that + +$$ +\angle B Q C=\angle P A B+\angle P D C \leq 90^{\circ} . +$$ + +Moreover, since $\angle P C Q=\angle P D Q$, we get + +$$ +\angle A D Q+\angle B C Q=\angle A D P+\angle P D Q+\angle B C P-\angle P C Q=\angle A D P+\angle B C P . +$$ + +The last sum is equal to $\angle A P B$, according to the observation (1) applied to $T=P$. And because $\angle A P B=\angle A Q B$, we obtain + +$$ +\angle A D Q+\angle B C Q=\angle A Q B \text {. } +$$ + +Applying now (1) to $T=Q$ we conclude that the circles $(B C Q)$ and $(D A Q)$ are externally tangent at $Q$. (We have assumed $P \neq Q$; but if $P=Q$ then the last conclusion holds trivially.) + +Finally consider the halfdiscs with diameters $B C$ and $D A$ constructed inwardly to the quadrilateral $A B C D$. They have centres at $M$ and $N$, the midpoints of $B C$ and $D A$ respectively. In view of (2) and (3), these two halfdiscs lie entirely inside the circles $(B Q C)$ and $(A Q D)$; and since these circles are tangent, the two halfdiscs cannot overlap. Hence $M N \geq \frac{1}{2} B C+\frac{1}{2} D A$. + +On the other hand, since $\overrightarrow{M N}=\frac{1}{2}(\overrightarrow{B A}+\overrightarrow{C D})$, we have $M N \leq \frac{1}{2}(A B+C D)$. Thus indeed $A B+C D \geq B C+D A$, as claimed. + +G9. Points $A_{1}, B_{1}, C_{1}$ are chosen on the sides $B C, C A, A B$ of a triangle $A B C$, respectively. The circumcircles of triangles $A B_{1} C_{1}, B C_{1} A_{1}, C A_{1} B_{1}$ intersect the circumcircle of triangle $A B C$ again at points $A_{2}, B_{2}, C_{2}$, respectively $\left(A_{2} \neq A, B_{2} \neq B, C_{2} \neq C\right)$. Points $A_{3}, B_{3}, C_{3}$ are symmetric to $A_{1}, B_{1}, C_{1}$ with respect to the midpoints of the sides $B C, C A, A B$ respectively. Prove that the triangles $A_{2} B_{2} C_{2}$ and $A_{3} B_{3} C_{3}$ are similar. + +(Russia) + +Solution. We will work with oriented angles between lines. For two straight lines $\ell, m$ in the plane, $\angle(\ell, m)$ denotes the angle of counterclockwise rotation which transforms line $\ell$ into a line parallel to $m$ (the choice of the rotation centre is irrelevant). This is a signed quantity; values differing by a multiple of $\pi$ are identified, so that + +$$ +\angle(\ell, m)=-\angle(m, \ell), \quad \angle(\ell, m)+\angle(m, n)=\angle(\ell, n) . +$$ + +If $\ell$ is the line through points $K, L$ and $m$ is the line through $M, N$, one writes $\angle(K L, M N)$ for $\angle(\ell, m)$; the characters $K, L$ are freely interchangeable; and so are $M, N$. + +The counterpart of the classical theorem about cyclic quadrilaterals is the following: If $K, L, M, N$ are four noncollinear points in the plane then + +$$ +K, L, M, N \text { are concyclic if and only if } \angle(K M, L M)=\angle(K N, L N) \text {. } +$$ + +Passing to the solution proper, we first show that the three circles $\left(A B_{1} C_{1}\right),\left(B C_{1} A_{1}\right)$, $\left(C A_{1} B_{1}\right)$ have a common point. So, let $\left(A B_{1} C_{1}\right)$ and $\left(B C_{1} A_{1}\right)$ intersect at the points $C_{1}$ and $P$. Then by (1) + +$$ +\begin{gathered} +\angle\left(P A_{1}, C A_{1}\right)=\angle\left(P A_{1}, B A_{1}\right)=\angle\left(P C_{1}, B C_{1}\right) \\ +=\angle\left(P C_{1}, A C_{1}\right)=\angle\left(P B_{1}, A B_{1}\right)=\angle\left(P B_{1}, C B_{1}\right) +\end{gathered} +$$ + +Denote this angle by $\varphi$. + +The equality between the outer terms shows, again by (1), that the points $A_{1}, B_{1}, P, C$ are concyclic. Thus $P$ is the common point of the three mentioned circles. + +From now on the basic property (1) will be used without explicit reference. We have + +$$ +\varphi=\angle\left(P A_{1}, B C\right)=\angle\left(P B_{1}, C A\right)=\angle\left(P C_{1}, A B\right) . +$$ + +![](https://cdn.mathpix.com/cropped/2024_04_17_e55ccc62f43b046023cag-49.jpg?height=778&width=1620&top_left_y=1938&top_left_x=226) + +Let lines $A_{2} P, B_{2} P, C_{2} P$ meet the circle $(A B C)$ again at $A_{4}, B_{4}, C_{4}$, respectively. As + +$$ +\angle\left(A_{4} A_{2}, A A_{2}\right)=\angle\left(P A_{2}, A A_{2}\right)=\angle\left(P C_{1}, A C_{1}\right)=\angle\left(P C_{1}, A B\right)=\varphi \text {, } +$$ + +we see that line $A_{2} A$ is the image of line $A_{2} A_{4}$ under rotation about $A_{2}$ by the angle $\varphi$. Hence the point $A$ is the image of $A_{4}$ under rotation by $2 \varphi$ about $O$, the centre of $(A B C)$. The same rotation sends $B_{4}$ to $B$ and $C_{4}$ to $C$. Triangle $A B C$ is the image of $A_{4} B_{4} C_{4}$ in this map. Thus + +$$ +\angle\left(A_{4} B_{4}, A B\right)=\angle\left(B_{4} C_{4}, B C\right)=\angle\left(C_{4} A_{4}, C A\right)=2 \varphi \text {. } +$$ + +Since the rotation by $2 \varphi$ about $O$ takes $B_{4}$ to $B$, we have $\angle\left(A B_{4}, A B\right)=\varphi$. Hence by (2) + +$$ +\angle\left(A B_{4}, P C_{1}\right)=\angle\left(A B_{4}, A B\right)+\angle\left(A B, P C_{1}\right)=\varphi+(-\varphi)=0 +$$ + +which means that $A B_{4} \| P C_{1}$. +![](https://cdn.mathpix.com/cropped/2024_04_17_e55ccc62f43b046023cag-50.jpg?height=784&width=1682&top_left_y=978&top_left_x=197) + +Let $C_{5}$ be the intersection of lines $P C_{1}$ and $A_{4} B_{4}$; define $A_{5}, B_{5}$ analogously. So $A B_{4} \| C_{1} C_{5}$ and, by (3) and (2), + +$$ +\angle\left(A_{4} B_{4}, P C_{1}\right)=\angle\left(A_{4} B_{4}, A B\right)+\angle\left(A B, P C_{1}\right)=2 \varphi+(-\varphi)=\varphi +$$ + +i.e., $\angle\left(B_{4} C_{5}, C_{5} C_{1}\right)=\varphi$. This combined with $\angle\left(C_{5} C_{1}, C_{1} A\right)=\angle\left(P C_{1}, A B\right)=\varphi$ (see (2)) proves that the quadrilateral $A B_{4} C_{5} C_{1}$ is an isosceles trapezoid with $A C_{1}=B_{4} C_{5}$. + +Interchanging the roles of $A$ and $B$ we infer that also $B C_{1}=A_{4} C_{5}$. And since $A C_{1}+B C_{1}=$ $A B=A_{4} B_{4}$, it follows that the point $C_{5}$ lies on the line segment $A_{4} B_{4}$ and partitions it into segments $A_{4} C_{5}, B_{4} C_{5}$ of lengths $B C_{1}\left(=A C_{3}\right)$ and $A C_{1}\left(=B C_{3}\right)$. In other words, the rotation which maps triangle $A_{4} B_{4} C_{4}$ onto $A B C$ carries $C_{5}$ onto $C_{3}$. Likewise, it sends $A_{5}$ to $A_{3}$ and $B_{5}$ to $B_{3}$. So the triangles $A_{3} B_{3} C_{3}$ and $A_{5} B_{5} C_{5}$ are congruent. It now suffices to show that the latter is similar to $A_{2} B_{2} C_{2}$. + +Lines $B_{4} C_{5}$ and $P C_{5}$ coincide respectively with $A_{4} B_{4}$ and $P C_{1}$. Thus by (4) + +$$ +\angle\left(B_{4} C_{5}, P C_{5}\right)=\varphi +$$ + +Analogously (by cyclic shift) $\varphi=\angle\left(C_{4} A_{5}, P A_{5}\right)$, which rewrites as + +$$ +\varphi=\angle\left(B_{4} A_{5}, P A_{5}\right) +$$ + +These relations imply that the points $P, B_{4}, C_{5}, A_{5}$ are concyclic. Analogously, $P, C_{4}, A_{5}, B_{5}$ and $P, A_{4}, B_{5}, C_{5}$ are concyclic quadruples. Therefore + +$$ +\angle\left(A_{5} B_{5}, C_{5} B_{5}\right)=\angle\left(A_{5} B_{5}, P B_{5}\right)+\angle\left(P B_{5}, C_{5} B_{5}\right)=\angle\left(A_{5} C_{4}, P C_{4}\right)+\angle\left(P A_{4}, C_{5} A_{4}\right) . +$$ + +On the other hand, since the points $A_{2}, B_{2}, C_{2}, A_{4}, B_{4}, C_{4}$ all lie on the circle $(A B C)$, we have + +$$ +\angle\left(A_{2} B_{2}, C_{2} B_{2}\right)=\angle\left(A_{2} B_{2}, B_{4} B_{2}\right)+\angle\left(B_{4} B_{2}, C_{2} B_{2}\right)=\angle\left(A_{2} A_{4}, B_{4} A_{4}\right)+\angle\left(B_{4} C_{4}, C_{2} C_{4}\right) . +$$ + +But the lines $A_{2} A_{4}, B_{4} A_{4}, B_{4} C_{4}, C_{2} C_{4}$ coincide respectively with $P A_{4}, C_{5} A_{4}, A_{5} C_{4}, P C_{4}$. So the sums on the right-hand sides of (5) and (6) are equal, leading to equality between their left-hand sides: $\angle\left(A_{5} B_{5}, C_{5} B_{5}\right)=\angle\left(A_{2} B_{2}, C_{2} B_{2}\right)$. Hence (by cyclic shift, once more) also $\angle\left(B_{5} C_{5}, A_{5} C_{5}\right)=\angle\left(B_{2} C_{2}, A_{2} C_{2}\right)$ and $\angle\left(C_{5} A_{5}, B_{5} A_{5}\right)=\angle\left(C_{2} A_{2}, B_{2} A_{2}\right)$. This means that the triangles $A_{5} B_{5} C_{5}$ and $A_{2} B_{2} C_{2}$ have their corresponding angles equal, and consequently they are similar. + +Comment 1. This is the way in which the proof has been presented by the proposer. Trying to work it out in the language of classical geometry, so as to avoid oriented angles, one is led to difficulties due to the fact that the reasoning becomes heavily case-dependent. Disposition of relevant points can vary in many respects. Angles which are equal in one case become supplementary in another. Although it seems not hard to translate all formulas from the shapes they have in one situation to the one they have in another, the real trouble is to identify all cases possible and rigorously verify that the key conclusions retain validity in each case. + +The use of oriented angles is a very efficient method to omit this trouble. It seems to be the most appropriate environment in which the solution can be elaborated. + +Comment 2. Actually, the fact that the circles $\left(A B_{1} C_{1}\right),\left(B C_{1} A_{1}\right)$ and $\left(C A_{1} B_{1}\right)$ have a common point does not require a proof; it is known as Miquel's theorem. + +G10. To each side $a$ of a convex polygon we assign the maximum area of a triangle contained in the polygon and having $a$ as one of its sides. Show that the sum of the areas assigned to all sides of the polygon is not less than twice the area of the polygon. + +(Serbia) + +## Solution 1. + +Lemma. Every convex $(2 n)$-gon, of area $S$, has a side and a vertex that jointly span a triangle of area not less than $S / n$. + +Proof. By main diagonals of the $(2 n)$-gon we shall mean those which partition the $(2 n)$-gon into two polygons with equally many sides. For any side $b$ of the $(2 n)$-gon denote by $\Delta_{b}$ the triangle $A B P$ where $A, B$ are the endpoints of $b$ and $P$ is the intersection point of the main diagonals $A A^{\prime}, B B^{\prime}$. We claim that the union of triangles $\Delta_{b}$, taken over all sides, covers the whole polygon. + +To show this, choose any side $A B$ and consider the main diagonal $A A^{\prime}$ as a directed segment. Let $X$ be any point in the polygon, not on any main diagonal. For definiteness, let $X$ lie on the left side of the ray $A A^{\prime}$. Consider the sequence of main diagonals $A A^{\prime}, B B^{\prime}, C C^{\prime}, \ldots$, where $A, B, C, \ldots$ are consecutive vertices, situated right to $A A^{\prime}$. + +The $n$-th item in this sequence is the diagonal $A^{\prime} A$ (i.e. $A A^{\prime}$ reversed), having $X$ on its right side. So there are two successive vertices $K, L$ in the sequence $A, B, C, \ldots$ before $A^{\prime}$ such that $X$ still lies to the left of $K K^{\prime}$ but to the right of $L L^{\prime}$. And this means that $X$ is in the triangle $\Delta_{\ell^{\prime}}, \ell^{\prime}=K^{\prime} L^{\prime}$. Analogous reasoning applies to points $X$ on the right of $A A^{\prime}$ (points lying on main diagonals can be safely ignored). Thus indeed the triangles $\Delta_{b}$ jointly cover the whole polygon. + +The sum of their areas is no less than $S$. So we can find two opposite sides, say $b=A B$ and $b^{\prime}=A^{\prime} B^{\prime}$ (with $A A^{\prime}, B B^{\prime}$ main diagonals) such that $\left[\Delta_{b}\right]+\left[\Delta_{b^{\prime}}\right] \geq S / n$, where $[\cdots]$ stands for the area of a region. Let $A A^{\prime}, B B^{\prime}$ intersect at $P$; assume without loss of generality that $P B \geq P B^{\prime}$. Then + +$$ +\left[A B A^{\prime}\right]=[A B P]+\left[P B A^{\prime}\right] \geq[A B P]+\left[P A^{\prime} B^{\prime}\right]=\left[\Delta_{b}\right]+\left[\Delta_{b^{\prime}}\right] \geq S / n +$$ + +proving the lemma. + +Now, let $\mathcal{P}$ be any convex polygon, of area $S$, with $m$ sides $a_{1}, \ldots, a_{m}$. Let $S_{i}$ be the area of the greatest triangle in $\mathcal{P}$ with side $a_{i}$. Suppose, contrary to the assertion, that + +$$ +\sum_{i=1}^{m} \frac{S_{i}}{S}<2 +$$ + +Then there exist rational numbers $q_{1}, \ldots, q_{m}$ such that $\sum q_{i}=2$ and $q_{i}>S_{i} / S$ for each $i$. + +Let $n$ be a common denominator of the $m$ fractions $q_{1}, \ldots, q_{m}$. Write $q_{i}=k_{i} / n$; so $\sum k_{i}=2 n$. Partition each side $a_{i}$ of $\mathcal{P}$ into $k_{i}$ equal segments, creating a convex $(2 n)$-gon of area $S$ (with some angles of size $180^{\circ}$ ), to which we apply the lemma. Accordingly, this refined polygon has a side $b$ and a vertex $H$ spanning a triangle $T$ of area $[T] \geq S / n$. If $b$ is a piece of a side $a_{i}$ of $\mathcal{P}$, then the triangle $W$ with base $a_{i}$ and summit $H$ has area + +$$ +[W]=k_{i} \cdot[T] \geq k_{i} \cdot S / n=q_{i} \cdot S>S_{i}, +$$ + +in contradiction with the definition of $S_{i}$. This ends the proof. + +Solution 2. As in the first solution, we allow again angles of size $180^{\circ}$ at some vertices of the convex polygons considered. + +To each convex $n$-gon $\mathcal{P}=A_{1} A_{2} \ldots A_{n}$ we assign a centrally symmetric convex $(2 n)$-gon $\mathcal{Q}$ with side vectors $\pm \overrightarrow{A_{i} A_{i+1}}, 1 \leq i \leq n$. The construction is as follows. Attach the $2 n$ vectors $\pm \overrightarrow{A_{i} A_{i+1}}$ at a common origin and label them $\overrightarrow{\mathbf{b}_{1}}, \overrightarrow{\mathbf{b}_{2}}, \ldots, \overrightarrow{\mathbf{b}_{2 n}}$ in counterclockwise direction; the choice of the first vector $\overrightarrow{\mathbf{b}_{1}}$ is irrelevant. The order of labelling is well-defined if $\mathcal{P}$ has neither parallel sides nor angles equal to $180^{\circ}$. Otherwise several collinear vectors with the same direction are labelled consecutively $\overrightarrow{\mathbf{b}_{j}}, \overrightarrow{\mathbf{b}_{j+1}}, \ldots, \overrightarrow{\mathbf{b}_{j+r}}$. One can assume that in such cases the respective opposite vectors occur in the order $-\overrightarrow{\mathbf{b}_{j}},-\overrightarrow{\mathbf{b}_{j+1}}, \ldots,-\overrightarrow{\mathbf{b}_{j+r}}$, ensuring that $\overrightarrow{\mathbf{b}_{j+n}}=-\overrightarrow{\mathbf{b}_{j}}$ for $j=1, \ldots, 2 n$. Indices are taken cyclically here and in similar situations below. + +Choose points $B_{1}, B_{2}, \ldots, B_{2 n}$ satisfying $\overrightarrow{B_{j} B_{j+1}}=\overrightarrow{\mathbf{b}_{j}}$ for $j=1, \ldots, 2 n$. The polygonal line $\mathcal{Q}=B_{1} B_{2} \ldots B_{2 n}$ is closed, since $\sum_{j=1}^{2 n} \overrightarrow{\mathbf{b}_{j}}=\overrightarrow{0}$. Moreover, $\mathcal{Q}$ is a convex $(2 n)$-gon due to the arrangement of the vectors $\overrightarrow{\mathbf{b}_{j}}$, possibly with $180^{\circ}$-angles. The side vectors of $\mathcal{Q}$ are $\pm \overrightarrow{A_{i} A_{i+1}}$, $1 \leq i \leq n$. So in particular $\mathcal{Q}$ is centrally symmetric, because it contains as side vectors $\overrightarrow{A_{i} A_{i+1}}$ and $-\overrightarrow{A_{i} A_{i+1}}$ for each $i=1, \ldots, n$. Note that $B_{j} B_{j+1}$ and $B_{j+n} B_{j+n+1}$ are opposite sides of $\mathcal{Q}$, $1 \leq j \leq n$. We call $\mathcal{Q}$ the associate of $\mathcal{P}$. + +Let $S_{i}$ be the maximum area of a triangle with side $A_{i} A_{i+1}$ in $\mathcal{P}, 1 \leq i \leq n$. We prove that + +$$ +\left[B_{1} B_{2} \ldots B_{2 n}\right]=2 \sum_{i=1}^{n} S_{i} +$$ + +and + +$$ +\left[B_{1} B_{2} \ldots B_{2 n}\right] \geq 4\left[A_{1} A_{2} \ldots A_{n}\right] +$$ + +It is clear that (1) and (2) imply the conclusion of the original problem. + +Lemma. For a side $A_{i} A_{i+1}$ of $\mathcal{P}$, let $h_{i}$ be the maximum distance from a point of $\mathcal{P}$ to line $A_{i} A_{i+1}$, $i=1, \ldots, n$. Denote by $B_{j} B_{j+1}$ the side of $\mathcal{Q}$ such that $\overrightarrow{A_{i} A_{i+1}}=\overrightarrow{B_{j} B_{j+1}}$. Then the distance between $B_{j} B_{j+1}$ and its opposite side in $\mathcal{Q}$ is equal to $2 h_{i}$. + +Proof. Choose a vertex $A_{k}$ of $\mathcal{P}$ at distance $h_{i}$ from line $A_{i} A_{i+1}$. Let $\mathbf{u}$ be the unit vector perpendicular to $A_{i} A_{i+1}$ and pointing inside $\mathcal{P}$. Denoting by $\mathbf{x} \cdot \mathbf{y}$ the dot product of vectors $\mathbf{x}$ and $\mathbf{y}$, we have + +$$ +h=\mathbf{u} \cdot \overrightarrow{A_{i} A_{k}}=\mathbf{u} \cdot\left(\overrightarrow{A_{i} A_{i+1}}+\cdots+\overrightarrow{A_{k-1} A_{k}}\right)=\mathbf{u} \cdot\left(\overrightarrow{A_{i} A_{i-1}}+\cdots+\overrightarrow{A_{k+1} A_{k}}\right) . +$$ + +In $\mathcal{Q}$, the distance $H_{i}$ between the opposite sides $B_{j} B_{j+1}$ and $B_{j+n} B_{j+n+1}$ is given by + +$$ +H_{i}=\mathbf{u} \cdot\left(\overrightarrow{B_{j} B_{j+1}}+\cdots+\overrightarrow{B_{j+n-1} B_{j+n}}\right)=\mathbf{u} \cdot\left(\overrightarrow{\mathbf{b}_{j}}+\overrightarrow{\mathbf{b}_{j+1}}+\cdots+\overrightarrow{\mathbf{b}_{j+n-1}}\right) . +$$ + +The choice of vertex $A_{k}$ implies that the $n$ consecutive vectors $\overrightarrow{\mathbf{b}_{j}}, \overrightarrow{\mathbf{b}_{j+1}}, \ldots, \overrightarrow{\mathbf{b}_{j+n-1}}$ are precisely $\overrightarrow{A_{i} A_{i+1}}, \ldots, \overrightarrow{A_{k-1} A_{k}}$ and $\overrightarrow{A_{i} A_{i-1}}, \ldots, \overrightarrow{A_{k+1} A_{k}}$, taken in some order. This implies $H_{i}=2 h_{i}$. + +For a proof of (1), apply the lemma to each side of $\mathcal{P}$. If $O$ the centre of $\mathcal{Q}$ then, using the notation of the lemma, + +$$ +\left[B_{j} B_{j+1} O\right]=\left[B_{j+n} B_{j+n+1} O\right]=\left[A_{i} A_{i+1} A_{k}\right]=S_{i} +$$ + +Summation over all sides of $\mathcal{P}$ yields (1). + +Set $d(\mathcal{P})=[\mathcal{Q}]-4[\mathcal{P}]$ for a convex polygon $\mathcal{P}$ with associate $\mathcal{Q}$. Inequality $(2)$ means that $d(\mathcal{P}) \geq 0$ for each convex polygon $\mathcal{P}$. The last inequality will be proved by induction on the +number $\ell$ of side directions of $\mathcal{P}$, i. e. the number of pairwise nonparallel lines each containing a side of $\mathcal{P}$. + +We choose to start the induction with $\ell=1$ as a base case, meaning that certain degenerate polygons are allowed. More exactly, we regard as degenerate convex polygons all closed polygonal lines of the form $X_{1} X_{2} \ldots X_{k} Y_{1} Y_{2} \ldots Y_{m} X_{1}$, where $X_{1}, X_{2}, \ldots, X_{k}$ are points in this order on a line segment $X_{1} Y_{1}$, and so are $Y_{m}, Y_{m-1}, \ldots, Y_{1}$. The initial construction applies to degenerate polygons; their associates are also degenerate, and the value of $d$ is zero. For the inductive step, consider a convex polygon $\mathcal{P}$ which determines $\ell$ side directions, assuming that $d(\mathcal{P}) \geq 0$ for polygons with smaller values of $\ell$. + +Suppose first that $\mathcal{P}$ has a pair of parallel sides, i. e. sides on distinct parallel lines. Let $A_{i} A_{i+1}$ and $A_{j} A_{j+1}$ be such a pair, and let $A_{i} A_{i+1} \leq A_{j} A_{j+1}$. Remove from $\mathcal{P}$ the parallelogram $R$ determined by vectors $\overrightarrow{A_{i} A_{i+1}}$ and $\overrightarrow{A_{i} A_{j+1}}$. Two polygons are obtained in this way. Translating one of them by vector $\overrightarrow{A_{i} A_{i+1}}$ yields a new convex polygon $\mathcal{P}^{\prime}$, of area $[\mathcal{P}]-[R]$ and with value of $\ell$ not exceeding the one of $\mathcal{P}$. The construction just described will be called operation A. +![](https://cdn.mathpix.com/cropped/2024_04_17_e55ccc62f43b046023cag-54.jpg?height=464&width=1440&top_left_y=1098&top_left_x=316) + +The associate of $\mathcal{P}^{\prime}$ is obtained from $\mathcal{Q}$ upon decreasing the lengths of two opposite sides by an amount of $2 A_{i} A_{i+1}$. By the lemma, the distance between these opposite sides is twice the distance between $A_{i} A_{i+1}$ and $A_{j} A_{j+1}$. Thus operation $\mathbf{A}$ decreases $[\mathcal{Q}]$ by the area of a parallelogram with base and respective altitude twice the ones of $R$, i. e. by $4[R]$. Hence $\mathbf{A}$ leaves the difference $d(\mathcal{P})=[\mathcal{Q}]-4[\mathcal{P}]$ unchanged. + +Now, if $\mathcal{P}^{\prime}$ also has a pair of parallel sides, apply operation $\mathbf{A}$ to it. Keep doing so with the subsequent polygons obtained for as long as possible. Now, A decreases the number $p$ of pairs of parallel sides in $\mathcal{P}$. Hence its repeated applications gradually reduce $p$ to 0 , and further applications of $\mathbf{A}$ will be impossible after several steps. For clarity, let us denote by $\mathcal{P}$ again the polygon obtained at that stage. + +The inductive step is complete if $\mathcal{P}$ is degenerate. Otherwise $\ell>1$ and $p=0$, i. e. there are no parallel sides in $\mathcal{P}$. Observe that then $\ell \geq 3$. Indeed, $\ell=2$ means that the vertices of $\mathcal{P}$ all lie on the boundary of a parallelogram, implying $p>0$. + +Furthermore, since $\mathcal{P}$ has no parallel sides, consecutive collinear vectors in the sequence $\left(\overrightarrow{\mathbf{b}_{k}}\right)$ (if any) correspond to consecutive $180^{\circ}$-angles in $\mathcal{P}$. Removing the vertices of such angles, we obtain a convex polygon with the same value of $d(\mathcal{P})$. + +In summary, if operation $\mathbf{A}$ is impossible for a nondegenerate polygon $\mathcal{P}$, then $\ell \geq 3$. In addition, one may assume that $\mathcal{P}$ has no angles of size $180^{\circ}$. + +The last two conditions then also hold for the associate $\mathcal{Q}$ of $\mathcal{P}$, and we perform the following construction. Since $\ell \geq 3$, there is a side $B_{j} B_{j+1}$ of $\mathcal{Q}$ such that the sum of the angles at $B_{j}$ and $B_{j+1}$ is greater than $180^{\circ}$. (Such a side exists in each convex $k$-gon for $k>4$.) Naturally, $B_{j+n} B_{j+n+1}$ is a side with the same property. Extend the pairs of sides $B_{j-1} B_{j}, B_{j+1} B_{j+2}$ +and $B_{j+n-1} B_{j+n}, B_{j+n+1} B_{j+n+2}$ to meet at $U$ and $V$, respectively. Let $\mathcal{Q}^{\prime}$ be the centrally symmetric convex $2(n+1)$-gon obtained from $\mathcal{Q}$ by inserting $U$ and $V$ into the sequence $B_{1}, \ldots, B_{2 n}$ as new vertices between $B_{j}, B_{j+1}$ and $B_{j+n}, B_{j+n+1}$, respectively. Informally, we adjoin to $\mathcal{Q}$ the congruent triangles $B_{j} B_{j+1} U$ and $B_{j+n} B_{j+n+1} V$. Note that $B_{j}, B_{j+1}, B_{j+n}$ and $B_{j+n+1}$ are kept as vertices of $\mathcal{Q}^{\prime}$, although $B_{j} B_{j+1}$ and $B_{j+n} B_{j+n+1}$ are no longer its sides. + +Let $A_{i} A_{i+1}$ be the side of $\mathcal{P}$ such that $\overrightarrow{A_{i} A_{i+1}}=\overrightarrow{B_{j} B_{j+1}}=\overrightarrow{\mathbf{b}_{j}}$. Consider the point $W$ such that triangle $A_{i} A_{i+1} W$ is congruent to triangle $B_{j} B_{j+1} U$ and exterior to $\mathcal{P}$. Insert $W$ into the sequence $A_{1}, A_{2}, \ldots, A_{n}$ as a new vertex between $A_{i}$ and $A_{i+1}$ to obtain an $(n+1)$-gon $\mathcal{P}^{\prime}$. We claim that $\mathcal{P}^{\prime}$ is convex and its associate is $\mathcal{Q}^{\prime}$. +![](https://cdn.mathpix.com/cropped/2024_04_17_e55ccc62f43b046023cag-55.jpg?height=532&width=1388&top_left_y=796&top_left_x=344) + +Vectors $\overrightarrow{A_{i} W}$ and $\overrightarrow{\mathbf{b}_{j-1}}$ are collinear and have the same direction, as well as vectors $\overrightarrow{W A_{i+1}}$ and $\overrightarrow{\mathbf{b}_{j+1}}$. Since $\overrightarrow{\mathbf{b}_{j-1}}, \overrightarrow{\mathbf{b}_{j}}, \overrightarrow{\mathbf{b}_{j+1}}$ are consecutive terms in the sequence $\left(\overrightarrow{\mathbf{b}_{k}}\right)$, the angle inequalities $\angle\left(\overrightarrow{\mathbf{b}_{j-1}}, \overrightarrow{\mathbf{b}_{j}}\right) \leq \angle\left(\overrightarrow{A_{i-1} A_{i}}, \overrightarrow{\mathbf{b}_{j}}\right)$ and $\angle\left(\overrightarrow{\mathbf{b}_{j}}, \overrightarrow{\mathbf{b}_{j+1}}\right) \leq \angle\left(\overrightarrow{\mathbf{b}_{j}}, \overrightarrow{A_{i+1} A_{i+2}}\right)$ hold true. They show that $\mathcal{P}^{\prime}$ is a convex polygon. To construct its associate, vectors $\pm \overrightarrow{A_{i} A_{i+1}}= \pm \overrightarrow{\mathbf{b}_{j}}$ must be deleted from the defining sequence $\left(\overrightarrow{\mathbf{b}_{k}}\right.$ ) of $\mathcal{Q}$, and the vectors $\pm \overrightarrow{A_{i} W}, \pm \overrightarrow{W A_{i+1}}$ must be inserted appropriately into it. The latter can be done as follows: + +$$ +\ldots, \overrightarrow{\mathbf{b}_{j-1}}, \overrightarrow{A_{i} W}, \overrightarrow{W A_{i+1}}, \overrightarrow{\mathbf{b}_{j+1}}, \ldots,-\overrightarrow{\mathbf{b}_{j-1}},-\overrightarrow{A_{i} W},-\overrightarrow{W A_{i+1}},-\overrightarrow{\mathbf{b}_{j+1}}, \ldots +$$ + +This updated sequence produces $\mathcal{Q}^{\prime}$ as the associate of $\mathcal{P}^{\prime}$. + +It follows from the construction that $\left[\mathcal{P}^{\prime}\right]=[\mathcal{P}]+\left[A_{i} A_{i+1} W\right]$ and $\left[\mathcal{Q}^{\prime}\right]=[\mathcal{Q}]+2\left[A_{i} A_{i+1} W\right]$. Therefore $d\left(\mathcal{P}^{\prime}\right)=d(\mathcal{P})-2\left[A_{i} A_{i+1} W\right]0$; without loss of generality confine attention to $y>0$. The equation rewritten as + +$$ +2^{x}\left(1+2^{x+1}\right)=(y-1)(y+1) +$$ + +shows that the factors $y-1$ and $y+1$ are even, exactly one of them divisible by 4 . Hence $x \geq 3$ and one of these factors is divisible by $2^{x-1}$ but not by $2^{x}$. So + +$$ +y=2^{x-1} m+\epsilon, \quad m \text { odd }, \quad \epsilon= \pm 1 +$$ + +Plugging this into the original equation we obtain + +$$ +2^{x}\left(1+2^{x+1}\right)=\left(2^{x-1} m+\epsilon\right)^{2}-1=2^{2 x-2} m^{2}+2^{x} m \epsilon, +$$ + +or, equivalently + +$$ +1+2^{x+1}=2^{x-2} m^{2}+m \epsilon . +$$ + +Therefore + +$$ +1-\epsilon m=2^{x-2}\left(m^{2}-8\right) . +$$ + +For $\epsilon=1$ this yields $m^{2}-8 \leq 0$, i.e., $m=1$, which fails to satisfy (2). + +For $\epsilon=-1$ equation (2) gives us + +$$ +1+m=2^{x-2}\left(m^{2}-8\right) \geq 2\left(m^{2}-8\right), +$$ + +implying $2 m^{2}-m-17 \leq 0$. Hence $m \leq 3$; on the other hand $m$ cannot be 1 by (2). Because $m$ is odd, we obtain $m=3$, leading to $x=4$. From (1) we get $y=23$. These values indeed satisfy the given equation. Recall that then $y=-23$ is also good. Thus we have the complete list of solutions $(x, y):(0,2),(0,-2),(4,23),(4,-23)$. + +N2. For $x \in(0,1)$ let $y \in(0,1)$ be the number whose $n$th digit after the decimal point is the $\left(2^{n}\right)$ th digit after the decimal point of $x$. Show that if $x$ is rational then so is $y$. + +(Canada) + +Solution. Since $x$ is rational, its digits repeat periodically starting at some point. We wish to show that this is also true for the digits of $y$, implying that $y$ is rational. + +Let $d$ be the length of the period of $x$ and let $d=2^{u} \cdot v$, where $v$ is odd. There is a positive integer $w$ such that + +$$ +2^{w} \equiv 1 \quad(\bmod v) +$$ + +(For instance, one can choose $w$ to be $\varphi(v)$, the value of Euler's function at $v$.) Therefore + +$$ +2^{n+w}=2^{n} \cdot 2^{w} \equiv 2^{n} \quad(\bmod v) +$$ + +for each $n$. Also, for $n \geq u$ we have + +$$ +2^{n+w} \equiv 2^{n} \equiv 0 \quad\left(\bmod 2^{u}\right) +$$ + +It follows that, for all $n \geq u$, the relation + +$$ +2^{n+w} \equiv 2^{n} \quad(\bmod d) +$$ + +holds. Thus, for $n$ sufficiently large, the $2^{n+w}$ th digit of $x$ is in the same spot in the cycle of $x$ as its $2^{n}$ th digit, and so these digits are equal. Hence the $(n+w)$ th digit of $y$ is equal to its $n$th digit. This means that the digits of $y$ repeat periodically with period $w$ from some point on, as required. + +N3. The sequence $f(1), f(2), f(3), \ldots$ is defined by + +$$ +f(n)=\frac{1}{n}\left(\left\lfloor\frac{n}{1}\right\rfloor+\left\lfloor\frac{n}{2}\right\rfloor+\cdots+\left\lfloor\frac{n}{n}\right\rfloor\right), +$$ + +where $\lfloor x\rfloor$ denotes the integer part of $x$. + +(a) Prove that $f(n+1)>f(n)$ infinitely often. + +(b) Prove that $f(n+1)f(n)$ and $d(n+1)1, d(n) \geq 2$ holds, with equality if and only if $n$ is prime. Since $f(6)=7 / 3>2$, it follows that $f(n)>2$ holds for all $n \geq 6$. + +Since there are infinitely many primes, $d(n+1)=2$ holds for infinitely many values of $n$, and for each such $n \geq 6$ we have $d(n+1)=2\max \{d(1), d(2), \ldots, d(n)\}$ for infinitely many $n$. For all such $n$, we have $d(n+1)>f(n)$. This completes the solution. + +N4. Let $P$ be a polynomial of degree $n>1$ with integer coefficients and let $k$ be any positive integer. Consider the polynomial $Q(x)=P(P(\ldots P(P(x)) \ldots))$, with $k$ pairs of parentheses. Prove that $Q$ has no more than $n$ integer fixed points, i.e. integers satisfying the equation $Q(x)=x$. + +(Romania) + +Solution. The claim is obvious if every integer fixed point of $Q$ is a fixed point of $P$ itself. For the sequel assume that this is not the case. Take any integer $x_{0}$ such that $Q\left(x_{0}\right)=x_{0}$, $P\left(x_{0}\right) \neq x_{0}$ and define inductively $x_{i+1}=P\left(x_{i}\right)$ for $i=0,1,2, \ldots$; then $x_{k}=x_{0}$. + +It is evident that + +$$ +P(u)-P(v) \text { is divisible by } u-v \text { for distinct integers } u, v \text {. } +$$ + +(Indeed, if $P(x)=\sum a_{i} x^{i}$ then each $a_{i}\left(u^{i}-v^{i}\right)$ is divisible by $u-v$.) Therefore each term in the chain of (nonzero) differences + +$$ +x_{0}-x_{1}, \quad x_{1}-x_{2}, \quad \ldots, \quad x_{k-1}-x_{k}, \quad x_{k}-x_{k+1} +$$ + +is a divisor of the next one; and since $x_{k}-x_{k+1}=x_{0}-x_{1}$, all these differences have equal absolute values. For $x_{m}=\min \left(x_{1}, \ldots, x_{k}\right)$ this means that $x_{m-1}-x_{m}=-\left(x_{m}-x_{m+1}\right)$. Thus $x_{m-1}=x_{m+1}\left(\neq x_{m}\right)$. It follows that consecutive differences in the sequence (2) have opposite signs. Consequently, $x_{0}, x_{1}, x_{2}, \ldots$ is an alternating sequence of two distinct values. In other words, every integer fixed point of $Q$ is a fixed point of the polynomial $P(P(x))$. Our task is to prove that there are at most $n$ such points. + +Let $a$ be one of them so that $b=P(a) \neq a$ (we have assumed that such an $a$ exists); then $a=P(b)$. Take any other integer fixed point $\alpha$ of $P(P(x))$ and let $P(\alpha)=\beta$, so that $P(\beta)=\alpha$; the numbers $\alpha$ and $\beta$ need not be distinct ( $\alpha$ can be a fixed point of $P$ ), but each of $\alpha, \beta$ is different from each of $a, b$. Applying property (1) to the four pairs of integers $(\alpha, a),(\beta, b)$, $(\alpha, b),(\beta, a)$ we get that the numbers $\alpha-a$ and $\beta-b$ divide each other, and also $\alpha-b$ and $\beta-a$ divide each other. Consequently + +$$ +\alpha-b= \pm(\beta-a), \quad \alpha-a= \pm(\beta-b) . +$$ + +Suppose we have a plus in both instances: $\alpha-b=\beta-a$ and $\alpha-a=\beta-b$. Subtraction yields $a-b=b-a$, a contradiction, as $a \neq b$. Therefore at least one equality in (3) holds with a minus sign. For each of them this means that $\alpha+\beta=a+b$; equivalently $a+b-\alpha-P(\alpha)=0$. + +Denote $a+b$ by $C$. We have shown that every integer fixed point of $Q$ other that $a$ and $b$ is a root of the polynomial $F(x)=C-x-P(x)$. This is of course true for $a$ and $b$ as well. And since $P$ has degree $n>1$, the polynomial $F$ has the same degree, so it cannot have more than $n$ roots. Hence the result. + +Comment. The first part of the solution, showing that integer fixed points of any iterate of $P$ are in fact fixed points of the second iterate $P \circ P$ is standard; moreover, this fact has already appeared in contests. We however do not consider this as a major drawback to the problem because the only tricky moment comes up only at the next stage of the reasoning - to apply the divisibility property (1) to points from distinct 2-orbits of $P$. Yet maybe it would be more appropriate to state the problem in a version involving $k=2$ only. + +N5. Find all integer solutions of the equation + +$$ +\frac{x^{7}-1}{x-1}=y^{5}-1 +$$ + +(Russia) + +Solution. The equation has no integer solutions. To show this, we first prove a lemma. + +Lemma. If $x$ is an integer and $p$ is a prime divisor of $\frac{x^{7}-1}{x-1}$ then either $p \equiv 1(\bmod 7)$ or $p=7$. + +Proof. Both $x^{7}-1$ and $x^{p-1}-1$ are divisible by $p$, by hypothesis and by Fermat's little theorem, respectively. Suppose that 7 does not divide $p-1$. Then $\operatorname{gcd}(p-1,7)=1$, so there exist integers $k$ and $m$ such that $7 k+(p-1) m=1$. We therefore have + +$$ +x \equiv x^{7 k+(p-1) m} \equiv\left(x^{7}\right)^{k} \cdot\left(x^{p-1}\right)^{m} \equiv 1 \quad(\bmod p) +$$ + +and so + +$$ +\frac{x^{7}-1}{x-1}=1+x+\cdots+x^{6} \equiv 7 \quad(\bmod p) +$$ + +It follows that $p$ divides 7 , hence $p=7$ must hold if $p \equiv 1(\bmod 7)$ does not, as stated. + +The lemma shows that each positive divisor $d$ of $\frac{x^{7}-1}{x-1}$ satisfies either $d \equiv 0(\bmod 7)$ or $d \equiv 1(\bmod 7)$. + +Now assume that $(x, y)$ is an integer solution of the original equation. Notice that $y-1>0$, because $\frac{x^{7}-1}{x-1}>0$ for all $x \neq 1$. Since $y-1$ divides $\frac{x^{7}-1}{x-1}=y^{5}-1$, we have $y \equiv 1(\bmod 7)$ or $y \equiv 2(\bmod 7)$ by the previous paragraph. In the first case, $1+y+y^{2}+y^{3}+y^{4} \equiv 5(\bmod 7)$, and in the second $1+y+y^{2}+y^{3}+y^{4} \equiv 3(\bmod 7)$. Both possibilities contradict the fact that the positive divisor $1+y+y^{2}+y^{3}+y^{4}$ of $\frac{x^{7}-1}{x-1}$ is congruent to 0 or 1 modulo 7 . So the given equation has no integer solutions. + +N6. Let $a>b>1$ be relatively prime positive integers. Define the weight of an integer $c$, denoted by $w(c)$, to be the minimal possible value of $|x|+|y|$ taken over all pairs of integers $x$ and $y$ such that + +$$ +a x+b y=c . +$$ + +An integer $c$ is called a local champion if $w(c) \geq w(c \pm a)$ and $w(c) \geq w(c \pm b)$. + +Find all local champions and determine their number. + +Solution. Call the pair of integers $(x, y)$ a representation of $c$ if $a x+b y=c$ and $|x|+|y|$ has the smallest possible value, i.e. $|x|+|y|=w(c)$. + +We characterise the local champions by the following three observations. + +Lemma 1. If $(x, y)$ a representation of a local champion $c$ then $x y<0$. + +Proof. Suppose indirectly that $x \geq 0$ and $y \geq 0$ and consider the values $w(c)$ and $w(c+a)$. All representations of the numbers $c$ and $c+a$ in the form $a u+b v$ can be written as + +$$ +c=a(x-k b)+b(y+k a), \quad c+a=a(x+1-k b)+b(y+k a) +$$ + +where $k$ is an arbitrary integer. + +Since $|x|+|y|$ is minimal, we have + +$$ +x+y=|x|+|y| \leq|x-k b|+|y+k a| +$$ + +for all $k$. On the other hand, $w(c+a) \leq w(c)$, so there exists a $k$ for which + +$$ +|x+1-k b|+|y+k a| \leq|x|+|y|=x+y . +$$ + +Then + +$$ +(x+1-k b)+(y+k a) \leq|x+1-k b|+|y+k a| \leq x+y \leq|x-k b|+|y+k a| . +$$ + +Comparing the first and the third expressions, we find $k(a-b)+1 \leq 0$ implying $k<0$. Comparing the second and fourth expressions, we get $|x+1-k b| \leq|x-k b|$, therefore $k b>x$; this is a contradiction. + +If $x, y \leq 0$ then we can switch to $-c,-x$ and $-y$. + +From this point, write $c=a x-b y$ instead of $c=a x+b y$ and consider only those cases where $x$ and $y$ are nonzero and have the same sign. By Lemma 1, there is no loss of generality in doing so. + +Lemma 2. Let $c=a x-b y$ where $|x|+|y|$ is minimal and $x, y$ have the same sign. The number $c$ is a local champion if and only if $|x|0$. + +The numbers $c-a$ and $c+b$ can be written as + +$$ +c-a=a(x-1)-b y \quad \text { and } \quad c+b=a x-b(y-1) +$$ + +and trivially $w(c-a) \leq(x-1)+y0$. We prove that we can choose $k=1$. + +Consider the function $f(t)=|x+1-b t|+|y-a t|-(x+y)$. This is a convex function and we have $f(0)=1$ and $f(k) \leq 0$. By Jensen's inequality, $f(1) \leq\left(1-\frac{1}{k}\right) f(0)+\frac{1}{k} f(k)<1$. But $f(1)$ is an integer. Therefore $f(1) \leq 0$ and + +$$ +|x+1-b|+|y-a| \leq x+y . +$$ + +Knowing $c=a(x-b)-b(y-a)$, we also have + +$$ +x+y \leq|x-b|+|y-a| . +$$ + +Combining the two inequalities yields $|x+1-b| \leq|x-b|$ which is equivalent to $xb$, we also have $0N$ and + +$$ +2^{b_{i}}+b_{i} \equiv i \quad(\bmod d) . +$$ + +This yields the claim for $m=b_{0}$. + +The base case $d=1$ is trivial. Take an $a>1$ and assume that the statement holds for all $d\max \left(2^{M}, N\right)$ and + +$$ +2^{b_{i}}+b_{i} \equiv i \quad(\bmod d) \quad \text { for } \quad i=0,1,2, \ldots, d-1 +$$ + +For each $i=0,1, \ldots, d-1$ consider the sequence + +$$ +2^{b_{i}}+b_{i}, \quad 2^{b_{i}+k}+\left(b_{i}+k\right), \ldots, \quad 2^{b_{i}+\left(a^{\prime}-1\right) k}+\left(b_{i}+\left(a^{\prime}-1\right) k\right) . +$$ + +Modulo $a$, these numbers are congruent to + +$$ +2^{b_{i}}+b_{i}, \quad 2^{b_{i}}+\left(b_{i}+k\right), \ldots, \quad 2^{b_{i}}+\left(b_{i}+\left(a^{\prime}-1\right) k\right), +$$ + +respectively. The $d$ sequences contain $a^{\prime} d=a$ numbers altogether. We shall now prove that no two of these numbers are congruent modulo $a$. + +Suppose that + +$$ +2^{b_{i}}+\left(b_{i}+m k\right) \equiv 2^{b_{j}}+\left(b_{j}+n k\right) \quad(\bmod a) +$$ + +for some values of $i, j \in\{0,1, \ldots, d-1\}$ and $m, n \in\left\{0,1, \ldots, a^{\prime}-1\right\}$. Since $d$ is a divisor of $a$, we also have + +$$ +2^{b_{i}}+\left(b_{i}+m k\right) \equiv 2^{b_{j}}+\left(b_{j}+n k\right) \quad(\bmod d) . +$$ + +Because $d$ is a divisor of $k$ and in view of $(1)$, we obtain $i \equiv j(\bmod d)$. As $i, j \in\{0,1, \ldots, d-1\}$, this just means that $i=j$. Substituting this into (3) yields $m k \equiv n k(\bmod a)$. Therefore $m k^{\prime} \equiv n k^{\prime}\left(\bmod a^{\prime}\right)$; and since $a^{\prime}$ and $k^{\prime}$ are coprime, we get $m \equiv n\left(\bmod a^{\prime}\right)$. Hence also $m=n$. + +It follows that the $a$ numbers that make up the $d$ sequences (2) satisfy all the requirements; they are certainly all greater than $N$ because we chose each $b_{i}>\max \left(2^{M}, N\right)$. So the statement holds for $a$, completing the induction. + diff --git a/IMO/md/en-IMO2007SL.md b/IMO/md/en-IMO2007SL.md new file mode 100644 index 0000000000000000000000000000000000000000..f4a0188b37dab5b2f2b37ff0c623327a1144c32c --- /dev/null +++ b/IMO/md/en-IMO2007SL.md @@ -0,0 +1,2314 @@ +$48^{\text {th }}$ International + +Mathematical + +Olympiad + +VIETNAM 2007 + +![](https://cdn.mathpix.com/cropped/2024_04_17_b8b307b9422db9068910g-01.jpg?height=311&width=531&top_left_y=144&top_left_x=1505) + +IMO 2007 + +Shortlisted + +Problems + +with Solutions + +July 19-31, 2007 +$48^{\text {th }}$ International Mathematical Olympiad Vietnam 2007 + +Shortlisted Problems with Solutions + +## Contents + +Contributing Countries \& Problem Selection Committee ..... 5 +Algebra ..... 7 +Problem A1 ..... 7 +Problem A2 ..... 10 +Problem A3 ..... 12 +Problem A4 ..... 14 +Problem A5 ..... 16 +Problem A6 ..... 20 +Problem A7 ..... 22 +Combinatorics ..... 25 +Problem C1 ..... 25 +Problem C2 ..... 28 +Problem C3 ..... 30 +Problem C4 ..... 31 +Problem C5 ..... 32 +Problem C6 ..... 34 +Problem C7 ..... 36 +Problem C8 ..... 37 +Geometry ..... 39 +Problem G1 ..... 39 +Problem G2 ..... 41 +Problem G3 ..... 42 +Problem G4 ..... 43 +Problem G5 ..... 44 +Problem G6 ..... 46 +Problem G7 ..... 49 +Problem G8 ..... 52 +Number Theory ..... 55 +Problem N1 ..... 55 +Problem N2 ..... 56 +Problem N3 ..... 57 +Problem N4 ..... 58 +Problem N5 ..... 60 +Problem N6 ..... 62 +Problem N7 ..... 63 + +# Contributing Countries + +Austria, Australia, Belgium, Bulgaria, Canada, Croatia,
Czech Republic, Estonia, Finland, Greece, India, Indonesia, Iran,
Japan, Korea (North), Korea (South), Lithuania, Luxembourg,
Mexico, Moldova, Netherlands, New Zealand, Poland, Romania,
Russia, Serbia, South Africa, Sweden, Thailand, Taiwan, Turkey,
Ukraine, United Kingdom, United States of America + +## Problem Selection Committee + +Ha Huy Khoai
Ilya Bogdanov
Tran Nam Dung
Le Tuan Hoa
Géza Kós + +## Algebra + +A1. Given a sequence $a_{1}, a_{2}, \ldots, a_{n}$ of real numbers. For each $i(1 \leq i \leq n)$ define + +$$ +d_{i}=\max \left\{a_{j}: 1 \leq j \leq i\right\}-\min \left\{a_{j}: i \leq j \leq n\right\} +$$ + +and let + +$$ +d=\max \left\{d_{i}: 1 \leq i \leq n\right\} +$$ + +(a) Prove that for arbitrary real numbers $x_{1} \leq x_{2} \leq \ldots \leq x_{n}$, + +$$ +\max \left\{\left|x_{i}-a_{i}\right|: 1 \leq i \leq n\right\} \geq \frac{d}{2} +$$ + +(b) Show that there exists a sequence $x_{1} \leq x_{2} \leq \ldots \leq x_{n}$ of real numbers such that we have equality in (1). + +(New Zealand) + +Solution 1. (a) Let $1 \leq p \leq q \leq r \leq n$ be indices for which + +$$ +d=d_{q}, \quad a_{p}=\max \left\{a_{j}: 1 \leq j \leq q\right\}, \quad a_{r}=\min \left\{a_{j}: q \leq j \leq n\right\} +$$ + +and thus $d=a_{p}-a_{r}$. (These indices are not necessarily unique.) + +![](https://cdn.mathpix.com/cropped/2024_04_17_b8b307b9422db9068910g-08.jpg?height=363&width=797&top_left_y=1669&top_left_x=595) + +For arbitrary real numbers $x_{1} \leq x_{2} \leq \ldots \leq x_{n}$, consider just the two quantities $\left|x_{p}-a_{p}\right|$ and $\left|x_{r}-a_{r}\right|$. Since + +$$ +\left(a_{p}-x_{p}\right)+\left(x_{r}-a_{r}\right)=\left(a_{p}-a_{r}\right)+\left(x_{r}-x_{p}\right) \geq a_{p}-a_{r}=d, +$$ + +we have either $a_{p}-x_{p} \geq \frac{d}{2}$ or $x_{r}-a_{r} \geq \frac{d}{2}$. Hence, + +$$ +\max \left\{\left|x_{i}-a_{i}\right|: 1 \leq i \leq n\right\} \geq \max \left\{\left|x_{p}-a_{p}\right|,\left|x_{r}-a_{r}\right|\right\} \geq \max \left\{a_{p}-x_{p}, x_{r}-a_{r}\right\} \geq \frac{d}{2} +$$ + +(b) Define the sequence $\left(x_{k}\right)$ as + +$$ +x_{1}=a_{1}-\frac{d}{2}, \quad x_{k}=\max \left\{x_{k-1}, a_{k}-\frac{d}{2}\right\} \quad \text { for } 2 \leq k \leq n +$$ + +We show that we have equality in (1) for this sequence. + +By the definition, sequence $\left(x_{k}\right)$ is non-decreasing and $x_{k}-a_{k} \geq-\frac{d}{2}$ for all $1 \leq k \leq n$. Next we prove that + +$$ +x_{k}-a_{k} \leq \frac{d}{2} \quad \text { for all } 1 \leq k \leq n +$$ + +Consider an arbitrary index $1 \leq k \leq n$. Let $\ell \leq k$ be the smallest index such that $x_{k}=x_{\ell}$. We have either $\ell=1$, or $\ell \geq 2$ and $x_{\ell}>x_{\ell-1}$. In both cases, + +$$ +x_{k}=x_{\ell}=a_{\ell}-\frac{d}{2} +$$ + +Since + +$$ +a_{\ell}-a_{k} \leq \max \left\{a_{j}: 1 \leq j \leq k\right\}-\min \left\{a_{j}: k \leq j \leq n\right\}=d_{k} \leq d +$$ + +equality (3) implies + +$$ +x_{k}-a_{k}=a_{\ell}-a_{k}-\frac{d}{2} \leq d-\frac{d}{2}=\frac{d}{2} +$$ + +We obtained that $-\frac{d}{2} \leq x_{k}-a_{k} \leq \frac{d}{2}$ for all $1 \leq k \leq n$, so + +$$ +\max \left\{\left|x_{i}-a_{i}\right|: 1 \leq i \leq n\right\} \leq \frac{d}{2} +$$ + +We have equality because $\left|x_{1}-a_{1}\right|=\frac{d}{2}$. + +Solution 2. We present another construction of a sequence $\left(x_{i}\right)$ for part (b). + +For each $1 \leq i \leq n$, let + +$$ +M_{i}=\max \left\{a_{j}: 1 \leq j \leq i\right\} \quad \text { and } \quad m_{i}=\min \left\{a_{j}: i \leq j \leq n\right\} +$$ + +For all $1 \leq in$, by (1) we have + +$$ +f(m)=f(n+(m-n)) \geq f(n)+f(f(m-n))-1 \geq f(n), +$$ + +so $f$ is nondecreasing. + +Function $f \equiv 1$ is an obvious solution. To find other solutions, assume that $f \not \equiv 1$ and take the smallest $a \in \mathbb{N}$ such that $f(a)>1$. Then $f(b) \geq f(a)>1$ for all integer $b \geq a$. + +Suppose that $f(n)>n$ for some $n \in \mathbb{N}$. Then we have + +$$ +f(f(n))=f((f(n)-n)+n) \geq f(f(n)-n)+f(f(n))-1 +$$ + +so $f(f(n)-n) \leq 1$ and hence $f(n)-n2007 ; +\end{array} \quad h_{j}(n)=\max \left\{1,\left\lfloor\frac{j n}{2007}\right\rfloor\right\}\right. +$$ + +Also the example for $j=2008$ can be generalized. In particular, choosing a divisor $d>1$ of 2007 , one can set + +$$ +f_{2008, d}(n)= \begin{cases}n, & d \nless\{n \\ n+1, & d \mid n\end{cases} +$$ + +A3. Let $n$ be a positive integer, and let $x$ and $y$ be positive real numbers such that $x^{n}+y^{n}=1$. Prove that + +$$ +\left(\sum_{k=1}^{n} \frac{1+x^{2 k}}{1+x^{4 k}}\right)\left(\sum_{k=1}^{n} \frac{1+y^{2 k}}{1+y^{4 k}}\right)<\frac{1}{(1-x)(1-y)} +$$ + +(Estonia) + +Solution 1. For each real $t \in(0,1)$, + +$$ +\frac{1+t^{2}}{1+t^{4}}=\frac{1}{t}-\frac{(1-t)\left(1-t^{3}\right)}{t\left(1+t^{4}\right)}<\frac{1}{t} +$$ + +Substituting $t=x^{k}$ and $t=y^{k}$, + +$$ +0<\sum_{k=1}^{n} \frac{1+x^{2 k}}{1+x^{4 k}}<\sum_{k=1}^{n} \frac{1}{x^{k}}=\frac{1-x^{n}}{x^{n}(1-x)} \quad \text { and } \quad 0<\sum_{k=1}^{n} \frac{1+y^{2 k}}{1+y^{4 k}}<\sum_{k=1}^{n} \frac{1}{y^{k}}=\frac{1-y^{n}}{y^{n}(1-y)} +$$ + +Since $1-y^{n}=x^{n}$ and $1-x^{n}=y^{n}$, + +$$ +\frac{1-x^{n}}{x^{n}(1-x)}=\frac{y^{n}}{x^{n}(1-x)}, \quad \frac{1-y^{n}}{y^{n}(1-y)}=\frac{x^{n}}{y^{n}(1-y)} +$$ + +and therefore + +$$ +\left(\sum_{k=1}^{n} \frac{1+x^{2 k}}{1+x^{4 k}}\right)\left(\sum_{k=1}^{n} \frac{1+y^{2 k}}{1+y^{4 k}}\right)<\frac{y^{n}}{x^{n}(1-x)} \cdot \frac{x^{n}}{y^{n}(1-y)}=\frac{1}{(1-x)(1-y)} . +$$ + +Solution 2. We prove + +$$ +\left(\sum_{k=1}^{n} \frac{1+x^{2 k}}{1+x^{4 k}}\right)\left(\sum_{k=1}^{n} \frac{1+y^{2 k}}{1+y^{4 k}}\right)<\frac{\left(\frac{1+\sqrt{2}}{2} \ln 2\right)^{2}}{(1-x)(1-y)}<\frac{0.7001}{(1-x)(1-y)} +$$ + +The idea is to estimate each term on the left-hand side with the same constant. To find the upper bound for the expression $\frac{1+x^{2 k}}{1+x^{4 k}}$, consider the function $f(t)=\frac{1+t}{1+t^{2}}$ in interval $(0,1)$. Since + +$$ +f^{\prime}(t)=\frac{1-2 t-t^{2}}{\left(1+t^{2}\right)^{2}}=\frac{(\sqrt{2}+1+t)(\sqrt{2}-1-t)}{\left(1+t^{2}\right)^{2}} +$$ + +the function increases in interval $(0, \sqrt{2}-1]$ and decreases in $[\sqrt{2}-1,1)$. Therefore the maximum is at point $t_{0}=\sqrt{2}-1$ and + +$$ +f(t)=\frac{1+t}{1+t^{2}} \leq f\left(t_{0}\right)=\frac{1+\sqrt{2}}{2}=\alpha . +$$ + +Applying this to each term on the left-hand side of (1), we obtain + +$$ +\left(\sum_{k=1}^{n} \frac{1+x^{2 k}}{1+x^{4 k}}\right)\left(\sum_{k=1}^{n} \frac{1+y^{2 k}}{1+y^{4 k}}\right) \leq n \alpha \cdot n \alpha=(n \alpha)^{2} +$$ + +To estimate $(1-x)(1-y)$ on the right-hand side, consider the function + +$$ +g(t)=\ln \left(1-t^{1 / n}\right)+\ln \left(1-(1-t)^{1 / n}\right) . +$$ + +Substituting $s$ for $1-t$, we have + +$$ +-n g^{\prime}(t)=\frac{t^{1 / n-1}}{1-t^{1 / n}}-\frac{s^{1 / n-1}}{1-s^{1 / n}}=\frac{1}{s t}\left(\frac{(1-t) t^{1 / n}}{1-t^{1 / n}}-\frac{(1-s) s^{1 / n}}{1-s^{1 / n}}\right)=\frac{h(t)-h(s)}{s t} . +$$ + +The function + +$$ +h(t)=t^{1 / n} \frac{1-t}{1-t^{1 / n}}=\sum_{i=1}^{n} t^{i / n} +$$ + +is obviously increasing for $t \in(0,1)$, hence for these values of $t$ we have + +$$ +g^{\prime}(t)>0 \Longleftrightarrow h(t)y$ for all $y \in \mathbb{R}^{+}$. Functional equation (1) yields $f(x+f(y))>f(x+y)$ and hence $f(y) \neq y$ immediately. If $f(y)f(y) +$$ + +contradiction. Therefore $f(y)>y$ for all $y \in \mathbb{R}^{+}$. + +For $x \in \mathbb{R}^{+}$define $g(x)=f(x)-x$; then $f(x)=g(x)+x$ and, as we have seen, $g(x)>0$. Transforming (1) for function $g(x)$ and setting $t=x+y$, + +$$ +\begin{aligned} +f(t+g(y)) & =f(t)+f(y) \\ +g(t+g(y))+t+g(y) & =(g(t)+t)+(g(y)+y) +\end{aligned} +$$ + +and therefore + +$$ +g(t+g(y))=g(t)+y \quad \text { for all } t>y>0 +$$ + +Next we prove that function $g(x)$ is injective. Suppose that $g\left(y_{1}\right)=g\left(y_{2}\right)$ for some numbers $y_{1}, y_{2} \in \mathbb{R}^{+}$. Then by $(2)$, + +$$ +g(t)+y_{1}=g\left(t+g\left(y_{1}\right)\right)=g\left(t+g\left(y_{2}\right)\right)=g(t)+y_{2} +$$ + +for all $t>\max \left\{y_{1}, y_{2}\right\}$. Hence, $g\left(y_{1}\right)=g\left(y_{2}\right)$ is possible only if $y_{1}=y_{2}$. + +Now let $u, v$ be arbitrary positive numbers and $t>u+v$. Applying (2) three times, + +$$ +g(t+g(u)+g(v))=g(t+g(u))+v=g(t)+u+v=g(t+g(u+v)) \text {. } +$$ + +By the injective property we conclude that $t+g(u)+g(v)=t+g(u+v)$, hence + +$$ +g(u)+g(v)=g(u+v) . +$$ + +Since function $g(v)$ is positive, equation (3) also shows that $g$ is an increasing function. + +Finally we prove that $g(x)=x$. Combining (2) and (3), we obtain + +$$ +g(t)+y=g(t+g(y))=g(t)+g(g(y)) +$$ + +and hence + +$$ +g(g(y))=y +$$ + +Suppose that there exists an $x \in \mathbb{R}^{+}$such that $g(x) \neq x$. By the monotonicity of $g$, if $x>g(x)$ then $g(x)>g(g(x))=x$. Similarly, if $x0$ ) imply $g(x)=c x$. So, after proving (3), it is sufficient to test functions $f(x)=(c+1) x$. + +Solution 2. We prove that $f(y)>y$ and introduce function $g(x)=f(x)-x>0$ in the same way as in Solution 1. + +For arbitrary $t>y>0$, substitute $x=t-y$ into (1) to obtain + +$$ +f(t+g(y))=f(t)+f(y) +$$ + +which, by induction, implies + +$$ +f(t+n g(y))=f(t)+n f(y) \quad \text { for all } t>y>0, n \in \mathbb{N} \text {. } +$$ + +Take two arbitrary positive reals $y$ and $z$ and a third fixed number $t>\max \{y, z\}$. For each positive integer $k$, let $\ell_{k}=\left\lfloor k \frac{g(y)}{g(z)}\right\rfloor$. Then $t+k g(y)-\ell_{k} g(z) \geq t>z$ and, applying (4) twice, + +$$ +\begin{aligned} +f\left(t+k g(y)-\ell_{k} g(z)\right)+\ell_{k} f(z) & =f(t+k g(y))=f(t)+k f(y) \\ +0<\frac{1}{k} f\left(t+k g(y)-\ell_{k} g(z)\right) & =\frac{f(t)}{k}+f(y)-\frac{\ell_{k}}{k} f(z) . +\end{aligned} +$$ + +As $k \rightarrow \infty$ we get + +$$ +0 \leq \lim _{k \rightarrow \infty}\left(\frac{f(t)}{k}+f(y)-\frac{\ell_{k}}{k} f(z)\right)=f(y)-\frac{g(y)}{g(z)} f(z)=f(y)-\frac{f(y)-y}{f(z)-z} f(z) +$$ + +and therefore + +$$ +\frac{f(y)}{y} \leq \frac{f(z)}{z} +$$ + +Exchanging variables $y$ and $z$, we obtain the reverse inequality. Hence, $\frac{f(y)}{y}=\frac{f(z)}{z}$ for arbitrary $y$ and $z$; so function $\frac{f(x)}{x}$ is constant, $f(x)=c x$. + +Substituting back into (1), we find that $f(x)=c x$ is a solution if and only if $c=2$. So the only solution for the problem is $f(x)=2 x$. + +A5. Let $c>2$, and let $a(1), a(2), \ldots$ be a sequence of nonnegative real numbers such that + +$$ +a(m+n) \leq 2 a(m)+2 a(n) \text { for all } m, n \geq 1 \text {, } +$$ + +and + +$$ +a\left(2^{k}\right) \leq \frac{1}{(k+1)^{c}} \quad \text { for all } k \geq 0 +$$ + +Prove that the sequence $a(n)$ is bounded. + +(Croatia) + +Solution 1. For convenience, define $a(0)=0$; then condition (1) persists for all pairs of nonnegative indices. + +Lemma 1. For arbitrary nonnegative indices $n_{1}, \ldots, n_{k}$, we have + +$$ +a\left(\sum_{i=1}^{k} n_{i}\right) \leq \sum_{i=1}^{k} 2^{i} a\left(n_{i}\right) +$$ + +and + +$$ +a\left(\sum_{i=1}^{k} n_{i}\right) \leq 2 k \sum_{i=1}^{k} a\left(n_{i}\right) +$$ + +Proof. Inequality (3) is proved by induction on $k$. The base case $k=1$ is trivial, while the induction step is provided by + +$a\left(\sum_{i=1}^{k+1} n_{i}\right)=a\left(n_{1}+\sum_{i=2}^{k+1} n_{i}\right) \leq 2 a\left(n_{1}\right)+2 a\left(\sum_{i=1}^{k} n_{i+1}\right) \leq 2 a\left(n_{1}\right)+2 \sum_{i=1}^{k} 2^{i} a\left(n_{i+1}\right)=\sum_{i=1}^{k+1} 2^{i} a\left(n_{i}\right)$. + +To establish (4), first the inequality + +$$ +a\left(\sum_{i=1}^{2^{d}} n_{i}\right) \leq 2^{d} \sum_{i=1}^{2^{d}} a\left(n_{i}\right) +$$ + +can be proved by an obvious induction on $d$. Then, turning to (4), we find an integer $d$ such that $2^{d-1}d$, and take some positive integer $f$ such that $M_{f}>d$. Applying (3), we get + +$$ +a(n)=a\left(\sum_{k=1}^{f} \sum_{M_{k-1} \leq i2$. We can assume that $s_{1} \leq s_{2} \leq \cdots \leq s_{k}$. Note that + +$$ +\sum_{i=1}^{k-1} 2^{-s_{i}} \leq 1-2^{-s_{k-1}} +$$ + +since the left-hand side is a fraction with the denominator $2^{s_{k-1}}$, and this fraction is less than 1. Define $s_{k-1}^{\prime}=s_{k-1}-1$ and $n_{k-1}^{\prime}=n_{k-1}+n_{k}$; then we have + +$$ +\sum_{i=1}^{k-2} 2^{-s_{i}}+2^{-s_{k-1}^{\prime}} \leq\left(1-2 \cdot 2^{-s_{k-1}}\right)+2^{1-s_{k-1}}=1 . +$$ + +Now, the induction hypothesis can be applied to achieve + +$$ +\begin{aligned} +a\left(\sum_{i=1}^{k} n_{i}\right)=a\left(\sum_{i=1}^{k-2} n_{i}+n_{k-1}^{\prime}\right) & \leq \sum_{i=1}^{k-2} 2^{s_{i}} a\left(n_{i}\right)+2^{s_{k-1}^{\prime}} a\left(n_{k-1}^{\prime}\right) \\ +& \leq \sum_{i=1}^{k-2} 2^{s_{i}} a\left(n_{i}\right)+2^{s_{k-1}-1} \cdot 2\left(a\left(n_{k-1}\right)+a\left(n_{k}\right)\right) \\ +& \leq \sum_{i=1}^{k-2} 2^{s_{i}} a\left(n_{i}\right)+2^{s_{k-1}} a\left(n_{k-1}\right)+2^{s_{k}} a\left(n_{k}\right) . +\end{aligned} +$$ + +Let $q=c / 2>1$. Take an arbitrary positive integer $n$ and write + +$$ +n=\sum_{i=1}^{k} 2^{u_{i}}, \quad 0 \leq u_{1}1$. + +Comment 1. In fact, Lemma 2 (applied to the case $n_{i}=2^{u_{i}}$ only) provides a sharp bound for any $a(n)$. Actually, let $b(k)=\frac{1}{(k+1)^{c}}$ and consider the sequence + +$$ +a(n)=\min \left\{\sum_{i=1}^{k} 2^{s_{i}} b\left(u_{i}\right) \mid k \in \mathbb{N}, \quad \sum_{i=1}^{k} 2^{-s_{i}} \leq 1, \quad \sum_{i=1}^{k} 2^{u_{i}}=n\right\} +$$ + +We show that this sequence satisfies the conditions of the problem. Take two arbitrary indices $m$ and $n$. Let + +$$ +\begin{aligned} +& a(m)=\sum_{i=1}^{k} 2^{s_{i}} b\left(u_{i}\right), \quad \sum_{i=1}^{k} 2^{-s_{i}} \leq 1, \quad \sum_{i=1}^{k} 2^{u_{i}}=m ; \\ +& a(n)=\sum_{i=1}^{l} 2^{r_{i}} b\left(w_{i}\right), \quad \sum_{i=1}^{l} 2^{-r_{i}} \leq 1, \quad \sum_{i=1}^{l} 2^{w_{i}}=n . +\end{aligned} +$$ + +Then we have + +$$ +\sum_{i=1}^{k} 2^{-1-s_{i}}+\sum_{i=1}^{l} 2^{-1-r_{i}} \leq \frac{1}{2}+\frac{1}{2}=1, \quad \sum_{i=1}^{k} 2^{u_{i}}+\sum_{i=1}^{l} 2^{w_{i}}=m+n +$$ + +so by (5) we obtain + +$$ +a(n+m) \leq \sum_{i=1}^{k} 2^{1+s_{i}} b\left(u_{i}\right)+\sum_{i=1}^{l} 2^{1+r_{i}} b\left(w_{i}\right)=2 a(m)+2 a(n) . +$$ + +Comment 2. The condition $c>2$ is sharp; we show that the sequence (5) is not bounded if $c \leq 2$. + +First, we prove that for an arbitrary $n$ the minimum in (5) is attained with a sequence $\left(u_{i}\right)$ consisting of distinct numbers. To the contrary, assume that $u_{k-1}=u_{k}$. Replace $u_{k-1}$ and $u_{k}$ by a single number $u_{k-1}^{\prime}=u_{k}+1$, and $s_{k-1}$ and $s_{k}$ by $s_{k-1}^{\prime}=\min \left\{s_{k-1}, s_{k}\right\}$. The modified sequences provide a better bound since + +$$ +2^{s_{k-1}^{\prime}} b\left(u_{k-1}^{\prime}\right)=2^{s_{k-1}^{\prime}} b\left(u_{k}+1\right)<2^{s_{k-1}} b\left(u_{k-1}\right)+2^{s_{k}} b\left(u_{k}\right) +$$ + +(we used the fact that $b(k)$ is decreasing). This is impossible. + +Hence, the claim is proved, and we can assume that the minimum is attained with $u_{1}<\cdots0.4513$. + +Comment 3. The solution can be improved in several ways to give somewhat better bounds for $c_{n}$. Here we show a variant which proves $c_{n}<0.4589$ for $n \geq 5$. + +The value of $c_{n}$ does not change if negative values are also allowed in (3). So the problem is equivalent to maximizing + +$$ +f\left(a_{1}, a_{2}, \ldots, a_{n}\right)=a_{1}^{2} a_{2}+a_{2}^{2} a_{3}+\ldots+a_{n}^{2} a_{1} +$$ + +on the unit sphere $a_{1}^{2}+a_{2}^{2}+\ldots+a_{n}^{2}=1$ in $\mathbb{R}^{n}$. Since the unit sphere is compact, the function has a maximum and we can apply the Lagrange multiplier method; for each maximum point there exists a real number $\lambda$ such that + +$$ +a_{k-1}^{2}+2 a_{k} a_{k+1}=\lambda \cdot 2 a_{k} \quad \text { for all } k=1,2, \ldots, n +$$ + +Then + +$$ +3 S=\sum_{k=1}^{n}\left(a_{k-1}^{2} a_{k}+2 a_{k}^{2} a_{k+1}\right)=\sum_{k=1}^{n} 2 \lambda a_{k}^{2}=2 \lambda +$$ + +and therefore + +$$ +a_{k-1}^{2}+2 a_{k} a_{k+1}=3 S a_{k} \quad \text { for all } k=1,2, \ldots, n \text {. } +$$ + +From (4) we can derive + +$$ +9 S^{2}=\sum_{k=1}^{n}\left(3 S a_{k}\right)^{2}=\sum_{k=1}^{n}\left(a_{k-1}^{2}+2 a_{k} a_{k+1}\right)^{2}=\sum_{k=1}^{n} a_{k}^{4}+4 \sum_{k=1}^{n} a_{k}^{2} a_{k+1}^{2}+4 \sum_{k=1}^{n} a_{k}^{2} a_{k+1} a_{k+2} +$$ + +and + +$$ +3 S^{2}=\sum_{k=1}^{n} 3 S a_{k-1}^{2} a_{k}=\sum_{k=1}^{n} a_{k-1}^{2}\left(a_{k-1}^{2}+2 a_{k} a_{k+1}\right)=\sum_{k=1}^{n} a_{k}^{4}+2 \sum_{k=1}^{n} a_{k}^{2} a_{k+1} a_{k+2} . +$$ + +Let $p$ be a positive number. Combining (5) and (6) and applying the AM-GM inequality, + +$$ +\begin{aligned} +(9+3 p) S^{2} & =(1+p) \sum_{k=1}^{n} a_{k}^{4}+4 \sum_{k=1}^{n} a_{k}^{2} a_{k+1}^{2}+(4+2 p) \sum_{k=1}^{n} a_{k}^{2} a_{k+1} a_{k+2} \\ +& \leq(1+p) \sum_{k=1}^{n} a_{k}^{4}+4 \sum_{k=1}^{n} a_{k}^{2} a_{k+1}^{2}+\sum_{k=1}^{n}\left(2(1+p) a_{k}^{2} a_{k+2}^{2}+\frac{(2+p)^{2}}{2(1+p)} a_{k}^{2} a_{k+1}^{2}\right) \\ +& =(1+p) \sum_{k=1}^{n}\left(a_{k}^{4}+2 a_{k}^{2} a_{k+1}^{2}+2 a_{k}^{2} a_{k+2}^{2}\right)+\left(4+\frac{(2+p)^{2}}{2(1+p)}-2(1+p)\right) \sum_{k=1}^{n} a_{k}^{2} a_{k+1}^{2} \\ +& \leq(1+p)\left(\sum_{k=1}^{n} a_{k}^{2}\right)^{2}+\frac{8+4 p-3 p^{2}}{2(1+p)} \sum_{k=1}^{n} a_{k}^{2} a_{k+1}^{2} \\ +& =(1+p)+\frac{8+4 p-3 p^{2}}{2(1+p)} \sum_{k=1}^{n} a_{k}^{2} a_{k+1}^{2} . +\end{aligned} +$$ + +Setting $p=\frac{2+2 \sqrt{7}}{3}$ which is the positive root of $8+4 p-3 p^{2}=0$, we obtain + +$$ +S \leq \sqrt{\frac{1+p}{9+3 p}}=\sqrt{\frac{5+2 \sqrt{7}}{33+6 \sqrt{7}}} \approx 0.458879 . +$$ + +A7. Let $n>1$ be an integer. In the space, consider the set + +$$ +S=\{(x, y, z) \mid x, y, z \in\{0,1, \ldots, n\}, x+y+z>0\} +$$ + +Find the smallest number of planes that jointly contain all $(n+1)^{3}-1$ points of $S$ but none of them passes through the origin. + +(Netherlands) + +Answer. $3 n$ planes. + +Solution. It is easy to find $3 n$ such planes. For example, planes $x=i, y=i$ or $z=i$ $(i=1,2, \ldots, n)$ cover the set $S$ but none of them contains the origin. Another such collection consists of all planes $x+y+z=k$ for $k=1,2, \ldots, 3 n$. + +We show that $3 n$ is the smallest possible number. + +Lemma 1. Consider a nonzero polynomial $P\left(x_{1}, \ldots, x_{k}\right)$ in $k$ variables. Suppose that $P$ vanishes at all points $\left(x_{1}, \ldots, x_{k}\right)$ such that $x_{1}, \ldots, x_{k} \in\{0,1, \ldots, n\}$ and $x_{1}+\cdots+x_{k}>0$, while $P(0,0, \ldots, 0) \neq 0$. Then $\operatorname{deg} P \geq k n$. + +Proof. We use induction on $k$. The base case $k=0$ is clear since $P \neq 0$. Denote for clarity $y=x_{k}$. + +Let $R\left(x_{1}, \ldots, x_{k-1}, y\right)$ be the residue of $P$ modulo $Q(y)=y(y-1) \ldots(y-n)$. Polynomial $Q(y)$ vanishes at each $y=0,1, \ldots, n$, hence $P\left(x_{1}, \ldots, x_{k-1}, y\right)=R\left(x_{1}, \ldots, x_{k-1}, y\right)$ for all $x_{1}, \ldots, x_{k-1}, y \in\{0,1, \ldots, n\}$. Therefore, $R$ also satisfies the condition of the Lemma; moreover, $\operatorname{deg}_{y} R \leq n$. Clearly, $\operatorname{deg} R \leq \operatorname{deg} P$, so it suffices to prove that $\operatorname{deg} R \geq n k$. + +Now, expand polynomial $R$ in the powers of $y$ : + +$$ +R\left(x_{1}, \ldots, x_{k-1}, y\right)=R_{n}\left(x_{1}, \ldots, x_{k-1}\right) y^{n}+R_{n-1}\left(x_{1}, \ldots, x_{k-1}\right) y^{n-1}+\cdots+R_{0}\left(x_{1}, \ldots, x_{k-1}\right) +$$ + +We show that polynomial $R_{n}\left(x_{1}, \ldots, x_{k-1}\right)$ satisfies the condition of the induction hypothesis. + +Consider the polynomial $T(y)=R(0, \ldots, 0, y)$ of degree $\leq n$. This polynomial has $n$ roots $y=1, \ldots, n$; on the other hand, $T(y) \not \equiv 0$ since $T(0) \neq 0$. Hence $\operatorname{deg} T=n$, and its leading coefficient is $R_{n}(0,0, \ldots, 0) \neq 0$. In particular, in the case $k=1$ we obtain that coefficient $R_{n}$ is nonzero. + +Similarly, take any numbers $a_{1}, \ldots, a_{k-1} \in\{0,1, \ldots, n\}$ with $a_{1}+\cdots+a_{k-1}>0$. Substituting $x_{i}=a_{i}$ into $R\left(x_{1}, \ldots, x_{k-1}, y\right)$, we get a polynomial in $y$ which vanishes at all points $y=0, \ldots, n$ and has degree $\leq n$. Therefore, this polynomial is null, hence $R_{i}\left(a_{1}, \ldots, a_{k-1}\right)=0$ for all $i=0,1, \ldots, n$. In particular, $R_{n}\left(a_{1}, \ldots, a_{k-1}\right)=0$. + +Thus, the polynomial $R_{n}\left(x_{1}, \ldots, x_{k-1}\right)$ satisfies the condition of the induction hypothesis. So, we have $\operatorname{deg} R_{n} \geq(k-1) n$ and $\operatorname{deg} P \geq \operatorname{deg} R \geq \operatorname{deg} R_{n}+n \geq k n$. + +Now we can finish the solution. Suppose that there are $N$ planes covering all the points of $S$ but not containing the origin. Let their equations be $a_{i} x+b_{i} y+c_{i} z+d_{i}=0$. Consider the polynomial + +$$ +P(x, y, z)=\prod_{i=1}^{N}\left(a_{i} x+b_{i} y+c_{i} z+d_{i}\right) +$$ + +It has total degree $N$. This polynomial has the property that $P\left(x_{0}, y_{0}, z_{0}\right)=0$ for any $\left(x_{0}, y_{0}, z_{0}\right) \in S$, while $P(0,0,0) \neq 0$. Hence by Lemma 1 we get $N=\operatorname{deg} P \geq 3 n$, as desired. + +Comment 1. There are many other collections of $3 n$ planes covering the set $S$ but not covering the origin. + +Solution 2. We present a different proof of the main Lemma 1. Here we confine ourselves to the case $k=3$, which is applied in the solution, and denote the variables by $x, y$ and $z$. (The same proof works for the general statement as well.) + +The following fact is known with various proofs; we provide one possible proof for the completeness. + +Lemma 2. For arbitrary integers $0 \leq m1$ be an integer. Find all sequences $a_{1}, a_{2}, \ldots, a_{n^{2}+n}$ satisfying the following conditions: + +(a) $a_{i} \in\{0,1\}$ for all $1 \leq i \leq n^{2}+n$; + +(b) $a_{i+1}+a_{i+2}+\ldots+a_{i+n}0$. By (2), it contains some "ones". Let the first "one" in this block be at the $u$ th position (that is, $\left.a_{u+v n}=1\right)$. By the induction hypothesis, the $(v-1)$ th and $v$ th blocks of $\left(a_{i}\right)$ have the form + +$$ +(\underbrace{0 \ldots 0 \ldots 0}_{n-v+1} \underbrace{1 \ldots 1}_{v-1})(\underbrace{0 \ldots 0}_{u-1} 1 * \ldots *), +$$ + +where each star can appear to be any binary digit. Observe that $u \leq n-v+1$, since the sum in this block is $v$. Then, the fragment of length $n$ bracketed above has exactly $(v-1)+1$ ones, i. e. $S(u+(v-1) n, u+v n]=v$. Hence, + +$$ +v=S(u+(v-1) n, u+v n]1$ rectangles such that their sides are parallel to the sides of the square. Any line, parallel to a side of the square and intersecting its interior, also intersects the interior of some rectangle. Prove that in this dissection, there exists a rectangle having no point on the boundary of the square. + +(Japan) + +Solution 1. Call the directions of the sides of the square horizontal and vertical. A horizontal or vertical line, which intersects the interior of the square but does not intersect the interior of any rectangle, will be called a splitting line. A rectangle having no point on the boundary of the square will be called an interior rectangle. + +Suppose, to the contrary, that there exists a dissection of the square into more than one rectangle, such that no interior rectangle and no splitting line appear. Consider such a dissection with the least possible number of rectangles. Notice that this number of rectangles is greater than 2, otherwise their common side provides a splitting line. + +If there exist two rectangles having a common side, then we can replace them by their union (see Figure 1). The number of rectangles was greater than 2, so in a new dissection it is greater than 1. Clearly, in the new dissection, there is also no splitting line as well as no interior rectangle. This contradicts the choice of the original dissection. + +Denote the initial square by $A B C D$, with $A$ and $B$ being respectively the lower left and lower right vertices. Consider those two rectangles $a$ and $b$ containing vertices $A$ and $B$, respectively. (Note that $a \neq b$, otherwise its top side provides a splitting line.) We can assume that the height of $a$ is not greater than that of $b$. Then consider the rectangle $c$ neighboring to the lower right corner of $a$ (it may happen that $c=b$ ). By aforementioned, the heights of $a$ and $c$ are distinct. Then two cases are possible. + +![](https://cdn.mathpix.com/cropped/2024_04_17_b8b307b9422db9068910g-29.jpg?height=214&width=323&top_left_y=1549&top_left_x=250) + +Figure 1 + +![](https://cdn.mathpix.com/cropped/2024_04_17_b8b307b9422db9068910g-29.jpg?height=339&width=580&top_left_y=1481&top_left_x=658) + +Figure 2 + +![](https://cdn.mathpix.com/cropped/2024_04_17_b8b307b9422db9068910g-29.jpg?height=339&width=583&top_left_y=1481&top_left_x=1322) + +Figure 3 + +Case 1. The height of $c$ is less than that of $a$. Consider the rectangle $d$ which is adjacent to both $a$ and $c$, i. e. the one containing the angle marked in Figure 2. This rectangle has no common point with $B C$ (since $a$ is not higher than $b$ ), as well as no common point with $A B$ or with $A D$ (obviously). Then $d$ has a common point with $C D$, and its left side provides a splitting line. Contradiction. + +Case 2. The height of $c$ is greater than that of $a$. Analogously, consider the rectangle $d$ containing the angle marked on Figure 3. It has no common point with $A D$ (otherwise it has a common side with $a$ ), as well as no common point with $A B$ or with $B C$ (obviously). Then $d$ has a common point with $C D$. Hence its right side provides a splitting line, and we get the contradiction again. + +Solution 2. Again, we suppose the contrary. Consider an arbitrary counterexample. Then we know that each rectangle is attached to at least one side of the square. Observe that a rectangle cannot be attached to two opposite sides, otherwise one of its sides lies on a splitting line. + +We say that two rectangles are opposite if they are attached to opposite sides of $A B C D$. We claim that there exist two opposite rectangles having a common point. + +Consider the union $L$ of all rectangles attached to the left. Assume, to the contrary, that $L$ has no common point with the rectangles attached to the right. Take a polygonal line $p$ connecting the top and the bottom sides of the square and passing close from the right to the boundary of $L$ (see Figure 4). Then all its points belong to the rectangles attached either to the top or to the bottom. Moreover, the upper end-point of $p$ belongs to a rectangle attached to the top, and the lower one belongs to an other rectangle attached to the bottom. Hence, there is a point on $p$ where some rectangles attached to the top and to the bottom meet each other. So, there always exists a pair of neighboring opposite rectangles. + +![](https://cdn.mathpix.com/cropped/2024_04_17_b8b307b9422db9068910g-30.jpg?height=417&width=414&top_left_y=1025&top_left_x=198) + +Figure 4 + +![](https://cdn.mathpix.com/cropped/2024_04_17_b8b307b9422db9068910g-30.jpg?height=419&width=534&top_left_y=1024&top_left_x=675) + +Figure 5 + +![](https://cdn.mathpix.com/cropped/2024_04_17_b8b307b9422db9068910g-30.jpg?height=546&width=576&top_left_y=955&top_left_x=1274) + +Figure 6 + +Now, take two opposite neighboring rectangles $a$ and $b$. We can assume that $a$ is attached to the left and $b$ is attached to the right. Let $X$ be their common point. If $X$ belongs to their horizontal sides (in particular, $X$ may appear to be a common vertex of $a$ and $b$ ), then these sides provide a splitting line (see Figure 5). Otherwise, $X$ lies on the vertical sides. Let $\ell$ be the line containing these sides. + +Since $\ell$ is not a splitting line, it intersects the interior of some rectangle. Let $c$ be such a rectangle, closest to $X$; we can assume that $c$ lies above $X$. Let $Y$ be the common point of $\ell$ and the bottom side of $c$ (see Figure 6). Then $Y$ is also a vertex of two rectangles lying below $c$. + +So, let $Y$ be the upper-right and upper-left corners of the rectangles $a^{\prime}$ and $b^{\prime}$, respectively. Then $a^{\prime}$ and $b^{\prime}$ are situated not lower than $a$ and $b$, respectively (it may happen that $a=a^{\prime}$ or $b=b^{\prime}$ ). We claim that $a^{\prime}$ is attached to the left. If $a=a^{\prime}$ then of course it is. If $a \neq a^{\prime}$ then $a^{\prime}$ is above $a$, below $c$ and to the left from $b^{\prime}$. Hence, it can be attached to the left only. + +Analogously, $b^{\prime}$ is attached to the right. Now, the top sides of these two rectangles pass through $Y$, hence they provide a splitting line again. This last contradiction completes the proof. + +C3. Find all positive integers $n$, for which the numbers in the set $S=\{1,2, \ldots, n\}$ can be colored red and blue, with the following condition being satisfied: the set $S \times S \times S$ contains exactly 2007 ordered triples $(x, y, z)$ such that (i) $x, y, z$ are of the same color and (ii) $x+y+z$ is divisible by $n$. + +(Netherlands) + +Answer. $n=69$ and $n=84$. + +Solution. Suppose that the numbers $1,2, \ldots, n$ are colored red and blue. Denote by $R$ and $B$ the sets of red and blue numbers, respectively; let $|R|=r$ and $|B|=b=n-r$. Call a triple $(x, y, z) \in S \times S \times S$ monochromatic if $x, y, z$ have the same color, and bichromatic otherwise. Call a triple $(x, y, z)$ divisible if $x+y+z$ is divisible by $n$. We claim that there are exactly $r^{2}-r b+b^{2}$ divisible monochromatic triples. + +For any pair $(x, y) \in S \times S$ there exists a unique $z_{x, y} \in S$ such that the triple $\left(x, y, z_{x, y}\right)$ is divisible; so there are exactly $n^{2}$ divisible triples. Furthermore, if a divisible triple $(x, y, z)$ is bichromatic, then among $x, y, z$ there are either one blue and two red numbers, or vice versa. In both cases, exactly one of the pairs $(x, y),(y, z)$ and $(z, x)$ belongs to the set $R \times B$. Assign such pair to the triple $(x, y, z)$. + +Conversely, consider any pair $(x, y) \in R \times B$, and denote $z=z_{x, y}$. Since $x \neq y$, the triples $(x, y, z),(y, z, x)$ and $(z, x, y)$ are distinct, and $(x, y)$ is assigned to each of them. On the other hand, if $(x, y)$ is assigned to some triple, then this triple is clearly one of those mentioned above. So each pair in $R \times B$ is assigned exactly three times. + +Thus, the number of bichromatic divisible triples is three times the number of elements in $R \times B$, and the number of monochromatic ones is $n^{2}-3 r b=(r+b)^{2}-3 r b=r^{2}-r b+b^{2}$, as claimed. + +So, to find all values of $n$ for which the desired coloring is possible, we have to find all $n$, for which there exists a decomposition $n=r+b$ with $r^{2}-r b+b^{2}=2007$. Therefore, $9 \mid r^{2}-r b+b^{2}=(r+b)^{2}-3 r b$. From this it consequently follows that $3|r+b, 3| r b$, and then $3|r, 3| b$. Set $r=3 s, b=3 c$. We can assume that $s \geq c$. We have $s^{2}-s c+c^{2}=223$. + +Furthermore, + +$$ +892=4\left(s^{2}-s c+c^{2}\right)=(2 c-s)^{2}+3 s^{2} \geq 3 s^{2} \geq 3 s^{2}-3 c(s-c)=3\left(s^{2}-s c+c^{2}\right)=669 +$$ + +so $297 \geq s^{2} \geq 223$ and $17 \geq s \geq 15$. If $s=15$ then + +$$ +c(15-c)=c(s-c)=s^{2}-\left(s^{2}-s c+c^{2}\right)=15^{2}-223=2 +$$ + +which is impossible for an integer $c$. In a similar way, if $s=16$ then $c(16-c)=33$, which is also impossible. Finally, if $s=17$ then $c(17-c)=66$, and the solutions are $c=6$ and $c=11$. Hence, $(r, b)=(51,18)$ or $(r, b)=(51,33)$, and the possible values of $n$ are $n=51+18=69$ and $n=51+33=84$. + +Comment. After the formula for the number of monochromatic divisible triples is found, the solution can be finished in various ways. The one presented is aimed to decrease the number of considered cases. + +C4. Let $A_{0}=\left(a_{1}, \ldots, a_{n}\right)$ be a finite sequence of real numbers. For each $k \geq 0$, from the sequence $A_{k}=\left(x_{1}, \ldots, x_{n}\right)$ we construct a new sequence $A_{k+1}$ in the following way. + +1. We choose a partition $\{1, \ldots, n\}=I \cup J$, where $I$ and $J$ are two disjoint sets, such that the expression + +$$ +\left|\sum_{i \in I} x_{i}-\sum_{j \in J} x_{j}\right| +$$ + +attains the smallest possible value. (We allow the sets $I$ or $J$ to be empty; in this case the corresponding sum is 0 .) If there are several such partitions, one is chosen arbitrarily. + +2. We set $A_{k+1}=\left(y_{1}, \ldots, y_{n}\right)$, where $y_{i}=x_{i}+1$ if $i \in I$, and $y_{i}=x_{i}-1$ if $i \in J$. + +Prove that for some $k$, the sequence $A_{k}$ contains an element $x$ such that $|x| \geq n / 2$. + +(Iran) + +## Solution. + +Lemma. Suppose that all terms of the sequence $\left(x_{1}, \ldots, x_{n}\right)$ satisfy the inequality $\left|x_{i}\right|1)$. By the induction hypothesis there exists a splitting $\{1, \ldots, n-1\}=I^{\prime} \cup J^{\prime}$ such that + +$$ +\left|\sum_{i \in I^{\prime}} x_{i}-\sum_{j \in J^{\prime}} x_{j}\right|n-2 \cdot \frac{n}{2}=0$. + +Thus we obtain $S_{q}>S_{q-1}>\cdots>S_{p}$. This is impossible since $A_{p}=A_{q}$ and hence $S_{p}=S_{q}$. + +C5. In the Cartesian coordinate plane define the strip $S_{n}=\{(x, y) \mid n \leq xb$. Then $a_{1}>b_{1} \geq 1$, so $a_{1} \geq 3$. + +Choose integers $k$ and $\ell$ such that $k a_{1}-\ell b_{1}=1$ and therefore $k a-\ell b=d$. Since $a_{1}$ and $b_{1}$ are odd, $k+\ell$ is odd as well. Hence, for every integer $n$, strips $S_{n}$ and $S_{n+k a-\ell b}=S_{n+d}$ have opposite colors. This also implies that the coloring is periodic with period $2 d$, i.e. strips $S_{n}$ and $S_{n+2 d}$ have the same color for every $n$. + +![](https://cdn.mathpix.com/cropped/2024_04_17_b8b307b9422db9068910g-33.jpg?height=573&width=671&top_left_y=1324&top_left_x=727) + +Figure 1 + +We will construct the desired rectangle $A B C D$ with $A B=C D=a$ and $B C=A D=b$ in a position such that vertex $A$ lies on the $x$-axis, and the projection of side $A B$ onto the $x$-axis is of length $2 d$ (see Figure 1). This is possible since $a=a_{1} d>2 d$. The coordinates of the vertices will have the forms + +$$ +A=(t, 0), \quad B=\left(t+2 d, y_{1}\right), \quad C=\left(u+2 d, y_{2}\right), \quad D=\left(u, y_{3}\right) . +$$ + +Let $\varphi=\sqrt{a_{1}^{2}-4}$. By Pythagoras' theorem, + +$$ +y_{1}=B B_{0}=\sqrt{a^{2}-4 d^{2}}=d \sqrt{a_{1}^{2}-4}=d \varphi \text {. } +$$ + +So, by the similar triangles $A D D_{0}$ and $B A B_{0}$, we have the constraint + +$$ +u-t=A D_{0}=\frac{A D}{A B} \cdot B B_{0}=\frac{b d}{a} \varphi +$$ + +for numbers $t$ and $u$. Computing the numbers $y_{2}$ and $y_{3}$ is not required since they have no effect to the colors. + +Observe that the number $\varphi$ is irrational, because $\varphi^{2}$ is an integer, but $\varphi$ is not: $a_{1}>\varphi \geq$ $\sqrt{a_{1}^{2}-2 a_{1}+2}>a_{1}-1$. + +By the periodicity, points $A$ and $B$ have the same color; similarly, points $C$ and $D$ have the same color. Furthermore, these colors depend only on the values of $t$ and $u$. So it is sufficient to choose numbers $t$ and $u$ such that vertices $A$ and $D$ have the same color. + +Let $w$ be the largest positive integer such that there exist $w$ consecutive strips $S_{n_{0}}, S_{n_{0}+1}, \ldots$, $S_{n_{0}+w-1}$ with the same color, say red. (Since $S_{n_{0}+d}$ must be blue, we have $w \leq d$.) We will choose $t$ from the interval $\left(n_{0}, n_{0}+w\right)$. + +![](https://cdn.mathpix.com/cropped/2024_04_17_b8b307b9422db9068910g-34.jpg?height=285&width=751&top_left_y=834&top_left_x=641) + +Figure 2 + +Consider the interval $I=\left(n_{0}+\frac{b d}{a} \varphi, n_{0}+\frac{b d}{a} \varphi+w\right)$ on the $x$-axis (see Figure 2). Its length is $w$, and the end-points are irrational. Therefore, this interval intersects $w+1$ consecutive strips. Since at most $w$ consecutive strips may have the same color, interval $I$ must contain both red and blue points. Choose $u \in I$ such that the line $x=u$ is red and set $t=u-\frac{b d}{a} \varphi$, according to the constraint (1). Then $t \in\left(n_{0}, n_{0}+w\right)$ and $A=(t, 0)$ is red as well as $D=\left(u, y_{3}\right)$. + +Hence, variables $u$ and $t$ can be set such that they provide a rectangle with four red vertices. + +Comment. The statement is false for squares, i.e. in the case $a=b$. If strips $S_{2 k a}, S_{2 k a+1}, \ldots$, $S_{(2 k+1) a-1}$ are red, and strips $S_{(2 k+1) a}, S_{(2 k+1) a+1}, \ldots, S_{(2 k+2) a-1}$ are blue for every integer $k$, then each square of size $a \times a$ has at least one red and at least one blue vertex as well. + +C6. In a mathematical competition some competitors are friends; friendship is always mutual. Call a group of competitors a clique if each two of them are friends. The number of members in a clique is called its size. + +It is known that the largest size of cliques is even. Prove that the competitors can be arranged in two rooms such that the largest size of cliques in one room is the same as the largest size of cliques in the other room. + +(Russia) + +Solution. We present an algorithm to arrange the competitors. Let the two rooms be Room $A$ and Room B. We start with an initial arrangement, and then we modify it several times by sending one person to the other room. At any state of the algorithm, $A$ and $B$ denote the sets of the competitors in the rooms, and $c(A)$ and $c(B)$ denote the largest sizes of cliques in the rooms, respectively. + +Step 1. Let $M$ be one of the cliques of largest size, $|M|=2 m$. Send all members of $M$ to Room $A$ and all other competitors to Room B. + +Since $M$ is a clique of the largest size, we have $c(A)=|M| \geq c(B)$. + +Step 2. While $c(A)>c(B)$, send one person from Room $A$ to Room $B$. + +![](https://cdn.mathpix.com/cropped/2024_04_17_b8b307b9422db9068910g-35.jpg?height=408&width=1028&top_left_y=1138&top_left_x=545) + +Note that $c(A)>c(B)$ implies that Room $A$ is not empty. + +In each step, $c(A)$ decreases by one and $c(B)$ increases by at most one. So at the end we have $c(A) \leq c(B) \leq c(A)+1$. + +We also have $c(A)=|A| \geq m$ at the end. Otherwise we would have at least $m+1$ members of $M$ in Room $B$ and at most $m-1$ in Room $A$, implying $c(B)-c(A) \geq(m+1)-(m-1)=2$. + +Step 3. Let $k=c(A)$. If $c(B)=k$ then $S T O P$. + +If we reached $c(A)=c(B)=k$ then we have found the desired arrangement. + +In all other cases we have $c(B)=k+1$. + +From the estimate above we also know that $k=|A|=|A \cap M| \geq m$ and $|B \cap M| \leq m$. + +Step 4. If there exists a competitor $x \in B \cap M$ and a clique $C \subset B$ such that $|C|=k+1$ and $x \notin C$, then move $x$ to Room $A$ and $S T O P$. + +![](https://cdn.mathpix.com/cropped/2024_04_17_b8b307b9422db9068910g-35.jpg?height=408&width=1002&top_left_y=2143&top_left_x=561) + +After moving $x$ back to Room $A$, we will have $k+1$ members of $M$ in Room $A$, thus $c(A)=k+1$. Due to $x \notin C, c(B)=|C|$ is not decreased, and after this step we have $c(A)=c(B)=k+1$. + +If there is no such competitor $x$, then in Room $B$, all cliques of size $k+1$ contain $B \cap M$ as a subset. + +Step 5. While $c(B)=k+1$, choose a clique $C \subset B$ such that $|C|=k+1$ and move one member of $C \backslash M$ to Room $A$. + +![](https://cdn.mathpix.com/cropped/2024_04_17_b8b307b9422db9068910g-36.jpg?height=411&width=1017&top_left_y=497&top_left_x=494) + +Note that $|C|=k+1>m \geq|B \cap M|$, so $C \backslash M$ cannot be empty. + +Every time we move a single person from Room $B$ to Room $A$, so $c(B)$ decreases by at most 1. Hence, at the end of this loop we have $c(B)=k$. + +In Room $A$ we have the clique $A \cap M$ with size $|A \cap M|=k$ thus $c(A) \geq k$. We prove that there is no clique of larger size there. Let $Q \subset A$ be an arbitrary clique. We show that $|Q| \leq k$. + +![](https://cdn.mathpix.com/cropped/2024_04_17_b8b307b9422db9068910g-36.jpg?height=411&width=1017&top_left_y=1185&top_left_x=494) + +In Room $A$, and specially in set $Q$, there can be two types of competitors: + +- Some members of $M$. Since $M$ is a clique, they are friends with all members of $B \cap M$. +- Competitors which were moved to Room $A$ in Step 5. Each of them has been in a clique with $B \cap M$ so they are also friends with all members of $B \cap M$. + +Hence, all members of $Q$ are friends with all members of $B \cap M$. Sets $Q$ and $B \cap M$ are cliques themselves, so $Q \cup(B \cap M)$ is also a clique. Since $M$ is a clique of the largest size, + +$$ +|M| \geq|Q \cup(B \cap M)|=|Q|+|B \cap M|=|Q|+|M|-|A \cap M| +$$ + +therefore + +$$ +|Q| \leq|A \cap M|=k +$$ + +Finally, after Step 5 we have $c(A)=c(B)=k$. + +Comment. Obviously, the statement is false without the assumption that the largest clique size is even. + +C7. Let $\alpha<\frac{3-\sqrt{5}}{2}$ be a positive real number. Prove that there exist positive integers $n$ and $p>\alpha \cdot 2^{n}$ for which one can select $2 p$ pairwise distinct subsets $S_{1}, \ldots, S_{p}, T_{1}, \ldots, T_{p}$ of the set $\{1,2, \ldots, n\}$ such that $S_{i} \cap T_{j} \neq \varnothing$ for all $1 \leq i, j \leq p$. + +(Austria) + +Solution. Let $k$ and $m$ be positive integers (to be determined later) and set $n=k m$. Decompose the set $\{1,2, \ldots, n\}$ into $k$ disjoint subsets, each of size $m$; denote these subsets by $A_{1}, \ldots, A_{k}$. Define the following families of sets: + +$$ +\begin{aligned} +\mathcal{S} & =\left\{S \subset\{1,2, \ldots, n\}: \forall i S \cap A_{i} \neq \varnothing\right\} \\ +\mathcal{T}_{1} & =\left\{T \subset\{1,2, \ldots, n\}: \quad \exists i A_{i} \subset T\right\}, \quad \mathcal{T}=\mathcal{T}_{1} \backslash \mathcal{S} . +\end{aligned} +$$ + +For each set $T \in \mathcal{T} \subset \mathcal{T}_{1}$, there exists an index $1 \leq i \leq k$ such that $A_{i} \subset T$. Then for all $S \in \mathcal{S}$, $S \cap T \supset S \cap A_{i} \neq \varnothing$. Hence, each $S \in \mathcal{S}$ and each $T \in \mathcal{T}$ have at least one common element. + +Below we show that the numbers $m$ and $k$ can be chosen such that $|\mathcal{S}|,|\mathcal{T}|>\alpha \cdot 2^{n}$. Then, choosing $p=\min \{|\mathcal{S}|,|\mathcal{T}|\}$, one can select the desired $2 p$ sets $S_{1}, \ldots, S_{p}$ and $T_{1}, \ldots, T_{p}$ from families $\mathcal{S}$ and $\mathcal{T}$, respectively. Since families $\mathcal{S}$ and $\mathcal{T}$ are disjoint, sets $S_{i}$ and $T_{j}$ will be pairwise distinct. + +To count the sets $S \in \mathcal{S}$, observe that each $A_{i}$ has $2^{m}-1$ nonempty subsets so we have $2^{m}-1$ choices for $S \cap A_{i}$. These intersections uniquely determine set $S$, so + +$$ +|\mathcal{S}|=\left(2^{m}-1\right)^{k} +$$ + +Similarly, if a set $H \subset\{1,2, \ldots, n\}$ does not contain a certain set $A_{i}$ then we have $2^{m}-1$ choices for $H \cap A_{i}$ : all subsets of $A_{i}$, except $A_{i}$ itself. Therefore, the complement of $\mathcal{T}_{1}$ contains $\left(2^{m}-1\right)^{k}$ sets and + +$$ +\left|\mathcal{T}_{1}\right|=2^{k m}-\left(2^{m}-1\right)^{k} . +$$ + +Next consider the family $\mathcal{S} \backslash \mathcal{T}_{1}$. If a set $S$ intersects all $A_{i}$ but does not contain any of them, then there exists $2^{m}-2$ possible values for each $S \cap A_{i}$ : all subsets of $A_{i}$ except $\varnothing$ and $A_{i}$. Therefore the number of such sets $S$ is $\left(2^{m}-2\right)^{k}$, so + +$$ +\left|\mathcal{S} \backslash \mathcal{T}_{1}\right|=\left(2^{m}-2\right)^{k} +$$ + +From (1), (2), and (3) we obtain + +$$ +|\mathcal{T}|=\left|\mathcal{T}_{1}\right|-\left|\mathcal{S} \cap \mathcal{T}_{1}\right|=\left|\mathcal{T}_{1}\right|-\left(|\mathcal{S}|-\left|\mathcal{S} \backslash \mathcal{T}_{1}\right|\right)=2^{k m}-2\left(2^{m}-1\right)^{k}+\left(2^{m}-2\right)^{k} +$$ + +Let $\delta=\frac{3-\sqrt{5}}{2}$ and $k=k(m)=\left[2^{m} \log \frac{1}{\delta}\right]$. Then + +$$ +\lim _{m \rightarrow \infty} \frac{|\mathcal{S}|}{2^{k m}}=\lim _{m \rightarrow \infty}\left(1-\frac{1}{2^{m}}\right)^{k}=\exp \left(-\lim _{m \rightarrow \infty} \frac{k}{2^{m}}\right)=\delta +$$ + +and similarly + +$$ +\lim _{m \rightarrow \infty} \frac{|\mathcal{T}|}{2^{k m}}=1-2 \lim _{m \rightarrow \infty}\left(1-\frac{1}{2^{m}}\right)^{k}+\lim _{m \rightarrow \infty}\left(1-\frac{2}{2^{m}}\right)^{k}=1-2 \delta+\delta^{2}=\delta +$$ + +Hence, if $m$ is sufficiently large then $\frac{|\mathcal{S}|}{2^{m k}}$ and $\frac{|\mathcal{T}|}{2^{m k}}$ are greater than $\alpha$ (since $\alpha<\delta$ ). So $|\mathcal{S}|,|\mathcal{T}|>\alpha \cdot 2^{m k}=\alpha \cdot 2^{n}$. + +Comment. It can be proved that the constant $\frac{3-\sqrt{5}}{2}$ is sharp. Actually, if $S_{1}, \ldots, S_{p}, T_{1}, \ldots, T_{p}$ are distinct subsets of $\{1,2, \ldots, n\}$ such that each $S_{i}$ intersects each $T_{j}$, then $p<\frac{3-\sqrt{5}}{2} \cdot 2^{n}$. + +C8. Given a convex $n$-gon $P$ in the plane. For every three vertices of $P$, consider the triangle determined by them. Call such a triangle good if all its sides are of unit length. + +Prove that there are not more than $\frac{2}{3} n$ good triangles. + +(Ukraine) + +Solution. Consider all good triangles containing a certain vertex $A$. The other two vertices of any such triangle lie on the circle $\omega_{A}$ with unit radius and center $A$. Since $P$ is convex, all these vertices lie on an arc of angle less than $180^{\circ}$. Let $L_{A} R_{A}$ be the shortest such arc, oriented clockwise (see Figure 1). Each of segments $A L_{A}$ and $A R_{A}$ belongs to a unique good triangle. We say that the good triangle with side $A L_{A}$ is assigned counterclockwise to $A$, and the second one, with side $A R_{A}$, is assigned clockwise to $A$. In those cases when there is a single good triangle containing vertex $A$, this triangle is assigned to $A$ twice. + +There are at most two assignments to each vertex of the polygon. (Vertices which do not belong to any good triangle have no assignment.) So the number of assignments is at most $2 n$. + +Consider an arbitrary good triangle $A B C$, with vertices arranged clockwise. We prove that $A B C$ is assigned to its vertices at least three times. Then, denoting the number of good triangles by $t$, we obtain that the number $K$ of all assignments is at most $2 n$, while it is not less than $3 t$. Then $3 t \leq K \leq 2 n$, as required. + +Actually, we prove that triangle $A B C$ is assigned either counterclockwise to $C$ or clockwise to $B$. Then, by the cyclic symmetry of the vertices, we obtain that triangle $A B C$ is assigned either counterclockwise to $A$ or clockwise to $C$, and either counterclockwise to $B$ or clockwise to $A$, providing the claim. +![](https://cdn.mathpix.com/cropped/2024_04_17_b8b307b9422db9068910g-38.jpg?height=586&width=678&top_left_y=1369&top_left_x=290) + +Figure 1 + +![](https://cdn.mathpix.com/cropped/2024_04_17_b8b307b9422db9068910g-38.jpg?height=549&width=617&top_left_y=1393&top_left_x=1119) + +Figure 2 + +Assume, to the contrary, that $L_{C} \neq A$ and $R_{B} \neq A$. Denote by $A^{\prime}, B^{\prime}, C^{\prime}$ the intersection points of circles $\omega_{A}, \omega_{B}$ and $\omega_{C}$, distinct from $A, B, C$ (see Figure 2). Let $C L_{C} L_{C}^{\prime}$ be the good triangle containing $C L_{C}$. Observe that the angle of $\operatorname{arc} L_{C} A$ is less than $120^{\circ}$. Then one of the points $L_{C}$ and $L_{C}^{\prime}$ belongs to $\operatorname{arc} B^{\prime} A$ of $\omega_{C}$; let this point be $X$. In the case when $L_{C}=B^{\prime}$ and $L_{C}^{\prime}=A$, choose $X=B^{\prime}$. + +Analogously, considering the good triangle $B R_{B}^{\prime} R_{B}$ which contains $B R_{B}$ as an edge, we see that one of the points $R_{B}$ and $R_{B}^{\prime}$ lies on arc $A C^{\prime}$ of $\omega_{B}$. Denote this point by $Y, Y \neq A$. Then angles $X A Y, Y A B, B A C$ and $C A X$ (oriented clockwise) are not greater than $180^{\circ}$. Hence, point $A$ lies in quadrilateral $X Y B C$ (either in its interior or on segment $X Y$ ). This is impossible, since all these five points are vertices of $P$. + +Hence, each good triangle has at least three assignments, and the statement is proved. + +Comment 1. Considering a diameter $A B$ of the polygon, one can prove that every good triangle containing either $A$ or $B$ has at least four assignments. This observation leads to $t \leq\left\lfloor\frac{2}{3}(n-1)\right\rfloor$. + +Comment 2. The result $t \leq\left\lfloor\frac{2}{3}(n-1)\right\rfloor$ is sharp. To construct a polygon with $n=3 k+1$ vertices and $t=2 k$ triangles, take a rhombus $A B_{1} C_{1} D_{1}$ with unit side length and $\angle B_{1}=60^{\circ}$. Then rotate it around $A$ by small angles obtaining rhombi $A B_{2} C_{2} D_{2}, \ldots, A B_{k} C_{k} D_{k}$ (see Figure 3). The polygon $A B_{1} \ldots B_{k} C_{1} \ldots C_{k} D_{1} \ldots D_{k}$ has $3 k+1$ vertices and contains $2 k$ good triangles. + +The construction for $n=3 k$ and $n=3 k-1$ can be obtained by deleting vertices $D_{n}$ and $D_{n-1}$. + +![](https://cdn.mathpix.com/cropped/2024_04_17_b8b307b9422db9068910g-39.jpg?height=331&width=602&top_left_y=311&top_left_x=1298) + +Figure 3 + +## Geometry + +G1. In triangle $A B C$, the angle bisector at vertex $C$ intersects the circumcircle and the perpendicular bisectors of sides $B C$ and $C A$ at points $R, P$, and $Q$, respectively. The midpoints of $B C$ and $C A$ are $S$ and $T$, respectively. Prove that triangles $R Q T$ and $R P S$ have the same area. + +(Czech Republic) + +Solution 1. If $A C=B C$ then triangle $A B C$ is isosceles, triangles $R Q T$ and $R P S$ are symmetric about the bisector $C R$ and the statement is trivial. If $A C \neq B C$ then it can be assumed without loss of generality that $A C\sqrt{E C^{2}-L C^{2}}=L E . +$$ + +Since quadrilateral $B C E D$ is cyclic, we have $\angle E D C=\angle E B C$, so the right triangles $B E L$ and $D E K$ are similar. Then $K E>L E$ implies $D K>B L$, and hence + +$$ +D F=D K-K F>B L-L C=B C=A D \text {. } +$$ + +But triangles $A D F$ and $G C F$ are similar, so we have $1>\frac{A D}{D F}=\frac{G C}{C F}$; this contradicts our assumption. + +The case $C F>G C$ is completely similar. We consequently obtain the converse inequalities $K F>L C, K E1$; also, lines $A_{1} C^{\prime}$ and $C A$ intersect at a point $Z$ on the extension of $C A$ beyond point $A$, hence $\frac{\left[A_{1} B_{1} C^{\prime}\right]}{\left[A_{1} B^{\prime} C^{\prime}\right]}=\frac{B_{1} Z}{B^{\prime} Z}>1$. Finally, since $A_{1} A^{\prime} \| B^{\prime} C^{\prime}$, we have $\left[A_{1} B_{1} C_{1}\right]>\left[A_{1} B_{1} C^{\prime}\right]>\left[A_{1} B^{\prime} C^{\prime}\right]=\left[A^{\prime} B^{\prime} C^{\prime}\right]=\frac{1}{4}[A B C]$. + +Now, from $\left[A_{1} B_{1} C_{1}\right]+\left[A C_{1} B_{1}\right]+\left[B A_{1} C_{1}\right]+\left[C B_{1} A_{1}\right]=[A B C]$ we obtain that one of the remaining triangles $A C_{1} B_{1}, B A_{1} C_{1}, C B_{1} A_{1}$ has an area less than $\frac{1}{4}[A B C]$, so it is less than $\left[A_{1} B_{1} C_{1}\right]$. + +Now we return to the problem. We say that triangle $A_{1} B_{1} C_{1}$ is small if $\left[A_{1} B_{1} C_{1}\right]$ is less than each of $\left[B B_{1} A_{1}\right]$ and $\left[C C_{1} B_{1}\right]$; otherwise this triangle is big (the similar notion is introduced for triangles $B_{1} C_{1} D_{1}, C_{1} D_{1} A_{1}, D_{1} A_{1} B_{1}$ ). If both triangles $A_{1} B_{1} C_{1}$ and $C_{1} D_{1} A_{1}$ are big, then $\left[A_{1} B_{1} C_{1}\right]$ is not less than the area of some border triangle, and $\left[C_{1} D_{1} A_{1}\right]$ is not less than the area of another one; hence, $S_{1}=\left[A_{1} B_{1} C_{1}\right]+\left[C_{1} D_{1} A_{1}\right] \geq S$. The same is valid for the pair of $B_{1} C_{1} D_{1}$ and $D_{1} A_{1} B_{1}$. So it is sufficient to prove that in one of these pairs both triangles are big. + +Suppose the contrary. Then there is a small triangle in each pair. Without loss of generality, assume that triangles $A_{1} B_{1} C_{1}$ and $D_{1} A_{1} B_{1}$ are small. We can assume also that $\left[A_{1} B_{1} C_{1}\right] \leq$ $\left[D_{1} A_{1} B_{1}\right]$. Note that in this case ray $D_{1} C_{1}$ intersects line $B C$. + +Consider two cases. + +![](https://cdn.mathpix.com/cropped/2024_04_17_b8b307b9422db9068910g-48.jpg?height=448&width=648&top_left_y=987&top_left_x=224) + +Figure 4 + +![](https://cdn.mathpix.com/cropped/2024_04_17_b8b307b9422db9068910g-48.jpg?height=420&width=886&top_left_y=1001&top_left_x=899) + +Figure 5 + +Case 1. Ray $C_{1} D_{1}$ intersects line $A B$ at some point $K$. Let ray $D_{1} C_{1}$ intersect line $B C$ at point $L$ (see Figure 4). Then we have $\left[A_{1} B_{1} C_{1}\right]<\left[C C_{1} B_{1}\right]<\left[L C_{1} B_{1}\right],\left[A_{1} B_{1} C_{1}\right]<\left[B B_{1} A_{1}\right]$ (both - since $\left[A_{1} B_{1} C_{1}\right]$ is small), and $\left[A_{1} B_{1} C_{1}\right] \leq\left[D_{1} A_{1} B_{1}\right]<\left[A A_{1} D_{1}\right]<\left[K A_{1} D_{1}\right]<\left[K A_{1} C_{1}\right]$ (since triangle $D_{1} A_{1} B_{1}$ is small). This contradicts the Lemma, applied for triangle $A_{1} B_{1} C_{1}$ inside $L K B$. + +Case 2. Ray $C_{1} D_{1}$ does not intersect $A B$. Then choose a "sufficiently far" point $K$ on ray $B A$ such that $\left[K A_{1} C_{1}\right]>\left[A_{1} B_{1} C_{1}\right]$, and that ray $K C_{1}$ intersects line $B C$ at some point $L$ (see Figure 5). Since ray $C_{1} D_{1}$ does not intersect line $A B$, the points $A$ and $D_{1}$ are on different sides of $K L$; then $A$ and $D$ are also on different sides, and $C$ is on the same side as $A$ and $B$. Then analogously we have $\left[A_{1} B_{1} C_{1}\right]<\left[C C_{1} B_{1}\right]<\left[L C_{1} B_{1}\right]$ and $\left[A_{1} B_{1} C_{1}\right]<\left[B B_{1} A_{1}\right]$ since triangle $A_{1} B_{1} C_{1}$ is small. This (together with $\left[A_{1} B_{1} C_{1}\right]<\left[K A_{1} C_{1}\right]$ ) contradicts the Lemma again. + +G7. Given an acute triangle $A B C$ with angles $\alpha, \beta$ and $\gamma$ at vertices $A, B$ and $C$, respectively, such that $\beta>\gamma$. Point $I$ is the incenter, and $R$ is the circumradius. Point $D$ is the foot of the altitude from vertex $A$. Point $K$ lies on line $A D$ such that $A K=2 R$, and $D$ separates $A$ and $K$. Finally, lines $D I$ and $K I$ meet sides $A C$ and $B C$ at $E$ and $F$, respectively. + +Prove that if $I E=I F$ then $\beta \leq 3 \gamma$. + +(Iran) + +Solution 1. We first prove that + +$$ +\angle K I D=\frac{\beta-\gamma}{2} +$$ + +even without the assumption that $I E=I F$. Then we will show that the statement of the problem is a consequence of this fact. + +Denote the circumcenter by $O$. On the circumcircle, let $P$ be the point opposite to $A$, and let the angle bisector $A I$ intersect the circle again at $M$. Since $A K=A P=2 R$, triangle $A K P$ is isosceles. It is known that $\angle B A D=\angle C A O$, hence $\angle D A I=\angle B A I-\angle B A D=\angle C A I-$ $\angle C A O=\angle O A I$, and $A M$ is the bisector line in triangle $A K P$. Therefore, points $K$ and $P$ are symmetrical about $A M$, and $\angle A M K=\angle A M P=90^{\circ}$. Thus, $M$ is the midpoint of $K P$, and $A M$ is the perpendicular bisector of $K P$. + +![](https://cdn.mathpix.com/cropped/2024_04_17_b8b307b9422db9068910g-50.jpg?height=1082&width=962&top_left_y=1252&top_left_x=524) + +Denote the perpendicular feet of incenter $I$ on lines $B C, A C$, and $A D$ by $A_{1}, B_{1}$, and $T$, respectively. Quadrilateral $D A_{1} I T$ is a rectangle, hence $T D=I A_{1}=I B_{1}$. + +Due to the right angles at $T$ and $B_{1}$, quadrilateral $A B_{1} I T$ is cyclic. Hence $\angle B_{1} T I=$ $\angle B_{1} A I=\angle C A M=\angle B A M=\angle B P M$ and $\angle I B_{1} T=\angle I A T=\angle M A K=\angle M A P=$ $\angle M B P$. Therefore, triangles $B_{1} T I$ and $B P M$ are similar and $\frac{I T}{I B_{1}}=\frac{M P}{M B}$. + +It is well-known that $M B=M C=M I$. Then right triangles $I T D$ and $K M I$ are also +similar, because $\frac{I T}{T D}=\frac{I T}{I B_{1}}=\frac{M P}{M B}=\frac{K M}{M I}$. Hence, $\angle K I M=\angle I D T=\angle I D A$, and + +$$ +\angle K I D=\angle M I D-\angle K I M=(\angle I A D+\angle I D A)-\angle I D A=\angle I A D . +$$ + +Finally, from the right triangle $A D B$ we can compute + +$$ +\angle K I D=\angle I A D=\angle I A B-\angle D A B=\frac{\alpha}{2}-\left(90^{\circ}-\beta\right)=\frac{\alpha}{2}-\frac{\alpha+\beta+\gamma}{2}+\beta=\frac{\beta-\gamma}{2} . +$$ + +Now let us turn to the statement and suppose that $I E=I F$. Since $I A_{1}=I B_{1}$, the right triangles $I E B_{1}$ and $I F A_{1}$ are congruent and $\angle I E B_{1}=\angle I F A_{1}$. Since $\beta>\gamma, A_{1}$ lies in the interior of segment $C D$ and $F$ lies in the interior of $A_{1} D$. Hence, $\angle I F C$ is acute. Then two cases are possible depending on the order of points $A, C, B_{1}$ and $E$. +![](https://cdn.mathpix.com/cropped/2024_04_17_b8b307b9422db9068910g-51.jpg?height=818&width=1486&top_left_y=907&top_left_x=317) + +If point $E$ lies between $C$ and $B_{1}$ then $\angle I F C=\angle I E A$, hence quadrilateral $C E I F$ is cyclic and $\angle F C E=180^{\circ}-\angle E I F=\angle K I D$. By (1), in this case we obtain $\angle F C E=\gamma=\angle K I D=$ $\frac{\beta-\gamma}{2}$ and $\beta=3 \gamma$. + +Otherwise, if point $E$ lies between $A$ and $B_{1}$, quadrilateral $C E I F$ is a deltoid such that $\angle I E C=\angle I F C<90^{\circ}$. Then we have $\angle F C E>180^{\circ}-\angle E I F=\angle K I D$. Therefore, $\angle F C E=\gamma>\angle K I D=\frac{\beta-\gamma}{2}$ and $\beta<3 \gamma$. + +Comment 1. In the case when quadrilateral $C E I F$ is a deltoid, one can prove the desired inequality without using (1). Actually, from $\angle I E C=\angle I F C<90^{\circ}$ it follows that $\angle A D I=90^{\circ}-\angle E D C<$ $\angle A E D-\angle E D C=\gamma$. Since the incircle lies inside triangle $A B C$, we have $A D>2 r$ (here $r$ is the inradius), which implies $D T8 a^{4}+2 b^{2}$ and inequality (1) cannot hold. Hence $a \leq 3$, and three cases are possible. + +Case 1: $a=1$. Then $k=2$ and $8+2 b^{2} \geq 7+3^{b}$, thus $2 b^{2}+1 \geq 3^{b}$. This is possible only if $b \leq 2$. If $b=1$ then $n=2$ and $\frac{k^{4}+n^{2}}{7^{k}-3^{n}}=\frac{2^{4}+2^{2}}{7^{2}-3^{2}}=\frac{1}{2}$, which is not an integer. If $b=2$ then $n=4$ and $\frac{k^{4}+n^{2}}{7^{k}-3^{n}}=\frac{2^{4}+4^{2}}{7^{2}-3^{4}}=-1$, so $(k, n)=(2,4)$ is a solution. + +Case 2: $a=2$. Then $k=4$ and $k^{4}+n^{2}=256+4 b^{2} \geq\left|7^{4}-3^{n}\right|=\left|49-3^{b}\right| \cdot\left(49+3^{b}\right)$. The smallest value of the first factor is 22 , attained at $b=3$, so $128+2 b^{2} \geq 11\left(49+3^{b}\right)$, which is impossible since $3^{b}>2 b^{2}$. + +Case 3: $a=3$. Then $k=6$ and $k^{4}+n^{2}=1296+4 b^{2} \geq\left|7^{6}-3^{n}\right|=\left|343-3^{b}\right| \cdot\left(343+3^{b}\right)$. Analogously, $\left|343-3^{b}\right| \geq 100$ and we have $324+b^{2} \geq 25\left(343+3^{b}\right)$, which is impossible again. + +We find that there exists a unique solution $(k, n)=(2,4)$. + +N2. Let $b, n>1$ be integers. Suppose that for each $k>1$ there exists an integer $a_{k}$ such that $b-a_{k}^{n}$ is divisible by $k$. Prove that $b=A^{n}$ for some integer $A$. + +(Canada) + +Solution. Let the prime factorization of $b$ be $b=p_{1}^{\alpha_{1}} \ldots p_{s}^{\alpha_{s}}$, where $p_{1}, \ldots, p_{s}$ are distinct primes. Our goal is to show that all exponents $\alpha_{i}$ are divisible by $n$, then we can set $A=p_{1}^{\alpha_{1} / n} \ldots p_{s}^{\alpha_{s} / n}$. + +Apply the condition for $k=b^{2}$. The number $b-a_{k}^{n}$ is divisible by $b^{2}$ and hence, for each $1 \leq i \leq s$, it is divisible by $p_{i}^{2 \alpha_{i}}>p_{i}^{\alpha_{i}}$ as well. Therefore + +$$ +a_{k}^{n} \equiv b \equiv 0 \quad\left(\bmod p_{i}^{\alpha_{i}}\right) +$$ + +and + +$$ +a_{k}^{n} \equiv b \not \equiv 0 \quad\left(\bmod p_{i}^{\alpha_{i}+1}\right) +$$ + +which implies that the largest power of $p_{i}$ dividing $a_{k}^{n}$ is $p_{i}^{\alpha_{i}}$. Since $a_{k}^{n}$ is a complete $n$th power, this implies that $\alpha_{i}$ is divisible by $n$. + +Comment. If $n=8$ and $b=16$, then for each prime $p$ there exists an integer $a_{p}$ such that $b-a_{p}^{n}$ is divisible by $p$. Actually, the congruency $x^{8}-16 \equiv 0(\bmod p)$ expands as + +$$ +\left(x^{2}-2\right)\left(x^{2}+2\right)\left(x^{2}-2 x+2\right)\left(x^{2}+2 x+2\right) \equiv 0 \quad(\bmod p) +$$ + +Hence, if -1 is a quadratic residue modulo $p$, then congruency $x^{2}+2 x+2=(x+1)^{2}+1 \equiv 0$ has a solution. Otherwise, one of congruencies $x^{2} \equiv 2$ and $x^{2} \equiv-2$ has a solution. + +Thus, the solution cannot work using only prime values of $k$. + +N3. Let $X$ be a set of 10000 integers, none of them is divisible by 47 . Prove that there exists a 2007-element subset $Y$ of $X$ such that $a-b+c-d+e$ is not divisible by 47 for any $a, b, c, d, e \in Y$. + +(Netherlands) + +Solution. Call a set $M$ of integers good if $47 \nmid a-b+c-d+e$ for any $a, b, c, d, e \in M$. + +Consider the set $J=\{-9,-7,-5,-3,-1,1,3,5,7,9\}$. We claim that $J$ is good. Actually, for any $a, b, c, d, e \in J$ the number $a-b+c-d+e$ is odd and + +$$ +-45=(-9)-9+(-9)-9+(-9) \leq a-b+c-d+e \leq 9-(-9)+9-(-9)+9=45 +$$ + +But there is no odd number divisible by 47 between -45 and 45 . + +For any $k=1, \ldots, 46$ consider the set + +$$ +A_{k}=\{x \in X \mid \exists j \in J: \quad k x \equiv j(\bmod 47)\} . +$$ + +If $A_{k}$ is not good, then $47 \mid a-b+c-d+e$ for some $a, b, c, d, e \in A_{k}$, hence $47 \mid k a-k b+$ $k c-k d+k e$. But set $J$ contains numbers with the same residues modulo 47 , so $J$ also is not good. This is a contradiction; therefore each $A_{k}$ is a good subset of $X$. + +Then it suffices to prove that there exists a number $k$ such that $\left|A_{k}\right| \geq 2007$. Note that each $x \in X$ is contained in exactly 10 sets $A_{k}$. Then + +$$ +\sum_{k=1}^{46}\left|A_{k}\right|=10|X|=100000 +$$ + +hence for some value of $k$ we have + +$$ +\left|A_{k}\right| \geq \frac{100000}{46}>2173>2007 . +$$ + +This completes the proof. + +Comment. For the solution, it is essential to find a good set consisting of 10 different residues. Actually, consider a set $X$ containing almost uniform distribution of the nonzero residues (i. e. each residue occurs 217 or 218 times). Let $Y \subset X$ be a good subset containing 2007 elements. Then the set $K$ of all residues appearing in $Y$ contains not less than 10 residues, and obviously this set is good. + +On the other hand, there is no good set $K$ consisting of 11 different residues. The CauchyDavenport theorem claims that for any sets $A, B$ of residues modulo a prime $p$, + +$$ +|A+B| \geq \min \{p,|A|+|B|-1\} . +$$ + +Hence, if $|K| \geq 11$, then $|K+K| \geq 21,|K+K+K| \geq 31>47-|K+K|$, hence $\mid K+K+K+$ $(-K)+(-K) \mid=47$, and $0 \equiv a+c+e-b-d(\bmod 47)$ for some $a, b, c, d, e \in K$. + +From the same reasoning, one can see that a good set $K$ containing 10 residues should satisfy equalities $|K+K|=19=2|K|-1$ and $|K+K+K|=28=|K+K|+|K|-1$. It can be proved that in this case set $K$ consists of 10 residues forming an arithmetic progression. As an easy consequence, one obtains that set $K$ has the form $a J$ for some nonzero residue $a$. + +N4. For every integer $k \geq 2$, prove that $2^{3 k}$ divides the number + +$$ +\left(\begin{array}{c} +2^{k+1} \\ +2^{k} +\end{array}\right)-\left(\begin{array}{c} +2^{k} \\ +2^{k-1} +\end{array}\right) +$$ + +but $2^{3 k+1}$ does not. + +(Poland) + +Solution. We use the notation $(2 n-1) ! !=1 \cdot 3 \cdots(2 n-1)$ and $(2 n) ! !=2 \cdot 4 \cdots(2 n)=2^{n} n$ ! for any positive integer $n$. Observe that $(2 n) !=(2 n) ! !(2 n-1) ! !=2^{n} n !(2 n-1) !$ !. + +For any positive integer $n$ we have + +$$ +\begin{aligned} +& \left(\begin{array}{l} +4 n \\ +2 n +\end{array}\right)=\frac{(4 n) !}{(2 n) !^{2}}=\frac{2^{2 n}(2 n) !(4 n-1) ! !}{(2 n) !^{2}}=\frac{2^{2 n}}{(2 n) !}(4 n-1) ! ! \\ +& \left(\begin{array}{c} +2 n \\ +n +\end{array}\right)=\frac{1}{(2 n) !}\left(\frac{(2 n) !}{n !}\right)^{2}=\frac{1}{(2 n) !}\left(2^{n}(2 n-1) ! !\right)^{2}=\frac{2^{2 n}}{(2 n) !}(2 n-1) ! !^{2} . +\end{aligned} +$$ + +Then expression (1) can be rewritten as follows: + +$$ +\begin{aligned} +\left(\begin{array}{c} +2^{k+1} \\ +2^{k} +\end{array}\right) & -\left(\begin{array}{c} +2^{k} \\ +2^{k-1} +\end{array}\right)=\frac{2^{2^{k}}}{\left(2^{k}\right) !}\left(2^{k+1}-1\right) ! !-\frac{2^{2^{k}}}{\left(2^{k}\right) !}\left(2^{k}-1\right) ! !^{2} \\ +& =\frac{2^{2^{k}}\left(2^{k}-1\right) ! !}{\left(2^{k}\right) !} \cdot\left(\left(2^{k}+1\right)\left(2^{k}+3\right) \ldots\left(2^{k}+2^{k}-1\right)-\left(2^{k}-1\right)\left(2^{k}-3\right) \ldots\left(2^{k}-2^{k}+1\right)\right) . +\end{aligned} +$$ + +We compute the exponent of 2 in the prime decomposition of each factor (the first one is a rational number but not necessarily an integer; it is not important). + +First, we show by induction on $n$ that the exponent of 2 in $\left(2^{n}\right)$ ! is $2^{n}-1$. The base case $n=1$ is trivial. Suppose that $\left(2^{n}\right) !=2^{2^{n}-1}(2 d+1)$ for some integer $d$. Then we have + +$$ +\left(2^{n+1}\right) !=2^{2^{n}}\left(2^{n}\right) !\left(2^{n+1}-1\right) ! !=2^{2^{n}} 2^{2^{n}-1} \cdot(2 d+1)\left(2^{n+1}-1\right) ! !=2^{2^{n+1}-1} \cdot(2 q+1) +$$ + +for some integer $q$. This finishes the induction step. + +Hence, the exponent of 2 in the first factor in $(2)$ is $2^{k}-\left(2^{k}-1\right)=1$. + +The second factor in (2) can be considered as the value of the polynomial + +$$ +P(x)=(x+1)(x+3) \ldots\left(x+2^{k}-1\right)-(x-1)(x-3) \ldots\left(x-2^{k}+1\right) . +$$ + +at $x=2^{k}$. Now we collect some information about $P(x)$. + +Observe that $P(-x)=-P(x)$, since $k \geq 2$. So $P(x)$ is an odd function, and it has nonzero coefficients only at odd powers of $x$. Hence $P(x)=x^{3} Q(x)+c x$, where $Q(x)$ is a polynomial with integer coefficients. + +Compute the exponent of 2 in $c$. We have + +$$ +\begin{aligned} +c & =2\left(2^{k}-1\right) ! ! \sum_{i=1}^{2^{k-1}} \frac{1}{2 i-1}=\left(2^{k}-1\right) ! ! \sum_{i=1}^{2^{k-1}}\left(\frac{1}{2 i-1}+\frac{1}{2^{k}-2 i+1}\right) \\ +& =\left(2^{k}-1\right) ! ! \sum_{i=1}^{2^{k-1}} \frac{2^{k}}{(2 i-1)\left(2^{k}-2 i+1\right)}=2^{k} \sum_{i=1}^{2^{k-1}} \frac{\left(2^{k}-1\right) ! !}{(2 i-1)\left(2^{k}-2 i+1\right)}=2^{k} S +\end{aligned} +$$ + +For any integer $i=1, \ldots, 2^{k-1}$, denote by $a_{2 i-1}$ the residue inverse to $2 i-1$ modulo $2^{k}$. Clearly, when $2 i-1$ runs through all odd residues, so does $a_{2 i-1}$, hence + +$$ +\begin{aligned} +& S=\sum_{i=1}^{2^{k-1}} \frac{\left(2^{k}-1\right) ! !}{(2 i-1)\left(2^{k}-2 i+1\right)} \equiv-\sum_{i=1}^{2^{k-1}} \frac{\left(2^{k}-1\right) ! !}{(2 i-1)^{2}} \equiv-\sum_{i=1}^{2^{k-1}}\left(2^{k}-1\right) ! ! a_{2 i-1}^{2} \\ +&=-\left(2^{k}-1\right) ! ! \sum_{i=1}^{2^{k-1}}(2 i-1)^{2}=-\left(2^{k}-1\right) ! ! \frac{2^{k-1}\left(2^{2 k}-1\right)}{3} \quad\left(\bmod 2^{k}\right) . +\end{aligned} +$$ + +Therefore, the exponent of 2 in $S$ is $k-1$, so $c=2^{k} S=2^{2 k-1}(2 t+1)$ for some integer $t$. + +Finally we obtain that + +$$ +P\left(2^{k}\right)=2^{3 k} Q\left(2^{k}\right)+2^{k} c=2^{3 k} Q\left(2^{k}\right)+2^{3 k-1}(2 t+1), +$$ + +which is divisible exactly by $2^{3 k-1}$. Thus, the exponent of 2 in $(2)$ is $1+(3 k-1)=3 k$. + +Comment. The fact that (1) is divisible by $2^{2 k}$ is known; but it does not help in solving this problem. + +N5. Find all surjective functions $f: \mathbb{N} \rightarrow \mathbb{N}$ such that for every $m, n \in \mathbb{N}$ and every prime $p$, the number $f(m+n)$ is divisible by $p$ if and only if $f(m)+f(n)$ is divisible by $p$. + +( $\mathbb{N}$ is the set of all positive integers.) + +(Iran) + +Answer. $f(n)=n$. + +Solution. Suppose that function $f: \mathbb{N} \rightarrow \mathbb{N}$ satisfies the problem conditions. + +Lemma. For any prime $p$ and any $x, y \in \mathbb{N}$, we have $x \equiv y(\bmod p)$ if and only if $f(x) \equiv f(y)$ $(\bmod p)$. Moreover, $p \mid f(x)$ if and only if $p \mid x$. + +Proof. Consider an arbitrary prime $p$. Since $f$ is surjective, there exists some $x \in \mathbb{N}$ such that $p \mid f(x)$. Let + +$$ +d=\min \{x \in \mathbb{N}: p \mid f(x)\} . +$$ + +By induction on $k$, we obtain that $p \mid f(k d)$ for all $k \in \mathbb{N}$. The base is true since $p \mid f(d)$. Moreover, if $p \mid f(k d)$ and $p \mid f(d)$ then, by the problem condition, $p \mid f(k d+d)=f((k+1) d)$ as required. + +Suppose that there exists an $x \in \mathbb{N}$ such that $d \not x$ but $p \mid f(x)$. Let + +$$ +y=\min \{x \in \mathbb{N}: d \not\{x, p \mid f(x)\} . +$$ + +By the choice of $d$, we have $y>d$, and $y-d$ is a positive integer not divisible by $d$. Then $p \nmid\{(y-d)$, while $p \mid f(d)$ and $p \mid f(d+(y-d))=f(y)$. This contradicts the problem condition. Hence, there is no such $x$, and + +$$ +p|f(x) \Longleftrightarrow d| x . +$$ + +Take arbitrary $x, y \in \mathbb{N}$ such that $x \equiv y(\bmod d)$. We have $p \mid f(x+(2 x d-x))=f(2 x d)$; moreover, since $d \mid 2 x d+(y-x)=y+(2 x d-x)$, we get $p \mid f(y+(2 x d-x))$. Then by the problem condition $p|f(x)+f(2 x d-x), p| f(y)+f(2 x d-x)$, and hence $f(x) \equiv-f(2 x d-x) \equiv f(y)$ $(\bmod p)$. + +On the other hand, assume that $f(x) \equiv f(y)(\bmod p)$. Again we have $p \mid f(x)+f(2 x d-x)$ which by our assumption implies that $p \mid f(x)+f(2 x d-x)+(f(y)-f(x))=f(y)+f(2 x d-x)$. Hence by the problem condition $p \mid f(y+(2 x d-x))$. Using (1) we get $0 \equiv y+(2 x d-x) \equiv y-x$ $(\bmod d)$. + +Thus, we have proved that + +$$ +x \equiv y \quad(\bmod d) \Longleftrightarrow f(x) \equiv f(y) \quad(\bmod p) +$$ + +We are left to show that $p=d$ : in this case (1) and (2) provide the desired statements. + +The numbers $1,2, \ldots, d$ have distinct residues modulo $d$. By (2), numbers $f(1), f(2), \ldots$, $f(d)$ have distinct residues modulo $p$; hence there are at least $d$ distinct residues, and $p \geq d$. On the other hand, by the surjectivity of $f$, there exist $x_{1}, \ldots, x_{p} \in \mathbb{N}$ such that $f\left(x_{i}\right)=i$ for any $i=1,2, \ldots, p$. By (2), all these $x_{i}$ 's have distinct residues modulo $d$. For the same reasons, $d \geq p$. Hence, $d=p$. + +Now we prove that $f(n)=n$ by induction on $n$. If $n=1$ then, by the Lemma, $p \not \nmid f(1)$ for any prime $p$, so $f(1)=1$, and the base is established. Suppose that $n>1$ and denote $k=f(n)$. Note that there exists a prime $q \mid n$, so by the Lemma $q \mid k$ and $k>1$. + +If $k>n$ then $k-n+1>1$, and there exists a prime $p \mid k-n+1$; we have $k \equiv n-1$ $(\bmod p)$. By the induction hypothesis we have $f(n-1)=n-1 \equiv k=f(n)(\bmod p)$. Now, by the Lemma we obtain $n-1 \equiv n(\bmod p)$ which cannot be true. + +Analogously, if $k1$, so there exists a prime $p \mid n-k+1$ and $n \equiv k-1(\bmod p)$. By the Lemma again, $k=f(n) \equiv$ $f(k-1)=k-1(\bmod p)$, which is also false. The only remaining case is $k=n$, so $f(n)=n$. + +Finally, the function $f(n)=n$ obviously satisfies the condition. + +N6. Let $k$ be a positive integer. Prove that the number $\left(4 k^{2}-1\right)^{2}$ has a positive divisor of the form $8 k n-1$ if and only if $k$ is even. + +(United Kingdom) + +Solution. The statement follows from the following fact. + +Lemma. For arbitrary positive integers $x$ and $y$, the number $4 x y-1$ divides $\left(4 x^{2}-1\right)^{2}$ if and only if $x=y$. + +Proof. If $x=y$ then $4 x y-1=4 x^{2}-1$ obviously divides $\left(4 x^{2}-1\right)^{2}$ so it is sufficient to consider the opposite direction. + +Call a pair $(x, y)$ of positive integers bad if $4 x y-1$ divides $\left(4 x^{2}-1\right)^{2}$ but $x \neq y$. In order to prove that bad pairs do not exist, we present two properties of them which provide an infinite descent. + +Property (i). If $(x, y)$ is a bad pair and $x1$ and $a x y-1$ divides $\left(a x^{2}-1\right)^{2}$ then $x=y$. + +N7. For a prime $p$ and a positive integer $n$, denote by $\nu_{p}(n)$ the exponent of $p$ in the prime factorization of $n$ !. Given a positive integer $d$ and a finite set $\left\{p_{1}, \ldots, p_{k}\right\}$ of primes. Show that there are infinitely many positive integers $n$ such that $d \mid \nu_{p_{i}}(n)$ for all $1 \leq i \leq k$. + +(India) + +Solution 1. For arbitrary prime $p$ and positive integer $n$, denote by $\operatorname{ord}_{p}(n)$ the exponent of $p$ in $n$. Thus, + +$$ +\nu_{p}(n)=\operatorname{ord}_{p}(n !)=\sum_{i=1}^{n} \operatorname{ord}_{p}(i) +$$ + +Lemma. Let $p$ be a prime number, $q$ be a positive integer, $k$ and $r$ be positive integers such that $p^{k}>r$. Then $\nu_{p}\left(q p^{k}+r\right)=\nu_{p}\left(q p^{k}\right)+\nu_{p}(r)$. + +Proof. We claim that $\operatorname{ord}_{p}\left(q p^{k}+i\right)=\operatorname{ord}_{p}(i)$ for all $01$. By the construction of the sequence, $p_{i}^{n_{\ell_{1}}}$ divides $n_{\ell_{2}}+\ldots+n_{\ell_{m}}$; clearly, $p_{i}^{n_{\ell_{1}}}>n_{\ell_{1}}$ for all $1 \leq i \leq k$. Therefore the Lemma can be applied for $p=p_{i}, k=r=n_{\ell_{1}}$ and $q p^{k}=n_{\ell_{2}}+\ldots+n_{\ell_{m}}$ to obtain + +$$ +f_{i}\left(n_{\ell_{1}}+n_{\ell_{2}}+\ldots+n_{\ell_{m}}\right)=f_{i}\left(n_{\ell_{1}}\right)+f_{i}\left(n_{\ell_{2}}+\ldots+n_{\ell_{m}}\right) \quad \text { for all } 1 \leq i \leq k +$$ + +and hence + +$$ +f\left(n_{\ell_{1}}+n_{\ell_{2}}+\ldots+n_{\ell_{m}}\right)=f\left(n_{\ell_{1}}\right)+f\left(n_{\ell_{2}}+\ldots+n_{\ell_{m}}\right)=f\left(n_{\ell_{1}}\right)+f\left(n_{\ell_{2}}\right)+\ldots+f\left(n_{\ell_{m}}\right) +$$ + +by the induction hypothesis. + +Now consider the values $f\left(n_{1}\right), f\left(n_{2}\right), \ldots$ There exist finitely many possible values of $f$. Hence, there exists an infinite sequence of indices $\ell_{1}<\ell_{2}<\ldots$ such that $f\left(n_{\ell_{1}}\right)=f\left(n_{\ell_{2}}\right)=\ldots$ and thus + +$$ +f\left(n_{\ell_{m+1}}+n_{\ell_{m+2}}+\ldots+n_{\ell_{m+d}}\right)=f\left(n_{\ell_{m+1}}\right)+\ldots+f\left(n_{\ell_{m+d}}\right)=d \cdot f\left(n_{\ell_{1}}\right)=(\overline{0}, \ldots, \overline{0}) +$$ + +for all $m$. We have found infinitely many suitable numbers. + +Solution 2. We use the same Lemma and definition of the function $f$. + +Let $S=\{f(n): n \in \mathbb{N}\}$. Obviously, set $S$ is finite. For every $s \in S$ choose the minimal $n_{s}$ such that $f\left(n_{s}\right)=s$. Denote $N=\max _{s \in S} n_{s}$. Moreover, let $g$ be an integer such that $p_{i}^{g}>N$ for each $i=1,2, \ldots, k$. Let $P=\left(p_{1} p_{2} \ldots p_{k}\right)^{g}$. + +We claim that + +$$ +\{f(n) \mid n \in[m P, m P+N]\}=S +$$ + +for every positive integer $m$. In particular, since $(\overline{0}, \ldots, \overline{0})=f(1) \in S$, it follows that for an arbitrary $m$ there exists $n \in[m P, m P+N]$ such that $f(n)=(\overline{0}, \ldots, \overline{0})$. So there are infinitely many suitable numbers. + +To prove (1), let $a_{i}=f_{i}(m P)$. Consider all numbers of the form $n_{m, s}=m P+n_{s}$ with $s=\left(s_{1}, \ldots, s_{k}\right) \in S$ (clearly, all $n_{m, s}$ belong to $[m P, m P+N]$ ). Since $n_{s} \leq NJuan Manuel Conde Calero
Géza Kós
Marcin Kuczma
Daniel Lasaosa Medarde
Ignasi Mundet i Riera
Svetoslav Savchev + +## Algebra + +A1. Find all functions $f:(0, \infty) \rightarrow(0, \infty)$ such that + +$$ +\frac{f(p)^{2}+f(q)^{2}}{f\left(r^{2}\right)+f\left(s^{2}\right)}=\frac{p^{2}+q^{2}}{r^{2}+s^{2}} +$$ + +for all $p, q, r, s>0$ with $p q=r s$. + +Solution. Let $f$ satisfy the given condition. Setting $p=q=r=s=1$ yields $f(1)^{2}=f(1)$ and hence $f(1)=1$. Now take any $x>0$ and set $p=x, q=1, r=s=\sqrt{x}$ to obtain + +$$ +\frac{f(x)^{2}+1}{2 f(x)}=\frac{x^{2}+1}{2 x} . +$$ + +This recasts into + +$$ +\begin{gathered} +x f(x)^{2}+x=x^{2} f(x)+f(x), \\ +(x f(x)-1)(f(x)-x)=0 . +\end{gathered} +$$ + +And thus, + +$$ +\text { for every } x>0, \text { either } f(x)=x \text { or } f(x)=\frac{1}{x} \text {. } +$$ + +Obviously, if + +$$ +f(x)=x \quad \text { for all } x>0 \quad \text { or } \quad f(x)=\frac{1}{x} \quad \text { for all } x>0 +$$ + +then the condition of the problem is satisfied. We show that actually these two functions are the only solutions. + +So let us assume that there exists a function $f$ satisfying the requirement, other than those in (2). Then $f(a) \neq a$ and $f(b) \neq 1 / b$ for some $a, b>0$. By (1), these values must be $f(a)=1 / a, f(b)=b$. Applying now the equation with $p=a, q=b, r=s=\sqrt{a b}$ we obtain $\left(a^{-2}+b^{2}\right) / 2 f(a b)=\left(a^{2}+b^{2}\right) / 2 a b ;$ equivalently, + +$$ +f(a b)=\frac{a b\left(a^{-2}+b^{2}\right)}{a^{2}+b^{2}} . +$$ + +We know however (see (1)) that $f(a b)$ must be either $a b$ or $1 / a b$. If $f(a b)=a b$ then by (3) $a^{-2}+b^{2}=a^{2}+b^{2}$, so that $a=1$. But, as $f(1)=1$, this contradicts the relation $f(a) \neq a$. Likewise, if $f(a b)=1 / a b$ then (3) gives $a^{2} b^{2}\left(a^{-2}+b^{2}\right)=a^{2}+b^{2}$, whence $b=1$, in contradiction to $f(b) \neq 1 / b$. Thus indeed the functions listed in (2) are the only two solutions. + +Comment. The equation has as many as four variables with only one constraint $p q=r s$, leaving three degrees of freedom and providing a lot of information. Various substitutions force various useful properties of the function searched. We sketch one more method to reach conclusion (1); certainly there are many others. + +Noticing that $f(1)=1$ and setting, first, $p=q=1, r=\sqrt{x}, s=1 / \sqrt{x}$, and then $p=x, q=1 / x$, $r=s=1$, we obtain two relations, holding for every $x>0$, + +$$ +f(x)+f\left(\frac{1}{x}\right)=x+\frac{1}{x} \quad \text { and } \quad f(x)^{2}+f\left(\frac{1}{x}\right)^{2}=x^{2}+\frac{1}{x^{2}} . +$$ + +Squaring the first and subtracting the second gives $2 f(x) f(1 / x)=2$. Subtracting this from the second relation of (4) leads to + +$$ +\left(f(x)-f\left(\frac{1}{x}\right)\right)^{2}=\left(x-\frac{1}{x}\right)^{2} \quad \text { or } \quad f(x)-f\left(\frac{1}{x}\right)= \pm\left(x-\frac{1}{x}\right) . +$$ + +The last two alternatives combined with the first equation of (4) imply the two alternatives of (1). + +A2. (a) Prove the inequality + +$$ +\frac{x^{2}}{(x-1)^{2}}+\frac{y^{2}}{(y-1)^{2}}+\frac{z^{2}}{(z-1)^{2}} \geq 1 +$$ + +for real numbers $x, y, z \neq 1$ satisfying the condition $x y z=1$. + +(b) Show that there are infinitely many triples of rational numbers $x, y, z$ for which this inequality turns into equality. + +Solution 1. (a) We start with the substitution + +$$ +\frac{x}{x-1}=a, \quad \frac{y}{y-1}=b, \quad \frac{z}{z-1}=c, \quad \text { i.e., } \quad x=\frac{a}{a-1}, \quad y=\frac{b}{b-1}, \quad z=\frac{c}{c-1} \text {. } +$$ + +The inequality to be proved reads $a^{2}+b^{2}+c^{2} \geq 1$. The new variables are subject to the constraints $a, b, c \neq 1$ and the following one coming from the condition $x y z=1$, + +$$ +(a-1)(b-1)(c-1)=a b c . +$$ + +This is successively equivalent to + +$$ +\begin{aligned} +a+b+c-1 & =a b+b c+c a, \\ +2(a+b+c-1) & =(a+b+c)^{2}-\left(a^{2}+b^{2}+c^{2}\right), \\ +a^{2}+b^{2}+c^{2}-2 & =(a+b+c)^{2}-2(a+b+c), \\ +a^{2}+b^{2}+c^{2}-1 & =(a+b+c-1)^{2} . +\end{aligned} +$$ + +Thus indeed $a^{2}+b^{2}+c^{2} \geq 1$, as desired. + +(b) From the equation $a^{2}+b^{2}+c^{2}-1=(a+b+c-1)^{2}$ we see that the proposed inequality becomes an equality if and only if both sums $a^{2}+b^{2}+c^{2}$ and $a+b+c$ have value 1 . The first of them is equal to $(a+b+c)^{2}-2(a b+b c+c a)$. So the instances of equality are described by the system of two equations + +$$ +a+b+c=1, \quad a b+b c+c a=0 +$$ + +plus the constraint $a, b, c \neq 1$. Elimination of $c$ leads to $a^{2}+a b+b^{2}=a+b$, which we regard as a quadratic equation in $b$, + +$$ +b^{2}+(a-1) b+a(a-1)=0, +$$ + +with discriminant + +$$ +\Delta=(a-1)^{2}-4 a(a-1)=(1-a)(1+3 a) . +$$ + +We are looking for rational triples $(a, b, c)$; it will suffice to have $a$ rational such that $1-a$ and $1+3 a$ are both squares of rational numbers (then $\Delta$ will be so too). Set $a=k / m$. We want $m-k$ and $m+3 k$ to be squares of integers. This is achieved for instance by taking $m=k^{2}-k+1$ (clearly nonzero); then $m-k=(k-1)^{2}, m+3 k=(k+1)^{2}$. Note that distinct integers $k$ yield distinct values of $a=k / m$. + +And thus, if $k$ is any integer and $m=k^{2}-k+1, a=k / m$ then $\Delta=\left(k^{2}-1\right)^{2} / m^{2}$ and the quadratic equation has rational roots $b=\left(m-k \pm k^{2} \mp 1\right) /(2 m)$. Choose e.g. the larger root, + +$$ +b=\frac{m-k+k^{2}-1}{2 m}=\frac{m+(m-2)}{2 m}=\frac{m-1}{m} . +$$ + +Computing $c$ from $a+b+c=1$ then gives $c=(1-k) / m$. The condition $a, b, c \neq 1$ eliminates only $k=0$ and $k=1$. Thus, as $k$ varies over integers greater than 1 , we obtain an infinite family of rational triples $(a, b, c)$ - and coming back to the original variables $(x=a /(a-1)$ etc. $)$-an infinite family of rational triples $(x, y, z)$ with the needed property. (A short calculation shows that the resulting triples are $x=-k /(k-1)^{2}, y=k-k^{2}, z=(k-1) / k^{2}$; but the proof was complete without listing them.) + +Comment 1. There are many possible variations in handling the equation system $a^{2}+b^{2}+c^{2}=1$, $a+b+c=1(a, b, c \neq 1)$ which of course describes a circle in the $(a, b, c)$-space (with three points excluded), and finding infinitely many rational points on it. + +Also the initial substitution $x=a /(a-1)$ (etc.) can be successfully replaced by other similar substitutions, e.g. $x=1-1 / \alpha$ (etc.); or $x=x^{\prime}-1$ (etc.); or $1-y z=u$ (etc.) - eventually reducing the inequality to $(\cdots)^{2} \geq 0$, the expression in the parentheses depending on the actual substitution. + +Depending on the method chosen, one arrives at various sequences of rational triples $(x, y, z)$ as needed; let us produce just one more such example: $x=(2 r-2) /(r+1)^{2}, y=(2 r+2) /(r-1)^{2}$, $z=\left(r^{2}-1\right) / 4$ where $r$ can be any rational number different from 1 or -1 . + +Solution 2 (an outline). (a) Without changing variables, just setting $z=1 / x y$ and clearing fractions, the proposed inequality takes the form + +$$ +(x y-1)^{2}\left(x^{2}(y-1)^{2}+y^{2}(x-1)^{2}\right)+(x-1)^{2}(y-1)^{2} \geq(x-1)^{2}(y-1)^{2}(x y-1)^{2} . +$$ + +With the notation $p=x+y, q=x y$ this becomes, after lengthy routine manipulation and a lot of cancellation + +$$ +q^{4}-6 q^{3}+2 p q^{2}+9 q^{2}-6 p q+p^{2} \geq 0 \text {. } +$$ + +It is not hard to notice that the expression on the left is just $\left(q^{2}-3 q+p\right)^{2}$, hence nonnegative. + +(Without introducing $p$ and $q$, one is of course led with some more work to the same expression, just written in terms of $x$ and $y$; but then it is not that easy to see that it is a square.) + +(b) To have equality, one needs $q^{2}-3 q+p=0$. Note that $x$ and $y$ are the roots of the quadratic trinomial (in a formal variable $t$ ): $t^{2}-p t+q$. When $q^{2}-3 q+p=0$, the discriminant equals + +$$ +\delta=p^{2}-4 q=\left(3 q-q^{2}\right)^{2}-4 q=q(q-1)^{2}(q-4) . +$$ + +Now it suffices to have both $q$ and $q-4$ squares of rational numbers (then $p=3 q-q^{2}$ and $\sqrt{\delta}$ are also rational, and so are the roots of the trinomial). On setting $q=(n / m)^{2}=4+(l / m)^{2}$ the requirement becomes $4 m^{2}+l^{2}=n^{2}$ (with $l, m, n$ being integers). This is just the Pythagorean equation, known to have infinitely many integer solutions. + +Comment 2. Part (a) alone might also be considered as a possible contest problem (in the category of easy problems). + +A3. Let $S \subseteq \mathbb{R}$ be a set of real numbers. We say that a pair $(f, g)$ of functions from $S$ into $S$ is a Spanish Couple on $S$, if they satisfy the following conditions: + +(i) Both functions are strictly increasing, i.e. $f(x)0, C>2$ and $B=1$ produce a Spanish couple (in the example above, $A=1, C=3$ ). The proposer's example results from taking $h(a)=a+1, G(a, b)=3^{a}+b$. + +A4. For an integer $m$, denote by $t(m)$ the unique number in $\{1,2,3\}$ such that $m+t(m)$ is a multiple of 3. A function $f: \mathbb{Z} \rightarrow \mathbb{Z}$ satisfies $f(-1)=0, f(0)=1, f(1)=-1$ and + +$$ +f\left(2^{n}+m\right)=f\left(2^{n}-t(m)\right)-f(m) \quad \text { for all integers } m, n \geq 0 \text { with } 2^{n}>m \text {. } +$$ + +Prove that $f(3 p) \geq 0$ holds for all integers $p \geq 0$. + +Solution. The given conditions determine $f$ uniquely on the positive integers. The signs of $f(1), f(2), \ldots$ seem to change quite erratically. However values of the form $f\left(2^{n}-t(m)\right)$ are sufficient to compute directly any functional value. Indeed, let $n>0$ have base 2 representation $n=2^{a_{0}}+2^{a_{1}}+\cdots+2^{a_{k}}, a_{0}>a_{1}>\cdots>a_{k} \geq 0$, and let $n_{j}=2^{a_{j}}+2^{a_{j-1}}+\cdots+2^{a_{k}}, j=0, \ldots, k$. Repeated applications of the recurrence show that $f(n)$ is an alternating sum of the quantities $f\left(2^{a_{j}}-t\left(n_{j+1}\right)\right)$ plus $(-1)^{k+1}$. (The exact formula is not needed for our proof.) + +So we focus attention on the values $f\left(2^{n}-1\right), f\left(2^{n}-2\right)$ and $f\left(2^{n}-3\right)$. Six cases arise; more specifically, + +$t\left(2^{2 k}-3\right)=2, t\left(2^{2 k}-2\right)=1, t\left(2^{2 k}-1\right)=3, t\left(2^{2 k+1}-3\right)=1, t\left(2^{2 k+1}-2\right)=3, t\left(2^{2 k+1}-1\right)=2$. + +Claim. For all integers $k \geq 0$ the following equalities hold: + +$$ +\begin{array}{lll} +f\left(2^{2 k+1}-3\right)=0, & f\left(2^{2 k+1}-2\right)=3^{k}, & f\left(2^{2 k+1}-1\right)=-3^{k}, \\ +f\left(2^{2 k+2}-3\right)=-3^{k}, & f\left(2^{2 k+2}-2\right)=-3^{k}, & f\left(2^{2 k+2}-1\right)=2 \cdot 3^{k} . +\end{array} +$$ + +Proof. By induction on $k$. The base $k=0$ comes down to checking that $f(2)=-1$ and $f(3)=2$; the given values $f(-1)=0, f(0)=1, f(1)=-1$ are also needed. Suppose the claim holds for $k-1$. For $f\left(2^{2 k+1}-t(m)\right)$, the recurrence formula and the induction hypothesis yield + +$$ +\begin{aligned} +& f\left(2^{2 k+1}-3\right)=f\left(2^{2 k}+\left(2^{2 k}-3\right)\right)=f\left(2^{2 k}-2\right)-f\left(2^{2 k}-3\right)=-3^{k-1}+3^{k-1}=0, \\ +& f\left(2^{2 k+1}-2\right)=f\left(2^{2 k}+\left(2^{2 k}-2\right)\right)=f\left(2^{2 k}-1\right)-f\left(2^{2 k}-2\right)=2 \cdot 3^{k-1}+3^{k-1}=3^{k}, \\ +& f\left(2^{2 k+1}-1\right)=f\left(2^{2 k}+\left(2^{2 k}-1\right)\right)=f\left(2^{2 k}-3\right)-f\left(2^{2 k}-1\right)=-3^{k-1}-2 \cdot 3^{k-1}=-3^{k} . +\end{aligned} +$$ + +For $f\left(2^{2 k+2}-t(m)\right)$ we use the three equalities just established: + +$$ +\begin{aligned} +& f\left(2^{2 k+2}-3\right)=f\left(2^{2 k+1}+\left(2^{2 k+1}-3\right)\right)=f\left(2^{2 k+1}-1\right)-f\left(2^{2 k+1}-3\right)=-3^{k}-0=-3^{k}, \\ +& f\left(2^{2 k+2}-2\right)=f\left(2^{2 k+1}+\left(2^{2 k+1}-2\right)\right)=f\left(2^{2 k+1}-3\right)-f\left(2^{2 k}-2\right)=0-3^{k}=-3^{k}, \\ +& f\left(2^{2 k+2}-1\right)=f\left(2^{2 k+1}+\left(2^{2 k+1}-1\right)\right)=f\left(2^{2 k+1}-2\right)-f\left(2^{2 k+1}-1\right)=3^{k}+3^{k}=2 \cdot 3^{k} . +\end{aligned} +$$ + +The claim follows. + +A closer look at the six cases shows that $f\left(2^{n}-t(m)\right) \geq 3^{(n-1) / 2}$ if $2^{n}-t(m)$ is divisible by 3 , and $f\left(2^{n}-t(m)\right) \leq 0$ otherwise. On the other hand, note that $2^{n}-t(m)$ is divisible by 3 if and only if $2^{n}+m$ is. Therefore, for all nonnegative integers $m$ and $n$, + +(i) $f\left(2^{n}-t(m)\right) \geq 3^{(n-1) / 2}$ if $2^{n}+m$ is divisible by 3 ; + +(ii) $f\left(2^{n}-t(m)\right) \leq 0$ if $2^{n}+m$ is not divisible by 3 . + +One more (direct) consequence of the claim is that $\left|f\left(2^{n}-t(m)\right)\right| \leq \frac{2}{3} \cdot 3^{n / 2}$ for all $m, n \geq 0$. + +The last inequality enables us to find an upper bound for $|f(m)|$ for $m$ less than a given power of 2 . We prove by induction on $n$ that $|f(m)| \leq 3^{n / 2}$ holds true for all integers $m, n \geq 0$ with $2^{n}>m$. + +The base $n=0$ is clear as $f(0)=1$. For the inductive step from $n$ to $n+1$, let $m$ and $n$ satisfy $2^{n+1}>m$. If $m<2^{n}$, we are done by the inductive hypothesis. If $m \geq 2^{n}$ then $m=2^{n}+k$ where $2^{n}>k \geq 0$. Now, by $\left|f\left(2^{n}-t(k)\right)\right| \leq \frac{2}{3} \cdot 3^{n / 2}$ and the inductive assumption, + +$$ +|f(m)|=\left|f\left(2^{n}-t(k)\right)-f(k)\right| \leq\left|f\left(2^{n}-t(k)\right)\right|+|f(k)| \leq \frac{2}{3} \cdot 3^{n / 2}+3^{n / 2}<3^{(n+1) / 2} . +$$ + +The induction is complete. + +We proceed to prove that $f(3 p) \geq 0$ for all integers $p \geq 0$. Since $3 p$ is not a power of 2 , its binary expansion contains at least two summands. Hence one can write $3 p=2^{a}+2^{b}+c$ where $a>b$ and $2^{b}>c \geq 0$. Applying the recurrence formula twice yields + +$$ +f(3 p)=f\left(2^{a}+2^{b}+c\right)=f\left(2^{a}-t\left(2^{b}+c\right)\right)-f\left(2^{b}-t(c)\right)+f(c) . +$$ + +Since $2^{a}+2^{b}+c$ is divisible by 3 , we have $f\left(2^{a}-t\left(2^{b}+c\right)\right) \geq 3^{(a-1) / 2}$ by (i). Since $2^{b}+c$ is not divisible by 3 , we have $f\left(2^{b}-t(c)\right) \leq 0$ by (ii). Finally $|f(c)| \leq 3^{b / 2}$ as $2^{b}>c \geq 0$, so that $f(c) \geq-3^{b / 2}$. Therefore $f(3 p) \geq 3^{(a-1) / 2}-3^{b / 2}$ which is nonnegative because $a>b$. + +A5. Let $a, b, c, d$ be positive real numbers such that + +$$ +a b c d=1 \quad \text { and } \quad a+b+c+d>\frac{a}{b}+\frac{b}{c}+\frac{c}{d}+\frac{d}{a} +$$ + +Prove that + +$$ +a+b+c+d<\frac{b}{a}+\frac{c}{b}+\frac{d}{c}+\frac{a}{d} +$$ + +Solution. We show that if $a b c d=1$, the sum $a+b+c+d$ cannot exceed a certain weighted mean of the expressions $\frac{a}{b}+\frac{b}{c}+\frac{c}{d}+\frac{d}{a}$ and $\frac{b}{a}+\frac{c}{b}+\frac{d}{c}+\frac{a}{d}$. + +By applying the AM-GM inequality to the numbers $\frac{a}{b}, \frac{a}{b}, \frac{b}{c}$ and $\frac{a}{d}$, we obtain + +$$ +a=\sqrt[4]{\frac{a^{4}}{a b c d}}=\sqrt[4]{\frac{a}{b} \cdot \frac{a}{b} \cdot \frac{b}{c} \cdot \frac{a}{d}} \leq \frac{1}{4}\left(\frac{a}{b}+\frac{a}{b}+\frac{b}{c}+\frac{a}{d}\right) +$$ + +Analogously, + +$$ +b \leq \frac{1}{4}\left(\frac{b}{c}+\frac{b}{c}+\frac{c}{d}+\frac{b}{a}\right), \quad c \leq \frac{1}{4}\left(\frac{c}{d}+\frac{c}{d}+\frac{d}{a}+\frac{c}{b}\right) \quad \text { and } \quad d \leq \frac{1}{4}\left(\frac{d}{a}+\frac{d}{a}+\frac{a}{b}+\frac{d}{c}\right) . +$$ + +Summing up these estimates yields + +$$ +a+b+c+d \leq \frac{3}{4}\left(\frac{a}{b}+\frac{b}{c}+\frac{c}{d}+\frac{d}{a}\right)+\frac{1}{4}\left(\frac{b}{a}+\frac{c}{b}+\frac{d}{c}+\frac{a}{d}\right) . +$$ + +In particular, if $a+b+c+d>\frac{a}{b}+\frac{b}{c}+\frac{c}{d}+\frac{d}{a}$ then $a+b+c+d<\frac{b}{a}+\frac{c}{b}+\frac{d}{c}+\frac{a}{d}$. + +Comment. The estimate in the above solution was obtained by applying the AM-GM inequality to each column of the $4 \times 4$ array + +$$ +\begin{array}{llll} +a / b & b / c & c / d & d / a \\ +a / b & b / c & c / d & d / a \\ +b / c & c / d & d / a & a / b \\ +a / d & b / a & c / b & d / c +\end{array} +$$ + +and adding up the resulting inequalities. The same table yields a stronger bound: If $a, b, c, d>0$ and $a b c d=1$ then + +$$ +\left(\frac{a}{b}+\frac{b}{c}+\frac{c}{d}+\frac{d}{a}\right)^{3}\left(\frac{b}{a}+\frac{c}{b}+\frac{d}{c}+\frac{a}{d}\right) \geq(a+b+c+d)^{4} +$$ + +It suffices to apply Hölder's inequality to the sequences in the four rows, with weights $1 / 4$ : + +$$ +\begin{gathered} +\left(\frac{a}{b}+\frac{b}{c}+\frac{c}{d}+\frac{d}{a}\right)^{1 / 4}\left(\frac{a}{b}+\frac{b}{c}+\frac{c}{d}+\frac{d}{a}\right)^{1 / 4}\left(\frac{b}{c}+\frac{c}{d}+\frac{d}{a}+\frac{a}{b}\right)^{1 / 4}\left(\frac{a}{d}+\frac{b}{a}+\frac{c}{b}+\frac{d}{c}\right)^{1 / 4} \\ +\geq\left(\frac{a a b a}{b b c d}\right)^{1 / 4}+\left(\frac{b b c b}{c c d a}\right)^{1 / 4}+\left(\frac{c c d c}{d d a b}\right)^{1 / 4}+\left(\frac{d d a d}{a a b c}\right)^{1 / 4}=a+b+c+d +\end{gathered} +$$ + +A6. Let $f: \mathbb{R} \rightarrow \mathbb{N}$ be a function which satisfies + +$$ +f\left(x+\frac{1}{f(y)}\right)=f\left(y+\frac{1}{f(x)}\right) \quad \text { for all } x, y \in \mathbb{R} . +$$ + +Prove that there is a positive integer which is not a value of $f$. + +Solution. Suppose that the statement is false and $f(\mathbb{R})=\mathbb{N}$. We prove several properties of the function $f$ in order to reach a contradiction. + +To start with, observe that one can assume $f(0)=1$. Indeed, let $a \in \mathbb{R}$ be such that $f(a)=1$, and consider the function $g(x)=f(x+a)$. By substituting $x+a$ and $y+a$ for $x$ and $y$ in (1), we have + +$$ +g\left(x+\frac{1}{g(y)}\right)=f\left(x+a+\frac{1}{f(y+a)}\right)=f\left(y+a+\frac{1}{f(x+a)}\right)=g\left(y+\frac{1}{g(x)}\right) . +$$ + +So $g$ satisfies the functional equation (1), with the additional property $g(0)=1$. Also, $g$ and $f$ have the same set of values: $g(\mathbb{R})=f(\mathbb{R})=\mathbb{N}$. Henceforth we assume $f(0)=1$. + +Claim 1. For an arbitrary fixed $c \in \mathbb{R}$ we have $\left\{f\left(c+\frac{1}{n}\right): n \in \mathbb{N}\right\}=\mathbb{N}$. + +Proof. Equation (1) and $f(\mathbb{R})=\mathbb{N}$ imply + +$f(\mathbb{R})=\left\{f\left(x+\frac{1}{f(c)}\right): x \in \mathbb{R}\right\}=\left\{f\left(c+\frac{1}{f(x)}\right): x \in \mathbb{R}\right\} \subset\left\{f\left(c+\frac{1}{n}\right): n \in \mathbb{N}\right\} \subset f(\mathbb{R})$. + +The claim follows. + +We will use Claim 1 in the special cases $c=0$ and $c=1 / 3$ : + +$$ +\left\{f\left(\frac{1}{n}\right): n \in \mathbb{N}\right\}=\left\{f\left(\frac{1}{3}+\frac{1}{n}\right): n \in \mathbb{N}\right\}=\mathbb{N} +$$ + +Claim 2. If $f(u)=f(v)$ for some $u, v \in \mathbb{R}$ then $f(u+q)=f(v+q)$ for all nonnegative rational $q$. Furthermore, if $f(q)=1$ for some nonnegative rational $q$ then $f(k q)=1$ for all $k \in \mathbb{N}$. + +Proof. For all $x \in \mathbb{R}$ we have by (1) + +$$ +f\left(u+\frac{1}{f(x)}\right)=f\left(x+\frac{1}{f(u)}\right)=f\left(x+\frac{1}{f(v)}\right)=f\left(v+\frac{1}{f(x)}\right) +$$ + +Since $f(x)$ attains all positive integer values, this yields $f(u+1 / n)=f(v+1 / n)$ for all $n \in \mathbb{N}$. Let $q=k / n$ be a positive rational number. Then $k$ repetitions of the last step yield + +$$ +f(u+q)=f\left(u+\frac{k}{n}\right)=f\left(v+\frac{k}{n}\right)=f(v+q) +$$ + +Now let $f(q)=1$ for some nonnegative rational $q$, and let $k \in \mathbb{N}$. As $f(0)=1$, the previous conclusion yields successively $f(q)=f(2 q), f(2 q)=f(3 q), \ldots, f((k-1) q)=f(k q)$, as needed. + +Claim 3. The equality $f(q)=f(q+1)$ holds for all nonnegative rational $q$. + +Proof. Let $m$ be a positive integer such that $f(1 / m)=1$. Such an $m$ exists by (2). Applying the second statement of Claim 2 with $q=1 / m$ and $k=m$ yields $f(1)=1$. + +Given that $f(0)=f(1)=1$, the first statement of Claim 2 implies $f(q)=f(q+1)$ for all nonnegative rational $q$. + +Claim 4. The equality $f\left(\frac{1}{n}\right)=n$ holds for every $n \in \mathbb{N}$. + +Proof. For a nonnegative rational $q$ we set $x=q, y=0$ in (1) and use Claim 3 to obtain + +$$ +f\left(\frac{1}{f(q)}\right)=f\left(q+\frac{1}{f(0)}\right)=f(q+1)=f(q) . +$$ + +By (2), for each $n \in \mathbb{N}$ there exists a $k \in \mathbb{N}$ such that $f(1 / k)=n$. Applying the last equation with $q=1 / k$, we have + +$$ +n=f\left(\frac{1}{k}\right)=f\left(\frac{1}{f(1 / k)}\right)=f\left(\frac{1}{n}\right) +$$ + +Now we are ready to obtain a contradiction. Let $n \in \mathbb{N}$ be such that $f(1 / 3+1 / n)=1$. Such an $n$ exists by (2). Let $1 / 3+1 / n=s / t$, where $s, t \in \mathbb{N}$ are coprime. Observe that $t>1$ as $1 / 3+1 / n$ is not an integer. Choose $k, l \in \mathbb{N}$ so that that $k s-l t=1$. + +Because $f(0)=f(s / t)=1$, Claim 2 implies $f(k s / t)=1$. Now $f(k s / t)=f(1 / t+l)$; on the other hand $f(1 / t+l)=f(1 / t)$ by $l$ successive applications of Claim 3 . Finally, $f(1 / t)=t$ by Claim 4, leading to the impossible $t=1$. The solution is complete. + +A7. Prove that for any four positive real numbers $a, b, c, d$ the inequality + +$$ +\frac{(a-b)(a-c)}{a+b+c}+\frac{(b-c)(b-d)}{b+c+d}+\frac{(c-d)(c-a)}{c+d+a}+\frac{(d-a)(d-b)}{d+a+b} \geq 0 +$$ + +holds. Determine all cases of equality. + +Solution 1. Denote the four terms by + +$$ +A=\frac{(a-b)(a-c)}{a+b+c}, \quad B=\frac{(b-c)(b-d)}{b+c+d}, \quad C=\frac{(c-d)(c-a)}{c+d+a}, \quad D=\frac{(d-a)(d-b)}{d+a+b} . +$$ + +The expression $2 A$ splits into two summands as follows, + +$$ +2 A=A^{\prime}+A^{\prime \prime} \quad \text { where } \quad A^{\prime}=\frac{(a-c)^{2}}{a+b+c}, \quad A^{\prime \prime}=\frac{(a-c)(a-2 b+c)}{a+b+c} ; +$$ + +this is easily verified. We analogously represent $2 B=B^{\prime}+B^{\prime \prime}, 2 C=C^{\prime}+C^{\prime \prime}, 2 B=D^{\prime}+D^{\prime \prime}$ and examine each of the sums $A^{\prime}+B^{\prime}+C^{\prime}+D^{\prime}$ and $A^{\prime \prime}+B^{\prime \prime}+C^{\prime \prime}+D^{\prime \prime}$ separately. + +Write $s=a+b+c+d$; the denominators become $s-d, s-a, s-b, s-c$. By the CauchySchwarz inequality, + +$$ +\begin{aligned} +& \left(\frac{|a-c|}{\sqrt{s-d}} \cdot \sqrt{s-d}+\frac{|b-d|}{\sqrt{s-a}} \cdot \sqrt{s-a}+\frac{|c-a|}{\sqrt{s-b}} \cdot \sqrt{s-b}+\frac{|d-b|}{\sqrt{s-c}} \cdot \sqrt{s-c}\right)^{2} \\ +& \quad \leq\left(\frac{(a-c)^{2}}{s-d}+\frac{(b-d)^{2}}{s-a}+\frac{(c-a)^{2}}{s-b}+\frac{(d-b)^{2}}{s-c}\right)(4 s-s)=3 s\left(A^{\prime}+B^{\prime}+C^{\prime}+D^{\prime}\right) . +\end{aligned} +$$ + +Hence + +$$ +A^{\prime}+B^{\prime}+C^{\prime}+D^{\prime} \geq \frac{(2|a-c|+2|b-d|)^{2}}{3 s} \geq \frac{16 \cdot|a-c| \cdot|b-d|}{3 s} . +$$ + +Next we estimate the absolute value of the other sum. We couple $A^{\prime \prime}$ with $C^{\prime \prime}$ to obtain + +$$ +\begin{aligned} +A^{\prime \prime}+C^{\prime \prime} & =\frac{(a-c)(a+c-2 b)}{s-d}+\frac{(c-a)(c+a-2 d)}{s-b} \\ +& =\frac{(a-c)(a+c-2 b)(s-b)+(c-a)(c+a-2 d)(s-d)}{(s-d)(s-b)} \\ +& =\frac{(a-c)(-2 b(s-b)-b(a+c)+2 d(s-d)+d(a+c))}{s(a+c)+b d} \\ +& =\frac{3(a-c)(d-b)(a+c)}{M}, \quad \text { with } \quad M=s(a+c)+b d . +\end{aligned} +$$ + +Hence by cyclic shift + +$$ +B^{\prime \prime}+D^{\prime \prime}=\frac{3(b-d)(a-c)(b+d)}{N}, \quad \text { with } \quad N=s(b+d)+c a +$$ + +Thus + +$$ +A^{\prime \prime}+B^{\prime \prime}+C^{\prime \prime}+D^{\prime \prime}=3(a-c)(b-d)\left(\frac{b+d}{N}-\frac{a+c}{M}\right)=\frac{3(a-c)(b-d) W}{M N} +$$ + +where + +$$ +W=(b+d) M-(a+c) N=b d(b+d)-a c(a+c) . +$$ + +Note that + +$$ +M N>(a c(a+c)+b d(b+d)) s \geq|W| \cdot s . +$$ + +Now (2) and (4) yield + +$$ +\left|A^{\prime \prime}+B^{\prime \prime}+C^{\prime \prime}+D^{\prime \prime}\right| \leq \frac{3 \cdot|a-c| \cdot|b-d|}{s} +$$ + +Combined with (1) this results in + +$$ +\begin{aligned} +2(A+B & +C+D)=\left(A^{\prime}+B^{\prime}+C^{\prime}+D^{\prime}\right)+\left(A^{\prime \prime}+B^{\prime \prime}+C^{\prime \prime}+D^{\prime \prime}\right) \\ +& \geq \frac{16 \cdot|a-c| \cdot|b-d|}{3 s}-\frac{3 \cdot|a-c| \cdot|b-d|}{s}=\frac{7 \cdot|a-c| \cdot|b-d|}{3(a+b+c+d)} \geq 0 +\end{aligned} +$$ + +This is the required inequality. From the last line we see that equality can be achieved only if either $a=c$ or $b=d$. Since we also need equality in (1), this implies that actually $a=c$ and $b=d$ must hold simultaneously, which is obviously also a sufficient condition. + +Solution 2. We keep the notations $A, B, C, D, s$, and also $M, N, W$ from the preceding solution; the definitions of $M, N, W$ and relations (3), (4) in that solution did not depend on the foregoing considerations. Starting from + +$$ +2 A=\frac{(a-c)^{2}+3(a+c)(a-c)}{a+b+c}-2 a+2 c +$$ + +we get + +$$ +\begin{aligned} +2(A & +C)=(a-c)^{2}\left(\frac{1}{s-d}+\frac{1}{s-b}\right)+3(a+c)(a-c)\left(\frac{1}{s-d}-\frac{1}{s-b}\right) \\ +& =(a-c)^{2} \frac{2 s-b-d}{M}+3(a+c)(a-c) \cdot \frac{d-b}{M}=\frac{p(a-c)^{2}-3(a+c)(a-c)(b-d)}{M} +\end{aligned} +$$ + +where $p=2 s-b-d=s+a+c$. Similarly, writing $q=s+b+d$ we have + +$$ +2(B+D)=\frac{q(b-d)^{2}-3(b+d)(b-d)(c-a)}{N} ; +$$ + +specific grouping of terms in the numerators has its aim. Note that $p q>2 s^{2}$. By adding the fractions expressing $2(A+C)$ and $2(B+D)$, + +$$ +2(A+B+C+D)=\frac{p(a-c)^{2}}{M}+\frac{3(a-c)(b-d) W}{M N}+\frac{q(b-d)^{2}}{N} +$$ + +with $W$ defined by (3). + +Substitution $x=(a-c) / M, y=(b-d) / N$ brings the required inequality to the form + +$$ +2(A+B+C+D)=M p x^{2}+3 W x y+N q y^{2} \geq 0 . +$$ + +It will be enough to verify that the discriminant $\Delta=9 W^{2}-4 M N p q$ of the quadratic trinomial $M p t^{2}+3 W t+N q$ is negative; on setting $t=x / y$ one then gets (6). The first inequality in (4) together with $p q>2 s^{2}$ imply $4 M N p q>8 s^{3}(a c(a+c)+b d(b+d))$. Since + +$$ +(a+c) s^{3}>(a+c)^{4} \geq 4 a c(a+c)^{2} \quad \text { and likewise } \quad(b+d) s^{3}>4 b d(b+d)^{2} +$$ + +the estimate continues as follows, + +$$ +4 M N p q>8\left(4(a c)^{2}(a+c)^{2}+4(b d)^{2}(b+d)^{2}\right)>32(b d(b+d)-a c(a+c))^{2}=32 W^{2} \geq 9 W^{2} +$$ + +Thus indeed $\Delta<0$. The desired inequality (6) hence results. It becomes an equality if and only if $x=y=0$; equivalently, if and only if $a=c$ and simultaneously $b=d$. + +Comment. The two solutions presented above do not differ significantly; large portions overlap. The properties of the number $W$ turn out to be crucial in both approaches. The Cauchy-Schwarz inequality, applied in the first solution, is avoided in the second, which requires no knowledge beyond quadratic trinomials. + +The estimates in the proof of $\Delta<0$ in the second solution seem to be very wasteful. However, they come close to sharp when the terms in one of the pairs $(a, c),(b, d)$ are equal and much bigger than those in the other pair. + +In attempts to prove the inequality by just considering the six cases of arrangement of the numbers $a, b, c, d$ on the real line, one soon discovers that the cases which create real trouble are precisely those in which $a$ and $c$ are both greater or both smaller than $b$ and $d$. + +## Solution 3. + +$$ +\begin{gathered} +(a-b)(a-c)(a+b+d)(a+c+d)(b+c+d)= \\ +=((a-b)(a+b+d))((a-c)(a+c+d))(b+c+d)= \\ +=\left(a^{2}+a d-b^{2}-b d\right)\left(a^{2}+a d-c^{2}-c d\right)(b+c+d)= \\ +=\left(a^{4}+2 a^{3} d-a^{2} b^{2}-a^{2} b d-a^{2} c^{2}-a^{2} c d+a^{2} d^{2}-a b^{2} d-a b d^{2}-a c^{2} d-a c d^{2}+b^{2} c^{2}+b^{2} c d+b c^{2} d+b c d^{2}\right)(b+c+d)= \\ +=a^{4} b+a^{4} c+a^{4} d+\left(b^{3} c^{2}+a^{2} d^{3}\right)-a^{2} c^{3}+\left(2 a^{3} d^{2}-b^{3} a^{2}+c^{3} b^{2}\right)+ \\ ++\left(b^{3} c d-c^{3} d a-d^{3} a b\right)+\left(2 a^{3} b d+c^{3} d b-d^{3} a c\right)+\left(2 a^{3} c d-b^{3} d a+d^{3} b c\right) \\ ++\left(-a^{2} b^{2} c+3 b^{2} c^{2} d-2 a c^{2} d^{2}\right)+\left(-2 a^{2} b^{2} d+2 b c^{2} d^{2}\right)+\left(-a^{2} b c^{2}-2 a^{2} c^{2} d-2 a b^{2} d^{2}+2 b^{2} c d^{2}\right)+ \\ ++\left(-2 a^{2} b c d-a b^{2} c d-a b c^{2} d-2 a b c d^{2}\right) \\ +\text { Introducing the notation } S_{x y z w}=\sum_{c y c} a^{x} b^{y} c^{z} d^{w}, \text { one can write } \\ +\sum_{c y c}(a-b)(a-c)(a+b+d)(a+c+d)(b+c+d)= \\ +=S_{4100}+S_{4010}+S_{4001}+2 S_{3200}-S_{3020}+2 S_{3002}-S_{3110}+2 S_{3101}+2 S_{3011}-3 S_{2120}-6 S_{2111}= \\ ++\left(S_{4100}+S_{4001}+\frac{1}{2} S_{3110}+\frac{1}{2} S_{3011}-3 S_{2120}\right)+ \\ ++\left(S_{4010}-S_{3020}-\frac{3}{2} S_{3110}+\frac{3}{2} S_{3011}+\frac{9}{16} S_{2210}+\frac{9}{16} S_{2201}-\frac{9}{8} S_{2111}\right)+ \\ ++\frac{9}{16}\left(S_{3200}-S_{2210}-S_{2201}+S_{3002}\right)+\frac{23}{16}\left(S_{3200}-2 S_{3101}+S_{3002}\right)+\frac{39}{8}\left(S_{3101}-S_{2111}\right), +\end{gathered} +$$ + +where the expressions + +$$ +\begin{gathered} +S_{4100}+S_{4001}+\frac{1}{2} S_{3110}+\frac{1}{2} S_{3011}-3 S_{2120}=\sum_{c y c}\left(a^{4} b+b c^{4}+\frac{1}{2} a^{3} b c+\frac{1}{2} a b c^{3}-3 a^{2} b c^{2}\right), \\ +S_{4010}-S_{3020}-\frac{3}{2} S_{3110}+\frac{3}{2} S_{3011}+\frac{9}{16} S_{2210}+\frac{9}{16} S_{2201}-\frac{9}{8} S_{2111}=\sum_{c y c} a^{2} c\left(a-c-\frac{3}{4} b+\frac{3}{4} d\right)^{2}, \\ +S_{3200}-S_{2210}-S_{2201}+S_{3002}=\sum_{c y c} b^{2}\left(a^{3}-a^{2} c-a c^{2}+c^{3}\right)=\sum_{c y c} b^{2}(a+c)(a-c)^{2}, \\ +S_{3200}-2 S_{3101}+S_{3002}=\sum_{c y c} a^{3}(b-d)^{2} \quad \text { and } \quad S_{3101}-S_{2111}=\frac{1}{3} \sum_{c y c} b d\left(2 a^{3}+c^{3}-3 a^{2} c\right) +\end{gathered} +$$ + +are all nonnegative. + +## Combinatorics + +C1. In the plane we consider rectangles whose sides are parallel to the coordinate axes and have positive length. Such a rectangle will be called a box. Two boxes intersect if they have a common point in their interior or on their boundary. + +Find the largest $n$ for which there exist $n$ boxes $B_{1}, \ldots, B_{n}$ such that $B_{i}$ and $B_{j}$ intersect if and only if $i \not \equiv j \pm 1(\bmod n)$. + +Solution. The maximum number of such boxes is 6 . One example is shown in the figure. + +![](https://cdn.mathpix.com/cropped/2024_04_17_e32cef0f3c5b05a0a6b1g-22.jpg?height=437&width=457&top_left_y=1135&top_left_x=777) + +Now we show that 6 is the maximum. Suppose that boxes $B_{1}, \ldots, B_{n}$ satisfy the condition. Let the closed intervals $I_{k}$ and $J_{k}$ be the projections of $B_{k}$ onto the $x$ - and $y$-axis, for $1 \leq k \leq n$. + +If $B_{i}$ and $B_{j}$ intersect, with a common point $(x, y)$, then $x \in I_{i} \cap I_{j}$ and $y \in J_{i} \cap J_{j}$. So the intersections $I_{i} \cap I_{j}$ and $J_{i} \cap J_{j}$ are nonempty. Conversely, if $x \in I_{i} \cap I_{j}$ and $y \in J_{i} \cap J_{j}$ for some real numbers $x, y$, then $(x, y)$ is a common point of $B_{i}$ and $B_{j}$. Putting it around, $B_{i}$ and $B_{j}$ are disjoint if and only if their projections on at least one coordinate axis are disjoint. + +For brevity we call two boxes or intervals adjacent if their indices differ by 1 modulo $n$, and nonadjacent otherwise. + +The adjacent boxes $B_{k}$ and $B_{k+1}$ do not intersect for each $k=1, \ldots, n$. Hence $\left(I_{k}, I_{k+1}\right)$ or $\left(J_{k}, J_{k+1}\right)$ is a pair of disjoint intervals, $1 \leq k \leq n$. So there are at least $n$ pairs of disjoint intervals among $\left(I_{1}, I_{2}\right), \ldots,\left(I_{n-1}, I_{n}\right),\left(I_{n}, I_{1}\right) ;\left(J_{1}, J_{2}\right), \ldots,\left(J_{n-1}, J_{n}\right),\left(J_{n}, J_{1}\right)$. + +Next, every two nonadjacent boxes intersect, hence their projections on both axes intersect, too. Then the claim below shows that at most 3 pairs among $\left(I_{1}, I_{2}\right), \ldots,\left(I_{n-1}, I_{n}\right),\left(I_{n}, I_{1}\right)$ are disjoint, and the same holds for $\left(J_{1}, J_{2}\right) \ldots,\left(J_{n-1}, J_{n}\right),\left(J_{n}, J_{1}\right)$. Consequently $n \leq 3+3=6$, as stated. Thus we are left with the claim and its justification. + +Claim. Let $\Delta_{1}, \Delta_{2}, \ldots, \Delta_{n}$ be intervals on a straight line such that every two nonadjacent intervals intersect. Then $\Delta_{k}$ and $\Delta_{k+1}$ are disjoint for at most three values of $k=1, \ldots, n$. + +Proof. Denote $\Delta_{k}=\left[a_{k}, b_{k}\right], 1 \leq k \leq n$. Let $\alpha=\max \left(a_{1}, \ldots, a_{n}\right)$ be the rightmost among the left endpoints of $\Delta_{1}, \ldots, \Delta_{n}$, and let $\beta=\min \left(b_{1}, \ldots, b_{n}\right)$ be the leftmost among their right endpoints. Assume that $\alpha=a_{2}$ without loss of generality. + +If $\alpha \leq \beta$ then $a_{i} \leq \alpha \leq \beta \leq b_{i}$ for all $i$. Every $\Delta_{i}$ contains $\alpha$, and thus no disjoint pair $\left(\Delta_{i}, \Delta_{i+1}\right)$ exists. + +If $\beta<\alpha$ then $\beta=b_{i}$ for some $i$ such that $a_{i}3$ and consider any nice permutation $\left(a_{1}, a_{2}, \ldots, a_{n}\right)$ of $\{1,2, \ldots, n\}$. Then $n-1$ must be a divisor of the number + +$$ +\begin{aligned} +& 2\left(a_{1}+a_{2}+\cdots+a_{n-1}\right)=2\left((1+2+\cdots+n)-a_{n}\right) \\ +& \quad=n(n+1)-2 a_{n}=(n+2)(n-1)+\left(2-2 a_{n}\right) . +\end{aligned} +$$ + +So $2 a_{n}-2$ must be divisible by $n-1$, hence equal to 0 or $n-1$ or $2 n-2$. This means that + +$$ +a_{n}=1 \quad \text { or } \quad a_{n}=\frac{n+1}{2} \quad \text { or } \quad a_{n}=n \text {. } +$$ + +Suppose that $a_{n}=(n+1) / 2$. Since the permutation is nice, taking $k=n-2$ we get that $n-2$ has to be a divisor of + +$$ +\begin{aligned} +2\left(a_{1}+a_{2}+\cdots+a_{n-2}\right) & =2\left((1+2+\cdots+n)-a_{n}-a_{n-1}\right) \\ +& =n(n+1)-(n+1)-2 a_{n-1}=(n+2)(n-2)+\left(3-2 a_{n-1}\right) . +\end{aligned} +$$ + +So $2 a_{n-1}-3$ should be divisible by $n-2$, hence equal to 0 or $n-2$ or $2 n-4$. Obviously 0 and $2 n-4$ are excluded because $2 a_{n-1}-3$ is odd. The remaining possibility $\left(2 a_{n-1}-3=n-2\right)$ leads to $a_{n-1}=(n+1) / 2=a_{n}$, which also cannot hold. This eliminates $(n+1) / 2$ as a possible value of $a_{n}$. Consequently $a_{n}=1$ or $a_{n}=n$. + +If $a_{n}=n$ then $\left(a_{1}, a_{2}, \ldots, a_{n-1}\right)$ is a nice permutation of $\{1,2, \ldots, n-1\}$. There are $F_{n-1}$ such permutations. Attaching $n$ to any one of them at the end creates a nice permutation of $\{1,2, \ldots, n\}$. + +If $a_{n}=1$ then $\left(a_{1}-1, a_{2}-1, \ldots, a_{n-1}-1\right)$ is a permutation of $\{1,2, \ldots, n-1\}$. It is also nice because the number + +$$ +2\left(\left(a_{1}-1\right)+\cdots+\left(a_{k}-1\right)\right)=2\left(a_{1}+\cdots+a_{k}\right)-2 k +$$ + +is divisible by $k$, for any $k \leq n-1$. And again, any one of the $F_{n-1}$ nice permutations $\left(b_{1}, b_{2}, \ldots, b_{n-1}\right)$ of $\{1,2, \ldots, n-1\}$ gives rise to a nice permutation of $\{1,2, \ldots, n\}$ whose last term is 1 , namely $\left(b_{1}+1, b_{2}+1, \ldots, b_{n-1}+1,1\right)$. + +The bijective correspondences established in both cases show that there are $F_{n-1}$ nice permutations of $\{1,2, \ldots, n\}$ with the last term 1 and also $F_{n-1}$ nice permutations of $\{1,2, \ldots, n\}$ with the last term $n$. Hence follows the recurrence $F_{n}=2 F_{n-1}$. With the base value $F_{3}=6$ this gives the outcome formula $F_{n}=3 \cdot 2^{n-2}$ for $n \geq 3$. + +C3. In the coordinate plane consider the set $S$ of all points with integer coordinates. For a positive integer $k$, two distinct points $A, B \in S$ will be called $k$-friends if there is a point $C \in S$ such that the area of the triangle $A B C$ is equal to $k$. A set $T \subset S$ will be called a $k$-clique if every two points in $T$ are $k$-friends. Find the least positive integer $k$ for which there exists a $k$-clique with more than 200 elements. + +Solution. To begin, let us describe those points $B \in S$ which are $k$-friends of the point $(0,0)$. By definition, $B=(u, v)$ satisfies this condition if and only if there is a point $C=(x, y) \in S$ such that $\frac{1}{2}|u y-v x|=k$. (This is a well-known formula expressing the area of triangle $A B C$ when $A$ is the origin.) + +To say that there exist integers $x, y$ for which $|u y-v x|=2 k$, is equivalent to saying that the greatest common divisor of $u$ and $v$ is also a divisor of $2 k$. Summing up, a point $B=(u, v) \in S$ is a $k$-friend of $(0,0)$ if and only if $\operatorname{gcd}(u, v)$ divides $2 k$. + +Translation by a vector with integer coordinates does not affect $k$-friendship; if two points are $k$-friends, so are their translates. It follows that two points $A, B \in S, A=(s, t), B=(u, v)$, are $k$-friends if and only if the point $(u-s, v-t)$ is a $k$-friend of $(0,0)$; i.e., if $\operatorname{gcd}(u-s, v-t) \mid 2 k$. + +Let $n$ be a positive integer which does not divide $2 k$. We claim that a $k$-clique cannot have more than $n^{2}$ elements. + +Indeed, all points $(x, y) \in S$ can be divided into $n^{2}$ classes determined by the remainders that $x$ and $y$ leave in division by $n$. If a set $T$ has more than $n^{2}$ elements, some two points $A, B \in T, A=(s, t), B=(u, v)$, necessarily fall into the same class. This means that $n \mid u-s$ and $n \mid v-t$. Hence $n \mid d$ where $d=\operatorname{gcd}(u-s, v-t)$. And since $n$ does not divide $2 k$, also $d$ does not divide $2 k$. Thus $A$ and $B$ are not $k$-friends and the set $T$ is not a $k$-clique. + +Now let $M(k)$ be the least positive integer which does not divide $2 k$. Write $M(k)=m$ for the moment and consider the set $T$ of all points $(x, y)$ with $0 \leq x, y200$. + +By the definition of $M(k), 2 k$ is divisible by the numbers $1,2, \ldots, M(k)-1$, but not by $M(k)$ itself. If $M(k)^{2}>200$ then $M(k) \geq 15$. Trying to hit $M(k)=15$ we get a contradiction immediately ( $2 k$ would have to be divisible by 3 and 5 , but not by 15 ). + +So let us try $M(k)=16$. Then $2 k$ is divisible by the numbers $1,2, \ldots, 15$, hence also by their least common multiple $L$, but not by 16 . And since $L$ is not a multiple of 16 , we infer that $k=L / 2$ is the least $k$ with $M(k)=16$. + +Finally, observe that if $M(k) \geq 17$ then $2 k$ must be divisible by the least common multiple of $1,2, \ldots, 16$, which is equal to $2 L$. Then $2 k \geq 2 L$, yielding $k>L / 2$. + +In conclusion, the least $k$ with the required property is equal to $L / 2=180180$. + +C4. Let $n$ and $k$ be fixed positive integers of the same parity, $k \geq n$. We are given $2 n$ lamps numbered 1 through $2 n$; each of them can be on or off. At the beginning all lamps are off. We consider sequences of $k$ steps. At each step one of the lamps is switched (from off to on or from on to off). + +Let $N$ be the number of $k$-step sequences ending in the state: lamps $1, \ldots, n$ on, lamps $n+1, \ldots, 2 n$ off. + +Let $M$ be the number of $k$-step sequences leading to the same state and not touching lamps $n+1, \ldots, 2 n$ at all. + +Find the ratio $N / M$. + +Solution. A sequence of $k$ switches ending in the state as described in the problem statement (lamps $1, \ldots, n$ on, lamps $n+1, \ldots, 2 n$ off) will be called an admissible process. If, moreover, the process does not touch the lamps $n+1, \ldots, 2 n$, it will be called restricted. So there are $N$ admissible processes, among which $M$ are restricted. + +In every admissible process, restricted or not, each one of the lamps $1, \ldots, n$ goes from off to on, so it is switched an odd number of times; and each one of the lamps $n+1, \ldots, 2 n$ goes from off to off, so it is switched an even number of times. + +Notice that $M>0$; i.e., restricted admissible processes do exist (it suffices to switch each one of the lamps $1, \ldots, n$ just once and then choose one of them and switch it $k-n$ times, which by hypothesis is an even number). + +Consider any restricted admissible process $\mathbf{p}$. Take any lamp $\ell, 1 \leq \ell \leq n$, and suppose that it was switched $k_{\ell}$ times. As noticed, $k_{\ell}$ must be odd. Select arbitrarily an even number of these $k_{\ell}$ switches and replace each of them by the switch of lamp $n+\ell$. This can be done in $2^{k_{\ell}-1}$ ways (because a $k_{\ell}$-element set has $2^{k_{\ell}-1}$ subsets of even cardinality). Notice that $k_{1}+\cdots+k_{n}=k$. + +These actions are independent, in the sense that the action involving lamp $\ell$ does not affect the action involving any other lamp. So there are $2^{k_{1}-1} \cdot 2^{k_{2}-1} \cdots 2^{k_{n}-1}=2^{k-n}$ ways of combining these actions. In any of these combinations, each one of the lamps $n+1, \ldots, 2 n$ gets switched an even number of times and each one of the lamps $1, \ldots, n$ remains switched an odd number of times, so the final state is the same as that resulting from the original process $\mathbf{p}$. + +This shows that every restricted admissible process $\mathbf{p}$ can be modified in $2^{k-n}$ ways, giving rise to $2^{k-n}$ distinct admissible processes (with all lamps allowed). + +Now we show that every admissible process $\mathbf{q}$ can be achieved in that way. Indeed, it is enough to replace every switch of a lamp with a label $\ell>n$ that occurs in $\mathbf{q}$ by the switch of the corresponding lamp $\ell-n$; in the resulting process $\mathbf{p}$ the lamps $n+1, \ldots, 2 n$ are not involved. + +Switches of each lamp with a label $\ell>n$ had occurred in $\mathbf{q}$ an even number of times. So the performed replacements have affected each lamp with a label $\ell \leq n$ also an even number of times; hence in the overall effect the final state of each lamp has remained the same. This means that the resulting process $\mathbf{p}$ is admissible - and clearly restricted, as the lamps $n+1, \ldots, 2 n$ are not involved in it any more. + +If we now take process $\mathbf{p}$ and reverse all these replacements, then we obtain process $\mathbf{q}$. These reversed replacements are nothing else than the modifications described in the foregoing paragraphs. + +Thus there is a one-to $-\left(2^{k-n}\right)$ correspondence between the $M$ restricted admissible processes and the total of $N$ admissible processes. Therefore $N / M=2^{k-n}$. + +C5. Let $S=\left\{x_{1}, x_{2}, \ldots, x_{k+\ell}\right\}$ be a $(k+\ell)$-element set of real numbers contained in the interval $[0,1] ; k$ and $\ell$ are positive integers. A $k$-element subset $A \subset S$ is called nice if + +$$ +\left|\frac{1}{k} \sum_{x_{i} \in A} x_{i}-\frac{1}{\ell} \sum_{x_{j} \in S \backslash A} x_{j}\right| \leq \frac{k+\ell}{2 k \ell} . +$$ + +Prove that the number of nice subsets is at least $\frac{2}{k+\ell}\left(\begin{array}{c}k+\ell \\ k\end{array}\right)$. + +Solution. For a $k$-element subset $A \subset S$, let $f(A)=\frac{1}{k} \sum_{x_{i} \in A} x_{i}-\frac{1}{\ell} \sum_{x_{j} \in S \backslash A} x_{j}$. Denote $\frac{k+\ell}{2 k \ell}=d$. By definition a subset $A$ is nice if $|f(A)| \leq d$. + +To each permutation $\left(y_{1}, y_{2}, \ldots, y_{k+\ell}\right)$ of the set $S=\left\{x_{1}, x_{2}, \ldots, x_{k+\ell}\right\}$ we assign $k+\ell$ subsets of $S$ with $k$ elements each, namely $A_{i}=\left\{y_{i}, y_{i+1}, \ldots, y_{i+k-1}\right\}, i=1,2, \ldots, k+\ell$. Indices are taken modulo $k+\ell$ here and henceforth. In other words, if $y_{1}, y_{2}, \ldots, y_{k+\ell}$ are arranged around a circle in this order, the sets in question are all possible blocks of $k$ consecutive elements. + +Claim. At least two nice sets are assigned to every permutation of $S$. + +Proof. Adjacent sets $A_{i}$ and $A_{i+1}$ differ only by the elements $y_{i}$ and $y_{i+k}, i=1, \ldots, k+\ell$. By the definition of $f$, and because $y_{i}, y_{i+k} \in[0,1]$, + +$$ +\left|f\left(A_{i+1}\right)-f\left(A_{i}\right)\right|=\left|\left(\frac{1}{k}+\frac{1}{\ell}\right)\left(y_{i+k}-y_{i}\right)\right| \leq \frac{1}{k}+\frac{1}{\ell}=2 d \text {. } +$$ + +Each element $y_{i} \in S$ belongs to exactly $k$ of the sets $A_{1}, \ldots, A_{k+\ell}$. Hence in $k$ of the expressions $f\left(A_{1}\right), \ldots, f\left(A_{k+\ell}\right)$ the coefficient of $y_{i}$ is $1 / k$; in the remaining $\ell$ expressions, its coefficient is $-1 / \ell$. So the contribution of $y_{i}$ to the sum of all $f\left(A_{i}\right)$ equals $k \cdot 1 / k-\ell \cdot 1 / \ell=0$. Since this holds for all $i$, it follows that $f\left(A_{1}\right)+\cdots+f\left(A_{k+\ell}\right)=0$. + +If $f\left(A_{p}\right)=\min f\left(A_{i}\right), f\left(A_{q}\right)=\max f\left(A_{i}\right)$, we obtain in particular $f\left(A_{p}\right) \leq 0, f\left(A_{q}\right) \geq 0$. Let $pq$ is analogous; and the claim is true for $p=q$ as $f\left(A_{i}\right)=0$ for all $i$ ). + +We are ready to prove that at least two of the sets $A_{1}, \ldots, A_{k+\ell}$ are nice. The interval $[-d, d]$ has length $2 d$, and we saw that adjacent numbers in the circular arrangement $f\left(A_{1}\right), \ldots, f\left(A_{k+\ell}\right)$ differ by at most $2 d$. Suppose that $f\left(A_{p}\right)<-d$ and $f\left(A_{q}\right)>d$. Then one of the numbers $f\left(A_{p+1}\right), \ldots, f\left(A_{q-1}\right)$ lies in $[-d, d]$, and also one of the numbers $f\left(A_{q+1}\right), \ldots, f\left(A_{p-1}\right)$ lies there. Consequently, one of the sets $A_{p+1}, \ldots, A_{q-1}$ is nice, as well as one of the sets $A_{q+1}, \ldots, A_{p-1}$. If $-d \leq f\left(A_{p}\right)$ and $f\left(A_{q}\right) \leq d$ then $A_{p}$ and $A_{q}$ are nice. + +Let now $f\left(A_{p}\right)<-d$ and $f\left(A_{q}\right) \leq d$. Then $f\left(A_{p}\right)+f\left(A_{q}\right)<0$, and since $\sum f\left(A_{i}\right)=0$, there is an $r \neq q$ such that $f\left(A_{r}\right)>0$. We have $02\left(u_{m-1}-u_{1}\right)$ for all $m \geq 3$. + +Indeed, assume that $u_{m}-u_{1} \leq 2\left(u_{m-1}-u_{1}\right)$ holds for some $m \geq 3$. This inequality can be written as $2\left(u_{m}-u_{m-1}\right) \leq u_{m}-u_{1}$. Take the unique $k$ such that $2^{k} \leq u_{m}-u_{1}<2^{k+1}$. Then $2\left(u_{m}-u_{m-1}\right) \leq u_{m}-u_{1}<2^{k+1}$ yields $u_{m}-u_{m-1}<2^{k}$. However the elements $z=u_{m}, x=u_{1}$, $y=u_{m-1}$ of $S_{a}$ then satisfy $z-y<2^{k}$ and $z-x \geq 2^{k}$, so that $z=u_{m}$ is $k$-good to $S_{a}$. + +Thus each term of the sequence $u_{2}-u_{1}, u_{3}-u_{1}, \ldots, u_{p}-u_{1}$ is more than twice the previous one. Hence $u_{p}-u_{1}>2^{p-1}\left(u_{2}-u_{1}\right) \geq 2^{p-1}$. But $u_{p} \in\left\{1,2,3, \ldots, 2^{n+1}\right\}$, so that $u_{p} \leq 2^{n+1}$. This yields $p-1 \leq n$, i. e. $p \leq n+1$. + +In other words, each set $S_{a}$ contains at most $n+1$ elements that are not good to it. + +To summarize the conclusions, mark with red all elements in the sets $S_{a}$ that are good to the respective set, and with blue the ones that are not good. Then the total number of red elements, counting multiplicities, is at most $n \cdot 2^{n+1}$ (each $z \in A$ can be marked red in at most $n$ sets). The total number of blue elements is at most $(n+1) 2^{n}$ (each set $S_{a}$ contains at most $n+1$ blue elements). Therefore the sum of cardinalities of $S_{1}, S_{2}, \ldots, S_{2^{n}}$ does not exceed $(3 n+1) 2^{n}$. By averaging, the smallest set has at most $3 n+1$ elements. + +Solution 2. We show that one of the sets $S_{a}$ has at most $2 n+1$ elements. In the sequel $|\cdot|$ denotes the cardinality of a (finite) set. + +Claim. For $n \geq 2$, suppose that $k$ subsets $S_{1}, \ldots, S_{k}$ of $\left\{1,2, \ldots, 2^{n}\right\}$ (not necessarily different) satisfy the condition of the problem. Then + +$$ +\sum_{i=1}^{k}\left(\left|S_{i}\right|-n\right) \leq(2 n-1) 2^{n-2} +$$ + +Proof. Observe that if the sets $S_{i}(1 \leq i \leq k)$ satisfy the condition then so do their arbitrary subsets $T_{i}(1 \leq i \leq k)$. The condition also holds for the sets $t+S_{i}=\left\{t+x \mid x \in S_{i}\right\}$ where $t$ is arbitrary. + +Note also that a set may occur more than once among $S_{1}, \ldots, S_{k}$ only if its cardinality is less than 3, in which case its contribution to the sum $\sum_{i=1}^{k}\left(\left|S_{i}\right|-n\right)$ is nonpositive (as $n \geq 2$ ). + +The proof is by induction on $n$. In the base case $n=2$ we have subsets $S_{i}$ of $\{1,2,3,4\}$. Only the ones of cardinality 3 and 4 need to be considered by the remark above; each one of +them occurs at most once among $S_{1}, \ldots, S_{k}$. If $S_{i}=\{1,2,3,4\}$ for some $i$ then no $S_{j}$ is a 3 -element subset in view of the condition, hence $\sum_{i=1}^{k}\left(\left|S_{i}\right|-2\right) \leq 2$. By the condition again, it is impossible that $S_{i}=\{1,3,4\}$ and $S_{j}=\{2,3,4\}$ for some $i, j$. So if $\left|S_{i}\right| \leq 3$ for all $i$ then at most 3 summands $\left|S_{i}\right|-2$ are positive, corresponding to 3 -element subsets. This implies $\sum_{i=1}^{k}\left(\left|S_{i}\right|-2\right) \leq 3$, therefore the conclusion is true for $n=2$. + +Suppose that the claim holds for some $n \geq 2$, and let the sets $S_{1}, \ldots, S_{k} \subseteq\left\{1,2, \ldots, 2^{n+1}\right\}$ satisfy the given property. Denote $U_{i}=S_{i} \cap\left\{1,2, \ldots, 2^{n}\right\}, V_{i}=S_{i} \cap\left\{2^{n}+1, \ldots, 2^{n+1}\right\}$. Let + +$$ +I=\left\{i|1 \leq i \leq k,| U_{i} \mid \neq 0\right\}, \quad J=\{1, \ldots, k\} \backslash I +$$ + +The sets $S_{j}$ with $j \in J$ are all contained in $\left\{2^{n}+1, \ldots, 2^{n+1}\right\}$, so the induction hypothesis applies to their translates $-2^{n}+S_{j}$ which have the same cardinalities. Consequently, this gives $\sum_{j \in J}\left(\left|S_{j}\right|-n\right) \leq(2 n-1) 2^{n-2}$, so that + +$$ +\sum_{j \in J}\left(\left|S_{j}\right|-(n+1)\right) \leq \sum_{j \in J}\left(\left|S_{j}\right|-n\right) \leq(2 n-1) 2^{n-2} +$$ + +For $i \in I$, denote by $v_{i}$ the least element of $V_{i}$. Observe that if $V_{a}$ and $V_{b}$ intersect, with $a2^{n}$, which implies $z=v_{a}$. + +It follows that if the element $v_{i}$ is removed from each $V_{i}$, a family of pairwise disjoint sets $W_{i}=V_{i} \backslash\left\{v_{i}\right\}$ is obtained, $i \in I$ (we assume $W_{i}=\emptyset$ if $V_{i}=\emptyset$ ). As $W_{i} \subseteq\left\{2^{n}+1, \ldots, 2^{n+1}\right\}$ for all $i$, we infer that $\sum_{i \in I}\left|W_{i}\right| \leq 2^{n}$. Therefore $\sum_{i \in I}\left(\left|V_{i}\right|-1\right) \leq \sum_{i \in I}\left|W_{i}\right| \leq 2^{n}$. + +On the other hand, the induction hypothesis applies directly to the sets $U_{i}, i \in I$, so that $\sum_{i \in \mathcal{I}}\left(\left|U_{i}\right|-n\right) \leq(2 n-1) 2^{n-2}$. In summary, + +$$ +\sum_{i \in I}\left(\left|S_{i}\right|-(n+1)\right)=\sum_{i \in I}\left(\left|U_{i}\right|-n\right)+\sum_{i \in I}\left(\left|V_{i}\right|-1\right) \leq(2 n-1) 2^{n-2}+2^{n} +$$ + +The estimates (1) and (2) are sufficient to complete the inductive step: + +$$ +\begin{aligned} +\sum_{i=1}^{k}\left(\left|S_{i}\right|-(n+1)\right) & =\sum_{i \in I}\left(\left|S_{i}\right|-(n+1)\right)+\sum_{j \in J}\left(\left|S_{j}\right|-(n+1)\right) \\ +& \leq(2 n-1) 2^{n-2}+2^{n}+(2 n-1) 2^{n-2}=(2 n+1) 2^{n-1} +\end{aligned} +$$ + +Returning to the problem, consider $k=2^{n}$ subsets $S_{1}, S_{2}, \ldots, S_{2^{n}}$ of $\left\{1,2,3, \ldots, 2^{n+1}\right\}$. If they satisfy the given condition, the claim implies $\sum_{i=1}^{2^{n}}\left(\left|S_{i}\right|-(n+1)\right) \leq(2 n+1) 2^{n-1}$. By averaging again, we see that the smallest set has at most $2 n+1$ elements. + +Comment. It can happen that each set $S_{i}$ has cardinality at least $n+1$. Here is an example by the proposer. + +For $i=1, \ldots, 2^{n}$, let $S_{i}=\left\{i+2^{k} \mid 0 \leq k \leq n\right\}$. Then $\left|S_{i}\right|=n+1$ for all $i$. Suppose that there exist $al$. Since $y \in S_{a}$ and $y180^{\circ}, +\end{gathered} +$$ + +point $P$ lies in triangle $C D Q$, and hence in $A B C D$. The proof is complete. + +![](https://cdn.mathpix.com/cropped/2024_04_17_e32cef0f3c5b05a0a6b1g-40.jpg?height=757&width=1008&top_left_y=1489&top_left_x=501) + +G7. Let $A B C D$ be a convex quadrilateral with $A B \neq B C$. Denote by $\omega_{1}$ and $\omega_{2}$ the incircles of triangles $A B C$ and $A D C$. Suppose that there exists a circle $\omega$ inscribed in angle $A B C$, tangent to the extensions of line segments $A D$ and $C D$. Prove that the common external tangents of $\omega_{1}$ and $\omega_{2}$ intersect on $\omega$. + +Solution. The proof below is based on two known facts. + +Lemma 1. Given a convex quadrilateral $A B C D$, suppose that there exists a circle which is inscribed in angle $A B C$ and tangent to the extensions of line segments $A D$ and $C D$. Then $A B+A D=C B+C D$. + +Proof. The circle in question is tangent to each of the lines $A B, B C, C D, D A$, and the respective points of tangency $K, L, M, N$ are located as with circle $\omega$ in the figure. Then + +$$ +A B+A D=(B K-A K)+(A N-D N), \quad C B+C D=(B L-C L)+(C M-D M) . +$$ + +Also $B K=B L, D N=D M, A K=A N, C L=C M$ by equalities of tangents. It follows that $A B+A D=C B+C D$. + +![](https://cdn.mathpix.com/cropped/2024_04_17_e32cef0f3c5b05a0a6b1g-41.jpg?height=1068&width=1079&top_left_y=1156&top_left_x=523) + +For brevity, in the sequel we write "excircle $A C$ " for the excircle of a triangle with side $A C$ which is tangent to line segment $A C$ and the extensions of the other two sides. + +Lemma 2. The incircle of triangle $A B C$ is tangent to its side $A C$ at $P$. Let $P P^{\prime}$ be the diameter of the incircle through $P$, and let line $B P^{\prime}$ intersect $A C$ at $Q$. Then $Q$ is the point of tangency of side $A C$ and excircle $A C$. + +Proof. Let the tangent at $P^{\prime}$ to the incircle $\omega_{1}$ meet $B A$ and $B C$ at $A^{\prime}$ and $C^{\prime}$. Now $\omega_{1}$ is the excircle $A^{\prime} C^{\prime}$ of triangle $A^{\prime} B C^{\prime}$, and it touches side $A^{\prime} C^{\prime}$ at $P^{\prime}$. Since $A^{\prime} C^{\prime} \| A C$, the homothety with centre $B$ and ratio $B Q / B P^{\prime}$ takes $\omega_{1}$ to the excircle $A C$ of triangle $A B C$. Because this homothety takes $P^{\prime}$ to $Q$, the lemma follows. + +Recall also that if the incircle of a triangle touches its side $A C$ at $P$, then the tangency point $Q$ of the same side and excircle $A C$ is the unique point on line segment $A C$ such that $A P=C Q$. + +We pass on to the main proof. Let $\omega_{1}$ and $\omega_{2}$ touch $A C$ at $P$ and $Q$, respectively; then $A P=(A C+A B-B C) / 2, C Q=(C A+C D-A D) / 2$. Since $A B-B C=C D-A D$ by Lemma 1, we obtain $A P=C Q$. It follows that in triangle $A B C$ side $A C$ and excircle $A C$ are tangent at $Q$. Likewise, in triangle $A D C$ side $A C$ and excircle $A C$ are tangent at $P$. Note that $P \neq Q$ as $A B \neq B C$. + +Let $P P^{\prime}$ and $Q Q^{\prime}$ be the diameters perpendicular to $A C$ of $\omega_{1}$ and $\omega_{2}$, respectively. Then Lemma 2 shows that points $B, P^{\prime}$ and $Q$ are collinear, and so are points $D, Q^{\prime}$ and $P$. + +Consider the diameter of $\omega$ perpendicular to $A C$ and denote by $T$ its endpoint that is closer to $A C$. The homothety with centre $B$ and ratio $B T / B P^{\prime}$ takes $\omega_{1}$ to $\omega$. Hence $B, P^{\prime}$ and $T$ are collinear. Similarly, $D, Q^{\prime}$ and $T$ are collinear since the homothety with centre $D$ and ratio $-D T / D Q^{\prime}$ takes $\omega_{2}$ to $\omega$. + +We infer that points $T, P^{\prime}$ and $Q$ are collinear, as well as $T, Q^{\prime}$ and $P$. Since $P P^{\prime} \| Q Q^{\prime}$, line segments $P P^{\prime}$ and $Q Q^{\prime}$ are then homothetic with centre $T$. The same holds true for circles $\omega_{1}$ and $\omega_{2}$ because they have $P P^{\prime}$ and $Q Q^{\prime}$ as diameters. Moreover, it is immediate that $T$ lies on the same side of line $P P^{\prime}$ as $Q$ and $Q^{\prime}$, hence the ratio of homothety is positive. In particular $\omega_{1}$ and $\omega_{2}$ are not congruent. + +In summary, $T$ is the centre of a homothety with positive ratio that takes circle $\omega_{1}$ to circle $\omega_{2}$. This completes the solution, since the only point with the mentioned property is the intersection of the the common external tangents of $\omega_{1}$ and $\omega_{2}$. + +## Number Theory + +N1. Let $n$ be a positive integer and let $p$ be a prime number. Prove that if $a, b, c$ are integers (not necessarily positive) satisfying the equations + +$$ +a^{n}+p b=b^{n}+p c=c^{n}+p a, +$$ + +then $a=b=c$. + +Solution 1. If two of $a, b, c$ are equal, it is immediate that all the three are equal. So we may assume that $a \neq b \neq c \neq a$. Subtracting the equations we get $a^{n}-b^{n}=-p(b-c)$ and two cyclic copies of this equation, which upon multiplication yield + +$$ +\frac{a^{n}-b^{n}}{a-b} \cdot \frac{b^{n}-c^{n}}{b-c} \cdot \frac{c^{n}-a^{n}}{c-a}=-p^{3} . +$$ + +If $n$ is odd then the differences $a^{n}-b^{n}$ and $a-b$ have the same sign and the product on the left is positive, while $-p^{3}$ is negative. So $n$ must be even. + +Let $d$ be the greatest common divisor of the three differences $a-b, b-c, c-a$, so that $a-b=d u, b-c=d v, \quad c-a=d w ; \operatorname{gcd}(u, v, w)=1, u+v+w=0$. + +From $a^{n}-b^{n}=-p(b-c)$ we see that $(a-b) \mid p(b-c)$, i.e., $u \mid p v$; and cyclically $v|p w, w| p u$. As $\operatorname{gcd}(u, v, w)=1$ and $u+v+w=0$, at most one of $u, v, w$ can be divisible by $p$. Supposing that the prime $p$ does not divide any one of them, we get $u|v, v| w, w \mid u$, whence $|u|=|v|=|w|=1$; but this quarrels with $u+v+w=0$. + +Thus $p$ must divide exactly one of these numbers. Let e.g. $p \mid u$ and write $u=p u_{1}$. Now we obtain, similarly as before, $u_{1}|v, v| w, w \mid u_{1}$ so that $\left|u_{1}\right|=|v|=|w|=1$. The equation $p u_{1}+v+w=0$ forces that the prime $p$ must be even; i.e. $p=2$. Hence $v+w=-2 u_{1}= \pm 2$, implying $v=w(= \pm 1)$ and $u=-2 v$. Consequently $a-b=-2(b-c)$. + +Knowing that $n$ is even, say $n=2 k$, we rewrite the equation $a^{n}-b^{n}=-p(b-c)$ with $p=2$ in the form + +$$ +\left(a^{k}+b^{k}\right)\left(a^{k}-b^{k}\right)=-2(b-c)=a-b . +$$ + +The second factor on the left is divisible by $a-b$, so the first factor $\left(a^{k}+b^{k}\right)$ must be $\pm 1$. Then exactly one of $a$ and $b$ must be odd; yet $a-b=-2(b-c)$ is even. Contradiction ends the proof. + +Solution 2. The beginning is as in the first solution. Assuming that $a, b, c$ are not all equal, hence are all distinct, we derive equation (1) with the conclusion that $n$ is even. Write $n=2 k$. + +Suppose that $p$ is odd. Then the integer + +$$ +\frac{a^{n}-b^{n}}{a-b}=a^{n-1}+a^{n-2} b+\cdots+b^{n-1} +$$ + +which is a factor in (1), must be odd as well. This sum of $n=2 k$ summands is odd only if $a$ and $b$ have different parities. The same conclusion holding for $b, c$ and for $c$, $a$, we get that $a, b, c, a$ alternate in their parities, which is clearly impossible. + +Thus $p=2$. The original system shows that $a, b, c$ must be of the same parity. So we may divide (1) by $p^{3}$, i.e. $2^{3}$, to obtain the following product of six integer factors: + +$$ +\frac{a^{k}+b^{k}}{2} \cdot \frac{a^{k}-b^{k}}{a-b} \cdot \frac{b^{k}+c^{k}}{2} \cdot \frac{b^{k}-c^{k}}{b-c} \cdot \frac{c^{k}+a^{k}}{2} \cdot \frac{c^{k}-a^{k}}{c-a}=-1 . +$$ + +Each one of the factors must be equal to $\pm 1$. In particular, $a^{k}+b^{k}= \pm 2$. If $k$ is even, this becomes $a^{k}+b^{k}=2$ and yields $|a|=|b|=1$, whence $a^{k}-b^{k}=0$, contradicting (2). + +Let now $k$ be odd. Then the sum $a^{k}+b^{k}$, with value $\pm 2$, has $a+b$ as a factor. Since $a$ and $b$ are of the same parity, this means that $a+b= \pm 2$; and cyclically, $b+c= \pm 2, c+a= \pm 2$. In some two of these equations the signs must coincide, hence some two of $a, b, c$ are equal. This is the desired contradiction. + +Comment. Having arrived at the equation (1) one is tempted to write down all possible decompositions of $-p^{3}$ (cube of a prime) into a product of three integers. This leads to cumbersome examination of many cases, some of which are unpleasant to handle. One may do that just for $p=2$, having earlier in some way eliminated odd primes from consideration. + +However, the second solution shows that the condition of $p$ being a prime is far too strong. What is actually being used in that solution, is that $p$ is either a positive odd integer or $p=2$. + +N2. Let $a_{1}, a_{2}, \ldots, a_{n}$ be distinct positive integers, $n \geq 3$. Prove that there exist distinct indices $i$ and $j$ such that $a_{i}+a_{j}$ does not divide any of the numbers $3 a_{1}, 3 a_{2}, \ldots, 3 a_{n}$. + +Solution. Without loss of generality, let $0a_{n-1}\right)$. Thus $3 a_{n-1}=a_{n}+a_{n-1}$, i. e. $a_{n}=2 a_{n-1}$. + +Similarly, if $j=n$ then $3 a_{n}=k\left(a_{n}+a_{n-1}\right)$ for some integer $k$, and only $k=2$ is possible. Hence $a_{n}=2 a_{n-1}$ holds true in both cases remaining, $j=n-1$ and $j=n$. + +Now $a_{n}=2 a_{n-1}$ implies that the sum $a_{n-1}+a_{1}$ is strictly between $a_{n} / 2$ and $a_{n}$. But $a_{n-1}$ and $a_{1}$ are distinct as $n \geq 3$, so it follows from the above that $a_{n-1}+a_{1}$ divides $a_{n}$. This provides the desired contradiction. + +N3. Let $a_{0}, a_{1}, a_{2}, \ldots$ be a sequence of positive integers such that the greatest common divisor of any two consecutive terms is greater than the preceding term; in symbols, $\operatorname{gcd}\left(a_{i}, a_{i+1}\right)>a_{i-1}$. Prove that $a_{n} \geq 2^{n}$ for all $n \geq 0$. + +Solution. Since $a_{i} \geq \operatorname{gcd}\left(a_{i}, a_{i+1}\right)>a_{i-1}$, the sequence is strictly increasing. In particular $a_{0} \geq 1, a_{1} \geq 2$. For each $i \geq 1$ we also have $a_{i+1}-a_{i} \geq \operatorname{gcd}\left(a_{i}, a_{i+1}\right)>a_{i-1}$, and consequently $a_{i+1} \geq a_{i}+a_{i-1}+1$. Hence $a_{2} \geq 4$ and $a_{3} \geq 7$. The equality $a_{3}=7$ would force equalities in the previous estimates, leading to $\operatorname{gcd}\left(a_{2}, a_{3}\right)=\operatorname{gcd}(4,7)>a_{1}=2$, which is false. Thus $a_{3} \geq 8$; the result is valid for $n=0,1,2,3$. These are the base cases for a proof by induction. + +Take an $n \geq 3$ and assume that $a_{i} \geq 2^{i}$ for $i=0,1, \ldots, n$. We must show that $a_{n+1} \geq 2^{n+1}$. Let $\operatorname{gcd}\left(a_{n}, a_{n+1}\right)=d$. We know that $d>a_{n-1}$. The induction claim is reached immediately in the following cases: + +$$ +\begin{array}{ll} +\text { if } \quad a_{n+1} \geq 4 d & \text { then } a_{n+1}>4 a_{n-1} \geq 4 \cdot 2^{n-1}=2^{n+1} \\ +\text { if } \quad a_{n} \geq 3 d & \text { then } a_{n+1} \geq a_{n}+d \geq 4 d>4 a_{n-1} \geq 4 \cdot 2^{n-1}=2^{n+1} \\ +\text { if } \quad a_{n}=d & \text { then } a_{n+1} \geq a_{n}+d=2 a_{n} \geq 2 \cdot 2^{n}=2^{n+1} +\end{array} +$$ + +The only remaining possibility is that $a_{n}=2 d$ and $a_{n+1}=3 d$, which we assume for the sequel. So $a_{n+1}=\frac{3}{2} a_{n}$. + +Let now $\operatorname{gcd}\left(a_{n-1}, a_{n}\right)=d^{\prime}$; then $d^{\prime}>a_{n-2}$. Write $a_{n}=m d^{\prime} \quad(m$ an integer $)$. Keeping in mind that $d^{\prime} \leq a_{n-1}9 a_{n-2} \geq 9 \cdot 2^{n-2}>2^{n+1} \\ +& \text { if } 3 \leq m \leq 4 \text { then } a_{n-1}<\frac{1}{2} \cdot 4 d^{\prime} \text {, and hence } a_{n-1}=d^{\prime} \\ +& \qquad a_{n+1}=\frac{3}{2} m a_{n-1} \geq \frac{3}{2} \cdot 3 a_{n-1} \geq \frac{9}{2} \cdot 2^{n-1}>2^{n+1} . +\end{aligned} +$$ + +So we are left with the case $m=5$, which means that $a_{n}=5 d^{\prime}, a_{n+1}=\frac{15}{2} d^{\prime}, a_{n-1}a_{n-3}$. Because $d^{\prime \prime}$ is a divisor of $a_{n-1}$, hence also of $2 d^{\prime}$, we may write $2 d^{\prime}=m^{\prime} d^{\prime \prime}$ ( $m^{\prime}$ an integer). Since $d^{\prime \prime} \leq a_{n-2}\frac{75}{4} a_{n-3} \geq \frac{75}{4} \cdot 2^{n-3}>2^{n+1} \\ +& \text { if } 3 \leq m^{\prime} \leq 4 \text { then } a_{n-2}<\frac{1}{2} \cdot 4 d^{\prime \prime} \text {, and hence } a_{n-2}=d^{\prime \prime} \\ +& \qquad a_{n+1}=\frac{15}{4} m^{\prime} a_{n-2} \geq \frac{15}{4} \cdot 3 a_{n-2} \geq \frac{45}{4} \cdot 2^{n-2}>2^{n+1} . +\end{aligned} +$$ + +Both of them have produced the induction claim. But now there are no cases left. Induction is complete; the inequality $a_{n} \geq 2^{n}$ holds for all $n$. + +N4. Let $n$ be a positive integer. Show that the numbers + +$$ +\left(\begin{array}{c} +2^{n}-1 \\ +0 +\end{array}\right), \quad\left(\begin{array}{c} +2^{n}-1 \\ +1 +\end{array}\right), \quad\left(\begin{array}{c} +2^{n}-1 \\ +2 +\end{array}\right), \quad \ldots, \quad\left(\begin{array}{c} +2^{n}-1 \\ +2^{n-1}-1 +\end{array}\right) +$$ + +are congruent modulo $2^{n}$ to $1,3,5, \ldots, 2^{n}-1$ in some order. + +Solution 1. It is well-known that all these numbers are odd. So the assertion that their remainders $\left(\bmod 2^{n}\right)$ make up a permutation of $\left\{1,3, \ldots, 2^{n}-1\right\}$ is equivalent just to saying that these remainders are all distinct. We begin by showing that + +$$ +\left(\begin{array}{c} +2^{n}-1 \\ +2 k +\end{array}\right)+\left(\begin{array}{c} +2^{n}-1 \\ +2 k+1 +\end{array}\right) \equiv 0 \quad\left(\bmod 2^{n}\right) \quad \text { and } \quad\left(\begin{array}{c} +2^{n}-1 \\ +2 k +\end{array}\right) \equiv(-1)^{k}\left(\begin{array}{c} +2^{n-1}-1 \\ +k +\end{array}\right) \quad\left(\bmod 2^{n}\right) +$$ + +The first relation is immediate, as the sum on the left is equal to $\left(\begin{array}{c}2^{n} \\ 2 k+1\end{array}\right)=\frac{2^{n}}{2 k+1}\left(\begin{array}{c}2^{n}-1 \\ 2 k\end{array}\right)$, hence is divisible by $2^{n}$. The second relation: + +$$ +\left(\begin{array}{c} +2^{n}-1 \\ +2 k +\end{array}\right)=\prod_{j=1}^{2 k} \frac{2^{n}-j}{j}=\prod_{i=1}^{k} \frac{2^{n}-(2 i-1)}{2 i-1} \cdot \prod_{i=1}^{k} \frac{2^{n-1}-i}{i} \equiv(-1)^{k}\left(\begin{array}{c} +2^{n-1}-1 \\ +k +\end{array}\right) \quad\left(\bmod 2^{n}\right) . +$$ + +This prepares ground for a proof of the required result by induction on $n$. The base case $n=1$ is obvious. Assume the assertion is true for $n-1$ and pass to $n$, denoting $a_{k}=\left(\begin{array}{c}2^{n-1}-1 \\ k\end{array}\right)$, $b_{m}=\left(\begin{array}{c}2^{n}-1 \\ m\end{array}\right)$. The induction hypothesis is that all the numbers $a_{k}\left(0 \leq k<2^{n-2}\right)$ are distinct $\left(\bmod 2^{n-1}\right)$; the claim is that all the numbers $b_{m}\left(0 \leq m<2^{n-1}\right)$ are distinct $\left(\bmod 2^{n}\right)$. + +The congruence relations (1) are restated as + +$$ +b_{2 k} \equiv(-1)^{k} a_{k} \equiv-b_{2 k+1} \quad\left(\bmod 2^{n}\right) +$$ + +Shifting the exponent in the first relation of (1) from $n$ to $n-1$ we also have the congruence $a_{2 i+1} \equiv-a_{2 i}\left(\bmod 2^{n-1}\right)$. We hence conclude: + +If, for some $j, k<2^{n-2}, a_{k} \equiv-a_{j}\left(\bmod 2^{n-1}\right)$, then $\{j, k\}=\{2 i, 2 i+1\}$ for some $i$. + +This is so because in the sequence $\left(a_{k}: k<2^{n-2}\right)$ each term $a_{j}$ is complemented to $0\left(\bmod 2^{n-1}\right)$ by only one other term $a_{k}$, according to the induction hypothesis. + +From (2) we see that $b_{4 i} \equiv a_{2 i}$ and $b_{4 i+3} \equiv a_{2 i+1}\left(\bmod 2^{n}\right)$. Let + +$$ +M=\left\{m: 0 \leq m<2^{n-1}, m \equiv 0 \text { or } 3(\bmod 4)\right\}, \quad L=\left\{l: 0 \leq l<2^{n-1}, l \equiv 1 \text { or } 2(\bmod 4)\right\} +$$ + +The last two congruences take on the unified form + +$$ +b_{m} \equiv a_{\lfloor m / 2\rfloor} \quad\left(\bmod 2^{n}\right) \quad \text { for all } \quad m \in M +$$ + +Thus all the numbers $b_{m}$ for $m \in M$ are distinct $\left(\bmod 2^{n}\right)$ because so are the numbers $a_{k}$ (they are distinct $\left(\bmod 2^{n-1}\right)$, hence also $\left(\bmod 2^{n}\right)$ ). + +Every $l \in L$ is paired with a unique $m \in M$ into a pair of the form $\{2 k, 2 k+1\}$. So (2) implies that also all the $b_{l}$ for $l \in L$ are distinct $\left(\bmod 2^{n}\right)$. It remains to eliminate the possibility that $b_{m} \equiv b_{l}\left(\bmod 2^{n}\right)$ for some $m \in M, l \in L$. + +Suppose that such a situation occurs. Let $m^{\prime} \in M$ be such that $\left\{m^{\prime}, l\right\}$ is a pair of the form $\{2 k, 2 k+1\}$, so that $($ see $(2)) b_{m^{\prime}} \equiv-b_{l}\left(\bmod 2^{n}\right)$. Hence $b_{m^{\prime}} \equiv-b_{m}\left(\bmod 2^{n}\right)$. Since both $m^{\prime}$ and $m$ are in $M$, we have by (4) $b_{m^{\prime}} \equiv a_{j}, b_{m} \equiv a_{k}\left(\bmod 2^{n}\right)$ for $j=\left\lfloor m^{\prime} / 2\right\rfloor, k=\lfloor m / 2\rfloor$. + +Then $a_{j} \equiv-a_{k}\left(\bmod 2^{n}\right)$. Thus, according to $(3), j=2 i, k=2 i+1$ for some $i$ (or vice versa). The equality $a_{2 i+1} \equiv-a_{2 i}\left(\bmod 2^{n}\right)$ now means that $\left(\begin{array}{c}2^{n-1}-1 \\ 2 i\end{array}\right)+\left(\begin{array}{c}2^{n-1}-1 \\ 2 i+1\end{array}\right) \equiv 0\left(\bmod 2^{n}\right)$. However, the sum on the left is equal to $\left(\begin{array}{c}2^{n-1} \\ 2 i+1\end{array}\right)$. A number of this form cannot be divisible by $2^{n}$. This is a contradiction which concludes the induction step and proves the result. + +Solution 2. We again proceed by induction, writing for brevity $N=2^{n-1}$ and keeping notation $a_{k}=\left(\begin{array}{c}N-1 \\ k\end{array}\right), b_{m}=\left(\begin{array}{c}2 N-1 \\ m\end{array}\right)$. Assume that the result holds for the sequence $\left(a_{0}, a_{1}, a_{2}, \ldots, a_{N / 2-1}\right)$. In view of the symmetry $a_{N-1-k}=a_{k}$ this sequence is a permutation of $\left(a_{0}, a_{2}, a_{4}, \ldots, a_{N-2}\right)$. So the induction hypothesis says that this latter sequence, taken $(\bmod N)$, is a permutation of $(1,3,5, \ldots, N-1)$. Similarly, the induction claim is that $\left(b_{0}, b_{2}, b_{4}, \ldots, b_{2 N-2}\right)$, taken $(\bmod 2 N)$, is a permutation of $(1,3,5, \ldots, 2 N-1)$. + +In place of the congruence relations (2) we now use the following ones, + +$$ +b_{4 i} \equiv a_{2 i} \quad(\bmod N) \quad \text { and } \quad b_{4 i+2} \equiv b_{4 i}+N \quad(\bmod 2 N) . +$$ + +Given this, the conclusion is immediate: the first formula of (5) together with the induction hypothesis tells us that $\left(b_{0}, b_{4}, b_{8}, \ldots, b_{2 N-4}\right)(\bmod N)$ is a permutation of $(1,3,5, \ldots, N-1)$. Then the second formula of $(5)$ shows that $\left(b_{2}, b_{6}, b_{10}, \ldots, b_{2 N-2}\right)(\bmod N)$ is exactly the same permutation; moreover, this formula distinguishes $(\bmod 2 N)$ each $b_{4 i}$ from $b_{4 i+2}$. + +Consequently, these two sequences combined represent $(\bmod 2 N)$ a permutation of the sequence $(1,3,5, \ldots, N-1, N+1, N+3, N+5, \ldots, N+N-1)$, and this is precisely the induction claim. + +Now we prove formulas (5); we begin with the second one. Since $b_{m+1}=b_{m} \cdot \frac{2 N-m-1}{m+1}$, + +$$ +b_{4 i+2}=b_{4 i} \cdot \frac{2 N-4 i-1}{4 i+1} \cdot \frac{2 N-4 i-2}{4 i+2}=b_{4 i} \cdot \frac{2 N-4 i-1}{4 i+1} \cdot \frac{N-2 i-1}{2 i+1} . +$$ + +The desired congruence $b_{4 i+2} \equiv b_{4 i}+N$ may be multiplied by the odd number $(4 i+1)(2 i+1)$, giving rise to a chain of successively equivalent congruences: + +$$ +\begin{aligned} +b_{4 i}(2 N-4 i-1)(N-2 i-1) & \equiv\left(b_{4 i}+N\right)(4 i+1)(2 i+1) & & (\bmod 2 N), \\ +b_{4 i}(2 i+1-N) & \equiv\left(b_{4 i}+N\right)(2 i+1) & & (\bmod 2 N), \\ +\left(b_{4 i}+2 i+1\right) N & \equiv 0 & & (\bmod 2 N) ; +\end{aligned} +$$ + +and the last one is satisfied, as $b_{4 i}$ is odd. This settles the second relation in (5). + +The first one is proved by induction on $i$. It holds for $i=0$. Assume $b_{4 i} \equiv a_{2 i}(\bmod 2 N)$ and consider $i+1$ : + +$$ +b_{4 i+4}=b_{4 i+2} \cdot \frac{2 N-4 i-3}{4 i+3} \cdot \frac{2 N-4 i-4}{4 i+4} ; \quad a_{2 i+2}=a_{2 i} \cdot \frac{N-2 i-1}{2 i+1} \cdot \frac{N-2 i-2}{2 i+2} . +$$ + +Both expressions have the fraction $\frac{N-2 i-2}{2 i+2}$ as the last factor. Since $2 i+21$. + +Direct verification shows that this function meets the requirements. + +Conversely, let $f: \mathbb{N} \rightarrow \mathbb{N}$ satisfy (i) and (ii). Applying (i) for $x=1$ gives $d(f(1))=1$, so $f(1)=1$. In the sequel we prove that (1) holds for all $n>1$. Notice that $f(m)=f(n)$ implies $m=n$ in view of (i). The formula $d\left(p_{1}^{b_{1}} \cdots p_{k}^{b_{k}}\right)=\left(b_{1}+1\right) \cdots\left(b_{k}+1\right)$ will be used throughout. + +Let $p$ be a prime. Since $d(f(p))=p$, the formula just mentioned yields $f(p)=q^{p-1}$ for some prime $q$; in particular $f(2)=q^{2-1}=q$ is a prime. We prove that $f(p)=p^{p-1}$ for all primes $p$. + +Suppose that $p$ is odd and $f(p)=q^{p-1}$ for a prime $q$. Applying (ii) first with $x=2$, $y=p$ and then with $x=p, y=2$ shows that $f(2 p)$ divides both $(2-1) p^{2 p-1} f(2)=p^{2 p-1} f(2)$ and $(p-1) 2^{2 p-1} f(p)=(p-1) 2^{2 p-1} q^{p-1}$. If $q \neq p$ then the odd prime $p$ does not divide $(p-1) 2^{2 p-1} q^{p-1}$, hence the greatest common divisor of $p^{2 p-1} f(2)$ and $(p-1) 2^{2 p-1} q^{p-1}$ is a divisor of $f(2)$. Thus $f(2 p)$ divides $f(2)$ which is a prime. As $f(2 p)>1$, we obtain $f(2 p)=f(2)$ which is impossible. So $q=p$, i. e. $f(p)=p^{p-1}$. + +For $p=2$ the same argument with $x=2, y=3$ and $x=3, y=2$ shows that $f(6)$ divides both $3^{5} f(2)$ and $2^{6} f(3)=2^{6} 3^{2}$. If the prime $f(2)$ is odd then $f(6)$ divides $3^{2}=9$, so $f(6) \in\{1,3,9\}$. However then $6=d(f(6)) \in\{d(1), d(3), d(9)\}=\{1,2,3\}$ which is false. In conclusion $f(2)=2$. + +Next, for each $n>1$ the prime divisors of $f(n)$ are among the ones of $n$. Indeed, let $p$ be the least prime divisor of $n$. Apply (ii) with $x=p$ and $y=n / p$ to obtain that $f(n)$ divides $(p-1) y^{n-1} f(p)=(p-1) y^{n-1} p^{p-1}$. Write $f(n)=\ell P$ where $\ell$ is coprime to $n$ and $P$ is a product of primes dividing $n$. Since $\ell$ divides $(p-1) y^{n-1} p^{p-1}$ and is coprime to $y^{n-1} p^{p-1}$, it divides $p-1$; hence $d(\ell) \leq \ell1$ ). So $f\left(p^{a}\right)=p^{b}$ for some $b \geq 1$, and (i) yields $p^{a}=d\left(f\left(p^{a}\right)\right)=d\left(p^{b}\right)=b+1$. Hence $f\left(p^{a}\right)=p^{p^{a}-1}$, as needed. + +Let us finally show that (1) is true for a general $n>1$ with prime factorization $n=p_{1}^{a_{1}} \cdots p_{k}^{a_{k}}$. We saw that the prime factorization of $f(n)$ has the form $f(n)=p_{1}^{b_{1}} \cdots p_{k}^{b_{k}}$. For $i=1, \ldots, k$, set $x=p_{i}^{a_{i}}$ and $y=n / x$ in (ii) to infer that $f(n)$ divides $\left(p_{i}^{a_{i}}-1\right) y^{n-1} f\left(p_{i}^{a_{i}}\right)$. Hence $p_{i}^{b_{i}}$ divides $\left(p_{i}^{a_{i}}-1\right) y^{n-1} f\left(p_{i}^{a_{i}}\right)$, and because $p_{i}^{b_{i}}$ is coprime to $\left(p_{i}^{a_{i}}-1\right) y^{n-1}$, it follows that $p_{i}^{b_{i}}$ divides $f\left(p_{i}^{a_{i}}\right)=p_{i}^{p_{i}^{a_{i}}-1}$. So $b_{i} \leq p_{i}^{a_{i}}-1$ for all $i=1, \ldots, k$. Combined with (i), these conclusions imply + +$$ +p_{1}^{a_{1}} \cdots p_{k}^{a_{k}}=n=d(f(n))=d\left(p_{1}^{b_{1}} \cdots p_{k}^{b_{k}}\right)=\left(b_{1}+1\right) \cdots\left(b_{k}+1\right) \leq p_{1}^{a_{1}} \cdots p_{k}^{a_{k}} . +$$ + +Hence all inequalities $b_{i} \leq p_{i}^{a_{i}}-1$ must be equalities, $i=1, \ldots, k$, implying that (1) holds true. The proof is complete. + +N6. Prove that there exist infinitely many positive integers $n$ such that $n^{2}+1$ has a prime divisor greater than $2 n+\sqrt{2 n}$. + +Solution. Let $p \equiv 1(\bmod 8)$ be a prime. The congruence $x^{2} \equiv-1(\bmod p)$ has two solutions in $[1, p-1]$ whose sum is $p$. If $n$ is the smaller one of them then $p$ divides $n^{2}+1$ and $n \leq(p-1) / 2$. We show that $p>2 n+\sqrt{10 n}$. + +Let $n=(p-1) / 2-\ell$ where $\ell \geq 0$. Then $n^{2} \equiv-1(\bmod p)$ gives + +$$ +\left(\frac{p-1}{2}-\ell\right)^{2} \equiv-1 \quad(\bmod p) \quad \text { or } \quad(2 \ell+1)^{2}+4 \equiv 0 \quad(\bmod p) +$$ + +Thus $(2 \ell+1)^{2}+4=r p$ for some $r \geq 0$. As $(2 \ell+1)^{2} \equiv 1 \equiv p(\bmod 8)$, we have $r \equiv 5(\bmod 8)$, so that $r \geq 5$. Hence $(2 \ell+1)^{2}+4 \geq 5 p$, implying $\ell \geq(\sqrt{5 p-4}-1) / 2$. Set $\sqrt{5 p-4}=u$ for clarity; then $\ell \geq(u-1) / 2$. Therefore + +$$ +n=\frac{p-1}{2}-\ell \leq \frac{1}{2}(p-u) +$$ + +Combined with $p=\left(u^{2}+4\right) / 5$, this leads to $u^{2}-5 u-10 n+4 \geq 0$. Solving this quadratic inequality with respect to $u \geq 0$ gives $u \geq(5+\sqrt{40 n+9}) / 2$. So the estimate $n \leq(p-u) / 2$ leads to + +$$ +p \geq 2 n+u \geq 2 n+\frac{1}{2}(5+\sqrt{40 n+9})>2 n+\sqrt{10 n} +$$ + +Since there are infinitely many primes of the form $8 k+1$, it follows easily that there are also infinitely many $n$ with the stated property. + +Comment. By considering the prime factorization of the product $\prod_{n=1}^{N}\left(n^{2}+1\right)$, it can be obtained that its greatest prime divisor is at least $c N \log N$. This could improve the statement as $p>n \log n$. + +However, the proof applies some advanced information about the distribution of the primes of the form $4 k+1$, which is inappropriate for high schools contests. + diff --git a/IMO/md/en-IMO2009SL.md b/IMO/md/en-IMO2009SL.md new file mode 100644 index 0000000000000000000000000000000000000000..9d9f92d2e96d1e078738bfa2c70c2bebdf2f7dad --- /dev/null +++ b/IMO/md/en-IMO2009SL.md @@ -0,0 +1,2790 @@ +![](https://cdn.mathpix.com/cropped/2024_11_18_b191c9407e8cb3d672cbg-01.jpg?height=502&width=902&top_left_y=154&top_left_x=191) + +Bremen + +# International Mathematical Olympiad + +Germany + +## 10 to 22 July 2009 + +## Problem Shortlist with solutions + +![](https://cdn.mathpix.com/cropped/2024_11_18_b191c9407e8cb3d672cbg-01.jpg?height=1350&width=2075&top_left_y=1276&top_left_x=1) + +## Problem Shortlist with Solutions + +We insistently ask everybody to consider the following IMO Regulations rule: + +## These Shortlist Problems
have to be kept strictly confidential until IMO 2010. + +## The Problem Selection Committee + +Konrad Engel, Karl Fegert, Andreas Felgenhauer, Hans-Dietrich Gronau, Roger Labahn, Bernd Mulansky, Jürgen Prestin, Christian Reiher, Peter Scholze, Eckard Specht, Robert Strich, Martin Welk +gratefully received +132 problem proposals submitted by 39 countries: +Algeria, Australia, Austria, Belarus, Belgium, Bulgaria, Colombia, Croatia, Czech Republic, El Salvador, Estonia, Finland, France, Greece, Hong Kong, Hungary, India, Ireland, Islamic Republic of Iran, Japan, Democratic People's Republic of Korea, Lithuania, Luxembourg, The former Yugoslav Republic of Macedonia, Mongolia, Netherlands, New Zealand, Pakistan, Peru, Poland, Romania, Russian Federation, Slovenia, South Africa, Taiwan, Turkey, Ukraine, United Kingdom, United States of America. + +Layout: Roger Labahn with $\mathrm{LATE}_{\mathrm{E}} \mathrm{X}$ \& $\mathrm{T}_{\mathrm{E}} \mathrm{X}$ +Drawings: Eckard Specht with nicefig 2.0 +![](https://cdn.mathpix.com/cropped/2024_11_18_b191c9407e8cb3d672cbg-05.jpg?height=798&width=1363&top_left_y=589&top_left_x=346) + +The Problem Selection Committee + +## Contents + +Problem Shortlist 4 +Algebra 12 +Combinatorics 26 +Geometry 47 +Number Theory 69 + +## Algebra + +## A1 CZE (Czech Republic) + +Find the largest possible integer $k$, such that the following statement is true: +Let 2009 arbitrary non-degenerated triangles be given. In every triangle the three sides are colored, such that one is blue, one is red and one is white. Now, for every color separately, let us sort the lengths of the sides. We obtain + +$$ +\begin{aligned} +& b_{1} \leq b_{2} \\ +& \leq \ldots \leq b_{2009} \quad \text { the lengths of the blue sides, } \\ +r_{1} & \leq r_{2} \leq \ldots \leq r_{2009} \quad \text { the lengths of the red sides, } \\ +\text { and } \quad w_{1} & \leq w_{2} \leq \ldots \leq w_{2009} \quad \text { the lengths of the white sides. } +\end{aligned} +$$ + +Then there exist $k$ indices $j$ such that we can form a non-degenerated triangle with side lengths $b_{j}, r_{j}, w_{j}$ 。 + +## A2 EST (Estonia) + +Let $a, b, c$ be positive real numbers such that $\frac{1}{a}+\frac{1}{b}+\frac{1}{c}=a+b+c$. Prove that + +$$ +\frac{1}{(2 a+b+c)^{2}}+\frac{1}{(2 b+c+a)^{2}}+\frac{1}{(2 c+a+b)^{2}} \leq \frac{3}{16} +$$ + +## A3 FRA (France) + +Determine all functions $f$ from the set of positive integers into the set of positive integers such that for all $x$ and $y$ there exists a non degenerated triangle with sides of lengths + +$$ +x, \quad f(y) \quad \text { and } \quad f(y+f(x)-1) +$$ + +## A4 BLR (Belarus) + +Let $a, b, c$ be positive real numbers such that $a b+b c+c a \leq 3 a b c$. Prove that + +$$ +\sqrt{\frac{a^{2}+b^{2}}{a+b}}+\sqrt{\frac{b^{2}+c^{2}}{b+c}}+\sqrt{\frac{c^{2}+a^{2}}{c+a}}+3 \leq \sqrt{2}(\sqrt{a+b}+\sqrt{b+c}+\sqrt{c+a}) . +$$ + +## A5 BLR (Belarus) + +Let $f$ be any function that maps the set of real numbers into the set of real numbers. Prove that there exist real numbers $x$ and $y$ such that + +$$ +f(x-f(y))>y f(x)+x +$$ + +## A6 USA (United States of America) + +Suppose that $s_{1}, s_{2}, s_{3}, \ldots$ is a strictly increasing sequence of positive integers such that the subsequences + +$$ +s_{s_{1}}, s_{s_{2}}, s_{s_{3}}, \ldots \quad \text { and } \quad s_{s_{1}+1}, s_{s_{2}+1}, s_{s_{3}+1}, \ldots +$$ + +are both arithmetic progressions. Prove that $s_{1}, s_{2}, s_{3}, \ldots$ is itself an arithmetic progression. + +## A7 JPN (Japan) + +Find all functions $f$ from the set of real numbers into the set of real numbers which satisfy for all real $x, y$ the identity + +$$ +f(x f(x+y))=f(y f(x))+x^{2} +$$ + +## Combinatorics + +## C1 NZL (New Zealand) + +Consider 2009 cards, each having one gold side and one black side, lying in parallel on a long table. Initially all cards show their gold sides. Two players, standing by the same long side of the table, play a game with alternating moves. Each move consists of choosing a block of 50 consecutive cards, the leftmost of which is showing gold, and turning them all over, so those which showed gold now show black and vice versa. The last player who can make a legal move wins. +(a) Does the game necessarily end? +(b) Does there exist a winning strategy for the starting player? + +## C2 ROU (Romania) + +For any integer $n \geq 2$, let $N(n)$ be the maximal number of triples $\left(a_{i}, b_{i}, c_{i}\right), i=1, \ldots, N(n)$, consisting of nonnegative integers $a_{i}, b_{i}$ and $c_{i}$ such that the following two conditions are satisfied: +(1) $a_{i}+b_{i}+c_{i}=n$ for all $i=1, \ldots, N(n)$, +(2) If $i \neq j$, then $a_{i} \neq a_{j}, b_{i} \neq b_{j}$ and $c_{i} \neq c_{j}$. + +Determine $N(n)$ for all $n \geq 2$. + +Comment. The original problem was formulated for $m$-tuples instead for triples. The numbers $N(m, n)$ are then defined similarly to $N(n)$ in the case $m=3$. The numbers $N(3, n)$ and $N(n, n)$ should be determined. The case $m=3$ is the same as in the present problem. The upper bound for $N(n, n)$ can be proved by a simple generalization. The construction of a set of triples attaining the bound can be easily done by induction from $n$ to $n+2$. + +## C3 RUS (Russian Federation) + +Let $n$ be a positive integer. Given a sequence $\varepsilon_{1}, \ldots, \varepsilon_{n-1}$ with $\varepsilon_{i}=0$ or $\varepsilon_{i}=1$ for each $i=1, \ldots, n-1$, the sequences $a_{0}, \ldots, a_{n}$ and $b_{0}, \ldots, b_{n}$ are constructed by the following rules: + +$$ +\begin{gathered} +a_{0}=b_{0}=1, \quad a_{1}=b_{1}=7, \\ +a_{i+1}=\left\{\begin{array}{ll} +2 a_{i-1}+3 a_{i}, & \text { if } \varepsilon_{i}=0, \\ +3 a_{i-1}+a_{i}, & \text { if } \varepsilon_{i}=1, +\end{array} \text { for each } i=1, \ldots, n-1,\right. \\ +b_{i+1}=\left\{\begin{array}{ll} +2 b_{i-1}+3 b_{i}, & \text { if } \varepsilon_{n-i}=0, \\ +3 b_{i-1}+b_{i}, & \text { if } \varepsilon_{n-i}=1, +\end{array} \text { for each } i=1, \ldots, n-1 .\right. +\end{gathered} +$$ + +Prove that $a_{n}=b_{n}$. + +## C4 NLD (Netherlands) + +For an integer $m \geq 1$, we consider partitions of a $2^{m} \times 2^{m}$ chessboard into rectangles consisting of cells of the chessboard, in which each of the $2^{m}$ cells along one diagonal forms a separate rectangle of side length 1 . Determine the smallest possible sum of rectangle perimeters in such a partition. + +## C5 NLD (Netherlands) + +Five identical empty buckets of 2-liter capacity stand at the vertices of a regular pentagon. Cinderella and her wicked Stepmother go through a sequence of rounds: At the beginning of every round, the Stepmother takes one liter of water from the nearby river and distributes it arbitrarily over the five buckets. Then Cinderella chooses a pair of neighboring buckets, empties them into the river, and puts them back. Then the next round begins. The Stepmother's goal is to make one of these buckets overflow. Cinderella's goal is to prevent this. Can the wicked Stepmother enforce a bucket overflow? + +## C6 BGR (Bulgaria) + +On a $999 \times 999$ board a limp rook can move in the following way: From any square it can move to any of its adjacent squares, i.e. a square having a common side with it, and every move must be a turn, i.e. the directions of any two consecutive moves must be perpendicular. A nonintersecting route of the limp rook consists of a sequence of pairwise different squares that the limp rook can visit in that order by an admissible sequence of moves. Such a non-intersecting route is called cyclic, if the limp rook can, after reaching the last square of the route, move directly to the first square of the route and start over. +How many squares does the longest possible cyclic, non-intersecting route of a limp rook visit? + +## C7 RUS (Russian Federation) + +Variant 1. A grasshopper jumps along the real axis. He starts at point 0 and makes 2009 jumps to the right with lengths $1,2, \ldots, 2009$ in an arbitrary order. Let $M$ be a set of 2008 positive integers less than 1005 - 2009. Prove that the grasshopper can arrange his jumps in such a way that he never lands on a point from $M$. + +Variant 2. Let $n$ be a nonnegative integer. A grasshopper jumps along the real axis. He starts at point 0 and makes $n+1$ jumps to the right with pairwise different positive integral lengths $a_{1}, a_{2}, \ldots, a_{n+1}$ in an arbitrary order. Let $M$ be a set of $n$ positive integers in the interval $(0, s)$, where $s=a_{1}+a_{2}+\cdots+a_{n+1}$. Prove that the grasshopper can arrange his jumps in such a way that he never lands on a point from $M$. + +## C8 AUT (Austria) + +For any integer $n \geq 2$, we compute the integer $h(n)$ by applying the following procedure to its decimal representation. Let $r$ be the rightmost digit of $n$. +(1) If $r=0$, then the decimal representation of $h(n)$ results from the decimal representation of $n$ by removing this rightmost digit 0 . +(2) If $1 \leq r \leq 9$ we split the decimal representation of $n$ into a maximal right part $R$ that solely consists of digits not less than $r$ and into a left part $L$ that either is empty or ends with a digit strictly smaller than $r$. Then the decimal representation of $h(n)$ consists of the decimal representation of $L$, followed by two copies of the decimal representation of $R-1$. For instance, for the number $n=17,151,345,543$, we will have $L=17,151, R=345,543$ and $h(n)=17,151,345,542,345,542$. +Prove that, starting with an arbitrary integer $n \geq 2$, iterated application of $h$ produces the integer 1 after finitely many steps. + +## Geometry + +## G1 BEL (Belgium) + +Let $A B C$ be a triangle with $A B=A C$. The angle bisectors of $A$ and $B$ meet the sides $B C$ and $A C$ in $D$ and $E$, respectively. Let $K$ be the incenter of triangle $A D C$. Suppose that $\angle B E K=45^{\circ}$. Find all possible values of $\angle B A C$. + +## G2 RUS (Russian Federation) + +Let $A B C$ be a triangle with circumcenter $O$. The points $P$ and $Q$ are interior points of the sides $C A$ and $A B$, respectively. The circle $k$ passes through the midpoints of the segments $B P$, $C Q$, and $P Q$. Prove that if the line $P Q$ is tangent to circle $k$ then $O P=O Q$. + +## G3 IRN (Islamic Republic of Iran) + +Let $A B C$ be a triangle. The incircle of $A B C$ touches the sides $A B$ and $A C$ at the points $Z$ and $Y$, respectively. Let $G$ be the point where the lines $B Y$ and $C Z$ meet, and let $R$ and $S$ be points such that the two quadrilaterals $B C Y R$ and $B C S Z$ are parallelograms. +Prove that $G R=G S$. + +## G4 UNK (United Kingdom) + +Given a cyclic quadrilateral $A B C D$, let the diagonals $A C$ and $B D$ meet at $E$ and the lines $A D$ and $B C$ meet at $F$. The midpoints of $A B$ and $C D$ are $G$ and $H$, respectively. Show that $E F$ is tangent at $E$ to the circle through the points $E, G$, and $H$. + +## G5 POL (Poland) + +Let $P$ be a polygon that is convex and symmetric to some point $O$. Prove that for some parallelogram $R$ satisfying $P \subset R$ we have + +$$ +\frac{|R|}{|P|} \leq \sqrt{2} +$$ + +where $|R|$ and $|P|$ denote the area of the sets $R$ and $P$, respectively. + +## G6 UKR (Ukraine) + +Let the sides $A D$ and $B C$ of the quadrilateral $A B C D$ (such that $A B$ is not parallel to $C D$ ) intersect at point $P$. Points $O_{1}$ and $O_{2}$ are the circumcenters and points $H_{1}$ and $H_{2}$ are the orthocenters of triangles $A B P$ and $D C P$, respectively. Denote the midpoints of segments $O_{1} H_{1}$ and $O_{2} H_{2}$ by $E_{1}$ and $E_{2}$, respectively. Prove that the perpendicular from $E_{1}$ on $C D$, the perpendicular from $E_{2}$ on $A B$ and the line $H_{1} H_{2}$ are concurrent. + +## G7 IRN (Islamic Republic of Iran) + +Let $A B C$ be a triangle with incenter $I$ and let $X, Y$ and $Z$ be the incenters of the triangles $B I C, C I A$ and $A I B$, respectively. Let the triangle $X Y Z$ be equilateral. Prove that $A B C$ is equilateral too. + +## G8 BGR (Bulgaria) + +Let $A B C D$ be a circumscribed quadrilateral. Let $g$ be a line through $A$ which meets the segment $B C$ in $M$ and the line $C D$ in $N$. Denote by $I_{1}, I_{2}$, and $I_{3}$ the incenters of $\triangle A B M$, $\triangle M N C$, and $\triangle N D A$, respectively. Show that the orthocenter of $\triangle I_{1} I_{2} I_{3}$ lies on $g$. + +## Number Theory + +## N1 AUS (Australia) + +A social club has $n$ members. They have the membership numbers $1,2, \ldots, n$, respectively. From time to time members send presents to other members, including items they have already received as presents from other members. In order to avoid the embarrassing situation that a member might receive a present that he or she has sent to other members, the club adds the following rule to its statutes at one of its annual general meetings: +"A member with membership number $a$ is permitted to send a present to a member with membership number $b$ if and only if $a(b-1)$ is a multiple of $n$." +Prove that, if each member follows this rule, none will receive a present from another member that he or she has already sent to other members. + +Alternative formulation: Let $G$ be a directed graph with $n$ vertices $v_{1}, v_{2}, \ldots, v_{n}$, such that there is an edge going from $v_{a}$ to $v_{b}$ if and only if $a$ and $b$ are distinct and $a(b-1)$ is a multiple of $n$. Prove that this graph does not contain a directed cycle. + +## N2 PER (Peru) + +A positive integer $N$ is called balanced, if $N=1$ or if $N$ can be written as a product of an even number of not necessarily distinct primes. Given positive integers $a$ and $b$, consider the polynomial $P$ defined by $P(x)=(x+a)(x+b)$. +(a) Prove that there exist distinct positive integers $a$ and $b$ such that all the numbers $P(1), P(2)$, $\ldots, P(50)$ are balanced. +(b) Prove that if $P(n)$ is balanced for all positive integers $n$, then $a=b$. + +## N3 EST (Estonia) + +Let $f$ be a non-constant function from the set of positive integers into the set of positive integers, such that $a-b$ divides $f(a)-f(b)$ for all distinct positive integers $a, b$. Prove that there exist infinitely many primes $p$ such that $p$ divides $f(c)$ for some positive integer $c$. + +## N4 PRK (Democratic People's Republic of Korea) + +Find all positive integers $n$ such that there exists a sequence of positive integers $a_{1}, a_{2}, \ldots, a_{n}$ satisfying + +$$ +a_{k+1}=\frac{a_{k}^{2}+1}{a_{k-1}+1}-1 +$$ + +for every $k$ with $2 \leq k \leq n-1$. + +## N5 HUN (Hungary) + +Let $P(x)$ be a non-constant polynomial with integer coefficients. Prove that there is no function $T$ from the set of integers into the set of integers such that the number of integers $x$ with $T^{n}(x)=x$ is equal to $P(n)$ for every $n \geq 1$, where $T^{n}$ denotes the $n$-fold application of $T$. + +## N6 TUR (Turkey) + +Let $k$ be a positive integer. Show that if there exists a sequence $a_{0}, a_{1}, \ldots$ of integers satisfying the condition + +$$ +a_{n}=\frac{a_{n-1}+n^{k}}{n} \quad \text { for all } n \geq 1 +$$ + +then $k-2$ is divisible by 3 . + +## N7 MNG (Mongolia) + +Let $a$ and $b$ be distinct integers greater than 1 . Prove that there exists a positive integer $n$ such that $\left(a^{n}-1\right)\left(b^{n}-1\right)$ is not a perfect square. + +## Algebra + +## A1 CZE (Czech Republic) + +Find the largest possible integer $k$, such that the following statement is true: +Let 2009 arbitrary non-degenerated triangles be given. In every triangle the three sides are colored, such that one is blue, one is red and one is white. Now, for every color separately, let us sort the lengths of the sides. We obtain + +$$ +\begin{array}{rlrl} +b_{1} & \leq b_{2} & \leq \ldots \leq b_{2009} \quad & \\ +& & \text { the lengths of the blue sides, } \\ +r_{1} & \leq r_{2} & \leq \ldots \leq r_{2009} \quad \text { the lengths of the red sides, } \\ +w_{1} & \leq w_{2} & \leq \ldots \leq w_{2009} \quad \text { the lengths of the white sides. } +\end{array} +$$ + +Then there exist $k$ indices $j$ such that we can form a non-degenerated triangle with side lengths $b_{j}, r_{j}, w_{j}$ 。 + +Solution. We will prove that the largest possible number $k$ of indices satisfying the given condition is one. +Firstly we prove that $b_{2009}, r_{2009}, w_{2009}$ are always lengths of the sides of a triangle. Without loss of generality we may assume that $w_{2009} \geq r_{2009} \geq b_{2009}$. We show that the inequality $b_{2009}+r_{2009}>w_{2009}$ holds. Evidently, there exists a triangle with side lengths $w, b, r$ for the white, blue and red side, respectively, such that $w_{2009}=w$. By the conditions of the problem we have $b+r>w, b_{2009} \geq b$ and $r_{2009} \geq r$. From these inequalities it follows + +$$ +b_{2009}+r_{2009} \geq b+r>w=w_{2009} +$$ + +Secondly we will describe a sequence of triangles for which $w_{j}, b_{j}, r_{j}$ with $j<2009$ are not the lengths of the sides of a triangle. Let us define the sequence $\Delta_{j}, j=1,2, \ldots, 2009$, of triangles, where $\Delta_{j}$ has +a blue side of length $2 j$, +a red side of length $j$ for all $j \leq 2008$ and 4018 for $j=2009$, +and a white side of length $j+1$ for all $j \leq 2007$, 4018 for $j=2008$ and 1 for $j=2009$. +Since + +$$ +\begin{array}{rrrl} +(j+1)+j>2 j & \geq j+1>j, & \text { if } & j \leq 2007 \\ +2 j+j>4018>2 j \quad>j, & \text { if } & j=2008 \\ +4018+1>2 j & =4018>1, & \text { if } & j=2009 +\end{array} +$$ + +such a sequence of triangles exists. Moreover, $w_{j}=j, r_{j}=j$ and $b_{j}=2 j$ for $1 \leq j \leq 2008$. Then + +$$ +w_{j}+r_{j}=j+j=2 j=b_{j} +$$ + +i.e., $b_{j}, r_{j}$ and $w_{j}$ are not the lengths of the sides of a triangle for $1 \leq j \leq 2008$. + +## A2 EST (Estonia) + +Let $a, b, c$ be positive real numbers such that $\frac{1}{a}+\frac{1}{b}+\frac{1}{c}=a+b+c$. Prove that + +$$ +\frac{1}{(2 a+b+c)^{2}}+\frac{1}{(2 b+c+a)^{2}}+\frac{1}{(2 c+a+b)^{2}} \leq \frac{3}{16} +$$ + +Solution 1. For positive real numbers $x, y, z$, from the arithmetic-geometric-mean inequality, + +$$ +2 x+y+z=(x+y)+(x+z) \geq 2 \sqrt{(x+y)(x+z)} +$$ + +we obtain + +$$ +\frac{1}{(2 x+y+z)^{2}} \leq \frac{1}{4(x+y)(x+z)} +$$ + +Applying this to the left-hand side terms of the inequality to prove, we get + +$$ +\begin{aligned} +\frac{1}{(2 a+b+c)^{2}} & +\frac{1}{(2 b+c+a)^{2}}+\frac{1}{(2 c+a+b)^{2}} \\ +& \leq \frac{1}{4(a+b)(a+c)}+\frac{1}{4(b+c)(b+a)}+\frac{1}{4(c+a)(c+b)} \\ +& =\frac{(b+c)+(c+a)+(a+b)}{4(a+b)(b+c)(c+a)}=\frac{a+b+c}{2(a+b)(b+c)(c+a)} . +\end{aligned} +$$ + +A second application of the inequality of the arithmetic-geometric mean yields + +$$ +a^{2} b+a^{2} c+b^{2} a+b^{2} c+c^{2} a+c^{2} b \geq 6 a b c +$$ + +or, equivalently, + +$$ +9(a+b)(b+c)(c+a) \geq 8(a+b+c)(a b+b c+c a) +$$ + +The supposition $\frac{1}{a}+\frac{1}{b}+\frac{1}{c}=a+b+c$ can be written as + +$$ +a b+b c+c a=a b c(a+b+c) . +$$ + +Applying the arithmetic-geometric-mean inequality $x^{2} y^{2}+x^{2} z^{2} \geq 2 x^{2} y z$ thrice, we get + +$$ +a^{2} b^{2}+b^{2} c^{2}+c^{2} a^{2} \geq a^{2} b c+a b^{2} c+a b c^{2} +$$ + +which is equivalent to + +$$ +(a b+b c+c a)^{2} \geq 3 a b c(a+b+c) +$$ + +Combining (1), (2), (3), and (4), we will finish the proof: + +$$ +\begin{aligned} +\frac{a+b+c}{2(a+b)(b+c)(c+a)} & =\frac{(a+b+c)(a b+b c+c a)}{2(a+b)(b+c)(c+a)} \cdot \frac{a b+b c+c a}{a b c(a+b+c)} \cdot \frac{a b c(a+b+c)}{(a b+b c+c a)^{2}} \\ +& \leq \frac{9}{2 \cdot 8} \cdot 1 \cdot \frac{1}{3}=\frac{3}{16} +\end{aligned} +$$ + +Solution 2. Equivalently, we prove the homogenized inequality + +$$ +\frac{(a+b+c)^{2}}{(2 a+b+c)^{2}}+\frac{(a+b+c)^{2}}{(a+2 b+c)^{2}}+\frac{(a+b+c)^{2}}{(a+b+2 c)^{2}} \leq \frac{3}{16}(a+b+c)\left(\frac{1}{a}+\frac{1}{b}+\frac{1}{c}\right) +$$ + +for all positive real numbers $a, b, c$. Without loss of generality we choose $a+b+c=1$. Thus, the problem is equivalent to prove for all $a, b, c>0$, fulfilling this condition, the inequality + +$$ +\frac{1}{(1+a)^{2}}+\frac{1}{(1+b)^{2}}+\frac{1}{(1+c)^{2}} \leq \frac{3}{16}\left(\frac{1}{a}+\frac{1}{b}+\frac{1}{c}\right) . +$$ + +Applying Jensen's inequality to the function $f(x)=\frac{x}{(1+x)^{2}}$, which is concave for $0 \leq x \leq 2$ and increasing for $0 \leq x \leq 1$, we obtain + +$$ +\alpha \frac{a}{(1+a)^{2}}+\beta \frac{b}{(1+b)^{2}}+\gamma \frac{c}{(1+c)^{2}} \leq(\alpha+\beta+\gamma) \frac{A}{(1+A)^{2}}, \quad \text { where } \quad A=\frac{\alpha a+\beta b+\gamma c}{\alpha+\beta+\gamma} +$$ + +Choosing $\alpha=\frac{1}{a}, \beta=\frac{1}{b}$, and $\gamma=\frac{1}{c}$, we can apply the harmonic-arithmetic-mean inequality + +$$ +A=\frac{3}{\frac{1}{a}+\frac{1}{b}+\frac{1}{c}} \leq \frac{a+b+c}{3}=\frac{1}{3}<1 +$$ + +Finally we prove (5): + +$$ +\begin{aligned} +\frac{1}{(1+a)^{2}}+\frac{1}{(1+b)^{2}}+\frac{1}{(1+c)^{2}} & \leq\left(\frac{1}{a}+\frac{1}{b}+\frac{1}{c}\right) \frac{A}{(1+A)^{2}} \\ +& \leq\left(\frac{1}{a}+\frac{1}{b}+\frac{1}{c}\right) \frac{\frac{1}{3}}{\left(1+\frac{1}{3}\right)^{2}}=\frac{3}{16}\left(\frac{1}{a}+\frac{1}{b}+\frac{1}{c}\right) . +\end{aligned} +$$ + +## A3 FRA (France) + +Determine all functions $f$ from the set of positive integers into the set of positive integers such that for all $x$ and $y$ there exists a non degenerated triangle with sides of lengths + +$$ +x, \quad f(y) \quad \text { and } \quad f(y+f(x)-1) . +$$ + +Solution. The identity function $f(x)=x$ is the only solution of the problem. +If $f(x)=x$ for all positive integers $x$, the given three lengths are $x, y=f(y)$ and $z=$ $f(y+f(x)-1)=x+y-1$. Because of $x \geq 1, y \geq 1$ we have $z \geq \max \{x, y\}>|x-y|$ and $z1$ we would conclude $f(y)=f(y+m)$ for all $y$ considering the triangle with the side lengths $1, f(y)$ and $f(y+m)$. Thus, $f$ would be $m$-periodic and, consequently, bounded. Let $B$ be a bound, $f(x) \leq B$. If we choose $x>2 B$ we obtain the contradiction $x>2 B \geq f(y)+f(y+f(x)-1)$. + +Step 2. For all positive integers $z$, we have $f(f(z))=z$. +Setting $x=z$ and $y=1$ this follows immediately from Step 1. + +Step 3. For all integers $z \geq 1$, we have $f(z) \leq z$. +Let us show, that the contrary leads to a contradiction. Assume $w+1=f(z)>z$ for some $z$. From Step 1 we know that $w \geq z \geq 2$. Let $M=\max \{f(1), f(2), \ldots, f(w)\}$ be the largest value of $f$ for the first $w$ integers. First we show, that no positive integer $t$ exists with + +$$ +f(t)>\frac{z-1}{w} \cdot t+M +$$ + +otherwise we decompose the smallest value $t$ as $t=w r+s$ where $r$ is an integer and $1 \leq s \leq w$. Because of the definition of $M$, we have $t>w$. Setting $x=z$ and $y=t-w$ we get from the triangle inequality + +$$ +z+f(t-w)>f((t-w)+f(z)-1)=f(t-w+w)=f(t) +$$ + +Hence, + +$$ +f(t-w) \geq f(t)-(z-1)>\frac{z-1}{w}(t-w)+M +$$ + +a contradiction to the minimality of $t$. +Therefore the inequality (1) fails for all $t \geq 1$, we have proven + +$$ +f(t) \leq \frac{z-1}{w} \cdot t+M +$$ + +instead. + +Now, using (2), we finish the proof of Step 3. Because of $z \leq w$ we have $\frac{z-1}{w}<1$ and we can choose an integer $t$ sufficiently large to fulfill the condition + +$$ +\left(\frac{z-1}{w}\right)^{2} t+\left(\frac{z-1}{w}+1\right) My f(x)+x +$$ + +Solution 1. Assume that + +$$ +f(x-f(y)) \leq y f(x)+x \quad \text { for all real } x, y +$$ + +Let $a=f(0)$. Setting $y=0$ in (1) gives $f(x-a) \leq x$ for all real $x$ and, equivalently, + +$$ +f(y) \leq y+a \quad \text { for all real } y +$$ + +Setting $x=f(y)$ in (1) yields in view of (2) + +$$ +a=f(0) \leq y f(f(y))+f(y) \leq y f(f(y))+y+a . +$$ + +This implies $0 \leq y(f(f(y))+1)$ and thus + +$$ +f(f(y)) \geq-1 \quad \text { for all } y>0 +$$ + +From (2) and (3) we obtain $-1 \leq f(f(y)) \leq f(y)+a$ for all $y>0$, so + +$$ +f(y) \geq-a-1 \quad \text { for all } y>0 +$$ + +Now we show that + +$$ +f(x) \leq 0 \quad \text { for all real } x +$$ + +Assume the contrary, i.e. there is some $x$ such that $f(x)>0$. Take any $y$ such that + +$$ +y0 +$$ + +and with (1) and (4) we obtain + +$$ +y f(x)+x \geq f(x-f(y)) \geq-a-1, +$$ + +whence + +$$ +y \geq \frac{-a-x-1}{f(x)} +$$ + +contrary to our choice of $y$. Thereby, we have established (5). +Setting $x=0$ in (5) leads to $a=f(0) \leq 0$ and (2) then yields + +$$ +f(x) \leq x \quad \text { for all real } x +$$ + +Now choose $y$ such that $y>0$ and $y>-f(-1)-1$ and set $x=f(y)-1$. From (11), (5) and +(6) we obtain + +$$ +f(-1)=f(x-f(y)) \leq y f(x)+x=y f(f(y)-1)+f(y)-1 \leq y(f(y)-1)-1 \leq-y-1, +$$ + +i.e. $y \leq-f(-1)-1$, a contradiction to the choice of $y$. + +Solution 2. Assume that + +$$ +f(x-f(y)) \leq y f(x)+x \quad \text { for all real } x, y +$$ + +Let $a=f(0)$. Setting $y=0$ in (7) gives $f(x-a) \leq x$ for all real $x$ and, equivalently, + +$$ +f(y) \leq y+a \quad \text { for all real } y +$$ + +Now we show that + +$$ +f(z) \geq 0 \quad \text { for all } z \geq 1 +$$ + +Let $z \geq 1$ be fixed, set $b=f(z)$ and assume that $b<0$. Setting $x=w+b$ and $y=z$ in (7) gives + +$$ +f(w)-z f(w+b) \leq w+b \quad \text { for all real } w +$$ + +Applying (10) to $w, w+b, \ldots, w+(n-1) b$, where $n=1,2, \ldots$, leads to + +$$ +\begin{gathered} +f(w)-z^{n} f(w+n b)=(f(w)-z f(w+b))+z(f(w+b)-z f(w+2 b)) \\ ++\cdots+z^{n-1}(f(w+(n-1) b)-z f(w+n b)) \\ +\leq(w+b)+z(w+2 b)+\cdots+z^{n-1}(w+n b) +\end{gathered} +$$ + +From (8) we obtain + +$$ +f(w+n b) \leq w+n b+a +$$ + +and, thus, we have for all positive integers $n$ + +$$ +f(w) \leq\left(1+z+\cdots+z^{n-1}+z^{n}\right) w+\left(1+2 z+\cdots+n z^{n-1}+n z^{n}\right) b+z^{n} a . +$$ + +With $w=0$ we get + +$$ +a \leq\left(1+2 z+\cdots+n z^{n-1}+n z^{n}\right) b+a z^{n} . +$$ + +In view of the assumption $b<0$ we find some $n$ such that + +$$ +a>(n b+a) z^{n} +$$ + +because the right hand side tends to $-\infty$ as $n \rightarrow \infty$. Now (12) and (13) give the desired contradiction and (9) is established. In addition, we have for $z=1$ the strict inequality + +$$ +f(1)>0 \text {. } +$$ + +Indeed, assume that $f(1)=0$. Then setting $w=-1$ and $z=1$ in (11) leads to + +$$ +f(-1) \leq-(n+1)+a +$$ + +which is false if $n$ is sufficiently large. +To complete the proof we set $t=\min \{-a,-2 / f(1)\}$. Setting $x=1$ and $y=t$ in (7) gives + +$$ +f(1-f(t)) \leq t f(1)+1 \leq-2+1=-1 . +$$ + +On the other hand, by (8) and the choice of $t$ we have $f(t) \leq t+a \leq 0$ and hence $1-f(t) \geq 1$. The inequality (9) yields + +$$ +f(1-f(t)) \geq 0 +$$ + +which contradicts (15). + +## A6 USA (United States of America) + +Suppose that $s_{1}, s_{2}, s_{3}, \ldots$ is a strictly increasing sequence of positive integers such that the subsequences + +$$ +s_{s_{1}}, s_{s_{2}}, s_{s_{3}}, \ldots \quad \text { and } \quad s_{s_{1}+1}, s_{s_{2}+1}, s_{s_{3}+1}, \ldots +$$ + +are both arithmetic progressions. Prove that $s_{1}, s_{2}, s_{3}, \ldots$ is itself an arithmetic progression. + +Solution 1. Let $D$ be the common difference of the progression $s_{s_{1}}, s_{s_{2}}, \ldots$. Let for $n=$ 1, 2, ... + +$$ +d_{n}=s_{n+1}-s_{n} +$$ + +We have to prove that $d_{n}$ is constant. First we show that the numbers $d_{n}$ are bounded. Indeed, by supposition $d_{n} \geq 1$ for all $n$. Thus, we have for all $n$ + +$$ +d_{n}=s_{n+1}-s_{n} \leq d_{s_{n}}+d_{s_{n}+1}+\cdots+d_{s_{n+1}-1}=s_{s_{n+1}}-s_{s_{n}}=D . +$$ + +The boundedness implies that there exist + +$$ +m=\min \left\{d_{n}: n=1,2, \ldots\right\} \quad \text { and } \quad M=\max \left\{d_{n}: n=1,2, \ldots\right\} +$$ + +It suffices to show that $m=M$. Assume that $mn$ if $d_{n}=n$, because in the case $s_{n}=n$ we would have $m=d_{n}=d_{s_{n}}=M$ in contradiction to the assumption $mn$. Consequently, there is a strictly increasing sequence $n_{1}, n_{2}, \ldots$ such that + +$$ +d_{s_{n_{1}}}=M, \quad d_{s_{n_{2}}}=m, \quad d_{s_{n_{3}}}=M, \quad d_{s_{n_{4}}}=m, \quad \ldots +$$ + +The sequence $d_{s_{1}}, d_{s_{2}}, \ldots$ is the sequence of pairwise differences of $s_{s_{1}+1}, s_{s_{2}+1}, \ldots$ and $s_{s_{1}}, s_{s_{2}}, \ldots$, hence also an arithmetic progression. Thus $m=M$. + +Solution 2. Let the integers $D$ and $E$ be the common differences of the progressions $s_{s_{1}}, s_{s_{2}}, \ldots$ and $s_{s_{1}+1}, s_{s_{2}+1}, \ldots$, respectively. Let briefly $A=s_{s_{1}}-D$ and $B=s_{s_{1}+1}-E$. Then, for all positive integers $n$, + +$$ +s_{s_{n}}=A+n D, \quad s_{s_{n}+1}=B+n E +$$ + +Since the sequence $s_{1}, s_{2}, \ldots$ is strictly increasing, we have for all positive integers $n$ + +$$ +s_{s_{n}}s_{n}+m$. Then $\left.m(m+1) \leq m\left(s_{n+1}-s_{n}\right) \leq s_{s_{n+1}}-s_{s_{n}}=(A+(n+1) D)-(A+n D)\right)=D=m(B-A)=m^{2}$, a contradiction. Hence $s_{1}, s_{2}, \ldots$ is an arithmetic progression with common difference $m$. + +## A7 JPN (Japan) + +Find all functions $f$ from the set of real numbers into the set of real numbers which satisfy for all real $x, y$ the identity + +$$ +f(x f(x+y))=f(y f(x))+x^{2} +$$ + +Solution 1. It is no hard to see that the two functions given by $f(x)=x$ and $f(x)=-x$ for all real $x$ respectively solve the functional equation. In the sequel, we prove that there are no further solutions. +Let $f$ be a function satisfying the given equation. It is clear that $f$ cannot be a constant. Let us first show that $f(0)=0$. Suppose that $f(0) \neq 0$. For any real $t$, substituting $(x, y)=\left(0, \frac{t}{f(0)}\right)$ into the given functional equation, we obtain + +$$ +f(0)=f(t) +$$ + +contradicting the fact that $f$ is not a constant function. Therefore, $f(0)=0$. Next for any $t$, substituting $(x, y)=(t, 0)$ and $(x, y)=(t,-t)$ into the given equation, we get + +$$ +f(t f(t))=f(0)+t^{2}=t^{2} +$$ + +and + +$$ +f(t f(0))=f(-t f(t))+t^{2} +$$ + +respectively. Therefore, we conclude that + +$$ +f(t f(t))=t^{2}, \quad f(-t f(t))=-t^{2}, \quad \text { for every real } t +$$ + +Consequently, for every real $v$, there exists a real $u$, such that $f(u)=v$. We also see that if $f(t)=0$, then $0=f(t f(t))=t^{2}$ so that $t=0$, and thus 0 is the only real number satisfying $f(t)=0$. +We next show that for any real number $s$, + +$$ +f(-s)=-f(s) +$$ + +This is clear if $f(s)=0$. Suppose now $f(s)<0$, then we can find a number $t$ for which $f(s)=-t^{2}$. As $t \neq 0$ implies $f(t) \neq 0$, we can also find number $a$ such that $a f(t)=s$. Substituting $(x, y)=(t, a)$ into the given equation, we get + +$$ +f(t f(t+a))=f(a f(t))+t^{2}=f(s)+t^{2}=0 +$$ + +and therefore, $t f(t+a)=0$, which implies $t+a=0$, and hence $s=-t f(t)$. Consequently, $f(-s)=f(t f(t))=t^{2}=-\left(-t^{2}\right)=-f(s)$ holds in this case. +Finally, suppose $f(s)>0$ holds. Then there exists a real number $t \neq 0$ for which $f(s)=t^{2}$. Choose a number $a$ such that $t f(a)=s$. Substituting $(x, y)=(t, a-t)$ into the given equation, we get $f(s)=f(t f(a))=f((a-t) f(t))+t^{2}=f((a-t) f(t))+f(s)$. So we have $f((a-t) f(t))=0$, from which we conclude that $(a-t) f(t)=0$. Since $f(t) \neq 0$, we get $a=t$ so that $s=t f(t)$ and thus we see $f(-s)=f(-t f(t))=-t^{2}=-f(s)$ holds in this case also. This observation finishes the proof of (3). +By substituting $(x, y)=(s, t),(x, y)=(t,-s-t)$ and $(x, y)=(-s-t, s)$ into the given equation, +we obtain + +$$ +\begin{array}{r} +f(s f(s+t)))=f(t f(s))+s^{2} \\ +f(t f(-s))=f((-s-t) f(t))+t^{2} +\end{array} +$$ + +and + +$$ +f((-s-t) f(-t))=f(s f(-s-t))+(s+t)^{2} +$$ + +respectively. Using the fact that $f(-x)=-f(x)$ holds for all $x$ to rewrite the second and the third equation, and rearranging the terms, we obtain + +$$ +\begin{aligned} +f(t f(s))-f(s f(s+t)) & =-s^{2} \\ +f(t f(s))-f((s+t) f(t)) & =-t^{2} \\ +f((s+t) f(t))+f(s f(s+t)) & =(s+t)^{2} +\end{aligned} +$$ + +Adding up these three equations now yields $2 f(t f(s))=2 t s$, and therefore, we conclude that $f(t f(s))=t s$ holds for every pair of real numbers $s, t$. By fixing $s$ so that $f(s)=1$, we obtain $f(x)=s x$. In view of the given equation, we see that $s= \pm 1$. It is easy to check that both functions $f(x)=x$ and $f(x)=-x$ satisfy the given functional equation, so these are the desired solutions. + +Solution 2. As in Solution 1 we obtain (1), (2) and (3). +Now we prove that $f$ is injective. For this purpose, let us assume that $f(r)=f(s)$ for some $r \neq s$. Then, by (2) + +$$ +r^{2}=f(r f(r))=f(r f(s))=f((s-r) f(r))+r^{2} +$$ + +where the last statement follows from the given functional equation with $x=r$ and $y=s-r$. Hence, $h=(s-r) f(r)$ satisfies $f(h)=0$ which implies $h^{2}=f(h f(h))=f(0)=0$, i.e., $h=0$. Then, by $s \neq r$ we have $f(r)=0$ which implies $r=0$, and finally $f(s)=f(r)=f(0)=0$. Analogously, it follows that $s=0$ which gives the contradiction $r=s$. +To prove $|f(1)|=1$ we apply (2) with $t=1$ and also with $t=f(1)$ and obtain $f(f(1))=1$ and $(f(1))^{2}=f(f(1) \cdot f(f(1)))=f(f(1))=1$. +Now we choose $\eta \in\{-1,1\}$ with $f(1)=\eta$. Using that $f$ is odd and the given equation with $x=1, y=z$ (second equality) and with $x=-1, y=z+2$ (fourth equality) we obtain + +$$ +\begin{aligned} +& f(z)+2 \eta=\eta(f(z \eta)+2)=\eta(f(f(z+1))+1)=\eta(-f(-f(z+1))+1) \\ +& =-\eta f((z+2) f(-1))=-\eta f((z+2)(-\eta))=\eta f((z+2) \eta)=f(z+2) +\end{aligned} +$$ + +Hence, + +$$ +f(z+2 \eta)=\eta f(\eta z+2)=\eta(f(\eta z)+2 \eta)=f(z)+2 +$$ + +Using this argument twice we obtain + +$$ +f(z+4 \eta)=f(z+2 \eta)+2=f(z)+4 +$$ + +Substituting $z=2 f(x)$ we have + +$$ +f(2 f(x))+4=f(2 f(x)+4 \eta)=f(2 f(x+2)) +$$ + +where the last equality follows from (4). Applying the given functional equation we proceed to + +$$ +f(2 f(x+2))=f(x f(2))+4=f(2 \eta x)+4 +$$ + +where the last equality follows again from (4) with $z=0$, i.e., $f(2)=2 \eta$. Finally, $f(2 f(x))=$ $f(2 \eta x)$ and by injectivity of $f$ we get $2 f(x)=2 \eta x$ and hence the two solutions. + +## Combinatorics + +## C1 NZL (New Zealand) + +Consider 2009 cards, each having one gold side and one black side, lying in parallel on a long table. Initially all cards show their gold sides. Two players, standing by the same long side of the table, play a game with alternating moves. Each move consists of choosing a block of 50 consecutive cards, the leftmost of which is showing gold, and turning them all over, so those which showed gold now show black and vice versa. The last player who can make a legal move wins. +(a) Does the game necessarily end? +(b) Does there exist a winning strategy for the starting player? + +Solution. (a) We interpret a card showing black as the digit 0 and a card showing gold as the digit 1. Thus each position of the 2009 cards, read from left to right, corresponds bijectively to a nonnegative integer written in binary notation of 2009 digits, where leading zeros are allowed. Each move decreases this integer, so the game must end. +(b) We show that there is no winning strategy for the starting player. We label the cards from right to left by $1, \ldots, 2009$ and consider the set $S$ of cards with labels $50 i, i=1,2, \ldots, 40$. Let $g_{n}$ be the number of cards from $S$ showing gold after $n$ moves. Obviously, $g_{0}=40$. Moreover, $\left|g_{n}-g_{n+1}\right|=1$ as long as the play goes on. Thus, after an odd number of moves, the nonstarting player finds a card from $S$ showing gold and hence can make a move. Consequently, this player always wins. + +## C2 ROU (Romania) + +For any integer $n \geq 2$, let $N(n)$ be the maximal number of triples $\left(a_{i}, b_{i}, c_{i}\right), i=1, \ldots, N(n)$, consisting of nonnegative integers $a_{i}, b_{i}$ and $c_{i}$ such that the following two conditions are satisfied: +(1) $a_{i}+b_{i}+c_{i}=n$ for all $i=1, \ldots, N(n)$, +(2) If $i \neq j$, then $a_{i} \neq a_{j}, b_{i} \neq b_{j}$ and $c_{i} \neq c_{j}$. + +Determine $N(n)$ for all $n \geq 2$. + +Comment. The original problem was formulated for $m$-tuples instead for triples. The numbers $N(m, n)$ are then defined similarly to $N(n)$ in the case $m=3$. The numbers $N(3, n)$ and $N(n, n)$ should be determined. The case $m=3$ is the same as in the present problem. The upper bound for $N(n, n)$ can be proved by a simple generalization. The construction of a set of triples attaining the bound can be easily done by induction from $n$ to $n+2$. + +Solution. Let $n \geq 2$ be an integer and let $\left\{T_{1}, \ldots, T_{N}\right\}$ be any set of triples of nonnegative integers satisfying the conditions (1) and (2). Since the $a$-coordinates are pairwise distinct we have + +$$ +\sum_{i=1}^{N} a_{i} \geq \sum_{i=1}^{N}(i-1)=\frac{N(N-1)}{2} +$$ + +Analogously, + +$$ +\sum_{i=1}^{N} b_{i} \geq \frac{N(N-1)}{2} \quad \text { and } \quad \sum_{i=1}^{N} c_{i} \geq \frac{N(N-1)}{2} +$$ + +Summing these three inequalities and applying (1) yields + +$$ +3 \frac{N(N-1)}{2} \leq \sum_{i=1}^{N} a_{i}+\sum_{i=1}^{N} b_{i}+\sum_{i=1}^{N} c_{i}=\sum_{i=1}^{N}\left(a_{i}+b_{i}+c_{i}\right)=n N, +$$ + +hence $3 \frac{N-1}{2} \leq n$ and, consequently, + +$$ +N \leq\left\lfloor\frac{2 n}{3}\right\rfloor+1 +$$ + +By constructing examples, we show that this upper bound can be attained, so $N(n)=\left\lfloor\frac{2 n}{3}\right\rfloor+1$. + +We distinguish the cases $n=3 k-1, n=3 k$ and $n=3 k+1$ for $k \geq 1$ and present the extremal examples in form of a table. + +| $n=3 k-1$ | | | +| :---: | :---: | :---: | +| $\left\lfloor\frac{2 n}{3}\right\rfloor+1=2 k$ | | | +| $a_{i}$ | $b_{i}$ | $c_{i}$ | +| 0 | $k+1$ | $2 k-2$ | +| 1 | $k+2$ | $2 k-4$ | +| $\vdots$ | $\vdots$ | $\vdots$ | +| $k-1$ | $2 k$ | 0 | +| $k$ | 0 | $2 k-1$ | +| $k+1$ | 1 | $2 k-3$ | +| $\vdots$ | $\vdots$ | $\vdots$ | +| $2 k-1$ | $k-1$ | 1 | + + +| $n=3 k$ | | | +| :---: | :---: | :---: | +| $\rfloor+1=2 k+1$ | | | +| $a_{i}$ | $b_{i}$ | $c_{i}$ | +| 0 | $k$ | $2 k$ | +| 1 | $k+1$ | $2 k-2$ | +| $\vdots$ | $\vdots$ | $\vdots$ | +| $k$ | $2 k$ | 0 | +| $k+1$ | 0 | $2 k-1$ | +| $k+2$ | 1 | $2 k-3$ | +| $\vdots$ | $\vdots$ | $\vdots$ | +| $2 k$ | $k-1$ | 1 | + + +| $n=3 k+1$ | | | +| :---: | :---: | :---: | +| $\left\lfloor\left\lfloor\frac{2 n}{3}\right\rfloor+1=2 k+1\right.$ | | | +| $a_{i}$ | $b_{i}$ | $c_{i}$ | +| 0 | $k$ | $2 k+1$ | +| 1 | $k+1$ | $2 k-1$ | +| $\vdots$ | $\vdots$ | $\vdots$ | +| $k$ | $2 k$ | 1 | +| $k+1$ | 0 | $2 k$ | +| $k+2$ | 1 | $2 k-2$ | +| $\vdots$ | $\vdots$ | $\vdots$ | +| $2 k$ | $k-1$ | 2 | + +It can be easily seen that the conditions (1) and (2) are satisfied and that we indeed have $\left\lfloor\frac{2 n}{3}\right\rfloor+1$ triples in each case. + +Comment. A cute combinatorial model is given by an equilateral triangle, partitioned into $n^{2}$ congruent equilateral triangles by $n-1$ equidistant parallels to each of its three sides. Two chess-like bishops placed at any two vertices of the small triangles are said to menace one another if they lie on a same parallel. The problem is to determine the largest number of bishops that can be placed so that none menaces another. A bishop may be assigned three coordinates $a, b, c$, namely the numbers of sides of small triangles they are off each of the sides of the big triangle. It is readily seen that the sum of these coordinates is always $n$, therefore fulfilling the requirements. + +## C3 RUS (Russian Federation) + +Let $n$ be a positive integer. Given a sequence $\varepsilon_{1}, \ldots, \varepsilon_{n-1}$ with $\varepsilon_{i}=0$ or $\varepsilon_{i}=1$ for each $i=1, \ldots, n-1$, the sequences $a_{0}, \ldots, a_{n}$ and $b_{0}, \ldots, b_{n}$ are constructed by the following rules: + +$$ +\begin{gathered} +a_{0}=b_{0}=1, \quad a_{1}=b_{1}=7, \\ +a_{i+1}=\left\{\begin{array}{ll} +2 a_{i-1}+3 a_{i}, & \text { if } \varepsilon_{i}=0, \\ +3 a_{i-1}+a_{i}, & \text { if } \varepsilon_{i}=1, +\end{array} \text { for each } i=1, \ldots, n-1,\right. \\ +b_{i+1}=\left\{\begin{array}{ll} +2 b_{i-1}+3 b_{i}, & \text { if } \varepsilon_{n-i}=0, \\ +3 b_{i-1}+b_{i}, & \text { if } \varepsilon_{n-i}=1, +\end{array} \text { for each } i=1, \ldots, n-1 .\right. +\end{gathered} +$$ + +Prove that $a_{n}=b_{n}$. + +Solution. For a binary word $w=\sigma_{1} \ldots \sigma_{n}$ of length $n$ and a letter $\sigma \in\{0,1\}$ let $w \sigma=$ $\sigma_{1} \ldots \sigma_{n} \sigma$ and $\sigma w=\sigma \sigma_{1} \ldots \sigma_{n}$. Moreover let $\bar{w}=\sigma_{n} \ldots \sigma_{1}$ and let $\emptyset$ be the empty word (of length 0 and with $\bar{\emptyset}=\emptyset$ ). Let $(u, v)$ be a pair of two real numbers. For binary words $w$ we define recursively the numbers $(u, v)^{w}$ as follows: + +$$ +\begin{gathered} +(u, v)^{\emptyset}=v, \quad(u, v)^{0}=2 u+3 v, \quad(u, v)^{1}=3 u+v, \\ +(u, v)^{w \sigma \varepsilon}= \begin{cases}3(u, v)^{w}+3(u, v)^{w \sigma}, & \text { if } \varepsilon=0, \\ +3(u, v)^{w}+(u, v)^{w \sigma}, & \text { if } \varepsilon=1\end{cases} +\end{gathered} +$$ + +It easily follows by induction on the length of $w$ that for all real numbers $u_{1}, v_{1}, u_{2}, v_{2}, \lambda_{1}$ and $\lambda_{2}$ + +$$ +\left(\lambda_{1} u_{1}+\lambda_{2} u_{2}, \lambda_{1} v_{1}+\lambda_{2} v_{2}\right)^{w}=\lambda_{1}\left(u_{1}, v_{1}\right)^{w}+\lambda_{2}\left(u_{2}, v_{2}\right)^{w} +$$ + +and that for $\varepsilon \in\{0,1\}$ + +$$ +(u, v)^{\varepsilon w}=\left(v,(u, v)^{\varepsilon}\right)^{w} . +$$ + +Obviously, for $n \geq 1$ and $w=\varepsilon_{1} \ldots \varepsilon_{n-1}$, we have $a_{n}=(1,7)^{w}$ and $b_{n}=(1,7)^{\bar{w}}$. Thus it is sufficient to prove that + +$$ +(1,7)^{w}=(1,7)^{\bar{w}} +$$ + +for each binary word $w$. We proceed by induction on the length of $w$. The assertion is obvious if $w$ has length 0 or 1 . Now let $w \sigma \varepsilon$ be a binary word of length $n \geq 2$ and suppose that the assertion is true for all binary words of length at most $n-1$. +Note that $(2,1)^{\sigma}=7=(1,7)^{\emptyset}$ for $\sigma \in\{0,1\},(1,7)^{0}=23$, and $(1,7)^{1}=10$. +First let $\varepsilon=0$. Then in view of the induction hypothesis and the equalities (1) and (2), we obtain + +$$ +\begin{aligned} +&(1,7)^{w \sigma 0}=2(1,7)^{w}+3(1,7)^{w \sigma}=2(1,7)^{\bar{w}}+3(1,7)^{\sigma \bar{w}}=2(2,1)^{\sigma \bar{w}}+3(1,7)^{\sigma \bar{w}} \\ +&=(7,23)^{\sigma \bar{w}}=(1,7)^{0 \sigma \bar{w}} +\end{aligned} +$$ + +Now let $\varepsilon=1$. Analogously, we obtain + +$$ +\begin{aligned} +&(1,7)^{w \sigma 1}=3(1,7)^{w}+(1,7)^{w \sigma}=3(1,7)^{\bar{w}}+(1,7)^{\sigma \bar{w}}=3(2,1)^{\sigma \bar{w}}+(1,7)^{\sigma \bar{w}} \\ +&=(7,10)^{\sigma \bar{w}}=(1,7)^{1 \sigma \bar{w}} +\end{aligned} +$$ + +Thus the induction step is complete, (3) and hence also $a_{n}=b_{n}$ are proved. + +Comment. The original solution uses the relation + +$$ +(1,7)^{\alpha \beta w}=\left((1,7)^{w},(1,7)^{\beta w}\right)^{\alpha}, \quad \alpha, \beta \in\{0,1\} +$$ + +which can be proved by induction on the length of $w$. Then (3) also follows by induction on the length of $w$ : + +$$ +(1,7)^{\alpha \beta w}=\left((1,7)^{w},(1,7)^{\beta w}\right)^{\alpha}=\left((1,7)^{\bar{w}},(1,7)^{\bar{w} \beta}\right)^{\alpha}=(1,7)^{\bar{w} \beta \alpha} . +$$ + +Here $w$ may be the empty word. + +## C4 NLD (Netherlands) + +For an integer $m \geq 1$, we consider partitions of a $2^{m} \times 2^{m}$ chessboard into rectangles consisting of cells of the chessboard, in which each of the $2^{m}$ cells along one diagonal forms a separate rectangle of side length 1 . Determine the smallest possible sum of rectangle perimeters in such a partition. + +Solution 1. For a $k \times k$ chessboard, we introduce in a standard way coordinates of the vertices of the cells and assume that the cell $C_{i j}$ in row $i$ and column $j$ has vertices $(i-1, j-1),(i-$ $1, j),(i, j-1),(i, j)$, where $i, j \in\{1, \ldots, k\}$. Without loss of generality assume that the cells $C_{i i}$, $i=1, \ldots, k$, form a separate rectangle. Then we may consider the boards $B_{k}=\bigcup_{1 \leq i0$, JEnSEN's inequality immediately shows that the minimum of the right hand sight of (1) is attained for $i=k / 2$. Hence the total perimeter of the optimal partition of $B_{k}$ is at least $2 k+2 k / 2 \log _{2} k / 2+2(k / 2) \log _{2}(k / 2)=D_{k}$. + +Solution 2. We start as in Solution 1 and present another proof that $m 2^{m+1}$ is a lower bound for the total perimeter of a partition of $B_{2^{m}}$ into $n$ rectangles. Let briefly $M=2^{m}$. For $1 \leq i \leq M$, let $r_{i}$ denote the number of rectangles in the partition that cover some cell from row $i$ and let $c_{j}$ be the number of rectangles that cover some cell from column $j$. Note that the total perimeter $p$ of all rectangles in the partition is + +$$ +p=2\left(\sum_{i=1}^{M} r_{i}+\sum_{i=1}^{M} c_{i}\right) . +$$ + +No rectangle can simultaneously cover cells from row $i$ and from column $i$ since otherwise it would also cover the cell $C_{i i}$. We classify subsets $S$ of rectangles of the partition as follows. We say that $S$ is of type $i, 1 \leq i \leq M$, if $S$ contains all $r_{i}$ rectangles that cover some cell from row $i$, but none of the $c_{i}$ rectangles that cover some cell from column $i$. Altogether there are $2^{n-r_{i}-c_{i}}$ subsets of type $i$. Now we show that no subset $S$ can be simultaneously of type $i$ and of type $j$ if $i \neq j$. Assume the contrary and let without loss of generality $i1$. By condition $\left(2^{\prime}\right)$, after the Stepmother has distributed her water we have $y_{0}+y_{1}+y_{2}+y_{3}+y_{4} \leq \frac{5}{2}$. Therefore, + +$$ +\left(y_{0}+y_{2}\right)+\left(y_{1}+y_{3}\right)+\left(y_{2}+y_{4}\right)+\left(y_{3}+y_{0}\right)+\left(y_{4}+y_{1}\right)=2\left(y_{0}+y_{1}+y_{2}+y_{3}+y_{4}\right) \leq 5 +$$ + +and hence there is a pair of non-neighboring buckets which is not critical, say $\left(B_{0}, B_{2}\right)$. Now, if both of the pairs $\left(B_{3}, B_{0}\right)$ and $\left(B_{2}, B_{4}\right)$ are critical, we must have $y_{1}<\frac{1}{2}$ and Cinderella can empty the buckets $B_{3}$ and $B_{4}$. This clearly leaves no critical pair of buckets and the total contents of all the buckets is then $y_{1}+\left(y_{0}+y_{2}\right) \leq \frac{3}{2}$. Therefore, conditions $\left(1^{\prime}\right)$ and $\left(2^{\prime}\right)$ are fulfilled. + +Now suppose that without loss of generality the pair $\left(B_{3}, B_{0}\right)$ is not critical. If in this case $y_{0} \leq \frac{1}{2}$, then one of the inequalities $y_{0}+y_{1}+y_{2} \leq \frac{3}{2}$ and $y_{0}+y_{3}+y_{4} \leq \frac{3}{2}$ must hold. But then Cinderella can empty $B_{3}$ and $B_{4}$ or $B_{1}$ and $B_{2}$, respectively and clearly fulfill the conditions. +Finally consider the case $y_{0}>\frac{1}{2}$. By $y_{0}+y_{1}+y_{2}+y_{3}+y_{4} \leq \frac{5}{2}$, at least one of the pairs $\left(B_{1}, B_{3}\right)$ and $\left(B_{2}, B_{4}\right)$ is not critical. Without loss of generality let this be the pair $\left(B_{1}, B_{3}\right)$. Since the pair $\left(B_{3}, B_{0}\right)$ is not critical and $y_{0}>\frac{1}{2}$, we must have $y_{3} \leq \frac{1}{2}$. But then, as before, Cinderella can maintain the two conditions at the beginning of the next round by either emptying $B_{1}$ and $B_{2}$ or $B_{4}$ and $B_{0}$. + +Comments on GREEDY approaches. A natural approach for Cinderella would be a GREEDY strategy as for example: Always remove as much water as possible from the system. It is straightforward to prove that GREEDY can avoid buckets of capacity $\frac{5}{2}$ from overflowing: If before the Stepmothers move one has $x_{0}+x_{1}+x_{2}+x_{3}+x_{4} \leq \frac{3}{2}$ then after her move the inequality $Y=y_{0}+y_{1}+y_{2}+y_{3}+y_{4} \leq \frac{5}{2}$ holds. If now Cinderella removes the two adjacent buckets with maximum total contents she removes at least $\frac{2 Y}{5}$ and thus the remaining buckets contain at most $\frac{3}{5} \cdot Y \leq \frac{3}{2}$. +But GREEDY is in general not strong enough to settle this problem as can be seen in the following example: + +- In an initial phase, the Stepmother brings all the buckets (after her move) to contents of at least $\frac{1}{2}-2 \epsilon$, where $\epsilon$ is an arbitrary small positive number. This can be done by always splitting the 1 liter she has to distribute so that all buckets have the same contents. After her $r$-th move the total contents of each of the buckets is then $c_{r}$ with $c_{1}=1$ and $c_{r+1}=1+\frac{3}{5} \cdot c_{r}$ and hence $c_{r}=\frac{5}{2}-\frac{3}{2} \cdot\left(\frac{3}{5}\right)^{r-1}$. So the contents of each single bucket indeed approaches $\frac{1}{2}$ (from below). In particular, any two adjacent buckets have total contents strictly less than 1 which enables the Stepmother to always refill the buckets that Cinderella just emptied and then distribute the remaining water evenly over all buckets. +- After that phase GREEDY faces a situation like this ( $\left.\frac{1}{2}-2 \epsilon, \frac{1}{2}-2 \epsilon, \frac{1}{2}-2 \epsilon, \frac{1}{2}-2 \epsilon, \frac{1}{2}-2 \epsilon\right)$ and leaves a situation of the form $\left(x_{0}, x_{1}, x_{2}, x_{3}, x_{4}\right)=\left(\frac{1}{2}-2 \epsilon, \frac{1}{2}-2 \epsilon, \frac{1}{2}-2 \epsilon, 0,0\right)$. +- Then the Stepmother can add the amounts $\left(0, \frac{1}{4}+\epsilon, \epsilon, \frac{3}{4}-2 \epsilon, 0\right)$ to achieve a situation like this: $\left(y_{0}, y_{1}, y_{2}, y_{3}, y_{4}\right)=\left(\frac{1}{2}-2 \epsilon, \frac{3}{4}-\epsilon, \frac{1}{2}-\epsilon, \frac{3}{4}-2 \epsilon, 0\right)$. +- Now $B_{1}$ and $B_{2}$ are the adjacent buckets with the maximum total contents and thus GREEDY empties them to yield $\left(x_{0}, x_{1}, x_{2}, x_{3}, x_{4}\right)=\left(\frac{1}{2}-2 \epsilon, 0,0, \frac{3}{4}-2 \epsilon, 0\right)$. +- Then the Stepmother adds $\left(\frac{5}{8}, 0,0, \frac{3}{8}, 0\right)$, which yields $\left(\frac{9}{8}-2 \epsilon, 0,0, \frac{9}{8}-2 \epsilon, 0\right)$. +- Now GREEDY can only empty one of the two nonempty buckets and in the next step the Stepmother adds her liter to the other bucket and brings it to $\frac{17}{8}-2 \epsilon$, i.e. an overflow. + +A harder variant. Five identical empty buckets of capacity $b$ stand at the vertices of a regular pentagon. Cinderella and her wicked Stepmother go through a sequence of rounds: At the beginning of every round, the Stepmother takes one liter of water from the nearby river and distributes it arbitrarily over the five buckets. Then Cinderella chooses a pair of neighboring buckets, empties them into the river, and puts them back. Then the next round begins. The Stepmother's goal is to make one of these buckets overflow. Cinderella's goal is to prevent this. Determine all bucket capacities $b$ for which the Stepmother can enforce a bucket to overflow. + +Solution to the harder variant. The answer is $b<2$. +The previous proof shows that for all $b \geq 2$ the Stepmother cannot enforce overflowing. Now if $b<2$, let $R$ be a positive integer such that $b<2-2^{1-R}$. In the first $R$ rounds the Stepmother now ensures that at least one of the (nonadjacent) buckets $B_{1}$ and $B_{3}$ have contents of at least $1-2^{1-r}$ at the beginning of round $r(r=1,2, \ldots, R)$. This is trivial for $r=1$ and if it holds at the beginning of round $r$, she can fill the bucket which contains at least $1-2^{1-r}$ liters with another $2^{-r}$ liters and put the rest of her water $-1-2^{-r}$ liters - in the other bucket. As Cinderella now can remove the water of at most one of the two buckets, the other bucket carries its contents into the next round. +At the beginning of the $R$-th round there are $1-2^{1-R}$ liters in $B_{1}$ or $B_{3}$. The Stepmother puts the entire liter into that bucket and produces an overflow since $b<2-2^{1-R}$. + +## C6 BGR (Bulgaria) + +On a $999 \times 999$ board a limp rook can move in the following way: From any square it can move to any of its adjacent squares, i.e. a square having a common side with it, and every move must be a turn, i.e. the directions of any two consecutive moves must be perpendicular. A nonintersecting route of the limp rook consists of a sequence of pairwise different squares that the limp rook can visit in that order by an admissible sequence of moves. Such a non-intersecting route is called cyclic, if the limp rook can, after reaching the last square of the route, move directly to the first square of the route and start over. +How many squares does the longest possible cyclic, non-intersecting route of a limp rook visit? + +Solution. The answer is $998^{2}-4=4 \cdot\left(499^{2}-1\right)$ squares. +First we show that this number is an upper bound for the number of cells a limp rook can visit. To do this we color the cells with four colors $A, B, C$ and $D$ in the following way: for $(i, j) \equiv(0,0) \bmod 2$ use $A$, for $(i, j) \equiv(0,1) \bmod 2$ use $B$, for $(i, j) \equiv(1,0) \bmod 2$ use $C$ and for $(i, j) \equiv(1,1) \bmod 2$ use $D$. From an $A$-cell the rook has to move to a $B$-cell or a $C$-cell. In the first case, the order of the colors of the cells visited is given by $A, B, D, C, A, B, D, C, A, \ldots$, in the second case it is $A, C, D, B, A, C, D, B, A, \ldots$ Since the route is closed it must contain the same number of cells of each color. There are only $499^{2} A$-cells. In the following we will show that the rook cannot visit all the $A$-cells on its route and hence the maximum possible number of cells in a route is $4 \cdot\left(499^{2}-1\right)$. +Assume that the route passes through every single $A$-cell. Color the $A$-cells in black and white in a chessboard manner, i.e. color any two $A$-cells at distance 2 in different color. Since the number of $A$-cells is odd the rook cannot always alternate between visiting black and white $A$-cells along its route. Hence there are two $A$-cells of the same color which are four rook-steps apart that are visited directly one after the other. Let these two $A$-cells have row and column numbers $(a, b)$ and $(a+2, b+2)$ respectively. +![](https://cdn.mathpix.com/cropped/2024_11_18_b191c9407e8cb3d672cbg-38.jpg?height=449&width=524&top_left_y=1883&top_left_x=766) + +There is up to reflection only one way the rook can take from $(a, b)$ to $(a+2, b+2)$. Let this way be $(a, b) \rightarrow(a, b+1) \rightarrow(a+1, b+1) \rightarrow(a+1, b+2) \rightarrow(a+2, b+2)$. Also let without loss of generality the color of the cell $(a, b+1)$ be $B$ (otherwise change the roles of columns and rows). +Now consider the $A$-cell $(a, b+2)$. The only way the rook can pass through it is via $(a-1, b+2) \rightarrow$ $(a, b+2) \rightarrow(a, b+3)$ in this order, since according to our assumption after every $A$-cell the rook passes through a $B$-cell. Hence, to connect these two parts of the path, there must be +a path connecting the cell $(a, b+3)$ and $(a, b)$ and also a path connecting $(a+2, b+2)$ and $(a-1, b+2)$. + +But these four cells are opposite vertices of a convex quadrilateral and the paths are outside of that quadrilateral and hence they must intersect. This is due to the following fact: +The path from $(a, b)$ to $(a, b+3)$ together with the line segment joining these two cells form a closed loop that has one of the cells $(a-1, b+2)$ and $(a+2, b+2)$ in its inside and the other one on the outside. Thus the path between these two points must cross the previous path. +But an intersection is only possible if a cell is visited twice. This is a contradiction. +Hence the number of cells visited is at most $4 \cdot\left(499^{2}-1\right)$. +The following picture indicates a recursive construction for all $n \times n$-chessboards with $n \equiv 3$ $\bmod 4$ which clearly yields a path that misses exactly one $A$-cell (marked with a dot, the center cell of the $15 \times 15$-chessboard) and hence, in the case of $n=999$ crosses exactly $4 \cdot\left(499^{2}-1\right)$ cells. +![](https://cdn.mathpix.com/cropped/2024_11_18_b191c9407e8cb3d672cbg-39.jpg?height=686&width=715&top_left_y=979&top_left_x=676) + +## C7 RUS (Russian Federation) + +Variant 1. A grasshopper jumps along the real axis. He starts at point 0 and makes 2009 jumps to the right with lengths $1,2, \ldots, 2009$ in an arbitrary order. Let $M$ be a set of 2008 positive integers less than $1005 \cdot 2009$. Prove that the grasshopper can arrange his jumps in such a way that he never lands on a point from $M$. + +Variant 2. Let $n$ be a nonnegative integer. A grasshopper jumps along the real axis. He starts at point 0 and makes $n+1$ jumps to the right with pairwise different positive integral lengths $a_{1}, a_{2}, \ldots, a_{n+1}$ in an arbitrary order. Let $M$ be a set of $n$ positive integers in the interval $(0, s)$, where $s=a_{1}+a_{2}+\cdots+a_{n+1}$. Prove that the grasshopper can arrange his jumps in such a way that he never lands on a point from $M$. + +Solution of Variant 1. We construct the set of landing points of the grasshopper. +Case 1. $M$ does not contain numbers divisible by 2009. +We fix the numbers $2009 k$ as landing points, $k=1,2, \ldots, 1005$. Consider the open intervals $I_{k}=(2009(k-1), 2009 k), k=1,2, \ldots, 1005$. We show that we can choose exactly one point outside of $M$ as a landing point in 1004 of these intervals such that all lengths from 1 to 2009 are realized. Since there remains one interval without a chosen point, the length 2009 indeed will appear. Each interval has length 2009, hence a new landing point in an interval yields with a length $d$ also the length $2009-d$. Thus it is enough to implement only the lengths from $D=\{1,2, \ldots, 1004\}$. We will do this in a greedy way. Let $n_{k}, k=1,2, \ldots, 1005$, be the number of elements of $M$ that belong to the interval $I_{k}$. We order these numbers in a decreasing way, so let $p_{1}, p_{2}, \ldots, p_{1005}$ be a permutation of $\{1,2, \ldots, 1005\}$ such that $n_{p_{1}} \geq n_{p_{2}} \geq \cdots \geq n_{p_{1005}}$. In $I_{p_{1}}$ we do not choose a landing point. Assume that landing points have already been chosen in the intervals $I_{p_{2}}, \ldots, I_{p_{m}}$ and the lengths $d_{2}, \ldots, d_{m}$ from $D$ are realized, $m=1, \ldots, 1004$. We show that there is some $d \in D \backslash\left\{d_{2}, \ldots, d_{m}\right\}$ that can be implemented with a new landing point in $I_{p_{m+1}}$. Assume the contrary. Then the $1004-(m-1)$ other lengths are obstructed by the $n_{p_{m+1}}$ points of $M$ in $I_{p_{m+1}}$. Each length $d$ can be realized by two landing points, namely $2009\left(p_{m+1}-1\right)+d$ and $2009 p_{m+1}-d$, hence + +$$ +n_{p_{m+1}} \geq 2(1005-m) +$$ + +Moreover, since $|M|=2008=n_{1}+\cdots+n_{1005}$, + +$$ +2008 \geq n_{p_{1}}+n_{p_{2}}+\cdots+n_{p_{m+1}} \geq(m+1) n_{p_{m+1}} +$$ + +Consequently, by (1) and (2), + +$$ +2008 \geq 2(m+1)(1005-m) +$$ + +The right hand side of the last inequality obviously attains its minimum for $m=1004$ and this minimum value is greater than 2008, a contradiction. +Case 2. $M$ does contain a number $\mu$ divisible by 2009. +By the pigeonhole principle there exists some $r \in\{1, \ldots, 2008\}$ such that $M$ does not contain numbers with remainder $r$ modulo 2009. We fix the numbers $2009(k-1)+r$ as landing points, $k=1,2, \ldots, 1005$. Moreover, $1005 \cdot 2009$ is a landing point. Consider the open intervals +$I_{k}=(2009(k-1)+r, 2009 k+r), k=1,2, \ldots, 1004$. Analogously to Case 1 , it is enough to show that we can choose in 1003 of these intervals exactly one landing point outside of $M \backslash\{\mu\}$ such that each of the lengths of $D=\{1,2, \ldots, 1004\} \backslash\{r\}$ are implemented. Note that $r$ and $2009-r$ are realized by the first and last jump and that choosing $\mu$ would realize these two differences again. Let $n_{k}, k=1,2, \ldots, 1004$, be the number of elements of $M \backslash\{\mu\}$ that belong to the interval $I_{k}$ and $p_{1}, p_{2}, \ldots, p_{1004}$ be a permutation of $\{1,2, \ldots, 1004\}$ such that $n_{p_{1}} \geq n_{p_{2}} \geq \cdots \geq n_{p_{1004}}$. With the same reasoning as in Case 1 we can verify that a greedy choice of the landing points in $I_{p_{2}}, I_{p_{3}}, \ldots, I_{p_{1004}}$ is possible. We only have to replace (1) by + +$$ +n_{p_{m+1}} \geq 2(1004-m) +$$ + +( $D$ has one element less) and (2) by + +$$ +2007 \geq n_{p_{1}}+n_{p_{2}}+\cdots+n_{p_{m+1}} \geq(m+1) n_{p_{m+1}} +$$ + +Comment. The cardinality 2008 of $M$ in the problem is the maximum possible value. For $M=\{1,2, \ldots, 2009\}$, the grasshopper necessarily lands on a point from $M$. + +Solution of Variant 2. First of all we remark that the statement in the problem implies a strengthening of itself: Instead of $|M|=n$ it is sufficient to suppose that $|M \cap(0, s-\bar{a}]| \leq n$, where $\bar{a}=\min \left\{a_{1}, a_{2}, \ldots, a_{n+1}\right\}$. This fact will be used in the proof. +We prove the statement by induction on $n$. The case $n=0$ is obvious. Let $n>0$ and let the assertion be true for all nonnegative integers less than $n$. Moreover let $a_{1}, a_{2}, \ldots, a_{n+1}, s$ and $M$ be given as in the problem. Without loss of generality we may assume that $a_{n+1}1$. If $T_{\bar{k}-1} \in M$, then (4.) follows immediately by the minimality of $\bar{k}$. If $T_{\bar{k}-1} \notin M$, by the smoothness of $\bar{k}-1$, we obtain a situation as in Claim 1 with $m=\bar{k}-1$ provided that $\mid M \cap\left(0, T_{\bar{k}-1}| | \geq \bar{k}-1\right.$. Hence, we may even restrict ourselves to $\mid M \cap\left(0, T_{\bar{k}-1} \mid \leq \bar{k}-2\right.$ in this case and Claim 3 is proved. +Choose an integer $v \geq 0$ with $\left|M \cap\left(0, T_{\bar{k}}\right)\right|=\bar{k}+v$. Let $r_{1}>r_{2}>\cdots>r_{l}$ be exactly those indices $r$ from $\{\bar{k}+1, \bar{k}+2, \ldots, n+1\}$ for which $T_{\bar{k}}+a_{r} \notin M$. Then + +$$ +n=|M|=\left|M \cap\left(0, T_{\bar{k}}\right)\right|+1+\left|M \cap\left(T_{\bar{k}}, s\right)\right| \geq \bar{k}+v+1+(n+1-\bar{k}-l) +$$ + +and consequently $l \geq v+2$. Note that +$T_{\bar{k}}+a_{r_{1}}-a_{1}f_{i}(\varepsilon)$ for all $i=0, \ldots, k$. +We will show the essential fact: +Fact 3. $f_{0}(n)>f_{0}(h(n))$. +Then the empty string will necessarily be reached after a finite number of applications of $h$. But starting from a string without leading zeros, $\varepsilon$ can only be reached via the strings $1 \rightarrow 00 \rightarrow 0 \rightarrow \varepsilon$. Hence also the number 1 will appear after a finite number of applications of $h$. +Proof of Fact 3. If the last digit $r$ of $n$ is 0 , then we write $n=x_{0} 0 \ldots 0 x_{m-1} 0 \varepsilon$ where the $x_{i}$ do not contain the digit 0 . Then $h(n)=x_{0} 0 \ldots 0 x_{m-1}$ and $f_{0}(n)-f_{0}(h(n))=f_{0}(\varepsilon)>0$. +So let the last digit $r$ of $n$ be at least 1 . Let $L=y k$ and $R=z r$ be the corresponding left and right parts where $y$ is some string, $k \leq r-1$ and the string $z$ consists only of digits not less +than $r$. Then $n=y k z r$ and $h(n)=y k z(r-1) z(r-1)$. Let $d(y)$ be the smallest digit of $y$. We consider two cases which do not exclude each other. + +Case 1. $d(y) \geq k$. +Then + +$$ +f_{k}(n)-f_{k}(h(n))=f_{k}(z r)-f_{k}(z(r-1) z(r-1)) . +$$ + +In view of Fact 1 this difference is positive if and only if + +$$ +f_{r-1}(z r)-f_{r-1}(z(r-1) z(r-1))>0 +$$ + +We have, using Fact 2, + +$$ +f_{r-1}(z r)=4^{f_{r}(z r)}=4^{f_{r}(z)+4^{f_{r+1}(z)}} \geq 4 \cdot 4^{f_{r}(z)}>4^{f_{r}(z)}+4^{f_{r}(z)}+4^{f_{r}(z)}=f_{r-1}(z(r-1) z(r-1)) . +$$ + +Here we use the additional definition $f_{10}(\varepsilon)=0$ if $r=9$. Consequently, $f_{k}(n)-f_{k}(h(n))>0$ and according to Fact $1, f_{0}(n)-f_{0}(h(n))>0$. +Case 2. $d(y) \leq k$. +We prove by induction on $d(y)=k, k-1, \ldots, 0$ that $f_{i}(n)-f_{i}(h(n))>0$ for all $i=0, \ldots, d(y)$. By Fact 1, it suffices to do so for $i=d(y)$. The initialization $d(y)=k$ was already treated in Case 1. Let $t=d(y)0 +$$ + +Thus the inequality $f_{d(y)}(n)-f_{d(y)}(h(n))>0$ is established and from Fact 1 it follows that $f_{0}(n)-f_{0}(h(n))>0$. + +Solution 2. We identify integers $n \geq 2$ with the digit-strings, briefly strings, of their decimal representation and extend the definition of $h$ to all non-empty strings with digits from 0 to 9. Moreover, let us define that the empty string, $\varepsilon$, is being mapped to the empty string. In the following all functions map the set of strings into the set of strings. For two functions $f$ and $g$ let $g \circ f$ be defined by $(g \circ f)(x)=g(f(x))$ for all strings $x$ and let, for non-negative integers $n$, $f^{n}$ denote the $n$-fold application of $f$. For any string $x$ let $s(x)$ be the smallest digit of $x$, and for the empty string let $s(\varepsilon)=\infty$. We define nine functions $g_{1}, \ldots, g_{9}$ as follows: Let $k \in\{1, \ldots, 9\}$ and let $x$ be a string. If $x=\varepsilon$ then $g_{k}(x)=\varepsilon$. Otherwise, write $x$ in the form $x=y z r$ where $y$ is either the empty string or ends with a digit smaller than $k, s(z) \geq k$ and $r$ is the rightmost digit of $x$. Then $g_{k}(x)=z r$. + +Lemma 1. We have $g_{k} \circ h=g_{k} \circ h \circ g_{k}$ for all $k=1, \ldots, 9$. +Proof of Lemma 1. Let $x=y z r$ be as in the definition of $g_{k}$. If $y=\varepsilon$, then $g_{k}(x)=x$, whence + +$$ +g_{k}(h(x))=g_{k}\left(h\left(g_{k}(x)\right) .\right. +$$ + +So let $y \neq \varepsilon$. +Case 1. $z$ contains a digit smaller than $r$. +Let $z=u a v$ where $a0\end{cases} +$$ + +and + +$$ +h\left(g_{k}(x)\right)=h(z r)=h(\text { uavr })= \begin{cases}\operatorname{uav} & \text { if } r=0 \\ \operatorname{uav}(r-1) v(r-1) & \text { if } r>0\end{cases} +$$ + +Since $y$ ends with a digit smaller than $k$, (1) is obviously true. +Case 2. $z$ does not contain a digit smaller than $r$. +Let $y=u v$ where $u$ is either the empty string or ends with a digit smaller than $r$ and $s(v) \geq r$. We have + +$$ +h(x)= \begin{cases}u v z & \text { if } r=0 \\ u v z(r-1) v z(r-1) & \text { if } r>0\end{cases} +$$ + +and + +$$ +h\left(g_{k}(x)\right)=h(z r)= \begin{cases}z & \text { if } r=0 \\ z(r-1) z(r-1) & \text { if } r>0\end{cases} +$$ + +Recall that $y$ and hence $v$ ends with a digit smaller than $k$, but all digits of $v$ are at least $r$. Now if $r>k$, then $v=\varepsilon$, whence the terminal digit of $u$ is smaller than $k$, which entails + +$$ +g_{k}(h(x))=z(r-1) z(r-1)=g_{k}\left(h\left(g_{k}(x)\right)\right) . +$$ + +If $r \leq k$, then + +$$ +g_{k}(h(x))=z(r-1)=g_{k}\left(h\left(g_{k}(x)\right)\right), +$$ + +so that in both cases (1) is true. Thus Lemma 1 is proved. +Lemma 2. Let $k \in\{1, \ldots, 9\}$, let $x$ be a non-empty string and let $n$ be a positive integer. If $h^{n}(x)=\varepsilon$ then $\left(g_{k} \circ h\right)^{n}(x)=\varepsilon$. +Proof of Lemma 2. We proceed by induction on $n$. If $n=1$ we have + +$$ +\varepsilon=h(x)=g_{k}(h(x))=\left(g_{k} \circ h\right)(x) . +$$ + +Now consider the step from $n-1$ to $n$ where $n \geq 2$. Let $h^{n}(x)=\varepsilon$ and let $y=h(x)$. Then $h^{n-1}(y)=\varepsilon$ and by the induction hypothesis $\left(g_{k} \circ h\right)^{n-1}(y)=\varepsilon$. In view of Lemma 1 , + +$$ +\begin{aligned} +& \varepsilon=\left(g_{k} \circ h\right)^{n-2}\left(\left(g_{k} \circ h\right)(y)\right)=\left(g_{k} \circ h\right)^{n-2}\left(g_{k}(h(y))\right. \\ +&=\left(g_{k} \circ h\right)^{n-2}\left(g_{k}\left(h\left(g_{k}(y)\right)\right)=\left(g_{k} \circ h\right)^{n-2}\left(g_{k}\left(h\left(g_{k}(h(x))\right)\right)=\left(g_{k} \circ h\right)^{n}(x)\right.\right. +\end{aligned} +$$ + +Thus the induction step is complete and Lemma 2 is proved. +We say that the non-empty string $x$ terminates if $h^{n}(x)=\varepsilon$ for some non-negative integer $n$. +Lemma 3. Let $x=y z r$ where $s(y) \geq k, s(z) \geq k, y$ ends with the digit $k$ and $z$ is possibly empty. If $y$ and $z r$ terminate then also $x$ terminates. +Proof of Lemma 3. Suppose that $y$ and $z r$ terminate. We proceed by induction on $k$. Let $k=0$. Obviously, $h(y w)=y h(w)$ for any non-empty string $w$. Let $h^{n}(z r)=\epsilon$. It follows easily by induction on $m$ that $h^{m}(y z r)=y h^{m}(z r)$ for $m=1, \ldots, n$. Consequently, $h^{n}(y z r)=y$. Since $y$ terminates, also $x=y z r$ terminates. +Now let the assertion be true for all nonnegative integers less than $k$ and let us prove it for $k$ where $k \geq 1$. It turns out that it is sufficient to prove that $y g_{k}(h(z r))$ terminates. Indeed: +Case 1. $r=0$. +Then $h(y z r)=y z=y g_{k}(h(z r))$. +Case 2. $0k$. +Then $h(y z r)=y h(z r)=y g_{k}(h(z r))$. +Note that $y g_{k}(h(z r))$ has the form $y z^{\prime} r^{\prime}$ where $s\left(z^{\prime}\right) \geq k$. By the same arguments it is sufficient to prove that $y g_{k}\left(h\left(z^{\prime} r^{\prime}\right)\right)=y\left(g_{k} \circ h\right)^{2}(z r)$ terminates and, by induction, that $y\left(g_{k} \circ h\right)^{m}(z r)$ terminates for some positive integer $m$. In view of Lemma 2 there is some $m$ such that $\left(g_{k} \circ\right.$ $h)^{m}(z r)=\epsilon$, so $x=y z r$ terminates if $y$ terminates. Thus Lemma 3 is proved. +Now assume that there is some string $x$ that does not terminate. We choose $x$ minimal. If $x \geq 10$, we can write $x$ in the form $x=y z r$ of Lemma 3 and by this lemma $x$ terminates since $y$ and $z r$ are smaller than $x$. If $x \leq 9$, then $h(x)=(x-1)(x-1)$ and $h(x)$ terminates again by Lemma 3 and the minimal choice of $x$. + +Solution 3. We commence by introducing some terminology. Instead of integers, we will consider the set $S$ of all strings consisting of the digits $0,1, \ldots, 9$, including the empty string $\epsilon$. If $\left(a_{1}, a_{2}, \ldots, a_{n}\right)$ is a nonempty string, we let $\rho(a)=a_{n}$ denote the terminal digit of $a$ and $\lambda(a)$ be the string with the last digit removed. We also define $\lambda(\epsilon)=\epsilon$ and denote the set of non-negative integers by $\mathbb{N}_{0}$. +Now let $k \in\{0,1,2, \ldots, 9\}$ denote any digit. We define a function $f_{k}: S \longrightarrow S$ on the set of strings: First, if the terminal digit of $n$ belongs to $\{0,1, \ldots, k\}$, then $f_{k}(n)$ is obtained from $n$ by deleting this terminal digit, i.e $f_{k}(n)=\lambda(n)$. Secondly, if the terminal digit of $n$ belongs to $\{k+1, \ldots, 9\}$, then $f_{k}(n)$ is obtained from $n$ by the process described in the problem. We also define $f_{k}(\epsilon)=\epsilon$. Note that up to the definition for integers $n \leq 1$, the function $f_{0}$ coincides with the function $h$ in the problem, through interpreting integers as digit strings. The argument will be roughly as follows. We begin by introducing a straightforward generalization of our claim about $f_{0}$. Then it will be easy to see that $f_{9}$ has all these stronger properties, which means that is suffices to show for $k \in\{0,1, \ldots, 8\}$ that $f_{k}$ possesses these properties provided that $f_{k+1}$ does. +We continue to use $k$ to denote any digit. The operation $f_{k}$ is said to be separating, if the followings holds: Whenever $a$ is an initial segment of $b$, there is some $N \in \mathbb{N}_{0}$ such that $f_{k}^{N}(b)=a$. The following two notions only apply to the case where $f_{k}$ is indeed separating, otherwise they remain undefined. For every $a \in S$ we denote the least $N \in \mathbb{N}_{0}$ for which $f_{k}^{N}(a)=\epsilon$ occurs by $g_{k}(a)$ (because $\epsilon$ is an initial segment of $a$, such an $N$ exists if $f_{k}$ is separating). If for every two strings $a$ and $b$ such that $a$ is a terminal segment of $b$ one has $g_{k}(a) \leq g_{k}(b)$, we say that $f_{k}$ is coherent. In case that $f_{k}$ is separating and coherent we call the digit $k$ seductive. +As $f_{9}(a)=\lambda(a)$ for all $a$, it is obvious that 9 is seductive. Hence in order to show that 0 is seductive, which clearly implies the statement of the problem, it suffices to take any $k \in\{0,1, \ldots, 8\}$ such that $k+1$ is seductive and to prove that $k$ has to be seductive as well. Note that in doing so, we have the function $g_{k+1}$ at our disposal. We have to establish two things and we begin with + +Step 1. $f_{k}$ is separating. + +Before embarking on the proof of this, we record a useful observation which is easily proved by induction on $M$. + +Claim 1. For any strings $A, B$ and any positive integer $M$ such that $f_{k}^{M-1}(B) \neq \epsilon$, we have + +$$ +f_{k}^{M}(A k B)=A k f_{k}^{M}(B) +$$ + +Now we call a pair $(a, b)$ of strings wicked provided that $a$ is an initial segment of $b$, but there is no $N \in \mathbb{N}_{0}$ such that $f_{k}^{N}(b)=a$. We need to show that there are none, so assume that there were such pairs. Choose a wicked pair $(a, b)$ for which $g_{k+1}(b)$ attains its minimal possible value. Obviously $b \neq \epsilon$ for any wicked pair $(a, b)$. Let $z$ denote the terminal digit of $b$. Observe that $a \neq b$, which means that $a$ is also an initial segment of $\lambda(b)$. To facilitate the construction of the eventual contradiction, we prove +Claim 2. There cannot be an $N \in \mathbb{N}_{0}$ such that + +$$ +f_{k}^{N}(b)=\lambda(b) +$$ + +Proof of Claim 2. For suppose that such an $N$ existed. Because $g_{k+1}(\lambda(b))k+1$ is impossible: Set $B=f_{k}(b)$. Then also $f_{k+1}(b)=B$, but $g_{k+1}(B)g_{k}(b)$. Observe that if $f_{k}$ was incoherent, which we shall assume from now on, then such pairs existed. Now among all aggressive pairs we choose one, say $(a, b)$, for which $g_{k}(b)$ attains its least possible value. Obviously $f_{k}(a)$ cannot be injectible into $f_{k}(b)$, for otherwise the pair $\left(f_{k}(a), f_{k}(b)\right)$ was aggressive and contradicted our choice of $(a, b)$. Let $\left(A_{1}, A_{2}, \ldots, A_{m}\right)$ and $\left(B_{1}, B_{2}, \ldots, B_{n}\right)$ be the decompositions of $a$ and $b$ and take a function $H:\{1,2, \ldots, m\} \longrightarrow\{1,2, \ldots, n\}$ exemplifying that $a$ is indeed injectible into $b$. If we had $H(m)|O A B|$, which is contradictory to the choice of $A$ and $B$. If it is one of the lines $b, a^{\prime}$ or $b^{\prime}$ almost identical arguments lead to a similar contradiction. +Let $R_{2}$ be the parallelogram $A B A^{\prime} B^{\prime}$. Since $A$ and $B$ are points of $P$, segment $A B \subset P$ and so $R_{2} \subset R_{1}$. Since $A, B, A^{\prime}$ and $B^{\prime}$ are midpoints of the sides of $R_{1}$, an easy argument yields + +$$ +\left|R_{1}\right|=2 \cdot\left|R_{2}\right| +$$ + +Let $R_{3}$ be the smallest parallelogram enclosing $P$ defined by lines parallel to $A B$ and $B A^{\prime}$. Obviously $R_{2} \subset R_{3}$ and every side of $R_{3}$ contains at least one point of the boundary of $P$. Denote by $C$ the intersection point of $a$ and $b$, by $X$ the intersection point of $A B$ and $O C$, and by $X^{\prime}$ the intersection point of $X C$ and the boundary of $R_{3}$. In a similar way denote by $D$ +the intersection point of $b$ and $a^{\prime}$, by $Y$ the intersection point of $A^{\prime} B$ and $O D$, and by $Y^{\prime}$ the intersection point of $Y D$ and the boundary of $R_{3}$. +Note that $O C=2 \cdot O X$ and $O D=2 \cdot O Y$, so there exist real numbers $x$ and $y$ with $1 \leq x, y \leq 2$ and $O X^{\prime}=x \cdot O X$ and $O Y^{\prime}=y \cdot O Y$. Corresponding sides of $R_{3}$ and $R_{2}$ are parallel which yields + +$$ +\left|R_{3}\right|=x y \cdot\left|R_{2}\right| +$$ + +The side of $R_{3}$ containing $X^{\prime}$ contains at least one point $X^{*}$ of $P$; due to the convexity of $P$ we have $A X^{*} B \subset P$. Since this side of the parallelogram $R_{3}$ is parallel to $A B$ we have $\left|A X^{*} B\right|=\left|A X^{\prime} B\right|$, so $\left|O A X^{\prime} B\right|$ does not exceed the area of $P$ confined to the sector defined by the rays $O B$ and $O A$. In a similar way we conclude that $\left|O B^{\prime} Y^{\prime} A^{\prime}\right|$ does not exceed the area of $P$ confined to the sector defined by the rays $O B$ and $O A^{\prime}$. Putting things together we have $\left|O A X^{\prime} B\right|=x \cdot|O A B|,\left|O B D A^{\prime}\right|=y \cdot\left|O B A^{\prime}\right|$. Since $|O A B|=\left|O B A^{\prime}\right|$, we conclude that $|P| \geq 2 \cdot\left|A X^{\prime} B Y^{\prime} A^{\prime}\right|=2 \cdot\left(x \cdot|O A B|+y \cdot\left|O B A^{\prime}\right|\right)=4 \cdot \frac{x+y}{2} \cdot|O A B|=\frac{x+y}{2} \cdot R_{2}$; this is in short + +$$ +\frac{x+y}{2} \cdot\left|R_{2}\right| \leq|P| +$$ + +Since all numbers concerned are positive, we can combine (1)-(3). Using the arithmetic-geometric-mean inequality we obtain + +$$ +\left|R_{1}\right| \cdot\left|R_{3}\right|=2 \cdot\left|R_{2}\right| \cdot x y \cdot\left|R_{2}\right| \leq 2 \cdot\left|R_{2}\right|^{2}\left(\frac{x+y}{2}\right)^{2} \leq 2 \cdot|P|^{2} +$$ + +This implies immediately the desired result $\left|R_{1}\right| \leq \sqrt{2} \cdot|P|$ or $\left|R_{3}\right| \leq \sqrt{2} \cdot|P|$. + +Solution 2. We construct the parallelograms $R_{1}, R_{2}$ and $R_{3}$ in the same way as in Solution 1 and will show that $\frac{\left|R_{1}\right|}{|P|} \leq \sqrt{2}$ or $\frac{\left|R_{3}\right|}{|P|} \leq \sqrt{2}$. +![](https://cdn.mathpix.com/cropped/2024_11_18_b191c9407e8cb3d672cbg-60.jpg?height=569&width=580&top_left_y=1720&top_left_x=738) + +Figure 2 +Recall that affine one-to-one maps of the plane preserve the ratio of areas of subsets of the plane. On the other hand, every parallelogram can be transformed with an affine map onto a square. It follows that without loss of generality we may assume that $R_{1}$ is a square (see Figure 2). +Then $R_{2}$, whose vertices are the midpoints of the sides of $R_{1}$, is a square too, and $R_{3}$, whose sides are parallel to the diagonals of $R_{1}$, is a rectangle. +Let $a>0, b \geq 0$ and $c \geq 0$ be the distances introduced in Figure 2. Then $\left|R_{1}\right|=2 a^{2}$ and +$\left|R_{3}\right|=(a+2 b)(a+2 c)$. +Points $A, A^{\prime}, B$ and $B^{\prime}$ are in the convex polygon $P$. Hence the square $A B A^{\prime} B^{\prime}$ is a subset of $P$. Moreover, each of the sides of the rectangle $R_{3}$ contains a point of $P$, otherwise $R_{3}$ would not be minimal. It follows that + +$$ +|P| \geq a^{2}+2 \cdot \frac{a b}{2}+2 \cdot \frac{a c}{2}=a(a+b+c) +$$ + +Now assume that both $\frac{\left|R_{1}\right|}{|P|}>\sqrt{2}$ and $\frac{\left|R_{3}\right|}{|P|}>\sqrt{2}$, then + +$$ +2 a^{2}=\left|R_{1}\right|>\sqrt{2} \cdot|P| \geq \sqrt{2} \cdot a(a+b+c) +$$ + +and + +$$ +(a+2 b)(a+2 c)=\left|R_{3}\right|>\sqrt{2} \cdot|P| \geq \sqrt{2} \cdot a(a+b+c) +$$ + +All numbers concerned are positive, so after multiplying these inequalities we get + +$$ +2 a^{2}(a+2 b)(a+2 c)>2 a^{2}(a+b+c)^{2} +$$ + +But the arithmetic-geometric-mean inequality implies the contradictory result + +$$ +2 a^{2}(a+2 b)(a+2 c) \leq 2 a^{2}\left(\frac{(a+2 b)+(a+2 c)}{2}\right)^{2}=2 a^{2}(a+b+c)^{2} +$$ + +Hence $\frac{\left|R_{1}\right|}{|P|} \leq \sqrt{2}$ or $\frac{\left|R_{3}\right|}{|P|} \leq \sqrt{2}$, as desired. + +## G6 UKR (Ukraine) + +Let the sides $A D$ and $B C$ of the quadrilateral $A B C D$ (such that $A B$ is not parallel to $C D$ ) intersect at point $P$. Points $O_{1}$ and $O_{2}$ are the circumcenters and points $H_{1}$ and $H_{2}$ are the orthocenters of triangles $A B P$ and $D C P$, respectively. Denote the midpoints of segments $O_{1} H_{1}$ and $O_{2} H_{2}$ by $E_{1}$ and $E_{2}$, respectively. Prove that the perpendicular from $E_{1}$ on $C D$, the perpendicular from $E_{2}$ on $A B$ and the line $H_{1} H_{2}$ are concurrent. + +Solution 1. We keep triangle $A B P$ fixed and move the line $C D$ parallel to itself uniformly, i.e. linearly dependent on a single parameter $\lambda$ (see Figure 1). Then the points $C$ and $D$ also move uniformly. Hence, the points $O_{2}, H_{2}$ and $E_{2}$ move uniformly, too. Therefore also the perpendicular from $E_{2}$ on $A B$ moves uniformly. Obviously, the points $O_{1}, H_{1}, E_{1}$ and the perpendicular from $E_{1}$ on $C D$ do not move at all. Hence, the intersection point $S$ of these two perpendiculars moves uniformly. Since $H_{1}$ does not move, while $H_{2}$ and $S$ move uniformly along parallel lines (both are perpendicular to $C D$ ), it is sufficient to prove their collinearity for two different positions of $C D$. +![](https://cdn.mathpix.com/cropped/2024_11_18_b191c9407e8cb3d672cbg-62.jpg?height=897&width=895&top_left_y=1162&top_left_x=592) + +Figure 1 +Let $C D$ pass through either point $A$ or point $B$. Note that by hypothesis these two cases are different. We will consider the case $A \in C D$, i.e. $A=D$. So we have to show that the perpendiculars from $E_{1}$ on $A C$ and from $E_{2}$ on $A B$ intersect on the altitude $A H$ of triangle $A B C$ (see Figure 2). +![](https://cdn.mathpix.com/cropped/2024_11_18_b191c9407e8cb3d672cbg-63.jpg?height=971&width=749&top_left_y=197&top_left_x=659) + +Figure 2 + +To this end, we consider the midpoints $A_{1}, B_{1}, C_{1}$ of $B C, C A, A B$, respectively. As $E_{1}$ is the center of Feuerbach's circle (nine-point circle) of $\triangle A B P$, we have $E_{1} C_{1}=E_{1} H$. Similarly, $E_{2} B_{1}=E_{2} H$. Note further that a point $X$ lies on the perpendicular from $E_{1}$ on $A_{1} C_{1}$ if and only if + +$$ +X C_{1}^{2}-X A_{1}^{2}=E_{1} C_{1}^{2}-E_{1} A_{1}^{2} +$$ + +Similarly, the perpendicular from $E_{2}$ on $A_{1} B_{1}$ is characterized by + +$$ +X A_{1}^{2}-X B_{1}^{2}=E_{2} A_{1}^{2}-E_{2} B_{1}^{2} +$$ + +The line $H_{1} H_{2}$, which is perpendicular to $B_{1} C_{1}$ and contains $A$, is given by + +$$ +X B_{1}^{2}-X C_{1}^{2}=A B_{1}^{2}-A C_{1}^{2} +$$ + +The three lines are concurrent if and only if + +$$ +\begin{aligned} +0 & =X C_{1}^{2}-X A_{1}^{2}+X A_{1}^{2}-X B_{1}^{2}+X B_{1}^{2}-X C_{1}^{2} \\ +& =E_{1} C_{1}^{2}-E_{1} A_{1}^{2}+E_{2} A_{1}^{2}-E_{2} B_{1}^{2}+A B_{1}^{2}-A C_{1}^{2} \\ +& =-E_{1} A_{1}^{2}+E_{2} A_{1}^{2}+E_{1} H^{2}-E_{2} H^{2}+A B_{1}^{2}-A C_{1}^{2} +\end{aligned} +$$ + +i.e. it suffices to show that + +$$ +E_{1} A_{1}^{2}-E_{2} A_{1}^{2}-E_{1} H^{2}+E_{2} H^{2}=\frac{A C^{2}-A B^{2}}{4} +$$ + +We have + +$$ +\frac{A C^{2}-A B^{2}}{4}=\frac{H C^{2}-H B^{2}}{4}=\frac{(H C+H B)(H C-H B)}{4}=\frac{H A_{1} \cdot B C}{2} +$$ + +Let $F_{1}, F_{2}$ be the projections of $E_{1}, E_{2}$ on $B C$. Obviously, these are the midpoints of $H P_{1}$, +$H P_{2}$, where $P_{1}, P_{2}$ are the midpoints of $P B$ and $P C$ respectively. Then + +$$ +\begin{aligned} +& E_{1} A_{1}^{2}-E_{2} A_{1}^{2}-E_{1} H^{2}+E_{2} H^{2} \\ +& =F_{1} A_{1}^{2}-F_{1} H^{2}-F_{2} A_{1}^{2}+F_{2} H^{2} \\ +& =\left(F_{1} A_{1}-F_{1} H\right)\left(F_{1} A_{1}+F_{1} H\right)-\left(F_{2} A_{1}-F_{2} H\right)\left(F_{2} A_{1}+F_{2} H\right) \\ +& =A_{1} H \cdot\left(A_{1} P_{1}-A_{1} P_{2}\right) \\ +& =\frac{A_{1} H \cdot B C}{2} \\ +& =\frac{A C^{2}-A B^{2}}{4} +\end{aligned} +$$ + +which proves the claim. + +Solution 2. Let the perpendicular from $E_{1}$ on $C D$ meet $P H_{1}$ at $X$, and the perpendicular from $E_{2}$ on $A B$ meet $P H_{2}$ at $Y$ (see Figure 3). Let $\varphi$ be the intersection angle of $A B$ and $C D$. Denote by $M, N$ the midpoints of $P H_{1}, P H_{2}$ respectively. +![](https://cdn.mathpix.com/cropped/2024_11_18_b191c9407e8cb3d672cbg-64.jpg?height=754&width=1069&top_left_y=1185&top_left_x=499) + +Figure 3 +We will prove now that triangles $E_{1} X M$ and $E_{2} Y N$ have equal angles at $E_{1}, E_{2}$, and supplementary angles at $X, Y$. +In the following, angles are understood as oriented, and equalities of angles modulo $180^{\circ}$. +Let $\alpha=\angle H_{2} P D, \psi=\angle D P C, \beta=\angle C P H_{1}$. Then $\alpha+\psi+\beta=\varphi, \angle E_{1} X H_{1}=\angle H_{2} Y E_{2}=\varphi$, thus $\angle M X E_{1}+\angle N Y E_{2}=180^{\circ}$. +By considering the Feuerbach circle of $\triangle A B P$ whose center is $E_{1}$ and which goes through $M$, we have $\angle E_{1} M H_{1}=\psi+2 \beta$. Analogous considerations with the Feuerbach circle of $\triangle D C P$ yield $\angle H_{2} N E_{2}=\psi+2 \alpha$. Hence indeed $\angle X E_{1} M=\varphi-(\psi+2 \beta)=(\psi+2 \alpha)-\varphi=\angle Y E_{2} N$. +It follows now that + +$$ +\frac{X M}{M E_{1}}=\frac{Y N}{N E_{2}} +$$ + +Furthermore, $M E_{1}$ is half the circumradius of $\triangle A B P$, while $P H_{1}$ is the distance of $P$ to the orthocenter of that triangle, which is twice the circumradius times the cosine of $\psi$. Together +with analogous reasoning for $\triangle D C P$ we have + +$$ +\frac{M E_{1}}{P H_{1}}=\frac{1}{4 \cos \psi}=\frac{N E_{2}}{P H_{2}} +$$ + +By multiplication, + +$$ +\frac{X M}{P H_{1}}=\frac{Y N}{P H_{2}} +$$ + +and therefore + +$$ +\frac{P X}{X H_{1}}=\frac{H_{2} Y}{Y P} +$$ + +Let $E_{1} X, E_{2} Y$ meet $H_{1} H_{2}$ in $R, S$ respectively. +Applying the intercept theorem to the parallels $E_{1} X, P H_{2}$ and center $H_{1}$ gives + +$$ +\frac{H_{2} R}{R H_{1}}=\frac{P X}{X H_{1}} +$$ + +while with parallels $E_{2} Y, P H_{1}$ and center $H_{2}$ we obtain + +$$ +\frac{H_{2} S}{S H_{1}}=\frac{H_{2} Y}{Y P} +$$ + +Combination of the last three equalities yields that $R$ and $S$ coincide. + +## G7 IRN (Islamic Republic of Iran) + +Let $A B C$ be a triangle with incenter $I$ and let $X, Y$ and $Z$ be the incenters of the triangles $B I C, C I A$ and $A I B$, respectively. Let the triangle $X Y Z$ be equilateral. Prove that $A B C$ is equilateral too. + +Solution. $A Z, A I$ and $A Y$ divide $\angle B A C$ into four equal angles; denote them by $\alpha$. In the same way we have four equal angles $\beta$ at $B$ and four equal angles $\gamma$ at $C$. Obviously $\alpha+\beta+\gamma=\frac{180^{\circ}}{4}=45^{\circ}$; and $0^{\circ}<\alpha, \beta, \gamma<45^{\circ}$. +![](https://cdn.mathpix.com/cropped/2024_11_18_b191c9407e8cb3d672cbg-66.jpg?height=629&width=938&top_left_y=802&top_left_x=562) + +Easy calculations in various triangles yield $\angle B I C=180^{\circ}-2 \beta-2 \gamma=180^{\circ}-\left(90^{\circ}-2 \alpha\right)=$ $90^{\circ}+2 \alpha$, hence (for $X$ is the incenter of triangle $B C I$, so $I X$ bisects $\angle B I C$ ) we have $\angle X I C=$ $\angle B I X=\frac{1}{2} \angle B I C=45^{\circ}+\alpha$ and with similar aguments $\angle C I Y=\angle Y I A=45^{\circ}+\beta$ and $\angle A I Z=\angle Z I B=45^{\circ}+\gamma$. Furthermore, we have $\angle X I Y=\angle X I C+\angle C I Y=\left(45^{\circ}+\alpha\right)+$ $\left(45^{\circ}+\beta\right)=135^{\circ}-\gamma, \angle Y I Z=135^{\circ}-\alpha$, and $\angle Z I X=135^{\circ}-\beta$. +Now we calculate the lengths of $I X, I Y$ and $I Z$ in terms of $\alpha, \beta$ and $\gamma$. The perpendicular from $I$ on $C X$ has length $I X \cdot \sin \angle C X I=I X \cdot \sin \left(90^{\circ}+\beta\right)=I X \cdot \cos \beta$. But $C I$ bisects $\angle Y C X$, so the perpendicular from $I$ on $C Y$ has the same length, and we conclude + +$$ +I X \cdot \cos \beta=I Y \cdot \cos \alpha +$$ + +To make calculations easier we choose a length unit that makes $I X=\cos \alpha$. Then $I Y=\cos \beta$ and with similar arguments $I Z=\cos \gamma$. +Since $X Y Z$ is equilateral we have $Z X=Z Y$. The law of Cosines in triangles $X Y I, Y Z I$ yields + +$$ +\begin{aligned} +& Z X^{2}=Z Y^{2} \\ +\Longrightarrow & I Z^{2}+I X^{2}-2 \cdot I Z \cdot I X \cdot \cos \angle Z I X=I Z^{2}+I Y^{2}-2 \cdot I Z \cdot I Y \cdot \cos \angle Y I Z \\ +\Longrightarrow & I X^{2}-I Y^{2}=2 \cdot I Z \cdot(I X \cdot \cos \angle Z I X-I Y \cdot \cos \angle Y I Z) \\ +\Longrightarrow & \underbrace{\cos ^{2} \alpha-\cos ^{2} \beta}_{\text {L.H.S. }}=\underbrace{2 \cdot \cos \gamma \cdot\left(\cos \alpha \cdot \cos \left(135^{\circ}-\beta\right)-\cos \beta \cdot \cos \left(135^{\circ}-\alpha\right)\right)}_{\text {R.H.S. }} . +\end{aligned} +$$ + +A transformation of the left-hand side (L.H.S.) yields + +$$ +\begin{aligned} +\text { L.H.S. } & =\cos ^{2} \alpha \cdot\left(\sin ^{2} \beta+\cos ^{2} \beta\right)-\cos ^{2} \beta \cdot\left(\sin ^{2} \alpha+\cos ^{2} \alpha\right) \\ +& =\cos ^{2} \alpha \cdot \sin ^{2} \beta-\cos ^{2} \beta \cdot \sin ^{2} \alpha +\end{aligned} +$$ + +$$ +\begin{aligned} +& =(\cos \alpha \cdot \sin \beta+\cos \beta \cdot \sin \alpha) \cdot(\cos \alpha \cdot \sin \beta-\cos \beta \cdot \sin \alpha) \\ +& =\sin (\beta+\alpha) \cdot \sin (\beta-\alpha)=\sin \left(45^{\circ}-\gamma\right) \cdot \sin (\beta-\alpha) +\end{aligned} +$$ + +whereas a transformation of the right-hand side (R.H.S.) leads to + +$$ +\begin{aligned} +\text { R.H.S. } & =2 \cdot \cos \gamma \cdot\left(\cos \alpha \cdot\left(-\cos \left(45^{\circ}+\beta\right)\right)-\cos \beta \cdot\left(-\cos \left(45^{\circ}+\alpha\right)\right)\right) \\ +& =2 \cdot \frac{\sqrt{2}}{2} \cdot \cos \gamma \cdot(\cos \alpha \cdot(\sin \beta-\cos \beta)+\cos \beta \cdot(\cos \alpha-\sin \alpha)) \\ +& =\sqrt{2} \cdot \cos \gamma \cdot(\cos \alpha \cdot \sin \beta-\cos \beta \cdot \sin \alpha) \\ +& =\sqrt{2} \cdot \cos \gamma \cdot \sin (\beta-\alpha) +\end{aligned} +$$ + +Equating L.H.S. and R.H.S. we obtain + +$$ +\begin{aligned} +& \sin \left(45^{\circ}-\gamma\right) \cdot \sin (\beta-\alpha)=\sqrt{2} \cdot \cos \gamma \cdot \sin (\beta-\alpha) \\ +\Longrightarrow & \sin (\beta-\alpha) \cdot\left(\sqrt{2} \cdot \cos \gamma-\sin \left(45^{\circ}-\gamma\right)\right)=0 \\ +\Longrightarrow & \alpha=\beta \quad \text { or } \quad \sqrt{2} \cdot \cos \gamma=\sin \left(45^{\circ}-\gamma\right) . +\end{aligned} +$$ + +But $\gamma<45^{\circ}$; so $\sqrt{2} \cdot \cos \gamma>\cos \gamma>\cos 45^{\circ}=\sin 45^{\circ}>\sin \left(45^{\circ}-\gamma\right)$. This leaves $\alpha=\beta$. With similar reasoning we have $\alpha=\gamma$, which means triangle $A B C$ must be equilateral. + +## G8 BGR (Bulgaria) + +Let $A B C D$ be a circumscribed quadrilateral. Let $g$ be a line through $A$ which meets the segment $B C$ in $M$ and the line $C D$ in $N$. Denote by $I_{1}, I_{2}$, and $I_{3}$ the incenters of $\triangle A B M$, $\triangle M N C$, and $\triangle N D A$, respectively. Show that the orthocenter of $\triangle I_{1} I_{2} I_{3}$ lies on $g$. + +Solution 1. Let $k_{1}, k_{2}$ and $k_{3}$ be the incircles of triangles $A B M, M N C$, and $N D A$, respectively (see Figure 1). We shall show that the tangent $h$ from $C$ to $k_{1}$ which is different from $C B$ is also tangent to $k_{3}$. +![](https://cdn.mathpix.com/cropped/2024_11_18_b191c9407e8cb3d672cbg-68.jpg?height=926&width=1063&top_left_y=842&top_left_x=499) + +Figure 1 +To this end, let $X$ denote the point of intersection of $g$ and $h$. Then $A B C X$ and $A B C D$ are circumscribed quadrilaterals, whence + +$$ +C D-C X=(A B+C D)-(A B+C X)=(B C+A D)-(B C+A X)=A D-A X +$$ + +i.e. + +$$ +A X+C D=C X+A D +$$ + +which in turn reveals that the quadrilateral $A X C D$ is also circumscribed. Thus $h$ touches indeed the circle $k_{3}$. +Moreover, we find that $\angle I_{3} C I_{1}=\angle I_{3} C X+\angle X C I_{1}=\frac{1}{2}(\angle D C X+\angle X C B)=\frac{1}{2} \angle D C B=$ $\frac{1}{2}\left(180^{\circ}-\angle M C N\right)=180^{\circ}-\angle M I_{2} N=\angle I_{3} I_{2} I_{1}$, from which we conclude that $C, I_{1}, I_{2}, I_{3}$ are concyclic. +Let now $L_{1}$ and $L_{3}$ be the reflection points of $C$ with respect to the lines $I_{2} I_{3}$ and $I_{1} I_{2}$ respectively. Since $I_{1} I_{2}$ is the angle bisector of $\angle N M C$, it follows that $L_{3}$ lies on $g$. By analogous reasoning, $L_{1}$ lies on $g$. +Let $H$ be the orthocenter of $\triangle I_{1} I_{2} I_{3}$. We have $\angle I_{2} L_{3} I_{1}=\angle I_{1} C I_{2}=\angle I_{1} I_{3} I_{2}=180^{\circ}-\angle I_{1} H I_{2}$, which entails that the quadrilateral $I_{2} H I_{1} L_{3}$ is cyclic. Analogously, $I_{3} H L_{1} I_{2}$ is cyclic. + +Then, working with oriented angles modulo $180^{\circ}$, we have + +$$ +\angle L_{3} H I_{2}=\angle L_{3} I_{1} I_{2}=\angle I_{2} I_{1} C=\angle I_{2} I_{3} C=\angle L_{1} I_{3} I_{2}=\angle L_{1} H I_{2}, +$$ + +whence $L_{1}, L_{3}$, and $H$ are collinear. By $L_{1} \neq L_{3}$, the claim follows. + +Comment. The last part of the argument essentially reproves the following fact: The Simson line of a point $P$ lying on the circumcircle of a triangle $A B C$ with respect to that triangle bisects the line segment connecting $P$ with the orthocenter of $A B C$. + +Solution 2. We start by proving that $C, I_{1}, I_{2}$, and $I_{3}$ are concyclic. +![](https://cdn.mathpix.com/cropped/2024_11_18_b191c9407e8cb3d672cbg-69.jpg?height=797&width=1015&top_left_y=818&top_left_x=526) + +Figure 2 +To this end, notice first that $I_{2}, M, I_{1}$ are collinear, as are $N, I_{2}, I_{3}$ (see Figure 2). Denote by $\alpha, \beta, \gamma, \delta$ the internal angles of $A B C D$. By considerations in triangle $C M N$, it follows that $\angle I_{3} I_{2} I_{1}=\frac{\gamma}{2}$. We will show that $\angle I_{3} C I_{1}=\frac{\gamma}{2}$, too. Denote by $I$ the incenter of $A B C D$. Clearly, $I_{1} \in B I, I_{3} \in D I, \angle I_{1} A I_{3}=\frac{\alpha}{2}$. +Using the abbreviation $[X, Y Z]$ for the distance from point $X$ to the line $Y Z$, we have because of $\angle B A I_{1}=\angle I A I_{3}$ and $\angle I_{1} A I=\angle I_{3} A D$ that + +$$ +\frac{\left[I_{1}, A B\right]}{\left[I_{1}, A I\right]}=\frac{\left[I_{3}, A I\right]}{\left[I_{3}, A D\right]} +$$ + +Furthermore, consideration of the angle sums in $A I B, B I C, C I D$ and $D I A$ implies $\angle A I B+$ $\angle C I D=\angle B I C+\angle D I A=180^{\circ}$, from which we see + +$$ +\frac{\left[I_{1}, A I\right]}{\left[I_{3}, C I\right]}=\frac{I_{1} I}{I_{3} I}=\frac{\left[I_{1}, C I\right]}{\left[I_{3}, A I\right]} . +$$ + +Because of $\left[I_{1}, A B\right]=\left[I_{1}, B C\right],\left[I_{3}, A D\right]=\left[I_{3}, C D\right]$, multiplication yields + +$$ +\frac{\left[I_{1}, B C\right]}{\left[I_{3}, C I\right]}=\frac{\left[I_{1}, C I\right]}{\left[I_{3}, C D\right]} . +$$ + +By $\angle D C I=\angle I C B=\gamma / 2$ it follows that $\angle I_{1} C B=\angle I_{3} C I$ which concludes the proof of the +above statement. +Let the perpendicular from $I_{1}$ on $I_{2} I_{3}$ intersect $g$ at $Z$. Then $\angle M I_{1} Z=90^{\circ}-\angle I_{3} I_{2} I_{1}=$ $90^{\circ}-\gamma / 2=\angle M C I_{2}$. Since we have also $\angle Z M I_{1}=\angle I_{2} M C$, triangles $M Z I_{1}$ and $M I_{2} C$ are similar. From this one easily proves that also $M I_{2} Z$ and $M C I_{1}$ are similar. Because $C, I_{1}, I_{2}$, and $I_{3}$ are concyclic, $\angle M Z I_{2}=\angle M I_{1} C=\angle N I_{3} C$, thus $N I_{2} Z$ and $N C I_{3}$ are similar, hence $N C I_{2}$ and $N I_{3} Z$ are similar. We conclude $\angle Z I_{3} I_{2}=\angle I_{2} C N=90^{\circ}-\gamma / 2$, hence $I_{1} I_{2} \perp Z I_{3}$. This completes the proof. + +## Number Theory + +## N1 AUS (Australia) + +A social club has $n$ members. They have the membership numbers $1,2, \ldots, n$, respectively. From time to time members send presents to other members, including items they have already received as presents from other members. In order to avoid the embarrassing situation that a member might receive a present that he or she has sent to other members, the club adds the following rule to its statutes at one of its annual general meetings: +"A member with membership number $a$ is permitted to send a present to a member with membership number $b$ if and only if $a(b-1)$ is a multiple of $n$." +Prove that, if each member follows this rule, none will receive a present from another member that he or she has already sent to other members. + +Alternative formulation: Let $G$ be a directed graph with $n$ vertices $v_{1}, v_{2}, \ldots, v_{n}$, such that there is an edge going from $v_{a}$ to $v_{b}$ if and only if $a$ and $b$ are distinct and $a(b-1)$ is a multiple of $n$. Prove that this graph does not contain a directed cycle. + +Solution 1. Suppose there is an edge from $v_{i}$ to $v_{j}$. Then $i(j-1)=i j-i=k n$ for some integer $k$, which implies $i=i j-k n$. If $\operatorname{gcd}(i, n)=d$ and $\operatorname{gcd}(j, n)=e$, then $e$ divides $i j-k n=i$ and thus $e$ also divides $d$. Hence, if there is an edge from $v_{i}$ to $v_{j}$, then $\operatorname{gcd}(j, n) \mid \operatorname{gcd}(i, n)$. +If there is a cycle in $G$, say $v_{i_{1}} \rightarrow v_{i_{2}} \rightarrow \cdots \rightarrow v_{i_{r}} \rightarrow v_{i_{1}}$, then we have + +$$ +\operatorname{gcd}\left(i_{1}, n\right)\left|\operatorname{gcd}\left(i_{r}, n\right)\right| \operatorname{gcd}\left(i_{r-1}, n\right)|\ldots| \operatorname{gcd}\left(i_{2}, n\right) \mid \operatorname{gcd}\left(i_{1}, n\right) +$$ + +which implies that all these greatest common divisors must be equal, say be equal to $t$. +Now we pick any of the $i_{k}$, without loss of generality let it be $i_{1}$. Then $i_{r}\left(i_{1}-1\right)$ is a multiple of $n$ and hence also (by dividing by $t$ ), $i_{1}-1$ is a multiple of $\frac{n}{t}$. Since $i_{1}$ and $i_{1}-1$ are relatively prime, also $t$ and $\frac{n}{t}$ are relatively prime. So, by the Chinese remainder theorem, the value of $i_{1}$ is uniquely determined modulo $n=t \cdot \frac{n}{t}$ by the value of $t$. But, as $i_{1}$ was chosen arbitrarily among the $i_{k}$, this implies that all the $i_{k}$ have to be equal, a contradiction. + +Solution 2. If $a, b, c$ are integers such that $a b-a$ and $b c-b$ are multiples of $n$, then also $a c-a=a(b c-b)+(a b-a)-(a b-a) c$ is a multiple of $n$. This implies that if there is an edge from $v_{a}$ to $v_{b}$ and an edge from $v_{b}$ to $v_{c}$, then there also must be an edge from $v_{a}$ to $v_{c}$. Therefore, if there are any cycles at all, the smallest cycle must have length 2. But suppose the vertices $v_{a}$ and $v_{b}$ form such a cycle, i. e., $a b-a$ and $a b-b$ are both multiples of $n$. Then $a-b$ is also a multiple of $n$, which can only happen if $a=b$, which is impossible. + +Solution 3. Suppose there was a cycle $v_{i_{1}} \rightarrow v_{i_{2}} \rightarrow \cdots \rightarrow v_{i_{r}} \rightarrow v_{i_{1}}$. Then $i_{1}\left(i_{2}-1\right)$ is a multiple of $n$, i.e., $i_{1} \equiv i_{1} i_{2} \bmod n$. Continuing in this manner, we get $i_{1} \equiv i_{1} i_{2} \equiv$ $i_{1} i_{2} i_{3} \equiv i_{1} i_{2} i_{3} \ldots i_{r} \bmod n$. But the same holds for all $i_{k}$, i. e., $i_{k} \equiv i_{1} i_{2} i_{3} \ldots i_{r} \bmod n$. Hence $i_{1} \equiv i_{2} \equiv \cdots \equiv i_{r} \bmod n$, which means $i_{1}=i_{2}=\cdots=i_{r}$, a contradiction. + +Solution 4. Let $n=k$ be the smallest value of $n$ for which the corresponding graph has a cycle. We show that $k$ is a prime power. +If $k$ is not a prime power, it can be written as a product $k=d e$ of relatively prime integers greater than 1. Reducing all the numbers modulo $d$ yields a single vertex or a cycle in the corresponding graph on $d$ vertices, because if $a(b-1) \equiv 0 \bmod k$ then this equation also holds modulo $d$. But since the graph on $d$ vertices has no cycles, by the minimality of $k$, we must have that all the indices of the cycle are congruent modulo $d$. The same holds modulo $e$ and hence also modulo $k=d e$. But then all the indices are equal, which is a contradiction. +Thus $k$ must be a prime power $k=p^{m}$. There are no edges ending at $v_{k}$, so $v_{k}$ is not contained in any cycle. All edges not starting at $v_{k}$ end at a vertex belonging to a non-multiple of $p$, and all edges starting at a non-multiple of $p$ must end at $v_{1}$. But there is no edge starting at $v_{1}$. Hence there is no cycle. + +Solution 5. Suppose there was a cycle $v_{i_{1}} \rightarrow v_{i_{2}} \rightarrow \cdots \rightarrow v_{i_{r}} \rightarrow v_{i_{1}}$. Let $q=p^{m}$ be a prime power dividing $n$. We claim that either $i_{1} \equiv i_{2} \equiv \cdots \equiv i_{r} \equiv 0 \bmod q$ or $i_{1} \equiv i_{2} \equiv \cdots \equiv i_{r} \equiv$ $1 \bmod q$. +Suppose that there is an $i_{s}$ not divisible by $q$. Then, as $i_{s}\left(i_{s+1}-1\right)$ is a multiple of $q, i_{s+1} \equiv$ $1 \bmod p$. Similarly, we conclude $i_{s+2} \equiv 1 \bmod p$ and so on. So none of the labels is divisible by $p$, but since $i_{s}\left(i_{s+1}-1\right)$ is a multiple of $q=p^{m}$ for all $s$, all $i_{s+1}$ are congruent to 1 modulo $q$. This proves the claim. +Now, as all the labels are congruent modulo all the prime powers dividing $n$, they must all be equal by the Chinese remainder theorem. This is a contradiction. + +## N2 PER (Peru) + +A positive integer $N$ is called balanced, if $N=1$ or if $N$ can be written as a product of an even number of not necessarily distinct primes. Given positive integers $a$ and $b$, consider the polynomial $P$ defined by $P(x)=(x+a)(x+b)$. +(a) Prove that there exist distinct positive integers $a$ and $b$ such that all the numbers $P(1), P(2)$, ..., $P(50)$ are balanced. +(b) Prove that if $P(n)$ is balanced for all positive integers $n$, then $a=b$. + +Solution. Define a function $f$ on the set of positive integers by $f(n)=0$ if $n$ is balanced and $f(n)=1$ otherwise. Clearly, $f(n m) \equiv f(n)+f(m) \bmod 2$ for all positive integers $n, m$. +(a) Now for each positive integer $n$ consider the binary sequence $(f(n+1), f(n+2), \ldots, f(n+$ $50)$ ). As there are only $2^{50}$ different such sequences there are two different positive integers $a$ and $b$ such that + +$$ +(f(a+1), f(a+2), \ldots, f(a+50))=(f(b+1), f(b+2), \ldots, f(b+50)) +$$ + +But this implies that for the polynomial $P(x)=(x+a)(x+b)$ all the numbers $P(1), P(2)$, $\ldots, P(50)$ are balanced, since for all $1 \leq k \leq 50$ we have $f(P(k)) \equiv f(a+k)+f(b+k) \equiv$ $2 f(a+k) \equiv 0 \bmod 2$. +(b) Now suppose $P(n)$ is balanced for all positive integers $n$ and $av_{p_{i}}(f(1))$ for all $i=1,2, \ldots, m$, e.g. $a=\left(p_{1} p_{2} \ldots p_{m}\right)^{\alpha}$ with $\alpha$ sufficiently large. Pick any such $a$. The condition of the problem then yields $a \mid(f(a+1)-f(1))$. Assume $f(a+1) \neq f(1)$. Then we must have $v_{p_{i}}(f(a+1)) \neq$ $v_{p_{i}}(f(1))$ for at least one $i$. This yields $v_{p_{i}}(f(a+1)-f(1))=\min \left\{v_{p_{i}}(f(a+1)), v_{p_{i}}(f(1))\right\} \leq$ $v_{p_{1}}(f(1))p_{1}^{\alpha_{1}+1} p_{2}^{\alpha_{2}+1} \ldots p_{m}^{\alpha_{m}+1} \cdot(f(r)+r)-r \\ +& \geq p_{1}^{\alpha_{1}+1} p_{2}^{\alpha_{2}+1} \ldots p_{m}^{\alpha_{m}+1}+(f(r)+r)-r \\ +& >p_{1}^{\alpha_{1}} p_{2}^{\alpha_{2}} \ldots p_{m}^{\alpha_{m}}+f(r) \\ +& \geq|f(M)-f(r)| . +\end{aligned} +$$ + +But since $M-r$ divides $f(M)-f(r)$ this can only be true if $f(r)=f(M)=f(1)$, which contradicts the choice of $r$. + +Comment. In the case that $f$ is a polynomial with integer coefficients the result is well-known, see e.g. W. Schwarz, Einführung in die Methoden der Primzahltheorie, 1969. + +## N4 PRK (Democratic People's Republic of Korea) + +Find all positive integers $n$ such that there exists a sequence of positive integers $a_{1}, a_{2}, \ldots, a_{n}$ satisfying + +$$ +a_{k+1}=\frac{a_{k}^{2}+1}{a_{k-1}+1}-1 +$$ + +for every $k$ with $2 \leq k \leq n-1$. + +Solution 1. Such a sequence exists for $n=1,2,3,4$ and no other $n$. Since the existence of such a sequence for some $n$ implies the existence of such a sequence for all smaller $n$, it suffices to prove that $n=5$ is not possible and $n=4$ is possible. +Assume first that for $n=5$ there exists a sequence of positive integers $a_{1}, a_{2}, \ldots, a_{5}$ satisfying the conditions + +$$ +\begin{aligned} +& a_{2}^{2}+1=\left(a_{1}+1\right)\left(a_{3}+1\right), \\ +& a_{3}^{2}+1=\left(a_{2}+1\right)\left(a_{4}+1\right), \\ +& a_{4}^{2}+1=\left(a_{3}+1\right)\left(a_{5}+1\right) . +\end{aligned} +$$ + +Assume $a_{1}$ is odd, then $a_{2}$ has to be odd as well and as then $a_{2}^{2}+1 \equiv 2 \bmod 4, a_{3}$ has to be even. But this is a contradiction, since then the even number $a_{2}+1$ cannot divide the odd number $a_{3}^{2}+1$. +Hence $a_{1}$ is even. +If $a_{2}$ is odd, $a_{3}^{2}+1$ is even (as a multiple of $a_{2}+1$ ) and hence $a_{3}$ is odd, too. Similarly we must have $a_{4}$ odd as well. But then $a_{3}^{2}+1$ is a product of two even numbers $\left(a_{2}+1\right)\left(a_{4}+1\right)$ and thus is divisible by 4 , which is a contradiction as for odd $a_{3}$ we have $a_{3}^{2}+1 \equiv 2 \bmod 4$. +Hence $a_{2}$ is even. Furthermore $a_{3}+1$ divides the odd number $a_{2}^{2}+1$ and so $a_{3}$ is even. Similarly, $a_{4}$ and $a_{5}$ are even as well. +Now set $x=a_{2}$ and $y=a_{3}$. From the given condition we get $(x+1) \mid\left(y^{2}+1\right)$ and $(y+1) \mid\left(x^{2}+1\right)$. We will prove that there is no pair of positive even numbers $(x, y)$ satisfying these two conditions, thus yielding a contradiction to the assumption. +Assume there exists a pair $\left(x_{0}, y_{0}\right)$ of positive even numbers satisfying the two conditions $\left(x_{0}+1\right) \mid\left(y_{0}^{2}+1\right)$ and $\left(y_{0}+1\right) \mid\left(x_{0}^{2}+1\right)$. +Then one has $\left(x_{0}+1\right) \mid\left(y_{0}^{2}+1+x_{0}^{2}-1\right)$, i.e., $\left(x_{0}+1\right) \mid\left(x_{0}^{2}+y_{0}^{2}\right)$, and similarly $\left(y_{0}+1\right) \mid\left(x_{0}^{2}+y_{0}^{2}\right)$. Any common divisor $d$ of $x_{0}+1$ and $y_{0}+1$ must hence also divide the number $\left(x_{0}^{2}+1\right)+\left(y_{0}^{2}+1\right)-\left(x_{0}^{2}+y_{0}^{2}\right)=2$. But as $x_{0}+1$ and $y_{0}+1$ are both odd, we must have $d=1$. Thus $x_{0}+1$ and $y_{0}+1$ are relatively prime and therefore there exists a positive integer $k$ such that + +$$ +k(x+1)(y+1)=x^{2}+y^{2} +$$ + +has the solution $\left(x_{0}, y_{0}\right)$. We will show that the latter equation has no solution $(x, y)$ in positive even numbers. + +Assume there is a solution. Pick the solution $\left(x_{1}, y_{1}\right)$ with the smallest sum $x_{1}+y_{1}$ and assume $x_{1} \geq y_{1}$. Then $x_{1}$ is a solution to the quadratic equation + +$$ +x^{2}-k\left(y_{1}+1\right) x+y_{1}^{2}-k\left(y_{1}+1\right)=0 . +$$ + +Let $x_{2}$ be the second solution, which by Vieta's theorem fulfills $x_{1}+x_{2}=k\left(y_{1}+1\right)$ and $x_{1} x_{2}=y_{1}^{2}-k\left(y_{1}+1\right)$. If $x_{2}=0$, the second equation implies $y_{1}^{2}=k\left(y_{1}+1\right)$, which is impossible, as $y_{1}+1>1$ cannot divide the relatively prime number $y_{1}^{2}$. Therefore $x_{2} \neq 0$. +Also we get $\left(x_{1}+1\right)\left(x_{2}+1\right)=x_{1} x_{2}+x_{1}+x_{2}+1=y_{1}^{2}+1$ which is odd, and hence $x_{2}$ must be even and positive. Also we have $x_{2}+1=\frac{y_{1}^{2}+1}{x_{1}+1} \leq \frac{y_{1}^{2}+1}{y_{1}+1} \leq y_{1} \leq x_{1}$. But this means that the pair $\left(x^{\prime}, y^{\prime}\right)$ with $x^{\prime}=y_{1}$ and $y^{\prime}=x_{2}$ is another solution of $k(x+1)(y+1)=x^{2}+y^{2}$ in even positive numbers with $x^{\prime}+y^{\prime}y +$$ + +and similarly in the other case. +Now, if $a_{3}$ was odd, then $\left(a_{2}+1\right)\left(a_{4}+1\right)=a_{3}^{2}+1 \equiv 2 \bmod 4$ would imply that one of $a_{2}$ or $a_{4}$ is even, this contradicts (1). Thus $a_{3}$ and hence also $a_{1}, a_{2}, a_{4}$ and $a_{5}$ are even. According to (1), one has $a_{1}a_{2}>a_{3}>a_{4}>a_{5}$ but due to the minimality of $a_{1}$ the first series of inequalities must hold. +Consider the identity +$\left(a_{3}+1\right)\left(a_{1}+a_{3}\right)=a_{3}^{2}-1+\left(a_{1}+1\right)\left(a_{3}+1\right)=a_{2}^{2}+a_{3}^{2}=a_{2}^{2}-1+\left(a_{2}+1\right)\left(a_{4}+1\right)=\left(a_{2}+1\right)\left(a_{2}+a_{4}\right)$. +Any common divisor of the two odd numbers $a_{2}+1$ and $a_{3}+1$ must also divide $\left(a_{2}+1\right)\left(a_{4}+\right.$ $1)-\left(a_{3}+1\right)\left(a_{3}-1\right)=2$, so these numbers are relatively prime. Hence the last identity shows that $a_{1}+a_{3}$ must be a multiple of $a_{2}+1$, i.e. there is an integer $k$ such that + +$$ +a_{1}+a_{3}=k\left(a_{2}+1\right) +$$ + +Now set $a_{0}=k\left(a_{1}+1\right)-a_{2}$. This is an integer and we have + +$$ +\begin{aligned} +\left(a_{0}+1\right)\left(a_{2}+1\right) & =k\left(a_{1}+1\right)\left(a_{2}+1\right)-\left(a_{2}-1\right)\left(a_{2}+1\right) \\ +& =\left(a_{1}+1\right)\left(a_{1}+a_{3}\right)-\left(a_{1}+1\right)\left(a_{3}+1\right)+2 \\ +& =\left(a_{1}+1\right)\left(a_{1}-1\right)+2=a_{1}^{2}+1 +\end{aligned} +$$ + +Thus $a_{0} \geq 0$. If $a_{0}>0$, then by (1) we would have $a_{0}1$ is relatively prime to $a_{1}^{2}$ and thus cannot be a divisior of this number. +Hence $n \geq 5$ is not possible. + +Comment 1. Finding the example for $n=4$ is not trivial and requires a tedious calculation, but it can be reduced to checking a few cases. The equations $\left(a_{1}+1\right)\left(a_{3}+1\right)=a_{2}^{2}+1$ and $\left(a_{2}+1\right)\left(a_{4}+1\right)=a_{3}^{2}+1$ imply, as seen in the proof, that $a_{1}$ is even and $a_{2}, a_{3}, a_{4}$ are odd. The case $a_{1}=2$ yields $a_{2}^{2} \equiv-1 \bmod 3$ which is impossible. Hence $a_{1}=4$ is the smallest possibility. In this case $a_{2}^{2} \equiv-1 \bmod 5$ and $a_{2}$ is odd, which implies $a_{2} \equiv 3$ or $a_{2} \equiv 7 \bmod 10$. Hence we have to start checking $a_{2}=7,13,17,23,27,33$ and in the last case we succeed. + +Comment 2. The choice of $a_{0}=k\left(a_{1}+1\right)-a_{2}$ in the second solution appears more natural if one considers that by the previous calculations one has $a_{1}=k\left(a_{2}+1\right)-a_{3}$ and $a_{2}=k\left(a_{3}+1\right)-a_{4}$. Alternatively, one can solve the equation (2) for $a_{3}$ and use $a_{2}^{2}+1=\left(a_{1}+1\right)\left(a_{3}+1\right)$ to get $a_{2}^{2}-k\left(a_{1}+1\right) a_{2}+a_{1}^{2}-k\left(a_{1}+1\right)=0$. Now $a_{0}$ is the second solution to this quadratic equation in $a_{2}$ (Vieta jumping). + +## N5 HUN (Hungary) + +Let $P(x)$ be a non-constant polynomial with integer coefficients. Prove that there is no function $T$ from the set of integers into the set of integers such that the number of integers $x$ with $T^{n}(x)=x$ is equal to $P(n)$ for every $n \geq 1$, where $T^{n}$ denotes the $n$-fold application of $T$. + +Solution 1. Assume there is a polynomial $P$ of degree at least 1 with the desired property for a given function $T$. Let $A(n)$ denote the set of all $x \in \mathbb{Z}$ such that $T^{n}(x)=x$ and let $B(n)$ denote the set of all $x \in \mathbb{Z}$ for which $T^{n}(x)=x$ and $T^{k}(x) \neq x$ for all $1 \leq k\sqrt{a b}, b^{\ell_{0}}>\sqrt{a b}$, we define the polynomial + +$$ +P(x)=\prod_{k=0, \ell=0}^{k_{0}-1, \ell_{0}-1}\left(a^{k} b^{\ell} x-\sqrt{a b}\right)=: \sum_{i=0}^{k_{0} \cdot \ell_{0}} d_{i} x^{i} +$$ + +with integer coefficients $d_{i}$. By our assumption, the zeros + +$$ +\frac{\sqrt{a b}}{a^{k} b^{\ell}}, \quad k=0, \ldots, k_{0}-1, \quad \ell=0, \ldots, \ell_{0}-1 +$$ + +of $P$ are pairwise distinct. +Furthermore, we consider the integer sequence + +$$ +y_{n}=\sum_{i=0}^{k_{0} \cdot \ell_{0}} d_{i} x_{n+i}, \quad n=1,2, \ldots +$$ + +By the theory of linear recursions, we obtain + +$$ +y_{n}=\sum_{\substack{k, \ell \geq 0 \\ k \geq k_{0} \text { or } \ell \geq \ell_{0}}} e_{k, \ell}\left(\frac{\sqrt{a b}}{a^{k} b^{\ell}}\right)^{n}, \quad n=1,2, \ldots +$$ + +with real numbers $e_{k, \ell}$. We have + +$$ +\left|y_{n}\right| \leq \sum_{\substack{k, \ell \geq 0 \\ k \geq k_{0} \text { or } \ell \geq \ell_{0}}}\left|e_{k, \ell}\right|\left(\frac{\sqrt{a b}}{a^{k} b^{\ell}}\right)^{n}=: M_{n} . +$$ + +Because the series in (4) is obtained by a finite linear combination of the absolutely convergent series (1), we conclude that in particular $M_{1}<\infty$. Since + +$$ +\frac{\sqrt{a b}}{a^{k} b^{\ell}} \leq \lambda:=\max \left\{\frac{\sqrt{a b}}{a^{k_{0}}}, \frac{\sqrt{a b}}{b^{\ell_{0}}}\right\} \quad \text { for all } k, \ell \geq 0 \text { such that } k \geq k_{0} \text { or } \ell \geq \ell_{0} +$$ + +we get the estimates $M_{n+1} \leq \lambda M_{n}, n=1,2, \ldots$ Our choice of $k_{0}$ and $\ell_{0}$ ensures $\lambda<1$, which implies $M_{n} \rightarrow 0$ and consequently $y_{n} \rightarrow 0$ as $n \rightarrow \infty$. It follows that $y_{n}=0$ for all sufficiently large $n$. +So, equation (3) reduces to $\sum_{i=0}^{k_{0} \cdot \ell_{0}} d_{i} x_{n+i}=0$. +Using the theory of linear recursions again, for sufficiently large $n$ we have + +$$ +x_{n}=\sum_{k=0, \ell=0}^{k_{0}-1, \ell_{0}-1} f_{k, \ell}\left(\frac{\sqrt{a b}}{a^{k} b^{\ell}}\right)^{n} +$$ + +for certain real numbers $f_{k, \ell}$. +Comparing with (2), we see that $f_{k, \ell}=c_{k, \ell}$ for all $k, \ell \geq 0$ with $kj_{0}$. But this means that + +$$ +\left(1-x^{\mu}\right)^{\frac{1}{2}}\left(1-x^{\nu}\right)^{\frac{1}{2}}=\sum_{j=0}^{j_{0}} g_{j} x^{j} +$$ + +for all real numbers $x \in(0,1)$. Squaring, we see that + +$$ +\left(1-x^{\mu}\right)\left(1-x^{\nu}\right) +$$ + +is the square of a polynomial in $x$. In particular, all its zeros are of order at least 2 , which implies $\mu=\nu$ by looking at roots of unity. So we obtain $\mu=\nu=1$, i. e., $a=b$, a contradiction. + +Solution 2. We set $a^{2}=A, b^{2}=B$, and $z_{n}=\sqrt{\left(A^{n}-1\right)\left(B^{n}-1\right)}$. Let us assume that $z_{n}$ is an integer for $n=1,2, \ldots$. Without loss of generality, we may suppose that $b0$ as $z_{n}>0$. As before, one obtains + +$$ +\begin{aligned} +& A^{n} B^{n}-A^{n}-B^{n}+1=z_{n}^{2} \\ +& =\left\{\delta_{0}(a b)^{n}-\delta_{1}\left(\frac{a}{b}\right)^{n}-\delta_{2}\left(\frac{a}{b^{3}}\right)^{n}-\cdots-\delta_{k}\left(\frac{a}{b^{2 k-1}}\right)^{n}\right\}^{2} \\ +& =\delta_{0}^{2} A^{n} B^{n}-2 \delta_{0} \delta_{1} A^{n}-\sum_{i=2}^{i=k}\left(2 \delta_{0} \delta_{i}-\sum_{j=1}^{j=i-1} \delta_{j} \delta_{i-j}\right)\left(\frac{A}{B^{i-1}}\right)^{n}+O\left(\frac{A}{B^{k}}\right)^{n} +\end{aligned} +$$ + +Easy asymptotic calculations yield $\delta_{0}=1, \delta_{1}=\frac{1}{2}, \delta_{i}=\frac{1}{2} \sum_{j=1}^{j=i-1} \delta_{j} \delta_{i-j}$ for $i=2,3, \ldots, k-2$, and then $a=b^{k-1}$. It follows that $k>2$ and there is some $P \in \mathbb{Q}[X]$ for which $(X-1)\left(X^{k-1}-1\right)=$ $P(X)^{2}$. But this cannot occur, for instance as $X^{k-1}-1$ has no double zeros. Thus our +assumption that $z_{n}$ was an integer for $n=1,2, \ldots$ turned out to be wrong, which solves the problem. + +Original formulation of the problem. $a, b$ are positive integers such that $a \cdot b$ is not a square of an integer. Prove that there exists a (infinitely many) positive integer $n$ such that ( $\left.a^{n}-1\right)\left(b^{n}-1\right)$ is not a square of an integer. + +Solution. Lemma. Let $c$ be a positive integer, which is not a perfect square. Then there exists an odd prime $p$ such that $c$ is not a quadratic residue modulo $p$. +Proof. Denoting the square-free part of $c$ by $c^{\prime}$, we have the equality $\left(\frac{c^{\prime}}{p}\right)=\left(\frac{c}{p}\right)$ of the corresponding LegEndre symbols. Suppose that $c^{\prime}=q_{1} \cdots q_{m}$, where $q_{1}<\cdots1$ ); so, the expression $\frac{4}{3}(3-d)(d-1)\left(d^{2}-3\right)$ in the right-hand part of $(2)$ is nonnegative, and the desired inequality is proved. +Comment. The rightmost inequality is easier than the leftmost one. In particular, Solutions 2 and 3 show that only the condition $a^{2}+b^{2}+c^{2}+d^{2}=12$ is needed for the former one. + +A3. Let $x_{1}, \ldots, x_{100}$ be nonnegative real numbers such that $x_{i}+x_{i+1}+x_{i+2} \leq 1$ for all $i=1, \ldots, 100$ (we put $x_{101}=x_{1}, x_{102}=x_{2}$ ). Find the maximal possible value of the sum + +$$ +S=\sum_{i=1}^{100} x_{i} x_{i+2} +$$ + +(Russia) +Answer. $\frac{25}{2}$. +Solution 1. Let $x_{2 i}=0, x_{2 i-1}=\frac{1}{2}$ for all $i=1, \ldots, 50$. Then we have $S=50 \cdot\left(\frac{1}{2}\right)^{2}=\frac{25}{2}$. So, we are left to show that $S \leq \frac{25}{2}$ for all values of $x_{i}$ 's satisfying the problem conditions. + +Consider any $1 \leq i \leq 50$. By the problem condition, we get $x_{2 i-1} \leq 1-x_{2 i}-x_{2 i+1}$ and $x_{2 i+2} \leq 1-x_{2 i}-x_{2 i+1}$. Hence by the AM-GM inequality we get + +$$ +\begin{aligned} +x_{2 i-1} x_{2 i+1} & +x_{2 i} x_{2 i+2} \leq\left(1-x_{2 i}-x_{2 i+1}\right) x_{2 i+1}+x_{2 i}\left(1-x_{2 i}-x_{2 i+1}\right) \\ +& =\left(x_{2 i}+x_{2 i+1}\right)\left(1-x_{2 i}-x_{2 i+1}\right) \leq\left(\frac{\left(x_{2 i}+x_{2 i+1}\right)+\left(1-x_{2 i}-x_{2 i+1}\right)}{2}\right)^{2}=\frac{1}{4} +\end{aligned} +$$ + +Summing up these inequalities for $i=1,2, \ldots, 50$, we get the desired inequality + +$$ +\sum_{i=1}^{50}\left(x_{2 i-1} x_{2 i+1}+x_{2 i} x_{2 i+2}\right) \leq 50 \cdot \frac{1}{4}=\frac{25}{2} +$$ + +Comment. This solution shows that a bit more general fact holds. Namely, consider $2 n$ nonnegative numbers $x_{1}, \ldots, x_{2 n}$ in a row (with no cyclic notation) and suppose that $x_{i}+x_{i+1}+x_{i+2} \leq 1$ for all $i=1,2, \ldots, 2 n-2$. Then $\sum_{i=1}^{2 n-2} x_{i} x_{i+2} \leq \frac{n-1}{4}$. + +The proof is the same as above, though if might be easier to find it (for instance, applying induction). The original estimate can be obtained from this version by considering the sequence $x_{1}, x_{2}, \ldots, x_{100}, x_{1}, x_{2}$. + +Solution 2. We present another proof of the estimate. From the problem condition, we get + +$$ +\begin{aligned} +S=\sum_{i=1}^{100} x_{i} x_{i+2} \leq \sum_{i=1}^{100} x_{i}\left(1-x_{i}-x_{i+1}\right) & =\sum_{i=1}^{100} x_{i}-\sum_{i=1}^{100} x_{i}^{2}-\sum_{i=1}^{100} x_{i} x_{i+1} \\ +& =\sum_{i=1}^{100} x_{i}-\frac{1}{2} \sum_{i=1}^{100}\left(x_{i}+x_{i+1}\right)^{2} +\end{aligned} +$$ + +By the AM-QM inequality, we have $\sum\left(x_{i}+x_{i+1}\right)^{2} \geq \frac{1}{100}\left(\sum\left(x_{i}+x_{i+1}\right)\right)^{2}$, so + +$$ +\begin{aligned} +S \leq \sum_{i=1}^{100} x_{i}-\frac{1}{200}\left(\sum_{i=1}^{100}\left(x_{i}+x_{i+1}\right)\right)^{2} & =\sum_{i=1}^{100} x_{i}-\frac{2}{100}\left(\sum_{i=1}^{100} x_{i}\right)^{2} \\ +& =\frac{2}{100}\left(\sum_{i=1}^{100} x_{i}\right)\left(\frac{100}{2}-\sum_{i=1}^{100} x_{i}\right) +\end{aligned} +$$ + +And finally, by the AM-GM inequality + +$$ +S \leq \frac{2}{100} \cdot\left(\frac{1}{2}\left(\sum_{i=1}^{100} x_{i}+\frac{100}{2}-\sum_{i=1}^{100} x_{i}\right)\right)^{2}=\frac{2}{100} \cdot\left(\frac{100}{4}\right)^{2}=\frac{25}{2} +$$ + +Comment. These solutions are not as easy as they may seem at the first sight. There are two different optimal configurations in which the variables have different values, and not all of sums of three consecutive numbers equal 1. Although it is easy to find the value $\frac{25}{2}$, the estimates must be done with care to preserve equality in the optimal configurations. + +A4. A sequence $x_{1}, x_{2}, \ldots$ is defined by $x_{1}=1$ and $x_{2 k}=-x_{k}, x_{2 k-1}=(-1)^{k+1} x_{k}$ for all $k \geq 1$. Prove that $x_{1}+x_{2}+\cdots+x_{n} \geq 0$ for all $n \geq 1$. +(Austria) +Solution 1. We start with some observations. First, from the definition of $x_{i}$ it follows that for each positive integer $k$ we have + +$$ +x_{4 k-3}=x_{2 k-1}=-x_{4 k-2} \quad \text { and } \quad x_{4 k-1}=x_{4 k}=-x_{2 k}=x_{k} +$$ + +Hence, denoting $S_{n}=\sum_{i=1}^{n} x_{i}$, we have + +$$ +\begin{gathered} +S_{4 k}=\sum_{i=1}^{k}\left(\left(x_{4 k-3}+x_{4 k-2}\right)+\left(x_{4 k-1}+x_{4 k}\right)\right)=\sum_{i=1}^{k}\left(0+2 x_{k}\right)=2 S_{k}, \\ +S_{4 k+2}=S_{4 k}+\left(x_{4 k+1}+x_{4 k+2}\right)=S_{4 k} . +\end{gathered} +$$ + +Observe also that $S_{n}=\sum_{i=1}^{n} x_{i} \equiv \sum_{i=1}^{n} 1=n(\bmod 2)$. +Now we prove by induction on $k$ that $S_{i} \geq 0$ for all $i \leq 4 k$. The base case is valid since $x_{1}=x_{3}=x_{4}=1, x_{2}=-1$. For the induction step, assume that $S_{i} \geq 0$ for all $i \leq 4 k$. Using the relations (1)-(3), we obtain + +$$ +S_{4 k+4}=2 S_{k+1} \geq 0, \quad S_{4 k+2}=S_{4 k} \geq 0, \quad S_{4 k+3}=S_{4 k+2}+x_{4 k+3}=\frac{S_{4 k+2}+S_{4 k+4}}{2} \geq 0 +$$ + +So, we are left to prove that $S_{4 k+1} \geq 0$. If $k$ is odd, then $S_{4 k}=2 S_{k} \geq 0$; since $k$ is odd, $S_{k}$ is odd as well, so we have $S_{4 k} \geq 2$ and hence $S_{4 k+1}=S_{4 k}+x_{4 k+1} \geq 1$. + +Conversely, if $k$ is even, then we have $x_{4 k+1}=x_{2 k+1}=x_{k+1}$, hence $S_{4 k+1}=S_{4 k}+x_{4 k+1}=$ $2 S_{k}+x_{k+1}=S_{k}+S_{k+1} \geq 0$. The step is proved. + +Solution 2. We will use the notation of $S_{n}$ and the relations (1)-(3) from the previous solution. + +Assume the contrary and consider the minimal $n$ such that $S_{n+1}<0$; surely $n \geq 1$, and from $S_{n} \geq 0$ we get $S_{n}=0, x_{n+1}=-1$. Hence, we are especially interested in the set $M=\left\{n: S_{n}=0\right\}$; our aim is to prove that $x_{n+1}=1$ whenever $n \in M$ thus coming to a contradiction. + +For this purpose, we first describe the set $M$ inductively. We claim that (i) $M$ consists only of even numbers, (ii) $2 \in M$, and (iii) for every even $n \geq 4$ we have $n \in M \Longleftrightarrow[n / 4] \in M$. Actually, (i) holds since $S_{n} \equiv n(\bmod 2)$, (ii) is straightforward, while (iii) follows from the relations $S_{4 k+2}=S_{4 k}=2 S_{k}$. + +Now, we are left to prove that $x_{n+1}=1$ if $n \in M$. We use the induction on $n$. The base case is $n=2$, that is, the minimal element of $M$; here we have $x_{3}=1$, as desired. + +For the induction step, consider some $4 \leq n \in M$ and let $m=[n / 4] \in M$; then $m$ is even, and $x_{m+1}=1$ by the induction hypothesis. We prove that $x_{n+1}=x_{m+1}=1$. If $n=4 m$ then we have $x_{n+1}=x_{2 m+1}=x_{m+1}$ since $m$ is even; otherwise, $n=4 m+2$, and $x_{n+1}=-x_{2 m+2}=x_{m+1}$, as desired. The proof is complete. + +Comment. Using the inductive definition of set $M$, one can describe it explicitly. Namely, $M$ consists exactly of all positive integers not containing digits 1 and 3 in their 4 -base representation. + +A5. Denote by $\mathbb{Q}^{+}$the set of all positive rational numbers. Determine all functions $f: \mathbb{Q}^{+} \rightarrow \mathbb{Q}^{+}$ which satisfy the following equation for all $x, y \in \mathbb{Q}^{+}$: + +$$ +f\left(f(x)^{2} y\right)=x^{3} f(x y) +$$ + +(Switzerland) +Answer. The only such function is $f(x)=\frac{1}{x}$. +Solution. By substituting $y=1$, we get + +$$ +f\left(f(x)^{2}\right)=x^{3} f(x) +$$ + +Then, whenever $f(x)=f(y)$, we have + +$$ +x^{3}=\frac{f\left(f(x)^{2}\right)}{f(x)}=\frac{f\left(f(y)^{2}\right)}{f(y)}=y^{3} +$$ + +which implies $x=y$, so the function $f$ is injective. +Now replace $x$ by $x y$ in (2), and apply (1) twice, second time to $\left(y, f(x)^{2}\right)$ instead of $(x, y)$ : + +$$ +f\left(f(x y)^{2}\right)=(x y)^{3} f(x y)=y^{3} f\left(f(x)^{2} y\right)=f\left(f(x)^{2} f(y)^{2}\right) +$$ + +Since $f$ is injective, we get + +$$ +\begin{aligned} +f(x y)^{2} & =f(x)^{2} f(y)^{2} \\ +f(x y) & =f(x) f(y) . +\end{aligned} +$$ + +Therefore, $f$ is multiplicative. This also implies $f(1)=1$ and $f\left(x^{n}\right)=f(x)^{n}$ for all integers $n$. +Then the function equation (1) can be re-written as + +$$ +\begin{aligned} +f(f(x))^{2} f(y) & =x^{3} f(x) f(y), \\ +f(f(x)) & =\sqrt{x^{3} f(x)} . +\end{aligned} +$$ + +Let $g(x)=x f(x)$. Then, by (3), we have + +$$ +\begin{aligned} +g(g(x)) & =g(x f(x))=x f(x) \cdot f(x f(x))=x f(x)^{2} f(f(x))= \\ +& =x f(x)^{2} \sqrt{x^{3} f(x)}=(x f(x))^{5 / 2}=(g(x))^{5 / 2} +\end{aligned} +$$ + +and, by induction, +for every positive integer $n$. +Consider (4) for a fixed $x$. The left-hand side is always rational, so $(g(x))^{(5 / 2)^{n}}$ must be rational for every $n$. We show that this is possible only if $g(x)=1$. Suppose that $g(x) \neq 1$, and let the prime factorization of $g(x)$ be $g(x)=p_{1}^{\alpha_{1}} \ldots p_{k}^{\alpha_{k}}$ where $p_{1}, \ldots, p_{k}$ are distinct primes and $\alpha_{1}, \ldots, \alpha_{k}$ are nonzero integers. Then the unique prime factorization of (4) is + +$$ +\underbrace{g(g(\ldots g}_{n+1}(x) \ldots))=(g(x))^{(5 / 2)^{n}}=p_{1}^{(5 / 2)^{n} \alpha_{1}} \cdots p_{k}^{(5 / 2)^{n} \alpha_{k}} +$$ + +where the exponents should be integers. But this is not true for large values of $n$, for example $\left(\frac{5}{2}\right)^{n} \alpha_{1}$ cannot be a integer number when $2^{n} \nmid \alpha_{1}$. Therefore, $g(x) \neq 1$ is impossible. + +Hence, $g(x)=1$ and thus $f(x)=\frac{1}{x}$ for all $x$. +The function $f(x)=\frac{1}{x}$ satisfies the equation (1): + +$$ +f\left(f(x)^{2} y\right)=\frac{1}{f(x)^{2} y}=\frac{1}{\left(\frac{1}{x}\right)^{2} y}=\frac{x^{3}}{x y}=x^{3} f(x y) +$$ + +Comment. Among $\mathbb{R}^{+} \rightarrow \mathbb{R}^{+}$functions, $f(x)=\frac{1}{x}$ is not the only solution. Another solution is $f_{1}(x)=x^{3 / 2}$. Using transfinite tools, infinitely many other solutions can be constructed. + +A6. Suppose that $f$ and $g$ are two functions defined on the set of positive integers and taking positive integer values. Suppose also that the equations $f(g(n))=f(n)+1$ and $g(f(n))=$ $g(n)+1$ hold for all positive integers. Prove that $f(n)=g(n)$ for all positive integer $n$. +(Germany) +Solution 1. Throughout the solution, by $\mathbb{N}$ we denote the set of all positive integers. For any function $h: \mathbb{N} \rightarrow \mathbb{N}$ and for any positive integer $k$, define $h^{k}(x)=\underbrace{h(h(\ldots h}_{k}(x) \ldots)$ ) (in particular, $\left.h^{0}(x)=x\right)$. + +Observe that $f\left(g^{k}(x)\right)=f\left(g^{k-1}(x)\right)+1=\cdots=f(x)+k$ for any positive integer $k$, and similarly $g\left(f^{k}(x)\right)=g(x)+k$. Now let $a$ and $b$ are the minimal values attained by $f$ and $g$, respectively; say $f\left(n_{f}\right)=a, g\left(n_{g}\right)=b$. Then we have $f\left(g^{k}\left(n_{f}\right)\right)=a+k, g\left(f^{k}\left(n_{g}\right)\right)=b+k$, so the function $f$ attains all values from the set $N_{f}=\{a, a+1, \ldots\}$, while $g$ attains all the values from the set $N_{g}=\{b, b+1, \ldots\}$. + +Next, note that $f(x)=f(y)$ implies $g(x)=g(f(x))-1=g(f(y))-1=g(y)$; surely, the converse implication also holds. Now, we say that $x$ and $y$ are similar (and write $x \sim y$ ) if $f(x)=f(y)$ (equivalently, $g(x)=g(y)$ ). For every $x \in \mathbb{N}$, we define $[x]=\{y \in \mathbb{N}: x \sim y\}$; surely, $y_{1} \sim y_{2}$ for all $y_{1}, y_{2} \in[x]$, so $[x]=[y]$ whenever $y \in[x]$. + +Now we investigate the structure of the sets $[x]$. +Claim 1. Suppose that $f(x) \sim f(y)$; then $x \sim y$, that is, $f(x)=f(y)$. Consequently, each class $[x]$ contains at most one element from $N_{f}$, as well as at most one element from $N_{g}$. +Proof. If $f(x) \sim f(y)$, then we have $g(x)=g(f(x))-1=g(f(y))-1=g(y)$, so $x \sim y$. The second statement follows now from the sets of values of $f$ and $g$. + +Next, we clarify which classes do not contain large elements. +Claim 2. For any $x \in \mathbb{N}$, we have $[x] \subseteq\{1,2, \ldots, b-1\}$ if and only if $f(x)=a$. Analogously, $[x] \subseteq\{1,2, \ldots, a-1\}$ if and only if $g(x)=b$. +Proof. We will prove that $[x] \nsubseteq\{1,2, \ldots, b-1\} \Longleftrightarrow f(x)>a$; the proof of the second statement is similar. + +Note that $f(x)>a$ implies that there exists some $y$ satisfying $f(y)=f(x)-1$, so $f(g(y))=$ $f(y)+1=f(x)$, and hence $x \sim g(y) \geq b$. Conversely, if $b \leq c \sim x$ then $c=g(y)$ for some $y \in \mathbb{N}$, which in turn follows $f(x)=f(g(y))=f(y)+1 \geq a+1$, and hence $f(x)>a$. + +Claim 2 implies that there exists exactly one class contained in $\{1, \ldots, a-1\}$ (that is, the class $\left[n_{g}\right]$ ), as well as exactly one class contained in $\{1, \ldots, b-1\}$ (the class $\left[n_{f}\right]$ ). Assume for a moment that $a \leq b$; then $\left[n_{g}\right]$ is contained in $\{1, \ldots, b-1\}$ as well, hence it coincides with $\left[n_{g}\right]$. So, we get that + +$$ +f(x)=a \Longleftrightarrow g(x)=b \Longleftrightarrow x \sim n_{f} \sim n_{g} . +$$ + +Claim 3. $a=b$. +Proof. By Claim 2, we have $[a] \neq\left[n_{f}\right]$, so $[a]$ should contain some element $a^{\prime} \geq b$ by Claim 2 again. If $a \neq a^{\prime}$, then $[a]$ contains two elements $\geq a$ which is impossible by Claim 1. Therefore, $a=a^{\prime} \geq b$. Similarly, $b \geq a$. + +Now we are ready to prove the problem statement. First, we establish the following +Claim 4. For every integer $d \geq 0, f^{d+1}\left(n_{f}\right)=g^{d+1}\left(n_{f}\right)=a+d$. +Proof. Induction on $d$. For $d=0$, the statement follows from (1) and Claim 3. Next, for $d>1$ from the induction hypothesis we have $f^{d+1}\left(n_{f}\right)=f\left(f^{d}\left(n_{f}\right)\right)=f\left(g^{d}\left(n_{f}\right)\right)=f\left(n_{f}\right)+d=a+d$. The equality $g^{d+1}\left(n_{f}\right)=a+d$ is analogous. + +Finally, for each $x \in \mathbb{N}$, we have $f(x)=a+d$ for some $d \geq 0$, so $f(x)=f\left(g^{d}\left(n_{f}\right)\right)$ and hence $x \sim g^{d}\left(n_{f}\right)$. It follows that $g(x)=g\left(g^{d}\left(n_{f}\right)\right)=g^{d+1}\left(n_{f}\right)=a+d=f(x)$ by Claim 4 . + +Solution 2. We start with the same observations, introducing the relation $\sim$ and proving Claim 1 from the previous solution. + +Note that $f(a)>a$ since otherwise we have $f(a)=a$ and hence $g(a)=g(f(a))=g(a)+1$, which is false. +Claim 2'. $a=b$. +Proof. We can assume that $a \leq b$. Since $f(a) \geq a+1$, there exists some $x \in \mathbb{N}$ such that $f(a)=f(x)+1$, which is equivalent to $f(a)=f(g(x))$ and $a \sim g(x)$. Since $g(x) \geq b \geq a$, by Claim 1 we have $a=g(x) \geq b$, which together with $a \leq b$ proves the Claim. + +Now, almost the same method allows to find the values $f(a)$ and $g(a)$. +Claim 3'. $f(a)=g(a)=a+1$. +Proof. Assume the contrary; then $f(a) \geq a+2$, hence there exist some $x, y \in \mathbb{N}$ such that $f(x)=f(a)-2$ and $f(y)=g(x)($ as $g(x) \geq a=b)$. Now we get $f(a)=f(x)+2=f\left(g^{2}(x)\right)$, so $a \sim g^{2}(x) \geq a$, and by Claim 1 we get $a=g^{2}(x)=g(f(y))=1+g(y) \geq 1+a$; this is impossible. The equality $g(a)=a+1$ is similar. + +Now, we are prepared for the proof of the problem statement. First, we prove it for $n \geq a$. Claim 4'. For each integer $x \geq a$, we have $f(x)=g(x)=x+1$. +Proof. Induction on $x$. The base case $x=a$ is provided by Claim $3^{\prime}$, while the induction step follows from $f(x+1)=f(g(x))=f(x)+1=(x+1)+1$ and the similar computation for $g(x+1)$. + +Finally, for an arbitrary $n \in \mathbb{N}$ we have $g(n) \geq a$, so by Claim $4^{\prime}$ we have $f(n)+1=$ $f(g(n))=g(n)+1$, hence $f(n)=g(n)$. +Comment. It is not hard now to describe all the functions $f: \mathbb{N} \rightarrow \mathbb{N}$ satisfying the property $f(f(n))=$ $f(n)+1$. For each such function, there exists $n_{0} \in \mathbb{N}$ such that $f(n)=n+1$ for all $n \geq n_{0}$, while for each $nr$, we inductively define + +$$ +a_{n}=\max _{1 \leq k \leq n-1}\left(a_{k}+a_{n-k}\right) +$$ + +Prove that there exist positive integers $\ell \leq r$ and $N$ such that $a_{n}=a_{n-\ell}+a_{\ell}$ for all $n \geq N$. +(Iran) +Solution 1. First, from the problem conditions we have that each $a_{n}(n>r)$ can be expressed as $a_{n}=a_{j_{1}}+a_{j_{2}}$ with $j_{1}, j_{2}r$ then we can proceed in the same way with $a_{j_{1}}$, and so on. Finally, we represent $a_{n}$ in a form + +$$ +\begin{gathered} +a_{n}=a_{i_{1}}+\cdots+a_{i_{k}}, \\ +1 \leq i_{j} \leq r, \quad i_{1}+\cdots+i_{k}=n . +\end{gathered} +$$ + +Moreover, if $a_{i_{1}}$ and $a_{i_{2}}$ are the numbers in (2) obtained on the last step, then $i_{1}+i_{2}>r$. Hence we can adjust (3) as + +$$ +1 \leq i_{j} \leq r, \quad i_{1}+\cdots+i_{k}=n, \quad i_{1}+i_{2}>r . +$$ + +On the other hand, suppose that the indices $i_{1}, \ldots, i_{k}$ satisfy the conditions (4). Then, denoting $s_{j}=i_{1}+\cdots+i_{j}$, from (1) we have + +$$ +a_{n}=a_{s_{k}} \geq a_{s_{k-1}}+a_{i_{k}} \geq a_{s_{k-2}}+a_{i_{k-1}}+a_{i_{k}} \geq \cdots \geq a_{i_{1}}+\cdots+a_{i_{k}} +$$ + +Summarizing these observations we get the following +Claim. For every $n>r$, we have + +$$ +a_{n}=\max \left\{a_{i_{1}}+\cdots+a_{i_{k}}: \text { the collection }\left(i_{1}, \ldots, i_{k}\right) \text { satisfies }(4)\right\} +$$ + +Now we denote + +$$ +s=\max _{1 \leq i \leq r} \frac{a_{i}}{i} +$$ + +and fix some index $\ell \leq r$ such that $s=\frac{a_{\ell}}{\ell}$. +Consider some $n \geq r^{2} \ell+2 r$ and choose an expansion of $a_{n}$ in the form (2), (4). Then we have $n=i_{1}+\cdots+i_{k} \leq r k$, so $k \geq n / r \geq r \ell+2$. Suppose that none of the numbers $i_{3}, \ldots, i_{k}$ equals $\ell$. Then by the pigeonhole principle there is an index $1 \leq j \leq r$ which appears among $i_{3}, \ldots, i_{k}$ at least $\ell$ times, and surely $j \neq \ell$. Let us delete these $\ell$ occurrences of $j$ from $\left(i_{1}, \ldots, i_{k}\right)$, and add $j$ occurrences of $\ell$ instead, obtaining a sequence $\left(i_{1}, i_{2}, i_{3}^{\prime}, \ldots, i_{k^{\prime}}^{\prime}\right)$ also satisfying (4). By Claim, we have + +$$ +a_{i_{1}}+\cdots+a_{i_{k}}=a_{n} \geq a_{i_{1}}+a_{i_{2}}+a_{i_{3}^{\prime}}+\cdots+a_{i_{k^{\prime}}^{\prime}} +$$ + +or, after removing the coinciding terms, $\ell a_{j} \geq j a_{\ell}$, so $\frac{a_{\ell}}{\ell} \leq \frac{a_{j}}{j}$. By the definition of $\ell$, this means that $\ell a_{j}=j a_{\ell}$, hence + +$$ +a_{n}=a_{i_{1}}+a_{i_{2}}+a_{i_{3}^{\prime}}+\cdots+a_{i_{k^{\prime}}^{\prime}} +$$ + +Thus, for every $n \geq r^{2} \ell+2 r$ we have found a representation of the form (2), (4) with $i_{j}=\ell$ for some $j \geq 3$. Rearranging the indices we may assume that $i_{k}=\ell$. + +Finally, observe that in this representation, the indices $\left(i_{1}, \ldots, i_{k-1}\right)$ satisfy the conditions (4) with $n$ replaced by $n-\ell$. Thus, from the Claim we get + +$$ +a_{n-\ell}+a_{\ell} \geq\left(a_{i_{1}}+\cdots+a_{i_{k-1}}\right)+a_{\ell}=a_{n} +$$ + +which by (1) implies + +$$ +a_{n}=a_{n-\ell}+a_{\ell} \quad \text { for each } n \geq r^{2} \ell+2 r +$$ + +as desired. + +Solution 2. As in the previous solution, we involve the expansion (2), (3), and we fix some index $1 \leq \ell \leq r$ such that + +$$ +\frac{a_{\ell}}{\ell}=s=\max _{1 \leq i \leq r} \frac{a_{i}}{i} +$$ + +Now, we introduce the sequence $\left(b_{n}\right)$ as $b_{n}=a_{n}-s n$; then $b_{\ell}=0$. +We prove by induction on $n$ that $b_{n} \leq 0$, and $\left(b_{n}\right)$ satisfies the same recurrence relation as $\left(a_{n}\right)$. The base cases $n \leq r$ follow from the definition of $s$. Now, for $n>r$ from the induction hypothesis we have + +$$ +b_{n}=\max _{1 \leq k \leq n-1}\left(a_{k}+a_{n-k}\right)-n s=\max _{1 \leq k \leq n-1}\left(b_{k}+b_{n-k}+n s\right)-n s=\max _{1 \leq k \leq n-1}\left(b_{k}+b_{n-k}\right) \leq 0 +$$ + +as required. +Now, if $b_{k}=0$ for all $1 \leq k \leq r$, then $b_{n}=0$ for all $n$, hence $a_{n}=s n$, and the statement is trivial. Otherwise, define + +$$ +M=\max _{1 \leq i \leq r}\left|b_{i}\right|, \quad \varepsilon=\min \left\{\left|b_{i}\right|: 1 \leq i \leq r, b_{i}<0\right\} +$$ + +Then for $n>r$ we obtain + +$$ +b_{n}=\max _{1 \leq k \leq n-1}\left(b_{k}+b_{n-k}\right) \geq b_{\ell}+b_{n-\ell}=b_{n-\ell} +$$ + +so + +$$ +0 \geq b_{n} \geq b_{n-\ell} \geq b_{n-2 \ell} \geq \cdots \geq-M +$$ + +Thus, in view of the expansion (2), (3) applied to the sequence $\left(b_{n}\right)$, we get that each $b_{n}$ is contained in a set + +$$ +T=\left\{b_{i_{1}}+b_{i_{2}}+\cdots+b_{i_{k}}: i_{1}, \ldots, i_{k} \leq r\right\} \cap[-M, 0] +$$ + +We claim that this set is finite. Actually, for any $x \in T$, let $x=b_{i_{1}}+\cdots+b_{i_{k}}\left(i_{1}, \ldots, i_{k} \leq r\right)$. Then among $b_{i_{j}}$ 's there are at most $\frac{M}{\varepsilon}$ nonzero terms (otherwise $x<\frac{M}{\varepsilon} \cdot(-\varepsilon)<-M$ ). Thus $x$ can be expressed in the same way with $k \leq \frac{M}{\varepsilon}$, and there is only a finite number of such sums. + +Finally, for every $t=1,2, \ldots, \ell$ we get that the sequence + +$$ +b_{r+t}, b_{r+t+\ell}, b_{r+t+2 \ell}, \ldots +$$ + +is non-decreasing and attains the finite number of values; therefore it is constant from some index. Thus, the sequence $\left(b_{n}\right)$ is periodic with period $\ell$ from some index $N$, which means that + +$$ +b_{n}=b_{n-\ell}=b_{n-\ell}+b_{\ell} \quad \text { for all } n>N+\ell +$$ + +and hence + +$$ +a_{n}=b_{n}+n s=\left(b_{n-\ell}+(n-\ell) s\right)+\left(b_{\ell}+\ell s\right)=a_{n-\ell}+a_{\ell} \quad \text { for all } n>N+\ell \text {, } +$$ + +as desired. + +A8. Given six positive numbers $a, b, c, d, e, f$ such that $a\sqrt{3(S+T)(S(b d+b f+d f)+T(a c+a e+c e))} +$$ + +(South Korea) +Solution 1. We define also $\sigma=a c+c e+a e, \tau=b d+b f+d f$. The idea of the solution is to interpret (1) as a natural inequality on the roots of an appropriate polynomial. + +Actually, consider the polynomial + +$$ +\begin{aligned} +& P(x)=(b+d+f)(x-a)(x-c)(x-e)+(a+c+e)(x-b)(x-d)(x-f) \\ +&=T\left(x^{3}-S x^{2}+\sigma x-a c e\right)+S\left(x^{3}-T x^{2}+\tau x-b d f\right) +\end{aligned} +$$ + +Surely, $P$ is cubic with leading coefficient $S+T>0$. Moreover, we have + +$$ +\begin{array}{ll} +P(a)=S(a-b)(a-d)(a-f)<0, & P(c)=S(c-b)(c-d)(c-f)>0 \\ +P(e)=S(e-b)(e-d)(e-f)<0, & P(f)=T(f-a)(f-c)(f-e)>0 +\end{array} +$$ + +Hence, each of the intervals $(a, c),(c, e),(e, f)$ contains at least one root of $P(x)$. Since there are at most three roots at all, we obtain that there is exactly one root in each interval (denote them by $\alpha \in(a, c), \beta \in(c, e), \gamma \in(e, f))$. Moreover, the polynomial $P$ can be factorized as + +$$ +P(x)=(T+S)(x-\alpha)(x-\beta)(x-\gamma) +$$ + +Equating the coefficients in the two representations (2) and (3) of $P(x)$ provides + +$$ +\alpha+\beta+\gamma=\frac{2 T S}{T+S}, \quad \alpha \beta+\alpha \gamma+\beta \gamma=\frac{S \tau+T \sigma}{T+S} +$$ + +Now, since the numbers $\alpha, \beta, \gamma$ are distinct, we have + +$$ +0<(\alpha-\beta)^{2}+(\alpha-\gamma)^{2}+(\beta-\gamma)^{2}=2(\alpha+\beta+\gamma)^{2}-6(\alpha \beta+\alpha \gamma+\beta \gamma) +$$ + +which implies + +$$ +\frac{4 S^{2} T^{2}}{(T+S)^{2}}=(\alpha+\beta+\gamma)^{2}>3(\alpha \beta+\alpha \gamma+\beta \gamma)=\frac{3(S \tau+T \sigma)}{T+S} +$$ + +or + +$$ +4 S^{2} T^{2}>3(T+S)(T \sigma+S \tau) +$$ + +which is exactly what we need. +Comment 1. In fact, one can locate the roots of $P(x)$ more narrowly: they should lie in the intervals $(a, b),(c, d),(e, f)$. + +Surely, if we change all inequality signs in the problem statement to non-strict ones, the (non-strict) inequality will also hold by continuity. One can also find when the equality is achieved. This happens in that case when $P(x)$ is a perfect cube, which immediately implies that $b=c=d=e(=\alpha=\beta=\gamma)$, together with the additional condition that $P^{\prime \prime}(b)=0$. Algebraically, + +$$ +\begin{array}{rlr} +6(T+S) b-4 T S=0 & \Longleftrightarrow & 3 b(a+4 b+f)=2(a+2 b)(2 b+f) \\ +& \Longleftrightarrow & f=\frac{b(4 b-a)}{2 a+b}=b\left(1+\frac{3(b-a)}{2 a+b}\right)>b +\end{array} +$$ + +This means that for every pair of numbers $a, b$ such that $0b$ such that the point $(a, b, b, b, b, f)$ is a point of equality. + +Solution 2. Let + +$$ +U=\frac{1}{2}\left((e-a)^{2}+(c-a)^{2}+(e-c)^{2}\right)=S^{2}-3(a c+a e+c e) +$$ + +and + +$$ +V=\frac{1}{2}\left((f-b)^{2}+(f-d)^{2}+(d-b)^{2}\right)=T^{2}-3(b d+b f+d f) +$$ + +Then + +$$ +\begin{aligned} +& \text { (L.H.S. })^{2}-(\text { R.H.S. })^{2}=(2 S T)^{2}-(S+T)(S \cdot 3(b d+b f+d f)+T \cdot 3(a c+a e+c e))= \\ +& \quad=4 S^{2} T^{2}-(S+T)\left(S\left(T^{2}-V\right)+T\left(S^{2}-U\right)\right)=(S+T)(S V+T U)-S T(T-S)^{2} +\end{aligned} +$$ + +and the statement is equivalent with + +$$ +(S+T)(S V+T U)>S T(T-S)^{2} +$$ + +By the Cauchy-Schwarz inequality, + +$$ +(S+T)(T U+S V) \geq(\sqrt{S \cdot T U}+\sqrt{T \cdot S V})^{2}=S T(\sqrt{U}+\sqrt{V})^{2} +$$ + +Estimate the quantities $\sqrt{U}$ and $\sqrt{V}$ by the QM-AM inequality with the positive terms $(e-c)^{2}$ and $(d-b)^{2}$ being omitted: + +$$ +\begin{aligned} +\sqrt{U}+\sqrt{V} & >\sqrt{\frac{(e-a)^{2}+(c-a)^{2}}{2}}+\sqrt{\frac{(f-b)^{2}+(f-d)^{2}}{2}} \\ +& >\frac{(e-a)+(c-a)}{2}+\frac{(f-b)+(f-d)}{2}=\left(f-\frac{d}{2}-\frac{b}{2}\right)+\left(\frac{e}{2}+\frac{c}{2}-a\right) \\ +& =(T-S)+\frac{3}{2}(e-d)+\frac{3}{2}(c-b)>T-S . +\end{aligned} +$$ + +The estimates (5) and (6) prove (4) and hence the statement. +Solution 3. We keep using the notations $\sigma$ and $\tau$ from Solution 1. Moreover, let $s=c+e$. Note that + +$$ +(c-b)(c-d)+(e-f)(e-d)+(e-f)(c-b)<0 +$$ + +since each summand is negative. This rewrites as + +$$ +\begin{gathered} +(b d+b f+d f)-(a c+c e+a e)<(c+e)(b+d+f-a-c-e), \text { or } \\ +\tau-\sigma\sqrt{3(S+T)(S(b d+b f+d f)+T(a c+a e+c e))} +$$ + +Comment 2. The expression (7) can be found by considering the sum of the roots of the quadratic polynomial $q(x)=(x-b)(x-d)(x-f)-(x-a)(x-c)(x-e)$. + +Solution 4. We introduce the expressions $\sigma$ and $\tau$ as in the previous solutions. The idea of the solution is to change the values of variables $a, \ldots, f$ keeping the left-hand side unchanged and increasing the right-hand side; it will lead to a simpler inequality which can be proved in a direct way. + +Namely, we change the variables (i) keeping the (non-strict) inequalities $a \leq b \leq c \leq d \leq$ $e \leq f$; (ii) keeping the values of sums $S$ and $T$ unchanged; and finally (iii) increasing the values of $\sigma$ and $\tau$. Then the left-hand side of (1) remains unchanged, while the right-hand side increases. Hence, the inequality (1) (and even a non-strict version of (1)) for the changed values would imply the same (strict) inequality for the original values. + +First, we find the sufficient conditions for (ii) and (iii) to be satisfied. +Lemma. Let $x, y, z>0$; denote $U(x, y, z)=x+y+z, v(x, y, z)=x y+x z+y z$. Suppose that $x^{\prime}+y^{\prime}=x+y$ but $|x-y| \geq\left|x^{\prime}-y^{\prime}\right| ;$ then we have $U\left(x^{\prime}, y^{\prime}, z\right)=U(x, y, z)$ and $v\left(x^{\prime}, y^{\prime}, z\right) \geq$ $v(x, y, z)$ with equality achieved only when $|x-y|=\left|x^{\prime}-y^{\prime}\right|$. +Proof. The first equality is obvious. For the second, we have + +$$ +\begin{aligned} +v\left(x^{\prime}, y^{\prime}, z\right)=z\left(x^{\prime}+y^{\prime}\right)+x^{\prime} y^{\prime} & =z\left(x^{\prime}+y^{\prime}\right)+\frac{\left(x^{\prime}+y^{\prime}\right)^{2}-\left(x^{\prime}-y^{\prime}\right)^{2}}{4} \\ +& \geq z(x+y)+\frac{(x+y)^{2}-(x-y)^{2}}{4}=v(x, y, z) +\end{aligned} +$$ + +with the equality achieved only for $\left(x^{\prime}-y^{\prime}\right)^{2}=(x-y)^{2} \Longleftrightarrow\left|x^{\prime}-y^{\prime}\right|=|x-y|$, as desired. + +Now, we apply Lemma several times making the following changes. For each change, we denote the new values by the same letters to avoid cumbersome notations. + +1. Let $k=\frac{d-c}{2}$. Replace $(b, c, d, e)$ by $(b+k, c+k, d-k, e-k)$. After the change we have $a2^{N-2}$. Consider the collection of all $2^{N-2}$ flags having yellow first squares and blue second ones. Obviously, both colors appear on the diagonal of each $N \times N$ square formed by these flags. + +We are left to show that $M_{N} \leq 2^{N-2}+1$, thus obtaining the desired answer. We start with establishing this statement for $N=4$. + +Suppose that we have 5 flags of length 4 . We decompose each flag into two parts of 2 squares each; thereby, we denote each flag as $L R$, where the $2 \times 1$ flags $L, R \in \mathcal{S}=\{\mathrm{BB}, \mathrm{BY}, \mathrm{YB}, \mathrm{YY}\}$ are its left and right parts, respectively. First, we make two easy observations on the flags $2 \times 1$ which can be checked manually. +(i) For each $A \in \mathcal{S}$, there exists only one $2 \times 1$ flag $C \in \mathcal{S}$ (possibly $C=A$ ) such that $A$ and $C$ cannot form a $2 \times 2$ square with monochrome diagonal (for part BB, that is YY, and for BY that is YB). +(ii) Let $A_{1}, A_{2}, A_{3} \in \mathcal{S}$ be three distinct elements; then two of them can form a $2 \times 2$ square with yellow diagonal, and two of them can form a $2 \times 2$ square with blue diagonal (for all parts but BB , a pair $(\mathrm{BY}, \mathrm{YB})$ fits for both statements, while for all parts but BY, these pairs are $(\mathrm{YB}, \mathrm{YY})$ and (BB, YB)). + +Now, let $\ell$ and $r$ be the numbers of distinct left and right parts of our 5 flags, respectively. The total number of flags is $5 \leq r \ell$, hence one of the factors (say, $r$ ) should be at least 3 . On the other hand, $\ell, r \leq 4$, so there are two flags with coinciding right part; let them be $L_{1} R_{1}$ and $L_{2} R_{1}\left(L_{1} \neq L_{2}\right)$. + +Next, since $r \geq 3$, there exist some flags $L_{3} R_{3}$ and $L_{4} R_{4}$ such that $R_{1}, R_{3}, R_{4}$ are distinct. Let $L^{\prime} R^{\prime}$ be the remaining flag. By (i), one of the pairs $\left(L^{\prime}, L_{1}\right)$ and $\left(L^{\prime}, L_{2}\right)$ can form a $2 \times 2$ square with monochrome diagonal; we can assume that $L^{\prime}, L_{2}$ form a square with a blue diagonal. Finally, the right parts of two of the flags $L_{1} R_{1}, L_{3} R_{3}, L_{4} R_{4}$ can also form a $2 \times 2$ square with a blue diagonal by (ii). Putting these $2 \times 2$ squares on the diagonal of a $4 \times 4$ square, we find a desired arrangement of four flags. + +We are ready to prove the problem statement by induction on $N$; actually, above we have proved the base case $N=4$. For the induction step, assume that $N>4$, consider any $2^{N-2}+1$ flags of length $N$, and arrange them into a large flag of size $\left(2^{N-2}+1\right) \times N$. This flag contains a non-monochrome column since the flags are distinct; we may assume that this column is the first one. By the pigeonhole principle, this column contains at least $\left\lceil\frac{2^{N-2}+1}{2}\right\rceil=2^{N-3}+1$ squares of one color (say, blue). We call the flags with a blue first square good. + +Consider all the good flags and remove the first square from each of them. We obtain at least $2^{N-3}+1 \geq M_{N-1}$ flags of length $N-1$; by the induction hypothesis, $N-1$ of them +can form a square $Q$ with the monochrome diagonal. Now, returning the removed squares, we obtain a rectangle $(N-1) \times N$, and our aim is to supplement it on the top by one more flag. + +If $Q$ has a yellow diagonal, then we can take each flag with a yellow first square (it exists by a choice of the first column; moreover, it is not used in $Q$ ). Conversely, if the diagonal of $Q$ is blue then we can take any of the $\geq 2^{N-3}+1-(N-1)>0$ remaining good flags. So, in both cases we get a desired $N \times N$ square. + +Solution 2. We present a different proof of the estimate $M_{N} \leq 2^{N-2}+1$. We do not use the induction, involving Hall's lemma on matchings instead. + +Consider arbitrary $2^{N-2}+1$ distinct flags and arrange them into a large $\left(2^{N-2}+1\right) \times N$ flag. Construct two bipartite graphs $G_{\mathrm{y}}=\left(V \cup V^{\prime}, E_{\mathrm{y}}\right)$ and $G_{\mathrm{b}}=\left(V \cup V^{\prime}, E_{\mathrm{b}}\right)$ with the common set of vertices as follows. Let $V$ and $V^{\prime}$ be the set of columns and the set of flags under consideration, respectively. Next, let the edge $(c, f)$ appear in $E_{\mathrm{y}}$ if the intersection of column $c$ and flag $f$ is yellow, and $(c, f) \in E_{\mathrm{b}}$ otherwise. Then we have to prove exactly that one of the graphs $G_{\mathrm{y}}$ and $G_{\mathrm{b}}$ contains a matching with all the vertices of $V$ involved. + +Assume that these matchings do not exist. By Hall's lemma, it means that there exist two sets of columns $S_{\mathrm{y}}, S_{\mathrm{b}} \subset V$ such that $\left|E_{\mathrm{y}}\left(S_{\mathrm{y}}\right)\right| \leq\left|S_{\mathrm{y}}\right|-1$ and $\left|E_{\mathrm{b}}\left(S_{\mathrm{b}}\right)\right| \leq\left|S_{\mathrm{b}}\right|-1$ (in the left-hand sides, $E_{\mathrm{y}}\left(S_{\mathrm{y}}\right)$ and $E_{\mathrm{b}}\left(S_{\mathrm{b}}\right)$ denote respectively the sets of all vertices connected to $S_{\mathrm{y}}$ and $S_{\mathrm{b}}$ in the corresponding graphs). Our aim is to prove that this is impossible. Note that $S_{\mathrm{y}}, S_{\mathrm{b}} \neq V$ since $N \leq 2^{N-2}+1$. + +First, suppose that $S_{\mathrm{y}} \cap S_{\mathrm{b}} \neq \varnothing$, so there exists some $c \in S_{\mathrm{y}} \cap S_{\mathrm{b}}$. Note that each flag is connected to $c$ either in $G_{\mathrm{y}}$ or in $G_{\mathrm{b}}$, hence $E_{\mathrm{y}}\left(S_{\mathrm{y}}\right) \cup E_{\mathrm{b}}\left(S_{\mathrm{b}}\right)=V^{\prime}$. Hence we have $2^{N-2}+1=\left|V^{\prime}\right| \leq\left|E_{\mathrm{y}}\left(S_{\mathrm{y}}\right)\right|+\left|E_{\mathrm{b}}\left(S_{\mathrm{b}}\right)\right| \leq\left|S_{\mathrm{y}}\right|+\left|S_{\mathrm{b}}\right|-2 \leq 2 N-4$; this is impossible for $N \geq 4$. + +So, we have $S_{\mathrm{y}} \cap S_{\mathrm{b}}=\varnothing$. Let $y=\left|S_{\mathrm{y}}\right|, b=\left|S_{\mathrm{b}}\right|$. From the construction of our graph, we have that all the flags in the set $V^{\prime \prime}=V^{\prime} \backslash\left(E_{\mathrm{y}}\left(S_{\mathrm{y}}\right) \cup E_{\mathrm{b}}\left(S_{\mathrm{b}}\right)\right)$ have blue squares in the columns of $S_{\mathrm{y}}$ and yellow squares in the columns of $S_{\mathrm{b}}$. Hence the only undetermined positions in these flags are the remaining $N-y-b$ ones, so $2^{N-y-b} \geq\left|V^{\prime \prime}\right| \geq\left|V^{\prime}\right|-\left|E_{\mathrm{y}}\left(S_{\mathrm{y}}\right)\right|-\left|E_{\mathrm{b}}\left(S_{\mathrm{b}}\right)\right| \geq$ $2^{N-2}+1-(y-1)-(b-1)$, or, denoting $c=y+b, 2^{N-c}+c>2^{N-2}+2$. This is impossible since $N \geq c \geq 2$. + +C3. 2500 chess kings have to be placed on a $100 \times 100$ chessboard so that +(i) no king can capture any other one (i.e. no two kings are placed in two squares sharing a common vertex); +(ii) each row and each column contains exactly 25 kings. + +Find the number of such arrangements. (Two arrangements differing by rotation or symmetry are supposed to be different.) +(Russia) +Answer. There are two such arrangements. +Solution. Suppose that we have an arrangement satisfying the problem conditions. Divide the board into $2 \times 2$ pieces; we call these pieces blocks. Each block can contain not more than one king (otherwise these two kings would attack each other); hence, by the pigeonhole principle each block must contain exactly one king. + +Now assign to each block a letter T or B if a king is placed in its top or bottom half, respectively. Similarly, assign to each block a letter L or R if a king stands in its left or right half. So we define $T$-blocks, $B$-blocks, $L$-blocks, and $R$-blocks. We also combine the letters; we call a block a TL-block if it is simultaneously T-block and L-block. Similarly we define TR-blocks, $B L$-blocks, and BR-blocks. The arrangement of blocks determines uniquely the arrangement of kings; so in the rest of the solution we consider the $50 \times 50$ system of blocks (see Fig. 1). We identify the blocks by their coordinate pairs; the pair $(i, j)$, where $1 \leq i, j \leq 50$, refers to the $j$ th block in the $i$ th row (or the $i$ th block in the $j$ th column). The upper-left block is $(1,1)$. + +The system of blocks has the following properties.. +( $\left.\mathrm{i}^{\prime}\right)$ If $(i, j)$ is a B-block then $(i+1, j)$ is a B-block: otherwise the kings in these two blocks can take each other. Similarly: if $(i, j)$ is a T-block then $(i-1, j)$ is a T-block; if $(i, j)$ is an L-block then $(i, j-1)$ is an L-block; if $(i, j)$ is an R-block then $(i, j+1)$ is an R-block. +(ii') Each column contains exactly 25 L-blocks and 25 R-blocks, and each row contains exactly 25 T -blocks and 25 B-blocks. In particular, the total number of L-blocks (or R-blocks, or T-blocks, or B-blocks) is equal to $25 \cdot 50=1250$. + +Consider any B-block of the form $(1, j)$. By ( $\mathrm{i}^{\prime}$ ), all blocks in the $j$ th column are B-blocks; so we call such a column $B$-column. By (ii'), we have 25 B-blocks in the first row, so we obtain 25 B-columns. These 25 B-columns contain 1250 B-blocks, hence all blocks in the remaining columns are T-blocks, and we obtain 25 T-columns. Similarly, there are exactly $25 L$-rows and exactly $25 R$-rows. + +Now consider an arbitrary pair of a T-column and a neighboring B-column (columns with numbers $j$ and $j+1$ ). +![](https://cdn.mathpix.com/cropped/2024_11_18_ba86565be3364967ae6fg-29.jpg?height=343&width=778&top_left_y=2127&top_left_x=436) + +Fig. 1 +![](https://cdn.mathpix.com/cropped/2024_11_18_ba86565be3364967ae6fg-29.jpg?height=320&width=361&top_left_y=2150&top_left_x=1330) + +Fig. 2 + +Case 1. Suppose that the $j$ th column is a T-column, and the $(j+1)$ th column is a Bcolumn. Consider some index $i$ such that the $i$ th row is an L-row; then $(i, j+1)$ is a BL-block. Therefore, $(i+1, j)$ cannot be a TR-block (see Fig. 2), hence $(i+1, j)$ is a TL-block, thus the +$(i+1)$ th row is an L-row. Now, choosing the $i$ th row to be the topmost L-row, we successively obtain that all rows from the $i$ th to the 50 th are L-rows. Since we have exactly 25 L-rows, it follows that the rows from the 1st to the 25 th are R-rows, and the rows from the 26 th to the 50th are L-rows. + +Now consider the neighboring R-row and L-row (that are the rows with numbers 25 and 26). Replacing in the previous reasoning rows by columns and vice versa, the columns from the 1 st to the 25 th are T-columns, and the columns from the 26 th to the 50 th are B-columns. So we have a unique arrangement of blocks that leads to the arrangement of kings satisfying the condition of the problem (see Fig. 3). +![](https://cdn.mathpix.com/cropped/2024_11_18_ba86565be3364967ae6fg-30.jpg?height=521&width=1020&top_left_y=796&top_left_x=204) + +Fig. 3 +![](https://cdn.mathpix.com/cropped/2024_11_18_ba86565be3364967ae6fg-30.jpg?height=524&width=509&top_left_y=792&top_left_x=1296) + +Fig. 4 + +Case 2. Suppose that the $j$ th column is a B-column, and the $(j+1)$ th column is a T-column. Repeating the arguments from Case 1, we obtain that the rows from the 1st to the 25th are L-rows (and all other rows are R-rows), the columns from the 1st to the 25 th are B-columns (and all other columns are T-columns), so we find exactly one more arrangement of kings (see Fig. 4). + +C4. Six stacks $S_{1}, \ldots, S_{6}$ of coins are standing in a row. In the beginning every stack contains a single coin. There are two types of allowed moves: +Move 1: If stack $S_{k}$ with $1 \leq k \leq 5$ contains at least one coin, you may remove one coin from $S_{k}$ and add two coins to $S_{k+1}$. +Move 2: If stack $S_{k}$ with $1 \leq k \leq 4$ contains at least one coin, then you may remove one coin from $S_{k}$ and exchange stacks $S_{k+1}$ and $S_{k+2}$. +Decide whether it is possible to achieve by a sequence of such moves that the first five stacks are empty, whereas the sixth stack $S_{6}$ contains exactly $2010^{2010^{2010}}$ coins. + +C4 ${ }^{\prime}$. Same as Problem C4, but the constant $2010^{2010^{2010}}$ is replaced by $2010^{2010}$. +(Netherlands) +Answer. Yes (in both variants of the problem). There exists such a sequence of moves. +Solution. Denote by $\left(a_{1}, a_{2}, \ldots, a_{n}\right) \rightarrow\left(a_{1}^{\prime}, a_{2}^{\prime}, \ldots, a_{n}^{\prime}\right)$ the following: if some consecutive stacks contain $a_{1}, \ldots, a_{n}$ coins, then it is possible to perform several allowed moves such that the stacks contain $a_{1}^{\prime}, \ldots, a_{n}^{\prime}$ coins respectively, whereas the contents of the other stacks remain unchanged. + +Let $A=2010^{2010}$ or $A=2010^{2010^{2010}}$, respectively. Our goal is to show that + +$$ +(1,1,1,1,1,1) \rightarrow(0,0,0,0,0, A) +$$ + +First we prove two auxiliary observations. +Lemma 1. $(a, 0,0) \rightarrow\left(0,2^{a}, 0\right)$ for every $a \geq 1$. +Proof. We prove by induction that $(a, 0,0) \rightarrow\left(a-k, 2^{k}, 0\right)$ for every $1 \leq k \leq a$. For $k=1$, apply Move 1 to the first stack: + +$$ +(a, 0,0) \rightarrow(a-1,2,0)=\left(a-1,2^{1}, 0\right) +$$ + +Now assume that $k0$, while in the latter one $S(T)=1^{3}+1^{3}+(-1)^{3}+(-1)^{3}=0$, as desired. + +Now we turn to the general problem. Consider a tournament $T$ with no bad companies and enumerate the players by the numbers from 1 to $n$. For every 4 players $i_{1}, i_{2}, i_{3}, i_{4}$ consider a "sub-tournament" $T_{i_{1} i_{2} i_{3} i_{4}}$ consisting of only these players and the games which they performed with each other. By the abovementioned, we have $S\left(T_{i_{1} i_{2} i_{3} i_{4}}\right) \geq 0$. Our aim is to prove that + +$$ +S(T)=\sum_{i_{1}, i_{2}, i_{3}, i_{4}} S\left(T_{i_{1} i_{2} i_{3} i_{4}}\right) +$$ + +where the sum is taken over all 4 -tuples of distinct numbers from the set $\{1, \ldots, n\}$. This way the problem statement will be established. + +We interpret the number $\left(w_{i}-\ell_{i}\right)^{3}$ as following. For $i \neq j$, let $\varepsilon_{i j}=1$ if the $i$ th player wins against the $j$ th one, and $\varepsilon_{i j}=-1$ otherwise. Then + +$$ +\left(w_{i}-\ell_{i}\right)^{3}=\left(\sum_{j \neq i} \varepsilon_{i j}\right)^{3}=\sum_{j_{1}, j_{2}, j_{3} \neq i} \varepsilon_{i j_{1}} \varepsilon_{i j_{2}} \varepsilon_{i j_{3}} . +$$ + +Hence, + +$$ +S(T)=\sum_{i \notin\left\{j_{1}, j_{2}, j_{3}\right\}} \varepsilon_{i j_{1}} \varepsilon_{i j_{2}} \varepsilon_{i j_{3}} +$$ + +To simplify this expression, consider all the terms in this sum where two indices are equal. If, for instance, $j_{1}=j_{2}$, then the term contains $\varepsilon_{i j_{1}}^{2}=1$, so we can replace this term by $\varepsilon_{i j_{3}}$. Make such replacements for each such term; obviously, after this change each term of the form $\varepsilon_{i j_{3}}$ will appear $P(T)$ times, hence + +$$ +S(T)=\sum_{\left|\left\{i, j_{1}, j_{2}, j_{3}\right\}\right|=4} \varepsilon_{i j_{1}} \varepsilon_{i j_{2}} \varepsilon_{i j_{3}}+P(T) \sum_{i \neq j} \varepsilon_{i j}=S_{1}(T)+P(T) S_{2}(T) +$$ + +We show that $S_{2}(T)=0$ and hence $S(T)=S_{1}(T)$ for each tournament. Actually, note that $\varepsilon_{i j}=-\varepsilon_{j i}$, and the whole sum can be split into such pairs. Since the sum in each pair is 0 , so is $S_{2}(T)$. + +Thus the desired equality (2) rewrites as + +$$ +S_{1}(T)=\sum_{i_{1}, i_{2}, i_{3}, i_{4}} S_{1}\left(T_{i_{1} i_{2} i_{3} i_{4}}\right) +$$ + +Now, if all the numbers $j_{1}, j_{2}, j_{3}$ are distinct, then the set $\left\{i, j_{1}, j_{2}, j_{3}\right\}$ is contained in exactly one 4 -tuple, hence the term $\varepsilon_{i j_{1}} \varepsilon_{i j_{2}} \varepsilon_{i j_{3}}$ appears in the right-hand part of (3) exactly once, as well as in the left-hand part. Clearly, there are no other terms in both parts, so the equality is established. + +Solution 2. Similarly to the first solution, we call the subsets of players as companies, and the $k$-element subsets will be called as $k$-companies. + +In any company of the players, call a player the local champion of the company if he defeated all other members of the company. Similarly, if a player lost all his games against the others in the company then call him the local loser of the company. Obviously every company has at most one local champion and at most one local loser. By the condition of the problem, whenever a 4-company has a local loser, then this company has a local champion as well. + +Suppose that $k$ is some positive integer, and let us count all cases when a player is the local champion of some $k$-company. The $i$ th player won against $w_{i}$ other player. To be the local champion of a $k$-company, he must be a member of the company, and the other $k-1$ members must be chosen from those whom he defeated. Therefore, the $i$ th player is the local champion of $\binom{w_{i}}{k-1} k$-companies. Hence, the total number of local champions of all $k$-companies is $\sum_{i=1}^{n}\binom{w_{i}}{k-1}$. + +Similarly, the total number of local losers of the $k$-companies is $\sum_{i=1}^{n}\binom{\ell_{i}}{k-1}$. +Now apply this for $k=2,3$ and 4. +Since every game has a winner and a loser, we have $\sum_{i=1}^{n} w_{i}=\sum_{i=1}^{n} \ell_{i}=\binom{n}{2}$, and hence + +$$ +\sum_{i=1}^{n}\left(w_{i}-\ell_{i}\right)=0 +$$ + +In every 3-company, either the players defeated one another in a cycle or the company has both a local champion and a local loser. Therefore, the total number of local champions and local losers in the 3-companies is the same, $\sum_{i=1}^{n}\binom{w_{i}}{2}=\sum_{i=1}^{n}\binom{\ell_{i}}{2}$. So we have + +$$ +\sum_{i=1}^{n}\left(\binom{w_{i}}{2}-\binom{\ell_{i}}{2}\right)=0 +$$ + +In every 4-company, by the problem's condition, the number of local losers is less than or equal to the number of local champions. Then the same holds for the total numbers of local +champions and local losers in all 4-companies, so $\sum_{i=1}^{n}\binom{w_{i}}{3} \geq \sum_{i=1}^{n}\binom{\ell_{i}}{3}$. Hence, + +$$ +\sum_{i=1}^{n}\left(\binom{w_{i}}{3}-\binom{\ell_{i}}{3}\right) \geq 0 +$$ + +Now we establish the problem statement (1) as a linear combination of (4), (5) and (6). It is easy check that + +$$ +(x-y)^{3}=24\left(\binom{x}{3}-\binom{y}{3}\right)+24\left(\binom{x}{2}-\binom{y}{2}\right)-\left(3(x+y)^{2}-4\right)(x-y) +$$ + +Apply this identity to $x=w_{1}$ and $y=\ell_{i}$. Since every player played $n-1$ games, we have $w_{i}+\ell_{i}=n-1$, and thus + +$$ +\left(w_{i}-\ell_{i}\right)^{3}=24\left(\binom{w_{i}}{3}-\binom{\ell_{i}}{3}\right)+24\left(\binom{w_{i}}{2}-\binom{\ell_{i}}{2}\right)-\left(3(n-1)^{2}-4\right)\left(w_{i}-\ell_{i}\right) +$$ + +Then + +$$ +\sum_{i=1}^{n}\left(w_{i}-\ell_{i}\right)^{3}=24 \underbrace{\sum_{i=1}^{n}\left(\binom{w_{i}}{3}-\binom{\ell_{i}}{3}\right)}_{\geq 0}+24 \underbrace{\sum_{i=1}^{n}\left(\binom{w_{i}}{2}-\binom{\ell_{i}}{2}\right)}_{0}-\left(3(n-1)^{2}-4\right) \underbrace{\sum_{i=1}^{n}\left(w_{i}-\ell_{i}\right)}_{0} \geq 0 +$$ + +C6. Given a positive integer $k$ and other two integers $b>w>1$. There are two strings of pearls, a string of $b$ black pearls and a string of $w$ white pearls. The length of a string is the number of pearls on it. + +One cuts these strings in some steps by the following rules. In each step: +(i) The strings are ordered by their lengths in a non-increasing order. If there are some strings of equal lengths, then the white ones precede the black ones. Then $k$ first ones (if they consist of more than one pearl) are chosen; if there are less than $k$ strings longer than 1 , then one chooses all of them. +(ii) Next, one cuts each chosen string into two parts differing in length by at most one. +(For instance, if there are strings of $5,4,4,2$ black pearls, strings of $8,4,3$ white pearls and $k=4$, then the strings of 8 white, 5 black, 4 white and 4 black pearls are cut into the parts $(4,4),(3,2),(2,2)$ and $(2,2)$, respectively.) + +The process stops immediately after the step when a first isolated white pearl appears. Prove that at this stage, there will still exist a string of at least two black pearls. +(Canada) +Solution 1. Denote the situation after the $i$ th step by $A_{i}$; hence $A_{0}$ is the initial situation, and $A_{i-1} \rightarrow A_{i}$ is the $i$ th step. We call a string containing $m$ pearls an $m$-string; it is an $m$-w-string or a $m$-b-string if it is white or black, respectively. + +We continue the process until every string consists of a single pearl. We will focus on three moments of the process: (a) the first stage $A_{s}$ when the first 1 -string (no matter black or white) appears; (b) the first stage $A_{t}$ where the total number of strings is greater than $k$ (if such moment does not appear then we put $t=\infty$ ); and (c) the first stage $A_{f}$ when all black pearls are isolated. It is sufficient to prove that in $A_{f-1}$ (or earlier), a 1-w-string appears. + +We start with some easy properties of the situations under consideration. Obviously, we have $s \leq f$. Moreover, all b-strings from $A_{f-1}$ become single pearls in the $f$ th step, hence all of them are 1- or 2-b-strings. + +Next, observe that in each step $A_{i} \rightarrow A_{i+1}$ with $i \leq t-1$, all $(>1)$-strings were cut since there are not more than $k$ strings at all; if, in addition, $i1$ as $s-1 \leq \min \{s, t\}$. Now, if $s=f$, then in $A_{s-1}$, there is no 1 -w-string as well as no ( $>2$ )-b-string. That is, $2=B_{s-1} \geq W_{s-1} \geq b_{s-1} \geq w_{s-1}>1$, hence all these numbers equal 2 . This means that in $A_{s-1}$, all strings contain 2 pearls, and there are $2^{s-1}$ black and $2^{s-1}$ white strings, which means $b=2 \cdot 2^{s-1}=w$. This contradicts the problem conditions. + +Hence we have $s \leq f-1$ and thus $s \leq t$. Therefore, in the $s$ th step each string is cut into two parts. Now, if a 1-b-string appears in this step, then from $w_{s-1} \leq b_{s-1}$ we see that a + +1-w-string appears as well; so, in each case in the sth step a 1-w-string appears, while not all black pearls become single, as desired. + +Case 2. Now assume that $t+1 \leq s$ and $t+2 \leq f$. Then in $A_{t}$ we have exactly $2^{t}$ white and $2^{t}$ black strings, all being larger than 1 , and $2^{t+1}>k \geq 2^{t}$ (the latter holds since $2^{t}$ is the total number of strings in $A_{t-1}$ ). Now, in the $(t+1)$ st step, exactly $k$ strings are cut, not more than $2^{t}$ of them being black; so the number of w-strings in $A_{t+1}$ is at least $2^{t}+\left(k-2^{t}\right)=k$. Since the number of w-strings does not decrease in our process, in $A_{f-1}$ we have at least $k$ white strings as well. + +Finally, in $A_{f-1}$, all b-strings are not larger than 2, and at least one 2-b-string is cut in the $f$ th step. Therefore, at most $k-1$ white strings are cut in this step, hence there exists a w-string $\mathcal{W}$ which is not cut in the $f$ th step. On the other hand, since a 2 -b-string is cut, all $(\geq 2)$-w-strings should also be cut in the $f$ th step; hence $\mathcal{W}$ should be a single pearl. This is exactly what we needed. + +Comment. In this solution, we used the condition $b \neq w$ only to avoid the case $b=w=2^{t}$. Hence, if a number $b=w$ is not a power of 2 , then the problem statement is also valid. + +Solution 2. We use the same notations as introduced in the first paragraph of the previous solution. We claim that at every stage, there exist a $u$-b-string and a $v$-w-string such that either +(i) $u>v \geq 1$, or +(ii) $2 \leq u \leq v<2 u$, and there also exist $k-1$ of ( $>v / 2$ )-strings other than considered above. + +First, we notice that this statement implies the problem statement. Actually, in both cases (i) and (ii) we have $u>1$, so at each stage there exists a ( $\geq 2$ )-b-string, and for the last stage it is exactly what we need. + +Now, we prove the claim by induction on the number of the stage. Obviously, for $A_{0}$ the condition (i) holds since $b>w$. Further, we suppose that the statement holds for $A_{i}$, and prove it for $A_{i+1}$. Two cases are possible. + +Case 1. Assume that in $A_{i}$, there are a $u$-b-string and a $v$-w-string with $u>v$. We can assume that $v$ is the length of the shortest w-string in $A_{i}$; since we are not at the final stage, we have $v \geq 2$. Now, in the $(i+1)$ st step, two subcases may occur. + +Subcase 1a. Suppose that either no $u$-b-string is cut, or both some $u$-b-string and some $v$-w-string are cut. Then in $A_{i+1}$, we have either a $u$-b-string and a $(\leq v)$-w-string (and (i) is valid), or we have a $\lceil u / 2\rceil$-b-string and a $\lfloor v / 2\rfloor$-w-string. In the latter case, from $u>v$ we get $\lceil u / 2\rceil>\lfloor v / 2\rfloor$, and (i) is valid again. + +Subcase $1 b$. Now, some $u$-b-string is cut, and no $v$-w-string is cut (and hence all the strings which are cut are longer than $v$ ). If $u^{\prime}=\lceil u / 2\rceil>v$, then the condition (i) is satisfied since we have a $u^{\prime}$-b-string and a $v$-w-string in $A_{i+1}$. Otherwise, notice that the inequality $u>v \geq 2$ implies $u^{\prime} \geq 2$. Furthermore, besides a fixed $u$-b-string, other $k-1$ of $(\geq v+1)$-strings should be cut in the $(i+1)$ st step, hence providing at least $k-1$ of $(\geq\lceil(v+1) / 2\rceil)$-strings, and $\lceil(v+1) / 2\rceil>v / 2$. So, we can put $v^{\prime}=v$, and we have $u^{\prime} \leq vv$, so each one results in a ( $>v / 2$ )string. Hence in $A_{i+1}$, there exist $k \geq k-1$ of $(>v / 2)$-strings other than the considered $u$ - and $v$-strings, and the condition (ii) is satisfied. + +Subcase 2c. In the remaining case, all $u$-b-strings are cut. This means that all $(\geq u)$-strings are cut as well, hence our $v$-w-string is cut. Therefore in $A_{i+1}$ there exists a $\lceil u / 2\rceil$-b-string together with a $\lfloor v / 2\rfloor$-w-string. Now, if $u^{\prime}=\lceil u / 2\rceil>\lfloor v / 2\rfloor=v^{\prime}$ then the condition (i) is fulfilled. Otherwise, we have $u^{\prime} \leq v^{\prime}$ $|f(W)|$; if there is no such set then we set $W=\varnothing$. Denote $W^{\prime}=f(W), U=V \backslash W, U^{\prime}=V^{\prime} \backslash W^{\prime}$. + +By our assumption and the Lemma condition, $|f(V)|=\left|V^{\prime}\right| \geq|V|$, hence $W \neq V$ and $U \neq \varnothing$. Permuting the coordinates, we can assume that $U^{\prime}=\left\{v_{i j}: 1 \leq i \leq \ell\right\}, W^{\prime}=\left\{v_{i j}: \ell+1 \leq i \leq k\right\}$. + +Consider the induced subgraph $G^{\prime}$ of $G$ on the vertices $U \cup U^{\prime}$. We claim that for every $X \subset U$, we get $\left|f(X) \cap U^{\prime}\right| \geq|X|$ (so $G^{\prime}$ satisfies the conditions of Hall's lemma). Actually, we have $|W| \geq|f(W)|$, so if $|X|>\left|f(X) \cap U^{\prime}\right|$ for some $X \subset U$, then we have + +$$ +|W \cup X|=|W|+|X|>|f(W)|+\left|f(X) \cap U^{\prime}\right|=\left|f(W) \cup\left(f(X) \cap U^{\prime}\right)\right|=|f(W \cup X)| +$$ + +This contradicts the maximality of $|W|$. +Thus, applying Hall's lemma, we can assign to each $L \in U$ some vertex $v_{i j} \in U^{\prime}$ so that to distinct elements of $U$, distinct vertices of $U^{\prime}$ are assigned. In this situation, we say that $L \in U$ corresponds to the $i$ th axis, and write $g(L)=i$. Since there are $n_{i}-1$ vertices of the form $v_{i j}$, we get that for each $1 \leq i \leq \ell$, not more than $n_{i}-1$ subgrids correspond to the $i$ th axis. + +Finally, we are ready to present the desired point. Since $W \neq V$, there exists a point $b=\left(b_{1}, b_{2}, \ldots, b_{k}\right) \in N \backslash\left(\cup_{L \in W} L\right)$. On the other hand, for every $1 \leq i \leq \ell$, consider any subgrid $L \in U$ with $g(L)=i$. This means exactly that $L$ is orthogonal to the $i$ th axis, and hence all its elements have the same $i$ th coordinate $c_{L}$. Since there are at most $n_{i}-1$ such subgrids, there exists a number $0 \leq a_{i} \leq n_{i}-1$ which is not contained in a set $\left\{c_{L}: g(L)=i\right\}$. Choose such number for every $1 \leq i \leq \ell$. Now we claim that point $a=\left(a_{1}, \ldots, a_{\ell}, b_{\ell+1}, \ldots, b_{k}\right)$ is not covered, hence contradicting the Lemma condition. + +Surely, point $a$ cannot lie in some $L \in U$, since all the points in $L$ have $g(L)$ th coordinate $c_{L} \neq a_{g(L)}$. On the other hand, suppose that $a \in L$ for some $L \in W$; recall that $b \notin L$. But the points $a$ and $b$ differ only at first $\ell$ coordinates, so $L$ should be orthogonal to at least one of the first $\ell$ axes, and hence our graph contains some edge $\left(L, v_{i j}\right)$ for $i \leq \ell$. It contradicts the definition of $W^{\prime}$. The Lemma is proved. + +Now we turn to the problem. Let $d_{j}$ be the step of the progression $P_{j}$. Note that since $n=$ l.c.m. $\left(d_{1}, \ldots, d_{s}\right)$, for each $1 \leq i \leq k$ there exists an index $j(i)$ such that $p_{i}^{\alpha_{i}} \mid d_{j(i)}$. We assume that $n>1$; otherwise the problem statement is trivial. + +For each $0 \leq m \leq n-1$ and $1 \leq i \leq k$, let $m_{i}$ be the residue of $m$ modulo $p_{i}^{\alpha_{i}}$, and let $m_{i}=\overline{r_{i \alpha_{i}} \ldots r_{i 1}}$ be the base $p_{i}$ representation of $m_{i}$ (possibly, with some leading zeroes). Now, we put into correspondence to $m$ the sequence $r(m)=\left(r_{11}, \ldots, r_{1 \alpha_{1}}, r_{21}, \ldots, r_{k \alpha_{k}}\right)$. Hence $r(m)$ lies in a $\underbrace{p_{1} \times \cdots \times p_{1}}_{\alpha_{1} \text { times }} \times \cdots \times \underbrace{p_{k} \times \cdots \times p_{k}}_{\alpha_{k} \text { times }}$ grid $N$. + +Surely, if $r(m)=r\left(m^{\prime}\right)$ then $p_{i}^{\alpha_{i}} \mid m_{i}-m_{i}^{\prime}$, which follows $p_{i}^{\alpha_{i}} \mid m-m^{\prime}$ for all $1 \leq i \leq k$; consequently, $n \mid m-m^{\prime}$. So, when $m$ runs over the set $\{0, \ldots, n-1\}$, the sequences $r(m)$ do not repeat; since $|N|=n$, this means that $r$ is a bijection between $\{0, \ldots, n-1\}$ and $N$. Now we will show that for each $1 \leq i \leq s$, the set $L_{i}=\left\{r(m): m \in P_{i}\right\}$ is a subgrid, and that for each axis there exists a subgrid orthogonal to this axis. Obviously, these subgrids cover $N$, and the condition (ii') follows directly from (ii). Hence the Lemma provides exactly the estimate we need. + +Consider some $1 \leq j \leq s$ and let $d_{j}=p_{1}^{\gamma_{1}} \ldots p_{k}^{\gamma_{k}}$. Consider some $q \in P_{j}$ and let $r(q)=$ $\left(r_{11}, \ldots, r_{k \alpha_{k}}\right)$. Then for an arbitrary $q^{\prime}$, setting $r\left(q^{\prime}\right)=\left(r_{11}^{\prime}, \ldots, r_{k \alpha_{k}}^{\prime}\right)$ we have + +$$ +q^{\prime} \in P_{j} \quad \Longleftrightarrow \quad p_{i}^{\gamma_{i}} \mid q-q^{\prime} \text { for each } 1 \leq i \leq k \quad \Longleftrightarrow \quad r_{i, t}=r_{i, t}^{\prime} \text { for all } t \leq \gamma_{i} +$$ + +Hence $L_{j}=\left\{\left(r_{11}^{\prime}, \ldots, r_{k \alpha_{k}}^{\prime}\right) \in N: r_{i, t}=r_{i, t}^{\prime}\right.$ for all $\left.t \leq \gamma_{i}\right\}$ which means that $L_{j}$ is a subgrid containing $r(q)$. Moreover, in $L_{j(i)}$, all the coordinates corresponding to $p_{i}$ are fixed, so it is orthogonal to all of their axes, as desired. + +Comment 1. The estimate in the problem is sharp for every $n$. One of the possible examples is the following one. For each $1 \leq i \leq k, 0 \leq j \leq \alpha_{i}-1,1 \leq k \leq p-1$, let + +$$ +P_{i, j, k}=k p_{i}^{j}+p_{i}^{j+1} \mathbb{Z} +$$ + +and add the progression $P_{0}=n \mathbb{Z}$. One can easily check that this set satisfies all the problem conditions. There also exist other examples. + +On the other hand, the estimate can be adjusted in the following sense. For every $1 \leq i \leq k$, let $0=\alpha_{i 0}, \alpha_{i 1}, \ldots, \alpha_{i h_{i}}$ be all the numbers of the form $\operatorname{ord}_{p_{i}}\left(d_{j}\right)$ in an increasing order (we delete the repeating occurences of a number, and add a number $0=\alpha_{i 0}$ if it does not occur). Then, repeating the arguments from the solution one can obtain that + +$$ +s \geq 1+\sum_{i=1}^{k} \sum_{j=1}^{h_{i}}\left(p^{\alpha_{j}-\alpha_{j-1}}-1\right) +$$ + +Note that $p^{\alpha}-1 \geq \alpha(p-1)$, and the equality is achieved only for $\alpha=1$. Hence, for reaching the minimal number of the progressions, one should have $\alpha_{i, j}=j$ for all $i, j$. In other words, for each $1 \leq j \leq \alpha_{i}$, there should be an index $t$ such that $\operatorname{ord}_{p_{i}}\left(d_{t}\right)=j$. + +Solution 2. We start with introducing some notation. For positive integer $r$, we denote $[r]=\{1,2, \ldots, r\}$. Next, we say that a set of progressions $\mathcal{P}=\left\{P_{1}, \ldots, P_{s}\right\}$ cover $\mathbb{Z}$ if each integer belongs to some of them; we say that this covering is minimal if no proper subset of $\mathcal{P}$ covers $\mathbb{Z}$. Obviously, each covering contains a minimal subcovering. + +Next, for a minimal covering $\left\{P_{1}, \ldots, P_{s}\right\}$ and for every $1 \leq i \leq s$, let $d_{i}$ be the step of progression $P_{i}$, and $h_{i}$ be some number which is contained in $P_{i}$ but in none of the other progressions. We assume that $n>1$, otherwise the problem is trivial. This implies $d_{i}>1$, otherwise the progression $P_{i}$ covers all the numbers, and $n=1$. + +We will prove a more general statement, namely the following +Claim. Assume that the progressions $P_{1}, \ldots, P_{s}$ and number $n=p_{1}^{\alpha_{1}} \ldots p_{k}^{\alpha_{k}}>1$ are chosen as in the problem statement. Moreover, choose some nonempty set of indices $I=\left\{i_{1}, \ldots, i_{t}\right\} \subseteq[k]$ and some positive integer $\beta_{i} \leq \alpha_{i}$ for every $i \in I$. Consider the set of indices + +$$ +T=\left\{j: 1 \leq j \leq s, \text { and } p_{i}^{\alpha_{i}-\beta_{i}+1} \mid d_{j} \text { for some } i \in I\right\} +$$ + +Then + +$$ +|T| \geq 1+\sum_{i \in I} \beta_{i}\left(p_{i}-1\right) +$$ + +Observe that the Claim for $I=[k]$ and $\beta_{i}=\alpha_{i}$ implies the problem statement, since the left-hand side in (2) is not greater than $s$. Hence, it suffices to prove the Claim. + +1. First, we prove the Claim assuming that all $d_{j}$ 's are prime numbers. If for some $1 \leq i \leq k$ we have at least $p_{i}$ progressions with the step $p_{i}$, then they do not intersect and hence cover all the integers; it means that there are no other progressions, and $n=p_{i}$; the Claim is trivial in this case. + +Now assume that for every $1 \leq i \leq k$, there are not more than $p_{i}-1$ progressions with step $p_{i}$; each such progression covers the numbers with a fixed residue modulo $p_{i}$, therefore there exists a residue $q_{i} \bmod p_{i}$ which is not touched by these progressions. By the Chinese Remainder Theorem, there exists a number $q$ such that $q \equiv q_{i}\left(\bmod p_{i}\right)$ for all $1 \leq i \leq k$; this number cannot be covered by any progression with step $p_{i}$, hence it is not covered at all. A contradiction. +2. Now, we assume that the general Claim is not valid, and hence we consider a counterexample $\left\{P_{1}, \ldots, P_{s}\right\}$ for the Claim; we can choose it to be minimal in the following sense: + +- the number $n$ is minimal possible among all the counterexamples; +- the sum $\sum_{i} d_{i}$ is minimal possible among all the counterexamples having the chosen value of $n$. + +As was mentioned above, not all numbers $d_{i}$ are primes; hence we can assume that $d_{1}$ is composite, say $p_{1} \mid d_{1}$ and $d_{1}^{\prime}=\frac{d_{1}}{p_{1}}>1$. Consider a progression $P_{1}^{\prime}$ having the step $d_{1}^{\prime}$, and containing $P_{1}$. We will focus on two coverings constructed as follows. +(i) Surely, the progressions $P_{1}^{\prime}, P_{2}, \ldots, P_{s}$ cover $\mathbb{Z}$, though this covering in not necessarily minimal. So, choose some minimal subcovering $\mathcal{P}^{\prime}$ in it; surely $P_{1}^{\prime} \in \mathcal{P}^{\prime}$ since $h_{1}$ is not covered by $P_{2}, \ldots, P_{s}$, so we may assume that $\mathcal{P}^{\prime}=\left\{P_{1}^{\prime}, P_{2}, \ldots, P_{s^{\prime}}\right\}$ for some $s^{\prime} \leq s$. Furthermore, the period of the covering $\mathcal{P}^{\prime}$ can appear to be less than $n$; so we denote this period by + +$$ +n^{\prime}=p_{1}^{\alpha_{1}-\sigma_{1}} \ldots p_{k}^{\alpha_{k}-\sigma_{k}}=\text { l.c.m. }\left(d_{1}^{\prime}, d_{2}, \ldots, d_{s^{\prime}}\right) +$$ + +Observe that for each $P_{j} \notin \mathcal{P}^{\prime}$, we have $h_{j} \in P_{1}^{\prime}$, otherwise $h_{j}$ would not be covered by $\mathcal{P}$. +(ii) On the other hand, each nonempty set of the form $R_{i}=P_{i} \cap P_{1}^{\prime}(1 \leq i \leq s)$ is also a progression with a step $r_{i}=$ l.c.m. $\left(d_{i}, d_{1}^{\prime}\right)$, and such sets cover $P_{1}^{\prime}$. Scaling these progressions with the ratio $1 / d_{1}^{\prime}$, we obtain the progressions $Q_{i}$ with steps $q_{i}=r_{i} / d_{1}^{\prime}$ which cover $\mathbb{Z}$. Now we choose a minimal subcovering $\mathcal{Q}$ of this covering; again we should have $Q_{1} \in \mathcal{Q}$ by the reasons of $h_{1}$. Now, denote the period of $\mathcal{Q}$ by + +$$ +n^{\prime \prime}=\text { l.c.m. }\left\{q_{i}: Q_{i} \in \mathcal{Q}\right\}=\frac{\text { l.c.m. }\left\{r_{i}: Q_{i} \in \mathcal{Q}\right\}}{d_{1}^{\prime}}=\frac{p_{1}^{\gamma_{1}} \ldots p_{k}^{\gamma_{k}}}{d_{1}^{\prime}} +$$ + +Note that if $h_{j} \in P_{1}^{\prime}$, then the image of $h_{j}$ under the scaling can be covered by $Q_{j}$ only; so, in this case we have $Q_{j} \in \mathcal{Q}$. + +Our aim is to find the desired number of progressions in coverings $\mathcal{P}$ and $\mathcal{Q}$. First, we have $n \geq n^{\prime}$, and the sum of the steps in $\mathcal{P}^{\prime}$ is less than that in $\mathcal{P}$; hence the Claim is valid for $\mathcal{P}^{\prime}$. We apply it to the set of indices $I^{\prime}=\left\{i \in I: \beta_{i}>\sigma_{i}\right\}$ and the exponents $\beta_{i}^{\prime}=\beta_{i}-\sigma_{i}$; hence the set under consideration is + +$$ +T^{\prime}=\left\{j: 1 \leq j \leq s^{\prime}, \text { and } p_{i}^{\left(\alpha_{i}-\sigma_{i}\right)-\beta_{i}^{\prime}+1}=p_{i}^{\alpha_{i}-\beta_{i}+1} \mid d_{j} \text { for some } i \in I^{\prime}\right\} \subseteq T \cap\left[s^{\prime}\right], +$$ + +and we obtain that + +$$ +\left|T \cap\left[s^{\prime}\right]\right| \geq\left|T^{\prime}\right| \geq 1+\sum_{i \in I^{\prime}}\left(\beta_{i}-\sigma_{i}\right)\left(p_{i}-1\right)=1+\sum_{i \in I}\left(\beta_{i}-\sigma_{i}\right)_{+}\left(p_{i}-1\right) +$$ + +where $(x)_{+}=\max \{x, 0\}$; the latter equality holds as for $i \notin I^{\prime}$ we have $\beta_{i} \leq \sigma_{i}$. +Observe that $x=(x-y)_{+}+\min \{x, y\}$ for all $x, y$. So, if we find at least + +$$ +G=\sum_{i \in I} \min \left\{\beta_{i}, \sigma_{i}\right\}\left(p_{i}-1\right) +$$ + +indices in $T \cap\left\{s^{\prime}+1, \ldots, s\right\}$, then we would have + +$$ +|T|=\left|T \cap\left[s^{\prime}\right]\right|+\left|T \cap\left\{s^{\prime}+1, \ldots, s\right\}\right| \geq 1+\sum_{i \in I}\left(\left(\beta_{i}-\sigma_{i}\right)_{+}+\min \left\{\beta_{i}, \sigma_{i}\right\}\right)\left(p_{i}-1\right)=1+\sum_{i \in I} \beta_{i}\left(p_{i}-1\right) +$$ + +thus leading to a contradiction with the choice of $\mathcal{P}$. We will find those indices among the indices of progressions in $\mathcal{Q}$. +3. Now denote $I^{\prime \prime}=\left\{i \in I: \sigma_{i}>0\right\}$ and consider some $i \in I^{\prime \prime}$; then $p_{i}^{\alpha_{i}} \nmid n^{\prime}$. On the other hand, there exists an index $j(i)$ such that $p_{i}^{\alpha_{i}} \mid d_{j(i)}$; this means that $d_{j(i)} \backslash n^{\prime}$ and hence $P_{j(i)}$ cannot appear in $\mathcal{P}^{\prime}$, so $j(i)>s^{\prime}$. Moreover, we have observed before that in this case $h_{j(i)} \in P_{1}^{\prime}$, hence $Q_{j(i)} \in \mathcal{Q}$. This means that $q_{j(i)} \mid n^{\prime \prime}$, therefore $\gamma_{i}=\alpha_{i}$ for each $i \in I^{\prime \prime}$ (recall here that $q_{i}=r_{i} / d_{1}^{\prime}$ and hence $\left.d_{j(i)}\left|r_{j(i)}\right| d_{1}^{\prime} n^{\prime \prime}\right)$. + +Let $d_{1}^{\prime}=p_{1}^{\tau_{1}} \ldots p_{k}^{\tau_{k}}$. Then $n^{\prime \prime}=p_{1}^{\gamma_{1}-\tau_{1}} \ldots p_{k}^{\gamma_{i}-\tau_{i}}$. Now, if $i \in I^{\prime \prime}$, then for every $\beta$ the condition $p_{i}^{\left(\gamma_{i}-\tau_{i}\right)-\beta+1} \mid q_{j}$ is equivalent to $p_{i}^{\alpha_{i}-\beta+1} \mid r_{j}$. + +Note that $n^{\prime \prime} \leq n / d_{1}^{\prime}0$. So, the set under consideration is + +$$ +\begin{aligned} +T^{\prime \prime} & =\left\{j: Q_{j} \in \mathcal{Q}, \text { and } p_{i}^{\left(\gamma_{i}-\tau_{i}\right)-\min \left\{\beta_{i}, \sigma_{i}\right\}+1} \mid q_{j} \text { for some } i \in I^{\prime \prime}\right\} \\ +& =\left\{j: Q_{j} \in \mathcal{Q}, \text { and } p_{i}^{\alpha_{i}-\min \left\{\beta_{i}, \sigma_{i}\right\}+1} \mid r_{j} \text { for some } i \in I^{\prime \prime}\right\}, +\end{aligned} +$$ + +and we obtain $\left|T^{\prime \prime}\right| \geq 1+G$. Finally, we claim that $T^{\prime \prime} \subseteq T \cap\left(\{1\} \cup\left\{s^{\prime}+1, \ldots, s\right\}\right)$; then we will obtain $\left|T \cap\left\{s^{\prime}+1, \ldots, s\right\}\right| \geq G$, which is exactly what we need. + +To prove this, consider any $j \in T^{\prime \prime}$. Observe first that $\alpha_{i}-\min \left\{\beta_{i}, \sigma_{i}\right\}+1>\alpha_{i}-\sigma_{i} \geq \tau_{i}$, hence from $p_{i}^{\alpha_{i}-\min \left\{\beta_{i}, \sigma_{i}\right\}+1} \mid r_{j}=$ l.c.m. $\left(d_{1}^{\prime}, d_{j}\right)$ we have $p_{i}^{\alpha_{i}-\min \left\{\beta_{i}, \sigma_{i}\right\}+1} \mid d_{j}$, which means that $j \in T$. Next, the exponent of $p_{i}$ in $d_{j}$ is greater than that in $n^{\prime}$, which means that $P_{j} \notin \mathcal{P}^{\prime}$. This may appear only if $j=1$ or $j>s^{\prime}$, as desired. This completes the proof. + +Comment 2. A grid analogue of the Claim is also valid. It reads as following. +Claim. Assume that the grid $N$ is covered by subgrids $L_{1}, L_{2}, \ldots, L_{s}$ so that +(ii') each subgrid contains a point which is not covered by other subgrids; +(iii) for each coordinate axis, there exists a subgrid $L_{i}$ orthogonal to this axis. + +Choose some set of indices $I=\left\{i_{1}, \ldots, i_{t}\right\} \subset[k]$, and consider the set of indices + +$$ +T=\left\{j: 1 \leq j \leq s \text {, and } L_{j} \text { is orthogonal to the } i \text { th axis for some } i \in I\right\} +$$ + +Then + +$$ +|T| \geq 1+\sum_{i \in I}\left(n_{i}-1\right) +$$ + +This Claim may be proved almost in the same way as in Solution 1. + +## Geometry + +G1. Let $A B C$ be an acute triangle with $D, E, F$ the feet of the altitudes lying on $B C, C A, A B$ respectively. One of the intersection points of the line $E F$ and the circumcircle is $P$. The lines $B P$ and $D F$ meet at point $Q$. Prove that $A P=A Q$. +(United Kingdom) +Solution 1. The line $E F$ intersects the circumcircle at two points. Depending on the choice of $P$, there are two different cases to consider. + +Case 1: The point $P$ lies on the ray $E F$ (see Fig. 1). +Let $\angle C A B=\alpha, \angle A B C=\beta$ and $\angle B C A=\gamma$. The quadrilaterals $B C E F$ and $C A F D$ are cyclic due to the right angles at $D, E$ and $F$. So, + +$$ +\begin{aligned} +& \angle B D F=180^{\circ}-\angle F D C=\angle C A F=\alpha, \\ +& \angle A F E=180^{\circ}-\angle E F B=\angle B C E=\gamma, \\ +& \angle D F B=180^{\circ}-\angle A F D=\angle D C A=\gamma . +\end{aligned} +$$ + +Since $P$ lies on the arc $A B$ of the circumcircle, $\angle P B A<\angle B C A=\gamma$. Hence, we have + +$$ +\angle P B D+\angle B D F=\angle P B A+\angle A B D+\angle B D F<\gamma+\beta+\alpha=180^{\circ}, +$$ + +and the point $Q$ must lie on the extensions of $B P$ and $D F$ beyond the points $P$ and $F$, respectively. + +From the cyclic quadrilateral $A P B C$ we get + +$$ +\angle Q P A=180^{\circ}-\angle A P B=\angle B C A=\gamma=\angle D F B=\angle Q F A . +$$ + +Hence, the quadrilateral $A Q P F$ is cyclic. Then $\angle A Q P=180^{\circ}-\angle P F A=\angle A F E=\gamma$. +We obtained that $\angle A Q P=\angle Q P A=\gamma$, so the triangle $A Q P$ is isosceles, $A P=A Q$. +![](https://cdn.mathpix.com/cropped/2024_11_18_ba86565be3364967ae6fg-45.jpg?height=680&width=763&top_left_y=2007&top_left_x=361) + +Fig. 1 +![](https://cdn.mathpix.com/cropped/2024_11_18_ba86565be3364967ae6fg-45.jpg?height=549&width=638&top_left_y=2127&top_left_x=1143) + +Fig. 2 + +Case 2: The point $P$ lies on the ray $F E$ (see Fig. 2). In this case the point $Q$ lies inside the segment $F D$. + +Similarly to the first case, we have + +$$ +\angle Q P A=\angle B C A=\gamma=\angle D F B=180^{\circ}-\angle A F Q +$$ + +Hence, the quadrilateral $A F Q P$ is cyclic. +Then $\angle A Q P=\angle A F P=\angle A F E=\gamma=\angle Q P A$. The triangle $A Q P$ is isosceles again, $\angle A Q P=\angle Q P A$ and thus $A P=A Q$. +Comment. Using signed angles, the two possible configurations can be handled simultaneously, without investigating the possible locations of $P$ and $Q$. + +Solution 2. For arbitrary points $X, Y$ on the circumcircle, denote by $\widehat{X Y}$ the central angle of the arc $X Y$. + +Let $P$ and $P^{\prime}$ be the two points where the line $E F$ meets the circumcircle; let $P$ lie on the $\operatorname{arc} A B$ and let $P^{\prime}$ lie on the $\operatorname{arc} C A$. Let $B P$ and $B P^{\prime}$ meet the line $D F$ and $Q$ and $Q^{\prime}$, respectively (see Fig. 3). We will prove that $A P=A P^{\prime}=A Q=A Q^{\prime}$. +![](https://cdn.mathpix.com/cropped/2024_11_18_ba86565be3364967ae6fg-46.jpg?height=667&width=760&top_left_y=1197&top_left_x=628) + +Fig. 3 +Like in the first solution, we have $\angle A F E=\angle B F P=\angle D F B=\angle B C A=\gamma$ from the cyclic quadrilaterals $B C E F$ and $C A F D$. + +By $\overparen{P B}+\overparen{P^{\prime} A}=2 \angle A F P^{\prime}=2 \gamma=2 \angle B C A=\overparen{A P}+\overparen{P B}$, we have + +$$ +\overline{A P}=\widetilde{P^{\prime} A}, \quad \angle P B A=\angle A B P^{\prime} \quad \text { and } \quad A P=A P^{\prime} +$$ + +Due to $\overparen{A P}=\widehat{P^{\prime}} A$, the lines $B P$ and $B Q^{\prime}$ are symmetrical about line $A B$. +Similarly, by $\angle B F P=\angle Q^{\prime} F B$, the lines $F P$ and $F Q^{\prime}$ are symmetrical about $A B$. It follows that also the points $P$ and $P^{\prime}$ are symmetrical to $Q^{\prime}$ and $Q$, respectively. Therefore, + +$$ +A P=A Q^{\prime} \quad \text { and } \quad A P^{\prime}=A Q +$$ + +The relations (1) and (2) together prove $A P=A P^{\prime}=A Q=A Q^{\prime}$. + +G2. Point $P$ lies inside triangle $A B C$. Lines $A P, B P, C P$ meet the circumcircle of $A B C$ again at points $K, L, M$, respectively. The tangent to the circumcircle at $C$ meets line $A B$ at $S$. Prove that $S C=S P$ if and only if $M K=M L$. + +Solution 1. We assume that $C A>C B$, so point $S$ lies on the ray $A B$. +From the similar triangles $\triangle P K M \sim \triangle P C A$ and $\triangle P L M \sim \triangle P C B$ we get $\frac{P M}{K M}=\frac{P A}{C A}$ and $\frac{L M}{P M}=\frac{C B}{P B}$. Multiplying these two equalities, we get + +$$ +\frac{L M}{K M}=\frac{C B}{C A} \cdot \frac{P A}{P B} +$$ + +Hence, the relation $M K=M L$ is equivalent to $\frac{C B}{C A}=\frac{P B}{P A}$. +Denote by $E$ the foot of the bisector of angle $B$ in triangle $A B C$. Recall that the locus of points $X$ for which $\frac{X A}{X B}=\frac{C A}{C B}$ is the Apollonius circle $\Omega$ with the center $Q$ on the line $A B$, and this circle passes through $C$ and $E$. Hence, we have $M K=M L$ if and only if $P$ lies on $\Omega$, that is $Q P=Q C$. +![](https://cdn.mathpix.com/cropped/2024_11_18_ba86565be3364967ae6fg-47.jpg?height=692&width=1086&top_left_y=1387&top_left_x=522) + +Fig. 1 + +Now we prove that $S=Q$, thus establishing the problem statement. We have $\angle C E S=$ $\angle C A E+\angle A C E=\angle B C S+\angle E C B=\angle E C S$, so $S C=S E$. Hence, the point $S$ lies on $A B$ as well as on the perpendicular bisector of $C E$ and therefore coincides with $Q$. + +Solution 2. As in the previous solution, we assume that $S$ lies on the ray $A B$. + +1. Let $P$ be an arbitrary point inside both the circumcircle $\omega$ of the triangle $A B C$ and the angle $A S C$, the points $K, L, M$ defined as in the problem. We claim that $S P=S C$ implies $M K=M L$. + +Let $E$ and $F$ be the points of intersection of the line $S P$ with $\omega$, point $E$ lying on the segment $S P$ (see Fig. 2). +![](https://cdn.mathpix.com/cropped/2024_11_18_ba86565be3364967ae6fg-48.jpg?height=641&width=658&top_left_y=288&top_left_x=679) + +Fig. 2 + +We have $S P^{2}=S C^{2}=S A \cdot S B$, so $\frac{S P}{S B}=\frac{S A}{S P}$, and hence $\triangle P S A \sim \triangle B S P$. Then $\angle B P S=\angle S A P$. Since $2 \angle B P S=\overparen{B E}+\overparen{L F}$ and $2 \angle S A P=\overparen{B E}+\overparen{E K}$ we have + +$$ +\overparen{L F}=\overparen{E K} +$$ + +On the other hand, from $\angle S P C=\angle S C P$ we have $\widehat{E C}+\widehat{M F}=\widehat{E C}+\widehat{E M}$, or + +$$ +\widetilde{M F}=\overparen{E M} . +$$ + +From (1) and (2) we get $\widehat{M F L}=\widehat{M F}+\widehat{F L}=\widehat{M E}+\widehat{E K}=\widehat{M E K}$ and hence $M K=M L$. The claim is proved. +2. We are left to prove the converse. So, assume that $M K=M L$, and introduce the points $E$ and $F$ as above. We have $S C^{2}=S E \cdot S F$; hence, there exists a point $P^{\prime}$ lying on the segment $E F$ such that $S P^{\prime}=S C$ (see Fig. 3). +![](https://cdn.mathpix.com/cropped/2024_11_18_ba86565be3364967ae6fg-48.jpg?height=743&width=1154&top_left_y=1933&top_left_x=431) + +Fig. 3 + +Assume that $P \neq P^{\prime}$. Let the lines $A P^{\prime}, B P^{\prime}, C P^{\prime}$ meet $\omega$ again at points $K^{\prime}, L^{\prime}, M^{\prime}$ respectively. Now, if $P^{\prime}$ lies on the segment $P F$ then by the first part of the solution we have $\widehat{M^{\prime} F L^{\prime}}=\widehat{M^{\prime} E K^{\prime}}$. On the other hand, we have $\widehat{M F L}>\widehat{M^{\prime} F L^{\prime}}=\widehat{M^{\prime} E K^{\prime}}>\widehat{M E K}$, therefore $\widehat{M F L}>\widehat{M E K}$ which contradicts $M K=M L$. + +Similarly, if point $P^{\prime}$ lies on the segment $E P$ then we get $\widehat{M F L}<\widehat{M E K}$ which is impossible. Therefore, the points $P$ and $P^{\prime}$ coincide and hence $S P=S P^{\prime}=S C$. + +Solution 3. We present a different proof of the converse direction, that is, $M K=M L \Rightarrow$ $S P=S C$. As in the previous solutions we assume that $C A>C B$, and the line $S P$ meets $\omega$ at $E$ and $F$. + +From $M L=M K$ we get $\widehat{M E K}=\widehat{M F L}$. Now we claim that $\widehat{M E}=\widehat{M F}$ and $\widehat{E K}=\widehat{F L}$. +To the contrary, suppose first that $\widehat{M E}>\widehat{M F}$; then $\widehat{E K}=\widehat{M E K}-\widehat{M E}<\widehat{M F L}-\widehat{M F}=$ $\overparen{F L}$. Now, the inequality $\overparen{M E}>\overparen{M F}$ implies $2 \angle S C M=\overparen{E C}+\overparen{M E}>\overparen{E C}+\overparen{M F}=2 \angle S P C$ and hence $S P>S C$. On the other hand, the inequality $\overparen{E K}<\overparen{F L}$ implies $2 \angle S P K=$ $\overparen{E K}+\widetilde{A F}<\overparen{F L}+\widetilde{A F}=2 \angle A B L$, hence + +$$ +\angle S P A=180^{\circ}-\angle S P K>180^{\circ}-\angle A B L=\angle S B P . +$$ + +![](https://cdn.mathpix.com/cropped/2024_11_18_ba86565be3364967ae6fg-49.jpg?height=712&width=1098&top_left_y=1226&top_left_x=519) + +Fig. 4 +Consider the point $A^{\prime}$ on the ray $S A$ for which $\angle S P A^{\prime}=\angle S B P$; in our case, this point lies on the segment $S A$ (see Fig. 4). Then $\triangle S B P \sim \triangle S P A^{\prime}$ and $S P^{2}=S B \cdot S A^{\prime}S C$. + +Similarly, one can prove that the inequality $\widehat{M E}<\widehat{M F}$ is also impossible. So, we get $\widehat{M E}=\widehat{M F}$ and therefore $2 \angle S C M=\widehat{E C}+\widehat{M E}=\widehat{E C}+\widehat{M F}=2 \angle S P C$, which implies $S C=S P$. + +G3. Let $A_{1} A_{2} \ldots A_{n}$ be a convex polygon. Point $P$ inside this polygon is chosen so that its projections $P_{1}, \ldots, P_{n}$ onto lines $A_{1} A_{2}, \ldots, A_{n} A_{1}$ respectively lie on the sides of the polygon. Prove that for arbitrary points $X_{1}, \ldots, X_{n}$ on sides $A_{1} A_{2}, \ldots, A_{n} A_{1}$ respectively, + +$$ +\max \left\{\frac{X_{1} X_{2}}{P_{1} P_{2}}, \ldots, \frac{X_{n} X_{1}}{P_{n} P_{1}}\right\} \geq 1 +$$ + +(Armenia) + +Solution 1. Denote $P_{n+1}=P_{1}, X_{n+1}=X_{1}, A_{n+1}=A_{1}$. +Lemma. Let point $Q$ lies inside $A_{1} A_{2} \ldots A_{n}$. Then it is contained in at least one of the circumcircles of triangles $X_{1} A_{2} X_{2}, \ldots, X_{n} A_{1} X_{1}$. +Proof. If $Q$ lies in one of the triangles $X_{1} A_{2} X_{2}, \ldots, X_{n} A_{1} X_{1}$, the claim is obvious. Otherwise $Q$ lies inside the polygon $X_{1} X_{2} \ldots X_{n}$ (see Fig. 1). Then we have + +$$ +\begin{aligned} +& \left(\angle X_{1} A_{2} X_{2}+\angle X_{1} Q X_{2}\right)+\cdots+\left(\angle X_{n} A_{1} X_{1}+\angle X_{n} Q X_{1}\right) \\ +& \quad=\left(\angle X_{1} A_{1} X_{2}+\cdots+\angle X_{n} A_{1} X_{1}\right)+\left(\angle X_{1} Q X_{2}+\cdots+\angle X_{n} Q X_{1}\right)=(n-2) \pi+2 \pi=n \pi +\end{aligned} +$$ + +hence there exists an index $i$ such that $\angle X_{i} A_{i+1} X_{i+1}+\angle X_{i} Q X_{i+1} \geq \frac{\pi n}{n}=\pi$. Since the quadrilateral $Q X_{i} A_{i+1} X_{i+1}$ is convex, this means exactly that $Q$ is contained the circumcircle of $\triangle X_{i} A_{i+1} X_{i+1}$, as desired. + +Now we turn to the solution. Applying lemma, we get that $P$ lies inside the circumcircle of triangle $X_{i} A_{i+1} X_{i+1}$ for some $i$. Consider the circumcircles $\omega$ and $\Omega$ of triangles $P_{i} A_{i+1} P_{i+1}$ and $X_{i} A_{i+1} X_{i+1}$ respectively (see Fig. 2); let $r$ and $R$ be their radii. Then we get $2 r=A_{i+1} P \leq 2 R$ (since $P$ lies inside $\Omega$ ), hence + +$$ +P_{i} P_{i+1}=2 r \sin \angle P_{i} A_{i+1} P_{i+1} \leq 2 R \sin \angle X_{i} A_{i+1} X_{i+1}=X_{i} X_{i+1} +$$ + +QED. +![](https://cdn.mathpix.com/cropped/2024_11_18_ba86565be3364967ae6fg-51.jpg?height=547&width=566&top_left_y=2011&top_left_x=508) + +Fig. 1 +![](https://cdn.mathpix.com/cropped/2024_11_18_ba86565be3364967ae6fg-51.jpg?height=458&width=527&top_left_y=2098&top_left_x=1093) + +Fig. 2 + +Solution 2. As in Solution 1, we assume that all indices of points are considered modulo $n$. +We will prove a bit stronger inequality, namely + +$$ +\max \left\{\frac{X_{1} X_{2}}{P_{1} P_{2}} \cos \alpha_{1}, \ldots, \frac{X_{n} X_{1}}{P_{n} P_{1}} \cos \alpha_{n}\right\} \geq 1 +$$ + +where $\alpha_{i}(1 \leq i \leq n)$ is the angle between lines $X_{i} X_{i+1}$ and $P_{i} P_{i+1}$. We denote $\beta_{i}=\angle A_{i} P_{i} P_{i-1}$ and $\gamma_{i}=\angle A_{i+1} P_{i} P_{i+1}$ for all $1 \leq i \leq n$. + +Suppose that for some $1 \leq i \leq n$, point $X_{i}$ lies on the segment $A_{i} P_{i}$, while point $X_{i+1}$ lies on the segment $P_{i+1} A_{i+2}$. Then the projection of the segment $X_{i} X_{i+1}$ onto the line $P_{i} P_{i+1}$ contains segment $P_{i} P_{i+1}$, since $\gamma_{i}$ and $\beta_{i+1}$ are acute angles (see Fig. 3). Therefore, $X_{i} X_{i+1} \cos \alpha_{i} \geq$ $P_{i} P_{i+1}$, and in this case the statement is proved. + +So, the only case left is when point $X_{i}$ lies on segment $P_{i} A_{i+1}$ for all $1 \leq i \leq n$ (the case when each $X_{i}$ lies on segment $A_{i} P_{i}$ is completely analogous). + +Now, assume to the contrary that the inequality + +$$ +X_{i} X_{i+1} \cos \alpha_{i}P_{i+1} Y_{i+1}^{\prime}$ (again since $\gamma_{i}$ and $\beta_{i+1}$ are acute; see Fig. 4). Hence, we have + +$$ +X_{i} P_{i} \cos \gamma_{i}>X_{i+1} P_{i+1} \cos \beta_{i+1}, \quad 1 \leq i \leq n +$$ + +Multiplying these inequalities, we get + +$$ +\cos \gamma_{1} \cos \gamma_{2} \cdots \cos \gamma_{n}>\cos \beta_{1} \cos \beta_{2} \cdots \cos \beta_{n} +$$ + +On the other hand, the sines theorem applied to triangle $P P_{i} P_{i+1}$ provides + +$$ +\frac{P P_{i}}{P P_{i+1}}=\frac{\sin \left(\frac{\pi}{2}-\beta_{i+1}\right)}{\sin \left(\frac{\pi}{2}-\gamma_{i}\right)}=\frac{\cos \beta_{i+1}}{\cos \gamma_{i}} +$$ + +Multiplying these equalities we get + +$$ +1=\frac{\cos \beta_{2}}{\cos \gamma_{1}} \cdot \frac{\cos \beta_{3}}{\cos \gamma_{2}} \cdots \frac{\cos \beta_{1}}{\cos \gamma_{n}} +$$ + +which contradicts (2). +![](https://cdn.mathpix.com/cropped/2024_11_18_ba86565be3364967ae6fg-52.jpg?height=817&width=1297&top_left_y=1873&top_left_x=351) + +Fig. 3 +Fig. 4 + +G4. Let $I$ be the incenter of a triangle $A B C$ and $\Gamma$ be its circumcircle. Let the line $A I$ intersect $\Gamma$ at a point $D \neq A$. Let $F$ and $E$ be points on side $B C$ and $\operatorname{arc} B D C$ respectively such that $\angle B A F=\angle C A E<\frac{1}{2} \angle B A C$. Finally, let $G$ be the midpoint of the segment $I F$. Prove that the lines $D G$ and $E I$ intersect on $\Gamma$. +(Hong Kong) +Solution 1. Let $X$ be the second point of intersection of line $E I$ with $\Gamma$, and $L$ be the foot of the bisector of angle $B A C$. Let $G^{\prime}$ and $T$ be the points of intersection of segment $D X$ with lines $I F$ and $A F$, respectively. We are to prove that $G=G^{\prime}$, or $I G^{\prime}=G^{\prime} F$. By the Menelaus theorem applied to triangle $A I F$ and line $D X$, it means that we need the relation + +$$ +1=\frac{G^{\prime} F}{I G^{\prime}}=\frac{T F}{A T} \cdot \frac{A D}{I D}, \quad \text { or } \quad \frac{T F}{A T}=\frac{I D}{A D} +$$ + +Let the line $A F$ intersect $\Gamma$ at point $K \neq A$ (see Fig. 1); since $\angle B A K=\angle C A E$ we have $\widehat{B K}=\widehat{C E}$, hence $K E \| B C$. Notice that $\angle I A T=\angle D A K=\angle E A D=\angle E X D=\angle I X T$, so the points $I, A, X, T$ are concyclic. Hence we have $\angle I T A=\angle I X A=\angle E X A=\angle E K A$, so $I T\|K E\| B C$. Therefore we obtain $\frac{T F}{A T}=\frac{I L}{A I}$. + +Since $C I$ is the bisector of $\angle A C L$, we get $\frac{I L}{A I}=\frac{C L}{A C}$. Furthermore, $\angle D C L=\angle D C B=$ $\angle D A B=\angle C A D=\frac{1}{2} \angle B A C$, hence the triangles $D C L$ and $D A C$ are similar; therefore we get $\frac{C L}{A C}=\frac{D C}{A D}$. Finally, it is known that the midpoint $D$ of $\operatorname{arc} B C$ is equidistant from points $I$, $B, C$, hence $\frac{D C}{A D}=\frac{I D}{A D}$. + +Summarizing all these equalities, we get + +$$ +\frac{T F}{A T}=\frac{I L}{A I}=\frac{C L}{A C}=\frac{D C}{A D}=\frac{I D}{A D} +$$ + +as desired. +![](https://cdn.mathpix.com/cropped/2024_11_18_ba86565be3364967ae6fg-53.jpg?height=767&width=715&top_left_y=1838&top_left_x=336) + +Fig. 1 +![](https://cdn.mathpix.com/cropped/2024_11_18_ba86565be3364967ae6fg-53.jpg?height=775&width=530&top_left_y=1828&top_left_x=1254) + +Fig. 2 + +Comment. The equality $\frac{A I}{I L}=\frac{A D}{D I}$ is known and can be obtained in many different ways. For instance, one can consider the inversion with center $D$ and radius $D C=D I$. This inversion takes $\widehat{B A C}$ to the segment $B C$, so point $A$ goes to $L$. Hence $\frac{I L}{D I}=\frac{A I}{A D}$, which is the desired equality. + +Solution 2. As in the previous solution, we introduce the points $X, T$ and $K$ and note that it suffice to prove the equality + +$$ +\frac{T F}{A T}=\frac{D I}{A D} \quad \Longleftrightarrow \quad \frac{T F+A T}{A T}=\frac{D I+A D}{A D} \quad \Longleftrightarrow \quad \frac{A T}{A D}=\frac{A F}{D I+A D} +$$ + +Since $\angle F A D=\angle E A I$ and $\angle T D A=\angle X D A=\angle X E A=\angle I E A$, we get that the triangles $A T D$ and $A I E$ are similar, therefore $\frac{A T}{A D}=\frac{A I}{A E}$. + +Next, we also use the relation $D B=D C=D I$. Let $J$ be the point on the extension of segment $A D$ over point $D$ such that $D J=D I=D C$ (see Fig. 2). Then $\angle D J C=$ $\angle J C D=\frac{1}{2}(\pi-\angle J D C)=\frac{1}{2} \angle A D C=\frac{1}{2} \angle A B C=\angle A B I$. Moreover, $\angle B A I=\angle J A C$, hence triangles $A B I$ and $A J C$ are similar, so $\frac{A B}{A J}=\frac{A I}{A C}$, or $A B \cdot A C=A J \cdot A I=(D I+A D) \cdot A I$. + +On the other hand, we get $\angle A B F=\angle A B C=\angle A E C$ and $\angle B A F=\angle C A E$, so triangles $A B F$ and $A E C$ are also similar, which implies $\frac{A F}{A C}=\frac{A B}{A E}$, or $A B \cdot A C=A F \cdot A E$. + +Summarizing we get + +$$ +(D I+A D) \cdot A I=A B \cdot A C=A F \cdot A E \quad \Rightarrow \quad \frac{A I}{A E}=\frac{A F}{A D+D I} \quad \Rightarrow \quad \frac{A T}{A D}=\frac{A F}{A D+D I} +$$ + +as desired. +Comment. In fact, point $J$ is an excenter of triangle $A B C$. + +G5. Let $A B C D E$ be a convex pentagon such that $B C \| A E, A B=B C+A E$, and $\angle A B C=$ $\angle C D E$. Let $M$ be the midpoint of $C E$, and let $O$ be the circumcenter of triangle $B C D$. Given that $\angle D M O=90^{\circ}$, prove that $2 \angle B D A=\angle C D E$. +(Ukraine) +Solution 1. Choose point $T$ on ray $A E$ such that $A T=A B$; then from $A E \| B C$ we have $\angle C B T=\angle A T B=\angle A B T$, so $B T$ is the bisector of $\angle A B C$. On the other hand, we have $E T=A T-A E=A B-A E=B C$, hence quadrilateral $B C T E$ is a parallelogram, and the midpoint $M$ of its diagonal $C E$ is also the midpoint of the other diagonal $B T$. + +Next, let point $K$ be symmetrical to $D$ with respect to $M$. Then $O M$ is the perpendicular bisector of segment $D K$, and hence $O D=O K$, which means that point $K$ lies on the circumcircle of triangle $B C D$. Hence we have $\angle B D C=\angle B K C$. On the other hand, the angles $B K C$ and $T D E$ are symmetrical with respect to $M$, so $\angle T D E=\angle B K C=\angle B D C$. + +Therefore, $\angle B D T=\angle B D E+\angle E D T=\angle B D E+\angle B D C=\angle C D E=\angle A B C=180^{\circ}-$ $\angle B A T$. This means that the points $A, B, D, T$ are concyclic, and hence $\angle A D B=\angle A T B=$ $\frac{1}{2} \angle A B C=\frac{1}{2} \angle C D E$, as desired. +![](https://cdn.mathpix.com/cropped/2024_11_18_ba86565be3364967ae6fg-55.jpg?height=621&width=1276&top_left_y=1140&top_left_x=425) + +Solution 2. Let $\angle C B D=\alpha, \angle B D C=\beta, \angle A D E=\gamma$, and $\angle A B C=\angle C D E=2 \varphi$. Then we have $\angle A D B=2 \varphi-\beta-\gamma, \angle B C D=180^{\circ}-\alpha-\beta, \angle A E D=360^{\circ}-\angle B C D-\angle C D E=$ $180^{\circ}-2 \varphi+\alpha+\beta$, and finally $\angle D A E=180^{\circ}-\angle A D E-\angle A E D=2 \varphi-\alpha-\beta-\gamma$. +![](https://cdn.mathpix.com/cropped/2024_11_18_ba86565be3364967ae6fg-55.jpg?height=530&width=1039&top_left_y=2008&top_left_x=546) + +Let $N$ be the midpoint of $C D$; then $\angle D N O=90^{\circ}=\angle D M O$, hence points $M, N$ lie on the circle with diameter $O D$. Now, if points $O$ and $M$ lie on the same side of $C D$, we have $\angle D M N=\angle D O N=\frac{1}{2} \angle D O C=\alpha$; in the other case, we have $\angle D M N=180^{\circ}-\angle D O N=\alpha ;$ +so, in both cases $\angle D M N=\alpha$ (see Figures). Next, since $M N$ is a midline in triangle $C D E$, we have $\angle M D E=\angle D M N=\alpha$ and $\angle N D M=2 \varphi-\alpha$. + +Now we apply the sine rule to the triangles $A B D, A D E$ (twice), $B C D$ and $M N D$ obtaining + +$$ +\begin{gathered} +\frac{A B}{A D}=\frac{\sin (2 \varphi-\beta-\gamma)}{\sin (2 \varphi-\alpha)}, \quad \frac{A E}{A D}=\frac{\sin \gamma}{\sin (2 \varphi-\alpha-\beta)}, \quad \frac{D E}{A D}=\frac{\sin (2 \varphi-\alpha-\beta-\gamma)}{\sin (2 \varphi-\alpha-\beta)} \\ +\frac{B C}{C D}=\frac{\sin \beta}{\sin \alpha}, \quad \frac{C D}{D E}=\frac{C D / 2}{D E / 2}=\frac{N D}{N M}=\frac{\sin \alpha}{\sin (2 \varphi-\alpha)} +\end{gathered} +$$ + +which implies + +$$ +\frac{B C}{A D}=\frac{B C}{C D} \cdot \frac{C D}{D E} \cdot \frac{D E}{A D}=\frac{\sin \beta \cdot \sin (2 \varphi-\alpha-\beta-\gamma)}{\sin (2 \varphi-\alpha) \cdot \sin (2 \varphi-\alpha-\beta)} +$$ + +Hence, the condition $A B=A E+B C$, or equivalently $\frac{A B}{A D}=\frac{A E+B C}{A D}$, after multiplying by the common denominator rewrites as + +$$ +\begin{gathered} +\sin (2 \varphi-\alpha-\beta) \cdot \sin (2 \varphi-\beta-\gamma)=\sin \gamma \cdot \sin (2 \varphi-\alpha)+\sin \beta \cdot \sin (2 \varphi-\alpha-\beta-\gamma) \\ +\Longleftrightarrow \cos (\gamma-\alpha)-\cos (4 \varphi-2 \beta-\alpha-\gamma)=\cos (2 \varphi-\alpha-2 \beta-\gamma)-\cos (2 \varphi+\gamma-\alpha) \\ +\Longleftrightarrow \cos (\gamma-\alpha)+\cos (2 \varphi+\gamma-\alpha)=\cos (2 \varphi-\alpha-2 \beta-\gamma)+\cos (4 \varphi-2 \beta-\alpha-\gamma) \\ +\Longleftrightarrow \cos \varphi \cdot \cos (\varphi+\gamma-\alpha)=\cos \varphi \cdot \cos (3 \varphi-2 \beta-\alpha-\gamma) \\ +\Longleftrightarrow \cos \varphi \cdot(\cos (\varphi+\gamma-\alpha)-\cos (3 \varphi-2 \beta-\alpha-\gamma))=0 \\ +\Longleftrightarrow \cos \varphi \cdot \sin (2 \varphi-\beta-\alpha) \cdot \sin (\varphi-\beta-\gamma)=0 +\end{gathered} +$$ + +Since $2 \varphi-\beta-\alpha=180^{\circ}-\angle A E D<180^{\circ}$ and $\varphi=\frac{1}{2} \angle A B C<90^{\circ}$, it follows that $\varphi=\beta+\gamma$, hence $\angle B D A=2 \varphi-\beta-\gamma=\varphi=\frac{1}{2} \angle C D E$, as desired. + +G6. The vertices $X, Y, Z$ of an equilateral triangle $X Y Z$ lie respectively on the sides $B C$, $C A, A B$ of an acute-angled triangle $A B C$. Prove that the incenter of triangle $A B C$ lies inside triangle $X Y Z$. + +G6 ${ }^{\prime}$. The vertices $X, Y, Z$ of an equilateral triangle $X Y Z$ lie respectively on the sides $B C, C A, A B$ of a triangle $A B C$. Prove that if the incenter of triangle $A B C$ lies outside triangle $X Y Z$, then one of the angles of triangle $A B C$ is greater than $120^{\circ}$. +(Bulgaria) +Solution 1 for G6. We will prove a stronger fact; namely, we will show that the incenter $I$ of triangle $A B C$ lies inside the incircle of triangle $X Y Z$ (and hence surely inside triangle $X Y Z$ itself). We denote by $d(U, V W)$ the distance between point $U$ and line $V W$. + +Denote by $O$ the incenter of $\triangle X Y Z$ and by $r, r^{\prime}$ and $R^{\prime}$ the inradii of triangles $A B C, X Y Z$ and the circumradius of $X Y Z$, respectively. Then we have $R^{\prime}=2 r^{\prime}$, and the desired inequality is $O I \leq r^{\prime}$. We assume that $O \neq I$; otherwise the claim is trivial. + +Let the incircle of $\triangle A B C$ touch its sides $B C, A C, A B$ at points $A_{1}, B_{1}, C_{1}$ respectively. The lines $I A_{1}, I B_{1}, I C_{1}$ cut the plane into 6 acute angles, each one containing one of the points $A_{1}, B_{1}, C_{1}$ on its border. We may assume that $O$ lies in an angle defined by lines $I A_{1}$, $I C_{1}$ and containing point $C_{1}$ (see Fig. 1). Let $A^{\prime}$ and $C^{\prime}$ be the projections of $O$ onto lines $I A_{1}$ and $I C_{1}$, respectively. + +Since $O X=R^{\prime}$, we have $d(O, B C) \leq R^{\prime}$. Since $O A^{\prime} \| B C$, it follows that $d\left(A^{\prime}, B C\right)=$ $A^{\prime} I+r \leq R^{\prime}$, or $A^{\prime} I \leq R^{\prime}-r$. On the other hand, the incircle of $\triangle X Y Z$ lies inside $\triangle A B C$, hence $d(O, A B) \geq r^{\prime}$, and analogously we get $d(O, A B)=C^{\prime} C_{1}=r-I C^{\prime} \geq r^{\prime}$, or $I C^{\prime} \leq r-r^{\prime}$. +![](https://cdn.mathpix.com/cropped/2024_11_18_ba86565be3364967ae6fg-57.jpg?height=475&width=815&top_left_y=1536&top_left_x=455) + +Fig. 1 +![](https://cdn.mathpix.com/cropped/2024_11_18_ba86565be3364967ae6fg-57.jpg?height=346&width=389&top_left_y=1666&top_left_x=1282) + +Fig. 2 + +Finally, the quadrilateral $I A^{\prime} O C^{\prime}$ is circumscribed due to the right angles at $A^{\prime}$ and $C^{\prime}$ (see Fig. 2). On its circumcircle, we have $\widehat{A^{\prime} O C^{\prime}}=2 \angle A^{\prime} I C^{\prime}<180^{\circ}=\widetilde{O C^{\prime} I}$, hence $180^{\circ} \geq$ $\widetilde{I C^{\prime}}>\widetilde{A^{\prime} O}$. This means that $I C^{\prime}>A^{\prime} O$. Finally, we have $O I \leq I A^{\prime}+A^{\prime} O90^{\circ}$ thus leading to a contradiction. + +Note that $\omega$ intersects each of the segments $X Y$ and $Y Z$ at two points; let $U, U^{\prime}$ and $V$, $V^{\prime}$ be the points of intersection of $\omega$ with $X Y$ and $Y Z$, respectively $\left(U Y>U^{\prime} Y, V Y>V^{\prime} Y\right.$; see Figs. 3 and 4). Note that $60^{\circ}=\angle X Y Z=\frac{1}{2}\left(\overparen{U V}-\widetilde{U^{\prime} V^{\prime}}\right) \leq \frac{1}{2} \overparen{U V}$, hence $\overparen{U V} \geq 120^{\circ}$. + +On the other hand, since $I$ lies in $\triangle A Y Z$, we get $\sqrt{U V^{\prime}}<180^{\circ}$, hence $\sqrt{U A_{1} U^{\prime}} \leq \sqrt{U A_{1} V^{\prime}}<$ $180^{\circ}-\widehat{U V} \leq 60^{\circ}$. + +Now, two cases are possible due to the order of points $Y, B_{1}$ on segment $A C$. +![](https://cdn.mathpix.com/cropped/2024_11_18_ba86565be3364967ae6fg-58.jpg?height=495&width=690&top_left_y=529&top_left_x=249) + +Fig. 3 +![](https://cdn.mathpix.com/cropped/2024_11_18_ba86565be3364967ae6fg-58.jpg?height=415&width=801&top_left_y=609&top_left_x=953) + +Fig. 4 + +Case 1. Let point $Y$ lie on the segment $A B_{1}$ (see Fig. 3). Then we have $\angle Y X C=$ $\frac{1}{2}\left(\widehat{A_{1} U^{\prime}}-\widehat{A_{1} U}\right) \leq \frac{1}{2} \widehat{U A_{1} U^{\prime}}<30^{\circ}$; analogously, we get $\angle X Y C \leq \frac{1}{2} \widehat{U A_{1} U^{\prime}}<30^{\circ}$. Therefore, $\angle Y C X=180^{\circ}-\angle Y X C-\angle X Y C>120^{\circ}$, as desired. + +Case 2. Now let point $Y$ lie on the segment $C B_{1}$ (see Fig. 4). Analogously, we obtain $\angle Y X C<30^{\circ}$. Next, $\angle I Y X>\angle Z Y X=60^{\circ}$, but $\angle I Y X<\angle I Y B_{1}$, since $Y B_{1}$ is a tangent and $Y X$ is a secant line to circle $\omega$ from point $Y$. Hence, we get $120^{\circ}<\angle I Y B_{1}+\angle I Y X=$ $\angle B_{1} Y X=\angle Y X C+\angle Y C X<30^{\circ}+\angle Y C X$, hence $\angle Y C X>120^{\circ}-30^{\circ}=90^{\circ}$, as desired. + +Comment. In the same way, one can prove a more general +Claim. Let the vertices $X, Y, Z$ of a triangle $X Y Z$ lie respectively on the sides $B C, C A, A B$ of a triangle $A B C$. Suppose that the incenter of triangle $A B C$ lies outside triangle $X Y Z$, and $\alpha$ is the least angle of $\triangle X Y Z$. Then one of the angles of triangle $A B C$ is greater than $3 \alpha-90^{\circ}$. + +Solution for G6'. Assume the contrary. As in Solution 2, we assume that the incenter $I$ of $\triangle A B C$ lies in $\triangle A Y Z$, and the tangency point $A_{1}$ of $\omega$ and $B C$ lies on segment $C X$. Surely, $\angle Y Z A \leq 180^{\circ}-\angle Y Z X=120^{\circ}$, hence points $I$ and $Y$ lie on one side of the perpendicular bisector to $X Y$; therefore $I X>I Y$. Moreover, $\omega$ intersects segment $X Y$ at two points, and therefore the projection $M$ of $I$ onto $X Y$ lies on the segment $X Y$. In this case, we will prove that $\angle C>120^{\circ}$. + +Let $Y K, Y L$ be two tangents from point $Y$ to $\omega$ (points $K$ and $A_{1}$ lie on one side of $X Y$; if $Y$ lies on $\omega$, we say $K=L=Y$ ); one of the points $K$ and $L$ is in fact a tangency point $B_{1}$ of $\omega$ and $A C$. From symmetry, we have $\angle Y I K=\angle Y I L$. On the other hand, since $I X>I Y$, we get $X M120^{\circ}$, as desired. +![](https://cdn.mathpix.com/cropped/2024_11_18_ba86565be3364967ae6fg-59.jpg?height=366&width=715&top_left_y=408&top_left_x=271) + +Fig. 5 +![](https://cdn.mathpix.com/cropped/2024_11_18_ba86565be3364967ae6fg-59.jpg?height=515&width=869&top_left_y=276&top_left_x=982) + +Fig. 6 + +Comment 1. The estimate claimed in $\mathrm{G}^{\prime}$ is sharp. Actually, if $\angle B A C>120^{\circ}$, one can consider an equilateral triangle $X Y Z$ with $Z=A, Y \in A C, X \in B C$ (such triangle exists since $\angle A C B<60^{\circ}$ ). It intersects with the angle bisector of $\angle B A C$ only at point $A$, hence it does not contain $I$. + +Comment 2. As in the previous solution, there is a generalization for an arbitrary triangle $X Y Z$, but here we need some additional condition. The statement reads as follows. +Claim. Let the vertices $X, Y, Z$ of a triangle $X Y Z$ lie respectively on the sides $B C, C A, A B$ of a triangle $A B C$. Suppose that the incenter of triangle $A B C$ lies outside triangle $X Y Z, \alpha$ is the least angle of $\triangle X Y Z$, and all sides of triangle $X Y Z$ are greater than $2 r \cot \alpha$, where $r$ is the inradius of $\triangle A B C$. Then one of the angles of triangle $A B C$ is greater than $2 \alpha$. + +The additional condition is needed to verify that $X M>Y M$ since it cannot be shown in the original way. Actually, we have $\angle M Y I>\alpha, I M2 r \cot \alpha$, then surely $X M>Y M$. + +On the other hand, this additional condition follows easily from the conditions of the original problem. Actually, if $I \in \triangle A Y Z$, then the diameter of $\omega$ parallel to $Y Z$ is contained in $\triangle A Y Z$ and is thus shorter than $Y Z$. Hence $Y Z>2 r>2 r \cot 60^{\circ}$. + +G7. Three circular arcs $\gamma_{1}, \gamma_{2}$, and $\gamma_{3}$ connect the points $A$ and $C$. These arcs lie in the same half-plane defined by line $A C$ in such a way that arc $\gamma_{2}$ lies between the arcs $\gamma_{1}$ and $\gamma_{3}$. Point $B$ lies on the segment $A C$. Let $h_{1}, h_{2}$, and $h_{3}$ be three rays starting at $B$, lying in the same half-plane, $h_{2}$ being between $h_{1}$ and $h_{3}$. For $i, j=1,2,3$, denote by $V_{i j}$ the point of intersection of $h_{i}$ and $\gamma_{j}$ (see the Figure below). + +Denote by $\widehat{V_{i j} V_{k j}} \widehat{V_{k \ell} V_{i \ell}}$ the curved quadrilateral, whose sides are the segments $V_{i j} V_{i \ell}, V_{k j} V_{k \ell}$ and $\operatorname{arcs} V_{i j} V_{k j}$ and $V_{i \ell} V_{k \ell}$. We say that this quadrilateral is circumscribed if there exists a circle touching these two segments and two arcs. + +Prove that if the curved quadrilaterals $\sqrt{V_{11} V_{21}} \sqrt{22} V_{12}, \sqrt{V_{12} V_{22}} \sqrt{23} V_{13}, \sqrt{V_{21} V_{31}} \sqrt{V_{32} V_{22}}$ are circumscribed, then the curved quadrilateral $\sqrt{V_{22} V_{32}} \sqrt{33} V_{23}$ is circumscribed, too. +![](https://cdn.mathpix.com/cropped/2024_11_18_ba86565be3364967ae6fg-61.jpg?height=495&width=815&top_left_y=798&top_left_x=655) + +Fig. 1 +(Hungary) +Solution. Denote by $O_{i}$ and $R_{i}$ the center and the radius of $\gamma_{i}$, respectively. Denote also by $H$ the half-plane defined by $A C$ which contains the whole configuration. For every point $P$ in the half-plane $H$, denote by $d(P)$ the distance between $P$ and line $A C$. Furthermore, for any $r>0$, denote by $\Omega(P, r)$ the circle with center $P$ and radius $r$. +Lemma 1. For every $1 \leq iR_{i}$ and $O_{j} P0$; then the +![](https://cdn.mathpix.com/cropped/2024_11_18_ba86565be3364967ae6fg-62.jpg?height=787&width=1338&top_left_y=269&top_left_x=336) +circle $\Omega(P, r)$ touches $\gamma_{i}$ externally and touches $\gamma_{j}$ internally, so $P$ belongs to the locus under investigation. +(b) Let $\vec{\rho}=\overrightarrow{A P}, \vec{\rho}_{i}=\overrightarrow{A O_{i}}$, and $\vec{\rho}_{j}=\overrightarrow{A O_{j}}$; let $d_{i j}=O_{i} O_{j}$, and let $\vec{v}$ be a unit vector orthogonal to $A C$ and directed toward $H$. Then we have $\left|\vec{\rho}_{i}\right|=R_{i},\left|\vec{\rho}_{j}\right|=R_{j},\left|\overrightarrow{O_{i} P}\right|=$ $\left|\vec{\rho}-\vec{\rho}_{i}\right|=R_{i}+r,\left|\overrightarrow{O_{j} P}\right|=\left|\vec{\rho}-\vec{\rho}_{j}\right|=R_{j}-r$, hence + +$$ +\begin{gathered} +\left(\vec{\rho}-\vec{\rho}_{i}\right)^{2}-\left(\vec{\rho}-\vec{\rho}_{j}\right)^{2}=\left(R_{i}+r\right)^{2}-\left(R_{j}-r\right)^{2}, \\ +\left(\vec{\rho}_{i}^{2}-\vec{\rho}_{j}^{2}\right)+2 \vec{\rho} \cdot\left(\vec{\rho}_{j}-\vec{\rho}_{i}\right)=\left(R_{i}^{2}-R_{j}^{2}\right)+2 r\left(R_{i}+R_{j}\right), \\ +d_{i j} \cdot d(P)=d_{i j} \vec{v} \cdot \vec{\rho}=\left(\vec{\rho}_{j}-\vec{\rho}_{i}\right) \cdot \vec{\rho}=r\left(R_{i}+R_{j}\right) . +\end{gathered} +$$ + +Therefore, + +$$ +r=\frac{d_{i j}}{R_{i}+R_{j}} \cdot d(P) +$$ + +and the value $v_{i j}=\frac{d_{i j}}{R_{i}+R_{j}}$ does not depend on $P$. +Lemma 3. The curved quadrilateral $\mathcal{Q}_{i j}=\sqrt{i, j V_{i+1, j}} V_{i+1, j+1} V_{i, j+1}$ is circumscribed if and only if $u_{i, i+1}=v_{j, j+1}$. +Proof. First suppose that the curved quadrilateral $\mathcal{Q}_{i j}$ is circumscribed and $\Omega(P, r)$ is its inscribed circle. By Lemma 1 and Lemma 2 we have $r=u_{i, i+1} \cdot d(P)$ and $r=v_{j, j+1} \cdot d(P)$ as well. Hence, $u_{i, i+1}=v_{j, j+1}$. + +To prove the opposite direction, suppose $u_{i, i+1}=v_{j, j+1}$. Let $P$ be the intersection of the angle bisector $\beta_{i, i+1}$ and the ellipse arc $\varepsilon_{j, j+1}$. Choose $r=u_{i, i+1} \cdot d(P)=v_{j, j+1} \cdot d(P)$. Then the circle $\Omega(P, r)$ is tangent to the half lines $h_{i}$ and $h_{i+1}$ by Lemma 1 , and it is tangent to the $\operatorname{arcs} \gamma_{j}$ and $\gamma_{j+1}$ by Lemma 2. Hence, the curved quadrilateral $\mathcal{Q}_{i j}$ is circumscribed. + +By Lemma 3, the statement of the problem can be reformulated to an obvious fact: If the equalities $u_{12}=v_{12}, u_{12}=v_{23}$, and $u_{23}=v_{12}$ hold, then $u_{23}=v_{23}$ holds as well. + +Comment 1. Lemma 2(b) (together with the easy Lemma 1(b)) is the key tool in this solution. If one finds this fact, then the solution can be finished in many ways. That is, one can find a circle touching three of $h_{2}, h_{3}, \gamma_{2}$, and $\gamma_{3}$, and then prove that it is tangent to the fourth one in either synthetic or analytical way. Both approaches can be successful. + +Here we present some discussion about this key Lemma. + +1. In the solution above we chose an analytic proof for Lemma 2(b) because we expect that most students will use coordinates or vectors to examine the locus of the centers, and these approaches are less case-sensitive. + +Here we outline a synthetic proof. We consider only the case when $P$ does not lie in the line $O_{i} O_{j}$. The other case can be obtained as a limit case, or computed in a direct way. + +Let $S$ be the internal homothety center between the circles of $\gamma_{i}$ and $\gamma_{j}$, lying on $O_{i} O_{j}$; this point does not depend on $P$. Let $U$ and $V$ be the points of tangency of circle $\sigma=\Omega(P, r)$ with $\gamma_{i}$ and $\gamma_{j}$, respectively (then $r=P U=P V$ ); in other words, points $U$ and $V$ are the intersection points of rays $O_{i} P, O_{j} P$ with arcs $\gamma_{i}, \gamma_{j}$ respectively (see Fig. 4). + +Due to the theorem on three homothety centers (or just to the Menelaus theorem applied to triangle $O_{i} O_{j} P$ ), the points $U, V$ and $S$ are collinear. Let $T$ be the intersection point of line $A C$ and the common tangent to $\sigma$ and $\gamma_{i}$ at $U$; then $T$ is the radical center of $\sigma, \gamma_{i}$ and $\gamma_{j}$, hence $T V$ is the common tangent to $\sigma$ and $\gamma_{j}$. + +Let $Q$ be the projection of $P$ onto the line $A C$. By the right angles, the points $U, V$ and $Q$ lie on the circle with diameter $P T$. From this fact and the equality $P U=P V$ we get $\angle U Q P=\angle U V P=$ $\angle V U P=\angle S U O_{i}$. Since $O_{i} S \| P Q$, we have $\angle S O_{i} U=\angle Q P U$. Hence, the triangles $S O_{i} U$ and $U P Q$ are similar and thus $\frac{r}{d(P)}=\frac{P U}{P Q}=\frac{O_{i} S}{O_{i} U}=\frac{O_{i} S}{R_{i}}$; the last expression is constant since $S$ is a constant point. +![](https://cdn.mathpix.com/cropped/2024_11_18_ba86565be3364967ae6fg-63.jpg?height=595&width=797&top_left_y=1570&top_left_x=361) + +Fig. 4 +![](https://cdn.mathpix.com/cropped/2024_11_18_ba86565be3364967ae6fg-63.jpg?height=647&width=546&top_left_y=1524&top_left_x=1212) + +Fig. 5 +2. Using some known facts about conics, the same statement can be proved in a very short way. Denote by $\ell$ the directrix of ellipse of $\varepsilon_{i j}$ related to the focus $O_{j}$; since $\varepsilon_{i j}$ is symmetrical about $O_{i} O_{j}$, we have $\ell \| A C$. Recall that for each point $P \in \varepsilon_{i j}$, we have $P O_{j}=\epsilon \cdot d_{\ell}(P)$, where $d_{\ell}(P)$ is the distance from $P$ to $\ell$, and $\epsilon$ is the eccentricity of $\varepsilon_{i j}$ (see Fig. 5). + +Now we have + +$$ +r=R_{j}-\left(R_{j}-r\right)=A O_{j}-P O_{j}=\epsilon\left(d_{\ell}(A)-d_{\ell}(P)\right)=\epsilon(d(P)-d(A))=\epsilon \cdot d(P), +$$ + +and $\epsilon$ does not depend on $P$. + +Comment 2. One can find a spatial interpretations of the problem and the solution. +For every point $(x, y)$ and radius $r>0$, represent the circle $\Omega((x, y), r)$ by the point $(x, y, r)$ in space. This point is the apex of the cone with base circle $\Omega((x, y), r)$ and height $r$. According to Lemma 1 , the circles which are tangent to $h_{i}$ and $h_{j}$ correspond to the points of a half line $\beta_{i j}^{\prime}$, starting at $B$. + +Now we translate Lemma 2. Take some $1 \leq i1$ since otherwise $1-\frac{1}{s_{1}}=0$. So we have $2 \leq s_{1} \leq s_{2}-1 \leq \cdots \leq s_{n}-(n-1)$, hence $s_{i} \geq i+1$ for each $i=1, \ldots, n$. Therefore + +$$ +\begin{aligned} +\frac{51}{2010} & =\left(1-\frac{1}{s_{1}}\right)\left(1-\frac{1}{s_{2}}\right) \ldots\left(1-\frac{1}{s_{n}}\right) \\ +& \geq\left(1-\frac{1}{2}\right)\left(1-\frac{1}{3}\right) \ldots\left(1-\frac{1}{n+1}\right)=\frac{1}{2} \cdot \frac{2}{3} \cdots \frac{n}{n+1}=\frac{1}{n+1} +\end{aligned} +$$ + +which implies + +$$ +n+1 \geq \frac{2010}{51}=\frac{670}{17}>39 +$$ + +so $n \geq 39$. +Now we are left to show that $n=39$ fits. Consider the set $\{2,3, \ldots, 33,35,36, \ldots, 40,67\}$ which contains exactly 39 numbers. We have + +$$ +\frac{1}{2} \cdot \frac{2}{3} \cdots \frac{32}{33} \cdot \frac{34}{35} \cdots \frac{39}{40} \cdot \frac{66}{67}=\frac{1}{33} \cdot \frac{34}{40} \cdot \frac{66}{67}=\frac{17}{670}=\frac{51}{2010} +$$ + +hence for $n=39$ there exists a desired example. +Comment. One can show that the example (1) is unique. +Answer for Problem N1' ${ }^{\prime} n=48$. +Solution for Problem N1'. Suppose that for some $n$ there exist the desired numbers. In the same way we obtain that $s_{i} \geq i+1$. Moreover, since the denominator of the fraction $\frac{42}{2010}=\frac{7}{335}$ is divisible by 67 , some of $s_{i}$ 's should be divisible by 67 , so $s_{n} \geq s_{i} \geq 67$. This means that + +$$ +\frac{42}{2010} \geq \frac{1}{2} \cdot \frac{2}{3} \cdots \frac{n-1}{n} \cdot\left(1-\frac{1}{67}\right)=\frac{66}{67 n} +$$ + +which implies + +$$ +n \geq \frac{2010 \cdot 66}{42 \cdot 67}=\frac{330}{7}>47 +$$ + +so $n \geq 48$. +Now we are left to show that $n=48$ fits. Consider the set $\{2,3, \ldots, 33,36,37, \ldots, 50,67\}$ which contains exactly 48 numbers. We have + +$$ +\frac{1}{2} \cdot \frac{2}{3} \cdots \frac{32}{33} \cdot \frac{35}{36} \cdots \frac{49}{50} \cdot \frac{66}{67}=\frac{1}{33} \cdot \frac{35}{50} \cdot \frac{66}{67}=\frac{7}{335}=\frac{42}{2010} +$$ + +hence for $n=48$ there exists a desired example. +Comment 1. In this version of the problem, the estimate needs one more step, hence it is a bit harder. On the other hand, the example in this version is not unique. Another example is + +$$ +\frac{1}{2} \cdot \frac{2}{3} \cdots \frac{46}{47} \cdot \frac{66}{67} \cdot \frac{329}{330}=\frac{1}{67} \cdot \frac{66}{330} \cdot \frac{329}{47}=\frac{7}{67 \cdot 5}=\frac{42}{2010} . +$$ + +Comment 2. N1' was the Proposer's formulation of the problem. We propose N1 according to the number of current IMO. + +N2. Find all pairs $(m, n)$ of nonnegative integers for which + +$$ +m^{2}+2 \cdot 3^{n}=m\left(2^{n+1}-1\right) +$$ + +(Australia) +Answer. $(6,3),(9,3),(9,5),(54,5)$. +Solution. For fixed values of $n$, the equation (1) is a simple quadratic equation in $m$. For $n \leq 5$ the solutions are listed in the following table. + +| case | equation | discriminant | integer roots | +| :--- | :--- | :--- | :--- | +| $n=0$ | $m^{2}-m+2=0$ | -7 | none | +| $n=1$ | $m^{2}-3 m+6=0$ | -15 | none | +| $n=2$ | $m^{2}-7 m+18=0$ | -23 | none | +| $n=3$ | $m^{2}-15 m+54=0$ | 9 | $m=6$ and $m=9$ | +| $n=4$ | $m^{2}-31 m+162=0$ | 313 | none | +| $n=5$ | $m^{2}-63 m+486=0$ | $2025=45^{2}$ | $m=9$ and $m=54$ | + +We prove that there is no solution for $n \geq 6$. +Suppose that $(m, n)$ satisfies (1) and $n \geq 6$. Since $m \mid 2 \cdot 3^{n}=m\left(2^{n+1}-1\right)-m^{2}$, we have $m=3^{p}$ with some $0 \leq p \leq n$ or $m=2 \cdot 3^{q}$ with some $0 \leq q \leq n$. + +In the first case, let $q=n-p$; then + +$$ +2^{n+1}-1=m+\frac{2 \cdot 3^{n}}{m}=3^{p}+2 \cdot 3^{q} +$$ + +In the second case let $p=n-q$. Then + +$$ +2^{n+1}-1=m+\frac{2 \cdot 3^{n}}{m}=2 \cdot 3^{q}+3^{p} +$$ + +Hence, in both cases we need to find the nonnegative integer solutions of + +$$ +3^{p}+2 \cdot 3^{q}=2^{n+1}-1, \quad p+q=n +$$ + +Next, we prove bounds for $p, q$. From (2) we get + +$$ +3^{p}<2^{n+1}=8^{\frac{n+1}{3}}<9^{\frac{n+1}{3}}=3^{\frac{2(n+1)}{3}} +$$ + +and + +$$ +2 \cdot 3^{q}<2^{n+1}=2 \cdot 8^{\frac{n}{3}}<2 \cdot 9^{\frac{n}{3}}=2 \cdot 3^{\frac{2 n}{3}}<2 \cdot 3^{\frac{2(n+1)}{3}} +$$ + +so $p, q<\frac{2(n+1)}{3}$. Combining these inequalities with $p+q=n$, we obtain + +$$ +\frac{n-2}{3}\frac{n-2}{3}$; in particular, we have $h>1$. On the left-hand side of (2), both terms are divisible by $3^{h}$, therefore $9\left|3^{h}\right| 2^{n+1}-1$. It is easy check that $\operatorname{ord}_{9}(2)=6$, so $9 \mid 2^{n+1}-1$ if and only if $6 \mid n+1$. Therefore, $n+1=6 r$ for some positive integer $r$, and we can write + +$$ +2^{n+1}-1=4^{3 r}-1=\left(4^{2 r}+4^{r}+1\right)\left(2^{r}-1\right)\left(2^{r}+1\right) +$$ + +Notice that the factor $4^{2 r}+4^{r}+1=\left(4^{r}-1\right)^{2}+3 \cdot 4^{r}$ is divisible by 3 , but it is never divisible by 9 . The other two factors in (4), $2^{r}-1$ and $2^{r}+1$ are coprime: both are odd and their difference is 2 . Since the whole product is divisible by $3^{h}$, we have either $3^{h-1} \mid 2^{r}-1$ or $3^{h-1} \mid 2^{r}+1$. In any case, we have $3^{h-1} \leq 2^{r}+1$. Then + +$$ +\begin{gathered} +3^{h-1} \leq 2^{r}+1 \leq 3^{r}=3^{\frac{n+1}{6}} \\ +\frac{n-2}{3}-10$ of the system of equations +(i) $\sum_{i=1}^{4} x_{i}^{2}=8 m^{2}$, +(ii) $\sum_{i=1}^{4} y_{i}^{2}=8 m^{2}$, +(iii) $\sum_{i=1}^{4} x_{i} y_{i}=-6 m^{2}$. + +We will show that such a solution does not exist. +Assume the contrary and consider a solution with minimal $m$. Note that if an integer $x$ is odd then $x^{2} \equiv 1(\bmod 8)$. Otherwise (i.e., if $x$ is even) we have $x^{2} \equiv 0(\bmod 8)$ or $x^{2} \equiv 4$ $(\bmod 8)$. Hence, by (i), we get that $x_{1}, x_{2}, x_{3}$ and $x_{4}$ are even. Similarly, by (ii), we get that $y_{1}, y_{2}, y_{3}$ and $y_{4}$ are even. Thus the LHS of (iii) is divisible by 4 and $m$ is also even. It follows that $\left(\frac{x_{1}}{2}, \frac{y_{1}}{2}, \frac{x_{2}}{2}, \frac{y_{2}}{2}, \frac{x_{3}}{2}, \frac{y_{3}}{2}, \frac{x_{4}}{2}, \frac{y_{4}}{2}, \frac{m}{2}\right)$ is a solution of the system of equations (i), (ii) and (iii), which contradicts the minimality of $m$. + +Solution 2. We prove that $n \leq 4$ is impossible. Define the numbers $a_{i}, b_{i}$ for $i=1,2,3,4$ as in the previous solution. + +By Euler's identity we have + +$$ +\begin{aligned} +\left(a_{1}^{2}+a_{2}^{2}+a_{3}^{2}+a_{4}^{2}\right)\left(b_{1}^{2}+b_{2}^{2}+b_{3}^{2}+b_{4}^{2}\right) & =\left(a_{1} b_{1}+a_{2} b_{2}+a_{3} b_{3}+a_{4} b_{4}\right)^{2}+\left(a_{1} b_{2}-a_{2} b_{1}+a_{3} b_{4}-a_{4} b_{3}\right)^{2} \\ +& +\left(a_{1} b_{3}-a_{3} b_{1}+a_{4} b_{2}-a_{2} b_{4}\right)^{2}+\left(a_{1} b_{4}-a_{4} b_{1}+a_{2} b_{3}-a_{3} b_{2}\right)^{2} . +\end{aligned} +$$ + +So, using the relations (1) from the Solution 1 we get that + +$$ +7=\left(\frac{m_{1}}{m}\right)^{2}+\left(\frac{m_{2}}{m}\right)^{2}+\left(\frac{m_{3}}{m}\right)^{2} +$$ + +where + +$$ +\begin{aligned} +& \frac{m_{1}}{m}=a_{1} b_{2}-a_{2} b_{1}+a_{3} b_{4}-a_{4} b_{3} \\ +& \frac{m_{2}}{m}=a_{1} b_{3}-a_{3} b_{1}+a_{4} b_{2}-a_{2} b_{4} \\ +& \frac{m_{3}}{m}=a_{1} b_{4}-a_{4} b_{1}+a_{2} b_{3}-a_{3} b_{2} +\end{aligned} +$$ + +and $m_{1}, m_{2}, m_{3} \in \mathbb{Z}, m \in \mathbb{N}$. +Let $m$ be a minimum positive integer number for which (2) holds. Then + +$$ +8 m^{2}=m_{1}^{2}+m_{2}^{2}+m_{3}^{2}+m^{2} +$$ + +As in the previous solution, we get that $m_{1}, m_{2}, m_{3}, m$ are all even numbers. Then $\left(\frac{m_{1}}{2}, \frac{m_{2}}{2}, \frac{m_{3}}{2}, \frac{m}{2}\right)$ is also a solution of ( 2 ) which contradicts the minimality of $m$. So, we have $n \geq 5$. The example with $n=5$ is already shown in Solution 1 . + +N4. Let $a, b$ be integers, and let $P(x)=a x^{3}+b x$. For any positive integer $n$ we say that the pair $(a, b)$ is $n$-good if $n \mid P(m)-P(k)$ implies $n \mid m-k$ for all integers $m, k$. We say that $(a, b)$ is very good if $(a, b)$ is $n$-good for infinitely many positive integers $n$. +(a) Find a pair $(a, b)$ which is 51 -good, but not very good. +(b) Show that all 2010-good pairs are very good. +(Turkey) +Solution. (a) We show that the pair $\left(1,-51^{2}\right)$ is good but not very good. Let $P(x)=x^{3}-51^{2} x$. Since $P(51)=P(0)$, the pair $\left(1,-51^{2}\right)$ is not $n$-good for any positive integer that does not divide 51 . Therefore, $\left(1,-51^{2}\right)$ is not very good. + +On the other hand, if $P(m) \equiv P(k)(\bmod 51)$, then $m^{3} \equiv k^{3}(\bmod 51)$. By Fermat's theorem, from this we obtain + +$$ +m \equiv m^{3} \equiv k^{3} \equiv k \quad(\bmod 3) \quad \text { and } \quad m \equiv m^{33} \equiv k^{33} \equiv k \quad(\bmod 17) . +$$ + +Hence we have $m \equiv k(\bmod 51)$. Therefore $\left(1,-51^{2}\right)$ is 51 -good. +(b) We will show that if a pair $(a, b)$ is 2010-good then $(a, b)$ is $67^{i}$-good for all positive integer $i$. +Claim 1. If $(a, b)$ is 2010 -good then $(a, b)$ is 67 -good. +Proof. Assume that $P(m)=P(k)(\bmod 67)$. Since 67 and 30 are coprime, there exist integers $m^{\prime}$ and $k^{\prime}$ such that $k^{\prime} \equiv k(\bmod 67), k^{\prime} \equiv 0(\bmod 30)$, and $m^{\prime} \equiv m(\bmod 67), m^{\prime} \equiv 0$ $(\bmod 30)$. Then we have $P\left(m^{\prime}\right) \equiv P(0) \equiv P\left(k^{\prime}\right)(\bmod 30)$ and $P\left(m^{\prime}\right) \equiv P(m) \equiv P(k) \equiv P\left(k^{\prime}\right)$ $(\bmod 67)$, hence $P\left(m^{\prime}\right) \equiv P\left(k^{\prime}\right)(\bmod 2010)$. This implies $m^{\prime} \equiv k^{\prime}(\bmod 2010)$ as $(a, b)$ is 2010 -good. It follows that $m \equiv m^{\prime} \equiv k^{\prime} \equiv k(\bmod 67)$. Therefore, $(a, b)$ is 67 -good. +Claim 2. If $(a, b)$ is 67 -good then $67 \mid a$. +Proof. Suppose that $67 \nmid a$. Consider the sets $\left\{a t^{2}(\bmod 67): 0 \leq t \leq 33\right\}$ and $\left\{-3 a s^{2}-b\right.$ $\bmod 67: 0 \leq s \leq 33\}$. Since $a \not \equiv 0(\bmod 67)$, each of these sets has 34 elements. Hence they have at least one element in common. If $a t^{2} \equiv-3 a s^{2}-b(\bmod 67)$ then for $m=t \pm s, k=\mp 2 s$ we have + +$$ +\begin{aligned} +P(m)-P(k)=a\left(m^{3}-k^{3}\right)+b(m-k) & =(m-k)\left(a\left(m^{2}+m k+k^{2}\right)+b\right) \\ +& =(t \pm 3 s)\left(a t^{2}+3 a s^{2}+b\right) \equiv 0 \quad(\bmod 67) +\end{aligned} +$$ + +Since $(a, b)$ is 67 -good, we must have $m \equiv k(\bmod 67)$ in both cases, that is, $t \equiv 3 s(\bmod 67)$ and $t \equiv-3 s(\bmod 67)$. This means $t \equiv s \equiv 0(\bmod 67)$ and $b \equiv-3 a s^{2}-a t^{2} \equiv 0(\bmod 67)$. But then $67 \mid P(7)-P(2)=67 \cdot 5 a+5 b$ and $67 \not 7-2$, contradicting that $(a, b)$ is 67 -good. +Claim 3. If $(a, b)$ is 2010 -good then $(a, b)$ is $67^{i}$-good all $i \geq 1$. +Proof. By Claim 2, we have $67 \mid a$. If $67 \mid b$, then $P(x) \equiv P(0)(\bmod 67)$ for all $x$, contradicting that $(a, b)$ is 67 -good. Hence, $67 \nmid b$. + +Suppose that $67^{i} \mid P(m)-P(k)=(m-k)\left(a\left(m^{2}+m k+k^{2}\right)+b\right)$. Since $67 \mid a$ and $67 \nmid b$, the second factor $a\left(m^{2}+m k+k^{2}\right)+b$ is coprime to 67 and hence $67^{i} \mid m-k$. Therefore, $(a, b)$ is $67^{i}$-good. +Comment 1. In the proof of Claim 2, the following reasoning can also be used. Since 3 is not a quadratic residue modulo 67 , either $a u^{2} \equiv-b(\bmod 67)$ or $3 a v^{2} \equiv-b(\bmod 67)$ has a solution. The settings $(m, k)=(u, 0)$ in the first case and $(m, k)=(v,-2 v)$ in the second case lead to $b \equiv 0$ $(\bmod 67)$. +Comment 2. The pair $(67,30)$ is $n$-good if and only if $n=d \cdot 67^{i}$, where $d \mid 30$ and $i \geq 0$. It shows that in part (b), one should deal with the large powers of 67 to reach the solution. The key property of number 67 is that it has the form $3 k+1$, so there exists a nontrivial cubic root of unity modulo 67 . + +N5. Let $\mathbb{N}$ be the set of all positive integers. Find all functions $f: \mathbb{N} \rightarrow \mathbb{N}$ such that the number $(f(m)+n)(m+f(n))$ is a square for all $m, n \in \mathbb{N}$. + +Answer. All functions of the form $f(n)=n+c$, where $c \in \mathbb{N} \cup\{0\}$. +Solution. First, it is clear that all functions of the form $f(n)=n+c$ with a constant nonnegative integer $c$ satisfy the problem conditions since $(f(m)+n)(f(n)+m)=(n+m+c)^{2}$ is a square. + +We are left to prove that there are no other functions. We start with the following Lemma. Suppose that $p \mid f(k)-f(\ell)$ for some prime $p$ and positive integers $k, \ell$. Then $p \mid k-\ell$. Proof. Suppose first that $p^{2} \mid f(k)-f(\ell)$, so $f(\ell)=f(k)+p^{2} a$ for some integer $a$. Take some positive integer $D>\max \{f(k), f(\ell)\}$ which is not divisible by $p$ and set $n=p D-f(k)$. Then the positive numbers $n+f(k)=p D$ and $n+f(\ell)=p D+(f(\ell)-f(k))=p(D+p a)$ are both divisible by $p$ but not by $p^{2}$. Now, applying the problem conditions, we get that both the numbers $(f(k)+n)(f(n)+k)$ and $(f(\ell)+n)(f(n)+\ell)$ are squares divisible by $p$ (and thus by $p^{2}$ ); this means that the multipliers $f(n)+k$ and $f(n)+\ell$ are also divisible by $p$, therefore $p \mid(f(n)+k)-(f(n)+\ell)=k-\ell$ as well. + +On the other hand, if $f(k)-f(\ell)$ is divisible by $p$ but not by $p^{2}$, then choose the same number $D$ and set $n=p^{3} D-f(k)$. Then the positive numbers $f(k)+n=p^{3} D$ and $f(\ell)+n=$ $p^{3} D+(f(\ell)-f(k))$ are respectively divisible by $p^{3}$ (but not by $p^{4}$ ) and by $p$ (but not by $p^{2}$ ). Hence in analogous way we obtain that the numbers $f(n)+k$ and $f(n)+\ell$ are divisible by $p$, therefore $p \mid(f(n)+k)-(f(n)+\ell)=k-\ell$. + +We turn to the problem. First, suppose that $f(k)=f(\ell)$ for some $k, \ell \in \mathbb{N}$. Then by Lemma we have that $k-\ell$ is divisible by every prime number, so $k-\ell=0$, or $k=\ell$. Therefore, the function $f$ is injective. + +Next, consider the numbers $f(k)$ and $f(k+1)$. Since the number $(k+1)-k=1$ has no prime divisors, by Lemma the same holds for $f(k+1)-f(k)$; thus $|f(k+1)-f(k)|=1$. + +Now, let $f(2)-f(1)=q,|q|=1$. Then we prove by induction that $f(n)=f(1)+q(n-1)$. The base for $n=1,2$ holds by the definition of $q$. For the step, if $n>1$ we have $f(n+1)=$ $f(n) \pm q=f(1)+q(n-1) \pm q$. Since $f(n) \neq f(n-2)=f(1)+q(n-2)$, we get $f(n)=f(1)+q n$, as desired. + +Finally, we have $f(n)=f(1)+q(n-1)$. Then $q$ cannot be -1 since otherwise for $n \geq f(1)+1$ we have $f(n) \leq 0$ which is impossible. Hence $q=1$ and $f(n)=(f(1)-1)+n$ for each $n \in \mathbb{N}$, and $f(1)-1 \geq 0$, as desired. + +N6. The rows and columns of a $2^{n} \times 2^{n}$ table are numbered from 0 to $2^{n}-1$. The cells of the table have been colored with the following property being satisfied: for each $0 \leq i, j \leq 2^{n}-1$, the $j$ th cell in the $i$ th row and the $(i+j)$ th cell in the $j$ th row have the same color. (The indices of the cells in a row are considered modulo $2^{n}$.) + +Prove that the maximal possible number of colors is $2^{n}$. + +Solution. Throughout the solution we denote the cells of the table by coordinate pairs; $(i, j)$ refers to the $j$ th cell in the $i$ th row. + +Consider the directed graph, whose vertices are the cells of the board, and the edges are the arrows $(i, j) \rightarrow(j, i+j)$ for all $0 \leq i, j \leq 2^{n}-1$. From each vertex $(i, j)$, exactly one edge passes $\left(\right.$ to $\left(j, i+j \bmod 2^{n}\right)$ ); conversely, to each cell $(j, k)$ exactly one edge is directed (from the cell $\left.\left(k-j \bmod 2^{n}, j\right)\right)$. Hence, the graph splits into cycles. + +Now, in any coloring considered, the vertices of each cycle should have the same color by the problem condition. On the other hand, if each cycle has its own color, the obtained coloring obviously satisfies the problem conditions. Thus, the maximal possible number of colors is the same as the number of cycles, and we have to prove that this number is $2^{n}$. + +Next, consider any cycle $\left(i_{1}, j_{1}\right),\left(i_{2}, j_{2}\right), \ldots$; we will describe it in other terms. Define a sequence $\left(a_{0}, a_{1}, \ldots\right)$ by the relations $a_{0}=i_{1}, a_{1}=j_{1}, a_{n+1}=a_{n}+a_{n-1}$ for all $n \geq 1$ (we say that such a sequence is a Fibonacci-type sequence). Then an obvious induction shows that $i_{k} \equiv a_{k-1}\left(\bmod 2^{n}\right), j_{k} \equiv a_{k}\left(\bmod 2^{n}\right)$. Hence we need to investigate the behavior of Fibonacci-type sequences modulo $2^{n}$. + +Denote by $F_{0}, F_{1}, \ldots$ the Fibonacci numbers defined by $F_{0}=0, F_{1}=1$, and $F_{n+2}=$ $F_{n+1}+F_{n}$ for $n \geq 0$. We also set $F_{-1}=1$ according to the recurrence relation. + +For every positive integer $m$, denote by $\nu(m)$ the exponent of 2 in the prime factorization of $m$, i.e. for which $2^{\nu(m)} \mid m$ but $2^{\nu(m)+1} \backslash m$. +Lemma 1. For every Fibonacci-type sequence $a_{0}, a_{1}, a_{2}, \ldots$, and every $k \geq 0$, we have $a_{k}=$ $F_{k-1} a_{0}+F_{k} a_{1}$. +Proof. Apply induction on $k$. The base cases $k=0,1$ are trivial. For the step, from the induction hypothesis we get + +$$ +a_{k+1}=a_{k}+a_{k-1}=\left(F_{k-1} a_{0}+F_{k} a_{1}\right)+\left(F_{k-2} a_{0}+F_{k-1} a_{1}\right)=F_{k} a_{0}+F_{k+1} a_{1} +$$ + +Lemma 2. For every $m \geq 3$, +(a) we have $\nu\left(F_{3 \cdot 2^{m-2}}\right)=m$; +(b) $d=3 \cdot 2^{m-2}$ is the least positive index for which $2^{m} \mid F_{d}$; +(c) $F_{3 \cdot 2^{m-2}+1} \equiv 1+2^{m-1}\left(\bmod 2^{m}\right)$. + +Proof. Apply induction on $m$. In the base case $m=3$ we have $\nu\left(F_{3 \cdot 2^{m-2}}\right)=F_{6}=8$, so $\nu\left(F_{3 \cdot 2^{m-2}}\right)=\nu(8)=3$, the preceding Fibonacci-numbers are not divisible by 8 , and indeed $F_{3 \cdot 2^{m-2}+1}=F_{7}=13 \equiv 1+4(\bmod 8)$. + +Now suppose that $m>3$ and let $k=3 \cdot 2^{m-3}$. By applying Lemma 1 to the Fibonacci-type sequence $F_{k}, F_{k+1}, \ldots$ we get + +$$ +\begin{gathered} +F_{2 k}=F_{k-1} F_{k}+F_{k} F_{k+1}=\left(F_{k+1}-F_{k}\right) F_{k}+F_{k+1} F_{k}=2 F_{k+1} F_{k}-F_{k}^{2} \\ +F_{2 k+1}=F_{k} \cdot F_{k}+F_{k+1} \cdot F_{k+1}=F_{k}^{2}+F_{k+1}^{2} +\end{gathered} +$$ + +By the induction hypothesis, $\nu\left(F_{k}\right)=m-1$, and $F_{k+1}$ is odd. Therefore we get $\nu\left(F_{k}^{2}\right)=$ $2(m-1)>(m-1)+1=\nu\left(2 F_{k} F_{k+1}\right)$, which implies $\nu\left(F_{2 k}\right)=m$, establishing statement (a). + +Moreover, since $F_{k+1}=1+2^{m-2}+a 2^{m-1}$ for some integer $a$, we get + +$$ +F_{2 k+1}=F_{k}^{2}+F_{k+1}^{2} \equiv 0+\left(1+2^{m-2}+a 2^{m-1}\right)^{2} \equiv 1+2^{m-1} \quad\left(\bmod 2^{m}\right) +$$ + +as desired in statement (c). +We are left to prove that $2^{m} \backslash F_{\ell}$ for $\ell<2 k$. Assume the contrary. Since $2^{m-1} \mid F_{\ell}$, from the induction hypothesis it follows that $\ell>k$. But then we have $F_{\ell}=F_{k-1} F_{\ell-k}+F_{k} F_{\ell-k+1}$, where the second summand is divisible by $2^{m-1}$ but the first one is not (since $F_{k-1}$ is odd and $\ell-k0$ such that $a_{k+p} \equiv a_{k}\left(\bmod 2^{n}\right)$ for all $k \geq 0$. +Lemma 3. Let $A=\left(a_{0}, a_{1}, \ldots\right)$ be a Fibonacci-type sequence such that $\mu(A)=k Mathematical Olympiad
12-24 July 2011
Amsterdam
The Netherlands + +Problem shortlist with solutions + +## IMO regulation:
these shortlist problems have to be kept strictly confidential until IMO 2012. + +The problem selection committee + +Bart de Smit (chairman), Ilya Bogdanov, Johan Bosman, + +Andries Brouwer, Gabriele Dalla Torre, Géza Kós, + +Hendrik Lenstra, Charles Leytem, Ronald van Luijk, + +Christian Reiher, Eckard Specht, Hans Sterk, Lenny Taelman + +The committee gratefully acknowledges the receipt of 142 problem proposals by the following 46 countries: + +Armenia, Australia, Austria, Belarus, Belgium, Bosnia and Herzegovina, Brazil, Bulgaria, Canada, Colombia, Cyprus, Denmark, Estonia, Finland, France, Germany, Greece, Hong Kong, Hungary, India, Islamic Republic of Iran, Ireland, Israel, Japan, Kazakhstan, Republic of Korea, Luxembourg, Malaysia, Mexico, Mongolia, Montenegro, Pakistan, Poland, Romania, Russian Federation, Saudi Arabia, Serbia, Slovakia, Slovenia, Sweden, Taiwan, Thailand, Turkey, Ukraine, United Kingdom, United States of America + +## Algebra + +## A1 + +For any set $A=\left\{a_{1}, a_{2}, a_{3}, a_{4}\right\}$ of four distinct positive integers with sum $s_{A}=a_{1}+a_{2}+a_{3}+a_{4}$, let $p_{A}$ denote the number of pairs $(i, j)$ with $1 \leq i\sqrt{2}$ and $a^{2}+b^{2}+c^{2}=3$. Prove that + +$$ +\frac{a}{(b+c-a)^{2}}+\frac{b}{(c+a-b)^{2}}+\frac{c}{(a+b-c)^{2}} \geq \frac{3}{(a b c)^{2}} +$$ + +## Combinatorics + +## C1 + +Let $n>0$ be an integer. We are given a balance and $n$ weights of weight $2^{0}, 2^{1}, \ldots, 2^{n-1}$. In a sequence of $n$ moves we place all weights on the balance. In the first move we choose a weight and put it on the left pan. In each of the following moves we choose one of the remaining weights and we add it either to the left or to the right pan. Compute the number of ways in which we can perform these $n$ moves in such a way that the right pan is never heavier than the left pan. + +## $\mathrm{C} 2$ + +Suppose that 1000 students are standing in a circle. Prove that there exists an integer $k$ with $100 \leq k \leq 300$ such that in this circle there exists a contiguous group of $2 k$ students, for which the first half contains the same number of girls as the second half. + +## C3 + +Let $\mathcal{S}$ be a finite set of at least two points in the plane. Assume that no three points of $\mathcal{S}$ are collinear. By a windmill we mean a process as follows. Start with a line $\ell$ going through a point $P \in \mathcal{S}$. Rotate $\ell$ clockwise around the pivot $P$ until the line contains another point $Q$ of $\mathcal{S}$. The point $Q$ now takes over as the new pivot. This process continues indefinitely, with the pivot always being a point from $\mathcal{S}$. + +Show that for a suitable $P \in \mathcal{S}$ and a suitable starting line $\ell$ containing $P$, the resulting windmill will visit each point of $\mathcal{S}$ as a pivot infinitely often. + +## $\mathrm{C} 4$ + +Determine the greatest positive integer $k$ that satisfies the following property: The set of positive integers can be partitioned into $k$ subsets $A_{1}, A_{2}, \ldots, A_{k}$ such that for all integers $n \geq 15$ and all $i \in\{1,2, \ldots, k\}$ there exist two distinct elements of $A_{i}$ whose sum is $n$. + +## C5 + +Let $m$ be a positive integer and consider a checkerboard consisting of $m$ by $m$ unit squares. At the midpoints of some of these unit squares there is an ant. At time 0 , each ant starts moving with speed 1 parallel to some edge of the checkerboard. When two ants moving in opposite directions meet, they both turn $90^{\circ}$ clockwise and continue moving with speed 1 . When more than two ants meet, or when two ants moving in perpendicular directions meet, the ants continue moving in the same direction as before they met. When an ant reaches one of the edges of the checkerboard, it falls off and will not re-appear. + +Considering all possible starting positions, determine the latest possible moment at which the last ant falls off the checkerboard or prove that such a moment does not necessarily exist. + +## C6 + +Let $n$ be a positive integer and let $W=\ldots x_{-1} x_{0} x_{1} x_{2} \ldots$ be an infinite periodic word consisting of the letters $a$ and $b$. Suppose that the minimal period $N$ of $W$ is greater than $2^{n}$. + +A finite nonempty word $U$ is said to appear in $W$ if there exist indices $k \leq \ell$ such that $U=x_{k} x_{k+1} \ldots x_{\ell}$. A finite word $U$ is called ubiquitous if the four words $U a, U b, a U$, and $b U$ all appear in $W$. Prove that there are at least $n$ ubiquitous finite nonempty words. + +## C7 + +On a square table of 2011 by 2011 cells we place a finite number of napkins that each cover a square of 52 by 52 cells. In each cell we write the number of napkins covering it, and we record the maximal number $k$ of cells that all contain the same nonzero number. Considering all possible napkin configurations, what is the largest value of $k$ ? + +## Geometry + +## G1 + +Let $A B C$ be an acute triangle. Let $\omega$ be a circle whose center $L$ lies on the side $B C$. Suppose that $\omega$ is tangent to $A B$ at $B^{\prime}$ and to $A C$ at $C^{\prime}$. Suppose also that the circumcenter $O$ of the triangle $A B C$ lies on the shorter $\operatorname{arc} B^{\prime} C^{\prime}$ of $\omega$. Prove that the circumcircle of $A B C$ and $\omega$ meet at two points. + +## G2 + +Let $A_{1} A_{2} A_{3} A_{4}$ be a non-cyclic quadrilateral. Let $O_{1}$ and $r_{1}$ be the circumcenter and the circumradius of the triangle $A_{2} A_{3} A_{4}$. Define $O_{2}, O_{3}, O_{4}$ and $r_{2}, r_{3}, r_{4}$ in a similar way. Prove that + +$$ +\frac{1}{O_{1} A_{1}^{2}-r_{1}^{2}}+\frac{1}{O_{2} A_{2}^{2}-r_{2}^{2}}+\frac{1}{O_{3} A_{3}^{2}-r_{3}^{2}}+\frac{1}{O_{4} A_{4}^{2}-r_{4}^{2}}=0 +$$ + +## G3 + +Let $A B C D$ be a convex quadrilateral whose sides $A D$ and $B C$ are not parallel. Suppose that the circles with diameters $A B$ and $C D$ meet at points $E$ and $F$ inside the quadrilateral. Let $\omega_{E}$ be the circle through the feet of the perpendiculars from $E$ to the lines $A B, B C$, and $C D$. Let $\omega_{F}$ be the circle through the feet of the perpendiculars from $F$ to the lines $C D, D A$, and $A B$. Prove that the midpoint of the segment $E F$ lies on the line through the two intersection points of $\omega_{E}$ and $\omega_{F}$. + +## G4 + +Let $A B C$ be an acute triangle with circumcircle $\Omega$. Let $B_{0}$ be the midpoint of $A C$ and let $C_{0}$ be the midpoint of $A B$. Let $D$ be the foot of the altitude from $A$, and let $G$ be the centroid of the triangle $A B C$. Let $\omega$ be a circle through $B_{0}$ and $C_{0}$ that is tangent to the circle $\Omega$ at a point $X \neq A$. Prove that the points $D, G$, and $X$ are collinear. + +## G5 + +Let $A B C$ be a triangle with incenter $I$ and circumcircle $\omega$. Let $D$ and $E$ be the second intersection points of $\omega$ with the lines $A I$ and $B I$, respectively. The chord $D E$ meets $A C$ at a point $F$, and $B C$ at a point $G$. Let $P$ be the intersection point of the line through $F$ parallel to $A D$ and the line through $G$ parallel to $B E$. Suppose that the tangents to $\omega$ at $A$ and at $B$ meet at a point $K$. Prove that the three lines $A E, B D$, and $K P$ are either parallel or concurrent. + +## G6 + +Let $A B C$ be a triangle with $A B=A C$, and let $D$ be the midpoint of $A C$. The angle bisector of $\angle B A C$ intersects the circle through $D, B$, and $C$ in a point $E$ inside the triangle $A B C$. The line $B D$ intersects the circle through $A, E$, and $B$ in two points $B$ and $F$. The lines $A F$ and $B E$ meet at a point $I$, and the lines $C I$ and $B D$ meet at a point $K$. Show that $I$ is the incenter of triangle $K A B$. + +## G7 + +Let $A B C D E F$ be a convex hexagon all of whose sides are tangent to a circle $\omega$ with center $O$. Suppose that the circumcircle of triangle $A C E$ is concentric with $\omega$. Let $J$ be the foot of the perpendicular from $B$ to $C D$. Suppose that the perpendicular from $B$ to $D F$ intersects the line $E O$ at a point $K$. Let $L$ be the foot of the perpendicular from $K$ to $D E$. Prove that $D J=D L$. + +## G8 + +Let $A B C$ be an acute triangle with circumcircle $\omega$. Let $t$ be a tangent line to $\omega$. Let $t_{a}, t_{b}$, and $t_{c}$ be the lines obtained by reflecting $t$ in the lines $B C, C A$, and $A B$, respectively. Show that the circumcircle of the triangle determined by the lines $t_{a}, t_{b}$, and $t_{c}$ is tangent to the circle $\omega$. + +## Number Theory + +## N1 + +For any integer $d>0$, let $f(d)$ be the smallest positive integer that has exactly $d$ positive divisors (so for example we have $f(1)=1, f(5)=16$, and $f(6)=12$ ). Prove that for every integer $k \geq 0$ the number $f\left(2^{k}\right)$ divides $f\left(2^{k+1}\right)$. + +## N2 + +Consider a polynomial $P(x)=\left(x+d_{1}\right)\left(x+d_{2}\right) \cdot \ldots \cdot\left(x+d_{9}\right)$, where $d_{1}, d_{2}, \ldots, d_{9}$ are nine distinct integers. Prove that there exists an integer $N$ such that for all integers $x \geq N$ the number $P(x)$ is divisible by a prime number greater than 20 . + +## N3 + +Let $n \geq 1$ be an odd integer. Determine all functions $f$ from the set of integers to itself such that for all integers $x$ and $y$ the difference $f(x)-f(y)$ divides $x^{n}-y^{n}$. + +## N4 + +For each positive integer $k$, let $t(k)$ be the largest odd divisor of $k$. Determine all positive integers $a$ for which there exists a positive integer $n$ such that all the differences + +$$ +t(n+a)-t(n), \quad t(n+a+1)-t(n+1), \quad \ldots, \quad t(n+2 a-1)-t(n+a-1) +$$ + +are divisible by 4 . + +## N5 + +Let $f$ be a function from the set of integers to the set of positive integers. Suppose that for any two integers $m$ and $n$, the difference $f(m)-f(n)$ is divisible by $f(m-n)$. Prove that for all integers $m, n$ with $f(m) \leq f(n)$ the number $f(n)$ is divisible by $f(m)$. + +## N6 + +Let $P(x)$ and $Q(x)$ be two polynomials with integer coefficients such that no nonconstant polynomial with rational coefficients divides both $P(x)$ and $Q(x)$. Suppose that for every positive integer $n$ the integers $P(n)$ and $Q(n)$ are positive, and $2^{Q(n)}-1$ divides $3^{P(n)}-1$. Prove that $Q(x)$ is a constant polynomial. + +## N7 + +Let $p$ be an odd prime number. For every integer $a$, define the number + +$$ +S_{a}=\frac{a}{1}+\frac{a^{2}}{2}+\cdots+\frac{a^{p-1}}{p-1} +$$ + +Let $m$ and $n$ be integers such that + +$$ +S_{3}+S_{4}-3 S_{2}=\frac{m}{n} +$$ + +Prove that $p$ divides $m$. + +## N8 + +Let $k$ be a positive integer and set $n=2^{k}+1$. Prove that $n$ is a prime number if and only if the following holds: there is a permutation $a_{1}, \ldots, a_{n-1}$ of the numbers $1,2, \ldots, n-1$ and a sequence of integers $g_{1}, g_{2}, \ldots, g_{n-1}$ such that $n$ divides $g_{i}^{a_{i}}-a_{i+1}$ for every $i \in\{1,2, \ldots, n-1\}$, where we set $a_{n}=a_{1}$. + +## A1 + +For any set $A=\left\{a_{1}, a_{2}, a_{3}, a_{4}\right\}$ of four distinct positive integers with sum $s_{A}=a_{1}+a_{2}+a_{3}+a_{4}$, let $p_{A}$ denote the number of pairs $(i, j)$ with $1 \leq ia_{1}+a_{3}$ and $a_{3}+a_{4}>a_{1}+a_{2}$. Hence $a_{2}+a_{4}$ and $a_{3}+a_{4}$ do not divide $s_{A}$. This proves $p_{A} \leq 4$. + +Now suppose $p_{A}=4$. By the previous argument we have + +$$ +\begin{array}{lll} +a_{1}+a_{4} \mid a_{2}+a_{3} & \text { and } & a_{2}+a_{3} \mid a_{1}+a_{4}, \\ +a_{1}+a_{2} \mid a_{3}+a_{4} & \text { and } & a_{3}+a_{4} \not a_{1}+a_{2}, \\ +a_{1}+a_{3} \mid a_{2}+a_{4} & \text { and } & a_{2}+a_{4} \not a_{1}+a_{3} . +\end{array} +$$ + +Hence, there exist positive integers $m$ and $n$ with $m>n \geq 2$ such that + +$$ +\left\{\begin{array}{l} +a_{1}+a_{4}=a_{2}+a_{3} \\ +m\left(a_{1}+a_{2}\right)=a_{3}+a_{4} \\ +n\left(a_{1}+a_{3}\right)=a_{2}+a_{4} +\end{array}\right. +$$ + +Adding up the first equation and the third one, we get $n\left(a_{1}+a_{3}\right)=2 a_{2}+a_{3}-a_{1}$. If $n \geq 3$, then $n\left(a_{1}+a_{3}\right)>3 a_{3}>2 a_{2}+a_{3}>2 a_{2}+a_{3}-a_{1}$. This is a contradiction. Therefore $n=2$. If we multiply by 2 the sum of the first equation and the third one, we obtain + +$$ +6 a_{1}+2 a_{3}=4 a_{2} +$$ + +while the sum of the first one and the second one is + +$$ +(m+1) a_{1}+(m-1) a_{2}=2 a_{3} . +$$ + +Adding up the last two equations we get + +$$ +(m+7) a_{1}=(5-m) a_{2} . +$$ + +It follows that $5-m \geq 1$, because the left-hand side of the last equation and $a_{2}$ are positive. Since we have $m>n=2$, the integer $m$ can be equal only to either 3 or 4 . Substituting $(3,2)$ and $(4,2)$ for $(m, n)$ and solving the previous system of equations, we find the families of solutions $\{d, 5 d, 7 d, 11 d\}$ and $\{d, 11 d, 19 d, 29 d\}$, where $d$ is any positive integer. + +## A 2 + +Determine all sequences $\left(x_{1}, x_{2}, \ldots, x_{2011}\right)$ of positive integers such that for every positive integer $n$ there is an integer $a$ with + +$$ +x_{1}^{n}+2 x_{2}^{n}+\cdots+2011 x_{2011}^{n}=a^{n+1}+1 . +$$ + +Answer. The only sequence that satisfies the condition is + +$$ +\left(x_{1}, \ldots, x_{2011}\right)=(1, k, \ldots, k) \quad \text { with } k=2+3+\cdots+2011=2023065 +$$ + +Solution. Throughout this solution, the set of positive integers will be denoted by $\mathbb{Z}_{+}$. + +Put $k=2+3+\cdots+2011=2023065$. We have + +$$ +1^{n}+2 k^{n}+\cdots 2011 k^{n}=1+k \cdot k^{n}=k^{n+1}+1 +$$ + +for all $n$, so $(1, k, \ldots, k)$ is a valid sequence. We shall prove that it is the only one. + +Let a valid sequence $\left(x_{1}, \ldots, x_{2011}\right)$ be given. For each $n \in \mathbb{Z}_{+}$we have some $y_{n} \in \mathbb{Z}_{+}$with + +$$ +x_{1}^{n}+2 x_{2}^{n}+\cdots+2011 x_{2011}^{n}=y_{n}^{n+1}+1 . +$$ + +Note that $x_{1}^{n}+2 x_{2}^{n}+\cdots+2011 x_{2011}^{n}<\left(x_{1}+2 x_{2}+\cdots+2011 x_{2011}\right)^{n+1}$, which implies that the sequence $\left(y_{n}\right)$ is bounded. In particular, there is some $y \in \mathbb{Z}_{+}$with $y_{n}=y$ for infinitely many $n$. + +Let $m$ be the maximum of all the $x_{i}$. Grouping terms with equal $x_{i}$ together, the sum $x_{1}^{n}+$ $2 x_{2}^{n}+\cdots+2011 x_{2011}^{n}$ can be written as + +$$ +x_{1}^{n}+2 x_{2}^{n}+\cdots+x_{2011}^{n}=a_{m} m^{n}+a_{m-1}(m-1)^{n}+\cdots+a_{1} +$$ + +with $a_{i} \geq 0$ for all $i$ and $a_{1}+\cdots+a_{m}=1+2+\cdots+2011$. So there exist arbitrarily large values of $n$, for which + +$$ +a_{m} m^{n}+\cdots+a_{1}-1-y \cdot y^{n}=0 \text {. } +$$ + +The following lemma will help us to determine the $a_{i}$ and $y$ : + +Lemma. Let integers $b_{1}, \ldots, b_{N}$ be given and assume that there are arbitrarily large positive integers $n$ with $b_{1}+b_{2} 2^{n}+\cdots+b_{N} N^{n}=0$. Then $b_{i}=0$ for all $i$. + +Proof. Suppose that not all $b_{i}$ are zero. We may assume without loss of generality that $b_{N} \neq 0$. + +Dividing through by $N^{n}$ gives + +$$ +\left|b_{N}\right|=\left|b_{N-1}\left(\frac{N-1}{N}\right)^{n}+\cdots+b_{1}\left(\frac{1}{N}\right)^{n}\right| \leq\left(\left|b_{N-1}\right|+\cdots+\left|b_{1}\right|\right)\left(\frac{N-1}{N}\right)^{n} . +$$ + +The expression $\left(\frac{N-1}{N}\right)^{n}$ can be made arbitrarily small for $n$ large enough, contradicting the assumption that $b_{N}$ be non-zero. + +We obviously have $y>1$. Applying the lemma to (1) we see that $a_{m}=y=m, a_{1}=1$, and all the other $a_{i}$ are zero. This implies $\left(x_{1}, \ldots, x_{2011}\right)=(1, m, \ldots, m)$. But we also have $1+m=a_{1}+\cdots+a_{m}=1+\cdots+2011=1+k$ so $m=k$, which is what we wanted to show. + +## A3 + +Determine all pairs $(f, g)$ of functions from the set of real numbers to itself that satisfy + +$$ +g(f(x+y))=f(x)+(2 x+y) g(y) +$$ + +for all real numbers $x$ and $y$. + +Answer. Either both $f$ and $g$ vanish identically, or there exists a real number $C$ such that $f(x)=x^{2}+C$ and $g(x)=x$ for all real numbers $x$. + +Solution. Clearly all these pairs of functions satisfy the functional equation in question, so it suffices to verify that there cannot be any further ones. Substituting $-2 x$ for $y$ in the given functional equation we obtain + +$$ +g(f(-x))=f(x) . +$$ + +Using this equation for $-x-y$ in place of $x$ we obtain + +$$ +f(-x-y)=g(f(x+y))=f(x)+(2 x+y) g(y) . +$$ + +Now for any two real numbers $a$ and $b$, setting $x=-b$ and $y=a+b$ we get + +$$ +f(-a)=f(-b)+(a-b) g(a+b) . +$$ + +If $c$ denotes another arbitrary real number we have similarly + +$$ +f(-b)=f(-c)+(b-c) g(b+c) +$$ + +as well as + +$$ +f(-c)=f(-a)+(c-a) g(c+a) . +$$ + +Adding all these equations up, we obtain + +$$ +((a+c)-(b+c)) g(a+b)+((a+b)-(a+c)) g(b+c)+((b+c)-(a+b)) g(a+c)=0 \text {. } +$$ + +Now given any three real numbers $x, y$, and $z$ one may determine three reals $a, b$, and $c$ such that $x=b+c, y=c+a$, and $z=a+b$, so that we get + +$$ +(y-x) g(z)+(z-y) g(x)+(x-z) g(y)=0 . +$$ + +This implies that the three points $(x, g(x)),(y, g(y))$, and $(z, g(z))$ from the graph of $g$ are collinear. Hence that graph is a line, i.e., $g$ is either a constant or a linear function. + +Let us write $g(x)=A x+B$, where $A$ and $B$ are two real numbers. Substituting $(0,-y)$ for $(x, y)$ in (2) and denoting $C=f(0)$, we have $f(y)=A y^{2}-B y+C$. Now, comparing the coefficients of $x^{2}$ in (1I) we see that $A^{2}=A$, so $A=0$ or $A=1$. + +If $A=0$, then (1) becomes $B=-B x+C$ and thus $B=C=0$, which provides the first of the two solutions mentioned above. + +Now suppose $A=1$. Then (1) becomes $x^{2}-B x+C+B=x^{2}-B x+C$, so $B=0$. Thus, $g(x)=x$ and $f(x)=x^{2}+C$, which is the second solution from above. + +Comment. Another way to show that $g(x)$ is either a constant or a linear function is the following. If we interchange $x$ and $y$ in the given functional equation and subtract this new equation from the given one, we obtain + +$$ +f(x)-f(y)=(2 y+x) g(x)-(2 x+y) g(y) . +$$ + +Substituting $(x, 0),(1, x)$, and $(0,1)$ for $(x, y)$, we get + +$$ +\begin{aligned} +& f(x)-f(0)=x g(x)-2 x g(0), \\ +& f(1)-f(x)=(2 x+1) g(1)-(x+2) g(x), \\ +& f(0)-f(1)=2 g(0)-g(1) . +\end{aligned} +$$ + +Taking the sum of these three equations and dividing by 2 , we obtain + +$$ +g(x)=x(g(1)-g(0))+g(0) . +$$ + +This proves that $g(x)$ is either a constant of a linear function. + +## A4 + +Determine all pairs $(f, g)$ of functions from the set of positive integers to itself that satisfy + +$$ +f^{g(n)+1}(n)+g^{f(n)}(n)=f(n+1)-g(n+1)+1 +$$ + +for every positive integer $n$. Here, $f^{k}(n)$ means $\underbrace{f(f(\ldots f}_{k}(n) \ldots))$. + +Answer. The only pair $(f, g)$ of functions that satisfies the equation is given by $f(n)=n$ and $g(n)=1$ for all $n$. + +Solution. The given relation implies + +$$ +f\left(f^{g(n)}(n)\right)1$, then (1) reads $f\left(f^{g(x-1)}(x-1)\right)n$. Substituting $x-1$ into (1) we have $f\left(f^{g(x-1)}(x-1)\right)n$. So $b=n$, and hence $y_{n}=n$, which proves (ii) ${ }_{n}$. Next, from (i) ${ }_{n}$ we now get $f(k)=n \Longleftrightarrow k=n$, so removing all the iterations of $f$ in (3) we obtain $x-1=b=n$, which proves (i) ${ }_{n+1}$. + +So, all the statements in (2) are valid and hence $f(n)=n$ for all $n$. The given relation between $f$ and $g$ now reads $n+g^{n}(n)=n+1-g(n+1)+1$ or $g^{n}(n)+g(n+1)=2$, from which it +immediately follows that we have $g(n)=1$ for all $n$. + +Comment. Several variations of the above solution are possible. For instance, one may first prove by induction that the smallest $n$ values of $f$ are exactly $f(1)<\cdotsn$, then $f(x)>x$ for all $x \geq n$. From this we conclude $f^{g(n)+1}(n)>f^{g(n)}(n)>\cdots>f(n)$. But we also have $f^{g(n)+1}(c-b)(c+b)>a^{2}$. + +Now we turn to the induction step. Let $n>1$ and put $t=\lfloor n / 2\rfloor\frac{n}{2} \cdot \frac{9 n}{2} \geq(n+1)^{2} \geq i^{2}$, + +so this triangle is obtuse. The proof is completed. + +## A6 + +Let $f$ be a function from the set of real numbers to itself that satisfies + +$$ +f(x+y) \leq y f(x)+f(f(x)) +$$ + +for all real numbers $x$ and $y$. Prove that $f(x)=0$ for all $x \leq 0$. + +Solution 1. Substituting $y=t-x$, we rewrite (II) as + +$$ +f(t) \leq t f(x)-x f(x)+f(f(x)) . +$$ + +Consider now some real numbers $a, b$ and use (2) with $t=f(a), x=b$ as well as with $t=f(b)$, $x=a$. We get + +$$ +\begin{aligned} +& f(f(a))-f(f(b)) \leq f(a) f(b)-b f(b), \\ +& f(f(b))-f(f(a)) \leq f(a) f(b)-a f(a) . +\end{aligned} +$$ + +Adding these two inequalities yields + +$$ +2 f(a) f(b) \geq a f(a)+b f(b) . +$$ + +Now, substitute $b=2 f(a)$ to obtain $2 f(a) f(b) \geq a f(a)+2 f(a) f(b)$, or $a f(a) \leq 0$. So, we get + +$$ +f(a) \geq 0 \text { for all } a<0 \text {. } +$$ + +Now suppose $f(x)>0$ for some real number $x$. From (21) we immediately get that for every $t<\frac{x f(x)-f(f(x))}{f(x)}$ we have $f(t)<0$. This contradicts (3); therefore + +$$ +f(x) \leq 0 \quad \text { for all real } x +$$ + +and by (3) again we get $f(x)=0$ for all $x<0$. + +We are left to find $f(0)$. Setting $t=x<0$ in (2) we get + +$$ +0 \leq 0-0+f(0), +$$ + +so $f(0) \geq 0$. Combining this with (4) we obtain $f(0)=0$. + +Solution 2. We will also use the condition of the problem in form (2)). For clarity we divide the argument into four steps. + +Step 1. We begin by proving that $f$ attains nonpositive values only. Assume that there exist some real number $z$ with $f(z)>0$. Substituting $x=z$ into (2) and setting $A=f(z)$, $B=-z f(z)-f(f(z))$ we get $f(t) \leq A t+B$ for all real $t$. Hence, if for any positive real number $t$ we substitute $x=-t, y=t$ into (11), we get + +$$ +\begin{aligned} +f(0) & \leq t f(-t)+f(f(-t)) \leq t(-A t+B)+A f(-t)+B \\ +& \leq-t(A t-B)+A(-A t+B)+B=-A t^{2}-\left(A^{2}-B\right) t+(A+1) B . +\end{aligned} +$$ + +But surely this cannot be true if we take $t$ to be large enough. This contradiction proves that we have indeed $f(x) \leq 0$ for all real numbers $x$. Note that for this reason (11) entails + +$$ +f(x+y) \leq y f(x) +$$ + +for all real numbers $x$ and $y$. + +Step 2. We proceed by proving that $f$ has at least one zero. If $f(0)=0$, we are done. Otherwise, in view of Step 1 we get $f(0)<0$. Observe that (5) tells us now $f(y) \leq y f(0)$ for all real numbers $y$. Thus we can specify a positive real number $a$ that is so large that $f(a)^{2}>-f(0)$. Put $b=f(a)$ and substitute $x=b$ and $y=-b$ into (5); we learn $-b^{2}0 \\ 0 & \text { if } x \leq 0\end{cases} +$$ + +automatically satisfies (11). Indeed, we have $f(x) \leq 0$ and hence also $f(f(x))=0$ for all real numbers $x$. So (11) reduces to (5); moreover, this inequality is nontrivial only if $x$ and $y$ are positive. In this last case it is provided by (6). + +Now it is not hard to come up with a nonzero function $g$ obeying (6). E.g. $g(z)=C e^{z}$ (where $C$ is a positive constant) fits since the inequality $e^{y}>y$ holds for all (positive) real numbers $y$. One may also consider the function $g(z)=e^{z}-1$; in this case, we even have that $f$ is continuous. + +## A7 + +Let $a, b$, and $c$ be positive real numbers satisfying $\min (a+b, b+c, c+a)>\sqrt{2}$ and $a^{2}+b^{2}+c^{2}=3$. Prove that + +$$ +\frac{a}{(b+c-a)^{2}}+\frac{b}{(c+a-b)^{2}}+\frac{c}{(a+b-c)^{2}} \geq \frac{3}{(a b c)^{2}} +$$ + +Throughout both solutions, we denote the sums of the form $f(a, b, c)+f(b, c, a)+f(c, a, b)$ by $\sum f(a, b, c)$. + +Solution 1. The condition $b+c>\sqrt{2}$ implies $b^{2}+c^{2}>1$, so $a^{2}=3-\left(b^{2}+c^{2}\right)<2$, i.e. $a<\sqrt{2}0$, and also $c+a-b>0$ and $a+b-c>0$ for similar reasons. + +We will use the variant of HÖLDER's inequality + +$$ +\frac{x_{1}^{p+1}}{y_{1}^{p}}+\frac{x_{1}^{p+1}}{y_{1}^{p}}+\ldots+\frac{x_{n}^{p+1}}{y_{n}^{p}} \geq \frac{\left(x_{1}+x_{2}+\ldots+x_{n}\right)^{p+1}}{\left(y_{1}+y_{2}+\ldots+y_{n}\right)^{p}} +$$ + +which holds for all positive real numbers $p, x_{1}, x_{2}, \ldots, x_{n}, y_{1}, y_{2}, \ldots, y_{n}$. Applying it to the left-hand side of (11) with $p=2$ and $n=3$, we get + +$$ +\sum \frac{a}{(b+c-a)^{2}}=\sum \frac{\left(a^{2}\right)^{3}}{a^{5}(b+c-a)^{2}} \geq \frac{\left(a^{2}+b^{2}+c^{2}\right)^{3}}{\left(\sum a^{5 / 2}(b+c-a)\right)^{2}}=\frac{27}{\left(\sum a^{5 / 2}(b+c-a)\right)^{2}} . +$$ + +To estimate the denominator of the right-hand part, we use an instance of SCHUR's inequality, namely + +$$ +\sum a^{3 / 2}(a-b)(a-c) \geq 0 +$$ + +which can be rewritten as + +$$ +\sum a^{5 / 2}(b+c-a) \leq a b c(\sqrt{a}+\sqrt{b}+\sqrt{c}) . +$$ + +Moreover, by the inequality between the arithmetic mean and the fourth power mean we also have + +$$ +\left(\frac{\sqrt{a}+\sqrt{b}+\sqrt{c}}{3}\right)^{4} \leq \frac{a^{2}+b^{2}+c^{2}}{3}=1 +$$ + +i.e., $\sqrt{a}+\sqrt{b}+\sqrt{c} \leq 3$. Hence, (2) yields + +$$ +\sum \frac{a}{(b+c-a)^{2}} \geq \frac{27}{(a b c(\sqrt{a}+\sqrt{b}+\sqrt{c}))^{2}} \geq \frac{3}{a^{2} b^{2} c^{2}} +$$ + +thus solving the problem. + +Comment. In this solution, one may also start from the following version of HÖLDER's inequality + +$$ +\left(\sum_{i=1}^{n} a_{i}^{3}\right)\left(\sum_{i=1}^{n} b_{i}^{3}\right)\left(\sum_{i=1}^{n} c_{i}^{3}\right) \geq\left(\sum_{i=1}^{n} a_{i} b_{i} c_{i}\right)^{3} +$$ + +applied as + +$$ +\sum \frac{a}{(b+c-a)^{2}} \cdot \sum a^{3}(b+c-a) \cdot \sum a^{2}(b+c-a) \geq 27 +$$ + +After doing that, one only needs the slightly better known instances + +$$ +\sum a^{3}(b+c-a) \leq(a+b+c) a b c \quad \text { and } \quad \sum a^{2}(b+c-a) \leq 3 a b c +$$ + +of Schur's Inequality. + +Solution 2. As in Solution 1, we mention that all the numbers $b+c-a, a+c-b, a+b-c$ are positive. We will use only this restriction and the condition + +$$ +a^{5}+b^{5}+c^{5} \geq 3 +$$ + +which is weaker than the given one. Due to the symmetry we may assume that $a \geq b \geq c$. + +In view of (3)), it suffices to prove the inequality + +$$ +\sum \frac{a^{3} b^{2} c^{2}}{(b+c-a)^{2}} \geq \sum a^{5} +$$ + +or, moving all the terms into the left-hand part, + +$$ +\sum \frac{a^{3}}{(b+c-a)^{2}}\left((b c)^{2}-(a(b+c-a))^{2}\right) \geq 0 +$$ + +Note that the signs of the expressions $(y z)^{2}-(x(y+z-x))^{2}$ and $y z-x(y+z-x)=(x-y)(x-z)$ are the same for every positive $x, y, z$ satisfying the triangle inequality. So the terms in (4) corresponding to $a$ and $c$ are nonnegative, and hence it is sufficient to prove that the sum of the terms corresponding to $a$ and $b$ is nonnegative. Equivalently, we need the relation + +$$ +\frac{a^{3}}{(b+c-a)^{2}}(a-b)(a-c)(b c+a(b+c-a)) \geq \frac{b^{3}}{(a+c-b)^{2}}(a-b)(b-c)(a c+b(a+c-b)) +$$ + +Obviously, we have + +$$ +a^{3} \geq b^{3} \geq 0, \quad 0(a-b)^{2} +$$ + +which holds since $c>a-b \geq 0$ and $a+b>a-b \geq 0$. + +## C1 + +Let $n>0$ be an integer. We are given a balance and $n$ weights of weight $2^{0}, 2^{1}, \ldots, 2^{n-1}$. In a sequence of $n$ moves we place all weights on the balance. In the first move we choose a weight and put it on the left pan. In each of the following moves we choose one of the remaining weights and we add it either to the left or to the right pan. Compute the number of ways in which we can perform these $n$ moves in such a way that the right pan is never heavier than the left pan. + +Answer. The number $f(n)$ of ways of placing the $n$ weights is equal to the product of all odd positive integers less than or equal to $2 n-1$, i.e. $f(n)=(2 n-1) ! !=1 \cdot 3 \cdot 5 \cdot \ldots \cdot(2 n-1)$. + +Solution 1. Assume $n \geq 2$. We claim + +$$ +f(n)=(2 n-1) f(n-1) . +$$ + +Firstly, note that after the first move the left pan is always at least 1 heavier than the right one. Hence, any valid way of placing the $n$ weights on the scale gives rise, by not considering weight 1 , to a valid way of placing the weights $2,2^{2}, \ldots, 2^{n-1}$. + +If we divide the weight of each weight by 2 , the answer does not change. So these $n-1$ weights can be placed on the scale in $f(n-1)$ valid ways. Now we look at weight 1 . If it is put on the scale in the first move, then it has to be placed on the left side, otherwise it can be placed either on the left or on the right side, because after the first move the difference between the weights on the left pan and the weights on the right pan is at least 2 . Hence, there are exactly $2 n-1$ different ways of inserting weight 1 in each of the $f(n-1)$ valid sequences for the $n-1$ weights in order to get a valid sequence for the $n$ weights. This proves the claim. + +Since $f(1)=1$, by induction we obtain for all positive integers $n$ + +$$ +f(n)=(2 n-1) ! !=1 \cdot 3 \cdot 5 \cdot \ldots \cdot(2 n-1) +$$ + +Comment 1. The word "compute" in the statement of the problem is probably too vague. An alternative but more artificial question might ask for the smallest $n$ for which the number of valid ways is divisible by 2011. In this case the answer would be 1006 . + +Comment 2. It is useful to remark that the answer is the same for any set of weights where each weight is heavier than the sum of the lighter ones. Indeed, in such cases the given condition is equivalent to asking that during the process the heaviest weight on the balance is always on the left pan. + +Comment 3. Instead of considering the lightest weight, one may also consider the last weight put on the balance. If this weight is $2^{n-1}$ then it should be put on the left pan. Otherwise it may be put on +any pan; the inequality would not be violated since at this moment the heaviest weight is already put onto the left pan. In view of the previous comment, in each of these $2 n-1$ cases the number of ways to place the previous weights is exactly $f(n-1)$, which yields (1). + +Solution 2. We present a different way of obtaining (1). Set $f(0)=1$. Firstly, we find a recurrent formula for $f(n)$. + +Assume $n \geq 1$. Suppose that weight $2^{n-1}$ is placed on the balance in the $i$-th move with $1 \leq i \leq n$. This weight has to be put on the left pan. For the previous moves we have $\left(\begin{array}{c}n-1 \\ i-1\end{array}\right)$ choices of the weights and from Comment 2 there are $f(i-1)$ valid ways of placing them on the balance. For later moves there is no restriction on the way in which the weights are to be put on the pans. Therefore, all $(n-i) ! 2^{n-i}$ ways are possible. This gives + +$$ +f(n)=\sum_{i=1}^{n}\left(\begin{array}{c} +n-1 \\ +i-1 +\end{array}\right) f(i-1)(n-i) ! 2^{n-i}=\sum_{i=1}^{n} \frac{(n-1) ! f(i-1) 2^{n-i}}{(i-1) !} +$$ + +Now we are ready to prove (11). Using $n-1$ instead of $n$ in (2) we get + +$$ +f(n-1)=\sum_{i=1}^{n-1} \frac{(n-2) ! f(i-1) 2^{n-1-i}}{(i-1) !} . +$$ + +Hence, again from (2) we get + +$$ +\begin{aligned} +f(n)=2(n-1) \sum_{i=1}^{n-1} & \frac{(n-2) ! f(i-1) 2^{n-1-i}}{(i-1) !}+f(n-1) \\ +& =(2 n-2) f(n-1)+f(n-1)=(2 n-1) f(n-1), +\end{aligned} +$$ + +QED. + +Comment. There exist different ways of obtaining the formula (2). Here we show one of them. + +Suppose that in the first move we use weight $2^{n-i+1}$. Then the lighter $n-i$ weights may be put on the balance at any moment and on either pan. This gives $2^{n-i} \cdot(n-1) ! /(i-1)$ ! choices for the moves (moments and choices of pan) with the lighter weights. The remaining $i-1$ moves give a valid sequence for the $i-1$ heavier weights and this is the only requirement for these moves, so there are $f(i-1)$ such sequences. Summing over all $i=1,2, \ldots, n$ we again come to (2). + +## C2 + +Suppose that 1000 students are standing in a circle. Prove that there exists an integer $k$ with $100 \leq k \leq 300$ such that in this circle there exists a contiguous group of $2 k$ students, for which the first half contains the same number of girls as the second half. + +Solution. Number the students consecutively from 1 to 1000. Let $a_{i}=1$ if the $i$ th student is a girl, and $a_{i}=0$ otherwise. We expand this notion for all integers $i$ by setting $a_{i+1000}=$ $a_{i-1000}=a_{i}$. Next, let + +$$ +S_{k}(i)=a_{i}+a_{i+1}+\cdots+a_{i+k-1} +$$ + +Now the statement of the problem can be reformulated as follows: + +There exist an integer $k$ with $100 \leq k \leq 300$ and an index $i$ such that $S_{k}(i)=S_{k}(i+k)$. + +Assume now that this statement is false. Choose an index $i$ such that $S_{100}(i)$ attains the maximal possible value. In particular, we have $S_{100}(i-100)-S_{100}(i)<0$ and $S_{100}(i)-S_{100}(i+100)>0$, for if we had an equality, then the statement would hold. This means that the function $S(j)-$ $S(j+100)$ changes sign somewhere on the segment $[i-100, i]$, so there exists some index $j \in$ $[i-100, i-1]$ such that + +$$ +S_{100}(j) \leq S_{100}(j+100)-1, \quad \text { but } \quad S_{100}(j+1) \geq S_{100}(j+101)+1 +$$ + +Subtracting the first inequality from the second one, we get $a_{j+100}-a_{j} \geq a_{j+200}-a_{j+100}+2$, so + +$$ +a_{j}=0, \quad a_{j+100}=1, \quad a_{j+200}=0 +$$ + +Substituting this into the inequalities of (1), we also obtain $S_{99}(j+1) \leq S_{99}(j+101) \leq S_{99}(j+1)$, which implies + +$$ +S_{99}(j+1)=S_{99}(j+101) . +$$ + +Now let $k$ and $\ell$ be the least positive integers such that $a_{j-k}=1$ and $a_{j+200+\ell}=1$. By symmetry, we may assume that $k \geq \ell$. If $k \geq 200$ then we have $a_{j}=a_{j-1}=\cdots=a_{j-199}=0$, so $S_{100}(j-199)=S_{100}(j-99)=0$, which contradicts the initial assumption. Hence $\ell \leq k \leq 199$. + +Finally, we have + +$$ +\begin{gathered} +S_{100+\ell}(j-\ell+1)=\left(a_{j-\ell+1}+\cdots+a_{j}\right)+S_{99}(j+1)+a_{j+100}=S_{99}(j+1)+1, \\ +S_{100+\ell}(j+101)=S_{99}(j+101)+\left(a_{j+200}+\cdots+a_{j+200+\ell-1}\right)+a_{j+200+\ell}=S_{99}(j+101)+1 . +\end{gathered} +$$ + +Comparing with (2) we get $S_{100+\ell}(j-\ell+1)=S_{100+\ell}(j+101)$ and $100+\ell \leq 299$, which again contradicts our assumption. + +Comment. It may be seen from the solution that the number 300 from the problem statement can be +replaced by 299. Here we consider some improvements of this result. Namely, we investigate which interval can be put instead of $[100,300]$ in order to keep the problem statement valid. + +First of all, the two examples + +![](https://cdn.mathpix.com/cropped/2024_04_17_481ddb3576adc2313a2eg-31.jpg?height=109&width=1054&top_left_y=454&top_left_x=501) + +and + +$$ +\underbrace{1,1, \ldots, 1}_{249}, \underbrace{0,0, \ldots, 0}_{251}, \underbrace{1,1, \ldots, 1}_{249}, \underbrace{0,0, \ldots, 0}_{251} +$$ + +show that the interval can be changed neither to $[84,248]$ nor to $[126,374]$. + +On the other hand, we claim that this interval can be changed to $[125,250]$. Note that this statement is invariant under replacing all 1's by 0's and vice versa. Assume, to the contrary, that there is no admissible $k \in[125,250]$. The arguments from the solution easily yield the following lemma. + +Lemma. Under our assumption, suppose that for some indices $i\left(i_{3}^{\prime}-i_{1}\right)+\left(i_{6}^{\prime}-i_{4}\right) \geq 500+500$. This final contradiction shows that our claim holds. + +One may even show that the interval in the statement of the problem may be replaced by $[125,249]$ (both these numbers cannot be improved due to the examples above). But a proof of this fact is a bit messy, and we do not present it here. + +## C3 + +Let $\mathcal{S}$ be a finite set of at least two points in the plane. Assume that no three points of $\mathcal{S}$ are collinear. By a windmill we mean a process as follows. Start with a line $\ell$ going through a point $P \in \mathcal{S}$. Rotate $\ell$ clockwise around the pivot $P$ until the line contains another point $Q$ of $\mathcal{S}$. The point $Q$ now takes over as the new pivot. This process continues indefinitely, with the pivot always being a point from $\mathcal{S}$. + +Show that for a suitable $P \in \mathcal{S}$ and a suitable starting line $\ell$ containing $P$, the resulting windmill will visit each point of $\mathcal{S}$ as a pivot infinitely often. + +Solution. Give the rotating line an orientation and distinguish its sides as the oranje side and the blue side. Notice that whenever the pivot changes from some point $T$ to another point $U$, after the change, $T$ is on the same side as $U$ was before. Therefore, the number of elements of $\mathcal{S}$ on the oranje side and the number of those on the blue side remain the same throughout the whole process (except for those moments when the line contains two points). +![](https://cdn.mathpix.com/cropped/2024_04_17_481ddb3576adc2313a2eg-32.jpg?height=422&width=1108&top_left_y=1198&top_left_x=474) + +First consider the case that $|\mathcal{S}|=2 n+1$ is odd. We claim that through any point $T \in \mathcal{S}$, there is a line that has $n$ points on each side. To see this, choose an oriented line through $T$ containing no other point of $\mathcal{S}$ and suppose that it has $n+r$ points on its oranje side. If $r=0$ then we have established the claim, so we may assume that $r \neq 0$. As the line rotates through $180^{\circ}$ around $T$, the number of points of $\mathcal{S}$ on its oranje side changes by 1 whenever the line passes through a point; after $180^{\circ}$, the number of points on the oranje side is $n-r$. Therefore there is an intermediate stage at which the oranje side, and thus also the blue side, contains $n$ points. + +Now select the point $P$ arbitrarily, and choose a line through $P$ that has $n$ points of $\mathcal{S}$ on each side to be the initial state of the windmill. We will show that during a rotation over $180^{\circ}$, the line of the windmill visits each point of $\mathcal{S}$ as a pivot. To see this, select any point $T$ of $\mathcal{S}$ and select a line $\ell$ through $T$ that separates $\mathcal{S}$ into equal halves. The point $T$ is the unique point of $\mathcal{S}$ through which a line in this direction can separate the points of $\mathcal{S}$ into equal halves (parallel translation would disturb the balance). Therefore, when the windmill line is parallel to $\ell$, it must be $\ell$ itself, and so pass through $T$. + +Next suppose that $|\mathcal{S}|=2 n$. Similarly to the odd case, for every $T \in \mathcal{S}$ there is an oriented +line through $T$ with $n-1$ points on its oranje side and $n$ points on its blue side. Select such an oriented line through an arbitrary $P$ to be the initial state of the windmill. + +We will now show that during a rotation over $360^{\circ}$, the line of the windmill visits each point of $\mathcal{S}$ as a pivot. To see this, select any point $T$ of $\mathcal{S}$ and an oriented line $\ell$ through $T$ that separates $\mathcal{S}$ into two subsets with $n-1$ points on its oranje and $n$ points on its blue side. Again, parallel translation would change the numbers of points on the two sides, so when the windmill line is parallel to $\ell$ with the same orientation, the windmill line must pass through $T$. + +Comment. One may shorten this solution in the following way. + +Suppose that $|\mathcal{S}|=2 n+1$. Consider any line $\ell$ that separates $\mathcal{S}$ into equal halves; this line is unique given its direction and contains some point $T \in \mathcal{S}$. Consider the windmill starting from this line. When the line has made a rotation of $180^{\circ}$, it returns to the same location but the oranje side becomes blue and vice versa. So, for each point there should have been a moment when it appeared as pivot, as this is the only way for a point to pass from on side to the other. + +Now suppose that $|\mathcal{S}|=2 n$. Consider a line having $n-1$ and $n$ points on the two sides; it contains some point $T$. Consider the windmill starting from this line. After having made a rotation of $180^{\circ}$, the windmill line contains some different point $R$, and each point different from $T$ and $R$ has changed the color of its side. So, the windmill should have passed through all the points. + +## $\mathrm{C} 4$ + +Determine the greatest positive integer $k$ that satisfies the following property: The set of positive integers can be partitioned into $k$ subsets $A_{1}, A_{2}, \ldots, A_{k}$ such that for all integers $n \geq 15$ and all $i \in\{1,2, \ldots, k\}$ there exist two distinct elements of $A_{i}$ whose sum is $n$. + +Answer. The greatest such number $k$ is 3 . + +Solution 1. There are various examples showing that $k=3$ does indeed have the property under consideration. E.g. one can take + +$$ +\begin{gathered} +A_{1}=\{1,2,3\} \cup\{3 m \mid m \geq 4\}, \\ +A_{2}=\{4,5,6\} \cup\{3 m-1 \mid m \geq 4\}, \\ +A_{3}=\{7,8,9\} \cup\{3 m-2 \mid m \geq 4\} +\end{gathered} +$$ + +To check that this partition fits, we notice first that the sums of two distinct elements of $A_{i}$ obviously represent all numbers $n \geq 1+12=13$ for $i=1$, all numbers $n \geq 4+11=15$ for $i=2$, and all numbers $n \geq 7+10=17$ for $i=3$. So, we are left to find representations of the numbers 15 and 16 as sums of two distinct elements of $A_{3}$. These are $15=7+8$ and $16=7+9$. + +Let us now suppose that for some $k \geq 4$ there exist sets $A_{1}, A_{2}, \ldots, A_{k}$ satisfying the given property. Obviously, the sets $A_{1}, A_{2}, A_{3}, A_{4} \cup \cdots \cup A_{k}$ also satisfy the same property, so one may assume $k=4$. + +Put $B_{i}=A_{i} \cap\{1,2, \ldots, 23\}$ for $i=1,2,3,4$. Now for any index $i$ each of the ten numbers $15,16, \ldots, 24$ can be written as sum of two distinct elements of $B_{i}$. Therefore this set needs to contain at least five elements. As we also have $\left|B_{1}\right|+\left|B_{2}\right|+\left|B_{3}\right|+\left|B_{4}\right|=23$, there has to be some index $j$ for which $\left|B_{j}\right|=5$. Let $B_{j}=\left\{x_{1}, x_{2}, x_{3}, x_{4}, x_{5}\right\}$. Finally, now the sums of two distinct elements of $A_{j}$ representing the numbers $15,16, \ldots, 24$ should be exactly all the pairwise sums of the elements of $B_{j}$. Calculating the sum of these numbers in two different ways, we reach + +$$ +4\left(x_{1}+x_{2}+x_{3}+x_{4}+x_{5}\right)=15+16+\ldots+24=195 . +$$ + +Thus the number 195 should be divisible by 4 , which is false. This contradiction completes our solution. + +Comment. There are several variation of the proof that $k$ should not exceed 3. E.g., one may consider the sets $C_{i}=A_{i} \cap\{1,2, \ldots, 19\}$ for $i=1,2,3,4$. As in the previous solution one can show that for some index $j$ one has $\left|C_{j}\right|=4$, and the six pairwise sums of the elements of $C_{j}$ should represent all numbers $15,16, \ldots, 20$. Let $C_{j}=\left\{y_{1}, y_{2}, y_{3}, y_{4}\right\}$ with $y_{1}1$. In the sequel, the word collision will be used to denote meeting of exactly two ants, moving in opposite directions. + +If at the beginning we place an ant on the southwest corner square facing east and an ant on the southeast corner square facing west, then they will meet in the middle of the bottom row at time $\frac{m-1}{2}$. After the collision, the ant that moves to the north will stay on the board for another $m-\frac{1}{2}$ time units and thus we have established an example in which the last ant falls off at time $\frac{m-1}{2}+m-\frac{1}{2}=\frac{3 m}{2}-1$. So, we are left to prove that this is the latest possible moment. + +Consider any collision of two ants $a$ and $a^{\prime}$. Let us change the rule for this collision, and enforce these two ants to turn anticlockwise. Then the succeeding behavior of all the ants does not change; the only difference is that $a$ and $a^{\prime}$ swap their positions. These arguments may be applied to any collision separately, so we may assume that at any collision, either both ants rotate clockwise or both of them rotate anticlockwise by our own choice. + +For instance, we may assume that there are only two types of ants, depending on their initial direction: $N E$-ants, which move only north or east, and $S W$-ants, moving only south and west. Then we immediately obtain that all ants will have fallen off the board after $2 m-1$ time units. However, we can get a better bound by considering the last moment at which a given ant collides with another ant. + +Choose a coordinate system such that the corners of the checkerboard are $(0,0),(m, 0),(m, m)$ and $(0, m)$. At time $t$, there will be no NE-ants in the region $\{(x, y): x+y2 m-t-1\}$. So if two ants collide at $(x, y)$ at time $t$, we have + +$$ +t+1 \leq x+y \leq 2 m-t-1 +$$ + +Analogously, we may change the rules so that each ant would move either alternatingly north and west, or alternatingly south and east. By doing so, we find that apart from (1) we also have $|x-y| \leq m-t-1$ for each collision at point $(x, y)$ and time $t$. + +To visualize this, put + +$$ +B(t)=\left\{(x, y) \in[0, m]^{2}: t+1 \leq x+y \leq 2 m-t-1 \text { and }|x-y| \leq m-t-1\right\} . +$$ + +An ant can thus only collide with another ant at time $t$ if it happens to be in the region $B(t)$. The following figure displays $B(t)$ for $t=\frac{1}{2}$ and $t=\frac{7}{2}$ in the case $m=6$ : + +![](https://cdn.mathpix.com/cropped/2024_04_17_481ddb3576adc2313a2eg-37.jpg?height=622&width=623&top_left_y=771&top_left_x=722) + +Now suppose that an NE-ant has its last collision at time $t$ and that it does so at the point $(x, y)$ (if the ant does not collide at all, it will fall off the board within $m-\frac{1}{2}<\frac{3 m}{2}-1$ time units, so this case can be ignored). Then we have $(x, y) \in B(t)$ and thus $x+y \geq t+1$ and $x-y \geq-(m-t-1)$. So we get + +$$ +x \geq \frac{(t+1)-(m-t-1)}{2}=t+1-\frac{m}{2} +$$ + +By symmetry we also have $y \geq t+1-\frac{m}{2}$, and hence $\min \{x, y\} \geq t+1-\frac{m}{2}$. After this collision, the ant will move directly to an edge, which will take at $\operatorname{most} m-\min \{x, y\}$ units of time. In sum, the total amount of time the ant stays on the board is at most + +$$ +t+(m-\min \{x, y\}) \leq t+m-\left(t+1-\frac{m}{2}\right)=\frac{3 m}{2}-1 +$$ + +By symmetry, the same bound holds for $\mathrm{SW}$-ants as well. + +## C6 + +Let $n$ be a positive integer and let $W=\ldots x_{-1} x_{0} x_{1} x_{2} \ldots$ be an infinite periodic word consisting of the letters $a$ and $b$. Suppose that the minimal period $N$ of $W$ is greater than $2^{n}$. + +A finite nonempty word $U$ is said to appear in $W$ if there exist indices $k \leq \ell$ such that $U=x_{k} x_{k+1} \ldots x_{\ell}$. A finite word $U$ is called ubiquitous if the four words $U a, U b, a U$, and $b U$ all appear in $W$. Prove that there are at least $n$ ubiquitous finite nonempty words. + +Solution. Throughout the solution, all the words are nonempty. For any word $R$ of length $m$, we call the number of indices $i \in\{1,2, \ldots, N\}$ for which $R$ coincides with the subword $x_{i+1} x_{i+2} \ldots x_{i+m}$ of $W$ the multiplicity of $R$ and denote it by $\mu(R)$. Thus a word $R$ appears in $W$ if and only if $\mu(R)>0$. Since each occurrence of a word in $W$ is both succeeded by either the letter $a$ or the letter $b$ and similarly preceded by one of those two letters, we have + +$$ +\mu(R)=\mu(R a)+\mu(R b)=\mu(a R)+\mu(b R) +$$ + +for all words $R$. + +We claim that the condition that $N$ is in fact the minimal period of $W$ guarantees that each word of length $N$ has multiplicity 1 or 0 depending on whether it appears or not. Indeed, if the words $x_{i+1} x_{i+2} \ldots x_{i+N}$ and $x_{j+1} \ldots x_{j+N}$ are equal for some $1 \leq i2^{n}$, at least one of the two words $a$ and $b$ has a multiplicity that is strictly larger than $2^{n-1}$. + +For each $k=0,1, \ldots, n-1$, let $U_{k}$ be a subword of $W$ whose multiplicity is strictly larger than $2^{k}$ and whose length is maximal subject to this property. Note that such a word exists in view of the two observations made in the two previous paragraphs. + +Fix some index $k \in\{0,1, \ldots, n-1\}$. Since the word $U_{k} b$ is longer than $U_{k}$, its multiplicity can be at most $2^{k}$, so in particular $\mu\left(U_{k} b\right)<\mu\left(U_{k}\right)$. Therefore, the word $U_{k} a$ has to appear by (11). For a similar reason, the words $U_{k} b, a U_{k}$, and $b U_{k}$ have to appear as well. Hence, the word $U_{k}$ is ubiquitous. Moreover, if the multiplicity of $U_{k}$ were strictly greater than $2^{k+1}$, then by (1) at least one of the two words $U_{k} a$ and $U_{k} b$ would have multiplicity greater than $2^{k}$ and would thus violate the maximality condition imposed on $U_{k}$. + +So we have $\mu\left(U_{0}\right) \leq 2<\mu\left(U_{1}\right) \leq 4<\ldots \leq 2^{n-1}<\mu\left(U_{n-1}\right)$, which implies in particular that the words $U_{0}, U_{1}, \ldots, U_{n-1}$ have to be distinct. As they have been proved to be ubiquitous as well, the problem is solved. + +Comment 1. There is an easy construction for obtaining ubiquitous words from appearing words whose multiplicity is at least two. Starting with any such word $U$ we may simply extend one of its occurrences in $W$ forwards and backwards as long as its multiplicity remains fixed, thus arriving at a +word that one might call the ubiquitous prolongation $p(U)$ of $U$. + +There are several variants of the argument in the second half of the solution using the concept of prolongation. For instance, one may just take all ubiquitous words $U_{1}, U_{2}, \ldots, U_{\ell}$ ordered by increasing multiplicity and then prove for $i \in\{1,2, \ldots, \ell\}$ that $\mu\left(U_{i}\right) \leq 2^{i}$. Indeed, assume that $i$ is a minimal counterexample to this statement; then by the arguments similar to those presented above, the ubiquitous prolongation of one of the words $U_{i} a, U_{i} b, a U_{i}$ or $b U_{i}$ violates the definition of $U_{i}$. + +Now the multiplicity of one of the two letters $a$ and $b$ is strictly greater than $2^{n-1}$, so passing to ubiquitous prolongations once more we obtain $2^{n-1}<\mu\left(U_{\ell}\right) \leq 2^{\ell}$, which entails $\ell \geq n$, as needed. + +Comment 2. The bound $n$ for the number of ubiquitous subwords in the problem statement is not optimal, but it is close to an optimal one in the following sense. There is a universal constant $C>0$ such that for each positive integer $n$ there exists an infinite periodic word $W$ whose minimal period is greater than $2^{n}$ but for which there exist fewer than $C n$ ubiquitous words. + +## C7 + +On a square table of 2011 by 2011 cells we place a finite number of napkins that each cover a square of 52 by 52 cells. In each cell we write the number of napkins covering it, and we record the maximal number $k$ of cells that all contain the same nonzero number. Considering all possible napkin configurations, what is the largest value of $k$ ? + +Answer. $2011^{2}-\left(\left(52^{2}-35^{2}\right) \cdot 39-17^{2}\right)=4044121-57392=3986729$. + +Solution 1. Let $m=39$, then $2011=52 m-17$. We begin with an example showing that there can exist 3986729 cells carrying the same positive number. + +![](https://cdn.mathpix.com/cropped/2024_04_17_481ddb3576adc2313a2eg-40.jpg?height=577&width=580&top_left_y=1022&top_left_x=744) + +To describe it, we number the columns from the left to the right and the rows from the bottom to the top by $1,2, \ldots, 2011$. We will denote each napkin by the coordinates of its lowerleft cell. There are four kinds of napkins: first, we take all napkins $(52 i+36,52 j+1)$ with $0 \leq j \leq i \leq m-2$; second, we use all napkins $(52 i+1,52 j+36)$ with $0 \leq i \leq j \leq m-2$; third, we use all napkins $(52 i+36,52 i+36)$ with $0 \leq i \leq m-2$; and finally the napkin $(1,1)$. Different groups of napkins are shown by different types of hatchings in the picture. + +Now except for those squares that carry two or more different hatchings, all squares have the number 1 written into them. The number of these exceptional cells is easily computed to be $\left(52^{2}-35^{2}\right) m-17^{2}=57392$. + +We are left to prove that 3986729 is an upper bound for the number of cells containing the same number. Consider any configuration of napkins and any positive integer $M$. Suppose there are $g$ cells with a number different from $M$. Then it suffices to show $g \geq 57392$. Throughout the solution, a line will mean either a row or a column. + +Consider any line $\ell$. Let $a_{1}, \ldots, a_{52 m-17}$ be the numbers written into its consecutive cells. For $i=1,2, \ldots, 52$, let $s_{i}=\sum_{t \equiv i(\bmod 52)} a_{t}$. Note that $s_{1}, \ldots, s_{35}$ have $m$ terms each, while $s_{36}, \ldots, s_{52}$ have $m-1$ terms each. Every napkin intersecting $\ell$ contributes exactly 1 to each $s_{i}$; +hence the number $s$ of all those napkins satisfies $s_{1}=\cdots=s_{52}=s$. Call the line $\ell$ rich if $s>(m-1) M$ and poor otherwise. + +Suppose now that $\ell$ is rich. Then in each of the sums $s_{36}, \ldots, s_{52}$ there exists a term greater than $M$; consider all these terms and call the corresponding cells the rich bad cells for this line. So, each rich line contains at least 17 cells that are bad for this line. + +If, on the other hand, $\ell$ is poor, then certainly $s2 H^{2}$, then the answer and the example are the same as in the previous case; otherwise the answer is $(2 m-1) S^{2}+2 S H(m-1)$, and the example is provided simply by $(m-1)^{2}$ nonintersecting napkins. + +Now we sketch the proof of both estimates for Case 2. We introduce a more appropriate notation based on that from Solution 2. Denote by $a_{-}$and $a_{+}$the number of cells of class $A$ that contain the number which is strictly less than $M$ and strictly greater than $M$, respectively. The numbers $b_{ \pm}, c_{ \pm}$, and $d_{ \pm}$are defined in a similar way. One may notice that the proofs of Claim 1 and Claims 2, 3 lead in fact to the inequalities + +$$ +m-1 \leq \frac{b_{-}+c_{-}}{2 S H}+\frac{d_{+}}{H^{2}} \quad \text { and } \quad 2 m-1 \leq \frac{a}{S^{2}}+\frac{b_{+}+c_{+}}{2 S H}+\frac{d_{+}}{H^{2}} +$$ + +(to obtain the first one, one needs to look at the big lines instead of the small ones). Combining these inequalities, one may obtain the desired estimates. + +These estimates can also be proved in some different ways, e.g. without distinguishing rich and poor cells. + +## G1 + +Let $A B C$ be an acute triangle. Let $\omega$ be a circle whose center $L$ lies on the side $B C$. Suppose that $\omega$ is tangent to $A B$ at $B^{\prime}$ and to $A C$ at $C^{\prime}$. Suppose also that the circumcenter $O$ of the triangle $A B C$ lies on the shorter arc $B^{\prime} C^{\prime}$ of $\omega$. Prove that the circumcircle of $A B C$ and $\omega$ meet at two points. + +Solution. The point $B^{\prime}$, being the perpendicular foot of $L$, is an interior point of side $A B$. Analogously, $C^{\prime}$ lies in the interior of $A C$. The point $O$ is located inside the triangle $A B^{\prime} C^{\prime}$, hence $\angle C O B<\angle C^{\prime} O B^{\prime}$. + +![](https://cdn.mathpix.com/cropped/2024_04_17_481ddb3576adc2313a2eg-45.jpg?height=837&width=674&top_left_y=895&top_left_x=697) + +Let $\alpha=\angle C A B$. The angles $\angle C A B$ and $\angle C^{\prime} O B^{\prime}$ are inscribed into the two circles with centers $O$ and $L$, respectively, so $\angle C O B=2 \angle C A B=2 \alpha$ and $2 \angle C^{\prime} O B^{\prime}=360^{\circ}-\angle C^{\prime} L B^{\prime}$. From the kite $A B^{\prime} L C^{\prime}$ we have $\angle C^{\prime} L B^{\prime}=180^{\circ}-\angle C^{\prime} A B^{\prime}=180^{\circ}-\alpha$. Combining these, we get + +$$ +2 \alpha=\angle C O B<\angle C^{\prime} O B^{\prime}=\frac{360^{\circ}-\angle C^{\prime} L B^{\prime}}{2}=\frac{360^{\circ}-\left(180^{\circ}-\alpha\right)}{2}=90^{\circ}+\frac{\alpha}{2}, +$$ + +So + +$$ +\alpha<60^{\circ} \text {. } +$$ + +Let $O^{\prime}$ be the reflection of $O$ in the line $B C$. In the quadrilateral $A B O^{\prime} C$ we have + +$$ +\angle C O^{\prime} B+\angle C A B=\angle C O B+\angle C A B=2 \alpha+\alpha<180^{\circ}, +$$ + +so the point $O^{\prime}$ is outside the circle $A B C$. Hence, $O$ and $O^{\prime}$ are two points of $\omega$ such that one of them lies inside the circumcircle, while the other one is located outside. Therefore, the two circles intersect. + +Comment. There are different ways of reducing the statement of the problem to the case $\alpha<60^{\circ}$. E.g., since the point $O$ lies in the interior of the isosceles triangle $A B^{\prime} C^{\prime}$, we have $O A
2 L B^{\prime}$, and this condition implies $\angle C A B=2 \angle B^{\prime} A L<2 \cdot 30^{\circ}=60^{\circ}$. + +## G2 + +Let $A_{1} A_{2} A_{3} A_{4}$ be a non-cyclic quadrilateral. Let $O_{1}$ and $r_{1}$ be the circumcenter and the circumradius of the triangle $A_{2} A_{3} A_{4}$. Define $O_{2}, O_{3}, O_{4}$ and $r_{2}, r_{3}, r_{4}$ in a similar way. Prove that + +$$ +\frac{1}{O_{1} A_{1}^{2}-r_{1}^{2}}+\frac{1}{O_{2} A_{2}^{2}-r_{2}^{2}}+\frac{1}{O_{3} A_{3}^{2}-r_{3}^{2}}+\frac{1}{O_{4} A_{4}^{2}-r_{4}^{2}}=0 +$$ + +Solution 1. Let $M$ be the point of intersection of the diagonals $A_{1} A_{3}$ and $A_{2} A_{4}$. On each diagonal choose a direction and let $x, y, z$, and $w$ be the signed distances from $M$ to the points $A_{1}, A_{2}, A_{3}$, and $A_{4}$, respectively. + +Let $\omega_{1}$ be the circumcircle of the triangle $A_{2} A_{3} A_{4}$ and let $B_{1}$ be the second intersection point of $\omega_{1}$ and $A_{1} A_{3}$ (thus, $B_{1}=A_{3}$ if and only if $A_{1} A_{3}$ is tangent to $\omega_{1}$ ). Since the expression $O_{1} A_{1}^{2}-r_{1}^{2}$ is the power of the point $A_{1}$ with respect to $\omega_{1}$, we get + +$$ +O_{1} A_{1}^{2}-r_{1}^{2}=A_{1} B_{1} \cdot A_{1} A_{3} +$$ + +On the other hand, from the equality $M B_{1} \cdot M A_{3}=M A_{2} \cdot M A_{4}$ we obtain $M B_{1}=y w / z$. Hence, we have + +$$ +O_{1} A_{1}^{2}-r_{1}^{2}=\left(\frac{y w}{z}-x\right)(z-x)=\frac{z-x}{z}(y w-x z) . +$$ + +Substituting the analogous expressions into the sought sum we get + +$$ +\sum_{i=1}^{4} \frac{1}{O_{i} A_{i}^{2}-r_{i}^{2}}=\frac{1}{y w-x z}\left(\frac{z}{z-x}-\frac{w}{w-y}+\frac{x}{x-z}-\frac{y}{y-w}\right)=0 +$$ + +as desired. + +Comment. One might reformulate the problem by assuming that the quadrilateral $A_{1} A_{2} A_{3} A_{4}$ is convex. This should not really change the difficulty, but proofs that distinguish several cases may become shorter. + +Solution 2. Introduce a Cartesian coordinate system in the plane. Every circle has an equation of the form $p(x, y)=x^{2}+y^{2}+l(x, y)=0$, where $l(x, y)$ is a polynomial of degree at most 1 . For any point $A=\left(x_{A}, y_{A}\right)$ we have $p\left(x_{A}, y_{A}\right)=d^{2}-r^{2}$, where $d$ is the distance from $A$ to the center of the circle and $r$ is the radius of the circle. + +For each $i$ in $\{1,2,3,4\}$ let $p_{i}(x, y)=x^{2}+y^{2}+l_{i}(x, y)=0$ be the equation of the circle with center $O_{i}$ and radius $r_{i}$ and let $d_{i}$ be the distance from $A_{i}$ to $O_{i}$. Consider the equation + +$$ +\sum_{i=1}^{4} \frac{p_{i}(x, y)}{d_{i}^{2}-r_{i}^{2}}=1 +$$ + +Since the coordinates of the points $A_{1}, A_{2}, A_{3}$, and $A_{4}$ satisfy (1) but these four points do not lie on a circle or on an line, equation (1) defines neither a circle, nor a line. Hence, the equation is an identity and the coefficient of the quadratic term $x^{2}+y^{2}$ also has to be zero, i.e. + +$$ +\sum_{i=1}^{4} \frac{1}{d_{i}^{2}-r_{i}^{2}}=0 +$$ + +Comment. Using the determinant form of the equation of the circle through three given points, the same solution can be formulated as follows. + +For $i=1,2,3,4$ let $\left(u_{i}, v_{i}\right)$ be the coordinates of $A_{i}$ and define + +$$ +\Delta=\left|\begin{array}{llll} +u_{1}^{2}+v_{1}^{2} & u_{1} & v_{1} & 1 \\ +u_{2}^{2}+v_{2}^{2} & u_{2} & v_{2} & 1 \\ +u_{3}^{2}+v_{3}^{2} & u_{3} & v_{3} & 1 \\ +u_{4}^{2}+v_{4}^{2} & u_{4} & v_{4} & 1 +\end{array}\right| \quad \text { and } \quad \Delta_{i}=\left|\begin{array}{lll} +u_{i+1} & v_{i+1} & 1 \\ +u_{i+2} & v_{i+2} & 1 \\ +u_{i+3} & v_{i+3} & 1 +\end{array}\right| +$$ + +where $i+1, i+2$, and $i+3$ have to be read modulo 4 as integers in the set $\{1,2,3,4\}$. + +Expanding $\left|\begin{array}{llll}u_{1} & v_{1} & 1 & 1 \\ u_{2} & v_{2} & 1 & 1 \\ u_{3} & v_{3} & 1 & 1 \\ u_{4} & v_{4} & 1 & 1\end{array}\right|=0$ along the third column, we get $\Delta_{1}-\Delta_{2}+\Delta_{3}-\Delta_{4}=0$. + +The circle through $A_{i+1}, A_{i+2}$, and $A_{i+3}$ is given by the equation + +$$ +\frac{1}{\Delta_{i}}\left|\begin{array}{cccc} +x^{2}+y^{2} & x & y & 1 \\ +u_{i+1}^{2}+v_{i+1}^{2} & u_{i+1} & v_{i+1} & 1 \\ +u_{i+2}^{2}+v_{i+2}^{2} & u_{i+2} & v_{i+2} & 1 \\ +u_{i+3}^{2}+v_{i+3}^{2} & u_{i+3} & v_{i+3} & 1 +\end{array}\right|=0 +$$ + +On the left-hand side, the coefficient of $x^{2}+y^{2}$ is equal to 1 . Substituting $\left(u_{i}, v_{i}\right)$ for $(x, y)$ in (2) we obtain the power of point $A_{i}$ with respect to the circle through $A_{i+1}, A_{i+2}$, and $A_{i+3}$ : + +$$ +d_{i}^{2}-r_{i}^{2}=\frac{1}{\Delta_{i}}\left|\begin{array}{cccc} +u_{i}^{2}+v_{i}^{2} & u_{i} & v_{i} & 1 \\ +u_{i+1}^{2}+v_{i+1}^{2} & u_{i+1} & v_{i+1} & 1 \\ +u_{i+2}^{2}+v_{i+2}^{2} & u_{i+2} & v_{i+2} & 1 \\ +u_{i+3}^{2}+v_{i+3}^{2} & u_{i+3} & v_{i+3} & 1 +\end{array}\right|=(-1)^{i+1} \frac{\Delta}{\Delta_{i}} +$$ + +Thus, we have + +$$ +\sum_{i=1}^{4} \frac{1}{d_{i}^{2}-r_{i}^{2}}=\frac{\Delta_{1}-\Delta_{2}+\Delta_{3}-\Delta_{4}}{\Delta}=0 +$$ + +## G3 + +Let $A B C D$ be a convex quadrilateral whose sides $A D$ and $B C$ are not parallel. Suppose that the circles with diameters $A B$ and $C D$ meet at points $E$ and $F$ inside the quadrilateral. Let $\omega_{E}$ be the circle through the feet of the perpendiculars from $E$ to the lines $A B, B C$, and $C D$. Let $\omega_{F}$ be the circle through the feet of the perpendiculars from $F$ to the lines $C D, D A$, and $A B$. Prove that the midpoint of the segment $E F$ lies on the line through the two intersection points of $\omega_{E}$ and $\omega_{F}$. + +Solution. Denote by $P, Q, R$, and $S$ the projections of $E$ on the lines $D A, A B, B C$, and $C D$ respectively. The points $P$ and $Q$ lie on the circle with diameter $A E$, so $\angle Q P E=\angle Q A E$; analogously, $\angle Q R E=\angle Q B E$. So $\angle Q P E+\angle Q R E=\angle Q A E+\angle Q B E=90^{\circ}$. By similar reasons, we have $\angle S P E+\angle S R E=90^{\circ}$, hence we get $\angle Q P S+\angle Q R S=90^{\circ}+90^{\circ}=180^{\circ}$, and the quadrilateral $P Q R S$ is inscribed in $\omega_{E}$. Analogously, all four projections of $F$ onto the sides of $A B C D$ lie on $\omega_{F}$. + +Denote by $K$ the meeting point of the lines $A D$ and $B C$. Due to the arguments above, there is no loss of generality in assuming that $A$ lies on segment $D K$. Suppose that $\angle C K D \geq 90^{\circ}$; then the circle with diameter $C D$ covers the whole quadrilateral $A B C D$, so the points $E, F$ cannot lie inside this quadrilateral. Hence our assumption is wrong. Therefore, the lines EP and $B C$ intersect at some point $P^{\prime}$, while the lines $E R$ and $A D$ intersect at some point $R^{\prime}$. + +![](https://cdn.mathpix.com/cropped/2024_04_17_481ddb3576adc2313a2eg-49.jpg?height=794&width=1376&top_left_y=1522&top_left_x=340) + +Figure 1 + +We claim that the points $P^{\prime}$ and $R^{\prime}$ also belong to $\omega_{E}$. Since the points $R, E, Q, B$ are concyclic, $\angle Q R K=\angle Q E B=90^{\circ}-\angle Q B E=\angle Q A E=\angle Q P E$. So $\angle Q R K=\angle Q P P^{\prime}$, which means that the point $P^{\prime}$ lies on $\omega_{E}$. Analogously, $R^{\prime}$ also lies on $\omega_{E}$. + +In the same manner, denote by $M$ and $N$ the projections of $F$ on the lines $A D$ and $B C$ +respectively, and let $M^{\prime}=F M \cap B C, N^{\prime}=F N \cap A D$. By the same arguments, we obtain that the points $M^{\prime}$ and $N^{\prime}$ belong to $\omega_{F}$. + +![](https://cdn.mathpix.com/cropped/2024_04_17_481ddb3576adc2313a2eg-50.jpg?height=782&width=1291&top_left_y=340&top_left_x=385) + +Figure 2 + +Now we concentrate on Figure 2, where all unnecessary details are removed. Let $U=N N^{\prime} \cap$ $P P^{\prime}, V=M M^{\prime} \cap R R^{\prime}$. Due to the right angles at $N$ and $P$, the points $N, N^{\prime}, P, P^{\prime}$ are concyclic, so $U N \cdot U N^{\prime}=U P \cdot U P^{\prime}$ which means that $U$ belongs to the radical axis $g$ of the circles $\omega_{E}$ and $\omega_{F}$. Analogously, $V$ also belongs to $g$. + +Finally, since $E U F V$ is a parallelogram, the radical axis $U V$ of $\omega_{E}$ and $\omega_{F}$ bisects $E F$. + +## G4 + +Let $A B C$ be an acute triangle with circumcircle $\Omega$. Let $B_{0}$ be the midpoint of $A C$ and let $C_{0}$ be the midpoint of $A B$. Let $D$ be the foot of the altitude from $A$, and let $G$ be the centroid of the triangle $A B C$. Let $\omega$ be a circle through $B_{0}$ and $C_{0}$ that is tangent to the circle $\Omega$ at a point $X \neq A$. Prove that the points $D, G$, and $X$ are collinear. + +Solution 1. If $A B=A C$, then the statement is trivial. So without loss of generality we may assume $A B0$, let $f(d)$ be the smallest positive integer that has exactly $d$ positive divisors (so for example we have $f(1)=1, f(5)=16$, and $f(6)=12$ ). Prove that for every integer $k \geq 0$ the number $f\left(2^{k}\right)$ divides $f\left(2^{k+1}\right)$. + +Solution 1. For any positive integer $n$, let $d(n)$ be the number of positive divisors of $n$. Let $n=\prod_{p} p^{a(p)}$ be the prime factorization of $n$ where $p$ ranges over the prime numbers, the integers $a(p)$ are nonnegative and all but finitely many $a(p)$ are zero. Then we have $d(n)=\prod_{p}(a(p)+1)$. Thus, $d(n)$ is a power of 2 if and only if for every prime $p$ there is a nonnegative integer $b(p)$ with $a(p)=2^{b(p)}-1=1+2+2^{2}+\cdots+2^{b(p)-1}$. We then have + +$$ +n=\prod_{p} \prod_{i=0}^{b(p)-1} p^{2^{i}}, \quad \text { and } \quad d(n)=2^{k} \quad \text { with } \quad k=\sum_{p} b(p) +$$ + +Let $\mathcal{S}$ be the set of all numbers of the form $p^{2^{r}}$ with $p$ prime and $r$ a nonnegative integer. Then we deduce that $d(n)$ is a power of 2 if and only if $n$ is the product of the elements of some finite subset $\mathcal{T}$ of $\mathcal{S}$ that satisfies the following condition: for all $t \in \mathcal{T}$ and $s \in \mathcal{S}$ with $s \mid t$ we have $s \in \mathcal{T}$. Moreover, if $d(n)=2^{k}$ then the corresponding set $\mathcal{T}$ has $k$ elements. + +Note that the set $\mathcal{T}_{k}$ consisting of the smallest $k$ elements from $\mathcal{S}$ obviously satisfies the condition above. Thus, given $k$, the smallest $n$ with $d(n)=2^{k}$ is the product of the elements of $\mathcal{T}_{k}$. This $n$ is $f\left(2^{k}\right)$. Since obviously $\mathcal{T}_{k} \subset \mathcal{T}_{k+1}$, it follows that $f\left(2^{k}\right) \mid f\left(2^{k+1}\right)$. + +Solution 2. This is an alternative to the second part of the Solution 1. Suppose $k$ is a nonnegative integer. From the first part of Solution 1 we see that $f\left(2^{k}\right)=\prod_{p} p^{a(p)}$ with $a(p)=2^{b(p)}-1$ and $\sum_{p} b(p)=k$. We now claim that for any two distinct primes $p, q$ with $b(q)>0$ we have + +$$ +m=p^{2^{b(p)}}>q^{2^{b(q)-1}}=\ell . +$$ + +To see this, note first that $\ell$ divides $f\left(2^{k}\right)$. With the first part of Solution 1 one can see that the integer $n=f\left(2^{k}\right) m / \ell$ also satisfies $d(n)=2^{k}$. By the definition of $f\left(2^{k}\right)$ this implies that $n \geq f\left(2^{k}\right)$ so $m \geq \ell$. Since $p \neq q$ the inequality (1) follows. + +Let the prime factorization of $f\left(2^{k+1}\right)$ be given by $f\left(2^{k+1}\right)=\prod_{p} p^{r(p)}$ with $r(p)=2^{s(p)}-1$. Since we have $\sum_{p} s(p)=k+1>k=\sum_{p} b(p)$ there is a prime $p$ with $s(p)>b(p)$. For any prime $q \neq p$ with $b(q)>0$ we apply inequality (1) twice and get + +$$ +q^{2^{s(q)}}>p^{2^{s(p)-1}} \geq p^{2^{b(p)}}>q^{2^{b(q)-1}} +$$ + +which implies $s(q) \geq b(q)$. It follows that $s(q) \geq b(q)$ for all primes $q$, so $f\left(2^{k}\right) \mid f\left(2^{k+1}\right)$. + +N2 + +Consider a polynomial $P(x)=\left(x+d_{1}\right)\left(x+d_{2}\right) \cdot \ldots \cdot\left(x+d_{9}\right)$, where $d_{1}, d_{2}, \ldots, d_{9}$ are nine distinct integers. Prove that there exists an integer $N$ such that for all integers $x \geq N$ the number $P(x)$ is divisible by a prime number greater than 20 . + +Solution 1. Note that the statement of the problem is invariant under translations of $x$; hence without loss of generality we may suppose that the numbers $d_{1}, d_{2}, \ldots, d_{9}$ are positive. + +The key observation is that there are only eight primes below 20, while $P(x)$ involves more than eight factors. + +We shall prove that $N=d^{8}$ satisfies the desired property, where $d=\max \left\{d_{1}, d_{2}, \ldots, d_{9}\right\}$. Suppose for the sake of contradiction that there is some integer $x \geq N$ such that $P(x)$ is composed of primes below 20 only. Then for every index $i \in\{1,2, \ldots, 9\}$ the number $x+d_{i}$ can be expressed as product of powers of the first 8 primes. + +Since $x+d_{i}>x \geq d^{8}$ there is some prime power $f_{i}>d$ that divides $x+d_{i}$. Invoking the pigeonhole principle we see that there are two distinct indices $i$ and $j$ such that $f_{i}$ and $f_{j}$ are powers of the same prime number. For reasons of symmetry, we may suppose that $f_{i} \leq f_{j}$. Now both of the numbers $x+d_{i}$ and $x+d_{j}$ are divisible by $f_{i}$ and hence so is their difference $d_{i}-d_{j}$. But as + +$$ +0<\left|d_{i}-d_{j}\right| \leq \max \left(d_{i}, d_{j}\right) \leq dD_{i}$ the numerator of the fraction we thereby get cannot be 1 , and hence it has to be divisible by some prime number $p_{i}<20$. + +By the pigeonhole principle, there are a prime number $p$ and two distinct indices $i$ and $j$ such that $p_{i}=p_{j}=p$. Let $p^{\alpha_{i}}$ and $p^{\alpha_{j}}$ be the greatest powers of $p$ dividing $x+d_{i}$ and $x+d_{j}$, respectively. Due to symmetry we may suppose $\alpha_{i} \leq \alpha_{j}$. But now $p^{\alpha_{i}}$ divides $d_{i}-d_{j}$ and hence also $D_{i}$, which means that all occurrences of $p$ in the numerator of the fraction $\left(x+d_{i}\right) / D_{i}$ cancel out, contrary to the choice of $p=p_{i}$. This contradiction proves our claim. + +Solution 3. Given a nonzero integer $N$ as well as a prime number $p$ we write $v_{p}(N)$ for the exponent with which $p$ occurs in the prime factorization of $|N|$. + +Evidently, if the statement of the problem were not true, then there would exist an infinite sequence $\left(x_{n}\right)$ of positive integers tending to infinity such that for each $n \in \mathbb{Z}_{+}$the integer $P\left(x_{n}\right)$ is not divisible by any prime number $>20$. Observe that the numbers $-d_{1},-d_{2}, \ldots,-d_{9}$ do not appear in this sequence. + +Now clearly there exists a prime $p_{1}<20$ for which the sequence $v_{p_{1}}\left(x_{n}+d_{1}\right)$ is not bounded; thinning out the sequence $\left(x_{n}\right)$ if necessary we may even suppose that + +$$ +v_{p_{1}}\left(x_{n}+d_{1}\right) \longrightarrow \infty . +$$ + +Repeating this argument eight more times we may similarly choose primes $p_{2}, \ldots, p_{9}<20$ and suppose that our sequence $\left(x_{n}\right)$ has been thinned out to such an extent that $v_{p_{i}}\left(x_{n}+d_{i}\right) \longrightarrow \infty$ holds for $i=2, \ldots, 9$ as well. In view of the pigeonhole principle, there are distinct indices $i$ and $j$ as well as a prime $p<20$ such that $p_{i}=p_{j}=p$. Setting $k=v_{p}\left(d_{i}-d_{j}\right)$ there now has to be some $n$ for which both $v_{p}\left(x_{n}+d_{i}\right)$ and $v_{p}\left(x_{n}+d_{j}\right)$ are greater than $k$. But now the numbers $x_{n}+d_{i}$ and $x_{n}+d_{j}$ are divisible by $p^{k+1}$ whilst their difference $d_{i}-d_{j}$ is not - a contradiction. + +Comment. This problem is supposed to be a relatively easy one, so one might consider adding the hypothesis that the numbers $d_{1}, d_{2}, \ldots, d_{9}$ be positive. Then certain merely technical issues are not going to arise while the main ideas required to solve the problems remain the same. + +Let $n \geq 1$ be an odd integer. Determine all functions $f$ from the set of integers to itself such that for all integers $x$ and $y$ the difference $f(x)-f(y)$ divides $x^{n}-y^{n}$. + +Answer. All functions $f$ of the form $f(x)=\varepsilon x^{d}+c$, where $\varepsilon$ is in $\{1,-1\}$, the integer $d$ is a positive divisor of $n$, and $c$ is an integer. + +Solution. Obviously, all functions in the answer satisfy the condition of the problem. We will show that there are no other functions satisfying that condition. + +Let $f$ be a function satisfying the given condition. For each integer $n$, the function $g$ defined by $g(x)=f(x)+n$ also satisfies the same condition. Therefore, by subtracting $f(0)$ from $f(x)$ we may assume that $f(0)=0$. + +For any prime $p$, the condition on $f$ with $(x, y)=(p, 0)$ states that $f(p)$ divides $p^{n}$. Since the set of primes is infinite, there exist integers $d$ and $\varepsilon$ with $0 \leq d \leq n$ and $\varepsilon \in\{1,-1\}$ such that for infinitely many primes $p$ we have $f(p)=\varepsilon p^{d}$. Denote the set of these primes by $P$. Since a function $g$ satisfies the given condition if and only if $-g$ satisfies the same condition, we may suppose $\varepsilon=1$. + +The case $d=0$ is easily ruled out, because 0 does not divide any nonzero integer. Suppose $d \geq 1$ and write $n$ as $m d+r$, where $m$ and $r$ are integers such that $m \geq 1$ and $0 \leq r \leq d-1$. Let $x$ be an arbitrary integer. For each prime $p$ in $P$, the difference $f(p)-f(x)$ divides $p^{n}-x^{n}$. Using the equality $f(p)=p^{d}$, we get + +$$ +p^{n}-x^{n}=p^{r}\left(p^{d}\right)^{m}-x^{n} \equiv p^{r} f(x)^{m}-x^{n} \equiv 0 \quad\left(\bmod p^{d}-f(x)\right) +$$ + +Since we have $r8$. For each positive integer $n$, there exists an $i \in\{0,1, \ldots, a-5\}$ such that $n+i=2 d$ for some odd $d$. We get + +$$ +t(n+i)=d \not \equiv d+2=t(n+i+4) \quad(\bmod 4) +$$ + +and + +$$ +t(n+a+i)=n+a+i \equiv n+a+i+4=t(n+a+i+4) \quad(\bmod 4) +$$ + +Therefore, the integers $t(n+a+i)-t(n+i)$ and $t(n+a+i+4)-t(n+i+4)$ cannot be both divisible by 4 , and therefore there are no winning pairs in this case. + +Case 3: $a=7$. For each positive integer $n$, there exists an $i \in\{0,1, \ldots, 6\}$ such that $n+i$ is either of the form $8 k+3$ or of the form $8 k+6$, where $k$ is a nonnegative integer. But we have + +$$ +t(8 k+3) \equiv 3 \not \equiv 1 \equiv 4 k+5=t(8 k+3+7) \quad(\bmod 4) +$$ + +and + +$$ +t(8 k+6)=4 k+3 \equiv 3 \not \equiv 1 \equiv t(8 k+6+7) \quad(\bmod 4) . +$$ + +Hence, there are no winning pairs of the form $(7, n)$. + +## N5 + +Let $f$ be a function from the set of integers to the set of positive integers. Suppose that for any two integers $m$ and $n$, the difference $f(m)-f(n)$ is divisible by $f(m-n)$. Prove that for all integers $m, n$ with $f(m) \leq f(n)$ the number $f(n)$ is divisible by $f(m)$. + +Solution 1. Suppose that $x$ and $y$ are two integers with $f(x)0 +$$ + +so $f(x-y) \leq f(y)-f(x)f(1)$. Note that such a number exists due to the symmetry of $f$ obtained in Claim 2. + +Claim 3. $f(n) \neq f(1)$ if and only if $a \mid n$. + +Proof. Since $f(1)=\cdots=f(a-1)0$, so $f(n+a) \leq$ $f(a)-f(n)b_{2}>\cdots>b_{k}$ be all these values. One may show (essentially in the same way as in Claim 3) that the set $S_{i}=\left\{n: f(n) \geq b_{i}\right\}$ consists exactly of all numbers divisible by some integer $a_{i} \geq 0$. One obviously has $a_{i} \mid a_{i-1}$, which implies $f\left(a_{i}\right) \mid f\left(a_{i-1}\right)$ by Claim 1. So, $b_{k}\left|b_{k-1}\right| \cdots \mid b_{1}$, thus proving the problem statement. + +Moreover, now it is easy to describe all functions satisfying the conditions of the problem. Namely, all these functions can be constructed as follows. Consider a sequence of nonnegative integers $a_{1}, a_{2}, \ldots, a_{k}$ and another sequence of positive integers $b_{1}, b_{2}, \ldots, b_{k}$ such that $\left|a_{k}\right|=1, a_{i} \neq a_{j}$ and $b_{i} \neq b_{j}$ for all $1 \leq i0$ this implies $M \leq 3^{P(m+a x-b y)}-1$. But $P(m+a x-b y)$ is listed among $P(1), P(2), \ldots, P(d)$, so + +$$ +M<3^{P(m+a x-b y)} \leq 3^{\max \{P(1), P(2), \ldots, P(d)\}} +$$ + +which contradicts (1). + +Comment. We present another variant of the solution above. + +Denote the degree of $P$ by $k$ and its leading coefficient by $p$. Consider any positive integer $n$ and let $a=Q(n)$. Again, denote by $b$ the multiplicative order of 3 modulo $2^{a}-1$. Since $2^{a}-1 \mid 3^{P(n)}-1$, we have $b \mid P(n)$. Moreover, since $2^{Q(n+a t)}-1 \mid 3^{P(n+a t)}-1$ and $a=Q(n) \mid Q(n+a t)$ for each positive integer $t$, we have $2^{a}-1 \mid 3^{P(n+a t)}-1$, hence $b \mid P(n+a t)$ as well. + +Therefore, $b$ divides $\operatorname{gcd}\{P(n+a t): t \geq 0\}$; hence it also divides the number + +$$ +\sum_{i=0}^{k}(-1)^{k-i}\left(\begin{array}{c} +k \\ +i +\end{array}\right) P(n+a i)=p \cdot k ! \cdot a^{k} +$$ + +Finally, we get $b \mid \operatorname{gcd}\left(P(n), k ! \cdot p \cdot Q(n)^{k}\right)$, which is bounded by the same arguments as in the beginning of the solution. So $3^{b}-1$ is bounded, and hence $2^{Q(n)}-1$ is bounded as well. + +## N7 + +Let $p$ be an odd prime number. For every integer $a$, define the number + +$$ +S_{a}=\frac{a}{1}+\frac{a^{2}}{2}+\cdots+\frac{a^{p-1}}{p-1} +$$ + +Let $m$ and $n$ be integers such that + +$$ +S_{3}+S_{4}-3 S_{2}=\frac{m}{n} +$$ + +Prove that $p$ divides $m$. + +Solution 1. For rational numbers $p_{1} / q_{1}$ and $p_{2} / q_{2}$ with the denominators $q_{1}, q_{2}$ not divisible by $p$, we write $p_{1} / q_{1} \equiv p_{2} / q_{2}(\bmod p)$ if the numerator $p_{1} q_{2}-p_{2} q_{1}$ of their difference is divisible by $p$. + +We start with finding an explicit formula for the residue of $S_{a}$ modulo $p$. Note first that for every $k=1, \ldots, p-1$ the number $\left(\begin{array}{l}p \\ k\end{array}\right)$ is divisible by $p$, and + +$$ +\frac{1}{p}\left(\begin{array}{l} +p \\ +k +\end{array}\right)=\frac{(p-1)(p-2) \cdots(p-k+1)}{k !} \equiv \frac{(-1) \cdot(-2) \cdots(-k+1)}{k !}=\frac{(-1)^{k-1}}{k} \quad(\bmod p) +$$ + +Therefore, we have + +$$ +S_{a}=-\sum_{k=1}^{p-1} \frac{(-a)^{k}(-1)^{k-1}}{k} \equiv-\sum_{k=1}^{p-1}(-a)^{k} \cdot \frac{1}{p}\left(\begin{array}{l} +p \\ +k +\end{array}\right) \quad(\bmod p) +$$ + +The number on the right-hand side is integer. Using the binomial formula we express it as + +$$ +-\sum_{k=1}^{p-1}(-a)^{k} \cdot \frac{1}{p}\left(\begin{array}{l} +p \\ +k +\end{array}\right)=-\frac{1}{p}\left(-1-(-a)^{p}+\sum_{k=0}^{p}(-a)^{k}\left(\begin{array}{l} +p \\ +k +\end{array}\right)\right)=\frac{(a-1)^{p}-a^{p}+1}{p} +$$ + +since $p$ is odd. So, we have + +$$ +S_{a} \equiv \frac{(a-1)^{p}-a^{p}+1}{p} \quad(\bmod p) +$$ + +Finally, using the obtained formula we get + +$$ +\begin{aligned} +S_{3}+S_{4}-3 S_{2} & \equiv \frac{\left(2^{p}-3^{p}+1\right)+\left(3^{p}-4^{p}+1\right)-3\left(1^{p}-2^{p}+1\right)}{p} \\ +& =\frac{4 \cdot 2^{p}-4^{p}-4}{p}=-\frac{\left(2^{p}-2\right)^{2}}{p} \quad(\bmod p) . +\end{aligned} +$$ + +By Fermat's theorem, $p \mid 2^{p}-2$, so $p^{2} \mid\left(2^{p}-2\right)^{2}$ and hence $S_{3}+S_{4}-3 S_{2} \equiv 0(\bmod p)$. + +Solution 2. One may solve the problem without finding an explicit formula for $S_{a}$. It is enough to find the following property. + +Lemma. For every integer $a$, we have $S_{a+1} \equiv S_{-a}(\bmod p)$. + +Proof. We expand $S_{a+1}$ using the binomial formula as + +$$ +S_{a+1}=\sum_{k=1}^{p-1} \frac{1}{k} \sum_{j=0}^{k}\left(\begin{array}{l} +k \\ +j +\end{array}\right) a^{j}=\sum_{k=1}^{p-1}\left(\frac{1}{k}+\sum_{j=1}^{k} a^{j} \cdot \frac{1}{k}\left(\begin{array}{l} +k \\ +j +\end{array}\right)\right)=\sum_{k=1}^{p-1} \frac{1}{k}+\sum_{j=1}^{p-1} a^{j} \sum_{k=j}^{p-1} \frac{1}{k}\left(\begin{array}{l} +k \\ +j +\end{array}\right) a^{k} . +$$ + +Note that $\frac{1}{k}+\frac{1}{p-k}=\frac{p}{k(p-k)} \equiv 0(\bmod p)$ for all $1 \leq k \leq p-1$; hence the first sum vanishes modulo $p$. For the second sum, we use the relation $\frac{1}{k}\left(\begin{array}{c}k \\ j\end{array}\right)=\frac{1}{j}\left(\begin{array}{c}k-1 \\ j-1\end{array}\right)$ to obtain + +$$ +S_{a+1} \equiv \sum_{j=1}^{p-1} \frac{a^{j}}{j} \sum_{k=1}^{p-1}\left(\begin{array}{c} +k-1 \\ +j-1 +\end{array}\right) \quad(\bmod p) +$$ + +Finally, from the relation + +$$ +\sum_{k=1}^{p-1}\left(\begin{array}{l} +k-1 \\ +j-1 +\end{array}\right)=\left(\begin{array}{c} +p-1 \\ +j +\end{array}\right)=\frac{(p-1)(p-2) \ldots(p-j)}{j !} \equiv(-1)^{j} \quad(\bmod p) +$$ + +we obtain + +$$ +S_{a+1} \equiv \sum_{j=1}^{p-1} \frac{a^{j}(-1)^{j}}{j !}=S_{-a} +$$ + +Now we turn to the problem. Using the lemma we get + +$$ +S_{3}-3 S_{2} \equiv S_{-2}-3 S_{2}=\sum_{\substack{1 \leq k \leq p-1 \\ k \text { is even }}} \frac{-2 \cdot 2^{k}}{k}+\sum_{\substack{1 \leq k \leq p-1 \\ k \text { is odd }}} \frac{-4 \cdot 2^{k}}{k}(\bmod p) . +$$ + +The first sum in (11) expands as + +$$ +\sum_{\ell=1}^{(p-1) / 2} \frac{-2 \cdot 2^{2 \ell}}{2 \ell}=-\sum_{\ell=1}^{(p-1) / 2} \frac{4^{\ell}}{\ell} +$$ + +Next, using Fermat's theorem, we expand the second sum in (11) as + +$$ +-\sum_{\ell=1}^{(p-1) / 2} \frac{2^{2 \ell+1}}{2 \ell-1} \equiv-\sum_{\ell=1}^{(p-1) / 2} \frac{2^{p+2 \ell}}{p+2 \ell-1}=-\sum_{m=(p+1) / 2}^{p-1} \frac{2 \cdot 4^{m}}{2 m}=-\sum_{m=(p+1) / 2}^{p-1} \frac{4^{m}}{m}(\bmod p) +$$ + +(here we set $m=\ell+\frac{p-1}{2}$ ). Hence, + +$$ +S_{3}-3 S_{2} \equiv-\sum_{\ell=1}^{(p-1) / 2} \frac{4^{\ell}}{\ell}-\sum_{m=(p+1) / 2}^{p-1} \frac{4^{m}}{m}=-S_{4} \quad(\bmod p) +$$ + +Let $k$ be a positive integer and set $n=2^{k}+1$. Prove that $n$ is a prime number if and only if the following holds: there is a permutation $a_{1}, \ldots, a_{n-1}$ of the numbers $1,2, \ldots, n-1$ and a sequence of integers $g_{1}, g_{2}, \ldots, g_{n-1}$ such that $n$ divides $g_{i}^{a_{i}}-a_{i+1}$ for every $i \in\{1,2, \ldots, n-1\}$, where we set $a_{n}=a_{1}$. + +Solution. Let $N=\{1,2, \ldots, n-1\}$. For $a, b \in N$, we say that $b$ follows $a$ if there exists an integer $g$ such that $b \equiv g^{a}(\bmod n)$ and denote this property as $a \rightarrow b$. This way we have a directed graph with $N$ as set of vertices. If $a_{1}, \ldots, a_{n-1}$ is a permutation of $1,2, \ldots, n-1$ such that $a_{1} \rightarrow a_{2} \rightarrow \ldots \rightarrow a_{n-1} \rightarrow a_{1}$ then this is a Hamiltonian cycle in the graph. + +Step I. First consider the case when $n$ is composite. Let $n=p_{1}^{\alpha_{1}} \ldots p_{s}^{\alpha_{s}}$ be its prime factorization. All primes $p_{i}$ are odd. + +Suppose that $\alpha_{i}>1$ for some $i$. For all integers $a, g$ with $a \geq 2$, we have $g^{a} \not \equiv p_{i}\left(\bmod p_{i}^{2}\right)$, because $g^{a}$ is either divisible by $p_{i}^{2}$ or it is not divisible by $p_{i}$. It follows that in any Hamiltonian cycle $p_{i}$ comes immediately after 1 . The same argument shows that $2 p_{i}$ also should come immediately after 1, which is impossible. Hence, there is no Hamiltonian cycle in the graph. + +Now suppose that $n$ is square-free. We have $n=p_{1} p_{2} \ldots p_{s}>9$ and $s \geq 2$. Assume that there exists a Hamiltonian cycle. There are $\frac{n-1}{2}$ even numbers in this cycle, and each number which follows one of them should be a quadratic residue modulo $n$. So, there should be at least $\frac{n-1}{2}$ nonzero quadratic residues modulo $n$. On the other hand, for each $p_{i}$ there exist exactly $\frac{p_{i}+1}{2}$ quadratic residues modulo $p_{i}$; by the Chinese Remainder Theorem, the number of quadratic residues modulo $n$ is exactly $\frac{p_{1}+1}{2} \cdot \frac{p_{2}+1}{2} \cdot \ldots \cdot \frac{p_{s}+1}{2}$, including 0 . Then we have a contradiction by + +$$ +\frac{p_{1}+1}{2} \cdot \frac{p_{2}+1}{2} \cdot \ldots \cdot \frac{p_{s}+1}{2} \leq \frac{2 p_{1}}{3} \cdot \frac{2 p_{2}}{3} \cdot \ldots \cdot \frac{2 p_{s}}{3}=\left(\frac{2}{3}\right)^{s} n \leq \frac{4 n}{9}<\frac{n-1}{2} +$$ + +This proves the "if"-part of the problem. + +Step II. Now suppose that $n$ is prime. For any $a \in N$, denote by $\nu_{2}(a)$ the exponent of 2 in the prime factorization of $a$, and let $\mu(a)=\max \left\{t \in[0, k] \mid 2^{t} \rightarrow a\right\}$. + +Lemma. For any $a, b \in N$, we have $a \rightarrow b$ if and only if $\nu_{2}(a) \leq \mu(b)$. + +Proof. Let $\ell=\nu_{2}(a)$ and $m=\mu(b)$. + +Suppose $\ell \leq m$. Since $b$ follows $2^{m}$, there exists some $g_{0}$ such that $b \equiv g_{0}^{2^{m}}(\bmod n)$. By $\operatorname{gcd}(a, n-1)=2^{\ell}$ there exist some integers $p$ and $q$ such that $p a-q(n-1)=2^{\ell}$. Choosing $g=g_{0}^{2^{m-\ell} p}$ we have $g^{a}=g_{0}^{2^{m-\ell} p a}=g_{0}^{2^{m}+2^{m-\ell} q(n-1)} \equiv g_{0}^{2^{m}} \equiv b(\bmod n)$ by FERMAT's theorem. Hence, $a \rightarrow b$. + +To prove the reverse statement, suppose that $a \rightarrow b$, so $b \equiv g^{a}(\bmod n)$ with some $g$. Then $b \equiv\left(g^{a / 2^{\ell}}\right)^{2^{\ell}}$, and therefore $2^{\ell} \rightarrow b$. By the definition of $\mu(b)$, we have $\mu(b) \geq \ell$. The lemma is +proved. + +Now for every $i$ with $0 \leq i \leq k$, let + +$$ +\begin{aligned} +A_{i} & =\left\{a \in N \mid \nu_{2}(a)=i\right\}, \\ +B_{i} & =\{a \in N \mid \mu(a)=i\}, \\ +\text { and } C_{i} & =\{a \in N \mid \mu(a) \geq i\}=B_{i} \cup B_{i+1} \cup \ldots \cup B_{k} . +\end{aligned} +$$ + +We claim that $\left|A_{i}\right|=\left|B_{i}\right|$ for all $0 \leq i \leq k$. Obviously we have $\left|A_{i}\right|=2^{k-i-1}$ for all $i=$ $0, \ldots, k-1$, and $\left|A_{k}\right|=1$. Now we determine $\left|C_{i}\right|$. We have $\left|C_{0}\right|=n-1$ and by Fermat's theorem we also have $C_{k}=\{1\}$, so $\left|C_{k}\right|=1$. Next, notice that $C_{i+1}=\left\{x^{2} \bmod n \mid x \in C_{i}\right\}$. For every $a \in N$, the relation $x^{2} \equiv a(\bmod n)$ has at most two solutions in $N$. Therefore we have $2\left|C_{i+1}\right| \leq\left|C_{i}\right|$, with the equality achieved only if for every $y \in C_{i+1}$, there exist distinct elements $x, x^{\prime} \in C_{i}$ such that $x^{2} \equiv x^{\prime 2} \equiv y(\bmod n)$ (this implies $\left.x+x^{\prime}=n\right)$. Now, since $2^{k}\left|C_{k}\right|=\left|C_{0}\right|$, we obtain that this equality should be achieved in each step. Hence $\left|C_{i}\right|=2^{k-i}$ for $0 \leq i \leq k$, and therefore $\left|B_{i}\right|=2^{k-i-1}$ for $0 \leq i \leq k-1$ and $\left|B_{k}\right|=1$. + +From the previous arguments we can see that for each $z \in C_{i}(0 \leq i0$. Finally, if $\lambda=k-1$, then $C$ contains $2^{k-1}$ which is the only element of $A_{k-1}$. Since $B_{k-1}=\left\{2^{k}\right\}=A_{k}$ and $B_{k}=\{1\}$, the cycle $C$ contains the path $2^{k-1} \rightarrow 2^{k} \rightarrow 1$ and it contains an odd number again. This completes the proof of the "only if"-part of the problem. + +Comment 1. The lemma and the fact $\left|A_{i}\right|=\left|B_{i}\right|$ together show that for every edge $a \rightarrow b$ of the Hamiltonian cycle, $\nu_{2}(a)=\mu(b)$ must hold. After this observation, the Hamiltonian cycle can be built in many ways. For instance, it is possible to select edges from $A_{i}$ to $B_{i}$ for $i=k, k-1, \ldots, 1$ in such a way that they form disjoint paths; at the end all these paths will have odd endpoints. In the final step, the paths can be closed to form a unique cycle. + +Comment 2. Step II is an easy consequence of some basic facts about the multiplicative group modulo the prime $n=2^{k}+1$. The Lemma follows by noting that this group has order $2^{k}$, so the $a$-th powers are exactly the $2^{\nu_{2}(a)}$-th powers. Using the existence of a primitive root $g$ modulo $n$ one sees that the map from $\{1,2, \ldots, n-1\}$ to itself that sends $a$ to $g^{a} \bmod n$ is a bijection that sends $A_{i}$ to $B_{i}$ for each $i \in\{0, \ldots, k\}$. + diff --git a/IMO/md/en-IMO2012SL.md b/IMO/md/en-IMO2012SL.md new file mode 100644 index 0000000000000000000000000000000000000000..aee35324f6beb2b9175c306a96d6d2a86e48c67f --- /dev/null +++ b/IMO/md/en-IMO2012SL.md @@ -0,0 +1,1639 @@ +# Shortlisted Problems with Solutions + +$53^{\text {rd }}$ International Mathematical Olympiad + +Mar del Plata, Argentina 2012 + +## The shortlisted problems should be kept strictly confidential until IMO 2013 + +## Contributing Countries + +The Organizing Committee and the Problem Selection Committee of IMO 2012 thank the following 40 countries for contributing 136 problem proposals: + +Australia, Austria, Belarus, Belgium, Bulgaria, Canada, Cyprus, Czech Republic, Denmark, Estonia, Finland, France, Germany, Greece, Hong Kong, India, Iran, Ireland, Israel, Japan, Kazakhstan, Luxembourg, Malaysia, Montenegro, Netherlands, Norway, Pakistan, Romania, Russia, Serbia, Slovakia, Slovenia, South Africa, South Korea, Sweden, Thailand, Ukraine, United Kingdom, United States of America, Uzbekistan + +## Problem Selection Committee + +Martín Avendaño + +Carlos di Fiore + +Géza Kós + +Svetoslav Savchev + +## Algebra + +A1. Find all the functions $f: \mathbb{Z} \rightarrow \mathbb{Z}$ such that + +$$ +f(a)^{2}+f(b)^{2}+f(c)^{2}=2 f(a) f(b)+2 f(b) f(c)+2 f(c) f(a) +$$ + +for all integers $a, b, c$ satisfying $a+b+c=0$. + +A2. Let $\mathbb{Z}$ and $\mathbb{Q}$ be the sets of integers and rationals respectively. + +a) Does there exist a partition of $\mathbb{Z}$ into three non-empty subsets $A, B, C$ such that the sets $A+B, B+C, C+A$ are disjoint? + +b) Does there exist a partition of $\mathbb{Q}$ into three non-empty subsets $A, B, C$ such that the sets $A+B, B+C, C+A$ are disjoint? + +Here $X+Y$ denotes the set $\{x+y \mid x \in X, y \in Y\}$, for $X, Y \subseteq \mathbb{Z}$ and $X, Y \subseteq \mathbb{Q}$. + +A3. Let $a_{2}, \ldots, a_{n}$ be $n-1$ positive real numbers, where $n \geq 3$, such that $a_{2} a_{3} \cdots a_{n}=1$. Prove that + +$$ +\left(1+a_{2}\right)^{2}\left(1+a_{3}\right)^{3} \cdots\left(1+a_{n}\right)^{n}>n^{n} . +$$ + +A4. Let $f$ and $g$ be two nonzero polynomials with integer coefficients and $\operatorname{deg} f>\operatorname{deg} g$. Suppose that for infinitely many primes $p$ the polynomial $p f+g$ has a rational root. Prove that $f$ has a rational root. + +A5. Find all functions $f: \mathbb{R} \rightarrow \mathbb{R}$ that satisfy the conditions + +$$ +f(1+x y)-f(x+y)=f(x) f(y) \quad \text { for all } x, y \in \mathbb{R} +$$ + +and $f(-1) \neq 0$. + +A6. Let $f: \mathbb{N} \rightarrow \mathbb{N}$ be a function, and let $f^{m}$ be $f$ applied $m$ times. Suppose that for every $n \in \mathbb{N}$ there exists a $k \in \mathbb{N}$ such that $f^{2 k}(n)=n+k$, and let $k_{n}$ be the smallest such $k$. Prove that the sequence $k_{1}, k_{2}, \ldots$ is unbounded. + +A7. We say that a function $f: \mathbb{R}^{k} \rightarrow \mathbb{R}$ is a metapolynomial if, for some positive integers $m$ and $n$, it can be represented in the form + +$$ +f\left(x_{1}, \ldots, x_{k}\right)=\max _{i=1, \ldots, m} \min _{j=1, \ldots, n} P_{i, j}\left(x_{1}, \ldots, x_{k}\right) +$$ + +where $P_{i, j}$ are multivariate polynomials. Prove that the product of two metapolynomials is also a metapolynomial. + +## Combinatorics + +C1. Several positive integers are written in a row. Iteratively, Alice chooses two adjacent numbers $x$ and $y$ such that $x>y$ and $x$ is to the left of $y$, and replaces the pair $(x, y)$ by either $(y+1, x)$ or $(x-1, x)$. Prove that she can perform only finitely many such iterations. + +C2. Let $n \geq 1$ be an integer. What is the maximum number of disjoint pairs of elements of the set $\{1,2, \ldots, n\}$ such that the sums of the different pairs are different integers not exceeding $n$ ? + +C3. In a $999 \times 999$ square table some cells are white and the remaining ones are red. Let $T$ be the number of triples $\left(C_{1}, C_{2}, C_{3}\right)$ of cells, the first two in the same row and the last two in the same column, with $C_{1}$ and $C_{3}$ white and $C_{2}$ red. Find the maximum value $T$ can attain. + +C4. Players $A$ and $B$ play a game with $N \geq 2012$ coins and 2012 boxes arranged around a circle. Initially $A$ distributes the coins among the boxes so that there is at least 1 coin in each box. Then the two of them make moves in the order $B, A, B, A, \ldots$ by the following rules: + +- On every move of his $B$ passes 1 coin from every box to an adjacent box. +- On every move of hers $A$ chooses several coins that were not involved in $B$ 's previous move and are in different boxes. She passes every chosen coin to an adjacent box. + +Player $A$ 's goal is to ensure at least 1 coin in each box after every move of hers, regardless of how $B$ plays and how many moves are made. Find the least $N$ that enables her to succeed. + +C5. The columns and the rows of a $3 n \times 3 n$ square board are numbered $1,2, \ldots, 3 n$. Every square $(x, y)$ with $1 \leq x, y \leq 3 n$ is colored asparagus, byzantium or citrine according as the modulo 3 remainder of $x+y$ is 0,1 or 2 respectively. One token colored asparagus, byzantium or citrine is placed on each square, so that there are $3 n^{2}$ tokens of each color. + +Suppose that one can permute the tokens so that each token is moved to a distance of at most $d$ from its original position, each asparagus token replaces a byzantium token, each byzantium token replaces a citrine token, and each citrine token replaces an asparagus token. Prove that it is possible to permute the tokens so that each token is moved to a distance of at most $d+2$ from its original position, and each square contains a token with the same color as the square. + +C6. Let $k$ and $n$ be fixed positive integers. In the liar's guessing game, Amy chooses integers $x$ and $N$ with $1 \leq x \leq N$. She tells Ben what $N$ is, but not what $x$ is. Ben may then repeatedly ask Amy whether $x \in S$ for arbitrary sets $S$ of integers. Amy will always answer with yes or no, but she might lie. The only restriction is that she can lie at most $k$ times in a row. After he has asked as many questions as he wants, Ben must specify a set of at most $n$ positive integers. If $x$ is in this set he wins; otherwise, he loses. Prove that: + +a) If $n \geq 2^{k}$ then Ben can always win. + +b) For sufficiently large $k$ there exist $n \geq 1.99^{k}$ such that Ben cannot guarantee a win. + +C7. There are given $2^{500}$ points on a circle labeled $1,2, \ldots, 2^{500}$ in some order. Prove that one can choose 100 pairwise disjoint chords joining some of these points so that the 100 sums of the pairs of numbers at the endpoints of the chosen chords are equal. + +## Geometry + +G1. In the triangle $A B C$ the point $J$ is the center of the excircle opposite to $A$. This excircle is tangent to the side $B C$ at $M$, and to the lines $A B$ and $A C$ at $K$ and $L$ respectively. The lines $L M$ and $B J$ meet at $F$, and the lines $K M$ and $C J$ meet at $G$. Let $S$ be the point of intersection of the lines $A F$ and $B C$, and let $T$ be the point of intersection of the lines $A G$ and $B C$. Prove that $M$ is the midpoint of $S T$. + +G2. Let $A B C D$ be a cyclic quadrilateral whose diagonals $A C$ and $B D$ meet at $E$. The extensions of the sides $A D$ and $B C$ beyond $A$ and $B$ meet at $F$. Let $G$ be the point such that $E C G D$ is a parallelogram, and let $H$ be the image of $E$ under reflection in $A D$. Prove that $D, H, F, G$ are concyclic. + +G3. In an acute triangle $A B C$ the points $D, E$ and $F$ are the feet of the altitudes through $A$, $B$ and $C$ respectively. The incenters of the triangles $A E F$ and $B D F$ are $I_{1}$ and $I_{2}$ respectively; the circumcenters of the triangles $A C I_{1}$ and $B C I_{2}$ are $O_{1}$ and $O_{2}$ respectively. Prove that $I_{1} I_{2}$ and $O_{1} O_{2}$ are parallel. + +G4. Let $A B C$ be a triangle with $A B \neq A C$ and circumcenter $O$. The bisector of $\angle B A C$ intersects $B C$ at $D$. Let $E$ be the reflection of $D$ with respect to the midpoint of $B C$. The lines through $D$ and $E$ perpendicular to $B C$ intersect the lines $A O$ and $A D$ at $X$ and $Y$ respectively. Prove that the quadrilateral $B X C Y$ is cyclic. + +G5. Let $A B C$ be a triangle with $\angle B C A=90^{\circ}$, and let $C_{0}$ be the foot of the altitude from $C$. Choose a point $X$ in the interior of the segment $C C_{0}$, and let $K, L$ be the points on the segments $A X, B X$ for which $B K=B C$ and $A L=A C$ respectively. Denote by $M$ the intersection of $A L$ and $B K$. Show that $M K=M L$. + +G6. Let $A B C$ be a triangle with circumcenter $O$ and incenter $I$. The points $D, E$ and $F$ on the sides $B C, C A$ and $A B$ respectively are such that $B D+B F=C A$ and $C D+C E=A B$. The circumcircles of the triangles $B F D$ and $C D E$ intersect at $P \neq D$. Prove that $O P=O I$. + +G7. Let $A B C D$ be a convex quadrilateral with non-parallel sides $B C$ and $A D$. Assume that there is a point $E$ on the side $B C$ such that the quadrilaterals $A B E D$ and $A E C D$ are circumscribed. Prove that there is a point $F$ on the side $A D$ such that the quadrilaterals $A B C F$ and $B C D F$ are circumscribed if and only if $A B$ is parallel to $C D$. + +G8. Let $A B C$ be a triangle with circumcircle $\omega$ and $\ell$ a line without common points with $\omega$. Denote by $P$ the foot of the perpendicular from the center of $\omega$ to $\ell$. The side-lines $B C, C A, A B$ intersect $\ell$ at the points $X, Y, Z$ different from $P$. Prove that the circumcircles of the triangles $A X P, B Y P$ and $C Z P$ have a common point different from $P$ or are mutually tangent at $P$. + +## Number Theory + +N1. Call admissible a set $A$ of integers that has the following property: + +$$ +\text { If } x, y \in A \text { (possibly } x=y \text { ) then } x^{2}+k x y+y^{2} \in A \text { for every integer } k \text {. } +$$ + +Determine all pairs $m, n$ of nonzero integers such that the only admissible set containing both $m$ and $n$ is the set of all integers. + +N2. Find all triples $(x, y, z)$ of positive integers such that $x \leq y \leq z$ and + +$$ +x^{3}\left(y^{3}+z^{3}\right)=2012(x y z+2) \text {. } +$$ + +N3. Determine all integers $m \geq 2$ such that every $n$ with $\frac{m}{3} \leq n \leq \frac{m}{2}$ divides the binomial coefficient $\left(\begin{array}{c}n \\ m-2 n\end{array}\right)$. + +N4. An integer $a$ is called friendly if the equation $\left(m^{2}+n\right)\left(n^{2}+m\right)=a(m-n)^{3}$ has a solution over the positive integers. + +a) Prove that there are at least 500 friendly integers in the set $\{1,2, \ldots, 2012\}$. + +b) Decide whether $a=2$ is friendly. + +N5. For a nonnegative integer $n$ define $\operatorname{rad}(n)=1$ if $n=0$ or $n=1$, and $\operatorname{rad}(n)=p_{1} p_{2} \cdots p_{k}$ where $p_{1}100$ and every integer $r$ there exist two integers $a$ and $b$ such that $p$ divides $a^{2}+b^{5}-r$. + +## Algebra + +A1. Find all the functions $f: \mathbb{Z} \rightarrow \mathbb{Z}$ such that + +$$ +f(a)^{2}+f(b)^{2}+f(c)^{2}=2 f(a) f(b)+2 f(b) f(c)+2 f(c) f(a) +$$ + +for all integers $a, b, c$ satisfying $a+b+c=0$. + +Solution. The substitution $a=b=c=0$ gives $3 f(0)^{2}=6 f(0)^{2}$, hence + +$$ +f(0)=0 \text {. } +$$ + +The substitution $b=-a$ and $c=0$ gives $\left((f(a)-f(-a))^{2}=0\right.$. Hence $f$ is an even function: + +$$ +f(a)=f(-a) \quad \text { for all } a \in \mathbb{Z} +$$ + +Now set $b=a$ and $c=-2 a$ to obtain $2 f(a)^{2}+f(2 a)^{2}=2 f(a)^{2}+4 f(a) f(2 a)$. Hence + +$$ +f(2 a)=0 \text { or } f(2 a)=4 f(a) \text { for all } a \in \mathbb{Z} +$$ + +If $f(r)=0$ for some $r \geq 1$ then the substitution $b=r$ and $c=-a-r$ gives $(f(a+r)-f(a))^{2}=0$. So $f$ is periodic with period $r$, i. e. + +$$ +f(a+r)=f(a) \text { for all } a \in \mathbb{Z} +$$ + +In particular, if $f(1)=0$ then $f$ is constant, thus $f(a)=0$ for all $a \in \mathbb{Z}$. This function clearly satisfies the functional equation. For the rest of the analysis, we assume $f(1)=k \neq 0$. + +By (3) we have $f(2)=0$ or $f(2)=4 k$. If $f(2)=0$ then $f$ is periodic of period 2 , thus $f($ even $)=0$ and $f($ odd $)=k$. This function is a solution for every $k$. We postpone the verification; for the sequel assume $f(2)=4 k \neq 0$. + +By (3) again, we have $f(4)=0$ or $f(4)=16 k$. In the first case $f$ is periodic of period 4 , and $f(3)=f(-1)=f(1)=k$, so we have $f(4 n)=0, f(4 n+1)=f(4 n+3)=k$, and $f(4 n+2)=4 k$ for all $n \in \mathbb{Z}$. This function is a solution too, which we justify later. For the rest of the analysis, we assume $f(4)=16 k \neq 0$. + +We show now that $f(3)=9 k$. In order to do so, we need two substitutions: + +$$ +\begin{gathered} +a=1, b=2, c=-3 \Longrightarrow f(3)^{2}-10 k f(3)+9 k^{2}=0 \Longrightarrow f(3) \in\{k, 9 k\}, \\ +a=1, b=3, c=-4 \Longrightarrow f(3)^{2}-34 k f(3)+225 k^{2}=0 \Longrightarrow f(3) \in\{9 k, 25 k\} . +\end{gathered} +$$ + +Therefore $f(3)=9 k$, as claimed. Now we prove inductively that the only remaining function is $f(x)=k x^{2}, x \in \mathbb{Z}$. We proved this for $x=0,1,2,3,4$. Assume that $n \geq 4$ and that $f(x)=k x^{2}$ holds for all integers $x \in[0, n]$. Then the substitutions $a=n, b=1, c=-n-1$ and $a=n-1$, $b=2, c=-n-1$ lead respectively to + +$$ +f(n+1) \in\left\{k(n+1)^{2}, k(n-1)^{2}\right\} \quad \text { and } \quad f(n+1) \in\left\{k(n+1)^{2}, k(n-3)^{2}\right\} \text {. } +$$ + +Since $k(n-1)^{2} \neq k(n-3)^{2}$ for $n \neq 2$, the only possibility is $f(n+1)=k(n+1)^{2}$. This completes the induction, so $f(x)=k x^{2}$ for all $x \geq 0$. The same expression is valid for negative values of $x$ since $f$ is even. To verify that $f(x)=k x^{2}$ is actually a solution, we need to check the identity $a^{4}+b^{4}+(a+b)^{4}=2 a^{2} b^{2}+2 a^{2}(a+b)^{2}+2 b^{2}(a+b)^{2}$, which follows directly by expanding both sides. + +Therefore the only possible solutions of the functional equation are the constant function $f_{1}(x)=0$ and the following functions: + +$$ +f_{2}(x)=k x^{2} \quad f_{3}(x)=\left\{\begin{array}{cc} +0 & x \text { even } \\ +k & x \text { odd } +\end{array} \quad f_{4}(x)=\left\{\begin{array}{ccc} +0 & x \equiv 0 & (\bmod 4) \\ +k & x \equiv 1 & (\bmod 2) \\ +4 k & x \equiv 2 & (\bmod 4) +\end{array}\right.\right. +$$ + +for any non-zero integer $k$. The verification that they are indeed solutions was done for the first two. For $f_{3}$ note that if $a+b+c=0$ then either $a, b, c$ are all even, in which case $f(a)=f(b)=f(c)=0$, or one of them is even and the other two are odd, so both sides of the equation equal $2 k^{2}$. For $f_{4}$ we use similar parity considerations and the symmetry of the equation, which reduces the verification to the triples $(0, k, k),(4 k, k, k),(0,0,0),(0,4 k, 4 k)$. They all satisfy the equation. + +Comment. We used several times the same fact: For any $a, b \in \mathbb{Z}$ the functional equation is a quadratic equation in $f(a+b)$ whose coefficients depend on $f(a)$ and $f(b)$ : + +$$ +f(a+b)^{2}-2(f(a)+f(b)) f(a+b)+(f(a)-f(b))^{2}=0 +$$ + +Its discriminant is $16 f(a) f(b)$. Since this value has to be non-negative for any $a, b \in \mathbb{Z}$, we conclude that either $f$ or $-f$ is always non-negative. Also, if $f$ is a solution of the functional equation, then $-f$ is also a solution. Therefore we can assume $f(x) \geq 0$ for all $x \in \mathbb{Z}$. Now, the two solutions of the quadratic equation are + +$$ +f(a+b) \in\left\{(\sqrt{f(a)}+\sqrt{f(b)})^{2},(\sqrt{f(a)}-\sqrt{f(b)})^{2}\right\} \quad \text { for all } a, b \in \mathbb{Z} +$$ + +The computation of $f(3)$ from $f(1), f(2)$ and $f(4)$ that we did above follows immediately by setting $(a, b)=(1,2)$ and $(a, b)=(1,-4)$. The inductive step, where $f(n+1)$ is derived from $f(n), f(n-1)$, $f(2)$ and $f(1)$, follows immediately using $(a, b)=(n, 1)$ and $(a, b)=(n-1,2)$. + +A2. Let $\mathbb{Z}$ and $\mathbb{Q}$ be the sets of integers and rationals respectively. + +a) Does there exist a partition of $\mathbb{Z}$ into three non-empty subsets $A, B, C$ such that the sets $A+B, B+C, C+A$ are disjoint? + +b) Does there exist a partition of $\mathbb{Q}$ into three non-empty subsets $A, B, C$ such that the sets $A+B, B+C, C+A$ are disjoint? + +Here $X+Y$ denotes the set $\{x+y \mid x \in X, y \in Y\}$, for $X, Y \subseteq \mathbb{Z}$ and $X, Y \subseteq \mathbb{Q}$. + +Solution 1. a) The residue classes modulo 3 yield such a partition: + +$$ +A=\{3 k \mid k \in \mathbb{Z}\}, \quad B=\{3 k+1 \mid k \in \mathbb{Z}\}, \quad C=\{3 k+2 \mid k \in \mathbb{Z}\} +$$ + +b) The answer is no. Suppose that $\mathbb{Q}$ can be partitioned into non-empty subsets $A, B, C$ as stated. Note that for all $a \in A, b \in B, c \in C$ one has + +$$ +a+b-c \in C, \quad b+c-a \in A, \quad c+a-b \in B +$$ + +Indeed $a+b-c \notin A$ as $(A+B) \cap(A+C)=\emptyset$, and similarly $a+b-c \notin B$, hence $a+b-c \in C$. The other two relations follow by symmetry. Hence $A+B \subset C+C, B+C \subset A+A, C+A \subset B+B$. + +The opposite inclusions also hold. Let $a, a^{\prime} \in A$ and $b \in B, c \in C$ be arbitrary. By (1) $a^{\prime}+c-b \in B$, and since $a \in A, c \in C$, we use (1) again to obtain + +$$ +a+a^{\prime}-b=a+\left(a^{\prime}+c-b\right)-c \in C . +$$ + +So $A+A \subset B+C$ and likewise $B+B \subset C+A, C+C \subset A+B$. In summary + +$$ +B+C=A+A, \quad C+A=B+B, \quad A+B=C+C . +$$ + +Furthermore suppose that $0 \in A$ without loss of generality. Then $B=\{0\}+B \subset A+B$ and $C=\{0\}+C \subset A+C$. So, since $B+C$ is disjoint with $A+B$ and $A+C$, it is also disjoint with $B$ and $C$. Hence $B+C$ is contained in $\mathbb{Z} \backslash(B \cup C)=A$. Because $B+C=A+A$, we obtain $A+A \subset A$. On the other hand $A=\{0\}+A \subset A+A$, implying $A=A+A=B+C$. + +Therefore $A+B+C=A+A+A=A$, and now $B+B=C+A$ and $C+C=A+B$ yield $B+B+B=A+B+C=A, C+C+C=A+B+C=A$. In particular if $r \in \mathbb{Q}=A \cup B \cup C$ is arbitrary then $3 r \in A$. + +However such a conclusion is impossible. Take any $b \in B(B \neq \emptyset)$ and let $r=b / 3 \in \mathbb{Q}$. Then $b=3 r \in A$ which is a contradiction. + +Solution 2. We prove that the example for $\mathbb{Z}$ from the first solution is unique, and then use this fact to solve part b). + +Let $\mathbb{Z}=A \cup B \cup C$ be a partition of $\mathbb{Z}$ with $A, B, C \neq \emptyset$ and $A+B, B+C, C+A$ disjoint. We need the relations (1) which clearly hold for $\mathbb{Z}$. Fix two consecutive integers from different sets, say $b \in B$ and $c=b+1 \in C$. For every $a \in A$ we have, in view of (1), $a-1=a+b-c \in C$ and $a+1=a+c-b \in B$. So every $a \in A$ is preceded by a number from $C$ and followed by a number from $B$. + +In particular there are pairs of the form $c, c+1$ with $c \in C, c+1 \in A$. For such a pair and any $b \in B$ analogous reasoning shows that each $b \in B$ is preceded by a number from $A$ and followed by a number from $C$. There are also pairs $b, b-1$ with $b \in B, b-1 \in A$. We use them in a similar way to prove that each $c \in C$ is preceded by a number from $B$ and followed by a number from $A$. + +By putting the observations together we infer that $A, B, C$ are the three congruence classes modulo 3. Observe that all multiples of 3 are in the set of the partition that contains 0 . + +Now we turn to part b). Suppose that there is a partition of $\mathbb{Q}$ with the given properties. Choose three rationals $r_{i}=p_{i} / q_{i}$ from the three sets $A, B, C, i=1,2,3$, and set $N=3 q_{1} q_{2} q_{3}$. + +Let $S \subset \mathbb{Q}$ be the set of fractions with denominators $N$ (irreducible or not). It is obtained through multiplication of every integer by the constant $1 / N$, hence closed under sums and differences. Moreover, if we identify each $k \in \mathbb{Z}$ with $k / N \in S$ then $S$ is essentially the set $\mathbb{Z}$ with respect to addition. The numbers $r_{i}$ belong to $S$ because + +$$ +r_{1}=\frac{3 p_{1} q_{2} q_{3}}{N}, \quad r_{2}=\frac{3 p_{2} q_{3} q_{1}}{N}, \quad r_{3}=\frac{3 p_{3} q_{1} q_{2}}{N} +$$ + +The partition $\mathbb{Q}=A \cup B \cup C$ of $\mathbb{Q}$ induces a partition $S=A^{\prime} \cup B^{\prime} \cup C^{\prime}$ of $S$, with $A^{\prime}=A \cap S$, $B^{\prime}=B \cap S, C^{\prime}=C \cap S$. Clearly $A^{\prime}+B^{\prime}, B^{\prime}+C^{\prime}, C^{\prime}+A^{\prime}$ are disjoint, so this partition has the properties we consider. + +By the uniqueness of the example for $\mathbb{Z}$ the sets $A^{\prime}, B^{\prime}, C^{\prime}$ are the congruence classes modulo 3 , multiplied by $1 / N$. Also all multiples of $3 / N$ are in the same set, $A^{\prime}, B^{\prime}$ or $C^{\prime}$. This holds for $r_{1}, r_{2}, r_{3}$ in particular as they are all multiples of $3 / N$. However $r_{1}, r_{2}, r_{3}$ are in different sets $A^{\prime}, B^{\prime}, C^{\prime}$ since they were chosen from different sets $A, B, C$. The contradiction ends the proof. + +Comment. The uniqueness of the example for $\mathbb{Z}$ can also be deduced from the argument in the first solution. + +A3. Let $a_{2}, \ldots, a_{n}$ be $n-1$ positive real numbers, where $n \geq 3$, such that $a_{2} a_{3} \cdots a_{n}=1$. Prove that + +$$ +\left(1+a_{2}\right)^{2}\left(1+a_{3}\right)^{3} \cdots\left(1+a_{n}\right)^{n}>n^{n} . +$$ + +Solution. The substitution $a_{2}=\frac{x_{2}}{x_{1}}, a_{3}=\frac{x_{3}}{x_{2}}, \ldots, a_{n}=\frac{x_{1}}{x_{n-1}}$ transforms the original problem into the inequality + +$$ +\left(x_{1}+x_{2}\right)^{2}\left(x_{2}+x_{3}\right)^{3} \cdots\left(x_{n-1}+x_{1}\right)^{n}>n^{n} x_{1}^{2} x_{2}^{3} \cdots x_{n-1}^{n} +$$ + +for all $x_{1}, \ldots, x_{n-1}>0$. To prove this, we use the AM-GM inequality for each factor of the left-hand side as follows: + +$$ +\begin{array}{rlcl} +\left(x_{1}+x_{2}\right)^{2} & & & \geq 2^{2} x_{1} x_{2} \\ +\left(x_{2}+x_{3}\right)^{3} & = & \left(2\left(\frac{x_{2}}{2}\right)+x_{3}\right)^{3} & \geq 3^{3}\left(\frac{x_{2}}{2}\right)^{2} x_{3} \\ +\left(x_{3}+x_{4}\right)^{4} & = & \left(3\left(\frac{x_{3}}{3}\right)+x_{4}\right)^{4} & \geq 4^{4}\left(\frac{x_{3}}{3}\right)^{3} x_{4} \\ +& \vdots & \vdots & \vdots +\end{array} +$$ + +Multiplying these inequalities together gives $\left({ }^{*}\right)$, with inequality sign $\geq$ instead of $>$. However for the equality to occur it is necessary that $x_{1}=x_{2}, x_{2}=2 x_{3}, \ldots, x_{n-1}=(n-1) x_{1}$, implying $x_{1}=(n-1) ! x_{1}$. This is impossible since $x_{1}>0$ and $n \geq 3$. Therefore the inequality is strict. + +Comment. One can avoid the substitution $a_{i}=x_{i} / x_{i-1}$. Apply the weighted AM-GM inequality to each factor $\left(1+a_{k}\right)^{k}$, with the same weights like above, to obtain + +$$ +\left(1+a_{k}\right)^{k}=\left((k-1) \frac{1}{k-1}+a_{k}\right)^{k} \geq \frac{k^{k}}{(k-1)^{k-1}} a_{k} +$$ + +Multiplying all these inequalities together gives + +$$ +\left(1+a_{2}\right)^{2}\left(1+a_{3}\right)^{3} \cdots\left(1+a_{n}\right)^{n} \geq n^{n} a_{2} a_{3} \cdots a_{n}=n^{n} . +$$ + +The same argument as in the proof above shows that the equality cannot be attained. + +A4. Let $f$ and $g$ be two nonzero polynomials with integer coefficients and $\operatorname{deg} f>\operatorname{deg} g$. Suppose that for infinitely many primes $p$ the polynomial $p f+g$ has a rational root. Prove that $f$ has a rational root. + +Solution 1. Since $\operatorname{deg} f>\operatorname{deg} g$, we have $|g(x) / f(x)|<1$ for sufficiently large $x$; more precisely, there is a real number $R$ such that $|g(x) / f(x)|<1$ for all $x$ with $|x|>R$. Then for all such $x$ and all primes $p$ we have + +$$ +|p f(x)+g(x)| \geq|f(x)|\left(p-\frac{|g(x)|}{|f(x)|}\right)>0 +$$ + +Hence all real roots of the polynomials $p f+g$ lie in the interval $[-R, R]$. + +Let $f(x)=a_{n} x^{n}+a_{n-1} x^{n-1}+\cdots+a_{0}$ and $g(x)=b_{m} x^{m}+b_{m-1} x^{m-1}+\cdots+b_{0}$ where $n>m, a_{n} \neq 0$ and $b_{m} \neq 0$. Upon replacing $f(x)$ and $g(x)$ by $a_{n}^{n-1} f\left(x / a_{n}\right)$ and $a_{n}^{n-1} g\left(x / a_{n}\right)$ respectively, we reduce the problem to the case $a_{n}=1$. In other words one can assume that $f$ is monic. Then the leading coefficient of $p f+g$ is $p$, and if $r=u / v$ is a rational root of $p f+g$ with $(u, v)=1$ and $v>0$, then either $v=1$ or $v=p$. + +First consider the case when $v=1$ infinitely many times. If $v=1$ then $|u| \leq R$, so there are only finitely many possibilities for the integer $u$. Therefore there exist distinct primes $p$ and $q$ for which we have the same value of $u$. Then the polynomials $p f+g$ and $q f+g$ share this root, implying $f(u)=g(u)=0$. So in this case $f$ and $g$ have an integer root in common. + +Now suppose that $v=p$ infinitely many times. By comparing the exponent of $p$ in the denominators of $p f(u / p)$ and $g(u / p)$ we get $m=n-1$ and $p f(u / p)+g(u / p)=0$ reduces to an equation of the form + +$$ +\left(u^{n}+a_{n-1} p u^{n-1}+\ldots+a_{0} p^{n}\right)+\left(b_{n-1} u^{n-1}+b_{n-2} p u^{n-2}+\ldots+b_{0} p^{n-1}\right)=0 . +$$ + +The equation above implies that $u^{n}+b_{n-1} u^{n-1}$ is divisible by $p$ and hence, since $(u, p)=1$, we have $u+b_{n-1}=p k$ with some integer $k$. On the other hand all roots of $p f+g$ lie in the interval $[-R, R]$, so that + +$$ +\begin{gathered} +\frac{\left|p k-b_{n-1}\right|}{p}=\frac{|u|}{p}5 / 4$ then $-x<5 / 4$ and so $g(2-x)-g(-x)=2$ by the above. On the other hand (3) gives $g(x)=2-g(2-x), g(x+2)=2-g(-x)$, so that $g(x+2)-g(x)=g(2-x)-g(-x)=2$. Thus (4) is true for all $x \in \mathbb{R}$. + +Now replace $x$ by $-x$ in (3) to obtain $g(-x)+g(2+x)=2$. In view of (4) this leads to $g(x)+g(-x)=0$, i. e. $g(-x)=-g(x)$ for all $x$. Taking this into account, we apply (1) to the pairs $(-x, y)$ and $(x,-y)$ : + +$g(1-x y)-g(-x+y)=(g(x)+1)(1-g(y)), \quad g(1-x y)-g(x-y)=(1-g(x))(g(y)+1)$. + +Adding up yields $g(1-x y)=1-g(x) g(y)$. Then $g(1+x y)=1+g(x) g(y)$ by (3). Now the original equation (1) takes the form $g(x+y)=g(x)+g(y)$. Hence $g$ is additive. + +By additvity $g(1+x y)=g(1)+g(x y)=1+g(x y)$; since $g(1+x y)=1+g(x) g(y)$ was shown above, we also have $g(x y)=g(x) g(y)$ ( $g$ is multiplicative). In particular $y=x$ gives $g\left(x^{2}\right)=g(x)^{2} \geq 0$ for all $x$, meaning that $g(x) \geq 0$ for $x \geq 0$. Since $g$ is additive and bounded from below on $[0,+\infty)$, it is linear; more exactly $g(x)=g(1) x=x$ for all $x \in \mathbb{R}$. + +In summary $f(x)=x-1, x \in \mathbb{R}$. It is straightforward that this function satisfies the requirements. + +Comment. There are functions that satisfy the given equation but vanish at -1 , for instance the constant function 0 and $f(x)=x^{2}-1, x \in \mathbb{R}$. + +A6. Let $f: \mathbb{N} \rightarrow \mathbb{N}$ be a function, and let $f^{m}$ be $f$ applied $m$ times. Suppose that for every $n \in \mathbb{N}$ there exists a $k \in \mathbb{N}$ such that $f^{2 k}(n)=n+k$, and let $k_{n}$ be the smallest such $k$. Prove that the sequence $k_{1}, k_{2}, \ldots$ is unbounded. + +Solution. We restrict attention to the set + +$$ +S=\left\{1, f(1), f^{2}(1), \ldots\right\} +$$ + +Observe that $S$ is unbounded because for every number $n$ in $S$ there exists a $k>0$ such that $f^{2 k}(n)=n+k$ is in $S$. Clearly $f$ maps $S$ into itself; moreover $f$ is injective on $S$. Indeed if $f^{i}(1)=f^{j}(1)$ with $i \neq j$ then the values $f^{m}(1)$ start repeating periodically from some point on, and $S$ would be finite. + +Define $g: S \rightarrow S$ by $g(n)=f^{2 k_{n}}(n)=n+k_{n}$. We prove that $g$ is injective too. Suppose that $g(a)=g(b)$ with $ak_{b}$. So, since $f$ is injective on $S$, we obtain + +$$ +f^{2\left(k_{a}-k_{b}\right)}(a)=b=a+\left(k_{a}-k_{b}\right) . +$$ + +However this contradicts the minimality of $k_{a}$ as $0n$ for $n \in S$, so $T$ is non-empty. For each $t \in T$ denote $C_{t}=\left\{t, g(t), g^{2}(t), \ldots\right\}$; call $C_{t}$ the chain starting at $t$. Observe that distinct chains are disjoint because $g$ is injective. Each $n \in S \backslash T$ has the form $n=g\left(n^{\prime}\right)$ with $n^{\prime}k$, i. e. $k_{n}>k$. In conclusion $k_{1}, k_{2}, \ldots$ is unbounded. + +A7. We say that a function $f: \mathbb{R}^{k} \rightarrow \mathbb{R}$ is a metapolynomial if, for some positive integers $m$ and $n$, it can be represented in the form + +$$ +f\left(x_{1}, \ldots, x_{k}\right)=\max _{i=1, \ldots, m} \min _{j=1, \ldots, n} P_{i, j}\left(x_{1}, \ldots, x_{k}\right) +$$ + +where $P_{i, j}$ are multivariate polynomials. Prove that the product of two metapolynomials is also a metapolynomial. + +Solution. We use the notation $f(x)=f\left(x_{1}, \ldots, x_{k}\right)$ for $x=\left(x_{1}, \ldots, x_{k}\right)$ and $[m]=\{1,2, \ldots, m\}$. Observe that if a metapolynomial $f(x)$ admits a representation like the one in the statement for certain positive integers $m$ and $n$, then they can be replaced by any $m^{\prime} \geq m$ and $n^{\prime} \geq n$. For instance, if we want to replace $m$ by $m+1$ then it is enough to define $P_{m+1, j}(x)=P_{m, j}(x)$ and note that repeating elements of a set do not change its maximum nor its minimum. So one can assume that any two metapolynomials are defined with the same $m$ and $n$. We reserve letters $P$ and $Q$ for polynomials, so every function called $P, P_{i, j}, Q, Q_{i, j}, \ldots$ is a polynomial function. + +We start with a lemma that is useful to change expressions of the form $\min \max f_{i, j}$ to ones of the form $\max \min g_{i, j}$. + +Lemma. Let $\left\{a_{i, j}\right\}$ be real numbers, for all $i \in[m]$ and $j \in[n]$. Then + +$$ +\min _{i \in[m]} \max _{j \in[n]} a_{i, j}=\max _{j_{1}, \ldots, j_{m} \in[n]} \min _{i \in[m]} a_{i, j_{i}} +$$ + +where the max in the right-hand side is over all vectors $\left(j_{1}, \ldots, j_{m}\right)$ with $j_{1}, \ldots, j_{m} \in[n]$. + +Proof. We can assume for all $i$ that $a_{i, n}=\max \left\{a_{i, 1}, \ldots, a_{i, n}\right\}$ and $a_{m, n}=\min \left\{a_{1, n}, \ldots, a_{m, n}\right\}$. The left-hand side is $=a_{m, n}$ and hence we need to prove the same for the right-hand side. If $\left(j_{1}, j_{2}, \ldots, j_{m}\right)=(n, n, \ldots, n)$ then $\min \left\{a_{1, j_{1}}, \ldots, a_{m, j_{m}}\right\}=\min \left\{a_{1, n}, \ldots, a_{m, n}\right\}=a_{m, n}$ which implies that the right-hand side is $\geq a_{m, n}$. It remains to prove the opposite inequality and this is equivalent to $\min \left\{a_{1, j_{1}}, \ldots, a_{m, j_{m}}\right\} \leq a_{m, n}$ for all possible $\left(j_{1}, j_{2}, \ldots, j_{m}\right)$. This is true because $\min \left\{a_{1, j_{1}}, \ldots, a_{m, j_{m}}\right\} \leq a_{m, j_{m}} \leq a_{m, n}$. + +We need to show that the family $\mathcal{M}$ of metapolynomials is closed under multiplication, but it turns out easier to prove more: that it is also closed under addition, maxima and minima. + +First we prove the assertions about the maxima and the minima. If $f_{1}, \ldots, f_{r}$ are metapolynomials, assume them defined with the same $m$ and $n$. Then + +$$ +f=\max \left\{f_{1}, \ldots, f_{r}\right\}=\max \left\{\max _{i \in[m]} \min _{j \in[n]} P_{i, j}^{1}, \ldots, \max _{i \in[m]} \min _{j \in[n]} P_{i, j}^{r}\right\}=\max _{s \in[r], i \in[m]} \min _{j \in[n]} P_{i, j}^{s} +$$ + +It follows that $f=\max \left\{f_{1}, \ldots, f_{r}\right\}$ is a metapolynomial. The same argument works for the minima, but first we have to replace $\min \max$ by $\max \min$, and this is done via the lemma. + +Another property we need is that if $f=\max \min P_{i, j}$ is a metapolynomial then so is $-f$. Indeed, $-f=\min \left(-\min P_{i, j}\right)=\min \max P_{i, j}$. + +To prove $\mathcal{M}$ is closed under addition let $f=\max \min P_{i, j}$ and $g=\max \min Q_{i, j}$. Then + +$$ +\begin{gathered} +f(x)+g(x)=\max _{i \in[m]} \min _{j \in[n]} P_{i, j}(x)+\max _{i \in[m]} \min _{j \in[n]} Q_{i, j}(x) \\ +=\max _{i_{1}, i_{2} \in[m]}\left(\min _{j \in[n]} P_{i_{1}, j}(x)+\min _{j \in[n]} Q_{i_{2}, j}(x)\right)=\max _{i_{1}, i_{2} \in[m]} \min _{j_{1}, j_{2} \in[n]}\left(P_{i_{1}, j_{1}}(x)+Q_{i_{2}, j_{2}}(x)\right), +\end{gathered} +$$ + +and hence $f(x)+g(x)$ is a metapolynomial. + +We proved that $\mathcal{M}$ is closed under sums, maxima and minima, in particular any function that can be expressed by sums, max, min, polynomials or even metapolynomials is in $\mathcal{M}$. + +We would like to proceed with multiplication along the same lines like with addition, but there is an essential difference. In general the product of the maxima of two sets is not equal +to the maximum of the product of the sets. We need to deal with the fact that $ay$ and $x$ is to the left of $y$, and replaces the pair $(x, y)$ by either $(y+1, x)$ or $(x-1, x)$. Prove that she can perform only finitely many such iterations. + +Solution 1. Note first that the allowed operation does not change the maximum $M$ of the initial sequence. Let $a_{1}, a_{2}, \ldots, a_{n}$ be the numbers obtained at some point of the process. Consider the sum + +$$ +S=a_{1}+2 a_{2}+\cdots+n a_{n} +$$ + +We claim that $S$ increases by a positive integer amount with every operation. Let the operation replace the pair ( $\left.a_{i}, a_{i+1}\right)$ by a pair $\left(c, a_{i}\right)$, where $a_{i}>a_{i+1}$ and $c=a_{i+1}+1$ or $c=a_{i}-1$. Then the new and the old value of $S$ differ by $d=\left(i c+(i+1) a_{i}\right)-\left(i a_{i}+(i+1) a_{i+1}\right)=a_{i}-a_{i+1}+i\left(c-a_{i+1}\right)$. The integer $d$ is positive since $a_{i}-a_{i+1} \geq 1$ and $c-a_{i+1} \geq 0$. + +On the other hand $S \leq(1+2+\cdots+n) M$ as $a_{i} \leq M$ for all $i=1, \ldots, n$. Since $S$ increases by at least 1 at each step and never exceeds the constant $(1+2+\cdots+n) M$, the process stops after a finite number of iterations. + +Solution 2. Like in the first solution note that the operations do not change the maximum $M$ of the initial sequence. Now consider the reverse lexicographical order for $n$-tuples of integers. We say that $\left(x_{1}, \ldots, x_{n}\right)<\left(y_{1}, \ldots, y_{n}\right)$ if $x_{n}y$ and $y \leq a \leq x$, we see that $s_{i}$ decreases by at least 1 . This concludes the proof. + +Comment. All three proofs work if $x$ and $y$ are not necessarily adjacent, and if the pair $(x, y)$ is replaced by any pair $(a, x)$, with $a$ an integer satisfying $y \leq a \leq x$. There is nothing special about the "weights" $1,2, \ldots, n$ in the definition of $S=\sum_{i=1}^{n} i a_{i}$ from the first solution. For any sequence $w_{1}2^{k}$ we show how Ben can find a number $y \in T$ that is different from $x$. By performing this step repeatedly he can reduce $T$ to be of size $2^{k} \leq n$ and thus win. + +Since only the size $m>2^{k}$ of $T$ is relevant, assume that $T=\left\{0,1, \ldots, 2^{k}, \ldots, m-1\right\}$. Ben begins by asking repeatedly whether $x$ is $2^{k}$. If Amy answers no $k+1$ times in a row, one of these answers is truthful, and so $x \neq 2^{k}$. Otherwise Ben stops asking about $2^{k}$ at the first answer yes. He then asks, for each $i=1, \ldots, k$, if the binary representation of $x$ has a 0 in the $i$ th digit. Regardless of what the $k$ answers are, they are all inconsistent with a certain number $y \in\left\{0,1, \ldots, 2^{k}-1\right\}$. The preceding answer yes about $2^{k}$ is also inconsistent with $y$. Hence $y \neq x$. Otherwise the last $k+1$ answers are not truthful, which is impossible. + +Either way, Ben finds a number in $T$ that is different from $x$, and the claim is proven. + +b) We prove that if $1<\lambda<2$ and $n=\left\lfloor(2-\lambda) \lambda^{k+1}\right\rfloor-1$ then Ben cannot guarantee a win. To complete the proof, then it suffices to take $\lambda$ such that $1.99<\lambda<2$ and $k$ large enough so that + +$$ +n=\left\lfloor(2-\lambda) \lambda^{k+1}\right\rfloor-1 \geq 1.99^{k} +$$ + +Consider the following strategy for Amy. First she chooses $N=n+1$ and $x \in\{1,2, \ldots, n+1\}$ arbitrarily. After every answer of hers Amy determines, for each $i=1,2, \ldots, n+1$, the number $m_{i}$ of consecutive answers she has given by that point that are inconsistent with $i$. To decide on her next answer, she then uses the quantity + +$$ +\phi=\sum_{i=1}^{n+1} \lambda^{m_{i}} +$$ + +No matter what Ben's next question is, Amy chooses the answer which minimizes $\phi$. + +We claim that with this strategy $\phi$ will always stay less than $\lambda^{k+1}$. Consequently no exponent $m_{i}$ in $\phi$ will ever exceed $k$, hence Amy will never give more than $k$ consecutive answers inconsistent with some $i$. In particular this applies to the target number $x$, so she will never lie more than $k$ times in a row. Thus, given the claim, Amy's strategy is legal. Since the strategy does not depend on $x$ in any way, Ben can make no deductions about $x$, and therefore he cannot guarantee a win. + +It remains to show that $\phi<\lambda^{k+1}$ at all times. Initially each $m_{i}$ is 0 , so this condition holds in the beginning due to $1<\lambda<2$ and $n=\left\lfloor(2-\lambda) \lambda^{k+1}\right\rfloor-1$. Suppose that $\phi<\lambda^{k+1}$ at some point, and Ben has just asked if $x \in S$ for some set $S$. According as Amy answers yes or no, the new value of $\phi$ becomes + +$$ +\phi_{1}=\sum_{i \in S} 1+\sum_{i \notin S} \lambda^{m_{i}+1} \quad \text { or } \quad \phi_{2}=\sum_{i \in S} \lambda^{m_{i}+1}+\sum_{i \notin S} 1 +$$ + +Since Amy chooses the option minimizing $\phi$, the new $\phi$ will equal $\min \left(\phi_{1}, \phi_{2}\right)$. Now we have + +$$ +\min \left(\phi_{1}, \phi_{2}\right) \leq \frac{1}{2}\left(\phi_{1}+\phi_{2}\right)=\frac{1}{2}\left(\sum_{i \in S}\left(1+\lambda^{m_{i}+1}\right)+\sum_{i \notin S}\left(\lambda^{m_{i}+1}+1\right)\right)=\frac{1}{2}(\lambda \phi+n+1) +$$ + +Because $\phi<\lambda^{k+1}$, the assumptions $\lambda<2$ and $n=\left\lfloor(2-\lambda) \lambda^{k+1}\right\rfloor-1$ lead to + +$$ +\min \left(\phi_{1}, \phi_{2}\right)<\frac{1}{2}\left(\lambda^{k+2}+(2-\lambda) \lambda^{k+1}\right)=\lambda^{k+1} +$$ + +The claim follows, which completes the solution. + +Comment. Given a fixed $k$, let $f(k)$ denote the minimum value of $n$ for which Ben can guarantee a victory. The problem asks for a proof that for large $k$ + +$$ +1.99^{k} \leq f(k) \leq 2^{k} +$$ + +A computer search shows that $f(k)=2,3,4,7,11,17$ for $k=1,2,3,4,5,6$. + +C7. There are given $2^{500}$ points on a circle labeled $1,2, \ldots, 2^{500}$ in some order. Prove that one can choose 100 pairwise disjoint chords joining some of these points so that the 100 sums of the pairs of numbers at the endpoints of the chosen chords are equal. + +Solution. The proof is based on the following general fact. + +Lemma. In a graph $G$ each vertex $v$ has degree $d_{v}$. Then $G$ contains an independent set $S$ of vertices such that $|S| \geq f(G)$ where + +$$ +f(G)=\sum_{v \in G} \frac{1}{d_{v}+1} +$$ + +Proof. Induction on $n=|G|$. The base $n=1$ is clear. For the inductive step choose a vertex $v_{0}$ in $G$ of minimum degree $d$. Delete $v_{0}$ and all of its neighbors $v_{1}, \ldots, v_{d}$ and also all edges with endpoints $v_{0}, v_{1}, \ldots, v_{d}$. This gives a new graph $G^{\prime}$. By the inductive assumption $G^{\prime}$ contains an independent set $S^{\prime}$ of vertices such that $\left|S^{\prime}\right| \geq f\left(G^{\prime}\right)$. Since no vertex in $S^{\prime}$ is a neighbor of $v_{0}$ in $G$, the set $S=S^{\prime} \cup\left\{v_{0}\right\}$ is independent in $G$. + +Let $d_{v}^{\prime}$ be the degree of a vertex $v$ in $G^{\prime}$. Clearly $d_{v}^{\prime} \leq d_{v}$ for every such vertex $v$, and also $d_{v_{i}} \geq d$ for all $i=0,1, \ldots, d$ by the minimal choice of $v_{0}$. Therefore + +$$ +f\left(G^{\prime}\right)=\sum_{v \in G^{\prime}} \frac{1}{d_{v}^{\prime}+1} \geq \sum_{v \in G^{\prime}} \frac{1}{d_{v}+1}=f(G)-\sum_{i=0}^{d} \frac{1}{d_{v_{i}}+1} \geq f(G)-\frac{d+1}{d+1}=f(G)-1 . +$$ + +Hence $|S|=\left|S^{\prime}\right|+1 \geq f\left(G^{\prime}\right)+1 \geq f(G)$, and the induction is complete. + +We pass on to our problem. For clarity denote $n=2^{499}$ and draw all chords determined by the given $2 n$ points. Color each chord with one of the colors $3,4, \ldots, 4 n-1$ according to the sum of the numbers at its endpoints. Chords with a common endpoint have different colors. For each color $c$ consider the following graph $G_{c}$. Its vertices are the chords of color $c$, and two chords are neighbors in $G_{c}$ if they intersect. Let $f\left(G_{c}\right)$ have the same meaning as in the lemma for all graphs $G_{c}$. + +Every chord $\ell$ divides the circle into two arcs, and one of them contains $m(\ell) \leq n-1$ given points. (In particular $m(\ell)=0$ if $\ell$ joins two consecutive points.) For each $i=0,1, \ldots, n-2$ there are $2 n$ chords $\ell$ with $m(\ell)=i$. Such a chord has degree at most $i$ in the respective graph. Indeed let $A_{1}, \ldots, A_{i}$ be all points on either arc determined by a chord $\ell$ with $m(\ell)=i$ and color $c$. Every $A_{j}$ is an endpoint of at most 1 chord colored $c, j=1, \ldots, i$. Hence at most $i$ chords of color $c$ intersect $\ell$. + +It follows that for each $i=0,1, \ldots, n-2$ the $2 n$ chords $\ell$ with $m(\ell)=i$ contribute at least $\frac{2 n}{i+1}$ to the sum $\sum_{c} f\left(G_{c}\right)$. Summation over $i=0,1, \ldots, n-2$ gives + +$$ +\sum_{c} f\left(G_{c}\right) \geq 2 n \sum_{i=1}^{n-1} \frac{1}{i} +$$ + +Because there are $4 n-3$ colors in all, averaging yields a color $c$ such that + +$$ +f\left(G_{c}\right) \geq \frac{2 n}{4 n-3} \sum_{i=1}^{n-1} \frac{1}{i}>\frac{1}{2} \sum_{i=1}^{n-1} \frac{1}{i} +$$ + +By the lemma there are at least $\frac{1}{2} \sum_{i=1}^{n-1} \frac{1}{i}$ pairwise disjoint chords of color $c$, i. e. with the same sum $c$ of the pairs of numbers at their endpoints. It remains to show that $\frac{1}{2} \sum_{i=1}^{n-1} \frac{1}{i} \geq 100$ for $n=2^{499}$. Indeed we have + +$$ +\sum_{i=1}^{n-1} \frac{1}{i}>\sum_{i=1}^{2^{400}} \frac{1}{i}=1+\sum_{k=1}^{400} \sum_{i=2^{k-1+1}}^{2^{k}} \frac{1}{i}>1+\sum_{k=1}^{400} \frac{2^{k-1}}{2^{k}}=201>200 +$$ + +This completes the solution. + +## Geometry + +G1. In the triangle $A B C$ the point $J$ is the center of the excircle opposite to $A$. This excircle is tangent to the side $B C$ at $M$, and to the lines $A B$ and $A C$ at $K$ and $L$ respectively. The lines $L M$ and $B J$ meet at $F$, and the lines $K M$ and $C J$ meet at $G$. Let $S$ be the point of intersection of the lines $A F$ and $B C$, and let $T$ be the point of intersection of the lines $A G$ and $B C$. Prove that $M$ is the midpoint of $S T$. + +Solution. Let $\alpha=\angle C A B, \beta=\angle A B C$ and $\gamma=\angle B C A$. The line $A J$ is the bisector of $\angle C A B$, so $\angle J A K=\angle J A L=\frac{\alpha}{2}$. By $\angle A K J=\angle A L J=90^{\circ}$ the points $K$ and $L$ lie on the circle $\omega$ with diameter $A J$. + +The triangle $K B M$ is isosceles as $B K$ and $B M$ are tangents to the excircle. Since $B J$ is the bisector of $\angle K B M$, we have $\angle M B J=90^{\circ}-\frac{\beta}{2}$ and $\angle B M K=\frac{\beta}{2}$. Likewise $\angle M C J=90^{\circ}-\frac{\gamma}{2}$ and $\angle C M L=\frac{\gamma}{2}$. Also $\angle B M F=\angle C M L$, therefore + +$$ +\angle L F J=\angle M B J-\angle B M F=\left(90^{\circ}-\frac{\beta}{2}\right)-\frac{\gamma}{2}=\frac{\alpha}{2}=\angle L A J . +$$ + +Hence $F$ lies on the circle $\omega$. (By the angle computation, $F$ and $A$ are on the same side of $B C$.) Analogously, $G$ also lies on $\omega$. Since $A J$ is a diameter of $\omega$, we obtain $\angle A F J=\angle A G J=90^{\circ}$. + +![](https://cdn.mathpix.com/cropped/2024_04_17_93f7104904c3717a5058g-29.jpg?height=703&width=1242&top_left_y=1276&top_left_x=407) + +The lines $A B$ and $B C$ are symmetric with respect to the external bisector $B F$. Because $A F \perp B F$ and $K M \perp B F$, the segments $S M$ and $A K$ are symmetric with respect to $B F$, hence $S M=A K$. By symmetry $T M=A L$. Since $A K$ and $A L$ are equal as tangents to the excircle, it follows that $S M=T M$, and the proof is complete. + +Comment. After discovering the circle $A F K J L G$, there are many other ways to complete the solution. For instance, from the cyclic quadrilaterals $J M F S$ and $J M G T$ one can find $\angle T S J=\angle S T J=\frac{\alpha}{2}$. Another possibility is to use the fact that the lines $A S$ and $G M$ are parallel (both are perpendicular to the external angle bisector $B J$ ), so $\frac{M S}{M T}=\frac{A G}{G T}=1$. + +G2. Let $A B C D$ be a cyclic quadrilateral whose diagonals $A C$ and $B D$ meet at $E$. The extensions of the sides $A D$ and $B C$ beyond $A$ and $B$ meet at $F$. Let $G$ be the point such that $E C G D$ is a parallelogram, and let $H$ be the image of $E$ under reflection in $A D$. Prove that $D, H, F, G$ are concyclic. + +Solution. We show first that the triangles $F D G$ and $F B E$ are similar. Since $A B C D$ is cyclic, the triangles $E A B$ and $E D C$ are similar, as well as $F A B$ and $F C D$. The parallelogram $E C G D$ yields $G D=E C$ and $\angle C D G=\angle D C E$; also $\angle D C E=\angle D C A=\angle D B A$ by inscribed angles. Therefore + +$$ +\begin{gathered} +\angle F D G=\angle F D C+\angle C D G=\angle F B A+\angle A B D=\angle F B E, \\ +\frac{G D}{E B}=\frac{C E}{E B}=\frac{C D}{A B}=\frac{F D}{F B} . +\end{gathered} +$$ + +It follows that $F D G$ and $F B E$ are similar, and so $\angle F G D=\angle F E B$. + +![](https://cdn.mathpix.com/cropped/2024_04_17_93f7104904c3717a5058g-30.jpg?height=954&width=974&top_left_y=948&top_left_x=538) + +Since $H$ is the reflection of $E$ with respect to $F D$, we conclude that + +$$ +\angle F H D=\angle F E D=180^{\circ}-\angle F E B=180^{\circ}-\angle F G D . +$$ + +This proves that $D, H, F, G$ are concyclic. + +Comment. Points $E$ and $G$ are always in the half-plane determined by the line $F D$ that contains $B$ and $C$, but $H$ is always in the other half-plane. In particular, $D H F G$ is cyclic if and only if $\angle F H D+\angle F G D=180^{\circ}$. + +G3. In an acute triangle $A B C$ the points $D, E$ and $F$ are the feet of the altitudes through $A$, $B$ and $C$ respectively. The incenters of the triangles $A E F$ and $B D F$ are $I_{1}$ and $I_{2}$ respectively; the circumcenters of the triangles $A C I_{1}$ and $B C I_{2}$ are $O_{1}$ and $O_{2}$ respectively. Prove that $I_{1} I_{2}$ and $O_{1} O_{2}$ are parallel. + +Solution. Let $\angle C A B=\alpha, \angle A B C=\beta, \angle B C A=\gamma$. We start by showing that $A, B, I_{1}$ and $I_{2}$ are concyclic. Since $A I_{1}$ and $B I_{2}$ bisect $\angle C A B$ and $\angle A B C$, their extensions beyond $I_{1}$ and $I_{2}$ meet at the incenter $I$ of the triangle. The points $E$ and $F$ are on the circle with diameter $B C$, so $\angle A E F=\angle A B C$ and $\angle A F E=\angle A C B$. Hence the triangles $A E F$ and $A B C$ are similar with ratio of similitude $\frac{A E}{A B}=\cos \alpha$. Because $I_{1}$ and $I$ are their incenters, we obtain $I_{1} A=I A \cos \alpha$ and $I I_{1}=I A-I_{1} A=2 I A \sin ^{2} \frac{\alpha}{2}$. By symmetry $I I_{2}=2 I B \sin ^{2} \frac{\beta}{2}$. The law of sines in the triangle $A B I$ gives $I A \sin \frac{\alpha}{2}=I B \sin \frac{\beta}{2}$. Hence + +$$ +I I_{1} \cdot I A=2\left(I A \sin \frac{\alpha}{2}\right)^{2}=2\left(I B \sin \frac{\beta}{2}\right)^{2}=I I_{2} \cdot I B +$$ + +Therefore $A, B, I_{1}$ and $I_{2}$ are concyclic, as claimed. + +![](https://cdn.mathpix.com/cropped/2024_04_17_93f7104904c3717a5058g-31.jpg?height=848&width=1466&top_left_y=998&top_left_x=295) + +In addition $I I_{1} \cdot I A=I I_{2} \cdot I B$ implies that $I$ has the same power with respect to the circles $\left(A C I_{1}\right),\left(B C I_{2}\right)$ and $\left(A B I_{1} I_{2}\right)$. Then $C I$ is the radical axis of $\left(A C I_{1}\right)$ and $\left(B C I_{2}\right)$; in particular $C I$ is perpendicular to the line of centers $O_{1} O_{2}$. + +Now it suffices to prove that $C I \perp I_{1} I_{2}$. Let $C I$ meet $I_{1} I_{2}$ at $Q$, then it is enough to check that $\angle I I_{1} Q+\angle I_{1} I Q=90^{\circ}$. Since $\angle I_{1} I Q$ is external for the triangle $A C I$, we have + +$$ +\angle I I_{1} Q+\angle I_{1} I Q=\angle I I_{1} Q+(\angle A C I+\angle C A I)=\angle I I_{1} I_{2}+\angle A C I+\angle C A I . +$$ + +It remains to note that $\angle I I_{1} I_{2}=\frac{\beta}{2}$ from the cyclic quadrilateral $A B I_{1} I_{2}$, and $\angle A C I=\frac{\gamma}{2}$, $\angle C A I=\frac{\alpha}{2}$. Therefore $\angle I I_{1} Q+\angle I_{1} I Q=\frac{\alpha}{2}+\frac{\beta}{2}+\frac{\gamma}{2}=90^{\circ}$, completing the proof. + +Comment. It follows from the first part of the solution that the common point $I_{3} \neq C$ of the circles $\left(A C I_{1}\right)$ and $\left(B C I_{2}\right)$ is the incenter of the triangle $C D E$. + +G4. Let $A B C$ be a triangle with $A B \neq A C$ and circumcenter $O$. The bisector of $\angle B A C$ intersects $B C$ at $D$. Let $E$ be the reflection of $D$ with respect to the midpoint of $B C$. The lines through $D$ and $E$ perpendicular to $B C$ intersect the lines $A O$ and $A D$ at $X$ and $Y$ respectively. Prove that the quadrilateral $B X C Y$ is cyclic. + +Solution. The bisector of $\angle B A C$ and the perpendicular bisector of $B C$ meet at $P$, the midpoint of the minor arc $\widehat{B C}$ (they are different lines as $A B \neq A C$ ). In particular $O P$ is perpendicular to $B C$ and intersects it at $M$, the midpoint of $B C$. + +Denote by $Y^{\prime}$ the reflexion of $Y$ with respect to $O P$. Since $\angle B Y C=\angle B Y^{\prime} C$, it suffices to prove that $B X C Y^{\prime}$ is cyclic. + +![](https://cdn.mathpix.com/cropped/2024_04_17_93f7104904c3717a5058g-32.jpg?height=1039&width=826&top_left_y=720&top_left_x=615) + +We have + +$$ +\angle X A P=\angle O P A=\angle E Y P \text {. } +$$ + +The first equality holds because $O A=O P$, and the second one because $E Y$ and $O P$ are both perpendicular to $B C$ and hence parallel. But $\left\{Y, Y^{\prime}\right\}$ and $\{E, D\}$ are pairs of symmetric points with respect to $O P$, it follows that $\angle E Y P=\angle D Y^{\prime} P$ and hence + +$$ +\angle X A P=\angle D Y^{\prime} P=\angle X Y^{\prime} P . +$$ + +The last equation implies that $X A Y^{\prime} P$ is cyclic. By the powers of $D$ with respect to the circles $\left(X A Y^{\prime} P\right)$ and $(A B P C)$ we obtain + +$$ +X D \cdot D Y^{\prime}=A D \cdot D P=B D \cdot D C +$$ + +It follows that $B X C Y^{\prime}$ is cyclic, as desired. + +G5. Let $A B C$ be a triangle with $\angle B C A=90^{\circ}$, and let $C_{0}$ be the foot of the altitude from $C$. Choose a point $X$ in the interior of the segment $C C_{0}$, and let $K, L$ be the points on the segments $A X, B X$ for which $B K=B C$ and $A L=A C$ respectively. Denote by $M$ the intersection of $A L$ and $B K$. Show that $M K=M L$. + +Solution. Let $C^{\prime}$ be the reflection of $C$ in the line $A B$, and let $\omega_{1}$ and $\omega_{2}$ be the circles with centers $A$ and $B$, passing through $L$ and $K$ respectively. Since $A C^{\prime}=A C=A L$ and $B C^{\prime}=B C=B K$, both $\omega_{1}$ and $\omega_{2}$ pass through $C$ and $C^{\prime}$. By $\angle B C A=90^{\circ}, A C$ is tangent to $\omega_{2}$ at $C$, and $B C$ is tangent to $\omega_{1}$ at $C$. Let $K_{1} \neq K$ be the second intersection of $A X$ and $\omega_{2}$, and let $L_{1} \neq L$ be the second intersection of $B X$ and $\omega_{1}$. + +![](https://cdn.mathpix.com/cropped/2024_04_17_93f7104904c3717a5058g-33.jpg?height=1051&width=1016&top_left_y=688&top_left_x=520) + +By the powers of $X$ with respect to $\omega_{2}$ and $\omega_{1}$, + +$$ +X K \cdot X K_{1}=X C \cdot X C^{\prime}=X L \cdot X L_{1}, +$$ + +so the points $K_{1}, L, K, L_{1}$ lie on a circle $\omega_{3}$. + +The power of $A$ with respect to $\omega_{2}$ gives + +$$ +A L^{2}=A C^{2}=A K \cdot A K_{1}, +$$ + +indicating that $A L$ is tangent to $\omega_{3}$ at $L$. Analogously, $B K$ is tangent to $\omega_{3}$ at $K$. Hence $M K$ and $M L$ are the two tangents from $M$ to $\omega_{3}$ and therefore $M K=M L$. + +G6. Let $A B C$ be a triangle with circumcenter $O$ and incenter $I$. The points $D, E$ and $F$ on the sides $B C, C A$ and $A B$ respectively are such that $B D+B F=C A$ and $C D+C E=A B$. The circumcircles of the triangles $B F D$ and $C D E$ intersect at $P \neq D$. Prove that $O P=O I$. + +Solution. By Miquel's theorem the circles $(A E F)=\omega_{A},(B F D)=\omega_{B}$ and $(C D E)=\omega_{C}$ have a common point, for arbitrary points $D, E$ and $F$ on $B C, C A$ and $A B$. So $\omega_{A}$ passes through the common point $P \neq D$ of $\omega_{B}$ and $\omega_{C}$. + +Let $\omega_{A}, \omega_{B}$ and $\omega_{C}$ meet the bisectors $A I, B I$ and $C I$ at $A \neq A^{\prime}, B \neq B^{\prime}$ and $C \neq C^{\prime}$ respectively. The key observation is that $A^{\prime}, B^{\prime}$ and $C^{\prime}$ do not depend on the particular choice of $D, E$ and $F$, provided that $B D+B F=C A, C D+C E=A B$ and $A E+A F=B C$ hold true (the last equality follows from the other two). For a proof we need the following fact. + +Lemma. Given is an angle with vertex $A$ and measure $\alpha$. A circle $\omega$ through $A$ intersects the angle bisector at $L$ and sides of the angle at $X$ and $Y$. Then $A X+A Y=2 A L \cos \frac{\alpha}{2}$. + +Proof. Note that $L$ is the midpoint of $\operatorname{arc} \widehat{X L Y}$ in $\omega$ and set $X L=Y L=u, X Y=v$. By PtOLEMY's theorem $A X \cdot Y L+A Y \cdot X L=A L \cdot X Y$, which rewrites as $(A X+A Y) u=A L \cdot v$. Since $\angle L X Y=\frac{\alpha}{2}$ and $\angle X L Y=180^{\circ}-\alpha$, we have $v=2 \cos \frac{\alpha}{2} u$ by the law of sines, and the claim follows. + +![](https://cdn.mathpix.com/cropped/2024_04_17_93f7104904c3717a5058g-34.jpg?height=551&width=671&top_left_y=1118&top_left_x=698) + +Apply the lemma to $\angle B A C=\alpha$ and the circle $\omega=\omega_{A}$, which intersects $A I$ at $A^{\prime}$. This gives $2 A A^{\prime} \cos \frac{\alpha}{2}=A E+A F=B C$; by symmetry analogous relations hold for $B B^{\prime}$ and $C C^{\prime}$. It follows that $A^{\prime}, B^{\prime}$ and $C^{\prime}$ are independent of the choice of $D, E$ and $F$, as stated. + +We use the lemma two more times with $\angle B A C=\alpha$. Let $\omega$ be the circle with diameter $A I$. Then $X$ and $Y$ are the tangency points of the incircle of $A B C$ with $A B$ and $A C$, and hence $A X=A Y=\frac{1}{2}(A B+A C-B C)$. So the lemma yields $2 A I \cos \frac{\alpha}{2}=A B+A C-B C$. Next, if $\omega$ is the circumcircle of $A B C$ and $A I$ intersects $\omega$ at $M \neq A$ then $\{X, Y\}=\{B, C\}$, and so $2 A M \cos \frac{\alpha}{2}=A B+A C$ by the lemma. To summarize, + +$$ +2 A A^{\prime} \cos \frac{\alpha}{2}=B C, \quad 2 A I \cos \frac{\alpha}{2}=A B+A C-B C, \quad 2 A M \cos \frac{\alpha}{2}=A B+A C +$$ + +These equalities imply $A A^{\prime}+A I=A M$, hence the segments $A M$ and $I A^{\prime}$ have a common midpoint. It follows that $I$ and $A^{\prime}$ are equidistant from the circumcenter $O$. By symmetry $O I=O A^{\prime}=O B^{\prime}=O C^{\prime}$, so $I, A^{\prime}, B^{\prime}, C^{\prime}$ are on a circle centered at $O$. + +To prove $O P=O I$, now it suffices to show that $I, A^{\prime}, B^{\prime}, C^{\prime}$ and $P$ are concyclic. Clearly one can assume $P \neq I, A^{\prime}, B^{\prime}, C^{\prime}$. + +We use oriented angles to avoid heavy case distinction. The oriented angle between the lines $l$ and $m$ is denoted by $\angle(l, m)$. We have $\angle(l, m)=-\angle(m, l)$ and $\angle(l, m)+\angle(m, n)=\angle(l, n)$ for arbitrary lines $l, m$ and $n$. Four distinct non-collinear points $U, V, X, Y$ are concyclic if and only if $\angle(U X, V X)=\angle(U Y, V Y)$. + +![](https://cdn.mathpix.com/cropped/2024_04_17_93f7104904c3717a5058g-35.jpg?height=1048&width=1059&top_left_y=184&top_left_x=493) + +Suppose for the moment that $A^{\prime}, B^{\prime}, P, I$ are distinct and noncollinear; then it is enough to check the equality $\angle\left(A^{\prime} P, B^{\prime} P\right)=\angle\left(A^{\prime} I, B^{\prime} I\right)$. Because $A, F, P, A^{\prime}$ are on the circle $\omega_{A}$, we have $\angle\left(A^{\prime} P, F P\right)=\angle\left(A^{\prime} A, F A\right)=\angle\left(A^{\prime} I, A B\right)$. Likewise $\angle\left(B^{\prime} P, F P\right)=\angle\left(B^{\prime} I, A B\right)$. Therefore + +$$ +\angle\left(A^{\prime} P, B^{\prime} P\right)=\angle\left(A^{\prime} P, F P\right)+\angle\left(F P, B^{\prime} P\right)=\angle\left(A^{\prime} I, A B\right)-\angle\left(B^{\prime} I, A B\right)=\angle\left(A^{\prime} I, B^{\prime} I\right) \text {. } +$$ + +Here we assumed that $P \neq F$. If $P=F$ then $P \neq D, E$ and the conclusion follows similarly (use $\angle\left(A^{\prime} F, B^{\prime} F\right)=\angle\left(A^{\prime} F, E F\right)+\angle(E F, D F)+\angle\left(D F, B^{\prime} F\right)$ and inscribed angles in $\left.\omega_{A}, \omega_{B}, \omega_{C}\right)$. + +There is no loss of generality in assuming $A^{\prime}, B^{\prime}, P, I$ distinct and noncollinear. If $A B C$ is an equilateral triangle then the equalities $\left(^{*}\right)$ imply that $A^{\prime}, B^{\prime}, C^{\prime}, I, O$ and $P$ coincide, so $O P=O I$. Otherwise at most one of $A^{\prime}, B^{\prime}, C^{\prime}$ coincides with $I$. If say $C^{\prime}=I$ then $O I \perp C I$ by the previous reasoning. It follows that $A^{\prime}, B^{\prime} \neq I$ and hence $A^{\prime} \neq B^{\prime}$. Finally $A^{\prime}, B^{\prime}$ and $I$ are noncollinear because $I, A^{\prime}, B^{\prime}, C^{\prime}$ are concyclic. + +Comment. The proposer remarks that the locus $\gamma$ of the points $P$ is an arc of the circle $\left(A^{\prime} B^{\prime} C^{\prime} I\right)$. The reflection $I^{\prime}$ of $I$ in $O$ belongs to $\gamma$; it is obtained by choosing $D, E$ and $F$ to be the tangency points of the three excircles with their respective sides. The rest of the circle $\left(A^{\prime} B^{\prime} C^{\prime} I\right)$, except $I$, can be included in $\gamma$ by letting $D, E$ and $F$ vary on the extensions of the sides and assuming signed lengths. For instance if $B$ is between $C$ and $D$ then the length $B D$ must be taken with a negative sign. The incenter $I$ corresponds to the limit case where $D$ tends to infinity. + +G7. Let $A B C D$ be a convex quadrilateral with non-parallel sides $B C$ and $A D$. Assume that there is a point $E$ on the side $B C$ such that the quadrilaterals $A B E D$ and $A E C D$ are circumscribed. Prove that there is a point $F$ on the side $A D$ such that the quadrilaterals $A B C F$ and $B C D F$ are circumscribed if and only if $A B$ is parallel to $C D$. + +Solution. Let $\omega_{1}$ and $\omega_{2}$ be the incircles and $O_{1}$ and $O_{2}$ the incenters of the quadrilaterals $A B E D$ and $A E C D$ respectively. A point $F$ with the stated property exists only if $\omega_{1}$ and $\omega_{2}$ are also the incircles of the quadrilaterals $A B C F$ and $B C D F$. + +![](https://cdn.mathpix.com/cropped/2024_04_17_93f7104904c3717a5058g-36.jpg?height=494&width=868&top_left_y=593&top_left_x=591) + +Let the tangents from $B$ to $\omega_{2}$ and from $C$ to $\omega_{1}$ (other than $B C$ ) meet $A D$ at $F_{1}$ and $F_{2}$ respectively. We need to prove that $F_{1}=F_{2}$ if and only if $A B \| C D$. + +Lemma. The circles $\omega_{1}$ and $\omega_{2}$ with centers $O_{1}$ and $O_{2}$ are inscribed in an angle with vertex $O$. The points $P, S$ on one side of the angle and $Q, R$ on the other side are such that $\omega_{1}$ is the incircle of the triangle $P Q O$, and $\omega_{2}$ is the excircle of the triangle $R S O$ opposite to $O$. Denote $p=O O_{1} \cdot O O_{2}$. Then exactly one of the following relations holds: + +$$ +O P \cdot O Rp>O Q \cdot O S, \quad O P \cdot O R=p=O Q \cdot O S +$$ + +Proof. Denote $\angle O P O_{1}=u, \angle O Q O_{1}=v, \angle O O_{2} R=x, \angle O O_{2} S=y, \angle P O Q=2 \varphi$. Because $P O_{1}, Q O_{1}, R O_{2}, S O_{2}$ are internal or external bisectors in the triangles $P Q O$ and $R S O$, we have + +$$ +u+v=x+y\left(=90^{\circ}-\varphi\right) . +$$ + +By the law of sines + +![](https://cdn.mathpix.com/cropped/2024_04_17_93f7104904c3717a5058g-36.jpg?height=417&width=880&top_left_y=1756&top_left_x=588) + +$$ +\frac{O P}{O O_{1}}=\frac{\sin (u+\varphi)}{\sin u} \quad \text { and } \quad \frac{O O_{2}}{O R}=\frac{\sin (x+\varphi)}{\sin x} +$$ + +Therefore, since $x, u$ and $\varphi$ are acute, + +$O P \cdot O R \geq p \Leftrightarrow \frac{O P}{O O_{1}} \geq \frac{O O_{2}}{O R} \Leftrightarrow \sin x \sin (u+\varphi) \geq \sin u \sin (x+\varphi) \Leftrightarrow \sin (x-u) \geq 0 \Leftrightarrow x \geq u$. + +Thus $O P \cdot O R \geq p$ is equivalent to $x \geq u$, with $O P \cdot O R=p$ if and only if $x=u$. + +Analogously, $p \geq O Q \cdot O S$ is equivalent to $v \geq y$, with $p=O Q \cdot O S$ if and only if $v=y$. On the other hand $x \geq u$ and $v \geq y$ are equivalent by (1), with $x=u$ if and only if $v=y$. The conclusion of the lemma follows from here. + +Going back to the problem, apply the lemma to the quadruples $\left\{B, E, D, F_{1}\right\},\{A, B, C, D\}$ and $\left\{A, E, C, F_{2}\right\}$. Assuming $O E \cdot O F_{1}>p$, we obtain + +$$ +O E \cdot O F_{1}>p \Rightarrow O B \cdot O D

p \Rightarrow O E \cdot O F_{2}

p$ implies + +$$ +O B \cdot O Dp>O E \cdot O F_{2} \text {. } +$$ + +Similarly, $O E \cdot O F_{1}p>O A \cdot O C \text { and } O E \cdot O F_{1}s>O^{\prime \prime} Q+O^{\prime} S, \quad O^{\prime} P+O^{\prime \prime} R=s=O^{\prime \prime} Q+O^{\prime} S . +$$ + +![](https://cdn.mathpix.com/cropped/2024_04_17_93f7104904c3717a5058g-37.jpg?height=300&width=780&top_left_y=1403&top_left_x=638) + +Once this is established, the proof of the original statement for $B C \| A D$ is analogous to the one in the intersecting case. One replaces products by sums of relevant segments. + +G8. Let $A B C$ be a triangle with circumcircle $\omega$ and $\ell$ a line without common points with $\omega$. Denote by $P$ the foot of the perpendicular from the center of $\omega$ to $\ell$. The side-lines $B C, C A, A B$ intersect $\ell$ at the points $X, Y, Z$ different from $P$. Prove that the circumcircles of the triangles $A X P, B Y P$ and $C Z P$ have a common point different from $P$ or are mutually tangent at $P$. + +Solution 1. Let $\omega_{A}, \omega_{B}, \omega_{C}$ and $\omega$ be the circumcircles of triangles $A X P, B Y P, C Z P$ and $A B C$ respectively. The strategy of the proof is to construct a point $Q$ with the same power with respect to the four circles. Then each of $P$ and $Q$ has the same power with respect to $\omega_{A}, \omega_{B}, \omega_{C}$ and hence the three circles are coaxial. In other words they have another common point $P^{\prime}$ or the three of them are tangent at $P$. + +We first give a description of the point $Q$. Let $A^{\prime} \neq A$ be the second intersection of $\omega$ and $\omega_{A}$; define $B^{\prime}$ and $C^{\prime}$ analogously. We claim that $A A^{\prime}, B B^{\prime}$ and $C C^{\prime}$ have a common point. Once this claim is established, the point just constructed will be on the radical axes of the three pairs of circles $\left\{\omega, \omega_{A}\right\},\left\{\omega, \omega_{B}\right\},\left\{\omega, \omega_{C}\right\}$. Hence it will have the same power with respect to $\omega, \omega_{A}, \omega_{B}, \omega_{C}$. + +![](https://cdn.mathpix.com/cropped/2024_04_17_93f7104904c3717a5058g-38.jpg?height=783&width=1242&top_left_y=965&top_left_x=407) + +We proceed to prove that $A A^{\prime}, B B^{\prime}$ and $C C^{\prime}$ intersect at one point. Let $r$ be the circumradius of triangle $A B C$. Define the points $X^{\prime}, Y^{\prime}, Z^{\prime}$ as the intersections of $A A^{\prime}, B B^{\prime}, C C^{\prime}$ with $\ell$. Observe that $X^{\prime}, Y^{\prime}, Z^{\prime}$ do exist. If $A A^{\prime}$ is parallel to $\ell$ then $\omega_{A}$ is tangent to $\ell$; hence $X=P$ which is a contradiction. Similarly, $B B^{\prime}$ and $C C^{\prime}$ are not parallel to $\ell$. + +From the powers of the point $X^{\prime}$ with respect to the circles $\omega_{A}$ and $\omega$ we get + +$$ +X^{\prime} P \cdot\left(X^{\prime} P+P X\right)=X^{\prime} P \cdot X^{\prime} X=X^{\prime} A^{\prime} \cdot X^{\prime} A=X^{\prime} O^{2}-r^{2} +$$ + +hence + +$$ +X^{\prime} P \cdot P X=X^{\prime} O^{2}-r^{2}-X^{\prime} P^{2}=O P^{2}-r^{2} . +$$ + +We argue analogously for the points $Y^{\prime}$ and $Z^{\prime}$, obtaining + +$$ +X^{\prime} P \cdot P X=Y^{\prime} P \cdot P Y=Z^{\prime} P \cdot P Z=O P^{2}-r^{2}=k^{2} +$$ + +In these computations all segments are regarded as directed segments. We keep the same convention for the sequel. + +We prove that the lines $A A^{\prime}, B B^{\prime}, C C^{\prime}$ intersect at one point by CEVA's theorem. To avoid distracting remarks we interpret everything projectively, i. e. whenever two lines are parallel they meet at a point on the line at infinity. + +Let $U, V, W$ be the intersections of $A A^{\prime}, B B^{\prime}, C C^{\prime}$ with $B C, C A, A B$ respectively. The idea is that although it is difficult to calculate the ratio $\frac{B U}{C U}$, it is easier to deal with the cross-ratio $\frac{B U}{C U} / \frac{B X}{C X}$ because we can send it to the line $\ell$. With this in mind we apply MEnELaus' theorem to the triangle $A B C$ and obtain $\frac{B X}{C X} \cdot \frac{C Y}{A Y} \cdot \frac{A Z}{B Z}=1$. Hence Ceva's ratio can be expressed as + +$$ +\frac{B U}{C U} \cdot \frac{C V}{A V} \cdot \frac{A W}{B W}=\frac{B U}{C U} / \frac{B X}{C X} \cdot \frac{C V}{A V} / \frac{C Y}{A Y} \cdot \frac{A W}{B W} / \frac{A Z}{B Z} +$$ + +![](https://cdn.mathpix.com/cropped/2024_04_17_93f7104904c3717a5058g-39.jpg?height=594&width=1145&top_left_y=565&top_left_x=453) + +Project the line $B C$ to $\ell$ from $A$. The cross-ratio between $B C$ and $U X$ equals the cross-ratio between $Z Y$ and $X^{\prime} X$. Repeating the same argument with the lines $C A$ and $A B$ gives + +$$ +\frac{B U}{C U} \cdot \frac{C V}{A V} \cdot \frac{A W}{B W}=\frac{Z X^{\prime}}{Y X^{\prime}} / \frac{Z X}{Y X} \cdot \frac{X Y^{\prime}}{Z Y^{\prime}} / \frac{X Y}{Z Y} \cdot \frac{Y Z^{\prime}}{X Z^{\prime}} / \frac{Y Z}{X Z} +$$ + +and hence + +$$ +\frac{B U}{C U} \cdot \frac{C V}{A V} \cdot \frac{A W}{B W}=(-1) \cdot \frac{Z X^{\prime}}{Y X^{\prime}} \cdot \frac{X Y^{\prime}}{Z Y^{\prime}} \cdot \frac{Y Z^{\prime}}{X Z^{\prime}} +$$ + +The equations (1) reduce the problem to a straightforward computation on the line $\ell$. For instance, the transformation $t \mapsto-k^{2} / t$ preserves cross-ratio and interchanges the points $X, Y, Z$ with the points $X^{\prime}, Y^{\prime}, Z^{\prime}$. Then + +$$ +\frac{B U}{C U} \cdot \frac{C V}{A V} \cdot \frac{A W}{B W}=(-1) \cdot \frac{Z X^{\prime}}{Y X^{\prime}} / \frac{Z Z^{\prime}}{Y Z^{\prime}} \cdot \frac{X Y^{\prime}}{Z Y^{\prime}} / \frac{X Z^{\prime}}{Z Z^{\prime}}=-1 +$$ + +We proved that CEvA's ratio equals -1 , so $A A^{\prime}, B B^{\prime}, C C^{\prime}$ intersect at one point $Q$. + +Comment 1. There is a nice projective argument to prove that $A X^{\prime}, B Y^{\prime}, C Z^{\prime}$ intersect at one point. Suppose that $\ell$ and $\omega$ intersect at a pair of complex conjugate points $D$ and $E$. Consider a projective transformation that takes $D$ and $E$ to $[i ; 1,0]$ and $[-i, 1,0]$. Then $\ell$ is the line at infinity, and $\omega$ is a conic through the special points $[i ; 1,0]$ and $[-i, 1,0]$, hence it is a circle. So one can assume that $A X, B Y, C Z$ are parallel to $B C, C A, A B$. The involution on $\ell$ taking $X, Y, Z$ to $X^{\prime}, Y^{\prime}, Z^{\prime}$ and leaving $D, E$ fixed is the involution changing each direction to its perpendicular one. Hence $A X, B Y, C Z$ are also perpendicular to $A X^{\prime}, B Y^{\prime}, C Z^{\prime}$. + +It follows from the above that $A X^{\prime}, B Y^{\prime}, C Z^{\prime}$ intersect at the orthocenter of triangle $A B C$. + +Comment 2. The restriction that the line $\ell$ does not intersect the circumcricle $\omega$ is unnecessary. The proof above works in general. In case $\ell$ intersects $\omega$ at $D$ and $E$ point $P$ is the midpoint of $D E$, and some equations can be interpreted differently. For instance + +$$ +X^{\prime} P \cdot X^{\prime} X=X^{\prime} A^{\prime} \cdot X^{\prime} A=X^{\prime} D \cdot X^{\prime} E +$$ + +and hence the pairs $X^{\prime} X$ and $D E$ are harmonic conjugates. This means that $X^{\prime}, Y^{\prime}, Z^{\prime}$ are the harmonic conjugates of $X, Y, Z$ with respect to the segment $D E$. + +Solution 2. First we prove that there is an inversion in space that takes $\ell$ and $\omega$ to parallel circles on a sphere. Let $Q R$ be the diameter of $\omega$ whose extension beyond $Q$ passes through $P$. Let $\Pi$ be the plane carrying our objects. In space, choose a point $O$ such that the line $Q O$ is perpendicular to $\Pi$ and $\angle P O R=90^{\circ}$, and apply an inversion with pole $O$ (the radius of the inversion does not matter). For any object $\mathcal{T}$ denote by $\mathcal{T}^{\prime}$ the image of $\mathcal{T}$ under this inversion. + +The inversion takes the plane $\Pi$ to a sphere $\Pi^{\prime}$. The lines in $\Pi$ are taken to circles through $O$, and the circles in $\Pi$ also are taken to circles on $\Pi^{\prime}$. + +![](https://cdn.mathpix.com/cropped/2024_04_17_93f7104904c3717a5058g-40.jpg?height=491&width=1288&top_left_y=560&top_left_x=384) + +Since the line $\ell$ and the circle $\omega$ are perpendicular to the plane $O P Q$, the circles $\ell^{\prime}$ and $\omega^{\prime}$ also are perpendicular to this plane. Hence, the planes of the circles $\ell^{\prime}$ and $\omega^{\prime}$ are parallel. + +Now consider the circles $A^{\prime} X^{\prime} P^{\prime}, B^{\prime} Y^{\prime} P^{\prime}$ and $C^{\prime} Z^{\prime} P^{\prime}$. We want to prove that either they have a common point (on $\Pi^{\prime}$ ), different from $P^{\prime}$, or they are tangent to each other. + +![](https://cdn.mathpix.com/cropped/2024_04_17_93f7104904c3717a5058g-40.jpg?height=819&width=870&top_left_y=1321&top_left_x=593) + +The point $X^{\prime}$ is the second intersection of the circles $B^{\prime} C^{\prime} O$ and $\ell^{\prime}$, other than $O$. Hence, the lines $O X^{\prime}$ and $B^{\prime} C^{\prime}$ are coplanar. Moreover, they lie in the parallel planes of $\ell^{\prime}$ and $\omega^{\prime}$. Therefore, $O X^{\prime}$ and $B^{\prime} C^{\prime}$ are parallel. Analogously, $O Y^{\prime}$ and $O Z^{\prime}$ are parallel to $A^{\prime} C^{\prime}$ and $A^{\prime} B^{\prime}$. + +Let $A_{1}$ be the second intersection of the circles $A^{\prime} X^{\prime} P^{\prime}$ and $\omega^{\prime}$, other than $A^{\prime}$. The segments $A^{\prime} A_{1}$ and $P^{\prime} X^{\prime}$ are coplanar, and therefore parallel. Now we know that $B^{\prime} C^{\prime}$ and $A^{\prime} A_{1}$ are parallel to $O X^{\prime}$ and $X^{\prime} P^{\prime}$ respectively, but these two segments are perpendicular because $O P^{\prime}$ is a diameter in $\ell^{\prime}$. We found that $A^{\prime} A_{1}$ and $B^{\prime} C^{\prime}$ are perpendicular, hence $A^{\prime} A_{1}$ is the altitude in the triangle $A^{\prime} B^{\prime} C^{\prime}$, starting from $A$. + +Analogously, let $B_{1}$ and $C_{1}$ be the second intersections of $\omega^{\prime}$ with the circles $B^{\prime} P^{\prime} Y^{\prime}$ and $C^{\prime} P^{\prime} Z^{\prime}$, other than $B^{\prime}$ and $C^{\prime}$ respectively. Then $B^{\prime} B_{1}$ and $C^{\prime} C_{1}$ are the other two altitudes in the triangle $A^{\prime} B^{\prime} C^{\prime}$. + +Let $H$ be the orthocenter of the triangle $A^{\prime} B^{\prime} C^{\prime}$. Let $W$ be the second intersection of the line $P^{\prime} H$ with the sphere $\Pi^{\prime}$, other than $P^{\prime}$. The point $W$ lies on the sphere $\Pi^{\prime}$, in the plane of the circle $A^{\prime} P^{\prime} X^{\prime}$, so $W$ lies on the circle $A^{\prime} P^{\prime} X^{\prime}$. Similarly, $W$ lies on the circles $B^{\prime} P^{\prime} Y^{\prime}$ and $C^{\prime} P^{\prime} Z^{\prime}$ as well; indeed $W$ is the second common point of the three circles. + +If the line $P^{\prime} H$ is tangent to the sphere then $W$ coincides with $P^{\prime}$, and $P^{\prime} H$ is the common tangent of the three circles. + +## Number Theory + +N1. Call admissible a set $A$ of integers that has the following property: + +$$ +\text { If } x, y \in A \text { (possibly } x=y \text { ) then } x^{2}+k x y+y^{2} \in A \text { for every integer } k \text {. } +$$ + +Determine all pairs $m, n$ of nonzero integers such that the only admissible set containing both $m$ and $n$ is the set of all integers. + +Solution. A pair of integers $m, n$ fulfills the condition if and only if $\operatorname{gcd}(m, n)=1$. Suppose that $\operatorname{gcd}(m, n)=d>1$. The set + +$$ +A=\{\ldots,-2 d,-d, 0, d, 2 d, \ldots\} +$$ + +is admissible, because if $d$ divides $x$ and $y$ then it divides $x^{2}+k x y+y^{2}$ for every integer $k$. Also $m, n \in A$ and $A \neq \mathbb{Z}$. + +Now let $\operatorname{gcd}(m, n)=1$, and let $A$ be an admissible set containing $m$ and $n$. We use the following observations to prove that $A=\mathbb{Z}$ : + +(i) $k x^{2} \in A$ for every $x \in A$ and every integer $k$. + +(ii) $(x+y)^{2} \in A$ for all $x, y \in A$. + +To justify (i) let $y=x$ in the definition of an admissible set; to justify (ii) let $k=2$. + +Since $\operatorname{gcd}(m, n)=1$, we also have $\operatorname{gcd}\left(m^{2}, n^{2}\right)=1$. Hence one can find integers $a, b$ such that $a m^{2}+b n^{2}=1$. It follows from (i) that $a m^{2} \in A$ and $b n^{2} \in A$. Now we deduce from (ii) that $1=\left(a m^{2}+b n^{2}\right)^{2} \in A$. But if $1 \in A$ then (i) implies $k \in A$ for every integer $k$. + +N2. Find all triples $(x, y, z)$ of positive integers such that $x \leq y \leq z$ and + +$$ +x^{3}\left(y^{3}+z^{3}\right)=2012(x y z+2) \text {. } +$$ + +Solution. First note that $x$ divides $2012 \cdot 2=2^{3} \cdot 503$. If $503 \mid x$ then the right-hand side of the equation is divisible by $503^{3}$, and it follows that $503^{2} \mid x y z+2$. This is false as $503 \mid x$. Hence $x=2^{m}$ with $m \in\{0,1,2,3\}$. If $m \geq 2$ then $2^{6} \mid 2012(x y z+2)$. However the highest powers of 2 dividing 2012 and $x y z+2=2^{m} y z+2$ are $2^{2}$ and $2^{1}$ respectively. So $x=1$ or $x=2$, yielding the two equations + +$$ +y^{3}+z^{3}=2012(y z+2), \quad \text { and } \quad y^{3}+z^{3}=503(y z+1) \text {. } +$$ + +In both cases the prime $503=3 \cdot 167+2$ divides $y^{3}+z^{3}$. We claim that $503 \mid y+z$. This is clear if $503 \mid y$, so let $503 \nmid y$ and $503 \nmid z$. Then $y^{502} \equiv z^{502}(\bmod 503)$ by FeRmat's little theorem. On the other hand $y^{3} \equiv-z^{3}(\bmod 503)$ implies $y^{3 \cdot 167} \equiv-z^{3 \cdot 167}(\bmod 503)$, i. e. $y^{501} \equiv-z^{501}(\bmod 503)$. It follows that $y \equiv-z(\bmod 503)$ as claimed. + +Therefore $y+z=503 k$ with $k \geq 1$. In view of $y^{3}+z^{3}=(y+z)\left((y-z)^{2}+y z\right)$ the two equations take the form + +$$ +\begin{aligned} +& k(y-z)^{2}+(k-4) y z=8 \\ +& k(y-z)^{2}+(k-1) y z=1 . +\end{aligned} +$$ + +In (1) we have $(k-4) y z \leq 8$, which implies $k \leq 4$. Indeed if $k>4$ then $1 \leq(k-4) y z \leq 8$, so that $y \leq 8$ and $z \leq 8$. This is impossible as $y+z=503 k \geq 503$. Note next that $y^{3}+z^{3}$ is even in the first equation. Hence $y+z=503 k$ is even too, meaning that $k$ is even. Thus $k=2$ or $k=4$. Clearly (1) has no integer solutions for $k=4$. If $k=2$ then (1) takes the form $(y+z)^{2}-5 y z=4$. Since $y+z=503 k=503 \cdot 2$, this leads to $5 y z=503^{2} \cdot 2^{2}-4$. However $503^{2} \cdot 2^{2}-4$ is not a multiple of 5 . Therefore (1) has no integer solutions. + +Equation (2) implies $0 \leq(k-1) y z \leq 1$, so that $k=1$ or $k=2$. Also $0 \leq k(y-z)^{2} \leq 1$, hence $k=2$ only if $y=z$. However then $y=z=1$, which is false in view of $y+z \geq 503$. Therefore $k=1$ and (2) takes the form $(y-z)^{2}=1$, yielding $z-y=|y-z|=1$. Combined with $k=1$ and $y+z=503 k$, this leads to $y=251, z=252$. + +In summary the triple $(2,251,252)$ is the only solution. + +N3. Determine all integers $m \geq 2$ such that every $n$ with $\frac{m}{3} \leq n \leq \frac{m}{2}$ divides the binomial coefficient $\left(\begin{array}{c}n \\ m-2 n\end{array}\right)$. + +Solution. The integers in question are all prime numbers. + +First we check that all primes satisfy the condition, and even a stronger one. Namely, if $p$ is a prime then every $n$ with $1 \leq n \leq \frac{p}{2}$ divides $\left(\begin{array}{c}n \\ p-2 n\end{array}\right)$. This is true for $p=2$ where $n=1$ is the only possibility. For an odd prime $p$ take $n \in\left[1, \frac{p}{2}\right]$ and consider the following identity of binomial coefficients: + +$$ +(p-2 n) \cdot\left(\begin{array}{c} +n \\ +p-2 n +\end{array}\right)=n \cdot\left(\begin{array}{c} +n-1 \\ +p-2 n-1 +\end{array}\right) +$$ + +Since $p \geq 2 n$ and $p$ is odd, all factors are non-zero. If $d=\operatorname{gcd}(p-2 n, n)$ then $d$ divides $p$, but $d \leq n1$, pick $n=k$. Then $\frac{m}{3} \leq n \leq \frac{m}{2}$ but $\left(\begin{array}{c}n \\ m-2 n\end{array}\right)=\left(\begin{array}{l}k \\ 0\end{array}\right)=1$ is not divisible by $k>1$. +- If $m$ is odd then there exist an odd prime $p$ and an integer $k \geq 1$ with $m=p(2 k+1)$. Pick $n=p k$, then $\frac{m}{3} \leq n \leq \frac{m}{2}$ by $k \geq 1$. However + +$$ +\frac{1}{n}\left(\begin{array}{c} +n \\ +m-2 n +\end{array}\right)=\frac{1}{p k}\left(\begin{array}{c} +p k \\ +p +\end{array}\right)=\frac{(p k-1)(p k-2) \cdots(p k-(p-1))}{p !} +$$ + +is not an integer, because $p$ divides the denominator but not the numerator. + +N4. An integer $a$ is called friendly if the equation $\left(m^{2}+n\right)\left(n^{2}+m\right)=a(m-n)^{3}$ has a solution over the positive integers. + +a) Prove that there are at least 500 friendly integers in the set $\{1,2, \ldots, 2012\}$. + +b) Decide whether $a=2$ is friendly. + +Solution. a) Every $a$ of the form $a=4 k-3$ with $k \geq 2$ is friendly. Indeed the numbers $m=2 k-1>0$ and $n=k-1>0$ satisfy the given equation with $a=4 k-3$ : + +$$ +\left(m^{2}+n\right)\left(n^{2}+m\right)=\left((2 k-1)^{2}+(k-1)\right)\left((k-1)^{2}+(2 k-1)\right)=(4 k-3) k^{3}=a(m-n)^{3} \text {. } +$$ + +Hence $5,9, \ldots, 2009$ are friendly and so $\{1,2, \ldots, 2012\}$ contains at least 502 friendly numbers. + +b) We show that $a=2$ is not friendly. Consider the equation with $a=2$ and rewrite its left-hand side as a difference of squares: + +$$ +\frac{1}{4}\left(\left(m^{2}+n+n^{2}+m\right)^{2}-\left(m^{2}+n-n^{2}-m\right)^{2}\right)=2(m-n)^{3} +$$ + +Since $m^{2}+n-n^{2}-m=(m-n)(m+n-1)$, we can further reformulate the equation as + +$$ +\left(m^{2}+n+n^{2}+m\right)^{2}=(m-n)^{2}\left(8(m-n)+(m+n-1)^{2}\right) . +$$ + +It follows that $8(m-n)+(m+n-1)^{2}$ is a perfect square. Clearly $m>n$, hence there is an integer $s \geq 1$ such that + +$$ +(m+n-1+2 s)^{2}=8(m-n)+(m+n-1)^{2} . +$$ + +Subtracting the squares gives $s(m+n-1+s)=2(m-n)$. Since $m+n-1+s>m-n$, we conclude that $s<2$. Therefore the only possibility is $s=1$ and $m=3 n$. However then the left-hand side of the given equation (with $a=2$ ) is greater than $m^{3}=27 n^{3}$, whereas its right-hand side equals $16 n^{3}$. The contradiction proves that $a=2$ is not friendly. + +Comment. A computer search shows that there are 561 friendly numbers in $\{1,2, \ldots, 2012\}$. + +N5. For a nonnegative integer $n$ define $\operatorname{rad}(n)=1$ if $n=0$ or $n=1$, and $\operatorname{rad}(n)=p_{1} p_{2} \cdots p_{k}$ where $p_{1}0$. + +Let $p$ be a prime number. The condition is that $f(n) \equiv 0(\bmod p)$ implies + +$$ +f\left(n^{\operatorname{rad}(n)}\right) \equiv 0 \quad(\bmod p) +$$ + +Since $\operatorname{rad}\left(n^{\operatorname{rad}(n)^{k}}\right)=\operatorname{rad}(n)$ for all $k$, repeated applications of the preceding implication show that if $p$ divides $f(n)$ then + +$$ +f\left(n^{\operatorname{rad}(n)^{k}}\right) \equiv 0 \quad(\bmod p) \quad \text { for all } k . +$$ + +The idea is to construct a prime $p$ and a positive integer $n$ such that $p-1$ divides $n$ and $p$ divides $f(n)$. In this case, for $k$ large enough $p-1$ divides $\operatorname{rad}(n)^{k}$. Hence if $(p, n)=1$ then $n^{r a d(n)^{k}} \equiv 1(\bmod p)$ by FERMAT's little theorem, so that + +$$ +f(1) \equiv f\left(n^{\operatorname{rad}(n)^{k}}\right) \equiv 0 \quad(\bmod p) . +$$ + +Suppose that $f(x)=g(x) x^{m}$ with $g(0) \neq 0$. Let $t$ be a positive integer, $p$ any prime factor of $g(-t)$ and $n=(p-1) t$. So $p-1$ divides $n$ and $f(n)=f((p-1) t) \equiv f(-t) \equiv 0(\bmod p)$, hence either $(p, n)>1$ or $(2)$ holds. If $(p,(p-1) t)>1$ then $p$ divides $t$ and $g(0) \equiv g(-t) \equiv 0(\bmod p)$, meaning that $p$ divides $g(0)$. + +In conclusion we proved that each prime factor of $g(-t)$ divides $g(0) f(1) \neq 0$, and thus the set of prime factors of $g(-t)$ when $t$ ranges through the positive integers is finite. This is known to imply that $g(x)$ is a constant polynomial, and so $f(x)=a x^{m}$. + +Solution 2. Let $f(x)$ be a polynomial with integer coefficients (not necessarily nonnegative) such that $\operatorname{rad}(f(n))$ divides $\operatorname{rad}\left(f\left(n^{\operatorname{rad}(n)}\right)\right)$ for any nonnegative integer $n$. We give a complete description of all polynomials with this property. More precisely, we claim that if $f(x)$ is such a polynomial and $\xi$ is a root of $f(x)$ then so is $\xi^{d}$ for every positive integer $d$. + +Therefore each root of $f(x)$ is zero or a root of unity. In particular, if a root of unity $\xi$ is a root of $f(x)$ then $1=\xi^{d}$ is a root too (for some positive integer $d$ ). In the original problem $f(x)$ has nonnegative coefficients. Then either $f(x)$ is the zero polynomial or $f(1)>0$ and $\xi=0$ is the only possible root. In either case $f(x)=a x^{m}$ with $a$ and $m$ nonnegative integers. + +To prove the claim let $\xi$ be a root of $f(x)$, and let $g(x)$ be an irreducible factor of $f(x)$ such that $g(\xi)=0$. If 0 or 1 are roots of $g(x)$ then either $\xi=0$ or $\xi=1$ (because $g(x)$ is irreducible) and we are done. So assume that $g(0), g(1) \neq 0$. By decomposing $d$ as a product of prime numbers, it is enough to consider the case $d=p$ prime. We argue for $p=2$. Since $\operatorname{rad}\left(2^{k}\right)=2$ for every $k$, we have + +$$ +\operatorname{rad}\left(f\left(2^{k}\right)\right) \mid \operatorname{rad}\left(f\left(2^{2 k}\right)\right) . +$$ + +Now we prove that $g(x)$ divides $f\left(x^{2}\right)$. Suppose that this is not the case. Then, since $g(x)$ is irreducible, there are integer-coefficient polynomials $a(x), b(x)$ and an integer $N$ such that + +$$ +a(x) g(x)+b(x) f\left(x^{2}\right)=N +$$ + +Each prime factor $p$ of $g\left(2^{k}\right)$ divides $f\left(2^{k}\right)$, so by $\operatorname{rad}\left(f\left(2^{k}\right)\right) \mid \operatorname{rad}\left(f\left(2^{2 k}\right)\right)$ it also divides $f\left(2^{2 k}\right)$. From the equation above with $x=2^{k}$ it follows that $p$ divides $N$. + +In summary, each prime divisor of $g\left(2^{k}\right)$ divides $N$, for all $k \geq 0$. Let $p_{1}, \ldots, p_{n}$ be the odd primes dividing $N$, and suppose that + +$$ +g(1)=2^{\alpha} p_{1}^{\alpha_{1}} \cdots p_{n}^{\alpha_{n}} +$$ + +If $k$ is divisible by $\varphi\left(p_{1}^{\alpha_{1}+1} \cdots p_{n}^{\alpha_{n}+1}\right)$ then + +$$ +2^{k} \equiv 1 \quad\left(\bmod p_{1}^{\alpha_{1}+1} \cdots p_{n}^{\alpha_{n}+1}\right) +$$ + +yielding + +$$ +g\left(2^{k}\right) \equiv g(1) \quad\left(\bmod p_{1}^{\alpha_{1}+1} \cdots p_{n}^{\alpha_{n}+1}\right) +$$ + +It follows that for each $i$ the maximal power of $p_{i}$ dividing $g\left(2^{k}\right)$ and $g(1)$ is the same, namely $p_{i}^{\alpha_{i}}$. On the other hand, for large enough $k$, the maximal power of 2 dividing $g\left(2^{k}\right)$ and $g(0) \neq 0$ is the same. From the above, for $k$ divisible by $\varphi\left(p_{1}^{\alpha_{1}+1} \cdots p_{n}^{\alpha_{n}+1}\right)$ and large enough, we obtain that $g\left(2^{k}\right)$ divides $g(0) \cdot g(1)$. This is impossible because $g(0), g(1) \neq 0$ are fixed and $g\left(2^{k}\right)$ is arbitrarily large. + +In conclusion, $g(x)$ divides $f\left(x^{2}\right)$. Recall that $\xi$ is a root of $f(x)$ such that $g(\xi)=0$; then $f\left(\xi^{2}\right)=0$, i. e. $\xi^{2}$ is a root of $f(x)$. + +Likewise if $\xi$ is a root of $f(x)$ and $p$ an arbitrary prime then $\xi^{p}$ is a root too. The argument is completely analogous, in the proof above just replace 2 by $p$ and "odd prime" by "prime different from $p . "$ + +Comment. The claim in the second solution can be proved by varying $n(\bmod p)$ in $(1)$. For instance, we obtain + +$$ +f\left(n^{\operatorname{rad}(n+p k)}\right) \equiv 0 \quad(\bmod p) +$$ + +for every positive integer $k$. One can prove that if $(n, p)=1$ then $\operatorname{rad}(n+p k)$ runs through all residue classes $r(\bmod p-1)$ with $(r, p-1)$ squarefree. Hence if $f(n) \equiv 0(\bmod p)$ then $f\left(n^{r}\right) \equiv 0(\bmod p)$ for all integers $r$. This implies the claim by an argument leading to the identity (3). + +N6. Let $x$ and $y$ be positive integers. If $x^{2^{n}}-1$ is divisible by $2^{n} y+1$ for every positive integer $n$, prove that $x=1$. + +Solution. First we prove the following fact: For every positive integer $y$ there exist infinitely many primes $p \equiv 3(\bmod 4)$ such that $p$ divides some number of the form $2^{n} y+1$. + +Clearly it is enough to consider the case $y$ odd. Let + +$$ +2 y+1=p_{1}^{e_{1}} \cdots p_{r}^{e_{r}} +$$ + +be the prime factorization of $2 y+1$. Suppose on the contrary that there are finitely many primes $p_{r+1}, \ldots, p_{r+s} \equiv 3(\bmod 4)$ that divide some number of the form $2^{n} y+1$ but do not divide $2 y+1$. + +We want to find an $n$ such that $p_{i}^{e_{i}} \| 2^{n} y+1$ for $1 \leq i \leq r$ and $p_{i} \nmid 2^{n} y+1$ for $r+1 \leq i \leq r+s$. For this it suffices to take + +$$ +n=1+\varphi\left(p_{1}^{e_{1}+1} \cdots p_{r}^{e_{r}+1} p_{r+1}^{1} \cdots p_{r+s}^{1}\right) +$$ + +because then + +$$ +2^{n} y+1 \equiv 2 y+1 \quad\left(\bmod p_{1}^{e_{1}+1} \cdots p_{r}^{e_{r}+1} p_{r+1}^{1} \cdots p_{r+s}^{1}\right) +$$ + +The last congruence means that $p_{1}^{e_{1}}, \ldots, p_{r}^{e_{r}}$ divide exactly $2^{n} y+1$ and no prime $p_{r+1}, \ldots, p_{r+s}$ divides $2^{n} y+1$. It follows that the prime factorization of $2^{n} y+1$ consists of the prime powers $p_{1}^{e_{1}}, \ldots, p_{r}^{e_{r}}$ and powers of primes $\equiv 1(\bmod 4)$. Because $y$ is odd, we obtain + +$$ +2^{n} y+1 \equiv p_{1}^{e_{1}} \cdots p_{r}^{e_{r}} \equiv 2 y+1 \equiv 3 \quad(\bmod 4) +$$ + +This is a contradiction since $n>1$, and so $2^{n} y+1 \equiv 1(\bmod 4)$. + +Now we proceed to the problem. If $p$ is a prime divisor of $2^{n} y+1$ the problem statement implies that $x^{d} \equiv 1(\bmod p)$ for $d=2^{n}$. By FERMAT's little theorem the same congruence holds for $d=p-1$, so it must also hold for $d=\left(2^{n}, p-1\right)$. For $p \equiv 3(\bmod 4)$ we have $\left(2^{n}, p-1\right)=2$, therefore in this case $x^{2} \equiv 1(\bmod p)$. + +In summary, we proved that every prime $p \equiv 3(\bmod 4)$ that divides some number of the form $2^{n} y+1$ also divides $x^{2}-1$. This is possible only if $x=1$, otherwise by the above $x^{2}-1$ would be a positive integer with infinitely many prime factors. + +Comment. For each $x$ and each odd prime $p$ the maximal power of $p$ dividing $x^{2^{n}}-1$ for some $n$ is bounded and hence the same must be true for the numbers $2^{n} y+1$. We infer that $p^{2}$ divides $2^{p-1}-1$ for each prime divisor $p$ of $2^{n} y+1$. However trying to reach a contradiction with this conclusion alone seems hopeless, since it is not even known if there are infinitely many primes $p$ without this property. + +N7. Find all $n \in \mathbb{N}$ for which there exist nonnegative integers $a_{1}, a_{2}, \ldots, a_{n}$ such that + +$$ +\frac{1}{2^{a_{1}}}+\frac{1}{2^{a_{2}}}+\cdots+\frac{1}{2^{a_{n}}}=\frac{1}{3^{a_{1}}}+\frac{2}{3^{a_{2}}}+\cdots+\frac{n}{3^{a_{n}}}=1 . +$$ + +Solution. Such numbers $a_{1}, a_{2}, \ldots, a_{n}$ exist if and only if $n \equiv 1(\bmod 4)$ or $n \equiv 2(\bmod 4)$. + +Let $\sum_{k=1}^{n} \frac{k}{3^{a} k}=1$ with $a_{1}, a_{2}, \ldots, a_{n}$ nonnegative integers. Then $1 \cdot x_{1}+2 \cdot x_{2}+\cdots+n \cdot x_{n}=3^{a}$ with $x_{1}, \ldots, x_{n}$ powers of 3 and $a \geq 0$. The right-hand side is odd, and the left-hand side has the same parity as $1+2+\cdots+n$. Hence the latter sum is odd, which implies $n \equiv 1,2(\bmod 4)$. Now we prove the converse. + +Call feasible a sequence $b_{1}, b_{2}, \ldots, b_{n}$ if there are nonnegative integers $a_{1}, a_{2}, \ldots, a_{n}$ such that + +$$ +\frac{1}{2^{a_{1}}}+\frac{1}{2^{a_{2}}}+\cdots+\frac{1}{2^{a_{n}}}=\frac{b_{1}}{3^{a_{1}}}+\frac{b_{2}}{3^{a_{2}}}+\cdots+\frac{b_{n}}{3^{a_{n}}}=1 +$$ + +Let $b_{k}$ be a term of a feasible sequence $b_{1}, b_{2}, \ldots, b_{n}$ with exponents $a_{1}, a_{2}, \ldots, a_{n}$ like above, and let $u, v$ be nonnegative integers with sum $3 b_{k}$. Observe that + +$$ +\frac{1}{2^{a_{k}+1}}+\frac{1}{2^{a_{k}+1}}=\frac{1}{2^{a_{k}}} \quad \text { and } \quad \frac{u}{3^{a_{k}+1}}+\frac{v}{3^{a_{k}+1}}=\frac{b_{k}}{3^{a_{k}}} +$$ + +It follows that the sequence $b_{1}, \ldots, b_{k-1}, u, v, b_{k+1}, \ldots, b_{n}$ is feasible. The exponents $a_{i}$ are the same for the unchanged terms $b_{i}, i \neq k$; the new terms $u, v$ have exponents $a_{k}+1$. + +We state the conclusion in reverse. If two terms $u, v$ of a sequence are replaced by one term $\frac{u+v}{3}$ and the obtained sequence is feasible, then the original sequence is feasible too. Denote by $\alpha_{n}$ the sequence $1,2, \ldots, n$. To show that $\alpha_{n}$ is feasible for $n \equiv 1,2(\bmod 4)$, we transform it by $n-1$ replacements $\{u, v\} \mapsto \frac{u+v}{3}$ to the one-term sequence $\alpha_{1}$. The latter is feasible, with $a_{1}=0$. Note that if $m$ and $2 m$ are terms of a sequence then $\{m, 2 m\} \mapsto m$, so $2 m$ can be ignored if necessary. + +Let $n \geq 16$. We prove that $\alpha_{n}$ can be reduced to $\alpha_{n-12}$ by 12 operations. Write $n=12 k+r$ where $k \geq 1$ and $0 \leq r \leq 11$. If $0 \leq r \leq 5$ then the last 12 terms of $\alpha_{n}$ can be partitioned into 2 singletons $\{12 k-6\},\{12 k\}$ and the following 5 pairs: + +$$ +\{12 k-6-i, 12 k-6+i\}, i=1, \ldots, 5-r ; \quad\{12 k-j, 12 k+j\}, j=1, \ldots, r . +$$ + +(There is only one kind of pairs if $r \in\{0,5\}$.) One can ignore $12 k-6$ and $12 k$ since $\alpha_{n}$ contains $6 k-3$ and $6 k$. Furthermore the 5 operations $\{12 k-6-i, 12 k-6+i\} \mapsto 8 k-4$ and $\{12 k-j, 12 k+j\} \mapsto 8 k$ remove the 10 terms in the pairs and bring in 5 new terms equal to $8 k-4$ or $8 k$. All of these can be ignored too as $4 k-2$ and $4 k$ are still present in the sequence. Indeed $4 k \leq n-12$ is equivalent to $8 k \geq 12-r$, which is true for $r \in\{4,5\}$. And if $r \in\{0,1,2,3\}$ then $n \geq 16$ implies $k \geq 2$, so $8 k \geq 12-r$ also holds. Thus $\alpha_{n}$ reduces to $\alpha_{n-12}$. + +The case $6 \leq r \leq 11$ is analogous. Consider the singletons $\{12 k\},\{12 k+6\}$ and the 5 pairs + +$$ +\{12 k-i, 12 k+i\}, i=1, \ldots, 11-r ; \quad\{12 k+6-j, 12 k+6+j\}, j=1, \ldots, r-6 +$$ + +Ignore the singletons like before, then remove the pairs via operations $\{12 k-i, 12 k+i\} \mapsto 8 k$ and $\{12 k+6-j, 12 k+6+j\} \mapsto 8 k+4$. The 5 newly-appeared terms $8 k$ and $8 k+4$ can be ignored too since $4 k+2 \leq n-12$ (this follows from $k \geq 1$ and $r \geq 6$ ). We obtain $\alpha_{n-12}$ again. + +The problem reduces to $2 \leq n \leq 15$. In fact $n \in\{2,5,6,9,10,13,14\}$ by $n \equiv 1,2(\bmod 4)$. The cases $n=2,6,10,14$ reduce to $n=1,5,9,13$ respectively because the last even term of $\alpha_{n}$ can be ignored. For $n=5$ apply $\{4,5\} \mapsto 3$, then $\{3,3\} \mapsto 2$, then ignore the 2 occurrences of 2 . For $n=9$ ignore 6 first, then apply $\{5,7\} \mapsto 4,\{4,8\} \mapsto 4,\{3,9\} \mapsto 4$. Now ignore the 3 occurrences of 4 , then ignore 2. Finally $n=13$ reduces to $n=10$ by $\{11,13\} \mapsto 8$ and ignoring 8 and 12 . The proof is complete. + +N8. Prove that for every prime $p>100$ and every integer $r$ there exist two integers $a$ and $b$ such that $p$ divides $a^{2}+b^{5}-r$. + +Solution 1. Throughout the solution, all congruence relations are meant modulo $p$. + +Fix $p$, and let $\mathcal{P}=\{0,1, \ldots, p-1\}$ be the set of residue classes modulo $p$. For every $r \in \mathcal{P}$, let $S_{r}=\left\{(a, b) \in \mathcal{P} \times \mathcal{P}: a^{2}+b^{5} \equiv r\right\}$, and let $s_{r}=\left|S_{r}\right|$. Our aim is to prove $s_{r}>0$ for all $r \in \mathcal{P}$. + +We will use the well-known fact that for every residue class $r \in \mathcal{P}$ and every positive integer $k$, there are at most $k$ values $x \in \mathcal{P}$ such that $x^{k} \equiv r$. + +Lemma. Let $N$ be the number of quadruples $(a, b, c, d) \in \mathcal{P}^{4}$ for which $a^{2}+b^{5} \equiv c^{2}+d^{5}$. Then + +$$ +N=\sum_{r \in \mathcal{P}} s_{r}^{2} +$$ + +and + +$$ +N \leq p\left(p^{2}+4 p-4\right) +$$ + +Proof. (a) For each residue class $r$ there exist exactly $s_{r}$ pairs $(a, b)$ with $a^{2}+b^{5} \equiv r$ and $s_{r}$ pairs $(c, d)$ with $c^{2}+d^{5} \equiv r$. So there are $s_{r}^{2}$ quadruples with $a^{2}+b^{5} \equiv c^{2}+d^{5} \equiv r$. Taking the sum over all $r \in \mathcal{P}$, the statement follows. + +(b) Choose an arbitrary pair $(b, d) \in \mathcal{P}$ and look for the possible values of $a, c$. + +1. Suppose that $b^{5} \equiv d^{5}$, and let $k$ be the number of such pairs $(b, d)$. The value $b$ can be chosen in $p$ different ways. For $b \equiv 0$ only $d=0$ has this property; for the nonzero values of $b$ there are at most 5 possible values for $d$. So we have $k \leq 1+5(p-1)=5 p-4$. + +The values $a$ and $c$ must satisfy $a^{2} \equiv c^{2}$, so $a \equiv \pm c$, and there are exactly $2 p-1$ such pairs $(a, c)$. + +2. Now suppose $b^{5} \not \equiv d^{5}$. In this case $a$ and $c$ must be distinct. By $(a-c)(a+c)=d^{5}-b^{5}$, the value of $a-c$ uniquely determines $a+c$ and thus $a$ and $c$ as well. Hence, there are $p-1$ suitable pairs $(a, c)$. + +Thus, for each of the $k$ pairs $(b, d)$ with $b^{5} \equiv d^{5}$ there are $2 p-1$ pairs $(a, c)$, and for each of the other $p^{2}-k$ pairs $(b, d)$ there are $p-1$ pairs $(a, c)$. Hence, + +$$ +N=k(2 p-1)+\left(p^{2}-k\right)(p-1)=p^{2}(p-1)+k p \leq p^{2}(p-1)+(5 p-4) p=p\left(p^{2}+4 p-4\right) +$$ + +To prove the statement of the problem, suppose that $S_{r}=\emptyset$ for some $r \in \mathcal{P}$; obviously $r \not \equiv 0$. Let $T=\left\{x^{10}: x \in \mathcal{P} \backslash\{0\}\right\}$ be the set of nonzero 10th powers modulo $p$. Since each residue class is the 10 th power of at most 10 elements in $\mathcal{P}$, we have $|T| \geq \frac{p-1}{10} \geq 4$ by $p>100$. + +For every $t \in T$, we have $S_{t r}=\emptyset$. Indeed, if $(x, y) \in S_{t r}$ and $t \equiv z^{10}$ then + +$$ +\left(z^{-5} x\right)^{2}+\left(z^{-2} y\right)^{5} \equiv t^{-1}\left(x^{2}+y^{5}\right) \equiv r +$$ + +so $\left(z^{-5} x, z^{-2} y\right) \in S_{r}$. So, there are at least $\frac{p-1}{10} \geq 4$ empty sets among $S_{1}, \ldots, S_{p-1}$, and there are at most $p-4$ nonzero values among $s_{0}, s_{2}, \ldots, s_{p-1}$. Then by the AM-QM inequality we obtain + +$$ +N=\sum_{r \in \mathcal{P} \backslash r T} s_{r}^{2} \geq \frac{1}{p-4}\left(\sum_{r \in \mathcal{P} \backslash r T} s_{r}\right)^{2}=\frac{|\mathcal{P} \times \mathcal{P}|^{2}}{p-4}=\frac{p^{4}}{p-4}>p\left(p^{2}+4 p-4\right), +$$ + +which is impossible by the lemma. + +Solution 2. If $5 \nmid p-1$, then all modulo $p$ residue classes are complete fifth powers and the statement is trivial. So assume that $p=10 k+1$ where $k \geq 10$. Let $g$ be a primitive root modulo $p$. + +We will use the following facts: + +(F1) If some residue class $x$ is not quadratic then $x^{(p-1) / 2} \equiv-1(\bmod p)$. + +(F2) For every integer $d$, as a simple corollary of the summation formula for geometric progressions, + +$$ +\sum_{i=0}^{2 k-1} g^{5 d i} \equiv\left\{\begin{array}{ll} +2 k & \text { if } 2 k \mid d \\ +0 & \text { if } 2 k \not \nless d +\end{array} \quad(\bmod p)\right. +$$ + +Suppose that, contrary to the statement, some modulo $p$ residue class $r$ cannot be expressed as $a^{2}+b^{5}$. Of course $r \not \equiv 0(\bmod p)$. By $(\mathrm{F} 1)$ we have $\left(r-b^{5}\right)^{(p-1) / 2}=\left(r-b^{5}\right)^{5 k} \equiv-1(\bmod p)$ for all residue classes $b$. + +For $t=1,2 \ldots, k-1$ consider the sums + +$$ +S(t)=\sum_{i=0}^{2 k-1}\left(r-g^{5 i}\right)^{5 k} g^{5 t i} +$$ + +By the indirect assumption and (F2), + +$$ +S(t)=\sum_{i=0}^{2 k-1}\left(r-\left(g^{i}\right)^{5}\right)^{5 k} g^{5 t i} \equiv \sum_{i=0}^{2 k-1}(-1) g^{5 t i} \equiv-\sum_{i=0}^{2 k-1} g^{5 t i} \equiv 0 \quad(\bmod p) +$$ + +because $2 k$ cannot divide $t$. + +On the other hand, by the binomial theorem, + +$$ +\begin{aligned} +S(t) & =\sum_{i=0}^{2 k-1}\left(\sum_{j=0}^{5 k}\left(\begin{array}{c} +5 k \\ +j +\end{array}\right) r^{5 k-j}\left(-g^{5 i}\right)^{j}\right) g^{5 t i}=\sum_{j=0}^{5 k}(-1)^{j}\left(\begin{array}{c} +5 k \\ +j +\end{array}\right) r^{5 k-j}\left(\sum_{i=0}^{2 k-1} g^{5(j+t) i}\right) \equiv \\ +& \equiv \sum_{j=0}^{5 k}(-1)^{j}\left(\begin{array}{c} +5 k \\ +j +\end{array}\right) r^{5 k-j}\left\{\begin{array}{ll} +2 k & \text { if } 2 k \mid j+t \\ +0 & \text { if } 2 k \not j j+t +\end{array}(\bmod p) .\right. +\end{aligned} +$$ + +Since $1 \leq j+t<6 k$, the number $2 k$ divides $j+t$ only for $j=2 k-t$ and $j=4 k-t$. Hence, + +$$ +\begin{gathered} +0 \equiv S(t) \equiv(-1)^{t}\left(\left(\begin{array}{c} +5 k \\ +2 k-t +\end{array}\right) r^{3 k+t}+\left(\begin{array}{c} +5 k \\ +4 k-t +\end{array}\right) r^{k+t}\right) \cdot 2 k \quad(\bmod p), \\ +\left(\begin{array}{c} +5 k \\ +2 k-t +\end{array}\right) r^{2 k}+\left(\begin{array}{c} +5 k \\ +4 k-t +\end{array}\right) \equiv 0 \quad(\bmod p) . +\end{gathered} +$$ + +Taking this for $t=1,2$ and eliminating $r$, we get + +$$ +\begin{aligned} +0 & \equiv\left(\begin{array}{c} +5 k \\ +2 k-2 +\end{array}\right)\left(\left(\begin{array}{c} +5 k \\ +2 k-1 +\end{array}\right) r^{2 k}+\left(\begin{array}{c} +5 k \\ +4 k-1 +\end{array}\right)\right)-\left(\begin{array}{c} +5 k \\ +2 k-1 +\end{array}\right)\left(\left(\begin{array}{c} +5 k \\ +2 k-2 +\end{array}\right) r^{2 k}+\left(\begin{array}{c} +5 k \\ +4 k-2 +\end{array}\right)\right) \\ +& =\left(\begin{array}{c} +5 k \\ +2 k-2 +\end{array}\right)\left(\begin{array}{c} +5 k \\ +4 k-1 +\end{array}\right)-\left(\begin{array}{c} +5 k \\ +2 k-1 +\end{array}\right)\left(\begin{array}{c} +5 k \\ +4 k-2 +\end{array}\right) \\ +& =\frac{(5 k) !^{2}}{(2 k-1) !(3 k+2) !(4 k-1) !(k+2) !}((2 k-1)(k+2)-(3 k+2)(4 k-1)) \\ +& =\frac{-(5 k) !^{2} \cdot 2 k(5 k+1)}{(2 k-1) !(3 k+2) !(4 k-1) !(k+2) !}(\bmod p) . +\end{aligned} +$$ + +But in the last expression none of the numbers is divisible by $p=10 k+1$, a contradiction. + +Comment 1. The argument in the second solution is valid whenever $k \geq 3$, that is for all primes $p=10 k+1$ except $p=11$. This is an exceptional case when the statement is not true; $r=7$ cannot be expressed as desired. + +Comment 2. The statement is true in a more general setting: for every positive integer $n$, for all sufficiently large $p$, each residue class modulo $p$ can be expressed as $a^{2}+b^{n}$. Choosing $t=3$ would allow using the Cauchy-Davenport theorem (together with some analysis on the case of equality). + +In the literature more general results are known. For instance, the statement easily follows from the Hasse-Weil bound. + diff --git a/IMO/md/en-IMO2013SL.md b/IMO/md/en-IMO2013SL.md new file mode 100644 index 0000000000000000000000000000000000000000..7c879da69fb4509dc9e92ead17d2e100f1f29f82 --- /dev/null +++ b/IMO/md/en-IMO2013SL.md @@ -0,0 +1,2442 @@ +# Shortlisted Problems with Solutions + +$54^{\text {th }}$ International Mathematical Olympiad + +Santa Marta, Colombia 2013 + +## Note of Confidentiality + +## The Shortlisted Problems should be kept strictly confidential until IMO 2014. + +## Contributing Countries + +The Organizing Committee and the Problem Selection Committee of IMO 2013 thank the following 50 countries for contributing 149 problem proposals. + +Argentina, Armenia, Australia, Austria, Belgium, Belarus, Brazil, Bulgaria, Croatia, Cyprus, Czech Republic, Denmark, El Salvador, Estonia, Finland, France, Georgia, Germany, Greece, Hungary, India, Indonesia, Iran, Ireland, Israel, Italy, Japan, Latvia, Lithuania, Luxembourg, Malaysia, Mexico, Netherlands, Nicaragua, Pakistan, Panama, Poland, Romania, Russia, Saudi Arabia, Serbia, Slovenia, Sweden, Switzerland, Tajikistan, Thailand, Turkey, U.S.A., Ukraine, United Kingdom + +## Problem Selection Committee + +Federico Ardila (chairman) + +Ilya I. Bogdanov + +Géza Kós + +Carlos Gustavo Tamm de Araújo Moreira (Gugu) + +Christian Reiher + +## Problems + +## Algebra + +A1. Let $n$ be a positive integer and let $a_{1}, \ldots, a_{n-1}$ be arbitrary real numbers. Define the sequences $u_{0}, \ldots, u_{n}$ and $v_{0}, \ldots, v_{n}$ inductively by $u_{0}=u_{1}=v_{0}=v_{1}=1$, and + +$$ +u_{k+1}=u_{k}+a_{k} u_{k-1}, \quad v_{k+1}=v_{k}+a_{n-k} v_{k-1} \quad \text { for } k=1, \ldots, n-1 . +$$ + +Prove that $u_{n}=v_{n}$. + +(France) + +A2. Prove that in any set of 2000 distinct real numbers there exist two pairs $a>b$ and $c>d$ with $a \neq c$ or $b \neq d$, such that + +$$ +\left|\frac{a-b}{c-d}-1\right|<\frac{1}{100000} +$$ + +(Lithuania) + +A3. Let $\mathbb{Q}_{>0}$ be the set of positive rational numbers. Let $f: \mathbb{Q}_{>0} \rightarrow \mathbb{R}$ be a function satisfying the conditions + +$$ +f(x) f(y) \geqslant f(x y) \text { and } f(x+y) \geqslant f(x)+f(y) +$$ + +for all $x, y \in \mathbb{Q}_{>0}$. Given that $f(a)=a$ for some rational $a>1$, prove that $f(x)=x$ for all $x \in \mathbb{Q}_{>0}$. + +(Bulgaria) + +A4. Let $n$ be a positive integer, and consider a sequence $a_{1}, a_{2}, \ldots, a_{n}$ of positive integers. Extend it periodically to an infinite sequence $a_{1}, a_{2}, \ldots$ by defining $a_{n+i}=a_{i}$ for all $i \geqslant 1$. If + +$$ +a_{1} \leqslant a_{2} \leqslant \cdots \leqslant a_{n} \leqslant a_{1}+n +$$ + +and + +$$ +a_{a_{i}} \leqslant n+i-1 \quad \text { for } i=1,2, \ldots, n +$$ + +prove that + +$$ +a_{1}+\cdots+a_{n} \leqslant n^{2} . +$$ + +(Germany) + +A5. Let $\mathbb{Z}_{\geqslant 0}$ be the set of all nonnegative integers. Find all the functions $f: \mathbb{Z}_{\geqslant 0} \rightarrow \mathbb{Z}_{\geqslant 0}$ satisfying the relation + +$$ +f(f(f(n)))=f(n+1)+1 +$$ + +for all $n \in \mathbb{Z}_{\geqslant 0}$. + +(Serbia) + +A6. Let $m \neq 0$ be an integer. Find all polynomials $P(x)$ with real coefficients such that + +$$ +\left(x^{3}-m x^{2}+1\right) P(x+1)+\left(x^{3}+m x^{2}+1\right) P(x-1)=2\left(x^{3}-m x+1\right) P(x) +$$ + +for all real numbers $x$. + +## Combinatorics + +C1. Let $n$ be a positive integer. Find the smallest integer $k$ with the following property: Given any real numbers $a_{1}, \ldots, a_{d}$ such that $a_{1}+a_{2}+\cdots+a_{d}=n$ and $0 \leqslant a_{i} \leqslant 1$ for $i=1,2, \ldots, d$, it is possible to partition these numbers into $k$ groups (some of which may be empty) such that the sum of the numbers in each group is at most 1 . + +(Poland) + +C2. In the plane, 2013 red points and 2014 blue points are marked so that no three of the marked points are collinear. One needs to draw $k$ lines not passing through the marked points and dividing the plane into several regions. The goal is to do it in such a way that no region contains points of both colors. + +Find the minimal value of $k$ such that the goal is attainable for every possible configuration of 4027 points. + +(Australia) + +C3. A crazy physicist discovered a new kind of particle which he called an imon, after some of them mysteriously appeared in his lab. Some pairs of imons in the lab can be entangled, and each imon can participate in many entanglement relations. The physicist has found a way to perform the following two kinds of operations with these particles, one operation at a time. + +(i) If some imon is entangled with an odd number of other imons in the lab, then the physicist can destroy it. + +(ii) At any moment, he may double the whole family of imons in his lab by creating a copy $I^{\prime}$ of each imon $I$. During this procedure, the two copies $I^{\prime}$ and $J^{\prime}$ become entangled if and only if the original imons $I$ and $J$ are entangled, and each copy $I^{\prime}$ becomes entangled with its original imon $I$; no other entanglements occur or disappear at this moment. + +Prove that the physicist may apply a sequence of such operations resulting in a family of imons, no two of which are entangled. + +(Japan) + +C4. Let $n$ be a positive integer, and let $A$ be a subset of $\{1, \ldots, n\}$. An $A$-partition of $n$ into $k$ parts is a representation of $n$ as a sum $n=a_{1}+\cdots+a_{k}$, where the parts $a_{1}, \ldots, a_{k}$ belong to $A$ and are not necessarily distinct. The number of different parts in such a partition is the number of (distinct) elements in the set $\left\{a_{1}, a_{2}, \ldots, a_{k}\right\}$. + +We say that an $A$-partition of $n$ into $k$ parts is optimal if there is no $A$-partition of $n$ into $r$ parts with $r\angle C$. Let $P$ and $Q$ be two different points on line $A C$ such that $\angle P B A=\angle Q B A=\angle A C B$ and $A$ is located between $P$ and $C$. Suppose that there exists an interior point $D$ of segment $B Q$ for which $P D=P B$. Let the ray $A D$ intersect the circle $A B C$ at $R \neq A$. Prove that $Q B=Q R$. + +(Georgia) + +G5. Let $A B C D E F$ be a convex hexagon with $A B=D E, B C=E F, C D=F A$, and $\angle A-\angle D=\angle C-\angle F=\angle E-\angle B$. Prove that the diagonals $A D, B E$, and $C F$ are concurrent. + +(Ukraine) + +G6. Let the excircle of the triangle $A B C$ lying opposite to $A$ touch its side $B C$ at the point $A_{1}$. Define the points $B_{1}$ and $C_{1}$ analogously. Suppose that the circumcentre of the triangle $A_{1} B_{1} C_{1}$ lies on the circumcircle of the triangle $A B C$. Prove that the triangle $A B C$ is right-angled. + +(Russia) + +## Number Theory + +N1. Let $\mathbb{Z}_{>0}$ be the set of positive integers. Find all functions $f: \mathbb{Z}_{>0} \rightarrow \mathbb{Z}_{>0}$ such that + +$$ +m^{2}+f(n) \mid m f(m)+n +$$ + +for all positive integers $m$ and $n$. + +(Malaysia) + +N2. Prove that for any pair of positive integers $k$ and $n$ there exist $k$ positive integers $m_{1}, m_{2}, \ldots, m_{k}$ such that + +$$ +1+\frac{2^{k}-1}{n}=\left(1+\frac{1}{m_{1}}\right)\left(1+\frac{1}{m_{2}}\right) \cdots\left(1+\frac{1}{m_{k}}\right) . +$$ + +(Japan) + +N3. Prove that there exist infinitely many positive integers $n$ such that the largest prime divisor of $n^{4}+n^{2}+1$ is equal to the largest prime divisor of $(n+1)^{4}+(n+1)^{2}+1$. + +(Belgium) + +N4. Determine whether there exists an infinite sequence of nonzero digits $a_{1}, a_{2}, a_{3}, \ldots$ and a positive integer $N$ such that for every integer $k>N$, the number $\overline{a_{k} a_{k-1} \ldots a_{1}}$ is a perfect square. + +(Iran) + +N5. Fix an integer $k \geqslant 2$. Two players, called Ana and Banana, play the following game of numbers: Initially, some integer $n \geqslant k$ gets written on the blackboard. Then they take moves in turn, with Ana beginning. A player making a move erases the number $m$ just written on the blackboard and replaces it by some number $m^{\prime}$ with $k \leqslant m^{\prime}0}$. (Here, $\mathbb{Z}_{>0}$ denotes the set of positive integers.) + +(Israel) + +N7. Let $\nu$ be an irrational positive number, and let $m$ be a positive integer. A pair $(a, b)$ of positive integers is called good if + +$$ +a\lceil b \nu\rceil-b\lfloor a \nu\rfloor=m . +$$ + +A good pair $(a, b)$ is called excellent if neither of the pairs $(a-b, b)$ and $(a, b-a)$ is good. (As usual, by $\lfloor x\rfloor$ and $\lceil x\rceil$ we denote the integer numbers such that $x-1<\lfloor x\rfloor \leqslant x$ and $x \leqslant\lceil x\rceili_{1}>\ldots>i_{t}>n-k \\ i_{j}-i_{j+1} \geqslant 2}} a_{i_{1}} \ldots a_{i_{t}} +$$ + +For $k=n$ the expressions (1) and (2) coincide, so indeed $u_{n}=v_{n}$. + +Solution 2. Define recursively a sequence of multivariate polynomials by + +$$ +P_{0}=P_{1}=1, \quad P_{k+1}\left(x_{1}, \ldots, x_{k}\right)=P_{k}\left(x_{1}, \ldots, x_{k-1}\right)+x_{k} P_{k-1}\left(x_{1}, \ldots, x_{k-2}\right), +$$ + +so $P_{n}$ is a polynomial in $n-1$ variables for each $n \geqslant 1$. Two easy inductive arguments show that + +$$ +u_{n}=P_{n}\left(a_{1}, \ldots, a_{n-1}\right), \quad v_{n}=P_{n}\left(a_{n-1}, \ldots, a_{1}\right) +$$ + +so we need to prove $P_{n}\left(x_{1}, \ldots, x_{n-1}\right)=P_{n}\left(x_{n-1}, \ldots, x_{1}\right)$ for every positive integer $n$. The cases $n=1,2$ are trivial, and the cases $n=3,4$ follow from $P_{3}(x, y)=1+x+y$ and $P_{4}(x, y, z)=$ $1+x+y+z+x z$. + +Now we proceed by induction, assuming that $n \geqslant 5$ and the claim hold for all smaller cases. Using $F(a, b)$ as an abbreviation for $P_{|a-b|+1}\left(x_{a}, \ldots, x_{b}\right)$ (where the indices $a, \ldots, b$ can be either in increasing or decreasing order), + +$$ +\begin{aligned} +F(n, 1) & =F(n, 2)+x_{1} F(n, 3)=F(2, n)+x_{1} F(3, n) \\ +& =\left(F(2, n-1)+x_{n} F(2, n-2)\right)+x_{1}\left(F(3, n-1)+x_{n} F(3, n-2)\right) \\ +& =\left(F(n-1,2)+x_{1} F(n-1,3)\right)+x_{n}\left(F(n-2,2)+x_{1} F(n-2,3)\right) \\ +& =F(n-1,1)+x_{n} F(n-2,1)=F(1, n-1)+x_{n} F(1, n-2) \\ +& =F(1, n), +\end{aligned} +$$ + +as we wished to show. + +Solution 3. Using matrix notation, we can rewrite the recurrence relation as + +$$ +\left(\begin{array}{c} +u_{k+1} \\ +u_{k+1}-u_{k} +\end{array}\right)=\left(\begin{array}{c} +u_{k}+a_{k} u_{k-1} \\ +a_{k} u_{k-1} +\end{array}\right)=\left(\begin{array}{cc} +1+a_{k} & -a_{k} \\ +a_{k} & -a_{k} +\end{array}\right)\left(\begin{array}{c} +u_{k} \\ +u_{k}-u_{k-1} +\end{array}\right) +$$ + +for $1 \leqslant k \leqslant n-1$, and similarly + +$$ +\left(v_{k+1} ; v_{k}-v_{k+1}\right)=\left(v_{k}+a_{n-k} v_{k-1} ;-a_{n-k} v_{k-1}\right)=\left(v_{k} ; v_{k-1}-v_{k}\right)\left(\begin{array}{cc} +1+a_{n-k} & -a_{n-k} \\ +a_{n-k} & -a_{n-k} +\end{array}\right) +$$ + +for $1 \leqslant k \leqslant n-1$. Hence, introducing the $2 \times 2$ matrices $A_{k}=\left(\begin{array}{cc}1+a_{k} & -a_{k} \\ a_{k} & -a_{k}\end{array}\right)$ we have + +$$ +\left(\begin{array}{c} +u_{k+1} \\ +u_{k+1}-u_{k} +\end{array}\right)=A_{k}\left(\begin{array}{c} +u_{k} \\ +u_{k}-u_{k-1} +\end{array}\right) \quad \text { and } \quad\left(v_{k+1} ; v_{k}-v_{k+1}\right)=\left(v_{k} ; v_{k-1}-v_{k}\right) A_{n-k} . +$$ + +for $1 \leqslant k \leqslant n-1$. Since $\left(\begin{array}{c}u_{1} \\ u_{1}-u_{0}\end{array}\right)=\left(\begin{array}{l}1 \\ 0\end{array}\right)$ and $\left(v_{1} ; v_{0}-v_{1}\right)=(1 ; 0)$, we get + +$$ +\left(\begin{array}{c} +u_{n} \\ +u_{n}-u_{n-1} +\end{array}\right)=A_{n-1} A_{n-2} \cdots A_{1} \cdot\left(\begin{array}{l} +1 \\ +0 +\end{array}\right) \quad \text { and } \quad\left(v_{n} ; v_{n-1}-v_{n}\right)=(1 ; 0) \cdot A_{n-1} A_{n-2} \cdots A_{1} \text {. } +$$ + +It follows that + +$$ +\left(u_{n}\right)=(1 ; 0)\left(\begin{array}{c} +u_{n} \\ +u_{n}-u_{n-1} +\end{array}\right)=(1 ; 0) \cdot A_{n-1} A_{n-2} \cdots A_{1} \cdot\left(\begin{array}{l} +1 \\ +0 +\end{array}\right)=\left(v_{n} ; v_{n-1}-v_{n}\right)\left(\begin{array}{l} +1 \\ +0 +\end{array}\right)=\left(v_{n}\right) . +$$ + +Comment 1. These sequences are related to the Fibonacci sequence; when $a_{1}=\cdots=a_{n-1}=1$, we have $u_{k}=v_{k}=F_{k+1}$, the $(k+1)$ st Fibonacci number. Also, for every positive integer $k$, the polynomial $P_{k}\left(x_{1}, \ldots, x_{k-1}\right)$ from Solution 2 is the sum of $F_{k+1}$ monomials. + +Comment 2. One may notice that the condition is equivalent to + +$$ +\frac{u_{k+1}}{u_{k}}=1+\frac{a_{k}}{1+\frac{a_{k-1}}{1+\ldots+\frac{a_{2}}{1+a_{1}}}} \quad \text { and } \quad \frac{v_{k+1}}{v_{k}}=1+\frac{a_{n-k}}{1+\frac{a_{n-k+1}}{1+\ldots+\frac{a_{n-2}}{1+a_{n-1}}}} +$$ + +so the problem claims that the corresponding continued fractions for $u_{n} / u_{n-1}$ and $v_{n} / v_{n-1}$ have the same numerator. + +Comment 3. An alternative variant of the problem is the following. + +Let $n$ be a positive integer and let $a_{1}, \ldots, a_{n-1}$ be arbitrary real numbers. Define the sequences $u_{0}, \ldots, u_{n}$ and $v_{0}, \ldots, v_{n}$ inductively by $u_{0}=v_{0}=0, u_{1}=v_{1}=1$, and + +$$ +u_{k+1}=a_{k} u_{k}+u_{k-1}, \quad v_{k+1}=a_{n-k} v_{k}+v_{k-1} \quad \text { for } k=1, \ldots, n-1 +$$ + +Prove that $u_{n}=v_{n}$. + +All three solutions above can be reformulated to prove this statement; one may prove + +$$ +u_{n}=v_{n}=\sum_{\substack{0=i_{0}0 +$$ + +or observe that + +$$ +\left(\begin{array}{c} +u_{k+1} \\ +u_{k} +\end{array}\right)=\left(\begin{array}{cc} +a_{k} & 1 \\ +1 & 0 +\end{array}\right)\left(\begin{array}{c} +u_{k} \\ +u_{k-1} +\end{array}\right) \quad \text { and } \quad\left(v_{k+1} ; v_{k}\right)=\left(v_{k} ; v_{k-1}\right)\left(\begin{array}{cc} +a_{k} & 1 \\ +1 & 0 +\end{array}\right) . +$$ + +Here we have + +$$ +\frac{u_{k+1}}{u_{k}}=a_{k}+\frac{1}{a_{k-1}+\frac{1}{a_{k-2}+\ldots+\frac{1}{a_{1}}}}=\left[a_{k} ; a_{k-1}, \ldots, a_{1}\right] +$$ + +and + +$$ +\frac{v_{k+1}}{v_{k}}=a_{n-k}+\frac{1}{a_{n-k+1}+\frac{1}{a_{n-k+2}+\ldots+\frac{1}{a_{n-1}}}}=\left[a_{n-k} ; a_{n-k+1}, \ldots, a_{n-1}\right] +$$ + +so this alternative statement is equivalent to the known fact that the continued fractions $\left[a_{n-1} ; a_{n-2}, \ldots, a_{1}\right]$ and $\left[a_{1} ; a_{2}, \ldots, a_{n-1}\right]$ have the same numerator. + +A2. Prove that in any set of 2000 distinct real numbers there exist two pairs $a>b$ and $c>d$ with $a \neq c$ or $b \neq d$, such that + +$$ +\left|\frac{a-b}{c-d}-1\right|<\frac{1}{100000} +$$ + +(Lithuania) + +Solution. For any set $S$ of $n=2000$ distinct real numbers, let $D_{1} \leqslant D_{2} \leqslant \cdots \leqslant D_{m}$ be the distances between them, displayed with their multiplicities. Here $m=n(n-1) / 2$. By rescaling the numbers, we may assume that the smallest distance $D_{1}$ between two elements of $S$ is $D_{1}=1$. Let $D_{1}=1=y-x$ for $x, y \in S$. Evidently $D_{m}=v-u$ is the difference between the largest element $v$ and the smallest element $u$ of $S$. + +If $D_{i+1} / D_{i}<1+10^{-5}$ for some $i=1,2, \ldots, m-1$ then the required inequality holds, because $0 \leqslant D_{i+1} / D_{i}-1<10^{-5}$. Otherwise, the reverse inequality + +$$ +\frac{D_{i+1}}{D_{i}} \geqslant 1+\frac{1}{10^{5}} +$$ + +holds for each $i=1,2, \ldots, m-1$, and therefore + +$$ +v-u=D_{m}=\frac{D_{m}}{D_{1}}=\frac{D_{m}}{D_{m-1}} \cdots \frac{D_{3}}{D_{2}} \cdot \frac{D_{2}}{D_{1}} \geqslant\left(1+\frac{1}{10^{5}}\right)^{m-1} . +$$ + +From $m-1=n(n-1) / 2-1=1000 \cdot 1999-1>19 \cdot 10^{5}$, together with the fact that for all $n \geqslant 1$, $\left(1+\frac{1}{n}\right)^{n} \geqslant 1+\left(\begin{array}{l}n \\ 1\end{array}\right) \cdot \frac{1}{n}=2$, we get + +$$ +\left(1+\frac{1}{10^{5}}\right)^{19 \cdot 10^{5}}=\left(\left(1+\frac{1}{10^{5}}\right)^{10^{5}}\right)^{19} \geqslant 2^{19}=2^{9} \cdot 2^{10}>500 \cdot 1000>2 \cdot 10^{5}, +$$ + +and so $v-u=D_{m}>2 \cdot 10^{5}$. + +Since the distance of $x$ to at least one of the numbers $u, v$ is at least $(u-v) / 2>10^{5}$, we have + +$$ +|x-z|>10^{5} . +$$ + +for some $z \in\{u, v\}$. Since $y-x=1$, we have either $z>y>x$ (if $z=v$ ) or $y>x>z$ (if $z=u$ ). If $z>y>x$, selecting $a=z, b=y, c=z$ and $d=x$ (so that $b \neq d$ ), we obtain + +$$ +\left|\frac{a-b}{c-d}-1\right|=\left|\frac{z-y}{z-x}-1\right|=\left|\frac{x-y}{z-x}\right|=\frac{1}{z-x}<10^{-5} . +$$ + +Otherwise, if $y>x>z$, we may choose $a=y, b=z, c=x$ and $d=z$ (so that $a \neq c$ ), and obtain + +$$ +\left|\frac{a-b}{c-d}-1\right|=\left|\frac{y-z}{x-z}-1\right|=\left|\frac{y-x}{x-z}\right|=\frac{1}{x-z}<10^{-5} +$$ + +The desired result follows. + +Comment. As the solution shows, the numbers 2000 and $\frac{1}{100000}$ appearing in the statement of the problem may be replaced by any $n \in \mathbb{Z}_{>0}$ and $\delta>0$ satisfying + +$$ +\delta(1+\delta)^{n(n-1) / 2-1}>2 +$$ + +A3. Let $\mathbb{Q}_{>0}$ be the set of positive rational numbers. Let $f: \mathbb{Q}_{>0} \rightarrow \mathbb{R}$ be a function satisfying the conditions + +$$ +\begin{aligned} +& f(x) f(y) \geqslant f(x y) \\ +& f(x+y) \geqslant f(x)+f(y) +\end{aligned} +$$ + +for all $x, y \in \mathbb{Q}_{>0}$. Given that $f(a)=a$ for some rational $a>1$, prove that $f(x)=x$ for all $x \in \mathbb{Q}_{>0}$. + +(Bulgaria) + +Solution. Denote by $\mathbb{Z}_{>0}$ the set of positive integers. + +Plugging $x=1, y=a$ into (1) we get $f(1) \geqslant 1$. Next, by an easy induction on $n$ we get from (2) that + +$$ +f(n x) \geqslant n f(x) \text { for all } n \in \mathbb{Z}_{>0} \text { and } x \in \mathbb{Q}_{>0} +$$ + +In particular, we have + +$$ +f(n) \geqslant n f(1) \geqslant n \quad \text { for all } n \in \mathbb{Z}_{>0} +$$ + +From (1) again we have $f(m / n) f(n) \geqslant f(m)$, so $f(q)>0$ for all $q \in \mathbb{Q}_{>0}$. + +Now, (2) implies that $f$ is strictly increasing; this fact together with (4) yields + +$$ +f(x) \geqslant f(\lfloor x\rfloor) \geqslant\lfloor x\rfloor>x-1 \quad \text { for all } x \geqslant 1 +$$ + +By an easy induction we get from (1) that $f(x)^{n} \geqslant f\left(x^{n}\right)$, so + +$$ +f(x)^{n} \geqslant f\left(x^{n}\right)>x^{n}-1 \quad \Longrightarrow \quad f(x) \geqslant \sqrt[n]{x^{n}-1} \text { for all } x>1 \text { and } n \in \mathbb{Z}_{>0} +$$ + +This yields + +$$ +f(x) \geqslant x \text { for every } x>1 \text {. } +$$ + +(Indeed, if $x>y>1$ then $x^{n}-y^{n}=(x-y)\left(x^{n-1}+x^{n-2} y+\cdots+y^{n}\right)>n(x-y)$, so for a large $n$ we have $x^{n}-1>y^{n}$ and thus $f(x)>y$.) + +Now, (1) and (5) give $a^{n}=f(a)^{n} \geqslant f\left(a^{n}\right) \geqslant a^{n}$, so $f\left(a^{n}\right)=a^{n}$. Now, for $x>1$ let us choose $n \in \mathbb{Z}_{>0}$ such that $a^{n}-x>1$. Then by (2) and (5) we get + +$$ +a^{n}=f\left(a^{n}\right) \geqslant f(x)+f\left(a^{n}-x\right) \geqslant x+\left(a^{n}-x\right)=a^{n} +$$ + +and therefore $f(x)=x$ for $x>1$. Finally, for every $x \in \mathbb{Q}_{>0}$ and every $n \in \mathbb{Z}_{>0}$, from (1) and (3) we get + +$$ +n f(x)=f(n) f(x) \geqslant f(n x) \geqslant n f(x) +$$ + +which gives $f(n x)=n f(x)$. Therefore $f(m / n)=f(m) / n=m / n$ for all $m, n \in \mathbb{Z}_{>0}$. + +Comment. The condition $f(a)=a>1$ is essential. Indeed, for $b \geqslant 1$ the function $f(x)=b x^{2}$ satisfies (1) and (2) for all $x, y \in \mathbb{Q}_{>0}$, and it has a unique fixed point $1 / b \leqslant 1$. + +A4. Let $n$ be a positive integer, and consider a sequence $a_{1}, a_{2}, \ldots, a_{n}$ of positive integers. Extend it periodically to an infinite sequence $a_{1}, a_{2}, \ldots$ by defining $a_{n+i}=a_{i}$ for all $i \geqslant 1$. If + +$$ +a_{1} \leqslant a_{2} \leqslant \cdots \leqslant a_{n} \leqslant a_{1}+n +$$ + +and + +$$ +a_{a_{i}} \leqslant n+i-1 \quad \text { for } i=1,2, \ldots, n +$$ + +prove that + +$$ +a_{1}+\cdots+a_{n} \leqslant n^{2} . +$$ + +(Germany) + +Solution 1. First, we claim that + +$$ +a_{i} \leqslant n+i-1 \quad \text { for } i=1,2, \ldots, n \text {. } +$$ + +Assume contrariwise that $i$ is the smallest counterexample. From $a_{n} \geqslant a_{n-1} \geqslant \cdots \geqslant a_{i} \geqslant n+i$ and $a_{a_{i}} \leqslant n+i-1$, taking into account the periodicity of our sequence, it follows that + +$$ +a_{i} \text { cannot be congruent to } i, i+1, \ldots, n-1 \text {, or } n(\bmod n) \text {. } +$$ + +Thus our assumption that $a_{i} \geqslant n+i$ implies the stronger statement that $a_{i} \geqslant 2 n+1$, which by $a_{1}+n \geqslant a_{n} \geqslant a_{i}$ gives $a_{1} \geqslant n+1$. The minimality of $i$ then yields $i=1$, and (4) becomes contradictory. This establishes our first claim. + +In particular we now know that $a_{1} \leqslant n$. If $a_{n} \leqslant n$, then $a_{1} \leqslant \cdots \leqslant \cdots a_{n} \leqslant n$ and the desired inequality holds trivially. Otherwise, consider the number $t$ with $1 \leqslant t \leqslant n-1$ such that + +$$ +a_{1} \leqslant a_{2} \leqslant \ldots \leqslant a_{t} \leqslant na_{a_{i}}$ ) belongs to $\left\{a_{i}+1, \ldots, n\right\}$, and for this reason $b_{i} \leqslant n-a_{i}$. + +It follows from the definition of the $b_{i} \mathrm{~s}$ and (5) that + +$$ +a_{t+1}+\ldots+a_{n} \leqslant n(n-t)+b_{1}+\ldots+b_{t} . +$$ + +Adding $a_{1}+\ldots+a_{t}$ to both sides and using that $a_{i}+b_{i} \leqslant n$ for $1 \leqslant i \leqslant t$, we get + +$$ +a_{1}+a_{2}+\cdots+a_{n} \leqslant n(n-t)+n t=n^{2} +$$ + +as we wished to prove. + +Solution 2. In the first quadrant of an infinite grid, consider the increasing "staircase" obtained by shading in dark the bottom $a_{i}$ cells of the $i$ th column for $1 \leqslant i \leqslant n$. We will prove that there are at most $n^{2}$ dark cells. + +To do it, consider the $n \times n$ square $S$ in the first quadrant with a vertex at the origin. Also consider the $n \times n$ square directly to the left of $S$. Starting from its lower left corner, shade in light the leftmost $a_{j}$ cells of the $j$ th row for $1 \leqslant j \leqslant n$. Equivalently, the light shading is obtained by reflecting the dark shading across the line $x=y$ and translating it $n$ units to the left. The figure below illustrates this construction for the sequence $6,6,6,7,7,7,8,12,12,14$. + +![](https://cdn.mathpix.com/cropped/2024_04_17_e0914cb4f8055c731538g-14.jpg?height=650&width=1052&top_left_y=667&top_left_x=534) + +We claim that there is no cell in $S$ which is both dark and light. Assume, contrariwise, that there is such a cell in column $i$. Consider the highest dark cell in column $i$ which is inside $S$. Since it is above a light cell and inside $S$, it must be light as well. There are two cases: + +Case 1. $a_{i} \leqslant n$ + +If $a_{i} \leqslant n$ then this dark and light cell is $\left(i, a_{i}\right)$, as highlighted in the figure. However, this is the $(n+i)$-th cell in row $a_{i}$, and we only shaded $a_{a_{i}}0$ such that $f(n)=b-1$; but then $f^{3}(n-1)=f(n)+1=b$, so $b \in R_{3}$. + +This yields + +$$ +3 k=\left|S_{1} \cup S_{2} \cup S_{3}\right| \leqslant 1+1+\left|S_{1}\right|=k+2, +$$ + +or $k \leqslant 1$. Therefore $k=1$, and the inequality above comes to equality. So we have $S_{1}=\{a\}$, $S_{2}=\{f(a)\}$, and $S_{3}=\left\{f^{2}(a)\right\}$ for some $a \in \mathbb{Z}_{\geqslant 0}$, and each one of the three options (i), (ii), and (iii) should be realized exactly once, which means that + +$$ +\left\{a, f(a), f^{2}(a)\right\}=\{0, a+1, f(0)+1\} . +$$ + +III. From (3), we get $a+1 \in\left\{f(a), f^{2}(a)\right\}$ (the case $a+1=a$ is impossible). If $a+1=f^{2}(a)$ then we have $f(a+1)=f^{3}(a)=f(a+1)+1$ which is absurd. Therefore + +$$ +f(a)=a+1 +$$ + +Next, again from (3) we have $0 \in\left\{a, f^{2}(a)\right\}$. Let us consider these two cases separately. Case 1. Assume that $a=0$, then $f(0)=f(a)=a+1=1$. Also from (3) we get $f(1)=f^{2}(a)=$ $f(0)+1=2$. Now, let us show that $f(n)=n+1$ by induction on $n$; the base cases $n \leqslant 1$ are established. Next, if $n \geqslant 2$ then the induction hypothesis implies + +$$ +n+1=f(n-1)+1=f^{3}(n-2)=f^{2}(n-1)=f(n), +$$ + +establishing the step. In this case we have obtained the first of two answers; checking that is satisfies (*) is straightforward. + +Case 2. Assume now that $f^{2}(a)=0$; then by (3) we get $a=f(0)+1$. By (4) we get $f(a+1)=$ $f^{2}(a)=0$, then $f(0)=f^{3}(a)=f(a+1)+1=1$, hence $a=f(0)+1=2$ and $f(2)=3$ by (4). To summarize, + +$$ +f(0)=1, \quad f(2)=3, \quad f(3)=0 . +$$ + +Now let us prove by induction on $m$ that (1) holds for all $n=4 k, 4 k+2,4 k+3$ with $k \leqslant m$ and for all $n=4 k+1$ with $k0$ and $B(1)=m+2>0$ since $n=2 m$. + +Therefore $B(x)=A(x+a+b)$. Writing $c=a+b \geqslant 1$ we compute + +$$ +0=A(x+c)-B(x)=(3 c-2 m) x^{2}+c(3 c-2 m) x+c^{2}(c-m) . +$$ + +Then we must have $3 c-2 m=c-m=0$, which gives $m=0$, a contradiction. We conclude that $f(x)=t x$ is the only solution. + +Solution 2. Multiplying (1) by $x$, we rewrite it as + +$$ +x\left(x^{3}-m x^{2}+1\right) P(x+1)+x\left(x^{3}+m x^{2}+1\right) P(x-1)=[(x+1)+(x-1)]\left(x^{3}-m x+1\right) P(x) . +$$ + +After regrouping, it becomes + +$$ +\left(x^{3}-m x^{2}+1\right) Q(x)=\left(x^{3}+m x^{2}+1\right) Q(x-1) \text {, } +$$ + +where $Q(x)=x P(x+1)-(x+1) P(x)$. If $\operatorname{deg} P \geqslant 2$ then $\operatorname{deg} Q=\operatorname{deg} P$, so $Q(x)$ has a finite multiset of complex roots, which we denote $R_{Q}$. Each root is taken with its multiplicity. Then the multiset of complex roots of $Q(x-1)$ is $R_{Q}+1=\left\{z+1: z \in R_{Q}\right\}$. + +Let $\left\{x_{1}, x_{2}, x_{3}\right\}$ and $\left\{y_{1}, y_{2}, y_{3}\right\}$ be the multisets of roots of the polynomials $A(x)=x^{3}-m x^{2}+1$ and $B(x)=x^{3}+m x^{2}+1$, respectively. From (2) we get the equality of multisets + +$$ +\left\{x_{1}, x_{2}, x_{3}\right\} \cup R_{Q}=\left\{y_{1}, y_{2}, y_{3}\right\} \cup\left(R_{Q}+1\right) . +$$ + +For every $r \in R_{Q}$, since $r+1$ is in the set of the right hand side, we must have $r+1 \in R_{Q}$ or $r+1=x_{i}$ for some $i$. Similarly, since $r$ is in the set of the left hand side, either $r-1 \in R_{Q}$ or $r=y_{i}$ for some $i$. This implies that, possibly after relabelling $y_{1}, y_{2}, y_{3}$, all the roots of (2) may be partitioned into three chains of the form $\left\{y_{i}, y_{i}+1, \ldots, y_{i}+k_{i}=x_{i}\right\}$ for $i=1,2,3$ and some integers $k_{1}, k_{2}, k_{3} \geqslant 0$. + +Now we analyze the roots of the polynomial $A_{a}(x)=x^{3}+a x^{2}+1$. Using calculus or elementary methods, we find that the local extrema of $A_{a}(x)$ occur at $x=0$ and $x=-2 a / 3$; their values are $A_{a}(0)=1>0$ and $A_{a}(-2 a / 3)=1+4 a^{3} / 27$, which is positive for integers $a \geqslant-1$ and negative for integers $a \leqslant-2$. So when $a \in \mathbb{Z}, A_{a}$ has three real roots if $a \leqslant-2$ and one if $a \geqslant-1$. + +Now, since $y_{i}-x_{i} \in \mathbb{Z}$ for $i=1,2,3$, the cubics $A_{m}$ and $A_{-m}$ must have the same number of real roots. The previous analysis then implies that $m=1$ or $m=-1$. Therefore the real root $\alpha$ of $A_{1}(x)=x^{3}+x^{2}+1$ and the real root $\beta$ of $A_{-1}(x)=x^{3}-x^{2}+1$ must differ by an integer. But this is impossible, because $A_{1}\left(-\frac{3}{2}\right)=-\frac{1}{8}$ and $A_{1}(-1)=1$ so $-1.5<\alpha<-1$, while $A_{-1}(-1)=-1$ and $A_{-1}\left(-\frac{1}{2}\right)=\frac{5}{8}$, so $-1<\beta<-0.5$. + +It follows that $\operatorname{deg} P \leqslant 1$. Then, as shown in Solution 1, we conclude that the solutions are $P(x)=t x$ for all real numbers $t$. + +## Combinatorics + +C1. Let $n$ be a positive integer. Find the smallest integer $k$ with the following property: Given any real numbers $a_{1}, \ldots, a_{d}$ such that $a_{1}+a_{2}+\cdots+a_{d}=n$ and $0 \leqslant a_{i} \leqslant 1$ for $i=1,2, \ldots, d$, it is possible to partition these numbers into $k$ groups (some of which may be empty) such that the sum of the numbers in each group is at most 1 . + +(Poland) + +Answer. $k=2 n-1$. + +Solution 1. If $d=2 n-1$ and $a_{1}=\cdots=a_{2 n-1}=n /(2 n-1)$, then each group in such a partition can contain at most one number, since $2 n /(2 n-1)>1$. Therefore $k \geqslant 2 n-1$. It remains to show that a suitable partition into $2 n-1$ groups always exists. + +We proceed by induction on $d$. For $d \leqslant 2 n-1$ the result is trivial. If $d \geqslant 2 n$, then since + +$$ +\left(a_{1}+a_{2}\right)+\ldots+\left(a_{2 n-1}+a_{2 n}\right) \leqslant n +$$ + +we may find two numbers $a_{i}, a_{i+1}$ such that $a_{i}+a_{i+1} \leqslant 1$. We "merge" these two numbers into one new number $a_{i}+a_{i+1}$. By the induction hypothesis, a suitable partition exists for the $d-1$ numbers $a_{1}, \ldots, a_{i-1}, a_{i}+a_{i+1}, a_{i+2}, \ldots, a_{d}$. This induces a suitable partition for $a_{1}, \ldots, a_{d}$. + +Solution 2. We will show that it is even possible to split the sequence $a_{1}, \ldots, a_{d}$ into $2 n-1$ contiguous groups so that the sum of the numbers in each groups does not exceed 1. Consider a segment $S$ of length $n$, and partition it into segments $S_{1}, \ldots, S_{d}$ of lengths $a_{1}, \ldots, a_{d}$, respectively, as shown below. Consider a second partition of $S$ into $n$ equal parts by $n-1$ "empty dots". + +| $a_{1}$ | $a_{2}$ | $a_{3}$ | $a_{4}$ | $a_{5}$ | $a_{6}$ | $a_{7}$ | $a_{8}, a_{9}, a_{10}$ | +| :--- | :--- | :--- | :--- | :--- | :--- | :--- | :--- | :--- | +| | | | | | | | | + +Assume that the $n-1$ empty dots are in segments $S_{i_{1}}, \ldots, S_{i_{n-1}}$. (If a dot is on the boundary of two segments, we choose the right segment). These $n-1$ segments are distinct because they have length at most 1. Consider the partition: + +$$ +\left\{a_{1}, \ldots, a_{i_{1}-1}\right\},\left\{a_{i_{1}}\right\},\left\{a_{i_{1}+1}, \ldots, a_{i_{2}-1}\right\},\left\{a_{i_{2}}\right\}, \ldots\left\{a_{i_{n-1}}\right\},\left\{a_{i_{n-1}+1}, \ldots, a_{d}\right\} +$$ + +In the example above, this partition is $\left\{a_{1}, a_{2}\right\},\left\{a_{3}\right\},\left\{a_{4}, a_{5}\right\},\left\{a_{6}\right\}, \varnothing,\left\{a_{7}\right\},\left\{a_{8}, a_{9}, a_{10}\right\}$. We claim that in this partition, the sum of the numbers in this group is at most 1 . + +For the sets $\left\{a_{i_{t}}\right\}$ this is obvious since $a_{i_{t}} \leqslant 1$. For the sets $\left\{a_{i_{t}}+1, \ldots, a_{i_{t+1}-1}\right\}$ this follows from the fact that the corresponding segments lie between two neighboring empty dots, or between an endpoint of $S$ and its nearest empty dot. Therefore the sum of their lengths cannot exceed 1. + +Solution 3. First put all numbers greater than $\frac{1}{2}$ in their own groups. Then, form the remaining groups as follows: For each group, add new $a_{i} \mathrm{~S}$ one at a time until their sum exceeds $\frac{1}{2}$. Since the last summand is at most $\frac{1}{2}$, this group has sum at most 1 . Continue this procedure until we have used all the $a_{i}$ s. Notice that the last group may have sum less than $\frac{1}{2}$. If the sum of the numbers in the last two groups is less than or equal to 1, we merge them into one group. In the end we are left with $m$ groups. If $m=1$ we are done. Otherwise the first $m-2$ have sums greater than $\frac{1}{2}$ and the last two have total sum greater than 1 . Therefore $n>(m-2) / 2+1$ so $m \leqslant 2 n-1$ as desired. + +Comment 1. The original proposal asked for the minimal value of $k$ when $n=2$. + +Comment 2. More generally, one may ask the same question for real numbers between 0 and 1 whose sum is a real number $r$. In this case the smallest value of $k$ is $k=\lceil 2 r\rceil-1$, as Solution 3 shows. + +Solutions 1 and 2 lead to the slightly weaker bound $k \leqslant 2\lceil r\rceil-1$. This is actually the optimal bound for partitions into consecutive groups, which are the ones contemplated in these two solutions. To see this, assume that $r$ is not an integer and let $c=(r+1-\lceil r\rceil) /(1+\lceil r\rceil)$. One easily checks that $0N$. + +C3. A crazy physicist discovered a new kind of particle which he called an imon, after some of them mysteriously appeared in his lab. Some pairs of imons in the lab can be entangled, and each imon can participate in many entanglement relations. The physicist has found a way to perform the following two kinds of operations with these particles, one operation at a time. + +(i) If some imon is entangled with an odd number of other imons in the lab, then the physicist can destroy it. + +(ii) At any moment, he may double the whole family of imons in his lab by creating a copy $I^{\prime}$ of each imon $I$. During this procedure, the two copies $I^{\prime}$ and $J^{\prime}$ become entangled if and only if the original imons $I$ and $J$ are entangled, and each copy $I^{\prime}$ becomes entangled with its original imon $I$; no other entanglements occur or disappear at this moment. + +Prove that the physicist may apply a sequence of such operations resulting in a family of imons, no two of which are entangled. + +(Japan) + +Solution 1. Let us consider a graph with the imons as vertices, and two imons being connected if and only if they are entangled. Recall that a proper coloring of a graph $G$ is a coloring of its vertices in several colors so that every two connected vertices have different colors. + +Lemma. Assume that a graph $G$ admits a proper coloring in $n$ colors $(n>1)$. Then one may perform a sequence of operations resulting in a graph which admits a proper coloring in $n-1$ colors. + +Proof. Let us apply repeatedly operation $(i)$ to any appropriate vertices while it is possible. Since the number of vertices decreases, this process finally results in a graph where all the degrees are even. Surely this graph also admits a proper coloring in $n$ colors $1, \ldots, n$; let us fix this coloring. + +Now apply the operation (ii) to this graph. A proper coloring of the resulting graph in $n$ colors still exists: one may preserve the colors of the original vertices and color the vertex $I^{\prime}$ in a color $k+1(\bmod n)$ if the vertex $I$ has color $k$. Then two connected original vertices still have different colors, and so do their two connected copies. On the other hand, the vertices $I$ and $I^{\prime}$ have different colors since $n>1$. + +All the degrees of the vertices in the resulting graph are odd, so one may apply operation $(i)$ to delete consecutively all the vertices of color $n$ one by one; no two of them are connected by an edge, so their degrees do not change during the process. Thus, we obtain a graph admitting a proper coloring in $n-1$ colors, as required. The lemma is proved. + +Now, assume that a graph $G$ has $n$ vertices; then it admits a proper coloring in $n$ colors. Applying repeatedly the lemma we finally obtain a graph admitting a proper coloring in one color, that is - a graph with no edges, as required. + +Solution 2. Again, we will use the graph language. + +I. We start with the following observation. + +Lemma. Assume that a graph $G$ contains an isolated vertex $A$, and a graph $G^{\circ}$ is obtained from $G$ by deleting this vertex. Then, if one can apply a sequence of operations which makes a graph with no edges from $G^{\circ}$, then such a sequence also exists for $G$. + +Proof. Consider any operation applicable to $G^{\circ}$ resulting in a graph $G_{1}^{\circ}$; then there exists a sequence of operations applicable to $G$ and resulting in a graph $G_{1}$ differing from $G_{1}^{\circ}$ by an addition of an isolated vertex $A$. Indeed, if this operation is of type $(i)$, then one may simply repeat it in $G$. + +Otherwise, the operation is of type (ii), and one may apply it to $G$ and then delete the vertex $A^{\prime}$ (it will have degree 1 ). + +Thus one may change the process for $G^{\circ}$ into a corresponding process for $G$ step by step. + +In view of this lemma, if at some moment a graph contains some isolated vertex, then we may simply delete it; let us call this operation (iii). + +II. Let $V=\left\{A_{1}^{0}, \ldots, A_{n}^{0}\right\}$ be the vertices of the initial graph. Let us describe which graphs can appear during our operations. Assume that operation (ii) was applied $m$ times. If these were the only operations applied, then the resulting graph $G_{n}^{m}$ has the set of vertices which can be enumerated as + +$$ +V_{n}^{m}=\left\{A_{i}^{j}: 1 \leqslant i \leqslant n, 0 \leqslant j \leqslant 2^{m}-1\right\} +$$ + +where $A_{i}^{0}$ is the common "ancestor" of all the vertices $A_{i}^{j}$, and the binary expansion of $j$ (adjoined with some zeroes at the left to have $m$ digits) "keeps the history" of this vertex: the $d$ th digit from the right is 0 if at the $d$ th doubling the ancestor of $A_{i}^{j}$ was in the original part, and this digit is 1 if it was in the copy. + +Next, the two vertices $A_{i}^{j}$ and $A_{k}^{\ell}$ in $G_{n}^{m}$ are connected with an edge exactly if either (1) $j=\ell$ and there was an edge between $A_{i}^{0}$ and $A_{k}^{0}$ (so these vertices appeared at the same application of operation (ii)); or (2) $i=k$ and the binary expansions of $j$ and $\ell$ differ in exactly one digit (so their ancestors became connected as a copy and the original vertex at some application of (ii)). + +Now, if some operations $(i)$ were applied during the process, then simply some vertices in $G_{n}^{m}$ disappeared. So, in any case the resulting graph is some induced subgraph of $G_{n}^{m}$. + +III. Finally, we will show that from each (not necessarily induced) subgraph of $G_{n}^{m}$ one can obtain a graph with no vertices by applying operations $(i),(i i)$ and $(i i i)$. We proceed by induction on $n$; the base case $n=0$ is trivial. + +For the induction step, let us show how to apply several operations so as to obtain a graph containing no vertices of the form $A_{n}^{j}$ for $j \in \mathbb{Z}$. We will do this in three steps. + +Step 1. We apply repeatedly operation (i) to any appropriate vertices while it is possible. In the resulting graph, all vertices have even degrees. + +Step 2. Apply operation (ii) obtaining a subgraph of $G_{n}^{m+1}$ with all degrees being odd. In this graph, we delete one by one all the vertices $A_{n}^{j}$ where the sum of the binary digits of $j$ is even; it is possible since there are no edges between such vertices, so all their degrees remain odd. After that, we delete all isolated vertices. + +Step 3. Finally, consider any remaining vertex $A_{n}^{j}$ (then the sum of digits of $j$ is odd). If its degree is odd, then we simply delete it. Otherwise, since $A_{n}^{j}$ is not isolated, we consider any vertex adjacent to it. It has the form $A_{k}^{j}$ for some $k\sqrt[3]{6 n}$, we will prove that there exist subsets $X$ and $Y$ of $S$ such that $|X|<|Y|$ and $\sum_{x \in X} x=\sum_{y \in Y} y$. Then, deleting the elements of $Y$ from our partition and adding the elements of $X$ to it, we obtain an $A$-partition of $n$ into less than $k_{\text {min }}$ parts, which is the desired contradiction. + +For each positive integer $k \leqslant s$, we consider the $k$-element subset + +$$ +S_{1,0}^{k}:=\left\{b_{1}, \ldots, b_{k}\right\} +$$ + +as well as the following $k$-element subsets $S_{i, j}^{k}$ of $S$ : + +$$ +S_{i, j}^{k}:=\left\{b_{1}, \ldots, b_{k-i}, b_{k-i+j+1}, b_{s-i+2}, \ldots, b_{s}\right\}, \quad i=1, \ldots, k, \quad j=1, \ldots, s-k +$$ + +Pictorially, if we represent the elements of $S$ by a sequence of dots in increasing order, and represent a subset of $S$ by shading in the appropriate dots, we have: + +![](https://cdn.mathpix.com/cropped/2024_04_17_e0914cb4f8055c731538g-27.jpg?height=124&width=967&top_left_y=1388&top_left_x=579) + +Denote by $\Sigma_{i, j}^{k}$ the sum of elements in $S_{i, j}^{k}$. Clearly, $\Sigma_{1,0}^{k}$ is the minimum sum of a $k$-element subset of $S$. Next, for all appropriate indices $i$ and $j$ we have + +$$ +\Sigma_{i, j}^{k}=\Sigma_{i, j+1}^{k}+b_{k-i+j+1}-b_{k-i+j+2}<\Sigma_{i, j+1}^{k} \quad \text { and } \quad \sum_{i, s-k}^{k}=\sum_{i+1,1}^{k}+b_{k-i}-b_{k-i+1}<\Sigma_{i+1,1}^{k} \text {. } +$$ + +Therefore + +$$ +1 \leqslant \Sigma_{1,0}^{k}<\Sigma_{1,1}^{k}<\Sigma_{1,2}^{k}<\cdots<\Sigma_{1, s-k}^{k}<\Sigma_{2,1}^{k}<\cdots<\Sigma_{2, s-k}^{k}<\Sigma_{3,1}^{k}<\cdots<\Sigma_{k, s-k}^{k} \leqslant n . +$$ + +To see this in the picture, we start with the $k$ leftmost points marked. At each step, we look for the rightmost point which can move to the right, and move it one unit to the right. We continue until the $k$ rightmost points are marked. As we do this, the corresponding sums clearly increase. + +For each $k$ we have found $k(s-k)+1$ different integers of the form $\Sigma_{i, j}^{k}$ between 1 and $n$. As we vary $k$, the total number of integers we are considering is + +$$ +\sum_{k=1}^{s}(k(s-k)+1)=s \cdot \frac{s(s+1)}{2}-\frac{s(s+1)(2 s+1)}{6}+s=\frac{s\left(s^{2}+5\right)}{6}>\frac{s^{3}}{6}>n . +$$ + +Since they are between 1 and $n$, at least two of these integers are equal. Consequently, there exist $1 \leqslant k\sqrt[3]{6 n}>1$. Without loss of generality we assume that $a_{k_{\min }}=b_{s}$. Let us distinguish two cases. + +Case 1. $b_{s} \geqslant \frac{s(s-1)}{2}+1$. + +Consider the partition $n-b_{s}=a_{1}+\cdots+a_{k_{\min }-1}$, which is clearly a minimum $A$-partition of $n-b_{s}$ with at least $s-1 \geqslant 1$ different parts. Now, from $n<\frac{s^{3}}{6}$ we obtain + +$$ +n-b_{s} \leqslant n-\frac{s(s-1)}{2}-1<\frac{s^{3}}{6}-\frac{s(s-1)}{2}-1<\frac{(s-1)^{3}}{6} +$$ + +so $s-1>\sqrt[3]{6\left(n-b_{s}\right)}$, which contradicts the choice of $n$. + +Case 2. $b_{s} \leqslant \frac{s(s-1)}{2}$. + +Set $b_{0}=0, \Sigma_{0,0}=0$, and $\Sigma_{i, j}=b_{1}+\cdots+b_{i-1}+b_{j}$ for $1 \leqslant i \leqslant jb_{s}$ such sums; so at least two of them, say $\Sigma_{i, j}$ and $\Sigma_{i^{\prime}, j^{\prime}}$, are congruent modulo $b_{s}$ (where $(i, j) \neq\left(i^{\prime}, j^{\prime}\right)$ ). This means that $\Sigma_{i, j}-\Sigma_{i^{\prime}, j^{\prime}}=r b_{s}$ for some integer $r$. Notice that for $i \leqslant ji^{\prime}$. Next, we observe that $\Sigma_{i, j}-\Sigma_{i^{\prime}, j^{\prime}}=\left(b_{i^{\prime}}-b_{j^{\prime}}\right)+b_{j}+b_{i^{\prime}+1}+\cdots+b_{i-1}$ and $b_{i^{\prime}} \leqslant b_{j^{\prime}}$ imply + +$$ +-b_{s}<-b_{j^{\prime}}<\Sigma_{i, j}-\Sigma_{i^{\prime}, j^{\prime}}<\left(i-i^{\prime}\right) b_{s}, +$$ + +so $0 \leqslant r \leqslant i-i^{\prime}-1$. + +Thus, we may remove the $i$ terms of $\Sigma_{i, j}$ in our $A$-partition, and replace them by the $i^{\prime}$ terms of $\Sigma_{i^{\prime}, j^{\prime}}$ and $r$ terms equal to $b_{s}$, for a total of $r+i^{\prime}a_{k+1} \geqslant 1$, which shows that + +$$ +n=a_{1}+\ldots+a_{k+1} +$$ + +is an $A$-partition of $n$ into $k+1$ different parts. Since $k h3$. Finally, in the sequence $d(a, y), d\left(a, b_{y}\right), d\left(a, c_{y}\right), d\left(a, d_{y}\right)$ the neighboring terms differ by at most 1 , the first term is less than 3 , and the last one is greater than 3 ; thus there exists one which is equal to 3 , as required. + +Comment 1. The upper bound 2550 is sharp. This can be seen by means of various examples; one of them is the "Roman Empire": it has one capital, called "Rome", that is connected to 51 semicapitals by internally disjoint paths of length 3. Moreover, each of these semicapitals is connected to 50 rural cities by direct flights. + +Comment 2. Observe that, under the conditions of the problem, there exists no bound for the size of $S_{1}(x)$ or $S_{2}(x)$. + +Comment 3. The numbers 100 and 2550 appearing in the statement of the problem may be replaced by $n$ and $\left\lfloor\frac{(n+1)^{2}}{4}\right\rfloor$ for any positive integer $n$. Still more generally, one can also replace the pair $(3,4)$ of distances under consideration by any pair $(r, s)$ of positive integers satisfying $rn$ is equivalent to the case $t0$ is very small. + +![](https://cdn.mathpix.com/cropped/2024_04_17_e0914cb4f8055c731538g-35.jpg?height=585&width=607&top_left_y=797&top_left_x=756) + +Figure 3 + +If $0 \leqslant a, b, c, d \leqslant n$ satisfy $a+c=b+d$, then $a \alpha+c \alpha=b \alpha+d \alpha$, so the chord from $a$ to $c$ is parallel to the chord from $b$ to $d$ in $A(\alpha)$. Hence in a cyclic arrangement all $k$-chords are parallel. In particular every cyclic arrangement is beautiful. + +Next we show that there are exactly $N+1$ distinct cyclic arrangements. To see this, let us see how $A(\alpha)$ changes as we increase $\alpha$ from 0 to 1 . The order of points $p$ and $q$ changes precisely when we cross a value $\alpha=f$ such that $\{p f\}=\{q f\}$; this can only happen if $f$ is one of the $N$ fractions $f_{1}, \ldots, f_{N}$. Therefore there are at most $N+1$ different cyclic arrangements. + +To show they are all distinct, recall that $f_{i}=a_{i} / b_{i}$ and let $\epsilon>0$ be a very small number. In the arrangement $A\left(f_{i}+\epsilon\right)$, point $k$ lands at $\frac{k a_{i}\left(\bmod b_{i}\right)}{b_{i}}+k \epsilon$. Therefore the points are grouped into $b_{i}$ clusters next to the points $0, \frac{1}{b_{i}}, \ldots, \frac{b_{i}-1}{b_{i}}$ of the circle. The cluster following $\frac{k}{b_{i}}$ contains the numbers congruent to $k a_{i}^{-1}$ modulo $b_{i}$, listed clockwise in increasing order. It follows that the first number after 0 in $A\left(f_{i}+\epsilon\right)$ is $b_{i}$, and the first number after 0 which is less than $b_{i}$ is $a_{i}^{-1}\left(\bmod b_{i}\right)$, which uniquely determines $a_{i}$. In this way we can recover $f_{i}$ from the cyclic arrangement. Note also that $A\left(f_{i}+\epsilon\right)$ is not the trivial arrangement where we list $0,1, \ldots, n$ in order clockwise. It follows that the $N+1$ cyclic arrangements $A(\epsilon), A\left(f_{1}+\epsilon\right), \ldots, A\left(f_{N}+\epsilon\right)$ are distinct. + +Let us record an observation which will be useful later: + +$$ +\text { if } f_{i}<\alpha1 / 2^{m}$. + +Next, any interval blackened by $B$ before the $r$ th move which intersects $\left(x_{r}, x_{r+1}\right)$ should be contained in $\left[x_{r}, x_{r+1}\right]$; by (ii), all such intervals have different lengths not exceeding $1 / 2^{m}$, so the total amount of ink used for them is less than $2 / 2^{m}$. Thus, the amount of ink used for the segment $\left[0, x_{r+1}\right]$ does not exceed the sum of $2 / 2^{m}, 3 x_{r}$ (used for $\left[0, x_{r}\right]$ ), and $1 / 2^{m}$ used for the +segment $I_{0}^{r}$. In total it gives at most $3\left(x_{r}+1 / 2^{m}\right)<3\left(x_{r}+\alpha\right)=3 x_{r+1}$. Thus condition $(i)$ is also verified in this case. The claim is proved. + +Finally, we can perform the desired estimation. Consider any situation in the game, say after the $(r-1)$ st move; assume that the segment $[0,1]$ is not completely black. By $(i i)$, in the segment $\left[x_{r}, 1\right]$ player $B$ has colored several segments of different lengths; all these lengths are negative powers of 2 not exceeding $1-x_{r}$; thus the total amount of ink used for this interval is at most $2\left(1-x_{r}\right)$. Using $(i)$, we obtain that the total amount of ink used is at most $3 x_{r}+2\left(1-x_{r}\right)<3$. Thus the pot is not empty, and therefore $A$ never wins. + +Comment 1. Notice that this strategy works even if the pot contains initially only 3 units of ink. + +Comment 2. There exist other strategies for $B$ allowing him to prevent emptying the pot before the whole interval is colored. On the other hand, let us mention some idea which does not work. + +Player $B$ could try a strategy in which the set of blackened points in each round is an interval of the type $[0, x]$. Such a strategy cannot work (even if there is more ink available). Indeed, under the assumption that $B$ uses such a strategy, let us prove by induction on $s$ the following statement: + +For any positive integer $s$, player $A$ has a strategy picking only positive integers $m \leqslant s$ in which, if player $B$ ever paints a point $x \geqslant 1-1 / 2^{s}$ then after some move, exactly the interval $\left[0,1-1 / 2^{s}\right]$ is blackened, and the amount of ink used up to this moment is at least s/2. + +For the base case $s=1$, player $A$ just picks $m=1$ in the first round. If for some positive integer $k$ player $A$ has such a strategy, for $s+1$ he can first rescale his strategy to the interval $[0,1 / 2]$ (sending in each round half of the amount of ink he would give by the original strategy). Thus, after some round, the interval $\left[0,1 / 2-1 / 2^{s+1}\right]$ becomes blackened, and the amount of ink used is at least $s / 4$. Now player $A$ picks $m=1 / 2$, and player $B$ spends $1 / 2$ unit of ink to blacken the interval [0,1/2]. After that, player $A$ again rescales his strategy to the interval $[1 / 2,1]$, and player $B$ spends at least $s / 4$ units of ink to blacken the interval $\left[1 / 2,1-1 / 2^{s+1}\right]$, so he spends in total at least $s / 4+1 / 2+s / 4=(s+1) / 2$ units of ink. + +Comment 3. In order to avoid finiteness issues, the statement could be replaced by the following one: + +Players $A$ and $B$ play a paintful game on the real numbers. Player $A$ has a paint pot with four units of black ink. A quantity $p$ of this ink suffices to blacken a (closed) real interval of length $p$. In the beginning of the game, player $A$ chooses (and announces) a positive integer $N$. In every round, player $A$ picks some positive integer $m \leqslant N$ and provides $1 / 2^{m}$ units of ink from the pot. The player $B$ picks an integer $k$ and blackens the interval from $k / 2^{m}$ to $(k+1) / 2^{m}$ (some parts of this interval may happen to be blackened before). The goal of player $A$ is to reach a situation where the pot is empty and the interval $[0,1]$ is not completely blackened. + +Decide whether there exists a strategy for player A to win. + +However, the Problem Selection Committee believes that this version may turn out to be harder than the original one. + +## Geometry + +G1. Let $A B C$ be an acute-angled triangle with orthocenter $H$, and let $W$ be a point on side $B C$. Denote by $M$ and $N$ the feet of the altitudes from $B$ and $C$, respectively. Denote by $\omega_{1}$ the circumcircle of $B W N$, and let $X$ be the point on $\omega_{1}$ which is diametrically opposite to $W$. Analogously, denote by $\omega_{2}$ the circumcircle of $C W M$, and let $Y$ be the point on $\omega_{2}$ which is diametrically opposite to $W$. Prove that $X, Y$ and $H$ are collinear. + +(Thaliand) + +Solution. Let $L$ be the foot of the altitude from $A$, and let $Z$ be the second intersection point of circles $\omega_{1}$ and $\omega_{2}$, other than $W$. We show that $X, Y, Z$ and $H$ lie on the same line. + +Due to $\angle B N C=\angle B M C=90^{\circ}$, the points $B, C, N$ and $M$ are concyclic; denote their circle by $\omega_{3}$. Observe that the line $W Z$ is the radical axis of $\omega_{1}$ and $\omega_{2}$; similarly, $B N$ is the radical axis of $\omega_{1}$ and $\omega_{3}$, and $C M$ is the radical axis of $\omega_{2}$ and $\omega_{3}$. Hence $A=B N \cap C M$ is the radical center of the three circles, and therefore $W Z$ passes through $A$. + +Since $W X$ and $W Y$ are diameters in $\omega_{1}$ and $\omega_{2}$, respectively, we have $\angle W Z X=\angle W Z Y=90^{\circ}$, so the points $X$ and $Y$ lie on the line through $Z$, perpendicular to $W Z$. + +![](https://cdn.mathpix.com/cropped/2024_04_17_e0914cb4f8055c731538g-39.jpg?height=770&width=1114&top_left_y=1103&top_left_x=511) + +The quadrilateral $B L H N$ is cyclic, because it has two opposite right angles. From the power of $A$ with respect to the circles $\omega_{1}$ and $B L H N$ we find $A L \cdot A H=A B \cdot A N=A W \cdot A Z$. If $H$ lies on the line $A W$ then this implies $H=Z$ immediately. Otherwise, by $\frac{A Z}{A H}=\frac{A L}{A W}$ the triangles $A H Z$ and $A W L$ are similar. Then $\angle H Z A=\angle W L A=90^{\circ}$, so the point $H$ also lies on the line $X Y Z$. + +Comment. The original proposal also included a second statement: + +Let $P$ be the point on $\omega_{1}$ such that $W P$ is parallel to $C N$, and let $Q$ be the point on $\omega_{2}$ such that $W Q$ is parallel to $B M$. Prove that $P, Q$ and $H$ are collinear if and only if $B W=C W$ or $A W \perp B C$. + +The Problem Selection Committee considered the first part more suitable for the competition. + +G2. Let $\omega$ be the circumcircle of a triangle $A B C$. Denote by $M$ and $N$ the midpoints of the sides $A B$ and $A C$, respectively, and denote by $T$ the midpoint of the $\operatorname{arc} B C$ of $\omega$ not containing $A$. The circumcircles of the triangles $A M T$ and $A N T$ intersect the perpendicular bisectors of $A C$ and $A B$ at points $X$ and $Y$, respectively; assume that $X$ and $Y$ lie inside the triangle $A B C$. The lines $M N$ and $X Y$ intersect at $K$. Prove that $K A=K T$. + +(Iran) + +Solution 1. Let $O$ be the center of $\omega$, thus $O=M Y \cap N X$. Let $\ell$ be the perpendicular bisector of $A T$ (it also passes through $O$ ). Denote by $r$ the operation of reflection about $\ell$. Since $A T$ is the angle bisector of $\angle B A C$, the line $r(A B)$ is parallel to $A C$. Since $O M \perp A B$ and $O N \perp A C$, this means that the line $r(O M)$ is parallel to the line $O N$ and passes through $O$, so $r(O M)=O N$. Finally, the circumcircle $\gamma$ of the triangle $A M T$ is symmetric about $\ell$, so $r(\gamma)=\gamma$. Thus the point $M$ maps to the common point of $O N$ with the arc $A M T$ of $\gamma$ - that is, $r(M)=X$. + +Similarly, $r(N)=Y$. Thus, we get $r(M N)=X Y$, and the common point $K$ of $M N$ nd $X Y$ lies on $\ell$. This means exactly that $K A=K T$. + +![](https://cdn.mathpix.com/cropped/2024_04_17_e0914cb4f8055c731538g-40.jpg?height=748&width=707&top_left_y=981&top_left_x=709) + +Solution 2. Let $L$ be the second common point of the line $A C$ with the circumcircle $\gamma$ of the triangle $A M T$. From the cyclic quadrilaterals $A B T C$ and $A M T L$ we get $\angle B T C=180^{\circ}-$ $\angle B A C=\angle M T L$, which implies $\angle B T M=\angle C T L$. Since $A T$ is an angle bisector in these quadrilaterals, we have $B T=T C$ and $M T=T L$. Thus the triangles $B T M$ and $C T L$ are congruent, so $C L=B M=A M$. + +Let $X^{\prime}$ be the common point of the line $N X$ with the external bisector of $\angle B A C$; notice that it lies outside the triangle $A B C$. Then we have $\angle T A X^{\prime}=90^{\circ}$ and $X^{\prime} A=X^{\prime} C$, so we get $\angle X^{\prime} A M=90^{\circ}+\angle B A C / 2=180^{\circ}-\angle X^{\prime} A C=180^{\circ}-\angle X^{\prime} C A=\angle X^{\prime} C L$. Thus the triangles $X^{\prime} A M$ and $X^{\prime} C L$ are congruent, and therefore + +$$ +\angle M X^{\prime} L=\angle A X^{\prime} C+\left(\angle C X^{\prime} L-\angle A X^{\prime} M\right)=\angle A X^{\prime} C=180^{\circ}-2 \angle X^{\prime} A C=\angle B A C=\angle M A L . +$$ + +This means that $X^{\prime}$ lies on $\gamma$. + +Thus we have $\angle T X N=\angle T X X^{\prime}=\angle T A X^{\prime}=90^{\circ}$, so $T X \| A C$. Then $\angle X T A=\angle T A C=$ $\angle T A M$, so the cyclic quadrilateral MATX is an isosceles trapezoid. Similarly, $N A T Y$ is an isosceles trapezoid, so again the lines $M N$ and $X Y$ are the reflections of each other about the perpendicular bisector of $A T$. Thus $K$ belongs to this perpendicular bisector. + +![](https://cdn.mathpix.com/cropped/2024_04_17_e0914cb4f8055c731538g-41.jpg?height=797&width=1001&top_left_y=214&top_left_x=562) + +Comment. There are several different ways of showing that the points $X$ and $M$ are symmetrical with respect to $\ell$. For instance, one can show that the quadrilaterals $A M O N$ and $T X O Y$ are congruent. We chose Solution 1 as a simple way of doing it. On the other hand, Solution 2 shows some other interesting properties of the configuration. + +Let us define $Y^{\prime}$, analogously to $X^{\prime}$, as the common point of $M Y$ and the external bisector of $\angle B A C$. One may easily see that in general the lines $M N$ and $X^{\prime} Y^{\prime}$ (which is the external bisector of $\angle B A C$ ) do not intersect on the perpendicular bisector of $A T$. Thus, any solution should involve some argument using the choice of the intersection points $X$ and $Y$. + +G3. In a triangle $A B C$, let $D$ and $E$ be the feet of the angle bisectors of angles $A$ and $B$, respectively. A rhombus is inscribed into the quadrilateral $A E D B$ (all vertices of the rhombus lie on different sides of $A E D B$ ). Let $\varphi$ be the non-obtuse angle of the rhombus. Prove that $\varphi \leqslant \max \{\angle B A C, \angle A B C\}$. + +(Serbia) + +Solution 1. Let $K, L, M$, and $N$ be the vertices of the rhombus lying on the sides $A E, E D, D B$, and $B A$, respectively. Denote by $d(X, Y Z)$ the distance from a point $X$ to a line $Y Z$. Since $D$ and $E$ are the feet of the bisectors, we have $d(D, A B)=d(D, A C), d(E, A B)=d(E, B C)$, and $d(D, B C)=d(E, A C)=0$, which implies + +$$ +d(D, A C)+d(D, B C)=d(D, A B) \quad \text { and } \quad d(E, A C)+d(E, B C)=d(E, A B) +$$ + +Since $L$ lies on the segment $D E$ and the relation $d(X, A C)+d(X, B C)=d(X, A B)$ is linear in $X$ inside the triangle, these two relations imply + +$$ +d(L, A C)+d(L, B C)=d(L, A B) . +$$ + +Denote the angles as in the figure below, and denote $a=K L$. Then we have $d(L, A C)=a \sin \mu$ and $d(L, B C)=a \sin \nu$. Since $K L M N$ is a parallelogram lying on one side of $A B$, we get + +$$ +d(L, A B)=d(L, A B)+d(N, A B)=d(K, A B)+d(M, A B)=a(\sin \delta+\sin \varepsilon) +$$ + +Thus the condition (1) reads + +$$ +\sin \mu+\sin \nu=\sin \delta+\sin \varepsilon +$$ + +![](https://cdn.mathpix.com/cropped/2024_04_17_e0914cb4f8055c731538g-42.jpg?height=593&width=653&top_left_y=1384&top_left_x=736) + +If one of the angles $\alpha$ and $\beta$ is non-acute, then the desired inequality is trivial. So we assume that $\alpha, \beta<\pi / 2$. It suffices to show then that $\psi=\angle N K L \leqslant \max \{\alpha, \beta\}$. + +Assume, to the contrary, that $\psi>\max \{\alpha, \beta\}$. Since $\mu+\psi=\angle C K N=\alpha+\delta$, by our assumption we obtain $\mu=(\alpha-\psi)+\delta<\delta$. Similarly, $\nu<\varepsilon$. Next, since $K N \| M L$, we have $\beta=\delta+\nu$, so $\delta<\beta<\pi / 2$. Similarly, $\varepsilon<\pi / 2$. Finally, by $\mu<\delta<\pi / 2$ and $\nu<\varepsilon<\pi / 2$, we obtain + +$$ +\sin \mu<\sin \delta \quad \text { and } \quad \sin \nu<\sin \varepsilon +$$ + +This contradicts (2). + +Comment. One can see that the equality is achieved if $\alpha=\beta$ for every rhombus inscribed into the quadrilateral $A E D B$. + +G4. Let $A B C$ be a triangle with $\angle B>\angle C$. Let $P$ and $Q$ be two different points on line $A C$ such that $\angle P B A=\angle Q B A=\angle A C B$ and $A$ is located between $P$ and $C$. Suppose that there exists an interior point $D$ of segment $B Q$ for which $P D=P B$. Let the ray $A D$ intersect the circle $A B C$ at $R \neq A$. Prove that $Q B=Q R$. + +(Georgia) + +Solution 1. Denote by $\omega$ the circumcircle of the triangle $A B C$, and let $\angle A C B=\gamma$. Note that the condition $\gamma<\angle C B A$ implies $\gamma<90^{\circ}$. Since $\angle P B A=\gamma$, the line $P B$ is tangent to $\omega$, so $P A \cdot P C=P B^{2}=P D^{2}$. By $\frac{P A}{P D}=\frac{P D}{P C}$ the triangles $P A D$ and $P D C$ are similar, and $\angle A D P=\angle D C P$. + +Next, since $\angle A B Q=\angle A C B$, the triangles $A B C$ and $A Q B$ are also similar. Then $\angle A Q B=$ $\angle A B C=\angle A R C$, which means that the points $D, R, C$, and $Q$ are concyclic. Therefore $\angle D R Q=$ $\angle D C Q=\angle A D P$. + +![](https://cdn.mathpix.com/cropped/2024_04_17_e0914cb4f8055c731538g-43.jpg?height=594&width=959&top_left_y=923&top_left_x=583) + +Figure 1 + +Now from $\angle A R B=\angle A C B=\gamma$ and $\angle P D B=\angle P B D=2 \gamma$ we get + +$$ +\angle Q B R=\angle A D B-\angle A R B=\angle A D P+\angle P D B-\angle A R B=\angle D R Q+\gamma=\angle Q R B +$$ + +so the triangle $Q R B$ is isosceles, which yields $Q B=Q R$. + +Solution 2. Again, denote by $\omega$ the circumcircle of the triangle $A B C$. Denote $\angle A C B=\gamma$. Since $\angle P B A=\gamma$, the line $P B$ is tangent to $\omega$. + +Let $E$ be the second intersection point of $B Q$ with $\omega$. If $V^{\prime}$ is any point on the ray $C E$ beyond $E$, then $\angle B E V^{\prime}=180^{\circ}-\angle B E C=180^{\circ}-\angle B A C=\angle P A B$; together with $\angle A B Q=$ $\angle P B A$ this shows firstly, that the rays $B A$ and $C E$ intersect at some point $V$, and secondly that the triangle $V E B$ is similar to the triangle $P A B$. Thus we have $\angle B V E=\angle B P A$. Next, $\angle A E V=\angle B E V-\gamma=\angle P A B-\angle A B Q=\angle A Q B$; so the triangles $P B Q$ and $V A E$ are also similar. + +Let $P H$ be an altitude in the isosceles triangle $P B D$; then $B H=H D$. Let $G$ be the intersection point of $P H$ and $A B$. By the symmetry with respect to $P H$, we have $\angle B D G=\angle D B G=\gamma=$ $\angle B E A$; thus $D G \| A E$ and hence $\frac{B G}{G A}=\frac{B D}{D E}$. Thus the points $G$ and $D$ correspond to each other in the similar triangles $P A B$ and $V E B$, so $\angle D V B=\angle G P B=90^{\circ}-\angle P B Q=90^{\circ}-\angle V A E$. Thus $V D \perp A E$. + +Let $T$ be the common point of $V D$ and $A E$, and let $D S$ be an altitude in the triangle $B D R$. The points $S$ and $T$ are the feet of corresponding altitudes in the similar triangles $A D E$ and $B D R$, so $\frac{B S}{S R}=\frac{A T}{T E}$. On the other hand, the points $T$ and $H$ are feet of corresponding altitudes in the similar triangles $V A E$ and $P B Q$, so $\frac{A T}{T E}=\frac{B H}{H Q}$. Thus $\frac{B S}{S R}=\frac{A T}{T E}=\frac{B H}{H Q}$, and the triangles $B H S$ and $B Q R$ are similar. + +Finally, $S H$ is a median in the right-angled triangle $S B D$; so $B H=H S$, and hence $B Q=Q R$. + +![](https://cdn.mathpix.com/cropped/2024_04_17_e0914cb4f8055c731538g-44.jpg?height=848&width=875&top_left_y=584&top_left_x=625) + +Figure 2 + +Solution 3. Denote by $\omega$ and $O$ the circumcircle of the triangle $A B C$ and its center, respectively. From the condition $\angle P B A=\angle B C A$ we know that $B P$ is tangent to $\omega$. + +Let $E$ be the second point of intersection of $\omega$ and $B D$. Due to the isosceles triangle $B D P$, the tangent of $\omega$ at $E$ is parallel to $D P$ and consequently it intersects $B P$ at some point $L$. Of course, $P D \| L E$. Let $M$ be the midpoint of $B E$, and let $H$ be the midpoint of $B R$. Notice that $\angle A E B=\angle A C B=\angle A B Q=\angle A B E$, so $A$ lies on the perpendicular bisector of $B E$; thus the points $L, A, M$, and $O$ are collinear. Let $\omega_{1}$ be the circle with diameter $B O$. Let $Q^{\prime}=H O \cap B E$; since $H O$ is the perpendicular bisector of $B R$, the statement of the problem is equivalent to $Q^{\prime}=Q$. + +Consider the following sequence of projections (see Fig. 3). + +1. Project the line $B E$ to the line $L B$ through the center $A$. (This maps $Q$ to $P$.) +2. Project the line $L B$ to $B E$ in parallel direction with $L E$. $(P \mapsto D$.) +3. Project the line $B E$ to the circle $\omega$ through its point $A .(D \mapsto R$.) +4. Scale $\omega$ by the ratio $\frac{1}{2}$ from the point $B$ to the circle $\omega_{1} .(R \mapsto H$. +5. Project $\omega_{1}$ to the line $B E$ through its point $O$. $\left(H \mapsto Q^{\prime}\right.$.) + +We prove that the composition of these transforms, which maps the line $B E$ to itself, is the identity. To achieve this, it suffices to show three fixed points. An obvious fixed point is $B$ which is fixed by all the transformations above. Another fixed point is $M$, its path being $M \mapsto L \mapsto$ $E \mapsto E \mapsto M \mapsto M$. + +![](https://cdn.mathpix.com/cropped/2024_04_17_e0914cb4f8055c731538g-45.jpg?height=800&width=786&top_left_y=218&top_left_x=279) + +Figure 3 + +![](https://cdn.mathpix.com/cropped/2024_04_17_e0914cb4f8055c731538g-45.jpg?height=786&width=759&top_left_y=233&top_left_x=1125) + +Figure 4 + +In order to show a third fixed point, draw a line parallel with $L E$ through $A$; let that line intersect $B E, L B$ and $\omega$ at $X, Y$ and $Z \neq A$, respectively (see Fig. 4). We show that $X$ is a fixed point. The images of $X$ at the first three transformations are $X \mapsto Y \mapsto X \mapsto Z$. From $\angle X B Z=\angle E A Z=\angle A E L=\angle L B A=\angle B Z X$ we can see that the triangle $X B Z$ is isosceles. Let $U$ be the midpoint of $B Z$; then the last two transformations do $Z \mapsto U \mapsto X$, and the point $X$ is fixed. + +Comment. Verifying that the point $E$ is fixed seems more natural at first, but it appears to be less straightforward. Here we outline a possible proof. + +Let the images of $E$ at the first three transforms above be $F, G$ and $I$. After comparing the angles depicted in Fig. 5 (noticing that the quadrilateral $A F B G$ is cyclic) we can observe that the tangent $L E$ of $\omega$ is parallel to $B I$. Then, similarly to the above reasons, the point $E$ is also fixed. + +![](https://cdn.mathpix.com/cropped/2024_04_17_e0914cb4f8055c731538g-45.jpg?height=789&width=751&top_left_y=1687&top_left_x=687) + +Figure 5 + +G5. Let $A B C D E F$ be a convex hexagon with $A B=D E, B C=E F, C D=F A$, and $\angle A-\angle D=\angle C-\angle F=\angle E-\angle B$. Prove that the diagonals $A D, B E$, and $C F$ are concurrent. + +(Ukraine) + +In all three solutions, we denote $\theta=\angle A-\angle D=\angle C-\angle F=\angle E-\angle B$ and assume without loss of generality that $\theta \geqslant 0$. + +Solution 1. Let $x=A B=D E, y=C D=F A, z=E F=B C$. Consider the points $P, Q$, and $R$ such that the quadrilaterals $C D E P, E F A Q$, and $A B C R$ are parallelograms. We compute + +$$ +\begin{aligned} +\angle P E Q & =\angle F E Q+\angle D E P-\angle E=\left(180^{\circ}-\angle F\right)+\left(180^{\circ}-\angle D\right)-\angle E \\ +& =360^{\circ}-\angle D-\angle E-\angle F=\frac{1}{2}(\angle A+\angle B+\angle C-\angle D-\angle E-\angle F)=\theta / 2 +\end{aligned} +$$ + +Similarly, $\angle Q A R=\angle R C P=\theta / 2$. + +![](https://cdn.mathpix.com/cropped/2024_04_17_e0914cb4f8055c731538g-46.jpg?height=491&width=827&top_left_y=947&top_left_x=649) + +If $\theta=0$, since $\triangle R C P$ is isosceles, $R=P$. Therefore $A B\|R C=P C\| E D$, so $A B D E$ is a parallelogram. Similarly, $B C E F$ and $C D F A$ are parallelograms. It follows that $A D, B E$ and $C F$ meet at their common midpoint. + +Now assume $\theta>0$. Since $\triangle P E Q, \triangle Q A R$, and $\triangle R C P$ are isosceles and have the same angle at the apex, we have $\triangle P E Q \sim \triangle Q A R \sim \triangle R C P$ with ratios of similarity $y: z: x$. Thus + +$\triangle P Q R$ is similar to the triangle with sidelengths $y, z$, and $x$. + +Next, notice that + +$$ +\frac{R Q}{Q P}=\frac{z}{y}=\frac{R A}{A F} +$$ + +and, using directed angles between rays, + +$$ +\begin{aligned} +\not(R Q, Q P) & =\Varangle(R Q, Q E)+\Varangle(Q E, Q P) \\ +& =\Varangle(R Q, Q E)+\Varangle(R A, R Q)=\Varangle(R A, Q E)=\Varangle(R A, A F) . +\end{aligned} +$$ + +Thus $\triangle P Q R \sim \triangle F A R$. Since $F A=y$ and $A R=z$, (1) then implies that $F R=x$. Similarly $F P=x$. Therefore $C R F P$ is a rhombus. + +We conclude that $C F$ is the perpendicular bisector of $P R$. Similarly, $B E$ is the perpendicular bisector of $P Q$ and $A D$ is the perpendicular bisector of $Q R$. It follows that $A D, B E$, and $C F$ are concurrent at the circumcenter of $P Q R$. + +Solution 2. Let $X=C D \cap E F, Y=E F \cap A B, Z=A B \cap C D, X^{\prime}=F A \cap B C, Y^{\prime}=$ $B C \cap D E$, and $Z^{\prime}=D E \cap F A$. From $\angle A+\angle B+\angle C=360^{\circ}+\theta / 2$ we get $\angle A+\angle B>180^{\circ}$ and $\angle B+\angle C>180^{\circ}$, so $Z$ and $X^{\prime}$ are respectively on the opposite sides of $B C$ and $A B$ from the hexagon. Similar conclusions hold for $X, Y, Y^{\prime}$, and $Z^{\prime}$. Then + +$$ +\angle Y Z X=\angle B+\angle C-180^{\circ}=\angle E+\angle F-180^{\circ}=\angle Y^{\prime} Z^{\prime} X^{\prime}, +$$ + +and similarly $\angle Z X Y=\angle Z^{\prime} X^{\prime} Y^{\prime}$ and $\angle X Y Z=\angle X^{\prime} Y^{\prime} Z^{\prime}$, so $\triangle X Y Z \sim \triangle X^{\prime} Y^{\prime} Z^{\prime}$. Thus there is a rotation $R$ which sends $\triangle X Y Z$ to a triangle with sides parallel to $\triangle X^{\prime} Y^{\prime} Z^{\prime}$. Since $A B=D E$ we have $R(\overrightarrow{A B})=\overrightarrow{D E}$. Similarly, $R(\overrightarrow{C D})=\overrightarrow{F A}$ and $R(\overrightarrow{E F})=\overrightarrow{B C}$. Therefore + +$$ +\overrightarrow{0}=\overrightarrow{A B}+\overrightarrow{B C}+\overrightarrow{C D}+\overrightarrow{D E}+\overrightarrow{E F}+\overrightarrow{F A}=(\overrightarrow{A B}+\overrightarrow{C D}+\overrightarrow{E F})+R(\overrightarrow{A B}+\overrightarrow{C D}+\overrightarrow{E F}) +$$ + +If $R$ is a rotation by $180^{\circ}$, then any two opposite sides of our hexagon are equal and parallel, so the three diagonals meet at their common midpoint. Otherwise, we must have + +$$ +\overrightarrow{A B}+\overrightarrow{C D}+\overrightarrow{E F}=\overrightarrow{0} +$$ + +or else we would have two vectors with different directions whose sum is $\overrightarrow{0}$. + +![](https://cdn.mathpix.com/cropped/2024_04_17_e0914cb4f8055c731538g-47.jpg?height=689&width=1333&top_left_y=1057&top_left_x=388) + +This allows us to consider a triangle $L M N$ with $\overrightarrow{L M}=\overrightarrow{E F}, \overrightarrow{M N}=\overrightarrow{A B}$, and $\overrightarrow{N L}=\overrightarrow{C D}$. Let $O$ be the circumcenter of $\triangle L M N$ and consider the points $O_{1}, O_{2}, O_{3}$ such that $\triangle A O_{1} B, \triangle C O_{2} D$, and $\triangle E O_{3} F$ are translations of $\triangle M O N, \triangle N O L$, and $\triangle L O M$, respectively. Since $F O_{3}$ and $A O_{1}$ are translations of $M O$, quadrilateral $A F O_{3} O_{1}$ is a parallelogram and $O_{3} O_{1}=F A=C D=N L$. Similarly, $O_{1} O_{2}=L M$ and $O_{2} O_{3}=M N$. Therefore $\triangle O_{1} O_{2} O_{3} \cong \triangle L M N$. Moreover, by means of the rotation $R$ one may check that these triangles have the same orientation. + +Let $T$ be the circumcenter of $\triangle O_{1} O_{2} O_{3}$. We claim that $A D, B E$, and $C F$ meet at $T$. Let us show that $C, T$, and $F$ are collinear. Notice that $C O_{2}=O_{2} T=T O_{3}=O_{3} F$ since they are all equal to the circumradius of $\triangle L M N$. Therefore $\triangle T O_{3} F$ and $\triangle C O_{2} T$ are isosceles. Using directed angles between rays again, we get + +$$ +\Varangle\left(T F, T O_{3}\right)=\Varangle\left(F O_{3}, F T\right) \quad \text { and } \quad \Varangle\left(T O_{2}, T C\right)=\Varangle\left(C T, C O_{2}\right) \text {. } +$$ + +Also, $T$ and $O$ are the circumcenters of the congruent triangles $\triangle O_{1} O_{2} O_{3}$ and $\triangle L M N$ so we have $\Varangle\left(T O_{3}, T O_{2}\right)=\Varangle(O N, O M)$. Since $C_{2}$ and $F O_{3}$ are translations of $N O$ and $M O$ respectively, this implies + +$$ +\Varangle\left(T O_{3}, T O_{2}\right)=\Varangle\left(C O_{2}, F O_{3}\right) . +$$ + +Adding the three equations in (2) and (3) gives + +$$ +\Varangle(T F, T C)=\Varangle(C T, F T)=-\not(T F, T C) +$$ + +which implies that $T$ is on $C F$. Analogous arguments show that it is on $A D$ and $B E$ also. The desired result follows. + +Solution 3. Place the hexagon on the complex plane, with $A$ at the origin and vertices labelled clockwise. Now $A, B, C, D, E, F$ represent the corresponding complex numbers. Also consider the complex numbers $a, b, c, a^{\prime}, b^{\prime}, c^{\prime}$ given by $B-A=a, D-C=b, F-E=c, E-D=a^{\prime}$, $A-F=b^{\prime}$, and $C-B=c^{\prime}$. Let $k=|a| /|b|$. From $a / b^{\prime}=-k e^{i \angle A}$ and $a^{\prime} / b=-k e^{i \angle D}$ we get that $\left(a^{\prime} / a\right)\left(b^{\prime} / b\right)=e^{-i \theta}$ and similarly $\left(b^{\prime} / b\right)\left(c^{\prime} / c\right)=e^{-i \theta}$ and $\left(c^{\prime} / c\right)\left(a^{\prime} / a\right)=e^{-i \theta}$. It follows that $a^{\prime}=a r$, $b^{\prime}=b r$, and $c^{\prime}=c r$ for a complex number $r$ with $|r|=1$, as shown below. + +![](https://cdn.mathpix.com/cropped/2024_04_17_e0914cb4f8055c731538g-48.jpg?height=512&width=1052&top_left_y=823&top_left_x=534) + +We have + +$$ +0=a+c r+b+a r+c+b r=(a+b+c)(1+r) . +$$ + +If $r=-1$, then the hexagon is centrally symmetric and its diagonals intersect at its center of symmetry. Otherwise + +$$ +a+b+c=0 \text {. } +$$ + +Therefore + +$$ +A=0, \quad B=a, \quad C=a+c r, \quad D=c(r-1), \quad E=-b r-c, \quad F=-b r . +$$ + +Now consider a point $W$ on $A D$ given by the complex number $c(r-1) \lambda$, where $\lambda$ is a real number with $0<\lambda<1$. Since $D \neq A$, we have $r \neq 1$, so we can define $s=1 /(r-1)$. From $r \bar{r}=|r|^{2}=1$ we get + +$$ +1+s=\frac{r}{r-1}=\frac{r}{r-r \bar{r}}=\frac{1}{1-\bar{r}}=-\bar{s} . +$$ + +Now, + +$$ +\begin{aligned} +W \text { is on } B E & \Longleftrightarrow c(r-1) \lambda-a\|a-(-b r-c)=b(r-1) \Longleftrightarrow c \lambda-a s\| b \\ +& \Longleftrightarrow-a \lambda-b \lambda-a s\|b \Longleftrightarrow a(\lambda+s)\| b . +\end{aligned} +$$ + +One easily checks that $r \neq \pm 1$ implies that $\lambda+s \neq 0$ since $s$ is not real. On the other hand, + +$$ +\begin{aligned} +W \text { on } C F & \Longleftrightarrow c(r-1) \lambda+b r\|-b r-(a+c r)=a(r-1) \Longleftrightarrow c \lambda+b(1+s)\| a \\ +& \Longleftrightarrow-a \lambda-b \lambda-b \bar{s}\|a \Longleftrightarrow b(\lambda+\bar{s})\| a \Longleftrightarrow b \| a(\lambda+s), +\end{aligned} +$$ + +where in the last step we use that $(\lambda+s)(\lambda+\bar{s})=|\lambda+s|^{2} \in \mathbb{R}_{>0}$. We conclude that $A D \cap B E=$ $C F \cap B E$, and the desired result follows. + +G6. Let the excircle of the triangle $A B C$ lying opposite to $A$ touch its side $B C$ at the point $A_{1}$. Define the points $B_{1}$ and $C_{1}$ analogously. Suppose that the circumcentre of the triangle $A_{1} B_{1} C_{1}$ lies on the circumcircle of the triangle $A B C$. Prove that the triangle $A B C$ is right-angled. + +(Russia) + +Solution 1. Denote the circumcircles of the triangles $A B C$ and $A_{1} B_{1} C_{1}$ by $\Omega$ and $\Gamma$, respectively. Denote the midpoint of the arc $C B$ of $\Omega$ containing $A$ by $A_{0}$, and define $B_{0}$ as well as $C_{0}$ analogously. By our hypothesis the centre $Q$ of $\Gamma$ lies on $\Omega$. + +Lemma. One has $A_{0} B_{1}=A_{0} C_{1}$. Moreover, the points $A, A_{0}, B_{1}$, and $C_{1}$ are concyclic. Finally, the points $A$ and $A_{0}$ lie on the same side of $B_{1} C_{1}$. Similar statements hold for $B$ and $C$. + +Proof. Let us consider the case $A=A_{0}$ first. Then the triangle $A B C$ is isosceles at $A$, which implies $A B_{1}=A C_{1}$ while the remaining assertions of the Lemma are obvious. So let us suppose $A \neq A_{0}$ from now on. + +By the definition of $A_{0}$, we have $A_{0} B=A_{0} C$. It is also well known and easy to show that $B C_{1}=$ $C B_{1}$. Next, we have $\angle C_{1} B A_{0}=\angle A B A_{0}=\angle A C A_{0}=\angle B_{1} C A_{0}$. Hence the triangles $A_{0} B C_{1}$ and $A_{0} C B_{1}$ are congruent. This implies $A_{0} C_{1}=A_{0} B_{1}$, establishing the first part of the Lemma. It also follows that $\angle A_{0} C_{1} A=\angle A_{0} B_{1} A$, as these are exterior angles at the corresponding vertices $C_{1}$ and $B_{1}$ of the congruent triangles $A_{0} B C_{1}$ and $A_{0} C B_{1}$. For that reason the points $A, A_{0}, B_{1}$, and $C_{1}$ are indeed the vertices of some cyclic quadrilateral two opposite sides of which are $A A_{0}$ and $B_{1} C_{1}$. + +Now we turn to the solution. Evidently the points $A_{1}, B_{1}$, and $C_{1}$ lie interior to some semicircle arc of $\Gamma$, so the triangle $A_{1} B_{1} C_{1}$ is obtuse-angled. Without loss of generality, we will assume that its angle at $B_{1}$ is obtuse. Thus $Q$ and $B_{1}$ lie on different sides of $A_{1} C_{1}$; obviously, the same holds for the points $B$ and $B_{1}$. So, the points $Q$ and $B$ are on the same side of $A_{1} C_{1}$. + +Notice that the perpendicular bisector of $A_{1} C_{1}$ intersects $\Omega$ at two points lying on different sides of $A_{1} C_{1}$. By the first statement from the Lemma, both points $B_{0}$ and $Q$ are among these points of intersection; since they share the same side of $A_{1} C_{1}$, they coincide (see Figure 1). + +![](https://cdn.mathpix.com/cropped/2024_04_17_e0914cb4f8055c731538g-49.jpg?height=781&width=827&top_left_y=1667&top_left_x=649) + +Figure 1 + +Now, by the first part of the Lemma again, the lines $Q A_{0}$ and $Q C_{0}$ are the perpendicular bisectors of $B_{1} C_{1}$ and $A_{1} B_{1}$, respectively. Thus + +$$ +\angle C_{1} B_{0} A_{1}=\angle C_{1} B_{0} B_{1}+\angle B_{1} B_{0} A_{1}=2 \angle A_{0} B_{0} B_{1}+2 \angle B_{1} B_{0} C_{0}=2 \angle A_{0} B_{0} C_{0}=180^{\circ}-\angle A B C +$$ + +recalling that $A_{0}$ and $C_{0}$ are the midpoints of the arcs $C B$ and $B A$, respectively. + +On the other hand, by the second part of the Lemma we have + +$$ +\angle C_{1} B_{0} A_{1}=\angle C_{1} B A_{1}=\angle A B C . +$$ + +From the last two equalities, we get $\angle A B C=90^{\circ}$, whereby the problem is solved. + +Solution 2. Let $Q$ again denote the centre of the circumcircle of the triangle $A_{1} B_{1} C_{1}$, that lies on the circumcircle $\Omega$ of the triangle $A B C$. We first consider the case where $Q$ coincides with one of the vertices of $A B C$, say $Q=B$. Then $B C_{1}=B A_{1}$ and consequently the triangle $A B C$ is isosceles at $B$. Moreover we have $B C_{1}=B_{1} C$ in any triangle, and hence $B B_{1}=B C_{1}=B_{1} C$; similarly, $B B_{1}=B_{1} A$. It follows that $B_{1}$ is the centre of $\Omega$ and that the triangle $A B C$ has a right angle at $B$. + +So from now on we may suppose $Q \notin\{A, B, C\}$. We start with the following well known fact. Lemma. Let $X Y Z$ and $X^{\prime} Y^{\prime} Z^{\prime}$ be two triangles with $X Y=X^{\prime} Y^{\prime}$ and $Y Z=Y^{\prime} Z^{\prime}$. + +(i) If $X Z \neq X^{\prime} Z^{\prime}$ and $\angle Y Z X=\angle Y^{\prime} Z^{\prime} X^{\prime}$, then $\angle Z X Y+\angle Z^{\prime} X^{\prime} Y^{\prime}=180^{\circ}$. + +(ii) If $\angle Y Z X+\angle X^{\prime} Z^{\prime} Y^{\prime}=180^{\circ}$, then $\angle Z X Y=\angle Y^{\prime} X^{\prime} Z^{\prime}$. + +Proof. For both parts, we may move the triangle $X Y Z$ through the plane until $Y=Y^{\prime}$ and $Z=Z^{\prime}$. Possibly after reflecting one of the two triangles about $Y Z$, we may also suppose that $X$ and $X^{\prime}$ lie on the same side of $Y Z$ if we are in case (i) and on different sides if we are in case (ii). In both cases, the points $X, Z$, and $X^{\prime}$ are collinear due to the angle condition (see Fig. 2). Moreover we have $X \neq X^{\prime}$, because in case (i) we assumed $X Z \neq X^{\prime} Z^{\prime}$ and in case (ii) these points even lie on different sides of $Y Z$. Thus the triangle $X X^{\prime} Y$ is isosceles at $Y$. The claim now follows by considering the equal angles at its base. + +![](https://cdn.mathpix.com/cropped/2024_04_17_e0914cb4f8055c731538g-50.jpg?height=304&width=460&top_left_y=1732&top_left_x=518) + +Figure 2(i) + +![](https://cdn.mathpix.com/cropped/2024_04_17_e0914cb4f8055c731538g-50.jpg?height=304&width=464&top_left_y=1732&top_left_x=1123) + +Figure 2(ii) + +Relabeling the vertices of the triangle $A B C$ if necessary we may suppose that $Q$ lies in the interior of the arc $A B$ of $\Omega$ not containing $C$. We will sometimes use tacitly that the six triangles $Q B A_{1}, Q A_{1} C, Q C B_{1}, Q B_{1} A, Q C_{1} A$, and $Q B C_{1}$ have the same orientation. + +As $Q$ cannot be the circumcentre of the triangle $A B C$, it is impossible that $Q A=Q B=Q C$ and thus we may also suppose that $Q C \neq Q B$. Now the above Lemma $(i)$ is applicable to the triangles $Q B_{1} C$ and $Q C_{1} B$, since $Q B_{1}=Q C_{1}$ and $B_{1} C=C_{1} B$, while $\angle B_{1} C Q=\angle C_{1} B Q$ holds as both angles appear over the same side of the chord $Q A$ in $\Omega$ (see Fig. 3). So we get + +$$ +\angle C Q B_{1}+\angle B Q C_{1}=180^{\circ} . +$$ + +We claim that $Q C=Q A$. To see this, let us assume for the sake of a contradiction that $Q C \neq Q A$. Then arguing similarly as before but now with the triangles $Q A_{1} C$ and $Q C_{1} A$ we get + +$$ +\angle A_{1} Q C+\angle C_{1} Q A=180^{\circ} \text {. } +$$ + +Adding this equation to (1), we get $\angle A_{1} Q B_{1}+\angle B Q A=360^{\circ}$, which is absurd as both summands lie in the interval $\left(0^{\circ}, 180^{\circ}\right)$. + +This proves $Q C=Q A$; so the triangles $Q A_{1} C$ and $Q C_{1} A$ are congruent their sides being equal, which in turn yields + +$$ +\angle A_{1} Q C=\angle C_{1} Q A \text {. } +$$ + +Finally our Lemma (ii) is applicable to the triangles $Q A_{1} B$ and $Q B_{1} A$. Indeed we have $Q A_{1}=Q B_{1}$ and $A_{1} B=B_{1} A$ as usual, and the angle condition $\angle A_{1} B Q+\angle Q A B_{1}=180^{\circ}$ holds as $A$ and $B$ lie on different sides of the chord $Q C$ in $\Omega$. Consequently we have + +$$ +\angle B Q A_{1}=\angle B_{1} Q A \text {. } +$$ + +From (1) and (3) we get + +$$ +\left(\angle B_{1} Q C+\angle B_{1} Q A\right)+\left(\angle C_{1} Q B-\angle B Q A_{1}\right)=180^{\circ} \text {, } +$$ + +i.e. $\angle C Q A+\angle A_{1} Q C_{1}=180^{\circ}$. In light of (2) this may be rewritten as $2 \angle C Q A=180^{\circ}$ and as $Q$ lies on $\Omega$ this implies that the triangle $A B C$ has a right angle at $B$. + +![](https://cdn.mathpix.com/cropped/2024_04_17_e0914cb4f8055c731538g-51.jpg?height=569&width=908&top_left_y=1404&top_left_x=606) + +Figure 3 + +Comment 1. One may also check that $Q$ is in the interior of $\Omega$ if and only if the triangle $A B C$ is acute-angled. + +Comment 2. The original proposal asked to prove the converse statement as well: if the triangle $A B C$ is right-angled, then the point $Q$ lies on its circumcircle. The Problem Selection Committee thinks that the above simplified version is more suitable for the competition. + +## Number Theory + +N1. Let $\mathbb{Z}_{>0}$ be the set of positive integers. Find all functions $f: \mathbb{Z}_{>0} \rightarrow \mathbb{Z}_{>0}$ such that + +$$ +m^{2}+f(n) \mid m f(m)+n +$$ + +for all positive integers $m$ and $n$. + +(Malaysia) + +Answer. $f(n)=n$. + +Solution 1. Setting $m=n=2$ tells us that $4+f(2) \mid 2 f(2)+2$. Since $2 f(2)+2<2(4+f(2))$, we must have $2 f(2)+2=4+f(2)$, so $f(2)=2$. Plugging in $m=2$ then tells us that $4+f(n) \mid 4+n$, which implies that $f(n) \leqslant n$ for all $n$. + +Setting $m=n$ gives $n^{2}+f(n) \mid n f(n)+n$, so $n f(n)+n \geqslant n^{2}+f(n)$ which we rewrite as $(n-1)(f(n)-n) \geqslant 0$. Therefore $f(n) \geqslant n$ for all $n \geqslant 2$. This is trivially true for $n=1$ also. + +It follows that $f(n)=n$ for all $n$. This function obviously satisfies the desired property. + +Solution 2. Setting $m=f(n)$ we get $f(n)(f(n)+1) \mid f(n) f(f(n))+n$. This implies that $f(n) \mid n$ for all $n$. + +Now let $m$ be any positive integer, and let $p>2 m^{2}$ be a prime number. Note that $p>m f(m)$ also. Plugging in $n=p-m f(m)$ we learn that $m^{2}+f(n)$ divides $p$. Since $m^{2}+f(n)$ cannot equal 1, it must equal $p$. Therefore $p-m^{2}=f(n) \mid n=p-m f(m)$. But $p-m f(m)|f(n)-n|$. It follows that $f$ is the identity function. + +N2. Prove that for any pair of positive integers $k$ and $n$ there exist $k$ positive integers $m_{1}, m_{2}, \ldots, m_{k}$ such that + +$$ +1+\frac{2^{k}-1}{n}=\left(1+\frac{1}{m_{1}}\right)\left(1+\frac{1}{m_{2}}\right) \cdots\left(1+\frac{1}{m_{k}}\right) . +$$ + +(Japan) + +Solution 1. We proceed by induction on $k$. For $k=1$ the statement is trivial. Assuming we have proved it for $k=j-1$, we now prove it for $k=j$. + +Case 1. $n=2 t-1$ for some positive integer $t$. + +Observe that + +$$ +1+\frac{2^{j}-1}{2 t-1}=\frac{2\left(t+2^{j-1}-1\right)}{2 t} \cdot \frac{2 t}{2 t-1}=\left(1+\frac{2^{j-1}-1}{t}\right)\left(1+\frac{1}{2 t-1}\right) . +$$ + +By the induction hypothesis we can find $m_{1}, \ldots, m_{j-1}$ such that + +$$ +1+\frac{2^{j-1}-1}{t}=\left(1+\frac{1}{m_{1}}\right)\left(1+\frac{1}{m_{2}}\right) \cdots\left(1+\frac{1}{m_{j-1}}\right) +$$ + +so setting $m_{j}=2 t-1$ gives the desired expression. + +Case 2. $n=2 t$ for some positive integer $t$. + +Now we have + +$$ +1+\frac{2^{j}-1}{2 t}=\frac{2 t+2^{j}-1}{2 t+2^{j}-2} \cdot \frac{2 t+2^{j}-2}{2 t}=\left(1+\frac{1}{2 t+2^{j}-2}\right)\left(1+\frac{2^{j-1}-1}{t}\right) +$$ + +noting that $2 t+2^{j}-2>0$. Again, we use that + +$$ +1+\frac{2^{j-1}-1}{t}=\left(1+\frac{1}{m_{1}}\right)\left(1+\frac{1}{m_{2}}\right) \cdots\left(1+\frac{1}{m_{j-1}}\right) . +$$ + +Setting $m_{j}=2 t+2^{j}-2$ then gives the desired expression. + +Solution 2. Consider the base 2 expansions of the residues of $n-1$ and $-n$ modulo $2^{k}$ : + +$$ +\begin{aligned} +n-1 & \equiv 2^{a_{1}}+2^{a_{2}}+\cdots+2^{a_{r}}\left(\bmod 2^{k}\right) & & \text { where } 0 \leqslant a_{1}q_{n-1} \text { and } q_{n}>q_{n+1}\right\} +$$ + +is infinite, since for each $n \in S$ one has + +$$ +p_{n}=\max \left\{q_{n}, q_{n-1}\right\}=q_{n}=\max \left\{q_{n}, q_{n+1}\right\}=p_{n+1} . +$$ + +Suppose on the contrary that $S$ is finite. Since $q_{2}=7<13=q_{3}$ and $q_{3}=13>7=q_{4}$, the set $S$ is non-empty. Since it is finite, we can consider its largest element, say $m$. + +Note that it is impossible that $q_{m}>q_{m+1}>q_{m+2}>\ldots$ because all these numbers are positive integers, so there exists a $k \geqslant m$ such that $q_{k}q_{\ell+1}$. By the minimality of $\ell$ we have $q_{\ell-1}k \geqslant m$, this contradicts the maximality of $m$, and hence $S$ is indeed infinite. + +Comment. Once the factorization of $n^{4}+n^{2}+1$ is found and the set $S$ is introduced, the problem is mainly about ruling out the case that + +$$ +q_{k}0}$. In the above solution, this is done by observing $q_{(k+1)^{2}}=\max \left(q_{k}, q_{k+1}\right)$. Alternatively one may notice that (1) implies that $q_{j+2}-q_{j} \geqslant 6$ for $j \geqslant k+1$, since every prime greater than 3 is congruent to -1 or 1 modulo 6 . Then there is some integer $C \geqslant 0$ such that $q_{n} \geqslant 3 n-C$ for all $n \geqslant k$. + +Now let the integer $t$ be sufficiently large (e.g. $t=\max \{k+1, C+3\}$ ) and set $p=q_{t-1} \geqslant 2 t$. Then $p \mid(t-1)^{2}+(t-1)+1$ implies that $p \mid(p-t)^{2}+(p-t)+1$, so $p$ and $q_{p-t}$ are prime divisors of $(p-t)^{2}+(p-t)+1$. But $p-t>t-1 \geqslant k$, so $q_{p-t}>q_{t-1}=p$ and $p \cdot q_{p-t}>p^{2}>(p-t)^{2}+(p-t)+1$, a contradiction. + +N4. Determine whether there exists an infinite sequence of nonzero digits $a_{1}, a_{2}, a_{3}, \ldots$ and a positive integer $N$ such that for every integer $k>N$, the number $\overline{a_{k} a_{k-1} \ldots a_{1}}$ is a perfect square. + +(Iran) + +Answer. No. + +Solution. Assume that $a_{1}, a_{2}, a_{3}, \ldots$ is such a sequence. For each positive integer $k$, let $y_{k}=$ $\overline{a_{k} a_{k-1} \ldots a_{1}}$. By the assumption, for each $k>N$ there exists a positive integer $x_{k}$ such that $y_{k}=x_{k}^{2}$. + +I. For every $n$, let $5^{\gamma_{n}}$ be the greatest power of 5 dividing $x_{n}$. Let us show first that $2 \gamma_{n} \geqslant n$ for every positive integer $n>N$. + +Assume, to the contrary, that there exists a positive integer $n>N$ such that $2 \gamma_{n}N$. + +II. Consider now any integer $k>\max \{N / 2,2\}$. Since $2 \gamma_{2 k+1} \geqslant 2 k+1$ and $2 \gamma_{2 k+2} \geqslant 2 k+2$, we have $\gamma_{2 k+1} \geqslant k+1$ and $\gamma_{2 k+2} \geqslant k+1$. So, from $y_{2 k+2}=a_{2 k+2} \cdot 10^{2 k+1}+y_{2 k+1}$ we obtain $5^{2 k+2} \mid y_{2 k+2}-y_{2 k+1}=a_{2 k+2} \cdot 10^{2 k+1}$ and thus $5 \mid a_{2 k+2}$, which implies $a_{2 k+2}=5$. Therefore, + +$$ +\left(x_{2 k+2}-x_{2 k+1}\right)\left(x_{2 k+2}+x_{2 k+1}\right)=x_{2 k+2}^{2}-x_{2 k+1}^{2}=y_{2 k+2}-y_{2 k+1}=5 \cdot 10^{2 k+1}=2^{2 k+1} \cdot 5^{2 k+2} . +$$ + +Setting $A_{k}=x_{2 k+2} / 5^{k+1}$ and $B_{k}=x_{2 k+1} / 5^{k+1}$, which are integers, we obtain + +$$ +\left(A_{k}-B_{k}\right)\left(A_{k}+B_{k}\right)=2^{2 k+1} . +$$ + +Both $A_{k}$ and $B_{k}$ are odd, since otherwise $y_{2 k+2}$ or $y_{2 k+1}$ would be a multiple of 10 which is false by $a_{1} \neq 0$; so one of the numbers $A_{k}-B_{k}$ and $A_{k}+B_{k}$ is not divisible by 4 . Therefore (1) yields $A_{k}-B_{k}=2$ and $A_{k}+B_{k}=2^{2 k}$, hence $A_{k}=2^{2 k-1}+1$ and thus + +$$ +x_{2 k+2}=5^{k+1} A_{k}=10^{k+1} \cdot 2^{k-2}+5^{k+1}>10^{k+1}, +$$ + +since $k \geqslant 2$. This implies that $y_{2 k+2}>10^{2 k+2}$ which contradicts the fact that $y_{2 k+2}$ contains $2 k+2$ digits. The desired result follows. + +Solution 2. Again, we assume that a sequence $a_{1}, a_{2}, a_{3}, \ldots$ satisfies the problem conditions, introduce the numbers $x_{k}$ and $y_{k}$ as in the previous solution, and notice that + +$$ +y_{k+1}-y_{k}=\left(x_{k+1}-x_{k}\right)\left(x_{k+1}+x_{k}\right)=10^{k} a_{k+1} +$$ + +for all $k>N$. Consider any such $k$. Since $a_{1} \neq 0$, the numbers $x_{k}$ and $x_{k+1}$ are not multiples of 10 , and therefore the numbers $p_{k}=x_{k+1}-x_{k}$ and $q_{k}=x_{k+1}+x_{k}$ cannot be simultaneously multiples of 20 , and hence one of them is not divisible either by 4 or by 5 . In view of (2), this means that the other one is divisible by either $5^{k}$ or by $2^{k-1}$. Notice also that $p_{k}$ and $q_{k}$ have the same parity, so both are even. + +On the other hand, we have $x_{k+1}^{2}=x_{k}^{2}+10^{k} a_{k+1} \geqslant x_{k}^{2}+10^{k}>2 x_{k}^{2}$, so $x_{k+1} / x_{k}>\sqrt{2}$, which implies that + +$$ +1<\frac{q_{k}}{p_{k}}=1+\frac{2}{x_{k+1} / x_{k}-1}<1+\frac{2}{\sqrt{2}-1}<6 . +$$ + +Thus, if one of the numbers $p_{k}$ and $q_{k}$ is divisible by $5^{k}$, then we have + +$$ +10^{k+1}>10^{k} a_{k+1}=p_{k} q_{k} \geqslant \frac{\left(5^{k}\right)^{2}}{6} +$$ + +and hence $(5 / 2)^{k}<60$ which is false for sufficiently large $k$. So, assuming that $k$ is large, we get that $2^{k-1}$ divides one of the numbers $p_{k}$ and $q_{k}$. Hence + +$$ +\left\{p_{k}, q_{k}\right\}=\left\{2^{k-1} \cdot 5^{r_{k}} b_{k}, 2 \cdot 5^{k-r_{k}} c_{k}\right\} \quad \text { with nonnegative integers } b_{k}, c_{k}, r_{k} \text { such that } b_{k} c_{k}=a_{k+1} \text {. } +$$ + +Moreover, from (3) we get + +$$ +6>\frac{2^{k-1} \cdot 5^{r_{k}} b_{k}}{2 \cdot 5^{k-r_{k}} c_{k}} \geqslant \frac{1}{36} \cdot\left(\frac{2}{5}\right)^{k} \cdot 5^{2 r_{k}} \quad \text { and } \quad 6>\frac{2 \cdot 5^{k-r_{k}} c_{k}}{2^{k-1} \cdot 5^{r_{k}} b_{k}} \geqslant \frac{4}{9} \cdot\left(\frac{5}{2}\right)^{k} \cdot 5^{-2 r_{k}} +$$ + +SO + +$$ +\alpha k+c_{1}c_{1} \text {. } +$$ + +Consequently, for $C=c_{2}-c_{1}+1-\alpha>0$ we have + +$$ +(k+1)-r_{k+1} \leqslant k-r_{k}+C . +$$ + +Next, we will use the following easy lemma. + +Lemma. Let $s$ be a positive integer. Then $5^{s+2^{s}} \equiv 5^{s}\left(\bmod 10^{s}\right)$. + +Proof. Euler's theorem gives $5^{2^{s}} \equiv 1\left(\bmod 2^{s}\right)$, so $5^{s+2^{s}}-5^{s}=5^{s}\left(5^{2^{s}}-1\right)$ is divisible by $2^{s}$ and $5^{s}$. + +Now, for every large $k$ we have + +$$ +x_{k+1}=\frac{p_{k}+q_{k}}{2}=5^{r_{k}} \cdot 2^{k-2} b_{k}+5^{k-r_{k}} c_{k} \equiv 5^{k-r_{k}} c_{k} \quad\left(\bmod 10^{r_{k}}\right) +$$ + +since $r_{k} \leqslant k-2$ by $(4)$; hence $y_{k+1} \equiv 5^{2\left(k-r_{k}\right)} c_{k}^{2}\left(\bmod 10^{r_{k}}\right)$. Let us consider some large integer $s$, and choose the minimal $k$ such that $2\left(k-r_{k}\right) \geqslant s+2^{s}$; it exists by (4). Set $d=2\left(k-r_{k}\right)-\left(s+2^{s}\right)$. By (4) we have $2^{s}<2\left(k-r_{k}\right)<\left(\frac{2}{\alpha}-2\right) r_{k}-\frac{2 c_{1}}{\alpha}$; if $s$ is large this implies $r_{k}>s$, so (6) also holds modulo $10^{s}$. Then (6) and the lemma give + +$$ +y_{k+1} \equiv 5^{2\left(k-r_{k}\right)} c_{k}^{2}=5^{s+2^{s}} \cdot 5^{d} c_{k}^{2} \equiv 5^{s} \cdot 5^{d} c_{k}^{2} \quad\left(\bmod 10^{s}\right) . +$$ + +By (5) and the minimality of $k$ we have $d \leqslant 2 C$, so $5^{d} c_{k}^{2} \leqslant 5^{2 C} \cdot 81=D$. Using $5^{4}<10^{3}$ we obtain + +$$ +5^{s} \cdot 5^{d} c_{k}^{2}<10^{3 s / 4} D<10^{s-1} +$$ + +for sufficiently large $s$. This, together with (7), shows that the sth digit from the right in $y_{k+1}$, which is $a_{s}$, is zero. This contradicts the problem condition. + +N5. Fix an integer $k \geqslant 2$. Two players, called Ana and Banana, play the following game of numbers: Initially, some integer $n \geqslant k$ gets written on the blackboard. Then they take moves in turn, with Ana beginning. A player making a move erases the number $m$ just written on the blackboard and replaces it by some number $m^{\prime}$ with $k \leqslant m^{\prime}m \geqslant k$ and $\operatorname{gcd}(m, n)=1$, then $n$ itself is bad, for Ana has the following winning strategy in the game with initial number $n$ : She proceeds by first playing $m$ and then using Banana's strategy for the game with starting number $m$. + +Otherwise, if some integer $n \geqslant k$ has the property that every integer $m$ with $n>m \geqslant k$ and $\operatorname{gcd}(m, n)=1$ is bad, then $n$ is good. Indeed, if Ana can make a first move at all in the game with initial number $n$, then she leaves it in a position where the first player has a winning strategy, so that Banana can defeat her. + +In particular, this implies that any two good numbers have a non-trivial common divisor. Also, $k$ itself is good. + +For brevity, we say that $n \longrightarrow x$ is a move if $n$ and $x$ are two coprime integers with $n>x \geqslant k$. + +Claim 1. If $n$ is good and $n^{\prime}$ is a multiple of $n$, then $n^{\prime}$ is also good. + +Proof. If $n^{\prime}$ were bad, there would have to be some move $n^{\prime} \longrightarrow x$, where $x$ is good. As $n^{\prime}$ is a multiple of $n$ this implies that the two good numbers $n$ and $x$ are coprime, which is absurd. + +Claim 2. If $r$ and $s$ denote two positive integers for which $r s \geqslant k$ is bad, then $r^{2} s$ is also bad. + +Proof. Since $r s$ is bad, there is a move $r s \longrightarrow x$ for some good $x$. Evidently $x$ is coprime to $r^{2} s$ as well, and hence the move $r^{2} s \longrightarrow x$ shows that $r^{2} s$ is indeed bad. + +Claim 3. If $p>k$ is prime and $n \geqslant k$ is bad, then $n p$ is also bad. + +Proof. Otherwise we choose a counterexample with $n$ being as small as possible. In particular, $n p$ is good. Since $n$ is bad, there is a move $n \longrightarrow x$ for some good $x$. Now $n p \longrightarrow x$ cannot be a valid move, which tells us that $x$ has to be divisible by $p$. So we can write $x=p^{r} y$, where $r$ and $y$ denote some positive integers, the latter of which is not divisible by $p$. + +Note that $y=1$ is impossible, for then we would have $x=p^{r}$ and the move $x \longrightarrow k$ would establish that $x$ is bad. In view of this, there is a least power $y^{\alpha}$ of $y$ that is at least as large as $k$. Since the numbers $n p$ and $y^{\alpha}$ are coprime and the former is good, the latter has to be bad. Moreover, the minimality of $\alpha$ implies $y^{\alpha}k$, but now we get the same contradiction using Claim 3 instead of Claim 2 . Thereby the problem is solved. + +Solution 2. We use the same analysis of the game of numbers as in the first five paragraphs of the first solution. Let us call a prime number $p$ small in case $p \leqslant k$ and big otherwise. We again call two integers similar if their sets of small prime factors coincide. + +Claim 4. For each integer $b \geqslant k$ having some small prime factor, there exists an integer $x$ similar to it with $b \geqslant x \geqslant k$ and having no big prime factors. + +Proof. Unless $b$ has a big prime factor we may simply choose $x=b$. Now let $p$ and $q$ denote a small and a big prime factor of $b$, respectively. Let $a$ be the product of all small prime factors of $b$. Further define $n$ to be the least non-negative integer for which the number $x=p^{n} a$ is at least as large as $k$. It suffices to show that $b>x$. This is clear in case $n=0$, so let us assume $n>0$ from now on. Then we have $x

b$. Applying Claim 4 to $b$ we get an integer $x$ with $b \geqslant x \geqslant k$ that is similar to $b$ and has no big prime divisors at all. By our assumption, $b^{\prime}$ and $x$ are coprime, and as $b^{\prime}$ is good this implies that $x$ is bad. Consequently there has to be some move $x \longrightarrow b^{*}$ such that $b^{*}$ is good. But now all the small prime factors of $b$ also appear in $x$ and thus they cannot divide $b^{*}$. Therefore the pair $\left(b^{*}, b\right)$ contradicts the supposed minimality of $b^{\prime}$. + +From that point, it is easy to complete the solution: assume that there are two similar integers $a$ and $b$ such that $a$ is bad and $b$ is good. Since $a$ is bad, there is a move $a \longrightarrow b^{\prime}$ for some good $b^{\prime}$. By Claim 5, there is a small prime $p$ dividing $b$ and $b^{\prime}$. Due to the similarity of $a$ and $b$, the prime $p$ has to divide $a$ as well, but this contradicts the fact that $a \longrightarrow b^{\prime}$ is a valid move. Thereby the problem is solved. + +Comment 2. There are infinitely many good numbers, e.g. all multiples of $k$. The increasing sequence $b_{0}, b_{1}, \ldots$, of all good numbers may be constructed recursively as follows: + +- Start with $b_{0}=k$. +- If $b_{n}$ has just been defined for some $n \geqslant 0$, then $b_{n+1}$ is the smallest number $b>b_{n}$ that is coprime to none of $b_{0}, \ldots, b_{n}$. + +This construction can be used to determine the set of good numbers for any specific $k$ as explained in the next comment. It is already clear that if $k=p^{\alpha}$ is a prime power, then a number $b \geqslant k$ is good if and only if it is divisible by $p$. + +Comment 3. Let $P>1$ denote the product of all small prime numbers. Then any two integers $a, b \geqslant k$ that are congruent modulo $P$ are similar. Thus the infinite word $W_{k}=\left(X_{k}, X_{k+1}, \ldots\right)$ defined by + +$$ +X_{i}= \begin{cases}A & \text { if } i \text { is bad } \\ B & \text { if } i \text { is good }\end{cases} +$$ + +for all $i \geqslant k$ is periodic and the length of its period divides $P$. As the prime power example shows, the true period can sometimes be much smaller than $P$. On the other hand, there are cases where the period is rather large; e.g., if $k=15$, the sequence of good numbers begins with 15,18,20,24,30,36,40,42, 45 and the period of $W_{15}$ is 30 . + +Comment 4. The original proposal contained two questions about the game of numbers, namely $(a)$ to show that if two numbers have the same prime factors then either both are good or both are bad, and (b) to show that the word $W_{k}$ introduced in the previous comment is indeed periodic. The Problem Selection Committee thinks that the above version of the problem is somewhat easier, even though it demands to prove a stronger result. + +N6. Determine all functions $f: \mathbb{Q} \longrightarrow \mathbb{Z}$ satisfying + +$$ +f\left(\frac{f(x)+a}{b}\right)=f\left(\frac{x+a}{b}\right) +$$ + +for all $x \in \mathbb{Q}, a \in \mathbb{Z}$, and $b \in \mathbb{Z}_{>0}$. (Here, $\mathbb{Z}_{>0}$ denotes the set of positive integers.) + +(Israel) + +Answer. There are three kinds of such functions, which are: all constant functions, the floor function, and the ceiling function. + +Solution 1. I. We start by verifying that these functions do indeed satisfy (1). This is clear for all constant functions. Now consider any triple $(x, a, b) \in \mathbb{Q} \times \mathbb{Z} \times \mathbb{Z}_{>0}$ and set + +$$ +q=\left\lfloor\frac{x+a}{b}\right\rfloor . +$$ + +This means that $q$ is an integer and $b q \leqslant x+a0}$. According to the behaviour of the restriction of $f$ to the integers we distinguish two cases. + +Case 1: There is some $m \in \mathbb{Z}$ such that $f(m) \neq m$. + +Write $f(m)=C$ and let $\eta \in\{-1,+1\}$ and $b$ denote the sign and absolute value of $f(m)-m$, respectively. Given any integer $r$, we may plug the triple $(m, r b-C, b)$ into (1), thus getting $f(r)=f(r-\eta)$. Starting with $m$ and using induction in both directions, we deduce from this that the equation $f(r)=C$ holds for all integers $r$. Now any rational number $y$ can be written in the form $y=\frac{p}{q}$ with $(p, q) \in \mathbb{Z} \times \mathbb{Z}_{>0}$, and substituting $(C-p, p-C, q)$ into (1) we get $f(y)=f(0)=C$. Thus $f$ is the constant function whose value is always $C$. + +Case 2: One has $f(m)=m$ for all integers $m$. + +Note that now the special case $b=1$ of (1) takes a particularly simple form, namely + +$$ +f(x)+a=f(x+a) \quad \text { for all }(x, a) \in \mathbb{Q} \times \mathbb{Z} +$$ + +Defining $f\left(\frac{1}{2}\right)=\omega$ we proceed in three steps. + +Step $A$. We show that $\omega \in\{0,1\}$. + +If $\omega \leqslant 0$, we may plug $\left(\frac{1}{2},-\omega, 1-2 \omega\right)$ into (1), obtaining $0=f(0)=f\left(\frac{1}{2}\right)=\omega$. In the contrary case $\omega \geqslant 1$ we argue similarly using the triple $\left(\frac{1}{2}, \omega-1,2 \omega-1\right)$. + +Step B. We show that $f(x)=\omega$ for all rational numbers $x$ with $0b \geqslant k+\omega$, which is absurd. Similarly, $m \geqslant r$ leads to $r a-m b0} +$$ + +Now suppose first that $x$ is not an integer but can be written in the form $\frac{p}{q}$ with $p \in \mathbb{Z}$ and $q \in \mathbb{Z}_{>0}$ both being odd. Let $d$ denote the multiplicative order of 2 modulo $q$ and let $m$ be any large integer. Plugging $n=d m$ into (6) and using (2) we get + +$$ +f(x)=\left[\frac{f\left(2^{d m} x\right)}{2^{d m}}\right]=\left[\frac{f(x)+\left(2^{d m}-1\right) x}{2^{d m}}\right]=\left[x+\frac{f(x)-x}{2^{d m}}\right] . +$$ + +Since $x$ is not an integer, the square bracket function is continuous at $x$; hence as $m$ tends to infinity the above fomula gives $f(x)=[x]$. To complete the argument we just need to observe that if some $y \in \mathbb{Q}$ satisfies $f(y)=[y]$, then (5) yields $f\left(\frac{y}{2}\right)=f\left(\frac{[y]}{2}\right)=\left[\frac{[y]}{2}\right]=\left[\frac{y}{2}\right]$. + +Solution 2. Here we just give another argument for the second case of the above solution. Again we use equation (2). It follows that the set $S$ of all zeros of $f$ contains for each $x \in \mathbb{Q}$ exactly one term from the infinite sequence $\ldots, x-2, x-1, x, x+1, x+2, \ldots$. + +Next we claim that + +$$ +\text { if }(p, q) \in \mathbb{Z} \times \mathbb{Z}_{>0} \text { and } \frac{p}{q} \in S \text {, then } \frac{p}{q+1} \in S \text { holds as well. } +$$ + +To see this we just plug $\left(\frac{p}{q}, p, q+1\right)$ into (1), thus getting $f\left(\frac{p}{q+1}\right)=f\left(\frac{p}{q}\right)=0$. + +From this we get that + +$$ +\text { if } x, y \in \mathbb{Q}, x>y>0 \text {, and } x \in S \text {, then } y \in S \text {. } +$$ + +Indeed, if we write $x=\frac{p}{q}$ and $y=\frac{r}{s}$ with $p, q, r, s \in \mathbb{Z}_{>0}$, then $p s>q r$ and (7) tells us + +$$ +0=f\left(\frac{p}{q}\right)=f\left(\frac{p r}{q r}\right)=f\left(\frac{p r}{q r+1}\right)=\ldots=f\left(\frac{p r}{p s}\right)=f\left(\frac{r}{s}\right) +$$ + +Essentially the same argument also establishes that + +$$ +\text { if } x, y \in \mathbb{Q}, x2 m$. Such a pair is necessarily a child of $(c, d-c)$, and thus a descendant of some pair $\left(c, d^{\prime}\right)$ with $m\{(b+k a) \nu\}=\left\{\left(b+k_{0} a\right) \nu\right\}+\left(k-k_{0}\right)\{a \nu\}$ for all $k>k_{0}$, which is absurd. + +Similarly, one can prove that $S_{m}$ contains finitely many pairs $(c, d)$ with $c>2 m$, thus finitely many elements at all. + +We are now prepared for proving the following crucial lemma. + +Lemma. Consider any pair $(a, b)$ with $f(a, b) \neq m$. Then the number $g(a, b)$ of its $m$-excellent descendants is equal to the number $h(a, b)$ of ways to represent the number $t=m-f(a, b)$ as $t=k a+\ell b$ with $k$ and $\ell$ being some nonnegative integers. + +Proof. We proceed by induction on the number $N$ of descendants of $(a, b)$ in $S_{m}$. If $N=0$ then clearly $g(a, b)=0$. Assume that $h(a, b)>0$; without loss of generality, we have $a \leqslant b$. Then, clearly, $m-f(a, b) \geqslant a$, so $f(a, b+a) \leqslant f(a, b)+a \leqslant m$ and $a \leqslant m$, hence $(a, b+a) \in S_{m}$ which is impossible. Thus in the base case we have $g(a, b)=h(a, b)=0$, as desired. + +Now let $N>0$. Assume that $f(a+b, b)=f(a, b)+b$ and $f(a, b+a)=f(a, b)$ (the other case is similar). If $f(a, b)+b \neq m$, then by the induction hypothesis we have + +$$ +g(a, b)=g(a+b, b)+g(a, b+a)=h(a+b, b)+h(a, b+a) +$$ + +Notice that both pairs $(a+b, b)$ and $(a, b+a)$ are descendants of $(a, b)$ and thus each of them has strictly less descendants in $S_{m}$ than $(a, b)$ does. + +Next, each one of the $h(a+b, b)$ representations of $m-f(a+b, b)=m-b-f(a, b)$ as the sum $k^{\prime}(a+b)+\ell^{\prime} b$ provides the representation $m-f(a, b)=k a+\ell b$ with $k=k^{\prime}1} d=\sum_{d \mid m} d +$$ + +as required. + +Comment. Let us present a sketch of an outline of a different solution. The plan is to check that the number of excellent pairs does not depend on the (irrational) number $\nu$, and to find this number for some appropriate value of $\nu$. For that, we first introduce some geometrical language. We deal only with the excellent pairs $(a, b)$ with $a \neq b$. + +Part I. Given an irrational positive $\nu$, for every positive integer $n$ we introduce two integral points $F_{\nu}(n)=$ $(n,\lfloor n \nu\rfloor)$ and $C_{\nu}(n)=(n,\lceil n \nu\rceil)$ on the coordinate plane $O x y$. Then $(*)$ reads as $\left[O F_{\nu}(a) C_{\nu}(b)\right]=m / 2$; here $[\cdot]$ stands for the signed area. Next, we rewrite in these terms the condition on a pair $(a, b)$ to be excellent. Let $\ell_{\nu}, \ell_{\nu}^{+}$, and $\ell_{\nu}^{-}$be the lines determined by the equations $y=\nu x, y=\nu x+1$, and $y=\nu x-1$, respectively. + +a). Firstly, we deal with all excellent pairs $(a, b)$ with $aa$ is excellent exactly when $p_{\nu}(a)$ lies between $b-a$ and $b$, and the point of $f_{\nu}(a)$ with abscissa $b$ is integral (which means that this point is $C_{\nu}(b)$ ). + +Notice now that, if $p_{\nu}(a)>a$, then the number of excellent pairs of the form $(a, b)$ (with $b>a$ ) is $\operatorname{gcd}(a,\lfloor a \nu\rfloor)$. + +![](https://cdn.mathpix.com/cropped/2024_04_17_e0914cb4f8055c731538g-67.jpg?height=396&width=422&top_left_y=1274&top_left_x=556) + +Figure 1 + +![](https://cdn.mathpix.com/cropped/2024_04_17_e0914cb4f8055c731538g-67.jpg?height=431&width=442&top_left_y=1254&top_left_x=1123) + +Figure 2 + +b). Analogously, considering the pairs $(a, b)$ with $a>b$, we fix the value of $b$, introduce the line $c_{\nu}(b)$ containing all the points $F$ with $\left[O F C_{\nu}(b)\right]=m / 2$, assume that this line contains an integral point (which means $\operatorname{gcd}(b,\lceil b \nu\rceil) \mid m$ ), and denote the common point of $c_{\nu}(b)$ and $\ell_{\nu}^{-}$by $Q_{\nu}(b)$, its abscissa being $q_{\nu}(b)$. Similarly to the previous case, we obtain that the pair $(a, b)$ is excellent exactly when $q_{\nu}(a)$ lies between $a-b$ and $a$, and the point of $c_{\nu}(b)$ with abscissa $a$ is integral (see Fig. 2). Again, if $q_{\nu}(b)>b$, then the number of excellent pairs of the form $(a, b)$ (with $a>b)$ is $\operatorname{gcd}(b,\lceil b \nu\rceil)$. + +Part II, sketchy. Having obtained such a description, one may check how the number of excellent pairs changes as $\nu$ grows. (Having done that, one may find this number for one appropriate value of $\nu$; for instance, it is relatively easy to make this calculation for $\nu \in\left(1,1+\frac{1}{m}\right)$.) + +Consider, for the initial value of $\nu$, some excellent pair $(a, t)$ with $a>t$. As $\nu$ grows, this pair eventually stops being excellent; this happens when the point $Q_{\nu}(t)$ passes through $F_{\nu}(a)$. At the same moment, the pair $(a+t, t)$ becomes excellent instead. + +This process halts when the point $Q_{\nu}(t)$ eventually disappears, i.e. when $\nu$ passes through the ratio of the coordinates of the point $T=C_{\nu}(t)$. Hence, the point $T$ afterwards is regarded as $F_{\nu}(t)$. Thus, all the old excellent pairs of the form $(a, t)$ with $a>t$ disappear; on the other hand, the same number of excellent pairs with the first element being $t$ just appear. + +Similarly, if some pair $(t, b)$ with $t0$. Show that there exists a positive integer $n$ such that + +$$ +\left(a_{n}-a_{n-1}\right)\left(b_{n}-b_{n-1}\right)<0 . +$$ + +(Denmark) +A3. For a sequence $x_{1}, x_{2}, \ldots, x_{n}$ of real numbers, we define its price as + +$$ +\max _{1 \leqslant i \leqslant n}\left|x_{1}+\cdots+x_{i}\right| +$$ + +Given $n$ real numbers, Dave and George want to arrange them into a sequence with a low price. Diligent Dave checks all possible ways and finds the minimum possible price $D$. Greedy George, on the other hand, chooses $x_{1}$ such that $\left|x_{1}\right|$ is as small as possible; among the remaining numbers, he chooses $x_{2}$ such that $\left|x_{1}+x_{2}\right|$ is as small as possible, and so on. Thus, in the $i^{\text {th }}$ step he chooses $x_{i}$ among the remaining numbers so as to minimise the value of $\left|x_{1}+x_{2}+\cdots+x_{i}\right|$. In each step, if several numbers provide the same value, George chooses one at random. Finally he gets a sequence with price $G$. + +Find the least possible constant $c$ such that for every positive integer $n$, for every collection of $n$ real numbers, and for every possible sequence that George might obtain, the resulting values satisfy the inequality $G \leqslant c D$. +(Georgia) +A4. Determine all functions $f: \mathbb{Z} \rightarrow \mathbb{Z}$ satisfying + +$$ +f(f(m)+n)+f(m)=f(n)+f(3 m)+2014 +$$ + +for all integers $m$ and $n$. + +A5. Consider all polynomials $P(x)$ with real coefficients that have the following property: for any two real numbers $x$ and $y$ one has + +$$ +\left|y^{2}-P(x)\right| \leqslant 2|x| \quad \text { if and only if } \quad\left|x^{2}-P(y)\right| \leqslant 2|y| +$$ + +Determine all possible values of $P(0)$. + +A6. Find all functions $f: \mathbb{Z} \rightarrow \mathbb{Z}$ such that + +$$ +n^{2}+4 f(n)=f(f(n))^{2} +$$ + +for all $n \in \mathbb{Z}$. + +## Combinatorics + +C1. Let $n$ points be given inside a rectangle $R$ such that no two of them lie on a line parallel to one of the sides of $R$. The rectangle $R$ is to be dissected into smaller rectangles with sides parallel to the sides of $R$ in such a way that none of these rectangles contains any of the given points in its interior. Prove that we have to dissect $R$ into at least $n+1$ smaller rectangles. +(Serbia) +C2. We have $2^{m}$ sheets of paper, with the number 1 written on each of them. We perform the following operation. In every step we choose two distinct sheets; if the numbers on the two sheets are $a$ and $b$, then we erase these numbers and write the number $a+b$ on both sheets. Prove that after $m 2^{m-1}$ steps, the sum of the numbers on all the sheets is at least $4^{m}$. +(Iran) +C3. Let $n \geqslant 2$ be an integer. Consider an $n \times n$ chessboard divided into $n^{2}$ unit squares. We call a configuration of $n$ rooks on this board happy if every row and every column contains exactly one rook. Find the greatest positive integer $k$ such that for every happy configuration of rooks, we can find a $k \times k$ square without a rook on any of its $k^{2}$ unit squares. +(Croatia) +C4. Construct a tetromino by attaching two $2 \times 1$ dominoes along their longer sides such that the midpoint of the longer side of one domino is a corner of the other domino. This construction yields two kinds of tetrominoes with opposite orientations. Let us call them Sand Z-tetrominoes, respectively. +![](https://cdn.mathpix.com/cropped/2024_11_18_62c85fb0b01b41aca29bg-07.jpg?height=181&width=321&top_left_y=1349&top_left_x=636) + +S-tetrominoes +![](https://cdn.mathpix.com/cropped/2024_11_18_62c85fb0b01b41aca29bg-07.jpg?height=167&width=318&top_left_y=1364&top_left_x=1115) + +Z-tetrominoes + +Assume that a lattice polygon $P$ can be tiled with S-tetrominoes. Prove than no matter how we tile $P$ using only S- and Z-tetrominoes, we always use an even number of Z-tetrominoes. +(Hungary) +C5. Consider $n \geqslant 3$ lines in the plane such that no two lines are parallel and no three have a common point. These lines divide the plane into polygonal regions; let $\mathcal{F}$ be the set of regions having finite area. Prove that it is possible to colour $\lceil\sqrt{n / 2}\rceil$ of the lines blue in such a way that no region in $\mathcal{F}$ has a completely blue boundary. (For a real number $x,\lceil x\rceil$ denotes the least integer which is not smaller than $x$.) + +C6. We are given an infinite deck of cards, each with a real number on it. For every real number $x$, there is exactly one card in the deck that has $x$ written on it. Now two players draw disjoint sets $A$ and $B$ of 100 cards each from this deck. We would like to define a rule that declares one of them a winner. This rule should satisfy the following conditions: + +1. The winner only depends on the relative order of the 200 cards: if the cards are laid down in increasing order face down and we are told which card belongs to which player, but not what numbers are written on them, we can still decide the winner. +2. If we write the elements of both sets in increasing order as $A=\left\{a_{1}, a_{2}, \ldots, a_{100}\right\}$ and $B=\left\{b_{1}, b_{2}, \ldots, b_{100}\right\}$, and $a_{i}>b_{i}$ for all $i$, then $A$ beats $B$. +3. If three players draw three disjoint sets $A, B, C$ from the deck, $A$ beats $B$ and $B$ beats $C$, then $A$ also beats $C$. + +How many ways are there to define such a rule? Here, we consider two rules as different if there exist two sets $A$ and $B$ such that $A$ beats $B$ according to one rule, but $B$ beats $A$ according to the other. +(Russia) +C7. Let $M$ be a set of $n \geqslant 4$ points in the plane, no three of which are collinear. Initially these points are connected with $n$ segments so that each point in $M$ is the endpoint of exactly two segments. Then, at each step, one may choose two segments $A B$ and $C D$ sharing a common interior point and replace them by the segments $A C$ and $B D$ if none of them is present at this moment. Prove that it is impossible to perform $n^{3} / 4$ or more such moves. +(Russia) +C8. A card deck consists of 1024 cards. On each card, a set of distinct decimal digits is written in such a way that no two of these sets coincide (thus, one of the cards is empty). Two players alternately take cards from the deck, one card per turn. After the deck is empty, each player checks if he can throw out one of his cards so that each of the ten digits occurs on an even number of his remaining cards. If one player can do this but the other one cannot, the one who can is the winner; otherwise a draw is declared. + +Determine all possible first moves of the first player after which he has a winning strategy. +(Russia) +C9. There are $n$ circles drawn on a piece of paper in such a way that any two circles intersect in two points, and no three circles pass through the same point. Turbo the snail slides along the circles in the following fashion. Initially he moves on one of the circles in clockwise direction. Turbo always keeps sliding along the current circle until he reaches an intersection with another circle. Then he continues his journey on this new circle and also changes the direction of moving, i.e. from clockwise to anticlockwise or vice versa. + +Suppose that Turbo's path entirely covers all circles. Prove that $n$ must be odd. + +## Geometry + +G1. The points $P$ and $Q$ are chosen on the side $B C$ of an acute-angled triangle $A B C$ so that $\angle P A B=\angle A C B$ and $\angle Q A C=\angle C B A$. The points $M$ and $N$ are taken on the rays $A P$ and $A Q$, respectively, so that $A P=P M$ and $A Q=Q N$. Prove that the lines $B M$ and $C N$ intersect on the circumcircle of the triangle $A B C$. +(Georgia) +G2. Let $A B C$ be a triangle. The points $K, L$, and $M$ lie on the segments $B C, C A$, and $A B$, respectively, such that the lines $A K, B L$, and $C M$ intersect in a common point. Prove that it is possible to choose two of the triangles $A L M, B M K$, and $C K L$ whose inradii sum up to at least the inradius of the triangle $A B C$. +(Estonia) +G3. Let $\Omega$ and $O$ be the circumcircle and the circumcentre of an acute-angled triangle $A B C$ with $A B>B C$. The angle bisector of $\angle A B C$ intersects $\Omega$ at $M \neq B$. Let $\Gamma$ be the circle with diameter $B M$. The angle bisectors of $\angle A O B$ and $\angle B O C$ intersect $\Gamma$ at points $P$ and $Q$, respectively. The point $R$ is chosen on the line $P Q$ so that $B R=M R$. Prove that $B R \| A C$. (Here we always assume that an angle bisector is a ray.) +(Russia) +G4. Consider a fixed circle $\Gamma$ with three fixed points $A, B$, and $C$ on it. Also, let us fix a real number $\lambda \in(0,1)$. For a variable point $P \notin\{A, B, C\}$ on $\Gamma$, let $M$ be the point on the segment $C P$ such that $C M=\lambda \cdot C P$. Let $Q$ be the second point of intersection of the circumcircles of the triangles $A M P$ and $B M C$. Prove that as $P$ varies, the point $Q$ lies on a fixed circle. +(United Kingdom) +G5. Let $A B C D$ be a convex quadrilateral with $\angle B=\angle D=90^{\circ}$. Point $H$ is the foot of the perpendicular from $A$ to $B D$. The points $S$ and $T$ are chosen on the sides $A B$ and $A D$, respectively, in such a way that $H$ lies inside triangle $S C T$ and + +$$ +\angle S H C-\angle B S C=90^{\circ}, \quad \angle T H C-\angle D T C=90^{\circ} . +$$ + +Prove that the circumcircle of triangle $S H T$ is tangent to the line $B D$. +(Iran) +G6. Let $A B C$ be a fixed acute-angled triangle. Consider some points $E$ and $F$ lying on the sides $A C$ and $A B$, respectively, and let $M$ be the midpoint of $E F$. Let the perpendicular bisector of $E F$ intersect the line $B C$ at $K$, and let the perpendicular bisector of $M K$ intersect the lines $A C$ and $A B$ at $S$ and $T$, respectively. We call the pair $(E, F)$ interesting, if the quadrilateral $K S A T$ is cyclic. + +Suppose that the pairs $\left(E_{1}, F_{1}\right)$ and $\left(E_{2}, F_{2}\right)$ are interesting. Prove that + +$$ +\frac{E_{1} E_{2}}{A B}=\frac{F_{1} F_{2}}{A C} . +$$ + +G7. Let $A B C$ be a triangle with circumcircle $\Omega$ and incentre $I$. Let the line passing through $I$ and perpendicular to $C I$ intersect the segment $B C$ and the $\operatorname{arc} B C($ not containing $A)$ of $\Omega$ at points $U$ and $V$, respectively. Let the line passing through $U$ and parallel to $A I$ intersect $A V$ at $X$, and let the line passing through $V$ and parallel to $A I$ intersect $A B$ at $Y$. Let $W$ and $Z$ be the midpoints of $A X$ and $B C$, respectively. Prove that if the points $I, X$, and $Y$ are collinear, then the points $I, W$, and $Z$ are also collinear. + +## Number Theory + +N1. Let $n \geqslant 2$ be an integer, and let $A_{n}$ be the set + +$$ +A_{n}=\left\{2^{n}-2^{k} \mid k \in \mathbb{Z}, 0 \leqslant k1$ be a given integer. Prove that infinitely many terms of the sequence $\left(a_{k}\right)_{k \geqslant 1}$, defined by + +$$ +a_{k}=\left\lfloor\frac{n^{k}}{k}\right\rfloor +$$ + +are odd. (For a real number $x,\lfloor x\rfloor$ denotes the largest integer not exceeding $x$.) +(Hong Kong) +N5. Find all triples $(p, x, y)$ consisting of a prime number $p$ and two positive integers $x$ and $y$ such that $x^{p-1}+y$ and $x+y^{p-1}$ are both powers of $p$. +(Belgium) +N6. Let $a_{1}0$. + +Notice that + +$$ +n z_{n+1}-\left(z_{0}+z_{1}+\cdots+z_{n}\right)=(n+1) z_{n+1}-\left(z_{0}+z_{1}+\cdots+z_{n}+z_{n+1}\right)=-d_{n+1} +$$ + +so the second inequality in (1) is equivalent to $d_{n+1} \leqslant 0$. Therefore, we have to prove that there is a unique index $n \geqslant 1$ that satisfies $d_{n}>0 \geqslant d_{n+1}$. + +By its definition the sequence $d_{1}, d_{2}, \ldots$ consists of integers and we have + +$$ +d_{1}=\left(z_{0}+z_{1}\right)-1 \cdot z_{1}=z_{0}>0 +$$ + +From +$d_{n+1}-d_{n}=\left(\left(z_{0}+\cdots+z_{n}+z_{n+1}\right)-(n+1) z_{n+1}\right)-\left(\left(z_{0}+\cdots+z_{n}\right)-n z_{n}\right)=n\left(z_{n}-z_{n+1}\right)<0$ +we can see that $d_{n+1}d_{2}>\ldots$ of integers such that its first element $d_{1}$ is positive. The sequence must drop below 0 at some point, and thus there is a unique index $n$, that is the index of the last positive term, satisfying $d_{n}>0 \geqslant d_{n+1}$. + +Comment. Omitting the assumption that $z_{1}, z_{2}, \ldots$ are integers allows the numbers $d_{n}$ to be all positive. In such cases the desired $n$ does not exist. This happens for example if $z_{n}=2-\frac{1}{2^{n}}$ for all integers $n \geqslant 0$. + +A2. Define the function $f:(0,1) \rightarrow(0,1)$ by + +$$ +f(x)= \begin{cases}x+\frac{1}{2} & \text { if } x<\frac{1}{2} \\ x^{2} & \text { if } x \geqslant \frac{1}{2}\end{cases} +$$ + +Let $a$ and $b$ be two real numbers such that $00$. Show that there exists a positive integer $n$ such that + +$$ +\left(a_{n}-a_{n-1}\right)\left(b_{n}-b_{n-1}\right)<0 . +$$ + +(Denmark) +Solution. Note that + +$$ +f(x)-x=\frac{1}{2}>0 +$$ + +if $x<\frac{1}{2}$ and + +$$ +f(x)-x=x^{2}-x<0 +$$ + +if $x \geqslant \frac{1}{2}$. So if we consider $(0,1)$ as being divided into the two subintervals $I_{1}=\left(0, \frac{1}{2}\right)$ and $I_{2}=\left[\frac{1}{2}, 1\right)$, the inequality + +$$ +\left(a_{n}-a_{n-1}\right)\left(b_{n}-b_{n-1}\right)=\left(f\left(a_{n-1}\right)-a_{n-1}\right)\left(f\left(b_{n-1}\right)-b_{n-1}\right)<0 +$$ + +holds if and only if $a_{n-1}$ and $b_{n-1}$ lie in distinct subintervals. +Let us now assume, to the contrary, that $a_{k}$ and $b_{k}$ always lie in the same subinterval. Consider the distance $d_{k}=\left|a_{k}-b_{k}\right|$. If both $a_{k}$ and $b_{k}$ lie in $I_{1}$, then + +$$ +d_{k+1}=\left|a_{k+1}-b_{k+1}\right|=\left|a_{k}+\frac{1}{2}-b_{k}-\frac{1}{2}\right|=d_{k} +$$ + +If, on the other hand, $a_{k}$ and $b_{k}$ both lie in $I_{2}$, then $\min \left(a_{k}, b_{k}\right) \geqslant \frac{1}{2}$ and $\max \left(a_{k}, b_{k}\right)=$ $\min \left(a_{k}, b_{k}\right)+d_{k} \geqslant \frac{1}{2}+d_{k}$, which implies + +$$ +d_{k+1}=\left|a_{k+1}-b_{k+1}\right|=\left|a_{k}^{2}-b_{k}^{2}\right|=\left|\left(a_{k}-b_{k}\right)\left(a_{k}+b_{k}\right)\right| \geqslant\left|a_{k}-b_{k}\right|\left(\frac{1}{2}+\frac{1}{2}+d_{k}\right)=d_{k}\left(1+d_{k}\right) \geqslant d_{k} +$$ + +This means that the difference $d_{k}$ is non-decreasing, and in particular $d_{k} \geqslant d_{0}>0$ for all $k$. +We can even say more. If $a_{k}$ and $b_{k}$ lie in $I_{2}$, then + +$$ +d_{k+2} \geqslant d_{k+1} \geqslant d_{k}\left(1+d_{k}\right) \geqslant d_{k}\left(1+d_{0}\right) +$$ + +If $a_{k}$ and $b_{k}$ both lie in $I_{1}$, then $a_{k+1}$ and $b_{k+1}$ both lie in $I_{2}$, and so we have + +$$ +d_{k+2} \geqslant d_{k+1}\left(1+d_{k+1}\right) \geqslant d_{k+1}\left(1+d_{0}\right)=d_{k}\left(1+d_{0}\right) +$$ + +In either case, $d_{k+2} \geqslant d_{k}\left(1+d_{0}\right)$, and inductively we get + +$$ +d_{2 m} \geqslant d_{0}\left(1+d_{0}\right)^{m} +$$ + +For sufficiently large $m$, the right-hand side is greater than 1 , but since $a_{2 m}, b_{2 m}$ both lie in $(0,1)$, we must have $d_{2 m}<1$, a contradiction. + +Thus there must be a positive integer $n$ such that $a_{n-1}$ and $b_{n-1}$ do not lie in the same subinterval, which proves the desired statement. + +A3. For a sequence $x_{1}, x_{2}, \ldots, x_{n}$ of real numbers, we define its price as + +$$ +\max _{1 \leqslant i \leqslant n}\left|x_{1}+\cdots+x_{i}\right| +$$ + +Given $n$ real numbers, Dave and George want to arrange them into a sequence with a low price. Diligent Dave checks all possible ways and finds the minimum possible price $D$. Greedy George, on the other hand, chooses $x_{1}$ such that $\left|x_{1}\right|$ is as small as possible; among the remaining numbers, he chooses $x_{2}$ such that $\left|x_{1}+x_{2}\right|$ is as small as possible, and so on. Thus, in the $i^{\text {th }}$ step he chooses $x_{i}$ among the remaining numbers so as to minimise the value of $\left|x_{1}+x_{2}+\cdots+x_{i}\right|$. In each step, if several numbers provide the same value, George chooses one at random. Finally he gets a sequence with price $G$. + +Find the least possible constant $c$ such that for every positive integer $n$, for every collection of $n$ real numbers, and for every possible sequence that George might obtain, the resulting values satisfy the inequality $G \leqslant c D$. +(Georgia) +Answer. $c=2$. +Solution. If the initial numbers are $1,-1,2$, and -2 , then Dave may arrange them as $1,-2,2,-1$, while George may get the sequence $1,-1,2,-2$, resulting in $D=1$ and $G=2$. So we obtain $c \geqslant 2$. + +Therefore, it remains to prove that $G \leqslant 2 D$. Let $x_{1}, x_{2}, \ldots, x_{n}$ be the numbers Dave and George have at their disposal. Assume that Dave and George arrange them into sequences $d_{1}, d_{2}, \ldots, d_{n}$ and $g_{1}, g_{2}, \ldots, g_{n}$, respectively. Put + +$$ +M=\max _{1 \leqslant i \leqslant n}\left|x_{i}\right|, \quad S=\left|x_{1}+\cdots+x_{n}\right|, \quad \text { and } \quad N=\max \{M, S\} +$$ + +We claim that + +$$ +\begin{aligned} +& D \geqslant S, \\ +& D \geqslant \frac{M}{2}, \quad \text { and } \\ +& G \leqslant N=\max \{M, S\} +\end{aligned} +$$ + +These inequalities yield the desired estimate, as $G \leqslant \max \{M, S\} \leqslant \max \{M, 2 S\} \leqslant 2 D$. +The inequality (1) is a direct consequence of the definition of the price. +To prove (2), consider an index $i$ with $\left|d_{i}\right|=M$. Then we have + +$$ +M=\left|d_{i}\right|=\left|\left(d_{1}+\cdots+d_{i}\right)-\left(d_{1}+\cdots+d_{i-1}\right)\right| \leqslant\left|d_{1}+\cdots+d_{i}\right|+\left|d_{1}+\cdots+d_{i-1}\right| \leqslant 2 D +$$ + +as required. +It remains to establish (3). Put $h_{i}=g_{1}+g_{2}+\cdots+g_{i}$. We will prove by induction on $i$ that $\left|h_{i}\right| \leqslant N$. The base case $i=1$ holds, since $\left|h_{1}\right|=\left|g_{1}\right| \leqslant M \leqslant N$. Notice also that $\left|h_{n}\right|=S \leqslant N$. + +For the induction step, assume that $\left|h_{i-1}\right| \leqslant N$. We distinguish two cases. +Case 1. Assume that no two of the numbers $g_{i}, g_{i+1}, \ldots, g_{n}$ have opposite signs. +Without loss of generality, we may assume that they are all nonnegative. Then one has $h_{i-1} \leqslant h_{i} \leqslant \cdots \leqslant h_{n}$, thus + +$$ +\left|h_{i}\right| \leqslant \max \left\{\left|h_{i-1}\right|,\left|h_{n}\right|\right\} \leqslant N +$$ + +Case 2. Among the numbers $g_{i}, g_{i+1}, \ldots, g_{n}$ there are positive and negative ones. + +Then there exists some index $j \geqslant i$ such that $h_{i-1} g_{j} \leqslant 0$. By the definition of George's sequence we have + +$$ +\left|h_{i}\right|=\left|h_{i-1}+g_{i}\right| \leqslant\left|h_{i-1}+g_{j}\right| \leqslant \max \left\{\left|h_{i-1}\right|,\left|g_{j}\right|\right\} \leqslant N +$$ + +Thus, the induction step is established. +Comment 1. One can establish the weaker inequalities $D \geqslant \frac{M}{2}$ and $G \leqslant D+\frac{M}{2}$ from which the result also follows. + +Comment 2. One may ask a more specific question to find the maximal suitable $c$ if the number $n$ is fixed. For $n=1$ or 2 , the answer is $c=1$. For $n=3$, the answer is $c=\frac{3}{2}$, and it is reached e.g., for the collection $1,2,-4$. Finally, for $n \geqslant 4$ the answer is $c=2$. In this case the arguments from the solution above apply, and the answer is reached e.g., for the same collection $1,-1,2,-2$, augmented by several zeroes. + +A4. Determine all functions $f: \mathbb{Z} \rightarrow \mathbb{Z}$ satisfying + +$$ +f(f(m)+n)+f(m)=f(n)+f(3 m)+2014 +$$ + +for all integers $m$ and $n$. +(Netherlands) +Answer. There is only one such function, namely $n \longmapsto 2 n+1007$. +Solution. Let $f$ be a function satisfying (1). Set $C=1007$ and define the function $g: \mathbb{Z} \rightarrow \mathbb{Z}$ by $g(m)=f(3 m)-f(m)+2 C$ for all $m \in \mathbb{Z}$; in particular, $g(0)=2 C$. Now (1) rewrites as + +$$ +f(f(m)+n)=g(m)+f(n) +$$ + +for all $m, n \in \mathbb{Z}$. By induction in both directions it follows that + +$$ +f(t f(m)+n)=t g(m)+f(n) +$$ + +holds for all $m, n, t \in \mathbb{Z}$. Applying this, for any $r \in \mathbb{Z}$, to the triples $(r, 0, f(0))$ and $(0,0, f(r))$ in place of $(m, n, t)$ we obtain + +$$ +f(0) g(r)=f(f(r) f(0))-f(0)=f(r) g(0) +$$ + +Now if $f(0)$ vanished, then $g(0)=2 C>0$ would entail that $f$ vanishes identically, contrary to (1). Thus $f(0) \neq 0$ and the previous equation yields $g(r)=\alpha f(r)$, where $\alpha=\frac{g(0)}{f(0)}$ is some nonzero constant. + +So the definition of $g$ reveals $f(3 m)=(1+\alpha) f(m)-2 C$, i.e., + +$$ +f(3 m)-\beta=(1+\alpha)(f(m)-\beta) +$$ + +for all $m \in \mathbb{Z}$, where $\beta=\frac{2 C}{\alpha}$. By induction on $k$ this implies + +$$ +f\left(3^{k} m\right)-\beta=(1+\alpha)^{k}(f(m)-\beta) +$$ + +for all integers $k \geqslant 0$ and $m$. +Since $3 \nmid 2014$, there exists by (1) some value $d=f(a)$ attained by $f$ that is not divisible by 3 . Now by (2) we have $f(n+t d)=f(n)+t g(a)=f(n)+\alpha \cdot t f(a)$, i.e., + +$$ +f(n+t d)=f(n)+\alpha \cdot t d +$$ + +for all $n, t \in \mathbb{Z}$. +Let us fix any positive integer $k$ with $d \mid\left(3^{k}-1\right)$, which is possible, since $\operatorname{gcd}(3, d)=1$. E.g., by the Euler-Fermat theorem, we may take $k=\varphi(|d|)$. Now for each $m \in \mathbb{Z}$ we get + +$$ +f\left(3^{k} m\right)=f(m)+\alpha\left(3^{k}-1\right) m +$$ + +from (5), which in view of (4) yields $\left((1+\alpha)^{k}-1\right)(f(m)-\beta)=\alpha\left(3^{k}-1\right) m$. Since $\alpha \neq 0$, the right hand side does not vanish for $m \neq 0$, wherefore the first factor on the left hand side cannot vanish either. It follows that + +$$ +f(m)=\frac{\alpha\left(3^{k}-1\right)}{(1+\alpha)^{k}-1} \cdot m+\beta +$$ + +So $f$ is a linear function, say $f(m)=A m+\beta$ for all $m \in \mathbb{Z}$ with some constant $A \in \mathbb{Q}$. Plugging this into (1) one obtains $\left(A^{2}-2 A\right) m+(A \beta-2 C)=0$ for all $m$, which is equivalent to the conjunction of + +$$ +A^{2}=2 A \quad \text { and } \quad A \beta=2 C +$$ + +The first equation is equivalent to $A \in\{0,2\}$, and as $C \neq 0$ the second one gives + +$$ +A=2 \quad \text { and } \quad \beta=C +$$ + +This shows that $f$ is indeed the function mentioned in the answer and as the numbers found in (7) do indeed satisfy the equations (6) this function is indeed as desired. + +Comment 1. One may see that $\alpha=2$. A more pedestrian version of the above solution starts with a direct proof of this fact, that can be obtained by substituting some special values into (1), e.g., as follows. + +Set $D=f(0)$. Plugging $m=0$ into (1) and simplifying, we get + +$$ +f(n+D)=f(n)+2 C +$$ + +for all $n \in \mathbb{Z}$. In particular, for $n=0, D, 2 D$ we obtain $f(D)=2 C+D, f(2 D)=f(D)+2 C=4 C+D$, and $f(3 D)=f(2 D)+2 C=6 C+D$. So substituting $m=D$ and $n=r-D$ into (1) and applying (8) with $n=r-D$ afterwards we learn + +$$ +f(r+2 C)+2 C+D=(f(r)-2 C)+(6 C+D)+2 C +$$ + +i.e., $f(r+2 C)=f(r)+4 C$. By induction in both directions it follows that + +$$ +f(n+2 C t)=f(n)+4 C t +$$ + +holds for all $n, t \in \mathbb{Z}$. +Claim. If $a$ and $b$ denote two integers with the property that $f(n+a)=f(n)+b$ holds for all $n \in \mathbb{Z}$, then $b=2 a$. +Proof. Applying induction in both directions to the assumption we get $f(n+t a)=f(n)+t b$ for all $n, t \in \mathbb{Z}$. Plugging $(n, t)=(0,2 C)$ into this equation and $(n, t)=(0, a)$ into $(9)$ we get $f(2 a C)-f(0)=$ $2 b C=4 a C$, and, as $C \neq 0$, the claim follows. + +Now by (1), for any $m \in \mathbb{Z}$, the numbers $a=f(m)$ and $b=f(3 m)-f(m)+2 C$ have the property mentioned in the claim, whence we have + +$$ +f(3 m)-C=3(f(m)-C) . +$$ + +In view of (3) this tells us indeed that $\alpha=2$. +Now the solution may be completed as above, but due to our knowledge of $\alpha=2$ we get the desired formula $f(m)=2 m+C$ directly without having the need to go through all linear functions. Now it just remains to check that this function does indeed satisfy (1). + +Comment 2. It is natural to wonder what happens if one replaces the number 2014 appearing in the statement of the problem by some arbitrary integer $B$. + +If $B$ is odd, there is no such function, as can be seen by using the same ideas as in the above solution. + +If $B \neq 0$ is even, however, then the only such function is given by $n \longmapsto 2 n+B / 2$. In case $3 \nmid B$ this was essentially proved above, but for the general case one more idea seems to be necessary. Writing $B=3^{\nu} \cdot k$ with some integers $\nu$ and $k$ such that $3 \nmid k$ one can obtain $f(n)=2 n+B / 2$ for all $n$ that are divisible by $3^{\nu}$ in the same manner as usual; then one may use the formula $f(3 n)=3 f(n)-B$ to establish the remaining cases. + +Finally, in case $B=0$ there are more solutions than just the function $n \longmapsto 2 n$. It can be shown that all these other functions are periodic; to mention just one kind of example, for any even integers $r$ and $s$ the function + +$$ +f(n)= \begin{cases}r & \text { if } n \text { is even, } \\ s & \text { if } n \text { is odd }\end{cases} +$$ + +also has the property under discussion. + +A5. Consider all polynomials $P(x)$ with real coefficients that have the following property: for any two real numbers $x$ and $y$ one has + +$$ +\left|y^{2}-P(x)\right| \leqslant 2|x| \quad \text { if and only if } \quad\left|x^{2}-P(y)\right| \leqslant 2|y| +$$ + +Determine all possible values of $P(0)$. +(Belgium) +Answer. The set of possible values of $P(0)$ is $(-\infty, 0) \cup\{1\}$. + +## Solution. + +Part I. We begin by verifying that these numbers are indeed possible values of $P(0)$. To see that each negative real number $-C$ can be $P(0)$, it suffices to check that for every $C>0$ the polynomial $P(x)=-\left(\frac{2 x^{2}}{C}+C\right)$ has the property described in the statement of the problem. Due to symmetry it is enough for this purpose to prove $\left|y^{2}-P(x)\right|>2|x|$ for any two real numbers $x$ and $y$. In fact we have + +$$ +\left|y^{2}-P(x)\right|=y^{2}+\frac{x^{2}}{C}+\frac{(|x|-C)^{2}}{C}+2|x| \geqslant \frac{x^{2}}{C}+2|x| \geqslant 2|x|, +$$ + +where in the first estimate equality can only hold if $|x|=C$, whilst in the second one it can only hold if $x=0$. As these two conditions cannot be met at the same time, we have indeed $\left|y^{2}-P(x)\right|>2|x|$. + +To show that $P(0)=1$ is possible as well, we verify that the polynomial $P(x)=x^{2}+1$ satisfies (1). Notice that for all real numbers $x$ and $y$ we have + +$$ +\begin{aligned} +\left|y^{2}-P(x)\right| \leqslant 2|x| & \Longleftrightarrow\left(y^{2}-x^{2}-1\right)^{2} \leqslant 4 x^{2} \\ +& \Longleftrightarrow 0 \leqslant\left(\left(y^{2}-(x-1)^{2}\right)\left((x+1)^{2}-y^{2}\right)\right. \\ +& \Longleftrightarrow 0 \leqslant(y-x+1)(y+x-1)(x+1-y)(x+1+y) \\ +& \Longleftrightarrow 0 \leqslant\left((x+y)^{2}-1\right)\left(1-(x-y)^{2}\right) . +\end{aligned} +$$ + +Since this inequality is symmetric in $x$ and $y$, we are done. +Part II. Now we show that no values other than those mentioned in the answer are possible for $P(0)$. To reach this we let $P$ denote any polynomial satisfying (1) and $P(0) \geqslant 0$; as we shall see, this implies $P(x)=x^{2}+1$ for all real $x$, which is actually more than what we want. + +First step: We prove that $P$ is even. +By (1) we have + +$$ +\left|y^{2}-P(x)\right| \leqslant 2|x| \Longleftrightarrow\left|x^{2}-P(y)\right| \leqslant 2|y| \Longleftrightarrow\left|y^{2}-P(-x)\right| \leqslant 2|x| +$$ + +for all real numbers $x$ and $y$. Considering just the equivalence of the first and third statement and taking into account that $y^{2}$ may vary through $\mathbb{R}_{\geqslant 0}$ we infer that + +$$ +[P(x)-2|x|, P(x)+2|x|] \cap \mathbb{R}_{\geqslant 0}=[P(-x)-2|x|, P(-x)+2|x|] \cap \mathbb{R}_{\geqslant 0} +$$ + +holds for all $x \in \mathbb{R}$. We claim that there are infinitely many real numbers $x$ such that $P(x)+2|x| \geqslant 0$. This holds in fact for any real polynomial with $P(0) \geqslant 0$; in order to see this, we may assume that the coefficient of $P$ appearing in front of $x$ is nonnegative. In this case the desired inequality holds for all sufficiently small positive real numbers. + +For such numbers $x$ satisfying $P(x)+2|x| \geqslant 0$ we have $P(x)+2|x|=P(-x)+2|x|$ by the previous displayed formula, and hence also $P(x)=P(-x)$. Consequently the polynomial $P(x)-P(-x)$ has infinitely many zeros, wherefore it has to vanish identically. Thus $P$ is indeed even. + +Second step: We prove that $P(t)>0$ for all $t \in \mathbb{R}$. +Let us assume for a moment that there exists a real number $t \neq 0$ with $P(t)=0$. Then there is some open interval $I$ around $t$ such that $|P(y)| \leqslant 2|y|$ holds for all $y \in I$. Plugging $x=0$ into (1) we learn that $y^{2}=P(0)$ holds for all $y \in I$, which is clearly absurd. We have thus shown $P(t) \neq 0$ for all $t \neq 0$. + +In combination with $P(0) \geqslant 0$ this informs us that our claim could only fail if $P(0)=0$. In this case there is by our first step a polynomial $Q(x)$ such that $P(x)=x^{2} Q(x)$. Applying (1) to $x=0$ and an arbitrary $y \neq 0$ we get $|y Q(y)|>2$, which is surely false when $y$ is sufficiently small. + +Third step: We prove that $P$ is a quadratic polynomial. +Notice that $P$ cannot be constant, for otherwise if $x=\sqrt{P(0)}$ and $y$ is sufficiently large, the first part of (1) is false whilst the second part is true. So the degree $n$ of $P$ has to be at least 1 . By our first step $n$ has to be even as well, whence in particular $n \geqslant 2$. + +Now assume that $n \geqslant 4$. Plugging $y=\sqrt{P(x)}$ into (1) we get $\left|x^{2}-P(\sqrt{P(x)})\right| \leqslant 2 \sqrt{P(x)}$ and hence + +$$ +P(\sqrt{P(x)}) \leqslant x^{2}+2 \sqrt{P(x)} +$$ + +for all real $x$. Choose positive real numbers $x_{0}, a$, and $b$ such that if $x \in\left(x_{0}, \infty\right)$, then $a x^{n}<$ $P(x)0$ denotes the leading coefficient of $P$, then $\lim _{x \rightarrow \infty} \frac{P(x)}{x^{n}}=d$, whence for instance the numbers $a=\frac{d}{2}$ and $b=2 d$ work provided that $x_{0}$ is chosen large enough. + +Now for all sufficiently large real numbers $x$ we have + +$$ +a^{n / 2+1} x^{n^{2} / 2}0$ and $b$ such that $P(x)=$ $a x^{2}+b$. Now if $x$ is large enough and $y=\sqrt{a} x$, the left part of (1) holds and the right part reads $\left|\left(1-a^{2}\right) x^{2}-b\right| \leqslant 2 \sqrt{a} x$. In view of the fact that $a>0$ this is only possible if $a=1$. Finally, substituting $y=x+1$ with $x>0$ into (1) we get + +$$ +|2 x+1-b| \leqslant 2 x \Longleftrightarrow|2 x+1+b| \leqslant 2 x+2 +$$ + +i.e., + +$$ +b \in[1,4 x+1] \Longleftrightarrow b \in[-4 x-3,1] +$$ + +for all $x>0$. Choosing $x$ large enough, we can achieve that at least one of these two statements holds; then both hold, which is only possible if $b=1$, as desired. + +Comment 1. There are some issues with this problem in that its most natural solutions seem to use some basic facts from analysis, such as the continuity of polynomials or the intermediate value theorem. Yet these facts are intuitively obvious and implicitly clear to the students competing at this level of difficulty, so that the Problem Selection Committee still thinks that the problem is suitable for the IMO. + +Comment 2. It seems that most solutions will in the main case, where $P(0)$ is nonnegative, contain an argument that is somewhat asymptotic in nature showing that $P$ is quadratic, and some part narrowing that case down to $P(x)=x^{2}+1$. + +Comment 3. It is also possible to skip the first step and start with the second step directly, but then one has to work a bit harder to rule out the case $P(0)=0$. Let us sketch one possibility of doing this: Take the auxiliary polynomial $Q(x)$ such that $P(x)=x Q(x)$. Applying (1) to $x=0$ and an arbitrary $y \neq 0$ we get $|Q(y)|>2$. Hence we either have $Q(z) \geqslant 2$ for all real $z$ or $Q(z) \leqslant-2$ for all real $z$. In particular there is some $\eta \in\{-1,+1\}$ such that $P(\eta) \geqslant 2$ and $P(-\eta) \leqslant-2$. Substituting $x= \pm \eta$ into (1) we learn + +$$ +\left|y^{2}-P(\eta)\right| \leqslant 2 \Longleftrightarrow|1-P(y)| \leqslant 2|y| \Longleftrightarrow\left|y^{2}-P(-\eta)\right| \leqslant 2 +$$ + +But for $y=\sqrt{P(\eta)}$ the first statement is true, whilst the third one is false. +Also, if one has not obtained the evenness of $P$ before embarking on the fourth step, one needs to work a bit harder there, but not in a way that is likely to cause major difficulties. + +Comment 4. Truly curious people may wonder about the set of all polynomials having property (1). As explained in the solution above, $P(x)=x^{2}+1$ is the only one with $P(0)=1$. On the other hand, it is not hard to notice that for negative $P(0)$ there are more possibilities than those mentioned above. E.g., as remarked by the proposer, if $a$ and $b$ denote two positive real numbers with $a b>1$ and $Q$ denotes a polynomial attaining nonnegative values only, then $P(x)=-\left(a x^{2}+b+Q(x)\right)$ works. + +More generally, it may be proved that if $P(x)$ satisfies (1) and $P(0)<0$, then $-P(x)>2|x|$ holds for all $x \in \mathbb{R}$ so that one just considers the equivalence of two false statements. One may generate all such polynomials $P$ by going through all combinations of a solution of the polynomial equation + +$$ +x=A(x) B(x)+C(x) D(x) +$$ + +and a real $E>0$, and setting + +$$ +P(x)=-\left(A(x)^{2}+B(x)^{2}+C(x)^{2}+D(x)^{2}+E\right) +$$ + +for each of them. + +A6. Find all functions $f: \mathbb{Z} \rightarrow \mathbb{Z}$ such that + +$$ +n^{2}+4 f(n)=f(f(n))^{2} +$$ + +for all $n \in \mathbb{Z}$. +(United Kingdom) +Answer. The possibilities are: + +- $f(n)=n+1$ for all $n$; +- or, for some $a \geqslant 1, \quad f(n)= \begin{cases}n+1, & n>-a, \\ -n+1, & n \leqslant-a ;\end{cases}$ +- or $f(n)= \begin{cases}n+1, & n>0, \\ 0, & n=0, \\ -n+1, & n<0 .\end{cases}$ + + +## Solution 1. + +Part I. Let us first check that each of the functions above really satisfies the given functional equation. If $f(n)=n+1$ for all $n$, then we have + +$$ +n^{2}+4 f(n)=n^{2}+4 n+4=(n+2)^{2}=f(n+1)^{2}=f(f(n))^{2} . +$$ + +If $f(n)=n+1$ for $n>-a$ and $f(n)=-n+1$ otherwise, then we have the same identity for $n>-a$ and + +$$ +n^{2}+4 f(n)=n^{2}-4 n+4=(2-n)^{2}=f(1-n)^{2}=f(f(n))^{2} +$$ + +otherwise. The same applies to the third solution (with $a=0$ ), where in addition one has + +$$ +0^{2}+4 f(0)=0=f(f(0))^{2} +$$ + +Part II. It remains to prove that these are really the only functions that satisfy our functional equation. We do so in three steps: + +Step 1: We prove that $f(n)=n+1$ for $n>0$. +Consider the sequence $\left(a_{k}\right)$ given by $a_{k}=f^{k}(1)$ for $k \geqslant 0$. Setting $n=a_{k}$ in (1), we get + +$$ +a_{k}^{2}+4 a_{k+1}=a_{k+2}^{2} +$$ + +Of course, $a_{0}=1$ by definition. Since $a_{2}^{2}=1+4 a_{1}$ is odd, $a_{2}$ has to be odd as well, so we set $a_{2}=2 r+1$ for some $r \in \mathbb{Z}$. Then $a_{1}=r^{2}+r$ and consequently + +$$ +a_{3}^{2}=a_{1}^{2}+4 a_{2}=\left(r^{2}+r\right)^{2}+8 r+4 +$$ + +Since $8 r+4 \neq 0, a_{3}^{2} \neq\left(r^{2}+r\right)^{2}$, so the difference between $a_{3}^{2}$ and $\left(r^{2}+r\right)^{2}$ is at least the distance from $\left(r^{2}+r\right)^{2}$ to the nearest even square (since $8 r+4$ and $r^{2}+r$ are both even). This implies that + +$$ +|8 r+4|=\left|a_{3}^{2}-\left(r^{2}+r\right)^{2}\right| \geqslant\left(r^{2}+r\right)^{2}-\left(r^{2}+r-2\right)^{2}=4\left(r^{2}+r-1\right) +$$ + +(for $r=0$ and $r=-1$, the estimate is trivial, but this does not matter). Therefore, we ave + +$$ +4 r^{2} \leqslant|8 r+4|-4 r+4 +$$ + +If $|r| \geqslant 4$, then + +$$ +4 r^{2} \geqslant 16|r| \geqslant 12|r|+16>8|r|+4+4|r|+4 \geqslant|8 r+4|-4 r+4 +$$ + +a contradiction. Thus $|r|<4$. Checking all possible remaining values of $r$, we find that $\left(r^{2}+r\right)^{2}+8 r+4$ is only a square in three cases: $r=-3, r=0$ and $r=1$. Let us now distinguish these three cases: + +- $r=-3$, thus $a_{1}=6$ and $a_{2}=-5$. For each $k \geqslant 1$, we have + +$$ +a_{k+2}= \pm \sqrt{a_{k}^{2}+4 a_{k+1}} +$$ + +and the sign needs to be chosen in such a way that $a_{k+1}^{2}+4 a_{k+2}$ is again a square. This yields $a_{3}=-4, a_{4}=-3, a_{5}=-2, a_{6}=-1, a_{7}=0, a_{8}=1, a_{9}=2$. At this point we have reached a contradiction, since $f(1)=f\left(a_{0}\right)=a_{1}=6$ and at the same time $f(1)=f\left(a_{8}\right)=a_{9}=2$. + +- $r=0$, thus $a_{1}=0$ and $a_{2}=1$. Then $a_{3}^{2}=a_{1}^{2}+4 a_{2}=4$, so $a_{3}= \pm 2$. This, however, is a contradiction again, since it gives us $f(1)=f\left(a_{0}\right)=a_{1}=0$ and at the same time $f(1)=f\left(a_{2}\right)=a_{3}= \pm 2$. +- $r=1$, thus $a_{1}=2$ and $a_{2}=3$. We prove by induction that $a_{k}=k+1$ for all $k \geqslant 0$ in this case, which we already know for $k \leqslant 2$ now. For the induction step, assume that $a_{k-1}=k$ and $a_{k}=k+1$. Then + +$$ +a_{k+1}^{2}=a_{k-1}^{2}+4 a_{k}=k^{2}+4 k+4=(k+2)^{2} +$$ + +so $a_{k+1}= \pm(k+2)$. If $a_{k+1}=-(k+2)$, then + +$$ +a_{k+2}^{2}=a_{k}^{2}+4 a_{k+1}=(k+1)^{2}-4 k-8=k^{2}-2 k-7=(k-1)^{2}-8 +$$ + +The latter can only be a square if $k=4$ (since 1 and 9 are the only two squares whose difference is 8 ). Then, however, $a_{4}=5, a_{5}=-6$ and $a_{6}= \pm 1$, so + +$$ +a_{7}^{2}=a_{5}^{2}+4 a_{6}=36 \pm 4 +$$ + +but neither 32 nor 40 is a perfect square. Thus $a_{k+1}=k+2$, which completes our induction. This also means that $f(n)=f\left(a_{n-1}\right)=a_{n}=n+1$ for all $n \geqslant 1$. + +Step 2: We prove that either $f(0)=1$, or $f(0)=0$ and $f(n) \neq 0$ for $n \neq 0$. +Set $n=0$ in (1) to get + +$$ +4 f(0)=f(f(0))^{2} +$$ + +This means that $f(0) \geqslant 0$. If $f(0)=0$, then $f(n) \neq 0$ for all $n \neq 0$, since we would otherwise have + +$$ +n^{2}=n^{2}+4 f(n)=f(f(n))^{2}=f(0)^{2}=0 +$$ + +If $f(0)>0$, then we know that $f(f(0))=f(0)+1$ from the first step, so + +$$ +4 f(0)=(f(0)+1)^{2} +$$ + +which yields $f(0)=1$. + +Step 3: We discuss the values of $f(n)$ for $n<0$. +Lemma. For every $n \geqslant 1$, we have $f(-n)=-n+1$ or $f(-n)=n+1$. Moreover, if $f(-n)=$ $-n+1$ for some $n \geqslant 1$, then also $f(-n+1)=-n+2$. +Proof. We prove this statement by strong induction on $n$. For $n=1$, we get + +$$ +1+4 f(-1)=f(f(-1))^{2} +$$ + +Thus $f(-1)$ needs to be nonnegative. If $f(-1)=0$, then $f(f(-1))=f(0)= \pm 1$, so $f(0)=1$ (by our second step). Otherwise, we know that $f(f(-1))=f(-1)+1$, so + +$$ +1+4 f(-1)=(f(-1)+1)^{2} +$$ + +which yields $f(-1)=2$ and thus establishes the base case. For the induction step, we consider two cases: + +- If $f(-n) \leqslant-n$, then + +$$ +f(f(-n))^{2}=(-n)^{2}+4 f(-n) \leqslant n^{2}-4 n<(n-2)^{2} +$$ + +so $|f(f(-n))| \leqslant n-3$ (for $n=2$, this case cannot even occur). If $f(f(-n)) \geqslant 0$, then we already know from the first two steps that $f(f(f(-n)))=f(f(-n))+1$, unless perhaps if $f(0)=0$ and $f(f(-n))=0$. However, the latter would imply $f(-n)=0$ (as shown in Step 2) and thus $n=0$, which is impossible. If $f(f(-n))<0$, we can apply the induction hypothesis to $f(f(-n))$. In either case, $f(f(f(-n)))= \pm f(f(-n))+1$. Therefore, + +$$ +f(-n)^{2}+4 f(f(-n))=f(f(f(-n)))^{2}=( \pm f(f(-n))+1)^{2} +$$ + +which gives us + +$$ +\begin{aligned} +n^{2} & \leqslant f(-n)^{2}=( \pm f(f(-n))+1)^{2}-4 f(f(-n)) \leqslant f(f(-n))^{2}+6|f(f(-n))|+1 \\ +& \leqslant(n-3)^{2}+6(n-3)+1=n^{2}-8 +\end{aligned} +$$ + +a contradiction. + +- Thus, we are left with the case that $f(-n)>-n$. Now we argue as in the previous case: if $f(-n) \geqslant 0$, then $f(f(-n))=f(-n)+1$ by the first two steps, since $f(0)=0$ and $f(-n)=0$ would imply $n=0$ (as seen in Step 2) and is thus impossible. If $f(-n)<0$, we can apply the induction hypothesis, so in any case we can infer that $f(f(-n))= \pm f(-n)+1$. We obtain + +$$ +(-n)^{2}+4 f(-n)=( \pm f(-n)+1)^{2} +$$ + +so either + +$$ +n^{2}=f(-n)^{2}-2 f(-n)+1=(f(-n)-1)^{2} +$$ + +which gives us $f(-n)= \pm n+1$, or + +$$ +n^{2}=f(-n)^{2}-6 f(-n)+1=(f(-n)-3)^{2}-8 +$$ + +Since 1 and 9 are the only perfect squares whose difference is 8 , we must have $n=1$, which we have already considered. + +Finally, suppose that $f(-n)=-n+1$ for some $n \geqslant 2$. Then + +$$ +f(-n+1)^{2}=f(f(-n))^{2}=(-n)^{2}+4 f(-n)=(n-2)^{2} +$$ + +so $f(-n+1)= \pm(n-2)$. However, we already know that $f(-n+1)=-n+2$ or $f(-n+1)=n$, so $f(-n+1)=-n+2$. + +Combining everything we know, we find the solutions as stated in the answer: + +- One solution is given by $f(n)=n+1$ for all $n$. +- If $f(n)$ is not always equal to $n+1$, then there is a largest integer $m$ (which cannot be positive) for which this is not the case. In view of the lemma that we proved, we must then have $f(n)=-n+1$ for any integer $n(|b|-4)^{2} +$$ + +because $|b| \geqslant|a|-1 \geqslant 9$. Thus (3) can be refined to + +$$ +|a|+3 \geqslant|f(a)| \geqslant|a|-1 \quad \text { for }|a| \geqslant E +$$ + +Now, from $c^{2}=a^{2}+4 b$ with $|b| \in[|a|-1,|a|+3]$ we get $c^{2}=(a \pm 2)^{2}+d$, where $d \in\{-16,-12,-8,-4,0,4,8\}$. Since $|a \pm 2| \geqslant 8$, this can happen only if $c^{2}=(a \pm 2)^{2}$, which in turn yields $b= \pm a+1$. To summarise, + +$$ +f(a)=1 \pm a \quad \text { for }|a| \geqslant E \text {. } +$$ + +We have shown that, with at most finitely many exceptions, $f(a)=1 \pm a$. Thus it will be convenient for our second step to introduce the sets + +$$ +Z_{+}=\{a \in \mathbb{Z}: f(a)=a+1\}, \quad Z_{-}=\{a \in \mathbb{Z}: f(a)=1-a\}, \quad \text { and } \quad Z_{0}=\mathbb{Z} \backslash\left(Z_{+} \cup Z_{-}\right) +$$ + +Step 2. Now we investigate the structure of the sets $Z_{+}, Z_{-}$, and $Z_{0}$. +4. Note that $f(E+1)=1 \pm(E+1)$. If $f(E+1)=E+2$, then $E+1 \in Z_{+}$. Otherwise we have $f(1+E)=-E$; then the original equation (1) with $n=E+1$ gives us $(E-1)^{2}=f(-E)^{2}$, so $f(-E)= \pm(E-1)$. By (4) this may happen only if $f(-E)=1-E$, so in this case $-E \in Z_{+}$. In any case we find that $Z_{+} \neq \varnothing$. +5. Now take any $a \in Z_{+}$. We claim that every integer $x \geqslant a$ also lies in $Z_{+}$. We proceed by induction on $x$, the base case $x=a$ being covered by our assumption. For the induction step, assume that $f(x-1)=x$ and plug $n=x-1$ into (1). We get $f(x)^{2}=(x+1)^{2}$, so either $f(x)=x+1$ or $f(x)=-(x+1)$. +Assume that $f(x)=-(x+1)$ and $x \neq-1$, since otherwise we already have $f(x)=x+1$. Plugging $n=x$ into (1), we obtain $f(-x-1)^{2}=(x-2)^{2}-8$, which may happen only if $x-2= \pm 3$ and $f(-x-1)= \pm 1$. Plugging $n=-x-1$ into (1), we get $f( \pm 1)^{2}=(x+1)^{2} \pm 4$, which in turn may happen only if $x+1 \in\{-2,0,2\}$. +Thus $x \in\{-1,5\}$ and at the same time $x \in\{-3,-1,1\}$, which gives us $x=-1$. Since this has already been excluded, we must have $f(x)=x+1$, which completes our induction. +6. Now we know that either $Z_{+}=\mathbb{Z}$ (if $Z_{+}$is not bounded below), or $Z_{+}=\left\{a \in \mathbb{Z}: a \geqslant a_{0}\right\}$, where $a_{0}$ is the smallest element of $Z_{+}$. In the former case, $f(n)=n+1$ for all $n \in \mathbb{Z}$, which is our first solution. So we assume in the following that $Z_{+}$is bounded below and has a smallest element $a_{0}$. +If $Z_{0}=\varnothing$, then we have $f(x)=x+1$ for $x \geqslant a_{0}$ and $f(x)=1-x$ for $x1$. If none of the line segments that form the borders between the rectangles is horizontal, then we have $k-1$ vertical segments dividing $R$ into $k$ rectangles. On each of them, there can only be one of the $n$ points, so $n \leqslant k-1$, which is exactly what we want to prove. + +Otherwise, consider the lowest horizontal line $h$ that contains one or more of these line segments. Let $R^{\prime}$ be the rectangle that results when everything that lies below $h$ is removed from $R$ (see the example in the figure below). + +The rectangles that lie entirely below $h$ form blocks of rectangles separated by vertical line segments. Suppose there are $r$ blocks and $k_{i}$ rectangles in the $i^{\text {th }}$ block. The left and right border of each block has to extend further upwards beyond $h$. Thus we can move any points that lie on these borders upwards, so that they now lie inside $R^{\prime}$. This can be done without violating the conditions, one only needs to make sure that they do not get to lie on a common horizontal line with one of the other given points. + +All other borders between rectangles in the $i^{\text {th }}$ block have to lie entirely below $h$. There are $k_{i}-1$ such line segments, each of which can contain at most one of the given points. Finally, there can be one point that lies on $h$. All other points have to lie in $R^{\prime}$ (after moving some of them as explained in the previous paragraph). +![](https://cdn.mathpix.com/cropped/2024_11_18_62c85fb0b01b41aca29bg-28.jpg?height=320&width=512&top_left_y=1322&top_left_x=772) + +Figure 2: Illustration of the inductive argument +We see that $R^{\prime}$ is divided into $k-\sum_{i=1}^{r} k_{i}$ rectangles. Applying the induction hypothesis to $R^{\prime}$, we find that there are at most + +$$ +\left(k-\sum_{i=1}^{r} k_{i}\right)-1+\sum_{i=1}^{r}\left(k_{i}-1\right)+1=k-r +$$ + +points. Since $r \geqslant 1$, this means that $n \leqslant k-1$, which completes our induction. + +C2. We have $2^{m}$ sheets of paper, with the number 1 written on each of them. We perform the following operation. In every step we choose two distinct sheets; if the numbers on the two sheets are $a$ and $b$, then we erase these numbers and write the number $a+b$ on both sheets. Prove that after $m 2^{m-1}$ steps, the sum of the numbers on all the sheets is at least $4^{m}$. +(Iran) +Solution. Let $P_{k}$ be the product of the numbers on the sheets after $k$ steps. +Suppose that in the $(k+1)^{\text {th }}$ step the numbers $a$ and $b$ are replaced by $a+b$. In the product, the number $a b$ is replaced by $(a+b)^{2}$, and the other factors do not change. Since $(a+b)^{2} \geqslant 4 a b$, we see that $P_{k+1} \geqslant 4 P_{k}$. Starting with $P_{0}=1$, a straightforward induction yields + +$$ +P_{k} \geqslant 4^{k} +$$ + +for all integers $k \geqslant 0$; in particular + +$$ +P_{m \cdot 2^{m-1}} \geqslant 4^{m \cdot 2^{m-1}}=\left(2^{m}\right)^{2^{m}} +$$ + +so by the AM-GM inequality, the sum of the numbers written on the sheets after $m 2^{m-1}$ steps is at least + +$$ +2^{m} \cdot \sqrt[2^{m}]{P_{m \cdot 2^{m-1}}} \geqslant 2^{m} \cdot 2^{m}=4^{m} +$$ + +Comment 1. It is possible to achieve the sum $4^{m}$ in $m 2^{m-1}$ steps. For example, starting from $2^{m}$ equal numbers on the sheets, in $2^{m-1}$ consecutive steps we can double all numbers. After $m$ such doubling rounds we have the number $2^{m}$ on every sheet. + +Comment 2. There are several versions of the solution above. E.g., one may try to assign to each positive integer $n$ a weight $w_{n}$ in such a way that the sum of the weights of the numbers written on the sheets increases, say, by at least 2 in each step. For this purpose, one needs the inequality + +$$ +2 w_{a+b} \geqslant w_{a}+w_{b}+2 +$$ + +to be satisfied for all positive integers $a$ and $b$. +Starting from $w_{1}=1$ and trying to choose the weights as small as possible, one may find that these weights can be defined as follows: For every positive integer $n$, one chooses $k$ to be the maximal integer such that $n \geqslant 2^{k}$, and puts + +$$ +w_{n}=k+\frac{n}{2^{k}}=\min _{d \in \mathbb{Z} \geq 0}\left(d+\frac{n}{2^{d}}\right) . +$$ + +Now, in order to prove that these weights satisfy (1), one may take arbitrary positive integers $a$ and $b$, and choose an integer $d \geqslant 0$ such that $w_{a+b}=d+\frac{a+b}{2^{d}}$. Then one has + +$$ +2 w_{a+b}=2 d+2 \cdot \frac{a+b}{2^{d}}=\left((d-1)+\frac{a}{2^{d-1}}\right)+\left((d-1)+\frac{b}{2^{d-1}}\right)+2 \geqslant w_{a}+w_{b}+2 +$$ + +Since the initial sum of the weights was $2^{m}$, after $m 2^{m-1}$ steps the sum is at least $(m+1) 2^{m}$. To finish the solution, one may notice that by (2) for every positive integer $a$ one has + +$$ +w_{a} \leqslant m+\frac{a}{2^{m}}, \quad \text { i.e., } \quad a \geqslant 2^{m}\left(-m+w_{a}\right) +$$ + +So the sum of the numbers $a_{1}, a_{2}, \ldots, a_{2^{m}}$ on the sheets can be estimated as + +$$ +\sum_{i=1}^{2^{m}} a_{i} \geqslant \sum_{i=1}^{2^{m}} 2^{m}\left(-m+w_{a_{i}}\right)=-m 2^{m} \cdot 2^{m}+2^{m} \sum_{i=1}^{2^{m}} w_{a_{i}} \geqslant-m 4^{m}+(m+1) 4^{m}=4^{m} +$$ + +as required. +For establishing the inequalities (1) and (3), one may also use the convexity argument, instead of the second definition of $w_{n}$ in (2). + +One may check that $\log _{2} n \leqslant w_{n} \leqslant \log _{2} n+1$; thus, in some rough sense, this approach is obtained by "taking the logarithm" of the solution above. + +Comment 3. An intuitive strategy to minimise the sum of numbers is that in every step we choose the two smallest numbers. We may call this the greedy strategy. In the following paragraphs we prove that the greedy strategy indeed provides the least possible sum of numbers. + +Claim. Starting from any sequence $x_{1}, \ldots, x_{N}$ of positive real numbers on $N$ sheets, for any number $k$ of steps, the greedy strategy achieves the lowest possible sum of numbers. + +Proof. We apply induction on $k$; for $k=1$ the statement is obvious. Let $k \geqslant 2$, and assume that the claim is true for smaller values. + +Every sequence of $k$ steps can be encoded as $S=\left(\left(i_{1}, j_{1}\right), \ldots,\left(i_{k}, j_{k}\right)\right)$, where, for $r=1,2, \ldots, k$, the numbers $i_{r}$ and $j_{r}$ are the indices of the two sheets that are chosen in the $r^{\text {th }}$ step. The resulting final sum will be some linear combination of $x_{1}, \ldots, x_{N}$, say, $c_{1} x_{1}+\cdots+c_{N} x_{N}$ with positive integers $c_{1}, \ldots, c_{N}$ that depend on $S$ only. Call the numbers $\left(c_{1}, \ldots, c_{N}\right)$ the characteristic vector of $S$. + +Choose a sequence $S_{0}=\left(\left(i_{1}, j_{1}\right), \ldots,\left(i_{k}, j_{k}\right)\right)$ of steps that produces the minimal sum, starting from $x_{1}, \ldots, x_{N}$, and let $\left(c_{1}, \ldots, c_{N}\right)$ be the characteristic vector of $S$. We may assume that the sheets are indexed in such an order that $c_{1} \geqslant c_{2} \geqslant \cdots \geqslant c_{N}$. If the sheets (and the numbers) are permuted by a permutation $\pi$ of the indices $(1,2, \ldots, N)$ and then the same steps are performed, we can obtain the $\operatorname{sum} \sum_{t=1}^{N} c_{t} x_{\pi(t)}$. By the rearrangement inequality, the smallest possible sum can be achieved when the numbers $\left(x_{1}, \ldots, x_{N}\right)$ are in non-decreasing order. So we can assume that also $x_{1} \leqslant x_{2} \leqslant \cdots \leqslant x_{N}$. + +Let $\ell$ be the largest index with $c_{1}=\cdots=c_{\ell}$, and let the $r^{\text {th }}$ step be the first step for which $c_{i_{r}}=c_{1}$ or $c_{j_{r}}=c_{1}$. The role of $i_{r}$ and $j_{r}$ is symmetrical, so we can assume $c_{i_{r}}=c_{1}$ and thus $i_{r} \leqslant \ell$. We show that $c_{j_{r}}=c_{1}$ and $j_{r} \leqslant \ell$ hold, too. + +Before the $r^{\text {th }}$ step, on the $i_{r}{ }^{\text {th }}$ sheet we had the number $x_{i_{r}}$. On the $j_{r}{ }^{\text {th }}$ sheet there was a linear combination that contains the number $x_{j_{r}}$ with a positive integer coefficient, and possibly some other terms. In the $r^{\text {th }}$ step, the number $x_{i_{r}}$ joins that linear combination. From this point, each sheet contains a linear combination of $x_{1}, \ldots, x_{N}$, with the coefficient of $x_{j_{r}}$ being not smaller than the coefficient of $x_{i_{r}}$. This is preserved to the end of the procedure, so we have $c_{j_{r}} \geqslant c_{i_{r}}$. But $c_{i_{r}}=c_{1}$ is maximal among the coefficients, so we have $c_{j_{r}}=c_{i_{r}}=c_{1}$ and thus $j_{r} \leqslant \ell$. + +Either from $c_{j_{r}}=c_{i_{r}}=c_{1}$ or from the arguments in the previous paragraph we can see that none of the $i_{r}{ }^{\text {th }}$ and the $j_{r}{ }^{\text {th }}$ sheets were used before step $r$. Therefore, the final linear combination of the numbers does not change if the step $\left(i_{r}, j_{r}\right)$ is performed first: the sequence of steps + +$$ +S_{1}=\left(\left(i_{r}, j_{r}\right),\left(i_{1}, j_{1}\right), \ldots,\left(i_{r-1}, j_{r-1}\right),\left(i_{r+1}, j_{r+1}\right), \ldots,\left(i_{N}, j_{N}\right)\right) +$$ + +also produces the same minimal sum at the end. Therefore, we can replace $S_{0}$ by $S_{1}$ and we may assume that $r=1$ and $c_{i_{1}}=c_{j_{1}}=c_{1}$. + +As $i_{1} \neq j_{1}$, we can see that $\ell \geqslant 2$ and $c_{1}=c_{2}=c_{i_{1}}=c_{j_{1}}$. Let $\pi$ be such a permutation of the indices $(1,2, \ldots, N)$ that exchanges 1,2 with $i_{r}, j_{r}$ and does not change the remaining indices. Let + +$$ +S_{2}=\left(\left(\pi\left(i_{1}\right), \pi\left(j_{1}\right)\right), \ldots,\left(\pi\left(i_{N}\right), \pi\left(j_{N}\right)\right)\right) +$$ + +Since $c_{\pi(i)}=c_{i}$ for all indices $i$, this sequence of steps produces the same, minimal sum. Moreover, in the first step we chose $x_{\pi\left(i_{1}\right)}=x_{1}$ and $x_{\pi\left(j_{1}\right)}=x_{2}$, the two smallest numbers. + +Hence, it is possible to achieve the optimal sum if we follow the greedy strategy in the first step. By the induction hypothesis, following the greedy strategy in the remaining steps we achieve the optimal sum. + +C3. Let $n \geqslant 2$ be an integer. Consider an $n \times n$ chessboard divided into $n^{2}$ unit squares. We call a configuration of $n$ rooks on this board happy if every row and every column contains exactly one rook. Find the greatest positive integer $k$ such that for every happy configuration of rooks, we can find a $k \times k$ square without a rook on any of its $k^{2}$ unit squares. +(Croatia) +Answer. $\lfloor\sqrt{n-1}\rfloor$. +Solution. Let $\ell$ be a positive integer. We will show that (i) if $n>\ell^{2}$ then each happy configuration contains an empty $\ell \times \ell$ square, but (ii) if $n \leqslant \ell^{2}$ then there exists a happy configuration not containing such a square. These two statements together yield the answer. +(i). Assume that $n>\ell^{2}$. Consider any happy configuration. There exists a row $R$ containing a rook in its leftmost square. Take $\ell$ consecutive rows with $R$ being one of them. Their union $U$ contains exactly $\ell$ rooks. Now remove the $n-\ell^{2} \geqslant 1$ leftmost columns from $U$ (thus at least one rook is also removed). The remaining part is an $\ell^{2} \times \ell$ rectangle, so it can be split into $\ell$ squares of size $\ell \times \ell$, and this part contains at most $\ell-1$ rooks. Thus one of these squares is empty. +(ii). Now we assume that $n \leqslant \ell^{2}$. Firstly, we will construct a happy configuration with no empty $\ell \times \ell$ square for the case $n=\ell^{2}$. After that we will modify it to work for smaller values of $n$. + +Let us enumerate the rows from bottom to top as well as the columns from left to right by the numbers $0,1, \ldots, \ell^{2}-1$. Every square will be denoted, as usual, by the pair $(r, c)$ of its row and column numbers. Now we put the rooks on all squares of the form $(i \ell+j, j \ell+i)$ with $i, j=0,1, \ldots, \ell-1$ (the picture below represents this arrangement for $\ell=3$ ). Since each number from 0 to $\ell^{2}-1$ has a unique representation of the form $i \ell+j(0 \leqslant i, j \leqslant \ell-1)$, each row and each column contains exactly one rook. +![](https://cdn.mathpix.com/cropped/2024_11_18_62c85fb0b01b41aca29bg-31.jpg?height=524&width=527&top_left_y=1554&top_left_x=773) + +Next, we show that each $\ell \times \ell$ square $A$ on the board contains a rook. Consider such a square $A$, and consider $\ell$ consecutive rows the union of which contains $A$. Let the lowest of these rows have number $p \ell+q$ with $0 \leqslant p, q \leqslant \ell-1$ (notice that $p \ell+q \leqslant \ell^{2}-\ell$ ). Then the rooks in this union are placed in the columns with numbers $q \ell+p,(q+1) \ell+p, \ldots,(\ell-1) \ell+p$, $p+1, \ell+(p+1), \ldots,(q-1) \ell+p+1$, or, putting these numbers in increasing order, + +$$ +p+1, \ell+(p+1), \ldots,(q-1) \ell+(p+1), q \ell+p,(q+1) \ell+p, \ldots,(\ell-1) \ell+p +$$ + +One readily checks that the first number in this list is at most $\ell-1$ (if $p=\ell-1$, then $q=0$, and the first listed number is $q \ell+p=\ell-1$ ), the last one is at least $(\ell-1) \ell$, and the difference between any two consecutive numbers is at most $\ell$. Thus, one of the $\ell$ consecutive columns intersecting $A$ contains a number listed above, and the rook in this column is inside $A$, as required. The construction for $n=\ell^{2}$ is established. + +It remains to construct a happy configuration of rooks not containing an empty $\ell \times \ell$ square for $n<\ell^{2}$. In order to achieve this, take the construction for an $\ell^{2} \times \ell^{2}$ square described above and remove the $\ell^{2}-n$ bottom rows together with the $\ell^{2}-n$ rightmost columns. We will have a rook arrangement with no empty $\ell \times \ell$ square, but several rows and columns may happen to be empty. Clearly, the number of empty rows is equal to the number of empty columns, so one can find a bijection between them, and put a rook on any crossing of an empty row and an empty column corresponding to each other. + +Comment. Part (i) allows several different proofs. E.g., in the last paragraph of the solution, it suffices to deal only with the case $n=\ell^{2}+1$. Notice now that among the four corner squares, at least one is empty. So the rooks in its row and in its column are distinct. Now, deleting this row and column we obtain an $\ell^{2} \times \ell^{2}$ square with $\ell^{2}-1$ rooks in it. This square can be partitioned into $\ell^{2}$ squares of size $\ell \times \ell$, so one of them is empty. + +C4. Construct a tetromino by attaching two $2 \times 1$ dominoes along their longer sides such that the midpoint of the longer side of one domino is a corner of the other domino. This construction yields two kinds of tetrominoes with opposite orientations. Let us call them Sand Z-tetrominoes, respectively. +![](https://cdn.mathpix.com/cropped/2024_11_18_62c85fb0b01b41aca29bg-33.jpg?height=238&width=809&top_left_y=446&top_left_x=629) + +Assume that a lattice polygon $P$ can be tiled with S-tetrominoes. Prove than no matter how we tile $P$ using only S - and Z -tetrominoes, we always use an even number of Z -tetrominoes. +(Hungary) +Solution 1. We may assume that polygon $P$ is the union of some squares of an infinite chessboard. Colour the squares of the chessboard with two colours as the figure below illustrates. +![](https://cdn.mathpix.com/cropped/2024_11_18_62c85fb0b01b41aca29bg-33.jpg?height=663&width=866&top_left_y=979&top_left_x=601) + +Observe that no matter how we tile $P$, any S-tetromino covers an even number of black squares, whereas any Z-tetromino covers an odd number of them. As $P$ can be tiled exclusively by S-tetrominoes, it contains an even number of black squares. But if some S-tetrominoes and some Z-tetrominoes cover an even number of black squares, then the number of Z-tetrominoes must be even. + +Comment. An alternative approach makes use of the following two colourings, which are perhaps somewhat more natural: +![](https://cdn.mathpix.com/cropped/2024_11_18_62c85fb0b01b41aca29bg-33.jpg?height=371&width=1297&top_left_y=2071&top_left_x=385) + +Let $s_{1}$ and $s_{2}$ be the number of $S$-tetrominoes of the first and second type (as shown in the figure above) respectively that are used in a tiling of $P$. Likewise, let $z_{1}$ and $z_{2}$ be the number of $Z$-tetrominoes of the first and second type respectively. The first colouring shows that $s_{1}+z_{2}$ is invariant modulo 2 , the second colouring shows that $s_{1}+z_{1}$ is invariant modulo 2 . Adding these two conditions, we find that $z_{1}+z_{2}$ is invariant modulo 2 , which is what we have to prove. Indeed, the sum of the two colourings (regarding white as 0 and black as 1 and adding modulo 2) is the colouring shown in the solution. + +Solution 2. Let us assign coordinates to the squares of the infinite chessboard in such a way that the squares of $P$ have nonnegative coordinates only, and that the first coordinate increases as one moves to the right, while the second coordinate increases as one moves upwards. Write the integer $3^{i} \cdot(-3)^{j}$ into the square with coordinates $(i, j)$, as in the following figure: + +| $\vdots$ | | | | | | +| :---: | :---: | :---: | :---: | :---: | :---: | +| 81 | $\vdots$ | | | | | +| -27 | -81 | $\vdots$ | | | | +| 9 | 27 | 81 | $\cdots$ | | | +| -3 | -9 | -27 | -81 | $\cdots$ | | +| 1 | 3 | 9 | 27 | 81 | $\cdots$ | + +The sum of the numbers written into four squares that can be covered by an $S$-tetromino is either of the form + +$$ +3^{i} \cdot(-3)^{j} \cdot\left(1+3+3 \cdot(-3)+3^{2} \cdot(-3)\right)=-32 \cdot 3^{i} \cdot(-3)^{j} +$$ + +(for the first type of $S$-tetrominoes), or of the form + +$$ +3^{i} \cdot(-3)^{j} \cdot\left(3+3 \cdot(-3)+(-3)+(-3)^{2}\right)=0 +$$ + +and thus divisible by 32 . For this reason, the sum of the numbers written into the squares of $P$, and thus also the sum of the numbers covered by $Z$-tetrominoes in the second covering, is likewise divisible by 32 . Now the sum of the entries of a $Z$-tetromino is either of the form + +$$ +3^{i} \cdot(-3)^{j} \cdot\left(3+3^{2}+(-3)+3 \cdot(-3)\right)=0 +$$ + +(for the first type of $Z$-tetrominoes), or of the form + +$$ +3^{i} \cdot(-3)^{j} \cdot\left(1+(-3)+3 \cdot(-3)+3 \cdot(-3)^{2}\right)=16 \cdot 3^{i} \cdot(-3)^{j} +$$ + +i.e., 16 times an odd number. Thus in order to obtain a total that is divisible by 32 , an even number of the latter kind of $Z$-tetrominoes needs to be used. Rotating everything by $90^{\circ}$, we find that the number of $Z$-tetrominoes of the first kind is even as well. So we have even proven slightly more than necessary. + +Comment 1. In the second solution, 3 and -3 can be replaced by other combinations as well. For example, for any positive integer $a \equiv 3(\bmod 4)$, we can write $a^{i} \cdot(-a)^{j}$ into the square with coordinates $(i, j)$ and apply the same argument. + +Comment 2. As the second solution shows, we even have the stronger result that the parity of the number of each of the four types of tetrominoes in a tiling of $P$ by S - and Z-tetrominoes is an invariant of $P$. This also remains true if there is no tiling of $P$ that uses only S-tetrominoes. + +C5. Consider $n \geqslant 3$ lines in the plane such that no two lines are parallel and no three have a common point. These lines divide the plane into polygonal regions; let $\mathcal{F}$ be the set of regions having finite area. Prove that it is possible to colour $\lceil\sqrt{n / 2}\rceil$ of the lines blue in such a way that no region in $\mathcal{F}$ has a completely blue boundary. (For a real number $x,\lceil x\rceil$ denotes the least integer which is not smaller than $x$.) +(Austria) +Solution. Let $L$ be the given set of lines. Choose a maximal (by inclusion) subset $B \subseteq L$ such that when we colour the lines of $B$ blue, no region in $\mathcal{F}$ has a completely blue boundary. Let $|B|=k$. We claim that $k \geqslant\lceil\sqrt{n / 2}\rceil$. + +Let us colour all the lines of $L \backslash B$ red. Call a point blue if it is the intersection of two blue lines. Then there are $\binom{k}{2}$ blue points. + +Now consider any red line $\ell$. By the maximality of $B$, there exists at least one region $A \in \mathcal{F}$ whose only red side lies on $\ell$. Since $A$ has at least three sides, it must have at least one blue vertex. Let us take one such vertex and associate it to $\ell$. + +Since each blue point belongs to four regions (some of which may be unbounded), it is associated to at most four red lines. Thus the total number of red lines is at most $4\binom{k}{2}$. On the other hand, this number is $n-k$, so + +$$ +n-k \leqslant 2 k(k-1), \quad \text { thus } \quad n \leqslant 2 k^{2}-k \leqslant 2 k^{2} +$$ + +and finally $k \geqslant\lceil\sqrt{n / 2}\rceil$, which gives the desired result. +Comment 1. The constant factor in the estimate can be improved in different ways; we sketch two of them below. On the other hand, the Problem Selection Committee is not aware of any results showing that it is sometimes impossible to colour $k$ lines satisfying the desired condition for $k \gg \sqrt{n}$. In this situation we find it more suitable to keep the original formulation of the problem. + +1. Firstly, we show that in the proof above one has in fact $k=|B| \geqslant\lceil\sqrt{2 n / 3}\rceil$. + +Let us make weighted associations as follows. Let a region $A$ whose only red side lies on $\ell$ have $k$ vertices, so that $k-2$ of them are blue. We associate each of these blue vertices to $\ell$, and put the weight $\frac{1}{k-2}$ on each such association. So the sum of the weights of all the associations is exactly $n-k$. + +Now, one may check that among the four regions adjacent to a blue vertex $v$, at most two are triangles. This means that the sum of the weights of all associations involving $v$ is at most $1+1+\frac{1}{2}+\frac{1}{2}=3$. This leads to the estimate + +$$ +n-k \leqslant 3\binom{k}{2} +$$ + +or + +$$ +2 n \leqslant 3 k^{2}-k<3 k^{2} +$$ + +which yields $k \geqslant\lceil\sqrt{2 n / 3}\rceil$. +2. Next, we even show that $k=|B| \geqslant\lceil\sqrt{n}\rceil$. For this, we specify the process of associating points to red lines in one more different way. + +Call a point red if it lies on a red line as well as on a blue line. Consider any red line $\ell$, and take an arbitrary region $A \in \mathcal{F}$ whose only red side lies on $\ell$. Let $r^{\prime}, r, b_{1}, \ldots, b_{k}$ be its vertices in clockwise order with $r^{\prime}, r \in \ell$; then the points $r^{\prime}, r$ are red, while all the points $b_{1}, \ldots, b_{k}$ are blue. Let us associate to $\ell$ the red point $r$ and the blue point $b_{1}$. One may notice that to each pair of a red point $r$ and a blue point $b$, at most one red line can be associated, since there is at most one region $A$ having $r$ and $b$ as two clockwise consecutive vertices. + +We claim now that at most two red lines are associated to each blue point $b$; this leads to the desired bound + +$$ +n-k \leqslant 2\binom{k}{2} \quad \Longleftrightarrow \quad n \leqslant k^{2} +$$ + +Assume, to the contrary, that three red lines $\ell_{1}, \ell_{2}$, and $\ell_{3}$ are associated to the same blue point $b$. Let $r_{1}, r_{2}$, and $r_{3}$ respectively be the red points associated to these lines; all these points are distinct. The point $b$ defines four blue rays, and each point $r_{i}$ is the red point closest to $b$ on one of these rays. So we may assume that the points $r_{2}$ and $r_{3}$ lie on one blue line passing through $b$, while $r_{1}$ lies on the other one. +![](https://cdn.mathpix.com/cropped/2024_11_18_62c85fb0b01b41aca29bg-36.jpg?height=432&width=532&top_left_y=475&top_left_x=768) + +Now consider the region $A$ used to associate $r_{1}$ and $b$ with $\ell_{1}$. Three of its clockwise consecutive vertices are $r_{1}, b$, and either $r_{2}$ or $r_{3}$ (say, $r_{2}$ ). Since $A$ has only one red side, it can only be the triangle $r_{1} b r_{2}$; but then both $\ell_{1}$ and $\ell_{2}$ pass through $r_{2}$, as well as some blue line. This is impossible by the problem assumptions. + +Comment 2. The condition that the lines be non-parallel is essentially not used in the solution, nor in the previous comment; thus it may be omitted. + +C6. We are given an infinite deck of cards, each with a real number on it. For every real number $x$, there is exactly one card in the deck that has $x$ written on it. Now two players draw disjoint sets $A$ and $B$ of 100 cards each from this deck. We would like to define a rule that declares one of them a winner. This rule should satisfy the following conditions: + +1. The winner only depends on the relative order of the 200 cards: if the cards are laid down in increasing order face down and we are told which card belongs to which player, but not what numbers are written on them, we can still decide the winner. +2. If we write the elements of both sets in increasing order as $A=\left\{a_{1}, a_{2}, \ldots, a_{100}\right\}$ and $B=\left\{b_{1}, b_{2}, \ldots, b_{100}\right\}$, and $a_{i}>b_{i}$ for all $i$, then $A$ beats $B$. +3. If three players draw three disjoint sets $A, B, C$ from the deck, $A$ beats $B$ and $B$ beats $C$, then $A$ also beats $C$. + +How many ways are there to define such a rule? Here, we consider two rules as different if there exist two sets $A$ and $B$ such that $A$ beats $B$ according to one rule, but $B$ beats $A$ according to the other. +(Russia) +Answer. 100. +Solution 1. We prove a more general statement for sets of cardinality $n$ (the problem being the special case $n=100$, then the answer is $n$ ). In the following, we write $A>B$ or $Bb_{k}$. This rule clearly satisfies all three conditions, and the rules corresponding to different $k$ are all different. Thus there are at least $n$ different rules. + +Part II. Now we have to prove that there is no other way to define such a rule. Suppose that our rule satisfies the conditions, and let $k \in\{1,2, \ldots, n\}$ be minimal with the property that + +$$ +A_{k}=\{1,2, \ldots, k, n+k+1, n+k+2, \ldots, 2 n\} \prec B_{k}=\{k+1, k+2, \ldots, n+k\} +$$ + +Clearly, such a $k$ exists, since this holds for $k=n$ by assumption. Now consider two disjoint sets $X=\left\{x_{1}, x_{2}, \ldots, x_{n}\right\}$ and $Y=\left\{y_{1}, y_{2}, \ldots, y_{n}\right\}$, both in increasing order (i.e., $x_{1}B_{k-1}$ by our choice of $k$, we also have $U>W$ (if $k=1$, this is trivial). +- The elements of $V \cup W$ are ordered in the same way as those of $A_{k} \cup B_{k}$, and since $A_{k} \prec B_{k}$ by our choice of $k$, we also have $V \prec W$. + +It follows that + +$$ +X \prec V \prec W \prec U \prec Y +$$ + +so $X(\{1\} \cup\{3 i-1 \mid 2 \leqslant i \leqslant n-1\} \cup\{3 n\}) . +$$ + +Likewise, if the second relation does not hold, then we must also have + +$$ +(\{1\} \cup\{3 i-1 \mid 2 \leqslant i \leqslant n-1\} \cup\{3 n\})>(\{3\} \cup\{3 i \mid 2 \leqslant i \leqslant n-1\} \cup\{3 n-1\}) +$$ + +Now condition 3 implies that + +$$ +(\{2\} \cup\{3 i-2 \mid 2 \leqslant i \leqslant n-1\} \cup\{3 n-2\})>(\{3\} \cup\{3 i \mid 2 \leqslant i \leqslant n-1\} \cup\{3 n-1\}) +$$ + +which contradicts the second condition. +Now we distinguish two cases, depending on which of the two relations actually holds: +First case: $(\{2\} \cup\{2 i-1 \mid 2 \leqslant i \leqslant n\})<(\{1\} \cup\{2 i \mid 2 \leqslant i \leqslant n\})$. +Let $A=\left\{a_{1}, a_{2}, \ldots, a_{n}\right\}$ and $B=\left\{b_{1}, b_{2}, \ldots, b_{n}\right\}$ be two disjoint sets, both in increasing order. We claim that the winner can be decided only from the values of $a_{2}, \ldots, a_{n}$ and $b_{2}, \ldots, b_{n}$, while $a_{1}$ and $b_{1}$ are actually irrelevant. Suppose that this was not the case, and assume without loss of generality that $a_{2}0$ be smaller than half the distance between any two of the numbers in $B_{x} \cup B_{y} \cup A$. For any set $M$, let $M \pm \varepsilon$ be the set obtained by adding/subtracting $\varepsilon$ to all elements of $M$. By our choice of $\varepsilon$, the relative order of the elements of $\left(B_{y}+\varepsilon\right) \cup A$ is still the same as for $B_{y} \cup A$, while the relative order of the elements of $\left(B_{x}-\varepsilon\right) \cup A$ is still the same as for $B_{x} \cup A$. Thus $A \prec B_{x}-\varepsilon$, but $A>B_{y}+\varepsilon$. Moreover, if $y>x$, then $B_{x}-\varepsilon \prec B_{y}+\varepsilon$ by condition 2, while otherwise the relative order of +the elements in $\left(B_{x}-\varepsilon\right) \cup\left(B_{y}+\varepsilon\right)$ is the same as for the two sets $\{2\} \cup\{2 i-1 \mid 2 \leqslant i \leqslant n\}$ and $\{1\} \cup\{2 i \mid 2 \leqslant i \leqslant n\}$, so that $B_{x}-\varepsilon_{\sigma} B$ if and only if $\left(a_{\sigma(1)}, \ldots, a_{\sigma(n)}\right)$ is lexicographically greater than $\left(b_{\sigma(1)}, \ldots, b_{\sigma(n)}\right)$. + +It seems, however, that this formulation adds rather more technicalities to the problem than additional ideas. + +This page is intentionally left blank + +C7. Let $M$ be a set of $n \geqslant 4$ points in the plane, no three of which are collinear. Initially these points are connected with $n$ segments so that each point in $M$ is the endpoint of exactly two segments. Then, at each step, one may choose two segments $A B$ and $C D$ sharing a common interior point and replace them by the segments $A C$ and $B D$ if none of them is present at this moment. Prove that it is impossible to perform $n^{3} / 4$ or more such moves. +(Russia) +Solution. A line is said to be red if it contains two points of $M$. As no three points of $M$ are collinear, each red line determines a unique pair of points of $M$. Moreover, there are precisely $\binom{n}{2}<\frac{n^{2}}{2}$ red lines. By the value of a segment we mean the number of red lines intersecting it in its interior, and the value of a set of segments is defined to be the sum of the values of its elements. We will prove that $(i)$ the value of the initial set of segments is smaller than $n^{3} / 2$ and that (ii) each step decreases the value of the set of segments present by at least 2 . Since such a value can never be negative, these two assertions imply the statement of the problem. + +To show $(i)$ we just need to observe that each segment has a value that is smaller than $n^{2} / 2$. Thus the combined value of the $n$ initial segments is indeed below $n \cdot n^{2} / 2=n^{3} / 2$. + +It remains to establish (ii). Suppose that at some moment we have two segments $A B$ and $C D$ sharing an interior point $S$, and that at the next moment we have the two segments $A C$ and $B D$ instead. Let $X_{A B}$ denote the set of red lines intersecting the segment $A B$ in its interior and let the sets $X_{A C}, X_{B D}$, and $X_{C D}$ be defined similarly. We are to prove that $\left|X_{A C}\right|+\left|X_{B D}\right|+2 \leqslant\left|X_{A B}\right|+\left|X_{C D}\right|$. + +As a first step in this direction, we claim that + +$$ +\left|X_{A C} \cup X_{B D}\right|+2 \leqslant\left|X_{A B} \cup X_{C D}\right| . +$$ + +Indeed, if $g$ is a red line intersecting, e.g. the segment $A C$ in its interior, then it has to intersect the triangle $A C S$ once again, either in the interior of its side $A S$, or in the interior of its side $C S$, or at $S$, meaning that it belongs to $X_{A B}$ or to $X_{C D}$ (see Figure 1). Moreover, the red lines $A B$ and $C D$ contribute to $X_{A B} \cup X_{C D}$ but not to $X_{A C} \cup X_{B D}$. Thereby (1) is proved. +![](https://cdn.mathpix.com/cropped/2024_11_18_62c85fb0b01b41aca29bg-41.jpg?height=404&width=390&top_left_y=1780&top_left_x=339) + +Figure 1 +![](https://cdn.mathpix.com/cropped/2024_11_18_62c85fb0b01b41aca29bg-41.jpg?height=452&width=360&top_left_y=1733&top_left_x=865) + +Figure 2 +![](https://cdn.mathpix.com/cropped/2024_11_18_62c85fb0b01b41aca29bg-41.jpg?height=448&width=366&top_left_y=1729&top_left_x=1356) + +Figure 3 + +Similarly but more easily one obtains + +$$ +\left|X_{A C} \cap X_{B D}\right| \leqslant\left|X_{A B} \cap X_{C D}\right| +$$ + +Indeed, a red line $h$ appearing in $X_{A C} \cap X_{B D}$ belongs, for similar reasons as above, also to $X_{A B} \cap X_{C D}$. To make the argument precise, one may just distinguish the cases $S \in h$ (see Figure 2) and $S \notin h$ (see Figure 3). Thereby (2) is proved. + +Adding (1) and (2) we obtain the desired conclusion, thus completing the solution of this problem. + +Comment 1. There is a problem belonging to the folklore, in the solution of which one may use the same kind of operation: + +Given $n$ red and $n$ green points in the plane, prove that one may draw $n$ nonintersecting segments each of which connects a red point with a green point. + +A standard approach to this problem consists in taking $n$ arbitrary segments connecting the red points with the green points, and to perform the same operation as in the above proposal whenever an intersection occurs. Now each time one performs such a step, the total length of the segments that are present decreases due to the triangle inequality. So, as there are only finitely many possibilities for the set of segments present, the process must end at some stage. + +In the above proposal, however, considering the sum of the Euclidean lengths of the segment that are present does not seem to help much, for even though it shows that the process must necessarily terminate after some finite number of steps, it does not seem to easily yield any upper bound on the number of these steps that grows polynomially with $n$. + +One may regard the concept of the value of a segment introduced in the above solution as an appropriately discretised version of Euclidean length suitable for obtaining such a bound. + +The Problem Selection Committee still believes the problem to be sufficiently original for the competition. + +Comment 2. There are some other essentially equivalent ways of presenting the same solution. E.g., put $M=\left\{A_{1}, A_{2}, \ldots, A_{n}\right\}$, denote the set of segments present at any moment by $\left\{e_{1}, e_{2}, \ldots, e_{n}\right\}$, and called a triple $(i, j, k)$ of indices with $i \neq j$ intersecting, if the line $A_{i} A_{j}$ intersects the segment $e_{k}$. It may then be shown that the number $S$ of intersecting triples satisfies $0 \leqslant S0$ ). It would be interesting to say more about the gap between $c n^{2}$ and $c n^{3}$. + +C8. A card deck consists of 1024 cards. On each card, a set of distinct decimal digits is written in such a way that no two of these sets coincide (thus, one of the cards is empty). Two players alternately take cards from the deck, one card per turn. After the deck is empty, each player checks if he can throw out one of his cards so that each of the ten digits occurs on an even number of his remaining cards. If one player can do this but the other one cannot, the one who can is the winner; otherwise a draw is declared. + +Determine all possible first moves of the first player after which he has a winning strategy. +(Russia) +Answer. All the moves except for taking the empty card. + +Solution. Let us identify each card with the set of digits written on it. For any collection of cards $C_{1}, C_{2}, \ldots, C_{k}$ denote by their sum the set $C_{1} \triangle C_{2} \triangle \cdots \triangle C_{k}$ consisting of all elements belonging to an odd number of the $C_{i}$ 's. Denote the first and the second player by $\mathcal{F}$ and $\mathcal{S}$, respectively. + +Since each digit is written on exactly 512 cards, the sum of all the cards is $\varnothing$. Therefore, at the end of the game the sum of all the cards of $\mathcal{F}$ will be the same as that of $\mathcal{S}$; denote this sum by $C$. Then the player who took $C$ can throw it out and get the desired situation, while the other one cannot. Thus, the player getting card $C$ wins, and no draw is possible. + +Now, given a nonempty card $B$, one can easily see that all the cards can be split into 512 pairs of the form $(X, X \triangle B)$ because $(X \triangle B) \triangle B=X$. The following lemma shows a property of such a partition that is important for the solution. +Lemma. Let $B \neq \varnothing$ be some card. Let us choose 512 cards so that exactly one card is chosen from every pair $(X, X \triangle B)$. Then the sum of all chosen cards is either $\varnothing$ or $B$. +Proof. Let $b$ be some element of $B$. Enumerate the pairs; let $X_{i}$ be the card not containing $b$ in the $i^{\text {th }}$ pair, and let $Y_{i}$ be the other card in this pair. Then the sets $X_{i}$ are exactly all the sets not containing $b$, therefore each digit $a \neq b$ is written on exactly 256 of these cards, so $X_{1} \triangle X_{2} \triangle \cdots \triangle X_{512}=\varnothing$. Now, if we replace some summands in this sum by the other elements from their pairs, we will simply add $B$ several times to this sum, thus the sum will either remain unchanged or change by $B$, as required. + +Now we consider two cases. +Case 1. Assume that $\mathcal{F}$ takes the card $\varnothing$ on his first move. In this case, we present a winning strategy for $\mathcal{S}$. + +Let $\mathcal{S}$ take an arbitrary card $A$. Assume that $\mathcal{F}$ takes card $B$ after that; then $\mathcal{S}$ takes $A \triangle B$. Split all 1024 cards into 512 pairs of the form $(X, X \triangle B)$; we call two cards in one pair partners. Then the four cards taken so far form two pairs $(\varnothing, B)$ and $(A, A \triangle B)$ belonging to $\mathcal{F}$ and $\mathcal{S}$, respectively. On each of the subsequent moves, when $\mathcal{F}$ takes some card, $\mathcal{S}$ should take the partner of this card in response. + +Consider the situation at the end of the game. Let us for a moment replace card $A$ belonging to $\mathcal{S}$ by $\varnothing$. Then he would have one card from each pair; by our lemma, the sum of all these cards would be either $\varnothing$ or $B$. Now, replacing $\varnothing$ back by $A$ we get that the actual sum of the cards of $\mathcal{S}$ is either $A$ or $A \triangle B$, and he has both these cards. Thus $\mathcal{S}$ wins. + +Case 2. Now assume that $\mathcal{F}$ takes some card $A \neq \varnothing$ on his first move. Let us present a winning strategy for $\mathcal{F}$ in this case. + +Assume that $\mathcal{S}$ takes some card $B \neq \varnothing$ on his first move; then $\mathcal{F}$ takes $A \triangle B$. Again, let us split all the cards into pairs of the form $(X, X \triangle B)$; then the cards which have not been taken yet form several complete pairs and one extra element (card $\varnothing$ has not been taken while its partner $B$ has). Now, on each of the subsequent moves, if $\mathcal{S}$ takes some element from a +complete pair, then $\mathcal{F}$ takes its partner. If $\mathcal{S}$ takes the extra element, then $\mathcal{F}$ takes an arbitrary card $Y$, and the partner of $Y$ becomes the new extra element. + +Thus, on his last move $\mathcal{S}$ is forced to take the extra element. After that player $\mathcal{F}$ has cards $A$ and $A \triangle B$, player $\mathcal{S}$ has cards $B$ and $\varnothing$, and $\mathcal{F}$ has exactly one element from every other pair. Thus the situation is the same as in the previous case with roles reversed, and $\mathcal{F}$ wins. + +Finally, if $\mathcal{S}$ takes $\varnothing$ on his first move then $\mathcal{F}$ denotes any card which has not been taken yet by $B$ and takes $A \triangle B$. After that, the same strategy as above is applicable. + +Comment 1. If one wants to avoid the unusual question about the first move, one may change the formulation as follows. (The difficulty of the problem would decrease somewhat.) + +A card deck consists of 1023 cards; on each card, a nonempty set of distinct decimal digits is written in such a way that no two of these sets coincide. Two players alternately take cards from the deck, one card per turn. When the deck is empty, each player checks if he can throw out one of his cards so that for each of the ten digits, he still holds an even number of cards with this digit. If one player can do this but the other one cannot, the one who can is the winner; otherwise a draw is declared. + +Determine which of the players (if any) has a winning strategy. +The winner in this version is the first player. The analysis of the game from the first two paragraphs of the previous solution applies to this version as well, except for the case $C=\varnothing$ in which the result is a draw. Then the strategy for $\mathcal{S}$ in Case 1 works for $\mathcal{F}$ in this version: the sum of all his cards at the end is either $A$ or $A \triangle B$, thus nonempty in both cases. + +Comment 2. Notice that all the cards form a vector space over $\mathbb{F}_{2}$, with $\triangle$ the operation of addition. Due to the automorphisms of this space, all possibilities for $\mathcal{F}$ 's first move except $\varnothing$ are equivalent. The same holds for the response by $\mathcal{S}$ if $\mathcal{F}$ takes the card $\varnothing$ on his first move. + +Comment 3. It is not that hard to show that in the initial game, $\mathcal{F}$ has a winning move, by the idea of "strategy stealing". + +Namely, assume that $\mathcal{S}$ has a winning strategy. Let us take two card decks and start two games, in which $\mathcal{S}$ will act by his strategy. In the first game, $\mathcal{F}$ takes an arbitrary card $A_{1}$; assume that $\mathcal{S}$ takes some $B_{1}$ in response. Then $\mathcal{F}$ takes the card $B_{1}$ at the second game; let the response by $\mathcal{S}$ be $A_{2}$. Then $\mathcal{F}$ takes $A_{2}$ in the first game and gets a response $B_{2}$, and so on. + +This process stops at some moment when in the second game $\mathcal{S}$ takes $A_{i}=A_{1}$. At this moment the players hold the same sets of cards in both games, but with roles reversed. Now, if some cards remain in the decks, $\mathcal{F}$ takes an arbitrary card from the first deck starting a similar cycle. + +At the end of the game, player $\mathcal{F}$ 's cards in the first game are exactly player $\mathcal{S}$ 's cards in the second game, and vice versa. Thus in one of the games $\mathcal{F}$ will win, which is impossible by our assumption. + +One may notice that the strategy in Case 2 is constructed exactly in this way from the strategy in Case 1 . This is possible since every response by $\mathcal{S}$ wins if $\mathcal{F}$ takes the card $\varnothing$ on his first move. + +C9. There are $n$ circles drawn on a piece of paper in such a way that any two circles intersect in two points, and no three circles pass through the same point. Turbo the snail slides along the circles in the following fashion. Initially he moves on one of the circles in clockwise direction. Turbo always keeps sliding along the current circle until he reaches an intersection with another circle. Then he continues his journey on this new circle and also changes the direction of moving, i.e. from clockwise to anticlockwise or vice versa. + +Suppose that Turbo's path entirely covers all circles. Prove that $n$ must be odd. +(India) +Solution 1. Replace every cross (i.e. intersection of two circles) by two small circle arcs that indicate the direction in which the snail should leave the cross (see Figure 1.1). Notice that the placement of the small arcs does not depend on the direction of moving on the curves; no matter which direction the snail is moving on the circle arcs, he will follow the same curves (see Figure 1.2). In this way we have a set of curves, that are the possible paths of the snail. Call these curves snail orbits or just orbits. Every snail orbit is a simple closed curve that has no intersection with any other orbit. +![](https://cdn.mathpix.com/cropped/2024_11_18_62c85fb0b01b41aca29bg-45.jpg?height=355&width=375&top_left_y=1025&top_left_x=406) + +Figure 1.1 +![](https://cdn.mathpix.com/cropped/2024_11_18_62c85fb0b01b41aca29bg-45.jpg?height=347&width=670&top_left_y=1020&top_left_x=930) + +Figure 1.2 + +We prove the following, more general statement. +(*) In any configuration of $n$ circles such that no two of them are tangent, the number of snail orbits has the same parity as the number $n$. (Note that it is not assumed that all circle pairs intersect.) + +This immediately solves the problem. +Let us introduce the following operation that will be called flipping a cross. At a cross, remove the two small arcs of the orbits, and replace them by the other two arcs. Hence, when the snail arrives at a flipped cross, he will continue on the other circle as before, but he will preserve the orientation in which he goes along the circle arcs (see Figure 2). +![](https://cdn.mathpix.com/cropped/2024_11_18_62c85fb0b01b41aca29bg-45.jpg?height=292&width=735&top_left_y=1964&top_left_x=663) + +Figure 2 +Consider what happens to the number of orbits when a cross is flipped. Denote by $a, b, c$, and $d$ the four arcs that meet at the cross such that $a$ and $b$ belong to the same circle. Before the flipping $a$ and $b$ were connected to $c$ and $d$, respectively, and after the flipping $a$ and $b$ are connected to $d$ and $c$, respectively. + +The orbits passing through the cross are closed curves, so each of the $\operatorname{arcs} a, b, c$, and $d$ is connected to another one by orbits outside the cross. We distinguish three cases. + +Case 1: $a$ is connected to $b$ and $c$ is connected to $d$ by the orbits outside the cross (see Figure 3.1). + +We show that this case is impossible. Remove the two small arcs at the cross, connect $a$ to $b$, and connect $c$ to $d$ at the cross. Let $\gamma$ be the new closed curve containing $a$ and $b$, and let $\delta$ be the new curve that connects $c$ and $d$. These two curves intersect at the cross. So one of $c$ and $d$ is inside $\gamma$ and the other one is outside $\gamma$. Then the two closed curves have to meet at least one more time, but this is a contradiction, since no orbit can intersect itself. +![](https://cdn.mathpix.com/cropped/2024_11_18_62c85fb0b01b41aca29bg-46.jpg?height=247&width=333&top_left_y=499&top_left_x=239) + +Figure 3.1 +![](https://cdn.mathpix.com/cropped/2024_11_18_62c85fb0b01b41aca29bg-46.jpg?height=232&width=552&top_left_y=501&top_left_x=632) + +Figure 3.2 +![](https://cdn.mathpix.com/cropped/2024_11_18_62c85fb0b01b41aca29bg-46.jpg?height=235&width=575&top_left_y=496&top_left_x=1249) + +Figure 3.3 + +Case 2: $a$ is connected to $c$ and $b$ is connected to $d$ (see Figure 3.2). +Before the flipping $a$ and $c$ belong to one orbit and $b$ and $d$ belong to another orbit. Flipping the cross merges the two orbits into a single orbit. Hence, the number of orbits decreases by 1. + +Case 3: $a$ is connected to $d$ and $b$ is connected to $c$ (see Figure 3.3). +Before the flipping the arcs $a, b, c$, and $d$ belong to a single orbit. Flipping the cross splits that orbit in two. The number of orbits increases by 1. + +As can be seen, every flipping decreases or increases the number of orbits by one, thus changes its parity. + +Now flip every cross, one by one. Since every pair of circles has 0 or 2 intersections, the number of crosses is even. Therefore, when all crosses have been flipped, the original parity of the number of orbits is restored. So it is sufficient to prove (*) for the new configuration, where all crosses are flipped. Of course also in this new configuration the (modified) orbits are simple closed curves not intersecting each other. + +Orient the orbits in such a way that the snail always moves anticlockwise along the circle arcs. Figure 4 shows the same circles as in Figure 1 after flipping all crosses and adding orientation. (Note that this orientation may be different from the orientation of the orbit as a planar curve; the orientation of every orbit may be negative as well as positive, like the middle orbit in Figure 4.) If the snail moves around an orbit, the total angle change in his moving direction, the total curvature, is either $+2 \pi$ or $-2 \pi$, depending on the orientation of the orbit. Let $P$ and $N$ be the number of orbits with positive and negative orientation, respectively. Then the total curvature of all orbits is $(P-N) \cdot 2 \pi$. +![](https://cdn.mathpix.com/cropped/2024_11_18_62c85fb0b01b41aca29bg-46.jpg?height=372&width=412&top_left_y=1961&top_left_x=499) + +Figure 4 +![](https://cdn.mathpix.com/cropped/2024_11_18_62c85fb0b01b41aca29bg-46.jpg?height=213&width=500&top_left_y=2115&top_left_x=1052) + +Figure 5 + +Double-count the total curvature of all orbits. Along every circle the total curvature is $2 \pi$. At every cross, the two turnings make two changes with some angles having the same absolute value but opposite signs, as depicted in Figure 5. So the changes in the direction at the crosses cancel out. Hence, the total curvature is $n \cdot 2 \pi$. + +Now we have $(P-N) \cdot 2 \pi=n \cdot 2 \pi$, so $P-N=n$. The number of (modified) orbits is $P+N$, that has a same parity as $P-N=n$. + +Solution 2. We present a different proof of (*). +We perform a sequence of small modification steps on the configuration of the circles in such a way that at the end they have no intersection at all (see Figure 6.1). We use two kinds of local changes to the structure of the orbits (see Figure 6.2): + +- Type-1 step: An arc of a circle is moved over an arc of another circle; such a step creates or removes two intersections. +- Type-2 step: An arc of a circle is moved through the intersection of two other circles. +![](https://cdn.mathpix.com/cropped/2024_11_18_62c85fb0b01b41aca29bg-47.jpg?height=313&width=701&top_left_y=803&top_left_x=295) + +Figure 6.1 +![](https://cdn.mathpix.com/cropped/2024_11_18_62c85fb0b01b41aca29bg-47.jpg?height=292&width=189&top_left_y=819&top_left_x=1139) + +Figure 6.2 + +We assume that in every step only one circle is moved, and that this circle is moved over at most one arc or intersection point of other circles. + +We will show that the parity of the number of orbits does not change in any step. As every circle becomes a separate orbit at the end of the procedure, this fact proves (*). + +Consider what happens to the number of orbits when a Type-1 step is performed. The two intersection points are created or removed in a small neighbourhood. Denote some points of the two circles where they enter or leave this neighbourhood by $a, b, c$, and $d$ in this order around the neighbourhood; let $a$ and $b$ belong to one circle and let $c$ and $d$ belong to the other circle. The two circle arcs may have the same or opposite orientations. Moreover, the four end-points of the two arcs are connected by the other parts of the orbits. This can happen in two ways without intersection: either $a$ is connected to $d$ and $b$ is connected to $c$, or $a$ is connected to $b$ and $c$ is connected to $d$. Altogether we have four cases, as shown in Figure 7. +![](https://cdn.mathpix.com/cropped/2024_11_18_62c85fb0b01b41aca29bg-47.jpg?height=276&width=1621&top_left_y=1873&top_left_x=221) + +Figure 7 +We can see that the number of orbits is changed by -2 or +2 in the leftmost case when the arcs have the same orientation, $a$ is connected to $d$, and $b$ is connected to $c$. In the other three cases the number of orbits is not changed. Hence, Type-1 steps do not change the parity of the number of orbits. + +Now consider a Type-2 step. The three circles enclose a small, triangular region; by the step, this triangle is replaced by another triangle. Again, the modification of the orbits is done in some small neighbourhood; the structure does not change outside. Each side of the triangle shaped region can be convex or concave; the number of concave sides can be $0,1,2$ or 3 , so there are 4 possible arrangements of the orbits inside the neighbourhood, as shown in Figure 8. +![](https://cdn.mathpix.com/cropped/2024_11_18_62c85fb0b01b41aca29bg-48.jpg?height=312&width=301&top_left_y=221&top_left_x=295) +all convex +![](https://cdn.mathpix.com/cropped/2024_11_18_62c85fb0b01b41aca29bg-48.jpg?height=345&width=1092&top_left_y=219&top_left_x=683) + +Figure 8 +Denote the points where the three circles enter or leave the neighbourhood by $a, b, c, d$, $e$, and $f$ in this order around the neighbourhood. As can be seen in Figure 8, there are only two essentially different cases; either $a, c, e$ are connected to $b, d, f$, respectively, or $a, c, e$ are connected to $f, b, d$, respectively. The step either preserves the set of connections or switches to the other arrangement. Obviously, in the earlier case the number of orbits is not changed; therefore we have to consider only the latter case. + +The points $a, b, c, d, e$, and $f$ are connected by the orbits outside, without intersection. If $a$ was connected to $c$, say, then this orbit would isolate $b$, so this is impossible. Hence, each of $a, b, c, d, e$ and $f$ must be connected either to one of its neighbours or to the opposite point. If say $a$ is connected to $d$, then this orbit separates $b$ and $c$ from $e$ and $f$, therefore $b$ must be connected to $c$ and $e$ must be connected to $f$. Altogether there are only two cases and their reverses: either each point is connected to one of its neighbours or two opposite points are connected and the the remaining neigh boring pairs are connected to each other. See Figure 9. +![](https://cdn.mathpix.com/cropped/2024_11_18_62c85fb0b01b41aca29bg-48.jpg?height=295&width=1628&top_left_y=1299&top_left_x=216) + +Figure 9 +We can see that if only neighbouring points are connected, then the number of orbits is changed by +2 or -2 . If two opposite points are connected ( $a$ and $d$ in the figure), then the orbits are re-arranged, but their number is unchanged. Hence, Type-2 steps also preserve the parity. This completes the proof of (*). + +Solution 3. Like in the previous solutions, we do not need all circle pairs to intersect but we assume that the circles form a connected set. Denote by $\mathcal{C}$ and $\mathcal{P}$ the sets of circles and their intersection points, respectively. + +The circles divide the plane into several simply connected, bounded regions and one unbounded region. Denote the set of these regions by $\mathcal{R}$. We say that an intersection point or a region is odd or even if it is contained inside an odd or even number of circles, respectively. Let $\mathcal{P}_{\text {odd }}$ and $\mathcal{R}_{\text {odd }}$ be the sets of odd intersection points and odd regions, respectively. + +Claim. + +$$ +\left|\mathcal{R}_{\text {odd }}\right|-\left|\mathcal{P}_{\text {odd }}\right| \equiv n \quad(\bmod 2) . +$$ + +Proof. For each circle $c \in \mathcal{C}$, denote by $R_{c}, P_{c}$, and $X_{c}$ the number of regions inside $c$, the number of intersection points inside $c$, and the number of circles intersecting $c$, respectively. The circles divide each other into several arcs; denote by $A_{c}$ the number of such arcs inside $c$. By double counting the regions and intersection points inside the circles we get + +$$ +\left|\mathcal{R}_{\mathrm{odd}}\right| \equiv \sum_{c \in \mathcal{C}} R_{c} \quad(\bmod 2) \quad \text { and } \quad\left|\mathcal{P}_{\mathrm{odd}}\right| \equiv \sum_{c \in \mathcal{C}} P_{c} \quad(\bmod 2) . +$$ + +For each circle $c$, apply EULER's polyhedron theorem to the (simply connected) regions in $c$. There are $2 X_{c}$ intersection points on $c$; they divide the circle into $2 X_{c}$ arcs. The polyhedron theorem yields $\left(R_{c}+1\right)+\left(P_{c}+2 X_{c}\right)=\left(A_{c}+2 X_{c}\right)+2$, considering the exterior of $c$ as a single region. Therefore, + +$$ +R_{c}+P_{c}=A_{c}+1 +$$ + +Moreover, we have four arcs starting from every interior points inside $c$ and a single arc starting into the interior from each intersection point on the circle. By double-counting the end-points of the interior arcs we get $2 A_{c}=4 P_{c}+2 X_{c}$, so + +$$ +A_{c}=2 P_{c}+X_{c} +$$ + +The relations (2) and (3) together yield + +$$ +R_{c}-P_{c}=X_{c}+1 +$$ + +By summing up (4) for all circles we obtain + +$$ +\sum_{c \in \mathcal{C}} R_{c}-\sum_{c \in \mathcal{C}} P_{c}=\sum_{c \in \mathcal{C}} X_{c}+|\mathcal{C}| +$$ + +which yields + +$$ +\left|\mathcal{R}_{\mathrm{odd}}\right|-\left|\mathcal{P}_{\mathrm{odd}}\right| \equiv \sum_{c \in \mathcal{C}} X_{c}+n \quad(\bmod 2) +$$ + +Notice that in $\sum_{c \in \mathcal{C}} X_{c}$ each intersecting circle pair is counted twice, i.e., for both circles in the pair, so + +$$ +\sum_{c \in \mathcal{C}} X_{c} \equiv 0 \quad(\bmod 2) +$$ + +which finishes the proof of the Claim. +Now insert the same small arcs at the intersections as in the first solution, and suppose that there is a single snail orbit $b$. + +First we show that the odd regions are inside the curve $b$, while the even regions are outside. Take a region $r \in \mathcal{R}$ and a point $x$ in its interior, and draw a ray $y$, starting from $x$, that does not pass through any intersection point of the circles and is neither tangent to any of the circles. As is well-known, $x$ is inside the curve $b$ if and only if $y$ intersects $b$ an odd number of times (see Figure 10). Notice that if an arbitrary circle $c$ contains $x$ in its interior, then $c$ intersects $y$ at a single point; otherwise, if $x$ is outside $c$, then $c$ has 2 or 0 intersections with $y$. Therefore, $y$ intersects $b$ an odd number of times if and only if $x$ is contained in an odd number of circles, so if and only if $r$ is odd. +![](https://cdn.mathpix.com/cropped/2024_11_18_62c85fb0b01b41aca29bg-49.jpg?height=321&width=626&top_left_y=2255&top_left_x=715) + +Figure 10 +Now consider an intersection point $p$ of two circles $c_{1}$ and $c_{2}$ and a small neighbourhood around $p$. Suppose that $p$ is contained inside $k$ circles. + +We have four regions that meet at $p$. Let $r_{1}$ be the region that lies outside both $c_{1}$ and $c_{2}$, let $r_{2}$ be the region that lies inside both $c_{1}$ and $c_{2}$, and let $r_{3}$ and $r_{4}$ be the two remaining regions, each lying inside exactly one of $c_{1}$ and $c_{2}$. The region $r_{1}$ is contained inside the same $k$ circles as $p$; the region $r_{2}$ is contained also by $c_{1}$ and $c_{2}$, so by $k+2$ circles in total; each of the regions $r_{3}$ and $r_{4}$ is contained inside $k+1$ circles. After the small arcs have been inserted at $p$, the regions $r_{1}$ and $r_{2}$ get connected, and the regions $r_{3}$ and $r_{4}$ remain separated at $p$ (see Figure 11). If $p$ is an odd point, then $r_{1}$ and $r_{2}$ are odd, so two odd regions are connected at $p$. Otherwise, if $p$ is even, then we have two even regions connected at $p$. +![](https://cdn.mathpix.com/cropped/2024_11_18_62c85fb0b01b41aca29bg-50.jpg?height=261&width=298&top_left_y=658&top_left_x=571) + +Figure 11 +![](https://cdn.mathpix.com/cropped/2024_11_18_62c85fb0b01b41aca29bg-50.jpg?height=286&width=481&top_left_y=645&top_left_x=1007) + +Figure 12 + +Consider the system of odd regions and their connections at the odd points as a graph. In this graph the odd regions are the vertices, and each odd point establishes an edge that connects two vertices (see Figure 12). As $b$ is a single closed curve, this graph is connected and contains no cycle, so the graph is a tree. Then the number of vertices must be by one greater than the number of edges, so + +$$ +\left|\mathcal{R}_{\text {odd }}\right|-\left|\mathcal{P}_{\text {odd }}\right|=1 . +$$ + +The relations (1) and (9) together prove that $n$ must be odd. + +Comment. For every odd $n$ there exists at least one configuration of $n$ circles with a single snail orbit. Figure 13 shows a possible configuration with 5 circles. In general, if a circle is rotated by $k \cdot \frac{360^{\circ}}{n}$ $(k=1,2, \ldots, n-1)$ around an interior point other than the centre, the circle and its rotated copies together provide a single snail orbit. +![](https://cdn.mathpix.com/cropped/2024_11_18_62c85fb0b01b41aca29bg-50.jpg?height=584&width=595&top_left_y=1641&top_left_x=736) + +Figure 13 + +## Geometry + +G1. The points $P$ and $Q$ are chosen on the side $B C$ of an acute-angled triangle $A B C$ so that $\angle P A B=\angle A C B$ and $\angle Q A C=\angle C B A$. The points $M$ and $N$ are taken on the rays $A P$ and $A Q$, respectively, so that $A P=P M$ and $A Q=Q N$. Prove that the lines $B M$ and $C N$ intersect on the circumcircle of the triangle $A B C$. +(Georgia) +Solution 1. Denote by $S$ the intersection point of the lines $B M$ and $C N$. Let moreover $\beta=\angle Q A C=\angle C B A$ and $\gamma=\angle P A B=\angle A C B$. From these equalities it follows that the triangles $A B P$ and $C A Q$ are similar (see Figure 1). Therefore we obtain + +$$ +\frac{B P}{P M}=\frac{B P}{P A}=\frac{A Q}{Q C}=\frac{N Q}{Q C} +$$ + +Moreover, + +$$ +\angle B P M=\beta+\gamma=\angle C Q N +$$ + +Hence the triangles $B P M$ and $N Q C$ are similar. This gives $\angle B M P=\angle N C Q$, so the triangles $B P M$ and $B S C$ are also similar. Thus we get + +$$ +\angle C S B=\angle B P M=\beta+\gamma=180^{\circ}-\angle B A C, +$$ + +which completes the solution. +![](https://cdn.mathpix.com/cropped/2024_11_18_62c85fb0b01b41aca29bg-51.jpg?height=618&width=595&top_left_y=1299&top_left_x=374) + +Figure 1 +![](https://cdn.mathpix.com/cropped/2024_11_18_62c85fb0b01b41aca29bg-51.jpg?height=643&width=624&top_left_y=1272&top_left_x=1090) + +Figure 2 + +Solution 2. As in the previous solution, denote by $S$ the intersection point of the lines $B M$ and $N C$. Let moreover the circumcircle of the triangle $A B C$ intersect the lines $A P$ and $A Q$ again at $K$ and $L$, respectively (see Figure 2). + +Note that $\angle L B C=\angle L A C=\angle C B A$ and similarly $\angle K C B=\angle K A B=\angle B C A$. It implies that the lines $B L$ and $C K$ meet at a point $X$, being symmetric to the point $A$ with respect to the line $B C$. Since $A P=P M$ and $A Q=Q N$, it follows that $X$ lies on the line $M N$. Therefore, using Pascal's theorem for the hexagon $A L B S C K$, we infer that $S$ lies on the circumcircle of the triangle $A B C$, which finishes the proof. + +Comment. Both solutions can be modified to obtain a more general result, with the equalities + +$$ +A P=P M \quad \text { and } \quad A Q=Q N +$$ + +replaced by + +$$ +\frac{A P}{P M}=\frac{Q N}{A Q} +$$ + +G2. Let $A B C$ be a triangle. The points $K, L$, and $M$ lie on the segments $B C, C A$, and $A B$, respectively, such that the lines $A K, B L$, and $C M$ intersect in a common point. Prove that it is possible to choose two of the triangles $A L M, B M K$, and $C K L$ whose inradii sum up to at least the inradius of the triangle $A B C$. +(Estonia) +Solution. Denote + +$$ +a=\frac{B K}{K C}, \quad b=\frac{C L}{L A}, \quad c=\frac{A M}{M B} . +$$ + +By Ceva's theorem, $a b c=1$, so we may, without loss of generality, assume that $a \geqslant 1$. Then at least one of the numbers $b$ or $c$ is not greater than 1 . Therefore at least one of the pairs $(a, b)$, $(b, c)$ has its first component not less than 1 and the second one not greater than 1 . Without loss of generality, assume that $1 \leqslant a$ and $b \leqslant 1$. + +Therefore, we obtain $b c \leqslant 1$ and $1 \leqslant c a$, or equivalently + +$$ +\frac{A M}{M B} \leqslant \frac{L A}{C L} \quad \text { and } \quad \frac{M B}{A M} \leqslant \frac{B K}{K C} . +$$ + +The first inequality implies that the line passing through $M$ and parallel to $B C$ intersects the segment $A L$ at a point $X$ (see Figure 1). Therefore the inradius of the triangle $A L M$ is not less than the inradius $r_{1}$ of triangle $A M X$. + +Similarly, the line passing through $M$ and parallel to $A C$ intersects the segment $B K$ at a point $Y$, so the inradius of the triangle $B M K$ is not less than the inradius $r_{2}$ of the triangle $B M Y$. Thus, to complete our solution, it is enough to show that $r_{1}+r_{2} \geqslant r$, where $r$ is the inradius of the triangle $A B C$. We prove that in fact $r_{1}+r_{2}=r$. +![](https://cdn.mathpix.com/cropped/2024_11_18_62c85fb0b01b41aca29bg-52.jpg?height=597&width=849&top_left_y=1369&top_left_x=621) + +Figure 1 +Since $M X \| B C$, the dilation with centre $A$ that takes $M$ to $B$ takes the incircle of the triangle $A M X$ to the incircle of the triangle $A B C$. Therefore + +$$ +\frac{r_{1}}{r}=\frac{A M}{A B}, \quad \text { and similarly } \quad \frac{r_{2}}{r}=\frac{M B}{A B} . +$$ + +Adding these equalities gives $r_{1}+r_{2}=r$, as required. +Comment. Alternatively, one can use Desargues' theorem instead of Ceva's theorem, as follows: The lines $A B, B C, C A$ dissect the plane into seven regions. One of them is bounded, and amongst the other six, three are two-sided and three are three-sided. Now define the points $P=B C \cap L M$, $Q=C A \cap M K$, and $R=A B \cap K L$ (in the projective plane). By Desargues' theorem, the points $P$, $Q, R$ lie on a common line $\ell$. This line intersects only unbounded regions. If we now assume (without loss of generality) that $P, Q$ and $R$ lie on $\ell$ in that order, then one of the segments $P Q$ or $Q R$ lies inside a two-sided region. If, for example, this segment is $P Q$, then the triangles $A L M$ and $B M K$ will satisfy the statement of the problem for the same reason. + +G3. Let $\Omega$ and $O$ be the circumcircle and the circumcentre of an acute-angled triangle $A B C$ with $A B>B C$. The angle bisector of $\angle A B C$ intersects $\Omega$ at $M \neq B$. Let $\Gamma$ be the circle with diameter $B M$. The angle bisectors of $\angle A O B$ and $\angle B O C$ intersect $\Gamma$ at points $P$ and $Q$, respectively. The point $R$ is chosen on the line $P Q$ so that $B R=M R$. Prove that $B R \| A C$. (Here we always assume that an angle bisector is a ray.) +(Russia) +Solution. Let $K$ be the midpoint of $B M$, i.e., the centre of $\Gamma$. Notice that $A B \neq B C$ implies $K \neq O$. Clearly, the lines $O M$ and $O K$ are the perpendicular bisectors of $A C$ and $B M$, respectively. Therefore, $R$ is the intersection point of $P Q$ and $O K$. + +Let $N$ be the second point of intersection of $\Gamma$ with the line $O M$. Since $B M$ is a diameter of $\Gamma$, the lines $B N$ and $A C$ are both perpendicular to $O M$. Hence $B N \| A C$, and it suffices to prove that $B N$ passes through $R$. Our plan for doing this is to interpret the lines $B N, O K$, and $P Q$ as the radical axes of three appropriate circles. + +Let $\omega$ be the circle with diameter $B O$. Since $\angle B N O=\angle B K O=90^{\circ}$, the points $N$ and $K$ lie on $\omega$. + +Next we show that the points $O, K, P$, and $Q$ are concyclic. To this end, let $D$ and $E$ be the midpoints of $B C$ and $A B$, respectively. Clearly, $D$ and $E$ lie on the rays $O Q$ and $O P$, respectively. By our assumptions about the triangle $A B C$, the points $B, E, O, K$, and $D$ lie in this order on $\omega$. It follows that $\angle E O R=\angle E B K=\angle K B D=\angle K O D$, so the line $K O$ externally bisects the angle $P O Q$. Since the point $K$ is the centre of $\Gamma$, it also lies on the perpendicular bisector of $P Q$. So $K$ coincides with the midpoint of the $\operatorname{arc} P O Q$ of the circumcircle $\gamma$ of triangle $P O Q$. + +Thus the lines $O K, B N$, and $P Q$ are pairwise radical axes of the circles $\omega, \gamma$, and $\Gamma$. Hence they are concurrent at $R$, as required. +![](https://cdn.mathpix.com/cropped/2024_11_18_62c85fb0b01b41aca29bg-53.jpg?height=877&width=1429&top_left_y=1466&top_left_x=316) + +G4. Consider a fixed circle $\Gamma$ with three fixed points $A, B$, and $C$ on it. Also, let us fix a real number $\lambda \in(0,1)$. For a variable point $P \notin\{A, B, C\}$ on $\Gamma$, let $M$ be the point on the segment $C P$ such that $C M=\lambda \cdot C P$. Let $Q$ be the second point of intersection of the circumcircles of the triangles $A M P$ and $B M C$. Prove that as $P$ varies, the point $Q$ lies on a fixed circle. +(United Kingdom) +Solution 1. Throughout the solution, we denote by $\Varangle(a, b)$ the directed angle between the lines $a$ and $b$. + +Let $D$ be the point on the segment $A B$ such that $B D=\lambda \cdot B A$. We will show that either $Q=D$, or $\Varangle(D Q, Q B)=\Varangle(A B, B C)$; this would mean that the point $Q$ varies over the constant circle through $D$ tangent to $B C$ at $B$, as required. + +Denote the circumcircles of the triangles $A M P$ and $B M C$ by $\omega_{A}$ and $\omega_{B}$, respectively. The lines $A P, B C$, and $M Q$ are pairwise radical axes of the circles $\Gamma, \omega_{A}$, and $\omega_{B}$, thus either they are parallel, or they share a common point $X$. + +Assume that these lines are parallel (see Figure 1). Then the segments $A P, Q M$, and $B C$ have a common perpendicular bisector; the reflection in this bisector maps the segment $C P$ to $B A$, and maps $M$ to $Q$. Therefore, in this case $Q$ lies on $A B$, and $B Q / A B=C M / C P=$ $B D / A B$; so we have $Q=D$. +![](https://cdn.mathpix.com/cropped/2024_11_18_62c85fb0b01b41aca29bg-54.jpg?height=644&width=549&top_left_y=1300&top_left_x=225) + +Figure 1 +![](https://cdn.mathpix.com/cropped/2024_11_18_62c85fb0b01b41aca29bg-54.jpg?height=709&width=1040&top_left_y=1230&top_left_x=802) + +Figure 2 + +Now assume that the lines $A P, Q M$, and $B C$ are concurrent at some point $X$ (see Figure 2). Notice that the points $A, B, Q$, and $X$ lie on a common circle $\Omega$ by Miquel's theorem applied to the triangle $X P C$. Let us denote by $Y$ the symmetric image of $X$ about the perpendicular bisector of $A B$. Clearly, $Y$ lies on $\Omega$, and the triangles $Y A B$ and $\triangle X B A$ are congruent. Moreover, the triangle $X P C$ is similar to the triangle $X B A$, so it is also similar to the triangle $Y A B$. + +Next, the points $D$ and $M$ correspond to each other in similar triangles $Y A B$ and $X P C$, since $B D / B A=C M / C P=\lambda$. Moreover, the triangles $Y A B$ and $X P C$ are equi-oriented, so $\Varangle(M X, X P)=\Varangle(D Y, Y A)$. On the other hand, since the points $A, Q, X$, and $Y$ lie on $\Omega$, we have $\Varangle(Q Y, Y A)=\Varangle(M X, X P)$. Therefore, $\Varangle(Q Y, Y A)=\Varangle(D Y, Y A)$, so the points $Y, D$, and $Q$ are collinear. + +Finally, we have $\Varangle(D Q, Q B)=\Varangle(Y Q, Q B)=\Varangle(Y A, A B)=\Varangle(A B, B X)=\Varangle(A B, B C)$, as desired. + +Comment. In the original proposal, $\lambda$ was supposed to be an arbitrary real number distinct from 0 and 1, and the point $M$ was defined by $\overrightarrow{C M}=\lambda \cdot \overrightarrow{C P}$. The Problem Selection Committee decided to add the restriction $\lambda \in(0,1)$ in order to avoid a large case distinction. + +Solution 2. As in the previous solution, we introduce the radical centre $X=A P \cap B C \cap M Q$ of the circles $\omega_{A}, \omega_{B}$, and $\Gamma$. Next, we also notice that the points $A, Q, B$, and $X$ lie on a common circle $\Omega$. + +If the point $P$ lies on the arc $B A C$ of $\Gamma$, then the point $X$ is outside $\Gamma$, thus the point $Q$ belongs to the ray $X M$, and therefore the points $P, A$, and $Q$ lie on the same side of $B C$. Otherwise, if $P$ lies on the arc $B C$ not containing $A$, then $X$ lies inside $\Gamma$, so $M$ and $Q$ lie on different sides of $B C$; thus again $Q$ and $A$ lie on the same side of $B C$. So, in each case the points $Q$ and $A$ lie on the same side of $B C$. +![](https://cdn.mathpix.com/cropped/2024_11_18_62c85fb0b01b41aca29bg-55.jpg?height=715&width=1009&top_left_y=839&top_left_x=523) + +Figure 3 +Now we prove that the ratio + +$$ +\frac{Q B}{\sin \angle Q B C}=\frac{Q B}{Q X} \cdot \frac{Q X}{\sin \angle Q B X} +$$ + +is constant. Since the points $A, Q, B$, and $X$ are concyclic, we have + +$$ +\frac{Q X}{\sin \angle Q B X}=\frac{A X}{\sin \angle A B C} +$$ + +Next, since the points $B, Q, M$, and $C$ are concyclic, the triangles $X B Q$ and $X M C$ are similar, so + +$$ +\frac{Q B}{Q X}=\frac{C M}{C X}=\lambda \cdot \frac{C P}{C X} +$$ + +Analogously, the triangles $X C P$ and $X A B$ are also similar, so + +$$ +\frac{C P}{C X}=\frac{A B}{A X} +$$ + +Therefore, we obtain + +$$ +\frac{Q B}{\sin \angle Q B C}=\lambda \cdot \frac{A B}{A X} \cdot \frac{A X}{\sin \angle A B C}=\lambda \cdot \frac{A B}{\sin \angle A B C} +$$ + +so this ratio is indeed constant. Thus the circle passing through $Q$ and tangent to $B C$ at $B$ is also constant, and $Q$ varies over this fixed circle. + +Comment. It is not hard to guess that the desired circle should be tangent to $B C$ at $B$. Indeed, the second paragraph of this solution shows that this circle lies on one side of $B C$; on the other hand, in the limit case $P=B$, the point $Q$ also coincides with $B$. + +Solution 3. Let us perform an inversion centred at $C$. Denote by $X^{\prime}$ the image of a point $X$ under this inversion. + +The circle $\Gamma$ maps to the line $\Gamma^{\prime}$ passing through the constant points $A^{\prime}$ and $B^{\prime}$, and containing the variable point $P^{\prime}$. By the problem condition, the point $M$ varies over the circle $\gamma$ which is the homothetic image of $\Gamma$ with centre $C$ and coefficient $\lambda$. Thus $M^{\prime}$ varies over the constant line $\gamma^{\prime} \| A^{\prime} B^{\prime}$ which is the homothetic image of $A^{\prime} B^{\prime}$ with centre $C$ and coefficient $1 / \lambda$, and $M=\gamma^{\prime} \cap C P^{\prime}$. Next, the circumcircles $\omega_{A}$ and $\omega_{B}$ of the triangles $A M P$ and $B M C$ map to the circumcircle $\omega_{A}^{\prime}$ of the triangle $A^{\prime} M^{\prime} P^{\prime}$ and to the line $B^{\prime} M^{\prime}$, respectively; the point $Q$ thus maps to the second point of intersection of $B^{\prime} M^{\prime}$ with $\omega_{A}^{\prime}$ (see Figure 4). +![](https://cdn.mathpix.com/cropped/2024_11_18_62c85fb0b01b41aca29bg-56.jpg?height=675&width=1029&top_left_y=913&top_left_x=519) + +Figure 4 + +Let $J$ be the (constant) common point of the lines $\gamma^{\prime}$ and $C A^{\prime}$, and let $\ell$ be the (constant) line through $J$ parallel to $C B^{\prime}$. Let $V$ be the common point of the lines $\ell$ and $B^{\prime} M^{\prime}$. Applying Pappus' theorem to the triples $\left(C, J, A^{\prime}\right)$ and $\left(V, B^{\prime}, M^{\prime}\right)$ we get that the points $C B^{\prime} \cap J V$, $J M^{\prime} \cap A^{\prime} B^{\prime}$, and $C M^{\prime} \cap A^{\prime} V$ are collinear. The first two of these points are ideal, hence so is the third, which means that $C M^{\prime} \| A^{\prime} V$. + +Now we have $\Varangle\left(Q^{\prime} A^{\prime}, A^{\prime} P^{\prime}\right)=\Varangle\left(Q^{\prime} M^{\prime}, M^{\prime} P^{\prime}\right)=\angle\left(V M^{\prime}, A^{\prime} V\right)$, which means that the triangles $B^{\prime} Q^{\prime} A^{\prime}$ and $B^{\prime} A^{\prime} V$ are similar, and $\left(B^{\prime} A^{\prime}\right)^{2}=B^{\prime} Q^{\prime} \cdot B^{\prime} V$. Thus $Q^{\prime}$ is the image of $V$ under the second (fixed) inversion with centre $B^{\prime}$ and radius $B^{\prime} A^{\prime}$. Since $V$ varies over the constant line $\ell, Q^{\prime}$ varies over some constant circle $\Theta$. Thus, applying the first inversion back we get that $Q$ also varies over some fixed circle. + +One should notice that this last circle is not a line; otherwise $\Theta$ would contain $C$, and thus $\ell$ would contain the image of $C$ under the second inversion. This is impossible, since $C B^{\prime} \| \ell$. + +G5. Let $A B C D$ be a convex quadrilateral with $\angle B=\angle D=90^{\circ}$. Point $H$ is the foot of the perpendicular from $A$ to $B D$. The points $S$ and $T$ are chosen on the sides $A B$ and $A D$, respectively, in such a way that $H$ lies inside triangle $S C T$ and + +$$ +\angle S H C-\angle B S C=90^{\circ}, \quad \angle T H C-\angle D T C=90^{\circ} . +$$ + +Prove that the circumcircle of triangle $S H T$ is tangent to the line $B D$. +(Iran) +Solution. Let the line passing through $C$ and perpendicular to the line $S C$ intersect the line $A B$ at $Q$ (see Figure 1). Then + +$$ +\angle S Q C=90^{\circ}-\angle B S C=180^{\circ}-\angle S H C, +$$ + +which implies that the points $C, H, S$, and $Q$ lie on a common circle. Moreover, since $S Q$ is a diameter of this circle, we infer that the circumcentre $K$ of triangle $S H C$ lies on the line $A B$. Similarly, we prove that the circumcentre $L$ of triangle $C H T$ lies on the line $A D$. +![](https://cdn.mathpix.com/cropped/2024_11_18_62c85fb0b01b41aca29bg-57.jpg?height=795&width=1052&top_left_y=959&top_left_x=516) + +Figure 1 +In order to prove that the circumcircle of triangle $S H T$ is tangent to $B D$, it suffices to show that the perpendicular bisectors of $H S$ and $H T$ intersect on the line $A H$. However, these two perpendicular bisectors coincide with the angle bisectors of angles $A K H$ and $A L H$. Therefore, in order to complete the solution, it is enough (by the bisector theorem) to show that + +$$ +\frac{A K}{K H}=\frac{A L}{L H} +$$ + +We present two proofs of this equality. +First proof. Let the lines $K L$ and $H C$ intersect at $M$ (see Figure 2). Since $K H=K C$ and $L H=L C$, the points $H$ and $C$ are symmetric to each other with respect to the line $K L$. Therefore $M$ is the midpoint of $H C$. Denote by $O$ the circumcentre of quadrilateral $A B C D$. Then $O$ is the midpoint of $A C$. Therefore we have $O M \| A H$ and hence $O M \perp B D$. This together with the equality $O B=O D$ implies that $O M$ is the perpendicular bisector of $B D$ and therefore $B M=D M$. + +Since $C M \perp K L$, the points $B, C, M$, and $K$ lie on a common circle with diameter $K C$. Similarly, the points $L, C, M$, and $D$ lie on a circle with diameter $L C$. Thus, using the sine law, we obtain + +$$ +\frac{A K}{A L}=\frac{\sin \angle A L K}{\sin \angle A K L}=\frac{D M}{C L} \cdot \frac{C K}{B M}=\frac{C K}{C L}=\frac{K H}{L H} +$$ + +which finishes the proof of (1). +![](https://cdn.mathpix.com/cropped/2024_11_18_62c85fb0b01b41aca29bg-58.jpg?height=806&width=875&top_left_y=294&top_left_x=202) + +Figure 2 +![](https://cdn.mathpix.com/cropped/2024_11_18_62c85fb0b01b41aca29bg-58.jpg?height=744&width=752&top_left_y=359&top_left_x=1126) + +Figure 3 + +Second proof. If the points $A, H$, and $C$ are collinear, then $A K=A L$ and $K H=L H$, so the equality (1) follows. Assume therefore that the points $A, H$, and $C$ do not lie in a line and consider the circle $\omega$ passing through them (see Figure 3). Since the quadrilateral $A B C D$ is cyclic, + +$$ +\angle B A C=\angle B D C=90^{\circ}-\angle A D H=\angle H A D . +$$ + +Let $N \neq A$ be the intersection point of the circle $\omega$ and the angle bisector of $\angle C A H$. Then $A N$ is also the angle bisector of $\angle B A D$. Since $H$ and $C$ are symmetric to each other with respect to the line $K L$ and $H N=N C$, it follows that both $N$ and the centre of $\omega$ lie on the line $K L$. This means that the circle $\omega$ is an Apollonius circle of the points $K$ and $L$. This immediately yields (1). + +Comment. Either proof can be used to obtain the following generalised result: +Let $A B C D$ be a convex quadrilateral and let $H$ be a point in its interior with $\angle B A C=\angle D A H$. The points $S$ and $T$ are chosen on the sides $A B$ and $A D$, respectively, in such a way that $H$ lies inside triangle SCT and + +$$ +\angle S H C-\angle B S C=90^{\circ}, \quad \angle T H C-\angle D T C=90^{\circ} . +$$ + +Then the circumcentre of triangle SHT lies on the line AH (and moreover the circumcentre of triangle SCT lies on $A C$ ). + +G6. Let $A B C$ be a fixed acute-angled triangle. Consider some points $E$ and $F$ lying on the sides $A C$ and $A B$, respectively, and let $M$ be the midpoint of $E F$. Let the perpendicular bisector of $E F$ intersect the line $B C$ at $K$, and let the perpendicular bisector of $M K$ intersect the lines $A C$ and $A B$ at $S$ and $T$, respectively. We call the pair $(E, F)$ interesting, if the quadrilateral $K S A T$ is cyclic. + +Suppose that the pairs $\left(E_{1}, F_{1}\right)$ and $\left(E_{2}, F_{2}\right)$ are interesting. Prove that + +$$ +\frac{E_{1} E_{2}}{A B}=\frac{F_{1} F_{2}}{A C} +$$ + +(Iran) +Solution 1. For any interesting pair $(E, F)$, we will say that the corresponding triangle $E F K$ is also interesting. + +Let $E F K$ be an interesting triangle. Firstly, we prove that $\angle K E F=\angle K F E=\angle A$, which also means that the circumcircle $\omega_{1}$ of the triangle $A E F$ is tangent to the lines $K E$ and $K F$. + +Denote by $\omega$ the circle passing through the points $K, S, A$, and $T$. Let the line $A M$ intersect the line $S T$ and the circle $\omega$ (for the second time) at $N$ and $L$, respectively (see Figure 1). + +Since $E F \| T S$ and $M$ is the midpoint of $E F, N$ is the midpoint of $S T$. Moreover, since $K$ and $M$ are symmetric to each other with respect to the line $S T$, we have $\angle K N S=\angle M N S=$ $\angle L N T$. Thus the points $K$ and $L$ are symmetric to each other with respect to the perpendicular bisector of $S T$. Therefore $K L \| S T$. + +Let $G$ be the point symmetric to $K$ with respect to $N$. Then $G$ lies on the line $E F$, and we may assume that it lies on the ray $M F$. One has + +$$ +\angle K G E=\angle K N S=\angle S N M=\angle K L A=180^{\circ}-\angle K S A +$$ + +(if $K=L$, then the angle $K L A$ is understood to be the angle between $A L$ and the tangent to $\omega$ at $L$ ). This means that the points $K, G, E$, and $S$ are concyclic. Now, since $K S G T$ is a parallelogram, we obtain $\angle K E F=\angle K S G=180^{\circ}-\angle T K S=\angle A$. Since $K E=K F$, we also have $\angle K F E=\angle K E F=\angle A$. + +After having proved this fact, one may finish the solution by different methods. +![](https://cdn.mathpix.com/cropped/2024_11_18_62c85fb0b01b41aca29bg-59.jpg?height=684&width=829&top_left_y=1874&top_left_x=245) + +Figure 1 +![](https://cdn.mathpix.com/cropped/2024_11_18_62c85fb0b01b41aca29bg-59.jpg?height=612&width=730&top_left_y=1944&top_left_x=1117) + +Figure 2 + +First method. We have just proved that all interesting triangles are similar to each other. This allows us to use the following lemma. + +Lemma. Let $A B C$ be an arbitrary triangle. Choose two points $E_{1}$ and $E_{2}$ on the side $A C$, two points $F_{1}$ and $F_{2}$ on the side $A B$, and two points $K_{1}$ and $K_{2}$ on the side $B C$, in a way that the triangles $E_{1} F_{1} K_{1}$ and $E_{2} F_{2} K_{2}$ are similar. Then the six circumcircles of the triangles $A E_{i} F_{i}$, $B F_{i} K_{i}$, and $C E_{i} K_{i}(i=1,2)$ meet at a common point $Z$. Moreover, $Z$ is the centre of the spiral similarity that takes the triangle $E_{1} F_{1} K_{1}$ to the triangle $E_{2} F_{2} K_{2}$. +Proof. Firstly, notice that for each $i=1,2$, the circumcircles of the triangles $A E_{i} F_{i}, B F_{i} K_{i}$, and $C K_{i} E_{i}$ have a common point $Z_{i}$ by Miquel's theorem. Moreover, we have +$\Varangle\left(Z_{i} F_{i}, Z_{i} E_{i}\right)=\Varangle(A B, C A), \quad \Varangle\left(Z_{i} K_{i}, Z_{i} F_{i}\right)=\Varangle(B C, A B), \quad \Varangle\left(Z_{i} E_{i}, Z_{i} K_{i}\right)=\Varangle(C A, B C)$. +This yields that the points $Z_{1}$ and $Z_{2}$ correspond to each other in similar triangles $E_{1} F_{1} K_{1}$ and $E_{2} F_{2} K_{2}$. Thus, if they coincide, then this common point is indeed the desired centre of a spiral similarity. + +Finally, in order to show that $Z_{1}=Z_{2}$, one may notice that $\Varangle\left(A B, A Z_{1}\right)=\Varangle\left(E_{1} F_{1}, E_{1} Z_{1}\right)=$ $\Varangle\left(E_{2} F_{2}, E_{2} Z_{2}\right)=\Varangle\left(A B, A Z_{2}\right)$ (see Figure 2). Similarly, one has $\Varangle\left(B C, B Z_{1}\right)=\Varangle\left(B C, B Z_{2}\right)$ and $\Varangle\left(C A, C Z_{1}\right)=\Varangle\left(C A, C Z_{2}\right)$. This yields $Z_{1}=Z_{2}$. + +Now, let $P$ and $Q$ be the feet of the perpendiculars from $B$ and $C$ onto $A C$ and $A B$, respectively, and let $R$ be the midpoint of $B C$ (see Figure 3). Then $R$ is the circumcentre of the cyclic quadrilateral $B C P Q$. Thus we obtain $\angle A P Q=\angle B$ and $\angle R P C=\angle C$, which yields $\angle Q P R=\angle A$. Similarly, we show that $\angle P Q R=\angle A$. Thus, all interesting triangles are similar to the triangle $P Q R$. +![](https://cdn.mathpix.com/cropped/2024_11_18_62c85fb0b01b41aca29bg-60.jpg?height=615&width=715&top_left_y=1326&top_left_x=316) + +Figure 3 +![](https://cdn.mathpix.com/cropped/2024_11_18_62c85fb0b01b41aca29bg-60.jpg?height=615&width=692&top_left_y=1326&top_left_x=1090) + +Figure 4 + +Denote now by $Z$ the common point of the circumcircles of $A P Q, B Q R$, and $C P R$. Let $E_{1} F_{1} K_{1}$ and $E_{2} F_{2} K_{2}$ be two interesting triangles. By the lemma, $Z$ is the centre of any spiral similarity taking one of the triangles $E_{1} F_{1} K_{1}, E_{2} F_{2} K_{2}$, and $P Q R$ to some other of them. Therefore the triangles $Z E_{1} E_{2}$ and $Z F_{1} F_{2}$ are similar, as well as the triangles $Z E_{1} F_{1}$ and $Z P Q$. Hence + +$$ +\frac{E_{1} E_{2}}{F_{1} F_{2}}=\frac{Z E_{1}}{Z F_{1}}=\frac{Z P}{Z Q} +$$ + +Moreover, the equalities $\angle A Z Q=\angle A P Q=\angle A B C=180^{\circ}-\angle Q Z R$ show that the point $Z$ lies on the line $A R$ (see Figure 4). Therefore the triangles $A Z P$ and $A C R$ are similar, as well as the triangles $A Z Q$ and $A B R$. This yields + +$$ +\frac{Z P}{Z Q}=\frac{Z P}{R C} \cdot \frac{R B}{Z Q}=\frac{A Z}{A C} \cdot \frac{A B}{A Z}=\frac{A B}{A C} +$$ + +which completes the solution. + +Second method. Now we will start from the fact that $\omega_{1}$ is tangent to the lines $K E$ and $K F$ (see Figure 5). We prove that if $(E, F)$ is an interesting pair, then + +$$ +\frac{A E}{A B}+\frac{A F}{A C}=2 \cos \angle A +$$ + +Let $Y$ be the intersection point of the segments $B E$ and $C F$. The points $B, K$, and $C$ are collinear, hence applying PASCAL's theorem to the degenerated hexagon AFFYEE, we infer that $Y$ lies on the circle $\omega_{1}$. + +Denote by $Z$ the second intersection point of the circumcircle of the triangle $B F Y$ with the line $B C$ (see Figure 6). By Miquel's theorem, the points $C, Z, Y$, and $E$ are concyclic. Therefore we obtain + +$$ +B F \cdot A B+C E \cdot A C=B Y \cdot B E+C Y \cdot C F=B Z \cdot B C+C Z \cdot B C=B C^{2} +$$ + +On the other hand, $B C^{2}=A B^{2}+A C^{2}-2 A B \cdot A C \cos \angle A$, by the cosine law. Hence + +$$ +(A B-A F) \cdot A B+(A C-A E) \cdot A C=A B^{2}+A C^{2}-2 A B \cdot A C \cos \angle A, +$$ + +which simplifies to the desired equality (1). +Let now $\left(E_{1}, F_{1}\right)$ and $\left(E_{2}, F_{2}\right)$ be two interesting pairs of points. Then we get + +$$ +\frac{A E_{1}}{A B}+\frac{A F_{1}}{A C}=\frac{A E_{2}}{A B}+\frac{A F_{2}}{A C} +$$ + +which gives the desired result. +![](https://cdn.mathpix.com/cropped/2024_11_18_62c85fb0b01b41aca29bg-61.jpg?height=709&width=915&top_left_y=1527&top_left_x=248) + +Figure 5 +![](https://cdn.mathpix.com/cropped/2024_11_18_62c85fb0b01b41aca29bg-61.jpg?height=569&width=615&top_left_y=1669&top_left_x=1229) + +Figure 6 + +Third method. Again, we make use of the fact that all interesting triangles are similar (and equi-oriented). Let us put the picture onto a complex plane such that $A$ is at the origin, and identify each point with the corresponding complex number. + +Let $E F K$ be any interesting triangle. The equalities $\angle K E F=\angle K F E=\angle A$ yield that the ratio $\nu=\frac{K-E}{F-E}$ is the same for all interesting triangles. This in turn means that the numbers $E$, $F$, and $K$ satisfy the linear equation + +$$ +K=\mu E+\nu F, \quad \text { where } \quad \mu=1-\nu +$$ + +Now let us choose the points $X$ and $Y$ on the rays $A B$ and $A C$, respectively, so that $\angle C X A=\angle A Y B=\angle A=\angle K E F$ (see Figure 7). Then each of the triangles $A X C$ and $Y A B$ is similar to any interesting triangle, which also means that + +$$ +C=\mu A+\nu X=\nu X \quad \text { and } \quad B=\mu Y+\nu A=\mu Y . +$$ + +Moreover, one has $X / Y=\overline{C / B}$. +Since the points $E, F$, and $K$ lie on $A C, A B$, and $B C$, respectively, one gets + +$$ +E=\rho Y, \quad F=\sigma X, \quad \text { and } \quad K=\lambda B+(1-\lambda) C +$$ + +for some real $\rho, \sigma$, and $\lambda$. In view of (3), the equation (2) now reads $\lambda B+(1-\lambda) C=K=$ $\mu E+\nu F=\rho B+\sigma C$, or + +$$ +(\lambda-\rho) B=(\sigma+\lambda-1) C +$$ + +Since the nonzero complex numbers $B$ and $C$ have different arguments, the coefficients in the brackets vanish, so $\rho=\lambda$ and $\sigma=1-\lambda$. Therefore, + +$$ +\frac{E}{Y}+\frac{F}{X}=\rho+\sigma=1 +$$ + +Now, if $\left(E_{1}, F_{1}\right)$ and $\left(E_{2}, F_{2}\right)$ are two distinct interesting pairs, one may apply (4) to both pairs. Subtracting, we get + +$$ +\frac{E_{1}-E_{2}}{Y}=\frac{F_{2}-F_{1}}{X}, \quad \text { so } \quad \frac{E_{1}-E_{2}}{F_{2}-F_{1}}=\frac{Y}{X}=\frac{\bar{B}}{\bar{C}} +$$ + +Taking absolute values provides the required result. +![](https://cdn.mathpix.com/cropped/2024_11_18_62c85fb0b01b41aca29bg-62.jpg?height=487&width=669&top_left_y=1664&top_left_x=696) + +Figure 7 + +Comment 1. One may notice that the triangle $P Q R$ is also interesting. +Comment 2. In order to prove that $\angle K E F=\angle K F E=\angle A$, one may also use the following well-known fact: +Let $A E F$ be a triangle with $A E \neq A F$, and let $K$ be the common point of the symmedian taken from $A$ and the perpendicular bisector of $E F$. Then the lines $K E$ and $K F$ are tangent to the circumcircle $\omega_{1}$ of the triangle $A E F$. + +In this case, however, one needs to deal with the case $A E=A F$ separately. + +Solution 2. Let $(E, F)$ be an interesting pair. This time we prove that + +$$ +\frac{A M}{A K}=\cos \angle A +$$ + +As in Solution 1, we introduce the circle $\omega$ passing through the points $K, S$, $A$, and $T$, together with the points $N$ and $L$ at which the line $A M$ intersect the line $S T$ and the circle $\omega$ for the second time, respectively. Let moreover $O$ be the centre of $\omega$ (see Figures 8 and 9). As in Solution 1, we note that $N$ is the midpoint of $S T$ and show that $K L \| S T$, which implies $\angle F A M=\angle E A K$. +![](https://cdn.mathpix.com/cropped/2024_11_18_62c85fb0b01b41aca29bg-63.jpg?height=752&width=1009&top_left_y=749&top_left_x=266) + +Figure 8 +![](https://cdn.mathpix.com/cropped/2024_11_18_62c85fb0b01b41aca29bg-63.jpg?height=552&width=549&top_left_y=952&top_left_x=1279) + +Figure 9 + +Suppose now that $K \neq L$ (see Figure 8). Then $K L \| S T$, and consequently the lines $K M$ and $K L$ are perpendicular. It implies that the lines $L O$ and $K M$ meet at a point $X$ lying on the circle $\omega$. Since the lines $O N$ and $X M$ are both perpendicular to the line $S T$, they are parallel to each other, and hence $\angle L O N=\angle L X K=\angle M A K$. On the other hand, $\angle O L N=\angle M K A$, so we infer that triangles $N O L$ and $M A K$ are similar. This yields + +$$ +\frac{A M}{A K}=\frac{O N}{O L}=\frac{O N}{O T}=\cos \angle T O N=\cos \angle A +$$ + +If, on the other hand, $K=L$, then the points $A, M, N$, and $K$ lie on a common line, and this line is the perpendicular bisector of $S T$ (see Figure 9). This implies that $A K$ is a diameter of $\omega$, which yields $A M=2 O K-2 N K=2 O N$. So also in this case we obtain + +$$ +\frac{A M}{A K}=\frac{2 O N}{2 O T}=\cos \angle T O N=\cos \angle A +$$ + +Thus (5) is proved. +Let $P$ and $Q$ be the feet of the perpendiculars from $B$ and $C$ onto $A C$ and $A B$, respectively (see Figure 10). We claim that the point $M$ lies on the line $P Q$. Consider now the composition of the dilatation with factor $\cos \angle A$ and centre $A$, and the reflection with respect to the angle bisector of $\angle B A C$. This transformation is a similarity that takes $B, C$, and $K$ to $P, Q$, and $M$, respectively. Since $K$ lies on the line $B C$, the point $M$ lies on the line $P Q$. +![](https://cdn.mathpix.com/cropped/2024_11_18_62c85fb0b01b41aca29bg-64.jpg?height=715&width=918&top_left_y=219&top_left_x=586) + +Figure 10 +Suppose that $E \neq P$. Then also $F \neq Q$, and by Menelaus' theorem, we obtain + +$$ +\frac{A Q}{F Q} \cdot \frac{F M}{E M} \cdot \frac{E P}{A P}=1 +$$ + +Using the similarity of the triangles $A P Q$ and $A B C$, we infer that + +$$ +\frac{E P}{F Q}=\frac{A P}{A Q}=\frac{A B}{A C}, \quad \text { and hence } \quad \frac{E P}{A B}=\frac{F Q}{A C} +$$ + +The last equality holds obviously also in case $E=P$, because then $F=Q$. Moreover, since the line $P Q$ intersects the segment $E F$, we infer that the point $E$ lies on the segment $A P$ if and only if the point $F$ lies outside of the segment $A Q$. + +Let now $\left(E_{1}, F_{1}\right)$ and $\left(E_{2}, F_{2}\right)$ be two interesting pairs. Then we obtain + +$$ +\frac{E_{1} P}{A B}=\frac{F_{1} Q}{A C} \quad \text { and } \quad \frac{E_{2} P}{A B}=\frac{F_{2} Q}{A C} . +$$ + +If $P$ lies between the points $E_{1}$ and $E_{2}$, we add the equalities above, otherwise we subtract them. In any case we obtain + +$$ +\frac{E_{1} E_{2}}{A B}=\frac{F_{1} F_{2}}{A C} +$$ + +which completes the solution. + +G7. Let $A B C$ be a triangle with circumcircle $\Omega$ and incentre $I$. Let the line passing through $I$ and perpendicular to $C I$ intersect the segment $B C$ and the $\operatorname{arc} B C$ (not containing $A$ ) of $\Omega$ at points $U$ and $V$, respectively. Let the line passing through $U$ and parallel to $A I$ intersect $A V$ at $X$, and let the line passing through $V$ and parallel to $A I$ intersect $A B$ at $Y$. Let $W$ and $Z$ be the midpoints of $A X$ and $B C$, respectively. Prove that if the points $I, X$, and $Y$ are collinear, then the points $I, W$, and $Z$ are also collinear. + +Solution 1. We start with some general observations. Set $\alpha=\angle A / 2, \beta=\angle B / 2, \gamma=\angle C / 2$. Then obviously $\alpha+\beta+\gamma=90^{\circ}$. Since $\angle U I C=90^{\circ}$, we obtain $\angle I U C=\alpha+\beta$. Therefore $\angle B I V=\angle I U C-\angle I B C=\alpha=\angle B A I=\angle B Y V$, which implies that the points $B, Y, I$, and $V$ lie on a common circle (see Figure 1). + +Assume now that the points $I, X$ and $Y$ are collinear. We prove that $\angle Y I A=90^{\circ}$. +Let the line $X U$ intersect $A B$ at $N$. Since the lines $A I, U X$, and $V Y$ are parallel, we get + +$$ +\frac{N X}{A I}=\frac{Y N}{Y A}=\frac{V U}{V I}=\frac{X U}{A I} +$$ + +implying $N X=X U$. Moreover, $\angle B I U=\alpha=\angle B N U$. This implies that the quadrilateral BUIN is cyclic, and since $B I$ is the angle bisector of $\angle U B N$, we infer that $N I=U I$. Thus in the isosceles triangle $N I U$, the point $X$ is the midpoint of the base $N U$. This gives $\angle I X N=90^{\circ}$, i.e., $\angle Y I A=90^{\circ}$. +![](https://cdn.mathpix.com/cropped/2024_11_18_62c85fb0b01b41aca29bg-65.jpg?height=880&width=966&top_left_y=1256&top_left_x=565) + +Figure 1 +Let $S$ be the midpoint of the segment $V C$. Let moreover $T$ be the intersection point of the lines $A X$ and $S I$, and set $x=\angle B A V=\angle B C V$. Since $\angle C I A=90^{\circ}+\beta$ and $S I=S C$, we obtain + +$$ +\angle T I A=180^{\circ}-\angle A I S=90^{\circ}-\beta-\angle C I S=90^{\circ}-\beta-\gamma-x=\alpha-x=\angle T A I, +$$ + +which implies that $T I=T A$. Therefore, since $\angle X I A=90^{\circ}$, the point $T$ is the midpoint of $A X$, i.e., $T=W$. + +To complete our solution, it remains to show that the intersection point of the lines $I S$ and $B C$ coincide with the midpoint of the segment $B C$. But since $S$ is the midpoint of the segment $V C$, it suffices to show that the lines $B V$ and $I S$ are parallel. + +Since the quadrilateral $B Y I V$ is cyclic, $\angle V B I=\angle V Y I=\angle Y I A=90^{\circ}$. This implies that $B V$ is the external angle bisector of the angle $A B C$, which yields $\angle V A C=\angle V C A$. Therefore $2 \alpha-x=2 \gamma+x$, which gives $\alpha=\gamma+x$. Hence $\angle S C I=\alpha$, so $\angle V S I=2 \alpha$. + +On the other hand, $\angle B V C=180^{\circ}-\angle B A C=180^{\circ}-2 \alpha$, which implies that the lines $B V$ and $I S$ are parallel. This completes the solution. + +Solution 2. As in Solution 1, we first prove that the points $B, Y, I, V$ lie on a common circle and $\angle Y I A=90^{\circ}$. The remaining part of the solution is based on the following lemma, which holds true for any triangle $A B C$, not necessarily with the property that $I, X, Y$ are collinear. Lemma. Let $A B C$ be the triangle inscribed in a circle $\Gamma$ and let $I$ be its incentre. Assume that the line passing through $I$ and perpendicular to the line $A I$ intersects the side $A B$ at the point $Y$. Let the circumcircle of the triangle $B Y I$ intersect the circle $\Gamma$ for the second time at $V$, and let the excircle of the triangle $A B C$ opposite to the vertex $A$ be tangent to the side $B C$ at $E$. Then + +$$ +\angle B A V=\angle C A E +$$ + +Proof. Let $\rho$ be the composition of the inversion with centre $A$ and radius $\sqrt{A B \cdot A C}$, and the symmetry with respect to $A I$. Clearly, $\rho$ interchanges $B$ and $C$. + +Let $J$ be the excentre of the triangle $A B C$ opposite to $A$ (see Figure 2). Then we have $\angle J A C=\angle B A I$ and $\angle J C A=90^{\circ}+\gamma=\angle B I A$, so the triangles $A C J$ and $A I B$ are similar, and therefore $A B \cdot A C=A I \cdot A J$. This means that $\rho$ interchanges $I$ and $J$. Moreover, since $Y$ lies on $A B$ and $\angle A I Y=90^{\circ}$, the point $Y^{\prime}=\rho(Y)$ lies on $A C$, and $\angle J Y^{\prime} A=90^{\circ}$. Thus $\rho$ maps the circumcircle $\gamma$ of the triangle $B Y I$ to a circle $\gamma^{\prime}$ with diameter $J C$. + +Finally, since $V$ lies on both $\Gamma$ and $\gamma$, the point $V^{\prime}=\rho(V)$ lies on the line $\rho(\Gamma)=A B$ as well as on $\gamma^{\prime}$, which in turn means that $V^{\prime}=E$. This implies the desired result. +![](https://cdn.mathpix.com/cropped/2024_11_18_62c85fb0b01b41aca29bg-66.jpg?height=709&width=572&top_left_y=1713&top_left_x=248) + +Figure 2 +![](https://cdn.mathpix.com/cropped/2024_11_18_62c85fb0b01b41aca29bg-66.jpg?height=872&width=989&top_left_y=1549&top_left_x=822) + +Figure 3 + +Now we turn to the solution of the problem. +Assume that the incircle $\omega_{1}$ of the triangle $A B C$ is tangent to $B C$ at $D$, and let the excircle $\omega_{2}$ of the triangle $A B C$ opposite to the vertex $A$ touch the side $B C$ at $E$ (see Figure 3). The homothety with centre $A$ that takes $\omega_{2}$ to $\omega_{1}$ takes the point $E$ to some point $F$, and the +tangent to $\omega_{1}$ at $F$ is parallel to $B C$. Therefore $D F$ is a diameter of $\omega_{1}$. Moreover, $Z$ is the midpoint of $D E$. This implies that the lines $I Z$ and $F E$ are parallel. + +Let $K=Y I \cap A E$. Since $\angle Y I A=90^{\circ}$, the lemma yields that $I$ is the midpoint of $X K$. This implies that the segments $I W$ and $A K$ are parallel. Therefore, the points $W, I$ and $Z$ are collinear. + +Comment 1. The properties $\angle Y I A=90^{\circ}$ and $V A=V C$ can be established in various ways. The main difficulty of the problem seems to find out how to use these properties in connection to the points $W$ and $Z$. + +In Solution 2 this principal part is more or less covered by the lemma, for which we have presented a direct proof. On the other hand, this lemma appears to be a combination of two well-known facts; let us formulate them in terms of the lemma statement. + +Let the line $I Y$ intersect $A C$ at $P$ (see Figure 4). The first fact states that the circumcircle $\omega$ of the triangle $V Y P$ is tangent to the segments $A B$ and $A C$, as well as to the circle $\Gamma$. The second fact states that for such a circle, the angles $B A V$ and $C A E$ are equal. + +The awareness of this lemma may help a lot in solving this problem; so the Jury might also consider a variation of the proposed problem, for which the lemma does not seem to be useful; see Comment 3. +![](https://cdn.mathpix.com/cropped/2024_11_18_62c85fb0b01b41aca29bg-67.jpg?height=946&width=1689&top_left_y=1149&top_left_x=189) + +Comment 2. The proposed problem stated the equivalence: the point $I$ lies on the line $X Y$ if and only if $I$ lies on the line $W Z$. Here we sketch the proof of the "if" part (see Figure 5). +As in Solution 2 , let $B C$ touch the circles $\omega_{1}$ and $\omega_{2}$ at $D$ and $E$, respectively. Since $I Z \| A E$ and $W$ lies on $I Z$, the line $D X$ is also parallel to $A E$. Therefore, the triangles $X U P$ and $A I Q$ are similar. Moreover, the line $D X$ is symmetric to $A E$ with respect to $I$, so $I P=I Q$, where $P=U V \cap X D$ and $Q=U V \cap A E$. Thus we obtain + +$$ +\frac{U V}{V I}=\frac{U X}{I A}=\frac{U P}{I Q}=\frac{U P}{I P} +$$ + +So the pairs $I U$ and $P V$ are harmonic conjugates, and since $\angle U D I=90^{\circ}$, we get $\angle V D B=\angle B D X=$ $\angle B E A$. Therefore the point $V^{\prime}$ symmetric to $V$ with respect to the perpendicular bisector of $B C$ lies on the line $A E$. So we obtain $\angle B A V=\angle C A E$. + +The rest can be obtained by simply reversing the arguments in Solution 2 . The points $B, V, I$, and $Y$ are concyclic. The lemma implies that $\angle Y I A=90^{\circ}$. Moreover, the points $B, U, I$, and $N$, where $N=U X \cap A B$, lie on a common circle, so $I N=I U$. Since $I Y \perp U N$, the point $X^{\prime}=I Y \cap U N$ is the midpoint of $U N$. But in the trapezoid $A Y V I$, the line $X U$ is parallel to the sides $A I$ and $Y V$, so $N X=U X^{\prime}$. This yields $X=X^{\prime}$. +The reasoning presented in Solution 1 can also be reversed, but it requires a lot of technicalities. Therefore the Problem Selection Committee proposes to consider only the "only if" part of the original proposal, which is still challenging enough. + +Comment 3. The Jury might also consider the following variation of the proposed problem. +Let $A B C$ be a triangle with circumcircle $\Omega$ and incentre I. Let the line through I perpendicular to CI intersect the segment $B C$ and the arc $B C$ (not containing $A$ ) of $\Omega$ at $U$ and $V$, respectively. Let the line through $U$ parallel to $A I$ intersect $A V$ at $X$. Prove that if the lines XI and AI are perpendicular, then the midpoint of the segment AC lies on the line XI (see Figure 6). +![](https://cdn.mathpix.com/cropped/2024_11_18_62c85fb0b01b41aca29bg-68.jpg?height=675&width=746&top_left_y=1273&top_left_x=184) + +Figure 6 +![](https://cdn.mathpix.com/cropped/2024_11_18_62c85fb0b01b41aca29bg-68.jpg?height=1012&width=980&top_left_y=930&top_left_x=892) + +Figure 7 + +Since the solution contains the arguments used above, we only sketch it. +Let $N=X U \cap A B$ (see Figure 7). Then $\angle B N U=\angle B A I=\angle B I U$, so the points $B, U, I$, and $N$ lie on a common circle. Therefore $I U=I N$, and since $I X \perp N U$, it follows that $N X=X U$. +Now set $Y=X I \cap A B$. The equality $N X=X U$ implies that + +$$ +\frac{V X}{V A}=\frac{X U}{A I}=\frac{N X}{A I}=\frac{Y X}{Y I} +$$ + +and therefore $Y V \| A I$. Hence $\angle B Y V=\angle B A I=\angle B I V$, so the points $B, V, I, Y$ are concyclic. Next we have $I Y \perp Y V$, so $\angle I B V=90^{\circ}$. This implies that $B V$ is the external angle bisector of the angle $A B C$, which gives $\angle V A C=\angle V C A$. +So in order to show that $M=X I \cap A C$ is the midpoint of $A C$, it suffices to prove that $\angle V M C=90^{\circ}$. But this follows immediately from the observation that the points $V, C, M$, and $I$ are concyclic, as $\angle M I V=\angle Y B V=180^{\circ}-\angle A C V$. +The converse statement is also true, but its proof requires some technicalities as well. + +## Number Theory + +N1. Let $n \geqslant 2$ be an integer, and let $A_{n}$ be the set + +$$ +A_{n}=\left\{2^{n}-2^{k} \mid k \in \mathbb{Z}, 0 \leqslant k2$, and take an integer $m>(n-2) 2^{n}+1$. If $m$ is even, then consider + +$$ +\frac{m}{2} \geqslant \frac{(n-2) 2^{n}+2}{2}=(n-2) 2^{n-1}+1>(n-3) 2^{n-1}+1 +$$ + +By the induction hypothesis, there is a representation of the form + +$$ +\frac{m}{2}=\left(2^{n-1}-2^{k_{1}}\right)+\left(2^{n-1}-2^{k_{2}}\right)+\cdots+\left(2^{n-1}-2^{k_{r}}\right) +$$ + +for some $k_{i}$ with $0 \leqslant k_{i}\frac{(n-2) 2^{n}+1-\left(2^{n}-1\right)}{2}=(n-3) 2^{n-1}+1 +$$ + +By the induction hypothesis, there is a representation of the form + +$$ +\frac{m-\left(2^{n}-1\right)}{2}=\left(2^{n-1}-2^{k_{1}}\right)+\left(2^{n-1}-2^{k_{2}}\right)+\cdots+\left(2^{n-1}-2^{k_{r}}\right) +$$ + +for some $k_{i}$ with $0 \leqslant k_{i}2$, assume that there exist integers $a, b$ with $a \geqslant 0, b \geqslant 1$ and $a+by$ due to symmetry. Then the integer $n=x-y$ is positive and (1) may be rewritten as + +$$ +\sqrt[3]{7(y+n)^{2}-13(y+n) y+7 y^{2}}=n+1 +$$ + +Raising this to the third power and simplifying the result one obtains + +$$ +y^{2}+y n=n^{3}-4 n^{2}+3 n+1 . +$$ + +To complete the square on the left hand side, we multiply by 4 and add $n^{2}$, thus getting + +$$ +(2 y+n)^{2}=4 n^{3}-15 n^{2}+12 n+4=(n-2)^{2}(4 n+1) +$$ + +This shows that the cases $n=1$ and $n=2$ are impossible, whence $n>2$, and $4 n+1$ is the square of the rational number $\frac{2 y+n}{n-2}$. Consequently, it has to be a perfect square, and, since it is odd as well, there has to exist some nonnegative integer $m$ such that $4 n+1=(2 m+1)^{2}$, i.e. + +$$ +n=m^{2}+m +$$ + +Notice that $n>2$ entails $m \geqslant 2$. Substituting the value of $n$ just found into the previous displayed equation we arrive at + +$$ +\left(2 y+m^{2}+m\right)^{2}=\left(m^{2}+m-2\right)^{2}(2 m+1)^{2}=\left(2 m^{3}+3 m^{2}-3 m-2\right)^{2} . +$$ + +Extracting square roots and taking $2 m^{3}+3 m^{2}-3 m-2=(m-1)\left(2 m^{2}+5 m+2\right)>0$ into account we derive $2 y+m^{2}+m=2 m^{3}+3 m^{2}-3 m-2$, which in turn yields + +$$ +y=m^{3}+m^{2}-2 m-1 +$$ + +Notice that $m \geqslant 2$ implies that $y=\left(m^{3}-1\right)+(m-2) m$ is indeed positive, as it should be. In view of $x=y+n=y+m^{2}+m$ it also follows that + +$$ +x=m^{3}+2 m^{2}-m-1, +$$ + +and that this integer is positive as well. +Comment. Alternatively one could ask to find all pairs $(x, y)$ of - not necessarily positive - integers solving (1). The answer to that question is a bit nicer than the answer above: the set of solutions are now described by + +$$ +\{x, y\}=\left\{m^{3}+m^{2}-2 m-1, m^{3}+2 m^{2}-m-1\right\} +$$ + +where $m$ varies through $\mathbb{Z}$. This may be shown using essentially the same arguments as above. We finally observe that the pair $(x, y)=(1,1)$, that appears to be sporadic above, corresponds to $m=-1$. + +N3. A coin is called a Cape Town coin if its value is $1 / n$ for some positive integer $n$. Given a collection of Cape Town coins of total value at most $99+\frac{1}{2}$, prove that it is possible to split this collection into at most 100 groups each of total value at most 1. +(Luxembourg) +Solution. We will show that for every positive integer $N$ any collection of Cape Town coins of total value at most $N-\frac{1}{2}$ can be split into $N$ groups each of total value at most 1 . The problem statement is a particular case for $N=100$. + +We start with some preparations. If several given coins together have a total value also of the form $\frac{1}{k}$ for a positive integer $k$, then we may merge them into one new coin. Clearly, if the resulting collection can be split in the required way then the initial collection can also be split. + +After each such merging, the total number of coins decreases, thus at some moment we come to a situation when no more merging is possible. At this moment, for every even $k$ there is at most one coin of value $\frac{1}{k}$ (otherwise two such coins may be merged), and for every odd $k>1$ there are at most $k-1$ coins of value $\frac{1}{k}$ (otherwise $k$ such coins may also be merged). + +Now, clearly, each coin of value 1 should form a single group; if there are $d$ such coins then we may remove them from the collection and replace $N$ by $N-d$. So from now on we may assume that there are no coins of value 1. + +Finally, we may split all the coins in the following way. For each $k=1,2, \ldots, N$ we put all the coins of values $\frac{1}{2 k-1}$ and $\frac{1}{2 k}$ into a group $G_{k}$; the total value of $G_{k}$ does not exceed + +$$ +(2 k-2) \cdot \frac{1}{2 k-1}+\frac{1}{2 k}<1 +$$ + +It remains to distribute the "small" coins of values which are less than $\frac{1}{2 N}$; we will add them one by one. In each step, take any remaining small coin. The total value of coins in the groups at this moment is at most $N-\frac{1}{2}$, so there exists a group of total value at most $\frac{1}{N}\left(N-\frac{1}{2}\right)=1-\frac{1}{2 N}$; thus it is possible to put our small coin into this group. Acting so, we will finally distribute all the coins. + +Comment 1. The algorithm may be modified, at least the step where one distributes the coins of values $\geqslant \frac{1}{2 N}$. One different way is to put into $G_{k}$ all the coins of values $\frac{1}{(2 k-1) 2^{s}}$ for all integer $s \geqslant 0$. One may easily see that their total value also does not exceed 1. + +Comment 2. The original proposal also contained another part, suggesting to show that a required splitting may be impossible if the total value of coins is at most 100 . There are many examples of such a collection, e.g. one may take 98 coins of value 1 , one coin of value $\frac{1}{2}$, two coins of value $\frac{1}{3}$, and four coins of value $\frac{1}{5}$. + +The Problem Selection Committee thinks that this part is less suitable for the competition. + +N4. Let $n>1$ be a given integer. Prove that infinitely many terms of the sequence $\left(a_{k}\right)_{k \geqslant 1}$, defined by + +$$ +a_{k}=\left\lfloor\frac{n^{k}}{k}\right\rfloor +$$ + +are odd. (For a real number $x,\lfloor x\rfloor$ denotes the largest integer not exceeding $x$.) +(Hong Kong) +Solution 1. If $n$ is odd, let $k=n^{m}$ for $m=1,2, \ldots$. Then $a_{k}=n^{n^{m}-m}$, which is odd for each $m$. + +Henceforth, assume that $n$ is even, say $n=2 t$ for some integer $t \geqslant 1$. Then, for any $m \geqslant 2$, the integer $n^{2^{m}}-2^{m}=2^{m}\left(2^{2^{m}-m} \cdot t^{2^{m}}-1\right)$ has an odd prime divisor $p$, since $2^{m}-m>1$. Then, for $k=p \cdot 2^{m}$, we have + +$$ +n^{k}=\left(n^{2^{m}}\right)^{p} \equiv\left(2^{m}\right)^{p}=\left(2^{p}\right)^{m} \equiv 2^{m} +$$ + +where the congruences are taken modulo $p\left(\right.$ recall that $2^{p} \equiv 2(\bmod p)$, by Fermat's little theorem). Also, from $n^{k}-2^{m}m$ ). Note that for different values of $m$, we get different values of $k$, due to the different powers of 2 in the prime factorisation of $k$. + +Solution 2. Treat the (trivial) case when $n$ is odd as in Solution 1. +Now assume that $n$ is even and $n>2$. Let $p$ be a prime divisor of $n-1$. +Proceed by induction on $i$ to prove that $p^{i+1}$ is a divisor of $n^{p^{i}}-1$ for every $i \geqslant 0$. The case $i=0$ is true by the way in which $p$ is chosen. Suppose the result is true for some $i \geqslant 0$. The factorisation + +$$ +n^{p^{i+1}}-1=\left(n^{p^{i}}-1\right)\left[n^{p^{i}(p-1)}+n^{p^{i}(p-2)}+\cdots+n^{p^{i}}+1\right], +$$ + +together with the fact that each of the $p$ terms between the square brackets is congruent to 1 modulo $p$, implies that the result is also true for $i+1$. + +Hence $\left\lfloor\frac{n^{p^{i}}}{p^{i}}\right\rfloor=\frac{n^{p^{i}}-1}{p^{i}}$, an odd integer for each $i \geqslant 1$. +Finally, we consider the case $n=2$. We observe that $3 \cdot 4^{i}$ is a divisor of $2^{3 \cdot 4^{i}}-4^{i}$ for every $i \geqslant 1$ : Trivially, $4^{i}$ is a divisor of $2^{3 \cdot 4^{i}}-4^{i}$, since $3 \cdot 4^{i}>2 i$. Furthermore, since $2^{3 \cdot 4^{i}}$ and $4^{i}$ are both congruent to 1 modulo 3, we have $3 \mid 2^{3 \cdot 4^{i}}-4^{i}$. Hence, $\left\lfloor\frac{2^{3 \cdot 4^{i}}}{3 \cdot 4^{i}}\right\rfloor=\frac{2^{3 \cdot 4^{i}}-4^{i}}{3 \cdot 4^{i}}=\frac{2^{3 \cdot 4^{i}-2 i}-1}{3}$, which is odd for every $i \geqslant 1$. + +Comment. The case $n$ even and $n>2$ can also be solved by recursively defining the sequence $\left(k_{i}\right)_{i \geqslant 1}$ by $k_{1}=1$ and $k_{i+1}=n^{k_{i}}-1$ for $i \geqslant 1$. Then $\left(k_{i}\right)$ is strictly increasing and it follows (by induction on $i$ ) that $k_{i} \mid n^{k_{i}}-1$ for all $i \geqslant 1$, so the $k_{i}$ are as desired. + +The case $n=2$ can also be solved as follows: Let $i \geqslant 2$. By Bertrand's postulate, there exists a prime number $p$ such that $2^{2^{i}-1}

0}: 2^{i} \text { divides } a p+1\right\} +$$ + +Recall that there exists $a$ with $1 \leqslant a<2^{i}$ such that $a p \equiv-1\left(\bmod 2^{i}\right)$, so each $a_{i}$ satisfies $1 \leqslant a_{i}<2^{i}$. This implies that $a_{i} p+1

2$, and we let $a$ and $b$ be positive integers such that $x^{p-1}+y=p^{a}$ and $x+y^{p-1}=p^{b}$. Assume further, without loss of generality, that $x \leqslant y$, so that $p^{a}=x^{p-1}+y \leqslant x+y^{p-1}=p^{b}$, which means that $a \leqslant b$ (and thus $\left.p^{a} \mid p^{b}\right)$. + +Now we have + +$$ +p^{b}=y^{p-1}+x=\left(p^{a}-x^{p-1}\right)^{p-1}+x . +$$ + +We take this equation modulo $p^{a}$ and take into account that $p-1$ is even, which gives us + +$$ +0 \equiv x^{(p-1)^{2}}+x \quad\left(\bmod p^{a}\right) +$$ + +If $p \mid x$, then $p^{a} \mid x$, since $x^{(p-1)^{2}-1}+1$ is not divisible by $p$ in this case. However, this is impossible, since $x \leqslant x^{p-1}2$. Thus $a=r+1$. Now since $p^{r} \leqslant x+1$, we get + +$$ +x=\frac{x^{2}+x}{x+1} \leqslant \frac{x^{p-1}+y}{x+1}=\frac{p^{a}}{x+1} \leqslant \frac{p^{a}}{p^{r}}=p, +$$ + +so we must have $x=p-1$ for $p$ to divide $x+1$. +It follows that $r=1$ and $a=2$. If $p \geqslant 5$, we obtain + +$$ +p^{a}=x^{p-1}+y>(p-1)^{4}=\left(p^{2}-2 p+1\right)^{2}>(3 p)^{2}>p^{2}=p^{a} +$$ + +a contradiction. So the only case that remains is $p=3$, and indeed $x=2$ and $y=p^{a}-x^{p-1}=5$ satisfy the conditions. + +Comment 1. In this solution, we are implicitly using a special case of the following lemma known as "lifting the exponent": +Lemma. Let $n$ be a positive integer, let $p$ be an odd prime, and let $v_{p}(m)$ denote the exponent of the highest power of $p$ that divides $m$. + +If $x$ and $y$ are integers not divisible by $p$ such that $p \mid x-y$, then we have + +$$ +v_{p}\left(x^{n}-y^{n}\right)=v_{p}(x-y)+v_{p}(n) +$$ + +Likewise, if $x$ and $y$ are integers not divisible by $p$ such that $p \mid x+y$, then we have + +$$ +v_{p}\left(x^{n}+y^{n}\right)=v_{p}(x+y)+v_{p}(n) . +$$ + +Comment 2. There exist various ways of solving the problem involving the "lifting the exponent" lemma. Let us sketch another one. + +The cases $x=y$ and $p \mid x$ are ruled out easily, so we assume that $p>2, x2$. If $p \mid x$, then also $p \mid y$. In this case, let $p^{k}$ and $p^{\ell}$ be the highest powers of $p$ that divide $x$ and $y$ respectively, and assume without loss of generality that $k \leqslant \ell$. Then $p^{k}$ divides $x+y^{p-1}$ while $p^{k+1}$ does not, but $p^{k}p$, so $x^{p-1}+y$ and $y^{p-1}+x$ are both at least equal to $p^{2}$. Now we have + +$$ +x^{p-1} \equiv-y \quad\left(\bmod p^{2}\right) \quad \text { and } \quad y^{p-1} \equiv-x \quad\left(\bmod p^{2}\right) +$$ + +These two congruences, together with the Euler-Fermat theorem, give us + +$$ +1 \equiv x^{p(p-1)} \equiv(-y)^{p} \equiv-y^{p} \equiv x y \quad\left(\bmod p^{2}\right) +$$ + +Since $x \equiv y \equiv-1(\bmod p), x-y$ is divisible by $p$, so $(x-y)^{2}$ is divisible by $p^{2}$. This means that + +$$ +(x+y)^{2}=(x-y)^{2}+4 x y \equiv 4 \quad\left(\bmod p^{2}\right) +$$ + +so $p^{2}$ divides $(x+y-2)(x+y+2)$. We already know that $x+y \equiv-2(\bmod p)$, so $x+y-2 \equiv$ $-4 \not \equiv 0(\bmod p)$. This means that $p^{2}$ divides $x+y+2$. + +Using the same notation as in the first solution, we subtract the two original equations to obtain + +$$ +p^{b}-p^{a}=y^{p-1}-x^{p-1}+x-y=(y-x)\left(y^{p-2}+y^{p-3} x+\cdots+x^{p-2}-1\right) +$$ + +The second factor is symmetric in $x$ and $y$, so it can be written as a polynomial of the elementary symmetric polynomials $x+y$ and $x y$ with integer coefficients. In particular, its value modulo +$p^{2}$ is characterised by the two congruences $x y \equiv 1\left(\bmod p^{2}\right)$ and $x+y \equiv-2\left(\bmod p^{2}\right)$. Since both congruences are satisfied when $x=y=-1$, we must have + +$$ +y^{p-2}+y^{p-3} x+\cdots+x^{p-2}-1 \equiv(-1)^{p-2}+(-1)^{p-3}(-1)+\cdots+(-1)^{p-2}-1 \quad\left(\bmod p^{2}\right), +$$ + +which simplifies to $y^{p-2}+y^{p-3} x+\cdots+x^{p-2}-1 \equiv-p\left(\bmod p^{2}\right)$. Thus the second factor in (1) is divisible by $p$, but not $p^{2}$. + +This means that $p^{a-1}$ has to divide the other factor $y-x$. It follows that + +$$ +0 \equiv x^{p-1}+y \equiv x^{p-1}+x \equiv x(x+1)\left(x^{p-3}-x^{p-4}+\cdots+1\right) \quad\left(\bmod p^{a-1}\right) . +$$ + +Since $x \equiv-1(\bmod p)$, the last factor is $x^{p-3}-x^{p-4}+\cdots+1 \equiv p-2(\bmod p)$ and in particular not divisible by $p$. We infer that $p^{a-1} \mid x+1$ and continue as in the first solution. + +Comment. Instead of reasoning by means of elementary symmetric polynomials, it is possible to provide a more direct argument as well. For odd $r,(x+1)^{2}$ divides $\left(x^{r}+1\right)^{2}$, and since $p$ divides $x+1$, we deduce that $p^{2}$ divides $\left(x^{r}+1\right)^{2}$. Together with the fact that $x y \equiv 1\left(\bmod p^{2}\right)$, we obtain + +$$ +0 \equiv y^{r}\left(x^{r}+1\right)^{2} \equiv x^{2 r} y^{r}+2 x^{r} y^{r}+y^{r} \equiv x^{r}+2+y^{r} \quad\left(\bmod p^{2}\right) . +$$ + +We apply this congruence with $r=p-2-2 k$ (where $0 \leqslant k<(p-2) / 2$ ) to find that + +$$ +x^{k} y^{p-2-k}+x^{p-2-k} y^{k} \equiv(x y)^{k}\left(x^{p-2-2 k}+y^{p-2-2 k}\right) \equiv 1^{k} \cdot(-2) \equiv-2 \quad\left(\bmod p^{2}\right) . +$$ + +Summing over all $k$ yields + +$$ +y^{p-2}+y^{p-3} x+\cdots+x^{p-2}-1 \equiv \frac{p-1}{2} \cdot(-2)-1 \equiv-p \quad\left(\bmod p^{2}\right) +$$ + +once again. + +N6. Let $a_{1}x_{n} +$$ + +holds for all integers $n \geqslant 0$, it is also strictly increasing. Since $x_{n+1}$ is by (1) coprime to $c$ for any $n \geqslant 0$, it suffices to prove that for each $n \geqslant 2$ there exists a prime number $p$ dividing $x_{n}$ but none of the numbers $x_{1}, \ldots, x_{n-1}$. Let us begin by establishing three preliminary claims. +Claim 1. If $i \equiv j(\bmod m)$ holds for some integers $i, j \geqslant 0$ and $m \geqslant 1$, then $x_{i} \equiv x_{j}\left(\bmod x_{m}\right)$ holds as well. +Proof. Evidently, it suffices to show $x_{i+m} \equiv x_{i}\left(\bmod x_{m}\right)$ for all integers $i \geqslant 0$ and $m \geqslant 1$. For this purpose we may argue for fixed $m$ by induction on $i$ using $x_{0}=0$ in the base case $i=0$. Now, if we have $x_{i+m} \equiv x_{i}\left(\bmod x_{m}\right)$ for some integer $i$, then the recursive equation (1) yields + +$$ +x_{i+m+1} \equiv c^{2}\left(x_{i+m}^{3}-4 x_{i+m}^{2}+5 x_{i+m}\right)+1 \equiv c^{2}\left(x_{i}^{3}-4 x_{i}^{2}+5 x_{i}\right)+1 \equiv x_{i+1} \quad\left(\bmod x_{m}\right), +$$ + +which completes the induction. +Claim 2. If the integers $i, j \geqslant 2$ and $m \geqslant 1$ satisfy $i \equiv j(\bmod m)$, then $x_{i} \equiv x_{j}\left(\bmod x_{m}^{2}\right)$ holds as well. +Proof. Again it suffices to prove $x_{i+m} \equiv x_{i}\left(\bmod x_{m}^{2}\right)$ for all integers $i \geqslant 2$ and $m \geqslant 1$. As above, we proceed for fixed $m$ by induction on $i$. The induction step is again easy using (1), but this time the base case $i=2$ requires some calculation. Set $L=5 c^{2}$. By (1) we have $x_{m+1} \equiv L x_{m}+1\left(\bmod x_{m}^{2}\right)$, and hence + +$$ +\begin{aligned} +x_{m+1}^{3}-4 x_{m+1}^{2}+5 x_{m+1} & \equiv\left(L x_{m}+1\right)^{3}-4\left(L x_{m}+1\right)^{2}+5\left(L x_{m}+1\right) \\ +& \equiv\left(3 L x_{m}+1\right)-4\left(2 L x_{m}+1\right)+5\left(L x_{m}+1\right) \equiv 2 \quad\left(\bmod x_{m}^{2}\right) +\end{aligned} +$$ + +which in turn gives indeed $x_{m+2} \equiv 2 c^{2}+1 \equiv x_{2}\left(\bmod x_{m}^{2}\right)$. +Claim 3. For each integer $n \geqslant 2$, we have $x_{n}>x_{1} \cdot x_{2} \cdots x_{n-2}$. +Proof. The cases $n=2$ and $n=3$ are clear. Arguing inductively, we assume now that the claim holds for some $n \geqslant 3$. Recall that $x_{2} \geqslant 3$, so by monotonicity and (2) we get $x_{n} \geqslant x_{3} \geqslant x_{2}\left(x_{2}-2\right)^{2}+x_{2}+1 \geqslant 7$. It follows that + +$$ +x_{n+1}>x_{n}^{3}-4 x_{n}^{2}+5 x_{n}>7 x_{n}^{2}-4 x_{n}^{2}>x_{n}^{2}>x_{n} x_{n-1} +$$ + +which by the induction hypothesis yields $x_{n+1}>x_{1} \cdot x_{2} \cdots x_{n-1}$, as desired. + +Now we direct our attention to the problem itself: let any integer $n \geqslant 2$ be given. By Claim 3 there exists a prime number $p$ appearing with a higher exponent in the prime factorisation of $x_{n}$ than in the prime factorisation of $x_{1} \cdots x_{n-2}$. In particular, $p \mid x_{n}$, and it suffices to prove that $p$ divides none of $x_{1}, \ldots, x_{n-1}$. + +Otherwise let $k \in\{1, \ldots, n-1\}$ be minimal such that $p$ divides $x_{k}$. Since $x_{n-1}$ and $x_{n}$ are coprime by (1) and $x_{1}=1$, we actually have $2 \leqslant k \leqslant n-2$. Write $n=q k+r$ with some integers $q \geqslant 0$ and $0 \leqslant r1$, so there exists a prime $p$ with $v_{p}(N)>0$. Since $N$ is a fraction of two odd numbers, $p$ is odd. + +By our lemma, + +$$ +0\frac{1}{2}, \quad \text { or } \quad\{x\}>\frac{1}{2}, \quad\{y\}>\frac{1}{2}, \quad\{x+y\}<\frac{1}{2}, +$$ + +where $\{x\}$ denotes the fractional part of $x$. +In the context of our problem, the first condition seems easier to deal with. Also, one may notice that + +$$ +\{x\}<\frac{1}{2} \Longleftrightarrow \varkappa(x)=0 \quad \text { and } \quad\{x\} \geqslant \frac{1}{2} \Longleftrightarrow \varkappa(x)=1, +$$ + +where + +$$ +\varkappa(x)=\lfloor 2 x\rfloor-2\lfloor x\rfloor . +$$ + +Now it is natural to consider the number + +$$ +M=\frac{\binom{2 a+2 b}{a+b}}{\binom{2 a}{a}\binom{2 b}{b}} +$$ + +since + +$$ +v_{p}(M)=\sum_{k=1}^{\infty}\left(\varkappa\left(\frac{2(a+b)}{p^{k}}\right)-\varkappa\left(\frac{2 a}{p^{k}}\right)-\varkappa\left(\frac{2 b}{p^{k}}\right)\right) . +$$ + +One may see that $M>1$, and that $v_{2}(M) \leqslant 0$. Thus, there exist an odd prime $p$ and a positive integer $k$ with + +$$ +\varkappa\left(\frac{2(a+b)}{p^{k}}\right)-\varkappa\left(\frac{2 a}{p^{k}}\right)-\varkappa\left(\frac{2 b}{p^{k}}\right)>0 . +$$ + +In view of (4), the last inequality yields + +$$ +\left\{\frac{a}{p^{k}}\right\}<\frac{1}{2}, \quad\left\{\frac{b}{p^{k}}\right\}<\frac{1}{2}, \quad \text { and } \quad\left\{\frac{a+b}{p^{k}}\right\}>\frac{1}{2} +$$ + +which is what we wanted to obtain. +Comment 2. Once one tries to prove the existence of suitable $p$ and $k$ satisfying (5), it seems somehow natural to suppose that $a \leqslant b$ and to add the restriction $p^{k}>a$. In this case the inequalities (5) can be rewritten as + +$$ +2 ak$. We would like to mention here that Sylvester's theorem itself does not seem to suffice for solving the problem. + diff --git a/IMO/md/en-IMO2015SL.md b/IMO/md/en-IMO2015SL.md new file mode 100644 index 0000000000000000000000000000000000000000..88e96b991bbdac35f5dd9fdffead6993b47498b8 --- /dev/null +++ b/IMO/md/en-IMO2015SL.md @@ -0,0 +1,2600 @@ +# Shortlisted Problems with Solutions + +## $(\sqrt{(1)}$
$56^{\text {th }}$
International Mathematical Olympiad + +![](https://cdn.mathpix.com/cropped/2024_11_18_62dbd531c72d0ff8164cg-01.jpg?height=1075&width=1626&top_left_y=1738&top_left_x=455) + +## Shortlisted Problems with Solutions + +$56^{\text {th }}$ International Mathematical Olympiad +Chiang Mai, Thailand, 4-16 +![](https://cdn.mathpix.com/cropped/2024_11_18_62dbd531c72d0ff8164cg-02.jpg?height=641&width=872&top_left_y=1741&top_left_x=595) + +## The shortlisted problems should be kept strictly confidential until IMO 2016. + +## Contributing Countries + +The Organizing Committee and the Problem Selection Committee of IMO 2015 thank the following 53 countries for contributing 155 problem proposals: + +Albania, Algeria, Armenia, Australia, Austria, Brazil, Bulgaria, Canada, Costa Rica, Croatia, Cyprus, Denmark, El Salvador, Estonia, Finland, France, Georgia, Germany, Greece, Hong Kong, Hungary, India, Iran, Ireland, Israel, Italy, Japan, Kazakhstan, Lithuania, Luxembourg, Montenegro, Morocco, Netherlands, Pakistan, Poland, Romania, Russia, Saudi Arabia, Serbia, Singapore, Slovakia, Slovenia, South Africa, South Korea, Sweden, Turkey, Turkmenistan, Taiwan, Tanzania, Ukraine, United Kingdom, U.S.A., Uzbekistan + +## Problem Selection Committee + +![](https://cdn.mathpix.com/cropped/2024_11_18_62dbd531c72d0ff8164cg-03.jpg?height=907&width=1328&top_left_y=1551&top_left_x=364) + +Dungjade Shiowattana, Ilya I. Bogdanov, Tirasan Khandhawit, Wittawat Kositwattanarerk, Géza Kós, Weerachai Neeranartvong, Nipun Pitimanaaree, Christian Reiher, Nat Sothanaphan, Warut Suksompong, Wuttisak Trongsiriwat, Wijit Yangjit + +Assistants: Jirawat Anunrojwong, Pakawut Jiradilok + +## Problems + +## Algebra + +A1. Suppose that a sequence $a_{1}, a_{2}, \ldots$ of positive real numbers satisfies + +$$ +a_{k+1} \geqslant \frac{k a_{k}}{a_{k}^{2}+(k-1)} +$$ + +for every positive integer $k$. Prove that $a_{1}+a_{2}+\cdots+a_{n} \geqslant n$ for every $n \geqslant 2$. +A2. Determine all functions $f: \mathbb{Z} \rightarrow \mathbb{Z}$ with the property that + +$$ +f(x-f(y))=f(f(x))-f(y)-1 +$$ + +holds for all $x, y \in \mathbb{Z}$. +(Croatia) +A3. Let $n$ be a fixed positive integer. Find the maximum possible value of + +$$ +\sum_{1 \leqslant rm \geqslant N$. + +C6. Let $S$ be a nonempty set of positive integers. We say that a positive integer $n$ is clean if it has a unique representation as a sum of an odd number of distinct elements from $S$. Prove that there exist infinitely many positive integers that are not clean. + +C7. In a company of people some pairs are enemies. A group of people is called unsociable if the number of members in the group is odd and at least 3 , and it is possible to arrange all its members around a round table so that every two neighbors are enemies. Given that there are at most 2015 unsociable groups, prove that it is possible to partition the company into 11 parts so that no two enemies are in the same part. + +## Geometry + +G1. Let $A B C$ be an acute triangle with orthocenter $H$. Let $G$ be the point such that the quadrilateral $A B G H$ is a parallelogram. Let $I$ be the point on the line $G H$ such that $A C$ bisects $H I$. Suppose that the line $A C$ intersects the circumcircle of the triangle $G C I$ at $C$ and $J$. Prove that $I J=A H$. +(Australia) +G2. Let $A B C$ be a triangle inscribed into a circle $\Omega$ with center $O$. A circle $\Gamma$ with center $A$ meets the side $B C$ at points $D$ and $E$ such that $D$ lies between $B$ and $E$. Moreover, let $F$ and $G$ be the common points of $\Gamma$ and $\Omega$. We assume that $F$ lies on the arc $A B$ of $\Omega$ not containing $C$, and $G$ lies on the arc $A C$ of $\Omega$ not containing $B$. The circumcircles of the triangles $B D F$ and $C E G$ meet the sides $A B$ and $A C$ again at $K$ and $L$, respectively. Suppose that the lines $F K$ and $G L$ are distinct and intersect at $X$. Prove that the points $A, X$, and $O$ are collinear. +(Greece) +G3. Let $A B C$ be a triangle with $\angle C=90^{\circ}$, and let $H$ be the foot of the altitude from $C$. A point $D$ is chosen inside the triangle $C B H$ so that $C H$ bisects $A D$. Let $P$ be the intersection point of the lines $B D$ and $C H$. Let $\omega$ be the semicircle with diameter $B D$ that meets the segment $C B$ at an interior point. A line through $P$ is tangent to $\omega$ at $Q$. Prove that the lines $C Q$ and $A D$ meet on $\omega$. +(Georgia) +G4. Let $A B C$ be an acute triangle, and let $M$ be the midpoint of $A C$. A circle $\omega$ passing through $B$ and $M$ meets the sides $A B$ and $B C$ again at $P$ and $Q$, respectively. Let $T$ be the point such that the quadrilateral $B P T Q$ is a parallelogram. Suppose that $T$ lies on the circumcircle of the triangle $A B C$. Determine all possible values of $B T / B M$. +(Russia) +G5. Let $A B C$ be a triangle with $C A \neq C B$. Let $D, F$, and $G$ be the midpoints of the sides $A B, A C$, and $B C$, respectively. A circle $\Gamma$ passing through $C$ and tangent to $A B$ at $D$ meets the segments $A F$ and $B G$ at $H$ and $I$, respectively. The points $H^{\prime}$ and $I^{\prime}$ are symmetric to $H$ and $I$ about $F$ and $G$, respectively. The line $H^{\prime} I^{\prime}$ meets $C D$ and $F G$ at $Q$ and $M$, respectively. The line $C M$ meets $\Gamma$ again at $P$. Prove that $C Q=Q P$. +(El Salvador) +G6. Let $A B C$ be an acute triangle with $A B>A C$, and let $\Gamma$ be its circumcircle. Let $H$, $M$, and $F$ be the orthocenter of the triangle, the midpoint of $B C$, and the foot of the altitude from $A$, respectively. Let $Q$ and $K$ be the two points on $\Gamma$ that satisfy $\angle A Q H=90^{\circ}$ and $\angle Q K H=90^{\circ}$. Prove that the circumcircles of the triangles $K Q H$ and $K F M$ are tangent to each other. +(Ukraine) +G7. Let $A B C D$ be a convex quadrilateral, and let $P, Q, R$, and $S$ be points on the sides $A B, B C, C D$, and $D A$, respectively. Let the line segments $P R$ and $Q S$ meet at $O$. Suppose that each of the quadrilaterals $A P O S, B Q O P, C R O Q$, and $D S O R$ has an incircle. Prove that the lines $A C, P Q$, and $R S$ are either concurrent or parallel to each other. +(Bulgaria) +G8. A triangulation of a convex polygon $\Pi$ is a partitioning of $\Pi$ into triangles by diagonals having no common points other than the vertices of the polygon. We say that a triangulation is a Thaiangulation if all triangles in it have the same area. + +Prove that any two different Thaiangulations of a convex polygon $\Pi$ differ by exactly two triangles. (In other words, prove that it is possible to replace one pair of triangles in the first Thaiangulation with a different pair of triangles so as to obtain the second Thaiangulation.) +(Bulgaria) + +## Number Theory + +N1. Determine all positive integers $M$ for which the sequence $a_{0}, a_{1}, a_{2}, \ldots$, defined by $a_{0}=\frac{2 M+1}{2}$ and $a_{k+1}=a_{k}\left\lfloor a_{k}\right\rfloor$ for $k=0,1,2, \ldots$, contains at least one integer term. +(Luxembourg) +N2. Let $a$ and $b$ be positive integers such that $a!b$ ! is a multiple of $a!+b!$. Prove that $3 a \geqslant 2 b+2$. +(United Kingdom) +N3. Let $m$ and $n$ be positive integers such that $m>n$. Define $x_{k}=(m+k) /(n+k)$ for $k=$ $1,2, \ldots, n+1$. Prove that if all the numbers $x_{1}, x_{2}, \ldots, x_{n+1}$ are integers, then $x_{1} x_{2} \cdots x_{n+1}-1$ is divisible by an odd prime. +(Austria) +N4. Suppose that $a_{0}, a_{1}, \ldots$ and $b_{0}, b_{1}, \ldots$ are two sequences of positive integers satisfying $a_{0}, b_{0} \geqslant 2$ and + +$$ +a_{n+1}=\operatorname{gcd}\left(a_{n}, b_{n}\right)+1, \quad b_{n+1}=\operatorname{lcm}\left(a_{n}, b_{n}\right)-1 +$$ + +for all $n \geqslant 0$. Prove that the sequence $\left(a_{n}\right)$ is eventually periodic; in other words, there exist integers $N \geqslant 0$ and $t>0$ such that $a_{n+t}=a_{n}$ for all $n \geqslant N$. +(France) +N5. Determine all triples $(a, b, c)$ of positive integers for which $a b-c, b c-a$, and $c a-b$ are powers of 2 . + +Explanation: A power of 2 is an integer of the form $2^{n}$, where $n$ denotes some nonnegative integer. +(Serbia) +N6. Let $\mathbb{Z}_{>0}$ denote the set of positive integers. Consider a function $f: \mathbb{Z}_{>0} \rightarrow \mathbb{Z}_{>0}$. For any $m, n \in \mathbb{Z}_{>0}$ we write $f^{n}(m)=\underbrace{f(f(\ldots f}_{n}(m) \ldots))$. Suppose that $f$ has the following two properties: +(i) If $m, n \in \mathbb{Z}_{>0}$, then $\frac{f^{n}(m)-m}{n} \in \mathbb{Z}_{>0}$; +(ii) The set $\mathbb{Z}_{>0} \backslash\left\{f(n) \mid n \in \mathbb{Z}_{>0}\right\}$ is finite. + +Prove that the sequence $f(1)-1, f(2)-2, f(3)-3, \ldots$ is periodic. +(Singapore) +N7. Let $\mathbb{Z}_{>0}$ denote the set of positive integers. For any positive integer $k$, a function $f: \mathbb{Z}_{>0} \rightarrow \mathbb{Z}_{>0}$ is called $k$-good if $\operatorname{gcd}(f(m)+n, f(n)+m) \leqslant k$ for all $m \neq n$. Find all $k$ such that there exists a $k$-good function. +(Canada) +N8. For every positive integer $n$ with prime factorization $n=\prod_{i=1}^{k} p_{i}^{\alpha_{i}}$, define + +$$ +\mho(n)=\sum_{i: p_{i}>10^{100}} \alpha_{i} . +$$ + +That is, $\mho(n)$ is the number of prime factors of $n$ greater than $10^{100}$, counted with multiplicity. +Find all strictly increasing functions $f: \mathbb{Z} \rightarrow \mathbb{Z}$ such that + +$$ +\mho(f(a)-f(b)) \leqslant \mho(a-b) \quad \text { for all integers } a \text { and } b \text { with } a>b . +$$ + +## Solutions + +## Algebra + +A1. Suppose that a sequence $a_{1}, a_{2}, \ldots$ of positive real numbers satisfies + +$$ +a_{k+1} \geqslant \frac{k a_{k}}{a_{k}^{2}+(k-1)} +$$ + +for every positive integer $k$. Prove that $a_{1}+a_{2}+\cdots+a_{n} \geqslant n$ for every $n \geqslant 2$. + +Solution. From the constraint (1), it can be seen that + +$$ +\frac{k}{a_{k+1}} \leqslant \frac{a_{k}^{2}+(k-1)}{a_{k}}=a_{k}+\frac{k-1}{a_{k}} +$$ + +and so + +$$ +a_{k} \geqslant \frac{k}{a_{k+1}}-\frac{k-1}{a_{k}} . +$$ + +Summing up the above inequality for $k=1, \ldots, m$, we obtain + +$$ +a_{1}+a_{2}+\cdots+a_{m} \geqslant\left(\frac{1}{a_{2}}-\frac{0}{a_{1}}\right)+\left(\frac{2}{a_{3}}-\frac{1}{a_{2}}\right)+\cdots+\left(\frac{m}{a_{m+1}}-\frac{m-1}{a_{m}}\right)=\frac{m}{a_{m+1}} +$$ + +Now we prove the problem statement by induction on $n$. The case $n=2$ can be done by applying (1) to $k=1$ : + +$$ +a_{1}+a_{2} \geqslant a_{1}+\frac{1}{a_{1}} \geqslant 2 +$$ + +For the induction step, assume that the statement is true for some $n \geqslant 2$. If $a_{n+1} \geqslant 1$, then the induction hypothesis yields + +$$ +\left(a_{1}+\cdots+a_{n}\right)+a_{n+1} \geqslant n+1 +$$ + +Otherwise, if $a_{n+1}<1$ then apply (2) as + +$$ +\left(a_{1}+\cdots+a_{n}\right)+a_{n+1} \geqslant \frac{n}{a_{n+1}}+a_{n+1}=\frac{n-1}{a_{n+1}}+\left(\frac{1}{a_{n+1}}+a_{n+1}\right)>(n-1)+2 +$$ + +That completes the solution. +Comment 1. It can be seen easily that having equality in the statement requires $a_{1}=a_{2}=1$ in the base case $n=2$, and $a_{n+1}=1$ in (3). So the equality $a_{1}+\cdots+a_{n}=n$ is possible only in the trivial case $a_{1}=\cdots=a_{n}=1$. + +Comment 2. After obtaining (2), there are many ways to complete the solution. We outline three such possibilities. + +- With defining $s_{n}=a_{1}+\cdots+a_{n}$, the induction step can be replaced by + +$$ +s_{n+1}=s_{n}+a_{n+1} \geqslant s_{n}+\frac{n}{s_{n}} \geqslant n+1 +$$ + +because the function $x \mapsto x+\frac{n}{x}$ increases on $[n, \infty)$. + +- By applying the AM-GM inequality to the numbers $a_{1}+\cdots+a_{k}$ and $k a_{k+1}$, we can conclude + +$$ +a_{1}+\cdots+a_{k}+k a_{k+1} \geqslant 2 k +$$ + +and sum it up for $k=1, \ldots, n-1$. + +- We can derive the symmetric estimate + +$$ +\sum_{1 \leqslant ib$ with $f(a)=f(b)$. A straightforward induction using (2) in the induction step reveals that we have $f(a+n)=f(b+n)$ for all nonnegative integers $n$. Consequently, the sequence $\gamma_{n}=f(b+n)$ is periodic and thus in particular bounded, which means that the numbers + +$$ +\varphi=\min _{n \geqslant 0} \gamma_{n} \quad \text { and } \quad \psi=\max _{n \geqslant 0} \gamma_{n} +$$ + +exist. +Let us pick any integer $y$ with $f(y)=\varphi$ and then an integer $x \geqslant a$ with $f(x-f(y))=\varphi$. Due to the definition of $\varphi$ and (3) we have + +$$ +\varphi \leqslant f(x+1)=f(x-f(y))+f(y)+1=2 \varphi+1 +$$ + +whence $\varphi \geqslant-1$. The same reasoning applied to $\psi$ yields $\psi \leqslant-1$. Since $\varphi \leqslant \psi$ holds trivially, it follows that $\varphi=\psi=-1$, or in other words that we have $f(t)=-1$ for all integers $t \geqslant a$. + +Finally, if any integer $y$ is given, we may find an integer $x$ which is so large that $x+1 \geqslant a$ and $x-f(y) \geqslant a$ hold. Due to (3) and the result from the previous paragraph we get + +$$ +f(y)=f(x+1)-f(x-f(y))-1=(-1)-(-1)-1=-1 . +$$ + +Thereby the problem is solved. +Solution 3. Set $d=f(0)$. By plugging $x=f(y)$ into (1) we obtain + +$$ +f^{3}(y)=f(y)+d+1 +$$ + +for all $y \in \mathbb{Z}$, where the left-hand side abbreviates $f(f(f(y)))$. When we replace $x$ in (1) by $f(x)$ we obtain $f(f(x)-f(y))=f^{3}(x)-f(y)-1$ and as a consequence of (4) this simplifies to + +$$ +f(f(x)-f(y))=f(x)-f(y)+d +$$ + +Now we consider the set + +$$ +E=\{f(x)-d \mid x \in \mathbb{Z}\} +$$ + +Given two integers $a$ and $b$ from $E$, we may pick some integers $x$ and $y$ with $f(x)=a+d$ and $f(y)=b+d$; now (5) tells us that $f(a-b)=(a-b)+d$, which means that $a-b$ itself exemplifies $a-b \in E$. Thus, + +$$ +E \text { is closed under taking differences. } +$$ + +Also, the definitions of $d$ and $E$ yield $0 \in E$. If $E=\{0\}$, then $f$ is a constant function and (1) implies that the only value attained by $f$ is indeed -1 . + +So let us henceforth suppose that $E$ contains some number besides zero. It is known that in this case (6) entails $E$ to be the set of all integer multiples of some positive integer $k$. Indeed, this holds for + +$$ +k=\min \{|x| \mid x \in E \text { and } x \neq 0\} +$$ + +as one may verify by an argument based on division with remainder. +Thus we have + +$$ +\{f(x) \mid x \in \mathbb{Z}\}=\{k \cdot t+d \mid t \in \mathbb{Z}\} +$$ + +Due to (5) and (7) we get + +$$ +f(k \cdot t)=k \cdot t+d +$$ + +for all $t \in \mathbb{Z}$, whence in particular $f(k)=k+d$. So by comparing the results of substituting $y=0$ and $y=k$ into (1) we learn that + +$$ +f(z+k)=f(z)+k +$$ + +holds for all integers $z$. In plain English, this means that on any residue class modulo $k$ the function $f$ is linear with slope 1. + +Now by (7) the set of all values attained by $f$ is such a residue class. Hence, there exists an absolute constant $c$ such that $f(f(x))=f(x)+c$ holds for all $x \in \mathbb{Z}$. Thereby (1) simplifies to + +$$ +f(x-f(y))=f(x)-f(y)+c-1 +$$ + +On the other hand, considering (1) modulo $k$ we obtain $d \equiv-1(\bmod k)$ because of (7). So by (7) again, $f$ attains the value -1 . + +Thus we may apply (9) to some integer $y$ with $f(y)=-1$, which gives $f(x+1)=f(x)+c$. So $f$ is a linear function with slope $c$. Hence, (8) leads to $c=1$, wherefore there is an absolute constant $d^{\prime}$ with $f(x)=x+d^{\prime}$ for all $x \in \mathbb{Z}$. Using this for $x=0$ we obtain $d^{\prime}=d$ and finally (4) discloses $d=1$, meaning that $f$ is indeed the successor function. + +A3. Let $n$ be a fixed positive integer. Find the maximum possible value of + +$$ +\sum_{1 \leqslant r1$ is odd. One can modify the arguments of the last part in order to work for every (not necessarily odd) sufficiently large value of $k$; namely, when $k$ is even, one may show that the sequence $P(1), P(2), \ldots, P(k)$ has different numbers of positive and negative terms. + +On the other hand, the problem statement with $k$ replaced by 2 is false, since the polynomials $P(x)=T(x)-T(x-1)$ and $Q(x)=T(x-1)-T(x)$ are block-similar in this case, due to the fact that $P(2 i-1)=-P(2 i)=Q(2 i)=-Q(2 i-1)=T(2 i-1)$ for all $i=1,2, \ldots, n$. Thus, every complete solution should use the relation $k>2$. + +One may easily see that the condition $n \geqslant 2$ is also substantial, since the polynomials $x$ and $k+1-x$ become block-similar if we set $n=1$. + +It is easily seen from the solution that the result still holds if we assume that the polynomials have degree at most $n$. + +Solution 2. We provide an alternative argument for part (b). +Assume again that there exist two distinct block-similar polynomials $P(x)$ and $Q(x)$ of degree $n$. Let $R(x)=P(x)-Q(x)$ and $S(x)=P(x)+Q(x)$. For brevity, we also denote the segment $[(i-1) k+1, i k]$ by $I_{i}$, and the set $\{(i-1) k+1,(i-1) k+2, \ldots, i k\}$ of all integer points in $I_{i}$ by $Z_{i}$. +Step 1. We prove that $R(x)$ has exactly one root in each segment $I_{i}, i=1,2, \ldots, n$, and all these roots are simple. + +Indeed, take any $i \in\{1,2, \ldots, n\}$ and choose some points $p^{-}, p^{+} \in Z_{i}$ so that + +$$ +P\left(p^{-}\right)=\min _{x \in Z_{i}} P(x) \quad \text { and } \quad P\left(p^{+}\right)=\max _{x \in Z_{i}} P(x) +$$ + +Since the sequences of values of $P$ and $Q$ in $Z_{i}$ are permutations of each other, we have $R\left(p^{-}\right)=P\left(p^{-}\right)-Q\left(p^{-}\right) \leqslant 0$ and $R\left(p^{+}\right)=P\left(p^{+}\right)-Q\left(p^{+}\right) \geqslant 0$. Since $R(x)$ is continuous, there exists at least one root of $R(x)$ between $p^{-}$and $p^{+}$- thus in $I_{i}$. + +So, $R(x)$ has at least one root in each of the $n$ disjoint segments $I_{i}$ with $i=1,2, \ldots, n$. Since $R(x)$ is nonzero and its degree does not exceed $n$, it should have exactly one root in each of these segments, and all these roots are simple, as required. + +## Step 2. We prove that $S(x)$ is constant. + +We start with the following claim. +Claim. For every $i=1,2, \ldots, n$, the sequence of values $S((i-1) k+1), S((i-1) k+2), \ldots$, $S(i k)$ cannot be strictly increasing. +Proof. Fix any $i \in\{1,2, \ldots, n\}$. Due to the symmetry, we may assume that $P(i k) \leqslant Q(i k)$. Choose now $p^{-}$and $p^{+}$as in Step 1. If we had $P\left(p^{+}\right)=P\left(p^{-}\right)$, then $P$ would be constant on $Z_{i}$, so all the elements of $Z_{i}$ would be the roots of $R(x)$, which is not the case. In particular, we have $p^{+} \neq p^{-}$. If $p^{-}>p^{+}$, then $S\left(p^{-}\right)=P\left(p^{-}\right)+Q\left(p^{-}\right) \leqslant Q\left(p^{+}\right)+P\left(p^{+}\right)=S\left(p^{+}\right)$, so our claim holds. + +We now show that the remaining case $p^{-}$ $Q\left(p^{+}\right)$. Then, like in Step 1 , we have $R\left(p^{-}\right) \leqslant 0, R\left(p^{+}\right)>0$, and $R(i k) \leqslant 0$, so $R(x)$ has a root in each of the intervals $\left[p^{-}, p^{+}\right)$and $\left(p^{+}, i k\right]$. This contradicts the result of Step 1. + +We are left only with the case $p^{-}0$. Next, let $p^{-}, q^{+} \in I_{i}$ be some points such that $P\left(p^{-}\right)=\min _{x \in Z_{i}} P(x)$ and $Q\left(q^{+}\right)=\max _{x \in Z_{i}} Q(x)$. Notice that $P\left(p^{-}\right) \leqslant Q(r)Q(r)$, so $r$ is different from $p^{-}$and $q^{+}$. + +Without loss of generality, we may assume that $p^{-}r$, then, similarly, $R\left(q^{+}\right) \leqslant 0\ell_{j}>r_{i}$. + +Clearly, there is no town which can sweep $T_{n}$ away from the right. Then we may choose the leftmost town $T_{k}$ which cannot be swept away from the right. One can observe now that no town $T_{i}$ with $i>k$ may sweep away some town $T_{j}$ with $jm$. As we have already observed, $p$ cannot be greater than $k$. On the other hand, $T_{m}$ cannot sweep $T_{p}$ away, so a fortiori it cannot sweep $T_{k}$ away. + +Claim 2. Any town $T_{m}$ with $m \neq k$ can be swept away by some other town. + +Proof. If $mk$. + +Let $T_{p}$ be a town among $T_{k}, T_{k+1}, \ldots, T_{m-1}$ having the largest right bulldozer. We claim that $T_{p}$ can sweep $T_{m}$ away. If this is not the case, then $r_{p}<\ell_{q}$ for some $q$ with $p1$. Firstly, we find a town which can be swept away by each of its neighbors (each town has two neighbors, except for the bordering ones each of which has one); we call such town a loser. Such a town exists, because there are $n-1$ pairs of neighboring towns, and in each of them there is only one which can sweep the other away; so there exists a town which is a winner in none of these pairs. + +Notice that a loser can be swept away, but it cannot sweep any other town away (due to its neighbors' protection). Now we remove a loser, and suggest its left bulldozer to its right neighbor (if it exists), and its right bulldozer to a left one (if it exists). Surely, a town accepts a suggestion if a suggested bulldozer is larger than the town's one of the same orientation. + +Notice that suggested bulldozers are useless in attack (by the definition of a loser), but may serve for defensive purposes. Moreover, each suggested bulldozer's protection works for the same pairs of remaining towns as before the removal. + +By the induction hypothesis, the new configuration contains exactly one town which cannot be swept away. The arguments above show that the initial one also satisfies this property. + +Solution 3. We separately prove that $(i)$ there exists a town which cannot be swept away, and that (ii) there is at most one such town. We also make use of the two observations from the previous solutions. +To prove ( $i$ ), assume contrariwise that every town can be swept away. Let $t_{1}$ be the leftmost town; next, for every $k=1,2, \ldots$ we inductively choose $t_{k+1}$ to be some town which can sweep $t_{k}$ away. Now we claim that for every $k=1,2, \ldots$, the town $t_{k+1}$ is to the right of $t_{k}$; this leads to the contradiction, since the number of towns is finite. + +Induction on $k$. The base case $k=1$ is clear due to the choice of $t_{1}$. Assume now that for all $j$ with $1 \leqslant jj$ and $a_{i}\frac{n}{2}$, then we have $O A_{i-n / 2+1}=A_{i} A_{i-n / 2+1}$. This completes the proof. + +An example of such a construction when $n=10$ is shown in Figure 1. +![](https://cdn.mathpix.com/cropped/2024_11_18_62dbd531c72d0ff8164cg-28.jpg?height=507&width=804&top_left_y=1574&top_left_x=272) + +Figure 1 +![](https://cdn.mathpix.com/cropped/2024_11_18_62dbd531c72d0ff8164cg-28.jpg?height=600&width=617&top_left_y=1479&top_left_x=1185) + +Figure 2 + +Comment (a). There are many ways to construct an example by placing equilateral triangles in a circle. Here we present one general method. + +Let $O$ be the center of a circle and let $A_{1}, B_{1}, \ldots, A_{k}, B_{k}$ be distinct points on the circle such that the triangle $O A_{i} B_{i}$ is equilateral for each $i$. Then $\mathcal{V}=\left\{O, A_{1}, B_{1}, \ldots, A_{k}, B_{k}\right\}$ is balanced. To construct a set of even cardinality, put extra points $C, D, E$ on the circle such that triangles $O C D$ and $O D E$ are equilateral (see Figure 2). Then $\mathcal{V}=\left\{O, A_{1}, B_{1}, \ldots, A_{k}, B_{k}, C, D, E\right\}$ is balanced. + +Part (b). We now show that there exists a balanced, center-free set containing $n$ points for all odd $n \geqslant 3$, and that one does not exist for any even $n \geqslant 3$. + +If $n$ is odd, then let $\mathcal{V}$ be the set of vertices of a regular $n$-gon. We have shown in part ( $a$ ) that $\mathcal{V}$ is balanced. We claim that $\mathcal{V}$ is also center-free. Indeed, if $P$ is a point such that +$P A=P B=P C$ for some three distinct vertices $A, B$ and $C$, then $P$ is the circumcenter of the $n$-gon, which is not contained in $\mathcal{V}$. + +Now suppose that $\mathcal{V}$ is a balanced, center-free set of even cardinality $n$. We will derive a contradiction. For a pair of distinct points $A, B \in \mathcal{V}$, we say that a point $C \in \mathcal{V}$ is associated with the pair $\{A, B\}$ if $A C=B C$. Since there are $\frac{n(n-1)}{2}$ pairs of points, there exists a point $P \in \mathcal{V}$ which is associated with at least $\left\lceil\frac{n(n-1)}{2} / n\right\rceil=\frac{n}{2}$ pairs. Note that none of these $\frac{n}{2}$ pairs can contain $P$, so that the union of these $\frac{n}{2}$ pairs consists of at most $n-1$ points. Hence there exist two such pairs that share a point. Let these two pairs be $\{A, B\}$ and $\{A, C\}$. Then $P A=P B=P C$, which is a contradiction. + +Comment (b). We can rephrase the argument in graph theoretic terms as follows. Let $\mathcal{V}$ be a balanced, center-free set consisting of $n$ points. For any pair of distinct vertices $A, B \in \mathcal{V}$ and for any $C \in \mathcal{V}$ such that $A C=B C$, draw directed edges $A \rightarrow C$ and $B \rightarrow C$. Then all pairs of vertices generate altogether at least $n(n-1)$ directed edges; since the set is center-free, these edges are distinct. So we must obtain a graph in which any two vertices are connected in both directions. Now, each vertex has exactly $n-1$ incoming edges, which means that $n-1$ is even. Hence $n$ is odd. + +C3. For a finite set $A$ of positive integers, we call a partition of $A$ into two disjoint nonempty subsets $A_{1}$ and $A_{2}$ good if the least common multiple of the elements in $A_{1}$ is equal to the greatest common divisor of the elements in $A_{2}$. Determine the minimum value of $n$ such that there exists a set of $n$ positive integers with exactly 2015 good partitions. +(Ukraine) +Answer. 3024. +Solution. Let $A=\left\{a_{1}, a_{2}, \ldots, a_{n}\right\}$, where $a_{1}1$ and $B$ can respond by choosing $a-1$ on the $k^{\text {th }}$ move instead. + +We now give an alternative winning strategy in the case $n$ is even and $n \geqslant 8$. We first present a winning strategy for the case when $A$ 's first pick is 1 . We consider two cases depending on $A$ 's second move. + +Case 1. A's second pick is 3 . Then $B$ chooses $n-3$ on the second move. On the $k^{\text {th }}$ move, $B$ chooses the number exactly 1 less than $A^{\prime}$ 's $k^{\text {th }}$ pick except that $B$ chooses 2 if $A$ 's $k^{\text {th }}$ pick is $n-2$ or $n-1$. + +Case 2. A's second pick is $a>3$. Then $B$ chooses $a-2$ on the second move. Afterwards on the $k^{\text {th }}$ move, $B$ picks the number exactly 1 less than $A^{\text {'s }} k^{\text {th }}$ pick. + +One may easily see that this strategy guarantees $B$ 's victory, when $A$ 's first pick is 1 . +The following claim shows how to extend the strategy to the general case. +Claim. Assume that $B$ has an explicit strategy leading to a victory after $A$ picks 1 on the first move. Then $B$ also has an explicit strategy leading to a victory after any first moves of $A$. +Proof. Let $S$ be an optimal strategy of $B$ after $A$ picks 1 on the first move. Assume that $A$ picks some number $a>1$ on this move; we show how $B$ can make use of $S$ in order to win in this case. + +In parallel to the real play, $B$ starts an imaginary play. The positions in these plays differ by flipping the segment $[1, a]$; so, if a player chooses some number $x$ in the real play, then the same player chooses a number $x$ or $a+1-x$ in the imaginary play, depending on whether $x>a$ or $x \leqslant a$. Thus $A$ 's first pick in the imaginary play is 1. + +Clearly, a number is chosen in the real play exactly if the corresponding number is chosen in the imaginary one. Next, if an unchosen number is neighboring to one chosen by $A$ in the imaginary play, then the corresponding number also has this property in the real play, so $A$ also cannot choose it. One can easily see that a similar statement with real and imaginary plays interchanged holds for $B$ instead of $A$. + +Thus, when $A$ makes some move in the real play, $B$ may imagine the corresponding legal move in the imaginary one. Then $B$ chooses the response according to $S$ in the imaginary game and makes the corresponding legal move in the real one. Acting so, $B$ wins the imaginary game, thus $B$ will also win the real one. + +Hence, $B$ has a winning strategy for all even $n$ greater or equal to 8 . +Notice that the claim can also be used to simplify the argument when $n$ is odd. +Comment 2. One may also employ symmetry when $n$ is odd. In particular, $B$ could use a mirror strategy. However, additional ideas are required to modify the strategy after $A$ picks $\frac{n+1}{2}$. + +C5. Consider an infinite sequence $a_{1}, a_{2}, \ldots$ of positive integers with $a_{i} \leqslant 2015$ for all $i \geqslant 1$. Suppose that for any two distinct indices $i$ and $j$ we have $i+a_{i} \neq j+a_{j}$. + +Prove that there exist two positive integers $b$ and $N$ such that + +$$ +\left|\sum_{i=m+1}^{n}\left(a_{i}-b\right)\right| \leqslant 1007^{2} +$$ + +whenever $n>m \geqslant N$. +(Australia) +Solution 1. We visualize the set of positive integers as a sequence of points. For each $n$ we draw an arrow emerging from $n$ that points to $n+a_{n}$; so the length of this arrow is $a_{n}$. Due to the condition that $m+a_{m} \neq n+a_{n}$ for $m \neq n$, each positive integer receives at most one arrow. There are some positive integers, such as 1 , that receive no arrows; these will be referred to as starting points in the sequel. When one starts at any of the starting points and keeps following the arrows, one is led to an infinite path, called its ray, that visits a strictly increasing sequence of positive integers. Since the length of any arrow is at most 2015, such a ray, say with starting point $s$, meets every interval of the form $[n, n+2014]$ with $n \geqslant s$ at least once. + +Suppose for the sake of contradiction that there would be at least 2016 starting points. Then we could take an integer $n$ that is larger than the first 2016 starting points. But now the interval $[n, n+2014]$ must be met by at least 2016 rays in distinct points, which is absurd. We have thereby shown that the number $b$ of starting points satisfies $1 \leqslant b \leqslant 2015$. Let $N$ denote any integer that is larger than all starting points. We contend that $b$ and $N$ are as required. + +To see this, let any two integers $m$ and $n$ with $n>m \geqslant N$ be given. The sum $\sum_{i=m+1}^{n} a_{i}$ gives the total length of the arrows emerging from $m+1, \ldots, n$. Taken together, these arrows form $b$ subpaths of our rays, some of which may be empty. Now on each ray we look at the first number that is larger than $m$; let $x_{1}, \ldots, x_{b}$ denote these numbers, and let $y_{1}, \ldots, y_{b}$ enumerate in corresponding order the numbers defined similarly with respect to $n$. Then the list of differences $y_{1}-x_{1}, \ldots, y_{b}-x_{b}$ consists of the lengths of these paths and possibly some zeros corresponding to empty paths. Consequently, we obtain + +$$ +\sum_{i=m+1}^{n} a_{i}=\sum_{j=1}^{b}\left(y_{j}-x_{j}\right) +$$ + +whence + +$$ +\sum_{i=m+1}^{n}\left(a_{i}-b\right)=\sum_{j=1}^{b}\left(y_{j}-n\right)-\sum_{j=1}^{b}\left(x_{j}-m\right) . +$$ + +Now each of the $b$ rays meets the interval $[m+1, m+2015]$ at some point and thus $x_{1}-$ $m, \ldots, x_{b}-m$ are $b$ distinct members of the set $\{1,2, \ldots, 2015\}$. Moreover, since $m+1$ is not a starting point, it must belong to some ray; so 1 has to appear among these numbers, wherefore + +$$ +1+\sum_{j=1}^{b-1}(j+1) \leqslant \sum_{j=1}^{b}\left(x_{j}-m\right) \leqslant 1+\sum_{j=1}^{b-1}(2016-b+j) +$$ + +The same argument applied to $n$ and $y_{1}, \ldots, y_{b}$ yields + +$$ +1+\sum_{j=1}^{b-1}(j+1) \leqslant \sum_{j=1}^{b}\left(y_{j}-n\right) \leqslant 1+\sum_{j=1}^{b-1}(2016-b+j) +$$ + +So altogether we get + +$$ +\begin{aligned} +\left|\sum_{i=m+1}^{n}\left(a_{i}-b\right)\right| & \leqslant \sum_{j=1}^{b-1}((2016-b+j)-(j+1))=(b-1)(2015-b) \\ +& \leqslant\left(\frac{(b-1)+(2015-b)}{2}\right)^{2}=1007^{2}, +\end{aligned} +$$ + +as desired. +Solution 2. Set $s_{n}=n+a_{n}$ for all positive integers $n$. By our assumptions, we have + +$$ +n+1 \leqslant s_{n} \leqslant n+2015 +$$ + +for all $n \in \mathbb{Z}_{>0}$. The members of the sequence $s_{1}, s_{2}, \ldots$ are distinct. We shall investigate the set + +$$ +M=\mathbb{Z}_{>0} \backslash\left\{s_{1}, s_{2}, \ldots\right\} +$$ + +Claim. At most 2015 numbers belong to $M$. +Proof. Otherwise let $m_{1}m \geqslant N$. Plugging $r=n$ and $r=m$ into (3) and subtracting the estimates that result, we deduce + +$$ +\sum_{i=m+1}^{n}\left(a_{i}-b\right)=\sum C_{n}-\sum C_{m} +$$ + +Since $C_{n}$ and $C_{m}$ are subsets of $\{1,2, \ldots, 2014\}$ with $\left|C_{n}\right|=\left|C_{m}\right|=b-1$, it is clear that the absolute value of the right-hand side of the above inequality attains its largest possible value if either $C_{m}=\{1,2, \ldots, b-1\}$ and $C_{n}=\{2016-b, \ldots, 2014\}$, or the other way around. In these two cases we have + +$$ +\left|\sum C_{n}-\sum C_{m}\right|=(b-1)(2015-b) +$$ + +so in the general case we find + +$$ +\left|\sum_{i=m+1}^{n}\left(a_{i}-b\right)\right| \leqslant(b-1)(2015-b) \leqslant\left(\frac{(b-1)+(2015-b)}{2}\right)^{2}=1007^{2} +$$ + +as desired. + +Comment. The sets $C_{n}$ may be visualized by means of the following process: Start with an empty blackboard. For $n \geqslant 1$, the following happens during the $n^{\text {th }}$ step. The number $a_{n}$ gets written on the blackboard, then all numbers currently on the blackboard are decreased by 1 , and finally all zeros that have arisen get swept away. + +It is not hard to see that the numbers present on the blackboard after $n$ steps are distinct and form the set $C_{n}$. Moreover, it is possible to complete a solution based on this idea. + +C6. Let $S$ be a nonempty set of positive integers. We say that a positive integer $n$ is clean if it has a unique representation as a sum of an odd number of distinct elements from $S$. Prove that there exist infinitely many positive integers that are not clean. + +Solution 1. Define an odd (respectively, even) representation of $n$ to be a representation of $n$ as a sum of an odd (respectively, even) number of distinct elements of $S$. Let $\mathbb{Z}_{>0}$ denote the set of all positive integers. + +Suppose, to the contrary, that there exist only finitely many positive integers that are not clean. Therefore, there exists a positive integer $N$ such that every integer $n>N$ has exactly one odd representation. + +Clearly, $S$ is infinite. We now claim the following properties of odd and even representations. +$\underline{P r o p e r t y}$ 1. Any positive integer $n$ has at most one odd and at most one even representation. +Proof. We first show that every integer $n$ has at most one even representation. Since $S$ is infinite, there exists $x \in S$ such that $x>\max \{n, N\}$. Then, the number $n+x$ must be clean, and $x$ does not appear in any even representation of $n$. If $n$ has more than one even representation, then we obtain two distinct odd representations of $n+x$ by adding $x$ to the even representations of $n$, which is impossible. Therefore, $n$ can have at most one even representation. + +Similarly, there exist two distinct elements $y, z \in S$ such that $y, z>\max \{n, N\}$. If $n$ has more than one odd representation, then we obtain two distinct odd representations of $n+y+z$ by adding $y$ and $z$ to the odd representations of $n$. This is again a contradiction. + +Property 2. Fix $s \in S$. Suppose that a number $n>N$ has no even representation. Then $n+2 a s$ has an even representation containing $s$ for all integers $a \geqslant 1$. +Proof. It is sufficient to prove the following statement: If $n$ has no even representation without $s$, then $n+2 s$ has an even representation containing $s$ (and hence no even representation without $s$ by Property 1). + +Notice that the odd representation of $n+s$ does not contain $s$; otherwise, we have an even representation of $n$ without $s$. Then, adding $s$ to this odd representation of $n+s$, we get that $n+2 s$ has an even representation containing $s$, as desired. + +Property 3. Every sufficiently large integer has an even representation. +Proof. Fix any $s \in S$, and let $r$ be an arbitrary element in $\{1,2, \ldots, 2 s\}$. Then, Property 2 implies that the set $Z_{r}=\{r+2 a s: a \geqslant 0\}$ contains at most one number exceeding $N$ with no even representation. Therefore, $Z_{r}$ contains finitely many positive integers with no even representation, and so does $\mathbb{Z}_{>0}=\bigcup_{r=1}^{2 s} Z_{r}$. + +In view of Properties 1 and 3 , we may assume that $N$ is chosen such that every $n>N$ has exactly one odd and exactly one even representation. In particular, each element $s>N$ of $S$ has an even representation. +Property 4. For any $s, t \in S$ with $NN$. Then, Property 4 implies that for every $i>k$ the even representation of $s_{i}$ contains all the numbers $s_{k}, s_{k+1}, \ldots, s_{i-1}$. Therefore, + +$$ +s_{i}=s_{k}+s_{k+1}+\cdots+s_{i-1}+R_{i}=\sigma_{i-1}-\sigma_{k-1}+R_{i} +$$ + +where $R_{i}$ is a sum of some of $s_{1}, \ldots, s_{k-1}$. In particular, $0 \leqslant R_{i} \leqslant s_{1}+\cdots+s_{k-1}=\sigma_{k-1}$. + +Let $j_{0}$ be an integer satisfying $j_{0}>k$ and $\sigma_{j_{0}}>2 \sigma_{k-1}$. Then (1) shows that, for every $j>j_{0}$, + +$$ +s_{j+1} \geqslant \sigma_{j}-\sigma_{k-1}>\sigma_{j} / 2 . +$$ + +Next, let $p>j_{0}$ be an index such that $R_{p}=\min _{i>j_{0}} R_{i}$. Then, + +$$ +s_{p+1}=s_{k}+s_{k+1}+\cdots+s_{p}+R_{p+1}=\left(s_{p}-R_{p}\right)+s_{p}+R_{p+1} \geqslant 2 s_{p} +$$ + +Therefore, there is no element of $S$ larger than $s_{p}$ but smaller than $2 s_{p}$. It follows that the even representation $\tau$ of $2 s_{p}$ does not contain any element larger than $s_{p}$. On the other hand, inequality (2) yields $2 s_{p}>s_{1}+\cdots+s_{p-1}$, so $\tau$ must contain a term larger than $s_{p-1}$. Thus, it must contain $s_{p}$. After removing $s_{p}$ from $\tau$, we have that $s_{p}$ has an odd representation not containing $s_{p}$, which contradicts Property 1 since $s_{p}$ itself also forms an odd representation of $s_{p}$. + +Solution 2. We will also use Property 1 from Solution 1. +We first define some terminology and notations used in this solution. Let $\mathbb{Z}_{\geqslant 0}$ denote the set of all nonnegative integers. All sums mentioned are regarded as sums of distinct elements of $S$. Moreover, a sum is called even or odd depending on the parity of the number of terms in it. All closed or open intervals refer to sets of all integers inside them, e.g., $[a, b]=\{x \in \mathbb{Z}: a \leqslant x \leqslant b\}$. + +Again, let $s_{1}$ $2^{n-1}-1+n m \geqslant \sigma_{n}$ for every sufficiently large $n$. We now claim the following. + +Claim 1. $\left(\sigma_{n}-s_{n+1}, s_{n+2}-s_{n+1}\right) \subseteq E_{n}$ for every sufficiently large $n$. +Proof. For sufficiently large $n$, all elements of $\left(\sigma_{n}, s_{n+2}\right)$ are clean. Clearly, the elements of $\left(\sigma_{n}, s_{n+2}\right)$ can be in neither $O_{n}$ nor $O \backslash O_{n+1}$. So, $\left(\sigma_{n}, s_{n+2}\right) \subseteq O_{n+1} \backslash O_{n}=s_{n+1}+E_{n}$, which yields the claim. + +Now, Claim 1 together with inequalities (3) implies that, for all sufficiently large $n$, + +$$ +E \supseteq E_{n} \supseteq\left(\sigma_{n}-s_{n+1}, s_{n+2}-s_{n+1}\right) \supseteq\left(2 n m, 2^{n-1}-(n+2) m\right) . +$$ + +This easily yields that $\mathbb{Z}_{\geqslant 0} \backslash E$ is also finite. Since $\mathbb{Z}_{\geqslant 0} \backslash O$ is also finite, by Property 1 , there exists a positive integer $N$ such that every integer $n>N$ has exactly one even and one odd representation. + +Step 3. We investigate the structures of $E_{n}$ and $O_{n}$. +Suppose that $z \in E_{2 n}$. Since $z$ can be represented as an even sum using $\left\{s_{1}, s_{2}, \ldots, s_{2 n}\right\}$, so can its complement $\sigma_{2 n}-z$. Thus, we get $E_{2 n}=\sigma_{2 n}-E_{2 n}$. Similarly, we have + +$$ +E_{2 n}=\sigma_{2 n}-E_{2 n}, \quad O_{2 n}=\sigma_{2 n}-O_{2 n}, \quad E_{2 n+1}=\sigma_{2 n+1}-O_{2 n+1}, \quad O_{2 n+1}=\sigma_{2 n+1}-E_{2 n+1} +$$ + +Claim 2. For every sufficiently large $n$, we have + +$$ +\left[0, \sigma_{n}\right] \supseteq O_{n} \supseteq\left(N, \sigma_{n}-N\right) \quad \text { and } \quad\left[0, \sigma_{n}\right] \supseteq E_{n} \supseteq\left(N, \sigma_{n}-N\right) +$$ + +Proof. Clearly $O_{n}, E_{n} \subseteq\left[0, \sigma_{n}\right]$ for every positive integer $n$. We now prove $O_{n}, E_{n} \supseteq\left(N, \sigma_{n}-N\right)$. Taking $n$ sufficiently large, we may assume that $s_{n+1} \geqslant 2^{n-1}+1-n m>\frac{1}{2}\left(2^{n-1}-1+n m\right) \geqslant \sigma_{n} / 2$. Therefore, the odd representation of every element of ( $\left.N, \sigma_{n} / 2\right]$ cannot contain a term larger than $s_{n}$. Thus, $\left(N, \sigma_{n} / 2\right] \subseteq O_{n}$. Similarly, since $s_{n+1}+s_{1}>\sigma_{n} / 2$, we also have $\left(N, \sigma_{n} / 2\right] \subseteq E_{n}$. Equations (4) then yield that, for sufficiently large $n$, the interval $\left(N, \sigma_{n}-N\right)$ is a subset of both $O_{n}$ and $E_{n}$, as desired. + +Step 4. We obtain a final contradiction. +Notice that $0 \in \mathbb{Z}_{\geqslant 0} \backslash O$ and $1 \in \mathbb{Z}_{\geqslant 0} \backslash E$. Therefore, the sets $\mathbb{Z}_{\geqslant 0} \backslash O$ and $\mathbb{Z}_{\geqslant 0} \backslash E$ are nonempty. Denote $o=\max \left(\mathbb{Z}_{\geqslant 0} \backslash O\right)$ and $e=\max \left(\mathbb{Z}_{\geqslant 0} \backslash E\right)$. Observe also that $e, o \leqslant N$. + +Taking $k$ sufficiently large, we may assume that $\sigma_{2 k}>2 N$ and that Claim 2 holds for all $n \geqslant 2 k$. Due to (4) and Claim 2, we have that $\sigma_{2 k}-e$ is the minimal number greater than $N$ which is not in $E_{2 k}$, i.e., $\sigma_{2 k}-e=s_{2 k+1}+s_{1}$. Similarly, + +$$ +\sigma_{2 k}-o=s_{2 k+1}, \quad \sigma_{2 k+1}-e=s_{2 k+2}, \quad \text { and } \quad \sigma_{2 k+1}-o=s_{2 k+2}+s_{1} +$$ + +Therefore, we have + +$$ +\begin{aligned} +s_{1} & =\left(s_{2 k+1}+s_{1}\right)-s_{2 k+1}=\left(\sigma_{2 k}-e\right)-\left(\sigma_{2 k}-o\right)=o-e \\ +& =\left(\sigma_{2 k+1}-e\right)-\left(\sigma_{2 k+1}-o\right)=s_{2 k+2}-\left(s_{2 k+2}+s_{1}\right)=-s_{1} +\end{aligned} +$$ + +which is impossible since $s_{1}>0$. + +C7. In a company of people some pairs are enemies. A group of people is called unsociable if the number of members in the group is odd and at least 3, and it is possible to arrange all its members around a round table so that every two neighbors are enemies. Given that there are at most 2015 unsociable groups, prove that it is possible to partition the company into 11 parts so that no two enemies are in the same part. +(Russia) +Solution 1. Let $G=(V, E)$ be a graph where $V$ is the set of people in the company and $E$ is the set of the enemy pairs - the edges of the graph. In this language, partitioning into 11 disjoint enemy-free subsets means properly coloring the vertices of this graph with 11 colors. + +We will prove the following more general statement. +Claim. Let $G$ be a graph with chromatic number $k \geqslant 3$. Then $G$ contains at least $2^{k-1}-k$ unsociable groups. + +Recall that the chromatic number of $G$ is the least $k$ such that a proper coloring + +$$ +V=V_{1} \sqcup \cdots \sqcup V_{k} +$$ + +exists. In view of $2^{11}-12>2015$, the claim implies the problem statement. +Let $G$ be a graph with chromatic number $k$. We say that a proper coloring (1) of $G$ is leximinimal, if the $k$-tuple $\left(\left|V_{1}\right|,\left|V_{2}\right|, \ldots,\left|V_{k}\right|\right)$ is lexicographically minimal; in other words, the following conditions are satisfied: the number $n_{1}=\left|V_{1}\right|$ is minimal; the number $n_{2}=\left|V_{2}\right|$ is minimal, subject to the previously chosen value of $n_{1} ; \ldots$; the number $n_{k-1}=\left|V_{k-1}\right|$ is minimal, subject to the previously chosen values of $n_{1}, \ldots, n_{k-2}$. + +The following lemma is the core of the proof. +Lemma 1. Suppose that $G=(V, E)$ is a graph with odd chromatic number $k \geqslant 3$, and let (1) be one of its leximinimal colorings. Then $G$ contains an odd cycle which visits all color classes $V_{1}, V_{2}, \ldots, V_{k}$. +Proof of Lemma 1. Let us call a cycle colorful if it visits all color classes. +Due to the definition of the chromatic number, $V_{1}$ is nonempty. Choose an arbitrary vertex $v \in V_{1}$. We construct a colorful odd cycle that has only one vertex in $V_{1}$, and this vertex is $v$. + +We draw a subgraph of $G$ as follows. Place $v$ in the center, and arrange the sets $V_{2}, V_{3}, \ldots, V_{k}$ in counterclockwise circular order around it. For convenience, let $V_{k+1}=V_{2}$. We will draw arrows to add direction to some edges of $G$, and mark the vertices these arrows point to. First we draw arrows from $v$ to all its neighbors in $V_{2}$, and mark all those neighbors. If some vertex $u \in V_{i}$ with $i \in\{2,3, \ldots, k\}$ is already marked, we draw arrows from $u$ to all its neighbors in $V_{i+1}$ which are not marked yet, and we mark all of them. We proceed doing this as long as it is possible. The process of marking is exemplified in Figure 1. + +Notice that by the rules of our process, in the final state, marked vertices in $V_{i}$ cannot have unmarked neighbors in $V_{i+1}$. Moreover, $v$ is connected to all marked vertices by directed paths. + +Now move each marked vertex to the next color class in circular order (see an example in Figure 3). In view of the arguments above, the obtained coloring $V_{1} \sqcup W_{2} \sqcup \cdots \sqcup W_{k}$ is proper. Notice that $v$ has a neighbor $w \in W_{2}$, because otherwise + +$$ +\left(V_{1} \backslash\{v\}\right) \sqcup\left(W_{2} \cup\{v\}\right) \sqcup W_{3} \sqcup \cdots \sqcup W_{k} +$$ + +would be a proper coloring lexicographically smaller than (1). If $w$ was unmarked, i.e., $w$ was an element of $V_{2}$, then it would be marked at the beginning of the process and thus moved to $V_{3}$, which did not happen. Therefore, $w$ is marked and $w \in V_{k}$. +![](https://cdn.mathpix.com/cropped/2024_11_18_62dbd531c72d0ff8164cg-40.jpg?height=780&width=1658&top_left_y=181&top_left_x=219) + +Figure 1 +Figure 2 +Since $w$ is marked, there exists a directed path from $v$ to $w$. This path moves through the sets $V_{2}, \ldots, V_{k}$ in circular order, so the number of edges in it is divisible by $k-1$ and thus even. Closing this path by the edge $w \rightarrow v$, we get a colorful odd cycle, as required. + +Proof of the claim. Let us choose a leximinimal coloring (1) of $G$. For every set $C \subseteq\{1,2, \ldots, k\}$ such that $|C|$ is odd and greater than 1 , we will provide an odd cycle visiting exactly those color classes whose indices are listed in the set $C$. This property ensures that we have different cycles for different choices of $C$, and it proves the claim because there are $2^{k-1}-k$ choices for the set $C$. + +Let $V_{C}=\bigcup_{c \in C} V_{c}$, and let $G_{C}$ be the induced subgraph of $G$ on the vertex set $V_{C}$. We also have the induced coloring of $V_{C}$ with $|C|$ colors; this coloring is of course proper. Notice further that the induced coloring is leximinimal: if we had a lexicographically smaller coloring $\left(W_{c}\right)_{c \in C}$ of $G_{C}$, then these classes, together the original color classes $V_{i}$ for $i \notin C$, would provide a proper coloring which is lexicographically smaller than (1). Hence Lemma 1, applied to the subgraph $G_{C}$ and its leximinimal coloring $\left(V_{c}\right)_{c \in C}$, provides an odd cycle that visits exactly those color classes that are listed in the set $C$. + +Solution 2. We provide a different proof of the claim from the previous solution. +We say that a graph is critical if deleting any vertex from the graph decreases the graph's chromatic number. Obviously every graph contains a critical induced subgraph with the same chromatic number. +Lemma 2. Suppose that $G=(V, E)$ is a critical graph with chromatic number $k \geqslant 3$. Then every vertex $v$ of $G$ is contained in at least $2^{k-2}-1$ unsociable groups. +Proof. For every set $X \subseteq V$, denote by $n(X)$ the number of neighbors of $v$ in the set $X$. +Since $G$ is critical, there exists a proper coloring of $G \backslash\{v\}$ with $k-1$ colors, so there exists a proper coloring $V=V_{1} \sqcup V_{2} \sqcup \cdots \sqcup V_{k}$ of $G$ such that $V_{1}=\{v\}$. Among such colorings, take one for which the sequence $\left(n\left(V_{2}\right), n\left(V_{3}\right), \ldots, n\left(V_{k}\right)\right)$ is lexicographically minimal. Clearly, $n\left(V_{i}\right)>0$ for every $i=2,3, \ldots, k$; otherwise $V_{2} \sqcup \ldots \sqcup V_{i-1} \sqcup\left(V_{i} \cup V_{1}\right) \sqcup V_{i+1} \sqcup \ldots V_{k}$ would be a proper coloring of $G$ with $k-1$ colors. + +We claim that for every $C \subseteq\{2,3, \ldots, k\}$ with $|C| \geqslant 2$ being even, $G$ contains an unsociable group so that the set of its members' colors is precisely $C \cup\{1\}$. Since the number of such sets $C$ is $2^{k-2}-1$, this proves the lemma. Denote the elements of $C$ by $c_{1}, \ldots, c_{2 \ell}$ in increasing order. For brevity, let $U_{i}=V_{c_{i}}$. Denote by $N_{i}$ the set of neighbors of $v$ in $U_{i}$. + +We show that for every $i=1, \ldots, 2 \ell-1$ and $x \in N_{i}$, the subgraph induced by $U_{i} \cup U_{i+1}$ contains a path that connects $x$ with another point in $N_{i+1}$. For the sake of contradiction, suppose that no such path exists. Let $S$ be the set of vertices that lie in the connected component of $x$ in the subgraph induced by $U_{i} \cup U_{i+1}$, and let $P=U_{i} \cap S$, and $Q=U_{i+1} \cap S$ (see Figure 3). Since $x$ is separated from $N_{i+1}$, the sets $Q$ and $N_{i+1}$ are disjoint. So, if we re-color $G$ by replacing $U_{i}$ and $U_{i+1}$ by $\left(U_{i} \cup Q\right) \backslash P$ and $\left(U_{i+1} \cup P\right) \backslash Q$, respectively, we obtain a proper coloring such that $n\left(U_{i}\right)=n\left(V_{c_{i}}\right)$ is decreased and only $n\left(U_{i+1}\right)=n\left(V_{c_{i+1}}\right)$ is increased. That contradicts the lexicographical minimality of $\left(n\left(V_{2}\right), n\left(V_{3}\right), \ldots, n\left(V_{k}\right)\right)$. +![](https://cdn.mathpix.com/cropped/2024_11_18_62dbd531c72d0ff8164cg-41.jpg?height=438&width=851&top_left_y=632&top_left_x=611) + +Figure 3 +Next, we build a path through $U_{1}, U_{2}, \ldots, U_{2 \ell}$ as follows. Let the starting point of the path be an arbitrary vertex $v_{1}$ in the set $N_{1}$. For $i \leqslant 2 \ell-1$, if the vertex $v_{i} \in N_{i}$ is already defined, connect $v_{i}$ to some vertex in $N_{i+1}$ in the subgraph induced by $U_{i} \cup U_{i+1}$, and add these edges to the path. Denote the new endpoint of the path by $v_{i+1}$; by the construction we have $v_{i+1} \in N_{i+1}$ again, so the process can be continued. At the end we have a path that starts at $v_{1} \in N_{1}$ and ends at some $v_{2 \ell} \in N_{2 \ell}$. Moreover, all edges in this path connect vertices in neighboring classes: if a vertex of the path lies in $U_{i}$, then the next vertex lies in $U_{i+1}$ or $U_{i-1}$. Notice that the path is not necessary simple, so take a minimal subpath of it. The minimal subpath is simple and connects the same endpoints $v_{1}$ and $v_{2 \ell}$. The property that every edge steps to a neighboring color class (i.e., from $U_{i}$ to $U_{i+1}$ or $U_{i-1}$ ) is preserved. So the resulting path also visits all of $U_{1}, \ldots, U_{2 \ell}$, and its length must be odd. Closing the path with the edges $v v_{1}$ and $v_{2 \ell} v$ we obtain the desired odd cycle (see Figure 4). +![](https://cdn.mathpix.com/cropped/2024_11_18_62dbd531c72d0ff8164cg-41.jpg?height=518&width=1489&top_left_y=1803&top_left_x=286) + +Figure 4 +Now we prove the claim by induction on $k \geqslant 3$. The base case $k=3$ holds by applying Lemma 2 to a critical subgraph. For the induction step, let $G_{0}$ be a critical $k$-chromatic subgraph of $G$, and let $v$ be an arbitrary vertex of $G_{0}$. By Lemma 2, $G_{0}$ has at least $2^{k-2}-1$ unsociable groups containing $v$. On the other hand, the graph $G_{0} \backslash\{v\}$ has chromatic number $k-1$, so it contains at least $2^{k-2}-(k-1)$ unsociable groups by the induction hypothesis. Altogether, this gives $2^{k-2}-1+2^{k-2}-(k-1)=2^{k-1}-k$ distinct unsociable groups in $G_{0}$ (and thus in $G$ ). + +Comment 1. The claim we proved is sharp. The complete graph with $k$ vertices has chromatic number $k$ and contains exactly $2^{k-1}-k$ unsociable groups. + +Comment 2. The proof of Lemma 2 works for odd values of $|C| \geqslant 3$ as well. Hence, the second solution shows the analogous statement that the number of even sized unsociable groups is at least $2^{k}-1-\binom{k}{2}$. + +## Geometry + +G1. Let $A B C$ be an acute triangle with orthocenter $H$. Let $G$ be the point such that the quadrilateral $A B G H$ is a parallelogram. Let $I$ be the point on the line $G H$ such that $A C$ bisects $H I$. Suppose that the line $A C$ intersects the circumcircle of the triangle $G C I$ at $C$ and $J$. Prove that $I J=A H$. +(Australia) +Solution 1. Since $H G \| A B$ and $B G \| A H$, we have $B G \perp B C$ and $C H \perp G H$. Therefore, the quadrilateral $B G C H$ is cyclic. Since $H$ is the orthocenter of the triangle $A B C$, we have $\angle H A C=90^{\circ}-\angle A C B=\angle C B H$. Using that $B G C H$ and $C G J I$ are cyclic quadrilaterals, we get + +$$ +\angle C J I=\angle C G H=\angle C B H=\angle H A C . +$$ + +Let $M$ be the intersection of $A C$ and $G H$, and let $D \neq A$ be the point on the line $A C$ such that $A H=H D$. Then $\angle M J I=\angle H A C=\angle M D H$. + +Since $\angle M J I=\angle M D H, \angle I M J=\angle H M D$, and $I M=M H$, the triangles $I M J$ and $H M D$ are congruent, and thus $I J=H D=A H$. +![](https://cdn.mathpix.com/cropped/2024_11_18_62dbd531c72d0ff8164cg-44.jpg?height=863&width=901&top_left_y=1062&top_left_x=583) + +Comment. Instead of introducing the point $D$, one can complete the solution by using the law of sines in the triangles $I J M$ and $A M H$, yielding + +$$ +\frac{I J}{I M}=\frac{\sin \angle I M J}{\sin \angle M J I}=\frac{\sin \angle A M H}{\sin \angle H A M}=\frac{A H}{M H}=\frac{A H}{I M} . +$$ + +Solution 2. Obtain $\angle C G H=\angle H A C$ as in the previous solution. In the parallelogram $A B G H$ we have $\angle B A H=\angle H G B$. It follows that + +$$ +\angle H M C=\angle B A C=\angle B A H+\angle H A C=\angle H G B+\angle C G H=\angle C G B . +$$ + +So the right triangles $C M H$ and $C G B$ are similar. Also, in the circumcircle of triangle $G C I$ we have similar triangles $M I J$ and $M C G$. Therefore, + +$$ +\frac{I J}{C G}=\frac{M I}{M C}=\frac{M H}{M C}=\frac{G B}{G C}=\frac{A H}{C G} +$$ + +Hence $I J=A H$. + +G2. Let $A B C$ be a triangle inscribed into a circle $\Omega$ with center $O$. A circle $\Gamma$ with center $A$ meets the side $B C$ at points $D$ and $E$ such that $D$ lies between $B$ and $E$. Moreover, let $F$ and $G$ be the common points of $\Gamma$ and $\Omega$. We assume that $F$ lies on the arc $A B$ of $\Omega$ not containing $C$, and $G$ lies on the arc $A C$ of $\Omega$ not containing $B$. The circumcircles of the triangles $B D F$ and $C E G$ meet the sides $A B$ and $A C$ again at $K$ and $L$, respectively. Suppose that the lines $F K$ and $G L$ are distinct and intersect at $X$. Prove that the points $A, X$, and $O$ are collinear. +(Greece) +Solution 1. It suffices to prove that the lines $F K$ and $G L$ are symmetric about $A O$. Now the segments $A F$ and $A G$, being chords of $\Omega$ with the same length, are clearly symmetric with respect to $A O$. Hence it is enough to show + +$$ +\angle K F A=\angle A G L . +$$ + +Let us denote the circumcircles of $B D F$ and $C E G$ by $\omega_{B}$ and $\omega_{C}$, respectively. To prove (1), we start from + +$$ +\angle K F A=\angle D F G+\angle G F A-\angle D F K +$$ + +In view of the circles $\omega_{B}, \Gamma$, and $\Omega$, this may be rewritten as + +$$ +\angle K F A=\angle C E G+\angle G B A-\angle D B K=\angle C E G-\angle C B G . +$$ + +Due to the circles $\omega_{C}$ and $\Omega$, we obtain $\angle K F A=\angle C L G-\angle C A G=\angle A G L$. Thereby the problem is solved. +![](https://cdn.mathpix.com/cropped/2024_11_18_62dbd531c72d0ff8164cg-45.jpg?height=729&width=1220&top_left_y=1389&top_left_x=418) + +Figure 1 + +Solution 2. Again, we denote the circumcircle of $B D K F$ by $\omega_{B}$. In addition, we set $\alpha=$ $\angle B A C, \varphi=\angle A B F$, and $\psi=\angle E D A=\angle A E D$ (see Figure 2). Notice that $A F=A G$ entails $\varphi=\angle G C A$, so all three of $\alpha, \varphi$, and $\psi$ respect the "symmetry" between $B$ and $C$ of our configuration. Again, we reduce our task to proving (1). + +This time, we start from + +$$ +2 \angle K F A=2(\angle D F A-\angle D F K) . +$$ + +Since the triangle $A F D$ is isosceles, we have + +$$ +\angle D F A=\angle A D F=\angle E D F-\psi=\angle B F D+\angle E B F-\psi . +$$ + +Moreover, because of the circle $\omega_{B}$ we have $\angle D F K=\angle C B A$. Altogether, this yields + +$$ +2 \angle K F A=\angle D F A+(\angle B F D+\angle E B F-\psi)-2 \angle C B A, +$$ + +which simplifies to + +$$ +2 \angle K F A=\angle B F A+\varphi-\psi-\angle C B A . +$$ + +Now the quadrilateral $A F B C$ is cyclic, so this entails $2 \angle K F A=\alpha+\varphi-\psi$. +Due to the "symmetry" between $B$ and $C$ alluded to above, this argument also shows that $2 \angle A G L=\alpha+\varphi-\psi$. This concludes the proof of (1). +![](https://cdn.mathpix.com/cropped/2024_11_18_62dbd531c72d0ff8164cg-46.jpg?height=547&width=926&top_left_y=726&top_left_x=571) + +Figure 2 + +Comment 1. As the first solution shows, the assumption that $A$ be the center of $\Gamma$ may be weakened to the following one: The center of $\Gamma$ lies on the line $O A$. The second solution may be modified to yield the same result. + +Comment 2. It might be interesting to remark that $\angle G D K=90^{\circ}$. To prove this, let $G^{\prime}$ denote the point on $\Gamma$ diametrically opposite to $G$. Because of $\angle K D F=\angle K B F=\angle A G F=\angle G^{\prime} D F$, the points $D, K$, and $G^{\prime}$ are collinear, which leads to the desired result. Notice that due to symmetry we also have $\angle L E F=90^{\circ}$. + +Moreover, a standard argument shows that the triangles $A G L$ and $B G E$ are similar. By symmetry again, also the triangles $A F K$ and $C D F$ are similar. + +There are several ways to derive a solution from these facts. For instance, one may argue that + +$$ +\begin{aligned} +\angle K F A & =\angle B F A-\angle B F K=\angle B F A-\angle E D G^{\prime}=\left(180^{\circ}-\angle A G B\right)-\left(180^{\circ}-\angle G^{\prime} G E\right) \\ +& =\angle A G E-\angle A G B=\angle B G E=\angle A G L . +\end{aligned} +$$ + +Comment 3. The original proposal did not contain the point $X$ in the assumption and asked instead to prove that the lines $F K, G L$, and $A O$ are concurrent. This differs from the version given above only insofar as it also requires to show that these lines cannot be parallel. The Problem Selection Committee removed this part from the problem intending to make it thus more suitable for the Olympiad. + +For the sake of completeness, we would still like to sketch one possibility for proving $F K \nVdash A O$ here. As the points $K$ and $O$ lie in the angular region $\angle F A G$, it suffices to check $\angle K F A+\angle F A O<180^{\circ}$. Multiplying by 2 and making use of the formulae from the second solution, we see that this is equivalent to $(\alpha+\varphi-\psi)+\left(180^{\circ}-2 \varphi\right)<360^{\circ}$, which in turn is an easy consequence of $\alpha<180^{\circ}$. + +G3. Let $A B C$ be a triangle with $\angle C=90^{\circ}$, and let $H$ be the foot of the altitude from $C$. A point $D$ is chosen inside the triangle $C B H$ so that $C H$ bisects $A D$. Let $P$ be the intersection point of the lines $B D$ and $C H$. Let $\omega$ be the semicircle with diameter $B D$ that meets the segment $C B$ at an interior point. A line through $P$ is tangent to $\omega$ at $Q$. Prove that the lines $C Q$ and $A D$ meet on $\omega$. +(Georgia) +Solution 1. Let $K$ be the projection of $D$ onto $A B$; then $A H=H K$ (see Figure 1). Since $P H \| D K$, we have + +$$ +\frac{P D}{P B}=\frac{H K}{H B}=\frac{A H}{H B} +$$ + +Let $L$ be the projection of $Q$ onto $D B$. Since $P Q$ is tangent to $\omega$ and $\angle D Q B=\angle B L Q=$ $90^{\circ}$, we have $\angle P Q D=\angle Q B P=\angle D Q L$. Therefore, $Q D$ and $Q B$ are respectively the internal and the external bisectors of $\angle P Q L$. By the angle bisector theorem, we obtain + +$$ +\frac{P D}{D L}=\frac{P Q}{Q L}=\frac{P B}{B L} +$$ + +The relations (1) and (2) yield $\frac{A H}{H B}=\frac{P D}{P B}=\frac{D L}{L B}$. So, the spiral similarity $\tau$ centered at $B$ and sending $A$ to $D$ maps $H$ to $L$. Moreover, $\tau$ sends the semicircle with diameter $A B$ passing through $C$ to $\omega$. Due to $C H \perp A B$ and $Q L \perp D B$, it follows that $\tau(C)=Q$. + +Hence, the triangles $A B D$ and $C B Q$ are similar, so $\angle A D B=\angle C Q B$. This means that the lines $A D$ and $C Q$ meet at some point $T$, and this point satisfies $\angle B D T=\angle B Q T$. Therefore, $T$ lies on $\omega$, as needed. +![](https://cdn.mathpix.com/cropped/2024_11_18_62dbd531c72d0ff8164cg-47.jpg?height=438&width=801&top_left_y=1397&top_left_x=225) + +Figure 1 +![](https://cdn.mathpix.com/cropped/2024_11_18_62dbd531c72d0ff8164cg-47.jpg?height=464&width=798&top_left_y=1367&top_left_x=1046) + +Figure 2 + +Comment 1. Since $\angle B A D=\angle B C Q$, the point $T$ lies also on the circumcircle of the triangle $A B C$. +Solution 2. Let $\Gamma$ be the circumcircle of $A B C$, and let $A D$ meet $\omega$ at $T$. Then $\angle A T B=$ $\angle A C B=90^{\circ}$, so $T$ lies on $\Gamma$ as well. As in the previous solution, let $K$ be the projection of $D$ onto $A B$; then $A H=H K$ (see Figure 2). + +Our goal now is to prove that the points $C, Q$, and $T$ are collinear. Let $C T$ meet $\omega$ again at $Q^{\prime}$. Then, it suffices to show that $P Q^{\prime}$ is tangent to $\omega$, or that $\angle P Q^{\prime} D=\angle Q^{\prime} B D$. + +Since the quadrilateral $B D Q^{\prime} T$ is cyclic and the triangles $A H C$ and $K H C$ are congruent, we have $\angle Q^{\prime} B D=\angle Q^{\prime} T D=\angle C T A=\angle C B A=\angle A C H=\angle H C K$. Hence, the right triangles $C H K$ and $B Q^{\prime} D$ are similar. This implies that $\frac{H K}{C K}=\frac{Q^{\prime} D}{B D}$, and thus $H K \cdot B D=C K \cdot Q^{\prime} D$. + +Notice that $P H \| D K$; therefore, we have $\frac{P D}{B D}=\frac{H K}{B K}$, and so $P D \cdot B K=H K \cdot B D$. Consequently, $P D \cdot B K=H K \cdot B D=C K \cdot Q^{\prime} D$, which yields $\frac{P D}{Q^{\prime} D}=\frac{C K}{B K}$. + +Since $\angle C K A=\angle K A C=\angle B D Q^{\prime}$, the triangles $C K B$ and $P D Q^{\prime}$ are similar, so $\angle P Q^{\prime} D=$ $\angle C B A=\angle Q^{\prime} B D$, as required. + +Comment 2. There exist several other ways to prove that $P Q^{\prime}$ is tangent to $\omega$. For instance, one may compute $\frac{P D}{P B}$ and $\frac{P Q^{\prime}}{P B}$ in terms of $A H$ and $H B$ to verify that $P Q^{\prime 2}=P D \cdot P B$, concluding that $P Q^{\prime}$ is tangent to $\omega$. + +Another possible approach is the following. As in Solution 2, we introduce the points $T$ and $Q^{\prime}$ and mention that the triangles $A B C$ and $D B Q^{\prime}$ are similar (see Figure 3). + +Let $M$ be the midpoint of $A D$, and let $L$ be the projection of $Q^{\prime}$ onto $A B$. Construct $E$ on the line $A B$ so that $E P$ is parallel to $A D$. Projecting from $P$, we get $(A, B ; H, E)=(A, D ; M, \infty)=-1$. Since $\frac{E A}{A B}=\frac{P D}{D B}$, the point $P$ is the image of $E$ under the similarity transform mapping $A B C$ to $D B Q^{\prime}$. Therefore, we have $(D, B ; L, P)=(A, B ; H, E)=-1$, which means that $Q^{\prime} D$ and $Q^{\prime} B$ are respectively the internal and the external bisectors of $\angle P Q^{\prime} L$. This implies that $P Q^{\prime}$ is tangent to $\omega$, as required. +![](https://cdn.mathpix.com/cropped/2024_11_18_62dbd531c72d0ff8164cg-48.jpg?height=415&width=1026&top_left_y=820&top_left_x=521) + +Figure 3 + +Solution 3. Introduce the points $T$ and $Q^{\prime}$ as in the previous solution. Note that $T$ lies on the circumcircle of $A B C$. Here we present yet another proof that $P Q^{\prime}$ is tangent to $\omega$. + +Let $\Omega$ be the circle completing the semicircle $\omega$. Construct a point $F$ symmetric to $C$ with respect to $A B$. Let $S \neq T$ be the second intersection point of $F T$ and $\Omega$ (see Figure 4). +![](https://cdn.mathpix.com/cropped/2024_11_18_62dbd531c72d0ff8164cg-48.jpg?height=772&width=855&top_left_y=1553&top_left_x=606) + +Figure 4 +Since $A C=A F$, we have $\angle D K C=\angle H C K=\angle C B A=\angle C T A=\angle D T S=180^{\circ}-$ $\angle S K D$. Thus, the points $C, K$, and $S$ are collinear. Notice also that $\angle Q^{\prime} K D=\angle Q^{\prime} T D=$ $\angle H C K=\angle K F H=180^{\circ}-\angle D K F$. This implies that the points $F, K$, and $Q^{\prime}$ are collinear. + +Applying PASCAL's theorem to the degenerate hexagon $K Q^{\prime} Q^{\prime} T S S$, we get that the tangents to $\Omega$ passing through $Q^{\prime}$ and $S$ intersect on $C F$. The relation $\angle Q^{\prime} T D=\angle D T S$ yields that $Q^{\prime}$ and $S$ are symmetric with respect to $B D$. Therefore, the two tangents also intersect on $B D$. Thus, the two tangents pass through $P$. Hence, $P Q^{\prime}$ is tangent to $\omega$, as needed. + +G4. Let $A B C$ be an acute triangle, and let $M$ be the midpoint of $A C$. A circle $\omega$ passing through $B$ and $M$ meets the sides $A B$ and $B C$ again at $P$ and $Q$, respectively. Let $T$ be the point such that the quadrilateral $B P T Q$ is a parallelogram. Suppose that $T$ lies on the circumcircle of the triangle $A B C$. Determine all possible values of $B T / B M$. +(Russia) +Answer. $\sqrt{2}$. +Solution 1. Let $S$ be the center of the parallelogram $B P T Q$, and let $B^{\prime} \neq B$ be the point on the ray $B M$ such that $B M=M B^{\prime}$ (see Figure 1). It follows that $A B C B^{\prime}$ is a parallelogram. Then, $\angle A B B^{\prime}=\angle P Q M$ and $\angle B B^{\prime} A=\angle B^{\prime} B C=\angle M P Q$, and so the triangles $A B B^{\prime}$ and $M Q P$ are similar. It follows that $A M$ and $M S$ are corresponding medians in these triangles. Hence, + +$$ +\angle S M P=\angle B^{\prime} A M=\angle B C A=\angle B T A . +$$ + +Since $\angle A C T=\angle P B T$ and $\angle T A C=\angle T B C=\angle B T P$, the triangles $T C A$ and $P B T$ are similar. Again, as $T M$ and $P S$ are corresponding medians in these triangles, we have + +$$ +\angle M T A=\angle T P S=\angle B Q P=\angle B M P . +$$ + +Now we deal separately with two cases. +Case 1. $\quad S$ does not lie on $B M$. Since the configuration is symmetric between $A$ and $C$, we may assume that $S$ and $A$ lie on the same side with respect to the line $B M$. + +Applying (1) and (2), we get + +$$ +\angle B M S=\angle B M P-\angle S M P=\angle M T A-\angle B T A=\angle M T B, +$$ + +and so the triangles $B S M$ and $B M T$ are similar. We now have $B M^{2}=B S \cdot B T=B T^{2} / 2$, so $B T=\sqrt{2} B M$. +Case 2. $\quad S$ lies on $B M$. It follows from (2) that $\angle B C A=\angle M T A=\angle B Q P=\angle B M P$ (see Figure 2). Thus, $P Q \| A C$ and $P M \| A T$. Hence, $B S / B M=B P / B A=B M / B T$, so $B T^{2}=2 B M^{2}$ and $B T=\sqrt{2} B M$. +![](https://cdn.mathpix.com/cropped/2024_11_18_62dbd531c72d0ff8164cg-49.jpg?height=960&width=730&top_left_y=1713&top_left_x=246) + +Figure 1 +![](https://cdn.mathpix.com/cropped/2024_11_18_62dbd531c72d0ff8164cg-49.jpg?height=749&width=729&top_left_y=1938&top_left_x=1092) + +Figure 2 + +Comment 1. Here is another way to show that the triangles $B S M$ and $B M T$ are similar. Denote by $\Omega$ the circumcircle of the triangle $A B C$. Let $R$ be the second point of intersection of $\omega$ and $\Omega$, and let $\tau$ be the spiral similarity centered at $R$ mapping $\omega$ to $\Omega$. Then, one may show that $\tau$ maps each point $X$ on $\omega$ to a point $Y$ on $\Omega$ such that $B, X$, and $Y$ are collinear (see Figure 3). If we let $K$ and $L$ be the second points of intersection of $B M$ with $\Omega$ and of $B T$ with $\omega$, respectively, then it follows that the triangle $M K T$ is the image of $S M L$ under $\tau$. We now obtain $\angle B S M=\angle T M B$, which implies the desired result. +![](https://cdn.mathpix.com/cropped/2024_11_18_62dbd531c72d0ff8164cg-50.jpg?height=758&width=757&top_left_y=752&top_left_x=204) + +Figure 3 +![](https://cdn.mathpix.com/cropped/2024_11_18_62dbd531c72d0ff8164cg-50.jpg?height=954&width=846&top_left_y=551&top_left_x=1002) + +Figure 4 + +Solution 2. Again, we denote by $\Omega$ the circumcircle of the triangle $A B C$. +Choose the points $X$ and $Y$ on the rays $B A$ and $B C$ respectively, so that $\angle M X B=\angle M B C$ and $\angle B Y M=\angle A B M$ (see Figure 4). Then the triangles $B M X$ and $Y M B$ are similar. Since $\angle X P M=\angle B Q M$, the points $P$ and $Q$ correspond to each other in these triangles. So, if $\overrightarrow{B P}=\mu \cdot \overrightarrow{B X}$, then $\overrightarrow{B Q}=(1-\mu) \cdot \overrightarrow{B Y}$. Thus + +$$ +\overrightarrow{B T}=\overrightarrow{B P}+\overrightarrow{B Q}=\overrightarrow{B Y}+\mu \cdot(\overrightarrow{B X}-\overrightarrow{B Y})=\overrightarrow{B Y}+\mu \cdot \overrightarrow{Y X} +$$ + +which means that $T$ lies on the line $X Y$. +Let $B^{\prime} \neq B$ be the point on the ray $B M$ such that $B M=M B^{\prime}$. Then $\angle M B^{\prime} A=$ $\angle M B C=\angle M X B$ and $\angle C B^{\prime} M=\angle A B M=\angle B Y M$. This means that the triangles $B M X$, $B A B^{\prime}, Y M B$, and $B^{\prime} C B$ are all similar; hence $B A \cdot B X=B M \cdot B B^{\prime}=B C \cdot B Y$. Thus there exists an inversion centered at $B$ which swaps $A$ with $X, M$ with $B^{\prime}$, and $C$ with $Y$. This inversion then swaps $\Omega$ with the line $X Y$, and hence it preserves $T$. Therefore, we have $B T^{2}=B M \cdot B B^{\prime}=2 B M^{2}$, and $B T=\sqrt{2} B M$. + +Solution 3. We begin with the following lemma. +Lemma. Let $A B C T$ be a cyclic quadrilateral. Let $P$ and $Q$ be points on the sides $B A$ and $B C$ respectively, such that $B P T Q$ is a parallelogram. Then $B P \cdot B A+B Q \cdot B C=B T^{2}$. +Proof. Let the circumcircle of the triangle $Q T C$ meet the line $B T$ again at $J$ (see Figure 5). The power of $B$ with respect to this circle yields + +$$ +B Q \cdot B C=B J \cdot B T +$$ + +We also have $\angle T J Q=180^{\circ}-\angle Q C T=\angle T A B$ and $\angle Q T J=\angle A B T$, and so the triangles $T J Q$ and $B A T$ are similar. We now have $T J / T Q=B A / B T$. Therefore, + +$$ +T J \cdot B T=T Q \cdot B A=B P \cdot B A +$$ + +Combining (3) and (4) now yields the desired result. +Let $X$ and $Y$ be the midpoints of $B A$ and $B C$ respectively (see Figure 6). Applying the lemma to the cyclic quadrilaterals $P B Q M$ and $A B C T$, we obtain + +$$ +B X \cdot B P+B Y \cdot B Q=B M^{2} +$$ + +and + +$$ +B P \cdot B A+B Q \cdot B C=B T^{2} +$$ + +Since $B A=2 B X$ and $B C=2 B Y$, we have $B T^{2}=2 B M^{2}$, and so $B T=\sqrt{2} B M$. +![](https://cdn.mathpix.com/cropped/2024_11_18_62dbd531c72d0ff8164cg-51.jpg?height=789&width=1464&top_left_y=933&top_left_x=296) + +Comment 2. Here we give another proof of the lemma using Ptolemy's theorem. We readily have + +$$ +T C \cdot B A+T A \cdot B C=A C \cdot B T +$$ + +The lemma now follows from + +$$ +\frac{B P}{T C}=\frac{B Q}{T A}=\frac{B T}{A C}=\frac{\sin \angle B C T}{\sin \angle A B C} +$$ + +G5. Let $A B C$ be a triangle with $C A \neq C B$. Let $D, F$, and $G$ be the midpoints of the sides $A B, A C$, and $B C$, respectively. A circle $\Gamma$ passing through $C$ and tangent to $A B$ at $D$ meets the segments $A F$ and $B G$ at $H$ and $I$, respectively. The points $H^{\prime}$ and $I^{\prime}$ are symmetric to $H$ and $I$ about $F$ and $G$, respectively. The line $H^{\prime} I^{\prime}$ meets $C D$ and $F G$ at $Q$ and $M$, respectively. The line $C M$ meets $\Gamma$ again at $P$. Prove that $C Q=Q P$. +(El Salvador) +Solution 1. We may assume that $C A>C B$. Observe that $H^{\prime}$ and $I^{\prime}$ lie inside the segments $C F$ and $C G$, respectively. Therefore, $M$ lies outside $\triangle A B C$ (see Figure 1). + +Due to the powers of points $A$ and $B$ with respect to the circle $\Gamma$, we have + +$$ +C H^{\prime} \cdot C A=A H \cdot A C=A D^{2}=B D^{2}=B I \cdot B C=C I^{\prime} \cdot C B +$$ + +Therefore, $C H^{\prime} \cdot C F=C I^{\prime} \cdot C G$. Hence, the quadrilateral $H^{\prime} I^{\prime} G F$ is cyclic, and so $\angle I^{\prime} H^{\prime} C=$ $\angle C G F$. + +Let $D F$ and $D G$ meet $\Gamma$ again at $R$ and $S$, respectively. We claim that the points $R$ and $S$ lie on the line $H^{\prime} I^{\prime}$. + +Observe that $F H^{\prime} \cdot F A=F H \cdot F C=F R \cdot F D$. Thus, the quadrilateral $A D H^{\prime} R$ is cyclic, and hence $\angle R H^{\prime} F=\angle F D A=\angle C G F=\angle I^{\prime} H^{\prime} C$. Therefore, the points $R, H^{\prime}$, and $I^{\prime}$ are collinear. Similarly, the points $S, H^{\prime}$, and $I^{\prime}$ are also collinear, and so all the points $R, H^{\prime}, Q, I^{\prime}, S$, and $M$ are all collinear. +![](https://cdn.mathpix.com/cropped/2024_11_18_62dbd531c72d0ff8164cg-52.jpg?height=689&width=803&top_left_y=1249&top_left_x=204) + +Figure 1 +![](https://cdn.mathpix.com/cropped/2024_11_18_62dbd531c72d0ff8164cg-52.jpg?height=684&width=826&top_left_y=1257&top_left_x=1023) + +Figure 2 + +Then, $\angle R S D=\angle R D A=\angle D F G$. Hence, the quadrilateral $R S G F$ is cyclic (see Figure 2). Therefore, $M H^{\prime} \cdot M I^{\prime}=M F \cdot M G=M R \cdot M S=M P \cdot M C$. Thus, the quadrilateral $C P I^{\prime} H^{\prime}$ is also cyclic. Let $\omega$ be its circumcircle. + +Notice that $\angle H^{\prime} C Q=\angle S D C=\angle S R C$ and $\angle Q C I^{\prime}=\angle C D R=\angle C S R$. Hence, $\triangle C H^{\prime} Q \sim \triangle R C Q$ and $\triangle C I^{\prime} Q \sim \triangle S C Q$, and therefore $Q H^{\prime} \cdot Q R=Q C^{2}=Q I^{\prime} \cdot Q S$. + +We apply the inversion with center $Q$ and radius $Q C$. Observe that the points $R, C$, and $S$ are mapped to $H^{\prime}, C$, and $I^{\prime}$, respectively. Therefore, the circumcircle $\Gamma$ of $\triangle R C S$ is mapped to the circumcircle $\omega$ of $\triangle H^{\prime} C I^{\prime}$. Since $P$ and $C$ belong to both circles and the point $C$ is preserved by the inversion, we have that $P$ is also mapped to itself. We then get $Q P^{2}=Q C^{2}$. Hence, $Q P=Q C$. + +Comment 1. The problem statement still holds when $\Gamma$ intersects the sides $C A$ and $C B$ outside segments $A F$ and $B G$, respectively. + +Solution 2. Let $X=H I \cap A B$, and let the tangent to $\Gamma$ at $C$ meet $A B$ at $Y$. Let $X C$ meet $\Gamma$ again at $X^{\prime}$ (see Figure 3). Projecting from $C, X$, and $C$ again, we have $(X, A ; D, B)=$ $\left(X^{\prime}, H ; D, I\right)=(C, I ; D, H)=(Y, B ; D, A)$. Since $A$ and $B$ are symmetric about $D$, it follows that $X$ and $Y$ are also symmetric about $D$. + +Now, Menelaus' theorem applied to $\triangle A B C$ with the line $H I X$ yields + +$$ +1=\frac{C H}{H A} \cdot \frac{B I}{I C} \cdot \frac{A X}{X B}=\frac{A H^{\prime}}{H^{\prime} C} \cdot \frac{C I^{\prime}}{I^{\prime} B} \cdot \frac{B Y}{Y A} . +$$ + +By the converse of Menelaus' theorem applied to $\triangle A B C$ with points $H^{\prime}, I^{\prime}, Y$, we get that the points $H^{\prime}, I^{\prime}, Y$ are collinear. +![](https://cdn.mathpix.com/cropped/2024_11_18_62dbd531c72d0ff8164cg-53.jpg?height=568&width=1382&top_left_y=744&top_left_x=340) + +Figure 3 +Let $T$ be the midpoint of $C D$, and let $O$ be the center of $\Gamma$. Let $C M$ meet $T Y$ at $N$. To avoid confusion, we clean some superfluous details out of the picture (see Figure 4). + +Let $V=M T \cap C Y$. Since $M T \| Y D$ and $D T=T C$, we get $C V=V Y$. Then CevA's theorem applied to $\triangle C T Y$ with the point $M$ yields + +$$ +1=\frac{T Q}{Q C} \cdot \frac{C V}{V Y} \cdot \frac{Y N}{N T}=\frac{T Q}{Q C} \cdot \frac{Y N}{N T} +$$ + +Therefore, $\frac{T Q}{Q C}=\frac{T N}{N Y}$. So, $N Q \| C Y$, and thus $N Q \perp O C$. +Note that the points $O, N, T$, and $Y$ are collinear. Therefore, $C Q \perp O N$. So, $Q$ is the orthocenter of $\triangle O C N$, and hence $O Q \perp C P$. Thus, $Q$ lies on the perpendicular bisector of $C P$, and therefore $C Q=Q P$, as required. +![](https://cdn.mathpix.com/cropped/2024_11_18_62dbd531c72d0ff8164cg-53.jpg?height=652&width=866&top_left_y=2027&top_left_x=595) + +Figure 4 + +Comment 2. The second part of Solution 2 provides a proof of the following more general statement, which does not involve a specific choice of $Q$ on $C D$. + +Let $Y C$ and $Y D$ be two tangents to a circle $\Gamma$ with center $O$ (see Figure 4). Let $\ell$ be the midline of $\triangle Y C D$ parallel to $Y D$. Let $Q$ and $M$ be two points on $C D$ and $\ell$, respectively, such that the line $Q M$ passes through $Y$. Then $O Q \perp C M$. + +G6. Let $A B C$ be an acute triangle with $A B>A C$, and let $\Gamma$ be its circumcircle. Let $H$, $M$, and $F$ be the orthocenter of the triangle, the midpoint of $B C$, and the foot of the altitude from $A$, respectively. Let $Q$ and $K$ be the two points on $\Gamma$ that satisfy $\angle A Q H=90^{\circ}$ and $\angle Q K H=90^{\circ}$. Prove that the circumcircles of the triangles $K Q H$ and $K F M$ are tangent to each other. +(Ukraine) +Solution 1. Let $A^{\prime}$ be the point diametrically opposite to $A$ on $\Gamma$. Since $\angle A Q A^{\prime}=90^{\circ}$ and $\angle A Q H=90^{\circ}$, the points $Q, H$, and $A^{\prime}$ are collinear. Similarly, if $Q^{\prime}$ denotes the point on $\Gamma$ diametrically opposite to $Q$, then $K, H$, and $Q^{\prime}$ are collinear. Let the line $A H F$ intersect $\Gamma$ again at $E$; it is known that $M$ is the midpoint of the segment $H A^{\prime}$ and that $F$ is the midpoint of $H E$. Let $J$ be the midpoint of $H Q^{\prime}$. + +Consider any point $T$ such that $T K$ is tangent to the circle $K Q H$ at $K$ with $Q$ and $T$ lying on different sides of $K H$ (see Figure 1). Then $\angle H K T=\angle H Q K$ and we are to prove that $\angle M K T=\angle C F K$. Thus it remains to show that $\angle H Q K=\angle C F K+\angle H K M$. Due to $\angle H Q K=90^{\circ}-\angle Q^{\prime} H A^{\prime}$ and $\angle C F K=90^{\circ}-\angle K F A$, this means the same as $\angle Q^{\prime} H A^{\prime}=$ $\angle K F A-\angle H K M$. Now, since the triangles $K H E$ and $A H Q^{\prime}$ are similar with $F$ and $J$ being the midpoints of corresponding sides, we have $\angle K F A=\angle H J A$, and analogously one may obtain $\angle H K M=\angle J Q H$. Thereby our task is reduced to verifying + +$$ +\angle Q^{\prime} H A^{\prime}=\angle H J A-\angle J Q H +$$ + +![](https://cdn.mathpix.com/cropped/2024_11_18_62dbd531c72d0ff8164cg-55.jpg?height=798&width=758&top_left_y=1257&top_left_x=289) + +Figure 1 +![](https://cdn.mathpix.com/cropped/2024_11_18_62dbd531c72d0ff8164cg-55.jpg?height=710&width=672&top_left_y=1344&top_left_x=1092) + +Figure 2 + +To avoid confusion, let us draw a new picture at this moment (see Figure 2). Owing to $\angle Q^{\prime} H A^{\prime}=\angle J Q H+\angle H J Q$ and $\angle H J A=\angle Q J A+\angle H J Q$, we just have to show that $2 \angle J Q H=\angle Q J A$. To this end, it suffices to remark that $A Q A^{\prime} Q^{\prime}$ is a rectangle and that $J$, being defined to be the midpoint of $H Q^{\prime}$, has to lie on the mid parallel of $Q A^{\prime}$ and $Q^{\prime} A$. + +Solution 2. We define the points $A^{\prime}$ and $E$ and prove that the ray $M H$ passes through $Q$ in the same way as in the first solution. Notice that the points $A^{\prime}$ and $E$ can play analogous roles to the points $Q$ and $K$, respectively: point $A^{\prime}$ is the second intersection of the line $M H$ with $\Gamma$, and $E$ is the point on $\Gamma$ with the property $\angle H E A^{\prime}=90^{\circ}$ (see Figure 3). + +In the circles $K Q H$ and $E A^{\prime} H$, the line segments $H Q$ and $H A^{\prime}$ are diameters, respectively; so, these circles have a common tangent $t$ at $H$, perpendicular to $M H$. Let $R$ be the radical center of the circles $A B C, K Q H$ and $E A^{\prime} H$. Their pairwise radical axes are the lines $Q K$, $A^{\prime} E$ and the line $t$; they all pass through $R$. Let $S$ be the midpoint of $H R$; by $\angle Q K H=$ +![](https://cdn.mathpix.com/cropped/2024_11_18_62dbd531c72d0ff8164cg-56.jpg?height=689&width=960&top_left_y=181&top_left_x=545) + +Figure 3 +$\angle H E A^{\prime}=90^{\circ}$, the quadrilateral $H E R K$ is cyclic and its circumcenter is $S$; hence we have $S K=S E=S H$. The line $B C$, being the perpendicular bisector of $H E$, passes through $S$. + +The circle $H M F$ also is tangent to $t$ at $H$; from the power of $S$ with respect to the circle $H M F$ we have + +$$ +S M \cdot S F=S H^{2}=S K^{2} +$$ + +So, the power of $S$ with respect to the circles $K Q H$ and $K F M$ is $S K^{2}$. Therefore, the line segment $S K$ is tangent to both circles at $K$. + +G7. Let $A B C D$ be a convex quadrilateral, and let $P, Q, R$, and $S$ be points on the sides $A B, B C, C D$, and $D A$, respectively. Let the line segments $P R$ and $Q S$ meet at $O$. Suppose that each of the quadrilaterals $A P O S, B Q O P, C R O Q$, and $D S O R$ has an incircle. Prove that the lines $A C, P Q$, and $R S$ are either concurrent or parallel to each other. +(Bulgaria) +Solution 1. Denote by $\gamma_{A}, \gamma_{B}, \gamma_{C}$, and $\gamma_{D}$ the incircles of the quadrilaterals $A P O S, B Q O P$, $C R O Q$, and $D S O R$, respectively. + +We start with proving that the quadrilateral $A B C D$ also has an incircle which will be referred to as $\Omega$. Denote the points of tangency as in Figure 1. It is well-known that $Q Q_{1}=O O_{1}$ (if $B C \| P R$, this is obvious; otherwise, one may regard the two circles involved as the incircle and an excircle of the triangle formed by the lines $O Q, P R$, and $B C$ ). Similarly, $O O_{1}=P P_{1}$. Hence we have $Q Q_{1}=P P_{1}$. The other equalities of segment lengths marked in Figure 1 can be proved analogously. These equalities, together with $A P_{1}=A S_{1}$ and similar ones, yield $A B+C D=A D+B C$, as required. +![](https://cdn.mathpix.com/cropped/2024_11_18_62dbd531c72d0ff8164cg-57.jpg?height=654&width=1377&top_left_y=958&top_left_x=342) + +Figure 1 + +Next, let us draw the lines parallel to $Q S$ through $P$ and $R$, and also draw the lines parallel to $P R$ through $Q$ and $S$. These lines form a parallelogram; denote its vertices by $A^{\prime}, B^{\prime}, C^{\prime}$, and $D^{\prime}$ as shown in Figure 2. + +Since the quadrilateral $A P O S$ has an incircle, we have $A P-A S=O P-O S=A^{\prime} S-A^{\prime} P$. It is well-known that in this case there also exists a circle $\omega_{A}$ tangent to the four rays $A P$, $A S, A^{\prime} P$, and $A^{\prime} S$. It is worth mentioning here that in case when, say, the lines $A B$ and $A^{\prime} B^{\prime}$ coincide, the circle $\omega_{A}$ is just tangent to $A B$ at $P$. We introduce the circles $\omega_{B}, \omega_{C}$, and $\omega_{D}$ in a similar manner. + +Assume that the radii of the circles $\omega_{A}$ and $\omega_{C}$ are different. Let $X$ be the center of the homothety having a positive scale factor and mapping $\omega_{A}$ to $\omega_{C}$. + +Now, Monge's theorem applied to the circles $\omega_{A}, \Omega$, and $\omega_{C}$ shows that the points $A, C$, and $X$ are collinear. Applying the same theorem to the circles $\omega_{A}, \omega_{B}$, and $\omega_{C}$, we see that the points $P, Q$, and $X$ are also collinear. Similarly, the points $R, S$, and $X$ are collinear, as required. + +If the radii of $\omega_{A}$ and $\omega_{C}$ are equal but these circles do not coincide, then the degenerate version of the same theorem yields that the three lines $A C, P Q$, and $R S$ are parallel to the line of centers of $\omega_{A}$ and $\omega_{C}$. + +Finally, we need to say a few words about the case when $\omega_{A}$ and $\omega_{C}$ coincide (and thus they also coincide with $\Omega, \omega_{B}$, and $\omega_{D}$ ). It may be regarded as the limit case in the following manner. +![](https://cdn.mathpix.com/cropped/2024_11_18_62dbd531c72d0ff8164cg-58.jpg?height=1012&width=1580&top_left_y=179&top_left_x=244) + +Figure 2 + +Let us fix the positions of $A, P, O$, and $S$ (thus we also fix the circles $\omega_{A}, \gamma_{A}, \gamma_{B}$, and $\gamma_{D}$ ). Now we vary the circle $\gamma_{C}$ inscribed into $\angle Q O R$; for each of its positions, one may reconstruct the lines $B C$ and $C D$ as the external common tangents to $\gamma_{B}, \gamma_{C}$ and $\gamma_{C}, \gamma_{D}$ different from $P R$ and $Q S$, respectively. After such variation, the circle $\Omega$ changes, so the result obtained above may be applied. + +Solution 2. Applying Menelaus' theorem to $\triangle A B C$ with the line $P Q$ and to $\triangle A C D$ with the line $R S$, we see that the line $A C$ meets $P Q$ and $R S$ at the same point (possibly at infinity) if and only if + +$$ +\frac{A P}{P B} \cdot \frac{B Q}{Q C} \cdot \frac{C R}{R D} \cdot \frac{D S}{S A}=1 +$$ + +So, it suffices to prove (1). +We start with the following result. +Lemma 1. Let $E F G H$ be a circumscribed quadrilateral, and let $M$ be its incenter. Then + +$$ +\frac{E F \cdot F G}{G H \cdot H E}=\frac{F M^{2}}{H M^{2}} +$$ + +Proof. Notice that $\angle E M H+\angle G M F=\angle F M E+\angle H M G=180^{\circ}, \angle F G M=\angle M G H$, and $\angle H E M=\angle M E F$ (see Figure 3). By the law of sines, we get + +$$ +\frac{E F}{F M} \cdot \frac{F G}{F M}=\frac{\sin \angle F M E \cdot \sin \angle G M F}{\sin \angle M E F \cdot \sin \angle F G M}=\frac{\sin \angle H M G \cdot \sin \angle E M H}{\sin \angle M G H \cdot \sin \angle H E M}=\frac{G H}{H M} \cdot \frac{H E}{H M} . +$$ + +![](https://cdn.mathpix.com/cropped/2024_11_18_62dbd531c72d0ff8164cg-59.jpg?height=569&width=646&top_left_y=452&top_left_x=225) + +Figure 3 +![](https://cdn.mathpix.com/cropped/2024_11_18_62dbd531c72d0ff8164cg-59.jpg?height=829&width=954&top_left_y=179&top_left_x=885) + +Figure 4 + +We denote by $I, J, K$, and $L$ the incenters of the quadrilaterals $A P O S, B Q O P, C R O Q$, and $D S O R$, respectively. Applying Lemma 1 to these four quadrilaterals we get + +$$ +\frac{A P \cdot P O}{O S \cdot S A} \cdot \frac{B Q \cdot Q O}{O P \cdot P B} \cdot \frac{C R \cdot R O}{O Q \cdot Q C} \cdot \frac{D S \cdot S O}{O R \cdot R D}=\frac{P I^{2}}{S I^{2}} \cdot \frac{Q J^{2}}{P J^{2}} \cdot \frac{R K^{2}}{Q K^{2}} \cdot \frac{S L^{2}}{R L^{2}} +$$ + +which reduces to + +$$ +\frac{A P}{P B} \cdot \frac{B Q}{Q C} \cdot \frac{C R}{R D} \cdot \frac{D S}{S A}=\frac{P I^{2}}{P J^{2}} \cdot \frac{Q J^{2}}{Q K^{2}} \cdot \frac{R K^{2}}{R L^{2}} \cdot \frac{S L^{2}}{S I^{2}} +$$ + +Next, we have $\angle I P J=\angle J O I=90^{\circ}$, and the line $O P$ separates $I$ and $J$ (see Figure 4). This means that the quadrilateral $I P J O$ is cyclic. Similarly, we get that the quadrilateral $J Q K O$ is cyclic with $\angle J Q K=90^{\circ}$. Thus, $\angle Q K J=\angle Q O J=\angle J O P=\angle J I P$. Hence, the right triangles $I P J$ and $K Q J$ are similar. Therefore, $\frac{P I}{P J}=\frac{Q K}{Q J}$. Likewise, we obtain $\frac{R K}{R L}=\frac{S I}{S L}$. These two equations together with (2) yield (1). +Comment. Instead of using the sine law, one may prove Lemma 1 by the following approach. +![](https://cdn.mathpix.com/cropped/2024_11_18_62dbd531c72d0ff8164cg-59.jpg?height=558&width=698&top_left_y=2028&top_left_x=685) + +Figure 5 +Let $N$ be the point such that $\triangle N H G \sim \triangle M E F$ and such that $N$ and $M$ lie on different sides of the line $G H$, as shown in Figure 5. Then $\angle G N H+\angle H M G=\angle F M E+\angle H M G=180^{\circ}$. So, +the quadrilateral $G N H M$ is cyclic. Thus, $\angle M N H=\angle M G H=\angle F G M$ and $\angle H M N=\angle H G N=$ $\angle E F M=\angle M F G$. Hence, $\triangle H M N \sim \triangle M F G$. Therefore, $\frac{H M}{H G}=\frac{H M}{H N} \cdot \frac{H N}{H G}=\frac{M F}{M G} \cdot \frac{E M}{E F}$. Similarly, we obtain $\frac{H M}{H E}=\frac{M F}{M E} \cdot \frac{G M}{G F}$. By multiplying these two equations, we complete the proof. + +Solution 3. We present another approach for showing (1) from Solution 2. +Lemma 2. Let $E F G H$ and $E^{\prime} F^{\prime} G^{\prime} H^{\prime}$ be circumscribed quadrilaterals such that $\angle E+\angle E^{\prime}=$ $\angle F+\angle F^{\prime}=\angle G+\angle G^{\prime}=\angle H+\angle H^{\prime}=180^{\circ}$. Then + +$$ +\frac{E F \cdot G H}{F G \cdot H E}=\frac{E^{\prime} F^{\prime} \cdot G^{\prime} H^{\prime}}{F^{\prime} G^{\prime} \cdot H^{\prime} E^{\prime}} +$$ + +Proof. Let $M$ and $M^{\prime}$ be the incenters of $E F G H$ and $E^{\prime} F^{\prime} G^{\prime} H^{\prime}$, respectively. We use the notation [XYZ] for the area of a triangle $X Y Z$. + +Taking into account the relation $\angle F M E+\angle F^{\prime} M^{\prime} E^{\prime}=180^{\circ}$ together with the analogous ones, we get + +$$ +\begin{aligned} +\frac{E F \cdot G H}{F G \cdot H E} & =\frac{[M E F] \cdot[M G H]}{[M F G] \cdot[M H E]}=\frac{M E \cdot M F \cdot \sin \angle F M E \cdot M G \cdot M H \cdot \sin \angle H M G}{M F \cdot M G \cdot \sin \angle G M F \cdot M H \cdot M E \cdot \sin \angle E M H} \\ +& =\frac{M^{\prime} E^{\prime} \cdot M^{\prime} F^{\prime} \cdot \sin \angle F^{\prime} M^{\prime} E^{\prime} \cdot M^{\prime} G^{\prime} \cdot M^{\prime} H^{\prime} \cdot \sin \angle H^{\prime} M^{\prime} G^{\prime}}{M^{\prime} F^{\prime} \cdot M^{\prime} G^{\prime} \cdot \sin \angle G^{\prime} M^{\prime} F^{\prime} \cdot M^{\prime} H^{\prime} \cdot M^{\prime} E^{\prime} \cdot \sin \angle E^{\prime} M^{\prime} H^{\prime}}=\frac{E^{\prime} F^{\prime} \cdot G^{\prime} H^{\prime}}{F^{\prime} G^{\prime} \cdot H^{\prime} E^{\prime}} . +\end{aligned} +$$ + +![](https://cdn.mathpix.com/cropped/2024_11_18_62dbd531c72d0ff8164cg-60.jpg?height=849&width=1089&top_left_y=1300&top_left_x=489) + +Figure 6 +Denote by $h$ the homothety centered at $O$ that maps the incircle of $C R O Q$ to the incircle of $A P O S$. Let $Q^{\prime}=h(Q), C^{\prime}=h(C), R^{\prime}=h(R), O^{\prime}=O, S^{\prime}=S, A^{\prime}=A$, and $P^{\prime}=P$. Furthermore, define $B^{\prime}=A^{\prime} P^{\prime} \cap C^{\prime} Q^{\prime}$ and $D^{\prime}=A^{\prime} S^{\prime} \cap C^{\prime} R^{\prime}$ as shown in Figure 6. Then + +$$ +\frac{A P \cdot O S}{P O \cdot S A}=\frac{A^{\prime} P^{\prime} \cdot O^{\prime} S^{\prime}}{P^{\prime} O^{\prime} \cdot S^{\prime} A^{\prime}} +$$ + +holds trivially. We also have + +$$ +\frac{C R \cdot O Q}{R O \cdot Q C}=\frac{C^{\prime} R^{\prime} \cdot O^{\prime} Q^{\prime}}{R^{\prime} O^{\prime} \cdot Q^{\prime} C^{\prime}} +$$ + +by the similarity of the quadrilaterals $C R O Q$ and $C^{\prime} R^{\prime} O^{\prime} Q^{\prime}$. + +Next, consider the circumscribed quadrilaterals $B Q O P$ and $B^{\prime} Q^{\prime} O^{\prime} P^{\prime}$ whose incenters lie on different sides of the quadrilaterals' shared side line $O P=O^{\prime} P^{\prime}$. Observe that $B Q \| B^{\prime} Q^{\prime}$ and that $B^{\prime}$ and $Q^{\prime}$ lie on the lines $B P$ and $Q O$, respectively. It is now easy to see that the two quadrilaterals satisfy the hypotheses of Lemma 2. Thus, we deduce + +$$ +\frac{B Q \cdot O P}{Q O \cdot P B}=\frac{B^{\prime} Q^{\prime} \cdot O^{\prime} P^{\prime}}{Q^{\prime} O^{\prime} \cdot P^{\prime} B^{\prime}} +$$ + +Similarly, we get + +$$ +\frac{D S \cdot O R}{S O \cdot R D}=\frac{D^{\prime} S^{\prime} \cdot O^{\prime} R^{\prime}}{S^{\prime} O^{\prime} \cdot R^{\prime} D^{\prime}} +$$ + +Multiplying these four equations, we obtain + +$$ +\frac{A P}{P B} \cdot \frac{B Q}{Q C} \cdot \frac{C R}{R D} \cdot \frac{D S}{S A}=\frac{A^{\prime} P^{\prime}}{P^{\prime} B^{\prime}} \cdot \frac{B^{\prime} Q^{\prime}}{Q^{\prime} C^{\prime}} \cdot \frac{C^{\prime} R^{\prime}}{R^{\prime} D^{\prime}} \cdot \frac{D^{\prime} S^{\prime}}{S^{\prime} A^{\prime}} +$$ + +Finally, we apply BRIANChon's theorem to the circumscribed hexagon $A^{\prime} P^{\prime} R^{\prime} C^{\prime} Q^{\prime} S^{\prime}$ and deduce that the lines $A^{\prime} C^{\prime}, P^{\prime} Q^{\prime}$, and $R^{\prime} S^{\prime}$ are either concurrent or parallel to each other. So, by Menelaus' theorem, we obtain + +$$ +\frac{A^{\prime} P^{\prime}}{P^{\prime} B^{\prime}} \cdot \frac{B^{\prime} Q^{\prime}}{Q^{\prime} C^{\prime}} \cdot \frac{C^{\prime} R^{\prime}}{R^{\prime} D^{\prime}} \cdot \frac{D^{\prime} S^{\prime}}{S^{\prime} A^{\prime}}=1 +$$ + +This equation together with (3) yield (1). + +G8. A triangulation of a convex polygon $\Pi$ is a partitioning of $\Pi$ into triangles by diagonals having no common points other than the vertices of the polygon. We say that a triangulation is a Thaiangulation if all triangles in it have the same area. + +Prove that any two different Thaiangulations of a convex polygon $\Pi$ differ by exactly two triangles. (In other words, prove that it is possible to replace one pair of triangles in the first Thaiangulation with a different pair of triangles so as to obtain the second Thaiangulation.) +(Bulgaria) +Solution 1. We denote by [S] the area of a polygon $S$. +Recall that each triangulation of a convex $n$-gon has exactly $n-2$ triangles. This means that all triangles in any two Thaiangulations of a convex polygon $\Pi$ have the same area. + +Let $\mathcal{T}$ be a triangulation of a convex polygon $\Pi$. If four vertices $A, B, C$, and $D$ of $\Pi$ form a parallelogram, and $\mathcal{T}$ contains two triangles whose union is this parallelogram, then we say that $\mathcal{T}$ contains parallelogram $A B C D$. Notice here that if two Thaiangulations $\mathcal{T}_{1}$ and $\mathcal{T}_{2}$ of $\Pi$ differ by two triangles, then the union of these triangles is a quadrilateral each of whose diagonals bisects its area, i.e., a parallelogram. + +We start with proving two properties of triangulations. +Lemma 1. A triangulation of a convex polygon $\Pi$ cannot contain two parallelograms. +Proof. Arguing indirectly, assume that $P_{1}$ and $P_{2}$ are two parallelograms contained in some triangulation $\mathcal{T}$. If they have a common triangle in $\mathcal{T}$, then we may assume that $P_{1}$ consists of triangles $A B C$ and $A D C$ of $\mathcal{T}$, while $P_{2}$ consists of triangles $A D C$ and $C D E$ (see Figure 1). But then $B C\|A D\| C E$, so the three vertices $B, C$, and $E$ of $\Pi$ are collinear, which is absurd. + +Assume now that $P_{1}$ and $P_{2}$ contain no common triangle. Let $P_{1}=A B C D$. The sides $A B$, $B C, C D$, and $D A$ partition $\Pi$ into several parts, and $P_{2}$ is contained in one of them; we may assume that this part is cut off from $P_{1}$ by $A D$. Then one may label the vertices of $P_{2}$ by $X$, $Y, Z$, and $T$ so that the polygon $A B C D X Y Z T$ is convex (see Figure 2; it may happen that $D=X$ and/or $T=A$, but still this polygon has at least six vertices). But the sum of the external angles of this polygon at $B, C, Y$, and $Z$ is already $360^{\circ}$, which is impossible. A final contradiction. +![](https://cdn.mathpix.com/cropped/2024_11_18_62dbd531c72d0ff8164cg-62.jpg?height=458&width=269&top_left_y=1687&top_left_x=251) + +Figure 1 +![](https://cdn.mathpix.com/cropped/2024_11_18_62dbd531c72d0ff8164cg-62.jpg?height=341&width=601&top_left_y=1800&top_left_x=613) + +Figure 2 +![](https://cdn.mathpix.com/cropped/2024_11_18_62dbd531c72d0ff8164cg-62.jpg?height=406&width=484&top_left_y=1733&top_left_x=1323) + +Figure 3 + +Lemma 2. Every triangle in a Thaiangulation $\mathcal{T}$ of $\Pi$ contains a side of $\Pi$. +Proof. Let $A B C$ be a triangle in $\mathcal{T}$. Apply an affine transform such that $A B C$ maps to an equilateral triangle; let $A^{\prime} B^{\prime} C^{\prime}$ be the image of this triangle, and $\Pi^{\prime}$ be the image of $\Pi$. Clearly, $\mathcal{T}$ maps into a Thaiangulation $\mathcal{T}^{\prime}$ of $\Pi^{\prime}$. + +Assume that none of the sides of $\triangle A^{\prime} B^{\prime} C^{\prime}$ is a side of $\Pi^{\prime}$. Then $\mathcal{T}^{\prime}$ contains some other triangles with these sides, say, $A^{\prime} B^{\prime} Z, C^{\prime} A^{\prime} Y$, and $B^{\prime} C^{\prime} X$; notice that $A^{\prime} Z B^{\prime} X C^{\prime} Y$ is a convex hexagon (see Figure 3). The sum of its external angles at $X, Y$, and $Z$ is less than $360^{\circ}$. So one of these angles (say, at $Z$ ) is less than $120^{\circ}$, hence $\angle A^{\prime} Z B^{\prime}>60^{\circ}$. Then $Z$ lies on a circular arc subtended by $A^{\prime} B^{\prime}$ and having angular measure less than $240^{\circ}$; consequently, the altitude $Z H$ of $\triangle A^{\prime} B^{\prime} Z$ is less than $\sqrt{3} A^{\prime} B^{\prime} / 2$. Thus $\left[A^{\prime} B^{\prime} Z\right]<\left[A^{\prime} B^{\prime} C^{\prime}\right]$, and $\mathcal{T}^{\prime}$ is not a Thaiangulation. A contradiction. + +Now we pass to the solution. We say that a triangle in a triangulation of $\Pi$ is an ear if it contains two sides of $\Pi$. Note that each triangulation of a polygon contains some ear. + +Arguing indirectly, we choose a convex polygon $\Pi$ with the least possible number of sides such that some two Thaiangulations $\mathcal{T}_{1}$ and $\mathcal{T}_{2}$ of $\Pi$ violate the problem statement (thus $\Pi$ has at least five sides). Consider now any ear $A B C$ in $\mathcal{T}_{1}$, with $A C$ being a diagonal of $\Pi$. If $\mathcal{T}_{2}$ also contains $\triangle A B C$, then one may cut $\triangle A B C$ off from $\Pi$, getting a polygon with a smaller number of sides which also violates the problem statement. This is impossible; thus $\mathcal{T}_{2}$ does not contain $\triangle A B C$. + +Next, $\mathcal{T}_{1}$ contains also another triangle with side $A C$, say $\triangle A C D$. By Lemma 2 , this triangle contains a side of $\Pi$, so $D$ is adjacent to either $A$ or $C$ on the boundary of $\Pi$. We may assume that $D$ is adjacent to $C$. + +Assume that $\mathcal{T}_{2}$ does not contain the triangle $B C D$. Then it contains two different triangles $B C X$ and $C D Y$ (possibly, with $X=Y$ ); since these triangles have no common interior points, the polygon $A B C D Y X$ is convex (see Figure 4). But, since $[A B C]=[B C X]=$ $[A C D]=[C D Y]$, we get $A X \| B C$ and $A Y \| C D$ which is impossible. Thus $\mathcal{T}_{2}$ contains $\triangle B C D$. + +Therefore, $[A B D]=[A B C]+[A C D]-[B C D]=[A B C]$, and $A B C D$ is a parallelogram contained in $\mathcal{T}_{1}$. Let $\mathcal{T}^{\prime}$ be the Thaiangulation of $\Pi$ obtained from $\mathcal{T}_{1}$ by replacing the diagonal $A C$ with $B D$; then $\mathcal{T}^{\prime}$ is distinct from $\mathcal{T}_{2}$ (otherwise $\mathcal{T}_{1}$ and $\mathcal{T}_{2}$ would differ by two triangles). Moreover, $\mathcal{T}^{\prime}$ shares a common ear $B C D$ with $\mathcal{T}_{2}$. As above, cutting this ear away we obtain that $\mathcal{T}_{2}$ and $\mathcal{T}^{\prime}$ differ by two triangles forming a parallelogram different from $A B C D$. Thus $\mathcal{T}^{\prime}$ contains two parallelograms, which contradicts Lemma 1. +![](https://cdn.mathpix.com/cropped/2024_11_18_62dbd531c72d0ff8164cg-63.jpg?height=455&width=1166&top_left_y=1372&top_left_x=199) + +Figure 4 +![](https://cdn.mathpix.com/cropped/2024_11_18_62dbd531c72d0ff8164cg-63.jpg?height=412&width=506&top_left_y=1416&top_left_x=1363) + +Figure 5 + +Comment 1. Lemma 2 is equivalent to the well-known Erdős-Debrunner inequality stating that for any triangle $P Q R$ and any points $A, B, C$ lying on the sides $Q R, R P$, and $P Q$, respectively, we have + +$$ +[A B C] \geqslant \min \{[A B R],[B C P],[C A Q]\} +$$ + +To derive this inequality from Lemma 2, one may assume that (1) does not hold, and choose some points $X, Y$, and $Z$ inside the triangles $B C P, C A Q$, and $A B R$, respectively, so that $[A B C]=$ $[A B Z]=[B C X]=[C A Y]$. Then a convex hexagon $A Z B X C Y$ has a Thaiangulation containing $\triangle A B C$, which contradicts Lemma 2. + +Conversely, assume that a Thaiangulation $\mathcal{T}$ of $\Pi$ contains a triangle $A B C$ none of whose sides is a side of $\Pi$, and let $A B Z, A Y C$, and $X B C$ be other triangles in $\mathcal{T}$ containing the corresponding sides. Then $A Z B X C Y$ is a convex hexagon. + +Consider the lines through $A, B$, and $C$ parallel to $Y Z, Z X$, and $X Y$, respectively. They form a triangle $X^{\prime} Y^{\prime} Z^{\prime}$ similar to $\triangle X Y Z$ (see Figure 5). By (1) we have + +$$ +[A B C] \geqslant \min \left\{\left[A B Z^{\prime}\right],\left[B C X^{\prime}\right],\left[C A Y^{\prime}\right]\right\}>\min \{[A B Z],[B C X],[C A Y]\}, +$$ + +so $\mathcal{T}$ is not a Thaiangulation. + +Solution 2. We will make use of the preliminary observations from Solution 1, together with Lemma 1. + +Arguing indirectly, we choose a convex polygon $\Pi$ with the least possible number of sides such that some two Thaiangulations $\mathcal{T}_{1}$ and $\mathcal{T}_{2}$ of $\Pi$ violate the statement (thus $\Pi$ has at least five sides). Assume that $\mathcal{T}_{1}$ and $\mathcal{T}_{2}$ share a diagonal $d$ splitting $\Pi$ into two smaller polygons $\Pi_{1}$ and $\Pi_{2}$. Since the problem statement holds for any of them, the induced Thaiangulations of each of $\Pi_{i}$ differ by two triangles forming a parallelogram (the Thaiangulations induced on $\Pi_{i}$ by $\mathcal{T}_{1}$ and $T_{2}$ may not coincide, otherwise $\mathcal{T}_{1}$ and $\mathcal{T}_{2}$ would differ by at most two triangles). But both these parallelograms are contained in $\mathcal{T}_{1}$; this contradicts Lemma 1. Therefore, $\mathcal{T}_{1}$ and $\mathcal{T}_{2}$ share no diagonal. Hence they also share no triangle. + +We consider two cases. +Case 1. Assume that some vertex $B$ of $\Pi$ is an endpoint of some diagonal in $\mathcal{T}_{1}$, as well as an endpoint of some diagonal in $\mathcal{T}_{2}$. + +Let $A$ and $C$ be the vertices of $\Pi$ adjacent to $B$. Then $\mathcal{T}_{1}$ contains some triangles $A B X$ and $B C Y$, while $\mathcal{T}_{2}$ contains some triangles $A B X^{\prime}$ and $B C Y^{\prime}$. Here, some of the points $X$, $X^{\prime}, Y$, and $Y^{\prime}$ may coincide; however, in view of our assumption together with the fact that $\mathcal{T}_{1}$ and $\mathcal{T}_{2}$ share no triangle, all four triangles $A B X, B C Y, A B X^{\prime}$, and $B C Y^{\prime}$ are distinct. + +Since $[A B X]=[B C Y]=\left[A B X^{\prime}\right]=\left[B C Y^{\prime}\right]$, we have $X X^{\prime} \| A B$ and $Y Y^{\prime} \| B C$. Now, if $X=Y$, then $X^{\prime}$ and $Y^{\prime}$ lie on different lines passing through $X$ and are distinct from that point, so that $X^{\prime} \neq Y^{\prime}$. In this case, we may switch the two Thaiangulations. So, hereafter we assume that $X \neq Y$. + +In the convex pentagon $A B C Y X$ we have either $\angle B A X+\angle A X Y>180^{\circ}$ or $\angle X Y C+$ $\angle Y C B>180^{\circ}$ (or both); due to the symmetry, we may assume that the first inequality holds. Let $r$ be the ray emerging from $X$ and co-directed with $\overrightarrow{A B}$; our inequality shows that $r$ points to the interior of the pentagon (and thus to the interior of $\Pi$ ). Therefore, the ray opposite to $r$ points outside $\Pi$, so $X^{\prime}$ lies on $r$; moreover, $X^{\prime}$ lies on the "arc" $C Y$ of $\Pi$ not containing $X$. So the segments $X X^{\prime}$ and $Y B$ intersect (see Figure 6). + +Let $O$ be the intersection point of the rays $r$ and $B C$. Since the triangles $A B X^{\prime}$ and $B C Y^{\prime}$ have no common interior points, $Y^{\prime}$ must lie on the "arc" $C X^{\prime}$ which is situated inside the triangle $X B O$. Therefore, the line $Y Y^{\prime}$ meets two sides of $\triangle X B O$, none of which may be $X B$ (otherwise the diagonals $X B$ and $Y Y^{\prime}$ would share a common point). Thus $Y Y^{\prime}$ intersects $B O$, which contradicts $Y Y^{\prime} \| B C$. +![](https://cdn.mathpix.com/cropped/2024_11_18_62dbd531c72d0ff8164cg-64.jpg?height=518&width=892&top_left_y=1928&top_left_x=588) + +Figure 6 + +Case 2. In the remaining case, each vertex of $\Pi$ is an endpoint of a diagonal in at most one of $\mathcal{T}_{1}$ and $\mathcal{T}_{2}$. On the other hand, a triangulation cannot contain two consecutive vertices with no diagonals from each. Therefore, the vertices of $\Pi$ alternatingly emerge diagonals in $\mathcal{T}_{1}$ and in $\mathcal{T}_{2}$. In particular, $\Pi$ has an even number of sides. + +Next, we may choose five consecutive vertices $A, B, C, D$, and $E$ of $\Pi$ in such a way that + +$$ +\angle A B C+\angle B C D>180^{\circ} \text { and } \angle B C D+\angle C D E>180^{\circ} . +$$ + +In order to do this, it suffices to choose three consecutive vertices $B, C$, and $D$ of $\Pi$ such that the sum of their external angles is at most $180^{\circ}$. This is possible, since $\Pi$ has at least six sides. +![](https://cdn.mathpix.com/cropped/2024_11_18_62dbd531c72d0ff8164cg-65.jpg?height=529&width=701&top_left_y=521&top_left_x=683) + +Figure 7 +We may assume that $\mathcal{T}_{1}$ has no diagonals from $B$ and $D$ (and thus contains the triangles $A B C$ and $C D E$ ), while $\mathcal{T}_{2}$ has no diagonals from $A, C$, and $E$ (and thus contains the triangle $B C D)$. Now, since $[A B C]=[B C D]=[C D E]$, we have $A D \| B C$ and $B E \| C D$ (see Figure 7). By (2) this yields that $A D>B C$ and $B E>C D$. Let $X=A C \cap B D$ and $Y=C E \cap B D$; then the inequalities above imply that $A X>C X$ and $E Y>C Y$. + +Finally, $\mathcal{T}_{2}$ must also contain some triangle $B D Z$ with $Z \neq C$; then the ray $C Z$ lies in the angle $A C E$. Since $[B C D]=[B D Z]$, the diagonal $B D$ bisects $C Z$. Together with the inequalities above, this yields that $Z$ lies inside the triangle $A C E$ (but $Z$ is distinct from $A$ and $E$ ), which is impossible. The final contradiction. + +Comment 2. Case 2 may also be accomplished with the use of Lemma 2. Indeed, since each triangulation of an $n$-gon contains $n-2$ triangles neither of which may contain three sides of $\Pi$, Lemma 2 yields that each Thaiangulation contains exactly two ears. But each vertex of $\Pi$ is a vertex of an ear either in $\mathcal{T}_{1}$ or in $\mathcal{T}_{2}$, so $\Pi$ cannot have more than four vertices. + +## Number Theory + +N1. Determine all positive integers $M$ for which the sequence $a_{0}, a_{1}, a_{2}, \ldots$, defined by $a_{0}=\frac{2 M+1}{2}$ and $a_{k+1}=a_{k}\left\lfloor a_{k}\right\rfloor$ for $k=0,1,2, \ldots$, contains at least one integer term. +(Luxembourg) +Answer. All integers $M \geqslant 2$. +Solution 1. Define $b_{k}=2 a_{k}$ for all $k \geqslant 0$. Then + +$$ +b_{k+1}=2 a_{k+1}=2 a_{k}\left\lfloor a_{k}\right\rfloor=b_{k}\left\lfloor\frac{b_{k}}{2}\right\rfloor . +$$ + +Since $b_{0}$ is an integer, it follows that $b_{k}$ is an integer for all $k \geqslant 0$. +Suppose that the sequence $a_{0}, a_{1}, a_{2}, \ldots$ does not contain any integer term. Then $b_{k}$ must be an odd integer for all $k \geqslant 0$, so that + +$$ +b_{k+1}=b_{k}\left\lfloor\frac{b_{k}}{2}\right\rfloor=\frac{b_{k}\left(b_{k}-1\right)}{2} +$$ + +Hence + +$$ +b_{k+1}-3=\frac{b_{k}\left(b_{k}-1\right)}{2}-3=\frac{\left(b_{k}-3\right)\left(b_{k}+2\right)}{2} +$$ + +for all $k \geqslant 0$. +Suppose that $b_{0}-3>0$. Then equation (2) yields $b_{k}-3>0$ for all $k \geqslant 0$. For each $k \geqslant 0$, define $c_{k}$ to be the highest power of 2 that divides $b_{k}-3$. Since $b_{k}-3$ is even for all $k \geqslant 0$, the number $c_{k}$ is positive for every $k \geqslant 0$. + +Note that $b_{k}+2$ is an odd integer. Therefore, from equation (2), we have that $c_{k+1}=c_{k}-1$. Thus, the sequence $c_{0}, c_{1}, c_{2}, \ldots$ of positive integers is strictly decreasing, a contradiction. So, $b_{0}-3 \leqslant 0$, which implies $M=1$. + +For $M=1$, we can check that the sequence is constant with $a_{k}=\frac{3}{2}$ for all $k \geqslant 0$. Therefore, the answer is $M \geqslant 2$. + +Solution 2. We provide an alternative way to show $M=1$ once equation (1) has been reached. We claim that $b_{k} \equiv 3\left(\bmod 2^{m}\right)$ for all $k \geqslant 0$ and $m \geqslant 1$. If this is true, then we would have $b_{k}=3$ for all $k \geqslant 0$ and hence $M=1$. + +To establish our claim, we proceed by induction on $m$. The base case $b_{k} \equiv 3(\bmod 2)$ is true for all $k \geqslant 0$ since $b_{k}$ is odd. Now suppose that $b_{k} \equiv 3\left(\bmod 2^{m}\right)$ for all $k \geqslant 0$. Hence $b_{k}=2^{m} d_{k}+3$ for some integer $d_{k}$. We have + +$$ +3 \equiv b_{k+1} \equiv\left(2^{m} d_{k}+3\right)\left(2^{m-1} d_{k}+1\right) \equiv 3 \cdot 2^{m-1} d_{k}+3 \quad\left(\bmod 2^{m}\right) +$$ + +so that $d_{k}$ must be even. This implies that $b_{k} \equiv 3\left(\bmod 2^{m+1}\right)$, as required. +Comment. The reason the number 3 which appears in both solutions is important, is that it is a nontrivial fixed point of the recurrence relation for $b_{k}$. + +N2. Let $a$ and $b$ be positive integers such that $a!b$ ! is a multiple of $a!+b!$. Prove that $3 a \geqslant 2 b+2$. +(United Kingdom) +Solution 1. If $a>b$, we immediately get $3 a \geqslant 2 b+2$. In the case $a=b$, the required inequality is equivalent to $a \geqslant 2$, which can be checked easily since $(a, b)=(1,1)$ does not satisfy $a!+b!\mid a!b!$. We now assume $aa$ !, which is impossible. We observe that $c!\mid M$ since $M$ is a product of $c$ consecutive integers. Thus $\operatorname{gcd}(1+M, c!)=1$, which implies + +$$ +1+M \left\lvert\, \frac{a!}{c!}=(c+1)(c+2) \cdots a\right. +$$ + +If $a \leqslant 2 c$, then $\frac{a!}{c!}$ is a product of $a-c \leqslant c$ integers not exceeding $a$ whereas $M$ is a product of $c$ integers exceeding $a$. Therefore, $1+M>\frac{a!}{c!}$, which is a contradiction. + +It remains to exclude the case $a=2 c+1$. Since $a+1=2(c+1)$, we have $c+1 \mid M$. Hence, we can deduce from (1) that $1+M \mid(c+2)(c+3) \cdots a$. Now $(c+2)(c+3) \cdots a$ is a product of $a-c-1=c$ integers not exceeding $a$; thus it is smaller than $1+M$. Again, we arrive at a contradiction. + +Comment 1. One may derive a weaker version of (1) and finish the problem as follows. After assuming $a \leqslant 2 c+1$, we have $\left\lfloor\frac{a}{2}\right\rfloor \leqslant c$, so $\left.\left\lfloor\frac{a}{2}\right\rfloor!\right\rvert\, M$. Therefore, + +$$ +1+M \left\lvert\,\left(\left\lfloor\frac{a}{2}\right\rfloor+1\right)\left(\left\lfloor\frac{a}{2}\right\rfloor+2\right) \cdots a\right. +$$ + +Observe that $\left(\left\lfloor\frac{a}{2}\right\rfloor+1\right)\left(\left\lfloor\frac{a}{2}\right\rfloor+2\right) \cdots a$ is a product of $\left\lceil\frac{a}{2}\right\rceil$ integers not exceeding $a$. This leads to a contradiction when $a$ is even since $\left\lceil\frac{a}{2}\right\rceil=\frac{a}{2} \leqslant c$ and $M$ is a product of $c$ integers exceeding $a$. + +When $a$ is odd, we can further deduce that $1+M \left\lvert\,\left(\frac{a+3}{2}\right)\left(\frac{a+5}{2}\right) \cdots a\right.$ since $\left.\left\lfloor\frac{a}{2}\right\rfloor+1=\frac{a+1}{2} \right\rvert\, a+1$. Now $\left(\frac{a+3}{2}\right)\left(\frac{a+5}{2}\right) \cdots a$ is a product of $\frac{a-1}{2} \leqslant c$ numbers not exceeding $a$, and we get a contradiction. + +Solution 2. As in Solution 1, we may assume that $aa+c$; otherwise, $a+1 \leqslant 2 c+2 \leqslant 2 p \leqslant a+c$ so $p \mid N-1$, again impossible. Thus, we have $p \in\left(\frac{a+c}{2}, a\right]$, and $p^{2} \nmid(a+c)$ ! since $2 p>a+c$. Therefore, $p^{2} \nmid N$ as well. + +If $a \leqslant c+2$, then the interval $\left(\frac{a+c}{2}, a\right]$ contains at most one integer and hence at most one prime number, which has to be $a$. Since $p^{2} \nmid N$, we must have $N=p=a$ or $N=1$, which is absurd since $N>a \geqslant 1$. Thus, we have $a \geqslant c+3$, and so $\frac{a+c+1}{2} \geqslant c+2$. It follows that $p$ lies in the interval $[c+2, a]$. + +Thus, every prime appearing in the prime factorization of $N$ lies in the interval $[c+2, a]$, and its exponent is exactly 1 . So we must have $N \mid(c+2)(c+3) \cdots a$. However, $(c+2)(c+3) \cdots a$ is a product of $a-c-1 \leqslant c$ numbers not exceeding $a$, so it is less than $N$. This is a contradiction. + +Comment 2. The original problem statement also asks to determine when the equality $3 a=2 b+2$ holds. It can be checked that the answer is $(a, b)=(2,2),(4,5)$. + +N3. Let $m$ and $n$ be positive integers such that $m>n$. Define $x_{k}=(m+k) /(n+k)$ for $k=$ $1,2, \ldots, n+1$. Prove that if all the numbers $x_{1}, x_{2}, \ldots, x_{n+1}$ are integers, then $x_{1} x_{2} \cdots x_{n+1}-1$ is divisible by an odd prime. +(Austria) +Solution. Assume that $x_{1}, x_{2}, \ldots, x_{n+1}$ are integers. Define the integers + +$$ +a_{k}=x_{k}-1=\frac{m+k}{n+k}-1=\frac{m-n}{n+k}>0 +$$ + +for $k=1,2, \ldots, n+1$. +Let $P=x_{1} x_{2} \cdots x_{n+1}-1$. We need to prove that $P$ is divisible by an odd prime, or in other words, that $P$ is not a power of 2 . To this end, we investigate the powers of 2 dividing the numbers $a_{k}$. + +Let $2^{d}$ be the largest power of 2 dividing $m-n$, and let $2^{c}$ be the largest power of 2 not exceeding $2 n+1$. Then $2 n+1 \leqslant 2^{c+1}-1$, and so $n+1 \leqslant 2^{c}$. We conclude that $2^{c}$ is one of the numbers $n+1, n+2, \ldots, 2 n+1$, and that it is the only multiple of $2^{c}$ appearing among these numbers. Let $\ell$ be such that $n+\ell=2^{c}$. Since $\frac{m-n}{n+\ell}$ is an integer, we have $d \geqslant c$. Therefore, $2^{d-c+1} \nmid a_{\ell}=\frac{m-n}{n+\ell}$, while $2^{d-c+1} \mid a_{k}$ for all $k \in\{1, \ldots, n+1\} \backslash\{\ell\}$. + +Computing modulo $2^{d-c+1}$, we get + +$$ +P=\left(a_{1}+1\right)\left(a_{2}+1\right) \cdots\left(a_{n+1}+1\right)-1 \equiv\left(a_{\ell}+1\right) \cdot 1^{n}-1 \equiv a_{\ell} \not \equiv 0 \quad\left(\bmod 2^{d-c+1}\right) +$$ + +Therefore, $2^{d-c+1} \nmid P$. +On the other hand, for any $k \in\{1, \ldots, n+1\} \backslash\{\ell\}$, we have $2^{d-c+1} \mid a_{k}$. So $P \geqslant a_{k} \geqslant 2^{d-c+1}$, and it follows that $P$ is not a power of 2 . + +Comment. Instead of attempting to show that $P$ is not a power of 2 , one may try to find an odd factor of $P$ (greater than 1) as follows: + +From $a_{k}=\frac{m-n}{n+k} \in \mathbb{Z}_{>0}$, we get that $m-n$ is divisible by $n+1, n+2, \ldots, 2 n+1$, and thus it is also divisible by their least common multiple $L$. So $m-n=q L$ for some positive integer $q$; hence $x_{k}=q \cdot \frac{L}{n+k}+1$. + +Then, since $n+1 \leqslant 2^{c}=n+\ell \leqslant 2 n+1 \leqslant 2^{c+1}-1$, we have $2^{c} \mid L$, but $2^{c+1} \nmid L$. So $\frac{L}{n+\ell}$ is odd, while $\frac{L}{n+k}$ is even for $k \neq \ell$. Computing modulo $2 q$ yields + +$$ +x_{1} x_{2} \cdots x_{n+1}-1 \equiv(q+1) \cdot 1^{n}-1 \equiv q \quad(\bmod 2 q) +$$ + +Thus, $x_{1} x_{2} \cdots x_{n+1}-1=2 q r+q=q(2 r+1)$ for some integer $r$. +Since $x_{1} x_{2} \cdots x_{n+1}-1 \geqslant x_{1} x_{2}-1 \geqslant(q+1)^{2}-1>q$, we have $r \geqslant 1$. This implies that $x_{1} x_{2} \cdots x_{n+1}-1$ is divisible by an odd prime. + +N4. Suppose that $a_{0}, a_{1}, \ldots$ and $b_{0}, b_{1}, \ldots$ are two sequences of positive integers satisfying $a_{0}, b_{0} \geqslant 2$ and + +$$ +a_{n+1}=\operatorname{gcd}\left(a_{n}, b_{n}\right)+1, \quad b_{n+1}=\operatorname{lcm}\left(a_{n}, b_{n}\right)-1 +$$ + +for all $n \geqslant 0$. Prove that the sequence $\left(a_{n}\right)$ is eventually periodic; in other words, there exist integers $N \geqslant 0$ and $t>0$ such that $a_{n+t}=a_{n}$ for all $n \geqslant N$. +(France) +Solution 1. Let $s_{n}=a_{n}+b_{n}$. Notice that if $a_{n} \mid b_{n}$, then $a_{n+1}=a_{n}+1, b_{n+1}=b_{n}-1$ and $s_{n+1}=s_{n}$. So, $a_{n}$ increases by 1 and $s_{n}$ does not change until the first index is reached with $a_{n} \nmid s_{n}$. Define + +$$ +W_{n}=\left\{m \in \mathbb{Z}_{>0}: m \geqslant a_{n} \text { and } m \nmid s_{n}\right\} \quad \text { and } \quad w_{n}=\min W_{n} +$$ + +Claim 1. The sequence $\left(w_{n}\right)$ is non-increasing. +Proof. If $a_{n} \mid b_{n}$ then $a_{n+1}=a_{n}+1$. Due to $a_{n} \mid s_{n}$, we have $a_{n} \notin W_{n}$. Moreover $s_{n+1}=s_{n}$; therefore, $W_{n+1}=W_{n}$ and $w_{n+1}=w_{n}$. + +Otherwise, if $a_{n} \nmid b_{n}$, then $a_{n} \nmid s_{n}$, so $a_{n} \in W_{n}$ and thus $w_{n}=a_{n}$. We show that $a_{n} \in W_{n+1}$; this implies $w_{n+1} \leqslant a_{n}=w_{n}$. By the definition of $W_{n+1}$, we need that $a_{n} \geqslant a_{n+1}$ and $a_{n} \nmid s_{n+1}$. The first relation holds because of $\operatorname{gcd}\left(a_{n}, b_{n}\right)\beta>\gamma . +$$ + +Depending on how large $a$ is, we divide the argument into two further cases. +Case 2.1. $\quad a=2$. +We first prove that $\gamma=0$. Assume for the sake of contradiction that $\gamma>0$. Then $c$ is even by (4) and, similarly, $b$ is even by (5) and (3). So the left-hand side of (2) is congruent to 2 modulo 4 , which is only possible if $b c=4$. As this contradicts (1), we have thereby shown that $\gamma=0$, i.e., that $c=2 b-1$. + +Now (3) yields $3 b-2=2^{\beta}$. Due to $b>2$ this is only possible if $\beta \geqslant 4$. If $\beta=4$, then we get $b=6$ and $c=2 \cdot 6-1=11$, which is a solution. It remains to deal with the case $\beta \geqslant 5$. Now (2) implies + +$$ +9 \cdot 2^{\alpha}=9 b(2 b-1)-18=(3 b-2)(6 b+1)-16=2^{\beta}\left(2^{\beta+1}+5\right)-16 +$$ + +and by $\beta \geqslant 5$ the right-hand side is not divisible by 32 . Thus $\alpha \leqslant 4$ and we get a contradiction to (5). + +Case 2.2. $\quad a \geqslant 3$. +Pick an integer $\vartheta \in\{-1,+1\}$ such that $c-\vartheta$ is not divisible by 4 . Now + +$$ +2^{\alpha}+\vartheta \cdot 2^{\beta}=\left(b c-a \vartheta^{2}\right)+\vartheta(c a-b)=(b+a \vartheta)(c-\vartheta) +$$ + +is divisible by $2^{\beta}$ and, consequently, $b+a \vartheta$ is divisible by $2^{\beta-1}$. On the other hand, $2^{\beta}=a c-b>$ $(a-1) c \geqslant 2 c$ implies in view of (1) that $a$ and $b$ are smaller than $2^{\beta-1}$. All this is only possible if $\vartheta=1$ and $a+b=2^{\beta-1}$. Now (3) yields + +$$ +a c-b=2(a+b) +$$ + +whence $4 b>a+3 b=a(c-1) \geqslant a b$, which in turn yields $a=3$. +So (6) simplifies to $c=b+2$ and (2) tells us that $b(b+2)-3=(b-1)(b+3)$ is a power of 2 . Consequently, the factors $b-1$ and $b+3$ are powers of 2 themselves. Since their difference is 4 , this is only possible if $b=5$ and thus $c=7$. Thereby the solution is complete. + +Solution 2. As in the beginning of the first solution, we observe that $a, b, c \geqslant 2$. Depending on the parities of $a, b$, and $c$ we distinguish three cases. + +Case 1. The numbers $a, b$, and $c$ are even. +Let $2^{A}, 2^{B}$, and $2^{C}$ be the largest powers of 2 dividing $a, b$, and $c$ respectively. We may assume without loss of generality that $1 \leqslant A \leqslant B \leqslant C$. Now $2^{B}$ is the highest power of 2 dividing $a c-b$, whence $a c-b=2^{B} \leqslant b$. Similarly, we deduce $b c-a=2^{A} \leqslant a$. Adding both estimates we get $(a+b) c \leqslant 2(a+b)$, whence $c \leqslant 2$. So $c=2$ and thus $A=B=C=1$; moreover, we must have had equality throughout, i.e., $a=2^{A}=2$ and $b=2^{B}=2$. We have thereby found the solution $(a, b, c)=(2,2,2)$. + +Case 2. The numbers $a, b$, and $c$ are odd. +If any two of these numbers are equal, say $a=b$, then $a c-b=a(c-1)$ has a nontrivial odd divisor and cannot be a power of 2 . Hence $a, b$, and $c$ are distinct. So we may assume without loss of generality that $a\beta$, and thus $2^{\beta}$ divides + +$$ +a \cdot 2^{\alpha}-b \cdot 2^{\beta}=a(b c-a)-b(a c-b)=b^{2}-a^{2}=(b+a)(b-a) +$$ + +Since $a$ is odd, it is not possible that both factors $b+a$ and $b-a$ are divisible by 4 . Consequently, one of them has to be a multiple of $2^{\beta-1}$. Hence one of the numbers $2(b+a)$ and $2(b-a)$ is divisible by $2^{\beta}$ and in either case we have + +$$ +a c-b=2^{\beta} \leqslant 2(a+b) +$$ + +This in turn yields $(a-1) b
\beta$ denote the integers satisfying + +$$ +2^{\alpha}=b c-a \quad \text { and } \quad 2^{\beta}=a c-b +$$ + +If $\beta=0$ it would follow that $a c-b=a b-c=1$ and hence that $b=c=1$, which is absurd. So $\beta$ and $\alpha$ are positive and consequently $a$ and $b$ are even. Substituting $c=a b-1$ into (9) we obtain + +$$ +\begin{aligned} +2^{\alpha} & =a b^{2}-(a+b), \\ +\text { and } \quad 2^{\beta} & =a^{2} b-(a+b) . +\end{aligned} +$$ + +The addition of both equation yields $2^{\alpha}+2^{\beta}=(a b-2)(a+b)$. Now $a b-2$ is even but not divisible by 4 , so the highest power of 2 dividing $a+b$ is $2^{\beta-1}$. For this reason, the equations (10) and (11) show that the highest powers of 2 dividing either of the numbers $a b^{2}$ and $a^{2} b$ is likewise $2^{\beta-1}$. Thus there is an integer $\tau \geqslant 1$ together with odd integers $A, B$, and $C$ such that $a=2^{\tau} A, b=2^{\tau} B, a+b=2^{3 \tau} C$, and $\beta=1+3 \tau$. + +Notice that $A+B=2^{2 \tau} C \geqslant 4 C$. Moreover, (11) entails $A^{2} B-C=2$. Thus $8=$ $4 A^{2} B-4 C \geqslant 4 A^{2} B-A-B \geqslant A^{2}(3 B-1)$. Since $A$ and $B$ are odd with $A0$, this may be weakened to $p^{2} q+p-q \leqslant p+q$. Hence $p^{2} q \leqslant 2 q$, which is only possible if $p=1$. + +Going back to (13), we get + +$$ +\left(d^{2} q-q-1\right) \mid d^{2}(q-1) +$$ + +Now $2\left(d^{2} q-q-1\right) \leqslant d^{2}(q-1)$ would entail $d^{2}(q+1) \leqslant 2(q+1)$ and thus $d=1$. But this would tell us that $a=d p=1$, which is absurd. This argument proves $2\left(d^{2} q-q-1\right)>d^{2}(q-1)$ and in the light of (14) it follows that $d^{2} q-q-1=d^{2}(q-1)$, i.e., $q=d^{2}-1$. Plugging this together with $p=1$ into (12) we infer $2^{\beta}=d^{3}\left(d^{2}-2\right)$. Hence $d$ and $d^{2}-2$ are powers of 2 . Consequently, $d=2, q=3$, $a=2, b=6$, and $c=11$, as desired. + +N6. Let $\mathbb{Z}_{>0}$ denote the set of positive integers. Consider a function $f: \mathbb{Z}_{>0} \rightarrow \mathbb{Z}_{>0}$. For any $m, n \in \mathbb{Z}_{>0}$ we write $f^{n}(m)=\underbrace{f(f(\ldots f}_{n}(m) \ldots))$. Suppose that $f$ has the following two properties: +(i) If $m, n \in \mathbb{Z}_{>0}$, then $\frac{f^{n}(m)-m}{n} \in \mathbb{Z}_{>0}$; +(ii) The set $\mathbb{Z}_{>0} \backslash\left\{f(n) \mid n \in \mathbb{Z}_{>0}\right\}$ is finite. + +Prove that the sequence $f(1)-1, f(2)-2, f(3)-3, \ldots$ is periodic. +(Singapore) +Solution. We split the solution into three steps. In the first of them, we show that the function $f$ is injective and explain how this leads to a useful visualization of $f$. Then comes the second step, in which most of the work happens: its goal is to show that for any $n \in \mathbb{Z}_{>0}$ the sequence $n, f(n), f^{2}(n), \ldots$ is an arithmetic progression. Finally, in the third step we put everything together, thus solving the problem. +$\underline{\text { Step 1. We commence by checking that } f \text { is injective. For this purpose, we consider any }}$ $m, k \in \mathbb{Z}_{>0}$ with $f(m)=f(k)$. By $(i)$, every positive integer $n$ has the property that + +$$ +\frac{k-m}{n}=\frac{f^{n}(m)-m}{n}-\frac{f^{n}(k)-k}{n} +$$ + +is a difference of two integers and thus integral as well. But for $n=|k-m|+1$ this is only possible if $k=m$. Thereby, the injectivity of $f$ is established. + +Now recall that due to condition (ii) there are finitely many positive integers $a_{1}, \ldots, a_{k}$ such that $\mathbb{Z}_{>0}$ is the disjoint union of $\left\{a_{1}, \ldots, a_{k}\right\}$ and $\left\{f(n) \mid n \in \mathbb{Z}_{>0}\right\}$. Notice that by plugging $n=1$ into condition $(i)$ we get $f(m)>m$ for all $m \in \mathbb{Z}_{>0}$. + +We contend that every positive integer $n$ may be expressed uniquely in the form $n=f^{j}\left(a_{i}\right)$ for some $j \geqslant 0$ and $i \in\{1, \ldots, k\}$. The uniqueness follows from the injectivity of $f$. The existence can be proved by induction on $n$ in the following way. If $n \in\left\{a_{1}, \ldots, a_{k}\right\}$, then we may take $j=0$; otherwise there is some $n^{\prime}0$; and $T=1$ and $A=0$ if $t=0$. For every integer $n \geqslant A$, the interval $\Delta_{n}=[n+1, n+T]$ contains exactly $T / T_{i}$ +elements of the $i^{\text {th }}$ row $(1 \leqslant i \leqslant t)$. Therefore, the number of elements from the last $(k-t)$ rows of the Table contained in $\Delta_{n}$ does not depend on $n \geqslant A$. It is not possible that none of these intervals $\Delta_{n}$ contains an element from the $k-t$ last rows, because infinitely many numbers appear in these rows. It follows that for each $n \geqslant A$ the interval $\Delta_{n}$ contains at least one member from these rows. + +This yields that for every positive integer $d$, the interval $[A+1, A+(d+1)(k-t) T]$ contains at least $(d+1)(k-t)$ elements from the last $k-t$ rows; therefore, there exists an index $x$ with $t+1 \leqslant x \leqslant k$, possibly depending on $d$, such that our interval contains at least $d+1$ elements from the $x^{\text {th }}$ row. In this situation we have + +$$ +f^{d}\left(a_{x}\right) \leqslant A+(d+1)(k-t) T . +$$ + +Finally, since there are finitely many possibilities for $x$, there exists an index $x \geqslant t+1$ such that the set + +$$ +X=\left\{d \in \mathbb{Z}_{>0} \mid f^{d}\left(a_{x}\right) \leqslant A+(d+1)(k-t) T\right\} +$$ + +is infinite. Thereby we have found the "dense row" promised above. +By assumption $(i)$, for every $d \in X$ the number + +$$ +\beta_{d}=\frac{f^{d}\left(a_{x}\right)-a_{x}}{d} +$$ + +is a positive integer not exceeding + +$$ +\frac{A+(d+1)(k-t) T}{d} \leqslant \frac{A d+2 d(k-t) T}{d}=A+2(k-t) T +$$ + +This leaves us with finitely many choices for $\beta_{d}$, which means that there exists a number $T_{x}$ such that the set + +$$ +Y=\left\{d \in X \mid \beta_{d}=T_{x}\right\} +$$ + +is infinite. Notice that we have $f^{d}\left(a_{x}\right)=a_{x}+d \cdot T_{x}$ for all $d \in Y$. +Now we are prepared to prove that the numbers in the $x^{\text {th }}$ row form an arithmetic progression, thus coming to a contradiction with our assumption. Let us fix any positive integer $j$. Since the set $Y$ is infinite, we can choose a number $y \in Y$ such that $y-j>\left|f^{j}\left(a_{x}\right)-\left(a_{x}+j T_{x}\right)\right|$. Notice that both numbers + +$$ +f^{y}\left(a_{x}\right)-f^{j}\left(a_{x}\right)=f^{y-j}\left(f^{j}\left(a_{x}\right)\right)-f^{j}\left(a_{x}\right) \quad \text { and } \quad f^{y}\left(a_{x}\right)-\left(a_{x}+j T_{x}\right)=(y-j) T_{x} +$$ + +are divisible by $y-j$. Thus, the difference between these numbers is also divisible by $y-j$. Since the absolute value of this difference is less than $y-j$, it has to vanish, so we get $f^{j}\left(a_{x}\right)=$ $a_{x}+j \cdot T_{x}$. + +Hence, it is indeed true that all rows of the Table are arithmetic progressions. +Step 3. Keeping the above notation in force, we denote the step of the $i^{\text {th }}$ row of the table by $T_{i}$. Now we claim that we have $f(n)-n=f(n+T)-(n+T)$ for all $n \in \mathbb{Z}_{>0}$, where + +$$ +T=\operatorname{lcm}\left(T_{1}, \ldots, T_{k}\right) +$$ + +To see this, let any $n \in \mathbb{Z}_{>0}$ be given and denote the index of the row in which it appears in the Table by $i$. Then we have $f^{j}(n)=n+j \cdot T_{i}$ for all $j \in \mathbb{Z}_{>0}$, and thus indeed + +$$ +f(n+T)-f(n)=f^{1+T / T_{i}}(n)-f(n)=\left(n+T+T_{i}\right)-\left(n+T_{i}\right)=T +$$ + +This concludes the solution. + +Comment 1. There are some alternative ways to complete the second part once the index $x$ corresponding to a "dense row" is found. For instance, one may show that for some integer $T_{x}^{*}$ the set + +$$ +Y^{*}=\left\{j \in \mathbb{Z}_{>0} \mid f^{j+1}\left(a_{x}\right)-f^{j}\left(a_{x}\right)=T_{x}^{*}\right\} +$$ + +is infinite, and then one may conclude with a similar divisibility argument. +Comment 2. It may be checked that, conversely, any way to fill out the Table with finitely many arithmetic progressions so that each positive integer appears exactly once, gives rise to a function $f$ satisfying the two conditions mentioned in the problem. For example, we may arrange the positive integers as follows: + +| 2 | 4 | 6 | 8 | 10 | $\ldots$ | +| :---: | :---: | :---: | :---: | :---: | :---: | +| 1 | 5 | 9 | 13 | 17 | $\ldots$ | +| 3 | 7 | 11 | 15 | 19 | $\ldots$ | + +This corresponds to the function + +$$ +f(n)= \begin{cases}n+2 & \text { if } n \text { is even } \\ n+4 & \text { if } n \text { is odd }\end{cases} +$$ + +As this example shows, it is not true that the function $n \mapsto f(n)-n$ has to be constant. + +N7. Let $\mathbb{Z}_{>0}$ denote the set of positive integers. For any positive integer $k$, a function $f: \mathbb{Z}_{>0} \rightarrow \mathbb{Z}_{>0}$ is called $k$-good if $\operatorname{gcd}(f(m)+n, f(n)+m) \leqslant k$ for all $m \neq n$. Find all $k$ such that there exists a $k$-good function. +(Canada) +Answer. $k \geqslant 2$. +Solution 1. For any function $f: \mathbb{Z}_{>0} \rightarrow \mathbb{Z}_{>0}$, let $G_{f}(m, n)=\operatorname{gcd}(f(m)+n, f(n)+m)$. Note that a $k$-good function is also $(k+1)$-good for any positive integer $k$. Hence, it suffices to show that there does not exist a 1-good function and that there exists a 2-good function. + +We first show that there is no 1 -good function. Suppose that there exists a function $f$ such that $G_{f}(m, n)=1$ for all $m \neq n$. Now, if there are two distinct even numbers $m$ and $n$ such that $f(m)$ and $f(n)$ are both even, then $2 \mid G_{f}(m, n)$, a contradiction. A similar argument holds if there are two distinct odd numbers $m$ and $n$ such that $f(m)$ and $f(n)$ are both odd. Hence we can choose an even $m$ and an odd $n$ such that $f(m)$ is odd and $f(n)$ is even. This also implies that $2 \mid G_{f}(m, n)$, a contradiction. + +We now construct a 2-good function. Define $f(n)=2^{g(n)+1}-n-1$, where $g$ is defined recursively by $g(1)=1$ and $g(n+1)=\left(2^{g(n)+1}\right)!$. + +For any positive integers $m>n$, set + +$$ +A=f(m)+n=2^{g(m)+1}-m+n-1, \quad B=f(n)+m=2^{g(n)+1}-n+m-1 +$$ + +We need to show that $\operatorname{gcd}(A, B) \leqslant 2$. First, note that $A+B=2^{g(m)+1}+2^{g(n)+1}-2$ is not divisible by 4 , so that $4 \nmid \operatorname{gcd}(A, B)$. Now we suppose that there is an odd prime $p$ for which $p \mid \operatorname{gcd}(A, B)$ and derive a contradiction. + +We first claim that $2^{g(m-1)+1} \geqslant B$. This is a rather weak bound; one way to prove it is as follows. Observe that $g(k+1)>g(k)$ and hence $2^{g(k+1)+1} \geqslant 2^{g(k)+1}+1$ for every positive integer $k$. By repeatedly applying this inequality, we obtain $2^{g(m-1)+1} \geqslant 2^{g(n)+1}+(m-1)-n=B$. + +Now, since $p \mid B$, we have $p-11$, introduce the set $X_{m}$ like in Solution 2 and define $f(m)$ so as to satisfy + +$$ +\begin{array}{rll} +f(m) \equiv f(m-p) & (\bmod p) & \text { for all } p \in X_{m} \text { with } p10^{100}} \alpha_{i} . +$$ + +That is, $\mho(n)$ is the number of prime factors of $n$ greater than $10^{100}$, counted with multiplicity. +Find all strictly increasing functions $f: \mathbb{Z} \rightarrow \mathbb{Z}$ such that + +$$ +\mho(f(a)-f(b)) \leqslant \mho(a-b) \quad \text { for all integers } a \text { and } b \text { with } a>b . +$$ + +(Brazil) +Answer. $f(x)=a x+b$, where $b$ is an arbitrary integer, and $a$ is an arbitrary positive integer with $\mho(a)=0$. +Solution. A straightforward check shows that all the functions listed in the answer satisfy the problem condition. It remains to show the converse. + +Assume that $f$ is a function satisfying the problem condition. Notice that the function $g(x)=f(x)-f(0)$ also satisfies this condition. Replacing $f$ by $g$, we assume from now on that $f(0)=0$; then $f(n)>0$ for any positive integer $n$. Thus, we aim to prove that there exists a positive integer $a$ with $\mho(a)=0$ such that $f(n)=a n$ for all $n \in \mathbb{Z}$. + +We start by introducing some notation. Set $N=10^{100}$. We say that a prime $p$ is large if $p>N$, and $p$ is small otherwise; let $\mathcal{S}$ be the set of all small primes. Next, we say that a positive integer is large or small if all its prime factors are such (thus, the number 1 is the unique number which is both large and small). For a positive integer $k$, we denote the greatest large divisor of $k$ and the greatest small divisor of $k$ by $L(k)$ and $S(k)$, respectively; thus, $k=L(k) S(k)$. + +We split the proof into three steps. +Step 1. We prove that for every large $k$, we have $k|f(a)-f(b) \Longleftrightarrow k| a-b$. In other words, $L(f(a)-f(b))=L(a-b)$ for all integers $a$ and $b$ with $a>b$. + +We use induction on $k$. The base case $k=1$ is trivial. For the induction step, assume that $k_{0}$ is a large number, and that the statement holds for all large numbers $k$ with $k\ell$. But then + +$$ +\mho(f(x)-f(y)) \geqslant \mho\left(\operatorname{lcm}\left(k_{0}, \ell\right)\right)>\mho(\ell)=\mho(x-y), +$$ + +which is impossible. +Now we complete the induction step. By Claim 1, for every integer $a$ each of the sequences + +$$ +f(a), f(a+1), \ldots, f\left(a+k_{0}-1\right) \quad \text { and } \quad f(a+1), f(a+2), \ldots, f\left(a+k_{0}\right) +$$ + +forms a complete residue system modulo $k_{0}$. This yields $f(a) \equiv f\left(a+k_{0}\right)\left(\bmod k_{0}\right)$. Thus, $f(a) \equiv f(b)\left(\bmod k_{0}\right)$ whenever $a \equiv b\left(\bmod k_{0}\right)$. + +Finally, if $a \not \equiv b\left(\bmod k_{0}\right)$ then there exists an integer $b^{\prime}$ such that $b^{\prime} \equiv b\left(\bmod k_{0}\right)$ and $\left|a-b^{\prime}\right|N$. +Proof. Let $d$ be the product of all small primes, and let $\alpha$ be a positive integer such that $2^{\alpha}>f(N)$. Then, for every $p \in \mathcal{S}$ the numbers $f(0), f(1), \ldots, f(N)$ are distinct modulo $p^{\alpha}$. Set $P=d^{\alpha}$ and $c=P+f(N)$. + +Choose any integer $t>N$. Due to the choice of $\alpha$, for every $p \in \mathcal{S}$ there exists at most one nonnegative integer $i \leqslant N$ with $p^{\alpha} \mid f(t)-f(i)$. Since $|\mathcal{S}||f(x)-a x|$. Since $n-x \equiv r-(r+1)=-1(\bmod N!)$, the number $|n-x|$ is large. Therefore, by Step 1 we have $f(x) \equiv f(n)=a n \equiv a x(\bmod n-x)$, so $n-x \mid f(x)-a x$. Due to the choice of $n$, this yields $f(x)=a x$. + +To complete Step 3, notice that the set $\mathcal{T}^{\prime}$ found in Step 2 contains infinitely many elements of some residue class $R_{i}$. Applying Claim 3, we successively obtain that $f(x)=a x$ for all $x \in R_{i+1}, R_{i+2}, \ldots, R_{i+N!}=R_{i}$. This finishes the solution. + +Comment 1. As the proposer also mentions, one may also consider the version of the problem where the condition (1) is replaced by the condition that $L(f(a)-f(b))=L(a-b)$ for all integers $a$ and $b$ with $a>b$. This allows to remove of Step 1 from the solution. + +Comment 2. Step 2 is the main step of the solution. We sketch several different approaches allowing to perform this step using statements which are weaker than Claim 2. +Approach 1. Let us again denote the product of all small primes by $d$. We focus on the values $f\left(d^{i}\right)$, $i \geqslant 0$. In view of Step 1, we have $L\left(f\left(d^{i}\right)-f\left(d^{k}\right)\right)=L\left(d^{i}-d^{k}\right)=d^{i-k}-1$ for all $i>k \geqslant 0$. + +Acting similarly to the beginning of the proof of Claim 2, one may choose a number $\alpha \geqslant 0$ such that the residues of the numbers $f\left(d^{i}\right), i=0,1, \ldots, N$, are distinct modulo $p^{\alpha}$ for each $p \in \mathcal{S}$. Then, for every $i>N$, there exists an exponent $k=k(i) \leqslant N$ such that $S\left(f\left(d^{i}\right)-f\left(d^{k}\right)\right)N$ such that $k(i)$ attains the same value $k_{0}$ for all $i \in I$, and such that, moreover, $S\left(f\left(d^{i}\right)-f\left(d^{k_{0}}\right)\right)$ attains the same value $s_{0}$ for all $i \in I$. Therefore, for all such $i$ we have + +$$ +f\left(d^{i}\right)=f\left(d^{k_{0}}\right)+L\left(f\left(d^{i}\right)-f\left(d^{k_{0}}\right)\right) \cdot S\left(f\left(d^{i}\right)-f\left(d^{k_{0}}\right)\right)=f\left(d^{k_{0}}\right)+\left(d^{i-k_{0}}-1\right) s_{0}, +$$ + +which means that $f$ is linear on the infinite set $\left\{d^{i}: i \in I\right\}$ (although with rational coefficients). +Finally, one may implement the relation $f\left(d^{i}\right) \equiv f(1)\left(\bmod d^{i}-1\right)$ in order to establish that in fact $f\left(d^{i}\right) / d^{i}$ is a (small and fixed) integer for all $i \in I$. + +Approach 2. Alternatively, one may start with the following lemma. +Lemma. There exists a positive constant $c$ such that + +$$ +L\left(\prod_{i=1}^{3 N}(f(k)-f(i))\right)=\prod_{i=1}^{3 N} L(f(k)-f(i)) \geqslant c(f(k))^{2 N} +$$ + +for all $k>3 N$. +Proof. Let $k$ be an integer with $k>3 N$. Set $\Pi=\prod_{i=1}^{3 N}(f(k)-f(i))$. +Notice that for every prime $p \in \mathcal{S}$, at most one of the numbers in the set + +$$ +\mathcal{H}=\{f(k)-f(i): 1 \leqslant i \leqslant 3 N\} +$$ + +is divisible by a power of $p$ which is greater than $f(3 N)$; we say that such elements of $\mathcal{H}$ are bad. Now, for each element $h \in \mathcal{H}$ which is not bad we have $S(h) \leqslant f(3 N)^{N}$, while the bad elements do not exceed $f(k)$. Moreover, there are less than $N$ bad elements in $\mathcal{H}$. Therefore, + +$$ +S(\Pi)=\prod_{h \in \mathcal{H}} S(h) \leqslant(f(3 N))^{3 N^{2}} \cdot(f(k))^{N} . +$$ + +This easily yields the lemma statement in view of the fact that $L(\Pi) S(\Pi)=\Pi \geqslant \mu(f(k))^{3 N}$ for some absolute constant $\mu$. + +As a corollary of the lemma, one may get a weaker version of Claim 2 stating that there exists a positive constant $C$ such that $f(k) \leqslant C k^{3 / 2}$ for all $k>3 N$. Indeed, from Step 1 we have + +$$ +k^{3 N} \geqslant \prod_{i=1}^{3 N} L(k-i)=\prod_{i=1}^{3 N} L(f(k)-f(i)) \geqslant c(f(k))^{2 N} +$$ + +so $f(k) \leqslant c^{-1 /(2 N)} k^{3 / 2}$. +To complete Step 2 now, set $a=f(1)$. Due to the estimates above, we may choose a positive integer $n_{0}$ such that $|f(n)-a n|<\frac{n(n-1)}{2}$ for all $n \geqslant n_{0}$. + +Take any $n \geqslant n_{0}$ with $n \equiv 2(\bmod N!)$. Then $L(f(n)-f(0))=L(n)=n / 2$ and $L(f(n)-f(1))=$ $L(n-1)=n-1$; these relations yield $f(n) \equiv f(0)=0 \equiv a n(\bmod n / 2)$ and $f(n) \equiv f(1)=a \equiv a n$ $(\bmod n-1)$, respectively. Thus, $\left.\frac{n(n-1)}{2} \right\rvert\, f(n)-a n$, which shows that $f(n)=a n$ in view of the estimate above. + +Comment 3. In order to perform Step 3, it suffices to establish the equality $f(n)=a n$ for any infinite set of values of $n$. However, if this set has some good structure, then one may find easier ways to complete this step. + +For instance, after showing, as in Approach 2 , that $f(n)=$ an for all $n \geqslant n_{0}$ with $n \equiv 2(\bmod N!)$, one may proceed as follows. Pick an arbitrary integer $x$ and take any large prime $p$ which is greater than $|f(x)-a x|$. By the Chinese Remainder Theorem, there exists a positive integer $n>\max \left(x, n_{0}\right)$ such that $n \equiv 2(\bmod N!)$ and $n \equiv x(\bmod p)$. By Step 1 , we have $f(x) \equiv f(n)=a n \equiv a x(\bmod p)$. Due to the choice of $p$, this is possible only if $f(x)=a x$. + +## CHIANG MAI, THAILAND 4-16 JULY 2015 + diff --git a/IMO/md/en-IMO2016SL.md b/IMO/md/en-IMO2016SL.md new file mode 100644 index 0000000000000000000000000000000000000000..318e37609f72f693af2a4a947a8093982fab1325 --- /dev/null +++ b/IMO/md/en-IMO2016SL.md @@ -0,0 +1,2426 @@ +## Shortlisted Problems with Solutions $57^{\text {th }}$ International Mathematical Olympiad Hong Kong, 2016 + +## Note of Confidentiality + +## The shortlisted problems should be kept strictly confidential until IMO 2017. + +## Contributing Countries + +The Organising Committee and the Problem Selection Committee of IMO 2016 thank the following 40 countries for contributing 121 problem proposals: + +Albania, Algeria, Armenia, Australia, Austria, Belarus, Belgium, Bulgaria, Colombia, Cyprus, Czech Republic, Denmark, Estonia, France, Georgia, Greece, Iceland, India, Iran, Ireland, Israel, Japan, Latvia, Luxembourg, Malaysia, Mexico, Mongolia, Netherlands, Philippines, Russia, Serbia, Slovakia, Slovenia, South Africa, Taiwan, Tanzania, Thailand, Trinidad and Tobago, Turkey, Ukraine. + +## Problem Selection Committee + +![](https://cdn.mathpix.com/cropped/2024_11_18_e5e735288fb13d37ab0cg-04.jpg?height=590&width=1215&top_left_y=1475&top_left_x=406) + +Front row from left: Yong-Gao Chen, Andy Liu, Tat Wing Leung (Chairman). +Back row from left: Yi-Jun Yao, Yun-Hao Fu, Yi-Jie He, +Zhongtao Wu, Heung Wing Joseph Lee, Chi Hong Chow, +Ka Ho Law, Tak Wing Ching. + +## Problems + +## Algebra + +A1. Let $a, b$ and $c$ be positive real numbers such that $\min \{a b, b c, c a\} \geqslant 1$. Prove that + +$$ +\sqrt[3]{\left(a^{2}+1\right)\left(b^{2}+1\right)\left(c^{2}+1\right)} \leqslant\left(\frac{a+b+c}{3}\right)^{2}+1 +$$ + +A2. Find the smallest real constant $C$ such that for any positive real numbers $a_{1}, a_{2}, a_{3}, a_{4}$ and $a_{5}$ (not necessarily distinct), one can always choose distinct subscripts $i, j, k$ and $l$ such that + +$$ +\left|\frac{a_{i}}{a_{j}}-\frac{a_{k}}{a_{l}}\right| \leqslant C +$$ + +A3. Find all integers $n \geqslant 3$ with the following property: for all real numbers $a_{1}, a_{2}, \ldots, a_{n}$ and $b_{1}, b_{2}, \ldots, b_{n}$ satisfying $\left|a_{k}\right|+\left|b_{k}\right|=1$ for $1 \leqslant k \leqslant n$, there exist $x_{1}, x_{2}, \ldots, x_{n}$, each of which is either -1 or 1 , such that + +$$ +\left|\sum_{k=1}^{n} x_{k} a_{k}\right|+\left|\sum_{k=1}^{n} x_{k} b_{k}\right| \leqslant 1 +$$ + +A4. Denote by $\mathbb{R}^{+}$the set of all positive real numbers. Find all functions $f: \mathbb{R}^{+} \rightarrow \mathbb{R}^{+}$such that + +$$ +x f\left(x^{2}\right) f(f(y))+f(y f(x))=f(x y)\left(f\left(f\left(x^{2}\right)\right)+f\left(f\left(y^{2}\right)\right)\right) +$$ + +for all positive real numbers $x$ and $y$. + +## A5. + +(a) Prove that for every positive integer $n$, there exists a fraction $\frac{a}{b}$ where $a$ and $b$ are integers satisfying $0k$, and announces them to the deputy leader and a contestant. The leader then secretly tells the deputy leader an $n$-digit binary string, and the deputy leader writes down all $n$-digit binary strings which differ from the leader's in exactly $k$ positions. (For example, if $n=3$ and $k=1$, and if the leader chooses 101, the deputy leader would write down 001, 111 and 100.) The contestant is allowed to look at the strings written by the deputy leader and guess the leader's string. What is the minimum number of guesses (in terms of $n$ and $k$ ) needed to guarantee the correct answer? + +C2. Find all positive integers $n$ for which all positive divisors of $n$ can be put into the cells of a rectangular table under the following constraints: + +- each cell contains a distinct divisor; +- the sums of all rows are equal; and +- the sums of all columns are equal. + +C3. Let $n$ be a positive integer relatively prime to 6 . We paint the vertices of a regular $n$-gon with three colours so that there is an odd number of vertices of each colour. Show that there exists an isosceles triangle whose three vertices are of different colours. +$\mathbf{C 4}$. Find all positive integers $n$ for which we can fill in the entries of an $n \times n$ table with the following properties: + +- each entry can be one of $I, M$ and $O$; +- in each row and each column, the letters $I, M$ and $O$ occur the same number of times; and +- in any diagonal whose number of entries is a multiple of three, the letters $I, M$ and $O$ occur the same number of times. + +C5. Let $n \geqslant 3$ be a positive integer. Find the maximum number of diagonals of a regular $n$-gon one can select, so that any two of them do not intersect in the interior or they are perpendicular to each other. + +C6. There are $n \geqslant 3$ islands in a city. Initially, the ferry company offers some routes between some pairs of islands so that it is impossible to divide the islands into two groups such that no two islands in different groups are connected by a ferry route. + +After each year, the ferry company will close a ferry route between some two islands $X$ and $Y$. At the same time, in order to maintain its service, the company will open new routes according to the following rule: for any island which is connected by a ferry route to exactly one of $X$ and $Y$, a new route between this island and the other of $X$ and $Y$ is added. + +Suppose at any moment, if we partition all islands into two nonempty groups in any way, then it is known that the ferry company will close a certain route connecting two islands from the two groups after some years. Prove that after some years there will be an island which is connected to all other islands by ferry routes. + +C7. Let $n \geqslant 2$ be an integer. In the plane, there are $n$ segments given in such a way that any two segments have an intersection point in the interior, and no three segments intersect at a single point. Jeff places a snail at one of the endpoints of each of the segments and claps his hands $n-1$ times. Each time when he claps his hands, all the snails move along their own segments and stay at the next intersection points until the next clap. Since there are $n-1$ intersection points on each segment, all snails will reach the furthest intersection points from their starting points after $n-1$ claps. +(a) Prove that if $n$ is odd then Jeff can always place the snails so that no two of them ever occupy the same intersection point. +(b) Prove that if $n$ is even then there must be a moment when some two snails occupy the same intersection point no matter how Jeff places the snails. + +C8. Let $n$ be a positive integer. Determine the smallest positive integer $k$ with the following property: it is possible to mark $k$ cells on a $2 n \times 2 n$ board so that there exists a unique partition of the board into $1 \times 2$ and $2 \times 1$ dominoes, none of which contains two marked cells. + +## Geometry + +G1. In a convex pentagon $A B C D E$, let $F$ be a point on $A C$ such that $\angle F B C=90^{\circ}$. Suppose triangles $A B F, A C D$ and $A D E$ are similar isosceles triangles with + +$$ +\angle F A B=\angle F B A=\angle D A C=\angle D C A=\angle E A D=\angle E D A +$$ + +Let $M$ be the midpoint of $C F$. Point $X$ is chosen such that $A M X E$ is a parallelogram. Show that $B D, E M$ and $F X$ are concurrent. + +G2. Let $A B C$ be a triangle with circumcircle $\Gamma$ and incentre $I$. Let $M$ be the midpoint of side $B C$. Denote by $D$ the foot of perpendicular from $I$ to side $B C$. The line through $I$ perpendicular to $A I$ meets sides $A B$ and $A C$ at $F$ and $E$ respectively. Suppose the circumcircle of triangle $A E F$ intersects $\Gamma$ at a point $X$ other than $A$. Prove that lines $X D$ and $A M$ meet on $\Gamma$. + +G3. Let $B=(-1,0)$ and $C=(1,0)$ be fixed points on the coordinate plane. A nonempty, bounded subset $S$ of the plane is said to be nice if +(i) there is a point $T$ in $S$ such that for every point $Q$ in $S$, the segment $T Q$ lies entirely in $S$; and +(ii) for any triangle $P_{1} P_{2} P_{3}$, there exists a unique point $A$ in $S$ and a permutation $\sigma$ of the indices $\{1,2,3\}$ for which triangles $A B C$ and $P_{\sigma(1)} P_{\sigma(2)} P_{\sigma(3)}$ are similar. + +Prove that there exist two distinct nice subsets $S$ and $S^{\prime}$ of the set $\{(x, y): x \geqslant 0, y \geqslant 0\}$ such that if $A \in S$ and $A^{\prime} \in S^{\prime}$ are the unique choices of points in (ii), then the product $B A \cdot B A^{\prime}$ is a constant independent of the triangle $P_{1} P_{2} P_{3}$. + +G4. Let $A B C$ be a triangle with $A B=A C \neq B C$ and let $I$ be its incentre. The line $B I$ meets $A C$ at $D$, and the line through $D$ perpendicular to $A C$ meets $A I$ at $E$. Prove that the reflection of $I$ in $A C$ lies on the circumcircle of triangle $B D E$. + +G5. Let $D$ be the foot of perpendicular from $A$ to the Euler line (the line passing through the circumcentre and the orthocentre) of an acute scalene triangle $A B C$. A circle $\omega$ with centre $S$ passes through $A$ and $D$, and it intersects sides $A B$ and $A C$ at $X$ and $Y$ respectively. Let $P$ be the foot of altitude from $A$ to $B C$, and let $M$ be the midpoint of $B C$. Prove that the circumcentre of triangle $X S Y$ is equidistant from $P$ and $M$. + +G6. Let $A B C D$ be a convex quadrilateral with $\angle A B C=\angle A D C<90^{\circ}$. The internal angle bisectors of $\angle A B C$ and $\angle A D C$ meet $A C$ at $E$ and $F$ respectively, and meet each other at point $P$. Let $M$ be the midpoint of $A C$ and let $\omega$ be the circumcircle of triangle $B P D$. Segments $B M$ and $D M$ intersect $\omega$ again at $X$ and $Y$ respectively. Denote by $Q$ the intersection point of lines $X E$ and $Y F$. Prove that $P Q \perp A C$. + +G7. Let $I$ be the incentre of a non-equilateral triangle $A B C, I_{A}$ be the $A$-excentre, $I_{A}^{\prime}$ be the reflection of $I_{A}$ in $B C$, and $l_{A}$ be the reflection of line $A I_{A}^{\prime}$ in $A I$. Define points $I_{B}, I_{B}^{\prime}$ and line $l_{B}$ analogously. Let $P$ be the intersection point of $l_{A}$ and $l_{B}$. +(a) Prove that $P$ lies on line $O I$ where $O$ is the circumcentre of triangle $A B C$. +(b) Let one of the tangents from $P$ to the incircle of triangle $A B C$ meet the circumcircle at points $X$ and $Y$. Show that $\angle X I Y=120^{\circ}$. + +G8. Let $A_{1}, B_{1}$ and $C_{1}$ be points on sides $B C, C A$ and $A B$ of an acute triangle $A B C$ respectively, such that $A A_{1}, B B_{1}$ and $C C_{1}$ are the internal angle bisectors of triangle $A B C$. Let $I$ be the incentre of triangle $A B C$, and $H$ be the orthocentre of triangle $A_{1} B_{1} C_{1}$. Show that + +$$ +A H+B H+C H \geqslant A I+B I+C I +$$ + +## Number Theory + +N1. For any positive integer $k$, denote the sum of digits of $k$ in its decimal representation by $S(k)$. Find all polynomials $P(x)$ with integer coefficients such that for any positive integer $n \geqslant 2016$, the integer $P(n)$ is positive and + +$$ +S(P(n))=P(S(n)) +$$ + +N2. Let $\tau(n)$ be the number of positive divisors of $n$. Let $\tau_{1}(n)$ be the number of positive divisors of $n$ which have remainders 1 when divided by 3 . Find all possible integral values of the fraction $\frac{\tau(10 n)}{\tau_{1}(10 n)}$. + +N3. Define $P(n)=n^{2}+n+1$. For any positive integers $a$ and $b$, the set + +$$ +\{P(a), P(a+1), P(a+2), \ldots, P(a+b)\} +$$ + +is said to be fragrant if none of its elements is relatively prime to the product of the other elements. Determine the smallest size of a fragrant set. + +N4. Let $n, m, k$ and $l$ be positive integers with $n \neq 1$ such that $n^{k}+m n^{l}+1$ divides $n^{k+l}-1$. Prove that + +- $m=1$ and $l=2 k$; or +- $l \mid k$ and $m=\frac{n^{k-l}-1}{n^{l}-1}$. + +N5. Let $a$ be a positive integer which is not a square number. Denote by $A$ the set of all positive integers $k$ such that + +$$ +k=\frac{x^{2}-a}{x^{2}-y^{2}} +$$ + +for some integers $x$ and $y$ with $x>\sqrt{a}$. Denote by $B$ the set of all positive integers $k$ such that (1) is satisfied for some integers $x$ and $y$ with $0 \leqslant x<\sqrt{a}$. Prove that $A=B$. + +N6. Denote by $\mathbb{N}$ the set of all positive integers. Find all functions $f: \mathbb{N} \rightarrow \mathbb{N}$ such that for all positive integers $m$ and $n$, the integer $f(m)+f(n)-m n$ is nonzero and divides $m f(m)+n f(n)$. + +N7. Let $n$ be an odd positive integer. In the Cartesian plane, a cyclic polygon $P$ with area $S$ is chosen. All its vertices have integral coordinates, and all squares of its side lengths are divisible by $n$. Prove that $2 S$ is an integer divisible by $n$. + +N8. Find all polynomials $P(x)$ of odd degree $d$ and with integer coefficients satisfying the following property: for each positive integer $n$, there exist $n$ positive integers $x_{1}, x_{2}, \ldots, x_{n}$ such that $\frac{1}{2}<\frac{P\left(x_{i}\right)}{P\left(x_{j}\right)}<2$ and $\frac{P\left(x_{i}\right)}{P\left(x_{j}\right)}$ is the $d$-th power of a rational number for every pair of indices $i$ and $j$ with $1 \leqslant i, j \leqslant n$. + +## Solutions + +## Algebra + +A1. Let $a, b$ and $c$ be positive real numbers such that $\min \{a b, b c, c a\} \geqslant 1$. Prove that + +$$ +\sqrt[3]{\left(a^{2}+1\right)\left(b^{2}+1\right)\left(c^{2}+1\right)} \leqslant\left(\frac{a+b+c}{3}\right)^{2}+1 +$$ + +Solution 1. We first show the following. + +- Claim. For any positive real numbers $x, y$ with $x y \geqslant 1$, we have + +$$ +\left(x^{2}+1\right)\left(y^{2}+1\right) \leqslant\left(\left(\frac{x+y}{2}\right)^{2}+1\right)^{2} +$$ + +Proof. Note that $x y \geqslant 1$ implies $\left(\frac{x+y}{2}\right)^{2}-1 \geqslant x y-1 \geqslant 0$. We find that $\left(x^{2}+1\right)\left(y^{2}+1\right)=(x y-1)^{2}+(x+y)^{2} \leqslant\left(\left(\frac{x+y}{2}\right)^{2}-1\right)^{2}+(x+y)^{2}=\left(\left(\frac{x+y}{2}\right)^{2}+1\right)^{2}$. + +Without loss of generality, assume $a \geqslant b \geqslant c$. This implies $a \geqslant 1$. Let $d=\frac{a+b+c}{3}$. Note that + +$$ +a d=\frac{a(a+b+c)}{3} \geqslant \frac{1+1+1}{3}=1 . +$$ + +Then we can apply (2) to the pair $(a, d)$ and the pair $(b, c)$. We get + +$$ +\left(a^{2}+1\right)\left(d^{2}+1\right)\left(b^{2}+1\right)\left(c^{2}+1\right) \leqslant\left(\left(\frac{a+d}{2}\right)^{2}+1\right)^{2}\left(\left(\frac{b+c}{2}\right)^{2}+1\right)^{2} +$$ + +Next, from + +$$ +\frac{a+d}{2} \cdot \frac{b+c}{2} \geqslant \sqrt{a d} \cdot \sqrt{b c} \geqslant 1 +$$ + +we can apply (2) again to the pair $\left(\frac{a+d}{2}, \frac{b+c}{2}\right)$. Together with (3), we have + +$$ +\left(a^{2}+1\right)\left(d^{2}+1\right)\left(b^{2}+1\right)\left(c^{2}+1\right) \leqslant\left(\left(\frac{a+b+c+d}{4}\right)^{2}+1\right)^{4}=\left(d^{2}+1\right)^{4} +$$ + +Therefore, $\left(a^{2}+1\right)\left(b^{2}+1\right)\left(c^{2}+1\right) \leqslant\left(d^{2}+1\right)^{3}$, and (1) follows by taking cube root of both sides. + +Comment. After justifying the Claim, one may also obtain (1) by mixing variables. Indeed, the function involved is clearly continuous, and hence it suffices to check that the condition $x y \geqslant 1$ is preserved under each mixing step. This is true since whenever $a b, b c, c a \geqslant 1$, we have + +$$ +\frac{a+b}{2} \cdot \frac{a+b}{2} \geqslant a b \geqslant 1 \quad \text { and } \quad \frac{a+b}{2} \cdot c \geqslant \frac{1+1}{2}=1 . +$$ + +Solution 2. Let $f(x)=\ln \left(1+x^{2}\right)$. Then the inequality (1) to be shown is equivalent to + +$$ +\frac{f(a)+f(b)+f(c)}{3} \leqslant f\left(\frac{a+b+c}{3}\right), +$$ + +while (2) becomes + +$$ +\frac{f(x)+f(y)}{2} \leqslant f\left(\frac{x+y}{2}\right) +$$ + +for $x y \geqslant 1$. +Without loss of generality, assume $a \geqslant b \geqslant c$. From the Claim in Solution 1, we have + +$$ +\frac{f(a)+f(b)+f(c)}{3} \leqslant \frac{f(a)+2 f\left(\frac{b+c}{2}\right)}{3} . +$$ + +Note that $a \geqslant 1$ and $\frac{b+c}{2} \geqslant \sqrt{b c} \geqslant 1$. Since + +$$ +f^{\prime \prime}(x)=\frac{2\left(1-x^{2}\right)}{\left(1+x^{2}\right)^{2}} +$$ + +we know that $f$ is concave on $[1, \infty)$. Then we can apply Jensen's Theorem to get + +$$ +\frac{f(a)+2 f\left(\frac{b+c}{2}\right)}{3} \leqslant f\left(\frac{a+2 \cdot \frac{b+c}{2}}{3}\right)=f\left(\frac{a+b+c}{3}\right) . +$$ + +This completes the proof. + +A2. Find the smallest real constant $C$ such that for any positive real numbers $a_{1}, a_{2}, a_{3}, a_{4}$ and $a_{5}$ (not necessarily distinct), one can always choose distinct subscripts $i, j, k$ and $l$ such that + +$$ +\left|\frac{a_{i}}{a_{j}}-\frac{a_{k}}{a_{l}}\right| \leqslant C . +$$ + +Answer. The smallest $C$ is $\frac{1}{2}$. +Solution. We first show that $C \leqslant \frac{1}{2}$. For any positive real numbers $a_{1} \leqslant a_{2} \leqslant a_{3} \leqslant a_{4} \leqslant a_{5}$, consider the five fractions + +$$ +\frac{a_{1}}{a_{2}}, \frac{a_{3}}{a_{4}}, \frac{a_{1}}{a_{5}}, \frac{a_{2}}{a_{3}}, \frac{a_{4}}{a_{5}} . +$$ + +Each of them lies in the interval $(0,1]$. Therefore, by the Pigeonhole Principle, at least three of them must lie in $\left(0, \frac{1}{2}\right]$ or lie in $\left(\frac{1}{2}, 1\right]$ simultaneously. In particular, there must be two consecutive terms in (2) which belong to an interval of length $\frac{1}{2}$ (here, we regard $\frac{a_{1}}{a_{2}}$ and $\frac{a_{4}}{a_{5}}$ as consecutive). In other words, the difference of these two fractions is less than $\frac{1}{2}$. As the indices involved in these two fractions are distinct, we can choose them to be $i, j, k, l$ and conclude that $C \leqslant \frac{1}{2}$. + +Next, we show that $C=\frac{1}{2}$ is best possible. Consider the numbers $1,2,2,2, n$ where $n$ is a large real number. The fractions formed by two of these numbers in ascending order are $\frac{1}{n}, \frac{2}{n}, \frac{1}{2}, \frac{2}{2}, \frac{2}{1}, \frac{n}{2}, \frac{n}{1}$. Since the indices $i, j, k, l$ are distinct, $\frac{1}{n}$ and $\frac{2}{n}$ cannot be chosen simultaneously. Therefore the minimum value of the left-hand side of (1) is $\frac{1}{2}-\frac{2}{n}$. When $n$ tends to infinity, this value approaches $\frac{1}{2}$, and so $C$ cannot be less than $\frac{1}{2}$. + +These conclude that $C=\frac{1}{2}$ is the smallest possible choice. +Comment. The conclusion still holds if $a_{1}, a_{2}, \ldots, a_{5}$ are pairwise distinct, since in the construction, we may replace the 2's by real numbers sufficiently close to 2 . + +There are two possible simplifications for this problem: +(i) the answer $C=\frac{1}{2}$ is given to the contestants; or +(ii) simply ask the contestants to prove the inequality (1) for $C=\frac{1}{2}$. + +A3. Find all integers $n \geqslant 3$ with the following property: for all real numbers $a_{1}, a_{2}, \ldots, a_{n}$ and $b_{1}, b_{2}, \ldots, b_{n}$ satisfying $\left|a_{k}\right|+\left|b_{k}\right|=1$ for $1 \leqslant k \leqslant n$, there exist $x_{1}, x_{2}, \ldots, x_{n}$, each of which is either -1 or 1 , such that + +$$ +\left|\sum_{k=1}^{n} x_{k} a_{k}\right|+\left|\sum_{k=1}^{n} x_{k} b_{k}\right| \leqslant 1 +$$ + +Answer. $n$ can be any odd integer greater than or equal to 3 . +Solution 1. For any even integer $n \geqslant 4$, we consider the case + +$$ +a_{1}=a_{2}=\cdots=a_{n-1}=b_{n}=0 \quad \text { and } \quad b_{1}=b_{2}=\cdots=b_{n-1}=a_{n}=1 +$$ + +The condition $\left|a_{k}\right|+\left|b_{k}\right|=1$ is satisfied for each $1 \leqslant k \leqslant n$. No matter how we choose each $x_{k}$, both sums $\sum_{k=1}^{n} x_{k} a_{k}$ and $\sum_{k=1}^{n} x_{k} b_{k}$ are odd integers. This implies $\left|\sum_{k=1}^{n} x_{k} a_{k}\right| \geqslant 1$ and $\left|\sum_{k=1}^{n} x_{k} b_{k}\right| \geqslant 1$, which shows (1) cannot hold. + +For any odd integer $n \geqslant 3$, we may assume without loss of generality $b_{k} \geqslant 0$ for $1 \leqslant k \leqslant n$ (this can be done by flipping the pair $\left(a_{k}, b_{k}\right)$ to $\left(-a_{k},-b_{k}\right)$ and $x_{k}$ to $-x_{k}$ if necessary) and $a_{1} \geqslant a_{2} \geqslant \cdots \geqslant a_{m} \geqslant 0>a_{m+1} \geqslant \cdots \geqslant a_{n}$. We claim that the choice $x_{k}=(-1)^{k+1}$ for $1 \leqslant k \leqslant n$ will work. Define + +$$ +s=\sum_{k=1}^{m} x_{k} a_{k} \quad \text { and } \quad t=-\sum_{k=m+1}^{n} x_{k} a_{k} . +$$ + +Note that + +$$ +s=\left(a_{1}-a_{2}\right)+\left(a_{3}-a_{4}\right)+\cdots \geqslant 0 +$$ + +by the assumption $a_{1} \geqslant a_{2} \geqslant \cdots \geqslant a_{m}$ (when $m$ is odd, there is a single term $a_{m}$ at the end, which is also positive). Next, we have + +$$ +s=a_{1}-\left(a_{2}-a_{3}\right)-\left(a_{4}-a_{5}\right)-\cdots \leqslant a_{1} \leqslant 1 +$$ + +Similarly, + +$$ +t=\left(-a_{n}+a_{n-1}\right)+\left(-a_{n-2}+a_{n-3}\right)+\cdots \geqslant 0 +$$ + +and + +$$ +t=-a_{n}+\left(a_{n-1}-a_{n-2}\right)+\left(a_{n-3}-a_{n-4}\right)+\cdots \leqslant-a_{n} \leqslant 1 . +$$ + +From the condition, we have $a_{k}+b_{k}=1$ for $1 \leqslant k \leqslant m$ and $-a_{k}+b_{k}=1$ for $m+1 \leqslant k \leqslant n$. It follows that $\sum_{k=1}^{n} x_{k} a_{k}=s-t$ and $\sum_{k=1}^{n} x_{k} b_{k}=1-s-t$. Hence it remains to prove + +$$ +|s-t|+|1-s-t| \leqslant 1 +$$ + +under the constraint $0 \leqslant s, t \leqslant 1$. By symmetry, we may assume $s \geqslant t$. If $1-s-t \geqslant 0$, then we have + +$$ +|s-t|+|1-s-t|=s-t+1-s-t=1-2 t \leqslant 1 +$$ + +If $1-s-t \leqslant 0$, then we have + +$$ +|s-t|+|1-s-t|=s-t-1+s+t=2 s-1 \leqslant 1 +$$ + +Hence, the inequality is true in both cases. +These show $n$ can be any odd integer greater than or equal to 3 . + +Solution 2. The even case can be handled in the same way as Solution 1. For the odd case, we prove by induction on $n$. + +Firstly, for $n=3$, we may assume without loss of generality $a_{1} \geqslant a_{2} \geqslant a_{3} \geqslant 0$ and $b_{1}=a_{1}-1$ (if $b_{1}=1-a_{1}$, we may replace each $b_{k}$ by $-b_{k}$ ). + +- Case 1. $b_{2}=a_{2}-1$ and $b_{3}=a_{3}-1$, in which case we take $\left(x_{1}, x_{2}, x_{3}\right)=(1,-1,1)$. + +Let $c=a_{1}-a_{2}+a_{3}$ so that $0 \leqslant c \leqslant 1$. Then $\left|b_{1}-b_{2}+b_{3}\right|=\left|a_{1}-a_{2}+a_{3}-1\right|=1-c$ and hence $|c|+\left|b_{1}-b_{2}+b_{3}\right|=1$. + +- Case 2. $b_{2}=1-a_{2}$ and $b_{3}=1-a_{3}$, in which case we take $\left(x_{1}, x_{2}, x_{3}\right)=(1,-1,1)$. + +Let $c=a_{1}-a_{2}+a_{3}$ so that $0 \leqslant c \leqslant 1$. Since $a_{3} \leqslant a_{2}$ and $a_{1} \leqslant 1$, we have + +$$ +c-1 \leqslant b_{1}-b_{2}+b_{3}=a_{1}+a_{2}-a_{3}-1 \leqslant 1-c +$$ + +This gives $\left|b_{1}-b_{2}+b_{3}\right| \leqslant 1-c$ and hence $|c|+\left|b_{1}-b_{2}+b_{3}\right| \leqslant 1$. + +- Case 3. $b_{2}=a_{2}-1$ and $b_{3}=1-a_{3}$, in which case we take $\left(x_{1}, x_{2}, x_{3}\right)=(-1,1,1)$. + +Let $c=-a_{1}+a_{2}+a_{3}$. If $c \geqslant 0$, then $a_{3} \leqslant 1$ and $a_{2} \leqslant a_{1}$ imply + +$$ +c-1 \leqslant-b_{1}+b_{2}+b_{3}=-a_{1}+a_{2}-a_{3}+1 \leqslant 1-c +$$ + +If $c<0$, then $a_{1} \leqslant a_{2}+1$ and $a_{3} \geqslant 0$ imply + +$$ +-c-1 \leqslant-b_{1}+b_{2}+b_{3}=-a_{1}+a_{2}-a_{3}+1 \leqslant 1+c +$$ + +In both cases, we get $\left|-b_{1}+b_{2}+b_{3}\right| \leqslant 1-|c|$ and hence $|c|+\left|-b_{1}+b_{2}+b_{3}\right| \leqslant 1$. + +- Case 4. $b_{2}=1-a_{2}$ and $b_{3}=a_{3}-1$, in which case we take $\left(x_{1}, x_{2}, x_{3}\right)=(-1,1,1)$. + +Let $c=-a_{1}+a_{2}+a_{3}$. If $c \geqslant 0$, then $a_{2} \leqslant 1$ and $a_{3} \leqslant a_{1}$ imply + +$$ +c-1 \leqslant-b_{1}+b_{2}+b_{3}=-a_{1}-a_{2}+a_{3}+1 \leqslant 1-c +$$ + +If $c<0$, then $a_{1} \leqslant a_{3}+1$ and $a_{2} \geqslant 0$ imply + +$$ +-c-1 \leqslant-b_{1}+b_{2}+b_{3}=-a_{1}-a_{2}+a_{3}+1 \leqslant 1+c +$$ + +In both cases, we get $\left|-b_{1}+b_{2}+b_{3}\right| \leqslant 1-|c|$ and hence $|c|+\left|-b_{1}+b_{2}+b_{3}\right| \leqslant 1$. +We have found $x_{1}, x_{2}, x_{3}$ satisfying (1) in each case for $n=3$. +Now, let $n \geqslant 5$ be odd and suppose the result holds for any smaller odd cases. Again we may assume $a_{k} \geqslant 0$ for each $1 \leqslant k \leqslant n$. By the Pigeonhole Principle, there are at least three indices $k$ for which $b_{k}=a_{k}-1$ or $b_{k}=1-a_{k}$. Without loss of generality, suppose $b_{k}=a_{k}-1$ for $k=1,2,3$. Again by the Pigeonhole Principle, as $a_{1}, a_{2}, a_{3}$ lies between 0 and 1 , the difference of two of them is at most $\frac{1}{2}$. By changing indices if necessary, we may assume $0 \leqslant d=a_{1}-a_{2} \leqslant \frac{1}{2}$. + +By the inductive hypothesis, we can choose $x_{3}, x_{4}, \ldots, x_{n}$ such that $a^{\prime}=\sum_{k=3}^{n} x_{k} a_{k}$ and $b^{\prime}=\sum_{k=3}^{n} x_{k} b_{k}$ satisfy $\left|a^{\prime}\right|+\left|b^{\prime}\right| \leqslant 1$. We may further assume $a^{\prime} \geqslant 0$. + +- Case 1. $b^{\prime} \geqslant 0$, in which case we take $\left(x_{1}, x_{2}\right)=(-1,1)$. + +We have $\left|-a_{1}+a_{2}+a^{\prime}\right|+\left|-\left(a_{1}-1\right)+\left(a_{2}-1\right)+b^{\prime}\right|=\left|-d+a^{\prime}\right|+\left|-d+b^{\prime}\right| \leqslant$ $\max \left\{a^{\prime}+b^{\prime}-2 d, a^{\prime}-b^{\prime}, b^{\prime}-a^{\prime}, 2 d-a^{\prime}-b^{\prime}\right\} \leqslant 1$ since $0 \leqslant a^{\prime}, b^{\prime}, a^{\prime}+b^{\prime} \leqslant 1$ and $0 \leqslant d \leqslant \frac{1}{2}$. + +- Case 2. $0>b^{\prime} \geqslant-a^{\prime}$, in which case we take $\left(x_{1}, x_{2}\right)=(-1,1)$. + +We have $\left|-a_{1}+a_{2}+a^{\prime}\right|+\left|-\left(a_{1}-1\right)+\left(a_{2}-1\right)+b^{\prime}\right|=\left|-d+a^{\prime}\right|+\left|-d+b^{\prime}\right|$. If $-d+a^{\prime} \geqslant 0$, this equals $a^{\prime}-b^{\prime}=\left|a^{\prime}\right|+\left|b^{\prime}\right| \leqslant 1$. If $-d+a^{\prime}<0$, this equals $2 d-a^{\prime}-b^{\prime} \leqslant 2 d \leqslant 1$. + +- Case 3. $b^{\prime}<-a^{\prime}$, in which case we take $\left(x_{1}, x_{2}\right)=(1,-1)$. + +We have $\left|a_{1}-a_{2}+a^{\prime}\right|+\left|\left(a_{1}-1\right)-\left(a_{2}-1\right)+b^{\prime}\right|=\left|d+a^{\prime}\right|+\left|d+b^{\prime}\right|$. If $d+b^{\prime} \geqslant 0$, this equals $2 d+a^{\prime}+b^{\prime}<2 d \leqslant 1$. If $d+b^{\prime}<0$, this equals $a^{\prime}-b^{\prime}=\left|a^{\prime}\right|+\left|b^{\prime}\right| \leqslant 1$. + +Therefore, we have found $x_{1}, x_{2}, \ldots, x_{n}$ satisfying (1) in each case. By induction, the property holds for all odd integers $n \geqslant 3$. + +A4. Denote by $\mathbb{R}^{+}$the set of all positive real numbers. Find all functions $f: \mathbb{R}^{+} \rightarrow \mathbb{R}^{+}$such that + +$$ +x f\left(x^{2}\right) f(f(y))+f(y f(x))=f(x y)\left(f\left(f\left(x^{2}\right)\right)+f\left(f\left(y^{2}\right)\right)\right) +$$ + +for all positive real numbers $x$ and $y$. +Answer. $f(x)=\frac{1}{x}$ for any $x \in \mathbb{R}^{+}$. +![](https://cdn.mathpix.com/cropped/2024_11_18_e5e735288fb13d37ab0cg-20.jpg?height=63&width=1663&top_left_y=565&top_left_x=177) hence $f(1)=1$. Swapping $x$ and $y$ in (1) and comparing with (1) again, we find + +$$ +x f\left(x^{2}\right) f(f(y))+f(y f(x))=y f\left(y^{2}\right) f(f(x))+f(x f(y)) . +$$ + +Taking $y=1$ in (2), we have $x f\left(x^{2}\right)+f(f(x))=f(f(x))+f(x)$, that is, + +$$ +f\left(x^{2}\right)=\frac{f(x)}{x} +$$ + +Take $y=1$ in (1) and apply (3) to $x f\left(x^{2}\right)$. We get $f(x)+f(f(x))=f(x)\left(f\left(f\left(x^{2}\right)\right)+1\right)$, which implies + +$$ +f\left(f\left(x^{2}\right)\right)=\frac{f(f(x))}{f(x)} +$$ + +For any $x \in \mathbb{R}^{+}$, we find that + +$$ +f\left(f(x)^{2}\right) \stackrel{(3)}{=} \frac{f(f(x))}{f(x)} \stackrel{(4)}{=} f\left(f\left(x^{2}\right)\right) \stackrel{(3)}{=} f\left(\frac{f(x)}{x}\right) +$$ + +It remains to show the following key step. + +- Claim. The function $f$ is injective. + +Proof. Using (3) and (4), we rewrite (1) as + +$$ +f(x) f(f(y))+f(y f(x))=f(x y)\left(\frac{f(f(x))}{f(x)}+\frac{f(f(y))}{f(y)}\right) . +$$ + +Take $x=y$ in (6) and apply (3). This gives $f(x) f(f(x))+f(x f(x))=2 \frac{f(f(x))}{x}$, which means + +$$ +f(x f(x))=f(f(x))\left(\frac{2}{x}-f(x)\right) +$$ + +Using (3), equation (2) can be rewritten as + +$$ +f(x) f(f(y))+f(y f(x))=f(y) f(f(x))+f(x f(y)) +$$ + +Suppose $f(x)=f(y)$ for some $x, y \in \mathbb{R}^{+}$. Then (8) implies + +$$ +f(y f(y))=f(y f(x))=f(x f(y))=f(x f(x)) +$$ + +Using (7), this gives + +$$ +f(f(y))\left(\frac{2}{y}-f(y)\right)=f(f(x))\left(\frac{2}{x}-f(x)\right) +$$ + +Noting $f(x)=f(y)$, we find $x=y$. This establishes the injectivity. + +By the Claim and (5), we get the only possible solution $f(x)=\frac{1}{x}$. It suffices to check that this is a solution. Indeed, the left-hand side of (1) becomes + +$$ +x \cdot \frac{1}{x^{2}} \cdot y+\frac{x}{y}=\frac{y}{x}+\frac{x}{y}, +$$ + +while the right-hand side becomes + +$$ +\frac{1}{x y}\left(x^{2}+y^{2}\right)=\frac{x}{y}+\frac{y}{x} . +$$ + +The two sides agree with each other. +Solution 2. Taking $x=y=1$ in (1), we get $f(1) f(f(1))+f(f(1))=2 f(1) f(f(1))$ and hence $f(1)=1$. Putting $x=1$ in (1), we have $f(f(y))+f(y)=f(y)\left(1+f\left(f\left(y^{2}\right)\right)\right)$ so that + +$$ +f(f(y))=f(y) f\left(f\left(y^{2}\right)\right) +$$ + +Putting $y=1$ in (1), we get $x f\left(x^{2}\right)+f(f(x))=f(x)\left(f\left(f\left(x^{2}\right)\right)+1\right)$. Using (9), this gives + +$$ +x f\left(x^{2}\right)=f(x) +$$ + +Replace $y$ by $\frac{1}{x}$ in (1). Then we have + +$$ +x f\left(x^{2}\right) f\left(f\left(\frac{1}{x}\right)\right)+f\left(\frac{f(x)}{x}\right)=f\left(f\left(x^{2}\right)\right)+f\left(f\left(\frac{1}{x^{2}}\right)\right) . +$$ + +The relation (10) shows $f\left(\frac{f(x)}{x}\right)=f\left(f\left(x^{2}\right)\right)$. Also, using (9) with $y=\frac{1}{x}$ and using (10) again, the last equation reduces to + +$$ +f(x) f\left(\frac{1}{x}\right)=1 +$$ + +Replace $x$ by $\frac{1}{x}$ and $y$ by $\frac{1}{y}$ in (1) and apply (11). We get + +$$ +\frac{1}{x f\left(x^{2}\right) f(f(y))}+\frac{1}{f(y f(x))}=\frac{1}{f(x y)}\left(\frac{1}{f\left(f\left(x^{2}\right)\right)}+\frac{1}{f\left(f\left(y^{2}\right)\right)}\right) . +$$ + +Clearing denominators, we can use (1) to simplify the numerators and obtain + +$$ +f(x y)^{2} f\left(f\left(x^{2}\right)\right) f\left(f\left(y^{2}\right)\right)=x f\left(x^{2}\right) f(f(y)) f(y f(x)) +$$ + +Using (9) and (10), this is the same as + +$$ +f(x y)^{2} f(f(x))=f(x)^{2} f(y) f(y f(x)) +$$ + +Substitute $y=f(x)$ in (12) and apply (10) (with $x$ replaced by $f(x)$ ). We have + +$$ +f(x f(x))^{2}=f(x) f(f(x)) +$$ + +Taking $y=x$ in (12), squaring both sides, and using (10) and (13), we find that + +$$ +f(f(x))=x^{4} f(x)^{3} +$$ + +Finally, we combine (9), (10) and (14) to get + +$$ +y^{4} f(y)^{3} \stackrel{(14)}{=} f(f(y)) \stackrel{(9)}{=} f(y) f\left(f\left(y^{2}\right)\right) \stackrel{(14)}{=} f(y) y^{8} f\left(y^{2}\right)^{3} \stackrel{(10)}{=} y^{5} f(y)^{4}, +$$ + +which implies $f(y)=\frac{1}{y}$. This is a solution by the checking in Solution 1. + +## A5. + +(a) Prove that for every positive integer $n$, there exists a fraction $\frac{a}{b}$ where $a$ and $b$ are integers satisfying $02016$ or $4 k2$ for $0 \leqslant j \leqslant 503$ in this case. So each term in the product lies strictly between 0 and 1 , and the whole product must be less than 1 , which is impossible. + +- Case 4. $4 k+2\frac{n+1}{2}$, we have $|l(x)|>|r(x)|$. + +If $n \equiv 3(\bmod 4)$, one may leave $l(x)=(x-1)(x-2) \cdots\left(x-\frac{n+1}{2}\right)$ on the left-hand side and $r(x)=\left(x-\frac{n+3}{2}\right)\left(x-\frac{x+5}{2}\right) \cdots(x-n)$ on the right-hand side. For $x<1$ or $\frac{n+1}{2}0>r(x)$. For $1\frac{n+3}{2}$, we have $|l(x)|>|r(x)|$. + +If $n \equiv 1(\bmod 4)$, as the proposer mentioned, the situation is a bit more out of control. Since the construction for $n-1 \equiv 0(\bmod 4)$ works, the answer can be either $n$ or $n-1$. For $n=5$, we can leave the products $(x-1)(x-2)(x-3)(x-4)$ and $(x-5)$. For $n=9$, the only example that works is $l(x)=(x-1)(x-2)(x-9)$ and $r(x)=(x-3)(x-4) \cdots(x-8)$, while there seems to be no such partition for $n=13$. + +A7. Denote by $\mathbb{R}$ the set of all real numbers. Find all functions $f: \mathbb{R} \rightarrow \mathbb{R}$ such that $f(0) \neq 0$ and + +$$ +f(x+y)^{2}=2 f(x) f(y)+\max \left\{f\left(x^{2}\right)+f\left(y^{2}\right), f\left(x^{2}+y^{2}\right)\right\} +$$ + +for all real numbers $x$ and $y$. + +## Answer. + +- $f(x)=-1$ for any $x \in \mathbb{R}$; or +- $f(x)=x-1$ for any $x \in \mathbb{R}$. + +Solution 1. Taking $x=y=0$ in (1), we get $f(0)^{2}=2 f(0)^{2}+\max \{2 f(0), f(0)\}$. If $f(0)>0$, then $f(0)^{2}+2 f(0)=0$ gives no positive solution. If $f(0)<0$, then $f(0)^{2}+f(0)=0$ gives $f(0)=-1$. Putting $y=0$ in (1), we have $f(x)^{2}=-2 f(x)+f\left(x^{2}\right)$, which is the same as $(f(x)+1)^{2}=f\left(x^{2}\right)+1$. Let $g(x)=f(x)+1$. Then for any $x \in \mathbb{R}$, we have + +$$ +g\left(x^{2}\right)=g(x)^{2} \geqslant 0 +$$ + +From (1), we find that $f(x+y)^{2} \geqslant 2 f(x) f(y)+f\left(x^{2}\right)+f\left(y^{2}\right)$. In terms of $g$, this becomes $(g(x+y)-1)^{2} \geqslant 2(g(x)-1)(g(y)-1)+g\left(x^{2}\right)+g\left(y^{2}\right)-2$. Using (2), this means + +$$ +(g(x+y)-1)^{2} \geqslant(g(x)+g(y)-1)^{2}-1 +$$ + +Putting $x=1$ in (2), we get $g(1)=0$ or 1 . The two cases are handled separately. + +- Case 1. $g(1)=0$, which is the same as $f(1)=-1$. + +We put $x=-1$ and $y=0$ in (1). This gives $f(-1)^{2}=-2 f(-1)-1$, which forces $f(-1)=-1$. Next, we take $x=-1$ and $y=1$ in (1) to get $1=2+\max \{-2, f(2)\}$. This clearly implies $1=2+f(2)$ and hence $f(2)=-1$, that is, $g(2)=0$. From (2), we can prove inductively that $g\left(2^{2^{n}}\right)=g(2)^{2^{n}}=0$ for any $n \in \mathbb{N}$. Substitute $y=2^{2^{n}}-x$ in (3). We obtain + +$$ +\left(g(x)+g\left(2^{2^{n}}-x\right)-1\right)^{2} \leqslant\left(g\left(2^{2^{n}}\right)-1\right)^{2}+1=2 +$$ + +For any fixed $x \geqslant 0$, we consider $n$ to be sufficiently large so that $2^{2^{n}}-x>0$. From (2), this implies $g\left(2^{2^{n}}-x\right) \geqslant 0$ so that $g(x) \leqslant 1+\sqrt{2}$. Using (2) again, we get + +$$ +g(x)^{2^{n}}=g\left(x^{2^{n}}\right) \leqslant 1+\sqrt{2} +$$ + +for any $n \in \mathbb{N}$. Therefore, $|g(x)| \leqslant 1$ for any $x \geqslant 0$. +If there exists $a \in \mathbb{R}$ for which $g(a) \neq 0$, then for sufficiently large $n$ we must have $g\left(\left(a^{2}\right)^{\frac{1}{2^{n}}}\right)=g\left(a^{2}\right)^{\frac{1}{2^{n}}}>\frac{1}{2}$. By taking $x=-y=-\left(a^{2}\right)^{\frac{1}{2^{n}}}$ in (1), we obtain + +$$ +\begin{aligned} +1 & =2 f(x) f(-x)+\max \left\{2 f\left(x^{2}\right), f\left(2 x^{2}\right)\right\} \\ +& =2(g(x)-1)(g(-x)-1)+\max \left\{2\left(g\left(x^{2}\right)-1\right), g\left(2 x^{2}\right)-1\right\} \\ +& \leqslant 2\left(-\frac{1}{2}\right)\left(-\frac{1}{2}\right)+0=\frac{1}{2} +\end{aligned} +$$ + +since $|g(-x)|=|g(x)| \in\left(\frac{1}{2}, 1\right]$ by (2) and the choice of $x$, and since $g(z) \leqslant 1$ for $z \geqslant 0$. This yields a contradiction and hence $g(x)=0$ must hold for any $x$. This means $f(x)=-1$ for any $x \in \mathbb{R}$, which clearly satisfies (1). + +- Case 2. $g(1)=1$, which is the same as $f(1)=0$. + +We put $x=-1$ and $y=1$ in (1) to get $1=\max \{0, f(2)\}$. This clearly implies $f(2)=1$ and hence $g(2)=2$. Setting $x=2 n$ and $y=2$ in (3), we have + +$$ +(g(2 n+2)-1)^{2} \geqslant(g(2 n)+1)^{2}-1 +$$ + +By induction on $n$, it is easy to prove that $g(2 n) \geqslant n+1$ for all $n \in \mathbb{N}$. For any real number $a>1$, we choose a large $n \in \mathbb{N}$ and take $k$ to be the positive integer such that $2 k \leqslant a^{2^{n}}<2 k+2$. From (2) and (3), we have + +$$ +\left(g(a)^{2^{n}}-1\right)^{2}+1=\left(g\left(a^{2^{n}}\right)-1\right)^{2}+1 \geqslant\left(g(2 k)+g\left(a^{2^{n}}-2 k\right)-1\right)^{2} \geqslant k^{2}>\frac{1}{4}\left(a^{2^{n}}-2\right)^{2} +$$ + +since $g\left(a^{2^{n}}-2 k\right) \geqslant 0$. For large $n$, this clearly implies $g(a)^{2^{n}}>1$. Thus, + +$$ +\left(g(a)^{2^{n}}\right)^{2}>\left(g(a)^{2^{n}}-1\right)^{2}+1>\frac{1}{4}\left(a^{2^{n}}-2\right)^{2} +$$ + +This yields + +$$ +g(a)^{2^{n}}>\frac{1}{2}\left(a^{2^{n}}-2\right) +$$ + +Note that + +$$ +\frac{a^{2^{n}}}{a^{2^{n}}-2}=1+\frac{2}{a^{2^{n}}-2} \leqslant\left(1+\frac{2}{2^{n}\left(a^{2^{n}}-2\right)}\right)^{2^{n}} +$$ + +by binomial expansion. This can be rewritten as + +$$ +\left(a^{2^{n}}-2\right)^{\frac{1}{2^{n}}} \geqslant \frac{a}{1+\frac{2}{2^{n}\left(a^{2^{n}}-2\right)}} +$$ + +Together with (4), we conclude $g(a) \geqslant a$ by taking $n$ sufficiently large. +Consider $x=n a$ and $y=a>1$ in (3). This gives $(g((n+1) a)-1)^{2} \geqslant(g(n a)+g(a)-1)^{2}-1$. By induction on $n$, it is easy to show $g(n a) \geqslant(n-1)(g(a)-1)+a$ for any $n \in \mathbb{N}$. We choose a large $n \in \mathbb{N}$ and take $k$ to be the positive integer such that $k a \leqslant 2^{2^{n}}<(k+1) a$. Using (2) and (3), we have +$2^{2^{n+1}}>\left(2^{2^{n}}-1\right)^{2}+1=\left(g\left(2^{2^{n}}\right)-1\right)^{2}+1 \geqslant\left(g\left(2^{2^{n}}-k a\right)+g(k a)-1\right)^{2} \geqslant((k-1)(g(a)-1)+a-1)^{2}$, from which it follows that + +$$ +2^{2^{n}} \geqslant(k-1)(g(a)-1)+a-1>\frac{2^{2^{n}}}{a}(g(a)-1)-2(g(a)-1)+a-1 +$$ + +holds for sufficiently large $n$. Hence, we must have $\frac{g(a)-1}{a} \leqslant 1$, which implies $g(a) \leqslant a+1$ for any $a>1$. Then for large $n \in \mathbb{N}$, from (3) and (2) we have + +$$ +4 a^{2^{n+1}}=\left(2 a^{2^{n}}\right)^{2} \geqslant\left(g\left(2 a^{2^{n}}\right)-1\right)^{2} \geqslant\left(2 g\left(a^{2^{n}}\right)-1\right)^{2}-1=\left(2 g(a)^{2^{n}}-1\right)^{2}-1 +$$ + +This implies + +$$ +2 a^{2^{n}}>\frac{1}{2}\left(1+\sqrt{4 a^{2^{n+1}}+1}\right) \geqslant g(a)^{2^{n}} +$$ + +When $n$ tends to infinity, this forces $g(a) \leqslant a$. Together with $g(a) \geqslant a$, we get $g(a)=a$ for all real numbers $a>1$, that is, $f(a)=a-1$ for all $a>1$. + +Finally, for any $x \in \mathbb{R}$, we choose $y$ sufficiently large in (1) so that $y, x+y>1$. This gives $(x+y-1)^{2}=2 f(x)(y-1)+\max \left\{f\left(x^{2}\right)+y^{2}-1, x^{2}+y^{2}-1\right\}$, which can be rewritten as + +$$ +2(x-1-f(x)) y=-x^{2}+2 x-2-2 f(x)+\max \left\{f\left(x^{2}\right), x^{2}\right\} +$$ + +As the right-hand side is fixed, this can only hold for all large $y$ when $f(x)=x-1$. We now check that this function satisfies (1). Indeed, we have + +$$ +\begin{aligned} +f(x+y)^{2} & =(x+y-1)^{2}=2(x-1)(y-1)+\left(x^{2}+y^{2}-1\right) \\ +& =2 f(x) f(y)+\max \left\{f\left(x^{2}\right)+f\left(y^{2}\right), f\left(x^{2}+y^{2}\right)\right\} . +\end{aligned} +$$ + +Solution 2. Taking $x=y=0$ in (1), we get $f(0)^{2}=2 f(0)^{2}+\max \{2 f(0), f(0)\}$. If $f(0)>0$, then $f(0)^{2}+2 f(0)=0$ gives no positive solution. If $f(0)<0$, then $f(0)^{2}+f(0)=0$ gives $f(0)=-1$. Putting $y=0$ in (1), we have + +$$ +f(x)^{2}=-2 f(x)+f\left(x^{2}\right) +$$ + +Replace $x$ by $-x$ in (5) and compare with (5) again. We get $f(x)^{2}+2 f(x)=f(-x)^{2}+2 f(-x)$, which implies + +$$ +f(x)=f(-x) \quad \text { or } \quad f(x)+f(-x)=-2 +$$ + +Taking $x=y$ and $x=-y$ respectively in (1) and comparing the two equations obtained, we have + +$$ +f(2 x)^{2}-2 f(x)^{2}=1-2 f(x) f(-x) . +$$ + +Combining (6) and (7) to eliminate $f(-x)$, we find that $f(2 x)$ can be $\pm 1$ (when $f(x)=f(-x)$ ) or $\pm(2 f(x)+1)$ (when $f(x)+f(-x)=-2$ ). + +We prove the following. + +- Claim. $f(x)+f(-x)=-2$ for any $x \in \mathbb{R}$. + +Proof. Suppose there exists $a \in \mathbb{R}$ such that $f(a)+f(-a) \neq-2$. Then $f(a)=f(-a) \neq-1$ and we may assume $a>0$. We first show that $f(a) \neq 1$. Suppose $f(a)=1$. Consider $y=a$ in (7). We get $f(2 a)^{2}=1$. Taking $x=y=a$ in (1), we have $1=2+\max \left\{2 f\left(a^{2}\right), f\left(2 a^{2}\right)\right\}$. From (5), $f\left(a^{2}\right)=3$ so that $1 \geqslant 2+6$. This is impossible, and thus $f(a) \neq 1$. + +As $f(a) \neq \pm 1$, we have $f(a)= \pm\left(2 f\left(\frac{a}{2}\right)+1\right)$. Similarly, $f(-a)= \pm\left(2 f\left(-\frac{a}{2}\right)+1\right)$. These two expressions are equal since $f(a)=f(-a)$. If $f\left(\frac{a}{2}\right)=f\left(-\frac{a}{2}\right)$, then the above argument works when we replace $a$ by $\frac{a}{2}$. In particular, we have $f(a)^{2}=f\left(2 \cdot \frac{a}{2}\right)^{2}=1$, which is a contradiction. Therefore, (6) forces $f\left(\frac{a}{2}\right)+f\left(-\frac{a}{2}\right)=-2$. Then we get + +$$ +\pm\left(2 f\left(\frac{a}{2}\right)+1\right)= \pm\left(-2 f\left(\frac{a}{2}\right)-3\right) . +$$ + +For any choices of the two signs, we either get a contradiction or $f\left(\frac{a}{2}\right)=-1$, in which case $f\left(\frac{a}{2}\right)=f\left(-\frac{a}{2}\right)$ and hence $f(a)= \pm 1$ again. Therefore, there is no such real number $a$ and the Claim follows. + +Replace $x$ and $y$ by $-x$ and $-y$ in (1) respectively and compare with (1). We get + +$$ +f(x+y)^{2}-2 f(x) f(y)=f(-x-y)^{2}-2 f(-x) f(-y) +$$ + +Using the Claim, this simplifies to $f(x+y)=f(x)+f(y)+1$. In addition, (5) can be rewritten as $(f(x)+1)^{2}=f\left(x^{2}\right)+1$. Therefore, the function $g$ defined by $g(x)=f(x)+1$ satisfies $g(x+y)=g(x)+g(y)$ and $g(x)^{2}=g\left(x^{2}\right)$. The latter relation shows $g(y)$ is nonnegative for $y \geqslant 0$. For such a function satisfying the Cauchy Equation $g(x+y)=g(x)+g(y)$, it must be monotonic increasing and hence $g(x)=c x$ for some constant $c$. + +From $(c x)^{2}=g(x)^{2}=g\left(x^{2}\right)=c x^{2}$, we get $c=0$ or 1 , which corresponds to the two functions $f(x)=-1$ and $f(x)=x-1$ respectively, both of which are solutions to (1) as checked in Solution 1. + +Solution 3. As in Solution 2, we find that $f(0)=-1$, + +$$ +(f(x)+1)^{2}=f\left(x^{2}\right)+1 +$$ + +and + +$$ +f(x)=f(-x) \quad \text { or } \quad f(x)+f(-x)=-2 +$$ + +for any $x \in \mathbb{R}$. We shall show that one of the statements in (9) holds for all $x \in \mathbb{R}$. Suppose $f(a)=f(-a)$ but $f(a)+f(-a) \neq-2$, while $f(b) \neq f(-b)$ but $f(b)+f(-b)=-2$. Clearly, $a, b \neq 0$ and $f(a), f(b) \neq-1$. + +Taking $y=a$ and $y=-a$ in (1) respectively and comparing the two equations obtained, we have $f(x+a)^{2}=f(x-a)^{2}$, that is, $f(x+a)= \pm f(x-a)$. This implies $f(x+2 a)= \pm f(x)$ for all $x \in \mathbb{R}$. Putting $x=b$ and $x=-2 a-b$ respectively, we find $f(2 a+b)= \pm f(b)$ and $f(-2 a-b)= \pm f(-b)= \pm(-2-f(b))$. Since $f(b) \neq-1$, the term $\pm(-2-f(b))$ is distinct from $\pm f(b)$ in any case. So $f(2 a+b) \neq f(-2 a-b)$. From (9), we must have $f(2 a+b)+f(-2 a-b)=-2$. Note that we also have $f(b)+f(-b)=-2$ where $|f(b)|,|f(-b)|$ are equal to $|f(2 a+b)|,|f(-2 a-b)|$ respectively. The only possible case is $f(2 a+b)=f(b)$ and $f(-2 a-b)=f(-b)$. + +Applying the argument to $-a$ instead of $a$ and using induction, we have $f(2 k a+b)=f(b)$ and $f(2 k a-b)=f(-b)$ for any integer $k$. Note that $f(b)+f(-b)=-2$ and $f(b) \neq-1$ imply one of $f(b), f(-b)$ is less than -1 . Without loss of generality, assume $f(b)<-1$. We consider $x=\sqrt{2 k a+b}$ in (8) for sufficiently large $k$ so that + +$$ +(f(x)+1)^{2}=f(2 k a+b)+1=f(b)+1<0 +$$ + +yields a contradiction. Therefore, one of the statements in (9) must hold for all $x \in \mathbb{R}$. + +- Case 1. $f(x)=f(-x)$ for any $x \in \mathbb{R}$. + +For any $a \in \mathbb{R}$, setting $x=y=\frac{a}{2}$ and $x=-y=\frac{a}{2}$ in (1) respectively and comparing these, we obtain $f(a)^{2}=f(0)^{2}=1$, which means $f(a)= \pm 1$ for all $a \in \mathbb{R}$. If $f(a)=1$ for some $a$, we may assume $a>0$ since $f(a)=f(-a)$. Taking $x=y=\sqrt{a}$ in (1), we get + +$$ +f(2 \sqrt{a})^{2}=2 f(\sqrt{a})^{2}+\max \{2, f(2 a)\}=2 f(\sqrt{a})^{2}+2 +$$ + +Note that the left-hand side is $\pm 1$ while the right-hand side is an even integer. This is a contradiction. Therefore, $f(x)=-1$ for all $x \in \mathbb{R}$, which is clearly a solution. + +- Case 2. $f(x)+f(-x)=-2$ for any $x \in \mathbb{R}$. + +This case can be handled in the same way as in Solution 2, which yields another solution $f(x)=x-1$. + +A8. Determine the largest real number $a$ such that for all $n \geqslant 1$ and for all real numbers $x_{0}, x_{1}, \ldots, x_{n}$ satisfying $0=x_{0}4 \sum_{k=1}^{n} \frac{k+1}{x_{k}} . +$$ + +This shows (1) holds for $a=\frac{4}{9}$. +Next, we show that $a=\frac{4}{9}$ is the optimal choice. Consider the sequence defined by $x_{0}=0$ and $x_{k}=x_{k-1}+k(k+1)$ for $k \geqslant 1$, that is, $x_{k}=\frac{1}{3} k(k+1)(k+2)$. Then the left-hand side of (1) equals + +$$ +\sum_{k=1}^{n} \frac{1}{k(k+1)}=\sum_{k=1}^{n}\left(\frac{1}{k}-\frac{1}{k+1}\right)=1-\frac{1}{n+1} +$$ + +while the right-hand side equals + +$$ +a \sum_{k=1}^{n} \frac{k+1}{x_{k}}=3 a \sum_{k=1}^{n} \frac{1}{k(k+2)}=\frac{3}{2} a \sum_{k=1}^{n}\left(\frac{1}{k}-\frac{1}{k+2}\right)=\frac{3}{2}\left(1+\frac{1}{2}-\frac{1}{n+1}-\frac{1}{n+2}\right) a . +$$ + +When $n$ tends to infinity, the left-hand side tends to 1 while the right-hand side tends to $\frac{9}{4} a$. Therefore $a$ has to be at most $\frac{4}{9}$. + +Hence the largest value of $a$ is $\frac{4}{9}$. +Solution 2. We shall give an alternative method to establish (1) with $a=\frac{4}{9}$. We define $y_{k}=x_{k}-x_{k-1}>0$ for $1 \leqslant k \leqslant n$. By the Cauchy-Schwarz Inequality, for $1 \leqslant k \leqslant n$, we have + +$$ +\left(y_{1}+y_{2}+\cdots+y_{k}\right)\left(\sum_{j=1}^{k} \frac{1}{y_{j}}\binom{j+1}{2}^{2}\right) \geqslant\left(\binom{2}{2}+\binom{3}{2}+\cdots+\binom{k+1}{2}\right)^{2}=\binom{k+2}{3}^{2} +$$ + +This can be rewritten as + +$$ +\frac{k+1}{y_{1}+y_{2}+\cdots+y_{k}} \leqslant \frac{36}{k^{2}(k+1)(k+2)^{2}}\left(\sum_{j=1}^{k} \frac{1}{y_{j}}\binom{j+1}{2}^{2}\right) . +$$ + +Summing (3) over $k=1,2, \ldots, n$, we get + +$$ +\frac{2}{y_{1}}+\frac{3}{y_{1}+y_{2}}+\cdots+\frac{n+1}{y_{1}+y_{2}+\cdots+y_{n}} \leqslant \frac{c_{1}}{y_{1}}+\frac{c_{2}}{y_{2}}+\cdots+\frac{c_{n}}{y_{n}} +$$ + +where for $1 \leqslant m \leqslant n$, + +$$ +\begin{aligned} +c_{m} & =36\binom{m+1}{2}^{2} \sum_{k=m}^{n} \frac{1}{k^{2}(k+1)(k+2)^{2}} \\ +& =\frac{9 m^{2}(m+1)^{2}}{4} \sum_{k=m}^{n}\left(\frac{1}{k^{2}(k+1)^{2}}-\frac{1}{(k+1)^{2}(k+2)^{2}}\right) \\ +& =\frac{9 m^{2}(m+1)^{2}}{4}\left(\frac{1}{m^{2}(m+1)^{2}}-\frac{1}{(n+1)^{2}(n+2)^{2}}\right)<\frac{9}{4} . +\end{aligned} +$$ + +From (4), the inequality (1) holds for $a=\frac{4}{9}$. This is also the upper bound as can be verified in the same way as Solution 1. + +## Combinatorics + +C1. The leader of an IMO team chooses positive integers $n$ and $k$ with $n>k$, and announces them to the deputy leader and a contestant. The leader then secretly tells the deputy leader an $n$-digit binary string, and the deputy leader writes down all $n$-digit binary strings which differ from the leader's in exactly $k$ positions. (For example, if $n=3$ and $k=1$, and if the leader chooses 101, the deputy leader would write down 001, 111 and 100.) The contestant is allowed to look at the strings written by the deputy leader and guess the leader's string. What is the minimum number of guesses (in terms of $n$ and $k$ ) needed to guarantee the correct answer? + +Answer. The minimum number of guesses is 2 if $n=2 k$ and 1 if $n \neq 2 k$. +Solution 1. Let $X$ be the binary string chosen by the leader and let $X^{\prime}$ be the binary string of length $n$ every digit of which is different from that of $X$. The strings written by the deputy leader are the same as those in the case when the leader's string is $X^{\prime}$ and $k$ is changed to $n-k$. In view of this, we may assume $k \geqslant \frac{n}{2}$. Also, for the particular case $k=\frac{n}{2}$, this argument shows that the strings $X$ and $X^{\prime}$ cannot be distinguished, and hence in that case the contestant has to guess at least twice. + +It remains to show that the number of guesses claimed suffices. Consider any string $Y$ which differs from $X$ in $m$ digits where $02$, the deputy leader would write down the same strings if the leader's string $X$ is replaced by $X^{\prime}$ obtained by changing each digit of $X$. This shows at least 2 guesses are needed. We shall show that 2 guesses suffice in this case. Suppose the first two digits of the leader's string are the same. Then among the strings written by the deputy leader, the prefices 01 and 10 will occur $\binom{2 k-2}{k-1}$ times each, while the prefices 00 and 11 will occur $\binom{2 k-2}{k}$ times each. The two numbers are interchanged if the first two digits of the leader's string are different. Since $\binom{2 k-2}{k-1} \neq\binom{ 2 k-2}{k}$, the contestant can tell whether the first two digits of the leader's string are the same or not. He can work out the relation of the first digit and the +other digits in the same way and reduce the leader's string to only 2 possibilities. The proof is complete. + +C2. Find all positive integers $n$ for which all positive divisors of $n$ can be put into the cells of a rectangular table under the following constraints: + +- each cell contains a distinct divisor; +- the sums of all rows are equal; and +- the sums of all columns are equal. + +Answer. 1. +Solution 1. Suppose all positive divisors of $n$ can be arranged into a rectangular table of size $k \times l$ where the number of rows $k$ does not exceed the number of columns $l$. Let the sum of numbers in each column be $s$. Since $n$ belongs to one of the columns, we have $s \geqslant n$, where equality holds only when $n=1$. + +For $j=1,2, \ldots, l$, let $d_{j}$ be the largest number in the $j$-th column. Without loss of generality, assume $d_{1}>d_{2}>\cdots>d_{l}$. Since these are divisors of $n$, we have + +$$ +d_{l} \leqslant \frac{n}{l} +$$ + +As $d_{l}$ is the maximum entry of the $l$-th column, we must have + +$$ +d_{l} \geqslant \frac{s}{k} \geqslant \frac{n}{k} . +$$ + +The relations (1) and (2) combine to give $\frac{n}{l} \geqslant \frac{n}{k}$, that is, $k \geqslant l$. Together with $k \leqslant l$, we conclude that $k=l$. Then all inequalities in (1) and (2) are equalities. In particular, $s=n$ and so $n=1$, in which case the conditions are clearly satisfied. + +Solution 2. Clearly $n=1$ works. Then we assume $n>1$ and let its prime factorization be $n=p_{1}^{r_{1}} p_{2}^{r_{2}} \cdots p_{t}^{r_{t}}$. Suppose the table has $k$ rows and $l$ columns with $1n$. Therefore, we have + +$$ +\begin{aligned} +\left(r_{1}+1\right)\left(r_{2}+1\right) \cdots\left(r_{t}+1\right) & =k l \leqslant l^{2}<\left(\frac{\sigma(n)}{n}\right)^{2} \\ +& =\left(1+\frac{1}{p_{1}}+\cdots+\frac{1}{p_{1}^{r_{1}}}\right)^{2} \cdots\left(1+\frac{1}{p_{t}}+\cdots+\frac{1}{p_{t}^{r_{t}}}\right)^{2} +\end{aligned} +$$ + +This can be rewritten as + +$$ +f\left(p_{1}, r_{1}\right) f\left(p_{2}, r_{2}\right) \cdots f\left(p_{t}, r_{t}\right)<1 +$$ + +where + +$$ +f(p, r)=\frac{r+1}{\left(1+\frac{1}{p}+\cdots+\frac{1}{p^{r}}\right)^{2}}=\frac{(r+1)\left(1-\frac{1}{p}\right)^{2}}{\left(1-\frac{1}{p^{r+1}}\right)^{2}} +$$ + +Direct computation yields + +$$ +f(2,1)=\frac{8}{9}, \quad f(2,2)=\frac{48}{49}, \quad f(3,1)=\frac{9}{8} . +$$ + +Also, we find that + +$$ +\begin{aligned} +& f(2, r) \geqslant\left(1-\frac{1}{2^{r+1}}\right)^{-2}>1 \quad \text { for } r \geqslant 3 \\ +& f(3, r) \geqslant \frac{4}{3}\left(1-\frac{1}{3^{r+1}}\right)^{-2}>\frac{4}{3}>\frac{9}{8} \quad \text { for } r \geqslant 2, \text { and } \\ +& f(p, r) \geqslant \frac{32}{25}\left(1-\frac{1}{p^{r+1}}\right)^{-2}>\frac{32}{25}>\frac{9}{8} \quad \text { for } p \geqslant 5 +\end{aligned} +$$ + +From these values and bounds, it is clear that (3) holds only when $n=2$ or 4 . In both cases, it is easy to see that the conditions are not satisfied. Hence, the only possible $n$ is 1 . + +C3. Let $n$ be a positive integer relatively prime to 6 . We paint the vertices of a regular $n$-gon with three colours so that there is an odd number of vertices of each colour. Show that there exists an isosceles triangle whose three vertices are of different colours. + +Solution. For $k=1,2,3$, let $a_{k}$ be the number of isosceles triangles whose vertices contain exactly $k$ colours. Suppose on the contrary that $a_{3}=0$. Let $b, c, d$ be the number of vertices of the three different colours respectively. We now count the number of pairs $(\triangle, E)$ where $\triangle$ is an isosceles triangle and $E$ is a side of $\triangle$ whose endpoints are of different colours. + +On the one hand, since we have assumed $a_{3}=0$, each triangle in the pair must contain exactly two colours, and hence each triangle contributes twice. Thus the number of pairs is $2 a_{2}$ 。 + +On the other hand, if we pick any two vertices $A, B$ of distinct colours, then there are three isosceles triangles having these as vertices, two when $A B$ is not the base and one when $A B$ is the base since $n$ is odd. Note that the three triangles are all distinct as $(n, 3)=1$. In this way, we count the number of pairs to be $3(b c+c d+d b)$. However, note that $2 a_{2}$ is even while $3(b c+c d+d b)$ is odd, as each of $b, c, d$ is. This yields a contradiction and hence $a_{3} \geqslant 1$. + +Comment. A slightly stronger version of this problem is to replace the condition $(n, 6)=1$ by $n$ being odd (where equilateral triangles are regarded as isosceles triangles). In that case, the only difference in the proof is that by fixing any two vertices $A, B$, one can find exactly one or three isosceles triangles having these as vertices. But since only parity is concerned in the solution, the proof goes the same way. + +The condition that there is an odd number of vertices of each colour is necessary, as can be seen from the following example. Consider $n=25$ and we label the vertices $A_{0}, A_{1}, \ldots, A_{24}$. Suppose colour 1 is used for $A_{0}$, colour 2 is used for $A_{5}, A_{10}, A_{15}, A_{20}$, while colour 3 is used for the remaining vertices. Then any isosceles triangle having colours 1 and 2 must contain $A_{0}$ and one of $A_{5}, A_{10}, A_{15}, A_{20}$. Clearly, the third vertex must have index which is a multiple of 5 so it is not of colour 3 . + +C4. Find all positive integers $n$ for which we can fill in the entries of an $n \times n$ table with the following properties: + +- each entry can be one of $I, M$ and $O$; +- in each row and each column, the letters $I, M$ and $O$ occur the same number of times; and +- in any diagonal whose number of entries is a multiple of three, the letters $I, M$ and $O$ occur the same number of times. + +Answer. $n$ can be any multiple of 9 . +Solution. We first show that such a table exists when $n$ is a multiple of 9 . Consider the following $9 \times 9$ table. + +$$ +\left(\begin{array}{ccccccccc} +I & I & I & M & M & M & O & O & O \\ +M & M & M & O & O & O & I & I & I \\ +O & O & O & I & I & I & M & M & M \\ +I & I & I & M & M & M & O & O & O \\ +M & M & M & O & O & O & I & I & I \\ +O & O & O & I & I & I & M & M & M \\ +I & I & I & M & M & M & O & O & O \\ +M & M & M & O & O & O & I & I & I \\ +O & O & O & I & I & I & M & M & M +\end{array}\right) +$$ + +It is a direct checking that the table (1) satisfies the requirements. For $n=9 k$ where $k$ is a positive integer, we form an $n \times n$ table using $k \times k$ copies of (1). For each row and each column of the table of size $n$, since there are three $I$ 's, three $M$ 's and three $O$ 's for any nine consecutive entries, the numbers of $I, M$ and $O$ are equal. In addition, every diagonal of the large table whose number of entries is divisible by 3 intersects each copy of (1) at a diagonal with number of entries divisible by 3 (possibly zero). Therefore, every such diagonal also contains the same number of $I, M$ and $O$. + +Next, consider any $n \times n$ table for which the requirements can be met. As the number of entries of each row should be a multiple of 3 , we let $n=3 k$ where $k$ is a positive integer. We divide the whole table into $k \times k$ copies of $3 \times 3$ blocks. We call the entry at the centre of such a $3 \times 3$ square a vital entry. We also call any row, column or diagonal that contains at least one vital entry a vital line. We compute the number of pairs $(l, c)$ where $l$ is a vital line and $c$ is an entry belonging to $l$ that contains the letter $M$. We let this number be $N$. + +On the one hand, since each vital line contains the same number of $I, M$ and $O$, it is obvious that each vital row and each vital column contain $k$ occurrences of $M$. For vital diagonals in either direction, we count there are exactly + +$$ +1+2+\cdots+(k-1)+k+(k-1)+\cdots+2+1=k^{2} +$$ + +occurrences of $M$. Therefore, we have $N=4 k^{2}$. + +On the other hand, there are $3 k^{2}$ occurrences of $M$ in the whole table. Note that each entry belongs to exactly 1 or 4 vital lines. Therefore, $N$ must be congruent to $3 k^{2} \bmod 3$. + +From the double counting, we get $4 k^{2} \equiv 3 k^{2}(\bmod 3)$, which forces $k$ to be a multiple of 3. Therefore, $n$ has to be a multiple of 9 and the proof is complete. + +C5. Let $n \geqslant 3$ be a positive integer. Find the maximum number of diagonals of a regular $n$-gon one can select, so that any two of them do not intersect in the interior or they are perpendicular to each other. + +Answer. $n-2$ if $n$ is even and $n-3$ if $n$ is odd. + +Solution 1. We consider two cases according to the parity of $n$. + +- Case 1. $n$ is odd. + +We first claim that no pair of diagonals is perpendicular. Suppose $A, B, C, D$ are vertices where $A B$ and $C D$ are perpendicular, and let $E$ be the vertex lying on the perpendicular bisector of $A B$. Let $E^{\prime}$ be the opposite point of $E$ on the circumcircle of the regular polygon. Since $E C=E^{\prime} D$ and $C, D, E$ are vertices of the regular polygon, $E^{\prime}$ should also belong to the polygon. This contradicts the fact that a regular polygon with an odd number of vertices does not contain opposite points on the circumcircle. +![](https://cdn.mathpix.com/cropped/2024_11_18_e5e735288fb13d37ab0cg-39.jpg?height=513&width=1250&top_left_y=1061&top_left_x=493) + +Therefore in the odd case we can only select diagonals which do not intersect. In the maximal case these diagonals should divide the regular $n$-gon into $n-2$ triangles, so we can select at most $n-3$ diagonals. This can be done, for example, by selecting all diagonals emanated from a particular vertex. + +- Case 2. $n$ is even. + +If there is no intersection, then the proof in the odd case works. Suppose there are two perpendicular diagonals selected. We consider the set $S$ of all selected diagonals parallel to one of them which intersect with some selected diagonals. Suppose $S$ contains $k$ diagonals and the number of distinct endpoints of the $k$ diagonals is $l$. + +Firstly, consider the longest diagonal in one of the two directions in $S$. No other diagonal in $S$ can start from either endpoint of that diagonal, since otherwise it has to meet another longer diagonal in $S$. The same holds true for the other direction. Ignoring these two longest diagonals and their four endpoints, the remaining $k-2$ diagonals share $l-4$ endpoints where each endpoint can belong to at most two diagonals. This gives $2(l-4) \geqslant 2(k-2)$, so that $k \leqslant l-2$. +![](https://cdn.mathpix.com/cropped/2024_11_18_e5e735288fb13d37ab0cg-40.jpg?height=430&width=1442&top_left_y=289&top_left_x=294) + +Consider a group of consecutive vertices of the regular $n$-gon so that each of the two outermost vertices is an endpoint of a diagonal in $S$, while the interior points are not. There are $l$ such groups. We label these groups $P_{1}, P_{2}, \ldots, P_{l}$ in this order. We claim that each selected diagonal outside $S$ must connect vertices of the same group $P_{i}$. Consider any diagonal $d$ joining vertices from distinct groups $P_{i}$ and $P_{j}$. Let $d_{1}$ and $d_{2}$ be two diagonals in $S$ each having one of the outermost points of $P_{i}$ as endpoint. Then $d$ must meet either $d_{1}, d_{2}$ or a diagonal in $S$ which is perpendicular to both $d_{1}$ and $d_{2}$. In any case $d$ should belong to $S$ by definition, which is a contradiction. + +Within the same group $P_{i}$, there are no perpendicular diagonals since the vertices belong to the same side of a diameter of the circumcircle. Hence there can be at most $\left|P_{i}\right|-2$ selected diagonals within $P_{i}$, including the one joining the two outermost points of $P_{i}$ when $\left|P_{i}\right|>2$. Therefore, the maximum number of diagonals selected is + +$$ +\sum_{i=1}^{l}\left(\left|P_{i}\right|-2\right)+k=\sum_{i=1}^{l}\left|P_{i}\right|-2 l+k=(n+l)-2 l+k=n-l+k \leqslant n-2 +$$ + +This upper bound can be attained as follows. We take any vertex $A$ and let $A^{\prime}$ be the vertex for which $A A^{\prime}$ is a diameter of the circumcircle. If we select all diagonals emanated from $A$ together with the diagonal $d^{\prime}$ joining the two neighbouring vertices of $A^{\prime}$, then the only pair of diagonals that meet each other is $A A^{\prime}$ and $d^{\prime}$, which are perpendicular to each other. In total we can take $n-2$ diagonals. +![](https://cdn.mathpix.com/cropped/2024_11_18_e5e735288fb13d37ab0cg-40.jpg?height=459&width=421&top_left_y=1798&top_left_x=803) + +Solution 2. The constructions and the odd case are the same as Solution 1. Instead of dealing separately with the case where $n$ is even, we shall prove by induction more generally that we can select at most $n-2$ diagonals for any cyclic $n$-gon with circumcircle $\Gamma$. + +The base case $n=3$ is trivial since there is no diagonal at all. Suppose the upper bound holds for any cyclic polygon with fewer than $n$ sides. For a cyclic $n$-gon, if there is a selected diagonal which does not intersect any other selected diagonal, then this diagonal divides the $n$-gon into an $m$-gon and an $l$-gon (with $m+l=n+2$ ) so that each selected diagonal belongs to one of them. Without loss of generality, we may assume the $m$-gon lies on the same side of a diameter of $\Gamma$. Then no two selected diagonals of the $m$-gon can intersect, and hence we can select at most $m-3$ diagonals. Also, we can apply the inductive hypothesis to the $l$-gon. This shows the maximum number of selected diagonals is $(m-3)+(l-2)+1=n-2$. + +It remains to consider the case when all selected diagonals meet at least one other selected diagonal. Consider a pair of selected perpendicular diagonals $d_{1}, d_{2}$. They divide the circumference of $\Gamma$ into four arcs, each of which lies on the same side of a diameter of $\Gamma$. If there are two selected diagonals intersecting each other and neither is parallel to $d_{1}$ or $d_{2}$, then their endpoints must belong to the same arc determined by $d_{1}, d_{2}$, and hence they cannot be perpendicular. This violates the condition, and hence all selected diagonals must have the same direction as one of $d_{1}, d_{2}$. +![](https://cdn.mathpix.com/cropped/2024_11_18_e5e735288fb13d37ab0cg-41.jpg?height=435&width=1234&top_left_y=1036&top_left_x=494) + +Take the longest selected diagonal in one of the two directions. We argue as in Solution 1 that its endpoints do not belong to any other selected diagonal. The same holds true for the longest diagonal in the other direction. Apart from these four endpoints, each of the remaining $n-4$ vertices can belong to at most two selected diagonals. Thus we can select at most $\frac{1}{2}(2(n-4)+4)=n-2$ diagonals. Then the proof follows by induction. + +C6. There are $n \geqslant 3$ islands in a city. Initially, the ferry company offers some routes between some pairs of islands so that it is impossible to divide the islands into two groups such that no two islands in different groups are connected by a ferry route. + +After each year, the ferry company will close a ferry route between some two islands $X$ and $Y$. At the same time, in order to maintain its service, the company will open new routes according to the following rule: for any island which is connected by a ferry route to exactly one of $X$ and $Y$, a new route between this island and the other of $X$ and $Y$ is added. + +Suppose at any moment, if we partition all islands into two nonempty groups in any way, then it is known that the ferry company will close a certain route connecting two islands from the two groups after some years. Prove that after some years there will be an island which is connected to all other islands by ferry routes. + +Solution. Initially, we pick any pair of islands $A$ and $B$ which are connected by a ferry route and put $A$ in set $\mathcal{A}$ and $B$ in set $\mathcal{B}$. From the condition, without loss of generality there must be another island which is connected to $A$. We put such an island $C$ in set $\mathcal{B}$. We say that two sets of islands form a network if each island in one set is connected to each island in the other set. + +Next, we shall included all islands to $\mathcal{A} \cup \mathcal{B}$ one by one. Suppose we have two sets $\mathcal{A}$ and $\mathcal{B}$ which form a network where $3 \leqslant|\mathcal{A} \cup \mathcal{B}|\angle C B A$. This shows $S$ and $O_{1}$ lie on different sides of $X Y$. As $S^{\prime}$ lies on ray $S O_{1}$, it follows that $S$ and $S^{\prime}$ cannot lie on the same side of $X Y$. Therefore, $S$ and $S^{\prime}$ are symmetric with respect to $X Y$. + +Let $d$ be the diameter of the circumcircle of triangle $X S Y$. As $S S^{\prime}$ is twice the distance from $S$ to $X Y$ and $S X=S Y$, we have $S S^{\prime}=2 \frac{S X^{2}}{d}$. It follows from (1) that $d=2 S O_{1}$. As $S O_{1}$ is the perpendicular bisector of $X Y$, point $O_{1}^{d}$ is the circumcentre of triangle $X S Y$. + +Solution 2. Denote the orthocentre and circumcentre of triangle $A B C$ by $H$ and $O$ respectively. Let $O_{1}$ be the circumcentre of triangle $X S Y$. Consider two other possible positions of $S$. We name them $S^{\prime}$ and $S^{\prime \prime}$ and define the analogous points $X^{\prime}, Y^{\prime}, O_{1}^{\prime}, X^{\prime \prime}, Y^{\prime \prime} O_{1}^{\prime \prime}$ accordingly. Note that $S, S^{\prime}, S^{\prime \prime}$ lie on the perpendicular bisector of $A D$. + +As $X X^{\prime}$ and $Y Y^{\prime}$ meet at $A$ and the circumcircles of triangles $A X Y$ and $A X^{\prime} Y^{\prime}$ meet at $D$, there is a spiral similarity with centre $D$ mapping $X Y$ to $X^{\prime} Y^{\prime}$. We find that + +$$ +\measuredangle S X Y=\frac{\pi}{2}-\measuredangle Y A X=\frac{\pi}{2}-\measuredangle Y^{\prime} A X^{\prime}=\measuredangle S^{\prime} X^{\prime} Y^{\prime} +$$ + +and similarly $\measuredangle S Y X=\measuredangle S^{\prime} Y^{\prime} X^{\prime}$. This shows triangles $S X Y$ and $S^{\prime} X^{\prime} Y^{\prime}$ are directly similar. Then the spiral similarity with centre $D$ takes points $S, X, Y, O_{1}$ to $S^{\prime}, X^{\prime}, Y^{\prime}, O_{1}^{\prime}$. Similarly, there is a spiral similarity with centre $D$ mapping $S, X, Y, O_{1}$ to $S^{\prime \prime}, X^{\prime \prime}, Y^{\prime \prime}, O_{1}^{\prime \prime}$. From these, we see that there is a spiral similarity taking the corresponding points $S, S^{\prime}, S^{\prime \prime}$ to points $O_{1}, O_{1}^{\prime}, O_{1}^{\prime \prime}$. In particular, $O_{1}, O_{1}^{\prime}, O_{1}^{\prime \prime}$ are collinear. +![](https://cdn.mathpix.com/cropped/2024_11_18_e5e735288fb13d37ab0cg-63.jpg?height=750&width=1071&top_left_y=251&top_left_x=592) + +It now suffices to show that $O_{1}$ lies on the perpendicular bisector of $P M$ for two special cases. + +Firstly, we take $S$ to be the midpoint of $A H$. Then $X$ and $Y$ are the feet of altitudes from $C$ and $B$ respectively. It is well-known that the circumcircle of triangle $X S Y$ is the nine-point circle of triangle $A B C$. Then $O_{1}$ is the nine-point centre and $O_{1} P=O_{1} M$. Indeed, $P$ and $M$ also lies on the nine-point circle. + +Secondly, we take $S^{\prime}$ to be the midpoint of $A O$. Then $X^{\prime}$ and $Y^{\prime}$ are the midpoints of $A B$ and $A C$ respectively. Then $X^{\prime} Y^{\prime} / / B C$. Clearly, $S^{\prime}$ lies on the perpendicular bisector of $P M$. This shows the perpendicular bisectors of $X^{\prime} Y^{\prime}$ and $P M$ coincide. Hence, we must have $O_{1}^{\prime} P=O_{1}^{\prime} M$. +![](https://cdn.mathpix.com/cropped/2024_11_18_e5e735288fb13d37ab0cg-63.jpg?height=548&width=1257&top_left_y=1610&top_left_x=481) + +G6. Let $A B C D$ be a convex quadrilateral with $\angle A B C=\angle A D C<90^{\circ}$. The internal angle bisectors of $\angle A B C$ and $\angle A D C$ meet $A C$ at $E$ and $F$ respectively, and meet each other at point $P$. Let $M$ be the midpoint of $A C$ and let $\omega$ be the circumcircle of triangle $B P D$. Segments $B M$ and $D M$ intersect $\omega$ again at $X$ and $Y$ respectively. Denote by $Q$ the intersection point of lines $X E$ and $Y F$. Prove that $P Q \perp A C$. + +## Solution 1. + +![](https://cdn.mathpix.com/cropped/2024_11_18_e5e735288fb13d37ab0cg-64.jpg?height=857&width=1492&top_left_y=604&top_left_x=273) + +Let $\omega_{1}$ be the circumcircle of triangle $A B C$. We first prove that $Y$ lies on $\omega_{1}$. Let $Y^{\prime}$ be the point on ray $M D$ such that $M Y^{\prime} \cdot M D=M A^{2}$. Then triangles $M A Y^{\prime}$ and $M D A$ are oppositely similar. Since $M C^{2}=M A^{2}=M Y^{\prime} \cdot M D$, triangles $M C Y^{\prime}$ and $M D C$ are also oppositely similar. Therefore, using directed angles, we have + +$$ +\measuredangle A Y^{\prime} C=\measuredangle A Y^{\prime} M+\measuredangle M Y^{\prime} C=\measuredangle M A D+\measuredangle D C M=\measuredangle C D A=\measuredangle A B C +$$ + +so that $Y^{\prime}$ lies on $\omega_{1}$. +Let $Z$ be the intersection point of lines $B C$ and $A D$. Since $\measuredangle P D Z=\measuredangle P B C=\measuredangle P B Z$, point $Z$ lies on $\omega$. In addition, from $\measuredangle Y^{\prime} B Z=\measuredangle Y^{\prime} B C=\measuredangle Y^{\prime} A C=\measuredangle Y^{\prime} A M=\measuredangle Y^{\prime} D Z$, we also know that $Y^{\prime}$ lies on $\omega$. Note that $\angle A D C$ is acute implies $M A \neq M D$ so $M Y^{\prime} \neq M D$. Therefore, $Y^{\prime}$ is the second intersection of $D M$ and $\omega$. Then $Y^{\prime}=Y$ and hence $Y$ lies on $\omega_{1}$. + +Next, by the Angle Bisector Theorem and the similar triangles, we have + +$$ +\frac{F A}{F C}=\frac{A D}{C D}=\frac{A D}{A M} \cdot \frac{C M}{C D}=\frac{Y A}{Y M} \cdot \frac{Y M}{Y C}=\frac{Y A}{Y C} +$$ + +Hence, $F Y$ is the internal angle bisector of $\angle A Y C$. +Let $B^{\prime}$ be the second intersection of the internal angle bisector of $\angle C B A$ and $\omega_{1}$. Then $B^{\prime}$ is the midpoint of arc $A C$ not containing $B$. Therefore, $Y B^{\prime}$ is the external angle bisector of $\angle A Y C$, so that $B^{\prime} Y \perp F Y$. + +Denote by $l$ the line through $P$ parallel to $A C$. Suppose $l$ meets line $B^{\prime} Y$ at $S$. From + +$$ +\begin{aligned} +\measuredangle P S Y & =\measuredangle\left(A C, B^{\prime} Y\right)=\measuredangle A C Y+\measuredangle C Y B^{\prime}=\measuredangle A C Y+\measuredangle C A B^{\prime}=\measuredangle A C Y+\measuredangle B^{\prime} C A \\ +& =\measuredangle B^{\prime} C Y=\measuredangle B^{\prime} B Y=\measuredangle P B Y +\end{aligned} +$$ + +the point $S$ lies on $\omega$. Similarly, the line through $X$ perpendicular to $X E$ also passes through the second intersection of $l$ and $\omega$, which is the point $S$. From $Q Y \perp Y S$ and $Q X \perp X S$, point $Q$ lies on $\omega$ and $Q S$ is a diameter of $\omega$. Therefore, $P Q \perp P S$ so that $P Q \perp A C$. + +Solution 2. Denote by $\omega_{1}$ and $\omega_{2}$ the circumcircles of triangles $A B C$ and $A D C$ respectively. Since $\angle A B C=\angle A D C$, we know that $\omega_{1}$ and $\omega_{2}$ are symmetric with respect to the midpoint $M$ of $A C$. + +Firstly, we show that $X$ lies on $\omega_{2}$. Let $X_{1}$ be the second intersection of ray $M B$ and $\omega_{2}$ and $X^{\prime}$ be its symmetric point with respect to $M$. Then $X^{\prime}$ lies on $\omega_{1}$ and $X^{\prime} A X_{1} C$ is a parallelogram. Hence, we have + +$$ +\begin{aligned} +\measuredangle D X_{1} B & =\measuredangle D X_{1} A+\measuredangle A X_{1} B=\measuredangle D C A+\measuredangle A X_{1} X^{\prime}=\measuredangle D C A+\measuredangle C X^{\prime} X_{1} \\ +& =\measuredangle D C A+\measuredangle C A B=\measuredangle(C D, A B) . +\end{aligned} +$$ + +![](https://cdn.mathpix.com/cropped/2024_11_18_e5e735288fb13d37ab0cg-65.jpg?height=914&width=1302&top_left_y=1191&top_left_x=449) + +Also, we have + +$$ +\measuredangle D P B=\measuredangle P D C+\angle(C D, A B)+\measuredangle A B P=\angle(C D, A B) . +$$ + +These yield $\measuredangle D X_{1} B=\measuredangle D P B$ and hence $X_{1}$ lies on $\omega$. It follows that $X_{1}=X$ and $X$ lies on $\omega_{2}$. Similarly, $Y$ lies on $\omega_{1}$. + +Next, we prove that $Q$ lies on $\omega$. Suppose the perpendicular bisector of $A C$ meet $\omega_{1}$ at $B^{\prime}$ and $M_{1}$ and meet $\omega_{2}$ at $D^{\prime}$ and $M_{2}$, so that $B, M_{1}$ and $D^{\prime}$ lie on the same side of $A C$. Note that $B^{\prime}$ lies on the angle bisector of $\angle A B C$ and similarly $D^{\prime}$ lies on $D P$. + +If we denote the area of $W_{1} W_{2} W_{3}$ by $\left[W_{1} W_{2} W_{3}\right]$, then + +$$ +\frac{B A \cdot X^{\prime} A}{B C \cdot X^{\prime} C}=\frac{\frac{1}{2} B A \cdot X^{\prime} A \sin \angle B A X^{\prime}}{\frac{1}{2} B C \cdot X^{\prime} C \sin \angle B C X^{\prime}}=\frac{\left[B A X^{\prime}\right]}{\left[B C X^{\prime}\right]}=\frac{M A}{M C}=1 +$$ + +As $B E$ is the angle bisector of $\angle A B C$, we have + +$$ +\frac{E A}{E C}=\frac{B A}{B C}=\frac{X^{\prime} C}{X^{\prime} A}=\frac{X A}{X C} +$$ + +Therefore, $X E$ is the angle bisector of $\angle A X C$, so that $M_{2}$ lies on the line joining $X, E, Q$. Analogously, $M_{1}, F, Q, Y$ are collinear. Thus, + +$$ +\begin{aligned} +\measuredangle X Q Y & =\measuredangle M_{2} Q M_{1}=\measuredangle Q M_{2} M_{1}+\measuredangle M_{2} M_{1} Q=\measuredangle X M_{2} D^{\prime}+\measuredangle B^{\prime} M_{1} Y \\ +& =\measuredangle X D D^{\prime}+\measuredangle B^{\prime} B Y=\measuredangle X D P+\measuredangle P B Y=\measuredangle X B P+\measuredangle P B Y=\measuredangle X B Y, +\end{aligned} +$$ + +which implies $Q$ lies on $\omega$. +Finally, as $M_{1}$ and $M_{2}$ are symmetric with respect to $M$, the quadrilateral $X^{\prime} M_{2} X M_{1}$ is a parallelogram. Consequently, + +$$ +\measuredangle X Q P=\measuredangle X B P=\measuredangle X^{\prime} B B^{\prime}=\measuredangle X^{\prime} M_{1} B^{\prime}=\measuredangle X M_{2} M_{1} . +$$ + +This shows $Q P / / M_{2} M_{1}$. As $M_{2} M_{1} \perp A C$, we get $Q P \perp A C$. +Solution 3. We first state two results which will be needed in our proof. + +- Claim 1. In $\triangle X^{\prime} Y^{\prime} Z^{\prime}$ with $X^{\prime} Y^{\prime} \neq X^{\prime} Z^{\prime}$, let $N^{\prime}$ be the midpoint of $Y^{\prime} Z^{\prime}$ and $W^{\prime}$ be the foot of internal angle bisector from $X^{\prime}$. Then $\tan ^{2} \measuredangle W^{\prime} X^{\prime} Z^{\prime}=\tan \measuredangle N^{\prime} X^{\prime} W^{\prime} \tan \measuredangle Z^{\prime} W^{\prime} X^{\prime}$. + +Proof. +![](https://cdn.mathpix.com/cropped/2024_11_18_e5e735288fb13d37ab0cg-66.jpg?height=416&width=562&top_left_y=1684&top_left_x=733) + +Without loss of generality, assume $X^{\prime} Y^{\prime}>X^{\prime} Z^{\prime}$. Then $W^{\prime}$ lies between $N^{\prime}$ and $Z^{\prime}$. The signs of both sides agree so it suffices to establish the relation for ordinary angles. Let $\angle W^{\prime} X^{\prime} Z^{\prime}=\alpha, \angle N^{\prime} X^{\prime} W^{\prime}=\beta$ and $\angle Z^{\prime} W^{\prime} X^{\prime}=\gamma$. We have + +$$ +\frac{\sin (\gamma-\alpha)}{\sin (\alpha-\beta)}=\frac{N^{\prime} X^{\prime}}{N^{\prime} Y^{\prime}}=\frac{N^{\prime} X^{\prime}}{N^{\prime} Z^{\prime}}=\frac{\sin (\gamma+\alpha)}{\sin (\alpha+\beta)} +$$ + +This implies + +$$ +\frac{\tan \gamma-\tan \alpha}{\tan \gamma+\tan \alpha}=\frac{\sin \gamma \cos \alpha-\cos \gamma \sin \alpha}{\sin \gamma \cos \alpha+\cos \gamma \sin \alpha}=\frac{\sin \alpha \cos \beta-\cos \alpha \sin \beta}{\sin \alpha \cos \beta+\cos \alpha \sin \beta}=\frac{\tan \alpha-\tan \beta}{\tan \alpha+\tan \beta} +$$ + +Expanding and simplifying, we get the desired result $\tan ^{2} \alpha=\tan \beta \tan \gamma$. + +- Claim 2. Let $A^{\prime} B^{\prime} C^{\prime} D^{\prime}$ be a quadrilateral inscribed in circle $\Gamma$. Let diagonals $A^{\prime} C^{\prime}$ and $B^{\prime} D^{\prime}$ meet at $E^{\prime}$, and $F^{\prime}$ be the intersection of lines $A^{\prime} B^{\prime}$ and $C^{\prime} D^{\prime}$. Let $M^{\prime}$ be the midpoint of $E^{\prime} F^{\prime}$. Then the power of $M^{\prime}$ with respect to $\Gamma$ is equal to $\left(M^{\prime} E^{\prime}\right)^{2}$. + +Proof. +![](https://cdn.mathpix.com/cropped/2024_11_18_e5e735288fb13d37ab0cg-67.jpg?height=624&width=904&top_left_y=1000&top_left_x=678) + +Let $O^{\prime}$ be the centre of $\Gamma$ and let $\Gamma^{\prime}$ be the circle with centre $M^{\prime}$ passing through $E^{\prime}$. Let $F_{1}$ be the inversion image of $F^{\prime}$ with respect to $\Gamma$. It is well-known that $E^{\prime}$ lies on the polar of $F^{\prime}$ with respect to $\Gamma$. This shows $E^{\prime} F_{1} \perp O^{\prime} F^{\prime}$ and hence $F_{1}$ lies on $\Gamma^{\prime}$. It follows that the inversion image of $\Gamma^{\prime}$ with respect to $\Gamma$ is $\Gamma^{\prime}$ itself. This shows $\Gamma^{\prime}$ is orthogonal to $\Gamma$, and thus the power of $M^{\prime}$ with respect to $\Gamma$ is the square of radius of $\Gamma^{\prime}$, which is $\left(M^{\prime} E^{\prime}\right)^{2}$. + +We return to the main problem. Let $Z$ be the intersection of lines $A D$ and $B C$, and $W$ be the intersection of lines $A B$ and $C D$. Since $\measuredangle P D Z=\measuredangle P B C=\measuredangle P B Z$, point $Z$ lies on $\omega$. Similarly, $W$ lies on $\omega$. Applying Claim 2 to the cyclic quadrilateral $Z B D W$, we know that the power of $M$ with respect to $\omega$ is $M A^{2}$. Hence, $M X \cdot M B=M A^{2}$. + +Suppose the line through $B$ perpendicular to $B E$ meets line $A C$ at $T$. Then $B E$ and $B T$ are the angle bisectors of $\angle C B A$. This shows $(T, E ; A, C)$ is harmonic. Thus, we have $M E \cdot M T=M A^{2}=M X \cdot M B$. It follows that $E, T, B, X$ are concyclic. +![](https://cdn.mathpix.com/cropped/2024_11_18_e5e735288fb13d37ab0cg-68.jpg?height=808&width=1424&top_left_y=252&top_left_x=285) + +The result is trivial for the special case $A D=C D$ since $P, Q$ lie on the perpendicular bisector of $A C$ in that case. Similarly, the case $A B=C B$ is trivial. It remains to consider the general cases where we can apply Claim 1 in the latter part of the proof. + +Let the projections from $P$ and $Q$ to $A C$ be $P^{\prime}$ and $Q^{\prime}$ respectively. Then $P Q \perp A C$ if and only if $P^{\prime}=Q^{\prime}$ if and only if $\frac{E P^{\prime}}{F P^{\prime}}=\frac{E Q^{\prime}}{F Q^{\prime}}$ in terms of directed lengths. Note that + +$$ +\frac{E P^{\prime}}{F P^{\prime}}=\frac{\tan \measuredangle E F P}{\tan \measuredangle F E P}=\frac{\tan \measuredangle A F D}{\tan \measuredangle A E B} +$$ + +Next, we have $\frac{E Q^{\prime}}{F Q^{\prime}}=\frac{\tan \measuredangle E F Q}{\tan \measuredangle F E Q}$ where $\measuredangle F E Q=\measuredangle T E X=\measuredangle T B X=\frac{\pi}{2}+\measuredangle E B M$ and by symmetry $\measuredangle E F Q=\frac{\pi}{2}+\measuredangle F D M$. Combining all these, it suffices to show + +$$ +\frac{\tan \measuredangle A F D}{\tan \measuredangle A E B}=\frac{\tan \measuredangle M B E}{\tan \measuredangle M D F} +$$ + +We now apply Claim 1 twice to get + +$$ +\tan \measuredangle A F D \tan \measuredangle M D F=\tan ^{2} \measuredangle F D C=\tan ^{2} \measuredangle E B A=\tan \measuredangle M B E \tan \measuredangle A E B . +$$ + +The result then follows. + +G7. Let $I$ be the incentre of a non-equilateral triangle $A B C, I_{A}$ be the $A$-excentre, $I_{A}^{\prime}$ be the reflection of $I_{A}$ in $B C$, and $l_{A}$ be the reflection of line $A I_{A}^{\prime}$ in $A I$. Define points $I_{B}, I_{B}^{\prime}$ and line $l_{B}$ analogously. Let $P$ be the intersection point of $l_{A}$ and $l_{B}$. +(a) Prove that $P$ lies on line $O I$ where $O$ is the circumcentre of triangle $A B C$. +(b) Let one of the tangents from $P$ to the incircle of triangle $A B C$ meet the circumcircle at points $X$ and $Y$. Show that $\angle X I Y=120^{\circ}$. + +## Solution 1. + +(a) Let $A^{\prime}$ be the reflection of $A$ in $B C$ and let $M$ be the second intersection of line $A I$ and the circumcircle $\Gamma$ of triangle $A B C$. As triangles $A B A^{\prime}$ and $A O C$ are isosceles with $\angle A B A^{\prime}=2 \angle A B C=\angle A O C$, they are similar to each other. Also, triangles $A B I_{A}$ and $A I C$ are similar. Therefore we have + +$$ +\frac{A A^{\prime}}{A I_{A}}=\frac{A A^{\prime}}{A B} \cdot \frac{A B}{A I_{A}}=\frac{A C}{A O} \cdot \frac{A I}{A C}=\frac{A I}{A O} +$$ + +Together with $\angle A^{\prime} A I_{A}=\angle I A O$, we find that triangles $A A^{\prime} I_{A}$ and $A I O$ are similar. +![](https://cdn.mathpix.com/cropped/2024_11_18_e5e735288fb13d37ab0cg-69.jpg?height=890&width=1025&top_left_y=1192&top_left_x=653) + +Denote by $P^{\prime}$ the intersection of line $A P$ and line $O I$. Using directed angles, we have + +$$ +\begin{aligned} +\measuredangle M A P^{\prime} & =\measuredangle I_{A}^{\prime} A I_{A}=\measuredangle I_{A}^{\prime} A A^{\prime}-\measuredangle I_{A} A A^{\prime}=\measuredangle A A^{\prime} I_{A}-\measuredangle(A M, O M) \\ +& =\measuredangle A I O-\measuredangle A M O=\measuredangle M O P^{\prime} . +\end{aligned} +$$ + +This shows $M, O, A, P^{\prime}$ are concyclic. + +Denote by $R$ and $r$ the circumradius and inradius of triangle $A B C$. Then + +$$ +I P^{\prime}=\frac{I A \cdot I M}{I O}=\frac{I O^{2}-R^{2}}{I O} +$$ + +is independent of $A$. Hence, $B P$ also meets line $O I$ at the same point $P^{\prime}$ so that $P^{\prime}=P$, and $P$ lies on $O I$. +(b) By Poncelet's Porism, the other tangents to the incircle of triangle $A B C$ from $X$ and $Y$ meet at a point $Z$ on $\Gamma$. Let $T$ be the touching point of the incircle to $X Y$, and let $D$ be the midpoint of $X Y$. We have + +$$ +\begin{aligned} +O D & =I T \cdot \frac{O P}{I P}=r\left(1+\frac{O I}{I P}\right)=r\left(1+\frac{O I^{2}}{O I \cdot I P}\right)=r\left(1+\frac{R^{2}-2 R r}{R^{2}-I O^{2}}\right) \\ +& =r\left(1+\frac{R^{2}-2 R r}{2 R r}\right)=\frac{R}{2}=\frac{O X}{2} +\end{aligned} +$$ + +This shows $\angle X Z Y=60^{\circ}$ and hence $\angle X I Y=120^{\circ}$. + +## Solution 2. + +(a) Note that triangles $A I_{B} C$ and $I_{A} B C$ are similar since their corresponding interior angles are equal. Therefore, the four triangles $A I_{B}^{\prime} C, A I_{B} C, I_{A} B C$ and $I_{A}^{\prime} B C$ are all similar. From $\triangle A I_{B}^{\prime} C \sim \triangle I_{A}^{\prime} B C$, we get $\triangle A I_{A}^{\prime} C \sim \triangle I_{B}^{\prime} B C$. From $\measuredangle A B P=\measuredangle I_{B}^{\prime} B C=\measuredangle A I_{A}^{\prime} C$ and $\measuredangle B A P=\measuredangle I_{A}^{\prime} A C$, the triangles $A B P$ and $A I_{A}^{\prime} C$ are directly similar. +![](https://cdn.mathpix.com/cropped/2024_11_18_e5e735288fb13d37ab0cg-70.jpg?height=820&width=1137&top_left_y=1455&top_left_x=494) + +Consider the inversion with centre $A$ and radius $\sqrt{A B \cdot A C}$ followed by the reflection in $A I$. Then $B$ and $C$ are mapped to each other, and $I$ and $I_{A}$ are mapped to each other. + +From the similar triangles obtained, we have $A P \cdot A I_{A}^{\prime}=A B \cdot A C$ so that $P$ is mapped to $I_{A}^{\prime}$ under the transformation. In addition, line $A O$ is mapped to the altitude from $A$, and hence $O$ is mapped to the reflection of $A$ in $B C$, which we call point $A^{\prime}$. Note that $A A^{\prime} I_{A} I_{A}^{\prime}$ is an isosceles trapezoid, which shows it is inscribed in a circle. The preimage of this circle is a straight line, meaning that $O, I, P$ are collinear. +(b) Denote by $R$ and $r$ the circumradius and inradius of triangle $A B C$. Note that by the above transformation, we have $\triangle A P O \sim \triangle A A^{\prime} I_{A}^{\prime}$ and $\triangle A A^{\prime} I_{A} \sim \triangle A I O$. Therefore, we find that + +$$ +P O=A^{\prime} I_{A}^{\prime} \cdot \frac{A O}{A I_{A}^{\prime}}=A I_{A} \cdot \frac{A O}{A^{\prime} I_{A}}=\frac{A I_{A}}{A^{\prime} I_{A}} \cdot A O=\frac{A O}{I O} \cdot A O +$$ + +This shows $P O \cdot I O=R^{2}$, and it follows that $P$ and $I$ are mapped to each other under the inversion with respect to the circumcircle $\Gamma$ of triangle $A B C$. Then $P X \cdot P Y$, which is the power of $P$ with respect to $\Gamma$, equals $P I \cdot P O$. This yields $X, I, O, Y$ are concyclic. + +Let $T$ be the touching point of the incircle to $X Y$, and let $D$ be the midpoint of $X Y$. Then + +$$ +O D=I T \cdot \frac{P O}{P I}=r \cdot \frac{P O}{P O-I O}=r \cdot \frac{R^{2}}{R^{2}-I O^{2}}=r \cdot \frac{R^{2}}{2 R r}=\frac{R}{2} +$$ + +This shows $\angle D O X=60^{\circ}$ and hence $\angle X I Y=\angle X O Y=120^{\circ}$. +Comment. A simplification of this problem is to ask part (a) only. Note that the question in part (b) implicitly requires $P$ to lie on $O I$, or otherwise the angle is not uniquely determined as we can find another tangent from $P$ to the incircle. + +G8. Let $A_{1}, B_{1}$ and $C_{1}$ be points on sides $B C, C A$ and $A B$ of an acute triangle $A B C$ respectively, such that $A A_{1}, B B_{1}$ and $C C_{1}$ are the internal angle bisectors of triangle $A B C$. Let $I$ be the incentre of triangle $A B C$, and $H$ be the orthocentre of triangle $A_{1} B_{1} C_{1}$. Show that + +$$ +A H+B H+C H \geqslant A I+B I+C I +$$ + +Solution. Without loss of generality, assume $\alpha=\angle B A C \leqslant \beta=\angle C B A \leqslant \gamma=\angle A C B$. Denote by $a, b, c$ the lengths of $B C, C A, A B$ respectively. We first show that triangle $A_{1} B_{1} C_{1}$ is acute. + +Choose points $D$ and $E$ on side $B C$ such that $B_{1} D / / A B$ and $B_{1} E$ is the internal angle bisector of $\angle B B_{1} C$. As $\angle B_{1} D B=180^{\circ}-\beta$ is obtuse, we have $B B_{1}>B_{1} D$. Thus, + +$$ +\frac{B E}{E C}=\frac{B B_{1}}{B_{1} C}>\frac{D B_{1}}{B_{1} C}=\frac{B A}{A C}=\frac{B A_{1}}{A_{1} C} +$$ + +Therefore, $B E>B A_{1}$ and $\frac{1}{2} \angle B B_{1} C=\angle B B_{1} E>\angle B B_{1} A_{1}$. Similarly, $\frac{1}{2} \angle B B_{1} A>\angle B B_{1} C_{1}$. It follows that + +$$ +\angle A_{1} B_{1} C_{1}=\angle B B_{1} A_{1}+\angle B B_{1} C_{1}<\frac{1}{2}\left(\angle B B_{1} C+\angle B B_{1} A\right)=90^{\circ} +$$ + +is acute. By symmetry, triangle $A_{1} B_{1} C_{1}$ is acute. +Let $B B_{1}$ meet $A_{1} C_{1}$ at $F$. From $\alpha \leqslant \gamma$, we get $a \leqslant c$, which implies + +$$ +B A_{1}=\frac{c a}{b+c} \leqslant \frac{a c}{a+b}=B C_{1} +$$ + +and hence $\angle B C_{1} A_{1} \leqslant \angle B A_{1} C_{1}$. As $B F$ is the internal angle bisector of $\angle A_{1} B C_{1}$, this shows $\angle B_{1} F C_{1}=\angle B F A_{1} \leqslant 90^{\circ}$. Hence, $H$ lies on the same side of $B B_{1}$ as $C_{1}$. This shows $H$ lies inside triangle $B B_{1} C_{1}$. Similarly, from $\alpha \leqslant \beta$ and $\beta \leqslant \gamma$, we know that $H$ lies inside triangles $C C_{1} B_{1}$ and $A A_{1} C_{1}$. +![](https://cdn.mathpix.com/cropped/2024_11_18_e5e735288fb13d37ab0cg-72.jpg?height=779&width=1010&top_left_y=1660&top_left_x=506) + +As $\alpha \leqslant \beta \leqslant \gamma$, we have $\alpha \leqslant 60^{\circ} \leqslant \gamma$. Then $\angle B I C \leqslant 120^{\circ} \leqslant \angle A I B$. Firstly, suppose $\angle A I C \geqslant 120^{\circ}$. + +Rotate points $B, I, H$ through $60^{\circ}$ about $A$ to $B^{\prime}, I^{\prime}, H^{\prime}$ so that $B^{\prime}$ and $C$ lie on different sides of $A B$. Since triangle $A I^{\prime} I$ is equilateral, we have + +$$ +A I+B I+C I=I^{\prime} I+B^{\prime} I^{\prime}+I C=B^{\prime} I^{\prime}+I^{\prime} I+I C . +$$ + +Similarly, + +$$ +A H+B H+C H=H^{\prime} H+B^{\prime} H^{\prime}+H C=B^{\prime} H^{\prime}+H^{\prime} H+H C +$$ + +As $\angle A I I^{\prime}=\angle A I^{\prime} I=60^{\circ}, \angle A I^{\prime} B^{\prime}=\angle A I B \geqslant 120^{\circ}$ and $\angle A I C \geqslant 120^{\circ}$, the quadrilateral $B^{\prime} I^{\prime} I C$ is convex and lies on the same side of $B^{\prime} C$ as $A$. + +Next, since $H$ lies inside triangle $A C C_{1}, H$ lies outside $B^{\prime} I^{\prime} I C$. Also, $H$ lying inside triangle $A B I$ implies $H^{\prime}$ lies inside triangle $A B^{\prime} I^{\prime}$. This shows $H^{\prime}$ lies outside $B^{\prime} I^{\prime} I C$ and hence the convex quadrilateral $B^{\prime} I^{\prime} I C$ is contained inside the quadrilateral $B^{\prime} H^{\prime} H C$. It follows that the perimeter of $B^{\prime} I^{\prime} I C$ cannot exceed the perimeter of $B^{\prime} H^{\prime} H C$. From (1) and (2), we conclude that + +$$ +A H+B H+C H \geqslant A I+B I+C I +$$ + +For the case $\angle A I C<120^{\circ}$, we can rotate $B, I, H$ through $60^{\circ}$ about $C$ to $B^{\prime}, I^{\prime}, H^{\prime}$ so that $B^{\prime}$ and $A$ lie on different sides of $B C$. The proof is analogous to the previous case and we still get the desired inequality. + +## Number Theory + +N1. For any positive integer $k$, denote the sum of digits of $k$ in its decimal representation by $S(k)$. Find all polynomials $P(x)$ with integer coefficients such that for any positive integer $n \geqslant 2016$, the integer $P(n)$ is positive and + +$$ +S(P(n))=P(S(n)) +$$ + +Answer. + +- $P(x)=c$ where $1 \leqslant c \leqslant 9$ is an integer; or +- $P(x)=x$. + +Solution 1. We consider three cases according to the degree of $P$. + +- Case 1. $P(x)$ is a constant polynomial. + +Let $P(x)=c$ where $c$ is an integer constant. Then (1) becomes $S(c)=c$. This holds if and only if $1 \leqslant c \leqslant 9$. + +- Case 2. $\operatorname{deg} P=1$. + +We have the following observation. For any positive integers $m, n$, we have + +$$ +S(m+n) \leqslant S(m)+S(n) +$$ + +and equality holds if and only if there is no carry in the addition $m+n$. +Let $P(x)=a x+b$ for some integers $a, b$ where $a \neq 0$. As $P(n)$ is positive for large $n$, we must have $a \geqslant 1$. The condition (1) becomes $S(a n+b)=a S(n)+b$ for all $n \geqslant 2016$. Setting $n=2025$ and $n=2020$ respectively, we get + +$$ +S(2025 a+b)-S(2020 a+b)=(a S(2025)+b)-(a S(2020)+b)=5 a +$$ + +On the other hand, (2) implies + +$$ +S(2025 a+b)=S((2020 a+b)+5 a) \leqslant S(2020 a+b)+S(5 a) +$$ + +These give $5 a \leqslant S(5 a)$. As $a \geqslant 1$, this holds only when $a=1$, in which case (1) reduces to $S(n+b)=S(n)+b$ for all $n \geqslant 2016$. Then we find that + +$$ +S(n+1+b)-S(n+b)=(S(n+1)+b)-(S(n)+b)=S(n+1)-S(n) +$$ + +If $b>0$, we choose $n$ such that $n+1+b=10^{k}$ for some sufficiently large $k$. Note that all the digits of $n+b$ are 9 's, so that the left-hand side of (3) equals $1-9 k$. As $n$ is a positive integer less than $10^{k}-1$, we have $S(n)<9 k$. Therefore, the right-hand side of (3) is at least $1-(9 k-1)=2-9 k$, which is a contradiction. + +The case $b<0$ can be handled similarly by considering $n+1$ to be a large power of 10 . Therefore, we conclude that $P(x)=x$, in which case (1) is trivially satisfied. + +- Case 3. $\operatorname{deg} P \geqslant 2$. + +Suppose the leading term of $P$ is $a_{d} n^{d}$ where $a_{d} \neq 0$. Clearly, we have $a_{d}>0$. Consider $n=10^{k}-1$ in (1). We get $S(P(n))=P(9 k)$. Note that $P(n)$ grows asymptotically as fast as $n^{d}$, so $S(P(n))$ grows asymptotically as no faster than a constant multiple of $k$. On the other hand, $P(9 k)$ grows asymptotically as fast as $k^{d}$. This shows the two sides of the last equation cannot be equal for sufficiently large $k$ since $d \geqslant 2$. + +Therefore, we conclude that $P(x)=c$ where $1 \leqslant c \leqslant 9$ is an integer, or $P(x)=x$. +Solution 2. Let $P(x)=a_{d} x^{d}+a_{d-1} x^{d-1}+\cdots+a_{0}$. Clearly $a_{d}>0$. There exists an integer $m \geqslant 1$ such that $\left|a_{i}\right|<10^{m}$ for all $0 \leqslant i \leqslant d$. Consider $n=9 \times 10^{k}$ for a sufficiently large integer $k$ in (1). If there exists an index $0 \leqslant i \leqslant d-1$ such that $a_{i}<0$, then all digits of $P(n)$ in positions from $10^{i k+m+1}$ to $10^{(i+1) k-1}$ are all 9 's. Hence, we have $S(P(n)) \geqslant 9(k-m-1)$. On the other hand, $P(S(n))=P(9)$ is a fixed constant. Therefore, (1) cannot hold for large $k$. This shows $a_{i} \geqslant 0$ for all $0 \leqslant i \leqslant d-1$. + +Hence, $P(n)$ is an integer formed by the nonnegative integers $a_{d} \times 9^{d}, a_{d-1} \times 9^{d-1}, \ldots, a_{0}$ by inserting some zeros in between. This yields + +$$ +S(P(n))=S\left(a_{d} \times 9^{d}\right)+S\left(a_{d-1} \times 9^{d-1}\right)+\cdots+S\left(a_{0}\right) . +$$ + +Combining with (1), we have + +$$ +S\left(a_{d} \times 9^{d}\right)+S\left(a_{d-1} \times 9^{d-1}\right)+\cdots+S\left(a_{0}\right)=P(9)=a_{d} \times 9^{d}+a_{d-1} \times 9^{d-1}+\cdots+a_{0} +$$ + +As $S(m) \leqslant m$ for any positive integer $m$, with equality when $1 \leqslant m \leqslant 9$, this forces each $a_{i} \times 9^{i}$ to be a positive integer between 1 and 9 . In particular, this shows $a_{i}=0$ for $i \geqslant 2$ and hence $d \leqslant 1$. Also, we have $a_{1} \leqslant 1$ and $a_{0} \leqslant 9$. If $a_{1}=1$ and $1 \leqslant a_{0} \leqslant 9$, we take $n=10^{k}+\left(10-a_{0}\right)$ for sufficiently large $k$ in (1). This yields a contradiction since + +$$ +S(P(n))=S\left(10^{k}+10\right)=2 \neq 11=P\left(11-a_{0}\right)=P(S(n)) +$$ + +The zero polynomial is also rejected since $P(n)$ is positive for large $n$. The remaining candidates are $P(x)=x$ or $P(x)=a_{0}$ where $1 \leqslant a_{0} \leqslant 9$, all of which satisfy (1), and hence are the only solutions. + +N2. Let $\tau(n)$ be the number of positive divisors of $n$. Let $\tau_{1}(n)$ be the number of positive divisors of $n$ which have remainders 1 when divided by 3 . Find all possible integral values of the fraction $\frac{\tau(10 n)}{\tau_{1}(10 n)}$. + +Answer. All composite numbers together with 2. +Solution. In this solution, we always use $p_{i}$ to denote primes congruent to $1 \bmod 3$, and use $q_{j}$ to denote primes congruent to $2 \bmod 3$. When we express a positive integer $m$ using its prime factorization, we also include the special case $m=1$ by allowing the exponents to be zeros. We first compute $\tau_{1}(m)$ for a positive integer $m$. + +- Claim. Let $m=3^{x} p_{1}^{a_{1}} p_{2}^{a_{2}} \cdots p_{s}^{a_{s}} q_{1}^{b_{1}} q_{2}^{b_{2}} \cdots q_{t}^{b_{t}}$ be the prime factorization of $m$. Then + +$$ +\tau_{1}(m)=\prod_{i=1}^{s}\left(a_{i}+1\right)\left\lceil\frac{1}{2} \prod_{j=1}^{t}\left(b_{j}+1\right)\right\rceil . +$$ + +Proof. To choose a divisor of $m$ congruent to $1 \bmod 3$, it cannot have the prime divisor 3, while there is no restriction on choosing prime factors congruent to $1 \bmod 3$. Also, we have to choose an even number of prime factors (counted with multiplicity) congruent to $2 \bmod 3$. + +If $\prod_{j=1}^{t}\left(b_{j}+1\right)$ is even, then we may assume without loss of generality $b_{1}+1$ is even. We can choose the prime factors $q_{2}, q_{3}, \ldots, q_{t}$ freely in $\prod_{j=2}^{t}\left(b_{j}+1\right)$ ways. Then the parity of the number of $q_{1}$ is uniquely determined, and hence there are $\frac{1}{2}\left(b_{1}+1\right)$ ways to choose the exponent of $q_{1}$. Hence (1) is verified in this case. + +If $\prod_{j=1}^{t}\left(b_{j}+1\right)$ is odd, we use induction on $t$ to count the number of choices. When $t=1$, there are $\left\lceil\frac{b_{1}+1}{2}\right\rceil$ choices for which the exponent is even and $\left\lfloor\frac{b_{1}+1}{2}\right\rfloor$ choices for which the exponent is odd. For the inductive step, we find that there are + +$$ +\left\lceil\frac{1}{2} \prod_{j=1}^{t-1}\left(b_{j}+1\right)\right\rceil \cdot\left\lceil\frac{b_{t}+1}{2}\right\rceil+\left\lfloor\frac{1}{2} \prod_{j=1}^{t-1}\left(b_{j}+1\right)\right\rfloor \cdot\left\lfloor\frac{b_{t}+1}{2}\right\rfloor=\left\lceil\frac{1}{2} \prod_{j=1}^{t}\left(b_{j}+1\right)\right\rceil +$$ + +choices with an even number of prime factors and hence $\left\lfloor\frac{1}{2} \prod_{j=1}^{t}\left(b_{j}+1\right)\right\rfloor$ choices with an odd number of prime factors. Hence (1) is also true in this case. + +Let $n=3^{x} 2^{y} 5^{z} p_{1}^{a_{1}} p_{2}^{a_{2}} \cdots p_{s}^{a_{s}} q_{1}^{b_{1}} q_{2}^{b_{2}} \cdots q_{t}^{b_{t}}$. Using the well-known formula for computing the divisor function, we get + +$$ +\tau(10 n)=(x+1)(y+2)(z+2) \prod_{i=1}^{s}\left(a_{i}+1\right) \prod_{j=1}^{t}\left(b_{j}+1\right) +$$ + +By the Claim, we have + +$$ +\tau_{1}(10 n)=\prod_{i=1}^{s}\left(a_{i}+1\right)\left\lceil\frac{1}{2}(y+2)(z+2) \prod_{j=1}^{t}\left(b_{j}+1\right)\right\rceil +$$ + +If $c=(y+2)(z+2) \prod_{j=1}^{t}\left(b_{j}+1\right)$ is even, then (2) and (3) imply + +$$ +\frac{\tau(10 n)}{\tau_{1}(10 n)}=2(x+1) +$$ + +In this case $\frac{\tau(10 n)}{\tau_{1}(10 n)}$ can be any even positive integer as $x$ runs through all nonnegative integers. +If $c$ is odd, which means $y, z$ are odd and each $b_{j}$ is even, then (2) and (3) imply + +$$ +\frac{\tau(10 n)}{\tau_{1}(10 n)}=\frac{2(x+1) c}{c+1} +$$ + +For this to be an integer, we need $c+1$ divides $2(x+1)$ since $c$ and $c+1$ are relatively prime. Let $2(x+1)=k(c+1)$. Then (4) reduces to + +$$ +\frac{\tau(10 n)}{\tau_{1}(10 n)}=k c=k(y+2)(z+2) \prod_{j=1}^{t}\left(b_{j}+1\right) +$$ + +Noting that $y, z$ are odd, the integers $y+2$ and $z+2$ are at least 3 . This shows the integer in this case must be composite. On the other hand, for any odd composite number $a b$ with $a, b \geqslant 3$, we may simply take $n=3^{\frac{a b-1}{2}} \cdot 2^{a-2} \cdot 5^{b-2}$ so that $\frac{\tau(10 n)}{\tau_{1}(10 n)}=a b$ from (5). + +We conclude that the fraction can be any even integer or any odd composite number. Equivalently, it can be 2 or any composite number. + +N3. Define $P(n)=n^{2}+n+1$. For any positive integers $a$ and $b$, the set + +$$ +\{P(a), P(a+1), P(a+2), \ldots, P(a+b)\} +$$ + +is said to be fragrant if none of its elements is relatively prime to the product of the other elements. Determine the smallest size of a fragrant set. + +Answer. 6. +Solution. We have the following observations. +(i) $(P(n), P(n+1))=1$ for any $n$. + +We have $(P(n), P(n+1))=\left(n^{2}+n+1, n^{2}+3 n+3\right)=\left(n^{2}+n+1,2 n+2\right)$. Noting that $n^{2}+n+1$ is odd and $\left(n^{2}+n+1, n+1\right)=(1, n+1)=1$, the claim follows. +(ii) $(P(n), P(n+2))=1$ for $n \not \equiv 2(\bmod 7)$ and $(P(n), P(n+2))=7$ for $n \equiv 2(\bmod 7)$. + +From $(2 n+7) P(n)-(2 n-1) P(n+2)=14$ and the fact that $P(n)$ is odd, $(P(n), P(n+2))$ must be a divisor of 7 . The claim follows by checking $n \equiv 0,1, \ldots, 6(\bmod 7)$ directly. +(iii) $(P(n), P(n+3))=1$ for $n \not \equiv 1(\bmod 3)$ and $3 \mid(P(n), P(n+3))$ for $n \equiv 1(\bmod 3)$. + +From $(n+5) P(n)-(n-1) P(n+3)=18$ and the fact that $P(n)$ is odd, $(P(n), P(n+3))$ must be a divisor of 9 . The claim follows by checking $n \equiv 0,1,2(\bmod 3)$ directly. + +Suppose there exists a fragrant set with at most 5 elements. We may assume it contains exactly 5 elements $P(a), P(a+1), \ldots, P(a+4)$ since the following argument also works with fewer elements. Consider $P(a+2)$. From (i), it is relatively prime to $P(a+1)$ and $P(a+3)$. Without loss of generality, assume $(P(a), P(a+2))>1$. From (ii), we have $a \equiv 2(\bmod 7)$. The same observation implies $(P(a+1), P(a+3))=1$. In order that the set is fragrant, $(P(a), P(a+3))$ and $(P(a+1), P(a+4))$ must both be greater than 1. From (iii), this holds only when both $a$ and $a+1$ are congruent to $1 \bmod 3$, which is a contradiction. + +It now suffices to construct a fragrant set of size 6. By the Chinese Remainder Theorem, we can take a positive integer $a$ such that + +$$ +a \equiv 7 \quad(\bmod 19), \quad a+1 \equiv 2 \quad(\bmod 7), \quad a+2 \equiv 1 \quad(\bmod 3) +$$ + +For example, we may take $a=197$. From (ii), both $P(a+1)$ and $P(a+3)$ are divisible by 7. From (iii), both $P(a+2)$ and $P(a+5)$ are divisible by 3 . One also checks from $19 \mid P(7)=57$ and $19 \mid P(11)=133$ that $P(a)$ and $P(a+4)$ are divisible by 19. Therefore, the set $\{P(a), P(a+1), \ldots, P(a+5)\}$ is fragrant. + +Therefore, the smallest size of a fragrant set is 6 . +Comment. "Fragrant Harbour" is the English translation of "Hong Kong". +A stronger version of this problem is to show that there exists a fragrant set of size $k$ for any $k \geqslant 6$. We present a proof here. + +For each even positive integer $m$ which is not divisible by 3 , since $m^{2}+3 \equiv 3(\bmod 4)$, we can find a prime $p_{m} \equiv 3(\bmod 4)$ such that $p_{m} \mid m^{2}+3$. Clearly, $p_{m}>3$. + +If $b=2 t \geqslant 6$, we choose $a$ such that $3 \mid 2(a+t)+1$ and $p_{m} \mid 2(a+t)+1$ for each $1 \leqslant m \leqslant b$ with $m \equiv 2,4(\bmod 6)$. For $0 \leqslant r \leqslant t$ and $3 \mid r$, we have $a+t \pm r \equiv 1(\bmod 3)$ so that $3 \mid P(a+t \pm r)$. For $0 \leqslant r \leqslant t$ and $(r, 3)=1$, we have + +$$ +4 P(a+t \pm r) \equiv(-1 \pm 2 r)^{2}+2(-1 \pm 2 r)+4=4 r^{2}+3 \equiv 0 \quad\left(\bmod p_{2 r}\right) . +$$ + +Hence, $\{P(a), P(a+1), \ldots, P(a+b)\}$ is fragrant. +If $b=2 t+1 \geqslant 7$ (the case $b=5$ has been done in the original problem), we choose $a$ such that $3 \mid 2(a+t)+1$ and $p_{m} \mid 2(a+t)+1$ for $1 \leqslant m \leqslant b$ with $m \equiv 2,4(\bmod 6)$, and that $a+b \equiv 9(\bmod 13)$. Note that $a$ exists by the Chinese Remainder Theorem since $p_{m} \neq 13$ for all $m$. The even case shows that $\{P(a), P(a+1), \ldots, P(a+b-1)\}$ is fragrant. Also, one checks from $13 \mid P(9)=91$ and $13 \mid P(3)=13$ that $P(a+b)$ and $P(a+b-6)$ are divisible by 13. The proof is thus complete. + +N4. Let $n, m, k$ and $l$ be positive integers with $n \neq 1$ such that $n^{k}+m n^{l}+1$ divides $n^{k+l}-1$. Prove that + +- $m=1$ and $l=2 k$; or +- $l \mid k$ and $m=\frac{n^{k-l}-1}{n^{l}-1}$. + +Solution 1. It is given that + +$$ +n^{k}+m n^{l}+1 \mid n^{k+l}-1 +$$ + +This implies + +$$ +n^{k}+m n^{l}+1 \mid\left(n^{k+l}-1\right)+\left(n^{k}+m n^{l}+1\right)=n^{k+l}+n^{k}+m n^{l} . +$$ + +We have two cases to discuss. + +- Case 1. $l \geqslant k$. + +Since $\left(n^{k}+m n^{l}+1, n\right)=1$,(2) yields + +$$ +n^{k}+m n^{l}+1 \mid n^{l}+m n^{l-k}+1 . +$$ + +In particular, we get $n^{k}+m n^{l}+1 \leqslant n^{l}+m n^{l-k}+1$. As $n \geqslant 2$ and $k \geqslant 1,(m-1) n^{l}$ is at least $2(m-1) n^{l-k}$. It follows that the inequality cannot hold when $m \geqslant 2$. For $m=1$, the above divisibility becomes + +$$ +n^{k}+n^{l}+1 \mid n^{l}+n^{l-k}+1 . +$$ + +Note that $n^{l}+n^{l-k}+12 m n^{l}+1>n^{l}+m n^{l-k}+1$, it follows that $n^{k}+m n^{l}+1=n^{l}+m n^{l-k}+1$, that is, + +$$ +m\left(n^{l}-n^{l-k}\right)=n^{l}-n^{k} . +$$ + +If $m \geqslant 2$, then $m\left(n^{l}-n^{l-k}\right) \geqslant 2 n^{l}-2 n^{l-k} \geqslant 2 n^{l}-n^{l}>n^{l}-n^{k}$ gives a contradiction. Hence $m=1$ and $l-k=k$, which means $m=1$ and $l=2 k$. + +- Case 2. $l2 n^{k}+m>n^{k}+n^{k-l}+m$, it follows that $n^{k}+m n^{l}+1=n^{k}+n^{k-l}+m$. This gives $m=\frac{n^{k-l}-1}{n^{l}-1}$. Note that $n^{l}-1 \mid n^{k-l}-1$ implies $l \mid k-l$ and hence $l \mid k$. The proof is thus complete. + +Comment. Another version of this problem is as follows: let $n, m, k$ and $l$ be positive integers with $n \neq 1$ such that $k$ and $l$ do not divide each other. Show that $n^{k}+m n^{l}+1$ does not divide $n^{k+l}-1$. + +N5. Let $a$ be a positive integer which is not a square number. Denote by $A$ the set of all positive integers $k$ such that + +$$ +k=\frac{x^{2}-a}{x^{2}-y^{2}} +$$ + +for some integers $x$ and $y$ with $x>\sqrt{a}$. Denote by $B$ the set of all positive integers $k$ such that (1) is satisfied for some integers $x$ and $y$ with $0 \leqslant x<\sqrt{a}$. Prove that $A=B$. + +Solution 1. We first prove the following preliminary result. + +- Claim. For fixed $k$, let $x, y$ be integers satisfying (1). Then the numbers $x_{1}, y_{1}$ defined by + +$$ +x_{1}=\frac{1}{2}\left(x-y+\frac{(x-y)^{2}-4 a}{x+y}\right), \quad y_{1}=\frac{1}{2}\left(x-y-\frac{(x-y)^{2}-4 a}{x+y}\right) +$$ + +are integers and satisfy (1) (with $x, y$ replaced by $x_{1}, y_{1}$ respectively). +Proof. Since $x_{1}+y_{1}=x-y$ and + +$$ +x_{1}=\frac{x^{2}-x y-2 a}{x+y}=-x+\frac{2\left(x^{2}-a\right)}{x+y}=-x+2 k(x-y), +$$ + +both $x_{1}$ and $y_{1}$ are integers. Let $u=x+y$ and $v=x-y$. The relation (1) can be rewritten as + +$$ +u^{2}-(4 k-2) u v+\left(v^{2}-4 a\right)=0 +$$ + +By Vieta's Theorem, the number $z=\frac{v^{2}-4 a}{u}$ satisfies + +$$ +v^{2}-(4 k-2) v z+\left(z^{2}-4 a\right)=0 +$$ + +Since $x_{1}$ and $y_{1}$ are defined so that $v=x_{1}+y_{1}$ and $z=x_{1}-y_{1}$, we can reverse the process and verify (1) for $x_{1}, y_{1}$. + +We first show that $B \subset A$. Take any $k \in B$ so that (1) is satisfied for some integers $x, y$ with $0 \leqslant x<\sqrt{a}$. Clearly, $y \neq 0$ and we may assume $y$ is positive. Since $a$ is not a square, we have $k>1$. Hence, we get $0 \leqslant x\sqrt{a} +$$ + +This implies $k \in A$ and hence $B \subset A$. + +Next, we shall show that $A \subset B$. Take any $k \in A$ so that (1) is satisfied for some integers $x, y$ with $x>\sqrt{a}$. Again, we may assume $y$ is positive. Among all such representations of $k$, we choose the one with smallest $x+y$. Define + +$$ +x_{1}=\frac{1}{2}\left|x-y+\frac{(x-y)^{2}-4 a}{x+y}\right|, \quad y_{1}=\frac{1}{2}\left(x-y-\frac{(x-y)^{2}-4 a}{x+y}\right) . +$$ + +By the Claim, $x_{1}, y_{1}$ are integers satisfying (1). Since $k>1$, we get $x>y>\sqrt{a}$. Therefore, we have $y_{1}>\frac{4 a}{x+y}>0$ and $\frac{4 a}{x+y}\sqrt{a}$, we get a contradiction due to the minimality of $x+y$. Therefore, we must have $0 \leqslant x_{1}<\sqrt{a}$, which means $k \in B$ so that $A \subset B$. + +The two subset relations combine to give $A=B$. +Solution 2. The relation (1) is equivalent to + +$$ +k y^{2}-(k-1) x^{2}=a +$$ + +Motivated by Pell's Equation, we prove the following, which is essentially the same as the Claim in Solution 1. + +- Claim. If $\left(x_{0}, y_{0}\right)$ is a solution to $(2)$, then $\left((2 k-1) x_{0} \pm 2 k y_{0},(2 k-1) y_{0} \pm 2(k-1) x_{0}\right)$ is also a solution to (2). + +Proof. We check directly that + +$$ +\begin{aligned} +& k\left((2 k-1) y_{0} \pm 2(k-1) x_{0}\right)^{2}-(k-1)\left((2 k-1) x_{0} \pm 2 k y_{0}\right)^{2} \\ += & \left(k(2 k-1)^{2}-(k-1)(2 k)^{2}\right) y_{0}^{2}+\left(k(2(k-1))^{2}-(k-1)(2 k-1)^{2}\right) x_{0}^{2} \\ += & k y_{0}^{2}-(k-1) x_{0}^{2}=a +\end{aligned} +$$ + +If (2) is satisfied for some $0 \leqslant x<\sqrt{a}$ and nonnegative integer $y$, then clearly (1) implies $y>x$. Also, we have $k>1$ since $a$ is not a square number. By the Claim, consider another solution to (2) defined by + +$$ +x_{1}=(2 k-1) x+2 k y, \quad y_{1}=(2 k-1) y+2(k-1) x +$$ + +It satisfies $x_{1} \geqslant(2 k-1) x+2 k(x+1)=(4 k-1) x+2 k>x$. Then we can replace the old solution by a new one which has a larger value in $x$. After a finite number of replacements, we must get a solution with $x>\sqrt{a}$. This shows $B \subset A$. + +If (2) is satisfied for some $x>\sqrt{a}$ and nonnegative integer $y$, by the Claim we consider another solution to (2) defined by + +$$ +x_{1}=|(2 k-1) x-2 k y|, \quad y_{1}=(2 k-1) y-2(k-1) x . +$$ + +From (2), we get $\sqrt{k} y>\sqrt{k-1} x$. This implies $k y>\sqrt{k(k-1)} x>(k-1) x$ and hence $(2 k-1) x-2 k yy$. Then it is clear that $(2 k-1) x-2 k y>-x$. These combine to give $x_{1}|x|$, and, in case of $x>\sqrt{a}$, another solution $\left(x_{2}, y_{2}\right)$ with $\left|x_{2}\right|<|x|$. + +Without loss of generality, assume $x, y \geqslant 0$. Let $u=x+y$ and $v=x-y$. Then $u \geqslant v$ and (1) becomes + +$$ +k=\frac{(u+v)^{2}-4 a}{4 u v} +$$ + +This is the same as + +$$ +v^{2}+(2 u-4 k u) v+u^{2}-4 a=0 +$$ + +Let $v_{1}=4 k u-2 u-v$. Then $u+v_{1}=4 k u-u-v \geqslant 8 u-u-v>u+v$. By Vieta's Theorem, $v_{1}$ satisfies + +$$ +v_{1}^{2}+(2 u-4 k u) v_{1}+u^{2}-4 a=0 +$$ + +This gives $k=\frac{\left(u+v_{1}\right)^{2}-4 a}{4 u v_{1}}$. As $k$ is an integer, $u+v_{1}$ must be even. Therefore, $x_{1}=\frac{u+v_{1}}{2}$ and $y_{1}=\frac{v_{1}-u}{2}$ are integers. By reversing the process, we can see that $\left(x_{1}, y_{1}\right)$ is a solution to (1), with $x_{1}=\frac{u+v_{1}}{2}>\frac{u+v}{2}=x \geqslant 0$. This completes the first half of the proof. + +Suppose $x>\sqrt{a}$. Then $u+v>2 \sqrt{a}$ and (3) can be rewritten as + +$$ +u^{2}+(2 v-4 k v) u+v^{2}-4 a=0 . +$$ + +Let $u_{2}=4 k v-2 v-u$. By Vieta's Theorem, we have $u u_{2}=v^{2}-4 a$ and + +$$ +u_{2}^{2}+(2 v-4 k v) u_{2}+v^{2}-4 a=0 +$$ + +By $u>0, u+v>2 \sqrt{a}$ and (3), we have $v>0$. If $u_{2} \geqslant 0$, then $v u_{2} \leqslant u u_{2}=v^{2}-4 a0$ and $u_{2}+v\nu_{p}\left(A_{1} A_{m+1}^{2}\right)$ for $2 \leqslant m \leqslant k-1$. + +Proof. The case $m=2$ is obvious since $\nu_{p}\left(A_{1} A_{2}^{2}\right) \geqslant p^{t}>\nu_{p}\left(A_{1} A_{3}^{2}\right)$ by the condition and the above assumption. + +Suppose $\nu_{p}\left(A_{1} A_{2}^{2}\right)>\nu_{p}\left(A_{1} A_{3}^{2}\right)>\cdots>\nu_{p}\left(A_{1} A_{m}^{2}\right)$ where $3 \leqslant m \leqslant k-1$. For the induction step, we apply Ptolemy's Theorem to the cyclic quadrilateral $A_{1} A_{m-1} A_{m} A_{m+1}$ to get + +$$ +A_{1} A_{m+1} \times A_{m-1} A_{m}+A_{1} A_{m-1} \times A_{m} A_{m+1}=A_{1} A_{m} \times A_{m-1} A_{m+1} +$$ + +which can be rewritten as + +$$ +\begin{aligned} +A_{1} A_{m+1}^{2} \times A_{m-1} A_{m}^{2}= & A_{1} A_{m-1}^{2} \times A_{m} A_{m+1}^{2}+A_{1} A_{m}^{2} \times A_{m-1} A_{m+1}^{2} \\ +& -2 A_{1} A_{m-1} \times A_{m} A_{m+1} \times A_{1} A_{m} \times A_{m-1} A_{m+1} +\end{aligned} +$$ + +From this, $2 A_{1} A_{m-1} \times A_{m} A_{m+1} \times A_{1} A_{m} \times A_{m-1} A_{m+1}$ is an integer. We consider the component of $p$ of each term in (1). By the inductive hypothesis, we have $\nu_{p}\left(A_{1} A_{m-1}^{2}\right)>\nu_{p}\left(A_{1} A_{m}^{2}\right)$. Also, we have $\nu_{p}\left(A_{m} A_{m+1}^{2}\right) \geqslant p^{t}>\nu_{p}\left(A_{m-1} A_{m+1}^{2}\right)$. These give + +$$ +\nu_{p}\left(A_{1} A_{m-1}^{2} \times A_{m} A_{m+1}^{2}\right)>\nu_{p}\left(A_{1} A_{m}^{2} \times A_{m-1} A_{m+1}^{2}\right) +$$ + +Next, we have $\nu_{p}\left(4 A_{1} A_{m-1}^{2} \times A_{m} A_{m+1}^{2} \times A_{1} A_{m}^{2} \times A_{m-1} A_{m+1}^{2}\right)=\nu_{p}\left(A_{1} A_{m-1}^{2} \times A_{m} A_{m+1}^{2}\right)+$ $\nu_{p}\left(A_{1} A_{m}^{2} \times A_{m-1} A_{m+1}^{2}\right)>2 \nu_{p}\left(A_{1} A_{m}^{2} \times A_{m-1} A_{m+1}^{2}\right)$ from (2). This implies + +$$ +\nu_{p}\left(2 A_{1} A_{m-1} \times A_{m} A_{m+1} \times A_{1} A_{m} \times A_{m-1} A_{m+1}\right)>\nu_{p}\left(A_{1} A_{m}^{2} \times A_{m-1} A_{m+1}^{2}\right) +$$ + +Combining (1), (2) and (3), we conclude that + +$$ +\nu_{p}\left(A_{1} A_{m+1}^{2} \times A_{m-1} A_{m}^{2}\right)=\nu_{p}\left(A_{1} A_{m}^{2} \times A_{m-1} A_{m+1}^{2}\right) +$$ + +By $\nu_{p}\left(A_{m-1} A_{m}^{2}\right) \geqslant p^{t}>\nu_{p}\left(A_{m-1} A_{m+1}^{2}\right)$, we get $\nu_{p}\left(A_{1} A_{m+1}^{2}\right)<\nu_{p}\left(A_{1} A_{m}^{2}\right)$. The Claim follows by induction. + +From the Claim, we get a chain of inequalities + +$$ +p^{t}>\nu_{p}\left(A_{1} A_{3}^{2}\right)>\nu_{p}\left(A_{1} A_{4}^{2}\right)>\cdots>\nu_{p}\left(A_{1} A_{k}^{2}\right) \geqslant p^{t} +$$ + +which yields a contradiction. Therefore, we can show by induction that $2 S$ is divisible by $n$. +Comment. The condition that $P$ is cyclic is crucial. As a counterexample, consider the rhombus with vertices $(0,3),(4,0),(0,-3),(-4,0)$. Each of its squares of side lengths is divisible by 5 , while $2 S=48$ is not. + +The proposer also gives a proof for the case $n$ is even. One just needs an extra technical step for the case $p=2$. + +N8. Find all polynomials $P(x)$ of odd degree $d$ and with integer coefficients satisfying the following property: for each positive integer $n$, there exist $n$ positive integers $x_{1}, x_{2}, \ldots, x_{n}$ such that $\frac{1}{2}<\frac{P\left(x_{i}\right)}{P\left(x_{j}\right)}<2$ and $\frac{P\left(x_{i}\right)}{P\left(x_{j}\right)}$ is the $d$-th power of a rational number for every pair of indices $i$ and $j$ with $1 \leqslant i, j \leqslant n$. + +Answer. $P(x)=a(r x+s)^{d}$ where $a, r, s$ are integers with $a \neq 0, r \geqslant 1$ and $(r, s)=1$. +Solution. Let $P(x)=a_{d} x^{d}+a_{d-1} x^{d-1}+\cdots+a_{0}$. Consider the substitution $y=d a_{d} x+a_{d-1}$. By defining $Q(y)=P(x)$, we find that $Q$ is a polynomial with rational coefficients without the term $y^{d-1}$. Let $Q(y)=b_{d} y^{d}+b_{d-2} y^{d-2}+b_{d-3} y^{d-3}+\cdots+b_{0}$ and $B=\max _{0 \leqslant i \leqslant d}\left\{\left|b_{i}\right|\right\}$ (where $b_{d-1}=0$ ). + +The condition shows that for each $n \geqslant 1$, there exist integers $y_{1}, y_{2}, \ldots, y_{n}$ such that $\frac{1}{2}<\frac{Q\left(y_{i}\right)}{Q\left(y_{j}\right)}<2$ and $\frac{Q\left(y_{i}\right)}{Q\left(y_{j}\right)}$ is the $d$-th power of a rational number for $1 \leqslant i, j \leqslant n$. Since $n$ can be arbitrarily large, we may assume all $x_{i}$ 's and hence $y_{i}$ 's are integers larger than some absolute constant in the following. + +By Dirichlet's Theorem, since $d$ is odd, we can find a sufficiently large prime $p$ such that $p \equiv 2(\bmod d)$. In particular, we have $(p-1, d)=1$. For this fixed $p$, we choose $n$ to be sufficiently large. Then by the Pigeonhole Principle, there must be $d+1$ of $y_{1}, y_{2}, \ldots, y_{n}$ which are congruent $\bmod p$. Without loss of generality, assume $y_{i} \equiv y_{j}(\bmod p)$ for $1 \leqslant i, j \leqslant d+1$. We shall establish the following. + +- Claim. $\frac{Q\left(y_{i}\right)}{Q\left(y_{1}\right)}=\frac{y_{i}^{d}}{y_{1}^{d}}$ for $2 \leqslant i \leqslant d+1$. + +Proof. Let $\frac{Q\left(y_{i}\right)}{Q\left(y_{1}\right)}=\frac{l^{d}}{m^{d}}$ where $(l, m)=1$ and $l, m>0$. This can be rewritten in the expanded form + +$$ +b_{d}\left(m^{d} y_{i}^{d}-l^{d} y_{1}^{d}\right)=-\sum_{j=0}^{d-2} b_{j}\left(m^{d} y_{i}^{j}-l^{d} y_{1}^{j}\right) +$$ + +Let $c$ be the common denominator of $Q$, so that $c Q(k)$ is an integer for any integer $k$. Note that $c$ depends only on $P$ and so we may assume $(p, c)=1$. Then $y_{1} \equiv y_{i}(\bmod p)$ implies $c Q\left(y_{1}\right) \equiv c Q\left(y_{i}\right)(\bmod p)$. + +- Case 1. $p \mid c Q\left(y_{1}\right)$. + +In this case, there is a cancellation of $p$ in the numerator and denominator of $\frac{c Q\left(y_{i}\right)}{c Q\left(y_{1}\right)}$, so that $m^{d} \leqslant p^{-1}\left|c Q\left(y_{1}\right)\right|$. Noting $\left|Q\left(y_{1}\right)\right|<2 B y_{1}^{d}$ as $y_{1}$ is large, we get + +$$ +m \leqslant p^{-\frac{1}{d}}(2 c B)^{\frac{1}{d}} y_{1} +$$ + +For large $y_{1}$ and $y_{i}$, the relation $\frac{1}{2}<\frac{Q\left(y_{i}\right)}{Q\left(y_{1}\right)}<2$ implies + +$$ +\frac{1}{3}<\frac{y_{i}^{d}}{y_{1}^{d}}<3 +$$ + +We also have + +$$ +\frac{1}{2}<\frac{l^{d}}{m^{d}}<2 +$$ + +Now, the left-hand side of (1) is + +$$ +b_{d}\left(m y_{i}-l y_{1}\right)\left(m^{d-1} y_{i}^{d-1}+m^{d-2} y_{i}^{d-2} l y_{1}+\cdots+l^{d-1} y_{1}^{d-1}\right) . +$$ + +Suppose on the contrary that $m y_{i}-l y_{1} \neq 0$. Then the absolute value of the above expression is at least $\left|b_{d}\right| m^{d-1} y_{i}^{d-1}$. On the other hand, the absolute value of the right-hand side of (1) is at most + +$$ +\begin{aligned} +\sum_{j=0}^{d-2} B\left(m^{d} y_{i}^{j}+l^{d} y_{1}^{j}\right) & \leqslant(d-1) B\left(m^{d} y_{i}^{d-2}+l^{d} y_{1}^{d-2}\right) \\ +& \leqslant(d-1) B\left(7 m^{d} y_{i}^{d-2}\right) \\ +& \leqslant 7(d-1) B\left(p^{-\frac{1}{d}}(2 c B)^{\frac{1}{d}} y_{1}\right) m^{d-1} y_{i}^{d-2} \\ +& \leqslant 21(d-1) B p^{-\frac{1}{d}}(2 c B)^{\frac{1}{d}} m^{d-1} y_{i}^{d-1} +\end{aligned} +$$ + +by using successively (3), (4), (2) and again (3). This shows + +$$ +\left|b_{d}\right| m^{d-1} y_{i}^{d-1} \leqslant 21(d-1) B p^{-\frac{1}{d}}(2 c B)^{\frac{1}{d}} m^{d-1} y_{i}^{d-1}, +$$ + +which is a contradiction for large $p$ as $b_{d}, B, c, d$ depend only on the polynomial $P$. Therefore, we have $m y_{i}-l y_{1}=0$ in this case. + +- Case 2. $\left(p, c Q\left(y_{1}\right)\right)=1$. + +From $c Q\left(y_{1}\right) \equiv c Q\left(y_{i}\right)(\bmod p)$, we have $l^{d} \equiv m^{d}(\bmod p)$. Since $(p-1, d)=1$, we use Fermat Little Theorem to conclude $l \equiv m(\bmod p)$. Then $p \mid m y_{i}-l y_{1}$. Suppose on the contrary that $m y_{i}-l y_{1} \neq 0$. Then the left-hand side of (1) has absolute value at least $\left|b_{d}\right| p m^{d-1} y_{i}^{d-1}$. Similar to Case 1, the right-hand side of (1) has absolute value at most + +$$ +21(d-1) B(2 c B)^{\frac{1}{d}} m^{d-1} y_{i}^{d-1} +$$ + +which must be smaller than $\left|b_{d}\right| p m^{d-1} y_{i}^{d-1}$ for large $p$. Again this yields a contradiction and hence $m y_{i}-l y_{1}=0$. + +In both cases, we find that $\frac{Q\left(y_{i}\right)}{Q\left(y_{1}\right)}=\frac{l^{d}}{m^{d}}=\frac{y_{i}^{d}}{y_{1}^{d}}$. +From the Claim, the polynomial $Q\left(y_{1}\right) y^{d}-y_{1}^{d} Q(y)$ has roots $y=y_{1}, y_{2}, \ldots, y_{d+1}$. Since its degree is at most $d$, this must be the zero polynomial. Hence, $Q(y)=b_{d} y^{d}$. This implies $P(x)=a_{d}\left(x+\frac{a_{d-1}}{d a_{d}}\right)^{d}$. Let $\frac{a_{d-1}}{d a_{d}}=\frac{s}{r}$ with integers $r, s$ where $r \geqslant 1$ and $(r, s)=1$. Since $P$ has integer coefficients, we need $r^{d} \mid a_{d}$. Let $a_{d}=r^{d} a$. Then $P(x)=a(r x+s)^{d}$. It is obvious that such a polynomial satisfies the conditions. + +Comment. In the proof, the use of prime and Dirichlet's Theorem can be avoided. One can easily show that each $P\left(x_{i}\right)$ can be expressed in the form $u v_{i}^{d}$ where $u, v_{i}$ are integers and $u$ cannot be divisible by the $d$-th power of a prime (note that $u$ depends only on $P$ ). By fixing a large integer $q$ and by choosing a large $n$, we can apply the Pigeonhole Principle and assume +$x_{1} \equiv x_{2} \equiv \cdots \equiv x_{d+1}(\bmod q)$ and $v_{1} \equiv v_{2} \equiv \cdots \equiv v_{d+1}(\bmod q)$. Then the remaining proof is similar to Case 2 of the Solution. + +Alternatively, we give another modification of the proof as follows. +We take a sufficiently large $n$ and consider the corresponding positive integers $y_{1}, y_{2}, \ldots, y_{n}$. For each $2 \leqslant i \leqslant n$, let $\frac{Q\left(y_{i}\right)}{Q\left(y_{1}\right)}=\frac{l_{i}^{d}}{m_{i}^{d}}$. + +As in Case 1, if there are $d$ indices $i$ such that the integers $\frac{c\left|Q\left(y_{1}\right)\right|}{m_{i}^{d}}$ are bounded below by a constant depending only on $P$, we can establish the Claim using those $y_{i}$ 's and complete the proof. Similarly, as in Case 2, if there are $d$ indices $i$ such that the integers $\left|m_{i} y_{i}-l_{i} y_{1}\right|$ are bounded below, then the proof goes the same. So it suffices to consider the case where $\frac{c\left|Q\left(y_{1}\right)\right|}{m_{i}^{d}} \leqslant M$ and $\left|m_{i} y_{i}-l_{i} y_{1}\right| \leqslant N$ for all $2 \leqslant i \leqslant n^{\prime}$ where $M, N$ are fixed constants and $n^{\prime}$ is large. Since there are only finitely many choices for $m_{i}$ and $m_{i} y_{i}-l_{i} y_{1}$, by the Pigeonhole Principle, we can assume without loss of generality $m_{i}=m$ and $m_{i} y_{i}-l_{i} y_{1}=t$ for $2 \leqslant i \leqslant d+2$. Then + +$$ +\frac{Q\left(y_{i}\right)}{Q\left(y_{1}\right)}=\frac{l_{i}^{d}}{m^{d}}=\frac{\left(m y_{i}-t\right)^{d}}{m^{d} y_{1}^{d}} +$$ + +so that $Q\left(y_{1}\right)(m y-t)^{d}-m^{d} y_{1}^{d} Q(y)$ has roots $y=y_{2}, y_{3}, \ldots, y_{d+2}$. Its degree is at most $d$ and hence it is the zero polynomial. Therefore, $Q(y)=\frac{b_{d}}{m^{d}}(m y-t)^{d}$. Indeed, $Q$ does not have the term $y^{d-1}$, which means $t$ should be 0 . This gives the corresponding $P(x)$ of the desired form. + +The two modifications of the Solution work equally well when the degree $d$ is even. + diff --git a/IMO/md/en-IMO2017SL.md b/IMO/md/en-IMO2017SL.md new file mode 100644 index 0000000000000000000000000000000000000000..672a0a9eb08ecc7bc3e4aa25377096c9154798d2 --- /dev/null +++ b/IMO/md/en-IMO2017SL.md @@ -0,0 +1,2968 @@ +![](https://cdn.mathpix.com/cropped/2024_11_18_ff08e3c077fa2f8c16b2g-01.jpg?height=326&width=572&top_left_y=294&top_left_x=468) + +# ImO2017
RIO DE JANEIRO - BRAZIL + +58 ${ }^{\text {th }}$ International Mathematical Olympiad + +## Shortlisted Problems (with solutions) + +![](https://cdn.mathpix.com/cropped/2024_11_18_ff08e3c077fa2f8c16b2g-03.jpg?height=321&width=569&top_left_y=299&top_left_x=464) + +## Shortlisted Problems (with solutions) + +$58^{\text {th }}$ International Mathematical Olympiad Rio de Janeiro, 12-23 July 2017 + +## The Shortlist has to be kept strictly confidential until the conclusion of the following International Mathematical Olympiad. IMO General Regulations §6.6 + +## Contributing Countries + +The Organizing Committee and the Problem Selection Committee of IMO 2017 thank the following 51 countries for contributing 150 problem proposals: + +Albania, Algeria, Armenia, Australia, Austria, Azerbaijan, Belarus, Belgium, Bulgaria, Cuba, Cyprus, Czech Republic, Denmark, Estonia, France, Georgia, Germany, Greece, Hong Kong, India, Iran, Ireland, Israel, Italy, Japan, Kazakhstan, Latvia, Lithuania, Luxembourg, Mexico, Montenegro, Morocco, Netherlands, Romania, Russia, Serbia, Singapore, Slovakia, Slovenia, South Africa, Sweden, Switzerland, Taiwan, Tajikistan, Tanzania, Thailand, Trinidad and Tobago, Turkey, Ukraine, United Kingdom, U.S.A. + +## Problem Selection Committee + +![](https://cdn.mathpix.com/cropped/2024_11_18_ff08e3c077fa2f8c16b2g-05.jpg?height=697&width=1157&top_left_y=1873&top_left_x=455) + +Carlos Gustavo Tamm de Araújo Moreira (Gugu) (chairman), Luciano Monteiro de Castro, Ilya I. Bogdanov, Géza Kós, Carlos Yuzo Shine, Zhuo Qun (Alex) Song, Ralph Costa Teixeira, Eduardo Tengan + +## Problems + +## Algebra + +A1. Let $a_{1}, a_{2}, \ldots, a_{n}, k$, and $M$ be positive integers such that + +$$ +\frac{1}{a_{1}}+\frac{1}{a_{2}}+\cdots+\frac{1}{a_{n}}=k \quad \text { and } \quad a_{1} a_{2} \ldots a_{n}=M +$$ + +If $M>1$, prove that the polynomial + +$$ +P(x)=M(x+1)^{k}-\left(x+a_{1}\right)\left(x+a_{2}\right) \cdots\left(x+a_{n}\right) +$$ + +has no positive roots. +(Trinidad and Tobago) +A2. Let $q$ be a real number. Gugu has a napkin with ten distinct real numbers written on it, and he writes the following three lines of real numbers on the blackboard: + +- In the first line, Gugu writes down every number of the form $a-b$, where $a$ and $b$ are two (not necessarily distinct) numbers on his napkin. +- In the second line, Gugu writes down every number of the form $q a b$, where $a$ and $b$ are two (not necessarily distinct) numbers from the first line. +- In the third line, Gugu writes down every number of the form $a^{2}+b^{2}-c^{2}-d^{2}$, where $a, b, c, d$ are four (not necessarily distinct) numbers from the first line. + +Determine all values of $q$ such that, regardless of the numbers on Gugu's napkin, every number in the second line is also a number in the third line. + +A3. Let $S$ be a finite set, and let $\mathcal{A}$ be the set of all functions from $S$ to $S$. Let $f$ be an element of $\mathcal{A}$, and let $T=f(S)$ be the image of $S$ under $f$. Suppose that $f \circ g \circ f \neq g \circ f \circ g$ for every $g$ in $\mathcal{A}$ with $g \neq f$. Show that $f(T)=T$. +(India) +A4. A sequence of real numbers $a_{1}, a_{2}, \ldots$ satisfies the relation + +$$ +a_{n}=-\max _{i+j=n}\left(a_{i}+a_{j}\right) \quad \text { for all } n>2017 +$$ + +Prove that this sequence is bounded, i.e., there is a constant $M$ such that $\left|a_{n}\right| \leqslant M$ for all positive integers $n$. + +A5. An integer $n \geqslant 3$ is given. We call an $n$-tuple of real numbers $\left(x_{1}, x_{2}, \ldots, x_{n}\right)$ Shiny if for each permutation $y_{1}, y_{2}, \ldots, y_{n}$ of these numbers we have + +$$ +\sum_{i=1}^{n-1} y_{i} y_{i+1}=y_{1} y_{2}+y_{2} y_{3}+y_{3} y_{4}+\cdots+y_{n-1} y_{n} \geqslant-1 +$$ + +Find the largest constant $K=K(n)$ such that + +$$ +\sum_{1 \leqslant i1 +\end{array} \quad \text { for } n=1,2, \ldots\right. +$$ + +Prove that at least one of the two numbers $a_{2017}$ and $a_{2018}$ must be greater than or equal to 2017 . +(Australia) +A8. Assume that a function $f: \mathbb{R} \rightarrow \mathbb{R}$ satisfies the following condition: +For every $x, y \in \mathbb{R}$ such that $(f(x)+y)(f(y)+x)>0$, we have $f(x)+y=f(y)+x$. +Prove that $f(x)+y \leqslant f(y)+x$ whenever $x>y$. +(Netherlands) + +## Combinatorics + +C1. A rectangle $\mathcal{R}$ with odd integer side lengths is divided into small rectangles with integer side lengths. Prove that there is at least one among the small rectangles whose distances from the four sides of $\mathcal{R}$ are either all odd or all even. +(Singapore) +C2. Let $n$ be a positive integer. Define a chameleon to be any sequence of $3 n$ letters, with exactly $n$ occurrences of each of the letters $a, b$, and $c$. Define a swap to be the transposition of two adjacent letters in a chameleon. Prove that for any chameleon $X$, there exists a chameleon $Y$ such that $X$ cannot be changed to $Y$ using fewer than $3 n^{2} / 2$ swaps. +(Australia) +C3. Sir Alex plays the following game on a row of 9 cells. Initially, all cells are empty. In each move, Sir Alex is allowed to perform exactly one of the following two operations: +(1) Choose any number of the form $2^{j}$, where $j$ is a non-negative integer, and put it into an empty cell. +(2) Choose two (not necessarily adjacent) cells with the same number in them; denote that number by $2^{j}$. Replace the number in one of the cells with $2^{j+1}$ and erase the number in the other cell. + +At the end of the game, one cell contains the number $2^{n}$, where $n$ is a given positive integer, while the other cells are empty. Determine the maximum number of moves that Sir Alex could have made, in terms of $n$. +(Thailand) +C4. Let $N \geqslant 2$ be an integer. $N(N+1)$ soccer players, no two of the same height, stand in a row in some order. Coach Ralph wants to remove $N(N-1)$ people from this row so that in the remaining row of $2 N$ players, no one stands between the two tallest ones, no one stands between the third and the fourth tallest ones, ..., and finally no one stands between the two shortest ones. Show that this is always possible. +(Russia) +C5. A hunter and an invisible rabbit play a game in the Euclidean plane. The hunter's starting point $H_{0}$ coincides with the rabbit's starting point $R_{0}$. In the $n^{\text {th }}$ round of the game $(n \geqslant 1)$, the following happens. +(1) First the invisible rabbit moves secretly and unobserved from its current point $R_{n-1}$ to some new point $R_{n}$ with $R_{n-1} R_{n}=1$. +(2) The hunter has a tracking device (e.g. dog) that returns an approximate position $R_{n}^{\prime}$ of the rabbit, so that $R_{n} R_{n}^{\prime} \leqslant 1$. +(3) The hunter then visibly moves from point $H_{n-1}$ to a new point $H_{n}$ with $H_{n-1} H_{n}=1$. + +Is there a strategy for the hunter that guarantees that after $10^{9}$ such rounds the distance between the hunter and the rabbit is below 100 ? + +C6. Let $n>1$ be an integer. An $n \times n \times n$ cube is composed of $n^{3}$ unit cubes. Each unit cube is painted with one color. For each $n \times n \times 1$ box consisting of $n^{2}$ unit cubes (of any of the three possible orientations), we consider the set of the colors present in that box (each color is listed only once). This way, we get $3 n$ sets of colors, split into three groups according to the orientation. It happens that for every set in any group, the same set appears in both of the other groups. Determine, in terms of $n$, the maximal possible number of colors that are present. +(Russia) +C7. For any finite sets $X$ and $Y$ of positive integers, denote by $f_{X}(k)$ the $k^{\text {th }}$ smallest positive integer not in $X$, and let + +$$ +X * Y=X \cup\left\{f_{X}(y): y \in Y\right\} +$$ + +Let $A$ be a set of $a>0$ positive integers, and let $B$ be a set of $b>0$ positive integers. Prove that if $A * B=B * A$, then + +$$ +\underbrace{A *(A * \cdots *(A *(A * A)) \ldots)}_{A \text { appears } b \text { times }}=\underbrace{B *(B * \cdots *(B *(B * B)) \ldots)}_{B \text { appears } a \text { times }} . +$$ + +C8. Let $n$ be a given positive integer. In the Cartesian plane, each lattice point with nonnegative coordinates initially contains a butterfly, and there are no other butterflies. The neighborhood of a lattice point $c$ consists of all lattice points within the axis-aligned $(2 n+1) \times(2 n+1)$ square centered at $c$, apart from $c$ itself. We call a butterfly lonely, crowded, or comfortable, depending on whether the number of butterflies in its neighborhood $N$ is respectively less than, greater than, or equal to half of the number of lattice points in $N$. + +Every minute, all lonely butterflies fly away simultaneously. This process goes on for as long as there are any lonely butterflies. Assuming that the process eventually stops, determine the number of comfortable butterflies at the final state. + +## Geometry + +G1. Let $A B C D E$ be a convex pentagon such that $A B=B C=C D, \angle E A B=\angle B C D$, and $\angle E D C=\angle C B A$. Prove that the perpendicular line from $E$ to $B C$ and the line segments $A C$ and $B D$ are concurrent. +(Italy) +G2. Let $R$ and $S$ be distinct points on circle $\Omega$, and let $t$ denote the tangent line to $\Omega$ at $R$. Point $R^{\prime}$ is the reflection of $R$ with respect to $S$. A point $I$ is chosen on the smaller arc $R S$ of $\Omega$ so that the circumcircle $\Gamma$ of triangle $I S R^{\prime}$ intersects $t$ at two different points. Denote by $A$ the common point of $\Gamma$ and $t$ that is closest to $R$. Line $A I$ meets $\Omega$ again at $J$. Show that $J R^{\prime}$ is tangent to $\Gamma$. +(Luxembourg) +G3. Let $O$ be the circumcenter of an acute scalene triangle $A B C$. Line $O A$ intersects the altitudes of $A B C$ through $B$ and $C$ at $P$ and $Q$, respectively. The altitudes meet at $H$. Prove that the circumcenter of triangle $P Q H$ lies on a median of triangle $A B C$. +(Ukraine) +G4. In triangle $A B C$, let $\omega$ be the excircle opposite $A$. Let $D, E$, and $F$ be the points where $\omega$ is tangent to lines $B C, C A$, and $A B$, respectively. The circle $A E F$ intersects line $B C$ at $P$ and $Q$. Let $M$ be the midpoint of $A D$. Prove that the circle $M P Q$ is tangent to $\omega$. +(Denmark) +G5. Let $A B C C_{1} B_{1} A_{1}$ be a convex hexagon such that $A B=B C$, and suppose that the line segments $A A_{1}, B B_{1}$, and $C C_{1}$ have the same perpendicular bisector. Let the diagonals $A C_{1}$ and $A_{1} C$ meet at $D$, and denote by $\omega$ the circle $A B C$. Let $\omega$ intersect the circle $A_{1} B C_{1}$ again at $E \neq B$. Prove that the lines $B B_{1}$ and $D E$ intersect on $\omega$. +(Ukraine) +G6. Let $n \geqslant 3$ be an integer. Two regular $n$-gons $\mathcal{A}$ and $\mathcal{B}$ are given in the plane. Prove that the vertices of $\mathcal{A}$ that lie inside $\mathcal{B}$ or on its boundary are consecutive. +(That is, prove that there exists a line separating those vertices of $\mathcal{A}$ that lie inside $\mathcal{B}$ or on its boundary from the other vertices of $\mathcal{A}$.) +(Czech Republic) +G7. A convex quadrilateral $A B C D$ has an inscribed circle with center $I$. Let $I_{a}, I_{b}, I_{c}$, and $I_{d}$ be the incenters of the triangles $D A B, A B C, B C D$, and $C D A$, respectively. Suppose that the common external tangents of the circles $A I_{b} I_{d}$ and $C I_{b} I_{d}$ meet at $X$, and the common external tangents of the circles $B I_{a} I_{c}$ and $D I_{a} I_{c}$ meet at $Y$. Prove that $\angle X I Y=90^{\circ}$. +(Kazakhstan) +G8. There are 2017 mutually external circles drawn on a blackboard, such that no two are tangent and no three share a common tangent. A tangent segment is a line segment that is a common tangent to two circles, starting at one tangent point and ending at the other one. Luciano is drawing tangent segments on the blackboard, one at a time, so that no tangent segment intersects any other circles or previously drawn tangent segments. Luciano keeps drawing tangent segments until no more can be drawn. Find all possible numbers of tangent segments when he stops drawing. +(Australia) + +## Number Theory + +N1. The sequence $a_{0}, a_{1}, a_{2}, \ldots$ of positive integers satisfies + +$$ +a_{n+1}=\left\{\begin{array}{ll} +\sqrt{a_{n}}, & \text { if } \sqrt{a_{n}} \text { is an integer } \\ +a_{n}+3, & \text { otherwise } +\end{array} \quad \text { for every } n \geqslant 0\right. +$$ + +Determine all values of $a_{0}>1$ for which there is at least one number $a$ such that $a_{n}=a$ for infinitely many values of $n$. +(South Africa) +N2. Let $p \geqslant 2$ be a prime number. Eduardo and Fernando play the following game making moves alternately: in each move, the current player chooses an index $i$ in the set $\{0,1, \ldots, p-1\}$ that was not chosen before by either of the two players and then chooses an element $a_{i}$ of the set $\{0,1,2,3,4,5,6,7,8,9\}$. Eduardo has the first move. The game ends after all the indices $i \in\{0,1, \ldots, p-1\}$ have been chosen. Then the following number is computed: + +$$ +M=a_{0}+10 \cdot a_{1}+\cdots+10^{p-1} \cdot a_{p-1}=\sum_{j=0}^{p-1} a_{j} \cdot 10^{j} +$$ + +The goal of Eduardo is to make the number $M$ divisible by $p$, and the goal of Fernando is to prevent this. + +Prove that Eduardo has a winning strategy. +(Morocco) +N3. Determine all integers $n \geqslant 2$ with the following property: for any integers $a_{1}, a_{2}, \ldots, a_{n}$ whose sum is not divisible by $n$, there exists an index $1 \leqslant i \leqslant n$ such that none of the numbers + +$$ +a_{i}, a_{i}+a_{i+1}, \ldots, a_{i}+a_{i+1}+\cdots+a_{i+n-1} +$$ + +is divisible by $n$. (We let $a_{i}=a_{i-n}$ when $i>n$.) +(Thailand) +N4. Call a rational number short if it has finitely many digits in its decimal expansion. For a positive integer $m$, we say that a positive integer $t$ is $m$-tastic if there exists a number $c \in\{1,2,3, \ldots, 2017\}$ such that $\frac{10^{t}-1}{c \cdot m}$ is short, and such that $\frac{10^{k}-1}{c \cdot m}$ is not short for any $1 \leqslant kq$ for which the number + +$$ +\frac{(p+q)^{p+q}(p-q)^{p-q}-1}{(p+q)^{p-q}(p-q)^{p+q}-1} +$$ + +is an integer. + +N6. Find the smallest positive integer $n$, or show that no such $n$ exists, with the following property: there are infinitely many distinct $n$-tuples of positive rational numbers ( $a_{1}, a_{2}, \ldots, a_{n}$ ) such that both + +$$ +a_{1}+a_{2}+\cdots+a_{n} \quad \text { and } \quad \frac{1}{a_{1}}+\frac{1}{a_{2}}+\cdots+\frac{1}{a_{n}} +$$ + +are integers. +(Singapore) +N7. Say that an ordered pair $(x, y)$ of integers is an irreducible lattice point if $x$ and $y$ are relatively prime. For any finite set $S$ of irreducible lattice points, show that there is a homogenous polynomial in two variables, $f(x, y)$, with integer coefficients, of degree at least 1 , such that $f(x, y)=1$ for each $(x, y)$ in the set $S$. + +Note: A homogenous polynomial of degree $n$ is any nonzero polynomial of the form + +$$ +f(x, y)=a_{0} x^{n}+a_{1} x^{n-1} y+a_{2} x^{n-2} y^{2}+\cdots+a_{n-1} x y^{n-1}+a_{n} y^{n} . +$$ + +N8 Let $p$ be an odd prime number and $\mathbb{Z}_{>0}$ be the set of positive integers. Suppose that a function $f: \mathbb{Z}_{>0} \times \mathbb{Z}_{>0} \rightarrow\{0,1\}$ satisfies the following properties: + +- $f(1,1)=0$; +- $f(a, b)+f(b, a)=1$ for any pair of relatively prime positive integers $(a, b)$ not both equal to 1 ; +- $f(a+b, b)=f(a, b)$ for any pair of relatively prime positive integers $(a, b)$. + +Prove that + +$$ +\sum_{n=1}^{p-1} f\left(n^{2}, p\right) \geqslant \sqrt{2 p}-2 +$$ + +## Solutions + +## Algebra + +A1. Let $a_{1}, a_{2}, \ldots, a_{n}, k$, and $M$ be positive integers such that + +$$ +\frac{1}{a_{1}}+\frac{1}{a_{2}}+\cdots+\frac{1}{a_{n}}=k \quad \text { and } \quad a_{1} a_{2} \ldots a_{n}=M +$$ + +If $M>1$, prove that the polynomial + +$$ +P(x)=M(x+1)^{k}-\left(x+a_{1}\right)\left(x+a_{2}\right) \cdots\left(x+a_{n}\right) +$$ + +has no positive roots. +(Trinidad and Tobago) +Solution 1. We first prove that, for $x>0$, + +$$ +a_{i}(x+1)^{1 / a_{i}} \leqslant x+a_{i}, +$$ + +with equality if and only if $a_{i}=1$. It is clear that equality occurs if $a_{i}=1$. +If $a_{i}>1$, the AM-GM inequality applied to a single copy of $x+1$ and $a_{i}-1$ copies of 1 yields + +$$ +\frac{(x+1)+\overbrace{1+1+\cdots+1}^{a_{i}-1 \text { ones }}}{a_{i}} \geqslant \sqrt[a_{i}]{(x+1) \cdot 1^{a_{i}-1}} \Longrightarrow a_{i}(x+1)^{1 / a_{i}} \leqslant x+a_{i} +$$ + +Since $x+1>1$, the inequality is strict for $a_{i}>1$. +Multiplying the inequalities (1) for $i=1,2, \ldots, n$ yields + +$$ +\prod_{i=1}^{n} a_{i}(x+1)^{1 / a_{i}} \leqslant \prod_{i=1}^{n}\left(x+a_{i}\right) \Longleftrightarrow M(x+1)^{\sum_{i=1}^{n} 1 / a_{i}}-\prod_{i=1}^{n}\left(x+a_{i}\right) \leqslant 0 \Longleftrightarrow P(x) \leqslant 0 +$$ + +with equality iff $a_{i}=1$ for all $i \in\{1,2, \ldots, n\}$. But this implies $M=1$, which is not possible. Hence $P(x)<0$ for all $x \in \mathbb{R}^{+}$, and $P$ has no positive roots. + +Comment 1. Inequality (1) can be obtained in several ways. For instance, we may also use the binomial theorem: since $a_{i} \geqslant 1$, + +$$ +\left(1+\frac{x}{a_{i}}\right)^{a_{i}}=\sum_{j=0}^{a_{i}}\binom{a_{i}}{j}\left(\frac{x}{a_{i}}\right)^{j} \geqslant\binom{ a_{i}}{0}+\binom{a_{i}}{1} \cdot \frac{x}{a_{i}}=1+x +$$ + +Both proofs of (1) mimic proofs to Bernoulli's inequality for a positive integer exponent $a_{i}$; we can use this inequality directly: + +$$ +\left(1+\frac{x}{a_{i}}\right)^{a_{i}} \geqslant 1+a_{i} \cdot \frac{x}{a_{i}}=1+x +$$ + +and so + +$$ +x+a_{i}=a_{i}\left(1+\frac{x}{a_{i}}\right) \geqslant a_{i}(1+x)^{1 / a_{i}} +$$ + +or its (reversed) formulation, with exponent $1 / a_{i} \leqslant 1$ : + +$$ +(1+x)^{1 / a_{i}} \leqslant 1+\frac{1}{a_{i}} \cdot x=\frac{x+a_{i}}{a_{i}} \Longrightarrow a_{i}(1+x)^{1 / a_{i}} \leqslant x+a_{i} . +$$ + +Solution 2. We will prove that, in fact, all coefficients of the polynomial $P(x)$ are non-positive, and at least one of them is negative, which implies that $P(x)<0$ for $x>0$. + +Indeed, since $a_{j} \geqslant 1$ for all $j$ and $a_{j}>1$ for some $j$ (since $a_{1} a_{2} \ldots a_{n}=M>1$ ), we have $k=\frac{1}{a_{1}}+\frac{1}{a_{2}}+\cdots+\frac{1}{a_{n}}1$, if (2) is true for a given $r-1$ and $x \neq 0$. + +A2. Let $q$ be a real number. Gugu has a napkin with ten distinct real numbers written on it, and he writes the following three lines of real numbers on the blackboard: + +- In the first line, Gugu writes down every number of the form $a-b$, where $a$ and $b$ are two (not necessarily distinct) numbers on his napkin. +- In the second line, Gugu writes down every number of the form qab, where $a$ and $b$ are two (not necessarily distinct) numbers from the first line. +- In the third line, Gugu writes down every number of the form $a^{2}+b^{2}-c^{2}-d^{2}$, where $a, b, c, d$ are four (not necessarily distinct) numbers from the first line. + +Determine all values of $q$ such that, regardless of the numbers on Gugu's napkin, every number in the second line is also a number in the third line. +(Austria) +Answer: -2, 0,2 . +Solution 1. Call a number $q$ good if every number in the second line appears in the third line unconditionally. We first show that the numbers 0 and $\pm 2$ are good. The third line necessarily contains 0 , so 0 is good. For any two numbers $a, b$ in the first line, write $a=x-y$ and $b=u-v$, where $x, y, u, v$ are (not necessarily distinct) numbers on the napkin. We may now write + +$$ +2 a b=2(x-y)(u-v)=(x-v)^{2}+(y-u)^{2}-(x-u)^{2}-(y-v)^{2} +$$ + +which shows that 2 is good. By negating both sides of the above equation, we also see that -2 is good. + +We now show that $-2,0$, and 2 are the only good numbers. Assume for sake of contradiction that $q$ is a good number, where $q \notin\{-2,0,2\}$. We now consider some particular choices of numbers on Gugu's napkin to arrive at a contradiction. + +Assume that the napkin contains the integers $1,2, \ldots, 10$. Then, the first line contains the integers $-9,-8, \ldots, 9$. The second line then contains $q$ and $81 q$, so the third line must also contain both of them. But the third line only contains integers, so $q$ must be an integer. Furthermore, the third line contains no number greater than $162=9^{2}+9^{2}-0^{2}-0^{2}$ or less than -162 , so we must have $-162 \leqslant 81 q \leqslant 162$. This shows that the only possibilities for $q$ are $\pm 1$. + +Now assume that $q= \pm 1$. Let the napkin contain $0,1,4,8,12,16,20,24,28,32$. The first line contains $\pm 1$ and $\pm 4$, so the second line contains $\pm 4$. However, for every number $a$ in the first line, $a \not \equiv 2(\bmod 4)$, so we may conclude that $a^{2} \equiv 0,1(\bmod 8)$. Consequently, every number in the third line must be congruent to $-2,-1,0,1,2(\bmod 8)$; in particular, $\pm 4$ cannot be in the third line, which is a contradiction. + +Solution 2. Let $q$ be a good number, as defined in the first solution, and define the polynomial $P\left(x_{1}, \ldots, x_{10}\right)$ as + +$$ +\prod_{i2017 +$$ + +Prove that this sequence is bounded, i.e., there is a constant $M$ such that $\left|a_{n}\right| \leqslant M$ for all positive integers $n$. +(Russia) +Solution 1. Set $D=2017$. Denote + +$$ +M_{n}=\max _{kD$; our first aim is to bound $a_{n}$ in terms of $m_{n}$ and $M_{n}$. +(i) There exist indices $p$ and $q$ such that $a_{n}=-\left(a_{p}+a_{q}\right)$ and $p+q=n$. Since $a_{p}, a_{q} \leqslant M_{n}$, we have $a_{n} \geqslant-2 M_{n}$. +(ii) On the other hand, choose an index $kD$ is lucky if $m_{n} \leqslant 2 M_{n}$. Two cases are possible. +Case 1. Assume that there exists a lucky index $n$. In this case, (1) yields $m_{n+1} \leqslant 2 M_{n}$ and $M_{n} \leqslant M_{n+1} \leqslant M_{n}$. Therefore, $M_{n+1}=M_{n}$ and $m_{n+1} \leqslant 2 M_{n}=2 M_{n+1}$. So, the index $n+1$ is also lucky, and $M_{n+1}=M_{n}$. Applying the same arguments repeatedly, we obtain that all indices $k>n$ are lucky (i.e., $m_{k} \leqslant 2 M_{k}$ for all these indices), and $M_{k}=M_{n}$ for all such indices. Thus, all of the $m_{k}$ and $M_{k}$ are bounded by $2 M_{n}$. +Case 2. Assume now that there is no lucky index, i.e., $2 M_{n}D$. Then (1) shows that for all $n>D$ we have $m_{n} \leqslant m_{n+1} \leqslant m_{n}$, so $m_{n}=m_{D+1}$ for all $n>D$. Since $M_{n}a_{i} \text { for each } i-2 \ell, \quad \text { and } \quad n>D +$$ + +We first show that there must be some good index $n$. By assumption, we may take an index $N$ such that $a_{N}>\max \{L,-2 \ell\}$. Choose $n$ minimally such that $a_{n}=\max \left\{a_{1}, a_{2}, \ldots, a_{N}\right\}$. Now, the first condition in (2) is satisfied because of the minimality of $n$, and the second and third conditions are satisfied because $a_{n} \geqslant a_{N}>L,-2 \ell$, and $L \geqslant a_{i}$ for every $i$ such that $1 \leqslant i \leqslant D$. + +Let $n$ be a good index. We derive a contradiction. We have that + +$$ +a_{n}+a_{u}+a_{v} \leqslant 0 +$$ + +whenever $u+v=n$. +We define the index $u$ to maximize $a_{u}$ over $1 \leqslant u \leqslant n-1$, and let $v=n-u$. Then, we note that $a_{u} \geqslant a_{v}$ by the maximality of $a_{u}$. + +Assume first that $v \leqslant D$. Then, we have that + +$$ +a_{N}+2 \ell \leqslant 0, +$$ + +because $a_{u} \geqslant a_{v} \geqslant \ell$. But this contradicts our assumption that $a_{n}>-2 \ell$ in the second criteria of (2). + +Now assume that $v>D$. Then, there exist some indices $w_{1}, w_{2}$ summing up to $v$ such that + +$$ +a_{v}+a_{w_{1}}+a_{w_{2}}=0 +$$ + +But combining this with (3), we have + +$$ +a_{n}+a_{u} \leqslant a_{w_{1}}+a_{w_{2}} +$$ + +Because $a_{n}>a_{u}$, we have that $\max \left\{a_{w_{1}}, a_{w_{2}}\right\}>a_{u}$. But since each of the $w_{i}$ is less than $v$, this contradicts the maximality of $a_{u}$. + +Comment 1. We present two harder versions of this problem below. +Version 1. Let $a_{1}, a_{2}, \ldots$ be a sequence of numbers that satisfies the relation + +$$ +a_{n}=-\max _{i+j+k=n}\left(a_{i}+a_{j}+a_{k}\right) \quad \text { for all } n>2017 +$$ + +Then, this sequence is bounded. +Proof. Set $D=2017$. Denote + +$$ +M_{n}=\max _{k2 D$; our first aim is to bound $a_{n}$ in terms of $m_{i}$ and $M_{i}$. Set $k=\lfloor n / 2\rfloor$. +(i) Choose indices $p, q$, and $r$ such that $a_{n}=-\left(a_{p}+a_{q}+a_{r}\right)$ and $p+q+r=n$. Without loss of generality, $p \geqslant q \geqslant r$. + +Assume that $p \geqslant k+1(>D)$; then $p>q+r$. Hence + +$$ +-a_{p}=\max _{i_{1}+i_{2}+i_{3}=p}\left(a_{i_{1}}+a_{i_{2}}+a_{i_{3}}\right) \geqslant a_{q}+a_{r}+a_{p-q-r} +$$ + +and therefore $a_{n}=-\left(a_{p}+a_{q}+a_{r}\right) \geqslant\left(a_{q}+a_{r}+a_{p-q-r}\right)-a_{q}-a_{r}=a_{p-q-r} \geqslant-m_{n}$. +Otherwise, we have $k \geqslant p \geqslant q \geqslant r$. Since $n<3 k$, we have $r2 D$ is lucky if $m_{n} \leqslant 2 M_{\lfloor n / 2\rfloor+1}+M_{\lfloor n / 2]}$. Two cases are possible. +Case 1. Assume that there exists a lucky index $n$; set $k=\lfloor n / 2\rfloor$. In this case, (4) yields $m_{n+1} \leqslant$ $2 M_{k+1}+M_{k}$ and $M_{n} \leqslant M_{n+1} \leqslant M_{n}$ (the last relation holds, since $m_{n}-M_{k+1}-M_{k} \leqslant\left(2 M_{k+1}+\right.$ $\left.M_{k}\right)-M_{k+1}-M_{k}=M_{k+1} \leqslant M_{n}$ ). Therefore, $M_{n+1}=M_{n}$ and $m_{n+1} \leqslant 2 M_{k+1}+M_{k}$; the last relation shows that the index $n+1$ is also lucky. + +Thus, all indices $N>n$ are lucky, and $M_{N}=M_{n} \geqslant m_{N} / 3$, whence all the $m_{N}$ and $M_{N}$ are bounded by $3 M_{n}$. +Case 2. Conversely, assume that there is no lucky index, i.e., $2 M_{\lfloor n / 2\rfloor+1}+M_{\lfloor n / 2\rfloor}2 D$. Then (4) shows that for all $n>2 D$ we have $m_{n} \leqslant m_{n+1} \leqslant m_{n}$, i.e., $m_{N}=m_{2 D+1}$ for all $N>2 D$. Since $M_{N}2017 +$$ + +Then, this sequence is bounded. +Proof. As in the solutions above, let $D=2017$. If the sequence is bounded above, say, by $Q$, then we have that $a_{n} \geqslant \min \left\{a_{1}, \ldots, a_{D},-k Q\right\}$ for all $n$, so the sequence is bounded. Assume for sake of contradiction that the sequence is not bounded above. Let $\ell=\min \left\{a_{1}, \ldots, a_{D}\right\}$, and $L=\max \left\{a_{1}, \ldots, a_{D}\right\}$. Call an index $n$ good if the following criteria hold: + +$$ +a_{n}>a_{i} \text { for each } i-k \ell, \quad \text { and } \quad n>D +$$ + +We first show that there must be some good index $n$. By assumption, we may take an index $N$ such that $a_{N}>\max \{L,-k \ell\}$. Choose $n$ minimally such that $a_{n}=\max \left\{a_{1}, a_{2}, \ldots, a_{N}\right\}$. Now, the first condition is satisfied because of the minimality of $n$, and the second and third conditions are satisfied because $a_{n} \geqslant a_{N}>L,-k \ell$, and $L \geqslant a_{i}$ for every $i$ such that $1 \leqslant i \leqslant D$. + +Let $n$ be a good index. We derive a contradiction. We have that + +$$ +a_{n}+a_{v_{1}}+\cdots+a_{v_{k}} \leqslant 0 +$$ + +whenever $v_{1}+\cdots+v_{k}=n$. +We define the sequence of indices $v_{1}, \ldots, v_{k-1}$ to greedily maximize $a_{v_{1}}$, then $a_{v_{2}}$, and so forth, selecting only from indices such that the equation $v_{1}+\cdots+v_{k}=n$ can be satisfied by positive integers $v_{1}, \ldots, v_{k}$. More formally, we define them inductively so that the following criteria are satisfied by the $v_{i}$ : + +1. $1 \leqslant v_{i} \leqslant n-(k-i)-\left(v_{1}+\cdots+v_{i-1}\right)$. +2. $a_{v_{i}}$ is maximal among all choices of $v_{i}$ from the first criteria. + +First of all, we note that for each $i$, the first criteria is always satisfiable by some $v_{i}$, because we are guaranteed that + +$$ +v_{i-1} \leqslant n-(k-(i-1))-\left(v_{1}+\cdots+v_{i-2}\right), +$$ + +which implies + +$$ +1 \leqslant n-(k-i)-\left(v_{1}+\cdots+v_{i-1}\right) . +$$ + +Secondly, the sum $v_{1}+\cdots+v_{k-1}$ is at most $n-1$. Define $v_{k}=n-\left(v_{1}+\cdots+v_{k-1}\right)$. Then, (6) is satisfied by the $v_{i}$. We also note that $a_{v_{i}} \geqslant a_{v_{j}}$ for all $i-k \ell$ in the second criteria of (5). + +Now assume that $v_{k}>D$, and then we must have some indices $w_{1}, \ldots, w_{k}$ summing up to $v_{k}$ such that + +$$ +a_{v_{k}}+a_{w_{1}}+\cdots+a_{w_{k}}=0 +$$ + +But combining this with (6), we have + +$$ +a_{n}+a_{v_{1}}+\cdots+a_{v_{k-1}} \leqslant a_{w_{1}}+\cdots+a_{w_{k}} . +$$ + +Because $a_{n}>a_{v_{1}} \geqslant \cdots \geqslant a_{v_{k-1}}$, we have that $\max \left\{a_{w_{1}}, \ldots, a_{w_{k}}\right\}>a_{v_{k-1}}$. But since each of the $w_{i}$ is less than $v_{k}$, in the definition of the $v_{k-1}$ we could have chosen one of the $w_{i}$ instead, which is a contradiction. + +Comment 2. It seems that each sequence satisfying the condition in Version 2 is eventually periodic, at least when its terms are integers. + +However, up to this moment, the Problem Selection Committee is not aware of a proof for this fact (even in the case $k=2$ ). + +A5. An integer $n \geqslant 3$ is given. We call an $n$-tuple of real numbers $\left(x_{1}, x_{2}, \ldots, x_{n}\right)$ Shiny if for each permutation $y_{1}, y_{2}, \ldots, y_{n}$ of these numbers we have + +$$ +\sum_{i=1}^{n-1} y_{i} y_{i+1}=y_{1} y_{2}+y_{2} y_{3}+y_{3} y_{4}+\cdots+y_{n-1} y_{n} \geqslant-1 +$$ + +Find the largest constant $K=K(n)$ such that + +$$ +\sum_{1 \leqslant i\ell$. +Case 1: $k>\ell$. +Consider all permutations $\phi:\{1,2, \ldots, n\} \rightarrow\{1,2, \ldots, n\}$ such that $\phi^{-1}(T)=\{2,4, \ldots, 2 \ell\}$. Note that there are $k!!$ ! such permutations $\phi$. Define + +$$ +f(\phi)=\sum_{i=1}^{n-1} x_{\phi(i)} x_{\phi(i+1)} +$$ + +We know that $f(\phi) \geqslant-1$ for every permutation $\phi$ with the above property. Averaging $f(\phi)$ over all $\phi$ gives + +$$ +-1 \leqslant \frac{1}{k!\ell!} \sum_{\phi} f(\phi)=\frac{2 \ell}{k \ell} M+\frac{2(k-\ell-1)}{k(k-1)} K +$$ + +where the equality holds because there are $k \ell$ products in $M$, of which $2 \ell$ are selected for each $\phi$, and there are $k(k-1) / 2$ products in $K$, of which $k-\ell-1$ are selected for each $\phi$. We now have + +$$ +K+L+M \geqslant K+L+\left(-\frac{k}{2}-\frac{k-\ell-1}{k-1} K\right)=-\frac{k}{2}+\frac{\ell}{k-1} K+L +$$ + +Since $k \leqslant n-1$ and $K, L \geqslant 0$, we get the desired inequality. +Case 2: $k=\ell=n / 2$. +We do a similar approach, considering all $\phi:\{1,2, \ldots, n\} \rightarrow\{1,2, \ldots, n\}$ such that $\phi^{-1}(T)=$ $\{2,4, \ldots, 2 \ell\}$, and defining $f$ the same way. Analogously to Case 1 , we have + +$$ +-1 \leqslant \frac{1}{k!\ell!} \sum_{\phi} f(\phi)=\frac{2 \ell-1}{k \ell} M +$$ + +because there are $k \ell$ products in $M$, of which $2 \ell-1$ are selected for each $\phi$. Now, we have that + +$$ +K+L+M \geqslant M \geqslant-\frac{n^{2}}{4(n-1)} \geqslant-\frac{n-1}{2} +$$ + +where the last inequality holds because $n \geqslant 4$. + +A6. Find all functions $f: \mathbb{R} \rightarrow \mathbb{R}$ such that + +$$ +f(f(x) f(y))+f(x+y)=f(x y) +$$ + +for all $x, y \in \mathbb{R}$. +(Albania) +Answer: There are 3 solutions: + +$$ +x \mapsto 0 \quad \text { or } \quad x \mapsto x-1 \quad \text { or } \quad x \mapsto 1-x \quad(x \in \mathbb{R}) . +$$ + +Solution. An easy check shows that all the 3 above mentioned functions indeed satisfy the original equation (*). + +In order to show that these are the only solutions, first observe that if $f(x)$ is a solution then $-f(x)$ is also a solution. Hence, without loss of generality we may (and will) assume that $f(0) \leqslant 0$ from now on. We have to show that either $f$ is identically zero or $f(x)=x-1$ $(\forall x \in \mathbb{R})$. + +Observe that, for a fixed $x \neq 1$, we may choose $y \in \mathbb{R}$ so that $x+y=x y \Longleftrightarrow y=\frac{x}{x-1}$, and therefore from the original equation (*) we have + +$$ +f\left(f(x) \cdot f\left(\frac{x}{x-1}\right)\right)=0 \quad(x \neq 1) +$$ + +In particular, plugging in $x=0$ in (1), we conclude that $f$ has at least one zero, namely $(f(0))^{2}$ : + +$$ +f\left((f(0))^{2}\right)=0 +$$ + +We analyze two cases (recall that $f(0) \leqslant 0$ ): +Case 1: $f(0)=0$. +Setting $y=0$ in the original equation we get the identically zero solution: + +$$ +f(f(x) f(0))+f(x)=f(0) \Longrightarrow f(x)=0 \text { for all } x \in \mathbb{R} +$$ + +From now on, we work on the main +Case 2: $f(0)<0$. +We begin with the following + +## Claim 1. + +$$ +f(1)=0, \quad f(a)=0 \Longrightarrow a=1, \quad \text { and } \quad f(0)=-1 . +$$ + +Proof. We need to show that 1 is the unique zero of $f$. First, observe that $f$ has at least one zero $a$ by (2); if $a \neq 1$ then setting $x=a$ in (1) we get $f(0)=0$, a contradiction. Hence from (2) we get $(f(0))^{2}=1$. Since we are assuming $f(0)<0$, we conclude that $f(0)=-1$. + +Setting $y=1$ in the original equation (*) we get + +$$ +f(f(x) f(1))+f(x+1)=f(x) \Longleftrightarrow f(0)+f(x+1)=f(x) \Longleftrightarrow f(x+1)=f(x)+1 \quad(x \in \mathbb{R}) +$$ + +An easy induction shows that + +$$ +f(x+n)=f(x)+n \quad(x \in \mathbb{R}, n \in \mathbb{Z}) . +$$ + +Now we make the following +Claim 2. $f$ is injective. +Proof. Suppose that $f(a)=f(b)$ with $a \neq b$. Then by (4), for all $N \in \mathbb{Z}$, + +$$ +f(a+N+1)=f(b+N)+1 +$$ + +Choose any integer $N<-b$; then there exist $x_{0}, y_{0} \in \mathbb{R}$ with $x_{0}+y_{0}=a+N+1, x_{0} y_{0}=b+N$. Since $a \neq b$, we have $x_{0} \neq 1$ and $y_{0} \neq 1$. Plugging in $x_{0}$ and $y_{0}$ in the original equation (*) we get + +$$ +\begin{aligned} +f\left(f\left(x_{0}\right) f\left(y_{0}\right)\right)+f(a+N+1)=f(b+N) & \Longleftrightarrow f\left(f\left(x_{0}\right) f\left(y_{0}\right)\right)+1=0 \\ +& \Longleftrightarrow f\left(f\left(x_{0}\right) f\left(y_{0}\right)+1\right)=0 \\ +& \Longleftrightarrow f\left(x_{0}\right) f\left(y_{0}\right)=0 +\end{aligned} +$$ + +However, by Claim 1 we have $f\left(x_{0}\right) \neq 0$ and $f\left(y_{0}\right) \neq 0$ since $x_{0} \neq 1$ and $y_{0} \neq 1$, a contradiction. + +Now the end is near. For any $t \in \mathbb{R}$, plug in $(x, y)=(t,-t)$ in the original equation (*) to get + +$$ +\begin{aligned} +f(f(t) f(-t))+f(0)=f\left(-t^{2}\right) & \Longleftrightarrow f(f(t) f(-t))=f\left(-t^{2}\right)+1 & & \text { by }(3) \\ +& \Longleftrightarrow f(f(t) f(-t))=f\left(-t^{2}+1\right) & & \text { by }(4) \\ +& \Longleftrightarrow f(t) f(-t)=-t^{2}+1 & & \text { by injectivity of } f . +\end{aligned} +$$ + +Similarly, plugging in $(x, y)=(t, 1-t)$ in $(*)$ we get + +$$ +\begin{aligned} +f(f(t) f(1-t))+f(1)=f(t(1-t)) & \Longleftrightarrow f(f(t) f(1-t))=f(t(1-t)) & \text { by }(3) \\ +& \Longleftrightarrow f(t) f(1-t)=t(1-t) & \text { by injectivity of } f . +\end{aligned} +$$ + +But since $f(1-t)=1+f(-t)$ by (4), we get + +$$ +\begin{aligned} +f(t) f(1-t)=t(1-t) & \Longleftrightarrow f(t)(1+f(-t))=t(1-t) \Longleftrightarrow f(t)+\left(-t^{2}+1\right)=t(1-t) \\ +& \Longleftrightarrow f(t)=t-1, +\end{aligned} +$$ + +as desired. + +Comment. Other approaches are possible. For instance, after Claim 1, we may define + +$$ +g(x) \stackrel{\text { def }}{=} f(x)+1 +$$ + +Replacing $x+1$ and $y+1$ in place of $x$ and $y$ in the original equation (*), we get + +$$ +f(f(x+1) f(y+1))+f(x+y+2)=f(x y+x+y+1) \quad(x, y \in \mathbb{R}) +$$ + +and therefore, using (4) (so that in particular $g(x)=f(x+1)$ ), we may rewrite $(*)$ as + +$$ +g(g(x) g(y))+g(x+y)=g(x y+x+y) \quad(x, y \in \mathbb{R}) +$$ + +We are now to show that $g(x)=x$ for all $x \in \mathbb{R}$ under the assumption (Claim 1 ) that 0 is the unique zero of $g$. +Claim 3. Let $n \in \mathbb{Z}$ and $x \in \mathbb{R}$. Then +(a) $g(x+n)=x+n$, and the conditions $g(x)=n$ and $x=n$ are equivalent. +(b) $g(n x)=n g(x)$. + +Proof. For part (a), just note that $g(x+n)=x+n$ is just a reformulation of (4). Then $g(x)=n \Longleftrightarrow$ $g(x-n)=0 \Longleftrightarrow x-n=0$ since 0 is the unique zero of $g$. For part (b), we may assume that $x \neq 0$ since the result is obvious when $x=0$. Plug in $y=n / x$ in (**) and use part (a) to get + +$$ +g\left(g(x) g\left(\frac{n}{x}\right)\right)+g\left(x+\frac{n}{x}\right)=g\left(n+x+\frac{n}{x}\right) \Longleftrightarrow g\left(g(x) g\left(\frac{n}{x}\right)\right)=n \Longleftrightarrow g(x) g\left(\frac{n}{x}\right)=n +$$ + +In other words, for $x \neq 0$ we have + +$$ +g(x)=\frac{n}{g(n / x)} +$$ + +In particular, for $n=1$, we get $g(1 / x)=1 / g(x)$, and therefore replacing $x \leftarrow n x$ in the last equation we finally get + +$$ +g(n x)=\frac{n}{g(1 / x)}=n g(x) +$$ + +as required. +Claim 4. The function $g$ is additive, i.e., $g(a+b)=g(a)+g(b)$ for all $a, b \in \mathbb{R}$. +Proof. Set $x \leftarrow-x$ and $y \leftarrow-y$ in $(* *)$; since $g$ is an odd function (by Claim 3(b) with $n=-1$ ), we get + +$$ +g(g(x) g(y))-g(x+y)=-g(-x y+x+y) +$$ + +Subtracting the last relation from $(* *)$ we have + +$$ +2 g(x+y)=g(x y+x+y)+g(-x y+x+y) +$$ + +and since by Claim $3(\mathrm{~b})$ we have $2 g(x+y)=g(2(x+y))$, we may rewrite the last equation as + +$$ +g(\alpha+\beta)=g(\alpha)+g(\beta) \quad \text { where } \quad\left\{\begin{array}{l} +\alpha=x y+x+y \\ +\beta=-x y+x+y +\end{array}\right. +$$ + +In other words, we have additivity for all $\alpha, \beta \in \mathbb{R}$ for which there are real numbers $x$ and $y$ satisfying + +$$ +x+y=\frac{\alpha+\beta}{2} \quad \text { and } \quad x y=\frac{\alpha-\beta}{2} +$$ + +i.e., for all $\alpha, \beta \in \mathbb{R}$ such that $\left(\frac{\alpha+\beta}{2}\right)^{2}-4 \cdot \frac{\alpha-\beta}{2} \geqslant 0$. Therefore, given any $a, b \in \mathbb{R}$, we may choose $n \in \mathbb{Z}$ large enough so that we have additivity for $\alpha=n a$ and $\beta=n b$, i.e., + +$$ +g(n a)+g(n b)=g(n a+n b) \Longleftrightarrow n g(a)+n g(b)=n g(a+b) +$$ + +by Claim $3(\mathrm{~b})$. Cancelling $n$, we get the desired result. (Alternatively, setting either $(\alpha, \beta)=(a, b)$ or $(\alpha, \beta)=(-a,-b)$ will ensure that $\left.\left(\frac{\alpha+\beta}{2}\right)^{2}-4 \cdot \frac{\alpha-\beta}{2} \geqslant 0\right)$. + +Now we may finish the solution. Set $y=1$ in $(* *)$, and use Claim 3 to get + +$$ +g(g(x) g(1))+g(x+1)=g(2 x+1) \Longleftrightarrow g(g(x))+g(x)+1=2 g(x)+1 \Longleftrightarrow g(g(x))=g(x) +$$ + +By additivity, this is equivalent to $g(g(x)-x)=0$. Since 0 is the unique zero of $g$ by assumption, we finally get $g(x)-x=0 \Longleftrightarrow g(x)=x$ for all $x \in \mathbb{R}$. + +A7. Let $a_{0}, a_{1}, a_{2}, \ldots$ be a sequence of integers and $b_{0}, b_{1}, b_{2}, \ldots$ be a sequence of positive integers such that $a_{0}=0, a_{1}=1$, and + +$$ +a_{n+1}=\left\{\begin{array}{ll} +a_{n} b_{n}+a_{n-1}, & \text { if } b_{n-1}=1 \\ +a_{n} b_{n}-a_{n-1}, & \text { if } b_{n-1}>1 +\end{array} \quad \text { for } n=1,2, \ldots\right. +$$ + +Prove that at least one of the two numbers $a_{2017}$ and $a_{2018}$ must be greater than or equal to 2017 . +(Australia) +Solution 1. The value of $b_{0}$ is irrelevant since $a_{0}=0$, so we may assume that $b_{0}=1$. +Lemma. We have $a_{n} \geqslant 1$ for all $n \geqslant 1$. +Proof. Let us suppose otherwise in order to obtain a contradiction. Let + +$$ +n \geqslant 1 \text { be the smallest integer with } a_{n} \leqslant 0 \text {. } +$$ + +Note that $n \geqslant 2$. It follows that $a_{n-1} \geqslant 1$ and $a_{n-2} \geqslant 0$. Thus we cannot have $a_{n}=$ $a_{n-1} b_{n-1}+a_{n-2}$, so we must have $a_{n}=a_{n-1} b_{n-1}-a_{n-2}$. Since $a_{n} \leqslant 0$, we have $a_{n-1} \leqslant a_{n-2}$. Thus we have $a_{n-2} \geqslant a_{n-1} \geqslant a_{n}$. + +Let + +$$ +r \text { be the smallest index with } a_{r} \geqslant a_{r+1} \geqslant a_{r+2} \text {. } +$$ + +Then $r \leqslant n-2$ by the above, but also $r \geqslant 2$ : if $b_{1}=1$, then $a_{2}=a_{1}=1$ and $a_{3}=a_{2} b_{2}+a_{1}>a_{2}$; if $b_{1}>1$, then $a_{2}=b_{1}>1=a_{1}$. + +By the minimal choice (2) of $r$, it follows that $a_{r-1}0$. In order to have $a_{r+1} \geqslant a_{r+2}$, we must have $a_{r+2}=a_{r+1} b_{r+1}-a_{r}$ so that $b_{r} \geqslant 2$. Putting everything together, we conclude that + +$$ +a_{r+1}=a_{r} b_{r} \pm a_{r-1} \geqslant 2 a_{r}-a_{r-1}=a_{r}+\left(a_{r}-a_{r-1}\right)>a_{r} +$$ + +which contradicts (2). +To complete the problem, we prove that $\max \left\{a_{n}, a_{n+1}\right\} \geqslant n$ by induction. The cases $n=0,1$ are given. Assume it is true for all non-negative integers strictly less than $n$, where $n \geqslant 2$. There are two cases: + +Case 1: $b_{n-1}=1$. +Then $a_{n+1}=a_{n} b_{n}+a_{n-1}$. By the inductive assumption one of $a_{n-1}, a_{n}$ is at least $n-1$ and the other, by the lemma, is at least 1 . Hence + +$$ +a_{n+1}=a_{n} b_{n}+a_{n-1} \geqslant a_{n}+a_{n-1} \geqslant(n-1)+1=n . +$$ + +Thus $\max \left\{a_{n}, a_{n+1}\right\} \geqslant n$, as desired. +Case 2: $b_{n-1}>1$. +Since we defined $b_{0}=1$ there is an index $r$ with $1 \leqslant r \leqslant n-1$ such that + +$$ +b_{n-1}, b_{n-2}, \ldots, b_{r} \geqslant 2 \quad \text { and } \quad b_{r-1}=1 +$$ + +We have $a_{r+1}=a_{r} b_{r}+a_{r-1} \geqslant 2 a_{r}+a_{r-1}$. Thus $a_{r+1}-a_{r} \geqslant a_{r}+a_{r-1}$. +Now we claim that $a_{r}+a_{r-1} \geqslant r$. Indeed, this holds by inspection for $r=1$; for $r \geqslant 2$, one of $a_{r}, a_{r-1}$ is at least $r-1$ by the inductive assumption, while the other, by the lemma, is at least 1 . Hence $a_{r}+a_{r-1} \geqslant r$, as claimed, and therefore $a_{r+1}-a_{r} \geqslant r$ by the last inequality in the previous paragraph. + +Since $r \geqslant 1$ and, by the lemma, $a_{r} \geqslant 1$, from $a_{r+1}-a_{r} \geqslant r$ we get the following two inequalities: + +$$ +a_{r+1} \geqslant r+1 \quad \text { and } \quad a_{r+1}>a_{r} . +$$ + +Now observe that + +$$ +a_{m}>a_{m-1} \Longrightarrow a_{m+1}>a_{m} \text { for } m=r+1, r+2, \ldots, n-1 \text {, } +$$ + +since $a_{m+1}=a_{m} b_{m}-a_{m-1} \geqslant 2 a_{m}-a_{m-1}=a_{m}+\left(a_{m}-a_{m-1}\right)>a_{m}$. Thus + +$$ +a_{n}>a_{n-1}>\cdots>a_{r+1} \geqslant r+1 \Longrightarrow a_{n} \geqslant n +$$ + +So $\max \left\{a_{n}, a_{n+1}\right\} \geqslant n$, as desired. +Solution 2. We say that an index $n>1$ is bad if $b_{n-1}=1$ and $b_{n-2}>1$; otherwise $n$ is good. The value of $b_{0}$ is irrelevant to the definition of $\left(a_{n}\right)$ since $a_{0}=0$; so we assume that $b_{0}>1$. +Lemma 1. (a) $a_{n} \geqslant 1$ for all $n>0$. +(b) If $n>1$ is good, then $a_{n}>a_{n-1}$. + +Proof. Induction on $n$. In the base cases $n=1,2$ we have $a_{1}=1 \geqslant 1, a_{2}=b_{1} a_{1} \geqslant 1$, and finally $a_{2}>a_{1}$ if 2 is good, since in this case $b_{1}>1$. + +Now we assume that the lemma statement is proved for $n=1,2, \ldots, k$ with $k \geqslant 2$, and prove it for $n=k+1$. Recall that $a_{k}$ and $a_{k-1}$ are positive by the induction hypothesis. +Case 1: $k$ is bad. +We have $b_{k-1}=1$, so $a_{k+1}=b_{k} a_{k}+a_{k-1} \geqslant a_{k}+a_{k-1}>a_{k} \geqslant 1$, as required. +Case 2: $k$ is good. +We already have $a_{k}>a_{k-1} \geqslant 1$ by the induction hypothesis. We consider three easy subcases. + +Subcase 2.1: $b_{k}>1$. +Then $a_{k+1} \geqslant b_{k} a_{k}-a_{k-1} \geqslant a_{k}+\left(a_{k}-a_{k-1}\right)>a_{k} \geqslant 1$. +Subcase 2.2: $b_{k}=b_{k-1}=1$. +Then $a_{k+1}=a_{k}+a_{k-1}>a_{k} \geqslant 1$. +Subcase 2.3: $b_{k}=1$ but $b_{k-1}>1$. +Then $k+1$ is bad, and we need to prove only (a), which is trivial: $a_{k+1}=a_{k}-a_{k-1} \geqslant 1$. +So, in all three subcases we have verified the required relations. +Lemma 2. Assume that $n>1$ is bad. Then there exists a $j \in\{1,2,3\}$ such that $a_{n+j} \geqslant$ $a_{n-1}+j+1$, and $a_{n+i} \geqslant a_{n-1}+i$ for all $1 \leqslant i0: b_{n+i-1}>1\right\} +$$ + +(possibly $m=+\infty$ ). We claim that $j=\min \{m, 3\}$ works. Again, we distinguish several cases, according to the value of $m$; in each of them we use Lemma 1 without reference. +Case 1: $m=1$, so $b_{n}>1$. +Then $a_{n+1} \geqslant 2 a_{n}+a_{n-1} \geqslant a_{n-1}+2$, as required. +Case 2: $m=2$, so $b_{n}=1$ and $b_{n+1}>1$. +Then we successively get + +$$ +\begin{gathered} +a_{n+1}=a_{n}+a_{n-1} \geqslant a_{n-1}+1 \\ +a_{n+2} \geqslant 2 a_{n+1}+a_{n} \geqslant 2\left(a_{n-1}+1\right)+a_{n}=a_{n-1}+\left(a_{n-1}+a_{n}+2\right) \geqslant a_{n-1}+4 +\end{gathered} +$$ + +which is even better than we need. + +Case 3: $m>2$, so $b_{n}=b_{n+1}=1$. +Then we successively get + +$$ +\begin{gathered} +a_{n+1}=a_{n}+a_{n-1} \geqslant a_{n-1}+1, \quad a_{n+2}=a_{n+1}+a_{n} \geqslant a_{n-1}+1+a_{n} \geqslant a_{n-1}+2, \\ +a_{n+3} \geqslant a_{n+2}+a_{n+1} \geqslant\left(a_{n-1}+1\right)+\left(a_{n-1}+2\right) \geqslant a_{n-1}+4 +\end{gathered} +$$ + +as required. +Lemmas 1 (b) and 2 provide enough information to prove that $\max \left\{a_{n}, a_{n+1}\right\} \geqslant n$ for all $n$ and, moreover, that $a_{n} \geqslant n$ often enough. Indeed, assume that we have found some $n$ with $a_{n-1} \geqslant n-1$. If $n$ is good, then by Lemma 1 (b) we have $a_{n} \geqslant n$ as well. If $n$ is bad, then Lemma 2 yields $\max \left\{a_{n+i}, a_{n+i+1}\right\} \geqslant a_{n-1}+i+1 \geqslant n+i$ for all $0 \leqslant i0$, we have $f(x)+y=f(y)+x$. +Prove that $f(x)+y \leqslant f(y)+x$ whenever $x>y$. +(Netherlands) +Solution 1. Define $g(x)=x-f(x)$. The condition on $f$ then rewrites as follows: +For every $x, y \in \mathbb{R}$ such that $((x+y)-g(x))((x+y)-g(y))>0$, we have $g(x)=g(y)$. +This condition may in turn be rewritten in the following form: +If $g(x) \neq g(y)$, then the number $x+y$ lies (non-strictly) between $g(x)$ and $g(y)$. +Notice here that the function $g_{1}(x)=-g(-x)$ also satisfies $(*)$, since + +$$ +\begin{aligned} +g_{1}(x) \neq g_{1}(y) \Longrightarrow & g(-x) \neq g(-y) \Longrightarrow \quad-(x+y) \text { lies between } g(-x) \text { and } g(-y) \\ +& \Longrightarrow \quad x+y \text { lies between } g_{1}(x) \text { and } g_{1}(y) +\end{aligned} +$$ + +On the other hand, the relation we need to prove reads now as + +$$ +g(x) \leqslant g(y) \quad \text { whenever } xX$. Similarly, if $X>2 x$, then $g$ attains at most two values on $[x ; X-x)$ - namely, $X$ and, possibly, some $YX$ and hence by (*) we get $X \leqslant a+x \leqslant g(a)$. + +Now, for any $b \in(X-x ; x)$ with $g(b) \neq X$ we similarly get $b+x \leqslant g(b)$. Therefore, the number $a+b$ (which is smaller than each of $a+x$ and $b+x$ ) cannot lie between $g(a)$ and $g(b)$, which by (*) implies that $g(a)=g(b)$. Hence $g$ may attain only two values on $(X-x ; x]$, namely $X$ and $g(a)>X$. + +To prove the second claim, notice that $g_{1}(-x)=-X<2 \cdot(-x)$, so $g_{1}$ attains at most two values on $(-X+x,-x]$, i.e., $-X$ and, possibly, some $-Y>-X$. Passing back to $g$, we get what we need. +Lemma 2. If $X<2 x$, then $g$ is constant on $(X-x ; x)$. Similarly, if $X>2 x$, then $g$ is constant on $(x ; X-x)$. +Proof. Again, it suffices to prove the first claim only. Assume, for the sake of contradiction, that there exist $a, b \in(X-x ; x)$ with $g(a) \neq g(b)$; by Lemma 1, we may assume that $g(a)=X$ and $Y=g(b)>X$. + +Notice that $\min \{X-a, X-b\}>X-x$, so there exists a $u \in(X-x ; x)$ such that $u<\min \{X-a, X-b\}$. By Lemma 1, we have either $g(u)=X$ or $g(u)=Y$. In the former case, by (*) we have $X \leqslant u+b \leqslant Y$ which contradicts $u2 x$, then $g(a)=X$ for all $a \in(x ; X-x)$. +Proof. Again, we only prove the first claim. +By Lemmas 1 and 2, this claim may be violated only if $g$ takes on a constant value $Y>X$ on $(X-x, x)$. Choose any $a, b \in(X-x ; x)$ with $a2 a$. Applying Lemma 2 to $a$ in place of $x$, we obtain that $g$ is constant on $(a, Y-a)$. By (2) again, we have $x \leqslant Y-bg(y)$ for some $xc$. Now, for any $x$ with $g(x)=c$, by (*) we have $c \leqslant x+y \leqslant g(y)$, or $c-y \leqslant x \leqslant g(y)-y$. Since $c$ and $y$ are constant, we get what we need. + +If $g(y)-c$. By the above arguments, we obtain that all the solutions of $g_{1}(-x)=-c$ are bounded, which is equivalent to what we need. + +As an immediate consequence, the function $g$ takes on infinitely many values, which shows that the next claim is indeed widely applicable. +Claim 2. If $g(x)g(y)$ for some $x0$ the equation $g(x)=-N x$ has at most one solution, and (ii) for every $N>1$ the equation $g(x)=N x$ has at least one solution. + +Claim ( $i$ ) is now trivial. Indeed, $g$ is proven to be non-decreasing, so $g(x)+N x$ is strictly increasing and thus has at most one zero. + +We proceed on claim $(i i)$. If $g(0)=0$, then the required root has been already found. Otherwise, we may assume that $g(0)>0$ and denote $c=g(0)$. We intend to prove that $x=c / N$ is the required root. Indeed, by monotonicity we have $g(c / N) \geqslant g(0)=c$; if we had $g(c / N)>c$, then (*) would yield $c \leqslant 0+c / N \leqslant g(c / N)$ which is false. Thus, $g(x)=c=N x$. + +Comment 2. There are plenty of functions $g$ satisfying (*) (and hence of functions $f$ satisfying the problem conditions). One simple example is $g_{0}(x)=2 x$. Next, for any increasing sequence $A=\left(\ldots, a_{-1}, a_{0}, a_{1}, \ldots\right)$ which is unbounded in both directions (i.e., for every $N$ this sequence contains terms greater than $N$, as well as terms smaller than $-N$ ), the function $g_{A}$ defined by + +$$ +g_{A}(x)=a_{i}+a_{i+1} \quad \text { whenever } x \in\left[a_{i} ; a_{i+1}\right) +$$ + +satisfies (*). Indeed, pick any $xx\} & \text { and } \quad s_{A-}(x)=\sup \{a \in A: aN_{t}(Y)$, and define $d^{*}(X, Y)$ to be the number of $(X, Y)$-inversions. Then for any two chameleons $Y$ and $Y^{\prime}$ differing by a single swap, we have $\left|d^{*}(X, Y)-d^{*}\left(X, Y^{\prime}\right)\right|=1$. Since $d^{*}(X, X)=0$, this yields $d(X, Y) \geqslant d^{*}(X, Y)$ for any pair of chameleons $X$ and $Y$. The bound $d^{*}$ may also be used in both Solution 1 and Solution 2. + +Comment 3. In fact, one may prove that the distance $d^{*}$ defined in the previous comment coincides with $d$. Indeed, if $X \neq Y$, then there exist an ( $X, Y$ )-inversion $(s, t)$. One can show that such $s$ and $t$ may be chosen to occupy consecutive positions in $Y$. Clearly, $s$ and $t$ correspond to different letters among $\{a, b, c\}$. So, swapping them in $Y$ we get another chameleon $Y^{\prime}$ with $d^{*}\left(X, Y^{\prime}\right)=d^{*}(X, Y)-1$. Proceeding in this manner, we may change $Y$ to $X$ in $d^{*}(X, Y)$ steps. + +Using this fact, one can show that the estimate in the problem statement is sharp for all $n \geqslant 2$. (For $n=1$ it is not sharp, since any permutation of three letters can be changed to an opposite one in no less than three swaps.) We outline the proof below. + +For any $k \geqslant 0$, define + +$$ +X_{2 k}=\underbrace{a b c a b c \ldots a b c}_{3 k \text { letters }} \underbrace{c b a c b a \ldots c b a}_{3 k \text { letters }} \text { and } \quad X_{2 k+3}=\underbrace{a b c a b c \ldots a b c}_{3 k \text { letters }} a b c b c a c a b \underbrace{c b a c b a \ldots c b a}_{3 k \text { letters }} . +$$ + +We claim that for every $n \geqslant 2$ and every chameleon $Y$, we have $d^{*}\left(X_{n}, Y\right) \leqslant\left\lceil 3 n^{2} / 2\right\rceil$. This will mean that for every $n \geqslant 2$ the number $3 n^{2} / 2$ in the problem statement cannot be changed by any number larger than $\left\lceil 3 n^{2} / 2\right\rceil$. + +For any distinct letters $u, v \in\{a, b, c\}$ and any two chameleons $X$ and $Y$, we define $d_{u, v}^{*}(X, Y)$ as the number of $(X, Y)$-inversions $(s, t)$ such that $s$ and $t$ are instances of $u$ and $v$ (in any of the two possible orders). Then $d^{*}(X, Y)=d_{a, b}^{*}(X, Y)+d_{b, c}^{*}(X, Y)+d_{c, a}^{*}(X, Y)$. + +We start with the case when $n=2 k$ is even; denote $X=X_{2 k}$. We show that $d_{a, b}^{*}(X, Y) \leqslant 2 k^{2}$ for any chameleon $Y$; this yields the required estimate. Proceed by the induction on $k$ with the trivial base case $k=0$. To perform the induction step, notice that $d_{a, b}^{*}(X, Y)$ is indeed the minimal number of swaps needed to change $Y_{\bar{c}}$ into $X_{\bar{c}}$. One may show that moving $a_{1}$ and $a_{2 k}$ in $Y$ onto the first and the last positions in $Y$, respectively, takes at most $2 k$ swaps, and that subsequent moving $b_{1}$ and $b_{2 k}$ onto the second and the second last positions takes at most $2 k-2$ swaps. After performing that, one may delete these letters from both $X_{\bar{c}}$ and $Y_{\bar{c}}$ and apply the induction hypothesis; so $X_{\bar{c}}$ can be obtained from $Y_{\bar{c}}$ using at most $2(k-1)^{2}+2 k+(2 k-2)=2 k^{2}$ swaps, as required. + +If $n=2 k+3$ is odd, the proof is similar but more technically involved. Namely, we claim that $d_{a, b}^{*}\left(X_{2 k+3}, Y\right) \leqslant 2 k^{2}+6 k+5$ for any chameleon $Y$, and that the equality is achieved only if $Y_{\bar{c}}=$ $b b \ldots b a a \ldots a$. The proof proceeds by a similar induction, with some care taken of the base case, as well as of extracting the equality case. Similar estimates hold for $d_{b, c}^{*}$ and $d_{c, a}^{*}$. Summing three such estimates, we obtain + +$$ +d^{*}\left(X_{2 k+3}, Y\right) \leqslant 3\left(2 k^{2}+6 k+5\right)=\left\lceil\frac{3 n^{2}}{2}\right\rceil+1 +$$ + +which is by 1 more than we need. But the equality could be achieved only if $Y_{\bar{c}}=b b \ldots b a a \ldots a$ and, similarly, $Y_{\bar{b}}=a a \ldots a c c \ldots c$ and $Y_{\bar{a}}=c c \ldots c b b \ldots b$. Since these three equalities cannot hold simultaneously, the proof is finished. + +C3. Sir Alex plays the following game on a row of 9 cells. Initially, all cells are empty. In each move, Sir Alex is allowed to perform exactly one of the following two operations: +(1) Choose any number of the form $2^{j}$, where $j$ is a non-negative integer, and put it into an empty cell. +(2) Choose two (not necessarily adjacent) cells with the same number in them; denote that number by $2^{j}$. Replace the number in one of the cells with $2^{j+1}$ and erase the number in the other cell. + +At the end of the game, one cell contains the number $2^{n}$, where $n$ is a given positive integer, while the other cells are empty. Determine the maximum number of moves that Sir Alex could have made, in terms of $n$. +(Thailand) +Answer: $2 \sum_{j=0}^{8}\binom{n}{j}-1$. +Solution 1. We will solve a more general problem, replacing the row of 9 cells with a row of $k$ cells, where $k$ is a positive integer. Denote by $m(n, k)$ the maximum possible number of moves Sir Alex can make starting with a row of $k$ empty cells, and ending with one cell containing the number $2^{n}$ and all the other $k-1$ cells empty. Call an operation of type (1) an insertion, and an operation of type (2) a merge. + +Only one move is possible when $k=1$, so we have $m(n, 1)=1$. From now on we consider $k \geqslant 2$, and we may assume Sir Alex's last move was a merge. Then, just before the last move, there were exactly two cells with the number $2^{n-1}$, and the other $k-2$ cells were empty. + +Paint one of those numbers $2^{n-1}$ blue, and the other one red. Now trace back Sir Alex's moves, always painting the numbers blue or red following this rule: if $a$ and $b$ merge into $c$, paint $a$ and $b$ with the same color as $c$. Notice that in this backward process new numbers are produced only by reversing merges, since reversing an insertion simply means deleting one of the numbers. Therefore, all numbers appearing in the whole process will receive one of the two colors. + +Sir Alex's first move is an insertion. Without loss of generality, assume this first number inserted is blue. Then, from this point on, until the last move, there is always at least one cell with a blue number. + +Besides the last move, there is no move involving a blue and a red number, since all merges involves numbers with the same color, and insertions involve only one number. Call an insertion of a blue number or merge of two blue numbers a blue move, and define a red move analogously. + +The whole sequence of blue moves could be repeated on another row of $k$ cells to produce one cell with the number $2^{n-1}$ and all the others empty, so there are at most $m(n-1, k)$ blue moves. + +Now we look at the red moves. Since every time we perform a red move there is at least one cell occupied with a blue number, the whole sequence of red moves could be repeated on a row of $k-1$ cells to produce one cell with the number $2^{n-1}$ and all the others empty, so there are at most $m(n-1, k-1)$ red moves. This proves that + +$$ +m(n, k) \leqslant m(n-1, k)+m(n-1, k-1)+1 . +$$ + +On the other hand, we can start with an empty row of $k$ cells and perform $m(n-1, k)$ moves to produce one cell with the number $2^{n-1}$ and all the others empty, and after that perform $m(n-1, k-1)$ moves on those $k-1$ empty cells to produce the number $2^{n-1}$ in one of them, leaving $k-2$ empty. With one more merge we get one cell with $2^{n}$ and the others empty, proving that + +$$ +m(n, k) \geqslant m(n-1, k)+m(n-1, k-1)+1 . +$$ + +It follows that + +$$ +m(n, k)=m(n-1, k)+m(n-1, k-1)+1, +$$ + +for $n \geqslant 1$ and $k \geqslant 2$. +If $k=1$ or $n=0$, we must insert $2^{n}$ on our first move and immediately get the final configuration, so $m(0, k)=1$ and $m(n, 1)=1$, for $n \geqslant 0$ and $k \geqslant 1$. These initial values, together with the recurrence relation (1), determine $m(n, k)$ uniquely. + +Finally, we show that + +$$ +m(n, k)=2 \sum_{j=0}^{k-1}\binom{n}{j}-1 +$$ + +for all integers $n \geqslant 0$ and $k \geqslant 1$. +We use induction on $n$. Since $m(0, k)=1$ for $k \geqslant 1,(2)$ is true for the base case. We make the induction hypothesis that (2) is true for some fixed positive integer $n$ and all $k \geqslant 1$. We have $m(n+1,1)=1=2\binom{n+1}{0}-1$, and for $k \geqslant 2$ the recurrence relation (1) and the induction hypothesis give us + +$$ +\begin{aligned} +& m(n+1, k)=m(n, k)+m(n, k-1)+1=2 \sum_{j=0}^{k-1}\binom{n}{j}-1+2 \sum_{j=0}^{k-2}\binom{n}{j}-1+1 \\ +& \quad=2 \sum_{j=0}^{k-1}\binom{n}{j}+2 \sum_{j=0}^{k-1}\binom{n}{j-1}-1=2 \sum_{j=0}^{k-1}\left(\binom{n}{j}+\binom{n}{j-1}\right)-1=2 \sum_{j=0}^{k-1}\binom{n+1}{j}-1, +\end{aligned} +$$ + +which completes the proof. + +Comment 1. After deducing the recurrence relation (1), it may be convenient to homogenize the recurrence relation by defining $h(n, k)=m(n, k)+1$. We get the new relation + +$$ +h(n, k)=h(n-1, k)+h(n-1, k), +$$ + +for $n \geqslant 1$ and $k \geqslant 2$, with initial values $h(0, k)=h(n, 1)=2$, for $n \geqslant 0$ and $k \geqslant 1$. +This may help one to guess the answer, and also with other approaches like the one we develop next. + +Comment 2. We can use a generating function to find the answer without guessing. We work with the homogenized recurrence relation (3). Define $h(n, 0)=0$ so that (3) is valid for $k=1$ as well. Now we set up the generating function $f(x, y)=\sum_{n, k \geqslant 0} h(n, k) x^{n} y^{k}$. Multiplying the recurrence relation (3) by $x^{n} y^{k}$ and summing over $n, k \geqslant 1$, we get + +$$ +\sum_{n, k \geqslant 1} h(n, k) x^{n} y^{k}=x \sum_{n, k \geqslant 1} h(n-1, k) x^{n-1} y^{k}+x y \sum_{n, k \geqslant 1} h(n-1, k-1) x^{n-1} y^{k-1} . +$$ + +Completing the missing terms leads to the following equation on $f(x, y)$ : + +$$ +f(x, y)-\sum_{n \geqslant 0} h(n, 0) x^{n}-\sum_{k \geqslant 1} h(0, k) y^{k}=x f(x, y)-x \sum_{n \geqslant 0} h(n, 0) x^{n}+x y f(x, y) . +$$ + +Substituting the initial values, we obtain + +$$ +f(x, y)=\frac{2 y}{1-y} \cdot \frac{1}{1-x(1+y)} . +$$ + +Developing as a power series, we get + +$$ +f(x, y)=2 \sum_{j \geqslant 1} y^{j} \cdot \sum_{n \geqslant 0}(1+y)^{n} x^{n} . +$$ + +The coefficient of $x^{n}$ in this power series is + +$$ +2 \sum_{j \geqslant 1} y^{j} \cdot(1+y)^{n}=2 \sum_{j \geqslant 1} y^{j} \cdot \sum_{i \geqslant 0}\binom{n}{i} y^{i} +$$ + +and extracting the coefficient of $y^{k}$ in this last expression we finally obtain the value for $h(n, k)$, + +$$ +h(n, k)=2 \sum_{j=0}^{k-1}\binom{n}{j} +$$ + +This proves that + +$$ +m(n, k)=2 \sum_{j=0}^{k-1}\binom{n}{j}-1 +$$ + +The generating function approach also works if applied to the non-homogeneous recurrence relation (1), but the computations are less straightforward. +Solution 2. Define merges and insertions as in Solution 1. After each move made by Sir Alex we compute the number $N$ of empty cells, and the $\operatorname{sum} S$ of all the numbers written in the cells. Insertions always increase $S$ by some power of 2 , and increase $N$ exactly by 1 . Merges do not change $S$ and decrease $N$ exactly by 1 . Since the initial value of $N$ is 0 and its final value is 1 , the total number of insertions exceeds that of merges by exactly one. So, to maximize the number of moves, we need to maximize the number of insertions. + +We will need the following lemma. +Lemma. If the binary representation of a positive integer $A$ has $d$ nonzero digits, then $A$ cannot be represented as a sum of fewer than $d$ powers of 2 . Moreover, any representation of $A$ as a sum of $d$ powers of 2 must coincide with its binary representation. +Proof. Let $s$ be the minimum number of summands in all possible representations of $A$ as sum of powers of 2 . Suppose there is such a representation with $s$ summands, where two of the summands are equal to each other. Then, replacing those two summands with the result of their sum, we obtain a representation with fewer than $s$ summands, which is a contradiction. We deduce that in any representation with $s$ summands, the summands are all distinct, so any such representation must coincide with the unique binary representation of $A$, and $s=d$. + +Now we split the solution into a sequence of claims. +Claim 1. After every move, the number $S$ is the sum of at most $k-1$ distinct powers of 2 . +Proof. If $S$ is the sum of $k$ (or more) distinct powers of 2 , the Lemma implies that the $k$ cells are filled with these numbers. This is a contradiction since no more merges or insertions can be made. + +Let $A(n, k-1)$ denote the set of all positive integers not exceeding $2^{n}$ with at most $k-1$ nonzero digits in its base 2 representation. Since every insertion increases the value of $S$, by Claim 1, the total number of insertions is at most $|A(n, k-1)|$. We proceed to prove that it is possible to achieve this number of insertions. +Claim 2. Let $A(n, k-1)=\left\{a_{1}, a_{2}, \ldots, a_{m}\right\}$, with $a_{1}$ | $\boldsymbol{x}_{\boldsymbol{N}, \boldsymbol{N}}$ | +| $\vee$ | $\vee$ | $\vee$ | | $\vee$ | $\vee$ | +| $x_{N+1,1}$ | $x_{N+1,2}$ | $x_{N+1,3}$ | $\cdots$ | $x_{N+1, N-1}$ | $\boldsymbol{x}_{\boldsymbol{N + 1 , N}}$ | + +Now we make the bold choice: from the original row of people, remove everyone but those with heights + +$$ +x_{1,1}>x_{2,1}>x_{2,2}>x_{3,2}>\cdots>x_{N, N-1}>x_{N, N}>x_{N+1, N} +$$ + +Of course this height order (*) is not necessarily their spatial order in the new row. We now need to convince ourselves that each pair $\left(x_{k, k} ; x_{k+1, k}\right)$ remains spatially together in this new row. But $x_{k, k}$ and $x_{k+1, k}$ belong to the same column/block of consecutive $N+1$ people; the only people that could possibly stand between them were also in this block, and they are all gone. + +Solution 2. Split the people into $N$ groups by height: group $G_{1}$ has the $N+1$ tallest ones, group $G_{2}$ has the next $N+1$ tallest, and so on, up to group $G_{N}$ with the $N+1$ shortest people. + +Now scan the original row from left to right, stopping as soon as you have scanned two people (consecutively or not) from the same group, say, $G_{i}$. Since we have $N$ groups, this must happen before or at the $(N+1)^{\text {th }}$ person of the row. Choose this pair of people, removing all the other people from the same group $G_{i}$ and also all people that have been scanned so far. The only people that could separate this pair's heights were in group $G_{i}$ (and they are gone); the only people that could separate this pair's positions were already scanned (and they are gone too). + +We are now left with $N-1$ groups (all except $G_{i}$ ). Since each of them lost at most one person, each one has at least $N$ unscanned people left in the row. Repeat the scanning process from left to right, choosing the next two people from the same group, removing this group and +everyone scanned up to that point. Once again we end up with two people who are next to each other in the remaining row and whose heights cannot be separated by anyone else who remains (since the rest of their group is gone). After picking these 2 pairs, we still have $N-2$ groups with at least $N-1$ people each. + +If we repeat the scanning process a total of $N$ times, it is easy to check that we will end up with 2 people from each group, for a total of $2 N$ people remaining. The height order is guaranteed by the grouping, and the scanning construction from left to right guarantees that each pair from a group stand next to each other in the final row. We are done. + +Solution 3. This is essentially the same as solution 1, but presented inductively. The essence of the argument is the following lemma. +Lemma. Assume that we have $N$ disjoint groups of at least $N+1$ people in each, all people have distinct heights. Then one can choose two people from each group so that among the chosen people, the two tallest ones are in one group, the third and the fourth tallest ones are in one group, ..., and the two shortest ones are in one group. +Proof. Induction on $N \geqslant 1$; for $N=1$, the statement is trivial. +Consider now $N$ groups $G_{1}, \ldots, G_{N}$ with at least $N+1$ people in each for $N \geqslant 2$. Enumerate the people by $1,2, \ldots, N(N+1)$ according to their height, say, from tallest to shortest. Find the least $s$ such that two people among $1,2, \ldots, s$ are in one group (without loss of generality, say this group is $G_{N}$ ). By the minimality of $s$, the two mentioned people in $G_{N}$ are $s$ and some $i\frac{1}{400} +$$ + +In particular, $\varepsilon^{2}+1=400 \varepsilon$, so + +$$ +y^{2}=d_{n}^{2}-2 \varepsilon d_{n}+\varepsilon^{2}+1=d_{n}^{2}+\varepsilon\left(400-2 d_{n}\right) +$$ + +Since $\varepsilon>\frac{1}{400}$ and we assumed $d_{n}<100$, this shows that $y^{2}>d_{n}^{2}+\frac{1}{2}$. So, as we claimed, with this list of radar pings, no matter what the hunter does, the rabbit might achieve $d_{n+200}^{2}>d_{n}^{2}+\frac{1}{2}$. The wabbit wins. + +Comment 1. Many different versions of the solution above can be found by replacing 200 with some other number $N$ for the number of hops the rabbit takes between reveals. If this is done, we have: + +$$ +\varepsilon=N-\sqrt{N^{2}-1}>\frac{1}{N+\sqrt{N^{2}-1}}>\frac{1}{2 N} +$$ + +and + +$$ +\varepsilon^{2}+1=2 N \varepsilon +$$ + +so, as long as $N>d_{n}$, we would find + +$$ +y^{2}=d_{n}^{2}+\varepsilon\left(2 N-2 d_{n}\right)>d_{n}^{2}+\frac{N-d_{n}}{N} +$$ + +For example, taking $N=101$ is already enough-the squared distance increases by at least $\frac{1}{101}$ every 101 rounds, and $101^{2} \cdot 10^{4}=1.0201 \cdot 10^{8}<10^{9}$ rounds are enough for the rabbit. If the statement is made sharper, some such versions might not work any longer. + +Comment 2. The original statement asked whether the distance could be kept under $10^{10}$ in $10^{100}$ rounds. + +C6. Let $n>1$ be an integer. An $n \times n \times n$ cube is composed of $n^{3}$ unit cubes. Each unit cube is painted with one color. For each $n \times n \times 1$ box consisting of $n^{2}$ unit cubes (of any of the three possible orientations), we consider the set of the colors present in that box (each color is listed only once). This way, we get $3 n$ sets of colors, split into three groups according to the orientation. It happens that for every set in any group, the same set appears in both of the other groups. Determine, in terms of $n$, the maximal possible number of colors that are present. +(Russia) +Answer: The maximal number is $\frac{n(n+1)(2 n+1)}{6}$. +Solution 1. Call a $n \times n \times 1$ box an $x$-box, a $y$-box, or a $z$-box, according to the direction of its short side. Let $C$ be the number of colors in a valid configuration. We start with the upper bound for $C$. + +Let $\mathcal{C}_{1}, \mathcal{C}_{2}$, and $\mathcal{C}_{3}$ be the sets of colors which appear in the big cube exactly once, exactly twice, and at least thrice, respectively. Let $M_{i}$ be the set of unit cubes whose colors are in $\mathcal{C}_{i}$, and denote $n_{i}=\left|M_{i}\right|$. + +Consider any $x$-box $X$, and let $Y$ and $Z$ be a $y$ - and a $z$-box containing the same set of colors as $X$ does. +Claim. $4\left|X \cap M_{1}\right|+\left|X \cap M_{2}\right| \leqslant 3 n+1$. +Proof. We distinguish two cases. +Case 1: $X \cap M_{1} \neq \varnothing$. +A cube from $X \cap M_{1}$ should appear in all three boxes $X, Y$, and $Z$, so it should lie in $X \cap Y \cap Z$. Thus $X \cap M_{1}=X \cap Y \cap Z$ and $\left|X \cap M_{1}\right|=1$. + +Consider now the cubes in $X \cap M_{2}$. There are at most $2(n-1)$ of them lying in $X \cap Y$ or $X \cap Z$ (because the cube from $X \cap Y \cap Z$ is in $M_{1}$ ). Let $a$ be some other cube from $X \cap M_{2}$. Recall that there is just one other cube $a^{\prime}$ sharing a color with $a$. But both $Y$ and $Z$ should contain such cube, so $a^{\prime} \in Y \cap Z$ (but $a^{\prime} \notin X \cap Y \cap Z$ ). The map $a \mapsto a^{\prime}$ is clearly injective, so the number of cubes $a$ we are interested in does not exceed $|(Y \cap Z) \backslash X|=n-1$. Thus $\left|X \cap M_{2}\right| \leqslant 2(n-1)+(n-1)=3(n-1)$, and hence $4\left|X \cap M_{1}\right|+\left|X \cap M_{2}\right| \leqslant 4+3(n-1)=3 n+1$. Case 2: $X \cap M_{1}=\varnothing$. + +In this case, the same argument applies with several changes. Indeed, $X \cap M_{2}$ contains at most $2 n-1$ cubes from $X \cap Y$ or $X \cap Z$. Any other cube $a$ in $X \cap M_{2}$ corresponds to some $a^{\prime} \in Y \cap Z$ (possibly with $a^{\prime} \in X$ ), so there are at most $n$ of them. All this results in $\left|X \cap M_{2}\right| \leqslant(2 n-1)+n=3 n-1$, which is even better than we need (by the assumptions of our case). + +Summing up the inequalities from the Claim over all $x$-boxes $X$, we obtain + +$$ +4 n_{1}+n_{2} \leqslant n(3 n+1) +$$ + +Obviously, we also have $n_{1}+n_{2}+n_{3}=n^{3}$. +Now we are prepared to estimate $C$. Due to the definition of the $M_{i}$, we have $n_{i} \geqslant i\left|\mathcal{C}_{i}\right|$, so + +$$ +C \leqslant n_{1}+\frac{n_{2}}{2}+\frac{n_{3}}{3}=\frac{n_{1}+n_{2}+n_{3}}{3}+\frac{4 n_{1}+n_{2}}{6} \leqslant \frac{n^{3}}{3}+\frac{3 n^{2}+n}{6}=\frac{n(n+1)(2 n+1)}{6} +$$ + +It remains to present an example of an appropriate coloring in the above-mentioned number of colors. For each color, we present the set of all cubes of this color. These sets are: + +1. $n$ singletons of the form $S_{i}=\{(i, i, i)\}$ (with $1 \leqslant i \leqslant n$ ); +2. $3\binom{n}{2}$ doubletons of the forms $D_{i, j}^{1}=\{(i, j, j),(j, i, i)\}, D_{i, j}^{2}=\{(j, i, j),(i, j, i)\}$, and $D_{i, j}^{3}=$ $\{(j, j, i),(i, i, j)\}$ (with $1 \leqslant i0$ positive integers, and let $B$ be a set of $b>0$ positive integers. Prove that if $A * B=B * A$, then + +$$ +\underbrace{A *(A * \cdots *(A *(A * A)) \ldots)}_{A \text { appears } b \text { times }}=\underbrace{B *(B * \cdots *(B *(B * B)) \ldots)}_{B \text { appears } a \text { times }} . +$$ + +Solution 1. For any function $g: \mathbb{Z}_{>0} \rightarrow \mathbb{Z}_{>0}$ and any subset $X \subset \mathbb{Z}_{>0}$, we define $g(X)=$ $\{g(x): x \in X\}$. We have that the image of $f_{X}$ is $f_{X}\left(\mathbb{Z}_{>0}\right)=\mathbb{Z}_{>0} \backslash X$. We now show a general lemma about the operation *, with the goal of showing that $*$ is associative. +Lemma 1. Let $X$ and $Y$ be finite sets of positive integers. The functions $f_{X * Y}$ and $f_{X} \circ f_{Y}$ are equal. +Proof. We have +$f_{X * Y}\left(\mathbb{Z}_{>0}\right)=\mathbb{Z}_{>0} \backslash(X * Y)=\left(\mathbb{Z}_{>0} \backslash X\right) \backslash f_{X}(Y)=f_{X}\left(\mathbb{Z}_{>0}\right) \backslash f_{X}(Y)=f_{X}\left(\mathbb{Z}_{>0} \backslash Y\right)=f_{X}\left(f_{Y}\left(\mathbb{Z}_{>0}\right)\right)$. +Thus, the functions $f_{X * Y}$ and $f_{X} \circ f_{Y}$ are strictly increasing functions with the same range. Because a strictly function is uniquely defined by its range, we have $f_{X * Y}=f_{X} \circ f_{Y}$. + +Lemma 1 implies that * is associative, in the sense that $(A * B) * C=A *(B * C)$ for any finite sets $A, B$, and $C$ of positive integers. We prove the associativity by noting + +$$ +\begin{gathered} +\mathbb{Z}_{>0} \backslash((A * B) * C)=f_{(A * B) * C}\left(\mathbb{Z}_{>0}\right)=f_{A * B}\left(f_{C}\left(\mathbb{Z}_{>0}\right)\right)=f_{A}\left(f_{B}\left(f_{C}\left(\mathbb{Z}_{>0}\right)\right)\right) \\ +=f_{A}\left(f_{B * C}\left(\mathbb{Z}_{>0}\right)=f_{A *(B * C)}\left(\mathbb{Z}_{>0}\right)=\mathbb{Z}_{>0} \backslash(A *(B * C))\right. +\end{gathered} +$$ + +In light of the associativity of *, we may drop the parentheses when we write expressions like $A *(B * C)$. We also introduce the notation + +$$ +X^{* k}=\underbrace{X *(X * \cdots *(X *(X * X)) \ldots)}_{X \text { appears } k \text { times }} +$$ + +Our goal is then to show that $A * B=B * A$ implies $A^{* b}=B^{* a}$. We will do so via the following general lemma. +Lemma 2. Suppose that $X$ and $Y$ are finite sets of positive integers satisfying $X * Y=Y * X$ and $|X|=|Y|$. Then, we must have $X=Y$. +Proof. Assume that $X$ and $Y$ are not equal. Let $s$ be the largest number in exactly one of $X$ and $Y$. Without loss of generality, say that $s \in X \backslash Y$. The number $f_{X}(s)$ counts the $s^{t h}$ number not in $X$, which implies that + +$$ +f_{X}(s)=s+\left|X \cap\left\{1,2, \ldots, f_{X}(s)\right\}\right| +$$ + +Since $f_{X}(s) \geqslant s$, we have that + +$$ +\left\{f_{X}(s)+1, f_{X}(s)+2, \ldots\right\} \cap X=\left\{f_{X}(s)+1, f_{X}(s)+2, \ldots\right\} \cap Y +$$ + +which, together with the assumption that $|X|=|Y|$, gives + +$$ +\left|X \cap\left\{1,2, \ldots, f_{X}(s)\right\}\right|=\left|Y \cap\left\{1,2, \ldots, f_{X}(s)\right\}\right| +$$ + +Now consider the equation + +$$ +t-|Y \cap\{1,2, \ldots, t\}|=s +$$ + +This equation is satisfied only when $t \in\left[f_{Y}(s), f_{Y}(s+1)\right)$, because the left hand side counts the number of elements up to $t$ that are not in $Y$. We have that the value $t=f_{X}(s)$ satisfies the above equation because of (1) and (2). Furthermore, since $f_{X}(s) \notin X$ and $f_{X}(s) \geqslant s$, we have that $f_{X}(s) \notin Y$ due to the maximality of $s$. Thus, by the above discussion, we must have $f_{X}(s)=f_{Y}(s)$. + +Finally, we arrive at a contradiction. The value $f_{X}(s)$ is neither in $X$ nor in $f_{X}(Y)$, because $s$ is not in $Y$ by assumption. Thus, $f_{X}(s) \notin X * Y$. However, since $s \in X$, we have $f_{Y}(s) \in Y * X$, a contradiction. + +We are now ready to finish the proof. Note first of all that $\left|A^{* b}\right|=a b=\left|B^{* a}\right|$. Moreover, since $A * B=B * A$, and $*$ is associative, it follows that $A^{* b} * B^{* a}=B^{* a} * A^{* b}$. Thus, by Lemma 2, we have $A^{* b}=B^{* a}$, as desired. + +Comment 1. Taking $A=X^{* k}$ and $B=X^{* l}$ generates many non-trivial examples where $A * B=B * A$. There are also other examples not of this form. For example, if $A=\{1,2,4\}$ and $B=\{1,3\}$, then $A * B=\{1,2,3,4,6\}=B * A$. +Solution 2. We will use Lemma 1 from Solution 1. Additionally, let $X^{* k}$ be defined as in Solution 1. If $X$ and $Y$ are finite sets, then + +$$ +f_{X}=f_{Y} \Longleftrightarrow f_{X}\left(\mathbb{Z}_{>0}\right)=f_{Y}\left(\mathbb{Z}_{>0}\right) \Longleftrightarrow\left(\mathbb{Z}_{>0} \backslash X\right)=\left(\mathbb{Z}_{>0} \backslash Y\right) \Longleftrightarrow X=Y, +$$ + +where the first equivalence is because $f_{X}$ and $f_{Y}$ are strictly increasing functions, and the second equivalence is because $f_{X}\left(\mathbb{Z}_{>0}\right)=\mathbb{Z}_{>0} \backslash X$ and $f_{Y}\left(\mathbb{Z}_{>0}\right)=\mathbb{Z}_{>0} \backslash Y$. + +Denote $g=f_{A}$ and $h=f_{B}$. The given relation $A * B=B * A$ is equivalent to $f_{A * B}=f_{B * A}$ because of (3), and by Lemma 1 of the first solution, this is equivalent to $g \circ h=h \circ g$. Similarly, the required relation $A^{* b}=B^{* a}$ is equivalent to $g^{b}=h^{a}$. We will show that + +$$ +g^{b}(n)=h^{a}(n) +$$ + +for all $n \in \mathbb{Z}_{>0}$, which suffices to solve the problem. +To start, we claim that (4) holds for all sufficiently large $n$. Indeed, let $p$ and $q$ be the maximal elements of $A$ and $B$, respectively; we may assume that $p \geqslant q$. Then, for every $n \geqslant p$ we have $g(n)=n+a$ and $h(n)=n+b$, whence $g^{b}(n)=n+a b=h^{a}(n)$, as was claimed. + +In view of this claim, if (4) is not identically true, then there exists a maximal $s$ with $g^{b}(s) \neq$ $h^{a}(s)$. Without loss of generality, we may assume that $g(s) \neq s$, for if we had $g(s)=h(s)=s$, then $s$ would satisfy (4). As $g$ is increasing, we then have $g(s)>s$, so (4) holds for $n=g(s)$. But then we have + +$$ +g\left(g^{b}(s)\right)=g^{b+1}(s)=g^{b}(n)=h^{a}(n)=h^{a}(g(s))=g\left(h^{a}(s)\right), +$$ + +where the last equality holds in view of $g \circ h=h \circ g$. By the injectivity of $g$, the above equality yields $g^{b}(s)=h^{a}(s)$, which contradicts the choice of $s$. Thus, we have proved that (4) is identically true on $\mathbb{Z}_{>0}$, as desired. + +Comment 2. We present another proof of Lemma 2 of the first solution. +Let $x=|X|=|Y|$. Say that $u$ is the smallest number in $X$ and $v$ is the smallest number in $Y$; assume without loss of generality that $u \leqslant v$. + +Let $T$ be any finite set of positive integers, and define $t=|T|$. Enumerate the elements of $X$ as $x_{1}m_{i}$, we have that $s_{m, i}=t+m n-c_{i}$. Furthermore, the $c_{i}$ do not depend on the choice of $T$. + +First, we show that this claim implies Lemma 2. We may choose $T=X$ and $T=Y$. Then, there is some $m^{\prime}$ such that for all $m \geqslant m^{\prime}$, we have + +$$ +f_{X * m}(X)=f_{(Y * X *(m-1))}(X) +$$ + +Because $u$ is the minimum element of $X, v$ is the minimum element of $Y$, and $u \leqslant v$, we have that + +$$ +\left(\bigcup_{m=m^{\prime}}^{\infty} f_{X * m}(X)\right) \cup X^{* m^{\prime}}=\left(\bigcup_{m=m^{\prime}}^{\infty} f_{\left(Y * X^{*(m-1)}\right)}(X)\right) \cup\left(Y * X^{*\left(m^{\prime}-1\right)}\right)=\{u, u+1, \ldots\}, +$$ + +and in both the first and second expressions, the unions are of pairwise distinct sets. By (5), we obtain $X^{* m^{\prime}}=Y * X^{*\left(m^{\prime}-1\right)}$. Now, because $X$ and $Y$ commute, we get $X^{* m^{\prime}}=X^{*\left(m^{\prime}-1\right)} * Y$, and so $X=Y$. + +We now prove the claim. +Proof of the claim. We induct downwards on $i$, first proving the statement for $i=n$, and so on. +Assume that $m$ is chosen so that all elements of $S_{m}$ are greater than all elements of $T$ (which is possible because $T$ is finite). For $i=n$, we have that $s_{m, n}>s_{k, n}$ for every $ki$ and $pi$ eventually do not depend on $T$, the sequence $s_{m, i}-t$ eventually does not depend on $T$ either, so the inductive step is complete. This proves the claim and thus Lemma 2. + +C8. Let $n$ be a given positive integer. In the Cartesian plane, each lattice point with nonnegative coordinates initially contains a butterfly, and there are no other butterflies. The neighborhood of a lattice point $c$ consists of all lattice points within the axis-aligned $(2 n+1) \times$ $(2 n+1)$ square centered at $c$, apart from $c$ itself. We call a butterfly lonely, crowded, or comfortable, depending on whether the number of butterflies in its neighborhood $N$ is respectively less than, greater than, or equal to half of the number of lattice points in $N$. + +Every minute, all lonely butterflies fly away simultaneously. This process goes on for as long as there are any lonely butterflies. Assuming that the process eventually stops, determine the number of comfortable butterflies at the final state. +(Bulgaria) +Answer: $n^{2}+1$. +Solution. We always identify a butterfly with the lattice point it is situated at. For two points $p$ and $q$, we write $p \geqslant q$ if each coordinate of $p$ is at least the corresponding coordinate of $q$. Let $O$ be the origin, and let $\mathcal{Q}$ be the set of initially occupied points, i.e., of all lattice points with nonnegative coordinates. Let $\mathcal{R}_{\mathrm{H}}=\{(x, 0): x \geqslant 0\}$ and $\mathcal{R}_{\mathrm{V}}=\{(0, y): y \geqslant 0\}$ be the sets of the lattice points lying on the horizontal and vertical boundary rays of $\mathcal{Q}$. Denote by $N(a)$ the neighborhood of a lattice point $a$. + +1. Initial observations. We call a set of lattice points up-right closed if its points stay in the set after being shifted by any lattice vector $(i, j)$ with $i, j \geqslant 0$. Whenever the butterflies form a up-right closed set $\mathcal{S}$, we have $|N(p) \cap \mathcal{S}| \geqslant|N(q) \cap \mathcal{S}|$ for any two points $p, q \in \mathcal{S}$ with $p \geqslant q$. So, since $\mathcal{Q}$ is up-right closed, the set of butterflies at any moment also preserves this property. We assume all forthcoming sets of lattice points to be up-right closed. + +When speaking of some set $\mathcal{S}$ of lattice points, we call its points lonely, comfortable, or crowded with respect to this set (i.e., as if the butterflies were exactly at all points of $\mathcal{S}$ ). We call a set $\mathcal{S} \subset \mathcal{Q}$ stable if it contains no lonely points. In what follows, we are interested only in those stable sets whose complements in $\mathcal{Q}$ are finite, because one can easily see that only a finite number of butterflies can fly away on each minute. + +If the initial set $\mathcal{Q}$ of butterflies contains some stable set $\mathcal{S}$, then, clearly no butterfly of this set will fly away. On the other hand, the set $\mathcal{F}$ of all butterflies in the end of the process is stable. This means that $\mathcal{F}$ is the largest (with respect to inclusion) stable set within $\mathcal{Q}$, and we are about to describe this set. +2. A description of a final set. The following notion will be useful. Let $\mathcal{U}=\left\{\vec{u}_{1}, \vec{u}_{2}, \ldots, \vec{u}_{d}\right\}$ be a set of $d$ pairwise non-parallel lattice vectors, each having a positive $x$ - and a negative $y$-coordinate. Assume that they are numbered in increasing order according to slope. We now define a $\mathcal{U}$-curve to be the broken line $p_{0} p_{1} \ldots p_{d}$ such that $p_{0} \in \mathcal{R}_{\mathrm{V}}, p_{d} \in \mathcal{R}_{\mathrm{H}}$, and $\overrightarrow{p_{i-1} p_{i}}=\vec{u}_{i}$ for all $i=1,2, \ldots, m$ (see the Figure below to the left). +![](https://cdn.mathpix.com/cropped/2024_11_18_ff08e3c077fa2f8c16b2g-53.jpg?height=546&width=766&top_left_y=2194&top_left_x=191) + +Construction of $\mathcal{U}$-curve +![](https://cdn.mathpix.com/cropped/2024_11_18_ff08e3c077fa2f8c16b2g-53.jpg?height=415&width=803&top_left_y=2322&top_left_x=1066) + +Construction of $\mathcal{D}$ + +Now, let $\mathcal{K}_{n}=\{(i, j): 1 \leqslant i \leqslant n,-n \leqslant j \leqslant-1\}$. Consider all the rays emerging at $O$ and passing through a point from $\mathcal{K}_{n}$; number them as $r_{1}, \ldots, r_{m}$ in increasing order according to slope. Let $A_{i}$ be the farthest from $O$ lattice point in $r_{i} \cap \mathcal{K}_{n}$, set $k_{i}=\left|r_{i} \cap \mathcal{K}_{n}\right|$, let $\vec{v}_{i}=\overrightarrow{O A_{i}}$, and finally denote $\mathcal{V}=\left\{\vec{v}_{i}: 1 \leqslant i \leqslant m\right\}$; see the Figure above to the right. We will concentrate on the $\mathcal{V}$-curve $d_{0} d_{1} \ldots d_{m}$; let $\mathcal{D}$ be the set of all lattice points $p$ such that $p \geqslant p^{\prime}$ for some (not necessarily lattice) point $p^{\prime}$ on the $\mathcal{V}$-curve. In fact, we will show that $\mathcal{D}=\mathcal{F}$. + +Clearly, the $\mathcal{V}$-curve is symmetric in the line $y=x$. Denote by $D$ the convex hull of $\mathcal{D}$. +3. We prove that the set $\mathcal{D}$ contains all stable sets. Let $\mathcal{S} \subset \mathcal{Q}$ be a stable set (recall that it is assumed to be up-right closed and to have a finite complement in $\mathcal{Q}$ ). Denote by $S$ its convex hull; clearly, the vertices of $S$ are lattice points. The boundary of $S$ consists of two rays (horizontal and vertical ones) along with some $\mathcal{V}_{*}$-curve for some set of lattice vectors $\mathcal{V}_{*}$. +Claim 1. For every $\vec{v}_{i} \in \mathcal{V}$, there is a $\vec{v}_{i}^{*} \in \mathcal{V}_{*}$ co-directed with $\vec{v}$ with $\left|\vec{v}_{i}^{*}\right| \geqslant|\vec{v}|$. +Proof. Let $\ell$ be the supporting line of $S$ parallel to $\vec{v}_{i}$ (i.e., $\ell$ contains some point of $S$, and the set $S$ lies on one side of $\ell$ ). Take any point $b \in \ell \cap \mathcal{S}$ and consider $N(b)$. The line $\ell$ splits the set $N(b) \backslash \ell$ into two congruent parts, one having an empty intersection with $\mathcal{S}$. Hence, in order for $b$ not to be lonely, at least half of the set $\ell \cap N(b)$ (which contains $2 k_{i}$ points) should lie in $S$. Thus, the boundary of $S$ contains a segment $\ell \cap S$ with at least $k_{i}+1$ lattice points (including $b$ ) on it; this segment corresponds to the required vector $\vec{v}_{i}^{*} \in \mathcal{V}_{*}$. +![](https://cdn.mathpix.com/cropped/2024_11_18_ff08e3c077fa2f8c16b2g-54.jpg?height=600&width=1626&top_left_y=1199&top_left_x=221) + +Claim 2. Each stable set $\mathcal{S} \subseteq \mathcal{Q}$ lies in $\mathcal{D}$. +Proof. To show this, it suffices to prove that the $\mathcal{V}_{*}$-curve lies in $D$, i.e., that all its vertices do so. Let $p^{\prime}$ be an arbitrary vertex of the $\mathcal{V}_{*^{-}}$-curve; $p^{\prime}$ partitions this curve into two parts, $\mathcal{X}$ (being down-right of $p$ ) and $\mathcal{Y}$ (being up-left of $p$ ). The set $\mathcal{V}$ is split now into two parts: $\mathcal{V}_{\mathcal{X}}$ consisting of those $\vec{v}_{i} \in \mathcal{V}$ for which $\vec{v}_{i}^{*}$ corresponds to a segment in $\mathcal{X}$, and a similar part $\mathcal{V}_{\mathcal{Y}}$. Notice that the $\mathcal{V}$-curve consists of several segments corresponding to $\mathcal{V}_{\mathcal{X}}$, followed by those corresponding to $\mathcal{V}_{\mathcal{Y}}$. Hence there is a vertex $p$ of the $\mathcal{V}$-curve separating $\mathcal{V}_{\mathcal{X}}$ from $\mathcal{V}_{\mathcal{Y}}$. Claim 1 now yields that $p^{\prime} \geqslant p$, so $p^{\prime} \in \mathcal{D}$, as required. + +Claim 2 implies that the final set $\mathcal{F}$ is contained in $\mathcal{D}$. +4. $\mathcal{D}$ is stable, and its comfortable points are known. Recall the definitions of $r_{i}$; let $r_{i}^{\prime}$ be the ray complementary to $r_{i}$. By our definitions, the set $N(O)$ contains no points between the rays $r_{i}$ and $r_{i+1}$, as well as between $r_{i}^{\prime}$ and $r_{i+1}^{\prime}$. +Claim 3. In the set $\mathcal{D}$, all lattice points of the $\mathcal{V}$-curve are comfortable. +Proof. Let $p$ be any lattice point of the $\mathcal{V}$-curve, belonging to some segment $d_{i} d_{i+1}$. Draw the line $\ell$ containing this segment. Then $\ell \cap \mathcal{D}$ contains exactly $k_{i}+1$ lattice points, all of which lie in $N(p)$ except for $p$. Thus, exactly half of the points in $N(p) \cap \ell$ lie in $\mathcal{D}$. It remains to show that all points of $N(p)$ above $\ell$ lie in $\mathcal{D}$ (recall that all the points below $\ell$ lack this property). + +Notice that each vector in $\mathcal{V}$ has one coordinate greater than $n / 2$; thus the neighborhood of $p$ contains parts of at most two segments of the $\mathcal{V}$-curve succeeding $d_{i} d_{i+1}$, as well as at most two of those preceding it. + +The angles formed by these consecutive segments are obtained from those formed by $r_{j}$ and $r_{j-1}^{\prime}$ (with $i-1 \leqslant j \leqslant i+2$ ) by shifts; see the Figure below. All the points in $N(p)$ above $\ell$ which could lie outside $\mathcal{D}$ lie in shifted angles between $r_{j}, r_{j+1}$ or $r_{j}^{\prime}, r_{j-1}^{\prime}$. But those angles, restricted to $N(p)$, have no lattice points due to the above remark. The claim is proved. +![](https://cdn.mathpix.com/cropped/2024_11_18_ff08e3c077fa2f8c16b2g-55.jpg?height=404&width=1052&top_left_y=592&top_left_x=503) + +Proof of Claim 3 +Claim 4. All the points of $\mathcal{D}$ which are not on the boundary of $D$ are crowded. +Proof. Let $p \in \mathcal{D}$ be such a point. If it is to the up-right of some point $p^{\prime}$ on the curve, then the claim is easy: the shift of $N\left(p^{\prime}\right) \cap \mathcal{D}$ by $\overrightarrow{p^{\prime} p}$ is still in $\mathcal{D}$, and $N(p)$ contains at least one more point of $\mathcal{D}$ - either below or to the left of $p$. So, we may assume that $p$ lies in a right triangle constructed on some hypothenuse $d_{i} d_{i+1}$. Notice here that $d_{i}, d_{i+1} \in N(p)$. + +Draw a line $\ell \| d_{i} d_{i+1}$ through $p$, and draw a vertical line $h$ through $d_{i}$; see Figure below. Let $\mathcal{D}_{\mathrm{L}}$ and $\mathcal{D}_{\mathrm{R}}$ be the parts of $\mathcal{D}$ lying to the left and to the right of $h$, respectively (points of $\mathcal{D} \cap h$ lie in both parts). +![](https://cdn.mathpix.com/cropped/2024_11_18_ff08e3c077fa2f8c16b2g-55.jpg?height=538&width=1120&top_left_y=1556&top_left_x=468) + +Notice that the vectors $\overrightarrow{d_{i} p}, \overrightarrow{d_{i+1} d_{i+2}}, \overrightarrow{d_{i} d_{i+1}}, \overrightarrow{d_{i-1} d_{i}}$, and $\overrightarrow{p d_{i+1}}$ are arranged in non-increasing order by slope. This means that $\mathcal{D}_{\mathrm{L}}$ shifted by $\overrightarrow{d_{i} p}$ still lies in $\mathcal{D}$, as well as $\mathcal{D}_{\mathrm{R}}$ shifted by $\overrightarrow{d_{i+1} p}$. As we have seen in the proof of Claim 3, these two shifts cover all points of $N(p)$ above $\ell$, along with those on $\ell$ to the left of $p$. Since $N(p)$ contains also $d_{i}$ and $d_{i+1}$, the point $p$ is crowded. + +Thus, we have proved that $\mathcal{D}=\mathcal{F}$, and have shown that the lattice points on the $\mathcal{V}$-curve are exactly the comfortable points of $\mathcal{D}$. It remains to find their number. + +Recall the definition of $\mathcal{K}_{n}$ (see Figure on the first page of the solution). Each segment $d_{i} d_{i+1}$ contains $k_{i}$ lattice points different from $d_{i}$. Taken over all $i$, these points exhaust all the lattice points in the $\mathcal{V}$-curve, except for $d_{1}$, and thus the number of lattice points on the $\mathcal{V}$-curve is $1+\sum_{i=1}^{m} k_{i}$. On the other hand, $\sum_{i=1}^{m} k_{i}$ is just the number of points in $\mathcal{K}_{n}$, so it equals $n^{2}$. Hence the answer to the problem is $n^{2}+1$. + +Comment 1. The assumption that the process eventually stops is unnecessary for the problem, as one can see that, in fact, the process stops for every $n \geqslant 1$. Indeed, the proof of Claims 3 and 4 do not rely essentially on this assumption, and they together yield that the set $\mathcal{D}$ is stable. So, only butterflies that are not in $\mathcal{D}$ may fly away, and this takes only a finite time. + +This assumption has been inserted into the problem statement in order to avoid several technical details regarding finiteness issues. It may also simplify several other arguments. + +Comment 2. The description of the final set $\mathcal{F}(=\mathcal{D})$ seems to be crucial for the solution; the Problem Selection Committee is not aware of any solution that completely avoids such a description. + +On the other hand, after the set $\mathcal{D}$ has been defined, the further steps may be performed in several ways. For example, in order to prove that all butterflies outside $\mathcal{D}$ will fly away, one may argue as follows. (Here we will also make use of the assumption that the process eventually stops.) + +First of all, notice that the process can be modified in the following manner: Each minute, exactly one of the lonely butterflies flies away, until there are no more lonely butterflies. The modified process necessarily stops at the same state as the initial one. Indeed, one may observe, as in solution above, that the (unique) largest stable set is still the final set for the modified process. + +Thus, in order to prove our claim, it suffices to indicate an order in which the butterflies should fly away in the new process; if we are able to exhaust the whole set $\mathcal{Q} \backslash \mathcal{D}$, we are done. + +Let $\mathcal{C}_{0}=d_{0} d_{1} \ldots d_{m}$ be the $\mathcal{V}$-curve. Take its copy $\mathcal{C}$ and shift it downwards so that $d_{0}$ comes to some point below the origin $O$. Now we start moving $\mathcal{C}$ upwards continuously, until it comes back to its initial position $\mathcal{C}_{0}$. At each moment when $\mathcal{C}$ meets some lattice points, we convince all the butterflies at those points to fly away in a certain order. We will now show that we always have enough arguments for butterflies to do so, which will finish our argument for the claim.. + +Let $\mathcal{C}^{\prime}=d_{0}^{\prime} d_{1}^{\prime} \ldots d_{m}^{\prime}$ be a position of $\mathcal{C}$ when it meets some butterflies. We assume that all butterflies under this current position of $\mathcal{C}$ were already convinced enough and flied away. Consider the lowest butterfly $b$ on $\mathcal{C}^{\prime}$. Let $d_{i}^{\prime} d_{i+1}^{\prime}$ be the segment it lies on; we choose $i$ so that $b \neq d_{i+1}^{\prime}$ (this is possible because $\mathcal{C}$ as not yet reached $\mathcal{C}_{0}$ ). + +Draw a line $\ell$ containing the segment $d_{i}^{\prime} d_{i+1}^{\prime}$. Then all the butterflies in $N(b)$ are situated on or above $\ell$; moreover, those on $\ell$ all lie on the segment $d_{i} d_{i+1}$. But this segment now contains at most $k_{i}$ butterflies (including $b$ ), since otherwise some butterfly had to occupy $d_{i+1}^{\prime}$ which is impossible by the choice of $b$. Thus, $b$ is lonely and hence may be convinced to fly away. + +After $b$ has flied away, we switch to the lowest of the remaining butterflies on $\mathcal{C}^{\prime}$, and so on. +Claims 3 and 4 also allow some different proofs which are not presented here. + +This page is intentionally left blank + +## Geometry + +G1. Let $A B C D E$ be a convex pentagon such that $A B=B C=C D, \angle E A B=\angle B C D$, and $\angle E D C=\angle C B A$. Prove that the perpendicular line from $E$ to $B C$ and the line segments $A C$ and $B D$ are concurrent. +(Italy) +Solution 1. Throughout the solution, we refer to $\angle A, \angle B, \angle C, \angle D$, and $\angle E$ as internal angles of the pentagon $A B C D E$. Let the perpendicular bisectors of $A C$ and $B D$, which pass respectively through $B$ and $C$, meet at point $I$. Then $B D \perp C I$ and, similarly, $A C \perp B I$. Hence $A C$ and $B D$ meet at the orthocenter $H$ of the triangle $B I C$, and $I H \perp B C$. It remains to prove that $E$ lies on the line $I H$ or, equivalently, $E I \perp B C$. + +Lines $I B$ and $I C$ bisect $\angle B$ and $\angle C$, respectively. Since $I A=I C, I B=I D$, and $A B=$ $B C=C D$, the triangles $I A B, I C B$ and $I C D$ are congruent. Hence $\angle I A B=\angle I C B=$ $\angle C / 2=\angle A / 2$, so the line $I A$ bisects $\angle A$. Similarly, the line $I D$ bisects $\angle D$. Finally, the line $I E$ bisects $\angle E$ because $I$ lies on all the other four internal bisectors of the angles of the pentagon. + +The sum of the internal angles in a pentagon is $540^{\circ}$, so + +$$ +\angle E=540^{\circ}-2 \angle A+2 \angle B . +$$ + +In quadrilateral $A B I E$, + +$$ +\begin{aligned} +\angle B I E & =360^{\circ}-\angle E A B-\angle A B I-\angle A E I=360^{\circ}-\angle A-\frac{1}{2} \angle B-\frac{1}{2} \angle E \\ +& =360^{\circ}-\angle A-\frac{1}{2} \angle B-\left(270^{\circ}-\angle A-\angle B\right) \\ +& =90^{\circ}+\frac{1}{2} \angle B=90^{\circ}+\angle I B C, +\end{aligned} +$$ + +which means that $E I \perp B C$, completing the proof. +![](https://cdn.mathpix.com/cropped/2024_11_18_ff08e3c077fa2f8c16b2g-58.jpg?height=618&width=730&top_left_y=1801&top_left_x=663) + +Solution 2. We present another proof of the fact that $E$ lies on line $I H$. Since all five internal bisectors of $A B C D E$ meet at $I$, this pentagon has an inscribed circle with center $I$. Let this circle touch side $B C$ at $T$. + +Applying Brianchon's theorem to the (degenerate) hexagon $A B T C D E$ we conclude that $A C, B D$ and $E T$ are concurrent, so point $E$ also lies on line $I H T$, completing the proof. + +Solution 3. We present yet another proof that $E I \perp B C$. In pentagon $A B C D E, \angle E<$ $180^{\circ} \Longleftrightarrow \angle A+\angle B+\angle C+\angle D>360^{\circ}$. Then $\angle A+\angle B=\angle C+\angle D>180^{\circ}$, so rays $E A$ and $C B$ meet at a point $P$, and rays $B C$ and $E D$ meet at a point $Q$. Now, + +$$ +\angle P B A=180^{\circ}-\angle B=180^{\circ}-\angle D=\angle Q D C +$$ + +and, similarly, $\angle P A B=\angle Q C D$. Since $A B=C D$, the triangles $P A B$ and $Q C D$ are congruent with the same orientation. Moreover, $P Q E$ is isosceles with $E P=E Q$. +![](https://cdn.mathpix.com/cropped/2024_11_18_ff08e3c077fa2f8c16b2g-59.jpg?height=524&width=1546&top_left_y=612&top_left_x=255) + +In Solution 1 we have proved that triangles $I A B$ and $I C D$ are also congruent with the same orientation. Then we conclude that quadrilaterals $P B I A$ and $Q D I C$ are congruent, which implies $I P=I Q$. Then $E I$ is the perpendicular bisector of $P Q$ and, therefore, $E I \perp$ $P Q \Longleftrightarrow E I \perp B C$. + +Comment. Even though all three solutions used the point $I$, there are solutions that do not need it. We present an outline of such a solution: if $J$ is the incenter of $\triangle Q C D$ (with $P$ and $Q$ as defined in Solution 3), then a simple angle chasing shows that triangles $C J D$ and $B H C$ are congruent. Then if $S$ is the projection of $J$ onto side $C D$ and $T$ is the orthogonal projection of $H$ onto side $B C$, one can verify that + +$$ +Q T=Q C+C T=Q C+D S=Q C+\frac{C D+D Q-Q C}{2}=\frac{P B+B C+Q C}{2}=\frac{P Q}{2}, +$$ + +so $T$ is the midpoint of $P Q$, and $E, H$ and $T$ all lie on the perpendicular bisector of $P Q$. + +G2. Let $R$ and $S$ be distinct points on circle $\Omega$, and let $t$ denote the tangent line to $\Omega$ at $R$. Point $R^{\prime}$ is the reflection of $R$ with respect to $S$. A point $I$ is chosen on the smaller arc $R S$ of $\Omega$ so that the circumcircle $\Gamma$ of triangle $I S R^{\prime}$ intersects $t$ at two different points. Denote by $A$ the common point of $\Gamma$ and $t$ that is closest to $R$. Line $A I$ meets $\Omega$ again at $J$. Show that $J R^{\prime}$ is tangent to $\Gamma$. +(Luxembourg) +Solution 1. In the circles $\Omega$ and $\Gamma$ we have $\angle J R S=\angle J I S=\angle A R^{\prime} S$. On the other hand, since $R A$ is tangent to $\Omega$, we get $\angle S J R=\angle S R A$. So the triangles $A R R^{\prime}$ and $S J R$ are similar, and + +$$ +\frac{R^{\prime} R}{R J}=\frac{A R^{\prime}}{S R}=\frac{A R^{\prime}}{S R^{\prime}} +$$ + +The last relation, together with $\angle A R^{\prime} S=\angle J R R^{\prime}$, yields $\triangle A S R^{\prime} \sim \triangle R^{\prime} J R$, hence $\angle S A R^{\prime}=\angle R R^{\prime} J$. It follows that $J R^{\prime}$ is tangent to $\Gamma$ at $R^{\prime}$. +![](https://cdn.mathpix.com/cropped/2024_11_18_ff08e3c077fa2f8c16b2g-60.jpg?height=880&width=757&top_left_y=1082&top_left_x=244) + +Solution 1 +![](https://cdn.mathpix.com/cropped/2024_11_18_ff08e3c077fa2f8c16b2g-60.jpg?height=1029&width=758&top_left_y=933&top_left_x=1066) + +Solution 2 + +Solution 2. As in Solution 1, we notice that $\angle J R S=\angle J I S=\angle A R^{\prime} S$, so we have $R J \| A R^{\prime}$. +Let $A^{\prime}$ be the reflection of $A$ about $S$; then $A R A^{\prime} R^{\prime}$ is a parallelogram with center $S$, and hence the point $J$ lies on the line $R A^{\prime}$. + +From $\angle S R^{\prime} A^{\prime}=\angle S R A=\angle S J R$ we get that the points $S, J, A^{\prime}, R^{\prime}$ are concyclic. This proves that $\angle S R^{\prime} J=\angle S A^{\prime} J=\angle S A^{\prime} R=\angle S A R^{\prime}$, so $J R^{\prime}$ is tangent to $\Gamma$ at $R^{\prime}$. + +G3. Let $O$ be the circumcenter of an acute scalene triangle $A B C$. Line $O A$ intersects the altitudes of $A B C$ through $B$ and $C$ at $P$ and $Q$, respectively. The altitudes meet at $H$. Prove that the circumcenter of triangle $P Q H$ lies on a median of triangle $A B C$. +(Ukraine) +Solution. Suppose, without loss of generality, that $A B
0$. + +Denote by $h$ the homothety mapping $\mathcal{C}$ to $\mathcal{A}$. We need now to prove that the vertices of $\mathcal{C}$ which stay in $\mathcal{B}$ after applying $h$ are consecutive. If $X \in \mathcal{B}$, the claim is easy. Indeed, if $k<1$, then the vertices of $\mathcal{A}$ lie on the segments of the form $X C$ ( $C$ being a vertex of $\mathcal{C}$ ) which lie in $\mathcal{B}$. If $k>1$, then the vertices of $\mathcal{A}$ lie on the extensions of such segments $X C$ beyond $C$, and almost all these extensions lie outside $\mathcal{B}$. The exceptions may occur only in case when $X$ lies on the boundary of $\mathcal{B}$, and they may cause one or two vertices of $\mathcal{A}$ stay on the boundary of $\mathcal{B}$. But even in this case those vertices are still consecutive. + +So, from now on we assume that $X \notin \mathcal{B}$. +Now, there are two vertices $B_{\mathrm{T}}$ and $\mathcal{B}_{\mathrm{B}}$ of $\mathcal{B}$ such that $\mathcal{B}$ is contained in the angle $\angle B_{\mathrm{T}} X B_{\mathrm{B}}$; if there are several options, say, for $B_{\mathrm{T}}$, then we choose the farthest one from $X$ if $k>1$, and the nearest one if $k<1$. For the visualization purposes, we refer the plane to Cartesian coordinates so that the $y$-axis is co-directional with $\overrightarrow{B_{\mathrm{B}} B_{\mathrm{T}}}$, and $X$ lies to the left of the line $B_{\mathrm{T}} B_{\mathrm{B}}$. Again, the perimeter of $\mathcal{B}$ is split by $B_{\mathrm{T}}$ and $B_{\mathrm{B}}$ into the right part $\mathcal{B}_{\mathrm{R}}$ and the left part $\mathcal{B}_{\mathrm{L}}$, and the set of vertices of $\mathcal{C}$ is split into two subsets $\mathcal{C}_{\mathrm{R}} \subset \mathcal{B}_{\mathrm{R}}$ and $\mathcal{C}_{\mathrm{L}} \subset \mathcal{B}_{\mathrm{L}}$. +![](https://cdn.mathpix.com/cropped/2024_11_18_ff08e3c077fa2f8c16b2g-67.jpg?height=572&width=546&top_left_y=959&top_left_x=281) + +Case 2, $X$ inside $\mathcal{B}$ +![](https://cdn.mathpix.com/cropped/2024_11_18_ff08e3c077fa2f8c16b2g-67.jpg?height=623&width=906&top_left_y=908&top_left_x=866) + +Subcase 2.1: $k>1$ + +Subcase 2.1: $k>1$. +In this subcase, all points from $\mathcal{B}_{\mathrm{R}}$ (and hence from $\mathcal{C}_{\mathrm{R}}$ ) move out from $\mathcal{B}$ under $h$, because they are the farthest points of $\mathcal{B}$ on the corresponding rays emanated from $X$. It remains to prove that the vertices of $\mathcal{C}_{\mathrm{L}}$ which stay in $\mathcal{B}$ under $h$ are consecutive. + +Again, let $C_{1}, C_{2}, C_{3}$ be three vertices in $\mathcal{C}_{\mathrm{L}}$ such that $C_{2}$ is between $C_{1}$ and $C_{3}$, and $h\left(C_{1}\right)$ and $h\left(C_{3}\right)$ lie in $\mathcal{B}$. Let $A_{i}=h\left(C_{i}\right)$. Then the ray $X C_{2}$ crosses the segment $C_{1} C_{3}$ beyond $C_{2}$, so this ray crosses $A_{1} A_{3}$ beyond $A_{2}$; this implies that $A_{2}$ lies in the triangle $A_{1} C_{2} A_{3}$, which is contained in $\mathcal{B}$. +![](https://cdn.mathpix.com/cropped/2024_11_18_ff08e3c077fa2f8c16b2g-67.jpg?height=515&width=909&top_left_y=2021&top_left_x=576) + +Subcase 2.2: $k<1$ +Subcase 2.2: $k<1$. +This case is completely similar to the previous one. All points from $\mathcal{B}_{\mathrm{L}}$ (and hence from $\mathcal{C}_{\mathrm{L}}$ move out from $\mathcal{B}$ under $h$, because they are the nearest points of $\mathcal{B}$ on the corresponding +rays emanated from $X$. Assume that $C_{1}, C_{2}$, and $C_{3}$ are three vertices in $\mathcal{C}_{\mathrm{R}}$ such that $C_{2}$ lies between $C_{1}$ and $C_{3}$, and $h\left(C_{1}\right)$ and $h\left(C_{3}\right)$ lie in $\mathcal{B}$; let $A_{i}=h\left(C_{i}\right)$. Then $A_{2}$ lies on the segment $X C_{2}$, and the segments $X A_{2}$ and $A_{1} A_{3}$ cross each other. Thus $A_{2}$ lies in the triangle $A_{1} C_{2} A_{3}$, which is contained in $\mathcal{B}$. + +Comment 1. In fact, Case 1 can be reduced to Case 2 via the following argument. +Assume that $\mathcal{A}$ and $\mathcal{C}$ are congruent. Apply to $\mathcal{A}$ a homothety centered at $O_{B}$ with a factor slightly smaller than 1 to obtain a polygon $\mathcal{A}^{\prime}$. With appropriately chosen factor, the vertices of $\mathcal{A}$ which were outside $/$ inside $\mathcal{B}$ stay outside/inside it, so it suffices to prove our claim for $\mathcal{A}^{\prime}$ instead of $\mathcal{A}$. And now, the polygon $\mathcal{A}^{\prime}$ is a homothetic image of $\mathcal{C}$, so the arguments from Case 2 apply. + +Comment 2. After the polygon $\mathcal{C}$ has been found, the rest of the solution uses only the convexity of the polygons, instead of regularity. Thus, it proves a more general statement: + +Assume that $\mathcal{A}, \mathcal{B}$, and $\mathcal{C}$ are three convex polygons in the plane such that $\mathcal{C}$ is inscribed into $\mathcal{B}$, and $\mathcal{A}$ can be obtained from it via either translation or positive homothety. Then the vertices of $\mathcal{A}$ that lie inside $\mathcal{B}$ or on its boundary are consecutive. +Solution 2. Let $O_{A}$ and $O_{B}$ be the centers of $\mathcal{A}$ and $\mathcal{B}$, respectively. Denote $[n]=\{1,2, \ldots, n\}$. +We start with introducing appropriate enumerations and notations. Enumerate the sidelines of $\mathcal{B}$ clockwise as $\ell_{1}, \ell_{2}, \ldots, \ell_{n}$. Denote by $\mathcal{H}_{i}$ the half-plane of $\ell_{i}$ that contains $\mathcal{B}$ ( $\mathcal{H}_{i}$ is assumed to contain $\left.\ell_{i}\right)$; by $B_{i}$ the midpoint of the side belonging to $\ell_{i}$; and finally denote $\overrightarrow{b_{i}}=\overrightarrow{B_{i} O_{B}}$. (As usual, the numbering is cyclic modulo $n$, so $\ell_{n+i}=\ell_{i}$ etc.) + +Now, choose a vertex $A_{1}$ of $\mathcal{A}$ such that the vector $\overrightarrow{O_{A} A_{1}}$ points "mostly outside $\mathcal{H}_{1}$ "; strictly speaking, this means that the scalar product $\left\langle\overrightarrow{O_{A} A_{1}}, \overrightarrow{b_{1}}\right\rangle$ is minimal. Starting from $A_{1}$, enumerate the vertices of $\mathcal{A}$ clockwise as $A_{1}, A_{2}, \ldots, A_{n}$; by the rotational symmetry, the choice of $A_{1}$ yields that the vector $\overrightarrow{O_{A} A_{i}}$ points "mostly outside $\mathcal{H}_{i}$ ", i.e., + +$$ +\left\langle\overrightarrow{O_{A} A_{i}}, \overrightarrow{b_{i}}\right\rangle=\min _{j \in[n]}\left\langle\overrightarrow{O_{A} A_{j}}, \overrightarrow{b_{i}}\right\rangle +$$ + +![](https://cdn.mathpix.com/cropped/2024_11_18_ff08e3c077fa2f8c16b2g-68.jpg?height=560&width=898&top_left_y=1516&top_left_x=579) + +Enumerations and notations +We intend to reformulate the problem in more combinatorial terms, for which purpose we introduce the following notion. Say that a subset $I \subseteq[n]$ is connected if the elements of this set are consecutive in the cyclic order (in other words, if we join each $i$ with $i+1 \bmod n$ by an edge, this subset is connected in the usual graph sense). Clearly, the union of two connected subsets sharing at least one element is connected too. Next, for any half-plane $\mathcal{H}$ the indices of vertices of, say, $\mathcal{A}$ that lie in $\mathcal{H}$ form a connected set. + +To access the problem, we denote + +$$ +M=\left\{j \in[n]: A_{j} \notin \mathcal{B}\right\}, \quad M_{i}=\left\{j \in[n]: A_{j} \notin \mathcal{H}_{i}\right\} \quad \text { for } i \in[n] +$$ + +We need to prove that $[n] \backslash M$ is connected, which is equivalent to $M$ being connected. On the other hand, since $\mathcal{B}=\bigcap_{i \in[n]} \mathcal{H}_{i}$, we have $M=\bigcup_{i \in[n]} M_{i}$, where the sets $M_{i}$ are easier to investigate. We will utilize the following properties of these sets; the first one holds by the definition of $M_{i}$, along with the above remark. +![](https://cdn.mathpix.com/cropped/2024_11_18_ff08e3c077fa2f8c16b2g-69.jpg?height=529&width=1183&top_left_y=175&top_left_x=439) + +The sets $M_{i}$ + +Property 1: Each set $M_{i}$ is connected. +Property 2: If $M_{i}$ is nonempty, then $i \in M_{i}$. +Proof. Indeed, we have + +$$ +j \in M_{i} \Longleftrightarrow A_{j} \notin \mathcal{H}_{i} \Longleftrightarrow\left\langle\overrightarrow{B_{i} A_{j}}, \overrightarrow{b_{i}}\right\rangle<0 \Longleftrightarrow\left\langle\overrightarrow{O_{A} A_{j}}, \overrightarrow{b_{i}}\right\rangle<\left\langle\overrightarrow{O_{A} B_{i}}, \overrightarrow{b_{i}}\right\rangle +$$ + +The right-hand part of the last inequality does not depend on $j$. Therefore, if some $j$ lies in $M_{i}$, then by (1) so does $i$. + +In view of Property 2, it is useful to define the set + +$$ +M^{\prime}=\left\{i \in[n]: i \in M_{i}\right\}=\left\{i \in[n]: M_{i} \neq \varnothing\right\} . +$$ + +Property 3: The set $M^{\prime}$ is connected. +Proof. To prove this property, we proceed on with the investigation started in (2) to write + +$$ +i \in M^{\prime} \Longleftrightarrow A_{i} \in M_{i} \Longleftrightarrow\left\langle\overrightarrow{B_{i}} \overrightarrow{A_{i}}, \overrightarrow{b_{i}}\right\rangle<0 \Longleftrightarrow\left\langle\overrightarrow{O_{B} O_{A}}, \overrightarrow{b_{i}}\right\rangle<\left\langle\overrightarrow{O_{B} B_{i}}, \overrightarrow{b_{i}}\right\rangle+\left\langle\overrightarrow{A_{i} O_{A}}, \overrightarrow{b_{i}}\right\rangle +$$ + +The right-hand part of the obtained inequality does not depend on $i$, due to the rotational symmetry; denote its constant value by $\mu$. Thus, $i \in M^{\prime}$ if and only if $\left\langle\overrightarrow{O_{B} O_{A}}, \overrightarrow{b_{i}}\right\rangle<\mu$. This condition is in turn equivalent to the fact that $B_{i}$ lies in a certain (open) half-plane whose boundary line is orthogonal to $O_{B} O_{A}$; thus, it defines a connected set. + +Now we can finish the solution. Since $M^{\prime} \subseteq M$, we have + +$$ +M=\bigcup_{i \in[n]} M_{i}=M^{\prime} \cup \bigcup_{i \in[n]} M_{i}, +$$ + +so $M$ can be obtained from $M^{\prime}$ by adding all the sets $M_{i}$ one by one. All these sets are connected, and each nonempty $M_{i}$ contains an element of $M^{\prime}$ (namely, $i$ ). Thus their union is also connected. + +Comment 3. Here we present a way in which one can come up with a solution like the one above. +Assume, for sake of simplicity, that $O_{A}$ lies inside $\mathcal{B}$. Let us first put onto the plane a very small regular $n$-gon $\mathcal{A}^{\prime}$ centered at $O_{A}$ and aligned with $\mathcal{A}$; all its vertices lie inside $\mathcal{B}$. Now we start blowing it up, looking at the order in which the vertices leave $\mathcal{B}$. To go out of $\mathcal{B}$, a vertex should cross a certain side of $\mathcal{B}$ (which is hard to describe), or, equivalently, to cross at least one sideline of $\mathcal{B}$ - and this event is easier to describe. Indeed, the first vertex of $\mathcal{A}^{\prime}$ to cross $\ell_{i}$ is the vertex $A_{i}^{\prime}$ (corresponding to $A_{i}$ in $\mathcal{A}$ ); more generally, the vertices $A_{j}^{\prime}$ cross $\ell_{i}$ in such an order that the scalar product $\left\langle\overrightarrow{O_{A}} \overrightarrow{A_{j}}, \overrightarrow{b_{i}}\right\rangle$ does not increase. For different indices $i$, these orders are just cyclic shifts of each other; and this provides some intuition for the notions and claims from Solution 2. + +G7. A convex quadrilateral $A B C D$ has an inscribed circle with center $I$. Let $I_{a}, I_{b}, I_{c}$, and $I_{d}$ be the incenters of the triangles $D A B, A B C, B C D$, and $C D A$, respectively. Suppose that the common external tangents of the circles $A I_{b} I_{d}$ and $C I_{b} I_{d}$ meet at $X$, and the common external tangents of the circles $B I_{a} I_{c}$ and $D I_{a} I_{c}$ meet at $Y$. Prove that $\angle X I Y=90^{\circ}$. +(Kazakhstan) +Solution. Denote by $\omega_{a}, \omega_{b}, \omega_{c}$ and $\omega_{d}$ the circles $A I_{b} I_{d}, B I_{a} I_{c}, C I_{b} I_{d}$, and $D I_{a} I_{c}$, let their centers be $O_{a}, O_{b}, O_{c}$ and $O_{d}$, and let their radii be $r_{a}, r_{b}, r_{c}$ and $r_{d}$, respectively. +Claim 1. $I_{b} I_{d} \perp A C$ and $I_{a} I_{c} \perp B D$. +Proof. Let the incircles of triangles $A B C$ and $A C D$ be tangent to the line $A C$ at $T$ and $T^{\prime}$, respectively. (See the figure to the left.) We have $A T=\frac{A B+A C-B C}{2}$ in triangle $A B C, A T^{\prime}=$ $\frac{A D+A C-C D}{2}$ in triangle $A C D$, and $A B-B C=A D-C D$ in quadrilateral $A B C D$, so + +$$ +A T=\frac{A C+A B-B C}{2}=\frac{A C+A D-C D}{2}=A T^{\prime} +$$ + +This shows $T=T^{\prime}$. As an immediate consequence, $I_{b} I_{d} \perp A C$. +The second statement can be shown analogously. +![](https://cdn.mathpix.com/cropped/2024_11_18_ff08e3c077fa2f8c16b2g-70.jpg?height=726&width=1729&top_left_y=1076&top_left_x=168) + +Claim 2. The points $O_{a}, O_{b}, O_{c}$ and $O_{d}$ lie on the lines $A I, B I, C I$ and $D I$, respectively. Proof. By symmetry it suffices to prove the claim for $O_{a}$. (See the figure to the right above.) + +Notice first that the incircles of triangles $A B C$ and $A C D$ can be obtained from the incircle of the quadrilateral $A B C D$ with homothety centers $B$ and $D$, respectively, and homothety factors less than 1 , therefore the points $I_{b}$ and $I_{d}$ lie on the line segments $B I$ and $D I$, respectively. + +As is well-known, in every triangle the altitude and the diameter of the circumcircle starting from the same vertex are symmetric about the angle bisector. By Claim 1, in triangle $A I_{d} I_{b}$, the segment $A T$ is the altitude starting from $A$. Since the foot $T$ lies inside the segment $I_{b} I_{d}$, the circumcenter $O_{a}$ of triangle $A I_{d} I_{b}$ lies in the angle domain $I_{b} A I_{d}$ in such a way that $\angle I_{b} A T=\angle O_{a} A I_{d}$. The points $I_{b}$ and $I_{d}$ are the incenters of triangles $A B C$ and $A C D$, so the lines $A I_{b}$ and $A I_{d}$ bisect the angles $\angle B A C$ and $\angle C A D$, respectively. Then + +$$ +\angle O_{a} A D=\angle O_{a} A I_{d}+\angle I_{d} A D=\angle I_{b} A T+\angle I_{d} A D=\frac{1}{2} \angle B A C+\frac{1}{2} \angle C A D=\frac{1}{2} \angle B A D, +$$ + +so $O_{a}$ lies on the angle bisector of $\angle B A D$, that is, on line $A I$. +The point $X$ is the external similitude center of $\omega_{a}$ and $\omega_{c}$; let $U$ be their internal similitude center. The points $O_{a}$ and $O_{c}$ lie on the perpendicular bisector of the common chord $I_{b} I_{d}$ of $\omega_{a}$ and $\omega_{c}$, and the two similitude centers $X$ and $U$ lie on the same line; by Claim 2, that line is parallel to $A C$. +![](https://cdn.mathpix.com/cropped/2024_11_18_ff08e3c077fa2f8c16b2g-71.jpg?height=1149&width=1529&top_left_y=179&top_left_x=269) + +From the similarity of the circles $\omega_{a}$ and $\omega_{c}$, from $O_{a} I_{b}=O_{a} I_{d}=O_{a} A=r_{a}$ and $O_{c} I_{b}=$ $O_{c} I_{d}=O_{c} C=r_{c}$, and from $A C \| O_{a} O_{c}$ we can see that + +$$ +\frac{O_{a} X}{O_{c} X}=\frac{O_{a} U}{O_{c} U}=\frac{r_{a}}{r_{c}}=\frac{O_{a} I_{b}}{O_{c} I_{b}}=\frac{O_{a} I_{d}}{O_{c} I_{d}}=\frac{O_{a} A}{O_{c} C}=\frac{O_{a} I}{O_{c} I} +$$ + +So the points $X, U, I_{b}, I_{d}, I$ lie on the Apollonius circle of the points $O_{a}, O_{c}$ with ratio $r_{a}: r_{c}$. In this Apollonius circle $X U$ is a diameter, and the lines $I U$ and $I X$ are respectively the internal and external bisectors of $\angle O_{a} I O_{c}=\angle A I C$, according to the angle bisector theorem. Moreover, in the Apollonius circle the diameter $U X$ is the perpendicular bisector of $I_{b} I_{d}$, so the lines $I X$ and $I U$ are the internal and external bisectors of $\angle I_{b} I I_{d}=\angle B I D$, respectively. + +Repeating the same argument for the points $B, D$ instead of $A, C$, we get that the line $I Y$ is the internal bisector of $\angle A I C$ and the external bisector of $\angle B I D$. Therefore, the lines $I X$ and $I Y$ respectively are the internal and external bisectors of $\angle B I D$, so they are perpendicular. + +Comment. In fact the points $O_{a}, O_{b}, O_{c}$ and $O_{d}$ lie on the line segments $A I, B I, C I$ and $D I$, respectively. For the point $O_{a}$ this can be shown for example by $\angle I_{d} O_{a} A+\angle A O_{a} I_{b}=\left(180^{\circ}-\right.$ $\left.2 \angle O_{a} A I_{d}\right)+\left(180^{\circ}-2 \angle I_{b} A O_{a}\right)=360^{\circ}-\angle B A D=\angle A D I+\angle D I A+\angle A I B+\angle I B A>\angle I_{d} I A+\angle A I I_{b}$. + +The solution also shows that the line $I Y$ passes through the point $U$, and analogously, $I X$ passes through the internal similitude center of $\omega_{b}$ and $\omega_{d}$. + +G8. There are 2017 mutually external circles drawn on a blackboard, such that no two are tangent and no three share a common tangent. A tangent segment is a line segment that is a common tangent to two circles, starting at one tangent point and ending at the other one. Luciano is drawing tangent segments on the blackboard, one at a time, so that no tangent segment intersects any other circles or previously drawn tangent segments. Luciano keeps drawing tangent segments until no more can be drawn. Find all possible numbers of tangent segments when he stops drawing. +(Australia) +Answer: If there were $n$ circles, there would always be exactly $3(n-1)$ segments; so the only possible answer is $3 \cdot 2017-3=6048$. +Solution 1. First, consider a particular arrangement of circles $C_{1}, C_{2}, \ldots, C_{n}$ where all the centers are aligned and each $C_{i}$ is eclipsed from the other circles by its neighbors - for example, taking $C_{i}$ with center $\left(i^{2}, 0\right)$ and radius $i / 2$ works. Then the only tangent segments that can be drawn are between adjacent circles $C_{i}$ and $C_{i+1}$, and exactly three segments can be drawn for each pair. So Luciano will draw exactly $3(n-1)$ segments in this case. +![](https://cdn.mathpix.com/cropped/2024_11_18_ff08e3c077fa2f8c16b2g-72.jpg?height=258&width=1335&top_left_y=1059&top_left_x=355) + +For the general case, start from a final configuration (that is, an arrangement of circles and segments in which no further segments can be drawn). The idea of the solution is to continuously resize and move the circles around the plane, one by one (in particular, making sure we never have 4 circles with a common tangent line), and show that the number of segments drawn remains constant as the picture changes. This way, we can reduce any circle/segment configuration to the particular one mentioned above, and the final number of segments must remain at $3 n-3$. + +Some preliminary considerations: look at all possible tangent segments joining any two circles. A segment that is tangent to a circle $A$ can do so in two possible orientations - it may come out of $A$ in clockwise or counterclockwise orientation. Two segments touching the same circle with the same orientation will never intersect each other. Each pair $(A, B)$ of circles has 4 choices of tangent segments, which can be identified by their orientations - for example, $(A+, B-)$ would be the segment which comes out of $A$ in clockwise orientation and comes out of $B$ in counterclockwise orientation. In total, we have $2 n(n-1)$ possible segments, disregarding intersections. + +Now we pick a circle $C$ and start to continuously move and resize it, maintaining all existing tangent segments according to their identifications, including those involving $C$. We can keep our choice of tangent segments until the configuration reaches a transition. We lose nothing if we assume that $C$ is kept at least $\varepsilon$ units away from any other circle, where $\varepsilon$ is a positive, fixed constant; therefore at a transition either: (1) a currently drawn tangent segment $t$ suddenly becomes obstructed; or (2) a currently absent tangent segment $t$ suddenly becomes unobstructed and available. +Claim. A transition can only occur when three circles $C_{1}, C_{2}, C_{3}$ are tangent to a common line $\ell$ containing $t$, in a way such that the three tangent segments lying on $\ell$ (joining the three circles pairwise) are not obstructed by any other circles or tangent segments (other than $C_{1}, C_{2}, C_{3}$ ). Proof. Since (2) is effectively the reverse of (1), it suffices to prove the claim for (1). Suppose $t$ has suddenly become obstructed, and let us consider two cases. + +Case 1: $t$ becomes obstructed by a circle +![](https://cdn.mathpix.com/cropped/2024_11_18_ff08e3c077fa2f8c16b2g-73.jpg?height=318&width=1437&top_left_y=292&top_left_x=315) + +Then the new circle becomes the third circle tangent to $\ell$, and no other circles or tangent segments are obstructing $t$. + +## Case 2: $t$ becomes obstructed by another tangent segment $t^{\prime}$ + +When two segments $t$ and $t^{\prime}$ first intersect each other, they must do so at a vertex of one of them. But if a vertex of $t^{\prime}$ first crossed an interior point of $t$, the circle associated to this vertex was already blocking $t$ (absurd), or is about to (we already took care of this in case 1). So we only have to analyze the possibility of $t$ and $t^{\prime}$ suddenly having a common vertex. However, if that happens, this vertex must belong to a single circle (remember we are keeping different circles at least $\varepsilon$ units apart from each other throughout the moving/resizing process), and therefore they must have different orientations with respect to that circle. +![](https://cdn.mathpix.com/cropped/2024_11_18_ff08e3c077fa2f8c16b2g-73.jpg?height=324&width=1351&top_left_y=1226&top_left_x=358) + +Thus, at the transition moment, both $t$ and $t^{\prime}$ are tangent to the same circle at a common point, that is, they must be on the same line $\ell$ and hence we again have three circles simultaneously tangent to $\ell$. Also no other circles or tangent segments are obstructing $t$ or $t^{\prime}$ (otherwise, they would have disappeared before this transition). + +Next, we focus on the maximality of a configuration immediately before and after a transition, where three circles share a common tangent line $\ell$. Let the three circles be $C_{1}, C_{2}, C_{3}$, ordered by their tangent points. The only possibly affected segments are the ones lying on $\ell$, namely $t_{12}, t_{23}$ and $t_{13}$. Since $C_{2}$ is in the middle, $t_{12}$ and $t_{23}$ must have different orientations with respect to $C_{2}$. For $C_{1}, t_{12}$ and $t_{13}$ must have the same orientation, while for $C_{3}, t_{13}$ and $t_{23}$ must have the same orientation. The figure below summarizes the situation, showing alternative positions for $C_{1}$ (namely, $C_{1}$ and $\left.C_{1}^{\prime}\right)$ and for $C_{3}\left(C_{3}\right.$ and $\left.C_{3}^{\prime}\right)$. +![](https://cdn.mathpix.com/cropped/2024_11_18_ff08e3c077fa2f8c16b2g-73.jpg?height=567&width=1009&top_left_y=2212&top_left_x=523) + +Now perturb the diagram slightly so the three circles no longer have a common tangent, while preserving the definition of $t_{12}, t_{23}$ and $t_{13}$ according to their identifications. First note that no other circles or tangent segments can obstruct any of these segments. Also recall that tangent segments joining the same circle at the same orientation will never obstruct each other. + +The availability of the tangent segments can now be checked using simple diagrams. +Case 1: $t_{13}$ passes through $C_{2}$ +![](https://cdn.mathpix.com/cropped/2024_11_18_ff08e3c077fa2f8c16b2g-74.jpg?height=549&width=1006&top_left_y=591&top_left_x=528) + +In this case, $t_{13}$ is not available, but both $t_{12}$ and $t_{23}$ are. +Case 2: $t_{13}$ does not pass through $C_{2}$ +![](https://cdn.mathpix.com/cropped/2024_11_18_ff08e3c077fa2f8c16b2g-74.jpg?height=536&width=1017&top_left_y=1388&top_left_x=522) + +Now $t_{13}$ is available, but $t_{12}$ and $t_{23}$ obstruct each other, so only one can be drawn. +In any case, exactly 2 out of these 3 segments can be drawn. Thus the maximal number of segments remains constant as we move or resize the circles, and we are done. + +Solution 2. First note that all tangent segments lying on the boundary of the convex hull of the circles are always drawn since they do not intersect anything else. Now in the final picture, aside from the $n$ circles, the blackboard is divided into regions. We can consider the picture as a plane (multi-)graph $G$ in which the circles are the vertices and the tangent segments are the edges. The idea of this solution is to find a relation between the number of edges and the number of regions in $G$; then, once we prove that $G$ is connected, we can use Euler's formula to finish the problem. + +The boundary of each region consists of 1 or more (for now) simple closed curves, each made of arcs and tangent segments. The segment and the arc might meet smoothly (as in $S_{i}$, $i=1,2, \ldots, 6$ in the figure below) or not (as in $P_{1}, P_{2}, P_{3}, P_{4}$; call such points sharp corners of the boundary). In other words, if a person walks along the border, her direction would suddenly turn an angle of $\pi$ at a sharp corner. +![](https://cdn.mathpix.com/cropped/2024_11_18_ff08e3c077fa2f8c16b2g-75.jpg?height=626&width=577&top_left_y=178&top_left_x=748) + +Claim 1. The outer boundary $B_{1}$ of any internal region has at least 3 sharp corners. +Proof. Let a person walk one lap along $B_{1}$ in the counterclockwise orientation. As she does so, she will turn clockwise as she moves along the circle arcs, and not turn at all when moving along the lines. On the other hand, her total rotation after one lap is $2 \pi$ in the counterclockwise direction! Where could she be turning counterclockwise? She can only do so at sharp corners, and, even then, she turns only an angle of $\pi$ there. But two sharp corners are not enough, since at least one arc must be present-so she must have gone through at least 3 sharp corners. + +Claim 2. Each internal region is simply connected, that is, has only one boundary curve. +Proof. Suppose, by contradiction, that some region has an outer boundary $B_{1}$ and inner boundaries $B_{2}, B_{3}, \ldots, B_{m}(m \geqslant 2)$. Let $P_{1}$ be one of the sharp corners of $B_{1}$. + +Now consider a car starting at $P_{1}$ and traveling counterclockwise along $B_{1}$. It starts in reverse, i.e., it is initially facing the corner $P_{1}$. Due to the tangent conditions, the car may travel in a way so that its orientation only changes when it is moving along an arc. In particular, this means the car will sometimes travel forward. For example, if the car approaches a sharp corner when driving in reverse, it would continue travel forward after the corner, instead of making an immediate half-turn. This way, the orientation of the car only changes in a clockwise direction since the car always travels clockwise around each arc. + +Now imagine there is a laser pointer at the front of the car, pointing directly ahead. Initially, the laser endpoint hits $P_{1}$, but, as soon as the car hits an arc, the endpoint moves clockwise around $B_{1}$. In fact, the laser endpoint must move continuously along $B_{1}$ ! Indeed, if the endpoint ever jumped (within $B_{1}$, or from $B_{1}$ to one of the inner boundaries), at the moment of the jump the interrupted laser would be a drawable tangent segment that Luciano missed (see figure below for an example). +![](https://cdn.mathpix.com/cropped/2024_11_18_ff08e3c077fa2f8c16b2g-75.jpg?height=637&width=587&top_left_y=2140&top_left_x=740) + +Now, let $P_{2}$ and $P_{3}$ be the next two sharp corners the car goes through, after $P_{1}$ (the previous lemma assures their existence). At $P_{2}$ the car starts moving forward, and at $P_{3}$ it will start to move in reverse again. So, at $P_{3}$, the laser endpoint is at $P_{3}$ itself. So while the car moved counterclockwise between $P_{1}$ and $P_{3}$, the laser endpoint moved clockwise between $P_{1}$ and $P_{3}$. That means the laser beam itself scanned the whole region within $B_{1}$, and it should have crossed some of the inner boundaries. + +Claim 3. Each region has exactly 3 sharp corners. +Proof. Consider again the car of the previous claim, with its laser still firmly attached to its front, traveling the same way as before and going through the same consecutive sharp corners $P_{1}, P_{2}$ and $P_{3}$. As we have seen, as the car goes counterclockwise from $P_{1}$ to $P_{3}$, the laser endpoint goes clockwise from $P_{1}$ to $P_{3}$, so together they cover the whole boundary. If there were a fourth sharp corner $P_{4}$, at some moment the laser endpoint would pass through it. But, since $P_{4}$ is a sharp corner, this means the car must be on the extension of a tangent segment going through $P_{4}$. Since the car is not on that segment itself (the car never goes through $P_{4}$ ), we would have 3 circles with a common tangent line, which is not allowed. +![](https://cdn.mathpix.com/cropped/2024_11_18_ff08e3c077fa2f8c16b2g-76.jpg?height=538&width=792&top_left_y=1007&top_left_x=643) + +We are now ready to finish the solution. Let $r$ be the number of internal regions, and $s$ be the number of tangent segments. Since each tangent segment contributes exactly 2 sharp corners to the diagram, and each region has exactly 3 sharp corners, we must have $2 s=3 r$. Since the graph corresponding to the diagram is connected, we can use Euler's formula $n-s+r=1$ and find $s=3 n-3$ and $r=2 n-2$. + +## Number Theory + +N1. The sequence $a_{0}, a_{1}, a_{2}, \ldots$ of positive integers satisfies + +$$ +a_{n+1}=\left\{\begin{array}{ll} +\sqrt{a_{n}}, & \text { if } \sqrt{a_{n}} \text { is an integer } \\ +a_{n}+3, & \text { otherwise } +\end{array} \quad \text { for every } n \geqslant 0\right. +$$ + +Determine all values of $a_{0}>1$ for which there is at least one number $a$ such that $a_{n}=a$ for infinitely many values of $n$. +(South Africa) +Answer: All positive multiples of 3. +Solution. Since the value of $a_{n+1}$ only depends on the value of $a_{n}$, if $a_{n}=a_{m}$ for two different indices $n$ and $m$, then the sequence is eventually periodic. So we look for the values of $a_{0}$ for which the sequence is eventually periodic. +Claim 1. If $a_{n} \equiv-1(\bmod 3)$, then, for all $m>n, a_{m}$ is not a perfect square. It follows that the sequence is eventually strictly increasing, so it is not eventually periodic. +Proof. A square cannot be congruent to -1 modulo 3 , so $a_{n} \equiv-1(\bmod 3)$ implies that $a_{n}$ is not a square, therefore $a_{n+1}=a_{n}+3>a_{n}$. As a consequence, $a_{n+1} \equiv a_{n} \equiv-1(\bmod 3)$, so $a_{n+1}$ is not a square either. By repeating the argument, we prove that, from $a_{n}$ on, all terms of the sequence are not perfect squares and are greater than their predecessors, which completes the proof. +Claim 2. If $a_{n} \not \equiv-1(\bmod 3)$ and $a_{n}>9$ then there is an index $m>n$ such that $a_{m}9, t$ is at least 3. The first square in the sequence $a_{n}, a_{n}+3, a_{n}+6, \ldots$ will be $(t+1)^{2},(t+2)^{2}$ or $(t+3)^{2}$, therefore there is an index $m>n$ such that $a_{m} \leqslant t+3n$ such that $a_{m}=3$. +Proof. First we notice that, by the definition of the sequence, a multiple of 3 is always followed by another multiple of 3 . If $a_{n} \in\{3,6,9\}$ the sequence will eventually follow the periodic pattern $3,6,9,3,6,9, \ldots$ If $a_{n}>9$, let $j$ be an index such that $a_{j}$ is equal to the minimum value of the set $\left\{a_{n+1}, a_{n+2}, \ldots\right\}$. We must have $a_{j} \leqslant 9$, otherwise we could apply Claim 2 to $a_{j}$ and get a contradiction on the minimality hypothesis. It follows that $a_{j} \in\{3,6,9\}$, and the proof is complete. +Claim 4. If $a_{n} \equiv 1(\bmod 3)$, then there is an index $m>n$ such that $a_{m} \equiv-1(\bmod 3)$. +Proof. In the sequence, 4 is always followed by $2 \equiv-1(\bmod 3)$, so the claim is true for $a_{n}=4$. If $a_{n}=7$, the next terms will be $10,13,16,4,2, \ldots$ and the claim is also true. For $a_{n} \geqslant 10$, we again take an index $j>n$ such that $a_{j}$ is equal to the minimum value of the set $\left\{a_{n+1}, a_{n+2}, \ldots\right\}$, which by the definition of the sequence consists of non-multiples of 3 . Suppose $a_{j} \equiv 1(\bmod 3)$. Then we must have $a_{j} \leqslant 9$ by Claim 2 and the minimality of $a_{j}$. It follows that $a_{j} \in\{4,7\}$, so $a_{m}=2j$, contradicting the minimality of $a_{j}$. Therefore, we must have $a_{j} \equiv-1(\bmod 3)$. + +It follows from the previous claims that if $a_{0}$ is a multiple of 3 the sequence will eventually reach the periodic pattern $3,6,9,3,6,9, \ldots$; if $a_{0} \equiv-1(\bmod 3)$ the sequence will be strictly increasing; and if $a_{0} \equiv 1(\bmod 3)$ the sequence will be eventually strictly increasing. + +So the sequence will be eventually periodic if, and only if, $a_{0}$ is a multiple of 3 . + +N2. Let $p \geqslant 2$ be a prime number. Eduardo and Fernando play the following game making moves alternately: in each move, the current player chooses an index $i$ in the set $\{0,1, \ldots, p-1\}$ that was not chosen before by either of the two players and then chooses an element $a_{i}$ of the set $\{0,1,2,3,4,5,6,7,8,9\}$. Eduardo has the first move. The game ends after all the indices $i \in\{0,1, \ldots, p-1\}$ have been chosen. Then the following number is computed: + +$$ +M=a_{0}+10 \cdot a_{1}+\cdots+10^{p-1} \cdot a_{p-1}=\sum_{j=0}^{p-1} a_{j} \cdot 10^{j} +$$ + +The goal of Eduardo is to make the number $M$ divisible by $p$, and the goal of Fernando is to prevent this. + +Prove that Eduardo has a winning strategy. +(Morocco) +Solution. We say that a player makes the move $\left(i, a_{i}\right)$ if he chooses the index $i$ and then the element $a_{i}$ of the set $\{0,1,2,3,4,5,6,7,8,9\}$ in this move. + +If $p=2$ or $p=5$ then Eduardo chooses $i=0$ and $a_{0}=0$ in the first move, and wins, since, independently of the next moves, $M$ will be a multiple of 10 . + +Now assume that the prime number $p$ does not belong to $\{2,5\}$. Eduardo chooses $i=p-1$ and $a_{p-1}=0$ in the first move. By Fermat's Little Theorem, $\left(10^{(p-1) / 2}\right)^{2}=10^{p-1} \equiv 1(\bmod p)$, so $p \mid\left(10^{(p-1) / 2}\right)^{2}-1=\left(10^{(p-1) / 2}+1\right)\left(10^{(p-1) / 2}-1\right)$. Since $p$ is prime, either $p \mid 10^{(p-1) / 2}+1$ or $p \mid 10^{(p-1) / 2}-1$. Thus we have two cases: +Case a: $10^{(p-1) / 2} \equiv-1(\bmod p)$ +In this case, for each move $\left(i, a_{i}\right)$ of Fernando, Eduardo immediately makes the move $\left(j, a_{j}\right)=$ $\left(i+\frac{p-1}{2}, a_{i}\right)$, if $0 \leqslant i \leqslant \frac{p-3}{2}$, or $\left(j, a_{j}\right)=\left(i-\frac{p-1}{2}, a_{i}\right)$, if $\frac{p-1}{2} \leqslant i \leqslant p-2$. We will have $10^{j} \equiv-10^{i}$ $(\bmod p)$, and so $a_{j} \cdot 10^{j}=a_{i} \cdot 10^{j} \equiv-a_{i} \cdot 10^{i}(\bmod p)$. Notice that this move by Eduardo is always possible. Indeed, immediately before a move by Fernando, for any set of the type $\{r, r+(p-1) / 2\}$ with $0 \leqslant r \leqslant(p-3) / 2$, either no element of this set was chosen as an index by the players in the previous moves or else both elements of this set were chosen as indices by the players in the previous moves. Therefore, after each of his moves, Eduardo always makes the sum of the numbers $a_{k} \cdot 10^{k}$ corresponding to the already chosen pairs $\left(k, a_{k}\right)$ divisible by $p$, and thus wins the game. +Case b: $10^{(p-1) / 2} \equiv 1(\bmod p)$ +In this case, for each move $\left(i, a_{i}\right)$ of Fernando, Eduardo immediately makes the move $\left(j, a_{j}\right)=$ $\left(i+\frac{p-1}{2}, 9-a_{i}\right)$, if $0 \leqslant i \leqslant \frac{p-3}{2}$, or $\left(j, a_{j}\right)=\left(i-\frac{p-1}{2}, 9-a_{i}\right)$, if $\frac{p-1}{2} \leqslant i \leqslant p-2$. The same argument as above shows that Eduardo can always make such move. We will have $10^{j} \equiv 10^{i}$ $(\bmod p)$, and so $a_{j} \cdot 10^{j}+a_{i} \cdot 10^{i} \equiv\left(a_{i}+a_{j}\right) \cdot 10^{i}=9 \cdot 10^{i}(\bmod p)$. Therefore, at the end of the game, the sum of all terms $a_{k} \cdot 10^{k}$ will be congruent to + +$$ +\sum_{i=0}^{\frac{p-3}{2}} 9 \cdot 10^{i}=10^{(p-1) / 2}-1 \equiv 0 \quad(\bmod p) +$$ + +and Eduardo wins the game. + +N3. Determine all integers $n \geqslant 2$ with the following property: for any integers $a_{1}, a_{2}, \ldots, a_{n}$ whose sum is not divisible by $n$, there exists an index $1 \leqslant i \leqslant n$ such that none of the numbers + +$$ +a_{i}, a_{i}+a_{i+1}, \ldots, a_{i}+a_{i+1}+\cdots+a_{i+n-1} +$$ + +is divisible by $n$. (We let $a_{i}=a_{i-n}$ when $i>n$.) +(Thailand) +Answer: These integers are exactly the prime numbers. +Solution. Let us first show that, if $n=a b$, with $a, b \geqslant 2$ integers, then the property in the statement of the problem does not hold. Indeed, in this case, let $a_{k}=a$ for $1 \leqslant k \leqslant n-1$ and $a_{n}=0$. The sum $a_{1}+a_{2}+\cdots+a_{n}=a \cdot(n-1)$ is not divisible by $n$. Let $i$ with $1 \leqslant i \leqslant n$ be an arbitrary index. Taking $j=b$ if $1 \leqslant i \leqslant n-b$, and $j=b+1$ if $n-b0$ such that $10^{t} \equiv 1$ $(\bmod \mathrm{cm})$ for some integer $c \in C$, that is, the set of orders of 10 modulo cm . In other words, + +$$ +S(m)=\left\{\operatorname{ord}_{c m}(10): c \in C\right\} +$$ + +Since there are $4 \cdot 201+3=807$ numbers $c$ with $1 \leqslant c \leqslant 2017$ and $\operatorname{gcd}(c, 10)=1$, namely those such that $c \equiv 1,3,7,9(\bmod 10)$, + +$$ +|S(m)| \leqslant|C|=807 +$$ + +Now we find $m$ such that $|S(m)|=807$. Let + +$$ +P=\{1

0}$. We will show that $k=c$. +Denote by $\nu_{p}(n)$ the number of prime factors $p$ in $n$, that is, the maximum exponent $\beta$ for which $p^{\beta} \mid n$. For every $\ell \geqslant 1$ and $p \in P$, the Lifting the Exponent Lemma provides + +$$ +\nu_{p}\left(10^{\ell \alpha}-1\right)=\nu_{p}\left(\left(10^{\alpha}\right)^{\ell}-1\right)=\nu_{p}\left(10^{\alpha}-1\right)+\nu_{p}(\ell)=\nu_{p}(m)+\nu_{p}(\ell) +$$ + +so + +$$ +\begin{aligned} +c m \mid 10^{k \alpha}-1 & \Longleftrightarrow \forall p \in P ; \nu_{p}(c m) \leqslant \nu_{p}\left(10^{k \alpha}-1\right) \\ +& \Longleftrightarrow \forall p \in P ; \nu_{p}(m)+\nu_{p}(c) \leqslant \nu_{p}(m)+\nu_{p}(k) \\ +& \Longleftrightarrow \forall p \in P ; \nu_{p}(c) \leqslant \nu_{p}(k) \\ +& \Longleftrightarrow c \mid k . +\end{aligned} +$$ + +The first such $k$ is $k=c$, so $\operatorname{ord}_{c m}(10)=c \alpha$. + +Comment. The Lifting the Exponent Lemma states that, for any odd prime $p$, any integers $a, b$ coprime with $p$ such that $p \mid a-b$, and any positive integer exponent $n$, + +$$ +\nu_{p}\left(a^{n}-b^{n}\right)=\nu_{p}(a-b)+\nu_{p}(n), +$$ + +and, for $p=2$, + +$$ +\nu_{2}\left(a^{n}-b^{n}\right)=\nu_{2}\left(a^{2}-b^{2}\right)+\nu_{p}(n)-1 . +$$ + +Both claims can be proved by induction on $n$. + +N5. Find all pairs $(p, q)$ of prime numbers with $p>q$ for which the number + +$$ +\frac{(p+q)^{p+q}(p-q)^{p-q}-1}{(p+q)^{p-q}(p-q)^{p+q}-1} +$$ + +is an integer. +(Japan) +Answer: The only such pair is $(3,2)$. +Solution. Let $M=(p+q)^{p-q}(p-q)^{p+q}-1$, which is relatively prime with both $p+q$ and $p-q$. Denote by $(p-q)^{-1}$ the multiplicative inverse of $(p-q)$ modulo $M$. + +By eliminating the term -1 in the numerator, + +$$ +\begin{aligned} +(p+q)^{p+q}(p-q)^{p-q}-1 & \equiv(p+q)^{p-q}(p-q)^{p+q}-1 \quad(\bmod M) \\ +(p+q)^{2 q} & \equiv(p-q)^{2 q} \quad(\bmod M) \\ +\left((p+q) \cdot(p-q)^{-1}\right)^{2 q} & \equiv 1 \quad(\bmod M) +\end{aligned} +$$ + +Case 1: $q \geqslant 5$. +Consider an arbitrary prime divisor $r$ of $M$. Notice that $M$ is odd, so $r \geqslant 3$. By (2), the multiplicative order of $\left((p+q) \cdot(p-q)^{-1}\right)$ modulo $r$ is a divisor of the exponent $2 q$ in (2), so it can be $1,2, q$ or $2 q$. + +By Fermat's theorem, the order divides $r-1$. So, if the order is $q$ or $2 q$ then $r \equiv 1(\bmod q)$. If the order is 1 or 2 then $r \mid(p+q)^{2}-(p-q)^{2}=4 p q$, so $r=p$ or $r=q$. The case $r=p$ is not possible, because, by applying Fermat's theorem, +$M=(p+q)^{p-q}(p-q)^{p+q}-1 \equiv q^{p-q}(-q)^{p+q}-1=\left(q^{2}\right)^{p}-1 \equiv q^{2}-1=(q+1)(q-1) \quad(\bmod p)$ +and the last factors $q-1$ and $q+1$ are less than $p$ and thus $p \nmid M$. Hence, all prime divisors of $M$ are either $q$ or of the form $k q+1$; it follows that all positive divisors of $M$ are congruent to 0 or 1 modulo $q$. + +Now notice that + +$$ +M=\left((p+q)^{\frac{p-q}{2}}(p-q)^{\frac{p+q}{2}}-1\right)\left((p+q)^{\frac{p-q}{2}}(p-q)^{\frac{p+q}{2}}+1\right) +$$ + +is the product of two consecutive positive odd numbers; both should be congruent to 0 or 1 modulo $q$. But this is impossible by the assumption $q \geqslant 5$. So, there is no solution in Case 1 . +Case 2: $q=2$. +By (1), we have $M \mid(p+q)^{2 q}-(p-q)^{2 q}=(p+2)^{4}-(p-2)^{4}$, so + +$$ +\begin{gathered} +(p+2)^{p-2}(p-2)^{p+2}-1=M \leqslant(p+2)^{4}-(p-2)^{4} \leqslant(p+2)^{4}-1, \\ +(p+2)^{p-6}(p-2)^{p+2} \leqslant 1 . +\end{gathered} +$$ + +If $p \geqslant 7$ then the left-hand side is obviously greater than 1 . For $p=5$ we have $(p+2)^{p-6}(p-2)^{p+2}=7^{-1} \cdot 3^{7}$ which is also too large. + +There remains only one candidate, $p=3$, which provides a solution: + +$$ +\frac{(p+q)^{p+q}(p-q)^{p-q}-1}{(p+q)^{p-q}(p-q)^{p+q}-1}=\frac{5^{5} \cdot 1^{1}-1}{5^{1} \cdot 1^{5}-1}=\frac{3124}{4}=781 . +$$ + +So in Case 2 the only solution is $(p, q)=(3,2)$. + +Case 3: $q=3$. +Similarly to Case 2, we have + +$$ +M \left\lvert\,(p+q)^{2 q}-(p-q)^{2 q}=64 \cdot\left(\left(\frac{p+3}{2}\right)^{6}-\left(\frac{p-3}{2}\right)^{6}\right)\right. +$$ + +Since $M$ is odd, we conclude that + +$$ +M \left\lvert\,\left(\frac{p+3}{2}\right)^{6}-\left(\frac{p-3}{2}\right)^{6}\right. +$$ + +and + +$$ +\begin{gathered} +(p+3)^{p-3}(p-3)^{p+3}-1=M \leqslant\left(\frac{p+3}{2}\right)^{6}-\left(\frac{p-3}{2}\right)^{6} \leqslant\left(\frac{p+3}{2}\right)^{6}-1 \\ +64(p+3)^{p-9}(p-3)^{p+3} \leqslant 1 +\end{gathered} +$$ + +If $p \geqslant 11$ then the left-hand side is obviously greater than 1 . If $p=7$ then the left-hand side is $64 \cdot 10^{-2} \cdot 4^{10}>1$. If $p=5$ then the left-hand side is $64 \cdot 8^{-4} \cdot 2^{8}=2^{2}>1$. Therefore, there is no solution in Case 3. + +N6. Find the smallest positive integer $n$, or show that no such $n$ exists, with the following property: there are infinitely many distinct $n$-tuples of positive rational numbers $\left(a_{1}, a_{2}, \ldots, a_{n}\right)$ such that both + +$$ +a_{1}+a_{2}+\cdots+a_{n} \quad \text { and } \quad \frac{1}{a_{1}}+\frac{1}{a_{2}}+\cdots+\frac{1}{a_{n}} +$$ + +are integers. +(Singapore) +Answer: $n=3$. +Solution 1. For $n=1, a_{1} \in \mathbb{Z}_{>0}$ and $\frac{1}{a_{1}} \in \mathbb{Z}_{>0}$ if and only if $a_{1}=1$. Next we show that +(i) There are finitely many $(x, y) \in \mathbb{Q}_{>0}^{2}$ satisfying $x+y \in \mathbb{Z}$ and $\frac{1}{x}+\frac{1}{y} \in \mathbb{Z}$ + +Write $x=\frac{a}{b}$ and $y=\frac{c}{d}$ with $a, b, c, d \in \mathbb{Z}_{>0}$ and $\operatorname{gcd}(a, b)=\operatorname{gcd}(c, d)=1$. Then $x+y \in \mathbb{Z}$ and $\frac{1}{x}+\frac{1}{y} \in \mathbb{Z}$ is equivalent to the two divisibility conditions + +$$ +b d \mid a d+b c \quad(1) \quad \text { and } \quad a c \mid a d+b c +$$ + +Condition (1) implies that $d|a d+b c \Longleftrightarrow d| b c \Longleftrightarrow d \mid b$ since $\operatorname{gcd}(c, d)=1$. Still from (1) we get $b|a d+b c \Longleftrightarrow b| a d \Longleftrightarrow b \mid d$ since $\operatorname{gcd}(a, b)=1$. From $b \mid d$ and $d \mid b$ we have $b=d$. +An analogous reasoning with condition (2) shows that $a=c$. Hence $x=\frac{a}{b}=\frac{c}{d}=y$, i.e., the problem amounts to finding all $x \in \mathbb{Q}_{>0}$ such that $2 x \in \mathbb{Z}_{>0}$ and $\frac{2}{x} \in \mathbb{Z}_{>0}$. Letting $n=2 x \in \mathbb{Z}_{>0}$, we have that $\frac{2}{x} \in \mathbb{Z}_{>0} \Longleftrightarrow \frac{4}{n} \in \mathbb{Z}_{>0} \Longleftrightarrow n=1,2$ or 4 , and there are finitely many solutions, namely $(x, y)=\left(\frac{1}{2}, \frac{1}{2}\right),(1,1)$ or $(2,2)$. +(ii) There are infinitely many triples $(x, y, z) \in \mathbb{Q}_{>0}^{2}$ such that $x+y+z \in \mathbb{Z}$ and $\frac{1}{x}+\frac{1}{y}+\frac{1}{z} \in \mathbb{Z}$. We will look for triples such that $x+y+z=1$, so we may write them in the form + +$$ +(x, y, z)=\left(\frac{a}{a+b+c}, \frac{b}{a+b+c}, \frac{c}{a+b+c}\right) \quad \text { with } a, b, c \in \mathbb{Z}_{>0} +$$ + +We want these to satisfy + +$$ +\frac{1}{x}+\frac{1}{y}+\frac{1}{z}=\frac{a+b+c}{a}+\frac{a+b+c}{b}+\frac{a+b+c}{c} \in \mathbb{Z} \Longleftrightarrow \frac{b+c}{a}+\frac{a+c}{b}+\frac{a+b}{c} \in \mathbb{Z} +$$ + +Fixing $a=1$, it suffices to find infinitely many pairs $(b, c) \in \mathbb{Z}_{>0}^{2}$ such that + +$$ +\frac{1}{b}+\frac{1}{c}+\frac{c}{b}+\frac{b}{c}=3 \Longleftrightarrow b^{2}+c^{2}-3 b c+b+c=0 +$$ + +To show that equation (*) has infinitely many solutions, we use Vieta jumping (also known as root flipping): starting with $b=2, c=3$, the following algorithm generates infinitely many solutions. Let $c \geqslant b$, and view (*) as a quadratic equation in $b$ for $c$ fixed: + +$$ +b^{2}-(3 c-1) \cdot b+\left(c^{2}+c\right)=0 +$$ + +Then there exists another root $b_{0} \in \mathbb{Z}$ of ( $\left.* *\right)$ which satisfies $b+b_{0}=3 c-1$ and $b \cdot b_{0}=c^{2}+c$. Since $c \geqslant b$ by assumption, + +$$ +b_{0}=\frac{c^{2}+c}{b} \geqslant \frac{c^{2}+c}{c}>c +$$ + +Hence from the solution $(b, c)$ we obtain another one $\left(c, b_{0}\right)$ with $b_{0}>c$, and we can then "jump" again, this time with $c$ as the "variable" in the quadratic (*). This algorithm will generate an infinite sequence of distinct solutions, whose first terms are +$(2,3),(3,6),(6,14),(14,35),(35,90),(90,234),(234,611),(611,1598),(1598,4182), \ldots$ + +Comment. Although not needed for solving this problem, we may also explicitly solve the recursion given by the Vieta jumping. Define the sequence $\left(x_{n}\right)$ as follows: + +$$ +x_{0}=2, \quad x_{1}=3 \quad \text { and } \quad x_{n+2}=3 x_{n+1}-x_{n}-1 \text { for } n \geqslant 0 +$$ + +Then the triple + +$$ +(x, y, z)=\left(\frac{1}{1+x_{n}+x_{n+1}}, \frac{x_{n}}{1+x_{n}+x_{n+1}}, \frac{x_{n+1}}{1+x_{n}+x_{n+1}}\right) +$$ + +satisfies the problem conditions for all $n \in \mathbb{N}$. It is easy to show that $x_{n}=F_{2 n+1}+1$, where $F_{n}$ denotes the $n$-th term of the Fibonacci sequence ( $F_{0}=0, F_{1}=1$, and $F_{n+2}=F_{n+1}+F_{n}$ for $n \geqslant 0$ ). +Solution 2. Call the $n$-tuples $\left(a_{1}, a_{2}, \ldots, a_{n}\right) \in \mathbb{Q}_{>0}^{n}$ satisfying the conditions of the problem statement good, and those for which + +$$ +f\left(a_{1}, \ldots, a_{n}\right) \stackrel{\text { def }}{=}\left(a_{1}+a_{2}+\cdots+a_{n}\right)\left(\frac{1}{a_{1}}+\frac{1}{a_{2}}+\cdots+\frac{1}{a_{n}}\right) +$$ + +is an integer pretty. Then good $n$-tuples are pretty, and if $\left(b_{1}, \ldots, b_{n}\right)$ is pretty then + +$$ +\left(\frac{b_{1}}{b_{1}+b_{2}+\cdots+b_{n}}, \frac{b_{2}}{b_{1}+b_{2}+\cdots+b_{n}}, \ldots, \frac{b_{n}}{b_{1}+b_{2}+\cdots+b_{n}}\right) +$$ + +is good since the sum of its components is 1 , and the sum of the reciprocals of its components equals $f\left(b_{1}, \ldots, b_{n}\right)$. We declare pretty $n$-tuples proportional to each other equivalent since they are precisely those which give rise to the same good $n$-tuple. Clearly, each such equivalence class contains exactly one $n$-tuple of positive integers having no common prime divisors. Call such $n$-tuple a primitive pretty tuple. Our task is to find infinitely many primitive pretty $n$-tuples. + +For $n=1$, there is clearly a single primitive 1 -tuple. For $n=2$, we have $f(a, b)=\frac{(a+b)^{2}}{a b}$, which can be integral (for coprime $a, b \in \mathbb{Z}_{>0}$ ) only if $a=b=1$ (see for instance (i) in the first solution). + +Now we construct infinitely many primitive pretty triples for $n=3$. Fix $b, c, k \in \mathbb{Z}_{>0}$; we will try to find sufficient conditions for the existence of an $a \in \mathbb{Q}_{>0}$ such that $f(a, b, c)=k$. Write $\sigma=b+c, \tau=b c$. From $f(a, b, c)=k$, we have that $a$ should satisfy the quadratic equation + +$$ +a^{2} \cdot \sigma+a \cdot\left(\sigma^{2}-(k-1) \tau\right)+\sigma \tau=0 +$$ + +whose discriminant is + +$$ +\Delta=\left(\sigma^{2}-(k-1) \tau\right)^{2}-4 \sigma^{2} \tau=\left((k+1) \tau-\sigma^{2}\right)^{2}-4 k \tau^{2} +$$ + +We need it to be a square of an integer, say, $\Delta=M^{2}$ for some $M \in \mathbb{Z}$, i.e., we want + +$$ +\left((k+1) \tau-\sigma^{2}\right)^{2}-M^{2}=2 k \cdot 2 \tau^{2} +$$ + +so that it suffices to set + +$$ +(k+1) \tau-\sigma^{2}=\tau^{2}+k, \quad M=\tau^{2}-k . +$$ + +The first relation reads $\sigma^{2}=(\tau-1)(k-\tau)$, so if $b$ and $c$ satisfy + +$$ +\tau-1 \mid \sigma^{2} \quad \text { i.e. } \quad b c-1 \mid(b+c)^{2} +$$ + +then $k=\frac{\sigma^{2}}{\tau-1}+\tau$ will be integral, and we find rational solutions to (1), namely + +$$ +a=\frac{\sigma}{\tau-1}=\frac{b+c}{b c-1} \quad \text { or } \quad a=\frac{\tau^{2}-\tau}{\sigma}=\frac{b c \cdot(b c-1)}{b+c} +$$ + +We can now find infinitely many pairs ( $b, c$ ) satisfying (2) by Vieta jumping. For example, if we impose + +$$ +(b+c)^{2}=5 \cdot(b c-1) +$$ + +then all pairs $(b, c)=\left(v_{i}, v_{i+1}\right)$ satisfy the above condition, where + +$$ +v_{1}=2, v_{2}=3, \quad v_{i+2}=3 v_{i+1}-v_{i} \quad \text { for } i \geqslant 0 +$$ + +For $(b, c)=\left(v_{i}, v_{i+1}\right)$, one of the solutions to (1) will be $a=(b+c) /(b c-1)=5 /(b+c)=$ $5 /\left(v_{i}+v_{i+1}\right)$. Then the pretty triple $(a, b, c)$ will be equivalent to the integral pretty triple + +$$ +\left(5, v_{i}\left(v_{i}+v_{i+1}\right), v_{i+1}\left(v_{i}+v_{i+1}\right)\right) +$$ + +After possibly dividing by 5 , we obtain infinitely many primitive pretty triples, as required. +Comment. There are many other infinite series of $(b, c)=\left(v_{i}, v_{i+1}\right)$ with $b c-1 \mid(b+c)^{2}$. Some of them are: + +$$ +\begin{array}{llll} +v_{1}=1, & v_{2}=3, & v_{i+1}=6 v_{i}-v_{i-1}, & \left(v_{i}+v_{i+1}\right)^{2}=8 \cdot\left(v_{i} v_{i+1}-1\right) ; \\ +v_{1}=1, & v_{2}=2, & v_{i+1}=7 v_{i}-v_{i-1}, & \left(v_{i}+v_{i+1}\right)^{2}=9 \cdot\left(v_{i} v_{i+1}-1\right) ; \\ +v_{1}=1, & v_{2}=5, & v_{i+1}=7 v_{i}-v_{i-1}, & \left(v_{i}+v_{i+1}\right)^{2}=9 \cdot\left(v_{i} v_{i+1}-1\right) +\end{array} +$$ + +(the last two are in fact one sequence prolonged in two possible directions). + +N7. Say that an ordered pair $(x, y)$ of integers is an irreducible lattice point if $x$ and $y$ are relatively prime. For any finite set $S$ of irreducible lattice points, show that there is a homogenous polynomial in two variables, $f(x, y)$, with integer coefficients, of degree at least 1 , such that $f(x, y)=1$ for each $(x, y)$ in the set $S$. + +Note: A homogenous polynomial of degree $n$ is any nonzero polynomial of the form + +$$ +f(x, y)=a_{0} x^{n}+a_{1} x^{n-1} y+a_{2} x^{n-2} y^{2}+\cdots+a_{n-1} x y^{n-1}+a_{n} y^{n} . +$$ + +Solution 1. First of all, we note that finding a homogenous polynomial $f(x, y)$ such that $f(x, y)= \pm 1$ is enough, because we then have $f^{2}(x, y)=1$. Label the irreducible lattice points $\left(x_{1}, y_{1}\right)$ through $\left(x_{n}, y_{n}\right)$. If any two of these lattice points $\left(x_{i}, y_{i}\right)$ and $\left(x_{j}, y_{j}\right)$ lie on the same line through the origin, then $\left(x_{j}, y_{j}\right)=\left(-x_{i},-y_{i}\right)$ because both of the points are irreducible. We then have $f\left(x_{j}, y_{j}\right)= \pm f\left(x_{i}, y_{i}\right)$ whenever $f$ is homogenous, so we can assume that no two of the lattice points are collinear with the origin by ignoring the extra lattice points. + +Consider the homogenous polynomials $\ell_{i}(x, y)=y_{i} x-x_{i} y$ and define + +$$ +g_{i}(x, y)=\prod_{j \neq i} \ell_{j}(x, y) +$$ + +Then $\ell_{i}\left(x_{j}, y_{j}\right)=0$ if and only if $j=i$, because there is only one lattice point on each line through the origin. Thus, $g_{i}\left(x_{j}, y_{j}\right)=0$ for all $j \neq i$. Define $a_{i}=g_{i}\left(x_{i}, y_{i}\right)$, and note that $a_{i} \neq 0$. + +Note that $g_{i}(x, y)$ is a degree $n-1$ polynomial with the following two properties: + +1. $g_{i}\left(x_{j}, y_{j}\right)=0$ if $j \neq i$. +2. $g_{i}\left(x_{i}, y_{i}\right)=a_{i}$. + +For any $N \geqslant n-1$, there also exists a polynomial of degree $N$ with the same two properties. Specifically, let $I_{i}(x, y)$ be a degree 1 homogenous polynomial such that $I_{i}\left(x_{i}, y_{i}\right)=1$, which exists since $\left(x_{i}, y_{i}\right)$ is irreducible. Then $I_{i}(x, y)^{N-(n-1)} g_{i}(x, y)$ satisfies both of the above properties and has degree $N$. + +We may now reduce the problem to the following claim: +Claim: For each positive integer $a$, there is a homogenous polynomial $f_{a}(x, y)$, with integer coefficients, of degree at least 1 , such that $f_{a}(x, y) \equiv 1(\bmod a)$ for all relatively prime $(x, y)$. + +To see that this claim solves the problem, take $a$ to be the least common multiple of the numbers $a_{i}(1 \leqslant i \leqslant n)$. Take $f_{a}$ given by the claim, choose some power $f_{a}(x, y)^{k}$ that has degree at least $n-1$, and subtract appropriate multiples of the $g_{i}$ constructed above to obtain the desired polynomial. + +We prove the claim by factoring $a$. First, if $a$ is a power of a prime ( $a=p^{k}$ ), then we may choose either: + +- $f_{a}(x, y)=\left(x^{p-1}+y^{p-1}\right)^{\phi(a)}$ if $p$ is odd; +- $f_{a}(x, y)=\left(x^{2}+x y+y^{2}\right)^{\phi(a)}$ if $p=2$. + +Now suppose $a$ is any positive integer, and let $a=q_{1} q_{2} \cdots q_{k}$, where the $q_{i}$ are prime powers, pairwise relatively prime. Let $f_{q_{i}}$ be the polynomials just constructed, and let $F_{q_{i}}$ be powers of these that all have the same degree. Note that + +$$ +\frac{a}{q_{i}} F_{q_{i}}(x, y) \equiv \frac{a}{q_{i}} \quad(\bmod a) +$$ + +for any relatively prime $x, y$. By Bézout's lemma, there is an integer linear combination of the $\frac{a}{q_{i}}$ that equals 1 . Thus, there is a linear combination of the $F_{q_{i}}$ such that $F_{q_{i}}(x, y) \equiv 1$ $(\bmod a)$ for any relatively prime $(x, y)$; and this polynomial is homogenous because all the $F_{q_{i}}$ have the same degree. + +Solution 2. As in the previous solution, label the irreducible lattice points $\left(x_{1}, y_{1}\right), \ldots,\left(x_{n}, y_{n}\right)$ and assume without loss of generality that no two of the points are collinear with the origin. We induct on $n$ to construct a homogenous polynomial $f(x, y)$ such that $f\left(x_{i}, y_{i}\right)=1$ for all $1 \leqslant i \leqslant n$. + +If $n=1$ : Since $x_{1}$ and $y_{1}$ are relatively prime, there exist some integers $c, d$ such that $c x_{1}+d y_{1}=1$. Then $f(x, y)=c x+d y$ is suitable. + +If $n \geqslant 2$ : By the induction hypothesis we already have a homogeneous polynomial $g(x, y)$ with $g\left(x_{1}, y_{1}\right)=\ldots=g\left(x_{n-1}, y_{n-1}\right)=1$. Let $j=\operatorname{deg} g$, + +$$ +g_{n}(x, y)=\prod_{k=1}^{n-1}\left(y_{k} x-x_{k} y\right) +$$ + +and $a_{n}=g_{n}\left(x_{n}, y_{n}\right)$. By assumption, $a_{n} \neq 0$. Take some integers $c, d$ such that $c x_{n}+d y_{n}=1$. We will construct $f(x, y)$ in the form + +$$ +f(x, y)=g(x, y)^{K}-C \cdot g_{n}(x, y) \cdot(c x+d y)^{L} +$$ + +where $K$ and $L$ are some positive integers and $C$ is some integer. We assume that $L=K j-n+1$ so that $f$ is homogenous. + +Due to $g\left(x_{1}, y_{1}\right)=\ldots=g\left(x_{n-1}, y_{n-1}\right)=1$ and $g_{n}\left(x_{1}, y_{1}\right)=\ldots=g_{n}\left(x_{n-1}, y_{n-1}\right)=0$, the property $f\left(x_{1}, y_{1}\right)=\ldots=f\left(x_{n-1}, y_{n-1}\right)=1$ is automatically satisfied with any choice of $K, L$, and $C$. + +Furthermore, + +$$ +f\left(x_{n}, y_{n}\right)=g\left(x_{n}, y_{n}\right)^{K}-C \cdot g_{n}\left(x_{n}, y_{n}\right) \cdot\left(c x_{n}+d y_{n}\right)^{L}=g\left(x_{n}, y_{n}\right)^{K}-C a_{n} +$$ + +If we have an exponent $K$ such that $g\left(x_{n}, y_{n}\right)^{K} \equiv 1\left(\bmod a_{n}\right)$, then we may choose $C$ such that $f\left(x_{n}, y_{n}\right)=1$. We now choose such a $K$. + +Consider an arbitrary prime divisor $p$ of $a_{n}$. By + +$$ +p \mid a_{n}=g_{n}\left(x_{n}, y_{n}\right)=\prod_{k=1}^{n-1}\left(y_{k} x_{n}-x_{k} y_{n}\right) +$$ + +there is some $1 \leqslant k0$.) + +Comment. It is possible to show that there is no constant $C$ for which, given any two irreducible lattice points, there is some homogenous polynomial $f$ of degree at most $C$ with integer coefficients that takes the value 1 on the two points. Indeed, if one of the points is $(1,0)$ and the other is $(a, b)$, the polynomial $f(x, y)=a_{0} x^{n}+a_{1} x^{n-1} y+\cdots+a_{n} y^{n}$ should satisfy $a_{0}=1$, and so $a^{n} \equiv 1(\bmod b)$. If $a=3$ and $b=2^{k}$ with $k \geqslant 3$, then $n \geqslant 2^{k-2}$. If we choose $2^{k-2}>C$, this gives a contradiction. + +N8. Let $p$ be an odd prime number and $\mathbb{Z}_{>0}$ be the set of positive integers. Suppose that a function $f: \mathbb{Z}_{>0} \times \mathbb{Z}_{>0} \rightarrow\{0,1\}$ satisfies the following properties: + +- $f(1,1)=0$; +- $f(a, b)+f(b, a)=1$ for any pair of relatively prime positive integers $(a, b)$ not both equal to 1 ; +- $f(a+b, b)=f(a, b)$ for any pair of relatively prime positive integers $(a, b)$. + +Prove that + +$$ +\sum_{n=1}^{p-1} f\left(n^{2}, p\right) \geqslant \sqrt{2 p}-2 +$$ + +(Italy) +Solution 1. Denote by $\mathbb{A}$ the set of all pairs of coprime positive integers. Notice that for every $(a, b) \in \mathbb{A}$ there exists a pair $(u, v) \in \mathbb{Z}^{2}$ with $u a+v b=1$. Moreover, if $\left(u_{0}, v_{0}\right)$ is one such pair, then all such pairs are of the form $(u, v)=\left(u_{0}+k b, v_{0}-k a\right)$, where $k \in \mathbb{Z}$. So there exists a unique such pair $(u, v)$ with $-b / 20$. +Proof. We induct on $a+b$. The base case is $a+b=2$. In this case, we have that $a=b=1$, $g(a, b)=g(1,1)=(0,1)$ and $f(1,1)=0$, so the claim holds. + +Assume now that $a+b>2$, and so $a \neq b$, since $a$ and $b$ are coprime. Two cases are possible. Case 1: $a>b$. + +Notice that $g(a-b, b)=(u, v+u)$, since $u(a-b)+(v+u) b=1$ and $u \in(-b / 2, b / 2]$. Thus $f(a, b)=1 \Longleftrightarrow f(a-b, b)=1 \Longleftrightarrow u>0$ by the induction hypothesis. +Case 2: $av b \geqslant 1-\frac{a b}{2}, \quad \text { so } \quad \frac{1+a}{2} \geqslant \frac{1}{b}+\frac{a}{2}>v \geqslant \frac{1}{b}-\frac{a}{2}>-\frac{a}{2} . +$$ + +Thus $1+a>2 v>-a$, so $a \geqslant 2 v>-a$, hence $a / 2 \geqslant v>-a / 2$, and thus $g(b, a)=(v, u)$. +Observe that $f(a, b)=1 \Longleftrightarrow f(b, a)=0 \Longleftrightarrow f(b-a, a)=0$. We know from Case 1 that $g(b-a, a)=(v, u+v)$. We have $f(b-a, a)=0 \Longleftrightarrow v \leqslant 0$ by the inductive hypothesis. Then, since $b>a \geqslant 1$ and $u a+v b=1$, we have $v \leqslant 0 \Longleftrightarrow u>0$, and we are done. + +The Lemma proves that, for all $(a, b) \in \mathbb{A}, f(a, b)=1$ if and only if the inverse of $a$ modulo $b$, taken in $\{1,2, \ldots, b-1\}$, is at most $b / 2$. Then, for any odd prime $p$ and integer $n$ such that $n \not \equiv 0(\bmod p), f\left(n^{2}, p\right)=1$ iff the inverse of $n^{2} \bmod p$ is less than $p / 2$. Since $\left\{n^{2} \bmod p: 1 \leqslant n \leqslant p-1\right\}=\left\{n^{-2} \bmod p: 1 \leqslant n \leqslant p-1\right\}$, including multiplicities (two for each quadratic residue in each set), we conclude that the desired sum is twice the number of quadratic residues that are less than $p / 2$, i.e., + +$$ +\left.\sum_{n=1}^{p-1} f\left(n^{2}, p\right)=2 \left\lvert\,\left\{k: 1 \leqslant k \leqslant \frac{p-1}{2} \text { and } k^{2} \bmod p<\frac{p}{2}\right\}\right. \right\rvert\, . +$$ + +Since the number of perfect squares in the interval $[1, p / 2)$ is $\lfloor\sqrt{p / 2}\rfloor>\sqrt{p / 2}-1$, we conclude that + +$$ +\sum_{n=1}^{p-1} f\left(n^{2}, p\right)>2\left(\sqrt{\frac{p}{2}}-1\right)=\sqrt{2 p}-2 +$$ + +Solution 2. We provide a different proof for the Lemma. For this purpose, we use continued fractions to find $g(a, b)=(u, v)$ explicitly. + +The function $f$ is completely determined on $\mathbb{A}$ by the following +Claim. Represent $a / b$ as a continued fraction; that is, let $a_{0}$ be an integer and $a_{1}, \ldots, a_{k}$ be positive integers such that $a_{k} \geqslant 2$ and + +$$ +\frac{a}{b}=a_{0}+\frac{1}{a_{1}+\frac{1}{a_{2}+\frac{1}{\cdots+\frac{1}{a_{k}}}}}=\left[a_{0} ; a_{1}, a_{2}, \ldots, a_{k}\right] . +$$ + +Then $f(a, b)=0 \Longleftrightarrow k$ is even. +Proof. We induct on $b$. If $b=1$, then $a / b=[a]$ and $k=0$. Then, for $a \geqslant 1$, an easy induction shows that $f(a, 1)=f(1,1)=0$. + +Now consider the case $b>1$. Perform the Euclidean division $a=q b+r$, with $0 \leqslant r0$ and define $q_{-1}=0$ if necessary. Then + +- $q_{k}=a_{k} q_{k-1}+q_{k-2}, \quad$ and +- $a q_{k-1}-b p_{k-1}=p_{k} q_{k-1}-q_{k} p_{k-1}=(-1)^{k-1}$. + +Assume that $k>0$. Then $a_{k} \geqslant 2$, and + +$$ +b=q_{k}=a_{k} q_{k-1}+q_{k-2} \geqslant a_{k} q_{k-1} \geqslant 2 q_{k-1} \Longrightarrow q_{k-1} \leqslant \frac{b}{2} +$$ + +with strict inequality for $k>1$, and + +$$ +(-1)^{k-1} q_{k-1} a+(-1)^{k} p_{k-1} b=1 +$$ + +Now we finish the proof of the Lemma. It is immediate for $k=0$. If $k=1$, then $(-1)^{k-1}=1$, so + +$$ +-b / 2<0 \leqslant(-1)^{k-1} q_{k-1} \leqslant b / 2 +$$ + +If $k>1$, we have $q_{k-1}0$, we find that $g(a, b)=\left((-1)^{k-1} q_{k-1},(-1)^{k} p_{k-1}\right)$, and so + +$$ +f(a, b)=1 \Longleftrightarrow k \text { is odd } \Longleftrightarrow u=(-1)^{k-1} q_{k-1}>0 +$$ + +Comment 1. The Lemma can also be established by observing that $f$ is uniquely defined on $\mathbb{A}$, defining $f_{1}(a, b)=1$ if $u>0$ in $g(a, b)=(u, v)$ and $f_{1}(a, b)=0$ otherwise, and verifying that $f_{1}$ satisfies all the conditions from the statement. + +It seems that the main difficulty of the problem is in conjecturing the Lemma. +Comment 2. The case $p \equiv 1(\bmod 4)$ is, in fact, easier than the original problem. We have, in general, for $1 \leqslant a \leqslant p-1$, +$f(a, p)=1-f(p, a)=1-f(p-a, a)=f(a, p-a)=f(a+(p-a), p-a)=f(p, p-a)=1-f(p-a, p)$. +If $p \equiv 1(\bmod 4)$, then $a$ is a quadratic residue modulo $p$ if and only if $p-a$ is a quadratic residue modulo $p$. Therefore, denoting by $r_{k}$ (with $1 \leqslant r_{k} \leqslant p-1$ ) the remainder of the division of $k^{2}$ by $p$, we get + +$$ +\sum_{n=1}^{p-1} f\left(n^{2}, p\right)=\sum_{n=1}^{p-1} f\left(r_{n}, p\right)=\frac{1}{2} \sum_{n=1}^{p-1}\left(f\left(r_{n}, p\right)+f\left(p-r_{n}, p\right)\right)=\frac{p-1}{2} +$$ + +Comment 3. The estimate for the sum $\sum_{n=1}^{p} f\left(n^{2}, p\right)$ can be improved by refining the final argument in Solution 1. In fact, one can prove that + +$$ +\sum_{n=1}^{p-1} f\left(n^{2}, p\right) \geqslant \frac{p-1}{16} +$$ + +By counting the number of perfect squares in the intervals $[k p,(k+1 / 2) p)$, we find that + +$$ +\sum_{n=1}^{p-1} f\left(n^{2}, p\right)=\sum_{k=0}^{p-1}\left(\left\lfloor\sqrt{\left(k+\frac{1}{2}\right) p}\right\rfloor-\lfloor\sqrt{k p}\rfloor\right) . +$$ + +Each summand of (2) is non-negative. We now estimate the number of positive summands. Suppose that a summand is zero, i.e., + +$$ +\left\lfloor\sqrt{\left(k+\frac{1}{2}\right) p}\right\rfloor=\lfloor\sqrt{k p}\rfloor=: q . +$$ + +Then both of the numbers $k p$ and $k p+p / 2$ lie within the interval $\left[q^{2},(q+1)^{2}\right)$. Hence + +$$ +\frac{p}{2}<(q+1)^{2}-q^{2} +$$ + +which implies + +$$ +q \geqslant \frac{p-1}{4} +$$ + +Since $q \leqslant \sqrt{k p}$, if the $k^{\text {th }}$ summand of (2) is zero, then + +$$ +k \geqslant \frac{q^{2}}{p} \geqslant \frac{(p-1)^{2}}{16 p}>\frac{p-2}{16} \Longrightarrow k \geqslant \frac{p-1}{16} +$$ + +So at least the first $\left\lceil\frac{p-1}{16}\right\rceil$ summands (from $k=0$ to $k=\left\lceil\frac{p-1}{16}\right\rceil-1$ ) are positive, and the result follows. + +Comment 4. The bound can be further improved by using different methods. In fact, we prove that + +$$ +\sum_{n=1}^{p-1} f\left(n^{2}, p\right) \geqslant \frac{p-3}{4} +$$ + +To that end, we use the Legendre symbol + +$$ +\left(\frac{a}{p}\right)= \begin{cases}0 & \text { if } p \mid a \\ 1 & \text { if } a \text { is a nonzero quadratic residue } \bmod p \\ -1 & \text { otherwise. }\end{cases} +$$ + +We start with the following Claim, which tells us that there are not too many consecutive quadratic residues or consecutive quadratic non-residues. + +Claim. $\sum_{n=1}^{p-1}\left(\frac{n}{p}\right)\left(\frac{n+1}{p}\right)=-1$. +Proof. We have $\left(\frac{n}{p}\right)\left(\frac{n+1}{p}\right)=\left(\frac{n(n+1)}{p}\right)$. For $1 \leqslant n \leqslant p-1$, we get that $n(n+1) \equiv n^{2}\left(1+n^{-1}\right)(\bmod p)$, hence $\left(\frac{n(n+1)}{p}\right)=\left(\frac{1+n^{-1}}{p}\right)$. Since $\left\{1+n^{-1} \bmod p: 1 \leqslant n \leqslant p-1\right\}=\{0,2,3, \ldots, p-1 \bmod p\}$, we find + +$$ +\sum_{n=1}^{p-1}\left(\frac{n}{p}\right)\left(\frac{n+1}{p}\right)=\sum_{n=1}^{p-1}\left(\frac{1+n^{-1}}{p}\right)=\sum_{n=1}^{p-1}\left(\frac{n}{p}\right)-1=-1 +$$ + +because $\sum_{n=1}^{p}\left(\frac{n}{p}\right)=0$. +Observe that (1) becomes + +$$ +\sum_{n=1}^{p-1} f\left(n^{2}, p\right)=2|S|, \quad S=\left\{r: 1 \leqslant r \leqslant \frac{p-1}{2} \text { and }\left(\frac{r}{p}\right)=1\right\} +$$ + +We connect $S$ with the sum from the claim by pairing quadratic residues and quadratic non-residues. To that end, define + +$$ +\begin{aligned} +S^{\prime} & =\left\{r: 1 \leqslant r \leqslant \frac{p-1}{2} \text { and }\left(\frac{r}{p}\right)=-1\right\} \\ +T & =\left\{r: \frac{p+1}{2} \leqslant r \leqslant p-1 \text { and }\left(\frac{r}{p}\right)=1\right\} \\ +T^{\prime} & =\left\{r: \frac{p+1}{2} \leqslant r \leqslant p-1 \text { and }\left(\frac{r}{p}\right)=-1\right\} +\end{aligned} +$$ + +Since there are exactly $(p-1) / 2$ nonzero quadratic residues modulo $p,|S|+|T|=(p-1) / 2$. Also we obviously have $|T|+\left|T^{\prime}\right|=(p-1) / 2$. Then $|S|=\left|T^{\prime}\right|$. + +For the sake of brevity, define $t=|S|=\left|T^{\prime}\right|$. If $\left(\frac{n}{p}\right)\left(\frac{n+1}{p}\right)=-1$, then exactly of one the numbers $\left(\frac{n}{p}\right)$ and $\left(\frac{n+1}{p}\right)$ is equal to 1 , so + +$$ +\left\lvert\,\left\{n: 1 \leqslant n \leqslant \frac{p-3}{2} \text { and }\left(\frac{n}{p}\right)\left(\frac{n+1}{p}\right)=-1\right\}|\leqslant|S|+|S-1|=2 t\right. +$$ + +On the other hand, if $\left(\frac{n}{p}\right)\left(\frac{n+1}{p}\right)=-1$, then exactly one of $\left(\frac{n}{p}\right)$ and $\left(\frac{n+1}{p}\right)$ is equal to -1 , and + +$$ +\left\lvert\,\left\{n: \frac{p+1}{2} \leqslant n \leqslant p-2 \text { and }\left(\frac{n}{p}\right)\left(\frac{n+1}{p}\right)=-1\right\}\left|\leqslant\left|T^{\prime}\right|+\left|T^{\prime}-1\right|=2 t .\right.\right. +$$ + +Thus, taking into account that the middle term $\left(\frac{(p-1) / 2}{p}\right)\left(\frac{(p+1) / 2}{p}\right)$ may happen to be -1 , + +$$ +\left.\left\lvert\,\left\{n: 1 \leqslant n \leqslant p-2 \text { and }\left(\frac{n}{p}\right)\left(\frac{n+1}{p}\right)=-1\right\}\right. \right\rvert\, \leqslant 4 t+1 +$$ + +This implies that + +$$ +\left.\left\lvert\,\left\{n: 1 \leqslant n \leqslant p-2 \text { and }\left(\frac{n}{p}\right)\left(\frac{n+1}{p}\right)=1\right\}\right. \right\rvert\, \geqslant(p-2)-(4 t+1)=p-4 t-3 +$$ + +and so + +$$ +-1=\sum_{n=1}^{p-1}\left(\frac{n}{p}\right)\left(\frac{n+1}{p}\right) \geqslant p-4 t-3-(4 t+1)=p-8 t-4 +$$ + +which implies $8 t \geqslant p-3$, and thus + +$$ +\sum_{n=1}^{p-1} f\left(n^{2}, p\right)=2 t \geqslant \frac{p-3}{4} +$$ + +Comment 5. It is possible to prove that + +$$ +\sum_{n=1}^{p-1} f\left(n^{2}, p\right) \geqslant \frac{p-1}{2} +$$ + +The case $p \equiv 1(\bmod 4)$ was already mentioned, and it is the equality case. If $p \equiv 3(\bmod 4)$, then, by a theorem of Dirichlet, we have + +$$ +\left.\left\lvert\,\left\{r: 1 \leqslant r \leqslant \frac{p-1}{2} \text { and }\left(\frac{r}{p}\right)=1\right\}\right. \right\rvert\,>\frac{p-1}{4} +$$ + +which implies the result. +See https://en.wikipedia.org/wiki/Quadratic_residue\#Dirichlet.27s_formulas for the full statement of the theorem. It seems that no elementary proof of it is known; a proof using complex analysis is available, for instance, in Chapter 7 of the book Quadratic Residues and Non-Residues: Selected Topics, by Steve Wright, available in https://arxiv.org/abs/1408.0235. +![](https://cdn.mathpix.com/cropped/2024_11_18_ff08e3c077fa2f8c16b2g-96.jpg?height=138&width=204&top_left_y=2241&top_left_x=315) + +BIÊNIO DA MATEMATICA BRASIL +![](https://cdn.mathpix.com/cropped/2024_11_18_ff08e3c077fa2f8c16b2g-96.jpg?height=249&width=683&top_left_y=2251&top_left_x=1052) +![](https://cdn.mathpix.com/cropped/2024_11_18_ff08e3c077fa2f8c16b2g-96.jpg?height=132&width=338&top_left_y=2607&top_left_x=1436) + + +[^0]: *The name Dirichlet interval is chosen for the reason that $g$ theoretically might act similarly to the Dirichlet function on this interval. + diff --git a/IMO/md/en-IMO2018SL.md b/IMO/md/en-IMO2018SL.md new file mode 100644 index 0000000000000000000000000000000000000000..1a6606d1ac11f77f626d3957849f13f6c437c985 --- /dev/null +++ b/IMO/md/en-IMO2018SL.md @@ -0,0 +1,2341 @@ +![](https://cdn.mathpix.com/cropped/2024_04_17_6da854f21230b13cd732g-01.jpg?height=280&width=1714&top_left_y=157&top_left_x=206) + +# SHORTLISTED PROBLEMS WITH SOLUTIONS + +![](https://cdn.mathpix.com/cropped/2024_04_17_6da854f21230b13cd732g-01.jpg?height=1622&width=1982&top_left_y=1299&top_left_x=77) + +![](https://cdn.mathpix.com/cropped/2024_04_17_6da854f21230b13cd732g-03.jpg?height=419&width=537&top_left_y=196&top_left_x=223) + +## Shortlisted Problems (with solutions) + +$59^{\text {th }}$ International Mathematical Olympiad Cluj-Napoca — Romania, 3-14 July 2018 + +## The Shortlist has to be kept strictly confidential until the conclusion of the following
International Mathematical Olympiad.
IMO General Regulations $\S 6.6$ + +## Contributing Countries + +The Organising Committee and the Problem Selection Committee of IMO 2018 thank the following 49 countries for contributing 168 problem proposals: + +Armenia, Australia, Austria, Azerbaijan, Belarus, Belgium, Bosnia and Herzegovina, Brazil, Bulgaria, Canada, China, Croatia, Cyprus, Czech Republic, Denmark, Estonia, Germany, Greece, Hong Kong, Iceland, India, Indonesia, Iran, Ireland, Israel, Japan, Kosovo, Luxembourg, Mexico, Moldova, Mongolia, Netherlands, Nicaragua, Poland, Russia, Serbia, Singapore, Slovakia, Slovenia, South Africa, South Korea, Switzerland, Taiwan, Tanzania, Thailand, Turkey, Ukraine, United Kingdom, U.S.A. + +## Problem Selection Committee + +![](https://cdn.mathpix.com/cropped/2024_04_17_6da854f21230b13cd732g-04.jpg?height=896&width=1145&top_left_y=1705&top_left_x=455) + +Calin Popescu, Radu Gologan, Marian Andronache, Mihail Baluna, + +Nicolae Beli, Ilya Bogdanov, Pavel Kozhevnikov, Géza Kós, Sever Moldoveanu + +## Problems + +## Algebra + +A1. Let $\mathbb{Q}_{>0}$ denote the set of all positive rational numbers. Determine all functions $f: \mathbb{Q}_{>0} \rightarrow \mathbb{Q}_{>0}$ satisfying + +for all $x, y \in \mathbb{Q}_{>0}$. + +$$ +f\left(x^{2} f(y)^{2}\right)=f(x)^{2} f(y) +$$ + +(Switzerland) + +A2. Find all positive integers $n \geqslant 3$ for which there exist real numbers $a_{1}, a_{2}, \ldots, a_{n}$, $a_{n+1}=a_{1}, a_{n+2}=a_{2}$ such that + +$$ +a_{i} a_{i+1}+1=a_{i+2} +$$ + +for all $i=1,2, \ldots, n$. + +(Slovakia) + +A3. Given any set $S$ of positive integers, show that at least one of the following two assertions holds: + +(1) There exist distinct finite subsets $F$ and $G$ of $S$ such that $\sum_{x \in F} 1 / x=\sum_{x \in G} 1 / x$; + +(2) There exists a positive rational number $r<1$ such that $\sum_{x \in F} 1 / x \neq r$ for all finite subsets $F$ of $S$. + +(Luxembourg) + +A4. Let $a_{0}, a_{1}, a_{2}, \ldots$ be a sequence of real numbers such that $a_{0}=0, a_{1}=1$, and for every $n \geqslant 2$ there exists $1 \leqslant k \leqslant n$ satisfying + +$$ +a_{n}=\frac{a_{n-1}+\cdots+a_{n-k}}{k} +$$ + +Find the maximal possible value of $a_{2018}-a_{2017}$. + +(Belgium) + +A5. Determine all functions $f:(0, \infty) \rightarrow \mathbb{R}$ satisfying + +$$ +\left(x+\frac{1}{x}\right) f(y)=f(x y)+f\left(\frac{y}{x}\right) +$$ + +for all $x, y>0$. + +(South Korea) + +A6. Let $m, n \geqslant 2$ be integers. Let $f\left(x_{1}, \ldots, x_{n}\right)$ be a polynomial with real coefficients such that + +$$ +f\left(x_{1}, \ldots, x_{n}\right)=\left\lfloor\frac{x_{1}+\ldots+x_{n}}{m}\right\rfloor \text { for every } x_{1}, \ldots, x_{n} \in\{0,1, \ldots, m-1\} +$$ + +Prove that the total degree of $f$ is at least $n$. + +(Brazil) + +A7. Find the maximal value of + +$$ +S=\sqrt[3]{\frac{a}{b+7}}+\sqrt[3]{\frac{b}{c+7}}+\sqrt[3]{\frac{c}{d+7}}+\sqrt[3]{\frac{d}{a+7}} +$$ + +where $a, b, c, d$ are nonnegative real numbers which satisfy $a+b+c+d=100$. + +## Combinatorics + +C1. Let $n \geqslant 3$ be an integer. Prove that there exists a set $S$ of $2 n$ positive integers satisfying the following property: For every $m=2,3, \ldots, n$ the set $S$ can be partitioned into two subsets with equal sums of elements, with one of subsets of cardinality $m$. + +(Iceland) + +C2. Queenie and Horst play a game on a $20 \times 20$ chessboard. In the beginning the board is empty. In every turn, Horst places a black knight on an empty square in such a way that his new knight does not attack any previous knights. Then Queenie places a white queen on an empty square. The game gets finished when somebody cannot move. + +Find the maximal positive $K$ such that, regardless of the strategy of Queenie, Horst can put at least $K$ knights on the board. + +(Armenia) + +C3. Let $n$ be a given positive integer. Sisyphus performs a sequence of turns on a board consisting of $n+1$ squares in a row, numbered 0 to $n$ from left to right. Initially, $n$ stones are put into square 0, and the other squares are empty. At every turn, Sisyphus chooses any nonempty square, say with $k$ stones, takes one of those stones and moves it to the right by at most $k$ squares (the stone should stay within the board). Sisyphus' aim is to move all $n$ stones to square $n$. + +Prove that Sisyphus cannot reach the aim in less than + +$$ +\left\lceil\frac{n}{1}\right\rceil+\left\lceil\frac{n}{2}\right\rceil+\left\lceil\frac{n}{3}\right\rceil+\cdots+\left\lceil\frac{n}{n}\right\rceil +$$ + +turns. (As usual, $\lceil x\rceil$ stands for the least integer not smaller than $x$.) + +(Netherlands) + +C4. An anti-Pascal pyramid is a finite set of numbers, placed in a triangle-shaped array so that the first row of the array contains one number, the second row contains two numbers, the third row contains three numbers and so on; and, except for the numbers in the bottom row, each number equals the absolute value of the difference of the two numbers below it. For instance, the triangle below is an anti-Pascal pyramid with four rows, in which every integer from 1 to $1+2+3+4=10$ occurs exactly once: + +$$ +\begin{array}{cccc} +& & 4 \\ +& 2 & 6 & \\ +& 5 \quad 7 \quad 1 \\ +8 & 3 & 10 & 9 . +\end{array} +$$ + +Is it possible to form an anti-Pascal pyramid with 2018 rows, using every integer from 1 to $1+2+\cdots+2018$ exactly once? + +(Iran) + +C5. Let $k$ be a positive integer. The organising committee of a tennis tournament is to schedule the matches for $2 k$ players so that every two players play once, each day exactly one match is played, and each player arrives to the tournament site the day of his first match, and departs the day of his last match. For every day a player is present on the tournament, the committee has to pay 1 coin to the hotel. The organisers want to design the schedule so as to minimise the total cost of all players' stays. Determine this minimum cost. + +(Russia) + +C6. Let $a$ and $b$ be distinct positive integers. The following infinite process takes place on an initially empty board. + +(i) If there is at least a pair of equal numbers on the board, we choose such a pair and increase one of its components by $a$ and the other by $b$. + +(ii) If no such pair exists, we write down two times the number 0 . + +Prove that, no matter how we make the choices in $(i)$, operation (ii) will be performed only finitely many times. + +(Serbia) + +C7. Consider 2018 pairwise crossing circles no three of which are concurrent. These circles subdivide the plane into regions bounded by circular edges that meet at vertices. Notice that there are an even number of vertices on each circle. Given the circle, alternately colour the vertices on that circle red and blue. In doing so for each circle, every vertex is coloured twice once for each of the two circles that cross at that point. If the two colourings agree at a vertex, then it is assigned that colour; otherwise, it becomes yellow. Show that, if some circle contains at least 2061 yellow points, then the vertices of some region are all yellow. + +(India) + +## Geometry + +G1. Let $A B C$ be an acute-angled triangle with circumcircle $\Gamma$. Let $D$ and $E$ be points on the segments $A B$ and $A C$, respectively, such that $A D=A E$. The perpendicular bisectors of the segments $B D$ and $C E$ intersect the small arcs $\overparen{A B}$ and $\overparen{A C}$ at points $F$ and $G$ respectively. Prove that $D E \| F G$. + +(Greece) + +G2. Let $A B C$ be a triangle with $A B=A C$, and let $M$ be the midpoint of $B C$. Let $P$ be a point such that $P B

1$ be a positive integer. Each cell of an $n \times n$ table contains an integer. Suppose that the following conditions are satisfied: + +(i) Each number in the table is congruent to 1 modulo $n$; + +(ii) The sum of numbers in any row, as well as the sum of numbers in any column, is congruent to $n$ modulo $n^{2}$. + +Let $R_{i}$ be the product of the numbers in the $i^{\text {th }}$ row, and $C_{j}$ be the product of the numbers in the $j^{\text {th }}$ column. Prove that the sums $R_{1}+\cdots+R_{n}$ and $C_{1}+\cdots+C_{n}$ are congruent modulo $n^{4}$. + +(Indonesia) + +N3. Define the sequence $a_{0}, a_{1}, a_{2}, \ldots$ by $a_{n}=2^{n}+2^{\lfloor n / 2\rfloor}$. Prove that there are infinitely many terms of the sequence which can be expressed as a sum of (two or more) distinct terms of the sequence, as well as infinitely many of those which cannot be expressed in such a way. + +(Serbia) + +N4. Let $a_{1}, a_{2}, \ldots, a_{n}, \ldots$ be a sequence of positive integers such that + +$$ +\frac{a_{1}}{a_{2}}+\frac{a_{2}}{a_{3}}+\cdots+\frac{a_{n-1}}{a_{n}}+\frac{a_{n}}{a_{1}} +$$ + +is an integer for all $n \geqslant k$, where $k$ is some positive integer. Prove that there exists a positive integer $m$ such that $a_{n}=a_{n+1}$ for all $n \geqslant m$. + +(Mongolia) + +N5. Four positive integers $x, y, z$, and $t$ satisfy the relations + +$$ +x y-z t=x+y=z+t +$$ + +Is it possible that both $x y$ and $z t$ are perfect squares? + +(Russia) + +N6. Let $f:\{1,2,3, \ldots\} \rightarrow\{2,3, \ldots\}$ be a function such that $f(m+n) \mid f(m)+f(n)$ for all pairs $m, n$ of positive integers. Prove that there exists a positive integer $c>1$ which divides all values of $f$. + +(Mexico) + +N7. + +Let $n \geqslant 2018$ be an integer, and let $a_{1}, a_{2}, \ldots, a_{n}, b_{1}, b_{2}, \ldots, b_{n}$ be pairwise distinct positive integers not exceeding $5 n$. Suppose that the sequence + +$$ +\frac{a_{1}}{b_{1}}, \frac{a_{2}}{b_{2}}, \ldots, \frac{a_{n}}{b_{n}} +$$ + +forms an arithmetic progression. Prove that the terms of the sequence are equal. + +## Solutions + +## Algebra + +A1. Let $\mathbb{Q}_{>0}$ denote the set of all positive rational numbers. Determine all functions $f: \mathbb{Q}_{>0} \rightarrow \mathbb{Q}_{>0}$ satisfying + +$$ +f\left(x^{2} f(y)^{2}\right)=f(x)^{2} f(y) +$$ + +for all $x, y \in \mathbb{Q}_{>0}$. + +(Switzerland) + +Answer: $f(x)=1$ for all $x \in \mathbb{Q}_{>0}$. + +Solution. Take any $a, b \in \mathbb{Q}_{>0}$. By substituting $x=f(a), y=b$ and $x=f(b), y=a$ into $(*)$ we get + +$$ +f(f(a))^{2} f(b)=f\left(f(a)^{2} f(b)^{2}\right)=f(f(b))^{2} f(a) +$$ + +which yields + +$$ +\frac{f(f(a))^{2}}{f(a)}=\frac{f(f(b))^{2}}{f(b)} \quad \text { for all } a, b \in \mathbb{Q}_{>0} +$$ + +In other words, this shows that there exists a constant $C \in \mathbb{Q}_{>0}$ such that $f(f(a))^{2}=C f(a)$, or + +$$ +\left(\frac{f(f(a))}{C}\right)^{2}=\frac{f(a)}{C} \quad \text { for all } a \in \mathbb{Q}_{>0} \text {. } +$$ + +Denote by $f^{n}(x)=\underbrace{f(f(\ldots(f}_{n}(x)) \ldots))$ the $n^{\text {th }}$ iteration of $f$. Equality (1) yields + +$$ +\frac{f(a)}{C}=\left(\frac{f^{2}(a)}{C}\right)^{2}=\left(\frac{f^{3}(a)}{C}\right)^{4}=\cdots=\left(\frac{f^{n+1}(a)}{C}\right)^{2^{n}} +$$ + +for all positive integer $n$. So, $f(a) / C$ is the $2^{n}$-th power of a rational number for all positive integer $n$. This is impossible unless $f(a) / C=1$, since otherwise the exponent of some prime in the prime decomposition of $f(a) / C$ is not divisible by sufficiently large powers of 2 . Therefore, $f(a)=C$ for all $a \in \mathbb{Q}_{>0}$. + +Finally, after substituting $f \equiv C$ into (*) we get $C=C^{3}$, whence $C=1$. So $f(x) \equiv 1$ is the unique function satisfying $(*)$. + +Comment 1. There are several variations of the solution above. For instance, one may start with finding $f(1)=1$. To do this, let $d=f(1)$. By substituting $x=y=1$ and $x=d^{2}, y=1$ into (*) we get $f\left(d^{2}\right)=d^{3}$ and $f\left(d^{6}\right)=f\left(d^{2}\right)^{2} \cdot d=d^{7}$. By substituting now $x=1, y=d^{2}$ we obtain $f\left(d^{6}\right)=d^{2} \cdot d^{3}=d^{5}$. Therefore, $d^{7}=f\left(d^{6}\right)=d^{5}$, whence $d=1$. + +After that, the rest of the solution simplifies a bit, since we already know that $C=\frac{f(f(1))^{2}}{f(1)}=1$. Hence equation (1) becomes merely $f(f(a))^{2}=f(a)$, which yields $f(a)=1$ in a similar manner. + +Comment 2. There exist nonconstant functions $f: \mathbb{R}^{+} \rightarrow \mathbb{R}^{+}$satisfying $(*)$ for all real $x, y>0-$ e.g., $f(x)=\sqrt{x}$. + +A2. Find all positive integers $n \geqslant 3$ for which there exist real numbers $a_{1}, a_{2}, \ldots, a_{n}$, $a_{n+1}=a_{1}, a_{n+2}=a_{2}$ such that + +$$ +a_{i} a_{i+1}+1=a_{i+2} +$$ + +for all $i=1,2, \ldots, n$. + +(Slovakia) + +Answer: $n$ can be any multiple of 3 . + +Solution 1. For the sake of convenience, extend the sequence $a_{1}, \ldots, a_{n+2}$ to an infinite periodic sequence with period $n$. ( $n$ is not necessarily the shortest period.) + +If $n$ is divisible by 3 , then $\left(a_{1}, a_{2}, \ldots\right)=(-1,-1,2,-1,-1,2, \ldots)$ is an obvious solution. + +We will show that in every periodic sequence satisfying the recurrence, each positive term is followed by two negative values, and after them the next number is positive again. From this, it follows that $n$ is divisible by 3 . + +If the sequence contains two consecutive positive numbers $a_{i}, a_{i+1}$, then $a_{i+2}=a_{i} a_{i+1}+1>1$, so the next value is positive as well; by induction, all numbers are positive and greater than 1 . But then $a_{i+2}=a_{i} a_{i+1}+1 \geqslant 1 \cdot a_{i+1}+1>a_{i+1}$ for every index $i$, which is impossible: our sequence is periodic, so it cannot increase everywhere. + +If the number 0 occurs in the sequence, $a_{i}=0$ for some index $i$, then it follows that $a_{i+1}=a_{i-1} a_{i}+1$ and $a_{i+2}=a_{i} a_{i+1}+1$ are two consecutive positive elements in the sequences and we get the same contradiction again. + +Notice that after any two consecutive negative numbers the next one must be positive: if $a_{i}<0$ and $a_{i+1}<0$, then $a_{i+2}=a_{1} a_{i+1}+1>1>0$. Hence, the positive and negative numbers follow each other in such a way that each positive term is followed by one or two negative values and then comes the next positive term. + +Consider the case when the positive and negative values alternate. So, if $a_{i}$ is a negative value then $a_{i+1}$ is positive, $a_{i+2}$ is negative and $a_{i+3}$ is positive again. + +Notice that $a_{i} a_{i+1}+1=a_{i+2}<00$ we conclude $a_{i}1$. The number $a_{i+3}$ must be negative. We show that $a_{i+4}$ also must be negative. + +Notice that $a_{i+3}$ is negative and $a_{i+4}=a_{i+2} a_{i+3}+1<10, +$$ + +therefore $a_{i+5}>a_{i+4}$. Since at most one of $a_{i+4}$ and $a_{i+5}$ can be positive, that means that $a_{i+4}$ must be negative. + +Now $a_{i+3}$ and $a_{i+4}$ are negative and $a_{i+5}$ is positive; so after two negative and a positive terms, the next three terms repeat the same pattern. That completes the solution. + +Solution 2. We prove that the shortest period of the sequence must be 3 . Then it follows that $n$ must be divisible by 3 . + +Notice that the equation $x^{2}+1=x$ has no real root, so the numbers $a_{1}, \ldots, a_{n}$ cannot be all equal, hence the shortest period of the sequence cannot be 1 . + +By applying the recurrence relation for $i$ and $i+1$, + +$$ +\begin{gathered} +\left(a_{i+2}-1\right) a_{i+2}=a_{i} a_{i+1} a_{i+2}=a_{i}\left(a_{i+3}-1\right), \quad \text { so } \\ +a_{i+2}^{2}-a_{i} a_{i+3}=a_{i+2}-a_{i} . +\end{gathered} +$$ + +By summing over $i=1,2, \ldots, n$, we get + +$$ +\sum_{i=1}^{n}\left(a_{i}-a_{i+3}\right)^{2}=0 +$$ + +That proves that $a_{i}=a_{i+3}$ for every index $i$, so the sequence $a_{1}, a_{2}, \ldots$ is indeed periodic with period 3. The shortest period cannot be 1 , so it must be 3 ; therefore, $n$ is divisible by 3 . + +Comment. By solving the system of equations $a b+1=c, \quad b c+1=a, \quad c a+1=b$, it can be seen that the pattern $(-1,-1,2)$ is repeated in all sequences satisfying the problem conditions. + +A3. Given any set $S$ of positive integers, show that at least one of the following two assertions holds: + +(1) There exist distinct finite subsets $F$ and $G$ of $S$ such that $\sum_{x \in F} 1 / x=\sum_{x \in G} 1 / x$; + +(2) There exists a positive rational number $r<1$ such that $\sum_{x \in F} 1 / x \neq r$ for all finite subsets $F$ of $S$. + +## (Luxembourg) + +Solution 1. Argue indirectly. Agree, as usual, that the empty sum is 0 to consider rationals in $[0,1)$; adjoining 0 causes no harm, since $\sum_{x \in F} 1 / x=0$ for no nonempty finite subset $F$ of $S$. For every rational $r$ in $[0,1)$, let $F_{r}$ be the unique finite subset of $S$ such that $\sum_{x \in F_{r}} 1 / x=r$. The argument hinges on the lemma below. + +Lemma. If $x$ is a member of $S$ and $q$ and $r$ are rationals in $[0,1)$ such that $q-r=1 / x$, then $x$ is a member of $F_{q}$ if and only if it is not one of $F_{r}$. + +Proof. If $x$ is a member of $F_{q}$, then + +$$ +\sum_{y \in F_{q} \backslash\{x\}} \frac{1}{y}=\sum_{y \in F_{q}} \frac{1}{y}-\frac{1}{x}=q-\frac{1}{x}=r=\sum_{y \in F_{r}} \frac{1}{y} +$$ + +so $F_{r}=F_{q} \backslash\{x\}$, and $x$ is not a member of $F_{r}$. Conversely, if $x$ is not a member of $F_{r}$, then + +$$ +\sum_{y \in F_{r} \cup\{x\}} \frac{1}{y}=\sum_{y \in F_{r}} \frac{1}{y}+\frac{1}{x}=r+\frac{1}{x}=q=\sum_{y \in F_{q}} \frac{1}{y} +$$ + +so $F_{q}=F_{r} \cup\{x\}$, and $x$ is a member of $F_{q}$. + +Consider now an element $x$ of $S$ and a positive rational $r<1$. Let $n=\lfloor r x\rfloor$ and consider the sets $F_{r-k / x}, k=0, \ldots, n$. Since $0 \leqslant r-n / x<1 / x$, the set $F_{r-n / x}$ does not contain $x$, and a repeated application of the lemma shows that the $F_{r-(n-2 k) / x}$ do not contain $x$, whereas the $F_{r-(n-2 k-1) / x}$ do. Consequently, $x$ is a member of $F_{r}$ if and only if $n$ is odd. + +Finally, consider $F_{2 / 3}$. By the preceding, $\lfloor 2 x / 3\rfloor$ is odd for each $x$ in $F_{2 / 3}$, so $2 x / 3$ is not integral. Since $F_{2 / 3}$ is finite, there exists a positive rational $\varepsilon$ such that $\lfloor(2 / 3-\varepsilon) x\rfloor=\lfloor 2 x / 3\rfloor$ for all $x$ in $F_{2 / 3}$. This implies that $F_{2 / 3}$ is a subset of $F_{2 / 3-\varepsilon}$ which is impossible. + +Comment. The solution above can be adapted to show that the problem statement still holds, if the condition $r<1$ in (2) is replaced with $r<\delta$, for an arbitrary positive $\delta$. This yields that, if $S$ does not satisfy (1), then there exist infinitely many positive rational numbers $r<1$ such that $\sum_{x \in F} 1 / x \neq r$ for all finite subsets $F$ of $S$. + +Solution 2. A finite $S$ clearly satisfies (2), so let $S$ be infinite. If $S$ fails both conditions, so does $S \backslash\{1\}$. We may and will therefore assume that $S$ consists of integers greater than 1 . Label the elements of $S$ increasingly $x_{1}2 x_{n}$ for some $n$, then $\sum_{x \in F} 1 / x2$, we have $\Delta_{n} \leqslant \frac{n-1}{n} \Delta_{n-1}$. + +Proof. Choose positive integers $k, \ell \leqslant n$ such that $M_{n}=S(n, k) / k$ and $m_{n}=S(n, \ell) / \ell$. We have $S(n, k)=a_{n-1}+S(n-1, k-1)$, so + +$$ +k\left(M_{n}-a_{n-1}\right)=S(n, k)-k a_{n-1}=S(n-1, k-1)-(k-1) a_{n-1} \leqslant(k-1)\left(M_{n-1}-a_{n-1}\right), +$$ + +since $S(n-1, k-1) \leqslant(k-1) M_{n-1}$. Similarly, we get + +$$ +\ell\left(a_{n-1}-m_{n}\right)=(\ell-1) a_{n-1}-S(n-1, \ell-1) \leqslant(\ell-1)\left(a_{n-1}-m_{n-1}\right) . +$$ + +Since $m_{n-1} \leqslant a_{n-1} \leqslant M_{n-1}$ and $k, \ell \leqslant n$, the obtained inequalities yield + +$$ +\begin{array}{ll} +M_{n}-a_{n-1} \leqslant \frac{k-1}{k}\left(M_{n-1}-a_{n-1}\right) \leqslant \frac{n-1}{n}\left(M_{n-1}-a_{n-1}\right) \quad \text { and } \\ +a_{n-1}-m_{n} \leqslant \frac{\ell-1}{\ell}\left(a_{n-1}-m_{n-1}\right) \leqslant \frac{n-1}{n}\left(a_{n-1}-m_{n-1}\right) . +\end{array} +$$ + +Therefore, + +$$ +\Delta_{n}=\left(M_{n}-a_{n-1}\right)+\left(a_{n-1}-m_{n}\right) \leqslant \frac{n-1}{n}\left(\left(M_{n-1}-a_{n-1}\right)+\left(a_{n-1}-m_{n-1}\right)\right)=\frac{n-1}{n} \Delta_{n-1} . +$$ + +Back to the problem, if $a_{n}=1$ for all $n \leqslant 2017$, then $a_{2018} \leqslant 1$ and hence $a_{2018}-a_{2017} \leqslant 0$. Otherwise, let $2 \leqslant q \leqslant 2017$ be the minimal index with $a_{q}<1$. We have $S(q, i)=i$ for all $i=1,2, \ldots, q-1$, while $S(q, q)=q-1$. Therefore, $a_{q}<1$ yields $a_{q}=S(q, q) / q=1-\frac{1}{q}$. + +Now we have $S(q+1, i)=i-\frac{1}{q}$ for $i=1,2, \ldots, q$, and $S(q+1, q+1)=q-\frac{1}{q}$. This gives us + +$$ +m_{q+1}=\frac{S(q+1,1)}{1}=\frac{S(q+1, q+1)}{q+1}=\frac{q-1}{q} \quad \text { and } \quad M_{q+1}=\frac{S(q+1, q)}{q}=\frac{q^{2}-1}{q^{2}} +$$ + +so $\Delta_{q+1}=M_{q+1}-m_{q+1}=(q-1) / q^{2}$. Denoting $N=2017 \geqslant q$ and using Claim 1 for $n=q+2, q+3, \ldots, N+1$ we finally obtain + +$$ +\Delta_{N+1} \leqslant \frac{q-1}{q^{2}} \cdot \frac{q+1}{q+2} \cdot \frac{q+2}{q+3} \cdots \frac{N}{N+1}=\frac{1}{N+1}\left(1-\frac{1}{q^{2}}\right) \leqslant \frac{1}{N+1}\left(1-\frac{1}{N^{2}}\right)=\frac{N-1}{N^{2}} +$$ + +as required. + +Comment 1. One may check that the maximal value of $a_{2018}-a_{2017}$ is attained at the unique sequence, which is presented in the solution above. + +Comment 2. An easier question would be to determine the maximal value of $\left|a_{2018}-a_{2017}\right|$. In this version, the answer $\frac{1}{2018}$ is achieved at + +$$ +a_{1}=a_{2}=\cdots=a_{2017}=1, \quad a_{2018}=\frac{a_{2017}+\cdots+a_{0}}{2018}=1-\frac{1}{2018} . +$$ + +To prove that this value is optimal, it suffices to notice that $\Delta_{2}=\frac{1}{2}$ and to apply Claim 1 obtaining + +$$ +\left|a_{2018}-a_{2017}\right| \leqslant \Delta_{2018} \leqslant \frac{1}{2} \cdot \frac{2}{3} \cdots \frac{2017}{2018}=\frac{1}{2018} . +$$ + +Solution 2. We present a different proof of the estimate $a_{2018}-a_{2017} \leqslant \frac{2016}{2017^{2}}$. We keep the same notations of $S(n, k), m_{n}$ and $M_{n}$ from the previous solution. + +Notice that $S(n, n)=S(n, n-1)$, as $a_{0}=0$. Also notice that for $0 \leqslant k \leqslant \ell \leqslant n$ we have $S(n, \ell)=S(n, k)+S(n-k, \ell-k)$. + +Claim 2. For every positive integer $n$, we have $m_{n} \leqslant m_{n+1}$ and $M_{n+1} \leqslant M_{n}$, so the segment $\left[m_{n+1}, M_{n+1}\right]$ is contained in $\left[m_{n}, M_{n}\right]$. + +Proof. Choose a positive integer $k \leqslant n+1$ such that $m_{n+1}=S(n+1, k) / k$. Then we have + +$$ +k m_{n+1}=S(n+1, k)=a_{n}+S(n, k-1) \geqslant m_{n}+(k-1) m_{n}=k m_{n}, +$$ + +which establishes the first inequality in the Claim. The proof of the second inequality is similar. + +Claim 3. For every positive integers $k \geqslant n$, we have $m_{n} \leqslant a_{k} \leqslant M_{n}$. + +Proof. By Claim 2, we have $\left[m_{k}, M_{k}\right] \subseteq\left[m_{k-1}, M_{k-1}\right] \subseteq \cdots \subseteq\left[m_{n}, M_{n}\right]$. Since $a_{k} \in\left[m_{k}, M_{k}\right]$, the claim follows. + +Claim 4. For every integer $n \geqslant 2$, we have $M_{n}=S(n, n-1) /(n-1)$ and $m_{n}=S(n, n) / n$. + +Proof. We use induction on $n$. The base case $n=2$ is routine. To perform the induction step, we need to prove the inequalities + +$$ +\frac{S(n, n)}{n} \leqslant \frac{S(n, k)}{k} \quad \text { and } \quad \frac{S(n, k)}{k} \leqslant \frac{S(n, n-1)}{n-1} +$$ + +for every positive integer $k \leqslant n$. Clearly, these inequalities hold for $k=n$ and $k=n-1$, as $S(n, n)=S(n, n-1)>0$. In the sequel, we assume that $k0$. + +(South Korea) + +Answer: $f(x)=C_{1} x+\frac{C_{2}}{x}$ with arbitrary constants $C_{1}$ and $C_{2}$. + +Solution 1. Fix a real number $a>1$, and take a new variable $t$. For the values $f(t), f\left(t^{2}\right)$, $f(a t)$ and $f\left(a^{2} t^{2}\right)$, the relation (1) provides a system of linear equations: + +$$ +\begin{array}{llll} +x=y=t: & \left(t+\frac{1}{t}\right) f(t) & =f\left(t^{2}\right)+f(1) \\ +x=\frac{t}{a}, y=a t: & \left(\frac{t}{a}+\frac{a}{t}\right) f(a t) & =f\left(t^{2}\right)+f\left(a^{2}\right) \\ +x=a^{2} t, y=t: & \left(a^{2} t+\frac{1}{a^{2} t}\right) f(t) & =f\left(a^{2} t^{2}\right)+f\left(\frac{1}{a^{2}}\right) \\ +x=y=a t: & \left(a t+\frac{1}{a t}\right) f(a t) & =f\left(a^{2} t^{2}\right)+f(1) +\end{array} +$$ + +In order to eliminate $f\left(t^{2}\right)$, take the difference of (2a) and (2b); from (2c) and (2d) eliminate $f\left(a^{2} t^{2}\right)$; then by taking a linear combination, eliminate $f(a t)$ as well: + +$$ +\begin{gathered} +\left(t+\frac{1}{t}\right) f(t)-\left(\frac{t}{a}+\frac{a}{t}\right) f(a t)=f(1)-f\left(a^{2}\right) \text { and } \\ +\left(a^{2} t+\frac{1}{a^{2} t}\right) f(t)-\left(a t+\frac{1}{a t}\right) f(a t)=f\left(1 / a^{2}\right)-f(1), \text { so } \\ +\left(\left(a t+\frac{1}{a t}\right)\left(t+\frac{1}{t}\right)-\left(\frac{t}{a}+\frac{a}{t}\right)\left(a^{2} t+\frac{1}{a^{2} t}\right)\right) f(t) \\ +=\left(a t+\frac{1}{a t}\right)\left(f(1)-f\left(a^{2}\right)\right)-\left(\frac{t}{a}+\frac{a}{t}\right)\left(f\left(1 / a^{2}\right)-f(1)\right) . +\end{gathered} +$$ + +Notice that on the left-hand side, the coefficient of $f(t)$ is nonzero and does not depend on $t$ : + +$$ +\left(a t+\frac{1}{a t}\right)\left(t+\frac{1}{t}\right)-\left(\frac{t}{a}+\frac{a}{t}\right)\left(a^{2} t+\frac{1}{a^{2} t}\right)=a+\frac{1}{a}-\left(a^{3}+\frac{1}{a^{3}}\right)<0 . +$$ + +After dividing by this fixed number, we get + +$$ +f(t)=C_{1} t+\frac{C_{2}}{t} +$$ + +where the numbers $C_{1}$ and $C_{2}$ are expressed in terms of $a, f(1), f\left(a^{2}\right)$ and $f\left(1 / a^{2}\right)$, and they do not depend on $t$. + +The functions of the form (3) satisfy the equation: + +$$ +\left(x+\frac{1}{x}\right) f(y)=\left(x+\frac{1}{x}\right)\left(C_{1} y+\frac{C_{2}}{y}\right)=\left(C_{1} x y+\frac{C_{2}}{x y}\right)+\left(C_{1} \frac{y}{x}+C_{2} \frac{x}{y}\right)=f(x y)+f\left(\frac{y}{x}\right) . +$$ + +Solution 2. We start with an observation. If we substitute $x=a \neq 1$ and $y=a^{n}$ in (1), we obtain + +$$ +f\left(a^{n+1}\right)-\left(a+\frac{1}{a}\right) f\left(a^{n}\right)+f\left(a^{n-1}\right)=0 . +$$ + +For the sequence $z_{n}=a^{n}$, this is a homogeneous linear recurrence of the second order, and its characteristic polynomial is $t^{2}-\left(a+\frac{1}{a}\right) t+1=(t-a)\left(t-\frac{1}{a}\right)$ with two distinct nonzero roots, namely $a$ and $1 / a$. As is well-known, the general solution is $z_{n}=C_{1} a^{n}+C_{2}(1 / a)^{n}$ where the index $n$ can be as well positive as negative. Of course, the numbers $C_{1}$ and $C_{2}$ may depend of the choice of $a$, so in fact we have two functions, $C_{1}$ and $C_{2}$, such that + +$$ +f\left(a^{n}\right)=C_{1}(a) \cdot a^{n}+\frac{C_{2}(a)}{a^{n}} \quad \text { for every } a \neq 1 \text { and every integer } n \text {. } +$$ + +The relation (4) can be easily extended to rational values of $n$, so we may conjecture that $C_{1}$ and $C_{2}$ are constants, and whence $f(t)=C_{1} t+\frac{C_{2}}{t}$. As it was seen in the previous solution, such functions indeed satisfy (1). + +The equation (1) is linear in $f$; so if some functions $f_{1}$ and $f_{2}$ satisfy (1) and $c_{1}, c_{2}$ are real numbers, then $c_{1} f_{1}(x)+c_{2} f_{2}(x)$ is also a solution of (1). In order to make our formulas simpler, define + +$$ +f_{0}(x)=f(x)-f(1) \cdot x \text {. } +$$ + +This function is another one satisfying (1) and the extra constraint $f_{0}(1)=0$. Repeating the same argument on linear recurrences, we can write $f_{0}(a)=K(a) a^{n}+\frac{L(a)}{a^{n}}$ with some functions $K$ and $L$. By substituting $n=0$, we can see that $K(a)+L(a)=f_{0}(1)=0$ for every $a$. Hence, + +$$ +f_{0}\left(a^{n}\right)=K(a)\left(a^{n}-\frac{1}{a^{n}}\right) +$$ + +Now take two numbers $a>b>1$ arbitrarily and substitute $x=(a / b)^{n}$ and $y=(a b)^{n}$ in (1): + +$$ +\begin{aligned} +\left(\frac{a^{n}}{b^{n}}+\frac{b^{n}}{a^{n}}\right) f_{0}\left((a b)^{n}\right) & =f_{0}\left(a^{2 n}\right)+f_{0}\left(b^{2 n}\right), \quad \text { so } \\ +\left(\frac{a^{n}}{b^{n}}+\frac{b^{n}}{a^{n}}\right) K(a b)\left((a b)^{n}-\frac{1}{(a b)^{n}}\right) & =K(a)\left(a^{2 n}-\frac{1}{a^{2 n}}\right)+K(b)\left(b^{2 n}-\frac{1}{b^{2 n}}\right), \quad \text { or equivalently } \\ +K(a b)\left(a^{2 n}-\frac{1}{a^{2 n}}+b^{2 n}-\frac{1}{b^{2 n}}\right) & =K(a)\left(a^{2 n}-\frac{1}{a^{2 n}}\right)+K(b)\left(b^{2 n}-\frac{1}{b^{2 n}}\right) . +\end{aligned} +$$ + +By dividing (5) by $a^{2 n}$ and then taking limit with $n \rightarrow+\infty$ we get $K(a b)=K(a)$. Then (5) reduces to $K(a)=K(b)$. Hence, $K(a)=K(b)$ for all $a>b>1$. + +Fix $a>1$. For every $x>0$ there is some $b$ and an integer $n$ such that $10$, at least one of $a_{1}, \ldots, a_{n}$ is positive; without loss of generality suppose $a_{1} \geqslant 1$. + +Consider the polynomials $F_{1}=\Delta_{1} F$ and $G_{1}=\Delta G$. On the grid $\left\{0, \ldots, a_{1}-1\right\} \times\left\{0, \ldots, a_{2}\right\} \times$ $\ldots \times\left\{0, \ldots, a_{n}\right\}$ we have + +$$ +\begin{aligned} +F_{1}\left(x_{1}, \ldots, x_{n}\right) & =F\left(x_{1}+1, x_{2}, \ldots, x_{n}\right)-F\left(x_{1}, x_{2}, \ldots, x_{n}\right)= \\ +& =G\left(x_{1}+\ldots+x_{n}+1\right)-G\left(x_{1}+\ldots+x_{n}\right)=G_{1}\left(x_{1}+\ldots+x_{n}\right) . +\end{aligned} +$$ + +Since $G$ is nonconstant, we have $\operatorname{deg} G_{1}=\operatorname{deg} G-1 \leqslant\left(a_{1}-1\right)+a_{2}+\ldots+a_{n}$. Therefore we can apply the induction hypothesis to $F_{1}$ and $G_{1}$ and conclude that $F_{1}$ is not the zero polynomial and $\operatorname{deg} F_{1} \geqslant \operatorname{deg} G_{1}$. Hence, $\operatorname{deg} F \geqslant \operatorname{deg} F_{1}+1 \geqslant \operatorname{deg} G_{1}+1=\operatorname{deg} G$. That finishes the proof. + +To prove the problem statement, take the unique polynomial $g(x)$ so that $g(x)=\left\lfloor\frac{x}{m}\right\rfloor$ for $x \in\{0,1, \ldots, n(m-1)\}$ and $\operatorname{deg} g \leqslant n(m-1)$. Notice that precisely $n(m-1)+1$ values of $g$ are prescribed, so $g(x)$ indeed exists and is unique. Notice further that the constraints $g(0)=g(1)=0$ and $g(m)=1$ together enforce $\operatorname{deg} g \geqslant 2$. + +By applying the lemma to $a_{1}=\ldots=a_{n}=m-1$ and the polynomials $f$ and $g$, we achieve $\operatorname{deg} f \geqslant \operatorname{deg} g$. Hence we just need a suitable lower bound on $\operatorname{deg} g$. + +Consider the polynomial $h(x)=g(x+m)-g(x)-1$. The degree of $g(x+m)-g(x)$ is $\operatorname{deg} g-1 \geqslant 1$, so $\operatorname{deg} h=\operatorname{deg} g-1 \geqslant 1$, and therefore $h$ cannot be the zero polynomial. On the other hand, $h$ vanishes at the points $0,1, \ldots, n(m-1)-m$, so $h$ has at least $(n-1)(m-1)$ roots. Hence, + +$$ +\operatorname{deg} f \geqslant \operatorname{deg} g=\operatorname{deg} h+1 \geqslant(n-1)(m-1)+1 \geqslant n +$$ + +Comment 1. In the lemma we have equality for the choice $F\left(x_{1}, \ldots, x_{n}\right)=G\left(x_{1}+\ldots+x_{n}\right)$, so it indeed transforms the problem to an equivalent single-variable question. + +Comment 2. If $m \geqslant 3$, the polynomial $h(x)$ can be replaced by $\Delta g$. Notice that + +$$ +(\Delta g)(x)= \begin{cases}1 & \text { if } x \equiv-1 \quad(\bmod m) \quad \text { for } x=0,1, \ldots, n(m-1)-1 \\ 0 & \text { otherwise }\end{cases} +$$ + +Hence, $\Delta g$ vanishes at all integers $x$ with $0 \leqslant x0$ and $2\left(k^{2}+1\right)$, respectively; in this case, the answer becomes + +$$ +2 \sqrt[3]{\frac{(k+1)^{2}}{k}} +$$ + +Even further, a linear substitution allows to extend the solutions to a version with 7 and 100 being replaced with arbitrary positive real numbers $p$ and $q$ satisfying $q \geqslant 4 p$. + +## Combinatorics + +C1. Let $n \geqslant 3$ be an integer. Prove that there exists a set $S$ of $2 n$ positive integers satisfying the following property: For every $m=2,3, \ldots, n$ the set $S$ can be partitioned into two subsets with equal sums of elements, with one of subsets of cardinality $m$. + +(Iceland) + +Solution. We show that one of possible examples is the set + +$$ +S=\left\{1 \cdot 3^{k}, 2 \cdot 3^{k}: k=1,2, \ldots, n-1\right\} \cup\left\{1, \frac{3^{n}+9}{2}-1\right\} +$$ + +It is readily verified that all the numbers listed above are distinct (notice that the last two are not divisible by 3 ). + +The sum of elements in $S$ is + +$$ +\Sigma=1+\left(\frac{3^{n}+9}{2}-1\right)+\sum_{k=1}^{n-1}\left(1 \cdot 3^{k}+2 \cdot 3^{k}\right)=\frac{3^{n}+9}{2}+\sum_{k=1}^{n-1} 3^{k+1}=\frac{3^{n}+9}{2}+\frac{3^{n+1}-9}{2}=2 \cdot 3^{n} +$$ + +Hence, in order to show that this set satisfies the problem requirements, it suffices to present, for every $m=2,3, \ldots, n$, an $m$-element subset $A_{m} \subset S$ whose sum of elements equals $3^{n}$. + +Such a subset is + +$$ +A_{m}=\left\{2 \cdot 3^{k}: k=n-m+1, n-m+2, \ldots, n-1\right\} \cup\left\{1 \cdot 3^{n-m+1}\right\} . +$$ + +Clearly, $\left|A_{m}\right|=m$. The sum of elements in $A_{m}$ is + +$$ +3^{n-m+1}+\sum_{k=n-m+1}^{n-1} 2 \cdot 3^{k}=3^{n-m+1}+\frac{2 \cdot 3^{n}-2 \cdot 3^{n-m+1}}{2}=3^{n} +$$ + +as required. + +Comment. Let us present a more general construction. Let $s_{1}, s_{2}, \ldots, s_{2 n-1}$ be a sequence of pairwise distinct positive integers satisfying $s_{2 i+1}=s_{2 i}+s_{2 i-1}$ for all $i=2,3, \ldots, n-1$. Set $s_{2 n}=s_{1}+s_{2}+$ $\cdots+s_{2 n-4}$. + +Assume that $s_{2 n}$ is distinct from the other terms of the sequence. Then the set $S=\left\{s_{1}, s_{2}, \ldots, s_{2 n}\right\}$ satisfies the problem requirements. Indeed, the sum of its elements is + +$$ +\Sigma=\sum_{i=1}^{2 n-4} s_{i}+\left(s_{2 n-3}+s_{2 n-2}\right)+s_{2 n-1}+s_{2 n}=s_{2 n}+s_{2 n-1}+s_{2 n-1}+s_{2 n}=2 s_{2 n}+2 s_{2 n-1} +$$ + +Therefore, we have + +$$ +\frac{\Sigma}{2}=s_{2 n}+s_{2 n-1}=s_{2 n}+s_{2 n-2}+s_{2 n-3}=s_{2 n}+s_{2 n-2}+s_{2 n-4}+s_{2 n-5}=\ldots, +$$ + +which shows that the required sets $A_{m}$ can be chosen as + +$$ +A_{m}=\left\{s_{2 n}, s_{2 n-2}, \ldots, s_{2 n-2 m+4}, s_{2 n-2 m+3}\right\} +$$ + +So, the only condition to be satisfied is $s_{2 n} \notin\left\{s_{1}, s_{2}, \ldots, s_{2 n-1}\right\}$, which can be achieved in many different ways (e.g., by choosing properly the number $s_{1}$ after specifying $s_{2}, s_{3}, \ldots, s_{2 n-1}$ ). + +The solution above is an instance of this general construction. Another instance, for $n>3$, is the set + +$$ +\left\{F_{1}, F_{2}, \ldots, F_{2 n-1}, F_{1}+\cdots+F_{2 n-4}\right\}, +$$ + +where $F_{1}=1, F_{2}=2, F_{n+1}=F_{n}+F_{n-1}$ is the usual Fibonacci sequence. + +C2. Queenie and Horst play a game on a $20 \times 20$ chessboard. In the beginning the board is empty. In every turn, Horst places a black knight on an empty square in such a way that his new knight does not attack any previous knights. Then Queenie places a white queen on an empty square. The game gets finished when somebody cannot move. + +Find the maximal positive $K$ such that, regardless of the strategy of Queenie, Horst can put at least $K$ knights on the board. + +(Armenia) + +Answer: $K=20^{2} / 4=100$. In case of a $4 N \times 4 M$ board, the answer is $K=4 N M$. + +Solution. We show two strategies, one for Horst to place at least 100 knights, and another strategy for Queenie that prevents Horst from putting more than 100 knights on the board. + +A strategy for Horst: Put knights only on black squares, until all black squares get occupied. + +Colour the squares of the board black and white in the usual way, such that the white and black squares alternate, and let Horst put his knights on black squares as long as it is possible. Two knights on squares of the same colour never attack each other. The number of black squares is $20^{2} / 2=200$. The two players occupy the squares in turn, so Horst will surely find empty black squares in his first 100 steps. + +A strategy for Queenie: Group the squares into cycles of length 4, and after each step of Horst, occupy the opposite square in the same cycle. + +Consider the squares of the board as vertices of a graph; let two squares be connected if two knights on those squares would attack each other. Notice that in a $4 \times 4$ board the squares can be grouped into 4 cycles of length 4 , as shown in Figure 1. Divide the board into parts of size $4 \times 4$, and perform the same grouping in every part; this way we arrange the 400 squares of the board into 100 cycles (Figure 2). + +![](https://cdn.mathpix.com/cropped/2024_04_17_6da854f21230b13cd732g-27.jpg?height=360&width=351&top_left_y=1579&top_left_x=310) + +Figure 1 + +![](https://cdn.mathpix.com/cropped/2024_04_17_6da854f21230b13cd732g-27.jpg?height=362&width=420&top_left_y=1578&top_left_x=772) + +Figure 2 + +![](https://cdn.mathpix.com/cropped/2024_04_17_6da854f21230b13cd732g-27.jpg?height=357&width=463&top_left_y=1578&top_left_x=1296) + +Figure 3 + +The strategy of Queenie can be as follows: Whenever Horst puts a new knight to a certain square $A$, which is part of some cycle $A-B-C-D-A$, let Queenie put her queen on the opposite square $C$ in that cycle (Figure 3). From this point, Horst cannot put any knight on $A$ or $C$ because those squares are already occupied, neither on $B$ or $D$ because those squares are attacked by the knight standing on $A$. Hence, Horst can put at most one knight on each cycle, that is at most 100 knights in total. + +Comment 1. Queenie's strategy can be prescribed by a simple rule: divide the board into $4 \times 4$ parts; whenever Horst puts a knight in a part $P$, Queenie reflects that square about the centre of $P$ and puts her queen on the reflected square. + +Comment 2. The result remains the same if Queenie moves first. In the first turn, she may put her first queen arbitrarily. Later, if she has to put her next queen on a square that already contains a queen, she may move arbitrarily again. + +C3. Let $n$ be a given positive integer. Sisyphus performs a sequence of turns on a board consisting of $n+1$ squares in a row, numbered 0 to $n$ from left to right. Initially, $n$ stones are put into square 0, and the other squares are empty. At every turn, Sisyphus chooses any nonempty square, say with $k$ stones, takes one of those stones and moves it to the right by at most $k$ squares (the stone should stay within the board). Sisyphus' aim is to move all $n$ stones to square $n$. + +Prove that Sisyphus cannot reach the aim in less than + +$$ +\left\lceil\frac{n}{1}\right\rceil+\left\lceil\frac{n}{2}\right\rceil+\left\lceil\frac{n}{3}\right\rceil+\cdots+\left\lceil\frac{n}{n}\right\rceil +$$ + +turns. (As usual, $\lceil x\rceil$ stands for the least integer not smaller than $x$.) + +(Netherlands) + +Solution. The stones are indistinguishable, and all have the same origin and the same final position. So, at any turn we can prescribe which stone from the chosen square to move. We do it in the following manner. Number the stones from 1 to $n$. At any turn, after choosing a square, Sisyphus moves the stone with the largest number from this square. + +This way, when stone $k$ is moved from some square, that square contains not more than $k$ stones (since all their numbers are at most $k$ ). Therefore, stone $k$ is moved by at most $k$ squares at each turn. Since the total shift of the stone is exactly $n$, at least $\lceil n / k\rceil$ moves of stone $k$ should have been made, for every $k=1,2, \ldots, n$. + +By summing up over all $k=1,2, \ldots, n$, we get the required estimate. + +Comment. The original submission contained the second part, asking for which values of $n$ the equality can be achieved. The answer is $n=1,2,3,4,5,7$. The Problem Selection Committee considered this part to be less suitable for the competition, due to technicalities. + +C4. An anti-Pascal pyramid is a finite set of numbers, placed in a triangle-shaped array so that the first row of the array contains one number, the second row contains two numbers, the third row contains three numbers and so on; and, except for the numbers in the bottom row, each number equals the absolute value of the difference of the two numbers below it. For instance, the triangle below is an anti-Pascal pyramid with four rows, in which every integer from 1 to $1+2+3+4=10$ occurs exactly once: + +$$ +\begin{array}{ccccc} +& & & \\ +& 2 & 6 & \\ +& 5 & 7 & \\ +8 & 3 & 10 & 9 . +\end{array} +$$ + +Is it possible to form an anti-Pascal pyramid with 2018 rows, using every integer from 1 to $1+2+\cdots+2018$ exactly once? + +(Iran) + +Answer: No, it is not possible. + +Solution. Let $T$ be an anti-Pascal pyramid with $n$ rows, containing every integer from 1 to $1+2+\cdots+n$, and let $a_{1}$ be the topmost number in $T$ (Figure 1). The two numbers below $a_{1}$ are some $a_{2}$ and $b_{2}=a_{1}+a_{2}$, the two numbers below $b_{2}$ are some $a_{3}$ and $b_{3}=a_{1}+a_{2}+a_{3}$, and so on and so forth all the way down to the bottom row, where some $a_{n}$ and $b_{n}=a_{1}+a_{2}+\cdots+a_{n}$ are the two neighbours below $b_{n-1}=a_{1}+a_{2}+\cdots+a_{n-1}$. Since the $a_{k}$ are $n$ pairwise distinct positive integers whose sum does not exceed the largest number in $T$, which is $1+2+\cdots+n$, it follows that they form a permutation of $1,2, \ldots, n$. + +![](https://cdn.mathpix.com/cropped/2024_04_17_6da854f21230b13cd732g-29.jpg?height=391&width=465&top_left_y=1415&top_left_x=430) + +Figure 1 + +![](https://cdn.mathpix.com/cropped/2024_04_17_6da854f21230b13cd732g-29.jpg?height=394&width=463&top_left_y=1411&top_left_x=1162) + +Figure 2 + +Consider now (Figure 2) the two 'equilateral' subtriangles of $T$ whose bottom rows contain the numbers to the left, respectively right, of the pair $a_{n}, b_{n}$. (One of these subtriangles may very well be empty.) At least one of these subtriangles, say $T^{\prime}$, has side length $\ell \geqslant\lceil(n-2) / 2\rceil$. Since $T^{\prime}$ obeys the anti-Pascal rule, it contains $\ell$ pairwise distinct positive integers $a_{1}^{\prime}, a_{2}^{\prime}, \ldots, a_{\ell}^{\prime}$, where $a_{1}^{\prime}$ is at the apex, and $a_{k}^{\prime}$ and $b_{k}^{\prime}=a_{1}^{\prime}+a_{2}^{\prime}+\cdots+a_{k}^{\prime}$ are the two neighbours below $b_{k-1}^{\prime}$ for each $k=2,3 \ldots, \ell$. Since the $a_{k}$ all lie outside $T^{\prime}$, and they form a permutation of $1,2, \ldots, n$, the $a_{k}^{\prime}$ are all greater than $n$. Consequently, + +$$ +\begin{array}{r} +b_{\ell}^{\prime} \geqslant(n+1)+(n+2)+\cdots+(n+\ell)=\frac{\ell(2 n+\ell+1)}{2} \\ +\geqslant \frac{1}{2} \cdot \frac{n-2}{2}\left(2 n+\frac{n-2}{2}+1\right)=\frac{5 n(n-2)}{8}, +\end{array} +$$ + +which is greater than $1+2+\cdots+n=n(n+1) / 2$ for $n=2018$. A contradiction. + +Comment. The above estimate may be slightly improved by noticing that $b_{\ell}^{\prime} \neq b_{n}$. This implies $n(n+1) / 2=b_{n}>b_{\ell}^{\prime} \geqslant\lceil(n-2) / 2\rceil(2 n+\lceil(n-2) / 2\rceil+1) / 2$, so $n \leqslant 7$ if $n$ is odd, and $n \leqslant 12$ if $n$ is even. It seems that the largest anti-Pascal pyramid whose entries are a permutation of the integers from 1 to $1+2+\cdots+n$ has 5 rows. + +C5. Let $k$ be a positive integer. The organising committee of a tennis tournament is to schedule the matches for $2 k$ players so that every two players play once, each day exactly one match is played, and each player arrives to the tournament site the day of his first match, and departs the day of his last match. For every day a player is present on the tournament, the committee has to pay 1 coin to the hotel. The organisers want to design the schedule so as to minimise the total cost of all players' stays. Determine this minimum cost. + +(Russia) + +Answer: The required minimum is $k\left(4 k^{2}+k-1\right) / 2$. + +Solution 1. Enumerate the days of the tournament $1,2, \ldots,\left(\begin{array}{c}2 k \\ 2\end{array}\right)$. Let $b_{1} \leqslant b_{2} \leqslant \cdots \leqslant b_{2 k}$ be the days the players arrive to the tournament, arranged in nondecreasing order; similarly, let $e_{1} \geqslant \cdots \geqslant e_{2 k}$ be the days they depart arranged in nonincreasing order (it may happen that a player arrives on day $b_{i}$ and departs on day $e_{j}$, where $i \neq j$ ). If a player arrives on day $b$ and departs on day $e$, then his stay cost is $e-b+1$. Therefore, the total stay cost is + +$$ +\Sigma=\sum_{i=1}^{2 k} e_{i}-\sum_{i=1}^{2 k} b_{i}+n=\sum_{i=1}^{2 k}\left(e_{i}-b_{i}+1\right) +$$ + +Bounding the total cost from below. To this end, estimate $e_{i+1}-b_{i+1}+1$. Before day $b_{i+1}$, only $i$ players were present, so at most $\left(\begin{array}{l}i \\ 2\end{array}\right)$ matches could be played. Therefore, $b_{i+1} \leqslant\left(\begin{array}{l}i \\ 2\end{array}\right)+1$. Similarly, at most $\left(\begin{array}{l}i \\ 2\end{array}\right)$ matches could be played after day $e_{i+1}$, so $e_{i} \geqslant\left(\begin{array}{c}2 k \\ 2\end{array}\right)-\left(\begin{array}{l}i \\ 2\end{array}\right)$. Thus, + +$$ +e_{i+1}-b_{i+1}+1 \geqslant\left(\begin{array}{c} +2 k \\ +2 +\end{array}\right)-2\left(\begin{array}{l} +i \\ +2 +\end{array}\right)=k(2 k-1)-i(i-1) +$$ + +This lower bound can be improved for $i>k$ : List the $i$ players who arrived first, and the $i$ players who departed last; at least $2 i-2 k$ players appear in both lists. The matches between these players were counted twice, though the players in each pair have played only once. Therefore, if $i>k$, then + +$$ +e_{i+1}-b_{i+1}+1 \geqslant\left(\begin{array}{c} +2 k \\ +2 +\end{array}\right)-2\left(\begin{array}{c} +i \\ +2 +\end{array}\right)+\left(\begin{array}{c} +2 i-2 k \\ +2 +\end{array}\right)=(2 k-i)^{2} +$$ + +An optimal tournament, We now describe a schedule in which the lower bounds above are all achieved simultaneously. Split players into two groups $X$ and $Y$, each of cardinality $k$. Next, partition the schedule into three parts. During the first part, the players from $X$ arrive one by one, and each newly arrived player immediately plays with everyone already present. During the third part (after all players from $X$ have already departed) the players from $Y$ depart one by one, each playing with everyone still present just before departing. + +In the middle part, everyone from $X$ should play with everyone from $Y$. Let $S_{1}, S_{2}, \ldots, S_{k}$ be the players in $X$, and let $T_{1}, T_{2}, \ldots, T_{k}$ be the players in $Y$. Let $T_{1}, T_{2}, \ldots, T_{k}$ arrive in this order; after $T_{j}$ arrives, he immediately plays with all the $S_{i}, i>j$. Afterwards, players $S_{k}$, $S_{k-1}, \ldots, S_{1}$ depart in this order; each $S_{i}$ plays with all the $T_{j}, i \leqslant j$, just before his departure, and $S_{k}$ departs the day $T_{k}$ arrives. For $0 \leqslant s \leqslant k-1$, the number of matches played between $T_{k-s}$ 's arrival and $S_{k-s}$ 's departure is + +$$ +\sum_{j=k-s}^{k-1}(k-j)+1+\sum_{j=k-s}^{k-1}(k-j+1)=\frac{1}{2} s(s+1)+1+\frac{1}{2} s(s+3)=(s+1)^{2} +$$ + +Thus, if $i>k$, then the number of matches that have been played between $T_{i-k+1}$ 's arrival, which is $b_{i+1}$, and $S_{i-k+1}$ 's departure, which is $e_{i+1}$, is $(2 k-i)^{2}$; that is, $e_{i+1}-b_{i+1}+1=(2 k-i)^{2}$, showing the second lower bound achieved for all $i>k$. + +If $i \leqslant k$, then the matches between the $i$ players present before $b_{i+1}$ all fall in the first part of the schedule, so there are $\left(\begin{array}{l}i \\ 2\end{array}\right)$ such, and $b_{i+1}=\left(\begin{array}{l}i \\ 2\end{array}\right)+1$. Similarly, after $e_{i+1}$, there are $i$ players left, all $\left(\begin{array}{l}i \\ 2\end{array}\right)$ matches now fall in the third part of the schedule, and $e_{i+1}=\left(\begin{array}{c}2 k \\ 2\end{array}\right)-\left(\begin{array}{c}i \\ 2\end{array}\right)$. The first lower bound is therefore also achieved for all $i \leqslant k$. + +Consequently, all lower bounds are achieved simultaneously, and the schedule is indeed optimal. + +Evaluation. Finally, evaluate the total cost for the optimal schedule: + +$$ +\begin{aligned} +\Sigma & =\sum_{i=0}^{k}(k(2 k-1)-i(i-1))+\sum_{i=k+1}^{2 k-1}(2 k-i)^{2}=(k+1) k(2 k-1)-\sum_{i=0}^{k} i(i-1)+\sum_{j=1}^{k-1} j^{2} \\ +& =k(k+1)(2 k-1)-k^{2}+\frac{1}{2} k(k+1)=\frac{1}{2} k\left(4 k^{2}+k-1\right) . +\end{aligned} +$$ + +Solution 2. Consider any tournament schedule. Label players $P_{1}, P_{2}, \ldots, P_{2 k}$ in order of their arrival, and label them again $Q_{2 k}, Q_{2 k-1}, \ldots, Q_{1}$ in order of their departure, to define a permutation $a_{1}, a_{2}, \ldots, a_{2 k}$ of $1,2, \ldots, 2 k$ by $P_{i}=Q_{a_{i}}$. + +We first describe an optimal tournament for any given permutation $a_{1}, a_{2}, \ldots, a_{2 k}$ of the indices $1,2, \ldots, 2 k$. Next, we find an optimal permutation and an optimal tournament. + +Optimisation for a fixed $a_{1}, \ldots, a_{2 k}$. We say that the cost of the match between $P_{i}$ and $P_{j}$ is the number of players present at the tournament when this match is played. Clearly, the Committee pays for each day the cost of the match of that day. Hence, we are to minimise the total cost of all matches. + +Notice that $Q_{2 k}$ 's departure does not precede $P_{2 k}$ 's arrival. Hence, the number of players at the tournament monotonically increases (non-strictly) until it reaches $2 k$, and then monotonically decreases (non-strictly). So, the best time to schedule the match between $P_{i}$ and $P_{j}$ is either when $P_{\max (i, j)}$ arrives, or when $Q_{\max \left(a_{i}, a_{j}\right)}$ departs, in which case the cost is $\min \left(\max (i, j), \max \left(a_{i}, a_{j}\right)\right)$. + +Conversely, assuming that $i>j$, if this match is scheduled between the arrivals of $P_{i}$ and $P_{i+1}$, then its cost will be exactly $i=\max (i, j)$. Similarly, one can make it cost $\max \left(a_{i}, a_{j}\right)$. Obviously, these conditions can all be simultaneously satisfied, so the minimal cost for a fixed sequence $a_{1}, a_{2}, \ldots, a_{2 k}$ is + +$$ +\Sigma\left(a_{1}, \ldots, a_{2 k}\right)=\sum_{1 \leqslant ia b-a-b$ is expressible in the form $x=s a+t b$, with integer $s, t \geqslant 0$. + +We will prove by induction on $s+t$ that if $x=s a+b t$, with $s, t$ nonnegative integers, then + +$$ +f(x)>\frac{f(0)}{2^{s+t}}-2 . +$$ + +The base case $s+t=0$ is trivial. Assume now that (3) is true for $s+t=v$. Then, if $s+t=v+1$ and $x=s a+t b$, at least one of the numbers $s$ and $t$ - say $s$ - is positive, hence by (2), + +$$ +f(x)=f(s a+t b) \geqslant \frac{f((s-1) a+t b)}{2}-1>\frac{1}{2}\left(\frac{f(0)}{2^{s+t-1}}-2\right)-1=\frac{f(0)}{2^{s+t}}-2 . +$$ + +Assume now that we must perform moves of type (ii) ad infinitum. Take $n=a b-a-b$ and suppose $b>a$. Since each of the numbers $n+1, n+2, \ldots, n+b$ can be expressed in the form $s a+t b$, with $0 \leqslant s \leqslant b$ and $0 \leqslant t \leqslant a$, after moves of type (ii) have been performed $2^{a+b+1}$ times and we have to add a new pair of zeros, each $f(n+k), k=1,2, \ldots, b$, is at least 2 . In this case (1) yields inductively $f(n+k) \geqslant 2$ for all $k \geqslant 1$. But this is absurd: after a finite number of moves, $f$ cannot attain nonzero values at infinitely many points. + +Solution 2. We start by showing that the result of the process in the problem does not depend on the way the operations are performed. For that purpose, it is convenient to modify the process a bit. + +Claim 1. Suppose that the board initially contains a finite number of nonnegative integers, and one starts performing type $(i)$ moves only. Assume that one had applied $k$ moves which led to a final arrangement where no more type (i) moves are possible. Then, if one starts from the same initial arrangement, performing type (i) moves in an arbitrary fashion, then the process will necessarily stop at the same final arrangement + +Proof. Throughout this proof, all moves are supposed to be of type (i). + +Induct on $k$; the base case $k=0$ is trivial, since no moves are possible. Assume now that $k \geqslant 1$. Fix some canonical process, consisting of $k$ moves $M_{1}, M_{2}, \ldots, M_{k}$, and reaching the final arrangement $A$. Consider any sample process $m_{1}, m_{2}, \ldots$ starting with the same initial arrangement and proceeding as long as possible; clearly, it contains at least one move. We need to show that this process stops at $A$. + +Let move $m_{1}$ consist in replacing two copies of $x$ with $x+a$ and $x+b$. If move $M_{1}$ does the same, we may apply the induction hypothesis to the arrangement appearing after $m_{1}$. Otherwise, the canonical process should still contain at least one move consisting in replacing $(x, x) \mapsto(x+a, x+b)$, because the initial arrangement contains at least two copies of $x$, while the final one contains at most one such. + +Let $M_{i}$ be the first such move. Since the copies of $x$ are indistinguishable and no other copy of $x$ disappeared before $M_{i}$ in the canonical process, the moves in this process can be permuted as $M_{i}, M_{1}, \ldots, M_{i-1}, M_{i+1}, \ldots, M_{k}$, without affecting the final arrangement. Now it suffices to perform the move $m_{1}=M_{i}$ and apply the induction hypothesis as above. + +Claim 2. Consider any process starting from the empty board, which involved exactly $n$ moves of type (ii) and led to a final arrangement where all the numbers are distinct. Assume that one starts with the board containing $2 n$ zeroes (as if $n$ moves of type (ii) were made in the beginning), applying type ( $i$ ) moves in an arbitrary way. Then this process will reach the same final arrangement. + +Proof. Starting with the board with $2 n$ zeros, one may indeed model the first process mentioned in the statement of the claim, omitting the type (ii) moves. This way, one reaches the same final arrangement. Now, Claim 1 yields that this final arrangement will be obtained when type (i) moves are applied arbitrarily. + +Claim 2 allows now to reformulate the problem statement as follows: There exists an integer $n$ such that, starting from $2 n$ zeroes, one may apply type (i) moves indefinitely. + +In order to prove this, we start with an obvious induction on $s+t=k \geqslant 1$ to show that if we start with $2^{s+t}$ zeros, then we can get simultaneously on the board, at some point, each of the numbers $s a+t b$, with $s+t=k$. + +Suppose now that $a4$, one can choose a point $A_{n+1}$ on the small arc $\widehat{C A_{n}}$, close enough to $C$, so that $A_{n} B+A_{n} A_{n+1}+B A_{n+1}$ is still greater than 4 . Thus each of these new triangles $B A_{n} A_{n+1}$ and $B A_{n+1} C$ has perimeter greater than 4, which completes the induction step. + +![](https://cdn.mathpix.com/cropped/2024_04_17_6da854f21230b13cd732g-42.jpg?height=366&width=691&top_left_y=1353&top_left_x=682) + +We proceed by showing that no $t>4$ satisfies the conditions of the problem. To this end, we assume that there exists a good collection $T$ of $n$ triangles, each of perimeter greater than $t$, and then bound $n$ from above. + +Take $\varepsilon>0$ such that $t=4+2 \varepsilon$. + +Claim. There exists a positive constant $\sigma=\sigma(\varepsilon)$ such that any triangle $\Delta$ with perimeter $2 s \geqslant 4+2 \varepsilon$, inscribed in $\omega$, has area $S(\Delta)$ at least $\sigma$. + +Proof. Let $a, b, c$ be the side lengths of $\Delta$. Since $\Delta$ is inscribed in $\omega$, each side has length at most 2. Therefore, $s-a \geqslant(2+\varepsilon)-2=\varepsilon$. Similarly, $s-b \geqslant \varepsilon$ and $s-c \geqslant \varepsilon$. By Heron's formula, $S(\Delta)=\sqrt{s(s-a)(s-b)(s-c)} \geqslant \sqrt{(2+\varepsilon) \varepsilon^{3}}$. Thus we can set $\sigma(\varepsilon)=\sqrt{(2+\varepsilon) \varepsilon^{3}}$. + +Now we see that the total area $S$ of all triangles from $T$ is at least $n \sigma(\varepsilon)$. On the other hand, $S$ does not exceed the area of the disk bounded by $\omega$. Thus $n \sigma(\varepsilon) \leqslant \pi$, which means that $n$ is bounded from above. + +Comment 1. One may prove the Claim using the formula $S=\frac{a b c}{4 R}$ instead of Heron's formula. + +Comment 2. In the statement of the problem condition $(i)$ could be replaced by a weaker one: each triangle from $T$ lies within $\omega$. This does not affect the solution above, but reduces the number of ways to prove the Claim. + +This page is intentionally left blank + +G4. A point $T$ is chosen inside a triangle $A B C$. Let $A_{1}, B_{1}$, and $C_{1}$ be the reflections of $T$ in $B C, C A$, and $A B$, respectively. Let $\Omega$ be the circumcircle of the triangle $A_{1} B_{1} C_{1}$. The lines $A_{1} T, B_{1} T$, and $C_{1} T$ meet $\Omega$ again at $A_{2}, B_{2}$, and $C_{2}$, respectively. Prove that the lines $A A_{2}, B B_{2}$, and $C C_{2}$ are concurrent on $\Omega$. + +(Mongolia) + +Solution. By $\Varangle(\ell, n)$ we always mean the directed angle of the lines $\ell$ and $n$, taken modulo $180^{\circ}$. + +Let $C C_{2}$ meet $\Omega$ again at $K$ (as usual, if $C C_{2}$ is tangent to $\Omega$, we set $T=C_{2}$ ). We show that the line $B B_{2}$ contains $K$; similarly, $A A_{2}$ will also pass through $K$. For this purpose, it suffices to prove that + +$$ +\Varangle\left(C_{2} C, C_{2} A_{1}\right)=\Varangle\left(B_{2} B, B_{2} A_{1}\right) . +$$ + +By the problem condition, $C B$ and $C A$ are the perpendicular bisectors of $T A_{1}$ and $T B_{1}$, respectively. Hence, $C$ is the circumcentre of the triangle $A_{1} T B_{1}$. Therefore, + +$$ +\Varangle\left(C A_{1}, C B\right)=\Varangle(C B, C T)=\Varangle\left(B_{1} A_{1}, B_{1} T\right)=\Varangle\left(B_{1} A_{1}, B_{1} B_{2}\right) . +$$ + +In circle $\Omega$ we have $\Varangle\left(B_{1} A_{1}, B_{1} B_{2}\right)=\Varangle\left(C_{2} A_{1}, C_{2} B_{2}\right)$. Thus, + +$$ +\Varangle\left(C A_{1}, C B\right)=\Varangle\left(B_{1} A_{1}, B_{1} B_{2}\right)=\Varangle\left(C_{2} A_{1}, C_{2} B_{2}\right) . +$$ + +Similarly, we get + +$$ +\Varangle\left(B A_{1}, B C\right)=\Varangle\left(C_{1} A_{1}, C_{1} C_{2}\right)=\Varangle\left(B_{2} A_{1}, B_{2} C_{2}\right) . +$$ + +The two obtained relations yield that the triangles $A_{1} B C$ and $A_{1} B_{2} C_{2}$ are similar and equioriented, hence + +$$ +\frac{A_{1} B_{2}}{A_{1} B}=\frac{A_{1} C_{2}}{A_{1} C} \quad \text { and } \quad \Varangle\left(A_{1} B, A_{1} C\right)=\Varangle\left(A_{1} B_{2}, A_{1} C_{2}\right) +$$ + +The second equality may be rewritten as $\Varangle\left(A_{1} B, A_{1} B_{2}\right)=\Varangle\left(A_{1} C, A_{1} C_{2}\right)$, so the triangles $A_{1} B B_{2}$ and $A_{1} C C_{2}$ are also similar and equioriented. This establishes (1). + +![](https://cdn.mathpix.com/cropped/2024_04_17_6da854f21230b13cd732g-44.jpg?height=839&width=1171&top_left_y=1782&top_left_x=448) + +Comment 1. In fact, the triangle $A_{1} B C$ is an image of $A_{1} B_{2} C_{2}$ under a spiral similarity centred at $A_{1}$; in this case, the triangles $A B B_{2}$ and $A C C_{2}$ are also spirally similar with the same centre. + +Comment 2. After obtaining (2) and (3), one can finish the solution in different ways. + +For instance, introducing the point $X=B C \cap B_{2} C_{2}$, one gets from these relations that the 4-tuples $\left(A_{1}, B, B_{2}, X\right)$ and $\left(A_{1}, C, C_{2}, X\right)$ are both cyclic. Therefore, $K$ is the Miquel point of the lines $B B_{2}$, $C C_{2}, B C$, and $B_{2} C_{2}$; this yields that the meeting point of $B B_{2}$ and $C C_{2}$ lies on $\Omega$. + +Yet another way is to show that the points $A_{1}, B, C$, and $K$ are concyclic, as + +$$ +\Varangle\left(K C, K A_{1}\right)=\Varangle\left(B_{2} C_{2}, B_{2} A_{1}\right)=\Varangle\left(B C, B A_{1}\right) . +$$ + +By symmetry, the second point $K^{\prime}$ of intersection of $B B_{2}$ with $\Omega$ is also concyclic to $A_{1}, B$, and $C$, hence $K^{\prime}=K$. + +![](https://cdn.mathpix.com/cropped/2024_04_17_6da854f21230b13cd732g-45.jpg?height=877&width=1188&top_left_y=681&top_left_x=434) + +Comment 3. The requirement that the common point of the lines $A A_{2}, B B_{2}$, and $C C_{2}$ should lie on $\Omega$ may seem to make the problem easier, since it suggests some approaches. On the other hand, there are also different ways of showing that the lines $A A_{2}, B B_{2}$, and $C C_{2}$ are just concurrent. + +In particular, the problem conditions yield that the lines $A_{2} T, B_{2} T$, and $C_{2} T$ are perpendicular to the corresponding sides of the triangle $A B C$. One may show that the lines $A T, B T$, and $C T$ are also perpendicular to the corresponding sides of the triangle $A_{2} B_{2} C_{2}$, i.e., the triangles $A B C$ and $A_{2} B_{2} C_{2}$ are orthologic, and their orthology centres coincide. It is known that such triangles are also perspective, i.e. the lines $A A_{2}, B B_{2}$, and $C C_{2}$ are concurrent (in projective sense). + +To show this mutual orthology, one may again apply angle chasing, but there are also other methods. Let $A^{\prime}, B^{\prime}$, and $C^{\prime}$ be the projections of $T$ onto the sides of the triangle $A B C$. Then $A_{2} T \cdot T A^{\prime}=$ $B_{2} T \cdot T B^{\prime}=C_{2} T \cdot T C^{\prime}$, since all three products equal (minus) half the power of $T$ with respect to $\Omega$. This means that $A_{2}, B_{2}$, and $C_{2}$ are the poles of the sidelines of the triangle $A B C$ with respect to some circle centred at $T$ and having pure imaginary radius (in other words, the reflections of $A_{2}, B_{2}$, and $C_{2}$ in $T$ are the poles of those sidelines with respect to some regular circle centred at $T$ ). Hence, dually, the vertices of the triangle $A B C$ are also the poles of the sidelines of the triangle $A_{2} B_{2} C_{2}$. + +G5. Let $A B C$ be a triangle with circumcircle $\omega$ and incentre $I$. A line $\ell$ intersects the lines $A I, B I$, and $C I$ at points $D, E$, and $F$, respectively, distinct from the points $A, B, C$, and $I$. The perpendicular bisectors $x, y$, and $z$ of the segments $A D, B E$, and $C F$, respectively determine a triangle $\Theta$. Show that the circumcircle of the triangle $\Theta$ is tangent to $\omega$. + +(Denmark) + +Preamble. Let $X=y \cap z, Y=x \cap z, Z=x \cap y$ and let $\Omega$ denote the circumcircle of the triangle $X Y Z$. Denote by $X_{0}, Y_{0}$, and $Z_{0}$ the second intersection points of $A I, B I$ and $C I$, respectively, with $\omega$. It is known that $Y_{0} Z_{0}$ is the perpendicular bisector of $A I, Z_{0} X_{0}$ is the perpendicular bisector of $B I$, and $X_{0} Y_{0}$ is the perpendicular bisector of $C I$. In particular, the triangles $X Y Z$ and $X_{0} Y_{0} Z_{0}$ are homothetic, because their corresponding sides are parallel. + +The solutions below mostly exploit the following approach. Consider the triangles $X Y Z$ and $X_{0} Y_{0} Z_{0}$, or some other pair of homothetic triangles $\Delta$ and $\delta$ inscribed into $\Omega$ and $\omega$, respectively. In order to prove that $\Omega$ and $\omega$ are tangent, it suffices to show that the centre $T$ of the homothety taking $\Delta$ to $\delta$ lies on $\omega$ (or $\Omega$ ), or, in other words, to show that $\Delta$ and $\delta$ are perspective (i.e., the lines joining corresponding vertices are concurrent), with their perspector lying on $\omega$ (or $\Omega$ ). + +We use directed angles throughout all the solutions. + +## Solution 1. + +Claim 1. The reflections $\ell_{a}, \ell_{b}$ and $\ell_{c}$ of the line $\ell$ in the lines $x, y$, and $z$, respectively, are concurrent at a point $T$ which belongs to $\omega$. + +![](https://cdn.mathpix.com/cropped/2024_04_17_6da854f21230b13cd732g-46.jpg?height=831&width=916&top_left_y=1338&top_left_x=570) + +Proof. Notice that $\Varangle\left(\ell_{b}, \ell_{c}\right)=\Varangle\left(\ell_{b}, \ell\right)+\Varangle\left(\ell, \ell_{c}\right)=2 \Varangle(y, \ell)+2 \Varangle(\ell, z)=2 \Varangle(y, z)$. But $y \perp B I$ and $z \perp C I$ implies $\Varangle(y, z)=\Varangle(B I, I C)$, so, since $2 \Varangle(B I, I C)=\Varangle(B A, A C)$, we obtain + +$$ +\Varangle\left(\ell_{b}, \ell_{c}\right)=\Varangle(B A, A C) . +$$ + +Since $A$ is the reflection of $D$ in $x$, $A$ belongs to $\ell_{a}$; similarly, $B$ belongs to $\ell_{b}$. Then (1) shows that the common point $T^{\prime}$ of $\ell_{a}$ and $\ell_{b}$ lies on $\omega$; similarly, the common point $T^{\prime \prime}$ of $\ell_{c}$ and $\ell_{b}$ lies on $\omega$. + +If $B \notin \ell_{a}$ and $B \notin \ell_{c}$, then $T^{\prime}$ and $T^{\prime \prime}$ are the second point of intersection of $\ell_{b}$ and $\omega$, hence they coincide. Otherwise, if, say, $B \in \ell_{c}$, then $\ell_{c}=B C$, so $\Varangle(B A, A C)=\Varangle\left(\ell_{b}, \ell_{c}\right)=\Varangle\left(\ell_{b}, B C\right)$, which shows that $\ell_{b}$ is tangent at $B$ to $\omega$ and $T^{\prime}=T^{\prime \prime}=B$. So $T^{\prime}$ and $T^{\prime \prime}$ coincide in all the cases, and the conclusion of the claim follows. + +Now we prove that $X, X_{0}, T$ are collinear. Denote by $D_{b}$ and $D_{c}$ the reflections of the point $D$ in the lines $y$ and $z$, respectively. Then $D_{b}$ lies on $\ell_{b}, D_{c}$ lies on $\ell_{c}$, and + +$$ +\begin{aligned} +\Varangle\left(D_{b} X, X D_{c}\right) & =\Varangle\left(D_{b} X, D X\right)+\Varangle\left(D X, X D_{c}\right)=2 \Varangle(y, D X)+2 \Varangle(D X, z)=2 \Varangle(y, z) \\ +& =\Varangle(B A, A C)=\Varangle(B T, T C), +\end{aligned} +$$ + +hence the quadrilateral $X D_{b} T D_{c}$ is cyclic. Notice also that since $X D_{b}=X D=X D_{c}$, the points $D, D_{b}, D_{c}$ lie on a circle with centre $X$. Using in this circle the diameter $D_{c} D_{c}^{\prime}$ yields $\Varangle\left(D_{b} D_{c}, D_{c} X\right)=90^{\circ}+\Varangle\left(D_{b} D_{c}^{\prime}, D_{c}^{\prime} X\right)=90^{\circ}+\Varangle\left(D_{b} D, D D_{c}\right)$. Therefore, + +$$ +\begin{gathered} +\Varangle\left(\ell_{b}, X T\right)=\Varangle\left(D_{b} T, X T\right)=\Varangle\left(D_{b} D_{c}, D_{c} X\right)=90^{\circ}+\Varangle\left(D_{b} D, D D_{c}\right) \\ +=90^{\circ}+\Varangle(B I, I C)=\Varangle(B A, A I)=\Varangle\left(B A, A X_{0}\right)=\Varangle\left(B T, T X_{0}\right)=\Varangle\left(\ell_{b}, X_{0} T\right) +\end{gathered} +$$ + +so the points $X, X_{0}, T$ are collinear. By a similar argument, $Y, Y_{0}, T$ and $Z, Z_{0}, T$ are collinear. As mentioned in the preamble, the statement of the problem follows. + +Comment 1. After proving Claim 1 one may proceed in another way. As it was shown, the reflections of $\ell$ in the sidelines of $X Y Z$ are concurrent at $T$. Thus $\ell$ is the Steiner line of $T$ with respect to $\triangle X Y Z$ (that is the line containing the reflections $T_{a}, T_{b}, T_{c}$ of $T$ in the sidelines of $X Y Z$ ). The properties of the Steiner line imply that $T$ lies on $\Omega$, and $\ell$ passes through the orthocentre $H$ of the triangle $X Y Z$. + +![](https://cdn.mathpix.com/cropped/2024_04_17_6da854f21230b13cd732g-47.jpg?height=811&width=1311&top_left_y=1248&top_left_x=378) + +Let $H_{a}, H_{b}$, and $H_{c}$ be the reflections of the point $H$ in the lines $x, y$, and $z$, respectively. Then the triangle $H_{a} H_{b} H_{c}$ is inscribed in $\Omega$ and homothetic to $A B C$ (by an easy angle chasing). Since $H_{a} \in \ell_{a}, H_{b} \in \ell_{b}$, and $H_{c} \in \ell_{c}$, the triangles $H_{a} H_{b} H_{c}$ and $A B C$ form a required pair of triangles $\Delta$ and $\delta$ mentioned in the preamble. + +Comment 2. The following observation shows how one may guess the description of the tangency point $T$ from Solution 1. + +Let us fix a direction and move the line $\ell$ parallel to this direction with constant speed. + +Then the points $D, E$, and $F$ are moving with constant speeds along the lines $A I, B I$, and $C I$, respectively. In this case $x, y$, and $z$ are moving with constant speeds, defining a family of homothetic triangles $X Y Z$ with a common centre of homothety $T$. Notice that the triangle $X_{0} Y_{0} Z_{0}$ belongs to this family (for $\ell$ passing through $I$ ). We may specify the location of $T$ considering the degenerate case when $x, y$, and $z$ are concurrent. In this degenerate case all the lines $x, y, z, \ell, \ell_{a}, \ell_{b}, \ell_{c}$ have a common point. Note that the lines $\ell_{a}, \ell_{b}, \ell_{c}$ remain constant as $\ell$ is moving (keeping its direction). Thus $T$ should be the common point of $\ell_{a}, \ell_{b}$, and $\ell_{c}$, lying on $\omega$. + +Solution 2. As mentioned in the preamble, it is sufficient to prove that the centre $T$ of the homothety taking $X Y Z$ to $X_{0} Y_{0} Z_{0}$ belongs to $\omega$. Thus, it suffices to prove that $\Varangle\left(T X_{0}, T Y_{0}\right)=$ $\Varangle\left(Z_{0} X_{0}, Z_{0} Y_{0}\right)$, or, equivalently, $\Varangle\left(X X_{0}, Y Y_{0}\right)=\Varangle\left(Z_{0} X_{0}, Z_{0} Y_{0}\right)$. + +Recall that $Y Z$ and $Y_{0} Z_{0}$ are the perpendicular bisectors of $A D$ and $A I$, respectively. Then, the vector $\vec{x}$ perpendicular to $Y Z$ and shifting the line $Y_{0} Z_{0}$ to $Y Z$ is equal to $\frac{1}{2} \overrightarrow{I D}$. Define the shifting vectors $\vec{y}=\frac{1}{2} \overrightarrow{I E}, \vec{z}=\frac{1}{2} \overrightarrow{I F}$ similarly. Consider now the triangle $U V W$ formed by the perpendiculars to $A I, B I$, and $C I$ through $D, E$, and $F$, respectively (see figure below). This is another triangle whose sides are parallel to the corresponding sides of $X Y Z$. + +Claim 2. $\overrightarrow{I U}=2 \overrightarrow{X_{0} X}, \overrightarrow{I V}=2 \overrightarrow{Y_{0} Y}, \overrightarrow{I W}=2 \overrightarrow{Z_{0} Z}$. + +Proof. We prove one of the relations, the other proofs being similar. To prove the equality of two vectors it suffices to project them onto two non-parallel axes and check that their projections are equal. + +The projection of $\overrightarrow{X_{0} X}$ onto $I B$ equals $\vec{y}$, while the projection of $\overrightarrow{I U}$ onto $I B$ is $\overrightarrow{I E}=2 \vec{y}$. The projections onto the other axis $I C$ are $\vec{z}$ and $\overrightarrow{I F}=2 \vec{z}$. Then $\overrightarrow{I U}=2 \overrightarrow{X_{0} X}$ follows. + +Notice that the line $\ell$ is the Simson line of the point $I$ with respect to the triangle $U V W$; thus $U, V, W$, and $I$ are concyclic. It follows from Claim 2 that $\Varangle\left(X X_{0}, Y Y_{0}\right)=\Varangle(I U, I V)=$ $\Varangle(W U, W V)=\Varangle\left(Z_{0} X_{0}, Z_{0} Y_{0}\right)$, and we are done. + +![](https://cdn.mathpix.com/cropped/2024_04_17_6da854f21230b13cd732g-48.jpg?height=888&width=1106&top_left_y=1161&top_left_x=475) + +Solution 3. Let $I_{a}, I_{b}$, and $I_{c}$ be the excentres of triangle $A B C$ corresponding to $A, B$, and $C$, respectively. Also, let $u, v$, and $w$ be the lines through $D, E$, and $F$ which are perpendicular to $A I, B I$, and $C I$, respectively, and let $U V W$ be the triangle determined by these lines, where $u=V W, v=U W$ and $w=U V$ (see figure above). + +Notice that the line $u$ is the reflection of $I_{b} I_{c}$ in the line $x$, because $u, x$, and $I_{b} I_{c}$ are perpendicular to $A D$ and $x$ is the perpendicular bisector of $A D$. Likewise, $v$ and $I_{a} I_{c}$ are reflections of each other in $y$, while $w$ and $I_{a} I_{b}$ are reflections of each other in $z$. It follows that $X, Y$, and $Z$ are the midpoints of $U I_{a}, V I_{b}$ and $W I_{c}$, respectively, and that the triangles $U V W$, $X Y Z$ and $I_{a} I_{b} I_{c}$ are either translates of each other or homothetic with a common homothety centre. + +Construct the points $T$ and $S$ such that the quadrilaterals $U V I W, X Y T Z$ and $I_{a} I_{b} S I_{c}$ are homothetic. Then $T$ is the midpoint of $I S$. Moreover, note that $\ell$ is the Simson line of the point $I$ with respect to the triangle $U V W$, hence $I$ belongs to the circumcircle of the triangle $U V W$, therefore $T$ belongs to $\Omega$. + +Consider now the homothety or translation $h_{1}$ that maps $X Y Z T$ to $I_{a} I_{b} I_{c} S$ and the homothety $h_{2}$ with centre $I$ and factor $\frac{1}{2}$. Furthermore, let $h=h_{2} \circ h_{1}$. The transform $h$ can be a homothety or a translation, and + +$$ +h(T)=h_{2}\left(h_{1}(T)\right)=h_{2}(S)=T +$$ + +hence $T$ is a fixed point of $h$. So, $h$ is a homothety with centre $T$. Note that $h_{2}$ maps the excentres $I_{a}, I_{b}, I_{c}$ to $X_{0}, Y_{0}, Z_{0}$ defined in the preamble. Thus the centre $T$ of the homothety taking $X Y Z$ to $X_{0} Y_{0} Z_{0}$ belongs to $\Omega$, and this completes the proof. + +G6. A convex quadrilateral $A B C D$ satisfies $A B \cdot C D=B C \cdot D A$. A point $X$ is chosen inside the quadrilateral so that $\angle X A B=\angle X C D$ and $\angle X B C=\angle X D A$. Prove that $\angle A X B+$ $\angle C X D=180^{\circ}$. + +(Poland) + +Solution 1. Let $B^{\prime}$ be the reflection of $B$ in the internal angle bisector of $\angle A X C$, so that $\angle A X B^{\prime}=\angle C X B$ and $\angle C X B^{\prime}=\angle A X B$. If $X, D$, and $B^{\prime}$ are collinear, then we are done. Now assume the contrary. + +On the ray $X B^{\prime}$ take a point $E$ such that $X E \cdot X B=X A \cdot X C$, so that $\triangle A X E \sim$ $\triangle B X C$ and $\triangle C X E \sim \triangle B X A$. We have $\angle X C E+\angle X C D=\angle X B A+\angle X A B<180^{\circ}$ and $\angle X A E+\angle X A D=\angle X D A+\angle X A D<180^{\circ}$, which proves that $X$ lies inside the angles $\angle E C D$ and $\angle E A D$ of the quadrilateral $E A D C$. Moreover, $X$ lies in the interior of exactly one of the two triangles $E A D, E C D$ (and in the exterior of the other). + +![](https://cdn.mathpix.com/cropped/2024_04_17_6da854f21230b13cd732g-50.jpg?height=491&width=600&top_left_y=891&top_left_x=728) + +The similarities mentioned above imply $X A \cdot B C=X B \cdot A E$ and $X B \cdot C E=X C \cdot A B$. Multiplying these equalities with the given equality $A B \cdot C D=B C \cdot D A$, we obtain $X A \cdot C D$. $C E=X C \cdot A D \cdot A E$, or, equivalently, + +$$ +\frac{X A \cdot D E}{A D \cdot A E}=\frac{X C \cdot D E}{C D \cdot C E} +$$ + +Lemma. Let $P Q R$ be a triangle, and let $X$ be a point in the interior of the angle $Q P R$ such that $\angle Q P X=\angle P R X$. Then $\frac{P X \cdot Q R}{P Q \cdot P R}<1$ if and only if $X$ lies in the interior of the triangle $P Q R$. Proof. The locus of points $X$ with $\angle Q P X=\angle P R X$ lying inside the angle $Q P R$ is an arc $\alpha$ of the circle $\gamma$ through $R$ tangent to $P Q$ at $P$. Let $\gamma$ intersect the line $Q R$ again at $Y$ (if $\gamma$ is tangent to $Q R$, then set $Y=R)$. The similarity $\triangle Q P Y \sim \triangle Q R P$ yields $P Y=\frac{P Q \cdot P R}{Q R}$. Now it suffices to show that $P X

P Y$ for $X \in \overparen{Y R}$. + +Case 2: $Y$ lies on the ray $Q R$ beyond $R$ (see the right figure below). + +In this case the whole arc $\alpha$ lies inside triangle $P Q R$, and between $m$ and $P Q$, thus $P X<$ $P Y$ for all $X \in \alpha$. +![](https://cdn.mathpix.com/cropped/2024_04_17_6da854f21230b13cd732g-51.jpg?height=406&width=1120&top_left_y=405&top_left_x=475) + +Applying the Lemma (to $\triangle E A D$ with the point $X$, and to $\triangle E C D$ with the point $X$ ), we obtain that exactly one of two expressions $\frac{X A \cdot D E}{A D \cdot A E}$ and $\frac{X C \cdot D E}{C D \cdot C E}$ is less than 1 , which contradicts $(*)$. + +Comment 1. One may show that $A B \cdot C D=X A \cdot X C+X B \cdot X D$. We know that $D, X, E$ are collinear and $\angle D C E=\angle C X D=180^{\circ}-\angle A X B$. Therefore, + +$$ +A B \cdot C D=X B \cdot \frac{\sin \angle A X B}{\sin \angle B A X} \cdot D E \cdot \frac{\sin \angle C E D}{\sin \angle D C E}=X B \cdot D E +$$ + +Furthermore, $X B \cdot D E=X B \cdot(X D+X E)=X B \cdot X D+X B \cdot X E=X B \cdot X D+X A \cdot X C$. + +Comment 2. For a convex quadrilateral $A B C D$ with $A B \cdot C D=B C \cdot D A$, it is known that $\angle D A C+\angle A B D+\angle B C A+\angle C D B=180^{\circ}$ (among other, it was used as a problem on the Regional round of All-Russian olympiad in 2012), but it seems that there is no essential connection between this fact and the original problem. + +Solution 2. The solution consists of two parts. In Part 1 we show that it suffices to prove that + +$$ +\frac{X B}{X D}=\frac{A B}{C D} +$$ + +and + +$$ +\frac{X A}{X C}=\frac{D A}{B C} +$$ + +In Part 2 we establish these equalities. + +Part 1. Using the sine law and applying (1) we obtain + +$$ +\frac{\sin \angle A X B}{\sin \angle X A B}=\frac{A B}{X B}=\frac{C D}{X D}=\frac{\sin \angle C X D}{\sin \angle X C D} +$$ + +so $\sin \angle A X B=\sin \angle C X D$ by the problem conditions. Similarly, (2) yields $\sin \angle D X A=$ $\sin \angle B X C$. If at least one of the pairs $(\angle A X B, \angle C X D)$ and $(\angle B X C, \angle D X A)$ consists of supplementary angles, then we are done. Otherwise, $\angle A X B=\angle C X D$ and $\angle D X A=\angle B X C$. In this case $X=A C \cap B D$, and the problem conditions yield that $A B C D$ is a parallelogram and hence a rhombus. In this last case the claim also holds. + +Part 2. To prove the desired equality (1), invert $A B C D$ at centre $X$ with unit radius; the images of points are denoted by primes. + +We have + +$$ +\angle A^{\prime} B^{\prime} C^{\prime}=\angle X B^{\prime} A^{\prime}+\angle X B^{\prime} C^{\prime}=\angle X A B+\angle X C B=\angle X C D+\angle X C B=\angle B C D . +$$ + +Similarly, the corresponding angles of quadrilaterals $A B C D$ and $D^{\prime} A^{\prime} B^{\prime} C^{\prime}$ are equal. + +Moreover, we have + +$$ +A^{\prime} B^{\prime} \cdot C^{\prime} D^{\prime}=\frac{A B}{X A \cdot X B} \cdot \frac{C D}{X C \cdot X D}=\frac{B C}{X B \cdot X C} \cdot \frac{D A}{X D \cdot D A}=B^{\prime} C^{\prime} \cdot D^{\prime} A^{\prime} +$$ + +![](https://cdn.mathpix.com/cropped/2024_04_17_6da854f21230b13cd732g-52.jpg?height=586&width=1084&top_left_y=455&top_left_x=495) + +Now we need the following Lemma. + +Lemma. Assume that the corresponding angles of convex quadrilaterals $X Y Z T$ and $X^{\prime} Y^{\prime} Z^{\prime} T^{\prime}$ are equal, and that $X Y \cdot Z T=Y Z \cdot T X$ and $X^{\prime} Y^{\prime} \cdot Z^{\prime} T^{\prime}=Y^{\prime} Z^{\prime} \cdot T^{\prime} X^{\prime}$. Then the two quadrilaterals are similar. + +Proof. Take the quadrilateral $X Y Z_{1} T_{1}$ similar to $X^{\prime} Y^{\prime} Z^{\prime} T^{\prime}$ and sharing the side $X Y$ with $X Y Z T$, such that $Z_{1}$ and $T_{1}$ lie on the rays $Y Z$ and $X T$, respectively, and $Z_{1} T_{1} \| Z T$. We need to prove that $Z_{1}=Z$ and $T_{1}=T$. Assume the contrary. Without loss of generality, $T X>X T_{1}$. Let segments $X Z$ and $Z_{1} T_{1}$ intersect at $U$. We have + +$$ +\frac{T_{1} X}{T_{1} Z_{1}}<\frac{T_{1} X}{T_{1} U}=\frac{T X}{Z T}=\frac{X Y}{Y Z}<\frac{X Y}{Y Z_{1}} +$$ + +thus $T_{1} X \cdot Y Z_{1}\beta$ be nonnegative integers. Then, for every integer $M \geqslant \beta+1$, there exists a nonnegative integer $\gamma$ such that + +$$ +\frac{\alpha+\gamma+1}{\beta+\gamma+1}=1+\frac{1}{M}=\frac{M+1}{M} . +$$ + +Proof. + +$$ +\frac{\alpha+\gamma+1}{\beta+\gamma+1}=1+\frac{1}{M} \Longleftrightarrow \frac{\alpha-\beta}{\beta+\gamma+1}=\frac{1}{M} \Longleftrightarrow \gamma=M(\alpha-\beta)-(\beta+1) \geqslant 0 . +$$ + +Now we can finish the solution. Without loss of generality, there exists an index $u$ such that $\alpha_{i}>\beta_{i}$ for $i=1,2, \ldots, u$, and $\alpha_{i}<\beta_{i}$ for $i=u+1, \ldots, t$. The conditions $n \nmid k$ and $k \nmid n$ mean that $1 \leqslant u \leqslant t-1$. + +Choose an integer $X$ greater than all the $\alpha_{i}$ and $\beta_{i}$. By the lemma, we can define the numbers $\gamma_{i}$ so as to satisfy + +$$ +\begin{array}{ll} +\frac{\alpha_{i}+\gamma_{i}+1}{\beta_{i}+\gamma_{i}+1}=\frac{u X+i}{u X+i-1} & \text { for } i=1,2, \ldots, u, \text { and } \\ +\frac{\beta_{u+i}+\gamma_{u+i}+1}{\alpha_{u+i}+\gamma_{u+i}+1}=\frac{(t-u) X+i}{(t-u) X+i-1} & \text { for } i=1,2, \ldots, t-u . +\end{array} +$$ + +Then we will have + +$$ +\frac{d(s n)}{d(s k)}=\prod_{i=1}^{u} \frac{u X+i}{u X+i-1} \cdot \prod_{i=1}^{t-u} \frac{(t-u) X+i-1}{(t-u) X+i}=\frac{u(X+1)}{u X} \cdot \frac{(t-u) X}{(t-u)(X+1)}=1, +$$ + +as required. + +Comment. The lemma can be used in various ways, in order to provide a suitable value of $s$. In particular, one may apply induction on the number $t$ of prime factors, using identities like + +$$ +\frac{n}{n-1}=\frac{n^{2}}{n^{2}-1} \cdot \frac{n+1}{n} +$$ + +N2. Let $n>1$ be a positive integer. Each cell of an $n \times n$ table contains an integer. Suppose that the following conditions are satisfied: + +(i) Each number in the table is congruent to 1 modulo $n$; + +(ii) The sum of numbers in any row, as well as the sum of numbers in any column, is congruent to $n$ modulo $n^{2}$. + +Let $R_{i}$ be the product of the numbers in the $i^{\text {th }}$ row, and $C_{j}$ be the product of the numbers in the $j^{\text {th }}$ column. Prove that the sums $R_{1}+\cdots+R_{n}$ and $C_{1}+\cdots+C_{n}$ are congruent modulo $n^{4}$. + +(Indonesia) + +Solution 1. Let $A_{i, j}$ be the entry in the $i^{\text {th }}$ row and the $j^{\text {th }}$ column; let $P$ be the product of all $n^{2}$ entries. For convenience, denote $a_{i, j}=A_{i, j}-1$ and $r_{i}=R_{i}-1$. We show that + +$$ +\sum_{i=1}^{n} R_{i} \equiv(n-1)+P \quad\left(\bmod n^{4}\right) +$$ + +Due to symmetry of the problem conditions, the sum of all the $C_{j}$ is also congruent to $(n-1)+P$ modulo $n^{4}$, whence the conclusion. + +By condition $(i)$, the number $n$ divides $a_{i, j}$ for all $i$ and $j$. So, every product of at least two of the $a_{i, j}$ is divisible by $n^{2}$, hence + +$R_{i}=\prod_{j=1}^{n}\left(1+a_{i, j}\right)=1+\sum_{j=1}^{n} a_{i, j}+\sum_{1 \leqslant j_{1}3$, so + +$$ +S_{n-1}=2^{n}+2^{\lceil n / 2\rceil}+2^{\lfloor n / 2\rfloor}-3>2^{n}+2^{\lfloor n / 2\rfloor}=a_{n} . +$$ + +Also notice that $S_{n-1}-a_{n}=2^{[n / 2]}-3a_{n}$. Denote $c=S_{n-1}-b$; then $S_{n-1}-a_{n}2^{t}-3$. + +Proof. The inequality follows from $t \geqslant 3$. In order to prove the equivalence, we apply Claim 1 twice in the following manner. + +First, since $S_{2 t-3}-a_{2 t-2}=2^{t-1}-3<2^{t}-3<2^{2 t-2}+2^{t-1}=a_{2 t-2}$, by Claim 1 we have $2^{t}-3 \sim S_{2 t-3}-\left(2^{t}-3\right)=2^{2 t-2}$. + +Second, since $S_{4 t-7}-a_{4 t-6}=2^{2 t-3}-3<2^{2 t-2}<2^{4 t-6}+2^{2 t-3}=a_{4 t-6}$, by Claim 1 we have $2^{2 t-2} \sim S_{4 t-7}-2^{2 t-2}=2^{4 t-6}-3$. + +Therefore, $2^{t}-3 \sim 2^{2 t-2} \sim 2^{4 t-6}-3$, as required. + +Now it is easy to find the required numbers. Indeed, the number $2^{3}-3=5=a_{0}+a_{1}$ is representable, so Claim 3 provides an infinite sequence of representable numbers + +$$ +2^{3}-3 \sim 2^{6}-3 \sim 2^{18}-3 \sim \cdots \sim 2^{t}-3 \sim 2^{4 t-6}-3 \sim \cdots \text {. } +$$ + +On the other hand, the number $2^{7}-3=125$ is non-representable (since by Claim 1 we have $125 \sim S_{6}-125=24 \sim S_{4}-24=17 \sim S_{3}-17=4$ which is clearly non-representable). So Claim 3 provides an infinite sequence of non-representable numbers + +$$ +2^{7}-3 \sim 2^{22}-3 \sim 2^{82}-3 \sim \cdots \sim 2^{t}-3 \sim 2^{4 t-6}-3 \sim \cdots . +$$ + +Solution 2. We keep the notion of representability and the notation $S_{n}$ from the previous solution. We say that an index $n$ is good if $a_{n}$ writes as a sum of smaller terms from the sequence $a_{0}, a_{1}, \ldots$. Otherwise we say it is bad. We must prove that there are infinitely many good indices, as well as infinitely many bad ones. + +Lemma 1. If $m \geqslant 0$ is an integer, then $4^{m}$ is representable if and only if either of $2 m+1$ and $2 m+2$ is good. + +Proof. The case $m=0$ is obvious, so we may assume that $m \geqslant 1$. Let $n=2 m+1$ or $2 m+2$. Then $n \geqslant 3$. We notice that + +$$ +S_{n-1}4^{s}$. + +Proof. We have $2^{4 k-2}s$. + +Now $4^{2}=a_{2}+a_{3}$ is representable, whereas $4^{6}=4096$ is not. Indeed, note that $4^{6}=2^{12}v_{p}\left(a_{n}\right)$, then $v_{p}\left(a_{n} / a_{n+1}\right)<0$, while $v_{p}\left(\left(a_{n+1}-a_{n}\right) / a_{1}\right) \geqslant 0$, so $(*)$ is not integer again. Thus, $v_{p}\left(a_{1}\right) \leqslant v_{p}\left(a_{n+1}\right) \leqslant v_{p}\left(a_{n}\right)$. + +The above arguments can now be applied successively to indices $n+1, n+2, \ldots$, showing that all the indices greater than $n$ are large, and the sequence $v_{p}\left(a_{n}\right), v_{p}\left(a_{n+1}\right), v_{p}\left(a_{n+2}\right), \ldots$ is nonincreasing — hence eventually constant. + +Case 2: There is no large index. + +We have $v_{p}\left(a_{1}\right)>v_{p}\left(a_{n}\right)$ for all $n \geqslant k$. If we had $v_{p}\left(a_{n+1}\right)0$. + +Suppose that the number $x+y=z+t$ is odd. Then $x$ and $y$ have opposite parity, as well as $z$ and $t$. This means that both $x y$ and $z t$ are even, as well as $x y-z t=x+y$; a contradiction. Thus, $x+y$ is even, so the number $s=\frac{x+y}{2}=\frac{z+t}{2}$ is a positive integer. + +Next, we set $b=\frac{|x-y|}{2}, d=\frac{|z-t|}{2}$. Now the problem conditions yield + +$$ +s^{2}=a^{2}+b^{2}=c^{2}+d^{2} +$$ + +and + +$$ +2 s=a^{2}-c^{2}=d^{2}-b^{2} +$$ + +(the last equality in (2) follows from (1)). We readily get from (2) that $a, d>0$. + +In the sequel we will use only the relations (1) and (2), along with the fact that $a, d, s$ are positive integers, while $b$ and $c$ are nonnegative integers, at most one of which may be zero. Since both relations are symmetric with respect to the simultaneous swappings $a \leftrightarrow d$ and $b \leftrightarrow c$, we assume, without loss of generality, that $b \geqslant c$ (and hence $b>0$ ). Therefore, $d^{2}=2 s+b^{2}>c^{2}$, whence + +$$ +d^{2}>\frac{c^{2}+d^{2}}{2}=\frac{s^{2}}{2} +$$ + +On the other hand, since $d^{2}-b^{2}$ is even by (2), the numbers $b$ and $d$ have the same parity, so $00$ imply $b=c=0$, which is impossible. + +Solution 2. We start with a complete description of all 4-tuples $(x, y, z, t)$ of positive integers satisfying (*). As in the solution above, we notice that the numbers + +$$ +s=\frac{x+y}{2}=\frac{z+t}{2}, \quad p=\frac{x-y}{2}, \quad \text { and } \quad q=\frac{z-t}{2} +$$ + +are integers (we may, and will, assume that $p, q \geqslant 0$ ). We have + +$$ +2 s=x y-z t=(s+p)(s-p)-(s+q)(s-q)=q^{2}-p^{2} +$$ + +so $p$ and $q$ have the same parity, and $q>p$. + +Set now $k=\frac{q-p}{2}, \ell=\frac{q+p}{2}$. Then we have $s=\frac{q^{2}-p^{2}}{2}=2 k \ell$ and hence + +$$ +\begin{array}{rlrl} +x & =s+p=2 k \ell-k+\ell, & y & =s-p=2 k \ell+k-\ell, \\ +z & =s+q=2 k \ell+k+\ell, & t=s-q=2 k \ell-k-\ell . +\end{array} +$$ + +Recall here that $\ell \geqslant k>0$ and, moreover, $(k, \ell) \neq(1,1)$, since otherwise $t=0$. + +Assume now that both $x y$ and $z t$ are squares. Then $x y z t$ is also a square. On the other hand, we have + +$$ +\begin{aligned} +x y z t=(2 k \ell-k+\ell) & (2 k \ell+k-\ell)(2 k \ell+k+\ell)(2 k \ell-k-\ell) \\ +& =\left(4 k^{2} \ell^{2}-(k-\ell)^{2}\right)\left(4 k^{2} \ell^{2}-(k+\ell)^{2}\right)=\left(4 k^{2} \ell^{2}-k^{2}-\ell^{2}\right)^{2}-4 k^{2} \ell^{2} +\end{aligned} +$$ + +Denote $D=4 k^{2} \ell^{2}-k^{2}-\ell^{2}>0$. From (6) we get $D^{2}>x y z t$. On the other hand, + +$$ +\begin{array}{r} +(D-1)^{2}=D^{2}-2\left(4 k^{2} \ell^{2}-k^{2}-\ell^{2}\right)+1=\left(D^{2}-4 k^{2} \ell^{2}\right)-\left(2 k^{2}-1\right)\left(2 \ell^{2}-1\right)+2 \\ +=x y z t-\left(2 k^{2}-1\right)\left(2 \ell^{2}-1\right)+20$ and $\ell \geqslant 2$. The converse is also true: every pair of positive integers $\ell \geqslant k>0$, except for the pair $k=\ell=1$, generates via (5) a 4-tuple of positive integers satisfying $(*)$. + +N6. Let $f:\{1,2,3, \ldots\} \rightarrow\{2,3, \ldots\}$ be a function such that $f(m+n) \mid f(m)+f(n)$ for all pairs $m, n$ of positive integers. Prove that there exists a positive integer $c>1$ which divides all values of $f$. + +(Mexico) + +Solution 1. For every positive integer $m$, define $S_{m}=\{n: m \mid f(n)\}$. + +Lemma. If the set $S_{m}$ is infinite, then $S_{m}=\{d, 2 d, 3 d, \ldots\}=d \cdot \mathbb{Z}_{>0}$ for some positive integer $d$. Proof. Let $d=\min S_{m}$; the definition of $S_{m}$ yields $m \mid f(d)$. + +Whenever $n \in S_{m}$ and $n>d$, we have $m|f(n)| f(n-d)+f(d)$, so $m \mid f(n-d)$ and therefore $n-d \in S_{m}$. Let $r \leqslant d$ be the least positive integer with $n \equiv r(\bmod d)$; repeating the same step, we can see that $n-d, n-2 d, \ldots, r \in S_{m}$. By the minimality of $d$, this shows $r=d$ and therefore $d \mid n$. + +Starting from an arbitrarily large element of $S_{m}$, the process above reaches all multiples of $d$; so they all are elements of $S_{m}$. + +The solution for the problem will be split into two cases. + +Case 1: The function $f$ is bounded. + +Call a prime $p$ frequent if the set $S_{p}$ is infinite, i.e., if $p$ divides $f(n)$ for infinitely many positive integers $n$; otherwise call $p$ sporadic. Since the function $f$ is bounded, there are only a finite number of primes that divide at least one $f(n)$; so altogether there are finitely many numbers $n$ such that $f(n)$ has a sporadic prime divisor. Let $N$ be a positive integer, greater than all those numbers $n$. + +Let $p_{1}, \ldots, p_{k}$ be the frequent primes. By the lemma we have $S_{p_{i}}=d_{i} \cdot \mathbb{Z}_{>0}$ for some $d_{i}$. Consider the number + +$$ +n=N d_{1} d_{2} \cdots d_{k}+1 +$$ + +Due to $n>N$, all prime divisors of $f(n)$ are frequent primes. Let $p_{i}$ be any frequent prime divisor of $f(n)$. Then $n \in S_{p_{i}}$, and therefore $d_{i} \mid n$. But $n \equiv 1\left(\bmod d_{i}\right)$, which means $d_{i}=1$. Hence $S_{p_{i}}=1 \cdot \mathbb{Z}_{>0}=\mathbb{Z}_{>0}$ and therefore $p_{i}$ is a common divisor of all values $f(n)$. + +Case 2: $f$ is unbounded. + +We prove that $f(1)$ divides all $f(n)$. + +Let $a=f(1)$. Since $1 \in S_{a}$, by the lemma it suffices to prove that $S_{a}$ is an infinite set. + +Call a positive integer $p$ a peak if $f(p)>\max (f(1), \ldots, f(p-1))$. Since $f$ is not bounded, there are infinitely many peaks. Let $1=p_{1}n+2$ and $h=f(p)>f(n)+2 a$. By (1) we have $f(p-1)=$ $f(p)-f(1)=h-a$ and $f(n+1)=f(p)-f(p-n-1)=h-f(p-n-1)$. From $h-a=f(p-1) \mid$ $f(n)+f(p-n-1)2018\end{cases}\right. +$$ + +Solution 2. Let $d_{n}=\operatorname{gcd}(f(n), f(1))$. From $d_{n+1} \mid f(1)$ and $d_{n+1}|f(n+1)| f(n)+f(1)$, we can see that $d_{n+1} \mid f(n)$; then $d_{n+1} \mid \operatorname{gcd}(f(n), f(1))=d_{n}$. So the sequence $d_{1}, d_{2}, \ldots$ is nonincreasing in the sense that every element is a divisor of the previous elements. Let $d=\min \left(d_{1}, d_{2}, \ldots\right)=\operatorname{gcd}\left(d_{1} \cdot d_{2}, \ldots\right)=\operatorname{gcd}(f(1), f(2), \ldots)$; we have to prove $d \geqslant 2$. + +For the sake of contradiction, suppose that the statement is wrong, so $d=1$; that means there is some index $n_{0}$ such that $d_{n}=1$ for every $n \geqslant n_{0}$, i.e., $f(n)$ is coprime with $f(1)$. + +Claim 1. If $2^{k} \geqslant n_{0}$ then $f\left(2^{k}\right) \leqslant 2^{k}$. + +Proof. By the condition, $f(2 n) \mid 2 f(n)$; a trivial induction yields $f\left(2^{k}\right) \mid 2^{k} f(1)$. If $2^{k} \geqslant n_{0}$ then $f\left(2^{k}\right)$ is coprime with $f(1)$, so $f\left(2^{k}\right)$ is a divisor of $2^{k}$. + +Claim 2. There is a constant $C$ such that $f(n)0}$ are coprime then $\operatorname{gcd}(f(a), f(b)) \mid f(1)$. In particular, if $a, b \geqslant n_{0}$ are coprime then $f(a)$ and $f(b)$ are coprime. + +Proof. Let $d=\operatorname{gcd}(f(a), f(b))$. We can replicate Euclid's algorithm. Formally, apply induction on $a+b$. If $a=1$ or $b=1$ then we already have $d \mid f(1)$. + +Without loss of generality, suppose $1C$ (that is possible, because there are arbitrarily long gaps between the primes). Then we establish a contradiction + +$$ +p_{N+1} \leqslant \max \left(f(1), f\left(q_{1}\right), \ldots, f\left(q_{N}\right)\right)<\max \left(1+C, q_{1}+C, \ldots, q_{N}+C\right)=p_{N}+C0$ and $c, d$ are coprime. + +We will show that too many denominators $b_{i}$ should be divisible by $d$. To this end, for any $1 \leqslant i \leqslant n$ and any prime divisor $p$ of $d$, say that the index $i$ is $p$-wrong, if $v_{p}\left(b_{i}\right)5 n, +$$ + +a contradiction. + +Claim 3. For every $0 \leqslant k \leqslant n-30$, among the denominators $b_{k+1}, b_{k+2}, \ldots, b_{k+30}$, at least $\varphi(30)=8$ are divisible by $d$. + +Proof. By Claim 1, the 2 -wrong, 3 -wrong and 5 -wrong indices can be covered by three arithmetic progressions with differences 2,3 and 5 . By a simple inclusion-exclusion, $(2-1) \cdot(3-1) \cdot(5-1)=8$ indices are not covered; by Claim 2, we have $d \mid b_{i}$ for every uncovered index $i$. + +Claim 4. $|\Delta|<\frac{20}{n-2}$ and $d>\frac{n-2}{20}$. + +Proof. From the sequence (1), remove all fractions with $b_{n}<\frac{n}{2}$, There remain at least $\frac{n}{2}$ fractions, and they cannot exceed $\frac{5 n}{n / 2}=10$. So we have at least $\frac{n}{2}$ elements of the arithmetic progression (1) in the interval $(0,10]$, hence the difference must be below $\frac{10}{n / 2-1}=\frac{20}{n-2}$. + +The second inequality follows from $\frac{1}{d} \leqslant \frac{|c|}{d}=|\Delta|$. + +Now we have everything to get the final contradiction. By Claim 3, we have $d \mid b_{i}$ for at least $\left\lfloor\frac{n}{30}\right\rfloor \cdot 8$ indices $i$. By Claim 4 , we have $d \geqslant \frac{n-2}{20}$. Therefore, + +$$ +5 n \geqslant \max \left\{b_{i}: d \mid b_{i}\right\} \geqslant\left(\left\lfloor\frac{n}{30}\right\rfloor \cdot 8\right) \cdot d>\left(\frac{n}{30}-1\right) \cdot 8 \cdot \frac{n-2}{20}>5 n . +$$ + +Comment 1. It is possible that all terms in (1) are equal, for example with $a_{i}=2 i-1$ and $b_{i}=4 i-2$ we have $\frac{a_{i}}{b_{i}}=\frac{1}{2}$. + +Comment 2. The bound $5 n$ in the statement is far from sharp; the solution above can be modified to work for $9 n$. For large $n$, the bound $5 n$ can be replaced by $n^{\frac{3}{2}-\varepsilon}$. + +The activities of the Problem Selection Committee were supported by + +DEDEMAN + +LA FANTANA vine la tine + diff --git a/IMO/md/en-IMO2019SL.md b/IMO/md/en-IMO2019SL.md new file mode 100644 index 0000000000000000000000000000000000000000..68527b3363e089855a414234121f399eef8e256e --- /dev/null +++ b/IMO/md/en-IMO2019SL.md @@ -0,0 +1,3254 @@ +![](https://cdn.mathpix.com/cropped/2024_11_18_9ee8c2ac2c2f5002ff14g-001.jpg?height=732&width=690&top_left_y=222&top_left_x=683) + +# 60TH INTERNATIONAL MATHEMATICAL OLYMPIAD + +July 11 1h - July 22 ${ }^{\text {nd }}$, Bath, United Kingdom + +## SHORTLISTED PROBLEMS WITH SOLUTIONS + +![](https://cdn.mathpix.com/cropped/2024_11_18_9ee8c2ac2c2f5002ff14g-003.jpg?height=287&width=307&top_left_y=199&top_left_x=332) + +IMO 2019 + +## Shortlisted Problems (with solutions) + +60 ${ }^{\text {th }}$ International Mathematical Olympiad +Bath — UK, 11th-22nd July 2019 + +## The Shortlist has to be kept strictly confidential until the conclusion of the following International Mathematical Olympiad. + +IMO General Regulations §6.6 + +## Contributing Countries + +The Organising Committee and the Problem Selection Committee of IMO 2019 thank the following 58 countries for contributing 204 problem proposals: + +Albania, Armenia, Australia, Austria, Belarus, Belgium, Brazil, Bulgaria, Canada, China, Croatia, Cuba, Cyprus, Czech Republic, Denmark, Ecuador, El Salvador, Estonia, Finland, France, Georgia, Germany, Greece, Hong Kong, Hungary, India, Indonesia, Iran, Ireland, Israel, Italy, Japan, Kazakhstan, Kosovo, Luxembourg, Mexico, Netherlands, New Zealand, Nicaragua, Nigeria, North Macedonia, Philippines, Poland, Russia, Serbia, Singapore, Slovakia, Slovenia, South Africa, South Korea, Sweden, Switzerland, Taiwan, Tanzania, Thailand, Ukraine, USA, Vietnam. + +Problem Selection Committee +![](https://cdn.mathpix.com/cropped/2024_11_18_9ee8c2ac2c2f5002ff14g-005.jpg?height=761&width=1648&top_left_y=1847&top_left_x=204) + +Tony Gardiner, Edward Crane, Alexander Betts, James Cranch, Joseph Myers (chair), James Aaronson, Andrew Carlotti, Géza Kós, Ilya I. Bogdanov, Jack Shotton + +## Problems + +## Algebra + +A1. Let $\mathbb{Z}$ be the set of integers. Determine all functions $f: \mathbb{Z} \rightarrow \mathbb{Z}$ such that, for all integers $a$ and $b$, + +$$ +f(2 a)+2 f(b)=f(f(a+b)) +$$ + +(South Africa) +A2. Let $u_{1}, u_{2}, \ldots, u_{2019}$ be real numbers satisfying + +$$ +u_{1}+u_{2}+\cdots+u_{2019}=0 \quad \text { and } \quad u_{1}^{2}+u_{2}^{2}+\cdots+u_{2019}^{2}=1 . +$$ + +Let $a=\min \left(u_{1}, u_{2}, \ldots, u_{2019}\right)$ and $b=\max \left(u_{1}, u_{2}, \ldots, u_{2019}\right)$. Prove that + +$$ +a b \leqslant-\frac{1}{2019} +$$ + +(Germany) +A3. Let $n \geqslant 3$ be a positive integer and let $\left(a_{1}, a_{2}, \ldots, a_{n}\right)$ be a strictly increasing sequence of $n$ positive real numbers with sum equal to 2 . Let $X$ be a subset of $\{1,2, \ldots, n\}$ such that the value of + +$$ +\left|1-\sum_{i \in X} a_{i}\right| +$$ + +is minimised. Prove that there exists a strictly increasing sequence of $n$ positive real numbers $\left(b_{1}, b_{2}, \ldots, b_{n}\right)$ with sum equal to 2 such that + +$$ +\sum_{i \in X} b_{i}=1 +$$ + +(New Zealand) +A4. Let $n \geqslant 2$ be a positive integer and $a_{1}, a_{2}, \ldots, a_{n}$ be real numbers such that + +$$ +a_{1}+a_{2}+\cdots+a_{n}=0 +$$ + +Define the set $A$ by + +$$ +A=\left\{(i, j)\left|1 \leqslant i0$, then he flips the $k^{\text {th }}$ coin over; otherwise he stops the process. (For example, the process starting with THT would be THT $\rightarrow H H T \rightarrow H T T \rightarrow T T T$, which takes three steps.) + +Letting $C$ denote the initial configuration (a sequence of $n H$ 's and $T$ 's), write $\ell(C)$ for the number of steps needed before all coins show $T$. Show that this number $\ell(C)$ is finite, and determine its average value over all $2^{n}$ possible initial configurations $C$. + +C4. On a flat plane in Camelot, King Arthur builds a labyrinth $\mathfrak{L}$ consisting of $n$ walls, each of which is an infinite straight line. No two walls are parallel, and no three walls have a common point. Merlin then paints one side of each wall entirely red and the other side entirely blue. + +At the intersection of two walls there are four corners: two diagonally opposite corners where a red side and a blue side meet, one corner where two red sides meet, and one corner where two blue sides meet. At each such intersection, there is a two-way door connecting the two diagonally opposite corners at which sides of different colours meet. + +After Merlin paints the walls, Morgana then places some knights in the labyrinth. The knights can walk through doors, but cannot walk through walls. + +Let $k(\mathfrak{L})$ be the largest number $k$ such that, no matter how Merlin paints the labyrinth $\mathfrak{L}$, Morgana can always place at least $k$ knights such that no two of them can ever meet. For each $n$, what are all possible values for $k(\mathfrak{L})$, where $\mathfrak{L}$ is a labyrinth with $n$ walls? +(Canada) +C5. On a certain social network, there are 2019 users, some pairs of which are friends, where friendship is a symmetric relation. Initially, there are 1010 people with 1009 friends each and 1009 people with 1010 friends each. However, the friendships are rather unstable, so events of the following kind may happen repeatedly, one at a time: + +Let $A, B$, and $C$ be people such that $A$ is friends with both $B$ and $C$, but $B$ and $C$ are not friends; then $B$ and $C$ become friends, but $A$ is no longer friends with them. +Prove that, regardless of the initial friendships, there exists a sequence of such events after which each user is friends with at most one other user. +(Croatia) + +C6. Let $n>1$ be an integer. Suppose we are given $2 n$ points in a plane such that no three of them are collinear. The points are to be labelled $A_{1}, A_{2}, \ldots, A_{2 n}$ in some order. We then consider the $2 n$ angles $\angle A_{1} A_{2} A_{3}, \angle A_{2} A_{3} A_{4}, \ldots, \angle A_{2 n-2} A_{2 n-1} A_{2 n}, \angle A_{2 n-1} A_{2 n} A_{1}$, $\angle A_{2 n} A_{1} A_{2}$. We measure each angle in the way that gives the smallest positive value (i.e. between $0^{\circ}$ and $180^{\circ}$ ). Prove that there exists an ordering of the given points such that the resulting $2 n$ angles can be separated into two groups with the sum of one group of angles equal to the sum of the other group. + +C7. There are 60 empty boxes $B_{1}, \ldots, B_{60}$ in a row on a table and an unlimited supply of pebbles. Given a positive integer $n$, Alice and Bob play the following game. + +In the first round, Alice takes $n$ pebbles and distributes them into the 60 boxes as she wishes. Each subsequent round consists of two steps: +(a) Bob chooses an integer $k$ with $1 \leqslant k \leqslant 59$ and splits the boxes into the two groups $B_{1}, \ldots, B_{k}$ and $B_{k+1}, \ldots, B_{60}$. +(b) Alice picks one of these two groups, adds one pebble to each box in that group, and removes one pebble from each box in the other group. + +Bob wins if, at the end of any round, some box contains no pebbles. Find the smallest $n$ such that Alice can prevent Bob from winning. +(Czech Republic) + +## C8. + +Alice has a map of Wonderland, a country consisting of $n \geqslant 2$ towns. For every pair of towns, there is a narrow road going from one town to the other. One day, all the roads are declared to be "one way" only. Alice has no information on the direction of the roads, but the King of Hearts has offered to help her. She is allowed to ask him a number of questions. For each question in turn, Alice chooses a pair of towns and the King of Hearts tells her the direction of the road connecting those two towns. + +Alice wants to know whether there is at least one town in Wonderland with at most one outgoing road. Prove that she can always find out by asking at most $4 n$ questions. + +Comment. This problem could be posed with an explicit statement about points being awarded for weaker bounds $c n$ for some $c>4$, in the style of IMO 2014 Problem 6. +(Thailand) + +## C9. + +For any two different real numbers $x$ and $y$, we define $D(x, y)$ to be the unique integer $d$ satisfying $2^{d} \leqslant|x-y|<2^{d+1}$. Given a set of reals $\mathcal{F}$, and an element $x \in \mathcal{F}$, we say that the scales of $x$ in $\mathcal{F}$ are the values of $D(x, y)$ for $y \in \mathcal{F}$ with $x \neq y$. + +Let $k$ be a given positive integer. Suppose that each member $x$ of $\mathcal{F}$ has at most $k$ different scales in $\mathcal{F}$ (note that these scales may depend on $x$ ). What is the maximum possible size of $\mathcal{F}$ ? +(Italy) + +## Geometry + +## G1. Let $A B C$ be a triangle. Circle $\Gamma$ passes through $A$, meets segments $A B$ and $A C$ + +again at points $D$ and $E$ respectively, and intersects segment $B C$ at $F$ and $G$ such that $F$ lies between $B$ and $G$. The tangent to circle $B D F$ at $F$ and the tangent to circle $C E G$ at $G$ meet at point $T$. Suppose that points $A$ and $T$ are distinct. Prove that line $A T$ is parallel to $B C$. +(Nigeria) +G2. Let $A B C$ be an acute-angled triangle and let $D, E$, and $F$ be the feet of altitudes from $A, B$, and $C$ to sides $B C, C A$, and $A B$, respectively. Denote by $\omega_{B}$ and $\omega_{C}$ the incircles of triangles $B D F$ and $C D E$, and let these circles be tangent to segments $D F$ and $D E$ at $M$ and $N$, respectively. Let line $M N$ meet circles $\omega_{B}$ and $\omega_{C}$ again at $P \neq M$ and $Q \neq N$, respectively. Prove that $M P=N Q$. +(Vietnam) +G3. In triangle $A B C$, let $A_{1}$ and $B_{1}$ be two points on sides $B C$ and $A C$, and let $P$ and $Q$ be two points on segments $A A_{1}$ and $B B_{1}$, respectively, so that line $P Q$ is parallel to $A B$. On ray $P B_{1}$, beyond $B_{1}$, let $P_{1}$ be a point so that $\angle P P_{1} C=\angle B A C$. Similarly, on ray $Q A_{1}$, beyond $A_{1}$, let $Q_{1}$ be a point so that $\angle C Q_{1} Q=\angle C B A$. Show that points $P, Q, P_{1}$, and $Q_{1}$ are concyclic. +(Ukraine) +G4. Let $P$ be a point inside triangle $A B C$. Let $A P$ meet $B C$ at $A_{1}$, let $B P$ meet $C A$ at $B_{1}$, and let $C P$ meet $A B$ at $C_{1}$. Let $A_{2}$ be the point such that $A_{1}$ is the midpoint of $P A_{2}$, let $B_{2}$ be the point such that $B_{1}$ is the midpoint of $P B_{2}$, and let $C_{2}$ be the point such that $C_{1}$ is the midpoint of $P C_{2}$. Prove that points $A_{2}, B_{2}$, and $C_{2}$ cannot all lie strictly inside the circumcircle of triangle $A B C$. +(Australia) +G5. Let $A B C D E$ be a convex pentagon with $C D=D E$ and $\angle E D C \neq 2 \cdot \angle A D B$. Suppose that a point $P$ is located in the interior of the pentagon such that $A P=A E$ and $B P=B C$. Prove that $P$ lies on the diagonal $C E$ if and only if $\operatorname{area}(B C D)+\operatorname{area}(A D E)=$ $\operatorname{area}(A B D)+\operatorname{area}(A B P)$. +(Hungary) +G6. Let $I$ be the incentre of acute-angled triangle $A B C$. Let the incircle meet $B C, C A$, and $A B$ at $D, E$, and $F$, respectively. Let line $E F$ intersect the circumcircle of the triangle at $P$ and $Q$, such that $F$ lies between $E$ and $P$. Prove that $\angle D P A+\angle A Q D=\angle Q I P$. +(Slovakia) +G7. The incircle $\omega$ of acute-angled scalene triangle $A B C$ has centre $I$ and meets sides $B C$, $C A$, and $A B$ at $D, E$, and $F$, respectively. The line through $D$ perpendicular to $E F$ meets $\omega$ again at $R$. Line $A R$ meets $\omega$ again at $P$. The circumcircles of triangles $P C E$ and $P B F$ meet again at $Q \neq P$. Prove that lines $D I$ and $P Q$ meet on the external bisector of angle $B A C$. +(India) +G8. +Let $\mathcal{L}$ be the set of all lines in the plane and let $f$ be a function that assigns to each line $\ell \in \mathcal{L}$ a point $f(\ell)$ on $\ell$. Suppose that for any point $X$, and for any three lines $\ell_{1}, \ell_{2}, \ell_{3}$ passing through $X$, the points $f\left(\ell_{1}\right), f\left(\ell_{2}\right), f\left(\ell_{3}\right)$ and $X$ lie on a circle. + +Prove that there is a unique point $P$ such that $f(\ell)=P$ for any line $\ell$ passing through $P$. +(Australia) + +## Number Theory + +N1. Find all pairs $(m, n)$ of positive integers satisfying the equation + +$$ +\left(2^{n}-1\right)\left(2^{n}-2\right)\left(2^{n}-4\right) \cdots\left(2^{n}-2^{n-1}\right)=m! +$$ + +(El Salvador) +N2. Find all triples $(a, b, c)$ of positive integers such that $a^{3}+b^{3}+c^{3}=(a b c)^{2}$. +(Nigeria) +N3. We say that a set $S$ of integers is rootiful if, for any positive integer $n$ and any $a_{0}, a_{1}, \ldots, a_{n} \in S$, all integer roots of the polynomial $a_{0}+a_{1} x+\cdots+a_{n} x^{n}$ are also in $S$. Find all rootiful sets of integers that contain all numbers of the form $2^{a}-2^{b}$ for positive integers $a$ and $b$. +(Czech Republic) +N4. Let $\mathbb{Z}_{>0}$ be the set of positive integers. A positive integer constant $C$ is given. Find all functions $f: \mathbb{Z}_{>0} \rightarrow \mathbb{Z}_{>0}$ such that, for all positive integers $a$ and $b$ satisfying $a+b>C$, + +$$ +a+f(b) \mid a^{2}+b f(a) +$$ + +(Croatia) +N5. Let $a$ be a positive integer. We say that a positive integer $b$ is $a$-good if $\binom{a n}{b}-1$ is divisible by $a n+1$ for all positive integers $n$ with $a n \geqslant b$. Suppose $b$ is a positive integer such that $b$ is $a$-good, but $b+2$ is not $a$-good. Prove that $b+1$ is prime. +(Netherlands) +N6. Let $H=\left\{\lfloor i \sqrt{2}\rfloor: i \in \mathbb{Z}_{>0}\right\}=\{1,2,4,5,7, \ldots\}$, and let $n$ be a positive integer. Prove that there exists a constant $C$ such that, if $A \subset\{1,2, \ldots, n\}$ satisfies $|A| \geqslant C \sqrt{n}$, then there exist $a, b \in A$ such that $a-b \in H$. (Here $\mathbb{Z}_{>0}$ is the set of positive integers, and $\lfloor z\rfloor$ denotes the greatest integer less than or equal to $z$.) +(Brazil) +N7. Prove that there is a constant $c>0$ and infinitely many positive integers $n$ with the following property: there are infinitely many positive integers that cannot be expressed as the sum of fewer than $c n \log (n)$ pairwise coprime $n^{\text {th }}$ powers. +(Canada) + +## N8. + +Let $a$ and $b$ be two positive integers. Prove that the integer + +$$ +a^{2}+\left\lceil\frac{4 a^{2}}{b}\right\rceil +$$ + +is not a square. (Here $\lceil z\rceil$ denotes the least integer greater than or equal to $z$.) + +## Solutions + +## Algebra + +A1. Let $\mathbb{Z}$ be the set of integers. Determine all functions $f: \mathbb{Z} \rightarrow \mathbb{Z}$ such that, for all integers $a$ and $b$, + +$$ +f(2 a)+2 f(b)=f(f(a+b)) +$$ + +(South Africa) +Answer: The solutions are $f(n)=0$ and $f(n)=2 n+K$ for any constant $K \in \mathbb{Z}$. +Common remarks. Most solutions to this problem first prove that $f$ must be linear, before determining all linear functions satisfying (1). + +Solution 1. Substituting $a=0, b=n+1$ gives $f(f(n+1))=f(0)+2 f(n+1)$. Substituting $a=1, b=n$ gives $f(f(n+1))=f(2)+2 f(n)$. + +In particular, $f(0)+2 f(n+1)=f(2)+2 f(n)$, and so $f(n+1)-f(n)=\frac{1}{2}(f(2)-f(0))$. Thus $f(n+1)-f(n)$ must be constant. Since $f$ is defined only on $\mathbb{Z}$, this tells us that $f$ must be a linear function; write $f(n)=M n+K$ for arbitrary constants $M$ and $K$, and we need only determine which choices of $M$ and $K$ work. + +Now, (1) becomes + +$$ +2 M a+K+2(M b+K)=M(M(a+b)+K)+K +$$ + +which we may rearrange to form + +$$ +(M-2)(M(a+b)+K)=0 +$$ + +Thus, either $M=2$, or $M(a+b)+K=0$ for all values of $a+b$. In particular, the only possible solutions are $f(n)=0$ and $f(n)=2 n+K$ for any constant $K \in \mathbb{Z}$, and these are easily seen to work. + +Solution 2. Let $K=f(0)$. +First, put $a=0$ in (1); this gives + +$$ +f(f(b))=2 f(b)+K +$$ + +for all $b \in \mathbb{Z}$. +Now put $b=0$ in (1); this gives + +$$ +f(2 a)+2 K=f(f(a))=2 f(a)+K +$$ + +where the second equality follows from (2). Consequently, + +$$ +f(2 a)=2 f(a)-K +$$ + +for all $a \in \mathbb{Z}$. +Substituting (2) and (3) into (1), we obtain + +$$ +\begin{aligned} +f(2 a)+2 f(b) & =f(f(a+b)) \\ +2 f(a)-K+2 f(b) & =2 f(a+b)+K \\ +f(a)+f(b) & =f(a+b)+K +\end{aligned} +$$ + +Thus, if we set $g(n)=f(n)-K$ we see that $g$ satisfies the Cauchy equation $g(a+b)=$ $g(a)+g(b)$. The solution to the Cauchy equation over $\mathbb{Z}$ is well-known; indeed, it may be proven by an easy induction that $g(n)=M n$ for each $n \in \mathbb{Z}$, where $M=g(1)$ is a constant. + +Therefore, $f(n)=M n+K$, and we may proceed as in Solution 1 . +Comment 1. Instead of deriving (3) by substituting $b=0$ into (1), we could instead have observed that the right hand side of (1) is symmetric in $a$ and $b$, and thus + +$$ +f(2 a)+2 f(b)=f(2 b)+2 f(a) +$$ + +Thus, $f(2 a)-2 f(a)=f(2 b)-2 f(b)$ for any $a, b \in \mathbb{Z}$, and in particular $f(2 a)-2 f(a)$ is constant. Setting $a=0$ shows that this constant is equal to $-K$, and so we obtain (3). + +Comment 2. Some solutions initially prove that $f(f(n))$ is linear (sometimes via proving that $f(f(n))-3 K$ satisfies the Cauchy equation). However, one can immediately prove that $f$ is linear by substituting something of the form $f(f(n))=M^{\prime} n+K^{\prime}$ into (2). + +A2. Let $u_{1}, u_{2}, \ldots, u_{2019}$ be real numbers satisfying + +$$ +u_{1}+u_{2}+\cdots+u_{2019}=0 \quad \text { and } \quad u_{1}^{2}+u_{2}^{2}+\cdots+u_{2019}^{2}=1 +$$ + +Let $a=\min \left(u_{1}, u_{2}, \ldots, u_{2019}\right)$ and $b=\max \left(u_{1}, u_{2}, \ldots, u_{2019}\right)$. Prove that + +$$ +a b \leqslant-\frac{1}{2019} +$$ + +(Germany) +Solution 1. Notice first that $b>0$ and $a<0$. Indeed, since $\sum_{i=1}^{2019} u_{i}^{2}=1$, the variables $u_{i}$ cannot be all zero, and, since $\sum_{i=1}^{2019} u_{i}=0$, the nonzero elements cannot be all positive or all negative. + +Let $P=\left\{i: u_{i}>0\right\}$ and $N=\left\{i: u_{i} \leqslant 0\right\}$ be the indices of positive and nonpositive elements in the sequence, and let $p=|P|$ and $n=|N|$ be the sizes of these sets; then $p+n=2019$. By the condition $\sum_{i=1}^{2019} u_{i}=0$ we have $0=\sum_{i=1}^{2019} u_{i}=\sum_{i \in P} u_{i}-\sum_{i \in N}\left|u_{i}\right|$, so + +$$ +\sum_{i \in P} u_{i}=\sum_{i \in N}\left|u_{i}\right| . +$$ + +After this preparation, estimate the sum of squares of the positive and nonpositive elements as follows: + +$$ +\begin{aligned} +& \sum_{i \in P} u_{i}^{2} \leqslant \sum_{i \in P} b u_{i}=b \sum_{i \in P} u_{i}=b \sum_{i \in N}\left|u_{i}\right| \leqslant b \sum_{i \in N}|a|=-n a b ; \\ +& \sum_{i \in N} u_{i}^{2} \leqslant \sum_{i \in N}|a| \cdot\left|u_{i}\right|=|a| \sum_{i \in N}\left|u_{i}\right|=|a| \sum_{i \in P} u_{i} \leqslant|a| \sum_{i \in P} b=-p a b . +\end{aligned} +$$ + +The sum of these estimates is + +$$ +1=\sum_{i=1}^{2019} u_{i}^{2}=\sum_{i \in P} u_{i}^{2}+\sum_{i \in N} u_{i}^{2} \leqslant-(p+n) a b=-2019 a b ; +$$ + +that proves $a b \leqslant \frac{-1}{2019}$. +Comment 1. After observing $\sum_{i \in P} u_{i}^{2} \leqslant b \sum_{i \in P} u_{i}$ and $\sum_{i \in N} u_{i}^{2} \leqslant|a| \sum_{i \in P}\left|u_{i}\right|$, instead of $(2,3)$ an alternative continuation is + +$$ +|a b| \geqslant \frac{\sum_{i \in P} u_{i}^{2}}{\sum_{i \in P} u_{i}} \cdot \frac{\sum_{i \in N} u_{i}^{2}}{\sum_{i \in N}\left|u_{i}\right|}=\frac{\sum_{i \in P} u_{i}^{2}}{\left(\sum_{i \in P} u_{i}\right)^{2}} \sum_{i \in N} u_{i}^{2} \geqslant \frac{1}{p} \sum_{i \in N} u_{i}^{2} +$$ + +(by the AM-QM or the Cauchy-Schwarz inequality) and similarly $|a b| \geqslant \frac{1}{n} \sum_{i \in P} u_{i}^{2}$. +Solution 2. As in the previous solution we conclude that $a<0$ and $b>0$. +For every index $i$, the number $u_{i}$ is a convex combination of $a$ and $b$, so + +$$ +u_{i}=x_{i} a+y_{i} b \quad \text { with some weights } 0 \leqslant x_{i}, y_{i} \leqslant 1, \text { with } x_{i}+y_{i}=1 \text {. } +$$ + +Let $X=\sum_{i=1}^{2019} x_{i}$ and $Y=\sum_{i=1}^{2019} y_{i}$. From $0=\sum_{i=1}^{2019} u_{i}=\sum_{i=1}^{2019}\left(x_{i} a+y_{i} b\right)=-|a| X+b Y$, we get + +$$ +|a| X=b Y +$$ + +From $\sum_{i=1}^{2019}\left(x_{i}+y_{i}\right)=2019$ we have + +$$ +X+Y=2019 +$$ + +The system of linear equations $(4,5)$ has a unique solution: + +$$ +X=\frac{2019 b}{|a|+b}, \quad Y=\frac{2019|a|}{|a|+b} +$$ + +Now apply the following estimate to every $u_{i}^{2}$ in their sum: + +$$ +u_{i}^{2}=x_{i}^{2} a^{2}+2 x_{i} y_{i} a b+y_{i}^{2} b^{2} \leqslant x_{i} a^{2}+y_{i} b^{2} +$$ + +we obtain that + +$$ +1=\sum_{i=1}^{2019} u_{i}^{2} \leqslant \sum_{i=1}^{2019}\left(x_{i} a^{2}+y_{i} b^{2}\right)=X a^{2}+Y b^{2}=\frac{2019 b}{|a|+b}|a|^{2}+\frac{2019|a|}{|a|+b} b^{2}=2019|a| b=-2019 a b . +$$ + +Hence, $a b \leqslant \frac{-1}{2019}$. +Comment 2. The idea behind Solution 2 is the following thought. Suppose we fix $a<0$ and $b>0$, fix $\sum u_{i}=0$ and vary the $u_{i}$ to achieve the maximum value of $\sum u_{i}^{2}$. Considering varying any two of the $u_{i}$ while preserving their sum: the maximum value of $\sum u_{i}^{2}$ is achieved when those two are as far apart as possible, so all but at most one of the $u_{i}$ are equal to $a$ or $b$. Considering a weighted version of the problem, we see the maximum (with fractional numbers of $u_{i}$ having each value) is achieved when $\frac{2019 b}{|a|+b}$ of them are $a$ and $\frac{2019|a|}{|a|+b}$ are $b$. + +In fact, this happens in the solution: the number $u_{i}$ is replaced by $x_{i}$ copies of $a$ and $y_{i}$ copies of $b$. + +A3. Let $n \geqslant 3$ be a positive integer and let $\left(a_{1}, a_{2}, \ldots, a_{n}\right)$ be a strictly increasing sequence of $n$ positive real numbers with sum equal to 2 . Let $X$ be a subset of $\{1,2, \ldots, n\}$ such that the value of + +$$ +\left|1-\sum_{i \in X} a_{i}\right| +$$ + +is minimised. Prove that there exists a strictly increasing sequence of $n$ positive real numbers $\left(b_{1}, b_{2}, \ldots, b_{n}\right)$ with sum equal to 2 such that + +$$ +\sum_{i \in X} b_{i}=1 +$$ + +(New Zealand) +Common remarks. In all solutions, we say an index set $X$ is $\left(a_{i}\right)$-minimising if it has the property in the problem for the given sequence $\left(a_{i}\right)$. Write $X^{c}$ for the complement of $X$, and $[a, b]$ for the interval of integers $k$ such that $a \leqslant k \leqslant b$. Note that + +$$ +\left|1-\sum_{i \in X} a_{i}\right|=\left|1-\sum_{i \in X^{c}} a_{i}\right|, +$$ + +so we may exchange $X$ and $X^{c}$ where convenient. Let + +$$ +\Delta=\sum_{i \in X^{c}} a_{i}-\sum_{i \in X} a_{i} +$$ + +and note that $X$ is $\left(a_{i}\right)$-minimising if and only if it minimises $|\Delta|$, and that $\sum_{i \in X} a_{i}=1$ if and only if $\Delta=0$. + +In some solutions, a scaling process is used. If we have a strictly increasing sequence of positive real numbers $c_{i}$ (typically obtained by perturbing the $a_{i}$ in some way) such that + +$$ +\sum_{i \in X} c_{i}=\sum_{i \in X^{c}} c_{i} +$$ + +then we may put $b_{i}=2 c_{i} / \sum_{j=1}^{n} c_{j}$. So it suffices to construct such a sequence without needing its sum to be 2 . + +The solutions below show various possible approaches to the problem. Solutions 1 and 2 perturb a few of the $a_{i}$ to form the $b_{i}$ (with scaling in the case of Solution 1, without scaling in the case of Solution 2). Solutions 3 and 4 look at properties of the index set $X$. Solution 3 then perturbs many of the $a_{i}$ to form the $b_{i}$, together with scaling. Rather than using such perturbations, Solution 4 constructs a sequence $\left(b_{i}\right)$ directly from the set $X$ with the required properties. Solution 4 can be used to give a complete description of sets $X$ that are $\left(a_{i}\right)$-minimising for some $\left(a_{i}\right)$. + +Solution 1. Without loss of generality, assume $\sum_{i \in X} a_{i} \leqslant 1$, and we may assume strict inequality as otherwise $b_{i}=a_{i}$ works. Also, $X$ clearly cannot be empty. + +If $n \in X$, add $\Delta$ to $a_{n}$, producing a sequence of $c_{i}$ with $\sum_{i \in X} c_{i}=\sum_{i \in X^{c}} c_{i}$, and then scale as described above to make the sum equal to 2 . Otherwise, there is some $k$ with $k \in X$ and $k+1 \in X^{c}$. Let $\delta=a_{k+1}-a_{k}$. + +- If $\delta>\Delta$, add $\Delta$ to $a_{k}$ and then scale. +- If $\delta<\Delta$, then considering $X \cup\{k+1\} \backslash\{k\}$ contradicts $X$ being $\left(a_{i}\right)$-minimising. +- If $\delta=\Delta$, choose any $j \neq k, k+1$ (possible since $n \geqslant 3$ ), and any $\epsilon$ less than the least of $a_{1}$ and all the differences $a_{i+1}-a_{i}$. If $j \in X$ then add $\Delta-\epsilon$ to $a_{k}$ and $\epsilon$ to $a_{j}$, then scale; otherwise, add $\Delta$ to $a_{k}$ and $\epsilon / 2$ to $a_{k+1}$, and subtract $\epsilon / 2$ from $a_{j}$, then scale. + +Solution 2. This is similar to Solution 1, but without scaling. As in that solution, without loss of generality, assume $\sum_{i \in X} a_{i}<1$. + +Suppose there exists $1 \leqslant j \leqslant n-1$ such that $j \in X$ but $j+1 \in X^{c}$. Then $a_{j+1}-a_{j} \geqslant \Delta$, because otherwise considering $X \cup\{j+1\} \backslash\{j\}$ contradicts $X$ being $\left(a_{i}\right)$-minimising. + +If $a_{j+1}-a_{j}>\Delta$, put + +$$ +b_{i}= \begin{cases}a_{j}+\Delta / 2, & \text { if } i=j \\ a_{j+1}-\Delta / 2, & \text { if } i=j+1 \\ a_{i}, & \text { otherwise }\end{cases} +$$ + +If $a_{j+1}-a_{j}=\Delta$, choose any $\epsilon$ less than the least of $\Delta / 2, a_{1}$ and all the differences $a_{i+1}-a_{i}$. If $|X| \geqslant 2$, choose $k \in X$ with $k \neq j$, and put + +$$ +b_{i}= \begin{cases}a_{j}+\Delta / 2-\epsilon, & \text { if } i=j \\ a_{j+1}-\Delta / 2, & \text { if } i=j+1 \\ a_{k}+\epsilon, & \text { if } i=k \\ a_{i}, & \text { otherwise }\end{cases} +$$ + +Otherwise, $\left|X^{c}\right| \geqslant 2$, so choose $k \in X^{c}$ with $k \neq j+1$, and put + +$$ +b_{i}= \begin{cases}a_{j}+\Delta / 2, & \text { if } i=j \\ a_{j+1}-\Delta / 2+\epsilon, & \text { if } i=j+1 \\ a_{k}-\epsilon, & \text { if } i=k \\ a_{i}, & \text { otherwise }\end{cases} +$$ + +If there is no $1 \leqslant j \leqslant n$ such that $j \in X$ but $j+1 \in X^{c}$, there must be some $1\Delta$, as otherwise considering $X \cup\{1\}$ contradicts $X$ being $\left(a_{i}\right)$-minimising. Now put + +$$ +b_{i}= \begin{cases}a_{1}-\Delta / 2, & \text { if } i=1 \\ a_{n}+\Delta / 2, & \text { if } i=n \\ a_{i}, & \text { otherwise }\end{cases} +$$ + +Solution 3. Without loss of generality, assume $\sum_{i \in X} a_{i} \leqslant 1$, so $\Delta \geqslant 0$. If $\Delta=0$ we can take $b_{i}=a_{i}$, so now assume that $\Delta>0$. + +Suppose that there is some $k \leqslant n$ such that $|X \cap[k, n]|>\left|X^{c} \cap[k, n]\right|$. If we choose the largest such $k$ then $|X \cap[k, n]|-\left|X^{c} \cap[k, n]\right|=1$. We can now find the required sequence $\left(b_{i}\right)$ by starting with $c_{i}=a_{i}$ for $i\frac{n-k+1}{2}$ and $|X \cap[\ell, n]|<\frac{n-\ell+1}{2}$. +We now construct our sequence $\left(b_{i}\right)$ using this claim. Let $k$ and $\ell$ be the greatest values satisfying the claim, and without loss of generality suppose $k=n$ and $\ell\sum_{i \in Y} a_{i}>1 +$$ + +contradicting $X$ being $\left(a_{i}\right)$-minimising. Otherwise, we always have equality, meaning that $X=Y$. But now consider $Z=Y \cup\{n-1\} \backslash\{n\}$. Since $n \geqslant 3$, we have + +$$ +\sum_{i \in Y} a_{i}>\sum_{i \in Z} a_{i}>\sum_{i \in Y^{c}} a_{i}=2-\sum_{i \in Y} a_{i} +$$ + +and so $Z$ contradicts $X$ being $\left(a_{i}\right)$-minimising. + +A4. Let $n \geqslant 2$ be a positive integer and $a_{1}, a_{2}, \ldots, a_{n}$ be real numbers such that + +$$ +a_{1}+a_{2}+\cdots+a_{n}=0 +$$ + +Define the set $A$ by + +$$ +A=\left\{(i, j)\left|1 \leqslant i0 +$$ + +Partition the indices into sets $P, Q, R$, and $S$ such that + +$$ +\begin{aligned} +P & =\left\{i \mid a_{i} \leqslant-1\right\} & R & =\left\{i \mid 01$ ). Therefore, + +$$ +t_{+}+t_{-} \leqslant \frac{p^{2}+s^{2}}{2}+p q+r s+p r+p s+q s=\frac{(p+q+r+s)^{2}}{2}-\frac{(q+r)^{2}}{2}=-\frac{(q+r)^{2}}{2} \leqslant 0 +$$ + +If $A$ is not empty and $p=s=0$, then there must exist $i \in Q, j \in R$ with $\left|a_{i}-a_{j}\right|>1$, and hence the earlier equality conditions cannot both occur. + +Comment. The RHS of the original inequality cannot be replaced with any constant $c<0$ (independent of $n$ ). Indeed, take + +$$ +a_{1}=-\frac{n}{n+2}, a_{2}=\cdots=a_{n-1}=\frac{1}{n+2}, a_{n}=\frac{2}{n+2} . +$$ + +Then $\sum_{(i, j) \in A} a_{i} a_{j}=-\frac{2 n}{(n+2)^{2}}$, which converges to zero as $n \rightarrow \infty$. + +This page is intentionally left blank + +A5. Let $x_{1}, x_{2}, \ldots, x_{n}$ be different real numbers. Prove that + +$$ +\sum_{1 \leqslant i \leqslant n} \prod_{j \neq i} \frac{1-x_{i} x_{j}}{x_{i}-x_{j}}= \begin{cases}0, & \text { if } n \text { is even } \\ 1, & \text { if } n \text { is odd }\end{cases} +$$ + +(Kazakhstan) +Common remarks. Let $G\left(x_{1}, x_{2}, \ldots, x_{n}\right)$ be the function of the $n$ variables $x_{1}, x_{2}, \ldots, x_{n}$ on the LHS of the required identity. + +Solution 1 (Lagrange interpolation). Since both sides of the identity are rational functions, it suffices to prove it when all $x_{i} \notin\{ \pm 1\}$. Define + +$$ +f(t)=\prod_{i=1}^{n}\left(1-x_{i} t\right) +$$ + +and note that + +$$ +f\left(x_{i}\right)=\left(1-x_{i}^{2}\right) \prod_{j \neq i} 1-x_{i} x_{j} +$$ + +Using the nodes $+1,-1, x_{1}, \ldots, x_{n}$, the Lagrange interpolation formula gives us the following expression for $f$ : + +$$ +\sum_{i=1}^{n} f\left(x_{i}\right) \frac{(x-1)(x+1)}{\left(x_{i}-1\right)\left(x_{i}+1\right)} \prod_{j \neq i} \frac{x-x_{j}}{x_{i}-x_{j}}+f(1) \frac{x+1}{1+1} \prod_{1 \leqslant i \leqslant n} \frac{x-x_{i}}{1-x_{i}}+f(-1) \frac{x-1}{-1-1} \prod_{1 \leqslant i \leqslant n} \frac{x-x_{i}}{1-x_{i}} +$$ + +The coefficient of $t^{n+1}$ in $f(t)$ is 0 , since $f$ has degree $n$. The coefficient of $t^{n+1}$ in the above expression of $f$ is + +$$ +\begin{aligned} +0 & =\sum_{1 \leqslant i \leqslant n} \frac{f\left(x_{i}\right)}{\prod_{j \neq i}\left(x_{i}-x_{j}\right) \cdot\left(x_{i}-1\right)\left(x_{i}+1\right)}+\frac{f(1)}{\prod_{1 \leqslant j \leqslant n}\left(1-x_{j}\right) \cdot(1+1)}+\frac{f(-1)}{\prod_{1 \leqslant j \leqslant n}\left(-1-x_{j}\right) \cdot(-1-1)} \\ +& =-G\left(x_{1}, \ldots, x_{n}\right)+\frac{1}{2}+\frac{(-1)^{n+1}}{2} +\end{aligned} +$$ + +Comment. The main difficulty is to think of including the two extra nodes $\pm 1$ and evaluating the coefficient $t^{n+1}$ in $f$ when $n+1$ is higher than the degree of $f$. + +It is possible to solve the problem using Lagrange interpolation on the nodes $x_{1}, \ldots, x_{n}$, but the definition of the polynomial being interpolated should depend on the parity of $n$. For $n$ even, consider the polynomial + +$$ +P(x)=\prod_{i}\left(1-x x_{i}\right)-\prod_{i}\left(x-x_{i}\right) +$$ + +Lagrange interpolation shows that $G$ is the coefficient of $x^{n-1}$ in the polynomial $P(x) /\left(1-x^{2}\right)$, i.e. 0 . For $n$ odd, consider the polynomial + +$$ +P(x)=\prod_{i}\left(1-x x_{i}\right)-x \prod_{i}\left(x-x_{i}\right) +$$ + +Now $G$ is the coefficient of $x^{n-1}$ in $P(x) /\left(1-x^{2}\right)$, which is 1 . + +Solution 2 (using symmetries). Observe that $G$ is symmetric in the variables $x_{1}, \ldots, x_{n}$. Define $V=\prod_{i0$. By Step 2, each of its monomials $\mu x^{a} y^{b} z^{c}$ of the highest total degree satisfies $a, b \geqslant c$. Applying other weak symmetries, we obtain $a, c \geqslant b$ and $b, c \geqslant a$; therefore, $P$ has a unique leading monomial of the form $\mu(x y z)^{c}$. The polynomial $P_{0}(x, y, z)=P(x, y, z)-\mu\left(x y z-x^{2}-y^{2}-z^{2}\right)^{c}$ has smaller total degree. Since $P_{0}$ is symmetric, it is representable as a polynomial function of $x y z-x^{2}-y^{2}-z^{2}$. Then $P$ is also of this form, completing the inductive step. + +Comment. We could alternatively carry out Step 1 by an induction on $n=\operatorname{deg}_{z} P$, in a manner similar to Step 3. If $n=0$, the statement holds. Assume that $n>0$ and check the leading component of $P$ with respect to $z$ : + +$$ +P(x, y, z)=Q_{n}(x, y) z^{n}+R(x, y, z) +$$ + +where $\operatorname{deg}_{z} Rv(x)$ and $v(x) \geqslant v(y)$. Hence this $f$ satisfies the functional equation and 0 is an $f$-rare integer. + +Comment 3. In fact, if $v$ is an $f$-rare integer for an $f$ satisfying the functional equation, then its fibre $X_{v}=\{v\}$ must be a singleton. We may assume without loss of generality that $v=0$. We've already seen in Solution 1 that 0 is either the greatest or least element of $X_{0}$; replacing $f$ with the function $x \mapsto-f(-x)$ if necessary, we may assume that 0 is the least element of $X_{0}$. We write $b$ for the largest element of $X_{0}$, supposing for contradiction that $b>0$, and write $N=(2 b)$ !. + +It now follows from (*) that we have + +$$ +f(f(N b)+b)=f(f(0)+b)=f(b)=0 +$$ + +from which we see that $f(N b)+b \in X_{0} \subseteq[0, b]$. It follows that $f(N b) \in[-b, 0)$, since by construction $N b \notin X_{v}$. Now it follows that $(f(N b)-0) \cdot(f(N b)-b)$ is a divisor of $N$, so from ( $\dagger$ ) we see that $f(N b)=f(0)=0$. This yields the desired contradiction. + +## Combinatorics + +C1. The infinite sequence $a_{0}, a_{1}, a_{2}, \ldots$ of (not necessarily different) integers has the following properties: $0 \leqslant a_{i} \leqslant i$ for all integers $i \geqslant 0$, and + +$$ +\binom{k}{a_{0}}+\binom{k}{a_{1}}+\cdots+\binom{k}{a_{k}}=2^{k} +$$ + +for all integers $k \geqslant 0$. +Prove that all integers $N \geqslant 0$ occur in the sequence (that is, for all $N \geqslant 0$, there exists $i \geqslant 0$ with $\left.a_{i}=N\right)$. +(Netherlands) +Solution. We prove by induction on $k$ that every initial segment of the sequence, $a_{0}, a_{1}, \ldots, a_{k}$, consists of the following elements (counted with multiplicity, and not necessarily in order), for some $\ell \geqslant 0$ with $2 \ell \leqslant k+1$ : + +$$ +0,1, \ldots, \ell-1, \quad 0,1, \ldots, k-\ell +$$ + +For $k=0$ we have $a_{0}=0$, which is of this form. Now suppose that for $k=m$ the elements $a_{0}, a_{1}, \ldots, a_{m}$ are $0,0,1,1,2,2, \ldots, \ell-1, \ell-1, \ell, \ell+1, \ldots, m-\ell-1, m-\ell$ for some $\ell$ with $0 \leqslant 2 \ell \leqslant m+1$. It is given that + +$$ +\binom{m+1}{a_{0}}+\binom{m+1}{a_{1}}+\cdots+\binom{m+1}{a_{m}}+\binom{m+1}{a_{m+1}}=2^{m+1} +$$ + +which becomes + +$$ +\begin{aligned} +\left(\binom{m+1}{0}+\binom{m+1}{1}\right. & \left.+\cdots+\binom{m+1}{\ell-1}\right) \\ +& +\left(\binom{m+1}{0}+\binom{m+1}{1}+\cdots+\binom{m+1}{m-\ell}\right)+\binom{m+1}{a_{m+1}}=2^{m+1} +\end{aligned} +$$ + +or, using $\binom{m+1}{i}=\binom{m+1}{m+1-i}$, that + +$$ +\begin{aligned} +\left(\binom{m+1}{0}+\binom{m+1}{1}\right. & \left.+\cdots+\binom{m+1}{\ell-1}\right) \\ +& +\left(\binom{m+1}{m+1}+\binom{m+1}{m}+\cdots+\binom{m+1}{\ell+1}\right)+\binom{m+1}{a_{m+1}}=2^{m+1} +\end{aligned} +$$ + +On the other hand, it is well known that + +$$ +\binom{m+1}{0}+\binom{m+1}{1}+\cdots+\binom{m+1}{m+1}=2^{m+1} +$$ + +and so, by subtracting, we get + +$$ +\binom{m+1}{a_{m+1}}=\binom{m+1}{\ell} +$$ + +From this, using the fact that the binomial coefficients $\binom{m+1}{i}$ are increasing for $i \leqslant \frac{m+1}{2}$ and decreasing for $i \geqslant \frac{m+1}{2}$, we conclude that either $a_{m+1}=\ell$ or $a_{m+1}=m+1-\ell$. In either case, $a_{0}, a_{1}, \ldots, a_{m+1}$ is again of the claimed form, which concludes the induction. + +As a result of this description, any integer $N \geqslant 0$ appears as a term of the sequence $a_{i}$ for some $0 \leqslant i \leqslant 2 N$. + +C2. You are given a set of $n$ blocks, each weighing at least 1 ; their total weight is $2 n$. Prove that for every real number $r$ with $0 \leqslant r \leqslant 2 n-2$ you can choose a subset of the blocks whose total weight is at least $r$ but at most $r+2$. +(Thailand) +Solution 1. We prove the following more general statement by induction on $n$. +Claim. Suppose that you have $n$ blocks, each of weight at least 1 , and of total weight $s \leqslant 2 n$. Then for every $r$ with $-2 \leqslant r \leqslant s$, you can choose some of the blocks whose total weight is at least $r$ but at most $r+2$. +Proof. The base case $n=1$ is trivial. To prove the inductive step, let $x$ be the largest block weight. Clearly, $x \geqslant s / n$, so $s-x \leqslant \frac{n-1}{n} s \leqslant 2(n-1)$. Hence, if we exclude a block of weight $x$, we can apply the inductive hypothesis to show the claim holds (for this smaller set) for any $-2 \leqslant r \leqslant s-x$. Adding the excluded block to each of those combinations, we see that the claim also holds when $x-2 \leqslant r \leqslant s$. So if $x-2 \leqslant s-x$, then we have covered the whole interval $[-2, s]$. But each block weight is at least 1 , so we have $x-2 \leqslant(s-(n-1))-2=s-(2 n-(n-1)) \leqslant s-(s-(n-1)) \leqslant s-x$, as desired. + +Comment. Instead of inducting on sets of blocks with total weight $s \leqslant 2 n$, we could instead prove the result only for $s=2 n$. We would then need to modify the inductive step to scale up the block weights before applying the induction hypothesis. + +Solution 2. Let $x_{1}, \ldots, x_{n}$ be the weights of the blocks in weakly increasing order. Consider the set $S$ of sums of the form $\sum_{j \in J} x_{j}$ for a subset $J \subseteq\{1,2, \ldots, n\}$. We want to prove that the mesh of $S$ - i.e. the largest distance between two adjacent elements - is at most 2. + +For $0 \leqslant k \leqslant n$, let $S_{k}$ denote the set of sums of the form $\sum_{i \in J} x_{i}$ for a subset $J \subseteq\{1,2, \ldots, k\}$. We will show by induction on $k$ that the mesh of $S_{k}$ is at most 2 . + +The base case $k=0$ is trivial (as $S_{0}=\{0\}$ ). For $k>0$ we have + +$$ +S_{k}=S_{k-1} \cup\left(x_{k}+S_{k-1}\right) +$$ + +(where $\left(x_{k}+S_{k-1}\right)$ denotes $\left\{x_{k}+s: s \in S_{k-1}\right\}$ ), so it suffices to prove that $x_{k} \leqslant \sum_{j\sum_{j(n+1-k)(k+1)+k-1 +$$ + +This rearranges to $n>k(n+1-k)$, which is false for $1 \leqslant k \leqslant n$, giving the desired contradiction. + +## C3. Let $n$ be a positive integer. Harry has $n$ coins lined up on his desk, each showing + +heads or tails. He repeatedly does the following operation: if there are $k$ coins showing heads and $k>0$, then he flips the $k^{\text {th }}$ coin over; otherwise he stops the process. (For example, the process starting with THT would be THT $\rightarrow H H T \rightarrow H T T \rightarrow T T T$, which takes three steps.) + +Letting $C$ denote the initial configuration (a sequence of $n H$ 's and $T$ 's), write $\ell(C)$ for the number of steps needed before all coins show $T$. Show that this number $\ell(C)$ is finite, and determine its average value over all $2^{n}$ possible initial configurations $C$. + +Answer: The average is $\frac{1}{4} n(n+1)$. +Common remarks. Throughout all these solutions, we let $E(n)$ denote the desired average value. + +Solution 1. We represent the problem using a directed graph $G_{n}$ whose vertices are the length- $n$ strings of $H$ 's and $T$ 's. The graph features an edge from each string to its successor (except for $T T \cdots T T$, which has no successor). We will also write $\bar{H}=T$ and $\bar{T}=H$. + +The graph $G_{0}$ consists of a single vertex: the empty string. The main claim is that $G_{n}$ can be described explicitly in terms of $G_{n-1}$ : + +- We take two copies, $X$ and $Y$, of $G_{n-1}$. +- In $X$, we take each string of $n-1$ coins and just append a $T$ to it. In symbols, we replace $s_{1} \cdots s_{n-1}$ with $s_{1} \cdots s_{n-1} T$. +- In $Y$, we take each string of $n-1$ coins, flip every coin, reverse the order, and append an $H$ to it. In symbols, we replace $s_{1} \cdots s_{n-1}$ with $\bar{s}_{n-1} \bar{s}_{n-2} \cdots \bar{s}_{1} H$. +- Finally, we add one new edge from $Y$ to $X$, namely $H H \cdots H H H \rightarrow H H \cdots H H T$. + +We depict $G_{4}$ below, in a way which indicates this recursive construction: +![](https://cdn.mathpix.com/cropped/2024_11_18_9ee8c2ac2c2f5002ff14g-034.jpg?height=515&width=1135&top_left_y=1850&top_left_x=466) + +We prove the claim inductively. Firstly, $X$ is correct as a subgraph of $G_{n}$, as the operation on coins is unchanged by an extra $T$ at the end: if $s_{1} \cdots s_{n-1}$ is sent to $t_{1} \cdots t_{n-1}$, then $s_{1} \cdots s_{n-1} T$ is sent to $t_{1} \cdots t_{n-1} T$. + +Next, $Y$ is also correct as a subgraph of $G_{n}$, as if $s_{1} \cdots s_{n-1}$ has $k$ occurrences of $H$, then $\bar{s}_{n-1} \cdots \bar{s}_{1} H$ has $(n-1-k)+1=n-k$ occurrences of $H$, and thus (provided that $k>0$ ), if $s_{1} \cdots s_{n-1}$ is sent to $t_{1} \cdots t_{n-1}$, then $\bar{s}_{n-1} \cdots \bar{s}_{1} H$ is sent to $\bar{t}_{n-1} \cdots \bar{t}_{1} H$. + +Finally, the one edge from $Y$ to $X$ is correct, as the operation does send $H H \cdots H H H$ to HH $\cdots$ HHT. + +To finish, note that the sequences in $X$ take an average of $E(n-1)$ steps to terminate, whereas the sequences in $Y$ take an average of $E(n-1)$ steps to reach $H H \cdots H$ and then an additional $n$ steps to terminate. Therefore, we have + +$$ +E(n)=\frac{1}{2}(E(n-1)+(E(n-1)+n))=E(n-1)+\frac{n}{2} +$$ + +We have $E(0)=0$ from our description of $G_{0}$. Thus, by induction, we have $E(n)=\frac{1}{2}(1+\cdots+$ $n)=\frac{1}{4} n(n+1)$, which in particular is finite. + +Solution 2. We consider what happens with configurations depending on the coins they start and end with. + +- If a configuration starts with $H$, the last $n-1$ coins follow the given rules, as if they were all the coins, until they are all $T$, then the first coin is turned over. +- If a configuration ends with $T$, the last coin will never be turned over, and the first $n-1$ coins follow the given rules, as if they were all the coins. +- If a configuration starts with $T$ and ends with $H$, the middle $n-2$ coins follow the given rules, as if they were all the coins, until they are all $T$. After that, there are $2 n-1$ more steps: first coins $1,2, \ldots, n-1$ are turned over in that order, then coins $n, n-1, \ldots, 1$ are turned over in that order. + +As this covers all configurations, and the number of steps is clearly finite for 0 or 1 coins, it follows by induction on $n$ that the number of steps is always finite. + +We define $E_{A B}(n)$, where $A$ and $B$ are each one of $H, T$ or $*$, to be the average number of steps over configurations of length $n$ restricted to those that start with $A$, if $A$ is not $*$, and that end with $B$, if $B$ is not * (so * represents "either $H$ or $T$ "). The above observations tell us that, for $n \geqslant 2$ : + +- $E_{H *}(n)=E(n-1)+1$. +- $E_{* T}(n)=E(n-1)$. +- $E_{H T}(n)=E(n-2)+1$ (by using both the observations for $H *$ and for $\left.* T\right)$. +- $E_{T H}(n)=E(n-2)+2 n-1$. + +Now $E_{H *}(n)=\frac{1}{2}\left(E_{H H}(n)+E_{H T}(n)\right)$, so $E_{H H}(n)=2 E(n-1)-E(n-2)+1$. Similarly, $E_{T T}(n)=2 E(n-1)-E(n-2)-1$. So + +$$ +E(n)=\frac{1}{4}\left(E_{H T}(n)+E_{H H}(n)+E_{T T}(n)+E_{T H}(n)\right)=E(n-1)+\frac{n}{2} +$$ + +We have $E(0)=0$ and $E(1)=\frac{1}{2}$, so by induction on $n$ we have $E(n)=\frac{1}{4} n(n+1)$. +Solution 3. Let $H_{i}$ be the number of heads in positions 1 to $i$ inclusive (so $H_{n}$ is the total number of heads), and let $I_{i}$ be 1 if the $i^{\text {th }}$ coin is a head, 0 otherwise. Consider the function + +$$ +t(i)=I_{i}+2\left(\min \left\{i, H_{n}\right\}-H_{i}\right) +$$ + +We claim that $t(i)$ is the total number of times coin $i$ is turned over (which implies that the process terminates). Certainly $t(i)=0$ when all coins are tails, and $t(i)$ is always a nonnegative integer, so it suffices to show that when the $k^{\text {th }}$ coin is turned over (where $k=H_{n}$ ), $t(k)$ goes down by 1 and all the other $t(i)$ are unchanged. We show this by splitting into cases: + +- If $ik, \min \left\{i, H_{n}\right\}=H_{n}$ both before and after the coin flip, and both $H_{n}$ and $H_{i}$ change by the same amount, so $t(i)$ is unchanged. +- If $i=k$ and the coin is heads, $I_{i}$ goes down by 1 , as do both $\min \left\{i, H_{n}\right\}=H_{n}$ and $H_{i}$; so $t(i)$ goes down by 1 . +- If $i=k$ and the coin is tails, $I_{i}$ goes up by $1, \min \left\{i, H_{n}\right\}=i$ is unchanged and $H_{i}$ goes up by 1 ; so $t(i)$ goes down by 1 . + +We now need to compute the average value of + +$$ +\sum_{i=1}^{n} t(i)=\sum_{i=1}^{n} I_{i}+2 \sum_{i=1}^{n} \min \left\{i, H_{n}\right\}-2 \sum_{i=1}^{n} H_{i} . +$$ + +The average value of the first term is $\frac{1}{2} n$, and that of the third term is $-\frac{1}{2} n(n+1)$. To compute the second term, we sum over choices for the total number of heads, and then over the possible values of $i$, getting + +$$ +2^{1-n} \sum_{j=0}^{n}\binom{n}{j} \sum_{i=1}^{n} \min \{i, j\}=2^{1-n} \sum_{j=0}^{n}\binom{n}{j}\left(n j-\binom{j}{2}\right) . +$$ + +Now, in terms of trinomial coefficients, + +$$ +\sum_{j=0}^{n} j\binom{n}{j}=\sum_{j=1}^{n}\binom{n}{n-j, j-1,1}=n \sum_{j=0}^{n-1}\binom{n-1}{j}=2^{n-1} n +$$ + +and + +$$ +\sum_{j=0}^{n}\binom{j}{2}\binom{n}{j}=\sum_{j=2}^{n}\binom{n}{n-j, j-2,2}=\binom{n}{2} \sum_{j=0}^{n-2}\binom{n-2}{j}=2^{n-2}\binom{n}{2} +$$ + +So the second term above is + +$$ +2^{1-n}\left(2^{n-1} n^{2}-2^{n-2}\binom{n}{2}\right)=n^{2}-\frac{n(n-1)}{4} +$$ + +and the required average is + +$$ +E(n)=\frac{1}{2} n+n^{2}-\frac{n(n-1)}{4}-\frac{1}{2} n(n+1)=\frac{n(n+1)}{4} . +$$ + +Solution 4. Harry has built a Turing machine to flip the coins for him. The machine is initially positioned at the $k^{\text {th }}$ coin, where there are $k$ heads (and the position before the first coin is considered to be the $0^{\text {th }}$ coin). The machine then moves according to the following rules, stopping when it reaches the position before the first coin: if the coin at its current position is $H$, it flips the coin and moves to the previous coin, while if the coin at its current position is $T$, it flips the coin and moves to the next position. + +Consider the maximal sequences of consecutive moves in the same direction. Suppose the machine has $a$ consecutive moves to the next coin, before a move to the previous coin. After those $a$ moves, the $a$ coins flipped in those moves are all heads, as is the coin the machine is now at, so at least the next $a+1$ moves will all be moves to the previous coin. Similarly, $a$ consecutive moves to the previous coin are followed by at least $a+1$ consecutive moves to +the next coin. There cannot be more than $n$ consecutive moves in the same direction, so this proves that the process terminates (with a move from the first coin to the position before the first coin). + +Thus we have a (possibly empty) sequence $a_{1}<\cdotsj$, meaning that, as we follow the moves backwards, each coin is always in the correct state when flipped to result in a move in the required direction. (Alternatively, since there are $2^{n}$ possible configurations of coins and $2^{n}$ possible such ascending sequences, the fact that the sequence of moves determines at most one configuration of coins, and thus that there is an injection from configurations of coins to such ascending sequences, is sufficient for it to be a bijection, without needing to show that coins are in the right state as we move backwards.) + +Solution 5. We explicitly describe what happens with an arbitrary sequence $C$ of $n$ coins. Suppose that $C$ contain $k$ heads at positions $1 \leqslant c_{1}1$ be an integer. Suppose we are given $2 n$ points in a plane such that no three of them are collinear. The points are to be labelled $A_{1}, A_{2}, \ldots, A_{2 n}$ in some order. We then consider the $2 n$ angles $\angle A_{1} A_{2} A_{3}, \angle A_{2} A_{3} A_{4}, \ldots, \angle A_{2 n-2} A_{2 n-1} A_{2 n}, \angle A_{2 n-1} A_{2 n} A_{1}$, $\angle A_{2 n} A_{1} A_{2}$. We measure each angle in the way that gives the smallest positive value (i.e. between $0^{\circ}$ and $180^{\circ}$ ). Prove that there exists an ordering of the given points such that the resulting $2 n$ angles can be separated into two groups with the sum of one group of angles equal to the sum of the other group. + +Comment. The first three solutions all use the same construction involving a line separating the points into groups of $n$ points each, but give different proofs that this construction works. Although Solution 1 is very short, the Problem Selection Committee does not believe any of the solutions is easy to find and thus rates this as a problem of medium difficulty. + +Solution 1. Let $\ell$ be a line separating the points into two groups ( $L$ and $R$ ) with $n$ points in each. Label the points $A_{1}, A_{2}, \ldots, A_{2 n}$ so that $L=\left\{A_{1}, A_{3}, \ldots, A_{2 n-1}\right\}$. We claim that this labelling works. + +Take a line $s=A_{2 n} A_{1}$. +(a) Rotate $s$ around $A_{1}$ until it passes through $A_{2}$; the rotation is performed in a direction such that $s$ is never parallel to $\ell$. +(b) Then rotate the new $s$ around $A_{2}$ until it passes through $A_{3}$ in a similar manner. +(c) Perform $2 n-2$ more such steps, after which $s$ returns to its initial position. + +The total (directed) rotation angle $\Theta$ of $s$ is clearly a multiple of $180^{\circ}$. On the other hand, $s$ was never parallel to $\ell$, which is possible only if $\Theta=0$. Now it remains to partition all the $2 n$ angles into those where $s$ is rotated anticlockwise, and the others. + +Solution 2. When tracing a cyclic path through the $A_{i}$ in order, with straight line segments between consecutive points, let $\theta_{i}$ be the exterior angle at $A_{i}$, with a sign convention that it is positive if the path turns left and negative if the path turns right. Then $\sum_{i=1}^{2 n} \theta_{i}=360 k^{\circ}$ for some integer $k$. Let $\phi_{i}=\angle A_{i-1} A_{i} A_{i+1}($ indices $\bmod 2 n)$, defined as in the problem; thus $\phi_{i}=180^{\circ}-\left|\theta_{i}\right|$. + +Let $L$ be the set of $i$ for which the path turns left at $A_{i}$ and let $R$ be the set for which it turns right. Then $S=\sum_{i \in L} \phi_{i}-\sum_{i \in R} \phi_{i}=(180(|L|-|R|)-360 k)^{\circ}$, which is a multiple of $360^{\circ}$ since the number of points is even. We will show that the points can be labelled such that $S=0$, in which case $L$ and $R$ satisfy the required condition of the problem. + +Note that the value of $S$ is defined for a slightly larger class of configurations: it is OK for two points to coincide, as long as they are not consecutive, and OK for three points to be collinear, as long as $A_{i}, A_{i+1}$ and $A_{i+2}$ do not appear on a line in that order. In what follows it will be convenient, although not strictly necessary, to consider such configurations. + +Consider how $S$ changes if a single one of the $A_{i}$ is moved along some straight-line path (not passing through any $A_{j}$ and not lying on any line $A_{j} A_{k}$, but possibly crossing such lines). Because $S$ is a multiple of $360^{\circ}$, and the angles change continuously, $S$ can only change when a point moves between $R$ and $L$. Furthermore, if $\phi_{j}=0$ when $A_{j}$ moves between $R$ and $L, S$ is unchanged; it only changes if $\phi_{j}=180^{\circ}$ when $A_{j}$ moves between those sets. + +For any starting choice of points, we will now construct a new configuration, with labels such that $S=0$, that can be perturbed into the original one without any $\phi_{i}$ passing through $180^{\circ}$, so that $S=0$ for the original configuration with those labels as well. + +Take some line such that there are $n$ points on each side of that line. The new configuration has $n$ copies of a single point on each side of the line, and a path that alternates between +sides of the line; all angles are 0 , so this configuration has $S=0$. Perturbing the points into their original positions, while keeping each point on its side of the line, no angle $\phi_{i}$ can pass through $180^{\circ}$, because no straight line can go from one side of the line to the other and back. So the perturbation process leaves $S=0$. + +Comment. More complicated variants of this solution are also possible; for example, a path defined using four quadrants of the plane rather than just two half-planes. + +Solution 3. First, let $\ell$ be a line in the plane such that there are $n$ points on one side and the other $n$ points on the other side. For convenience, assume $\ell$ is horizontal (otherwise, we can rotate the plane). Then we can use the terms "above", "below", "left" and "right" in the usual way. We denote the $n$ points above the line in an arbitrary order as $P_{1}, P_{2}, \ldots, P_{n}$, and the $n$ points below the line as $Q_{1}, Q_{2}, \ldots, Q_{n}$. + +If we connect $P_{i}$ and $Q_{j}$ with a line segment, the line segment will intersect with the line $\ell$. Denote the intersection as $I_{i j}$. If $P_{i}$ is connected to $Q_{j}$ and $Q_{k}$, where $jk$, then the sign of $\angle Q_{j} P_{i} Q_{k}$ is taken to be the same as for $\angle Q_{k} P_{i} Q_{j}$. + +Similarly, we can define the sign of $\angle P_{j} Q_{i} P_{k}$ with $j1$ boxes, the answer is $n=\left\lfloor\frac{N}{2}+1\right\rfloor\left\lceil\frac{N}{2}+1\right\rceil-1$. +Common remarks. We present solutions for the general case of $N>1$ boxes, and write $M=\left\lfloor\frac{N}{2}+1\right\rfloor\left\lceil\frac{N}{2}+1\right\rceil-1$ for the claimed answer. For $1 \leqslant kk$. We may then assume, without loss of generality, that Alice again picks the left group. +Proof. Suppose Alice picks the right group in the second round. Then the combined effect of the two rounds is that each of the boxes $B_{k+1}, \ldots, B_{\ell}$ lost two pebbles (and the other boxes are unchanged). Hence this configuration is strictly dominated by that before the first round, and it suffices to consider only Alice's other response. + +Solution 1 (Alice). Alice initially distributes pebbles according to $V_{\left\lceil\frac{N}{2}\right\rceil}$. Suppose the current configuration of pebbles dominates $V_{i}$. If Bob makes a $k$-move with $k \geqslant i$ then Alice picks the left group, which results in a configuration that dominates $V_{i+1}$. Likewise, if Bob makes a $k$-move with $k4$, in the style of IMO 2014 Problem 6. +(Thailand) +Solution. We will show Alice needs to ask at most $4 n-7$ questions. Her strategy has the following phases. In what follows, $S$ is the set of towns that Alice, so far, does not know to have more than one outgoing road (so initially $|S|=n$ ). + +Phase 1. Alice chooses any two towns, say $A$ and $B$. Without loss of generality, suppose that the King of Hearts' answer is that the road goes from $A$ to $B$. + +At the end of this phase, Alice has asked 1 question. +Phase 2. During this phase there is a single (variable) town $T$ that is known to have at least one incoming road but not yet known to have any outgoing roads. Initially, $T$ is $B$. Alice does the following $n-2$ times: she picks a town $X$ she has not asked about before, and asks the direction of the road between $T$ and $X$. If it is from $X$ to $T, T$ is unchanged; if it is from $T$ to $X, X$ becomes the new choice of town $T$, as the previous $T$ is now known to have an outgoing road. + +At the end of this phase, Alice has asked a total of $n-1$ questions. The final town $T$ is not yet known to have any outgoing roads, while every other town has exactly one outgoing road known. The undirected graph of roads whose directions are known is a tree. + +Phase 3. During this phase, Alice asks about the directions of all roads between $T$ and another town she has not previously asked about, stopping if she finds two outgoing roads from $T$. This phase involves at most $n-2$ questions. If she does not find two outgoing roads from $T$, she has answered her original question with at most $2 n-3 \leqslant 4 n-7$ questions, so in what follows we suppose that she does find two outgoing roads, asking a total of $k$ questions in this phase, where $2 \leqslant k \leqslant n-2$ (and thus $n \geqslant 4$ for what follows). + +For every question where the road goes towards $T$, the town at the other end is removed from $S$ (as it already had one outgoing road known), while the last question resulted in $T$ being removed from $S$. So at the end of this phase, $|S|=n-k+1$, while a total of $n+k-1$ questions have been asked. Furthermore, the undirected graph of roads within $S$ whose directions are known contains no cycles (as $T$ is no longer a member of $S$, all questions asked in this phase involved $T$ and the graph was a tree before this phase started). Every town in $S$ has exactly one outgoing road known (not necessarily to another town in $S$ ). + +Phase 4. During this phase, Alice repeatedly picks any pair of towns in $S$ for which she does not know the direction of the road between them. Because every town in $S$ has exactly one outgoing road known, this always results in the removal of one of those two towns from $S$. Because there are no cycles in the graph of roads of known direction within $S$, this can continue until there are at most 2 towns left in $S$. + +If it ends with $t$ towns left, $n-k+1-t$ questions were asked in this phase, so a total of $2 n-t$ questions have been asked. + +Phase 5. During this phase, Alice asks about all the roads from the remaining towns in $S$ that she has not previously asked about. She has definitely already asked about any road between those towns (if $t=2$ ). She must also have asked in one of the first two phases about +at least one other road involving one of those towns (as those phases resulted in a tree with $n>2$ vertices). So she asks at most $t(n-t)-1$ questions in this phase. + +At the end of this phase, Alice knows whether any town has at most one outgoing road. If $t=1$, at most $3 n-3 \leqslant 4 n-7$ questions were needed in total, while if $t=2$, at most $4 n-7$ questions were needed in total. + +Comment 1. The version of this problem originally submitted asked only for an upper bound of $5 n$, which is much simpler to prove. The Problem Selection Committee preferred a version with an asymptotically optimal constant. In the following comment, we will show that the constant is optimal. + +Comment 2. We will show that Alice cannot always find out by asking at most $4 n-3\left(\log _{2} n\right)-$ 15 questions, if $n \geqslant 8$. + +To show this, we suppose the King of Hearts is choosing the directions as he goes along, only picking the direction of a road when Alice asks about it for the first time. We provide a strategy for the King of Hearts that ensures that, after the given number of questions, the map is still consistent both with the existence of a town with at most one outgoing road, and with the nonexistence of such a town. His strategy has the following phases. When describing how the King of Hearts' answer to a question is determined below, we always assume he is being asked about a road for the first time (otherwise, he just repeats his previous answer for that road). This strategy is described throughout in graph-theoretic terms (vertices and edges rather than towns and roads). + +Phase 1. In this phase, we consider the undirected graph formed by edges whose directions are known. The phase terminates when there are exactly 8 connected components whose undirected graphs are trees. The following invariant is maintained: in a component with $k$ vertices whose undirected graph is a tree, every vertex has at most $\left[\log _{2} k\right\rfloor$ edges into it. + +- If the King of Hearts is asked about an edge between two vertices in the same component, or about an edge between two components at least one of which is not a tree, he chooses any direction for that edge arbitrarily. +- If he is asked about an edge between a vertex in component $A$ that has $a$ vertices and is a tree and a vertex in component $B$ that has $b$ vertices and is a tree, suppose without loss of generality that $a \geqslant b$. He then chooses the edge to go from $A$ to $B$. In this case, the new number of edges into any vertex is at most $\max \left\{\left\lfloor\log _{2} a\right\rfloor,\left\lfloor\log _{2} b\right\rfloor+1\right\} \leqslant\left\lfloor\log _{2}(a+b)\right\rfloor$. + +In all cases, the invariant is preserved, and the number of tree components either remains unchanged or goes down by 1. Assuming Alice does not repeat questions, the process must eventually terminate with 8 tree components, and at least $n-8$ questions having been asked. + +Note that each tree component contains at least one vertex with no outgoing edges. Colour one such vertex in each tree component red. + +Phase 2. Let $V_{1}, V_{2}$ and $V_{3}$ be the three of the red vertices whose components are smallest (so their components together have at most $\left\lfloor\frac{3}{8} n\right\rfloor$ vertices, with each component having at most $\left\lfloor\frac{3}{8} n-2\right\rfloor$ vertices). Let sets $C_{1}, C_{2}, \ldots$ be the connected components after removing the $V_{j}$. By construction, there are no edges with known direction between $C_{i}$ and $C_{j}$ for $i \neq j$, and there are at least five such components. + +If at any point during this phase, the King of Hearts is asked about an edge within one of the $C_{i}$, he chooses an arbitrary direction. If he is asked about an edge between $C_{i}$ and $C_{j}$ for $i \neq j$, he answers so that all edges go from $C_{i}$ to $C_{i+1}$ and $C_{i+2}$, with indices taken modulo the number of components, and chooses arbitrarily for other pairs. This ensures that all vertices other than the $V_{j}$ will have more than one outgoing edge. + +For edges involving one of the $V_{j}$ he answers as follows, so as to remain consistent for as long as possible with both possibilities for whether one of those vertices has at most one outgoing edge. Note that as they were red vertices, they have no outgoing edges at the start of this phase. For edges between two of the $V_{j}$, he answers that the edges go from $V_{1}$ to $V_{2}$, from $V_{2}$ to $V_{3}$ and from $V_{3}$ to $V_{1}$. For edges between $V_{j}$ and some other vertex, he always answers that the edge goes into $V_{j}$, except for the last such edge for which he is asked the question for any given $V_{j}$, for which he answers that the +edge goes out of $V_{j}$. Thus, as long as at least one of the $V_{j}$ has not had the question answered for all the vertices that are not among the $V_{j}$, his answers are still compatible both with all vertices having more than one outgoing edge, and with that $V_{j}$ having only one outgoing edge. + +At the start of this phase, each of the $V_{j}$ has at most $\left\lfloor\log _{2}\left\lfloor\frac{3}{8} n-2\right\rfloor\right\rfloor<\left(\log _{2} n\right)-1$ incoming edges. Thus, Alice cannot determine whether some vertex has only one outgoing edge within $3(n-$ $\left.3-\left(\left(\log _{2} n\right)-1\right)\right)-1$ questions in this phase; that is, $4 n-3\left(\log _{2} n\right)-15$ questions total. + +Comment 3. We can also improve the upper bound slightly, to $4 n-2\left(\log _{2} n\right)+1$. (We do not know where the precise minimum number of questions lies between $4 n-3\left(\log _{2} n\right)+O(1)$ and $4 n-2\left(\log _{2} n\right)+$ $O(1)$.) Suppose $n \geqslant 5$ (otherwise no questions are required at all). + +To do this, we replace Phases 1 and 2 of the given solution with a different strategy that also results in a spanning tree where one vertex $V$ is not known to have any outgoing edges, and all other vertices have exactly one outgoing edge known, but where there is more control over the numbers of incoming edges. In Phases 3 and 4 we then take more care about the order in which pairs of towns are chosen, to ensure that each of the remaining towns has already had a question asked about at least $\log _{2} n+O(1)$ edges. + +Define trees $T_{m}$ with $2^{m}$ vertices, exactly one of which (the root) has no outgoing edges and the rest of which have exactly one outgoing edge, as follows: $T_{0}$ is a single vertex, while $T_{m}$ is constructed by joining the roots of two copies of $T_{m-1}$ with an edge in either direction. If $n=2^{m}$ we can readily ask $n-1$ questions, resulting in a tree $T_{m}$ for the edges with known direction: first ask about $2^{m-1}$ disjoint pairs of vertices, then about $2^{m-2}$ disjoint pairs of the roots of the resulting $T_{1}$ trees, and so on. For the general case, where $n$ is not a power of 2 , after $k$ stages of this process we have $\left\lfloor n / 2^{k}\right\rfloor$ trees, each of which is like $T_{k}$ but may have some extra vertices (but, however, a unique root). If there are an even number of trees, then ask about pairs of their roots. If there are an odd number (greater than 1) of trees, when a single $T_{k}$ is left over, ask about its root together with that of one of the $T_{k+1}$ trees. + +Say $m=\left\lfloor\log _{2} n\right\rfloor$. The result of that process is a single $T_{m}$ tree, possibly with some extra vertices but still a unique root $V$. That root has at least $m$ incoming edges, and we may list vertices $V_{0}$, $\ldots, V_{m-1}$ with edges to $V$, such that, for all $0 \leqslant i1$ we have $D\left(x_{i}, x_{j}\right)>d$, since + +$$ +\left|x_{i}-x_{j}\right|=\left|x_{i+1}-x_{i}\right|+\left|x_{j}-x_{i+1}\right| \geqslant 2^{d}+2^{d}=2^{d+1} . +$$ + +Now choose the minimal $i \leqslant t$ and the maximal $j \geqslant t+1$ such that $D\left(x_{i}, x_{i+1}\right)=$ $D\left(x_{i+1}, x_{i+2}\right)=\cdots=D\left(x_{j-1}, x_{j}\right)=d$. + +Let $E$ be the set of all the $x_{s}$ with even indices $i \leqslant s \leqslant j, O$ be the set of those with odd indices $i \leqslant s \leqslant j$, and $R$ be the rest of the elements (so that $\mathcal{S}$ is the disjoint union of $E, O$ and $R$ ). Set $\mathcal{S}_{O}=R \cup O$ and $\mathcal{S}_{E}=R \cup E$; we have $\left|\mathcal{S}_{O}\right|<|\mathcal{S}|$ and $\left|\mathcal{S}_{E}\right|<|\mathcal{S}|$, so $w\left(\mathcal{S}_{O}\right), w\left(\mathcal{S}_{E}\right) \leqslant 1$ by the inductive hypothesis. + +Clearly, $r_{\mathcal{S}_{O}}(x) \leqslant r_{\mathcal{S}}(x)$ and $r_{\mathcal{S}_{E}}(x) \leqslant r_{\mathcal{S}}(x)$ for any $x \in R$, and thus + +$$ +\begin{aligned} +\sum_{x \in R} 2^{-r_{\mathcal{S}}(x)} & =\frac{1}{2} \sum_{x \in R}\left(2^{-r_{\mathcal{S}}(x)}+2^{-r_{\mathcal{S}}(x)}\right) \\ +& \leqslant \frac{1}{2} \sum_{x \in R}\left(2^{-r_{\mathcal{S}_{O}}(x)}+2^{-r_{\mathcal{S}_{E}}(x)}\right) +\end{aligned} +$$ + +On the other hand, for every $x \in O$, there is no $y \in \mathcal{S}_{O}$ such that $D_{\mathcal{S}_{O}}(x, y)=d$ (as all candidates from $\mathcal{S}$ were in $E$ ). Hence, we have $r_{\mathcal{S}_{O}}(x) \leqslant r_{\mathcal{S}}(x)-1$, and thus + +$$ +\sum_{x \in O} 2^{-r \mathcal{S}_{\mathcal{S}}(x)} \leqslant \frac{1}{2} \sum_{x \in O} 2^{-r s_{O}(x)} +$$ + +Similarly, for every $x \in E$, we have + +$$ +\sum_{x \in E} 2^{-r_{\mathcal{S}}(x)} \leqslant \frac{1}{2} \sum_{x \in E} 2^{-r_{\mathcal{S}_{E}}(x)} +$$ + +We can then combine these to give + +$$ +\begin{aligned} +w(S) & =\sum_{x \in R} 2^{-r_{\mathcal{S}}(x)}+\sum_{x \in O} 2^{-r_{\mathcal{S}}(x)}+\sum_{x \in E} 2^{-r_{\mathcal{S}}(x)} \\ +& \leqslant \frac{1}{2} \sum_{x \in R}\left(2^{-r_{\mathcal{S}_{O}}(x)}+2^{-r_{\mathcal{S}_{E}}(x)}\right)+\frac{1}{2} \sum_{x \in O} 2^{-r_{\mathcal{S}_{O}}(x)}+\frac{1}{2} \sum_{x \in E} 2^{-r_{\mathcal{S}_{E}}(x)} \\ +& \left.=\frac{1}{2}\left(\sum_{x \in \mathcal{S}_{O}} 2^{-r_{S_{O}}(x)}+\sum_{x \in \mathcal{S}_{E}} 2^{-r s_{S_{E}}(x)}\right) \quad \text { (since } \mathcal{S}_{O}=O \cup R \text { and } \mathcal{S}_{E}=E \cup R\right) \\ +& \left.\left.=\frac{1}{2}\left(w\left(\mathcal{S}_{O}\right)+w\left(\mathcal{S}_{E}\right)\right)\right) \quad \text { (by definition of } w(\cdot)\right) \\ +& \leqslant 1 \quad \text { (by the inductive hypothesis) } +\end{aligned} +$$ + +which completes the induction. +Comment 1. The sets $O$ and $E$ above are not the only ones we could have chosen. Indeed, we could instead have used the following definitions: + +Let $d$ be the maximal scale between two distinct elements of $\mathcal{S}$; that is, $d=D\left(x_{1}, x_{n}\right)$. Let $O=\left\{x \in \mathcal{S}: D\left(x, x_{n}\right)=d\right\}$ (a 'left' part of the set) and let $E=\left\{x \in \mathcal{S}: D\left(x_{1}, x\right)=d\right\}$ (a 'right' part of the set). Note that these two sets are disjoint, and nonempty (since they contain $x_{1}$ and $x_{n}$ respectively). The rest of the proof is then the same as in Solution 1. + +Comment 2. Another possible set $\mathcal{F}$ containing $2^{k}$ members could arise from considering a binary tree of height $k$, allocating a real number to each leaf, and trying to make the scale between the values of two leaves dependent only on the (graph) distance between them. The following construction makes this more precise. + +We build up sets $\mathcal{F}_{k}$ recursively. Let $\mathcal{F}_{0}=\{0\}$, and then let $\mathcal{F}_{k+1}=\mathcal{F}_{k} \cup\left\{x+3 \cdot 4^{k}: x \in \mathcal{F}_{k}\right\}$ (i.e. each half of $\mathcal{F}_{k+1}$ is a copy of $F_{k}$ ). We have that $\mathcal{F}_{k}$ is contained in the interval $\left[0,4^{k+1}\right.$ ), and so it follows by induction on $k$ that every member of $F_{k+1}$ has $k$ different scales in its own half of $F_{k+1}$ (by the inductive hypothesis), and only the single scale $2 k+1$ in the other half of $F_{k+1}$. + +Both of the constructions presented here have the property that every member of $\mathcal{F}$ has exactly $k$ different scales in $\mathcal{F}$. Indeed, it can be seen that this must hold (up to a slight perturbation) for any such maximal set. Suppose there were some element $x$ with only $k-1$ different scales in $\mathcal{F}$ (and every other element had at most $k$ different scales). Then we take some positive real $\epsilon$, and construct a new set $\mathcal{F}^{\prime}=\{y: y \in \mathcal{F}, y \leqslant x\} \cup\{y+\epsilon: y \in \mathcal{F}, y \geqslant x\}$. We have $\left|\mathcal{F}^{\prime}\right|=|\mathcal{F}|+1$, and if $\epsilon$ is sufficiently small then $\mathcal{F}^{\prime}$ will also satisfy the property that no member has more than $k$ different scales in $\mathcal{F}^{\prime}$. + +This observation might be used to motivate the idea of weighting members of an arbitrary set $\mathcal{S}$ of reals according to how many different scales they have in $\mathcal{S}$. + +## Geometry + +G1. Let $A B C$ be a triangle. Circle $\Gamma$ passes through $A$, meets segments $A B$ and $A C$ again at points $D$ and $E$ respectively, and intersects segment $B C$ at $F$ and $G$ such that $F$ lies between $B$ and $G$. The tangent to circle $B D F$ at $F$ and the tangent to circle $C E G$ at $G$ meet at point $T$. Suppose that points $A$ and $T$ are distinct. Prove that line $A T$ is parallel to $B C$. +(Nigeria) +Solution. Notice that $\angle T F B=\angle F D A$ because $F T$ is tangent to circle $B D F$, and moreover $\angle F D A=\angle C G A$ because quadrilateral $A D F G$ is cyclic. Similarly, $\angle T G B=\angle G E C$ because $G T$ is tangent to circle $C E G$, and $\angle G E C=\angle C F A$. Hence, + +$$ +\angle T F B=\angle C G A \text { and } \quad \angle T G B=\angle C F A +$$ + +![](https://cdn.mathpix.com/cropped/2024_11_18_9ee8c2ac2c2f5002ff14g-058.jpg?height=624&width=1326&top_left_y=930&top_left_x=365) + +Triangles $F G A$ and $G F T$ have a common side $F G$, and by (1) their angles at $F, G$ are the same. So, these triangles are congruent. So, their altitudes starting from $A$ and $T$, respectively, are equal and hence $A T$ is parallel to line $B F G C$. + +Comment. Alternatively, we can prove first that $T$ lies on $\Gamma$. For example, this can be done by showing that $\angle A F T=\angle A G T$ using (1). Then the statement follows as $\angle T A F=\angle T G F=\angle G F A$. + +## G2. Let $A B C$ be an acute-angled triangle and let $D, E$, and $F$ be the feet of altitudes + +from $A, B$, and $C$ to sides $B C, C A$, and $A B$, respectively. Denote by $\omega_{B}$ and $\omega_{C}$ the incircles of triangles $B D F$ and $C D E$, and let these circles be tangent to segments $D F$ and $D E$ at $M$ and $N$, respectively. Let line $M N$ meet circles $\omega_{B}$ and $\omega_{C}$ again at $P \neq M$ and $Q \neq N$, respectively. Prove that $M P=N Q$. +(Vietnam) +Solution. Denote the centres of $\omega_{B}$ and $\omega_{C}$ by $O_{B}$ and $O_{C}$, let their radii be $r_{B}$ and $r_{C}$, and let $B C$ be tangent to the two circles at $T$ and $U$, respectively. +![](https://cdn.mathpix.com/cropped/2024_11_18_9ee8c2ac2c2f5002ff14g-059.jpg?height=801&width=1014&top_left_y=659&top_left_x=521) + +From the cyclic quadrilaterals $A F D C$ and $A B D E$ we have + +$$ +\angle M D O_{B}=\frac{1}{2} \angle F D B=\frac{1}{2} \angle B A C=\frac{1}{2} \angle C D E=\angle O_{C} D N, +$$ + +so the right-angled triangles $D M O_{B}$ and $D N O_{C}$ are similar. The ratio of similarity between the two triangles is + +$$ +\frac{D N}{D M}=\frac{O_{C} N}{O_{B} M}=\frac{r_{C}}{r_{B}} . +$$ + +Let $\varphi=\angle D M N$ and $\psi=\angle M N D$. The lines $F M$ and $E N$ are tangent to $\omega_{B}$ and $\omega_{C}$, respectively, so + +$$ +\angle M T P=\angle F M P=\angle D M N=\varphi \quad \text { and } \quad \angle Q U N=\angle Q N E=\angle M N D=\psi +$$ + +(It is possible that $P$ or $Q$ coincides with $T$ or $U$, or lie inside triangles $D M T$ or $D U N$, respectively. To reduce case-sensitivity, we may use directed angles or simply ignore angles $M T P$ and $Q U N$.) + +In the circles $\omega_{B}$ and $\omega_{C}$ the lengths of chords $M P$ and $N Q$ are + +$$ +M P=2 r_{B} \cdot \sin \angle M T P=2 r_{B} \cdot \sin \varphi \quad \text { and } \quad N Q=2 r_{C} \cdot \sin \angle Q U N=2 r_{C} \cdot \sin \psi +$$ + +By applying the sine rule to triangle $D N M$ we get + +$$ +\frac{D N}{D M}=\frac{\sin \angle D M N}{\sin \angle M N D}=\frac{\sin \varphi}{\sin \psi} +$$ + +Finally, putting the above observations together, we get + +$$ +\frac{M P}{N Q}=\frac{2 r_{B} \sin \varphi}{2 r_{C} \sin \psi}=\frac{r_{B}}{r_{C}} \cdot \frac{\sin \varphi}{\sin \psi}=\frac{D M}{D N} \cdot \frac{\sin \varphi}{\sin \psi}=\frac{\sin \psi}{\sin \varphi} \cdot \frac{\sin \varphi}{\sin \psi}=1, +$$ + +so $M P=N Q$ as required. + +G3. In triangle $A B C$, let $A_{1}$ and $B_{1}$ be two points on sides $B C$ and $A C$, and let $P$ and $Q$ be two points on segments $A A_{1}$ and $B B_{1}$, respectively, so that line $P Q$ is parallel to $A B$. On ray $P B_{1}$, beyond $B_{1}$, let $P_{1}$ be a point so that $\angle P P_{1} C=\angle B A C$. Similarly, on ray $Q A_{1}$, beyond $A_{1}$, let $Q_{1}$ be a point so that $\angle C Q_{1} Q=\angle C B A$. Show that points $P, Q, P_{1}$, and $Q_{1}$ are concyclic. +(Ukraine) +Solution 1. Throughout the solution we use oriented angles. +Let rays $A A_{1}$ and $B B_{1}$ intersect the circumcircle of $\triangle A C B$ at $A_{2}$ and $B_{2}$, respectively. By + +$$ +\angle Q P A_{2}=\angle B A A_{2}=\angle B B_{2} A_{2}=\angle Q B_{2} A_{2} +$$ + +points $P, Q, A_{2}, B_{2}$ are concyclic; denote the circle passing through these points by $\omega$. We shall prove that $P_{1}$ and $Q_{1}$ also lie on $\omega$. +![](https://cdn.mathpix.com/cropped/2024_11_18_9ee8c2ac2c2f5002ff14g-060.jpg?height=615&width=858&top_left_y=909&top_left_x=602) + +By + +$$ +\angle C A_{2} A_{1}=\angle C A_{2} A=\angle C B A=\angle C Q_{1} Q=\angle C Q_{1} A_{1}, +$$ + +points $C, Q_{1}, A_{2}, A_{1}$ are also concyclic. From that we get + +$$ +\angle Q Q_{1} A_{2}=\angle A_{1} Q_{1} A_{2}=\angle A_{1} C A_{2}=\angle B C A_{2}=\angle B A A_{2}=\angle Q P A_{2} +$$ + +so $Q_{1}$ lies on $\omega$. +It follows similarly that $P_{1}$ lies on $\omega$. +Solution 2. First consider the case when lines $P P_{1}$ and $Q Q_{1}$ intersect each other at some point $R$. + +Let line $P Q$ meet the sides $A C$ and $B C$ at $E$ and $F$, respectively. Then + +$$ +\angle P P_{1} C=\angle B A C=\angle P E C +$$ + +so points $C, E, P, P_{1}$ lie on a circle; denote that circle by $\omega_{P}$. It follows analogously that points $C, F, Q, Q_{1}$ lie on another circle; denote it by $\omega_{Q}$. + +Let $A Q$ and $B P$ intersect at $T$. Applying Pappus' theorem to the lines $A A_{1} P$ and $B B_{1} Q$ provides that points $C=A B_{1} \cap B A_{1}, R=A_{1} Q \cap B_{1} P$ and $T=A Q \cap B P$ are collinear. + +Let line $R C T$ meet $P Q$ and $A B$ at $S$ and $U$, respectively. From $A B \| P Q$ we obtain + +$$ +\frac{S P}{S Q}=\frac{U B}{U A}=\frac{S F}{S E} +$$ + +so + +$$ +S P \cdot S E=S Q \cdot S F +$$ + +![](https://cdn.mathpix.com/cropped/2024_11_18_9ee8c2ac2c2f5002ff14g-061.jpg?height=792&width=1072&top_left_y=204&top_left_x=498) + +So, point $S$ has equal powers with respect to $\omega_{P}$ and $\omega_{Q}$, hence line $R C S$ is their radical axis; then $R$ also has equal powers to the circles, so $R P \cdot R P_{1}=R Q \cdot R Q_{1}$, proving that points $P, P_{1}, Q, Q_{1}$ are indeed concyclic. + +Now consider the case when $P P_{1}$ and $Q Q_{1}$ are parallel. Like in the previous case, let $A Q$ and $B P$ intersect at $T$. Applying Pappus' theorem again to the lines $A A_{1} P$ and $B B_{1} Q$, in this limit case it shows that line $C T$ is parallel to $P P_{1}$ and $Q Q_{1}$. + +Let line $C T$ meet $P Q$ and $A B$ at $S$ and $U$, as before. The same calculation as in the previous case shows that $S P \cdot S E=S Q \cdot S F$, so $S$ lies on the radical axis between $\omega_{P}$ and $\omega_{Q}$. +![](https://cdn.mathpix.com/cropped/2024_11_18_9ee8c2ac2c2f5002ff14g-061.jpg?height=800&width=1043&top_left_y=1413&top_left_x=512) + +Line $C S T$, that is the radical axis between $\omega_{P}$ and $\omega_{Q}$, is perpendicular to the line $\ell$ of centres of $\omega_{P}$ and $\omega_{Q}$. Hence, the chords $P P_{1}$ and $Q Q_{1}$ are perpendicular to $\ell$. So the quadrilateral $P P_{1} Q_{1} Q$ is an isosceles trapezium with symmetry axis $\ell$, and hence is cyclic. + +Comment. There are several ways of solving the problem involving Pappus' theorem. For example, one may consider the points $K=P B_{1} \cap B C$ and $L=Q A_{1} \cap A C$. Applying Pappus' theorem to the lines $A A_{1} P$ and $Q B_{1} B$ we get that $K, L$, and $P Q \cap A B$ are collinear, i.e. that $K L \| A B$. Therefore, cyclicity of $P, Q, P_{1}$, and $Q_{1}$ is equivalent to that of $K, L, P_{1}$, and $Q_{1}$. The latter is easy after noticing that $C$ also lies on that circle. Indeed, e.g. $\angle(L K, L C)=\angle(A B, A C)=\angle\left(P_{1} K, P_{1} C\right)$ shows that $K$ lies on circle $K L C$. + +This approach also has some possible degeneracy, as the points $K$ and $L$ may happen to be ideal. + +G4. Let $P$ be a point inside triangle $A B C$. Let $A P$ meet $B C$ at $A_{1}$, let $B P$ meet $C A$ at $B_{1}$, and let $C P$ meet $A B$ at $C_{1}$. Let $A_{2}$ be the point such that $A_{1}$ is the midpoint of $P A_{2}$, let $B_{2}$ be the point such that $B_{1}$ is the midpoint of $P B_{2}$, and let $C_{2}$ be the point such that $C_{1}$ is the midpoint of $P C_{2}$. Prove that points $A_{2}, B_{2}$, and $C_{2}$ cannot all lie strictly inside the circumcircle of triangle $A B C$. +(Australia) +![](https://cdn.mathpix.com/cropped/2024_11_18_9ee8c2ac2c2f5002ff14g-062.jpg?height=763&width=715&top_left_y=578&top_left_x=662) + +Solution 1. Since + +$$ +\angle A P B+\angle B P C+\angle C P A=2 \pi=(\pi-\angle A C B)+(\pi-\angle B A C)+(\pi-\angle C B A), +$$ + +at least one of the following inequalities holds: + +$$ +\angle A P B \geqslant \pi-\angle A C B, \quad \angle B P C \geqslant \pi-\angle B A C, \quad \angle C P A \geqslant \pi-\angle C B A . +$$ + +Without loss of generality, we assume that $\angle B P C \geqslant \pi-\angle B A C$. We have $\angle B P C>\angle B A C$ because $P$ is inside $\triangle A B C$. So $\angle B P C \geqslant \max (\angle B A C, \pi-\angle B A C)$ and hence + +$$ +\sin \angle B P C \leqslant \sin \angle B A C . +$$ + +Let the rays $A P, B P$, and $C P$ cross the circumcircle $\Omega$ again at $A_{3}, B_{3}$, and $C_{3}$, respectively. We will prove that at least one of the ratios $\frac{P B_{1}}{B_{1} B_{3}}$ and $\frac{P C_{1}}{C_{1} C_{3}}$ is at least 1 , which yields that one of the points $B_{2}$ and $C_{2}$ does not lie strictly inside $\Omega$. + +Because $A, B, C, B_{3}$ lie on a circle, the triangles $C B_{1} B_{3}$ and $B B_{1} A$ are similar, so + +$$ +\frac{C B_{1}}{B_{1} B_{3}}=\frac{B B_{1}}{B_{1} A} +$$ + +Applying the sine rule we obtain + +$$ +\frac{P B_{1}}{B_{1} B_{3}}=\frac{P B_{1}}{C B_{1}} \cdot \frac{C B_{1}}{B_{1} B_{3}}=\frac{P B_{1}}{C B_{1}} \cdot \frac{B B_{1}}{B_{1} A}=\frac{\sin \angle A C P}{\sin \angle B P C} \cdot \frac{\sin \angle B A C}{\sin \angle P B A} . +$$ + +Similarly, + +$$ +\frac{P C_{1}}{C_{1} C_{3}}=\frac{\sin \angle P B A}{\sin \angle B P C} \cdot \frac{\sin \angle B A C}{\sin \angle A C P} +$$ + +Multiplying these two equations we get + +$$ +\frac{P B_{1}}{B_{1} B_{3}} \cdot \frac{P C_{1}}{C_{1} C_{3}}=\frac{\sin ^{2} \angle B A C}{\sin ^{2} \angle B P C} \geqslant 1 +$$ + +using (*), which yields the desired conclusion. + +Comment. It also cannot happen that all three points $A_{2}, B_{2}$, and $C_{2}$ lie strictly outside $\Omega$. The same proof works almost literally, starting by assuming without loss of generality that $\angle B P C \leqslant \pi-\angle B A C$ and using $\angle B P C>\angle B A C$ to deduce that $\sin \angle B P C \geqslant \sin \angle B A C$. It is possible for $A_{2}, B_{2}$, and $C_{2}$ all to lie on the circumcircle; from the above solution we may derive that this happens if and only if $P$ is the orthocentre of the triangle $A B C$, (which lies strictly inside $A B C$ if and only if $A B C$ is acute). + +Solution 2. Define points $A_{3}, B_{3}$, and $C_{3}$ as in Solution 1. Assume for the sake of contradiction that $A_{2}, B_{2}$, and $C_{2}$ all lie strictly inside circle $A B C$. It follows that $P A_{1}0$ and $\alpha+\beta+\gamma=1$. Then + +$$ +A_{1}=\frac{\beta B+\gamma C}{\beta+\gamma}=\frac{1}{1-\alpha} P-\frac{\alpha}{1-\alpha} A, +$$ + +so + +$$ +A_{2}=2 A_{1}-P=\frac{1+\alpha}{1-\alpha} P-\frac{2 \alpha}{1-\alpha} A +$$ + +Hence + +$$ +\left|A_{2}\right|^{2}=\left(\frac{1+\alpha}{1-\alpha}\right)^{2}|P|^{2}+\left(\frac{2 \alpha}{1-\alpha}\right)^{2}|A|^{2}-\frac{4 \alpha(1+\alpha)}{(1-\alpha)^{2}} A \cdot P +$$ + +Using $|A|^{2}=1$ we obtain + +$$ +\frac{(1-\alpha)^{2}}{2(1+\alpha)}\left|A_{2}\right|^{2}=\frac{1+\alpha}{2}|P|^{2}+\frac{2 \alpha^{2}}{1+\alpha}-2 \alpha A \cdot P +$$ + +Likewise + +$$ +\frac{(1-\beta)^{2}}{2(1+\beta)}\left|B_{2}\right|^{2}=\frac{1+\beta}{2}|P|^{2}+\frac{2 \beta^{2}}{1+\beta}-2 \beta B \cdot P +$$ + +and + +$$ +\frac{(1-\gamma)^{2}}{2(1+\gamma)}\left|C_{2}\right|^{2}=\frac{1+\gamma}{2}|P|^{2}+\frac{2 \gamma^{2}}{1+\gamma}-2 \gamma C \cdot P +$$ + +Summing (1), (2) and (3) we obtain on the LHS the positive linear combination + +$$ +\text { LHS }=\frac{(1-\alpha)^{2}}{2(1+\alpha)}\left|A_{2}\right|^{2}+\frac{(1-\beta)^{2}}{2(1+\beta)}\left|B_{2}\right|^{2}+\frac{(1-\gamma)^{2}}{2(1+\gamma)}\left|C_{2}\right|^{2} +$$ + +and on the RHS the quantity + +$$ +\left(\frac{1+\alpha}{2}+\frac{1+\beta}{2}+\frac{1+\gamma}{2}\right)|P|^{2}+\left(\frac{2 \alpha^{2}}{1+\alpha}+\frac{2 \beta^{2}}{1+\beta}+\frac{2 \gamma^{2}}{1+\gamma}\right)-2(\alpha A \cdot P+\beta B \cdot P+\gamma C \cdot P) . +$$ + +The first term is $2|P|^{2}$ and the last term is $-2 P \cdot P$, so + +$$ +\begin{aligned} +\text { RHS } & =\left(\frac{2 \alpha^{2}}{1+\alpha}+\frac{2 \beta^{2}}{1+\beta}+\frac{2 \gamma^{2}}{1+\gamma}\right) \\ +& =\frac{3 \alpha-1}{2}+\frac{(1-\alpha)^{2}}{2(1+\alpha)}+\frac{3 \beta-1}{2}+\frac{(1-\beta)^{2}}{2(1+\beta)}+\frac{3 \gamma-1}{2}+\frac{(1-\gamma)^{2}}{2(1+\gamma)} \\ +& =\frac{(1-\alpha)^{2}}{2(1+\alpha)}+\frac{(1-\beta)^{2}}{2(1+\beta)}+\frac{(1-\gamma)^{2}}{2(1+\gamma)} +\end{aligned} +$$ + +Here we used the fact that + +$$ +\frac{3 \alpha-1}{2}+\frac{3 \beta-1}{2}+\frac{3 \gamma-1}{2}=0 . +$$ + +We have shown that a linear combination of $\left|A_{1}\right|^{2},\left|B_{1}\right|^{2}$, and $\left|C_{1}\right|^{2}$ with positive coefficients is equal to the sum of the coefficients. Therefore at least one of $\left|A_{1}\right|^{2},\left|B_{1}\right|^{2}$, and $\left|C_{1}\right|^{2}$ must be at least 1 , as required. + +Comment. This proof also works when $P$ is any point for which $\alpha, \beta, \gamma>-1, \alpha+\beta+\gamma=1$, and $\alpha, \beta, \gamma \neq 1$. (In any cases where $\alpha=1$ or $\beta=1$ or $\gamma=1$, some points in the construction are not defined.) + +This page is intentionally left blank + +## G5. Let $A B C D E$ be a convex pentagon with $C D=D E$ and $\angle E D C \neq 2 \cdot \angle A D B$. + +Suppose that a point $P$ is located in the interior of the pentagon such that $A P=A E$ and $B P=B C$. Prove that $P$ lies on the diagonal $C E$ if and only if area $(B C D)+\operatorname{area}(A D E)=$ $\operatorname{area}(A B D)+\operatorname{area}(A B P)$. +(Hungary) +Solution 1. Let $P^{\prime}$ be the reflection of $P$ across line $A B$, and let $M$ and $N$ be the midpoints of $P^{\prime} E$ and $P^{\prime} C$ respectively. Convexity ensures that $P^{\prime}$ is distinct from both $E$ and $C$, and hence from both $M$ and $N$. We claim that both the area condition and the collinearity condition in the problem are equivalent to the condition that the (possibly degenerate) right-angled triangles $A P^{\prime} M$ and $B P^{\prime} N$ are directly similar (equivalently, $A P^{\prime} E$ and $B P^{\prime} C$ are directly similar). +![](https://cdn.mathpix.com/cropped/2024_11_18_9ee8c2ac2c2f5002ff14g-066.jpg?height=535&width=549&top_left_y=806&top_left_x=756) + +For the equivalence with the collinearity condition, let $F$ denote the foot of the perpendicular from $P^{\prime}$ to $A B$, so that $F$ is the midpoint of $P P^{\prime}$. We have that $P$ lies on $C E$ if and only if $F$ lies on $M N$, which occurs if and only if we have the equality $\angle A F M=\angle B F N$ of signed angles modulo $\pi$. By concyclicity of $A P^{\prime} F M$ and $B F P^{\prime} N$, this is equivalent to $\angle A P^{\prime} M=\angle B P^{\prime} N$, which occurs if and only if $A P^{\prime} M$ and $B P^{\prime} N$ are directly similar. +![](https://cdn.mathpix.com/cropped/2024_11_18_9ee8c2ac2c2f5002ff14g-066.jpg?height=306&width=441&top_left_y=1663&top_left_x=813) + +For the other equivalence with the area condition, we have the equality of signed areas $\operatorname{area}(A B D)+\operatorname{area}(A B P)=\operatorname{area}\left(A P^{\prime} B D\right)=\operatorname{area}\left(A P^{\prime} D\right)+\operatorname{area}\left(B D P^{\prime}\right)$. Using the identity $\operatorname{area}(A D E)-\operatorname{area}\left(A P^{\prime} D\right)=\operatorname{area}(A D E)+\operatorname{area}\left(A D P^{\prime}\right)=2$ area $(A D M)$, and similarly for $B$, we find that the area condition is equivalent to the equality + +$$ +\operatorname{area}(D A M)=\operatorname{area}(D B N) . +$$ + +Now note that $A$ and $B$ lie on the perpendicular bisectors of $P^{\prime} E$ and $P^{\prime} C$, respectively. If we write $G$ and $H$ for the feet of the perpendiculars from $D$ to these perpendicular bisectors respectively, then this area condition can be rewritten as + +$$ +M A \cdot G D=N B \cdot H D +$$ + +(In this condition, we interpret all lengths as signed lengths according to suitable conventions: for instance, we orient $P^{\prime} E$ from $P^{\prime}$ to $E$, orient the parallel line $D H$ in the same direction, and orient the perpendicular bisector of $P^{\prime} E$ at an angle $\pi / 2$ clockwise from the oriented segment $P^{\prime} E$ - we adopt the analogous conventions at $B$.) +![](https://cdn.mathpix.com/cropped/2024_11_18_9ee8c2ac2c2f5002ff14g-067.jpg?height=529&width=575&top_left_y=204&top_left_x=758) + +To relate the signed lengths $G D$ and $H D$ to the triangles $A P^{\prime} M$ and $B P^{\prime} N$, we use the following calculation. +Claim. Let $\Gamma$ denote the circle centred on $D$ with both $E$ and $C$ on the circumference, and $h$ the power of $P^{\prime}$ with respect to $\Gamma$. Then we have the equality + +$$ +G D \cdot P^{\prime} M=H D \cdot P^{\prime} N=\frac{1}{4} h \neq 0 . +$$ + +Proof. Firstly, we have $h \neq 0$, since otherwise $P^{\prime}$ would lie on $\Gamma$, and hence the internal angle bisectors of $\angle E D P^{\prime}$ and $\angle P^{\prime} D C$ would pass through $A$ and $B$ respectively. This would violate the angle inequality $\angle E D C \neq 2 \cdot \angle A D B$ given in the question. + +Next, let $E^{\prime}$ denote the second point of intersection of $P^{\prime} E$ with $\Gamma$, and let $E^{\prime \prime}$ denote the point on $\Gamma$ diametrically opposite $E^{\prime}$, so that $E^{\prime \prime} E$ is perpendicular to $P^{\prime} E$. The point $G$ lies on the perpendicular bisectors of the sides $P^{\prime} E$ and $E E^{\prime \prime}$ of the right-angled triangle $P^{\prime} E E^{\prime \prime}$; it follows that $G$ is the midpoint of $P^{\prime} E^{\prime \prime}$. Since $D$ is the midpoint of $E^{\prime} E^{\prime \prime}$, we have that $G D=\frac{1}{2} P^{\prime} E^{\prime}$. Since $P^{\prime} M=\frac{1}{2} P^{\prime} E$, we have $G D \cdot P^{\prime} M=\frac{1}{4} P^{\prime} E^{\prime} \cdot P^{\prime} E=\frac{1}{4} h$. The other equality $H D \cdot P^{\prime} N$ follows by exactly the same argument. +![](https://cdn.mathpix.com/cropped/2024_11_18_9ee8c2ac2c2f5002ff14g-067.jpg?height=727&width=672&top_left_y=1664&top_left_x=726) + +From this claim, we see that the area condition is equivalent to the equality + +$$ +\left(M A: P^{\prime} M\right)=\left(N B: P^{\prime} N\right) +$$ + +of ratios of signed lengths, which is equivalent to direct similarity of $A P^{\prime} M$ and $B P^{\prime} N$, as desired. + +Solution 2. Along the perpendicular bisector of $C E$, define the linear function + +$$ +f(X)=\operatorname{area}(B C X)+\operatorname{area}(A X E)-\operatorname{area}(A B X)-\operatorname{area}(A B P), +$$ + +where, from now on, we always use signed areas. Thus, we want to show that $C, P, E$ are collinear if and only if $f(D)=0$. +![](https://cdn.mathpix.com/cropped/2024_11_18_9ee8c2ac2c2f5002ff14g-068.jpg?height=433&width=512&top_left_y=503&top_left_x=772) + +Let $P^{\prime}$ be the reflection of $P$ across line $A B$. The point $P^{\prime}$ does not lie on the line $C E$. To see this, we let $A^{\prime \prime}$ and $B^{\prime \prime}$ be the points obtained from $A$ and $B$ by dilating with scale factor 2 about $P^{\prime}$, so that $P$ is the orthogonal projection of $P^{\prime}$ onto $A^{\prime \prime} B^{\prime \prime}$. Since $A$ lies on the perpendicular bisector of $P^{\prime} E$, the triangle $A^{\prime \prime} E P^{\prime}$ is right-angled at $E$ (and $B^{\prime \prime} C P^{\prime}$ similarly). If $P^{\prime}$ were to lie on $C E$, then the lines $A^{\prime \prime} E$ and $B^{\prime \prime} C$ would be perpendicular to $C E$ and $A^{\prime \prime}$ and $B^{\prime \prime}$ would lie on the opposite side of $C E$ to $D$. It follows that the line $A^{\prime \prime} B^{\prime \prime}$ does not meet triangle $C D E$, and hence point $P$ does not lie inside $C D E$. But then $P$ must lie inside $A B C E$, and it is clear that such a point cannot reflect to a point $P^{\prime}$ on $C E$. + +We thus let $O$ be the centre of the circle $C E P^{\prime}$. The lines $A O$ and $B O$ are the perpendicular bisectors of $E P^{\prime}$ and $C P^{\prime}$, respectively, so + +$$ +\begin{aligned} +\operatorname{area}(B C O)+\operatorname{area}(A O E) & =\operatorname{area}\left(O P^{\prime} B\right)+\operatorname{area}\left(P^{\prime} O A\right)=\operatorname{area}\left(P^{\prime} B O A\right) \\ +& =\operatorname{area}(A B O)+\operatorname{area}\left(B A P^{\prime}\right)=\operatorname{area}(A B O)+\operatorname{area}(A B P), +\end{aligned} +$$ + +and hence $f(O)=0$. +Notice that if point $O$ coincides with $D$ then points $A, B$ lie in angle domain $C D E$ and $\angle E O C=2 \cdot \angle A O B$, which is not allowed. So, $O$ and $D$ must be distinct. Since $f$ is linear and vanishes at $O$, it follows that $f(D)=0$ if and only if $f$ is constant zero - we want to show this occurs if and only if $C, P, E$ are collinear. +![](https://cdn.mathpix.com/cropped/2024_11_18_9ee8c2ac2c2f5002ff14g-068.jpg?height=556&width=1254&top_left_y=1906&top_left_x=408) + +In the one direction, suppose firstly that $C, P, E$ are not collinear, and let $T$ be the centre of the circle $C E P$. The same calculation as above provides + +$$ +\operatorname{area}(B C T)+\operatorname{area}(A T E)=\operatorname{area}(P B T A)=\operatorname{area}(A B T)-\operatorname{area}(A B P) +$$ + +$$ +f(T)=-2 \operatorname{area}(A B P) \neq 0 +$$ + +Hence, the linear function $f$ is nonconstant with its zero is at $O$, so that $f(D) \neq 0$. +In the other direction, suppose that the points $C, P, E$ are collinear. We will show that $f$ is constant zero by finding a second point (other than $O$ ) at which it vanishes. +![](https://cdn.mathpix.com/cropped/2024_11_18_9ee8c2ac2c2f5002ff14g-069.jpg?height=410&width=1098&top_left_y=366&top_left_x=479) + +Let $Q$ be the reflection of $P$ across the midpoint of $A B$, so $P A Q B$ is a parallelogram. It is easy to see that $Q$ is on the perpendicular bisector of $C E$; for instance if $A^{\prime}$ and $B^{\prime}$ are the points produced from $A$ and $B$ by dilating about $P$ with scale factor 2, then the projection of $Q$ to $C E$ is the midpoint of the projections of $A^{\prime}$ and $B^{\prime}$, which are $E$ and $C$ respectively. The triangles $B C Q$ and $A Q E$ are indirectly congruent, so + +$$ +f(Q)=(\operatorname{area}(B C Q)+\operatorname{area}(A Q E))-(\operatorname{area}(A B Q)-\operatorname{area}(B A P))=0-0=0 +$$ + +The points $O$ and $Q$ are distinct. To see this, consider the circle $\omega$ centred on $Q$ with $P^{\prime}$ on the circumference; since triangle $P P^{\prime} Q$ is right-angled at $P^{\prime}$, it follows that $P$ lies outside $\omega$. On the other hand, $P$ lies between $C$ and $E$ on the line $C P E$. It follows that $C$ and $E$ cannot both lie on $\omega$, so that $\omega$ is not the circle $C E P^{\prime}$ and $Q \neq O$. + +Since $O$ and $Q$ are distinct zeroes of the linear function $f$, we have $f(D)=0$ as desired. +Comment 1. The condition $\angle E D C \neq 2 \cdot \angle A D B$ cannot be omitted. If $D$ is the centre of circle $C E P^{\prime}$, then the condition on triangle areas is satisfied automatically, without having $P$ on line $C E$. + +Comment 2. The "only if" part of this problem is easier than the "if" part. For example, in the second part of Solution 2, the triangles $E A Q$ and $Q B C$ are indirectly congruent, so the sum of their areas is 0 , and $D C Q E$ is a kite. Now one can easily see that $\angle(A Q, D E)=\angle(C D, C B)$ and $\angle(B Q, D C)=\angle(E D, E A)$, whence area $(B C D)=\operatorname{area}(A Q D)+\operatorname{area}(E Q A)$ and $\operatorname{area}(A D E)=$ $\operatorname{area}(B D Q)+\operatorname{area}(B Q C)$, which yields the result. + +Comment 3. The origin of the problem is the following observation. Let $A B D H$ be a tetrahedron and consider the sphere $\mathcal{S}$ that is tangent to the four face planes, internally to planes $A D H$ and $B D H$ and externally to $A B D$ and $A B H$ (or vice versa). It is known that the sphere $\mathcal{S}$ exists if and only if area $(A D H)+\operatorname{area}(B D H) \neq \operatorname{area}(A B H)+\operatorname{area}(A B D)$; this relation comes from the usual formula for the volume of the tetrahedron. + +Let $T, T_{a}, T_{b}, T_{d}$ be the points of tangency between the sphere and the four planes, as shown in the picture. Rotate the triangle $A B H$ inward, the triangles $B D H$ and $A D H$ outward, into the triangles $A B P, B D C$ and $A D E$, respectively, in the plane $A B D$. Notice that the points $T_{d}, T_{a}, T_{b}$ are rotated to $T$, so we have $H T_{a}=H T_{b}=H T_{d}=P T=C T=E T$. Therefore, the point $T$ is the centre of the circle $C E P$. Hence, if the sphere exists then $C, E, P$ cannot be collinear. + +If the condition $\angle E D C \neq 2 \cdot \angle A D B$ is replaced by the constraint that the angles $\angle E D A, \angle A D B$ and $\angle B D C$ satisfy the triangle inequality, it enables reconstructing the argument with the tetrahedron and the tangent sphere. +![](https://cdn.mathpix.com/cropped/2024_11_18_9ee8c2ac2c2f5002ff14g-070.jpg?height=1069&width=1306&top_left_y=825&top_left_x=381) + +## G6. Let $I$ be the incentre of acute-angled triangle $A B C$. Let the incircle meet $B C, C A$, + +and $A B$ at $D, E$, and $F$, respectively. Let line $E F$ intersect the circumcircle of the triangle at $P$ and $Q$, such that $F$ lies between $E$ and $P$. Prove that $\angle D P A+\angle A Q D=\angle Q I P$. +(Slovakia) +Solution 1. Let $N$ and $M$ be the midpoints of the arcs $\widehat{B C}$ of the circumcircle, containing and opposite vertex $A$, respectively. By $\angle F A E=\angle B A C=\angle B N C$, the right-angled kites $A F I E$ and $N B M C$ are similar. Consider the spiral similarity $\varphi$ (dilation in case of $A B=A C$ ) that moves $A F I E$ to $N B M C$. The directed angle in which $\varphi$ changes directions is $\angle(A F, N B)$, same as $\angle(A P, N P)$ and $\angle(A Q, N Q)$; so lines $A P$ and $A Q$ are mapped to lines $N P$ and $N Q$, respectively. Line $E F$ is mapped to $B C$; we can see that the intersection points $P=E F \cap A P$ and $Q=E F \cap A Q$ are mapped to points $B C \cap N P$ and $B C \cap N Q$, respectively. Denote these points by $P^{\prime}$ and $Q^{\prime}$, respectively. +![](https://cdn.mathpix.com/cropped/2024_11_18_9ee8c2ac2c2f5002ff14g-071.jpg?height=689&width=1512&top_left_y=883&top_left_x=272) + +Let $L$ be the midpoint of $B C$. We claim that points $P, Q, D, L$ are concyclic (if $D=L$ then line $B C$ is tangent to circle $P Q D$ ). Let $P Q$ and $B C$ meet at $Z$. By applying Menelaus' theorem to triangle $A B C$ and line $E F Z$, we have + +$$ +\frac{B D}{D C}=\frac{B F}{F A} \cdot \frac{A E}{E C}=-\frac{B Z}{Z C}, +$$ + +so the pairs $B, C$ and $D, Z$ are harmonic. It is well-known that this implies $Z B \cdot Z C=Z D \cdot Z L$. (The inversion with pole $Z$ that swaps $B$ and $C$ sends $Z$ to infinity and $D$ to the midpoint of $B C$, because the cross-ratio is preserved.) Hence, $Z D \cdot Z L=Z B \cdot Z C=Z P \cdot Z Q$ by the power of $Z$ with respect to the circumcircle; this proves our claim. + +By $\angle M P P^{\prime}=\angle M Q Q^{\prime}=\angle M L P^{\prime}=\angle M L Q^{\prime}=90^{\circ}$, the quadrilaterals $M L P P^{\prime}$ and $M L Q Q^{\prime}$ are cyclic. Then the problem statement follows by + +$$ +\begin{aligned} +\angle D P A+\angle A Q D & =360^{\circ}-\angle P A Q-\angle Q D P=360^{\circ}-\angle P N Q-\angle Q L P \\ +& =\angle L P N+\angle N Q L=\angle P^{\prime} M L+\angle L M Q^{\prime}=\angle P^{\prime} M Q^{\prime}=\angle P I Q . +\end{aligned} +$$ + +Solution 2. Define the point $M$ and the same spiral similarity $\varphi$ as in the previous solution. (The point $N$ is not necessary.) It is well-known that the centre of the spiral similarity that maps $F, E$ to $B, C$ is the Miquel point of the lines $F E, B C, B F$ and $C E$; that is, the second intersection of circles $A B C$ and $A E F$. Denote that point by $S$. + +By $\varphi(F)=B$ and $\varphi(E)=C$ the triangles $S B F$ and $S C E$ are similar, so we have + +$$ +\frac{S B}{S C}=\frac{B F}{C E}=\frac{B D}{C D} +$$ + +By the converse of the angle bisector theorem, that indicates that line $S D$ bisects $\angle B S C$ and hence passes through $M$. + +Let $K$ be the intersection point of lines $E F$ and $S I$. Notice that $\varphi$ sends points $S, F, E, I$ to $S, B, C, M$, so $\varphi(K)=\varphi(F E \cap S I)=B C \cap S M=D$. By $\varphi(I)=M$, we have $K D \| I M$. +![](https://cdn.mathpix.com/cropped/2024_11_18_9ee8c2ac2c2f5002ff14g-072.jpg?height=669&width=706&top_left_y=839&top_left_x=675) + +We claim that triangles $S P I$ and $S D Q$ are similar, and so are triangles $S P D$ and $S I Q$. Let ray $S I$ meet the circumcircle again at $L$. Note that the segment $E F$ is perpendicular to the angle bisector $A M$. Then by $\angle A M L=\angle A S L=\angle A S I=90^{\circ}$, we have $M L \| P Q$. Hence, $\widetilde{P L}=\widetilde{M Q}$ and therefore $\angle P S L=\angle M S Q=\angle D S Q$. By $\angle Q P S=\angle Q M S$, the triangles $S P K$ and $S M Q$ are similar. Finally, + +$$ +\frac{S P}{S I}=\frac{S P}{S K} \cdot \frac{S K}{S I}=\frac{S M}{S Q} \cdot \frac{S D}{S M}=\frac{S D}{S Q} +$$ + +shows that triangles $S P I$ and $S D Q$ are similar. The second part of the claim can be proved analogously. + +Now the problem statement can be proved by + +$$ +\angle D P A+\angle A Q D=\angle D P S+\angle S Q D=\angle Q I S+\angle S I P=\angle Q I P . +$$ + +Solution 3. Denote the circumcircle of triangle $A B C$ by $\Gamma$, and let rays $P D$ and $Q D$ meet $\Gamma$ again at $V$ and $U$, respectively. We will show that $A U \perp I P$ and $A V \perp I Q$. Then the problem statement will follow as + +$$ +\angle D P A+\angle A Q D=\angle V U A+\angle A V U=180^{\circ}-\angle U A V=\angle Q I P . +$$ + +Let $M$ be the midpoint of arc $\widehat{B U V C}$ and let $N$ be the midpoint of arc $\widehat{C A B}$; the lines $A I M$ and $A N$ being the internal and external bisectors of angle $B A C$, respectively, are perpendicular. Let the tangents drawn to $\Gamma$ at $B$ and $C$ meet at $R$; let line $P Q$ meet $A U, A I, A V$ and $B C$ at $X, T, Y$ and $Z$, respectively. + +As in Solution 1, we observe that the pairs $B, C$ and $D, Z$ are harmonic. Projecting these points from $Q$ onto the circumcircle, we can see that $B, C$ and $U, P$ are also harmonic. Analogously, the pair $V, Q$ is harmonic with $B, C$. Consider the inversion about the circle with centre $R$, passing through $B$ and $C$. Points $B$ and $C$ are fixed points, so this inversion exchanges every point of $\Gamma$ by its harmonic pair with respect to $B, C$. In particular, the inversion maps points $B, C, N, U, V$ to points $B, C, M, P, Q$, respectively. + +Combine the inversion with projecting $\Gamma$ from $A$ to line $P Q$; the points $B, C, M, P, Q$ are projected to $F, E, T, P, Q$, respectively. +![](https://cdn.mathpix.com/cropped/2024_11_18_9ee8c2ac2c2f5002ff14g-073.jpg?height=1006&width=1384&top_left_y=1082&top_left_x=339) + +The combination of these two transformations is projective map from the lines $A B, A C$, $A N, A U, A V$ to $I F, I E, I T, I P, I Q$, respectively. On the other hand, we have $A B \perp I F$, $A C \perp I E$ and $A N \perp A T$, so the corresponding lines in these two pencils are perpendicular. This proves $A U \perp I P$ and $A V \perp I Q$, and hence completes the solution. + +## G7. The incircle $\omega$ of acute-angled scalene triangle $A B C$ has centre $I$ and meets sides $B C$, + +$C A$, and $A B$ at $D, E$, and $F$, respectively. The line through $D$ perpendicular to $E F$ meets $\omega$ again at $R$. Line $A R$ meets $\omega$ again at $P$. The circumcircles of triangles $P C E$ and $P B F$ meet again at $Q \neq P$. Prove that lines $D I$ and $P Q$ meet on the external bisector of angle $B A C$. +(India) +Common remarks. Throughout the solution, $\angle(a, b)$ denotes the directed angle between lines $a$ and $b$, measured modulo $\pi$. + +## Solution 1. + +Step 1. The external bisector of $\angle B A C$ is the line through $A$ perpendicular to $I A$. Let $D I$ meet this line at $L$ and let $D I$ meet $\omega$ at $K$. Let $N$ be the midpoint of $E F$, which lies on $I A$ and is the pole of line $A L$ with respect to $\omega$. Since $A N \cdot A I=A E^{2}=A R \cdot A P$, the points $R$, $N, I$, and $P$ are concyclic. As $I R=I P$, the line $N I$ is the external bisector of $\angle P N R$, so $P N$ meets $\omega$ again at the point symmetric to $R$ with respect to $A N-i . e$ at $K$. + +Let $D N$ cross $\omega$ again at $S$. Opposite sides of any quadrilateral inscribed in the circle $\omega$ meet on the polar line of the intersection of the diagonals with respect to $\omega$. Since $L$ lies on the polar line $A L$ of $N$ with respect to $\omega$, the line $P S$ must pass through $L$. Thus it suffices to prove that the points $S, Q$, and $P$ are collinear. +![](https://cdn.mathpix.com/cropped/2024_11_18_9ee8c2ac2c2f5002ff14g-074.jpg?height=875&width=1350&top_left_y=1176&top_left_x=361) + +Step 2. Let $\Gamma$ be the circumcircle of $\triangle B I C$. Notice that + +$$ +\begin{aligned} +& \angle(B Q, Q C)=\angle(B Q, Q P)+\angle(P Q, Q C)=\angle(B F, F P)+\angle(P E, E C) \\ +&=\angle(E F, E P)+\angle(F P, F E)=\angle(F P, E P)=\angle(D F, D E)=\angle(B I, I C) +\end{aligned} +$$ + +so $Q$ lies on $\Gamma$. Let $Q P$ meet $\Gamma$ again at $T$. It will now suffice to prove that $S, P$, and $T$ are collinear. Notice that $\angle(B I, I T)=\angle(B Q, Q T)=\angle(B F, F P)=\angle(F K, K P)$. Note $F D \perp F K$ and $F D \perp B I$ so $F K \| B I$ and hence $I T$ is parallel to the line $K N P$. Since $D I=I K$, the line $I T$ crosses $D N$ at its midpoint $M$. +Step 3. Let $F^{\prime}$ and $E^{\prime}$ be the midpoints of $D E$ and $D F$, respectively. Since $D E^{\prime} \cdot E^{\prime} F=D E^{\prime 2}=$ $B E^{\prime} \cdot E^{\prime} I$, the point $E^{\prime}$ lies on the radical axis of $\omega$ and $\Gamma$; the same holds for $F^{\prime}$. Therefore, this radical axis is $E^{\prime} F^{\prime}$, and it passes through $M$. Thus $I M \cdot M T=D M \cdot M S$, so $S, I, D$, and $T$ are concyclic. This shows $\angle(D S, S T)=\angle(D I, I T)=\angle(D K, K P)=\angle(D S, S P)$, whence the points $S, P$, and $T$ are collinear, as desired. +![](https://cdn.mathpix.com/cropped/2024_11_18_9ee8c2ac2c2f5002ff14g-075.jpg?height=1089&width=992&top_left_y=198&top_left_x=532) + +Comment. Here is a longer alternative proof in step 1 that $P, S$, and $L$ are collinear, using a circular inversion instead of the fact that opposite sides of a quadrilateral inscribed in a circle $\omega$ meet on the polar line with respect to $\omega$ of the intersection of the diagonals. Let $G$ be the foot of the altitude from $N$ to the line $D I K L$. Observe that $N, G, K, S$ are concyclic (opposite right angles) so + +$$ +\angle D I P=2 \angle D K P=\angle G K N+\angle D S P=\angle G S N+\angle N S P=\angle G S P, +$$ + +hence $I, G, S, P$ are concyclic. We have $I G \cdot I L=I N \cdot I A=r^{2}$ since $\triangle I G N \sim \triangle I A L$. Inverting the circle $I G S P$ in circle $\omega$, points $P$ and $S$ are fixed and $G$ is taken to $L$ so we find that $P, S$, and $L$ are collinear. + +Solution 2. We start as in Solution 1. Namely, we introduce the same points $K, L, N$, and $S$, and show that the triples $(P, N, K)$ and $(P, S, L)$ are collinear. We conclude that $K$ and $R$ are symmetric in $A I$, and reduce the problem statement to showing that $P, Q$, and $S$ are collinear. + +Step 1. Let $A R$ meet the circumcircle $\Omega$ of $A B C$ again at $X$. The lines $A R$ and $A K$ are isogonal in the angle $B A C$; it is well known that in this case $X$ is the tangency point of $\Omega$ with the $A$-mixtilinear circle. It is also well known that for this point $X$, the line $X I$ crosses $\Omega$ again at the midpoint $M^{\prime}$ of arc $B A C$. + +Step 2. Denote the circles $B F P$ and $C E P$ by $\Omega_{B}$ and $\Omega_{C}$, respectively. Let $\Omega_{B}$ cross $A R$ and $E F$ again at $U$ and $Y$, respectively. We have + +$$ +\angle(U B, B F)=\angle(U P, P F)=\angle(R P, P F)=\angle(R F, F A) +$$ + +so $U B \| R F$. +![](https://cdn.mathpix.com/cropped/2024_11_18_9ee8c2ac2c2f5002ff14g-076.jpg?height=1009&width=1474&top_left_y=883&top_left_x=248) + +Next, we show that the points $B, I, U$, and $X$ are concyclic. Since + +$$ +\angle(U B, U X)=\angle(R F, R X)=\angle(A F, A R)+\angle(F R, F A)=\angle\left(M^{\prime} B, M^{\prime} X\right)+\angle(D R, D F) +$$ + +it suffices to prove $\angle(I B, I X)=\angle\left(M^{\prime} B, M^{\prime} X\right)+\angle(D R, D F)$, or $\angle\left(I B, M^{\prime} B\right)=\angle(D R, D F)$. But both angles equal $\angle(C I, C B)$, as desired. (This is where we used the fact that $M^{\prime}$ is the midpoint of $\operatorname{arc} B A C$ of $\Omega$.) + +It follows now from circles $B U I X$ and $B P U F Y$ that + +$$ +\begin{aligned} +\angle(I U, U B)=\angle(I X, B X)=\angle\left(M^{\prime} X, B X\right)= & \frac{\pi-\angle A}{2} \\ +& =\angle(E F, A F)=\angle(Y F, B F)=\angle(Y U, B U) +\end{aligned} +$$ + +so the points $Y, U$, and $I$ are collinear. +Let $E F$ meet $B C$ at $W$. We have + +$$ +\angle(I Y, Y W)=\angle(U Y, F Y)=\angle(U B, F B)=\angle(R F, A F)=\angle(C I, C W) +$$ + +so the points $W, Y, I$, and $C$ are concyclic. + +Similarly, if $V$ and $Z$ are the second meeting points of $\Omega_{C}$ with $A R$ and $E F$, we get that the 4-tuples $(C, V, I, X)$ and $(B, I, Z, W)$ are both concyclic. + +Step 3. Let $Q^{\prime}=C Y \cap B Z$. We will show that $Q^{\prime}=Q$. +First of all, we have + +$$ +\begin{aligned} +& \angle\left(Q^{\prime} Y, Q^{\prime} B\right)=\angle(C Y, Z B)=\angle(C Y, Z Y)+\angle(Z Y, B Z) \\ +& =\angle(C I, I W)+\angle(I W, I B)=\angle(C I, I B)=\frac{\pi-\angle A}{2}=\angle(F Y, F B), +\end{aligned} +$$ + +so $Q^{\prime} \in \Omega_{B}$. Similarly, $Q^{\prime} \in \Omega_{C}$. Thus $Q^{\prime} \in \Omega_{B} \cap \Omega_{C}=\{P, Q\}$ and it remains to prove that $Q^{\prime} \neq P$. If we had $Q^{\prime}=P$, we would have $\angle(P Y, P Z)=\angle\left(Q^{\prime} Y, Q^{\prime} Z\right)=\angle(I C, I B)$. This would imply + +$$ +\angle(P Y, Y F)+\angle(E Z, Z P)=\angle(P Y, P Z)=\angle(I C, I B)=\angle(P E, P F), +$$ + +so circles $\Omega_{B}$ and $\Omega_{C}$ would be tangent at $P$. That is excluded in the problem conditions, so $Q^{\prime}=Q$. +![](https://cdn.mathpix.com/cropped/2024_11_18_9ee8c2ac2c2f5002ff14g-077.jpg?height=966&width=1468&top_left_y=1070&top_left_x=251) + +Step 4. Now we are ready to show that $P, Q$, and $S$ are collinear. +Notice that $A$ and $D$ are the poles of $E W$ and $D W$ with respect to $\omega$, so $W$ is the pole of $A D$. Hence, $W I \perp A D$. Since $C I \perp D E$, this yields $\angle(I C, W I)=\angle(D E, D A)$. On the other hand, $D A$ is a symmedian in $\triangle D E F$, so $\angle(D E, D A)=\angle(D N, D F)=\angle(D S, D F)$. Therefore, + +$$ +\begin{aligned} +\angle(P S, P F)=\angle(D S, D F)=\angle(D E, D A)= & \angle(I C, I W) \\ +& =\angle(Y C, Y W)=\angle(Y Q, Y F)=\angle(P Q, P F), +\end{aligned} +$$ + +which yields the desired collinearity. + +## G8. Let $\mathcal{L}$ be the set of all lines in the plane and let $f$ be a function that assigns to each + +line $\ell \in \mathcal{L}$ a point $f(\ell)$ on $\ell$. Suppose that for any point $X$, and for any three lines $\ell_{1}, \ell_{2}, \ell_{3}$ passing through $X$, the points $f\left(\ell_{1}\right), f\left(\ell_{2}\right), f\left(\ell_{3}\right)$ and $X$ lie on a circle. + +Prove that there is a unique point $P$ such that $f(\ell)=P$ for any line $\ell$ passing through $P$. +(Australia) +Common remarks. The condition on $f$ is equivalent to the following: There is some function $g$ that assigns to each point $X$ a circle $g(X)$ passing through $X$ such that for any line $\ell$ passing through $X$, the point $f(\ell)$ lies on $g(X)$. (The function $g$ may not be uniquely defined for all points, if some points $X$ have at most one value of $f(\ell)$ other than $X$; for such points, an arbitrary choice is made.) + +If there were two points $P$ and $Q$ with the given property, $f(P Q)$ would have to be both $P$ and $Q$, so there is at most one such point, and it will suffice to show that such a point exists. + +Solution 1. We provide a complete characterisation of the functions satisfying the given condition. + +Write $\angle\left(\ell_{1}, \ell_{2}\right)$ for the directed angle modulo $180^{\circ}$ between the lines $\ell_{1}$ and $\ell_{2}$. Given a point $P$ and an angle $\alpha \in\left(0,180^{\circ}\right)$, for each line $\ell$, let $\ell^{\prime}$ be the line through $P$ satisfying $\angle\left(\ell^{\prime}, \ell\right)=\alpha$, and let $h_{P, \alpha}(\ell)$ be the intersection point of $\ell$ and $\ell^{\prime}$. We will prove that there is some pair $(P, \alpha)$ such that $f$ and $h_{P, \alpha}$ are the same function. Then $P$ is the unique point in the problem statement. + +Given an angle $\alpha$ and a point $P$, let a line $\ell$ be called $(P, \alpha)$-good if $f(\ell)=h_{P, \alpha}(\ell)$. Let a point $X \neq P$ be called $(P, \alpha)$-good if the circle $g(X)$ passes through $P$ and some point $Y \neq P, X$ on $g(X)$ satisfies $\angle(P Y, Y X)=\alpha$. It follows from this definition that if $X$ is $(P, \alpha)$ good then every point $Y \neq P, X$ of $g(X)$ satisfies this angle condition, so $h_{P, \alpha}(X Y)=Y$ for every $Y \in g(X)$. Equivalently, $f(\ell) \in\left\{X, h_{P, \alpha}(\ell)\right\}$ for each line $\ell$ passing through $X$. This shows the following lemma. +Lemma 1. If $X$ is $(P, \alpha)$-good and $\ell$ is a line passing through $X$ then either $f(\ell)=X$ or $\ell$ is $(P, \alpha)$-good. +Lemma 2. If $X$ and $Y$ are different $(P, \alpha)$-good points, then line $X Y$ is $(P, \alpha)$-good. +Proof. If $X Y$ is not $(P, \alpha)$-good then by the previous Lemma, $f(X Y)=X$ and similarly $f(X Y)=Y$, but clearly this is impossible as $X \neq Y$. + +Lemma 3. If $\ell_{1}$ and $\ell_{2}$ are different $(P, \alpha)$-good lines which intersect at $X \neq P$, then either $f\left(\ell_{1}\right)=X$ or $f\left(\ell_{2}\right)=X$ or $X$ is $(P, \alpha)$-good. +Proof. If $f\left(\ell_{1}\right), f\left(\ell_{2}\right) \neq X$, then $g(X)$ is the circumcircle of $X, f\left(\ell_{1}\right)$ and $f\left(\ell_{2}\right)$. Since $\ell_{1}$ and $\ell_{2}$ are $(P, \alpha)$-good lines, the angles + +$$ +\angle\left(P f\left(\ell_{1}\right), f\left(\ell_{1}\right) X\right)=\angle\left(P f\left(\ell_{2}\right), f\left(\ell_{2}\right) X\right)=\alpha +$$ + +so $P$ lies on $g(X)$. Hence, $X$ is $(P, \alpha)$-good. +Lemma 4. If $\ell_{1}, \ell_{2}$ and $\ell_{3}$ are different $(P, \alpha)$ good lines which intersect at $X \neq P$, then $X$ is $(P, \alpha)$-good. +Proof. This follows from the previous Lemma since at most one of the three lines $\ell_{i}$ can satisfy $f\left(\ell_{i}\right)=X$ as the three lines are all $(P, \alpha)$-good. +Lemma 5. If $A B C$ is a triangle such that $A, B, C, f(A B), f(A C)$ and $f(B C)$ are all different points, then there is some point $P$ and some angle $\alpha$ such that $A, B$ and $C$ are $(P, \alpha)$-good points and $A B, B C$ and $C A$ are $(P, \alpha)-\operatorname{good}$ lines. +![](https://cdn.mathpix.com/cropped/2024_11_18_9ee8c2ac2c2f5002ff14g-079.jpg?height=415&width=612&top_left_y=207&top_left_x=728) + +Proof. Let $D, E, F$ denote the points $f(B C), f(A C), f(A B)$, respectively. Then $g(A)$, $g(B)$ and $g(C)$ are the circumcircles of $A E F, B D F$ and $C D E$, respectively. Let $P \neq F$ be the second intersection of circles $g(A)$ and $g(B)$ (or, if these circles are tangent at $F$, then $P=F$ ). By Miquel's theorem (or an easy angle chase), $g(C)$ also passes through $P$. Then by the cyclic quadrilaterals, the directed angles + +$$ +\angle(P D, D C)=\angle(P F, F B)=\angle(P E, E A)=\alpha +$$ + +for some angle $\alpha$. Hence, lines $A B, B C$ and $C A$ are all $(P, \alpha)$-good, so by Lemma $3, A, B$ and $C$ are $(P, \alpha)$-good. (In the case where $P=D$, the line $P D$ in the equation above denotes the line which is tangent to $g(B)$ at $P=D$. Similar definitions are used for $P E$ and $P F$ in the cases where $P=E$ or $P=F$.) + +Consider the set $\Omega$ of all points $(x, y)$ with integer coordinates $1 \leqslant x, y \leqslant 1000$, and consider the set $L_{\Omega}$ of all horizontal, vertical and diagonal lines passing through at least one point in $\Omega$. A simple counting argument shows that there are 5998 lines in $L_{\Omega}$. For each line $\ell$ in $L_{\Omega}$ we colour the point $f(\ell)$ red. Then there are at most 5998 red points. Now we partition the points in $\Omega$ into 10000 ten by ten squares. Since there are at most 5998 red points, at least one of these squares $\Omega_{10}$ contains no red points. Let $(m, n)$ be the bottom left point in $\Omega_{10}$. Then the triangle with vertices $(m, n),(m+1, n)$ and $(m, n+1)$ satisfies the condition of Lemma 5 , so these three vertices are all $(P, \alpha)$-good for some point $P$ and angle $\alpha$, as are the lines joining them. From this point on, we will simply call a point or line good if it is $(P, \alpha)$-good for this particular pair $(P, \alpha)$. Now by Lemma 1, the line $x=m+1$ is good, as is the line $y=n+1$. Then Lemma 3 implies that $(m+1, n+1)$ is good. By applying these two lemmas repeatedly, we can prove that the line $x+y=m+n+2$ is good, then the points $(m, n+2)$ and $(m+2, n)$ then the lines $x=m+2$ and $y=n+2$, then the points $(m+2, n+1),(m+1, n+2)$ and $(m+2, n+2)$ and so on until we have prove that all points in $\Omega_{10}$ are good. + +Now we will use this to prove that every point $S \neq P$ is good. Since $g(S)$ is a circle, it passes through at most two points of $\Omega_{10}$ on any vertical line, so at most 20 points in total. Moreover, any line $\ell$ through $S$ intersects at most 10 points in $\Omega_{10}$. Hence, there are at least eight lines $\ell$ through $S$ which contain a point $Q$ in $\Omega_{10}$ which is not on $g(S)$. Since $Q$ is not on $g(S)$, the point $f(\ell) \neq Q$. Hence, by Lemma 1 , the line $\ell$ is good. Hence, at least eight good lines pass through $S$, so by Lemma 4, the point $S$ is good. Hence, every point $S \neq P$ is good, so by Lemma 2 , every line is good. In particular, every line $\ell$ passing through $P$ is good, and therefore satisfies $f(\ell)=P$, as required. + +Solution 2. Note that for any distinct points $X, Y$, the circles $g(X)$ and $g(Y)$ meet on $X Y$ at the point $f(X Y) \in g(X) \cap g(Y) \cap(X Y)$. We write $s(X, Y)$ for the second intersection point of circles $g(X)$ and $g(Y)$. +Lemma 1. Suppose that $X, Y$ and $Z$ are not collinear, and that $f(X Y) \notin\{X, Y\}$ and similarly for $Y Z$ and $Z X$. Then $s(X, Y)=s(Y, Z)=s(Z, X)$. +Proof. The circles $g(X), g(Y)$ and $g(Z)$ through the vertices of triangle $X Y Z$ meet pairwise on the corresponding edges (produced). By Miquel's theorem, the second points of intersection of any two of the circles coincide. (See the diagram for Lemma 5 of Solution 1.) + +Now pick any line $\ell$ and any six different points $Y_{1}, \ldots, Y_{6}$ on $\ell \backslash\{f(\ell)\}$. Pick a point $X$ not on $\ell$ or any of the circles $g\left(Y_{i}\right)$. Reordering the indices if necessary, we may suppose that $Y_{1}, \ldots, Y_{4}$ do not lie on $g(X)$, so that $f\left(X Y_{i}\right) \notin\left\{X, Y_{i}\right\}$ for $1 \leqslant i \leqslant 4$. By applying the above lemma to triangles $X Y_{i} Y_{j}$ for $1 \leqslant i0$, there is a point $P_{\epsilon}$ with $g\left(P_{\epsilon}\right)$ of radius at most $\epsilon$. Then there is a point $P$ with the given property. +Proof. Consider a sequence $\epsilon_{i}=2^{-i}$ and corresponding points $P_{\epsilon_{i}}$. Because the two circles $g\left(P_{\epsilon_{i}}\right)$ and $g\left(P_{\epsilon_{j}}\right)$ meet, the distance between $P_{\epsilon_{i}}$ and $P_{\epsilon_{j}}$ is at most $2^{1-i}+2^{1-j}$. As $\sum_{i} \epsilon_{i}$ converges, these points converge to some point $P$. For all $\epsilon>0$, the point $P$ has distance at most $2 \epsilon$ from $P_{\epsilon}$, and all circles $g(X)$ pass through a point with distance at most $2 \epsilon$ from $P_{\epsilon}$, so distance at most $4 \epsilon$ from $P$. A circle that passes distance at most $4 \epsilon$ from $P$ for all $\epsilon>0$ must pass through $P$, so by Lemma 1 the point $P$ has the given property. + +Lemma 3. Suppose no two of the circles $g(X)$ cut. Then there is a point $P$ with the given property. +Proof. Consider a circle $g(X)$ with centre $Y$. The circle $g(Y)$ must meet $g(X)$ without cutting it, so has half the radius of $g(X)$. Repeating this argument, there are circles with arbitrarily small radius and the result follows by Lemma 2. + +Lemma 4. Suppose there are six different points $A, B_{1}, B_{2}, B_{3}, B_{4}, B_{5}$ such that no three are collinear, no four are concyclic, and all the circles $g\left(B_{i}\right)$ cut pairwise at $A$. Then there is a point $P$ with the given property. +Proof. Consider some line $\ell$ through $A$ that does not pass through any of the $B_{i}$ and is not tangent to any of the $g\left(B_{i}\right)$. Fix some direction along that line, and let $X_{\epsilon}$ be the point on $\ell$ that has distance $\epsilon$ from $A$ in that direction. In what follows we consider only those $\epsilon$ for which $X_{\epsilon}$ does not lie on any $g\left(B_{i}\right)$ (this restriction excludes only finitely many possible values of $\epsilon$ ). + +Consider the circle $g\left(X_{\epsilon}\right)$. Because no four of the $B_{i}$ are concyclic, at most three of them lie on this circle, so at least two of them do not. There must be some sequence of $\epsilon \rightarrow 0$ such that it is the same two of the $B_{i}$ for all $\epsilon$ in that sequence, so now restrict attention to that sequence, and suppose without loss of generality that $B_{1}$ and $B_{2}$ do not lie on $g\left(X_{\epsilon}\right)$ for any $\epsilon$ in that sequence. + +Then $f\left(X_{\epsilon} B_{1}\right)$ is not $B_{1}$, so must be the other point of intersection of $X_{\epsilon} B_{1}$ with $g\left(B_{1}\right)$, and the same applies with $B_{2}$. Now consider the three points $X_{\epsilon}, f\left(X_{\epsilon} B_{1}\right)$ and $f\left(X_{\epsilon} B_{2}\right)$. As $\epsilon \rightarrow 0$, the angle at $X_{\epsilon}$ tends to $\angle B_{1} A B_{2}$ or $180^{\circ}-\angle B_{1} A B_{2}$, which is not 0 or $180^{\circ}$ because no three of the points were collinear. All three distances between those points are bounded above by constant multiples of $\epsilon$ (in fact, if the triangle is scaled by a factor of $1 / \epsilon$, it tends to a fixed triangle). Thus the circumradius of those three points, which is the radius of $g\left(X_{\epsilon}\right)$, is also bounded above by a constant multiple of $\epsilon$, and so the result follows by Lemma 2 . + +Lemma 5. Suppose there are two points $A$ and $B$ such that $g(A)$ and $g(B)$ cut. Then there is a point $P$ with the given property. +Proof. Suppose that $g(A)$ and $g(B)$ cut at $C$ and $D$. One of those points, without loss of generality $C$, must be $f(A B)$, and so lie on the line $A B$. We now consider two cases, according to whether $D$ also lies on that line. + +## Case 1: D does not lie on that line. + +In this case, consider a sequence of $X_{\epsilon}$ at distance $\epsilon$ from $D$, tending to $D$ along some line that is not a tangent to either circle, but perturbed slightly (by at most $\epsilon^{2}$ ) to ensure that no three of the points $A, B$ and $X_{\epsilon}$ are collinear and no four are concyclic. + +Consider the points $f\left(X_{\epsilon} A\right)$ and $f\left(X_{\epsilon} B\right)$, and the circles $g\left(X_{\epsilon}\right)$ on which they lie. The point $f\left(X_{\epsilon} A\right)$ might be either $A$ or the other intersection of $X_{\epsilon} A$ with the circle $g(A)$, and the same applies for $B$. If, for some sequence of $\epsilon \rightarrow 0$, both those points are the other point of intersection, the same argument as in the proof of Lemma 4 applies to find arbitrarily small circles. Otherwise, we have either infinitely many of those circles passing through $A$, or infinitely many passing through $B$; without loss of generality, suppose infinitely many through $A$. + +We now show we can find five points $B_{i}$ satisfying the conditions of Lemma 4 (together with $A$ ). Let $B_{1}$ be any of the $X_{\epsilon}$ for which $g\left(X_{\epsilon}\right)$ passes through $A$. Then repeat the following four times, for $2 \leqslant i \leqslant 5$. + +Consider some line $\ell=X_{\epsilon} A$ (different from those considered for previous $i$ ) that is not tangent to any of the $g\left(B_{j}\right)$ for $j1.3 \cdot 10^{12}$. +For $n \geqslant 7$ we prove (3) by the following inequalities: + +$$ +\begin{aligned} +\left(\frac{n(n-1)}{2}\right)! & =15!\cdot 16 \cdot 17 \cdots \frac{n(n-1)}{2}>2^{36} \cdot 16^{\frac{n(n-1)}{2}-15} \\ +& =2^{2 n(n-1)-24}=2^{n^{2}} \cdot 2^{n(n-2)-24}>2^{n^{2}} . +\end{aligned} +$$ + +Putting together (2) and (3), for $n \geqslant 6$ we get a contradiction, since + +$$ +L_{n}<2^{n^{2}}<\left(\frac{n(n-1)}{2}\right)!a^{3} . +$$ + +So $3 a^{3} \geqslant(a b c)^{2}>a^{3}$ and hence $3 a \geqslant b^{2} c^{2}>a$. Now $b^{3}+c^{3}=a^{2}\left(b^{2} c^{2}-a\right) \geqslant a^{2}$, and so + +$$ +18 b^{3} \geqslant 9\left(b^{3}+c^{3}\right) \geqslant 9 a^{2} \geqslant b^{4} c^{4} \geqslant b^{3} c^{5} +$$ + +so $18 \geqslant c^{5}$ which yields $c=1$. +Now, note that we must have $a>b$, as otherwise we would have $2 b^{3}+1=b^{4}$ which has no positive integer solutions. So + +$$ +a^{3}-b^{3} \geqslant(b+1)^{3}-b^{3}>1 +$$ + +and + +$$ +2 a^{3}>1+a^{3}+b^{3}>a^{3} +$$ + +which implies $2 a^{3}>a^{2} b^{2}>a^{3}$ and so $2 a>b^{2}>a$. Therefore + +$$ +4\left(1+b^{3}\right)=4 a^{2}\left(b^{2}-a\right) \geqslant 4 a^{2}>b^{4} +$$ + +so $4>b^{3}(b-4)$; that is, $b \leqslant 4$. +Now, for each possible value of $b$ with $2 \leqslant b \leqslant 4$ we obtain a cubic equation for $a$ with constant coefficients. These are as follows: + +$$ +\begin{array}{rlrl} +b=2: & & & a^{3}-4 a^{2}+9=0 \\ +b=3: & & a^{3}-9 a^{2}+28=0 \\ +b=4: & & a^{3}-16 a^{2}+65=0 . +\end{array} +$$ + +The only case with an integer solution for $a$ with $b \leqslant a$ is $b=2$, leading to $(a, b, c)=(3,2,1)$. +Comment 1.1. Instead of writing down each cubic equation explicitly, we could have just observed that $a^{2} \mid b^{3}+1$, and for each choice of $b$ checked each square factor of $b^{3}+1$ for $a^{2}$. + +We could also have observed that, with $c=1$, the relation $18 b^{3} \geqslant b^{4} c^{4}$ becomes $b \leqslant 18$, and we can simply check all possibilities for $b$ (instead of working to prove that $b \leqslant 4$ ). This check becomes easier after using the factorisation $b^{3}+1=(b+1)\left(b^{2}-b+1\right)$ and observing that no prime besides 3 can divide both of the factors. + +Comment 1.2. Another approach to finish the problem after establishing that $c \leqslant 1$ is to set $k=b^{2} c^{2}-a$, which is clearly an integer and must be positive as it is equal to $\left(b^{3}+c^{3}\right) / a^{2}$. Then we divide into cases based on whether $k=1$ or $k \geqslant 2$; in the first case, we have $b^{3}+1=a^{2}=\left(b^{2}-1\right)^{2}$ whose only positive root is $b=2$, and in the second case we have $b^{2} \leqslant 3 a$, and so + +$$ +b^{4} \leqslant(3 a)^{2} \leqslant \frac{9}{2}\left(k a^{2}\right)=\frac{9}{2}\left(b^{3}+1\right), +$$ + +which implies that $b \leqslant 4$. + +Solution 2. Again, we will start by proving that $c=1$. Suppose otherwise that $c \geqslant 2$. We have $a^{3}+b^{3}+c^{3} \leqslant 3 a^{3}$, so $b^{2} c^{2} \leqslant 3 a$. Since $c \geqslant 2$, this tells us that $b \leqslant \sqrt{3 a / 4}$. As the right-hand side of the original equation is a multiple of $a^{2}$, we have $a^{2} \leqslant 2 b^{3} \leqslant 2(3 a / 4)^{3 / 2}$. In other words, $a \leqslant \frac{27}{16}<2$, which contradicts the assertion that $a \geqslant c \geqslant 2$. So there are no solutions in this case, and so we must have $c=1$. + +Now, the original equation becomes $a^{3}+b^{3}+1=a^{2} b^{2}$. Observe that $a \geqslant 2$, since otherwise $a=b=1$ as $a \geqslant b$. + +The right-hand side is a multiple of $a^{2}$, so the left-hand side must be as well. Thus, $b^{3}+1 \geqslant$ $a^{2}$. Since $a \geqslant b$, we also have + +$$ +b^{2}=a+\frac{b^{3}+1}{a^{2}} \leqslant 2 a+\frac{1}{a^{2}} +$$ + +and so $b^{2} \leqslant 2 a$ since $b^{2}$ is an integer. Thus $(2 a)^{3 / 2}+1 \geqslant b^{3}+1 \geqslant a^{2}$, from which we deduce $a \leqslant 8$. + +Now, for each possible value of $a$ with $2 \leqslant a \leqslant 8$ we obtain a cubic equation for $b$ with constant coefficients. These are as follows: + +$$ +\begin{array}{ll} +a=2: & \\ +b^{3}-4 b^{2}+9=0 \\ +a=3: & \\ +b^{3}-9 b^{2}+28=0 \\ +a=4: & \\ +a=5: 16 b^{2}+65=0 \\ +a=6: & \\ +b^{3}-25 b^{2}+126=0 \\ +a=7: & \\ +b^{3}-36 b^{2}+217=0 \\ +a=8: & \\ +b^{3}-49 b^{2}+344=0 \\ +b^{3}-64 b^{2}+513=0 . +\end{array} +$$ + +The only case with an integer solution for $b$ with $a \geqslant b$ is $a=3$, leading to $(a, b, c)=(3,2,1)$. +Comment 2.1. As in Solution 1, instead of writing down each cubic equation explicitly, we could have just observed that $b^{2} \mid a^{3}+1$, and for each choice of $a$ checked each square factor of $a^{3}+1$ for $b^{2}$. + +Comment 2.2. This solution does not require initially proving that $c=1$, in which case the bound would become $a \leqslant 108$. The resulting cases could, in principle, be checked by a particularly industrious student. + +Solution 3. Set $k=\left(b^{3}+c^{3}\right) / a^{2} \leqslant 2 a$, and rewrite the original equation as $a+k=(b c)^{2}$. Since $b^{3}$ and $c^{3}$ are positive integers, we have $(b c)^{3} \geqslant b^{3}+c^{3}-1=k a^{2}-1$, so + +$$ +a+k \geqslant\left(k a^{2}-1\right)^{2 / 3} +$$ + +As in Comment 1.2, $k$ is a positive integer; for each value of $k \geqslant 1$, this gives us a polynomial inequality satisfied by $a$ : + +$$ +k^{2} a^{4}-a^{3}-5 k a^{2}-3 k^{2} a-\left(k^{3}-1\right) \leqslant 0 . +$$ + +We now prove that $a \leqslant 3$. Indeed, + +$$ +0 \geqslant \frac{k^{2} a^{4}-a^{3}-5 k a^{2}-3 k^{2} a-\left(k^{3}-1\right)}{k^{2}} \geqslant a^{4}-a^{3}-5 a^{2}-3 a-k \geqslant a^{4}-a^{3}-5 a^{2}-5 a +$$ + +which fails when $a \geqslant 4$. +This leaves ten triples with $3 \geqslant a \geqslant b \geqslant c \geqslant 1$, which may be checked manually to give $(a, b, c)=(3,2,1)$. + +Solution 4. Again, observe that $b^{3}+c^{3}=a^{2}\left(b^{2} c^{2}-a\right)$, so $b \leqslant a \leqslant b^{2} c^{2}-1$. +We consider the function $f(x)=x^{2}\left(b^{2} c^{2}-x\right)$. It can be seen that that on the interval $\left[0, b^{2} c^{2}-1\right]$ the function $f$ is increasing if $x<\frac{2}{3} b^{2} c^{2}$ and decreasing if $x>\frac{2}{3} b^{2} c^{2}$. Consequently, it must be the case that + +$$ +b^{3}+c^{3}=f(a) \geqslant \min \left(f(b), f\left(b^{2} c^{2}-1\right)\right) +$$ + +First, suppose that $b^{3}+c^{3} \geqslant f\left(b^{2} c^{2}-1\right)$. This may be written $b^{3}+c^{3} \geqslant\left(b^{2} c^{2}-1\right)^{2}$, and so + +$$ +2 b^{3} \geqslant b^{3}+c^{3} \geqslant\left(b^{2} c^{2}-1\right)^{2}>b^{4} c^{4}-2 b^{2} c^{2} \geqslant b^{4} c^{4}-2 b^{3} c^{4} +$$ + +Thus, $(b-2) c^{4}<2$, and the only solutions to this inequality have $(b, c)=(2,2)$ or $b \leqslant 3$ and $c=1$. It is easy to verify that the only case giving a solution for $a \geqslant b$ is $(a, b, c)=(3,2,1)$. + +Otherwise, suppose that $b^{3}+c^{3}=f(a) \geqslant f(b)$. Then, we have + +$$ +2 b^{3} \geqslant b^{3}+c^{3}=a^{2}\left(b^{2} c^{2}-a\right) \geqslant b^{2}\left(b^{2} c^{2}-b\right) . +$$ + +Consequently $b c^{2} \leqslant 3$, with strict inequality in the case that $b \neq c$. Hence $c=1$ and $b \leqslant 2$. Both of these cases have been considered already, so we are done. + +Comment 4.1. Instead of considering which of $f(b)$ and $f\left(b^{2} c^{2}-1\right)$ is less than $f(a)$, we may also proceed by explicitly dividing into cases based on whether $a \geqslant \frac{2}{3} b^{2} c^{2}$ or $a<\frac{2}{3} b^{2} c^{2}$. The first case may now be dealt with as follows. We have $b^{3} c^{3}+1 \geqslant b^{3}+c^{3}$ as $b^{3}$ and $c^{3}$ are positive integers, so we have + +$$ +b^{3} c^{3}+1 \geqslant b^{3}+c^{3} \geqslant a^{2} \geqslant \frac{4}{9} b^{4} c^{4} +$$ + +This implies $b c \leqslant 2$, and hence $c=1$ and $b \leqslant 2$. + +N3. We say that a set $S$ of integers is rootiful if, for any positive integer $n$ and any $a_{0}, a_{1}, \ldots, a_{n} \in S$, all integer roots of the polynomial $a_{0}+a_{1} x+\cdots+a_{n} x^{n}$ are also in $S$. Find all rootiful sets of integers that contain all numbers of the form $2^{a}-2^{b}$ for positive integers $a$ and $b$. +(Czech Republic) +Answer: The set $\mathbb{Z}$ of all integers is the only such rootiful set. +Solution 1. The set $\mathbb{Z}$ of all integers is clearly rootiful. We shall prove that any rootiful set $S$ containing all the numbers of the form $2^{a}-2^{b}$ for $a, b \in \mathbb{Z}_{>0}$ must be all of $\mathbb{Z}$. + +First, note that $0=2^{1}-2^{1} \in S$ and $2=2^{2}-2^{1} \in S$. Now, $-1 \in S$, since it is a root of $2 x+2$, and $1 \in S$, since it is a root of $2 x^{2}-x-1$. Also, if $n \in S$ then $-n$ is a root of $x+n$, so it suffices to prove that all positive integers must be in $S$. + +Now, we claim that any positive integer $n$ has a multiple in $S$. Indeed, suppose that $n=2^{\alpha} \cdot t$ for $\alpha \in \mathbb{Z}_{\geqslant 0}$ and $t$ odd. Then $t \mid 2^{\phi(t)}-1$, so $n \mid 2^{\alpha+\phi(t)+1}-2^{\alpha+1}$. Moreover, $2^{\alpha+\phi(t)+1}-2^{\alpha+1} \in S$, and so $S$ contains a multiple of every positive integer $n$. + +We will now prove by induction that all positive integers are in $S$. Suppose that $0,1, \ldots, n-$ $1 \in S$; furthermore, let $N$ be a multiple of $n$ in $S$. Consider the base- $n$ expansion of $N$, say $N=a_{k} n^{k}+a_{k-1} n^{k-1}+\cdots+a_{1} n+a_{0}$. Since $0 \leqslant a_{i}0}$. + +We show that, in fact, every integer $k$ with $|k|>2$ can be expressed as a root of a polynomial whose coefficients are of the form $2^{a}-2^{b}$. Observe that it suffices to consider the case where $k$ is positive, as if $k$ is a root of $a_{n} x^{n}+\cdots+a_{1} x+a_{0}=0$, then $-k$ is a root of $(-1)^{n} a_{n} x^{n}+\cdots-$ $a_{1} x+a_{0}=0$. + +Note that + +$$ +\left(2^{a_{n}}-2^{b_{n}}\right) k^{n}+\cdots+\left(2^{a_{0}}-2^{b_{0}}\right)=0 +$$ + +is equivalent to + +$$ +2^{a_{n}} k^{n}+\cdots+2^{a_{0}}=2^{b_{n}} k^{n}+\cdots+2^{b_{0}} +$$ + +Hence our aim is to show that two numbers of the form $2^{a_{n}} k^{n}+\cdots+2^{a_{0}}$ are equal, for a fixed value of $n$. We consider such polynomials where every term $2^{a_{i}} k^{i}$ is at most $2 k^{n}$; in other words, where $2 \leqslant 2^{a_{i}} \leqslant 2 k^{n-i}$, or, equivalently, $1 \leqslant a_{i} \leqslant 1+(n-i) \log _{2} k$. Therefore, there must be $1+\left\lfloor(n-i) \log _{2} k\right\rfloor$ possible choices for $a_{i}$ satisfying these constraints. + +The number of possible polynomials is then + +$$ +\prod_{i=0}^{n}\left(1+\left\lfloor(n-i) \log _{2} k\right\rfloor\right) \geqslant \prod_{i=0}^{n-1}(n-i) \log _{2} k=n!\left(\log _{2} k\right)^{n} +$$ + +where the inequality holds as $1+\lfloor x\rfloor \geqslant x$. +As there are $(n+1)$ such terms in the polynomial, each at most $2 k^{n}$, such a polynomial must have value at most $2 k^{n}(n+1)$. However, for large $n$, we have $n!\left(\log _{2} k\right)^{n}>2 k^{n}(n+1)$. Therefore there are more polynomials than possible values, so some two must be equal, as required. + +N4. Let $\mathbb{Z}_{>0}$ be the set of positive integers. A positive integer constant $C$ is given. Find all functions $f: \mathbb{Z}_{>0} \rightarrow \mathbb{Z}_{>0}$ such that, for all positive integers $a$ and $b$ satisfying $a+b>C$, + +$$ +a+f(b) \mid a^{2}+b f(a) +$$ + +(Croatia) +Answer: The functions satisfying (*) are exactly the functions $f(a)=k a$ for some constant $k \in \mathbb{Z}_{>0}$ (irrespective of the value of $C$ ). + +Common remarks. It is easy to verify that the functions $f(a)=k a$ satisfy (*). Thus, in the proofs below, we will only focus on the converse implication: that condition (*) implies that $f=k a$. + +A common minor part of these solutions is the derivation of some relatively easy bounds on the function $f$. An upper bound is easily obtained by setting $a=1$ in (*), giving the inequality + +$$ +f(b) \leqslant b \cdot f(1) +$$ + +for all sufficiently large $b$. The corresponding lower bound is only marginally more difficult to obtain: substituting $b=1$ in the original equation shows that + +$$ +a+f(1) \mid\left(a^{2}+f(a)\right)-(a-f(1)) \cdot(a+f(1))=f(1)^{2}+f(a) +$$ + +for all sufficiently large $a$. It follows from this that one has the lower bound + +$$ +f(a) \geqslant a+f(1) \cdot(1-f(1)) +$$ + +again for all sufficiently large $a$. +Each of the following proofs makes use of at least one of these bounds. +Solution 1. First, we show that $b \mid f(b)^{2}$ for all $b$. To do this, we choose a large positive integer $n$ so that $n b-f(b) \geqslant C$. Setting $a=n b-f(b)$ in (*) then shows that + +$$ +n b \mid(n b-f(b))^{2}+b f(n b-f(b)) +$$ + +so that $b \mid f(b)^{2}$ as claimed. +Now in particular we have that $p \mid f(p)$ for every prime $p$. If we write $f(p)=k(p) \cdot p$, then the bound $f(p) \leqslant f(1) \cdot p$ (valid for $p$ sufficiently large) shows that some value $k$ of $k(p)$ must be attained for infinitely many $p$. We will show that $f(a)=k a$ for all positive integers $a$. To do this, we substitute $b=p$ in (*), where $p$ is any sufficiently large prime for which $k(p)=k$, obtaining + +$$ +a+k p \mid\left(a^{2}+p f(a)\right)-a(a+k p)=p f(a)-p k a . +$$ + +For suitably large $p$ we have $\operatorname{gcd}(a+k p, p)=1$, and hence we have + +$$ +a+k p \mid f(a)-k a +$$ + +But the only way this can hold for arbitrarily large $p$ is if $f(a)-k a=0$. This concludes the proof. + +Comment. There are other ways to obtain the divisibility $p \mid f(p)$ for primes $p$, which is all that is needed in this proof. For instance, if $f(p)$ were not divisible by $p$ then the arithmetic progression $p^{2}+b f(p)$ would attain prime values for infinitely many $b$ by Dirichlet's Theorem: hence, for these pairs p , b, we would have $p+f(b)=p^{2}+b f(p)$. Substituting $a \mapsto b$ and $b \mapsto p$ in $(*)$ then shows that $\left(f(p)^{2}-p^{2}\right)(p-1)$ is divisible by $b+f(p)$ and hence vanishes, which is impossible since $p \nmid f(p)$ by assumption. + +Solution 2. First, we substitute $b=1$ in (*) and rearrange to find that + +$$ +\frac{f(a)+f(1)^{2}}{a+f(1)}=f(1)-a+\frac{a^{2}+f(a)}{a+f(1)} +$$ + +is a positive integer for sufficiently large $a$. Since $f(a) \leqslant a f(1)$, for all sufficiently large $a$, it follows that $\frac{f(a)+f(1)^{2}}{a+f(1)} \leqslant f(1)$ also and hence there is a positive integer $k$ such that $\frac{f(a)+f(1)^{2}}{a+f(1)}=k$ for infinitely many values of $a$. In other words, + +$$ +f(a)=k a+f(1) \cdot(k-f(1)) +$$ + +for infinitely many $a$. +Fixing an arbitrary choice of $a$ in (*), we have that + +$$ +\frac{a^{2}+b f(a)}{a+k b+f(1) \cdot(k-f(1))} +$$ + +is an integer for infinitely many $b$ (the same $b$ as above, maybe with finitely many exceptions). On the other hand, for $b$ taken sufficiently large, this quantity becomes arbitrarily close to $\frac{f(a)}{k}$; this is only possible if $\frac{f(a)}{k}$ is an integer and + +$$ +\frac{a^{2}+b f(a)}{a+k b+f(1) \cdot(k-f(1))}=\frac{f(a)}{k} +$$ + +for infinitely many $b$. This rearranges to + +$$ +\frac{f(a)}{k} \cdot(a+f(1) \cdot(k-f(1)))=a^{2} +$$ + +Hence $a^{2}$ is divisible by $a+f(1) \cdot(k-f(1))$, and hence so is $f(1)^{2}(k-f(1))^{2}$. The only way this can occur for all $a$ is if $k=f(1)$, in which case $(* *)$ provides that $f(a)=k a$ for all $a$, as desired. + +Solution 3. Fix any two distinct positive integers $a$ and $b$. From (*) it follows that the two integers + +$$ +\left(a^{2}+c f(a)\right) \cdot(b+f(c)) \text { and }\left(b^{2}+c f(b)\right) \cdot(a+f(c)) +$$ + +are both multiples of $(a+f(c)) \cdot(b+f(c))$ for all sufficiently large $c$. Taking an appropriate linear combination to eliminate the $c f(c)$ term, we find after expanding out that the integer + +$$ +\left[a^{2} f(b)-b^{2} f(a)\right] \cdot f(c)+[(b-a) f(a) f(b)] \cdot c+[a b(a f(b)-b f(a))] +$$ + +is also a multiple of $(a+f(c)) \cdot(b+f(c))$. +But as $c$ varies, $(\dagger)$ is bounded above by a positive multiple of $c$ while $(a+f(c)) \cdot(b+f(c))$ is bounded below by a positive multiple of $c^{2}$. The only way that such a divisibility can hold is if in fact + +$$ +\left[a^{2} f(b)-b^{2} f(a)\right] \cdot f(c)+[(b-a) f(a) f(b)] \cdot c+[a b(a f(b)-b f(a))]=0 +$$ + +for sufficiently large $c$. Since the coefficient of $c$ in this linear relation is nonzero, it follows that there are constants $k, \ell$ such that $f(c)=k c+\ell$ for all sufficiently large $c$; the constants $k$ and $\ell$ are necessarily integers. + +The value of $\ell$ satisfies + +$$ +\left[a^{2} f(b)-b^{2} f(a)\right] \cdot \ell+[a b(a f(b)-b f(a))]=0 +$$ + +and hence $b \mid \ell a^{2} f(b)$ for all $a$ and $b$. Taking $b$ sufficiently large so that $f(b)=k b+\ell$, we thus have that $b \mid \ell^{2} a^{2}$ for all sufficiently large $b$; this implies that $\ell=0$. From ( $\dagger \dagger \dagger$ ) it then follows that $\frac{f(a)}{a}=\frac{f(b)}{b}$ for all $a \neq b$, so that there is a constant $k$ such that $f(a)=k a$ for all $a$ ( $k$ is equal to the constant defined earlier). + +Solution 4. Let $\Gamma$ denote the set of all points $(a, f(a))$, so that $\Gamma$ is an infinite subset of the upper-right quadrant of the plane. For a point $A=(a, f(a))$ in $\Gamma$, we define a point $A^{\prime}=\left(-f(a),-f(a)^{2} / a\right)$ in the lower-left quadrant of the plane, and let $\Gamma^{\prime}$ denote the set of all such points $A^{\prime}$. +![](https://cdn.mathpix.com/cropped/2024_11_18_9ee8c2ac2c2f5002ff14g-095.jpg?height=552&width=652&top_left_y=449&top_left_x=702) + +Claim. For any point $A \in \Gamma$, the set $\Gamma$ is contained in finitely many lines through the point $A^{\prime}$. Proof. Let $A=(a, f(a))$. The functional equation (with $a$ and $b$ interchanged) can be rewritten as $b+f(a) \mid a f(b)-b f(a)$, so that all but finitely many points in $\Gamma$ are contained in one of the lines with equation + +$$ +a y-f(a) x=m(x+f(a)) +$$ + +for $m$ an integer. Geometrically, these are the lines through $A^{\prime}=\left(-f(a),-f(a)^{2} / a\right)$ with gradient $\frac{f(a)+m}{a}$. Since $\Gamma$ is contained, with finitely many exceptions, in the region $0 \leqslant y \leqslant$ $f(1) \cdot x$ and the point $A^{\prime}$ lies strictly in the lower-left quadrant of the plane, there are only finitely many values of $m$ for which this line meets $\Gamma$. This concludes the proof of the claim. + +Now consider any distinct points $A, B \in \Gamma$. It is clear that $A^{\prime}$ and $B^{\prime}$ are distinct. A line through $A^{\prime}$ and a line through $B^{\prime}$ only meet in more than one point if these two lines are equal to the line $A^{\prime} B^{\prime}$. It then follows from the above claim that the line $A^{\prime} B^{\prime}$ must contain all but finitely many points of $\Gamma$. If $C$ is another point of $\Gamma$, then the line $A^{\prime} C^{\prime}$ also passes through all but finitely many points of $\Gamma$, which is only possible if $A^{\prime} C^{\prime}=A^{\prime} B^{\prime}$. + +We have thus seen that there is a line $\ell$ passing through all points of $\Gamma^{\prime}$ and through all but finitely many points of $\Gamma$. We claim that this line passes through the origin $O$ and passes through every point of $\Gamma$. To see this, note that by construction $A, O, A^{\prime}$ are collinear for every point $A \in \Gamma$. Since $\ell=A A^{\prime}$ for all but finitely many points $A \in \Gamma$, it thus follows that $O \in \ell$. Thus any $A \in \Gamma$ lies on the line $\ell=A^{\prime} O$. + +Since $\Gamma$ is contained in a line through $O$, it follows that there is a real constant $k$ (the gradient of $\ell$ ) such that $f(a)=k a$ for all $a$. The number $k$ is, of course, a positive integer. + +Comment. Without the $a+b>C$ condition, this problem is approachable by much more naive methods. For instance, using the given divisibility for $a, b \in\{1,2,3\}$ one can prove by a somewhat tedious case-check that $f(2)=2 f(1)$ and $f(3)=3 f(1)$; this then forms the basis of an induction establishing that $f(n)=n f(1)$ for all $n$. + +N5. Let $a$ be a positive integer. We say that a positive integer $b$ is $a$-good if $\binom{a n}{b}-1$ is divisible by $a n+1$ for all positive integers $n$ with $a n \geqslant b$. Suppose $b$ is a positive integer such that $b$ is $a$-good, but $b+2$ is not $a$-good. Prove that $b+1$ is prime. +(Netherlands) +Solution 1. For $p$ a prime and $n$ a nonzero integer, we write $v_{p}(n)$ for the $p$-adic valuation of $n$ : the largest integer $t$ such that $p^{t} \mid n$. + +We first show that $b$ is $a$-good if and only if $b$ is even, and $p \mid a$ for all primes $p \leqslant b$. +To start with, the condition that $a n+1 \left\lvert\,\binom{ a n}{b}-1\right.$ can be rewritten as saying that + +$$ +\frac{a n(a n-1) \cdots(a n-b+1)}{b!} \equiv 1 \quad(\bmod a n+1) +$$ + +Suppose, on the one hand, there is a prime $p \leqslant b$ with $p \nmid a$. Take $t=v_{p}(b!)$. Then there exist positive integers $c$ such that $a c \equiv 1\left(\bmod p^{t+1}\right)$. If we take $c$ big enough, and then take $n=(p-1) c$, then $a n=a(p-1) c \equiv p-1\left(\bmod p^{t+1}\right)$ and $a n \geqslant b$. Since $p \leqslant b$, one of the terms of the numerator an $(a n-1) \cdots(a n-b+1)$ is $a n-p+1$, which is divisible by $p^{t+1}$. Hence the $p$-adic valuation of the numerator is at least $t+1$, but that of the denominator is exactly $t$. This means that $p \left\lvert\,\binom{ a n}{b}\right.$, so $p \nmid\binom{a n}{b}-1$. As $p \mid a n+1$, we get that $a n+1 \nmid\binom{a n}{b}$, so $b$ is not $a$-good. + +On the other hand, if for all primes $p \leqslant b$ we have $p \mid a$, then every factor of $b$ ! is coprime to $a n+1$, and hence invertible modulo $a n+1$ : hence $b$ ! is also invertible modulo $a n+1$. Then equation (1) reduces to: + +$$ +a n(a n-1) \cdots(a n-b+1) \equiv b!\quad(\bmod a n+1) +$$ + +However, we can rewrite the left-hand side as follows: + +$$ +a n(a n-1) \cdots(a n-b+1) \equiv(-1)(-2) \cdots(-b) \equiv(-1)^{b} b!\quad(\bmod a n+1) +$$ + +Provided that $a n>1$, if $b$ is even we deduce $(-1)^{b} b!\equiv b$ ! as needed. On the other hand, if $b$ is odd, and we take $a n+1>2(b!)$, then we will not have $(-1)^{b} b!\equiv b$ !, so $b$ is not $a$-good. This completes the claim. + +To conclude from here, suppose that $b$ is $a$-good, but $b+2$ is not. Then $b$ is even, and $p \mid a$ for all primes $p \leqslant b$, but there is a prime $q \leqslant b+2$ for which $q \nmid a$ : so $q=b+1$ or $q=b+2$. We cannot have $q=b+2$, as that is even too, so we have $q=b+1$ : in other words, $b+1$ is prime. + +Solution 2. We show only half of the claim of the previous solution: we show that if $b$ is $a$-good, then $p \mid a$ for all primes $p \leqslant b$. We do this with Lucas' theorem. + +Suppose that we have $p \leqslant b$ with $p \nmid a$. Then consider the expansion of $b$ in base $p$; there will be some digit (not the final digit) which is nonzero, because $p \leqslant b$. Suppose it is the $p^{t}$ digit for $t \geqslant 1$. + +Now, as $n$ varies over the integers, an +1 runs over all residue classes modulo $p^{t+1}$; in particular, there is a choice of $n$ (with $a n>b$ ) such that the $p^{0}$ digit of $a n$ is $p-1$ (so $p \mid a n+1$ ) and the $p^{t}$ digit of $a n$ is 0 . Consequently, $p \mid a n+1$ but $p \left\lvert\,\binom{ a n}{b}\right.$ (by Lucas' theorem) so $p \nmid\binom{a n}{b}-1$. Thus $b$ is not $a$-good. + +Now we show directly that if $b$ is $a$-good but $b+2$ fails to be so, then there must be a prime dividing $a n+1$ for some $n$, which also divides $(b+1)(b+2)$. Indeed, the ratio between $\binom{a n}{b+2}$ and $\binom{a n}{b}$ is $(b+1)(b+2) /(a n-b)(a n-b-1)$. We know that there must be a choice of $a n+1$ such that the former binomial coefficient is 1 modulo $a n+1$ but the latter is not, which means that the given ratio must not be $1 \bmod a n+1$. If $b+1$ and $b+2$ are both coprime to $a n+1$ then +the ratio is 1 , so that must not be the case. In particular, as any prime less than $b$ divides $a$, it must be the case that either $b+1$ or $b+2$ is prime. + +However, we can observe that $b$ must be even by insisting that $a n+1$ is prime (which is possible by Dirichlet's theorem) and hence $\binom{a n}{b} \equiv(-1)^{b}=1$. Thus $b+2$ cannot be prime, so $b+1$ must be prime. + +N6. Let $H=\left\{\lfloor i \sqrt{2}\rfloor: i \in \mathbb{Z}_{>0}\right\}=\{1,2,4,5,7, \ldots\}$, and let $n$ be a positive integer. Prove that there exists a constant $C$ such that, if $A \subset\{1,2, \ldots, n\}$ satisfies $|A| \geqslant C \sqrt{n}$, then there exist $a, b \in A$ such that $a-b \in H$. (Here $\mathbb{Z}_{>0}$ is the set of positive integers, and $\lfloor z\rfloor$ denotes the greatest integer less than or equal to $z$.) +(Brazil) +Common remarks. In all solutions, we will assume that $A$ is a set such that $\{a-b: a, b \in A\}$ is disjoint from $H$, and prove that $|A|1-\frac{1}{\sqrt{2}} . +$$ + +To see why, observe that $n \in H$ if and only if $00}$. In other words, $01$ since $a_{i}-a_{1} \notin H$. Furthermore, we must have $\left\{a_{i} / \sqrt{2}\right\}<\left\{a_{j} / \sqrt{2}\right\}$ whenever $i1 / \sqrt{2}>$ $1-1 / \sqrt{2}$, contradicting (1). + +Now, we have a sequence $0=a_{1}\frac{1}{2 d \sqrt{2}} +$$ + +To see why this is the case, let $h=\lfloor d / \sqrt{2}\rfloor$, so $\{d / \sqrt{2}\}=d / \sqrt{2}-h$. Then + +$$ +\left\{\frac{d}{\sqrt{2}}\right\}\left(\frac{d}{\sqrt{2}}+h\right)=\frac{d^{2}-2 h^{2}}{2} \geqslant \frac{1}{2} +$$ + +since the numerator is a positive integer. Because $d / \sqrt{2}+h<2 d / \sqrt{2}$, inequality (2) follows. +Let $d_{i}=a_{i+1}-a_{i}$, for $1 \leqslant i\sum_{i}\left\{\frac{d_{i}}{\sqrt{2}}\right\}>\frac{1}{2 \sqrt{2}} \sum_{i} \frac{1}{d_{i}} \geqslant \frac{(k-1)^{2}}{2 \sqrt{2}} \frac{1}{\sum_{i} d_{i}}>\frac{(k-1)^{2}}{2 \sqrt{2}} \cdot \frac{1}{n} . +$$ + +Here, the first inequality holds because $\left\{a_{k} / \sqrt{2}\right\}<1-1 / \sqrt{2}$, the second follows from (2), the third follows from an easy application of the AM-HM inequality (or Cauchy-Schwarz), and the fourth follows from the fact that $\sum_{i} d_{i}=a_{k}k-1 +$$ + +which provides the required bound on $k$. + +Solution 2. Let $\alpha=2+\sqrt{2}$, so $(1 / \alpha)+(1 / \sqrt{2})=1$. Thus, $J=\left\{\lfloor i \alpha\rfloor: i \in \mathbb{Z}_{>0}\right\}$ is the complementary Beatty sequence to $H$ (in other words, $H$ and $J$ are disjoint with $H \cup J=\mathbb{Z}_{>0}$ ). Write $A=\left\{a_{1}0}$. + +For any $j>i$, we have $a_{j}-a_{i}=\left\lfloor\alpha b_{j}\right\rfloor-\left\lfloor\alpha b_{i}\right\rfloor$. Because $a_{j}-a_{i} \in J$, we also have $a_{j}-a_{i}=\lfloor\alpha t\rfloor$ for some positive integer $t$. Thus, $\lfloor\alpha t\rfloor=\left\lfloor\alpha b_{j}\right\rfloor-\left\lfloor\alpha b_{i}\right\rfloor$. The right hand side must equal either $\left\lfloor\alpha\left(b_{j}-b_{i}\right)\right\rfloor$ or $\left\lfloor\alpha\left(b_{j}-b_{i}\right)\right\rfloor-1$, the latter of which is not a member of $J$ as $\alpha>2$. Therefore, $t=b_{j}-b_{i}$ and so we have $\left\lfloor\alpha b_{j}\right\rfloor-\left\lfloor\alpha b_{i}\right\rfloor=\left\lfloor\alpha\left(b_{j}-b_{i}\right)\right\rfloor$. + +For $1 \leqslant i1 /(2 d \sqrt{2})$ for positive integers $d)$ proves that $1>\left((k-1)^{2} /(2 \sqrt{2})\right)(\alpha / n)$, which again rearranges to give + +$$ +\sqrt{2 \sqrt{2}-2} \cdot \sqrt{n}>k-1 +$$ + +Comment. The use of Beatty sequences in Solution 2 is essentially a way to bypass (1). Both Solutions 1 and 2 use the fact that $\sqrt{2}<2$; the statement in the question would still be true if $\sqrt{2}$ did not have this property (for instance, if it were replaced with $\alpha$ ), but any argument along the lines of Solutions 1 or 2 would be more complicated. + +Solution 3. Again, define $J=\mathbb{Z}_{>0} \backslash H$, so all differences between elements of $A$ are in $J$. We start by making the following observation. Suppose we have a set $B \subseteq\{1,2, \ldots, n\}$ such that all of the differences between elements of $B$ are in $H$. Then $|A| \cdot|B| \leqslant 2 n$. + +To see why, observe that any two sums of the form $a+b$ with $a \in A, b \in B$ are different; otherwise, we would have $a_{1}+b_{1}=a_{2}+b_{2}$, and so $\left|a_{1}-a_{2}\right|=\left|b_{2}-b_{1}\right|$. However, then the left hand side is in $J$ whereas the right hand side is in $H$. Thus, $\{a+b: a \in A, b \in B\}$ is a set of size $|A| \cdot|B|$ all of whose elements are no greater than $2 n$, yielding the claimed inequality. + +With this in mind, it suffices to construct a set $B$, all of whose differences are in $H$ and whose size is at least $C^{\prime} \sqrt{n}$ for some constant $C^{\prime}>0$. + +To do so, we will use well-known facts about the negative Pell equation $X^{2}-2 Y^{2}=-1$; in particular, that there are infinitely many solutions and the values of $X$ are given by the recurrence $X_{1}=1, X_{2}=7$ and $X_{m}=6 X_{m-1}-X_{m-2}$. Therefore, we may choose $X$ to be a solution with $\sqrt{n} / 6\left(\frac{X}{\sqrt{2}}-Y\right) \geqslant \frac{-3}{\sqrt{2 n}}, +$$ + +from which it follows that $\{X / \sqrt{2}\}>1-(3 / \sqrt{2 n})$. Combined with (1), this shows that all differences between elements of $B$ are in $H$. + +Comment. Some of the ideas behind Solution 3 may be used to prove that the constant $C=\sqrt{2 \sqrt{2}-2}$ (from Solutions 1 and 2) is optimal, in the sense that there are arbitrarily large values of $n$ and sets $A_{n} \subseteq\{1,2, \ldots, n\}$ of size roughly $C \sqrt{n}$, all of whose differences are contained in $J$. + +To see why, choose $X$ to come from a sufficiently large solution to the Pell equation $X^{2}-2 Y^{2}=1$, so $\{X / \sqrt{2}\} \approx 1 /(2 X \sqrt{2})$. In particular, $\{X\},\{2 X\}, \ldots,\{[2 X \sqrt{2}(1-1 / \sqrt{2})\rfloor X\}$ are all less than $1-1 / \sqrt{2}$. Thus, by (1) any positive integer of the form $i X$ for $1 \leqslant i \leqslant\lfloor 2 X \sqrt{2}(1-1 / \sqrt{2})\rfloor$ lies in $J$. + +Set $n \approx 2 X^{2} \sqrt{2}(1-1 / \sqrt{2})$. We now have a set $A=\{i X: i \leqslant\lfloor 2 X \sqrt{2}(1-1 / \sqrt{2})\rfloor\}$ containing roughly $2 X \sqrt{2}(1-1 / \sqrt{2})$ elements less than or equal to $n$ such that all of the differences lie in $J$, and we can see that $|A| \approx C \sqrt{n}$ with $C=\sqrt{2 \sqrt{2}-2}$. + +Solution 4. As in Solution 3, we will provide a construction of a large set $B \subseteq\{1,2, \ldots, n\}$, all of whose differences are in $H$. + +Choose $Y$ to be a solution to the Pell-like equation $X^{2}-2 Y^{2}= \pm 1$; such solutions are given by the recurrence $Y_{1}=1, Y_{2}=2$ and $Y_{m}=2 Y_{m-1}+Y_{m-2}$, and so we can choose $Y$ such that $n /(3 \sqrt{2})\sqrt{Y}$ elements; if that subsequence is decreasing, we may reflect (using (4) or (5)) to ensure that it is increasing. Call the subsequence $\left\{y_{1} \sqrt{2}\right\},\left\{y_{2} \sqrt{2}\right\}, \ldots,\left\{y_{t} \sqrt{2}\right\}$ for $t>\sqrt{Y}$. + +Now, set $B=\left\{\left\lfloor y_{i} \sqrt{2}\right\rfloor: 1 \leqslant i \leqslant t\right\}$. We have $\left\lfloor y_{j} \sqrt{2}\right\rfloor-\left\lfloor y_{i} \sqrt{2}\right\rfloor=\left\lfloor\left(y_{j}-y_{i}\right) \sqrt{2}\right\rfloor$ for $i\sqrt{Y}>\sqrt{n} / \sqrt{3 \sqrt{2}}$, this is the required set. + +Comment. Any solution to this problem will need to use the fact that $\sqrt{2}$ cannot be approximated well by rationals, either directly or implicitly (for example, by using facts about solutions to Pelllike equations). If $\sqrt{2}$ were replaced by a value of $\theta$ with very good rational approximations (from below), then an argument along the lines of Solution 3 would give long arithmetic progressions in $\{\lfloor i \theta\rfloor: 0 \leqslant i0$ and infinitely many positive integers $n$ with the following property: there are infinitely many positive integers that cannot be expressed as the sum of fewer than $c n \log (n)$ pairwise coprime $n^{\text {th }}$ powers. +(Canada) +Solution 1. Suppose, for an integer $n$, that we can find another integer $N$ satisfying the following property: +$n$ is divisible by $\varphi\left(p^{e}\right)$ for every prime power $p^{e}$ exactly dividing $N$. +This property ensures that all $n^{\text {th }}$ powers are congruent to 0 or 1 modulo each such prime power $p^{e}$, and hence that any sum of $m$ pairwise coprime $n^{\text {th }}$ powers is congruent to $m$ or $m-1$ modulo $p^{e}$, since at most one of the $n^{\text {th }}$ powers is divisible by $p$. Thus, if $k$ denotes the number of distinct prime factors of $N$, we find by the Chinese Remainder Theorem at most $2^{k} m$ residue classes modulo $N$ which are sums of at most $m$ pairwise coprime $n^{\text {th }}$ powers. In particular, if $N>2^{k} m$ then there are infinitely many positive integers not expressible as a sum of at most $m$ pairwise coprime $n^{\text {th }}$ powers. + +It thus suffices to prove that there are arbitrarily large pairs $(n, N)$ of integers satisfying $(\dagger)$ such that + +$$ +N>c \cdot 2^{k} n \log (n) +$$ + +for some positive constant $c$. +We construct such pairs as follows. Fix a positive integer $t$ and choose (distinct) prime numbers $p \mid 2^{2^{t-1}}+1$ and $q \mid 2^{2^{t}}+1$; we set $N=p q$. It is well-known that $2^{t} \mid p-1$ and $2^{t+1} \mid q-1$, hence + +$$ +n=\frac{(p-1)(q-1)}{2^{t}} +$$ + +is an integer and the pair $(n, N)$ satisfies $(\dagger)$. +Estimating the size of $N$ and $n$ is now straightforward. We have + +$$ +\log _{2}(n) \leqslant 2^{t-1}+2^{t}-t<2^{t+1}<2 \cdot \frac{N}{n} +$$ + +which rearranges to + +$$ +N>\frac{1}{8} \cdot 2^{2} n \log _{2}(n) +$$ + +and so we are done if we choose $c<\frac{1}{8 \log (2)} \approx 0.18$. +Comment 1. The trick in the above solution was to find prime numbers $p$ and $q$ congruent to 1 modulo some $d=2^{t}$ and which are not too large. An alternative way to do this is via Linnik's Theorem, which says that there are absolute constants $b$ and $L>1$ such that for any coprime integers $a$ and $d$, there is a prime congruent to $a$ modulo $d$ and of size $\leqslant b d^{L}$. If we choose some $d$ not divisible by 3 and choose two distinct primes $p, q \leqslant b \cdot(3 d)^{L}$ congruent to 1 modulo $d$ (and, say, distinct modulo 3 ), then we obtain a pair $(n, N)$ satisfying $(\dagger)$ with $N=p q$ and $n=\frac{(p-1)(q-1)}{d}$. A straightforward computation shows that + +$$ +N>C n^{1+\frac{1}{2 L-1}} +$$ + +for some constant $C$, which is in particular larger than any $c \cdot 2^{2} n \log (n)$ for $p$ large. Thus, the statement of the problem is true for any constant $c$. More strongly, the statement of the problem is still true when $c n \log (n)$ is replaced by $n^{1+\delta}$ for a sufficiently small $\delta>0$. + +Solution 2, obtaining better bounds. As in the preceding solution, we seek arbitrarily large pairs of integers $n$ and $N$ satisfying ( $\dagger$ ) such that $N>c 2^{k} n \log (n)$. + +This time, to construct such pairs, we fix an integer $x \geqslant 4$, set $N$ to be the lowest common multiple of $1,2, \ldots, 2 x$, and set $n$ to be twice the lowest common multiple of $1,2, \ldots, x$. The pair $(n, N)$ does indeed satisfy the condition, since if $p^{e}$ is a prime power divisor of $N$ then $\frac{\varphi\left(p^{e}\right)}{2} \leqslant x$ is a factor of $\frac{n}{2}=\operatorname{lcm}_{r \leqslant x}(r)$. + +Now $2 N / n$ is the product of all primes having a power lying in the interval $(x, 2 x]$, and hence $2 N / n>x^{\pi(2 x)-\pi(x)}$. Thus for sufficiently large $x$ we have + +$$ +\log \left(\frac{2 N}{2^{\pi(2 x)} n}\right)>(\pi(2 x)-\pi(x)) \log (x)-\log (2) \pi(2 x) \sim x +$$ + +using the Prime Number Theorem $\pi(t) \sim t / \log (t)$. +On the other hand, $n$ is a product of at most $\pi(x)$ prime powers less than or equal to $x$, and so we have the upper bound + +$$ +\log (n) \leqslant \pi(x) \log (x) \sim x +$$ + +again by the Prime Number Theorem. Combined with the above inequality, we find that for any $\epsilon>0$, the inequality + +$$ +\log \left(\frac{N}{2^{\pi(2 x)} n}\right)>(1-\epsilon) \log (n) +$$ + +holds for sufficiently large $x$. Rearranging this shows that + +$$ +N>2^{\pi(2 x)} n^{2-\epsilon}>2^{\pi(2 x)} n \log (n) +$$ + +for all sufficiently large $x$ and we are done. +Comment 2. The stronger bound $N>2^{\pi(2 x)} n^{2-\epsilon}$ obtained in the above proof of course shows that infinitely many positive integers cannot be written as a sum of at most $n^{2-\epsilon}$ pairwise coprime $n^{\text {th }}$ powers. + +By refining the method in Solution 2, these bounds can be improved further to show that infinitely many positive integers cannot be written as a sum of at most $n^{\alpha}$ pairwise coprime $n^{\text {th }}$ powers for any positive $\alpha>0$. To do this, one fixes a positive integer $d$, sets $N$ equal to the product of the primes at most $d x$ which are congruent to 1 modulo $d$, and $n=d \mathrm{lcm}_{r \leqslant x}(r)$. It follows as in Solution 2 that $(n, N)$ satisfies $(\dagger)$. + +Now the Prime Number Theorem in arithmetic progressions provides the estimates $\log (N) \sim \frac{d}{\varphi(d)} x$, $\log (n) \sim x$ and $\pi(d x) \sim \frac{d x}{\log (x)}$ for any fixed $d$. Combining these provides a bound + +$$ +N>2^{\pi(d x)} n^{d / \varphi(d)-\epsilon} +$$ + +for any positive $\epsilon$, valid for $x$ sufficiently large. Since the ratio $\frac{d}{\varphi(d)}$ can be made arbitrarily large by a judicious choice of $d$, we obtain the $n^{\alpha}$ bound claimed. + +Comment 3. While big results from analytic number theory such as the Prime Number Theorem or Linnik's Theorem certainly can be used in this problem, they do not seem to substantially simplify matters: all known solutions involve first reducing to condition ( $\dagger$ ), and even then analytic results do not make it clear how to proceed. For this reason, we regard this problem as suitable for the IMO. + +Rather than simplifying the problem, what nonelementary results from analytic number theory allow one to achieve is a strengthening of the main bound, typically replacing the $n \log (n)$ growth with a power $n^{1+\delta}$. However, we believe that such stronger bounds are unlikely to be found by students in the exam. + +The strongest bound we know how to achieve using purely elementary methods is a bound of the form $N>2^{k} n \log (n)^{M}$ for any positive integer $M$. This is achieved by a variant of the argument in Solution 1, choosing primes $p_{0}, \ldots, p_{M}$ with $p_{i} \mid 2^{2^{t+i-1}}+1$ and setting $N=\prod_{i} p_{i}$ and $n=$ $2^{-t M} \prod_{i}\left(p_{i}-1\right)$. + +N8. +Let $a$ and $b$ be two positive integers. Prove that the integer + +$$ +a^{2}+\left\lceil\frac{4 a^{2}}{b}\right\rceil +$$ + +is not a square. (Here $\lceil z\rceil$ denotes the least integer greater than or equal to z.) +(Russia) +Solution 1. Arguing indirectly, assume that + +$$ +a^{2}+\left\lceil\frac{4 a^{2}}{b}\right\rceil=(a+k)^{2}, \quad \text { or } \quad\left\lceil\frac{(2 a)^{2}}{b}\right\rceil=(2 a+k) k +$$ + +Clearly, $k \geqslant 1$. In other words, the equation + +$$ +\left\lceil\frac{c^{2}}{b}\right\rceil=(c+k) k +$$ + +has a positive integer solution $(c, k)$, with an even value of $c$. +Choose a positive integer solution of (1) with minimal possible value of $k$, without regard to the parity of $c$. From + +$$ +\frac{c^{2}}{b}>\left\lceil\frac{c^{2}}{b}\right\rceil-1=c k+k^{2}-1 \geqslant c k +$$ + +and + +$$ +\frac{(c-k)(c+k)}{b}<\frac{c^{2}}{b} \leqslant\left\lceil\frac{c^{2}}{b}\right\rceil=(c+k) k +$$ + +it can be seen that $c>b k>c-k$, so + +$$ +c=k b+r \quad \text { with some } 00$ and $0a$, so + +$$ +\begin{aligned} +& c^{2}-1 (with solutions) + +## Shortlisted Problems
(with solutions) + +$61^{\text {st }}$ International Mathematical Olympiad Saint-Petersburg — Russia, 18th-28th September 2020 + +## The Shortlist has to be kept strictly confidential until the conclusion of the following International Mathematical Olympiad.
IMO General Regulations §6.6 + +## Contributing Countries + +The Organising Committee and the Problem Selection Committee of IMO 2020 thank the following 39 countries for contributing 149 problem proposals: + +Armenia, Australia, Austria, Belgium, Brazil, Canada, Croatia, Cuba, Cyprus, Czech Republic, Denmark, Estonia, France, Georgia, Germany, Hong Kong, Hungary, India, Iran, Ireland, Israel, Kosovo, Latvia, Luxembourg, Mongolia, Netherlands, North Macedonia, Philippines, Poland, Slovakia, Slovenia, South Africa, Taiwan, Thailand, Trinidad and Tobago, Ukraine, United Kingdom, USA, Venezuela + +## Problem Selection Committee + +![](https://cdn.mathpix.com/cropped/2024_11_18_bcdd154f2030efbe2fa4g-05.jpg?height=610&width=1006&top_left_y=1822&top_left_x=525) + +Ilya I. Bogdanov, Sergey Berlov, Alexander Gaifullin, Alexander S. Golovanov, Géza Kós, Pavel Kozhevnikov, Dmitry Krachun, Ivan Mitrofanov, Fedor Petrov, Paul Vaderlind + +## Problems + +## Algebra + +A1. Version 1. Let $n$ be a positive integer, and set $N=2^{n}$. Determine the smallest real number $a_{n}$ such that, for all real $x$, + +$$ +\sqrt[N]{\frac{x^{2 N}+1}{2}} \leqslant a_{n}(x-1)^{2}+x +$$ + +Version 2. For every positive integer $N$, determine the smallest real number $b_{N}$ such that, for all real $x$, + +$$ +\sqrt[N]{\frac{x^{2 N}+1}{2}} \leqslant b_{N}(x-1)^{2}+x +$$ + +(Ireland) +A2. Let $\mathcal{A}$ denote the set of all polynomials in three variables $x, y, z$ with integer coefficients. Let $\mathcal{B}$ denote the subset of $\mathcal{A}$ formed by all polynomials which can be expressed as + +$$ +(x+y+z) P(x, y, z)+(x y+y z+z x) Q(x, y, z)+x y z R(x, y, z) +$$ + +with $P, Q, R \in \mathcal{A}$. Find the smallest non-negative integer $n$ such that $x^{i} y^{j} z^{k} \in \mathcal{B}$ for all nonnegative integers $i, j, k$ satisfying $i+j+k \geqslant n$. +(Venezuela) +A3. Suppose that $a, b, c, d$ are positive real numbers satisfying $(a+c)(b+d)=a c+b d$. Find the smallest possible value of + +$$ +\frac{a}{b}+\frac{b}{c}+\frac{c}{d}+\frac{d}{a} +$$ + +A4. Let $a, b, c, d$ be four real numbers such that $a \geqslant b \geqslant c \geqslant d>0$ and $a+b+c+d=1$. Prove that + +$$ +(a+2 b+3 c+4 d) a^{a} b^{b} c^{c} d^{d}<1 +$$ + +(Belgium) +A5. A magician intends to perform the following trick. She announces a positive integer $n$, along with $2 n$ real numbers $x_{1}<\ldots90^{\circ}, \angle C D A>90^{\circ}$, and $\angle D A B=\angle B C D$. Denote by $E$ and $F$ the reflections of $A$ in lines $B C$ and $C D$, respectively. Suppose that the segments $A E$ and $A F$ meet the line $B D$ at $K$ and $L$, respectively. Prove that the circumcircles of triangles $B E K$ and $D F L$ are tangent to each other. +(Slovakia) +G4. In the plane, there are $n \geqslant 6$ pairwise disjoint disks $D_{1}, D_{2}, \ldots, D_{n}$ with radii $R_{1} \geqslant R_{2} \geqslant \ldots \geqslant R_{n}$. For every $i=1,2, \ldots, n$, a point $P_{i}$ is chosen in disk $D_{i}$. Let $O$ be an arbitrary point in the plane. Prove that + +$$ +O P_{1}+O P_{2}+\ldots+O P_{n} \geqslant R_{6}+R_{7}+\ldots+R_{n} +$$ + +(A disk is assumed to contain its boundary.) +(Iran) +G5. Let $A B C D$ be a cyclic quadrilateral with no two sides parallel. Let $K, L, M$, and $N$ be points lying on sides $A B, B C, C D$, and $D A$, respectively, such that $K L M N$ is a rhombus with $K L \| A C$ and $L M \| B D$. Let $\omega_{1}, \omega_{2}, \omega_{3}$, and $\omega_{4}$ be the incircles of triangles $A N K$, $B K L, C L M$, and $D M N$, respectively. Prove that the internal common tangents to $\omega_{1}$ and $\omega_{3}$ and the internal common tangents to $\omega_{2}$ and $\omega_{4}$ are concurrent. +(Poland) +G6. Let $I$ and $I_{A}$ be the incenter and the $A$-excenter of an acute-angled triangle $A B C$ with $A B
b_{n}$ for infinitely many $n$, and $b_{n}>a_{n}$ for infinitely many $n$ ? +(b) Do there exist infinitely many primes $p$ for which there exist $p$-sequences $\left(a_{0}, a_{1}, a_{2}, \ldots\right)$ and $\left(b_{0}, b_{1}, b_{2}, \ldots\right)$ such that $a_{0}b_{n}$ for all $n \geqslant 1$ ? +(United Kingdom) +N5. Determine all functions $f$ defined on the set of all positive integers and taking non-negative integer values, satisfying the three conditions: +(i) $f(n) \neq 0$ for at least one $n$; +(ii) $f(x y)=f(x)+f(y)$ for every positive integers $x$ and $y$; +(iii) there are infinitely many positive integers $n$ such that $f(k)=f(n-k)$ for all $k0$, we should have + +$$ +\frac{(1+t)^{2 N}+1}{2} \leqslant\left(1+t+a_{n} t^{2}\right)^{N} . +$$ + +Expanding the brackets we get + +$$ +\left(1+t+a_{n} t^{2}\right)^{N}-\frac{(1+t)^{2 N}+1}{2}=\left(N a_{n}-\frac{N^{2}}{2}\right) t^{2}+c_{3} t^{3}+\ldots+c_{2 N} t^{2 N} +$$ + +with some coefficients $c_{3}, \ldots, c_{2 N}$. Since $a_{n}0$, then the left hand sides of $\mathcal{I}(N,-x)$ and $\mathcal{I}(N, x)$ coincide, while the right hand side of $\mathcal{I}(N,-x)$ is larger than that of $\mathcal{I}(N,-x)$ (their difference equals $2(N-1) x \geqslant 0)$. Therefore, $\mathcal{I}(N,-x)$ follows from $\mathcal{I}(N, x)$. So, hereafter we suppose that $x>0$. + +Divide $\mathcal{I}(N, x)$ by $x$ and let $t=(x-1)^{2} / x=x-2+1 / x$; then $\mathcal{I}(n, x)$ reads as + +$$ +f_{N}:=\frac{x^{N}+x^{-N}}{2} \leqslant\left(1+\frac{N}{2} t\right)^{N} +$$ + +The key identity is the expansion of $f_{N}$ as a polynomial in $t$ : + +## Lemma. + +$$ +f_{N}=N \sum_{k=0}^{N} \frac{1}{N+k}\binom{N+k}{2 k} t^{k} +$$ + +Proof. Apply induction on $N$. We will make use of the straightforward recurrence relation + +$$ +f_{N+1}+f_{N-1}=(x+1 / x) f_{N}=(2+t) f_{N} +$$ + +The base cases $N=1,2$ are straightforward: + +$$ +f_{1}=1+\frac{t}{2}, \quad f_{2}=\frac{1}{2} t^{2}+2 t+1 +$$ + +For the induction step from $N-1$ and $N$ to $N+1$, we compute the coefficient of $t^{k}$ in $f_{N+1}$ using the formula $f_{N+1}=(2+t) f_{N}-f_{N-1}$. For $k=0$ that coefficient equals 1 , for $k>0$ it equals + +$$ +\begin{aligned} +& 2 \frac{N}{N+k}\binom{N+k}{2 k}+\frac{N}{N+k-1}\binom{N+k-1}{2 k-2}-\frac{N-1}{N+k-1}\binom{N+k-1}{2 k} \\ +& =\frac{(N+k-1)!}{(2 k)!(N-k)!}\left(2 N+\frac{2 k(2 k-1) N}{(N+k-1)(N-k+1)}-\frac{(N-1)(N-k)}{N+k-1}\right) \\ +& =\frac{(N+k-1)!}{(2 k)!(N-k+1)!}\left(2 N(N-k+1)+3 k N+k-N^{2}-N\right)=\frac{\binom{N+k+1}{2 k}}{(N+k+1)}(N+1) +\end{aligned} +$$ + +that completes the induction. +Turning back to the problem, in order to prove (2) we write + +$$ +\left(1+\frac{N}{2} t\right)^{N}-f_{N}=\left(1+\frac{N}{2} t\right)^{N}-N \sum_{k=0}^{N} \frac{1}{N+k}\binom{N+k}{2 k} t^{k}=\sum_{k=0}^{N} \alpha_{k} t^{k} +$$ + +where + +$$ +\begin{aligned} +\alpha_{k} & =\left(\frac{N}{2}\right)^{k}\binom{N}{k}-\frac{N}{N+k}\binom{N+k}{2 k} \\ +& =\left(\frac{N}{2}\right)^{k}\binom{N}{k}\left(1-2^{k} \frac{(1+1 / N)(1+2 / N) \cdot \ldots \cdot(1+(k-1) / N)}{(k+1) \cdot \ldots \cdot(2 k)}\right) \\ +& \geqslant\left(\frac{N}{2}\right)^{k}\binom{N}{k}\left(1-2^{k} \frac{2 \cdot 3 \cdot \ldots \cdot k}{(k+1) \cdot \ldots \cdot(2 k)}\right)=\left(\frac{N}{2}\right)^{k}\binom{N}{k}\left(1-\prod_{j=1}^{k} \frac{2 j}{k+j}\right) \geqslant 0 +\end{aligned} +$$ + +and (2) follows. + +Solution 2.2 (for Version 2). Here we present another proof of the inequality (2) for $x>0$, or, equivalently, for $t=(x-1)^{2} / x \geqslant 0$. Instead of finding the coefficients of the polynomial $f_{N}=f_{N}(t)$ we may find its roots, which is in a sense more straightforward. Note that the recurrence (4) and the initial conditions $f_{0}=1, f_{1}=1+t / 2$ imply that $f_{N}$ is a polynomial in $t$ of degree $N$. It also follows by induction that $f_{N}(0)=1, f_{N}^{\prime}(0)=N^{2} / 2$ : the recurrence relations read as $f_{N+1}(0)+f_{N-1}(0)=2 f_{N}(0)$ and $f_{N+1}^{\prime}(0)+f_{N-1}^{\prime}(0)=2 f_{N}^{\prime}(0)+f_{N}(0)$, respectively. + +Next, if $x_{k}=\exp \left(\frac{i \pi(2 k-1)}{2 N}\right)$ for $k \in\{1,2, \ldots, N\}$, then + +$$ +-t_{k}:=2-x_{k}-\frac{1}{x_{k}}=2-2 \cos \frac{\pi(2 k-1)}{2 N}=4 \sin ^{2} \frac{\pi(2 k-1)}{4 N}>0 +$$ + +and + +$$ +f_{N}\left(t_{k}\right)=\frac{x_{k}^{N}+x_{k}^{-N}}{2}=\frac{\exp \left(\frac{i \pi(2 k-1)}{2}\right)+\exp \left(-\frac{i \pi(2 k-1)}{2}\right)}{2}=0 . +$$ + +So the roots of $f_{N}$ are $t_{1}, \ldots, t_{N}$ and by the AM-GM inequality we have + +$$ +\begin{aligned} +f_{N}(t)=\left(1-\frac{t}{t_{1}}\right)\left(1-\frac{t}{t_{2}}\right) \ldots\left(1-\frac{t}{t_{N}}\right) & \leqslant\left(1-\frac{t}{N}\left(\frac{1}{t_{1}}+\ldots+\frac{1}{t_{n}}\right)\right)^{N}= \\ +\left(1+\frac{t f_{N}^{\prime}(0)}{N}\right)^{N} & =\left(1+\frac{N}{2} t\right)^{N} +\end{aligned} +$$ + +Comment. The polynomial $f_{N}(t)$ equals to $\frac{1}{2} T_{N}(t+2)$, where $T_{n}$ is the $n^{\text {th }}$ Chebyshev polynomial of the first kind: $T_{n}(2 \cos s)=2 \cos n s, T_{n}(x+1 / x)=x^{n}+1 / x^{n}$. + +Solution 2.3 (for Version 2). Here we solve the problem when $N \geqslant 1$ is an arbitrary real number. For a real number $a$ let + +$$ +f(x)=\left(\frac{x^{2 N}+1}{2}\right)^{\frac{1}{N}}-a(x-1)^{2}-x +$$ + +Then $f(1)=0$, + +$$ +f^{\prime}(x)=\left(\frac{x^{2 N}+1}{2}\right)^{\frac{1}{N}-1} x^{2 N-1}-2 a(x-1)-1 \quad \text { and } \quad f^{\prime}(1)=0 +$$ + +$f^{\prime \prime}(x)=(1-N)\left(\frac{x^{2 N}+1}{2}\right)^{\frac{1}{N}-2} x^{4 N-2}+(2 N-1)\left(\frac{x^{2 N}+1}{2}\right)^{\frac{1}{N}-1} x^{2 N-2}-2 a \quad$ and $\quad f^{\prime \prime}(1)=N-2 a$. +So if $a<\frac{N}{2}$, the function $f$ has a strict local minimum at point 1 , and the inequality $f(x) \leqslant$ $0=f(1)$ does not hold. This proves $b_{N} \geqslant N / 2$. + +For $a=\frac{N}{2}$ we have $f^{\prime \prime}(1)=0$ and + +$$ +f^{\prime \prime \prime}(x)=\frac{1}{2}(1-N)(1-2 N)\left(\frac{x^{2 N}+1}{2}\right)^{\frac{1}{N}-3} x^{2 N-3}\left(1-x^{2 N}\right) \quad \begin{cases}>0 & \text { if } 01\end{cases} +$$ + +Hence, $f^{\prime \prime}(x)<0$ for $x \neq 1 ; f^{\prime}(x)>0$ for $x<1$ and $f^{\prime}(x)<0$ for $x>1$, finally $f(x)<0$ for $x \neq 1$. + +Comment. Version 2 is much more difficult, of rather A5 or A6 difficulty. The induction in Version 1 is rather straightforward, while all three above solutions of Version 2 require some creativity. + +This page is intentionally left blank + +A2. Let $\mathcal{A}$ denote the set of all polynomials in three variables $x, y, z$ with integer coefficients. Let $\mathcal{B}$ denote the subset of $\mathcal{A}$ formed by all polynomials which can be expressed as + +$$ +(x+y+z) P(x, y, z)+(x y+y z+z x) Q(x, y, z)+x y z R(x, y, z) +$$ + +with $P, Q, R \in \mathcal{A}$. Find the smallest non-negative integer $n$ such that $x^{i} y^{j} z^{k} \in \mathcal{B}$ for all nonnegative integers $i, j, k$ satisfying $i+j+k \geqslant n$. +(Venezuela) +Answer: $n=4$. +Solution. We start by showing that $n \leqslant 4$, i.e., any monomial $f=x^{i} y^{j} z^{k}$ with $i+j+k \geqslant 4$ belongs to $\mathcal{B}$. Assume that $i \geqslant j \geqslant k$, the other cases are analogous. + +Let $x+y+z=p, x y+y z+z x=q$ and $x y z=r$. Then + +$$ +0=(x-x)(x-y)(x-z)=x^{3}-p x^{2}+q x-r +$$ + +therefore $x^{3} \in \mathcal{B}$. Next, $x^{2} y^{2}=x y q-(x+y) r \in \mathcal{B}$. +If $k \geqslant 1$, then $r$ divides $f$, thus $f \in \mathcal{B}$. If $k=0$ and $j \geqslant 2$, then $x^{2} y^{2}$ divides $f$, thus we have $f \in \mathcal{B}$ again. Finally, if $k=0, j \leqslant 1$, then $x^{3}$ divides $f$ and $f \in \mathcal{B}$ in this case also. + +In order to prove that $n \geqslant 4$, we show that the monomial $x^{2} y$ does not belong to $\mathcal{B}$. Assume the contrary: + +$$ +x^{2} y=p P+q Q+r R +$$ + +for some polynomials $P, Q, R$. If polynomial $P$ contains the monomial $x^{2}$ (with nonzero coefficient), then $p P+q Q+r R$ contains the monomial $x^{3}$ with the same nonzero coefficient. So $P$ does not contain $x^{2}, y^{2}, z^{2}$ and we may write + +$$ +x^{2} y=(x+y+z)(a x y+b y z+c z x)+(x y+y z+z x)(d x+e y+f z)+g x y z +$$ + +where $a, b, c ; d, e, f ; g$ are the coefficients of $x y, y z, z x ; x, y, z ; x y z$ in the polynomials $P$; $Q ; R$, respectively (the remaining coefficients do not affect the monomials of degree 3 in $p P+q Q+r R)$. By considering the coefficients of $x y^{2}$ we get $e=-a$, analogously $e=-b$, $f=-b, f=-c, d=-c$, thus $a=b=c$ and $f=e=d=-a$, but then the coefficient of $x^{2} y$ in the right hand side equals $a+d=0 \neq 1$. + +Comment 1. The general question is the following. Call a polynomial $f\left(x_{1}, \ldots, x_{n}\right)$ with integer coefficients nice, if $f(0,0, \ldots, 0)=0$ and $f\left(x_{\pi_{1}}, \ldots, x_{\pi_{n}}\right)=f\left(x_{1}, \ldots, x_{n}\right)$ for any permutation $\pi$ of $1, \ldots, n$ (in other words, $f$ is symmetric and its constant term is zero.) Denote by $\mathcal{I}$ the set of polynomials of the form + +$$ +p_{1} q_{1}+p_{2} q_{2}+\ldots+p_{m} q_{m} +$$ + +where $m$ is an integer, $q_{1}, \ldots, q_{m}$ are polynomials with integer coefficients, and $p_{1}, \ldots, p_{m}$ are nice polynomials. Find the least $N$ for which any monomial of degree at least $N$ belongs to $\mathcal{I}$. + +The answer is $n(n-1) / 2+1$. The lower bound follows from the following claim: the polynomial + +$$ +F\left(x_{1}, \ldots, x_{n}\right)=x_{2} x_{3}^{2} x_{4}^{3} \cdot \ldots \cdot x_{n}^{n-1} +$$ + +does not belong to $\mathcal{I}$. +Assume that $F=\sum p_{i} q_{i}$, according to (2). By taking only the monomials of degree $n(n-1) / 2$, we can additionally assume that every $p_{i}$ and every $q_{i}$ is homogeneous, $\operatorname{deg} p_{i}>0$, and $\operatorname{deg} p_{i}+\operatorname{deg} q_{i}=$ $\operatorname{deg} F=n(n-1) / 2$ for all $i$. + +Consider the alternating sum + +$$ +\sum_{\pi} \operatorname{sign}(\pi) F\left(x_{\pi_{1}}, \ldots, x_{\pi_{n}}\right)=\sum_{i=1}^{m} p_{i} \sum_{\pi} \operatorname{sign}(\pi) q_{i}\left(x_{\pi_{1}}, \ldots, x_{\pi_{n}}\right):=S +$$ + +where the summation is done over all permutations $\pi$ of $1, \ldots n$, and $\operatorname{sign}(\pi)$ denotes the sign of the permutation $\pi$. Since $\operatorname{deg} q_{i}=n(n-1) / 2-\operatorname{deg} p_{i}1$ and that the proposition is proved for smaller values of $n$. + +We proceed by an internal induction on $S:=\left|\left\{i: c_{i}=0\right\}\right|$. In the base case $S=0$ the monomial $h$ is divisible by the nice polynomial $x_{1} \cdot \ldots x_{n}$, therefore $h \in \mathcal{I}$. Now assume that $S>0$ and that the claim holds for smaller values of $S$. Let $T=n-S$. We may assume that $c_{T+1}=\ldots=c_{n}=0$ and $h=x_{1} \cdot \ldots \cdot x_{T} g\left(x_{1}, \ldots, x_{n-1}\right)$, where $\operatorname{deg} g=n(n-1) / 2-T+1 \geqslant(n-1)(n-2) / 2+1$. Using the outer induction hypothesis we represent $g$ as $p_{1} q_{1}+\ldots+p_{m} q_{m}$, where $p_{i}\left(x_{1}, \ldots, x_{n-1}\right)$ are nice polynomials in $n-1$ variables. There exist nice homogeneous polynomials $P_{i}\left(x_{1}, \ldots, x_{n}\right)$ such that $P_{i}\left(x_{1}, \ldots, x_{n-1}, 0\right)=p_{i}\left(x_{1}, \ldots, x_{n-1}\right)$. In other words, $\Delta_{i}:=p_{i}\left(x_{1}, \ldots, x_{n-1}\right)-P_{i}\left(x_{1}, \ldots, x_{n-1}, x_{n}\right)$ is divisible by $x_{n}$, let $\Delta_{i}=x_{n} g_{i}$. We get + +$$ +h=x_{1} \cdot \ldots \cdot x_{T} \sum p_{i} q_{i}=x_{1} \cdot \ldots \cdot x_{T} \sum\left(P_{i}+x_{n} g_{i}\right) q_{i}=\left(x_{1} \cdot \ldots \cdot x_{T} x_{n}\right) \sum g_{i} q_{i}+\sum P_{i} q_{i} \in \mathcal{I} +$$ + +The first term belongs to $\mathcal{I}$ by the inner induction hypothesis. This completes both inductions. +Comment 2. The solutions above work smoothly for the versions of the original problem and its extensions to the case of $n$ variables, where all polynomials are assumed to have real coefficients. In the version with integer coefficients, the argument showing that $x^{2} y \notin \mathcal{B}$ can be simplified: it is not hard to show that in every polynomial $f \in \mathcal{B}$, the sum of the coefficients of $x^{2} y, x^{2} z, y^{2} x, y^{2} z, z^{2} x$ and $z^{2} y$ is even. A similar fact holds for any number of variables and also implies that $N \geqslant n(n-1) / 2+1$ in terms of the previous comment. + +A3. Suppose that $a, b, c, d$ are positive real numbers satisfying $(a+c)(b+d)=a c+b d$. Find the smallest possible value of + +$$ +S=\frac{a}{b}+\frac{b}{c}+\frac{c}{d}+\frac{d}{a} +$$ + +(Israel) +Answer: The smallest possible value is 8. +Solution 1. To show that $S \geqslant 8$, apply the AM-GM inequality twice as follows: + +$$ +\left(\frac{a}{b}+\frac{c}{d}\right)+\left(\frac{b}{c}+\frac{d}{a}\right) \geqslant 2 \sqrt{\frac{a c}{b d}}+2 \sqrt{\frac{b d}{a c}}=\frac{2(a c+b d)}{\sqrt{a b c d}}=\frac{2(a+c)(b+d)}{\sqrt{a b c d}} \geqslant 2 \cdot \frac{2 \sqrt{a c} \cdot 2 \sqrt{b d}}{\sqrt{a b c d}}=8 . +$$ + +The above inequalities turn into equalities when $a=c$ and $b=d$. Then the condition $(a+c)(b+d)=a c+b d$ can be rewritten as $4 a b=a^{2}+b^{2}$. So it is satisfied when $a / b=2 \pm \sqrt{3}$. Hence, $S$ attains value 8 , e.g., when $a=c=1$ and $b=d=2+\sqrt{3}$. + +Solution 2. By homogeneity we may suppose that $a b c d=1$. Let $a b=C, b c=A$ and $c a=B$. Then $a, b, c$ can be reconstructed from $A, B$ and $C$ as $a=\sqrt{B C / A}, b=\sqrt{A C / B}$ and $c=\sqrt{A B / C}$. Moreover, the condition $(a+c)(b+d)=a c+b d$ can be written in terms of $A, B, C$ as + +$$ +A+\frac{1}{A}+C+\frac{1}{C}=b c+a d+a b+c d=(a+c)(b+d)=a c+b d=B+\frac{1}{B} . +$$ + +We then need to minimize the expression + +$$ +\begin{aligned} +S & :=\frac{a d+b c}{b d}+\frac{a b+c d}{a c}=\left(A+\frac{1}{A}\right) B+\left(C+\frac{1}{C}\right) \frac{1}{B} \\ +& =\left(A+\frac{1}{A}\right)\left(B-\frac{1}{B}\right)+\left(A+\frac{1}{A}+C+\frac{1}{C}\right) \frac{1}{B} \\ +& =\left(A+\frac{1}{A}\right)\left(B-\frac{1}{B}\right)+\left(B+\frac{1}{B}\right) \frac{1}{B} . +\end{aligned} +$$ + +Without loss of generality assume that $B \geqslant 1$ (otherwise, we may replace $B$ by $1 / B$ and swap $A$ and $C$, this changes neither the relation nor the function to be maximized). Therefore, we can write + +$$ +S \geqslant 2\left(B-\frac{1}{B}\right)+\left(B+\frac{1}{B}\right) \frac{1}{B}=2 B+\left(1-\frac{1}{B}\right)^{2}=: f(B) +$$ + +Clearly, $f$ increases on $[1, \infty)$. Since + +$$ +B+\frac{1}{B}=A+\frac{1}{A}+C+\frac{1}{C} \geqslant 4, +$$ + +we have $B \geqslant B^{\prime}$, where $B^{\prime}=2+\sqrt{3}$ is the unique root greater than 1 of the equation $B^{\prime}+1 / B^{\prime}=4$. Hence, + +$$ +S \geqslant f(B) \geqslant f\left(B^{\prime}\right)=2\left(B^{\prime}-\frac{1}{B^{\prime}}\right)+\left(B^{\prime}+\frac{1}{B^{\prime}}\right) \frac{1}{B^{\prime}}=2 B^{\prime}-\frac{2}{B^{\prime}}+\frac{4}{B^{\prime}}=8 +$$ + +It remains to note that when $A=C=1$ and $B=B^{\prime}$ we have the equality $S=8$. + +Solution 3. We present another proof of the inequality $S \geqslant 8$. We start with the estimate + +$$ +\left(\frac{a}{b}+\frac{c}{d}\right)+\left(\frac{b}{c}+\frac{d}{a}\right) \geqslant 2 \sqrt{\frac{a c}{b d}}+2 \sqrt{\frac{b d}{a c}} +$$ + +Let $y=\sqrt{a c}$ and $z=\sqrt{b d}$, and assume, without loss of generality, that $a c \geqslant b d$. By the AM-GM inequality, we have + +$$ +y^{2}+z^{2}=a c+b d=(a+c)(b+d) \geqslant 2 \sqrt{a c} \cdot 2 \sqrt{b d}=4 y z . +$$ + +Substituting $x=y / z$, we get $4 x \leqslant x^{2}+1$. For $x \geqslant 1$, this holds if and only if $x \geqslant 2+\sqrt{3}$. +Now we have + +$$ +2 \sqrt{\frac{a c}{b d}}+2 \sqrt{\frac{b d}{a c}}=2\left(x+\frac{1}{x}\right) . +$$ + +Clearly, this is minimized by setting $x(\geqslant 1)$ as close to 1 as possible, i.e., by taking $x=2+\sqrt{3}$. Then $2(x+1 / x)=2((2+\sqrt{3})+(2-\sqrt{3}))=8$, as required. + +A4. Let $a, b, c, d$ be four real numbers such that $a \geqslant b \geqslant c \geqslant d>0$ and $a+b+c+d=1$. Prove that + +$$ +(a+2 b+3 c+4 d) a^{a} b^{b} c^{c} d^{d}<1 +$$ + +(Belgium) +Solution 1. The weighted AM-GM inequality with weights $a, b, c, d$ gives + +$$ +a^{a} b^{b} c^{c} d^{d} \leqslant a \cdot a+b \cdot b+c \cdot c+d \cdot d=a^{2}+b^{2}+c^{2}+d^{2} +$$ + +so it suffices to prove that $(a+2 b+3 c+4 d)\left(a^{2}+b^{2}+c^{2}+d^{2}\right)<1=(a+b+c+d)^{3}$. This can be done in various ways, for example: + +$$ +\begin{aligned} +(a+b+c+d)^{3}> & a^{2}(a+3 b+3 c+3 d)+b^{2}(3 a+b+3 c+3 d) \\ +& \quad+c^{2}(3 a+3 b+c+3 d)+d^{2}(3 a+3 b+3 c+d) \\ +\geqslant & \left(a^{2}+b^{2}+c^{2}+d^{2}\right) \cdot(a+2 b+3 c+4 d) +\end{aligned} +$$ + +Solution 2. From $b \geqslant d$ we get + +$$ +a+2 b+3 c+4 d \leqslant a+3 b+3 c+3 d=3-2 a +$$ + +If $a<\frac{1}{2}$, then the statement can be proved by + +$$ +(a+2 b+3 c+4 d) a^{a} b^{b} c^{c} d^{d} \leqslant(3-2 a) a^{a} a^{b} a^{c} a^{d}=(3-2 a) a=1-(1-a)(1-2 a)<1 +$$ + +From now on we assume $\frac{1}{2} \leqslant a<1$. +By $b, c, d<1-a$ we have + +$$ +b^{b} c^{c} d^{d}<(1-a)^{b} \cdot(1-a)^{c} \cdot(1-a)^{d}=(1-a)^{1-a} . +$$ + +Therefore, + +$$ +(a+2 b+3 c+4 d) a^{a} b^{b} c^{c} d^{d}<(3-2 a) a^{a}(1-a)^{1-a} +$$ + +For $00 +$$ + +so $g$ is strictly convex on $(0,1)$. +By $g\left(\frac{1}{2}\right)=\log 2+2 \cdot \frac{1}{2} \log \frac{1}{2}=0$ and $\lim _{x \rightarrow 1-} g(x)=0$, we have $g(x) \leqslant 0$ (and hence $f(x) \leqslant 1$ ) for all $x \in\left[\frac{1}{2}, 1\right)$, and therefore + +$$ +(a+2 b+3 c+4 d) a^{a} b^{b} c^{c} d^{d}0$ with $\sum_{i} a_{i}=1$, the inequality + +$$ +\left(\sum_{i} i a_{i}\right) \prod_{i} a_{i}^{a_{i}} \leqslant 1 +$$ + +does not necessarily hold. Indeed, let $a_{2}=a_{3}=\ldots=a_{n}=\varepsilon$ and $a_{1}=1-(n-1) \varepsilon$, where $n$ and $\varepsilon \in(0,1 / n)$ will be chosen later. Then + +$$ +\left(\sum_{i} i a_{i}\right) \prod_{i} a_{i}^{a_{i}}=\left(1+\frac{n(n-1)}{2} \varepsilon\right) \varepsilon^{(n-1) \varepsilon}(1-(n-1) \varepsilon)^{1-(n-1) \varepsilon} +$$ + +If $\varepsilon=C / n^{2}$ with an arbitrary fixed $C>0$ and $n \rightarrow \infty$, then the factors $\varepsilon^{(n-1) \varepsilon}=\exp ((n-1) \varepsilon \log \varepsilon)$ and $(1-(n-1) \varepsilon)^{1-(n-1) \varepsilon}$ tend to 1 , so the limit of (1) in this set-up equals $1+C / 2$. This is not simply greater than 1 , but it can be arbitrarily large. + +A5. A magician intends to perform the following trick. She announces a positive integer $n$, along with $2 n$ real numbers $x_{1}<\ldots\max _{z \in \mathcal{O}(0)}|z|$, this yields $f(a)=f(-a)$ and $f^{2 a^{2}}(0)=0$. Therefore, the sequence $\left(f^{k}(0): k=0,1, \ldots\right)$ is purely periodic with a minimal period $T$ which divides $2 a^{2}$. Analogously, $T$ divides $2(a+1)^{2}$, therefore, $T \mid \operatorname{gcd}\left(2 a^{2}, 2(a+1)^{2}\right)=2$, i.e., $f(f(0))=0$ and $a(f(a)-f(-a))=f^{2 a^{2}}(0)=0$ for all $a$. Thus, + +$$ +\begin{array}{ll} +f(a)=f(-a) \quad \text { for all } a \neq 0 \\ +\text { in particular, } & f(1)=f(-1)=0 +\end{array} +$$ + +Next, for each $n \in \mathbb{Z}$, by $E(n, 1-n)$ we get + +$$ +n f(n)+(1-n) f(1-n)=f^{n^{2}+(1-n)^{2}}(1)=f^{2 n^{2}-2 n}(0)=0 +$$ + +Assume that there exists some $m \neq 0$ such that $f(m) \neq 0$. Choose such an $m$ for which $|m|$ is minimal possible. Then $|m|>1$ due to $(\boldsymbol{\phi}) ; f(|m|) \neq 0$ due to ( $\boldsymbol{\phi})$; and $f(1-|m|) \neq 0$ due to $(\Omega)$ for $n=|m|$. This contradicts to the minimality assumption. + +So, $f(n)=0$ for $n \neq 0$. Finally, $f(0)=f^{3}(0)=f^{4}(2)=2 f(2)=0$. Clearly, the function $f(x) \equiv 0$ satisfies the problem condition, which provides the first of the two answers. +Case 2: All orbits are infinite. +Since the orbits $\mathcal{O}(a)$ and $\mathcal{O}(a-1)$ differ by finitely many terms for all $a \in \mathbb{Z}$, each two orbits $\mathcal{O}(a)$ and $\mathcal{O}(b)$ have infinitely many common terms for arbitrary $a, b \in \mathbb{Z}$. + +For a minute, fix any $a, b \in \mathbb{Z}$. We claim that all pairs $(n, m)$ of nonnegative integers such that $f^{n}(a)=f^{m}(b)$ have the same difference $n-m$. Arguing indirectly, we have $f^{n}(a)=f^{m}(b)$ and $f^{p}(a)=f^{q}(b)$ with, say, $n-m>p-q$, then $f^{p+m+k}(b)=f^{p+n+k}(a)=f^{q+n+k}(b)$, for all nonnegative integers $k$. This means that $f^{\ell+(n-m)-(p-q)}(b)=f^{\ell}(b)$ for all sufficiently large $\ell$, i.e., that the sequence $\left(f^{n}(b)\right)$ is eventually periodic, so $\mathcal{O}(b)$ is finite, which is impossible. + +Now, for every $a, b \in \mathbb{Z}$, denote the common difference $n-m$ defined above by $X(a, b)$. We have $X(a-1, a)=1$ by (1). Trivially, $X(a, b)+X(b, c)=X(a, c)$, as if $f^{n}(a)=f^{m}(b)$ and $f^{p}(b)=f^{q}(c)$, then $f^{p+n}(a)=f^{p+m}(b)=f^{q+m}(c)$. These two properties imply that $X(a, b)=b-a$ for all $a, b \in \mathbb{Z}$. + +But (1) yields $f^{a^{2}+1}(f(a-1))=f^{a^{2}}(f(a))$, so + +$$ +1=X(f(a-1), f(a))=f(a)-f(a-1) \quad \text { for all } a \in \mathbb{Z} +$$ + +Recalling that $f(-1)=0$, we conclude by (two-sided) induction on $x$ that $f(x)=x+1$ for all $x \in \mathbb{Z}$. + +Finally, the obtained function also satisfies the assumption. Indeed, $f^{n}(x)=x+n$ for all $n \geqslant 0$, so + +$$ +f^{a^{2}+b^{2}}(a+b)=a+b+a^{2}+b^{2}=a f(a)+b f(b) +$$ + +Comment. There are many possible variations of the solution above, but it seems that finiteness of orbits seems to be a crucial distinction in all solutions. However, the case distinction could be made in different ways; in particular, there exist some versions of Case 1 which work whenever there is at least one finite orbit. + +We believe that Case 2 is conceptually harder than Case 1. + +A7. Let $n$ and $k$ be positive integers. Prove that for $a_{1}, \ldots, a_{n} \in\left[1,2^{k}\right]$ one has + +$$ +\sum_{i=1}^{n} \frac{a_{i}}{\sqrt{a_{1}^{2}+\ldots+a_{i}^{2}}} \leqslant 4 \sqrt{k n} +$$ + +(Iran) +Solution 1. Partition the set of indices $\{1,2, \ldots, n\}$ into disjoint subsets $M_{1}, M_{2}, \ldots, M_{k}$ so that $a_{\ell} \in\left[2^{j-1}, 2^{j}\right]$ for $\ell \in M_{j}$. Then, if $\left|M_{j}\right|=: p_{j}$, we have + +$$ +\sum_{\ell \in M_{j}} \frac{a_{\ell}}{\sqrt{a_{1}^{2}+\ldots+a_{\ell}^{2}}} \leqslant \sum_{i=1}^{p_{j}} \frac{2^{j}}{2^{j-1} \sqrt{i}}=2 \sum_{i=1}^{p_{j}} \frac{1}{\sqrt{i}} +$$ + +where we used that $a_{\ell} \leqslant 2^{j}$ and in the denominator every index from $M_{j}$ contributes at least $\left(2^{j-1}\right)^{2}$. Now, using $\sqrt{i}-\sqrt{i-1}=\frac{1}{\sqrt{i}+\sqrt{i-1}} \geqslant \frac{1}{2 \sqrt{i}}$, we deduce that + +$$ +\sum_{\ell \in M_{j}} \frac{a_{\ell}}{\sqrt{a_{1}^{2}+\ldots+a_{\ell}^{2}}} \leqslant 2 \sum_{i=1}^{p_{j}} \frac{1}{\sqrt{i}} \leqslant 2 \sum_{i=1}^{p_{j}} 2(\sqrt{i}-\sqrt{i-1})=4 \sqrt{p_{j}} +$$ + +Therefore, summing over $j=1, \ldots, k$ and using the QM-AM inequality, we obtain + +$$ +\sum_{\ell=1}^{n} \frac{a_{\ell}}{\sqrt{a_{1}^{2}+\ldots+a_{\ell}^{2}}} \leqslant 4 \sum_{j=1}^{k} \sqrt{\left|M_{j}\right|} \leqslant 4 \sqrt{k \sum_{j=1}^{k}\left|M_{j}\right|}=4 \sqrt{k n} +$$ + +Comment. Consider the function $f\left(a_{1}, \ldots, a_{n}\right)=\sum_{i=1}^{n} \frac{a_{i}}{\sqrt{a_{1}^{2}+\ldots+a_{i}^{2}}}$. One can see that rearranging the variables in increasing order can only increase the value of $f\left(a_{1}, \ldots, a_{n}\right)$. Indeed, if $a_{j}>a_{j+1}$ for some index $j$ then we have + +$$ +f\left(a_{1}, \ldots, a_{j-1}, a_{j+1}, a_{j}, a_{j+2}, \ldots, a_{n}\right)-f\left(a_{1}, \ldots, a_{n}\right)=\frac{a}{S}+\frac{b}{\sqrt{S^{2}-a^{2}}}-\frac{b}{S}-\frac{a}{\sqrt{S^{2}-b^{2}}} +$$ + +where $a=a_{j}, b=a_{j+1}$, and $S=\sqrt{a_{1}^{2}+\ldots+a_{j+1}^{2}}$. The positivity of the last expression above follows from + +$$ +\frac{b}{\sqrt{S^{2}-a^{2}}}-\frac{b}{S}=\frac{a^{2} b}{S \sqrt{S^{2}-a^{2}} \cdot\left(S+\sqrt{S^{2}-a^{2}}\right)}>\frac{a b^{2}}{S \sqrt{S^{2}-b^{2}} \cdot\left(S+\sqrt{S^{2}-b^{2}}\right)}=\frac{a}{\sqrt{S^{2}-b^{2}}}-\frac{a}{S} . +$$ + +Comment. If $ky-1$, hence + +$$ +f(x)>\frac{y-1}{f(y)} \quad \text { for all } x \in \mathbb{R}^{+} +$$ + +If $y>1$, this provides a desired positive lower bound for $f(x)$. +Now, let $M=\inf _{x \in \mathbb{R}^{+}} f(x)>0$. Then, for all $y \in \mathbb{R}^{+}$, + +$$ +M \geqslant \frac{y-1}{f(y)}, \quad \text { or } \quad f(y) \geqslant \frac{y-1}{M} +$$ + +Lemma 1. The function $f(x)$ (and hence $\phi(x)$ ) is bounded on any segment $[p, q]$, where $0\max \left\{C, \frac{C}{y}\right\}$. Then all three numbers $x, x y$, and $x+f(x y)$ are greater than $C$, so (*) reads as + +$$ +(x+x y+1)+1+y=(x+1) f(y)+1, \text { hence } f(y)=y+1 +$$ + +Comment 1. It may be useful to rewrite (*) in the form + +$$ +\phi(x+f(x y))+\phi(x y)=\phi(x) \phi(y)+x \phi(y)+y \phi(x)+\phi(x)+\phi(y) +$$ + +This general identity easily implies both (1) and (5). +Comment 2. There are other ways to prove that $f(x) \geqslant x+1$. Once one has proved this, they can use this stronger estimate instead of (3) in the proof of Lemma 1. Nevertheless, this does not make this proof simpler. So proving that $f(x) \geqslant x+1$ does not seem to be a serious progress towards the solution of the problem. In what follows, we outline one possible proof of this inequality. + +First of all, we improve inequality (3) by noticing that, in fact, $f(x) f(y) \geqslant y-1+M$, and hence + +$$ +f(y) \geqslant \frac{y-1}{M}+1 +$$ + +Now we divide the argument into two steps. +Step 1: We show that $M \leqslant 1$. +Suppose that $M>1$; recall the notation $a=f(1)$. Substituting $y=1 / x$ in (*), we get + +$$ +f(x+a)=f(x) f\left(\frac{1}{x}\right)+1-\frac{1}{x} \geqslant M f(x), +$$ + +provided that $x \geqslant 1$. By a straightforward induction on $\lceil(x-1) / a\rceil$, this yields + +$$ +f(x) \geqslant M^{(x-1) / a} +$$ + +Now choose an arbitrary $x_{0} \in \mathbb{R}^{+}$and define a sequence $x_{0}, x_{1}, \ldots$ by $x_{n+1}=x_{n}+f\left(x_{n}\right) \geqslant x_{n}+M$ for all $n \geqslant 0$; notice that the sequence is unbounded. On the other hand, by (4) we get + +$$ +a x_{n+1}>a f\left(x_{n}\right)=f\left(x_{n+1}\right) \geqslant M^{\left(x_{n+1}-1\right) / a}, +$$ + +which cannot hold when $x_{n+1}$ is large enough. + +Step 2: We prove that $f(y) \geqslant y+1$ for all $y \in \mathbb{R}^{+}$. +Arguing indirectly, choose $y \in \mathbb{R}^{+}$such that $f(y)n k$, so $a_{n} \geqslant k+1$. Now the $n-k+1$ numbers $a_{k}, a_{k+1}, \ldots, a_{n}$ are all greater than $k$; but there are only $n-k$ such values; this is not possible. + +If $a_{n}=n$ then $a_{1}, a_{2}, \ldots, a_{n-1}$ must be a permutation of the numbers $1, \ldots, n-1$ satisfying $a_{1} \leqslant 2 a_{2} \leqslant \ldots \leqslant(n-1) a_{n-1}$; there are $P_{n-1}$ such permutations. The last inequality in (*), $(n-1) a_{n-1} \leqslant n a_{n}=n^{2}$, holds true automatically. + +If $\left(a_{n-1}, a_{n}\right)=(n, n-1)$, then $a_{1}, \ldots, a_{n-2}$ must be a permutation of $1, \ldots, n-2$ satisfying $a_{1} \leqslant \ldots \leqslant(n-2) a_{n-2}$; there are $P_{n-2}$ such permutations. The last two inequalities in (*) hold true automatically by $(n-2) a_{n-2} \leqslant(n-2)^{2}k$. If $t=k$ then we are done, so assume $t>k$. + +Notice that one of the numbers among the $t-k$ numbers $a_{k}, a_{k+1}, \ldots, a_{t-1}$ is at least $t$, because there are only $t-k-1$ values between $k$ and $t$. Let $i$ be an index with $k \leqslant ik+1$. Then the chain of inequalities $k t=k a_{k} \leqslant \ldots \leqslant t a_{t}=k t$ should also turn into a chain of equalities. From this point we can find contradictions in several ways; for example by pointing to $a_{t-1}=\frac{k t}{t-1}=k+\frac{k}{t-1}$ which cannot be an integer, or considering +the product of the numbers $(k+1) a_{k+1}, \ldots,(t-1) a_{t-1}$; the numbers $a_{k+1}, \ldots, a_{t-1}$ are distinct and greater than $k$, so + +$$ +(k t)^{t-k-1}=(k+1) a_{k+1} \cdot(k+2) a_{k+2} \cdot \ldots \cdot(t-1) a_{t-1} \geqslant((k+1)(k+2) \cdot \ldots \cdot(t-1))^{2} +$$ + +Notice that $(k+i)(t-i)=k t+i(t-k-i)>k t$ for $1 \leqslant i(k t)^{t-k-1} +$$ + +Therefore, the case $t>k+1$ is not possible. + +C2. In a regular 100-gon, 41 vertices are colored black and the remaining 59 vertices are colored white. Prove that there exist 24 convex quadrilaterals $Q_{1}, \ldots, Q_{24}$ whose corners are vertices of the 100 -gon, so that + +- the quadrilaterals $Q_{1}, \ldots, Q_{24}$ are pairwise disjoint, and +- every quadrilateral $Q_{i}$ has three corners of one color and one corner of the other color. +(Austria) +Solution. Call a quadrilateral skew-colored, if it has three corners of one color and one corner of the other color. We will prove the following +Claim. If the vertices of a convex $(4 k+1)$-gon $P$ are colored black and white such that each color is used at least $k$ times, then there exist $k$ pairwise disjoint skew-colored quadrilaterals whose vertices are vertices of $P$. (One vertex of $P$ remains unused.) + +The problem statement follows by removing 3 arbitrary vertices of the 100-gon and applying the Claim to the remaining 97 vertices with $k=24$. +Proof of the Claim. We prove by induction. For $k=1$ we have a pentagon with at least one black and at least one white vertex. If the number of black vertices is even then remove a black vertex; otherwise remove a white vertex. In the remaining quadrilateral, there are an odd number of black and an odd number of white vertices, so the quadrilateral is skew-colored. + +For the induction step, assume $k \geqslant 2$. Let $b$ and $w$ be the numbers of black and white vertices, respectively; then $b, w \geqslant k$ and $b+w=4 k+1$. Without loss of generality we may assume $w \geqslant b$, so $k \leqslant b \leqslant 2 k$ and $2 k+1 \leqslant w \leqslant 3 k+1$. + +We want to find four consecutive vertices such that three of them are white, the fourth one is black. Denote the vertices by $V_{1}, V_{2}, \ldots, V_{4 k+1}$ in counterclockwise order, such that $V_{4 k+1}$ is black, and consider the following $k$ groups of vertices: + +$$ +\left(V_{1}, V_{2}, V_{3}, V_{4}\right),\left(V_{5}, V_{6}, V_{7}, V_{8}\right), \ldots,\left(V_{4 k-3}, V_{4 k-2}, V_{4 k-1}, V_{4 k}\right) +$$ + +In these groups there are $w$ white and $b-1$ black vertices. Since $w>b-1$, there is a group, $\left(V_{i}, V_{i+1}, V_{i+2}, V_{i+3}\right)$ that contains more white than black vertices. If three are white and one is black in that group, we are done. Otherwise, if $V_{i}, V_{i+1}, V_{i+2}, V_{i+3}$ are all white then let $V_{j}$ be the first black vertex among $V_{i+4}, \ldots, V_{4 k+1}$ (recall that $V_{4 k+1}$ is black); then $V_{j-3}, V_{j-2}$ and $V_{j-1}$ are white and $V_{j}$ is black. + +Now we have four consecutive vertices $V_{i}, V_{i+1}, V_{i+2}, V_{i+3}$ that form a skew-colored quadrilateral. The remaining vertices form a convex ( $4 k-3$ )-gon; $w-3$ of them are white and $b-1$ are black. Since $b-1 \geqslant k-1$ and $w-3 \geqslant(2 k+1)-3>k-1$, we can apply the Claim with $k-1$. + +Comment. It is not true that the vertices of the 100 -gon can be split into 25 skew-colored quadrilaterals. A possible counter-example is when the vertices $V_{1}, V_{3}, V_{5}, \ldots, V_{81}$ are black and the other vertices, $V_{2}, V_{4}, \ldots, V_{80}$ and $V_{82}, V_{83}, \ldots, V_{100}$ are white. For having 25 skew-colored quadrilaterals, there should be 8 containing three black vertices. But such a quadrilateral splits the other 96 vertices into four sets in such a way that at least two sets contain odd numbers of vertices and therefore they cannot be grouped into disjoint quadrilaterals. +![](https://cdn.mathpix.com/cropped/2024_11_18_bcdd154f2030efbe2fa4g-34.jpg?height=318&width=552&top_left_y=2462&top_left_x=752) + +C3. Let $n$ be an integer with $n \geqslant 2$. On a slope of a mountain, $n^{2}$ checkpoints are marked, numbered from 1 to $n^{2}$ from the bottom to the top. Each of two cable car companies, $A$ and $B$, operates $k$ cable cars numbered from 1 to $k$; each cable car provides a transfer from some checkpoint to a higher one. For each company, and for any $i$ and $j$ with $1 \leqslant im_{p-1}$. +Choose $k$ to be the smallest index satisfying $m_{k}>m_{p-1}$; by our assumptions, we have $1\left|R_{2}\right|$. We will find a saddle subpair ( $R^{\prime}, C^{\prime}$ ) of ( $R_{1}, C_{1}$ ) with $\left|R^{\prime}\right| \leqslant\left|R_{2}\right|$; clearly, this implies the desired statement. +Step 1: We construct maps $\rho: R_{1} \rightarrow R_{1}$ and $\sigma: C_{1} \rightarrow C_{1}$ such that $\left|\rho\left(R_{1}\right)\right| \leqslant\left|R_{2}\right|$, and $a\left(\rho\left(r_{1}\right), c_{1}\right) \geqslant a\left(r_{1}, \sigma\left(c_{1}\right)\right)$ for all $r_{1} \in R_{1}$ and $c_{1} \in C_{1}$. + +Since $\left(R_{1}, C_{1}\right)$ is a saddle pair, for each $r_{2} \in R_{2}$ there is $r_{1} \in R_{1}$ such that $a\left(r_{1}, c_{1}\right) \geqslant a\left(r_{2}, c_{1}\right)$ for all $c_{1} \in C_{1}$; denote one such an $r_{1}$ by $\rho_{1}\left(r_{2}\right)$. Similarly, we define four functions + +$$ +\begin{array}{llllll} +\rho_{1}: R_{2} \rightarrow R_{1} & \text { such that } & a\left(\rho_{1}\left(r_{2}\right), c_{1}\right) \geqslant a\left(r_{2}, c_{1}\right) & \text { for all } & r_{2} \in R_{2}, & c_{1} \in C_{1} ; \\ +\rho_{2}: R_{1} \rightarrow R_{2} & \text { such that } & a\left(\rho_{2}\left(r_{1}\right), c_{2}\right) \geqslant a\left(r_{1}, c_{2}\right) & \text { for all } & r_{1} \in R_{1}, & c_{2} \in C_{2} ; \\ +\sigma_{1}: C_{2} \rightarrow C_{1} & \text { such that } & a\left(r_{1}, \sigma_{1}\left(c_{2}\right)\right) \leqslant a\left(r_{1}, c_{2}\right) & \text { for all } & r_{1} \in R_{1}, & c_{2} \in C_{2} ; \\ +\sigma_{2}: C_{1} \rightarrow C_{2} & \text { such that } & a\left(r_{2}, \sigma_{2}\left(c_{1}\right)\right) \leqslant a\left(r_{2}, c_{1}\right) & \text { for all } & r_{2} \in R_{2}, & c_{1} \in C_{1} . +\end{array} +$$ + +Set now $\rho=\rho_{1} \circ \rho_{2}: R_{1} \rightarrow R_{1}$ and $\sigma=\sigma_{1} \circ \sigma_{2}: C_{1} \rightarrow C_{1}$. We have + +$$ +\left|\rho\left(R_{1}\right)\right|=\left|\rho_{1}\left(\rho_{2}\left(R_{1}\right)\right)\right| \leqslant\left|\rho_{1}\left(R_{2}\right)\right| \leqslant\left|R_{2}\right| . +$$ + +Moreover, for all $r_{1} \in R_{1}$ and $c_{1} \in C_{1}$, we get + +$$ +\begin{aligned} +a\left(\rho\left(r_{1}\right), c_{1}\right)=a\left(\rho_{1}\left(\rho_{2}\left(r_{1}\right)\right), c_{1}\right) \geqslant a\left(\rho_{2}\left(r_{1}\right), c_{1}\right) & \geqslant a\left(\rho_{2}\left(r_{1}\right), \sigma_{2}\left(c_{1}\right)\right) \\ +& \geqslant a\left(r_{1}, \sigma_{2}\left(c_{1}\right)\right) \geqslant a\left(r_{1}, \sigma_{1}\left(\sigma_{2}\left(c_{1}\right)\right)\right)=a\left(r_{1}, \sigma\left(c_{1}\right)\right) +\end{aligned} +$$ + +as desired. +Step 2: Given maps $\rho$ and $\sigma$, we construct a proper saddle subpair $\left(R^{\prime}, C^{\prime}\right)$ of $\left(R_{1}, C_{1}\right)$. +The properties of $\rho$ and $\sigma$ yield that + +$$ +a\left(\rho^{i}\left(r_{1}\right), c_{1}\right) \geqslant a\left(\rho^{i-1}\left(r_{1}\right), \sigma\left(c_{1}\right)\right) \geqslant \ldots \geqslant a\left(r_{1}, \sigma^{i}\left(c_{1}\right)\right) +$$ + +for each positive integer $i$ and all $r_{1} \in R_{1}, c_{1} \in C_{1}$. +Consider the images $R^{i}=\rho^{i}\left(R_{1}\right)$ and $C^{i}=\sigma^{i}\left(C_{1}\right)$. Clearly, $R_{1}=R^{0} \supseteq R^{1} \supseteq R^{2} \supseteq \ldots$ and $C_{1}=C^{0} \supseteq C^{1} \supseteq C^{2} \supseteq \ldots$. Since both chains consist of finite sets, there is an index $n$ such that $R^{n}=R^{n+1}=\ldots$ and $C^{n}=C^{n+1}=\ldots$. Then $\rho^{n}\left(R^{n}\right)=R^{2 n}=R^{n}$, so $\rho^{n}$ restricted to $R^{n}$ is a bijection. Similarly, $\sigma^{n}$ restricted to $C^{n}$ is a bijection from $C^{n}$ to itself. Therefore, there exists a positive integer $k$ such that $\rho^{n k}$ acts identically on $R^{n}$, and $\sigma^{n k}$ acts identically on $C^{n}$. + +We claim now that $\left(R^{n}, C^{n}\right)$ is a saddle subpair of $\left(R_{1}, C_{1}\right)$, with $\left|R^{n}\right| \leqslant\left|R^{1}\right|=\left|\rho\left(R_{1}\right)\right| \leqslant$ $\left|R_{2}\right|$, which is what we needed. To check that this is a saddle pair, take any row $r^{\prime}$; since $\left(R_{1}, C_{1}\right)$ is a saddle pair, there exists $r_{1} \in R_{1}$ such that $a\left(r_{1}, c_{1}\right) \geqslant a\left(r^{\prime}, c_{1}\right)$ for all $c_{1} \in C_{1}$. Set now $r_{*}=\rho^{n k}\left(r_{1}\right) \in R^{n}$. Then, for each $c \in C^{n}$ we have $c=\sigma^{n k}(c)$ and hence + +$$ +a\left(r_{*}, c\right)=a\left(\rho^{n k}\left(r_{1}\right), c\right) \geqslant a\left(r_{1}, \sigma^{n k}(c)\right)=a\left(r_{1}, c\right) \geqslant a\left(r^{\prime}, c\right) +$$ + +which establishes condition $(i)$. Condition (ii) is checked similarly. + +Solution 2. Denote by $\mathcal{R}$ and $\mathcal{C}$ the set of all rows and the set of all columns of the table, respectively. Let $\mathcal{T}$ denote the given table; for a set $R$ of rows and a set $C$ of columns, let $\mathcal{T}[R, C]$ denote the subtable obtained by intersecting rows from $R$ and columns from $C$. + +We say that row $r_{1}$ exceeds row $r_{2}$ in range of columns $C$ (where $C \subseteq \mathcal{C}$ ) and write $r_{1} \succeq_{C} r_{2}$ or $r_{2} \leq_{C} r_{1}$, if $a\left(r_{1}, c\right) \geqslant a\left(r_{2}, c\right)$ for all $c \in C$. We say that a row $r_{1}$ is equal to a row $r_{2}$ in range of columns $C$ and write $r_{1} \equiv_{C} r_{2}$, if $a\left(r_{1}, c\right)=a\left(r_{2}, c\right)$ for all $c \in C$. We introduce similar notions, and use the same notation, for columns. Then conditions ( $i$ ) and (ii) in the definition of a saddle pair can be written as $(i)$ for each $r^{\prime} \in \mathcal{R}$ there exists $r \in R$ such that $r \geq_{C} r^{\prime}$; and (ii) for each $c^{\prime} \in \mathcal{C}$ there exists $c \in C$ such that $c \leq_{R} c^{\prime}$. + +Lemma. Suppose that $(R, C)$ is a minimal pair. Remove from the table several rows outside of $R$ and/or several columns outside of $C$. Then $(R, C)$ remains a minimal pair in the new table. +Proof. Obviously, $(R, C)$ remains a saddle pair. Suppose $\left(R^{\prime}, C^{\prime}\right)$ is a proper subpair of $(R, C)$. Since $(R, C)$ is a saddle pair, for each row $r^{*}$ of the initial table, there is a row $r \in R$ such that $r \geq_{C} r^{*}$. If ( $R^{\prime}, C^{\prime}$ ) became saddle after deleting rows not in $R$ and/or columns not in $C$, there would be a row $r^{\prime} \in R^{\prime}$ satisfying $r^{\prime} \geq_{C^{\prime}} r$. Therefore, we would obtain that $r^{\prime} \geq_{C^{\prime \prime}} r^{*}$, which is exactly condition $(i)$ for the pair ( $R^{\prime}, C^{\prime}$ ) in the initial table; condition (ii) is checked similarly. Thus, ( $\left.R^{\prime}, C^{\prime}\right)$ was saddle in the initial table, which contradicts the hypothesis that $(R, C)$ was minimal. Hence, $(R, C)$ remains minimal after deleting rows and/or columns. + +By the Lemma, it suffices to prove the statement of the problem in the case $\mathcal{R}=R_{1} \cup R_{2}$ and $\mathcal{C}=C_{1} \cup C_{2}$. Further, suppose that there exist rows that belong both to $R_{1}$ and $R_{2}$. Duplicate every such row, and refer one copy of it to the set $R_{1}$, and the other copy to the set $R_{2}$. Then $\left(R_{1}, C_{1}\right)$ and $\left(R_{2}, C_{2}\right)$ will remain minimal pairs in the new table, with the same numbers of rows and columns, but the sets $R_{1}$ and $R_{2}$ will become disjoint. Similarly duplicating columns in $C_{1} \cap C_{2}$, we make $C_{1}$ and $C_{2}$ disjoint. Thus it is sufficient to prove the required statement in the case $R_{1} \cap R_{2}=\varnothing$ and $C_{1} \cap C_{2}=\varnothing$. + +The rest of the solution is devoted to the proof of the following claim including the statement of the problem. +Claim. Suppose that $\left(R_{1}, C_{1}\right)$ and $\left(R_{2}, C_{2}\right)$ are minimal pairs in table $\mathcal{T}$ such that $R_{2}=\mathcal{R} \backslash R_{1}$ and $C_{2}=\mathcal{C} \backslash C_{1}$. Then $\left|R_{1}\right|=\left|R_{2}\right|,\left|C_{1}\right|=\left|C_{2}\right| ;$ moreover, there are four bijections + +$$ +\begin{array}{llllll} +\rho_{1}: R_{2} \rightarrow R_{1} & \text { such that } & \rho_{1}\left(r_{2}\right) \equiv_{C_{1}} r_{2} & \text { for all } & r_{2} \in R_{2} ; \\ +\rho_{2}: R_{1} \rightarrow R_{2} & \text { such that } & \rho_{2}\left(r_{1}\right) \equiv_{C_{2}} r_{1} & \text { for all } & r_{1} \in R_{1} ; \\ +\sigma_{1}: C_{2} \rightarrow C_{1} & \text { such that } & \sigma_{1}\left(c_{2}\right) \equiv_{R_{1}} c_{2} & \text { for all } & c_{2} \in C_{2} ; \\ +\sigma_{2}: C_{1} \rightarrow C_{2} & \text { such that } & \sigma_{2}\left(c_{1}\right) \equiv_{R_{2}} c_{1} & \text { for all } & c_{1} \in C_{1} . +\end{array} +$$ + +We prove the Claim by induction on $|\mathcal{R}|+|\mathcal{C}|$. In the base case we have $\left|R_{1}\right|=\left|R_{2}\right|=$ $\left|C_{1}\right|=\left|C_{2}\right|=1$; let $R_{i}=\left\{r_{i}\right\}$ and $C_{i}=\left\{c_{i}\right\}$. Since $\left(R_{1}, C_{1}\right)$ and $\left(R_{2}, C_{2}\right)$ are saddle pairs, we have $a\left(r_{1}, c_{1}\right) \geqslant a\left(r_{2}, c_{1}\right) \geqslant a\left(r_{2}, c_{2}\right) \geqslant a\left(r_{1}, c_{2}\right) \geqslant a\left(r_{1}, c_{1}\right)$, hence, the table consists of four equal numbers, and the statement follows. + +To prove the inductive step, introduce the maps $\rho_{1}, \rho_{2}, \sigma_{1}$, and $\sigma_{2}$ as in Solution 1 , see (1). Suppose first that all four maps are surjective. Then, in fact, we have $\left|R_{1}\right|=\left|R_{2}\right|,\left|C_{1}\right|=\left|C_{2}\right|$, and all maps are bijective. Moreover, for all $r_{2} \in R_{2}$ and $c_{2} \in C_{2}$ we have + +$$ +\begin{aligned} +a\left(r_{2}, c_{2}\right) \leqslant a\left(r_{2}, \sigma_{2}^{-1}\left(c_{2}\right)\right) \leqslant a\left(\rho_{1}\left(r_{2}\right), \sigma_{2}^{-1}\left(c_{2}\right)\right) \leqslant a\left(\rho_{1}\left(r_{2}\right)\right. & \left., \sigma_{1}^{-1} \circ \sigma_{2}^{-1}\left(c_{2}\right)\right) \\ +& \leqslant a\left(\rho_{2} \circ \rho_{1}\left(r_{2}\right), \sigma_{1}^{-1} \circ \sigma_{2}^{-1}\left(c_{2}\right)\right) +\end{aligned} +$$ + +Summing up, we get + +$$ +\sum_{\substack{r_{2} \in R_{2} \\ c_{2} \in C_{2}}} a\left(r_{2}, c_{2}\right) \leqslant \sum_{\substack{r_{2} \in R_{2} \\ c_{2} \in C_{2}}} a\left(\rho_{2} \circ \rho_{1}\left(r_{2}\right), \sigma_{1}^{-1} \circ \sigma_{2}^{-1}\left(c_{2}\right)\right) . +$$ + +Since $\rho_{1} \circ \rho_{2}$ and $\sigma_{1}^{-1} \circ \sigma_{2}^{-1}$ are permutations of $R_{2}$ and $C_{2}$, respectively, this inequality is in fact equality. Therefore, all inequalities in (4) turn into equalities, which establishes the inductive step in this case. + +It remains to show that all four maps are surjective. For the sake of contradiction, we assume that $\rho_{1}$ is not surjective. Now let $R_{1}^{\prime}=\rho_{1}\left(R_{2}\right)$ and $C_{1}^{\prime}=\sigma_{1}\left(C_{2}\right)$, and set $R^{*}=R_{1} \backslash R_{1}^{\prime}$ and $C^{*}=C_{1} \backslash C_{1}^{\prime}$. By our assumption, $R^{*} \neq \varnothing$. + +Let $\mathcal{Q}$ be the table obtained from $\mathcal{T}$ by removing the rows in $R^{*}$ and the columns in $C^{*}$; in other words, $\mathcal{Q}=\mathcal{T}\left[R_{1}^{\prime} \cup R_{2}, C_{1}^{\prime} \cup C_{2}\right]$. By the definition of $\rho_{1}$, for each $r_{2} \in R_{2}$ we have $\rho_{1}\left(r_{2}\right) \geq_{C_{1}} r_{2}$, so a fortiori $\rho_{1}\left(r_{2}\right) \geq_{C_{1}^{\prime}} r_{2}$; moreover, $\rho_{1}\left(r_{2}\right) \in R_{1}^{\prime}$. Similarly, $C_{1}^{\prime} \ni \sigma_{1}\left(c_{2}\right) \leq_{R_{1}^{\prime}} c_{2}$ for each $c_{2} \in C_{2}$. This means that $\left(R_{1}^{\prime}, C_{1}^{\prime}\right)$ is a saddle pair in $\mathcal{Q}$. Recall that $\left(R_{2}, C_{2}\right)$ remains a minimal pair in $\mathcal{Q}$, due to the Lemma. + +Therefore, $\mathcal{Q}$ admits a minimal pair $\left(\bar{R}_{1}, \bar{C}_{1}\right)$ such that $\bar{R}_{1} \subseteq R_{1}^{\prime}$ and $\bar{C}_{1} \subseteq C_{1}^{\prime}$. For a minute, confine ourselves to the subtable $\overline{\mathcal{Q}}=\mathcal{Q}\left[\bar{R}_{1} \cup R_{2}, \bar{C}_{1} \cup C_{2}\right]$. By the Lemma, the pairs $\left(\bar{R}_{1}, \bar{C}_{1}\right)$ and $\left(R_{2}, C_{2}\right)$ are also minimal in $\overline{\mathcal{Q}}$. By the inductive hypothesis, we have $\left|R_{2}\right|=\left|\bar{R}_{1}\right| \leqslant\left|R_{1}^{\prime}\right|=\left|\rho_{1}\left(R_{2}\right)\right| \leqslant\left|R_{2}\right|$, so all these inequalities are in fact equalities. This implies that $\bar{R}_{2}=R_{2}^{\prime}$ and that $\rho_{1}$ is a bijection $R_{2} \rightarrow R_{1}^{\prime}$. Similarly, $\bar{C}_{1}=C_{1}^{\prime}$, and $\sigma_{1}$ is a bijection $C_{2} \rightarrow C_{1}^{\prime}$. In particular, $\left(R_{1}^{\prime}, C_{1}^{\prime}\right)$ is a minimal pair in $\mathcal{Q}$. + +Now, by inductive hypothesis again, we have $\left|R_{1}^{\prime}\right|=\left|R_{2}\right|,\left|C_{1}^{\prime}\right|=\left|C_{2}\right|$, and there exist four bijections + +$$ +\begin{array}{ccccc} +\rho_{1}^{\prime}: R_{2} \rightarrow R_{1}^{\prime} & \text { such that } & \rho_{1}^{\prime}\left(r_{2}\right) \equiv_{C_{1}^{\prime}} r_{2} & \text { for all } & r_{2} \in R_{2} ; \\ +\rho_{2}^{\prime}: R_{1}^{\prime} \rightarrow R_{2} & \text { such that } & \rho_{2}^{\prime}\left(r_{1}\right) \equiv_{C_{2}} r_{1} & \text { for all } & r_{1} \in R_{1}^{\prime} ; \\ +\sigma_{1}^{\prime}: C_{2} \rightarrow C_{1}^{\prime} & \text { such that } & \sigma_{1}^{\prime}\left(c_{2}\right) \equiv_{R_{1}^{\prime}} c_{2} & \text { for all } & c_{2} \in C_{2} ; \\ +\sigma_{2}^{\prime}: C_{1}^{\prime} \rightarrow C_{2} & \text { such that } & \sigma_{2}^{\prime}\left(c_{1}\right) \equiv_{R_{2}} c_{1} & \text { for all } & c_{1} \in C_{1}^{\prime} . +\end{array} +$$ + +Notice here that $\sigma_{1}$ and $\sigma_{1}^{\prime}$ are two bijections $C_{2} \rightarrow C_{1}^{\prime}$ satisfying $\sigma_{1}^{\prime}\left(c_{2}\right) \equiv_{R_{1}^{\prime}} c_{2} \geq_{R_{1}} \sigma_{1}\left(c_{2}\right)$ for all $c_{2} \in C_{2}$. Now, if $\sigma_{1}^{\prime}\left(c_{2}\right) \neq \sigma_{1}\left(c_{2}\right)$ for some $c_{2} \in C_{2}$, then we could remove column $\sigma_{1}^{\prime}\left(c_{2}\right)$ from $C_{1}^{\prime}$ obtaining another saddle pair $\left(R_{1}^{\prime}, C_{1}^{\prime} \backslash\left\{\sigma_{1}^{\prime}\left(c_{2}\right)\right\}\right)$ in $\mathcal{Q}$. This is impossible for a minimal pair $\left(R_{1}^{\prime}, C_{1}^{\prime}\right)$; hence the maps $\sigma_{1}$ and $\sigma_{1}^{\prime}$ coincide. + +Now we are prepared to show that $\left(R_{1}^{\prime}, C_{1}^{\prime}\right)$ is a saddle pair in $\mathcal{T}$, which yields a desired contradiction (since ( $R_{1}, C_{1}$ ) is not minimal). By symmetry, it suffices to find, for each $r^{\prime} \in \mathcal{R}$, a row $r_{1} \in R_{1}^{\prime}$ such that $r_{1} \geq_{C_{1}^{\prime}} r^{\prime}$. If $r^{\prime} \in R_{2}$, then we may put $r_{1}=\rho_{1}\left(r^{\prime}\right)$; so, in the sequel we assume $r^{\prime} \in R_{1}$. + +There exists $r_{2} \in R_{2}$ such that $r^{\prime} \leq_{C_{2}} r_{2}$; set $r_{1}=\left(\rho_{2}^{\prime}\right)^{-1}\left(r_{2}\right) \in R_{1}^{\prime}$ and recall that $r_{1} \equiv_{C_{2}}$ $r_{2} \geq_{C_{2}} r^{\prime}$. Therefore, implementing the bijection $\sigma_{1}=\sigma_{1}^{\prime}$, for each $c_{1} \in C_{1}^{\prime}$ we get + +$$ +a\left(r^{\prime}, c_{1}\right) \leqslant a\left(r^{\prime}, \sigma_{1}^{-1}\left(c_{1}\right)\right) \leqslant a\left(r_{1}, \sigma_{1}^{-1}\left(c_{1}\right)\right)=a\left(r_{1}, \sigma_{1}^{\prime} \circ \sigma_{1}^{-1}\left(c_{1}\right)\right)=a\left(r_{1}, c_{1}\right) +$$ + +which shows $r^{\prime} \leq_{C_{1}^{\prime}} r_{1}$, as desired. The inductive step is completed. +Comment 1. For two minimal pairs $\left(R_{1}, C_{1}\right)$ and $\left(R_{2}, C_{2}\right)$, Solution 2 not only proves the required equalities $\left|R_{1}\right|=\left|R_{2}\right|$ and $\left|C_{1}\right|=\left|C_{2}\right|$, but also shows the existence of bijections (3). In simple words, this means that the four subtables $\mathcal{T}\left[R_{1}, C_{1}\right], \mathcal{T}\left[R_{1}, C_{2}\right], \mathcal{T}\left[R_{2}, C_{1}\right]$, and $\mathcal{T}\left[R_{2}, C_{2}\right]$ differ only by permuting rows/columns. Notice that the existence of such bijections immediately implies that $\left(R_{1}, C_{2}\right)$ and $\left(R_{2}, C_{1}\right)$ are also minimal pairs. + +This stronger claim may also be derived directly from the arguments in Solution 1, even without the assumptions $R_{1} \cap R_{2}=\varnothing$ and $C_{1} \cap C_{2}=\varnothing$. Indeed, if $\left|R_{1}\right|=\left|R_{2}\right|$ and $\left|C_{1}\right|=\left|C_{2}\right|$, then similar arguments show that $R^{n}=R_{1}, C^{n}=C_{1}$, and for any $r \in R^{n}$ and $c \in C^{n}$ we have + +$$ +a(r, c)=a\left(\rho^{n k}(r), c\right) \geqslant a\left(\rho^{n k-1}(r), \sigma(c)\right) \geqslant \ldots \geqslant a\left(r, \sigma^{n k}(c)\right)=a(r, c) . +$$ + +This yields that all above inequalities turn into equalities. Moreover, this yields that all inequalities in (2) turn into equalities. Hence $\rho_{1}, \rho_{2}, \sigma_{1}$, and $\sigma_{2}$ satisfy (3). + +It is perhaps worth mentioning that one cannot necessarily find the maps in (3) so as to satisfy $\rho_{1}=\rho_{2}^{-1}$ and $\sigma_{1}=\sigma_{2}^{-1}$, as shown by the table below. + +| 1 | 0 | 0 | 1 | +| :--- | :--- | :--- | :--- | +| 0 | 1 | 1 | 0 | +| 1 | 0 | 1 | 0 | +| 0 | 1 | 0 | 1 | + +Comment 2. One may use the following, a bit more entertaining formulation of the same problem. +On a specialized market, a finite number of products are being sold, and there are finitely many retailers each selling all the products by some prices. Say that retailer $r_{1}$ dominates retailer $r_{2}$ with respect to a set of products $P$ if $r_{1}$ 's price of each $p \in P$ does not exceed $r_{2}$ 's price of $p$. Similarly, product $p_{1}$ exceeds product $p_{2}$ with respect to a set of retailers $R$, if $r$ 's price of $p_{1}$ is not less than $r$ 's price of $p_{2}$, for each $r \in R$. + +Say that a set $R$ of retailers and a set $P$ of products form a saddle pair if for each retailer $r^{\prime}$ there is $r \in R$ dominating $r^{\prime}$ with respect to $P$, and for each product $p^{\prime}$ there is $p \in P$ exceeding $p^{\prime}$ with respect to $R$. A saddle pair $(R, P)$ is called a minimal pair if for each saddle pair $\left(R^{\prime}, P^{\prime}\right)$ with $R^{\prime} \subseteq R$ and $P^{\prime} \subseteq P$, we have $R^{\prime}=R$ and $P^{\prime}=P$. + +Prove that any two minimal pairs contain the same number of retailers. + +## C8. Players $A$ and $B$ play a game on a blackboard that initially contains 2020 copies + +of the number 1. In every round, player $A$ erases two numbers $x$ and $y$ from the blackboard, and then player $B$ writes one of the numbers $x+y$ and $|x-y|$ on the blackboard. The game terminates as soon as, at the end of some round, one of the following holds: +(1) one of the numbers on the blackboard is larger than the sum of all other numbers; +(2) there are only zeros on the blackboard. + +Player $B$ must then give as many cookies to player $A$ as there are numbers on the blackboard. Player $A$ wants to get as many cookies as possible, whereas player $B$ wants to give as few as possible. Determine the number of cookies that $A$ receives if both players play optimally. +(Austria) + +## Answer: 7. + +Solution. For a positive integer $n$, we denote by $S_{2}(n)$ the sum of digits in its binary representation. We prove that, in fact, if a board initially contains an even number $n>1$ of ones, then A can guarantee to obtain $S_{2}(n)$, but not more, cookies. The binary representation of 2020 is $2020=\overline{11111100100}_{2}$, so $S_{2}(2020)=7$, and the answer follows. + +A strategy for $A$. At any round, while possible, A chooses two equal nonzero numbers on the board. Clearly, while $A$ can make such choice, the game does not terminate. On the other hand, $A$ can follow this strategy unless the game has already terminated. Indeed, if $A$ always chooses two equal numbers, then each number appearing on the board is either 0 or a power of 2 with non-negative integer exponent, this can be easily proved using induction on the number of rounds. At the moment when $A$ is unable to follow the strategy all nonzero numbers on the board are distinct powers of 2 . If the board contains at least one such power, then the largest of those powers is greater than the sum of the others. Otherwise there are only zeros on the blackboard, in both cases the game terminates. + +For every number on the board, define its range to be the number of ones it is obtained from. We can prove by induction on the number of rounds that for any nonzero number $k$ written by $B$ its range is $k$, and for any zero written by $B$ its range is a power of 2 . Thus at the end of each round all the ranges are powers of two, and their sum is $n$. Since $S_{2}(a+b) \leqslant S_{2}(a)+S_{2}(b)$ for any positive integers $a$ and $b$, the number $n$ cannot be represented as a sum of less than $S_{2}(n)$ powers of 2 . Thus at the end of each round the board contains at least $S_{2}(n)$ numbers, while $A$ follows the above strategy. So $A$ can guarantee at least $S_{2}(n)$ cookies for himself. + +Comment. There are different proofs of the fact that the presented strategy guarantees at least $S_{2}(n)$ cookies for $A$. For instance, one may denote by $\Sigma$ the sum of numbers on the board, and by $z$ the number of zeros. Then the board contains at least $S_{2}(\Sigma)+z$ numbers; on the other hand, during the game, the number $S_{2}(\Sigma)+z$ does not decrease, and its initial value is $S_{2}(n)$. The claim follows. + +A strategy for $B$. Denote $s=S_{2}(n)$. +Let $x_{1}, \ldots, x_{k}$ be the numbers on the board at some moment of the game after $B$ 's turn or at the beginning of the game. Say that a collection of $k$ signs $\varepsilon_{1}, \ldots, \varepsilon_{k} \in\{+1,-1\}$ is balanced if + +$$ +\sum_{i=1}^{k} \varepsilon_{i} x_{i}=0 +$$ + +We say that a situation on the board is good if $2^{s+1}$ does not divide the number of balanced collections. An appropriate strategy for $B$ can be explained as follows: Perform a move so that the situation remains good, while it is possible. We intend to show that in this case $B$ will not lose more than $S_{2}(n)$ cookies. For this purpose, we prove several lemmas. + +For a positive integer $k$, denote by $\nu_{2}(k)$ the exponent of the largest power of 2 that divides $k$. Recall that, by Legendre's formula, $\nu_{2}(n!)=n-S_{2}(n)$ for every positive integer $n$. + +Lemma 1. The initial situation is good. +Proof. In the initial configuration, the number of balanced collections is equal to $\binom{n}{n / 2}$. We have + +$$ +\nu_{2}\left(\binom{n}{n / 2}\right)=\nu_{2}(n!)-2 \nu_{2}((n / 2)!)=\left(n-S_{2}(n)\right)-2\left(\frac{n}{2}-S_{2}(n / 2)\right)=S_{2}(n)=s +$$ + +Hence $2^{\text {s+1 }}$ does not divide the number of balanced collections, as desired. +Lemma 2. B may play so that after each round the situation remains good. +Proof. Assume that the situation $\left(x_{1}, \ldots, x_{k}\right)$ before a round is good, and that $A$ erases two numbers, $x_{p}$ and $x_{q}$. + +Let $N$ be the number of all balanced collections, $N_{+}$be the number of those having $\varepsilon_{p}=\varepsilon_{q}$, and $N_{-}$be the number of other balanced collections. Then $N=N_{+}+N_{-}$. Now, if $B$ replaces $x_{p}$ and $x_{q}$ by $x_{p}+x_{q}$, then the number of balanced collections will become $N_{+}$. If $B$ replaces $x_{p}$ and $x_{q}$ by $\left|x_{p}-x_{q}\right|$, then this number will become $N_{-}$. Since $2^{s+1}$ does not divide $N$, it does not divide one of the summands $N_{+}$and $N_{-}$, hence $B$ can reach a good situation after the round. +Lemma 3. Assume that the game terminates at a good situation. Then the board contains at most $s$ numbers. +Proof. Suppose, one of the numbers is greater than the sum of the other numbers. Then the number of balanced collections is 0 and hence divisible by $2^{s+1}$. Therefore, the situation is not good. + +Then we have only zeros on the blackboard at the moment when the game terminates. If there are $k$ of them, then the number of balanced collections is $2^{k}$. Since the situation is good, we have $k \leqslant s$. + +By Lemmas 1 and 2, $B$ may act in such way that they keep the situation good. By Lemma 3, when the game terminates, the board contains at most $s$ numbers. This is what we aimed to prove. + +Comment 1. If the initial situation had some odd number $n>1$ of ones on the blackboard, player $A$ would still get $S_{2}(n)$ cookies, provided that both players act optimally. The proof of this fact is similar to the solution above, after one makes some changes in the definitions. Such changes are listed below. + +Say that a collection of $k$ signs $\varepsilon_{1}, \ldots, \varepsilon_{k} \in\{+1,-1\}$ is positive if + +$$ +\sum_{i=1}^{k} \varepsilon_{i} x_{i}>0 +$$ + +For every index $i=1,2, \ldots, k$, we denote by $N_{i}$ the number of positive collections such that $\varepsilon_{i}=1$. Finally, say that a situation on the board is good if $2^{s-1}$ does not divide at least one of the numbers $N_{i}$. Now, a strategy for $B$ again consists in preserving the situation good, after each round. + +Comment 2. There is an easier strategy for $B$, allowing, in the game starting with an even number $n$ of ones, to lose no more than $d=\left\lfloor\log _{2}(n+2)\right\rfloor-1$ cookies. If the binary representation of $n$ contains at most two zeros, then $d=S_{2}(n)$, and hence the strategy is optimal in that case. We outline this approach below. + +First of all, we can assume that $A$ never erases zeros from the blackboard. Indeed, $A$ may skip such moves harmlessly, ignoring the zeros in the further process; this way, $A$ 's win will just increase. + +We say that a situation on the blackboard is pretty if the numbers on the board can be partitioned into two groups with equal sums. Clearly, if the situation before some round is pretty, then $B$ may play so as to preserve this property after the round. The strategy for $B$ is as follows: + +- $B$ always chooses a move that leads to a pretty situation. +- If both possible moves of $B$ lead to pretty situations, then $B$ writes the sum of the two numbers erased by $A$. +Since the situation always remains pretty, the game terminates when all numbers on the board are zeros. + +Suppose that, at the end of the game, there are $m \geqslant d+1=\left\lfloor\log _{2}(n+2)\right\rfloor$ zeros on the board; then $2^{m}-1>n / 2$. + +Now we analyze the whole process of the play. Let us number the zeros on the board in order of appearance. During the play, each zero had appeared after subtracting two equal numbers. Let $n_{1}, \ldots, n_{m}$ be positive integers such that the first zero appeared after subtracting $n_{1}$ from $n_{1}$, the second zero appeared after subtracting $n_{2}$ from $n_{2}$, and so on. Since the sum of the numbers on the blackboard never increases, we have $2 n_{1}+\ldots+2 n_{m} \leqslant n$, hence + +$$ +n_{1}+\ldots+n_{m} \leqslant n / 2<2^{m}-1 +$$ + +There are $2^{m}$ subsets of the set $\{1,2, \ldots, m\}$. For $I \subseteq\{1,2, \ldots, m\}$, denote by $f(I)$ the sum $\sum_{i \in I} n_{i}$. There are less than $2^{m}$ possible values for $f(I)$, so there are two distinct subsets $I$ and $J$ with $f(I)=f(J)$. Replacing $I$ and $J$ with $I \backslash J$ and $J \backslash I$, we assume that $I$ and $J$ are disjoint. + +Let $i_{0}$ be the smallest number in $I \cup J$; without loss of generality, $i_{0} \in I$. Consider the round when $A$ had erased two numbers equal to $n_{i_{0}}$, and $B$ had put the $i_{0}{ }^{\text {th }}$ zero instead, and the situation before that round. + +For each nonzero number $z$ which is on the blackboard at this moment, we can keep track of it during the further play until it enters one of the numbers $n_{i}, i \geqslant i_{0}$, which then turn into zeros. For every $i=i_{0}, i_{0}+1, \ldots, m$, we denote by $X_{i}$ the collection of all numbers on the blackboard that finally enter the first copy of $n_{i}$, and by $Y_{i}$ the collection of those finally entering the second copy of $n_{i}$. Thus, each of $X_{i_{0}}$ and $Y_{i_{0}}$ consists of a single number. Since $A$ never erases zeros, the 2(m-iol $i_{0}$ ) defined sets are pairwise disjoint. + +Clearly, for either of the collections $X_{i}$ and $Y_{i}$, a signed sum of its elements equals $n_{i}$, for a proper choice of the signs. Therefore, for every $i=i_{0}, i_{0}+1, \ldots, m$ one can endow numbers in $X_{i} \cup Y_{i}$ with signs so that their sum becomes any of the numbers $-2 n_{i}, 0$, or $2 n_{i}$. Perform such endowment so as to get $2 n_{i}$ from each collection $X_{i} \cup Y_{i}$ with $i \in I,-2 n_{j}$ from each collection $X_{j} \cup Y_{j}$ with $j \in J$, and 0 from each remaining collection. The obtained signed sum of all numbers on the blackboard equals + +$$ +\sum_{i \in I} 2 n_{i}-\sum_{i \in J} 2 n_{i}=0 +$$ + +and the numbers in $X_{i_{0}}$ and $Y_{i_{0}}$ have the same (positive) sign. +This means that, at this round, $B$ could add up the two numbers $n_{i_{0}}$ to get a pretty situation. According to the strategy, $B$ should have performed that, instead of subtracting the numbers. This contradiction shows that $m \leqslant d$, as desired. + +This page is intentionally left blank + +## Geometry + +G1. Let $A B C$ be an isosceles triangle with $B C=C A$, and let $D$ be a point inside side $A B$ such that $A D90^{\circ}, \angle C D A>90^{\circ}$, and + +$\angle D A B=\angle B C D$. Denote by $E$ and $F$ the reflections of $A$ in lines $B C$ and $C D$, respectively. Suppose that the segments $A E$ and $A F$ meet the line $B D$ at $K$ and $L$, respectively. Prove that the circumcircles of triangles $B E K$ and $D F L$ are tangent to each other. +(Slovakia) +Solution 1. Denote by $A^{\prime}$ the reflection of $A$ in $B D$. We will show that that the quadrilaterals $A^{\prime} B K E$ and $A^{\prime} D L F$ are cyclic, and their circumcircles are tangent to each other at point $A^{\prime}$. + +From the symmetry about line $B C$ we have $\angle B E K=\angle B A K$, while from the symmetry in $B D$ we have $\angle B A K=\angle B A^{\prime} K$. Hence $\angle B E K=\angle B A^{\prime} K$, which implies that the quadrilateral $A^{\prime} B K E$ is cyclic. Similarly, the quadrilateral $A^{\prime} D L F$ is also cyclic. +![](https://cdn.mathpix.com/cropped/2024_11_18_bcdd154f2030efbe2fa4g-54.jpg?height=632&width=1229&top_left_y=818&top_left_x=419) + +For showing that circles $A^{\prime} B K E$ and $A^{\prime} D L F$ are tangent it suffices to prove that + +$$ +\angle A^{\prime} K B+\angle A^{\prime} L D=\angle B A^{\prime} D . +$$ + +Indeed, by $A K \perp B C$, $A L \perp C D$, and again the symmetry in $B D$ we have + +$$ +\angle A^{\prime} K B+\angle A^{\prime} L D=180^{\circ}-\angle K A^{\prime} L=180^{\circ}-\angle K A L=\angle B C D=\angle B A D=\angle B A^{\prime} D, +$$ + +as required. +Comment 1. The key to the solution above is introducing the point $A^{\prime}$; then the angle calculations can be done in many different ways. + +Solution 2. Note that $\angle K A L=180^{\circ}-\angle B C D$, since $A K$ and $A L$ are perpendicular to $B C$ and $C D$, respectively. Reflect both circles $(B E K)$ and $(D F L)$ in $B D$. Since $\angle K E B=\angle K A B$ and $\angle D F L=\angle D A L$, the images are the circles $(K A B)$ and $(L A D)$, respectively; so they meet at $A$. We need to prove that those two reflections are tangent at $A$. + +For this purpose, we observe that + +$$ +\angle A K B+\angle A L D=180^{\circ}-\angle K A L=\angle B C D=\angle B A D . +$$ + +Thus, there exists a ray $A P$ inside angle $\angle B A D$ such that $\angle B A P=\angle A K B$ and $\angle D A P=$ $\angle D L A$. Hence the line $A P$ is a common tangent to the circles $(K A B)$ and $(L A D)$, as desired. + +Comment 2. The statement of the problem remains true for a more general configuration, e.g., if line $B D$ intersect the extension of segment $A E$ instead of the segment itself, etc. The corresponding restrictions in the statement are given to reduce case sensitivity. + +G4. In the plane, there are $n \geqslant 6$ pairwise disjoint disks $D_{1}, D_{2}, \ldots, D_{n}$ with radii $R_{1} \geqslant R_{2} \geqslant \ldots \geqslant R_{n}$. For every $i=1,2, \ldots, n$, a point $P_{i}$ is chosen in disk $D_{i}$. Let $O$ be an arbitrary point in the plane. Prove that + +$$ +O P_{1}+O P_{2}+\ldots+O P_{n} \geqslant R_{6}+R_{7}+\ldots+R_{n} +$$ + +(A disk is assumed to contain its boundary.) +(Iran) +Solution. We will make use of the following lemma. +Lemma. Let $D_{1}, \ldots, D_{6}$ be disjoint disks in the plane with radii $R_{1}, \ldots, R_{6}$. Let $P_{i}$ be a point in $D_{i}$, and let $O$ be an arbitrary point. Then there exist indices $i$ and $j$ such that $O P_{i} \geqslant R_{j}$. Proof. Let $O_{i}$ be the center of $D_{i}$. Consider six rays $O O_{1}, \ldots, O O_{6}$ (if $O=O_{i}$, then the ray $O O_{i}$ may be assumed to have an arbitrary direction). These rays partition the plane into six angles (one of which may be non-convex) whose measures sum up to $360^{\circ}$; hence one of the angles, say $\angle O_{i} O O_{j}$, has measure at most $60^{\circ}$. Then $O_{i} O_{j}$ cannot be the unique largest side in (possibly degenerate) triangle $O O_{i} O_{j}$, so, without loss of generality, $O O_{i} \geqslant O_{i} O_{j} \geqslant R_{i}+R_{j}$. Therefore, $O P_{i} \geqslant O O_{i}-R_{i} \geqslant\left(R_{i}+R_{j}\right)-R_{i}=R_{j}$, as desired. + +Now we prove the required inequality by induction on $n \geqslant 5$. The base case $n=5$ is trivial. For the inductive step, apply the Lemma to the six largest disks, in order to find indices $i$ and $j$ such that $1 \leqslant i, j \leqslant 6$ and $O P_{i} \geqslant R_{j} \geqslant R_{6}$. Removing $D_{i}$ from the configuration and applying the inductive hypothesis, we get + +$$ +\sum_{k \neq i} O P_{k} \geqslant \sum_{\ell \geqslant 7} R_{\ell} . +$$ + +Adding up this inequality with $O P_{i} \geqslant R_{6}$ we establish the inductive step. +Comment 1. It is irrelevant to the problem whether the disks contain their boundaries or not. This condition is included for clarity reasons only. The problem statement remains true, and the solution works verbatim, if the disks are assumed to have disjoint interiors. + +Comment 2. There are several variations of the above solution. In particular, while performing the inductive step, one may remove the disk with the largest value of $O P_{i}$ and apply the inductive hypothesis to the remaining disks (the Lemma should still be applied to the six largest disks). + +Comment 3. While proving the Lemma, one may reduce it to a particular case when the disks are congruent, as follows: Choose the smallest radius $r$ of the disks in the Lemma statement, and then replace, for each $i$, the $i^{\text {th }}$ disk with its homothetic copy, using the homothety centered at $P_{i}$ with ratio $r / R_{i}$. + +This argument shows that the Lemma is tightly connected to a circle packing problem, see, e.g., https://en.wikipedia.org/wiki/Circle_packing_in_a_circle. The known results on that problem provide versions of the Lemma for different numbers of disks, which lead to different inequalities of the same kind. E.g., for 4 disks the best possible estimate in the Lemma is $O P_{i} \geqslant(\sqrt{2}-1) R_{j}$, while for 13 disks it has the form $O P_{i} \geqslant \sqrt{5} R_{j}$. Arguing as in the above solution, one obtains the inequalities + +$$ +\sum_{i=1}^{n} O P_{i} \geqslant(\sqrt{2}-1) \sum_{j=4}^{n} R_{j} \quad \text { and } \quad \sum_{i=1}^{n} O P_{i} \geqslant \sqrt{5} \sum_{j=13}^{n} R_{j} . +$$ + +However, there are some harder arguments which allow to improve these inequalities, meaning that the $R_{j}$ with large indices may be taken with much greater factors. + +## G5. Let $A B C D$ be a cyclic quadrilateral with no two sides parallel. Let $K, L, M$, and $N$ + +be points lying on sides $A B, B C, C D$, and $D A$, respectively, such that $K L M N$ is a rhombus with $K L \| A C$ and $L M \| B D$. Let $\omega_{1}, \omega_{2}, \omega_{3}$, and $\omega_{4}$ be the incircles of triangles $A N K$, $B K L, C L M$, and $D M N$, respectively. Prove that the internal common tangents to $\omega_{1}$ and $\omega_{3}$ and the internal common tangents to $\omega_{2}$ and $\omega_{4}$ are concurrent. +(Poland) +Solution 1. Let $I_{i}$ be the center of $\omega_{i}$, and let $r_{i}$ be its radius for $i=1,2,3,4$. Denote by $T_{1}$ and $T_{3}$ the points of tangency of $\omega_{1}$ and $\omega_{3}$ with $N K$ and $L M$, respectively. Suppose that the internal common tangents to $\omega_{1}$ and $\omega_{3}$ meet at point $S$, which is the center of homothety $h$ with negative ratio (namely, with ratio $-\frac{r_{3}}{r_{1}}$ ) mapping $\omega_{1}$ to $\omega_{3}$. This homothety takes $T_{1}$ to $T_{3}$ (since the tangents to $\omega_{1}$ and $\omega_{3}$ at $T_{1}$ to $T_{3}$ are parallel), hence $S$ is a point on the segment $T_{1} T_{3}$ with $T_{1} S: S T_{3}=r_{1}: r_{3}$. + +Construct segments $S_{1} S_{3} \| K L$ and $S_{2} S_{4} \| L M$ through $S$ with $S_{1} \in N K, S_{2} \in K L$, $S_{3} \in L M$, and $S_{4} \in M N$. Note that $h$ takes $S_{1}$ to $S_{3}$, hence $I_{1} S_{1} \| I_{3} S_{3}$, and $S_{1} S: S S_{3}=r_{1}: r_{3}$. We will prove that $S_{2} S: S S_{4}=r_{2}: r_{4}$ or, equivalently, $K S_{1}: S_{1} N=r_{2}: r_{4}$. This will yield the problem statement; indeed, applying similar arguments to the intersection point $S^{\prime}$ of the internal common tangents to $\omega_{2}$ and $\omega_{4}$, we see that $S^{\prime}$ satisfies similar relations, and there is a unique point inside $K L M N$ satisfying them. Therefore, $S^{\prime}=S$. +![](https://cdn.mathpix.com/cropped/2024_11_18_bcdd154f2030efbe2fa4g-56.jpg?height=670&width=1409&top_left_y=1207&top_left_x=326) + +Further, denote by $I_{A}, I_{B}, I_{C}, I_{D}$ and $r_{A}, r_{B}, r_{C}, r_{D}$ the incenters and inradii of triangles $D A B, A B C, B C D$, and $C D A$, respectively. One can shift triangle $C L M$ by $\overrightarrow{L K}$ to glue it with triangle $A K N$ into a quadrilateral $A K C^{\prime} N$ similar to $A B C D$. In particular, this shows that $r_{1}: r_{3}=r_{A}: r_{C}$; similarly, $r_{2}: r_{4}=r_{B}: r_{D}$. Moreover, the same shift takes $S_{3}$ to $S_{1}$, and it also takes $I_{3}$ to the incenter $I_{3}^{\prime}$ of triangle $K C^{\prime} N$. Since $I_{1} S_{1} \| I_{3} S_{3}$, the points $I_{1}, S_{1}, I_{3}^{\prime}$ are collinear. Thus, in order to complete the solution, it suffices to apply the following Lemma to quadrilateral $A K C^{\prime} N$. +Lemma 1. Let $A B C D$ be a cyclic quadrilateral, and define $I_{A}, I_{C}, r_{B}$, and $r_{D}$ as above. Let $I_{A} I_{C}$ meet $B D$ at $X$; then $B X: X D=r_{B}: r_{D}$. +Proof. Consider an inversion centered at $X$; the images under that inversion will be denoted by primes, e.g., $A^{\prime}$ is the image of $A$. + +By properties of inversion, we have + +$$ +\angle I_{C}^{\prime} I_{A}^{\prime} D^{\prime}=\angle X I_{A}^{\prime} D^{\prime}=\angle X D I_{A}=\angle B D A / 2=\angle B C A / 2=\angle A C I_{B} +$$ + +We obtain $\angle I_{A}^{\prime} I_{C}^{\prime} D^{\prime}=\angle C A I_{B}$ likewise; therefore, $\triangle I_{C}^{\prime} I_{A}^{\prime} D^{\prime} \sim \triangle A C I_{B}$. In the same manner, we get $\triangle I_{C}^{\prime} I_{A}^{\prime} B^{\prime} \sim \triangle A C I_{D}$, hence the quadrilaterals $I_{C}^{\prime} B^{\prime} I_{A}^{\prime} D^{\prime}$ and $A I_{D} C I_{B}$ are also similar. But the diagonals $A C$ and $I_{B} I_{D}$ of quadrilateral $A I_{D} C I_{B}$ meet at a point $Y$ such that $I_{B} Y$ : +$Y I_{D}=r_{B}: r_{D}$. By similarity, we get $D^{\prime} X: B^{\prime} X=r_{B}: r_{D}$ and hence $B X: X D=D^{\prime} X:$ $B^{\prime} X=r_{B}: r_{D}$. + +Comment 1. The solution above shows that the problem statement holds also for any parallelogram $K L M N$ whose sides are parallel to the diagonals of $A B C D$, as no property specific for a rhombus has been used. This solution works equally well when two sides of quadrilateral $A B C D$ are parallel. + +Comment 2. The problem may be reduced to Lemma 1 by using different tools, e.g., by using mass point geometry, linear motion of $K, L, M$, and $N$, etc. + +Lemma 1 itself also can be proved in different ways. We present below one alternative proof. +Proof. In the circumcircle of $A B C D$, let $K^{\prime}, L^{\prime} . M^{\prime}$, and $N^{\prime}$ be the midpoints of arcs $A B, B C$, $C D$, and $D A$ containing no other vertices of $A B C D$, respectively. Thus, $K^{\prime}=C I_{B} \cap D I_{A}$, etc. In the computations below, we denote by $[P]$ the area of a polygon $P$. We use similarities $\triangle I_{A} B K^{\prime} \sim$ $\triangle I_{A} D N^{\prime}, \triangle I_{B} K^{\prime} L^{\prime} \sim \triangle I_{B} A C$, etc., as well as congruences $\triangle I_{B} K^{\prime} L^{\prime}=\triangle B K^{\prime} L^{\prime}$ and $\triangle I_{D} M^{\prime} N^{\prime}=$ $\triangle D M^{\prime} N^{\prime}$ (e.g., the first congruence holds because $K^{\prime} L^{\prime}$ is a common internal bisector of angles $B K^{\prime} I_{B}$ and $B L^{\prime} I_{B}$ ). + +We have + +$$ +\begin{aligned} +& \frac{B X}{D X}=\frac{\left[I_{A} B I_{C}\right]}{\left[I_{A} D I_{C}\right]}=\frac{B I_{A} \cdot B I_{C} \cdot \sin I_{A} B I_{C}}{D I_{A} \cdot D I_{C} \cdot \sin I_{A} D I_{C}}=\frac{B I_{A}}{D I_{A}} \cdot \frac{B I_{C}}{D I_{C}} \cdot \frac{\sin N^{\prime} B M^{\prime}}{\sin K^{\prime} D L^{\prime}} \\ +& =\frac{B K^{\prime}}{D N^{\prime}} \cdot \frac{B L^{\prime}}{D M^{\prime}} \cdot \frac{\sin N^{\prime} D M^{\prime}}{\sin K^{\prime} B L^{\prime}}=\frac{B K^{\prime} \cdot B L^{\prime} \cdot \sin K^{\prime} B L^{\prime}}{D N^{\prime} \cdot D M^{\prime} \cdot \sin N^{\prime} D M^{\prime}} \cdot \frac{\sin ^{2} N^{\prime} D M^{\prime}}{\sin ^{2} K^{\prime} B L^{\prime}} \\ +& \quad=\frac{\left[K^{\prime} B L^{\prime}\right]}{\left[M^{\prime} D N^{\prime}\right]} \cdot \frac{N^{\prime} M^{\prime 2}}{K^{\prime} L^{\prime 2}}=\frac{\left[K^{\prime} I_{B} L^{\prime}\right] \cdot \frac{A^{\prime} C^{\prime 2}}{K^{\prime} L^{\prime 2}}}{\left[M^{\prime} I_{D} N^{\prime}\right] \cdot \frac{A^{\prime} C^{\prime 2}}{N^{\prime} M^{\prime 2}}}=\frac{\left[A I_{B} C\right]}{\left[A I_{D} C\right]}=\frac{r_{B}}{r_{D}}, +\end{aligned} +$$ + +as required. +Solution 2. This solution is based on the following general Lemma. +Lemma 2. Let $E$ and $F$ be distinct points, and let $\omega_{i}, i=1,2,3,4$, be circles lying in the same halfplane with respect to $E F$. For distinct indices $i, j \in\{1,2,3,4\}$, denote by $O_{i j}^{+}$ (respectively, $O_{i j}^{-}$) the center of homothety with positive (respectively, negative) ratio taking $\omega_{i}$ to $\omega_{j}$. Suppose that $E=O_{12}^{+}=O_{34}^{+}$and $F=O_{23}^{+}=O_{41}^{+}$. Then $O_{13}^{-}=O_{24}^{-}$. +Proof. Applying Monge's theorem to triples of circles $\omega_{1}, \omega_{2}, \omega_{4}$ and $\omega_{1}, \omega_{3}, \omega_{4}$, we get that both points $O_{24}^{-}$and $O_{13}^{-}$lie on line $E O_{14}^{-}$. Notice that this line is distinct from $E F$. Similarly we obtain that both points $O_{24}^{-}$and $O_{13}^{-}$lie on $F O_{34}^{-}$. Since the lines $E O_{14}^{-}$and $F O_{34}^{-}$are distinct, both points coincide with the meeting point of those lines. +![](https://cdn.mathpix.com/cropped/2024_11_18_bcdd154f2030efbe2fa4g-57.jpg?height=716&width=1837&top_left_y=2073&top_left_x=108) + +Turning back to the problem, let $A B$ intersect $C D$ at $E$ and let $B C$ intersect $D A$ at $F$. Assume, without loss of generality, that $B$ lies on segments $A E$ and $C F$. We will show that the points $E$ and $F$, and the circles $\omega_{i}$ satisfy the conditions of Lemma 2 , so the problem statement follows. In the sequel, we use the notation of $O_{i j}^{ \pm}$from the statement of Lemma 2, applied to circles $\omega_{1}, \omega_{2}, \omega_{3}$, and $\omega_{4}$. + +Using the relations $\triangle E C A \sim \triangle E B D, K N \| B D$, and $M N \| A C$. we get + +$$ +\frac{A N}{N D}=\frac{A N}{A D} \cdot \frac{A D}{N D}=\frac{K N}{B D} \cdot \frac{A C}{N M}=\frac{A C}{B D}=\frac{A E}{E D} +$$ + +Therefore, by the angle bisector theorem, point $N$ lies on the internal angle bisector of $\angle A E D$. We prove similarly that $L$ also lies on that bisector, and that the points $K$ and $M$ lie on the internal angle bisector of $\angle A F B$. + +Since $K L M N$ is a rhombus, points $K$ and $M$ are symmetric in line $E L N$. Hence, the convex quadrilateral determined by the lines $E K, E M, K L$, and $M L$ is a kite, and therefore it has an incircle $\omega_{0}$. Applying Monge's theorem to $\omega_{0}, \omega_{2}$, and $\omega_{3}$, we get that $O_{23}^{+}$lies on $K M$. On the other hand, $O_{23}^{+}$lies on $B C$, as $B C$ is an external common tangent to $\omega_{2}$ and $\omega_{3}$. It follows that $F=O_{23}^{+}$. Similarly, $E=O_{12}^{+}=O_{34}^{+}$, and $F=O_{41}^{+}$. + +Comment 3. The reduction to Lemma 2 and the proof of Lemma 2 can be performed with the use of different tools, e.g., by means of Menelaus theorem, by projecting harmonic quadruples, by applying Monge's theorem in some other ways, etc. + +This page is intentionally left blank + +## G6. Let $I$ and $I_{A}$ be the incenter and the $A$-excenter of an acute-angled triangle $A B C$ + +with $A B(t-1) \frac{\sqrt{3}}{2}$, or + +$$ +t<1+\frac{4 \sqrt{M}}{\sqrt{3}}<4 \sqrt{M} +$$ + +as $M \geqslant 1$. +Combining the estimates (1), (2), and (3), we finally obtain + +$$ +\frac{1}{4 \delta} \leqslant t<4 \sqrt{M}<4 \sqrt{2 n \delta}, \quad \text { or } \quad 512 n \delta^{3}>1 +$$ + +which does not hold for the chosen value of $\delta$. +Comment 1. As the proposer mentions, the exponent $-1 / 3$ in the problem statement is optimal. In fact, for any $n \geqslant 2$, there is a configuration $\mathcal{S}$ of $n$ points in the plane such that any two points in $\mathcal{S}$ are at least 1 apart, but every line $\ell$ separating $\mathcal{S}$ is at most $c^{\prime} n^{-1 / 3} \log n$ apart from some point in $\mathcal{S}$; here $c^{\prime}$ is some absolute constant. + +The original proposal suggested to prove the estimate of the form $\mathrm{cn}^{-1 / 2}$. That version admits much easier solutions. E.g., setting $\delta=\frac{1}{16} n^{-1 / 2}$ and applying (1), we see that $\mathcal{S}$ is contained in a disk $D$ of radius $\frac{1}{8} n^{1 / 2}$. On the other hand, for each point $X$ of $\mathcal{S}$, let $D_{X}$ be the disk of radius $\frac{1}{2}$ centered at $X$; all these disks have disjoint interiors and lie within the disk concentric to $D$, of radius $\frac{1}{16} n^{1 / 2}+\frac{1}{2}<\frac{1}{2} n^{1 / 2}$. Comparing the areas, we get + +$$ +n \cdot \frac{\pi}{4} \leqslant \pi\left(\frac{n^{1 / 2}}{16}+\frac{1}{2}\right)^{2}<\frac{\pi n}{4} +$$ + +which is a contradiction. +The Problem Selection Committee decided to choose a harder version for the Shortlist. +Comment 2. In this comment, we discuss some versions of the solution above, which avoid concentrating on the diameter of $\mathcal{S}$. We start with introducing some terminology suitable for those versions. + +Put $\delta=c n^{-1 / 3}$ for a certain sufficiently small positive constant $c$. For the sake of contradiction, suppose that, for some set $\mathcal{S}$ satisfying the conditions in the problem statement, there is no separating line which is at least $\delta$ apart from each point of $\mathcal{S}$. + +Let $C$ be the convex hull of $\mathcal{S}$. A line is separating if and only if it meets $C$ (we assume that a line passing through a point of $\mathcal{S}$ is always separating). Consider a strip between two parallel separating lines $a$ and $a^{\prime}$ which are, say, $\frac{1}{4}$ apart from each other. Define a slice determined by the strip as the intersection of $\mathcal{S}$ with the strip. The length of the slice is the diameter of the projection of the slice to $a$. + +In this terminology, the arguments used in the proofs of (2) and (3) show that for any slice $\mathcal{T}$ of length $L$, we have + +$$ +\frac{1}{8 \delta} \leqslant|\mathcal{T}| \leqslant 1+\frac{4}{\sqrt{15}} L +$$ + +The key idea of the solution is to apply these estimates to a peel slice, where line $a$ does not cross the interior of $C$. In the above solution, this idea was applied to one carefully chosen peel slice. Here, we outline some different approach involving many of them. We always assume that $n$ is sufficiently large. + +Consider a peel slice determined by lines $a$ and $a^{\prime}$, where $a$ contains no interior points of $C$. We orient $a$ so that $C$ lies to the left of $a$. Line $a$ is called a supporting line of the slice, and the obtained direction is the direction of the slice; notice that the direction determines uniquely the supporting line and hence the slice. Fix some direction $\mathbf{v}_{0}$, and for each $\alpha \in[0,2 \pi)$ denote by $\mathcal{T}_{\alpha}$ the peel slice whose direction is $\mathbf{v}_{0}$ rotated by $\alpha$ counterclockwise. + +When speaking about the slice, we always assume that the figure is rotated so that its direction is vertical from the bottom to the top; then the points in $\mathcal{T}$ get a natural order from the bottom to the top. In particular, we may speak about the top half $\mathrm{T}(\mathcal{T})$ consisting of $\lfloor|\mathcal{T}| / 2\rfloor$ topmost points in $\mathcal{T}$, and similarly about its bottom half $\mathrm{B}(\mathcal{T})$. By (4), each half contains at least 10 points when $n$ is large. Claim. Consider two angles $\alpha, \beta \in[0, \pi / 2]$ with $\beta-\alpha \geqslant 40 \delta=: \phi$. Then all common points of $\mathcal{T}_{\alpha}$ and $\mathcal{T}_{\beta}$ lie in $\mathrm{T}\left(\mathcal{T}_{\alpha}\right) \cap \mathrm{B}\left(\mathcal{T}_{\beta}\right)$. +![](https://cdn.mathpix.com/cropped/2024_11_18_bcdd154f2030efbe2fa4g-71.jpg?height=738&width=1338&top_left_y=201&top_left_x=366) + +Proof. By symmetry, it suffices to show that all those points lie in $\mathrm{T}\left(\mathcal{T}_{\alpha}\right)$. Let $a$ be the supporting line of $\mathcal{T}_{\alpha}$, and let $\ell$ be a line perpendicular to the direction of $\mathcal{T}_{\beta}$. Let $P_{1}, \ldots, P_{k}$ list all points in $\mathcal{T}_{\alpha}$, numbered from the bottom to the top; by (4), we have $k \geqslant \frac{1}{8} \delta^{-1}$. + +Introduce the Cartesian coordinates so that the (oriented) line $a$ is the $y$-axis. Let $P_{i}$ be any point in $\mathrm{B}\left(\mathcal{T}_{\alpha}\right)$. The difference of ordinates of $P_{k}$ and $P_{i}$ is at least $\frac{\sqrt{15}}{4}(k-i)>\frac{1}{3} k$, while their abscissas differ by at most $\frac{1}{4}$. This easily yields that the projections of those points to $\ell$ are at least + +$$ +\frac{k}{3} \sin \phi-\frac{1}{4} \geqslant \frac{1}{24 \delta} \cdot 20 \delta-\frac{1}{4}>\frac{1}{4} +$$ + +apart from each other, and $P_{k}$ is closer to the supporting line of $\mathcal{T}_{\beta}$ than $P_{i}$, so that $P_{i}$ does not belong to $\mathcal{T}_{\beta}$. + +Now, put $\alpha_{i}=40 \delta i$, for $i=0,1, \ldots,\left\lfloor\frac{1}{40} \delta^{-1} \cdot \frac{\pi}{2}\right\rfloor$, and consider the slices $\mathcal{T}_{\alpha_{i}}$. The Claim yields that each point in $\mathcal{S}$ is contained in at most two such slices. Hence, the union $\mathcal{U}$ of those slices contains at least + +$$ +\frac{1}{2} \cdot \frac{1}{8 \delta} \cdot \frac{1}{40 \delta} \cdot \frac{\pi}{2}=\frac{\lambda}{\delta^{2}} +$$ + +points (for some constant $\lambda$ ), and each point in $\mathcal{U}$ is at most $\frac{1}{4}$ apart from the boundary of $C$. +It is not hard now to reach a contradiction with (1). E.g., for each point $X \in \mathcal{U}$, consider a closest point $f(X)$ on the boundary of $C$. Obviously, $f(X) f(Y) \geqslant X Y-\frac{1}{2} \geqslant \frac{1}{2} X Y$ for all $X, Y \in \mathcal{U}$. This yields that the perimeter of $C$ is at least $\mu \delta^{-2}$, for some constant $\mu$, and hence the diameter of $\mathcal{S}$ is of the same order. + +Alternatively, one may show that the projection of $\mathcal{U}$ to the line at the angle of $\pi / 4$ with $\mathbf{v}_{0}$ has diameter at least $\mu \delta^{-2}$ for some constant $\mu$. + +## Number Theory + +N1. Given a positive integer $k$, show that there exists a prime $p$ such that one can choose distinct integers $a_{1}, a_{2}, \ldots, a_{k+3} \in\{1,2, \ldots, p-1\}$ such that $p$ divides $a_{i} a_{i+1} a_{i+2} a_{i+3}-i$ for all $i=1,2, \ldots, k$. +(South Africa) +Solution. First we choose distinct positive rational numbers $r_{1}, \ldots, r_{k+3}$ such that + +$$ +r_{i} r_{i+1} r_{i+2} r_{i+3}=i \quad \text { for } 1 \leqslant i \leqslant k +$$ + +Let $r_{1}=x, r_{2}=y, r_{3}=z$ be some distinct primes greater than $k$; the remaining terms satisfy $r_{4}=\frac{1}{r_{1} r_{2} r_{3}}$ and $r_{i+4}=\frac{i+1}{i} r_{i}$. It follows that if $r_{i}$ are represented as irreducible fractions, the numerators are divisible by $x$ for $i \equiv 1(\bmod 4)$, by $y$ for $i \equiv 2(\bmod 4)$, by $z$ for $i \equiv 3(\bmod 4)$ and by none for $i \equiv 0(\bmod 4)$. Notice that $r_{i}3$ dividing a number of the form $x^{2}-x+1$ with integer $x$ there are two unconnected islands in $p$-Landia. + +For brevity's sake, when a bridge connects the islands numbered $m$ and $n$, we shall speak simply that it connects $m$ and $n$. + +A bridge connects $m$ and $n$ if $n \equiv m^{2}+1(\bmod p)$ or $m \equiv n^{2}+1(\bmod p)$. If $m^{2}+1 \equiv n$ $(\bmod p)$, we draw an arrow starting at $m$ on the bridge connecting $m$ and $n$. Clearly only one arrow starts at $m$ if $m^{2}+1 \not \equiv m(\bmod p)$, and no arrows otherwise. The total number of bridges does not exceed the total number of arrows. + +Suppose $x^{2}-x+1 \equiv 0(\bmod p)$. We may assume that $1 \leqslant x \leqslant p$; then there is no arrow starting at $x$. Since $(1-x)^{2}-(1-x)+1=x^{2}-x+1,(p+1-x)^{2}+1 \equiv(p+1-x)(\bmod p)$, and there is also no arrow starting at $p+1-x$. If $x=p+1-x$, that is, $x=\frac{p+1}{2}$, then $4\left(x^{2}-x+1\right)=p^{2}+3$ and therefore $x^{2}-x+1$ is not divisible by $p$. Thus the islands $x$ and $p+1-x$ are different, and no arrows start at either of them. It follows that the total number of bridges in $p$-Landia does not exceed $p-2$. + +Let $1,2, \ldots, p$ be the vertices of a graph $G_{p}$, where an edge connects $m$ and $n$ if and only if there is a bridge between $m$ and $n$. The number of vertices of $G_{p}$ is $p$ and the number of edges is less than $p-1$. This means that the graph is not connected, which means that there are two islands not connected by a chain of bridges. + +It remains to prove that there are infinitely many primes $p$ dividing $x^{2}-x+1$ for some integer $x$. Let $p_{1}, p_{2}, \ldots, p_{k}$ be any finite set of such primes. The number $\left(p_{1} p_{2} \cdot \ldots \cdot p_{k}\right)^{2}-p_{1} p_{2} \cdot \ldots \cdot p_{k}+1$ is greater than 1 and not divisible by any $p_{i}$; therefore it has another prime divisor with the required property. + +Solution 2. One can show, by using only arithmetical methods, that for infinitely many $p$, the kingdom of $p$-Ladia contains two islands connected to no other island, except for each other. + +Let arrows between islands have the same meaning as in the previous solution. Suppose that positive $a3$. It follows that $a b \equiv a(1-a) \equiv 1$ $(\bmod p)$. If an arrow goes from $t$ to $a$, then $t$ must satisfy the congruence $t^{2}+1 \equiv a \equiv a^{2}+1$ $(\bmod p)$; the only such $t \neq a$ is $p-a$. Similarly, the only arrow going to $b$ goes from $p-b$. If one of the numbers $p-a$ and $p-b$, say, $p-a$, is not at the end of any arrow, the pair $a, p-a$ is not connected with the rest of the islands. This is true if at least one of the congruences $x^{2}+1 \equiv-a, x^{2}+1 \equiv-b$ has no solutions, that is, either $-a-1$ or $-b-1$ is a quadratic non-residue modulo $p$. + +Note that $x^{2}-x+1 \equiv x^{2}-(a+b) x+a b \equiv(x-a)(x-b)(\bmod p)$. Substituting $x=-1$ we get $(-1-a)(-1-b) \equiv 3(\bmod p)$. If 3 is a quadratic non-residue modulo $p$, so is one of the numbers $-1-a$ and $-1-b$. + +Thus it is enough to find infinitely many primes $p>3$ dividing $x^{2}-x+1$ for some integer $x$ and such that 3 is a quadratic non-residue modulo $p$. + +If $x^{2}-x+1 \equiv 0(\bmod p)$ then $(2 x-1)^{2} \equiv-3(\bmod p)$, that is, -3 is a quadratic residue modulo $p$, so 3 is a quadratic non-residue if and only if -1 is also a non-residue, in other words, $p \equiv-1(\bmod 4)$. + +Similarly to the first solution, let $p_{1}, \ldots, p_{k}$ be primes congruent to -1 modulo 4 and dividing numbers of the form $x^{2}-x+1$. The number $\left(2 p_{1} \cdot \ldots \cdot p_{k}\right)^{2}-2 p_{1} \cdot \ldots \cdot p_{k}+1$ is +not divisible by any $p_{i}$ and is congruent to -1 modulo 4 , therefore, it has some prime divisor $p \equiv-1(\bmod 4)$ which has the required properties. + +N3. Let $n$ be an integer with $n \geqslant 2$. Does there exist a sequence $\left(a_{1}, \ldots, a_{n}\right)$ of positive integers with not all terms being equal such that the arithmetic mean of every two terms is equal to the geometric mean of some (one or more) terms in this sequence? +(Estonia) +Answer: No such sequence exists. +Solution 1. Suppose that $a_{1}, \ldots, a_{n}$ satisfy the required properties. Let $d=\operatorname{gcd}\left(a_{1} \ldots, a_{n}\right)$. If $d>1$ then replace the numbers $a_{1}, \ldots, a_{n}$ by $\frac{a_{1}}{d}, \ldots, \frac{a_{n}}{d}$; all arithmetic and all geometric means will be divided by $d$, so we obtain another sequence satisfying the condition. Hence, without loss of generality, we can assume that $\operatorname{gcd}\left(a_{1} \ldots, a_{n}\right)=1$. + +We show two numbers, $a_{m}$ and $a_{k}$ such that their arithmetic mean, $\frac{a_{m}+a_{k}}{2}$ is different from the geometric mean of any (nonempty) subsequence of $a_{1} \ldots, a_{n}$. That proves that there cannot exist such a sequence. + +Choose the index $m \in\{1, \ldots, n\}$ such that $a_{m}=\max \left(a_{1}, \ldots, a_{n}\right)$. Note that $a_{m} \geqslant 2$, because $a_{1}, \ldots, a_{n}$ are not all equal. Let $p$ be a prime divisor of $a_{m}$. + +Let $k \in\{1, \ldots, n\}$ be an index such that $a_{k}=\max \left\{a_{i}: p \nmid a_{i}\right\}$. Due to $\operatorname{gcd}\left(a_{1} \ldots, a_{n}\right)=1$, not all $a_{i}$ are divisible by $p$, so such a $k$ exists. Note that $a_{m}>a_{k}$ because $a_{m} \geqslant a_{k}, p \mid a_{m}$ and $p \nmid a_{k}$. + +Let $b=\frac{a_{m}+a_{k}}{2}$; we will show that $b$ cannot be the geometric mean of any subsequence of $a_{1}, \ldots, a_{n}$. + +Consider the geometric mean, $g=\sqrt[t]{a_{i_{1}} \cdot \ldots \cdot a_{i_{t}}}$ of an arbitrary subsequence of $a_{1}, \ldots, a_{n}$. +If none of $a_{i_{1}}, \ldots, a_{i_{t}}$ is divisible by $p$, then they are not greater than $a_{k}$, so + +$$ +g=\sqrt[t]{a_{i_{1}} \cdot \ldots \cdot a_{i_{t}}} \leqslant a_{k}<\frac{a_{m}+a_{k}}{2}=b +$$ + +and therefore $g \neq b$. +Otherwise, if at least one of $a_{i_{1}}, \ldots, a_{i_{t}}$ is divisible by $p$, then $2 g=2 \sqrt{t} \sqrt{a_{i_{1}} \cdot \ldots \cdot a_{i_{t}}}$ is either not an integer or is divisible by $p$, while $2 b=a_{m}+a_{k}$ is an integer not divisible by $p$, so $g \neq b$ again. + +Solution 2. Like in the previous solution, we assume that the numbers $a_{1}, \ldots, a_{n}$ have no common divisor greater than 1 . The arithmetic mean of any two numbers in the sequence is half of an integer; on the other hand, it is a (some integer order) root of an integer. This means each pair's mean is an integer, so all terms in the sequence must be of the same parity; hence they all are odd. Let $d=\min \left\{\operatorname{gcd}\left(a_{i}, a_{j}\right): a_{i} \neq a_{j}\right\}$. By reordering the sequence we can assume that $\operatorname{gcd}\left(a_{1}, a_{2}\right)=d$, the sum $a_{1}+a_{2}$ is maximal among such pairs, and $a_{1}>a_{2}$. + +We will show that $\frac{a_{1}+a_{2}}{2}$ cannot be the geometric mean of any subsequence of $a_{1} \ldots, a_{n}$. +Let $a_{1}=x d$ and $a_{2}=y d$ where $x, y$ are coprime, and suppose that there exist some $b_{1}, \ldots, b_{t} \in\left\{a_{1}, \ldots, a_{n}\right\}$ whose geometric mean is $\frac{a_{1}+a_{2}}{2}$. Let $d_{i}=\operatorname{gcd}\left(a_{1}, b_{i}\right)$ for $i=1,2, \ldots, t$ and let $D=d_{1} d_{2} \cdot \ldots \cdot d_{t}$. Then + +$$ +D=d_{1} d_{2} \cdot \ldots \cdot d_{t} \left\lvert\, b_{1} b_{2} \cdot \ldots \cdot b_{t}=\left(\frac{a_{1}+a_{2}}{2}\right)^{t}=\left(\frac{x+y}{2}\right)^{t} d^{t}\right. +$$ + +We claim that $D \mid d^{t}$. Consider an arbitrary prime divisor $p$ of $D$. Let $\nu_{p}(x)$ denote the exponent of $p$ in the prime factorization of $x$. If $p \left\lvert\, \frac{x+y}{2}\right.$, then $p \nmid x, y$, so $p$ is coprime with $x$; hence, $\nu_{p}\left(d_{i}\right) \leqslant \nu_{p}\left(a_{1}\right)=\nu_{p}(x d)=\nu_{p}(d)$ for every $1 \leqslant i \leqslant t$, therefore $\nu_{p}(D)=\sum_{i} \nu_{p}\left(d_{i}\right) \leqslant$ $t \nu_{p}(d)=\nu_{p}\left(d^{t}\right)$. Otherwise, if $p$ is coprime to $\frac{x+y}{2}$, we have $\nu_{p}(D) \leqslant \nu_{p}\left(d^{t}\right)$ trivially. The claim has been proved. + +Notice that $d_{i}=\operatorname{gcd}\left(b_{i}, a_{1}\right) \geqslant d$ for $1 \leqslant i \leqslant t$ : if $b_{i} \neq a_{1}$ then this follows from the definition of $d$; otherwise we have $b_{i}=a_{1}$, so $d_{i}=a_{1} \geqslant d$. Hence, $D=d_{1} \cdot \ldots \cdot d_{t} \geqslant d^{t}$, and the claim forces $d_{1}=\ldots=d_{t}=d$. + +Finally, by $\frac{a_{1}+a_{2}}{2}>a_{2}$ there must be some $b_{k}$ which is greater than $a_{2}$. From $a_{1}>a_{2} \geqslant$ $d=\operatorname{gcd}\left(a_{1}, b_{k}\right)$ it follows that $a_{1} \neq b_{k}$. Now the have a pair $a_{1}, b_{k}$ such that $\operatorname{gcd}\left(a_{1}, b_{k}\right)=d$ but $a_{1}+b_{k}>a_{1}+a_{2}$; that contradicts the choice of $a_{1}$ and $a_{2}$. + +Comment. The original problem proposal contained a second question asking if there exists a nonconstant sequence $\left(a_{1}, \ldots, a_{n}\right)$ of positive integers such that the geometric mean of every two terms is equal the arithmetic mean of some terms. + +For $n \geqslant 3$ such a sequence is $(4,1,1, \ldots, 1)$. The case $n=2$ can be done by the trivial estimates + +$$ +\min \left(a_{1}, a_{2}\right)<\sqrt{a_{1} a_{2}}<\frac{a_{1}+a_{2}}{2}<\max \left(a_{1}, a_{2}\right) +$$ + +The Problem Selection Committee found this variant less interesting and suggests using only the first question. + +N4. For any odd prime $p$ and any integer $n$, let $d_{p}(n) \in\{0,1, \ldots, p-1\}$ denote the remainder when $n$ is divided by $p$. We say that $\left(a_{0}, a_{1}, a_{2}, \ldots\right)$ is a $p$-sequence, if $a_{0}$ is a positive integer coprime to $p$, and $a_{n+1}=a_{n}+d_{p}\left(a_{n}\right)$ for $n \geqslant 0$. +(a) Do there exist infinitely many primes $p$ for which there exist $p$-sequences $\left(a_{0}, a_{1}, a_{2}, \ldots\right)$ and $\left(b_{0}, b_{1}, b_{2}, \ldots\right)$ such that $a_{n}>b_{n}$ for infinitely many $n$, and $b_{n}>a_{n}$ for infinitely many $n$ ? +(b) Do there exist infinitely many primes $p$ for which there exist $p$-sequences $\left(a_{0}, a_{1}, a_{2}, \ldots\right)$ and $\left(b_{0}, b_{1}, b_{2}, \ldots\right)$ such that $a_{0}b_{n}$ for all $n \geqslant 1$ ? +(United Kingdom) +Answer: Yes, for both parts. +Solution. Fix some odd prime $p$, and let $T$ be the smallest positive integer such that $p \mid 2^{T}-1$; in other words, $T$ is the multiplicative order of 2 modulo $p$. + +Consider any $p$-sequence $\left(x_{n}\right)=\left(x_{0}, x_{1}, x_{2}, \ldots\right)$. Obviously, $x_{n+1} \equiv 2 x_{n}(\bmod p)$ and therefore $x_{n} \equiv 2^{n} x_{0}(\bmod p)$. This yields $x_{n+T} \equiv x_{n}(\bmod p)$ and therefore $d\left(x_{n+T}\right)=d\left(x_{n}\right)$ for all $n \geqslant 0$. It follows that the sum $d\left(x_{n}\right)+d\left(x_{n+1}\right)+\ldots+d\left(x_{n+T-1}\right)$ does not depend on $n$ and is thus a function of $x_{0}$ (and $p$ ) only; we shall denote this sum by $S_{p}\left(x_{0}\right)$, and extend the function $S_{p}(\cdot)$ to all (not necessarily positive) integers. Therefore, we have $x_{n+k T}=x_{n}+k S_{p}\left(x_{0}\right)$ for all positive integers $n$ and $k$. Clearly, $S_{p}\left(x_{0}\right)=S_{p}\left(2^{t} x_{0}\right)$ for every integer $t \geqslant 0$. + +In both parts, we use the notation + +$$ +S_{p}^{+}=S_{p}(1)=\sum_{i=0}^{T-1} d_{p}\left(2^{i}\right) \quad \text { and } \quad S_{p}^{-}=S_{p}(-1)=\sum_{i=0}^{T-1} d_{p}\left(p-2^{i}\right) +$$ + +(a) Let $q>3$ be a prime and $p$ a prime divisor of $2^{q}+1$ that is greater than 3 . We will show that $p$ is suitable for part (a). Notice that $9 \nmid 2^{q}+1$, so that such a $p$ exists. Moreover, for any two odd primes $qb_{0}$ and $a_{1}=p+2b_{0}+k S_{p}^{+}=b_{k \cdot 2 q} \quad \text { and } \quad a_{k \cdot 2 q+1}=a_{1}+k S_{p}^{+}S_{p}\left(y_{0}\right)$ but $x_{0}y_{n}$ for every $n \geqslant q+q \cdot \max \left\{y_{r}-x_{r}: r=0,1, \ldots, q-1\right\}$. Now, since $x_{0}S_{p}^{-}$, then we can put $x_{0}=1$ and $y_{0}=p-1$. Otherwise, if $S_{p}^{+}p$ must be divisible by $p$. Indeed, if $n=p k+r$ is a good number, $k>0,00$. Let $\mathcal{B}$ be the set of big primes, and let $p_{1}p_{1} p_{2}$, and let $p_{1}^{\alpha}$ be the largest power of $p_{1}$ smaller than $n$, so that $n \leqslant p_{1}^{\alpha+1}0$. If $f(k)=f(n-k)$ for all $k$, it implies that $\binom{n-1}{k}$ is not divisible by $p$ for all $k=1,2, \ldots, n-2$. It is well known that it implies $n=a \cdot p^{s}, a0, f(q)>0$, there exist only finitely many $n$ which are equal both to $a \cdot p^{s}, a0$ for at least two primes less than $n$. Let $p_{0}$ be the prime with maximal $g(f, p)$ among all primes $p1$, let $p_{1}, \ldots, p_{k}$ be all primes between 1 and $N$ and $p_{k+1}, \ldots, p_{k+s}$ be all primes between $N+1$ and $2 N$. Since for $j \leqslant k+s$ all prime divisors of $p_{j}-1$ do not exceed $N$, we have + +$$ +\prod_{j=1}^{k+s}\left(p_{j}-1\right)=\prod_{i=1}^{k} p_{i}^{c_{i}} +$$ + +with some fixed exponents $c_{1}, \ldots, c_{k}$. Choose a huge prime number $q$ and consider a number + +$$ +n=\left(p_{1} \cdot \ldots \cdot p_{k}\right)^{q-1} \cdot\left(p_{k+1} \cdot \ldots \cdot p_{k+s}\right) +$$ + +Then + +$$ +\varphi(d(n))=\varphi\left(q^{k} \cdot 2^{s}\right)=q^{k-1}(q-1) 2^{s-1} +$$ + +and + +$$ +d(\varphi(n))=d\left(\left(p_{1} \cdot \ldots \cdot p_{k}\right)^{q-2} \prod_{i=1}^{k+s}\left(p_{i}-1\right)\right)=d\left(\prod_{i=1}^{k} p_{i}^{q-2+c_{i}}\right)=\prod_{i=1}^{k}\left(q-1+c_{i}\right) +$$ + +so + +$$ +\frac{\varphi(d(n))}{d(\varphi(n))}=\frac{q^{k-1}(q-1) 2^{s-1}}{\prod_{i=1}^{k}\left(q-1+c_{i}\right)}=2^{s-1} \cdot \frac{q-1}{q} \cdot \prod_{i=1}^{k} \frac{q}{q-1+c_{i}} +$$ + +which can be made arbitrarily close to $2^{s-1}$ by choosing $q$ large enough. It remains to show that $s$ can be arbitrarily large, i.e. that there can be arbitrarily many primes between $N$ and $2 N$. + +This follows, for instance, from the well-known fact that $\sum \frac{1}{p}=\infty$, where the sum is taken over the set $\mathbb{P}$ of prime numbers. Indeed, if, for some constant $\stackrel{p}{C}$, there were always at most $C$ primes between $2^{\ell}$ and $2^{\ell+1}$, we would have + +$$ +\sum_{p \in \mathbb{P}} \frac{1}{p}=\sum_{\ell=0}^{\infty} \sum_{\substack{p \in \mathbb{P} \\ p \in\left[2^{\ell}, 2^{\ell+1}\right)}} \frac{1}{p} \leqslant \sum_{\ell=0}^{\infty} \frac{C}{2^{\ell}}<\infty, +$$ + +which is a contradiction. +Comment 1. Here we sketch several alternative elementary self-contained ways to perform the last step of the solution above. In particular, they avoid using divergence of $\sum \frac{1}{p}$. + +Suppose that for some constant $C$ and for every $k=1,2, \ldots$ there exist at most $C$ prime numbers between $2^{k}$ and $2^{k+1}$. Consider the prime factorization of the factorial $\left(2^{n}\right)!=\prod p^{\alpha_{p}}$. We have $\alpha_{p}=\left\lfloor 2^{n} / p\right\rfloor+\left\lfloor 2^{n} / p^{2}\right\rfloor+\ldots$. Thus, for $p \in\left[2^{k}, 2^{k+1}\right)$, we get $\alpha_{p} \leqslant 2^{n} / 2^{k}+2^{n} / 2^{k+1}+\ldots=2^{n-k+1}$, therefore $p^{\alpha_{p}} \leqslant 2^{(k+1) 2^{n-k+1}}$. Combining this with the bound $(2 m)!\geqslant m(m+1) \cdot \ldots \cdot(2 m-1) \geqslant m^{m}$ for $m=2^{n-1}$ we get + +$$ +2^{(n-1) \cdot 2^{n-1}} \leqslant\left(2^{n}\right)!\leqslant \prod_{k=1}^{n-1} 2^{C(k+1) 2^{n-k+1}} +$$ + +or + +$$ +\sum_{k=1}^{n-1} C(k+1) 2^{1-k} \geqslant \frac{n-1}{2} +$$ + +that fails for large $n$ since $C(k+1) 2^{1-k}<1 / 3$ for all but finitely many $k$. +In fact, a much stronger inequality can be obtained in an elementary way: Note that the formula for $\nu_{p}(n!)$ implies that if $p^{\alpha}$ is the largest power of $p$ dividing $\binom{n}{n / 2}$, then $p^{\alpha} \leqslant n$. By looking at prime factorization of $\binom{n}{n / 2}$ we instantaneously infer that + +$$ +\pi(n) \geqslant \log _{n}\binom{n}{n / 2} \geqslant \frac{\log \left(2^{n} / n\right)}{\log n} \geqslant \frac{n}{2 \log n} . +$$ + +This, in particular, implies that for infinitely many $n$ there are at least $\frac{n}{3 \log n}$ primes between $n$ and $2 n$. +Solution 2. In this solution we will use the Prime Number Theorem which states that + +$$ +\pi(m)=\frac{m}{\log m} \cdot(1+o(1)) +$$ + +as $m$ tends to infinity. Here and below $\pi(m)$ denotes the number of primes not exceeding $m$, and $\log$ the natural logarithm. + +Let $m>5$ be a large positive integer and let $n:=p_{1} p_{2} \cdot \ldots \cdot p_{\pi(m)}$ be the product of all primes not exceeding $m$. Then $\varphi(d(n))=\varphi\left(2^{\pi(m)}\right)=2^{\pi(m)-1}$. Consider the number + +$$ +\varphi(n)=\prod_{k=1}^{\pi(m)}\left(p_{k}-1\right)=\prod_{s=1}^{\pi(m / 2)} q_{s}^{\alpha_{s}} +$$ + +where $q_{1}, \ldots, q_{\pi(m / 2)}$ are primes not exceeding $m / 2$. Note that every term $p_{k}-1$ contributes at most one prime $q_{s}>\sqrt{m}$ into the product $\prod_{s} q_{s}^{\alpha_{s}}$, so we have + +$$ +\sum_{s: q_{s}>\sqrt{m}} \alpha_{s} \leqslant \pi(m) \Longrightarrow \sum_{s: q_{s}>\sqrt{m}}\left(1+\alpha_{s}\right) \leqslant \pi(m)+\pi(m / 2) . +$$ + +Hence, applying the AM-GM inequality and the inequality $(A / x)^{x} \leqslant e^{A / e}$, we obtain + +$$ +\prod_{s: q_{s}>\sqrt{m}}\left(\alpha_{s}+1\right) \leqslant\left(\frac{\pi(m)+\pi(m / 2)}{\ell}\right)^{\ell} \leqslant \exp \left(\frac{\pi(m)+\pi(m / 2)}{e}\right) +$$ + +where $\ell$ is the number of primes in the interval $(\sqrt{m}, m]$. +We then use a trivial bound $\alpha_{i} \leqslant \log _{2}(\varphi(n)) \leqslant \log _{2} n<\log _{2}\left(m^{m}\right)a, c$, then $b \nmid a+c$. Otherwise, since $b \neq a+c$, we would have $b \leqslant(a+c) / 2<\max \{a, c\}$. + +Solution 1. We prove the following stronger statement. +Claim. Let $\mathcal{S}$ be a good set consisting of $n \geqslant 2$ positive integers. Then the elements of $\mathcal{S}$ may be ordered as $a_{1}, a_{2}, \ldots, a_{n}$ so that $a_{i} \nmid a_{i-1}+a_{i+1}$ and $a_{i} \nmid a_{i-1}-a_{i+1}$, for all $i=2,3, \ldots, n-1$. Proof. Say that the ordering $a_{1}, \ldots, a_{n}$ of $\mathcal{S}$ is nice if it satisfies the required property. + +We proceed by induction on $n$. The base case $n=2$ is trivial, as there are no restrictions on the ordering. + +To perform the step of induction, suppose that $n \geqslant 3$. Let $a=\max \mathcal{S}$, and set $\mathcal{T}=\mathcal{S} \backslash\{a\}$. Use the inductive hypothesis to find a nice ordering $b_{1}, \ldots, b_{n-1}$ of $\mathcal{T}$. We will show that $a$ may be inserted into this sequence so as to reach a nice ordering of $\mathcal{S}$. In other words, we will show that there exists a $j \in\{1,2, \ldots, n\}$ such that the ordering + +$$ +N_{j}=\left(b_{1}, \ldots, b_{j-1}, a, b_{j}, b_{j+1}, \ldots, b_{n-1}\right) +$$ + +is nice. +Assume that, for some $j$, the ordering $N_{j}$ is not nice, so that some element $x$ in it divides either the sum or the difference of two adjacent ones. This did not happen in the ordering of $\mathcal{T}$, hence $x \in\left\{b_{j-1}, a, b_{j}\right\}$ (if, say, $b_{j-1}$ does not exist, then $x \in\left\{a, b_{j}\right\}$; a similar agreement is applied hereafter). But the case $x=a$ is impossible: $a$ cannot divide $b_{j-1}-b_{j}$, since $0<\left|b_{j-1}-b_{j}\right|a_{j+1}>\ldots>$ $a_{n}$; and +(ii) $f$-avoidance: If $a3 b$. + +A2. For every integer $n \geqslant 1$ consider the $n \times n$ table with entry $\left\lfloor\frac{i j}{n+1}\right\rfloor$ at the intersection of row $i$ and column $j$, for every $i=1, \ldots, n$ and $j=1, \ldots, n$. Determine all integers $n \geqslant 1$ for which the sum of the $n^{2}$ entries in the table is equal to $\frac{1}{4} n^{2}(n-1)$. + +A3. Given a positive integer $n$, find the smallest value of $\left\lfloor\frac{a_{1}}{1}\right\rfloor+\left\lfloor\frac{a_{2}}{2}\right\rfloor+\cdots+\left\lfloor\frac{a_{n}}{n}\right\rfloor$ over all permutations $\left(a_{1}, a_{2}, \ldots, a_{n}\right)$ of $(1,2, \ldots, n)$. + +A4. Show that for all real numbers $x_{1}, \ldots, x_{n}$ the following inequality holds: + +$$ +\sum_{i=1}^{n} \sum_{j=1}^{n} \sqrt{\left|x_{i}-x_{j}\right|} \leqslant \sum_{i=1}^{n} \sum_{j=1}^{n} \sqrt{\left|x_{i}+x_{j}\right|} +$$ + +A5. Let $n \geqslant 2$ be an integer, and let $a_{1}, a_{2}, \ldots, a_{n}$ be positive real numbers such that $a_{1}+a_{2}+\cdots+a_{n}=1$. Prove that + +$$ +\sum_{k=1}^{n} \frac{a_{k}}{1-a_{k}}\left(a_{1}+a_{2}+\cdots+a_{k-1}\right)^{2}<\frac{1}{3} +$$ + +A6. Let $A$ be a finite set of (not necessarily positive) integers, and let $m \geqslant 2$ be an integer. Assume that there exist non-empty subsets $B_{1}, B_{2}, B_{3}, \ldots, B_{m}$ of $A$ whose elements add up to the sums $m^{1}, m^{2}, m^{3}, \ldots, m^{m}$, respectively. Prove that $A$ contains at least $m / 2$ elements. + +A7. Let $n \geqslant 1$ be an integer, and let $x_{0}, x_{1}, \ldots, x_{n+1}$ be $n+2$ non-negative real numbers that satisfy $x_{i} x_{i+1}-x_{i-1}^{2} \geqslant 1$ for all $i=1,2, \ldots, n$. Show that + +$$ +x_{0}+x_{1}+\cdots+x_{n}+x_{n+1}>\left(\frac{2 n}{3}\right)^{3 / 2} +$$ + +A8. Determine all functions $f: \mathbb{R} \rightarrow \mathbb{R}$ that satisfy + +$$ +(f(a)-f(b))(f(b)-f(c))(f(c)-f(a))=f\left(a b^{2}+b c^{2}+c a^{2}\right)-f\left(a^{2} b+b^{2} c+c^{2} a\right) +$$ + +for all real numbers $a, b, c$. + +## Combinatorics + +C1. Let $S$ be an infinite set of positive integers, such that there exist four pairwise distinct $a, b, c, d \in S$ with $\operatorname{gcd}(a, b) \neq \operatorname{gcd}(c, d)$. Prove that there exist three pairwise distinct $x, y, z \in S$ such that $\operatorname{gcd}(x, y)=\operatorname{gcd}(y, z) \neq \operatorname{gcd}(z, x)$. + +C2. Let $n \geqslant 3$ be an integer. An integer $m \geqslant n+1$ is called $n$-colourful if, given infinitely many marbles in each of $n$ colours $C_{1}, C_{2}, \ldots, C_{n}$, it is possible to place $m$ of them around a circle so that in any group of $n+1$ consecutive marbles there is at least one marble of colour $C_{i}$ for each $i=1, \ldots, n$. + +Prove that there are only finitely many positive integers which are not $n$-colourful. Find the largest among them. + +C3. A thimblerigger has 2021 thimbles numbered from 1 through 2021. The thimbles are arranged in a circle in arbitrary order. The thimblerigger performs a sequence of 2021 moves; in the $k^{\text {th }}$ move, he swaps the positions of the two thimbles adjacent to thimble $k$. + +Prove that there exists a value of $k$ such that, in the $k^{\text {th }}$ move, the thimblerigger swaps some thimbles $a$ and $b$ such that $ak \geqslant 1$. There are $2 n+1$ students standing in a circle. Each student $S$ has $2 k$ neighbours - namely, the $k$ students closest to $S$ on the right, and the $k$ students closest to $S$ on the left. + +Suppose that $n+1$ of the students are girls, and the other $n$ are boys. Prove that there is a girl with at least $k$ girls among her neighbours. + +C6. A hunter and an invisible rabbit play a game on an infinite square grid. First the hunter fixes a colouring of the cells with finitely many colours. The rabbit then secretly chooses a cell to start in. Every minute, the rabbit reports the colour of its current cell to the hunter, and then secretly moves to an adjacent cell that it has not visited before (two cells are adjacent if they share a side). The hunter wins if after some finite time either + +- the rabbit cannot move; or +- the hunter can determine the cell in which the rabbit started. + +Decide whether there exists a winning strategy for the hunter. + +C7. Consider a checkered $3 m \times 3 m$ square, where $m$ is an integer greater than 1 . A frog sits on the lower left corner cell $S$ and wants to get to the upper right corner cell $F$. The frog can hop from any cell to either the next cell to the right or the next cell upwards. + +Some cells can be sticky, and the frog gets trapped once it hops on such a cell. A set $X$ of cells is called blocking if the frog cannot reach $F$ from $S$ when all the cells of $X$ are sticky. A blocking set is minimal if it does not contain a smaller blocking set. +(a) Prove that there exists a minimal blocking set containing at least $3 m^{2}-3 m$ cells. +(b) Prove that every minimal blocking set contains at most $3 m^{2}$ cells. + +Note. An example of a minimal blocking set for $m=2$ is shown below. Cells of the set $X$ are marked by letters $x$. +![](https://cdn.mathpix.com/cropped/2024_11_18_7107063274cfa469597ag-07.jpg?height=324&width=335&top_left_y=843&top_left_x=866) + +C8. Determine the largest $N$ for which there exists a table $T$ of integers with $N$ rows and 100 columns that has the following properties: +(i) Every row contains the numbers $1,2, \ldots, 100$ in some order. +(ii) For any two distinct rows $r$ and $s$, there is a column $c$ such that $|T(r, c)-T(s, c)| \geqslant 2$. + +Here $T(r, c)$ means the number at the intersection of the row $r$ and the column $c$. + +## Geometry + +G1. Let $A B C D$ be a parallelogram such that $A C=B C$. A point $P$ is chosen on the extension of the segment $A B$ beyond $B$. The circumcircle of the triangle $A C D$ meets the segment $P D$ again at $Q$, and the circumcircle of the triangle $A P Q$ meets the segment $P C$ again at $R$. Prove that the lines $C D, A Q$, and $B R$ are concurrent. + +## G2. Let $A B C D$ be a convex quadrilateral circumscribed around a circle with centre $I$. + +Let $\omega$ be the circumcircle of the triangle $A C I$. The extensions of $B A$ and $B C$ beyond $A$ and $C$ meet $\omega$ at $X$ and $Z$, respectively. The extensions of $A D$ and $C D$ beyond $D$ meet $\omega$ at $Y$ and $T$, respectively. Prove that the perimeters of the (possibly self-intersecting) quadrilaterals $A D T X$ and $C D Y Z$ are equal. + +## G3. + +Version 1. Let $n$ be a fixed positive integer, and let S be the set of points $(x, y)$ on the Cartesian plane such that both coordinates $x$ and $y$ are nonnegative integers smaller than $2 n$ (thus $|S|=4 n^{2}$ ). Assume that $\mathcal{F}$ is a set consisting of $n^{2}$ quadrilaterals such that all their vertices lie in $S$, and each point in $S$ is a vertex of exactly one of the quadrilaterals in $\mathcal{F}$. + +Determine the largest possible sum of areas of all $n^{2}$ quadrilaterals in $\mathcal{F}$. +Version 2. Let $n$ be a fixed positive integer, and let S be the set of points $(x, y)$ on the Cartesian plane such that both coordinates $x$ and $y$ are nonnegative integers smaller than $2 n$ (thus $|\mathrm{S}|=4 n^{2}$ ). Assume that $\mathcal{F}$ is a set of polygons such that all vertices of polygons in $\mathcal{F}$ lie in S , and each point in S is a vertex of exactly one of the polygons in $\mathcal{F}$. + +Determine the largest possible sum of areas of all polygons in $\mathcal{F}$. +G4. Let $A B C D$ be a quadrilateral inscribed in a circle $\Omega$. Let the tangent to $\Omega$ at $D$ intersect the rays $B A$ and $B C$ at points $E$ and $F$, respectively. A point $T$ is chosen inside the triangle $A B C$ so that $T E \| C D$ and $T F \| A D$. Let $K \neq D$ be a point on the segment $D F$ such that $T D=T K$. Prove that the lines $A C, D T$ and $B K$ intersect at one point. + +G5. Let $A B C D$ be a cyclic quadrilateral whose sides have pairwise different lengths. Let $O$ be the circumcentre of $A B C D$. The internal angle bisectors of $\angle A B C$ and $\angle A D C$ meet $A C$ at $B_{1}$ and $D_{1}$, respectively. Let $O_{B}$ be the centre of the circle which passes through $B$ and is tangent to $A C$ at $D_{1}$. Similarly, let $O_{D}$ be the centre of the circle which passes through $D$ and is tangent to $A C$ at $B_{1}$. + +Assume that $B D_{1} \| D B_{1}$. Prove that $O$ lies on the line $O_{B} O_{D}$. +G6. Determine all integers $n \geqslant 3$ satisfying the following property: every convex $n$-gon whose sides all have length 1 contains an equilateral triangle of side length 1. +(Every polygon is assumed to contain its boundary.) + +G7. A point $D$ is chosen inside an acute-angled triangle $A B C$ with $A B>A C$ so that $\angle B A D=\angle D A C$. A point $E$ is constructed on the segment $A C$ so that $\angle A D E=\angle D C B$. Similarly, a point $F$ is constructed on the segment $A B$ so that $\angle A D F=\angle D B C$. A point $X$ is chosen on the line $A C$ so that $C X=B X$. Let $O_{1}$ and $O_{2}$ be the circumcentres of the triangles $A D C$ and $D X E$. Prove that the lines $B C, E F$, and $O_{1} O_{2}$ are concurrent. + +G8. Let $\omega$ be the circumcircle of a triangle $A B C$, and let $\Omega_{A}$ be its excircle which is tangent to the segment $B C$. Let $X$ and $Y$ be the intersection points of $\omega$ and $\Omega_{A}$. Let $P$ and $Q$ be the projections of $A$ onto the tangent lines to $\Omega_{A}$ at $X$ and $Y$, respectively. The tangent line at $P$ to the circumcircle of the triangle $A P X$ intersects the tangent line at $Q$ to the circumcircle of the triangle $A Q Y$ at a point $R$. Prove that $A R \perp B C$. + +## Number Theory + +N1. Determine all integers $n \geqslant 1$ for which there exists a pair of positive integers $(a, b)$ such that no cube of a prime divides $a^{2}+b+3$ and + +$$ +\frac{a b+3 b+8}{a^{2}+b+3}=n +$$ + +N2. Let $n \geqslant 100$ be an integer. The numbers $n, n+1, \ldots, 2 n$ are written on $n+1$ cards, one number per card. The cards are shuffled and divided into two piles. Prove that one of the piles contains two cards such that the sum of their numbers is a perfect square. + +N3. Find all positive integers $n$ with the following property: the $k$ positive divisors of $n$ have a permutation $\left(d_{1}, d_{2}, \ldots, d_{k}\right)$ such that for every $i=1,2, \ldots, k$, the number $d_{1}+\cdots+d_{i}$ is a perfect square. + +N4. Alice is given a rational number $r>1$ and a line with two points $B \neq R$, where point $R$ contains a red bead and point $B$ contains a blue bead. Alice plays a solitaire game by performing a sequence of moves. In every move, she chooses a (not necessarily positive) integer $k$, and a bead to move. If that bead is placed at point $X$, and the other bead is placed at $Y$, then Alice moves the chosen bead to point $X^{\prime}$ with $\overrightarrow{Y X^{\prime}}=r^{k} \overrightarrow{Y X}$. + +Alice's goal is to move the red bead to the point $B$. Find all rational numbers $r>1$ such that Alice can reach her goal in at most 2021 moves. + +N5. Prove that there are only finitely many quadruples $(a, b, c, n)$ of positive integers such that + +$$ +n!=a^{n-1}+b^{n-1}+c^{n-1} . +$$ + +N6. Determine all integers $n \geqslant 2$ with the following property: every $n$ pairwise distinct integers whose sum is not divisible by $n$ can be arranged in some order $a_{1}, a_{2}, \ldots, a_{n}$ so that $n$ divides $1 \cdot a_{1}+2 \cdot a_{2}+\cdots+n \cdot a_{n}$. + +N7. Let $a_{1}, a_{2}, a_{3}, \ldots$ be an infinite sequence of positive integers such that $a_{n+2 m}$ divides $a_{n}+a_{n+m}$ for all positive integers $n$ and $m$. Prove that this sequence is eventually periodic, i.e. there exist positive integers $N$ and $d$ such that $a_{n}=a_{n+d}$ for all $n>N$. + +N8. For a polynomial $P(x)$ with integer coefficients let $P^{1}(x)=P(x)$ and $P^{k+1}(x)=$ $P\left(P^{k}(x)\right)$ for $k \geqslant 1$. Find all positive integers $n$ for which there exists a polynomial $P(x)$ with integer coefficients such that for every integer $m \geqslant 1$, the numbers $P^{m}(1), \ldots, P^{m}(n)$ leave exactly $\left\lceil n / 2^{m}\right\rceil$ distinct remainders when divided by $n$. + +## Solutions + +## Algebra + +A1. Let $n$ be an integer, and let $A$ be a subset of $\left\{0,1,2,3, \ldots, 5^{n}\right\}$ consisting of $4 n+2$ numbers. Prove that there exist $a, b, c \in A$ such that $a3 b$. + +Solution 1. (By contradiction) Suppose that there exist $4 n+2$ non-negative integers $x_{0}<$ $x_{1}<\cdots5^{n} \cdot 1 +$$ + +Solution 2. Denote the maximum element of $A$ by $c$. For $k=0, \ldots, 4 n-1$ let + +$$ +A_{k}=\left\{x \in A:\left(1-(2 / 3)^{k}\right) c \leqslant x<\left(1-(2 / 3)^{k+1}\right) c\right\} +$$ + +Note that + +$$ +\left(1-(2 / 3)^{4 n}\right) c=c-(16 / 81)^{n} c>c-(1 / 5)^{n} c \geqslant c-1 +$$ + +which shows that the sets $A_{0}, A_{1}, \ldots, A_{4 n-1}$ form a partition of $A \backslash\{c\}$. Since $A \backslash\{c\}$ has $4 n+1$ elements, by the pigeonhole principle some set $A_{k}$ does contain at least two elements of $A \backslash\{c\}$. Denote these two elements $a$ and $b$ and assume $a3 b +$$ + +as desired. + +A2. For every integer $n \geqslant 1$ consider the $n \times n$ table with entry $\left\lfloor\frac{i j}{n+1}\right\rfloor$ at the intersection of row $i$ and column $j$, for every $i=1, \ldots, n$ and $j=1, \ldots, n$. Determine all integers $n \geqslant 1$ for which the sum of the $n^{2}$ entries in the table is equal to $\frac{1}{4} n^{2}(n-1)$. + +Answer: All integers $n$ for which $n+1$ is a prime. +Solution 1. First, observe that every pair $x, y$ of real numbers for which the sum $x+y$ is integer satisfies + +$$ +\lfloor x\rfloor+\lfloor y\rfloor \geqslant x+y-1 +$$ + +The inequality is strict if $x$ and $y$ are integers, and it holds with equality otherwise. +We estimate the sum $S$ as follows. + +$$ +\begin{aligned} +2 S=\sum_{1 \leqslant i, j \leqslant n}\left(\left\lfloor\frac{i j}{n+1}\right\rfloor+\left\lfloor\frac{i j}{n+1}\right\rfloor\right)= & \sum_{1 \leqslant i, j \leqslant n}\left(\left\lfloor\frac{i j}{n+1}\right\rfloor+\left\lfloor\frac{(n+1-i) j}{n+1}\right\rfloor\right) \\ +& \geqslant \sum_{1 \leqslant i, j \leqslant n}(j-1)=\frac{(n-1) n^{2}}{2} . +\end{aligned} +$$ + +The inequality in the last line follows from (1) by setting $x=i j /(n+1)$ and $y=(n+1-$ i) $j /(n+1)$, so that $x+y=j$ is integral. + +Now $S=\frac{1}{4} n^{2}(n-1)$ if and only if the inequality in the last line holds with equality, which means that none of the values $i j /(n+1)$ with $1 \leqslant i, j \leqslant n$ may be integral. + +Hence, if $n+1$ is composite with factorisation $n+1=a b$ for $2 \leqslant a, b \leqslant n$, one gets a strict inequality for $i=a$ and $j=b$. If $n+1$ is a prime, then $i j /(n+1)$ is never integral and $S=\frac{1}{4} n^{2}(n-1)$. + +Solution 2. To simplify the calculation with indices, extend the table by adding a phantom column of index 0 with zero entries (which will not change the sum of the table). Fix a row $i$ with $1 \leqslant i \leqslant n$, and let $d:=\operatorname{gcd}(i, n+1)$ and $k:=(n+1) / d$. For columns $j=0, \ldots, n$, define the remainder $r_{j}:=i j \bmod (n+1)$. We first prove the following +Claim. For every integer $g$ with $1 \leqslant g \leqslant d$, the remainders $r_{j}$ with indices $j$ in the range + +$$ +(g-1) k \leqslant j \leqslant g k-1 +$$ + +form a permutation of the $k$ numbers $0 \cdot d, 1 \cdot d, 2 \cdot d, \ldots,(k-1) \cdot d$. +Proof. If $r_{j^{\prime}}=r_{j}$ holds for two indices $j^{\prime}$ and $j$ in (2), then $i\left(j^{\prime}-j\right) \equiv 0 \bmod (n+1)$, so that $j^{\prime}-j$ is a multiple of $k$; since $\left|j^{\prime}-j\right| \leqslant k-1$, this implies $j^{\prime}=j$. Hence, the $k$ remainders are pairwise distinct. Moreover, each remainder $r_{j}=i j \bmod (n+1)$ is a multiple of $d=\operatorname{gcd}(i, n+1)$. This proves the claim. + +We then have + +$$ +\sum_{j=0}^{n} r_{j}=\sum_{g=1}^{d} \sum_{\ell=0}^{(n+1) / d-1} \ell d=d^{2} \cdot \frac{1}{2}\left(\frac{n+1}{d}-1\right) \frac{n+1}{d}=\frac{(n+1-d)(n+1)}{2} +$$ + +By using (3), compute the sum $S_{i}$ of row $i$ as follows: + +$$ +\begin{aligned} +S_{i}=\sum_{j=0}^{n}\left\lfloor\frac{i j}{n+1}\right\rfloor= & \sum_{j=0}^{n} \frac{i j-r_{j}}{n+1}=\frac{i}{n+1} \sum_{j=0}^{n} j-\frac{1}{n+1} \sum_{j=0}^{n} r_{j} \\ +& =\frac{i}{n+1} \cdot \frac{n(n+1)}{2}-\frac{1}{n+1} \cdot \frac{(n+1-d)(n+1)}{2}=\frac{(i n-n-1+d)}{2} . +\end{aligned} +$$ + +Equation (4) yields the following lower bound on the row sum $S_{i}$, which holds with equality if and only if $d=\operatorname{gcd}(i, n+1)=1$ : + +$$ +S_{i} \geqslant \frac{(i n-n-1+1)}{2}=\frac{n(i-1)}{2} +$$ + +By summing up the bounds (5) for the rows $i=1, \ldots, n$, we get the following lower bound for the sum of all entries in the table + +$$ +\sum_{i=1}^{n} S_{i} \geqslant \sum_{i=1}^{n} \frac{n}{2}(i-1)=\frac{n^{2}(n-1)}{4} +$$ + +In (6) we have equality if and only if equality holds in (5) for each $i=1, \ldots, n$, which happens if and only if $\operatorname{gcd}(i, n+1)=1$ for each $i=1, \ldots, n$, which is equivalent to the fact that $n+1$ is a prime. Thus the sum of the table entries is $\frac{1}{4} n^{2}(n-1)$ if and only if $n+1$ is a prime. + +Comment. To simplify the answer, in the problem statement one can make a change of variables by introducing $m:=n+1$ and writing everything in terms of $m$. The drawback is that the expression for the sum will then be $\frac{1}{4}(m-1)^{2}(m-2)$ which seems more artificial. + +A3. Given a positive integer $n$, find the smallest value of $\left\lfloor\frac{a_{1}}{1}\right\rfloor+\left\lfloor\frac{a_{2}}{2}\right\rfloor+\cdots+\left\lfloor\frac{a_{n}}{n}\right\rfloor$ over all permutations $\left(a_{1}, a_{2}, \ldots, a_{n}\right)$ of $(1,2, \ldots, n)$. + +Answer: The minimum of such sums is $\left\lfloor\log _{2} n\right\rfloor+1$; so if $2^{k} \leqslant n<2^{k+1}$, the minimum is $k+1$. +Solution 1. Suppose that $2^{k} \leqslant n<2^{k+1}$ with some nonnegative integer $k$. First we show a permutation $\left(a_{1}, a_{2}, \ldots, a_{n}\right)$ such that $\left\lfloor\frac{a_{1}}{1}\right\rfloor+\left\lfloor\frac{a_{2}}{2}\right\rfloor+\cdots+\left\lfloor\frac{a_{n}}{n}\right\rfloor=k+1$; then we will prove that $\left\lfloor\frac{a_{1}}{1}\right\rfloor+\left\lfloor\frac{a_{2}}{2}\right\rfloor+\cdots+\left\lfloor\frac{a_{n}}{n}\right\rfloor \geqslant k+1$ for every permutation. Hence, the minimal possible value will be $k+1$. +I. Consider the permutation + +$$ +\begin{gathered} +\left(a_{1}\right)=(1), \quad\left(a_{2}, a_{3}\right)=(3,2), \quad\left(a_{4}, a_{5}, a_{6}, a_{7}\right)=(7,4,5,6) \\ +\left(a_{2^{k-1}}, \ldots, a_{2^{k}-1}\right)=\left(2^{k}-1,2^{k-1}, 2^{k-1}+1, \ldots, 2^{k}-2\right) \\ +\left(a_{2^{k}}, \ldots, a_{n}\right)=\left(n, 2^{k}, 2^{k}+1, \ldots, n-1\right) +\end{gathered} +$$ + +This permutation consists of $k+1$ cycles. In every cycle $\left(a_{p}, \ldots, a_{q}\right)=(q, p, p+1, \ldots, q-1)$ we have $q<2 p$, so + +$$ +\sum_{i=p}^{q}\left\lfloor\frac{a_{i}}{i}\right\rfloor=\left\lfloor\frac{q}{p}\right\rfloor+\sum_{i=p+1}^{q}\left\lfloor\frac{i-1}{i}\right\rfloor=1 ; +$$ + +The total sum over all cycles is precisely $k+1$. +II. In order to establish the lower bound, we prove a more general statement. + +Claim. If $b_{1}, \ldots, b_{2^{k}}$ are distinct positive integers then + +$$ +\sum_{i=1}^{2^{k}}\left\lfloor\frac{b_{i}}{i}\right\rfloor \geqslant k+1 +$$ + +From the Claim it follows immediately that $\sum_{i=1}^{n}\left\lfloor\frac{a_{i}}{i}\right\rfloor \geqslant \sum_{i=1}^{2^{k}}\left\lfloor\frac{a_{i}}{i}\right\rfloor \geqslant k+1$. +Proof of the Claim. Apply induction on $k$. For $k=1$ the claim is trivial, $\left\lfloor\frac{b_{1}}{1}\right\rfloor \geqslant 1$. Suppose the Claim holds true for some positive integer $k$, and consider $k+1$. + +If there exists an index $j$ such that $2^{k}k +$$ + +As the left-hand side is an integer, it must be at least $k+1$. + +A4. Show that for all real numbers $x_{1}, \ldots, x_{n}$ the following inequality holds: + +$$ +\sum_{i=1}^{n} \sum_{j=1}^{n} \sqrt{\left|x_{i}-x_{j}\right|} \leqslant \sum_{i=1}^{n} \sum_{j=1}^{n} \sqrt{\left|x_{i}+x_{j}\right|} +$$ + +Solution 1. If we add $t$ to all the variables then the left-hand side remains constant and the right-hand side becomes + +$$ +H(t):=\sum_{i=1}^{n} \sum_{j=1}^{n} \sqrt{\left|x_{i}+x_{j}+2 t\right|} +$$ + +Let $T$ be large enough such that both $H(-T)$ and $H(T)$ are larger than the value $L$ of the lefthand side of the inequality we want to prove. Not necessarily distinct points $p_{i, j}:=-\left(x_{i}+x_{j}\right) / 2$ together with $T$ and $-T$ split the real line into segments and two rays such that on each of these segments and rays the function $H(t)$ is concave since $f(t):=\sqrt{|\ell+2 t|}$ is concave on both intervals $(-\infty,-\ell / 2]$ and $[-\ell / 2,+\infty)$. Let $[a, b]$ be the segment containing zero. Then concavity implies $H(0) \geqslant \min \{H(a), H(b)\}$ and, since $H( \pm T)>L$, it suffices to prove the inequalities $H\left(-\left(x_{i}+x_{j}\right) / 2\right) \geqslant L$, that is to prove the original inequality in the case when all numbers are shifted in such a way that two variables $x_{i}$ and $x_{j}$ add up to zero. In the following we denote the shifted variables still by $x_{i}$. + +If $i=j$, i.e. $x_{i}=0$ for some index $i$, then we can remove $x_{i}$ which will decrease both sides by $2 \sum_{k} \sqrt{\left|x_{k}\right|}$. Similarly, if $x_{i}+x_{j}=0$ for distinct $i$ and $j$ we can remove both $x_{i}$ and $x_{j}$ which decreases both sides by + +$$ +2 \sqrt{2\left|x_{i}\right|}+2 \cdot \sum_{k \neq i, j}\left(\sqrt{\left|x_{k}+x_{i}\right|}+\sqrt{\left|x_{k}+x_{j}\right|}\right) +$$ + +In either case we reduced our inequality to the case of smaller $n$. It remains to note that for $n=0$ and $n=1$ the inequality is trivial. + +Solution 2. For real $p$ consider the integral + +$$ +I(p)=\int_{0}^{\infty} \frac{1-\cos (p x)}{x \sqrt{x}} d x +$$ + +which clearly converges to a strictly positive number. By changing the variable $y=|p| x$ one notices that $I(p)=\sqrt{|p|} I(1)$. Hence, by using the trigonometric formula $\cos (\alpha-\beta)-\cos (\alpha+$ $\beta)=2 \sin \alpha \sin \beta$ we obtain +$\sqrt{|a+b|}-\sqrt{|a-b|}=\frac{1}{I(1)} \int_{0}^{\infty} \frac{\cos ((a-b) x)-\cos ((a+b) x)}{x \sqrt{x}} d x=\frac{1}{I(1)} \int_{0}^{\infty} \frac{2 \sin (a x) \sin (b x)}{x \sqrt{x}} d x$, +from which our inequality immediately follows: + +$$ +\sum_{i=1}^{n} \sum_{j=1}^{n} \sqrt{\left|x_{i}+x_{j}\right|}-\sum_{i=1}^{n} \sum_{j=1}^{n} \sqrt{\left|x_{i}-x_{j}\right|}=\frac{2}{I(1)} \int_{0}^{\infty} \frac{\left(\sum_{i=1}^{n} \sin \left(x_{i} x\right)\right)^{2}}{x \sqrt{x}} d x \geqslant 0 +$$ + +Comment 1. A more general inequality + +$$ +\sum_{i=1}^{n} \sum_{j=1}^{n}\left|x_{i}-x_{j}\right|^{r} \leqslant \sum_{i=1}^{n} \sum_{j=1}^{n}\left|x_{i}+x_{j}\right|^{r} +$$ + +holds for any $r \in[0,2]$. The first solution can be repeated verbatim for any $r \in[0,1]$ but not for $r>1$. In the second solution, by putting $x^{r+1}$ in the denominator in place of $x \sqrt{x}$ we can prove the inequality for any $r \in(0,2)$ and the cases $r=0,2$ are easy to check by hand. +Comment 2. In fact, the integral from Solution 2 can be computed explicitly, we have $I(1)=\sqrt{2 \pi}$. + +A5. Let $n \geqslant 2$ be an integer, and let $a_{1}, a_{2}, \ldots, a_{n}$ be positive real numbers such that $a_{1}+a_{2}+\cdots+a_{n}=1$. Prove that + +$$ +\sum_{k=1}^{n} \frac{a_{k}}{1-a_{k}}\left(a_{1}+a_{2}+\cdots+a_{k-1}\right)^{2}<\frac{1}{3} +$$ + +Solution 1. For all $k \leqslant n$, let + +$$ +s_{k}=a_{1}+a_{2}+\cdots+a_{k} \quad \text { and } \quad b_{k}=\frac{a_{k} s_{k-1}^{2}}{1-a_{k}} +$$ + +with the convention that $s_{0}=0$. Note that $b_{k}$ is exactly a summand in the sum we need to estimate. We shall prove the inequality + +$$ +b_{k}<\frac{s_{k}^{3}-s_{k-1}^{3}}{3} +$$ + +Indeed, it suffices to check that + +$$ +\begin{aligned} +(1) & \Longleftrightarrow 0<\left(1-a_{k}\right)\left(\left(s_{k-1}+a_{k}\right)^{3}-s_{k-1}^{3}\right)-3 a_{k} s_{k-1}^{2} \\ +& \Longleftrightarrow 0<\left(1-a_{k}\right)\left(3 s_{k-1}^{2}+3 s_{k-1} a_{k}+a_{k}^{2}\right)-3 s_{k-1}^{2} \\ +& \Longleftrightarrow 0<-3 a_{k} s_{k-1}^{2}+3\left(1-a_{k}\right) s_{k-1} a_{k}+\left(1-a_{k}\right) a_{k}^{2} \\ +& \Longleftrightarrow 0<3\left(1-a_{k}-s_{k-1}\right) s_{k-1} a_{k}+\left(1-a_{k}\right) a_{k}^{2} +\end{aligned} +$$ + +which holds since $a_{k}+s_{k-1}=s_{k} \leqslant 1$ and $a_{k} \in(0,1)$. +Thus, adding inequalities (1) for $k=1, \ldots, n$, we conclude that + +$$ +b_{1}+b_{2}+\cdots+b_{n}<\frac{s_{n}^{3}-s_{0}^{3}}{3}=\frac{1}{3} +$$ + +as desired. +Comment 1. There are many ways of proving (1) which can be written as + +$$ +\frac{a s^{2}}{1-a}-\frac{(a+s)^{3}-s^{3}}{3}<0 +$$ + +for non-negative $a$ and $s$ satisfying $a+s \leqslant 1$ and $a>0$. +E.g., note that for any fixed $a$ the expression in (2) is quadratic in $s$ with the leading coefficient $a /(1-a)-a>0$. Hence, it is convex as a function in $s$, so it suffices to check the inequality at $s=0$ and $s=1-a$. The former case is trivial and in the latter case the inequality can be rewritten as + +$$ +a s-\frac{3 a s(a+s)+a^{3}}{3}<0, +$$ + +which is trivial since $a+s=1$. +Solution 2. First, let us define + +$$ +S\left(a_{1}, \ldots, a_{n}\right):=\sum_{k=1}^{n} \frac{a_{k}}{1-a_{k}}\left(a_{1}+a_{2}+\cdots+a_{k-1}\right)^{2} +$$ + +For some index $i$, denote $a_{1}+\cdots+a_{i-1}$ by $s$. If we replace $a_{i}$ with two numbers $a_{i} / 2$ and $a_{i} / 2$, i.e. replace the tuple $\left(a_{1}, \ldots, a_{n}\right)$ with $\left(a_{1}, \ldots, a_{i-1}, a_{i} / 2, a_{i} / 2, a_{i+1}, \ldots, a_{n}\right)$, the sum will increase by + +$$ +\begin{aligned} +S\left(a_{1}, \ldots, a_{i} / 2, a_{i} / 2, \ldots, a_{n}\right)-S\left(a_{1}, \ldots, a_{n}\right) & =\frac{a_{i} / 2}{1-a_{i} / 2}\left(s^{2}+\left(s+a_{i} / 2\right)^{2}\right)-\frac{a_{i}}{1-a_{i}} s^{2} \\ +& =a_{i} \frac{\left(1-a_{i}\right)\left(2 s^{2}+s a_{i}+a_{i}^{2} / 4\right)-\left(2-a_{i}\right) s^{2}}{\left(2-a_{i}\right)\left(1-a_{i}\right)} \\ +& =a_{i} \frac{\left(1-a_{i}-s\right) s a_{i}+\left(1-a_{i}\right) a_{i}^{2} / 4}{\left(2-a_{i}\right)\left(1-a_{i}\right)} +\end{aligned} +$$ + +which is strictly positive. So every such replacement strictly increases the sum. By repeating this process and making maximal number in the tuple tend to zero, we keep increasing the sum which will converge to + +$$ +\int_{0}^{1} x^{2} d x=\frac{1}{3} . +$$ + +This completes the proof. +Solution 3. We sketch a probabilistic version of the first solution. Let $x_{1}, x_{2}, x_{3}$ be drawn uniformly and independently at random from the segment [0,1]. Let $I_{1} \cup I_{2} \cup \cdots \cup I_{n}$ be a partition of $[0,1]$ into segments of length $a_{1}, a_{2}, \ldots, a_{n}$ in this order. Let $J_{k}:=I_{1} \cup \cdots \cup I_{k-1}$ for $k \geqslant 2$ and $J_{1}:=\varnothing$. Then + +$$ +\begin{aligned} +\frac{1}{3}= & \sum_{k=1}^{n} \mathbb{P}\left\{x_{1} \geqslant x_{2}, x_{3} ; x_{1} \in I_{k}\right\} \\ += & \sum_{k=1}^{n}\left(\mathbb{P}\left\{x_{1} \in I_{k} ; x_{2}, x_{3} \in J_{k}\right\}\right. \\ ++ & 2 \cdot \mathbb{P}\left\{x_{1} \geqslant x_{2} ; x_{1}, x_{2} \in I_{k} ; x_{3} \in J_{k}\right\} \\ +& \left.+\mathbb{P}\left\{x_{1} \geqslant x_{2}, x_{3} ; x_{1}, x_{2}, x_{3} \in I_{k}\right\}\right) \\ += & \sum_{k=1}^{n}\left(a_{k}\left(a_{1}+\cdots+a_{k-1}\right)^{2}+2 \cdot \frac{a_{k}^{2}}{2} \cdot\left(a_{1}+\cdots+a_{k-1}\right)+\frac{a_{k}^{3}}{3}\right) \\ +> & \sum_{k=1}^{n}\left(a_{k}\left(a_{1}+\cdots+a_{k-1}\right)^{2}+a_{k}^{2}\left(a_{1}+\cdots+a_{k-1}\right) \cdot \frac{a_{1}+\cdots+a_{k-1}}{1-a_{k}}\right) +\end{aligned} +$$ + +where for the last inequality we used that $1-a_{k} \geqslant a_{1}+\cdots+a_{k-1}$. This completes the proof since + +$$ +a_{k}+\frac{a_{k}^{2}}{1-a_{k}}=\frac{a_{k}}{1-a_{k}} +$$ + +A6. Let $A$ be a finite set of (not necessarily positive) integers, and let $m \geqslant 2$ be an integer. Assume that there exist non-empty subsets $B_{1}, B_{2}, B_{3}, \ldots, B_{m}$ of $A$ whose elements add up to the sums $m^{1}, m^{2}, m^{3}, \ldots, m^{m}$, respectively. Prove that $A$ contains at least $m / 2$ elements. + +Solution. Let $A=\left\{a_{1}, \ldots, a_{k}\right\}$. Assume that, on the contrary, $k=|A|0$ and large enough $m$ we get $k \geqslant(1 / 2-\varepsilon) m$. The proof uses the fact that the combinations $\sum c_{i}$ ! with $c_{i} \in\{0,1, \ldots, i\}$ are all distinct. + +Comment 2. The problem statement holds also if $A$ is a set of real numbers (not necessarily integers), the above proofs work in the real case. + +A7. Let $n \geqslant 1$ be an integer, and let $x_{0}, x_{1}, \ldots, x_{n+1}$ be $n+2$ non-negative real numbers that satisfy $x_{i} x_{i+1}-x_{i-1}^{2} \geqslant 1$ for all $i=1,2, \ldots, n$. Show that + +$$ +x_{0}+x_{1}+\cdots+x_{n}+x_{n+1}>\left(\frac{2 n}{3}\right)^{3 / 2} +$$ + +## Solution 1. + +Lemma 1.1. If $a, b, c$ are non-negative numbers such that $a b-c^{2} \geqslant 1$, then + +$$ +(a+2 b)^{2} \geqslant(b+2 c)^{2}+6 +$$ + +Proof. $(a+2 b)^{2}-(b+2 c)^{2}=(a-b)^{2}+2(b-c)^{2}+6\left(a b-c^{2}\right) \geqslant 6$. +Lemma 1.2. $\sqrt{1}+\cdots+\sqrt{n}>\frac{2}{3} n^{3 / 2}$. +Proof. Bernoulli's inequality $(1+t)^{3 / 2}>1+\frac{3}{2} t$ for $0>t \geqslant-1$ (or, alternatively, a straightforward check) gives + +$$ +(k-1)^{3 / 2}=k^{3 / 2}\left(1-\frac{1}{k}\right)^{3 / 2}>k^{3 / 2}\left(1-\frac{3}{2 k}\right)=k^{3 / 2}-\frac{3}{2} \sqrt{k} . +$$ + +Summing up (*) over $k=1,2, \ldots, n$ yields + +$$ +0>n^{3 / 2}-\frac{3}{2}(\sqrt{1}+\cdots+\sqrt{n}) . +$$ + +Now put $y_{i}:=2 x_{i}+x_{i+1}$ for $i=0,1, \ldots, n$. We get $y_{0} \geqslant 0$ and $y_{i}^{2} \geqslant y_{i-1}^{2}+6$ for $i=1,2, \ldots, n$ by Lemma 1.1. Thus, an easy induction on $i$ gives $y_{i} \geqslant \sqrt{6 i}$. Using this estimate and Lemma 1.2 we get + +$$ +3\left(x_{0}+\ldots+x_{n+1}\right) \geqslant y_{1}+\ldots+y_{n} \geqslant \sqrt{6}(\sqrt{1}+\sqrt{2}+\ldots+\sqrt{n})>\sqrt{6} \cdot \frac{2}{3} n^{3 / 2}=3\left(\frac{2 n}{3}\right)^{3 / 2} +$$ + +Solution 2. Say that an index $i \in\{0,1, \ldots, n+1\}$ is good, if $x_{i} \geqslant \sqrt{\frac{2}{3}} i$, otherwise call the index $i$ bad. +Lemma 2.1. There are no two consecutive bad indices. +Proof. Assume the contrary and consider two bad indices $j, j+1$ with minimal possible $j$. Since 0 is good, we get $j>0$, thus by minimality $j-1$ is a good index and we have + +$$ +\frac{2}{3} \sqrt{j(j+1)}>x_{j} x_{j+1} \geqslant x_{j-1}^{2}+1 \geqslant \frac{2}{3}(j-1)+1=\frac{2}{3} \cdot \frac{j+(j+1)}{2} +$$ + +that contradicts the AM-GM inequality for numbers $j$ and $j+1$. +Lemma 2.2. If an index $j \leqslant n-1$ is good, then + +$$ +x_{j+1}+x_{j+2} \geqslant \sqrt{\frac{2}{3}}(\sqrt{j+1}+\sqrt{j+2}) . +$$ + +Proof. We have + +$$ +x_{j+1}+x_{j+2} \geqslant 2 \sqrt{x_{j+1} x_{j+2}} \geqslant 2 \sqrt{x_{j}^{2}+1} \geqslant 2 \sqrt{\frac{2}{3} j+1} \geqslant \sqrt{\frac{2}{3} j+\frac{2}{3}}+\sqrt{\frac{2}{3} j+\frac{4}{3}}, +$$ + +the last inequality follows from concavity of the square root function, or, alternatively, from the AM-QM inequality for the numbers $\sqrt{\frac{2}{3} j+\frac{2}{3}}$ and $\sqrt{\frac{2}{3} j+\frac{4}{3}}$. + +Let $S_{i}=x_{1}+\ldots+x_{i}$ and $T_{i}=\sqrt{\frac{2}{3}}(\sqrt{1}+\ldots+\sqrt{i})$. +Lemma 2.3. If an index $i$ is good, then $S_{i} \geqslant T_{i}$. +Proof. Induction on $i$. The base case $i=0$ is clear. Assume that the claim holds for good indices less than $i$ and prove it for a good index $i>0$. + +If $i-1$ is good, then by the inductive hypothesis we get $S_{i}=S_{i-1}+x_{i} \geqslant T_{i-1}+\sqrt{\frac{2}{3} i}=T_{i}$. +If $i-1$ is bad, then $i>1$, and $i-2$ is good by Lemma 2.1. Then using Lemma 2.2 and the inductive hypothesis we get + +$$ +S_{i}=S_{i-2}+x_{i-1}+x_{i} \geqslant T_{i-2}+\sqrt{\frac{2}{3}}(\sqrt{i-1}+\sqrt{i})=T_{i} +$$ + +Since either $n$ or $n+1$ is good by Lemma 2.1, Lemma 2.3 yields in both cases $S_{n+1} \geqslant T_{n}$, and it remains to apply Lemma 1.2 from Solution 1. + +Comment 1. Another way to get (*) is the integral bound + +$$ +k^{3 / 2}-(k-1)^{3 / 2}=\int_{k-1}^{k} \frac{3}{2} \sqrt{x} d x<\frac{3}{2} \sqrt{k} . +$$ + +Comment 2. If $x_{i}=\sqrt{2 / 3} \cdot(\sqrt{i}+1)$, the conditions of the problem hold. Indeed, the inequality to check is + +$$ +(\sqrt{i}+1)(\sqrt{i+1}+1)-(\sqrt{i-1}+1)^{2} \geqslant 3 / 2 +$$ + +that rewrites as + +$$ +\sqrt{i}+\sqrt{i+1}-2 \sqrt{i-1} \geqslant(i+1 / 2)-\sqrt{i(i+1)}=\frac{1 / 4}{i+1 / 2+\sqrt{i(i+1)}}, +$$ + +which follows from + +$$ +\sqrt{i}-\sqrt{i-1}=\frac{1}{\sqrt{i}+\sqrt{i-1}}>\frac{1}{2 i} +$$ + +For these numbers we have $x_{0}+\ldots+x_{n+1}=\left(\frac{2 n}{3}\right)^{3 / 2}+O(n)$, thus the multiplicative constant $(2 / 3)^{3 / 2}$ in the problem statement is sharp. + +A8. Determine all functions $f: \mathbb{R} \rightarrow \mathbb{R}$ that satisfy + +$$ +(f(a)-f(b))(f(b)-f(c))(f(c)-f(a))=f\left(a b^{2}+b c^{2}+c a^{2}\right)-f\left(a^{2} b+b^{2} c+c^{2} a\right) +$$ + +for all real numbers $a, b, c$. +Answer: $f(x)=\alpha x+\beta$ or $f(x)=\alpha x^{3}+\beta$ where $\alpha \in\{-1,0,1\}$ and $\beta \in \mathbb{R}$. +Solution. It is straightforward to check that above functions satisfy the equation. Now let $f(x)$ satisfy the equation, which we denote $E(a, b, c)$. Then clearly $f(x)+C$ also does; therefore, we may suppose without loss of generality that $f(0)=0$.We start with proving +Lemma. Either $f(x) \equiv 0$ or $f$ is injective. +Proof. Denote by $\Theta \subseteq \mathbb{R}^{2}$ the set of points $(a, b)$ for which $f(a)=f(b)$. Let $\Theta^{*}=\{(x, y) \in \Theta$ : $x \neq y\}$. The idea is that if $(a, b) \in \Theta$, then by $E(a, b, x)$ we get + +$$ +H_{a, b}(x):=\left(a b^{2}+b x^{2}+x a^{2}, a^{2} b+b^{2} x+x^{2} a\right) \in \Theta +$$ + +for all real $x$. Reproducing this argument starting with $(a, b) \in \Theta^{*}$, we get more and more points in $\Theta$. There are many ways to fill in the details, we give below only one of them. + +Assume that $(a, b) \in \Theta^{*}$. Note that + +$$ +g_{-}(x):=\left(a b^{2}+b x^{2}+x a^{2}\right)-\left(a^{2} b+b^{2} x+x^{2} a\right)=(a-b)(b-x)(x-a) +$$ + +and + +$$ +g_{+}(x):=\left(a b^{2}+b x^{2}+x a^{2}\right)+\left(a^{2} b+b^{2} x+x^{2} a\right)=\left(x^{2}+a b\right)(a+b)+x\left(a^{2}+b^{2}\right) . +$$ + +Hence, there exists $x$ for which both $g_{-}(x) \neq 0$ and $g_{+}(x) \neq 0$. This gives a point $(\alpha, \beta)=$ $H_{a, b}(x) \in \Theta^{*}$ for which $\alpha \neq-\beta$. Now compare $E(\alpha, 1,0)$ and $E(\beta, 1,0)$. The left-hand side expressions coincide, on right-hand side we get $f(\alpha)-f\left(\alpha^{2}\right)=f(\beta)-f\left(\beta^{2}\right)$, respectively. Hence, $f\left(\alpha^{2}\right)=f\left(\beta^{2}\right)$ and we get a point $\left(\alpha_{1}, \beta_{1}\right):=\left(\alpha^{2}, \beta^{2}\right) \in \Theta^{*}$ with both coordinates $\alpha_{1}, \beta_{1}$ non-negative. Continuing squaring the coordinates, we get a point $(\gamma, \delta) \in \Theta^{*}$ for which $\delta>5 \gamma \geqslant 0$. Our nearest goal is to get a point $(0, r) \in \Theta^{*}$. If $\gamma=0$, this is already done. If $\gamma>0$, denote by $x$ a real root of the quadratic equation $\delta \gamma^{2}+\gamma x^{2}+x \delta^{2}=0$, which exists since the discriminant $\delta^{4}-4 \delta \gamma^{3}$ is positive. Also $x<0$ since this equation cannot have non-negative root. For the point $H_{\delta, \gamma}(x)=:(0, r) \in \Theta$ the first coordinate is 0 . The difference of coordinates equals $-r=(\delta-\gamma)(\gamma-x)(x-\delta)<0$, so $r \neq 0$ as desired. + +Now, let $(0, r) \in \Theta^{*}$. We get $H_{0, r}(x)=\left(r x^{2}, r^{2} x\right) \in \Theta$. Thus $f\left(r x^{2}\right)=f\left(r^{2} x\right)$ for all $x \in \mathbb{R}$. Replacing $x$ to $-x$ we get $f\left(r x^{2}\right)=f\left(r^{2} x\right)=f\left(-r^{2} x\right)$, so $f$ is even: $(a,-a) \in \Theta$ for all $a$. Then $H_{a,-a}(x)=\left(a^{3}-a x^{2}+x a^{2},-a^{3}+a^{2} x+x^{2} a\right) \in \Theta$ for all real $a, x$. Putting $x=\frac{1+\sqrt{5}}{2} a$ we obtain $\left(0,(1+\sqrt{5}) a^{3}\right) \in \Theta$ which means that $f(y)=f(0)=0$ for every real $y$. + +Hereafter we assume that $f$ is injective and $f(0)=0$. By $E(a, b, 0)$ we get + +$$ +f(a) f(b)(f(a)-f(b))=f\left(a^{2} b\right)-f\left(a b^{2}\right) . +$$ + +Let $\kappa:=f(1)$ and note that $\kappa=f(1) \neq f(0)=0$ by injectivity. Putting $b=1$ in ( $\Omega$ ) we get + +$$ +\kappa f(a)(f(a)-\kappa)=f\left(a^{2}\right)-f(a) . +$$ + +Subtracting the same equality for $-a$ we get + +$$ +\kappa(f(a)-f(-a))(f(a)+f(-a)-\kappa)=f(-a)-f(a) . +$$ + +Now, if $a \neq 0$, by injectivity we get $f(a)-f(-a) \neq 0$ and thus + +$$ +f(a)+f(-a)=\kappa-\kappa^{-1}=: \lambda . +$$ + +It follows that + +$$ +f(a)-f(b)=f(-b)-f(-a) +$$ + +for all non-zero $a, b$. Replace non-zero numbers $a, b$ in ( $\odot$ ) with $-a,-b$, respectively, and add the two equalities. Due to ( $\boldsymbol{\uparrow}$ ) we get + +$$ +(f(a)-f(b))(f(a) f(b)-f(-a) f(-b))=0 +$$ + +thus $f(a) f(b)=f(-a) f(-b)=(\lambda-f(a))(\lambda-f(b))$ for all non-zero $a \neq b$. If $\lambda \neq 0$, this implies $f(a)+f(b)=\lambda$ that contradicts injectivity when we vary $b$ with fixed $a$. Therefore, $\lambda=0$ and $\kappa= \pm 1$. Thus $f$ is odd. Replacing $f$ with $-f$ if necessary (this preserves the original equation) we may suppose that $f(1)=1$. + +Now, (\&) yields $f\left(a^{2}\right)=f^{2}(a)$. Summing relations $(\bigcirc)$ for pairs $(a, b)$ and $(a,-b)$, we get $-2 f(a) f^{2}(b)=-2 f\left(a b^{2}\right)$, i.e. $f(a) f\left(b^{2}\right)=f\left(a b^{2}\right)$. Putting $b=\sqrt{x}$ for each non-negative $x$ we get $f(a x)=f(a) f(x)$ for all real $a$ and non-negative $x$. Since $f$ is odd, this multiplicativity relation is true for all $a, x$. Also, from $f\left(a^{2}\right)=f^{2}(a)$ we see that $f(x) \geqslant 0$ for $x \geqslant 0$. Next, $f(x)>0$ for $x>0$ by injectivity. + +Assume that $f(x)$ for $x>0$ does not have the form $f(x)=x^{\tau}$ for a constant $\tau$. The known property of multiplicative functions yields that the graph of $f$ is dense on $(0, \infty)^{2}$. In particular, we may find positive $b<1 / 10$ for which $f(b)>1$. Also, such $b$ can be found if $f(x)=x^{\tau}$ for some $\tau<0$. Then for all $x$ we have $x^{2}+x b^{2}+b \geqslant 0$ and so $E(1, b, x)$ implies that + +$$ +f\left(b^{2}+b x^{2}+x\right)=f\left(x^{2}+x b^{2}+b\right)+(f(b)-1)(f(x)-f(b))(f(x)-1) \geqslant 0-\left((f(b)-1)^{3} / 4\right. +$$ + +is bounded from below (the quadratic trinomial bound $(t-f(1))(t-f(b)) \geqslant-(f(b)-1)^{2} / 4$ for $t=f(x)$ is used). Hence, $f$ is bounded from below on $\left(b^{2}-\frac{1}{4 b},+\infty\right)$, and since $f$ is odd it is bounded from above on $\left(0, \frac{1}{4 b}-b^{2}\right)$. This is absurd if $f(x)=x^{\tau}$ for $\tau<0$, and contradicts to the above dense graph condition otherwise. + +Therefore, $f(x)=x^{\tau}$ for $x>0$ and some constant $\tau>0$. Dividing $E(a, b, c)$ by $(a-b)(b-$ $c)(c-a)=\left(a b^{2}+b c^{2}+c a^{2}\right)-\left(a^{2} b+b^{2} c+c^{2} a\right)$ and taking a limit when $a, b, c$ all go to 1 (the divided ratios tend to the corresponding derivatives, say, $\frac{a^{\tau}-b^{\tau}}{a-b} \rightarrow\left(x^{\tau}\right)_{x=1}^{\prime}=\tau$ ), we get $\tau^{3}=\tau \cdot 3^{\tau-1}, \tau^{2}=3^{\tau-1}, F(\tau):=3^{\tau / 2-1 / 2}-\tau=0$. Since function $F$ is strictly convex, it has at most two roots, and we get $\tau \in\{1,3\}$. + +## Combinatorics + +C1. Let $S$ be an infinite set of positive integers, such that there exist four pairwise distinct $a, b, c, d \in S$ with $\operatorname{gcd}(a, b) \neq \operatorname{gcd}(c, d)$. Prove that there exist three pairwise distinct $x, y, z \in S$ such that $\operatorname{gcd}(x, y)=\operatorname{gcd}(y, z) \neq \operatorname{gcd}(z, x)$. + +Solution. There exists $\alpha \in S$ so that $\{\operatorname{gcd}(\alpha, s) \mid s \in S, s \neq \alpha\}$ contains at least two elements. Since $\alpha$ has only finitely many divisors, there is a $d \mid \alpha$ such that the set $B=\{\beta \in$ $S \mid \operatorname{gcd}(\alpha, \beta)=d\}$ is infinite. Pick $\gamma \in S$ so that $\operatorname{gcd}(\alpha, \gamma) \neq d$. Pick $\beta_{1}, \beta_{2} \in B$ so that $\operatorname{gcd}\left(\beta_{1}, \gamma\right)=\operatorname{gcd}\left(\beta_{2}, \gamma\right)=: d^{\prime}$. If $d=d^{\prime}$, then $\operatorname{gcd}\left(\alpha, \beta_{1}\right)=\operatorname{gcd}\left(\gamma, \beta_{1}\right) \neq \operatorname{gcd}(\alpha, \gamma)$. If $d \neq d^{\prime}$, then either $\operatorname{gcd}\left(\alpha, \beta_{1}\right)=\operatorname{gcd}\left(\alpha, \beta_{2}\right)=d$ and $\operatorname{gcd}\left(\beta_{1}, \beta_{2}\right) \neq d$ or $\operatorname{gcd}\left(\gamma, \beta_{1}\right)=\operatorname{gcd}\left(\gamma, \beta_{2}\right)=d^{\prime}$ and $\operatorname{gcd}\left(\beta_{1}, \beta_{2}\right) \neq d^{\prime}$. + +Comment. The situation can be modelled as a complete graph on the infinite vertex set $S$, where every edge $\{s, t\}$ is colored by $c(s, t):=\operatorname{gcd}(s, t)$. For every vertex the incident edges carry only finitely many different colors, and by the problem statement at least two different colors show up on the edge set. The goal is to show that there exists a bi-colored triangle (a triangle, whose edges carry exactly two different colors). + +For the proof, consider a vertex $v$ whose incident edges carry at least two different colors. Let $X \subset S$ be an infinite subset so that $c(v, x) \equiv c_{1}$ for all $x \in X$. Let $y \in S$ be a vertex so that $c(v, y) \neq c_{1}$. Let $x_{1}, x_{2} \in X$ be two vertices with $c\left(y, x_{1}\right)=c\left(y, x_{2}\right)=c_{2}$. If $c_{1}=c_{2}$, then the triangle $v, y, x_{1}$ is bi-colored. If $c_{1} \neq c_{2}$, then one of $v, x_{1}, x_{2}$ and $y, x_{1}, x_{2}$ is bi-colored. + +C2. Let $n \geqslant 3$ be an integer. An integer $m \geqslant n+1$ is called $n$-colourful if, given infinitely many marbles in each of $n$ colours $C_{1}, C_{2}, \ldots, C_{n}$, it is possible to place $m$ of them around a circle so that in any group of $n+1$ consecutive marbles there is at least one marble of colour $C_{i}$ for each $i=1, \ldots, n$. + +Prove that there are only finitely many positive integers which are not $n$-colourful. Find the largest among them. + +Answer: $m_{\max }=n^{2}-n-1$. +Solution. First suppose that there are $n(n-1)-1$ marbles. Then for one of the colours, say blue, there are at most $n-2$ marbles, which partition the non-blue marbles into at most $n-2$ groups with at least $(n-1)^{2}>n(n-2)$ marbles in total. Thus one of these groups contains at least $n+1$ marbles and this group does not contain any blue marble. + +Now suppose that the total number of marbles is at least $n(n-1)$. Then we may write this total number as $n k+j$ with some $k \geqslant n-1$ and with $0 \leqslant j \leqslant n-1$. We place around a circle $k-j$ copies of the colour sequence $[1,2,3, \ldots, n]$ followed by $j$ copies of the colour sequence $[1,1,2,3, \ldots, n]$. + +C3. A thimblerigger has 2021 thimbles numbered from 1 through 2021. The thimbles are arranged in a circle in arbitrary order. The thimblerigger performs a sequence of 2021 moves; in the $k^{\text {th }}$ move, he swaps the positions of the two thimbles adjacent to thimble $k$. + +Prove that there exists a value of $k$ such that, in the $k^{\text {th }}$ move, the thimblerigger swaps some thimbles $a$ and $b$ such that $ak \geqslant 1$. There are $2 n+1$ students standing in a circle. Each student $S$ has $2 k$ neighbours - namely, the $k$ students closest to $S$ on the right, and the $k$ students closest to $S$ on the left. + +Suppose that $n+1$ of the students are girls, and the other $n$ are boys. Prove that there is a girl with at least $k$ girls among her neighbours. + +Solution. We replace the girls by 1's, and the boys by 0 's, getting the numbers $a_{1}, a_{2}, \ldots, a_{2 n+1}$ arranged in a circle. We extend this sequence periodically by letting $a_{2 n+1+k}=a_{k}$ for all $k \in \mathbb{Z}$. We get an infinite periodic sequence + +$$ +\ldots, a_{1}, a_{2}, \ldots, a_{2 n+1}, a_{1}, a_{2}, \ldots, a_{2 n+1}, \ldots +$$ + +Consider the numbers $b_{i}=a_{i}+a_{i-k-1}-1 \in\{-1,0,1\}$ for all $i \in \mathbb{Z}$. We know that + +$$ +b_{m+1}+b_{m+2}+\cdots+b_{m+2 n+1}=1 \quad(m \in \mathbb{Z}) +$$ + +in particular, this yields that there exists some $i_{0}$ with $b_{i_{0}}=1$. Now we want to find an index $i$ such that + +$$ +b_{i}=1 \quad \text { and } \quad b_{i+1}+b_{i+2}+\cdots+b_{i+k} \geqslant 0 +$$ + +This will imply that $a_{i}=1$ and + +$$ +\left(a_{i-k}+a_{i-k+1}+\cdots+a_{i-1}\right)+\left(a_{i+1}+a_{i+2}+\cdots+a_{i+k}\right) \geqslant k +$$ + +as desired. +Suppose, to the contrary, that for every index $i$ with $b_{i}=1$ the sum $b_{i+1}+b_{i+2}+\cdots+b_{i+k}$ is negative. We start from some index $i_{0}$ with $b_{i_{0}}=1$ and construct a sequence $i_{0}, i_{1}, i_{2}, \ldots$, where $i_{j}(j>0)$ is the smallest possible index such that $i_{j}>i_{j-1}+k$ and $b_{i_{j}}=1$. We can choose two numbers among $i_{0}, i_{1}, \ldots, i_{2 n+1}$ which are congruent modulo $2 n+1$ (without loss of generality, we may assume that these numbers are $i_{0}$ and $i_{T}$ ). + +On the one hand, for every $j$ with $0 \leqslant j \leqslant T-1$ we have + +$$ +S_{j}:=b_{i_{j}}+b_{i_{j}+1}+b_{i_{j}+2}+\cdots+b_{i_{j+1}-1} \leqslant b_{i_{j}}+b_{i_{j}+1}+b_{i_{j}+2}+\cdots+b_{i_{j}+k} \leqslant 0 +$$ + +since $b_{i_{j}+k+1}, \ldots, b_{i_{j+1}-1} \leqslant 0$. On the other hand, since $\left(i_{T}-i_{0}\right) \mid(2 n+1)$, from (1) we deduce + +$$ +S_{0}+\cdots+S_{T-1}=\sum_{i=i_{0}}^{i_{T}-1} b_{i}=\frac{i_{T}-i_{0}}{2 n+1}>0 +$$ + +This contradiction finishes the solution. +Comment 1. After the problem is reduced to finding an index $i$ satisfying (2), one can finish the solution by applying the (existence part of) following statement. +Lemma (Raney). If $\left\langle x_{1}, x_{2}, \ldots, x_{m}\right\rangle$ is any sequence of integers whose sum is +1 , exactly one of the cyclic shifts $\left\langle x_{1}, x_{2}, \ldots, x_{m}\right\rangle,\left\langle x_{2}, \ldots, x_{m}, x_{1}\right\rangle, \ldots,\left\langle x_{m}, x_{1}, \ldots, x_{m-1}\right\rangle$ has all of its partial sums positive. + +A (possibly wider known) version of this lemma, which also can be used in order to solve the problem, is the following +Claim (Gas stations problem). Assume that there are several fuel stations located on a circular route which together contain just enough gas to make one trip around. Then one can make it all the way around, starting at the right station with an empty tank. + +Both Raney's theorem and the Gas stations problem admit many different (parallel) proofs. Their ideas can be disguised in direct solutions of the problem at hand (as it, in fact, happens in the above solution); such solutions may avoid the introduction of the $b_{i}$. Below, in Comment 2 we present a variant of such solution, while in Comment 3 we present an alternative proof of Raney's theorem. + +Comment 2. Here is a version of the solution which avoids the use of the $b_{i}$. +Suppose the contrary. Introduce the numbers $a_{i}$ as above. Starting from any index $s_{0}$ with $a_{s_{0}}=1$, we construct a sequence $s_{0}, s_{1}, s_{2}, \ldots$ by letting $s_{i}$ to be the smallest index larger than $s_{i-1}+k$ such that $a_{s_{i}}=1$, for $i=1,2, \ldots$. Choose two indices among $s_{1}, \ldots, s_{2 n+1}$ which are congruent modulo $2 n+1$; we assume those two are $s_{0}$ and $s_{T}$, with $s_{T}-s_{0}=t(2 n+1)$. Notice here that $s_{T+1}-s_{T}=s_{1}-s_{0}$. + +For every $i=0,1,2, \ldots, T$, put + +$$ +L_{i}=s_{i+1}-s_{i} \quad \text { and } \quad S_{i}=a_{s_{i}}+a_{s_{i}+1}+\cdots+a_{s_{i+1}-1} . +$$ + +Now, by the indirect assumption, for every $i=1,2, \ldots, T$, we have + +$$ +a_{s_{i}-k}+a_{s_{i}-k+1}+\cdots+a_{s_{i}+k} \leqslant a_{s_{i}}+(k-1)=k . +$$ + +Recall that $a_{j}=0$ for all $j$ with $s_{i}+kB C$. Suppose that $A D>D C$, and let $H=A C \cap B D$. Then the rays $B B_{1}$ and $D D_{1}$ lie on one side of $B D$, as they contain the midpoints of the arcs $A D C$ and $A B C$, respectively. However, if $B D_{1} \| D B_{1}$, then $B_{1}$ and $D_{1}$ should be separated by $H$. This contradiction shows that $A D\overparen{C B}$. In the right-angled triangle $B B_{1} B_{2}$, the point $L_{B}$ is a point on the hypothenuse such that $L_{B} B_{1}=L_{B} B$, so $L_{B}$ is the midpoint of $B_{1} B_{2}$. + +Since $D D_{1}$ is the internal angle bisector of $\angle A D C$, we have + +$$ +\angle B D D_{1}=\frac{\angle B D A-\angle C D B}{2}=\frac{\angle B C A-\angle C A B}{2}=\angle B B_{2} D_{1}, +$$ + +so the points $B, B_{2}, D$, and $D_{1}$ lie on some circle $\omega_{B}$. Similarly, $L_{D}$ is the midpoint of $D_{1} D_{2}$, and the points $D, D_{2}, B$, and $B_{1}$ lie on some circle $\omega_{D}$. + +Now we have + +$$ +\angle B_{2} D B_{1}=\angle B_{2} D B-\angle B_{1} D B=\angle B_{2} D_{1} B-\angle B_{1} D_{2} B=\angle D_{2} B D_{1} . +$$ + +Therefore, the corresponding sides of the triangles $D B_{1} B_{2}$ and $B D_{1} D_{2}$ are parallel, and the triangles are homothetical (in $H$ ). So their corresponding medians $D L_{B}$ and $B L_{D}$ are also parallel. +![](https://cdn.mathpix.com/cropped/2024_11_18_7107063274cfa469597ag-53.jpg?height=535&width=1492&top_left_y=1914&top_left_x=285) + +Yet alternatively, after obtaining the circles $\omega_{B}$ and $\omega_{D}$, one may notice that $H$ lies on their radical axis $B D$, whence $H B_{1} \cdot H D_{2}=H D_{1} \cdot H B_{2}$, or + +$$ +\frac{H B_{1}}{H D_{1}}=\frac{H B_{2}}{H D_{1}} . +$$ + +Since $h\left(D_{1}\right)=B_{1}$, this yields $h\left(D_{2}\right)=B_{2}$ and hence $h\left(L_{D}\right)=L_{B}$. + +Comment 3. Since $h$ preserves the line $A C$ and maps $B \mapsto D$ and $D_{1} \mapsto B_{1}$, we have $h\left(\gamma_{B}\right)=\gamma_{D}$. Therefore, $h\left(O_{B}\right)=O_{D}$; in particular, $H$ also lies on $O_{B} O_{D}$. + +Solution 2. Let $B D_{1}$ and $T_{B} D_{1}$ meet $\Omega$ again at $X_{B}$ and $Y_{B}$, respectively. Then + +$$ +\angle B D_{1} C=\angle B T_{B} D_{1}=\angle B T_{B} Y_{B}=\angle B X_{B} Y_{B} +$$ + +which shows that $X_{B} Y_{B} \| A C$. Similarly, let $D B_{1}$ and $T_{D} B_{1}$ meet $\Omega$ again at $X_{D}$ and $Y_{D}$, respectively; then $X_{D} Y_{D} \| A C$. + +Let $M_{D}$ and $M_{B}$ be the midpoints of the arcs $A B C$ and $A D C$, respectively; then the points $D_{1}$ and $B_{1}$ lie on $D M_{D}$ and $B M_{B}$, respectively. Let $K$ be the midpoint of $A C$ (which lies on $M_{B} M_{D}$ ). Applying Pascal's theorem to $M_{D} D X_{D} X_{B} B M_{B}$, we obtain that the points $D_{1}=M_{D} D \cap X_{B} B, B_{1}=D X_{D} \cap B M_{B}$, and $X_{D} X_{B} \cap M_{B} M_{D}$ are collinear, which means that $X_{B} X_{D}$ passes through $K$. Due to symmetry, the diagonals of an isosceles trapezoid $X_{B} Y_{B} X_{D} Y_{D}$ cross at $K$. +![](https://cdn.mathpix.com/cropped/2024_11_18_7107063274cfa469597ag-54.jpg?height=1143&width=1164&top_left_y=888&top_left_x=446) + +Let $b$ and $d$ denote the distances from the lines $X_{B} Y_{B}$ and $X_{D} Y_{D}$, respectively, to $A C$. Then we get + +$$ +\frac{X_{B} Y_{B}}{X_{D} Y_{D}}=\frac{b}{d}=\frac{D_{1} X_{B}}{B_{1} X_{D}} +$$ + +where the second equation holds in view of $D_{1} X_{B} \| B_{1} X_{D}$. Therefore, the triangles $D_{1} X_{B} Y_{B}$ and $B_{1} X_{D} Y_{D}$ are similar. The triangles $D_{1} T_{B} B$ and $B_{1} T_{D} D$ are similar to them and hence to each other. Since $B D_{1} \| D B_{1}$, these triangles are also homothetical. This yields $B T_{B} \| D T_{D}$, as desired. + +Comment 4. The original problem proposal asked to prove that the relations $B D_{1} \| D B_{1}$ and $O \in O_{1} O_{2}$ are equivalent. After obtaining $B D_{1} \| D B_{1} \Rightarrow O \in O_{1} O_{2}$, the converse proof is either repeated backwards mutatis mutandis, or can be obtained by the usual procedure of varying some points in the construction. + +The Problem Selection Committee chose the current version, because it is less technical, yet keeps most of the ideas. + +G6. Determine all integers $n \geqslant 3$ satisfying the following property: every convex $n$-gon whose sides all have length 1 contains an equilateral triangle of side length 1. +(Every polygon is assumed to contain its boundary.) +Answer: All odd $n \geqslant 3$. +Solution. First we show that for every even $n \geqslant 4$ there exists a polygon violating the required statement. Consider a regular $k$-gon $A_{0} A_{1}, \ldots A_{k-1}$ with side length 1 . Let $B_{1}, B_{2}, \ldots, B_{n / 2-1}$ be the points symmetric to $A_{1}, A_{2}, \ldots, A_{n / 2-1}$ with respect to the line $A_{0} A_{n / 2}$. Then $P=$ $A_{0} A_{1} A_{2} \ldots A_{n / 2-1} A_{n / 2} B_{n / 2-1} B_{n / 2-2} \ldots B_{2} B_{1}$ is a convex $n$-gon whose sides all have length 1 . If $k$ is big enough, $P$ is contained in a strip of width $1 / 2$, which clearly does not contain any equilateral triangle of side length 1. +![](https://cdn.mathpix.com/cropped/2024_11_18_7107063274cfa469597ag-56.jpg?height=175&width=1363&top_left_y=826&top_left_x=346) + +Assume now that $n=2 k+1$. As the case $k=1$ is trivially true, we assume $k \geqslant 2$ henceforth. Consider a convex $(2 k+1)$-gon $P$ whose sides all have length 1 . Let $d$ be its longest diagonal. The endpoints of $d$ split the perimeter of $P$ into two polylines, one of which has length at least $k+1$. Hence we can label the vertices of $P$ so that $P=A_{0} A_{1} \ldots A_{2 k}$ and $d=A_{0} A_{\ell}$ with $\ell \geqslant k+1$. We will show that, in fact, the polygon $A_{0} A_{1} \ldots A_{\ell}$ contains an equilateral triangle of side length 1 . + +Suppose that $\angle A_{\ell} A_{0} A_{1} \geqslant 60^{\circ}$. Since $d$ is the longest diagonal, we have $A_{1} A_{\ell} \leqslant A_{0} A_{\ell}$, so $\angle A_{0} A_{1} A_{\ell} \geqslant \angle A_{\ell} A_{0} A_{1} \geqslant 60^{\circ}$. It follows that there exists a point $X$ inside the triangle $A_{0} A_{1} A_{\ell}$ such that the triangle $A_{0} A_{1} X$ is equilateral, and this triangle is contained in $P$. Similar arguments apply if $\angle A_{\ell-1} A_{\ell} A_{0} \geqslant 60^{\circ}$. +![](https://cdn.mathpix.com/cropped/2024_11_18_7107063274cfa469597ag-56.jpg?height=224&width=730&top_left_y=1524&top_left_x=663) + +From now on, assume $\angle A_{\ell} A_{0} A_{1}<60^{\circ}$ and $A_{\ell-1} A_{\ell} A_{0}<60^{\circ}$. +Consider an isosceles trapezoid $A_{0} Y Z A_{\ell}$ such that $A_{0} A_{\ell} \| Y Z, A_{0} Y=Z A_{\ell}=1$, and $\angle A_{\ell} A_{0} Y=\angle Z A_{\ell} A_{0}=60^{\circ}$. Suppose that $A_{0} A_{1} \ldots A_{\ell}$ is contained in $A_{0} Y Z A_{\ell}$. Note that the perimeter of $A_{0} A_{1} \ldots A_{\ell}$ equals $\ell+A_{0} A_{\ell}$ and the perimeter of $A_{0} Y Z A_{\ell}$ equals $2 A_{0} A_{\ell}+1$. +![](https://cdn.mathpix.com/cropped/2024_11_18_7107063274cfa469597ag-56.jpg?height=209&width=1320&top_left_y=1957&top_left_x=371) + +Recall a well-known fact stating that if a convex polygon $P_{1}$ is contained in a convex polygon $P_{2}$, then the perimeter of $P_{1}$ is at most the perimeter of $P_{2}$. Hence we obtain + +$$ +\ell+A_{0} A_{\ell} \leqslant 2 A_{0} A_{\ell}+1, \quad \text { i.e. } \quad \ell-1 \leqslant A_{0} A_{\ell} . +$$ + +On the other hand, the triangle inequality yields + +$$ +A_{0} A_{\ell}1 / 2$ and $P A_{\ell}>1 / 2$. Choose points $Q \in A_{0} P, R \in P A_{\ell}$, and $S \in P A_{m}$ such that $P Q=P R=1 / 2$ and $P S=\sqrt{3} / 2$. Then $Q R S$ is an equilateral triangle of side length 1 contained in $A_{0} A_{1} \ldots A_{\ell}$. +![](https://cdn.mathpix.com/cropped/2024_11_18_7107063274cfa469597ag-57.jpg?height=335&width=1332&top_left_y=404&top_left_x=365) + +Comment. In fact, for every odd $n$ a stronger statement holds, which is formulated in terms defined in the solution above: there exists an equilateral triangle $A_{i} A_{i+1} B$ contained in $A_{0} A_{1} \ldots A_{\ell}$ for some $0 \leqslant i<\ell$. We sketch an indirect proof below. + +As above, we get $\angle A_{\ell} A_{0} A_{1}<60^{\circ}$ and $A_{\ell-1} A_{\ell} A_{0}<60^{\circ}$. Choose an index $m \in[1, \ell-1]$ maximising the distance between $A_{m}$ and $A_{0} A_{\ell}$. Arguments from the above solution yield $1120^{\circ}$ and $\angle A_{m-1} A_{m} A_{\ell}>\angle A_{0} A_{m} A_{\ell} \geqslant 60^{\circ}$. We construct an equilateral triangle $A_{m-1} A_{m} B$ as in the figure below. If $B$ lies in $A_{0} A_{m-1} A_{m} A_{\ell}$, then we are done. Otherwise $B$ and $A_{m}$ lie on different sides of $A_{0} A_{\ell}$. As before, let $P$ be the projection of $A_{m}$ to $A_{0} A_{\ell}$. We will show that + +$$ +A_{0} A_{1}+A_{1} A_{2}+\ldots+A_{m-1} A_{m}A C$ so that $\angle B A D=\angle D A C$. A point $E$ is constructed on the segment $A C$ so that $\angle A D E=\angle D C B$. Similarly, a point $F$ is constructed on the segment $A B$ so that $\angle A D F=\angle D B C$. A point $X$ is chosen on the line $A C$ so that $C X=B X$. Let $O_{1}$ and $O_{2}$ be the circumcentres of the triangles $A D C$ and $D X E$. Prove that the lines $B C, E F$, and $O_{1} O_{2}$ are concurrent. + +Common remarks. Let $Q$ be the isogonal conjugate of $D$ with respect to the triangle $A B C$. Since $\angle B A D=\angle D A C$, the point $Q$ lies on $A D$. Then $\angle Q B A=\angle D B C=\angle F D A$, so the points $Q, D, F$, and $B$ are concyclic. Analogously, the points $Q, D, E$, and $C$ are concyclic. Thus $A F \cdot A B=A D \cdot A Q=A E \cdot A C$ and so the points $B, F, E$, and $C$ are also concyclic. +![](https://cdn.mathpix.com/cropped/2024_11_18_7107063274cfa469597ag-60.jpg?height=478&width=1204&top_left_y=732&top_left_x=426) + +Let $T$ be the intersection of $B C$ and $F E$. +Claim. TD $D^{2}=T B \cdot T C=T F \cdot T E$. +Proof. We will prove that the circles $(D E F)$ and $(B D C)$ are tangent to each other. Indeed, using the above arguments, we get + +$$ +\begin{aligned} +& \angle B D F=\angle A F D-\angle A B D=\left(180^{\circ}-\angle F A D-\angle F D A\right)-(\angle A B C-\angle D B C) \\ += & 180^{\circ}-\angle F A D-\angle A B C=180^{\circ}-\angle D A E-\angle F E A=\angle F E D+\angle A D E=\angle F E D+\angle D C B, +\end{aligned} +$$ + +which implies the desired tangency. +Since the points $B, C, E$, and $F$ are concyclic, the powers of the point $T$ with respect to the circles $(B D C)$ and $(E D F)$ are equal. So their radical axis, which coincides with the common tangent at $D$, passes through $T$, and hence $T D^{2}=T E \cdot T F=T B \cdot T C$. + +Solution 1. Let $T A$ intersect the circle $(A B C)$ again at $M$. Due to the circles ( $B C E F$ ) and $(A M C B)$, and using the above Claim, we get $T M \cdot T A=T F \cdot T E=T B \cdot T C=T D^{2}$; in particular, the points $A, M, E$, and $F$ are concyclic. + +Under the inversion with centre $T$ and radius $T D$, the point $M$ maps to $A$, and $B$ maps to $C$, which implies that the circle $(M B D)$ maps to the circle $(A D C)$. Their common point $D$ lies on the circle of the inversion, so the second intersection point $K$ also lies on that circle, which means $T K=T D$. It follows that the point $T$ and the centres of the circles ( $K D E$ ) and $(A D C)$ lie on the perpendicular bisector of $K D$. + +Since the center of $(A D C)$ is $O_{1}$, it suffices to show now that the points $D, K, E$, and $X$ are concyclic (the center of the corresponding circle will be $O_{2}$ ). + +The lines $B M, D K$, and $A C$ are the pairwise radical axes of the circles $(A B C M),(A C D K)$ and $(B M D K)$, so they are concurrent at some point $P$. Also, $M$ lies on the circle $(A E F)$, thus + +$$ +\begin{aligned} +\Varangle(E X, X B) & =\Varangle(C X, X B)=\Varangle(X C, B C)+\Varangle(B C, B X)=2 \Varangle(A C, C B) \\ +& =\Varangle(A C, C B)+\Varangle(E F, F A)=\Varangle(A M, B M)+\Varangle(E M, M A)=\Varangle(E M, B M), +\end{aligned} +$$ + +so the points $M, E, X$, and $B$ are concyclic. Therefore, $P E \cdot P X=P M \cdot P B=P K \cdot P D$, so the points $E, K, D$, and $X$ are concyclic, as desired. +![](https://cdn.mathpix.com/cropped/2024_11_18_7107063274cfa469597ag-61.jpg?height=935&width=1466&top_left_y=195&top_left_x=295) + +Comment 1. We present here a different solution which uses similar ideas. +Perform the inversion $\iota$ with centre $T$ and radius $T D$. It swaps $B$ with $C$ and $E$ with $F$; the point $D$ maps to itself. Let $X^{\prime}=\iota(X)$. Observe that the points $E, F, X$, and $X^{\prime}$ are concyclic, as well as the points $B, C, X$, and $X^{\prime}$. Then + +$$ +\begin{aligned} +& \Varangle\left(C X^{\prime}, X^{\prime} F\right)=\Varangle\left(C X^{\prime}, X^{\prime} X\right)+\Varangle\left(X^{\prime} X, X^{\prime} F\right)=\Varangle(C B, B X)+\Varangle(E X, E F) \\ +&=\Varangle(X C, C B)+\Varangle(E C, E F)=\Varangle(C A, C B)+\Varangle(B C, B F)=\Varangle(C A, A F), +\end{aligned} +$$ + +therefore the points $C, X^{\prime}, A$, and $F$ are concyclic. +Let $X^{\prime} F$ intersect $A C$ at $P$, and let $K$ be the second common point of $D P$ and the circle $(A C D)$. Then + +$$ +P K \cdot P D=P A \cdot P C=P X^{\prime} \cdot P F=P E \cdot P X +$$ + +hence, the points $K, X, D$, and $E$ lie on some circle $\omega_{1}$, while the points $K, X^{\prime}, D$, and $F$ lie on some circle $\omega_{2}$. (These circles are distinct since $\angle E X F+\angle E D F<\angle E A F+\angle D C B+\angle D B C<180^{\circ}$ ). The inversion $\iota$ swaps $\omega_{1}$ with $\omega_{2}$ and fixes their common point $D$, so it fixes their second common point $K$. Thus $T D=T K$ and the perpendicular bisector of $D K$ passes through $T$, as well as through the centres of the circles $(C D K A)$ and $(D E K X)$. +![](https://cdn.mathpix.com/cropped/2024_11_18_7107063274cfa469597ag-61.jpg?height=775&width=1154&top_left_y=1994&top_left_x=451) + +Solution 2. We use only the first part of the Common remarks, namely, the facts that the tuples $(C, D, Q, E)$ and $(B, C, E, F)$ are both concyclic. We also introduce the point $T=$ $B C \cap E F$. Let the circle $(C D E)$ meet $B C$ again at $E_{1}$. Since $\angle E_{1} C Q=\angle D C E$, the $\operatorname{arcs} D E$ and $Q E_{1}$ of the circle $(C D Q)$ are equal, so $D Q \| E E_{1}$. + +Since $B F E C$ is cyclic, the line $A D$ forms equal angles with $B C$ and $E F$, hence so does $E E_{1}$. Therefore, the triangle $E E_{1} T$ is isosceles, $T E=T E_{1}$, and $T$ lies on the common perpendicular bisector of $E E_{1}$ and $D Q$. + +Let $U$ and $V$ be the centres of circles $(A D E)$ and $(C D Q E)$, respectively. Then $U O_{1}$ is the perpendicular bisector of $A D$. Moreover, the points $U, V$, and $O_{2}$ belong to the perpendicular bisector of $D E$. Since $U O_{1} \| V T$, in order to show that $O_{1} O_{2}$ passes through $T$, it suffices to show that + +$$ +\frac{O_{2} U}{O_{2} V}=\frac{O_{1} U}{T V} +$$ + +Denote angles $A, B$, and $C$ of the triangle $A B C$ by $\alpha, \beta$, and $\gamma$, respectively. Projecting onto $A C$ we obtain + +$$ +\frac{O_{2} U}{O_{2} V}=\frac{(X E-A E) / 2}{(X E+E C) / 2}=\frac{A X}{C X}=\frac{A X}{B X}=\frac{\sin (\gamma-\beta)}{\sin \alpha} +$$ + +The projection of $O_{1} U$ onto $A C$ is $(A C-A E) / 2=C E / 2$; the angle between $O_{1} U$ and $A C$ is $90^{\circ}-\alpha / 2$, so + +$$ +\frac{O_{1} U}{E C}=\frac{1}{2 \sin (\alpha / 2)} +$$ + +Next, we claim that $E, V, C$, and $T$ are concyclic. Indeed, the point $V$ lies on the perpendicular bisector of $C E$, as well as on the internal angle bisector of $\angle C T F$. Therefore, $V$ coincides with the midpoint of the arc $C E$ of the circle (TCE). + +Now we have $\angle E V C=2 \angle E E_{1} C=180^{\circ}-(\gamma-\beta)$ and $\angle V E T=\angle V E_{1} T=90^{\circ}-\angle E_{1} E C=$ $90^{\circ}-\alpha / 2$. Therefore, + +$$ +\frac{E C}{T V}=\frac{\sin \angle E T C}{\sin \angle V E T}=\frac{\sin (\gamma-\beta)}{\cos (\alpha / 2)} . +$$ + +![](https://cdn.mathpix.com/cropped/2024_11_18_7107063274cfa469597ag-62.jpg?height=986&width=1512&top_left_y=1774&top_left_x=266) + +Recalling (2) and multiplying (3) and (4) we establish (1): + +$$ +\frac{O_{2} U}{O_{2} V}=\frac{\sin (\gamma-\beta)}{\sin \alpha}=\frac{1}{2 \sin (\alpha / 2)} \cdot \frac{\sin (\gamma-\beta)}{\cos (\alpha / 2)}=\frac{O_{1} U}{E C} \cdot \frac{E C}{T V}=\frac{O_{1} U}{T V} +$$ + +Solution 3. Notice that $\angle A Q E=\angle Q C B$ and $\angle A Q F=\angle Q B C$; so, if we replace the point $D$ with $Q$ in the problem set up, the points $E, F$, and $T$ remain the same. So, by the Claim, we have $T Q^{2}=T B \cdot T C=T D^{2}$. + +Thus, there exists a circle $\Gamma$ centred at $T$ and passing through $D$ and $Q$. We denote the second meeting point of the circles $\Gamma$ and $(A D C)$ by $K$. Let the line $A C$ meet the circle ( $D E K$ ) again at $Y$; we intend to prove that $Y=X$. As in Solution 1, this will yield that the point $T$, as well as the centres $O_{1}$ and $O_{2}$, all lie on the perpendicular bisector of $D K$. + +Let $L=A D \cap B C$. We perform an inversion centred at $C$; the images of the points will be denoted by primes, e.g., $A^{\prime}$ is the image of $A$. We obtain the following configuration, constructed in a triangle $A^{\prime} C L^{\prime}$. + +The points $D^{\prime}$ and $Q^{\prime}$ are chosen on the circumcircle $\Omega$ of $A^{\prime} L^{\prime} C$ such that $\Varangle\left(L^{\prime} C, D^{\prime} C\right)=$ $\Varangle\left(Q^{\prime} C, A^{\prime} C\right)$, which means that $A^{\prime} L^{\prime} \| D^{\prime} Q^{\prime}$. The lines $D^{\prime} Q^{\prime}$ and $A^{\prime} C$ meet at $E^{\prime}$. + +A circle $\Gamma^{\prime}$ centred on $C L^{\prime}$ passes through $D^{\prime}$ and $Q^{\prime}$. Notice here that $B^{\prime}$ lies on the segment $C L^{\prime}$, and that $\angle A^{\prime} B^{\prime} C=\angle B A C=2 \angle L A C=2 \angle A^{\prime} L^{\prime} C$, so that $B^{\prime} L^{\prime}=B^{\prime} A^{\prime}$, and $B^{\prime}$ lies on the perpendicular bisector of $A^{\prime} L^{\prime}$ (which coincides with that of $D^{\prime} Q^{\prime}$ ). All this means that $B^{\prime}$ is the centre of $\Gamma^{\prime}$. + +Finally, $K^{\prime}$ is the second meeting point of $A^{\prime} D^{\prime}$ and $\Gamma^{\prime}$, and $Y^{\prime}$ is the second meeting point of the circle $\left(D^{\prime} K^{\prime} E^{\prime}\right)$ and the line $A^{\prime} E^{\prime}$, We have $\Varangle\left(Y^{\prime} K^{\prime}, K^{\prime} A^{\prime}\right)=\Varangle\left(Y^{\prime} E^{\prime}, E^{\prime} D^{\prime}\right)=$ $\Varangle\left(Y^{\prime} A^{\prime}, A^{\prime} L^{\prime}\right)$, so $A^{\prime} L^{\prime}$ is tangent to the circumcircle $\omega$ of the triangle $Y^{\prime} A^{\prime} K^{\prime}$. + +Let $O$ and $O^{*}$ be the centres of $\Omega$ and $\omega$, respectively. Then $O^{*} A^{\prime} \perp A^{\prime} L^{\prime} \perp B^{\prime} O$. The projections of vectors $\overrightarrow{O^{*} A^{\prime}}$ and $\overrightarrow{B^{\prime} O}$ onto $K^{\prime} D^{\prime}$ are equal to $\overrightarrow{K^{\prime} A^{\prime}} / 2=\overrightarrow{K^{\prime} D^{\prime}} / 2-\overrightarrow{A^{\prime} D^{\prime}} / 2$. So $\overrightarrow{O^{*} A^{\prime}}=\overrightarrow{B^{\prime} O}$, or equivalently $\overrightarrow{A^{\prime} O}=\overrightarrow{O^{*} B^{\prime}}$. Projecting this equality onto $A^{\prime} C$, we see that the projection of $\overrightarrow{O^{*} \overrightarrow{B^{\prime}}}$ equals $\overrightarrow{A^{\prime} C} / 2$. Since $O^{*}$ is projected to the midpoint of $A^{\prime} Y^{\prime}$, this yields that $B^{\prime}$ is projected to the midpoint of $C Y^{\prime}$, i.e., $B^{\prime} Y^{\prime}=B^{\prime} C$ and $\angle B^{\prime} Y^{\prime} C=\angle B^{\prime} C Y^{\prime}$. In the original figure, this rewrites as $\angle C B Y=\angle B C Y$, so $Y$ lies on the perpendicular bisector of $B C$, as desired. +![](https://cdn.mathpix.com/cropped/2024_11_18_7107063274cfa469597ag-63.jpg?height=843&width=1306&top_left_y=1783&top_left_x=381) + +Comment 2. The point $K$ appears to be the same in Solutions 1 and 3 (and Comment 1 as well). One can also show that $K$ lies on the circle passing through $A, X$, and the midpoint of the arc $B A C$. + +Comment 3. There are different proofs of the facts from the Common remarks, namely, the cyclicity of $B, C, E$, and $F$, and the Claim. We present one such alternative proof here. + +We perform the composition $\phi$ of a homothety with centre $A$ and the reflection in $A D$, which maps $E$ to $B$. Let $U=\phi(D)$. Then $\Varangle(B C, C D)=\Varangle(A D, D E)=\Varangle(B U, U D)$, so the points $B, U, C$, and $D$ are concyclic. Therefore, $\Varangle(C U, U D)=\Varangle(C B, B D)=\Varangle(A D, D F)$, so $\phi(F)=C$. Then the coefficient of the homothety is $A C / A F=A B / A E$, and thus points $C, E, F$, and $B$ are concyclic. + +Denote the centres of the circles $(E D F)$ and $(B U C D)$ by $O_{3}$ and $O_{4}$, respectively. Then $\phi\left(O_{3}\right)=$ $O_{4}$, hence $\Varangle\left(O_{3} D, D A\right)=-\Varangle\left(O_{4} U, U A\right)=\Varangle\left(O_{4} D, D A\right)$, whence the circle $(B D C)$ is tangent to the circle ( $E D F$ ). + +Now, the radical axes of circles $(D E F),(B D C)$ and $(B C E F)$ intersect at $T$, and the claim follows. +![](https://cdn.mathpix.com/cropped/2024_11_18_7107063274cfa469597ag-64.jpg?height=869&width=941&top_left_y=682&top_left_x=563) + +This suffices for Solution 1 to work. However, Solutions 2 and 3 need properties of point $Q$, established in Common remarks before Solution 1. + +Comment 4. In the original problem proposal, the point $X$ was hidden. Instead, a circle $\gamma$ was constructed such that $D$ and $E$ lie on $\gamma$, and its center is collinear with $O_{1}$ and $T$. The problem requested to prove that, in a fixed triangle $A B C$, independently from the choice of $D$ on the bisector of $\angle B A C$, all circles $\gamma$ pass through a fixed point. + +G8. Let $\omega$ be the circumcircle of a triangle $A B C$, and let $\Omega_{A}$ be its excircle which is tangent to the segment $B C$. Let $X$ and $Y$ be the intersection points of $\omega$ and $\Omega_{A}$. Let $P$ and $Q$ be the projections of $A$ onto the tangent lines to $\Omega_{A}$ at $X$ and $Y$, respectively. The tangent line at $P$ to the circumcircle of the triangle $A P X$ intersects the tangent line at $Q$ to the circumcircle of the triangle $A Q Y$ at a point $R$. Prove that $A R \perp B C$. + +Solution 1. Let $D$ be the point of tangency of $B C$ and $\Omega_{A}$. Let $D^{\prime}$ be the point such that $D D^{\prime}$ is a diameter of $\Omega_{A}$. Let $R^{\prime}$ be (the unique) point such that $A R^{\prime} \perp B C$ and $R^{\prime} D^{\prime} \| B C$. We shall prove that $R^{\prime}$ coincides with $R$. + +Let $P X$ intersect $A B$ and $D^{\prime} R^{\prime}$ at $S$ and $T$, respectively. Let $U$ be the ideal common point of the parallel lines $B C$ and $D^{\prime} R^{\prime}$. Note that the (degenerate) hexagon $A S X T U C$ is circumscribed around $\Omega_{A}$, hence by the Brianchon theorem $A T, S U$, and $X C$ concur at a point which we denote by $V$. Then $V S \| B C$. It follows that $\Varangle(S V, V X)=\Varangle(B C, C X)=$ $\Varangle(B A, A X)$, hence $A X S V$ is cyclic. Therefore, $\Varangle(P X, X A)=\Varangle(S V, V A)=\Varangle\left(R^{\prime} T, T A\right)$. Since $\angle A P T=\angle A R^{\prime} T=90^{\circ}$, the quadrilateral $A P R^{\prime} T$ is cyclic. Hence, + +$$ +\Varangle(X A, A P)=90^{\circ}-\Varangle(P X, X A)=90^{\circ}-\Varangle\left(R^{\prime} T, T A\right)=\Varangle\left(T A, A R^{\prime}\right)=\Varangle\left(T P, P R^{\prime}\right) . +$$ + +It follows that $P R^{\prime}$ is tangent to the circle $(A P X)$. +Analogous argument shows that $Q R^{\prime}$ is tangent to the circle $(A Q Y)$. Therefore, $R=R^{\prime}$ and $A R \perp B C$. +![](https://cdn.mathpix.com/cropped/2024_11_18_7107063274cfa469597ag-65.jpg?height=995&width=1380&top_left_y=1333&top_left_x=349) + +Comment 1. After showing $\Varangle(P X, X A)=\Varangle\left(R^{\prime} T, T A\right)$ one can finish the solution as follows. There exists a spiral similarity mapping the triangle $A T R^{\prime}$ to the triangle $A X P$. So the triangles $A T X$ and $A R^{\prime} P$ are similar and equioriented. Thus, $\Varangle(T X, X A)=\Varangle\left(R^{\prime} P, P A\right)$, which implies that $P R^{\prime}$ is tangent to the circle ( $A P X$ ). + +Solution 2. Let $J$ and $r$ be the center and the radius of $\Omega_{A}$. Denote the diameter of $\omega$ by $d$ and its center by $O$. By Euler's formula, $O J^{2}=(d / 2)^{2}+d r$, so the power of $J$ with respect to $\omega$ equals $d r$. + +Let $J X$ intersect $\omega$ again at $L$. Then $J L=d$. Let $L K$ be a diameter of $\omega$ and let $M$ be the midpoint of $J K$. Since $J L=L K$, we have $\angle L M K=90^{\circ}$, so $M$ lies on $\omega$. Let $R^{\prime}$ be the point such that $R^{\prime} P$ is tangent to the circle $(A P X)$ and $A R^{\prime} \perp B C$. Note that the line $A R^{\prime}$ is symmetric to the line $A O$ with respect to $A J$. +![](https://cdn.mathpix.com/cropped/2024_11_18_7107063274cfa469597ag-66.jpg?height=690&width=1438&top_left_y=406&top_left_x=316) + +Lemma. Let $M$ be the midpoint of the side $J K$ in a triangle $A J K$. Let $X$ be a point on the circle $(A M K)$ such that $\angle J X K=90^{\circ}$. Then there exists a point $T$ on the line $K X$ such that the triangles $A K J$ and $A J T$ are similar and equioriented. +Proof. Note that $M X=M K$. We construct a parallelogram $A J N K$. Let $T$ be a point on $K X$ such that $\Varangle(N J, J A)=\Varangle(K J, J T)$. Then + +$$ +\Varangle(J N, N A)=\Varangle(K A, A M)=\Varangle(K X, X M)=\Varangle(M K, K X)=\Varangle(J K, K T) . +$$ + +So there exists a spiral similarity with center $J$ mapping the triangle $A J N$ to the triangle $T J K$. Therefore, the triangles $N J K$ and $A J T$ are similar and equioriented. It follows that the triangles $A K J$ and $A J T$ are similar and equioriented. +![](https://cdn.mathpix.com/cropped/2024_11_18_7107063274cfa469597ag-66.jpg?height=1155&width=1087&top_left_y=1607&top_left_x=493) + +Back to the problem, we construct a point $T$ as in the lemma. We perform the composition $\phi$ of inversion with centre $A$ and radius $A J$ and reflection in $A J$. It is known that every triangle $A E F$ is similar and equioriented to $A \phi(F) \phi(E)$. + +So $\phi(K)=T$ and $\phi(T)=K$. Let $P^{*}=\phi(P)$ and $R^{*}=\phi\left(R^{\prime}\right)$. Observe that $\phi(T K)$ is a circle with diameter $A P^{*}$. Let $A A^{\prime}$ be a diameter of $\omega$. Then $P^{*} K \perp A K \perp A^{\prime} K$, so $A^{\prime}$ lies on $P^{*} K$. The triangles $A R^{\prime} P$ and $A P^{*} R^{*}$ are similar and equioriented, hence +$\Varangle\left(A A^{\prime}, A^{\prime} P^{*}\right)=\Varangle\left(A A^{\prime}, A^{\prime} K\right)=\Varangle(A X, X P)=\Varangle(A X, X P)=\Varangle\left(A P, P R^{\prime}\right)=\Varangle\left(A R^{*}, R^{*} P^{*}\right)$, +so $A, A^{\prime}, R^{*}$, and $P^{*}$ are concyclic. Since $A^{\prime}$ and $R^{*}$ lie on $A O$, we obtain $R^{*}=A^{\prime}$. So $R^{\prime}=\phi\left(A^{\prime}\right)$, and $\phi\left(A^{\prime}\right) P$ is tangent to the circle $(A P X)$. + +An identical argument shows that $\phi\left(A^{\prime}\right) Q$ is tangent to the circle $(A Q Y)$. Therefore, $R=$ $\phi\left(A^{\prime}\right)$ and $A R \perp B C$. + +Comment 2. One of the main ideas of Solution 2 is to get rid of the excircle, along with points $B$ and $C$. After doing so we obtain the following fact, which is, essentially, proved in Solution 2. + +Let $\omega$ be the circumcircle of a triangle $A K_{1} K_{2}$. Let $J$ be a point such that the midpoints of $J K_{1}$ and $J K_{2}$ lie on $\omega$. Points $X$ and $Y$ are chosen on $\omega$ so that $\angle J X K_{1}=\angle J Y K_{2}=90^{\circ}$. Let $P$ and $Q$ be the projections of $A$ onto $X K_{1}$ and $Y K_{2}$, respectively. The tangent line at $P$ to the circumcircle of the triangle $A P X$ intersects the tangent line at $Q$ to the circumcircle of the triangle $A Q Y$ at a point $R$. Then the reflection of the line $A R$ in $A J$ passes through the centre $O$ of $\omega$. + +## Number Theory + +N1. Determine all integers $n \geqslant 1$ for which there exists a pair of positive integers $(a, b)$ such that no cube of a prime divides $a^{2}+b+3$ and + +$$ +\frac{a b+3 b+8}{a^{2}+b+3}=n +$$ + +Answer: The only integer with that property is $n=2$. +Solution. As $b \equiv-a^{2}-3\left(\bmod a^{2}+b+3\right)$, the numerator of the given fraction satisfies + +$$ +a b+3 b+8 \equiv a\left(-a^{2}-3\right)+3\left(-a^{2}-3\right)+8 \equiv-(a+1)^{3} \quad\left(\bmod a^{2}+b+3\right) +$$ + +As $a^{2}+b+3$ is not divisible by $p^{3}$ for any prime $p$, if $a^{2}+b+3$ divides $(a+1)^{3}$ then it does also divide $(a+1)^{2}$. Since + +$$ +0<(a+1)^{2}<2\left(a^{2}+b+3\right) +$$ + +we conclude $(a+1)^{2}=a^{2}+b+3$. This yields $b=2(a-1)$ and $n=2$. The choice $(a, b)=(2,2)$ with $a^{2}+b+3=9$ shows that $n=2$ indeed is a solution. + +N2. Let $n \geqslant 100$ be an integer. The numbers $n, n+1, \ldots, 2 n$ are written on $n+1$ cards, one number per card. The cards are shuffled and divided into two piles. Prove that one of the piles contains two cards such that the sum of their numbers is a perfect square. + +Solution. To solve the problem it suffices to find three squares and three cards with numbers $a, b, c$ on them such that pairwise sums $a+b, b+c, a+c$ are equal to the chosen squares. By choosing the three consecutive squares $(2 k-1)^{2},(2 k)^{2},(2 k+1)^{2}$ we arrive at the triple + +$$ +(a, b, c)=\left(2 k^{2}-4 k, \quad 2 k^{2}+1, \quad 2 k^{2}+4 k\right) +$$ + +We need a value for $k$ such that + +$$ +n \leqslant 2 k^{2}-4 k, \quad \text { and } \quad 2 k^{2}+4 k \leqslant 2 n +$$ + +A concrete $k$ is suitable for all $n$ with + +$$ +n \in\left[k^{2}+2 k, 2 k^{2}-4 k+1\right]=: I_{k} +$$ + +For $k \geqslant 9$ the intervals $I_{k}$ and $I_{k+1}$ overlap because + +$$ +(k+1)^{2}+2(k+1) \leqslant 2 k^{2}-4 k+1 +$$ + +Hence $I_{9} \cup I_{10} \cup \ldots=[99, \infty)$, which proves the statement for $n \geqslant 99$. +Comment 1. There exist approaches which only work for sufficiently large $n$. +One possible approach is to consider three cards with numbers $70 k^{2}, 99 k^{2}, 126 k^{2}$ on them. Then their pairwise sums are perfect squares and so it suffices to find $k$ such that $70 k^{2} \geqslant n$ and $126 k^{2} \leqslant 2 n$ which exists for sufficiently large $n$. + +Another approach is to prove, arguing by contradiction, that $a$ and $a-2$ are in the same pile provided that $n$ is large enough and $a$ is sufficiently close to $n$. For that purpose, note that every pair of neighbouring numbers in the sequence $a, x^{2}-a, a+(2 x+1), x^{2}+2 x+3-a, a-2$ adds up to a perfect square for any $x$; so by choosing $x=\lfloor\sqrt{2 a}\rfloor+1$ and assuming that $n$ is large enough we conclude that $a$ and $a-2$ are in the same pile for any $a \in[n+2,3 n / 2]$. This gives a contradiction since it is easy to find two numbers from $[n+2,3 n / 2]$ of the same parity which sum to a square. + +It then remains to separately cover the cases of small $n$ which appears to be quite technical. +Comment 2. An alternative formulation for this problem could ask for a proof of the statement for all $n>10^{6}$. An advantage of this formulation is that some solutions, e.g. those mentioned in Comment 1 need not contain a technical part which deals with the cases of small $n$. However, the original formulation seems to be better because the bound it gives for $n$ is almost sharp, see the next comment for details. + +Comment 3. The statement of the problem is false for $n=98$. As a counterexample, the first pile may contain the even numbers from 98 to 126 , the odd numbers from 129 to 161 , and the even numbers from 162 to 196. + +N3. Find all positive integers $n$ with the following property: the $k$ positive divisors of $n$ have a permutation $\left(d_{1}, d_{2}, \ldots, d_{k}\right)$ such that for every $i=1,2, \ldots, k$, the number $d_{1}+\cdots+d_{i}$ is a perfect square. + +Answer: $n=1$ and $n=3$. +Solution. For $i=1,2, \ldots, k$ let $d_{1}+\ldots+d_{i}=s_{i}^{2}$, and define $s_{0}=0$ as well. Obviously $0=s_{0}2 i>d_{i}>d_{i-1}>\ldots>d_{1}$, so $j \leqslant i$ is not possible. The only possibility is $j=i+1$. + +Hence, + +$$ +\begin{gathered} +s_{i+1}+i=d_{i+1}=s_{i+1}^{2}-s_{i}^{2}=s_{i+1}^{2}-i^{2} \\ +s_{i+1}^{2}-s_{i+1}=i(i+1) +\end{gathered} +$$ + +By solving this equation we get $s_{i+1}=i+1$ and $d_{i+1}=2 i+1$, that finishes the proof. +Now we know that the positive divisors of the number $n$ are $1,3,5, \ldots, n-2, n$. The greatest divisor is $d_{k}=2 k-1=n$ itself, so $n$ must be odd. The second greatest divisor is $d_{k-1}=n-2$; then $n-2$ divides $n=(n-2)+2$, so $n-2$ divides 2 . Therefore, $n$ must be 1 or 3 . + +The numbers $n=1$ and $n=3$ obviously satisfy the requirements: for $n=1$ we have $k=1$ and $d_{1}=1^{2}$; for $n=3$ we have $k=2, d_{1}=1^{2}$ and $d_{1}+d_{2}=1+3=2^{2}$. + +This page is intentionally left blank + +N4. Alice is given a rational number $r>1$ and a line with two points $B \neq R$, where point $R$ contains a red bead and point $B$ contains a blue bead. Alice plays a solitaire game by performing a sequence of moves. In every move, she chooses a (not necessarily positive) integer $k$, and a bead to move. If that bead is placed at point $X$, and the other bead is placed at $Y$, then Alice moves the chosen bead to point $X^{\prime}$ with $\overrightarrow{Y X^{\prime}}=r^{k} \overrightarrow{Y X}$. + +Alice's goal is to move the red bead to the point $B$. Find all rational numbers $r>1$ such that Alice can reach her goal in at most 2021 moves. + +Answer: All $r=(b+1) / b$ with $b=1, \ldots, 1010$. +Solution. Denote the red and blue beads by $\mathcal{R}$ and $\mathcal{B}$, respectively. Introduce coordinates on the line and identify the points with their coordinates so that $R=0$ and $B=1$. Then, during the game, the coordinate of $\mathcal{R}$ is always smaller than the coordinate of $\mathcal{B}$. Moreover, the distance between the beads always has the form $r^{\ell}$ with $\ell \in \mathbb{Z}$, since it only multiplies by numbers of this form. Denote the value of the distance after the $m^{\text {th }}$ move by $d_{m}=r^{\alpha_{m}}$, $m=0,1,2, \ldots$ (after the $0^{\text {th }}$ move we have just the initial position, so $\alpha_{0}=0$ ). + +If some bead is moved in two consecutive moves, then Alice could instead perform a single move (and change the distance from $d_{i}$ directly to $d_{i+2}$ ) which has the same effect as these two moves. So, if Alice can achieve her goal, then she may as well achieve it in fewer (or the same) number of moves by alternating the moves of $\mathcal{B}$ and $\mathcal{R}$. In the sequel, we assume that Alice alternates the moves, and that $\mathcal{R}$ is shifted altogether $t$ times. + +If $\mathcal{R}$ is shifted in the $m^{\text {th }}$ move, then its coordinate increases by $d_{m}-d_{m+1}$. Therefore, the total increment of $\mathcal{R}$ 's coordinate, which should be 1 , equals + +$$ +\begin{aligned} +\text { either } \quad\left(d_{0}-d_{1}\right)+\left(d_{2}-d_{3}\right)+\cdots+\left(d_{2 t-2}-d_{2 t-1}\right) & =1+\sum_{i=1}^{t-1} r^{\alpha_{2 i}}-\sum_{i=1}^{t} r^{\alpha_{2 i-1}}, \\ +\text { or } \quad\left(d_{1}-d_{2}\right)+\left(d_{3}-d_{4}\right)+\cdots+\left(d_{2 t-1}-d_{2 t}\right) & =\sum_{i=1}^{t} r^{\alpha_{2 i-1}}-\sum_{i=1}^{t} r^{\alpha_{2 i}}, +\end{aligned} +$$ + +depending on whether $\mathcal{R}$ or $\mathcal{B}$ is shifted in the first move. Moreover, in the former case we should have $t \leqslant 1011$, while in the latter one we need $t \leqslant 1010$. So both cases reduce to an equation + +$$ +\sum_{i=1}^{n} r^{\beta_{i}}=\sum_{i=1}^{n-1} r^{\gamma_{i}}, \quad \beta_{i}, \gamma_{i} \in \mathbb{Z} +$$ + +for some $n \leqslant 1011$. Thus, if Alice can reach her goal, then this equation has a solution for $n=1011$ (we can add equal terms to both sums in order to increase $n$ ). + +Conversely, if (1) has a solution for $n=1011$, then Alice can compose a corresponding sequence of distances $d_{0}, d_{1}, d_{2}, \ldots, d_{2021}$ and then realise it by a sequence of moves. So the problem reduces to the solvability of (1) for $n=1011$. + +Assume that, for some rational $r$, there is a solution of (1). Write $r$ in lowest terms as $r=a / b$. Substitute this into (1), multiply by the common denominator, and collect all terms on the left hand side to get + +$$ +\sum_{i=1}^{2 n-1}(-1)^{i} a^{\mu_{i}} b^{N-\mu_{i}}=0, \quad \mu_{i} \in\{0,1, \ldots, N\} +$$ + +for some $N \geqslant 0$. We assume that there exist indices $j_{-}$and $j_{+}$such that $\mu_{j_{-}}=0$ and $\mu_{j_{+}}=N$. + +Reducing (2) modulo $a-b$ (so that $a \equiv b$ ), we get + +$$ +0=\sum_{i=1}^{2 n-1}(-1)^{i} a^{\mu_{i}} b^{N-\mu_{i}} \equiv \sum_{i=1}^{2 n-1}(-1)^{i} b^{\mu_{i}} b^{N-\mu_{i}}=-b^{N} \quad \bmod (a-b) +$$ + +Since $\operatorname{gcd}(a-b, b)=1$, this is possible only if $a-b=1$. +Reducing (2) modulo $a+b$ (so that $a \equiv-b$ ), we get + +$$ +0=\sum_{i=1}^{2 n-1}(-1)^{i} a^{\mu_{i}} b^{N-\mu_{i}} \equiv \sum_{i=1}^{2 n-1}(-1)^{i}(-1)^{\mu_{i}} b^{\mu_{i}} b^{N-\mu_{i}}=S b^{N} \quad \bmod (a+b) +$$ + +for some odd (thus nonzero) $S$ with $|S| \leqslant 2 n-1$. Since $\operatorname{gcd}(a+b, b)=1$, this is possible only if $a+b \mid S$. So $a+b \leqslant 2 n-1$, and hence $b=a-1 \leqslant n-1=1010$. + +Thus we have shown that any sought $r$ has the form indicated in the answer. It remains to show that for any $b=1,2, \ldots, 1010$ and $a=b+1$, Alice can reach the goal. For this purpose, in (1) we put $n=a, \beta_{1}=\beta_{2}=\cdots=\beta_{a}=0$, and $\gamma_{1}=\gamma_{2}=\cdots=\gamma_{b}=1$. + +Comment 1. Instead of reducing modulo $a+b$, one can reduce modulo $a$ and modulo $b$. The first reduction shows that the number of terms in (2) with $\mu_{i}=0$ is divisible by $a$, while the second shows that the number of terms with $\mu_{i}=N$ is divisible by $b$. + +Notice that, in fact, $N>0$, as otherwise (2) contains an alternating sum of an odd number of equal terms, which is nonzero. Therefore, all terms listed above have different indices, and there are at least $a+b$ of them. + +Comment 2. Another way to investigate the solutions of equation (1) is to consider the Laurent polynomial + +$$ +L(x)=\sum_{i=1}^{n} x^{\beta_{i}}-\sum_{i=1}^{n-1} x^{\gamma_{i}} . +$$ + +We can pick a sufficiently large integer $d$ so that $P(x)=x^{d} L(x)$ is a polynomial in $\mathbb{Z}[x]$. Then + +$$ +P(1)=1, +$$ + +and + +$$ +1 \leqslant|P(-1)| \leqslant 2021 +$$ + +If $r=p / q$ with integers $p>q \geqslant 1$ is a rational number with the properties listed in the problem statement, then $P(p / q)=L(p / q)=0$. As $P(x)$ has integer coefficients, + +$$ +(p-q x) \mid P(x) +$$ + +Plugging $x=1$ into (5) gives $(p-q) \mid P(1)=1$, which implies $p=q+1$. Moreover, plugging $x=-1$ into (5) gives $(p+q) \mid P(-1)$, which, along with (4), implies $p+q \leqslant 2021$ and $q \leqslant 1010$. Hence $x=(q+1) / q$ for some integer $q$ with $1 \leqslant q \leqslant 1010$. + +N5. Prove that there are only finitely many quadruples $(a, b, c, n)$ of positive integers such that + +$$ +n!=a^{n-1}+b^{n-1}+c^{n-1} . +$$ + +Solution. For fixed $n$ there are clearly finitely many solutions; we will show that there is no solution with $n>100$. So, assume $n>100$. By the AM-GM inequality, + +$$ +\begin{aligned} +n! & =2 n(n-1)(n-2)(n-3) \cdot(3 \cdot 4 \cdots(n-4)) \\ +& \leqslant 2(n-1)^{4}\left(\frac{3+\cdots+(n-4)}{n-6}\right)^{n-6}=2(n-1)^{4}\left(\frac{n-1}{2}\right)^{n-6}<\left(\frac{n-1}{2}\right)^{n-1} +\end{aligned} +$$ + +thus $a, b, c<(n-1) / 2$. +For every prime $p$ and integer $m \neq 0$, let $\nu_{p}(m)$ denote the $p$-adic valuation of $m$; that is, the greatest non-negative integer $k$ for which $p^{k}$ divides $m$. Legendre's formula states that + +$$ +\nu_{p}(n!)=\sum_{s=1}^{\infty}\left\lfloor\frac{n}{p^{s}}\right\rfloor +$$ + +and a well-know corollary of this formula is that + +$$ +\nu_{p}(n!)<\sum_{s=1}^{\infty} \frac{n}{p^{s}}=\frac{n}{p-1} +$$ + +If $n$ is odd then $a^{n-1}, b^{n-1}, c^{n-1}$ are squares, and by considering them modulo 4 we conclude that $a, b$ and $c$ must be even. Hence, $2^{n-1} \mid n$ ! but that is impossible for odd $n$ because $\nu_{2}(n!)=\nu_{2}((n-1)!)a+b$. On the other hand, $p \mid c$ implies that $p100$. + +Comment 1. The original version of the problem asked to find all solutions to the equation. The solution to that version is not much different but is more technical. + +Comment 2. To find all solutions we can replace the bound $a, b, c<(n-1) / 2$ for all $n$ with a weaker bound $a, b, c \leqslant n / 2$ only for even $n$, which is a trivial application of AM-GM to the tuple $(2,3, \ldots, n)$. Then we may use the same argument for odd $n$ (it works for $n \geqslant 5$ and does not require any bound on $a, b, c)$, and for even $n$ the same solution works for $n \geqslant 6$ unless we have $a+b=n-1$ and $2 \nu_{p}(n-1)=\nu_{p}(n!)$. This is only possible for $p=3$ and $n=10$ in which case we can consider the original equation modulo 7 to deduce that $7 \mid a b c$ which contradicts the fact that $7^{9}>10$ !. Looking at $n \leqslant 4$ we find four solutions, namely, + +$$ +(a, b, c, n)=(1,1,2,3),(1,2,1,3),(2,1,1,3),(2,2,2,4) +$$ + +Comment 3. For sufficiently large $n$, the inequality $a, b, c<(n-1) / 2$ also follows from Stirling's formula. + +N6. Determine all integers $n \geqslant 2$ with the following property: every $n$ pairwise distinct integers whose sum is not divisible by $n$ can be arranged in some order $a_{1}, a_{2}, \ldots, a_{n}$ so that $n$ divides $1 \cdot a_{1}+2 \cdot a_{2}+\cdots+n \cdot a_{n}$. + +Answer: All odd integers and all powers of 2. + +Solution. If $n=2^{k} a$, where $a \geqslant 3$ is odd and $k$ is a positive integer, we can consider a set containing the number $2^{k}+1$ and $n-1$ numbers congruent to 1 modulo $n$. The sum of these numbers is congruent to $2^{k}$ modulo $n$ and therefore is not divisible by $n$; for any permutation $\left(a_{1}, a_{2}, \ldots, a_{n}\right)$ of these numbers + +$$ +1 \cdot a_{1}+2 \cdot a_{2}+\cdots+n \cdot a_{n} \equiv 1+\cdots+n \equiv 2^{k-1} a\left(2^{k} a+1\right) \not \equiv 0 \quad\left(\bmod 2^{k}\right) +$$ + +and $a$ fortiori $1 \cdot a_{1}+2 \cdot a_{2}+\cdots+n \cdot a_{n}$ is not divisible by $n$. +From now on, we suppose that $n$ is either odd or a power of 2 . Let $S$ be the given set of integers, and $s$ be the sum of elements of $S$. +Lemma 1. If there is a permutation $\left(a_{i}\right)$ of $S$ such that $(n, s)$ divides $\sum_{i=1}^{n} i a_{i}$, then there is a permutation $\left(b_{i}\right)$ of $S$ such that $n$ divides $\sum_{i=1}^{n} i b_{i}$. +Proof. Let $r=\sum_{i=1}^{n} i a_{i}$. Consider the permutation $\left(b_{i}\right)$ defined by $b_{i}=a_{i+x}$, where $a_{j+n}=a_{j}$. For this permutation, we have + +$$ +\sum_{i=1}^{n} i b_{i}=\sum_{i=1}^{n} i a_{i+x} \equiv \sum_{i=1}^{n}(i-x) a_{i} \equiv r-s x \quad(\bmod n) +$$ + +Since $(n, s)$ divides $r$, the congruence $r-s x \equiv 0(\bmod n)$ admits a solution. +Lemma 2. Every set $T$ of $k m$ integers, $m>1$, can be partitioned into $m$ sets of $k$ integers so that in every set either the sum of elements is not divisible by $k$ or all the elements leave the same remainder upon division by $k$. +Proof. The base case, $m=2$. If $T$ contains $k$ elements leaving the same remainder upon division by $k$, we form one subset $A$ of these elements; the remaining elements form a subset $B$. If $k$ does not divide the sum of all elements of $B$, we are done. Otherwise it is enough to exchange any element of $A$ with any element of $B$ not congruent to it modulo $k$, thus making sums of both $A$ and $B$ not divisible by $k$. This cannot be done only when all the elements of $T$ are congruent modulo $k$; in this case any partition will do. + +If no $k$ elements of $T$ have the same residue modulo $k$, there are three elements $a, b, c \in T$ leaving pairwise distinct remainders upon division by $k$. Let $t$ be the sum of elements of $T$. It suffices to find $A \subset T$ such that $|A|=k$ and $\sum_{x \in A} x \not \equiv 0, t(\bmod k)$ : then neither the sum of elements of $A$ nor the sum of elements of $B=T \backslash A$ is divisible by $k$. Consider $U^{\prime} \subset T \backslash\{a, b, c\}$ with $\left|U^{\prime}\right|=k-1$. The sums of elements of three sets $U^{\prime} \cup\{a\}, U^{\prime} \cup\{b\}, U^{\prime} \cup\{c\}$ leave three different remainders upon division by $k$, and at least one of them is not congruent either to 0 or to $t$. + +Now let $m>2$. If $T$ contains $k$ elements leaving the same remainder upon division by $k$, we form one subset $A$ of these elements and apply the inductive hypothesis to the remaining $k(m-1)$ elements. Otherwise, we choose any $U \subset T,|U|=k-1$. Since all the remaining elements cannot be congruent modulo $k$, there is $a \in T \backslash U$ such that $a \not \equiv-\sum_{x \in U} x(\bmod k)$. Now we can take $A=U \cup\{a\}$ and apply the inductive hypothesis to $T \backslash A$. + +Now we are ready to prove the statement of the problem for all odd $n$ and $n=2^{k}$. The proof is by induction. + +If $n$ is prime, the statement follows immediately from Lemma 1 , since in this case $(n, s)=1$. Turning to the general case, we can find prime $p$ and an integer $t$ such that $p^{t} \mid n$ and $p^{t} \nmid s$. By Lemma 2, we can partition $S$ into $p$ sets of $\frac{n}{p}=k$ elements so that in every set either the sum of numbers is not divisible by $k$ or all numbers have the same residue modulo $k$. + +For sets in the first category, by the inductive hypothesis there is a permutation $\left(a_{i}\right)$ such that $k \mid \sum_{i=1}^{k} i a_{i}$. + +If $n$ (and therefore $k$ ) is odd, then for each permutation $\left(b_{i}\right)$ of a set in the second category we have + +$$ +\sum_{i=1}^{k} i b_{i} \equiv b_{1} \frac{k(k+1)}{2} \equiv 0 \quad(\bmod k) +$$ + +By combining such permutation for all sets of the partition, we get a permutation ( $c_{i}$ ) of $S$ such that $k \mid \sum_{i=1}^{n} i c_{i}$. Since this sum is divisible by $k$, and $k$ is divisible by $(n, s)$, we are done by Lemma 1 . + +If $n=2^{s}$, we have $p=2$ and $k=2^{s-1}$. Then for each of the subsets there is a permutation $\left(a_{1}, \ldots, a_{k}\right)$ such that $\sum_{i=1}^{k} i a_{i}$ is divisible by $2^{s-2}=\frac{k}{2}$ : if the subset belongs to the first category, the expression is divisible even by $k$, and if it belongs to the second one, + +$$ +\sum_{i=1}^{k} i a_{i} \equiv a_{1} \frac{k(k+1)}{2} \equiv 0\left(\bmod \frac{k}{2}\right) +$$ + +Now the numbers of each permutation should be multiplied by all the odd or all the even numbers not exceeding $n$ in increasing order so that the resulting sums are divisible by $k$ : + +$$ +\sum_{i=1}^{k}(2 i-1) a_{i} \equiv \sum_{i=1}^{k} 2 i a_{i} \equiv 2 \sum_{i=1}^{k} i a_{i} \equiv 0 \quad(\bmod k) +$$ + +Combining these two sums, we again get a permutation $\left(c_{i}\right)$ of $S$ such that $k \mid \sum_{i=1}^{n} i c_{i}$, and finish the case by applying Lemma 1. + +Comment. We cannot dispense with the condition that $n$ does not divide the sum of all elements. Indeed, for each $n>1$ and the set consisting of $1,-1$, and $n-2$ elements divisible by $n$ the required permutation does not exist. + +N7. Let $a_{1}, a_{2}, a_{3}, \ldots$ be an infinite sequence of positive integers such that $a_{n+2 m}$ divides $a_{n}+a_{n+m}$ for all positive integers $n$ and $m$. Prove that this sequence is eventually periodic, i.e. there exist positive integers $N$ and $d$ such that $a_{n}=a_{n+d}$ for all $n>N$. + +Solution. We will make repeated use of the following simple observation: +Lemma 1. If a positive integer $d$ divides $a_{n}$ and $a_{n-m}$ for some $m$ and $n>2 m$, it also divides $a_{n-2 m}$. If $d$ divides $a_{n}$ and $a_{n-2 m}$, it also divides $a_{n-m}$. +Proof. Both parts are obvious since $a_{n}$ divides $a_{n-2 m}+a_{n-m}$. +Claim. The sequence $\left(a_{n}\right)$ is bounded. +Proof. Suppose the contrary. Then there exist infinitely many indices $n$ such that $a_{n}$ is greater than each of the previous terms $a_{1}, a_{2}, \ldots, a_{n-1}$. Let $a_{n}=k$ be such a term, $n>10$. For each $s<\frac{n}{2}$ the number $a_{n}=k$ divides $a_{n-s}+a_{n-2 s}<2 k$, therefore + +$$ +a_{n-s}+a_{n-2 s}=k +$$ + +In particular, + +$$ +a_{n}=a_{n-1}+a_{n-2}=a_{n-2}+a_{n-4}=a_{n-4}+a_{n-8} +$$ + +that is, $a_{n-1}=a_{n-4}$ and $a_{n-2}=a_{n-8}$. It follows from Lemma 1 that $a_{n-1}$ divides $a_{n-1-3 s}$ for $3 sN$, then $a_{j}=t$ for infinitely many $j$. + +Clearly the sequence $\left(a_{n+N}\right)_{n>0}$ satisfies the divisibility condition, and it is enough to prove that this sequence is eventually periodic. Thus truncating the sequence if necessary, we can assume that each number appears infinitely many times in the sequence. Let $k$ be the maximum number appearing in the sequence. +Lemma 2. If a positive integer $d$ divides $a_{n}$ for some $n$, then the numbers $i$ such that $d$ divides $a_{i}$ form an arithmetical progression with an odd difference. +Proof. Let $i_{1}\frac{k}{2}$, it follows from Lemma 1 that $a_{n-1}$ divides $a_{n-1-3 s}$ when $3 s\frac{k}{2}$, all the terms $a_{n-2-6 s}$ with $6 s1$. We can choose $s$ so that $a_{3 s+3}=k$. Therefore $T$, which we already know to be odd and divisible by 3 , is greater than 3 , that is, at least 9 . Then $a_{3 s-3} \neq k$, and the only other possibility is $a_{3 s-3}=k / 3$. However, $a_{3 s+3}=k$ must divide $a_{3 s}+a_{3 s-3}=2 k / 3$, which is impossible. We have proved then that $a_{3 s}=k$ for all $s>1$, which is the case (ii). + +N8. For a polynomial $P(x)$ with integer coefficients let $P^{1}(x)=P(x)$ and $P^{k+1}(x)=$ $P\left(P^{k}(x)\right)$ for $k \geqslant 1$. Find all positive integers $n$ for which there exists a polynomial $P(x)$ with integer coefficients such that for every integer $m \geqslant 1$, the numbers $P^{m}(1), \ldots, P^{m}(n)$ leave exactly $\left\lceil n / 2^{m}\right\rceil$ distinct remainders when divided by $n$. + +Answer: All powers of 2 and all primes. +Solution. Denote the set of residues modulo $\ell$ by $\mathbb{Z}_{\ell}$. Observe that $P$ can be regarded as a function $\mathbb{Z}_{\ell} \rightarrow \mathbb{Z}_{\ell}$ for any positive integer $\ell$. Denote the cardinality of the set $P^{m}\left(\mathbb{Z}_{\ell}\right)$ by $f_{m, \ell}$. Note that $f_{m, n}=\left\lceil n / 2^{m}\right\rceil$ for all $m \geqslant 1$ if and only if $f_{m+1, n}=\left\lceil f_{m, n} / 2\right\rceil$ for all $m \geqslant 0$. + +Part 1. The required polynomial exists when $n$ is a power of 2 or a prime. +If $n$ is a power of 2 , set $P(x)=2 x$. +If $n=p$ is an odd prime, every function $f: \mathbb{Z}_{p} \rightarrow \mathbb{Z}_{p}$ coincides with some polynomial with integer coefficients. So we can pick the function that sends $x \in\{0,1, \ldots, p-1\}$ to $\lfloor x / 2\rfloor$. + +Part 2. The required polynomial does not exist when $n$ is not a prime power. +Let $n=a b$ where $a, b>1$ and $\operatorname{gcd}(a, b)=1$. Note that, since $\operatorname{gcd}(a, b)=1$, + +$$ +f_{m, a b}=f_{m, a} f_{m, b} +$$ + +by the Chinese remainder theorem. Also, note that, if $f_{m, \ell}=f_{m+1, \ell}$, then $P$ permutes the image of $P^{m}$ on $\mathbb{Z}_{\ell}$, and therefore $f_{s, \ell}=f_{m, \ell}$ for all $s>m$. So, as $f_{m, a b}=1$ for sufficiently large $m$, we have for each $m$ + +$$ +f_{m, a}>f_{m+1, a} \quad \text { or } \quad f_{m, a}=1, \quad f_{m, b}>f_{m+1, b} \quad \text { or } \quad f_{m, b}=1 . +$$ + +Choose the smallest $m$ such that $f_{m+1, a}=1$ or $f_{m+1, b}=1$. Without loss of generality assume that $f_{m+1, a}=1$. Then $f_{m+1, a b}=f_{m+1, b}1$. Let us choose the smallest $k$ for which this is so. To each residue in $P\left(S_{r}\right)$ we assign its residue modulo $p^{k-1}$; denote the resulting set by $\bar{P}(S, r)$. We have $|\bar{P}(S, r)|=p^{k-2}$ by virtue of minimality of $k$. Then $\left|P\left(S_{r}\right)\right|0}$ be the set of positive real numbers. Find all functions $f: \mathbb{R}_{>0} \rightarrow \mathbb{R}_{>0}$ such that, for every $x \in \mathbb{R}_{>0}$, there exists a unique $y \in \mathbb{R}_{>0}$ satisfying + +$$ +x f(y)+y f(x) \leqslant 2 +$$ + +(Netherlands) +A4. Let $n \geqslant 3$ be an integer, and let $x_{1}, x_{2}, \ldots, x_{n}$ be real numbers in the interval $[0,1]$. Let $s=x_{1}+x_{2}+\ldots+x_{n}$, and assume that $s \geqslant 3$. Prove that there exist integers $i$ and $j$ with $1 \leqslant i2^{s-3} +$$ + +(Trinidad and Tobago) +A5. Find all positive integers $n \geqslant 2$ for which there exist $n$ real numbers $a_{1}<\cdots0$ such that the $\frac{1}{2} n(n-1)$ differences $a_{j}-a_{i}$ for $1 \leqslant i3$ be a positive integer. Suppose that $n$ children are arranged in a circle, and $n$ coins are distributed between them (some children may have no coins). At every step, a child with at least 2 coins may give 1 coin to each of their immediate neighbours on the right and left. Determine all initial distributions of coins from which it is possible that, after a finite number of steps, each child has exactly one coin. + +C5. Let $m, n \geqslant 2$ be integers, let $X$ be a set with $n$ elements, and let $X_{1}, X_{2}, \ldots, X_{m}$ be pairwise distinct non-empty, not necessary disjoint subsets of $X$. A function $f: X \rightarrow$ $\{1,2, \ldots, n+1\}$ is called nice if there exists an index $k$ such that + +$$ +\sum_{x \in X_{k}} f(x)>\sum_{x \in X_{i}} f(x) \text { for all } i \neq k +$$ + +Prove that the number of nice functions is at least $n^{n}$. +(Germany) +C6. Let $n$ be a positive integer. We start with $n$ piles of pebbles, each initially containing a single pebble. One can perform moves of the following form: choose two piles, take an equal number of pebbles from each pile and form a new pile out of these pebbles. For each positive integer $n$, find the smallest number of non-empty piles that one can obtain by performing a finite sequence of moves of this form. +(Croatia) +C7. Lucy starts by writing $s$ integer-valued 2022-tuples on a blackboard. After doing that, she can take any two (not necessarily distinct) tuples $\mathbf{v}=\left(v_{1}, \ldots, v_{2022}\right)$ and $\mathbf{w}=\left(w_{1}, \ldots, w_{2022}\right)$ that she has already written, and apply one of the following operations to obtain a new tuple: + +$$ +\begin{aligned} +& \mathbf{v}+\mathbf{w}=\left(v_{1}+w_{1}, \ldots, v_{2022}+w_{2022}\right) \\ +& \mathbf{v} \vee \mathbf{w}=\left(\max \left(v_{1}, w_{1}\right), \ldots, \max \left(v_{2022}, w_{2022}\right)\right) +\end{aligned} +$$ + +and then write this tuple on the blackboard. +It turns out that, in this way, Lucy can write any integer-valued 2022-tuple on the blackboard after finitely many steps. What is the smallest possible number $s$ of tuples that she initially wrote? +(Czech Republic) + +## C8. + +Alice fills the fields of an $n \times n$ board with numbers from 1 to $n^{2}$, each number being used exactly once. She then counts the total number of good paths on the board. A good path is a sequence of fields of arbitrary length (including 1) such that: +(i) The first field in the sequence is one that is only adjacent to fields with larger numbers, +(ii) Each subsequent field in the sequence is adjacent to the previous field, +(iii) The numbers written on the fields in the sequence are in increasing order. + +Two fields are considered adjacent if they share a common side. Find the smallest possible number of good paths Alice can obtain, as a function of $n$. +(Serbia) +C9. Let $\mathbb{Z}_{\geqslant 0}$ be the set of non-negative integers, and let $f: \mathbb{Z}_{\geqslant 0} \times \mathbb{Z}_{\geqslant 0} \rightarrow \mathbb{Z}_{\geqslant 0}$ be a bijection such that whenever $f\left(x_{1}, y_{1}\right)>f\left(x_{2}, y_{2}\right)$, we have $f\left(x_{1}+1, y_{1}\right)>f\left(x_{2}+1, y_{2}\right)$ and $f\left(x_{1}, y_{1}+1\right)>f\left(x_{2}, y_{2}+1\right)$. + +Let $N$ be the number of pairs of integers $(x, y)$, with $0 \leqslant x, y<100$, such that $f(x, y)$ is odd. Find the smallest and largest possible value of $N$. + +## Geometry + +G1. Let $A B C D E$ be a convex pentagon such that $B C=D E$. Assume there is a point $T$ inside $A B C D E$ with $T B=T D, T C=T E$ and $\angle T B A=\angle A E T$. Let lines $C D$ and $C T$ intersect line $A B$ at points $P$ and $Q$, respectively, and let lines $C D$ and $D T$ intersect line $A E$ at points $R$ and $S$, respectively. Assume that points $P, B, A, Q$ and $R, E, A, S$ respectively, are collinear and occur on their lines in this order. Prove that the points $P, S, Q, R$ are concyclic. +(Slovakia) +G2. In the acute-angled triangle $A B C$, the point $F$ is the foot of the altitude from $A$, and $P$ is a point on the segment $A F$. The lines through $P$ parallel to $A C$ and $A B$ meet $B C$ at $D$ and $E$, respectively. Points $X \neq A$ and $Y \neq A$ lie on the circles $A B D$ and $A C E$, respectively, such that $D A=D X$ and $E A=E Y$. + +Prove that $B, C, X$ and $Y$ are concyclic. +(Netherlands) +G3. Let $A B C D$ be a cyclic quadrilateral. Assume that the points $Q, A, B, P$ are collinear in this order, in such a way that the line $A C$ is tangent to the circle $A D Q$, and the line $B D$ is tangent to the circle $B C P$. Let $M$ and $N$ be the midpoints of $B C$ and $A D$, respectively. Prove that the following three lines are concurrent: line $C D$, the tangent of circle $A N Q$ at point $A$, and the tangent to circle $B M P$ at point $B$. +(Slovakia) +G4. Let $A B C$ be an acute-angled triangle with $A C>A B$, let $O$ be its circumcentre, and let $D$ be a point on the segment $B C$. The line through $D$ perpendicular to $B C$ intersects the lines $A O, A C$ and $A B$ at $W, X$ and $Y$, respectively. The circumcircles of triangles $A X Y$ and $A B C$ intersect again at $Z \neq A$. + +Prove that if $O W=O D$, then $D Z$ is tangent to the circle $A X Y$. +(United Kingdom) +G5. Let $A B C$ be a triangle, and let $\ell_{1}$ and $\ell_{2}$ be two parallel lines. For $i=1,2$, let $\ell_{i}$ meet the lines $B C, C A$, and $A B$ at $X_{i}, Y_{i}$, and $Z_{i}$, respectively. Suppose that the line through $X_{i}$ perpendicular to $B C$, the line through $Y_{i}$ perpendicular to $C A$, and finally the line through $Z_{i}$ perpendicular to $A B$, determine a non-degenerate triangle $\Delta_{i}$. + +Show that the circumcircles of $\Delta_{1}$ and $\Delta_{2}$ are tangent to each other. +(Vietnam) +G6. In an acute-angled triangle $A B C$, point $H$ is the foot of the altitude from $A$. Let $P$ be a moving point such that the bisectors $k$ and $\ell$ of angles $P B C$ and $P C B$, respectively, intersect each other on the line segment $A H$. Let $k$ and $A C$ meet at $E$, let $\ell$ and $A B$ meet at $F$, and let $E F$ and $A H$ meet at $Q$. Prove that, as $P$ varies, the line $P Q$ passes through a fixed point. +(Iran) +G7. Let $A B C$ and $A^{\prime} B^{\prime} C^{\prime}$ be two triangles having the same circumcircle $\omega$, and the same orthocentre $H$. Let $\Omega$ be the circumcircle of the triangle determined by the lines $A A^{\prime}, B B^{\prime}$ and $C C^{\prime}$. Prove that $H$, the centre of $\omega$, and the centre of $\Omega$ are collinear. +(Denmark) +G8. +Let $A A^{\prime} B C C^{\prime} B^{\prime}$ be a convex cyclic hexagon such that $A C$ is tangent to the incircle of the triangle $A^{\prime} B^{\prime} C^{\prime}$, and $A^{\prime} C^{\prime}$ is tangent to the incircle of the triangle $A B C$. Let the lines $A B$ and $A^{\prime} B^{\prime}$ meet at $X$ and let the lines $B C$ and $B^{\prime} C^{\prime}$ meet at $Y$. + +Prove that if $X B Y B^{\prime}$ is a convex quadrilateral, then it has an incircle. + +## Number Theory + +N1. A number is called Norwegian if it has three distinct positive divisors whose sum is equal to 2022. Determine the smallest Norwegian number. +(Note: The total number of positive divisors of a Norwegian number is allowed to be larger than 3.) +(Cyprus) +N2. Find all positive integers $n>2$ such that + +$$ +n!\mid \prod_{\substack{p1$ be a positive integer, and let $d>1$ be a positive integer coprime to $a$. Let $x_{1}=1$ and, for $k \geqslant 1$, define + +$$ +x_{k+1}= \begin{cases}x_{k}+d & \text { if } a \text { doesn't divide } x_{k} \\ x_{k} / a & \text { if } a \text { divides } x_{k}\end{cases} +$$ + +Find the greatest positive integer $n$ for which there exists an index $k$ such that $x_{k}$ is divisible by $a^{n}$. +(Croatia) +N4. Find all triples of positive integers $(a, b, p)$ with $p$ prime and + +$$ +a^{p}=b!+p +$$ + +(Belgium) +N5. For each $1 \leqslant i \leqslant 9$ and $T \in \mathbb{N}$, define $d_{i}(T)$ to be the total number of times the digit $i$ appears when all the multiples of 1829 between 1 and $T$ inclusive are written out in base 10. + +Show that there are infinitely many $T \in \mathbb{N}$ such that there are precisely two distinct values among $d_{1}(T), d_{2}(T), \ldots, d_{9}(T)$. +(United Kingdom) +N6. Let $Q$ be a set of prime numbers, not necessarily finite. For a positive integer $n$ consider its prime factorisation; define $p(n)$ to be the sum of all the exponents and $q(n)$ to be the sum of the exponents corresponding only to primes in $Q$. A positive integer $n$ is called special if $p(n)+p(n+1)$ and $q(n)+q(n+1)$ are both even integers. Prove that there is a constant $c>0$ independent of the set $Q$ such that for any positive integer $N>100$, the number of special integers in $[1, N]$ is at least $c N$. +(For example, if $Q=\{3,7\}$, then $p(42)=3, q(42)=2, p(63)=3, q(63)=3, p(2022)=3$, $q(2022)=1$. +(Costa Rica) +N7. Let $k$ be a positive integer and let $S$ be a finite set of odd prime numbers. Prove that there is at most one way (modulo rotation and reflection) to place the elements of $S$ around a circle such that the product of any two neighbors is of the form $x^{2}+x+k$ for some positive integer $x$. +(U.S.A.) + +N8. Prove that $5^{n}-3^{n}$ is not divisible by $2^{n}+65$ for any positive integer $n$. + +## Solutions + +## Algebra + +A1. Let $\left(a_{n}\right)_{n \geqslant 1}$ be a sequence of positive real numbers with the property that + +$$ +\left(a_{n+1}\right)^{2}+a_{n} a_{n+2} \leqslant a_{n}+a_{n+2} +$$ + +for all positive integers $n$. Show that $a_{2022} \leqslant 1$. +(Nigeria) +Solution. We begin by observing that $\left(a_{n+1}\right)^{2}-1 \leqslant a_{n}+a_{n+2}-a_{n} a_{n+2}-1$, which is equivalent to + +$$ +\left(a_{n+1}\right)^{2}-1 \leqslant\left(1-a_{n}\right)\left(a_{n+2}-1\right) +$$ + +Suppose now that there exists a positive integer $n$ such that $a_{n+1}>1$ and $a_{n+2}>1$. Since $\left(a_{n+1}\right)^{2}-1 \leqslant\left(1-a_{n}\right)\left(a_{n+2}-1\right)$, we deduce that $0<1-a_{n}<1<1+a_{n+2}$, thus $\left(a_{n+1}\right)^{2}-1<\left(a_{n+2}+1\right)\left(a_{n+2}-1\right)=\left(a_{n+2}\right)^{2}-1$. + +On the other hand, $\left(a_{n+2}\right)^{2}-1 \leqslant\left(1-a_{n+3}\right)\left(a_{n+1}-1\right)<\left(1+a_{n+1}\right)\left(a_{n+1}-1\right)=\left(a_{n+1}\right)^{2}-1$, a contradiction. We have shown that we cannot have two consecutive terms, except maybe $a_{1}$ and $a_{2}$, strictly greater than 1 . + +Finally, suppose $a_{2022}>1$. This implies that $a_{2021} \leqslant 1$ and $a_{2023} \leqslant 1$. Therefore $0<$ $\left(a_{2022}\right)^{2}-1 \leqslant\left(1-a_{2021}\right)\left(a_{2023}-1\right) \leqslant 0$, a contradiction. We conclude that $a_{2022} \leqslant 1$. + +A2. Let $k \geqslant 2$ be an integer. Find the smallest integer $n \geqslant k+1$ with the property that there exists a set of $n$ distinct real numbers such that each of its elements can be written as a sum of $k$ other distinct elements of the set. +(Slovakia) +Answer: $n=k+4$. +Solution. First we show that $n \geqslant k+4$. Suppose that there exists such a set with $n$ numbers and denote them by $a_{1}a_{1}+\cdots+a_{k} \geqslant a_{k+1}$, which gives a contradiction. + +If $n=k+2$ then we have $a_{1} \geqslant a_{2}+\cdots+a_{k+1} \geqslant a_{k+2}$, that again gives a contradiction. +If $n=k+3$ then we have $a_{1} \geqslant a_{2}+\cdots+a_{k+1}$ and $a_{3}+\cdots+a_{k+2} \geqslant a_{k+3}$. Adding the two inequalities we get $a_{1}+a_{k+2} \geqslant a_{2}+a_{k+3}$, again a contradiction. + +It remains to give an example of a set with $k+4$ elements satisfying the condition of the problem. We start with the case when $k=2 l$ and $l \geqslant 1$. In that case, denote by $A_{i}=\{-i, i\}$ and take the set $A_{1} \cup \cdots \cup A_{l+2}$, which has exactly $k+4=2 l+4$ elements. We are left to show that this set satisfies the required condition. + +Note that if a number $i$ can be expressed in the desired way, then so can $-i$ by negating the expression. Therefore, we consider only $1 \leqslant i \leqslant l+2$. + +If $i0}$ be the set of positive real numbers. Find all functions $f: \mathbb{R}_{>0} \rightarrow \mathbb{R}_{>0}$ such that, for every $x \in \mathbb{R}_{>0}$, there exists a unique $y \in \mathbb{R}_{>0}$ satisfying + +$$ +x f(y)+y f(x) \leqslant 2 . +$$ + +(Netherlands) +Answer: The function $f(x)=1 / x$ is the only solution. +Solution 1. First we prove that the function $f(x)=1 / x$ satisfies the condition of the problem statement. The AM-GM inequality gives + +$$ +\frac{x}{y}+\frac{y}{x} \geqslant 2 +$$ + +for every $x, y>0$, with equality if and only if $x=y$. This means that, for every $x>0$, there exists a unique $y>0$ such that + +$$ +\frac{x}{y}+\frac{y}{x} \leqslant 2, +$$ + +namely $y=x$. +Let now $f: \mathbb{R}_{>0} \rightarrow \mathbb{R}_{>0}$ be a function that satisfies the condition of the problem statement. We say that a pair of positive real numbers $(x, y)$ is $\operatorname{good}$ if $x f(y)+y f(x) \leqslant 2$. Observe that if $(x, y)$ is good, then so is $(y, x)$. +Lemma 1.0. If $(x, y)$ is good, then $x=y$. +Proof. Assume that there exist positive real numbers $x \neq y$ such that $(x, y)$ is good. The uniqueness assumption says that $y$ is the unique positive real number such that $(x, y)$ is good. In particular, $(x, x)$ is not a good pair. This means that + +$$ +x f(x)+x f(x)>2 +$$ + +and thus $x f(x)>1$. Similarly, $(y, x)$ is a good pair, so $(y, y)$ is not a good pair, which implies $y f(y)>1$. We apply the AM-GM inequality to obtain + +$$ +x f(y)+y f(x) \geqslant 2 \sqrt{x f(y) \cdot y f(x)}=2 \sqrt{x f(x) \cdot y f(y)}>2 . +$$ + +This is a contradiction, since $(x, y)$ is a good pair. +By assumption, for any $x>0$, there always exists a good pair containing $x$, however Lemma 1 implies that the only good pair that can contain $x$ is $(x, x)$, so + +$$ +x f(x) \leqslant 1 \quad \Longleftrightarrow \quad f(x) \leqslant \frac{1}{x} +$$ + +for every $x>0$. +In particular, with $x=1 / f(t)$ for $t>0$, we obtain + +$$ +\frac{1}{f(t)} \cdot f\left(\frac{1}{f(t)}\right) \leqslant 1 +$$ + +Hence + +$$ +t \cdot f\left(\frac{1}{f(t)}\right) \leqslant t f(t) \leqslant 1 +$$ + +We claim that $(t, 1 / f(t))$ is a good pair for every $t>0$. Indeed, + +$$ +t \cdot f\left(\frac{1}{f(t)}\right)+\frac{1}{f(t)} f(t)=t \cdot f\left(\frac{1}{f(t)}\right)+1 \leqslant 2 +$$ + +Lemma 1 implies that $t=1 / f(t) \Longleftrightarrow f(t)=1 / t$ for every $t>0$. + +Solution 1.1. We give an alternative way to prove that $f(x)=1 / x$ assuming $f(x) \leqslant 1 / x$ for every $x>0$. + +Indeed, if $f(x)<1 / x$ then for every $a>0$ with $f(x)<1 / a<1 / x$ (and there are at least two of them), we have + +$$ +a f(x)+x f(a)<1+\frac{x}{a}<2 +$$ + +Hence $(x, a)$ is a good pair for every such $a$, a contradiction. We conclude that $f(x)=1 / x$. +Solution 1.2. We can also conclude from Lemma 1 and $f(x) \leqslant 1 / x$ as follows. +Lemma 2. The function $f$ is decreasing. +Proof. Let $y>x>0$. Lemma 1 says that $(x, y)$ is not a good pair, but $(y, y)$ is. Hence + +$$ +x f(y)+y f(x)>2 \geqslant 2 y f(y)>y f(y)+x f(y) +$$ + +where we used $y>x$ (and $f(y)>0$ ) in the last inequality. This implies that $f(x)>f(y)$, showing that $f$ is decreasing. + +We now prove that $f(x)=1 / x$ for all $x$. Fix a value of $x$ and note that for $y>x$ we must have $x f(x)+y f(x)>x f(y)+y f(x)>2$ (using that $f$ is decreasing for the first step), hence $f(x)>\frac{2}{x+y}$. The last inequality is true for every $y>x>0$. If we fix $x$ and look for the supremum of the expression $\frac{2}{x+y}$ over all $y>x$, we get + +$$ +f(x) \geqslant \frac{2}{x+x}=\frac{1}{x} +$$ + +Since we already know that $f(x) \leqslant 1 / x$, we conclude that $f(x)=1 / x$. +Solution 2.0. As in the first solution, we note that $f(x)=1 / x$ is a solution, and we set out to prove that it is the only one. We write $g(x)$ for the unique positive real number such that $(x, g(x))$ is a good pair. In this solution, we prove Lemma 2 without assuming Lemma 1. +Lemma 2. The function $f$ is decreasing. +Proof. Consider $x$ 2. Combining these two inequalities yields + +$$ +x f(g(y))+g(y) f(x)>2 \geqslant y f(g(y))+g(y) f(y) +$$ + +or $f(g(y))(x-y)>g(y)(f(y)-f(x))$. Because $g(y)$ and $f(g(y))$ are both positive while $x-y$ is negative, it follows that $f(y)2$ and $y f(y)+y f(y)>2$, which implies $x f(x)+y f(y)>2$. Now + +$$ +x f(x)+y f(y)>2 \geqslant x f(y)+y f(x) +$$ + +implies $(x-y)(f(x)-f(y))>0$, which contradicts the fact that $f$ is decreasing. So $y=x$ is the unique $y$ such that $(x, y)$ is a good pair, and in particular we have $f(x) \leqslant 1 / x$. + +We can now conclude the proof as in any of the Solutions 1.x. +Solution 3.0. As in the other solutions we verify that the function $f(x)=1 / x$ is a solution. We first want to prove the following lemma: +Lemma 3. For all $x \in \mathbb{R}_{>0}$ we actually have $x f(g(x))+g(x) f(x)=2$ (that is: the inequality is actually an equality). + +Proof. We proceed by contradiction: Assume there exists some number $x>0$ such that for $y=g(x)$ we have $x f(y)+y f(x)<2$. Then for any $0<\epsilon<\frac{2-x f(y)-y f(x)}{2 f(x)}$ we have, by uniqueness of $y$, that $x f(y+\epsilon)+(y+\epsilon) f(x)>2$. Therefore + +$$ +\begin{aligned} +f(y+\epsilon) & >\frac{2-(y+\epsilon) f(x)}{x}=\frac{2-y f(x)-\epsilon f(x)}{x} \\ +& >\frac{2-y f(x)-\frac{2-x f(y)-y f(x)}{2}}{x} \\ +& =\frac{2-x f(y)-y f(x)}{2 x}+f(y)>f(y) . +\end{aligned} +$$ + +Furthermore, for every such $\epsilon$ we have $g(y+\epsilon) f(y+\epsilon)+(y+\epsilon) f(g(y+\epsilon)) \leqslant 2$ and $g(y+\epsilon) f(y)+y f(g(y+\epsilon))>2($ since $y \neq y+\epsilon=g(g(y+\epsilon))$ ). This gives us the two inequalities + +$$ +f(g(y+\epsilon)) \leqslant \frac{2-g(y+\epsilon) f(y+\epsilon)}{y+\epsilon} \quad \text { and } \quad f(g(y+\epsilon))>\frac{2-g(y+\epsilon) f(y)}{y} . +$$ + +Combining these two inequalities and rearranging the terms leads to the inequality + +$$ +2 \epsilon0}$ we have + +$$ +x f(y)+y f(x) \geqslant 2 +$$ + +since for $y=g(x)$ we have equality and by uniqueness for $y \neq g(x)$ the inequality is strict. +In particular for every $x \in \mathbb{R}_{>0}$ and for $y=x$ we have $2 x f(x) \geqslant 2$, or equivalently $f(x) \geqslant 1 / x$ for all $x \in \mathbb{R}_{>0}$. With this inequality we obtain for all $x \in \mathbb{R}_{>0}$ + +$$ +2 \geqslant x f(g(x))+g(x) f(x) \geqslant \frac{x}{g(x)}+\frac{g(x)}{x} \geqslant 2 +$$ + +where the first inequality comes from the problem statement. Consequently each of these inequalities must actually be an equality, and in particular we obtain $f(x)=1 / x$ for all $x \in \mathbb{R}_{>0}$. + +Solution 4. Again, let us prove that $f(x)=1 / x$ is the only solution. Let again $g(x)$ be the unique positive real number such that $(x, g(x))$ is a good pair. +Lemma 4. The function $f$ is strictly convex. +Proof. Consider the function $q_{s}(x)=f(x)+s x$ for some real number $s$. If $f$ is not strictly convex, then there exist $u2^{s-3} +$$ + +(Trinidad and Tobago) + +## Solution. + +Let $1 \leqslant ac 2^{s} . +$$ + +The above shows that $c=1 / 8$ is the best possible. A somewhat simpler ending to the proof can be given for $c=1 / 32$. +End of solution for $c=1 / 32$. +As in the original solution, we arrive at + +$$ +s<2^{u+1} x_{a}+2^{v+1} x_{b}+((b-v)-(a+u)-1)=b-a+\left(2^{u+1-\alpha}+2^{v+1-\beta}-(u+v+1)\right) . +$$ + +Now $2^{b-a} x_{a} x_{b} \geqslant 2^{b-a} 2^{-u-1} 2^{-v-1}$, so it is enough to show $s-50$ such that the $\frac{1}{2} n(n-1)$ differences $a_{j}-a_{i}$ for $1 \leqslant i1$ of $x^{2}-x-1=0$ (the golden ratio) and set $\left(a_{1}, a_{2}, a_{3}\right)=$ $\left(0, r, r+r^{2}\right)$. Then + +$$ +\left(a_{2}-a_{1}, a_{3}-a_{2}, a_{3}-a_{1}\right)=\left(r, r^{2}, r+r^{2}=r^{3}\right) +$$ + +For $n=4$, take the root $r \in(1,2)$ of $x^{3}-x-1=0$ (such a root exists because $1^{3}-1-1<0$ and $\left.2^{3}-2-1>0\right)$ and set $\left(a_{1}, a_{2}, a_{3}, a_{4}\right)=\left(0, r, r+r^{2}, r+r^{2}+r^{3}\right)$. Then + +$$ +\left(a_{2}-a_{1}, a_{3}-a_{2}, a_{4}-a_{3}, a_{3}-a_{1}, a_{4}-a_{2}, a_{4}-a_{1}\right)=\left(r, r^{2}, r^{3}, r^{4}, r^{5}, r^{6}\right) +$$ + +For $n \geqslant 5$, we will proceed by contradiction. Suppose there exist numbers $a_{1}<\cdots1$ satisfying the conditions of the problem. We start with a lemma: +Lemma. We have $r^{n-1}>2$. +Proof. There are only $n-1$ differences $a_{j}-a_{i}$ with $j=i+1$, so there exists an exponent $e \leqslant n$ and a difference $a_{j}-a_{i}$ with $j \geqslant i+2$ such that $a_{j}-a_{i}=r^{e}$. This implies + +$$ +r^{n} \geqslant r^{e}=a_{j}-a_{i}=\left(a_{j}-a_{j-1}\right)+\left(a_{j-1}-a_{i}\right)>r+r=2 r, +$$ + +thus $r^{n-1}>2$ as desired. +To illustrate the general approach, we first briefly sketch the idea behind the argument in the special case $n=5$. In this case, we clearly have $a_{5}-a_{1}=r^{10}$. Note that there are 3 ways to rewrite $a_{5}-a_{1}$ as a sum of two differences, namely + +$$ +\left(a_{5}-a_{4}\right)+\left(a_{4}-a_{1}\right),\left(a_{5}-a_{3}\right)+\left(a_{3}-a_{1}\right),\left(a_{5}-a_{2}\right)+\left(a_{2}-a_{1}\right) . +$$ + +Using the lemma above and convexity of the function $f(n)=r^{n}$, we argue that those three ways must be $r^{10}=r^{9}+r^{1}=r^{8}+r^{4}=r^{7}+r^{6}$. That is, the "large" exponents keep dropping by 1 , while the "small" exponents keep increasing by $n-2, n-3, \ldots, 2$. Comparing any two such equations, we then get a contradiction unless $n \leqslant 4$. + +Now we go back to the full proof for any $n \geqslant 5$. Denote $b=\frac{1}{2} n(n-1)$. Clearly, we have $a_{n}-a_{1}=r^{b}$. Consider the $n-2$ equations of the form: + +$$ +a_{n}-a_{1}=\left(a_{n}-a_{i}\right)+\left(a_{i}-a_{1}\right) \text { for } i \in\{2, \ldots, n-1\} +$$ + +In each equation, one of the two terms on the right-hand side must be at least $\frac{1}{2}\left(a_{n}-a_{1}\right)$. But from the lemma we have $r^{b-(n-1)}=r^{b} / r^{n-1}<\frac{1}{2}\left(a_{n}-a_{1}\right)$, so there are at most $n-2$ sufficiently large elements in $\left\{r^{k} \mid 1 \leqslant k1$ and $f(r)=r^{n}$ is convex, we have + +$$ +r^{b-1}-r^{b-2}>r^{b-2}-r^{b-3}>\ldots>r^{b-(n-3)}-r^{b-(n-2)} +$$ + +implying + +$$ +r^{\alpha_{2}}-r^{\alpha_{1}}>r^{\alpha_{3}}-r^{\alpha_{2}}>\ldots>r^{\alpha_{n-2}}-r^{\alpha_{n-3}} . +$$ + +Convexity of $f(r)=r^{n}$ further implies + +$$ +\alpha_{2}-\alpha_{1}>\alpha_{3}-\alpha_{2}>\ldots>\alpha_{n-2}-\alpha_{n-3} +$$ + +Note that $\alpha_{n-2}-\alpha_{n-3} \geqslant 2$ : Otherwise we would have $\alpha_{n-2}-\alpha_{n-3}=1$ and thus + +$$ +r^{\alpha_{n-3}} \cdot(r-1)=r^{\alpha_{n-2}}-r^{\alpha_{n-3}}=r^{b-(n-3)}-r^{b-(n-2)}=r^{b-(n-2)} \cdot(r-1), +$$ + +implying that $\alpha_{n-3}=b-(n-2)$, a contradiction. Therefore, we have + +$$ +\begin{aligned} +\alpha_{n-2}-\alpha_{1} & =\left(\alpha_{n-2}-\alpha_{n-3}\right)+\cdots+\left(\alpha_{2}-\alpha_{1}\right) \\ +& \geqslant 2+3+\cdots+(n-2) \\ +& =\frac{1}{2}(n-2)(n-1)-1=\frac{1}{2} n(n-3) +\end{aligned} +$$ + +On the other hand, from $\alpha_{n-2} \leqslant b-(n-1)$ and $\alpha_{1} \geqslant 1$ we get + +$$ +\alpha_{n-2}-\alpha_{1} \leqslant b-n=\frac{1}{2} n(n-1)-n=\frac{1}{2} n(n-3), +$$ + +implying that equalities must occur everywhere and the claim about the small terms follows. +Now, assuming $n-2 \geqslant 2$, we have the two different equations: + +$$ +r^{b}=r^{b-(n-2)}+r^{b-(n-2)-1} \text { and } r^{b}=r^{b-(n-3)}+r^{b-(n-2)-3}, +$$ + +which can be rewritten as + +$$ +r^{n-1}=r+1 \quad \text { and } \quad r^{n+1}=r^{4}+1 +$$ + +Simple algebra now gives + +$$ +r^{4}+1=r^{n+1}=r^{n-1} \cdot r^{2}=r^{3}+r^{2} \Longrightarrow(r-1)\left(r^{3}-r-1\right)=0 . +$$ + +Since $r \neq 1$, using Equation (1) we conclude $r^{3}=r+1=r^{n-1}$, thus $n=4$, which gives a contradiction. + +A6. Let $\mathbb{R}$ be the set of real numbers. We denote by $\mathcal{F}$ the set of all functions $f: \mathbb{R} \rightarrow \mathbb{R}$ such that + +$$ +f(x+f(y))=f(x)+f(y) +$$ + +for every $x, y \in \mathbb{R}$. Find all rational numbers $q$ such that for every function $f \in \mathcal{F}$, there exists some $z \in \mathbb{R}$ satisfying $f(z)=q z$. +(Indonesia) +Answer: The desired set of rational numbers is $\left\{\frac{n+1}{n}: n \in \mathbb{Z}, n \neq 0\right\}$. +Solution. Let $Z$ be the set of all rational numbers $q$ such that for every function $f \in \mathcal{F}$, there exists some $z \in \mathbb{R}$ satisfying $f(z)=q z$. Let further + +$$ +S=\left\{\frac{n+1}{n}: n \in \mathbb{Z}, n \neq 0\right\} +$$ + +We prove that $Z=S$ by showing the two inclusions: $S \subseteq Z$ and $Z \subseteq S$. +We first prove that $S \subseteq Z$. Let $f \in \mathcal{F}$ and let $P(x, y)$ be the relation $f(x+f(y))=f(x)+$ $f(y)$. First note that $P(0,0)$ gives $f(f(0))=2 f(0)$. Then, $P(0, f(0))$ gives $f(2 f(0))=3 f(0)$. We claim that + +$$ +f(k f(0))=(k+1) f(0) +$$ + +for every integer $k \geqslant 1$. The claim can be proved by induction. The cases $k=1$ and $k=2$ have already been established. Assume that $f(k f(0))=(k+1) f(0)$ and consider $P(0, k f(0))$ which gives + +$$ +f((k+1) f(0))=f(0)+f(k f(0))=(k+2) f(0) +$$ + +This proves the claim. We conclude that $\frac{k+1}{k} \in Z$ for every integer $k \geqslant 1$. Note that $P(-f(0), 0)$ gives $f(-f(0))=0$. We now claim that + +$$ +f(-k f(0))=(-k+1) f(0) +$$ + +for every integer $k \geqslant 1$. The proof by induction is similar to the one above. We conclude that $\frac{-k+1}{-k} \in Z$ for every integer $k \geqslant 1$. This shows that $S \subseteq Z$. + +We now prove that $Z \subseteq S$. Let $p$ be a rational number outside the set $S$. We want to prove that $p$ does not belong to $Z$. To that end, we construct a function $f \in \mathcal{F}$ such that $f(z) \neq p z$ for every $z \in \mathbb{R}$. The strategy is to first construct a function + +$$ +g:[0,1) \rightarrow \mathbb{Z} +$$ + +and then define $f$ as $f(x)=g(\{x\})+\lfloor x\rfloor$. This function $f$ belongs to $\mathcal{F}$. Indeed, + +$$ +\begin{aligned} +f(x+f(y)) & =g(\{x+f(y)\})+\lfloor x+f(y)\rfloor \\ +& =g(\{x+g(\{y\})+\lfloor y\rfloor\})+\lfloor x+g(\{y\})+\lfloor y\rfloor\rfloor \\ +& =g(\{x\})+\lfloor x\rfloor+g(\{y\})+\lfloor y\rfloor \\ +& =f(x)+f(y) +\end{aligned} +$$ + +where we used that $g$ only takes integer values. +Lemma 1. For every $\alpha \in[0,1)$, there exists $m \in \mathbb{Z}$ such that + +$$ +m+n \neq p(\alpha+n) +$$ + +for every $n \in \mathbb{Z}$. + +Proof. Note that if $p=1$ the claim is trivial. If $p \neq 1$, then the claim is equivalent to the existence of an integer $m$ such that + +$$ +\frac{m-p \alpha}{p-1} +$$ + +is never an integer. Assume the contrary. That would mean that both + +$$ +\frac{m-p \alpha}{p-1} \quad \text { and } \quad \frac{(m+1)-p \alpha}{p-1} +$$ + +are integers, and so is their difference. The latter is equal to + +$$ +\frac{1}{p-1} +$$ + +Since we assumed $p \notin S, 1 /(p-1)$ is never an integer. This is a contradiction. +Define $g:[0,1) \rightarrow \mathbb{Z}$ by $g(\alpha)=m$ for any integer $m$ that satisfies the conclusion of Lemma 1. Note that $f(z) \neq p z$ if and and only if + +$$ +g(\{z\})+\lfloor z\rfloor \neq p(\{z\}+\lfloor z\rfloor) +$$ + +The latter is guaranteed by the construction of the function $g$. We conclude that $p \notin Z$ as desired. This shows that $Z \subset S$. + +A7. For a positive integer $n$ we denote by $s(n)$ the sum of the digits of $n$. Let $P(x)=$ $x^{n}+a_{n-1} x^{n-1}+\cdots+a_{1} x+a_{0}$ be a polynomial, where $n \geqslant 2$ and $a_{i}$ is a positive integer for all $0 \leqslant i \leqslant n-1$. Could it be the case that, for all positive integers $k, s(k)$ and $s(P(k))$ have the same parity? +(Belarus) +Answer: No. For any such polynomial there exists a positive integer $k$ such that $s(k)$ and $s(P(k))$ have different parities. + +Solution. With the notation above, we begin by choosing a positive integer $t$ such that + +$$ +10^{t}>\max \left\{\frac{100^{n-1} a_{n-1}}{\left(10^{\frac{1}{n-1}}-9^{\frac{1}{n-1}}\right)^{n-1}}, \frac{a_{n-1}}{9} 10^{n-1}, \frac{a_{n-1}}{9}\left(10 a_{n-1}\right)^{n-1}, \ldots, \frac{a_{n-1}}{9}\left(10 a_{0}\right)^{n-1}\right\} +$$ + +As a direct consequence of $10^{t}$ being bigger than the first quantity listed in the above set, we get that the interval + +$$ +I=\left[\left(\frac{9}{a_{n-1}} 10^{t}\right)^{\frac{1}{n-1}},\left(\frac{1}{a_{n-1}} 10^{t+1}\right)^{\frac{1}{n-1}}\right) +$$ + +contains at least 100 consecutive positive integers. +Let $X$ be a positive integer in $I$ such that $X$ is congruent to $1 \bmod 100$. Since $X \in I$ we have + +$$ +9 \cdot 10^{t} \leqslant a_{n-1} X^{n-1}<10^{t+1} +$$ + +thus the first digit (from the left) of $a_{n-1} X^{n-1}$ must be 9 . +Next, we observe that $a_{n-1}\left(10 a_{i}\right)^{n-1}<9 \cdot 10^{t} \leqslant a_{n-1} X^{n-1}$, thus $10 a_{i}10^{\alpha i} a_{n-1} X^{n-1}>$ $10^{\alpha i} a_{i} X^{i}$, the terms $a_{i} 10^{\alpha i} X^{i}$ do not interact when added; in particular, there is no carryover caused by addition. Thus we have $s\left(P\left(10^{\alpha} X\right)\right)=s\left(X^{n}\right)+s\left(a_{n-1} X^{n-1}\right)+\cdots+s\left(a_{0}\right)$. + +We now look at $P\left(10^{\alpha-1} X\right)=10^{(\alpha-1) n} X^{n}+a_{n-1} 10^{(\alpha-1)(n-1)} X^{n-1}+\cdots+a_{0}$. Firstly, if $i10^{(\alpha-1) i} a_{n-1} X^{n-1} \geqslant 10^{(\alpha-1) i+1} a_{i} X^{i}$, thus all terms $10^{(\alpha-1) i} a_{i} X^{i}$ for $0 \leqslant i \leqslant n-1$ come in 'blocks', exactly as in the previous case. + +Finally, $10^{(\alpha-1) n+1}>10^{(\alpha-1)(n-1)} a_{n-1} X^{n-1} \geqslant 10^{(\alpha-1) n}$, thus $10^{(\alpha-1)(n-1)} a_{n-1} X^{n-1}$ has exactly $(\alpha-1) n+1$ digits, and its first digit is 9 , as established above. On the other hand, $10^{(\alpha-1) n} X^{n}$ has exactly $(\alpha-1) n$ zeros, followed by $01($ as $X$ is $1 \bmod 100)$. Therefore, when we add the terms, the 9 and 1 turn into 0 , the 0 turns into 1 , and nothing else is affected. + +Putting everything together, we obtain + +$$ +s\left(P\left(10^{\alpha-1} X\right)\right)=s\left(X^{n}\right)+s\left(a_{n-1} X^{n-1}\right)+\cdots+s\left(a_{0}\right)-9=s\left(P\left(10^{\alpha} X\right)\right)-9 +$$ + +thus $s\left(P\left(10^{\alpha} X\right)\right)$ and $s\left(P\left(10^{\alpha-1} X\right)\right)$ have different parities, as claimed. + +A8. For a positive integer $n$, an $n$-sequence is a sequence $\left(a_{0}, \ldots, a_{n}\right)$ of non-negative integers satisfying the following condition: if $i$ and $j$ are non-negative integers with $i+j \leqslant n$, then $a_{i}+a_{j} \leqslant n$ and $a_{a_{i}+a_{j}}=a_{i+j}$. + +Let $f(n)$ be the number of $n$-sequences. Prove that there exist positive real numbers $c_{1}, c_{2}$ and $\lambda$ such that + +$$ +c_{1} \lambda^{n}k$ for some $i$, and small if no such $i$ exists. For now we will assume that $\left(a_{i}\right)$ is not the identity sequence (in other words, that $a_{i} \neq i$ for some $i$ ). +Lemma 1. If $a_{r}=a_{s}$ and $r, sr$, and let $d$ be the minimum positive integer such that $a_{r+d}=a_{r}$. Then + +1. The subsequence $\left(a_{r}, a_{r+1}, \ldots, a_{n}\right)$ is periodic with minimal period $d$. That is, for $uv_{0}$. Thus $u_{0}=v_{0}$, so $d \mid u-v$. +2. If $r=0$ there is nothing to prove. Otherwise $a_{0}=a_{2 a_{0}}$ so $2 a_{0}=0$. Then we have $a_{a_{i}}=a_{i}$ for all $i$, so $a_{i}=i$ for $ik+1$. We show that $a_{i} \leqslant k$ for all $i$ by induction. Note that Lemma 2 already establishes this for $i \leqslant k$. We must have $d \mid a_{d / 2}$ and $a_{d / 2} \leqslant kk$, if $a_{j} \leqslant k$ for $jk+1$ and $a_{i}=i$ for all $0 \leqslant ik$. + +We already have $a_{i}=i$ for $id$, this means that $a_{i}=i$ for $i \leqslant k$. + +Finally, one can show inductively that $a_{i}=i$ for $kc_{1} \lambda^{n}$ for some $c_{1}$, we note that + +$$ +f(n)>g\left(k+1,\left\lfloor\frac{k+1}{3}\right\rfloor\right) \geqslant 3^{\lfloor(k+1) / 3\rfloor}>3^{n / 6}-1 . +$$ + +To show that $f(n)2 \cdot 3^{-1 / 6} \approx 1.66537$. + +With a careful analysis one can show that the best possible value of $c_{2}$ is $\frac{236567}{4930} 3^{1 / 3} \approx$ 69.20662 . + +## Combinatorics + +C1. A $\pm 1$-sequence is a sequence of 2022 numbers $a_{1}, \ldots, a_{2022}$, each equal to either +1 or -1 . Determine the largest $C$ so that, for any $\pm 1$-sequence, there exists an integer $k$ and indices $1 \leqslant t_{1}<\ldots\frac{3 n+1}{2}$, let $a=k-n-1, b=2 n-k+1$. Then $k>2 a+b, k>2 b+a$, so the configuration $A^{a} C^{b} A^{b} C^{a}$ will always have four blocks: + +$$ +A^{a} C^{b} A^{b} C^{a} \rightarrow C^{a} A^{a} C^{b} A^{b} \rightarrow A^{b} C^{a} A^{a} C^{b} \rightarrow C^{b} A^{b} C^{a} A^{a} \rightarrow A^{a} C^{b} A^{b} C^{a} \rightarrow \ldots +$$ + +Therefore a pair $(n, k)$ can have the desired property only if $n \leqslant k \leqslant \frac{3 n+1}{2}$. We claim that all such pairs in fact do have the desired property. Clearly, the number of blocks in a configuration cannot increase, so whenever the operation is applied, it either decreases or remains constant. We show that unless there are only two blocks, after a finite amount of steps the number of blocks will decrease. + +Consider an arbitrary configuration with $c \geqslant 3$ blocks. We note that as $k \geqslant n$, the leftmost block cannot be moved, because in this case all $n$ coins of one type are in the leftmost block, meaning there are only two blocks. If a block which is not the leftmost or rightmost block is moved, its neighbor blocks will be merged, causing the number of blocks to decrease. + +Hence the only case in which the number of blocks does not decrease in the next step is if the rightmost block is moved. If $c$ is odd, the leftmost and the rightmost blocks are made of the same metal, so this would merge two blocks. Hence $c \geqslant 4$ must be even. Suppose there is a configuration of $c$ blocks with the $i$-th block having size $a_{i}$ so that the operation always moves the rightmost block: + +$$ +A^{a_{1}} \ldots A^{a_{c-1}} C^{a_{c}} \rightarrow C^{a_{c}} A^{a_{1}} \ldots A^{a_{c-1}} \rightarrow A^{a_{c-1}} C^{a_{c}} A^{a_{1}} \ldots C^{a_{c-2}} \rightarrow \ldots +$$ + +Because the rightmost block is always moved, $k \geqslant 2 n+1-a_{i}$ for all $i$. Because $\sum a_{i}=2 n$, summing this over all $i$ we get $c k \geqslant 2 c n+c-\sum a_{i}=2 c n+c-2 n$, so $k \geqslant 2 n+1-\frac{2 n}{c} \geqslant \frac{3 n}{2}+1$. But this contradicts $k \leqslant \frac{3 n+1}{2}$. Hence at some point the operation will not move the rightmost block, meaning that the number of blocks will decrease, as desired. + +C3. In each square of a garden shaped like a $2022 \times 2022$ board, there is initially a tree of height 0 . A gardener and a lumberjack alternate turns playing the following game, with the gardener taking the first turn: + +- The gardener chooses a square in the garden. Each tree on that square and all the surrounding squares (of which there are at most eight) then becomes one unit taller. +- The lumberjack then chooses four different squares on the board. Each tree of positive height on those squares then becomes one unit shorter. + +We say that a tree is majestic if its height is at least $10^{6}$. Determine the largest number $K$ such that the gardener can ensure there are eventually $K$ majestic trees on the board, no matter how the lumberjack plays. +(Colombia) +Answer: $K=5 \cdot \frac{2022^{2}}{9}=2271380$. In general, for a $3 N \times 3 N$ board, $K=5 N^{2}$. +Solution. We solve the problem for a general $3 N \times 3 N$ board. First, we prove that the lumberjack has a strategy to ensure there are never more than $5 N^{2}$ majestic trees. Giving the squares of the board coordinates in the natural manner, colour each square where at least one of its coordinates are divisible by 3 , shown below for a $9 \times 9$ board: +![](https://cdn.mathpix.com/cropped/2024_11_18_86efb4389f14b52b08e1g-28.jpg?height=900&width=914&top_left_y=1343&top_left_x=571) + +Then, as each $3 \times 3$ square on the board contains exactly 5 coloured squares, each move of the gardener will cause at most 4 trees on non-coloured squares to grow. The lumberjack may therefore cut those trees, ensuring no tree on a non-coloured square has positive height after his turn. Hence there cannot ever be more majestic trees than coloured squares, which is $5 N^{2}$. + +Next, we prove the gardener may ensure there are $5 N^{2}$ majestic trees. In fact, we prove this statement in a modified game which is more difficult for the gardener: on the lumberjack's turn in the modified game, he may decrement the height of all trees on the board except those the gardener did not just grow, in addition to four of the trees the gardener just grew. Clearly, a sequence of moves for the gardener which ensures that there are $K$ majestic trees in the modified game also ensures this in the original game. + +Let $M=\binom{9}{5}$; we say that a map is one of the $M$ possible ways to mark 5 squares on a $3 \times 3$ board. In the modified game, after the gardener chooses a $3 \times 3$ subboard on the board, the lumberjack chooses a map in this subboard, and the total result of the two moves is that each tree marked on the map increases its height by 1 , each tree in the subboard which is not in the map remains unchanged, and each tree outside the subboard decreases its height by 1. Also note that if the gardener chooses a $3 \times 3$ subboard $M l$ times, the lumberjack will have to choose some map at least $l$ times, so there will be at least 5 trees which each have height $\geqslant l$. + +The strategy for the gardener will be to divide the board into $N^{2}$ disjoint $3 \times 3$ subboards, number them $0, \ldots, N^{2}-1$ in some order. Then, for $b=N^{2}-1, \ldots, 0$ in order, he plays $10^{6} M(M+1)^{b}$ times on subboard number $b$. Hence, on subboard number $b$, the moves on that subboard will first ensure 5 of its trees grows by at least $10^{6}(M+1)^{b}$, and then each move after that will decrease their heights by 1 . (As the trees on subboard $b$ had height 0 before the gardener started playing there, no move made on subboards $\geqslant b$ decreased their heights.) As the gardener makes $10^{6} M(M+1)^{b-1}+\ldots=10^{6}\left((M+1)^{b}-1\right)$ moves after he finishes playing on subboard $b$, this means that on subboard $b$, there will be 5 trees of height at least $10^{6}(M+1)^{b}-10^{6}\left((M+1)^{b}-1\right)=10^{6}$, hence each of the subboard has 5 majestic trees, which was what we wanted. + +## C4. + +Let $n>3$ be a positive integer. Suppose that $n$ children are arranged in a circle, and $n$ coins are distributed between them (some children may have no coins). At every step, a child with at least 2 coins may give 1 coin to each of their immediate neighbours on the right and left. Determine all initial distributions of coins from which it is possible that, after a finite number of steps, each child has exactly one coin. +(Israel) +Answer: All distributions where $\sum_{i=1}^{n} i c_{i}=\frac{n(n+1)}{2}(\bmod n)$, where $c_{i}$ denotes the number of coins the $i$-th child starts with. + +## Solution 1. + +Number the children $1, \ldots, n$, and denote the number of coins the $i$-th child has by $c_{i}$. A step of this process consists of reducing some $c_{i}$ by 2 , and increasing $c_{i-1}, c_{i+1}$ by 1 . (Indices are considered $(\bmod n)$.) Because $(i-1)-2 i+(i+1)=0$, the quantity $\sum_{i=1}^{n} i c_{i}(\bmod n)$ will be invariant under this process. Hence a necessary condition for the children to end up with an uniform distribution of coins is that + +$$ +\sum_{i=1}^{n} i c_{i}=\frac{n(n+1)}{2} \quad(\bmod n) +$$ + +We will show that this condition is also sufficient. Consider an arbitrary initial distribution of coins. First, whenever child $i$ has more than one coin and $i \neq n$, have child $i$ pass coins to its neighbors. (Child $i$ does nothing.) Then, after some amount of such steps, it must eventually become impossible to do any more steps because no child except perhaps child $i$ has more than 1 coin. (To see this, consider e.g. the quantity $\sum_{i=1}^{n-1} i^{2} c_{i}$, which (as $(i-1)^{2}+(i+1)^{2}>2 i^{2}$ ) increases at each step.) + +Hence we can reach a state of the form $\left(z_{1}, \ldots, z_{n-1}, M\right)$, where $z_{i}=0$ or 1 . Call such states semi-uniform states of irregularity $M$. +Lemma. If there is a string of children having coins $a, \overbrace{1, \ldots, 1}^{k \text { ones }}, b, \overbrace{1, \ldots, 1}^{k \text { ones }}, c$, with $b \geqslant 2$, after some sequence of steps we may reach the state $a+1, \overbrace{1, \ldots, 1}^{k \text { ones }}, b-2, \overbrace{1, \ldots, 1}^{k \text { ones }}, c+1$. We call performing this sequence of steps long-passing coins. +Proof. This is simply repeated application of the operation. We prove the lemma by induction on $k$. For $k=0$, this is just the operation of the problem. If $k=1$, have the child with $b$ coins pass coins, then both of their neighbors pass coins, then the child with $b$ coins pass coins again. For $k \geqslant 2$, first, have the child with $b$ coins pass coins, then have both their neigbors send coins, giving the state + +$$ +a, \overbrace{1, \ldots, 1}^{k-2 \text { ones }}, 2,0, b, 0,2, \overbrace{1, \ldots, 1}^{k-2 \text { ones }}, c . +$$ + +Now set aside the children with $a, b$ and $c$ coins, and have each child with 2 coins give them to their neighbors until there are no such children remaining. This results in the state + +$$ +a+1,0, \overbrace{1, \ldots, 1}^{k-2}, b, \overbrace{1, \ldots, 1}^{k-2 \text { ones }}, 0, c+1 +$$ + +By the induction hypothesis, we can have the child with $b$ coins may pass a coin to each of the children with 0 coins, proving the lemma. + +Claim. We can reach a semi-uniform state of irregularity $M \leqslant 2$. +Proof. If $M>3$, because there are only $n$ coins in total, there must be at least two children with 0 coins. Consider the arc of the circle spanned by the two such children closest to the child with $M$ coins. It has the form + +$$ +0, \overbrace{1, \ldots, 1}^{a \text { ones }}, M, \overbrace{1, \ldots, 1}^{b \text { ones }}, 0 +$$ + +If $a=b$, applying the previous lemma we can have the child with $M$ coins long-pass a coin to each of the children with 0 coins, which yields a semi-uniform state with lower $M$. Otherwise, WLOG $a>b$, so we can have the child with $M$ coins long-pass a coin to each of the children at distance $b$ from it, reaching a state of the form $(\alpha:=a-b-1, \beta:=b)$ + +$$ +0, \overbrace{1, \ldots, 1}^{\alpha \text { ones }}, 2, \overbrace{1, \ldots, 1}^{\beta \text { ones }}, M-2, \overbrace{1, \ldots, 1}^{c \text { ones }} +$$ + +The children in the rightmost string of ones need make no further moves, so consider only the leftmost string. If $\alpha<\beta$, have the child with 2 coins long-pass coins to the child with 0 coins to its left and some child with 1 coin to its right, reaching a new state of the form $0, \overbrace{1, \ldots, 1}^{\alpha \text { ones }}, 2, \overbrace{1, \ldots, 1}^{\beta \text { ones }}, M-2$ with a smaller $\beta$. As $\beta$ cannot decrease indefinitely, eventually $\alpha \geqslant \beta$. If $\alpha=\beta$, have the child with 2 coins long-pass to the child with $M$ coins and the child with 0 coins, reaching a semi-uniform state of irregularity $M-1$ as desired. Otherwise, $\alpha<\beta$, so have the child with 2 coins long-pass to the child with $M$ coins and a child with 1 coin, reaching a state of the form + +$$ +0, \overbrace{1, \ldots, 1}^{x \text { ones }}, 2, \overbrace{1, \ldots, 1}^{y \text { ones }}, 0, \overbrace{1, \ldots, 1}^{z \text { ones }}, M-1 +$$ + +Now, consider only the substring between the two children with 0 coins, which has the form $0, \overbrace{1, \ldots, 1}^{x \text { ones }}, 2, \overbrace{1, \ldots, 1}^{y \text { ones }}, 0$. Repeatedly have the child in this substring with 2 coins long-pass to the closest child with 0 coins and some other child. If the other child has 1 coin, we have a new strictly shorter substring of the form $0, \overbrace{1, \ldots, 1}^{x \text { ones }}, 2, \overbrace{1, \ldots, 1}^{y \text { ones }}, 0$. Hence eventually it must happen that the other child also has 0 coins, at which point we reach a semi-uniform state of irregularity $M-1$, proving the claim. + +We have now shown that we can reach a semi-regular state of irregularity $M \leqslant 2$, If $M=1$, each child must have one coin, as desired. Otherwise, we must have $M=2$, so there is one child with 0 coins, one child with 2 coins, and the remaining children all have 1 coin. Recall that the state we started with satisfied the invariant + +$$ +\sum_{i=1}^{n} i c_{i}=\frac{n(n+1)}{2} \quad(\bmod n) +$$ + +Because each step preserves this invariant, this must also be true of the current state. Number the children so that the child with $M$ coins is child number $n$, and suppose the child with 0 coins is child $k$. Then $\frac{n(n+1)}{2}=\sum_{i=1}^{n} i c_{i}=\left(\sum_{i=1}^{n} 1 \cdot c_{i}\right)-k=\frac{n(n+1)}{2}-k(\bmod n)$, so $k=0$ $(\bmod n)$. But this is impossible, as no child except the child with $M$ coins has an index divisible by $n$. Hence we cannot end up in a semi-regular state of irregularity 2 , so we are done. + +Solution 2. Encode the sequence $c_{i}$ as a polynomial $p(x)=\sum_{i} a_{i} x_{i}$. The cyclic nature of the problem makes it natural to work modulo $x^{n}-1$. Child $i$ performing a step is equivalent to adding $x^{i}(x-1)^{2}$ to the polynomial, and we want to reach the polynomial $q(x)=1+x+\ldots+$ $x^{n-1}$. Since we only add multiples of $(x-1)^{2}$, this is only possible if $p(x)=q(x)$ modulo the ideal generated by $x^{n}-1$ and $(x-1)^{2}$, i.e. + +$$ +\left(x^{n}-1,(x-1)^{2}\right)=(x-1)\left(\frac{x^{n}-1}{x-1}, x-1\right)=(x-1) \cdot(n,(x-1)) +$$ + +This is equivalent to $p(1)=q(1)$ (which simply translates to the condition that there are $n$ coins) and $p^{\prime}(1)=q^{\prime}(1)(\bmod n)$, which translates to the invariant described in solution 1. + +C5. Let $m, n \geqslant 2$ be integers, let $X$ be a set with $n$ elements, and let $X_{1}, X_{2}, \ldots, X_{m}$ be pairwise distinct non-empty, not necessary disjoint subsets of $X$. A function $f: X \rightarrow$ $\{1,2, \ldots, n+1\}$ is called nice if there exists an index $k$ such that + +$$ +\sum_{x \in X_{k}} f(x)>\sum_{x \in X_{i}} f(x) \text { for all } i \neq k +$$ + +Prove that the number of nice functions is at least $n^{n}$. +(Germany) +Solution. For a subset $Y \subseteq X$, we write $f(Y)$ for $\sum_{y \in Y} f(y)$. Note that a function $f: X \rightarrow$ $\{1, \ldots, n+1\}$ is nice, if and only if $f\left(X_{i}\right)$ is maximized by a unique index $i \in\{1, \ldots, m\}$. + +We will first investigate the set $\mathcal{F}$ of functions $f: X \rightarrow\{1, \ldots, n\}$; note that $|\mathcal{F}|=n^{n}$. For every function $f \in \mathcal{F}$, define a corresponding function $f^{+}: X \rightarrow\{1,2, \ldots, n+1\}$ in the following way: Pick some set $X_{l}$ that maximizes the value $f\left(X_{l}\right)$. + +- For all $x \in X_{l}$, define $f^{+}(x)=f(x)+1$. +- For all $x \in X \backslash X_{l}$, define $f^{+}(x)=f(x)$. + +Claim. The resulting function $f^{+}$is nice. +Proof. Note that $f^{+}\left(X_{i}\right)=f\left(X_{i}\right)+\left|X_{i} \cap X_{l}\right|$ holds for all $X_{i}$. We show that $f^{+}\left(X_{i}\right)$ is maximized at the unique index $i=l$. Hence consider some arbitrary index $j \neq l$. Then $X_{l} \subset X_{j}$ is impossible, as this would imply $f\left(X_{j}\right)>f\left(X_{l}\right)$ and thereby contradict the choice of set $X_{l}$; this in particular yields $\left|X_{l}\right|>\left|X_{j} \cap X_{l}\right|$. + +$$ +f^{+}\left(X_{l}\right)=f\left(X_{l}\right)+\left|X_{l}\right| \geqslant f\left(X_{j}\right)+\left|X_{l}\right|>f\left(X_{j}\right)+\left|X_{j} \cap X_{l}\right|=f^{+}\left(X_{j}\right) +$$ + +The first inequality follows since $X_{l}$ was chosen to maximize the value $f\left(X_{l}\right)$. The second (strict) inequality follows from $\left|X_{l}\right|>\left|X_{j} \cap X_{l}\right|$ as observed above. This completes the proof of the claim. + +Next observe that function $f$ can be uniquely reconstructed from $f^{+}$: the claim yields that $f^{+}$has a unique maximizer $X_{l}$, and by decreasing the value of $f^{+}$on $X_{l}$ by 1 , we get we can fully determine the values of $f$. As each of the $n^{n}$ functions $f \in \mathcal{F}$ yields a (unique) corresponding nice function $f^{+}: X \rightarrow\{1,2, \ldots, n+1\}$, the proof is complete. + +C6. Let $n$ be a positive integer. We start with $n$ piles of pebbles, each initially containing a single pebble. One can perform moves of the following form: choose two piles, take an equal number of pebbles from each pile and form a new pile out of these pebbles. For each positive integer $n$, find the smallest number of non-empty piles that one can obtain by performing a finite sequence of moves of this form. +(Croatia) +Answer: 1 if $n$ is a power of two, and 2 otherwise. +Solution 1. The solution we describe is simple, but not the most effective one. +We can combine two piles of $2^{k-1}$ pebbles to make one pile of $2^{k}$ pebbles. In particular, given $2^{k}$ piles of one pebble, we may combine them as follows: + +$$ +\begin{array}{lcc} +2^{k} \text { piles of } 1 \text { pebble } & \rightarrow & 2^{k-1} \text { piles of } 2 \text { pebbles } \\ +2^{k-1} \text { piles of } 2 \text { pebbles } & \rightarrow & 2^{k-2} \text { piles of } 4 \text { pebbles } \\ +2^{k-2} \text { piles of } 4 \text { pebbles } & \rightarrow & 2^{k-3} \text { piles of } 8 \text { pebbles } \\ +& \vdots \\ +2 \text { piles of } 2^{k-1} \text { pebbles } & \rightarrow 1 \text { pile of } 2^{k} \text { pebbles } +\end{array} +$$ + +This proves the desired result in the case when $n$ is a power of 2 . +If $n$ is not a power of 2 , choose $N$ such that $2^{N}1$. In order to make a single pile of $n$ pebbles, we would have to start with a distribution in which the number of pebbles in each pile is divisible by the integer $m$. This is impossible when we start with all piles containing a single pebble. + +Remarks on starting configurations From any starting configuration that is not a single pile, if there are at least two piles with at least two pebbles, we can remove one pebble from two such piles, and form a new pile with 2 pebbles. We can repeat this until we have one pile of 2 pebbles and the rest are single pebble piles, and then proceed as in the solution. Hence, if $n$ is a power of two, we can make a single pile from any starting configuration. + +If $n$ is of the form $n=2^{k} m$ where $m>1$ is odd, then we can make a single pile from any starting configuration in which the number of pebbles in each pile is divisible by the integer $m$, otherwise two piles is the best we can do. Half of this is proven already. For the other half, assume we start with a configuration in which the number of pebbles in each pile is divisible by the integer $m$. Replace each pile of $t m$ pebbles with a pile of $t$ boulders. We now have a total of $2^{k}$ boulders, hence we can make them into one pile of $2^{k}$ boulders. Replacing the boulders with pebbles again, we are done. + +Solution 2. We show an alternative strategy if $n$ is not a power of 2 . Write $n$ in binary form: $n=2^{i_{1}}+2^{i_{2}}+\cdots+2^{i_{k}}$, where $i_{1}>i_{2}>\cdots>i_{k}$. Now we make piles of sizes $2^{i_{1}}, 2^{i_{2}}, \ldots, 2^{i_{k}}$. We call the pile with $2^{i_{1}}$ the large pile, all the others are the small piles. + +The strategy is the following: take the two smallest small pile. If they have the same size of $2^{a}$, we make a pile of size $2^{a+1}$. If they have different sizes, we double the smaller pile using the large pile (we allow the large pile to have a negative number of pebbles: later we prove that it is not possible). We call all the new piles small. When we have only one small pile, we terminate the process: we have at most 2 piles. + +After each move we have one less number of piles, and all the piles have cardinality power of 2. The number of pebbles is decreasing, and at the end of the process, it has a size of $n-2^{i_{2}+1} \geqslant n-2^{i_{1}}>0$, thus we can manage to have two piles. + +Solution 3. Throughout the solution, we will consider the moves in reverse order. Namely, imagine we have some piles of pebbles, and we are allowed to perform moves as follows: take a pile with an even number of pebbles, split it into two equal halves and add the pebbles from each half to a different pile, possibly forming new piles (we may assume for convenience that there are infinitely many empty piles at any given moment). Given a configuration of piles $\mathcal{C}$, we will use $|\mathcal{C}|$ to denote the number of non-empty piles in $\mathcal{C}$. Given two configurations $\mathcal{C}_{1}, \mathcal{C}_{2}$, we will say that $\mathcal{C}_{2}$ is reachable from $\mathcal{C}_{1}$ if $\mathcal{C}_{2}$ can be obtained by performing a finite sequence of moves starting from $\mathcal{C}_{1}$. Call a configuration of piles $\mathcal{C}$ + +- simple if each (non-empty) pile in $\mathcal{C}$ consists of a single pebble; +- good if at least one (non-empty) pile in $\mathcal{C}$ has an even number of pebbles and the numbers of pebbles on the piles in $\mathcal{C}$ have no non-trivial odd common divisor ( $\mathcal{C}$ has the odd divisor property); +- solvable if there exists a simple configuration which is reachable from $\mathcal{C}$. + +The problem asks to find the smallest number of non-empty piles in a solvable configuration consisting of $n$ pebbles. We begin the process of answering this question by making the following observation: +Lemma 1. Let $\mathcal{C}$ be a configuration of piles. Let $\mathcal{C}^{\prime}$ be a configuration obtained by applying a single move to $\mathcal{C}$. Then +(i) if $\mathcal{C}^{\prime}$ has the odd divisor property, then so does $\mathcal{C}$; +(ii) the converse to (i) holds if $\left|\mathcal{C}^{\prime}\right| \geqslant|\mathcal{C}|$. + +Proof. Suppose that the move consists of splitting a pile of size $2 a$ and adding $a$ pebbles to each of two piles of sizes $b$ and $c$. Here, $a$ is a positive integer and $b, c$ are non-negative integers. Thus, $\mathcal{C}^{\prime}$ can be obtained from $\mathcal{C}$ by replacing the piles of sizes $2 a, b, c$ by two piles of sizes $a+b$ and $a+c$. Note that the extra assumption $\left|\mathcal{C}^{\prime}\right| \geqslant|\mathcal{C}|$ of part (ii) holds if and only if at least one of $b, c$ is zero. +(i) Suppose $\mathcal{C}$ doesn't have the odd divisor property, i.e. there exists an odd integer $d>1$ such that the size of each pile in $\mathcal{C}$ is divisible by $d$. In particular, $2 a, b, c$ are multiples of $d$, so since $d$ is odd, it follows that $a, b, c$ are all divisible by $d$. Thus, $a+b$ and $a+c$ are also divisible by $d$, so $d$ divides the size of each pile in $\mathcal{C}^{\prime}$. In conclusion, $\mathcal{C}^{\prime}$ doesn't have the odd divisor property. +(ii) If $\mathcal{C}^{\prime}$ doesn't have the odd divisor property and at least one of $b, c$ is zero, then there exists an odd integer $d>1$ such that the size of each pile in $\mathcal{C}^{\prime}$ is divisible by $d$. In particular, $d$ divides $a+b$ and $a+c$, so since at least one of these numbers is equal to $a$, it follows that $d$ divides $a$. But then $d$ must divide all three of $a, b$ and $c$, and hence it certainly divides $2 a, b$ and $c$. Thus, $\mathcal{C}$ doesn't have the odd divisor property, as desired. +Lemma 2. If $\mathcal{C}_{2}$ is reachable from $\mathcal{C}_{1}$ and $\mathcal{C}_{2}$ has the odd divisor property, then so does $\mathcal{C}_{1}$. In particular, any solvable configuration has the odd divisor property. +Proof. The first statement follows by inductively applying part (i) of Lemma 1. The second statement follows from the first because every simple configuration has the odd divisor property. + +The main claim is the following: +Lemma 3. Let $\mathcal{C}$ be a good configuration. Then there exists a configuration $\mathcal{C}^{\prime}$ with the following properties: + +- $\mathcal{C}^{\prime}$ is reachable from $\mathcal{C}$ and $\left|\mathcal{C}^{\prime}\right|>|\mathcal{C}|$; +- $\mathcal{C}^{\prime}$ is either simple or good. + +Proof. Call a configuration terminal if it is a counterexample to the claim. The following claim is at the heart of the proof: +Claim. Let $a_{1}, \ldots, a_{k}$ be the numbers of pebbles on the non-empty piles of a terminal configuration $\mathcal{C}$. Then there exists a unique $i \in[k]$ such that $a_{i}$ is even. Moreover, for all $t \geqslant 1$ we have $a_{j} \equiv \frac{a_{i}}{2}\left(\bmod 2^{t}\right)$ for all $j \neq i$. +Proof of Claim. Since the configuration is good, there must exist $i \in[k]$ such that $a_{i}$ is even. Moreover, by assumption, if we split the pile with $a_{i}$ pebbles into two equal halves, the resulting configuration will not be good. By part (ii) of Lemma 2,the only way this can happen is that $\frac{a_{i}}{2}$ and $a_{j}$ for all $j \neq i$ are odd. To prove the second assertion, we proceed by induction on $t$, with the case $t=1$ already being established. If $t \geqslant 2$, then split the pile with $a_{i}$ pebbles into two equal halves and move one half to the pile with $a_{j}$ pebbles. Since $\frac{a_{i}}{2}$ and $a_{j}$ are both odd, $a_{j}+\frac{a_{i}}{2}$ is even, so by part (ii) of Lemma 2 , the resulting configuration $\mathcal{C}^{\prime}$ is good. Thus, $\mathcal{C}^{\prime}$ is terminal, so by the induction hypothesis, we have $\frac{a_{i}}{2} \equiv \frac{1}{2}\left(a_{j}+\frac{a_{i}}{2}\right)\left(\bmod 2^{t-1}\right)$, whence $a_{j} \equiv \frac{a_{i}}{2}($ $\bmod 2^{t}$ ), as desired. + +Suppose for contradiction that there exists a configuration as in the Claim. It follows that there exists $i \in[k]$ and an odd integer $x$ such that $a_{i}=2 x$ and $a_{j}=x$ for all $j \neq i$. Thus, $x$ is an odd common divisor of $a_{1}, \ldots, a_{k}$, so by the odd divisor property, we must have $x=1$. But then we can obtain a simple configuration by splitting the only pile with two pebbles into two piles each consisting of a single pebble, which is a contradiction. + +With the aid of Lemmas 2 and 3, it is not hard to characterise all solvable configurations: Lemma 4. A configuration of piles $\mathcal{C}$ is solvable if and only if it is simple or good. + +Proof. $(\Longrightarrow)$ Suppose $\mathcal{C}$ is not simple. Then since we have to be able to perform at least one move, there must be at least one non-empty pile in $\mathcal{C}$ with an even number of pebbles. Moreover, by Lemma $2, \mathcal{C}$ has the odd divisor property, so it must be good. +$(\Longleftarrow)$ This follows by repeatedly applying Lemma 3 until we reach a simple configuration. Note that the process must stop eventually since the number of non-empty piles is bounded from above. + +Finally, the answer to the problem is implied by the following corollary of Lemma 4: +Lemma 5. Let $n$ be a positive integer. Then +(i) a configuration consisting of a single pile of $n$ pebbles is solvable if and only if $n$ is a power of two; +(ii) if $n \geqslant 2$, then the configuration consisting of piles of sizes 2 and $n-2$ is good. + +Proof. (i) By Lemma 4, this configuration is solvable if and only if either $n=1$ or $n$ is even and has no non-trivial odd divisor, so the conclusion follows. +(ii) Since 2 is even and has no non-trivial odd divisor, this configuration is certainly good, so the conclusion follows by Lemma 4. + +Common remarks. Instead of choosing pebbles from two piles, one could allow choosing an equal number of pebbles from each of $k$ piles, where $k \geqslant 2$ is a fixed (prime) integer. However, this seems to yield a much harder problem - if $k=3$, numerical evidence suggests the same answer as in the case $k=2$ (with powers of two replaced by powers of three), but the case $k=5$ is already unclear. + +C7. Lucy starts by writing $s$ integer-valued 2022-tuples on a blackboard. After doing that, she can take any two (not necessarily distinct) tuples $\mathbf{v}=\left(v_{1}, \ldots, v_{2022}\right)$ and $\mathbf{w}=\left(w_{1}, \ldots, w_{2022}\right)$ that she has already written, and apply one of the following operations to obtain a new tuple: + +$$ +\begin{aligned} +& \mathbf{v}+\mathbf{w}=\left(v_{1}+w_{1}, \ldots, v_{2022}+w_{2022}\right) \\ +& \mathbf{v} \vee \mathbf{w}=\left(\max \left(v_{1}, w_{1}\right), \ldots, \max \left(v_{2022}, w_{2022}\right)\right) +\end{aligned} +$$ + +and then write this tuple on the blackboard. +It turns out that, in this way, Lucy can write any integer-valued 2022-tuple on the blackboard after finitely many steps. What is the smallest possible number $s$ of tuples that she initially wrote? +(Czech Republic) +Answer: The smallest possible number is $s=3$. + +Solution. We solve the problem for $n$-tuples for any $n \geqslant 3$ : we will show that the answer is $s=3$, regardless of the value of $n$. + +First, let us briefly introduce some notation. For an $n$-tuple $\mathbf{v}$, we will write $\mathbf{v}_{i}$ for its $i$-th coordinate (where $1 \leqslant i \leqslant n$ ). For a positive integer $n$ and a tuple $\mathbf{v}$ we will denote by $n \cdot \mathbf{v}$ the tuple obtained by applying addition on $\mathbf{v}$ with itself $n$ times. Furthermore, we denote by $\mathbf{e}(i)$ the tuple which has $i$-th coordinate equal to one and all the other coordinates equal to zero. We say that a tuple is positive if all its coordinates are positive, and negative if all its coordinates are negative. + +We will show that three tuples suffice, and then that two tuples do not suffice. + +Three tuples suffice. Write $\mathbf{c}$ for the constant-valued tuple $\mathbf{c}=(-1, \ldots,-1)$. +We note that it is enough for Lucy to be able to make the tuples $\mathbf{e}(1), \ldots, \mathbf{e}(n), \mathbf{c}$; from those any other tuple $\mathbf{v}$ can be made as follows. First we choose some positive integer $k$ such that $k+\mathbf{v}_{i}>0$ for all $i$. Then, by adding a positive number of copies of $\mathbf{c}, \mathbf{e}(1), \ldots, \mathbf{e}(n)$, she can make + +$$ +k \mathbf{c}+\left(k+\mathbf{v}_{1}\right) \cdot \mathbf{e}(1)+\cdots+\left(k+\mathbf{v}_{n}\right) \cdot \mathbf{e}(n) +$$ + +which we claim is equal to $\mathbf{v}$. Indeed, this can be checked by comparing coordinates: the $i$-th coordinate of the right-hand side is $-k+\left(k+\mathbf{v}_{i}\right)=\mathbf{v}_{i}$ as needed. + +Lucy can take her three starting tuples to be $\mathbf{a}, \mathbf{b}$ and $\mathbf{c}$, such that $\mathbf{a}_{i}=-i^{2}, \mathbf{b}_{i}=i$ and $\mathbf{c}=-1$ (as above). + +For any $1 \leqslant j \leqslant n$, write $\mathbf{d}(j)$ for the tuple $2 \cdot \mathbf{a}+4 j \cdot \mathbf{b}+\left(2 j^{2}-1\right) \cdot \mathbf{c}$, which Lucy can make by adding together $\mathbf{a}, \mathbf{b}$ and $\mathbf{c}$ repeatedly. This has $i$ th term + +$$ +\begin{aligned} +\mathbf{d}(j)_{i} & =2 \mathbf{a}_{i}+4 j \mathbf{b}_{i}+\left(2 j^{2}-1\right) \mathbf{c}_{i} \\ +& =-2 i^{2}+4 i j-\left(2 j^{2}-1\right) \\ +& =1-2(i-j)^{2} +\end{aligned} +$$ + +This is 1 if $j=i$, and at most -1 otherwise. Hence Lucy can produce the tuple $\mathbf{1}=(1, \ldots, 1)$ as $\mathbf{d}(1) \vee \cdots \vee \mathbf{d}(n)$. + +She can then produce the constant tuple $\mathbf{0}=(0, \ldots, 0)$ as $\mathbf{1}+\mathbf{c}$, and for any $1 \leqslant j \leqslant n$ she can then produce the tuple $\mathbf{e}(j)$ as $\mathbf{d}(j) \vee \mathbf{0}$. Since she can now produce $\mathbf{e}(1), \ldots, \mathbf{e}(n)$ and already had c, she can (as we argued earlier) produce any integer-valued tuple. + +Two tuples do not suffice. We start with an observation: Let $a$ be a non-negative real number and suppose that two tuples $\mathbf{v}$ and $\mathbf{w}$ satisfy $\mathbf{v}_{j} \geqslant a \mathbf{v}_{k}$ and $\mathbf{w}_{j} \geqslant a \mathbf{w}_{k}$ for some $1 \leqslant j, k \leqslant n$. Then we claim that the same inequality holds for $\mathbf{v}+\mathbf{w}$ and $\mathbf{v} \vee \mathbf{w}$ : Indeed, the property for the sum is verified by an easy computation: + +$$ +(\mathbf{v}+\mathbf{w})_{j}=\mathbf{v}_{j}+\mathbf{w}_{j} \geqslant a \mathbf{v}_{k}+a \mathbf{w}_{k}=a(\mathbf{v}+\mathbf{w})_{k} +$$ + +For the second operation, we denote by $\mathbf{m}$ the tuple $\mathbf{v} \vee \mathbf{w}$. Then $\mathbf{m}_{j} \geqslant \mathbf{v}_{j} \geqslant a \mathbf{v}_{k}$ and $\mathbf{m}_{j} \geqslant \mathbf{w}_{j} \geqslant a \mathbf{w}_{k}$. Since $\mathbf{m}_{k}=\mathbf{v}_{k}$ or $\mathbf{m}_{k}=\mathbf{w}_{k}$, the observation follows. + +As a consequence of this observation we have that if all starting tuples satisfy such an inequality, then all generated tuples will also satisfy it, and so we would not be able to obtain every integer-valued tuple. + +Let us now prove that Lucy needs at least three starting tuples. For contradiction, let us suppose that Lucy started with only two tuples $\mathbf{v}$ and $\mathbf{w}$. We are going to distinguish two cases. In the first case, suppose we can find a coordinate $i$ such that $\mathbf{v}_{i}, \mathbf{w}_{i} \geqslant 0$. Both operations preserve the sign, thus we can not generate any tuple that has negative $i$-th coordinate. Similarly for $\mathbf{v}_{i}, \mathbf{w}_{i} \leqslant 0$. + +Suppose the opposite, i.e., for every $i$ we have either $\mathbf{v}_{i}>0>\mathbf{w}_{i}$, or $\mathbf{v}_{i}<0<\mathbf{w}_{i}$. Since we assumed that our tuples have at least three coordinates, by pigeonhole principle there exist two coordinates $j \neq k$ such that $\mathbf{v}_{j}$ has the same sign as $\mathbf{v}_{k}$ and $\mathbf{w}_{j}$ has the same sign as $\mathbf{w}_{k}$ (because there are only two possible combinations of signs). + +Without loss of generality assume that $\mathbf{v}_{j}, \mathbf{v}_{k}>0$ and $\mathbf{w}_{j}, \mathbf{w}_{k}<0$. Let us denote the positive real number $\mathbf{v}_{j} / \mathbf{v}_{k}$ by $a$. If $\mathbf{w}_{j} / \mathbf{w}_{k} \leqslant a$, then both inequalities $\mathbf{v}_{j} \geqslant a \mathbf{v}_{k}$ and $\mathbf{w}_{j} \geqslant a \mathbf{w}_{k}$ are satisfied. On the other hand, if $\mathbf{w}_{j} / \mathbf{w}_{k} \leqslant a$, then both $\mathbf{v}_{k} \geqslant(1 / a) \mathbf{v}_{j}$ and $\mathbf{w}_{k} \geqslant(1 / a) \mathbf{w}_{j}$ are satisfied. In either case, we have found the desired inequality satisfied by both starting tuples, a contradiction with the observation above. + +## Common remarks. + +1. For $n \in\{1,2\}$, two starting $n$-tuples are necessary and sufficient. +2. The operations,$+ \vee$ used in this problem are studied in the area of tropical geometry. However, as far as we know, familiarity with tropical geometry does not help when solving the problem. + +## C8. + +Alice fills the fields of an $n \times n$ board with numbers from 1 to $n^{2}$, each number being used exactly once. She then counts the total number of good paths on the board. A good path is a sequence of fields of arbitrary length (including 1) such that: +(i) The first field in the sequence is one that is only adjacent to fields with larger numbers, +(ii) Each subsequent field in the sequence is adjacent to the previous field, +(iii) The numbers written on the fields in the sequence are in increasing order. + +Two fields are considered adjacent if they share a common side. Find the smallest possible number of good paths Alice can obtain, as a function of $n$. +(Serbia) +Answer: $2 n^{2}-2 n+1$. + +## Solution. + +We will call any field that is only adjacent to fields with larger numbers a well. Other fields will be called non-wells. Let us make a second $n \times n$ board $B$ where in each field we will write the number of good sequences which end on the corresponding field in the original board $A$. We will thus look for the minimal possible value of the sum of all entries in $B$. + +We note that any well has just one good path ending in it, consisting of just the well, and that any other field has the number of good paths ending in it equal to the sum of this quantity for all the adjacent fields with smaller values, since a good path can only come into the field from a field of lower value. Therefore, if we fill in the fields in $B$ in increasing order with respect to their values in $A$, it follows that each field not adjacent to any already filled field will receive a 1, while each field adjacent to already filled fields will receive the sum of the numbers already written on these adjacent fields. + +We note that there is at least one well in $A$, that corresponding with the field with the entry 1 in $A$. Hence, the sum of values of fields in $B$ corresponding to wells in $A$ is at least 1 . We will now try to minimize the sum of the non-well entries, i.e. of the entries in $B$ corresponding to the non-wells in $A$. We note that we can ascribe to each pair of adjacent fields the value of the lower assigned number and that the sum of non-well entries will then equal to the sum of the ascribed numbers. Since the lower number is still at least 1, the sum of non-well entries will at least equal the number of pairs of adjacent fields, which is $2 n(n-1)$. Hence, the total minimum sum of entries in $B$ is at least $2 n(n-1)+1=2 n^{2}-2 n+1$. The necessary conditions for the minimum to be achieved is for there to be only one well and for no two entries in $B$ larger than 1 to be adjacent to each other. + +We will now prove that the lower limit of $2 n^{2}-2 n+1$ entries can be achieved. This amounts to finding a way of marking a certain set of squares, those that have a value of 1 in $B$, such that no two unmarked squares are adjacent and that the marked squares form a connected tree with respect to adjacency. + +For $n=1$ and $n=2$ the markings are respectively the lone field and the L-trimino. Now, for $n>2$, let $s=2$ for $n \equiv 0,2 \bmod 3$ and $s=1$ for $n \equiv 1 \bmod 3$. We will take indices $k$ and $l$ to be arbitrary non-negative integers. For $n \geqslant 3$ we will construct a path of marked squares in the first two columns consisting of all squares of the form $(1, i)$ where $i$ is not of the form $6 k+s$ and $(2, j)$ where $j$ is of the form $6 k+s-1,6 k+s$ or $6+s+1$. Obviously, this path is connected. Now, let us consider the fields $(2,6 k+s)$ and $(1,6 k+s+3)$. For each considered field $(i, j)$ we will mark all squares of the form $(l, j)$ for $l>i$ and $(i+2 k, j \pm 1)$. One can easily see that no set of marked fields will produce a cycle, that the only fields of the unmarked form $(1,6 k+s),(2+2 l+1,6 k+s \pm 1)$ and $(2+2 l, 6 k+s+3 \pm 1)$ and that no two are adjacent, since +the consecutive considered fields are in columns of opposite parity. Examples of markings are given for $n=3,4,5,6,7$, and the corresponding constructions for $A$ and $B$ are given for $n=5$. +![](https://cdn.mathpix.com/cropped/2024_11_18_86efb4389f14b52b08e1g-41.jpg?height=352&width=1648&top_left_y=384&top_left_x=204) + +## Common remarks. + +- The construction can be achieved in different ways. For example, it can also be done recursively; we can complete any construction for $n$ to a construction for $n+1$. +- It is a natural idea to change the direction of the path: that way it can start anywhere, but only can end in a well, which exactly means that we cannot extend the path. This is just a reformulation of the problem, but can give some intuitions. + +C9. Let $\mathbb{Z}_{\geqslant 0}$ be the set of non-negative integers, and let $f: \mathbb{Z}_{\geqslant 0} \times \mathbb{Z}_{\geqslant 0} \rightarrow \mathbb{Z}_{\geqslant 0}$ be a bijection such that whenever $f\left(x_{1}, y_{1}\right)>f\left(x_{2}, y_{2}\right)$, we have $f\left(x_{1}+1, y_{1}\right)>f\left(x_{2}+1, y_{2}\right)$ and $f\left(x_{1}, y_{1}+1\right)>f\left(x_{2}, y_{2}+1\right)$. + +Let $N$ be the number of pairs of integers $(x, y)$, with $0 \leqslant x, y<100$, such that $f(x, y)$ is odd. Find the smallest and largest possible value of $N$. + +Answer: The optimal bounds are $2500 \leqslant N \leqslant 7500$. +Solution. We defer the constructions to the end of the solution. Instead, we begin by characterizing all such functions $f$, prove a formula and key property for such functions, and then solve the problem, providing constructions. + +Characterization Suppose $f$ satisfies the given relation. The condition can be written more strongly as + +$$ +\begin{aligned} +f\left(x_{1}, y_{1}\right)>f\left(x_{2}, y_{2}\right) & \Longleftrightarrow f\left(x_{1}+1, y_{1}\right)>f\left(x_{2}+1, y_{2}\right) \\ +& \Longleftrightarrow f\left(x_{1}, y_{1}+1\right)>f\left(x_{2}, y_{2}+1\right) . +\end{aligned} +$$ + +In particular, this means for any $(k, l) \in \mathbb{Z}^{2}, f(x+k, y+l)-f(x, y)$ has the same sign for all $x$ and $y$. + +Call a non-zero vector $(k, l) \in \mathbb{Z}_{\geqslant 0} \times \mathbb{Z}_{\geqslant 0}$ a needle if $f(x+k, y)-f(x, y+l)>0$ for all $x$ and $y$. It is not hard to see that needles and non-needles are both closed under addition, and thus under scalar division (whenever the quotient lives in $\mathbb{Z}^{2}$ ). + +In addition, call a positive rational number $\frac{k}{l}$ a grade if the vector $(k, l)$ is a needle. (Since needles are closed under scalar multiples and quotients, this is well-defined.) +Claim. Grades are closed upwards. +Proof. Consider positive rationals $k_{1} / l_{1}0$ and $(1,0)$ is a needle). + +Thus $\left(k_{2}, l_{2}\right)$ is a needle, as wanted. +Claim. A grade exists. +Proof. If no positive integer $n$ is a grade, then $f(1,0)>f(0, n)$ for all $n$ which is impossible. +Similarly, there is an $n$ such that $f(0,1)x_{2}+y_{2} \alpha$ then $f\left(x_{1}, y_{1}\right)>f\left(x_{2}, y_{2}\right)$. +Proof. If both $x_{1} \geqslant x_{2}$ and $y_{1} \geqslant y_{2}$ this is clear. +Suppose $x_{1} \geqslant x_{2}$ and $y_{1}\alpha$ is a grade. This gives $f\left(x_{1}, y_{1}\right)>f\left(x_{2}, y_{2}\right)$. Suppose $x_{1}0} \mid x+(y+1) \alpha \leqslant a+b \alpha<(x+1)+(y+1) \alpha\right\}+ \\ +\#\left\{(a, 0) \in \mathbb{Z}_{\geqslant 0} \times \mathbb{Z}_{\geqslant 0} \mid(x+1)+y \alpha \leqslant a<(x+1)+(y+1) \alpha\right\}= \\ +\#\left\{(a, b) \in \mathbb{Z}_{\geqslant 0} \times \mathbb{Z}_{\geqslant 0} \mid x+y \alpha \leqslant a+b \alpha<(x+1)+y \alpha\right\}+1=f(x+1, y)-f(x, y) . +\end{gathered} +$$ + +From this claim we immediately get that $2500 \leqslant N \leqslant 7500$; now we show that those bounds are indeed sharp. + +Remember that if $\alpha$ is irrational then + +$$ +f(a, b)=\#\left\{(x, y) \in \mathbb{Z}_{\geqslant 0} \times \mathbb{Z}_{\geqslant 0} \mid x+y \alphaA B$, let $O$ be its circumcentre, and let $D$ be a point on the segment $B C$. The line through $D$ perpendicular to $B C$ intersects the lines $A O, A C$ and $A B$ at $W, X$ and $Y$, respectively. The circumcircles of triangles $A X Y$ and $A B C$ intersect again at $Z \neq A$. + +Prove that if $O W=O D$, then $D Z$ is tangent to the circle $A X Y$. +(United Kingdom) + +Solution 1. Let $A O$ intersect $B C$ at $E$. As $E D W$ is a right-angled triangle and $O$ is on $W E$, the condition $O W=O D$ means $O$ is the circumcentre of this triangle. So $O D=O E$ which establishes that $D, E$ are reflections in the perpendicular bisector of $B C$. + +Now observe: + +$$ +180^{\circ}-\angle D X Z=\angle Z X Y=\angle Z A Y=\angle Z C D +$$ + +which shows $C D X Z$ is cyclic. +![](https://cdn.mathpix.com/cropped/2024_11_18_86efb4389f14b52b08e1g-51.jpg?height=955&width=706&top_left_y=1076&top_left_x=681) + +We next show that $A Z \| B C$. To do this, introduce point $Z^{\prime}$ on circle $A B C$ such that $A Z^{\prime} \| B C$. By the previous result, it suffices to prove that $C D X Z^{\prime}$ is cyclic. Notice that triangles $B A E$ and $C Z^{\prime} D$ are reflections in the perpendicular bisector of $B C$. Using this and that $A, O, E$ are collinear: + +$$ +\angle D Z^{\prime} C=\angle B A E=\angle B A O=90^{\circ}-\frac{1}{2} \angle A O B=90^{\circ}-\angle C=\angle D X C, +$$ + +so $D X Z^{\prime} C$ is cyclic, giving $Z \equiv Z^{\prime}$ as desired. +Using $A Z \| B C$ and $C D X Z$ cyclic we get: + +$$ +\angle A Z D=\angle C D Z=\angle C X Z=\angle A Y Z, +$$ + +which by the converse of alternate segment theorem shows $D Z$ is tangent to circle $A X Y$. + +Solution 2. Notice that point $Z$ is the Miquel-point of lines $A C, B C, B A$ and $D Y$; then $B, D, Z, Y$ and $C, D, X, Y$ are concyclic. Moreover, $Z$ is the centre of the spiral similarity that maps $B C$ to $Y X$. + +By $B C \perp Y X$, the angle of that similarity is $90^{\circ}$; hence the circles $A B C Z$ and $A X Y Z$ are perpendicular, therefore the radius $O Z$ in circle $A B C Z$ is tangent to circle $A X Y Z$. +![](https://cdn.mathpix.com/cropped/2024_11_18_86efb4389f14b52b08e1g-52.jpg?height=949&width=934&top_left_y=528&top_left_x=561) + +By $O W=O D$, the triangle $O W D$ is isosceles, and + +$$ +\angle Z O A=2 \angle Z B A=2 \angle Z B Y=2 \angle Z D Y=\angle O D W+\angle D W O, +$$ + +so $D$ lies on line $Z O$ that is tangent to circle $A X Y$. + +G5. Let $A B C$ be a triangle, and let $\ell_{1}$ and $\ell_{2}$ be two parallel lines. For $i=1,2$, let $\ell_{i}$ meet the lines $B C, C A$, and $A B$ at $X_{i}, Y_{i}$, and $Z_{i}$, respectively. Suppose that the line through $X_{i}$ perpendicular to $B C$, the line through $Y_{i}$ perpendicular to $C A$, and finally the line through $Z_{i}$ perpendicular to $A B$, determine a non-degenerate triangle $\Delta_{i}$. + +Show that the circumcircles of $\Delta_{1}$ and $\Delta_{2}$ are tangent to each other. +(Vietnam) +Solution 1. Throughout the solutions, $\Varangle(p, q)$ will denote the directed angle between lines $p$ and $q$, taken modulo $180^{\circ}$. + +Let the vertices of $\Delta_{i}$ be $D_{i}, E_{i}, F_{i}$, such that lines $E_{i} F_{i}, F_{i} D_{i}$ and $D_{i} E_{i}$ are the perpendiculars through $X, Y$ and $Z$, respectively, and denote the circumcircle of $\Delta_{i}$ by $\omega_{i}$. + +In triangles $D_{1} Y_{1} Z_{1}$ and $D_{2} Y_{2} Z_{2}$ we have $Y_{1} Z_{1} \| Y_{2} Z_{2}$ because they are parts of $\ell_{1}$ and $\ell_{2}$. Moreover, $D_{1} Y_{1} \| D_{2} Y_{2}$ are perpendicular to $A C$ and $D_{1} Z_{1} \| D_{2} Z_{2}$ are perpendicular to $A B$, so the two triangles are homothetic and their homothetic centre is $Y_{1} Y_{2} \cap Z_{1} Z_{2}=A$. Hence, line $D_{1} D_{2}$ passes through $A$. Analogously, line $E_{1} E_{2}$ passes through $B$ and $F_{1} F_{2}$ passes through $C$. +![](https://cdn.mathpix.com/cropped/2024_11_18_86efb4389f14b52b08e1g-53.jpg?height=872&width=1300&top_left_y=1049&top_left_x=384) + +The corresponding sides of $\Delta_{1}$ and $\Delta_{2}$ are parallel, because they are perpendicular to the respective sides of triangle $A B C$. Hence, $\Delta_{1}$ and $\Delta_{2}$ are either homothetic, or they can be translated to each other. Using that $B, X_{2}, Z_{2}$ and $E_{2}$ are concyclic, $C, X_{2}, Y_{2}$ and $F_{2}$ are concyclic, $Z_{2} E_{2} \perp A B$ and $Y_{2}, F_{2} \perp A C$ we can calculate + +$$ +\begin{aligned} +\Varangle\left(E_{1} E_{2}, F_{1} F_{2}\right) & =\Varangle\left(E_{1} E_{2}, X_{1} X_{2}\right)+\Varangle\left(X_{1} X_{2}, F_{1} F_{2}\right)=\Varangle\left(B E_{2}, B X_{2}\right)+\Varangle\left(C X_{2}, C F_{2}\right) \\ +& =\Varangle\left(Z_{2} E_{2}, Z_{2} X_{2}\right)+\Varangle\left(Y_{2} X_{2}, Y_{2} F_{2}\right)=\Varangle\left(Z_{2} E_{2}, \ell_{2}\right)+\Varangle\left(\ell_{2}, Y_{2} F_{2}\right) \\ +& =\Varangle\left(Z_{2} E_{2}, Y_{2} F_{2}\right)=\Varangle(A B, A C) \not \equiv 0, +\end{aligned} +$$ + +and conclude that lines $E_{1} E_{2}$ and $F_{1} F_{2}$ are not parallel. Hence, $\Delta_{1}$ and $\Delta_{2}$ are homothetic; the lines $D_{1} D_{2}, E_{1} E_{2}$, and $F_{1} F_{2}$ are concurrent at the homothetic centre of the two triangles. Denote this homothetic centre by $H$. + +For $i=1,2$, using (1), and that $A, Y_{i}, Z_{i}$ and $D_{i}$ are concyclic, + +$$ +\begin{aligned} +\Varangle\left(H E_{i}, H F_{i}\right) & =\Varangle\left(E_{1} E_{2}, F_{1} F_{2}\right)=\Varangle(A B, A C) \\ +& =\Varangle\left(A Z_{i}, A Y_{i}\right)=\Varangle\left(D_{i} Z_{i}, D_{i} Y_{i}\right)=\Varangle\left(D_{i} E_{i}, D_{i} F_{i}\right), +\end{aligned} +$$ + +so $H$ lies on circle $\omega_{i}$. +The same homothety that maps $\Delta_{1}$ to $\Delta_{2}$, sends $\omega_{1}$ to $\omega_{2}$ as well. Point $H$, that is the centre of the homothety, is a common point of the two circles, That finishes proving that $\omega_{1}$ and $\omega_{2}$ are tangent to each other. + +Solution 2. As in the first solution, let the vertices of $\Delta_{i}$ be $D_{i}, E_{i}, F_{i}$, such that $E_{i} F_{i}, F_{i} D_{i}$ and $D_{i} E_{i}$ are the perpendiculars through $X_{i}, Y_{i}$ and $Z_{i}$, respectively. In the same way we conclude that $\left(A, D_{1}, D_{2}\right),\left(B, E_{1}, E_{2}\right)$ and $\left(C, F_{1}, F_{2}\right)$ are collinear. + +The corresponding sides of triangles $A B C$ and $D_{i} E_{i} F_{i}$ are perpendicular to each other. Hence, there is a spiral similarity with rotation $\pm 90^{\circ}$ that maps $A B C$ to $D_{i} E_{i} F_{i}$; let $M_{i}$ be the centre of that similarity. Hence, $\Varangle\left(M_{i} A, M_{i} D_{i}\right)=\Varangle\left(M_{i} B, M_{i} E_{i}\right)=\Varangle\left(M_{i} C, M_{i} F_{i}\right)=90^{\circ}$. The circle with diameter $A D_{i}$ passes through $M_{i}, Y_{i}, Z_{i}$, so $M_{i}, A, Y_{i}, Z_{i}, D_{i}$ are concyclic; analogously $\left(M_{i}, B, X_{i}, Z_{i}, E_{i}\right.$ ) and ( $\left.M_{i}, C, X_{i}, Y_{i}, F_{i}\right)$ are concyclic. + +By applying Desargues' theorem to triangles $A B C$ and $D_{i} E_{i} F_{i}$ we conclude that the lines $A D_{i}, B E_{i}$ and $B F_{i}$ are concurrent; let their intersection be $H$. Since $\left(A, D_{1} \cdot D_{2}\right),\left(B, E_{1} \cdot E_{2}\right)$ and $\left(C, F_{1}, F_{2}\right)$ are collinear, we obtain the same point $H$ for $i=1$ and $i=2$. +![](https://cdn.mathpix.com/cropped/2024_11_18_86efb4389f14b52b08e1g-54.jpg?height=849&width=858&top_left_y=1100&top_left_x=602) + +By $\Varangle(C B, C H)=\Varangle\left(C X_{i}, C F_{i}\right)=\Varangle\left(Y_{i} X_{i}, Y_{i} F_{i}\right)=\Varangle\left(Y_{i} Z_{i}, Y_{i} D_{i}\right)=\Varangle\left(A Z_{i}, A D_{i}\right)=$ $\Varangle(A B, A H)$, point $H$ lies on circle $A B C$. + +Analogously, from $\Varangle\left(F_{i} D_{i}, F_{i} H\right)=\Varangle\left(F_{i} Y_{i}, F_{i} C\right)=\Varangle\left(X_{i} Y_{i}, X_{i} C\right)=\Varangle\left(X_{i} Z_{i}, X_{i} B\right)=$ $\Varangle\left(E_{i} Z_{i}, E_{i} B\right)=\Varangle\left(E_{i} D_{i}, E_{i} H\right)$, we can see that point $H$ lies on circle $D_{i} E_{i} F_{i}$ as well. Therefore, circles $A B C$ and $D_{i} E_{i} F_{i}$ intersect at point $H$. + +The spiral similarity moves the circle $A B C$ to circle $D_{i} E_{i} F_{i}$, so the two circles are perpendicular. Hence, both circles $D_{1} E_{1} F_{1}$ and $D_{2} E_{2} F_{2}$ are tangent to the radius of circle $A B C$ at $H$. + +Comment 1. As the last picture suggests, the circles $A B C$ and $D_{i} E_{i} F_{i}$ pass through $M_{i}$. In fact, point $M_{i}$, being the second intersection of circles $D_{i} E_{i} F_{i}$ and $D_{i} Y_{i} Z_{i}$, the Miquel point of the lines $A Y_{i}, A Z_{i}, C X_{i}$ and $X_{i} Y_{i}$, so it is concyclic with $A, B, C$. Similarly, $M_{i}$ the Miquel point of lines $D_{i} E_{i}$, $E_{i} F_{i}, F_{i} Y_{i}$ and $X_{i} Y_{i}$, so it is concyclic with $D_{i}, E_{i}, D_{i}$. + +Comment 2. Instead of two lines $\ell_{1}$ and $\ell_{2}$, it is possible to reformulate the problem with a single, varying line $\ell$ : + +Let $A B C$ be a triangle, and let $\ell$ be a varying line whose direction is fixed. Let $\ell$ intersect lines $B C$, $C A$, and $A B$ at $X, Y$, and $Z$, respectively. Suppose that the line through $X$, perpendicular to $B C$, the line through $Y$, perpendicular to $C A$, and the line through $Z$, perpendicular to $A B$, determine a non-degenerate triangle $\Delta$. + +Show that as $\ell$ varies, the circumcircles of the obtained triangles $\Delta$ pass through a fixed point, and these circles are tangent to each other. + +A reasonable approach is finding the position of line $\ell$ when the triangle $D E F$ degenerates to a single point. That happens when the line $X Y Z$ is the Simson line respect to point $D=E=F$ on the circumcircle $A B C$. Based on this observation a possible variant of the solution is as follows. + +Let $H$ be the second intersection of circles $A B C$ and line $A D$. Like in the solutions above, we can find that the line $A D$ is fixed, so $H$ is independent of the position of line $\ell$. +![](https://cdn.mathpix.com/cropped/2024_11_18_86efb4389f14b52b08e1g-55.jpg?height=846&width=795&top_left_y=819&top_left_x=633) + +From $\Varangle(H F, H D)=\Varangle(H C, H A)=\Varangle(B C, B A)=\Varangle(B X, B Z)=\Varangle(E X, E Z)=\Varangle(E F, E D)$ we can see that circle $\Delta$ passes through $H$. Hence, all circles $D E F$ passes through a fixed point. + +The corresponding sides of triangles $A B C$ and $D E F$ are perpendicular, so their circumcircle are perpendicular; that proves that circle $D E F$ is tangent to the radius of circle $A B C$ at $H$. + +G6. In an acute-angled triangle $A B C$, point $H$ is the foot of the altitude from $A$. Let $P$ be a moving point such that the bisectors $k$ and $\ell$ of angles $P B C$ and $P C B$, respectively, intersect each other on the line segment $A H$. Let $k$ and $A C$ meet at $E$, let $\ell$ and $A B$ meet at $F$, and let $E F$ and $A H$ meet at $Q$. Prove that, as $P$ varies, the line $P Q$ passes through a fixed point. +(Iran) +Solution 1. Let the reflections of the line $B C$ with respect to the lines $A B$ and $A C$ intersect at point $K$. We will prove that $P, Q$ and $K$ are collinear, so $K$ is the common point of the varying line $P Q$. + +Let lines $B E$ and $C F$ intersect at $I$. For every point $O$ and $d>0$, denote by $(O, d)$ the circle centred at $O$ with radius $d$, and define $\omega_{I}=(I, I H)$ and $\omega_{A}=(A, A H)$. Let $\omega_{K}$ and $\omega_{P}$ be the incircle of triangle $K B C$ and the $P$-excircle of triangle $P B C$, respectively. + +Since $I H \perp B C$ and $A H \perp B C$, the circles $\omega_{A}$ and $\omega_{I}$ are tangent to each other at $H$. So, $H$ is the external homothetic centre of $\omega_{A}$ and $\omega_{I}$. From the complete quadrangle $B C E F$ we have $(A, I ; Q, H)=-1$, therefore $Q$ is the internal homothetic centre of $\omega_{A}$ and $\omega_{I}$. Since $B A$ and $C A$ are the external bisectors of angles $\angle K B C$ and $\angle K C B$, circle $\omega_{A}$ is the $K$-excircle in triangle $B K C$. Hence, $K$ is the external homothetic centre of $\omega_{A}$ and $\omega_{K}$. Also it is clear that $P$ is the external homothetic centre of $\omega_{I}$ and $\omega_{P}$. Let point $T$ be the tangency point of $\omega_{P}$ and $B C$, and let $T^{\prime}$ be the tangency point of $\omega_{K}$ and $B C$. Since $\omega_{I}$ is the incircle and $\omega_{P}$ is the $P$-excircle of $P B C, T C=B H$ and since $\omega_{K}$ is the incircle and $\omega_{A}$ is the $K$-excircle of $K B C, T^{\prime} C=B H$. Therefore $T C=T^{\prime} C$ and $T \equiv T^{\prime}$. It yields that $\omega_{K}$ and $\omega_{P}$ are tangent to each other at $T$. +![](https://cdn.mathpix.com/cropped/2024_11_18_86efb4389f14b52b08e1g-56.jpg?height=930&width=1169&top_left_y=1491&top_left_x=449) + +Let point $S$ be the internal homothetic centre of $\omega_{A}$ and $\omega_{P}$, and let $S^{\prime}$ be the internal homothetic centre of $\omega_{I}$ and $\omega_{K}$. It's obvious that $S$ and $S^{\prime}$ lie on $B C$. We claim that $S \equiv S^{\prime}$. To prove our claim, let $r_{A}, r_{I}, r_{P}$, and $r_{K}$ be the radii of $\omega_{A}, \omega_{I}, \omega_{P}$ and $\omega_{k}$, respectively. + +It is well known that if the sides of a triangle are $a, b, c$, its semiperimeter is $s=(a+b+c) / 2$, and the radii of the incircle and the $a$-excircle are $r$ and $r_{a}$, respectively, then $r \cdot r_{a}=(s-b)(s-c)$. Applying this fact to triangle $P B C$ we get $r_{I} \cdot r_{P}=B H \cdot C H$. The same fact in triangle $K C B$ +yields $r_{K} \cdot r_{A}=C T \cdot B T$. Since $B H=C T$ and $B T=C H$, from these two we get + +$$ +\frac{H S}{S T}=\frac{r_{A}}{r_{P}}=\frac{r_{I}}{r_{K}}=\frac{H S^{\prime}}{S^{\prime} T} +$$ + +so $S=S^{\prime}$ indeed. +Finally, by applying the generalised Monge's theorem to the circles $\omega_{A}, \omega_{I}$, and $\omega_{K}$ (with two pairs of internal and one pair of external common tangents), we can see that points $Q$, $S$, and $K$ are collinear. Similarly one can show that $Q, S$ and $P$ are collinear, and the result follows. + +Solution 2. Again, let $B E$ and $C F$ meet at $I$, that is the incentre in triangle $B C P$; then $P I$ is the third angle bisector. From the tangent segments of the incircle we have $B P-C P=$ $B H-C H$; hence, the possible points $P$ lie on a branch of a hyperbola $\mathcal{H}$ with foci $B, C$, and $H$ is a vertex of $\mathcal{H}$. Since $P I$ bisects the angle between the radii $B P$ and $C P$ of the hyperbola, line $P I$ is tangent to $\mathcal{H}$. +![](https://cdn.mathpix.com/cropped/2024_11_18_86efb4389f14b52b08e1g-57.jpg?height=1066&width=1306&top_left_y=1049&top_left_x=378) + +Let $K$ be the second intersection of $P Q$ and $\mathcal{H}$, we will show that $A K$ is tangent to $\mathcal{H}$ at $K$; this property determines $K$. + +Let $G=K I \cap A P$ and $M=P I \cap A K$. From the complete quadrangle $B C E F$ we can see that $(H, Q ; I, A)$ is harmonic, so in the complete quadrangle $A P I K$, point $H$ lies on line $G M$. + +Consider triangle $A I M$. Its side $A I$ is tangent to $\mathcal{H}$ at $H$, the side $I M$ is tangent to $\mathcal{H}$ at $P$, and $K$ is a common point of the third side $A M$ and the hyperbola such that the lines $A P$, $I K$ and $M H$ are concurrent at the generalised Gergonne-point $G$. It follows that the third side, $A M$ is also tangent to $\mathcal{H}$ at $K$. +(Alternatively, in the last step we can apply the converse of Brianchon's theorem to the degenerate hexagon $A H I P M K$. By the theorem there is a conic section $\mathcal{H}^{\prime}$ such that lines $A I, I M$ and $M A$ are tangent to $\mathcal{H}^{\prime}$ at $H, P$ and $K$, respectively. But the three points $H, K$ and $P$, together with the tangents at $H$ and $P$ uniquely determine $\mathcal{H}^{\prime}$, so indeed $\mathcal{H}^{\prime}=\mathcal{H}$.) + +G7. Let $A B C$ and $A^{\prime} B^{\prime} C^{\prime}$ be two triangles having the same circumcircle $\omega$, and the same orthocentre $H$. Let $\Omega$ be the circumcircle of the triangle determined by the lines $A A^{\prime}, B B^{\prime}$ and $C C^{\prime}$. Prove that $H$, the centre of $\omega$, and the centre of $\Omega$ are collinear. +(Denmark) +Solution. In what follows, $\Varangle(p, q)$ will denote the directed angle between lines $p$ and $q$, taken modulo $180^{\circ}$. Denote by $O$ the centre of $\omega$. In any triangle, the homothety with ratio $-\frac{1}{2}$ centred at the centroid of the triangle takes the vertices to the midpoints of the opposite sides and it takes the orthocentre to the circumcentre. Therefore the triangles $A B C$ and $A^{\prime} B^{\prime} C^{\prime}$ share the same centroid $G$ and the midpoints of their sides lie on a circle $\rho$ with centre on $O H$. We will prove that $\omega, \Omega$, and $\rho$ are coaxial, so in particular it follows that their centres are collinear on OH . + +Let $D=B B^{\prime} \cap C C^{\prime}, E=C C^{\prime} \cap A A^{\prime}, F=A A^{\prime} \cap B B^{\prime}, S=B C^{\prime} \cap B^{\prime} C$, and $T=B C \cap B^{\prime} C^{\prime}$. Since $D, S$, and $T$ are the intersections of opposite sides and of the diagonals in the quadrilateral $B B^{\prime} C C^{\prime}$ inscribed in $\omega$, by Brocard's theorem triangle $D S T$ is self-polar with respect to $\omega$, i.e. each vertex is the pole of the opposite side. We apply this in two ways. +![](https://cdn.mathpix.com/cropped/2024_11_18_86efb4389f14b52b08e1g-58.jpg?height=835&width=1589&top_left_y=1053&top_left_x=239) + +First, from $D$ being the pole of $S T$ it follows that the inverse $D^{*}$ of $D$ with respect to $\omega$ is the projection of $D$ onto $S T$. In particular, $D^{*}$ lies on the circle with diameter $S D$. If $N$ denotes the midpoint of $S D$ and $R$ the radius of $\omega$, then the power of $O$ with respect to this circle is $O N^{2}-N D^{2}=O D \cdot O D^{*}=R^{2}$. By rearranging, we see that $N D^{2}$ is the power of $N$ with respect to $\omega$. + +Second, from $T$ being the pole of $S D$ it follows that $O T$ is perpendicular to $S D$. Let $M$ and $M^{\prime}$ denote the midpoints of $B C$ and $B^{\prime} C^{\prime}$. Then since $O M \perp B C$ and $O M^{\prime} \perp B^{\prime} C^{\prime}$ it follows that $O M M^{\prime} T$ is cyclic and + +$$ +\Varangle(S D, B C)=\Varangle(O T, O M)=\Varangle\left(B^{\prime} C^{\prime}, M M^{\prime}\right) . +$$ + +From $B B^{\prime} C C^{\prime}$ being cyclic we also have $\Varangle\left(B C, B B^{\prime}\right)=\Varangle\left(C C^{\prime}, B^{\prime} C^{\prime}\right)$, hence we obtain + +$$ +\begin{aligned} +\Varangle\left(S D, B B^{\prime}\right) & =\Varangle(S D, B C)+\Varangle\left(B C, B B^{\prime}\right) \\ +& =\Varangle\left(B^{\prime} C^{\prime}, M M^{\prime}\right)+\Varangle\left(C C^{\prime}, B^{\prime} C^{\prime}\right)=\Varangle\left(C C^{\prime}, M M^{\prime}\right) . +\end{aligned} +$$ + +Now from the homothety mentioned in the beginning, we know that $M M^{\prime}$ is parallel to $A A^{\prime}$, hence the above implies that $\Varangle\left(S D, B B^{\prime}\right)=\Varangle\left(C C^{\prime}, A A^{\prime}\right)$, which shows that $\Omega$ is tangent to $S D$ at $D$. In particular, $N D^{2}$ is also the power of $N$ with respect to $\Omega$. + +Additionally, from $B B^{\prime} C C^{\prime}$ being cyclic it follows that triangles $D B C$ and $D C^{\prime} B^{\prime}$ are inversely similar, so $\Varangle\left(B B^{\prime}, D M^{\prime}\right)=\Varangle\left(D M, C C^{\prime}\right)$. This yields + +$$ +\begin{aligned} +\Varangle\left(S D, D M^{\prime}\right) & =\Varangle\left(S D, B B^{\prime}\right)+\Varangle\left(B B^{\prime}, D M^{\prime}\right) \\ +& =\Varangle\left(C C^{\prime}, M M^{\prime}\right)+\Varangle\left(D M, C C^{\prime}\right)=\Varangle\left(D M, M M^{\prime}\right), +\end{aligned} +$$ + +which shows that the circle $D M M^{\prime}$ is also tangent to $S D$. Since $N, M$, and $M^{\prime}$ are collinear on the Newton-Gauss line of the complete quadrilateral determined by the lines $B B^{\prime}, C C^{\prime}, B C^{\prime}$, and $B^{\prime} C$, it follows that $N D^{2}=N M \cdot N M^{\prime}$. Hence $N$ has the same power with respect to $\omega$, $\Omega$, and $\rho$. + +By the same arguments there exist points on the tangents to $\Omega$ at $E$ and $F$ which have the same power with respect to $\omega, \Omega$, and $\rho$. The tangents to a given circle at three distinct points cannot be concurrent, hence we obtain at least two distinct points with the same power with respect to $\omega, \Omega$, and $\rho$. Hence the three circles are coaxial, as desired. + +Comment 1. Instead of invoking the Newton-Gauss line, one can also use a nice symmetry argument: If from the beginning we swapped the labels of $B^{\prime}$ and $C^{\prime}$, then in the proof above the labels of $D$ and $S$ would be swapped while the labels of $M$ and $M^{\prime}$ do not change. The consequence is that the circle $S M M^{\prime}$ is also tangent to $S D$. Since $N$ is the midpoint of $S D$ it then has the same power with respect to circles $D M M^{\prime}$ and $S M M^{\prime}$, so it lies on their radical axis $M M^{\prime}$. + +Comment 2. There exists another triple of points on the common radical axis of $\omega, \Omega$, and $\rho$ which can be used to solve the problem. We outline one such solution. +![](https://cdn.mathpix.com/cropped/2024_11_18_86efb4389f14b52b08e1g-59.jpg?height=752&width=1026&top_left_y=1297&top_left_x=518) + +Let $L$ and $L^{\prime}$ denote the feet of the altitudes from $A$ and $A^{\prime}$ in triangle $A B C$ and $A^{\prime} B^{\prime} C^{\prime}$, respectively. Since $\rho$ is the nine-point circle of the two triangles it contains both $L$ and $L^{\prime}$. Furthermore, $H A \cdot H L$ and $H A^{\prime} \cdot H L^{\prime}$ both equal twice the power of $H$ with respect to $\rho$, so $A, A^{\prime}, L, L^{\prime}$ are concyclic as well. + +Now let $\ell=A A^{\prime}$ and denote $P=L L^{\prime} \cap \ell, K=B C \cap \ell$, and $K^{\prime}=B^{\prime} C^{\prime} \cap \ell$. As $M M^{\prime} \| \ell$ (shown in the previous solution) and $L L^{\prime} M M^{\prime}$ is cyclic + +$$ +\Varangle(B C, \ell)=\Varangle\left(B C, M M^{\prime}\right)=\Varangle\left(L L^{\prime}, B^{\prime} C^{\prime}\right) +$$ + +so $K, K^{\prime}, L$, and $L^{\prime}$ are also concyclic. From the cyclic quadrilaterals $A A^{\prime} L L^{\prime}$ and $K K^{\prime} L L^{\prime}$ we get $P A \cdot P A^{\prime}=P L \cdot P L^{\prime}=P K \cdot P K^{\prime}$. This implies that $P$ is the centre of the (unique) involution $\sigma$ on $\ell$ that swaps $A, A^{\prime}$ and $K, K^{\prime}$. On the other hand, by Desargues' involution theorem applied to the line $\ell$, the quadrilateral $B B^{\prime} C C^{\prime}$, and its circumcircle $\omega$, the involution $\sigma$ also swaps $E$ and $F$. Hence + +$$ +P A \cdot P A^{\prime}=P L \cdot P L^{\prime}=P E \cdot P F +$$ + +However, this means that $P$ has the same power with respect to $\omega, \Omega$, and $\rho$, and by the same arguments there exist points on $B B^{\prime}$ and $C C^{\prime}$ with this property. + +G8. Let $A A^{\prime} B C C^{\prime} B^{\prime}$ be a convex cyclic hexagon such that $A C$ is tangent to the incircle of the triangle $A^{\prime} B^{\prime} C^{\prime}$, and $A^{\prime} C^{\prime}$ is tangent to the incircle of the triangle $A B C$. Let the lines $A B$ and $A^{\prime} B^{\prime}$ meet at $X$ and let the lines $B C$ and $B^{\prime} C^{\prime}$ meet at $Y$. + +Prove that if $X B Y B^{\prime}$ is a convex quadrilateral, then it has an incircle. +(Australia) + +Solution. Denote by $\omega$ and $\omega^{\prime}$ the incircles of $\triangle A B C$ and $\triangle A^{\prime} B^{\prime} C^{\prime}$ and let $I$ and $I^{\prime}$ be the centres of these circles. Let $N$ and $N^{\prime}$ be the second intersections of $B I$ and $B^{\prime} I^{\prime}$ with $\Omega$, the circumcircle of $A^{\prime} B C C^{\prime} B^{\prime} A$, and let $O$ be the centre of $\Omega$. Note that $O N \perp A C, O N^{\prime} \perp A^{\prime} C^{\prime}$ and $O N=O N^{\prime}$ so $N N^{\prime}$ is parallel to the angle bisector $I I^{\prime}$ of $A C$ and $A^{\prime} C^{\prime}$. Thus $I I^{\prime} \| N N^{\prime}$ which is antiparallel to $B B^{\prime}$ with respect to $B I$ and $B^{\prime} I^{\prime}$. Therefore $B, I, I^{\prime}, B^{\prime}$ are concyclic. +![](https://cdn.mathpix.com/cropped/2024_11_18_86efb4389f14b52b08e1g-60.jpg?height=958&width=1072&top_left_y=926&top_left_x=498) + +Further define $P$ as the intersection of $A C$ and $A^{\prime} C^{\prime}$ and $M$ as the antipode of $N^{\prime}$ in $\Omega$. Consider the circle $\Gamma_{1}$ with centre $N$ and radius $N A=N C$ and the circle $\Gamma_{2}$ with centre $M$ and radius $M A^{\prime}=M C^{\prime}$. Their radical axis passes through $P$ and is perpendicular to $M N \perp N N^{\prime} \| I P$, so $I$ lies on their radical axis. Therefore, since $I$ lies on $\Gamma_{1}$, it must also lie on $\Gamma_{2}$. Thus, if we define $Z$ as the second intersection of $M I$ with $\Omega$, we have that $I$ is the incentre of triangle $Z A^{\prime} C^{\prime}$. (Note that the point $Z$ can also be constructed directly via Poncelet's porism.) + +Consider the incircle $\omega_{c}$ with centre $I_{c}$ of triangle $C^{\prime} B^{\prime} Z$. Note that $\angle Z I C^{\prime}=90^{\circ}+$ $\frac{1}{2} \angle Z A^{\prime} C^{\prime}=90^{\circ}+\frac{1}{2} \angle Z B^{\prime} C^{\prime}=\angle Z I_{c} C^{\prime}$, so $Z, I, I_{c}, C^{\prime}$ are concyclic. Similarly $B^{\prime}, I^{\prime}, I_{c}, C^{\prime}$ are concyclic. + +The external centre of dilation from $\omega$ to $\omega_{c}$ is the intersection of $I I_{c}$ and $C^{\prime} Z$ ( $D$ in the picture), that is the radical centre of circles $\Omega, C^{\prime} I_{c} I Z$ and $I I^{\prime} I_{c}$. Similarly, the external centre of dilation from $\omega^{\prime}$ to $\omega_{c}$ is the intersection of $I^{\prime} I_{c}$ and $B^{\prime} C^{\prime}$ ( $D^{\prime}$ in the picture), that is the radical centre of circles $\Omega, B^{\prime} I^{\prime} I_{c} C^{\prime}$ and $I I^{\prime} I_{c}$. Therefore the Monge line of $\omega, \omega^{\prime}$ and $\omega_{c}$ is line $D D^{\prime}$, and the radical axis of $\Omega$ and circle $I I^{\prime} I_{c}$ coincide. Hence the external centre $T$ of dilation from $\omega$ to $\omega^{\prime}$ is also on the radical axis of $\Omega$ and circle $I I^{\prime} I_{c}$. +![](https://cdn.mathpix.com/cropped/2024_11_18_86efb4389f14b52b08e1g-61.jpg?height=1029&width=964&top_left_y=228&top_left_x=546) + +Now since $B, I, I^{\prime}, B^{\prime}$ are concyclic, the intersection $T^{\prime}$ of $B B^{\prime}$ and $I I^{\prime}$ is on the radical axis of $\Omega$ and circle $I I^{\prime} I_{c}$. Thus $T^{\prime}=T$ and $T$ lies on line $B B^{\prime}$. Finally, construct a circle $\Omega_{0}$ tangent to $A^{\prime} B^{\prime}, B^{\prime} C^{\prime}, A B$ on the same side of these lines as $\omega^{\prime}$. The centre of dilation from $\omega^{\prime}$ to $\Omega_{0}$ is $B^{\prime}$, so by Monge's theorem the external centre of dilation from $\Omega_{0}$ to $\omega$ must be on the line $T B B^{\prime}$. However, it is on line $A B$, so it must be $B$ and $B C$ must be tangent to $\Omega_{0}$ as desired. +![](https://cdn.mathpix.com/cropped/2024_11_18_86efb4389f14b52b08e1g-61.jpg?height=918&width=1357&top_left_y=1637&top_left_x=361) + +## Number Theory + +N1. A number is called Norwegian if it has three distinct positive divisors whose sum is equal to 2022. Determine the smallest Norwegian number. +(Note: The total number of positive divisors of a Norwegian number is allowed to be larger than 3.) +(Cyprus) +Answer: 1344 +Solution. Observe that 1344 is a Norwegian number as 6, 672 and 1344 are three distinct divisors of 1344 and $6+672+1344=2022$. It remains to show that this is the smallest such number. + +Assume for contradiction that $N<1344$ is Norwegian and let $N / a, N / b$ and $N / c$ be the three distinct divisors of $N$, with $a\frac{2022}{1344}=\frac{337}{224}=\frac{3}{2}+\frac{1}{224} +$$ + +If $a>1$ then + +$$ +\frac{1}{a}+\frac{1}{b}+\frac{1}{c} \leqslant \frac{1}{2}+\frac{1}{3}+\frac{1}{4}=\frac{13}{12}<\frac{3}{2} +$$ + +so it must be the case that $a=1$. Similarly, it must hold that $b<4$ since otherwise + +$$ +1+\frac{1}{b}+\frac{1}{c} \leqslant 1+\frac{1}{4}+\frac{1}{5}<\frac{3}{2} . +$$ + +This leaves two cases to check, $b=2$ and $b=3$. +Case $b=3$. Then + +$$ +\frac{1}{c}>\frac{3}{2}+\frac{1}{224}-1-\frac{1}{3}>\frac{1}{6}, +$$ + +so $c=4$ or $c=5$. If $c=4$ then + +$$ +2022=N\left(1+\frac{1}{3}+\frac{1}{4}\right)=\frac{19}{12} N +$$ + +but this is impossible as $19 \nmid 2022$. If $c=5$ then + +$$ +2022=N\left(1+\frac{1}{3}+\frac{1}{5}\right)=\frac{23}{15} N +$$ + +which again is impossible, as $23 \nmid 2022$. +Case $b=2$. Note that $c<224$ since + +$$ +\frac{1}{c}>\frac{3}{2}+\frac{1}{224}-1-\frac{1}{2}=\frac{1}{224} +$$ + +It holds that + +$$ +2022=N\left(1+\frac{1}{2}+\frac{1}{c}\right)=\frac{3 c+2}{2 c} N \Rightarrow(3 c+2) N=4044 c +$$ + +Since $(c, 3 c-2)=(c, 2) \in\{1,2\}$, then $3 c+2 \mid 8088=2^{3} \cdot 3 \cdot 337$ which implies that $3 c+2 \mid 2^{3} \cdot 337$. But since $3 c+2 \geqslant 3 \cdot 3+2>8=2^{3}$ and $3 c+2 \neq 337$, then it must hold that $3 c+2 \geqslant 2 \cdot 337$, contradicting $c<224$. + +N2. Find all positive integers $n>2$ such that + +$$ +n!\mid \prod_{\substack{p2$ and 3 divides one of $p_{m-2}, p_{m-1}=p_{m-2}+2$ and $p_{m}=p_{m-2}+4$. This implies $p_{m-2}=3$ and thus $p_{m}=7$, giving $7 \leqslant n<11$. + +Finally, a quick computation shows that $7!\mid \prod_{p1$ be a positive integer, and let $d>1$ be a positive integer coprime to $a$. Let $x_{1}=1$ and, for $k \geqslant 1$, define + +$$ +x_{k+1}= \begin{cases}x_{k}+d & \text { if } a \text { doesn't divide } x_{k} \\ x_{k} / a & \text { if } a \text { divides } x_{k}\end{cases} +$$ + +Find the greatest positive integer $n$ for which there exists an index $k$ such that $x_{k}$ is divisible by $a^{n}$. +(Croatia) + +Answer: $n$ is the exponent with $d0}: 0d$ then $y-d \in S$ but $a \cdot y \notin S$; otherwise, if $yd, \\ +a \cdot y & \text { if } y1$ be an index such that $x_{k_{1}}=1$. Then + +$$ +x_{k_{1}}=1, \quad x_{k_{1}-1}=f^{-1}(1)=a, x_{k_{1}-2}=f^{-1}(a)=a^{2}, \ldots, \quad x_{k_{1}-n}=a^{n} +$$ + +Solution 3. Like in the first solution, $x_{k}$ is relatively prime to $d$ and $x_{k}a^{n} / a=a^{n-1}$ the LHS is strictly less than $a^{v-n}$. This implies that on the RHS, the coefficients of $a^{v-n}, a^{v-n+1}, \ldots$ must all be zero, i.e. $z_{v-n}=z_{v-n+1}=\cdots=z_{v-1}=0$. This implies that there are $n$ consecutive decreasing indices in the original sequence. + +N4. Find all triples of positive integers $(a, b, p)$ with $p$ prime and + +$$ +a^{p}=b!+p +$$ + +(Belgium) +Answer: $(2,2,2)$ and $(3,4,3)$. +Solution 1. Clearly, $a>1$. We consider three cases. +Case 1: We have $ab$ which is also impossible since in this case we have $b!\leqslant a!a>1$. +Case 2: We have $a>p$. In this case $b!=a^{p}-p>p^{p}-p \geqslant p!$ so $b>p$ which means that $a^{p}=b!+p$ is divisible by $p$. Hence, $a$ is divisible by $p$ and $b!=a^{p}-p$ is not divisible by $p^{2}$. This means that $b<2 p$. If $a(2 p-1)!+p \geqslant b!+p$. + +Comment. The inequality $p^{2 p}>(2 p-1)!+p$ can be shown e.g. by using + +$$ +(2 p-1)!=[1 \cdot(2 p-1)] \cdot[2 \cdot(2 p-2)] \cdots \cdots[(p-1)(p+1)] \cdot p<\left(\left(\frac{2 p}{2}\right)^{2}\right)^{p-1} \cdot p=p^{2 p-1} +$$ + +where the inequality comes from applying AM-GM to each of the terms in square brackets. +Case 3: We have $a=p$. In this case $b!=p^{p}-p$. One can check that the values $p=2,3$ lead to the claimed solutions and $p=5$ does not lead to a solution. So we now assume that $p \geqslant 7$. We have $b!=p^{p}-p>p$ ! and so $b \geqslant p+1$ which implies that +$v_{2}((p+1)!) \leqslant v_{2}(b!)=v_{2}\left(p^{p-1}-1\right) \stackrel{L T E}{=} 2 v_{2}(p-1)+v_{2}(p+1)-1=v_{2}\left(\frac{p-1}{2} \cdot(p-1) \cdot(p+1)\right)$, +where in the middle we used lifting-the-exponent lemma. On the RHS we have three factors of $(p+1)!$. But, due to $p+1 \geqslant 8$, there are at least 4 even numbers among $1,2, \ldots, p+1$, so this case is not possible. + +Solution 2. The cases $a \neq p$ are covered as in solution 1 , as are $p=2,3$. For $p \geqslant 5$ we have $b!=p\left(p^{p-1}-1\right)$. By Zsigmondy's Theorem there exists some prime $q$ that divides $p^{p-1}-1$ but does not divide $p^{k}-1$ for $k(2 p-1)^{p-1} p>p^{p}>p^{p}-p$, a contradiction. + +Solution 3. The cases $a \neq p$ are covered as in solution 1 , as are $p=2,3$. Also $b>p$, as $p^{p}>p!+p$ for $p>2$. The cases $p=5,7,11$ are also checked manually, so assume $p \geqslant 13$. +Let $q \mid p+1$ be an odd prime. By LTE + +$$ +v_{q}\left(p^{p}-p\right)=v_{q}\left(\left(p^{2}\right)^{\frac{p-1}{2}}-1\right)=v_{q}\left(p^{2}-1\right)+v_{q}\left(\frac{p-1}{2}\right)=v_{q}(p+1) +$$ + +But $b \geqslant p+1$, so then $v_{q}(b!)>v_{q}(p+1)$, since $qq^{d-1} \geqslant 2 d +$$ + +provided $d \geqslant 2$ and $q>3$, or $d \geqslant 3$. +If $q=3, d=2$ and $p \geqslant 13$ then $v_{q}(b!) \geqslant v_{q}(p!) \geqslant v_{q}(13!)=5>2 d$. Either way, $d \leqslant 1$. +If $p>2 q+1$ (so $p>3 q$, as $q \mid p-1$ ) then + +$$ +v_{q}(b!) \geqslant v_{q}((3 q)!)=3, +$$ + +so we must have $q \geqslant \frac{p}{2}$, in other words, $p-1=2 q$. This implies that $p=2^{k}-1$ and $q=2^{k-1}-1$ are both prime, but it is not possible to have two consecutive Mersenne primes. + +Solution 4. Let $a=p, b>p$ and $p \geqslant 5$ (the remaining cases are dealt with as in solution 3). Modulo $(p+1)^{2}$ it holds that +$p^{p}-p=(p+1-1)^{p}-p \equiv\binom{p}{1}(p+1)(-1)^{p-1}+(-1)^{p}-p=p(p+1)-1-p=p^{2}-1 \not \equiv 0 \quad \bmod \left((p+1)^{2}\right)$. +Since $p \geqslant 5$, the numbers 2 and $\frac{p+1}{2}$ are distinct and less than or equal to $p$. Therefore, $p+1 \mid p$, and so $(p+1)^{2} \mid(p+1)$ !. +But $b \geqslant p+1$, so $b!\equiv 0 \not \equiv p^{p}-p \bmod (p+1)^{2}$, a contradiction. + +N5. For each $1 \leqslant i \leqslant 9$ and $T \in \mathbb{N}$, define $d_{i}(T)$ to be the total number of times the digit $i$ appears when all the multiples of 1829 between 1 and $T$ inclusive are written out in base 10. + +Show that there are infinitely many $T \in \mathbb{N}$ such that there are precisely two distinct values among $d_{1}(T), d_{2}(T), \ldots, d_{9}(T)$. +(United Kingdom) +Solution. Let $n:=1829$. First, we choose some $k$ such that $n \mid 10^{k}-1$. For instance, any multiple of $\varphi(n)$ would work since $n$ is coprime to 10 . We will show that either $T=10^{k}-1$ or $T=10^{k}-2$ has the desired property, which completes the proof since $k$ can be taken to be arbitrary large. + +For this it suffices to show that $\#\left\{d_{i}\left(10^{k}-1\right): 1 \leqslant i \leqslant 9\right\} \leqslant 2$. Indeed, if + +$$ +\#\left\{d_{i}\left(10^{k}-1\right): 1 \leqslant i \leqslant 9\right\}=1 +$$ + +then, since $10^{k}-1$ which consists of all nines is a multiple of $n$, we have + +$$ +d_{i}\left(10^{k}-2\right)=d_{i}\left(10^{k}-1\right) \text { for } i \in\{1, \ldots, 8\}, \text { and } d_{9}\left(10^{k}-2\right)0$ independent of the set $Q$ such that for any positive integer $N>100$, the number of special integers in $[1, N]$ is at least $c N$. +(For example, if $Q=\{3,7\}$, then $p(42)=3, q(42)=2, p(63)=3, q(63)=3, p(2022)=3$, $q(2022)=1$.) +(Costa Rica) +Solution. Let us call two positive integers $m, n$ friends if $p(m)+p(n)$ and $q(m)+q(n)$ are both even integers. We start by noting that the pairs $(p(k), q(k))$ modulo 2 can take at most 4 different values; thus, among any five different positive integers there are two which are friends. + +In addition, both functions $p$ and $q$ satisfy $f(a b)=f(a)+f(b)$ for any $a, b$. Therefore, if $m$ and $n$ are divisible by $d$, then both $p$ and $q$ satisfy the equality $f(m)+f(n)=f(m / d)+$ $f(n / d)+2 f(d)$. This implies that $m, n$ are friends if and only if $m / d, n / d$ are friends. + +Let us call a set of integers $\left\{n_{1}, n_{2}, \ldots, n_{5}\right\}$ an interesting set if for any indexes $i, j$, the difference $d_{i j}=\left|n_{i}-n_{j}\right|$ divides both $n_{i}$ and $n_{j}$. We claim that if elements of an interesting set are all positive, then we can obtain a special integer. Indeed, if we were able to construct such a set, then there would be a pair of integers $\left\{n_{i}, n_{j}\right\}$ which are friends, according to the first observation. Additionally, the second observation yields that the quotients $n_{i} / d_{i j}, n_{j} / d_{i j}$ form a pair of friends, which happen to be consecutive integers, thus giving a special integer as desired. + +In order to construct a family of interesting sets, we can start by observing that the set $\{0,6,8,9,12\}$ is an interesting set. Using that $72=2^{3} \cdot 3^{2}$ is the least common multiple of all pairwise differences in this set, we obtain a family of interesting sets by considering + +$$ +\{72 k, 72 k+6,72 k+8,72 k+9,72 k+12\} +$$ + +for any $k \geqslant 1$. If we consider the quotients (of these numbers by the appropriate differences), then we obtain that the set + +$$ +S_{k}=\{6 k, 8 k, 9 k, 12 k, 12 k+1,18 k+2,24 k+2,24 k+3,36 k+3,72 k+8\}, +$$ + +has at least one special integer. In particular, the interval $[1,100 k]$ contains the sets $S_{1}, S_{2}, \ldots, S_{k}$, each of which has a special number. Any special number can be contained in at most ten sets $S_{k}$, from where we conclude that the number of special integers in $[1,100 k]$ is at least $k / 10$. + +Finally, let $N=100 k+r$, with $k \geqslant 1$ and $0 \leqslant r<100$, so that we have $N<100(k+1) \leqslant$ $200 k$. Then the number of special integers in $[1, N]$ is at least $k / 10>N / 2000$, as we wanted to prove. + +Comment 1. The statement is also true for $N \geqslant 15$ as at least one of the numbers $7,14,15$ is special. +Comment 2. Another approach would be to note that if $p(2 n), p(2 n+1), p(2 n+2)$ all have the same parity then one of the numbers $n, 2 n, 2 n+1$ is special. Indeed, if $q(n)+q(n+1)$ is even then $n$ is special since $p(n)+p(n+1) \equiv p(2 n)+p(2 n+2) \equiv 0(\bmod 2)$. Otherwise, if $q(n)+q(n+1)$ is odd, so is $q(2 n)+q(2 n+2)$ which implies that exactly one of the numbers $2 n, 2 n+1$ is special. + +Unfortunately, it seems hard to show that the set of such $n$ has positive density: see a recent paper https://arxiv.org/abs/1509.01545 for the proof that all eight patterns of the parities of $p(n), p(n+1), p(n+2)$ appear for a positive proportion of positive integers. + +This page is intentionally left blank + +N7. Let $k$ be a positive integer and let $S$ be a finite set of odd prime numbers. Prove that there is at most one way (modulo rotation and reflection) to place the elements of $S$ around a circle such that the product of any two neighbors is of the form $x^{2}+x+k$ for some positive integer $x$. + +Solution. Let us allow the value $x=0$ as well; we prove the same statement under this more general constraint. Obviously that implies the statement with the original conditions. + +Call a pair $\{p, q\}$ of primes with $p \neq q$ special if $p q=x^{2}+x+k$ for some nonnegative integer $x$. The following claim is the key mechanism of the problem: +Claim. +(a) For every prime $r$, there are at most two primes less than $r$ forming a special pair with $r$. +(b) If such $p$ and $q$ exist, then $\{p, q\}$ is itself special. + +We present two proofs of the claim. +Proof 1. We are interested in integers $1 \leqslant x\sqrt{m}$ choices for $x$ and $y$, so there are more than $m$ possible pairs $(x, y)$. Hence, two of these sums are congruent modulo $m$ : $5^{k+1} x_{1}+3^{k+1} y_{1} \equiv 5^{k+1} x_{2}+3^{k+1} y_{2}(\bmod m)$. + +Now choose $a=x_{1}-x_{2}$ and $b=y_{1}-y_{2}$; at least one of $a, b$ is nonzero, and + +$$ +5^{k+1} a+3^{k+1} b \equiv 0 \quad(\bmod m), \quad|a|,|b| \leqslant \sqrt{m} +$$ + +From + +$$ +0 \equiv\left(5^{k+1} a\right)^{2}-\left(3^{k+1} b\right)^{2}=5^{n+1} a^{2}-3^{n+1} b^{2} \equiv 5 \cdot 3^{n} a^{2}-3^{n+1} b^{2}=3^{n}\left(5 a^{2}-3 b^{2}\right) \quad(\bmod m) +$$ + +we can see that $\left|5 a^{2}-3 b^{2}\right|$ is a multiple of $m$. Since at least one of $a$ and $b$ is nonzero, $5 a^{2} \neq 3 b^{2}$. Hence, by the choice of $a, b$, we have $0<\left|5 a^{2}-3 b^{2}\right| \leqslant \max \left(5 a^{2}, 3 b^{2}\right) \leqslant 5 m$. That shows that $m_{1} \leqslant 5 m$. +II. Next, we show that $m_{1}$ cannot be divisible by 2,3 and 5 . Since $m_{1}$ equals either $\left|5 a^{2}-3 b^{2}\right|$ or $\left|a^{2}-15 b^{2}\right|$ with some integers $a, b$, we have six cases to check. In all six cases, we will get a contradiction by presenting another multiple of $m$, smaller than $m_{1}$. + +- If $5 \mid m_{1}$ and $m_{1}=\left|5 a^{2}-3 b^{2}\right|$, then $5 \mid b$ and $\left|a^{2}-15\left(\frac{b}{5}\right)^{2}\right|=\frac{m_{1}}{5}1$ and $c$ are integers, and $X, Y$ are positive integers such that $X \leqslant mm$, then $c x^{2} \equiv d y^{2}(\bmod m)$ has a solution such that at least one of $x, y$ is nonzero, $|x|5$ of $5 a^{2}-3 b^{2}$ or $a^{2}-15 b^{2}$, we have + +$$ +1=\left(\frac{15}{p}\right)=\left(\frac{3}{p}\right)\left(\frac{5}{p}\right)=(-1)^{\frac{p-1}{2}}\left(\frac{p}{3}\right)\left(\frac{p}{5}\right) +$$ + +where $\left(\frac{a}{p}\right)$ stands for the Legendre symbol. Considering the remainders of $p$ when divided by 4,3 and 5, (1) leads to + +$$ +p \equiv \pm 1, \pm 7, \pm 11 \text { or } \pm 17 \bmod 60 +$$ + +These remainders form a subgroup of the reduced remainders modulo 60. Since 13 and 37 are not elements in this subgroup, the number $m=2^{n}+65$ cannot be a product of such primes. + +Instead of handling the prime divisors of $m$ separately, we can use Jacobi symbols for further simplification, as shown in the next solution. + +Solution 2. Suppose again that $5^{n} \equiv 3^{n}\left(\bmod m=2^{n}+65\right)$. Like in the first solution, we conclude that $n$ must be odd, and $n \geqslant 3$, so $8 \mid 2^{n}$. + +Using Jacobi symbols, + +$$ +-1=\left(\frac{2^{n}+65}{5}\right)=\left(\frac{5}{2^{n}+65}\right)=\left(\frac{5^{n}}{2^{n}+65}\right)=\left(\frac{3^{n}}{2^{n}+65}\right)=\left(\frac{3}{2^{n}+65}\right)=\left(\frac{2^{n}+65}{3}\right)=1 +$$ + +contradiction. + +## - + +![](https://cdn.mathpix.com/cropped/2024_11_18_86efb4389f14b52b08e1g-75.jpg?height=98&width=726&top_left_y=279&top_left_x=114) + ++ - + +\$4=4 \times 1.20\$1 +![](https://cdn.mathpix.com/cropped/2024_11_18_86efb4389f14b52b08e1g-75.jpg?height=84&width=889&top_left_y=1990&top_left_x=379) +Cly +D - +1 vaiv +![](https://cdn.mathpix.com/cropped/2024_11_18_86efb4389f14b52b08e1g-75.jpg?height=58&width=116&top_left_y=2247&top_left_x=536) +1N ![](https://cdn.mathpix.com/cropped/2024_11_18_86efb4389f14b52b08e1g-75.jpg?height=135&width=646&top_left_y=2301&top_left_x=348)![](https://cdn.mathpix.com/cropped/2024_11_18_86efb4389f14b52b08e1g-75.jpg?height=1874&width=164&top_left_y=419&top_left_x=1607) +![](https://cdn.mathpix.com/cropped/2024_11_18_86efb4389f14b52b08e1g-75.jpg?height=130&width=432&top_left_y=1647&top_left_x=0) +r / A M - 18 +![](https://cdn.mathpix.com/cropped/2024_11_18_86efb4389f14b52b08e1g-75.jpg?height=73&width=652&top_left_y=2395&top_left_x=139) +![](https://cdn.mathpix.com/cropped/2024_11_18_86efb4389f14b52b08e1g-75.jpg?height=58&width=637&top_left_y=2447&top_left_x=154) +A ..... ![](https://cdn.mathpix.com/cropped/2024_11_18_86efb4389f14b52b08e1g-75.jpg?height=75&width=415&top_left_y=2501&top_left_x=148) +![](https://cdn.mathpix.com/cropped/2024_11_18_86efb4389f14b52b08e1g-75.jpg?height=61&width=452&top_left_y=2539&top_left_x=605) +![](https://cdn.mathpix.com/cropped/2024_11_18_86efb4389f14b52b08e1g-75.jpg?height=75&width=532&top_left_y=2587&top_left_x=599) + diff --git a/IMO/md/en-compendium.md b/IMO/md/en-compendium.md new file mode 100644 index 0000000000000000000000000000000000000000..8d472ad3534294feb9756aaecbadf3833071c8f7 --- /dev/null +++ b/IMO/md/en-compendium.md @@ -0,0 +1,19482 @@ +Dušan Djukić
Vladimir Janković
Ivan Matic
Nikola Petrović + +# The IMO Compendium + +A Collection of Problems Suggested for the International Mathematical Olympiads: + +1959-2004 + +With 200 Figures + +## Preface + +The International Mathematical Olympiad (IMO) is nearing its fiftieth anniversary and has already created a very rich legacy and firmly established itself as the most prestigious mathematical competition in which a high-school student could aspire to participate. Apart from the opportunity to tackle interesting and very challenging mathematical problems, the IMO represents a great opportunity for high-school students to see how they measure up against students from the rest of the world. Perhaps even more importantly, it is an opportunity to make friends and socialize with students who have similar interests, possibly even to become acquainted with their future colleagues on this first leg of their journey into the world of professional and scientific mathematics. Above all, however pleasing or disappointing the final score may be, preparing for an IMO and participating in one is an adventure that will undoubtedly linger in one's memory for the rest of one's life. It is to the high-school-aged aspiring mathematician and IMO participant that we devote this entire book. + +The goal of this book is to include all problems ever shortlisted for the IMOs in a single volume. Up to this point, only scattered manuscripts traded among different teams have been available, and a number of manuscripts were lost for many years or unavailable to many. + +In this book, all manuscripts have been collected into a single compendium of mathematics problems of the kind that usually appear on the IMOs. Therefore, we believe that this book will be the definitive and authoritative source for high-school students preparing for the IMO, and we suspect that it will be of particular benefit in countries lacking adequate preparation literature. A high-school student could spend an enjoyable year going through the numerous problems and novel ideas presented in the solutions and emerge ready to tackle even the most difficult problems on an IMO. In addition, the skill acquired in the process of successfully attacking difficult mathematics problems will prove to be invaluable in a serious and prosperous career in mathematics. + +However, we must caution our aspiring IMO participant on the use of this book. Any book of problems, no matter how large, quickly depletes itself if +the reader merely glances at a problem and then five minutes later, having determined that the problem seems unsolvable, glances at the solution. + +The authors therefore propose the following plan for working through the book. Each problem is to be attempted at least half an hour before the reader looks at the solution. The reader is strongly encouraged to keep trying to solve the problem without looking at the solution as long as he or she is coming up with fresh ideas and possibilities for solving the problem. Only after all venues seem to have been exhausted is the reader to look at the solution, and then only in order to study it in close detail, carefully noting any previously unseen ideas or methods used. To condense the subject matter of this already very large book, most solutions have been streamlined, omitting obvious derivations and algebraic manipulations. Thus, reading the solutions requires a certain mathematical maturity, and in any case, the solutions, especially in geometry, are intended to be followed through with pencil and paper, the reader filling in all the omitted details. We highly recommend that the reader mark such unsolved problems and return to them in a few months to see whether they can be solved this time without looking at the solutions. We believe this to be the most efficient and systematic way (as with any book of problems) to raise one's level of skill and mathematical maturity. + +We now leave our reader with final words of encouragement to persist in this journey even when the difficulties seem insurmountable and a sincere wish to the reader for all mathematical success one can hope to aspire to. + +Belgrade, October 2004 + +Dušan Djukić +Vladimir Janković +Ivan Matić +Nikola Petrović + +For the most current information regarding The IMO Compendium you are invited to go to our website: www.imo.org.yu. At this site you can also find, for several of the years, scanned versions of available original shortlist and longlist problems, which should give an illustration of the original state the IMO materials we used were in. + +We are aware that this book may still contain errors. If you find any, please notify us at imo@matf.bg.ac.yu. A full list of discovered errors can be found at our website. If you have any questions, comments, or suggestions regarding both our book and our website, please do not hesitate to write to us at the above email address. We would be more than happy to hear from you. + +## Acknowledgements + +The making of this book would have never been possible without the help of numerous individuals, whom we wish to thank. + +First and foremost, obtaining manuscripts containing suggestions for IMOs was vital in order for us to provide the most complete listing of problems possible. We obtained manuscripts for many of the years from the former and current IMO team leaders of Yugoslavia / Serbia and Montenegro, who carefully preserved these valuable papers throughout the years. Special thanks are due to Prof. Vladimir Mićić, for some of the oldest manuscripts, and to Prof. Zoran Kadelburg. We also thank Prof. Djordje Dugošija and Prof. Pavle Mladenović. In collecting shortlisted and longlisted problems we were also assisted by Prof. Ioan Tomescu from Romania and Hà Duy Hưng from Vietnam. + +A lot of work was invested in cleaning up our giant manuscript of errors. Special thanks in this respect go to David Kramer, our copy-editor, and to Prof. Titu Andreescu and his group for checking, in great detail, the validity of the solutions in this manuscript, and for their proposed corrections and alternative solutions to several problems. We also thank Prof. Abderrahim Ouardini from France for sending us the list of countries of origin for the shortlisted problems of 1998, Prof. Dorin Andrica for helping us compile the list of books for reference, and Prof. Ljubomir Čukić for proofreading part of the manuscript and helping us correct several errors. + +We would also like to express our thanks to all anonymous authors of the IMO problems. It is a pity that authors' names are not registered together with their proposed problems. Without them, the IMO would obviously not be what it is today. In many cases, the original solutions of the authors were used, and we duly acknowledge this immense contribution to our book, though once again, we regret that we cannot do this individually. In the same vein, we also thank all the students participating in the IMOs, since we have also included some of their original solutions in this book. + +The illustrations of geometry problems were done in WinGCLC, a program created by Prof. Predrag Janičić. This program is specifically designed for creating geometric pictures of unparalleled complexity quickly and efficiently. Even though it is still in its testing phase, its capabilities and utility are already remarkable and worthy of highest compliment. + +Finally, we would like to thank our families for all their love and support during the making of this book. + +## Contents + +Preface ..... v +1 Introduction ..... 1 +1.1 The International Mathematical Olympiad ..... 1 +1.2 The IMO Compendium ..... 2 +2 Basic Concepts and Facts ..... 5 +2.1 Algebra ..... 5 +2.1.1 Polynomials ..... 5 +2.1.2 Recurrence Relations ..... 6 +2.1.3 Inequalities ..... 7 +2.1.4 Groups and Fields ..... 9 +2.2 Analysis ..... 10 +2.3 Geometry ..... 12 +2.3.1 Triangle Geometry ..... 12 +2.3.2 Vectors in Geometry ..... 13 +2.3.3 Barycenters ..... 14 +2.3.4 Quadrilaterals ..... 14 +2.3.5 Circle Geometry ..... 15 +2.3.6 Inversion ..... 16 +2.3.7 Geometric Inequalities ..... 16 +2.3.8 Trigonometry ..... 17 +2.3.9 Formulas in Geometry ..... 18 +2.4 Number Theory ..... 19 +2.4.1 Divisibility and Congruences ..... 19 +2.4.2 Exponential Congruences ..... 20 +2.4.3 Quadratic Diophantine Equations ..... 21 +2.4.4 Farey Sequences ..... 22 +2.5 Combinatorics ..... 22 +2.5.1 Counting of Objects ..... 22 +2.5.2 Graph Theory ..... 23 +3 Problems ..... 27 +3.1 IMO 1959 ..... 27 +3.1.1 Contest Problems ..... 27 +3.2 IMO 1960 ..... 29 +3.2.1 Contest Problems ..... 29 +3.3 IMO 1961 ..... 30 +3.3.1 Contest Problems ..... 30 +3.4 IMO 1962 ..... 31 +3.4.1 Contest Problems ..... 31 +3.5 IMO 1963 ..... 32 +3.5.1 Contest Problems ..... 32 +3.6 IMO 1964 ..... 33 +3.6.1 Contest Problems ..... 33 +3.7 IMO 1965 ..... 34 +3.7.1 Contest Problems ..... 34 +3.8 IMO 1966 ..... 35 +3.8.1 Contest Problems ..... 35 +3.8.2 Some Longlisted Problems 1959-1966 ..... 36 +3.9 IMO 1967 ..... 42 +3.9.1 Contest Problems ..... 42 +3.9.2 Longlisted Problems ..... 42 +3.10 IMO 1968 ..... 51 +3.10.1 Contest Problems ..... 51 +3.10.2 Shortlisted Problems ..... 52 +3.11 IMO 1969 ..... 55 +3.11.1 Contest Problems ..... 55 +3.11.2 Longlisted Problems ..... 55 +3.12 IMO 1970 ..... 64 +3.12.1 Contest Problems ..... 64 +3.12.2 Longlisted Problems ..... 65 +3.12.3 Shortlisted Problems ..... 72 +3.13 IMO 1971 ..... 74 +3.13.1 Contest Problems ..... 74 +3.13.2 Longlisted Problems ..... 75 +3.13.3 Shortlisted Problems ..... 81 +3.14 IMO 1972 ..... 84 +3.14.1 Contest Problems ..... 84 +3.14.2 Longlisted Problems ..... 84 +3.14.3 Shortlisted Problems ..... 89 +3.15 IMO 1973 ..... 91 +3.15.1 Contest Problems ..... 91 +3.15.2 Shortlisted Problems ..... 92 +3.16 IMO 1974 ..... 94 +3.16.1 Contest Problems ..... 94 +3.16.2 Longlisted Problems ..... 95 +3.16.3 Shortlisted Problems ..... 100 +3.17 IMO 1975 ..... 103 +3.17.1 Contest Problems ..... 103 +3.17.2 Shortlisted Problems ..... 103 +3.18 IMO 1976 ..... 106 +3.18.1 Contest Problems ..... 106 +3.18.2 Longlisted Problems ..... 106 +3.18.3 Shortlisted Problems ..... 112 +3.19 IMO 1977 ..... 114 +3.19.1 Contest Problems ..... 114 +3.19.2 Longlisted Problems ..... 114 +3.19.3 Shortlisted Problems ..... 120 +3.20 IMO 1978 ..... 123 +3.20.1 Contest Problems ..... 123 +3.20.2 Longlisted Problems ..... 123 +3.20.3 Shortlisted Problems ..... 128 +3.21 IMO 1979 ..... 131 +3.21.1 Contest Problems ..... 131 +3.21.2 Longlisted Problems ..... 132 +3.21.3 Shortlisted Problems ..... 139 +3.22 IMO 1981 ..... 143 +3.22.1 Contest Problems ..... 143 +3.22.2 Shortlisted Problems ..... 144 +3.23 IMO 1982 ..... 147 +3.23.1 Contest Problems ..... 147 +3.23.2 Longlisted Problems ..... 148 +3.23.3 Shortlisted Problems ..... 153 +3.24 IMO 1983 ..... 157 +3.24.1 Contest Problems ..... 157 +3.24.2 Longlisted Problems ..... 157 +3.24.3 Shortlisted Problems ..... 165 +3.25 IMO 1984 ..... 169 +3.25.1 Contest Problems ..... 169 +3.25.2 Longlisted Problems ..... 169 +3.25.3 Shortlisted Problems ..... 176 +3.26 IMO 1985 ..... 180 +3.26.1 Contest Problems ..... 180 +3.26.2 Longlisted Problems ..... 180 +3.26.3 Shortlisted Problems ..... 190 +3.27 IMO 1986 ..... 193 +3.27.1 Contest Problems ..... 193 +3.27.2 Longlisted Problems ..... 194 +3.27.3 Shortlisted Problems ..... 201 +3.28 IMO 1987 ..... 204 +3.28.1 Contest Problems ..... 204 +3.28.2 Longlisted Problems ..... 204 +3.28.3 Shortlisted Problems ..... 212 +3.29 IMO 1988 ..... 216 +3.29.1 Contest Problems ..... 216 +3.29.2 Longlisted Problems ..... 217 +3.29.3 Shortlisted Problems ..... 226 +3.30 IMO 1989 ..... 231 +3.30.1 Contest Problems ..... 231 +3.30.2 Longlisted Problems ..... 232 +3.30.3 Shortlisted Problems ..... 244 +3.31 IMO 1990 ..... 249 +3.31.1 Contest Problems ..... 249 +3.31.2 Shortlisted Problems ..... 250 +3.32 IMO 1991 ..... 254 +3.32.1 Contest Problems ..... 254 +3.32.2 Shortlisted Problems ..... 254 +3.33 IMO 1992 ..... 259 +3.33.1 Contest Problems ..... 259 +3.33.2 Longlisted Problems ..... 259 +3.33.3 Shortlisted Problems ..... 269 +3.34 IMO 1993 ..... 272 +3.34.1 Contest Problems ..... 272 +3.34.2 Shortlisted Problems ..... 273 +3.35 IMO 1994 ..... 277 +3.35.1 Contest Problems ..... 277 +3.35.2 Shortlisted Problems ..... 277 +3.36 IMO 1995 ..... 281 +3.36.1 Contest Problems ..... 281 +3.36.2 Shortlisted Problems ..... 281 +3.37 IMO 1996 ..... 286 +3.37.1 Contest Problems ..... 286 +3.37.2 Shortlisted Problems ..... 287 +3.38 IMO 1997 ..... 292 +3.38.1 Contest Problems ..... 292 +3.38.2 Shortlisted Problems ..... 293 +3.39 IMO 1998 ..... 297 +3.39.1 Contest Problems ..... 297 +3.39.2 Shortlisted Problems ..... 297 +3.40 IMO 1999 ..... 302 +3.40.1 Contest Problems ..... 302 +3.40.2 Shortlisted Problems ..... 302 +3.41 IMO 2000 ..... 307 +3.41.1 Contest Problems ..... 307 +3.41.2 Shortlisted Problems ..... 308 +3.42 IMO 2001 ..... 312 +3.42.1 Contest Problems ..... 312 +3.42.2 Shortlisted Problems ..... 312 +3.43 IMO 2002 ..... 317 +3.43.1 Contest Problems ..... 317 +3.43.2 Shortlisted Problems ..... 318 +3.44 IMO 2003 ..... 322 +3.44.1 Contest Problems ..... 322 +3.44.2 Shortlisted Problems ..... 323 +3.45 IMO 2004 ..... 327 +3.45.1 Contest Problems ..... 327 +3.45.2 Shortlisted Problems ..... 328 +4 Solutions ..... 333 +4.1 Contest Problems 1959 ..... 333 +4.2 Contest Problems 1960 ..... 335 +4.3 Contest Problems 1961 ..... 337 +4.4 Contest Problems 1962 ..... 339 +4.5 Contest Problems 1963 ..... 340 +4.6 Contest Problems 1964 ..... 341 +4.7 Contest Problems 1965 ..... 343 +4.8 Contest Problems 1966 ..... 345 +4.9 Longlisted Problems 1967 ..... 347 +4.10 Shortlisted Problems 1968 ..... 361 +4.11 Contest Problems 1969 ..... 367 +4.12 Shortlisted Problems 1970 ..... 370 +4.13 Shortlisted Problems 1971 ..... 377 +4.14 Shortlisted Problems 1972 ..... 384 +4.15 Shortlisted Problems 1973 ..... 389 +4.16 Shortlisted Problems 1974 ..... 395 +4.17 Shortlisted Problems 1975 ..... 401 +4.18 Shortlisted Problems 1976 ..... 406 +4.19 Longlisted Problems 1977 ..... 410 +4.20 Shortlisted Problems 1978 ..... 426 +4.21 Shortlisted Problems 1979 ..... 434 +4.22 Shortlisted Problems 1981 ..... 442 +4.23 Shortlisted Problems 1982 ..... 451 +4.24 Shortlisted Problems 1983 ..... 457 +4.25 Shortlisted Problems 1984 ..... 466 +4.26 Shortlisted Problems 1985 ..... 473 +4.27 Shortlisted Problems 1986 ..... 481 +4.28 Shortlisted Problems 1987 ..... 489 +4.29 Shortlisted Problems 1988 ..... 500 +4.30 Shortlisted Problems 1989 ..... 516 +4.31 Shortlisted Problems 1990 ..... 530 +4.32 Shortlisted Problems 1991 ..... 544 +4.33 Shortlisted Problems 1992 ..... 558 +4.34 Shortlisted Problems 1993 ..... 568 +4.35 Shortlisted Problems 1994 ..... 581 +4.36 Shortlisted Problems 1995 ..... 589 +4.37 Shortlisted Problems 1996 ..... 602 +4.38 Shortlisted Problems 1997 ..... 618 +4.39 Shortlisted Problems 1998 ..... 632 +4.40 Shortlisted Problems 1999 ..... 646 +4.41 Shortlisted Problems 2000 ..... 661 +4.42 Shortlisted Problems 2001 ..... 675 +4.43 Shortlisted Problems 2002 ..... 689 +4.44 Shortlisted Problems 2003 ..... 701 +4.45 Shortlisted Problems 2004 ..... 715 +A Notation and Abbreviations ..... 731 +A. 1 Notation ..... 731 +A. 2 Abbreviations ..... 732 +B Codes of the Countries of Origin ..... 735 +References ..... 737 + +## Introduction + +### 1.1 The International Mathematical Olympiad + +The International Mathematical Olympiad (IMO) is the most important and prestigious mathematical competition for high-school students. It has played a significant role in generating wide interest in mathematics among high school students, as well as identifying talent. + +In the beginning, the IMO was a much smaller competition than it is today. In 1959, the following seven countries gathered to compete in the first IMO: Bulgaria, Czechoslovakia, German Democratic Republic, Hungary, Poland, Romania, and the Soviet Union. Since then, the competition has been held annually. Gradually, other Eastern-block countries, countries from Western Europe, and ultimately numerous countries from around the world and every continent joined in. (The only year in which the IMO was not held was 1980, when for financial reasons no one stepped in to host it. Today this is hardly a problem, and hosts are lined up several years in advance.) In the 45th IMO, held in Athens, no fewer than 85 countries took part. + +The format of the competition quickly became stable and unchanging. Each country may send up to six contestants and each contestant competes individually (without any help or collaboration). The country also sends a team leader, who participates in problem selection and is thus isolated from the rest of the team until the end of the competition, and a deputy leader, who looks after the contestants. + +The IMO competition lasts two days. On each day students are given four and a half hours to solve three problems, for a total of six problems. The first problem is usually the easiest on each day and the last problem the hardest, though there have been many notable exceptions. ((IMO96-5) is one of the most difficult problems from all the Olympiads, having been fully solved by only six students out of several hundred!) Each problem is worth 7 points, making 42 points the maximum possible score. The number of points obtained by a contestant on each problem is the result of intense negotiations and, ultimately, agreement among the problem coordinators, assigned by the +host country, and the team leader and deputy, who defend the interests of their contestants. This system ensures a relatively objective grade that is seldom off by more than two or three points. + +Though countries naturally compare each other's scores, only individual prizes, namely medals and honorable mentions, are awarded on the IMO. Fewer than one twelfth of participants are awarded the gold medal, fewer than one fourth are awarded the gold or silver medal, and fewer than one half are awarded the gold, silver or bronze medal. Among the students not awarded a medal, those who score 7 points on at least one problem are awarded an honorable mention. This system of determining awards works rather well. It ensures, on the one hand, strict criteria and appropriate recognition for each level of performance, giving every contestant something to strive for. On the other hand, it also ensures a good degree of generosity that does not greatly depend on the variable difficulty of the problems proposed. + +According to the statistics, the hardest Olympiad was that in 1971, followed by those in 1996, 1993, and 1999. The Olympiad in which the winning team received the lowest score was that in 1977, followed by those in 1960 and 1999. + +The selection of the problems consists of several steps. Participant countries send their proposals, which are supposed to be novel, to the IMO organizers. The organizing country does not propose problems. From the received proposals (the longlisted problems), the problem committee selects a shorter list (the shortlisted problems), which is presented to the IMO jury, consisting of all the team leaders. From the short-listed problems the jury chooses six problems for the IMO. + +Apart from its mathematical and competitive side, the IMO is also a very large social event. After their work is done, the students have three days to enjoy events and excursions organized by the host country, as well as to interact and socialize with IMO participants from around the world. All this makes for a truly memorable experience. + +### 1.2 The IMO Compendium + +Olympiad problems have been published in many books [65]. However, the remaining shortlisted and longlisted problems have not been systematically collected and published, and therefore many of them are unknown to mathematicians interested in this subject. Some partial collections of shortlisted and longlisted problems can be found in the references, though usually only for one year. References [1], [30], [41], [60] contain problems from multiple years. In total, these books cover roughly $50 \%$ of the problems found in this book. + +The goal of this book is to present, in a single volume, our comprehensive collection of problems proposed for the IMO. It consists of all problems selected for the IMO competitions, shortlisted problems from the 10th IMO +and from the 12th through 44th IMOs, and longlisted problems from nineteen IMOs. We do not have shortlisted problems from the 9th and the 11th IMOs, and we could not discover whether competition problems at those two IMOs were selected from the longlisted problems or whether there existed shortlisted problems that have not been preserved. Since IMO organizers usually do not distribute longlisted problems to the representatives of participant countries, our collection is incomplete. The practice of distributing these longlists effectively ended in 1989. A selection of problems from the first eight IMOs has been taken from [60]. + +The book is organized as follows. For each year, the problems that were given on the IMO contest are presented, along with the longlisted and/or shortlisted problems, if applicable. We present solutions to all shortlisted problems. The problems appearing on the IMOs are solved among the other shortlisted problems. The longlisted problems have not been provided with solutions, except for the two IMOs held in Yugoslavia (for patriotic reasons), since that would have made the book unreasonably long. This book has thus the added benefit for professors and team coaches of being a suitable book from which to assign problems. For each problem, we indicate the country that proposed it with a three-letter code. A complete list of country codes and the corresponding countries is given in the appendix. In all shortlists, we also indicate which problems were selected for the contest. We occasionally make references in our solutions to other problems in a straightforward way. After indicating with LL, SL, or IMO whether the problem is from a longlist, shortlist, or contest, we indicate the year of the IMO and then the number of the problem. For example, (SL89-15) refers to the fifteenth problem of the shortlist of 1989. + +We also present a rough list of all formulas and theorems not obviously derivable that were called upon in our proofs. Since we were largely concerned with only the theorems used in proving the problems of this book, we believe that the list is a good compilation of the most useful theorems for IMO problem solving. + +The gathering of such a large collection of problems into a book required a massive amount of editing. We reformulated the problems whose original formulations were not precise or clear. We translated the problems that were not in English. Some of the solutions are taken from the author of the problem or other sources, while others are original solutions of the authors of this book. Many of the non-original solutions were significantly edited before being included. We do not make any guarantee that the problems in this book fully correspond to the actual shortlisted or longlisted problems. However, we believe this book to be the closest possible approximation to such a list. + +## Basic Concepts and Facts + +The following is a list of the most basic concepts and theorems frequently used in this book. We encourage the reader to become familiar with them and perhaps read up on them further in other literature. + +### 2.1 Algebra + +### 2.1.1 Polynomials + +Theorem 2.1. The quadratic equation $a x^{2}+b x+c=0(a, b, c \in \mathbb{R}, a \neq 0)$ has solutions + +$$ +x_{1,2}=\frac{-b \pm \sqrt{b^{2}-4 a c}}{2 a} . +$$ + +The discriminant $D$ of the quadratic equation is defined as $D=b^{2}-4 a c$. For $D<0$ the solutions are complex and conjugate to each other, for $D=0$ the solutions degenerate to one real solution, and for $D>0$ the equation has two distinct real solutions. + +Definition 2.2. Binomial coefficients $\binom{n}{k}, n, k \in \mathbb{N}_{0}, k \leq n$, are defined as + +$$ +\binom{n}{i}=\frac{n!}{i!(n-i)!} . +$$ + +They satisfy $\binom{n}{i}+\binom{n}{i-1}=\binom{n+1}{i}$ for $i>0$ and also $\binom{n}{0}+\binom{n}{1}+\cdots+\binom{n}{n}=2^{n}$, $\binom{n}{0}-\binom{n}{1}+\cdots+(-1)^{n}\binom{n}{n}=0,\binom{n+m}{k}=\sum_{i=0}^{k}\binom{n}{i}\binom{m}{k-i}$. + +Theorem 2.3 ((Newton's) binomial formula). For $x, y \in \mathbb{C}$ and $n \in \mathbb{N}$, + +$$ +(x+y)^{n}=\sum_{i=0}^{n}\binom{n}{i} x^{n-i} y^{i} . +$$ + +Theorem 2.4 (Bézout's theorem). A polynomial $P(x)$ is divisible by the binomial $x-a(a \in \mathbb{C})$ if and only if $P(a)=0$. + +Theorem 2.5 (The rational root theorem). If $x=p / q$ is a rational zero of a polynomial $P(x)=a_{n} x^{n}+\cdots+a_{0}$ with integer coefficients and $(p, q)=1$, then $p \mid a_{0}$ and $q \mid a_{n}$. + +Theorem 2.6 (The fundamental theorem of algebra). Every nonconstant polynomial with coefficients in $\mathbb{C}$ has a complex root. + +Theorem 2.7 (Eisenstein's criterion (extended)). Let $P(x)=a_{n} x^{n}+$ $\cdots+a_{1} x+a_{0}$ be a polynomial with integer coefficients. If there exist a prime $p$ and an integer $k \in\{0,1, \ldots, n-1\}$ such that $p \mid a_{0}, a_{1}, \ldots, a_{k}, p \nmid a_{k+1}$, and $p^{2} \nmid a_{0}$, then there exists an irreducible factor $Q(x)$ of $P(x)$ whose degree is at least $k$. In particular, if $p$ can be chosen such that $k=n-1$, then $P(x)$ is irreducible. + +Definition 2.8. Symmetric polynomials in $x_{1}, \ldots, x_{n}$ are polynomials that do not change on permuting the variables $x_{1}, \ldots, x_{n}$. Elementary symmetric polynomials are $\sigma_{k}\left(x_{1}, \ldots, x_{n}\right)=\sum x_{i_{1}} \cdots x_{i_{k}}$ (the sum is over all $k$-element subsets $\left\{i_{1}, \ldots, i_{k}\right\}$ of $\left.\{1,2, \ldots, n\}\right)$. + +Theorem 2.9. Every symmetric polynomial in $x_{1}, \ldots, x_{n}$ can be expressed as a polynomial in the elementary symmetric polynomials $\sigma_{1}, \ldots, \sigma_{n}$. + +Theorem 2.10 (Vieta's formulas). Let $\alpha_{1}, \ldots, \alpha_{n}$ and $c_{1}, \ldots, c_{n}$ be complex numbers such that + +$$ +\left(x-\alpha_{1}\right)\left(x-\alpha_{2}\right) \cdots\left(x-\alpha_{n}\right)=x^{n}+c_{1} x^{n-1}+c_{2} x^{n-2}+\cdots+c_{n} +$$ + +Then $c_{k}=(-1)^{k} \sigma_{k}\left(\alpha_{1}, \ldots, \alpha_{n}\right)$ for $k=1,2, \ldots, n$. +Theorem 2.11 (Newton's formulas on symmetric polynomials). Let $\sigma_{k}=\sigma_{k}\left(x_{1}, \ldots, x_{n}\right)$ and let $s_{k}=x_{1}^{k}+x_{2}^{k}+\cdots+x_{n}^{k}$, where $x_{1}, \ldots, x_{n}$ are arbitrary complex numbers. Then + +$$ +k \sigma_{k}=s_{1} \sigma_{k-1}-s_{2} \sigma_{k-2}+\cdots+(-1)^{k} s_{k-1} \sigma_{1}+(-1)^{k-1} s_{k} +$$ + +### 2.1.2 Recurrence Relations + +Definition 2.12. A recurrence relation is a relation that determines the elements of a sequence $x_{n}, n \in \mathbb{N}_{0}$, as a function of previous elements. A recurrence relation of the form + +$$ +(\forall n \geq k) \quad x_{n}+a_{1} x_{n-1}+\cdots+a_{k} x_{n-k}=0 +$$ + +for constants $a_{1}, \ldots, a_{k}$ is called a linear homogeneous recurrence relation of order $k$. We define the characteristic polynomial of the relation as $P(x)=$ $x^{k}+a_{1} x^{k-1}+\cdots+a_{k}$. + +Theorem 2.13. Using the notation introduced in the above definition, let $P(x)$ factorize as $P(x)=\left(x-\alpha_{1}\right)^{k_{1}}\left(x-\alpha_{2}\right)^{k_{2}} \cdots\left(x-\alpha_{r}\right)^{k_{r}}$, where $\alpha_{1}, \ldots, \alpha_{r}$ are distinct complex numbers and $k_{1}, \ldots, k_{r}$ are positive integers. The general solution of this recurrence relation is in this case given by + +$$ +x_{n}=p_{1}(n) \alpha_{1}^{n}+p_{2}(n) \alpha_{2}^{n}+\cdots+p_{r}(n) \alpha_{r}^{n}, +$$ + +where $p_{i}$ is a polynomial of degree less than $k_{i}$. In particular, if $P(x)$ has $k$ distinct roots, then all $p_{i}$ are constant. + +If $x_{0}, \ldots, x_{k-1}$ are set, then the coefficients of the polynomials are uniquely determined. + +### 2.1.3 Inequalities + +Theorem 2.14. The quadratic function is always positive; i.e., $(\forall x \in \mathbb{R}) x^{2} \geq$ 0 . By substituting different expressions for $x$, many of the inequalities below are obtained. + +## Theorem 2.15 (Bernoulli's inequalities). + +1. If $n \geq 1$ is an integer and $x>-1$ a real number then $(1+x)^{n} \geq 1+n x$. +2. If $a>1$ or $a<0$ then for $x>-1$ the following inequality holds: $(1+x)^{\alpha} \geq$ $1+\alpha x$. +3. If $a \in(0,1)$ then for $x>-1$ the following inequality holds: $(1+x)^{\alpha} \leq$ $1+\alpha x$. + +Theorem 2.16 (The mean inequalities). For positive real numbers $x_{1}, x_{2}$, $\ldots, x_{n}$ it follows that $Q M \geq A M \geq G M \geq H M$, where + +$$ +\begin{aligned} +Q M & =\sqrt{\frac{x_{1}^{2}+\cdots+x_{n}^{2}}{n}}, & A M=\frac{x_{1}+\cdots+x_{n}}{n}, \\ +G M & =\sqrt[n]{x_{1} \cdots x_{n}}, & H M=\frac{n}{1 / x_{1}+\cdots+1 / x_{n}} . +\end{aligned} +$$ + +Each of these inequalities becomes an equality if and only if $x_{1}=x_{2}=$ $\cdots=x_{n}$. The numbers $Q M, A M, G M$, and $H M$ are respectively called the quadratic mean, the arithmetic mean, the geometric mean, and the harmonic mean of $x_{1}, x_{2}, \ldots, x_{n}$. + +Theorem 2.17 (The general mean inequality). Let $x_{1}, \ldots, x_{n}$ be positive real numbers. For each $p \in \mathbb{R}$ we define the mean of order $p$ of $x_{1}, \ldots, x_{n}$ by $M_{p}=\left(\frac{x_{1}^{p}+\cdots+x_{n}^{p}}{n}\right)^{1 / p}$ for $p \neq 0$, and $M_{q}=\lim _{p \rightarrow q} M_{p}$ for $q \in\{ \pm \infty, 0\}$. In particular, $\max x_{i}, Q M, A M, G M, H M$, and $\min x_{i}$ are $M_{\infty}, M_{2}, M_{1}, M_{0}$, $M_{-1}$, and $M_{-\infty}$ respectively. Then + +$$ +M_{p} \leq M_{q} \quad \text { whenever } \quad p \leq q . +$$ + +Theorem 2.18 (Cauchy-Schwarz inequality). Let $a_{i}, b_{i}, i=1,2, \ldots, n$, be real numbers. Then + +$$ +\left(\sum_{i=1}^{n} a_{i} b_{i}\right)^{2} \leq\left(\sum_{i=1}^{n} a_{i}^{2}\right)\left(\sum_{i=1}^{n} b_{i}^{2}\right) +$$ + +Equality occurs if and only if there exists $c \in \mathbb{R}$ such that $b_{i}=c a_{i}$ for $i=$ $1, \ldots, n$. + +Theorem 2.19 (Hölder's inequality). Let $a_{i}, b_{i}, i=1,2, \ldots, n$, be nonnegative real numbers, and let $p, q$ be positive real numbers such that $1 / p+1 / q=1$. Then + +$$ +\sum_{i=1}^{n} a_{i} b_{i} \leq\left(\sum_{i=1}^{n} a_{i}^{p}\right)^{1 / p}\left(\sum_{i=1}^{n} b_{i}^{q}\right)^{1 / q} +$$ + +Equality occurs if and only if there exists $c \in \mathbb{R}$ such that $b_{i}=c a_{i}$ for $i=1, \ldots, n$. The Cauchy-Schwarz inequality is a special case of Hölder's inequality for $p=q=2$. + +Theorem 2.20 (Minkowski's inequality). Let $a_{i}, b_{i}(i=1,2, \ldots, n)$ be nonnegative real numbers and $p$ any real number not smaller than 1. Then + +$$ +\left(\sum_{i=1}^{n}\left(a_{i}+b_{i}\right)^{p}\right)^{1 / p} \leq\left(\sum_{i=1}^{n} a_{i}^{p}\right)^{1 / p}+\left(\sum_{i=1}^{n} b_{i}^{p}\right)^{1 / p} +$$ + +For $p>1$ equality occurs if and only if there exists $c \in \mathbb{R}$ such that $b_{i}=c a_{i}$ for $i=1, \ldots, n$. For $p=1$ equality occurs in all cases. + +Theorem 2.21 (Chebyshev's inequality). Let $a_{1} \geq a_{2} \geq \cdots \geq a_{n}$ and $b_{1} \geq b_{2} \geq \cdots \geq b_{n}$ be real numbers. Then + +$$ +n \sum_{i=1}^{n} a_{i} b_{i} \geq\left(\sum_{i=1}^{n} a_{i}\right)\left(\sum_{i=1}^{n} b_{i}\right) \geq n \sum_{i=1}^{n} a_{i} b_{n+1-i} +$$ + +The two inequalities become equalities at the same time when $a_{1}=a_{2}=\cdots=$ $a_{n}$ or $b_{1}=b_{2}=\cdots=b_{n}$. + +Definition 2.22. A real function $f$ defined on an interval $I$ is convex if $f(\alpha x+$ $\beta y) \leq \alpha f(x)+\beta f(y)$. for all $x, y \in I$ and all $\alpha, \beta>0$ such that $\alpha+\beta=1$. A function $f$ is said to be concave if the opposite inequality holds, i.e., if $-f$ is convex. + +Theorem 2.23. If $f$ is continuous on an interval $I$, then $f$ is convex on that interval if and only if + +$$ +f\left(\frac{x+y}{2}\right) \leq \frac{f(x)+f(y)}{2} \quad \text { for all } x, y \in I +$$ + +Theorem 2.24. If $f$ is differentiable, then it is convex if and only if the derivative $f^{\prime}$ is nondecreasing. Similarly, differentiable function $f$ is concave if and only if $f^{\prime}$ is nonincreasing. + +Theorem 2.25 (Jensen's inequality). If $f: I \rightarrow \mathbb{R}$ is a convex function, then the inequality + +$$ +f\left(\alpha_{1} x_{1}+\cdots+\alpha_{n} x_{n}\right) \leq \alpha_{1} f\left(x_{1}\right)+\cdots+\alpha_{n} f\left(x_{n}\right) +$$ + +holds for all $\alpha_{i} \geq 0, \alpha_{1}+\cdots+\alpha_{n}=1$, and $x_{i} \in I$. For a concave function the opposite inequality holds. + +Theorem 2.26 (Muirhead's inequality). Given $x_{1}, x_{2}, \ldots, x_{n} \in \mathbb{R}^{+}$and an $n$-tuple $\mathbf{a}=\left(a_{1}, \cdots, a_{n}\right)$ of positive real numbers, we define + +$$ +T_{\mathbf{a}}\left(x_{1}, \ldots, x_{n}\right)=\sum y_{1}^{a_{1}} \ldots y_{n}^{a_{n}} +$$ + +the sum being taken over all permutations $y_{1}, \ldots, y_{n}$ of $x_{1}, \ldots, x_{n}$. We say that an $n$-tuple $\mathbf{a}$ majorizes an $n$-tuple $\mathbf{b}$ if $a_{1}+\cdots+a_{n}=b_{1}+\cdots+b_{n}$ and $a_{1}+\cdots+a_{k} \geq b_{1}+\cdots+b_{k}$ for each $k=1, \ldots, n-1$. If a nonincreasing $n$-tuple $\mathbf{a}$ majorizes a nonincreasing $n$-tuple $\mathbf{b}$, then the following inequality holds: + +$$ +T_{\mathbf{a}}\left(x_{1}, \ldots, x_{n}\right) \geq T_{\mathbf{b}}\left(x_{1}, \ldots, x_{n}\right) +$$ + +Equality occurs if and only if $x_{1}=x_{2}=\cdots=x_{n}$. +Theorem 2.27 (Schur's inequality). Using the notation introduced for Muirhead's inequality, + +$$ +T_{\lambda+2 \mu, 0,0}\left(x_{1}, x_{2}, x_{3}\right)+T_{\lambda, \mu, \mu}\left(x_{1}, x_{2}, x_{3}\right) \geq 2 T_{\lambda+\mu, \mu, 0}\left(x_{1}, x_{2}, x_{3}\right) +$$ + +where $\lambda, \mu \in \mathbb{R}^{+}$. Equality occurs if and only if $x_{1}=x_{2}=x_{3}$ or $x_{1}=x_{2}$, $x_{3}=0$ (and in analogous cases). + +### 2.1.4 Groups and Fields + +Definition 2.28. A group is a nonempty set $G$ equipped with an operation * satisfying the following conditions: +(i) $a *(b * c)=(a * b) * c$ for all $a, b, c \in G$. +(ii) There exists a (unique) additive identity $e \in G$ such that $e * a=a * e=a$ for all $a \in G$. +(iii) For each $a \in G$ there exists a (unique) additive inverse $a^{-1}=b \in G$ such that $a * b=b * a=e$. + +If $n \in \mathbb{Z}$, we define $a^{n}$ as $a * a * \cdots * a$ ( $n$ times) if $n \geq 0$, and as $\left(a^{-1}\right)^{-n}$ otherwise. + +Definition 2.29. A group $\mathcal{G}=(G, *)$ is commutative or abelian if $a * b=b * a$ for all $a, b \in G$. + +Definition 2.30. A set $A$ generates a group $(G, *)$ if every element of $G$ can be obtained using powers of the elements of $A$ and the operation $*$. In other words, if $A$ is the generator of a group $G$ then every element $g \in G$ can be written as $a_{1}^{i_{1}} * \cdots * a_{n}^{i_{n}}$, where $a_{j} \in A$ and $i_{j} \in \mathbb{Z}$ for every $j=1,2, \ldots, n$. + +Definition 2.31. The order of $a \in G$ is the smallest $n \in \mathbb{N}$ such that $a^{n}=e$, if it exists. The order of a group is the number of its elements, if it is finite. Each element of a finite group has a finite order. + +Theorem 2.32 (Lagrange's theorem). In a finite group, the order of an element divides the order of the group. + +Definition 2.33. A ring is a nonempty set $R$ equipped with two operations + and $\cdot$ such that $(R,+)$ is an abelian group and for any $a, b, c \in R$, +(i) $(a \cdot b) \cdot c=a \cdot(b \cdot c)$; +(ii) $(a+b) \cdot c=a \cdot c+b \cdot c$ and $c \cdot(a+b)=c \cdot a+c \cdot b$. + +A ring is commutative if $a \cdot b=b \cdot a$ for any $a, b \in R$ and with identity if there exists a multiplicative identity $i \in R$ such that $i \cdot a=a \cdot i=a$ for all $a \in R$. + +Definition 2.34. A field is a commutative ring with identity in which every element $a$ other than the additive identity has a multiplicative inverse $a^{-1}$ such that $a \cdot a^{-1}=a^{-1} \cdot a=i$. + +Theorem 2.35. The following are common examples of groups, rings, and fields: + +Groups: $\left(\mathbb{Z}_{n},+\right),\left(\mathbb{Z}_{p} \backslash\{0\}, \cdot\right),(\mathbb{Q},+),(\mathbb{R},+),(\mathbb{R} \backslash\{0\}, \cdot)$. +Rings: $\left(\mathbb{Z}_{n},+, \cdot\right),(\mathbb{Z},+, \cdot),(\mathbb{Z}[x],+, \cdot),(\mathbb{R}[x],+, \cdot)$. +Fields: $\left(\mathbb{Z}_{p},+, \cdot\right),(\mathbb{Q},+, \cdot),(\mathbb{Q}(\sqrt{2}),+, \cdot),(\mathbb{R},+, \cdot),(\mathbb{C},+, \cdot)$. + +### 2.2 Analysis + +Definition 2.36. A sequence $\left\{a_{n}\right\}_{n=1}^{\infty}$ has a limit $a=\lim _{n \rightarrow \infty} a_{n}$ (also denoted by $a_{n} \rightarrow a$ ) if + +$$ +(\forall \varepsilon>0)\left(\exists n_{\varepsilon} \in \mathbb{N}\right)\left(\forall n \geq n_{\varepsilon}\right)\left|a_{n}-a\right|<\varepsilon +$$ + +A function $f:(a, b) \rightarrow \mathbb{R}$ has a limit $y=\lim _{x \rightarrow c} f(x)$ if + +$$ +(\forall \varepsilon>0)(\exists \delta>0)(\forall x \in(a, b)) 0<|x-c|<\delta \Rightarrow|f(x)-y|<\varepsilon +$$ + +Definition 2.37. A sequence $x_{n}$ converges to $x \in \mathbb{R}$ if $\lim _{n \rightarrow \infty} x_{n}=x$. A series $\sum_{n=1}^{\infty} x_{n}$ converges to $s \in \mathbb{R}$ if and only if $\lim _{m \rightarrow \infty} \sum_{n=1}^{m} x_{n}=s$. A sequence or series that does not converge is said to diverge. + +Theorem 2.38. $A$ sequence $a_{n}$ is convergent if it is monotonic and bounded. +Definition 2.39. A function $f$ is continuous on $[a, b]$ if for every $x_{0} \in[a, b]$, $\lim _{x \rightarrow x_{0}} f(x)=f\left(x_{0}\right)$. + +Definition 2.40. A function $f:(a, b) \rightarrow \mathbb{R}$ is differentiable at a point $x_{0} \in$ $(a, b)$ if the following limit exists: + +$$ +f^{\prime}\left(x_{0}\right)=\lim _{x \rightarrow x_{0}} \frac{f(x)-f\left(x_{0}\right)}{x-x_{0}} +$$ + +A function is differentiable on $(a, b)$ if it is differentiable at every $x_{0} \in(a, b)$. The function $f^{\prime}$ is called the derivative of $f$. We similarly define the second derivative $f^{\prime \prime}$ as the derivative of $f^{\prime}$, and so on. + +Theorem 2.41. A differentiable function is also continuous. If $f$ and $g$ are differentiable, then $f g, \alpha f+\beta g\left(\alpha, \beta \in \mathbb{R}\right.$ ), $f \circ g, 1 / f$ (if $f \neq 0$ ), $f^{-1}$ (if welldefined) are also differentiable. It holds that $(\alpha f+\beta g)^{\prime}=\alpha f^{\prime}+\beta g^{\prime},(f g)^{\prime}=$ $f^{\prime} g+f g^{\prime},(f \circ g)^{\prime}=\left(f^{\prime} \circ g\right) \cdot g^{\prime},(1 / f)^{\prime}=-f^{\prime} / f^{2},(f / g)^{\prime}=\left(f^{\prime} g-f g^{\prime}\right) / g^{2}$, $\left(f^{-1}\right)^{\prime}=1 /\left(f^{\prime} \circ f^{-1}\right)$. + +Theorem 2.42. The following are derivatives of some elementary functions (a denotes a real constant): $\left(x^{a}\right)^{\prime}=a x^{a-1},(\ln x)^{\prime}=1 / x,\left(a^{x}\right)^{\prime}=a^{x} \ln a$, $(\sin x)^{\prime}=\cos x,(\cos x)^{\prime}=-\sin x$. + +Theorem 2.43 (Fermat's theorem). Let $f:[a, b] \rightarrow \mathbb{R}$ be a differentiable function. The function $f$ attains its maximum and minimum in this interval. If $x_{0} \in(a, b)$ is an extremum (i.e., a maximum or minimum), then $f^{\prime}\left(x_{0}\right)=0$. + +Theorem 2.44 (Rolle's theorem). Let $f(x)$ be a continuously differentiable function defined on $[a, b]$, where $a, b \in \mathbb{R}, a0$. + +Definition 2.68. The mass center (barycenter) of the set of mass points $\left(A_{i}, m_{i}\right), i=1,2, \ldots, n$, is the point $T$ such that $\sum_{i} m_{i} \overrightarrow{T A_{i}}=0$. + +Theorem 2.69 (Leibniz's theorem). Let $T$ be the mass center of the set of mass points $\left\{\left(A_{i}, m_{i}\right) \mid i=1,2, \ldots, n\right\}$ of total mass $m=m_{1}+\cdots+m_{n}$, and let $X$ be an arbitrary point. Then + +$$ +\sum_{i=1}^{n} m_{i} X A_{i}^{2}=\sum_{i=1}^{n} m_{i} T A_{i}^{2}+m X T^{2} +$$ + +Specifically, if $T$ is the centroid of $\triangle A B C$ and $X$ an arbitrary point, then + +$$ +A X^{2}+B X^{2}+C X^{2}=A T^{2}+B T^{2}+C T^{2}+3 X T^{2} +$$ + +### 2.3.4 Quadrilaterals + +Theorem 2.70. $A$ quadrilateral $A B C D$ is cyclic (i.e., there exists a circumcircle of $A B C D$ ) if and only if $\angle A C B=\angle A D B$ and if and only if $\angle A D C+\angle A B C=180^{\circ}$. + +Theorem 2.71 (Ptolemy's theorem). A convex quadrilateral $A B C D$ is cyclic if and only if + +$$ +A C \cdot B D=A B \cdot C D+A D \cdot B C +$$ + +For an arbitrary quadrilateral $A B C D$ we have Ptolemy's inequality (see 2.3.7, Geometric Inequalities). + +Theorem 2.72 (Casey's theorem). Let $k_{1}, k_{2}, k_{3}, k_{4}$ be four circles that all touch a given circle $k$. Let $t_{i j}$ be the length of a segment determined by an external common tangent of circles $k_{i}$ and $k_{j}(i, j \in\{1,2,3,4\})$ if both $k_{i}$ and $k_{j}$ touch $k$ internally, or both touch $k$ externally. Otherwise, $t_{i j}$ is set to be the internal common tangent. Then one of the products $t_{12} t_{34}, t_{13} t_{24}$, and $t_{14} t_{23}$ is the sum of the other two. + +Some of the circles $k_{1}, k_{2}, k_{3}, k_{4}$ may be degenerate, i.e. of 0 radius and thus reduced to being points. In particular, for three points $A, B, C$ on a circle $k$ and a circle $k^{\prime}$ touching $k$ at a point on the arc of $A C$ not containing $B$, we have $A C \cdot b=A B \cdot c+a \cdot B C$, where $a, b$, and $c$ are the lengths of the tangent segments from points $A, B$, and $C$ to $k^{\prime}$. Ptolemy's theorem is a special case of Casey's theorem when all four circles are degenerate. + +Theorem 2.73. $A$ convex quadrilateral $A B C D$ is tangent (i.e., there exists an incircle of $A B C D$ ) if and only if + +$$ +A B+C D=B C+D A +$$ + +Theorem 2.74. For arbitrary points $A, B, C, D$ in space, $A C \perp B D$ if and only if + +$$ +A B^{2}+C D^{2}=B C^{2}+D A^{2} +$$ + +Theorem 2.75 (Newton's theorem). Let $A B C D$ be a quadrilateral, $A D \cap$ $B C=E$, and $A B \cap D C=F$ (such points $A, B, C, D, E, F$ form a complete quadrilateral). Then the midpoints of $A C, B D$, and $E F$ are collinear. If $A B C D$ is tangent, then the incenter also lies on this line. + +Theorem 2.76 (Brocard's theorem). Let $A B C D$ be a quadrilateral inscribed in a circle with center $O$, and let $P=A B \cap C D, Q=A D \cap B C$, $R=A C \cap B D$. Then $O$ is the orthocenter of $\triangle P Q R$. + +### 2.3.5 Circle Geometry + +Theorem 2.77 (Pascal's theorem). If $A_{1}, A_{2}, A_{3}, B_{1}, B_{2}, B_{3}$ are distinct points on a conic $\gamma$ (e.g., circle), then points $X_{1}=A_{2} B_{3} \cap A_{3} B_{2}, X_{2}=$ $A_{1} B_{3} \cap A_{3} B_{1}$, and $X_{3}=A_{1} B_{2} \cap A_{2} B_{1}$ are collinear. The special result when $\gamma$ consists of two lines is called Pappus's theorem. + +Theorem 2.78 (Brianchon's theorem). Let $A B C D E F$ be an arbitrary convex hexagon circumscribed about a conic (e.g., circle). Then $A D, B E$ and CF meet in a point. + +Theorem 2.79 (The butterfy theorem). Let $A B$ be a segment of circle $k$ and $C$ its midpoint. Let $p$ and $q$ be two different lines through $C$ that, respectively, intersect $k$ on one side of $A B$ in $P$ and $Q$ and on the other in $P^{\prime}$ and $Q^{\prime}$. Let $E$ and $F$ respectively be the intersections of $P Q^{\prime}$ and $P^{\prime} Q$ with $A B$. Then it follows that $C E=C F$. + +Definition 2.80. The power of a point $X$ with respect to a circle $k(O, r)$ is defined by $\mathcal{P}(X)=O X^{2}-r^{2}$. For an arbitrary line $l$ through $X$ that intersects $k$ at $A$ and $B(A=B$ when $l$ is a tangent), it follows that $\mathcal{P}(X)=\overrightarrow{X A} \cdot \overrightarrow{X B}$. + +Definition 2.81. The radical axis of two circles is the locus of points that have equal powers with respect to both circles. The radical axis of circles $k_{1}\left(O_{1}, r_{1}\right)$ and $k_{2}\left(O_{2}, r_{2}\right)$ is a line perpendicular to $O_{1} O_{2}$. The radical axes of three distinct circles are concurrent or mutually parallel. If concurrent, the intersection of the three axes is called the radical center. + +Definition 2.82. The pole of a line $l \not \ni O$ with respect to a circle $k(O, r)$ is a point $A$ on the other side of $l$ from $O$ such that $O A \perp l$ and $d(O, l) \cdot O A=r^{2}$. In particular, if $l$ intersects $k$ in two points, its pole will be the intersection of the tangents to $k$ at these two points. + +Definition 2.83. The polar of the point $A$ from the previous definition is the line $l$. In particular, if $A$ is a point outside $k$ and $A M, A N$ are tangents to $k$ $(M, N \in k)$, then $M N$ is the polar of $A$. +Poles and polares are generally defined in a similar way with respect to arbitrary non-degenerate conics. + +Theorem 2.84. If $A$ belongs to a polar of $B$, then $B$ belongs to a polar of $A$. + +### 2.3.6 Inversion + +Definition 2.85. An inversion of the plane $\pi$ around the circle $k(O, r)$ (which belongs to $\pi$ ), is a transformation of the set $\pi \backslash\{O\}$ onto itself such that every point $P$ is transformed into a point $P^{\prime}$ on $\left(O P\right.$ such that $O P \cdot O P^{\prime}=r^{2}$. In the following statements we implicitly assume exclusion of $O$. + +Theorem 2.86. The fixed points of the inversion are on the circle $k$. The inside of $k$ is transformed into the outside and vice versa. + +Theorem 2.87. If $A, B$ transform into $A^{\prime}, B^{\prime}$ after an inversion, then $\angle O A B$ $=\angle O B^{\prime} A^{\prime}$, and also $A B B^{\prime} A^{\prime}$ is cyclic and perpendicular to $k$. $A$ circle perpendicular to $k$ transforms into itself. Inversion preserves angles between continuous curves (which includes lines and circles). + +Theorem 2.88. An inversion transforms lines not containing $O$ into circles containing $O$, lines containing $O$ into themselves, circles not containing $O$ into circles not containing $O$, circles containing $O$ into lines not containing $O$ 。 + +### 2.3.7 Geometric Inequalities + +Theorem 2.89 (The triangle inequality). For any three points $A, B, C$ in a plane $A B+B C \geq A C$. Equality occurs when $A, B, C$ are collinear and $\mathcal{B}(A, B, C)$. + +Theorem 2.90 (Ptolemy's inequality). For any four points $A, B, C, D$, + +$$ +A C \cdot B D \leq A B \cdot C D+A D \cdot B C +$$ + +Theorem 2.91 (The parallelogram inequality). For any four points $A$, $B, C, D$, + +$$ +A B^{2}+B C^{2}+C D^{2}+D A^{2} \geq A C^{2}+B D^{2} +$$ + +Equality occurs if and only if $A B C D$ is a parallelogram. +Theorem 2.92. For a given triangle $\triangle A B C$ the point $X$ for which $A X+$ $B X+C X$ is minimal is Toricelli's point when all angles of $\triangle A B C$ are less than or equal to $120^{\circ}$, and is the vertex of the obtuse angle otherwise. The point $X_{2}$ for which $A X_{2}^{2}+B X_{2}^{2}+C X_{2}^{2}$ is minimal is the centroid (see Leibniz's theorem). + +Theorem 2.93 (The Erdös-Mordell inequality). Let $P$ be a point in the interior of $\triangle A B C$ and $X, Y, Z$ projections of $P$ onto $B C, A C, A B$, respectively. Then + +$$ +P A+P B+P C \geq 2(P X+P Y+P Z) +$$ + +Equality holds if and only if $\triangle A B C$ is equilateral and $P$ is its center. + +### 2.3.8 Trigonometry + +Definition 2.94. The trigonometric circle is the unit circle centered at the origin $O$ of a coordinate plane. Let $A$ be the point $(1,0)$ and $P(x, y)$ be a point on the trigonometric circle such that $\measuredangle A O P=\alpha$. We define $\sin \alpha=y$, $\cos \alpha=x, \tan \alpha=y / x$, and $\cot \alpha=x / y$. + +Theorem 2.95. The functions $\sin$ and cos are periodic with period $2 \pi$. The functions $\tan$ and cot are periodic with period $\pi$. The following simple identities hold: $\sin ^{2} x+\cos ^{2} x=1, \sin 0=\sin \pi=0, \sin (-x)=-\sin x, \cos (-x)=$ $\cos x, \sin (\pi / 2)=1, \sin (\pi / 4)=1 / \sqrt{2}, \sin (\pi / 6)=1 / 2, \cos x=\sin (\pi / 2-x)$. From these identities other identities can be easily derived. + +Theorem 2.96. Additive formulas for trigonometric functions: + +$$ +\begin{array}{ll} +\sin (\alpha \pm \beta)=\sin \alpha \cos \beta \pm \cos \alpha \sin \beta, & \cos (\alpha \pm \beta)=\cos \alpha \cos \beta \mp \sin \alpha \sin \beta, \\ +\tan (\alpha \pm \beta)=\frac{\tan \alpha \pm \tan \beta}{1 \mp \tan \alpha \tan \beta}, & \cot (\alpha \pm \beta)=\frac{\cot \alpha \cot \beta \mp 1}{\cot \alpha \pm \cot \beta} . +\end{array} +$$ + +Theorem 2.97. Formulas for trigonometric functions of $2 x$ and $3 x$ : + +$$ +\begin{aligned} +\sin 2 x & =2 \sin x \cos x, & & \sin 3 x=3 \sin x-4 \sin ^{3} x, \\ +\cos 2 x & =2 \cos ^{2} x-1, & & \cos 3 x=4 \cos ^{3} x-3 \cos x, \\ +\tan 2 x & =\frac{2 \tan x}{1-\tan ^{2} x}, & & \tan 3 x=\frac{3 \tan x-\tan ^{3} x}{1-3 \tan ^{2} x} . +\end{aligned} +$$ + +Theorem 2.98. For any $x \in \mathbb{R}, \sin x=\frac{2 t}{1+t^{2}}$ and $\cos x=\frac{1-t^{2}}{1+t^{2}}$, where $t=$ $\tan \frac{x}{2}$. + +Theorem 2.99. Transformations from product to sum: + +$$ +\begin{aligned} +& 2 \cos \alpha \cos \beta=\cos (\alpha+\beta)+\cos (\alpha-\beta) \\ +& 2 \sin \alpha \cos \beta=\sin (\alpha+\beta)+\sin (\alpha-\beta) \\ +& 2 \sin \alpha \sin \beta=\cos (\alpha-\beta)-\cos (\alpha+\beta) +\end{aligned} +$$ + +Theorem 2.100. The angles $\alpha, \beta, \gamma$ of a triangle satisfy + +$$ +\begin{aligned} +\cos ^{2} \alpha+\cos ^{2} \beta+\cos ^{2} \gamma+2 \cos \alpha \cos \beta \cos \gamma & =1 \\ +\tan \alpha+\tan \beta+\tan \gamma & =\tan \alpha \tan \beta \tan \gamma +\end{aligned} +$$ + +Theorem 2.101 (De Moivre's formula). If $i^{2}=-1$, then + +$$ +(\cos x+i \sin x)^{n}=\cos n x+i \sin n x +$$ + +### 2.3.9 Formulas in Geometry + +Theorem 2.102 (Heron's formula). The area of a triangle $A B C$ with sides $a, b, c$ and semiperimeter $s$ is given by + +$$ +S=\sqrt{s(s-a)(s-b)(s-c)}=\frac{1}{4} \sqrt{2 a^{2} b^{2}+2 a^{2} c^{2}+2 b^{2} c^{2}-a^{4}-b^{4}-c^{4}} . +$$ + +Theorem 2.103 (The law of sines). The sides $a, b, c$ and angles $\alpha, \beta, \gamma$ of a triangle $A B C$ satisfy + +$$ +\frac{a}{\sin \alpha}=\frac{b}{\sin \beta}=\frac{c}{\sin \gamma}=2 R +$$ + +where $R$ is the circumradius of $\triangle A B C$. +Theorem 2.104 (The law of cosines). The sides and angles of $\triangle A B C$ satisfy + +$$ +c^{2}=a^{2}+b^{2}-2 a b \cos \gamma +$$ + +Theorem 2.105. The circumradius $R$ and inradius $r$ of a triangle $A B C$ satisfy $R=\frac{a b c}{4 S}$ and $r=\frac{2 S}{a+b+c}=R(\cos \alpha+\cos \beta+\cos \gamma-1)$. If $x, y, z d e-$ note the distances of the circumcenter in an acute triangle to the sides, then $x+y+z=R+r$. + +Theorem 2.106 (Euler's formula). If $O$ and $I$ are the circumcenter and incenter of $\triangle A B C$, then $O I^{2}=R(R-2 r)$, where $R$ and $r$ are respectively the circumradius and the inradius of $\triangle A B C$. Consequently, $R \geq 2 r$. + +Theorem 2.107. The area $S$ of a quadrilateral $A B C D$ with sides $a, b, c, d$, semiperimeter $p$, and angles $\alpha, \gamma$ at vertices $A, C$ respectively is given by + +$$ +S=\sqrt{(p-a)(p-b)(p-c)(p-d)-a b c d \cos ^{2} \frac{\alpha+\gamma}{2}} . +$$ + +If $A B C D$ is a cyclic quadrilateral, the above formula reduces to + +$$ +S=\sqrt{(p-a)(p-b)(p-c)(p-d)} +$$ + +Theorem 2.108 (Euler's theorem for pedal triangles). Let $X, Y, Z$ be the feet of the perpendiculars from a point $P$ to the sides of a triangle $A B C$. Let $O$ denote the circumcenter and $R$ the circumradius of $\triangle A B C$. Then + +$$ +S_{X Y Z}=\frac{1}{4}\left|1-\frac{O P^{2}}{R^{2}}\right| S_{A B C} . +$$ + +Moreover, $S_{X Y Z}=0$ if and only if $P$ lies on the circumcircle of $\triangle A B C$ (see Simson's line). + +Theorem 2.109. If overrightarrowa $=\left(a_{1}, a_{2}, a_{3}\right), \vec{b}=\left(b_{1}, b_{2}, b_{3}\right), \vec{c}=\left(c_{1}, c_{2}, c_{3}\right)$ are three vectors in coordinate space, then + +$$ +\begin{gathered} +\vec{a} \cdot \vec{b}=a_{1} b_{1}+a_{2} b_{2}+a_{3} b_{3}, \quad \vec{a} \times \vec{b}=\left(a_{1} b_{2}-a_{2} b_{1}, a_{2} b_{3}-a_{3} b_{2}, a_{3} b_{1}-a_{1} b_{3}\right), \\ +{[\vec{a}, \vec{b}, \vec{c}]=\left\|\begin{array}{lll} +a_{1} & a_{2} & a_{3} \\ +b_{1} & b_{2} & b_{3} \\ +c_{1} & c_{2} & c_{3} +\end{array}\right\|} +\end{gathered} . +$$ + +Theorem 2.110. The area of a triangle $A B C$ and the volume of a tetrahedron $A B C D$ are equal to $|\overrightarrow{A B} \times \overrightarrow{A C}|$ and $|[\overrightarrow{A B}, \overrightarrow{A C}, \overrightarrow{A D}]|$, respectively. + +Theorem 2.111 (Cavalieri's principle). If the sections of two solids by the same plane always have equal area, then the volumes of the two solids are equal. + +### 2.4 Number Theory + +### 2.4.1 Divisibility and Congruences + +Definition 2.112. The greatest common divisor $(a, b)=\operatorname{gcd}(a, b)$ of $a, b \in \mathbb{N}$ is the largest positive integer that divides both $a$ and $b$. Positive integers $a$ and $b$ are coprime or relatively prime if $(a, b)=1$. The least common multiple $[a, b]=\operatorname{lcm}(a, b)$ of $a, b \in \mathbb{N}$ is the smallest positive integer that is divisible by both $a$ and $b$. It holds that $[a, b](a, b)=a b$. The above concepts are easily generalized to more than two numbers; i.e., we also define $\left(a_{1}, a_{2}, \ldots, a_{n}\right)$ and $\left[a_{1}, a_{2}, \ldots, a_{n}\right]$. + +Theorem 2.113 (Euclid's algorithm). Since $(a, b)=(|a-b|, a)=(\mid a-$ $b \mid, b)$ it follows that starting from positive integers $a$ and $b$ one eventually obtains $(a, b)$ by repeatedly replacing $a$ and $b$ with $|a-b|$ and $\min \{a, b\}$ until the two numbers are equal. The algorithm can be generalized to more than two numbers. + +Theorem 2.114 (Corollary to Euclid's algorithm). For each $a, b \in \mathbb{N}$ there exist $x, y \in \mathbb{Z}$ such that $a x+b y=(a, b)$. The number $(a, b)$ is the smallest positive number for which such $x$ and $y$ can be found. + +Theorem 2.115 (Second corollary to Euclid's algorithm). For a, m, $n \in$ $\mathbb{N}$ and $a>1$ it follows that $\left(a^{m}-1, a^{n}-1\right)=a^{(m, n)}-1$. + +Theorem 2.116 (Fundamental theorem of arithmetic). Every positive integer can be uniquely represented as a product of primes, up to their order. + +Theorem 2.117. The fundamental theorem of arithmetic also holds in some other rings, such as $\mathbb{Z}[i]=\{a+b i \mid a, b \in \mathbb{Z}\}, \mathbb{Z}[\sqrt{2}], \mathbb{Z}[\sqrt{-2}]$, $\mathbb{Z}[\omega]$ (where $\omega$ is a complex third root of 1). In these cases, the factorization into primes is unique up to the order and divisors of 1. + +Definition 2.118. Integers $a, b$ are congruent modulo $n \in \mathbb{N}$ if $n \mid a-b$. We then write $a \equiv b(\bmod n)$. + +Theorem 2.119 (Chinese remainder theorem). If $m_{1}, m_{2}, \ldots, m_{k}$ are positive integers pairwise relatively prime and $a_{1}, \ldots, a_{k}, c_{1}, \ldots, c_{k}$ are integers such that $\left(a_{i}, m_{i}\right)=1(i=1, \ldots, n)$, then the system of congruences + +$$ +a_{i} x \equiv c_{i}\left(\bmod m_{i}\right), \quad i=1,2, \ldots, n +$$ + +has a unique solution modulo $m_{1} m_{2} \cdots m_{k}$. + +### 2.4.2 Exponential Congruences + +Theorem 2.120 (Wilson's theorem). If $p$ is a prime, then $p \mid(p-1)!+1$. +Theorem 2.121 (Fermat's (little) theorem). Let $p$ be a prime number and $a$ be an integer with $(a, p)=1$. Then $a^{p-1} \equiv 1(\bmod p)$. This theorem is a special case of Euler's theorem. + +Definition 2.122. Euler's function $\varphi(n)$ is defined for $n \in \mathbb{N}$ as the number of positive integers less than $n$ and coprime to $n$. It holds that + +$$ +\varphi(n)=n\left(1-\frac{1}{p_{1}}\right) \cdots\left(1-\frac{1}{p_{k}}\right) +$$ + +where $n=p_{1}^{\alpha_{1}} \cdots p_{k}^{\alpha_{k}}$ is the factorization of $n$ into primes. + +Theorem 2.123 (Euler's theorem). Let $n$ be a natural number and a be an integer with $(a, n)=1$. Then $a^{\varphi(n)} \equiv 1(\bmod n)$. + +Theorem 2.124 (Existence of primitive roots). Let $p$ be a prime. There exists $g \in\{1,2, \ldots, p-1\}$ (called a primitive root modulo $p$ ) such that the set $\left\{1, g, g^{2}, \ldots, g^{p-2}\right\}$ is equal to $\{1,2, \ldots, p-1\}$ modulo $p$. + +Definition 2.125. Let $p$ be a prime and $\alpha$ be a nonnegative integer. We say that $p^{\alpha}$ is the exact power of $p$ that divides an integer $a$ (and $\alpha$ the exact exponent) if $p^{\alpha} \mid a$ and $p^{\alpha+1} \nmid a$. + +Theorem 2.126. Let $a, n$ be positive integers and $p$ be an odd prime. If $p^{\alpha}$ $(\alpha \in \mathbb{N}$ ) is the exact power of $p$ that divides $a-1$, then for any integer $\beta \geq 0$, $p^{\alpha+\beta} \mid a^{n}-1$ if and only if $p^{\beta} \mid n$. (See (SL97-14).) + +A similar statement holds for $p=2$. If $2^{\alpha}(\alpha \in \mathbb{N})$ is the exact power of 2 that divides $a^{2}-1$, then for any integer $\beta \geq 0,2^{\alpha+\beta} \mid a^{n}-1$ if and only if $2^{\beta+1} \mid n$. (See (SL89-27).) + +### 2.4.3 Quadratic Diophantine Equations + +Theorem 2.127. The solutions of $a^{2}+b^{2}=c^{2}$ in integers are given by $a=$ $t\left(m^{2}-n^{2}\right), b=2 t m n, c=t\left(m^{2}+n^{2}\right)$ (provided that $b$ is even), where $t, m, n \in$ $\mathbb{Z}$. The triples $(a, b, c)$ are called Pythagorean (or primitive Pythagorean if $\operatorname{gcd}(a, b, c)=1)$. + +Definition 2.128. Given $D \in \mathbb{N}$ that is not a perfect square, a Pell's equation is an equation of the form $x^{2}-D y^{2}=1$, where $x, y \in \mathbb{Z}$. + +Theorem 2.129. If $\left(x_{0}, y_{0}\right)$ is the least (nontrivial) solution in $\mathbb{N}$ of the Pell's equation $x^{2}-D y^{2}=1$, then all the integer solutions $(x, y)$ are given by $x+y \sqrt{D}= \pm\left(x_{0}+y_{0} \sqrt{D}\right)^{n}$, where $n \in \mathbb{Z}$. + +Definition 2.130. An integer $a$ is a quadratic residue modulo a prime $p$ if there exists $x \in \mathbb{Z}$ such that $x^{2} \equiv a(\bmod p)$. Otherwise, $a$ is a quadratic nonresidue modulo $p$. + +Definition 2.131. Legendre's symbol for an integer $a$ and a prime $p$ is defined by + +$$ +\left(\frac{a}{p}\right)= \begin{cases}1 & \text { if } a \text { is a quadratic residue } \bmod p \text { and } p \nmid a ; \\ 0 & \text { if } p \mid a ; \\ -1 & \text { otherwise. }\end{cases} +$$ + +Clearly $\left(\frac{a}{p}\right)=\left(\frac{a+p}{p}\right)$ and $\left(\frac{a^{2}}{p}\right)=1$ if $p \nmid a$. Legendre's symbol is multiplicative, i.e., $\left(\frac{a}{p}\right)\left(\frac{b}{p}\right)=\left(\frac{a b}{p}\right)$. + +Theorem 2.132 (Euler's criterion). For each odd prime $p$ and integer a not divisible by $p, a^{\frac{p-1}{2}} \equiv\left(\frac{a}{p}\right)(\bmod p)$. + +Theorem 2.133. For a prime $p>3,\left(\frac{-1}{p}\right),\left(\frac{2}{p}\right)$ and $\left(\frac{-3}{p}\right)$ are equal to 1 if and only if $p \equiv 1(\bmod 4), p \equiv \pm 1(\bmod 8)$ and $p \equiv 1(\bmod 6)$, respectively. + +Theorem 2.134 (Gauss's Reciprocity law). For any two distinct odd primes $p$ and $q$, + +$$ +\left(\frac{p}{q}\right)\left(\frac{q}{p}\right)=(-1)^{\frac{p-1}{2} \cdot \frac{q-1}{2}} +$$ + +Definition 2.135. Jacobi's symbol for an integer $a$ and an odd positive integer $b$ is defined as + +$$ +\left(\frac{a}{b}\right)=\left(\frac{a}{p_{1}}\right)^{\alpha_{1}} \cdots\left(\frac{a}{p_{k}}\right)^{\alpha_{k}} +$$ + +where $b=p_{1}^{\alpha_{1}} \cdots p_{k}^{\alpha_{k}}$ is the factorization of $b$ into primes. +Theorem 2.136. If $\left(\frac{a}{b}\right)=-1$, then $a$ is a quadratic nonresidue modulo $b$, but the converse is false. All the above identities for Legendre symbols except Euler's criterion remain true for Jacobi symbols. + +### 2.4.4 Farey Sequences + +Definition 2.137. For any positive integer $n$, the Farey sequence $F_{n}$ is the sequence of rational numbers $a / b$ with $0 \leq a \leq b \leq n$ and $(a, b)=1$ arranged in increasing order. For instance, $F_{3}=\left\{\frac{0}{1}, \frac{1}{3}, \frac{1}{2}, \frac{2}{3}, \frac{1}{1}\right\}$. +Theorem 2.138. If $p_{1} / q_{1}, p_{2} / q_{2}$, and $p_{3} / q_{3}$ are three successive terms in a Farey sequence, then + +$$ +p_{2} q_{1}-p_{1} q_{2}=1 \quad \text { and } \quad \frac{p_{1}+p_{3}}{q_{1}+q_{3}}=\frac{p_{2}}{q_{2}} +$$ + +### 2.5 Combinatorics + +### 2.5.1 Counting of Objects + +Many combinatorial problems involving the counting of objects satisfying a given set of properties can be properly reduced to an application of one of the following concepts. + +Definition 2.139. A variation of order $n$ over $k$ is a 1 to 1 mapping of $\{1,2, \ldots, k\}$ into $\{1,2, \ldots, n\}$. For a given $n$ and $k$, where $n \geq k$, the number of different variations is $V_{n}^{k}=\frac{n!}{(n-k)!}$. + +Definition 2.140. A variation with repetition of order $n$ over $k$ is an arbitrary mapping of $\{1,2, \ldots, k\}$ into $\{1,2, \ldots, n\}$. For a given $n$ and $k$ the number of different variations with repetition is $\bar{V}_{n}^{k}=k^{n}$. + +Definition 2.141. A permutation of order $n$ is a bijection of $\{1,2, \ldots, n\}$ into itself (a special case of variation for $k=n$ ). For a given $n$ the number of different permutations is $P_{n}=n$ !. + +Definition 2.142. A combination of order $n$ over $k$ is a $k$-element subset of $\{1,2, \ldots, n\}$. For a given $n$ and $k$ the number of different combinations is $C_{n}^{k}=\binom{n}{k}$ 。 + +Definition 2.143. A permutation with repetition of order $n$ is a bijection of $\{1,2, \ldots, n\}$ into a multiset of $n$ elements. A multiset is defined to be a set in which certain elements are deemed mutually indistinguishable (for example, as in $\{1,1,2,3\})$. + +If $\{1,2 \ldots, s\}$ denotes a set of different elements in the multiset and the element $i$ appears $\alpha_{i}$ times in the multiset, then number of different permutations with repetition is $P_{n, \alpha_{1}, \ldots, \alpha_{s}}=\frac{n!}{\alpha_{1}!\cdot \alpha_{2}!\cdots \alpha_{s}!}$. A combination is a special case of permutation with repetition for a multiset with two different elements. + +Theorem 2.144 (The pigeonhole principle). If a set of $n k+1$ different elements is partitioned into $n$ mutually disjoint subsets, then at least one subset will contain at least $k+1$ elements. + +Theorem 2.145 (The inclusion-exclusion principle). Let $S_{1}, S_{2}, \ldots, S_{n}$ be a family of subsets of the set $S$. The number of elements of $S$ contained in none of the subsets is given by the formula + +$$ +\left|S \backslash\left(S_{1} \cup \cdots \cup S_{n}\right)\right|=|S|-\sum_{k=1}^{n} \sum_{1 \leq i_{1}<\cdots Budapest-Veszprem, Hungary, July 6-16, 1961 + +### 3.3.1 Contest Problems + +First Day + +1. (HUN) Solve the following system of equations: + +$$ +\begin{aligned} +x+y+z & =a, \\ +x^{2}+y^{2}+z^{2} & =b^{2}, \\ +x y & =z^{2} +\end{aligned} +$$ + +where $a$ and $b$ are given real numbers. What conditions must hold on $a$ and $b$ for the solutions to be positive and distinct? +2. (POL) Let $a, b$, and $c$ be the lengths of a triangle whose area is $S$. Prove that + +$$ +a^{2}+b^{2}+c^{2} \geq 4 S \sqrt{3} +$$ + +In what case does equality hold? +3. (BUL) Solve the equation $\cos ^{n} x-\sin ^{n} x=1$, where $n$ is a given positive integer. + +## Second Day + +4. (GDR) In the interior of $\triangle P_{1} P_{2} P_{3}$ a point $P$ is given. Let $Q_{1}, Q_{2}$, and $Q_{3}$ respectively be the intersections of $P P_{1}, P P_{2}$, and $P P_{3}$ with the opposing edges of $\triangle P_{1} P_{2} P_{3}$. Prove that among the ratios $P P_{1} / P Q_{1}, P P_{2} / P Q_{2}$, and $P P_{3} / P Q_{3}$ there exists at least one not larger than 2 and at least one not smaller than 2. +5. (CZS) Construct a triangle $A B C$ if the following elements are given: $A C=b, A B=c$, and $\measuredangle A M B=\omega\left(\omega<90^{\circ}\right)$, where $M$ is the midpoint of $B C$. Prove that the construction has a solution if and only if + +$$ +b \tan \frac{\omega}{2} \leq c Prague-Hluboka, Czechoslovakia, July 7-15, 1962 + +### 3.4.1 Contest Problems + +First Day + +1. (POL) Find the smallest natural number $n$ with the following properties: +(a) In decimal representation it ends with 6. +(b) If we move this digit to the front of the number, we get a number 4 times larger. +2. (HUN) Find all real numbers $x$ for which + +$$ +\sqrt{3-x}-\sqrt{x+1}>\frac{1}{2} . +$$ + +3. (CZS) A cube $A B C D A^{\prime} B^{\prime} C^{\prime} D^{\prime}$ is given. The point $X$ is moving at a constant speed along the square $A B C D$ in the direction from $A$ to $B$. The point $Y$ is moving with the same constant speed along the square $B C C^{\prime} B^{\prime}$ in the direction from $B^{\prime}$ to $C^{\prime}$. Initially, $X$ and $Y$ start out from $A$ and $B^{\prime}$ respectively. Find the locus of all the midpoints of $X Y$. + +Second Day +4. (ROM) Solve the equation + +$$ +\cos ^{2} x+\cos ^{2} 2 x+\cos ^{2} 3 x=1 . +$$ + +5. (BUL) On the circle $k$ three points $A, B$, and $C$ are given. Construct the fourth point on the circle $D$ such that one can inscribe a circle in $A B C D$. +6. (GDR) Let $A B C$ be an isosceles triangle with circumradius $r$ and inradius $\rho$. Prove that the distance $d$ between the circumcenter and incenter is given by + +$$ +d=\sqrt{r(r-2 \rho)} . +$$ + +7. (USS) Prove that a tetrahedron $S A B C$ has five different spheres that touch all six lines determined by its edges if and only if it is regular. + +### 3.5 The Fifth IMO
Wroclaw, Poland, July 5-13, 1963 + +### 3.5.1 Contest Problems + +First Day + +1. (CZS) Determine all real solutions of the equation $\sqrt{x^{2}-p}+2 \sqrt{x^{2}-1}=$ $x$, where $p$ is a real number. +2. (USS) Find the locus of points in space that are vertices of right angles of which one ray passes through a given point and the other intersects a given segment. +3. (HUN) Prove that if all the angles of a convex $n$-gon are equal and the lengths of consecutive edges $a_{1}, \ldots, a_{n}$ satisfy $a_{1} \geq a_{2} \geq \cdots \geq a_{n}$, then $a_{1}=a_{2}=\cdots=a_{n}$. + +Second Day +4. (USS) Find all solutions $x_{1}, \ldots, x_{5}$ to the system of equations + +$$ +\left\{\begin{array}{l} +x_{5}+x_{2}=y x_{1}, \\ +x_{1}+x_{3}=y x_{2}, \\ +x_{2}+x_{4}=y x_{3}, \\ +x_{3}+x_{5}=y x_{4}, \\ +x_{4}+x_{1}=y x_{5}, +\end{array}\right. +$$ + +where $y$ is a real parameter. +5. (GDR) Prove that $\cos \frac{\pi}{7}-\cos \frac{2 \pi}{7}+\cos \frac{3 \pi}{7}=\frac{1}{2}$. +6. (HUN) Five students $A, B, C, D$, and $E$ have taken part in a certain competition. Before the competition, two persons $X$ and $Y$ tried to guess the rankings. $X$ thought that the ranking would be $A, B, C, D, E$; and $Y$ thought that the ranking would be $D, A, E, C, B$. At the end, it was revealed that $X$ didn't guess correctly any rankings of the participants, and moreover, didn't guess any of the orderings of pairs of consecutive participants. On the other hand, $Y$ guessed the correct rankings of two participants and the correct ordering of two pairs of consecutive participants. Determine the rankings of the competition. + +### 3.6 The Sixth IMO
Moscow, Soviet Union, June 30-July 10, 1964 + +### 3.6.1 Contest Problems + +First Day + +1. (CZS) (a) Find all natural numbers $n$ such that the number $2^{n}-1$ is divisible by 7 . +(b) Prove that for all natural numbers $n$ the number $2^{n}+1$ is not divisible by 7 . +2. (HUN) Denote by $a, b, c$ the lengths of the sides of a triangle. Prove that + +$$ +a^{2}(b+c-a)+b^{2}(c+a-b)+c^{2}(a+b-c) \leq 3 a b c +$$ + +3. (YUG) The incircle is inscribed in a triangle $A B C$ with sides $a, b, c$. Three tangents to the incircle are drawn, each of which is parallel to one side of the triangle $A B C$. These tangents form three smaller triangles (internal to $\triangle A B C$ ) with the sides of $\triangle A B C$. In each of these triangles an incircle is inscribed. Determine the sum of areas of all four incircles. + +Second Day +4. (HUN) Each of 17 students talked with every other student. They all talked about three different topics. Each pair of students talked about one topic. Prove that there are three students that talked about the same topic among themselves. +5. (ROM) Five points are given in the plane. Among the lines that connect these five points, no two coincide and no two are parallel or perpendicular. Through each point we construct an altitude to each of the other lines. What is the maximal number of intersection points of these altitudes (excluding the initial five points)? +6. (POL) Given a tetrahedron $A B C D$, let $D_{1}$ be the centroid of the triangle $A B C$ and let $A_{1}, B_{1}, C_{1}$ be the intersection points of the lines parallel to $D D_{1}$ and passing through the points $A, B, C$ with the opposite faces of the tetrahedron. Prove that the volume of the tetrahedron $A B C D$ is onethird the volume of the tetrahedron $A_{1} B_{1} C_{1} D_{1}$. Does the result remain true if the point $D_{1}$ is replaced with any point inside the triangle $A B C$ ? + +### 3.7 The Seventh IMO Berlin, DR Germany, July 3-13, 1965 + +### 3.7.1 Contest Problems + +First Day + +1. (YUG) Find all real numbers $x \in[0,2 \pi]$ such that + +$$ +2 \cos x \leq|\sqrt{1+\sin 2 x}-\sqrt{1-\sin 2 x}| \leq \sqrt{2} +$$ + +2. (POL) Consider the system of equations + +$$ +\left\{\begin{array}{l} +a_{11} x_{1}+a_{12} x_{2}+a_{13} x_{3}=0 \\ +a_{21} x_{1}+a_{22} x_{2}+a_{23} x_{3}=0 \\ +a_{31} x_{1}+a_{32} x_{2}+a_{33} x_{3}=0 +\end{array}\right. +$$ + +whose coefficients satisfy the following conditions: +(a) $a_{11}, a_{22}, a_{33}$ are positive real numbers; +(b) all other coefficients are negative; +(c) in each of the equations the sum of the coefficients is positive. Prove that $x_{1}=x_{2}=x_{3}=0$ is the only solution to the system. +3. (CZS) A tetrahedron $A B C D$ is given. The lengths of the edges $A B$ and $C D$ are $a$ and $b$, respectively, the distance between the lines $A B$ and $C D$ is $d$, and the angle between them is equal to $\omega$. The tetrahedron is divided into two parts by the plane $\pi$ parallel to the lines $A B$ and $C D$. Calculate the ratio of the volumes of the parts if the ratio between the distances of the plane $\pi$ from $A B$ and $C D$ is equal to $k$. + +Second Day +4. (USS) Find four real numbers $x_{1}, x_{2}, x_{3}, x_{4}$ such that the sum of any of the numbers and the product of other three is equal to 2 . +5. (ROM) Given a triangle $O A B$ such that $\angle A O B=\alpha<90^{\circ}$, let $M$ be an arbitrary point of the triangle different from $O$. Denote by $P$ and $Q$ the feet of the perpendiculars from $M$ to $O A$ and $O B$, respectively. Let $H$ be the orthocenter of the triangle $O P Q$. Find the locus of points $H$ when: +(a) $M$ belongs to the segment $A B$; +(b) $M$ belongs to the interior of $\triangle O A B$. +6. (POL) We are given $n \geq 3$ points in the plane. Let $d$ be the maximal distance between two of the given points. Prove that the number of pairs of points whose distance is equal to $d$ is less than or equal to $n$. + +### 3.8 The Eighth IMO
Sofia, Bulgaria, July 3-13, 1966 + +### 3.8.1 Contest Problems + +First Day + +1. (USS) Three problems $A, B$, and $C$ were given on a mathematics olympiad. All 25 students solved at least one of these problems. The number of students who solved $B$ and not $A$ is twice the number of students who solved $C$ and not $A$. The number of students who solved only $A$ is greater by 1 than the number of students who along with $A$ solved at least one other problem. Among the students who solved only one problem, half solved $A$. How many students solved only $B$ ? +2. (HUN) If $a, b$, and $c$ are the sides and $\alpha, \beta$, and $\gamma$ the respective angles of the triangle for which $a+b=\tan \frac{\gamma}{2}(a \tan \alpha+b \tan \beta)$, prove that the triangle is isosceles. +3. (BUL) Prove that the sum of distances from the center of the circumsphere of the regular tetrahedron to its four vertices is less than the sum of distances from any other point to the four vertices. + +Second Day +4. (YUG) Prove the following equality: + +$$ +\frac{1}{\sin 2 x}+\frac{1}{\sin 4 x}+\frac{1}{\sin 8 x}+\cdots+\frac{1}{\sin 2^{n} x}=\cot x-\cot 2^{n} x +$$ + +where $n \in \mathbb{N}$ and $x \notin \pi \mathbb{Z} / 2^{k}$ for every $k \in \mathbb{N}$. +5. (CZS) Solve the following system of equations: + +$$ +\begin{aligned} +& \left|a_{1}-a_{2}\right| x_{2}+\left|a_{1}-a_{3}\right| x_{3}+\left|a_{1}-a_{4}\right| x_{4}=1, \\ +& \left|a_{2}-a_{1}\right| x_{1}+\left|a_{2}-a_{3}\right| x_{3}+\left|a_{2}-a_{4}\right| x_{4}=1, \\ +& \left|a_{3}-a_{1}\right| x_{1}+\left|a_{3}-a_{2}\right| x_{2}+\left|a_{3}-a_{4}\right| x_{4}=1, \\ +& \left|a_{4}-a_{1}\right| x_{1}+\left|a_{4}-a_{2}\right| x_{2}+\left|a_{4}-a_{3}\right| x_{3}=1, +\end{aligned} +$$ + +where $a_{1}, a_{2}, a_{3}$, and $a_{4}$ are mutually distinct real numbers. +6. (POL) Let $M, K$, and $L$ be points on $(A B),(B C)$, and $(C A)$, respectively. Prove that the area of at least one of the three triangles $\triangle M A L$, $\triangle K B M$, and $\triangle L C K$ is less than or equal to one-fourth the area of $\triangle A B C$. + +### 3.8.2 Some Longlisted Problems 1959-1966 + +1. (CZS) We are given $n>3$ points in the plane, no three of which lie on a line. Does there necessarily exist a circle that passes through at least three of the given points and contains none of the other given points in its interior? +2. (GDR) Given $n$ positive real numbers $a_{1}, a_{2}, \ldots, a_{n}$ such that $a_{1} a_{2} \cdots a_{n}$ $=1$, prove that + +$$ +\left(1+a_{1}\right)\left(1+a_{2}\right) \cdots\left(1+a_{n}\right) \geq 2^{n} . +$$ + +3. (BUL) A regular triangular prism has height $h$ and a base of side length $a$. Both bases have small holes in the centers, and the inside of the three vertical walls has a mirror surface. Light enters through the small hole in the top base, strikes each vertical wall once and leaves through the hole in the bottom. Find the angle at which the light enters and the length of its path inside the prism. +4. (POL) Five points in the plane are given, no three of which are collinear. Show that some four of them form a convex quadrilateral. +5. (USS) Prove the inequality + +$$ +\tan \frac{\pi \sin x}{4 \sin \alpha}+\tan \frac{\pi \cos x}{4 \cos \alpha}>1 +$$ + +for any $x, \alpha$ with $0 \leq x \leq \pi / 2$ and $\pi / 6\frac{3}{10 n}+\log _{10} n$; +(b) $\log n!>\frac{3 n}{10}\left(\frac{1}{2}+\frac{1}{3}+\cdots+\frac{1}{n}-1\right)$. +31. (ROM) Solve the equation $\left|x^{2}-1\right|+\left|x^{2}-4\right|=m x$ as a function of the parameter $m$. Which pairs $(x, m)$ of integers satisfy this equation? +32. (BUL) The sides $a, b, c$ of a triangle $A B C$ form an arithmetic progression; the sides of another triangle $A_{1} B_{1} C_{1}$ also form an arithmetic progression. + +Suppose that $\angle A=\angle A_{1}$. Prove that the triangles $A B C$ and $A_{1} B_{1} C_{1}$ are similar. +33. (BUL) Two circles touch each other from inside, and an equilateral triangle is inscribed in the larger circle. From the vertices of the triangle one draws segments tangent to the smaller circle. Prove that the length of one of these segments equals the sum of the lengths of the other two. +34. (BUL) Determine all pairs of positive integers $(x, y)$ satisfying the equation $2^{x}=3^{y}+5$. +35. (POL) If $a, b, c, d$ are integers such that $a d$ is odd and $b c$ is even, prove that at least one root of the polynomial $a x^{3}+b x^{2}+c x+d$ is irrational. +36. (POL) Let $A B C D$ be a cyclic quadrilateral. Show that the centroids of the triangles $A B C, C D A, B C D, D A B$ lie on a circle. +37. (POL) Prove that the perpendiculars drawn from the midpoints of the sides of a cyclic quadrilateral to the opposite sides meet at one point. +38. (ROM) Two concentric circles have radii $R$ and $r$ respectively. Determine the greatest possible number of circles that are tangent to both these circles and mutually nonintersecting. Prove that this number lies between $\frac{3}{2} \cdot \frac{\sqrt{R}+\sqrt{r}}{\sqrt{R}-\sqrt{r}}-1$ and $\frac{63}{20} \cdot \frac{R+r}{R-r}$. +39. (ROM) In a plane, a circle with center $O$ and radius $R$ and two points $A, B$ are given. +(a) Draw a chord $C D$ parallel to $A B$ so that $A C$ and $B D$ intersect at a point $P$ on the circle. +(b) Prove that there are two possible positions of point $P$, say $P_{1}, P_{2}$, and find the distance between them if $O A=a, O B=b, A B=d$. +40. (CZS) For a positive real number $p$, find all real solutions to the equation + +$$ +\sqrt{x^{2}+2 p x-p^{2}}-\sqrt{x^{2}-2 p x-p^{2}}=1 +$$ + +41. (CZS) If $A_{1} A_{2} \ldots A_{n}$ is a regular $n$-gon $(n \geq 3)$, how many different obtuse triangles $A_{i} A_{j} A_{k}$ exist? +42. (CZS) Let $a_{1}, a_{2}, \ldots, a_{n}(n \geq 2)$ be a sequence of integers. Show that there is a subsequence $a_{k_{1}}, a_{k_{2}}, \ldots, a_{k_{m}}$, where $1 \leq k_{1}1$, and the series terminates. +Show also that $x$ can be expressed as the sum of reciprocals of different integers, each of which is greater than $10^{6}$. +19. (GBR 6) The $n$ points $P_{1}, P_{2}, \ldots, P_{n}$ are placed inside or on the boundary of a disk of radius 1 in such a way that the minimum distance $d_{n}$ between any two of these points has its largest possible value $D_{n}$. Calculate $D_{n}$ for $n=2$ to 7 and justify your answer. +20. (HUN 1) In space, $n$ points $(n \geq 3)$ are given. Every pair of points determines some distance. Suppose all distances are different. Connect every point with the nearest point. Prove that it is impossible to obtain a polygonal line in such a way. ${ }^{1}$ +21. (HUN 2) Without using any tables, find the exact value of the product + +$$ +P=\cos \frac{\pi}{15} \cos \frac{2 \pi}{15} \cos \frac{3 \pi}{15} \cos \frac{4 \pi}{15} \cos \frac{5 \pi}{15} \cos \frac{6 \pi}{15} \cos \frac{7 \pi}{15} +$$ + +22. (HUN 3) The distance between the centers of the circles $k_{1}$ and $k_{2}$ with radii $r$ is equal to $r$. Points $A$ and $B$ are on the circle $k_{1}$, symmetric with respect to the line connecting the centers of the circles. Point $P$ is an arbitrary point on $k_{2}$. Prove that + +$$ +P A^{2}+P B^{2} \geq 2 r^{2} +$$ + +When does equality hold? +23. (HUN 4) Prove that for an arbitrary pair of vectors $f$ and $g$ in the plane, the inequality + +$$ +a f^{2}+b f g+c g^{2} \geq 0 +$$ + +holds if and only if the following conditions are fulfilled: $a \geq 0, c \geq 0$, $4 a c \geq b^{2}$. +24. (HUN 5) ${ }^{\text {IMO6 }}$ Father has left to his children several identical gold coins. According to his will, the oldest child receives one coin and one-seventh of the remaining coins, the next child receives two coins and one-seventh of the remaining coins, the third child receives three coins and one-seventh of the remaining coins, and so on through the youngest child. If every child inherits an integer number of coins, find the number of children and the number of coins. +25. (HUN 6) Three disks of diameter $d$ are touching a sphere at their centers. Moreover, each disk touches the other two disks. How do we choose the radius $R$ of the sphere so that the axis of the whole figure makes an angle + +[^0]of $60^{\circ}$ with the line connecting the center of the sphere with the point on the disks that is at the largest distance from the axis? (The axis of the figure is the line having the property that rotation of the figure through $120^{\circ}$ about that line brings the figure to its initial position. The disks are all on one side of the plane, pass through the center of the sphere, and are orthogonal to the axes.) +26. (ITA 1) Let $A B C D$ be a regular tetrahedron. To an arbitrary point $M$ on one edge, say $C D$, corresponds the point $P=P(M)$, which is the intersection of two lines $A H$ and $B K$, drawn from $A$ orthogonally to $B M$ and from $B$ orthogonally to $A M$. What is the locus of $P$ as $M$ varies? +27. (ITA 2) Which regular polygons can be obtained (and how) by cutting a cube with a plane? +28. (ITA 3) Find values of the parameter $u$ for which the expression +$$ +y=\frac{\tan (x-u)+\tan x+\tan (x+u)}{\tan (x-u) \tan x \tan (x+u)} +$$ +does not depend on $x$. +29. (ITA 4) ${ }^{\mathrm{IMO} 4}$ The triangles $A_{0} B_{0} C_{0}$ and $A^{\prime} B^{\prime} C^{\prime}$ have all their angles acute. Describe how to construct one of the triangles $A B C$, similar to $A^{\prime} B^{\prime} C^{\prime}$ and circumscribing $A_{0} B_{0} C_{0}$ (so that $A, B, C$ correspond to $A^{\prime}$, $B^{\prime}, C^{\prime}$, and $A B$ passes through $C_{0}, B C$ through $A_{0}$, and $C A$ through $B_{0}$ ). Among these triangles $A B C$, describe, and prove, how to construct the triangle with the maximum area. +30. (MON 1) Given $m+n$ numbers $a_{i}(i=1,2, \ldots, m), b_{j}(j=1,2, \ldots, n)$, determine the number of pairs $\left(a_{i}, b_{j}\right)$ for which $|i-j| \geq k$, where $k$ is a nonnegative integer. +31. (MON 2) An urn contains balls of $k$ different colors; there are $n_{i}$ balls of the $i$ th color. Balls are drawn at random from the urn, one by one, without replacement. Find the smallest number of draws necessary for getting $m$ balls of the same color. +32. (MON 3) Determine the volume of the body obtained by cutting the ball of radius $R$ by the trihedron with vertex in the center of that ball if its dihedral angles are $\alpha, \beta, \gamma$. +33. (MON 4) In what case does the system +\[ + +$$ +\begin{aligned} +& x+y+m z=a, \\ +& x+m y+z=b, \\ +& m x+y+z=c, +\end{aligned} +$$ +\] + +have a solution? Find the conditions under which the unique solution of the above system is an arithmetic progression. +34. (MON 5) The faces of a convex polyhedron are six squares and eight equilateral triangles, and each edge is a common side for one triangle and one square. All dihedral angles obtained from the triangle and square with a common edge are equal. Prove that it is possible to circumscribe a sphere around this polyhedron and compute the ratio of the squares of the volumes of the polyhedron and of the ball whose boundary is the circumscribed sphere. +35. (MON 6) Prove the identity + +$$ +\sum_{k=0}^{n}\binom{n}{k}\left(\tan \frac{x}{2}\right)^{2 k}\left[1+2^{k} \frac{1}{\left(1-\tan ^{2}(x / 2)\right)^{k}}\right]=\sec ^{2 n} \frac{x}{2}+\sec ^{n} x +$$ + +36. (POL 1) Prove that the center of the sphere circumscribed around a tetrahedron $A B C D$ coincides with the center of a sphere inscribed in that tetrahedron if and only if $A B=C D, A C=B D$, and $A D=B C$. +37. (POL 2) Prove that for arbitrary positive numbers the following inequality holds: + +$$ +\frac{1}{a}+\frac{1}{b}+\frac{1}{c} \leq \frac{a^{8}+b^{8}+c^{8}}{a^{3} b^{3} c^{3}} +$$ + +38. (POL 3) Does there exist an integer such that its cube is equal to $3 n^{2}+3 n+7$, where $n$ is integer? +39. (POL 4) Show that the triangle whose angles satisfy the equality + +$$ +\frac{\sin ^{2} A+\sin ^{2} B+\sin ^{2} C}{\cos ^{2} A+\cos ^{2} B+\cos ^{2} C}=2 +$$ + +is a right-angled triangle. +40. (POL 5) ${ }^{\mathrm{IMO} 2}$ Exactly one side of a tetrahedron is of length greater than 1. Show that its volume is less than or equal to $1 / 8$. +41. (POL 6) A line $l$ is drawn through the intersection point $H$ of the altitudes of an acute-angled triangle. Prove that the symmetric images $l_{a}$, $l_{b}, l_{c}$ of $l$ with respect to sides $B C, C A, A B$ have one point in common, which lies on the circumcircle of $A B C$. +42. (ROM 1) Decompose into real factors the expression $1-\sin ^{5} x-\cos ^{5} x$. +43. (ROM 2) The equation + +$$ +x^{5}+5 \lambda x^{4}-x^{3}+(\lambda \alpha-4) x^{2}-(8 \lambda+3) x+\lambda \alpha-2=0 +$$ + +is given. +(a) Determine $\alpha$ such that the given equation has exactly one root independent of $\lambda$. +(b) Determine $\alpha$ such that the given equation has exactly two roots independent of $\lambda$. +44. (ROM 3) Suppose $p$ and $q$ are two different positive integers and $x$ is a real number. Form the product $(x+p)(x+q)$. +(a) Find the sum $S(x, n)=\sum(x+p)(x+q)$, where $p$ and $q$ take values from 1 to $n$. +(b) Do there exist integer values of $x$ for which $S(x, n)=0$ ? +45. (ROM 4) (a) Solve the equation + +$$ +\sin ^{3} x+\sin ^{3}\left(\frac{2 \pi}{3}+x\right)+\sin ^{3}\left(\frac{4 \pi}{3}+x\right)+\frac{3}{4} \cos 2 x=0 . +$$ + +(b) Suppose the solutions are in the form of arcs $A B$ of the trigonometric circle (where $A$ is the beginning of arcs of the trigonometric circle), and $P$ is a regular $n$-gon inscribed in the circle with one vertex at $A$. +(1) Find the subset of arcs with the endpoint $B$ at a vertex of the regular dodecagon. +(2) Prove that the endpoint $B$ cannot be at a vertex of $P$ if $2,3 \nmid n$ or $n$ is prime. +46. (ROM 5) If $x, y, z$ are real numbers satisfying the relations $x+y+z=1$ and $\arctan x+\arctan y+\arctan z=\pi / 4$, prove that + +$$ +x^{2 n+1}+y^{2 n+1}+z^{2 n+1}=1 +$$ + +for all positive integers $n$. +47. (ROM 6) Prove the inequality +$x_{1} x_{2} \cdots x_{k}\left(x_{1}^{n-1}+x_{2}^{n-1}+\cdots+x_{k}^{n-1}\right) \leq x_{1}^{n+k-1}+x_{2}^{n+k-1}+\cdots+x_{k}^{n+k-1}$, where $x_{i}>0(i=1,2, \ldots, k), k \in N, n \in N$. +48. (SWE 1) Determine all positive roots of the equation $x^{x}=1 / \sqrt{2}$. +49. (SWE 2) Let $n$ and $k$ be positive integers such that $1 \leq n \leq N+1$, $1 \leq k \leq N+1$. Show that + +$$ +\min _{n \neq k}|\sin n-\sin k|<\frac{2}{N} +$$ + +50. (SWE 3) The function $\varphi(x, y, z)$, defined for all triples $(x, y, z)$ of real numbers, is such that there are two functions $f$ and $g$ defined for all pairs of real numbers such that + +$$ +\varphi(x, y, z)=f(x+y, z)=g(x, y+z) +$$ + +for all real $x, y$, and $z$. Show that there is a function $h$ of one real variable such that + +$$ +\varphi(x, y, z)=h(x+y+z) +$$ + +for all real $x, y$, and $z$. +51. (SWE 4) A subset $S$ of the set of integers $0, \ldots, 99$ is said to have property A if it is impossible to fill a crossword puzzle with 2 rows and 2 columns with numbers in $S$ ( 0 is written as 00,1 as 01 , and so on). Determine the maximal number of elements in sets $S$ with property A. +52. (SWE 5) In the plane a point $O$ and a sequence of points $P_{1}, P_{2}, P_{3}, \ldots$ are given. The distances $O P_{1}, O P_{2}, O P_{3}, \ldots$ are $r_{1}, r_{2}, r_{3}, \ldots$, where $r_{1} \leq$ $r_{2} \leq r_{3} \leq \cdots$. Let $\alpha$ satisfy $0<\alpha<1$. Suppose that for every $n$ the distance from the point $P_{n}$ to any other point of the sequence is greater than or equal to $r_{n}^{\alpha}$. Determine the exponent $\beta$, as large as possible, such that for some $C$ independent of $n,{ }^{2}$ + +$$ +r_{n} \geq C n^{\beta}, \quad n=1,2, \ldots +$$ + +53. (SWE 6) In making Euclidean constructions in geometry it is permitted to use a straightedge and compass. In the constructions considered in this question, no compasses are permitted, but the straightedge is assumed to have two parallel edges, which can be used for constructing two parallel lines through two given points whose distance is at least equal to the breadth of the ruler. Then the distance between the parallel lines is equal to the breadth of the straightedge. Carry through the following constructions with such a straightedge. Construct: +(a) The bisector of a given angle. +(b) The midpoint of a given rectilinear segment. +(c) The center of a circle through three given noncollinear points. +(d) A line through a given point parallel to a given line. +54. (USS 1) Is it possible to put 100 (or 200) points on a wooden cube such that by all rotations of the cube the points map into themselves? Justify your answer. +55. (USS 2) Find all $x$ for which for all $n$, + +$$ +\sin x+\sin 2 x+\sin 3 x+\cdots+\sin n x \leq \frac{\sqrt{3}}{2} +$$ + +56. (USS 3) In a group of interpreters each one speaks one or several foreign languages; 24 of them speak Japanese, 24 Malay, 24 Farsi. Prove that it is possible to select a subgroup in which exactly 12 interpreters speak Japanese, exactly 12 speak Malay, and exactly 12 speak Farsi. +57. (USS 4) ${ }^{\mathrm{IMO} 5}$ Consider the sequence $\left(c_{n}\right)$ : + +$$ +\begin{gathered} +c_{1}=a_{1}+a_{2}+\cdots+a_{8}, \\ +c_{2}=a_{1}^{2}+a_{2}^{2}+\cdots+a_{8}^{2}, \\ +\cdots \\ +\cdots \cdots \cdots \\ +c_{n}=a_{1}^{n}+a_{2}^{n}+\cdots+a_{8}^{n}, +\end{gathered} +$$ + +[^1]where $a_{1}, a_{2}, \ldots, a_{8}$ are real numbers, not all equal to zero. Given that among the numbers of the sequence $\left(c_{n}\right)$ there are infinitely many equal to zero, determine all the values of $n$ for which $c_{n}=0$. +58. (USS 5) A linear binomial $l(z)=A z+B$ with complex coefficients $A$ and $B$ is given. It is known that the maximal value of $|l(z)|$ on the segment $-1 \leq x \leq 1(y=0)$ of the real line in the complex plane $(z=x+i y)$ is equal to $M$. Prove that for every $z$ +$$ +|l(z)| \leq M \rho, +$$ +where $\rho$ is the sum of distances from the point $P=z$ to the points $Q_{1}$ : $z=1$ and $Q_{3}: z=-1$. +59. (USS 6) On the circle with center $O$ and radius 1 the point $A_{0}$ is fixed and points $A_{1}, A_{2}, \ldots, A_{999}, A_{1000}$ are distributed in such a way that $\angle A_{0} O A_{k}=k$ (in radians). Cut the circle at points $A_{0}, A_{1}, \ldots, A_{1000}$. How many arcs with different lengths are obtained? + +### 3.10 The Tenth IMO + +## Moscow-Leningrad, Soviet Union, July 5-18, 1968 + +### 3.10.1 Contest Problems + +First Day + +1. Prove that there exists a unique triangle whose side lengths are consecutive natural numbers and one of whose angles is twice the measure of one of the others. +2. Find all positive integers $x$ for which $p(x)=x^{2}-10 x-22$, where $p(x)$ denotes the product of the digits of $x$. +3. Let $a, b, c$ be real numbers. Prove that the system of equations + +$$ +\left\{\begin{array}{r} +a x_{1}^{2}+b x_{1}+c=x_{2} \\ +a x_{2}^{2}+b x_{2}+c=x_{3} \\ +\cdots \cdots \cdots \cdots \\ +a x_{n-1}^{2}+b x_{n-1}+c=x_{n} \\ +a x_{n}^{2}+b x_{n}+c=x_{1} +\end{array}\right. +$$ + +(a) has no real solutions if $(b-1)^{2}-4 a c<0$; +(b) has a unique real solution if $(b-1)^{2}-4 a c=0$; +(c) has more than one real solution if $(b-1)^{2}-4 a c>0$. + +## Second Day + +4. Prove that in any tetrahedron there is a vertex such that the lengths of its sides through that vertex are sides of a triangle. +5. Let $a>0$ be a real number and $f(x)$ a real function defined on all of $\mathbb{R}$, satisfying for all $x \in \mathbb{R}$, + +$$ +f(x+a)=\frac{1}{2}+\sqrt{f(x)-f(x)^{2}} +$$ + +(a) Prove that the function $f$ is periodic; i.e., there exists $b>0$ such that for all $x, f(x+b)=f(x)$. +(b) Give an example of such a nonconstant function for $a=1$. +6. Let $[x]$ denote the integer part of $x$, i.e., the greatest integer not exceeding $x$. If $n$ is a positive integer, express as a simple function of $n$ the sum + +$$ +\left[\frac{n+1}{2}\right]+\left[\frac{n+2}{4}\right]+\cdots+\left[\frac{n+2^{i}}{2^{i+1}}\right]+\cdots +$$ + +### 3.10.2 Shortlisted Problems + +1. (SWE 2) Two ships sail on the sea with constant speeds and fixed directions. It is known that at 9:00 the distance between them was 20 miles; at 9:35, 15 miles; and at 9:55, 13 miles. At what moment were the ships the smallest distance from each other, and what was that distance? +2. (ROM 5) ${ }^{\mathrm{IMO} 1}$ Prove that there exists a unique triangle whose side lengths are consecutive natural numbers and one of whose angles is twice the measure of one of the others. +3. (POL 4) ${ }^{\mathrm{IMO} 4}$ Prove that in any tetrahedron there is a vertex such that the lengths of its sides through that vertex are sides of a triangle. +4. (BUL 2) ${ }^{\mathrm{IMO} 3}$ Let $a, b, c$ be real numbers. Prove that the system of equations + +$$ +\left\{\begin{array}{r} +a x_{1}^{2}+b x_{1}+c=x_{2} \\ +a x_{2}^{2}+b x_{2}+c=x_{3} \\ +\cdots \cdots \cdots \cdots \\ +a x_{n-1}^{2}+b x_{n-1}+c=x_{n} \\ +a x_{n}^{2}+b x_{n}+c=x_{1} +\end{array}\right. +$$ + +has a unique real solution if and only if $(b-1)^{2}-4 a c=0$. +Remark. It is assumed that $a \neq 0$. +5. (BUL 5) Let $h_{n}$ be the apothem (distance from the center to one of the sides) of a regular $n$-gon $(n \geq 3)$ inscribed in a circle of radius $r$. Prove the inequality + +$$ +(n+1) h_{n+1}-n h_{n}>r . +$$ + +Also prove that if $r$ on the right side is replaced with a greater number, the inequality will not remain true for all $n \geq 3$. +6. (HUN 1) If $a_{i}(i=1,2, \ldots, n)$ are distinct non-zero real numbers, prove that the equation + +$$ +\frac{a_{1}}{a_{1}-x}+\frac{a_{2}}{a_{2}-x}+\cdots+\frac{a_{n}}{a_{n}-x}=n +$$ + +has at least $n-1$ real roots. +7. (HUN 5) Prove that the product of the radii of three circles exscribed to a given triangle does not exceed $\frac{3 \sqrt{3}}{8}$ times the product of the side lengths of the triangle. When does equality hold? +8. (ROM 2) Given an oriented line $\Delta$ and a fixed point $A$ on it, consider all trapezoids $A B C D$ one of whose bases $A B$ lies on $\Delta$, in the positive direction. Let $E, F$ be the midpoints of $A B$ and $C D$ respectively. +Find the loci of vertices $B, C, D$ of trapezoids that satisfy the following: +(i) $|A B| \leq a \quad(a$ fixed); +(ii) $|E F|=l \quad(l$ fixed); +(iii) the sum of squares of the nonparallel sides of the trapezoid is constant. Remark. The constants are chosen so that such trapezoids exist. +9. (ROM 3) Let $A B C$ be an arbitrary triangle and $M$ a point inside it. Let $d_{a}, d_{b}, d_{c}$ be the distances from $M$ to sides $B C, C A, A B ; a, b, c$ the lengths of the sides respectively, and $S$ the area of the triangle $A B C$. Prove the inequality + +$$ +a b d_{a} d_{b}+b c d_{b} d_{c}+c a d_{c} d_{a} \leq \frac{4 S^{2}}{3} +$$ + +Prove that the left-hand side attains its maximum when $M$ is the centroid of the triangle. +10. (ROM 4) Consider two segments of length $a, b(a>b)$ and a segment of length $c=\sqrt{a b}$. +(a) For what values of $a / b$ can these segments be sides of a triangle? +(b) For what values of $a / b$ is this triangle right-angled, obtuse-angled, or acute-angled? +11. (ROM 6) Find all solutions $\left(x_{1}, x_{2}, \ldots, x_{n}\right)$ of the equation +$1+\frac{1}{x_{1}}+\frac{x_{1}+1}{x_{1} x_{2}}+\frac{\left(x_{1}+1\right)\left(x_{2}+1\right)}{x_{1} x_{2} x_{3}}+\cdots+\frac{\left(x_{1}+1\right) \cdots\left(x_{n-1}+1\right)}{x_{1} x_{2} \cdots x_{n}}=0$. +12. (POL 1) If $a$ and $b$ are arbitrary positive real numbers and $m$ an integer, prove that + +$$ +\left(1+\frac{a}{b}\right)^{m}+\left(1+\frac{b}{a}\right)^{m} \geq 2^{m+1} +$$ + +13. (POL 5) Given two congruent triangles $A_{1} A_{2} A_{3}$ and $B_{1} B_{2} B_{3}\left(A_{i} A_{k}=\right.$ $B_{i} B_{k}$ ), prove that there exists a plane such that the orthogonal projections of these triangles onto it are congruent and equally oriented. +14. (BUL 5) A line in the plane of a triangle $A B C$ intersects the sides $A B$ and $A C$ respectively at points $X$ and $Y$ such that $B X=C Y$. Find the locus of the center of the circumcircle of triangle $X A Y$. +15. (GBR 1) ${ }^{\mathrm{IMO} 6}$ Let $[x]$ denote the integer part of $x$, i.e., the greatest integer not exceeding $x$. If $n$ is a positive integer, express as a simple function of $n$ the sum + +$$ +\left[\frac{n+1}{2}\right]+\left[\frac{n+2}{4}\right]+\cdots+\left[\frac{n+2^{i}}{2^{i+1}}\right]+\cdots +$$ + +16. (GBR 3) A polynomial $p(x)=a_{0} x^{k}+a_{1} x^{k-1}+\cdots+a_{k}$ with integer coefficients is said to be divisible by an integer $m$ if $p(x)$ is divisible by $m$ for all integers $x$. Prove that if $p(x)$ is divisible by $m$, then $k!a_{0}$ is also divisible by $m$. Also prove that if $a_{0}, k, m$ are nonnegative integers for which $k!a_{0}$ is divisible by $m$, there exists a polynomial $p(x)=a_{0} x^{k}+\cdots+$ $a_{k}$ divisible by $m$. +17. (GBR 4) Given a point $O$ and lengths $x, y, z$, prove that there exists an equilateral triangle $A B C$ for which $O A=x, O B=y, O C=z$, if and only if $x+y \geq z, y+z \geq x, z+x \geq y$ (the points $O, A, B, C$ are coplanar). +18. (ITA 2) If an acute-angled triangle $A B C$ is given, construct an equilateral triangle $A^{\prime} B^{\prime} C^{\prime}$ in space such that lines $A A^{\prime}, B B^{\prime}, C C^{\prime}$ pass through a given point. +19. (ITA 5) We are given a fixed point on the circle of radius 1, and going from this point along the circumference in the positive direction on curved distances $0,1,2, \ldots$ from it we obtain points with abscisas $n=0,1,2, \ldots$ respectively. How many points among them should we take to ensure that some two of them are less than the distance $1 / 5$ apart? +20. (CZS 1) Given $n(n \geq 3)$ points in space such that every three of them form a triangle with one angle greater than or equal to $120^{\circ}$, prove that these points can be denoted by $A_{1}, A_{2}, \ldots, A_{n}$ in such a way that for each $i, j, k, 1 \leq i0$ be a real number and $f(x)$ a real function defined on all of $\mathbb{R}$, satisfying for all $x \in \mathbb{R}$, + +$$ +f(x+a)=\frac{1}{2}+\sqrt{f(x)-f(x)^{2}} . +$$ + +(a) Prove that the function $f$ is periodic; i.e., there exists $b>0$ such that for all $x, f(x+b)=f(x)$. +(b) Give an example of such a nonconstant function for $a=1$. + +[^2] +### 3.11 The Eleventh IMO Bucharest, Romania, July 5-20, 1969 + +### 3.11.1 Contest Problems + +## First Day (July 10) + +1. Prove that there exist infinitely many natural numbers $a$ with the following property: the number $z=n^{4}+a$ is not prime for any natural number $n$. +2. Let $a_{1}, a_{2}, \ldots, a_{n}$ be real constants and + +$$ +y(x)=\cos \left(a_{1}+x\right)+\frac{\cos \left(a_{2}+x\right)}{2}+\frac{\cos \left(a_{3}+x\right)}{2^{2}}+\cdots+\frac{\cos \left(a_{n}+x\right)}{2^{n-1}} . +$$ + +If $x_{1}, x_{2}$ are real and $y\left(x_{1}\right)=y\left(x_{2}\right)=0$, prove that $x_{1}-x_{2}=m \pi$ for some integer $m$. +3. Find conditions on the positive real number $a$ such that there exists a tetrahedron $k$ of whose edges $(k=1,2,3,4,5)$ have length $a$, and the other $6-k$ edges have length 1 . + +Second Day (July 11) +4. Let $A B$ be a diameter of a circle $\gamma$. A point $C$ different from $A$ and $B$ is on the circle $\gamma$. Let $D$ be the projection of the point $C$ onto the line $A B$. Consider three other circles $\gamma_{1}, \gamma_{2}$, and $\gamma_{3}$ with the common tangent $A B: \gamma_{1}$ inscribed in the triangle $A B C$, and $\gamma_{2}$ and $\gamma_{3}$ tangent to both (the segment) $C D$ and $\gamma$. Prove that $\gamma_{1}, \gamma_{2}$, and $\gamma_{3}$ have two common tangents. +5. Given $n$ points in the plane such that no three of them are collinear, prove that one can find at least $\binom{n-3}{2}$ convex quadrilaterals with their vertices at these points. +6. Under the conditions $x_{1}, x_{2}>0, x_{1} y_{1}>z_{1}^{2}$, and $x_{2} y_{2}>z_{2}^{2}$, prove the inequality + +$$ +\frac{8}{\left(x_{1}+x_{2}\right)\left(y_{1}+y_{2}\right)-\left(z_{1}+z_{2}\right)^{2}} \leq \frac{1}{x_{1} y_{1}-z_{1}^{2}}+\frac{1}{x_{2} y_{2}-z_{2}^{2}} +$$ + +### 3.11.2 Longlisted Problems + +1. (BEL 1) A parabola $P_{1}$ with equation $x^{2}-2 p y=0$ and parabola $P_{2}$ with equation $x^{2}+2 p y=0, p>0$, are given. A line $t$ is tangent to $P_{2}$. Find the locus of pole $M$ of the line $t$ with respect to $P_{1}$. +2. (BEL 2) (a) Find the equations of regular hyperbolas passing through the points $A(\alpha, 0), B(\beta, 0)$, and $C(0, \gamma)$. +(b) Prove that all such hyperbolas pass through the orthocenter $H$ of the triangle $A B C$. +(c) Find the locus of the centers of these hyperbolas. +(d) Check whether this locus coincides with the nine-point circle of the triangle $A B C$. +3. (BEL 3) Construct the circle that is tangent to three given circles. +4. (BEL 4) Let $O$ be a point on a nondegenerate conic. A right angle with vertex $O$ intersects the conic at points $A$ and $B$. Prove that the line $A B$ passes through a fixed point located on the normal to the conic through the point $O$. +5. (BEL 5) Let $G$ be the centroid of the triangle $O A B$. +(a) Prove that all conics passing through the points $O, A, B, G$ are hyperbolas. +(b) Find the locus of the centers of these hyperbolas. +6. (BEL 6) Evaluate $(\cos (\pi / 4)+i \sin (\pi / 4))^{10}$ in two different ways and prove that + +$$ +\binom{10}{1}-\binom{10}{3}+\frac{1}{2}\binom{10}{5}=2^{4} +$$ + +7. (BUL 1) Prove that the equation $\sqrt{x^{3}+y^{3}+z^{3}}=1969$ has no integral solutions. +8. (BUL 2) Find all functions $f$ defined for all $x$ that satisfy the condition + +$$ +x f(y)+y f(x)=(x+y) f(x) f(y) +$$ + +for all $x$ and $y$. Prove that exactly two of them are continuous. +9. (BUL 3) One hundred convex polygons are placed on a square with edge of length 38 cm . The area of each of the polygons is smaller than $\pi \mathrm{cm}^{2}$, and the perimeter of each of the polygons is smaller than $2 \pi \mathrm{~cm}$. Prove that there exists a disk with radius 1 in the square that does not intersect any of the polygons. +10. (BUL 4) Let $M$ be the point inside the right-angled triangle $A B C$ ( $\angle C=90^{\circ}$ ) such that + +$$ +\angle M A B=\angle M B C=\angle M C A=\varphi . +$$ + +Let $\psi$ be the acute angle between the medians of $A C$ and $B C$. Prove that $\frac{\sin (\varphi+\psi)}{\sin (\varphi-\psi)}=5$. +11. (BUL 5) Let $Z$ be a set of points in the plane. Suppose that there exists a pair of points that cannot be joined by a polygonal line not passing through any point of $Z$. Let us call such a pair of points unjoinable. Prove that for each real $r>0$ there exists an unjoinable pair of points separated by distance $r$. +12. (CZS 1) Given a unit cube, find the locus of the centroids of all tetrahedra whose vertices lie on the sides of the cube. +13. (CZS 2) Let $p$ be a prime odd number. Is it possible to find $p-1$ natural numbers $n+1, n+2, \ldots, n+p-1$ such that the sum of the squares of these numbers is divisible by the sum of these numbers? +14. (CZS 3) Let $a$ and $b$ be two positive real numbers. If $x$ is a real solution of the equation $x^{2}+p x+q=0$ with real coefficients $p$ and $q$ such that $|p| \leq a,|q| \leq b$, prove that + +$$ +|x| \leq \frac{1}{2}\left(a+\sqrt{a^{2}+4 b}\right) . +$$ + +Conversely, if $x$ satisfies (1), prove that there exist real numbers $p$ and $q$ with $|p| \leq a,|q| \leq b$ such that $x$ is one of the roots of the equation $x^{2}+p x+q=0$. +15. (CZS 4) Let $K_{1}, \ldots, K_{n}$ be nonnegative integers. Prove that + +$$ +K_{1}!K_{2}!\cdots K_{n}!\geq[K / n]!^{n} +$$ + +where $K=K_{1}+\cdots+K_{n}$. +16. (CZS 5) A convex quadrilateral $A B C D$ with sides $A B=a, B C=b$, $C D=c, D A=d$ and angles $\alpha=\angle D A B, \beta=\angle A B C, \gamma=\angle B C D$, and $\delta=\angle C D A$ is given. Let $s=(a+b+c+d) / 2$ and $P$ be the area of the quadrilateral. Prove that + +$$ +P^{2}=(s-a)(s-b)(s-c)(s-d)-a b c d \cos ^{2} \frac{\alpha+\gamma}{2} +$$ + +17. (CZS 6) Let $d$ and $p$ be two real numbers. Find the first term of an arithmetic progression $a_{1}, a_{2}, a_{3}, \ldots$ with difference $d$ such that $a_{1} a_{2} a_{3} a_{4}=p$. Find the number of solutions in terms of $d$ and $p$. +18. (FRA 1) Let $a$ and $b$ be two nonnegative integers. Denote by $H(a, b)$ the set of numbers $n$ of the form $n=p a+q b$, where $p$ and $q$ are positive integers. Determine $H(a)=H(a, a)$. Prove that if $a \neq b$, it is enough to know all the sets $H(a, b)$ for coprime numbers $a, b$ in order to know all the sets $H(a, b)$. Prove that in the case of coprime numbers $a$ and $b, H(a, b)$ contains all numbers greater than or equal to $\omega=(a-1)(b-1)$ and also $\omega / 2$ numbers smaller than $\omega$. +19. (FRA 2) Let $n$ be an integer that is not divisible by any square greater than 1 . Denote by $x_{m}$ the last digit of the number $x^{m}$ in the number system with base $n$. For which integers $x$ is it possible for $x_{m}$ to be 0 ? Prove that the sequence $x_{m}$ is periodic with period $t$ independent of $x$. For which $x$ do we have $x_{t}=1$. Prove that if $m$ and $x$ are relatively prime, then $0_{m}, 1_{m}, \ldots,(n-1)_{m}$ are different numbers. Find the minimal period $t$ in terms of $n$. If $n$ does not meet the given condition, prove that it is possible to have $x_{m}=0 \neq x_{1}$ and that the sequence is periodic starting only from some number $k>1$. +20. (FRA 3) A polygon (not necessarily convex) with vertices in the lattice points of a rectangular grid is given. The area of the polygon is $S$. If $I$ is the number of lattice points that are strictly in the interior of the polygon and $B$ the number of lattice points on the border of the polygon, find the number $T=2 S-B-2 I+2$. +21. (FRA 4) A right-angled triangle $O A B$ has its right angle at the point $B$. An arbitrary circle with center on the line $O B$ is tangent to the line $O A$. Let $A T$ be the tangent to the circle different from $O A$ ( $T$ is the point of tangency). Prove that the median from $B$ of the triangle $O A B$ intersects $A T$ at a point $M$ such that $M B=M T$. +22. (FRA 5) Let $\alpha(n)$ be the number of pairs $(x, y)$ of integers such that $x+y=n, 0 \leq y \leq x$, and let $\beta(n)$ be the number of triples $(x, y, z)$ such that $x+y+z=n$ and $0 \leq z \leq y \leq x$. Find a simple relation between $\alpha(n)$ and the integer part of the number $\frac{n+2}{2}$ and the relation among $\beta(n)$, $\beta(n-3)$ and $\alpha(n)$. Then evaluate $\beta(n)$ as a function of the residue of $n$ modulo 6 . What can be said about $\beta(n)$ and $1+\frac{n(n+6)}{12}$ ? And what about $\frac{(n+3)^{2}}{6}$ ? +Find the number of triples $(x, y, z)$ with the property $x+y+z \leq n$, $0 \leq z \leq y \leq x$ as a function of the residue of $n$ modulo 6 . What can be said about the relation between this number and the number $\frac{(n+6)\left(2 n^{2}+9 n+12\right)}{72}$ ? +23. (FRA 6) Consider the integer $d=\frac{a^{b}-1}{c}$, where $a, b$, and $c$ are positive integers and $c \leq a$. Prove that the set $G$ of integers that are between 1 and $d$ and relatively prime to $d$ (the number of such integers is denoted by $\varphi(d)$ ) can be partitioned into $n$ subsets, each of which consists of $b$ elements. What can be said about the rational number $\frac{\varphi(d)}{b}$ ? +24. (GBR 1) The polynomial $P(x)=a_{0} x^{k}+a_{1} x^{k-1}+\cdots+a_{k}$, where $a_{0}, \ldots, a_{k}$ are integers, is said to be divisible by an integer $m$ if $P(x)$ is a multiple of $m$ for every integral value of $x$. Show that if $P(x)$ is divisible by $m$, then $a_{0} \cdot k$ ! is a multiple of $m$. Also prove that if $a, k, m$ are positive integers such that $a k!$ is a multiple of $m$, then a polynomial $P(x)$ with leading term $a x^{k}$ can be found that is divisible by $m$. +25. (GBR 2) Let $a, b, x, y$ be positive integers such that $a$ and $b$ have no common divisor greater than 1. Prove that the largest number not expressible in the form $a x+b y$ is $a b-a-b$. If $N(k)$ is the largest number not expressible in the form $a x+b y$ in only $k$ ways, find $N(k)$. +26. (GBR 3) A smooth solid consists of a right circular cylinder of height $h$ and base-radius $r$, surmounted by a hemisphere of radius $r$ and center $O$. The solid stands on a horizontal table. One end of a string is attached to a point on the base. The string is stretched (initially being kept in the vertical plane) over the highest point of the solid and held down at the point $P$ on the hemisphere such that $O P$ makes an angle $\alpha$ with +the horizontal. Show that if $\alpha$ is small enough, the string will slacken if slightly displaced and no longer remain in a vertical plane. If then pulled tight through $P$, show that it will cross the common circular section of the hemisphere and cylinder at a point $Q$ such that $\angle S O Q=\phi, S$ being where it initially crossed this section, and $\sin \phi=\frac{r \tan \alpha}{h}$. +27. (GBR 4) The segment $A B$ perpendicularly bisects $C D$ at $X$. Show that, subject to restrictions, there is a right circular cone whose axis passes through $X$ and on whose surface lie the points $A, B, C, D$. What are the restrictions? +28. (GBR 5) Let us define $u_{0}=0, u_{1}=1$ and for $n \geq 0, u_{n+2}=a u_{n+1}+b u_{n}$, $a$ and $b$ being positive integers. Express $u_{n}$ as a polynomial in $a$ and $b$. Prove the result. Given that $b$ is prime, prove that $b$ divides $a\left(u_{b}-1\right)$. +29. (GDR 1) Find all real numbers $\lambda$ such that the equation + +$$ +\sin ^{4} x-\cos ^{4} x=\lambda\left(\tan ^{4} x-\cot ^{4} x\right) +$$ + +(a) has no solution, +(b) has exactly one solution, +(c) has exactly two solutions, +(d) has more than two solutions (in the interval $(0, \pi / 4)$ ). +30. (GDR 2) ${ }^{\mathrm{IMO} 1}$ Prove that there exist infinitely many natural numbers $a$ with the following property: The number $z=n^{4}+a$ is not prime for any natural number $n$. +31. (GDR 3) Find the number of permutations $a_{1}, \ldots, a_{n}$ of the set $\{1,2, \ldots, n\}$ such that $\left|a_{i}-a_{i+1}\right| \neq 1$ for all $i=1,2, \ldots, n-1$. Find a recurrence formula and evaluate the number of such permutations for $n \leq 6$. +32. (GDR 4) Find the maximal number of regions into which a sphere can be partitioned by $n$ circles. +33. (GDR 5) Given a ring $G$ in the plane bounded by two concentric circles with radii $R$ and $R / 2$, prove that we can cover this region with 8 disks of radius $2 R / 5$. (A region is covered if each of its points is inside or on the border of some disk.) +34. (HUN 1) Let $a$ and $b$ be arbitrary integers. Prove that if $k$ is an integer not divisible by 3 , then $(a+b)^{2 k}+a^{2 k}+b^{2 k}$ is divisible by $a^{2}+a b+b^{2}$. +35. (HUN 2) Prove that + +$$ +1+\frac{1}{2^{3}}+\frac{1}{3^{3}}+\cdots+\frac{1}{n^{3}}<\frac{5}{4} +$$ + +36. (HUN 3) In the plane 4000 points are given such that each line passes through at most 2 of these points. Prove that there exist 1000 disjoint quadrilaterals in the plane with vertices at these points. +37. (HUN 4) ${ }^{\mathrm{IMO} 2}$ If $a_{1}, a_{2}, \ldots, a_{n}$ are real constants, and if + +$$ +y=\cos \left(a_{1}+x\right)+2 \cos \left(a_{2}+x\right)+\cdots+n \cos \left(a_{n}+x\right) +$$ + +has two zeros $x_{1}$ and $x_{2}$ whose difference is not a multiple of $\pi$, prove that $y \equiv 0$. +38. (HUN 5) Let $r$ and $m(r \leq m)$ be natural numbers and $A_{k}=\frac{2 k-1}{2 m} \pi$. Evaluate + +$$ +\frac{1}{m^{2}} \sum_{k=1}^{m} \sum_{l=1}^{m} \sin \left(r A_{k}\right) \sin \left(r A_{l}\right) \cos \left(r A_{k}-r A_{l}\right) +$$ + +39. (HUN 6) Find the positions of three points $A, B, C$ on the boundary of a unit cube such that $\min \{A B, A C, B C\}$ is the greatest possible. +40. (MON 1) Find the number of five-digit numbers with the following properties: there are two pairs of digits such that digits from each pair are equal and are next to each other, digits from different pairs are different, and the remaining digit (which does not belong to any of the pairs) is different from the other digits. +41. (MON 2) Given two numbers $x_{0}$ and $x_{1}$, let $\alpha$ and $\beta$ be coefficients of the equation $1-\alpha y-\beta y^{2}=0$. Under the given conditions, find an expression for the solution of the system + +$$ +x_{n+2}-\alpha x_{n+1}-\beta x_{n}=0, \quad n=0,1,2, \ldots +$$ + +42. (MON 3) Let $A_{k}(1 \leq k \leq h)$ be $n$-element sets such that each two of them have a nonempty intersection. Let $A$ be the union of all the sets $A_{k}$, and let $B$ be a subset of $A$ such that for each $k(1 \leq k \leq h)$ the intersection of $A_{k}$ and $B$ consists of exactly two different elements $a_{k}$ and $b_{k}$. Find all subsets $X$ of the set $A$ with $r$ elements satisfying the condition that for at least one index $k$, both elements $a_{k}$ and $b_{k}$ belong to $X$. +43. (MON 4) Let $p$ and $q$ be two prime numbers greater than 3 . Prove that if their difference is $2^{n}$, then for any two integers $m$ and $n$, the number $S=p^{2 m+1}+q^{2 m+1}$ is divisible by 3. +44. (MON 5) Find the radius of the circle circumscribed about the isosceles triangle whose sides are the solutions of the equation $x^{2}-a x+b=0$. +45. (MON 6) ${ }^{\mathrm{IMO5}}$ Given $n$ points in the plane such that no three of them are collinear, prove that one can find at least $\binom{n-3}{2}$ convex quadrilaterals with their vertices at these points. +46. (NET 1) The vertices of an $(n+1)$-gon are placed on the edges of a regular $n$-gon so that the perimeter of the $n$-gon is divided into equal parts. How does one choose these $n+1$ points in order to obtain the $(n+1)$ gon with +(a) maximal area; +(b) minimal area? +47. (NET 2) ${ }^{\mathrm{IMO} 4}$ Let $A$ and $B$ be points on the circle $\gamma$. A point $C$, different from $A$ and $B$, is on the circle $\gamma$. Let $D$ be the projection of the point $C$ onto the line $A B$. Consider three other circles $\gamma_{1}, \gamma_{2}$, and $\gamma_{3}$ with the common tangent $A B$ : $\gamma_{1}$ inscribed in the triangle $A B C$, and $\gamma_{2}$ and $\gamma_{3}$ tangent to both (the segment) $C D$ and $\gamma$. Prove that $\gamma_{1}, \gamma_{2}$, and $\gamma_{3}$ have two common tangents. +48. (NET 3) Let $x_{1}, x_{2}, x_{3}, x_{4}$, and $x_{5}$ be positive integers satisfying + +$$ +\begin{array}{r} +x_{1}+x_{2}+x_{3}+x_{4}+x_{5}=1000 \\ +x_{1}-x_{2}+x_{3}-x_{4}+x_{5}>0 \\ +x_{1}+x_{2}-x_{3}+x_{4}-x_{5}>0 \\ +-x_{1}+x_{2}+x_{3}-x_{4}+x_{5}>0 \\ +x_{1}-x_{2}+x_{3}+x_{4}-x_{5}>0 \\ +-x_{1}+x_{2}-x_{3}+x_{4}+x_{5}>0 +\end{array} +$$ + +(a) Find the maximum of $\left(x_{1}+x_{3}\right)^{x_{2}+x_{4}}$. +(b) In how many different ways can we choose $x_{1}, \ldots, x_{5}$ to obtain the desired maximum? +49. (NET 4) A boy has a set of trains and pieces of railroad track. Each piece is a quarter of circle, and by concatenating these pieces, the boy obtained a closed railway. The railway does not intersect itself. In passing through this railway, the train sometimes goes in the clockwise direction, and sometimes in the opposite direction. Prove that the train passes an even number of times through the pieces in the clockwise direction and an even number of times in the counterclockwise direction. Also, prove that the number of pieces is divisible by 4. +50. (NET 5) The bisectors of the exterior angles of a pentagon $B_{1} B_{2} B_{3} B_{4} B_{5}$ form another pentagon $A_{1} A_{2} A_{3} A_{4} A_{5}$. Construct $B_{1} B_{2} B_{3} B_{4} B_{5}$ from the given pentagon $A_{1} A_{2} A_{3} A_{4} A_{5}$. +51. (NET 6) A curve determined by + +$$ +y=\sqrt{x^{2}-10 x+52}, \quad 0 \leq x \leq 100 +$$ + +is constructed in a rectangular grid. Determine the number of squares cut by the curve. +52. (POL 1) Prove that a regular polygon with an odd number of edges cannot be partitioned into four pieces with equal areas by two lines that pass through the center of polygon. +53. (POL 2) Given two segments $A B$ and $C D$ not in the same plane, find the locus of points $M$ such that + +$$ +M A^{2}+M B^{2}=M C^{2}+M D^{2} +$$ + +54. (POL 3) Given a polynomial $f(x)$ with integer coefficients whose value is divisible by 3 for three integers $k, k+1$, and $k+2$, prove that $f(m)$ is divisible by 3 for all integers $m$. +55. (POL 4) ${ }^{\mathrm{IMO} 3}$ Find the conditions on the positive real number $a$ such that there exists a tetrahedron $k$ of whose edges $(k=1,2,3,4,5)$ have length $a$, and the other $6-k$ edges have length 1. +56. (POL 5) Let $a$ and $b$ be two natural numbers that have an equal number $n$ of digits in their decimal expansions. The first $m$ digits (from left to right) of the numbers $a$ and $b$ are equal. Prove that if $m>n / 2$, then + +$$ +a^{1 / n}-b^{1 / n}<\frac{1}{n} . +$$ + +57. (POL 6) On the sides $A B$ and $A C$ of triangle $A B C$ two points $K$ and $L$ are given such that $\frac{K B}{A K}+\frac{L C}{A L}=1$. Prove that $K L$ passes through the centroid of $A B C$. +58. (SWE 1) Six points $P_{1}, \ldots, P_{6}$ are given in 3 -dimensional space such that no four of them lie in the same plane. Each of the line segments $P_{j} P_{k}$ is colored black or white. Prove that there exists one triangle $P_{j} P_{k} P_{l}$ whose edges are of the same color. +59. (SWE 2) For each $\lambda(0<\lambda<1$ and $\lambda \neq 1 / n$ for all $n=1,2,3, \ldots)$ construct a continuous function $f$ such that there do not exist $x, y$ with $0<\lambda2$, + +$$ +(n!)!>n[(n-1)!]^{n!} +$$ + +65. (USS 2) Prove that for $a>b^{2}$, + +$$ +\sqrt{a-b \sqrt{a+b \sqrt{a-b \sqrt{a+\cdots}}}}=\sqrt{a-\frac{3}{4} b^{2}}-\frac{1}{2} b . +$$ + +66. (USS 3) (a) Prove that if $0 \leq a_{0} \leq a_{1} \leq a_{2}$, then + +$$ +\left(a_{0}+a_{1} x-a_{2} x^{2}\right)^{2} \leq\left(a_{0}+a_{1}+a_{2}\right)^{2}\left(1+\frac{1}{2} x+\frac{1}{3} x^{2}+\frac{1}{2} x^{3}+x^{4}\right) +$$ + +(b) Formulate and prove the analogous result for polynomials of third degree. +67. (USS 4) ${ }^{\text {IMO6 }}$ Under the conditions $x_{1}, x_{2}>0, x_{1} y_{1}>z_{1}^{2}$, and $x_{2} y_{2}>z_{2}^{2}$, prove the inequality + +$$ +\frac{8}{\left(x_{1}+x_{2}\right)\left(y_{1}+y_{2}\right)-\left(z_{1}+z_{2}\right)^{2}} \leq \frac{1}{x_{1} y_{1}-z_{1}^{2}}+\frac{1}{x_{2} y_{2}-z_{2}^{2}} +$$ + +68. (USS 5) Given 5 points in the plane, no three of which are collinear, prove that we can choose 4 points among them that form a convex quadrilateral. +69. (YUG 1) Suppose that positive real numbers $x_{1}, x_{2}, x_{3}$ satisfy + +$$ +x_{1} x_{2} x_{3}>1, \quad x_{1}+x_{2}+x_{3}<\frac{1}{x_{1}}+\frac{1}{x_{2}}+\frac{1}{x_{3}} +$$ + +Prove that: +(a) None of $x_{1}, x_{2}, x_{3}$ equals 1 . +(b) Exactly one of these numbers is less than 1. +70. (YUG 2) A park has the shape of a convex pentagon of area $5 \sqrt{3}$ ha $\left(=50000 \sqrt{3} \mathrm{~m}^{2}\right)$. A man standing at an interior point $O$ of the park notices that he stands at a distance of at most 200 m from each vertex of the pentagon. Prove that he stands at a distance of at least 100 m from each side of the pentagon. +71. (YUG 3) Let four points $A_{i}(i=1,2,3,4)$ in the plane determine four triangles. In each of these triangles we choose the smallest angle. The sum of these angles is denoted by $S$. What is the exact placement of the points $A_{i}$ if $S=180^{\circ}$ ? + +### 3.12 The Twelfth IMO Budapest-Keszthely, Hungary, July 8-22, 1970 + +### 3.12.1 Contest Problems + +First Day (July 13) + +1. Given a point $M$ on the side $A B$ of the triangle $A B C$, let $r_{1}$ and $r_{2}$ be the radii of the inscribed circles of the triangles $A C M$ and $B C M$ respectively while $\rho_{1}$ and $\rho_{2}$ are the radii of the excircles of the triangles $A C M$ and $B C M$ at the sides $A M$ and $B M$ respectively. Let $r$ and $\rho$ denote the respective radii of the inscribed circle and the excircle at the side $A B$ of the triangle $A B C$. Prove that + +$$ +\frac{r_{1}}{\rho_{1}} \frac{r_{2}}{\rho_{2}}=\frac{r}{\rho} . +$$ + +2. Let $a$ and $b$ be the bases of two number systems and let + +$$ +\begin{array}{ll} +A_{n}={\overline{x_{1} x_{2} \ldots x_{n}}}^{(a)}, & A_{n+1}={\overline{x_{0} x_{1} x_{2} \ldots x_{n}}}^{(a)}, \\ +B_{n}={\overline{x_{1} x_{2} \ldots x_{n}}}^{(b)}, & B_{n+1}={\overline{x_{0} x_{1} x_{2} \ldots x_{n}}}^{(b)}, +\end{array} +$$ + +be numbers in the number systems with respective bases $a$ and $b$, so that $x_{0}, x_{1}, x_{2}, \ldots, x_{n}$ denote digits in the number system with base $a$ as well as in the number system with base $b$. Suppose that neither $x_{0}$ nor $x_{1}$ is zero. Prove that $a>b$ if and only if + +$$ +\frac{A_{n}}{A_{n+1}}<\frac{B_{n}}{B_{n+1}} . +$$ + +3. Let $1=a_{0} \leq a_{1} \leq a_{2} \leq \cdots \leq a_{n} \leq \cdots$ be a sequence of real numbers. Consider the sequence $b_{1}, b_{2}, \ldots$ defined by + +$$ +b_{n}=\sum_{k=1}^{n}\left(1-\frac{a_{k-1}}{a_{k}}\right) \frac{1}{\sqrt{a_{k}}} . +$$ + +Prove that: +(a) For all natural numbers $n, 0 \leq b_{n}<2$. +(b) Given an arbitrary $0 \leq b<2$, there is a sequence $a_{0}, a_{1}, \ldots, a_{n}, \ldots$ of the above type such that $b_{n}>b$ is true for an infinity of natural numbers $n$. + +## Second Day (July 14) + +4. For what natural numbers $n$ can the product of some of the numbers $n, n+1, n+2, n+3, n+4, n+5$ be equal to the product of the remaining ones? +5. In the tetrahedron $A B C D$, the edges $B D$ and $C D$ are mutually perpendicular, and the projection of the vertex $D$ to the plane $A B C$ is the intersection of the altitudes of the triangle $A B C$. Prove that + +$$ +(A B+B C+C A)^{2} \leq 6\left(D A^{2}+D B^{2}+D C^{2}\right) +$$ + +For which tetrahedra does equality hold? +6. Given 100 points in the plane, no three of which are on the same line, consider all triangles that have all their vertices chosen from the 100 given points. Prove that at most $70 \%$ of these triangles are acute-angled. + +### 3.12.2 Longlisted Problems + +1. (AUT 1) Prove that + +$$ +\frac{b c}{b+c}+\frac{c a}{c+a}+\frac{a b}{a+b} \leq \frac{1}{2}(a+b+c) \quad(a, b, c>0) +$$ + +2. (AUT 2) Prove that the two last digits of $9^{9^{9}}$ and $9^{9^{9^{9}}}$ in decimal representation are equal. +3. (AUT 3) Prove that for $a, b \in \mathbb{N}$, $a!b$ ! divides $(a+b)$ !. +4. (AUT 4) Solve the system of equations + +$$ +\begin{aligned} +& x^{2}+x y=a^{2}+a b \\ +& y^{2}+x y=a^{2}-a b, \quad a, b \text { real, } a \neq 0 . +\end{aligned} +$$ + +5. (AUT 5) Prove that $\sqrt[n]{\frac{1}{n+1}+\frac{2}{n+1}+\cdots+\frac{n}{n+1}} \geq 1$ for $n \geq 2$. +6. (BEL 1) Prove that the equation in $x$ + +$$ +\sum_{i=1}^{n} \frac{b_{i}}{x-a_{i}}=c, \quad b_{i}>0, \quad a_{1}1$ be a natural number, $a \geq 1$ a real number, and $x_{1}, x_{2}, \ldots, x_{n}$ numbers such that $x_{1}=1, \frac{x_{k+1}}{x_{k}}=a+\alpha_{k}$ for $k=1,2, \ldots, n-$ 1 , where $\alpha_{k}$ are real numbers with $\alpha_{k} \leq \frac{1}{k(k+1)}$. Prove that + +$$ +\sqrt[n-1]{x_{n}}\sqrt{2 a^{2}+h^{2}}$, and walks on the ground around the hole. The edges of the hole are smooth, so that the rope can freely slide along it. Find the shape and area of the territory accessible to the dog (whose size is neglected). +57. (USS 3) Let the numbers $1,2, \ldots, n^{2}$ be written in the cells of an $n \times n$ square board so that the entries in each column are arranged increasingly. What are the smallest and greatest possible sums of the numbers in the $k$ th row? ( $k$ a positive integer, $1 \leq k \leq n$.) +58. (USS 4) (SL70-12). +59. (USS 5) (SL70-7). + +### 3.12.3 Shortlisted Problems + +1. (BEL 3) Consider a regular $2 n$-gon and the $n$ diagonals of it that pass through its center. Let $P$ be a point of the inscribed circle and let $a_{1}, a_{2}, \ldots, a_{n}$ be the angles in which the diagonals mentioned are visible from the point $P$. Prove that + +$$ +\sum_{i=1}^{n} \tan ^{2} a_{i}=2 n \frac{\cos ^{2} \frac{\pi}{2 n}}{\sin ^{4} \frac{\pi}{2 n}} +$$ + +2. (ROM 1) ${ }^{\mathrm{IMO} 2}$ Let $a$ and $b$ be the bases of two number systems and let + +$$ +\begin{array}{ll} +A_{n}={\overline{x_{1} x_{2} \ldots x_{n}}}^{(a)}, & A_{n+1}={\overline{x_{0} x_{1} x_{2} \ldots x_{n}}}^{(a)}, \\ +B_{n}={\overline{x_{1} x_{2} \ldots x_{n}}}^{(b)}, & B_{n+1}={\overline{x_{0} x_{1} x_{2} \ldots x_{n}}}^{(b)}, +\end{array} +$$ + +be numbers in the number systems with respective bases $a$ and $b$, so that $x_{0}, x_{1}, x_{2}, \ldots, x_{n}$ denote digits in the number system with base $a$ as well as in the number system with base $b$. Suppose that neither $x_{0}$ nor $x_{1}$ is zero. Prove that $a>b$ if and only if + +$$ +\frac{A_{n}}{A_{n+1}}<\frac{B_{n}}{B_{n+1}} . +$$ + +3. (BUL 6) ${ }^{\mathrm{IMO}}$ In the tetrahedron $S A B C$ the angle $B S C$ is a right angle, and the projection of the vertex $S$ to the plane $A B C$ is the intersection of the altitudes of the triangle $A B C$. Let $z$ be the radius of the inscribed circle of the triangle $A B C$. Prove that + +$$ +S A^{2}+S B^{2}+S C^{2} \geq 18 z^{2} +$$ + +4. (CZS 1) $)^{\mathrm{IMO} 4}$ For what natural numbers $n$ can the product of some of the numbers $n, n+1, n+2, n+3, n+4, n+5$ be equal to the product of the remaining ones? +5. (CZS 3) Let $M$ be an interior point of the tetrahedron $A B C D$. Prove that + +$$ +\begin{aligned} +& \overrightarrow{M A} \operatorname{vol}(M B C D)+\overrightarrow{M B} \operatorname{vol}(M A C D) \\ +& \quad+\overrightarrow{M C} \operatorname{vol}(M A B D)+\overrightarrow{M D} \operatorname{vol}(M A B C)=0 +\end{aligned} +$$ + +( $\operatorname{vol}(P Q R S)$ denotes the volume of the tetrahedron $P Q R S)$. +6. (FRA 1) In the triangle $A B C$ let $B^{\prime}$ and $C^{\prime}$ be the midpoints of the sides $A C$ and $A B$ respectively and $H$ the foot of the altitude passing through the vertex $A$. Prove that the circumcircles of the triangles $A B^{\prime} C^{\prime}, B C^{\prime} H$, and $B^{\prime} C H$ have a common point $I$ and that the line $H I$ passes through the midpoint of the segment $B^{\prime} C^{\prime}$. +7. (USS 5) For which digits $a$ do exist integers $n \geq 4$ such that each digit of $\frac{n(n+1)}{2}$ equals $a$ ? +8. (POL 2) ${ }^{\mathrm{IMO} 1}$ Given a point $M$ on the side $A B$ of the triangle $A B C$, let $r_{1}$ and $r_{2}$ be the radii of the inscribed circles of the triangles $A C M$ and $B C M$ respectively and let $\rho_{1}$ and $\rho_{2}$ be the radii of the excircles of the triangles $A C M$ and $B C M$ at the sides $A M$ and $B M$ respectively. Let $r$ and $\rho$ denote the radii of the inscribed circle and the excircle at the side $A B$ of the triangle $A B C$ respectively. Prove that + +$$ +\frac{r_{1}}{\rho_{1}} \frac{r_{2}}{\rho_{2}}=\frac{r}{\rho} +$$ + +9. (GDR 3) Let $u_{1}, u_{2}, \ldots, u_{n}, v_{1}, v_{2}, \ldots, v_{n}$ be real numbers. Prove that + +$$ +1+\sum_{i=1}^{n}\left(u_{i}+v_{i}\right)^{2} \leq \frac{4}{3}\left(1+\sum_{i=1}^{n} u_{i}^{2}\right)\left(1+\sum_{i=1}^{n} v_{i}^{2}\right) +$$ + +In what case does equality hold? +10. (SWE 4) ${ }^{\mathrm{IMO} 3}$ Let $1=a_{0} \leq a_{1} \leq a_{2} \leq \cdots \leq a_{n} \leq \cdots$ be a sequence of real numbers. Consider the sequence $b_{1}, b_{2}, \ldots$ defined by: + +$$ +b_{n}=\sum_{k=1}^{n}\left(1-\frac{a_{k-1}}{a_{k}}\right) \frac{1}{\sqrt{a_{k}}} +$$ + +Prove that: +(a) For all natural numbers $n, 0 \leq b_{n}<2$. +(b) Given an arbitrary $0 \leq b<2$, there is a sequence $a_{0}, a_{1}, \ldots, a_{n}, \ldots$ of the above type such that $b_{n}>b$ is true for infinitely many natural numbers $n$. +11. (SWE 6) Let $P, Q, R$ be polynomials and let $S(x)=P\left(x^{3}\right)+x Q\left(x^{3}\right)+$ $x^{2} R\left(x^{3}\right)$ be a polynomial of degree $n$ whose roots $x_{1}, \ldots, x_{n}$ are distinct. Construct with the aid of the polynomials $P, Q, R$ a polynomial $T$ of degree $n$ that has the roots $x_{1}^{3}, x_{2}^{3}, \ldots, x_{n}^{3}$. +12. (USS 4) ${ }^{\mathrm{IMO} 6}$ We are given 100 points in the plane, no three of which are on the same line. Consider all triangles that have all vertices chosen from the 100 given points. Prove that at most $70 \%$ of these triangles are acute angled. + +### 3.13 The Thirteenth IMO Bratislava-Zilina, Czechoslovakia, July 10-21, 1971 + +### 3.13.1 Contest Problems + +First Day (July 13) + +1. Prove that the following statement is true for $n=3$ and for $n=5$, and false for all other $n>2$ : +For any real numbers $a_{1}, a_{2}, \ldots, a_{n}$, + +$$ +\begin{gathered} +\left(a_{1}-a_{2}\right)\left(a_{1}-a_{3}\right) \cdots\left(a_{1}-a_{n}\right)+\left(a_{2}-a_{1}\right)\left(a_{2}-a_{3}\right) \cdots\left(a_{2}-a_{n}\right)+\ldots \\ ++\left(a_{n}-a_{1}\right)\left(a_{n}-a_{2}\right) \cdots\left(a_{n}-a_{n-1}\right) \geq 0 +\end{gathered} +$$ + +2. Given a convex polyhedron $P_{1}$ with 9 vertices $A_{1}, \ldots, A_{9}$, let us denote by $P_{2}, P_{3}, \ldots, P_{9}$ the images of $P_{1}$ under the translations mapping the vertex $A_{1}$ to $A_{2}, A_{3}, \ldots, A_{9}$, respectively. Prove that among the polyhedra $P_{1}, \ldots, P_{9}$ at least two have a common interior point. +3. Prove that the sequence $2^{n}-3(n>1)$ contains a subsequence of numbers relatively prime in pairs. + +## Second Day (July 14) + +4. Given a tetrahedron $A B C D$ all of whose faces are acute-angled triangles, set + +$$ +\sigma=\measuredangle D A B+\measuredangle B C D-\measuredangle A B C-\measuredangle C D A +$$ + +Consider all closed broken lines $X Y Z T X$ whose vertices $X, Y, Z, T$ lie in the interior of segments $A B, B C, C D, D A$ respectively. Prove that: +(a) if $\sigma \neq 0$, then there is no broken line $X Y Z T$ of minimal length; +(b) if $\sigma=0$, then there are infinitely many such broken lines of minimal length. That length equals $2 A C \sin (\alpha / 2)$, where + +$$ +\alpha=\measuredangle B A C+\measuredangle C A D+\measuredangle D A B +$$ + +5. Prove that for every natural number $m \geq 1$ there exists a finite set $S_{m}$ of points in the plane satisfying the following condition: If $A$ is any point in $S_{m}$, then there are exactly $m$ points in $S_{m}$ whose distance to $A$ equals 1. +6. Consider the $n \times n$ array of nonnegative integers + +$$ +\left(\begin{array}{cccc} +a_{11} & a_{12} & \ldots & a_{1 n} \\ +a_{21} & a_{22} & \cdots & a_{2 n} \\ +\vdots & \vdots & & \vdots \\ +a_{n 1} & a_{n 2} & \ldots & a_{n n} +\end{array}\right) +$$ + +with the following property: If an element $a_{i j}$ is zero, then the sum of the elements of the $i$ th row and the $j$ th column is greater than or equal to $n$. Prove that the sum of all the elements is greater than or equal to $\geq \frac{1}{2} n^{2}$. + +### 3.13.2 Longlisted Problems + +1. (AUT 1) The points $S(i, j)$ with integer Cartesian coordinates $0\frac{76}{16} n$ holds. +3. (AUT 3) Let $a, b, c$ be positive real numbers, $0
a>0$, and $d>b>0$. +Show that if there exists a right circular cone with vertex $V$, with the properties: +(1) its axis passes through $O$, and +(2) its curved surface passes through $A, B, C$ and $D$, then + +$$ +O V^{2}=\frac{d^{2} b^{2}(c+a)^{2}-c^{2} a^{2}(d+b)^{2}}{c a(d-b)^{2}-d b(c-a)^{2}} +$$ + +[^3]Show also that if $\frac{c+a}{d+b}$ lies between $\frac{c a}{d b}$ and $\sqrt{\frac{c a}{d b}}$, and $\frac{c-a}{d-b}=\frac{c a}{d b}$, then for a suitable choice of $\theta$, a right circular cone exists with properties (1) and (2). +16. (GBR 3) (SL71-4). + +Original formulation. Two (intersecting) circles are given and a point $P$ through which it is possible to draw a straight line on which the circles intercept two equal chords. Describe a construction by straightedge and compass for the straight line and prove the validity of your construction. +17. (GDR 1) (SL71-3). + +Original formulation. Find all solutions of the system + +$$ +\begin{aligned} +x+y+z & =3, \\ +x^{3}+y^{3}+z^{3} & =15, \\ +x^{5}+y^{5}+z^{5} & =83 . +\end{aligned} +$$ + +18. (GDR 2) Let $a_{1}, a_{2}, \ldots, a_{n}$ be positive numbers, $m_{g}=\left(a_{1} a_{2} \cdots a_{n}\right)^{1 / n}$ their geometric mean, and $m_{a}=\left(a_{1}+a_{2}+\cdots+a_{n}\right) / n$ their arithmetic mean. Prove that + +$$ +\left(1+m_{g}\right)^{n} \leq\left(1+a_{1}\right) \cdots\left(1+a_{n}\right) \leq\left(1+m_{a}\right)^{n} +$$ + +19. (GDR 3) In a triangle $P_{1} P_{2} P_{3}$ let $P_{i} Q_{i}$ be the altitude from $P_{i}$ for $i=1,2,3$ ( $Q_{i}$ being the foot of the altitude). The circle with diameter $P_{i} Q_{i}$ meets the two corresponding sides at two points different from $P_{i}$. Denote the length of the segment whose endpoints are these two points by $l_{i}$. Prove that $l_{1}=l_{2}=l_{3}$. +20. (GDR 4) Let $M$ be the circumcenter of a triangle $A B C$. The line through $M$ perpendicular to $C M$ meets the lines $C A$ and $C B$ at $Q$ and $P$ respectively. Prove that + +$$ +\frac{\overline{C P}}{\overline{C M}} \frac{\overline{C Q}}{\overline{C M}} \frac{\overline{A B}}{\overline{P Q}}=2 . +$$ + +21. (HUN 1) (SL71-5). +22. (HUN 2) We are given an $n \times n$ board, where $n$ is an odd number. In each cell of the board either +1 or -1 is written. Let $a_{k}$ and $b_{k}$ denote the products of numbers in the $k$ th row and in the $k$ th column respectively. Prove that the sum $a_{1}+a_{2}+\cdots+a_{n}+b_{1}+b_{2}+\cdots+b_{n}$ cannot be equal to zero. +23. (HUN 3) Find all integer solutions of the equation + +$$ +x^{2}+y^{2}=(x-y)^{3} +$$ + +24. (HUN 4) Let $A, B$, and $C$ denote the angles of a triangle. If $\sin ^{2} A+$ $\sin ^{2} B+\sin ^{2} C=2$, prove that the triangle is right-angled. +25. (HUN 5) Let $A B C, A A_{1} A_{2}, B B_{1} B_{2}, C C_{1} C_{2}$ be four equilateral triangles in the plane satisfying only that they are all positively oriented (i.e., in the counterclockwise direction). Denote the midpoints of the segments $A_{2} B_{1}, B_{2} C_{1}, C_{2} A_{1}$ by $P, Q, R$ in this order. Prove that the triangle $P Q R$ is equilateral. +26. (HUN 6) An infinite set of rectangles in the Cartesian coordinate plane is given. The vertices of each of these rectangles have coordinates $(0,0),(p, 0),(p, q),(0, q)$ for some positive integers $p, q$. Show that there must exist two among them one of which is entirely contained in the other. +27. (HUN 7) (SL71-6). +28. (NET 1) (SL71-7). + +Original formulation. A tetrahedron $A B C D$ is given. The sum of angles of the tetrahedron at the vertex $A$ (namely $\angle B A C, \angle C A D, \angle D A B$ ) is denoted by $\alpha$, and $\beta, \gamma, \delta$ are defined analogously. Let $P, Q, R, S$ be variable points on edges of the tetrahedron: $P$ on $A D, Q$ on $B D, R$ on $B C$, and $S$ on $A C$, none of them at some vertex of $A B C D$. Prove that: +(a) if $\alpha+\beta \neq 2 \pi$, then $P Q+Q R+R S+S P$ attains no minimal value; +(b) if $\alpha+\beta=2 \pi$, then + +$$ +A B \sin \frac{\alpha}{2}=C D \sin \frac{\gamma}{2} \quad \text { and } \quad P Q+Q R+R S+S P \geq 2 A B \sin \frac{\alpha}{2} +$$ + +29. (NET 2) A rhombus with its incircle is given. At each vertex of the rhombus a circle is constructed that touches the incircle and two edges of the rhombus. These circles have radii $r_{1}, r_{2}$, while the incircle has radius $r$. Given that $r_{1}$ and $r_{2}$ are natural numbers and that $r_{1} r_{2}=r$, find $r_{1}, r_{2}$, and $r$. +30. (NET 3) Prove that the system of equations + +$$ +\begin{aligned} +& 2 y z+x-y-z=a, \\ +& 2 x z-x+y-z=a, \\ +& 2 x y-x-y+z=a, +\end{aligned} +$$ + +$a$ being a parameter, cannot have five distinct solutions. For what values of $a$ does this system have four distinct integer solutions? +31. (NET 4) (SL71-8). +32. (NET 5) Two half-lines $a$ and $b$, with the common endpoint $O$, make an acute angle $\alpha$. Let $A$ on $a$ and $B$ on $b$ be points such that $O A=O B$, and let $b^{\prime}$ be the line through $A$ parallel to $b$. Let $\beta$ be the circle with center $B$ and radius $B O$. We construct a sequence of half-lines $c_{1}, c_{2}, c_{3}, \ldots$, all lying inside the angle $\alpha$, in the following manner: +(i) $c_{1}$ is given arbitrarily; +(ii) for every natural number $k$, the circle $\beta$ intercepts on $c_{k}$ a segment that is of the same length as the segment cut on $b^{\prime}$ by $a$ and $c_{k+1}$. +Prove that the angle determined by the lines $c_{k}$ and $b$ has a limit as $k$ tends to infinity and find that limit. +33. (NET 6) A square $2 n \times 2 n$ grid is given. Let us consider all possible paths along grid lines, going from the center of the grid to the border, such that (1) no point of the grid is reached more than once, and (2) each of the squares homothetic to the grid having its center at the grid center is passed through only once. +(a) Prove that the number of all such paths is equal to $4 \prod_{i=2}^{n}(16 i-9)$. +(b) Find the number of pairs of such paths that divide the grid into two congruent figures. +(c) How many quadruples of such paths are there that divide the grid into four congruent parts? +34. (POL 1) (SL71-9). +35. (POL 2) (SL71-10). +36. (POL 3) (SL71-11). +37. (POL 4) Let $S$ be a circle, and $\alpha=\left\{A_{1}, \ldots, A_{n}\right\}$ a family of open arcs in $S$. Let $N(\alpha)=n$ denote the number of elements in $\alpha$. We say that $\alpha$ is a covering of $S$ if $\bigcup_{k=1}^{n} A_{k} \supset S$. +Let $\alpha=\left\{A_{1}, \ldots, A_{n}\right\}$ and $\beta=\left\{B_{1}, \ldots, B_{m}\right\}$ be two coverings of $S$. Show that we can choose from the family of all sets $A_{i} \cap B_{j}, i=1,2, \ldots, n$, $j=1,2, \ldots, m$, a covering $\gamma$ of $S$ such that $N(\gamma) \leq N(\alpha)+N(\beta)$. +38. (POL 5) Let $A, B, C$ be three points with integer coordinates in the plane and $K$ a circle with radius $R$ passing through $A, B, C$. Show that $A B \cdot B C \cdot C A \geq 2 R$, and if the center of $K$ is in the origin of the coordinates, show that $A B \cdot B C \cdot C A \geq 4 R$. +39. (POL 6) (SL71-12). +40. (SWE 1) Prove that + +$$ +\left(1-\frac{1}{2^{3}}\right)\left(1-\frac{1}{3^{3}}\right)\left(1-\frac{1}{4^{3}}\right) \cdots\left(1-\frac{1}{n^{3}}\right)>\frac{1}{2}, \quad n=2,3, \ldots +$$ + +41. (SWE 2) Consider the set of grid points $(m, n)$ in the plane, $m, n$ integers. Let $\sigma$ be a finite subset and define + +$$ +S(\sigma)=\sum_{(m, n) \in \sigma}(100-|m|-|n|) +$$ + +Find the maximum of $S$, taken over the set of all such subsets $\sigma$. +42. (SWE 3) Let $L_{i}, i=1,2,3$, be line segments on the sides of an equilateral triangle, one segment on each side, with lengths $l_{i}, i=1,2,3$. By $L_{i}^{*}$ we +denote the segment of length $l_{i}$ with its midpoint on the midpoint of the corresponding side of the triangle. Let $M(L)$ be the set of points in the plane whose orthogonal projections on the sides of the triangle are in $L_{1}, L_{2}$, and $L_{3}$, respectively; $M\left(L^{*}\right)$ is defined correspondingly. Prove that if $l_{1} \geq l_{2}+l_{3}$, we have that the area of $M(L)$ is less than or equal to the area of $M\left(L^{*}\right)$. +43. (SWE 4) Show that for nonnegative real numbers $a, b$ and integers $n \geq 2$, + +$$ +\frac{a^{n}+b^{n}}{2} \geq\left(\frac{a+b}{2}\right)^{n} +$$ + +When does equality hold? +44. (SWE 5) (SL71-13). +45. (SWE 6) Let $m$ and $n$ denote integers greater than 1 , and let $\nu(n)$ be the number of primes less than or equal to $n$. Show that if the equation $\frac{n}{\nu(n)}=m$ has a solution, then so does the equation $\frac{n}{\nu(n)}=m-1$. +46. (USS 1) (SL71-14). +47. (USS 2) (SL71-15). +48. (USS 3) A sequence of real numbers $x_{1}, x_{2}, \ldots, x_{n}$ is given such that $x_{i+1}=x_{i}+\frac{1}{30000} \sqrt{1-x_{i}^{2}}, i=1,2, \ldots$, and $x_{1}=0$. Can $n$ be equal to 50000 if $x_{n}<1$ ? +49. (USS 4) Diagonals of a convex quadrilateral $A B C D$ intersect at a point $O$. Find all angles of this quadrilateral if $\measuredangle O B A=30^{\circ}, \measuredangle O C B=$ $45^{\circ}, \measuredangle O D C=45^{\circ}$, and $\measuredangle O A D=30^{\circ}$. +50. (USS 5) (SL71-16). +51. (USS 6) Suppose that the sides $A B$ and $D C$ of a convex quadrilateral $A B C D$ are not parallel. On the sides $B C$ and $A D$, pairs of points $(M, N)$ and $(K, L)$ are chosen such that $B M=M N=N C$ and $A K=K L=L D$. Prove that the areas of triangles $O K M$ and $O L N$ are different, where $O$ is the intersection point of $A B$ and $C D$. +52. (YUG 1) (SL71-17). +53. (YUG 2) Denote by $x_{n}(p)$ the multiplicity of the prime $p$ in the canonical representation of the number $n$ ! as a product of primes. Prove that $\frac{x_{n}(p)}{n}<$ $\frac{1}{p-1}$ and $\lim _{n \rightarrow \infty} \frac{x_{n}(p)}{n}=\frac{1}{p-1}$. +54. (YUG 3) A set $M$ is formed of $\binom{2 n}{n}$ men, $n=1,2, \ldots$. Prove that we can choose a subset $P$ of the set $M$ consisting of $n+1$ men such that one of the following conditions is satisfied: +(1) every member of the set $P$ knows every other member of the set $P$; +(2) no member of the set $P$ knows any other member of the set $P$. +55. (YUG 4) Prove that the polynomial $x^{4}+\lambda x^{3}+\mu x^{2}+\nu x+1$ has no real roots if $\lambda, \mu, \nu$ are real numbers satisfying + +$$ +|\lambda|+|\mu|+|\nu| \leq \sqrt{2} +$$ + +### 3.13.3 Shortlisted Problems + +1. (BUL 2) Consider a sequence of polynomials $P_{0}(x), P_{1}(x), P_{2}(x), \ldots$, $P_{n}(x), \ldots$, where $P_{0}(x)=2, P_{1}(x)=x$ and for every $n \geq 1$ the following equality holds: + +$$ +P_{n+1}(x)+P_{n-1}(x)=x P_{n}(x) +$$ + +Prove that there exist three real numbers $a, b, c$ such that for all $n \geq 1$, + +$$ +\left(x^{2}-4\right)\left[P_{n}^{2}(x)-4\right]=\left[a P_{n+1}(x)+b P_{n}(x)+c P_{n-1}(x)\right]^{2} +$$ + +2. (BUL 5) ${ }^{\mathrm{IMO} 5}$ Prove that for every natural number $m \geq 1$ there exists a finite set $S_{m}$ of points in the plane satisfying the following condition: If $A$ is any point in $S_{m}$, then there are exactly $m$ points in $S_{m}$ whose distance to $A$ equals 1. +3. (GDR 1) Knowing that the system + +$$ +\begin{aligned} +x+y+z & =3, \\ +x^{3}+y^{3}+z^{3} & =15, \\ +x^{4}+y^{4}+z^{4} & =35, +\end{aligned} +$$ + +has a real solution $x, y, z$ for which $x^{2}+y^{2}+z^{2}<10$, find the value of $x^{5}+y^{5}+z^{5}$ for that solution. +4. (GBR 3) We are given two mutually tangent circles in the plane, with radii $r_{1}, r_{2}$. A line intersects these circles in four points, determining three segments of equal length. Find this length as a function of $r_{1}$ and $r_{2}$ and the condition for the solvability of the problem. +5. (HUN 1) ${ }^{\mathrm{IMO}}$ Let $a, b, c, d, e$ be real numbers. Prove that the expression + +$$ +\begin{gathered} +(a-b)(a-c)(a-d)(a-e)+(b-a)(b-c)(b-d)(b-e)+(c-a)(c-b)(c-d)(c-e) \\ ++(d-a)(d-b)(d-c)(d-e)+(e-a)(e-b)(e-c)(e-d) +\end{gathered} +$$ + +is nonnegative. +6. (HUN 7) Let $n \geq 2$ be a natural number. Find a way to assign natural numbers to the vertices of a regular $2^{n}$-gon such that the following conditions are satisfied: +(1) only digits 1 and 2 are used; +(2) each number consists of exactly $n$ digits; +(3) different numbers are assigned to different vertices; +(4) the numbers assigned to two neighboring vertices differ at exactly one digit. +7. (NET 1) ${ }^{\mathrm{IMO} /}$ Given a tetrahedron $A B C D$ whose all faces are acuteangled triangles, set + +$$ +\sigma=\measuredangle D A B+\measuredangle B C D-\measuredangle A B C-\measuredangle C D A +$$ + +Consider all closed broken lines $X Y Z T X$ whose vertices $X, Y, Z, T$ lie in the interior of segments $A B, B C, C D, D A$ respectively. Prove that: +(a) if $\sigma \neq 0$, then there is no broken line $X Y Z T$ of minimal length; +(b) if $\sigma=0$, then there are infinitely many such broken lines of minimal length. That length equals $2 A C \sin (\alpha / 2)$, where + +$$ +\alpha=\measuredangle B A C+\measuredangle C A D+\measuredangle D A B +$$ + +8. (NET 4) Determine whether there exist distinct real numbers $a, b, c, t$ for which: +(i) the equation $a x^{2}+b t x+c=0$ has two distinct real roots $x_{1}, x_{2}$, +(ii) the equation $b x^{2}+c t x+a=0$ has two distinct real roots $x_{2}, x_{3}$, +(iii) the equation $c x^{2}+a t x+b=0$ has two distinct real roots $x_{3}, x_{1}$. +9. (POL 1) Let $T_{k}=k-1$ for $k=1,2,3,4$ and + +$$ +T_{2 k-1}=T_{2 k-2}+2^{k-2}, \quad T_{2 k}=T_{2 k-5}+2^{k} \quad(k \geq 3) +$$ + +Show that for all $k$, + +$$ +1+T_{2 n-1}=\left[\frac{12}{7} 2^{n-1}\right] \quad \text { and } \quad 1+T_{2 n}=\left[\frac{17}{7} 2^{n-1}\right] +$$ + +where $[x]$ denotes the greatest integer not exceeding $x$. +10. (POL 2) ${ }^{\mathrm{IMO} 3}$ Prove that the sequence $2^{n}-3(n>1)$ contains a subsequence of numbers relatively prime in pairs. +11. (POL 3) The matrix + +$$ +\left(\begin{array}{ccc} +a_{11} & \ldots & a_{1 n} \\ +\vdots & \ldots & \vdots \\ +a_{n 1} & \ldots & a_{n n} +\end{array}\right) +$$ + +satisfies the inequality $\sum_{j=1}^{n}\left|a_{j 1} x_{1}+\cdots+a_{j n} x_{n}\right| \leq M$ for each choice of numbers $x_{i}$ equal to $\pm 1$. Show that + +$$ +\left|a_{11}+a_{22}+\cdots+a_{n n}\right| \leq M +$$ + +12. (POL 6) Two congruent equilateral triangles $A B C$ and $A^{\prime} B^{\prime} C^{\prime}$ in the plane are given. Show that the midpoints of the segments $A A^{\prime}, B B^{\prime}, C C^{\prime}$ either are collinear or form an equilateral triangle. +13. (SWE 5) ${ }^{\mathrm{IMO} 6}$ Consider the $n \times n$ array of nonnegative integers + +$$ +\left(\begin{array}{cccc} +a_{11} & a_{12} & \ldots & a_{1 n} \\ +a_{21} & a_{22} & \ldots & a_{2 n} \\ +\vdots & \vdots & & \vdots \\ +a_{n 1} & a_{n 2} & \ldots & a_{n n} +\end{array}\right) +$$ + +with the following property: If an element $a_{i j}$ is zero, then the sum of the elements of the $i$ th row and the $j$ th column is greater than or equal to $n$. Prove that the sum of all the elements is greater than or equal to $\frac{1}{2} n^{2}$. +14. (USS 1) A broken line $A_{1} A_{2} \ldots A_{n}$ is drawn in a $50 \times 50$ square, so that the distance from any point of the square to the broken line is less than 1. Prove that its total length is greater than 1248. +15. (USS 2) Natural numbers from 1 to 99 (not necessarily distinct) are written on 99 cards. It is given that the sum of the numbers on any subset of cards (including the set of all cards) is not divisible by 100. Show that all the cards contain the same number. +16. (USS 5) ${ }^{\mathrm{IMO} 2}$ Given a convex polyhedron $P_{1}$ with 9 vertices $A_{1}, \ldots, A_{9}$, let us denote by $P_{2}, P_{3}, \ldots, P_{9}$ the images of $P_{1}$ under the translations mapping the vertex $A_{1}$ to $A_{2}, A_{3}, \ldots, A_{9}$ respectively. Prove that among the polyhedra $P_{1}, \ldots, P_{9}$ at least two have a common interior point. +17. (YUG 1) Prove the inequality + +$$ +\frac{a_{1}+a_{3}}{a_{1}+a_{2}}+\frac{a_{2}+a_{4}}{a_{2}+a_{3}}+\frac{a_{3}+a_{1}}{a_{3}+a_{4}}+\frac{a_{4}+a_{2}}{a_{4}+a_{1}} \geq 4 +$$ + +where $a_{i}>0, i=1,2,3,4$. + +### 3.14 The Fourteenth IMO
Warsaw-Toruna, Poland, July 5-17, 1972 + +### 3.14.1 Contest Problems + +First Day (July 10) + +1. A set of 10 positive integers is given such that the decimal expansion of each of them has two digits. Prove that there are two disjoint subsets of the set with equal sums of their elements. +2. Prove that for each $n \geq 4$ every cyclic quadrilateral can be decomposed into $n$ cyclic quadrilaterals. +3. Let $m$ and $n$ be nonnegative integers. Prove that $\frac{(2 m)!(2 n)!}{m!n!(m+n)!}$ is an integer $(0!=1)$. + +Second Day (July 11) +4. Find all solutions in positive real numbers $x_{i}(i=1,2,3,4,5)$ of the following system of inequalities: + +$$ +\begin{aligned} +& \left(x_{1}^{2}-x_{3} x_{5}\right)\left(x_{2}^{2}-x_{3} x_{5}\right) \leq 0 \\ +& \left(x_{2}^{2}-x_{4} x_{1}\right)\left(x_{3}^{2}-x_{4} x_{1}\right) \leq 0 \\ +& \left(x_{3}^{2}-x_{5} x_{2}\right)\left(x_{4}^{2}-x_{5} x_{2}\right) \leq 0 \\ +& \left(x_{4}^{2}-x_{1} x_{3}\right)\left(x_{5}^{2}-x_{1} x_{3}\right) \leq 0 \\ +& \left(x_{5}^{2}-x_{2} x_{4}\right)\left(x_{1}^{2}-x_{2} x_{4}\right) \leq 0 +\end{aligned} +$$ + +5. Let $f$ and $\varphi$ be real functions defined in the interval $(-\infty, \infty)$ satisfying the functional equation + +$$ +f(x+y)+f(x-y)=2 \varphi(y) f(x) +$$ + +for arbitrary real $x, y$ (give examples of such functions). Prove that if $f(x)$ is not identically 0 and $|f(x)| \leq 1$ for all $x$, then $|\varphi(x)| \leq 1$ for all $x$. +6. Given four distinct parallel planes, show that a regular tetrahedron exists with a vertex on each plane. + +### 3.14.2 Longlisted Problems + +1. (BUL 1) Find all integer solutions of the equation + +$$ +1+x+x^{2}+x^{3}+x^{4}=y^{4} +$$ + +2. (BUL 2) Find all real values of the parameter $a$ for which the system of equations + +$$ +\begin{aligned} +& x^{4}=y z-x^{2}+a \\ +& y^{4}=z x-y^{2}+a \\ +& z^{4}=x y-z^{2}+a +\end{aligned} +$$ + +has at most one real solution. +3. (BUL 3) On a line a set of segments is given of total length less than $n$. Prove that every set of $n$ points of the line can be translated in some direction along the line for a distance smaller than $n / 2$ so that none of the points remain on the segments. +4. (BUL 4) Given a triangle, prove that the points of intersection of three pairs of trisectors of the inner angles at the sides lying closest to those sides are vertices of an equilateral triangle. +5. (BUL 5) Given a pyramid whose base is an $n$-gon inscribable in a circle, let $H$ be the projection of the top vertex of the pyramid to its base. Prove that the projections of $H$ to the lateral edges of the pyramid lie on a circle. +6. (BUL 6) Prove the inequality + +$$ +(n+1) \cos \frac{\pi}{n+1}-n \cos \frac{\pi}{n}>1 +$$ + +for all natural numbers $n \geq 2$. +7. (BUL 7) (SL72-1). +8. (CZS 1) (SL72-2). +9. (CZS 2) Given natural numbers $k$ and $n, k \leq n, n \geq 3$, find the set of all values in the interval $(0, \pi)$ that the $k$ th-largest among the interior angles of a convex $n$ gon can take. +10. (CZS 3) Given five points in the plane, no three of which are collinear, prove that there can be found at least two obtuse-angled triangles with vertices at the given points. Construct an example in which there are exactly two such triangles. +11. (CZS 4) (SL72-3). +12. (CZS 5) A circle $k=(S, r)$ is given and a hexagon $A A^{\prime} B B^{\prime} C C^{\prime}$ inscribed in it. The lengths of sides of the hexagon satisfy $A A^{\prime}=A^{\prime} B, B B^{\prime}=B^{\prime} C$, $C C^{\prime}=C^{\prime} A$. Prove that the area $P$ of triangle $A B C$ is not greater than the area $P^{\prime}$ of triangle $A^{\prime} B^{\prime} C^{\prime}$. When does $P=P^{\prime}$ hold? +13. (CZS 6) Given a sphere $K$, determine the set of all points $A$ that are vertices of some parallelograms $A B C D$ that satisfy $A C \leq B D$ and whose entire diagonal $B D$ is contained in $K$. +14. (GBR 1) (SL72-7). +15. (GBR 2) (SL72-8). +16. (GBR 3) Consider the set $S$ of all the different odd positive integers that are not multiples of 5 and that are less than $30 m, m$ being a positive integer. What is the smallest integer $k$ such that in any subset of $k$ integers from $S$ there must be two integers one of which divides the other? Prove your result. +17. (GBR 4) A solid right circular cylinder with height $h$ and base-radius $r$ has a solid hemisphere of radius $r$ resting upon it. The center of the hemisphere $O$ is on the axis of the cylinder. Let $P$ be any point on the surface of the hemisphere and $Q$ the point on the base circle of the cylinder that is furthest from $P$ (measuring along the surface of the combined solid). A string is stretched over the surface from $P$ to $Q$ so as to be as short as possible. Show that if the string is not in a plane, the straight line $P O$ when produced cuts the curved surface of the cylinder. +18. (GBR 5) We have $p$ players participating in a tournament, each player playing against every other player exactly once. A point is scored for each victory, and there are no draws. A sequence of nonnegative integers $s_{1} \leq s_{2} \leq s_{3} \leq \cdots \leq s_{p}$ is given. Show that it is possible for this sequence to be a set of final scores of the players in the tournament if and only if +(i) $\sum_{i=1}^{p} s_{i}=\frac{1}{2} p(p-1) \quad$ and +(ii) for all $k Moscow, Soviet Union, July 5-16, 1973 + +### 3.15.1 Contest Problems + +First Day (July 9) + +1. Let $O$ be a point on the line $l$ and $\overrightarrow{O P_{1}}, \overrightarrow{O P_{2}}, \ldots, \overrightarrow{O P_{n}}$ unit vectors such that points $P_{1}, P_{2}, \ldots, P_{n}$ and line $l$ lie in the same plane and all points $P_{i}$ lie in the same half-plane determined by $l$. Prove that if $n$ is odd, then + +$$ +\left\|\overrightarrow{O P_{1}}+\overrightarrow{O P_{2}}+\cdots+\overrightarrow{O P_{n}}\right\| \geq 1 +$$ + +$(\|\overrightarrow{O M}\|$ is the length of vector $\overrightarrow{O M})$. +2. Does there exist a finite set $M$ of points in space, not all in the same plane, such that for each two points $A, B \in M$ there exist two other points $C, D \in M$ such that lines $A B$ and $C D$ are parallel but not equal? +3. Determine the minimum of $a^{2}+b^{2}$ if $a$ and $b$ are real numbers for which the equation + +$$ +x^{4}+a x^{3}+b x^{2}+a x+1=0 +$$ + +has at least one real solution. +Second Day (July 10) +4. A soldier has to investigate whether there are mines in an area that has the form of equilateral triangle. The radius of his detector's range is equal to one-half the altitude of the triangle. The soldier starts from one vertex of the triangle. Determine the smallest path through which the soldier has to pass in order to check the entire region. +5. Let $G$ be the set of functions $f: \mathbb{R} \rightarrow \mathbb{R}$ of the form $f(x)=a x+b$, where $a$ and $b$ are real numbers and $a \neq 0$. Suppose that $G$ satisfies the following conditions: +(1) If $f, g \in G$, then $g \circ f \in G$, where $(g \circ f)(x)=g[f(x)]$. +(2) If $f \in G$ and $f(x)=a x+b$, then the inverse $f^{-1}$ of $f$ belongs to $G$ $\left(f^{-1}(x)=(x-b) / a\right)$. +(3) For each $f \in G$ there exists a number $x_{f} \in \mathbb{R}$ such that $f\left(x_{f}\right)=x_{f}$. Prove that there exists a number $k \in \mathbb{R}$ such that $f(k)=k$ for all $f \in G$. +6. Let $a_{1}, a_{2}, \ldots, a_{n}$ be positive numbers and $q$ a given real number, $01$. Prove that there exist infinitely many prime numbers $p$ that divide $u_{p-1}$. +3. (BUL 3) Let $A B C D$ be an arbitrary quadrilateral. Let squares $A B B_{1} A_{2}$, $B C C_{1} B_{2}, C D D_{1} C_{2}, D A A_{1} D_{2}$ be constructed in the exterior of the quadrilateral. Furthermore, let $A A_{1} P A_{2}$ and $C C_{1} Q C_{2}$ be parallelograms. For any arbitrary point $P$ in the interior of $A B C D$, parallelograms $R A S C$ and $R P T Q$ are constructed. Prove that these two parallelograms have two vertices in common. +4. (BUL 4) Let $K_{a}, K_{b}, K_{c}$ with centers $O_{a}, O_{b}, O_{c}$ be the excircles of a triangle $A B C$, touching the interiors of the sides $B C, C A, A B$ at points $T_{a}, T_{b}, T_{c}$ respectively. +Prove that the lines $O_{a} T_{a}, O_{b} T_{b}, O_{c} T_{c}$ are concurrent in a point $P$ for which $P O_{a}=P O_{b}=P O_{c}=2 R$ holds, where $R$ denotes the circumradius of $A B C$. Also prove that the circumcenter $O$ of $A B C$ is the midpoint of the segment $P J$, where $J$ is the incenter of $A B C$. +5. (BUL 5) A straight cone is given inside a rectangular parallelepiped $B$, with the apex at one of the vertices, say $T$, of the parallelepiped, and the base touching the three faces opposite to $T$. Its axis lies at the long diagonal through $T$. If $V_{1}$ and $V_{2}$ are the volumes of the cone and the parallelepiped respectively, prove that + +$$ +V_{1} \leq \frac{\sqrt{3} \pi V_{2}}{27} +$$ + +6. (CUB 1) Prove that the product of two natural numbers with their sum cannot be the third power of a natural number. +7. (CUB 2) Let $P$ be a prime number and $n$ a natural number. Prove that the product + +$$ +N=\frac{1}{p^{n^{2}}} \prod_{i=1 ; 2 \nmid i}^{2 n-1}\left[((p-1) i)!\binom{p^{2} i}{p i}\right] +$$ + +is a natural number that is not divisible by $p$. +8. (CUB 3) (SL74-9). +9. (CZS 1) Solve the following system of linear equations with unknown $x_{1}, \ldots, x_{n}(n \geq 2)$ and parameters $c_{1}, \ldots, c_{n}$ : + +$$ +\begin{array}{rlrl} +2 x_{1}-x_{2} & & =c_{1} \\ +-x_{1}+2 x_{2}-x_{3} & & & =c_{2} \\ +-x_{2}+2 x_{3}-x_{4} & & =c_{3} \\ +\ldots & \ldots & \ldots & \cdots \\ +& & -x_{n-2}+2 x_{n-1}-x_{n} & =c_{n-1} \\ +& -x_{n-1}+2 x_{n} & =c_{n} +\end{array} +$$ + +10. (CZS 2) A regular octagon $P$ is given whose incircle $k$ has diameter 1. About $k$ is circumscribed a regular 16-gon, which is also inscribed in $P$, cutting from $P$ eight isosceles triangles. To the octagon $P$, three of these triangles are added so that exactly two of them are adjacent and no two of them are opposite to each other. Every 11-gon so obtained is said to be $P^{\prime}$. +Prove the following statement: Given a finite set $M$ of points lying in $P$ such that every two points of this set have a distance not exceeding 1 , one of the 11-gons $P^{\prime}$ contains all of $M$. +11. (CZS 3) Given a line $p$ and a triangle $\triangle$ in the plane, construct an equilateral triangle one of whose vertices lies on the line $p$, while the other two halve the perimeter of $\triangle$. +12. (CZS 4) A circle $K$ with radius $r$, a point $D$ on $K$, and a convex angle with vertex $S$ and rays $a$ and $b$ are given in the plane. Construct a parallelogram $A B C D$ such that $A$ and $B$ lie on $a$ and $b$ respectively, $S A+S B=r$, and $C$ lies on $K$. +13. (FIN 1) Prove that $2^{147}-1$ is divisible by 343. +14. (FIN 2) Let $n$ and $k$ be natural numbers and $a_{1}, a_{2}, \ldots, a_{n}$ positive real numbers satisfying $a_{1}+a_{2}+\cdots+a_{n}=1$. Prove that + +$$ +a_{1}^{-k}+a_{2}^{-k}+\cdots+a_{n}^{-k} \geq n^{k+1} +$$ + +15. (FIN 3) (SL74-10). +16. (GBR 1) A pack of $2 n$ cards contains $n$ different pairs of cards. Each pair consists of two identical cards, either of which is called the twin of the other. A game is played between two players $A$ and $B$. A third person called the dealer shuffles the pack and deals the cards one by one face upward onto the table. One of the players, called the receiver, takes the card dealt, provided he does not have already its twin. If he does already have the twin, his opponent takes the dealt card and becomes the receiver. $A$ is initially the receiver and takes the first card dealt. The player who first obtains a complete set of $n$ different cards wins the game. What fraction of all possible arrangements of the pack lead to $A$ winning? Prove the correctness of your answer. +17. (GBR 2) Show that there exists a set $S$ of 15 distinct circles on the surface of a sphere, all having the same radius and such that 5 touch exactly 5 others, 5 touch exactly 4 others, and 5 touch exactly 3 others. +18. (GBR 3) (SL74-5). +19. (GBR 4) (Alternative to GBR 2) Prove that there exists, for $n \geq 4$, a set $S$ of $3 n$ equal circles in spacethat can be partitioned into three subsets $s_{5}, s_{4}$, and $s_{3}$, each containing $n$ circles, such that each circle in $s_{r}$ touches exactly $r$ circles in $S$. +20. (NET 1) For which natural numbers $n$ do there exist $n$ natural numbers $a_{i}(1 \leq i \leq n)$ such that $\sum_{i=1}^{n} a_{i}^{-2}=1$ ? +21. (NET 2) Let $M$ be a nonempty subset of $\mathbb{Z}^{+}$such that for every element $x$ in $M$, the numbers $4 x$ and $[\sqrt{x}]$ also belong to $M$. Prove that $M=\mathbb{Z}^{+}$. +22. (NET 3) (SL74-8). +23. (POL 1) (SL74-2). +24. (POL 2) (SL74-7). +25. (POL 3) Let $f: \mathbb{R} \rightarrow \mathbb{R}$ be of the form $f(x)=x+\varepsilon \sin x$, where $0<|\varepsilon| \leq 1$. Define for any $x \in \mathbb{R}$, + +$$ +x_{n}=\underbrace{f \circ \cdots \circ f}_{n \text { times }}(x) +$$ + +Show that for every $x \in \mathbb{R}$ there exists an integer $k$ such that $\lim _{n \rightarrow \infty} x_{n}$ $=k \pi$. +26. (POL 4) Let $g(k)$ be the number of partitions of a $k$-element set $M$, i.e., the number of families $\left\{A_{1}, A_{2}, \ldots, A_{s}\right\}$ of nonempty subsets of $M$ such that $A_{i} \cap A_{j}=\emptyset$ for $i \neq j$ and $\bigcup_{i=1}^{n} A_{i}=M$. Prove that + +$$ +n^{n} \leq g(2 n) \leq(2 n)^{2 n} \quad \text { for every } n +$$ + +27. (ROM 1) Let $C_{1}$ and $C_{2}$ be circles in the same plane, $P_{1}$ and $P_{2}$ arbitrary points on $C_{1}$ and $C_{2}$ respectively, and $Q$ the midpoint of segment $P_{1} P_{2}$. Find the locus of points $Q$ as $P_{1}$ and $P_{2}$ go through all possible positions. Alternative version. Let $C_{1}, C_{2}, C_{3}$ be three circles in the same plane. Find the locus of the centroid of triangle $P_{1} P_{2} P_{3}$ as $P_{1}, P_{2}$, and $P_{3}$ go through all possible positions on $C_{1}, C_{2}$, and $C_{3}$ respectively. +28. (ROM 2) Let $M$ be a finite set and $P=\left\{M_{1}, M_{2}, \ldots, M_{k}\right\}$ a partition of $M$ (i.e., $\bigcup_{i=1}^{k} M_{i}=M, M_{i} \neq \emptyset, M_{i} \cap M_{j}=\emptyset$ for all $i, j \in\{1,2, \ldots, k\}$, $i \neq j$ ). We define the following elementary operation on $P$ : + +Choose $i, j \in\{1,2, \ldots, k\}$, such that $i \neq j$ and $M_{i}$ has $a$ elements and $M_{j}$ has $b$ elements such that $a \geq b$. Then take $b$ elements from $M_{i}$ and place them into $M_{j}$, i.e., $M_{j}$ becomes the union of itself unifies and a $b$-element subset of $M_{i}$, while the same subset is subtracted from $M_{i}$ (if $a=b, M_{i}$ is thus removed from the partition). +Let a finite set $M$ be given. Prove that the property "for every partition $P$ of $M$ there exists a sequence $P=P_{1}, P_{2}, \ldots, P_{r}$ such that $P_{i+1}$ is obtained +from $P_{i}$ by an elementary operation and $P_{r}=\{M\}$ " is equivalent to "the number of elements of $M$ is a power of $2 . "$ +29. (ROM 3) Let $A, B, C, D$ be points in space. If for every point $M$ on the segment $A B$ the sum + +$$ +\operatorname{area}(A M C)+\operatorname{area}(C M D)+\operatorname{area}(D M B) +$$ + +is constant show that the points $A, B, C, D$ lie in the same plane. +30. (ROM 4) (SL74-6). +31. (ROM 5) Let $y^{\alpha}=\sum_{i=1}^{n} x_{i}^{\alpha}$, where $\alpha \neq 0, y>0, x_{i}>0$ are real numbers, and let $\lambda \neq \alpha$ be a real number. Prove that $y^{\lambda}>\sum_{i=1}^{n} x_{i}^{\lambda}$ if $\alpha(\lambda-\alpha)>0$, and $y^{\lambda}<\sum_{i=1}^{n} x_{i}^{\lambda}$ if $\alpha(\lambda-\alpha)<0$. +32. (SWE 1) Let $a_{1}, a_{2}, \ldots, a_{n}$ be $n$ real numbers such that $0
n$. Prove that + +$$ +2(m-n)^{2}\left(m^{2}-n^{2}+1\right) \geq 2 m^{2}-2 m n+1 +$$ + +51. (YUG 2) There are $n$ points on a flat piece of paper, any two of them at a distance of at least 2 from each other. An inattentive pupil spills ink on a part of the paper such that the total area of the damaged part equals $3 / 2$. Prove that there exist two vectors of equal length less than 1 and with their sum having a given direction, such that after a translation by either of these two vectors no points of the given set remain in the damaged area. +52. (YUG 3) A fox stands in the center of the field which has the form of an equilateral triangle, and a rabbit stands at one of its vertices. The fox can move through the whole field, while the rabbit can move only along the border of the field. The maximal speeds of the fox and rabbit are equal to $u$ and $v$, respectively. Prove that: +(a) If $2 u>v$, the fox can catch the rabbit, no matter how the rabbit moves. +(b) If $2 u \leq v$, the rabbit can always run away from the fox. + +### 3.16.3 Shortlisted Problems + +1. I 1 (USA 4) ${ }^{\mathrm{IMO}}$ Alice, Betty, and Carol took the same series of examinations. There was one grade of $A$, one grade of $B$, and one grade of $C$ for each examination, where $A, B, C$ are different positive integers. The final test scores were + +| Alice | Betty | Carol | +| :---: | :---: | :---: | +| 20 | 10 | 9 | + +If Betty placed first in the arithmetic examination, who placed second in the spelling examination? +2. I 2 (POL 1) Prove that the squares with sides $1 / 1,1 / 2,1 / 3, \ldots$ may be put into the square with side $3 / 2$ in such a way that no two of them have any interior point in common. +3. I 3 (SWE 3) ${ }^{\mathrm{IMO}}$ Let $P(x)$ be a polynomial with integer coefficients. If $n(P)$ is the number of (distinct) integers $k$ such that $P^{2}(k)=1$, prove that + +$$ +n(P)-\operatorname{deg}(P) \leq 2 +$$ + +where $\operatorname{deg}(P)$ denotes the degree of the polynomial $P$. +4. I 4 (USS 4) The sum of the squares of five real numbers $a_{1}, a_{2}, a_{3}, a_{4}, a_{5}$ equals 1. Prove that the least of the numbers $\left(a_{i}-a_{j}\right)^{2}$, where $i, j=$ $1,2,3,4,5$ and $i \neq j$, does not exceed $1 / 10$. +5. I 5 (GBR 3) Let $A_{r}, B_{r}, C_{r}$ be points on the circumference of a given circle $S$. From the triangle $A_{r} B_{r} C_{r}$, called $\triangle_{r}$, the triangle $\triangle_{r+1}$ is obtained by constructing the points $A_{r+1}, B_{r+1}, C_{r+1}$ on $S$ such that $A_{r+1} A_{r}$ is parallel to $B_{r} C_{r}, B_{r+1} B_{r}$ is parallel to $C_{r} A_{r}$, and $C_{r+1} C_{r}$ is parallel to $A_{r} B_{r}$. Each angle of $\triangle_{1}$ is an integer number of degrees and those integers are not multiples of 45 . Prove that at least two of the triangles $\triangle_{1}, \triangle_{2}, \ldots, \triangle_{15}$ are congruent. +6. I 6 (ROM 4) ${ }^{\mathrm{IMO} 3}$ Does there exist a natural number $n$ for which the number + +$$ +\sum_{k=0}^{n}\binom{2 n+1}{2 k+1} 2^{3 k} +$$ + +is divisible by 5 ? +7. II 1 (POL 2) Let $a_{i}, b_{i}$ be coprime positive integers for $i=1,2, \ldots, k$, and $m$ the least common multiple of $b_{1}, \ldots, b_{k}$. Prove that the greatest common divisor of $a_{1} \frac{m}{b_{1}}, \ldots, a_{k} \frac{m}{b_{k}}$ equals the greatest common divisor of $a_{1}, \ldots, a_{k}$. +8. II 2 (NET 3) ${ }^{\mathrm{IMO} 5}$ If $a, b, c, d$ are arbitrary positive real numbers, find all possible values of + +$$ +S=\frac{a}{a+b+d}+\frac{b}{a+b+c}+\frac{c}{b+c+d}+\frac{d}{a+c+d} . +$$ + +9. II 3 (CUB 3) Let $x, y, z$ be real numbers each of whose absolute value is different from $1 / \sqrt{3}$ such that $x+y+z=x y z$. Prove that + +$$ +\frac{3 x-x^{3}}{1-3 x^{2}}+\frac{3 y-y^{3}}{1-3 y^{2}}+\frac{3 z-z^{3}}{1-3 z^{2}}=\frac{3 x-x^{3}}{1-3 x^{2}} \cdot \frac{3 y-y^{3}}{1-3 y^{2}} \cdot \frac{3 z-z^{3}}{1-3 z^{2}} +$$ + +10. II 4 (FIN 3) ${ }^{\mathrm{IMO} 2}$ Let $\triangle A B C$ be a triangle. Prove that there exists a point $D$ on the side $A B$ such that $C D$ is the geometric mean of $A D$ and $B D$ if and only if $\sqrt{\sin A \sin B} \leq \sin \frac{C}{2}$. +11. II 5 (BUL 1) ${ }^{\mathrm{IMO} 4}$ Consider a partition of an $8 \times 8$ chessboard into $p$ rectangles whose interiors are disjoint such that each of them has an equal number of white and black cells. Assume that $a_{1}0, a_{i+1}>a_{i}$.) Prove that there are infinitely many $m$ for which positive integers $x, y, h, k$ can be found such that $00, a_{i+1}>a_{i}$.) Prove that there are infinitely many $m$ for which positive integers $x, y, h, k$ can be found such that $0B M \cdot D N +$$ + +6. (CZS 1) For each point $X$ of a given polytope, denote by $f(X)$ the sum of the distances of the point $X$ from all the planes of the faces of the polytope. +Prove that if $f$ attains its maximum at an interior point of the polytope, then $f$ is constant. +7. (CZS 2) Let $P$ be a fixed point and $T$ a given triangle that contains the point $P$. Translate the triangle $T$ by a given vector $\mathbf{v}$ and denote by $T^{\prime}$ this new triangle. Let $r, R$, respectively, be the radii of the smallest disks centered at $P$ that contain the triangles $T, T^{\prime}$, respectively. +Prove that + +$$ +r+|\mathbf{v}| \leq 3 R +$$ + +and find an example to show that equality can occur. +8. (CZS 3) (SL76-3). +9. (CZS 4) Find all (real) solutions of the system + +$$ +\begin{aligned} +& 3 x_{1}-x_{2}-x_{3} \quad-x_{5} \quad=0, \\ +& \begin{array}{lll} +-x_{1}+3 x_{2} & -x_{4} & -x_{6} +\end{array}=0, \\ +& \begin{array}{ccc} +-x_{1} & +3 x_{3}-x_{4} & -x_{7} +\end{array}=0, \\ +& -x_{2}-x_{3}+3 x_{4} \quad-x_{8}=0, \\ +& -x_{1} \quad+3 x_{5}-x_{6}-x_{7} \quad=0, \\ +& \begin{array}{rll} +-x_{2} & & -x_{5}+3 x_{6} \quad-x_{8}=0, \\ +-x_{3} & -x_{5}+3 x_{7}-x_{8}=0, +\end{array} \\ +& -x_{4} \quad-x_{6}-x_{7}+3 x_{8}=0 . +\end{aligned} +$$ + +10. (FIN 1) Show that the reciprocal of any number of the form $2\left(m^{2}+\right.$ $m+1$ ), where $m$ is a positive integer, can be represented as a sum of consecutive terms in the sequence $\left(a_{j}\right)_{j=1}^{\infty}$, + +$$ +a_{j}=\frac{1}{j(j+1)(j+2)} . +$$ + +11. (FIN 2) (SL76-9). +12. (FIN 3) Five points lie on the surface of a ball of unit radius. Find the maximum of the smallest distance between any two of them. +13. (GBR 1a) (SL76-4). +14. (GBR 1b) A sequence $\left\{u_{n}\right\}$ of integers is defined by + +$$ +\begin{aligned} +u_{1} & =2, \quad u_{2}=u_{3}=7, \\ +u_{n+1} & =u_{n} u_{n-1}-u_{n-2}, \quad \text { for } n \geq 3 +\end{aligned} +$$ + +Prove that for each $n \geq 1, u_{n}$ differs by 2 from an integral square. +15. (GBR 2) Let $A B C$ and $A^{\prime} B^{\prime} C^{\prime}$ be any two coplanar triangles. Let $L$ be a point such that $A L\left\|B C, A^{\prime} L\right\| B^{\prime} C^{\prime}$, and $M, N$ similarly defined. The line $B C$ meets $B^{\prime} C^{\prime}$ at $P$, and similarly defined are $Q$ and $R$. Prove that $P L, Q M, R N$ are concurrent. +16. (GBR 3) Prove that there is a positive integer $n$ such that the decimal representation of $7^{n}$ contains a block of at least $m$ consecutive zeros, where $m$ is any given positive integer. +17. (GBR 4) Show that there exists a convex polyhedron with all its vertices on the surface of a sphere and with all its faces congruent isosceles triangles whose ratio of sides are $\sqrt{3}: \sqrt{3}: 2$. +18. (GDR 1) Prove that the number $19^{1976}+76^{1976}$ : +(a) is divisible by the (Fermat) prime number $F_{4}=2^{2^{4}}+1$; +(b) is divisible by at least four distinct primes other than $F_{4}$. +19. (GDR 2) For a positive integer $n$, let $6^{(n)}$ be the natural number whose decimal representation consists of $n$ digits 6 . Let us define, for all natural numbers $m, k$ with $1 \leq k \leq m$, + +$$ +\left[\begin{array}{c} +m \\ +k +\end{array}\right]=\frac{6^{(m)} \cdot 6^{(m-1)} \cdots 6^{(m-k+1)}}{6^{(1)} \cdot 6^{(2)} \cdots 6^{(k)}} +$$ + +Prove that for all $m, k,\left[\begin{array}{c}m \\ k\end{array}\right]$ is a natural number whose decimal representation consists of exactly $k(m+k-1)-1$ digits. +20. (GDR 3) Let $\left(a_{n}\right), n=0,1, \ldots$, be a sequence of real numbers such that $a_{0}=0$ and + +$$ +a_{n+1}^{3}=\frac{1}{2} a_{n}^{2}-1, \quad n=0,1, \ldots +$$ + +Prove that there exists a positive number $q, q<1$, such that for all $n=1,2, \ldots$, + +$$ +\left|a_{n+1}-a_{n}\right| \leq q\left|a_{n}-a_{n-1}\right|, +$$ + +and give one such $q$ explicitly. +21. (GDR 4) Find the largest positive real number $p$ (if it exists) such that the inequality + +$$ +x_{1}^{2}+x_{2}^{2}+\cdots+x_{n}^{2} \geq p\left(x_{1} x_{2}+x_{2} x_{3}+\cdots+x_{n-1} x_{n}\right) +$$ + +is satisfied for all real numbers $x_{i}$, and (a) $n=2$; (b) $n=5$. +Find the largest positive real number $p$ (if it exists) such that the inequality (1) holds for all real numbers $x_{i}$ and all natural numbers $n, n \geq 2$. +22. (GDR 5) A regular pentagon $A_{1} A_{2} A_{3} A_{4} A_{5}$ with side length $s$ is given. At each point $A_{i}$ a sphere $K_{i}$ of radius $s / 2$ is constructed. There are two spheres $K_{1}{ }^{\prime}$ and $K_{2}{ }^{\prime}$ eah of radius $s / 2$ touching all the five spheres $K_{i}$. Decide whether $K_{1}{ }^{\prime}$ and $K_{2}{ }^{\prime}$ intersect each other, touch each other, or have no common points. +23. (NET 1) Prove that in a Euclidean plane there are infinitely many concentric circles $C$ such that all triangles inscribed in $C$ have at least one irrational side. +24. (NET 2) Let $0 \leq x_{1} \leq x_{2} \leq \cdots \leq x_{n} \leq 1$. Prove that for all $A \geq 1$ there exists an interval $I$ of length $2 \sqrt[n]{A}$ such that for all $x \in I$, + +$$ +\left|\left(x-x_{1}\right)\left(x-x_{2}\right) \cdots\left(x-x_{n}\right)\right| \leq A +$$ + +25. (NET 3) (SL76-5). +26. (NET 4) (SL76-6). +27. (NET 5) In a plane three points $P, Q, R$, not on a line, are given. Let $k, l, m$ be positive numbers. Construct a triangle $A B C$ whose sides pass through $P, Q$, and $R$ such that +$P$ divides the segment $A B$ in the ratio $1: k$, $Q$ divides the segment $B C$ in the ratio $1: l$, and $R$ divides the segment $C A$ in the ratio 1: $m$. +28. (POL 1a) Let $Q$ be a unit square in the plane: $Q=[0,1] \times[0,1]$. Let $T: Q \rightarrow Q$ be defined as follows: + +$$ +T(x, y)= \begin{cases}(2 x, y / 2) & \text { if } 0 \leq x \leq 1 / 2 \\ (2 x-1, y / 2+1 / 2) & \text { if } 1 / 20$ such that $T^{n}(D) \cap D \neq \emptyset$. +29. (POL 1b) (SL76-7). +30. (POL 2) Prove that if $P(x)=(x-a)^{k} Q(x)$, where $k$ is a positive integer, $a$ is a nonzero real number, $Q(x)$ is a nonzero polynomial, then $P(x)$ has at least $k+1$ nonzero coefficients. +31. (POL 3) Into every lateral face of a quadrangular pyramid a circle is inscribed. The circles inscribed into adjacent faces are tangent (have one point in common). Prove that the points of contact of the circles with the base of the pyramid lie on a circle. +32. (POL 4) We consider the infinite chessboard covering the whole plane. In every field of the chessboard there is a nonnegative real number. Every number is the arithmetic mean of the numbers in the four adjacent fields of the chessboard. Prove that the numbers occurring in the fields of the chessboard are all equal. +33. (SWE 1) A finite set of points $P$ in the plane has the following property: Every line through two points in $P$ contains at least one more point belonging to $P$. Prove that all points in $P$ lie on a straight line. +34. (SWE 2) Let $\left\{a_{n}\right\}_{0}^{\infty}$ and $\left\{b_{n}\right\}_{0}^{\infty}$ be two sequences determined by the recursion formulas + +$$ +\begin{aligned} +& a_{n+1}=a_{n}+b_{n}, \\ +& b_{n+1}=3 a_{n}+b_{n}, \quad n=0,1,2, \ldots, +\end{aligned} +$$ + +and the initial values $a_{0}=b_{0}=1$. Prove that there exists a uniquely determined constant $c$ such that $n\left|c a_{n}-b_{n}\right|<2$ for all nonnegative integers $n$. +35. (SWE 3) (SL76-8). +36. (USA 1) Three concentric circles with common center $O$ are cut by a common chord in successive points $A, B, C$. Tangents drawn to the circles at the points $A, B, C$ enclose a triangular region. If the distance from point $O$ to the common chord is equal to $p$, prove that the area of the region enclosed by the tangents is equal to + +$$ +\frac{A B \cdot B C \cdot C A}{2 p} +$$ + +37. (USA 2) From a square board 11 squares long and 11 squares wide, the central square is removed. Prove that the remaining 120 squares cannot be covered by 15 strips each 8 units long and one unit wide. +38. (USA 3) Let $x=\sqrt{a}+\sqrt{b}$, where $a$ and $b$ are natural numbers, $x$ is not an integer, and $x<1976$. Prove that the fractional part of $x$ exceeds $10^{-19.76}$. +39. (USA 4) In $\triangle A B C$, the inscribed circle is tangent to side $B C$ at $X$. Segment $A X$ is drawn. Prove that the line joining the midpoint of segment +$A X$ to the midpoint of side $B C$ passes through the center $I$ of the inscribed circle. +40. (USA 5) Let $g(x)$ be a fixed polynomial and define $f(x)$ by $f(x)=$ $x^{2}+x g\left(x^{3}\right)$. Show that $f(x)$ is not divisible by $x^{2}-x+1$. +41. (USA 6) (SL76-10). +42. (USS 1) For a point $O$ inside a triangle $A B C$, denote by $A_{1}, B_{1}, C_{1}$ the respective intersection points of $A O, B O, C O$ with the corresponding sides. Let $n_{1}=\frac{A O}{A_{1} O}, n_{2}=\frac{B O}{B_{1} O}, n_{3}=\frac{C O}{C_{1} O}$. What possible values of $n_{1}, n_{2}, n_{3}$ can all be positive integers? +43. (USS 2) Prove that if for a polynomial $P(x, y)$ we have + +$$ +P(x-1, y-2 x+1)=P(x, y) +$$ + +then there exists a polynomial $\Phi(x)$ with $P(x, y)=\Phi\left(y-x^{2}\right)$. +44. (USS 3) A circle of radius 1 rolls around a circle of radius $\sqrt{2}$. Initially, the tangent point is colored red. Afterwards, the red points map from one circle to another by contact. How many red points will be on the bigger circle when the center of the smaller one has made $n$ circuits around the bigger one? +45. (USS 4) We are given $n(n \geq 5)$ circles in a plane. Suppose that every three of them have a common point. Prove that all $n$ circles have a common point. +46. (USS 5) For $a \geq 0, b \geq 0, c \geq 0, d \geq 0$, prove the inequality + +$$ +a^{4}+b^{4}+c^{4}+d^{4}+2 a b c d \geq a^{2} b^{2}+a^{2} c^{2}+a^{2} d^{2}+b^{2} c^{2}+b^{2} d^{2}+c^{2} d^{2} +$$ + +47. (VIE 1) (SL76-11). +48. (VIE 2) (SL76-12). +49. (VIE 3) Determine whether there exist 1976 nonsimilar triangles with angles $\alpha, \beta, \gamma$, each of them satisfying the relations + +$$ +\frac{\sin \alpha+\sin \beta+\sin \gamma}{\cos \alpha+\cos \beta+\cos \gamma}=\frac{12}{7} \quad \text { and } \quad \sin \alpha \sin \beta \sin \gamma=\frac{12}{25} +$$ + +50. (VIE 4) Find a function $f(x)$ defined for all real values of $x$ such that for all $x$, + +$$ +f(x+2)-f(x)=x^{2}+2 x+4 +$$ + +and if $x \in[0,2)$, then $f(x)=x^{2}$. +51. (YUG 1) Four swallows are catching a fly. At first, the swallows are at the four vertices of a tetrahedron, and the fly is in its interior. Their maximal speeds are equal. Prove that the swallows can catch the fly. + +### 3.18.3 Shortlisted Problems + +1. (BUL 1) Let $A B C$ be a triangle with bisectors $A A_{1}, B B_{1}, C C_{1}\left(A_{1} \in\right.$ $B C$, etc.) and $M$ their common point. Consider the triangles $M B_{1} A$, $M C_{1} A, M C_{1} B, M A_{1} B, M A_{1} C, M B_{1} C$, and their inscribed circles. Prove that if four of these six inscribed circles have equal radii, then $A B=$ $B C=C A$. +2. (BUL 3) Let $a_{0}, a_{1}, \ldots, a_{n}, a_{n+1}$ be a sequence of real numbers satisfying the following conditions: + +$$ +\begin{aligned} +a_{0} & =a_{n+1}=0 \\ +\left|a_{k-1}-2 a_{k}+a_{k+1}\right| & \leq 1 \quad(k=1,2, \ldots, n) +\end{aligned} +$$ + +Prove that $\left|a_{k}\right| \leq \frac{k(n+1-k)}{2}(k=0,1, \ldots, n+1)$. +3. (CZS 3) ${ }^{\mathrm{IMO} 1}$ In a convex quadrangle with area $32 \mathrm{~cm}^{2}$, the sum of the lengths of two nonadjacent edges and of the length of one diagonal is equal to 16 cm . +(a) What is the length of the other diagonal? +(b) What are the lengths of the edges of the quadrangle if the perimeter is a minimum? +(c) Is it possible to choose the edges in such a way that the perimeter is a maximum? +4. (GBR 1a) ${ }^{\mathrm{IMO} 6}$ For all positive integral $n$, $u_{n+1}=u_{n}\left(u_{n-1}^{2}-2\right)-u_{1}$, $u_{0}=2$, and $u_{1}=5 / 2$. Prove that + +$$ +3 \log _{2}\left[u_{n}\right]=2^{n}-(-1)^{n} +$$ + +where $[x]$ is the integral part of $x$. +5. (NET 3) ${ }^{\mathrm{IMO} 5}$ Let a set of $p$ equations be given, + +$$ +\begin{gathered} +a_{11} x_{1}+\cdots+a_{1 q} x_{q}=0 \\ +a_{21} x_{1}+\cdots+a_{2 q} x_{q}=0 \\ +\vdots \\ +a_{p 1} x_{1}+\cdots+a_{p q} x_{q}=0, +\end{gathered} +$$ + +with coefficients $a_{i j}$ satisfying $a_{i j}=-1,0$, or +1 for all $i=1, \ldots, p$ and $j=1, \ldots, q$. Prove that if $q=2 p$, there exists a solution $x_{1}, \ldots, x_{q}$ of this system such that all $x_{j}(j=1, \ldots, q)$ are integers satisfying $\left|x_{j}\right| \leq q$ and $x_{j} \neq 0$ for at least one value of $j$. +6. (NET 4) ${ }^{\mathrm{IMO} 3} \mathrm{~A}$ rectangular box can be filled completely with unit cubes. If one places cubes with volume 2 in the box such that their edges are parallel to the edges of the box, one can fill exactly $40 \%$ of the box. Determine all possible (interior) sizes of the box. +7. (POL 1b) Let $I=(0,1]$ be the unit interval of the real line. For a given number $a \in(0,1)$ we define a map $T: I \rightarrow I$ by the formula + +$$ +T(x, y)= \begin{cases}x+(1-a) & \text { if } 00$ such that $T^{n}(J) \cap J \neq \emptyset$. +8. (SWE 3) Let $P$ be a polynomial with real coefficients such that $P(x)>0$ if $x>0$. Prove that there exist polynomials $Q$ and $R$ with nonnegative coefficients such that $P(x)=\frac{Q(x)}{R(x)}$ if $x>0$. +9. (FIN 2) ${ }^{\mathrm{IMO} 2}$ Let $P_{1}(x)=x^{2}-2, P_{j}(x)=P_{1}\left(P_{j-1}(x)\right), j=2,3, \ldots$. Show that for arbitrary $n$ the roots of the equation $P_{n}(x)=x$ are real and different from one another. +10. (USA 6) ${ }^{\mathrm{IMO}}$ Find the largest number obtainable as the product of positive integers whose sum is 1976. +11. (VIE 1) Prove that there exist infinitely many positive integers $n$ such that the decimal representation of $5^{n}$ contains a block of 1976 consecutive zeros. +12. (VIE 2) The polynomial $1976\left(x+x^{2}+\cdots+x^{n}\right)$ is decomposed into a sum of polynomials of the form $a_{1} x+a_{2} x^{2}+\ldots+a_{n} x^{n}$, where $a_{1}, a_{2}, \cdots, a_{n}$ are distinct positive integers not greater than $n$. Find all values of $n$ for which such a decomposition is possible. + +### 3.19 The Nineteenth IMO Belgrade-Arandjelovac, Yugoslavia, July 1-13, 1977 + +### 3.19.1 Contest Problems + +First Day (July 6) + +1. Equilateral triangles $A B K, B C L, C D M, D A N$ are constructed inside the square $A B C D$. Prove that the midpoints of the four segments $K L$, $L M, M N, N K$ and the midpoints of the eight segments $A K, B K, B L$, $C L, C M, D M, D N, A N$ are the twelve vertices of a regular dodecagon. +2. In a finite sequence of real numbers the sum of any seven successive terms is negative, and the sum of any eleven successive terms is positive. Determine the maximum number of terms in the sequence. +3. Let $n$ be a given integer greater than 2 , and let $V_{n}$ be the set of integers $1+k n$, where $k=1,2, \ldots$ A number $m \in V_{n}$ is called indecomposable in $V_{n}$ if there do not exist numbers $p, q \in V_{n}$ such that $p q=m$. Prove that there exists a number $r \in V_{n}$ that can be expressed as the product of elements indecomposable in $V_{n}$ in more than one way. (Expressions that differ only in order of the elements of $V_{n}$ will be considered the same.) + +Second Day (July 7) +4. Let $a, b, A, B$ be given constant real numbers and + +$$ +f(x)=1-a \cos x-b \sin x-A \cos 2 x-B \sin 2 x . +$$ + +Prove that if $f(x) \geq 0$ for all real $x$, then + +$$ +a^{2}+b^{2} \leq 2 \quad \text { and } \quad A^{2}+B^{2} \leq 1 +$$ + +5. Let $a$ and $b$ be natural numbers and let $q$ and $r$ be the quotient and remainder respectively when $a^{2}+b^{2}$ is divided by $a+b$. Determine the numbers $a$ and $b$ if $q^{2}+r=1977$. +6. Let $f: \mathbb{N} \rightarrow \mathbb{N}$ be a function that satisfies the inequality $f(n+1)>f(f(n))$ for all $n \in \mathbb{N}$. Prove that $f(n)=n$ for all natural numbers $n$. + +### 3.19.2 Longlisted Problems + +1. (BUL 1) A pentagon $A B C D E$ inscribed in a circle for which $B CC S$. +2. (BUL 2) (SL77-1). +3. (BUL 3) In a company of $n$ persons, each person has no more than $d$ acquaintances, and in that company there exists a group of $k$ persons, $k \geq d$, who are not acquainted with each other. Prove that the number of acquainted pairs is not greater than $\left[n^{2} / 4\right]$. +4. (BUL 4) We are given $n$ points in space. Some pairs of these points are connected by line segments so that the number of segments equals $\left[n^{2} / 4\right]$, and a connected triangle exists. Prove that any point from which the maximal number of segments starts is a vertex of a connected triangle. +5. (CZS 1) (SL77-2). +6. (CZS 2) Let $x_{1}, x_{2}, \ldots, x_{n}(n \geq 1)$ be real numbers such that $0 \leq x_{j} \leq \pi$, $j=1,2, \ldots, n$. Prove that if $\sum_{j=1}^{n}\left(\cos x_{j}+1\right)$ is an odd integer, then $\sum_{j=1}^{n} \sin x_{j} \geq 1$. +7. (CZS 3) Prove the following assertion: If $c_{1}, c_{2}, \ldots, c_{n}(n \geq 2)$ are real numbers such that + +$$ +(n-1)\left(c_{1}^{2}+c_{2}^{2}+\cdots+c_{n}^{2}\right)=\left(c_{1}+c_{2}+\cdots+c_{n}\right)^{2} +$$ + +then either all these numbers are nonnegative or all these numbers are nonpositive. +8. (CZS 4) A hexahedron $A B C D E$ is made of two regular congruent tetrahedra $A B C D$ and $A B C E$. Prove that there exists only one isometry $\mathcal{Z}$ that maps points $A, B, C, D, E$ onto $B, C, A, E, D$, respectively. Find all points $X$ on the surface of hexahedron whose distance from $\mathcal{Z}(X)$ is minimal. +9. (CZS 5) Let $A B C D$ be a regular tetrahedron and $\mathcal{Z}$ an isometry mapping $A, B, C, D$ into $B, C, D, A$, respectively. Find the set $\mathcal{M}$ of all points $X$ of the face $A B C$ whose distance from $\mathcal{Z}(X)$ is equal to a given number $t$. Find necessary and sufficient conditions for the set $\mathcal{M}$ to be nonempty. +10. (FRG 1) (SL77-3). +11. (FRG 2) Let $n$ and $z$ be integers greater than 1 and $(n, z)=1$. Prove: +(a) At least one of the numbers $z_{i}=1+z+z^{2}+\cdots+z^{i}, i=0,1, \ldots, n-1$, is divisible by $n$. +(b) If $(z-1, n)=1$, then at least one of the numbers $z_{i}, i=0,1, \ldots, n-2$, is divisible by $n$. +12. (FRG 3) Let $z$ be an integer $>1$ and let $M$ be the set of all numbers of the form $z_{k}=1+z+\cdots+z^{k}, k=0,1, \ldots$. Determine the set $T$ of divisors of at least one of the numbers $z_{k}$ from $M$. +13. (FRG 4) (SL77-4). +14. (FRG 5) (SL77-5). +15. (GDR 1) Let $n$ be an integer greater than 1 . In the Cartesian coordinate system we consider all squares with integer vertices $(x, y)$ such that $1 \leq$ $x, y \leq n$. Denote by $p_{k}(k=0,1,2, \ldots)$ the number of pairs of points that are vertices of exactly $k$ such squares. Prove that $\sum_{k}(k-1) p_{k}=0$. +16. (GDR 2) (SL77-6). +17. (GDR 3) A ball $K$ of radius $r$ is touched from the outside by mutually equal balls of radius $R$. Two of these balls are tangent to each other. Moreover, for two balls $K_{1}$ and $K_{2}$ tangent to $K$ and tangent to each other there exist two other balls tangent to $K_{1}, K_{2}$ and also to $K$. How many balls are tangent to $K$ ? For a given $r$ determine $R$. +18. (GDR 4) Given an isosceles triangle $A B C$ with a right angle at $C$, construct the center $M$ and radius $r$ of a circle cutting on segments $A B, B C, C A$ the segments $D E, F G$, and $H K$, respectively, such that $\angle D M E+\angle F M G+\angle H M K=180^{\circ}$ and $D E: F G: H K=A B: B C:$ $C A$. +19. (GBR 1) Given any integer $m>1$ prove that there exist infinitely many positive integers $n$ such that the last $m$ digits of $5^{n}$ are a sequence $a_{m}, a_{m-1}, \ldots, a_{1}=5\left(0 \leq a_{j}<10\right)$ in which each digit except the last is of opposite parity to its successor (i.e., if $a_{i}$ is even, then $a_{i-1}$ is odd, and if $a_{i}$ is odd, then $a_{i-1}$ is even). +20. (GBR 2) (SL77-7). +21. (GBR 3) Given that $x_{1}+x_{2}+x_{3}=y_{1}+y_{2}+y_{3}=x_{1} y_{1}+x_{2} y_{2}+x_{3} y_{3}=0$, prove that + +$$ +\frac{x_{1}^{2}}{x_{1}^{2}+x_{2}^{2}+x_{3}^{2}}+\frac{y_{1}^{2}}{y_{1}^{2}+y_{2}^{2}+y_{3}^{2}}=\frac{2}{3} +$$ + +22. (GBR 4) (SL77-8). +23. (HUN 1) (SL77-9). +24. (HUN 2) Determine all real functions $f(x)$ that are defined and continuous on the interval $(-1,1)$ and that satisfy the functional equation + +$$ +f(x+y)=\frac{f(x)+f(y)}{1-f(x) f(y)} \quad(x, y, x+y \in(-1,1)) . +$$ + +25. (HUN 3) Prove the identity + +$$ +(z+a)^{n}=z^{n}+a \sum_{k=1}^{n}\binom{n}{k}(a-k b)^{k-1}(z+k b)^{n-k} . +$$ + +26. (NET 1) Let $p$ be a prime number greater than 5 . Let $V$ be the collection of all positive integers $n$ that can be written in the form $n=k p+1$ or $n=k p-1(k=1,2, \ldots)$. A number $n \in V$ is called indecomposable in $V$ if it is impossible to find $k, l \in V$ such that $n=k l$. Prove that there exists +a number $N \in V$ that can be factorized into indecomposable factors in $V$ in more than one way. +27. (NET 2) (SL77-10). +28. (NET 3) (SL77-11). +29. (NET 4) (SL77-12). +30. (NET 5) A triangle $A B C$ with $\angle A=30^{\circ}$ and $\angle C=54^{\circ}$ is given. On $B C$ a point $D$ is chosen such that $\angle C A D=12^{\circ}$. On $A B$ a point $E$ is chosen such that $\angle A C E=6^{\circ}$. Let $S$ be the point of intersection of $A D$ and $C E$. Prove that $B S=B C$. +31. (POL 1) Let $f$ be a function defined on the set of pairs of nonzero rational numbers whose values are positive real numbers. Suppose that $f$ satisfies the following conditions: +(1) $f(a b, c)=f(a, c) f(b, c), f(c, a b)=f(c, a) f(c, b)$; +(2) $f(a, 1-a)=1$. + +Prove that $f(a, a)=f(a,-a)=1, f(a, b) f(b, a)=1$. +32. (POL 2) In a room there are nine men. Among every three of them there are two mutually acquainted. Prove that some four of them are mutually acquainted. +33. (POL 3) A circle $K$ centered at $(0,0)$ is given. Prove that for every vector $\left(a_{1}, a_{2}\right)$ there is a positive integer $n$ such that the circle $K$ translated by the vector $n\left(a_{1}, a_{2}\right)$ contains a lattice point (i.e., a point both of whose coordinates are integers). +34. (POL 4) (SL77-13). +35. (ROM 1) Find all numbers $N=\overline{a_{1} a_{2} \ldots a_{n}}$ for which $9 \times \overline{a_{1} a_{2} \ldots a_{n}}=$ $\overline{a_{n} \ldots a_{2} a_{1}}$ such that at most one of the digits $a_{1}, a_{2}, \ldots, a_{n}$ is zero. +36. (ROM 2) Consider a sequence of numbers $\left(a_{1}, a_{2}, \ldots, a_{2^{n}}\right)$. Define the operation + +$$ +S\left(\left(a_{1}, a_{2}, \ldots, a_{2^{n}}\right)\right)=\left(a_{1} a_{2}, a_{2} a_{3}, \ldots, a_{2^{n}-1} a_{2^{n}}, a_{2^{n}} a_{1}\right) +$$ + +Prove that whatever the sequence $\left(a_{1}, a_{2}, \ldots, a_{2^{n}}\right)$ is, with $a_{i} \in\{-1,1\}$ for $i=1,2, \ldots, 2^{n}$, after finitely many applications of the operation we get the sequence $(1,1, \ldots, 1)$. +37. (ROM 3) Let $A_{1}, A_{2}, \ldots, A_{n+1}$ be positive integers such that $\left(A_{i}, A_{n+1}\right)$ $=1$ for every $i=1,2, \ldots, n$. Show that the equation + +$$ +x_{1}^{A_{1}}+x_{2}^{A_{2}}+\cdots+x_{n}^{A_{n}}=x_{n+1}^{A_{n+1}} +$$ + +has an infinite set of solutions $\left(x_{1}, x_{2}, \ldots, x_{n+1}\right)$ in positive integers. +38. (ROM 4) Let $m_{j}>0$ for $j=1,2, \ldots, n$ and $a_{1} \leq \cdots \leq a_{n}3\left(\sum_{j=1}^{n} m_{j}\right)\left[\sum_{j=1}^{n} m_{j}\left(a_{j} b_{j}+b_{j} c_{j}+c_{j} a_{j}\right)\right] . +$$ + +39. (ROM 5) Consider 37 distinct points in space, all with integer coordinates. Prove that we may find among them three distinct points such that their barycenter has integers coordinates. +40. (SWE 1) The numbers $1,2,3, \ldots, 64$ are placed on a chessboard, one number in each square. Consider all squares on the chessboard of size $2 \times 2$. Prove that there are at least three such squares for which the sum of the 4 numbers contained exceeds 100. +41. (SWE 2) A wheel consists of a fixed circular disk and a mobile circular ring. On the disk the numbers $1,2,3, \ldots, N$ are marked, and on the ring $N$ integers $a_{1}, a_{2}, \ldots, a_{N}$ of sum 1 are marked (see the figure). The ring can be turned into $N$ different positions in which the numbers on the disk and on the ring match each other. Multiply every number on the ring with the corresponding number on the disk and form the sum of $N$ products. In this way a +![](https://cdn.mathpix.com/cropped/2024_11_18_8e985d6b9c83aa3e9d0eg-127.jpg?height=349&width=362&top_left_y=824&top_left_x=919) +sum is obtained for every position of the ring. Prove that the $N$ sums are different. +42. (SWE 3) The sequence $a_{n, k}, k=1,2,3, \ldots, 2^{n}, n=0,1,2, \ldots$, is defined by the following recurrence formula: + +$$ +\begin{aligned} +a_{1} & =2, \quad a_{n, k}=2 a_{n-1, k}^{3}, \quad a_{n, k+2^{n-1}}=\frac{1}{2} a_{n-1, k}^{3} \\ +\text { for } k & =1,2,3, \ldots, 2^{n-1}, n=0,1,2, \ldots . +\end{aligned} +$$ + +Prove that the numbers $a_{n, k}$ are all different. +43. (FIN 1) Evaluate + +$$ +S=\sum_{k=1}^{n} k(k+1) \cdots(k+p), +$$ + +where $n$ and $p$ are positive integers. +44. (FIN 2) Let $E$ be a finite set of points in space such that $E$ is not contained in a plane and no three points of $E$ are collinear. Show that $E$ contains the vertices of a tetrahedron $T=A B C D$ such that $T \cap E=$ $\{A, B, C, D\}$ (including interior points of $T$ ) and such that the projection of $A$ onto the plane $B C D$ is inside a triangle that is similar to the triangle $B C D$ and whose sides have midpoints $B, C, D$. +45. (FIN 2') (SL77-14). +46. (FIN 3) Let $f$ be a strictly increasing function defined on the set of real numbers. For $x$ real and $t$ positive, set + +$$ +g(x, t)=\frac{f(x+t)-f(x)}{f(x)-f(x-t)} +$$ + +Assume that the inequalities + +$$ +2^{-1}x_{1}>\cdots>x_{n}$. Prove that at least one of the numbers $\left|F\left(x_{0}\right)\right|,\left|F\left(x_{1}\right)\right|,\left|F\left(x_{2}\right)\right|, \ldots$, $\left|F\left(x_{n}\right)\right|$, where + +$$ +F(x)=x^{n}+a_{1} x^{n-1}+\cdots+a_{n}, \quad a_{i} \in \mathbb{R}, \quad i=1, \ldots, n +$$ + +is greater than $\frac{n!}{2^{n}}$. + +### 3.19.3 Shortlisted Problems + +1. (BUL 2) ${ }^{\mathrm{IMO} 06}$ Let $f: \mathbb{N} \rightarrow \mathbb{N}$ be a function that satisfies the inequality $f(n+1)>f(f(n))$ for all $n \in \mathbb{N}$. Prove that $f(n)=n$ for all natural numbers $n$. +2. (CZS 1) A lattice point in the plane is a point both of whose coordinates are integers. Each lattice point has four neighboring points: upper, lower, left, and right. Let $k$ be a circle with radius $r \geq 2$, that does not pass through any lattice point. An interior boundary point is a lattice point lying inside the circle $k$ that has a neighboring point lying outside $k$. Similarly, an exterior boundary point is a lattice point lying outside the circle $k$ that has a neighboring point lying inside $k$. Prove that there are four more exterior boundary points than interior boundary points. +3. (FRG 1) ${ }^{\mathrm{IMO} 5}$ Let $a$ and $b$ be natural numbers and let $q$ and $r$ be the quotient and remainder respectively when $a^{2}+b^{2}$ is divided by $a+b$. Determine the numbers $a$ and $b$ if $q^{2}+r=1977$. +4. (FRG 4) Describe all closed bounded figures $\Phi$ in the plane any two points of which are connectable by a semicircle lying in $\Phi$. +5. (FRG 5) There are $2^{n}$ words of length $n$ over the alphabet $\{0,1\}$. Prove that the following algorithm generates the sequence $w_{0}, w_{1}, \ldots, w_{2^{n}-1}$ of all these words such that any two consecutive words differ in exactly one digit. +(1) $w_{0}=00 \ldots 0$ ( $n$ zeros). +(2) Suppose $w_{m-1}=a_{1} a_{2} \ldots a_{n}, a_{i} \in\{0,1\}$. Let $e(m)$ be the exponent of 2 in the representation of $n$ as a product of primes, and let $j=$ $1+e(m)$. Replace the digit $a_{j}$ in the word $w_{m-1}$ by $1-a_{j}$. The obtained word is $w_{m}$. +6. (GDR 2) Let $n$ be a positive integer. How many integer solutions $(i, j, k, l), 1 \leq i, j, k, l \leq n$, does the following system of inequalities have: + +$$ +\begin{aligned} +& 1 \leq-j+k+l \leq n \\ +& 1 \leq \quad i-k+l \leq n \\ +& 1 \leq \quad i-j+l \leq n \\ +& 1 \leq \quad i+j-k \leq n ? +\end{aligned} +$$ + +7. (GBR 2) ${ }^{\mathrm{IMO} 4}$ Let $a, b, A, B$ be given constant real numbers and + +$$ +f(x)=1-a \cos x-b \sin x-A \cos 2 x-B \sin 2 x +$$ + +Prove that if $f(x) \geq 0$ for all real $x$, then + +$$ +a^{2}+b^{2} \leq 2 \quad \text { and } \quad A^{2}+B^{2} \leq 1 +$$ + +8. (GBR 4) Let $S$ be a convex quadrilateral $A B C D$ and $O$ a point inside it. The feet of the perpendiculars from $O$ to $A B, B C, C D, D A$ are $A_{1}, B_{1}$, $C_{1}, D_{1}$ respectively. The feet of the perpendiculars from $O$ to the sides of $S_{i}$, the quadrilateral $A_{i} B_{i} C_{i} D_{i}$, are $A_{i+1} B_{i+1} C_{i+1} D_{i+1}$, where $i=1,2,3$. Prove that $S_{4}$ is similar to $S$. +9. (HUN 1) For which positive integers $n$ do there exist two polynomials $f$ and $g$ with integer coefficients of $n$ variables $x_{1}, x_{2}, \ldots, x_{n}$ such that the following equality is satisfied: + +$$ +\left(\sum_{i=1}^{n} x_{i}\right) f\left(x_{1}, x_{2}, \ldots, x_{n}\right)=g\left(x_{1}^{2}, x_{2}^{2}, \ldots, x_{n}^{2}\right) ? +$$ + +10. (NET 2) ${ }^{\mathrm{IMO} 3}$ Let $n$ be an integer greater than 2 . Define $V=\{1+k n \mid$ $k=1,2, \ldots\}$. A number $p \in V$ is called indecomposable in $V$ if it is not possible to find numbers $q_{1}, q_{2} \in V$ such that $q_{1} q_{2}=p$. Prove that there exists a number $N \in V$ that can be factorized into indecomposable factors in $V$ in more than one way. +11. (NET 3) Let $n$ be an integer greater than 1 . Define +$x_{1}=n, \quad y_{1}=1, \quad x_{i+1}=\left[\frac{x_{i}+y_{i}}{2}\right], \quad y_{i+1}=\left[\frac{n}{x_{i+1}}\right] \quad$ for $i=1,2, \ldots$, +where $[z]$ denotes the largest integer less than or equal to $z$. Prove that + +$$ +\min \left\{x_{1}, x_{2}, \ldots x_{n}\right\}=[\sqrt{n}] . +$$ + +12. (NET 4) ${ }^{\mathrm{IMO}}$ On the sides of a square $A B C D$ one constructs inwardly equilateral triangles $A B K, B C L, C D M, D A N$. Prove that the midpoints of the four segments $K L, L M, M N, N K$, together with the midpoints of the eight segments $A K, B K, B L, C L, C M, D M, D N, A N$, are the 12 vertices of a regular dodecagon. +13. (POL 4) Let $B$ be a set of $k$ sequences each having $n$ terms equal to 1 or -1 . The product of two such sequences $\left(a_{1}, a_{2}, \ldots, a_{n}\right)$ and $\left(b_{1}, b_{2}, \ldots, b_{n}\right)$ is defined as $\left(a_{1} b_{1}, a_{2} b_{2}, \ldots, a_{n} b_{n}\right)$. Prove that there exists a sequence $\left(c_{1}, c_{2}, \ldots, c_{n}\right)$ such that the intersection of $B$ and the set containing all sequences from $B$ multiplied by $\left(c_{1}, c_{2}, \ldots, c_{n}\right)$ contains at most $k^{2} / 2^{n}$ sequences. +14. (FIN 2‘) Let $E$ be a finite set of points such that $E$ is not contained in a plane and no three points of $E$ are collinear. Show that at least one of the following alternatives holds: +(i) $E$ contains five points that are vertices of a convex pyramid having no other points in common with $E$; +(ii) some plane contains exactly three points from $E$. +15. (VIE 1) ${ }^{\mathrm{IMO} 2}$ The length of a finite sequence is defined as the number of terms of this sequence. Determine the maximal possible length of a finite sequence that satisfies the following condition: The sum of each seven successive terms is negative, and the sum of each eleven successive terms is positive. +16. (VIE 3) Let $E$ be a set of $n$ points in the plane $(n \geq 3)$ whose coordinates are integers such that any three points from $E$ are vertices of a nondegenerate triangle whose centroid doesn't have both coordinates integers. Determine the maximal $n$. + +### 3.20 The Twentieth IMO Bucharest, Romania, 1978 + +### 3.20.1 Contest Problems + +First Day (July 6) + +1. Let $n>m \geq 1$ be natural numbers such that the groups of the last three digits in the decimal representation of $1978^{m}, 1978^{n}$ coincide. Find the ordered pair $(m, n)$ of such $m, n$ for which $m+n$ is minimal. +2. Given any point $P$ in the interior of a sphere with radius $R$, three mutually perpendicular segments $P A, P B, P C$ are drawn terminating on the sphere and having one common vertex in $P$. Consider the rectangular parallelepiped of which $P A, P B, P C$ are coterminal edges. Find the locus of the point $Q$ that is diagonally opposite $P$ in the parallelepiped when $P$ and the sphere are fixed. +3. Let $\{f(n)\}$ be a strictly increasing sequence of positive integers: $0<$ $f(1)1$ there exists a triangle whose sides have lengths $P_{1}(X)=X^{4}+X^{3}+2 X^{2}+X+1, P_{2}(X)=2 X^{3}+X^{2}+2 X+1$, and $P_{3}(X)=X^{4}-1$. Prove that all these triangles have the same greatest angle and calculate it. +7. (CUB 3) (SL78-3). +8. (CZS 1) For two given triangles $A_{1} A_{2} A_{3}$ and $B_{1} B_{2} B_{3}$ with areas $\Delta_{A}$ and $\Delta_{B}$, respectively, $A_{i} A_{k} \geq B_{i} B_{k}, i, k=1,2,3$. Prove that $\Delta_{A} \geq \Delta_{B}$ if the triangle $A_{1} A_{2} A_{3}$ is not obtuse-angled. +9. (CZS 2) (SL78-4). +10. (CZS 3) Show that for any natural number $n$ there exist two prime numbers $p$ and $q, p \neq q$, such that $n$ divides their difference. +11. (CZS 4) Find all natural numbers $n<1978$ with the following property: If $m$ is a natural number, $10$ that has two real roots $x_{1}, x_{2}$. Prove that the absolute values of both roots are less than or equal to 1 if and only if $a+b+c \geq 0, a-b+c \geq 0$, and $a-c \geq 0$. +26. (GDR 2) (SL78-5). +27. (GDR 3) Determine the sixth number after the decimal point in the number $(\sqrt{1978}+[\sqrt{1978}])^{20}$. +28. (GDR 4) Let $c, s$ be real functions defined on $\mathbb{R} \backslash\{0\}$ that are nonconstant on any interval and satisfy + +$$ +c\left(\frac{x}{y}\right)=c(x) c(y)-s(x) s(y) \quad \text { for any } x \neq 0, y \neq 0 +$$ + +Prove that: +(a) $c(1 / x)=c(x), s(1 / x)=-s(x)$ for any $x \neq 0$, and also $c(1)=1$, $s(1)=s(-1)=0$; +(b) $c$ and $s$ are either both even or both odd functions (a function $f$ is even if $f(x)=f(-x)$ for all $x$, and odd if $f(x)=-f(-x)$ for all $x$ ). +Find functions $c, s$ that also satisfy $c(x)+s(x)=x^{n}$ for all $x$, where $n$ is a given positive integer. +29. (GDR 5) (Variant of GDR 4) Given a nonconstant function $f: \mathbb{R}^{+} \rightarrow \mathbb{R}$ such that $f(x y)=f(x) f(y)$ for any $x, y>0$, find functions $c, s: \mathbb{R}^{+} \rightarrow \mathbb{R}$ that satisfy $c(x / y)=c(x) c(y)-s(x) s(y)$ for all $x, y>0$ and $c(x)+s(x)=$ $f(x)$ for all $x>0$. +30. (NET 1) (SL78-10). +31. (NET 2) Let the polynomials + +$$ +\begin{aligned} +& P(x)=x^{n}+a_{n-1} x^{n-1}+\cdots+a_{1} x+a_{0} \\ +& Q(x)=x^{m}+b_{m-1} x^{m-1}+\cdots+b_{1} x+b_{0} +\end{aligned} +$$ + +be given satisfying the identity $P(x)^{2}=\left(x^{2}-1\right) Q(x)^{2}+1$. Prove the identity + +$$ +P^{\prime}(x)=n Q(x) +$$ + +32. (NET 3) Let $\mathcal{C}$ be the circumcircle of the square with vertices $(0,0)$, $(0,1978),(1978,0),(1978,1978)$ in the Cartesian plane. Prove that $\mathcal{C}$ contains no other point for which both coordinates are integers. +33. (SWE 1) A sequence $\left(a_{n}\right)_{0}^{\infty}$ of real numbers is called convex if $2 a_{n} \leq$ $a_{n-1}+a_{n+1}$ for all positive integers $n$. Let $\left(b_{n}\right)_{0}^{\infty}$ be a sequence of positive numbers and assume that the sequence $\left(\alpha^{n} b_{n}\right)_{0}^{\infty}$ is convex for any choice of $\alpha>0$. Prove that the sequence $\left(\log b_{n}\right)_{0}^{\infty}$ is convex. +34. (SWE 2) (SL78-11). +35. (SWE 3) A sequence $\left(a_{n}\right)_{0}^{N}$ of real numbers is called concave if $2 a_{n} \geq$ $a_{n-1}+a_{n+1}$ for all integers $n, 1 \leq n \leq N-1$. +(a) Prove that there exists a constant $C>0$ such that + +$$ +\left(\sum_{n=0}^{N} a_{n}\right)^{2} \geq C(N-1) \sum_{n=0}^{N} a_{n}^{2} +$$ + +for all concave positive sequences $\left(a_{n}\right)_{0}^{N}$. +(b) Prove that (1) holds with $C=3 / 4$ and that this constant is best possible. +36. (TUR 1) The integers 1 through 1000 are located on the circumference of a circle in natural order. Starting with 1, every fifteenth number (i.e., $1,16,31, \ldots)$ is marked. The marking is continued until an already marked number is reached. How many of the numbers will be left unmarked? +37. (TUR 2) Simplify + +$$ +\frac{1}{\log _{a}(a b c)}+\frac{1}{\log _{b}(a b c)}+\frac{1}{\log _{c}(a b c)} +$$ + +where $a, b, c$ are positive real numbers. +38. (TUR 3) Given a circle, construct a chord that is trisected by two given noncollinear radii. +39. (TUR 4) $A$ is a $2 m$-digit positive integer each of whose digits is $1 . B$ is an $m$-digit positive integer each of whose digits is 4 . Prove that $A+B+1$ is a perfect square. +40. (TUR 5) If $C_{n}^{p}=\frac{n!}{p!(n-p)!}(p \geq 1)$, prove the identity + +$$ +C_{n}^{p}=C_{n-1}^{p-1}+C_{n-2}^{p-1}+\cdots+C_{p}^{p-1}+C_{p-1}^{p-1} +$$ + +and then evaluate the sum + +$$ +S=1 \cdot 2 \cdot 3+2 \cdot 3 \cdot 4+\cdots+97 \cdot 98 \cdot 99 +$$ + +41. (USA 1) (SL78-12). +42. (USA 2) $A, B, C, D, E$ are points on a circle $O$ with radius equal to $r$. Chords $A B$ and $D E$ are parallel to each other and have length equal to $x$. Diagonals $A C, A D, B E, C E$ are drawn. If segment $X Y$ on $O$ meets $A C$ at $X$ and $E C$ at $Y$, prove that lines $B X$ and $D Y$ meet at $Z$ on the circle. +43. (USA 3) If $p$ is a prime greater than 3 , show that at least one of the numbers $\frac{3}{p^{2}}, \frac{4}{p^{2}}, \ldots, \frac{p-2}{p^{2}}$ is expressible in the form $\frac{1}{x}+\frac{1}{y}$, where $x$ and $y$ are positive integers. +44. (USA 4) In $\triangle A B C$ with $\angle C=60^{\circ}$, prove that $\frac{c}{a}+\frac{c}{b} \geq 2$. +45. (USA 5) If $r>s>0$ and $a>b>c$, prove that + +$$ +a^{r} b^{s}+b^{r} c^{s}+c^{r} a^{s} \geq a^{s} b^{r}+b^{s} c^{r}+c^{s} a^{r} . +$$ + +46. (USA 6) (SL78-13). +47. (VIE 1) Given the expression + +$$ +P_{n}(x)=\frac{1}{2^{n}}\left[\left(x+\sqrt{x^{2}-1}\right)^{n}+\left(x-\sqrt{x^{2}-1}\right)^{n}\right] +$$ + +prove: +(a) $P_{n}(x)$ satisfies the identity $P_{n}(x)-x P_{n-1}(x)+\frac{1}{4} P_{n-2}(x) \equiv 0$. +(b) $P_{n}(x)$ is a polynomial in $x$ of degree $n$. +48. (VIE 2) (SL78-14). +49. (VIE 3) Let $A, B, C, D$ be four arbitrary distinct points in space. +(a) Prove that using the segments $A B+C D, A C+B D$ and $A D+B C$ it is always possible to construct a triangle $T$ that is nondegenerate and has no obtuse angle. +(b) What should these four points satisfy in order for the triangle $T$ to be right-angled? +50. (VIE 4) A variable tetrahedron $A B C D$ has the following properties: Its edge lengths can change as well as its vertices, but the opposite edges remain equal $(B C=D A, C A=D B, A B=D C)$; and the vertices $A, B, C$ lie respectively on three fixed spheres with the same center $P$ and radii $3,4,12$. What is the maximal length of $P D$ ? +51. (VIE 5) Find the relations among the angles of the triangle $A B C$ whose altitude $A H$ and median $A M$ satisfy $\angle B A H=\angle C A M$. +52. (YUG 1) (SL78-15). +53. (YUG 2) (SL78-16). +54. (YUG 3) Let $p, q$ and $r$ be three lines in space such that there is no plane that is parallel to all three of them. Prove that there exist three planes $\alpha, \beta$, and $\gamma$, containing $p, q$, and $r$ respectively, that are perpendicular to each other $(\alpha \perp \beta, \beta \perp \gamma, \gamma \perp \alpha)$. + +### 3.20.3 Shortlisted Problems + +1. (BUL 1) The set $M=\{1,2, \ldots, 2 n\}$ is partitioned into $k$ nonintersecting subsets $M_{1}, M_{2}, \ldots, M_{k}$, where $n \geq k^{3}+k$. Prove that there exist even numbers $2 j_{1}, 2 j_{2}, \ldots, 2 j_{k+1}$ in $M$ that are in one and the same subset $M_{i}$ $(1 \leq i \leq k)$ such that the numbers $2 j_{1}-1,2 j_{2}-1, \ldots, 2 j_{k+1}-1$ are also in one and the same subset $M_{j}(1 \leq j \leq k)$. +2. (BUL 4) Two identically oriented equilateral triangles, $A B C$ with center $S$ and $A^{\prime} B^{\prime} C$, are given in the plane. We also have $A^{\prime} \neq S$ and $B^{\prime} \neq S$. If $M$ is the midpoint of $A^{\prime} B$ and $N$ the midpoint of $A B^{\prime}$, prove that the triangles $S B^{\prime} M$ and $S A^{\prime} N$ are similar. +3. (CUB 3) ${ }^{\mathrm{IMO} 1}$ Let $n>m \geq 1$ be natural numbers such that the groups of the last three digits in the decimal representation of $1978^{m}, 1978^{n}$ coincide. Find the ordered pair $(m, n)$ of such $m, n$ for which $m+n$ is minimal. +4. (CZS 2) Let $T_{1}$ be a triangle having $a, b, c$ as lengths of its sides and let $T_{2}$ be another triangle having $u, v, w$ as lengths of its sides. If $P, Q$ are the areas of the two triangles, prove that + +$$ +16 P Q \leq a^{2}\left(-u^{2}+v^{2}+w^{2}\right)+b^{2}\left(u^{2}-v^{2}+w^{2}\right)+c^{2}\left(u^{2}+v^{2}-w^{2}\right) +$$ + +When does equality hold? +5. (GDR 2) For every integer $d \geq 1$, let $M_{d}$ be the set of all positive integers that cannot be written as a sum of an arithmetic progression with difference $d$, having at least two terms and consisting of positive integers. Let $A=M_{1}, B=M_{2} \backslash\{2\}, C=M_{3}$. Prove that every $c \in C$ may be written in a unique way as $c=a b$ with $a \in A, b \in B$. +6. (FRA 2) $)^{\mathrm{IMO} 5}$ Let $\varphi:\{1,2,3, \ldots\} \rightarrow\{1,2,3, \ldots\}$ be injective. Prove that for all $n$, + +$$ +\sum_{k=1}^{n} \frac{\varphi(k)}{k^{2}} \geq \sum_{k=1}^{n} \frac{1}{k} +$$ + +7. (FRA 5) We consider three distinct half-lines $O x, O y, O z$ in a plane. Prove the existence and uniqueness of three points $A \in O x, B \in O y$, $C \in O z$ such that the perimeters of the triangles $O A B, O B C, O C A$ are all equal to a given number $2 p>0$. +8. (GBR 4) Let $S$ be the set of all the odd positive integers that are not multiples of 5 and that are less than $30 \mathrm{~m}, \mathrm{~m}$ being an arbitrary positive integer. What is the smallest integer $k$ such that in any subset of $k$ integers from $S$ there must be two different integers, one of which divides the other? +9. $\mathbf{( G B R} \mathbf{5})^{\mathrm{IMO} 3}$ Let $\{f(n)\}$ be a strictly increasing sequence of positive integers: $02$. +49. (NET 3) Let there be given two sequences of integers $f_{i}(1), f_{i}(2), \ldots$ ( $i=1,2)$ satisfying: +(i) $f_{i}(n m)=f_{i}(n) f_{i}(m)$ if $\operatorname{gcd}(n, m)=1$; +(ii) for every prime $P$ and all $k=2,3,4, \ldots, f_{i}\left(P^{k}\right)=f_{i}(P) f_{i}\left(P^{k-1}\right)-$ $P^{2} f\left(P^{k-2}\right)$. +Moreover, for every prime $P$ : +(iii) $f_{1}(P)=2 P$, +(iv) $f_{2}(P)<2 P$. + +Prove that $\left|f_{2}(n)\right|1$ be given and a sequence $\left(n_{k}\right)$ of positive integers such that $\frac{n_{k+1}}{n_{k}}>\lambda$ for $k=1,2, \ldots$ Prove that there exists a positive integer $c$ such that no positive integer $n$ can be represented in more than $c$ ways in the form $n=n_{k}+n_{j}$ or $n=n_{r}-n_{s}$. +53. (POL 4) An infinite increasing sequence of positive integers $n_{j}$ ( $j=$ $1,2, \ldots$ ) has the property that for a certain $c, \frac{1}{N} \sum_{n_{j} \leq N} n_{j} \leq c$, for every $N>0$ +Prove that there exist finitely many sequences $m_{j}^{(i)}(i=1,2, \ldots, k)$ such that + +$$ +\begin{aligned} +& \left\{n_{1}, n_{2}, \ldots\right\}=\bigcup_{i=1}^{k}\left\{m_{1}^{(i)}, m_{2}^{(i)}, \ldots\right\} \quad \text { and } \\ +& m_{j+1}^{(i)}>2 m_{j}^{(i)} \quad(1 \leq i \leq k, j=1,2, \ldots) . +\end{aligned} +$$ + +54. (ROM 1) (SL79-19). +55. (ROM 2) Let $a, b$ be coprime integers. Show that the equation $a x^{2}+$ $b y^{2}=z^{3}$ has an infinite set of solutions $(x, y, z)$ with $x, y, z \in \mathbb{Z}$ and $x, y$ mutually coprime (in each solution). +56. (ROM 3) Show that for every natural number $n, n \sqrt{2}-[n \sqrt{2}]>\frac{1}{2 n \sqrt{2}}$ and that for every $\varepsilon>0$ there exists a natural number $n$ with $n \sqrt{2}-$ $[n \sqrt{2}]<\frac{1}{2 n \sqrt{2}}+\varepsilon$. +57. (ROM 4) Let $M$ be a set, and $A, B, C$ given subsets of $M$. Find a necessary and sufficient condition for the existence of a set $X \subset M$ for which $(X \cup A) \backslash(X \cap B)=C$. Describe all such sets $X$. +58. (ROM 5) Prove that there exists a natural number $k_{0}$ such that for every natural number $k>k_{0}$ we may find a finite number of lines in the plane, not all parallel to one of them, that divide the plane exactly in $k$ regions. Find $k_{0}$. +59. (SWE 1) Determine the maximum value of $x^{2} y^{2} z^{2} w$ when $x, y, z, w \geq 0$ and + +$$ +2 x+x y+z+y z w=1 +$$ + +60. (SWE 2) (SL79-20). +61. (SWE 3) Let $a_{1} \leq a_{2} \leq \cdots \leq a_{n}$ and $b_{1} \leq b_{2} \leq \cdots \leq b_{n}$ be two sequences such that $\sum_{k=1}^{m} a_{k} \geq \sum_{k=1}^{m} b_{k}$ for all $m \leq n$ with equality for $m=n$. Let $f$ be a convex function defined on the real numbers. Prove that + +$$ +\sum_{k=1}^{n} f\left(a_{k}\right) \leq \sum_{k=1}^{n} f\left(b_{k}\right) +$$ + +62. (SWE 4) $T$ is a given triangle with vertices $P_{1}, P_{2}, P_{3}$. Consider an arbitrary subdivision of $T$ into finitely many subtriangles such that no vertex of a subtriangle lies strictly between two vertices of another subtriangle. To each vertex $V$ of the subtriangles there is assigned a number $n(V)$ according to the following rules: +(i) If $V=P_{i}$, then $n(V)=i$. +(ii) If $V$ lies on the side $P_{i} P_{j}$ of $T$, then $n(V)=i$ or $j$. +(iii) If $V$ lies inside the triangle $T$, then $n(V)$ is any of the numbers $1,2,3$. Prove that there exists at least one subtriangle whose vertices are numbered 1, 2, and 3 . +63. (USA 1) If $a_{1}, a_{2}, \ldots, a_{n}$ denote the lengths of the sides of an arbitrary $n$-gon, prove that + +$$ +2 \geq \frac{a_{1}}{s-a_{1}}+\frac{a_{2}}{s-a_{2}}+\cdots+\frac{a_{n}}{s-a_{n}} \geq \frac{n}{n-1} +$$ + +where $s=a_{1}+a_{2}+\cdots+a_{n}$. +64. (USA 2) From point $P$ on arc $B C$ of the circumcircle about triangle $A B C, P X$ is constructed perpendicular to $B C, P Y$ is perpendicular to $A C$, and $P Z$ perpendicular to $A B$ (all extended if necessary). Prove that + +$$ +\frac{B C}{P X}=\frac{A C}{P Y}+\frac{A B}{P Z} +$$ + +65. (USA 3) Given $f(x) \leq x$ for all real $x$ and + +$$ +f(x+y) \leq f(x)+f(y) \quad \text { for all real } x, y, +$$ + +prove that $f(x)=x$ for all $x$. +66. (USA 4) (SL79-23). +67. (USA 5) (SL79-24). +68. (USA 6) (SL79-25). +69. (USS 1) (SL79-21). +70. (USS 2) There are 1979 equilateral triangles: $T_{1}, T_{2}, \ldots, T_{1979}$. A side of triangle $T_{k}$ is equal to $1 / k, k=1,2, \ldots, 1979$. At what values of a number $a$ can one place all these triangles into the equilateral triangle with side length $a$ so that they don't intersect (points of contact are allowed)? +71. (USS 3) (SL79-22). +72. (VIE 1) Let $f(x)$ be a polynomial with integer coefficients. Prove that if $f(x)$ equals 1979 for four different integer values of $x$, then $f(x)$ cannot be equal to $2 \times 1979$ for any integral value of $x$. +73. (VIE 2) In a plane a finite number of equal circles are given. These circles are mutually nonintersecting (they may be externally tangent). Prove that one can use at most four colors for coloring these circles so that two circles tangent to each other are of different colors. What is the smallest number of circles that requires four colors? +74. (VIE 3) Given an equilateral triangle $A B C$ of side $a$ in a plane, let $M$ be a point on the circumcircle of the triangle. Prove that the sum $s=M A^{4}+M B^{4}+M C^{4}$ is independent of the position of the point $M$ on the circle, and determine that constant value as a function of $a$. +75. (VIE 4) Given an equilateral triangle $A B C$, let $M$ be an arbitrary point in space. +(a) Prove that one can construct a triangle from the segments $M A, M B$, $M C$. +(b) Suppose that $P$ and $Q$ are two points symmetric with respect to the center $O$ of $A B C$. Prove that the two triangles constructed from the segments $P A, P B, P C$ and $Q A, Q B, Q C$ are of equal area. +76. (VIE 5) Suppose that a triangle whose sides are of integer lengths is inscribed in a circle of diameter 6.25. Find the sides of the triangle. +77. (YUG 1) By $h(n)$, where $n$ is an integer greater than 1 , let us denote the greatest prime divisor of the number $n$. Are there infinitely many numbers $n$ for which $h(n)a_{100}$. +20. (SWE 2) Given the integer $n>1$ and the real number $a>0$ determine the maximum of $\sum_{i=1}^{n-1} x_{i} x_{i+1}$ taken over all nonnegative numbers $x_{i}$ with sum $a$. +21. (USS 1) Let $N$ be the number of integral solutions of the equation + +$$ +x^{2}-y^{2}=z^{3}-t^{3} +$$ + +satisfying the condition $0 \leq x, y, z, t \leq 10^{6}$, and let $M$ be the number of integral solutions of the equation + +$$ +x^{2}-y^{2}=z^{3}-t^{3}+1 +$$ + +satisfying the condition $0 \leq x, y, z, t \leq 10^{6}$. Prove that $N>M$. +22. (USS 3) ${ }^{\mathrm{IMO} 3}$ There are two circles in the plane. Let a point $A$ be one of the points of intersection of these circles. Two points begin moving simultaneously with constant speeds from the point $A$, each point along its own circle. The two points return to the point $A$ at the same time. Prove that there is a point $P$ in the plane such that at every moment of time the distances from the point $P$ to the moving points are equal. +23. (USA 4) Find all natural numbers $n$ for which $2^{8}+2^{11}+2^{n}$ is a perfect square. +24. (USA 5) A circle O with center $O$ on base $B C$ of an isosceles triangle $A B C$ is tangent to the equal sides $A B, A C$. If point $P$ on $A B$ and point $Q$ on $A C$ are selected such that $P B \times C Q=(B C / 2)^{2}$, prove that line segment $P Q$ is tangent to circle O , and prove the converse. +25. (USA 6) ${ }^{\mathrm{IMO} 4}$ Given a point $P$ in a given plane $\pi$ and also a given point $Q$ not in $\pi$, show how to determine a point $R$ in $\pi$ such that $\frac{Q P+P R}{Q R}$ is a maximum. +26. (YUG 4) Prove that the functional equations + +$$ +\begin{aligned} +f(x+y) & =f(x)+f(y) \\ +\text { and } \quad f(x+y+x y) & =f(x)+f(y)+f(x y) \quad(x, y \in \mathbb{R}) +\end{aligned} +$$ + +are equivalent. + +### 3.22 The Twenty-Second IMO Washington DC, United States of America, July 8-20, 1981 + +### 3.22.1 Contest Problems + +## First Day (July 13) + +1. Find the point $P$ inside the triangle $A B C$ for which + +$$ +\frac{B C}{P D}+\frac{C A}{P E}+\frac{A B}{P F} +$$ + +is minimal, where $P D, P E, P F$ are the perpendiculars from $P$ to $B C$, $C A, A B$ respectively. +2. Let $f(n, r)$ be the arithmetic mean of the minima of all $r$-subsets of the set $\{1,2, \ldots, n\}$. Prove that $f(n, r)=\frac{n+1}{r+1}$. +3. Determine the maximum value of $m^{2}+n^{2}$ where $m$ and $n$ are integers satisfying + +$$ +m, n \in\{1,2, \ldots, 1981\} \quad \text { and } \quad\left(n^{2}-m n-m^{2}\right)^{2}=1 +$$ + +Second Day (July 14) +4. (a) For which values of $n>2$ is there a set of $n$ consecutive positive integers such that the largest number in the set in the set is a divisor of the least common multiple of the remaining $n-1$ numbers? +(b) For which values of $n>2$ is there a unique set having the stated property? +5. Three equal circles touch the sides of a triangle and have one common point $O$. Show that the center of the circle inscribed in and of the circle circumscribed about the triangle $A B C$ and the point $O$ are collinear. +6. Assume that $f(x, y)$ is defined for all positive integers $x$ and $y$, and that the following equations are satisfied: + +$$ +\begin{aligned} +f(0, y) & =y+1 \\ +f(x+1,0) & =f(x, 1) \\ +f(x+1, y+1) & =f(x, f(x+1, y)) +\end{aligned} +$$ + +Determine $f(4,1981)$. + +### 3.22.2 Shortlisted Problems + +1. (BEL) ${ }^{\mathrm{IMO} 4}$ (a) For which values of $n>2$ is there a set of $n$ consecutive positive integers such that the largest number in the set is a divisor of the least common multiple of the remaining $n-1$ numbers? +(b) For which values of $n>2$ is there a unique set having the stated property? +2. (BUL) A sphere $S$ is tangent to the edges $A B, B C, C D, D A$ of a tetrahedron $A B C D$ at the points $E, F, G, H$ respectively. The points $E, F, G, H$ are the vertices of a square. Prove that if the sphere is tangent to the edge $A C$, then it is also tangent to the edge $B D$. +3. (CAN) Find the minimum value of + +$$ +\max (a+b+c, b+c+d, c+d+e, d+e+f, e+f+g) +$$ + +subject to the constraints +(i) $a, b, c, d, e, f, g \geq 0$, +(ii) $a+b+c+d+e+f+g=1$. +4. (CAN) Let $\left\{f_{n}\right\}$ be the Fibonacci sequence $\{1,1,2,3,5, \ldots\}$. +(a) Find all pairs $(a, b)$ of real numbers such that for each $n, a f_{n}+b f_{n+1}$ is a member of the sequence. +(b) Find all pairs $(u, v)$ of positive real numbers such that for each $n$, $u f_{n}^{2}+v f_{n+1}^{2}$ is a member of the sequence. +5. (COL) A cube is assembled with 27 white cubes. The larger cube is then painted black on the outside and disassembled. A blind man reassembles it. What is the probability that the cube is now completely black on the outside? Give an approximation of the size of your answer. +6. (CUB) Let $P(z)$ and $Q(z)$ be complex-variable polynomials, with degree not less than 1. Let + +$$ +P_{k}=\{z \in \mathbb{C} \mid P(z)=k\}, \quad Q_{k}=\{z \in \mathbb{C} \mid Q(z)=k\} +$$ + +Let also $P_{0}=Q_{0}$ and $P_{1}=Q_{1}$. Prove that $P(z) \equiv Q(z)$. +7. (FIN) ${ }^{\mathrm{IMO} 6}$ Assume that $f(x, y)$ is defined for all positive integers $x$ and $y$, and that the following equations are satisfied: + +$$ +\begin{aligned} +f(0, y) & =y+1 \\ +f(x+1,0) & =f(x, 1) \\ +f(x+1, y+1) & =f(x, f(x+1, y)) +\end{aligned} +$$ + +Determine $f(2,2), f(3,3)$ and $f(4,4)$. +Alternative version: Determine $f(4,1981)$. +8. (FRG) ${ }^{\mathrm{IMO} 2}$ Let $f(n, r)$ be the arithmetic mean of the minima of all $r$ subsets of the set $\{1,2, \ldots, n\}$. Prove that $f(n, r)=\frac{n+1}{r+1}$. +9. (FRG) A sequence $\left(a_{n}\right)$ is defined by means of the recursion + +$$ +a_{1}=1, \quad a_{n+1}=\frac{1+4 a_{n}+\sqrt{1+24 a_{n}}}{16} +$$ + +Find an explicit formula for $a_{n}$. +10. (FRA) Determine the smallest natural number $n$ having the following property: For every integer $p, p \geq n$, it is possible to subdivide (partition) a given square into $p$ squares (not necessarily equal). +11. (NET) On a semicircle with unit radius four consecutive chords $A B, B C$, $C D, D E$ with lengths $a, b, c, d$, respectively, are given. Prove that + +$$ +a^{2}+b^{2}+c^{2}+d^{2}+a b c+b c d<4 +$$ + +12. (NET) ${ }^{\mathrm{IMO} 3}$ Determine the maximum value of $m^{2}+n^{2}$ where $m$ and $n$ are integers satisfying + +$$ +m, n \in\{1,2, \ldots, 100\} \quad \text { and } \quad\left(n^{2}-m n-m^{2}\right)^{2}=1 +$$ + +13. (ROM) Let $P$ be a polynomial of degree $n$ satisfying + +$$ +P(k)=\binom{n+1}{k}^{-1} \quad \text { for } k=0,1, \ldots, n +$$ + +Determine $P(n+1)$. +14. (ROM) Prove that a convex pentagon (a five-sided polygon) $A B C D E$ with equal sides and for which the interior angles satisfy the condition $\angle A \geq \angle B \geq \angle C \geq \angle D \geq \angle E$ is a regular pentagon. +15. (GBR) ${ }^{\mathrm{IMO} 1}$ Find the point $P$ inside the triangle $A B C$ for which + +$$ +\frac{B C}{P D}+\frac{C A}{P E}+\frac{A B}{P F} +$$ + +is minimal, where $P D, P E, P F$ are the perpendiculars from $P$ to $B C, C A$, $A B$ respectively. +16. (GBR) A sequence of real numbers $u_{1}, u_{2}, u_{3}, \ldots$ is determined by $u_{1}$ and the following recurrence relation for $n \geq 1$ : + +$$ +4 u_{n+1}=\sqrt[3]{64 u_{n}+15} +$$ + +Describe, with proof, the behavior of $u_{n}$ as $n \rightarrow \infty$. +17. (USS) ${ }^{\text {IMO5 }}$ Three equal circles touch the sides of a triangle and have one common point $O$. Show that the center of the circle inscribed in and of the circle circumscribed about the triangle $A B C$ and the point $O$ are collinear. +18. (USS) Several equal spherical planets are given in outer space. On the surface of each planet there is a set of points that is invisible from any of the remaining planets. Prove that the sum of the areas of all these sets is equal to the area of the surface of one planet. +19. (YUG) A finite set of unit circles is given in a plane such that the area of their union $U$ is $S$. Prove that there exists a subset of mutually disjoint circles such that the area of their union is greater that $\frac{2 S}{9}$. + +### 3.23 The Twenty-Third IMO Budapest, Hungary, July 5-14, 1982 + +### 3.23.1 Contest Problems + +First Day (July 9) + +1. The function $f(n)$ is defined for all positive integers $n$ and takes on nonnegative integer values. Also, for all $m, n$, + +$$ +\begin{gathered} +f(m+n)-f(m)-f(n)=0 \quad \text { or } 1 \\ +f(2)=0, \quad f(3)>0, \quad \text { and } \quad f(9999)=3333 +\end{gathered} +$$ + +Determine $f(1982)$. +2. A nonisosceles triangle $A_{1} A_{2} A_{3}$ is given with sides $a_{1}, a_{2}, a_{3}$ ( $a_{i}$ is the side opposite to $A_{i}$ ). For all $i=1,2,3, M_{i}$ is the midpoint of side $a_{i}$, $T_{i}$ is the point where the incircle touches side $a_{i}$, and the reflection of $T_{i}$ in the interior bisector of $A_{i}$ yields the point $S_{i}$. Prove that the lines $M_{1} S_{1}, M_{2} S_{2}$, and $M_{3} S_{3}$ are concurrent. +3. Consider the infinite sequences $\left\{x_{n}\right\}$ of positive real numbers with the following properties: + +$$ +x_{0}=1 \quad \text { and for all } i \geq 0, x_{i+1} \leq x_{i} +$$ + +(a) Prove that for every such sequence there is an $n \geq 1$ such that + +$$ +\frac{x_{0}^{2}}{x_{1}}+\frac{x_{1}^{2}}{x_{2}}+\cdots+\frac{x_{n-1}^{2}}{x_{n}} \geq 3.999 +$$ + +(b) Find such a sequence for which $\frac{x_{0}^{2}}{x_{1}}+\frac{x_{1}^{2}}{x_{2}}+\cdots+\frac{x_{n-1}^{2}}{x_{n}}<4$ for all $n$. + +Second Day (July 10) +4. Prove that if $n$ is a positive integer such that the equation $x^{3}-3 x y^{2}+y^{3}=$ $n$ has a solution in integers $(x, y)$, then it has at least three such solutions. Show that the equation has no solution in integers when $n=2891$. +5. The diagonals $A C$ and $C E$ of the regular hexagon $A B C D E F$ are divided by the inner points $M$ and $N$, respectively, so that $\frac{A M}{A C}=\frac{C N}{C E}=r$. Determine $r$ if $B, M$, and $N$ are collinear. +6. Let $S$ be a square with sides of length 100 and let $L$ be a path within $S$ that does not meet itself and that is composed of linear segments $A_{0} A_{1}, A_{1} A_{2}, \ldots, A_{n-1} A_{n}$ with $A_{0} \neq A_{n}$. Suppose that for every point $P$ of the boundary of $S$ there is a point of $L$ at a distance from $P$ not greater than $\frac{1}{2}$. Prove that there are two points $X$ and $Y$ in $L$ such that the distance between $X$ and $Y$ is not greater than 1 and the length of the part of $L$ that lies between $X$ and $Y$ is not smaller than 198. + +### 3.23.2 Longlisted Problems + +1. (AUS 1) It is well known that the binomial coefficients $\binom{n}{k}=\frac{n!}{k!(n-k)!}$, $0 \leq k \leq n$, are positive integers. The factorial $n$ ! is defined inductively by $0!=1, n!=n \cdot(n-1)$ ! for $n \geq 1$. +(a) Prove that $\frac{1}{n+1}\binom{2 n}{n}$ is an integer for $n \geq 0$. +(b) Given a positive integer $k$, determine the smallest integer $C_{k}$ with the property that $\frac{C_{k}}{n+k+1}\binom{2 n}{n+k}$ is an integer for all $n \geq k$. +2. (AUS 2) Given a finite number of angular regions $A_{1}, \ldots, A_{k}$ in a plane, each $A_{i}$ being bounded by two half-lines meeting at a vertex and provided with a + or - sign, we assign to each point $P$ of the plane and not on a bounding half-line the number $k-l$, where $k$ is the number of + regions and $l$ the number of - regions that contain $P$. (Note that the boundary of $A_{i}$ does not belong to $A_{i}$.) +For instance, in the figure we have two + regions $Q A P$ and $R C Q$, and one - region $R B P$. Every point inside $\triangle A B C$ receives the number +![](https://cdn.mathpix.com/cropped/2024_11_18_8e985d6b9c83aa3e9d0eg-157.jpg?height=208&width=255&top_left_y=806&top_left_x=957) ++1 , while every point not inside $\triangle A B C$ and not on a boundary halfline the number 0 . We say that the interior of $\triangle A B C$ is represented as a sum of the signed angular regions $Q A P, R B P$, and $R C Q$. +(a) Show how to represent the interior of any convex planar polygon as a sum of signed angular regions. +(b) Show how to represent the interior of a tetrahedron as a sum of signed solid angular regions, that is, regions bounded by three planes intersecting at a vertex and provided with a + or - sign. +3. (AUS 3) Given $n$ points $X_{1}, X_{2}, \ldots, X_{n}$ in the interval $0 \leq X_{i} \leq 1$, $i=1,2, \ldots, n$, show that there is a point $y, 0 \leq y \leq 1$, such that + +$$ +\frac{1}{n} \sum_{i=1}^{n}\left|y-X_{i}\right|=\frac{1}{2} +$$ + +4. (AUS 4) (SL82-14). + +Original formulation. Let $A B C D$ be a convex planar quadrilateral and let $A_{1}$ denote the circumcenter of $\triangle B C D$. Define $B_{1}, C_{1}, D_{1}$ in a corresponding way. +(a) Prove that either all of $A_{1}, B_{1}, C_{1}, D_{1}$ coincide in one point, or they are all distinct. Assuming the latter case, show that $A_{1}, C_{1}$ are on opposite sides of the line $B_{1} D_{1}$, and similarly, $B_{1}, D_{1}$ are on opposite sides of the line $A_{1} C_{1}$. (This establishes the convexity of the quadrilateral $\left.A_{1} B_{1} C_{1} D_{1}.\right)$ +(b) Denote by $A_{2}$ the circumcenter of $B_{1} C_{1} D_{1}$, and define $B_{2}, C_{2}, D_{2}$ in an analogous way. Show that the quadrilateral $A_{2} B_{2} C_{2} D_{2}$ is similar to the quadrilateral $A B C D$. +(c) If the quadrilateral $A_{1} B_{1} C_{1} D_{1}$ was obtained from the quadrilateral $A B C D$ by the above process, what condition must be satisfied by the four points $A_{1}, B_{1}, C_{1}, D_{1}$ ? Assuming that the four points $A_{1}, B_{1}, C_{1}, D_{1}$ satisfying this condition are given, describe a construction by straightedge and compass to obtain the original quadrilateral $A B C D$. (It is not necessary to actually perform the construction). +5. (BEL 1) Among all triangles with a given perimeter, find the one with the maximal radius of its incircle. +6. (BEL 2) On the three distinct lines $a, b$, and $c$ three points $A, B$, and $C$ are given, respectively. Construct three collinear points $X, Y, Z$ on lines $a, b, c$, respectively, such that $\frac{B Y}{A X}=2$ and $\frac{C Z}{A X}=3$. +7. (BEL 3) Find all solutions $(x, y) \in \mathbb{Z}^{2}$ of the equation + +$$ +x^{3}-y^{3}=2 x y+8 +$$ + +8. (BRA 1) (SL82-10). +9. (BRA 2) Let $n$ be a natural number, $n \geq 2$, and let $\phi$ be Euler's function; i.e., $\phi(n)$ is the number of positive integers not exceeding $n$ and coprime to $n$. Given any two real numbers $\alpha$ and $\beta, 0 \leq \alpha<\beta \leq 1$, prove that there exists a natural number $m$ such that + +$$ +\alpha<\frac{\phi(m)}{m}<\beta +$$ + +10. (BRA 3) Let $r_{1}, \ldots, r_{n}$ be the radii of $n$ spheres. Call $S_{1}, S_{2}, \ldots, S_{n}$ the areas of the set of points of each sphere from which one cannot see any point of any other sphere. Prove that + +$$ +\frac{S_{1}}{r_{1}^{2}}+\frac{S_{2}}{r_{2}^{2}}+\cdots+\frac{S_{n}}{r_{n}^{2}}=4 \pi +$$ + +11. (BRA 4) A rectangular pool table has a hole at each of three of its corners. The lengths of sides of the table are the real numbers $a$ and $b$. A billiard ball is shot from the fourth corner along its angle bisector. The ball falls in one of the holes. What should the relation between $a$ and $b$ be for this to happen? +12. (BRA 5) Let there be 3399 numbers arbitrarily chosen among the first 6798 integers $1,2, \ldots, 6798$ in such a way that none of them divides another. Prove that there are exactly 1982 numbers in $\{1,2, \ldots, 6798\}$ that must end up being chosen. +13. (BUL 1) A regular $n$-gonal truncated pyramid is circumscribed around a sphere. Denote the areas of the base and the lateral surfaces of the pyramid by $S_{1}, S_{2}$, and $S$, respectively. Let $\sigma$ be the area of the polygon whose vertices are the tangential points of the sphere and the lateral faces of the pyramid. Prove that + +$$ +\sigma S=4 S_{1} S_{2} \cos ^{2} \frac{\pi}{n} +$$ + +14. (BUL 2) (SL82-4). +15. (CAN 1) Show that the set $S$ of natural numbers $n$ for which $3 / n$ cannot be written as the sum of two reciprocals of natural numbers ( $S=$ $\{n \mid 3 / n \neq 1 / p+1 / q$ for any $p, q \in \mathbb{N}\}$ ) is not the union of finitely many arithmetic progressions. +16. (CAN 2) (SL82-7). +17. (CAN 3) (SL82-11). +18. (CAN 4) You are given an algebraic system admitting addition and multiplication for which all the laws of ordinary arithmetic are valid except commutativity of multiplication. Show that + +$$ +\left(a+a b^{-1} a\right)^{-1}+(a+b)^{-1}=a^{-1} +$$ + +where $x^{-1}$ is the element for which $x^{-1} x=x x^{-1}=e$, where $e$ is the element of the system such that for all $a$ the equality $e a=a e=a$ holds. +19. (CAN 5) (SL82-15). +20. (CZS 1) Consider a cube $C$ and two planes $\sigma, \tau$, which divide Euclidean space into several regions. Prove that the interior of at least one of these regions meets at least three faces of the cube. +21. (CZS 2) All edges and all diagonals of regular hexagon $A_{1} A_{2} A_{3} A_{4} A_{5} A_{6}$ are colored blue or red such that each triangle $A_{j} A_{k} A_{m}, \quad 1 \leq j1} u_{n / d, k / d} +$$ + +(the empty sum is defined to be equal to zero). Prove that $n \mid u_{n, k}$ for every natural number $n$ and for every $k(1 \leq k \leq n)$. +39. (POL 2) Let $S$ be the unit circle with center $O$ and let $P_{1}, P_{2}, \ldots, P_{n}$ be points of $S$ such that the sum of vectors $v_{i}=\overrightarrow{O P_{i}}$ is the zero vector. Prove that the inequality $\sum_{i=1}^{n} X P_{i} \geq n$ holds for every point $X$. +40. (POL 3) We consider a game on an infinite chessboard similar to that of solitaire: If two adjacent fields are occupied by pawns and the next field is empty (the three fields lie on a vertical or horizontal line), then we may remove these two pawns and put one of them on the third field. Prove that if in the initial position pawns fill a $3 k \times n$ rectangle, then it is impossible to reach a position with only one pawn on the board. +41. (POL 4) (SL82-8). +42. (POL 5) Let $\mathcal{F}$ be the family of all $k$-element subsets of the set $\{1,2, \ldots, 2 k+1\}$. Prove that there exists a bijective function $f: \mathcal{F} \rightarrow \mathcal{F}$ such that for every $A \in \mathcal{F}$, the sets $A$ and $f(A)$ are disjoint. +43. (TUN 1) (a) What is the maximal number of acute angles in a convex polygon? +(b) Consider $m$ points in the interior of a convex $n$-gon. The $n$-gon is partitioned into triangles whose vertices are among the $n+m$ given points (the vertices of the $n$-gon and the given points). Each of the $m$ points in the interior is a vertex of at least one triangle. Find the number of triangles obtained. +44. (TUN 2) Let $A$ and $B$ be positions of two ships $M$ and $N$, respectively, at the moment when $N$ saw $M$ moving with constant speed $v$ following the line $A x$. In search of help, $N$ moves with speed $k v(k<1)$ along the line $B y$ in order to meet $M$ as soon as possible. Denote by $C$ the point of meeting of the two ships, and set + +$$ +A B=d, \quad \angle B A C=\alpha, \quad 0 \leq \alpha<\frac{\pi}{2} +$$ + +Determine the angle $\angle A B C=\beta$ and time $t$ that $N$ needs in order to meet $M$. +45. (TUN 3) (SL82-20). +46. (USA 1) Prove that if a diagonal is drawn in a quadrilateral inscribed in a circle, the sum of the radii of the circles inscribed in the two triangles thus formed is the same, no matter which diagonal is drawn. +47. (USA 2) Evaluate $\sec ^{\prime \prime} \frac{\pi}{4}+\sec ^{\prime \prime} \frac{3 \pi}{4}+\sec ^{\prime \prime} \frac{5 \pi}{4}+\sec ^{\prime \prime} \frac{7 \pi}{4}$. (Here $\sec ^{\prime \prime}$ means the second derivative of sec.) +48. (USA 3) Given a finite sequence of complex numbers $c_{1}, c_{2}, \ldots, c_{n}$, show that there exists an integer $k(1 \leq k \leq n)$ such that for every finite sequence $a_{1}, a_{2}, \ldots, a_{n}$ of real numbers with $1 \geq a_{1} \geq a_{2} \geq \cdots \geq a_{n} \geq 0$, the following inequality holds: + +$$ +\left|\sum_{m=1}^{n} a_{m} c_{m} n\right| \leq\left|\sum_{m=1}^{n} c_{m}\right| +$$ + +49. (USA 4) Simplify + +$$ +\sum_{k=0}^{n} \frac{(2 n)!}{(k!)^{2}((n-k)!)^{2}} +$$ + +50. (USS 1) Let $O$ be the midpoint of the axis of a right circular cylinder. Let $A$ and $B$ be diametrically opposite points of one base, and $C$ a point of the other base circle that does not belong to the plane $O A B$. Prove that the sum of dihedral angles of the trihedral $O A B C$ is equal to $2 \pi$. +51. (USS 2) Let $n$ numbers $x_{1}, x_{2}, \ldots, x_{n}$ be chosen in such a way that $1 \geq x_{1} \geq x_{2} \geq \cdots \geq x_{n} \geq 0$. Prove that + +$$ +\left(1+x_{1}+x_{2}+\cdots+x_{n}\right)^{\alpha} \leq 1+x_{1}^{\alpha}+2^{\alpha-1} x_{2}^{\alpha}+\cdots+n^{\alpha-1} x_{n}^{\alpha} +$$ + +if $0 \leq \alpha \leq 1$. +52. (USS 3) We are given $2 n$ natural numbers + +$$ +1,1,2,2,3,3, \ldots, n-1, n-1, n, n +$$ + +Find all $n$ for which these numbers can be arranged in a row such that for each $k \leq n$, there are exactly $k$ numbers between the two numbers $k$. +53. (USS 4) (SL82-3). +54. (USS 5) (SL82-17). +55. (VIE 1) (SL82-6). +56. (VIE 2) Let $f(x)=a x^{2}+b x+c$ and $g(x)=c x^{2}+b x+a$. If $|f(0)| \leq 1$, $|f(1)| \leq 1,|f(-1)| \leq 1$, prove that for $|x| \leq 1$, +(a) $|f(x)| \leq 5 / 4$, +(b) $|g(x)| \leq 2$. +57. (YUG 1) (SL82-2). + +### 3.23.3 Shortlisted Problems + +1. A1 (GBR 3) ${ }^{\mathrm{IMO}}$ The function $f(n)$ is defined for all positive integers $n$ and takes on nonnegative integer values. Also, for all $m, n$, + +$$ +\begin{gathered} +f(m+n)-f(m)-f(n)=0 \text { or } 1 \\ +f(2)=0, \quad f(3)>0, \quad \text { and } \quad f(9999)=3333 +\end{gathered} +$$ + +Determine $f(1982)$. +2. A2 (YUG 1) Let $K$ be a convex polygon in the plane and suppose that $K$ is positioned in the coordinate system in such a way that + +$$ +\operatorname{area}\left(K \cap Q_{i}\right)=\frac{1}{4} \text { area } K(i=1,2,3,4,), +$$ + +where the $Q_{i}$ denote the quadrants of the plane. Prove that if $K$ contains no nonzero lattice point, then the area of $K$ is less than 4. +3. A3 (USS 4) ${ }^{\mathrm{IMO} 3}$ Consider the infinite sequences $\left\{x_{n}\right\}$ of positive real numbers with the following properties: + +$$ +x_{0}=1 \quad \text { and for all } i \geq 0, x_{i+1} \leq x_{i} +$$ + +(a) Prove that for every such sequence there is an $n \geq 1$ such that $\frac{x_{0}^{2}}{x_{1}}+$ $\frac{x_{1}^{2}}{x_{2}}+\cdots+\frac{x_{n-1}^{2}}{x_{n}} \geq 3.999$. +(b) Find such a sequence for which $\frac{x_{0}^{2}}{x_{1}}+\frac{x_{1}^{2}}{x_{2}}+\cdots+\frac{x_{n-1}^{2}}{x_{n}}<4$ for all $n$. +4. A4 (BUL 2) Determine all real values of the parameter $a$ for which the equation + +$$ +16 x^{4}-a x^{3}+(2 a+17) x^{2}-a x+16=0 +$$ + +has exactly four distinct real roots that form a geometric progression. +5. A5 (NET 2) ${ }^{\mathrm{IMO}}$ Let $A_{1} A_{2} A_{3} A_{4} A_{5} A_{6}$ be a regular hexagon. Each of its diagonals $A_{i-1} A_{i+1}$ is divided into the same ratio $\frac{\lambda}{1-\lambda}$, where $0<\lambda<1$, by a point $B_{i}$ in such a way that $A_{i}, B_{i}$, and $B_{i+2}$ are collinear ( $i \equiv$ $1, \ldots, 6(\bmod 6))$. Compute $\lambda$. +6. A6 (VIE 1) ${ }^{\text {IMO6 }}$ Let $S$ be a square with sides of length 100 and let $L$ be a path within $S$ that does not meet itself and that is composed of linear segments $A_{0} A_{1}, A_{1} A_{2}, \ldots, A_{n-1} A_{n}$ with $A_{0} \neq A_{n}$. Suppose that for every point $P$ of the boundary of $S$ there is a point of $L$ at a distance from $P$ not greater than $\frac{1}{2}$. Prove that there are two points $X$ and $Y$ in $L$ such that the distance between $X$ and $Y$ is not greater than 1 and the length of that part of $L$ that lies between $X$ and $Y$ is not smaller than 198. +7. B1 (CAN 2) Let $p(x)$ be a cubic polynomial with integer coefficients with leading coefficient 1 and with one of its roots equal to the product of the other two. Show that $2 p(-1)$ is a multiple of $p(1)+p(-1)-2(1+p(0))$. +8. B2 (POL 4) A convex, closed figure lies inside a given circle. The figure is seen from every point of the circumference at a right angle (that is, the two rays drawn from the point and supporting the convex figure are perpendicular). Prove that the center of the circle is a center of symmetry of the figure. +9. B3 (GBR 1) Let $A B C$ be a triangle, and let $P$ be a point inside it such that $\measuredangle P A C=\measuredangle P B C$. The perpendiculars from $P$ to $B C$ and $C A$ meet these lines at $L$ and $M$, respectively, and $D$ is the midpoint of $A B$. Prove that $D L=D M$. +10. B4 (BRA 1) A box contains $p$ white balls and $q$ black balls. Beside the box there is a pile of black balls. Two balls are taken out of the box. If they have the same color, a black ball from the pile is put into the box. If they have different colors, the white ball is put back into the box. This procedure is repeated until the last two balls are removed from the box and one last ball is put in. What is the probability that this last ball is white? +11. B5 (CAN 3) (a) Find the rearrangement $\left\{a_{1}, \ldots, a_{n}\right\}$ of $\{1,2, \ldots, n\}$ that maximizes + +$$ +a_{1} a_{2}+a_{2} a_{3}+\cdots+a_{n} a_{1}=Q +$$ + +(b) Find the rearrangement that minimizes $Q$. +12. B6 (FIN 3) Four distinct circles $C, C_{1}, C_{2}, C_{3}$ and a line $L$ are given in the plane such that $C$ and $L$ are disjoint and each of the circles $C_{1}, C_{2}, C_{3}$ touches the other two, as well as $C$ and $L$. Assuming the radius of $C$ to be 1 , determine the distance between its center and $L$. +13. C1 (NET 1) ${ }^{\mathrm{IMO} 2} \mathrm{~A}$ scalene triangle $A_{1} A_{2} A_{3}$ is given with sides $a_{1}, a_{2}, a_{3}$ ( $a_{i}$ is the side opposite to $A_{i}$ ). For all $i=1,2,3, M_{i}$ is the midpoint of side $a_{i}, T_{i}$ is the point where the incircle touches side $a_{i}$, and the reflection of $T_{i}$ in the interior bisector of $A_{i}$ yields the point $S_{i}$. Prove that the lines $M_{1} S_{1}, M_{2} S_{2}$, and $M_{3} S_{3}$ are concurrent. +14. C2 (AUS 4) Let $A B C D$ be a convex plane quadrilateral and let $A_{1}$ denote the circumcenter of $\triangle B C D$. Define $B_{1}, C_{1}, D_{1}$ in a corresponding way. +(a) Prove that either all of $A_{1}, B_{1}, C_{1}, D_{1}$ coincide in one point, or they are all distinct. Assuming the latter case, show that $A_{1}, C_{1}$ are on opposite sides of the line $B_{1} D_{1}$, and similarly, $B_{1}, D_{1}$ are on opposite sides of the line $A_{1} C_{1}$. (This establishes the convexity of the quadrilateral $A_{1} B_{1} C_{1} D_{1}$.) +(b) Denote by $A_{2}$ the circumcenter of $B_{1} C_{1} D_{1}$, and define $B_{2}, C_{2}, D_{2}$ in an analogous way. Show that the quadrilateral $A_{2} B_{2} C_{2} D_{2}$ is similar to the quadrilateral $A B C D$. +15. C3 (CAN 5) Show that + +$$ +\frac{1-s^{a}}{1-s} \leq(1+s)^{a-1} +$$ + +holds for every $1 \neq s>0$ real and $00$, and + +$$ +f(x)=a_{0}+\sum_{j=0}^{\infty} b\left(a_{0}\right) \cdots b\left(a_{j}\right) a_{j+1} +$$ + +show that $0a_{i+1}$ for $i$ running from 1 up to $n-1$ ? +26. (FRG 2) Let $a, b, c$ be positive integers satisfying $(a, b)=(b, c)=(c, a)=$ 1. Show that $2 a b c-a b-b c-c a$ cannot be represented as $b c x+c a y+a b z$ with nonnegative integers $x, y, z$. +27. (FRG 3) (SL83-18). +28. (GBR 1) Show that if the sides $a, b, c$ of a triangle satisfy the equation + +$$ +2\left(a b^{2}+b c^{2}+c a^{2}\right)=a^{2} b+b^{2} c+c^{2} a+3 a b c +$$ + +then the triangle is equilateral. Show also that the equation can be satisfied by positive real numbers that are not the sides of a triangle. +29. (GBR 2) Let $O$ be a point outside a given circle. Two lines $O A B, O C D$ through $O$ meet the circle at $A, B, C, D$, where $A, C$ are the midpoints of $O B, O D$, respectively. Additionally, the acute angle $\theta$ between the lines is equal to the acute angle at which each line cuts the circle. Find $\cos \theta$ and show that the tangents at $A, D$ to the circle meet on the line $B C$. +30. (GBR 3) Prove the existence of a unique sequence $\left\{u_{n}\right\}(n=0,1,2 \ldots)$ of positive integers such that + +$$ +u_{n}^{2}=\sum_{r=0}^{n}\binom{n+r}{r} u_{n-r} \quad \text { for all } n \geq 0 +$$ + +where $\binom{m}{r}$ is the usual binomial coefficient. +31. (GBR 4) (SL83-12). +32. (GBR 5) Let $a, b, c$ be positive real numbers and let $[x]$ denote the greatest integer that does not exceed the real number $x$. Suppose that $f$ is a function defined on the set of nonnegative integers $n$ and taking real values such that $f(0)=0$ and + +$$ +f(n) \leq a n+f([b n])+f([c n]), \quad \text { for all } n \geq 1 +$$ + +Prove that if $b+c<1$, there is a real number $k$ such that + +$$ +f(n) \leq k n \quad \text { for all } n \text {, } +$$ + +while if $b+c=1$, there is a real number $K$ such that $f(n) \leq K n \log _{2} n$ for all $n \geq 2$. Show that if $b+c=1$, there may not be a real number $k$ that satisfies (1). +33. (GDR 1) (SL83-16). +34. (GDR 2) In a plane are given $n$ points $P_{i}(i=1,2, \ldots, n)$ and two angles $\alpha$ and $\beta$. Over each of the segments $P_{i} P_{i=1}\left(P_{n+1}=P_{1}\right)$ a point $Q_{i}$ is constructed such that for all $i$ : +(i) upon moving from $P_{i}$ to $P_{i+1}, Q_{i}$ is seen on the same side of $P_{i} P_{i+1}$, +(ii) $\angle P_{i+1} P_{i} Q_{i}=\alpha$, +(iii) $\angle P_{i} P_{i+1} Q_{i}=\beta$. + +Furthermore, let $g$ be a line in the same plane with the property that all the points $P_{i}, Q_{i}$ lie on the same side of $g$. Prove that + +$$ +\sum_{i=1}^{n} d\left(P_{i}, g\right)=\sum_{i=1}^{n} d\left(Q_{i}, g\right) +$$ + +where $d(M, g)$ denotes the distance from point $M$ to line $g$. +35. (GDR 3) (SL83-17). +36. (ISR 1) The set $X$ has 1983 members. There exists a family of subsets $\left\{S_{1}, S_{2}, \ldots, S_{k}\right\}$ such that: +(i) the union of any three of these subsets is the entire set $X$, while +(ii) the union of any two of them contains at most 1979 members. + +What is the largest possible value of $k$ ? +37. (ISR 2) The points $A_{1}, A_{2}, \ldots, A_{1983}$ are set on the circumference of a circle and each is given one of the values $\pm 1$. Show that if the number of points with the value +1 is greater than 1789 , then at least 1207 of the points will have the property that the partial sums that can be formed by taking the numbers from them to any other point, in either direction, are strictly positive. +38. (KUW 1) Let $\left\{u_{n}\right\}$ be the sequence defined by its first two terms $u_{0}, u_{1}$ and the recursion formula + +$$ +u_{n+2}=u_{n}-u_{n+1} +$$ + +(a) Show that $u_{n}$ can be written in the form $u_{n}=\alpha a^{n}+\beta b^{n}$, where $a, b, \alpha, \beta$ are constants independent of $n$ that have to be determined. +(b) If $S_{n}=u_{0}+u_{1}+\cdots+u_{n}$, prove that $S_{n}+u_{n-1}$ is a constant independent of $n$. Determine this constant. +39. (KUW 2) If $\alpha$ is the real root of the equation + +$$ +E(x)=x^{3}-5 x-50=0 +$$ + +such that $x_{n+1}=\left(5 x_{n}+50\right)^{1 / 3}$ and $x_{1}=5$, where $n$ is a positive integer, prove that: +(a) $x_{n+1}^{3}-\alpha^{3}=5\left(x_{n}-\alpha\right)$ +(b) $\alphaj \geq 1} \cos ^{2}\left(x_{i}-x_{j}\right) \geq \frac{n(n-2)}{4} +$$ + +73. (VIE 1) Let $A B C$ be a nonequilateral triangle. Prove that there exist two points $P$ and $Q$ in the plane of the triangle, one in the interior and one in the exterior of the circumcircle of $A B C$, such that the orthogonal projections of any of these two points on the sides of the triangle are vertices of an equilateral triangle. +74. (VIE 2) In a plane we are given two distinct points $A, B$ and two lines $a, b$ passing through $B$ and $A$ respectively $(~ a \ni B, b \ni A$ ) such that the line $A B$ is equally inclined to $a$ and $b$. Find the locus of points $M$ in the plane such that the product of distances from $M$ to $A$ and $a$ equals the +product of distances from $M$ to $B$ and $b$ (i.e., $M A \cdot M A^{\prime}=M B \cdot M B^{\prime}$, where $A^{\prime}$ and $B^{\prime}$ are the feet of the perpendiculars from $M$ to $a$ and $b$ respectively). +75. (VIE 3) Find the sum of the fiftieth powers of all sides and diagonals of a regular 100-gon inscribed in a circle of radius $R$. + +### 3.24.3 Shortlisted Problems + +1. (AUS 1) The localities $P_{1}, P_{2}, \ldots, P_{1983}$ are served by ten international airlines $A_{1}, A_{2}, \ldots, A_{10}$. It is noticed that there is direct service (without stops) between any two of these localities and that all airline schedules offer round-trip flights. Prove that at least one of the airlines can offer a round trip with an odd number of landings. +2. (BEL 1) Let $n$ be a positive integer. Let $\sigma(n)$ be the sum of the natural divisors $d$ of $n$ (including 1 and $n$ ). We say that an integer $m \geq 1$ is superabundant (P.Erdös, 1944) if $\forall k \in\{1,2, \ldots, m-1\}, \frac{\sigma(m)}{m}>\frac{\sigma(k)}{k}$. Prove that there exists an infinity of superabundant numbers. +3. (BEL 3) $)^{\mathrm{IMO} 4}$ We say that a set $E$ of points of the Euclidian plane is "Pythagorean" if for any partition of $E$ into two sets $A$ and $B$, at least one of the sets contains the vertices of a right-angled triangle. Decide whether the following sets are Pythagorean: +(a) a circle; +(b) an equilateral triangle (that is, the set of three vertices and the points of the three edges). +4. (BEL 5) On the sides of the triangle $A B C$, three similar isosceles triangles $A B P(A P=P B), A Q C(A Q=Q C)$, and $B R C(B R=R C)$ are constructed. The first two are constructed externally to the triangle $A B C$, but the third is placed in the same half-plane determined by the line $B C$ as the triangle $A B C$. Prove that $A P R Q$ is a parallelogram. +5. (BRA 1) Consider the set of all strictly decreasing sequences of $n$ natural numbers having the property that in each sequence no term divides any other term of the sequence. Let $A=\left(a_{j}\right)$ and $B=\left(b_{j}\right)$ be any two such sequences. We say that $A$ precedes $B$ if for some $k, a_{k}0$. Show that $01$ there are $a, b \in M$ such that $a+b=m$. +(ii) If $a, b, c, d \in M, a, b, c, d>10$ and $a+b=c+d$, then $a=c$ or $a=d$. +16. (GDR 1) Let $F(n)$ be the set of polynomials $P(x)=a_{0}+a_{1} x+\cdots+a_{n} x^{n}$, with $a_{0}, a_{1}, \ldots, a_{n} \in \mathbb{R}$ and $0 \leq a_{0}=a_{n} \leq a_{1}=a_{n-1} \leq \cdots \leq a_{[n / 2]}=$ $a_{[(n+1) / 2]}$. Prove that if $f \in F(m)$ and $g \in F(n)$, then $f g \in F(m+n)$. +17. (GDR 3) Let $P_{1}, P_{2}, \ldots, P_{n}$ be distinct points of the plane, $n \geq 2$. Prove that + +$$ +\max _{1 \leq i\frac{\sqrt{3}}{2}(\sqrt{n}-1) \min _{1 \leq i0$. +21. (SWE 1) Find the greatest integer less than or equal to $\sum_{k=1}^{2^{1983}} k^{1 / 1983-1}$. +22. (SWE 4) Let $n$ be a positive integer having at least two different prime factors. Show that there exists a permutation $a_{1}, a_{2}, \ldots, a_{n}$ of the integers $1,2, \ldots, n$ such that + +$$ +\sum_{k=1}^{n} k \cdot \cos \frac{2 \pi a_{k}}{n}=0 +$$ + +23. (USS 1) ${ }^{\mathrm{IMO} 2}$ Let $K$ be one of the two intersection points of the circles $W_{1}$ and $W_{2}$. Let $O_{1}$ and $O_{2}$ be the centers of $W_{1}$ and $W_{2}$. The two common tangents to the circles meet $W_{1}$ and $W_{2}$ respectively in $P_{1}$ and $P_{2}$, the first tangent, and $Q_{1}$ and $Q_{2}$, the second tangent. Let $M_{1}$ and $M_{2}$ be the midpoints of $P_{1} Q_{1}$ and $P_{2} Q_{2}$, respectively. Prove that + +$$ +\angle O_{1} K O_{2}=\angle M_{1} K M_{2} +$$ + +24. (USS 2) Let $d_{n}$ be the last nonzero digit of the decimal representation of $n$ !. Prove that $d_{n}$ is aperiodic; that is, there do not exist $T$ and $n_{0}$ such that for all $n \geq n_{0}, d_{n+T}=d_{n}$. +25. (USS 3) Prove that every partition of 3-dimensional space into three disjoint subsets has the following property: One of these subsets contains all possible distances; i.e., for every $a \in \mathbb{R}_{+}$, there are points $M$ and $N$ inside that subset such that distance between $M$ and $N$ is exactly $a$. + +### 3.25 The Twenty-Fifth IMO Prague, Czechoslovakia, June 29-July 10, 1984 + +### 3.25.1 Contest Problems + +First Day (July 4) + +1. Let $x, y, z$ be nonnegative real numbers with $x+y+z=1$. Show that + +$$ +0 \leq x y+y z+z x-2 x y z \leq \frac{7}{27} +$$ + +2. Find two positive integers $a, b$ such that none of the numbers $a, b, a+b$ is divisible by 7 and $(a+b)^{7}-a^{7}-b^{7}$ is divisible by $7^{7}$. +3. In a plane two different points $O$ and $A$ are given. For each point $X \neq O$ of the plane denote by $\alpha(X)$ the angle $A O X$ measured in radians $(0 \leq$ $\alpha(X)<2 \pi)$ and by $C(X)$ the circle with center $O$ and radius $O X+\frac{\alpha(X)}{O X}$. Suppose each point of the plane is colored by one of a finite number of colors. Show that there exists a point $X$ with $\alpha(X)>0$ such that its color appears somewhere on the circle $C(X)$. + +Second Day (July 5) +4. Let $A B C D$ be a convex quadrilateral for which the circle of diameter $A B$ is tangent to the line $C D$. Show that the circle of diameter $C D$ is tangent to the line $A B$ if and only if the lines $B C$ and $A D$ are parallel. +5. Let $d$ be the sum of the lengths of all diagonals of a convex polygon of $n$ $(n>3)$ vertices, and let $p$ be its perimeter. Prove that + +$$ +\frac{n-3}{2}<\frac{d}{p}<\frac{1}{2}\left(\left[\frac{n}{2}\right]\left[\frac{n+1}{2}\right]-2\right) +$$ + +6. Let $a, b, c, d$ be odd positive integers such that $ax_{j}$. Let $d(n, k)$ be the number of such permutations with exactly $k$ discordant pairs. +(a) Find $d(n, 2)$. +(b) Show that + +$$ +d(n, k)=d(n, k-1)+d(n-1, k)-d(n-1, k-1) +$$ + +with $d(n, k)=0$ for $k<0$ and $d(n, 0)=1$ for $n \geq 1$. Compute with this recursion a table of $d(n, k)$ for $n=1$ to 6 . +23. (FRG 4) A $2 \times 2 \times 12$ box fixed in space is to be filled with twenty-four $1 \times 1 \times 2$ bricks. In how many ways can this be done? +24. (FRG 5) (SL84-7). + +Original formulation. Consider several types of 4-cell figures: +(a) +![](https://cdn.mathpix.com/cropped/2024_11_18_8e985d6b9c83aa3e9d0eg-181.jpg?height=102&width=107&top_left_y=1372&top_left_x=317) +(b) +![](https://cdn.mathpix.com/cropped/2024_11_18_8e985d6b9c83aa3e9d0eg-181.jpg?height=62&width=173&top_left_y=1388&top_left_x=505) +(c) +![](https://cdn.mathpix.com/cropped/2024_11_18_8e985d6b9c83aa3e9d0eg-181.jpg?height=98&width=146&top_left_y=1374&top_left_x=749) +(d) +![](https://cdn.mathpix.com/cropped/2024_11_18_8e985d6b9c83aa3e9d0eg-181.jpg?height=98&width=141&top_left_y=1374&top_left_x=969) +(e) +![](https://cdn.mathpix.com/cropped/2024_11_18_8e985d6b9c83aa3e9d0eg-181.jpg?height=102&width=159&top_left_y=1372&top_left_x=1176) + +Find, with proof, for which of these types of figures it is not possible to number the fields of the $8 \times 8$ chessboard using the numbers $1,2, \ldots, 64$ in such a way that the sum of the four numbers in each of its parts congruent to the given figure is divisible by 4. +25. (GBR 1) (SL84-10). +26. (GBR 2) A cylindrical container has height 6 cm and radius 4 cm . It rests on a circular hoop, also of radius 4 cm , fixed in a horizontal plane with its axis vertical and with each circular rim of the cylinder touching the hoop at two points. +The cylinder is now moved so that each of its circular rims still touches the hoop in two points. Find with proof the locus of one of the cylinder's vertical ends. +27. (GBR 3) The function $f(n)$ is defined on the nonnegative integers $n$ by: $f(0)=0, f(1)=1$, + +$$ +f(n)=f\left(n-\frac{1}{2} m(m-1)\right)-f\left(\frac{1}{2} m(m+1)-n\right), +$$ + +for $\frac{1}{2} m(m-1)\beta>\gamma>0$. From the equation $f_{1}(x)=0$ one constructs the equation $f_{2}(x)=x^{3}+a_{2} x^{2}+b_{2} x+c_{2}=x\left(x+b_{1}\right)^{2}-\left(a_{1} x+c_{1}\right)^{2}=0$. Continuing this process, we get equations $f_{3}, \ldots, f_{n}$. Prove that + +$$ +\lim _{n \rightarrow \infty} \sqrt[2^{n-1}]{-a_{n}}=\alpha +$$ + +32. (LUX 2) (SL84-15). +33. (MON 1) (SL84-4). +34. (MON 2) One country has $n$ cities and every two of them are linked by a railroad. A railway worker should travel by train exactly once through the entire railroad system (reaching each city exactly once). If it is impossible for worker to travel by train between two cities, he can travel by plane. What is the minimal number of flights that the worker will have to use? +35. (MON 3) Prove that there exist distinct natural numbers $m_{1}, m_{2}, \ldots$, $m_{k}$ satisfying the conditions + +$$ +\pi^{-1984}<25-\left(\frac{1}{m_{1}}+\frac{1}{m_{2}}+\cdots+\frac{1}{m_{k}}\right)<\pi^{-1960} +$$ + +where $\pi$ is the ratio between circle and its diameter. +36. (MON 4) The set $\{1,2, \ldots, 49\}$ is divided into three subsets. Prove that at least one of these subsets contains three different numbers $a, b, c$ such that $a+b=c$. +37. (MOR 1) Denote by $[x]$ the greatest integer not exceeding $x$. For all real $k>1$, define two sequences: + +$$ +a_{n}(k)=[n k] \quad \text { and } \quad b_{n}(k)=\left[\frac{n k}{k-1}\right] . +$$ + +If $A(k)=\left\{a_{n}(k): n \in \mathbb{N}\right\}$ and $B(k)=\left\{b_{n}(k): n \in \mathbb{N}\right\}$, prove that $A(k)$ and $B(k)$ form a partition of $\mathbb{N}$ if and only if $k$ is irrational. +38. (MOR 2) Determine all continuous functions $f$ such that + +$$ +\left(\forall(x, y) \in \mathbb{R}^{2}\right) \quad f(x+y) f(x-y)=(f(x) f(y))^{2} . +$$ + +39. (MOR 3) Let $A B C$ be an isosceles triangle, $A B=A C, \angle A=20^{\circ}$. Let $D$ be a point on $A B$, and $E$ a point on $A C$ such that $\angle A C D=20^{\circ}$ and $\angle A B E=30^{\circ}$. What is the measure of the angle $\angle C D E$ ? +40. (NET 1) (SL84-12). +41. (NET 2) Determine positive integers $p, q$, and $r$ such that the diagonal of a block consisting of $p \times q \times r$ unit cubes passes through exactly 1984 of the unit cubes, while its length is minimal. (The diagonal is said to pass through a unit cube if it has more than one point in common with the unit cube.) +42. (NET 3) Triangle $A B C$ is given for which $B C=A C+\frac{1}{2} A B$. The point $P$ divides $A B$ such that $R P: P A=1: 3$. Prove that $\angle C A P=2 \angle C P A$. +43. (POL 1) (SL84-16). +44. (POL 2) (SL84-9). +45. (POL 3) Let $X$ be an arbitrary nonempty set contained in the plane and let sets $A_{1}, A_{2}, \ldots, A_{m}$ and $B_{1}, B_{2}, \ldots, B_{n}$ be its images under parallel translations. Let us suppose that + +$$ +A_{1} \cup A_{2} \cup \cdots \cup A_{m} \subset B_{1} \cup B_{2} \cup \cdots \cup B_{n} +$$ + +and that the sets $A_{1}, A_{2}, \ldots, A_{m}$ are disjoint. Prove that $m \leq n$. +46. (ROM 1) Let $\left(a_{n}\right)_{n \geq 1}$ and $\left(b_{n}\right)_{n \geq 1}$ be two sequences of natural numbers such that $a_{n+1}=n a_{n}+1, b_{n+1}=n b_{n}-1$ for every $n \geq 1$. Show that these two sequences can have only a finite number of terms in common. +47. (ROM 2) (SL84-8). +48. (ROM 3) Let $A B C$ be a triangle with interior angle bisectors $A A_{1}$, $B B_{1}, C C_{1}$ and incenter $I$. If $\sigma\left[I A_{1} B\right]+\sigma\left[I B_{1} C\right]+\sigma\left[I C_{1} A\right]=\frac{1}{2} \sigma[A B C]$, where $\sigma[A B C]$ denotes the area of $A B C$, show that $A B C$ is isosceles. +49. (ROM 4) Let $n>1$ and $x_{i} \in \mathbb{R}$ for $i=1, \ldots, n$. Set $S_{k}=x_{1}^{k}+x_{2}^{k}+$ $\cdots+x_{n}^{k}$ for $k \geq 1$. If $S_{1}=S_{2}=\cdots=S_{n+1}$, show that $x_{i} \in\{0,1\}$ for every $i=1,2, \ldots, n$. +50. (ROM 5) (SL84-14). +51. (SPA 1) Two cyclists leave simultaneously a point $P$ in a circular runway with constant velocities $v_{1}, v_{2}\left(v_{1}>v_{2}\right)$ and in the same sense. A pedestrian leaves $P$ at the same time, moving with velocity $v_{3}=\frac{v_{1}+v_{2}}{12}$. If the pedestrian and the cyclists move in opposite directions, the pedestrian meets the second cyclist 91 seconds after he meets the first. If the pedestrian moves in the same direction as the cyclists, the first cyclist overtakes him 187 seconds before the second does. Find the point where the first cyclist overtakes the second cyclist the first time. +52. (SPA 2) Construct a scalene triangle such that + +$$ +a(\tan B-\tan C)=b(\tan A-\tan C) +$$ + +53. (SPA 3) Find a sequence of natural numbers $a_{i}$ such that $a_{i}=\sum_{r=1}^{i+4} d_{r}$, where $d_{r} \neq d_{s}$ for $r \neq s$ and $d_{r}$ divides $a_{i}$. +54. (SPA 4) Let $P$ be a convex planar polygon with equal angles. Let $l_{1}, \ldots, l_{n}$ be its sides. Show that a necessary and sufficient condition for $P$ to be regular is that the sum of the ratios $\frac{l_{i}}{l_{i+1}}\left(i=1, \ldots, n ; l_{n+1}=l_{1}\right)$ equals the number of sides. +55. (SPA 5) Let $a, b, c$ be natural numbers such that $a+b+c=2 p q\left(p^{30}+q^{30}\right)$, $p>q$ being two given positive integers. +(a) Prove that $k=a^{3}+b^{3}+c^{3}$ is not a prime number. +(b) Prove that if $a \cdot b \cdot c$ is maximum, then 1984 divides $k$. +56. (SWE 1) Let $a, b, c$ be nonnegative integers such that $a \leq b \leq c, 2 b \neq$ $a+c$ and $\frac{a+b+c}{3}$ is an integer. Is it possible to find three nonnegative integers $d, e$, and $f$ such that $d \leq e \leq f, f \neq c$, and such that $a^{2}+b^{2}+c^{2}=$ $d^{2}+e^{2}+f^{2} ?$ +57. (SWE 2) Let $a, b, c, d$ be a permutation of the numbers $1,9,8,4$ and let $n=(10 a+b)^{10 c+d}$. Find the probability that $1984!$ is divisible by $n$. +58. (SWE 3) Let $\left(a_{n}\right)_{1}^{\infty}$ be a sequence such that $a_{n} \leq a_{n+m} \leq a_{n}+a_{m}$ for all positive integers $n$ and $m$. Prove that $\frac{a_{n}}{n}$ has a limit as $n$ approaches infinity. +59. (USA 1) Determine the smallest positive integer $m$ such that $529^{n}+m$. $132^{n}$ is divisible by 262417 for all odd positive integers $n$. +60. (USA 2) (SL84-20). +61. (USA 3) A fair coin is tossed repeatedly until there is a run of an odd number of heads followed by a tail. Determine the expected number of tosses. +62. (USA 4) From a point $P$ exterior to a circle $K$, two rays are drawn intersecting $K$ in the respective pairs of points $A, A^{\prime}$ and $B, B^{\prime}$. For any other pair of points $C, C^{\prime}$ on $K$, let $D$ be the point of intersection of the circumcircles of triangles $P A C$ and $P B^{\prime} C^{\prime}$ other than point $P$. Similarly, let $D^{\prime}$ be the point of intersection of the circumcircles of triangles $P A^{\prime} C^{\prime}$ and $P B C$ other than point $P$. Prove that the points $P, D$, and $D^{\prime}$ are collinear. +63. (USA 5) (SL84-18). +64. (USS 1) For a matrix $\left(p_{i j}\right)$ of the format $m \times n$ with real entries, set + +$$ +a_{i}=\sum_{j=1}^{n} p_{i j} \text { for } i=1, \ldots, m \text { and } \quad b_{j}=\sum_{i=1}^{m} p_{i j} \text { for } j=1, \ldots, n +$$ + +By integering a real number we mean replacing the number with the integer closest to it. +Prove that integering the numbers $a_{i}, b_{j}, p_{i j}$ can be done in such a way that (1) still holds. +65. (USS 2) A tetrahedron is inscribed in a sphere of radius 1 such that the center of the sphere is inside the tetrahedron. +Prove that the sum of lengths of all edges of the tetrahedron is greater than 6. +66. (USS 3) (SL84-3). + +Original formulation. All the divisors of a positive integer $n$ arranged in increasing order are $x_{1}\frac{5}{6} R +$$ + +68. (USS 5) In the Martian language every finite sequence of letters of the Latin alphabet letters is a word. The publisher "Martian Words" makes a collection of all words in many volumes. In the first volume there are only one-letter words, in the second, two-letter words, etc., and the numeration of the words in each of the volumes continues the numeration of the previous volume. Find the word whose numeration is equal to the sum of numerations of the words Prague, Olympiad, Mathematics. + +### 3.25.3 Shortlisted Problems + +1. (FRA 1) Find all solutions of the following system of $n$ equations in $n$ variables: + +$$ +\begin{aligned} +& x_{1}\left|x_{1}\right|-\left(x_{1}-a\right)\left|x_{1}-a\right|=x_{2}\left|x_{2}\right| \\ +& x_{2}\left|x_{2}\right|-\left(x_{2}-a\right)\left|x_{2}-a\right|=x_{3}\left|x_{3}\right| \\ +& \cdots \\ +& x_{n}\left|x_{n}\right|-\left(x_{n}-a\right)\left|x_{n}-a\right|=x_{1}\left|x_{1}\right| +\end{aligned} +$$ + +where $a$ is a given number. +2. (CAN 2) Prove: +(a) There are infinitely many triples of positive integers $m, n, p$ such that $4 m n-m-n=p^{2}-1$. +(b) There are no positive integers $m, n, p$ such that $4 m n-m-n=p^{2}$. +3. (USS 3) Find all positive integers $n$ such that + +$$ +n=d_{6}^{2}+d_{7}^{2}-1 +$$ + +where $1=d_{1}3)$ vertices and let $p$ be its perimeter. Prove that + +$$ +\frac{n-3}{2}<\frac{d}{p}<\frac{1}{2}\left(\left[\frac{n}{2}\right]\left[\frac{n+1}{2}\right]-2\right) . +$$ + +5. (FRG 1) ${ }^{\mathrm{IMO} 1}$ Let $x, y, z$ be nonnegative real numbers with $x+y+z=1$. Show that + +$$ +0 \leq x y+y z+z x-2 x y z \leq \frac{7}{27} +$$ + +6. (CAN 3) Let $c$ be a positive integer. The sequence $\left\{f_{n}\right\}$ is defined as follows: + +$$ +f_{1}=1, \quad f_{2}=c, \quad f_{n+1}=2 f_{n}-f_{n-1}+2 \quad(n \geq 2) +$$ + +Show that for each $k \in \mathbb{N}$ there exists $r \in \mathbb{N}$ such that $f_{k} f_{k+1}=f_{r}$. +7. (FRG 5) +(a) Decide whether the fields of the $8 \times 8$ chessboard can be numbered by the numbers $1,2, \ldots, 64$ in such a way that the sum of the four numbers in each of its parts of one of the forms +![](https://cdn.mathpix.com/cropped/2024_11_18_8e985d6b9c83aa3e9d0eg-186.jpg?height=171&width=587&top_left_y=1708&top_left_x=531) +is divisible by four. +(b) Solve the analogous problem for +![](https://cdn.mathpix.com/cropped/2024_11_18_8e985d6b9c83aa3e9d0eg-186.jpg?height=159&width=575&top_left_y=1974&top_left_x=539) +8. (ROM 2) $)^{\mathrm{IMO} 3}$ In a plane two different points $O$ and $A$ are given. For each point $X \neq O$ of the plane denote by $\alpha(X)$ the angle $A O X$ measured in radians $(0 \leq \alpha(X)<2 \pi)$ and by $C(X)$ the circle with center $O$ and radius $O X+\frac{\alpha(X)}{O X}$. Suppose each point of the plane is colored by one of a finite number of colors. Show that there exists a point $X$ with $\alpha(X)>0$ such that its color appears somewhere on the circle $C(X)$. +9. (POL 2) Let $a, b, c$ be positive numbers with $\sqrt{a}+\sqrt{b}+\sqrt{c}=\frac{\sqrt{3}}{2}$. Prove that the system of equations + +$$ +\begin{aligned} +& \sqrt{y-a}+\sqrt{z-a}=1, \\ +& \sqrt{z-b}+\sqrt{x-b}=1 \\ +& \sqrt{x-c}+\sqrt{y-c}=1 +\end{aligned} +$$ + +has exactly one solution $(x, y, z)$ in real numbers. +10. (GBR 1) Prove that the product of five consecutive positive integers cannot be the square of an integer. +11. (CAN 1) Let $n$ be a natural number and $a_{1}, a_{2}, \ldots, a_{2 n}$ mutually distinct integers. Find all integers $x$ satisfying + +$$ +\left(x-a_{1}\right) \cdot\left(x-a_{2}\right) \cdots\left(x-a_{2 n}\right)=(-1)^{n}(n!)^{2} +$$ + +12. (NET 1) ${ }^{\mathrm{IMO} 2}$ Find two positive integers $a, b$ such that none of the numbers $a, b, a+b$ is divisible by 7 and $(a+b)^{7}-a^{7}-b^{7}$ is divisible by $7^{7}$. +13. (BUL 5) Prove that the volume of a tetrahedron inscribed in a right circular cylinder of volume 1 does not exceed $\frac{2}{3 \pi}$. +14. (ROM 5) ${ }^{\mathrm{IMO} 4}$ Let $A B C D$ be a convex quadrilateral for which the circle with diameter $A B$ is tangent to the line $C D$. Show that the circle with diameter $C D$ is tangent to the line $A B$ if and only if the lines $B C$ and $A D$ are parallel. +15. (LUX 2) Angles of a given triangle $A B C$ are all smaller than $120^{\circ}$. Equilateral triangles $A F B, B D C$ and $C E A$ are constructed in the exterior of $\triangle A B C$. +(a) Prove that the lines $A D, B E$, and $C F$ pass through one point $S$. +(b) Prove that $S D+S E+S F=2(S A+S B+S C)$. +16. (POL 1) ${ }^{\mathrm{IMO} 6}$ Let $a, b, c, d$ be odd positive integers such that $ax_{j}$. Let $d(n, k)$ be the number of such permutations with exactly $k$ discordant pairs. Find $d(n, 2)$ and $d(n, 3)$. +18. (USA 5) Inside triangle $A B C$ there are three circles $k_{1}, k_{2}, k_{3}$ each of which is tangent to two sides of the triangle and to its incircle $k$. The radii of $k_{1}, k_{2}, k_{3}$ are 1, 4, and 9 . Determine the radius of $k$. +19. (CAN 5) The triangular array $\left(a_{n, k}\right)$ of numbers is given by $a_{n, 1}=1 / n$, for $n=1,2, \ldots, a_{n, k+1}=a_{n-1, k}-a_{n, k}$, for $1 \leq k \leq n-1$. Find the harmonic mean of the 1985th row. +20. (USA 2) Determine all pairs $(a, b)$ of positive real numbers with $a \neq 1$ such that + +$$ +\log _{a} b<\log _{a+1}(b+1) +$$ + +### 3.26 The Twenty-Sixth IMO + +## Joutsa, Finland, June 29-July 11, 1985 + +### 3.26.1 Contest Problems + +First Day (July 4) + +1. A circle whose center is on the side $E D$ of the cyclic quadrilateral $B C D E$ touches the other three sides. Prove that $E B+C D=E D$. +2. Each of the numbers in the set $N=\{1,2,3, \ldots, n-1\}$, where $n \geq 3$, is colored with one of two colors, say red or black, so that: +(i) $i$ and $n-i$ always receive the same color, and +(ii) for some $j \in N$ relatively prime to $n, i$ and $|j-i|$ receive the same color for all $i \in N, i \neq j$. +Prove that all numbers in $N$ must receive the same color. +3. The weight $w(p)$ of a polynomial $p, p(x)=\sum_{i=0}^{n} a_{i} x^{i}$, with integer coefficients $a_{i}$ is defined as the number of its odd coefficients. For $i=0,1,2, \ldots$, let $q_{i}(x)=(1+x)^{i}$. Prove that for any finite sequence $0 \leq i_{1}0$ recursively by + +$$ +f_{1}(x)=x, \quad f_{n+1}(x)=f_{n}(x)\left(f_{n}(x)+\frac{1}{n}\right) . +$$ + +Prove that there exists one and only one positive number $a$ such that $01, j>1$, and $(k, j)=m$, then $f(k j)=f(m)(f(k / m)+f(j / m))$. What values can $f(1984)$ and $f(1985)$ take? +4. (BEL 1) Let $x, y$, and $z$ be real numbers satisfying $x+y+z=x y z$. Prove that + +$$ +x\left(1-y^{2}\right)\left(1-z^{2}\right)+y\left(1-z^{2}\right)\left(1-x^{2}\right)+z\left(1-x^{2}\right)\left(1-y^{2}\right)=4 x y z +$$ + +5. (BEL 2) (SL85-16). +6. (BEL 3) On a one-way street, an unending sequence of cars of width $a$, length $b$ passes with velocity $v$. The cars are separated by the distance $c$. A pedestrian crosses the street perpendicularly with velocity $w$, without paying attention to the cars. +(a) What is the probability that the pedestrian crosses the street uninjured? +(b) Can he improve this probability by crossing the road in a direction other than perpendicular? +7. (BRA 1) A convex quadrilateral is inscribed in a circle of radius 1. Prove that the difference between its perimeter and the sum of the lengths of its diagonals is greater than zero and less than 2. +8. (BRA 2) Let $K$ be a convex set in the $x y$-plane, symmetric with respect to the origin and having area greater than 4 . Prove that there exists a point $(m, n) \neq(0,0)$ in $K$ such that $m$ and $n$ are integers. +9. (BRA 3) (SL85-2). +10. (BUL 1) (SL85-13). +11. (BUL 2) Let $a$ and $b$ be integers and $n$ a positive integer. Prove that + +$$ +\frac{b^{n-1} a(a+b)(a+2 b) \cdots(a+(n-1) b)}{n!} +$$ + +is an integer. +12. (CAN 1) Find the maximum value of + +$$ +\sin ^{2} \theta_{1}+\sin ^{2} \theta_{2}+\cdots+\sin ^{2} \theta_{n} +$$ + +subject to the restrictions $0 \leq \theta_{i} \leq \pi, \theta_{1}+\theta_{2}+\cdots+\theta_{n}=\pi$. +13. (CAN 2) Find the average of the quantity + +$$ +\left(a_{1}-a_{2}\right)^{2}+\left(a_{2}-a_{3}\right)^{2}+\cdots+\left(a_{n-1}-a_{n}\right)^{2} +$$ + +taken over all permutations $\left(a_{1}, a_{2}, \ldots, a_{n}\right)$ of $(1,2, \ldots, n)$. +14. (CAN 3) Let $k$ be a positive integer. Define $u_{0}=0, u_{1}=1$, and $u_{n}=k u_{n-1}-u_{n-2}, n \geq 2$. Show that for each integer $n$, the number $u_{1}^{3}+u_{2}^{3}+\cdots+u_{n}^{3}$ is a multiple of $u_{1}+u_{2}+\cdots+u_{n}$. +15. (CAN 4) Superchess is played on on a $12 \times 12$ board, and it uses superknights, which move between opposite corner cells of any $3 \times 4$ subboard. Is it possible for a superknight to visit every other cell of a superchessboard exactly once and return to its starting cell? +16. (CAN 5) (SL85-18). +17. (CUB 1) Set + +$$ +A_{n}=\sum_{k=1}^{n} \frac{k^{6}}{2^{k}} +$$ + +Find $\lim _{n \rightarrow \infty} A_{n}$. +18. (CYP 1) The circles $(R, r)$ and $(P, \rho)$, where $r>\rho$, touch externally at $A$. Their direct common tangent touches $(R, r)$ at $B$ and $(P, \rho)$ at $C$. The line $R P$ meets the circle $(P, \rho)$ again at $D$ and the line $B C$ at $E$. If $|B C|=6|D E|$, prove that: +(a) the lengths of the sides of the triangle $R B E$ are in an arithmetic progression, and +(b) $|A B|=2|A C|$. +19. (CYP 2) Solve the system of simultaneous equations + +$$ +\begin{aligned} +\sqrt{x}-1 / y-2 w+3 z & =1 \\ +x+1 / y^{2}-4 w^{2}-9 z^{2} & =3 \\ +x \sqrt{x}-1 / y^{3}-8 w^{3}+27 z^{3} & =-5 \\ +x^{2}+1 / y^{4}-16 w^{4}-81 z^{4} & =15 +\end{aligned} +$$ + +20. (CZS 1) Let $T$ be the set of all lattice points (i.e., all points with integer coordinates) in three-dimensional space. Two such points $(x, y, z)$ and $(u, v, w)$ are called neighbors if $|x-u|+|y-v|+|z-w|=1$. Show that there exists a subset $S$ of $T$ such that for each $p \in T$, there is exactly one point of $S$ among $p$ and its neighbors. +21. (CZS 2) Let $A$ be a set of positive integers such that for any two elements $x, y$ of $A,|x-y| \geq \frac{x y}{25}$. Prove that $A$ contains at most nine elements. Give an example of such a set of nine elements. +22. (CZS 3) (SL85-7). +23. (CZS 4) Let $\mathbb{N}=\{1,2,3, \ldots\}$. For real $x, y$, set $S(x, y)=\{s \mid s=$ $[n x+y], n \in \mathbb{N}\}$. Prove that if $r>1$ is a rational number, there exist real numbers $u$ and $v$ such that + +$$ +S(r, 0) \cap S(u, v)=\emptyset, S(r, 0) \cup S(u, v)=\mathbb{N} +$$ + +24. (FRA 1) Let $d \geq 1$ be an integer that is not the square of an integer. Prove that for every integer $n \geq 1$, + +$$ +(n \sqrt{d}+1)|\sin (n \pi \sqrt{d})| \geq 1 +$$ + +25. (FRA 2) Find eight positive integers $n_{1}, n_{2}, \ldots, n_{8}$ with the following property: For every integer $k,-1985 \leq k \leq 1985$, there are eight integers $\alpha_{1}, \alpha_{2}, \ldots, \alpha_{8}$, each belonging to the set $\{-1,0,1\}$, such that $k=\sum_{i=1}^{8} \alpha_{i} n_{i}$. +26. (FRA 3) (SL85-15). +27. (FRA 4) Let $O$ be a point on the oriented Euclidean plane and (i,j) a directly oriented orthonormal basis. Let $C$ be the circle of radius 1, centered at $O$. For every real number $t$ and nonnegative integer $n$ let $M_{n}$ be the point on $C$ for which $\left\langle\mathbf{i}, \overrightarrow{O M_{n}}\right\rangle=\cos 2^{n} t\left(\right.$ or $\left.\overrightarrow{O M_{n}}=\cos 2^{n} t \mathbf{i}+\sin 2^{n} t \mathbf{j}\right)$. Let $k \geq 2$ be an integer. Find all real numbers $t \in[0,2 \pi)$ that satisfy +(i) $M_{0}=M_{k}$, and +(ii) if one starts from $M_{0}$ and goes once around $C$ in the positive direction, one meets successively the points $M_{0}, M_{1}, \ldots, M_{k-2}, M_{k-1}$, in this order. +28. (FRG 1) Let $M$ be the set of the lengths of an octahedron whose sides are congruent quadrangles. Prove that $M$ has at most three elements. +(FRG 1a) Let an octahedron whose sides are congruent quadrangles be given. Prove that each of these quadrangles has two equal sides meeting at a common vertex. +29. (FRG 2) Call a four-digit number $(x y z t)_{B}$ in the number system with base $B$ stable if $(x y z t)_{B}=(d c b a)_{B}-(a b c d)_{B}$, where $a \leq b \leq c \leq d$ are the digits of $(x y z t)_{B}$ in ascending order. Determine all stable numbers in the number system with base $B$. +(FRG 2a) The same problem with $B=1985$. +(FRG 2b) With assumptions as in FRG 2, determine the number of bases $B \leq 1985$ such that there is a stable number with base $B$. +30. (GBR 1) A plane rectangular grid is given and a "rational point" is defined as a point $(x, y)$ where $x$ and $y$ are both rational numbers. Let $A, B, A^{\prime}, B^{\prime}$ be four distinct rational points. Let $P$ be a point such that $\frac{A^{\prime} B^{\prime}}{A B}=\frac{B^{\prime} P}{B C}=\frac{P A^{\prime}}{P A}$. In other words, the triangles $A B P, A^{\prime} B^{\prime} P$ are directly or oppositely similar. Prove that $P$ is in general a rational point and find the exceptional positions of $A^{\prime}$ and $B^{\prime}$ relative to $A$ and $B$ such that there exists a $P$ that is not a rational point. +31. (GBR 2) Let $E_{1}, E_{2}$, and $E_{3}$ be three mutually intersecting ellipses, all in the same plane. Their foci are respectively $F_{2}, F_{3} ; F_{3}, F_{1}$; and $F_{1}, F_{2}$. The three foci are not on a straight line. Prove that the common chords of each pair of ellipses are concurrent. +32. (GBR 3) A collection of $2 n$ letters contains 2 each of $n$ different letters. The collection is partitioned into $n$ pairs, each pair containing 2 letters, which may be the same or different. Denote the number of distinct partitions by $u_{n}$. (Partitions differing in the order of the pairs in the partition or in the order of the two letters in the pairs are not considered distinct.) +Prove that $u_{n+1}=(n+1) u_{n}-\frac{n(n-1)}{2} u_{n-2}$. +(GBR 3a) A pack of $n$ cards contains $n$ pairs of 2 identical cards. It is shuffled and 2 cards are dealt to each of $n$ different players. Let $p_{n}$ be the probability that every one of the $n$ players is dealt two identical cards. Prove that $\frac{1}{p_{n+1}}=\frac{n+1}{p_{n}}-\frac{n(n-1)}{2 p_{n-2}}$. +33. (GBR 4) (SL85-12). +34. (GBR 5) (SL85-20). +35. (GDR 1) We call a coloring $f$ of the elements in the set $M=\{(x, y) \mid$ $x=0,1, \ldots, k n-1 ; y=0,1, \ldots, l n-1\}$ with $n$ colors allowable if every color appears exactly $k$ and $l$ times in each row and column and there are no rectangles with sides parallel to the coordinate axes such that all the vertices in $M$ have the same color. Prove that every allowable coloring $f$ satisfies $k l \leq n(n+1)$. +36. (GDR 2) Determine whether there exist 100 distinct lines in the plane having exactly 1985 distinct points of intersection. +37. (GDR 3) Prove that a triangle with angles $\alpha, \beta, \gamma$, circumradius $R$, and area $A$ satisfies + +$$ +\tan \frac{\alpha}{2}+\tan \frac{\beta}{2}+\tan \frac{\gamma}{2} \leq \frac{9 R^{2}}{4 A} +$$ + +38. (IRE 1) (SL85-21). +39. (IRE 2) Given a triangle $A B C$ and external points $X, Y$, and $Z$ such that $\measuredangle B A Z=\measuredangle C A Y, \measuredangle C B X=\measuredangle A B Z$, and $\measuredangle A C Y=\measuredangle B C X$, prove that $A X, B Y$, and $C Z$ are concurrent. +40. (IRE 3) Each of the numbers $x_{1}, x_{2}, \ldots, x_{n}$ equals 1 or -1 and + +$$ +\begin{aligned} +& x_{1} x_{2} x_{3} x_{4}+x_{2} x_{3} x_{4} x_{5}+\cdots+x_{n-3} x_{n-2} x_{n-1} x_{n} \\ +& \quad+x_{n-2} x_{n-1} x_{n} x_{1}+x_{n-1} x_{n} x_{1} x_{2}+x_{n} x_{1} x_{2} x_{3}=0 +\end{aligned} +$$ + +Prove that $n$ is divisible by 4 . +41. (IRE 4) (SL85-14). +42. (ISR 1) Prove that the product of two sides of a triangle is always greater than the product of the diameters of the inscribed circle and the circumscribed circle. +43. (ISR 2) Suppose that 1985 points are given inside a unit cube. Show that one can always choose 32 of them in such a way that every (possibly +degenerate) closed polygon with these points as vertices has a total length of less than $8 \sqrt{3}$. +44. (ISR 3) (SL85-19). +45. (ITA 1) Two persons, $X$ and $Y$, play with a die. $X$ wins a game if the outcome is 1 or $2 ; Y$ wins in the other cases. A player wins a match if he wins two consecutive games. For each player determine the probability of winning a match within 5 games. Determine the probabilities of winning in an unlimited number of games. If $X$ bets 1 , how much must $Y$ bet for the game to be fair? +46. (ITA 2) Let $C$ be the curve determined by the equation $y=x^{3}$ in the rectangular coordinate system. Let $t$ be the tangent to $C$ at a point $P$ of $C ; t$ intersects $C$ at another point $Q$. Find the equation of the set $L$ of the midpoints $M$ of $P Q$ as $P$ describes $C$. Is the correspondence associating $P$ and $M$ a bijection of $C$ on $L$ ? Find a similarity that transforms $C$ into $L$. +47. (ITA 3) Let $F$ be the correspondence associating with every point $P=$ $(x, y)$ the point $P^{\prime}=\left(x^{\prime}, y^{\prime}\right)$ such that + +$$ +x^{\prime}=a x+b, \quad y^{\prime}=a y+2 b . +$$ + +Show that if $a \neq 1$, all lines $P P^{\prime}$ are concurrent. Find the equation of the set of points corresponding to $P=(1,1)$ for $b=a^{2}$. Show that the composition of two mappings of type (1) is of the same type. +48. (ITA 4) In a given country, all inhabitants are knights or knaves. A knight never lies; a knave always lies. We meet three persons, $A, B$, and $C$. Person $A$ says, "If $C$ is a knight, $B$ is a knave." Person $C$ says, " $A$ and I are different; one is a knight and the other is a knave." Who are the knights, and who are the knaves? +49. (MON 1) (SL85-1). +50. (MON 2) From each of the vertices of a regular $n$-gon a car starts to move with constant speed along the perimeter of the $n$-gon in the same direction. Prove that if all the cars end up at a vertex $A$ at the same time, then they never again meet at any other vertex of the $n$-gon. Can they meet again at $A$ ? +51. (MON 3) Let $f_{1}=\left(a_{1}, a_{2}, \ldots, a_{n}\right), n>2$, be a sequence of integers. From $f_{1}$ one constructs a sequence $f_{k}$ of sequences as follows: if $f_{k}=$ $\left(c_{1}, c_{2}, \ldots, c_{n}\right)$, then $f_{k+1}=\left(c_{i_{1}}, c_{i_{2}}, c_{i_{3}}+1, c_{i_{4}}+1, \ldots, c_{i_{n}}+1\right)$, where $\left(c_{i_{1}}, c_{i_{2}}, \ldots, c_{i_{n}}\right)$ is a permutation of $\left(c_{1}, c_{2}, \ldots, c_{n}\right)$. Give a necessary and sufficient condition for $f_{1}$ under which it is possible for $f_{k}$ to be a constant sequence $\left(b_{1}, b_{2}, \ldots, b_{n}\right), b_{1}=b_{2}=\cdots=b_{n}$, for some $k$. +52. (MON 4) In the triangle $A B C$, let $B_{1}$ be on $A C, E$ on $A B, G$ on $B C$, and let $E G$ be parallel to $A C$. Furthermore, let $E G$ be tangent to the +inscribed circle of the triangle $A B B_{1}$ and intersect $B B_{1}$ at $F$. Let $r, r_{1}$, and $r_{2}$ be the inradii of the triangles $A B C, A B B_{1}$, and $B F G$, respectively. Prove that $r=r_{1}+r_{2}$. +53. (MON 5) For each $P$ inside the triangle $A B C$, let $A(P), B(P)$, and $C(P)$ be the points of intersection of the lines $A P, B P$, and $C P$ with the sides opposite to $A, B$, and $C$, respectively. Determine $P$ in such a way that the area of the triangle $A(P) B(P) C(P)$ is as large as possible. +54. (MOR 1) Set $S_{n}=\sum_{p=1}^{n}\left(p^{5}+p^{7}\right)$. Determine the greatest common divisor of $S_{n}$ and $S_{3 n}$. +55. (MOR 2) The points $A, B, C$ are in this order on line $D$, and $A B=4 B C$. Let $M$ be a variable point on the perpendicular to $D$ through $C$. Let $M T_{1}$ and $M T_{2}$ be tangents to the circle with center $A$ and radius $A B$. Determine the locus of the orthocenter of the triangle $M T_{1} T_{2}$. +56. (MOR 3) Let $A B C D$ be a rhombus with angle $\angle A=60^{\circ}$. Let $E$ be a point, different from $D$, on the line $A D$. The lines $C E$ and $A B$ intersect at $F$. The lines $D F$ and $B E$ intersect at $M$. Determine the angle $\measuredangle B M D$ as a function of the position of $E$ on $A D$. +57. (NET 1) The solid $S$ is defined as the intersection of the six spheres with the six edges of a regular tetrahedron $T$, with edge length 1 , as diameters. Prove that $S$ contains two points at a distance $\frac{1}{\sqrt{6}}$. +(NET 1a) Using the same assumptions, prove that no pair of points in $S$ has a distance larger than $\frac{1}{\sqrt{6}}$. +58. (NET 2) Prove that there are infinitely many pairs $(k, N)$ of positive integers such that $1+2+\cdots+k=(k+1)+(k+2)+\cdots+N$. +59. (NET 3) (SL85-3). +60. (NOR 1) The sequence $\left(s_{n}\right)$, where $s_{n}=\sum_{k=1}^{n} \sin k, n=1,2, \ldots$, is bounded. Find an upper and lower bound. +61. (NOR 2) Consider the set $A=\{0,1,2, \ldots, 9\}$ and let $\left(B_{1}, B_{2}, \ldots, B_{k}\right)$ be a collection of nonempty subsets of $A$ such that $B_{i} \cap B_{j}$ has at most two elements for $i \neq j$. What is the maximal value of $k$ ? +62. (NOR 3) A "large" circular disk is attached to a vertical wall. It rotates clockwise with one revolution per minute. An insect lands on the disk and immediately starts to climb vertically upward with constant speed $\frac{\pi}{3} \mathrm{~cm}$ per second (relative to the disk). Describe the path of the insect +(a) relative to the disk; +(b) relative to the wall. +63. (POL 1) (SL85-6). +64. (POL 2) Let $p$ be a prime. For which $k$ can the set $\{1,2, \ldots, k\}$ be partitioned into $p$ subsets with equal sums of elements? +65. (POL 3) Define the functions $f, F: \mathbb{N} \rightarrow \mathbb{N}$, by + +$$ +f(n)=\left[\frac{3-\sqrt{5}}{2} n\right], \quad F(k)=\min \left\{n \in \mathbb{N} \mid f^{k}(n)>0\right\} +$$ + +where $f^{k}=f \circ \cdots \circ f$ is $f$ iterated $n$ times. Prove that $F(k+2)=$ $3 F(k+1)-F(k)$ for all $k \in \mathbb{N}$. +66. (ROM 1) (SL85-5). +67. (ROM 2) Let $k \geq 2$ and $n_{1}, n_{2}, \ldots, n_{k} \geq 1$ natural numbers having the property $n_{2}\left|2^{n_{1}}-1, n_{3}\right| 2^{n_{2}}-1, \ldots, n_{k} \mid 2^{n_{k-1}}-1$, and $n_{1} \mid 2^{n_{k}}-1$. Show that $n_{1}=n_{2}=\cdots=n_{k}=1$. +68. (ROM 3) Show that the sequence $\left\{a_{n}\right\}_{n \geq 1}$ defined by $a_{n}=[n \sqrt{2}]$ contains an infinite number of integer powers of 2 . ( $[x]$ is the integer part of $x$.) +69. (ROM 4) Let $A$ and $B$ be two finite disjoint sets of points in the plane such that any three distinct points in $A \cup B$ are not collinear. Assume that at least one of the sets $A, B$ contains at least five points. Show that there exists a triangle all of whose vertices are contained in $A$ or in $B$ that does not contain in its interior any point from the other set. +70. (ROM 5) Let $C$ be a class of functions $f: \mathbb{N} \rightarrow \mathbb{N}$ that contains the functions $S(x)=x+1$ and $E(x)=x-\left[\sqrt{x}^{2}\right.$ for every $x \in \mathbb{N}$. ( $[x]$ is the integer part of $x$.) If $C$ has the property that for every $f, g \in C$, $f+g, f g, f \circ g \in C$, show that the function $\max (f(x)-g(x), 0)$ is in $C$. +71. (ROM 6) For every integer $r>1$ find the smallest integer $h(r)>1$ having the following property: For any partition of the set $\{1,2, \ldots, h(r)\}$ into $r$ classes, there exist integers $a \geq 0,1 \leq x \leq y$ such that the numbers $a+x, a+y, a+x+y$ are contained in the same class of the partition. +72. (SPA 1) Construct a triangle $A B C$ given the side $A B$ and the distance $O H$ from the circumcenter $O$ to the orthocenter $H$, assuming that $O H$ and $A B$ are parallel. +73. (SPA 2) Let $A_{1} A_{2}, B_{1} B_{2}, C_{1} C_{2}$ be three equal segments on the three sides of an equilateral triangle. Prove that in the triangle formed by the lines $B_{2} C_{1}, C_{2} A_{1}, A_{2} B_{1}$, the segments $B_{2} C_{1}, C_{2} A_{1}, A_{2} B_{1}$ are proportional to the sides in which they are contained. +74. (SPA 3) Find the triples of positive integers $x, y, z$ satisfying + +$$ +\frac{1}{x}+\frac{1}{y}+\frac{1}{z}=\frac{4}{5} +$$ + +75. (SPA 4) Let $A B C D$ be a rectangle, $A B=a, B C=b$. Consider the family of parallel and equidistant straight lines (the distance between two consecutive lines being $d$ ) that are at an the angle $\phi, 0 \leq \phi \leq 90^{\circ}$, +with respect to $A B$. Let $L$ be the sum of the lengths of all the segments intersecting the rectangle. Find: +(a) how $L$ varies, +(b) a necessary and sufficient condition for $L$ to be a constant, and +(c) the value of this constant. +76. (SWE 1) Are there integers $m$ and $n$ such that + +$$ +5 m^{2}-6 m n+7 n^{2}=1985 ? +$$ + +77. (SWE 2) Two equilateral triangles are inscribed in a circle with radius $r$. Let $A$ be the area of the set consisting of all points interior to both triangles. Prove that $2 A \geq r^{2} \sqrt{3}$. +78. (SWE 3) (SL85-17). +79. (SWE 4) Let $a, b$, and $c$ be real numbers such that + +$$ +\frac{1}{b c-a^{2}}+\frac{1}{c a-b^{2}}+\frac{1}{a b-c^{2}}=0 +$$ + +Prove that + +$$ +\frac{a}{\left(b c-a^{2}\right)^{2}}+\frac{b}{\left(c a-b^{2}\right)^{2}}+\frac{c}{\left(a b-c^{2}\right)^{2}}=0 . +$$ + +80. (TUR 1) Let $E=\{1,2, \ldots, 16\}$ and let $M$ be the collection of all $4 \times 4$ matrices whose entries are distinct members of $E$. If a matrix $A=$ $\left(a_{i j}\right)_{4 \times 4}$ is chosen randomly from $M$, compute the probability $p(k)$ of $\max _{i} \min _{j} a_{i j}=k$ for $k \in E$. Furthermore, determine $l \in E$ such that $p(l)=\max \{p(k) \mid k \in E\}$. +81. (TUR 2) Given the side $a$ and the corresponding altitude $h_{a}$ of a triangle $A B C$, find a relation between $a$ and $h_{a}$ such that it is possible to construct, with straightedge and compass, triangle $A B C$ such that the altitudes of $A B C$ form a right triangle admitting $h_{a}$ as hypotenuse. +82. (TUR 3) Find all cubic polynomials $x^{3}+a x^{2}+b x+c$ admitting the rational numbers $a, b$, and $c$ as roots. +83. (TUR 4) Let $\Gamma_{i}, i=0,1,2, \ldots$, be a circle of radius $r_{i}$ inscribed in an angle of measure $2 \alpha$ such that each $\Gamma_{i}$ is externally tangent to $\Gamma_{i+1}$ and $r_{i+1}R$. The points $M$ and $N$ are chosen on $\Lambda$ in such a way that the circle with diameter $M N$ is externally tangent to the given circle. Show that there exists a point $A$ in the plane such that all the segments $M N$ are seen in a constant angle from $A$. + +### 3.26.3 Shortlisted Problems + +## Proposals of the Problem Selection Committee. + +1. (MON 1) ${ }^{\mathrm{IMO} 4}$ Given a set $M$ of 1985 positive integers, none of which has a prime divisor larger than 26 , prove that the set has four distinct elements whose geometric mean is an integer. +2. (BRA 3) A polyhedron has 12 faces and is such that: +(i) all faces are isosceles triangles, +(ii) all edges have length either $x$ or $y$, +(iii) at each vertex either 3 or 6 edges meet, and +(iv) all dihedral angles are equal. + +Find the ratio $x / y$. +3. (NET 3) ${ }^{\mathrm{IMO} 3}$ The weight $w(p)$ of a polynomial $p, p(x)=\sum_{i=0}^{n} a_{i} x^{i}$, with integer coefficients $a_{i}$ is defined as the number of its odd coefficients. For $i=0,1,2, \ldots$, let $q_{i}(x)=(1+x)^{i}$. Prove that for any finite sequence $0 \leq i_{1}0$. Prove that each $P_{m}(x, y, z)$ is symmetric, in other words, is unaltered by any permutation of $x, y, z$. +13. 4a.(BUL 1) Let $m$ boxes be given, with some balls in each box. Let $n0$ recursively by + +$$ +f_{1}(x)=x, \quad f_{n+1}(x)=f_{n}(x)\left(f_{n}(x)+\frac{1}{n}\right) . +$$ + +Prove that there exists one and only one positive number $a$ such that $02)$ in such a way that any two fields that have a common edge or a vertex are assigned numbers differing by at most $n+1$. What is the total number of such placements? +43. (HUN 2) (SL86-10). +44. (IRE 1) (SL86-14). +45. (IRE 2) Given $n$ real numbers $a_{1} \leq a_{2} \leq \cdots \leq a_{n}$, define + +$$ +M_{1}=\frac{1}{n} \sum_{i=1}^{n} a_{i}, \quad M_{2}=\frac{2}{n(n-1)} \sum_{1 \leq i3$. Find at least $3(n+1)$ [easier version: $2(n+1)]$ sequences of positive integers $x, y, z$ satisfying + +$$ +x y z=p^{n}(x+y+z) +$$ + +that do not differ only by permutation. +5. (FRG 1) ${ }^{\mathrm{IMO1}}$ The set $S=\{2,5,13\}$ has the property that for every $a, b \in S, a \neq b$, the number $a b-1$ is a perfect square. Show that for every positive integer $d$ not in $S$, the set $S \cup\{d\}$ does not have the above property. +6. (NET 1) Find four positive integers each not exceeding 70000 and each having more than 100 divisors. +7. (FRA 5) Let real numbers $x_{1}, x_{2}, \ldots, x_{n}$ satisfy $01$ there are integers $e_{i}$ not all 0 and with $\left|e_{i}\right|0$. Prove that there exists a function $u:[1,+\infty) \rightarrow \mathbb{R}$ satisfying $u\left(\frac{x+1 / x}{2}\right)=f(x)$ for all $x>0$. +8. (BEL 2) Determine the least possible value of the natural number $n$ such that $n$ ! ends in exactly 1987 zeros. +9. (BEL 3) In the set of 20 elements $\{1,2,3,4,5,6,7,8,9,0, A, B, C$, $D, J, K, L, U, X, Y, Z\}$ we have made a random sequence of 28 throws. What is the probability that the sequence $C U B A J U L Y 1987$ appears in this order in the sequence already thrown? +10. (FIN 1) In a Cartesian coordinate system, the circle $C_{1}$ has center $O_{1}(-2,0)$ and radius 3 . Denote the point $(1,0)$ by $A$ and the origin by $O$. Prove that there is a constant $c>0$ such that for every $X$ that is exterior to $C_{1}$, + +$$ +O X-1 \geq c \min \left\{A X, A X^{2}\right\} +$$ + +Find the largest possible $c$. +11. (FIN 2) Let $S \subset[0,1]$ be a set of 5 points with $\{0,1\} \subset S$. The graph of a real function $f:[0,1] \rightarrow[0,1]$ is continuous and increasing, and it is linear on every subinterval $I$ in $[0,1]$ such that the endpoints but no interior points of $I$ are in $S$. We want to compute, using a computer, the extreme values of $g(x, t)=\frac{f(x+t)-f(x)}{f(x)-f(x-t)}$ for $x-t, x+t \in[0,1]$. At how many points $(x, t)$ is it necessary to compute $g(x, t)$ with the computer? +12. (FIN 3) (SL87-3). +13. (FIN 4) $A$ be an infinite set of positive integers such that every $n \in A$ is the product of at most 1987 prime numbers. Prove that there is an infinite set $B \subset A$ and a number $p$ such that the greatest common divisor of any two distinct numbers in $B$ is $b$. +14. (FRA 1) Given $n$ real numbers $01$ be a real number, and let $n$ be the largest integer smaller than $r$. Consider an arbitrary real number $x$ with $0 \leq x \leq \frac{n}{r-1}$. By a base-r expansion of $x$ we mean a representation of $x$ in the form + +$$ +x=\frac{a_{1}}{r}+\frac{a_{2}}{r^{2}}+\frac{a_{3}}{r^{3}}+\cdots, +$$ + +where the $a_{i}$ are integers with $0 \leq a_{i}\gamma, \beta+\gamma>\alpha, \gamma+\alpha>\beta$. Prove that it is possible to draw a triangle with the lengths of the $\operatorname{sides} \sin \alpha, \sin \beta$, $\sin \gamma$. Moreover, prove that its area is less than + +$$ +\frac{1}{8}(\sin 2 \alpha+\sin 2 \beta+\sin 2 \gamma) +$$ + +69. (USS 3) (SL87-20). +70. (USS 4) (SL87-21). +71. (USS 5) To every natural number $k, k \geq 2$, there corresponds a sequence $a_{n}(k)$ according to the following rule: + +$$ +a_{0}=k, \quad a_{n}=\tau\left(a_{n-1}\right) \text { for } n \geq 1, +$$ + +in which $\tau(a)$ is the number of different divisors of $a$. Find all $k$ for which the sequence $a_{n}(k)$ does not contain the square of an integer. +72. (VIE 1) Is it possible to cover a rectangle of dimensions $m \times n$ with bricks that have the trimino angular shape (an arrangement of three unit squares forming the letter L) if: +(a) $m \times n=1985 \times 1987$; +(b) $m \times n=1987 \times 1989$ ? +73. (VIE 2) Let $f(x)$ be a periodic function of period $T>0$ defined over $\mathbb{R}$. Its first derivative is continuous on $\mathbb{R}$. Prove that there exist $x, y \in[0, T)$ such that $x \neq y$ and + +$$ +f(x) f^{\prime}(y)=f(y) f^{\prime}(x) . +$$ + +74. (VIE 3) (SL87-22). +75. (VIE 4) Let $a_{k}$ be positive numbers such that $a_{1} \geq 1$ and $a_{k+1}-a_{k} \geq 1$ $(k=1,2, \ldots)$. Prove that for every $n \in \mathbb{N}$, + +$$ +\sum_{k=1}^{n} \frac{1}{a_{k+1} \sqrt[1987]{a_{k}}}<1987 +$$ + +76. (VIE 5) Given two sequences of positive numbers $\left\{a_{k}\right\}$ and $\left\{b_{k}\right\}(k \in \mathbb{N})$ such that +(i) $a_{k}y$ and $f(y)-y \geq v \geq f(x)-x$, then $f(z)=v+z$, for some number $z$ between $x$ and $y$. +(ii) The equation $f(x)=0$ has at least one solution, and among the solutions of this equation, there is one that is not smaller than all the other solutions; +(iii) $f(0)=1$. +(iv) $f(1987) \leq 1988$. +(v) $f(x) f(y)=f(x f(y)+y f(x)-x y)$. + +Find $f(1987)$. +2. (USA 3) At a party attended by $n$ married couples, each person talks to everyone else at the party except his or her spouse. The conversations involve sets of persons or cliques $C_{1}, C_{2}, \ldots, C_{k}$ with the following property: no couple are members of the same clique, but for every other pair of persons there is exactly one clique to which both members belong. Prove that if $n \geq 4$, then $k \geq 2 n$. +3. (FIN 3) Does there exist a second-degree polynomial $p(x, y)$ in two variables such that every nonnegative integer $n$ equals $p(k, m)$ for one and only one ordered pair $(k, m)$ of nonnegative integers? +4. (FRA 5) Let $A B C D E F G H$ be a parallelepiped with $A E\|B F\| C G \| D H$. Prove the inequality + +$$ +A F+A H+A C \leq A B+A D+A E+A G +$$ + +In what cases does equality hold? +5. (GBR 1) Find, with proof, the point $P$ in the interior of an acute-angled triangle $A B C$ for which $B L^{2}+C M^{2}+A N^{2}$ is a minimum, where $L, M, N$ are the feet of the perpendiculars from $P$ to $B C, C A, A B$ respectively. +6. (GRE 4) Show that if $a, b, c$ are the lengths of the sides of a triangle and if $2 S=a+b+c$, then + +$$ +\frac{a^{n}}{b+c}+\frac{b^{n}}{c+a}+\frac{c^{n}}{a+b} \geq\left(\frac{2}{3}\right)^{n-2} S^{n-1}, \quad n \geq 1 +$$ + +7. (NET 1) Given five real numbers $u_{0}, u_{1}, u_{2}, u_{3}, u_{4}$, prove that it is always possible to find five real numbers $v_{0}, v_{1}, v_{2}, v_{3}, v_{4}$ that satisfy the following conditions: +(i) $u_{i}-v_{i} \in \mathbb{N}$. +(ii) $\sum_{0 \leq i1$. Prove that for any integers $a_{1}, a_{2}, \ldots, a_{m}$ and $b_{1}, b_{2}$, $\ldots, b_{k}$ there must be two products $a_{i} b_{j}$ and $a_{s} b_{t}((i, j) \neq(s, t))$ that give the same residue when divided by $m k$. +9. (HUN 2) Does there exist a set $M$ in usual Euclidean space such that for every plane $\lambda$ the intersection $M \cap \lambda$ is finite and nonempty? +10. (ICE 3) Let $S_{1}$ and $S_{2}$ be two spheres with distinct radii that touch externally. The spheres lie inside a cone $C$, and each sphere touches the cone in a full circle. Inside the cone there are $n$ additional solid spheres arranged in a ring in such a way that each solid sphere touches the cone $C$, both of the spheres $S_{1}$ and $S_{2}$ externally, as well as the two neighboring solid spheres. What are the possible values of $n$ ? +11. (POL 1) Find the number of partitions of the set $\{1,2, \ldots, n\}$ into three subsets $A_{1}, A_{2}, A_{3}$, some of which may be empty, such that the following conditions are satisfied: +(i) After the elements of every subset have been put in ascending order, every two consecutive elements of any subset have different parity. +(ii) If $A_{1}, A_{2}, A_{3}$ are all nonempty, then in exactly one of them the minimal number is even. +12. (POL 5) Given a nonequilateral triangle $A B C$, the vertices listed counterclockwise, find the locus of the centroids of the equilateral triangles $A^{\prime} B^{\prime} C^{\prime}$ (the vertices listed counterclockwise) for which the triples of points $A, B^{\prime}, C^{\prime} ; A^{\prime}, B, C^{\prime}$; and $A^{\prime}, B^{\prime}, C$ are collinear. +13. (GDR 2) ${ }^{\mathrm{IMO} 5}$ Is it possible to put 1987 points in the Euclidean plane such that the distance between each pair of points is irrational and each three points determine a nondegenerate triangle with rational area? +14. (FRG 1) How many words with $n$ digits can be formed from the alphabet $\{0,1,2,3,4\}$, if neighboring digits must differ by exactly one? +15. (FRG 2) $)^{\mathrm{IMO} 3}$ Suppose $x_{1}, x_{2}, \ldots, x_{n}$ are real numbers with $x_{1}^{2}+x_{2}^{2}+$ $\cdots+x_{n}^{2}=1$. Prove that for any integer $k>1$ there are integers $e_{i}$ not all 0 and with $\left|e_{i}\right|\gamma, \beta+\gamma>\alpha, \gamma+\alpha>\beta$. Prove that with the segments of lengths $\sin \alpha, \sin \beta, \sin \gamma$ we can construct a triangle and that its area is not greater than + +$$ +\frac{1}{8}(\sin 2 \alpha+\sin 2 \beta+\sin 2 \gamma) +$$ + +20. (USS 3) ${ }^{\text {IMO6 }}$ Let $f(x)=x^{2}+x+p, p \in \mathbb{N}$. Prove that if the numbers $f(0), f(1), \ldots, f([\sqrt{p / 3}])$ are primes, then all the numbers $f(0), f(1), \ldots$, $f(p-2)$ are primes. +21. (USS 4) ${ }^{\mathrm{IMO} 2}$ The prolongation of the bisector $A L(L \in B C)$ in the acuteangled triangle $A B C$ intersects the circumscribed circle at point $N$. From point $L$ to the sides $A B$ and $A C$ are drawn the perpendiculars $L K$ and $L M$ respectively. Prove that the area of the triangle $A B C$ is equal to the area of the quadrilateral $A K N M$. +22. (VIE 3) ${ }^{\mathrm{IMO} 4}$ Does there exist a function $f: \mathbb{N} \rightarrow \mathbb{N}$, such that $f(f(n))=$ $n+1987$ for every natural number $n$ ? +23. (YUG 2) Prove that for every natural number $k(k \geq 2)$ there exists an irrational number $r$ such that for every natural number $m$, + +$$ +\left[r^{m}\right] \equiv-1 \quad(\bmod k) +$$ + +Remark. An easier variant: Find $r$ as a root of a polynomial of second degree with integer coefficients. + +### 3.29 The Twenty-Ninth IMO Canberra, Australia, July 9-21, 1988 + +### 3.29.1 Contest Problems + +First Day (July 15) + +1. Consider two concentric circles of radii $R$ and $r(R>r)$ with center $O$. Fix $P$ on the small circle and consider the variable chord $P A$ of the small circle. Points $B$ and $C$ lie on the large circle; $B, P, C$ are collinear and $B C$ is perpendicular to $A P$. +(a) For which value(s) of $\angle O P A$ is the sum $B C^{2}+C A^{2}+A B^{2}$ extremal? +(b) What are the possible positions of the midpoints $U$ of $B A$ and $V$ of $A C$ as $\measuredangle O P A$ varies? +2. Let $n$ be an even positive integer. Let $A_{1}, A_{2}, \ldots, A_{n+1}$ be sets having $n$ elements each such that any two of them have exactly one element in common, while every element of their union belongs to at least two of the given sets. For which $n$ can one assign to every element of the union one of the numbers 0 and 1 in such a manner that each of the sets has exactly $n / 2$ zeros? +3. A function $f$ defined on the positive integers (and taking positive integer values) is given by + +$$ +\begin{aligned} +f(1) & =1, \quad f(3)=3 \\ +f(2 n) & =f(n) \\ +f(4 n+1) & =2 f(2 n+1)-f(n) \\ +f(4 n+3) & =3 f(2 n+1)-2 f(n) +\end{aligned} +$$ + +for all positive integers $n$. Determine with proof the number of positive integers less than or equal to 1988 for which $f(n)=n$. + +Second Day (July 16) +4. Show that the solution set of the inequality + +$$ +\sum_{k=1}^{70} \frac{k}{x-k} \geq \frac{5}{4} +$$ + +is the union of disjoint half-open intervals with the sum of lengths 1988. +5. In a right-angled triangle $A B C$ let $A D$ be the altitude drawn to the hypotenuse and let the straight line joining the incenters of the triangles $A B D, A C D$ intersect the sides $A B, A C$ at the points $K, L$ respectively. If $E$ and $E_{1}$ denote the areas of the triangles $A B C$ and $A K L$ respectively, show that $\frac{E}{E_{1}} \geq 2$. +6. Let $a$ and $b$ be two positive integers such that $a b+1$ divides $a^{2}+b^{2}$. Show that $\frac{a^{2}+b^{2}}{a b+1}$ is a perfect square. + +### 3.29.2 Longlisted Problems + +1. (BUL 1) (SL88-1). +2. (BUL 2) Let $a_{n}=\left[\sqrt{(n+1)^{2}+n^{2}}\right], n=1,2, \ldots$, where $[x]$ denotes the integer part of $x$. Prove that +(a) there are infinitely many positive integers $m$ such that $a_{m+1}-a_{m}>1$; +(b) there are infinitely many positive integers $m$ such that $a_{m+1}-a_{m}=1$. +3. (BUL 3) (SL88-2). +4. (CAN 1) (SL88-3). +5. (CUB 1) Let $k$ be a positive integer and $M_{k}$ the set of all the integers that are between $2 k^{2}+k$ and $2 k^{2}+3 k$, both included. Is it possible to partition $M_{k}$ into two subsets $A$ and $B$ such that + +$$ +\sum_{x \in A} x^{2}=\sum_{x \in B} x^{2} ? +$$ + +6. (CZS 1) (SL88-4). +7. (CZS 2) (SL88-5). +8. (CZS 3) (SL88-6). +9. (FRA 1) If $a_{0}$ is a positive real number, consider the sequence $\left\{a_{n}\right\}$ defined by + +$$ +a_{n+1}=\frac{a_{n}^{2}-1}{n+1} \quad \text { for } n \geq 0 +$$ + +Show that there exists a real number $a>0$ such that: +(i) for all real $a_{0} \geq a$, the sequence $\left\{a_{n}\right\} \rightarrow+\infty(n \rightarrow \infty)$; +(ii) for all real $a_{0}q^{n}=v$, where $p$ and $q$ are prime numbers, and $m$ and $n$ are natural numbers, show that + +$$ +|A M|=1, \quad|B M|=9, \quad|D N|=8, \quad|P Q|=8 +$$ + +32. (HKG 3) Assuming that the roots of $x^{3}+p x^{2}+q x+r=0$ are all real and positive, find a relation between $p, q$, and $r$ that gives a necessary condition for the roots to be exactly the cosines of three angles of a triangle. +33. (HKG 4) Find a necessary and sufficient condition on the natural number $n$ for the equation $x^{n}+(2+x)^{n}+(2-x)^{n}=0$ to have a real root. +34. (HKG 5) Express the number 1988 as the sum of some positive integers in such a way that the product of these positive integers is maximal. +35. (HKG 6) In the triangle $A B C$, let $D, E$, and $F$ be the midpoints of the three sides, $X, Y$, and $Z$ the feet of the three altitudes, $H$ the orthocenter, and $P, Q$, and $R$ the midpoints of the line segments joining $H$ to the three vertices. Show that the nine points $D, E, F, P, Q, R, X, Y, Z$ lie on a circle. +36. (HUN 1) (SL88-14). +37. (HUN 2) Let $n$ points be given on the surface of a sphere. Show that the surface can be divided into $n$ congruent regions such that each of them contains exactly one of the given points. +38. (HUN 3) In a multiple choice test there were 4 questions and 3 possible answers for each question. A group of students was tested and it turned out that for any 3 of them there was a question that the three students answered differently. What is the maximal possible number of students tested? +39. (ICE 1) (SL88-15). +40. (ICE 2) A sequence of numbers $a_{n}, n=1,2, \ldots$, is defined as follows: $a_{1}=1 / 2$, and for each $n \geq 2$, + +$$ +a_{n}=\left(\frac{2 n-3}{2 n}\right) a_{n-1} +$$ + +Prove that $\sum_{k=1}^{n} a_{k}<1$ for all $n \geq 1$. +41. (INA 1) +(a) Let $A B C$ be a triangle with $A B=12$ and $A C=16$. Suppose $M$ is the midpoint of side $B C$ and points $E$ and $F$ are chosen on sides $A C$ and $A B$ respectively, and suppose that the lines $E F$ and $A M$ intersect at $G$. If $A E=2 A F$ then find the ratio $E G / G F$. +(b) Let $E$ be a point external to a circle and suppose that two chords $E A B$ and $E C D$ meet at an angle of $40^{\circ}$. If $A B=B C=C D$, find the size of $\angle A C D$. +42. (INA 2) +(a) Four balls of radius 1 are mutually tangent, three resting an the floor and the fourth resting on the others. A tetrahedron, each of whose edges has length $s$, is circumscribed around the balls. Find the value of $s$. +(b) Suppose that $A B C D$ and $E F G H$ are opposite faces of a rectangular solid, with $\angle D H C=45^{\circ}$ and $\angle F H B=60^{\circ}$. Find the cosine of $\angle B H D$. +43. (INA 3) +(a) The polynomial $x^{2 k}+1+(x+1)^{2 k}$ is not divisible by $x^{2}+x+1$. Find the value of $k$. +(b) If $p, q$, and $r$ are distinct roots of $x^{3}-x^{2}+x-2=0$, find the value of $p^{3}+q^{3}+r^{3}$. +(c) If $r$ is the remainder when each of the numbers 1059, 1417, and 2312 is divided by $d$, where $d$ is an integer greater than one, find the value of $d-r$. +(d) What is the smallest positive odd integer $n$ such that the product of $2^{1 / 7}, 2^{3 / 7}, \ldots, 2^{(2 n+1) / 7}$ is greater than $1000 ?$ +44. (INA 4) +(a) Let $g(x)=x^{5}+x^{4}+x^{3}+x^{2}+x+1$. What is the remainder when the polynomial $g\left(x^{12}\right)$ is divided by the polynomial $g(x)$ ? +(b) If $k$ is a positive integer and $f$ is a function such that for every positive number $x, f\left(x^{2}+1\right)^{\sqrt{x}}=k$, find the value of $f\left(\frac{9+y^{2}}{y^{2}}\right)^{\sqrt{12 / y}}$ for every positive number $y$. +(c) The function $f$ satisfies the functional equation $f(x)+f(y)=f(x+$ $y)-x y-1$ for every pair $x, y$ of real numbers. If $f(1)=1$, find the number of integers $n$ for which $f(n)=n$. +45. (INA 5) +(a) Consider a circle $K$ with diameter $A B$, a circle $L$ tangent to $A B$ and to $K$, and a circle $M$ tangent to circle $K$, circle $L$, and $A B$. Calculate the ratio of the area of circle $K$ to the area of circle $M$. +(b) In triangle $A B C, A B=A C$ and $\measuredangle C A B=80^{\circ}$. If points $D, E$, and $F$ lie on sides $B C, A C$, and $A B$, respectively, and $C E=C D$ and $B F=B D$, find the measure of $\measuredangle E D F$. +46. (INA 6) +(a) Calculate $x=\frac{(11+6 \sqrt{2}) \sqrt{11-6 \sqrt{2}}-(11-6 \sqrt{2}) \sqrt{11+6 \sqrt{2}}}{(\sqrt{\sqrt{5}+2}+\sqrt{\sqrt{5}-2})-(\sqrt{\sqrt{5}+1})}$. +(b) For each positive number $x$, let $k=\frac{(x+1 / x)^{6}-\left(x^{6}+1 / x^{6}\right)-2}{(x+1 / x)^{3}+\left(x^{3}+1 / x^{3}\right)}$. Calculate the minimum value of $k$. +47. (IRE 1) (SL88-16). +48. (IRE 2) Find all plane triangles whose sides have integer length and whose incircles have unit radius. +49. (IRE 3) Let $-15$ is a prime, then $n$ divides $g(n)(g(n)+1)$. +51. (ISR 1) Let $A_{1}, A_{2}, \ldots, A_{29}$ be 29 different sequences of positive integers. For $1 \leq i200$. +52. (ISR 2) (SL88-17). +53. (KOR 1) Let $x=p, y=q, z=r, w=s$ be the unique solution of the system of linear equations + +$$ +x+a_{i} y+a_{i}^{2} z+a_{i}^{3} w=a_{i}^{4}, \quad i=1,2,3,4 +$$ + +Express the solution of the following system in terms of $p, q, r$, and $s$ : + +$$ +x+a_{i}^{2} y+a_{i}^{4} z+a_{i}^{6} w=a_{i}^{8}, \quad i=1,2,3,4 +$$ + +Assume the uniqueness of the solution. +54. (KOR 2) (SL88-22). +55. (KOR 3) Find all positive integers $x$ such that the product of all digits of $x$ is given by $x^{2}-10 x-22$. +56. (KOR 4) The Fibonacci sequence is defined by + +$$ +a_{n+1}=a_{n}+a_{n-1} \quad(n \geq 1), \quad a_{0}=0, a_{1}=a_{2}=1 +$$ + +Find the greatest common divisor of the 1960th and 1988th terms of the Fibonacci sequence. +57. (KOR 5) Let $C$ be a cube with edges of length 2. Construct a solid with fourteen faces by cutting off all eight corners of $C$, keeping the new faces perpendicular to the diagonals of the cube and keeping the newly formed faces identical. If at the conclusion of this process the fourteen faces so formed have the same area, find the area of each face of the new solid. +58. (KOR 6) For each pair of positive integers $k$ and $n$, let $S_{k}(n)$ be the base- $k$ digit sum of $n$. Prove that there are at most two primes $p$ less than 20,000 for which $S_{31}(p)$ is a composite number. +59. (LUX 1) (SL88-18). +60. (MEX 1) (SL88-19). +61. (MEX 2) Prove that the numbers $A, B$, and $C$ are equal, where we define $A$ as the number of ways that we can cover a $2 \times n$ rectangle with $2 \times 1$ rectangles, $B$ as the number of sequences of ones and twos that add up to $n$, and $C$ as + +$$ +\begin{cases}\binom{m}{0}+\binom{m+1}{2}+\cdots+\binom{2 m}{2 m} & \text { if } n=2 m \\ \binom{m+1}{1}+\binom{m+2}{3}+\cdots+\binom{2 m+1}{2 m+1} & \text { if } n=2 m+1\end{cases} +$$ + +62. (MON 1) The positive integer $n$ has the property that in any set of $n$ integers chosen from the integers $1,2, \ldots, 1988$, twenty-nine of them form an arithmetic progression. Prove that $n>1788$. +63. (MON 2) Let $A B C D$ be a quadrilateral. Let $A^{\prime} B C D^{\prime}$ be the reflection of $A B C D$ in $B C$, while $A^{\prime \prime} B^{\prime} C D^{\prime}$ is the reflection of $A^{\prime} B C D^{\prime}$ in $C D^{\prime}$ and $A^{\prime \prime} B^{\prime \prime} C^{\prime} D^{\prime}$ is the reflection of $A^{\prime \prime} B^{\prime} C D^{\prime}$ in $D^{\prime} A^{\prime \prime}$. Show that if the lines $A A^{\prime \prime}$ and $B B^{\prime \prime}$ are parallel, then $A B C D$ is a cyclic quadrilateral. +64. (MON 3) Given $n$ points $A_{1}, A_{2}, \ldots, A_{n}$, no three collinear, show that the $n$-gon $A_{1} A_{2} \ldots A_{n}$ can be inscribed in a circle if and only if + +$$ +\begin{aligned} +& A_{1} A_{2} \cdot A_{3} A_{n} \cdots A_{n-1} A_{n}+A_{2} A_{3} \cdot A_{4} A_{n} \cdots A_{n-1} A_{n} \cdot A_{1} A_{n}+\cdots \\ +& \quad+A_{n-1} A_{n-2} \cdot A_{1} A_{n} \cdots A_{n-3} A_{n}=A_{1} A_{n-1} \cdot A_{2} A_{n} \cdots A_{n-2} A_{n} . +\end{aligned} +$$ + +65. (MON 4) (SL88-20). +66. (MON 5) Suppose $\alpha_{i}>0, \beta_{i}>0$ for $1 \leq i \leq n(n>1)$ and that $\sum_{i=1}^{n} \alpha_{i}=\sum_{i=1}^{n} \beta_{i}=\pi$. Prove that + +$$ +\sum_{i=1}^{n} \frac{\cos \beta_{i}}{\sin \alpha_{i}} \leq \sum_{i=1}^{n} \cot \alpha_{i} +$$ + +67. (NET 1) Given a set of 1988 points in the plane, no three points of the set collinear, the points of a subset with 1788 points are colored blue, and the remaining 200 are colored red. Prove that there exists a line in the plane such that each of the two parts into which the line divides the plane contains 894 blue points and 100 red points. +68. (NET 2) Let $S$ be the set of all sequences $\left\{a_{i} \mid 1 \leq i \leq 7, a_{i}=0\right.$ or 1$\}$. The distance between two elements $\left\{a_{i}\right\}$ and $\left\{b_{i}\right\}$ of $S$ is defined as $\sum_{i=1}^{7}\left|a_{i}-b_{i}\right|$. Let $T$ be a subset of $S$ in which any two elements have a distance apart greater than or equal to 3 . Prove that $T$ contains at most 16 elements. Give an example of such a subset with 16 elements. +69. (POL 1) For a convex polygon $P$ in the plane let $P^{\prime}$ denote the convex polygon with vertices at the midpoints of the sides of $P$. Given an integer $n \geq 3$, determine sharp bounds for the ratio $\frac{\operatorname{area}\left(P^{\prime}\right)}{\operatorname{area}(P)}$ over all convex $n$-gons $P$. +70. (POL 2) In 3-dimensional space a point $O$ is given and a finite set $A$ of segments with the sum of the lengths equal to 1988. Prove that there exists a plane disjoint from $A$ such that the distance from it to $O$ does not exceed 574. +71. (POL 3) Given integers $a_{1}, \ldots, a_{10}$, prove that there exists a nonzero sequence $\left(x_{1}, \ldots, x_{10}\right)$ such that all $x_{i}$ belong to $\{-1,0,1\}$ and the number $\sum_{i=1}^{10} x_{i} a_{i}$ is divisible by 1001. +72. (POL 4) (SL88-21). +73. (SIN 1) In a group of $n$ people each one knows exactly three others. They are seated around a table. We say that the seating is perfect if everyone knows the two sitting by their sides. Show that if there is a perfect seating $S$ for the group, then there is always another perfect seating that cannot be obtained from $S$ by rotation or reflection. +74. (SIN 2) (SL88-23). +75. (SPA 1) Let $A B C$ be a triangle with inradius $r$ and circumradius $R$. Show that + +$$ +\sin \frac{A}{2} \sin \frac{B}{2}+\sin \frac{B}{2} \sin \frac{C}{2}+\sin \frac{C}{2} \sin \frac{A}{2} \leq \frac{5}{8}+\frac{r}{4 R} +$$ + +76. (SPA 2) The quadrilateral $A_{1} A_{2} A_{3} A_{4}$ is cyclic and its sides are $a_{1}=$ $A_{1} A_{2}, a_{2}=A_{2} A_{3}, a_{3}=A_{3} A_{4}$, and $a_{4}=A_{4} A_{1}$. The respective circles +with centers $I_{i}$ and radii $\rho_{i}$ are tangent externally to each side $a_{i}$ and to the sides $a_{i+1}$ and $a_{i-1}$ extended $\left(a_{0}=a_{4}\right)$. Show that + +$$ +\prod_{i=1}^{4} \frac{a_{i}}{\rho_{i}}=4\left(\csc A_{1}+\csc A_{2}\right)^{2} +$$ + +77. (SPA 3) Consider $h+1$ chessboards. Number the squares of each board from 1 to 64 in such a way that when the perimeters of any two boards of the collection are brought into coincidence in any possible manner, no two squares in the same position have the same number. What is the maximum value of $h$ ? +78. (SWE 1) A two-person game is played with nine boxes arranged in a $3 \times 3$ square, initially empty, and with white and black stones. At each move a player puts three stones, not necessarily of the same color, in three boxes in either a horizontal or a vertical row. No box can contain stones of different colors: If, for instance, a player puts a white stone in a box containing black stones, the white stone and one of the black stones are removed from the box. The game is over when the center box and the corner boxes each contain one black stone and the other boxes are empty. At one stage of the game $x$ boxes contained one black stone each and the other boxes were empty. Determine all possible values of $x$. +79. (SWE 2) (SL88-24). +80. (SWE 3) Let $S$ be an infinite set of integers containing zero and such that the distance between successive numbers never exceeds a given fixed number. Consider the following procedure: Given a set $X$ of integers, we construct a new set consisting of all numbers $x \pm s$, where $x$ belongs to $X$ and $s$ belongs to $S$. +Starting from $S_{0}=\{0\}$ we successively construct sets $S_{1}, S_{2}, S_{3}, \ldots$ using this procedure. Show that after a finite number of steps we do not obtain any new sets; i.e., $S_{k}=S_{k_{0}}$ for $k \geq k_{0}$. +81. (USA 1) There are $n \geq 3$ job openings at a factory, ranked 1 to $n$ in order of increasing pay. There are $n$ job applicants, ranked 1 to $n$ in order of increasing ability. Applicant $i$ is qualified for job $j$ if and only if $i \geq j$. The applicants arrive one at a time in random order. Each in turn is hired to the highest-ranking job for which he or she is qualified and that is lower in rank than any job already filled. (Under these rules, job 1 is always filled and hiring terminates thereafter.) +Show that applicants $n$ and $n-1$ have the same probability of being hired. +82. (USA 2) The triangle $A B C$ has a right angle at $C$. The point $P$ is located on segment $A C$ such that triangles $P B A$ and $P B C$ have congruent inscribed circles. Express the length $x=P C$ in terms of $a=B C, b=C A$, and $c=A B$. +83. (USA 3) (SL88-29). +84. (USS 1) (SL88-30). +85. (USS 2) (SL88-31). +86. (USS 3) Let $a, b, c$ be integers different from zero. It is known that the equation $a x^{2}+b y^{2}+c z^{2}=0$ has a solution $(x, y, z)$ in integers different from the solution $x=y=z=0$. Prove that the equation $a x^{2}+b y^{2}+c z^{2}=$ 1 has a solution in rational numbers. +87. (USS 4) All the irreducible positive rational numbers such that the product of the numerator and the denominator is less than 1988 are written in increasing order. Prove that any two adjacent fractions $a / b$ and $c / d$, $a / b\sqrt{3 \cos \frac{\pi}{7}} +$$ + +92. (VIE 4) Let $p \geq 2$ be a natural number. Prove that there exists an integer $n_{0}$ such that + +$$ +\sum_{i=1}^{n_{0}} \frac{1}{i \sqrt[p]{i+1}}>p +$$ + +93. (VIE 5) Given a natural number $n$, find all polynomials $P(x)$ of degree less than $n$ satisfying the following condition: + +$$ +\sum_{i=0}^{n} P(i)(-1)^{i}\binom{n}{i}=0 +$$ + +94. (VIE 6) Let $n+1(n \geq 1)$ positive integers be given such that for each integer, the set of all prime numbers dividing this integer is a subset of +the set of $n$ given prime numbers. Prove that among these $n+1$ integers one can find numbers (possibly one number) whose product is a perfect square. + +### 3.29.3 Shortlisted Problems + +1. (BUL 1) An integer sequence is defined by + +$$ +a_{n}=2 a_{n-1}+a_{n-2} \quad(n>1), \quad a_{0}=0, \quad a_{1}=1 +$$ + +Prove that $2^{k}$ divides $a_{n}$ if and only if $2^{k}$ divides $n$. +2. (BUL 3) Let $n$ be a positive integer. Find the number of odd coefficients of the polynomial + +$$ +u_{n}(x)=\left(x^{2}+x+1\right)^{n} +$$ + +3. (CAN 1) The triangle $A B C$ is inscribed in a circle. The interior bisectors of the angles $A, B$, and $C$ meet the circle again at $A^{\prime}, B^{\prime}$, and $C^{\prime}$ respectively. Prove that the area of triangle $A^{\prime} B^{\prime} C^{\prime}$ is greater than or equal to the area of triangle $A B C$. +4. (CZS 1) An $n \times n$ chessboard $(n \geq 2)$ is numbered by the numbers $1,2, \ldots, n^{2}$ (every number occurs once). Prove that there exist two neighboring (which share a common edge) squares such that their numbers differ by at least $n$. +5. (CZS 2) ${ }^{\mathrm{IMO} 2}$ Let $n$ be an even positive integer. Let $A_{1}, A_{2}, \ldots, A_{n+1}$ be sets having $n$ elements each such that any two of them have exactly one element in common while every element of their union belongs to at least two of the given sets. For which $n$ can one assign to every element of the union one of the numbers 0 and 1 in such a manner that each of the sets has exactly $n / 2$ zeros? +6. (CZS 3) In a given tetrahedron $A B C D$ let $K$ and $L$ be the centers of edges $A B$ and $C D$ respectively. Prove that every plane that contains the line $K L$ divides the tetrahedron into two parts of equal volume. +7. (FRA 2) Let $a$ be the greatest positive root of the equation $x^{3}-3 x^{2}+1=$ 0 . Show that $\left[a^{1788}\right]$ and $\left[a^{1988}\right]$ are both divisible by 17 . ( $[x]$ denotes the integer part of $x$.) +8. (FRA 3) Let $u_{1}, u_{2}, \ldots, u_{m}$ be $m$ vectors in the plane, each of length less than or equal to 1 , which add up to zero. Show that one can rearrange $u_{1}, u_{2}, \ldots, u_{m}$ as a sequence $v_{1}, v_{2}, \ldots, v_{m}$ such that each partial sum $v_{1}, v_{1}+v_{2}, v_{1}+v_{2}+v_{3}, \ldots, v_{1}+v_{2}+\cdots+v_{m}$ has length less than or equal to $\sqrt{5}$. +9. (FRG 1) ${ }^{\text {IMO6 }}$ Let $a$ and $b$ be two positive integers such that $a b+1$ divides $a^{2}+b^{2}$. Show that $\frac{a^{2}+b^{2}}{a b+1}$ is a perfect square. +10. (GDR 1) Let $N=\{1,2, \ldots, n\}, n \geq 2$. A collection $F=\left\{A_{1}, \ldots, A_{t}\right\}$ of subsets $A_{i} \subseteq N, i=1, \ldots, t$, is said to be separating if for every pair $\{x, y\} \subseteq N$, there is a set $A_{i} \in F$ such that $A_{i} \cap\{x, y\}$ contains just one element. A collection $F$ is said to be covering if every element of $N$ is contained in at least one set $A_{i} \in F$. What is the smallest value $f(n)$ of $t$ such that there is a set $F=\left\{A_{1}, \ldots, A_{t}\right\}$ that is simultaneously separating and covering? +11. (GDR 3) The lock on a safe consists of three wheels, each of which may be set in eight different positions. Due to a defect in the safe mechanism the door will open if any two of the three wheels are in the correct position. What is the smallest number of combinations that must be tried if one is to guarantee being able to open the safe (assuming that the "right combination" is not known)? +12. (GRE 2) In a triangle $A B C$, choose any points $K \in B C, L \in A C$, $M \in A B, N \in L M, R \in M K$, and $F \in K L$. If $E_{1}, E_{2}, E_{3}, E_{4}, E_{5}$, $E_{6}$, and $E$ denote the areas of the triangles $A M R, C K R, B K F, A L F$, $B N M, C L N$, and $A B C$ respectively, show that + +$$ +E \geq 8 \sqrt[6]{E_{1} E_{2} E_{3} E_{4} E_{5} E_{6}} +$$ + +Remark. Points $K, L, M, N, R, F$ lie on segments $B C, A C, A B, L M$, $M K, K L$ respectively. +13. (GRE 3) ${ }^{\mathrm{IMO5}}$ In a right-angled triangle $A B C$, let $A D$ be the altitude drawn to the hypotenuse and let the straight line joining the incenters of the triangles $A B D, A C D$ intersect the sides $A B, A C$ at the points $K, L$ respectively. If $E$ and $E_{1}$ denote the areas of the triangles $A B C$ and $A K L$ respectively, show that $\frac{E}{E_{1}} \geq 2$. +14. (HUN 1) For what values of $n$ does there exist an $n \times n$ array of entries $-1,0$, or 1 such that the $2 n$ sums obtained by summing the elements of the rows and the columns are all different? +15. (ICE 1) Let $A B C$ be an acute-angled triangle. Three lines $L_{A}, L_{B}$, and $L_{C}$ are constructed through the vertices $A, B$, and $C$ respectively according to the following prescription: Let $H$ be the foot of the altitude drawn from the vertex $A$ to the side $B C$; let $S_{A}$ be the circle with diameter $A H$; let $S_{A}$ meet the sides $A B$ and $A C$ at $M$ and $N$ respectively, where $M$ and $N$ are distinct from $A$; then $L_{A}$ is the line through $A$ perpendicular to $M N$. The lines $L_{B}$ and $L_{C}$ are constructed similarly. Prove that $L_{A}$, $L_{B}$, and $L_{C}$ are concurrent. +16. (IRE 1) $)^{\mathrm{IMO} 4}$ Show that the solution set of the inequality + +$$ +\sum_{k=1}^{70} \frac{k}{x-k} \geq \frac{5}{4} +$$ + +is a union of disjoint intervals the sum of whose lengths is 1988. +17. (ISR 2) In the convex pentagon $A B C D E$, the sides $B C, C D, D E$ have the same length. Moreover, each diagonal of the pentagon is parallel to a side ( $A C$ is parallel to $D E, B D$ is parallel to $A E$, etc.). Prove that $A B C D E$ is a regular pentagon. +18. (LUX 1) ${ }^{\mathrm{IMO}}$ Consider two concentric circles of radii $R$ and $r(R>r)$ with center $O$. Fix $P$ on the small circle and consider the variable chord $P A$ of the small circle. Points $B$ and $C$ lie on the large circle; $B, P, C$ are collinear and $B C$ is perpendicular to $A P$. +(a) For what value(s) of $\angle O P A$ is the sum $B C^{2}+C A^{2}+A B^{2}$ extremal? +(b) What are the possible positions of the midpoints $U$ of $B A$ and $V$ of $A C$ as $\angle O P A$ varies? +19. (MEX 1) Let $f(n)$ be a function defined on the set of all positive integers and having its values in the same set. Suppose that $f(f(m)+f(n))=m+n$ for all positive integers $n, m$. Find all possible values for $f(1988)$. +20. (MON 4) Find the least natural number $n$ such that if the set $\{1,2, \ldots, n\}$ is arbitrarily divided into two nonintersecting subsets, then one of the subsets contains three distinct numbers such that the product of two of them equals the third. +21. (POL 4) Forty-nine students solve a set of three problems. The score for each problem is a whole number of points from 0 to 7 . Prove that there exist two students $A$ and $B$ such that for each problem, $A$ will score at least as many points as $B$. +22. (KOR 2) Let $p$ be the product of two consecutive integers greater than 2. Show that there are no integers $x_{1}, x_{2}, \ldots, x_{p}$ satisfying the equation + +$$ +\sum_{i=1}^{p} x_{i}^{2}-\frac{4}{4 p+1}\left(\sum_{i=1}^{p} x_{i}\right)^{2}=1 +$$ + +Alternative formulation. Show that there are only two values of $p$ for which there are integers $x_{1}, x_{2}, \ldots, x_{p}$ satisfying the above inequality. +23. (SIN 2) Let $Q$ be the center of the inscribed circle of a triangle $A B C$. Prove that for any point $P$, +$a(P A)^{2}+b(P B)^{2}+c(P C)^{2}=a(Q A)^{2}+b(Q B)^{2}+c(Q C)^{2}+(a+b+c)(Q P)^{2}$, +where $a=B C, b=C A$, and $c=A B$. +24. (SWE 2) Let $\left\{a_{k}\right\}_{1}^{\infty}$ be a sequence of nonnegative real numbers such that $a_{k}-2 a_{k+1}+a_{k+2} \geq 0$ and $\sum_{j=1}^{k} a_{j} \leq 1$ for all $k=1,2, \ldots$. Prove that $0 \leq\left(a_{k}-a_{k+1}\right)<\frac{2}{k^{2}}$ for all $k=1,2, \ldots$. +25. (GBR 1) A positive integer is called a double number if its decimal representation consists of a block of digits, not commencing with 0 , followed immediately by an identical block. For instance, 360360 is a double number, but 36036 is not. Show that there are infinitely many double numbers that are perfect squares. +26. (GBR 2) ${ }^{\mathrm{IMO} 3} \mathrm{~A}$ function $f$ defined on the positive integers (and taking positive integer values) is given by + +$$ +\begin{aligned} +f(1) & =1, \quad f(3)=3 \\ +f(2 n) & =f(n) \\ +f(4 n+1) & =2 f(2 n+1)-f(n) \\ +f(4 n+3) & =3 f(2 n+1)-2 f(n) +\end{aligned} +$$ + +for all positive integers $n$. Determine with proof the number of positive integers less than or equal to 1988 for which $f(n)=n$. +27. (GBR 4) The triangle $A B C$ is acute-angled. Let $L$ be any line in the plane of the triangle and let $u, v, w$ be the lengths of the perpendiculars from $A, B, C$ respectively to $L$. Prove that + +$$ +u^{2} \tan A+v^{2} \tan B+w^{2} \tan C \geq 2 \Delta +$$ + +where $\Delta$ is the area of the triangle, and determine the lines $L$ for which equality holds. +28. (GBR 5) The sequence $\left\{a_{n}\right\}$ of integers is defined by $a_{1}=2, a_{2}=7$, and + +$$ +-\frac{1}{2}1$. +29. (USA 3) A number of signal lights are equally spaced along a one-way railroad track, labeled in order $1,2, \ldots, N(N \geq 2)$. As a safety rule, a train is not allowed to pass a signal if any other train is in motion on the length of track between it and the following signal. However, there is no limit to the number of trains that can be parked motionless at a signal, one behind the other. (Assume that the trains have zero length.) +A series of $K$ freight trains must be driven from Signal 1 to Signal $N$. Each train travels at a distinct but constant speed (i.e., the speed is fixed and different from that of each of the other trains) at all times when it is not blocked by the safety rule. Show that regardless of the order in which the trains are arranged, the same time will elapse between the first train's departure from Signal 1 and the last train's arrival at Signal $N$. +30. (USS 1) A point $M$ is chosen on the side $A C$ of the triangle $A B C$ in such a way that the radii of the circles inscribed in the triangles $A B M$ and $B M C$ are equal. Prove that + +$$ +B M^{2}=\Delta \cot \frac{B}{2} +$$ + +where $\Delta$ is the area of the triangle $A B C$. +31. (USS 2) Around a circular table an even number of persons have a discussion. After a break they sit again around the circular table in a different order. Prove that there are at least two people such that the number of participants sitting between them before and after the break is the same. + +### 3.30 The Thirtieth IMO
Braunschweig-Niedersachen, FR Germany, July 13-24, 1989 + +### 3.30.1 Contest Problems + +First Day (July 18) + +1. Prove that the set $\{1,2, \ldots, 1989\}$ can be expressed as the disjoint union of 17 subsets $A_{1}, A_{2}, \ldots, A_{17}$ such that: +(i) each $A_{i}$ contains the same number of elements; +(ii) the sum of all elements of each $A_{i}$ is the same for $i=1,2, \ldots, 17$. +2. Let $A B C$ be a triangle. The bisector of angle $A$ meets the circumcircle of triangle $A B C$ in $A_{1}$. Points $B_{1}$ and $C_{1}$ are defined similarly. Let $A A_{1}$ meet the lines that bisect the two external angles at $B$ and $C$ in point $A^{0}$. Define $B^{0}$ and $C^{0}$ similarly. If $S_{X_{1} X_{2} \ldots X_{n}}$ denotes the area of the polygon $X_{1} X_{2} \ldots X_{n}$, prove that + +$$ +S_{A^{0} B^{0} C^{0}}=2 S_{A C_{1} B A_{1} C B_{1}} \geq 4 S_{A B C} +$$ + +3. Given a set $S$ in the plane containing $n$ points and satisfying the conditions +(i) no three points of $S$ are collinear, +(ii) for every point $P$ of $S$ there exist at least $k$ points in $S$ that have the same distance to $P$, +prove that the following inequality holds: + +$$ +k<\frac{1}{2}+\sqrt{2 n} +$$ + +Second Day (July 19) +4. The quadrilateral $A B C D$ has the following properties: +(i) $A B=A D+B C$; +(ii) there is a point $P$ inside it at a distance $x$ from the side $C D$ such that $A P=x+A D$ and $B P=x+B C$. +Show that + +$$ +\frac{1}{\sqrt{x}} \geq \frac{1}{\sqrt{A D}}+\frac{1}{\sqrt{B C}} +$$ + +5. For which positive integers $n$ does there exist a positive integer $N$ such that none of the integers $1+N, 2+N, \ldots, n+N$ is the power of a prime number? +6. We consider permutations $\left(x_{1}, \ldots, x_{2 n}\right)$ of the set $\{1, \ldots, 2 n\}$ such that $\left|x_{i}-x_{i+1}\right|=n$ for at least one $i \in\{1, \ldots, 2 n-1\}$. For every natural number $n$, find out whether permutations with this property are more or less numerous than the remaining permutations of $\{1, \ldots, 2 n\}$. + +### 3.30.2 Longlisted Problems + +1. (AUS 1) In the set $S_{n}=\{1,2, \ldots, n\}$ a new multiplication $a * b$ is defined with the following properties: +(i) $c=a * b$ is in $S_{n}$ for any $a \in S_{n}, b \in S_{n}$. +(ii) If the ordinary product $a \cdot b$ is less than or equal to $n$, then $a * b=a \cdot b$. +(iii) The ordinary rules of multiplication hold for $*$, i.e., +(1) $a * b=b * a$ (commutativity) +(2) $(a * b) * c=a *(b * c)$ (associativity) +(3) If $a * b=a * c$ then $b=c$ (cancellation law). + +Find a suitable multiplication table for the new product for $n=11$ and $n=12$. +2. (AUS 2) (SL89-1). +3. (AUS 3) (SL89-2). +4. (AUS 4) (SL89-3). +5. (BUL 1) The sequences $a_{0}, a_{1}, \ldots$ and $b_{0}, b_{1}, \ldots$ are defined by the equalities + +$$ +a_{0}=\frac{\sqrt{2}}{2}, \quad a_{n+1}=\frac{\sqrt{2}}{2} \sqrt{1-\sqrt{1-a_{n}^{2}}}, \quad n=0,1,2, \ldots +$$ + +and + +$$ +b_{0}=1, \quad b_{n+1}=\frac{\sqrt{1+b_{n}^{2}}-1}{b_{n}}, \quad n=0,1,2, \ldots +$$ + +Prove the inequalities + +$$ +2^{n+2} a_{n}<\pi<2^{n+2} b_{n}, \quad \text { for every } n=0,1,2, \ldots +$$ + +6. (BUL 2) The circles $c_{1}$ and $c_{2}$ are tangent at the point $A$. A straight line $l$ through $A$ intersects $c_{1}$ and $c_{2}$ at points $C_{1}$ and $C_{2}$ respectively. A circle $c$, which contains $C_{1}$ and $C_{2}$, meets $c_{1}$ and $c_{2}$ at points $B_{1}$ and $B_{2}$ respectively. Let $\kappa$ be the circle circumscribed around triangle $A B_{1} B_{2}$. The circle $k$ tangent to $\kappa$ at the point $A$ meets $c_{1}$ and $c_{2}$ at the points $D_{1}$ and $D_{2}$ respectively. Prove that +(a) the points $C_{1}, C_{2}, D_{1}, D_{2}$ are concyclic or collinear; +(b) the points $B_{1}, B_{2}, D_{1}, D_{2}$ are concyclic if and only if $A C_{1}$ and $A C_{2}$ are diameters of $c_{1}$ and $c_{2}$. +7. (BUL 3) (SL89-4). +8. (COL 1) (SL89-5). +9. (COL 2) Let $m$ be a positive integer and define $f(m)$ to be the number of factors of 2 in $m$ ! (that is, the greatest positive integer $k$ such that $2^{k} \mid m!$ ). Prove that there are infinitely many positive integers $m$ such that $m-f(m)=1989$. +10. (CUB 1) Given the equation + +$$ +4 x^{3}+4 x^{2} y-15 x y^{2}-18 y^{3}-12 x^{2}+6 x y+36 y^{2}+5 x-10 y=0 +$$ + +find all positive integer solutions. +11. (CUB 2) Given the equation + +$$ +y^{4}+4 y^{2} x-11 y^{2}+4 x y-8 y+8 x^{2}-40 x+52=0 +$$ + +find all real solutions. +12. (CUB 3) Let $P(x)$ be a polynomial such that the following inequalities are satisfied: + +$$ +\begin{aligned} +& P(0)>0 \\ +& P(1)>P(0) \\ +& P(2)>2 P(1)-P(0) \\ +& P(3)>3 P(2)-3 P(1)+P(0) +\end{aligned} +$$ + +and also for every natural number $n, P(n+4)>4 P(n+3)-6 P(n+2)+$ $4 P(n+1)-P(n)$. Prove that for every positive natural number $n, P(n)$ is positive. +13. (CUB 4) Let $n$ be a natural number not greater than 44 . Prove that for any function $f$ defined over $\mathbb{N}^{2}$ whose images are in the set $\{1,2, \ldots, n\}$, there are four ordered pairs $(i, j),(i, k),(l, j)$, and $(l, k)$ such that $f(i, j)=$ $f(i, k)=f(l, j)=f(l, k)$, where $i, j, k, l$ are chosen in such a way that there are natural numbers $n, p$ that satisfy + +$$ +1989 m \leq i0, n>0$. Prove that $c_{m, n}=c_{n, m}$ for all $m \geq 0, n \geq 0$. +28. (GBR 4) Let $b_{1}, b_{2}, \ldots, b_{1989}$ be positive real numbers such that the equations + +$$ +x_{r-1}-2 x_{r}+x_{r+1}+b_{r} x_{r}=0 \quad(1 \leq r \leq 1989) +$$ + +have a solution with $x_{0}=x_{1990}=0$ but not all of $x_{1}, \ldots, x_{1989}$ are equal to zero. Prove that + +$$ +b_{1}+b_{2}+\cdots+b_{1989} \geq \frac{2}{995} +$$ + +29. (GRE 1) Let $L$ denote the set of all lattice points of the plane (points with integral coordinates). Show that for any three points $A, B, C$ of $L$ there is a fourth point $D$, different from $A, B, C$, such that the interiors of the segments $A D, B D, C D$ contain no points of $L$. Is the statement true if one considers four points of $L$ instead of three? +30. (GRE 2) In a triangle $A B C$ for which $6(a+b+c) r^{2}=a b c$, we consider a point $M$ on the inscribed circle and the projections $D, E, F$ of $M$ on the sides $B C, A C$, and $A B$ respectively. Let $S, S_{1}$ denote the areas of the triangles $A B C$ and $D E F$ respectively. Find the maximum and minimum values of the quotient $\frac{S}{S_{1}}$ (here $r$ denotes the inradius of $A B C$ and, as usual, $a=B C, b=A C, c=A B)$. +31. (GRE 3) (SL89-10). +32. (HKG 1) Let $A B C$ be an equilateral triangle. Let $D, E, F, M, N$, and $P$ bee the mid-points of $B C, C A, A B, F D, F B$, and $D C$ respectively. +(a) Show that the line segments $A M, E N$, and $F P$ are concurrent. +(b) Let $O$ be the point of intersection of $A M, E N$, and $F P$. Find $O M$ : $O F: O N: O E: O P: O A$. +33. (HKG 2) Let $n$ be a positive integer. Show that $(\sqrt{2}+1)^{n}=\sqrt{m}+$ $\sqrt{m-1}$ for some positive integer $m$. +34. (HKG 3) Given an acute triangle find a point inside the triangle such that the sum of the distances from this point to the three vertices is the least. +35. (HKG 4) Find all square numbers $S_{1}$ and $S_{2}$ such that $S_{1}-S_{2}=1989$. +36. (HKG 5) Prove the identity + +$$ +1+\frac{1}{2}-\frac{2}{3}+\frac{1}{4}+\frac{1}{5}-\frac{2}{6}+\cdots+\frac{1}{478}+\frac{1}{479}-\frac{2}{480}=2 \sum_{k=0}^{159} \frac{641}{(161+k)(480-k)} +$$ + +37. (HUN 1) (SL89-11). +38. (HUN 2) Connecting the vertices of a regular $n$-gon we obtain a closed (not necessarily convex) $n$-gon. Show that if $n$ is even, then there are two parallel segments among the connecting segments and if $n$ is odd then there cannot be exactly two parallel segments. +39. (HUN 3) (SL89-12). +40. (ICE 1) A sequence of real numbers $x_{0}, x_{1}, x_{2}, \ldots$ is defined as follows: $x_{0}=1989$ and for each $n \geq 1$ + +$$ +x_{n}=-\frac{1989}{n} \sum_{k=0}^{n-1} x_{k} +$$ + +Calculate the value of $\sum_{n=0}^{1989} 2^{n} x_{n}$. +41. (ICE 2) Alice has two urns. Each urn contains four balls and on each ball a natural number is written. She draws one ball from each urn at random, notes the sum of the numbers written on them, and replaces the balls in the urns from which she took them. This she repeats a large number of times. Bill, on examining the numbers recorded, notices that the frequency with which each sum occurs is the same as if it were the sum of two natural numbers drawn at random from the range 1 to 4 . What can he deduce about the numbers on the balls? +42. (ICE 3) (SL89-13). +43. (INA 1) Let $f(x)=a \sin ^{2} x+b \sin x+c$, where $a, b$, and $c$ are real numbers. Find all values of $a, b$, and $c$ such that the following three conditions are satisfied simultaneously: +(i) $f(x)=381$ if $\sin x=1 / 2$. +(ii) The absolute maximum of $f(x)$ is 444 . +(iii) The absolute minimum of $f(x)$ is 364 . +44. (INA 2) Let $A$ and $B$ be fixed distinct points on the $X$ axis, none of which coincides with the origin $O(0,0)$, and let $C$ be a point on the $Y$ axis of an orthogonal Cartesian coordinate system. Let $g$ be a line through the origin $O(0,0)$ and perpendicular to the line $A C$. Find the locus of the point of intersection of the lines $g$ and $B C$ as $C$ varies along the $Y$ axis. (Give an equation and a description of the locus.) +45. (INA 3) The expressions $a+b+c, a b+a c+b c$, and $a b c$ are called the elementary symmetric expressions on the three letters $a, b, c$; symmetric because if we interchange any two letters, say $a$ and $c$, the expressions remain algebraically the same. The common degree of its terms is called the order of the expression. +Let $S_{k}(n)$ denote the elementary expression on $k$ different letters of order $n$; for example $S_{4}(3)=a b c+a b d+a c d+b c d$. There are four terms in $S_{4}(3)$. How many terms are there in $S_{9891}(1989)$ ? (Assume that we have 9891 different letters.) +46. (INA 4) Given two distinct numbers $b_{1}$ and $b_{2}$, their product can be formed in two ways: $b_{1} \times b_{2}$ and $b_{2} \times b_{1}$. Given three distinct numbers, $b_{1}, b_{2}, b_{3}$, their product can be formed in twelve ways: $b_{1} \times\left(b_{2} \times b_{3}\right) ;\left(b_{1} \times\right.$ $\left.b_{2}\right) \times b_{3} ; b_{1} \times\left(b_{3} \times b_{2}\right) ;\left(b_{1} \times b_{3}\right) \times b_{2} ; b_{2} \times\left(b_{1} \times b_{3}\right) ;\left(b_{2} \times b_{1}\right) \times b_{3} ;$ $b_{2} \times\left(b_{3} \times b_{1}\right) ;\left(b_{2} \times b_{3}\right) \times b_{1} ; b_{3} \times\left(b_{1} \times b_{2}\right) ;\left(b_{3} \times b_{1}\right) \times b_{2} ; b_{3} \times\left(b_{2} \times b_{1}\right) ;$ $\left(b_{3} \times b_{2}\right) \times b_{1}$. In how many ways can the product of $n$ distinct letters be formed? +47. (INA 5) Let $\log _{2}^{2} x-4 \log _{2} x-m^{2}-2 m-13=0$ be an equation in $x$. Prove: +(a) For any real value of $m$ the equation has has two distinct solutions. +(b) The product of the solutions of the equation does not depend on $m$. +(c) One of the solutions of the equation is less than 1 , while the other solution is greater than 1. +Find the minimum value of the larger solution and the maximum value of the smaller solution. +48. (INA 6) Let $S$ be the point of intersection of the two lines $l_{1}: 7 x-5 y+$ $8=0$ and $l_{2}: 3 x+4 y-13=0$. Let $P=(3,7), Q=(11,13)$, and let $A$ and $B$ be points on the line $P Q$ such that $P$ is between $A$ and $Q$, and $B$ is between $P$ and $Q$, and such that $P A / A Q=P B / B Q=2 / 3$. Without finding the coordinates of $B$ find the equations of the lines $S A$ and $S B$. +49. (IND 1) Let $A, B$ denote two distinct fixed points in space. Let $X, P$ denote variable points (in space), while $K, N, n$ denote positive integers. Call $(X, K, N, P)$ admissible if $(N-K) P A+K \cdot P B \geq N \cdot P X$. Call $(X, K, N)$ admissible if $(X, K, N, P)$ is admissible for all choices of $P$. Call $(X, N)$ admissible if $(X, K, N)$ is admissible for some choice of $K$ in the interval $01)$. Determine: +(a) the set of admissible $X$; +(b) the set of $X$ for which $(X, 1989)$ is admissible but not $(X, n), n<1989$. +50. (IND 2) (SL89-14). +51. (IND 3) Let $t(n)$, for $n=3,4,5, \ldots$, represent the number of distinct, incongruent, integer-sided triangles whose perimeter is $n$; e.g., $t(3)=1$. Prove that + +$$ +t(2 n-1)-t(2 n)=\left[\frac{n}{6}\right] \text { or }\left[\frac{n}{6}+1\right] +$$ + +52. (IRE 1) (SL89-15). +53. (IRE 2) Let $f(x)=\left(x-a_{1}\right)\left(x-a_{2}\right) \cdots\left(x-a_{n}\right)-2$, where $n \geq 3$ and $a_{1}, a_{2}, \ldots, a_{n}$ are distinct integers. Suppose that $f(x)=g(x) h(x)$, where $g(x), h(x)$ are both nonconstant polynomials with integer coefficients. Prove that $n=3$. +54. (IRE 3) Let $f$ be a function from the real numbers to the real numbers such that $f(1)=1, f(a+b)=f(a)+f(b)$ for all $a, b$, and $f(x) f(1 / x)=1$ for all $x \neq 0$. +Prove that $f(x)=x$ for all real numbers $x$. +55. (IRE 4) Let $[x]$ denote the greatest integer less than or equal to $x$. Let $\alpha$ be the positive root of the equation $x^{2}-1989 x-1=0$. Prove that there exist infinitely many natural numbers $n$ that satisfy the equation + +$$ +[\alpha n+1989 \alpha[\alpha n]]=1989 n+\left(1989^{2}+1\right)[\alpha n] +$$ + +56. (IRE 5) Let $n=2 k-1$, where $k \geq 6$ is an integer. Let $T$ be the set of all $n$-tuples $\left(x_{1}, x_{2}, \ldots, x_{n}\right)$ where $x_{i}$ is 0 or $1(i=1,2, \ldots, n)$. For $\mathbf{x}=\left(x_{1}, \ldots, x_{n}\right)$ and $\mathbf{y}=\left(y_{1}, \ldots, y_{n}\right)$ in $T$, let $d(\mathbf{x}, \mathbf{y})$ denote the number of integers $j$ with $1 \leq j \leq n$ such that $x_{j} \neq y_{j}$. (In particular $d(\mathbf{x}, \mathbf{x})=0$.) + +Suppose that there exists a subset $S$ of $T$ with $2^{k}$ elements that has the following property: Given any element $\mathbf{x}$ in $T$, there is a unique element $\mathbf{y}$ in $S$ with $d(\mathbf{x}, \mathbf{y}) \leq 3$. Prove that $n=23$. +57. (ISR 1) (SL89-16). +58. (ISR 2) Let $P_{1}(x), P_{2}(x), \ldots, P_{n}(x)$ be polynomials with real coefficients. Show that there exist real polynomials $A_{r}(x), B_{r}(x)(r=1,2,3)$ such that + +$$ +\begin{aligned} +\sum_{s=1}^{n}\left(P_{s}(x)\right)^{2} & =\left(A_{1}(x)\right)^{2}+\left(B_{1}(x)\right)^{2} \\ +& =\left(A_{2}(x)\right)^{2}+x\left(B_{2}(x)\right)^{2} \\ +& =\left(A_{3}(x)\right)^{2}-x\left(B_{3}(x)\right)^{2} . +\end{aligned} +$$ + +59. (ISR 3) Let $v_{1}, v_{2}, \ldots, v_{1989}$ be a set of coplanar vectors with $\left|v_{r}\right| \leq 1$ for $1 \leq r \leq 1989$. Show that it is possible to find $\epsilon_{r}(1 \leq r \leq 1989)$, each equal to $\pm 1$, such that + +$$ +\left|\sum_{r=1}^{1989} \epsilon_{r} v_{r}\right| \leq \sqrt{3} +$$ + +60. (KOR 1) A real-valued function $f$ on $\mathbb{Q}$ satisfies the following conditions for arbitrary $\alpha, \beta \in \mathbb{Q}$ : +(i) $f(0)=0$, +(ii) $f(\alpha)>0$ if $\alpha \neq 0$, +(iii) $f(\alpha \beta)=f(\alpha) f(\beta)$, +(iv) $f(\alpha+\beta) \leq f(\alpha)+f(\beta)$, +(v) $f(m) \leq 1989$ for all $m \in \mathbb{Z}$. + +Prove that $f(\alpha+\beta)=\max \{f(\alpha), f(\beta)\}$ if $f(\alpha) \neq f(\beta)$. +Here, $\mathbb{Z}, \mathbb{Q}$ denote the sets of integers and rational numbers, respectively. +61. (KOR 2) Let $A$ be a set of positive integers such that no positive integer greater than 1 divides all the elements of $A$. Prove that any sufficiently large positive integer can be written as a sum of elements of $A$. (Elements may occur several times in the sum.) +62. (KOR 3) (SL89-25). +63. (KOR 4) (SL89-26). +64. (KOR 5) Let a regular $(2 n+1)$-gon be inscribed in a circle of radius $r$. We consider all the triangles whose vertices are from those of the regular $(2 n+1)$-gon. +(a) How many triangles among them contain the center of the circle in their interior? +(b) Find the sum of the areas of all those triangles that contain the center of the circle in their interior. +65. (LUX 1) A regular $n$-gon $A_{1} A_{2} A_{3} \ldots A_{k} \ldots A_{n}$ inscribed in a circle of radius $R$ is given. If $S$ is a point on the circle, calculate $T=S A_{1}^{2}+S A_{2}^{2}+$ $\cdots+S A_{n}^{2}$. +66. (MON 1) (SL89-17). +67. (MON 2) A family of sets $A_{1}, A_{2}, \ldots, A_{n}$ has the following properties: +(i) Each $A_{i}$ contains 30 elements. +(ii) $A_{i} \cap A_{j}$ contains exactly one element for all $i, j, 1 \leq iT_{1989}(2)$. Justify your answer. +87. (POR 6) A balance has a left pan, a right pan, and a pointer that moves along a graduated ruler. Like many other grocer balances, this one works as follows: An object of weight $L$ is placed in the left pan and another of weight $R$ in the right pan, the pointer stops at the number $R-L$ on the graduated ruler. +There are $n(\geq 2)$ bags of coins, each containing $\frac{n(n-1)}{2}+1$ coins. All coins look the same (shape, color, and so on). Of the bags, $n-1$ contain genuine coins, all with the same weight. The remaining bag (we don't know which one it is) contains counterfeit coins. All counterfeit coins have the same weight, and this weight is different from the weight of the genuine coins. A legal weighing consists of placing a certain number of coins in one of the pans, putting a certain number of coins in the other pan, and reading the number given by the pointer in the graduated ruler. With just two legal weighings it is possible to identify the bag containing counterfeit coins. Find a way to do this and explain it. +88. (ROM 1) (SL89-27). +89. (ROM 2) (SL89-28). +90. (ROM 3) Prove that the sequence $\left(a_{n}\right)_{n \geq 0}, a_{n}=[n \sqrt{2}]$, contains an infinite number of perfect squares. +91. (ROM 4) (SL89-29). +92. (ROM 5) Find the set of all $a \in \mathbb{R}$ for which there is no infinite sequence $\left(x_{n}\right)_{n \geq 0} \subset \mathbb{R}$ satisfying $x_{0}=a, x_{n+1}=\frac{x_{n}+\alpha}{\beta x_{n}+1}, n=0,1, \ldots$, where $\alpha \beta>0$. +93. (ROM 6) For $\Phi: \mathbb{N} \rightarrow \mathbb{Z}$ let us define $M_{\Phi}=\{f: \mathbb{N} \rightarrow \mathbb{Z} ; f(x)>$ $F(\Phi(x)), \forall x \in \mathbb{N}\}$. +(a) Prove that if $M_{\Phi_{1}}=M_{\Phi_{2}} \neq \emptyset$, then $\Phi_{1}=\Phi_{2}$. +(b) Does this property remain true if $M_{\Phi}=\{f: \mathbb{N} \rightarrow \mathbb{N} ; f(x)>$ $F(\Phi(x)), \forall x \in \mathbb{N}\}$ ? +94. (SWE 1) Prove that $a0$ and a complex number $z \neq 0$ with $\arg z \neq \pi$, define + +$$ +B(c, z)=\{b \in \mathbb{R}| | w-z|1$ be a fixed integer. Define functions $f_{0}(x)=0$, $f_{1}(x)=1-\cos x$, and for $k>0$, + +$$ +f_{k+1}(x)=2 f_{k}(x) \cos x-f_{k-1}(x) +$$ + +If $F(x)=f_{1}(x)+f_{2}(x)+\cdots+f_{n}(x)$, prove that +(a) $01$ for $\frac{\pi}{n+1}1$ the equation + +$$ +\frac{x^{n}}{n!}+\frac{x^{n-1}}{(n-1)!}+\cdots+\frac{x^{2}}{2!}+\frac{x}{1!}+1=0 +$$ + +has no rational roots. +5. (COL 1) Consider the polynomial $p(x)=x^{n}+n x^{n-1}+a_{2} x^{n-2}+\cdots+a_{n}$ having all real roots. If $r_{1}^{16}+r_{2}^{16}+\cdots+r_{n}^{16}=n$, where the $r_{j}$ are the roots of $p(x)$, find all such roots. +6. (CZS 1) For a triangle $A B C$, let $k$ be its circumcircle with radius $r$. The bisectors of the inner angles $A, B$, and $C$ of the triangle intersect respectively the circle $k$ again at points $A^{\prime}, B^{\prime}$, and $C^{\prime}$. Prove the inequality + +$$ +16 Q^{3} \geq 27 r^{4} P +$$ + +where $Q$ and $P$ are the areas of the triangles $A^{\prime} B^{\prime} C^{\prime}$ and $A B C$ respectively. +7. (FIN 1) Show that any two points lying inside a regular $n$-gon $E$ can be joined by two circular arcs lying inside $E$ and meeting at an angle of at least $\left(1-\frac{2}{n}\right) \pi$. +8. (FRA 2) Let $R$ be a rectangle that is the union of a finite number of rectangles $R_{i}, 1 \leq i \leq n$, satisfying the following conditions: +(i) The sides of every rectangle $R_{i}$ are parallel to the sides of $R$. +(ii) The interiors of any two different $R_{i}$ are disjoint. +(iii) Every $R_{i}$ has at least one side of integral length. Prove that $R$ has at least one side of integral length. +9. (FRA 4) For all integers $n, n \geq 0$, there exist uniquely determined integers $a_{n}, b_{n}, c_{n}$ such that + +$$ +(1+4 \sqrt[3]{2}-4 \sqrt[3]{4})^{n}=a_{n}+b_{n} \sqrt[3]{2}+c_{n} \sqrt[3]{4} +$$ + +Prove that $c_{n}=0$ implies $n=0$. +10. (GRE 3) Let $g: \mathbb{C} \rightarrow \mathbb{C}, w \in \mathbb{C}, a \in \mathbb{C}, w^{3}=1(w \neq 1)$. Show that there is one and only one function $f: \mathbb{C} \rightarrow \mathbb{C}$ such that + +$$ +f(z)+f(w z+a)=g(z), \quad z \in \mathbb{C} +$$ + +Find the function $f$. +11. (HUN 1) Define sequence $a_{n}$ by $\sum_{d \mid n} a_{d}=2^{n}$. Show that $n \mid a_{n}$. +12. (HUN 3) At $n$ distinct points of a circular race course there are $n$ cars ready to start. Each car moves at a constant speed and covers the circle in an hour. On hearing the initial signal, each of them selects a direction and starts moving immediately. If two cars meet, both of them change directions and go on without loss of speed. +Show that at a certain moment each car will be at its starting point. +13. (ICE 3) ${ }^{\mathrm{IMO}}$ The quadrilateral $A B C D$ has the following properties: +(i) $A B=A D+B C$; +(ii) there is a point $P$ inside it at a distance $x$ from the side $C D$ such that $A P=x+A D$ and $B P=x+B C$. +Show that + +$$ +\frac{1}{\sqrt{x}} \geq \frac{1}{\sqrt{A D}}+\frac{1}{\sqrt{B C}} +$$ + +14. (IND 2) A bicentric quadrilateral is one that is both inscribable in and circumscribable about a circle. Show that for such a quadrilateral, the centers of the two associated circles are collinear with the point of intersection of the diagonals. +15. (IRE 1) Let $a, b, c, d, m, n$ be positive integers such that $a^{2}+b^{2}+c^{2}+d^{2}=$ 1989, $a+b+c+d=m^{2}$, and the largest of $a, b, c, d$ is $n^{2}$. Determine, with proof, the values of $m$ and $n$. +16. (ISR 1) The set $\left\{a_{0}, a_{1}, \ldots, a_{n}\right\}$ of real numbers satisfies the following conditions: +(i) $a_{0}=a_{n}=0$; +(ii) for $1 \leq k \leq n-1$, + +$$ +a_{k}=c+\sum_{i=k}^{n-1} a_{i-k}\left(a_{i}+a_{i+1}\right) . +$$ + +Prove that $c \leq \frac{1}{4 n}$. +17. (MON 1) Given seven points in the plane, some of them are connected by segments so that: +(i) among any three of the given points, two are connected by a segment; +(i) the number of segments is minimal. + +How many segments does a figure satisfying (i) and (ii) contain? Give an example of such a figure. +18. (MON 4) Given a convex polygon $A_{1} A_{2} \ldots A_{n}$ with area $S$, and a point $M$ in the same plane, determine the area of polygon $M_{1} M_{2} \ldots M_{n}$, where $M_{i}$ is the image of $M$ under rotation $\mathcal{R}_{A_{i}}^{\alpha}$ around $A_{i}$ by $\alpha, i=1,2, \ldots, n$. +19. (MON 6) A positive integer is written in each square of an $m \times n$ board. The allowed move is to add an integer $k$ to each of two adjacent numbers in such a way that no negative numbers are obtained. (Two squares are adjacent if they have a common side.) Find a necessary and sufficient condition for it to be possible for all the numbers to be zero by a finite sequence of moves. +20. (NET 1) ${ }^{\text {IMO3 }}$ Given a set $S$ in the plane containing $n$ points and satisfying the conditions: +(i) no three points of $S$ are collinear, +(ii) for every point $P$ of $S$ there exist at least $k$ points in $S$ that have the same distance to $P$, +prove that the following inequality holds: + +$$ +k<\frac{1}{2}+\sqrt{2 n} +$$ + +21. (NET 2) Prove that the intersection of a plane and a regular tetrahedron can be an obtuse-angled triangle and that the obtuse angle in any such triangle is always smaller than $120^{\circ}$. +22. (PHI 1) ${ }^{\text {IMO1 }}$ Prove that the set $\{1,2, \ldots, 1989\}$ can be expressed as the disjoint union of 17 subsets $A_{1}, A_{2}, \ldots, A_{17}$ such that: +(i) each $A_{i}$ contains the same number of elements; +(ii) the sum of all elements of each $A_{i}$ is the same for $i=1,2, \ldots, 17$. +23. (POL 2) ${ }^{\mathrm{IMO} 6} \mathrm{We}$ consider permutations $\left(x_{1}, \ldots, x_{2 n}\right)$ of the set $\{1, \ldots$, $2 n\}$ such that $\left|x_{i}-x_{i+1}\right|=n$ for at least one $i \in\{1, \ldots, 2 n-1\}$. For every natural number $n$, find out whether permutations with this property are more or less numerous than the remaining permutations of $\{1, \ldots, 2 n\}$. +24. (POL 5) For points $A_{1}, \ldots, A_{5}$ on the sphere of radius 1 , what is the maximum value that $\min _{1 \leq i, j \leq 5} A_{i} A_{j}$ can take? Determine all configurations for which this maximum is attained. (Or: determine the diameter of any set $\left\{A_{1}, \ldots, A_{5}\right\}$ for which this maximum is attained.) +25. (KOR 3) Let $a, b$ be integers that are not perfect squares. Prove that if + +$$ +x^{2}-a y^{2}-b z^{2}+a b w^{2}=0 +$$ + +has a nontrivial solution in integers, then so does + +$$ +x^{2}-a y^{2}-b z^{2}=0 +$$ + +26. (KOR 4) Let $n$ be a positive integer and let $a, b$ be given real numbers. Determine the range of $x_{0}$ for which + +$$ +\sum_{i=0}^{n} x_{i}=a \quad \text { and } \quad \sum_{i=0}^{n} x_{i}^{2}=b +$$ + +where $x_{0}, x_{1}, \ldots, x_{n}$ are real variables. +27. (ROM 1) Let $m$ be a positive odd integer, $m \geq 2$. Find the smallest positive integer $n$ such that $2^{1989}$ divides $m^{n}-1$. +28. (ROM 2) Consider in a plane $\Pi$ the points $O, A_{1}, A_{2}, A_{3}, A_{4}$ such that $\sigma\left(O A_{i} A_{j}\right) \geq 1$ for all $i, j=1,2,3,4, i \neq j$. Prove that there is at least one pair $i_{0}, j_{0} \in\{1,2,3,4\}$ such that $\sigma\left(O A_{i_{0}} A_{j_{0}}\right) \geq \sqrt{2}$. +(We have denoted by $\sigma\left(O A_{i} A_{j}\right)$ the area of triangle $O A_{i} A_{j}$.) +29. (ROM 4) A flock of 155 birds sit down on a circle $C$. Two birds $P_{i}, P_{j}$ are mutually visible if $m\left(P_{i} P_{j}\right) \leq 10^{\circ}$. Find the smallest number of mutually visible pairs of birds. (One assumes that a position (point) on $C$ can be occupied simultaneously by several birds.) +30. (SWE 2) ${ }^{\mathrm{IMO} 5}$ For which positive integers $n$ does there exist a positive integer $N$ such that none of the integers $1+N, 2+N, \ldots, n+N$ is the power of a prime number? +31. (SWE 3) Let $a_{1} \geq a_{2} \geq a_{3}$ be given positive integers and let $N\left(a_{1}, a_{2}, a_{3}\right)$ be the number of solutions $\left(x_{1}, x_{2}, x_{3}\right)$ of the equation + +$$ +\frac{a_{1}}{x_{1}}+\frac{a_{2}}{x_{2}}+\frac{a_{3}}{x_{3}}=1 +$$ + +where $x_{1}, x_{2}$, and $x_{3}$ are positive integers. Show that + +$$ +N\left(a_{1}, a_{2}, a_{3}\right) \leq 6 a_{1} a_{2}\left(3+\ln \left(2 a_{1}\right)\right) +$$ + +32. (USA 3) The vertex $A$ of the acute triangle $A B C$ is equidistant from the circumcenter $O$ and the orthocenter $H$. Determine all possible values for the measure of angle $A$. + +### 3.31 The Thirty-First IMO Beijing, China, July 8-19, 1990 + +### 3.31.1 Contest Problems + +First Day (July 12) + +1. Given a circle with two chords $A B, C D$ that meet at $E$, let $M$ be a point of chord $A B$ other than $E$. Draw the circle through $D, E$, and $M$. The tangent line to the circle $D E M$ at $E$ meets the lines $B C, A C$ at $F, G$, respectively. Given $\frac{A M}{A B}=\lambda$, find $\frac{G E}{E F}$. +2. On a circle, $2 n-1(n \geq 3)$ different points are given. Find the minimal natural number $N$ with the property that whenever $N$ of the given points are colored black, there exist two black points such that the interior of one of the corresponding arcs contains exactly $n$ of the given $2 n-1$ points. +3. Find all positive integers $n$ having the property that $\frac{2^{n}+1}{n^{2}}$ is an integer. Second Day (July 13) +4. Let $\mathbb{Q}^{+}$be the set of positive rational numbers. Construct a function $f: \mathbb{Q}^{+} \rightarrow \mathbb{Q}^{+}$such that + +$$ +f(x f(y))=\frac{f(x)}{y}, \quad \text { for all } x, y \text { in } \mathbb{Q}^{+} +$$ + +5. Two players $A$ and $B$ play a game in which they choose numbers alternately according to the following rule: At the beginning, an initial natural number $n_{0}>1$ is given. Knowing $n_{2 k}$, player $A$ may choose any $n_{2 k+1} \in \mathbb{N}$ such that + +$$ +n_{2 k} \leq n_{2 k+1} \leq n_{2 k}^{2} +$$ + +Then player $B$ chooses a number $n_{2 k+2} \in \mathbb{N}$ such that + +$$ +\frac{n_{2 k+1}}{n_{2 k+2}}=p^{r} +$$ + +where $p$ is a prime number and $r \in \mathbb{N}$. +It is stipulated that player $A$ wins the game if he (she) succeeds in choosing the number 1990, and player $B$ wins if he (she) succeeds in choosing 1. For which natural numbers $n_{0}$ can player $A$ manage to win the game, for which $n_{0}$ can player $B$ manage to win, and for which $n_{0}$ can players $A$ and $B$ each force a tie? +6. Is there a 1990-gon with the following properties (i) and (ii)? +(i) All angles are equal; +(ii) The lengths of the 1990 sides are a permutation of the numbers $1^{2}, 2^{2}, \ldots, 1989^{2}, 1990^{2}$. + +### 3.31.2 Shortlisted Problems + +1. (AUS 3) The integer 9 can be written as a sum of two consecutive integers: $9=4+5$. Moreover, it can be written as a sum of (more than one) consecutive positive integers in exactly two ways: $9=4+5=2+3+4$. Is there an integer that can be written as a sum of 1990 consecutive integers and that can be written as a sum of (more than one) consecutive positive integers in exactly 1990 ways? +2. (CAN 1) Given $n$ countries with three representatives each, $m$ committees $A(1), A(2), \ldots A(m)$ are called $a$ cycle if +(i) each committee has $n$ members, one from each country; +(ii) no two committees have the same membership; +(iii) for $i=1,2, \ldots, m$, committee $A(i)$ and committee $A(i+1)$ have no member in common, where $A(m+1)$ denotes $A(1)$; +(iv) if $1<|i-j|90^{\circ}$. +6. (FRG 2) ${ }^{\mathrm{IMO} 5}$ Two players $A$ and $B$ play a game in which they choose numbers alternately according to the following rule: At the beginning, an initial natural number $n_{0}>1$ is given. Knowing $n_{2 k}$, player $A$ may choose any $n_{2 k+1} \in \mathbb{N}$ such that + +$$ +n_{2 k} \leq n_{2 k+1} \leq n_{2 k}^{2} +$$ + +Then player $B$ chooses a number $n_{2 k+2} \in \mathbb{N}$ such that + +$$ +\frac{n_{2 k+1}}{n_{2 k+2}}=p^{r} +$$ + +where $p$ is a prime number and $r \in \mathbb{N}$. +It is stipulated that player $A$ wins the game if he (she) succeeds in choosing the number 1990, and player $B$ wins if he (she) succeeds in choosing 1. + +For which natural numbers $n_{0}$ can player $A$ manage to win the game, for which $n_{0}$ can player $B$ manage to win, and for which $n_{0}$ can players $A$ and $B$ each force a tie? +7. (GRE 2) Let $f(0)=f(1)=0$ and + +$$ +f(n+2)=4^{n+2} f(n+1)-16^{n+1} f(n)+n \cdot 2^{n^{2}}, \quad n=0,1,2,3, \ldots +$$ + +Show that the numbers $f(1989), f(1990), f(1991)$ are divisible by 13. +8. (HUN 1) For a given positive integer $k$ denote the square of the sum of its digits by $f_{1}(k)$ and let $f_{n+1}(k)=f_{1}\left(f_{n}(k)\right)$. +Determine the value of $f_{1991}\left(2^{1990}\right)$. +9. (HUN 3) The incenter of the triangle $A B C$ is $K$. The midpoint of $A B$ is $C_{1}$ and that of $A C$ is $B_{1}$. The lines $C_{1} K$ and $A C$ meet at $B_{2}$, the lines $B_{1} K$ and $A B$ at $C_{2}$. If the areas of the triangles $A B_{2} C_{2}$ and $A B C$ are equal, what is the measure of angle $\angle C A B$ ? +10. (ICE 2) A plane cuts a right circular cone into two parts. The plane is tangent to the circumference of the base of the cone and passes through the midpoint of the altitude. Find the ratio of the volume of the smaller part to the volume of the whole cone. +11. (IND $\left.3^{\prime}\right)^{\mathrm{IMO}}$ Given a circle with two chords $A B, C D$ that meet at $E$, let $M$ be a point of chord $A B$ other than $E$. Draw the circle through $D, E$, and $M$. The tangent line to the circle $D E M$ at $E$ meets the lines $B C, A C$ at $F, G$, respectively. Given $\frac{A M}{A B}=\lambda$, find $\frac{G E}{E F}$. +12. (IRE 1) Let $A B C$ be a triangle and $L$ the line through $C$ parallel to the side $A B$. Let the internal bisector of the angle at $A$ meet the side $B C$ at $D$ and the line $L$ at $E$ and let the internal bisector of the angle at $B$ meet the side $A C$ at $F$ and the line $L$ at $G$. If $G F=D E$, prove that $A C=B C$. +13. (IRE 2) An eccentric mathematician has a ladder with $n$ rungs that he always ascends and descends in the following way: When he ascends, each step he takes covers $a$ rungs of the ladder, and when he descends, each step he takes covers $b$ rungs of the ladder, where $a$ and $b$ are fixed positive integers. By a sequence of ascending and descending steps he can climb from ground level to the top rung of the ladder and come back down to ground level again. Find, with proof, the minimum value of $n$, expressed in terms of $a$ and $b$. +14. (JAP 2) In the coordinate plane a rectangle with vertices $(0,0),(m, 0)$, $(0, n),(m, n)$ is given where both $m$ and $n$ are odd integers. The rectangle is partitioned into triangles in such a way that +(i) each triangle in the partition has at least one side (to be called a "good" side) that lies on a line of the form $x=j$ or $y=k$, where $j$ and $k$ are integers, and the altitude on this side has length 1 ; +(ii) each "bad" side (i.e., a side of any triangle in the partition that is not a "good" one) is a common side of two triangles in the partition. +Prove that there exist at least two triangles in the partition each of which has two good sides. +15. (MEX 2) Determine for which positive integers $k$ the set + +$$ +X=\{1990,1990+1,1990+2, \ldots, 1990+k\} +$$ + +can be partitioned into two disjoint subsets $A$ and $B$ such that the sum of the elements of $A$ is equal to the sum of the elements of $B$. +16. (NET 1) ${ }^{\text {IMO6 }}$ Is there a 1990-gon with the following properties (i) and (ii)? +(i) All angles are equal; +(ii) The lengths of the 1990 sides are a permutation of the numbers $1^{2}, 2^{2}, \ldots, 1989^{2}, 1990^{2}$. +17. (NET 3) Unit cubes are made into beads by drilling a hole through them along a diagonal. The beads are put on a string in such a way that they can move freely in space under the restriction that the vertices of two neighboring cubes are touching. Let $A$ be the beginning vertex and $B$ be the end vertex. Let there be $p \times q \times r$ cubes on the string $(p, q, r \geq 1)$. +(a) Determine for which values of $p, q$, and $r$ it is possible to build a block with dimensions $p, q$, and $r$. Give reasons for your answers. +(b) The same question as (a) with the extra condition that $A=B$. +18. (NOR) Let $a, b$ be natural numbers with $1 \leq a \leq b$, and $M=\left[\frac{a+b}{2}\right]$. Define the function $f: \mathbb{Z} \rightarrow \mathbb{Z}$ by + +$$ +f(n)= \begin{cases}n+a, & \text { if } n6$ and let $a_{1}1$, construct an infinite and bounded sequence $x_{0}, x_{1}, x_{2}, \ldots$ such that for all natural numbers $i$ and $j, i \neq j$, the following inequality holds: + +$$ +\left|x_{i}-x_{j} \| i-j\right|^{a} \geq 1 +$$ + +### 3.32.2 Shortlisted Problems + +1. (PHI 3) Let $A B C$ be any triangle and $P$ any point in its interior. Let $P_{1}, P_{2}$ be the feet of the perpendiculars from $P$ to the two sides $A C$ and $B C$. Draw $A P$ and $B P$, and from $C$ drop perpendiculars to $A P$ and $B P$. Let $Q_{1}$ and $Q_{2}$ be the feet of these perpendiculars. Prove that the lines $Q_{1} P_{2}, Q_{2} P_{1}$, and $A B$ are concurrent. +2. (JAP 5) For an acute triangle $A B C, M$ is the midpoint of the segment $B C, P$ is a point on the segment $A M$ such that $P M=B M, H$ is the foot of the perpendicular line from $P$ to $B C, Q$ is the point of intersection of segment $A B$ and the line passing through $H$ that is perpendicular to $P B$, and finally, $R$ is the point of intersection of the segment $A C$ and the line passing through $H$ that is perpendicular to $P C$. +Show that the circumcircle of $\triangle Q H R$ is tangent to the side $B C$ at point $H$. +3. (PRK 1) Let $S$ be any point on the circumscribed circle of $\triangle P Q R$. Then the feet of the perpendiculars from $S$ to the three sides of the triangle lie on the same straight line. Denote this line by $l(S, P Q R)$. Suppose that the hexagon $A B C D E F$ is inscribed in a circle. Show that the four lines $l(A, B D F), l(B, A C E), l(D, A B F)$, and $l(E, A B C)$ intersect at one point if and only if $C D E F$ is a rectangle. +4. (FRA 2) ${ }^{\mathrm{IMO5}}$ Let $A B C$ be a triangle and $M$ an interior point in $A B C$. Show that at least one of the angles $\measuredangle M A B, \measuredangle M B C$, and $\measuredangle M C A$ is less than or equal to $30^{\circ}$. +5. (SPA 4) In the triangle $A B C$, with $\measuredangle A=60^{\circ}$, a parallel $I F$ to $A C$ is drawn through the incenter $I$ of the triangle, where $F$ lies on the side $A B$. The point $P$ on the side $B C$ is such that $3 B P=B C$. Show that $\measuredangle B F P=\measuredangle B / 2$. +6. (USS 4) ${ }^{\mathrm{IMO}}$ Prove for each triangle $A B C$ the inequality + +$$ +\frac{1}{4}<\frac{I A \cdot I B \cdot I C}{l_{A} l_{B} l_{C}} \leq \frac{8}{27} +$$ + +where $I$ is the incenter and $l_{A}, l_{B}, l_{C}$ are the lengths of the angle bisectors of $A B C$. +7. (CHN 2) Let $O$ be the center of the circumsphere of a tetrahedron $A B C D$. Let $L, M, N$ be the midpoints of $B C, C A, A B$ respectively, and assume that $A B+B C=A D+C D, B C+C A=B D+A D$, and $C A+A B=$ $C D+B D$. Prove that $\angle L O M=\angle M O N=\angle N O L$. +8. (NET 1) Let $S$ be a set of $n$ points in the plane. No three points of $S$ are collinear. Prove that there exists a set $P$ containing $2 n-5$ points satisfying the following condition: In the interior of every triangle whose three vertices are elements of $S$ lies a point that is an element of $P$. +9. (FRA 3) In the plane we are given a set $E$ of 1991 points, and certain pairs of these points are joined with a path. We suppose that for every point of $E$, there exist at least 1593 other points of $E$ to which it is joined by a path. Show that there exist six points of $E$ every pair of which are joined by a path. +Alternative version. Is it possible to find a set $E$ of 1991 points in the plane and paths joining certain pairs of the points in $E$ such that every +point of $E$ is joined with a path to at least 1592 other points of $E$, and in every subset of six points of $E$ there exist at least two points that are not joined? +10. (USA 5) ${ }^{\mathrm{IMO} 4}$ Suppose $G$ is a connected graph with $n$ edges. Prove that it is possible to label the edges of $G$ from 1 to $n$ in such a way that in every vertex $v$ of $G$ with two or more incident edges, the set of numbers labeling those edges has no common divisor greater than 1. +11. (AUS 4) Prove that + +$$ +\sum_{m=0}^{995} \frac{(-1)^{m}}{1991-m}\binom{1991-m}{m}=\frac{1}{1991} +$$ + +12. $(\mathbf{C H N} 3)^{\mathrm{IMO} 3}$ Let $S=\{1,2,3, \ldots, 280\}$. Find the minimal natural number $n$ such that in any $n$-element subset of $S$ there are five numbers that are pairwise relatively prime. +13. (POL 4) Given any integer $n \geq 2$, assume that the integers $a_{1}, a_{2}, \ldots, a_{n}$ are not divisible by $n$ and, moreover, that $n$ does not divide $a_{1}+a_{2}+$ $\cdots+a_{n}$. Prove that there exist at least $n$ different sequences $\left(e_{1}, e_{2}, \cdots, e_{n}\right)$ consisting of zeros or ones such that $e_{1} a_{1}+e_{2} a_{2}+\cdots+e_{n} a_{n}$ is divisible by $n$. +14. (POL 3) Let $a, b, c$ be integers and $p$ an odd prime number. Prove that if $f(x)=a x^{2}+b x+c$ is a perfect square for $2 p-1$ consecutive integer values of $x$, then $p$ divides $b^{2}-4 a c$. +15. (USS 2) Let $a_{n}$ be the last nonzero digit in the decimal representation of the number $n$ !. Does the sequence $a_{1}, a_{2}, \ldots, a_{n}, \ldots$ become periodic after a finite number of terms? +16. (ROM 1) ${ }^{\mathrm{IMO} 2}$ Let $n>6$ and $a_{1}1$, construct an infinite and bounded sequence $x_{0}, x_{1}, x_{2}, \ldots$ such that for all natural numbers $i$ and $j, i \neq j$, the following inequality holds: + +$$ +\left|x_{i}-x_{j}\right||i-j|^{a} \geq 1 +$$ + +29. (FIN 2) We call a set $S$ on the real line $\mathbb{R}$ superinvariant if for any stretching $A$ of the set by the transformation taking $x$ to $A(x)=x_{0}+$ $a\left(x-x_{0}\right)$ there exists a translation $B, B(x)=x+b$, such that the images of $S$ under $A$ and $B$ agree; i.e., for any $x \in S$ there is a $y \in S$ such that $A(x)=B(y)$ and for any $t \in S$ there is a $u \in S$ such that $B(t)=A(u)$. Determine all superinvariant sets. +Remark. It is assumed that $a>0$. +30. (BUL 3) Two students $A$ and $B$ are playing the following game: Each of them writes down on a sheet of paper a positive integer and gives the sheet to the referee. The referee writes down on a blackboard two integers, one of which is the sum of the integers written by the players. After that, the referee asks student $A$ : "Can you tell the integer written by the other student?" If $A$ answers "no," the referee puts the same question to student $B$. If $B$ answers "no," the referee puts the question back to $A$, and so on. Assume that both students are intelligent and truthful. Prove that after a finite number of questions, one of the students will answer "yes." + +### 3.33 The Thirty-Third IMO Moscow, Russia, July 10-21, 1992 + +### 3.33.1 Contest Problems + +First Day (July 15) + +1. Find all integer triples $(p, q, r)$ such that $11$ and $\frac{1}{x}+\frac{1}{y}+\frac{1}{z}=2$, then + +$$ +\sqrt{x+y+z} \geq \sqrt{x-1}+\sqrt{y-1}+\sqrt{z-1} +$$ + +22. (GBR 2) (SL92-21). +23. (HKG 1) An Egyptian number is a positive integer that can be expressed as a sum of positive integers, not necessarily distinct, such that the sum of their reciprocals is 1 . For example, $32=2+3+9+18$ is Egyptian because $\frac{1}{2}+\frac{1}{3}+\frac{1}{9}+\frac{1}{18}=1$. Prove that all integers greater than 23 are Egyptian. +24. (ICE 1) Let $\mathbb{Q}^{+}$denote the set of nonnegative rational numbers. Show that there exists exactly one function $f: \mathbb{Q}^{+} \rightarrow \mathbb{Q}^{+}$satisfying the following conditions: +(i) if $0m \quad\left(x_{k_{1}}=x_{k_{7}}\right) +$$ +50. (MON 3) Let $N$ be a point inside the triangle $A B C$. Through the midpoints of the segments $A N, B N$, and $C N$ the lines parallel to the opposite sides of $\triangle A B C$ are constructed. Let $A_{N}, B_{N}$, and $C_{N}$ be the intersection points of these lines. If $N$ is the orthocenter of the triangle $A B C$, prove that the nine-point circles of $\triangle A B C$ and $\triangle A_{N} B_{N} C_{N}$ coincide. +Remark. The statement of the original problem was that the nine-point circles of the triangles $A_{N} B_{N} C_{N}$ and $A_{M} B_{M} C_{M}$ coincide, where $N$ and $M$ are the orthocenter and the centroid of $\triangle A B C$. This statement is false. +51. (NET 1) (SL92-12). +52. (NET 2) Let $n$ be an integer $>1$. In a circular arrangement of $n$ lamps $L_{0}, \ldots, L_{n-1}$, each one of which can be either ON or OFF, we start with the situation that all lamps are ON, and then carry out a sequence of steps, Step $_{0}, S_{\text {Step }}^{1}, \ldots \ldots$ If $L_{j-1}(j$ is taken $\bmod n)$ is ON, then $S_{t e p}^{j}$ changes the status of $L_{j}$ (it goes from ON to OFF or from OFF to ON) but does not change the status of any of the other lamps. If $L_{j-1}$ is OFF , then Step $_{j}$ does not change anything at all. Show that: +(a) There is a positive integer $M(n)$ such that after $M(n)$ steps all lamps are ON again. +(b) If $n$ has the form $2^{k}$, then all lamps are ON after $n^{2}-1$ steps. +(c) If $n$ has the form $2^{k}+1$, then all lamps are ON after $n^{2}-n+1$ steps. +53. (NZL 1) (SL92-13). +54. (POL 1) Suppose that $n>m \geq 1$ are integers such that the string of digits 143 occurs somewhere in the decimal representation of the fraction $m / n$. Prove that $n>125$ +55. (POL 2) (SL92-14). +56. (POL 3) A directed graph (any two distinct vertices joined by at most one directed line) has the following property: If $x, u$, and $v$ are three distinct vertices such that $x \rightarrow u$ and $x \rightarrow v$, then $u \rightarrow w$ and $v \rightarrow w$ for some vertex $w$. Suppose that $x \rightarrow u \rightarrow y \rightarrow \cdots \rightarrow z$ is a path of length $n$, that cannot be extended to the right (no arrow goes away from $z$ ). Prove that every path beginning at $x$ arrives after $n$ steps at $z$. +57. (POL 4) For positive numbers $a, b, c$ define $A=(a+b+c) / 3, G=$ $(a b c)^{1 / 3}, H=3 /\left(a^{-1}+b^{-1}+c^{-1}\right)$. Prove that +$$ +\left(\frac{A}{G}\right)^{3} \geq \frac{1}{4}+\frac{3}{4} \cdot \frac{A}{H} +$$ +for every $a, b, c>0$. +58. (POR 1) Let $A B C$ be a triangle. Denote by $a, b$, and $c$ the lengths of the sides opposite to the angles $A, B$, and $C$, respectively. Prove that ${ }^{7}$ +$$ +\frac{b c}{a+b+c}=\frac{\sin A+\sin B+\sin C}{\cos (A / 2) \sin (B / 2) \sin (C / 2)} . +$$ +59. (PRK 1) Let a regular 7-gon $A_{0} A_{1} A_{2} A_{3} A_{4} A_{5} A_{6}$ be inscribed in a circle. Prove that for any two points $P, Q$ on the $\operatorname{arc} A_{0} A_{6}$ the following equality holds: +$$ +\sum_{i=0}^{6}(-1)^{i} P A_{i}=\sum_{i=0}^{6}(-1)^{i} Q A_{i} +$$ +60. (PRK 2) (SL92-15). +61. (PRK 3) There are a board with $2 n \cdot 2 n\left(=4 n^{2}\right)$ squares and $4 n^{2}-1$ cards numbered with different natural numbers. These cards are put one by one on each of the squares. One square is empty. We can move a card to an empty square from one of the adjacent squares (two squares are adjacent if they have a common edge). Is it possible to exchange two cards on two adjacent squares of a column (or a row) in a finite number of movements? +62. (ROM 1) Let $c_{1}, \ldots, c_{n}(n \geq 2)$ be real numbers such that $0 \leq \sum c_{i} \leq n$. Prove that there exist integers $x_{1}, \ldots, x_{n}$ such that $\sum k_{i}=0$ and $1-n \leq$ $c_{i}+n k_{i} \leq n$ for every $i=1, \ldots, n$. +63. (ROM 2) Let $a$ and $b$ be integers. Prove that $\frac{2 a^{2}-1}{b^{2}+2}$ is not an integer. +64. (ROM 3) For any positive integer $n$ consider all representations $n=$ $a_{1}+\cdots+a_{k}$, where $a_{1}>a_{2}>\cdots>a_{k}>0$ are integers such that for all $i \in\{1,2, \ldots, k-1\}$, the number $a_{i}$ is divisible by $a_{i+1}$. Find the longest such representation of the number 1992. +65. (SAF 1) If $A, B, C$, and $D$ are four distinct points in space, prove that there is a plane $P$ on which the orthogonal projections of $A, B, C$, and $D$ form a parallelogram (possibly degenerate). +66. (SPA 1) A circle of radius $\rho$ is tangent to the sides $A B$ and $A C$ of the triangle $A B C$, and its center $K$ is at a distance $p$ from $B C$. +(a) Prove that $a(p-\rho)=2 s(r-\rho)$, where $r$ is the inradius and $2 s$ the perimeter of $A B C$. +(b) Prove that if the circle intersect $B C$ at $D$ and $E$, then +$$ +D E=\frac{4 \sqrt{r r_{1}(\rho-r)\left(r_{1}-\rho\right)}}{\left(r_{1}-r\right)} +$$ +where $r_{1}$ is the exradius corresponding to the vertex $A$. + +[^5]67. (SPA 2) In a triangle, a symmedian is a line through a vertex that is symmetric to the median with the respect to the internal bisector (all relative to the same vertex). In the triangle $A B C$, the median $m_{a}$ meets $B C$ at $A^{\prime}$ and the circumcircle again at $A_{1}$. The symmedian $s_{a}$ meets $B C$ at $M$ and the circumcircle again at $A_{2}$. Given that the line $A_{1} A_{2}$ contains the circumcenter $O$ of the triangle, prove that: +(a) $\frac{A A^{\prime}}{A M}=\frac{b^{2}+c^{2}}{2 b c}$; +(b) $1+4 b^{2} c^{2}=a^{2}\left(b^{2}+c^{2}\right)$. +68. (SPA 3) Show that the numbers $\tan (r \pi / 15)$, where $r$ is a positive integer less than 15 and relatively prime to 15 , satisfy +$$ +x^{8}-92 x^{6}+134 x^{4}-28 x^{2}+1=0 . +$$ +69. (SWE 1) (SL92-17). +70. (THA 1) Let two circles $A$ and $B$ with unequal radii $r$ and $R$, respectively, be tangent internally at the point $A_{0}$. If there exists a sequence of distinct circles $\left(C_{n}\right)$ such that each circle is tangent to both $A$ and $B$, and each circle $C_{n+1}$ touches circle $C_{n}$ at the point $A_{n}$, prove that +$$ +\sum_{n=1}^{\infty}\left|A_{n+1} A_{n}\right|<\frac{4 \pi R r}{R+r} +$$ +71. (THA 2) Let $P_{1}(x, y)$ and $P_{2}(x, y)$ be two relatively prime polynomials with complex coefficients. Let $Q(x, y)$ and $R(x, y)$ be polynomials with complex coefficients and each of degree not exceeding $d$. Prove that there exist two integers $A_{1}, A_{2}$ not simultaneously zero with $\left|A_{i}\right| \leq d+1(i=$ $1,2)$ and such that the polynomial $A_{1} P_{1}(x, y)+A_{2} P_{2}(x, y)$ is coprime to $Q(x, y)$ and $R(x, y)$. +72. (TUR 1) In a school six different courses are taught: mathematics, physics, biology, music, history, geography. The students were required to rank these courses according to their preferences, where equal preferences were allowed. It turned out that: +(i) mathematics was ranked among the most preferred courses by all students; +(ii) no student ranked music among the least preferred ones; +(iii) all students preferred history to geography and physics to biology; and (iv) no two rankings were the same. + +Find the greatest possible value for the number of students in this school. +73. (TUR 2) Let $\left\{A_{n} \mid n=1,2, \ldots\right\}$ be a set of points in the plane such that for each $n$, the disk with center $A_{n}$ and radius $2^{n}$ contains no other point $A_{j}$. For any given positive real numbers $a2$. Let $A_{0}, A_{1}, A_{2}, \ldots$ be a sequence of points on a circle of radius 1 such that the minor arc from $A_{k-1}$ to $A_{k}$ runs clockwise and such that + +$$ +\mu\left(A_{k-1} A_{k}\right)=\frac{4 F_{2 k+1}}{F_{2 k+1}^{2}+1} +$$ + +for $k \geq 1$, where $\mu(X Y)$ denotes the radian measure of the arc $X Y$ in the clockwise direction. What is the limit of the radian measure of arc $A_{0} A_{n}$ as $n$ approaches infinity? +79. (USA 2) (SL92-18). +80. (USA 3) Given a graph with $n$ vertices and a positive integer $m$ that is less than $n$, prove that the graph contains a set of $m+1$ vertices in which the difference between the largest degree of any vertex in the set and the smallest degree of any vertex in the set is at most $m-1$. +81. (USA 4) Suppose that points $X, Y, Z$ are located on sides $B C, C A$, and $A B$, respectively, of $\triangle A B C$ in such a way that $\triangle X Y Z$ is similar to $\triangle A B C$. Prove that the orthocenter of $\triangle X Y Z$ is the circumcenter of $\triangle A B C$. +82. (VIE 1) Let $f(x)=x^{m}+a_{1} x^{m-1}+\cdots+a_{m-1} x+a_{m}$ and $g(x)=$ $x^{n}+b_{1} x^{n-1}+\cdots+b_{n-1}+b_{n}$ be two polynomials with real coefficients such that for each real number $x, f(x)$ is the square of an integer if and only if so is $g(x)$. Prove that if $n+m>0$, then there exists a polynomial $h(x)$ with real coefficients such that $f(x) \cdot g(x)=(h(x))^{2}$. + +### 3.33.3 Shortlisted Problems + +1. (AUS 2) Prove that for any positive integer $m$ there exist an infinite number of pairs of integers $(x, y)$ such that (i) $x$ and $y$ are relatively prime; (ii) $y$ divides $x^{2}+m$; (iii) $x$ divides $y^{2}+m$. +2. (CHN 1) Let $\mathbb{R}^{+}$be the set of all nonnegative real numbers. Given two positive real numbers $a$ and $b$, suppose that a mapping $f: \mathbb{R}^{+} \rightarrow \mathbb{R}^{+}$ satisfies the functional equation + +$$ +f(f(x))+a f(x)=b(a+b) x +$$ + +Prove that there exists a unique solution of this equation. +3. (CHN 2) The diagonals of a quadrilateral $A B C D$ are perpendicular: $A C \perp B D$. Four squares, $A B E F, B C G H, C D I J, D A K L$, are erected externally on its sides. The intersection points of the pairs of straight lines $C L, D F ; D F, A H ; A H, B J ; B J, C L$ are denoted by $P_{1}, Q_{1}, R_{1}, S_{1}$, respectively, and the intersection points of the pairs of straight lines $A I, B K$; $B K, C E ; C E, D G ; D G, A I$ are denoted by $P_{2}, Q_{2}, R_{2}, S_{2}$, respectively. Prove that $P_{1} Q_{1} R_{1} S_{1} \cong P_{2} Q_{2} R_{2} S_{2}$. +4. $(\mathbf{C H N} 3)^{\mathrm{IMO}}$ Given nine points in space, no four of which are coplanar, find the minimal natural number $n$ such that for any coloring with red or blue of $n$ edges drawn between these nine points there always exists a triangle having all edges of the same color. +5. (COL 3) Let $A B C D$ be a convex quadrilateral such that $A C=$ $B D$. Equilateral triangles are constructed on the sides of the quadrilateral. Let $O_{1}, O_{2}, O_{3}, O_{4}$ be the centers of the triangles constructed on $A B, B C, C D, D A$ respectively. Show that $O_{1} O_{3}$ is perpendicular to $O_{2} O_{4}$. +6. (IND 2) ${ }^{\mathrm{IMO} 2}$ Find all functions $f: \mathbb{R} \rightarrow \mathbb{R}$ such that + +$$ +f\left(x^{2}+f(y)\right)=y+f(x)^{2} \quad \text { for all } x, y \text { in } \mathbb{R} +$$ + +7. (IND 4) Circles $G, G_{1}, G_{2}$ are three circles related to each other as follows: Circles $G_{1}$ and $G_{2}$ are externally tangent to one another at a point $W$ and both these circles are internally tangent to the circle $G$. Points $A, B, C$ are located on the circle $G$ as follows: Line $B C$ is a direct common tangent to the pair of circles $G_{1}$ and $G_{2}$, and line $W A$ is the transverse common tangent at $W$ to $G_{1}$ and $G_{2}$, with $W$ and $A$ lying on the same side of the line $B C$. Prove that $W$ is the incenter of the triangle $A B C$. +8. (IND 5) Show that in the plane there exists a convex polygon of 1992 sides satisfying the following conditions: +(i) its side lengths are $1,2,3, \ldots, 1992$ in some order; +(ii) the polygon is circumscribable about a circle. + +Alternative formulation. Does there exist a 1992-gon with side lengths $1,2,3, \ldots, 1992$ circumscribed about a circle? Answer the same question for a 1990-gon. +9. (IRN 1) Let $f(x)$ be a polynomial with rational coefficients and $\alpha$ be a real number such that $\alpha^{3}-\alpha=f(\alpha)^{3}-f(\alpha)=33^{1992}$. Prove that for each $n \geq 1$, + +$$ +\left(f^{(n)}(\alpha)\right)^{3}-f^{(n)}(\alpha)=33^{1992} +$$ + +where $f^{(n)}(x)=f(f(\ldots f(x)))$, and $n$ is a positive integer. +10. (ITA 1) ${ }^{\mathrm{IMO5}}$ Let $V$ be a finite subset of Euclidean space consisting of points $(x, y, z)$ with integer coordinates. Let $S_{1}, S_{2}, S_{3}$ be the projections of $V$ onto the $y z, x z, x y$ planes, respectively. Prove that + +$$ +|V|^{2} \leq\left|S_{1}\right|\left|S_{2}\right|\left|S_{3}\right| +$$ + +( $|X|$ denotes the number of elements of $X$ ). +11. (JAP 2) In a triangle $A B C$, let $D$ and $E$ be the intersections of the bisectors of $\angle A B C$ and $\angle A C B$ with the sides $A C, A B$, respectively. Determine the angles $\angle A, \angle B, \angle C$ if + +$$ +\measuredangle B D E=24^{\circ}, \quad \measuredangle C E D=18^{\circ} . +$$ + +12. (NET 1) Let $f, g$, and $a$ be polynomials with real coefficients, $f$ and $g$ in one variable and $a$ in two variables. Suppose + +$$ +f(x)-f(y)=a(x, y)(g(x)-g(y)) \quad \text { for all } x, y \in \mathbb{R} +$$ + +Prove that there exists a polynomial $h$ with $f(x)=h(g(x))$ for all $x \in \mathbb{R}$. +13. (NZL 1) ${ }^{\mathrm{IMO1}}$ Find all integer triples $(p, q, r)$ such that $13$ be a prime and suppose there exists an integer $z$ such that $p$ divides $f(z)$. Prove that there exist integers $z_{1}, z_{2}, \ldots, z_{8}$ such that if + +$$ +g(x)=\left(x-z_{1}\right)\left(x-z_{2}\right) \cdots\left(x-z_{8}\right) +$$ + +then all coefficients of $f(x)-g(x)$ are divisible by $p$. +20. (FRA 1) ${ }^{\mathrm{IMO} 4}$ In the plane, let there be given a circle $C$, a line $l$ tangent to $C$, and a point $M$ on $l$. Find the locus of points $P$ that have the following property: There exist two points $Q$ and $R$ on $l$ such that $M$ is the midpoint of $Q R$ and $C$ is the incircle of $P Q R$. +21. (GBR 2) ${ }^{\mathrm{IMO} 6}$ For each positive integer $n$, denote by $s(n)$ the greatest integer such that for all positive integers $k \leq s(n), n^{2}$ can be expressed as a sum of squares of $k$ positive integers. +(a) Prove that $s(n) \leq n^{2}-14$ for all $n \geq 4$. +(b) Find a number $n$ such that $s(n)=\overline{n^{2}}-14$. +(c) Prove that there exist infinitely many positive integers $n$ such that $s(n)=n^{2}-14$. + +### 3.34 The Thirty-Fourth IMO Istanbul, Turkey, July 13-24, 1993 + +### 3.34.1 Contest Problems + +## First Day (July 18) + +1. Let $n>1$ be an integer and let $f(x)=x^{n}+5 x^{n-1}+3$. Prove that there do not exist polynomials $g(x), h(x)$, each having integer coefficients and degree at least one, such that $f(x)=g(x) h(x)$. +2. $A, B, C, D$ are four points in the plane, with $C, D$ on the same side of the line $A B$, such that $A C \cdot B D=A D \cdot B C$ and $\measuredangle A D B=90^{\circ}+\measuredangle A C B$. Find the ratio + +$$ +\frac{A B \cdot C D}{A C \cdot B D} +$$ + +and prove that circles $A C D, B C D$ are orthogonal. (Intersecting circles are said to be orthogonal if at either common point their tangents are perpendicular.) +3. On an infinite chessboard, a solitaire game is played as follows: At the start, we have $n^{2}$ pieces occupying $n^{2}$ squares that form a square of side $n$. The only allowed move is a jump horizontally or vertically over an occupied square to an unoccupied one, and the piece that has been jumped over is removed. For what positive integers $n$ can the game end with only one piece remaining on the board? + +Second Day (July 19) +4. For three points $A, B, C$ in the plane we define $m(A B C)$ to be the smallest length of the three altitudes of the triangle $A B C$, where in the case of $A, B, C$ collinear, $m(A B C)=0$. Let $A, B, C$ be given points in the plane. Prove that for any point $X$ in the plane, + +$$ +m(A B C) \leq m(A B X)+m(A X C)+m(X B C) +$$ + +5. Let $\mathbb{N}=\{1,2,3, \ldots\}$. Determine whether there exists a strictly increasing function $f: \mathbb{N} \rightarrow \mathbb{N}$ with the following properties: + +$$ +\begin{aligned} +f(1) & =2 \\ +f(f(n)) & =f(n)+n \quad(n \in \mathbb{N}) +\end{aligned} +$$ + +6 . Let $n$ be an integer greater than 1 . In a circular arrangement of $n$ lamps $L_{0}, \ldots, L_{n-1}$, each one of that can be either ON or OFF, we start with the situation where all lamps are ON, and then carry out a sequence of steps, $S t e p_{0}, S t e p_{1}, \ldots$ If $L_{j-1}(j$ is taken $\bmod n)$ is ON, then $S t e p_{j}$ changes the status of $L_{j}$ (it goes from ON to OFF or from OFF to ON) but does not change the status of any of the other lamps. If $L_{j-1}$ is OFF , then $S t e p_{j}$ does not change anything at all. Show that: +(a) There is a positive integer $M(n)$ such that after $M(n)$ steps all lamps are ON again. +(b) If $n$ has the form $2^{k}$, then all lamps are ON after $n^{2}-1$ steps. +(c) If $n$ has the form $2^{k}+1$, then all lamps are ON after $n^{2}-n+1$ steps. + +### 3.34.2 Shortlisted Problems + +1. (BRA 1) Show that there exists a finite set $A \subset \mathbb{R}^{2}$ such that for every $X \in A$ there are points $Y_{1}, Y_{2}, \ldots, Y_{1993}$ in $A$ such that the distance between $X$ and $Y_{i}$ is equal to 1 , for every $i$. +2. (CAN 2) Let triangle $A B C$ be such that its circumradius $R$ is equal to 1. Let $r$ be the inradius of $A B C$ and let $p$ be the inradius of the orthic triangle $A^{\prime} B^{\prime} C^{\prime}$ of triangle $A B C$. +Prove that $p \leq 1-\frac{1}{3}(1+r)^{2}$. +Remark. The orthic triangle is the triangle whose vertices are the feet of the altitudes of $A B C$. +3. (SPA 1) Consider the triangle $A B C$, its circumcircle $k$ with center $O$ and radius $R$, and its incircle with center $I$ and radius $r$. Another circle $k_{c}$ is tangent to the sides $C A, C B$ at $D, E$, respectively, and it is internally tangent to $k$. +Show that the incenter $I$ is the midpoint of $D E$. +4. (SPA 2) In the triangle $A B C$, let $D, E$ be points on the side $B C$ such that $\angle B A D=\angle C A E$. If $M, N$ are, respectively, the points of tangency with $B C$ of the incircles of the triangles $A B D$ and $A C E$, show that + +$$ +\frac{1}{M B}+\frac{1}{M D}=\frac{1}{N C}+\frac{1}{N E} +$$ + +5. (FIN 3) ${ }^{\mathrm{IMO} 3}$ On an infinite chessboard, a solitaire game is played as follows: At the start, we have $n^{2}$ pieces occupying $n^{2}$ squares that form a square of side $n$. The only allowed move is a jump horizontally or vertically over an occupied square to an unoccupied one, and the piece that has been jumped over is removed. For what positive integers $n$ can the game end with only one piece remaining on the board? +6. (GER 1) ${ }^{\mathrm{IMO} 5}$ Let $\mathbb{N}=\{1,2,3, \ldots\}$. Determine whether there exists a strictly increasing function $f: \mathbb{N} \rightarrow \mathbb{N}$ with the following properties: + +$$ +\begin{aligned} +f(1) & =2 \\ +f(f(n)) & =f(n)+n \quad(n \in \mathbb{N}) +\end{aligned} +$$ + +7. (GEO 3) Let $a, b, c$ be given integers $a>0, a c-b^{2}=P=P_{1} \cdots P_{m}$ where $P_{1}, \ldots, P_{m}$ are (distinct) prime numbers. Let $M(n)$ denote the number of pairs of integers $(x, y)$ for which + +$$ +a x^{2}+2 b x y+c y^{2}=n +$$ + +Prove that $M(n)$ is finite and $M(n)=M\left(P^{k} \cdot n\right)$ for every integer $k \geq 0$. +8. (IND 1) Define a sequence $\langle f(n)\rangle_{n=1}^{\infty}$ of positive integers by $f(1)=1$ and + +$$ +f(n)= \begin{cases}f(n-1)-n, & \text { if } f(n-1)>n ; \\ f(n-1)+n, & \text { if } f(n-1) \leq n,\end{cases} +$$ + +for $n \geq 2$. Let $S=\{n \in \mathbb{N} \mid f(n)=1993\}$. +(a) Prove that $S$ is an infinite set. +(b) Find the least positive integer in $S$. +(c) If all the elements of $S$ are written in ascending order as $n_{1}34 +$$ + +10. (IND 5) A natural number $n$ is said to have the property $P$ if whenever $n$ divides $a^{n}-1$ for some integer $a, n^{2}$ also necessarily divides $a^{n}-1$. +(a) Show that every prime number has property $P$. +(b) Show that there are infinitely many composite numbers $n$ that possess property $P$. +11. (IRE 1) ${ }^{\mathrm{IMO1}}$ Let $n>1$ be an integer and let $f(x)=x^{n}+5 x^{n-1}+3$. Prove that there do not exist polynomials $g(x), h(x)$, each having integer coefficients and degree at least one, such that $f(x)=g(x) h(x)$. +12. (IRE 2) Let $n, k$ be positive integers with $k \leq n$ and let $S$ be a set containing $n$ distinct real numbers. Let $T$ be the set of all real numbers of the form $x_{1}+x_{2}+\cdots+x_{k}$, where $x_{1}, x_{2}, \ldots, x_{k}$ are distinct elements of $S$. Prove that $T$ contains at least $k(n-k)+1$ distinct elements. +13. (IRE 3) Let $S$ be the set of all pairs $(m, n)$ of relatively prime positive integers $m, n$ with $n$ even and $m1$ and $b^{n}-1 \mid a$. Show that the representation of the number $a$ in the base $b$ contains at least $n$ digits different from zero. +20. (ROM 3) Let $c_{1}, \ldots, c_{n} \in \mathbb{R}(n \geq 2)$ such that $0 \leq \sum_{i=1}^{n} c_{i} \leq n$. Show that we can find integers $k_{1}, \ldots, k_{n}$ such that $\sum_{i=1}^{n} k_{i}=0$ and + +$$ +1-n \leq c_{i}+n k_{i} \leq n \quad \text { for every } i=1, \ldots, n +$$ + +21. (GBR 1) A circle $S$ is said to cut a circle $\Sigma$ diametrally if their common chord is a diameter of $\Sigma$. +Let $S_{A}, S_{B}, S_{C}$ be three circles with distinct centers $A, B, C$ respectively. Prove that $A, B, C$ are collinear if and only if there is no unique circle $S$ that cuts each of $S_{A}, S_{B}, S_{C}$ diametrally. Prove further that if there exists more than one circle $S$ that cuts each of $S_{A}, S_{B}, S_{C}$ diametrally, then all such circles pass through two fixed points. Locate these points in relation to the circles $S_{A}, S_{B}, S_{C}$. +22. (GBR 2) ${ }^{\mathrm{IMO} 2} A, B, C, D$ are four points in the plane, with $C, D$ on the same side of the line $A B$, such that $A C \cdot B D=A D \cdot B C$ and $\measuredangle A D B=$ $90^{\circ}+\measuredangle A C B$. Find the ratio + +$$ +\frac{A B \cdot C D}{A C \cdot B D} +$$ + +and prove that circles $A C D, B C D$ are orthogonal. (Intersecting circles are said to be orthogonal if at either common point their tangents are perpendicular.) +23. (GBR 3) A finite set of (distinct) positive integers is called a " $D S$-set" if each of the integers divides the sum of them all. Prove that every finite set of positive integers is a subset of some $D S$-set. +24. (USA 3) Prove that + +$$ +\frac{a}{b+2 c+3 d}+\frac{b}{c+2 d+3 a}+\frac{c}{d+2 a+3 b}+\frac{d}{a+2 b+3 c} \geq \frac{2}{3} +$$ + +for all positive real numbers $a, b, c, d$. +25. (VIE 1) Solve the following system of equations, in which $a$ is a given number satisfying $|a|>1$ : + +$$ +\begin{aligned} +x_{1}^{2} & =a x_{2}+1, \\ +x_{2}^{2} & =a x_{3}+1 \\ +\cdots & \cdots \\ +x_{999}^{2} & =a x_{1000}+1 \\ +x_{1000}^{2} & =a x_{1}+1 +\end{aligned} +$$ + +26. (VIE 2) Let $a, b, c, d$ be four nonnegative numbers satisfying $a+b+c+d=$ 1. Prove the inequality + +$$ +a b c+b c d+c d a+d a b \leq \frac{1}{27}+\frac{176}{27} a b c d +$$ + +### 3.35 The Thirty-Fifth IMO Hong Kong, July 9-22, 1994 + +### 3.35.1 Contest Problems + +First Day (July 13) + +1. Let $m$ and $n$ be positive integers. The set $A=\left\{a_{1}, a_{2}, \ldots, a_{m}\right\}$ is a subset of $1,2, \ldots, n$. Whenever $a_{i}+a_{j} \leq n, 1 \leq i \leq j \leq m, a_{i}+a_{j}$ also belongs to $A$. Prove that + +$$ +\frac{a_{1}+a_{2}+\cdots+a_{m}}{m} \geq \frac{n+1}{2} +$$ + +2. $N$ is an arbitrary point on the bisector of $\angle B A C . P$ and $O$ are points on the lines $A B$ and $A N$, respectively, such that $\measuredangle A N P=90^{\circ}=\measuredangle A P O . Q$ is an arbitrary point on $N P$, and an arbitrary line through $Q$ meets the lines $A B$ and $A C$ at $E$ and $F$ respectively. Prove that $\measuredangle O Q E=90^{\circ}$ if and only if $Q E=Q F$. +3. For any positive integer $k, A_{k}$ is the subset of $\{k+1, k+2, \ldots, 2 k\}$ consisting of all elements whose digits in base 2 contain exactly three 1's. Let $f(k)$ denote the number of elements in $A_{k}$. +(a) Prove that for any positive integer $m, f(k)=m$ has at least one solution. +(b) Determine all positive integers $m$ for which $f(k)=m$ has a unique solution. + +## Second Day (July 14) + +4. Determine all pairs $(m, n)$ of positive integers such that $\frac{n^{3}+1}{m n-1}$ is an integer. + +5 . Let $S$ be the set of real numbers greater than -1 . Find all functions $f: S \rightarrow S$ such that + +$$ +f(x+f(y)+x f(y))=y+f(x)+y f(x) \quad \text { for all } x \text { and } y \text { in } S, +$$ + +and $f(x) / x$ is strictly increasing for $-1k$. Remove this card, slide all cards from the $(k+1)$ st to the $l$ th position one place to the right, and replace the card $l$ in the $l$ th position. +(a) Prove that the game lasts at most $2^{n}-1$ moves. +(b) Prove that there exists a unique initial configuration for which the game lasts exactly $2^{n}-1$ moves. +10. C5 (SWE) At a round table are 1994 girls, playing a game with a deck of $n$ cards. Initially, one girl holds all the cards. In each turn, if at least one girl holds at least two cards, one of these girls must pass a card to each of her two neighbors. The game ends when and only when each girl is holding at most one card. +(a) Prove that if $n \geq 1994$, then the game cannot end. +(b) Prove that if $n<1994$, then the game must end. +11. C6 (FIN) On an infinite square grid, two players alternately mark symbols on empty cells. The first player always marks $X$ 's, the second $O$ 's. One symbol is marked per turn. The first player wins if there are 11 consecutive $X$ 's in a row, column, or diagonal. Prove that the second player can prevent the first from winning. +12. C7 (BRA) Prove that for any integer $n \geq 2$, there exists a set of $2^{n-1}$ points in the plane such that no 3 lie on a line and no $2 n$ are the vertices of a convex $2 n$-gon. +13. G1 (FRA) A semicircle $\Gamma$ is drawn on one side of a straight line $l$. $C$ and $D$ are points on $\Gamma$. The tangents to $\Gamma$ at $C$ and $D$ meet $l$ at $B$ and $A$ respectively, with the center of the semicircle between them. Let $E$ be the point of intersection of $A C$ and $B D$, and $F$ the point on $l$ such that $E F$ is perpendicular to $l$. Prove that $E F$ bisects $\angle C F D$. +14. G2 (UKR) $A B C D$ is a quadrilateral with $B C$ parallel to $A D . M$ is the midpoint of $C D, P$ that of $M A$ and $Q$ that of $M B$. The lines $D P$ and $C Q$ meet at $N$. Prove that $N$ is not outside triangle $A B M .{ }^{8}$ +15. G3 (RUS) A circle $\omega$ is tangent to two parallel lines $l_{1}$ and $l_{2}$. A second circle $\omega_{1}$ is tangent to $l_{1}$ at $A$ and to $\omega$ externally at $C$. A third circle $\omega_{2}$ is tangent to $l_{2}$ at $B$, to $\omega$ externally at $D$, and to $\omega_{1}$ externally at $E$. $A D$ intersects $B C$ at $Q$. Prove that $Q$ is the circumcenter of triangle $C D E$. + +[^6]16. G4 (AUS-ARM) ${ }^{\mathrm{IMO} 2} N$ is an arbitrary point on the bisector of $\angle B A C$. $P$ and $O$ are points on the lines $A B$ and $A N$, respectively, such that $\measuredangle A N P=90^{\circ}=\measuredangle A P O . Q$ is an arbitrary point on $N P$, and an arbitrary line through $Q$ meets the lines $A B$ and $A C$ at $E$ and $F$ respectively. Prove that $\measuredangle O Q E=90^{\circ}$ if and only if $Q E=Q F$. +17. G5 (CYP) A line $l$ does not meet a circle $\omega$ with center $O . E$ is the point on $l$ such that $O E$ is perpendicular to $l . M$ is any point on $l$ other than $E$. The tangents from $M$ to $\omega$ touch it at $A$ and $B . C$ is the point on $M A$ such that $E C$ is perpendicular to $M A . D$ is the point on $M B$ such that $E D$ is perpendicular to $M B$. The line $C D$ cuts $O E$ at $F$. Prove that the location of $F$ is independent of that of $M$. +18. N1 (BUL) $M$ is a subset of $\{1,2,3, \ldots, 15\}$ such that the product of any three distinct elements of $M$ is not a square. Determine the maximum number of elements in $M$. +19. N2 (AUS) ${ }^{\mathrm{IMO} 4}$ Determine all pairs $(m, n)$ of positive integers such that $\frac{n^{3}+1}{m n-1}$ is an integer. +20. N3 (FIN) ${ }^{\text {IMO6 }}$ Find a set $A$ of positive integers such that for any infinite set $P$ of prime numbers, there exist positive integers $m \in A$ and $n \notin A$, both the product of the same number of distinct elements of $P$. +21. N4 (FRA) For any positive integer $x_{0}$, three sequences $\left\{x_{n}\right\},\left\{y_{n}\right\}$, and $\left\{z_{n}\right\}$ are defined as follows: +(i) $y_{0}=4$ and $z_{0}=1$; +(ii) if $x_{n}$ is even for $n \geq 0, x_{n+1}=\frac{x_{n}}{2}, y_{n+1}=2 y_{n}$, and $z_{n+1}=z_{n}$; +(iii) if $x_{n}$ is odd for $n \geq 0, x_{n+1}=x_{n}-\frac{y_{n}}{2}-z_{n}, y_{n+1}=y_{n}$, and $z_{n+1}=$ $y_{n}+z_{n}$. +The integer $x_{0}$ is said to be good if $x_{n}=0$ for some $n \geq 1$. Find the number of good integers less than or equal to 1994. +22. N5 (ROM) ${ }^{\mathrm{IMO} 3}$ For any positive integer $k, A_{k}$ is the subset of $\{k+1, k+$ $2, \ldots, 2 k\}$ consisting of all elements whose digits in base 2 contain exactly three 1's. Let $f(k)$ denote the number of elements in $A_{k}$. +(a) Prove that for any positive integer $m, f(k)=m$ has at least one solution. +(b) Determine all positive integers $m$ for which $f(k)=m$ has a unique solution. +23. N6 (LAT) Let $x_{1}$ and $x_{2}$ be relatively prime positive integers. For $n \geq 2$, define $x_{n+1}=x_{n} x_{n-1}+1$. +(a) Prove that for every $i>1$, there exists $j>i$ such that $x_{i}^{i}$ divides $x_{j}^{j}$. +(b) Is it true that $x_{1}$ must divide $x_{j}^{j}$ for some $j>1$ ? +24. N7 (GBR) A wobbly number is a positive integer whose digits in base 10 are alternately nonzero and zero, the units digit being nonzero. Determine all positive integers that do not divide any wobbly number. + +### 3.36 The Thirty-Sixth IMO Toronto, Canada, July 13-25, 1995 + +### 3.36.1 Contest Problems + +First Day (July 19) + +1. Let $A, B, C$, and $D$ be distinct points on a line, in that order. The circles with diameters $A C$ and $B D$ intersect at $X$ and $Y . O$ is an arbitrary point on the line $X Y$ but not on $A D . C O$ intersects the circle with diameter $A C$ again at $M$, and $B O$ intersects the other circle again at $N$. Prove that the lines $A M, D N$, and $X Y$ are concurrent. +2. Let $a, b$, and $c$ be positive real numbers such that $a b c=1$. Prove that + +$$ +\frac{1}{a^{3}(b+c)}+\frac{1}{b^{3}(a+c)}+\frac{1}{c^{3}(a+b)} \geq \frac{3}{2} +$$ + +3. Determine all integers $n>3$ such that there are $n$ points $A_{1}, A_{2}, \ldots, A_{n}$ in the plane that satisfy the following two conditions simultaneously: +(a) No three lie on the same line. +(b) There exist real numbers $p_{1}, p_{2}, \ldots, p_{n}$ such that the area of $\triangle A_{i} A_{j} A_{k}$ is equal to $p_{i}+p_{j}+p_{k}$, for $1 \leq i\left(\sum_{i=1}^{n-1}(n-i) x_{i}\right)\left(\sum_{j=2}^{n}(j-1) x_{j}\right) . +$$ + +7. G1 (BUL) ${ }^{\mathrm{IMO} 1}$ Let $A, B, C$, and $D$ be distinct points on a line, in that order. The circles with diameters $A C$ and $B D$ intersect at $X$ and $Y . O$ is an arbitrary point on the line $X Y$ but not on $A D$. $C O$ intersects the circle with diameter $A C$ again at $M$, and $B O$ intersects the other circle again at $N$. Prove that the lines $A M, D N$, and $X Y$ are concurrent. +8. G2 (GER) Let $A, B$, and $C$ be noncollinear points. Prove that there is a unique point $X$ in the plane of $A B C$ such that $X A^{2}+X B^{2}+A B^{2}=$ $X B^{2}+X C^{2}+B C^{2}=X C^{2}+X A^{2}+C A^{2}$. +9. G3 (TUR) The incircle of $A B C$ touches $B C, C A$, and $A B$ at $D, E$, and $F$ respectively. $X$ is a point inside $A B C$ such that the incircle of $X B C$ touches $B C$ at $D$ also, and touches $C X$ and $X B$ at $Y$ and $Z$, respectively. Prove that $E F Z Y$ is a cyclic quadrilateral. +10. G4 (UKR) An acute triangle $A B C$ is given. Points $A_{1}$ and $A_{2}$ are taken on the side $B C$ (with $A_{2}$ between $A_{1}$ and $C$ ), $B_{1}$ and $B_{2}$ on the side $A C$ (with $B_{2}$ between $B_{1}$ and $A$ ), and $C_{1}$ and $C_{2}$ on the side $A B$ (with $C_{2}$ between $C_{1}$ and $B$ ) such that + +$$ +\angle A A_{1} A_{2}=\angle A A_{2} A_{1}=\angle B B_{1} B_{2}=\angle B B_{2} B_{1}=\angle C C_{1} C_{2}=\angle C C_{2} C_{1} . +$$ + +The lines $A A_{1}, B B_{1}$, and $C C_{1}$ form a triangle, and the lines $A A_{2}, B B_{2}$, and $C C_{2}$ form a second triangle. Prove that all six vertices of these two triangles lie on a single circle. +11. G5 (NZL) ${ }^{\mathrm{IMO} 5}$ Let $A B C D E F$ be a convex hexagon with $A B=B C=$ $C D, D E=E F=F A$, and $\measuredangle B C D=\measuredangle E F A=\pi / 3$ (that is, $60^{\circ}$ ). Let $G$ and $H$ be two points interior to the hexagon such that angles $A G B$ and $D H E$ are both $2 \pi / 3$ (that is, $120^{\circ}$ ). Prove that $A G+G B+G H+D H+$ $H E \geq C F$. +12. G6 (USA) Let $A_{1} A_{2} A_{3} A_{4}$ be a tetrahedron, $G$ its centroid, and $A_{1}^{\prime}, A_{2}^{\prime}, A_{3}^{\prime}$, and $A_{4}^{\prime}$ the points where the circumsphere of $A_{1} A_{2} A_{3} A_{4}$ intersects $G A_{1}, G A_{2}, G A_{3}$, and $G A_{4}$, respectively. Prove that + +$$ +G A_{1} \cdot G A_{2} \cdot G A_{3} \cdot G A_{4} \leq G A_{1}^{\prime} \cdot G A_{2}^{\prime} \cdot G A_{3}^{\prime} \cdot G A_{4}^{\prime} +$$ + +and + +$$ +\frac{1}{G A_{1}^{\prime}}+\frac{1}{G A_{2}^{\prime}}+\frac{1}{G A_{3}^{\prime}}+\frac{1}{G A_{4}^{\prime}} \leq \frac{1}{G A_{1}}+\frac{1}{G A_{2}}+\frac{1}{G A_{3}}+\frac{1}{G A_{4}} +$$ + +13. G7 (LAT) $O$ is a point inside a convex quadrilateral $A B C D$ of area $S . K, L, M$, and $N$ are interior points of the sides $A B, B C, C D$, and $D A$ respectively. If $O K B L$ and $O M D N$ are parallelograms, prove that $\sqrt{S} \geq \sqrt{S_{1}}+\sqrt{S_{2}}$, where $S_{1}$ and $S_{2}$ are the areas of $O N A K$ and $O L C M$ respectively. +14. G8 (COL) Let $A B C$ be a triangle. A circle passing through $B$ and $C$ intersects the sides $A B$ and $A C$ again at $C^{\prime}$ and $B^{\prime}$, respectively. Prove that $B B^{\prime}, C C^{\prime}$, and $H H^{\prime}$ are concurrent, where $H$ and $H^{\prime}$ are the orthocenters of triangles $A B C$ and $A B^{\prime} C^{\prime}$ respectively. +15. N1 (ROM) Let $k$ be a positive integer. Prove that there are infinitely many perfect squares of the form $n 2^{k}-7$, where $n$ is a positive integer. +16. N2 (RUS) Let $\mathbb{Z}$ denote the set of all integers. Prove that for any integers $A$ and $B$, one can find an integer $C$ for which $M_{1}=\left\{x^{2}+A x+B: x \in \mathbb{Z}\right\}$ and $M_{2}=\left\{2 x^{2}+2 x+C: x \in \mathbb{Z}\right\}$ do not intersect. +17. N3 $(\mathbf{C Z E})^{\mathrm{IMO} 3}$ Determine all integers $n>3$ such that there are $n$ points $A_{1}, A_{2}, \ldots, A_{n}$ in the plane that satisfy the following two conditions simultaneously: +(a) No three lie on the same line. +(b) There exist real numbers $p_{1}, p_{2}, \ldots, p_{n}$ such that the area of $\triangle A_{i} A_{j} A_{k}$ is equal to $p_{i}+p_{j}+p_{k}$, for $1 \leq i1$ that satisfies the following condition? +The set of positive integers can be partitioned into $n$ nonempty subsets such that an arbitrary sum of $n-1$ integers, one taken from each of any $n-1$ of the subsets, lies in the remaining subset. +22. N8 (GER) Let $p$ be an odd prime. Determine positive integers $x$ and $y$ for which $x \leq y$ and $\sqrt{2 p}-\sqrt{x}-\sqrt{y}$ is nonnegative and as small as possible. +23. S1 (UKR) Does there exist a sequence $F(1), F(2), F(3), \ldots$ of nonnegative integers that simultaneously satisfies the following three conditions? +(a) Each of the integers $0,1,2, \ldots$ occurs in the sequence. +(b) Each positive integer occurs in the sequence infinitely often. +(c) For any $n \geq 2$, + +$$ +F\left(F\left(n^{163}\right)\right)=F(F(n))+F(F(361)) . +$$ + +24. S2 (POL) ${ }^{\text {IMO4 }}$ The positive real numbers $x_{0}, x_{1}, \ldots, x_{1995}$ satisfy $x_{0}=$ $x_{1995}$ and + +$$ +x_{i-1}+\frac{2}{x_{i-1}}=2 x_{i}+\frac{1}{x_{i}} +$$ + +for $i=1,2, \ldots, 1995$. Find the maximum value that $x_{0}$ can have. +25. S3 (POL) For an integer $x \geq 1$, let $p(x)$ be the least prime that does not divide $x$, and define $q(x)$ to be the product of all primes less than $p(x)$. In particular, $p(1)=2$. For $x$ such that $p(x)=2$, define $q(x)=1$. Consider the sequence $x_{0}, x_{1}, x_{2}, \ldots$ defined by $x_{0}=1$ and + +$$ +x_{n+1}=\frac{x_{n} p\left(x_{n}\right)}{q\left(x_{n}\right)} +$$ + +for $n \geq 0$. Find all $n$ such that $x_{n}=1995$. +26. S4 (NZL) Suppose that $x_{1}, x_{2}, x_{3}, \ldots$ are positive real numbers for which + +$$ +x_{n}^{n}=\sum_{j=0}^{n-1} x_{n}^{j} +$$ + +for $n=1,2,3, \ldots$ Prove that for all $n$, + +$$ +2-\frac{1}{2^{n-1}} \leq x_{n}<2-\frac{1}{2^{n}} +$$ + +27. S5 (FIN) For positive integers $n$, the numbers $f(n)$ are defined inductively as follows: $f(1)=1$, and for every positive integer $n, f(n+1)$ is the greatest integer $m$ such that there is an arithmetic progression of positive integers $a_{1}0$. Let $p=\max \left\{\left|a_{1}\right|, \ldots,\left|a_{n}\right|\right\}$. Prove that $p=a_{1}$ and that + +$$ +\left(x-a_{1}\right)\left(x-a_{2}\right) \cdots\left(x-a_{n}\right) \leq x^{n}-a_{1}^{n} +$$ + +for all $x>a_{1}$. +3. A3 (GRE) Let $a>2$ be given, and define recursively + +$$ +a_{0}=1, \quad a_{1}=a, \quad a_{n+1}=\left(\frac{a_{n}^{2}}{a_{n-1}^{2}}-2\right) a_{n} +$$ + +Show that for all $k \in \mathbb{N}$, we have + +$$ +\frac{1}{a_{0}}+\frac{1}{a_{1}}+\frac{1}{a_{2}}+\cdots+\frac{1}{a_{k}}<\frac{1}{2}\left(2+a-\sqrt{a^{2}-4}\right) . +$$ + +4. A4 (KOR) Let $a_{1}, a_{2}, \ldots, a_{n}$ be nonnegative real numbers, not all zero. +(a) Prove that $x^{n}-a_{1} x^{n-1}-\cdots-a_{n-1} x-a_{n}=0$ has precisely one positive real root. +(b) Let $A=\sum_{j=1}^{n} a_{j}, B=\sum_{j=1}^{n} j a_{j}$, and let $R$ be the positive real root of the equation in part (a). Prove that + +$$ +A^{A} \leq R^{B} +$$ + +5. A5 (ROM) Let $P(x)$ be the real polynomial function $P(x)=a x^{3}+$ $b x^{2}+c x+d$. Prove that if $|P(x)| \leq 1$ for all $x$ such that $|x| \leq 1$, then + +$$ +|a|+|b|+|c|+|d| \leq 7 +$$ + +6. A6 (IRE) Let $n$ be an even positive integer. Prove that there exists a positive integer $k$ such that + +$$ +k=f(x)(x+1)^{n}+g(x)\left(x^{n}+1\right) +$$ + +for some polynomials $f(x), g(x)$ having integer coefficients. If $k_{0}$ denotes the least such $k$, determine $k_{0}$ as a function of $n$. + +A6 ${ }^{\prime}$ Let $n$ be an even positive integer. Prove that there exists a positive integer $k$ such that + +$$ +k=f(x)(x+1)^{n}+g(x)\left(x^{n}+1\right) +$$ + +for some polynomials $f(x), g(x)$ having integer coefficients. If $k_{0}$ denotes the least such $k$, show that $k_{0}=2^{q}$, where $q$ is the odd integer determined by $n=q 2^{r}, r \in \mathbb{N}$. +A6" Prove that for each positive integer $n$, there exist polynomials $f(x), g(x)$ having integer coefficients such that + +$$ +f(x)(x+1)^{2^{n}}+g(x)\left(x^{2^{n}}+1\right)=2 . +$$ + +7. A7 (ARM) Let $f$ be a function from the set of real numbers $\mathbb{R}$ into itself such that for all $x \in \mathbb{R}$, we have $|f(x)| \leq 1$ and + +$$ +f\left(x+\frac{13}{42}\right)+f(x)=f\left(x+\frac{1}{6}\right)+f\left(x+\frac{1}{7}\right) . +$$ + +Prove that $f$ is a periodic function (that is, there exists a nonzero real number $c$ such that $f(x+c)=f(x)$ for all $x \in \mathbb{R})$. +8. A8 (ROM) ${ }^{\mathrm{IMO} 3}$ Let $\mathbb{N}_{0}$ denote the set of nonnegative integers. Find all functions $f$ from $\mathbb{N}_{0}$ into itself such that + +$$ +f(m+f(n))=f(f(m))+f(n), \quad \forall m, n \in \mathbb{N}_{0} +$$ + +9. A9 (POL) Let the sequence $a(n), n=1,2,3, \ldots$, be generated as follows: $a(1)=0$, and for $n>1$, + +$$ +a(n)=a([n / 2])+(-1)^{\frac{n(n+1)}{2}} . \quad(\text { Here }[t]=\text { the greatest integer } \leq t .) +$$ + +(a) Determine the maximum and minimum value of $a(n)$ over $n \leq 1996$ and find all $n \leq 1996$ for which these extreme values are attained. +(b) How many terms $a(n), n \leq 1996$, are equal to 0 ? +10. G1 (GBR) Let triangle $A B C$ have orthocenter $H$, and let $P$ be a point on its circumcircle, distinct from $A, B, C$. Let $E$ be the foot of the altitude $B H$, let $P A Q B$ and $P A R C$ be parallelograms, and let $A Q$ meet $H R$ in $X$. Prove that $E X$ is parallel to $A P$. +11. G2 (CAN) ${ }^{\mathrm{IMO} 2}$ Let $P$ be a point inside $\triangle A B C$ such that + +$$ +\angle A P B-\angle C=\angle A P C-\angle B . +$$ + +Let $D, E$ be the incenters of $\triangle A P B, \triangle A P C$ respectively. Show that $A P, B D$ and $C E$ meet in a point. +12. G3 (GBR) Let $A B C$ be an acute-angled triangle with $B C>C A$. Let $O$ be the circumcenter, $H$ its orthocenter, and $F$ the foot of its altitude $C H$. Let the perpendicular to $O F$ at $F$ meet the side $C A$ at $P$. Prove that $\angle F H P=\angle B A C$. +Possible second part: What happens if $|B C| \leq|C A|$ (the triangle still being acute-angled)? +13. G4 (USA) Let $\triangle A B C$ be an equilateral triangle and let $P$ be a point in its interior. Let the lines $A P, B P, C P$ meet the sides $B C, C A, A B$ in the points $A_{1}, B_{1}, C_{1}$ respectively. Prove that + +$$ +A_{1} B_{1} \cdot B_{1} C_{1} \cdot C_{1} A_{1} \geq A_{1} B \cdot B_{1} C \cdot C_{1} A +$$ + +14. G5 (ARM) ${ }^{\mathrm{IMO5}}$ Let $A B C D E F$ be a convex hexagon such that $A B$ is parallel to $D E, B C$ is parallel to $E F$, and $C D$ is parallel to $A F$. Let $R_{A}, R_{C}, R_{E}$ be the circumradii of triangles $F A B, B C D, D E F$ respectively, and let $P$ denote the perimeter of the hexagon. Prove that + +$$ +R_{A}+R_{C}+R_{E} \geq \frac{P}{2} +$$ + +15. G6 (ARM) Let the sides of two rectangles be $\{a, b\}$ and $\{c, d\}$ with $aR_{B}+R_{D}$ if and only if + +$$ +\angle A+\angle C>\angle B+\angle D . +$$ + +18. G9 (UKR) In the plane are given a point $O$ and a polygon $\mathcal{F}$ (not necessarily convex). Let $P$ denote the perimeter of $\mathcal{F}, D$ the sum of the distances from $O$ to the vertices of $\mathcal{F}$, and $H$ the sum of the distances from $O$ to the lines containing the sides of $\mathcal{F}$. Prove that + +$$ +D^{2}-H^{2} \geq \frac{P^{2}}{4} +$$ + +19. N1 (UKR) Four integers are marked on a circle. At each step we simultaneously replace each number by the difference between this number and the next number on the circle, in a given direction (that is, the numbers $a, b, c, d$ are replaced by $a-b, b-c, c-d, d-a)$. Is it possible after 1996 such steps to have numbers $a, b, c, d$ such that the numbers $|b c-a d|,|a c-b d|,|a b-c d|$ are primes? +20. N2 (RUS) ${ }^{\mathrm{IMO} 4}$ The positive integers $a$ and $b$ are such that the numbers $15 a+16 b$ and $16 a-15 b$ are both squares of positive integers. What is the least possible value that can be taken on by the smaller of these two squares? +21. N3 (BUL) A finite sequence of integers $a_{0}, a_{1}, \ldots, a_{n}$ is called quadratic if for each $i \in\{1,2, \ldots, n\}$ we have the equality $\left|a_{i}-a_{i-1}\right|=i^{2}$. +(a) Prove that for any two integers $b$ and $c$, there exist a natural number $n$ and a quadratic sequence with $a_{0}=b$ and $a_{n}=c$. +(b) Find the smallest natural number $n$ for which there exists a quadratic sequence with $a_{0}=0$ and $a_{n}=1996$. +22. N4 (BUL) Find all positive integers $a$ and $b$ for which + +$$ +\left[\frac{a^{2}}{b}\right]+\left[\frac{b^{2}}{a}\right]=\left[\frac{a^{2}+b^{2}}{a b}\right]+a b +$$ + +where as usual, $[t]$ refers to greatest integer that is less than or equal to $t$. +23. N5 (ROM) Let $\mathbb{N}_{0}$ denote the set of nonnegative integers. Find a bijective function $f$ from $\mathbb{N}_{0}$ into $\mathbb{N}_{0}$ such that for all $m, n \in \mathbb{N}_{0}$, + +$$ +f(3 m n+m+n)=4 f(m) f(n)+f(m)+f(n) . +$$ + +24. C1 (FIN) ${ }^{\mathrm{IMO} 1}$ We are given a positive integer $r$ and a rectangular board $A B C D$ with dimensions $|A B|=20,|B C|=12$. The rectangle is divided into a grid of $20 \times 12$ unit squares. The following moves are permitted on the board: One can move from one square to another only if the distance between the centers of the two squares is $\sqrt{r}$. The task is to find a sequence of moves leading from the square corresponding to vertex $A$ to the square corresponding to vertex $B$. +(a) Show that the task cannot be done if $r$ is divisible by 2 or 3 . +(b) Prove that the task is possible when $r=73$. +(c) Is there a solution when $r=97$ ? +25. C2 (UKR) An $(n-1) \times(n-1)$ square is divided into $(n-1)^{2}$ unit squares in the usual manner. Each of the $n^{2}$ vertices of these squares is to be colored red or blue. Find the number of different colorings such that each unit square has exactly two red vertices. (Two coloring schemes are regarded as different if at least one vertex is colored differently in the two schemes.) +26. C3 (USA) Let $k, m, n$ be integers such that $11$, then the $k$ th term from the left in $R_{n}$ is equal to 1 if and only if the $k$ th term from the right in $R_{n}$ is different from 1. +3. (GER) For each finite set $U$ of nonzero vectors in the plane we define $l(U)$ to be the length of the vector that is the sum of all vectors in $U$. Given a finite set $V$ of nonzero vectors in the plane, a subset $B$ of $V$ is said to be maximal if $l(B)$ is greater than or equal to $l(A)$ for each nonempty subset $A$ of $V$. +(a) Construct sets of 4 and 5 vectors that have 8 and 10 maximal subsets respectively. +(b) Show that for any set $V$ consisting of $n \geq 1$ vectors, the number of maximal subsets is less than or equal to $2 n$. +4. (IRN) ${ }^{\mathrm{IMO} 4} \mathrm{An} n \times n$ matrix with entries from $\{1,2, \ldots, 2 n-1\}$ is called a coveralls matrix if for each $i$ the union of the $i$ th row and the $i$ th column contains $2 n-1$ distinct entries. Show that: +(a) There exist no coveralls matrices for $n=1997$. +(b) Coveralls matrices exist for infinitely many values of $n$. +5. (ROM) Let $A B C D$ be a regular tetrahedron and $M, N$ distinct points in the planes $A B C$ and $A D C$ respectively. Show that the segments $M N, B N, M D$ are the sides of a triangle. +6. (IRE) (a) Let $n$ be a positive integer. Prove that there exist distinct positive integers $x, y, z$ such that + +$$ +x^{n-1}+y^{n}=z^{n+1} . +$$ + +(b) Let $a, b, c$ be positive integers such that $a$ and $b$ are relatively prime and $c$ is relatively prime either to $a$ or to $b$. Prove that there exist +infinitely many triples $(x, y, z)$ of distinct positive integers $x, y, z$ such that + +$$ +x^{a}+y^{b}=z^{c} . +$$ + +Original formulation: Let $a, b, c, n$ be positive integers such that $n$ is odd and $a c$ is relatively prime to $2 b$. Prove that there exist distinct positive integers $x, y, z$ such that +(i) $x^{a}+y^{b}=z^{c}$, and +(ii) $x y z$ is relatively prime to $n$. +7. (RUS) Let $A B C D E F$ be a convex hexagon such that $A B=B C, C D=$ $D E, E F=F A$. Prove that + +$$ +\frac{B C}{B E}+\frac{D E}{D A}+\frac{F A}{F C} \geq \frac{3}{2} +$$ + +When does equality occur? +8. (GBR) ${ }^{\mathrm{IMO} 2}$ Four different points $A, B, C, D$ are chosen on a circle $\Gamma$ such that the triangle $B C D$ is not right-angled. Prove that: +(a) The perpendicular bisectors of $A B$ and $A C$ meet the line $A D$ at certain points $W$ and $V$, respectively, and that the lines $C V$ and $B W$ meet at a certain point $T$. +(b) The length of one of the line segments $A D, B T$, and $C T$ is the sum of the lengths of the other two. +Original formulation. In triangle $A B C$ the angle at $A$ is the smallest. A line through $A$ meets the circumcircle again at the point $U$ lying on the $\operatorname{arc} B C$ opposite to $A$. The perpendicular bisectors of $C A$ and $A B$ meet $A U$ at $V$ and $W$, respectively, and the lines $C V, B W$ meet at $T$. Show that $A U=T B+T C$. +9. (USA) Let $A_{1} A_{2} A_{3}$ be a nonisosceles triangle with incenter $I$. Let $C_{i}$, $i=1,2,3$, be the smaller circle through $I$ tangent to $A_{i} A_{i+1}$ and $A_{i} A_{i+2}$ (the addition of indices being mod 3 ). Let $B_{i}, i=1,2,3$, be the second point of intersection of $C_{i+1}$ and $C_{i+2}$. Prove that the circumcenters of the triangles $A_{1} B_{1} I, A_{2} B_{2} I, A_{3} B_{3} I$ are collinear. +10. ( $\mathbf{C Z E}$ ) Find all positive integers $k$ for which the following statement is true: +If $F(x)$ is a polynomial with integer coefficients satisfying the condition + +$$ +0 \leq F(c) \leq k \quad \text { for each } c \in\{0,1, \ldots, k+1\} +$$ + +then $F(0)=F(1)=\cdots=F(k+1)$. +11. (NET) Let $P(x)$ be a polynomial with real coefficients such that $P(x)>$ 0 for all $x \geq 0$. Prove that there exists a positive integer $n$ such that $(1+x)^{n} P(x)$ is a polynomial with nonnegative coefficients. +12. (ITA) Let $p$ be a prime number and let $f(x)$ be a polynomial of degree $d$ with integer coefficients such that: +(i) $f(0)=0, f(1)=1$; +(ii) for every positive integer $n$, the remainder of the division of $f(n)$ by $p$ is either 0 or 1. +Prove that $d \geq p-1$. +13. (IND) In town $A$, there are $n$ girls and $n$ boys, and each girl knows each boy. In town $B$, there are $n$ girls $g_{1}, g_{2}, \ldots, g_{n}$ and $2 n-1$ boys $b_{1}, b_{2}, \ldots$, $b_{2 n-1}$. The girl $g_{i}, i=1,2, \ldots, n$, knows the boys $b_{1}, b_{2}, \ldots, b_{2 i-1}$, and no others. For all $r=1,2, \ldots, n$, denote by $A(r), B(r)$ the number of different ways in which $r$ girls from town $A$, respectively town $B$, can dance with $r$ boys from their own town, forming $r$ pairs, each girl with a boy she knows. Prove that $A(r)=B(r)$ for each $r=1,2, \ldots, n$. +14. (IND) Let $b, m, n$ be positive integers such that $b>1$ and $m \neq n$. Prove that if $b^{m}-1$ and $b^{n}-1$ have the same prime divisors, then $b+1$ is a power of 2 . +15. (RUS) An infinite arithmetic progression whose terms are positive integers contains the square of an integer and the cube of an integer. Show that it contains the sixth power of an integer. +16. (BLR) In an acute-angled triangle $A B C$, let $A D, B E$ be altitudes and $A P, B Q$ internal bisectors. Denote by $I$ and $O$ the incenter and the circumcenter of the triangle, respectively. Prove that the points $D, E$, and $I$ are collinear if and only if the points $P, Q$, and $O$ are collinear. +17. $(\mathbf{C Z E})^{\mathrm{IMO} 5}$ Find all pairs of integers $x, y \geq 1$ satisfying the equation $x^{y^{2}}=y^{x}$. +18. (GBR) The altitudes through the vertices $A, B, C$ of an acute-angled triangle $A B C$ meet the opposite sides at $D, E, F$, respectively. The line through $D$ parallel to $E F$ meets the lines $A C$ and $A B$ at $Q$ and $R$, respectively. The line $E F$ meets $B C$ at $P$. Prove that the circumcircle of the triangle $P Q R$ passes through the midpoint of $B C$. +19. (IRE) Let $a_{1} \geq \cdots \geq a_{n} \geq a_{n+1}=0$ be a sequence of real numbers. Prove that + +$$ +\sqrt{\sum_{k=1}^{n} a_{k}} \leq \sum_{k=1}^{n} \sqrt{k}\left(\sqrt{a_{k}}-\sqrt{a_{k+1}}\right) +$$ + +20. (IRE) Let $D$ be an internal point on the side $B C$ of a triangle $A B C$. The line $A D$ meets the circumcircle of $A B C$ again at $X$. Let $P$ and $Q$ be the feet of the perpendiculars from $X$ to $A B$ and $A C$, respectively, and let $\gamma$ be the circle with diameter $X D$. Prove that the line $P Q$ is tangent to $\gamma$ if and only if $A B=A C$. +21. (RUS) ${ }^{\mathrm{IMO} 3}$ Let $x_{1}, x_{2}, \ldots, x_{n}$ be real numbers satisfying the conditions + +$$ +\left|x_{1}+x_{2}+\cdots+x_{n}\right|=1 \quad \text { and } \quad\left|x_{i}\right| \leq \frac{n+1}{2} \quad \text { for } \quad i=1,2, \ldots, n +$$ + +Show that there exists a permutation $y_{1}, \ldots, y_{n}$ of the sequence $x_{1}, \ldots, x_{n}$ such that + +$$ +\left|y_{1}+2 y_{2}+\cdots+n y_{n}\right| \leq \frac{n+1}{2} +$$ + +22. (UKR) (a) Do there exist functions $f: \mathbb{R} \rightarrow \mathbb{R}$ and $g: \mathbb{R} \rightarrow \mathbb{R}$ such that + +$$ +f(g(x))=x^{2} \quad \text { and } \quad g(f(x))=x^{3} \quad \text { for all } x \in \mathbb{R} ? +$$ + +(b) Do there exist functions $f: \mathbb{R} \rightarrow \mathbb{R}$ and $g: \mathbb{R} \rightarrow \mathbb{R}$ such that + +$$ +f(g(x))=x^{2} \quad \text { and } \quad g(f(x))=x^{4} \quad \text { for all } x \in \mathbb{R} ? +$$ + +23. (GBR) Let $A B C D$ be a convex quadrilateral and $O$ the intersection of its diagonals $A C$ and $B D$. If + +$$ +O A \sin \angle A+O C \sin \angle C=O B \sin \angle B+O D \sin \angle D, +$$ + +prove that $A B C D$ is cyclic. +24. (LIT) ${ }^{\mathrm{IMO} 6}$ For a positive integer $n$, let $f(n)$ denote the number of ways to represent $n$ as the sum of powers of 2 with nonnegative integer exponents. Representations that differ only in the ordering in their summands are not considered to be distinct. (For instance, $f(4)=4$ because the number 4 can be represented in the following four ways: $4 ; 2+2 ; 2+1+1 ; 1+1+1+1$.) Prove that the inequality + +$$ +2^{n^{2} / 4}3$ be a prime number. For each nonempty subset $T$ of $\{0,1,2,3, \ldots, p-1\}$ let $E(T)$ be the set of all $(p-1)$-tuples $\left(x_{1}, \ldots, x_{p-1}\right)$, where each $x_{i} \in T$ and $x_{1}+2 x_{2}+\cdots+(p-1) x_{p-1}$ is divisible by $p$ and let $|E(T)|$ denote the number of elements in $E(T)$. +Prove that + +$$ +|E(\{0,1,3\})| \geq|E(\{0,1,2\})|, +$$ + +with equality if and only if $p=5$. + +### 3.41 The Forty-First IMO
Taejon, South Korea, July 13-25, 2000 + +### 3.41.1 Contest Problems + +First day (July 18) + +1. Two circles $G_{1}$ and $G_{2}$ intersect at $M$ and $N$. Let $A B$ be the line tangent to these circles at $A$ and $B$, respectively, such that $M$ lies closer to $A B$ than $N$. Let $C D$ be the line parallel to $A B$ and passing through $M$, with $C$ on $G_{1}$ and $D$ on $G_{2}$. Lines $A C$ and $B D$ meet at $E$; lines $A N$ and $C D$ meet at $P$; lines $B N$ and $C D$ meet at $Q$. Show that $E P=E Q$. +2. Let $a, b, c$ be positive real numbers with product 1 . Prove that + +$$ +\left(a-1+\frac{1}{b}\right)\left(b-1+\frac{1}{c}\right)\left(c-1+\frac{1}{a}\right) \leq 1 . +$$ + +3. Let $n \geq 2$ be a positive integer and $\lambda$ a positive real number. Initially there are $n$ fleas on a horizontal line, not all at the same point. We define a move of choosing two fleas at some points $A$ and $B$, with $A$ to the left of $B$, and letting the flea from $A$ jump over the flea from $B$ to the point $C$ such that $B C / A B=\lambda$. +Determine all values of $\lambda$ such that for any point $M$ on the line and for any initial position of the $n$ fleas, there exists a sequence of moves that will take them all to the position right of $M$. + +Second Day (July 19) +4. A magician has one hundred cards numbered 1 to 100 . He puts them into three boxes, a red one, a white one, and a blue one, so that each box contains at least one card. A member of the audience draws two cards from two different boxes and announces the sum of numbers on those cards. Given this information, the magician locates the box from which no card has been drawn. How many ways are there to put the cards in the three boxes so that the trick works? +5. Does there exist a positive integer $n$ such that $n$ has exactly 2000 prime divisors and $2^{n}+1$ is divisible by $n$ ? +6. $A_{1} A_{2} A_{3}$ is an acute-angled triangle. The foot of the altitude from $A_{i}$ is $K_{i}$, and the incircle touches the side opposite $A_{i}$ at $L_{i}$. The line $K_{1} K_{2}$ is reflected in the line $L_{1} L_{2}$. Similarly, the line $K_{2} K_{3}$ is reflected in $L_{2} L_{3}$ and $K_{3} K_{1}$ is reflected in $L_{3} L_{1}$. Show that the three new lines form a triangle with vertices on the incircle. + +### 3.41.2 Shortlisted Problems + +1. $\mathbf{C 1}$ (HUN) ${ }^{\mathrm{IMO}}$ A magician has one hundred cards numbered 1 to 100. He puts them into three boxes, a red one, a white one, and a blue one, so that each box contains at least one card. A member of the audience draws two cards from two different boxes and announces the sum of numbers on those cards. Given this information, the magician locates the box from which no card has been drawn. How many ways are there to put the cards in the three boxes so that the trick works? +2. C2 (ITA) A brick staircase with three steps of width 2 is made of twelve unit cubes. Determine all integers $n$ for which it is possible to build a cube of side $n$ using such bricks. +![](https://cdn.mathpix.com/cropped/2024_11_18_8e985d6b9c83aa3e9d0eg-317.jpg?height=175&width=141&top_left_y=739&top_left_x=725) +3. C3 (COL) Let $n \geq 4$ be a fixed positive integer. Given a set $S=$ $\left\{P_{1}, P_{2}, \ldots, P_{n}\right\}$ of points in the plane such that no three are collinear and no four concyclic, let $a_{t}, 1 \leq t \leq n$, be the number of circles $P_{i} P_{j} P_{k}$ that contain $P_{t}$ in their interior, and let + +$$ +m(S)=a_{1}+a_{2}+\cdots+a_{n} +$$ + +Prove that there exists a positive integer $f(n)$, depending only on $n$, such that the points of $S$ are the vertices of a convex polygon if and only if $m(S)=f(n)$. +4. C4 (CZE) Let $n$ and $k$ be positive integers such that $n / 22 a$ and $c>2 b$. Show that there exists a real number $t$ with the property that all the three numbers $t a, t b, t c$ have their fractional parts lying in the interval (1/3,2/3]. +9. A3 (BLR) Find all pairs of functions $f: \mathbb{R} \rightarrow \mathbb{R}, g: \mathbb{R} \rightarrow \mathbb{R}$ such that + +$$ +f(x+g(y))=x f(y)-y f(x)+g(x) \quad \text { for all } x, y \in R +$$ + +10. A4 (GBR) The function $F$ is defined on the set of nonnegative integers and takes nonnegative integer values satisfying the following conditions: For every $n \geq 0$, +(i) $F(4 n)=F(2 n)+F(n)$; +(ii) $F(4 n+2)=F(4 n)+1$; +(iii) $F(2 n+1)=F(2 n)+1$. + +Prove that for each positive integer $m$, the number of integers $n$ with $0 \leq n<2^{m}$ and $F(4 n)=F(3 n)$ is $F\left(2^{m+1}\right)$. +11. A5 (BLR) ${ }^{\mathrm{IMO} 3}$ Let $n \geq 2$ be a positive integer and $\lambda$ a positive real number. Initially there are $n$ fleas on a horizontal line, not all at the same point. We define a move of choosing two fleas at some points $A$ and $B$, with $A$ to the left of $B$, and letting the flea from $A$ jump over the flea from $B$ to the point $C$ such that $B C / A B=\lambda$. +Determine all values of $\lambda$ such that for any point $M$ on the line and for any initial position of the $n$ fleas, there exists a sequence of moves that will take them all to the position right of $M$. +12. A6 (IRE) A nonempty set $A$ of real numbers is called a $B_{3}$-set if the conditions $a_{1}, a_{2}, a_{3}, a_{4}, a_{5}, a_{6} \in A$ and $a_{1}+a_{2}+a_{3}=a_{4}+a_{5}+a_{6}$ imply that the sequences $\left(a_{1}, a_{2}, a_{3}\right)$ and $\left(a_{4}, a_{5}, a_{6}\right)$ are identical up to a permutation. Let $A=\left\{a_{0}=0b>c>d$ be positive integers and suppose + +$$ +a c+b d=(b+d+a-c)(b+d-a+c) +$$ + +Prove that $a b+c d$ is not prime. + +### 3.42.2 Shortlisted Problems + +1. A1 (IND) Let $T$ denote the set of all ordered triples $(p, q, r)$ of nonnegative integers. Find all functions $f: T \rightarrow \mathbb{R}$ such that + +$$ +f(p, q, r)=\left\{\begin{array}{rrr} +0 & & \text { if } p q r=0 \\ +1+\frac{1}{6} & (f(p+1, q-1, r)+f(p-1, q+1, r) & \\ +& +f(p-1, q, r+1)+f(p+1, q, r-1) & \\ +& +f(p, q+1, r-1)+f(p, q-1, r+1)) & \\ +& \text { otherwise. } +\end{array}\right. +$$ + +2. A2 (POL) Let $a_{0}, a_{1}, a_{2}, \ldots$ be an arbitrary infinite sequence of positive numbers. Show that the inequality $1+a_{n}>a_{n-1} \sqrt[n]{2}$ holds for infinitely many positive integers $n$. +3. A3 (ROM) Let $x_{1}, x_{2}, \ldots, x_{n}$ be arbitrary real numbers. Prove the inequality + +$$ +\frac{x_{1}}{1+x_{1}^{2}}+\frac{x_{2}}{1+x_{1}^{2}+x_{2}^{2}}+\cdots+\frac{x_{n}}{1+x_{1}^{2}+\cdots+x_{n}^{2}}<\sqrt{n} +$$ + +4. A4 (LIT) Find all functions $f: \mathbb{R} \rightarrow \mathbb{R}$ satisfying + +$$ +f(x y)(f(x)-f(y))=(x-y) f(x) f(y) +$$ + +for all $x, y$. +5. A5 (BUL) Find all positive integers $a_{1}, a_{2}, \ldots, a_{n}$ such that + +$$ +\frac{99}{100}=\frac{a_{0}}{a_{1}}+\frac{a_{1}}{a_{2}}+\cdots+\frac{a_{n-1}}{a_{n}} +$$ + +where $a_{0}=1$ and $\left(a_{k+1}-1\right) a_{k-1} \geq a_{k}^{2}\left(a_{k}-1\right)$ for $k=1,2, \ldots, n-1$. +6. A6 (KOR) ${ }^{\mathrm{IMO} 2}$ Prove that for all positive real numbers $a, b, c$, + +$$ +\frac{a}{\sqrt{a^{2}+8 b c}}+\frac{a}{\sqrt{b^{2}+8 c a}}+\frac{c}{\sqrt{c^{2}+8 a b}} \geq 1 +$$ + +7. $\mathbf{C 1}$ (COL) Let $A=\left(a_{1}, a_{2}, \ldots, a_{2001}\right)$ be a sequence of positive integers. Let $m$ be the number of 3 -element subsequences $\left(a_{i}, a_{j}, a_{k}\right)$ with $1 \leq i<$ $jb>c>d$ be positive integers and suppose + +$$ +a c+b d=(b+d+a-c)(b+d-a+c) . +$$ + +Prove that $a b+c d$ is not prime. +28. N6 (RUS) Is it possible to find 100 positive integers not exceeding 25,000 such that all pairwise sums of them are different? + +### 3.43 The Forty-Third IMO Glasgow, United Kingdom, July 19-30, 2002 + +### 3.43.1 Contest Problems + +First Day (July 24) + +1. Let $n$ be a positive integer. Each point $(x, y)$ in the plane, where $x$ and $y$ are nonnegative integers with $x+y=n$, is colored red or blue, subject to the following condition: If a point $(x, y)$ is red, then so are all points $\left(x^{\prime}, y^{\prime}\right)$ with $x^{\prime} \leq x$ and $y^{\prime} \leq y$. Let $A$ be the number of ways to choose $n$ blue points with distinct $x$-coordinates, and let $B$ be the number of ways to choose $n$ blue points with distinct $y$-coordinates. Prove that $A=B$. +2. The circle $S$ has center $O$, and $B C$ is a diameter of $S$. Let $A$ be a point of $S$ such that $\measuredangle A O B<120^{\circ}$. Let $D$ be the midpoint of the $\operatorname{arc} A B$ that does not contain $C$. The line through $O$ parallel to $D A$ meets the line $A C$ at $I$. The perpendicular bisector of $O A$ meets $S$ at $E$ and at $F$. Prove that $I$ is the incenter of the triangle $C E F$. +3. Find all pairs of positive integers $m, n \geq 3$ for which there exist infinitely many positive integers $a$ such that + +$$ +\frac{a^{m}+a-1}{a^{n}+a^{2}-1} +$$ + +is itself an integer. +Second Day (July 25) +4. Let $n \geq 2$ be a positive integer, with divisors $1=d_{1} Athens, Greece, July 7-19, 2004 + +### 3.45.1 Contest Problems + +First Day (July 12) + +1. Let $A B C$ be an acute-angled triangle with $A B \neq A C$. The circle with diameter $B C$ intersects the sides $A B$ and $A C$ at $M$ and $N$, respectively. Denote by $O$ the midpoint of $B C$. The bisectors of the angles $B A C$ and $M O N$ intersect at $R$. Prove that the circumcircles of the triangles $B M R$ and $C N R$ have a common point lying on the line segment $B C$. +2. Find all polynomials $P(x)$ with real coefficients that satisfy the equality + +$$ +P(a-b)+P(b-c)+P(c-a)=2 P(a+b+c) +$$ + +for all triples $a, b, c$ of real numbers such that $a b+b c+c a=0$. +3. Determine all $m \times n$ rectangles that can be covered with hooks made up of 6 unit squares, as in the figure: +![](https://cdn.mathpix.com/cropped/2024_11_18_8e985d6b9c83aa3e9d0eg-336.jpg?height=148&width=142&top_left_y=1060&top_left_x=724) + +Rotations and reflections of hooks are allowed. The rectangle must be covered without gaps and overlaps. No part of a hook may cover area outside the rectangle. + +Second Day (July 13) +4. Let $n \geq 3$ be an integer and $t_{1}, t_{2}, \ldots, t_{n}$ positive real numbers such that + +$$ +n^{2}+1>\left(t_{1}+t_{2}+\cdots+t_{n}\right)\left(\frac{1}{t_{1}}+\frac{1}{t_{2}}+\cdots+\frac{1}{t_{n}}\right) . +$$ + +Show that $t_{i}, t_{j}, t_{k}$ are the side lengths of a triangle for all $i, j, k$ with $1 \leq i\left(t_{1}+t_{2}+\cdots+t_{n}\right)\left(\frac{1}{t_{1}}+\frac{1}{t_{2}}+\cdots+\frac{1}{t_{n}}\right) . +$$ + +Show that $t_{i}, t_{j}, t_{k}$ are the side lengths of a triangle for all $i, j, k$ with $1 \leq i0$ and $a b+b c+c a=1$. Prove the inequality + +$$ +\sqrt[3]{\frac{1}{a}+6 b}+\sqrt[3]{\frac{1}{b}+6 c}+\sqrt[3]{\frac{1}{c}+6 a} \leq \frac{1}{a b c} +$$ + +6. A6 (RUS) Find all functions $f: \mathbb{R} \rightarrow \mathbb{R}$ satisfying the equation + +$$ +f\left(x^{2}+y^{2}+2 f(x y)\right)=(f(x+y))^{2} \quad \text { for all } x, y \in \mathbb{R} +$$ + +7. A7 (IRE) Let $a_{1}, a_{2}, \ldots, a_{n}$ be positive real numbers, $n>1$. Denote by $g_{n}$ their geometric mean, and by $A_{1}, A_{2}, \ldots, A_{n}$ the sequence of arithmetic means defined by $A_{k}=\frac{a_{1}+a_{2}+\cdots+a_{k}}{k}, k=1,2, \ldots, n$. Let $G_{n}$ be the geometric mean of $A_{1}, A_{2}, \ldots, A_{n}$. Prove the inequality + +$$ +n \sqrt[n]{\frac{G_{n}}{A_{n}}}+\frac{g_{n}}{G_{n}} \leq n+1 +$$ + +and establish the cases of equality. +8. C1 (PUR) There are 10001 students at a university. Some students join together to form several clubs (a student may belong to different clubs). Some clubs join together to form several societies (a club may belong to different societies). There are a total of $k$ societies. Suppose that the following conditions hold: +(i) Each pair of students are in exactly one club. +(ii) For each student and each society, the student is in exactly one club of the society. +(iii) Each club has an odd number of students. In addition, a club with $2 m+1$ students ( $m$ is a positive integer) is in exactly $m$ societies. +Find all possible values of $k$. +9. C2 (GER) Let $n$ and $k$ be positive integers. There are given $n$ circles in the plane. Every two of them intersect at two distinct points, and all points of intersection they determine are distinct. Each intersection point must be colored with one of $n$ distinct colors so that each color is used at least once, and exactly $k$ distinct colors occur on each circle. Find all values of $n \geq 2$ and $k$ for which such a coloring is possible. +10. C3 (AUS) The following operation is allowed on a finite graph: Choose an arbitrary cycle of length 4 (if there is any), choose an arbitrary edge in that cycle, and delete it from the graph. For a fixed integer $n \geq 4$, find the least number of edges of a graph that can be obtained by repeated applications of this operation from the complete graph on $n$ vertices (where each pair of vertices are joined by an edge). +11. C4 (POL) Consider a matrix of size $n \times n$ whose entries are real numbers of absolute value not exceeding 1 , and the sum of all entries is 0 . Let $n$ be an even positive integer. Determine the least number $C$ such that every such matrix necessarily has a row or a column with the sum of its entries not exceeding $C$ in absolute value. +12. C5 (NZL) Let $N$ be a positive integer. Two players $A$ and $B$, taking turns, write numbers from the set $\{1, \ldots, N\}$ on a blackboard. $A$ begins the game by writing 1 on his first move. Then, if a player has written $n$ on a certain move, his adversary is allowed to write $n+1$ or $2 n$ (provided the number he writes does not exceed $N$ ). The player who writes $N$ wins. We say that $N$ is of type $A$ or of type $B$ according as $A$ or $B$ has a winning strategy. +(a) Determine whether $N=2004$ is of type $A$ or of type $B$. +(b) Find the least $N>2004$ whose type is different from that of 2004. +13. C6 (IRN) For an $n \times n$ matrix $A$, let $X_{i}$ be the set of entries in row $i$, and $Y_{j}$ the set of entries in column $j, 1 \leq i, j \leq n$. We say that $A$ is golden if $X_{1}, \ldots, X_{n}, Y_{1}, \ldots, Y_{n}$ are distinct sets. Find the least integer $n$ such that there exists a $2004 \times 2004$ golden matrix with entries in the set $\{1,2, \ldots, n\}$. +14. C7 (EST) ${ }^{\mathrm{IMO} 3}$ Determine all $m \times n$ rectangles that can be covered with hooks made up of 6 unit squares, as in the figure: +![](https://cdn.mathpix.com/cropped/2024_11_18_8e985d6b9c83aa3e9d0eg-338.jpg?height=153&width=150&top_left_y=1988&top_left_x=722) + +Rotations and reflections of hooks are allowed. The rectangle must be covered without gaps and overlaps. No part of a hook may cover area outside the rectangle. +15. C8 (POL) For a finite graph $G$, let $f(G)$ be the number of triangles and $g(G)$ the number of tetrahedra formed by edges of $G$. Find the least constant $c$ such that + +$$ +g(G)^{3} \leq c \cdot f(G)^{4} \text { for every graph } G +$$ + +16. G1 (ROM) ${ }^{\mathrm{IMO} 1}$ Let $A B C$ be an acute-angled triangle with $A B \neq A C$. The circle with diameter $B C$ intersects the sides $A B$ and $A C$ at $M$ and $N$, respectively. Denote by $O$ the midpoint of $B C$. The bisectors of the angles $B A C$ and $M O N$ intersect at $R$. Prove that the circumcircles of the triangles $B M R$ and $C N R$ have a common point lying on the line segment $B C$. +17. G2 (KAZ) The circle $\Gamma$ and the line $\ell$ do not intersect. Let $A B$ be the diameter of $\Gamma$ perpendicular to $\ell$, with $B$ closer to $\ell$ than $A$. An arbitrary point $C \neq A, B$ is chosen on $\Gamma$. The line $A C$ intersects $\ell$ at $D$. The line $D E$ is tangent to $\Gamma$ at $E$, with $B$ and $E$ on the same side of $A C$. Let $B E$ intersect $\ell$ at $F$, and let $A F$ intersect $\Gamma$ at $G \neq A$. Prove that the reflection of $G$ in $A B$ lies on the line $C F$. +18. G3 (KOR) Let $O$ be the circumcenter of an acute-angled triangle $A B C$ with $\angle B<\angle C$. The line $A O$ meets the side $B C$ at $D$. The circumcenters of the triangles $A B D$ and $A C D$ are $E$ and $F$, respectively. Extend the sides $B A$ and $C A$ beyond $A$, and choose on the respective extension points $G$ and $H$ such that $A G=A C$ and $A H=A B$. Prove that the quadrilateral $E F G H$ is a rectangle if and only if $\angle A C B-\angle A B C=60^{\circ}$. +19. $\mathbf{G} 4 \mathbf{( P O L})^{\mathrm{IMO}}$ In a convex quadrilateral $A B C D$ the diagonal $B D$ does not bisect the angles $A B C$ and $C D A$. The point $P$ lies inside $A B C D$ and satisfies + +$$ +\angle P B C=\angle D B A \quad \text { and } \quad \angle P D C=\angle B D A . +$$ + +Prove that $A B C D$ is a cyclic quadrilateral if and only if $A P=C P$. +20. G5 (SMN) Let $A_{1} A_{2} \ldots A_{n}$ be a regular $n$-gon. The points $B_{1}, \ldots, B_{n-1}$ are defined as follows: +(i) If $i=1$ or $i=n-1$, then $B_{i}$ is the midpoint of the side $A_{i} A_{i+1}$. +(ii) If $i \neq 1, i \neq n-1$, and $S$ is the intersection point of $A_{1} A_{i+1}$ and $A_{n} A_{i}$, then $B_{i}$ is the intersection point of the bisector of the angle $A_{i} S A_{i+1}$ with $A_{i} A_{i+1}$. +Prove the equality + +$$ +\angle A_{1} B_{1} A_{n}+\angle A_{1} B_{2} A_{n}+\cdots+\angle A_{1} B_{n-1} A_{n}=180^{\circ} . +$$ + +21. G6 (GBR) Let $\mathcal{P}$ be a convex polygon. Prove that there is a convex hexagon that is contained in $\mathcal{P}$ and that occupies at least 75 percent of the area of $\mathcal{P}$. +22. G7 (RUS) For a given triangle $A B C$, let $X$ be a variable point on the line $B C$ such that $C$ lies between $B$ and $X$ and the incircles of the triangles $A B X$ and $A C X$ intersect at two distinct points $P$ and $Q$. Prove that the line $P Q$ passes through a point independent of $X$. +23. G8 (SMN) A cyclic quadrilateral $A B C D$ is given. The lines $A D$ and $B C$ intersect at $E$, with $C$ between $B$ and $E$; the diagonals $A C$ and $B D$ intersect at $F$. Let $M$ be the midpoint of the side $C D$, and let $N \neq M$ be a point on the circumcircle of the triangle $A B M$ such that $A N / B N=$ $A M / B M$. Prove that the points $E, F$, and $N$ are collinear. +24. N1 (BLR) Let $\tau(n)$ denote the number of positive divisors of the positive integer $n$. Prove that there exist infinitely many positive integers $a$ such that the equation + +$$ +\tau(a n)=n +$$ + +does not have a positive integer solution $n$. +25. N2 (RUS) The function $\psi$ from the set $\mathbb{N}$ of positive integers into itself is defined by the equality + +$$ +\psi(n)=\sum_{k=1}^{n}(k, n), \quad n \in \mathbb{N} +$$ + +where $(k, n)$ denotes the greatest common divisor of $k$ and $n$. +(a) Prove that $\psi(m n)=\psi(m) \psi(n)$ for every two relatively prime $m, n \in$ $\mathbb{N}$. +(b) Prove that for each $a \in \mathbb{N}$ the equation $\psi(x)=a x$ has a solution. +(c) Find all $a \in \mathbb{N}$ such that the equation $\psi(x)=a x$ has a unique solution. +26. N3 (IRN) A function $f$ from the set of positive integers $\mathbb{N}$ into itself is such that for all $m, n \in \mathbb{N}$ the number $\left(m^{2}+n\right)^{2}$ is divisible by $f^{2}(m)+$ $f(n)$. Prove that $f(n)=n$ for each $n \in \mathbb{N}$. +27. N4 (POL) Let $k$ be a fixed integer greater than 1 , and let $m=4 k^{2}-5$. Show that there exist positive integers $a$ and $b$ such that the sequence $\left(x_{n}\right)$ defined by + +$$ +x_{0}=a, \quad x_{1}=b, \quad x_{n+2}=x_{n+1}+x_{n} \quad \text { for } \quad n=0,1,2, \ldots +$$ + +has all of its terms relatively prime to $m$. +28. N5 (IRN) ${ }^{\mathrm{IMO} 6}$ We call a positive integer alternate if its decimal digits are alternately odd and even. Find all positive integers $n$ such that $n$ has an alternate multiple. +29. N6 (IRE) Given an integer $n>1$, denote by $P_{n}$ the product of all positive integers $x$ less than $n$ and such that $n$ divides $x^{2}-1$. For each $n>1$, find the remainder of $P_{n}$ on division by $n$. +30. N7 (BUL) Let $p$ be an odd prime and $n$ a positive integer. In the coordinate plane, eight distinct points with integer coordinates lie on a circle with diameter of length $p^{n}$. Prove that there exists a triangle with vertices at three of the given points such that the squares of its side lengths are integers divisible by $p^{n+1}$. + +## Solutions + +### 4.1 Solutions to the Contest Problems of IMO 1959 + +1. The desired result $(14 n+3,21 n+4)=1$ follows from + +$$ +3(14 n+3)-2(21 n+4)=1 +$$ + +2. For the square roots to be real we must have $2 x-1 \geq 0 \Rightarrow x \geq 1 / 2$ and $x \geq \sqrt{2 x-1} \Rightarrow x^{2} \geq 2 x-1 \Rightarrow(x-1)^{2} \geq 0$, which always holds. Then we have $\sqrt{x+\sqrt{2 x-1}}+\sqrt{x-\sqrt{2 x-1}}=c \Longleftrightarrow$ + +$$ +c^{2}=2 x+2 \sqrt{x^{2}-\sqrt{2 x-1}^{2}}=2 x+2|x-1|= \begin{cases}2, & 1 / 2 \leq x \leq 1 \\ 4 x-2, & x \geq 1\end{cases} +$$ + +(a) $c^{2}=2$. The equation holds for $1 / 2 \leq x \leq 1$. +(b) $c^{2}=1$. The equation has no solution. +(c) $c^{2}=4$. The equation holds for $4 x-2=4 \Rightarrow x=3 / 2$. +3. Multiplying the equality by $4\left(a \cos ^{2} x-b \cos x+c\right)$, we obtain $4 a^{2} \cos ^{4} x+$ $2\left(4 a c-2 b^{2}\right) \cos ^{2} x+4 c^{2}=0$. Plugging in $2 \cos ^{2} x=1+\cos 2 x$ we obtain (after quite a bit of manipulation): + +$$ +a^{2} \cos ^{2} 2 x+\left(2 a^{2}+4 a c-2 b^{2}\right) \cos 2 x+\left(a^{2}+4 a c-2 b^{2}+4 c^{2}\right)=0 +$$ + +For $a=4, b=2$, and $c=-1$ we get $4 \cos ^{2} x+2 \cos x-1=0$ and $16 \cos ^{2} 2 x+8 \cos 2 x-4=0 \Rightarrow 4 \cos ^{2} 2 x+2 \cos 2 x-1=0$. +4. Analysis. Let $a$ and $b$ be the other two sides of the triangle. From the conditions of the problem we have $c^{2}=a^{2}+b^{2}$ and $c / 2=\sqrt{a b} \Leftrightarrow 3 / 2 c^{2}=$ $a^{2}+b^{2}+2 a b=(a+b)^{2} \Leftrightarrow \sqrt{3 / 2} c=a+b$. Given a desired $\triangle A B C$ let $D$ be a point on $(A C$ such that $C D=C B$. In that case, $A D=a+b=\sqrt{3 / 2} c$, and also, since $B C=C D$, it follows that $\angle A D B=45^{\circ}$. +Construction. From a segment of length $c$ we elementarily construct a segment $A D$ of length $\sqrt{3 / 2} c$. We then construct a ray ( $D X$ such that +$\angle A D X=45^{\circ}$ and a circle $k(A, c)$ that intersects the ray at point $B$. Finally, we construct the perpendicular from $B$ to $A D$; point $C$ is the foot of that perpendicular. +Proof. It holds that $A B=c$, and, since $C B=C D$, it also holds that $A C+$ $C B=A C+C D=A D=\sqrt{3 / 2} c$. From this it follows that $\sqrt{A C \cdot C B}=$ $c / 2$. Since $B C$ is perpendicular to $A D$, it follows that $\measuredangle B C A=90^{\circ}$. Thus $A B C$ is the desired triangle. +Discussion. Since $A B \sqrt{2}=\sqrt{2} c>\sqrt{3 / 2} c=A D>A B$, the circle $k$ intersects the ray $D X$ in exactly two points, which correspond to two symmetric solutions. +5. (a) It suffices to prove that $A F \perp B C$, since then for the intersection point $X$ we have $\angle A X C=\angle B X F=90^{\circ}$, implying that $X$ belongs to the circumcircles of both squares and thus that $X=N$. The relation $A F \perp B C$ holds because from $M A=M C, M F=M B$, and $\angle A M C=\angle F M B$ it follows that $\triangle A M F$ is obtained by rotating $\triangle B M C$ by $90^{\circ}$ around $M$. +(b) Since $N$ is on the circumcircle of $B M F E$, it follows that $\angle A N M=$ $\angle M N B=45^{\circ}$. Hence $M N$ is the bisector of $\angle A N B$. It follows that $M N$ passes through the midpoint of the $\operatorname{arc} \widehat{A B}$ of the circle with diameter $A B$ (i.e., the circumcircle of $\triangle A B N$ ) not containing $N$. +(c) Let us introduce a coordinate system such that $A=(0,0), B=(b, 0)$, and $M=(m, 0)$. Setting in general $W=\left(x_{W}, y_{W}\right)$ for an arbitrary point $W$ and denoting by $R$ the midpoint of $P Q$, we have $y_{R}=\left(y_{P}+\right.$ $\left.y_{Q}\right) / 2=(m+b-m) / 4=b / 4$ and $x_{R}=\left(x_{P}+x_{Q}\right) / 2=(m+m+b) / 4=$ $(2 m+b) / 4$, the parameter $m$ varying from 0 to $b$. Thus the locus of all points $R$ is the closed segment $R_{1} R_{2}$ where $R_{1}=(b / 4, b / 4)$ and $R_{2}=(b / 4,3 b / 4)$. +6. Analysis. For $A B \| C D$ to hold evidently neither must intersect $p$ and hence constructing lines $r$ in $\alpha$ through $A$ and $s$ in $\beta$ through $C$, both being parallel to $p$, we get that $B \in r$ and $D \in s$. Hence the problem reduces to a planar problem in $\gamma$, determined by $r$ and $s$. Denote by $A^{\prime}$ the foot of the perpendicular from $A$ to $s$. Since $A B C D$ is isosceles and has an incircle, it follows $A D=B C=(A B+C D) / 2=A^{\prime} C$. The remaining parts of the problem are now obvious. + +### 4.2 Solutions to the Contest Problems of IMO 1960 + +1. Given the number $\overline{a c b}$, since $11 \mid \overline{a c b}$, it follows that $c=a+b$ or $c=$ $a+b-11$. In the first case, $a^{2}+b^{2}+(a+b)^{2}=10 a+b$, and in the second case, $a^{2}+b^{2}+(a+b-11)^{2}=10(a-1)+b$. In the first case the LHS is even, and hence $b \in\{0,2,4,6,8\}$, while in the second case it is odd, and hence $b \in\{1,3,5,7,9\}$. Analyzing the 10 quadratic equations for $a$ we obtain that the only valid solutions are 550 and 803. +2. The LHS term is well-defined for $x \geq-1 / 2$ and $x \neq 0$. Furthermore, $4 x^{2} /(1-\sqrt{1+2 x})^{2}=(1+\sqrt{1+2 x})^{2}$. Since $f(x)=(1+\sqrt{1+2 x})^{2}-2 x-$ $9=2 \sqrt{1+2 x}-7$ is increasing and since $f(45 / 8)=0$, it follows that the inequality holds precisely for $-1 / 2 \leq x<45 / 8$ and $x \neq 0$. +3. Let $B^{\prime} C^{\prime}$ be the middle of the $n=2 k+1$ segments and let $D$ be the foot of the perpendicular from $A$ to the hypotenuse. Let us assume $\mathcal{B}\left(C, D, C^{\prime}, B^{\prime}, B\right)$. Then from $C Da b$ there are two solutions, if $h^{2}=a b$ there is only one solution, and if $h^{2}
a^{2}>b^{2}$ and $a>0$. +2. Using $S=b c \sin \alpha / 2, a^{2}=b^{2}+c^{2}-2 b c \cos \alpha$ and $(\sqrt{3} \sin \alpha+\cos \alpha) / 2=$ $\cos \left(\alpha-60^{\circ}\right)$ we have + +$$ +\begin{gathered} +a^{2}+b^{2}+c^{2} \geq 4 S \sqrt{3} \Leftrightarrow b^{2}+c^{2} \geq b c(\sqrt{3} \sin \alpha+\cos \alpha) \Leftrightarrow \\ +\Leftrightarrow(b-c)^{2}+2 b c\left(1-\cos \left(\alpha-60^{\circ}\right)\right) \geq 0, +\end{gathered} +$$ + +where equality holds if and only if $b=c$ and $\alpha=60^{\circ}$, i.e., if the triangle is equilateral. +3. For $n \geq 2$ we have + +$$ +\begin{aligned} +1 & =\cos ^{n} x-\sin ^{n} x \leq\left|\cos ^{n} x-\sin ^{n} x\right| \\ +& \leq\left|\cos ^{n} x\right|+\left|\sin ^{n} x\right| \leq \cos ^{2} x+\sin ^{2} x=1 +\end{aligned} +$$ + +Hence $\sin ^{2} x=\left|\sin ^{n} x\right|$ and $\cos ^{2} x=\left|\cos ^{n} x\right|$, from which it follows that $\sin x, \cos x \in\{1,0,-1\} \Rightarrow x \in \pi \mathbb{Z} / 2$. By inspection one obtains the set of solutions +$\{m \pi \mid m \in \mathbb{Z}\}$ for even $n$ and $\{2 m \pi, 2 m \pi-\pi / 2 \mid m \in \mathbb{Z}\}$ for odd $n$. +For $n=1$ we have $1=\cos x-\sin x=-\sqrt{2} \sin (x-\pi / 4)$, which yields the set of solutions + +$$ +\{2 m \pi, 2 m \pi-\pi / 2 \mid m \in \mathbb{Z}\} +$$ + +4. Let $x_{i}=P P_{i} / P Q_{i}$ for $i=1,2,3$. For all $i$ we have + +$$ +\frac{1}{x_{i}+1}=\frac{P Q_{i}}{P_{i} Q_{i}}=\frac{S_{P P_{j} P_{k}}}{S_{P_{1} P_{2} P_{3}}} +$$ + +where the indices $j$ and $k$ are distinct and different from $i$. Hence we have + +$$ +\begin{aligned} +f\left(x_{1}, x_{2}, x_{3}\right) & =\frac{1}{x_{1}+1}+\frac{1}{x_{2}+1}+\frac{1}{x_{3}+1} \\ +& =\frac{S\left(P P_{2} P_{3}\right)+S\left(P P_{1} P_{3}\right)+S\left(P P_{2} P_{3}\right)}{S\left(P_{1} P_{2} P_{3}\right)}=1 +\end{aligned} +$$ + +It follows that $1 /\left(x_{i}+1\right) \geq 1 / 3$ for some $i$ and $1 /\left(x_{j}+1\right) \leq 1 / 3$ for some $j$. Consequently, $x_{i} \leq 2$ and $x_{j} \geq 2$. +5. Analysis. Let $C_{1}$ be the midpoint of $A B$. In $\triangle A M B$ we have $M C_{1}=b / 2$, $A B=c$, and $\angle A M B=\omega$. Thus, given $A B=c$, the point $M$ is at the intersection of the circle $k\left(C^{\prime}, b / 2\right)$ and the set of points $e$ that view $A B$ at an angle of $\omega$. The construction of $A B C$ is now obvious. +Discussion. It suffices to establish the conditions for which $k$ and $e$ intersect. Let $E$ be the midpoint of one of the arcs that make up $e$. A necessary and sufficient condition for $k$ to intersect $e$ is + +$$ +\frac{c}{2}=C^{\prime} A \leq \frac{b}{2} \leq C^{\prime} E=\frac{c}{2} \cot \frac{\omega}{2} \Leftrightarrow b \tan \frac{\omega}{2} \leq c1 / 2$ and $f(1-\sqrt{31} / 8)=\sqrt{(1 / 4+\sqrt{31} / 4)^{2}}-$ $\sqrt{(1 / 4-\sqrt{31} / 4)^{2}}=1 / 2$. Hence the inequality is satisfied for $-1 \leq x<$ $1-\sqrt{31} / 8$. +3. By inspecting the four different stages of this periodic motion we easily obtain that the locus of the midpoints of $X Y$ is the edges of $M N C Q$, where $M, N$, and $Q$ are the centers of $A B B^{\prime} A^{\prime}, B C C^{\prime} B^{\prime}$, and $A B C D$, respectively. +4. Since $\cos 2 x=1+\cos ^{2} x$ and $\cos \alpha+\cos \beta=2 \cos \left(\frac{\alpha+\beta}{2}\right) \cos \left(\frac{\alpha-\beta}{2}\right)$, we have $\cos ^{2} x+\cos ^{2} 2 x+\cos ^{2} 3 x=1 \Leftrightarrow \cos 2 x+\cos 4 x+2 \cos ^{2} 3 x=$ $2 \cos 3 x(\cos x+\cos 3 x)=0 \Leftrightarrow 4 \cos 3 x \cos 2 x \cos x=0$. Hence the solutions are $x \in\{\pi / 2+m \pi, \pi / 4+m \pi / 2, \pi / 6+m \pi / 3 \mid m \in \mathbb{Z}\}$. +5. Analysis. Let $A B C D$ be the desired quadrilateral. Let us assume w.l.o.g. that $A B>B C$ (for $A B=B C$ the construction is trivial). For a tangent quadrilateral we have $A D-D C=A B-B C$. Let $X$ be a point on $A D$ such that $D X=D C$. We then have $A X=A B-B C$ and $\measuredangle A X C=$ $\measuredangle A D C+\measuredangle C D X=180^{\circ}-\angle A B C / 2$. Constructing $X$ and hence $D$ is now obvious. +6. This problem is a special case, when the triangle is isosceles, of Euler's formula, which holds for all triangles. +7. The spheres are arranged in a similar manner as in the planar case where we have one incircle and three excircles. Here we have one "insphere" and four "exspheres" corresponding to each of the four sides. Each vertex of the tetrahedron effectively has three tangent lines drawn from it to each of the five spheres. Repeatedly using the equality of the three tangent segments from a vertex (in the same vein as for tangent planar quadrilaterals) we obtain $S A+B C=S B+C A=S C+A B$ from the insphere. From the exsphere opposite of $S$ we obtain $S A-B C=S B-C A=S C-A B$, hence $S A=S B=S C$ and $A B=B C=C A$. By symmetry, we also have $A B=A C=A S$. Hence indeed, all the edges of the tetrahedron are equal in length and thus we have shown that the tetrahedron is regular. + +### 4.5 Solutions to the Contest Problems of IMO 1963 + +1. Obviously, $x \geq 0$; hence squaring the given equation yields an equivalent equation $5 x^{2}-p-4+4 \sqrt{\left(x^{2}-1\right)\left(x^{2}-p\right)}=x^{2}$, i.e., $4 \sqrt{\left(x^{2}-1\right)\left(x^{2}-p\right)}=$ $(p+4)-4 x^{2}$. If $4 x^{2} \leq(p+4)$, we may square the equation once again to get $-16(p+1) x^{2}+16 p=-8(p+4) x^{2}+(p+4)^{2}$, which is equivalent to $x^{2}=(4-p)^{2} /[4(4-2 p)]$, i.e., $x=(4-p) /(2 \sqrt{4-2 p})$. For this to be a solution we must have $p \leq 2$ and $(4-p)^{2} /(4-2 p)=4 x^{2} \leq(p+4)$. Hence $4 / 3 \leq p \leq 2$. Otherwise there is no solution. +2. Let $A$ be the given point, $B C$ the given segment, and $\mathcal{B}_{1}, \mathcal{B}_{2}$ the closed balls with the diameters $A B$ and $A C$ respectively. Consider one right angle $\angle A O K$ with $K \in[B C]$. If $B^{\prime}, C^{\prime}$ are the feet of the perpendiculars from $B, C$ to $A O$ respectively, then $O$ lies on the segment $B^{\prime} C^{\prime}$, which implies that it lies on exactly one of the segments $A B^{\prime}, A C^{\prime}$. Hence $O$ belongs to exactly one of the balls $\mathcal{B}_{1}, \mathcal{B}_{2}$; i.e., $O \in \mathcal{B}_{1} \Delta \mathcal{B}_{2}$. This is obviously the required locus. +3. Let $\overrightarrow{O A_{1}}, \overrightarrow{O A_{2}}, \ldots, \overrightarrow{O A_{n}}$ be the vectors corresponding respectively to the edges $a_{1}, a_{2}, \ldots, a_{n}$ of the polygon. By the conditions of the problem, these vectors satisfy $\overrightarrow{O A_{1}}+\cdots+\overrightarrow{O A_{n}}=\overrightarrow{0}, \angle A_{1} O A_{2}=\angle A_{2} O A_{3}=\cdots=$ $\angle A_{n} O A_{1}=2 \pi / n$ and $O A_{1} \geq O A_{2} \geq \cdots \geq O A_{n}$. Our task is to prove that $O A_{1}=\cdots=O A_{n}$. +Let $l$ be the line through $O$ perpendicular to $O A_{n}$, and $B_{1}, \ldots, B_{n-1}$ the projections of $A_{1}, \ldots, A_{n-1}$ onto $l$ respectively. By the assumptions, the sum of the $\overrightarrow{O B_{i}}$ 's is $\overrightarrow{0}$. On the other hand, since $O B_{i} \leq O B_{n-i}$ for all $i \leq n / 2$, all the sums $\overrightarrow{O B_{i}}+\overrightarrow{O B_{n-i}}$ lie on the same side of the point $O$. Hence all these sums must be equal to $\overrightarrow{0}$. Consequently, $O A_{i}=O A_{n-i}$, from which the result immediately follows. +4. Summing up all the equations yields $2\left(x_{1}+x_{2}+x_{3}+x_{4}+x_{5}\right)=y\left(x_{1}+\right.$ $x_{2}+x_{3}+x_{4}+x_{5}$ ). If $y=2$, then the given equations imply $x_{1}-x_{2}=$ $x_{2}-x_{3}=\cdots=x_{5}-x_{1}$; hence $x_{1}=x_{2}=\cdots=x_{5}$, which is clearly a solution. If $y \neq 2$, then $x_{1}+\cdots+x_{5}=0$, and summing the first three equalities gives $x_{2}=y\left(x_{1}+x_{2}+x_{3}\right)$. Using that $x_{1}+x_{3}=y x_{2}$ we obtain $x_{2}=\left(y^{2}+y\right) x_{2}$, i.e., $\left(y^{2}+y-1\right) x_{2}=0$. If $y^{2}+y-1 \neq 0$, then $x_{2}=0$, and similarly $x_{1}=\cdots=x_{5}=0$. If $y^{2}+y-1=0$, it is easy to prove that the last two equations are the consequence of the first three. Thus choosing any values for $x_{1}$ and $x_{5}$ will give exactly one solution for $x_{2}, x_{3}, x_{4}$. +5. The LHS of the desired identity equals $S=\cos (\pi / 7)+\cos (3 \pi / 7)+$ $\cos (5 \pi / 7)$. Now + +$$ +S \sin \frac{\pi}{7}=\frac{\sin \frac{2 \pi}{7}}{2}+\frac{\sin \frac{4 \pi}{7}-\sin \frac{2 \pi}{7}}{2}+\frac{\sin \frac{6 \pi}{7}-\sin \frac{4 \pi}{7}}{2}=\frac{\sin \frac{6 \pi}{7}}{2} \Rightarrow S=\frac{1}{2} . +$$ + +6. The result is $E D A C B$. + +### 4.6 Solutions to the Contest Problems of IMO 1964 + +1. Let $n=3 k+r$, where $0 \leq r<2$. Then $2^{n}=2^{3 k+r}=8^{k} \cdot 2^{r} \equiv 2^{r}(\bmod 7)$. Thus the remainder of $2^{n}$ modulo 7 is $1,2,4$ if $n \equiv 0,1,2(\bmod 3)$. Hence $2^{n}-1$ is divisible by 7 if and only if $3 \mid n$, while $2^{n}+1$ is never divisible by 7 . +2. By substituting $a=x+y, b=y+z$, and $c=z+x(x, y, z>0)$ the given inequality becomes + +$$ +6 x y z \leq x^{2} y+x y^{2}+y^{2} z+y z^{2}+z^{2} x+z x^{2}, +$$ + +which follows immediately by the AM-GM inequality applied to $x^{2} y, x y^{2}$, $x^{2} z, x z^{2}, y^{2} z, y z^{2}$. +3. Let $r$ be the radius of the incircle of $\triangle A B C, r_{a}, r_{b}, r_{c}$ the radii of the smaller circles corresponding to $A, B, C$, and $h_{a}, h_{b}, h_{c}$ the altitudes from $A, B, C$ respectively. The coefficient of similarity between the smaller triangle at $A$ and the triangle $A B C$ is $1-2 r / h_{a}$, from which we easily obtain $r_{a}=\left(h_{a}-2 r\right) r / h_{a}=(s-a) r / s$. Similarly, $r_{b}=(s-b) r / s$ and $r_{c}=(s-c) r / s$. Now a straightforward computation gives that the sum of areas of the four circles is given by + +$$ +\Sigma=\frac{(b+c-a)(c+a-b)(a+b-c)\left(a^{2}+b^{2}+c^{2}\right) \pi}{(a+b+c)^{3}} +$$ + +4. Let us call the topics $T_{1}, T_{2}, T_{3}$. Consider an arbitrary student $A$. By the pigeonhole principle there is a topic, say $T_{3}$, he discussed with at least 6 other students. If two of these 6 students discussed $T_{3}$, then we are done. Suppose now that the 6 students discussed only $T_{1}$ and $T_{2}$ and choose one of them, say $B$. By the pigeonhole principle he discussed one of the topics, say $T_{2}$, with three of these students. If two of these three students also discussed $T_{2}$, then we are done. Otherwise, all the three students discussed only $T_{1}$, which completes the task. +5. Let us first compute the number of intersection points of the perpendiculars passing through two distinct points $B$ and $C$. The perpendiculars from $B$ to the lines through $C$ other than $B C$ meet all perpendiculars from $C$, which counts to $3 \cdot 6=18$ intersection points. Each perpendicular from $B$ to the 3 lines not containing $C$ can intersect at most 5 of the perpendiculars passing through $C$, which counts to another $3 \cdot 5=15$ intersection points. Thus there are $18+15=33$ intersection points corresponding to $B, C$. +It follows that the required total number is at most $10 \cdot 33=330$. But some of these points, namely the orthocenters of the triangles with vertices at the given points, are counted thrice. There are 10 such points. Hence the maximal number of intersection points is $330-2 \cdot 10=310$. + +Remark. The jury considered only the combinatorial part of the problem and didn't require an example in which 310 points appear. However, it is "easily" verified that, for instance, the set of points $A(1,1), B(e, \pi)$, $C\left(e^{2}, \pi^{2}\right), D\left(e^{3}, \pi^{3}\right), E\left(e^{4}, \pi^{4}\right)$ works. +6. We shall prove that the statement is valid in the general case, for an arbitrary point $D_{1}$ inside $\triangle A B C$. Since $D_{1}$ belongs to the plane $A B C$, there are real numbers $a, b, c$ such that $(a+b+c) \overrightarrow{D D_{1}}=a \overrightarrow{D A}+b \overrightarrow{D B}+c \overrightarrow{D C}$. Since $A A_{1} \| D D_{1}$, it holds that $\overrightarrow{A A_{1}}=k \overrightarrow{D D_{1}}$ for some $k \in \mathbb{R}$. Now it is easy to get $\overrightarrow{D A_{1}}=-(b \overrightarrow{D B}+c \overrightarrow{D C}) / a, \overrightarrow{D B_{1}}=-(a \overrightarrow{D A}+c \overrightarrow{D C}) / b$, and $\overrightarrow{D C_{1}}=-(a \overrightarrow{D A}+b \overrightarrow{D B}) / c$. This implies + +$$ +\begin{aligned} +& \overrightarrow{D_{1} A_{1}}=-\frac{a^{2} \overrightarrow{D A}+b(a+2 b+c) \overrightarrow{D B}+c(a+b+2 c) \overrightarrow{D C}}{a(a+b+c)} \\ +& \overrightarrow{D_{1} B_{1}}=-\frac{a(2 a+b+c) \overrightarrow{D A}+b^{2} \overrightarrow{D B}+c(a+b+2 c) \overrightarrow{D C}}{b(a+b+c)}, \text { and } \\ +& \overrightarrow{D_{1} C_{1}}=-\frac{a(2 a+b+c) \overrightarrow{D A}+b(a+2 b+c) \overrightarrow{D B}+c^{2} \overrightarrow{D C}}{c(a+b+c)} +\end{aligned} +$$ + +By using + +$$ +6 V_{D_{1} A_{1} B_{1} C_{1}}=\left|\left[\overrightarrow{D_{1} A_{1}}, \overrightarrow{D_{1} B_{1}}, \overrightarrow{D_{1} C_{1}}\right]\right| \text { and } 6 V_{D A B C}=|[\overrightarrow{D A}, \overrightarrow{D B}, \overrightarrow{D C}]| +$$ + +we get + +$$ +V_{D_{1} A_{1} B_{1} C_{1}}=\frac{\left\|\begin{array}{ccc} +a^{2} & b(a+2 b+c) & c(a+b+2 c) \\ +a(2 a+b+c) & b^{2} & c(a+b+2 c) \\ +a(2 a+b+c) b(a+2 b+c) & c^{2} +\end{array}\right\|}{6 a b c(a+b+c)^{3}}=3 V_{D A B C} +$$ + +### 4.7 Solutions to the Contest Problems of IMO 1965 + +1. Let us set $S=|\sqrt{1+\sin 2 x}-\sqrt{1-\sin 2 x}|$. Observe that $S^{2}=2-$ $2 \sqrt{1-\sin ^{2} 2 x}=2-2|\cos 2 x| \leq 2$, implying $S \leq \sqrt{2}$. Thus the righthand inequality holds for all $x$. +It remains to investigate the left-hand inequality. If $\pi / 2 \leq x \leq 3 \pi / 2$, then $\cos x \leq 0$ and the inequality trivially holds. Assume now that $\cos x>$ 0 . Then the inequality is equivalent to $2+2 \cos 2 x=4 \cos ^{2} x \leq S^{2}=$ $2-2|\cos 2 x|$, which is equivalent to $\cos 2 x \leq 0$, i.e., to $x \in[\pi / 4, \pi / 2] \cup$ $[3 \pi / 2,7 \pi / 4]$. Hence the solution set is $\pi / 4 \leq x \leq 7 \pi / 4$. +2. Suppose that $\left(x_{1}, x_{2}, x_{3}\right)$ is a solution. We may assume w.l.o.g. that $\left|x_{1}\right| \geq$ $\left|x_{2}\right| \geq\left|x_{3}\right|$. Suppose that $\left|x_{1}\right|>0$. From the first equation we obtain that + +$$ +0=\left|x_{1}\right| \cdot\left|a_{11}+a_{12} \frac{x_{2}}{x_{1}}+a_{13} \frac{x_{3}}{x_{1}}\right| \geq\left|x_{1}\right| \cdot\left(a_{11}-\left|a_{12}\right|-\left|a_{13}\right|\right)>0 +$$ + +which is a contradiction. Hence $\left|x_{1}\right|=0$ and consequently $x_{1}=x_{2}=x_{3}=$ 0. +3. Let $d$ denote the distance between the lines $A B$ and $C D$. Being parallel to $A B$ and $C D$, the plane $\pi$ intersects the faces of the tetrahedron in a parallelogram $E F G H$. Let $X \in A B$ be a points such that $H X \| D B$. +Clearly $V_{A E H B F G}=V_{A X E H}+$ $V_{X E H B F G}$. Let $M N$ be the common perpendicular to lines $A B$ and $C D(M \in A B, N \in C D)$ and let $M N, B N$ meet the plane $\pi$ at $Q$ and $R$ respectively. Then it holds that $B R / R N=M Q / Q N=k$ and consequently $A X / X B=A E / E C=$ $A H / H D=B F / F C=B G / G D=$ $k$. Now we have $V_{A X E H} / V_{A B C D}=$ +![](https://cdn.mathpix.com/cropped/2024_11_18_8e985d6b9c83aa3e9d0eg-352.jpg?height=417&width=535&top_left_y=1138&top_left_x=819) +$k^{3} /(k+1)^{3}$ 。 +Furthermore, if $h=3 V_{A B C D} / S_{A B C}$ is the height of $A B C D$ from $D$, then + +$$ +\begin{aligned} +V_{X E H B F G} & =\frac{1}{2} S_{X B F E} \frac{k}{k+1} h \text { and } \\ +S_{X B F E} & =S_{A B C}-S_{A X E}-S_{E F C}=\frac{(k+1)^{2}-1-k^{2}}{(k+1)^{2}}=\frac{2 k}{(1+k)^{2}} +\end{aligned} +$$ + +These relations give us $V_{X E H B F G} / V_{A B C D}=3 k^{2} /(1+k)^{3}$. Finally, + +$$ +\frac{V_{A E H B F G}}{V_{A B C D}}=\frac{k^{3}+3 k^{2}}{(k+1)^{3}} +$$ + +Similarly, $V_{C E F D H G} / V_{A B C D}=(3 k+1) /(k+1)^{3}$, and hence the required ratio is $\left(k^{3}+3 k^{2}\right) /(3 k+1)$. +4. It is easy to see that all $x_{i}$ are nonzero. Let $x_{1} x_{2} x_{3} x_{4}=p$. The given system of equations can be rewritten as $x_{i}+p / x_{i}=2, i=1,2,3,4$. The equation $x+p / x=2$ has at most two real solutions, say $y$ and $z$. Then each $x_{i}$ is equal either to $y$ or to $z$. There are three cases: +(i) $x_{1}=x_{2}=x_{3}=x_{4}=y$. Then $y+y^{3}=2$ and hence $y=1$. +(ii) $x_{1}=x_{2}=x_{3}=y, x_{4}=z$. Then $z+y^{3}=y+y^{2} z=2$. It is easy to obtain that the only possibilities for $(y, z)$ are $(-1,3)$ and $(1,1)$. +(iii) $x_{1}=x_{2}=y, x_{3}=x_{4}$. In this case the only possibility is $y=z=1$. + +Hence the solutions for $\left(x_{1}, x_{2}, x_{3}, x_{4}\right)$ are $(1,1,1,1),(-1,-1,-1,3)$, and the cyclic permutations. +5. (a) Let $A^{\prime}$ and $B^{\prime}$ denote the feet of the perpendiculars from $A$ and $B$ to $O B$ and $O A$ respectively. We claim that $H \in A^{\prime} B^{\prime}$. Indeed, since $M P H Q$ is a parallelogram, we have $B^{\prime} P / B^{\prime} A=B M / B A=$ $M Q / A A^{\prime}=P H / A A^{\prime}$, which implies by Thales's theorem that $H \in$ $A^{\prime} B^{\prime}$. It is easy to see that the locus of $H$ is the whole segment $A^{\prime} B^{\prime}$. +(b) In this case the locus of points $H$ is obviously the interior of the triangle $O A^{\prime} B^{\prime}$. +6. We recall the simple statement that every two diameters of a set must have a common point. +Consider any point $B$ that is an endpoint of $k \geq 2$ diameters $B C_{1}, B C_{2}$, $\ldots, B C_{k}$. We may assume w.l.o.g. that all the points $C_{1}, \ldots, C_{k}$ lie on the $\operatorname{arc} C_{1} C_{k}$, whose center is $B$ and measure does not exceed $60^{\circ}$. We observe that for $1$ $S\left(K L C_{1}\right)>S\left(K B_{1} C_{1}\right)=S\left(A_{1} B_{1} C_{1}\right)=S / 4$. Hence, by the pigeonhole principle one of the remaining three triangles $\triangle M A L, \triangle K B M$, and $\triangle L C K$ must have an area less than or equal to $S / 4$. This completes the proof. + +### 4.9 Solutions to the Longlisted Problems of IMO 1967 + +1. Let us denote the $n$th term of the given sequence by $a_{n}$. Then + +$$ +\begin{aligned} +a_{n} & =\frac{1}{3}\left(\frac{10^{3 n+3}-10^{2 n+3}}{9}+7 \frac{10^{2 n+2}-10^{n+1}}{9}+\frac{10^{n+2}-1}{9}\right) \\ +& =\frac{1}{27}\left(10^{3 n+3}-3 \cdot 10^{2 n+2}+3 \cdot 10^{n+1}-1\right)=\left(\frac{10^{n+1}-1}{3}\right)^{3} . +\end{aligned} +$$ + +2. $(n!)^{2 / n}=\left((1 \cdot 2 \cdots n)^{1 / n}\right)^{2} \leq\left(\frac{1+2+\cdots+n}{n}\right)^{2}=\left(\frac{n+1}{2}\right)^{2} \leq \frac{1}{3} n^{2}+\frac{1}{2} n+\frac{1}{6}$. +3. Consider the function $f:[0, \pi / 2] \rightarrow \mathbb{R}$ defined by $f(x)=1-x^{2} / 2+$ $x^{4} / 16-\cos x$. +It is easy to calculate that $f^{\prime}(0)=f^{\prime \prime}(0)=f^{\prime \prime \prime}(0)=0$ and $f^{\prime \prime \prime \prime}(x)=$ $3 / 2-\cos x$. +Since $f^{\prime \prime \prime \prime}(x)>0, f^{\prime \prime \prime}(x)$ is increasing. Together with $f^{\prime \prime \prime}(0)=0$, this gives $f^{\prime \prime \prime}(x)>0$ for $x>0$; hence $f^{\prime \prime}(x)$ is increasing, etc. Continuing in the same way we easily conclude that $f(x)>0$. +4. (a) Let $A B C D$ be a parallelogram, and $K, L$ the midpoints of segments $B C$ and $C D$ respectively. The sides of $\triangle A K L$ are equal and parallel to the medians of $\triangle A B C$. +(b) Using the formulas $4 m_{a}^{2}=2 b^{2}+2 c^{2}-a^{2}$ etc., it is easy to obtain that $m_{a}^{2}+m_{b}^{2}=m_{c}^{2}$ is equivalent to $a^{2}+b^{2}=5 c^{2}$. Then + +$$ +5\left(a^{2}+b^{2}-c^{2}\right)=4\left(a^{2}+b^{2}\right) \geq 8 a b +$$ + +5. If one of $x, y, z$ is equal to 1 or -1 , then we obtain solutions $(-1,-1,-1)$ and $(1,1,1)$. We claim that these are the only solutions to the system. Let $f(t)=t^{2}+t-1$. If among $x, y, z$ one is greater than 1 , say $x>1$, we have $xf(x)=y>f(y)=z>f(z)=x$, a contradiction. This proves our claim. +6. The given system has two solutions: $(-2,-1)$ and $(-14 / 3,13 / 3)$. +7. Let $S_{k}=x_{1}^{k}+x_{2}^{k}+\cdots+x_{n}^{k}$ and let $\sigma_{k}, k=1,2, \ldots, n$ denote the $k$ th elementary symmetric polynomial in $x_{1}, \ldots, x_{n}$. The given system can be written as $S_{k}=a^{k}, k=1, \ldots, n$. Using Newton's formulas + +$$ +k \sigma_{k}=S_{1} \sigma_{k-1}-S_{2} \sigma_{k-2}+\cdots+(-1)^{k} S_{k-1} \sigma_{1}+(-1)^{k-1} S_{k}, \quad k=1,2, \ldots, n +$$ + +the system easily leads to $\sigma_{1}=a$ and $\sigma_{k}=0$ for $k=2, \ldots, n$. By Vieta's formulas, $x_{1}, x_{2}, \ldots, x_{n}$ are the roots of the polynomial $x^{n}-a x^{n-1}$, i.e., $a, 0,0, \ldots, 0$ in some order. +Remark. This solution does not use the assumption that the $x_{j}$ 's are real. +8. The circles $K_{A}, K_{B}, K_{C}, K_{D}$ cover the parallelogram if and only if for every point $X$ inside the parallelogram, the length of one of the segments $X A, X B, X C, X D$ does not exceed 1. +Let $O$ and $r$ be the center and radius of the circumcircle of $\triangle A B D$. For every point $X$ inside $\triangle A B D$, it holds that $X A \leq r$ or $X B \leq r$ or $X D \leq r$. Similarly, for $X$ inside $\triangle B C D, X B \leq r$ or $X C \leq r$ or $X D \leq r$. Hence $K_{A}, K_{B}, K_{C}, K_{D}$ cover the parallelogram if and only if $r \leq 1$, which is equivalent to $\angle A B D \geq 30^{\circ}$. However, this last is exactly equivalent to $a=A B=2 r \sin \angle A D B \leq 2 \sin \left(\alpha+30^{\circ}\right)=\sqrt{3} \sin \alpha+\cos \alpha$. +9. The incenter of any such triangle lies inside the circle $k$. We shall show that every point $S$ interior to the circle $S$ is the incenter of one such triangle. If $S$ lies on the segment $A B$, then it is obviously the incenter of an isosceles triangle inscribed in $k$ that has $A B$ as an axis of symmetry. Let us now suppose $S$ does not lie on $A B$. Let $X$ and $Y$ be the intersection points of lines $A S$ and $B S$ with $k$, and let $Z$ be the foot of the perpendicular from $S$ to $A B$. Since the quadrilateral $B Z S X$ is cyclic, we have $\angle Z X S=$ $\angle A B S=\angle S X Y$ and analogously $\angle Z Y S=\angle S Y X$, which implies that $S$ is the incenter of $\triangle X Y Z$. +10. Let $n$ be the number of triangles and let $b$ and $i$ be the numbers of vertices on the boundary and in the interior of the square, respectively. +Since all the triangles are acute, each of the vertices of the square belongs to at least two triangles. Additionally, every vertex on the boundary belongs to at least three, and every vertex in the interior belongs to at least five triangles. Therefore + +$$ +3 n \geq 8+3 b+5 i +$$ + +Moreover, the sum of angles at any vertex that lies in the interior, on the boundary, or at a vertex of the square is equal to $2 \pi, \pi, \pi / 2$ respectively. The sum of all angles of the triangles equals $n \pi$, which gives us $n \pi=4 \cdot \pi / 2+b \pi+2 i \pi$, i.e., $n=$ $2+b+2 i$. This relation together with (1) easily yields that $i \geq 2$. Since each of the vertices inside the square belongs to at least five triangles, and at most two contain both, it follows that $n \geq 8$. +![](https://cdn.mathpix.com/cropped/2024_11_18_8e985d6b9c83aa3e9d0eg-357.jpg?height=525&width=489&top_left_y=1547&top_left_x=813) + +It is shown in the figure that the square can be decomposed into eight acute triangles. Obviously one of them can have an arbitrarily small perimeter. +11. We have to find the number $p_{n}$ of triples of positive integers $(a, b, c)$ satisfying $a \leq b \leq c \leq n$ and $a+b>c$. Let us denote by $p_{n}(k)$ the number of such triples with $c=k, k=1,2, \ldots, n$. For $k$ even, $p_{n}(k)=k+(k-2)+(k-4)+\cdots+2=\left(k^{2}+2 k\right) / 4$, and for $k$ odd, $p_{n}(k)=\left(k^{2}+2 k+1\right) / 4$. Hence $p_{n}=p_{n}(1)+p_{n}(2)+\cdots+p_{n}(n)= \begin{cases}n(n+2)(2 n+5) / 24, & \text { for } 2 \mid n, \\ (n+1)(n+3)(2 n+1) / 24, & \text { for } 2 \nmid n .\end{cases}$ +12. Let us denote by $M_{n}$ the set of points of the segment $A B$ obtained from $A$ and $B$ by not more than $n$ iterations of $(*)$. It can be proved by induction that + +$$ +M_{n}=\left\{X \in A B \left\lvert\, A X=\frac{3 k}{4^{n}}\right. \text { or } \frac{3 k-2}{4^{n}} \text { for some } k \in \mathbb{N}\right\} +$$ + +Thus (a) immediately follows from $M=\bigcup M_{n}$. It also follows that if $a, b \in \mathbb{N}$ and $a / b \in M$, then $3 \mid a(b-a)$. Therefore $1 / 2 \notin M$. +13. The maximum area is $3 \sqrt{3} r^{2} / 4$ (where $r$ is the radius of the semicircle) and is attained in the case of a trapezoid with two vertices at the endpoints of the diameter of the semicircle and the other two vertices dividing the semicircle into three equal arcs. +14. We have that + +$$ +\left|\frac{p}{q}-\sqrt{2}\right|=\frac{|p-q \sqrt{2}|}{q}=\frac{\left|p^{2}-2 q^{2}\right|}{q(p+q \sqrt{2})} \geq \frac{1}{q(p+q \sqrt{2})} +$$ + +because $\left|p^{2}-2 q^{2}\right| \geq 1$. +The greatest solution to the equation $\left|p^{2}-2 q^{2}\right|=1$ with $p, q \leq 100$ is $(p, q)=(99,70)$. It is easy to verify using (1) that $\frac{99}{70}$ best approximates $\sqrt{2}$ among the fractions $p / q$ with $p, q \leq 100$. +Second solution. By using some basic facts about Farey sequences one can find that $\frac{41}{29}<\sqrt{2}<\frac{99}{70}$ and that $\frac{41}{29}<\frac{p}{q}<\frac{99}{70}$ implies $p \geq 41+99>100$ because $99 \cdot 29-41 \cdot 70=1$. Of the two fractions $41 / 29$ and $99 / 70$, the latter is closer to $\sqrt{2}$. +15. Given that $\tan \alpha \in \mathbb{Q}$, we have that $\tan \beta$ is rational if and only if $\tan \gamma$ is rational, where $\gamma=\beta-\alpha$ and $2 \gamma=\alpha$. Putting $t=\tan \gamma$ we obtain $\frac{p}{q}=\tan 2 \gamma=\frac{2 t}{1-t^{2}}$, which leads to the quadratic equation $p t^{2}+2 q t-p=0$. This equation has rational solutions if and only if its discriminant $4\left(p^{2}+q^{2}\right)$ is a perfect square, and the result follows. +16. First let us notice that all the numbers $z_{m_{1}, m_{2}}=m_{1} r_{1}+m_{2} r_{2}$ ( $m_{1}, m_{2} \in$ $\mathbb{Z})$ are distinct, since $r_{1} / r_{2}$ is irrational. Thus for any $n \in \mathbb{N}$ the interval $\left[-n\left(\left|r_{1}\right|+\left|r_{2}\right|\right), n\left(\left|r_{1}\right|+\left|r_{2}\right|\right)\right]$ contains $(2 n+1)^{2}$ numbers $z_{m_{1}, m_{2}}$, +where $\left|m_{1}\right|,\left|m_{2}\right| \leq n$. Therefore some two of these $(2 n+1)^{2}$ numbers, say $z_{m_{1}, m_{2}}, z_{n_{1}, n_{2}}$, differ by at most $\frac{2 n\left(\left|r_{1}\right|+\left|r_{2}\right|\right)}{(2 n+1)^{2}-1}=\frac{\left(\left|r_{1}\right|+\left|r_{2}\right|\right)}{2(n+1)}$. By taking $n$ large enough we can achieve that + +$$ +z_{q_{1}, q_{2}}=\left|z_{m_{1}, m_{2}}-z_{n_{1}, n_{2}}\right| \leq p +$$ + +If now $k$ is the integer such that $k z_{q_{1}, q_{2}} \leq x<(k+1) z_{q_{1}, q_{2}}$, then $z_{k q_{1}, k q_{2}}=$ $k z_{q_{1}, q_{2}}$ differs from $x$ by at most $p$, as desired. +17. Using $c_{r}-c_{s}=(r-s)(r+s+1)$ we can easily get + +$$ +\frac{\left(c_{m+1}-c_{k}\right) \cdots\left(c_{m+n}-c_{k}\right)}{c_{1} c_{2} \cdots c_{n}}=\frac{(m-k+n)!}{(m-k)!n!} \cdot \frac{(m+k+n+1)!}{(m+k+1)!(n+1)!} +$$ + +The first factor $\frac{(m-k+n)!}{(m-k)!n!}=\binom{m-k+n}{n}$ is clearly an integer. The second factor is also an integer because by the assumption, $m+k+1$ and ( $m+$ $k)!(n+1)$ ! are coprime, and $(m+k+n+1)$ ! is divisible by both; hence it is also divisible by their product. +18. In the first part, it is sufficient to show that each rational number of the form $m / n!, m, n \in \mathbb{N}$, can be written uniquely in the required form. We prove this by induction on $n$. +The statement is trivial for $n=1$. Let us assume it holds for $n-1$, and let there be given a rational number $m / n$ !. Let us take $a_{n} \in\{0, \ldots, n-1\}$ such that $m-a_{n}=n m_{1}$ for some $m_{1} \in \mathbb{N}$. By the inductive hypothesis, there are unique $a_{1} \in \mathbb{N}_{0}, a_{i} \in\{0, \ldots, i-1\}(i=1, \ldots, n-1)$ such that $m_{1} /(n-1)!=\sum_{i=1}^{n-1} a_{i} / i$ !, and then + +$$ +\frac{m}{n!}=\frac{m_{1}}{(n-1)!}+\frac{a_{n}}{n!}=\sum_{i=1}^{n} \frac{a_{i}}{i!} +$$ + +as desired. On the other hand, if $m / n!=\sum_{i=1}^{n} a_{i} / i$ !, multiplying by $n$ ! we see that $m-a_{n}$ must be a multiple of $n$, so the choice of $a_{n}$ was unique and therefore the representation itself. This completes the induction. +In particular, since $a_{i} \mid i!$ and $i!/ a_{i}>(i-1)!\geq(i-1)!/ a_{i-1}$, we conclude that each rational $q, 00$ be a rational number. For any integer $m>10^{6}$, let $n>m$ be the greatest integer such that $y=$ $x-\frac{1}{m}-\frac{1}{m+1}-\cdots-\frac{1}{n}>0$. Then $y$ can be written as the sum of reciprocals of different positive integers, which all must be greater than $n$. The result follows immediately. +19. Suppose $n \leq 6$. Let us decompose the disk by its radii into $n$ congruent regions, so that one of the points $P_{j}$ lies on the boundaries of two of these regions. Then one of these regions contains two of the $n$ given points. Since the diameter of each of these regions is $2 \sin \frac{\pi}{n}$, we have $d_{n} \leq 2 \sin \frac{\pi}{n}$. This +value is attained if $P_{i}$ are the vertices of a regular $n$-gon inscribed in the boundary circle. Hence $D_{n}=2 \sin \frac{\pi}{n}$. +For $n=7$ we have $D_{7} \leq D_{6}=1$. This value is attained if six of the seven points form a regular hexagon inscribed in the boundary circle and the seventh is at the center. Hence $D_{7}=1$. +20. The statement so formulated is false. It would be true under the additional assumption that the polygonal line is closed. However, from the offered solution, which is not clear, it does not seem that the proposer had this in mind. +21. Using the formula + +$$ +\cos x \cos 2 x \cos 4 x \cdots \cos 2^{n-1} x=\frac{\sin 2^{n} x}{2^{n} \sin x} +$$ + +which is shown by simple induction, we obtain + +$$ +\begin{gathered} +\cos \frac{\pi}{15} \cos \frac{2 \pi}{15} \cos \frac{4 \pi}{15} \cos \frac{7 \pi}{15}=-\cos \frac{\pi}{15} \cos \frac{2 \pi}{15} \cos \frac{4 \pi}{15} \cos \frac{8 \pi}{15}=\frac{1}{16} \\ +\cos \frac{3 \pi}{15} \cos \frac{6 \pi}{15} +\end{gathered}=\frac{1}{4}, \quad \cos \frac{5 \pi}{15}=\frac{1}{2} . +$$ + +Multiplying these equalities, we get that the required product $P$ equals 1/128. +22. Let $O_{1}$ and $O_{2}$ be the centers of circles $k_{1}$ and $k_{2}$ and let $C$ be the midpoint of the segment $A B$. Using the well-known relation for elements of a triangle, we obtain + +$$ +P A^{2}+P B^{2}=2 P C^{2}+2 C A^{2} \geq 2 O_{1} C^{2}+2 C A^{2}=2 O_{1} A^{2}=2 r^{2} +$$ + +Equality holds if $P$ coincides with $O_{1}$ or if $A$ and $B$ coincide with $O_{2}$. +23. Suppose that $a \geq 0, c \geq 0,4 a c \geq b^{2}$. If $a=0$, then $b=0$, and the inequality reduces to the obvious $c g^{2} \geq 0$. Also, if $a>0$, then + +$$ +a f^{2}+b f g+c g^{2}=a\left(f+\frac{b}{2 a} g\right)^{2}+\frac{4 a c-b^{2}}{4 a} g^{2} \geq 0 +$$ + +Suppose now that $a f^{2}+b f g+c g^{2} \geq 0$ holds for an arbitrary pair of vectors $f, g$. Substituting $f$ by $t g(t \in \mathbb{R})$ we get that $\left(a t^{2}+b t+c\right) g^{2} \geq 0$ holds for any real number $t$. Therefore $a \geq 0, c \geq 0,4 a c \geq b^{2}$. +24. Let the $k$ th child receive $x_{k}$ coins. By the condition of the problem, the number of coins that remain after him was $6\left(x_{k}-k\right)$. This gives us a recurrence relation + +$$ +x_{k+1}=k+1+\frac{6\left(x_{k}-k\right)-k-1}{7}=\frac{6}{7} x_{k}+\frac{6}{7}, +$$ + +which, together with the condition $x_{1}=1+(m-1) / 7$, yields + +$$ +x_{k}=\frac{6^{k-1}}{7^{k}}(m-36)+6 \text { for } 1 \leq k \leq n . +$$ + +Since we are given $x_{n}=n$, we obtain $6^{n-1}(m-36)=7^{n}(n-6)$. It follows that $6^{n-1} \mid n-6$, which is possible only for $n=6$. Hence, $n=6$ and $m=36$. +25. The answer is $R=(4+\sqrt{3}) d / 6$. +26. Let $L$ be the midpoint of the edge $A B$. Since $P$ is the orthocenter of $\triangle A B M$ and $M L$ is its altitude, $P$ lies on $M L$ and therefore belongs to the triangular area $L C D$. Moreover, from the similarity of triangles $A L P$ and $M L B$ we have $L P \cdot L M=L A \cdot L B=a^{2} / 4$, where $a$ is the side length of tetrahedron $A B C D$. It easily follows that the locus of $P$ is the image of the segment $C D$ under the inversion of the plane $L C D$ with center $L$ and radius $a / 2$. This locus is the arc of a circle with center $L$ and endpoints at the orthocenters of triangles $A B C$ and $A B D$. +27. Regular polygons with 3,4 , and 6 sides can be obtained by cutting a cube with a plane, as shown in the figure. A polygon with more than 6 sides cannot be obtained in such a way, for a cube has 6 faces. Also, if a pentagon is obtained by cutting a +![](https://cdn.mathpix.com/cropped/2024_11_18_8e985d6b9c83aa3e9d0eg-361.jpg?height=299&width=312&top_left_y=991&top_left_x=935) +cube with a plane, then its sides lying on opposite faces are parallel; hence it cannot be regular. +28. The given expression can be transformed into + +$$ +y=\frac{4 \cos 2 u+2}{\cos 2 u-\cos 2 x}-3 . +$$ + +It does not depend on $x$ if and only if $\cos 2 u=-1 / 2$, i.e., $u= \pm \pi / 3+k \pi$ for some $k \in \mathbb{Z}$. +29. Let arc $l_{a}$ be the locus of points $A$ lying on the opposite side from $A_{0}$ with respect to the line $B_{0} C_{0}$ such that $\angle B_{0} A C_{0}=\angle A^{\prime}$. Let $k_{a}$ be the circle containing $l_{a}$, and let $S_{a}$ be the center of $k_{a}$. We similarly define $l_{b}, l_{c}, k_{b}, k_{c}, S_{b}, S_{c}$. It is easy to show that circles $k_{a}, k_{b}, k_{c}$ have a common point $S$ inside $\triangle A B C$. Let $A_{1}, B_{1}, C_{1}$ be the points on the arcs $l_{a}, l_{b}, l_{c}$ diametrically opposite to $S$ with respect to $S_{a}, S_{b}, S_{c}$ respectively. Then $A_{0} \in B_{1} C_{1}$ because $\angle B_{1} A_{0} S=\angle C_{1} A_{0} S=90^{\circ}$; similarly, $B_{0} \in A_{1} C_{1}$ and $C_{0} \in A_{1} B_{1}$. Hence the triangle $A_{1} B_{1} C_{1}$ is circumscribed about $\triangle A_{0} B_{0} C_{0}$ and similar to $\triangle A^{\prime} B^{\prime} C^{\prime}$. +Moreover, we claim that $\triangle A_{1} B_{1} C_{1}$ is the triangle $A B C$ with the desired properties having the maximum side $B C$ and hence the maximum area. + +Indeed, if $A B C$ is any other such triangle and $S_{b}^{\prime}, S_{c}^{\prime}$ are the projections of $S_{b}$ and $S_{c}$ onto the line $B C$, it holds that $B C=2 S_{b}^{\prime} S_{c}^{\prime} \leq 2 S_{b} S_{c}=B_{1} C_{1}$, which proves the maximality of $B_{1} C_{1}$. +30. We assume w.l.o.g. that $m \leq n$. Let $r$ and $s$ be the numbers of pairs for which $i-j \geq k$ and of those for which $j-i \geq k$. The desired number is $r+s$. We easily find that + +$$ +\begin{aligned} +& r= \begin{cases}(m-k)(m-k+1) / 2, & k\sqrt{3} / 2$. +Suppose now that $2 \pi / 3 \leq x<2 \pi$. We have + +$$ +\sin x+\cdots+\sin n x=\frac{\cos \frac{x}{2}-\cos \frac{2 n+1}{2} x}{2 \sin \frac{x}{2}} \leq \frac{\cos \frac{x}{2}+1}{2 \sin \frac{x}{2}}=\frac{\cot \frac{x}{4}}{2} \leq \frac{\sqrt{3}}{2} . +$$ + +For $x=2 \pi$ the given inequality clearly holds for all $n$. Hence, the inequality holds for all $n$ if and only if $2 \pi / 3+2 k \pi \leq x \leq 2 \pi+2 k \pi$ for some integer $k$. +56. We shall prove by induction on $n$ the following statement: If in some group of interpreters exactly $n$ persons, $n \geq 2$, speak each of the three languages, then it is possible to select a subgroup in which each language is spoken by exactly two persons. +The statement of the problem easily follows from this: it suffices to select six such groups. + +The case $n=2$ is trivial. Let us assume $n \geq 2$, and let $N_{j}, N_{m}, N_{f}, N_{j m}$, $N_{j f}, N_{m f}, N_{j m f}$ be the sets of those interpreters who speak only Japanese, only Malay, only Farsi, only Japanese and Malay, only Japanese and Farsi, only Malay and Farsi, and all the three languages, respectively, and $n_{j}, n_{m}$, $n_{f}, n_{j m}, n_{j f}, n_{m f}, n_{j m f}$ the cardinalities of these sets, respectively. By the condition of the problem, $n_{j}+n_{j m}+n_{j f}+n_{j m f}=n_{m}+n_{j m}+n_{m f}+n_{j m f}=$ $n_{f}+n_{j f}+n_{m f}+n_{j m f}=24$, and consequently + +$$ +n_{j}-n_{m f}=n_{m}-n_{j f}=n_{f}-n_{j m}=c +$$ + +Now if $c<0$, then $n_{j m}, n_{j f}, n_{m f}>0$, and it is enough to select one interpreter from each of the sets $N_{j m}, N_{j f}, N_{m f}$. If $c>0$, then $n_{j}, n_{m}, n_{f}>0$, and it is enough to select one interpreter from each of the sets $N_{j}, N_{m}, N_{f}$ and then use the inductive assumption. Also, if $c=0$, then w.l.o.g. $n_{j}=n_{m f}>0$, and it is enough to select one interpreter from each of the sets $N_{j}, N_{m f}$ and then use the inductive hypothesis. This completes the induction. +57. Obviously $c_{n}>0$ for all even $n$. Thus $c_{n}=0$ is possible only for an odd $n$. Let us assume $a_{1} \leq a_{2} \leq \cdots \leq a_{8}$ : in particular, $a_{1} \leq 0 \leq a_{8}$. +If $\left|a_{1}\right|<\left|a_{8}\right|$, then there exists $n_{0}$ such that for every odd $n>n_{0}, 7\left|a_{1}\right|^{n}<$ $a_{8}^{n} \Rightarrow a_{1}^{n}+\cdots+a_{7}^{n}+a_{8}^{n}>7 a_{1}^{n}+a_{8}^{n}>0$, contradicting the condition that $c_{n}=0$ for infinitely many $n$. Similarly $\left|a_{1}\right|>\left|a_{8}\right|$ is impossible, and we conclude that $a_{1}=-a_{8}$. +Continuing in the same manner we can show that $a_{2}=-a_{7}, a_{3}=-a_{6}$ and $a_{4}=-a_{5}$. Hence $c_{n}=0$ for every odd $n$. +58. The following sequence of equalities and inequalities gives an even stronger estimate than needed. + +$$ +\begin{aligned} +|l(z)| & =|A z+B|=\frac{1}{2}|(z+1)(A+B)+(z-1)(A-B)| \\ +& =\frac{1}{2}|(z+1) f(1)+(z-1) f(-1)| \\ +& \leq \frac{1}{2}(|z+1| \cdot|f(1)|+|z-1| \cdot|f(-1)|) \\ +& \leq \frac{1}{2}(|z+1|+|z-1|) M=\frac{1}{2} \rho M . +\end{aligned} +$$ + +59. By the $\operatorname{arc} A B$ we shall always mean the positive $\operatorname{arc} A B$. We denote by $|A B|$ the length of arc $A B$. Let a basic arc be one of the $n+1$ arcs into which the circle is partitioned by the points $A_{0}, A_{1}, \ldots, A_{n}$, where $n \in \mathbb{N}$. Suppose that $A_{p} A_{0}$ and $A_{0} A_{q}$ are the basic arcs with an endpoint at $A_{0}$, and that $x_{n}, y_{n}$ are their lengths, respectively. We show by induction on $n$ that for each $n$ the length of a basic arc is equal to $x_{n}, y_{n}$ or $x_{n}+y_{n}$. The statement is trivial for $n=1$. Assume that it holds for $n$, and let $A_{i} A_{n+1}, A_{n+1} A_{j}$ be basic arcs. We shall prove that these two arcs have lengths $x_{n}, y_{n}$, or $x_{n}+y_{n}$. If $i, j$ are both strictly positive, then $\left|A_{i} A_{n+1}\right|=$ +$\left|A_{i-1} A_{n}\right|$ and $\left|A_{n+1} A_{j}\right|=\left|A_{n} A_{j-1}\right|$ are equal to $x_{n}, y_{n}$, or $x_{n}+y_{n}$ by the inductive hypothesis. +Let us assume now that $i=0$, i.e., that $A_{p} A_{n+1}$ and $A_{n+1} A_{0}$ are basic arcs. Then $\left|A_{p} A_{n+1}\right|=\left|A_{0} A_{n+1-p}\right| \geq\left|A_{0} A_{q}\right|=y_{n}$ and similarly $\left|A_{n+1} A_{q}\right| \geq x_{n}$, but $\left|A_{p} A_{q}\right|=x_{n}+y_{n}$, from which it follows that $\left|A_{p} A_{n+1}\right|=\left|A_{0} A_{q}\right|=y_{n}$ and consequently $n+1=p+q$. Also, $x_{n+1}=\left|A_{n+1} A_{0}\right|=y_{n}-x_{n}$ and $y_{n+1}=y_{n}$. Now, all basic arcs have lengths $y_{n}-x_{n}, x_{n}, y_{n}, x_{n}+y_{n}$. A presence of a basic arc of length $x_{n}+y_{n}$ would spoil our inductive step. However, if any basic arc $A_{k} A_{l}$ has length $x_{n}+y_{n}$, then we must have $l-q=k-p$ because $2 \pi$ is irrational, and therefore the arc $A_{k} A_{l}$ contains either the point $A_{k-p}$ (if $k \geq p$ ) or the point $A_{k+q}$ (if $k1 \\ +\left(p_{k}, p_{k}+q_{k}\right), \text { if }\left\{p_{k} /(2 \pi)\right\}+\left\{q_{k} /(2 \pi)\right\}<1 +\end{array}\right. +$$ + +It is now "easy" to calculate that $p_{19}=p_{20}=333, q_{19}=377, q_{20}=710$, and thus $n_{19}=709<1000<1042=n_{20}$. It follows that the lengths of the basic arcs for $n=1000$ take exactly three different values. + +### 4.10 Solutions to the Shortlisted Problems of IMO 1968 + +1. Since the ships are sailing with constant speeds and directions, the second ship is sailing at a constant speed and direction in reference to the first ship. Let $A$ be the constant position of the first ship in this frame. Let $B_{1}$, $B_{2}, B_{3}$, and $B$ on line $b$ defining the trajectory of the ship be positions of the second ship with respect to the first ship at 9:00, 9:35, 9:55, and at the moment the two ships were closest. Then we have the following equations for distances (in miles): + +$$ +\begin{gathered} +A B_{1}=20, \quad A B_{2}=15, \quad A B_{3}=13 \\ +B_{1} B_{2}: B_{2} B_{3}=7: 4, \quad A B_{i}^{2}=A B^{2}+B B_{i}^{2} +\end{gathered} +$$ + +Since $B B_{1}>B B_{2}>B B_{3}$, it follows that $\mathcal{B}\left(B_{3}, B, B_{2}, B_{1}\right)$ or $\mathcal{B}\left(B, B_{3}, B_{2}\right.$, $B_{1}$ ). We get a system of three quadratic equations with three unknowns: $A B, B B_{3}$ and $B_{3} B_{2}$ ( $B B_{3}$ being negative if $\mathcal{B}\left(B_{3}, B, B_{1}, B_{2}\right)$, positive otherwise). This can be solved by eliminating $A B$ and then $B B_{3}$. The unique solution ends up being + +$$ +A B=12, \quad B B_{3}=5, \quad B_{3} B_{2}=4 +$$ + +and consequently, the two ships are closest at 10:20 when they are at a distance of 12 miles. +2. The sides $a, b, c$ of a triangle $A B C$ with $\angle A B C=2 \angle B A C$ satisfy $b^{2}=$ $a(a+c)$ (this statement is the lemma in (SL98-7)). Taking into account the remaining condition that $a, b, c$ are consecutive integers with $ax>\sin x$ for all $01 \\ +\Leftrightarrow & 1+2 n\left(1-\frac{1}{1+\pi^{2} /\left(4 n^{2}\right)}\right)-\frac{\pi^{2}}{2(n+1)}>1 \\ +\Leftrightarrow & 1+\frac{\pi^{2}}{2}\left(\frac{1}{n+\pi^{2} /(4 n)}-\frac{1}{n+1}\right)>1, +\end{aligned} +$$ + +where the last inequality holds because $\pi^{2}<4 n$. It is also apparent that as $n$ tends to infinity the term in parentheses tends to 0 , and hence it is not possible to strengthen the bound. This completes the proof. +6. We define $f(x)=\frac{a_{1}}{a_{1}-x}+\frac{a_{2}}{a_{2}-x}+\cdots+\frac{a_{n}}{a_{n}-x}$. Let us assume w.l.o.g. $a_{1}1$. Then $a=k b$ and $c=\sqrt{k} b$, and $a>c>b$. The segments $a, b, c$ form a triangle if and only if $k<\sqrt{k}+1$, which holds if and only if $1120^{\circ}-60^{\circ}=60^{\circ}$ (because $\angle A_{1} A_{j} A_{i}<60^{\circ}$ ); hence $\angle A_{i} A_{j} A_{k} \geq 120^{\circ}$. This proves that the denotation is correct. +Remark. It is easy to show that the diameter is unique. Hence the denotation is also unique. +21. The given conditions are equivalent to $y-a_{0}$ being divisible by $a_{0}, a_{0}+$ $a_{1}, a_{0}+a_{2}, \ldots, a_{0}+a_{n}$, i.e., to $y=k\left[a_{0}, a_{0}+a_{1}, \ldots, a_{0}+a_{n}\right]+a_{0}, k \in \mathbb{N}_{0}$. +22. It can be shown by induction on the number of digits of $x$ that $p(x) \leq x$ for all $x \in \mathbb{N}$. It follows that $x^{2}-10 x-22 \leq x$, which implies $x \leq 12$. + +Since $0k-j$ and $9^{k-j-1} 9$ !/ $(9-j)$ ! otherwise. If the $i$ th digit is not 0 , then the above results are multiplied by 8 . +25. The answer is + +$$ +\sum_{1 \leq p1$. We then have $z=n^{4}+4 m^{4}=$ $\left(n^{2}+2 m^{2}\right)^{2}-(2 m n)^{2}=\left(n^{2}+2 m^{2}+2 m n\right)\left(n^{2}+2 m^{2}-2 m n\right)$. Since $n^{2}+2 m^{2}-2 m n=(n-m)^{2}+m^{2} \geq m^{2}>1$, it follows that $z$ must be composite. Thus we have found infinitely many $a$ that satisfy the condition of the problem. +2. Using $\cos (a+x)=\cos a \cos x-\sin a \sin x$, we obtain $f(x)=A \sin x+$ $B \cos x$ where $A=-\sin a_{1}-\sin a_{2} / 2-\cdots-\sin a_{n} / 2^{n-1}$ and $B=\cos a_{1}+$ $\cos a_{2} / 2+\cdots+\cos a_{n} / 2^{n-1}$. Numbers $A$ and $B$ cannot both be equal to 0 , for otherwise $f$ would be identically equal to 0 , while on the other hand, we have $f\left(-a_{1}\right)=\cos \left(a_{1}-a_{1}\right)+\cos \left(a_{2}-a_{1}\right) / 2+\cdots+\cos \left(a_{n}-a_{1}\right) / 2^{n-1} \geq$ $1-1 / 2-\cdots-1 / 2^{n-1}=1 / 2^{n-1}>0$. Setting $A=C \cos \phi$ and $B=C \sin \phi$, where $C \neq 0$ (such $C$ and $\phi$ always exist), we get $f(x)=C \sin (x+\phi)$. It follows that the zeros of $f$ are of the form $x_{0} \in-\phi+\pi \mathbb{Z}$, from which $f\left(x_{1}\right)=f\left(x_{2}\right) \Rightarrow x_{1}-x_{2}=m \pi$ immediately follows. +3. We have several cases: +$1^{\circ} k=1$. W.l.o.g. let $A B=a$ and the remaining segments have length 1. Let $M$ be the midpoint of $C D$. Then $A M=B M=\sqrt{3} / 2(\triangle C D A$ and $\triangle C D B$ are equilateral) and $01$. Assume $A B=A C=A D=a$. Varying $A$ along the line perpendicular to the plane $B C D$ and through the center of $\triangle B C D$ we achieve all values of $a>1 / \sqrt{3}$. For $a<1 / \sqrt{3}$ we can observe a similar tetrahedron with three edges of length $1 / a$ and three of length 1 and proceed as before. +$4^{\circ} k=4$. By observing the similar tetrahedron we reduce this case to $k=2$ with length $1 / a$ instead of $a$. Thus we get $a>\sqrt{2-\sqrt{3}}$. +$5^{\circ} k=5$. We reduce to $k=1$ and get $a>1 / \sqrt{3}$. +4. Let $O$ be the midpoint of $A B$, i.e., the center of $\gamma$. Let $O_{1}, O_{2}$, and $O_{3}$ respectively be the centers of $\gamma_{1}, \gamma_{2}$, and $\gamma_{3}$ and let $r_{1}, r_{2}, r_{3}$ respectively +be the radii of $\gamma_{1}, \gamma_{2}$ and $\gamma_{3}$. Let $C_{1}, C_{2}$, and $C_{3}$ respectively be the points of tangency of $\gamma_{1}, \gamma_{2}$ and $\gamma_{3}$ with $A B$. Let $D_{2}$ and $D_{3}$ respectively be the points of tangency of $\gamma_{2}$ and $\gamma_{3}$ with $C D$. Finally, let $G_{2}$ and $G_{3}$ respectively be the points of tangency of $\gamma_{2}$ and $\gamma_{3}$ with $\gamma$. We have $\mathcal{B}\left(G_{2}, O_{2}, O\right)$, $G_{2} O_{2}=O_{2} D_{2}$, and $G_{2} O=O B$. Hence, $G_{2}, D_{2}, B$ are collinear. Similarly, $G_{3}, D_{3}, A$ are collinear. It follows that $A G_{2} D_{2} D$ and $B G_{3} D_{3} D$ are cyclic, since $\angle A G_{2} D_{2}=\angle D_{2} D A=\angle D_{3} D B=\angle B G_{3} D_{3}=90^{\circ}$. Hence $B C_{2}^{2}=B D_{2} \cdot B G_{2}=B D \cdot B A=B C^{2} \Rightarrow B C_{2}=B C$ and hence $A C_{2}=A B-B C$. Similarly, $A C_{3}=A C$. We thus have $A C_{1}=(A C+A B-B C) / 2=\left(A C_{3}+A C_{2}\right) / 2$. Hence, $C_{1}$ is the midpoint of $C_{2} C_{3}$. We also have $r_{2}+r_{3}=C_{2} C_{3}=A C+B C-A B=2 r_{1}$, from which it follows that $O_{1}, O_{2}, O_{3}$ are collinear. +Second solution. We shall prove the statement for arbitrary points $A, B, C$ on $\gamma$. +Let us apply the inversion $\psi$ with respect to the circle $\gamma_{1}$. We denote by $\widehat{X}$ the image of an object $X$ under $\psi$. Also, $\psi$ maps lines $B C, C A, A B$ onto circles $\widehat{a}, \widehat{b}, \widehat{c}$, respectively. Circles $\widehat{a}, \widehat{b}, \widehat{c}$ pass through the center $O_{1}$ of $\gamma_{1}$ and have radii equal to the radius of $\widehat{\gamma}$. Let $P, Q, R$ be the centers of $\widehat{a}, \widehat{b}, \widehat{c}$ respectively. +The line $C D$ maps onto a circle $k$ through $\widehat{C}$ and $O_{1}$ that is perpendicular to $\widehat{c}$. Therefore its center $K$ lies in the intersection of the tangent $t$ to $\widehat{c}$ and the line $P Q$ (which bisects $\left.\widehat{C} O_{1}\right)$. Let $O$ be a point such that $R O_{1} K O$ is a parallelogram and $\gamma_{2}^{\prime}, \gamma_{3}^{\prime}$ the circles centered at $O$ tangent to $k$. It is easy to see that $\gamma_{2}^{\prime}$ and $\gamma_{3}^{\prime}$ are also tangent to $\widehat{c}$, since $O R$ and $O K$ have lengths equal to the radii of $k$ and $\widehat{c}$. Hence $\gamma_{2}^{\prime}$ and $\gamma_{3}^{\prime}$ are the images of $\gamma_{2}$ and $\gamma_{3}$ under $\psi$. Moreover, since $Q \widehat{A} O K$ and $P \widehat{B} O K$ are parallelograms and $Q, P, K$ are collinear, it follows that $\widehat{A}, \widehat{B}, O$ are also collinear. Hence the centers of $\gamma_{1}, \gamma_{2}, \gamma_{3}$ are collinear, lying on the line $O_{1} O$, and the statement follows. + +Third solution. Moreover, the statement holds for an arbitrary point $D \in B C$. Let $E, F, G, H$ be the points of tangency of $\gamma_{2}$ with $A B, C D$ and of $\gamma_{3}$ with $A B, C D$, respectively. Let $O_{i}$ be the center of $\gamma_{i}, i=1,2,3$. As is shown in the third solution of (SL93-3), $E F$ and $G H$ meet at $O_{1}$. Hence the problem of proving the collinearity of $O_{1}, O_{2}, O_{3}$ reduces to the following simple problem: + +Let $D, E, F, G, H$ be points such that $D \in E G, F \in D H$ and $D E=D F, D G=D H$. Let $O_{1}, O_{2}, O_{3}$ be points such that $\angle O_{2} E D=$ $\angle O_{2} F D=90^{\circ}, \angle O_{3} G D=\angle O_{3} H D=90^{\circ}$, and $O_{1}=E F \cap G H$. Then $O_{1}, O_{2}, O_{3}$ are collinear. +Let $K_{2}=D O_{2} \cap E F$ and $K_{3}=D O_{3} \cap G H$. Then $O_{2} K_{2} / O_{2} D=$ $D K_{3} / D O_{3}=K_{2} O_{1} / D O_{3}$ and hence by Thales' theorem $O_{1} \in O_{2} O_{3}$. +5. We first prove the following lemma. + +Lemma. If of five points in a plane no three belong to a single line, then there exist four that are the vertices of a convex quadrilateral. + +Proof. If the convex hull of the five points $A, B, C, D, E$ is a pentagon or a quadrilateral, the statement automatically holds. If the convex hull is a triangle, then w.l.o.g. let $\triangle A B C$ be that triangle and $D, E$ points in its interior. Let the line $D E$ w.l.o.g. intersect $[A B]$ and $[A C]$. Then $B, C, D, E$ form the desired quadrilateral. +We now observe each quintuplet of points within the set. There are $\binom{n}{5}$ such quintuplets, and for each of them there is at least one quadruplet of points forming a convex quadrilateral. Each quadruplet, however, will be counted up to $n-4$ times. Hence we have found at least $\frac{1}{n-4}\binom{n}{5}$ quadruplets. Since $\frac{1}{n-4}\binom{n}{5} \geq\binom{ n-3}{2} \Leftrightarrow(n-5)(n-6)(n+8) \geq 0$, which always holds, it follows that we have found at least $\binom{n-3}{2}$ desired quadruplets of points. +6. Define $u_{1}=\sqrt{x_{1} y_{1}}+z_{1}, u_{2}=\sqrt{x_{2} y_{2}}+z_{2}, v_{1}=\sqrt{x_{1} y_{1}}-z_{1}$, and $v_{2}=$ $\sqrt{x_{2} y_{2}}-z_{2}$. By expanding both sides of the equation we can easily verify $\left(x_{1}+x_{2}\right)\left(y_{1}+y_{2}\right)-\left(z_{1}+z_{2}\right)^{2}=\left(u_{1}+u_{2}\right)\left(v_{1}+v_{2}\right)+\left(\sqrt{x_{1} y_{2}}-\sqrt{x_{2} y_{1}}\right)^{2} \geq$ $\left(u_{1}+u_{2}\right)\left(v_{1}+v_{2}\right)$. Since $x_{i} y_{i}-z_{i}^{2}=u_{i} v_{i}$ for $i=1,2$, it suffices to prove + +$$ +\begin{aligned} +& \frac{8}{\left(u_{1}+u_{2}\right)\left(v_{1}+v_{2}\right)} \leq \frac{1}{u_{1} v_{1}}+\frac{1}{u_{2} v_{2}} \\ +\Leftrightarrow & 8 u_{1} u_{2} v_{1} v_{2} \leq\left(u_{1}+u_{2}\right)\left(v_{1}+v_{2}\right)\left(u_{1} v_{1}+u_{2} v_{2}\right) +\end{aligned} +$$ + +which trivially follows from the AM-GM inequalities $2 \sqrt{u_{1} u_{2}} \leq u_{1}+u_{2}$, $2 \sqrt{v_{1} v_{2}} \leq v_{1}+v_{2}$ and $2 \sqrt{u_{1} v_{1} u_{2} v_{2}} \leq u_{1} v_{1}+u_{2} v_{2}$. +Equality holds if and only if $x_{1} y_{2}=x_{2} y_{1}, u_{1}=u_{2}$ and $v_{1}=v_{2}$, i.e. if and only if $x_{1}=x_{2}, y_{1}=y_{2}$ and $z_{1}=z_{2}$. +Second solution. Let us define $f(x, y, z)=1 /\left(x y-z^{2}\right)$. The problem actually states that + +$$ +2 f\left(\frac{x_{1}+x_{2}}{2}, \frac{y_{1}+y_{2}}{2}, \frac{z_{1}+z_{2}}{2}\right) \leq f\left(x_{1}, y_{1}, z_{1}\right)+f\left(x_{2}, y_{2}, z_{2}\right) +$$ + +i.e., that the function $f$ is convex on the set $D=\left\{(x, y, z) \in \mathbb{R}^{2} \mid x y-\right.$ $\left.z^{2}>0\right\}$. It is known that a twice continuously differentiable function $f\left(t_{1}, t_{2}, \ldots, t_{n}\right)$ is convex if and only if its Hessian $\left[f_{i j}^{\prime \prime}\right]_{i, j=1}^{n}$ is positive semidefinite, or equivalently, if its principal minors $D_{k}=\operatorname{det}\left[f_{i j}^{\prime \prime}\right]_{i, j=1}^{k}, k=$ $1,2, \ldots, n$, are nonnegative. In the case of our $f$ this is directly verified: $D_{1}=2 y^{2} /\left(x y-z^{2}\right)^{3}, D_{2}=3 x y+z^{2} /\left(x y-z^{2}\right)^{5}, D_{3}=6 /\left(x y-z^{2}\right)^{6}$ are obviously positive. + +### 4.12 Solutions to the Shortlisted Problems of IMO 1970 + +1. Denote respectively by $R$ and $r$ the radii of the circumcircle and incircle, by $A_{1}, \ldots, A_{n}, B_{1}, \ldots, B_{n}$, the vertices of the $2 n$-gon and by $O$ its center. Let $P^{\prime}$ be the point symmetric to $P$ with respect to $O$. Then $A_{i} P^{\prime} B_{i} P$ is a parallelogram, and applying cosine theorem on triangles $A_{i} B_{i} P$ and $P P^{\prime} B_{i}$ yields + +$$ +\begin{aligned} +4 R^{2} & =P A_{i}^{2}+P B_{i}^{2}-2 P A_{i} \cdot P B_{i} \cos a_{i} \\ +4 r^{2} & =P B_{i}^{2}+P^{\prime} B_{i}^{2}-2 P B_{i} \cdot P^{\prime} B_{i} \cos \angle P B_{i} P^{\prime} +\end{aligned} +$$ + +Since $A_{i} P^{\prime} B_{i} P$ is a parallelogram, we have that $P^{\prime} B_{i}=P A_{i}$ and $\angle P B_{i} P^{\prime}=\pi-a_{i}$. Subtracting the expression for $4 r^{2}$ from the one for $4 R^{2}$ yields $4\left(R^{2}-r^{2}\right)=-4 P A_{i} \cdot P B_{i} \cos a_{i}=-8 S_{\triangle A_{i} B_{i} P} \cot a_{i}$, hence we conclude that + +$$ +\tan ^{2} a_{i}=\frac{4 S_{\triangle A_{i} B_{i} P}^{2}}{\left(R^{2}-r^{2}\right)^{2}} +$$ + +Denote by $M_{i}$ the foot of the perpendicular from $P$ to $A_{i} B_{i}$ and let $m_{i}=$ $P M_{i}$. Then $S_{\triangle A_{i} B_{i} P}=R m_{i}$. Substituting this into (1) and adding up these relations for $i=1,2, \ldots, n$, we obtain + +$$ +\sum_{i=1}^{n} \tan ^{2} a_{i}=\frac{4 R^{2}}{\left(R^{2}-r^{2}\right)^{2}}\left(\sum_{i=1}^{n} m_{i}^{2}\right) +$$ + +Note that all the points $M_{i}$ lie on a circle with diameter $O P$ and form a regular $n$-gon. Denote its center by $F$. We have that $m_{i}^{2}=\left\|\overrightarrow{P M_{i}}\right\|^{2}=$ $\left\|\overrightarrow{F M_{i}}-\overrightarrow{F P}\right\|^{2}=\left\|\overrightarrow{F M}_{i}^{2}\right\|+\left\|\overrightarrow{F P}^{2}\right\|-2\left\langle\overrightarrow{F M_{i}}, \overrightarrow{F P}\right\rangle=r^{2} / 2-2\left\langle\overrightarrow{F M_{i}}, \overrightarrow{F P}\right\rangle$. From this it follows that $\sum_{i=1}^{n} m_{i}^{2}=2 n(r / 2)^{2}-2 \sum_{i=1}^{n}\left\langle\overrightarrow{F M_{i}}, \overrightarrow{F P}\right\rangle=$ $2 n(r / 2)^{2}-2\left\langle\sum_{i=1}^{n} \overrightarrow{F M_{i}}, \overrightarrow{F P}\right\rangle=2 n(r / 2)^{2}$, because $\sum_{i=1}^{n} \overrightarrow{F M_{i}}=\overrightarrow{0}$. Thus + +$$ +\sum_{i=1}^{n} \tan ^{2} a_{i}=\frac{4 R^{2}}{\left(R^{2}-r^{2}\right)^{2}} 2 n\left(\frac{r}{2}\right)^{2}=2 n \frac{(r / R)^{2}}{\left(1-(r / R)^{2}\right)^{2}}=2 n \frac{\cos ^{2} \frac{\pi}{2 n}}{\sin ^{4} \frac{\pi}{2 n}} +$$ + +Remark. For $n=1$ there is no regular 2-gon. However, if we think of a 2-gon as a line segment, the statement will remain true. +2. Suppose that $a>b$. Consider the polynomial $P(X)=x_{1} X^{n-1}+x_{2} X^{n-2}+$ $\cdots+x_{n-1} X+x_{n}$. We have $A_{n}=P(a), B_{n}=P(b), A_{n+1}=x_{0} a^{n}+$ $P(a)$, and $B_{n+1}=x_{0} b^{n}+P(b)$. Now $A_{n} / A_{n+1}b$, we have that $a^{i}>b^{i}$ and hence $x_{i} a^{n} b^{n-i} \geq x_{i} b^{n} a^{n-i}$ (also, for $i \geq 1$ the inequality is strict). Summing up all these inequalities for $i=1, \ldots, n$ we get $a^{n} P(b)>b^{n} P(a)$, which completes the proof for $a>b$. + +On the other hand, for $aB_{n} / B_{n+1}$, while for $a=b$ we have equality. Thus $A_{n} / A_{n+1}<$ $B_{n} / B_{n+1} \Leftrightarrow a>b$. + +3 . We shall use the following lemma +Lemma. If an altitude of a tetrahedron passes through the orthocenter of the opposite side, then each of the other altitudes possesses the same property. +Proof. Denote the tetrahedron by $S A B C$ and let $a=B C, b=C A$, $c=A B, m=S A, n=S B, p=S C$. It is enough to prove that an altitude passes through the orthocenter of the opposite side if and only if $a^{2}+m^{2}=b^{2}+n^{2}=c^{2}+p^{2}$. +Suppose that the foot $S^{\prime}$ of the altitude from $S$ is the orthocenter of $A B C$. Then $S S^{\prime} \perp A B C \Rightarrow S B^{2}-S C^{2}=S^{\prime} B^{2}-S^{\prime} C^{2}$. But from $A S^{\prime} \perp B C$ it follows that $A B^{2}-A C^{2}=S^{\prime} B^{2}-S^{\prime} C^{2}$. From these two equalities it can be concluded that $n^{2}-p^{2}=c^{2}-b^{2}$, or equivalently, $n^{2}+b^{2}=c^{2}+p^{2}$. Analogously, $a^{2}+m^{2}=n^{2}+b^{2}$, so we have proved the first part of the equivalence. +Now suppose that $a^{2}+m^{2}=b^{2}+n^{2}=c^{2}+p^{2}$. Defining $S^{\prime}$ as before, we get $n^{2}-p^{2}=S^{\prime} B^{2}-S^{\prime} C^{2}$. From the condition $n^{2}-p^{2}=c^{2}-b^{2}$ $\left(\Leftrightarrow b^{2}+n^{2}=c^{2}+p^{2}\right.$ ) we conclude that $A S^{\prime} \perp B C$. In the same way $C S^{\prime} \perp A B$, which proves that $S^{\prime}$ is the orthocenter of $\triangle A B C$. The lemma is thus proven. +Now using the lemma it is easy to see that if one of the angles at $S$ is right, than so are the others. Indeed, suppose that $\angle A S B=\pi / 2$. From the lemma we have that the altitude from $C$ passes through the orthocenter of $\triangle A S B$, which is $S$, so $C S \perp A S B$ and $\angle C S A=\angle C S B=\pi / 2$. +Therefore $m^{2}+n^{2}=c^{2}, n^{2}+p^{2}=a^{2}$, and $p^{2}+m^{2}=b^{2}$, so it follows that $m^{2}+n^{2}+p^{2}=\left(a^{2}+b^{2}+c^{2}\right) / 2$. By the inequality between the arithmetic and quadric means, we have that $\left(a^{2}+b^{2}+c^{2}\right) / 2 \geq 2 s^{2} / 3$, where $s$ denotes the semiperimeter of $\triangle A B C$. It remains to be shown that $2 s^{2} / 3 \geq 18 r^{2}$. Since $S_{\triangle A B C}=s r$, this is equivalent to $2 s^{4} / 3 \geq$ $18 S_{A B C}^{2}=18 s(s-a)(s-b)(s-c)$ by Heron's formula. This reduces to $s^{3} \geq 27(s-a)(s-b)(s-c)$, which is an obvious consequence of the AM-GM mean inequality. +Remark. In the place of the lemma one could prove that the opposite edges of the tetrahedron are mutually perpendicular and proceed in the same way. +4. Suppose that $n$ is such a natural number. If a prime number $p$ divides any of the numbers $n, n+1, \ldots, n+5$, then it must divide another one of them, so the only possibilities are $p=2,3,5$. Moreover, $n+1, n+2, n+3, n+4$ have no prime divisors other than 2 and 3 (if some prime number greater than 3 divides one of them, then none of the remaining numbers can have that divisor). Since two of these numbers are odd, they must be powers of + +3 (greater than 1). However, there are no two powers of 3 whose difference is 2 . Therefore there is no such natural number $n$. +Second solution. Obviously, none of $n, n+1, \ldots, n+5$ is divisible by 7; hence they form a reduced system of residues. We deduce that $n(n+$ 1) $\cdots(n+5) \equiv 1 \cdot 2 \cdots 6 \equiv-1(\bmod 7)$. If $\{n, \ldots, n+5\}$ can be partitioned into two subsets with the same products, both congruent to, say, $p$ modulo 7 , then $p^{2} \equiv-1(\bmod 7)$, which is impossible. +Remark. Erdős has proved that a set $n, n+1, \ldots, n+m$ of consecutive natural numbers can never be partitioned into two subsets with equal products of elements. +5. Denote respectively by $A_{1}, B_{1}, C_{1}$ and $D_{1}$ the points of intersection of the lines $A M, B M, C M$, and $D M$ with the opposite sides of the tetrahedron. Since $\operatorname{vol}(M B C D)=\operatorname{vol}(A B C D) \overrightarrow{M A_{1}} / \overrightarrow{A A_{1}}$, the relation we have to prove is equivalent to + +$$ +\overrightarrow{M A} \cdot \frac{\overrightarrow{M A_{1}}}{\overrightarrow{A A_{1}}}+\overrightarrow{M B} \cdot \frac{\overrightarrow{M B_{1}}}{\overrightarrow{B B_{1}}}+\overrightarrow{M C} \cdot \frac{\overrightarrow{M C_{1}}}{\overrightarrow{C C_{1}}}+\overrightarrow{M D} \cdot \frac{\overrightarrow{M D_{1}}}{\overrightarrow{D D_{1}}}=0 +$$ + +There exist unique real numbers $\alpha, \beta, \gamma$, and $\delta$ such that $\alpha+\beta+\gamma+\delta=1$ and for every point $O$ in space + +$$ +\overrightarrow{O M}=\alpha \overrightarrow{O A}+\beta \overrightarrow{O B}+\gamma \overrightarrow{O C}+\delta \overrightarrow{O D} +$$ + +(This follows easily from $\overrightarrow{O M}=\overrightarrow{O A}+\overrightarrow{A M}=\overrightarrow{O A}+k \overrightarrow{A B}+l \overrightarrow{A C}+m \overrightarrow{A D}=$ $\overrightarrow{A B}+k(\overrightarrow{O B}-\overrightarrow{O A})+l(\overrightarrow{O C}-\overrightarrow{O A})+m(\overrightarrow{O D}-\overrightarrow{O A})$ for some $k, l, m \in \mathbb{R}$.) Further, from the condition that $A_{1}$ belongs to the plane $B C D$ we obtain for every $O$ in space the following equality for some $\beta^{\prime}, \gamma^{\prime}, \delta^{\prime}$ : + +$$ +\overrightarrow{O A_{1}}=\beta^{\prime} \overrightarrow{O B}+\gamma^{\prime} \overrightarrow{O C}+\delta^{\prime} \overrightarrow{O D} +$$ + +However, for $\lambda=\overrightarrow{M A_{1}} / \overrightarrow{A A_{1}}, \overrightarrow{O M}=\lambda \overrightarrow{O A}+(1-\lambda) \overrightarrow{O A_{1}}$; hence substituting (2) and (3) in this expression and equating coefficients for $\overrightarrow{O A}$ we obtain $\lambda=\overrightarrow{M A_{1}} / \overrightarrow{A A_{1}}=\alpha$. Analogously, $\beta=\overrightarrow{M B_{1}} / \overrightarrow{B B_{1}}, \gamma=\overrightarrow{M C_{1}} / \overrightarrow{C C_{1}}$, and $\delta=\overrightarrow{M D_{1}} / \overrightarrow{D D_{1}}$; hence (1) follows immediately for $O=M$. +Remark. The statement of the problem actually follows from the fact that $M$ is the center of mass of the system with masses $\operatorname{vol}(M B C D)$, $\operatorname{vol}(M A C D), \operatorname{vol}(M A B D), \operatorname{vol}(M A B C)$ at $A, B, C, D$ respectively. Our proof is actually a formal verification of this fact. +6. Let $F$ be the midpoint of $B^{\prime} C^{\prime}, A^{\prime}$ the midpoint of $B C$, and $I$ the intersection point of the line $H F$ and the circle circumscribed about $\triangle B H C^{\prime}$. Denote by $M$ the intersection point of the line $A A^{\prime}$ with the circumscribed circle about the triangle $A B C$. Triangles $H B^{\prime} C^{\prime}$ and $A B C$ are similar. Since $\angle C^{\prime} I F=\angle A B C=\angle A^{\prime} M C, \angle C^{\prime} F I=\angle A A^{\prime} B=\angle M A^{\prime} C$, +$2 C^{\prime} F=C^{\prime} B^{\prime}$, and $2 A^{\prime} C=C B$, it follows that $\triangle C^{\prime} I B^{\prime} \sim \triangle C M B$, hence $\angle F I B^{\prime}=\angle A^{\prime} M B=\angle A C B$. Now one concludes that $I$ belongs to the circumscribed circles of $\triangle A B^{\prime} C^{\prime}$ (since $\left.\angle C^{\prime} I B^{\prime}=180^{\circ}-\angle C^{\prime} A B^{\prime}\right)$ and $\triangle H C B^{\prime}$. +Second Solution. We denote the angles of $\triangle A B C$ by $\alpha, \beta, \gamma$. Evidently $\triangle A B C \sim \triangle H C^{\prime} B^{\prime}$. Within $\triangle H C^{\prime} B^{\prime}$ there exists a unique point $I$ such that $\angle H I B^{\prime}=180^{\circ}-\gamma, \angle H I C^{\prime}=180^{\circ}-\beta$, and $\angle C^{\prime} I B^{\prime}=180^{\circ}-\alpha$, and all three circles must contain this point. Let $H I$ and $B^{\prime} C^{\prime}$ intersect in $F$. It remains to show that $F B^{\prime}=F C^{\prime}$. From $\angle H I B^{\prime}+\angle H B^{\prime} F=180^{\circ}$ we obtain $\angle I H B^{\prime}=\angle I B^{\prime} F$. Similarly, $\angle I H C^{\prime}=\angle I C^{\prime} F$. Thus circles around $\triangle I H C^{\prime}$ and $\triangle I H B^{\prime}$ are both tangent to $B^{\prime} C^{\prime}$, giving us $F B^{\prime 2}=$ $F I \cdot F H=F C^{\prime 2}$. +7. For $a=5$ one can take $n=10$, while for $a=6$ one takes $n=11$. Now assume $a \notin\{5,6\}$. +If there exists an integer $n$ such that each digit of $n(n+1) / 2$ is equal to $a$, then there is an integer $k$ such that $n(n+1) / 2=\left(10^{k}-1\right) a / 9$. After multiplying both sides of the equation by 72 , one obtains $36 n^{2}+36 n=$ $8 a \cdot 10^{k}-8 a$, which is equivalent to + +$$ +9(2 n+1)^{2}=8 a \cdot 10^{k}-8 a+9 +$$ + +So $8 a \cdot 10^{k}-8 a+9$ is the square of some odd integer. This means that its last digit is 1,5 , or 9 . Therefore $a \in\{1,3,5,6,8\}$. +If $a=3$ or $a=8$, the number on the RHS of (1) is divisible by 5 , but not by 25 (for $k \geq 2$ ), and thus cannot be a square. It remains to check the case $a=1$. In that case, (1) becomes $9(2 n+1)^{2}=8 \cdot 10^{k}+1$, or equivalently $[3(2 n+1)-1][3(2 n+1)+1]=8 \cdot 10^{k} \Rightarrow(3 n+1)(3 n+2)=2 \cdot 10^{k}$. Since the factors $3 n+1,3 n+2$ are relatively prime, this implies that one of them is $2^{k+1}$ and the other one is $5^{k}$. It is directly checked that their difference really equals 1 only for $k=1$ and $n=1$, which is excluded. Hence, the desired $n$ exists only for $a \in\{5,6\}$. +8. Let $A C=b, B C=a, A M=x, B M=y, C M=l$. Denote by $I_{1}$ the incenter and by $S_{1}$ the center of the excircle of $\triangle A M C$. Suppose that $P_{1}$ and $Q_{1}$ are feet of perpendiculars from $I_{1}$ and $S_{1}$, respectively, to the line $A C$. Then $\triangle I_{1} C P_{1} \sim \triangle S_{1} C Q_{1}$, hence $r_{1} / \rho_{1}=C P_{1} / C Q_{1}$. We have $C P_{1}=(A C+M C-A M) / 2=(b+l-x) / 2$ and $C Q_{1}=$ $(A C+M C+A M) / 2=(b+l+x) / 2$. Hence + +$$ +\frac{r_{1}}{\rho_{1}}=\frac{b+l-x}{b+l+x} +$$ + +We similarly obtain + +$$ +\frac{r_{2}}{\rho_{2}}=\frac{b+l-y}{b+l+y} \text { and } \frac{r}{\rho}=\frac{a+b-x-y}{a+b+x+y} +$$ + +What we have to prove is now equivalent to + +$$ +\frac{(b+l-x)(a+l-y)}{(b+l+x)(a+l+y)}=\frac{a+b-x-y}{a+b+x+y} +$$ + +Multiplying both sides of (1) by $(a+l+y)(b+l+x)(a+b+x+y)$ we obtain an expression that reduces to $l^{2} x+l^{2} y+x^{2} y+x y^{2}=b^{2} y+a^{2} x$. Dividing both sides by $c=x+y$, we get that (1) is equivalent to $l^{2}=$ $b^{2} y /(x+y)+a^{2} x /(x+y)-x y$, which is exactly Stewart's theorem for $l$. This finally proves the desired result. +9. Let us set $a=\sqrt{\sum_{i=1}^{n} u_{i}^{2}}$ and $b=\sqrt{\sum_{i=1}^{n} v_{i}^{2}}$. By Minkowski's inequality (for $p=2$ ) we have $\sum_{i=1}^{n}\left(u_{i}+v_{i}\right)^{2} \leq(a+b)^{2}$. Hence the LHS of the desired inequality is not greater than $1+(a+b)^{2}$, while the RHS is equal to $4\left(1+a^{2}\right)\left(1+b^{2}\right) / 3$. Now it is sufficient to prove that + +$$ +3+3(a+b)^{2} \leq 4\left(1+a^{2}\right)\left(1+b^{2}\right) +$$ + +The last inequality can be reduced to the trivial $0 \leq(a-b)^{2}+(2 a b-1)^{2}$. The equality in the initial inequality holds if and only if $u_{i} / v_{i}=c$ for some $c \in \mathbb{R}$ and $a=b=1 / \sqrt{2}$. +10. (a) Since $a_{n-1}1$, and let $a_{k}=q^{k}, k=1,2, \ldots$. Then $\left(1-a_{k-1} / a_{k}\right) / \sqrt{a_{k}}=(1-1 / q) / q^{k / 2}$, and consequently + +$$ +b_{n}=\left(1-\frac{1}{q}\right) \sum_{k=1}^{n} \frac{1}{q^{k / 2}}=\frac{\sqrt{q}+1}{q}\left(1-\frac{1}{q^{n / 2}}\right) . +$$ + +Since $(\sqrt{q}+1) / q$ can be arbitrarily close to 2 , one can set $q$ such that $(\sqrt{q}+1) / q>b$. Then $b_{n} \geq b$ for all sufficiently large $n$. +Second solution. +(a) Note that + +$$ +b_{n}=\sum_{k=1}^{n}\left(1-\frac{a_{k-1}}{a_{k}}\right) \frac{1}{\sqrt{a_{k}}}=\sum_{k=1}^{n}\left(a_{k}-a_{k-1}\right) \cdot \frac{1}{a_{k}^{3 / 2}} +$$ + +hence $b_{n}$ represents exactly the lower Darboux sum for the function $f(x)=x^{-3 / 2}$ on the interval $\left[a_{0}, a_{n}\right]$. Then $b_{n} \leq \int_{a_{0}}^{a_{n}} x^{-3 / 2} d x<$ $\int_{1}^{+\infty} x^{-3 / 2} d x=2$. +(b) For each $b<2$ there exists a number $\alpha>1$ such that $\int_{1}^{\alpha} x^{-3 / 2} d x>$ $b+(2-b) / 2$. Now, by Darboux's theorem, there exists an array $1=$ $a_{0} \leq a_{1} \leq \cdots \leq a_{n}=\alpha$ such that the corresponding Darboux sums are arbitrarily close to the value of the integral. In particular, there is an array $a_{0}, \ldots, a_{n}$ with $b_{n}>b$. +11. Let $S(x)=\left(x-x_{1}\right)\left(x-x_{2}\right) \cdots\left(x-x_{n}\right)$. We have $x^{3}-x_{i}^{3}=\left(x-x_{i}\right)(\omega x-$ $\left.x_{i}\right)\left(\omega^{2} x-x_{i}\right)$, where $\omega$ is a primitive third root of 1 . Multiplying these equalities for $i=1, \ldots, n$ we obtain + +$$ +T\left(x^{3}\right)=\left(x^{3}-x_{1}^{3}\right)\left(x^{3}-x_{2}^{3}\right) \cdots\left(x^{3}-x_{n}^{3}\right)=S(x) S(\omega x) S\left(\omega^{2} x\right) +$$ + +Since $S(\omega x)=P\left(x^{3}\right)+\omega x Q\left(x^{3}\right)+\omega^{2} x^{2} R\left(x^{3}\right)$ and $S\left(\omega^{2} x\right)=P\left(x^{3}\right)+$ $\omega^{2} x Q\left(x^{3}\right)+\omega x^{2} R\left(x^{3}\right)$, the above expression reduces to + +$$ +T\left(x^{3}\right)=P^{3}\left(x^{3}\right)+x^{3} Q^{3}\left(x^{3}\right)+x^{6} R^{3}\left(x^{3}\right)-3 P\left(x^{3}\right) Q\left(x^{3}\right) R\left(x^{3}\right) +$$ + +Therefore the zeros of the polynomial + +$$ +T(x)=P^{3}(x)+x Q^{3}(x)+x^{2} R^{3}(x)-3 P(x) Q(x) R(x) +$$ + +are exactly $x_{1}^{3}, \ldots, x_{n}^{3}$. It is easily verified that $\operatorname{deg} T=\operatorname{deg} S=n$, and hence $T$ is the desired polynomial. +12. Lemma. Five points are given in the plane such that no three of them are collinear. Then there are at least three triangles with vertices at these points that are not acute-angled. +Proof. We consider three cases, according to whether the convex hull of these points is a triangle, quadrilateral, or pentagon. +(i) Let a triangle $A B C$ be the convex hull and two other points $D$ and $E$ lie inside the triangle. At least two of the triangles $A D B, B D C$ and $C D A$ have obtuse angles at the point $D$. Similarly, at least two of the triangles $A E B, B E C$ and $C E A$ are obtuse-angled. Thus there are at least four non-acute-angled triangles. +(ii) Suppose that $A B C D$ is the convex hull and that $E$ is a point of its interior. At least one angle of the quadrilateral is not acute, determining one non-acute-angled triangle. Also, the point $E$ lies in the interior of either $\triangle A B C$ or $\triangle C D A$ hence, as in the previous case, it determines another two obtuse-angled triangles. +(iii) It is easy to see that at least two of the angles of the pentagon are not acute. We may assume that these two angles are among the angles corresponding to vertices $A, B$, and $C$. Now consider the quadrilateral $A C D E$. At least one its angles is not acute. Hence, there are at least three triangles that are not acute-angled. + +Now we consider all combinations of 5 points chosen from the given 100. There are $\binom{100}{5}$ such combinations, and for each of them there are at least three non-acute-angled triangles with vertices in it. On the other hand, vertices of each of the triangles are counted $\binom{97}{2}$ times. Hence there are at least $3\binom{100}{5} /\binom{97}{2}$ non-acute-angled triangles with vertices in the given 100 points. Since the number of all triangles with vertices in the given points is $\binom{100}{3}$, the ratio between the number of acute-angled triangles and the number of all triangles cannot be greater than + +$$ +1-\frac{3\binom{100}{5}}{\binom{97}{2}\binom{100}{3}}=0.7 +$$ + +### 4.13 Solutions to the Shortlisted Problems of IMO 1971 + +1. Assuming that $a, b, c$ in (1) exist, let us find what their values should be. Since $P_{2}(x)=x^{2}-2$, equation (1) for $n=1$ becomes $\left(x^{2}-4\right)^{2}=$ $\left[a\left(x^{2}-2\right)+b x+2 c\right]^{2}$. Therefore, there are two possibilities for $(a, b, c)$ : $(1,0,-1)$ and $(-1,0,1)$. In both cases we must prove that + +$$ +\left(x^{2}-4\right)\left[P_{n}(x)^{2}-4\right]=\left[P_{n+1}(x)-P_{n-1}(x)\right]^{2} +$$ + +It suffices to prove (2) for all $x$ in the interval $[-2,2]$. In this interval we can set $x=2 \cos t$ for some real $t$. We prove by induction that + +$$ +P_{n}(x)=2 \cos n t \quad \text { for all } n +$$ + +This is trivial for $n=0,1$. Assume (3) holds for some $n-1$ and $n$. Then $P_{n+1}(x)=4 \cos t \cos n t-2 \cos (n-1) t=2 \cos (n+1) t$ by the additive formula for the cosine. This completes the induction. +Now (2) reduces to the obviously correct equality + +$$ +16 \sin ^{2} t \sin ^{2} n t=(2 \cos (n+1) t-2 \cos (n-1) t)^{2} +$$ + +Second solution. If $x$ is fixed, the linear recurrence relation $P_{n+1}(x)+$ $P_{n-1}(x)=x P_{n}(x)$ can be solved in the standard way. The characteristic polynomial $t^{2}-x t+1$ has zeros $t_{1,2}$ with $t_{1}+t_{2}=x$ and $t_{1} t_{2}=1$; hence, the general $P_{n}(x)$ has the form $a t_{1}^{n}+b t_{2}^{n}$ for some constants $a$, $b$. From $P_{0}=2$ and $P_{1}=x$ we obtain that + +$$ +P_{n}(x)=t_{1}^{n}+t_{2}^{n} . +$$ + +Plugging in these values and using $t_{1} t_{2}=1$ one easily verifies (2). +2 . We will construct such a set $S_{m}$ of $2^{m}$ points. +Take vectors $u_{1}, \ldots, u_{m}$ in a given plane, such that $\left|u_{i}\right|=1 / 2$ and $0 \neq\left|c_{1} u_{1}+c_{2} u_{2}+\cdots+c_{n} u_{n}\right| \neq 1 / 2$ for any choice of numbers $c_{i}$ equal to 0 or $\pm 1$. Such vectors are easily constructed by induction on $m$ : For $u_{1}, \ldots, u_{m-1}$ fixed, there are only finitely many vector values $u_{m}$ that violate the upper condition, and we may set $u_{m}$ to be any other vector of length $1 / 2$. +Let $S_{m}$ be the set of all points $M_{0}+\varepsilon_{1} u_{1}+\varepsilon_{2} u_{2}+\cdots+\varepsilon_{m} u_{m}$, where $M_{0}$ is any fixed point in the plane and $\varepsilon_{i}= \pm 1$ for $i=1, \ldots, m$. Then $S_{m}$ obviously satisfies the condition of the problem. +3. Let $x, y, z$ be a solution of the given system with $x^{2}+y^{2}+z^{2}=\alpha<10$. Then + +$$ +x y+y z+z x=\frac{(x+y+z)^{2}-\left(x^{2}+y^{2}+z^{2}\right)}{2}=\frac{9-\alpha}{2} . +$$ + +Furthermore, $3 x y z=x^{3}+y^{3}+z^{3}-(x+y+z)\left(x^{2}+y^{2}+z^{2}-x y-y z-z x\right)$, which gives us $x y z=3(9-\alpha) / 2-4$. We now have + +$$ +\begin{aligned} +35= & x^{4}+y^{4}+z^{4}=\left(x^{3}+y^{3}+z^{3}\right)(x+y+z) \\ +& -\left(x^{2}+y^{2}+z^{2}\right)(x y+y z+z x)+x y z(x+y+z) \\ += & 45-\frac{\alpha(9-\alpha)}{2}+\frac{9(9-\alpha)}{2}-12 . +\end{aligned} +$$ + +The solutions in $\alpha$ are $\alpha=7$ and $\alpha=11$. Therefore $\alpha=7$, xyz $=-1$, $x y+x z+y z=1$, and + +$$ +\begin{aligned} +x^{5}+y^{5}+z^{5}= & \left(x^{4}+y^{4}+z^{4}\right)(x+y+z) \\ +& -\left(x^{3}+y^{3}+z^{3}\right)(x y+x z+y z)+x y z\left(x^{2}+y^{2}+z^{2}\right) \\ += & 35 \cdot 3-15 \cdot 1+7 \cdot(-1)=83 . +\end{aligned} +$$ + +4. In the coordinate system in which the $x$-axis passes through the centers of the circles and the $y$-axis is their common tangent, the circles have equations + +$$ +x^{2}+y^{2}+2 r_{1} x=0, \quad x^{2}+y^{2}-2 r_{2} x=0 . +$$ + +Let $p$ be the desired line with equation $y=a x+b$. The abscissas of points of intersection of $p$ with both circles satisfy one of + +$$ +\left(1+a^{2}\right) x^{2}+2\left(a b+r_{1}\right) x+b^{2}=0, \quad\left(1+a^{2}\right) x^{2}+2\left(a b-r_{2}\right) x+b^{2}=0 +$$ + +Let us denote the lengths of the chords and their projections onto the $x$-axis by $d$ and $d_{1}$, respectively. From these equations it follows that + +$$ +d_{1}^{2}=\frac{4\left(a b+r_{1}\right)^{2}}{\left(1+a^{2}\right)^{2}}-\frac{4 b^{2}}{1+a^{2}}=\frac{4\left(a b-r_{2}\right)^{2}}{\left(1+a^{2}\right)^{2}}-\frac{4 b^{2}}{1+a^{2}} +$$ + +Consider the point of intersection of $p$ with the $y$-axis. This point has equal powers with respect to both circles. Hence, if that point divides the segment determined on $p$ by the two circles on two segments of lengths $x$ and $y$, this power equals $x(x+d)=y(y+d)$, which implies $x=y=d / 2$. Thus each of the equations in (1) has two roots, one of which is thrice the other. This fact gives us $\left(a b+r_{1}\right)^{2}=4\left(1+a^{2}\right) b^{2} / 3$. From (1) and this we obtain + +$$ +\begin{gathered} +a b=\frac{r_{2}-r_{1}}{2}, \quad 4 b^{2}+a^{2} b^{2}=3\left[\left(a b+r_{1}\right)^{2}-a^{2} b^{2}\right]=3 r_{1} r_{2} \\ +a^{2}=\frac{4\left(r_{2}-r_{1}\right)^{2}}{14 r_{1} r_{2}-r_{1}^{2}-r_{2}^{2}}, \quad b^{2}=\frac{14 r_{1} r_{2}-r_{1}^{2}-r_{2}^{2}}{16} \\ +d_{1}^{2}=\frac{\left(14 r_{1} r_{2}-r_{1}^{2}-r_{2}^{2}\right)^{2}}{36\left(r_{1}+r_{2}\right)^{2}} +\end{gathered} +$$ + +Finally, since $d^{2}=d_{1}^{2}\left(1+a^{2}\right)$, we conclude that + +$$ +d^{2}=\frac{1}{12}\left(14 r_{1} r_{2}-r_{1}^{2}-r_{2}^{2}\right), +$$ + +and that the problem is solvable if and only if $7-4 \sqrt{3} \leq \frac{r_{1}}{r_{2}} \leq 7+4 \sqrt{3}$. +5. Without loss of generality, we may assume that $a \geq b \geq c \geq d \geq e$. Then $a-b=-(b-a) \geq 0, a-c \geq b-c \geq 0, a-d \geq b-d \geq 0$ and $a-e \geq b-e \geq 0$, and hence + +$$ +(a-b)(a-c)(a-d)(a-e)+(b-a)(b-c)(b-d)(b-e) \geq 0 +$$ + +Analogously, $(d-a)(d-b)(d-c)(d-e)+(e-a)(e-b)(e-c)(e-d) \geq 0$. Finally, $(c-a)(c-b)(c-d)(c-e) \geq 0$ as a product of two nonnegative numbers, from which the inequality stated in the problem follows. +Remark. The problem in an alternative formulation, accepted for the IMO, asked to prove that the analogous inequality + +$$ +\begin{gathered} +\left(a_{1}-a_{2}\right)\left(a_{1}-a_{2}\right) \cdots\left(a_{1}-a_{n}\right)+\left(a_{2}-a_{1}\right)\left(a_{2}-a_{3}\right) \cdots\left(a_{2}-a_{n}\right)+\cdots \\ ++\left(a_{n}-a_{1}\right)\left(a_{n}-a_{2}\right) \cdots\left(a_{n}-a_{n-1}\right) \geq 0 +\end{gathered} +$$ + +holds for arbitrary real numbers $a_{i}$ if and only if $n=3$ or $n=5$. +The case $n=3$ is analogous to $n=5$. For $n=4$, a counterexample is $a_{1}=0, a_{2}=a_{3}=a_{4}=1$, while for $n>5$ one can take $a_{1}=a_{2}=\cdots=$ $a_{n-4}=0, a_{n-3}=a_{n-2}=a_{n-1}=2, a_{n}=1$ as a counterexample. +6. The proof goes by induction on $n$. For $n=2$, the following numeration satisfies the conditions (a)-(d): $C_{1}=11, C_{2}=12, C_{3}=22, C_{4}=21$. +Suppose that $n>2$, and that the numeration $C_{1}, C_{2}, \ldots, C_{2^{n-1}}$ of a regular $2^{n-1}$-gon, in cyclical order, satisfies (i)-(iv). Then one can assign to the vertices of a $2^{n}$-gon cyclically the following numbers: + +$$ +\overline{1 C_{1}}, \overline{1 C_{2}}, \ldots, \overline{1 C_{2^{n-1}}}, \overline{2 C_{2^{n-1}}}, \ldots, \overline{2 C_{2}}, \overline{2 C_{1}} +$$ + +The conditions (i), (ii) obviously hold, while (iii) and (iv) follow from the inductive assumption. +7. (a) Suppose that $X, Y, Z$ are fixed on segments $A B, B C, C D$. It is proven in a standard way that if $\angle A T X \neq \angle Z T D$, then $Z T+T X$ can be reduced. It follows that if there exists a broken line $X Y Z T X$ of minimal length, then the following conditions hold: + +$$ +\begin{aligned} +& \angle D A B=\pi-\angle A T X-\angle A X T \\ +& \angle A B C=\pi-\angle B X Y-\angle B Y X=\pi-\angle A X T-\angle C Y Z \\ +& \angle B C D=\pi-\angle C Y Z-\angle C Z Y \\ +& \angle C D A=\pi-\angle D T Z-\angle D Z T=\pi-\angle A T X-\angle C Z Y . +\end{aligned} +$$ + +Thus $\sigma=0$. +(b) Now let $\sigma=0$. Let us cut the surface of the tetrahedron along the edges $A C, C D$, and $D B$ and set it down into a plane. Consider the plane figure $\mathcal{S}=A C D^{\prime} B D^{\prime \prime} C^{\prime}$ thus obtained made up of triangles $B C D^{\prime}, A B C, A B D^{\prime \prime}$, and $A C^{\prime} D^{\prime \prime}$, with $Z^{\prime}, T^{\prime}, Z^{\prime \prime}$ respectively on $C D^{\prime}, A D^{\prime \prime}, C^{\prime} D^{\prime \prime}$ (here $C^{\prime}$ corresponds to $C$, etc.). Since +$\angle C^{\prime} D^{\prime \prime} A+\angle D^{\prime \prime} A B+\angle A B C+\angle B C D^{\prime}=0$ as an oriented angle (because $\sigma=0$ ), the lines $C D^{\prime}$ and $C^{\prime} D^{\prime \prime}$ are parallel and equally oriented; i.e., $C D^{\prime} D^{\prime \prime} C^{\prime}$ is a parallelogram. +The broken line $X Y Z T X$ has minimal length if and only if $Z^{\prime \prime}, T^{\prime}, X$, $Y, Z^{\prime}$ are collinear (where $Z^{\prime} Z^{\prime \prime} \|$ $C C^{\prime}$ ), and then this length equals $Z^{\prime} Z^{\prime \prime}=C C^{\prime}=2 A C \sin (\alpha / 2)$. There is an infinity of such lines, one for every line $Z^{\prime} Z^{\prime \prime}$ parallel to $C C^{\prime}$ that meets the interiors of all the segments $C B, B A, A D^{\prime \prime}$. Such +![](https://cdn.mathpix.com/cropped/2024_11_18_8e985d6b9c83aa3e9d0eg-389.jpg?height=300&width=516&top_left_y=408&top_left_x=835) +$Z^{\prime} Z^{\prime \prime}$ exist. Indeed, the triangles $C A B$ and $D^{\prime \prime} A B$ are acute-angled, and thus the segment $A B$ has a common interior point with the parallelogram $C D^{\prime} D^{\prime \prime} C^{\prime}$. Therefore the desired result follows. +8. Suppose that $a, b, c, t$ satisfy all the conditions. Then $a b c \neq 0$ and + +$$ +x_{1} x_{2}=\frac{c}{a}, \quad x_{2} x_{3}=\frac{a}{b}, \quad x_{3} x_{1}=\frac{b}{c} . +$$ + +Multiplying these equations, we obtain $x_{1}^{2} x_{2}^{2} x_{3}^{2}=1$, and hence $x_{1} x_{2} x_{3}=$ $\varepsilon= \pm 1$. From (1) we get $x_{1}=\varepsilon b / a, x_{2}=\varepsilon c / b, x_{3}=\varepsilon a / c$. Substituting $x_{1}$ in the first equation, we get $a b^{2} / a^{2}+t \varepsilon b^{2} / a+c=0$, which gives us + +$$ +b^{2}(1+t \varepsilon)=-a c . +$$ + +Analogously, $c^{2}(1+t \varepsilon)=-a b$ and $a^{2}(1+t \varepsilon)=-b c$, and therefore $(1+$ $t \varepsilon)^{3}=-1$; i.e., $1+t \varepsilon=-1$, since it is real. This also implies together with (1) that $b^{2}=a c, c^{2}=a b$, and $a^{2}=b c$, and consequently + +$$ +a=b=c \text {. } +$$ + +Thus the three equations in the problem are equal, which is impossible. Hence, such $a, b, c, t$ do not exist. +9. We use induction. Since $T_{1}=0, T_{2}=1, T_{3}=2, T_{4}=3, T_{5}=5, T_{6}=8$, the statement is true for $n=1,2,3$. Suppose that both formulas from the problem hold for some $n \geq 3$. Then + +$$ +\begin{aligned} +& T_{2 n+1}=1+T_{2 n}+2^{n-1}=\left[\frac{17}{7} 2^{n-1}+2^{n-1}\right]=\left[\frac{12}{7} 2^{n}\right] \\ +& T_{2 n+2}=1+T_{2 n-3}+2^{n+1}=\left[\frac{12}{7} 2^{n-2}+2^{n+1}\right]=\left[\frac{17}{7} 2^{n}\right] . +\end{aligned} +$$ + +Therefore the formulas hold for $n+1$, which completes the proof. +10. We use induction. Suppose that every two of the numbers $a_{1}=2^{n_{1}}-$ $3, a_{2}=2^{n_{2}}-3, \ldots, a_{k}=2^{n_{k}}-3$, where $2=n_{1}n_{k}$. +11. We use induction. The statement for $n=1$ is trivial. Suppose that it holds for $n=k$ and consider $n=k+1$. From the given condition, we have + +$$ +\begin{gathered} +\sum_{j=1}^{k}\left|a_{j, 1} x_{1}+\cdots+a_{j, k} x_{k}+a_{j, k+1}\right| \\ ++\left|a_{k+1,1} x_{1}+\cdots+a_{k+1, k} x_{k}+a_{k+1, k+1}\right| \leq M \\ +\sum_{j=1}^{k}\left|a_{j, 1} x_{1}+\cdots+a_{j, k} x_{k}-a_{j, k+1}\right| \\ ++\left|a_{k+1,1} x_{1}+\cdots+a_{k+1, k} x_{k}-a_{k+1, k+1}\right| \leq M +\end{gathered} +$$ + +for each choice of $x_{i}= \pm 1$. Since $|a+b|+|a-b| \geq 2|a|$ for all $a, b$, we obtain + +$$ +\begin{aligned} +2 \sum_{j=1}^{k}\left|a_{j 1} x_{1}+\cdots+a_{j k} x_{k}\right|+2\left|a_{k+1, k+1}\right| & \leq 2 M, \text { that is, } \\ +\sum_{j=1}^{k}\left|a_{j 1} x_{1}+\cdots+a_{j k} x_{k}\right| & \leq M-\left|a_{k+1, k+1}\right| +\end{aligned} +$$ + +Now by the inductive assumption $\sum_{j=1}^{k}\left|a_{j j}\right| \leq M-\left|a_{k+1, k+1}\right|$, which is equivalent to the desired inequality. +12. Let us start with the case $A=A^{\prime}$. If the triangles $A B C$ and $A^{\prime} B^{\prime} C^{\prime}$ are oppositely oriented, then they are symmetric with respect to some axis, and the statement is true. Suppose that they are equally oriented. There is a rotation around $A$ by $60^{\circ}$ that maps $A B B^{\prime}$ onto $A C C^{\prime}$. This rotation also maps the midpoint $B_{0}$ of $B B^{\prime}$ onto the midpoint $C_{0}$ of $C C^{\prime}$, hence the triangle $A B_{0} C_{0}$ is equilateral. +In the general case, when $A \neq A^{\prime}$, let us denote by $T$ the translation that maps $A$ onto $A^{\prime}$. Let $X^{\prime}$ be the image of a point $X$ under the (unique) isometry mapping $A B C$ onto $A^{\prime} B^{\prime} C^{\prime}$, and $X^{\prime \prime}$ the image of $X$ under $T$. Furthermore, let $X_{0}, X_{0}^{\prime}$ be the midpoints of segments $X X^{\prime}, X^{\prime} X^{\prime \prime}$. Then $X_{0}$ is the image of $X_{0}^{\prime}$ under the translation $-(1 / 2) T$. However, since it has already been proven that the triangle $A_{0}^{\prime} B_{0}^{\prime} C_{0}^{\prime}$ is equilateral, its image $A_{0} B_{0} C_{0}$ under $(1 / 2) T$ is also equilateral. The statement of the problem is thus proven. +13. Let $p$ be the least of all the sums of elements in one row or column. If $p \geq n / 2$, then the sum of all elements of the array is $s \geq n p \geq n^{2} / 2$. + +Now suppose that $p1248$. For each segment $l_{i}=A_{i} A_{i+1}$ of the broken line, consider the figure $V_{i}$ obtained by a circle of radius 1 whose center moves along it, and let $\overline{V_{i}}$ be obtained by cutting off the circle of radius 1 with center at the starting point of $l_{i}$. The area of $\overline{V_{i}}$ is equal to $2 A_{i} A_{i+1}$. It is clear that the union of all the figures $\overline{V_{i}}$ together with a semicircle with center in $A_{1}$ and a semicircle with center in $A_{n}$ contains $V$ completely. Therefore + +$$ +S(V) \leq \pi+2 A_{1} A_{2}+2 A_{2} A_{3}+\cdots+2 A_{n-1} A_{n}=\pi+2 L . +$$ + +This completes the proof. +15. Assume the opposite. Then one can numerate the cards 1 to 99, with a number $n_{i}$ written on the card $i$, so that $n_{98} \neq n_{99}$. Denote by $x_{i}$ the remainder of $n_{1}+n_{2}+\cdots+n_{i}$ upon division by 100 , for $i=1,2, \ldots, 99$. All $x_{i}$ must be distinct: Indeed, if $x_{i}=x_{j}, i0$, $10^{k \beta} \equiv 1\left(\bmod \left(10^{\beta}-1\right) m\right)$ (this is in fact a consequence of Fermat's theorem). The number + +$$ +\frac{10^{k \beta}-1}{10^{\beta}-1} M_{\beta}=10^{(k-1) \beta} M_{\beta}+10^{(k-2) \beta} M_{\beta}+\cdots+M_{\beta} +$$ + +is a multiple of $n=5^{\beta} m$ with the required property. +7. (i) Consider the circumscribing cube $O Q_{1} P R_{1} O_{1} Q P_{1} R$ (that is, the cube in which the edges of the tetrahedron are small diagonals), of side $b=a \sqrt{2} / 2$. The left-hand side is the sum of squares of the projections of the edges of the tetrahedron onto a perpendicular $l$ to $\pi$. On the other hand, if $l$ +![](https://cdn.mathpix.com/cropped/2024_11_18_8e985d6b9c83aa3e9d0eg-395.jpg?height=363&width=333&top_left_y=609&top_left_x=936) +forms angles $\varphi_{1}, \varphi_{2}, \varphi_{3}$ with $O O_{1}, O Q_{1}, O R_{1}$ respectively, then the projections of $O P$ and $Q R$ onto $l$ have lengths $b\left(\cos \varphi_{2}+\cos \varphi_{3}\right)$ and $b\left|\cos \varphi_{2}-\cos \varphi_{3}\right|$. Summing up all these expressions, we obtain + +$$ +4 b^{2}\left(\cos ^{2} \varphi_{1}+\cos ^{2} \varphi_{2}+\cos ^{2} \varphi_{3}\right)=4 b^{2}=2 a^{2} +$$ + +(ii) We construct a required tetrahedron of edge length $a$ given in (i). Take $O$ arbitrarily on $\pi_{0}$, and let $p, q, r$ be the distances of $O$ from $\pi_{1}, \pi_{2}, \pi_{3}$. Since $a>p, q, r,|p-q|$, we can choose $P$ on $\pi_{1}$ anywhere at distance $a$ from $O$, and $Q$ at one of the two points on $\pi_{2}$ at distance $a$ from both $O$ and $P$. Consider the fourth vertex of the tetrahedron: its distance from $\pi_{0}$ will satisfy the equation from (i); i.e., there are two values for this distance; clearly, one of them is $r$, putting $R$ on $\pi_{3}$. +8. Let $f(m, n)=\frac{(2 m)!(2 n)!}{m!n!(m+n)!}$. Then it is directly shown that + +$$ +f(m, n)=4 f(m, n-1)-f(m+1, n-1), +$$ + +and thus $n$ may be successively reduced until one obtains $f(m, n)=$ $\sum_{r} c_{r} f(r, 0)$. Now $f(r, 0)$ is a simple binomial coefficient, and the $c_{r}$ 's are integers. +Second solution. For each prime $p$, the greatest exponents of $p$ that divide the numerator $(2 m)!(2 n)$ ! and denominator $m!n!(m+n)$ ! are respectively + +$$ +\sum_{k>0}\left(\left[\frac{2 m}{p^{k}}\right]+\left[\frac{2 n}{p^{k}}\right]\right) \quad \text { and } \quad \sum_{k>0}\left(\left[\frac{m}{p^{k}}\right]+\left[\frac{n}{p^{k}}\right]+\left[\frac{m+n}{p^{k}}\right]\right) +$$ + +hence it suffices to show that the first exponent is not less than the second one for every $p$. This follows from the fact that for each real $x,[2 x]+[2 y] \geq$ +$[x]+[y]+[x+y]$, which is straightforward to prove (for example, using $[2 x]=[x]+[x+1 / 2])$. +9. Clearly $x_{1}=x_{2}=x_{3}=x_{4}=x_{5}$ is a solution. We shall show that this describes all solutions. +Suppose that not all $x_{i}$ are equal. Then among $x_{3}, x_{5}, x_{2}, x_{4}, x_{1}$ two consecutive are distinct: Assume w.l.o.g. that $x_{3} \neq x_{5}$. Moreover, since $\left(1 / x_{1}, \ldots, 1 / x_{5}\right)$ is a solution whenever $\left(x_{1}, \ldots, x_{5}\right)$ is, we may assume that $x_{3}x_{3}$. Then $x_{5}^{2}>x_{1} x_{3}$, which together with (iv) gives $x_{4}^{2} \leq x_{1} x_{3}x_{5} x_{3}$, a contradiction. +Consider next the case $x_{1}>x_{2}$. We infer from (i) that $x_{1} \geq \sqrt{x_{3} x_{5}}>x_{3}$ and $x_{2} \leq \sqrt{x_{3} x_{5}}x_{2}$. +Second solution. + +$$ +\begin{aligned} +0 & \geq L_{1}=\left(x_{1}^{2}-x_{3} x_{5}\right)\left(x_{2}^{2}-x_{3} x_{5}\right)=x_{1}^{2} x_{2}^{2}+x_{3}^{2} x_{5}^{2}-\left(x_{1}^{2}+x_{2}^{2}\right) x_{3} x_{5} \\ +& \geq x_{1}^{2} x_{2}^{2}+x_{3}^{2} x_{5}^{2}-\frac{1}{2}\left(x_{1}^{2} x_{3}^{2}+x_{1}^{2} x_{5}^{2}+x_{2}^{2} x_{3}^{2}+x_{2}^{2} x_{5}^{2}\right) +\end{aligned} +$$ + +and analogously for $L_{2}, \ldots, L_{5}$. Therefore $L_{1}+L_{2}+L_{3}+L_{4}+L_{5} \geq 0$, with the only case of equality $x_{1}=x_{2}=x_{3}=x_{4}=x_{5}$. +10. Consider first a triangle. It can be decomposed into $k=3$ cyclic quadrilaterals by perpendiculars from some interior point of it to the sides; also, it can be decomposed into a cyclic quadrilateral and a triangle, and it follows by induction that this decomposition is possible for every $k$. Since every triangle can be cut into two triangles, the required decomposition is possible for each $n \geq 6$. It remains to treat the cases $n=4$ and $n=5$. $n=4$. If the center $O$ of the circumcircle is inside a cyclic quadrilateral +$A B C D$, then the required decomposition is effected by perpendiculars from $O$ to the four sides. Otherwise, let $C$ and $D$ be the vertices of the obtuse angles of the quadrilateral. Draw the perpendiculars at $C$ and $D$ to the lines $B C$ and $A D$ respectively, and choose points $P$ and $Q$ on them such that $P Q \| A B$. Then the required decomposition is effected by $C P, P Q, Q D$ and the perpendiculars from $P$ and $Q$ to $A B$. $n=5$. If $A B C D$ is an isosceles trapezoid with $A B \| C D$ and $A D=B C$, then it is trivially decomposed by lines parallel to $A B$. Otherwise, $A B C D$ can be decomposed into a cyclic quadrilateral and a trapezoid; this trapezoid can be cut into an isosceles trapezoid and a triangle, which can further be cut into three cyclic quadrilaterals and an isosceles trapezoid. + +Remark. It can be shown that the assertion is not true for $n=2$ and $n=3$. +11. Let $\angle A=2 x, \angle B=2 y, \angle C=2 z$. +(a) Denote by $M_{i}$ the center of $K_{i}, i=1,2, \ldots$ If $N_{1}, N_{2}$ are the projections of $M_{1}, M_{2}$ onto $A B$, we have $A N_{1}=r_{1} \cot x, N_{2} B=r_{2} \cot y$, and $N_{1} N_{2}=\sqrt{\left(r_{1}+r_{2}\right)^{2}-\left(r_{1}-r_{2}\right)^{2}}=2 \sqrt{r_{1} r_{2}}$. The required relation between $r_{1}, r_{2}$ follows from $A B=A N_{1}+N_{1} N_{2}+N_{2} B$. +If this relation is further considered as a quadratic equation in $\sqrt{r_{2}}$, then its discriminant, which equals + +$$ +\Delta=4\left(r(\cot x+\cot y) \cot y-r_{1}(\cot x \cot y-1)\right), +$$ + +must be nonnegative, and therefore $r_{1} \leq r \cot y \cot z$. Then $t_{1}, t_{2}, \ldots$ exist, and we can assume that $t_{i} \in[0, \pi / 2]$. +(b) Substituting $r_{1}=r \cot y \cot z \sin ^{2} t_{1}, r_{2}=r \cot z \cot x \sin ^{2} t_{2}$ in the relation of (a) we obtain that $\sin ^{2} t_{1}+\sin ^{2} t_{2}+k^{2}+2 k \sin t_{1} \sin t_{2}=1$, where we set $k=\sqrt{\tan x \tan y}$. It follows that $\left(k+\sin t_{1} \sin t_{2}\right)^{2}=$ $\left(1-\sin ^{2} t_{1}\right)\left(1-\sin ^{2} t_{2}\right)=\cos ^{2} t_{1} \cos ^{2} t_{2}$, and hence + +$$ +\cos \left(t_{1}+t_{2}\right)=\cos t_{1} \cos t_{2}-\sin t_{1} \sin t_{2}=k=\sqrt{\tan x \tan y} +$$ + +which is constant. Writing the analogous relations for each $t_{i}, t_{i+1}$ we conclude that $t_{1}+t_{2}=t_{4}+t_{5}, t_{2}+t_{3}=t_{5}+t_{6}$, and $t_{3}+t_{4}=t_{6}+t_{7}$. It follows that $t_{1}=t_{7}$, i.e., $K_{1}=K_{7}$. +12. First we observe that it is not essential to require the subsets to be disjoint (if they aren't, one simply excludes their intersection). There are $2^{10}-1=$ 1023 different subsets and at most 990 different sums. By the pigeonhole principle there are two different subsets with equal sums. + +### 4.15 Solutions to the Shortlisted Problems of IMO 1973 + +1. The condition of the point $P$ can be written in the form $\frac{A P^{2}}{A P \cdot P A_{1}}+\frac{B P^{2}}{B P \cdot P B_{1}}+$ $\frac{C P^{2}}{C P \cdot P C_{1}}+\frac{D P^{2}}{D P \cdot P D_{1}}=4$. All the four denominators are equal to $R^{2}-O P^{2}$, i.e., to the power of $P$ with respect to $S$. Thus the condition becomes + +$$ +A P^{2}+B P^{2}+C P^{2}+D P^{2}=4\left(R^{2}-O P^{2}\right) +$$ + +Let $M$ and $N$ be the midpoints of segments $A B$ and $C D$ respectively, and $G$ the midpoint of $M N$, or the centroid of $A B C D$. By Stewart's formula, an arbitrary point $P$ satisfies + +$$ +\begin{aligned} +A P^{2}+B P^{2}+C P^{2}+D P^{2} & =2 M P^{2}+2 N P^{2}+\frac{1}{2} A B^{2}+\frac{1}{2} C D^{2} \\ +& =4 G P^{2}+M N^{2}+\frac{1}{2}\left(A B^{2}+C D^{2}\right) +\end{aligned} +$$ + +Particularly, for $P \equiv O$ we get $4 R^{2}=4 O G^{2}+M N^{2}+\frac{1}{2}\left(A B^{2}+C D^{2}\right)$, and the above equality becomes + +$$ +A P^{2}+B P^{2}+C P^{2}+D P^{2}=4 G P^{2}+4 R^{2}-4 O G^{2} +$$ + +Therefore (1) is equivalent to $O G^{2}=O P^{2}+G P^{2} \Leftrightarrow \angle O P G=90^{\circ}$. Hence the locus of points $P$ is the sphere with diameter $O G$. Now the converse is easy. +2. Let $D^{\prime}$ be the reflection of $D$ across $A$. Since $B C A D^{\prime}$ is then a parallelogram, the condition $B D \geq A C$ is equivalent to $B D \geq B D^{\prime}$, which is in turn equivalent to $\angle B A D \geq \angle B A D^{\prime}$, i.e. to $\angle B A D \geq 90^{\circ}$. Thus the needed locus is actually the locus of points $A$ for which there exist points $B, D$ inside $K$ with $\angle B A D=90^{\circ}$. Such points $B, D$ exist if and only if the two tangents from $A$ to $K$, say $A P$ and $A Q$, determine an obtuse angle. Then if $P, Q \in K$, we have $\angle P A O=\angle Q A O=\varphi>45^{\circ}$; hence $O A=\frac{O P}{\sin \varphi}0$. Point $G$ belongs to a plane $A B C$ with $A \in O x, B \in O y, C \in O z$ if and only if there exist positive real numbers $\lambda, \mu, \nu$ with sum 1 such that $\lambda \overrightarrow{O A}+\mu \overrightarrow{O B}+\nu \overrightarrow{O C}=\overrightarrow{O G}$, which is equivalent to $\lambda \alpha=\mu \beta=\nu \gamma=1$. Such $\lambda, \mu, \nu$ exist if and only if + +$$ +\alpha, \beta, \gamma>0 \quad \text { and } \quad \frac{1}{\alpha}+\frac{1}{\beta}+\frac{1}{\gamma}=1 +$$ + +Since the volume of $O A B C$ is proportional to the product $\alpha \beta \gamma$, it is minimized when $\frac{1}{\alpha} \cdot \frac{1}{\beta} \cdot \frac{1}{\gamma}$ is maximized, which occurs when $\alpha=\beta=\gamma=3$ and $G$ is the centroid of $\triangle A B C$. +10. Let +$b_{k}=a_{1} q^{k-1}+\cdots+a_{k-1} q+a_{k}+a_{k+1} q+\cdots+a_{n} q^{n-k}, \quad k=1,2, \ldots, n$. +We show that these numbers satisfy the required conditions. Obviously $b_{k} \geq a_{k}$. Further, + +$$ +b_{k+1}-q b_{k}=-\left[\left(q^{2}-1\right) a_{k+1}+\cdots+q^{n-k-1}\left(q^{2}-1\right) a_{n}\right]>0 +$$ + +we analogously obtain $q b_{k+1}-b_{k}<0$. Finally, + +$$ +\begin{aligned} +b_{1}+b_{2}+\cdots+b_{n}= & a_{1}\left(q^{n-1}+\cdots+q+1\right)+\ldots \\ +& +a_{k}\left(q^{n-k}+\cdots+q+1+q+\cdots+q^{k-1}\right)+\ldots \\ +\leq & \left(a_{1}+a_{2}+\cdots+a_{n}\right)\left(1+2 q+2 q^{2}+\cdots+2 q^{n-1}\right) \\ +< & \frac{1+q}{1-q}\left(a_{1}+\cdots+a_{n}\right) +\end{aligned} +$$ + +11. Putting $x+\frac{1}{x}=t$ we also get $x^{2}+\frac{1}{x^{2}}=t^{2}-2$, and the given equation reduces to $t^{2}+a t+b-2=0$. Since $x=\frac{t \pm \sqrt{t^{2}-4}}{2}, x$ will be real if and only if $|t| \geq 2, t \in \mathbb{R}$. Thus we need the minimum value of $a^{2}+b^{2}$ under the condition $a t+b=-\left(t^{2}-2\right),|t| \geq 2$. +However, by the Cauchy-Schwarz inequality we have + +$$ +\left(a^{2}+b^{2}\right)\left(t^{2}+1\right) \geq(a t+b)^{2}=\left(t^{2}-2\right)^{2} +$$ + +It follows that $a^{2}+b^{2} \geq h(t)=\frac{\left(t^{2}-2\right)^{2}}{t^{2}+1}$. Since $h(t)=\left(t^{2}+1\right)+\frac{9}{t^{2}+1}-6$ is increasing for $t \geq 2$, we conclude that $a^{2}+b^{2} \geq h(2)=\frac{4}{5}$. +The cases of equality are easy to examine: These are $a= \pm \frac{4}{5}$ and $b=-\frac{2}{5}$. Second solution. In fact, there was no need for considering $x=t+1 / t$. By the Cauchy-Schwarz inequality we have $\left(a^{2}+2 b^{2}+a^{2}\right)\left(x^{6}+x^{4} / 2+x^{2}\right) \geq$ $\left(a x^{3}+b x^{2}+a x\right)^{2}=\left(x^{4}+1\right)^{2}$. Hence + +$$ +a^{2}+b^{2} \geq \frac{\left(x^{4}+1\right)^{2}}{2 x^{6}+x^{4}+2 x^{2}} \geq \frac{4}{5} +$$ + +with equality for $x=1$. +12. Observe that the absolute values of the determinants of the given matrices are invariant under all the admitted operations. The statement follows from $\operatorname{det} A=16 \neq \operatorname{det} B=0$. +13. Let $S_{1}, S_{2}, S_{3}, S_{4}$ denote the areas of the faces of the tetrahedron, $V$ its volume, $h_{1}, h_{2}, h_{3}, h_{4}$ its altitudes, and $r$ the radius of its inscribed sphere. Since + +$$ +3 V=S_{1} h_{1}=S_{2} h_{2}=S_{3} h_{3}=S_{4} h_{4}=\left(S_{1}+S_{2}+S_{3}+S_{4}\right) r, +$$ + +it follows that + +$$ +\frac{1}{h_{1}}+\frac{1}{h_{2}}+\frac{1}{h_{3}}+\frac{1}{h_{4}}=\frac{1}{r} . +$$ + +In our case, $h_{1}, h_{2}, h_{3}, h_{4} \geq 1$, hence $r \geq 1 / 4$. On the other hand, it is clear that a sphere of radius greater than $1 / 4$ cannot be inscribed in a tetrahedron all of whose altitudes have length equal to 1 . Thus the answer is $1 / 4$. +14. Suppose that the soldier starts at the vertex $A$ of the equilateral triangle $A B C$ of side length $a$. Let $\varphi, \psi$ be the arcs of circles with centers $B$ and $C$ and radii $a \sqrt{3} / 4$ respectively, that lie inside the triangle. In order to check the vertices $B, C$, he must visit some points $D \in \varphi$ and $E \in \psi$. +![](https://cdn.mathpix.com/cropped/2024_11_18_8e985d6b9c83aa3e9d0eg-401.jpg?height=324&width=394&top_left_y=1496&top_left_x=887) + +Thus his path cannot be shorter than the path $A D E$ (or $A E D$ ) itself. The length of the path $A D E$ is $A D+D E \geq A D+D C-a \sqrt{3} / 4$. Let $F$ be the reflection of $C$ across the line $M N$, where $M, N$ are the midpoints of $A B$ and $B C$. Then $D C \geq D F$ and hence $A D+D C \geq A D+D F \geq A F$. Consequently $A D+D E \geq A F-a \frac{\sqrt{3}}{4}=a\left(\frac{\sqrt{7}}{2}-\frac{\sqrt{3}}{4}\right)$, with equality if and only if $D$ is the midpoint of $\operatorname{arc} \varphi$ and $E=(C D) \cap \psi$. + +Moreover, it is easy to verify that, in following the path $A D E$, the soldier will check the whole region. Therefore this path (as well as the one symmetric to it) is shortest possible path that the soldier can take in order to check the entire field. +15. If $z=\cos \theta+i \sin \theta$, then $z-z^{-1}=2 i \sin \theta$. Now put $z=\cos \frac{\pi}{2 n+1}+$ $i \sin \frac{\pi}{2 n+1}$. Using de Moivre's formula we transform the required equality into + +$$ +A=\prod_{k=1}^{n}\left(z^{k}-z^{-k}\right)=i^{n} \sqrt{2 n+1} +$$ + +On the other hand, the complex numbers $z^{2 k}(k=-n,-n+1, \ldots, n)$ are the roots of $x^{2 n+1}-1$, and hence + +$$ +\prod_{k=1}^{n}\left(x-z^{2 k}\right)\left(x-z^{-2 k}\right)=\frac{x^{2 n+1}-1}{x-1}=x^{2 n}+\cdots+x+1 +$$ + +Now we go back to proving (1). We have + +$$ +(-1)^{n} z^{n(n+1) / 2} A=\prod_{k=1}^{n}\left(1-z^{2 k}\right) \quad \text { and } \quad z^{-n(n+1) / 2} A=\prod_{k=1}^{n}\left(1-z^{-2 k}\right) +$$ + +Multiplying these two equalities, we obtain $(-1)^{n} A^{2}=\prod_{k=1}^{n}\left(1-z^{2 k}\right)(1-$ $\left.z^{-2 k}\right)=2 n+1$, by (2). Therefore $A= \pm i^{-n} \sqrt{2 n+1}$. This actually implies that the required product is $\pm \sqrt{2 n+1}$, but it must be positive, since all the sines are, and the result follows. +16. First, we have $P(x)=Q(x) R(x)$ for $Q(x)=x^{m}-|a|^{m} e^{i \theta}$ and $R(x)=$ $x^{m}-|a|^{m} e^{-i \theta}$, where $e^{i \varphi}$ means of course $\cos \varphi+i \sin \varphi$. It remains to factor both $Q$ and $R$. Suppose that $Q(x)=\left(x-q_{1}\right) \cdots\left(x-q_{m}\right)$ and $R(x)=\left(x-r_{1}\right) \cdots\left(x-r_{m}\right)$. +Considering $Q(x)$, we see that $\left|q_{k}^{m}\right|=|a|^{m}$ and also $\left|q_{k}\right|=|a|$ for $k=$ $1, \ldots, m$. Thus we may put $q_{k}=|a| e^{i \beta_{k}}$ and obtain by de Moivre's formula $q_{k}^{m}=|a|^{m} e^{i m \beta_{k}}$. It follows that $m \beta_{k}=\theta+2 j \pi$ for some $j \in \mathbb{Z}$, and we have exactly $m$ possibilities for $\beta_{k}$ modulo $2 \pi$ : $\beta_{k}=\frac{\theta+2(k-1) \pi}{m}$ for $k=1,2, \ldots, m$. +Thus $q_{k}=|a| e^{i \beta_{k}}$; analogously we obtain for $R(x)$ that $r_{k}=|a| e^{-i \beta_{k}}$. Consequently, +$x^{m}-|a|^{m} e^{i \theta}=\prod_{k=1}^{m}\left(x-|a| e^{i \beta_{k}}\right) \quad$ and $\quad x^{m}-|a|^{m} e^{-i \theta}=\prod_{k=1}^{m}\left(x-|a| e^{-i \beta_{k}}\right)$. +Finally, grouping the $k$ th factors of both polynomials, we get + +$$ +\begin{aligned} +P(x) & =\prod_{k=1}^{m}\left(x-|a| e^{i \beta_{k}}\right)\left(x-|a| e^{-i \beta_{k}}\right)=\prod_{k=1}^{m}\left(x^{2}-2|a| x \cos \beta_{k}+a^{2}\right) \\ +& =\prod_{k=1}^{m}\left(x^{2}-2|a| x \cos \frac{\theta+2(k-1) \pi}{m}+a^{2}\right) +\end{aligned} +$$ + +17. Let $f_{1}(x)=a x+b$ and $f_{2}(x)=c x+d$ be two functions from $\mathcal{F}$. We define $g(x)=f_{1} \circ f_{2}(x)=a c x+(a d+b)$ and $h(x)=f_{2} \circ f_{1}(x)=a c x+(b c+d)$. + +By the condition for $\mathcal{F}$, both $g(x)$ and $h(x)$ belong to $\mathcal{F}$. Moreover, there exists $h^{-1}(x)=\frac{x-(b c+d)}{a c}$, and + +$$ +h^{-1} \circ g(x)=\frac{a c x+(a d+b)-(b c+d)}{a c}=x+\frac{(a d+b)-(b c+d)}{a c} +$$ + +belongs to $\mathcal{F}$. Now it follows that we must have $a d+b=b c+d$ for every $f_{1}, f_{2} \in \mathcal{F}$, which is equivalent to $\frac{b}{a-1}=\frac{d}{c-1}=k$. But these formulas exactly describe the fixed points of $f_{1}$ and $f_{2}: f_{1}(x)=a x+b=x \Rightarrow x=$ $\frac{b}{a-1}$. Hence all the functions in $\mathcal{F}$ fix the point $k$. + +### 4.16 Solutions to the Shortlisted Problems of IMO 1974 + +1. Denote by $n$ the number of exams. We have $n(A+B+C)=20+10+9=39$, and since $A, B, C$ are distinct, their sum is at least 6 ; therefore $n=3$ and $A+B+C=13$. +Assume w.l.o.g. that $A>B>C$. Since Betty gained $A$ points in arithmetic, but fewer than 13 points in total, she had $C$ points in both remaining exams (in spelling as well). Furthermore, Carol also gained fewer than 13 points, but with at least $B$ points on two examinations (on which Betty scored $C$ ), including spelling. If she had $A$ in spelling, then she would have at least $A+B+C=13$ points in total, a contradiction. Hence, Carol scored $B$ and placed second in spelling. +Remark. Moreover, it follows that Alice, Betty, and Carol scored $B+A+A$, $A+C+C$, and $C+B+B$ respectively, and that $A=8, B=4, C=1$. +2. We denote by $q_{i}$ the square with side $\frac{1}{i}$. Let us divide the big square into rectangles $r_{i}$ by parallel lines, where the size of $r_{i}$ is $\frac{3}{2} \times \frac{1}{2^{i}}$ for $i=2,3, \ldots$ and $\frac{3}{2} \times 1$ for $i=1$ (this can be done because $1+\sum_{i=2}^{\infty} \frac{1}{2^{i}}=\frac{3}{2}$ ). In rectangle $r_{1}$, one can put the squares $q_{1}, q_{2}, q_{3}$, as is done on the figure. Also, since $\frac{1}{2^{i}}+\cdots+\frac{1}{2^{i+1}-1}<2^{i} \cdot \frac{1}{2^{i}}=1<\frac{3}{2}$, in each $r_{i}, i \geq 2$, one can put $q_{2^{i}}, \ldots, q_{2^{i+1}-1}$. This completes the proof. +![](https://cdn.mathpix.com/cropped/2024_11_18_8e985d6b9c83aa3e9d0eg-404.jpg?height=252&width=254&top_left_y=1103&top_left_x=664) + +Remark. It can be shown that the squares $q_{1}, q_{2}$ cannot fit in any square of side less than $\frac{3}{2}$. +3. For $\operatorname{deg}(P) \leq 2$ the statement is obvious, since $n(P) \leq \operatorname{deg}\left(P^{2}\right)=$ $2 \operatorname{deg}(P) \leq \operatorname{deg}(P)+2$ 。 +Suppose now that $\operatorname{deg}(P) \geq 3$ and $n(P)>\operatorname{deg}(P)+2$. Then there is at least one integer $b$ for which $P(b)=-1$, and at least one $x$ with $P(x)=1$. We may assume w.l.o.g. that $b=0$ (if necessary, we consider the polynomial $P(x+b)$ instead). If $k_{1}, \ldots, k_{m}$ are all integers for which $P\left(k_{i}\right)=1$, then $P(x)=Q(x)\left(x-k_{1}\right) \cdots\left(x-k_{m}\right)+1$ for some polynomial $Q(x)$ with integer coefficients. Setting $x=0$ we obtain $(-1)^{m} Q(0) k_{1} \cdots k_{m}=1-P(0)=2$. It follows that $k_{1} \cdots k_{m} \mid 2$, and hence $m$ is at most 3 . The same holds for the polynomial $-P(x)$, and thus $P(x)=-1$ also has at most 3 integer solutions. This counts for 6 solutions of $P^{2}(x)=1$ in total, implying the statement for $\operatorname{deg}(P) \geq 4$. It remains to verify the statement for $n=3$. If $\operatorname{deg}(P)=3$ and $n(P)=6$, then it follows from the above consideration that $P(x)$ is either $-\left(x^{2}-\right.$ $1)(x-2)+1$ or $\left(x^{2}-1\right)(x+2)+1$. It is directly checked that $n(P)$ equals only 4 in both cases. +4. Assume w.l.o.g. that $a_{1} \leq a_{2} \leq a_{3} \leq a_{4} \leq a_{5}$. If $m$ is the least value of $\left|a_{i}-a_{j}\right|, i \neq j$, then $a_{i+1}-a_{i} \geq m$ for $i=1,2, \ldots, 5$, and consequently $a_{i}-a_{j} \geq(i-j) m$ for any $i, j \in\{1, \ldots, 5\}, i>j$. Then it follows that + +$$ +\sum_{i>j}\left(a_{i}-a_{j}\right)^{2} \geq m^{2} \sum_{i>j}(i-j)^{2}=50 m^{2} +$$ + +On the other hand, by the condition of the problem, + +$$ +\sum_{i>j}\left(a_{i}-a_{j}\right)^{2}=5 \sum_{i=1}^{5} a_{i}^{2}-\left(a_{1}+\cdots+a_{5}\right)^{2} \leq 5 +$$ + +Therefore $50 m^{2} \leq 5$; i.e., $m^{2} \leq \frac{1}{10}$. +5. All the angles are assumed to be oriented and measured modulo $180^{\circ}$. Denote by $\alpha_{i}, \beta_{i}, \gamma_{i}$ the angles of triangle $\triangle_{i}$, at $A_{i}, B_{i}, C_{i}$ respectively. Let us determine the angles of $\triangle_{i+1}$. If $D_{i}$ is the intersection of lines $B_{i} B_{i+1}$ and $C_{i} C_{i+1}$, we have $\angle B_{i+1} A_{i+1} C_{i+1}=\angle D_{i} B_{i} C_{i+1}=\angle B_{i} D_{i} C_{i+1}+$ $\angle D_{i} C_{i+1} B_{i}=\angle B_{i} D_{i} C_{i}-\angle B_{i} C_{i+1} C_{i}=-2 \angle B_{i} A_{i} C_{i}$. We conclude that + +$$ +\alpha_{i+1}=-2 \alpha_{i}, \quad \text { and analogously } \quad \beta_{i+1}=-2 \beta_{i}, \quad \gamma_{i+1}=-2 \gamma_{i} +$$ + +Therefore $\alpha_{r+t}=(-2)^{t} \alpha_{r}$. However, since $(-2)^{12} \equiv 1(\bmod 45)$ and consequently $(-2)^{14} \equiv(-2)^{2}(\bmod 180)$, it follows that $\alpha_{15}=\alpha_{3}$, since all values are modulo $180^{\circ}$. Analogously, $\beta_{15}=\beta_{3}$ and $\gamma_{15}=\gamma_{3}$, and moreover, $\triangle_{3}$ and $\triangle_{15}$ are inscribed in the same circle; hence $\triangle_{3} \cong \triangle_{15}$. +6. We set + +$$ +\begin{aligned} +& x=\sum_{k=0}^{n}\binom{2 n+1}{2 k+1} 2^{3 k}=\frac{1}{\sqrt{8}} \sum_{k=0}^{n}\binom{2 n+1}{2 k+1} \sqrt{8}^{2 k+1}, \\ +& y=\sum_{k=0}^{n}\binom{2 n+1}{2 k} 2^{3 k}=\sum_{k=0}^{n}\binom{2 n+1}{2 k} \sqrt{8}^{2 k} +\end{aligned} +$$ + +Both $x$ and $y$ are positive integers. Also, from the binomial formula we obtain + +$$ +y+x \sqrt{8}=\sum_{i=0}^{2 n+1}\binom{2 n+1}{i} \sqrt{8}^{i}=(1+\sqrt{8})^{2 n+1} +$$ + +and similarly + +$$ +y-x \sqrt{8}=(1-\sqrt{8})^{2 n+1} . +$$ + +Multiplying these equalities, we get $y^{2}-8 x^{2}=(1+\sqrt{8})^{2 n+1}(1-\sqrt{8})^{2 n+1}=$ $-7^{2 n+1}$. Reducing modulo 5 gives us + +$$ +3 x^{2}-y^{2} \equiv 2^{2 n+1} \equiv 2 \cdot(-1)^{n} +$$ + +Now we see that if $x$ is divisible by 5 , then $y^{2} \equiv \pm 2(\bmod 5)$, which is impossible. Therefore $x$ is never divisible by 5 . +Second solution. Another standard way is considering recurrent formulas. If we set + +$$ +x_{m}=\sum_{k}\binom{m}{2 k+1} 8^{k}, \quad y_{m}=\sum_{k}\binom{m}{2 k} 8^{k} +$$ + +then since $\binom{a}{b}=\binom{a-1}{b}+\binom{a-1}{b-1}$, it follows that $x_{m+1}=x_{m}+y_{m}$ and $y_{m+1}=8 x_{m}+y_{m}$; therefore $x_{m+1}=2 x_{m}+7 x_{m-1}$. We need to show that none of $x_{2 n+1}$ are divisible by 5 . Considering the sequence $\left\{x_{m}\right\}$ modulo 5 , we get that $x_{m}=0,1,2,1,1,4,0,3,1,3,3,2,0,4,3,4,4,1, \ldots$ Zeros occur in the initial position of blocks of length 6 , where each subsequent block is obtained by multiplying the previous one by 3 (modulo 5 ). Consequently, $x_{m}$ is divisible by 5 if and only if $m$ is a multiple of 6 , which cannot happen if $m=2 n+1$. +7. Consider an arbitrary prime number $p$. If $p \mid m$, then there exists $b_{i}$ that is divisible by the same power of $p$ as $m$. Then $p$ divides neither $a_{i} \frac{m}{b_{i}}$ nor $a_{i}$, because $\left(a_{i}, b_{i}\right)=1$. If otherwise $p \nmid m$, then $\frac{m}{b_{i}}$ is not divisible by $p$ for any $i$, hence $p$ divides $a_{i}$ and $a_{i} \frac{m}{b_{i}}$ to the same power. Therefore $\left(a_{1}, \ldots, a_{k}\right)$ and $\left(a_{1} \frac{m}{b_{1}}, \ldots, a_{k} \frac{m}{b_{k}}\right)$ have the same factorization; hence they are equal. Second solution. For $k=2$ we easily verify the formula $\left(m \frac{a_{1}}{b_{1}}, m \frac{a_{2}}{b_{2}}\right)=$ $\frac{m}{b_{1} b_{2}}\left(a_{1} b_{2}, a_{2} b_{1}\right)=\frac{1}{b_{1} b_{2}}\left[b_{1}, b_{2}\right]\left(a_{1}, a_{2}\right)\left(b_{1}, b_{2}\right)=\left(a_{1}, a_{2}\right)$, since $\left[b_{1}, b_{2}\right]$. $\left(b_{1}, b_{2}\right)=b_{1} b_{2}$. We proceed by induction: + +$$ +\begin{aligned} +\left(a_{1} \frac{m}{b_{1}}, \ldots, a_{k} \frac{m}{b_{k}}, a_{k+1} \frac{m}{b_{k+1}}\right) & =\left(\frac{m}{\left[b_{1}, \ldots, b_{k}\right]}\left(a_{1}, \ldots, a_{k}\right), a_{k+1} \frac{m}{b_{k+1}}\right) \\ +& =\left(a_{1}, \ldots, a_{k}, a_{k+1}\right) . +\end{aligned} +$$ + +8. It is clear that + +$$ +\begin{gathered} +\frac{a}{a+b+c+d}+\frac{b}{a+b+c+d}+\frac{c}{a+b+c+d}+\frac{d}{a+b+c+d}2 a_{n}$ for all $n$. For $n=1$ the claim is trivial. If it holds for $i \leq n$, then $a_{i} \leq 2^{i-n} a_{n}$; thus we obtain from (1) + +$$ +a_{n+1}>a_{n}\left(3-\frac{1}{2}-\frac{1}{2^{2}}-\cdots-\frac{1}{2^{n}}\right)>2 a_{n} +$$ + +Therefore $a_{n} \geq 2^{n}$ for all $n$ (moreover, one can show from (1) that $a_{n} \geq$ $(n+2) 2^{n-1}$ ); hence there exist good words of length $n$. +Remark. If there are two nonallowed words (instead of one) of each length greater than 1, the statement of the problem need not remain true. + +### 4.17 Solutions to the Shortlisted Problems of IMO 1975 + +1. First, we observe that there cannot exist three routes of the form $(A, B, C)$, $(A, B, D),(A, C, D)$, for if $E, F$ are the remaining two ports, there can be only one route covering $A, E$, namely, $(A, E, F)$. Thus if $(A, B, C)$, $(A, B, D)$ are two routes, the one covering $A, C$ must be w.l.o.g. $(A, C, E)$. The other roots are uniquely determined: These are $(A, D, F),(A, E, F)$, $(B, D, E),(B, E, F),(B, C, F),(C, D, E),(C, D, F)$. +2. Since there are finitely many arrangements of the $z_{i}$ 's, assume that $z_{1}, \ldots, z_{n}$ is the one for which $\sum_{i=1}^{n}\left(x_{i}-z_{i}\right)^{2}$ is minimal. We claim that in this case $i\Delta a_{n+1}$. Suppose that for some $n, \Delta a_{n}<0$ : Then for each $k \geq n, \Delta a_{k}<\Delta a_{n}$; hence $a_{n}-a_{n+m}=\Delta a_{n}+\cdots+\Delta a_{n+m-1}0$, or equivalently, let the graph of $f_{a}$ lie below the graph of $f$. In this case also $f(2 a)>f(a)$, since otherwise, the graphs of $f$ and $f_{a}$ would intersect between $a$ and $2 a$. Continuing in this way we are led to $0=f(0)<$ $f(a)f(1-a)>f(1-2 a)>\cdots>f(1-n a)$. Choosing values of $f$ at $i a, 1-i a, i=1, \ldots, n$, so that they satisfy $f(1-n a)<\cdots1, a_{k_{i}} \equiv a_{k_{1}}\left(\bmod a_{1}\right)$; hence $a_{k_{i}}=a_{k_{1}}+y a_{1}$ for some integer $y>0$. It follows that for every $r=0,1, \ldots, a_{1}-1$ there is exactly one member of the corresponding $\left(a_{k_{i}}\right)_{i \geq 1}$ that cannot be represented as $x a_{l}+y a_{m}$, and hence at most $a_{1}+1$ members of $\left(a_{k}\right)$ in total are not representable in the given form. +12. Since $\sin 2 x_{i}=2 \sin x_{i} \cos x_{i}$ and $\sin \left(x_{i}+x_{i+1}\right)+\sin \left(x_{i}-x_{i+1}\right)=$ $2 \sin x_{i} \cos x_{i+1}$, the inequality from the problem is equivalent to + +$$ +\begin{gathered} +\left(\cos x_{1}-\cos x_{2}\right) \sin x_{1}+\left(\cos x_{2}-\cos x_{3}\right) \sin x_{2}+\cdots \\ +\cdots+\left(\cos x_{\nu-1}-\cos x_{\nu}\right) \sin x_{\nu-1}<\frac{\pi}{4} +\end{gathered} +$$ + +Consider the unit circle with center at $O(0,0)$ and points $M_{i}\left(\cos x_{i}, \sin x_{i}\right)$ on it. Also, choose the points $N_{i}\left(\cos x_{i}, 0\right)$ and $M_{i}^{\prime}\left(\cos x_{i+1}, \sin x_{i}\right)$. It is clear that $\left(\cos x_{i}-\cos x_{i+1}\right) \sin x_{i}$ is equal to the area of the rectangle $M_{i} N_{i} N_{i+1} M_{i}^{\prime}$. Since all these rectangles are disjoint and lie inside the quarter circle in the first quadrant whose area is $\frac{\pi}{4}$, inequality (1) follows. +13. Suppose that $A_{k} A_{k+1} \cap A_{m} A_{m+1} \neq \emptyset$ for some $k, m>k+1$. Without loss of generality we may suppose that $k=0, m=n-1$ and that no two segments $A_{k} A_{k+1}$ and $A_{m} A_{m+1}$ intersect for $0 \leq kA_{0} A_{1}$, hence $\angle A_{0} A_{1} A_{2}>$ $\angle A_{1} A_{2} A_{3}$, a contradiction. +Let $n=4$. From $A_{3} A_{2}>A_{1} A_{2}$ we conclude that $\angle A_{3} A_{1} A_{2}>\angle A_{1} A_{3} A_{2}$. Using the inequality $\angle A_{0} A_{3} A_{2}>\angle A_{0} A_{1} A_{2}$ we obtain that $\angle A_{0} A_{3} A_{1}>$ $\angle A_{0} A_{1} A_{3}$ implying $A_{0} A_{1}>A_{0} A_{3}$. Now we have $A_{2} A_{3}\cdots>\alpha_{n-1}$; hence $\alpha_{n-1}<\frac{360^{\circ}}{n-1} \leq 90^{\circ}$. Consequently $\angle A_{n-2} A_{n-1} A_{0} \geq 90^{\circ}$ and $A_{0} A_{n-2}>A_{n-1} A_{n-2}$. On the other hand, $A_{0} A_{n-2}2$. Adding these up we obtain $x_{n}^{2} \geq x_{0}^{2}+2 n$, which proves the first inequality. +On the other hand, $x_{k+1}=x_{k}+\frac{1}{x_{k}} \leq x_{k}+0.2$ (for $x_{k} \geq 5$ ), and one also deduces from (1) that $x_{k+1}^{2}-x_{k}^{2}-0.2\left(x_{k+1}-x_{k}\right)=\left(x_{k+1}+x_{k}-\right.$ $0.2)\left(x_{k+1}-x_{k}\right) \leq 2$. Again, adding these inequalities up, $(k=0, \ldots, n-1)$ yields + +$$ +x_{n}^{2} \leq 2 n+x_{0}^{2}+0.2\left(x_{n}-x_{0}\right)=2 n+24+0.2 x_{n} +$$ + +Solving the corresponding quadratic equation, we obtain $x_{n}<0.1+$ $\sqrt{2 n+24.01}<0.1++\sqrt{2 n+25}$. +15. Assume that the center of the circle is at the origin $O(0,0)$, and that the points $A_{1}, A_{2}, \ldots, A_{1975}$ are arranged on the upper half-circle so that $\angle A_{i} O A_{1}=\alpha_{i}\left(\alpha_{1}=0\right)$. The distance $A_{i} A_{j}$ equals $2 \sin \frac{\alpha_{j}-\alpha_{i}}{2}=$ $2 \sin \frac{\alpha_{j}}{2} \cos \frac{\alpha_{i}}{2}-\cos \frac{\alpha_{j}}{2} \sin \frac{\alpha_{i}}{2}$, and it will be rational if all $\sin \frac{\alpha_{k}}{2}, \cos \frac{\alpha_{k}}{2}$ are rational. +Finally, observe that there exist infinitely many angles $\alpha$ such that both $\sin \alpha, \cos \alpha$ are rational, and that such $\alpha$ can be arbitrarily small. For example, take $\alpha$ so that $\sin \alpha=\frac{2 t}{t^{2}+1}$ and $\cos \alpha=\frac{t^{2}-1}{t^{2}+1}$ for any $t \in \mathbb{Q}$. + +### 4.18 Solutions to the Shortlisted Problems of IMO 1976 + +1. Let $r$ denote the common inradius. Some two of the four triangles with the inradii $\rho$ have cross angles at $M$ : Suppose these are $\triangle A M B_{1}$ and $\triangle B M A_{1}$. We shall show that $\triangle A M B_{1} \cong \triangle B M A_{1}$. Indeed, the altitudes of these two triangles are both equal to $r$, the inradius of $\triangle A B C$, and their interior angles at $M$ are equal to some angle $\varphi$. If $P$ is the point of tangency of the incircle of $\triangle A_{1} M B$ with $M B$, then $\frac{r}{\rho}=\frac{A_{1} M+B M+A_{1} B}{A_{1} B}$, which also implies $\frac{r-2 \rho}{\rho}=\frac{A_{1} M+B M-A_{1} B}{A_{1} B}=\frac{2 M P}{A_{1} B}=\frac{2 r \cot (\varphi / 2)}{A_{1} B}$. Since similarly $\frac{r-2 \rho}{\rho}=\frac{2 r \cot (\varphi / 2)}{B_{1} A}$, we obtain $A_{1} B=B_{1} A$ and consequently $\triangle A M B_{1} \cong \triangle B M A_{1}$. Thus $\angle B A C=\angle A B C$ and $C C_{1} \perp A B$. There are two alternatives for the other two incircles: +(i) If the inradii of $A M C_{1}$ and $A M B_{1}$ are equal to $r$, it is easy to obtain that $\triangle A M C_{1} \cong \triangle A M B_{1}$. Hence $\angle A B_{1} M=\angle A C_{1} M=90^{\circ}$, and $\triangle A B C$ is equilateral. +(ii) The inradii of $A M B_{1}$ and $C M B_{1}$ are equal to $r$. Put $x=\angle M A C_{1}=$ $\angle M B C_{1}$. In this case $\varphi=2 x$ and $\angle B_{1} M C=90^{\circ}-x$. Now we have $\frac{A B_{1}}{C B_{1}}=\frac{S_{A M B_{1}}}{S_{C M B_{1}}}=\frac{A M+M B_{1}+A B_{1}}{C M+M B_{1}+C B_{1}}=\frac{A M+M B_{1}-A B_{1}}{C M+M B_{1}-C B_{1}}=\frac{\mathrm{cot} x}{\cot \left(45^{\circ}-x / 2\right)}$. On the other hand, $\frac{A B_{1}}{C B_{1}}=\frac{A B}{B C}=2 \cos 2 x$. Thus we have an equation for $x: \tan \left(45^{\circ}-x / 2\right)=2 \cos 2 x \tan x$, or equivalently + +$$ +2 \tan \left(45^{\circ}-\frac{x}{2}\right) \sin \left(45^{\circ}-\frac{x}{2}\right) \cos \left(45^{\circ}-\frac{x}{2}\right)=2 \cos 2 x \sin x . +$$ + +Hence $\sin 3 x-\sin x=2 \sin ^{2}\left(45^{\circ}-\frac{x}{2}\right)=1-\sin x$, implying $\sin 3 x=1$, i.e., $x=30^{\circ}$. Therefore $\triangle A B C$ is equilateral. +2. Let us put $b_{i}=i(n+1-i) / 2$, and let $c_{i}=a_{i}-b_{i}, i=0,1, \ldots, n+1$. It is easy to verify that $b_{0}=b_{n+1}=0$ and $b_{i-1}-2 b_{i}+b_{i+1}=-1$. Subtracting this inequality from $a_{i-1}-2 a_{i}+a_{i+1} \geq-1$, we obtain $c_{i-1}-2 c_{i}+c_{i+1} \geq 0$, i.e., $2 c_{i} \leq c_{i-1}+c_{i+1}$. We also have $c_{0}=c_{n+1}=0$. + +Suppose that there exists $i \in\{1, \ldots, n\}$ for which $c_{i}>0$, and let $c_{k}$ be the maximal such $c_{i}$. Assuming w.l.o.g. that $c_{k-1}2$, then $a / b \leq 5 / 3$, and if $a>5$, then $a / b \leq 3 / 2$. If $a_{1}>2$, then $\frac{a_{1}}{b_{1}} \cdot \frac{a_{2}}{b_{2}} \cdot \frac{a_{3}}{b_{3}}<(5 / 3)^{3}<5$, a contradiction. Hence $a_{1}=2$. If also $a_{2}=2$, then $a_{3} / b_{3}=5 / 4 \leq \sqrt[3]{2}$, which is impossible. Also, if $a_{2} \geq 6$, then $\frac{a_{2}}{b_{2}} \cdot \frac{a_{3}}{b_{3}} \leq(1.5)^{2}<2.5$, again a contradiction. We thus have the following cases: +(i) $a_{1}=2, a_{2}=3$, then $a_{3} / b_{3}=5 / 3$, which holds only if $a_{3}=5$; +(ii) $a_{1}=2, a_{2}=4$, then $a_{3} / b_{3}=15 / 8$, which is impossible; +(iii) $a_{1}=2, a_{2}=5$, then $a_{3} / b_{3}=3 / 2$, which holds only if $a_{3}=6$. + +The only possible sizes of the box are therefore $(2,3,5)$ and $(2,5,6)$. +7. The map $T$ transforms the interval $(0, a]$ onto $(1-a, 1]$ and the interval $(a, 1]$ onto $(0,1-a]$. Clearly $T$ preserves the measure. Since the measure of the interval $[0,1]$ is finite, there exist two positive integers $k, l>k$ such that $T^{k}(J)$ and $T^{l}(J)$ are not disjoint. But the map $T$ is bijective; hence $T^{l-k}(J)$ and $J$ are not disjoint. +8. Every polynomial with real coefficients can be factored as a product of linear and quadratic polynomials with real coefficients. Thus it suffices to prove the result only for a quadratic polynomial $P(x)=x^{2}-2 a x+b^{2}$, with $a>0$ and $b^{2}>a^{2}$. +Using the identity + +$$ +\left(x^{2}+b^{2}\right)^{2 n}-(2 a x)^{2 n}=\left(x^{2}-2 a x+b^{2}\right) \sum_{k=0}^{2 n-1}\left(x^{2}+b^{2}\right)^{k}(2 a x)^{2 n-k-1} +$$ + +we have solved the problem if we can choose $n$ such that $b^{2 n}\binom{2 n}{n}>2^{2 n} a^{2 n}$. However, it is is easy to show that $2 n\binom{2 n}{n}<2^{2 n}$; hence it is enough to take $n$ such that $(b / a)^{2 n}>2 n$. Since $\lim _{n \rightarrow \infty}(2 n)^{1 /(2 n)}=12$, then by simple induction $P_{n}(x)>x$ for all $n$. Similarly, if $x<-1$, then $P_{1}(x)>2$, which implies $P_{n}(x)>2$ for all $n$. It follows that all real roots of the equation $P_{n}(x)=x$ lie in the interval $[-2,2]$, and thus have the form $x=2 \cos t$. +Now we observe that $P_{1}(2 \cos t)=4 \cos ^{2} t-2=2 \cos 2 t$, and in general $P_{n}(2 \cos t)=2 \cos 2^{n} t$. Our equation becomes + +$$ +\cos 2^{n} t=\cos t +$$ + +which indeed has $2^{n}$ different solutions $t=\frac{2 \pi m}{2^{n}-1}\left(m=0,1, \ldots, 2^{n-1}-1\right)$ and $t=\frac{2 \pi m}{2^{n}+1}\left(m=1,2, \ldots, 2^{n-1}\right)$. +10. Let $a_{1}a_{i}$. Since $4=2^{2}$, we may assume that all $a_{i}$ are either 2 or 3 , and $M=2^{k} 3^{l}$, where $2 k+3 l=1976$. +(4) $k \geq 3$ does not yield the maximal value, since $2 \cdot 2 \cdot 2<3 \cdot 3$. + +Hence $k \leq 2$ and $2 k \equiv 1976(\bmod 3)$ gives us $k=1, l=658$ and $M=2 \cdot 3^{658}$. +11. We shall show by induction that $5^{2^{k}}-1=2^{k+2} q_{k}$ for each $k=0,1, \ldots$, where $q_{k} \in \mathbb{N}$. Indeed, the statement is true for $k=0$, and if it holds for some $k$ then $5^{2^{k+1}}-1=\left(5^{2^{k}}+1\right)\left(5^{2^{k}}-1\right)=2^{k+3} d_{k+1}$ where $d_{k+1}=$ $\left(5^{2^{k}}+1\right) d_{k} / 2$ is an integer by the inductive hypothesis. +Let us now choose $n=2^{k}+k+2$. We have $5^{n}=10^{k+2} q_{k}+5^{k+2}$. It follows from $5^{4}<10^{3}$ that $5^{k+2}$ has at most $[3(k+2) / 4]+2$ nonzero digits, while $10^{k+2} q_{k}$ ends in $k+2$ zeros. Hence the decimal representation of $5^{n}$ contains at least $[(k+2) / 4]-2$ consecutive zeros. Now it suffices to take $k>4 \cdot 1978$. +12. Suppose the decomposition into $k$ polynomials is possible. The sum of coefficients of each polynomial $a_{1} x+a_{2} x^{2}+\cdots+a_{n} x^{n}$ equals $1+\cdots+$ $n=n(n+1) / 2$ while the sum of coefficients of $1976\left(x+x^{2}+\cdots+x^{n}\right)$ is $1976 n$. Hence we must have $1976 n=k n(n+1) / 2$, which reduces to $(n+1) \mid 3952=2^{4} \cdot 13 \cdot 19$. In other words, $n$ is of the form $n=2^{\alpha} 13^{\beta} 19^{\gamma}-1$, with $0 \leq \alpha \leq 4,0 \leq \beta \leq 1,0 \leq \gamma \leq 1$. We can immediately eliminate the values $n=0$ and $n=3951$ that correspond to $\alpha=\beta=\gamma=0$ and $\alpha=4, \beta=\gamma=1$. +We claim that all other values $n$ are permitted. There are two cases. +$\alpha \leq 3$. In this case $k=3952 /(n+1)$ is even. The simple choice of the polynomials $P=x+2 x^{2}+\cdots+n x^{n}$ and $P^{\prime}=n x+(n-1) x^{2}+\cdots+x^{n}$ suffices, since $k\left(P+P^{\prime}\right) / 2=1976\left(x+x^{2}+\cdots+x^{n}\right)$. +$\alpha=4$. Then $k$ is odd. Consider $(k-3) / 2$ pairs $\left(P, P^{\prime}\right)$ of the former case and + +$$ +\begin{aligned} +P_{1}= & {\left[n x+(n-1) x^{3}+\cdots+\frac{n+1}{2} x^{n}\right] } \\ +& +\left[\frac{n-1}{2} x^{2}+\frac{n-3}{2} x^{4}+\cdots+x^{n-1}\right] \\ +P_{2}= & {\left[\frac{n+1}{2} x+\frac{n-1}{2} x^{3}+\cdots+x^{n}\right] } \\ +& +\left[n x^{2}+(n-1) x^{4}+\cdots+\frac{n+3}{2} x^{n-1}\right] +\end{aligned} +$$ + +Then $P+P_{1}+P_{2}=3(n+1)\left(x+x^{2}+\cdots+x^{n}\right) / 2$ and therefore $(k-3)\left(P+P^{\prime}\right) / 2+\left(P+P_{1}+P_{2}\right)=1976\left(x+x^{2}+\cdots+x^{n}\right)$. +It follows that the desired decomposition is possible if and only if $1C S$ is equivalent to $B P>C P$. The conditions of the problem imply that $P A>P B$ and $P A>P E$. The locus of such points $P$ is the region of the plane that is determined by the perpendicular bisectors of segments $A B$ and $A E$ and that contains the point diametrically opposite $A$. But since $A Bf(n)$ whenever $x>n$. The case $n=0$ is trivial. Suppose that $n \geq 1$ and that $x>k$ implies $f(x)>f(k)$ for all $kn$, then $m-1 \geq n$ and consequently $f(m-1) \geq n$. But in this case the inequality $f(m)>$ $f(f(m-1))$ contradicts the minimality property of $m$. The inductive proof is thus completed. +It follows that $f$ is strictly increasing, so $f(n+1)>f(f(n))$ implies that $n+1>f(n)$. But since $f(n) \geq n$ we must have $f(n)=n$. +3. Let $v_{1}, v_{2}, \ldots, v_{k}$ be $k$ persons who are not acquainted with each other. Let us denote by $m$ the number of acquainted couples and by $d_{j}$ the number of acquaintances of person $v_{j}$. Then +$m \leq d_{k+1}+d_{k+2}+\cdots+d_{n} \leq d(n-k) \leq k(n-k) \leq\left(\frac{k+(n-k)}{2}\right)^{2}=\frac{n^{2}}{4}$. +4. Consider any vertex $v_{n}$ from which the maximal number $d$ of segments start, and suppose it is not a vertex of a triangle. Let $\mathcal{A}=$ $\left\{v_{1}, v_{2}, \ldots, v_{d}\right\}$ be the set of points that are connected to $v_{n}$, and let $\mathcal{B}=\left\{v_{d+1}, v_{d+2}, \ldots, v_{n}\right\}$ be the set of the other points. Since $v_{n}$ is not a vertex of a triangle, there is no segment both of whose vertices lie in $\mathcal{A}$; i.e., each segment has an end in $\mathcal{B}$. Thus, if $d_{j}$ denotes the number of segments at $v_{j}$ and $m$ denotes the total number of segments, we have + +$$ +m \leq d_{d+1}+d_{d+2}+\cdots+d_{n} \leq d(n-d) \leq\left[\frac{n^{2}}{4}\right]=m +$$ + +This means that each inequality must be equality, implying that each point in $\mathcal{B}$ is a vertex of $d$ segments, and each of these segments has the other end in $\mathcal{A}$. Then there is no triangle at all, which is a contradiction. +5. Let us denote by $I$ and $E$ the sets of interior boundary points and exterior boundary points. Let $A B C D$ be the square inscribed in the circle $k$ with sides parallel to the coordinate axes. Lines $A B, B C, C D, D A$ divide the +plane into 9 regions: $\mathcal{R}, \mathcal{R}_{A}, \mathcal{R}_{B}$, $\mathcal{R}_{C}, \mathcal{R}_{D}, \mathcal{R}_{A B}, \mathcal{R}_{B C}, \mathcal{R}_{C D}, \mathcal{R}_{D A}$. There is a unique pair of lattice points $A_{I} \in \mathcal{R}, A_{E} \in$ $\mathcal{R}_{A}$ that are opposite vertices of a unit square. We similarly define $B_{I}, C_{I}, D_{I}, B_{E}, C_{E}, D_{E}$. Let us form a graph $G$ by connecting each point from $E$ lying in $\mathcal{R}_{A B}$ (respectively $\mathcal{R}_{B C}, \mathcal{R}_{C D}, \mathcal{R}_{D A}$ ) to its up- +![](https://cdn.mathpix.com/cropped/2024_11_18_8e985d6b9c83aa3e9d0eg-420.jpg?height=389&width=505&top_left_y=271&top_left_x=825) +per (respectively left, lower, right) neighbor point (which clearly belongs to $I$ ). It is easy to see that: +(i) All vertices from $I$ other than $A_{I}, B_{I}, C_{I}, D_{I}$ have degree 1. +(ii) $A_{E}$ is not in $E$ if and only if $A_{I} \in I$ and $\operatorname{deg} A_{I}=2$. +(iii) No other lattice points inside $\mathcal{R}_{A}$ belong to $E$. + +Thus if $m$ is the number of edges of the graph $G$ and $s$ is the number of points among $A_{E}, B_{E}, C_{E}$, and $D_{E}$ that are in $E$, using (i)-(iii) we easily obtain $|E|=m+s$ and $|I|=m-(4-s)=|E|+4$. +6. Let $\langle y\rangle$ denote the distance from $y \in \mathbb{R}$ to the closest even integer. We claim that + +$$ +\langle 1+\cos x\rangle \leq \sin x \quad \text { for all } x \in[0, \pi] +$$ + +Indeed, if $\cos x \geq 0$, then $\langle 1+\cos x\rangle=1-\cos x \leq 1-\cos ^{2} x=\sin ^{2} x \leq$ $\sin x$; the proof is similar if $\cos x<0$. +We note that $\langle x+y\rangle \leq\langle x\rangle+\langle y\rangle$ holds for all $x, y \in \mathbb{R}$. Therefore + +$$ +\sum_{j=1}^{n} \sin x_{j} \geq \sum_{j=1}^{n}\left\langle 1+\cos x_{j}\right\rangle \geq\left\langle\sum_{j=1}^{n}\left(1+\cos x_{j}\right)\right\rangle=1 +$$ + +7. Let us suppose that $c_{1} \leq c_{2} \leq \cdots \leq c_{n}$ and that $c_{1}<01977-90=1887$ it follows that $q>43$; hence $q=44$ and $r=41$. It remains to find positive integers $a$ and $b$ satisfying $a^{2}+b^{2}=44(a+b)+41$, or equivalently + +$$ +(a-22)^{2}+(b-22)^{2}=1009 +$$ + +The only solutions to this Diophantine equation are $(|a-22|,|b-22|) \in$ $\{(15,28),(28,15)\}$, which yield $(a, b) \in\{(7,50),(37,50),(50,7),(50,37)\}$. +11. (a) Suppose to the contrary that none of the numbers $z_{0}, z_{1}, \ldots, z_{n-1}$ is divisible by $n$. Then two of these numbers, say $z_{k}$ and $z_{l}(0 \leq k90^{\circ}$ ). Let $h^{\prime}$ and $\overline{h^{\prime}}$ be the two semicircles with diameter $A P$, where $M \in h^{\prime}$. Since $\overline{h^{\prime}}$ contains a point $C$ such that $B C>A B$, it cannot be contained in $\Phi$, implying that $h^{\prime} \subset \Phi$. Hence $M$ belongs to $\Phi$. Since $\Phi$ contains no points outside the circle $k$, it must coincide with the disk determined by $k$. On the other hand, any disk has the required property. +14. We prove by induction on $n$ that independently of the word $w_{0}$, the given algorithm generates all words of length $n$. This is clear for $n=1$. Suppose now the statement is true for $n-1$, and that we are given a word $w_{0}=$ +$c_{1} c_{2} \ldots c_{n}$ of length $n$. Obviously, the words $w_{0}, w_{1}, \ldots, w_{2^{n-1}-1}$ all have the $n$th digit $c_{n}$, and by the inductive hypothesis these are all words whose $n$th digit is $c_{n}$. Similarly, by the inductive hypothesis $w_{2^{n-1}}, \ldots, w_{2^{n-1}}$ are all words whose $n$th digit is $1-c_{n}$, and the induction is complete. +15. Each segment is an edge of at most two squares and a diagonal of at most one square. Therefore $p_{k}=0$ for $k>3$, and we have to prove that + +$$ +p_{0}=p_{2}+2 p_{3} +$$ + +Let us calculate the number $q(n)$ of considered squares. Each of these squares is inscribed in a square with integer vertices and sides parallel to the coordinate axes. There are $(n-s)^{2}$ squares of side $s$ with integer vertices and sides parallel to the coordinate axes, and each of them circumscribes exactly $s$ of the considered squares. It follows that $q(n)=\sum_{s=1}^{n-1}(n-s)^{2} s=n^{2}\left(n^{2}-1\right) / 12$. Computing the number of edges and diagonals of the considered squares in two ways, we obtain that + +$$ +p_{1}+2 p_{2}+3 p_{3}=6 q(n) +$$ + +On the other hand, the total number of segments with endpoints in the considered integer points is given by + +$$ +p_{0}+p_{1}+p_{2}+p_{3}=\binom{n^{2}}{2}=\frac{n^{2}\left(n^{2}-1\right)}{2}=6 q(n) . +$$ + +Now (1) follows immediately from (2) and (3). +16. For $i=k$ and $j=l$ the system is reduced to $1 \leq i, j \leq n$, and has exactly $n^{2}$ solutions. Let us assume that $i \neq k$ or $j \neq l$. The points $A(i, j), B(k, l)$, $C(-j+k+l, i-k+l), D(i-j+l, i+j-k)$ are vertices of a negatively oriented square with integer vertices lying inside the square $[1, n] \times[1, n]$, and each of these squares corresponds to exactly 4 solutions to the system. By the previous problem there are exactly $q(n)=n^{2}\left(n^{2}-1\right) / 12$ such squares. Hence the number of solutions is equal to $n^{2}+4 q(n)=n^{2}\left(n^{2}+2\right) / 3$. +17. Centers of the balls that are tangent to $K$ are vertices of a regular polyhedron with triangular faces, with edge length $2 R$ and radius of circumscribed sphere $r+R$. Therefore the number $n$ of these balls is 4,6 , or 20 . It is straightforward to obtain that: +(i) If $n=4$, then $r+R=2 R(\sqrt{6} / 4)$, whence $R=r(2+\sqrt{6})$. +(ii) If $n=6$, then $r+R=2 R(\sqrt{2} / 2)$, whence $R=r(1+\sqrt{2})$. +(iii) If $n=20$, then $r+R=2 R \sqrt{5+\sqrt{5}} / 8$, whence $R=r[\sqrt{5-2 \sqrt{5}}+$ + +$$ +(3-\sqrt{5}) / 2] +$$ + +18. Let $U$ be the midpoint of the segment $A B$. The point $M$ belongs to $C U$ and $C M=(\sqrt{5}-1) C U / 2, r=C U \sqrt{\sqrt{5}-2}$. +19. We shall prove the statement by induction on $m$. For $m=2$ it is trivial, since each power of 5 greater than 5 ends in 25 . Suppose that the statement is true for some $m \geq 2$, and that the last $m$ digits of $5^{n}$ alternate in parity. It can be shown by induction that the maximum power of 2 that divides $5^{2^{m-2}}-1$ is $2^{m}$, and consequently the difference $5^{n+2^{m-2}}-5^{n}$ is divisible by $10^{m}$ but not by $2 \cdot 10^{m}$. It follows that the last $m$ digits of the numbers $5^{n+2^{m-2}}$ and $5^{n}$ coincide, but the digits at the position $m+1$ have opposite parity. Hence the last $m+1$ digits of one of these two powers of 5 alternate in parity. The inductive proof is completed. +20. There exist $u, v$ such that $a \cos x+b \sin x=r \cos (x-u)$ and $A \cos 2 x+$ $B \sin 2 x=R \cos 2(x-v)$, where $r=\sqrt{a^{2}+b^{2}}$ and $R=\sqrt{A^{2}+B^{2}}$. Then $1-f(x)=r \cos (x-u)+R \cos 2(x-v) \leq 1$ holds for all $x \in \mathbb{R}$. +There exists $x \in \mathbb{R}$ such that $\cos (x-u) \geq 0$ and $\cos 2(x-v)=1$ (indeed, either $x=v$ or $x=v+\pi$ works). It follows that $R \leq 1$. Similarly, there exists $x \in \mathbb{R}$ such that $\cos (x-u)=1 / \sqrt{2}$ and $\cos 2(x-v) \geq 0$ (either $x=u-\pi / 4$ or $x=u+\pi / 4$ works). It follows that $r \leq \sqrt{2}$. +Remark. The proposition of this problem contained as an addendum the following, more difficult, inequality: + +$$ +\sqrt{a^{2}+b^{2}}+\sqrt{A^{2}+B^{2}} \leq 2 . +$$ + +The proof follows from the existence of $x \in \mathbb{R}$ such that $\cos (x-u) \geq 1 / 2$ and $\cos 2(x-v) \geq 1 / 2$. +21. Let us consider the vectors $v_{1}=\left(x_{1}, x_{2}, x_{3}\right), v_{2}=\left(y_{1}, y_{2}, y_{3}\right), v_{3}=(1,1,1)$ in space. The given equalities express the condition that these three vectors are mutually perpendicular. Also, $\frac{x_{1}^{2}}{x_{1}^{2}+x_{2}^{2}+x_{3}^{2}}, \frac{y_{1}^{2}}{y_{1}^{2}+y_{2}^{2}+y_{3}^{2}}$, and $1 / 3$ are the squares of the projections of the vector $(1,0,0)$ onto the directions of $v_{1}, v_{2}, v_{3}$, respectively. The result follows from the fact that the sum of squares of projections of a unit vector on three mutually perpendicular directions is 1. +22. Since the quadrilateral $O A_{1} B B_{1}$ is cyclic, $\angle O A_{1} B_{1}=\angle O B C$. By using the analogous equalities we obtain $\angle O A_{4} B_{4}=\angle O B_{3} C_{3}=\angle O C_{2} D_{2}=$ $\angle O D_{1} A_{1}=\angle O A B$, and similarly $\angle O B_{4} A_{4}=\angle O B A$. Hence $\triangle O A_{4} B_{4} \sim$ $\triangle O A B$. Analogously, we have for the other three pairs of triangles $\triangle O B_{4} C_{4} \sim \triangle O B C, \triangle O C_{4} D_{4} \sim \triangle O C D, \triangle O D_{4} A_{4} \sim \triangle O D A$, and consequently $A B C D \sim A_{4} B_{4} C_{4} D_{4}$. +23. Every polynomial $q\left(x_{1}, \ldots, x_{n}\right)$ with integer coefficients can be expressed in the form $q=r_{1}+x_{1} r_{2}$, where $r_{1}, r_{2}$ are polynomials in $x_{1}, \ldots, x_{n}$ with integer coefficients in which the variable $x_{1}$ occurs only with even exponents. Thus if $q_{1}=r_{1}-x_{1} r_{2}$, the polynomial $q q_{1}=r_{1}^{2}-x_{1}^{2} r_{2}^{2}$ contains $x_{1}$ only with even exponents. We can continue inductively constructing polynomials $q_{j}, j=2,3, \ldots, n$, such that $q q_{1} q_{2} \cdots q_{j}$ contains each of +variables $x_{1}, x_{2}, \ldots, x_{j}$ only with even exponents. Thus the polynomial $q q_{1} \cdots q_{n}$ is a polynomial in $x_{1}^{2}, \ldots, x_{n}^{2}$. +The polynomials $f$ and $g$ exist for every $n \in \mathbb{N}$. In fact, it suffices to construct $q_{1}, \ldots, q_{n}$ for the polynomial $q=x_{1}+\cdots+x_{n}$ and take $f=$ $q_{1} q_{2} \cdots q_{n}$. +24. Setting $x=y=0$ gives us $f(0)=0$. Let us put $g(x)=\arctan f(x)$. The given functional equation becomes $\tan g(x+y)=\tan (g(x)+g(y))$; hence + +$$ +g(x+y)=g(x)+g(y)+k(x, y) \pi +$$ + +where $k(x, y)$ is an integer function. But $k(x, y)$ is continuous and $k(0,0)=$ 0 , therefore $k(x, y)=0$. Thus we obtain the classical Cauchy's functional equation $g(x+y)=g(x)+g(y)$ on the interval $(-1,1)$, all of whose continuous solutions are of the form $g(x)=a x$ for some real $a$. Moreover, $g(x) \in(-\pi, \pi)$ implies $|a| \leq \pi / 2$. +Therefore $f(x)=\tan a x$ for some $|a| \leq \pi / 2$, and this is indeed a solution to the given equation. +25. Let + +$$ +f_{n}(z)=z^{n}+a \sum_{k=1}^{n}\binom{n}{k}(a-k b)^{k-1}(z+k b)^{n-k} . +$$ + +We shall prove by induction on $n$ that $f_{n}(z)=(z+a)^{n}$. This is trivial for $n=1$. Suppose that the statement is true for some positive integer $n-1$. Then + +$$ +\begin{aligned} +f_{n}^{\prime}(z) & =n z^{n-1}+a \sum_{k=1}^{n-1}\binom{n}{k}(n-k)(a-k b)^{k-1}(z+k b)^{n-k-1} \\ +& =n z^{n-1}+n a \sum_{k=1}^{n-1}\binom{n-1}{k}(a-k b)^{k-1}(z+k b)^{n-k-1} \\ +& =n f_{n-1}(z)=n(z+a)^{n-1} +\end{aligned} +$$ + +It remains to prove that $f_{n}(-a)=0$. For $z=-a$ we have by the lemma of (SL81-13), + +$$ +\begin{aligned} +f_{n}(-a) & =(-a)^{n}+a \sum_{k=1}^{n}\binom{n}{k}(-1)^{n-k}(a-k b)^{n-1} \\ +& =a \sum_{k=0}^{n}\binom{n}{k}(-1)^{n-k}(a-k b)^{n-1}=0 . +\end{aligned} +$$ + +26. The result is an immediate consequence (for $G=\{-1,1\}$ ) of the following generalization. +(1) Let $G$ be a proper subgroup of $\mathbb{Z}_{n}^{*}$ (the multiplicative group of residue classes modulo $n$ coprime to $n$ ), and let $V$ be the union of elements +of $G$. A number $m \in V$ is called indecomposable in $V$ if there do not exist numbers $p, q \in V, p, q \notin\{-1,1\}$, such that $p q=m$. There exists a number $r \in V$ that can be expressed as a product of elements indecomposable in $V$ in more than one way. +First proof. We shall start by proving the following lemma. +Lemma. There are infinitely many primes not in $V$ that do not divide $n$. +Proof. There is at least one such prime: In fact, any number other than $\pm 1$ not in $V$ must have a prime factor not in $V$, since $V$ is closed under multiplication. If there were a finite number of such primes, say $p_{1}, p_{2}, \ldots, p_{k}$, then one of the numbers $p_{1} p_{2} \cdots p_{k}+n, p_{1}^{2} p_{2} \cdots p_{k}+n$ is not in $V$ and is coprime to $n$ and $p_{1}, \ldots, p_{k}$, which is a contradiction. [This lemma is actually a direct consequence of Dirichlet's theorem.] Let us consider two such primes $p, q$ that are congruent modulo $n$. Let $p^{k}$ be the least power of $p$ that is in $V$. Then $p^{k}, q^{k}, p^{k-1} q, p q^{k-1}$ belong to $V$ and are indecomposable in $V$. It follows that + +$$ +r=p^{k} \cdot q^{k}=p^{k-1} q \cdot p q^{k-1} +$$ + +has the desired property. +Second proof. Let $p$ be any prime not in $V$ that does not divide $n$, and let $p^{k}$ be the least power of $p$ that is in $V$. Obviously $p^{k}$ is indecomposable in $V$. Then the number + +$$ +r=p^{k} \cdot\left(p^{k-1}+n\right)(p+n)=p\left(p^{k-1}+n\right) \cdot p^{k-1}(p+n) +$$ + +has at least two different factorizations into indecomposable factors. +27. The result is a consequence of the generalization from the previous problem for $G=\{1\}$. +Remark. There is an explicit example: $r=(n-1)^{2} \cdot(2 n-1)^{2}=[(n-$ 1) $(2 n-1)]^{2}$. +28. The recurrent relations give us that + +$$ +x_{i+1}=\left[\frac{x_{i}+\left[n / x_{i}\right]}{2}\right]=\left[\frac{x_{i}+n / x_{i}}{2}\right] \geq[\sqrt{n}] +$$ + +On the other hand, if $x_{i}>[\sqrt{n}]$ for some $i$, then we have $x_{i+1}\left(x_{i}+n / x_{i}\right) / 2$, i.e., to $x_{i}^{2}>n$. Therefore $x_{i}=[\sqrt{n}]$ holds for at least one $i \leq n-[\sqrt{n}]+1$. Remark. If $n+1$ is a perfect square, then $x_{i}=[\sqrt{n}]$ implies $x_{i+1}=$ $[\sqrt{n}]+1$. Otherwise, $x_{i}=[\sqrt{n}]$ implies $x_{i+1}=[\sqrt{n}]$. +29. Let us denote the midpoints of segments $L M, A N, B L, M N, B K, C M$, $N K, C L, D N, K L, D M, A K$ by $P_{1}, P_{2}, P_{3}, P_{4}, P_{5}, P_{6}, P_{7}, P_{8}, P_{9}, P_{10}$, $P_{11}, P_{12}$, respectively. + +We shall prove that the dodecagon $P_{1} P_{2} P_{3} \ldots P_{11} P_{12}$ is regular. From $B L=B A$ and $\angle A B L=30^{\circ}$ it follows that $\angle B A L=75^{\circ}$. Similarly $\angle D A M=75^{\circ}$, and therefore $\angle L A M=60^{\circ}$, which together with $A L=A M$ implies that the triangle $A L M$ is equilateral. Now, from the triangles $O L M$ and $A L N$, we get +![](https://cdn.mathpix.com/cropped/2024_11_18_8e985d6b9c83aa3e9d0eg-426.jpg?height=383&width=433&top_left_y=233&top_left_x=863) +$O P_{1}=L M / 2, O P_{2}=A L / 2$ and $O P_{2} \| A L$. Hence $O P_{1}=O P_{2}$, $\angle P_{1} O P_{2}=\angle P_{1} A L=30^{\circ}$ and $\angle P_{2} O M=\angle L A D=15^{\circ}$. The desired result follows from symmetry. +30. Suppose $\angle S B A=x$. By the trigonometric form of Ceva's theorem we have + +$$ +\frac{\sin \left(96^{\circ}-x\right)}{\sin x} \frac{\sin 18^{\circ}}{\sin 12^{\circ}} \frac{\sin 6^{\circ}}{\sin 48^{\circ}}=1 +$$ + +We claim that $x=12^{\circ}$ is a solution of this equation. To prove this, it is enough to show that $\sin 84^{\circ} \sin 6^{\circ} \sin 18^{\circ}=\sin 48^{\circ} \sin 12^{\circ} \sin 12^{\circ}$, which is equivalent to $\sin 18^{\circ}=2 \sin 48^{\circ} \sin 12^{\circ}=\cos 36^{\circ}-\cos 60^{\circ}$. The last equality can be checked directly. +Since the equation is equivalent to $\left(\sin 96^{\circ} \cot x-\cos 96^{\circ}\right) \sin 6^{\circ} \sin 18^{\circ}=$ $\sin 48^{\circ} \sin 12^{\circ}$, the solution $x \in[0, \pi)$ is unique. Hence $x=12^{\circ}$. +Second solution. We know that if $a, b, c, a^{\prime}, b^{\prime}, c^{\prime}$ are points on the unit circle in the complex plane, the lines $a a^{\prime}, b b^{\prime}, c c^{\prime}$ are concurrent if and only if + +$$ +\left(a-b^{\prime}\right)\left(b-c^{\prime}\right)\left(c-a^{\prime}\right)=\left(a-c^{\prime}\right)\left(b-a^{\prime}\right)\left(c-b^{\prime}\right) +$$ + +We shall prove that $x=12^{\circ}$. We may suppose that $A B C$ is the triangle in the complex plane with vertices $a=1, b=\epsilon^{9}, c=\epsilon^{14}$, where $\epsilon=$ $\cos \frac{\pi}{15}+i \sin \frac{\pi}{15}$. If $a^{\prime}=\epsilon^{12}, b^{\prime}=\epsilon^{28}, c^{\prime}=\epsilon$, our task is the same as proving that lines $a a^{\prime}, b b^{\prime}, c c^{\prime}$ are concurrent, or by (1) that + +$$ +\left(1-\epsilon^{28}\right)\left(\epsilon^{9}-\epsilon\right)\left(\epsilon^{14}-\epsilon^{12}\right)-(1-\epsilon)\left(\epsilon^{9}-\epsilon^{12}\right)\left(\epsilon^{14}-\epsilon^{28}\right)=0 . +$$ + +The last equality holds, since the left-hand side is divisible by the minimum polynomial of $\epsilon: z^{8}+z^{7}-z^{5}-z^{4}-z^{3}+z+1$. +31. We obtain from (1) that $f(1, c)=f(1, c) f(1, c)$; hence $f(1, c)=1$ and consequently $f(-1, c) f(-1, c)=f(1, c)=1$, i.e. $f(-1, c)=1$. Analogously, $f(c, 1)=f(c,-1)=1$. +Clearly $f(1,1)=f(-1,1)=f(1,-1)=1$. Now let us assume that $a \neq 1$. Observe that $f\left(x^{-1}, y\right)=f\left(x, y^{-1}\right)=f(x, y)^{-1}$. Thus by (1) and (2) we get + +$$ +\begin{aligned} +1 & =f(a, 1-a) f(1 / a, 1-1 / a) \\ +& =f(a, 1-a) f\left(a, \frac{1}{1-1 / a}\right)=f\left(a, \frac{1-a}{1-1 / a}\right)=f(a,-a) +\end{aligned} +$$ + +We now have $f(a, a)=f(a,-1) f(a,-a)=1 \cdot 1=1$ and $1=f(a b, a b)=$ $f(a, a b) f(b, a b)=f(a, a) f(a, b) f(b, a) f(b, b)=f(a, b) f(b, a)$. +32. It is a known result that among six persons there are 3 mutually acquainted or 3 mutually unacquainted. By the condition of the problem the last case is excluded. +If there is a man in the room who is not acquainted with four of the others, then these four men are mutually acquainted. Otherwise, each man is acquainted with at least five others, and since the sum of numbers of acquaintances of all men in the room is even, one of the men is acquainted with at least six men. Among these six there are three mutually acquainted, and they together with the first one make a group of four mutually acquainted men. +33. Let $r$ be the radius of $K$ and $s>\sqrt{2} / r$ an integer. Consider the points $A_{k}\left(k a_{1}-\left[k a_{1}\right], k a_{2}-\left[k a_{2}\right]\right)$, where $k=0,1,2, \ldots, s^{2}$. Since all these points are in the unit square, two of them, say $A_{p}, A_{q}, q>p$, are in a small square with side $1 / s$, and consequently $A_{p} A_{q} \leq \sqrt{2} / s1$ +there exist $q \in \mathbb{N}, r \in\{-1,0,1\}$ such that $s=3 q+r$, and $q$ has a unique representation of the form (1). +43. Since $\left.k(k+1) \cdots(k+p)=(p+1)!\binom{k+p}{p+1}=(p+1)!\left[\begin{array}{c}k+p+1 \\ p+2\end{array}\right)-\binom{k+p}{p+2}\right]$, it follows that +$\sum_{k=1}^{n} k(k+1) \cdots(k+p)=(p+1)!\binom{n+p+1}{p+2}=\frac{n(n+1) \cdots(n+p+1)}{p+2}$. +44. Let $d(X, \sigma)$ denote the distance from a point $X$ to a plane $\sigma$. Let us consider the pair $(A, \pi)$ where $A \in E$ and $\pi$ is a plane containing some three points $B, C, D \in E$ such that $d(A, \pi)$ is the smallest possible. We may suppose that $B, C, D$ are selected such that $\triangle B C D$ contains no other points of $E$. Let $A^{\prime}$ be the projection of $A$ on $\pi$, and let $l_{b}, l_{c}, l_{d}$ be lines through $B, C, D$ parallel to $C D, D B, B C$ respectively. If $A^{\prime}$ is in the half-plane determined by $l_{d}$ not containing $B C$, then $d(D, A B C) \leq d\left(A^{\prime}, A B C\right)|x|$. There is no loss of generality in assuming $x>0$. +To obtain the estimate from below, set + +$$ +\begin{aligned} +a_{1} & =f\left(-\frac{x+t}{2}\right)-f(-(x+t)), & a_{2}=f(0)-f\left(-\frac{x+t}{2}\right), \\ +a_{3} & =f\left(\frac{x+t}{2}\right)-f(0), & a_{4}=f(x+t)-f\left(\frac{x+t}{2}\right) . +\end{aligned} +$$ + +Since $-(x+t)\frac{a_{4}}{a_{1}+a_{2}+a_{3}}>\frac{a_{3} / 2}{4 a_{3}+2 a_{3}+a_{3}}=14^{-1} . +$$ + +To obtain the estimate from above, set + +$$ +\begin{aligned} +b_{1} & =f(0)-f\left(-\frac{x+t}{3}\right), & b_{2}=f\left(\frac{x+t}{3}\right)-f(0), \\ +b_{3} & =f\left(\frac{2(x+t)}{3}\right)-f\left(\frac{x+t}{3}\right), & b_{4}=f(x+t)-f\left(\frac{2(x+t)}{3}\right) . +\end{aligned} +$$ + +If $t<2 x$, then $x-t<-(x+t) / 3$ and therefore $f(x)-f(x-t) \geq b_{1}$. If $t \geq 2 x$, then $(x+t) / 3 \leq x$ and therefore $f(x)-f(x-t) \geq b_{2}$. Since $2^{-1}2 C D / 3$, then $N$ coincides with C . +48. Let a plane cut the edges $A B, B C, C D, D A$ at points $K, L, M, N$ respectively. +Let $D^{\prime}, A^{\prime}, B^{\prime}$ be distinct points in the plane $A B C$ such that the triangles $B C D^{\prime}, C D^{\prime} A^{\prime}, D^{\prime} A^{\prime} B^{\prime}$ are equilateral, and $M^{\prime} \in\left[C D^{\prime}\right], N^{\prime} \in\left[D^{\prime} A^{\prime}\right]$, and $K^{\prime} \in\left[A^{\prime} B^{\prime}\right]$ such that $C M^{\prime}=C M$, $A^{\prime} N^{\prime}=A N$, and $A^{\prime} K^{\prime}=A K$. The perimeter $P$ of the quadrilateral $K L M N$ is equal to the length of the polygonal line $K L M^{\prime} N^{\prime} K^{\prime}$, which is not less than $K K^{\prime}$. It follows that $P \geq 2 a$. +![](https://cdn.mathpix.com/cropped/2024_11_18_8e985d6b9c83aa3e9d0eg-430.jpg?height=265&width=548&top_left_y=997&top_left_x=810) + +Let us consider all quadrilaterals $K L M N$ that are obtained by intersecting the tetrahedron by a plane parallel to a fixed plane $\alpha$. The lengths of the segments $K L, L M, M N, N K$ are linear functions in $A K$, and so is $P$. Thus $P$ takes its maximum at an endpoint of the interval, i.e., when the plane $K L M N$ passes through one of the vertices $A, B, C, D$, and it is easy to see that in this case $P \leq 3 a$. +49. If one of $p, q$, say $p$, is zero, then $-q$ is a perfect square. Conversely, $(p, q)=\left(0,-t^{2}\right)$ and $(p, q)=\left(-t^{2}, 0\right)$ satisfy the conditions for $t \in \mathbb{Z}$. +We now assume that $p, q$ are nonzero. If the trinomial $x^{2}+p x+q$ has two integer roots $x_{1}, x_{2}$, then $|q|=\left|x_{1} x_{2}\right| \geq\left|x_{1}\right|+\left|x_{2}\right|-1 \geq|p|-1$. Similarly, if $x^{2}+q x+p$ has integer roots, then $|p| \geq|q|-1$ and $q^{2}-4 p$ is a square. Thus we have two cases to investigate: +(i) $|p|=|q|$. Then $p^{2}-4 q=p^{2} \pm 4 p$ is a square, so $(p, q)=(4,4)$. +(ii) $|p|=|q| \pm 1$. The solutions for $(p, q)$ are $(t,-1-t)$ for $t \in \mathbb{Z}$ and $(5,6)$, $(6,5)$. +50. Suppose that $P_{n}(x)=n$ for $x \in\left\{x_{1}, x_{2}, \ldots, x_{n}\right\}$. Then + +$$ +P_{n}(x)=\left(x-x_{1}\right)\left(x-x_{2}\right) \cdots\left(x-x_{n}\right)+n . +$$ + +From $P_{n}(0)=0$ we obtain $n=\left|x_{1} x_{2} \cdots x_{n}\right| \geq 2^{n-2}$ (because at least $n-2$ factors are different from $\pm 1$ ) and therefore $n \geq 2^{n-2}$. It follows that $n \leq 4$. +For each positive integer $n \leq 4$ there exists a polynomial $P_{n}$. Here is the list of such polynomials: + +$$ +\begin{array}{ll} +n=1: \pm x, & n=2: 2 x^{2}, x^{2} \pm x,-x^{2} \pm 3 x \\ +n=3: \pm\left(x^{3}-x\right)+3 x^{2}, & n=4:-x^{4}+5 x^{2} . +\end{array} +$$ + +51. We shall use the following algorithm: + +Choose a segment of maximum length ("basic" segment) and put on it unused segments of the opposite color without overlapping, each time of the maximum possible length, as long as it is possible. Repeat the procedure with remaining segments until all the segments are used. +Let us suppose that the last basic segment is black. Then the length of the used part of any white basic segment is greater than the free part, and consequently at least one-half of the length of the white segments has been used more than once. Therefore all basic segments have total length at most 1.5 and can be distributed on a segment of length 1.51. +On the other hand, if we are given two white segments of lengths 0.5 and two black segments of lengths 0.999 and 0.001 , we cannot distribute them on a segment of length less than 1.499. +52. The maximum and minimum are $2 R \sqrt{4-2 k^{2}}$ and $2 R\left(1+\sqrt{1-k^{2}}\right)$ respectively. +53. The discriminant of the given equation considered as a quadratic equation in $b$ is $196-75 a^{2}$. Thus $75 a^{2} \leq 196$ and hence $-1 \leq a \leq 1$. Now the integer solutions of the given equation are easily found: $(-1,3),(0,0),(1,2)$. +54. We shall use the following lemma. + +Lemma. If a real function $f$ is convex on the interval $I$ and $x, y, z \in I$, $x \leq y \leq z$, then + +$$ +(y-z) f(x)+(z-x) f(y)+(x-y) f(z) \leq 0 . +$$ + +Proof. The inequality is obvious for $x=y=z$. If $xs_{k+m}$ for $0 \leq k \leq l-m$ and $s_{k}a_{k}$ in the ring, then $q n=r m$, which implies $m^{\prime}\left|q, n^{\prime}\right| p$ and thus $k=p+q \geq m^{\prime}+n^{\prime}$. But since all $i_{1}, i_{2}, \ldots, i_{k}$ are congruent modulo $d$, we have $k \leq m^{\prime}+n^{\prime}-1$, a contradiction. Hence there exists a sequence of length $m+n-d-1$ with the required property. +58. The following inequality (Finsler and Hadwiger, 1938) is sharper than the one we have to prove: + +$$ +2 a b+2 b c+2 c a-a^{2}-b^{2}-c^{2} \geq 4 S \sqrt{3} +$$ + +First proof. Let us set $2 x=b+c-a, 2 y=c+a-b, 2 z=a+b-c$. + +Then $x, y, z>0$ and the inequality (1) becomes + +$$ +y^{2} z^{2}+z^{2} x^{2}+x^{2} y^{2} \geq x y z(x+y+z) +$$ + +which is equivalent to the obvious inequality $(x y-y z)^{2}+(y z-z x)^{2}+$ $(z x-x y)^{2} \geq 0$. +Second proof. Using the known relations for a triangle + +$$ +\begin{aligned} +a^{2}+b^{2}+c^{2} & =2 s^{2}-2 r^{2}-8 r R, \\ +a b+b c+c a & =s^{2}+r^{2}+4 r R, \\ +S & =r s, +\end{aligned} +$$ + +where $r$ and $R$ are the radii of the incircle and the circumcircle, $s$ the semiperimeter and $S$ the area, we can transform (1) into + +$$ +s \sqrt{3} \leq 4 R+r . +$$ + +The last inequality is a consequence of the inequalities $2 r \leq R$ and $s^{2} \leq$ $4 R^{2}+4 R r+3 r^{2}$, where the last one follows from the equality $H I^{2}=$ $4 R^{2}+4 R r+3 r^{2}-s^{2}$ ( $H$ and $I$ being the orthocenter and the incenter of the triangle). +59. Let us consider the set $R$ of pairs of coordinates of the points from $E$ reduced modulo 3 . If some element of $R$ occurs thrice, then the corresponding points are vertices of a triangle with integer barycenter. Also, no three elements from $E$ can have distinct $x$-coordinates and distinct $y$ coordinates. By an easy discussion we can conclude that the set $R$ contains at most four elements. Hence $|E| \leq 8$. +An example of a set $E$ consisting of 8 points that satisfies the required condition is + +$$ +E=\{(0,0),(1,0),(0,1),(1,1),(3,6),(4,6),(3,7),(4,7)\} +$$ + +60. By Lagrange's interpolation formula we have + +$$ +F(x)=\sum_{j=0}^{n} F\left(x_{j}\right) \frac{\prod_{i \neq j}\left(x-x_{j}\right)}{\prod_{i \neq j}\left(x_{i}-x_{j}\right)} . +$$ + +Since the leading coefficient in $F(x)$ is 1 , it follows that + +$$ +1=\sum_{j=0}^{n} \frac{F\left(x_{j}\right)}{\prod_{i \neq j}\left(x_{i}-x_{j}\right)} +$$ + +Since + +$$ +\left|\prod_{i \neq j}\left(x_{i}-x_{j}\right)\right|=\prod_{i=0}^{j-1}\left|x_{i}-x_{j}\right| \prod_{i=j+1}^{n}\left|x_{i}-x_{j}\right| \geq j!(n-j)! +$$ + +we have + +$$ +1 \leq \sum_{j=0}^{n} \frac{\left|F\left(x_{j}\right)\right|}{\left|\prod_{i \neq j}\left(x_{i}-x_{j}\right)\right|} \leq \frac{1}{n!} \sum_{j=0}^{n}\binom{n}{j}\left|F\left(x_{j}\right)\right| \leq \frac{2^{n}}{n!} \max \left|F\left(x_{j}\right)\right| . +$$ + +Now the required inequality follows immediately. + +### 4.20 Solutions to the Shortlisted Problems of IMO 1978 + +1. There exists an $M_{s}$ that contains at least $2 n / k=2\left(k^{2}+1\right)$ elements. It follows that $M_{s}$ contains either at least $k^{2}+1$ even numbers or at least $k^{2}+1$ odd numbers. In the former case, consider the predecessors of those $k^{2}+1$ numbers: among them, at least $\frac{k^{2}+1}{k+1}>k$, i.e., at least $k+1$, belong to the same subset, say $M_{t}$. Then we choose $s, t$. The latter case is similar. Second solution. For all $i, j \in\{1,2, \ldots, k\}$, consider the set $N_{i j}=\{r \mid$ $\left.2 r \in M_{i}, 2 r-1 \in M_{j}\right\}$. Then $\left\{N_{i j} \mid i, j\right\}$ is a partition of $\{1,2, \ldots, n\}$ into $k^{2}$ subsets. For $n \geq k^{3}+1$ one of these subsets contains at least $k+1$ elements, and the statement follows. +Remark. The statement is not necessarily true when $n=k^{3}$. +2. Consider the transformation $\phi$ of the plane defined as the homothety $\mathcal{H}$ with center $B$ and coefficient 2 followed by the rotation $\mathcal{R}$ about the center $O$ through an angle of $60^{\circ}$. Being direct, this mapping must be a rotational homothety. We also see that $\mathcal{H}$ maps $S$ into the point symmetric to $S$ with respect to $O A$, and $\mathcal{R}$ takes it back to $S$. Hence $S$ is a fixed point, and is consequently also the center of $\phi$. Therefore $\phi$ is the rotational homothety about $S$ with the angle $60^{\circ}$ +![](https://cdn.mathpix.com/cropped/2024_11_18_8e985d6b9c83aa3e9d0eg-435.jpg?height=319&width=462&top_left_y=920&top_left_x=858) +and coefficient 2. (In fact, this could also be seen from the fact that $\phi$ preserves angles of triangles and maps the segment $S R$ onto $S B$, where $R$ is the midpoint of $A B$.) +Since $\phi(M)=B^{\prime}$, we conclude that $\angle M S B^{\prime}=60^{\circ}$ and $S B^{\prime} / S M=2$. Similarly, $\angle N S A^{\prime}=60^{\circ}$ and $S A^{\prime} / S N=2$, so triangles $M S B^{\prime}$ and $N S A^{\prime}$ are indeed similar. +Second solution. Probably the simplest way here is using complex numbers. Put the origin at $O$ and complex numbers $a, a^{\prime}$ at points $A, A^{\prime}$, and denote the primitive sixth root of 1 by $\omega$. Then the numbers at $B, B^{\prime}$, $S$ and $N$ are $\omega a, \omega a^{\prime},(a+\omega a) / 3$, and $\left(a+\omega a^{\prime}\right) / 2$ respectively. Now it is easy to verify that $(n-s)=\omega\left(a^{\prime}-s\right) / 2$, i.e., that $\angle N S A^{\prime}=60^{\circ}$ and $S A^{\prime} / S N=2$. +3. What we need are $m, n$ for which $1978^{m}\left(1978^{n-m}-1\right)$ is divisible by $1000=8 \cdot 125$. Since $1978^{n-m}-1$ is odd, it follows that $1978^{m}$ is divisible by 8 , so $m \geq 3$. +Also, $1978^{n-m}-1$ is divisible by 125 , i.e., $1978^{n-m} \equiv 1(\bmod 125)$. Note that $1978 \equiv-2(\bmod 5)$, and consequently also $-2^{n-m} \equiv 1$. Hence $4 \mid n-m=4 k, k \geq 1$. It remains to find the least $k$ such that $1978^{4 k} \equiv 1$ $(\bmod 125)$. Since $1978^{4} \equiv(-22)^{4}=484^{2} \equiv(-16)^{2}=256 \equiv 6$, we reduce it to $6^{k} \equiv 1$. Now $6^{k}=(1+5)^{k} \equiv 1+5 k+25\binom{k}{2}(\bmod 125)$, which +reduces to $125 \mid 5 k(5 k-3)$. But $5 k-3$ is not divisible by 5 , and so $25 \mid k$. Therefore $100 \mid n-m$, and the desired values are $m=3, n=103$. +4. Let $\gamma, \varphi$ be the angles of $T_{1}$ and $T_{2}$ opposite to $c$ and $w$ respectively. By the cosine theorem, the inequality is transformed into + +$$ +\begin{aligned} +& a^{2}\left(2 v^{2}-2 u v \cos \varphi\right)+b^{2}\left(2 u^{2}-2 u v \cos \varphi\right) \\ +& \quad+2\left(a^{2}+b^{2}-2 a b \cos \gamma\right) u v \cos \varphi \geq 4 a b u v \sin \gamma \sin \varphi +\end{aligned} +$$ + +This is equivalent to $2\left(a^{2} v^{2}+b^{2} u^{2}\right)-4 a b u v(\cos \gamma \cos \varphi+\sin \gamma \sin \varphi) \geq 0$, i.e., to + +$$ +2(a v-b u)^{2}+4 a b u v(1-\cos (\gamma-\varphi)) \geq 0 +$$ + +which is clearly satisfied. Equality holds if and only if $\gamma=\varphi$ and $a / b=$ $u / v$, i.e., when the triangles are similar, $a$ corresponding to $u$ and $b$ to $v$. +5. We first explicitly describe the elements of the sets $M_{1}, M_{2}$. $x \notin M_{1}$ is equivalent to $x=a+(a+1)+\cdots+(a+n-1)=n(2 a+n-1) / 2$ for some natural numbers $n, a, n \geq 2$. Among $n$ and $2 a+n-1$, one is odd and the other even, and both are greater than 1 ; so $x$ has an odd factor $\geq 3$. On the other hand, for every $x$ with an odd divisor $p>3$ it is easy to see that there exist corresponding $a, n$. Therefore $M_{1}=\left\{2^{k} \mid k=0,1,2, \ldots\right\}$. +$x \notin M_{2}$ is equivalent to $x=a+(a+2)+\cdots+(a+2(n-1))=n(a+n-1)$, where $n \geq 2$, i.e. to $x$ being composite. Therefore $M_{2}=\{1\} \cup\{p \mid$ $p=$ prime $\}$. +$x \notin M_{3}$ is equivalent to $x=a+(a+3)+\cdots+(a+3(n-1))=$ $n(2 a+3(n-1)) / 2$. +It remains to show that every $c \in M_{3}$ can be written as $c=2^{k} p$ with $p$ prime. Suppose the opposite, that $c=2^{k} p q$, where $p, q$ are odd and $q \geq p \geq 3$. Then there exist positive integers $a, n(n \geq 2)$ such that $c=n(2 a+3(n-1)) / 2$ and hence $c \notin M_{3}$. Indeed, if $k=0$, then $n=2$ and $2 a+3=p q$ work; otherwise, setting $n=p$ one obtains $a=2^{k} q-$ $3(p-1) / 2 \geq 2 q-3(p-1) / 2 \geq(p+3) / 2>1$. +6. For fixed $n$ and the set $\{\varphi(1), \ldots, \varphi(n)\}$, there are finitely many possibilities for mapping $\varphi$ to $\{1, \ldots, n\}$. Suppose $\varphi$ is the one among these for which $\sum_{k=1}^{n} \varphi(k) / k^{2}$ is minimal. If $i0 +\end{aligned} +$$ + +which contradicts the assumption. This shows that $\varphi(1)<\cdots<\varphi(n)$, and consequently $\varphi(k) \geq k$ for all $k$. Hence + +$$ +\sum_{k=1}^{n} \frac{\varphi(k)}{k^{2}} \geq \sum_{k=1}^{n} \frac{k}{k^{2}}=\sum_{k=1}^{n} \frac{1}{k} +$$ + +7. Let $x=O A, y=O B, z=O C, \alpha=\angle B O C, \beta=\angle C O A, \gamma=\angle A O B$. The conditions yield the equation $x+y+\sqrt{x^{2}+y^{2}-2 x y \cos \gamma}=2 p$, which transforms to $(2 p-x-y)^{2}=x^{2}+y^{2}-2 x y \cos \gamma$, i.e. $(p-x)(p-y)=$ $x y(1-\cos \gamma)$. Thus + +$$ +\frac{p-x}{x} \cdot \frac{p-y}{y}=1-\cos \gamma +$$ + +and analogously $\frac{p-y}{y} \cdot \frac{p-z}{z}=1-\cos \alpha, \frac{p-z}{z} \cdot \frac{p-x}{x}=1-\cos \beta$. Setting $u=\frac{p-x}{x}, v=\frac{p-y}{y}, w=\frac{p-z}{z}$, the above system becomes + +$$ +u v=1-\cos \gamma, \quad v w=1-\cos \alpha, \quad w u=1-\cos \beta +$$ + +This system has a unique solution in positive real numbers $u, v, w$ : $u=\sqrt{\frac{(1-\cos \beta)(1-\cos \gamma)}{1-\cos \alpha}}$, etc. Finally, the values of $x, y, z$ are uniquely determined from $u, v, w$. +Remark. It is not necessary that the three lines be in the same plane. Also, there could be any odd number of lines instead of three. +8. Take the subset $\left\{a_{i}\right\}=\{1,7,11,13,17,19,23,29, \ldots, 30 m-1\}$ of $S$ containing all the elements of $S$ that are not multiples of 3 . There are 8 m such elements. Every element in $S$ can be uniquely expressed as $3^{t} a_{i}$ for some $i$ and $t \geq 0$. In a subset of $S$ with $8 m+1$ elements, two of them will have the same $a_{i}$, hance one will divide the other. +On the other hand, for each $i=1,2, \ldots, 8 m$ choose $t \geq 0$ such that $10 \mathrm{~m}<$ $b_{i}=3^{t} a_{i}<30 \mathrm{~m}$. Then there are $8 \mathrm{~m} b_{i}$ 's in the interval $(10 \mathrm{~m}, 30 \mathrm{~m})$, and the quotient of any two of them is less than 3 , so none of them can divide any other. Thus the answer is 8 m . +9. Since the $n$th missing number (gap) is $f(f(n))+1$ and $f(f(n))$ is a member of the sequence, there are exactly $n-1$ gaps less than $f(f(n))$. This leads to + +$$ +f(f(n))=f(n)+n-1 +$$ + +Since 1 is not a gap, we have $f(1)=1$. The first gap is $f(f(1))+1=2$. Two consecutive integers cannot both be gaps (the predecessor of a gap is of the form $f(f(m))$ ). Now we deduce $f(2)=3$; a repeated application of the formula above gives $f(3)=3+1=4, f(4)=4+2=6, f(6)=9$, $f(9)=14, f(14)=22, f(22)=35, f(35)=56, f(56)=90, f(90)=145$, $f(145)=234, f(234)=378$. +Also, $f(f(35))+1=91$ is a gap, so $f(57)=92$. Then by $(1), f(92)=148$, $f(148)=239, f(239)=386$. Finally, here $f(f(148))+1=387$ is a gap, so $f(240)=388$. + +Second solution. As above, we arrive at formula (1). Then by simple induction it follows that $f\left(F_{n}+1\right)=F_{n+1}+1$, where $F_{k}$ is the Fibonacci sequence ( $F_{1}=F_{2}=1$ ). +We now prove by induction (on $n$ ) that $f\left(F_{n}+x\right)=F_{n+1}+f(x)$ for all $x$ with $1 \leq x \leq F_{n-1}$. This is trivially true for $n=0,1$. Supposing that it holds for $n-1$, we shall prove it for $n$ : +(i) If $x=f(y)$ for some $y$, then by the inductive assumption and (1) + +$$ +\begin{aligned} +f\left(F_{n}+x\right) & =f\left(F_{n}+f(y)\right)=f\left(f\left(F_{n-1}+y\right)\right) \\ +& =F_{n}+f(y)+F_{n-1}+y-1=F_{n+1}+f(x) +\end{aligned} +$$ + +(ii) If $x=f(f(y))+1$ is a gap, then $f\left(F_{n}+x-1\right)+1=F_{n+1}+f(x-1)+1$ is a gap also: + +$$ +\begin{aligned} +F_{n+1}+f(x)+1 & =F_{n+1}+f(f(f(y)))+1 \\ +& =f\left(F_{n}+f(f(y))\right)+1=f\left(f\left(F_{n-1}+f(y)\right)\right)+1 +\end{aligned} +$$ + +It follows that $f\left(F_{n}+x\right)=F_{n+1}+f(x-1)+2=F_{n+1}+f(x)$. +Now, since we know that each positive integer $x$ is expressible as $x=$ $F_{k_{1}}+F_{k_{2}}+\cdots+F_{k_{r}}$, where $0R$. + +We claim that the locus of $Q$ is the whole sphere $\left(O, \sqrt{3 R^{2}-O P^{2}}\right)$. Choose any point $Q$ on this sphere. Since $O Q>R>O P$, the sphere +with diameter $P Q$ intersects $S$ on a circle. Let $C$ be an arbitrary point on this circle, and $X$ the point opposite $C$ in the rectangle $P C Q X$. By the lemma, $O P^{2}+O Q^{2}=O C^{2}+O X^{2}$, hence $O X^{2}=2 R^{2}-O P^{2}>R^{2}$. The plane passing through $P$ and perpendicular to $P C$ intersects $S$ in a circle $\gamma$; both $P, X$ belong to this plane, $P$ being inside and $X$ outside the circle, so that the circle with diameter $P X$ intersects $\gamma$ at some point $B$. Finally, we choose $A$ to be the point opposite $B$ in the rectangle $P B X A$ : we deduce that $O A^{2}+O B^{2}=O P^{2}+O X^{2}$, and consequently $A \in S$. By the construction, there is a rectangular parallelepiped through $P, A, B, C, X, Q$. +14. We label the cells of the cube by $\left(a_{1}, a_{2}, a_{3}\right), a_{i} \in\{1,2, \ldots, 2 n+1\}$, in a natural way: for example, as Cartesian coordinates of centers of the cells $\left((1,1,1)\right.$ is one corner, etc.). Notice that there should be $(2 n+1)^{3}-$ $2 n(2 n+1) \cdot 2(n+1)=2 n+1$ void cells, i.e., those not covered by any piece of soap. +$n=1$. In this case, six pieces of soap $1 \times 2 \times 2$ can be placed on the following positions: $[(1,1,1),(2,2,1)],[(3,1,1),(3,2,2)],[(2,3,1),(3,3,2)]$ and the symmetric ones with respect to the center of the box. (Here $[A, B]$ denotes the rectangle with opposite corners at $A, B$.) +$n$ is even. Each of the $2 n+1$ planes $P_{k}=\left\{\left(a_{1}, a_{2}, k\right) \mid a_{i}=1, \ldots, 2 n+1\right\}$ can receive $2 n$ pieces of soap: In fact, $P_{k}$ can be partitioned into four $n \times(n+1)$ rectangles at the corners and the central cell, while an $n \times(n+1)$ rectangle can receive $n / 2$ pieces of soap. +$n$ is odd, $n>1$. Let us color a cell $\left(a_{1}, a_{2}, a_{3}\right)$ blue, red, or yellow if exactly three, two or one $a_{i}$ respectively is equal to $n+1$. Thus there are 1 blue, $6 n$ red, and $12 n^{2}$ yellow cells. We notice that each piece of soap must contain at least one colored cell (because $2(n+1)>2 n+1)$. Also, every piece of soap contains an even number (actually, $1 \cdot 2,1(n+1)$, or $2(n+1)$ ) of cells in $P_{k}$. On the other hand, $2 n+1$ cells are void, i.e., one in each plane. + +There are several cases for a piece of soap $S$ : +(i) $S$ consists of 1 blue, $n+1$ red and $n$ yellow cells; +(ii) $S$ consists of 2 red and $2 n$ yellow cells (and no blue cells); +(iii) $S$ contains 1 red cell, $n+1$ yellow cells, and the are rest uncolored; +(iv) $S$ contains 2 yellow cells and no blue or red ones. + +From the descriptions of the last three cases, we can deduce that if $S$ contains $r$ red cells and no blue, then it contains exactly $2+(n-1) r$ red ones. $\quad(*)$ +Now, let $B_{1}, \ldots, B_{k}$ be all boxes put in the cube, with a possible exception for the one covering the blue cell: thus $k=2 n(2 n+1)$ if the blue cell is void, or $k=2 n(2 n+1)-1$ otherwise. Let $r_{i}$ and $y_{i}$ respectively be the numbers of red and yellow cells inside $B_{i}$. By (*) we have $y_{1}+\cdots+y_{k}=2 k+(n-1)\left(r_{1}+\cdots+r_{k}\right)$. If the blue cell is void, then $r_{1}+\cdots+r_{k}=6 n$ and consequently $y_{1}+\cdots+y_{k}=$ +$4 n(2 n+1)+6 n(n-1)=14 n^{2}-2 n$, which is impossible because there are only $12 n^{2}<14 n^{2}-2 n$ yellow cells. Otherwise, $r_{1}+\cdots+r_{k} \geq 5 n-2$ (because $n+1$ red cells are covered by the box containing the blue cell, and one can be void) and consequently $y_{1}+\cdots+y_{k} \geq 4 n(2 n+$ $1)-2+(n-1)(5 n-2)=13 n^{2}-3 n$; since there are $n$ more yellow cells in the box containing the blue one, this counts for $13 n^{2}-2 n>12 n^{2}$ ( $n \geq 3$ ), again impossible. +Remark. The following solution of the case $n$ odd is simpler, but does not work for $n=3$. For $k=1,2,3$, let $m_{k}$ be the number of pieces whose long sides are perpendicular to the plane $\pi_{k}\left(a_{k}=n+1\right)$. Each of these $m_{k}$ pieces covers exactly 2 cells of $\pi_{k}$, while any other piece covers $n+1$, $2(n+1)$, or none. It follows that $4 n^{2}+4 n-2 m_{k}$ is divisible by $n+1$, and so is $2 m_{k}$. This further implies that $2 m_{1}+2 m_{2}+2 m_{3}=4 n(2 n+1)$ is a multiple of $n+1$, which is impossible for each odd $n$ except $n=1$ and $n=3$. +15. Let $C_{n}=\left\{a_{1}, \ldots, a_{n}\right\}\left(C_{0}=\emptyset\right)$ and $P_{n}=\left\{f(B) \mid B \subseteq C_{n}\right\}$. We claim that $P_{n}$ contains at least $n+1$ distinct elements. First note that $P_{0}=\{0\}$ contains one element. Suppose that $P_{n+1}=P_{n}$ for some $n$. Since $P_{n+1}=$ $\left\{a_{n+1}+r \mid r \in P_{n}\right\}$, it follows that for each $r \in P_{n}$, also $r+b_{n} \in P_{n}$. Then obviously $0 \in P_{n}$ implies $k b_{n} \in P_{n}$ for all $k$; therefore $P_{n}=P$ has at least $p \geq n+1$ elements. Otherwise, if $P_{n+1} \supset P_{n}$ for all $n$, then $\left|P_{n+1}\right| \geq\left|P_{n}\right|+1$ and hence $\left|P_{n}\right| \geq n+1$, as claimed. Consequently, $\left|P_{p-1}\right| \geq p$. (All the operations here are performed modulo $p$.) +16. Clearly $|x| \leq 1$. As $x$ runs over $[-1,1]$, the vector $u=\left(a x, a \sqrt{1-x^{2}}\right)$ runs over all vectors of length $a$ in the plane having a nonnegative vertical component. Putting $v=\left(b y, b \sqrt{1-y^{2}}\right), w=\left(c z, c \sqrt{1-z^{2}}\right)$, the system becomes $u+v=w$, with vectors $u, v, w$ of lengths $a, b, c$ respectively in the upper half-plane. Then $a, b, c$ are sides of a (possibly degenerate) triangle; i.e, $|a-b| \leq c \leq a+b$ is a necessary condition. + +Conversely, if $a, b, c$ satisfy this condition, one constructs a triangle $O M N$ with $O M=a, O N=b, M N=c$. If the vectors $\overrightarrow{O M}, \overrightarrow{O N}$ have a positive nonnegative component, then so does their sum. For every such triangle, putting $u=\overrightarrow{O M}, v=\overrightarrow{O N}$, and $w=\overrightarrow{O M}+\overrightarrow{O N}$ gives a solution, and every solution is given by one such triangle. This triangle is uniquely determined up to congruence: $\alpha=\angle M O N=\angle(u, v)$ and $\beta=\angle(u, w)$. +Therefore, all solutions of the system are + +$$ +\begin{aligned} +& x=\cos t, \quad y=\cos (t+\alpha), \quad z=y=\cos (t+\beta), \quad t \in[0, \pi-\alpha] \quad \text { or } \\ +& x=\cos t, \quad y=\cos (t-\alpha), \quad z=y=\cos (t-\beta), \quad t \in[\alpha, \pi] . +\end{aligned} +$$ + +17. Let $z_{0} \geq 1$ be a positive integer. Supposing that the statement is true for all triples $(x, y, z)$ with $z1$ and $x_{0}2\left(z_{0}-z_{0}\right)=0$. Moreover, $x y-z^{2}=x_{0}\left(x_{0}+y_{0}-\right.$ $\left.2 z_{0}\right)-\left(z_{0}-x_{0}\right)^{2}=x_{0} y_{0}-z_{0}^{2}=1$ and $z0$. Now it follows from the first part that there exist integers $a, b$ such that $x=p=a^{2}+b^{2}$. +Second solution. Another possibility is using arithmetic of Gaussian integers. +Lemma. Suppose $m, n, p, q$ are elements of $\mathbb{Z}$ or any other unique factorization domain, with $m n=p q$. then there exist elements $a, b, c, d$ such that $m=a b, n=c d, p=a c, q=b d$. +Proof is direct, for example using factorization of $a, b, c, d$ into primes. +We now apply this lemma to the Gaussian integers in our case (because $\mathbb{Z}[i]$ has the unique factorization property), having in mind that $x y=$ $z^{2}+1=(z+i)(z-i)$. We obtain +(1) $x=a b$, +(2) $y=c d$, +(3) $z+i=a c$, +(4) $z-i=b d$ +for some $a, b, c, d \in \mathbb{Z}[i]$. Let $a=a_{1}+a_{2} i$, etc. By (3) and (4), $\operatorname{gcd}\left(a_{1}, a_{2}\right)=$ $\cdots=\operatorname{gcd}\left(d_{1}, d_{2}\right)$. Then (1) and (2) give us $b=\bar{a}, c=\bar{d}$. The statement follows at once: $x=a b=a \bar{a}=a_{1}^{2}+a_{2}^{2}, y=d \bar{d}=d_{1}^{2}+d_{2}^{2}$ and $z+i=$ $\left(a_{1} d_{1}+a_{2} d_{2}\right)+\imath\left(a_{2} d_{1}-a_{1} d_{2}\right) \Rightarrow z=a_{1} d_{1}+a_{2} d_{2}$. + +### 4.21 Solutions to the Shortlisted Problems of IMO 1979 + +1. We prove more generally, by induction on $n$, that any $2 n$-gon with equal edges and opposite edges parallel to each other can be dissected. For $n=2$ the only possible such $2 n$-gon is a single lozenge, so our theorem holds in this case. We will now show that it holds for general $n$. Assume by induction that it holds for $n-1$. Let $A_{1} A_{2} \ldots A_{2 n}$ be an arbitrary $2 n$-gon with equal edges and opposite edges parallel to each other. Then we can construct points $B_{i}$ for $i=3,4, \ldots, n$ such that $\overrightarrow{A_{i} B_{i}}=\overrightarrow{A_{2} A_{1}}=\overrightarrow{A_{n+1} A_{n+2}}$. We set $B_{2}=A_{2 n+1}=A_{1}$ and $B_{n+1}=A_{n+2}$. It follows that $A_{i} B_{i} B_{i+1} A_{i+1}$ for $i=2,3,4, \ldots, n$ are all lozenges. It also follows that $B_{i} B_{i+1}$ for $i=2,3,4, \ldots, n$ are equal to the edges of $A_{1} A_{2} \ldots A_{2 n}$ and parallel to $A_{i} A_{i+1}$ and hence to $A_{n+i} A_{n+i+1}$. Thus $B_{2} \ldots B_{n+1} A_{n+3} \ldots A_{2 n}$ is a $2(n-1)$-gon with equal edges and opposite sides parallel and hence, by the induction hypothesis, can be dissected into lozenges. We have thus provided a dissection for $A_{1} A_{2} \ldots A_{2 n}$. This completes the proof. +2. The only way to arrive at the latter alternative is to draw four different socks in the first drawing or to draw only one pair in the first drawing and then draw two different socks in the last drawing. We will call these probabilities respectively $p_{1}, p_{2}, p_{3}$. We calculate them as follows: + +$$ +p_{1}=\frac{\binom{5}{4} 2^{4}}{\binom{10}{4}}=\frac{8}{21}, \quad p_{2}=\frac{5\binom{4}{2} 2^{2}}{\binom{10}{4}}=\frac{4}{7}, \quad p_{3}=\frac{4}{\binom{6}{2}}=\frac{4}{15} . +$$ + +We finally calculate the desired probability: $P=p_{1}+p_{2} p_{3}=\frac{8}{15}$. +3. An obvious solution is $f(x)=0$. We now look for nonzero solutions. We note that plugging in $x=0$ we get $f(0)^{2}=f(0)$; hence $f(0)=0$ or $f(0)=1$. If $f(0)=0$, then $f$ is of the form $f(x)=x^{k} g(x)$, where $g(0) \neq 0$. Plugging this formula into $f(x) f\left(2 x^{2}\right)=f\left(2 x^{3}+x\right)$ we get $2^{k} x^{2 k} g(x) g\left(2 x^{2}\right)=\left(2 x^{2}+1\right)^{k} g\left(2 x^{3}+x\right)$. Plugging in $x=0$ gives us $g(0)=0$, which is a contradiction. Hence $f(0)=1$. +For an arbitrary root $\alpha$ of the polynomial $f, 2 \alpha^{3}+\alpha$ must also be a root. Let $\alpha$ be a root of the largest modulus. If $|\alpha|>1$ then $\left|2 \alpha^{3}+\alpha\right|>$ $2|\alpha|^{3}-|\alpha|>|\alpha|$, which is impossible. It follows that $|\alpha| \leq 1$ and hence all roots of $f$ have modules less than or equal to 1 . But the product of all roots of $f$ is $|f(0)|=1$, which implies that all the roots have modulus 1. Consequently, for a root $\alpha$ it holds that $|\alpha|=\left|2 \alpha^{3}-\alpha\right|=1$. This is possible only if $\alpha= \pm \imath$. Since the coefficients of $f$ are real it follows that $f$ must be of the form $f(x)=\left(x^{2}+1\right)^{k}$ where $k \in \mathbb{N}_{0}$. These polynomials satisfy the original formula. Hence, the solutions for $f$ are $f(x)=0$ and $f(x)=\left(x^{2}+1\right)^{k}, k \in \mathbb{N}_{0}$. +4. Let us prove first that the edges $A_{1} A_{2}, A_{2} A_{3}, \ldots, A_{5} A_{1}$ are of the same color. Assume the contrary, and let w.l.o.g. $A_{1} A_{2}$ be red and $A_{2} A_{3}$ be +green. Three of the segments $A_{2} B_{l}(l=1,2,3,4,5)$, say $A_{2} B_{i}, A_{2} B_{j}, A_{2} B_{k}$, have to be of the same color, let it w.l.o.g. be red. Then $A_{1} B_{i}, A_{1} B_{j}, A_{1} B_{k}$ must be green. At least one of the sides of triangle $B_{i} B_{j} B_{k}$, say $B_{i} B_{j}$, must be an edge of the prism. Then looking at the triangles $A_{1} B_{i} B_{j}$ and $A_{2} B_{i} B_{j}$ we deduce that $B_{i} B_{j}$ can be neither green nor red, which is a contradiction. Hence all five edges of the pentagon $A_{1} A_{2} A_{3} A_{4} A_{5}$ have the same color. Similarly, all five edges of $B_{1} B_{2} B_{3} B_{4} B_{5}$ have the same color. We now show that the two colors are the same. Assume otherwise, i.e., that w.l.o.g. the $A$ edges are painted red and the $B$ edges green. Let us call segments of the form $A_{i} B_{j}$ diagonal ( $i$ and $j$ may be equal). We now count the diagonal segments by grouping the red segments based on their $A$ point, and the green segments based on their $B$ point. As above, the assumption that three of $A_{i} B_{j}$ for fixed $i$ are red leads to a contradiction. Hence at most two diagonal segments out of each $A_{i}$ may be red, which counts up to at most 10 red segments. Similarly, at most 10 diagonal segments can be green. But then we can paint at most 20 diagonal segments out of 25 , which is a contradiction. Hence all edges in the pentagons $A_{1} A_{2} A_{3} A_{4} A_{5}$ and $B_{1} B_{2} B_{3} B_{4} B_{5}$ have the same color. +5. Let $A=\{x \mid(x, y) \in M\}$ and $B=\{y \mid(x, y) \in M$. Then $A$ and $B$ are disjoint and hence + +$$ +|M| \leq|A| \cdot|B| \leq \frac{(|A|+|B|)^{2}}{4} \leq\left[\frac{n^{2}}{4}\right] +$$ + +These cardinalities can be achieved for $M=\{(a, b) \mid a=1,2, \ldots,[n / 2]$, $b=[n / 2]+1, \ldots, n\}$. +6. Setting $q=x^{2}+x-p$, the given equation becomes + +$$ +\sqrt{(x+1)^{2}-2 q}+\sqrt{(x+2)^{2}-q}=\sqrt{(2 x+3)^{2}-3 q} . +$$ + +Taking squares of both sides we get $2 \sqrt{\left((x+1)^{2}-2 q\right)\left((x+2)^{2}-q\right)}=$ $2(x+1)(x+2)$. Taking squares again we get + +$$ +q\left(2 q-2(x+2)^{2}-(x+1)^{2}\right)=0 +$$ + +If $2 q=2(x+2)^{2}+(x+1)^{2}$, at least one of the expressions under the three square roots in (1) is negative, and in that case the square root is not well-defined. Thus, we must have $q=0$. +Now (1) is equivalent to $|x+1|+|x+2|=|2 x+3|$, which holds if and only if $x \notin(-2,-1)$. The number of real solutions $x$ of $q=x^{2}+x-p=0$ which are not in the interval $(-2,-1)$ is zero if $p<-1 / 4$, one if $p=-1 / 4$ or $02 / n^{2}$ is fixed, then $a b=\left(s^{2}-(a-b)^{2}\right) / 4$ is minimized when $|a-b|$ is maximized, i.e., when $b=1 / n^{2}$. Hence $y_{1} y_{2} \cdots y_{n}$ is minimal when $y_{2}=y_{3}=\cdots=y_{n}=1 / n^{2}$. Then $y_{1}=\left(n^{2}-n+1\right) / n^{2}$ and therefore $P_{\min }=\sqrt{n^{2}-n+1} / n^{n}$. +12. The first criterion ensures that all sets in an $S$-family are distinct. Since the number of different families of subsets is finite, $h$ has to exist. In fact, we will show that $h=11$. First of all, if there exists $X \in F$ such that $|X| \geq 5$, then by (3) there exists $Y \in F$ such that $X \cup Y=R$. In this case $|F|$ is at most 2. Similarly, for $|X|=4$, for the remaining two elements either there exists a subset in $F$ that contains both, in which case we obtain the previous case, or there exist different $Y$ and $Z$ containing them, in which case $X \cup Y \cup Z=R$, which must not happen. Hence we can assume $|X| \leq 4$ for all $X \in F$. +Assume $|X|=1$ for some $X$. In that case other sets must not contain that subset and hence must be contained in the remaining 5 -element subset. These elements must not be subsets of each other. From elementary combinatorics, the largest number of subsets of a 5 -element set of which none is subset of another is $\binom{5}{2}=10$. This occurs when we take all 2-element subsets. These subsets also satisfy (2). Hence $|F|_{\max }=11$ in this case. Otherwise, let us assume $|X|=3$ for some $X$. Let us define the following families of subsets: $G=\{Z=Y \backslash X \mid Y \in F\}$ and $H=\{Z=Y \cap X \mid Y \in$ $F\}$. Then no two sets in $G$ must complement each other in $R \backslash X$, and $G$ must cover this set. Hence $G$ contains exactly the sets of each of the remaining 3 elements. For each element of $G$ no two sets in $H$ of which one is a subset of another may be paired with it. There can be only 3 such subsets selected within a 3 -element set $X$. Hence the number of remaining sets is smaller than $3 \cdot 3=9$. Hence in this case $|F|_{\max }=10$. + +In the remaining case all subsets have two elements. There are $\binom{6}{2}=15$ of them. But for every three that complement each other one must be discarded; hence the maximal number for $F$ in this case is $2 \cdot 15 / 3=10$. It follows that $h=11$. +13. From elementary trigonometry we have $\sin 3 t=3 \sin t-4 \sin ^{3} t$. Hence, if we denote $y=\sin 20^{\circ}$, we have $\sqrt{3} / 2=\sin 60^{\circ}=3 y-4 y^{3}$. Obviously $00$ for $0 \leq x<1 / 2$. Now the desired inequality $\frac{20}{60}=\frac{1}{3}<\sin 20^{\circ}<\frac{21}{60}=\frac{7}{20}$ follows from + +$$ +f\left(\frac{1}{3}\right)<\frac{\sqrt{3}}{2}0$ for $x>1$, the function $f(x)$ takes its maximum at a point $x$ for which $f^{\prime}(x)=(1-\ln x) / x^{2}=0$. Hence + +$$ +\max f(x)=f(e)=e^{1 / e} . +$$ + +It follows that the set of values of $f(x)$ for $x \in \mathbb{R}^{+}$is the interval $\left(-\infty, e^{1 / e}\right)$, and consequently the desired set of bases $a$ of logarithms is $(0,1) \cup\left(1, e^{1 / e}\right]$. +15. We note that +$\sum_{i=1}^{5} i\left(a-i^{2}\right)^{2} x_{i}=a^{2} \sum_{i=1}^{5} i x_{i}-2 a \sum_{i=1}^{5} i^{3} x_{i}+\sum_{i=1}^{5} i^{5} x_{i}=a^{2} \cdot a-2 a \cdot a^{2}+a^{3}=0$. +Since the terms in the sum on the left are all nonnegative, it follows that all the terms have to be 0 . Thus, either $x_{i}=0$ for all $i$, in which case $a=0$, or $a=j^{2}$ for some $j$ and $x_{i}=0$ for $i \neq j$. In this case, $x_{j}=a / j=j$. Hence, the only possible values of $a$ are $\{0,1,4,9,16,25\}$. +16. Obviously, no two elements of $F$ can be complements of each other. If one of the sets has one element, then the conclusion is trivial. If there exist two different 2-element sets, then they must contain a common element, which in turn must then be contained in all other sets. Thus we can assume that there exists at most one 2 -element subset of $K$ in $F$. Since there can be at most 6 subsets of more than 3 elements of a 5 -element set, it follows that at least 9 out of 10 possible 3 -element subsets of $K$ belong to $F$. Let us assume, without loss of generality, that all sets but $\{c, d, e\}$ belong to $F$. Then sets $\{a, b, c\},\{a, d, e\}$, and $\{b, c, d\}$ have no common element, which is a contradiction. Hence it follows that all sets have a common element. +17. Let $K, L$, and $M$ be intersections of $C Q$ and $B R, A R$ and $C P$, and $A Q$ and $B P$, respectively. Let $\angle X$ denote the angle of the hexagon $K Q M P L R$ at the vertex $X$, where $X$ is one of the six points. By an elementary calculation of angles we get +$\angle K=140^{\circ}, \angle L=130^{\circ}, \angle M=150^{\circ}, \angle P=100^{\circ}, \angle Q=95^{\circ}, \angle R=105^{\circ}$. +Since $\angle K B C=\angle K C B$, it follows that $K$ is on the symmetry line of $A B C$ through $A$. Analogous statements hold for $L$ and $M$. Let $K_{R}$ and $K_{Q}$ be points symmetric to $K$ with respect to $A R$ and $A Q$, respectively. Since $\angle A K_{Q} Q=\angle A K_{Q} K_{R}=70^{\circ}$ and $\angle A K_{R} R=\angle A K_{R} K_{Q}=70^{\circ}$, it follows that $K_{R}, R, Q$, and $K_{Q}$ are collinear. Hence $\angle Q R K=$ $2 \angle R-180^{\circ}$ and $\angle R Q K=2 \angle Q-$ $180^{\circ}$. We analogously get $\angle P R L=$ $2 \angle R-180^{\circ}, \angle R P L=2 \angle P-$ $180^{\circ}, \angle Q P M=2 \angle P-180^{\circ}$ and $\angle P Q M=2 \angle Q-180^{\circ}$. From these formulas we easily get $\angle R P Q=$ $60^{\circ}, \angle R Q P=75^{\circ}$, and $\angle Q R P=$ $45^{\circ}$. +18. Let us write all $a_{i}$ in binary representation. For $S \subseteq\{1,2, \ldots, m\}$ let us define $b(S)$ as the number in whose binary representation ones appear in exactly the slots where ones appear in all $a_{i}$ where $i \subseteq S$ and don't appear in any other $a_{i}$. Some $b(S)$, including $b(\emptyset)$, will equal 0 , and hence there are fewer than $2^{m}$ different positive $b(S)$. We note that no two positive $b\left(S_{1}\right)$ and $b\left(S_{2}\right)\left(S_{1} \neq S_{2}\right)$ have ones in the same decimal places. Hence sums of distinct $b(S)$ 's are distinct. Moreover + +$$ +a_{i}=\sum_{i \in S} b(S) +$$ + +and hence the positive $b(S)$ are indeed the numbers $b_{1}, \ldots, b_{n}$ whose existence we had to prove. +19. Let us define $i_{j}$ for two positive integers $i$ and $j$ in the following way: $i_{1}=i$ and $i_{j+1}=i^{i_{j}}$ for all positive integers $j$. Thus we must find the smallest $m$ such that $100_{m}>3_{100}$. Since $100_{1}=100>27=3_{2}$, we inductively have $100_{j}=10^{100_{j-1}}>3^{100_{j-1}}>3^{3_{j}}=3_{j+1}$ and hence $m \leq 99$. We now prove that $m=99$ by proving $100_{98}<3_{100}$. We note that $\left(100_{1}\right)^{2}=10^{4}<27^{4}=3^{12}<3^{27}=3_{3}$. We also note for $d>12$ (which trivially holds for all $d=100_{i}$ ) that if $c>d^{2}$, then we have + +$$ +3^{c}>3^{d^{2}}>3^{12 d}=\left(3^{12}\right)^{d}>10000^{d}=\left(100^{d}\right)^{2} +$$ + +Hence from $3_{3}>\left(100_{1}\right)^{2}$ it inductively follows that $3_{j}>\left(100_{j-2}\right)^{2}>$ $100_{j-2}$ and hence that $100_{99}>3_{100}>100_{98}$. Hence $m=99$. +20. Let $x_{k}=\max \left\{x_{1}, x_{2}, \ldots, x_{n}\right\}$. Then $x_{i} x_{i+1} \leq x_{i} x_{k}$ for $i=1,2, \ldots, k-1$ and $x_{i} x_{i+1} \leq x_{k} x_{i+1}$ for $i=k, \ldots, n-1$. Summing up these inequalities for $i=1,2, \ldots, n-1$ we obtain + +$$ +\sum_{i=1}^{n-1} \leq x_{k}\left(x_{1}+\cdots+x_{k-1}+x_{k+1}+\cdots+x_{n}\right)=x_{k}\left(a-x_{k}\right) \leq \frac{a^{2}}{4} +$$ + +We note that the value $a^{2} / 4$ is attained for $x_{1}=x_{2}=a / 2$ and $x_{3}=\cdots=$ $x_{n}=0$. Hence $a^{2} / 4$ is the required maximum. +21. Let $f(n)$ be the number of different ways $n \in \mathbb{N}$ can be expressed as $x^{2}+y^{3}$ where $x, y \in\left\{0,1, \ldots, 10^{6}\right\}$. Clearly $f(n)=0$ for $n<0$ or $n>10^{12}+10^{18}$. The first equation can be written as $x^{2}+t^{3}=y^{2}+z^{3}=n$, whereas the second equation can be written as $x^{2}+t^{3}=n+1, y^{2}+z^{3}=n$. Hence we obtain the following formulas for $M$ and $N$ : + +$$ +M=\sum_{i=0}^{m} f(i)^{2}, \quad N=\sum_{i=0}^{m-1} f(i) f(i+1) . +$$ + +Using the AM-GM inequality we get + +$$ +\begin{aligned} +N & =\sum_{i=0}^{m-1} f(i) f(i+1) \\ +& \leq \sum_{i=0}^{m-1} \frac{f(i)^{2}+f(i+1)^{2}}{2}=\frac{f(0)^{2}}{2}+\sum_{i=1}^{m-1} f(i)^{2}+\frac{f(m)^{2}}{2}0$. This completes our proof. +22. Let the centers of the two circles be denoted by $O$ and $O_{1}$ and their respective radii by $r$ and $r_{1}$, and let the positions of the points on the circles at time $t$ be denoted by $M(t)$ and $N(t)$. Let $Q$ be the point such that $O A O_{1} Q$ is a parallelogram. We will show that $Q$ is the point $P$ we are looking for, i.e., that $Q M(t)=$ $Q N(t)$ for all $t$. We note that $O Q=$ $O_{1} A=r_{1}, O_{1} Q=O A=r$ and +![](https://cdn.mathpix.com/cropped/2024_11_18_8e985d6b9c83aa3e9d0eg-449.jpg?height=353&width=469&top_left_y=1504&top_left_x=850) +$\angle Q O A=\angle Q O_{1} A=\phi$. Since the two points return to $A$ at the same time, it follows that $\angle M(t) O A=\angle N(t) O_{1} A=\omega t$. Therefore $\angle Q O M(t)=$ $\angle Q O_{1} N(t)=\phi+\omega t$, from which it follows that $\triangle Q O M(t) \cong \triangle Q O_{1} N(t)$. Hence $Q M(t)=Q N(t)$, as we claimed. +23. It is easily verified that no solutions exist for $n \leq 8$. Let us now assume that $n>8$. We note that $2^{8}+2^{11}+2^{n}=2^{8} \cdot\left(9+2^{n-8}\right)$. Hence $9+2^{n-8}$ +must also be a square, say $9+2^{n-8}=x^{2}, x \in \mathbb{N}$, i.e., $2^{n-8}=x^{2}-9=$ $(x-3)(x+3)$. Thus $x-3$ and $x+3$ are both powers of 2 , which is possible only for $x=5$ and $n=12$. Hence, $n=12$ is the only solution. +24. Clearly $O$ is the midpoint of $B C$. Let $M$ and $N$ be the points of tangency of the circle with $A B$ and $A C$, respectively, and let $\angle B A C=2 \varphi$. Then $\angle B O M=\angle C O N=\varphi$. +Let us assume that $P Q$ touches the circle in $X$. If we set $\angle P O M=$ $\angle P O X=x$ and $\angle Q O N=\angle Q O X=y$, then $2 x+2 y=\angle M O N=$ $180^{\circ}-2 \varphi$, i.e., $y=90^{\circ}-\varphi-x$. It follows that $\angle O Q C=180^{\circ}-\angle Q O C-$ $\angle O C Q=180^{\circ}-(\varphi+y)-\left(90^{\circ}-\varphi\right)=90^{\circ}-y=x+\varphi=\angle B O P$. Hence the triangles $B O P$ and $C Q O$ are similar, and consequently $B P \cdot C Q=$ $B O \cdot C O=(B C / 2)^{2}$. +Conversely, let $B P \cdot C Q=(B C / 2)^{2}$ and let $Q^{\prime}$ be the point on $(A C)$ such that $P Q^{\prime}$ is tangent to the circle. Then $B P \cdot C Q^{\prime}=(B C / 2)^{2}$, which implies $Q \equiv Q^{\prime}$. +25. Let us first look for such a point $R$ on a line $l$ in $\pi$ going through $P$. Let $\angle Q P R=2 \theta$. Consider a point $Q^{\prime}$ on $l$ such that $Q^{\prime} P=Q P$. Then we have + +$$ +\frac{Q P+P R}{Q R}=\frac{R Q^{\prime}}{Q R}=\frac{\sin Q^{\prime} Q R}{\sin Q Q^{\prime} R} +$$ + +Since $Q Q^{\prime} P$ is fixed, the maximal value of the expression occurs when $\angle Q Q^{\prime} R=90^{\circ}$. In this case $(Q P+P R) / Q R=1 / \sin \theta$. Looking at all possible lines $l$, we see that $\theta$ is minimized when $l$ equals the projection of $P Q$ onto $\pi$. Hence, the point $R$ is the intersection of the projection of $P Q$ onto $\pi$ and the plane through $Q$ perpendicular to $P Q$. +26. Let us assume that $f(x+y)=f(x)+f(y)$ for all reals. In this case we trivially apply the equation to get $f(x+y+x y)=f(x+y)+f(x y)=$ $f(x)+f(y)+f(x y)$. Hence the equivalence is proved in the first direction. Now let us assume that $f(x+y+x y)=f(x)+f(y)+f(x y)$ for all reals. Plugging in $x=y=0$ we get $f(0)=0$. Plugging in $y=-1$ we get $f(x)=-f(-x)$. Plugging in $y=1$ we get $f(2 x+1)=2 f(x)+f(1)$ and hence $f(2(u+v+u v)+1)=2 f(u+v+u v)+f(1)=2 f(u v)+$ $2 f(u)+2 f(v)+f(1)$ for all real $u$ and $v$. On the other hand, plugging in $x=u$ and $y=2 v+1$ we get $f(2(u+v+u v)+1)=f(u+(2 v+$ 1) $+u(2 v+1))=f(u)+2 f(v)+f(1)+f(2 u v+u)$. Hence it follows that $2 f(u v)+2 f(u)+2 f(v)+f(1)=f(u)+2 f(v)+f(1)+f(2 u v+u)$, i.e., + +$$ +f(2 u v+u)=2 f(u v)+f(u) +$$ + +Plugging in $v=-1 / 2$ we get $0=2 f(-u / 2)+f(u)=-2 f(u / 2)+f(u)$. Hence, $f(u)=2 f(u / 2)$ and consequently $f(2 x)=2 f(x)$ for all reals. Now (1) reduces to $f(2 u v+u)=f(2 u v)+f(u)$. Plugging in $u=y$ and $x=2 u v$, we obtain $f(x)+f(y)=f(x+y)$ for all nonzero reals $x$ and $y$. Since $f(0)=0$, it trivially holds that $f(x+y)=f(x)+f(y)$ when one of $x$ and $y$ is 0 . + +### 4.22 Solutions to the Shortlisted Problems of IMO 1981 + +1. Assume that the set $\{a-n+1, a-n+2, \ldots, a\}$ of $n$ consecutive numbers satisfies the condition $a \mid \operatorname{lcm}[a-n+1, \ldots, a-1]$. Let $a=p_{1}^{\alpha_{1}} p_{2}^{\alpha_{2}} \ldots p_{r}^{\alpha_{r}}$ be the canonic representation of $a$, where $p_{1}0$. Then for each $j=1,2, \ldots, r$, there exists $m, m=$ $1,2, \ldots, n-1$, such that $p_{j}^{\alpha_{j}} \mid a-m$, i.e., such that $p_{j}^{\alpha_{j}} \mid m$. Thus $p_{j}^{\alpha_{j}} \leq$ $n-1$. If $r=1$, then $a=p_{1}^{\alpha_{1}} \leq n-1$, which is impossible. Therefore $r \geq 2$. But then there must exist two distinct prime numbers less than $n$; hence $n \geq 4$. +For $n=4$, we must have $p_{1}^{\alpha_{1}}, p_{2}^{\alpha_{2}} \leq 3$, which leads to $p_{1}=2, p_{2}=3$, $\alpha_{1}=\alpha_{2}=1$. Therefore $a=6$, and $\{3,4,5,6\}$ is a unique set satisfying the condition of the problem. +For every $n \geq 5$ there exist at least two such sets. In fact, for $n=5$ we easily find two sets: $\{2,3,4,5,6\}$ and $\{8,9,10,11,12\}$. Suppose that $n \geq 6$. Let $r, s, t$ be natural numbers such that $2^{r} \leq n-1<2^{r+1}$, $3^{s} \leq n-1<3^{s+1}, 5^{t} \leq n-1<5^{t+1}$. Taking $a=2^{r} \cdot 3^{s}$ and $a=2^{r} \cdot 5^{t}$ we obtain two distinct sets with the required property. Thus the answers are (a) $n \geq 4$ and (b) $n=4$. +2. Lemma. Let $E, F, G, H, I$, and $K$ be points on edges $A B, B C, C D, D A$, $A C$, and $B D$ of a tetrahedron. Then there is a sphere that touches the edges at these points if and only if + +$$ +\begin{aligned} +& A E=A H=A I, \quad B E=B F=B K, \\ +& C F=C G=C I, \quad D G=D H=D K . +\end{aligned} +$$ + +Proof. The "only if" side of the equivalence is obvious. +We now assume (*). Denote by $\epsilon, \phi, \gamma, \eta, \iota$, and $\kappa$ planes through $E, F, G, H, I, K$ perpendicular to $A B, B C, C D, D A, A C$ and $B D$ respectively. Since the three planes $\epsilon, \eta$, and $\iota$ are not mutually parallel, they intersect in a common point $O$. Clearly, $\triangle A E O \cong$ +![](https://cdn.mathpix.com/cropped/2024_11_18_8e985d6b9c83aa3e9d0eg-451.jpg?height=324&width=496&top_left_y=1374&top_left_x=845) +$\triangle A H O \cong \triangle A I O$; hence $O E=O H=O I=r$, and the sphere $\sigma(O, r)$ is tangent to $A B, A D, A C$. +To prove that $\sigma$ is also tangent to $B C, C D, B D$ it suffices to show that planes $\phi, \gamma$, and $\kappa$ also pass through $O$. Without loss of generality we can prove this for just $\phi$. By the conditions for $E, F, I$, these are exactly the points of tangency of the incircle of $\triangle A B C$ and its sides, and if $S$ is the incenter, then $S E \perp A B, S F \perp B C, S I \perp A C$. Hence $\epsilon, \iota$, and $\phi$ all pass through $S$ and are perpendicular to the plane $A B C$, and consequently all share the line $l$ through $S$ perpendicular to $A B C$. + +Since $l=\epsilon \cap \iota$, the point $O$ will be situated on $l$, and hence $\phi$ will also contain $O$. This completes our proof of the lemma. +Let $A H=A E=x, B E=B F=y, C F=C G=z$, and $D G=D H=w$. If the sphere is also tangent to $A C$ at some point $I$, then $A I=x$ and $I C=z$. Using the stated lemma it suffices to prove that if $A C=x+z$, then $B D=y+w$. +Let $E F=F G=G H=H I=t, \angle B A D=\alpha, \angle A B C=\beta, \angle B C D=\gamma$, and $\angle A D C=\delta$. We get + +$$ +t^{2}=E H^{2}=A E^{2}+A H^{2}-2 \cdot A E \cdot A H \cos \alpha=2 x^{2}(1-\cos \alpha) +$$ + +We similarly conclude that $t^{2}=2 y^{2}(1-\cos \beta)=2 z^{2}(1-\cos \gamma)=2 w^{2}(1-$ $\cos \delta$ ). Further, using that $A B=x+y, B C=y+z, \cos \beta=1-t^{2} / 2 y^{2}$, we obtain + +$$ +A C^{2}=A B^{2}+B C^{2}-2 A B \cdot B C \cos \beta=(x-z)^{2}+t^{2}\left(\frac{x}{y}+1\right)\left(\frac{z}{y}+1\right) +$$ + +Analogously, from the triangle $A D C$ we get $A C^{2}=(x-z)^{2}+t^{2}(x / w+$ $1)(z / w+1)$, which gives $(x / y+1)(z / y+1)=(x / w+1)(z / w+1)$. Since $f(s)=(x / s+1)(z / s+1)$ is a decreasing function in $s$, it follows that $y=w$; similarly $x=z$. +Hence $C F=C G=x$ and $D G=D H=y$. Hence $A C \| E F$ and $A C: t=$ $A C: E F=A B: E B=(x+y): y$; i.e., $A C=t(x+y) / y$. Similarly, from the triangle $A B D$, we get that $B D=t(x+y) / x$. Hence if $A C=x+z=2 x$, it follows that $2 x=t(x+y) / y \Rightarrow 2 x y=t(x+y) \Rightarrow B D=t(x+y) / x=$ $2 y=y+w$. This completes the proof. +Second solution. Without loss of generality, assume that $E F=2$. Consider the Cartesian system in which points $O, E, F, G, H$ respectively have coordinates $(0,0,0),(-1,-1, a),(1,-1, a),(1,1, a),(-1,1, a)$. Line $A H$ is perpendicular to $O H$ and $A E$ is perpendicular to $O E$; hence from Pythagoras's theorem $A O^{2}=A H^{2}+H O^{2}=A E^{2}+E O^{2}=A E^{2}+H O^{2}$, which implies $A H=A E$. Therefore the $y$-coordinate of $A$ is zero; analogously the $x$-coordinates of $B$ and $D$ and the $y$-coordinate of $C$ are 0 . Let $A$ have coordinates $\left(x_{0}, 0, z_{1}\right)$ : then $\overrightarrow{E A}\left(x_{0}+1,1, z_{1}-a\right) \perp \overrightarrow{E O}(1,1,-a)$, i.e., $\overrightarrow{E A} \cdot \overrightarrow{E O}=x_{0}+2+a\left(a-z_{1}\right)=0$. Similarly, for $B\left(0, y_{0}, z_{2}\right)$ we have $y_{0}+2+a\left(a-z_{2}\right)=0$. This gives us + +$$ +z_{1}=\frac{x_{0}+a^{2}+2}{a}, \quad z_{2}=\frac{y_{0}+a^{2}+2}{a} +$$ + +We haven't used yet that $A\left(x_{0}, 0, z_{1}\right), E(-1,-1, a)$ and $B\left(0, y_{0}, z_{2}\right)$ are collinear, so let $A^{\prime}, B^{\prime}, E^{\prime}$ be the feet of perpendiculars from $A, B, E$ to the plane $x y$. The line $A^{\prime} B^{\prime}$, given by $y_{0} x+x_{0} y=x_{0} y_{0}, z=0$, contains the point $E^{\prime}(-1,-1,0)$, from which we obtain + +$$ +\left(x_{0}+1\right)\left(y_{0}+1\right)=1 +$$ + +In the same way, from the points $B$ and $C$ we get relations similar to (1) and (2) and conclude that $C$ has the coordinates $C\left(-x_{0}, 0, z_{1}\right)$. Similarly we get $D\left(0,-y_{0}, z_{2}\right)$. The condition that $A C$ is tangent to the sphere $\sigma(O, O E)$ is equivalent to $z_{1}=\sqrt{a^{2}+2}$, i.e., to $x_{0}=a \sqrt{a^{2}+2}-\left(a^{2}+2\right)$. But then (2) implies that $y_{0}=-a \sqrt{a^{2}+2}-\left(a^{2}+2\right)$ and $z_{2}=-\sqrt{a^{2}+2}$, which means that the sphere $\sigma$ is tangent to $B D$ as well. This finishes the proof. +3. Denote $\max (a+b+c, b+c+d, c+d+e, d+e+f, e+f+g)$ by $p$. We have + +$$ +(a+b+c)+(c+d+e)+(e+f+g)=1+c+e \leq 3 p +$$ + +which implies that $p \geq 1 / 3$. However, $p=1 / 3$ is achieved by taking $(a, b, c, d, e, f, g)=(1 / 3,0,0,1 / 3,0,0,1 / 3)$. Therefore the answer is $1 / 3$. +Remark. In fact, one can prove a more general statement in the same way. Given positive integers $n, k, n \geq k$, if $a_{1}, a_{2}, \ldots, a_{n}$ are nonnegative real numbers with sum 1 , then the minimum value of $\max _{i=1, \ldots, n-k+1}\left\{a_{i}+\right.$ $\left.a_{i+1}+\cdots+a_{i+k-1}\right\}$ is $1 / r$, where $r$ is the integer with $k(r-1)0$ is an integer depending on $n$. By (1), this is equivalent to $a\left(\alpha^{n}-(-1)^{n} \alpha^{-n}\right)+$ $b\left(\alpha^{n+1}+(-1)^{n} \alpha^{-n-1}\right)=\alpha^{k_{n}}-(-1)^{k_{n}} \alpha^{-k_{n}}$, i.e., + +$$ +\alpha^{k_{n}-n}=a+b \alpha-\alpha^{-2 n}(-1)^{n}\left(a-b \alpha^{-1}-(-\alpha)^{n-k_{n}}\right) \rightarrow a+b \alpha +$$ + +as $n \rightarrow \infty$. Hence, since $k_{n}$ is an integer, $k_{n}-n$ must be constant from some point on: $k_{n}=n+k$ and $\alpha^{k}=a+b \alpha$. Then it follows from (2) that $\alpha^{-k}=a-b \alpha^{-1}$, and from (1) we conclude that $a f_{n}+b f_{n+1}=$ $f_{k+n}$ holds for every $n$. Putting $n=1$ and $n=2$ in the previous relation and solving the obtained system of equations we get $a=f_{k-1}$, $b=f_{k}$. It is easy to verify that such $a$ and $b$ satisfy the conditions. +(b) As in (a), suppose that $u f_{n}^{2}+v f_{n+1}^{2}=f_{l_{n}}$ for all $n$. This leads to + +$$ +\begin{aligned} +u+v \alpha^{2}-\sqrt{5} \alpha^{l_{n}-2 n}= & 2(u-v)(-1)^{n} \alpha^{-2 n} \\ +& -\left(u \alpha^{-4 n}+v \alpha^{-4 n-2}+(-1)^{l_{n}} \sqrt{5} \alpha^{-l_{n}-2 n}\right) \\ +\rightarrow & 0 +\end{aligned} +$$ + +as $n \rightarrow \infty$. Thus $u+v \alpha^{2}=\sqrt{5} \alpha^{l_{n}-2 n}$, and $l_{n}-2 n=k$ is equal to a constant. Putting this into the above equation and multiplying by $\alpha^{2 n}$ we get $u-v \rightarrow 0$ as $n \rightarrow \infty$, i.e., $u=v$. Finally, substituting $n=1$ and $n=2$ in $u f_{n}^{2}+u f_{n+1}^{2}=f_{l_{n}}$ we easily get that the only possibility is $u=v=1$ and $k=1$. It is easy to verify that such $u$ and $v$ satisfy the conditions. +5. There are four types of small cubes upon disassembling: +(1) 8 cubes with three faces, painted black, at one corner; +(2) 12 cubes with two black faces, both at one edge; +(3) 6 cubes with one black face; +(4) 1 completely white cube. + +All cubes of type (1) must go to corners, and be placed in a correct way (one of three): for this step we have $3^{8} \cdot 8$ ! possibilities. Further, all cubes of type (2) must go in a correct way (one of two) to edges, admitting $2^{12} \cdot 12$ ! possibilities; similarly, there are $4^{6} \cdot 6$ ! ways for cubes of type (3), and 24 ways for the cube of type (4). Thus the total number of good reassemblings is $3^{8} 8!\cdot 2^{12} 12!\cdot 4^{6} 6!\cdot 24$, while the number of all possible reassemblings is $24^{27} \cdot 27!$. The desired probability is $\frac{3^{8} 8!\cdot 2^{12} 12!\cdot 4^{6} 6!\cdot 24}{24^{27} \cdot 27!}$. It is not necessary to calculate these numbers to find out that the blind man practically has no chance to reassemble the cube in a right way: in fact, the probability is of order $1.8 \cdot 10^{-37}$. +6. Assume w.l.o.g. that $n=\operatorname{deg} P \geq \operatorname{deg} Q$, and let $P_{0}=\left\{z_{1}, z_{2}, \ldots, z_{k}\right\}$, $P_{1}=\left\{z_{k+1}, z_{k+2}, \ldots z_{k+m}\right\}$. The polynomials $P$ and $Q$ match at $k+m$ points $z_{1}, z_{2}, \ldots, z_{k+m}$; hence if we prove that $k+m>n$, the result will follow. +By the assumption, +$P(x)=\left(x-z_{1}\right)^{\alpha_{1}} \cdots\left(x-z_{k}\right)^{\alpha_{k}}=\left(x-z_{k+1}\right)^{\alpha_{k+1}} \cdots\left(x-z_{k+m}\right)^{\alpha_{k+m}}+1$ +for some positive integers $\alpha_{1}, \ldots, \alpha_{k+m}$. Let us consider $P^{\prime}(x)$. As we know, it is divisible by $\left(x-z_{i}\right)^{\alpha_{i}-1}$ for $i=1,2, \ldots, k+m$; i.e., + +$$ +\prod_{i=1}^{k+m}\left(x-z_{i}\right)^{\alpha_{i}-1} \mid P^{\prime}(x) +$$ + +Therefore $2 n-k-m=\operatorname{deg} \prod_{i=1}^{k+m}\left(x-z_{i}\right)^{\alpha_{i}-1} \leq \operatorname{deg} P^{\prime}=n-1$, i.e., $k+m \geq n+1$, as we claimed. +7. We immediately find that $f(1,0)=f(0,1)=2$. Then $f(1, y+1)=$ $f(0, f(1, y))=f(1, y)+1$; hence $f(1, y)=y+2$ for $y \geq 0$. Next we find that $f(2,0)=f(1,1)=3$ and $f(2, y+1)=f(1, f(2, y))=f(2, y)+2$, from which $f(2, y)=2 y+3$. Particularly, $f(2,2)=7$. Further, $f(3,0)=$ $f(2,1)=5$ and $f(3, y+1)=f(2, f(3, y))=2 f(3, y)+3$. This gives by induction $f(3, y)=2^{y+3}-3$. For $y=3, f(3,3)=61$. Finally, from $f(4,0)=f(3,1)=13$ and $f(4, y+1)=f(3, f(4, y))=2^{f(4, y)+3}-3$, we conclude that + +$$ +f(4, y)=2^{2 y^{2}}-3 \quad(y+3 \text { twos }) +$$ + +8. Since the number $k, k=1,2, \ldots, n-r+1$, is the minimum in exactly $\binom{n-k}{r-1} r$-element subsets of $\{1,2, \ldots, n\}$, it follows that + +$$ +f(n, r)=\frac{1}{\binom{n}{r}} \sum_{k=1}^{n-r+1} k\binom{n-k}{r-1} +$$ + +To calculate the sum in the above expression, using the equality $\binom{r+j}{j}=$ $\sum_{i=0}^{j}\binom{r+i-1}{r-1}$, we note that + +$$ +\begin{aligned} +\sum_{k=1}^{n-r+1} k\binom{n-k}{r-1} & =\sum_{j=0}^{n-r}\left(\sum_{i=0}^{j}\binom{r+i-1}{r-1}\right) \\ +& =\sum_{j=0}^{n-r}\binom{r+j}{r}=\binom{n+1}{r+1}=\frac{n+1}{r+1}\binom{n}{r} . +\end{aligned} +$$ + +Therefore $f(n, r)=(n+1) /(r+1)$. +9. If we put $1+24 a_{n}=b_{n}^{2}$, the given recurrent relation becomes + +$$ +\frac{2}{3} b_{n+1}^{2}=\frac{3}{2}+\frac{b_{n}^{2}}{6}+b_{n}=\frac{2}{3}\left(\frac{3}{2}+\frac{b_{n}}{2}\right)^{2}, \quad \text { i.e., } \quad b_{n+1}=\frac{3+b_{n}}{2} +$$ + +where $b_{1}=5$. To solve this recurrent equation, we set $c_{n}=2^{n-1} b_{n}$. From (1) we obtain + +$$ +\begin{aligned} +c_{n+1} & =c_{n}+3 \cdot 2^{n-1}=\cdots=c_{1}+3\left(1+2+2^{2}+\cdots+2^{n-1}\right) \\ +& =5+3\left(2^{n}-1\right)=3 \cdot 2^{n}+2 +\end{aligned} +$$ + +Therefore $b_{n}=3+2^{-n+2}$ and consequently + +$$ +a_{n}=\frac{b_{n}^{2}-1}{24}=\frac{1}{3}\left(1+\frac{3}{2^{n}}+\frac{1}{2^{2 n-1}}\right)=\frac{1}{3}\left(1+\frac{1}{2^{n-1}}\right)\left(1+\frac{1}{2^{n}}\right) . +$$ + +10. It is easy to see that partitioning into $p=2 k$ squares is possible for $k \geq 2$ (Fig. 1). Furthermore, whenever it is possible to partition the square into $p$ squares, there is a partition of the square into $p+3$ squares: namely, in the partition into $p$ squares, divide one of them into four new squares. +![](https://cdn.mathpix.com/cropped/2024_11_18_8e985d6b9c83aa3e9d0eg-455.jpg?height=272&width=240&top_left_y=1678&top_left_x=393) + +Fig. 1 +![](https://cdn.mathpix.com/cropped/2024_11_18_8e985d6b9c83aa3e9d0eg-455.jpg?height=272&width=308&top_left_y=1678&top_left_x=919) + +Fig. 2 + +This implies that both $p=2 k$ and $p=2 k+3$ are possible if $k \geq 2$, and therefore all $p \geq 6$ are possible. + +On the other hand, partitioning the square into 5 squares is not possible. Assuming it is possible, one of its sides would be covered by exactly two squares, which cannot be of the same size (Fig. 2). The rest of the big square cannot be partitioned into three squares. Hence, the answer is $n=6$. +11. Let us denote the center of the semicircle by $O$, and $\angle A O B=2 \alpha$, $\angle B O C=2 \beta, A C=m, C E=n$. +We claim that $a^{2}+b^{2}+n^{2}+a b n=4$. Indeed, since $a=2 \sin \alpha, b=2 \sin \beta$, $n=2 \cos (\alpha+\beta)$, we have + +$$ +\begin{aligned} +a^{2} & +b^{2}+n^{2}+a b n \\ +& =4\left(\sin ^{2} \alpha+\sin ^{2} \beta+\cos ^{2}(\alpha+\beta)+2 \sin \alpha \sin \beta \cos (\alpha+\beta)\right) \\ +& =4+4\left(-\frac{\cos 2 \alpha}{2}-\frac{\cos 2 \beta}{2}+\cos (\alpha+\beta) \cos (\alpha-\beta)\right) \\ +& =4+4(\cos (\alpha+\beta) \cos (\alpha-\beta)-\cos (\alpha+\beta) \cos (\alpha-\beta))=4 +\end{aligned} +$$ + +Analogously, $c^{2}+d^{2}+m^{2}+c d m=4$. By adding both equalities and subtracting $m^{2}+n^{2}=4$ we obtain + +$$ +a^{2}+b^{2}+c^{2}+d^{2}+a b n+c d m=4 +$$ + +Since $n>c$ and $m>b$, the desired inequality follows. +12. We will solve the contest problem (in which $m, n \in\{1,2, \ldots, 1981\}$ ). For $m=1, n$ can be either 1 or 2 . If $m>1$, then $n(n-m)=m^{2} \pm 1>0$; hence $n-m>0$. Set $p=n-m$. Since $m^{2}-m p-p^{2}=m^{2}-p(m+p)=$ $-\left(n^{2}-n m-m^{2}\right)$, we see that $(m, n)$ is a solution of the equation if and only if $(p, m)$ is a solution too. Therefore, all the solutions of the equation are given as two consecutive members of the Fibonacci sequence + +$$ +1,1,2,3,5,8,13,21,34,55,89,144,233,377,610,987,1597,2584, \ldots +$$ + +So the required maximum is $987^{2}+1597^{2}$. +13. Lemma. For any polynomial $P$ of degree at most $n$, + +$$ +\sum_{i=0}^{n+1}(-1)^{i}\binom{n+1}{i} P(i)=0 +$$ + +Proof. We shall use induction on $n$. For $n=0$ it is trivial. Assume that it is true for $n=k$ and suppose that $P(x)$ is a polynomial of degree $k+1$. Then $P(x)-P(x+1)$ clearly has degree at most $k$; hence (1) gives + +$$ +\begin{aligned} +0 & =\sum_{i=0}^{k+1}(-1)^{i}\binom{k+1}{i}(P(i)-P(i+1)) \\ +& =\sum_{i=0}^{k+1}(-1)^{i}\binom{k+1}{i} P(i)+\sum_{i=1}^{k+2}(-1)^{i}\binom{k+1}{i-1} P(i) \\ +& =\sum_{i=0}^{k+2}(-1)^{i}\binom{k+2}{i} P(i) +\end{aligned} +$$ + +This completes the proof of the lemma. +Now we apply the lemma to obtain the value of $P(n+1)$. Since $P(i)=$ $\binom{n+1}{i}^{-1}$ for $i=0,1, \ldots, n$, we have + +$$ +0=\sum_{i=0}^{n+1}(-1)^{i}\binom{n+1}{i} P(i)=(-1)^{n+1} P(n+1)+ \begin{cases}1, & 2 \mid n ; \\ 0, & 2 \nmid n .\end{cases} +$$ + +It follows that $P(n+1)= \begin{cases}1, & 2 \mid n ; \\ 0, & 2 \nmid n .\end{cases}$ +14. We need the following lemma. + +Lemma. If a convex quadrilateral $P Q R S$ satisfies $P S=Q R$ and $\angle S P Q \geq$ $\angle R Q P$, then $\angle Q R S \geq \angle P S R$. +Proof. If the lines $P S$ and $Q R$ are parallel, then this quadrilateral is a parallelogram, and the statement is trivial. Otherwise, let $X$ be the point of intersection of lines $P S$ and $Q R$. +Assume that $\angle S P Q+\angle R Q P>180^{\circ}$. Then $\angle X P Q \leq \angle X Q P$ implies that $X P \geq X Q$, and consequently $X S \geq X R$. Hence, $\angle Q R S=$ $\angle X R S \geq \angle X S R=\angle P S R$. +Similarly, if $\angle S P Q+\angle R Q P<180^{\circ}$, then $\angle X P Q \geq \angle X Q P$, from which it follows that $X P \leq X Q$, and thus $X S \leq X R$; hence $\angle Q R S=$ $180^{\circ}-\angle X R S \geq 180^{\circ}-\angle X S R=\angle P S R$. +Now we apply the lemma to the quadrilateral $A B C D$. Since $\angle B \geq \angle C$ and $A B=C D$, it follows that $\angle C D A \geq \angle B A D$, which together with $\angle E D A=\angle E A D$ gives $\angle D \geq \angle A$. Thus $\angle A=\angle B=\angle C=\angle D$. Analogously, by applying the lemma to $B C D E$ we obtain $\angle E \geq \angle B$, and hence $\angle B=\angle C=\angle D=\angle E$. +15. Set $B C=a, C A=b, A B=c$, and denote the area of $\triangle A B C$ by $P$, and $a / P D+b / P E+c / P F$ by $S$. Since $a \cdot P D+b \cdot P E+c \cdot P F=2 P$, by the Cauchy-Schwarz inequality we have + +$$ +2 P S=(a \cdot P D+b \cdot P E+c \cdot P F)\left(\frac{a}{P D}+\frac{b}{P E}+\frac{c}{P F}\right) \geq(a+b+c)^{2} +$$ + +with equality if and only if $P D=P E=P F$, i.e., $P$ is the incenter of $\triangle A B C$. In that case, $S$ attains its minimum: + +$$ +S_{\min }=\frac{(a+b+c)^{2}}{2 P} +$$ + +16. The sequence $\left\{u_{n}\right\}$ is bounded, whatever $u_{1}$ is. Indeed, assume the opposite, and let $u_{m}$ be the first member of the sequence such that $\left|u_{m}\right|>\max \left\{2,\left|u_{1}\right|\right\}$. Then $\left|u_{m-1}\right|=\left|u_{m}^{3}-15 / 64\right|>\left|u_{m}\right|$, which is impossible. +Next, let us see for what values of $u_{m}, u_{m+1}$ is greater, equal, or smaller, respectively. +If $u_{m+1}=u_{m}$, then $u_{m}=u_{m+1}^{3}-15 / 64=u_{m}^{3}-15 / 64$; i.e., $u_{m}$ is a root of $x^{3}-x-15 / 64=0$. This equation factors as $(x+1 / 4)\left(x^{2}-x / 4-\right.$ $15 / 16)=0$, and hence $u_{m}$ is equal to $x_{1}=(1-\sqrt{61}) / 8, x_{2}=-1 / 4$, or $x_{3}=(1+\sqrt{61}) / 8$, and these are the only possible limits of the sequence. Each of $u_{m+1}>u_{m}, u_{m+1}0$ respectively. Thus the former is satisfied for $u_{m}$ in the interval $I_{1}=\left(-\infty, x_{1}\right)$ or $I_{3}=\left(x_{2}, x_{3}\right)$, while the latter is satisfied for $u_{m}$ in $I_{2}=\left(x_{1}, x_{2}\right)$ or $I_{4}=\left(x_{3}, \infty\right)$. Moreover, since the function $f(x)=\sqrt[3]{x+15 / 64}$ is strictly increasing with fixed points $x_{1}, x_{2}, x_{3}$, it follows that $u_{m}$ will never escape from the interval $I_{1}, I_{2}, I_{3}$, or $I_{4}$ to which it belongs initially. Therefore: +(1) if $u_{1}$ is one of $x_{1}, x_{2}, x_{3}$, the sequence $\left\{u_{m}\right\}$ is constant; +(2) if $u_{1} \in I_{1}$, then the sequence is strictly increasing and tends to $x_{1}$; +(3) if $u_{1} \in I_{2}$, then the sequence is strictly decreasing and tends to $x_{1}$; +(4) if $u_{1} \in I_{3}$, then the sequence is strictly increasing and tends to $x_{3}$; +(5) if $u_{1} \in I_{4}$, then the sequence is strictly decreasing and tends to $x_{3}$. +17. Let us denote by $S_{A}, S_{B}, S_{C}$ the centers of the given circles, where $S_{A}$ lies on the bisector of $\angle A$, etc. Then $S_{A} S_{B}\left\|A B, S_{B} S_{C}\right\| B C, S_{C} S_{A} \| C A$, so that the inner bisectors of the angles of triangle $A B C$ are also inner bisectors of the angles of $\triangle S_{A} S_{B} S_{C}$. These two triangles thus have a common incenter $S$, which is also the center of the homothety $\chi$ mapping $\triangle S_{A} S_{B} S_{C}$ onto $\triangle A B C$. +The point $O$ is the circumcenter of triangle $S_{A} S_{B} S_{C}$, and so is mapped by $\chi$ onto the circumcenter $P$ of $A B C$. This means that $O, P$, and the center $S$ of $\chi$ are collinear. +18. Let $C$ be the convex hull of the set of the planets: its border consists of parts of planes, parts of cylinders, and parts of the surfaces of some planets. These parts of planets consist exactly of all the invisible points; any point on a planet that is inside $C$ is visible. Thus it remains to show that the areas of all the parts of planets lying on the border of $C$ add up to the area of one planet. +As we have seen, an invisible part of a planet is bordered by some main spherical arcs, parallel two by two. Now fix any planet $P$, and translate these arcs onto arcs on the surface of $P$. All these arcs partition the surface of $P$ into several parts, each of which corresponds to the invisible part of +one of the planets. This correspondence is bijective, and therefore the statement follows. +19. Consider the partition of plane $\pi$ into regular hexagons, each having inradius 2. Fix one of these hexagons, denoted by $\gamma$. For any other hexagon $x$ in the partition, there exists a unique translation $\tau_{x}$ taking it onto $\gamma$. Define the mapping $\varphi: \pi \rightarrow \gamma$ as follows: If $A$ belongs to the interior of a hexagon $x$, then $\varphi(A)=\tau_{x}(A)$ (if $A$ is on the border of some hexagon, it does not actually matter where its image is). +The total area of the images of the union of the given circles equals $S$, while the area of the hexagon $\gamma$ is $8 \sqrt{3}$. Thus there exists a point $B$ of $\gamma$ that is covered at least $\frac{S}{8 \sqrt{3}}$ times, i.e., such that $\varphi^{-1}(B)$ consists of at least $\frac{S}{8 \sqrt{3}}$ distinct points of the plane that belong to some of the circles. For any of these points, take a circle that contains it. All these circles are disjoint, with total area not less than $\frac{\pi}{8 \sqrt{3}} S \geq 2 S / 9$. +Remark. The statement becomes false if the constant $2 / 9$ is replaced by any number greater than $1 / 4$. In that case a counterexample is, for example, a set of unit circles inside a circle of radius 2 covering a sufficiently large part of its area. + +### 4.23 Solutions to the Shortlisted Problems of IMO 1982 + +1. From $f(1)+f(1) \leq f(2)=0$ we obtain $f(1)=0$. Since $04000$. +(b) The sequence $x_{n}=1 / 2^{n}$ obviously satisfies the required condition. + +Second solution to part (a). For each $n \in \mathbb{N}$, let us find a constant $c_{n}$ such that the inequality $x_{0}^{2} / x_{1}+\cdots+x_{n-1}^{2} / x_{n} \geq c_{n} x_{0}$ holds for any sequence $x_{0} \geq x_{1} \geq \cdots \geq x_{n}>0$. +For $n=1$ we can take $c_{1}=1$. Assuming that $c_{n}$ exists, we have + +$$ +\frac{x_{0}^{2}}{x_{1}}+\left(\frac{x_{1}^{2}}{x_{2}}+\cdots+\frac{x_{n}^{2}}{x_{n+1}}\right) \geq \frac{x_{0}^{2}}{x_{1}}+c_{n} x_{1} \geq 2 \sqrt{x_{0}^{2} c_{n}}=x_{0} \cdot 2 \sqrt{c_{n}} +$$ + +Thus we can take $c_{n+1}=2 \sqrt{c_{n}}$. Then inductively $c_{n}=2^{2-1 / 2^{n-2}}$, and since $c_{n} \rightarrow 4$ as $n \rightarrow \infty$, the result follows. +Third solution. Since $\left\{x_{n}\right\}$ is decreasing, there exists $\lim _{n \rightarrow \infty} x_{n}=x \geq 0$. If $x>0$, then $x_{n-1}^{2} / x_{n} \geq x_{n} \geq x$ holds for each $n$, and the result is trivial. + +If otherwise $x=0$, then we note that $x_{n-1}^{2} / x_{n} \geq 4\left(x_{n-1}-x_{n}\right)$ for each $n$, with equality if and only if $x_{n-1}=2 x_{n}$. Hence + +$$ +\lim _{n \rightarrow \infty} \sum_{k=1}^{n} \frac{x_{k-1}^{2}}{x_{k}} \geq \lim _{n \rightarrow \infty} \sum_{k=1}^{n} 4\left(x_{k-1}-x_{k}\right)=4 x_{0}=4 +$$ + +Equality holds if and only if $x_{n-1}=2 x_{n}$ for all $n$, and consequently $x_{n}=1 / 2^{n}$. +4. Suppose that $a$ satisfies the requirements of the problem and that $x, q x$, $q^{2} x, q^{3} x$ are the roots of the given equation. Then $x \neq 0$ and we may assume that $|q|>1$, so that $|x|<|q x|<\left|q^{2} x\right|<\left|q^{3} x\right|$. Since the equation is symmetric, $1 / x$ is also a root and therefore $1 / x=q^{3} x$, i.e., $q=x^{-2 / 3}$. It follows that the roots are $x, x^{1 / 3}, x^{-1 / 3}, x^{-1}$. Now by Vieta's formula we have $x+x^{1 / 3}+x^{-1 / 3}+x^{-1}=a / 16$ and $x^{4 / 3}+x^{2 / 3}+2+x^{-2 / 3}+x^{-4 / 3}=$ $(2 a+17) / 16$. On setting $z=x^{1 / 3}+x^{-1 / 3}$ these equations become + +$$ +\begin{aligned} +z^{3}-2 z & =a / 16 \\ +\left(z^{2}-2\right)^{2}+z^{2}-2 & =(2 a+17) / 16 +\end{aligned} +$$ + +Substituting $a=16\left(z^{3}-2 z\right)$ in the second equation leads to $z^{4}-2 z^{3}-$ $3 z^{2}+4 z+15 / 16=0$. We observe that this polynomial factors as $(z+$ $3 / 2)(z-5 / 2)\left(z^{2}-z-1 / 4\right)$. Since $|z|=\left|x^{1 / 3}+x^{-1 / 3}\right| \geq 2$, the only viable value is $z=5 / 2$. Consequently $a=170$ and the roots are $1 / 8,1 / 2,2,8$. +5. We first observe that $\triangle A_{5} B_{4} A_{4} \cong$ $\triangle A_{3} B_{2} A_{2}$. Since $\angle A_{5} A_{3} A_{2}=90^{\circ}$, we have $\angle A_{2} B_{4} A_{4}=\angle A_{2} B_{4} A_{3}+$ $\angle A_{3} B_{4} A_{4}=\left(90^{\circ}-\angle B_{2} A_{2} A_{3}\right)+$ $\left(\angle B_{4} A_{5} A_{4}+\angle A_{5} A_{4} B_{4}\right)=90^{\circ}+$ $\angle B_{4} A_{5} A_{4}=120^{\circ}$. Hence $B_{4}$ belongs to the circle with center $A_{3}$ and radius $A_{3} A_{4}$, so $A_{3} A_{4}=A_{3} B_{4}$. +![](https://cdn.mathpix.com/cropped/2024_11_18_8e985d6b9c83aa3e9d0eg-461.jpg?height=311&width=428&top_left_y=1245&top_left_x=866) + +Thus $\lambda=A_{3} B_{4} / A_{3} A_{5}=A_{3} A_{4} / A_{3} A_{5}=1 / \sqrt{3}$. +6. Denote by $d(U, V)$ the distance between points or sets of points $U$ and $V$. For $P, Q \in L$ we shall denote by $L_{P Q}$ the part of $L$ between points $P$ and $Q$ and by $l_{P Q}$ the length of this part. Let us denote by $S_{i}(i=1,2,3,4)$ the vertices of $S$ and by $T_{i}$ points of $L$ such that $S_{i} T_{i} \leq 1 / 2$ in such a way that $l_{A_{0} T_{1}}$ is the least of the $l_{A_{0} T_{i}}$ 's, $S_{2}$ and $S_{4}$ are neighbors of $S_{1}$, and $l_{A_{0} T_{2}}A_{1} C$. Similarly, $C_{1} C>C_{1} A$. Hence the perpendicular bisector $l_{A C}$ of $A C$ separates points $A_{1}$ and $C_{1}$. Since $B_{1}, D_{1}$ lie on $l_{A C}$, this means that $A_{1}$ and $C_{1}$ are on opposite sides $B_{1} D_{1}$. Similarly one can show that $B_{1}$ and $D_{1}$ are on opposite sides of $A_{1} C_{1}$. +(b) Since $A_{2} B_{2} \perp C_{1} D_{1}$ and $C_{1} D_{1} \perp A B$, it follows that $A_{2} B_{2} \| A B$. Similarly $A_{2} C_{2}\left\|A C, A_{2} D_{2}\right\| A D, B_{2} C_{2}\left\|B C, B_{2} D_{2}\right\| B D$, and $C_{2} D_{2} \| C D$. Hence $\triangle A_{2} B_{2} C_{2} \sim \triangle A B C$ and $\triangle A_{2} D_{2} C_{2} \sim \triangle A D C$, and the result follows. +15. Let $a=k / n$, where $n, k \in \mathbb{N}, n \geq k$. Putting $t^{n}=s$, the given inequality becomes $\frac{1-t^{k}}{1-t^{n}} \leq\left(1+t^{n}\right)^{k / n-1}$, or equivalently + +$$ +\left(1+t+\cdots+t^{k-1}\right)^{n}\left(1+t^{n}\right)^{n-k} \leq\left(1+t+\cdots+t^{n-1}\right)^{n} +$$ + +This is clearly true for $k=n$. Therefore it is enough to prove that the lefthand side of the above inequality is an increasing function of $k$. We are led to show that $\left(1+t+\cdots+t^{k-1}\right)^{n}\left(1+t^{n}\right)^{n-k} \leq\left(1+t+\cdots+t^{k}\right)^{n}\left(1+t^{n}\right)^{n-k-1}$. This is equivalent to $1+t^{n} \leq A^{n}$, where $A=\frac{1+t+\cdots+t^{k}}{1+t+\cdots+t^{k-1}}$. But this easily follows, since + +$$ +\begin{aligned} +A^{n}-t^{n} & =(A-t)\left(A^{n-1}+A^{n-2} t+\cdots+t^{n-1}\right) \\ +& \geq(A-t)\left(1+t+\cdots+t^{n-1}\right)=\frac{1+t+\cdots+t^{n-1}}{1+t+\cdots+t^{k-1}} \geq 1 +\end{aligned} +$$ + +Remark. The original problem asked to prove the inequality for real $a$. +16. It is easy to verify that whenever $(x, y)$ is a solution of the equation $x^{3}-3 x y^{2}+y^{3}=n$, so are the pairs $(y-x,-x)$ and $(-y, x-y)$. No two of these three solutions are equal unless $x=y=n=0$. +Observe that $2981 \equiv 2(\bmod 9)$. Since $x^{3}, y^{3} \equiv 0, \pm 1(\bmod 9), x^{3}-$ $3 x y^{2}+y^{3}$ cannot give the remainder 2 when divided by 9 . Hence the above equation for $n=2981$ has no integer solutions. +17. Let $A$ be the origin of the Cartesian plane. Suppose that $B C: A C=k$ and that $(a, b)$ and $\left(a_{1}, b_{1}\right)$ are coordinates of the points $C$ and $C_{1}$, respectively. Then the coordinates of the point $B$ are $(a, b)+k(-b, a)=(a-k b, b+k a)$, +while the coordinates of $B_{1}$ are $\left(a_{1}, b_{1}\right)+k\left(b_{1},-a_{1}\right)=\left(a+k b_{1}, b_{1}-k a_{1}\right)$. Thus the lines $B C_{1}$ and $C B_{1}$ are given by the equations $\frac{x-a_{1}}{y-b_{1}}=\frac{x-(a-k b)}{y-(b+k a)}$ and $\frac{x-a}{y-b}=\frac{x-\left(a_{1}+k b_{1}\right)}{y-\left(b_{1}-k a_{1}\right)}$ respectively. After multiplying, these equations transform into the forms + +$$ +\begin{aligned} +B C_{1}: & k a x+k b y & =k a a_{1}+k b b_{1}+b a_{1}-a b_{1}-\left(b-b_{1}\right) x+\left(a-a_{1}\right) y \\ +C B_{1}: & k a_{1} x+k b_{1} y & =k a a_{1}+k b b_{1}+b a_{1}-a b_{1}-\left(b-b_{1}\right) x+\left(a-a_{1}\right) y . +\end{aligned} +$$ + +The coordinates $\left(x_{0}, y_{0}\right)$ of the point $M$ satisfy these equations, from which we deduce that $k a x_{0}+k b y_{0}=k a_{1} x_{0}+k b_{1} y_{0}$. This yields $\frac{x_{0}}{y_{0}}=-\frac{b_{1}-b}{a_{1}-a}$, implying that the lines $C C_{1}$ and $A M$ are perpendicular. +18. Set the coordinate system with the axes $x, y, z$ along the lines $l_{1}, l_{2}, l_{3}$ respectively. The coordinates $(a, b, c)$ of $M$ satisfy $a^{2}+b^{2}+c^{2}=R^{2}$, and so $S_{M}$ is given by the equation $(x-a)^{2}+(y-b)^{2}+(z-c)^{2}=R^{2}$. Hence the coordinates of $P_{1}$ are $(x, 0,0)$ with $(x-a)^{2}+b^{2}+c^{2}=R^{2}$, implying that either $x=2 a$ or $x=0$. Thus by the definition we obtain $x=2 a$. Similarly, the coordinates of $P_{2}$ and $P_{3}$ are $(0,2 b, 0)$ and $(0,0,2 c)$ respectively. Now, the centroid of $\triangle P_{1} P_{2} P_{3}$ has the coordinates $(2 a / 3,2 b / 3,2 c / 3)$. Therefore the required locus of points is the sphere with center $O$ and radius $2 R / 3$. +19. Let us set $x=m / n$. Since $f(x)=(m+n) / \sqrt{m^{2}+n^{2}}=(x+1) / \sqrt{1+x^{2}}$ is a continuous function of $x, f(x)$ takes all values between any two values of $f$; moreover, the corresponding $x$ can be rational. This completes the proof. + +Remark. Since $f$ is increasing for $x \geq 1,1 \leq x2^{n}$ cities. We shall prove that there is a round trip by at least one $A_{i}$ containing an odd number of stops. +For $n=1$ the statement is trivial, since one airline serves at least 3 cities and hence $P_{1} P_{2} P_{3} P_{1}$ is a round trip with 3 landings. We use induction on $n$, and assume that $n>1$. Suppose the contrary, that all round trips by $A_{n}$ consist of an even number of stops. Then we can separate the cities into two nonempty classes $Q=\left\{Q_{1}, \ldots, Q_{r}\right\}$ and $R=\left\{R_{1}, \ldots, R_{s}\right\}$ (where $r+s=N$ ), so that each flight by $A_{n}$ runs between a $Q$-city and an $R$-city. (Indeed, take any city $Q_{1}$ served by $A_{n}$; include each city linked to $Q_{1}$ by $A_{n}$ in $R$, then include in $Q$ each city linked by $A_{n}$ to any $R$-city, etc. Since all round trips are even, no contradiction can arise.) At least one of $r, s$ is larger than $2^{n-1}$, say $r>2^{n-1}$. But, only $A_{1}, \ldots, A_{n-1}$ run between cities in $\left\{Q_{1}, \ldots, Q_{r}\right\}$; hence by the induction hypothesis at least one of them flies a round trip with an odd number of landings, a contradiction. It only remains to notice that for $n=10,2^{n}=1024<1983$. +Remark. If there are $N=2^{n}$ cities, there is a schedule with $n$ airlines that contain no odd round trip by any of the airlines. Let the cities be $P_{k}$, $k=0, \ldots, 2^{n}-1$, and write $k$ in the binary system as an $n$-digit number $\overline{a_{1} \ldots a_{n}}$ (e.g., $1=(0 \ldots 001)_{2}$ ). Link $P_{k}$ and $P_{l}$ by $A_{i}$ if the $i$ th digits $k$ and $l$ are distinct but the first $i-1$ digits are the same. All round trips under $A_{i}$ are even, since the $i$ th digit alternates. +2. By definition, $\sigma(n)=\sum_{d \mid n} d=\sum_{d \mid n} n / d=n \sum_{d \mid n} 1 / d$, hence $\sigma(n) / n=$ $\sum_{d \mid n} 1 / d$. In particular, $\sigma(n!) / n!=\sum_{d \mid n!} 1 / d \geq \sum_{k=1}^{n} 1 / k$. It follows that the sequence $\sigma(n) / n$ is unbounded, and consequently there exist an infinite number of integers $n$ such that $\sigma(n) / n$ is strictly greater than $\sigma(k) / k$ for $k2(n-k)+1$. So we cannot begin with $x_{k+1}$ for any $k>0$. +Now assume that there is an equality for some $k$. There are two cases: +(i) Suppose $x_{1}+x_{2}+\cdots+x_{i} \leq 2 i(i=1, \ldots, k)$ and $x_{1}+\cdots+x_{k}=2 k+1$, $x_{1}+\cdots+x_{k+l} \leq 2(k+l)+1(1 \leq l \leq n-1-k)$. For $i \leq k-1$ we have $x_{i+1}+\cdots+x_{n}=2(n+1)-\left(x_{1}+\cdots+x_{i}\right)>2(n-i)+1$, so we cannot take $r=i$. If there is a $j \geq 1$ such that $x_{1}+x_{2}+\cdots+x_{k+j} \leq 2(k+j)$, then also $x_{k+j+1}+\cdots+x_{n}>2(n-k-j)+1$. If $(\forall j \geq 1) x_{1}+\cdots+x_{k+j}=$ $2(k+j)+1$, then $x_{n}=3$ and $x_{k+1}=\cdots=x_{n-1}=2$. In this case we directly verify that we cannot take $r=k+j$. However, we can also take $r=k$ : for $k+l \leq n-1, x_{k+1}+\cdots+x_{k+l} \leq 2(k+l)+1-(2 k+1)=2 l$, also $x_{k+1}+\cdots+x_{n}=2(n-k)+1$, and moreover $x_{1} \leq 2, x_{1}+x_{2} \leq 4, \ldots$. +(ii) Suppose $x_{1}+\cdots+x_{i} \leq 2 i(1 \leq i \leq n-2)$ and $x_{1}+\cdots+x_{n-1}=2 n-1$. Then we can obviously take $r=n-1$. On the other hand, for any +$1 \leq i \leq n-2, x_{i+1}+\cdots+x_{n-1}+x_{n}=\left(x_{1}+\cdots+x_{n-1}\right)-\left(x_{1}+\cdots+\right.$ $\left.x_{i}\right)+3>2(n-i)+1$, so we cannot take another $r \neq 0$. +7. Clearly, each $a_{n}$ is positive and $\sqrt{a_{n+1}}=\sqrt{a_{n}} \sqrt{a+1}+\sqrt{a_{n}+1} \sqrt{a}$. Notice that $\sqrt{a_{n+1}+1}=\sqrt{a+1} \sqrt{a_{n}+1}+\sqrt{a} \sqrt{a_{n}}$. Therefore + +$$ +\begin{aligned} +& (\sqrt{a+1}-\sqrt{a})\left(\sqrt{a_{n}+1}-\sqrt{a_{n}}\right) \\ +& \quad=\left(\sqrt{a+1} \sqrt{a_{n}+1}+\sqrt{a} \sqrt{a_{n}}\right)-\left(\sqrt{a_{n}} \sqrt{a+1}+\sqrt{a_{n}+1} \sqrt{a}\right) \\ +& \quad=\sqrt{a_{n+1}+1}-\sqrt{a_{n+1}} +\end{aligned} +$$ + +By induction, $\sqrt{a_{n+1}}-\sqrt{a_{n}}=(\sqrt{a+1}-\sqrt{a})^{n}$. Similarly, $\sqrt{a_{n+1}}+\sqrt{a_{n}}=$ $(\sqrt{a+1}+\sqrt{a})^{n}$. Hence, + +$$ +\sqrt{a_{n}}=\frac{1}{2}\left[(\sqrt{a+1}+\sqrt{a})^{n}-(\sqrt{a+1}-\sqrt{a})^{n}\right] +$$ + +from which the result follows. +8. Situations in which the condition of the statement is fulfilled are the following: +$S_{1}: N_{1}(t)=N_{2}(t)=N_{3}(t)$ +$S_{2}: N_{i}(t)=N_{j}(t)=h, N_{k}(t)=h+1$, where $(i, j, k)$ is a permutation of the set $\{1,2,3\}$. In this case the first student to leave must be from row $k$. This leads to the situation $S_{1}$. +$S_{3}: N_{i}(t)=h, N_{j}(t)=N_{k}(t)=h+1,((i, j, k)$ is a permutation of the set $\{1,2,3\})$. In this situation the first student leaving the room belongs to row $j$ (or $k$ ) and the second to row $k$ (or $j$ ). After this we arrive at the situation $S_{1}$. +Hence, the initial situation is $S_{1}$ and after each triple of students leaving the room the situation $S_{1}$ must recur. We shall compute the probability $P_{h}$ that from a situation $S_{1}$ with $3 h$ students in the room $(h \leq n)$ one arrives at a situation $S_{1}$ with $3(h-1)$ students in the room: + +$$ +P_{h}=\frac{(3 h) \cdot(2 h) \cdot h}{(3 h) \cdot(3 h-1) \cdot(3 h-2)}=\frac{3!h^{3}}{3 h(3 h-1)(3 h-2)} +$$ + +Since the room becomes empty after the repetition of $n$ such processes, which are independent, we obtain for the probability sought + +$$ +P=\prod_{h=1}^{n} P_{h}=\frac{(3!)^{n}(n!)^{3}}{(3 n)!} +$$ + +9. For any triangle of sides $a, b, c$ there exist 3 nonnegative numbers $x, y, z$ such that $a=y+z, b=z+x, c=x+y$ (these numbers correspond to the division of the sides of a triangle by the point of contact of the incircle). The inequality becomes + +$$ +(y+z)^{2}(z+x)(y-x)+(z+x)^{2}(x+y)(z-y)+(x+y)^{2}(y+z)(x-z) \geq 0 +$$ + +Expanding, we get $x y^{3}+y z^{3}+z x^{3} \geq x y z(x+y+z)$. This follows from Cauchy's inequality $\left(x y^{3}+y z^{3}+z x^{3}\right)(z+x+y) \geq(\sqrt{x y z}(x+y+z))^{2}$ with equality if and only if $x y^{3} / z=y z^{3} / x=z x^{3} / y$, or equivalently $x=y=z$, i.e., $a=b=c$. +10. Choose $P(x)=\frac{p}{q}\left((q x-1)^{2 n+1}+1\right), I=[1 / 2 q, 3 / 2 q]$. Then all the coefficients of $P$ are integers, and + +$$ +\left|P(x)-\frac{p}{q}\right|=\left|\frac{p}{q}(q x-1)^{2 n+1}\right| \leq\left|\frac{p}{q}\right| \frac{1}{2^{2 n+1}}, +$$ + +for $x \in I$. The desired inequality follows if $n$ is chosen large enough. +11. First suppose that the binary representation of $x$ is finite: $x=0, a_{1} a_{2} \ldots a_{n}$ $=\sum_{j=1}^{n} a_{j} 2^{-j}, a_{i} \in\{0,1\}$. We shall prove by induction on $n$ that + +$$ +f(x)=\sum_{j=1}^{n} b_{0} \ldots b_{j-1} a_{j}, \quad \text { where } b_{k}=\left\{\begin{array}{lr} +-b & \text { if } a_{k}=0 \\ +1-b & \text { if } a_{k}=1 +\end{array}\right. +$$ + +(Here $a_{0}=0$.) Indeed, by the recursion formula, +$a_{1}=0 \Rightarrow f(x)=b f\left(\sum_{j=1}^{n-1} a_{j+1} 2^{-j}\right)=b \sum_{j=1}^{n-1} b_{1} \ldots b_{j} a_{j+1}$ hence $f(x)=$ $\sum_{j=0}^{n-1} b_{0} \ldots b_{j} a_{j+1}$ as $b_{0}=b_{1}=b ;$ +$a_{1}=1 \Rightarrow f(x)=b+(1-b) f\left(\sum_{j=1}^{n-1} a_{j+1} 2^{-j}\right)=\sum_{j=0}^{n-1} b_{0} \ldots b_{j} a_{j+1}$, as $b_{0}=b, b_{1}=1-b$. +Clearly, $f(0)=0, f(1)=1, f(1 / 2)=b>1 / 2$. Assume $x=\sum_{j=0}^{n} a_{j} 2^{-j}$, and for $k \geq 2, v=x+2^{-n-k+1}, u=x+2^{-n-k}=(v+x) / 2$. Then $f(v)=$ $f(x)+b_{0} \ldots b_{n} b^{k-2}$ and $f(u)=f(x)+b_{0} \ldots b_{n} b^{k-1}>(f(v)+f(x)) / 2$. This means that the point $(u, f(u))$ lies above the line joining $(x, f(x))$ and $(v, f(v))$. By induction, every $(x, f(x))$, where $x$ has a finite binary expansion, lies above the line joining $(0,0)$ and $(1 / 2, b)$ if $0x$. For the second inequality, observe that + +$$ +\begin{aligned} +f(x)-x & =\sum_{j=1}^{\infty}\left(b_{0} \ldots b_{j-1}-2^{-j}\right) a_{j} \\ +& <\sum_{j=1}^{\infty}\left(b^{j}-2^{-j}\right) a_{j}<\sum_{j=1}^{\infty}\left(b^{j}-2^{-j}\right)=\frac{b}{1-b}-1=c . +\end{aligned} +$$ + +By continuity, these inequalities also hold for $x$ with infinite binary representations. +12. Putting $y=x$ in (1) we see that there exist positive real numbers $z$ such that $f(z)=z$ (this is true for every $z=x f(x)$ ). Let $a$ be any of them. + +Then $f\left(a^{2}\right)=f(a f(a))=a f(a)=a^{2}$, and by induction, $f\left(a^{n}\right)=a^{n}$. If $a>1$, then $a^{n} \rightarrow+\infty$ as $n \rightarrow \infty$, and we have a contradiction with (2). Again, $a=f(a)=f(1 \cdot a)=a f(1)$, so $f(1)=1$. Then, $a f\left(a^{-1}\right)=$ $f\left(a^{-1} f(a)\right)=f(1)=1$, and by induction, $f\left(a^{-n}\right)=a^{-n}$. This shows that $a \nless 1$. Hence, $a=1$. It follows that $x f(x)=1$, i.e., $f(x)=1 / x$ for all $x$. This function satisfies (1) and (2), so $f(x)=1 / x$ is the unique solution. +13. Given any coloring of the $3 \times 1983-2$ points of the axes, we prove that there is a unique coloring of $E$ having the given property and extending this coloring. The first thing to notice is that given any rectangle $R_{1}$ parallel to a coordinate plane and whose edges are parallel to the axes, there is an even number $r_{1}$ of red vertices on $R_{1}$. Indeed, let $R_{2}$ and $R_{3}$ be two other rectangles that are translated from $R_{1}$ orthogonally to $R_{1}$ and let $r_{2}, r_{3}$ be the numbers of red vertices on $R_{2}$ and $R_{3}$ respectively. Then $r_{1}+r_{2}$, $r_{1}+r_{3}$, and $r_{2}+r_{3}$ are multiples of 4 , so $r_{1}=\left(r_{1}+r_{2}+r_{1}+r_{3}-r_{2}-r_{3}\right) / 2$ is even. +Since any point of a coordinate plane is a vertex of a rectangle whose remaining three vertices lie on the corresponding axes, this determines uniquely the coloring of the coordinate planes. Similarly, the coloring of the inner points of the parallelepiped is completely determined. The solution is hence $2^{3 \times 1983-2}=2^{5947}$. +14. Let $T_{n}$ be the set of all nonnegative integers whose ternary representations consist of at most $n$ digits and do not contain a digit 2 . The cardinality of $T_{n}$ is $2^{n}$, and the greatest integer in $T_{n}$ is $11 \ldots 1=3^{0}+3^{1}+\cdots+3^{n-1}=$ $\left(3^{n}-1\right) / 2$. We claim that there is no arithmetic triple in $T_{n}$. To see this, suppose $x, y, z \in T_{n}$ and $2 y=x+z$. Then $2 y$ has only 0 's and 2's in its ternary representation, and a number of this form can be the sum of two integers $x, z \in T_{n}$ in only one way, namely $x=z=y$. But $\left|T_{10}\right|=2^{10}=1024$ and $\max T_{10}=\left(3^{10}-1\right) / 2=29524<30000$. Thus the answer is yes. +15. There is no such set. Suppose $M$ satisfies $(a)$ and $(b)$ and let $q_{n}=$ $|\{a \in M: a \leq n\}|$. Consider the differences $b-a$, where $a, b \in M$ and $10\sqrt{3} / 2 \cdot(\sqrt{n}-1) a +$$ + +Proof of the lemma. If a nonobtuse triangle with sides $a \geq b \geq c$ has a circumscribed circle of radius $R$, we have $R=a /(2 \sin \alpha) \leq a / \sqrt{3}$. Now we show that there exists a disk $D$ of radius $R$ containing $A=$ $\left\{P_{1}, \ldots, P_{n}\right\}$ whose border $C$ is such that $C \cap A$ is not included in an open semicircle, and hence contains either two diametrically opposite points and $R \leq b / 2$, or an acute-angled triangle and $R \leq b / \sqrt{3}$. +Among all disks whose borders pass through three points of $A$ and that contain all of $A$, let $D$ be the one of least radius. Suppose that $C \cap A$ is contained in an arc of central angle less than $180^{\circ}$, and that $P_{i}, P_{j}$ are its endpoints. Then there exists a circle through $P_{i}, P_{j}$ of smaller radius that contains $A$, a contradiction. Thus $D$ has the required property, and the assertion follows. +18. Let $\left(x_{0}, y_{0}, z_{0}\right)$ be one solution of $b c x+c a y+a b z=n$ (not necessarily nonnegative). By subtracting $b c x_{0}+c a y_{0}+a b z_{0}=n$ we get + +$$ +b c\left(x-x_{0}\right)+c a\left(y-y_{0}\right)+a b\left(z-z_{0}\right)=0 . +$$ + +Since $(a, b)=(a, c)=1$, we must have $a \mid x-x_{0}$ or $x-x_{0}=a s$. Substituting this in the last equation gives + +$$ +b c s+c\left(y-y_{0}\right)+b\left(z-z_{0}\right)=0 +$$ + +Since $(b, c)=1$, we have $b \mid y-y_{0}$ or $y-y_{0}=b t$. If we substitute this in the last equation we get $b c s+b c t+b\left(z-z_{0}\right)=0$, or $c s+c t+z-z_{0}=0$, or $z-z_{0}=-c(s+t)$. In $x=x_{0}+a s$ and $y=y_{0}+b t$, we can choose $s$ and $t$ such that $0 \leq x \leq a-1$ and $0 \leq y \leq b-1$. If $n>2 a b c-b c-c a-a b$, then $a b z=n-b c x-a c y>2 a b c-a b-b c-c a-b c(a-1)-c a(b-1)=-a b$ or $z>-1$, i.e., $z \geq 0$. Hence, it is representable as $b c x+c a y+a b z$ with $x, y, z \geq 0$. +Now we prove that $2 a b c-b c-c a-a b$ is not representable as $b c x+c a y+a b z$ with $x, y, z \geq 0$. Suppose that $b c x+c a y+a b z=2 a b c-a b-b c-c a$ with $x, y, z \geq 0$. Then + +$$ +b c(x+1)+c a(y+1)+a b(z+1)=2 a b c +$$ + +with $x+1, y+1, z+1 \geq 1$. Since $(a, b)=(a, c)=1$, we have $a \mid x+1$ and thus $a \leq x+1$. Similarly $b \leq y+1$ and $c \leq z+1$. Thus $b c a+c a b+a b c \leq 2 a b c$, a contradiction. +19. For all $n$, there exists a unique polynomial $P_{n}$ of degree $n$ such that $P_{n}(k)=F_{k}$ for $n+2 \leq k \leq 2 n+2$ and $P_{n}(2 n+3)=F_{2 n+3}-1$. For $n=0$, we have $F_{1}=F_{2}=1, F_{3}=2, P_{0}=1$. Now suppose that $P_{n-1}$ has been constructed and let $P_{n}$ be the polynomial of degree $n$ satisfying $P_{n}(X+2)-P_{n}(X+1)=P_{n-1}(X)$ and $P_{n}(n+2)=F_{n+2}$. (The mapping $\mathbb{R}_{n}[X] \rightarrow \mathbb{R}_{n-1}[X] \times \mathbb{R}, P \mapsto(Q, P(n+2)$ ), where $Q(X)=$ $P(X+2)-P(X+1)$, is bijective, since it is injective and those two spaces have the same dimension; clearly $\operatorname{deg} Q=\operatorname{deg} P-1$.) Thus for $n+2 \leq k \leq 2 n+2$ we have $P_{n}(k+1)=P_{n}(k)+F_{k-1}$ and $P_{n}(n+2)=F_{n+2}$; hence by induction on $k, P_{n}(k)=F_{k}$ for $n+2 \leq k \leq 2 n+2$ and + +$$ +P_{n}(2 n+3)=F_{2 n+2}+P_{n-1}(2 n+1)=F_{2 n+3}-1 +$$ + +Finally, $P_{990}$ is exactly the polynomial $P$ of the terms of the problem, for $P_{990}-P$ has degree less than or equal to 990 and vanishes at the 991 points $k=992, \ldots, 1982$. +20. If $\left(x_{1}, x_{2}, \ldots, x_{n}\right)$ satisfies the system with parameter $a$, then $\left(-x_{1},-x_{2}\right.$, $\ldots,-x_{n}$ ) satisfies the system with parameter $-a$. Hence it is sufficient to consider only $a \geq 0$. +Let $\left(x_{1}, \ldots, x_{n}\right)$ be a solution. Suppose $x_{1} \leq a, x_{2} \leq a, \ldots, x_{n} \leq a$. Summing the equations we get + +$$ +\left(x_{1}-a\right)^{2}+\cdots+\left(x_{n}-a\right)^{2}=0 +$$ + +and see that $(a, a, \ldots, a)$ is the only such solution. Now suppose that $x_{k} \geq a$ for some $k$. According to the $k$ th equation, + +$$ +x_{k+1}\left|x_{k+1}\right|=x_{k}^{2}-\left(x_{k}-a\right)^{2}=a\left(2 x_{k}-a\right) \geq a^{2} +$$ + +which implies that $x_{k+1} \geq a$ as well (here $x_{n+1}=x_{1}$ ). Consequently, all $x_{1}, x_{2}, \ldots, x_{n}$ are greater than or equal to $a$, and as above $(a, a, \ldots, a)$ is the only solution. +21. Using the identity + +$$ +a^{n}-b^{n}=(a-b) \sum_{m=0}^{n-1} a^{n-m-1} b^{m} +$$ + +with $a=k^{1 / n}$ and $b=(k-1)^{1 / n}$ one obtains + +$$ +1<\left(k^{1 / n}-(k-1)^{1 / n}\right) n k^{1-1 / n} \text { for all integers } n>1 \text { and } k \geq 1 +$$ + +This gives us the inequality $k^{1 / n-1}1$ and $k \geq 1$. In a similar way one proves that $n\left((k+1)^{1 / n}-k^{1 / n}\right)1$ and $k \geq 1$. Hence for $n>1$ and $m>1$ it holds that + +$$ +\begin{aligned} +n \sum_{k=1}^{m}\left((k+1)^{1 / n}-k^{1 / n}\right) & <\sum_{k=1}^{m} k^{1 / n-1} \\ +& 1$ and $t>1$. For $1 \leq k \leq n$ put $k=v s+u$, where $0 \leq v \leq t-1$ and $1 \leq u \leq s$, and let $a_{k}=a_{v s+u}$ be the unique integer in the set $\{1,2,3, \ldots, n\}$ such that $v s+u t-a_{v s+u}$ is a multiple of $n$. To prove that this construction gives a permutation, assume that $a_{k_{1}}=a_{k_{2}}$, where $k_{i}=v_{i} s+u_{i}, i=1,2$. Then $\left(v_{1}-v_{2}\right) s+\left(u_{1}-u_{2}\right) t$ is a multiple of $n=s t$. It follows that $t$ divides $\left(v_{1}-v_{2}\right)$, while $\left|v_{1}-v_{2}\right| \leq t-1$, and that $s$ divides $\left(u_{1}-u_{2}\right)$, while $\left|u_{1}-u_{2}\right| \leq s-1$. Hence, $v_{1}=v_{2}, u_{1}=u_{2}$, and $k_{1}=k_{2}$. We have proved that $a_{1}, \ldots, a_{n}$ is a permutation of $\{1,2, \ldots, n\}$ and hence + +$$ +\sum_{k=1}^{n} k \cos \frac{2 \pi a_{k}}{n}=\sum_{v=0}^{t-1}\left(\sum_{u=1}^{s}(v s+u) \cos \left(\frac{2 \pi v}{t}+\frac{2 \pi u}{s}\right)\right) +$$ + +Using $\sum_{u=1}^{s} \cos (2 \pi u / s)=\sum_{u=1}^{s} \sin (2 \pi u / s)=0$ and the additive formulas for cosine, one finds that + +$$ +\begin{aligned} +\sum_{k=1}^{n} k \cos \frac{2 \pi a_{k}}{n}= & \sum_{v=0}^{t-1}\left(\cos \frac{2 \pi v}{t} \sum_{u=1}^{s} u \cos \frac{2 \pi u}{s}-\sin \frac{2 \pi v}{t} \sum_{u=1}^{s} u \sin \frac{2 \pi u}{s}\right) \\ += & \left(\sum_{u=1}^{s} u \cos \frac{2 \pi u}{s}\right)\left(\sum_{v=0}^{t-1} \cos \frac{2 \pi v}{t}\right) \\ +& -\left(\sum_{u=1}^{s} u \sin \frac{2 \pi u}{s}\right)\left(\sum_{v=0}^{t-1} \sin \frac{2 \pi v}{t}\right)=0 +\end{aligned} +$$ + +23. We note that $\angle O_{1} K O_{2}=\angle M_{1} K M_{2}$ is equivalent to $\angle O_{1} K M_{1}=$ $\angle O_{2} K M_{2}$. Let $S$ be the intersection point of the common tangents, and let $L$ be the second point of intersection of $S K$ and $W_{1}$. Since $\triangle S O_{1} P_{1} \sim \triangle S P_{1} M_{1}$, we have $S K$. $S L=S P_{1}^{2}=S O_{1} \cdot S M_{1}$ which implies that points $O_{1}, L, K, M_{1}$ lie on a circle. Hence $\angle O_{1} K M_{1}=$ $\angle O_{1} L M_{1}=\angle O_{2} K M_{2}$. +![](https://cdn.mathpix.com/cropped/2024_11_18_8e985d6b9c83aa3e9d0eg-474.jpg?height=295&width=551&top_left_y=959&top_left_x=809) +24. See the solution of (SL91-15). +25. Suppose the contrary, that $\mathbb{R}^{3}=P_{1} \cup P_{2} \cup P_{3}$ is a partition such that $a_{1} \in \mathbb{R}^{+}$is not realized by $P_{1}, a_{2} \in \mathbb{R}^{+}$is not realized by $P_{2}$ and $a_{3} \in \mathbb{R}^{+}$ not realized by $P_{3}$, where w.l.o.g. $a_{1} \geq a_{2} \geq a_{3}$. +If $P_{1}=\emptyset=P_{2}$, then $P_{3}=\mathbb{R}^{3}$, which is impossible. +If $P_{1}=\emptyset$, and $X \in P_{2}$, the sphere centered at $X$ with radius $a_{2}$ is included in $P_{3}$ and $a_{3} \leq a_{2}$ is realized, which is impossible. +If $P_{1} \neq \emptyset$, let $X_{1} \in P_{1}$. The sphere $S$ centered in $X_{1}$, of radius $a_{1}$ is included in $P_{2} \cap P_{3}$. Since $a_{1} \geq a_{3}, S \not \subset P_{3}$. Let $X_{2} \in P_{2} \cap S$. The circle $\left\{Y \in S \mid d\left(X_{2}, Y\right)=a_{2}\right\}$ is included in $P_{3}$, but $a_{2} \leq a_{1}$; hence it has radius $r=a_{2} \sqrt{1-a_{2}^{2} /\left(4 a_{1}^{2}\right)} \geq a_{2} \sqrt{3} / 2$ and $a_{3} \leq a_{2} \leq a_{2} \sqrt{3}<2 r$; hence $a_{3}$ is realized by $P_{3}$. + +### 4.25 Solutions to the Shortlisted Problems of IMO 1984 + +1. This is the same problem as (SL83-20). +2. (a) For $m=t(t-1) / 2$ and $n=t(t+1) / 2$ we have $4 m n-m-n=$ $\left(t^{2}-1\right)^{2}-1$ +(b) Suppose that $4 m n-m-n=p^{2}$, or equivalently, $(4 m-1)(4 n-1)=$ $4 p^{2}+1$. The number $4 m-1$ has at least one prime divisor, say $q$, that is of the form $4 k+3$. Then $4 p^{2} \equiv-1(\bmod q)$. However, by Fermat's theorem we have + +$$ +1 \equiv(2 p)^{q-1}=\left(4 p^{2}\right)^{\frac{q-1}{2}} \equiv(-1)^{\frac{q-1}{2}}(\bmod q) +$$ + +which is impossible since $(q-1) / 2=2 k+1$ is odd. +3. From the equality $n=d_{6}^{2}+d_{7}^{2}-1$ we see that $d_{6}$ and $d_{7}$ are relatively prime and $d_{7}\left|d_{6}^{2}-1=\left(d_{6}-1\right)\left(d_{6}+1\right), d_{6}\right| d_{7}^{2}-1=\left(d_{7}-1\right)\left(d_{7}+1\right)$. Suppose that $d_{6}=a b, d_{7}=c d$ with $1A_{i} A_{i+1}+A_{j} A_{j+1}$. Summing all such $n(n-3) / 2$ inequalities, we obtain $2 d>(n-3) p$, proving the first inequality. +Let us now prove the second inequality. We notice that for each diagonal $A_{i} A_{i+j}$ (we may assume w.l.o.g. that $j \leq[n / 2]$ ) the following relation holds: + +$$ +A_{i} A_{i+j}y^{\prime}$ and $z>z^{\prime}$, but then $\sqrt{y-a}+\sqrt{z-a}>\sqrt{y^{\prime}-a}+\sqrt{z^{\prime}-a}$, which is a contradiction. +We shall now prove the existence of at least one solution. Let $P$ be an arbitrary point in the plane and $K, L, M$ points such that $P K=\sqrt{a}$, $P L=\sqrt{b}, P M=\sqrt{c}$, and $\angle K P L=\angle L P M=\angle M P K=120^{\circ}$. The lines through $K, L, M$ perpendicular respectively to $P K, P L, P M$ form an equilateral triangle $A B C$, where $K \in B C, L \in A C$, and $M \in A B$. Since its area equals $A B^{2} \sqrt{3} / 4=S_{\triangle B P C}+S_{\triangle A P C}+$ $S_{\triangle A P B}=A B(\sqrt{a}+\sqrt{b}+\sqrt{c}) / 2$, it follows that $A B=1$. Therefore $x=P A^{2}, y=P B^{2}$, and $z=P C^{2}$ is a solution of the system (indeed, $\sqrt{y-a}+\sqrt{z-a}=\sqrt{P B^{2}-P K^{2}}+\sqrt{P C^{2}-P K^{2}}=B K+C K=1$, etc.). +10. Suppose that the product of some five consecutive numbers is a square. It is easily seen that among them at least one, say $n$, is divisible neither by 2 nor 3 . Since $n$ is coprime to the remaining four numbers, it is itself a square of a number $m$ of the form $6 k \pm 1$. Thus $n=(6 k \pm 1)^{2}=24 r+1$, where $r=k(3 k \pm 1) / 2$. Note that neither of the numbers $24 r-1,24 r+5$ is one of our five consecutive numbers because it is not a square. Hence the five numbers must be $24 r, 24 r+1, \ldots, 24 r+4$. However, the number $24 r+4=(6 k \pm 1)^{2}+3$ is divisible by $6 r+1$, which implies that it is a square as well. It follows that these two squares are 1 and 4 , which is impossible. +11. Suppose that an integer $x$ satisfies the equation. Then the numbers $x-$ $a_{1}, x-a_{2}, \ldots, x-a_{2 n}$ are $2 n$ distinct integers whose product is $1 \cdot(-1)$. $2 \cdot(-2) \cdots n \cdot(-n)$. +From here it is obvious that the numbers $x-a_{1}, x-a_{2}, \ldots, x-a_{2 n}$ are some reordering of the numbers $-n,-n+1, \ldots,-1,1, \ldots, n-1, n$. It follows that their sum is 0 , and therefore $x=\left(a_{1}+a_{2}+\cdots+a_{2 n}\right) / 2 n$. This is +the only solution if $\left\{a_{1}, a_{2}, \ldots, a_{2 n}\right\}=\{x-n, \ldots, x-1, x+1, \ldots, x+n\}$ for some $x \in \mathbb{N}$. Otherwise there is no solution. +12. By the binomial formula we have + +$$ +\begin{aligned} +(a+b)^{7}-a^{7}-b^{7} & =7 a b\left[\left(a^{5}+b^{5}\right)+3 a b\left(a^{3}+b^{3}\right)+5 a^{2} b^{2}(a+b)\right] \\ +& =7 a b(a+b)\left(a^{2}+a b+b^{2}\right)^{2} +\end{aligned} +$$ + +Thus it will be enough to find $a$ and $b$ such that $7 \nmid a, b$ and $7^{3} \mid a^{2}+a b+b^{2}$. Such numbers must satisfy $(a+b)^{2}>a^{2}+a b+b^{2} \geq 7^{3}=343$, implying $a+b \geq 19$. Trying $a=1$ we easily find the example $(a, b)=(1,18)$. +13. Let $Z$ be the given cylinder of radius $r$, altitude $h$, and volume $\pi r^{2} h=1, k_{1}$ and $k_{2}$ the circles surrounding its bases, and $V$ the volume of an inscribed tetrahedron $A B C D$. +We claim that there is no loss of generality in assuming that $A, B, C, D$ all lie on $k_{1} \cup k_{2}$. Indeed, if the vertices $A, B, C$ are fixed and $D$ moves along a segment $E F$ parallel to the axis of the cylinder $\left(E \in k_{1}, F \in k_{2}\right)$, the maximum distance of $D$ from the plane $A B C$ (and consequently the maximum value of $V$ ) is achieved either at $E$ or at $F$. Hence we shall consider only the following two cases: +(i) $A, B \in k_{1}$ and $C, D \in k_{2}$. Let $P, Q$ be the projections of $A, B$ on the plane of $k_{2}$, and $R, S$ the projections of $C, D$ on the plane of $k_{1}$, respectively. Then $V$ is one-third of the volume $V^{\prime}$ of the prism $A R B S C P D Q$ with bases $A R B S$ and $C P D Q$. The area of the quadrilateral $A R B S$ inscribed in $k_{1}$ does not exceed the area of the square inscribed therein, which is $2 r^{2}$. Hence $3 V=V^{\prime} \leq 2 r^{2} h=2 / \pi$. +(ii) $A, B, C \in k_{1}$ and $D \in k_{2}$. The area of the triangle $A B C$ does not exceed the area of an equilateral triangle inscribed in $k_{1}$, which is $3 \sqrt{3} r^{2} / 4$. Consequently, $V \leq \frac{\sqrt{3}}{4} r^{2} h=\frac{\sqrt{3}}{4 \pi}<\frac{2}{3 \pi}$. +14. Let $M$ and $N$ be the midpoints of $A B$ and $C D$, and let $M^{\prime}, N^{\prime}$ be their projections on $C D$ and $A B$, respectively. We know that $M M^{\prime}=A B /$, and hence + +$$ +S_{A B C D}=S_{A M D}+S_{B M C}+S_{C M D}=\frac{1}{2}\left(S_{A B D}+S_{A B C}\right)+\frac{1}{4} A B \cdot C D +$$ + +The line $A B$ is tangent to the circle with diameter $C D$ if and only if $N N^{\prime}=C D / 2$, or equivalently, + +$$ +S_{A B C D}=S_{A N D}+S_{B N C}+S_{A N B}=\frac{1}{2}\left(S_{B C D}+S_{A C D}\right)+\frac{1}{4} A B \cdot C D +$$ + +By (1), this is further equivalent to $S_{A B C}+S_{A B D}=S_{B C D}+S_{A C D}$. But since $S_{A B C}+S_{A C D}=S_{A B D}+S_{B C D}=S_{A B C D}$, this reduces to $S_{A B C}=S_{B C D}$, i.e., to $B C \| A D$. +15. (a) Since rotation by $60^{\circ}$ around $A$ transforms the triangle $C A F$ into $\triangle E A B$, it follows that $\measuredangle(C F, E B)=60^{\circ}$. We similarly deduce that +$\measuredangle(E B, A D)=\measuredangle(A D, F C)=60^{\circ}$. Let $S$ be the intersection point of $B E$ and $A D$. Since $\measuredangle C S E=\measuredangle C A E=60^{\circ}$, it follows that $E A S C$ is cyclic. Therefore $\measuredangle(A S, S C)=60^{\circ}=\measuredangle(A D, F C)$, which implies that $S$ lies on $C F$ as well. +(b) A rotation of $E A S C$ around $E$ by $60^{\circ}$ transforms $A$ into $C$ and $S$ into a point $T$ for which $S E=S T=S C+C T=S C+S A$. Summing the equality $S E=S C+S A$ and the analogous equalities $S D=S B+S C$ and $S F=S A+S B$ yields the result. +16. From the first two conditions we can easily conclude that $a+d>b+c$ (indeed, $(d+a)^{2}-(d-a)^{2}=(c+b)^{2}-(c-b)^{2}=4 a d=4 b c$ and $d-a>c-b>0)$. Thus $k>m$. +From $d=2^{k}-a$ and $c=2^{m}-b$ we get $a\left(2^{k}-a\right)=b\left(2^{m}-b\right)$, or equivalently, + +$$ +(b+a)(b-a)=2^{m}\left(b-2^{k-m} a\right) +$$ + +Since $2^{k-m} a$ is even and $b$ is odd, the highest power of 2 that divides the right-hand side of $(1)$ is $m$. Hence $(b+a)(b-a)$ is divisible by $2^{m}$ but not by $2^{m+1}$, which implies $b+a=2^{m_{1}} p$ and $b-a=2^{m_{2}} q$, where $m_{1}, m_{2} \geq 1$, $m_{1}+m_{2}=m$, and $p, q$ are odd. +Furthermore, $b=\left(2^{m_{1}} p+2^{m_{2}} q\right) / 2$ and $a=\left(2^{m_{1}} p-2^{m_{2}} q\right) / 2$ are odd, so either $m_{1}=1$ or $m_{2}=1$. Note that $m_{1}=1$ is not possible, since it would imply that $b-a=2^{m-1} q \geq 2^{m-1}$, although $b+c=2^{m}$ and $bm$, it follows that $a=1$. +Remark. Now it is not difficult to prove that all fourtuplets $(a, b, c, d)$ that satisfy the given conditions are of the form $\left(1,2^{m-1}-1,2^{m-1}+1,2^{2 m-2}-\right.$ 1 ), where $m \in \mathbb{N}, m \geq 3$. +17. For any $m=0,1, \ldots, n-1$, we shall find the number of permutations $\left(x_{1}, x_{2}, \ldots, x_{n}\right)$ with exactly $k$ discordant pairs such that $x_{n}=n-m$. This $x_{n}$ is a member of exactly $m$ discordant pairs, and hence the permutation $\left(x_{1}, \ldots, x_{n-1}\right.$ of the set $\{1,2, \ldots, n\} \backslash\{m\}$ must have exactly $k-m$ discordant pairs: there are $d(n-1, k-m)$ such permutations. Therefore + +$$ +\begin{aligned} +d(n, k) & =d(n-1, k)+d(n-1, k-1) \cdots+d(n-1, k-n+1) \\ +& =d(n-1, k)+d(n, k-1) +\end{aligned} +$$ + +(note that $d(n, k)$ is 0 if $k<0$ or $k>\binom{n}{2}$ ). +We now proceed to calculate $d(n, 2)$ and $d(n, 3)$. Trivially, $d(n, 0)=1$. It follows that $d(n, 1)=d(n-1,1)+d(n, 0)=d(n-1,1)+1$, which yields $d(n, 1)=d(1,1)+n-1=n-1$. + +Further, $d(n, 2)=d(n-1,2)+d(n, 1)=d(n-1,2)+n-1=d(2,2)+$ $2+3+\cdots+n-1=\left(n^{2}-n-2\right) / 2$. +Finally, using the known formula $1^{2}+2^{2}+\cdots+k^{2}=k(k+1)(2 k+1) / 6$, we have $d(n, 3)=d(n-1,3)+d(n, 2)=d(n-1,3)+\left(n^{2}-n-2\right) / 2=$ $d(2,3)+\sum_{i=3}^{n}\left(n^{2}-n-2\right) / 2=\left(n^{3}-7 n+6\right) / 6$. +18. Suppose that circles $k_{1}\left(O_{1}, r_{1}\right), k_{2}\left(O_{2}, r_{2}\right)$, and $k_{3}\left(O_{3}, r_{3}\right)$ touch the edges of the angles $\angle B A C, \angle A B C$, and $\angle A C B$, respectively. Denote also by $O$ and $r$ the center and radius of the incircle. Let $P$ be the point of tangency of the incircle with $A B$ and let $F$ be the foot of the perpendicular from $O_{1}$ to $O P$. From $\triangle O_{1} F O$ we obtain $\cot (\alpha / 2)=2 \sqrt{r r_{1}} /\left(r-r_{1}\right)$ and analogously $\cot (\beta / 2)=2 \sqrt{r r_{2}} /\left(r-r_{2}\right), \cot (\gamma / 2)=2 \sqrt{r r_{3}} /\left(r-r_{3}\right)$. We will now use a well-known trigonometric identity for the angles of a triangle: + +$$ +\cot \frac{\alpha}{2}+\cot \frac{\beta}{2}+\cot \frac{\gamma}{2}=\cot \frac{\alpha}{2} \cdot \cot \frac{\beta}{2} \cdot \cot \frac{\gamma}{2} . +$$ + +(This identity follows from $\tan (\gamma / 2)=\cot (\alpha / 2+\beta / 2)$ and the formula for the cotangent of a sum.) +Plugging in the obtained cotangents, we get + +$$ +\begin{aligned} +\frac{2 \sqrt{r r_{1}}}{r-r_{1}}+\frac{2 \sqrt{r r_{2}}}{r-r_{2}}+\frac{2 \sqrt{r r_{3}}}{r-r_{3}}= & \frac{2 \sqrt{r r_{1}}}{r-r_{1}} \cdot \frac{2 \sqrt{r r_{2}}}{r-r_{2}} \cdot \frac{2 \sqrt{r r_{3}}}{r-r_{3}} \Rightarrow \\ +& \sqrt{r_{1}}\left(r-r_{2}\right)\left(r-r_{3}\right)+\sqrt{r_{2}}\left(r-r_{1}\right)\left(r-r_{3}\right) \\ +& +\sqrt{r_{3}}\left(r-r_{1}\right)\left(r-r_{2}\right)=4 r \sqrt{r_{1} r_{2} r_{3}} . +\end{aligned} +$$ + +For $r_{1}=1, r_{2}=4$, and $r_{3}=9$ we get +$(r-4)(r-9)+2(r-1)(r-9)+3(r-1)(r-4)=24 r \Rightarrow 6(r-1)(r-11)=0$. +Clearly, $r=11$ is the only viable value for $r$. +19. First, we shall prove that the numbers in the $n$th row are exactly the numbers + +$$ +\frac{1}{n\binom{n-1}{0}}, \frac{1}{n\binom{n-1}{1}}, \frac{1}{n\binom{n-1}{2}}, \ldots, \frac{1}{n\binom{n-1}{n-1}} +$$ + +The proof of this fact can be done by induction. For small $n$, the statement can be easily verified. Assuming that the statement is true for some $n$, we have that the $k$ th element in the $(n+1)$ st row is, as is directly verified, + +$$ +\frac{1}{n\binom{n-1}{k-1}}-\frac{1}{(n+1)\binom{n}{k-1}}=\frac{1}{(n+1)\binom{n}{k}} +$$ + +Thus (1) is proved. Now the geometric mean of the elements of the $n$th row becomes: + +$$ +\frac{1}{n \sqrt[n]{\binom{n-1}{0} \cdot\binom{n-1}{1} \cdots\binom{n-1}{n-1}}} \geq \frac{1}{n\left(\frac{\binom{n-1}{0}+\binom{n-1}{1}+\cdots+\binom{n-1}{n-1}}{n}\right)}=\frac{1}{2^{n-1}} +$$ + +The desired result follows directly from substituting $n=1984$. +20. Define the set $S=\mathbb{R}^{+} \backslash\{1\}$. The given inequality is equivalent to $\ln b / \ln a<\ln (b+1) / \ln (a+1)$. +If $b=1$, it is obvious that each $a \in S$ satisfies this inequality. Suppose now that $b$ is also in $S$. +Let us define on $S$ a function $f(x)=\ln (x+1) / \ln x$. Since $\ln (x+1)>\ln x$ and $1 / x>1 / x+1>0$, we have + +$$ +f^{\prime}(x)=\frac{\frac{\ln x}{x+1}-\frac{\ln (x+1)}{x}}{\ln ^{2} x}<0 \quad \text { for all } x +$$ + +Hence $f$ is always decreasing. We also note that $f(x)<0$ for $x<1$ and that $f(x)>0$ for $x>1$ (at $x=1$ there is a discontinuity). +Let us assume $b>1$. From $\ln b / \ln a<\ln (b+1) / \ln (a+1)$ we get $f(b)>$ $f(a)$. This holds for $b>a$ or for $a<1$. +Now let us assume $b<1$. This time we get $f(b)1$. +Hence all the solutions to $\log _{a} b<\log _{a+1}(b+1)$ are $\{b=1, a \in S\}$, $\{a>b>1\},\{b>1>a\},\{a513$ distinct two-element subsets of $M$ each having a square as the product of elements. Reasoning as above, we find at least one (in fact many) pair of such squares whose product is a fourth power. +2. The polyhedron has $3 \cdot 12 / 2=18$ edges, and by Euler's formula, 8 vertices. Let $v_{1}$ and $v_{2}$ be the numbers of vertices at which respectively 3 and 6 edges meet. Then $v_{1}+v_{2}=8$ and $3 v_{1}+6 v_{2}=2 \cdot 18$, implying that $v_{1}=4$. Let $A, B, C, D$ be the vertices at which three edges meet. Since the dihedral angles are equal, all the edges meeting at $A$, say $A E, A F, A G$, must have equal length, say $x$. (If $x=A E=A F \neq A G=y$, and $A E F$, $A F G$, and $A G E$ are isosceles, $\angle E A F \neq \angle F A G$, in contradiction to the equality of the dihedral angles.) It is easy to see that at $E, F$, and $G$ six edges meet. One proceeds to conclude that if $H$ is the fourth vertex of this kind, $E F G H$ must be a regular tetrahedron of edge length $y$, and the other vertices $A, B, C$, and $D$ are tops of isosceles pyramids based on $E F G, E F H, F G H$, and $G E H$. Let the plane through $A, B, C$ meet $E F$, $H F$, and $G F$, at $E^{\prime}, H^{\prime}$, and $G^{\prime}$. Then $A E^{\prime} B H^{\prime} C G^{\prime}$ is a regular hexagon, and since $x=F A=F E^{\prime}$, we have $E^{\prime} G^{\prime}=x$ and $A E^{\prime}=x / \sqrt{3}$. From the isosceles triangles $A E F$ and $F A E^{\prime}$ we obtain finally, with $\measuredangle E F A=\alpha$, $\frac{y}{2 x}=\cos \alpha=1-2 \sin ^{2}(\alpha / 2), x /(2 x \sqrt{3})=\sin (\alpha / 2)$, and $y / x=5 / 3$. +3. We shall write $P \equiv Q$ for two polynomials $P$ and $Q$ if $P(x)-Q(x)$ has even coefficients. +We observe that $(1+x)^{2^{m}} \equiv 1+x^{2^{m}}$ for every $m \in \mathbb{N}$. Consequently, for every polynomial $p$ with degree less than $k=2^{m}, w\left(p \cdot q_{k}\right)=2 w(p)$. +Now we prove the inequality from the problem by induction on $i_{n}$. If $i_{n} \leq 1$, the inequality is trivial. Assume it is true for any sequence with $i_{1}<\cdotsj$, then by (ii), $\langle k j\rangle \sim\langle k j\rangle-j=\langle(k-1) j\rangle$. If otherwise $\langle k j\rangle0$, then the midpoint $P$ of $M N$ lies inside the circle $C_{(m+n) / 2}$. This is trivial if $m=n$, so let $m \neq n$. For fixed $M, P$ is in the image $C_{n}^{\prime}$ of $C_{n}$ under the homothety with center $M$ and coefficient $1 / 2$. The center of the circle $C_{n}^{\prime}$ is at the midpoint of $O_{n} M$. If we let both $M$ and $N$ vary, $P$ will be on the union of circles with radius $r_{n} / 2$ and centers in the image of $C_{m}$ under the homothety with center $O_{n}$ and coefficient $1 / 2$. Hence $P$ is not outside the circle centered at the midpoint $O_{m} O_{n}$ and with radius $\left(r_{m}+r_{n}\right) / 2$. It remains to show that $r_{(m+n) / 2}>\left(r_{m}+r_{n}\right) / 2$. But this inequality is easily reduced to $(m-n)^{2}>0$, which is true. +6. Let us set + +$$ +\begin{aligned} +& x_{n, i}=\sqrt[i]{i+\sqrt[i+1]{i+1+\cdots+\sqrt[n]{n}}} \\ +& y_{n, i}=x_{n+1, i}^{i-1}+x_{n+1, i}^{i-2} x_{n, i}+\cdots+x_{n, i}^{i-1} +\end{aligned} +$$ + +In particular, $x_{n, 2}=x_{n}$ and $x_{n, i}=0$ for $i>n$. We observe that for $n>i>2$, + +$$ +x_{n+1, i}-x_{n, i}=\frac{x_{n+1, i}^{i}-x_{n, i}^{i}}{y_{n, i}}=\frac{x_{n+1, i+1}-x_{n, i+1}}{y_{n, i}} . +$$ + +Since $y_{n, i}>i x_{n, i}^{i-1} \geq i^{1+(i-1) / i} \geq i^{3 / 2}$ and $x_{n+1, n+1}-x_{n, n+1}=\sqrt[n+1]{n+1}$, simple induction gives + +$$ +x_{n+1}-x_{n} \leq \frac{\sqrt[n+1]{n+1}}{(n!)^{3 / 2}}<\frac{1}{n!} \quad \text { for } n>2 +$$ + +The inequality for $n=2$ is directly verified. +7. Let $k_{i} \geq 0$ be the largest integer such that $p^{k_{i}} \mid x_{i}, i=1, \ldots, n$, and $y_{i}=x_{i} / p^{k_{i}}$. We may assume that $k=k_{1}+\cdots+k_{n}$. All the $y_{i}$ must be +distinct. Indeed, if $y_{i}=y_{j}$ and $k_{i}>k_{j}$, then $x_{i} \geq p x_{j} \geq 2 x_{i} \geq 2 x_{1}$, which is impossible. Thus $y_{1} y_{2} \ldots y_{n}=P / p^{k} \geq n!$. +If equality holds, we must have $y_{i}=1, y_{j}=2$ and $y_{k}=3$ for some $i, j, k$. Thus $p \geq 5$, which implies that either $y_{i} / y_{j} \leq 1 / 2$ or $y_{i} / y_{j} \geq 5 / 2$, which is impossible. Hence the inequality is strict. +8. Among ten consecutive integers that divide $n$, there must exist numbers divisible by $2^{3}, 3^{2}, 5$, and 7 . Thus the desired number has the form $n=$ $2^{\alpha_{1}} 3^{\alpha_{2}} 5^{\alpha_{3}} 7^{\alpha_{4}} 11^{\alpha_{5}} \cdots$, where $\alpha_{1} \geq 3, \alpha_{2} \geq 2, \alpha_{3} \geq 1, \alpha_{4} \geq 1$. Since $n$ has $\left(\alpha_{1}+1\right)\left(\alpha_{2}+1\right)\left(\alpha_{3}+1\right) \cdots$ distinct factors, and $\left(\alpha_{1}+1\right)\left(\alpha_{2}+1\right)\left(\alpha_{3}+\right.$ $1)\left(\alpha_{4}+1\right) \geq 48$, we must have $\left(\alpha_{5}+1\right) \cdots \leq 3$. Hence at most one $\alpha_{j}$, $j>4$, is positive, and in the minimal $n$ this must be $\alpha_{5}$. Checking through the possible combinations satisfying $\left(\alpha_{1}+1\right)\left(\alpha_{2}+1\right) \cdots\left(\alpha_{5}+1\right)=144$ one finds that the minimal $n$ is $2^{5} \cdot 3^{2} \cdot 5 \cdot 7 \cdot 11=110880$. +9. Let $\vec{a}, \vec{b}, \vec{c}, \vec{d}$ denote the vectors $\overrightarrow{O A}, \overrightarrow{O B}, \overrightarrow{O C}, \overrightarrow{O D}$ respectively. Then $|\vec{a}|=|\vec{b}|=|\vec{c}|=|\vec{d}|=1$. The centroids of the faces are $(\vec{b}+\vec{c}+\vec{d}) / 3$, $(\vec{a}+\vec{c}+\vec{d}) / 3$, etc., and each of these is at distance $1 / 3$ from $P=$ $(\vec{a}+\vec{b}+\vec{c}+\vec{d}) / 3$; hence the required radius is $1 / 3$. To compute $|P|$ as a function of the edges of $A B C D$, observe that $A B^{2}=(\vec{b}-\vec{a})^{2}=$ $2-2 \vec{a} \cdot \vec{b}$ etc. Now + +$$ +\begin{aligned} +P^{2} & =\frac{|\vec{a}+\vec{b}+\vec{c}+\vec{d}|^{2}}{9} \\ +& =\frac{16-2\left(A B^{2}+B C^{2}+A C^{2}+A D^{2}+B D^{2}+C D^{2}\right)}{9} +\end{aligned} +$$ + +10. If $M$ is at a vertex of the regular tetrahedron $A B C D(A B=1)$, then one can take $M^{\prime}$ at the center of the opposite face of the tetrahedron. +Let $M$ be on the face $(A B C)$ of the tetrahedron, excluding the vertices. Consider a continuous mapping $f$ of $\mathbb{C}$ onto the surface $S$ of $A B C D$ that maps $m+n e^{2 \pi / 3}$ for $m, n \in$ $\mathbb{Z}$ onto $A, B, C, D$ if $(m, n) \equiv$ $(1,1),(1,0),(0,1),(0,0)(\bmod 2)$ re- +![](https://cdn.mathpix.com/cropped/2024_11_18_8e985d6b9c83aa3e9d0eg-484.jpg?height=464&width=446&top_left_y=1356&top_left_x=857) +spectively, and maps each unit equilateral triangle with vertices of the form $m+n e^{2 \pi / 3}$ isometrically onto the corresponding face of $A B C D$. The point $M$ then has one preimage $M_{j}, j=1,2, \ldots, 6$, in each of the six preimages of $\triangle A B C$ having two vertices on the unit circle. The $M_{j}$ 's form a convex centrally symmetric (possibly degenerate) hexagon. Of the triangles formed by two adjacent sides of this hexagon consider the one, say $M_{1} M_{2} M_{3}$, with the smallest radius of circumcircle and denote by $\widehat{M^{\prime}}$ +its circumcenter. Then we can choose $M^{\prime}=f\left(\widehat{M^{\prime}}\right)$. Indeed, the images of the segments $M_{1} \widehat{M^{\prime}}, M_{2} \widehat{M^{\prime}}, M_{3} \widehat{M^{\prime}}$ are three different shortest paths on $S$ from $M$ to $M^{\prime}$. +11. Let $-x_{1}, \ldots,-x_{6}$ be the roots of the polynomial. Let $s_{k, i}(k \leq i \leq 6)$ denote the sum of all products of $k$ of the numbers $x_{1}, \ldots, x_{i}$. By Vieta's formula we have $a_{k}=s_{k, 6}$ for $k=1, \ldots, 6$. Since $s_{k, i}=s_{k-1, i-1} x_{i}+$ $s_{k, i-1}$, one can compute the $a_{k}$ by the following scheme (the horizontal and vertical arrows denote multiplications and additions respectively): +![](https://cdn.mathpix.com/cropped/2024_11_18_8e985d6b9c83aa3e9d0eg-485.jpg?height=457&width=609&top_left_y=646&top_left_x=486) +12. We shall prove by induction on $m$ that $P_{m}(x, y, z)$ is symmetric and that + +$$ +(x+y) P_{m}(x, z, y+1)-(x+z) P_{m}(x, y, z+1)=(y-z) P_{m}(x, y, z) +$$ + +holds for all $x, y, z$. This is trivial for $m=0$. Assume now that it holds for $m=n-1$. +Since obviously $P_{n}(x, y, z)=P_{n}(y, x, z)$, the symmetry of $P_{n}$ will follow if we prove that $P_{n}(x, y, z)=P_{n}(x, z, y)$. Using (1) we have $P_{n}(x, z, y)-$ $P_{n}(x, y, z)=(y+z)\left[(x+y) P_{n-1}(x, z, y+1)-(x+z) P_{n-1}(x, y, z+1)\right]-\left(y^{2}-\right.$ $\left.z^{2}\right) P_{n-1}(x, y, z)=(y+z)(y-z) P_{n-1}(x, y, z)-\left(y^{2}-z^{2}\right) P_{n-1}(x, y, z)=0$. It remains to prove (1) for $m=n$. Using the already established symmetry we have + +$$ +\begin{aligned} +& (x+y) P_{n}(x, z, y+1)-(x+z) P_{n}(x, y, z+1) \\ +& =(x+y) P_{n}(y+1, z, x)-(x+z) P_{n}(z+1, y, x) \\ +& =(x+y)\left[(y+x+1)(z+x) P_{n-1}(y+1, z, x+1)-x^{2} P_{n-1}(y+1, z, x)\right] \\ +& \quad-(x+z)\left[(z+x+1)(y+x) P_{n-1}(z+1, y, x+1)-x^{2} P_{n-1}(z+1, y, x)\right] \\ +& =(x+y)(x+z)(y-z) P_{n-1}(x+1, y, z)-x^{2}(y-z) P_{n-1}(x, y, z) \\ +& =(y-z) P_{n}(z, y, x)=(y-z) P_{n}(x, y, z), +\end{aligned} +$$ + +as claimed. +13. If $m$ and $n$ are relatively prime, there exist positive integers $p, q$ such that $p m=q n+1$. Thus by putting $m$ balls in some boxes $p$ times we can +achieve that one box receives $q+1$ balls while all others receive $q$ balls. Repeating this process sufficiently many times, we can obtain an equal distribution of the balls. +Now assume $\operatorname{gcd}(m, n)>1$. If initially there is only one ball in the boxes, then after $k$ operations the number of balls will be $1+k m$, which is never divisible by $n$. Hence the task cannot be done. +14. It suffices to prove the existence of a good point in the case of exactly 661 -1 's. We prove by induction on $k$ that in any arrangement with $3 k+2$ points $k$ of which are -1 's a good point exists. For $k=1$ this is clear by inspection. Assume that the assertion holds for all arrangements of $3 n+2$ points and consider an arrangement of $3(n+1)+2$ points. Now there exists a sequence of consecutive -1 's surrounded by two +1 's. There is a point $P$ which is good for the arrangement obtained by removing the two +1 's bordering the sequence of -1 's and one of these -1 's. Since $P$ is out of this sequence, clearly the removal either leaves a partial sum as it was or diminishes it by 1 , so $P$ is good for the original arrangement. +Second solution. Denote the number on an arbitrary point by $a_{1}$, and the numbers on successive points going in the positive direction by $a_{2}, a_{3}, \ldots$ (in particular, $a_{k+1985}=a_{k}$ ). We define the partial sums $s_{0}=0, s_{n}=$ $a_{1}+a_{2}+\cdots+a_{n}$ for all positive integers $n$; then $s_{k+1985}=s_{k}+s_{1985}$ and $s_{1985} \geq 663$. Since $s_{1985 m} \geq 663 m$ and $3 \cdot 663 m>1985(m+2)+1$ for large $m$, not all values $0,1,2, \ldots 663 m$ can appear thrice among the $1985(m+2)+1$ sums $s_{-1985}, s_{-1984}, \ldots, s_{1985(m+1)}$ (and none of them appears out of this set). Thus there is an integral value $s>0$ that appears at most twice as a partial sum, say $s_{k}=s_{l}=s, ks$ must hold for all $i>l$, and $s_{i}s$ and $s_{q}Y Z$, and $X^{\prime} Y^{\prime} \cdot Y^{\prime} Z^{\prime}=X Y \cdot Y Z$. Suppose that $2 X^{\prime} Y^{\prime}>X Y$ (otherwise, we may cut off congruent rectangles from both the original ones until we reduce them to the case of $2 X^{\prime} Y^{\prime}>X Y$ ). Let $U \in X Y$ and $V \in Z T$ be points such that $Y U=T V=X^{\prime} Y^{\prime}$ and $W \in X V$ be a point such that $U W \| X T$. Then translating $\triangle X U W$ to a triangle $V Z R$ and $\triangle X V T$ to a triangle $W R S$ results in a rectangle $U Y R S$ congruent to $X^{\prime} Y^{\prime} Z^{\prime} T^{\prime}$. +Thus we have partitioned $K$ and $K^{\prime}$ into translation-invariant parts. Although not all the parts are triangles, we may simply triangulate them. +16. Let the three circles be $\alpha(A, a), \beta(B, b)$, and $\gamma(C, c)$, and assume $c \leq a, b$. We denote by $\mathcal{R}_{X, \varphi}$ the rotation around $X$ through an angle $\varphi$. Let $P Q R$ be an equilateral triangle, say of positive orientation (the case of negatively oriented $\triangle P Q R$ is analogous), with $P \in \alpha, Q \in \beta$, and $R \in \gamma$. Then $Q=\mathcal{R}_{P,-60^{\circ}}(R) \in \mathcal{R}_{P,-60^{\circ}}(\gamma) \cap \beta$. +Since the center of $\mathcal{R}_{P,-60^{\circ}}(\gamma)$ is $\mathcal{R}_{P,-60^{\circ}}(C)=\mathcal{R}_{C, 60^{\circ}}(P)$ and it lies on $\mathcal{R}_{C, 60^{\circ}}(\alpha)$, the union of circles $\mathcal{R}_{P,-60^{\circ}}(\gamma)$ as $P$ varies on $\alpha$ is the annulus $\mathcal{U}$ with center $A^{\prime}=\mathcal{R}_{C, 60^{\circ}}(A)$ and radii $a-c$ and $a+c$. Hence there is a solution if and only if $\mathcal{U} \cap \beta$ is nonempty. +17. The statement of the problem is equivalent to the statement that there is one and only one $a$ such that $1-1 / nx$ for all $x>0$ implies that $y_{n}-x_{n}0$ with $\prod_{i=1}^{n} y_{i}=1$. Then $\frac{1}{1+y_{n-1}}+\frac{1}{1+y_{n}}>\frac{1}{1+y_{n-1} y_{n}}$, which is equivalent to $1+y_{n} y_{n-1}(1+$ $\left.y_{n}+y_{n-1}\right)>0$. Hence by the inductive hypothesis + +$$ +\sum_{i=1}^{n} \frac{1}{1+y_{i}} \geq \sum_{i=1}^{n-2} \frac{1}{1+y_{i}}+\frac{1}{1+y_{n-1} y_{n}} \geq 1 +$$ + +Remark. The constant $n-1$ is best possible (take for example $x_{i}=a^{i}$ with $a$ arbitrarily large). +19. Suppose that for some $n>6$ there is a regular $n$-gon with vertices having integer coordinates, and that $A_{1} A_{2} \ldots A_{n}$ is the smallest such $n$-gon, of side length $a$. If $O$ is the origin and $B_{i}$ the point such that $\overrightarrow{O B_{i}}=\overrightarrow{A_{i-1} A_{i}}$, $i=1,2, \ldots, n$ (where $A_{0}=A_{n}$ ), then $B_{i}$ has integer coordinates and $B_{1} B_{2} \ldots B_{n}$ is a regular polygon of side length $2 a \sin (\pi / n)2$, then setting in (i) $x=w-2, y=2$, we get $f(w)=f((w-$ 2) $f(w)) f(2)=0$. Thus + +$$ +f(x)=0 \quad \text { if and only if } \quad x \geq 2 +$$ + +Now let $0 \leq y<2$ and $x \geq 0$. The LHS in (i) is zero if and only if $x f(y) \geq 2$, while the RHS is zero if and only if $x+y \geq 2$. It follows that $x \geq 2 / f(y)$ if and only if $x \geq 2-y$. Therefore + +$$ +f(y)=\left\{\begin{array}{cl} +\frac{2}{2-y} & \text { for } 0 \leq y<2 \\ +0 & \text { for } y \geq 2 +\end{array}\right. +$$ + +The confirmation that $f$ satisfies the given conditions is straightforward. +2. No. If $a$ were rational, its decimal expansion would be periodic from some point. Let $p$ be the number of decimals in the period. Since $f\left(10^{2 p}\right)$ has $2 n p$ zeros, it contains a full periodic part; hence the period would consist only of zeros, which is impossible. +3. Let $E$ be the point where the boy turned westward, reaching the shore at $D$. Let the ray $D E$ cut $A C$ at $F$ and the shore again at $G$. Then $E F=$ $A E=x$ (because $A E F$ is an equilateral triangle) and $F G=D E=y$. From $A E \cdot E B=D E \cdot E G$ we obtain $x(86-x)=y(x+y)$. If $x$ is odd, then $x(86-x)$ is odd, while $y(x+y)$ is even. Hence $x$ is even, and so $y$ must also be even. Let $y=2 y_{1}$. The above equation can be rewritten as + +$$ +\left(x+y_{1}-43\right)^{2}+\left(2 y_{1}\right)^{2}=\left(43-y_{1}\right)^{2} \text {. } +$$ + +Since $y_{1}<43$, we have $\left(2 y_{1}, 43-y_{1}\right)=1$, and thus $\left(\left|x+y_{1}-43\right|, 2 y_{1}, 43-\right.$ $\left.y_{1}\right)$ is a primitive Pythagorean triple. Consequently there exist integers $a>b>0$ such that $y_{1}=a b$ and $43-y_{1}=a^{2}+b^{2}$. We obtain that $a^{2}+b^{2}+a b=43$, which has the unique solution $a=6, b=1$. Hence $y=12$ and $x=2$ or $x=72$. +Remark. The Diophantine equation $x(86-x)=y(x+y)$ can be also solved directly. Namely, we have that $x(344-3 x)=(2 y+x)^{2}$ is a square, and since $x$ is even, we have $(x, 344-3 x)=2$ or 4 . Consequently $x, 344-3 x$ are either both squares or both two times squares. The rest is easy. +4. Let $x=p^{\alpha} x^{\prime}, y=p^{\beta} y^{\prime}, z=p^{\gamma} z^{\prime}$ with $p \nmid x^{\prime} y^{\prime} z^{\prime}$ and $\alpha \geq \beta \geq \gamma$. From the given equation it follows that $p^{n}(x+y)=z\left(x y-p^{n}\right)$ and consequently $z^{\prime} \mid x+y$. Since also $p^{\gamma} \mid x+y$, we have $z \mid x+y$, i.e., $x+y=q z$. The given equation together with the last condition gives us + +$$ +x y=p^{n}(q+1) \quad \text { and } \quad x+y=q z . +$$ + +Conversely, every solution of (1) gives a solution of the given equation. + +For $q=1$ and $q=2$ we obtain the following classes of $n+1$ solutions each: + +$$ +\begin{array}{ll} +q=1:(x, y, z)=\left(2 p^{i}, p^{n-i}, 2 p^{i}+p^{n-i}\right) & \text { for } i=0,1,2, \ldots, n \\ +q=2:(x, y, z)=\left(3 p^{j}, p^{n-j}, \frac{3 p^{j}+p^{n-j}}{2}\right) & \text { for } j=0,1,2, \ldots, n +\end{array} +$$ + +For $n=2 k$ these two classes have a common solution $\left(2 p^{k}, p^{k}, 3 p^{k}\right)$; otherwise, all these solutions are distinct. One further solution is given by $(x, y, z)=\left(1, p^{n}\left(p^{n}+3\right) / 2, p^{2}+2\right)$, not included in the above classes for $p>3$. Thus we have found $2(n+1)$ solutions. +Another type of solution is obtained if we put $q=p^{k}+p^{n-k}$. This yields the solutions + +$$ +(x, y, z)=\left(p^{k}, p^{n}+p^{n-k}+p^{2 n-2 k}, p^{n-k}+1\right) \quad \text { for } k=0,1, \ldots, n +$$ + +For $k0$. +13. Let us consider the infinite integer lattice and assume that having reached a point $(x, n)$ or $(n, y)$, the particle continues moving east and north following the rules of the game. The required probability $p_{k}$ is equal to the probability of getting to one of the points $E_{1}(n, n+k), E_{2}(n+k, n)$, but without passing through $(n, n+k-1)$ or $(n+k-1, n)$. Thus $p$ is equal to the probability $p_{1}$ of getting to $E_{1}(n, n+k)$ via $D_{1}(n-1, n+k)$ plus the probability $p_{2}$ of getting to $E_{2}(n+k, n)$ via $D_{2}(n+k, n-1)$. Both $p_{1}$ and $p_{2}$ are easily seen to be equal to $\binom{2 n+k-1}{n-1} 2^{-2 n-k}$, and therefore $p=\binom{2 n+k-1}{n-1} 2^{-2 n-k+1}$. +14. We shall use the following simple fact. + +Lemma. If $\widehat{k}$ is the image of a circle $k$ under an inversion centered at a point $Z$, and $O_{1}, O_{2}$ are centers of $k$ and $\widehat{k}$, then $O_{1}, O_{2}$, and $Z$ are collinear. +Proof. The result follows immediately from the symmetry with respect to the line $Z O_{1}$. +Let $I$ be the center of the inscribed circle $i$. Since $I X \cdot I A=I E^{2}$, the inversion with respect to $i$ takes points $A$ into $X$, and analogously $B, C$ into $Y, Z$ respectively. It follows from the lemma that the center of circle $A B C$, the center of circle $X Y Z$, and point $I$ are collinear. +15. (a) This is the same problem as $S L 82$-14. +(b) If $S$ is the midpoint of $A C$, we have $B^{\prime} S=A C \frac{\cos \angle D}{2 \sin \angle D}, D^{\prime} S=$ $A C \frac{\cos \angle B}{2 \sin \angle B}, B^{\prime} D^{\prime}=A C\left|\frac{\sin (\angle B+\angle D)}{2 \sin \angle B \sin \angle D}\right|$. These formulas are true also if $\angle B>90^{\circ}$ or $\angle D>90^{\circ}$. We similarly obtain that $A^{\prime \prime} C^{\prime \prime}=$ $B^{\prime} D^{\prime}\left|\frac{\sin \left(\angle A^{\prime}+\angle C^{\prime}\right)}{2 \sin \angle A^{\prime} \sin \angle C^{\prime}}\right|$. Therefore + +$$ +A^{\prime \prime} C^{\prime \prime}=A C \frac{\sin ^{2}(\angle A+\angle C)}{4 \sin \angle A \sin \angle B \sin \angle C \sin \angle D} +$$ + +16. Let $Z$ be the center of the polygon. + +Suppose that at some moment we have $A \in P_{i-1} P_{i}$ and $B \in$ $P_{i} P_{i+1}$, where $P_{i-1}, P_{i}, P_{i+1}$ are adjacent vertices of the polygon. Since $\angle A O B=180^{\circ}-\angle P_{i-1} P_{i} P_{i+1}$, the quadrilateral $A P_{i} B O$ is cyclic. Hence $\angle A P_{i} O=\angle A B O=\angle A P_{i} Z$, which means that $O \in P_{i} Z$. +![](https://cdn.mathpix.com/cropped/2024_11_18_8e985d6b9c83aa3e9d0eg-495.jpg?height=329&width=390&top_left_y=251&top_left_x=903) + +Moreover, from $O P_{i}=2 r \sin \angle P_{i} A O$, where $r$ is the radius of circle $A P_{i} B O$, we obtain that $Z P_{i} \leq O P_{i} \leq Z P_{i} / \cos (\pi / n)$. Thus $O$ traces a segment $Z Z_{i}$ as $A$ and $B$ move along $P_{i-1} P_{i}$ and $P_{i} P_{i+1}$ respectively, where $Z_{i}$ is a point on the ray $P_{i} Z$ with $P_{i} Z_{i} \cos (\pi / n)=P_{i} Z$. When $A, B$ move along the whole circumference of the polygon, $O$ traces an asterisk consisting of $n$ segments of equal length emanating from $Z$ and pointing away from the vertices. +17. We use complex numbers to represent the position of a point in the plane. For convenience, let $A_{1}, A_{2}, A_{3}, A_{4}, A_{5}, \ldots$ be $A, B, C, A, B, \ldots$ respectively, and let $P_{0}$ be the origin. After the $k$ th step, the position of $P_{k}$ will be $P_{k}=A_{k}+\left(P_{k-1}-A_{k}\right) u, k=1,2,3, \ldots$, where $u=e^{4 \pi \tau / 3}$. We easily obtain + +$$ +P_{k}=(1-u)\left(A_{k}+u A_{k-1}+u^{2} A_{k-2}+\cdots+u^{k-1} A_{1}\right) +$$ + +The condition $P_{0} \equiv P_{1986}$ is equivalent to $A_{1986}+u A_{1985}+\cdots+u^{1984} A_{2}+$ $u^{1985} A_{1}=0$, which, having in mind that $A_{1}=A_{4}=A_{7}=\cdots, A_{2}=A_{5}=$ $A_{8}=\cdots, A_{3}=A_{6}=A_{9}=\cdots$, reduces to + +$$ +662\left(A_{3}+u A_{2}+u^{2} A_{1}\right)=\left(1+u^{3}+\cdots+u^{1983}\right)\left(A_{3}+u A_{2}+u^{2} A_{1}\right)=0 +$$ + +It follows that $A_{3}-A_{1}=u\left(A_{1}-A_{2}\right)$, and the assertion follows. +Second solution. Let $f_{P}$ denote the rotation with center $P$ through $120^{\circ}$ clockwise. Let $f_{1}=f_{A}$. Then $f_{1}\left(P_{0}\right)=P_{1}$. Let $B^{\prime}=f_{1}(B), C^{\prime}=f_{1}(C)$, and $f_{2}=f_{B^{\prime}}$. Then $f_{2}\left(P_{1}\right)=P_{2}$ and $f_{2}\left(A B^{\prime} C^{\prime}\right)=A^{\prime} B^{\prime} C^{\prime \prime}$. Finally, let $f_{3}=f_{C^{\prime \prime}}$ and $f_{3}\left(A^{\prime} B^{\prime} C^{\prime \prime}\right)=A^{\prime \prime} B^{\prime \prime} C^{\prime \prime}$. Then $g=f_{3} f_{2} f_{1}$ is a translation sending $P_{0}$ to $P_{3}$ and $C$ to $C^{\prime \prime}$. Now $P_{1986}=P_{0}$ implies that $g^{662}$ is the identity, and thus $C=C^{\prime \prime}$. +Let $K$ be such that $A B K$ is equilateral and positively oriented. We observe that $f_{2} f_{1}(K)=K$; therefore the rotation $f_{2} f_{1}$ satisfies $f_{2} f_{1}(P) \neq P$ for $P \neq K$. Hence $f_{2} f_{1}(C)=C^{\prime \prime}=C$ implies $K=C$. +18. We shall use the following criterion for a quadrangle to be circumscribable. Lemma. The quadrangle $A Y D Z$ is circumscribable if and only if $D B-$ $D C=A B-A C$. +Proof. Suppose that $A Y D Z$ is circumscribable and that the incircle is tangent to $A Z, Z D, D Y, Y A$ at $M, N, P, Q$ respectively. Then $D B-D C=P B-N C=M B-Q C=A B-A C$. Conversely, assume +that $D B-D C=A B-A C$ and let a tangent from $D$ to the incircle of the triangle $A C Z$ meet $C Z$ and $C A$ at $D^{\prime} \neq Z$ and $Y^{\prime} \neq A$ respectively. According to the first part we have $D^{\prime} B-D^{\prime} C=A B-A C$. It follows that $\left|D^{\prime} B-D B\right|=$ +![](https://cdn.mathpix.com/cropped/2024_11_18_8e985d6b9c83aa3e9d0eg-496.jpg?height=315&width=387&top_left_y=236&top_left_x=900) +$\left|D^{\prime} C-D C\right|=D D^{\prime}$, implying that $D^{\prime} \equiv D$. +Let us assume that $D Z B X$ and $D X C Y$ are circumscribable. Using the lemma we obtain $D C-D A=B C-B A$ and $D A-D B=C A-C B$. Adding these two inequalities yields $D C-D B=A C-A B$, and the statement follows from the lemma. +19. Let $M$ and $N$ be the midpoints of segments $A B$ and $C D$, respectively. The given conditions imply that $\triangle A B D \cong \triangle B A C$ and $\triangle C D A \cong \triangle D C B$; hence $M C=M D$ and $N A=N B$. It follows that $M$ and $N$ both lie on the perpendicular bisectors of $A B$ and $C D$, and consequently $M N$ is the common perpendicular bisector of $A B$ and $C D$. Points $B$ and $C$ are symmetric to $A$ and $D$ with respect to $M N$. Now if $P$ is a point in space and $P^{\prime}$ the point symmetric to $P$ with respect to $M N$, we have $B P=A P^{\prime}, C P=D P^{\prime}$, and thus $f(P)=A P+A P^{\prime}+D P+D P^{\prime}$. Let $P P^{\prime}$ intersect $M N$ in $Q$. Then $A P+A P^{\prime} \geq 2 A Q$ and $D P+D P^{\prime} \geq 2 D Q$, from which it follows that $f(P) \geq 2(A Q+D Q)=f(Q)$. It remains to minimize $f(Q)$ with $Q$ moving along the line $M N$. +Let us rotate point $D$ around $M N$ to a point $D^{\prime}$ that belongs to the plane $A M N$, on the side of $M N$ opposite to $A$. Then $f(Q)=2\left(A Q+D^{\prime} Q\right) \geq$ $A D^{\prime}$, and equality occurs when $Q$ is the intersection of $A D^{\prime}$ and $M N$. Thus $\min f(Q)=A D^{\prime}$. We note that $4 M D^{2}=2 A D^{2}+2 B D^{2}-A B^{2}=$ $2 a^{2}+2 b^{2}-A B^{2}$ and $4 M N^{2}=4 M D^{2}-C D^{2}=2 a^{2}+2 b^{2}-A B^{2}-C D^{2}$. Now, $A D^{\prime 2}=\left(A M+D^{\prime} N\right)^{2}+M N^{2}$, which together with $A M+D^{\prime} N=$ $(a+b) / 2$ gives us + +$$ +A D^{\prime 2}=\frac{a^{2}+b^{2}+A B \cdot C D}{2}=\frac{a^{2}+b^{2}+c^{2}}{2} +$$ + +We conclude that $\min f(Q)=\sqrt{\left(a^{2}+b^{2}+c^{2}\right) / 2}$. +20. If the faces of the tetrahedron $A B C D$ are congruent triangles, we must have $A B=C D, A C=B D$, and $A D=B C$. Then the sum of angles at $A$ is $\angle B A C+\angle C A D+\angle D A B=\angle B D C+\angle C B D+\angle D C B=180^{\circ}$. +We now assume that the sum of angles at each vertex is $180^{\circ}$. Let us construct triangles $B C D^{\prime}, C A D^{\prime \prime}, A B D^{\prime \prime \prime}$ in the plane $A B C$, exterior to $\triangle A B C$, such that $\triangle B C D^{\prime} \cong \triangle B C D, \triangle C A D^{\prime \prime} \cong \triangle C A D$, and $\triangle A B D^{\prime \prime \prime} \cong \triangle A B D$. Then by the assumption, $A \in D^{\prime \prime} D^{\prime \prime \prime}, B \in D^{\prime \prime \prime} D^{\prime}$, and $C \in D^{\prime} D^{\prime \prime}$. Since also $D^{\prime \prime} A=D^{\prime \prime \prime} A=D A$, etc., $A, B, C$ are the mid- +points of segments $D^{\prime \prime} D^{\prime \prime \prime}, D^{\prime \prime \prime} D^{\prime}, D^{\prime} D^{\prime \prime}$ respectively. Thus the triangles $A B C, B C D^{\prime}, C A D^{\prime \prime}, A B D^{\prime \prime \prime}$ are congruent, and the statement follows. +21. Since the sum of all edges of $A B C D$ is 3 , the statement of the problem is an immediate consequence of the following statement: +Lemma. Let $r$ be the inradius of a triangle with sides $a, b, c$. Then $a+$ $b+c \geq 6 \sqrt{3} \cdot r$, with equality if and only if the triangle is equilateral. Proof. If $S$ and $p$ denotes the area and semiperimeter of the triangle, by Heron's formula and the AM-GM inequality we have + +$$ +\begin{aligned} +p r & =S=\sqrt{p(p-a)(p-b)(p-c)} \\ +& \leq \sqrt{p\left(\frac{(p-a)+(p-b)+(p-c)}{3}\right)^{3}}=\sqrt{\frac{p^{4}}{27}}=\frac{p^{2}}{3 \sqrt{3}}, +\end{aligned} +$$ + +i.e., $p \geq 3 \sqrt{3} \cdot r$, which is equivalent to the claim. + +### 4.28 Solutions to the Shortlisted Problems of IMO 1987 + +1. By (ii), $f(x)=0$ has at least one solution, and there is the greatest among them, say $x_{0}$. Then by (v), for any $x$, + +$$ +0=f(x) f\left(x_{0}\right)=f\left(x f\left(x_{0}\right)+x_{0} f(x)-x_{0} x\right)=f\left(x_{0}(f(x)-x)\right) +$$ + +It follows that $x_{0} \geq x_{0}(f(x)-x)$. +Suppose $x_{0}>0$. By (i) and (iii), since $f\left(x_{0}\right)-x_{0}<00 +\end{aligned} +$$ + +if not all $x_{1}, x_{2}, \ldots, x_{2 n}$ are zero, which is a contradiction. Hence $k \geq$ $2 n$. +Remark. The condition $n \geq 4$ is essential. For a party attended by 3 couples $\{(1,4),(2,5),(3,6)\}$, there is a collection of 4 cliques satisfying the conditions: $\{(1,2,3),(3,4,5),(5,6,1),(2,4,6)\}$. +3. The answer: yes. Set + +$$ +p(k, m)=k+[1+2+\cdots+(k+m)]=\frac{(k+m)^{2}+3 k+m}{2} . +$$ + +It is obviously of the desired type. +4. Setting $x_{1}=\overrightarrow{A B}, x_{2}=\overrightarrow{A D}, x_{3}=\overrightarrow{A E}$, we have to prove that + +$$ +\left\|x_{1}+x_{2}\right\|+\left\|x_{2}+x_{3}\right\|+\left\|x_{3}+x_{1}\right\| \leq\left\|x_{1}\right\|+\left\|x_{2}\right\|+\left\|x_{3}\right\|+\left\|x_{1}+x_{2}+x_{3}\right\| . +$$ + +We have + +$$ +\begin{aligned} +& \left(\left\|x_{1}\right\|+\left\|x_{2}\right\|+\left\|x_{3}\right\|\right)^{2}-\left\|x_{1}+x_{2}+x_{3}\right\|^{2} \\ +& \quad=2 \sum_{1 \leq i0$ will occur exactly $p_{n-1}(k-1)$ times, so that the sum of the $d_{i}$ 's is $\sum_{k=1}^{n} k p_{n-1}(k-1)=\sum_{k=0}^{n-1}(k+$ 1) $p_{n-1}(k)=2(n-1)$ !. Summation over $i$ yields + +$$ +Z=\sum_{k=0}^{n} k^{2} p_{n}(k)=2 n!. +$$ + +From (0), (1), and (2), we conclude that + +$$ +\sum_{k=0}^{n}(k-1)^{2} p_{n}(k)=\sum_{k=0}^{n} k^{2} p_{n}(k)-2 \sum_{k=0}^{n} k p_{n}(k)+\sum_{k=0}^{n} p_{n}(k)=n! +$$ + +Remark. Only the first part of this problem was given on the IMO. +17. The number of 4 -colorings of the set $M$ is equal to $4^{1987}$. Let $A$ be the number of arithmetic progressions in $M$ with 10 terms. The number of colorings containing a monochromatic arithmetic progression with 10 terms is less than $4 A \cdot 4^{1977}$. So, if $A<4^{9}$, then there exist 4 -colorings with the required property. + +Now we estimate the value of $A$. If the first term of a 10-term progression is $k$ and the difference is $d$, then $1 \leq k \leq 1978$ and $d \leq\left[\frac{1987-k}{9}\right]$; hence + +$$ +A=\sum_{k=1}^{1978}\left[\frac{1987-k}{9}\right]<\frac{1986+1985+\cdots+9}{9}=\frac{1995 \cdot 1978}{18}<4^{9} +$$ + +18. Note first that the statement that some $a+x, a+y, a+x+y$ belong to a class $C$ is equivalent to the following statement: +(1) There are positive integers $p, q \in C$ such that $p2 k$. +19. The facts given in the problem allow us to draw a triangular pyramid with angles $2 \alpha, 2 \beta, 2 \gamma$ at the top and lateral edges of length $1 / 2$. At the base there is a triangle whose side lengths are exactly $\sin \alpha, \sin \beta, \sin \gamma$. The area of this triangle does not exceed the sum of areas of the lateral sides, which equals $(\sin 2 \alpha+\sin 2 \beta+\sin 2 \gamma) / 8$. +20. Let $y$ be the smallest nonnegative integer with $y \leq p-2$ for which $f(y)$ is a composite number. Denote by $q$ the smallest prime divisor of $f(y)$. We claim that $y2 y$. Suppose the contrary, that $q \leq 2 y$. Since + +$$ +f(y)-f(x)=(y-x)(y+x+1) +$$ + +we observe that $f(y)-f(q-1-y)=(2 y-q+1) q$, from which it follows that $f(q-1-y)$ is divisible by $q$. But by the assumptions, $q-1-yq+p-y-1 \geq q . +$$ + +Therefore $q \geq 2 y+1$. Now, since $f(y)$, being composite, cannot be equal to $q$, and $q$ is its smallest prime divisor, we obtain that $f(y) \geq q^{2}$. Consequently, + +$$ +y^{2}+y+p \geq q^{2} \geq(2 y+1)^{2}=4 y^{2}+4 y+1 \Rightarrow 3\left(y^{2}+y\right) \leq p-1 +$$ + +and from this we easily conclude that $y<\sqrt{p / 3}$, which contradicts the condition of the problem. In this way, all the numbers + +$$ +f(0), f(1), \ldots, f(p-2) +$$ + +must be prime. +21. Let $P$ be the second point of intersection of segment $B C$ and the circle circumscribed about quadrilateral $A K L M$. Denote by $E$ the intersection point of the lines $K N$ and $B C$ and by $F$ the intersection point of the lines $M N$ and $B C$. Then $\angle B C N=\angle B A N$ and $\angle M A L=$ $\angle M P L$, as angles on the same arc. Since $A L$ is a bisector, $\angle B C N=$ $\angle B A L=\angle M A L=\angle M P L$, and +![](https://cdn.mathpix.com/cropped/2024_11_18_8e985d6b9c83aa3e9d0eg-507.jpg?height=452&width=414&top_left_y=813&top_left_x=866) +consequently $P M \| N C$. Similarly we prove $K P \| B N$. Then the quadrilaterals $B K P N$ and $N P M C$ are trapezoids; hence + +$$ +S_{B K E}=S_{N P E} \quad \text { and } \quad S_{N P F}=S_{C M F} +$$ + +Therefore $S_{A B C}=S_{A K N M}$. +22. Suppose that there exists such function $f$. Then we obtain + +$$ +f(n+1987)=f(f(f(n)))=f(n)+1987 \quad \text { for all } n \in \mathbb{N} +$$ + +and from here, by induction, $f(n+1987 t)=f(n)+1987 t$ for all $n, t \in \mathbb{N}$. Further, for any $r \in\{0,1, \ldots, 1986\}$, let $f(r)=1987 k+l, k, l \in \mathbb{N}$, $l \leq 1986$. We have + +$$ +r+1987=f(f(r))=f(l+1987 k)=f(l)+1987 k, +$$ + +and consequently there are two possibilities: +(i) $k=1 \Rightarrow f(r)=l+1987$ and $f(l)=r$; +(ii) $k=0 \Rightarrow f(r)=l$ and $f(l)=r+1987$; +in both cases, $r \neq l$. In this way, the set $\{0,1, \ldots, 1986\}$ decomposes to pairs $\{a, b\}$ such that + +$$ +f(a)=b \text { and } f(b)=a+1987, \quad \text { or } \quad f(b)=a \text { and } f(a)=b+1987 . +$$ + +But the set $\{0,1, \ldots, 1986\}$ has an odd number of elements, and cannot be decomposed into pairs. Contradiction. +23. If we prove the existence of $p, q \in \mathbb{N}$ such that the roots $r, s$ of + +$$ +f(x)=x^{2}-k p \cdot x+k q=0 +$$ + +are irrational real numbers with $01$ ), then we are done, because from $r+s, r s \equiv 0(\bmod k)$ we get $r^{m}+s^{m} \equiv 0$ $(\bmod k)$, and $00>f(1)$, i.e., + +$$ +k q>0>k(q-p)+1 \quad \Rightarrow \quad p>q>0 +$$ + +The irrationality of $r$ can be obtained by taking $q=p-1$, because the discriminant $D=(k p)^{2}-4 k p+4 k$, for $(k p-2)^{2}1$. From the equalities $b_{n}=p b_{n-1}+q b_{n-2}$, $b_{n+1}=p b_{n}+q b_{n-1}, b_{n+2}=p b_{n+1}+q b_{n}$, eliminating $b_{n+1}$ and $b_{n-1}$ we obtain that $b_{n+2}=\left(p^{2}+2 q\right) b_{n}-q^{2} b_{n-2}$. So the sequence $b_{0}, b_{2}, b_{4}, \ldots$ has the property + +$$ +b_{2 n}=P b_{2 n-2}+Q b_{2 n-4}, \quad P=p^{2}+2 q, \quad Q=-q^{2} . +$$ + +We shall solve the problem by induction. The sequence $a_{n}$ has $p=2$, $q=1$, and hence $P=6, Q=-1$. +Let $k=1$. Then $a_{0}=0, a_{1}=1$, and $a_{n}$ is of the same parity as $a_{n-2}$; i.e., it is even if and only if $n$ is even. +Let $k \geq 1$. We assume that for $n=2^{k} m$, the numbers $a_{n}$ are divisible by $2^{k}$, but divisible by $2^{k+1}$ if and only if $m$ is even. We assume also that the sequence $c_{0}, c_{1}, \ldots$, with $c_{m}=a_{m \cdot 2^{k}}$, satisfies the condition $c_{n}=$ $p c_{n-1}-c_{n-2}$, where $p \equiv 2(\bmod 4)($ for $k=1$ it is true). We shall prove the same statement for $k+1$. According to (1), $c_{2 n}=P c_{2 n-2}-c_{2 n-4}$, where $P=p^{2}-2$. Obviously $P \equiv 2(\bmod 4)$. Since $P=4 s+2$ for some integer $s$, and $c_{2 n}=2^{k+1} d_{2 n}, c_{0}=0, c_{1} \equiv 2^{k}\left(\bmod 2^{k+1}\right)$, and $c_{2}=p c_{1} \equiv 2^{k+1}$ $\left(\bmod 2^{k+2}\right)$, we have + +$$ +c_{2 n}=(4 s+2) 2^{k+1} d_{2 n-2}-c_{2 n-4} \equiv c_{2 n-4}\left(\bmod 2^{k+2}\right) +$$ + +i.e., $0 \equiv c_{0} \equiv c_{4} \equiv c_{8} \equiv \cdots$ and $2^{k+1} \equiv c_{2} \equiv c_{6} \equiv \cdots\left(\bmod 2^{k+2}\right)$, which proves the statement. +Second solution. The recursion is solved by + +$$ +a_{n}=\frac{1}{2 \sqrt{2}}\left((1+\sqrt{2})^{n}-(1-\sqrt{2})^{n}\right)=\binom{n}{1}+2\binom{n}{3}+2^{2}\binom{n}{5}+\cdots . +$$ + +Let $n=2^{k} m$ with $m$ odd; then for $p>0$ the summand + +$$ +2^{p}\binom{n}{2 p+1}=2^{k+p} m \frac{(n-1) \ldots(n-2 p)}{(2 p+1)!}=2^{k+p} \frac{m}{2 p+1}\binom{n-1}{2 p} +$$ + +is divisible by $2^{k+p}$, because the denominator $2 p+1$ is odd. Hence + +$$ +a_{n}=n+\sum_{p>0} 2^{p}\binom{n}{2 p+1}=2^{k} m+2^{k+1} N +$$ + +for some integer $N$, so that $a_{n}$ is exactly divisible by $2^{k}$. +Third solution. It can be proven by induction that $a_{2 n}=2 a_{n}\left(a_{n}+a_{n+1}\right)$. The required result follows easily, again by induction on $k$. +2. For polynomials $f(x), g(x)$ with integer coefficients, we use the notation $f(x) \sim g(x)$ if all the coefficients of $f-g$ are even. Let $n=2^{s}$. It is immediately shown by induction that $\left(x^{2}+x+1\right)^{2^{s}} \sim x^{2^{s+1}}+x^{2^{s}}+1$, and the required number for $n=2^{s}$ is 3 . + +Let $n=2^{s}-1$. If $s$ is odd, then $n \equiv 1(\bmod 3)$, while for $s$ even, $n \equiv 0$ $(\bmod 3)$. Consider the polynomial + +$$ +R_{s}(x)= \begin{cases}(x+1)\left(x^{2 n-1}+x^{2 n-4}+\cdots+x^{n+3}\right)+x^{n+1} & \\ +x^{n}+x^{n-1}+(x+1)\left(x^{n-4}+x^{n-7}+\cdots+1\right), & 2 \nmid s \\ (x+1)\left(x^{2 n-1}+x^{2 n-4}+\cdots+x^{n+2}\right)+x^{n} & \\ +(x+1)\left(x^{n-3}+x^{n-6}+\cdots+1\right), & 2 \mid s\end{cases} +$$ + +It is easily checked that $\left(x^{2}+x+1\right) R_{s}(x) \sim x^{2^{s+1}}+x^{2^{s}}+1 \sim\left(x^{2}+x+1\right)^{2^{s}}$, so that $R_{s}(x) \sim\left(x^{2}+x+1\right)^{2^{s}-1}$. In this case, the number of odd coefficients is $\left(2^{s+2}-(-1)^{s}\right) / 3$. +Now we pass to the general case. Let the number $n$ be represented in the binary system as + +$$ +n=\underbrace{11 \ldots 1}_{a_{k}} \underbrace{00 \ldots 0}_{b_{k}} \underbrace{11 \ldots 1}_{a_{k-1}} \underbrace{00 \ldots 0}_{b_{k-1}} \cdots \underbrace{11 \ldots 1}_{a_{1}} \underbrace{00 \ldots 0}_{b_{1}} +$$ + +$b_{i}>0(i>1), b_{1} \geq 0$, and $a_{i}>0$. Then $n=\sum_{i=1}^{k} 2^{s_{i}}\left(2^{a_{i}}-1\right)$, where $s_{i}=b_{1}+a_{1}+b_{2}+a_{2}+\cdots+b_{i}$, and hence + +$$ +u_{n}(x)=\left(x^{2}+x+1\right)^{n}=\prod_{i=1}^{k}\left(x^{2}+x+1\right)^{2^{s_{i}}\left(2^{a_{i}}-1\right)} \sim \prod_{i=1}^{k} R_{a_{i}}\left(x^{2^{s_{i}}}\right) +$$ + +Let $R_{a_{i}}\left(x^{2^{s_{i}}}\right) \sim x^{r_{i, 1}}+\cdots+x^{r_{i, d_{i}}}$; clearly $r_{i, j}$ is divisible by $2^{s_{i}}$ and $r_{i, j} \leq 2^{s_{i}+1}\left(2^{a_{i}}-1\right)<2^{s_{i+1}}$, so that for any $j, r_{i, j}$ can have nonzero binary digits only in some position $t, s_{i} \leq t \leq s_{i+1}-1$. Therefore, in + +$$ +\prod_{i=1}^{k} R_{a_{i}}\left(x^{2^{s_{i}}}\right) \sim \prod_{i=1}^{k}\left(x^{r_{i, 1}}+\cdots+x^{r_{i, d_{i}}}\right)=\sum_{i=1}^{k} \sum_{p_{i}=1}^{d_{i}} x^{r_{1, p_{1}}+r_{2, p_{2}}+\cdots+r_{k, p_{k}}} +$$ + +all the exponents $r_{1, p_{1}}+r_{2, p_{2}}+\cdots+r_{k, p_{k}}$ are different, so that the number of odd coefficients in $u_{n}(x)$ is + +$$ +\prod_{i=1}^{k} d_{i}=\prod_{i=1}^{k} \frac{2^{a_{i}+2}-(-1)^{a_{i}}}{3} +$$ + +3. Let $R$ be the circumradius, $r$ the inradius, $s$ the semiperimeter, $\Delta$ the area of $A B C$ and $\Delta^{\prime}$ the area of $A^{\prime} B^{\prime} C^{\prime}$. The angles of triangle $A^{\prime} B^{\prime} C^{\prime}$ are $A^{\prime}=90^{\circ}-A / 2, B^{\prime}=90^{\circ}-B / 2$, and $C^{\prime}=90^{\circ}-C / 2$, and hence + +$$ +\Delta=2 R^{2} \sin A \sin B \sin C +$$ + +and $\Delta^{\prime}=2 R^{2} \sin A^{\prime} \sin B^{\prime} \sin C^{\prime}=2 R^{2} \cos \frac{A}{2} \cos \frac{B}{2} \cos \frac{C}{2}$. +Hence, + +$$ +\frac{\Delta}{\Delta^{\prime}}=\frac{\sin A \sin B \sin C}{\cos \frac{A}{2} \cos \frac{B}{2} \cos \frac{C}{2}}=8 \sin \frac{A}{2} \sin \frac{B}{2} \sin \frac{C}{2}=\frac{2 r}{R} +$$ + +where we have used that $r=A I \sin (A / 2)=\cdots=4 R \sin (A / 2) \cdot \sin (B / 2)$. $\sin (C / 2)$. Euler's inequality $2 r \leq R$ shows that $\Delta \leq \Delta^{\prime}$. +Second solution. Let $H$ be orthocenter of triangle $A B C$, and $H_{a}, H_{b}, H_{c}$ points symmetric to $H$ with respect to $B C, C A, A B$, respectively. Since $\angle B H_{a} C=\angle B H C=180^{\circ}-\angle A$, points $H_{a}, H_{b}, H_{c}$ lie on the circumcircle of $A B C$, and the area of the hexagon $A H_{c} B H_{a} C H_{b}$ is double the area of $A B C$. (1) +Let us apply the analogous result for the triangle $A^{\prime} B^{\prime} C^{\prime}$. Since its orthocenter is the incenter $I$ of $A B C$, and the point symmetric to $I$ with respect to $B^{\prime} C^{\prime}$ is the point $A$, we find by (1) that the area of the hexagon $A C^{\prime} B A^{\prime} C B^{\prime}$ is double the area of $A^{\prime} B^{\prime} C^{\prime}$. +But it is clear that the area of $\Delta C H_{a} B$ is less than or equal to the area of $\Delta C A^{\prime} B$ etc.; hence, the area of $A H_{c} B H_{a} C H_{b}$ does not exceed the area of $A C^{\prime} B A^{\prime} C B^{\prime}$. The statement follows immediately. +4. Suppose that the numbers of any two neighboring squares differ by at most $n-1$. For $k=1,2, \ldots, n^{2}-n$, let $A_{k}, B_{k}$, and $C_{k}$ denote, respectively, the sets of squares numbered by $1,2, \ldots, k$; of squares numbered by $k+$ $n, k+n+1, \ldots, n^{2}$; and of squares numbered by $k+1, \ldots, k+n-1$. By the assumption, the squares from $A_{k}$ and $B_{k}$ have no edge in common; $C_{k}$ has $n-1$ elements only. Consequently, for each $k$ there exists a row and a column all belonging either to $A_{k}$, or to $B_{k}$. +For $k=1$, it must belong to $B_{k}$, while for $k=n^{2}-n$ it belongs to $A_{k}$. Let $k$ be the smallest index such that $A_{k}$ contains a whole row and a whole column. Since $B_{k-1}$ has that property too, it must have at least two squares in common with $A_{k}$, which is impossible. +5. Let $n=2 k$ and let $A=\left\{A_{1}, \ldots, A_{2 k+1}\right\}$ denote the family of sets with the desired properties. Since every element of their union $B$ belongs to at least two sets of $A$, it follows that $A_{j}=\bigcup_{i \neq j} A_{i} \cap A_{j}$ holds for every $1 \leq j \leq 2 k+1$. Since each intersection in the sum has at most one element and $A_{j}$ has $2 k$ elements, it follows that every element of $A_{j}$, i.e., in general of $B$, is a member of exactly two sets. +We now prove that $k$ is even, assuming that the marking described in the problem exists. We have already shown that for every two indices $1 \leq j \leq 2 k+1$ and $i \neq j$ there exists a unique element contained in both $A_{i}$ and $A_{j}$. On a $2 k \times 2 k$ matrix let us mark in the $i$ th column and $j$ th row for $i \neq j$ the number that was joined to the element of $B$ in $A_{i} \cap A_{j}$. In the $i$ th row and column let us mark the number of the element of $B$ in $A_{i} \cap A_{2 k+1}$. In each row from the conditions of the marking there must be an even number of zeros. Hence, the total number of zeros in the matrix is even. The matrix is symmetric with respect to its main diagonal; hence it has an even number of zeros outside its main diagonal. Hence, the number of zeros on the main diagonal must also be even and this number equals the number of elements in $A_{2 k+1}$ that are marked with 0 , which is $k$. Hence $k$ must be even. + +For even $k$ we note that the dimensions of a $2 k \times 2 k$ matrix are divisible by 4 . Tiling the entire matrix with the $4 \times 4$ submatrix + +$$ +Q=\left[\begin{array}{llll} +0 & 1 & 0 & 1 \\ +1 & 0 & 1 & 0 \\ +0 & 1 & 1 & 0 \\ +1 & 0 & 0 & 1 +\end{array}\right] +$$ + +we obtain a marking that indeed satisfies all the conditions of the problem; hence we have shown that the marking is possible if and only if $k$ is even. +6. Let $\omega$ be the plane through $A B$, parallel to $C D$. Define the point transformation $f: X \mapsto X^{\prime}$ in space as follows. If $X \in K L$, then $X^{\prime}=X$; otherwise, let $\omega_{X}$ be the plane through $X$ parallel to $\omega$ : then $X^{\prime}$ is the point symmetric to $X$ with respect to the intersection point of $K L$ with $\omega_{X}$. Clearly, $f(A)=B, f(B)=A, f(C)=D, f(D)=C$; hence $f$ maps the tetrahedron onto itself. +We shall show that $f$ preserves volumes. Let $s: X \mapsto X^{\prime \prime}$ denote the symmetry with respect to $K L$, and $g$ the transformation mapping $X^{\prime \prime}$ into $X^{\prime}$; then $f=g \circ s$. If points $X_{1}^{\prime \prime}=s\left(X_{1}\right)$ and $X_{2}^{\prime \prime}=s\left(X_{2}\right)$ have the property that $X_{1}^{\prime \prime} X_{2}^{\prime \prime}$ is parallel to $K L$, then the segments $X_{1}^{\prime \prime} X_{2}^{\prime \prime}$ and $X_{1}^{\prime} X_{2}^{\prime}$ have the same length and lie on the same line. Then by Cavalieri's principle $g$ preserves volume, and so does $f$. +Now, if $\alpha$ is any plane containing the line $K L$, the two parts of the tetrahedron on which it is partitioned by $\alpha$ are transformed into each other by $f$, and therefore have the same volumes. +Second solution. Suppose w.l.o.g. that the plane $\alpha$ through $K L$ meets the interiors of edges $A C$ and $B D$ at $X$ and $Y$. Let $\overrightarrow{A X}=\lambda \overrightarrow{A C}$ and $\overrightarrow{B Y}=\mu \overrightarrow{B D}$, for $0 \leq \lambda, \mu \leq 1$. Then the vectors $\overrightarrow{K X}=\lambda \overrightarrow{A C}-\overrightarrow{A B} / 2$, $\overrightarrow{K Y}=\mu \overrightarrow{B D}+\overrightarrow{A B} / 2, \overrightarrow{K L}=\overrightarrow{A C} / 2+$ $\overrightarrow{B D} / 2$ are coplanar; i.e., there exist real numbers $a, b, c$, not all zero, such that +![](https://cdn.mathpix.com/cropped/2024_11_18_8e985d6b9c83aa3e9d0eg-512.jpg?height=426&width=529&top_left_y=1294&top_left_x=829) + +$$ +\overrightarrow{0}=a \overrightarrow{K X}+b \overrightarrow{K Y}+c \overrightarrow{K L}=(\lambda a+c / 2) \overrightarrow{A C}+(\mu b+c / 2) \overrightarrow{B D}+\frac{b-a}{2} \overrightarrow{A B} +$$ + +Since $\overrightarrow{A C}, \overrightarrow{B D}, \overrightarrow{A B}$ are linearly independent, we must have $a=b$ and $\lambda=\mu$. We need to prove that the volume of the polyhedron $K X L Y B C$, which is one of the parts of the tetrahedron $A B C D$ partitioned by $\alpha$, equals half of the volume $V$ of $A B C D$. Indeed, we obtain + +$$ +V_{K X L Y B C}=V_{K X L C}+V_{K B Y L C}=\frac{1}{4}(1-\lambda) V+\frac{1}{4}(1+\mu) V=\frac{1}{2} V . +$$ + +7. The algebraic equation $x^{3}-3 x^{2}+1=0$ admits three real roots $\beta, \gamma, a$, with + +$$ +-0.6<\beta<-0.5, \quad 0.6<\gamma<0.7, \quad \sqrt{8}0$ for some $i$, then $\left|s+v_{i}\right|>|s|$ ), and similarly $x_{1}, \ldots, x_{p} \geq 0$. Finally, suppose by the one-dimensional case that $y_{1}, \ldots, y_{p}$ and $y_{p+1}, \ldots, y_{m}$ are permuted in such a way that all the sums $y_{1}+\cdots+y_{i}$ and $y_{p+1}+\cdots+y_{p+i}$ are $\leq 1$ in absolute value. +We apply the construction of the one-dimensional case to $x_{1}, \ldots, x_{m}$ taking, as described above, positive $z_{i}$ 's from $x_{1}, x_{2}, \ldots, x_{p}$ and negative ones +from $x_{p+1}, \ldots, x_{m}$, but so that the order is preserved; this way we get a permutation $x_{\sigma_{1}}, x_{\sigma_{2}}, \ldots, x_{\sigma_{m}}$. It is then clear that each sum $y_{\sigma_{1}}+y_{\sigma_{2}}+$ $\cdots+y_{\sigma_{k}}$ decomposes into the sum $\left(y_{1}+y_{2}+\cdots+y_{l}\right)+\left(y_{p+1}+\cdots+y_{p+n}\right)$ (because of the preservation of order), and that each of these sums is less than or equal to 1 in absolute value. Thus each sum $u_{\sigma_{1}}+\cdots+u_{\sigma_{k}}$ is composed of a vector of length at most 2 and an orthogonal vector of length at most 1 , and so is itself of length at most $\sqrt{5}$. +9. Let us assume $\frac{a^{2}+b^{2}}{a b+1}=k \in \mathbb{N}$. We then have $a^{2}-k a b+b^{2}=k$. Let us assume that $k$ is not an integer square, which implies $k \geq 2$. Now we observe the minimal pair $(a, b)$ such that $a^{2}-k a b+b^{2}=k$ holds. We may assume w.l.o.g. that $a \geq b$. For $a=b$ we get $k=(2-k) a^{2} \leq 0$; hence we must have $a>b$. +Let us observe the quadratic equation $x^{2}-k b x+b^{2}-k=0$, which has solutions $a$ and $a_{1}$. Since $a+a_{1}=k b$, it follows that $a_{1} \in \mathbb{Z}$. Since $a>k b$ implies $k>a+b^{2}>k b$ and $a=k b$ implies $k=b^{2}$, it follows that $ak$. Since $a a_{1}=b^{2}-k>0$ and $a>0$, it follows that $a_{1} \in \mathbb{N}$ and $a_{1}=\frac{b^{2}-k}{a}<\frac{a^{2}-1}{a}k} a_{i j}+\sum_{j \leq n-k} a_{i j}-\sum_{j>n-k} a_{i j}= \\ +& =2 \sum_{i=1}^{k} \sum_{j=1}^{n-k} a_{i j}-2 \sum_{i=k+1}^{n} \sum_{j=n-k+1}^{n} a_{i j} \leq 4 k(n-k) . +\end{aligned} +$$ + +This yields $n^{2} \leq 4 k(n-k)$, i.e., $(n-2 k)^{2} \leq 0$, and thus $n$ must be even. We proceed to show by induction that for all even $n$ an array of the given type exists. For $n=2$ the array in Fig. 1 is good. Let such an $n \times n$ array be given for some even $n \geq 2$, with $c_{1}=n, c_{2}=-n+1, c_{3}=$ $n-2, \ldots, c_{n-1}=2, c_{n}=-1$ and $r_{1}=n-1, r_{2}=-n+2, \ldots, r_{n-1}=1$, $r_{n}=0$. Upon enlarging this array as indicated in Fig. 2, the positive sums are increased by 2 , the nonpositive sums are decreased by 2 , and the missing sums $-1,0,1,2$ occur in the new rows and columns, so that the obtained array $(n+2) \times(n+2)$ is of the same type. +![](https://cdn.mathpix.com/cropped/2024_11_18_8e985d6b9c83aa3e9d0eg-517.jpg?height=89&width=87&top_left_y=1080&top_left_x=487) + +Fig. 1 + +| $n \times n$ | | | 1 | 1 | +| :---: | :---: | :---: | :---: | :---: | +| | | | | | +| | | | 1 | 1 | +| | | | - | | -1 | +| 1)-1 | 1 | - | 1 | 0 | +| 1)-1 | | 1. | 1 | -1 | + +Fig. 2 +15. Referring to the description of $L_{A}$, we have $\angle A M N=\angle A H N=90^{\circ}-$ $\angle H A C=\angle C$, and similarly $\angle A N M=\angle B$. Since the triangle $A B C$ is acute-angled, the line $L_{A}$ lies inside the angle $A$. Hence if $P=L_{A} \cap B C$ and $Q=L_{B} \cap A C$, we get $\angle B A P=90^{\circ}-\angle C$; hence $A P$ passes through the circumcenter $O$ of $\triangle A B C$. Similarly we prove that $L_{B}$ and $L_{C}$ contains the circumcenter $O$ also. It follows that $L_{A}, L_{B}$ and $L_{C}$ intersect at the point $O$. +Remark. Without identifying the point of intersection, one can prove the concurrence of the three lines using Ceva's theorem, in usual or trigonometric form. +16. Let $f(x)=\sum_{k=1}^{70} \frac{k}{x-k}$. For all integers $i=1, \ldots, 70$ we have that $f(x)$ tends to plus infinity as $x$ tends downward to $i$, and $f(x)$ tends to minus infinity as $x$ tends upward to $i$. As $x$ tends to infinity, $f(x)$ tends to 0 . Hence it follows that there exist $x_{1}, x_{2}, \ldots, x_{70}$ such that $1a_{k}+m \delta$, and consequently $a_{k+m}>1$ for large enough $m$, a contradiction. Thus $a_{k}-a_{k+1} \geq 0$ for all $k$. +Suppose that $a_{k}-a_{k+1}>2 / k^{2}$. Then for all $i2 / k^{2}$, so that $a_{i}-a_{k+1}>2(k+1-i) / k^{2}$, i.e., $a_{i}>2(k+1-i) / k^{2}, i=1,2, \ldots, k$. But this implies $a_{1}+a_{2}+\cdots+a_{k}>2 / k^{2}+4 / k^{2}+\cdots+2 k / k^{2}=k(k+1) / k^{2}$, which is impossible. Therefore $a_{k}-a_{k+1} \leq 2 / k^{2}$ for all $k$. +25. Observe that $1001=7 \cdot 143$, i.e., $10^{3}=-1+7 a, a=143$. Then by the binomial theorem, $10^{21}=(-1+7 a)^{7}=-1+7^{2} b$ for some integer $b$, so that we also have $10^{21 n} \equiv-1(\bmod 49)$ for any odd integer $n>0$. Hence $N=\frac{9}{49}\left(10^{21 n}+1\right)$ is an integer of $21 n$ digits, and $N\left(10^{21 n}+1\right)=$ $\left(\frac{3}{7}\left(10^{21 n}+1\right)\right)^{2}$ is a double number that is a perfect square. +26. The overline in this problem will exclusively denote binary representation. We will show by induction that if $n=\overline{c_{k} c_{k-1} \ldots c_{0}}=\sum_{i=0}^{k} c_{i} 2^{i}$ is the binary representation of $n\left(c_{i} \in\{0,1\}\right)$, then $f(n)=\overline{c_{0} c_{1} \ldots c_{k}}=$ $\sum_{i=0}^{k} c_{i} 2^{k-i}$ is the number whose binary representation is the palindrome of the binary representation of $n$. This evidently holds for $n \in\{1,2,3\}$. + +Let us assume that the claim holds for all numbers up to $n-1$ and show it holds for $n=\overline{c_{k} c_{k-1} \ldots c_{0}}$. We observe three cases: +(i) $c_{0}=0 \Rightarrow n=2 m \Rightarrow f(n)=f(m)=\overline{0 c_{1} \ldots c_{k}}=\overline{c_{0} c_{1} \ldots c_{k}}$. +(ii) $c_{0}=1, c_{1}=0 \Rightarrow n=4 m+1 \Rightarrow f(n)=2 f(2 m+1)-f(m)=$ $2 \cdot \overline{1 c_{2} \ldots c_{k}}-\overline{c_{2} \ldots c_{k}}=2^{k}+2 \cdot \overline{c_{2} \ldots c_{k}}-\overline{c_{2} \ldots c_{k}}=\overline{10 c_{2} \ldots c_{k}}=$ $\overline{c_{0} c_{1} \ldots c_{k}}$. +(iii) $c_{0}=1, c_{1}=1 \Rightarrow n=4 m+3 \Rightarrow f(n)=3 f(2 m+1)-2 f(m)=$ $3 \cdot \overline{1 c_{2} \ldots c_{k}}-2 \cdot \overline{c_{2} \ldots c_{k}}=2^{k}+2^{k-1}+3 \cdot \overline{c_{2} \ldots c_{k}}-2 \cdot \overline{c_{2} \ldots c_{k}}=$ $\overline{11 c_{2} \ldots c_{k}}=\overline{c_{0} c_{1} \ldots c_{k}}$. +We thus have to find the number of palindromes in binary representation smaller than $1998=\overline{11111000100}$. We note that for all $m \in \mathbb{N}$ the numbers of $2 m$ - and $(2 m-1)$-digit binary palindromes are both equal to $2^{m-1}$. We also note that $\overline{11111011111}$ and $\overline{11111111111}$ are the only 11-digit palindromes larger than 1998. Hence we count all palindromes of up to 11 digits and exclude the largest two. The number of $n \leq 1998$ such that $f(n)=n$ is thus equal to $1+1+2+2+4+4+8+8+16+16+32-2=92$. +27. Consider a Cartesian system with the $x$-axis on the line $B C$ and origin at the foot of the perpendicular from $A$ to $B C$, so that $A$ lies on the $y$-axis. Let $A$ be $(0, \alpha), B(-\beta, 0), C(\gamma, 0)$, where $\alpha, \beta, \gamma>0$ (because $A B C$ is acute-angled). Then +$\tan B=\frac{\alpha}{\beta}, \quad \tan C=\frac{\alpha}{\gamma} \quad$ and $\quad \tan A=-\tan (B+C)=\frac{\alpha(\beta+\gamma)}{\alpha^{2}-\beta \gamma} ;$ +here $\tan A>0$, so $\alpha^{2}>\beta \gamma$. Let $L$ have equation $x \cos \theta+y \sin \theta+p=0$. Then + +$$ +\begin{aligned} +& u^{2} \tan A+v^{2} \tan B+w^{2} \tan C \\ +& =\frac{\alpha(\beta+\gamma)}{\alpha^{2}-\beta \gamma}(\alpha \sin \theta+p)^{2}+\frac{\alpha}{\beta}(-\beta \cos \theta+p)^{2}+\frac{\alpha}{\gamma}(\gamma \cos \theta+p)^{2} \\ +& =\left(\alpha^{2} \sin ^{2} \theta+2 \alpha p \sin \theta+p^{2}\right) \frac{\alpha(\beta+\gamma)}{\alpha^{2}-\beta \gamma}+\alpha(\beta+\gamma) \cos ^{2} \theta+\frac{\alpha(\beta+\gamma)}{\beta \gamma} p^{2} \\ +& =\frac{\alpha(\beta+\gamma)}{\beta \gamma\left(\alpha^{2}-\beta \gamma\right)}\left(\alpha^{2} p^{2}+2 \alpha p \beta \gamma \sin \theta+\alpha^{2} \beta \gamma \sin ^{2} \theta+\beta \gamma\left(\alpha^{2}-\beta \gamma\right) \cos ^{2} \theta\right) \\ +& =\frac{\alpha(\beta+\gamma)}{\beta \gamma\left(\alpha^{2}-\beta \gamma\right)}\left[(\alpha p+\beta \gamma \sin \theta)^{2}+\beta \gamma\left(\alpha^{2}-\beta \gamma\right)\right] \geq \alpha(\beta+\gamma)=2 \Delta +\end{aligned} +$$ + +with equality when $\alpha p+\beta \gamma \sin \theta=0$, i.e., if and only if $L$ passes through $(0, \beta \gamma / \alpha)$, which is the orthocenter of the triangle. +28. The sequence is uniquely determined by the conditions, and $a_{1}=2, a_{2}=$ $7, a_{3}=25, a_{4}=89, a_{5}=317, \ldots$; it satisfies $a_{n}=3 a_{n-1}+2 a_{n-2}$ for $n=3,4,5$. We show that the sequence $b_{n}$ given by $b_{1}=2, b_{2}=7$, $b_{n}=3 b_{n-1}+2 b_{n-2}$ has the same inequality property, i.e., that $b_{n}=a_{n}$ : +$b_{n+1} b_{n-1}-b_{n}^{2}=\left(3 b_{n}+2 b_{n-1}\right) b_{n-1}-b_{n}\left(3 b_{n-1}+2 b_{n-2}\right)=-2\left(b_{n} b_{n-2}-b_{n-1}^{2}\right)$ +for $n>2$ gives that $b_{n+1} b_{n-1}-b_{n}^{2}=(-2)^{n-2}$ for all $n \geq 2$. But then + +$$ +\left|b_{n+1}-\frac{b_{n}^{2}}{b_{n-1}}\right|=\frac{2^{n-2}}{b_{n-1}}<\frac{1}{2}, +$$ + +since it is easily shown that $b_{n-1}>2^{n-1}$ for all $n$. It is obvious that $a_{n}=b_{n}$ are odd for $n>1$. +29. Let the first train start from Signal 1 at time 0, and let $t_{j}$ be the time it takes for the $j$ th train in the series to travel from one signal to the next. By induction on $k$, we show that Train $k$ arrives at signal $n$ at time $s_{k}+(n-2) m_{k}$, where $s_{k}=t_{1}+\cdots+t_{k}$ and $m_{k}=\max _{j=1, \ldots, k} t_{j}$. +For $k=1$ the statement is clear. We now suppose that it is true for $k$ trains and for every $n$, and add a $(k+1)$ th train behind the others at Signal 1. There are two cases to consider: +(i) $t_{k+1} \geq m_{k}$, i.e., $m_{k+1}=t_{k+1}$. Then Train $k+1$ leaves Signal 1 when all the others reach Signal 2, which by the induction happens at time $s_{k}$. Since by the induction hypothesis Train $k$ arrives at Signal $i+1$ at time $s_{k}+(i-1) m_{k} \leq s_{k}+(i-1) t_{k+1}$, Train $k+1$ is never forced to stop. The journey finishes at time $s_{k}+(n-1) t_{k+1}=s_{k+1}+(n-2) m_{k+1}$. +(ii) $t_{k+1}0$ and prime number $p, p^{k}$ doesn't divide $k!$. This follows from the fact that the highest exponent $r$ of $p$ for which $p^{r} \mid k$ ! is + +$$ +r=\left[\frac{k}{p}\right]+\left[\frac{k}{p^{2}}\right]+\cdots<\frac{k}{p}+\frac{k}{p^{2}}+\cdots=\frac{k}{p-1}0$, we can write $k=2^{l} k^{\prime}, k^{\prime}$ being odd and $l$ a nonnegative integer. Let us set $v(k)=l$, and define $\beta_{n}=v\left(b_{n}\right), \gamma_{n}=v\left(c_{n}\right)$. We prove the following lemmas: +Lemma 1. For every integer $p \geq 0, b_{2^{p}}$ and $c_{2^{p}}$ are nonzero, and $\beta_{2^{p}}=$ $\gamma_{2^{p}}=p+2$. +Proof. By induction on $p$. For $p=0, b_{1}=4$ and $c_{1}=-4$, so the assertion is true. Suppose that it holds for $p$. Then + +$$ +(1+4 \sqrt[3]{2}-4 \sqrt[3]{4})^{2^{p+1}}=\left(a+2^{p+2}\left(b^{\prime} \sqrt[3]{2}+c^{\prime} \sqrt[3]{4}\right)\right)^{2} \text { with } a, b^{\prime}, \text { and } c^{\prime} \text { odd. } +$$ + +Then we easily obtain that $(1+4 \sqrt[3]{2}-4 \sqrt[3]{4})^{2^{p+1}}=A+2^{p+3}(B \sqrt[3]{2}+$ $C \sqrt[3]{4}$ ), where $A, B=a b^{\prime}+2^{p+1} E, C=a c^{\prime}+2^{p+1} F$ are odd. Therefore Lemma 1 holds for $p+1$. +Lemma 2. Suppose that for integers $n, m \geq 0, \beta_{n}=\gamma_{n}=\lambda>\beta_{m}=$ $\gamma_{m}=\mu$. Then $b_{n+m}, c_{n+m}$ are nonzero and $\beta_{n+m}=\gamma_{n+m}=\mu$. +Proof. Calculating $\left(a^{\prime}+2^{\lambda}\left(b^{\prime} \sqrt[3]{2}+c^{\prime} \sqrt[3]{4}\right)\right)\left(a^{\prime \prime}+2^{\mu}\left(b^{\prime \prime} \sqrt[3]{2}+c^{\prime \prime} \sqrt[3]{4}\right)\right)$, with $a^{\prime}, b^{\prime}, c^{\prime}, a^{\prime \prime}, b^{\prime \prime}, c^{\prime \prime}$ odd, we easily obtain the product $A+2^{\mu}(B \sqrt[3]{2}+$ $C \sqrt[3]{4})$, where $A, B=a^{\prime} b^{\prime \prime}+2^{\lambda-\mu} E$, and $C=a^{\prime} c^{\prime \prime}+2^{\lambda-\mu} F$ are odd, which proves Lemma 2. +Since every integer $n>0$ can be written as $n=2^{p_{r}}+\cdots+2^{p_{1}}$, with $0 \leq p_{1}<\cdots1, S$ can be split into $d$ identical blocks. Let $x_{n}$ be the number of nonrepeating binary sequences of length $n$. The total number of binary sequences of length $n$ is obviously $2^{n}$. Any sequence of length $n$ can be produced by repeating its unique longest nonrepeating initial block according to need. Hence, we obtain the recursion relation $\sum_{d \mid n} x_{d}=2^{n}$. This, along with $x_{1}=2$, gives us $a_{n}=x_{n}$ for all $n$. +We now have that the sequences counted by $x_{n}$ can be grouped into groups of $n$, the sequences in the same group being cyclic shifts of each other. Hence, $n \mid x_{n}=a_{n}$. +12. Assume that each car starts with a unique ranking number. Suppose that while turning back at a meeting point two cars always exchanged their ranking numbers. We can observe that ranking numbers move at a constant speed and direction. One hour later, after several exchanges, each starting point will be occupied by a car of the same ranking number and proceeding in the same direction as the one that started from there one hour ago. +We now give the cars back their original ranking numbers. Since the sequence of the cars along the track cannot be changed, the only possibility is that the original situation has been rotated, maybe onto itself. Hence for some $d \mid n$, after $d$ hours each car will be at its starting position and orientation. +13. Let us construct the circles $\sigma_{1}$ with center $A$ and radius $R_{1}=A D, \sigma_{2}$ with center $B$ and radius $R_{2}=B C$, and $\sigma_{3}$ with center $P$ and radius $x$. The points $C$ and $D$ lie on $\sigma_{2}$ and $\sigma_{1}$ respectively, and $C D$ is tangent to $\sigma_{3}$. From this it is plain that the greatest value of $x$ occurs when $C D$ is also tangent to $\sigma_{1}$ and $\sigma_{2}$. We shall show that in this case the required inequality is really an equality, i.e., that $\frac{1}{\sqrt{x}}=\frac{1}{\sqrt{A D}}+\frac{1}{\sqrt{B C}}$. Then the inequality will immediately follow. +Denote the point of tangency of $C D$ with $\sigma_{3}$ by $M$. By the Pythagorean theorem we have $C D=\sqrt{\left(R_{1}+R_{2}\right)^{2}-\left(R_{1}-R_{2}\right)^{2}}=2 \sqrt{R_{1} R_{2}}$. On the +other hand, $C D=C M+M D=2 \sqrt{R_{2} x}+2 \sqrt{R_{1} x}$. Hence, we obtain $\frac{1}{\sqrt{x}}=\frac{1}{\sqrt{R_{1}}}+\frac{1}{\sqrt{R_{2}}}$. +14. Lemma 1. In a quadrilateral $A B C D$ circumscribed about a circle, with points of tangency $P, Q, R, S$ on $D A, A B, B C, C D$ respectively, the lines $A C, B D, P R, Q S$ concur. +Proof. Follows immediately, for example, from Brianchon's theorem. +Lemma 2. Let a variable chord $X Y$ of a circle $C(I, r)$ subtend a right angle at a fixed point $Z$ within the circle. Then the locus of the midpoint $P$ of $X Y$ is a circle whose center is at the midpoint $M$ of $I Z$ and whose radius is $\sqrt{r^{2} / 2-I Z^{2} / 4}$. +Proof. From $\angle X Z Y=90^{\circ}$ follows $\overrightarrow{Z X} \cdot \overrightarrow{Z Y}=(\overrightarrow{I X}-\overrightarrow{I Z}) \cdot(\overrightarrow{I Y}-\overrightarrow{I Z})=0$. Therefore, + +$$ +\begin{aligned} +\overrightarrow{M P}^{2} & =(\overrightarrow{M I}+\overrightarrow{I P})^{2}=\frac{1}{4}(-\overrightarrow{I Z}+\overrightarrow{I X}+\overrightarrow{I Y})^{2} \\ +& =\frac{1}{4}\left(I X^{2}+I Y^{2}-I Z^{2}+2(\overrightarrow{I X}-\overrightarrow{I Z}) \cdot(\overrightarrow{I Y}-\overrightarrow{I Z})\right) \\ +& =\frac{1}{2} r^{2}-\frac{1}{4} I Z^{2} +\end{aligned} +$$ + +Lemma 3. Using notation as in Lemma 1, if $A B C D$ is cyclic, $P R$ is perpendicular to $Q S$. +Proof. Consider the inversion in $C(I, r)$, mapping $A$ to $A^{\prime}$ etc. $(P, Q, R, S$ are fixed). As is easily seen, $A^{\prime}, B^{\prime}, C^{\prime}, D^{\prime}$ will lie at the midpoints of $P Q, Q R, R S, S P$, respectively. $A^{\prime} B^{\prime} C^{\prime} D^{\prime}$ is a parallelogram, but also cyclic, since inversion preserves circles; thus it must be a rectangle, and so $P R \perp Q S$. +Now we return to the main result. Let $I$ and $O$ be the incenter and circumcenter, $Z$ the intersection of the diagonals, and $P, Q, R, S, A^{\prime}, B^{\prime}, C^{\prime}, D^{\prime}$ points as defined in Lemmas 1 and 3. From Lemma 3, the chords $P Q, Q R, R S, S P$ subtend $90^{\circ}$ at $Z$. Therefore by Lemma 2 the points $A^{\prime}, B^{\prime}, C^{\prime}, D^{\prime}$ lie on a circle whose center is the midpoint $Y$ of $I Z$. Since this circle is the image of the circle $A B C D$ under the considered inversion (centered at $I$ ), it follows that $I, O, Y$ are collinear, and hence so are $I, O, Z$. +Remark. This is the famous Newton's theorem for bicentric quadrilaterals. +15. By Cauchy's inequality, $44<\sqrt{1989}a^{2}+b^{2}+c^{2}=$ $1989-d^{2}$, and so $d^{2}-49 d+206>0$. This inequality does not hold for $5 \leq d \leq 44$. Since $d \geq \sqrt{1989 / 4}>22$, $d$ must be at least 45 , which is impossible because $45^{2}>1989$. Thus we must have $m^{2}=81$ and $m=9$. Now, $4 d>81$ implies $d \geq 21$. On the other hand, $d<\sqrt{1989}$, and hence +$d=25$ or $d=36$. Suppose that $d=25$ and put $a=25-p, b=25-q$, $c=25-r$ with $p, q, r \geq 0$. From $a+b+c=56$ it follows that $p+q+r=19$, which, together with $(25-p)^{2}+(25-q)^{2}+(25-r)^{2}=1364$, gives us $p^{2}+q^{2}+r^{2}=439>361=(p+q+r)^{2}$, a contradiction. Therefore $d=36$ and $n=6$. +Remark. A little more calculation yields the unique solution $a=12$, $b=15, c=18, d=36$. +16. Define $S_{k}=\sum_{i=0}^{k} a_{i}(k=0,1, \ldots, n)$ and $S_{-1}=0$. We note that $S_{n-1}=$ $S_{n}$. Hence + +$$ +\begin{aligned} +S_{n} & =\sum_{k=0}^{n-1} a_{k}=n c+\sum_{k=0}^{n-1} \sum_{i=k}^{n-1} a_{i-k}\left(a_{i}+a_{i+1}\right) \\ +& =n c+\sum_{i=0}^{n-1} \sum_{k=0}^{i} a_{i-k}\left(a_{i}+a_{i+1}\right)=n c+\sum_{i=0}^{n-1}\left(a_{i}+a_{i+1}\right) \sum_{k=0}^{i} a_{i-k} \\ +& =n c+\sum_{i=0}^{n-1}\left(S_{i+1}-S_{i-1}\right) S_{i}=n c+S_{n}^{2} +\end{aligned} +$$ + +i.e., $S_{n}^{2}-S_{n}+n c=0$. Since $S_{n}$ is real, the discriminant of the quadratic equation must be positive, and hence $c \leq \frac{1}{4 n}$. +17. A figure consisting of 9 lines is shown below. +![](https://cdn.mathpix.com/cropped/2024_11_18_8e985d6b9c83aa3e9d0eg-531.jpg?height=237&width=318&top_left_y=1210&top_left_x=627) + +Now we show that 8 lines are not sufficient. Assume the opposite. By the pigeonhole principle, there is a vertex, say $A$, that is joined to at most 2 other vertices. Let $B, C, D, E$ denote the vertices to which $A$ is not joined, and $F, G$ the other two vertices. Then any two vertices of $B, C, D, E$ must be mutually joined for an edge to exist within the triangle these two points form with A. This accounts for 6 segments. Since only two segments remain, among $A, F$, and $G$ at least two are not joined. Taking these two and one of $B, C, D, E$ that is not joined to any of them (it obviously exists), we get a triple of points, no two of which are joined; a contradiction. + +Second solution. Since (a) is equivalent to the fact that no three points make a "blank triangle," by Turan's theorem the number of "blank edges" cannot exceed $\left[7^{2} / 4\right]=12$, leaving at least $7 \cdot 6 / 2-12=9$ segments. For general $n$, the answer is $[(n-1) / 2]^{2}$. +18. Consider the triangle $M A_{i} M_{i}$. Obviously, the point $M_{i}$ is the image of $A_{i}$ under the composition $C$ of rotation $R_{M}^{\alpha / 2-90^{\circ}}$ and homothety $H_{M}^{2 \sin (\alpha / 2)}$. Therefore, the polygon $M_{1} M_{2} \ldots M_{n}$ is obtained as the image of $A_{1} A_{2} \ldots A_{n}$ under the rotational homothety $C$ with coefficient $2 \sin (\alpha / 2)$. Therefore $S_{M_{1} M_{2} \ldots M_{n}}=4 \sin ^{2}(\alpha / 2) \cdot S$. +19. Let us color the board in a chessboard fashion. Denote by $S_{b}$ and $S_{w}$ respectively the sum of numbers in the black and in the white squares. It is clear that every allowed move leaves the difference $S_{b}-S_{w}$ unchanged. Therefore a necessary condition for annulling all the numbers is $S_{b}=S_{w}$. We now show it is sufficient. Assuming $S_{b}=S_{w}$ let us observe a triple of (different) cells $a, b, c$ with respective values $x_{a}, x_{b}, x_{c}$ where $a$ and $c$ are both adjacent to $b$. We first prove that we can reduce $x_{a}$ to be 0 if $x_{a}>0$. If $x_{a} \leq x_{b}$, we subtract $x_{a}$ from both $a$ and $b$. If $x_{a}>x_{b}$, we add $x_{a}-x_{b}$ to $b$ and $c$ and proceed as in the previous case. Applying the reduction in sequence, along the entire board, we reduce all cells except two neighboring cells to be 0 . Since $S_{b}=S_{w}$ is invariant, the two cells must have equal values and we can thus reduce them both to 0 . +20. Suppose $k \geq 1 / 2+\sqrt{2 n}$. Consider a point $P$ in $S$. There are at least $k$ points in $S$ having all the same distance to $P$, so there are at least $\binom{k}{2}$ pairs of points $A, B$ with $A P=B P$. Since this is true for every point $P \in S$, there are at least $n\binom{k}{2}$ triples of points $(A, B, P)$ for which $A P=B P$ holds. However, + +$$ +\begin{aligned} +n\binom{k}{2} & =n \frac{k(k-1)}{2} \geq \frac{n}{2}\left(\sqrt{2 n}+\frac{1}{2}\right)\left(\sqrt{2 n}-\frac{1}{2}\right) \\ +& =\frac{n}{2}\left(2 n-\frac{1}{4}\right)>n(n-1)=2\binom{n}{2} +\end{aligned} +$$ + +Since $\binom{n}{2}$ is the number of all possible pairs $(A, B)$ with $A, B \in S$, there must exist a pair of points $A, B$ with more than two points $P_{i}$ such that $A P_{i}=B P_{i}$. These points $P_{i}$ are collinear (they lie on the perpendicular bisector of $A B$ ), contradicting condition (1). +21. In order to obtain a triangle as the intersection we must have three points $P, Q, R$ on three sides of the tetrahedron passing through one vertex, say $T$. It is clear that we may suppose w.l.o.g. that $P$ is a vertex, and $Q$ and $R$ lie on the edges $T P_{1}$ and $T P_{2}\left(P_{1}, P_{2}\right.$ are vertices) or on their extensions respectively. Suppose that $\overrightarrow{T Q}=\lambda \overrightarrow{T P_{1}}$ and $\overrightarrow{T R}=\mu \overrightarrow{T P_{2}}$, where $\lambda, \mu>0$. Then + +$$ +\cos \angle Q P R=\frac{\overrightarrow{P Q} \cdot \overrightarrow{P R}}{\overline{P Q} \cdot \overline{P R}}=\frac{(\lambda-1)(\mu-1)+1}{2 \sqrt{\lambda^{2}-\lambda+1} \sqrt{\mu^{2}-\mu+1}} +$$ + +In order to obtain an obtuse angle (with $\cos <0$ ) we must choose $\mu<1$ and $\lambda>\frac{2-\mu}{1-\mu}>1$. Since $\sqrt{\lambda^{2}-\lambda+1}>\lambda-1$ and $\sqrt{\mu^{2}-\mu+1}>1-\mu$, we get that for $(\lambda-1)(\mu-1)+1<0$, + +$$ +\cos \angle Q P R>\frac{1-(1-\mu)(\lambda-1)}{2(1-\mu)(\lambda-1)}>-\frac{1}{2} ; \quad \text { hence } \angle Q P R<120^{\circ} +$$ + +Remark. After obtaining the formula for $\cos \angle Q P R$, the official solution was as follows: For fixed $\mu_{0}<1$ and $\lambda>1, \cos \angle Q P R$ is a decreasing function of $\lambda$ : indeed, + +$$ +\frac{\partial \cos \angle Q P R}{\partial \lambda}=\frac{\mu-(3-\mu) \lambda}{4\left(\lambda^{2}-\lambda+1\right)^{3 / 2}\left(\mu^{2}-\mu+1\right)^{1 / 2}}<0 +$$ + +Similarly, for a fixed, sufficiently large $\lambda_{0}, \cos \angle Q P R$ is decreasing for $\mu$ decreasing to 0 . Since $\lim _{\lambda \rightarrow 0, \mu \rightarrow 0+} \cos \angle Q P R=-1 / 2$, we conclude that $\angle Q P R<120^{\circ}$. +22. The statement remains valid if 17 is replaced by any divisor $k$ of $1989=3^{2}$. $13 \cdot 17,12$, is the twin of $x_{1}$, then + +$$ +f\left(x_{1}, x_{2}, \ldots, x_{2 n}\right)=\left(x_{2}, \ldots, x_{k-1}, x_{1}, x_{k}, \ldots, x_{2 n}\right) +$$ + +The mapping $f$ is injective, but not surjective. Thus $F_{0}(n)0$ (because $a, b<0$ is impossible, and $a, b \neq 0$ from the condition of the problem). Let $\left(x_{0}, y_{0}, z_{0}, w_{0}\right) \neq$ $(0,0,0,0)$ be a solution of $x^{2}-a y^{2}-b z^{2}+a b w^{2}$. Then + +$$ +x_{0}^{2}-a y_{0}^{2}=b\left(z_{0}^{2}-a w_{0}^{2}\right) +$$ + +Multiplying both sides by $\left(z_{0}^{2}-a w_{0}^{2}\right)$, we get + +$$ +\begin{gathered} +\left(x_{0}^{2}-a y_{0}^{2}\right)\left(z_{0}^{2}-a w_{0}^{2}\right)-b\left(z_{0}^{2}-a w_{0}^{2}\right)^{2}=0 \\ +\Leftrightarrow\left(x_{0} z_{0}-a y_{0} w_{0}\right)^{2}-a\left(y_{0} z_{0}-x_{0} w_{0}\right)^{2}-b\left(z_{0}^{2}-a w_{0}^{2}\right)^{2}=0 . +\end{gathered} +$$ + +Hence, for $x_{1}=x_{0} z_{0}-a y_{0} w_{0}, \quad y_{1}=y_{0} z_{0}-x_{0} w_{0}, \quad z_{1}=z_{0}^{2}-a w_{0}^{2}$, we have + +$$ +x_{1}^{2}-a y_{1}^{2}-b z_{1}^{2}=0 . +$$ + +If $\left(x_{1}, y_{1}, z_{1}\right)$ is the trivial solution, then $z_{1}=0$ implies $z_{0}=w_{0}=0$ and similarly $x_{0}=y_{0}=0$ because $a$ is not a perfect square. This contradicts the initial assumption. +26. By the Cauchy-Schwarz inequality, + +$$ +\left(\sum_{i=1}^{n} x_{i}\right)^{2} \leq n \sum_{i=1}^{n} x_{i}^{2} +$$ + +Since $\sum_{i=1}^{n} x_{i}=a-x_{0}$ and $\sum_{i=1}^{n} x_{i}^{2}=b-x_{0}^{2}$, we have $\left(a-x_{0}\right)^{2} \leq$ $n\left(b-x_{0}^{2}\right)$, i.e., + +$$ +(n+1) x_{0}^{2}-2 a x_{0}+\left(a^{2}-n b\right) \leq 0 . +$$ + +The discriminant of this quadratic is $D=4 n(n+1)\left[b-a^{2} /(n+1)\right]$, so we conclude that +(i) if $a^{2}>(n+1) b$, then such an $x_{0}$ does not exist; +(ii) if $a^{2}=(n+1) b$, then $x_{0}=a / n+1 ; \quad$ and +(iii) if $a^{2}<(n+1) b$, then $\frac{a-\sqrt{D} / 2}{n+1} \leq x_{0} \leq \frac{a+\sqrt{D} / 2}{n+1}$. + +It is easy to see that these conditions for $x_{0}$ are also sufficient. +27. Let $n$ be the required exponent, and suppose $n=2^{k} q$, where $q$ is an odd integer. Then we have + +$$ +m^{n}-1=\left(m^{2^{k}}-1\right)\left[\left(m^{2^{k}(q-1)}+\cdots+m^{2^{k}}+1\right]=\left(m^{2^{k}}-1\right) A\right. +$$ + +where $A$ is odd. Therefore $m^{n}-1$ and $m^{2^{k}}-1$ are divisible by the same power of 2 , and so $n=2^{k}$. +Next, we observe that + +$$ +\begin{aligned} +m^{2^{k}}-1 & =\left(m^{2^{k-1}}-1\right)\left(m^{2^{k-1}}+1\right)=\ldots \\ +& =\left(m^{2}-1\right)\left(m^{2}+1\right)\left(m^{4}+1\right) \cdots\left(m^{2^{k-1}}+1\right) +\end{aligned} +$$ + +Let $s$ be the maximal positive integer for which $m \equiv \pm 1\left(\bmod 2^{s}\right)$. Then $m^{2}-1$ is divisible by $2^{s+1}$ and not divisible by $2^{s+2}$. All the numbers $m^{2}+1, m^{4}+1, \ldots, m^{2^{k-1}}+1$ are divisible by 2 and not by 4 . Hence $m^{2^{k}}-1$ is divisible by $2^{s+k}$ and not by $2^{s+k+1}$. +It follows from the above consideration that the smallest exponent $n$ equals $2^{1989-s}$ if $s \leq 1989$, and $n=1$ if $s>1989$. +28. Assume w.l.o.g. that the rays $O A_{1}, O A_{2}, O A_{3}, O A_{4}$ are arranged clockwise. Setting $O A_{1}=a, O A_{2}=b, O A_{3}=c, O A_{4}=d$, and $\angle A_{1} O A_{2}=x$, $\angle A_{2} O A_{3}=y, \angle A_{3} O A_{4}=z$, we have + +$$ +\begin{aligned} +& S_{1}=\sigma\left(O A_{1} A_{2}\right)=\frac{1}{2} a b|\sin x|, S_{2}=\sigma\left(O A_{1} A_{3}\right)=\frac{1}{2} a c|\sin (x+y)|, \\ +& S_{3}=\sigma\left(O A_{1} A_{4}\right)=\frac{1}{2} a d|\sin (x+y+z)|, S_{4}=\sigma\left(O A_{2} A_{3}\right)=\frac{1}{2} b c|\sin y|, \\ +& S_{5}=\sigma\left(O A_{2} A_{4}\right)=\frac{1}{2} b d|\sin (y+z)|, S_{6}=\sigma\left(O A_{3} A_{4}\right)=\frac{1}{2} c d|\sin z| . +\end{aligned} +$$ + +Since $\sin (x+y+z) \sin y+\sin x \sin z=\sin (x+y) \sin (y+z)$, it follows that there exists a choice of $k, l \in\{0,1\}$ such that + +$$ +S_{1} S_{6}+(-1)^{k} S_{2} S_{5}+(-1)^{l} S_{3} S_{4}=0 +$$ + +For example (w.l.o.g.), if $S_{3} S_{4}=S_{1} S_{6}+S_{2} S_{5}$, we have + +$$ +\left(\max _{1 \leq i \leq 6} S_{i}\right)^{2} \geq S_{3} S_{4}=S_{1} S_{6}+S_{2} S_{5} \geq 1+1=2 +$$ + +i.e., $\max _{1 \leq i \leq 6} S_{i} \geq \sqrt{2}$ as claimed. +29. Let $P_{i}$, sitting at the place $A$, and $P_{j}$ sitting at $B$, be two birds that can see each other. Let $k$ and $l$ respectively be the number of birds visible from $B$ but not from $A$, and the number of those visible from $A$ but not from +$B$. Assume that $k \geq l$. Then if all birds from $B$ fly to $A$, each of them will see $l$ new birds, but won't see $k$ birds anymore. Hence the total number of mutually visible pairs does not increase, while the number of distinct positions occupied by at least one bird decreases by one. Repeating this operation as many times as possible one can arrive at a situation in which two birds see each other if and only if they are in the same position. The number of such distinct positions is at most 35 , while the total number of mutually visible pairs is not greater than at the beginning. Thus the problem is equivalent to the following one: +(1) If $x_{i} \geq 0$ are integers with $\sum_{j=1}^{35} x_{j}=155$, find the least possible value of $\sum_{j=1}^{35}\left(x_{j}^{2}-x_{j}\right) / 2$. +If $x_{j} \geq x_{i}+2$ for some $i, j$, then the sum of $\left(x_{j}^{2}-x_{j}\right) / 2$ decreases (for $x_{j}-x_{i}-2$ ) if $x_{i}, x_{j}$ are replaced with $x_{i}+1, x_{j}-1$. Consequently, our sum attains its minimum when the $x_{i}$ 's differ from each other by at most 1 . In this case, all the $x_{i}$ 's are equal to either $[155 / 35]=4$ or $[155 / 35]+1=5$, where $155=20 \cdot 4+15 \cdot 5$. It follows that the (minimum possible) number of mutually visible pairs is $20 \cdot \frac{4 \cdot 3}{2}+15 \cdot \frac{5 \cdot 4}{2}=270$. +Second solution for (1). Considering the graph consisting of birds as vertices and pairs of mutually nonvisible birds as edges, we see that there is no complete 36 -subgraph. Turan's theorem gives the answer immediately. (See problem (SL89-17).) +30. For all $n$ such $N$ exists. For a given $n$ choose $N=(n+1)!^{2}+1$. Then $1+j$ is a proper factor of $N+j$ for $1 \leq j \leq n$. So if $N+j=p^{m}$ is a power of a prime $p$, then $1+j=p^{r}$ for some integer $r, 1 \leq r90^{\circ}$. +6. Let $W$ denote the set of all $n_{0}$ for which player $A$ has a winning strategy, $L$ the set of all $n_{0}$ for which player $B$ has a winning strategy, and $T$ the set of all $n_{0}$ for which a tie is ensured. + +Lemma. Assume $\{m, m+1, \ldots 1990\} \subseteq W$ and that there exists $s \leq 1990$ such that $s / p^{r} \geq m$, where $p^{r}$ is the largest degree of a prime that divides $s$. Then all integers $x$ such that $\sqrt{s} \leq x1990$, it follows that for $n_{0} \in\{45, \ldots, 1990\}$ player $A$ can choose 1990 in the first move. Hence $\{45, \ldots, 1990\} \subseteq W$. Using $m=45$ and selecting $s=420=2^{2} \cdot 3 \cdot 5 \cdot 7$ we apply the lemma to get that all integers $x$ such that $\sqrt{420}<21 \leq x \leq 1990$ are in $W$. Again, using $m=21$ and selecting $s=168=2^{3} \cdot 3 \cdot 7$ we apply the lemma to get that all integers $x$ such that $\sqrt{168}<13 \leq x \leq 1990$ are in $W$. Selecting $s=105$ we obtain the new value for $m$ at $m=11$. Selecting $s=60$ we obtain $m=8$. Thus $\{8, \ldots, 1990\} \subseteq W$. +For $n_{0}>1990$ there exists $r \in N$ such that $2^{r} \cdot 3^{2}1$, the final solution is $L=\{2,3,4,5\}, T=\{6,7\}$, and $W=\{x \in \mathbb{N} \mid x \geq 8\}$. +7. Let $f(n)=g(n) 2^{n^{2}}$ for all $n$. The recursion then transforms into $g(n+$ 2) $-2 g(n+1)+g(n)=n \cdot 16^{-n-1}$ for $n \in \mathbb{N}_{0}$. By summing this equation from 0 to $n-1$, we get + +$$ +g(n+1)-g(n)=\frac{1}{15^{2}} \cdot\left(1-(15 n+1) 16^{-n}\right) +$$ + +By summing up again from 0 to $n-1$ we get $g(n)=\frac{1}{15^{3}} \cdot(15 n-32+$ $\left.(15 n+2) 16^{-n+1}\right)$. Hence + +$$ +f(n)=\frac{1}{15^{3}} \cdot\left(15 n+2+(15 n-32) 16^{n-1}\right) \cdot 2^{(n-2)^{2}} +$$ + +Now let us look at the values of $f(n)$ modulo 13: + +$$ +f(n) \equiv 15 n+2+(15 n-32) 16^{n-1} \equiv 2 n+2+(2 n-6) 3^{n-1} +$$ + +We have $3^{3} \equiv 1(\bmod 13)$. Plugging in $n \equiv 1(\bmod 13)$ and $n \equiv 1(\bmod$ 3 ) for $n=1990$ gives us $f(1990) \equiv 0(\bmod 13)$. We similarly calculate $f(1989) \equiv 0$ and $f(1991) \equiv 0(\bmod 13)$. +8. Since $2^{1990}<8^{700}<10^{700}$, we have $f_{1}\left(2^{1990}\right)<(9 \cdot 700)^{2}<4 \cdot 10^{7}$. We then have $f_{2}\left(2^{1990}\right)<(3+9 \cdot 7)^{2}<4900$ and finally $f_{3}\left(2^{1990}\right)<(3+9 \cdot 3)^{2}=30^{2}$. It is easily shown that $f_{k}(n) \equiv f_{k-1}(n)^{2}(\bmod 9)$. Since $2^{6} \equiv 1(\bmod 9)$, we have $2^{1990} \equiv 2^{4} \equiv 7$ (all congruences in this problem will be $\bmod 9$ ). It follows that $f_{1}\left(2^{1990}\right) \equiv 7^{2} \equiv 4$ and $f_{2}\left(2^{1990}\right) \equiv 4^{2} \equiv 7$. Indeed, it follows that $f_{2 k}\left(2^{1990}\right) \equiv 7$ and $f_{2 k+1}\left(2^{1990}\right) \equiv 4$ for all integer $k>0$. Thus $f_{3}\left(2^{1990}\right)=r^{2}$ where $r<30$ is an integer and $r \equiv f_{2}\left(2^{1990}\right) \equiv 7$. It follows that $r \in\{7,16,25\}$ and hence $f_{3}\left(2^{1990}\right) \in\{49,256,625\}$. It follows that $f_{4}\left(2^{1990}\right)=169, f_{5}\left(2^{1990}\right)=256$, and inductively $f_{2 k}\left(2^{1990}\right)=169$ and $f_{2 k+1}\left(2^{1990}\right)=256$ for all integer $k>1$. Hence $f_{1991}\left(2^{1990}\right)=256$. +9. Let $a, b, c$ be the lengths of the sides of $\triangle A B C, s=\frac{a+b+c}{2}, r$ the inradius of the triangle, and $c_{1}$ and $b_{1}$ the lengths of $A B_{2}$ and $A C_{2}$ respectively. As usual we will denote by $S(X Y Z)$ the area of $\triangle X Y Z$. We have + +$$ +\begin{gathered} +S\left(A C_{1} B_{2}\right)=\frac{A C_{1} \cdot A B_{2}}{A C \cdot A B} S(A B C)=\frac{c_{1} r s}{2 b} \\ +S\left(A K B_{2}\right)=\frac{c_{1} r}{2}, \quad S\left(A C_{1} K\right)=\frac{c r}{4} +\end{gathered} +$$ + +From $S\left(A C_{1} B_{2}\right)=S\left(A K B_{2}\right)+S\left(A C_{1} K\right)$ we get $\frac{c_{1} r s}{2 b}=\frac{c_{1} r}{2}+\frac{c r}{4}$; therefore $(a-b+c) c_{1}=b c$. By looking at the area of $\triangle A B_{1} C_{2}$ we similarly obtain $(a+b-c) b_{1}=b c$. From these two equations and from $S(A B C)=S\left(A B_{2} C_{2}\right)$, from which we have $b_{1} c_{1}=b c$, we obtain + +$$ +a^{2}-(b-c)^{2}=b c \Rightarrow \frac{b^{2}+c^{2}-a^{2}}{2 b c}=\cos (\angle B A C)=\frac{1}{2} \Rightarrow \angle B A C=60^{\circ} +$$ + +10. Let $r$ be the radius of the base and $h$ the height of the cone. We may assume w.l.o.g. that $r=1$. Let $A$ be the top of the cone, $B C$ the diameter of the circumference of the base such that the plane touches the circumference at $B, O$ the center of the base, and $H$ the midpoint of $O A$ (also belonging to the plane). Let $B H$ cut the sheet of the cone at $D$. By applying Menelaus's theorem to $\triangle A O C$ and $\triangle B H O$, we conclude that $\frac{A D}{D C}=\frac{C B}{B O} \cdot \frac{O H}{H A}=\frac{1}{2}$ and $\frac{H D}{D B}=\frac{H A}{A O} \cdot \frac{O C}{C B}=\frac{1}{4}$. +The plane cuts the cone in an ellipse whose major axis is $B D$. Let $E$ be the center of this ellipse and $F G$ its minor axis. We have $\frac{B E}{E D}=\frac{1}{2}$. Let $E^{\prime}, F^{\prime}, G^{\prime}$ be radial projections of $E, F, G$ from $A$ onto the base of the cone. Then $E$ sits on $B C$. Let $h(X)$ denote the height of a point $X$ with respect to the base of the cone. We have $h(E)=h(D) / 2=h / 3$. + +Hence $E F=2 E^{\prime} F^{\prime} / 3$. Applying Menelaus's theorem to $\triangle B H O$ we get $\frac{O E^{\prime}}{E^{\prime} B}=\frac{B E}{E H} \cdot \frac{H A}{A O}=1$. Hence $E F=\frac{2}{3} \frac{\sqrt{3}}{2}=\frac{1}{\sqrt{3}}$. +Let $d$ denote the distance from $A$ to the plane. Let $V_{1}$ and $V$ denote the volume of the cone above the plane (on the same side of the plane as $A$ ) and the total volume of the cone. We have + +$$ +\begin{aligned} +\frac{V_{1}}{V} & =\frac{B E \cdot E F \cdot d}{h}=\frac{(2 B H / 3)(1 / \sqrt{3})\left(2 S_{A H B} / B H\right)}{h} \\ +& =\frac{(2 / 3)(1 / \sqrt{3})(h / 2)}{h}=\frac{1}{3 \sqrt{3}} . +\end{aligned} +$$ + +Since this ratio is smaller than $1 / 2$, we have indeed selected the correct volume for our ratio. +11. Assume $\mathcal{B}(A, E, M, B)$. Since $A, B, C, D$ lie on a circle, we have $\angle G C E=$ $\angle M B D$ and $\angle M A D=\angle F C E$. Since $F D$ is tangent to the circle around $\triangle E M D$ at $E$, we have $\angle M D E=\angle F E B=\angle A E G$. Consequently, $\angle C E F=180^{\circ}-\angle C E A-\angle F E B=180^{\circ}-\angle M E D-\angle M D E=\angle E M D$ and $\angle C E G=180^{\circ}-\angle C E F=180^{\circ}-\angle E M D=\angle D M B$. It follows that $\triangle C E F \sim \triangle A M D$ and $\triangle C E G \sim$ $\triangle B M D$. From the first similarity we obtain $C E \cdot M D=A M \cdot E F$, and from the second we obtain $C E$. $M D=B M \cdot E G$. Hence + +$$ +\begin{gathered} +A M \cdot E F=B M \cdot E G \Longrightarrow \\ +\frac{G E}{E F}=\frac{A M}{B M}=\frac{\lambda}{1-\lambda} . +\end{gathered} +$$ + +![](https://cdn.mathpix.com/cropped/2024_11_18_8e985d6b9c83aa3e9d0eg-543.jpg?height=371&width=353&top_left_y=1023&top_left_x=894) + +If $\mathcal{B}(A, M, E, B)$, interchanging the roles of $A$ and $B$ we similarly obtain $\frac{G E}{E F}=\frac{\lambda}{1-\lambda}$. +12. Let $d(X, l)$ denote the distance of a point $X$ from a line $l$. Using the elementary facts that $A F: F C=c: a$ and $B D: D C=c: b$, we obtain $d(F, L)=\frac{a}{a+c} h_{c}$ and $d(D, L)=\frac{b}{b+c} h_{c}$, where $h_{a}$ is the altitude of $\triangle A B C$ from $A$. We also have $\angle F G C=\beta / 2, \angle D E C=\alpha / 2$. It follows that + +$$ +D E=\frac{d(D, L)}{\sin (\alpha / 2)} \quad \text { and } \quad F G=\frac{d(F, L)}{\sin (\beta / 2)} +$$ + +Now suppose that $a>b$. Since the function $f(x)=\frac{x}{x+c}$ is strictly increasing, we deduce $d(F, L)>d(D, L)$. Furthermore, $\sin (\alpha / 2)>\sin (\beta / 2)$, so we get from (1) that $F G>D E$. +Similarly, $a1$, the scientist can climb +only onto the rungs divisible by $k$ and we can just observe these rungs to obtain the situation equivalent to $a^{\prime}=a / k, b^{\prime}=b / k$, and $n^{\prime}=a^{\prime}+b^{\prime}-1$. Thus let us assume that $(a, b)=1$ and show that $n=a+b-1$. +We obviously have $n>a$. Consider $n=a+b-k, k \geq 1$, and let us assume without loss of generality that $a>b$ (otherwise, we can reverse the problem starting from the top rung in our round trip). Then we can uniquely define the numbers $r_{i}, 0 \leq r_{i}b-1$ we can move only $b$ rungs downward. If we end up at $b-k

0$ that $f^{k}(0)=0$. Since $f(m)=m+a$ or $f(m)=m-b$, it follows that $k$ can be written as $k=r+s$, where $r a-s b=0$. Since $a$ and $b$ are relatively prime, it follows that $k \geq a+b$. +Let us now prove that $f^{a+b}(0)=0$. In this case $a+b=r+s$ and hence $f^{a+b}(0)=(a+b-s) a-s b=(a+b)(a-s)$. Since $a+b \mid f^{a+b}(0)$ and $f^{a+b}(0) \in S$, it follows that $f^{a+b}(0)=0$. Thus for $(a, b)=1$ it follows that $k=a+b$. For other $a$ and $b$ we have $k=\frac{a+b}{(a, b)}$. +19. Let $d_{1}, d_{2}, d_{3}, d_{4}$ be the distances of the point $P$ to the tetrahedron. Let $d$ be the height of the regular tetrahedron. Let $x_{i}=d_{i} / d$. Clearly, $x_{1}+$ $x_{2}+x_{3}+x_{4}=1$, and given this condition, the parameters vary freely as we vary $P$ within the tetrahedron. The four tetrahedra have volumes $x_{1}^{3}, x_{2}^{3}, x_{3}^{3}$, and $x_{4}^{3}$, and the four parallelepipeds have volumes of $6 x_{2} x_{3} x_{4}$, $6 x_{1} x_{3} x_{4}, 6 x_{1} x_{2} x_{4}$, and $6 x_{1} x_{2} x_{3}$. Hence, using $x_{1}+x_{2}+x_{3}+x_{4}=1$ and setting $g(x)=x^{2}(1-x)$, we directly verify that + +$$ +\begin{aligned} +f(P) & =f\left(x_{1}, x_{2}, x_{3}, x_{4}\right)=1-\sum_{i=1}^{4} x_{i}^{3}-6 \sum_{1 \leq i0$ we have $g\left(x_{i}+x_{j}\right)+g(0) \geq$ $g\left(x_{i}\right)+g\left(x_{j}\right)$. Equality holds only when $x_{i}+x_{j}=2 / 3$. +Assuming without loss of generality $x_{1} \geq x_{2} \geq x_{3} \geq x_{4}$, we have $g\left(x_{1}\right)+$ $g\left(x_{2}\right)+g\left(x_{3}\right)+g\left(x_{4}\right)k$ and hence there exist two numbers $x, y \in S(n)$ such that $k \mid x-y$. +Let us show that $w=|x-y|$ is the desired number. By definition $k \mid w$. We also have + +$$ +w<1.2 \cdot 10^{n-1} \leq 1.2 \cdot\left(2^{3} \sqrt{2}\right)^{n-1} \leq 1.2 \cdot k^{3} \sqrt{k} \leq k^{4} +$$ + +Finally, since $x, y \in S(n)$, it follows that $w=|x-y|$ can be written using only the digits $\{0,1,8,9\}$. This completes the proof. +21. We must solve the congruence $\left(1+2^{p}+2^{n-p}\right) N \equiv 1\left(\bmod 2^{n}\right)$. Since $(1+$ $2^{p}+2^{n-p}$ ) and $2^{n}$ are coprime, there clearly exists a unique $N$ satisfying this equation and $0b$ we have in binary representation + +$$ +2^{a}-2^{b}=\underbrace{11 \ldots 11}_{a-b \text { times }} \underbrace{00 \ldots 00}_{b \text { times }} +$$ + +the binary representation of $N$ is calculated as follows: + +$$ +N=\left\{\begin{array}{cc} +\underbrace{11 \ldots 11}_{p \text { times }} \underbrace{11 \ldots 11}_{p \text { times }} \underbrace{00 \ldots 00}_{p \text { times }} \ldots \underbrace{11 \ldots 11}_{p \text { times }} \underbrace{00 \ldots 00}_{p-1 \text { times }} 1, & 2 \nmid \frac{n}{p} \\ +\underbrace{11 \ldots 11}_{p-1 \text { times }} \underbrace{00 \ldots 00}_{p+1} \underbrace{11 \ldots 11}_{p \text { times }} \underbrace{00 \ldots 00}_{p \text { times }} \ldots \underbrace{11 \ldots 11}_{p \text { times }} \underbrace{00 \ldots 00}_{p-1 \text { times }} 1, & 2 \left\lvert\, \frac{n}{p}\right. +\end{array}\right. +$$ + +22. We can assume without loss of generality that each connection is serviced by only one airline and the problem reduces to finding two disjoint monochromatic cycles of the same color and of odd length on a complete graph of 10 points colored by two colors. We use the following two standard lemmas: +Lemma 1. Given a complete graph on six points whose edges are colored with two colors there exists a monochromatic triangle. +$\operatorname{Proof}$. Let us denote the vertices by $c_{1}, c_{2}, c_{3}, c_{4}, c_{5}, c_{6}$. By the pigeonhole principle at least three vertices out of $c_{1}$, say $c_{2}, c_{3}, c_{4}$, are of the same color, let us call it red. Assuming that at least one of the edges connecting points $c_{2}, c_{3}, c_{4}$ is red, the connected points along with $c_{1}$ form a red triangle. Otherwise, edges connecting $c_{2}, c_{3}, c_{4}$ are all of the opposite color, let us call it blue, and hence in all cases we have a monochromatic triangle. +Lemma 2. Given a complete graph on five points whose edges are colored two colors there exists a monochromatic triangle or a monochromatic cycle of length five. +Proof. Let us denote the vertices by $c_{1}, c_{2}, c_{3}, c_{4}, c_{5}$. Assume that out of a point $c_{i}$ three vertices are of the same color. We can then proceed as in Lemma 1 to obtain a monochromatic triangle. Otherwise, each +point is connected to other points with exactly two red and two blue vertices. Hence, we obtain monochromatic cycles starting from a single point and moving along the edges of the same color. Since each cycle must be of length at least three (i.e., we cannot have more than one cycle of one color), it follows that for both red and blue we must have one cycle of length five of that color. +We now apply the lemmas. Let us denote the vertices by $c_{1}, c_{2}, \ldots, c_{10}$. We apply Lemma 1 to vertices $c_{1}, \ldots, c_{6}$ to obtain a monochromatic triangle. Out of the seven remaining vertices we select 6 and again apply Lemma 1 to obtain another monochromatic triangle. If they are of the same color, we are done. Otherwise, out of the nine edges connecting the two triangles of opposite color at least 5 are of the same color, we can assume blue w.l.o.g., and hence a vertex of a red triangle must contain at least two blue edges whose endpoints are connected with a blue edge. Hence there exist two triangles of different colors joined at a vertex. These take up five points. Applying Lemma 2 on the five remaining points, we obtain a monochromatic cycle of odd length that is of the same color as one of the two joined triangles and disjoint from both of them. +23. Let us assume $n>1$. Obviously $n$ is odd. Let $p \geq 3$ be the smallest prime divisor of $n$. In this case $(p-1, n)=1$. Since $2^{n}+1 \mid 2^{2 n}-1$, we have that $p \mid 2^{2 n}-1$. Thus it follows from Fermat's little theorem and elementary number theory that $p \mid\left(2^{2 n}-1,2^{p-1}-1\right)=2^{(2 n, p-1)}-1$. Since $(2 n, p-1) \leq 2$, it follows that $p \mid 3$ and hence $p=3$. +Let us assume now that $n$ is of the form $n=3^{k} d$, where $2,3 \nmid d$. We first prove that $k=1$. +Lemma. If $2^{m}-1$ is divisible by $3^{r}$, then $m$ is divisible by $3^{r-1}$. +Proof. This is the lemma from (SL97-14) with $p=3, a=2^{2}, k=m$, $\alpha=1$, and $\beta=r$. +Since $3^{2 k}$ divides $n^{2} \mid 2^{2 n}-1$, we can apply the lemma to $m=2 n$ and $r=2 k$ to conclude that $3^{2 k-1} \mid n=3^{k} d$. Hence $k=1$. +Finally, let us assume $d>1$ and let $q$ be the smallest prime factor of $d$. Obviously $q$ is odd, $q \geq 5$, and $(n, q-1) \in\{1,3\}$. We then have $q \mid 2^{2 n}-1$ and $q \mid 2^{q-1}-1$. Consequently, $q \mid 2^{(2 n, q-1)}-1=2^{2(n, q-1)}-1$, which divides $2^{6}-1=63=3^{2} \cdot 7$, so we must have $q=7$. However, in that case we obtain $7|n| 2^{n}+1$, which is a contradiction, since powers of two can only be congruent to 1,2 and 4 modulo 7 . It thus follows that $d=1$ and $n=3$. Hence $n>1 \Rightarrow n=3$. +It is easily verified that $n=1$ and $n=3$ are indeed solutions. Hence these are the only solutions. +24. Let us denote $A=b+c+d, B=a+c+d, C=a+b+d, D=a+b+c$. Since $a b+b c+c d+d a=1$ the numbers $A, B, C, D$ are all positive. By trivially applying the AM-GM inequality we have: + +$$ +a^{2}+b^{2}+c^{2}+d^{2} \geq a b+b c+c d+d a=1 . +$$ + +We will prove the inequality assuming only that $A, B, C, D$ are positive and $a^{2}+b^{2}+c^{2}+d^{2} \geq 1$. In this case we may assume without loss of generality that $a \geq b \geq c \geq d \geq 0$. Hence $a^{3} \geq b^{3} \geq c^{3} \geq d^{3} \geq 0$ and $\frac{1}{A} \geq \frac{1}{B} \geq \frac{1}{C} \geq \frac{1}{D}>0$. Using the Chebyshev and Cauchy inequalities we obtain: + +$$ +\begin{aligned} +& \frac{a^{3}}{A}+\frac{b^{3}}{B}+\frac{c^{3}}{C}+\frac{d^{3}}{D} \\ +& \geq \frac{1}{4}\left(a^{3}+b^{3}+c^{3}+d^{3}\right)\left(\frac{1}{A}+\frac{1}{B}+\frac{1}{C}+\frac{1}{D}\right) \\ +& \geq \frac{1}{16}\left(a^{2}+b^{2}+c^{2}+d^{2}\right)(a+b+c+d)\left(\frac{1}{A}+\frac{1}{B}+\frac{1}{C}+\frac{1}{D}\right) \\ +& =\frac{1}{48}\left(a^{2}+b^{2}+c^{2}+d^{2}\right)(A+B+C+D)\left(\frac{1}{A}+\frac{1}{B}+\frac{1}{C}+\frac{1}{D}\right) \geq \frac{1}{3} +\end{aligned} +$$ + +This completes the proof. +25. Plugging in $x=1$ we get $f(f(y))=f(1) / y$ and hence $f\left(y_{1}\right)=f\left(y_{2}\right)$ implies $y_{1}=y_{2}$ i.e. that the function is bijective. Plugging in $y=1$ gives us $f(x f(1))=f(x) \Rightarrow x f(1)=x \Rightarrow f(1)=1$. Hence $f(f(y))=1 / y$. Plugging in $y=f(z)$ implies $1 / f(z)=f(1 / z)$. Finally setting $y=f(1 / t)$ into the original equation gives us $f(x t)=f(x) / f(1 / t)=f(x) f(t)$. Conversely, any functional equation on $\mathbb{Q}^{+}$satisfying (i) $f(x t)=f(x) f(t)$ and (ii) $f(f(x))=\frac{1}{x}$ for all $x, t \in \mathbb{Q}^{+}$also satisfies the original functional equation: $f(x f(y))=f(x) f(f(y))=\frac{f(x)}{y}$. Hence it suffices to find a function satisfying (i) and (ii). +We note that all elements $q \in \mathbb{Q}^{+}$are of the form $q=\prod_{i=1}^{n} p_{i}^{a_{i}}$ where $p_{i}$ are prime and $a_{i} \in \mathbb{Z}$. The criterion ( $a$ ) implies $f(q)=f\left(\prod_{i=1}^{n} p_{i}^{a_{i}}\right)=$ $\prod_{i=1}^{n} f\left(p_{i}\right)^{a_{i}}$. Thus it is sufficient to define the function on all primes. For the function to satisfy $(b)$ it is necessary and sufficient for it to satisfy $f(f(p))=\frac{1}{p}$ for all primes $p$. Let $q_{i}$ denote the $i$-th smallest prime. We define our function $f$ as follows: + +$$ +f\left(q_{2 k-1}\right)=q_{2 k}, \quad f\left(q_{2 k}\right)=\frac{1}{q_{2 k-1}}, \quad k \in \mathbb{N} +$$ + +Such a function clearly satisfies $(b)$ and along with the additional condition $f(x t)=f(x) f(t)$ it is well defined for all elements of $\mathbb{Q}^{+}$and it satisfies the original functional equation. +26. We note that $|P(x) / x| \rightarrow \infty$. Hence, there exists an integer number $M$ such that $M>\left|q_{1}\right|$ and $|P(x)| \leq|x| \Rightarrow|x|\left|q_{i}\right| \geq M$ and this ultimately contradicts $\left|q_{1}\right|0\right)$ both belonging to the set $\{j T+1, j T+2, \ldots, j T+T\}$ such that $q_{m_{j}}=q_{m_{j}+k_{j}}$. Since $k_{j}\sin ^{3} 30^{\circ}=\frac{1}{8} . +\end{aligned} +$$ + +On the other hand, since $\angle M A C+\angle M B A+\angle M C B<180^{\circ}-3 \cdot 30^{\circ}=90^{\circ}$, Jensen's inequality applied on the concave function $\ln \sin x(x \in[0, \pi])$ gives us $\sin \angle M A C \sin \angle M B A \sin \angle M C B<\sin ^{3} 30^{\circ}$, contradicting (*). + +Second solution. Denote the intersections of $P A, P B, P C$ with $B C, C A$, $A B$ by $A_{1}, B_{1}, C_{1}$, respectively. Suppose that each of the angles $\angle P A B$, $\angle P B C, \angle P C A$ is greater than $30^{\circ}$ and denote $P A=2 x, P B=2 y, P C=$ $2 z$. Then $P C_{1}>x, P A_{1}>y, P B_{1}>z$. On the other hand, we know that + +$$ +\frac{P C_{1}}{P C+P C_{1}}+\frac{P A_{1}}{P A+P A_{1}}+\frac{P B_{1}}{P B+P B_{1}}=\frac{S_{A B P}}{S_{A B C}}+\frac{S_{P B C}}{S_{A B C}}+\frac{S_{A P C}}{S_{A B C}}=1 . +$$ + +Since the function $\frac{t}{p+t}$ is increasing, we obtain $\frac{x}{2 z+x}+\frac{y}{2 x+y}+\frac{z}{2 y+z}<1$. But on the contrary, Cauchy-Schwartz inequality (or alternatively Jensen's inequality) yields + +$$ +\frac{x}{2 z+x}+\frac{y}{2 x+y}+\frac{z}{2 y+z} \geq \frac{(x+y+z)^{2}}{x(2 z+x)+y(2 x+y)+z(2 y+z)}=1 . +$$ + +5. Let $P_{1}$ be the point on the side $B C$ such that $\angle B F P_{1}=\beta / 2$. Then $\angle B P_{1} F=180^{\circ}-3 \beta / 2$, and the sine law gives us $\frac{B F}{B P_{1}}=\frac{\sin (3 \beta / 2)}{\sin (\beta / 2)}=$ $3-4 \sin ^{2}(\beta / 2)=1+2 \cos \beta$. +Now we calculate $\frac{B F}{B P}$. We have $\angle B I F=120^{\circ}-\beta / 2, \angle B F I=60^{\circ}$ and $\angle B I C=120^{\circ}, \angle B C I=\gamma / 2=60^{\circ}-\beta / 2$. By the sine law, + +$$ +B F=B I \frac{\sin \left(120^{\circ}-\beta / 2\right)}{\sin 60^{\circ}}, \quad B P=\frac{1}{3} B C=B I \frac{\sin 120^{\circ}}{3 \sin \left(60^{\circ}-\beta / 2\right)} . +$$ + +It follows that $\frac{B F}{B P}=\frac{3 \sin \left(60^{\circ}-\beta / 2\right) \sin \left(60^{\circ}+\beta / 2\right)}{\sin ^{2} 60^{\circ}}=4 \sin \left(60^{\circ}-\beta / 2\right) \sin \left(60^{\circ}+\right.$ $\beta / 2)=2\left(\cos \beta-\cos 120^{\circ}\right)=2 \cos \beta+1=\frac{B F}{B P_{1}}$. Therefore $P \equiv P_{1}$. +6. Let $a, b, c$ be sides of the triangle. Let $A_{1}$ be the intersection of line $A I$ with $B C$. By the known fact, $B A_{1}: A_{1} C=c: b$ and $A I: I A_{1}=A B: B A_{1}$, hence $B A_{1}=\frac{a c}{b+c}$ and $\frac{A I}{I A_{1}}=\frac{A B}{B A_{1}}=\frac{b+c}{a}$. Consequently $\frac{A I}{l_{A}}=\frac{b+c}{a+b+c}$. Put $a=n+p, b=p+m, c=m+n$ : it is obvious that $m, n, p$ are positive. Our inequality becomes + +$$ +2<\frac{(2 m+n+p)(m+2 n+p)(m+n+2 p)}{(m+n+p)^{3}} \leq \frac{64}{27} +$$ + +The right side inequality immediately follows from the inequality between arithmetic and geometric means applied on $2 m+n+p, m+2 n+p$ and $m+n+2 p$. For the left side inequality, denote by $T=m+n+p$. Then we can write $(2 m+n+p)(m+2 n+p)(m+n+2 p)=(T+m)(T+n)(T+p)$ and +$(T+m)(T+n)(T+p)=T^{3}+(m+n+p) T^{2}+(m n+n p+p n) T+m n p>2 T^{3}$. +Remark. The inequalities cannot be improved. In fact, $\frac{A I \cdot B I \cdot C I}{l_{A} l_{B} l_{C}}$ is equal to $8 / 27$ for $a=b=c$, while it can be arbitrarily close to $1 / 4$ if $a=b$ and $c$ is sufficiently small. +7. The given equations imply $A B=C D, A C=B D, A D=B C$. Let $L_{1}$, $M_{1}, N_{1}$ be the midpoints of $A D, B D, C D$ respectively. Then the above +equalities yield + +$$ +\begin{gathered} +L_{1} M_{1}=A B / 2=L M \\ +L_{1} M_{1}\|A B\| L M \\ +L_{1} M=C D / 2=L M_{1} \\ +L_{1} M\|C D\| L M_{1} +\end{gathered} +$$ + +Thus $L, M, L_{1}, M_{1}$ are coplanar and $L M L_{1} M_{1}$ is a rhombus as well as +![](https://cdn.mathpix.com/cropped/2024_11_18_8e985d6b9c83aa3e9d0eg-556.jpg?height=317&width=410&top_left_y=248&top_left_x=888) +$M N M_{1} N_{1}$ and $L N L_{1} N_{1}$. Then the segments $L L_{1}, M M_{1}, N N_{1}$ have the common midpoint $Q$ and $Q L \perp Q M$, $Q L \perp Q N, Q M \perp Q N$. We also infer that the line $N N_{1}$ is perpendicular to the plane $L M L_{1} M_{1}$ and hence to the line $A B$. Thus $Q A=Q B$, and similarly, $Q B=Q C=Q D$, hence $Q$ is just the center $O$, and $\angle L O M=$ $\angle M O N=\angle N O L=90^{\circ}$. +8. Let $P_{1}\left(x_{1}, y_{1}\right), P_{2}\left(x_{2}, y_{2}\right), \ldots, P_{n}\left(x_{n}, y_{n}\right)$ be the $n$ points of $S$ in the coordinate plane. We may assume $x_{1}216$. + +We claim that the answer is $n=217$. First notice that the set $A_{2} \cup A_{3} \cup$ $A_{5} \cup A_{7}$ consists of four prime $(2,3,5,7)$ and 212 composite numbers. The set $S \backslash A$ contains exactly 8 composite numbers: namely, $11^{2}, 11 \cdot 13,11$. $17,11 \cdot 19,11 \cdot 23,13^{2}, 13 \cdot 17,13 \cdot 19$. Thus $S$ consists of the unity, 220 composite numbers and 59 primes. +Let $A$ be a 217 -element subset of $S$, and suppose that there are no five pairwise relatively prime numbers in $A$. Then $A$ can contain at most 4 primes (or unity and three primes) and at least 213 composite numbers. Hence the set $S \backslash A$ contains at most 7 composite numbers. Consequently, at least one of the following 8 five-element sets is disjoint with $S \backslash A$, and is thus entirely contained in $A$ : + +$$ +\begin{array}{ll} +\{2 \cdot 23,3 \cdot 19,5 \cdot 17,7 \cdot 13,11 \cdot 11\}, & \{2 \cdot 29,3 \cdot 23,5 \cdot 19,7 \cdot 17,11 \cdot 13\}, \\ +\{2 \cdot 31,3 \cdot 29,5 \cdot 23,7 \cdot 19,11 \cdot 17\}, & \{2 \cdot 37,3 \cdot 31,5 \cdot 29,7 \cdot 23,11 \cdot 19\}, \\ +\{2 \cdot 41,3 \cdot 37,5 \cdot 31,7 \cdot 29,11 \cdot 23\}, & \{2 \cdot 43,3 \cdot 41,5 \cdot 37,7 \cdot 31,13 \cdot 17\}, \\ +\{2 \cdot 47,3 \cdot 43,5 \cdot 41,7 \cdot 37,13 \cdot 19\}, & \{2 \cdot 2,3 \cdot 3,5 \cdot 5,7 \cdot 7,13 \cdot 13\} . +\end{array} +$$ + +As each of these sets consists of five numbers relatively prime in pairs, the claim is proved. +13. Call a sequence $e_{1}, \ldots, e_{n}$ good if $e_{1} a_{1}+\cdots+e_{n} a_{n}$ is divisible by $n$. Among the sums $s_{0}=0, s_{1}=a_{1}, s_{2}=a_{1}+a_{2}, \ldots, s_{n}=a_{1}+\cdots+a_{n}$, two give the same remainder modulo $n$, and their difference corresponds to a good sequence. To show that, permuting the $a_{i}$ 's, we can find $n-1$ different sequences, we use the following +Lemma. Let $A$ be a $k \times n(k \leq n-2)$ matrix of zeros and ones, whose every row contains at least one 0 and at least two 1 's. Then it is possible to permute columns of $A$ is such a way that in any row 1 's do not form a block. +Proof. We will use the induction on $k$. The case $k=1$ and arbitrary $n \geq 3$ is trivial. Suppose that $k \geq 2$ and that for $k-1$ and any $n \geq k+1$ the lemma is true. Consider a $k \times n$ matrix $A, n \geq k+2$. We mark an element $a_{i j}$ if either it is the only zero in the $i$-th row, or one of the 1 's in the row if it contains exactly two 1 's. Since $n \geq 4$, every row contains at most two marked elements, which adds up to at most $2 k<2 n$ marked elements in total. It follows that there is a column with at most one marked element. Assume w.l.o.g. that it is the first column and that $a_{1 j}$ isn't marked for $j>1$. The matrix $B$, obtained by omitting the first row and first column from $A$, satisfies the conditions of the lemma. Therefore, we can permute columns of $B$ and get the required form. Considered as a permutation of column of $A$, this permutation may leave a block of 1's only in the first row of $A$. In the case that it is so, if $a_{11}=1$ we put the first column in the last place, otherwise we put it between any two columns having 1's in the first row. The obtained matrix has the required property. +Suppose now that we have got $k$ different nontrivial good sequences $e_{1}^{i}, \ldots, e_{n}^{i}, i=1, \ldots, k$, and that $k \leq n-2$. The matrix $A=\left(e_{j}^{i}\right)$ +fulfils the conditions of Lemma, hence there is a permutation $\sigma$ from Lemma. Now among the sums $s_{0}=0, s_{1}=a_{\sigma(1)}, s_{2}=a_{\sigma(1)}+a_{\sigma(2)}$, $\ldots, s_{n}=a_{\sigma(1)}+\cdots+a_{\sigma(n)}$, two give the same remainder modulo $n$. Let $s_{p} \equiv s_{q}(\bmod n), pm>$ 0 , as large as we like, such that $10^{k} \equiv 10^{m}(\bmod T)$, using for example Euler's theorem. It is obvious that $a_{10^{k}-1}=a_{10^{k}}$ and hence, taking $k$ sufficiently large and using the periodicity, we see that + +$$ +a_{2 \cdot 10^{k}-10^{m}-1}=a_{10^{k}-1}=a_{10^{k}}=a_{2 \cdot 10^{k}-10^{m}} +$$ + +Since $\left(2 \cdot 10^{k}-10^{m}\right)!=\left(2 \cdot 10^{k}-10^{m}\right)\left(2 \cdot 10^{k}-10^{m}-1\right)!$ and the last nonzero digit of $2 \cdot 10^{k}-10^{m}$ is nine, we must have $a_{2 \cdot 10^{k}-10^{m}-1}=5$ (if $s$ is a digit, the last digit of $9 s$ is $s$ only if $s=5$ ). But this means that 5 divides $n$ ! with a greater power than 2 does, which is impossible. Indeed, if the exponents of these powers are $\alpha_{2}, \alpha_{5}$ respectively, then $\alpha_{5}=$ $[n / 5]+\left[n / 5^{2}\right]+\cdots \leq \alpha_{2}=[n / 2]+\left[n / 2^{2}\right]+\cdots$. +16. Let $p$ be the least prime number that does not divide $n$ : thus $a_{1}=1$ and $a_{2}=p$. Since $a_{2}-a_{1}=a_{3}-a_{2}=\cdots=r$, the $a_{i}$ 's are $1, p, 2 p-1,3 p-2, \ldots$ We have the following cases: +$p=2$. Then $r=1$ and the numbers $1,2,3, \ldots, n-1$ are relatively prime to $n$, hence $n$ is a prime. +$p=3$. Then $r=2$, so every odd number less than $n$ is relatively prime to $n$, from which we deduce that $n$ has no odd divisors. Therefore $n=2^{k}$ for some $k \in \mathbb{N}$. +$p>3$. Then $r=p-1$ and $a_{k+1}=a_{1}+k(p-1)=1+k(p-1)$. Since $n-1$ also must belong to the progression, we have $p-1 \mid n-2$. Let $q$ be any prime divisor of $p-1$. Then also $q \mid n-2$. On the other hand, since $q0)$ modulo 3 , we get that $5^{z} \equiv 1(\bmod 3)$, hence $z$ is even, say $z=2 z_{1}$. The equation then becomes $3^{x}=5^{2 z_{1}}-4^{y}=\left(5^{z_{1}}-2^{y}\right)\left(5^{z_{1}}+2^{y}\right)$. Each factor $5^{z_{1}}-2^{y}$ and $5^{z_{1}}+2^{y}$ is a power of 3 , for which the only possibility is $5^{z_{1}}+2^{y}=3^{x}$ and $5^{z_{1}}-2^{y}=$ 1. Again modulo 3 these equations reduce to $(-1)^{z_{1}}+(-1)^{y}=0$ and $(-1)^{z_{1}}-(-1)^{y}=1$, implying that $z_{1}$ is odd and $y$ is even. Particularly, $y \geq 2$. Reducing the equation $5^{z_{1}}+2^{y}=3^{x}$ modulo 4 we get that $3^{x} \equiv 1$, hence $x$ is even. Now if $y>2$, modulo 8 this equation yields $5 \equiv 5^{z_{1}} \equiv$ $3^{x} \equiv 1$, a contradiction. Hence $y=2, z_{1}=1$. The only solution of the original equation is $x=y=z=2$. +18. For integers $a>0, n>0$ and $\alpha \geq 0$, we shall write $a^{\alpha} \| n$ when $a^{\alpha} \mid n$ and $a^{\alpha+1} \nmid n$. +Lemma. For every odd number $a \geq 3$ and an integer $n \geq 0$ it holds that + +$$ +a^{n+1} \|(a+1)^{a^{n}}-1 \quad \text { and } \quad a^{n+1} \|(a-1)^{a^{n}}+1 +$$ + +Proof. We shall prove the first relation by induction (the second is analogous). For $n=0$ the statement is obvious. Suppose that it holds for some $n$, i.e. that $(1+a)^{a^{n}}=1+N a^{n+1}, a \nmid N$. Then + +$$ +(1+a)^{a^{n+1}}=\left(1+N a^{n+1}\right)^{a}=1+a \cdot N a^{n+1}+\binom{a}{2} N^{2} a^{2 n+2}+M a^{3 n+3} +$$ + +for some integer $M$. Since $\binom{a}{2}$ is divisible by $a$ for $a$ odd, we deduce that the part of the above sum behind $1+a \cdot N a^{n+1}$ is divisible by $a^{n+3}$. Hence $(1+a)^{a^{n+1}}=1+N^{\prime} a^{n+2}$, where $a \nmid N^{\prime}$. +It follows immediately from Lemma that + +$$ +1991^{1993} \| 1990^{1991^{1992}}+1 \quad \text { and } \quad 1991^{1991} \| 1992^{1991^{1990}}-1 +$$ + +Adding these two relations we obtain immediately that $k=1991$ is the desired value. +19. Set $x=\cos (\pi a)$. The given equation is equivalent to $4 x^{3}+4 x^{2}-3 x-2=0$, which factorizes as $(2 x+1)\left(2 x^{2}+x-2\right)=0$. +The case $2 x+1=0$ yields $\cos (\pi a)=-1 / 2$ and $a=2 / 3$. It remains to show that if $x$ satisfies $2 x^{2}+x-2=0$ then $a$ is not rational. The polynomial equation $2 x^{2}+x-2=0$ has two real roots, $x_{1,2}=\frac{-1 \pm \sqrt{17}}{4}$, and since $|x| \leq 1$ we must have $x=\cos \pi a=\frac{-1+\sqrt{17}}{4}$. +We now prove by induction that, for every integer $n \geq 0, \cos \left(2^{n} \pi a\right)=$ $\frac{a_{n}+b_{n} \sqrt{17}}{4}$ for some odd integers $a_{n}, b_{n}$. The case $n=0$ is trivial. Also, if $\cos \left(2^{n} \pi a\right)=\frac{a_{n}+b_{n} \sqrt{17}}{4}$, then + +$$ +\begin{aligned} +\cos \left(2^{n+1} \pi a\right) & =2 \cos ^{2}\left(2^{n} \pi a\right)-1 \\ +& =\frac{1}{4}\left(\frac{a_{n}^{2}+17 b_{n}^{2}-8}{2}+a_{n} b_{n} \sqrt{17}\right)=\frac{a_{n+1}+b_{n+1} \sqrt{17}}{4} . +\end{aligned} +$$ + +By the inductive step that $a_{n}, b_{n}$ are odd, it is obvious that $a_{n+1}, b_{n+1}$ are also odd. This proves the claim. +Note also that, since $a_{n+1}=\frac{1}{2}\left(a_{n}^{2}+17 b_{n}^{2}-8\right)>a_{n}$, the sequence $\left\{a_{n}\right\}$ is strictly increasing. Hence the set of values of $\cos \left(2^{n} \pi a\right), n=0,1,2, \ldots$, is infinite (because $\sqrt{17}$ is irrational). However, if $a$ were rational, then the set of values of $\cos m \pi a, m=1,2, \ldots$, would be finite, a contradiction. Therefore the only possible value for $a$ is $2 / 3$. +20. We prove the result with 1991 replaced by any positive integer $k$. For natural numbers $p, q$, let $\epsilon=(\alpha p-[\alpha p])(\alpha q-[\alpha q])$. Then $0<\epsilon<1$ and + +$$ +\epsilon=\alpha^{2} p q-\alpha(p[\alpha q]+q[\alpha p])+[\alpha p][\alpha q] . +$$ + +Multiplying this equality by $\alpha-k$ and using $\alpha^{2}=k \alpha+1$, i.e. $\alpha(\alpha-k)=1$, we get + +$$ +(\alpha-k) \epsilon=\alpha(p q+[\alpha p][\alpha q])-(p[\alpha q]+q[\alpha p]+k[\alpha p][\alpha q]) +$$ + +Since $0<(\alpha-k) \epsilon<1$, we have $[\alpha(p * q)]=p[\alpha q]+q[\alpha p]+k[\alpha p][\alpha q]$. Now + +$$ +\begin{aligned} +(p * q) * r & =(p * q) r+[\alpha(p * q)][\alpha r]= \\ +& =p q r+[\alpha p][\alpha q] r+[\alpha q][\alpha r] p+[\alpha r][\alpha p] q+k[\alpha p][\alpha q][\alpha r] . +\end{aligned} +$$ + +Since the last expression is symmetric, the same formula is obtained for $p *(q * r)$. +21. The polynomial $g(x)$ factorizes as $g(x)=f(x)^{2}-9=(f(x)-3)(f(x)+3)$. If one of the equations $f(x)+3=0$ and $f(x)-3=0$ has no integer solutions, then the number of integer solutions of $g(x)=0$ clearly does not exceed 1991. +Suppose now that both $f(x)+3=0$ and $f(x)-3=0$ have integer solutions. Let $x_{1}, \ldots, x_{k}$ be distinct integer solutions of the former, and $x_{k+1}, \ldots, x_{k+l}$ be distinct integer solutions of the latter equation. There exist monic polynomials $p(x), q(x)$ with integer coefficients such that $f(x)+3=\left(x-x_{1}\right)\left(x-x_{2}\right) \ldots\left(x-x_{k}\right) p(x)$ and $f(x)-3=$ $\left(x-x_{k+1}\right)\left(x-x_{k+2}\right) \ldots\left(x-x_{k+l}\right) q(x)$. Thus we obtain +$\left(x-x_{1}\right)\left(x-x_{2}\right) \ldots\left(x-x_{k}\right) p(x)-\left(x-x_{k+1}\right)\left(x-x_{k+2}\right) \ldots\left(x-x_{k+l}\right) q(x)=6$. +Putting $x=x_{k+1}$ we get $\left(x_{k+1}-x_{1}\right)\left(x_{k+1}-x_{2}\right) \cdots\left(x_{k+1}-x_{k}\right) \mid 6$, and since the product of more than four distinct integers cannot divide 6 , this implies $k \leq 4$. Similarly $l \leq 4$; hence $g(x)=0$ has at most 8 distinct integer solutions. +Remark. The proposer provided a solution for the upper bound of 1995 roots which was essentially the same as that of (IMO74-6). +22. Suppose w.l.o.g. that the center of the square is at the origin $O(0,0)$. We denote the curve $y=f(x)=x^{3}+a x^{2}+b x+c$ by $\gamma$ and the vertices of the square by $A, B, C, D$ in this order. +At first, the symmetry with respect to the point $O$ maps $\gamma$ into the curve $\bar{\gamma}\left(y=f(-x)=x^{3}-a x^{2}+b x-c\right)$. Obviously $\bar{\gamma}$ also passes through $A, B, C, D$, and thus has four different intersection points with $\gamma$. Then $2 a x^{2}+2 c$ has at least four distinct solution, which implies $a=c=0$. Particularly, $\gamma$ passes through $O$ and intersects all quadrants, and hence $b<0$. +Further, the curve $\gamma^{\prime}$, obtained by rotation of $\gamma$ around $O$ for $90^{\circ}$, has an equation $-x=f(y)$ and also contains the points $A, B, C, D$ and $O$. The intersection points $(x, y)$ of $\gamma \cap \gamma^{\prime}$ are determined by $-x=f(f(x))$, and hence they are roots of a polynomial $p(x)=f(f(x))+x$ of 9-th degree. + +But the number of times that one cubic actually crosses the other in each quadrant is in the general case even (draw the picture!), and since $A B C D$ is the only square lying on $\gamma \cap \gamma^{\prime}$, the intersection points $A, B, C, D$ must be double. It follows that + +$$ +p(x)=x[(x-r)(x+r)(x-s)(x+s)]^{2}, +$$ + +where $r, s$ are the $x$-coordinates of $A$ and $B$. On the other hand, $p(x)$ is defined by $\left(x^{3}+b x\right)^{3}+b\left(x^{3}+b x\right)+x$, and therefore equating of coefficients with (1) yields + +$$ +\begin{array}{cc} +3 b=-2\left(r^{2}+s^{2}\right), & 3 b^{2}=\left(r^{2}+s^{2}\right)^{2}+2 r^{2} s^{2}, \\ +b\left(b^{2}+1\right)=-2 r^{2} s^{2}\left(r^{2}+s^{2}\right), & b^{2}+1=r^{4} s^{4} . +\end{array} +$$ + +Straightforward solving this system of equations gives $b=-\sqrt{8}$ and $r^{2}+$ $s^{2}=\sqrt{18}$. +The line segment from $O$ to $(r, s)$ is half a diagonal of the square, and thus a side of the square has length $a=\sqrt{2\left(r^{2}+s^{2}\right)}=\sqrt[4]{72}$. +23. From (i), replacing $m$ by $f(f(m))$, we get + +$$ +f(f(f(m))+f(f(n)))=-f(f(f(f(m))+1))-n +$$ + +analogously $\quad f(f(f(n))+f(f(m)))=-f(f(f(f(n))+1))-m$. +From these relations we get $f(f(f(f(m))+1))-f(f(f(f(n))+1))=m-n$. Again from (i), + +$$ +\begin{aligned} +& f(f(f(f(m))+1))=f(-m-f(f(2))) \\ +& \text { and } \quad f(f(f(f(n))+1))=f(-n-f(f(2))) . +\end{aligned} +$$ + +Setting $f(f(2))=k$ we obtain $f(-m-k)-f(-n-k)=m-n$ for all integers $m, n$. This implies $f(m)=f(0)-m$. Then also $f(f(m))=m$, and using this in (i) we finally get + +$$ +f(n)=-n-1 \quad \text { for all integers } n +$$ + +Particularly $f(1991)=-1992$. +From (ii) we obtain $g(n)=g(-n-1)$ for all integers $n$. Since $g$ is a polynomial, it must also satisfy $g(x)=g(-x-1)$ for all real $x$. Let us now express $g$ as a polynomial on $x+1 / 2: g(x)=h(x+1 / 2)$. Then $h$ satisfies $h(x+1 / 2)=h(-x-1 / 2)$, i.e. $h(y)=h(-y)$, hence it is a polynomial in $y^{2}$; thus $g$ is a polynomial in $(x+1 / 2)^{2}=x^{2}+x+1 / 4$. Hence $g(n)=p\left(n^{2}+n\right)$ (for some polynomial $p$ ) is the most general form of $g$. +24. Let $y_{k}=a_{k}-a_{k+1}+a_{k+2}-\cdots+a_{k+n-1}$ for $k=1,2, \ldots, n$, where we define $x_{i+n}=x_{i}$ for $1 \leq i \leq n$. We then have $y_{1}+y_{2}=2 a_{1}, y_{2}+y_{3}=$ $2 a_{2}, \ldots, y_{n}+y_{1}=2 a_{n}$. +(i) Let $n=4 k-1$ for some integer $k>0$. Then for each $i=1,2, \ldots, n$ we have that $y_{i}=\left(a_{i}+a_{i+1}+\cdots+a_{i-1}\right)-2\left(a_{i+1}+a_{i+3}+\cdots+a_{i-2}\right)=1+$ $2+\cdots+(4 k-1)-2\left(a_{i+1}+a_{i+3}+\cdots+a_{i-2}\right)$ is even. Suppose now that $a_{1}, \ldots, a_{n}$ is a good permutation. Then each $y_{i}$ is positive and even, so $y_{i} \geq 2$. But for some $t \in\{1, \ldots, n\}$ we must have $a_{t}=1$, and thus $y_{t}+y_{t+1}=2 a_{t}=2$ which is impossible. Hence the numbers $n=4 k-1$ are not good. +(ii) Let $n=4 k+1$ for some integer $k>0$. Then $2,4, \ldots, 4 k, 4 k+1,4 k-$ $1, \ldots, 3,1$ is a permutation with the desired property. Indeed, in this case $y_{1}=y_{4 k+1}=1, y_{2}=y_{4 k}=3, \ldots, y_{2 k}=y_{2 k+2}=4 k-1$, $y_{2 k+1}=4 k+1$. +Therefore all nice numbers are given by $4 k+1, k \in \mathbb{N}$. +25. Since replacing $x_{1}$ by 1 can only reduce the set of indices $i$ for which the desired inequality holds, we may assume $x_{1}=1$. Similarly we may assume $x_{n}=0$. Now we can let $i$ be the largest index such that $x_{i}>1 / 2$. Then $x_{i+1} \leq 1 / 2$, hence + +$$ +x_{i}\left(1-x_{i+1}\right) \geq \frac{1}{4}=\frac{1}{4} x_{1}\left(1-x_{n}\right) . +$$ + +26. Without loss of generality we can assume $b_{1} \geq b_{2} \geq \cdots \geq b_{n}$. We denote by $A_{i}$ the product $a_{1} a_{2} \ldots a_{i-1} a_{i+1} \ldots a_{n}$. If for some $i0 +$$ + +because $x_{k-1}+x_{k} \leq \frac{2}{3}\left(x_{1}+x_{k-1}+x_{k-2}\right) \leq \frac{2}{3}$. Repeating this procedure we can reduce the number of nonzero $x_{i}$ 's to two, increasing the value of $F$ in each step. It remains to maximize $F$ over $n$-tuples $\left(x_{1}, x_{2}, 0, \ldots, 0\right)$ with $x_{1}, x_{2} \geq 0, x_{1}+x_{2}=1$ : in this case $F$ equals $x_{1} x_{2}$ and attains its maximum value $\frac{1}{4}$ when $x_{1}=x_{2}=\frac{1}{2}, x_{3}=\ldots, x_{n}=0$. +28. Let $x_{n}=c(n \sqrt{2}-[n \sqrt{2}])$ for some constant $c>0$. For $i>j$, putting $p=[i \sqrt{2}]-[j \sqrt{2}]$, we have +$\left|x_{i}-x_{j}\right|=c|(i-j) \sqrt{2}-p|=\frac{\left|2(i-j)^{2}-p^{2}\right| c}{(i-j) \sqrt{2}+p} \geq \frac{c}{(i-j) \sqrt{2}+p} \geq \frac{c}{4(i-j)}$, +because $p<(i-j) \sqrt{2}+1$. Taking $c=4$, we obtain that for any $i>j$, $(i-j)\left|x_{i}-x_{j}\right| \geq 1$. Of course, this implies $(i-j)^{a}\left|x_{i}-x_{j}\right| \geq 1$ for any $a>1$ 。 +Remark. The constant 4 can be replaced with $3 / 2+\sqrt{2}$. +Second solution. Another example of a sequence $\left\{x_{n}\right\}$ is constructed in the following way: $x_{1}=0, x_{2}=1, x_{3}=2$ and $x_{3^{k} i+m}=x_{m}+\frac{i}{3^{k}}$ for $i=1,2$ and $1 \leq m \leq 3^{k}$. It is easily shown that $|i-j| \cdot\left|x_{i}-x_{j}\right| \geq 1 / 3$ for any $i \neq j$. +Third solution. If $n=b_{0}+2 b_{1}+\cdots+2^{k} b_{k}, b_{i} \in\{0,1\}$, then one can set $x_{n}$ to be $=b_{0}+2^{-a} b_{1}+\cdots+2^{-k a} b_{k}$. In this case it holds that $|i-j|^{a}\left|x_{i}-x_{j}\right| \geq$ $\frac{2^{a}-2}{2^{a}-1}$. +29. One easily observes that the following sets are super-invariant: one-point set, its complement, closed and open half-lines or their complements, and the whole real line. To show that these are the only possibilities, we first observe that $S$ is super-invariant if and only if for each $a>0$ there is a $b$ such that $x \in S \Leftrightarrow a x+b \in S$. +(i) Suppose that for some $a$ there are two such $b$ 's: $b_{1}$ and $b_{2}$. Then $x \in$ $S \Leftrightarrow a x+b_{1} \in S$ and $x \in S \Leftrightarrow a x+b_{2} \in S$, which implies that $S$ is periodic: $y \in S \Leftrightarrow y+\frac{b_{1}-b_{2}}{a} \in S$. Since $S$ is identical to a translate of any stretching of $S$, all positive numbers are periods of $S$. Therefore $S \equiv \mathbb{R}$. +(ii) Assume that, for each $a, b=f(a)$ is unique. Then for any $a_{1}$ and $a_{2}$, + +$$ +\begin{aligned} +x \in S & \Leftrightarrow a_{1} x+f\left(a_{1}\right) \in S \Leftrightarrow a_{1} a_{2} x+a_{2} f\left(a_{1}\right)+f\left(a_{2}\right) \in S \\ +& \Leftrightarrow a_{2} x+f\left(a_{2}\right) \in S \Leftrightarrow a_{1} a_{2} x+a_{1} f\left(a_{2}\right)+f\left(a_{1}\right) \in S . +\end{aligned} +$$ + +As above it follows that $a_{1} f\left(a_{2}\right)+f\left(a_{1}\right)=a_{2} f\left(a_{1}\right)+f\left(a_{2}\right)$, or equivalently $f\left(a_{1}\right)\left(a_{2}-1\right)=f\left(a_{2}\right)\left(a_{1}-1\right)$. Hence (for some $\left.c\right), f(a)=c(a-1)$ for all $a$. Now $x \in S \Leftrightarrow a x+c(a-1) \in S$ actually means that $y-c \in S \Leftrightarrow a y-c \in S$ for all $a$. Then it is easy to conclude that $\{y-c \mid y \in S\}$ is either a half-line or the whole line, and so is $S$. +30. Let $a$ and $b$ be the integers written by $A$ and $B$ respectively, and let $xx-r_{n-1}, B$ would know that $a+b>x$ and hence $a+b=y$, while if $b0$, there exists an $m$ for which $s_{m}-r_{m}<0$, a contradiction. + +### 4.33 Solutions to the Shortlisted Problems of IMO 1992 + +1. Assume that a pair $(x, y)$ with $xy$ is a positive integer). This will imply the desired result, since starting from the pair $(1,1)$ we can obtain arbitrarily many solutions. First, we show that $\operatorname{gcd}\left(x_{1}, y\right)=1$. Suppose to the contrary that $\operatorname{gcd}\left(x_{1}, y\right)$ $=d>1$. Then $d\left|x_{1}\right| y^{2}+m \Rightarrow d \mid m$, which implies $d|y| x^{2}+m \Rightarrow d \mid x$. But this last is impossible, since $\operatorname{gcd}(x, y)=1$. Thus it remains to show that $x_{1} \mid y^{2}+m$ and $y \mid x_{1}^{2}+m$. The former relation is obvious. Since $\operatorname{gcd}(x, y)=1$, the latter is equivalent to $y \mid\left(x x_{1}\right)^{2}+m x^{2}=y^{4}+2 m y^{2}+$ $m^{2}+m x^{2}$, which is true because $y \mid m\left(m+x^{2}\right)$ by the assumption. Hence $\left(y, x_{1}\right)$ indeed satisfies all the required conditions. +Remark. The original problem asked to prove the existence of a pair $(x, y)$ of positive integers satisfying the given conditions such that $x+y \leq m+1$. The problem in this formulation is trivial, since the pair $x=y=1$ satisfies the conditions. Moreover, this is sometimes the only solution with $x+y \leq m+1$. For example, for $m=3$ the least nontrivial solution is $\left(x_{0}, y_{0}\right)=(1,4)$. +2. Let us define $x_{n}$ inductively as $x_{n}=f\left(x_{n-1}\right)$, where $x_{0} \geq 0$ is a fixed real number. It follows from the given equation in $f$ that $x_{n+2}=-a x_{n+1}+$ $b(a+b) x_{n}$. The general solution to this equation is of the form + +$$ +x_{n}=\lambda_{1} b^{n}+\lambda_{2}(-a-b)^{n}, +$$ + +where $\lambda_{1}, \lambda_{2} \in \mathbb{R}$ satisfy $x_{0}=\lambda_{1}+\lambda_{2}$ and $x_{1}=\lambda_{1} b-\lambda_{2}(a+b)$. In order to have $x_{n} \geq 0$ for all $n$ we must have $\lambda_{2}=0$. Hence $x_{0}=\lambda_{1}$ and $f\left(x_{0}\right)=x_{1}=\lambda_{1} b=b x_{0}$. Since $x_{0}$ was arbitrary, we conclude that $f(x)=b x$ is the only possible solution of the functional equation. It is easily verified that this is indeed a solution. +3. Consider two squares $A B^{\prime} C D^{\prime}$ and $A^{\prime} B C^{\prime} D$. Since $A C \perp B D$, these two squares are homothetic, which implies that the lines $A A^{\prime}, B B^{\prime}, C C^{\prime}, D D^{\prime}$ are concurrent at a certain point $O$. Since the rotation about $A$ by $90^{\circ}$ takes $\triangle A B K$ into $\triangle A F D$, it follows that $B K \perp D F$. Denote by $T$ the intersection of $B K$ and $D F$. The rotation about some point $X$ by $90^{\circ}$ maps $B K$ into $D F$ if and only if $T X$ bisects an angle between $B K$ and $D F$. Therefore $\angle F T A=$ +![](https://cdn.mathpix.com/cropped/2024_11_18_8e985d6b9c83aa3e9d0eg-567.jpg?height=342&width=470&top_left_y=1659&top_left_x=847) +$\angle A T K=45^{\circ}$. Moreover, the quadrilateral $B A^{\prime} D T$ is cyclic, which implies that $\angle B T A^{\prime}=B D A^{\prime}=45^{\circ}$ and consequently that the points $A, T, A^{\prime}$ are collinear. It follows that the +point $O$ lies on a bisector of $\angle B T D$ and therefore the rotation $\mathcal{R}$ about $O$ by $90^{\circ}$ takes $B K$ into $D F$. Analogously, $\mathcal{R}$ maps the lines $C E, D G, A I$ into $A H, B J, C L$. Hence the quadrilateral $P_{1} Q_{1} R_{1} S_{1}$ is the image of the quadrilateral $P_{2} Q_{2} R_{2} S_{2}$, and the result follows. +4. There are 36 possible edges in total. If not more than 3 edges are left undrawn, then we can choose 6 of the given 9 points no two of which are connected by an undrawn edge. These 6 points together with the edges between them form a two-colored complete graph, and thus by a wellknown result there exists at least one monochromatic triangle. It follows that $n \leq 33$. +In order to show that $n=33$, we shall give an example of a graph with 32 edges that does not contain a monochromatic triangle. Let us start with a complete graph $C_{5}$ with 5 vertices. Its edges can be colored in two colors so that there is no monochromatic triangle (Fig. 1). Furthermore, given a graph $\mathcal{H}$ with $k$ vertices without monochromatic triangles, we can add to it a new vertex, join it to all vertices of $\mathcal{H}$ except $A$, and color each edge $B X$ in the same way as $A X$. The obtained graph obviously contains no monochromatic triangles. Applying this construction four times to the graph $C_{5}$ we get an example like that of Fig. 2. +![](https://cdn.mathpix.com/cropped/2024_11_18_8e985d6b9c83aa3e9d0eg-568.jpg?height=338&width=396&top_left_y=1069&top_left_x=326) + +Fig. 1 +![](https://cdn.mathpix.com/cropped/2024_11_18_8e985d6b9c83aa3e9d0eg-568.jpg?height=326&width=396&top_left_y=1079&top_left_x=868) + +Fig. 2 + +Second solution. For simplicity, we call the colors red and blue. +Let $r(k, l)$ be the least positive integer $r$ such that each complete $r$-graph whose edges are colored in red and blue contains either a complete red $k$-graph or a complete blue $l$-graph. Also, let $t(n, k)$ be the greatest possible number of edges in a graph with $n$ vertices that does not contain a complete $k$-graph. These numbers exist by the theorems of Ramsey and Turán. +Let us assume that $r(k, l)0$ for all $x>0$. It also follows that $f(x)<0$ for $x<0$. In other words, $f$ preserves sign. +Now setting $x>0$ and $y=-f(x)$ in the given functional equation we obtain + +$$ +f(x-f(x))=f\left(\sqrt{x}^{2}+f(-x)\right)=-x+f(\sqrt{x})^{2}=-(x-f(x)) . +$$ + +But since $f$ preserves sign, this implies that $f(x)=x$ for $x>0$. Moreover, since $f(-x)=-f(x)$, it follows that $f(x)=x$ for all $x$. It is easily verified that this is indeed a solution. +7. Let $G_{1}, G_{2}$ touch the chord $B C$ at $P, Q$ and touch the circle $G$ at $R, S$ respectively. Let $D$ be the midpoint of the complementary $\operatorname{arc} B C$ of $G$. The homothety centered at $R$ mapping $G_{1}$ onto $G$ also maps the line $B C$ onto a tangent of $G$ parallel to $B C$. It follows that this line touches $G$ at point $D$, which is therefore the image of $P$ under the homothety. Hence $R, P$, and $D$ are collinear. Since $\angle D B P=\angle D C B=\angle D R B$, it follows that $\triangle D B P \sim \triangle D R B$ and consequently that $D P \cdot D R=D B^{2}$. Similarly, points $S, Q, D$ are collinear and satisfy $D Q \cdot D S=D B^{2}=D P \cdot D R$. + +Hence $D$ lies on the radical axis of the circles $G_{1}$ and $G_{2}$, i.e., on their common tangent $A W$, which also implies that $A W$ bisects the angle $B A D$. Furthermore, since $D B=D C=D W=\sqrt{D P \cdot D R}$, it follows from the lemma of (SL99-14) that $W$ is the incenter of $\triangle A B C$. +Remark. According to the third solution of (SL93-3), both $P W$ and $Q W$ contain the incenter of $\triangle A B C$, and the result is immediate. The problem can also be solved by inversion centered at $W$. +8. For simplicity, we shall write $n$ instead of 1992. + +Lemma. There exists a tangent $n$-gon $A_{1} A_{2} \ldots A_{n}$ with sides $A_{1} A_{2}=a_{1}$, $A_{2} A_{3}=a_{2}, \ldots, A_{n} A_{1}=a_{n}$ if and only if the system + +$$ +x_{1}+x_{2}=a_{1}, x_{2}+x_{3}=a_{2}, \ldots, x_{n}+x_{1}=a_{n} +$$ + +has a solution $\left(x_{1}, \ldots, x_{n}\right)$ in positive reals. +Proof. Suppose that such an $n$-gon $A_{1} A_{2} \ldots A_{n}$ exists. Let the side $A_{i} A_{i+1}$ touch the inscribed circle at point $P_{i}$ (where $\left.A_{n+1}=A_{1}\right)$. Then $x_{1}=$ $A_{1} P_{n}=A_{1} P_{1}, x_{2}=A_{2} P_{1}=A_{2} P_{2}, \ldots, x_{n}=A_{n} P_{n-1}=A_{n} P_{n}$ is clearly a positive solution of (1). +Now suppose that the system (1) has a positive real solution $\left(x_{1}, \ldots\right.$, $x_{n}$ ). Let us draw a polygonal line $A_{1} A_{2} \ldots A_{n+1}$ touching a circle of radius $r$ at points $P_{1}, P_{2}, \ldots, P_{n}$ respectively such that $A_{1} P_{1}=$ $A_{n+1} P_{n}=x_{1}$ and $A_{i} P_{i}=A_{i} P_{i-1}=x_{i}$ for $i=2, \ldots, n$. Observe that $O A_{1}=O A_{n+1}=\sqrt{x_{1}^{2}+r^{2}}$ and the function $f(r)=\angle A_{1} O A_{2}+$ $\angle A_{2} O A_{3}+\cdots+\angle A_{n} O A_{n+1}=$ $2\left(\arctan \frac{x_{1}}{r}+\cdots+\arctan \frac{x_{n}}{r}\right)$ is continuous. Thus $A_{1} A_{2} \ldots A_{n+1}$ is a closed simple polygonal line if and only if $f(r)=360^{\circ}$. But such an $r$ exists, since $f(r) \rightarrow 0$ +![](https://cdn.mathpix.com/cropped/2024_11_18_8e985d6b9c83aa3e9d0eg-570.jpg?height=331&width=444&top_left_y=1156&top_left_x=887) +when $r \rightarrow \infty$ and $f(r) \rightarrow \infty$ when $r \rightarrow 0$. This proves the second direction of the lemma. +For $n=4 k$, the system (1) is solvable in positive reals if $a_{i}=i$ for $i \equiv 1,2$ $(\bmod 4), a_{i}=i+1$ for $i \equiv 3$ and $a_{i}=i-1$ for $i \equiv 0(\bmod 4)$. Indeed, one solution is given by $x_{i}=1 / 2$ for $i \equiv 1, x_{i}=3 / 2$ for $i \equiv 3$ and $x_{i}=i-3 / 2$ for $i \equiv 0,2(\bmod 4)$. +Remark. For $n=4 k+2$ there is no such $n$-gon. In fact, solvability of the system (1) implies $a_{1}+a_{3}+\cdots=a_{2}+a_{4}+\cdots$, while in the case $n=4 k+2$ the sum $a_{1}+a_{2}+\cdots+a_{n}$ is odd. +9. Since the equation $x^{3}-x-c=0$ has only one real root for every $c>$ $2 /(3 \sqrt{3}), \alpha$ is the unique real root of $x^{3}-x-33^{1992}=0$. Hence $f^{n}(\alpha)=$ $f(\alpha)=\alpha$. +Remark. Consider any irreducible polynomial $g(x)$ in the place of $x^{3}-$ $x-33^{1992}$. The problem amounts to proving that if $\alpha$ and $f(\alpha)$ are roots +of $g$, then any $f^{(n)}(\alpha)$ is also a root of $g$. In fact, since $g(f(x))$ vanishes at $x=\alpha$, it must be divisible by the minimal polynomial of $\alpha$, that is, $g(x)$. It follows by induction that $g\left(f^{(n)}(x)\right)$ is divisible by $g(x)$ for all $n \in \mathbb{N}$, and hence $g\left(f^{(n)}(\alpha)\right)=0$. +10. Let us set $S(x)=\{(y, z) \mid(x, y, z) \in V\}, S_{y}(x)=\left\{z \mid(x, z) \in S_{y}\right\}$ and $S_{z}(x)=\left\{y \mid(x, y) \in S_{z}\right\}$. Clearly $S(x) \subset S_{x}$ and $S(x) \subset S_{y}(x) \times S_{z}(x)$. It follows that + +$$ +\begin{aligned} +|V| & =\sum_{x}|S(x)| \leq \sum_{x} \sqrt{\left|S_{x}\right|\left|S_{y}(x)\right|\left|S_{z}(x)\right|} \\ +& =\sqrt{\left|S_{x}\right|} \sum_{x} \sqrt{\left|S_{y}(x)\right|\left|S_{z}(x)\right|} +\end{aligned} +$$ + +Using the Cauchy-Schwarz inequality we also get + +$$ +\sum_{x} \sqrt{\left|S_{y}(x)\right|\left|S_{z}(x)\right|} \leq \sqrt{\sum_{x}\left|S_{y}(x)\right|} \sqrt{\sum_{x}\left|S_{z}(x)\right|}=\sqrt{\left|S_{y}\right|\left|S_{z}\right|} +$$ + +Now (1) and (2) together yield $|V| \leq \sqrt{\left|S_{x}\right|\left|S_{y}\right|\left|S_{z}\right|}$. +11. Let $I$ be the incenter of $\triangle A B C$. Since $90^{\circ}+\alpha / 2=\angle B I C=\angle D I E=$ $138^{\circ}$, we obtain that $\angle A=96^{\circ}$. +![](https://cdn.mathpix.com/cropped/2024_11_18_8e985d6b9c83aa3e9d0eg-571.jpg?height=295&width=1043&top_left_y=1086&top_left_x=260) + +Let $D^{\prime}$ and $E^{\prime}$ be the points symmetric to $D$ and $E$ with respect to $C E$ and $B D$ respectively, and let $S$ be the intersection point of $E D^{\prime}$ and $B D$. Then $\angle B D E^{\prime}=24^{\circ}$ and $\angle D^{\prime} D E^{\prime}=\angle D^{\prime} D E-\angle E^{\prime} D E=24^{\circ}$, which means that $D E^{\prime}$ bisects the angle $S D D^{\prime}$. Moreover, $\angle E^{\prime} S B=\angle E S B=$ $\angle E D S+\angle D E S=60^{\circ}$ and hence $S E^{\prime}$ bisects the angle $D^{\prime} S B$. It follows that $E^{\prime}$ is the excenter of $\triangle D^{\prime} D S$ and consequently $\angle D^{\prime} D C=\angle D D^{\prime} C=$ $\angle S D^{\prime} E^{\prime}=\left(180^{\circ}-72^{\circ}\right) / 2=54^{\circ}$. Finally, $\angle C=180^{\circ}-2 \cdot 54^{\circ}=72^{\circ}$ and $\angle B=12^{\circ}$. +12. Let us set $\operatorname{deg} f=n$ and $\operatorname{deg} g=m$. We shall prove the result by induction on $n$. If $n1$. It is easy to see that $\alpha\left(n_{m}\right)=2^{m}-m$. On the other hand, squaring and simplifying yields $n_{m}^{2}=1+\sum_{i1$ be a constant to be chosen later, and let $N_{i}=2^{m_{i}} n_{i}-1$ where $m_{i}>\alpha\left(n_{i}\right)$ is such that $m_{i} / \alpha\left(n_{i}\right) \rightarrow \theta$ as $i \rightarrow \infty$. Then $\alpha\left(N_{i}\right)=\alpha\left(n_{i}\right)+m_{i}-1$, whereas $N_{i}^{2}=2^{2 m_{i}} n_{i}^{2}-2^{m_{i}+1} n_{i}+1$ and $\alpha\left(N_{i}^{2}\right)=\alpha\left(n_{i}^{2}\right)-\alpha\left(n_{i}\right)+m_{i}$. It follows that + +$$ +\lim _{i \rightarrow \infty} \frac{\alpha\left(N_{i}^{2}\right)}{\alpha\left(N_{i}\right)}=\lim _{i \rightarrow \infty} \frac{\alpha\left(n_{i}^{2}\right)+(\theta-1) \alpha\left(n_{i}\right)}{(1+\theta) \alpha\left(n_{i}\right)}=\frac{\theta-1}{\theta+1} +$$ + +which is equal to $\gamma \in[0,1]$ for $\theta=\frac{1+\gamma}{1-\gamma}$ (for $\gamma=1$ we set $m_{i} / \alpha\left(n_{i}\right) \rightarrow$ $\infty)$. +(3) Let be given a sequence $\left(n_{i}\right)_{i=1}^{\infty}$ with $\alpha\left(n_{i}^{2}\right) / \alpha\left(n_{i}\right) \rightarrow \gamma$. Taking $m_{i}>$ $\alpha\left(n_{i}\right)$ and $N_{i}=2^{m_{i}} n_{i}+1$ we easily find that $\alpha\left(N_{i}\right)=\alpha\left(n_{i}\right)+1$ and $\alpha\left(N_{i}^{2}\right)=\alpha\left(n_{i}^{2}\right)+\alpha\left(n_{i}\right)+1$. Hence $\alpha\left(N_{i}^{2}\right) / \alpha\left(N_{i}\right)=\gamma+1$. Continuing this procedure we can construct a sequence $t_{i}$ such that $\alpha\left(t_{i}^{2}\right) / \alpha\left(t_{i}\right)=$ $\gamma+k$ for an arbitrary $k \in \mathbb{N}$. +18. Let us define inductively $f^{1}(x)=f(x)=\frac{1}{x+1}$ and $f^{n}(x)=f\left(f^{n-1}(x)\right)$, and let $g_{n}(x)=x+f(x)+f^{2}(x)+\cdots+f^{n}(x)$. We shall prove first the following statement. + +Lemma. The function $g_{n}(x)$ is strictly increasing on $[0,1]$, and $g_{n-1}(1)=$ $F_{1} / F_{2}+F_{2} / F_{3}+\cdots+F_{n} / F_{n+1}$. +Proof. Since $f(x)-f(y)=\frac{y-x}{(1+x)(1+y)}$ is smaller in absolute value than $x-y$, it follows that $x>y$ implies $f^{2 k}(x)>f^{2 k}(y)$ and $f^{2 k+1}(x)<$ $f^{2 k+1}(y)$, and moreover that for every integer $k \geq 0$, + +$$ +\left[f^{2 k}(x)-f^{2 k}(y)\right]+\left[f^{2 k+1}(x)-f^{2 k+1}(y)\right]>0 +$$ + +Hence if $x>y$, we have $g_{n}(x)-g_{n}(y)=(x-y)+[f(x)-f(y)]+\cdots+$ $\left[f^{n}(x)-f^{n}(y)\right]>0$, which yields the first part of the lemma. +The second part follows by simple induction, since $f^{k}(1)=F_{k+1} / F_{k+2}$. If some $x_{i}=0$ and consequently $x_{j}=0$ for all $j \geq i$, then the problem reduces to the problem with $i-1$ instead of $n$. Thus we may assume that all $x_{1}, \ldots, x_{n}$ are different from 0 . If we write $a_{i}=\left[1 / x_{i}\right]$, then $x_{i}=\frac{1}{a_{i}+x_{i+1}}$. Thus we can regard $x_{i}$ as functions of $x_{n}$ depending on $a_{1}, \ldots, a_{n-1}$. +Suppose that $x_{n}, a_{n-1}, \ldots, a_{3}, a_{2}$ are fixed. Then $x_{2}, x_{3}, \ldots, x_{n}$ are all fixed, and $x_{1}=\frac{1}{a_{1}+x_{2}}$ is maximal when $a_{1}=1$. Hence the sum $S=$ $x_{1}+x_{2}+\cdots+x_{n}$ is maximized for $a_{1}=1$. +We shall show by induction on $i$ that $S$ is maximized for $a_{1}=a_{2}=\cdots=$ $a_{i}=1$. In fact, assuming that the statement holds for $i-1$ and thus $a_{1}=$ $\cdots=a_{i-1}=1$, having $x_{n}, a_{n-1}, \ldots, a_{i+1}$ fixed we have that $x_{n}, \ldots, x_{i+1}$ are also fixed, and that $x_{i-1}=f\left(x_{i}\right), \ldots, x_{1}=f^{i-1}\left(x_{i}\right)$. Hence by the lemma, $S=g_{i-1}\left(x_{i}\right)+x_{i+1}+\cdots+x_{n}$ is maximal when $x_{i}=\frac{1}{a_{i}+x_{i+1}}$ is maximal, that is, for $a_{i}=1$. Thus the induction is complete. +It follows that $x_{1}+\cdots+x_{n}$ is maximal when $a_{1}=\cdots=a_{n-1}=1$, so that $x_{1}+\cdots+x_{n}=g_{n-1}\left(x_{1}\right)$. By the lemma, the latter does not exceed $g_{n-1}(1)$. This completes the proof. +Remark. The upper bound is the best possible, because it is approached by taking $x_{n}$ close to 1 and inductively (in reverse) defining $x_{i-1}=\frac{1}{1+x_{i}}=$ $\frac{1}{a_{i}+x_{i}}$. +19. Observe that $f(x)=\left(x^{4}+2 x^{2}+3\right)^{2}-8\left(x^{2}-1\right)^{2}=\left[x^{4}+2(1-\sqrt{2}) x^{2}+\right.$ $3+2 \sqrt{2}]\left[x^{4}+2(1+\sqrt{2}) x^{2}+3-2 \sqrt{2}\right]$. Now it is easy to find that the roots of $f$ are + +$$ +x_{1,2,3,4}= \pm i(i \sqrt[4]{2} \pm 1) \quad \text { and } \quad x_{5,6,7,8}= \pm i(\sqrt[4]{2} \pm 1) +$$ + +In other words, $x_{k}=\alpha_{i}+\beta_{j}$, where $\alpha_{i}^{2}=-1$ and $\beta_{j}^{4}=2$. +We claim that any root of $f$ can be obtained from any other using rational functions. In fact, we have + +$$ +\begin{aligned} +& x^{3}=-\alpha_{i}-3 \beta_{j}+3 \alpha_{i} \beta_{j}^{2}+\beta_{j}^{3} \\ +& x^{5}=11 \alpha_{i}+7 \beta_{j}-10 \alpha_{i} \beta_{j}^{2}-10 \beta_{j}^{3} \\ +& x^{7}=-71 \alpha_{i}-49 \beta_{j}+35 \alpha_{i} \beta_{j}^{2}+37 \beta_{j}^{3} +\end{aligned} +$$ + +from which we easily obtain that +$\alpha_{i}=24^{-1}\left(127 x+5 x^{3}+19 x^{5}+5 x^{7}\right), \quad \beta_{j}=24^{-1}\left(151 x+5 x^{3}+19 x^{5}+5 x^{7}\right)$. +Since all other values of $\alpha$ and $\beta$ can be obtained as rational functions of $\alpha_{i}$ and $\beta_{j}$, it follows that all the roots $x_{l}$ are rational functions of a particular root $x_{k}$. +We now note that if $x_{1}$ is an integer such that $f\left(x_{1}\right)$ is divisible by $p$, then $p>3$ and $x_{1} \in \mathbb{Z}_{p}$ is a root of the polynomial $f$. By the previous consideration, all remaining roots $x_{2}, \ldots, x_{8}$ of $f$ over the field $\mathbb{Z}_{p}$ are rational functions of $x_{1}$, since 24 is invertible in $Z_{p}$. Then $f(x)$ factors as + +$$ +f(x)=\left(x-x_{1}\right)\left(x-x_{2}\right) \cdots\left(x-x_{8}\right), +$$ + +and the result follows. +20. Denote by $U$ the point of tangency of the circle $C$ and the line $l$. Let $X$ and $U^{\prime}$ be the points symmetric to $U$ with respect to $S$ and $M$ respectively; these points do not depend on the choice of $P$. Also, let $C^{\prime}$ be the excircle of $\triangle P Q R$ corresponding to $P, S^{\prime}$ the center of $C^{\prime}$, and $W, W^{\prime}$ the points of tangency of $C$ and $C^{\prime}$ with the line $P Q$ respectively. Obviously, $\triangle W S P \sim \triangle W^{\prime} S^{\prime} P$. Since $S X \| S^{\prime} U^{\prime}$ and $S X: S^{\prime} U^{\prime}=$ $S W: S^{\prime} W^{\prime}=S P: S^{\prime} P$, we deduce that $\Delta S X P \sim \Delta S^{\prime} U^{\prime} P$, and consequently that $P$ lies on the line $X U^{\prime}$. On the other hand, it is easy to show that each point $P$ of the ray $U^{\prime} X$ over $X$ satisfies the required condition. Thus the desired locus is +![](https://cdn.mathpix.com/cropped/2024_11_18_8e985d6b9c83aa3e9d0eg-575.jpg?height=543&width=539&top_left_y=908&top_left_x=799) +the extension of $U^{\prime} X$ over $X$. +21. (a) Representing $n^{2}$ as a sum of $n^{2}-13$ squares is equivalent to representing 13 as a sum of numbers of the form $x^{2}-1, x \in \mathbb{N}$, such as $0,3,8,15, \ldots$ But it is easy to check that this is impossible, and hence $s(n) \leq n^{2}-14$. +(b) Let us prove that $s(13)=13^{2}-14=155$. Observe that + +$$ +\begin{aligned} +13^{2} & =8^{2}+8^{2}+4^{2}+4^{2}+3^{2} \\ +& =8^{2}+8^{2}+4^{2}+4^{2}+2^{2}+2^{2}+1^{2} \\ +& =8^{2}+8^{2}+4^{2}+3^{2}+3^{2}+2^{2}+1^{2}+1^{2}+1^{2} +\end{aligned} +$$ + +Given any representation of $n^{2}$ as a sum of $m$ squares one of which is even, we can construct a representation as a sum of $m+3$ squares by dividing the odd square into four equal squares. Thus the first equality enables us to construct representations with $5,8,11, \ldots, 155$ squares, the second to construct ones with $7,10,13, \ldots, 154$ squares, and the +third with $9,12, \ldots, 153$ squares. It remains only to represent $13^{2}$ as a sum of $k=2,3,4,6$ squares. This can be done as follows: + +$$ +\begin{aligned} +13^{2} & =12^{2}+5^{2}=12^{2}+4^{2}+3^{2} \\ +& =11^{2}+4^{2}+4^{2}+4^{2}=12^{2}+3^{2}+2^{2}+2^{2}+2^{2}+2^{2} +\end{aligned} +$$ + +(c) We shall prove that whenever $s(n)=n^{2}-14$ for some $n \geq 13$, it also holds that $s(2 n)=(2 n)^{2}-14$. This will imply that $s(n)=n^{2}-14$ for any $n=2^{t} \cdot 13$. +If $n^{2}=x_{1}^{2}+\cdots+x_{r}^{2}$, then we have $(2 n)^{2}=\left(2 x_{1}\right)^{2}+\cdots+\left(2 x_{r}\right)^{2}$. Replacing $\left(2 x_{i}\right)^{2}$ with $x_{i}^{2}+x_{i}^{2}+x_{i}^{2}+x_{i}^{2}$ as long as it is possible we can obtain representations of $(2 n)^{2}$ consisting of $r, r+3, \ldots, 4 r$ squares. This gives representations of $(2 n)^{2}$ into $k$ squares for any $k \leq 4 n^{2}-62$. Further, we observe that each number $m \geq 14$ can be written as a sum of $k \geq m$ numbers of the form $x^{2}-1, x \in \mathbb{N}$, which is easy to verify. Therefore if $k \leq 4 n^{2}-14$, it follows that $4 n^{2}-k$ is a sum of $k$ numbers of the form $x^{2}-1$ (since $k \geq 4 n^{2}-k \geq 14$ ), and consequently $4 n^{2}$ is a sum of $k$ squares. +Remark. One can find exactly the value of $f(n)$ for each $n$ : + +$$ +f(n)= \begin{cases}1, & \text { if } n \text { has a prime divisor congruent to } 3 \bmod 4 \\ 2, & \text { if } n \text { is of the form } 5 \cdot 2^{k}, k \text { a positive integer; } \\ n^{2}-14, & \text { otherwise. }\end{cases} +$$ + +### 4.34 Solutions to the Shortlisted Problems of IMO 1993 + +1. First we notice that for a rational point $O$ (i.e., with rational coordinates), there exist 1993 rational points in each quadrant of the unit circle centered at $O$. In fact, it suffices to take + +$$ +X=\left\{\left.O+\left( \pm \frac{t^{2}-1}{t^{2}+1}, \pm \frac{2 t}{t^{2}+1}\right) \right\rvert\, t=1,2, \ldots, 1993\right\} +$$ + +Now consider the set $A=\{(i / q, j / q) \mid i, j=0,1, \ldots, 2 q\}$, where $q=$ $\prod_{i=1}^{1993}\left(t^{2}+1\right)$. We claim that $A$ gives a solution for the problem. Indeed, for any $P \in A$ there is a quarter of the unit circle centered at $P$ that is contained in the square $[0,2] \times[0,2]$. As explained above, there are 1993 rational points on this quarter circle, and by definition of $q$ they all belong to $A$. +Remark. Substantially the same problem was proposed by Bulgaria for IMO 71: see (SL71-2), where we give another possible construction of a set $A$. +2. It is well known that $r \leq \frac{1}{2} R$. Therefore $\frac{1}{3}(1+r)^{2} \leq \frac{1}{3}\left(1+\frac{1}{2}\right)^{2}=\frac{3}{4}$. + +It remains only to show that $p \leq \frac{1}{4}$. We note that $p$ does not exceed one half of the circumradius of $\triangle A^{\prime} B^{\prime} C^{\prime}$. However, by the theorem on the nine-point circle, this circumradius is equal to $\frac{1}{2} R$, and the conclusion follows. +Second solution. By a well-known relation we have $\cos A+\cos B+\cos C=$ $1+\frac{r}{R}(=1+r$ when $R=1)$. Next, recalling that the incenter of $\triangle A^{\prime} B^{\prime} C^{\prime}$ is at the orthocenter of $\triangle A B C$, we easily obtain $p=2 \cos A \cos B \cos C$. Cosines of angles of a triangle satisfy the identity $\cos ^{2} A+\cos ^{2} B+\cos ^{2} C+$ $2 \cos A \cos B \cos C=1$ (the proof is straightforward: see (SL81-11)). Thus + +$$ +\begin{aligned} +p+\frac{1}{3}(1+r)^{2} & =2 \cos A \cos B \cos C+\frac{1}{3}(\cos A+\cos B+\cos C)^{2} \\ +& \leq 2 \cos A \cos B \cos C+\cos ^{2} A+\cos ^{2} B+\cos ^{2} C=1 +\end{aligned} +$$ + +3. Let $O_{1}$ and $\rho$ be the center and radius of $k_{c}$. It is clear that $C, I, O_{1}$ are collinear and $C I / C O_{1}=r / \rho$. By Stewart's theorem applied to $\triangle O C O_{1}$, + +$$ +O I^{2}=\frac{r}{\rho} O O_{1}^{2}+\left(1-\frac{r}{\rho}\right) O C^{2}-C I \cdot I O_{1} . +$$ + +Since $O O_{1}=R-\rho, O C=R$ and by Euler's formula $O I^{2}=R^{2}-2 R r$, substituting these values in (1) gives $C I \cdot I O_{1}=r \rho$, or equivalently $C O_{1}$. $I O_{1}=\rho^{2}=D O_{1}^{2}$. Hence the triangles $C O_{1} D$ and $D O_{1} I$ are similar, implying $\angle D I O_{1}=90^{\circ}$. Since $C D=C E$ and the line $C O_{1}$ bisects the segment $D E$, it follows that $I$ is the midpoint of $D E$. +Second solution. Under the inversion with center $C$ and power $a b, k_{c}$ is transformed into the excircle of $\widehat{A} \widehat{B} C$ corresponding to $C$. Thus $C D=$ +$\frac{a b}{s}$, where $s$ is the common semiperimeter of $\triangle A B C$ and $\triangle \widehat{A} \widehat{B} C$, and consequently the distance from $D$ to $B C$ is $\frac{a b}{s} \sin C=\frac{2 S_{A B C}}{s}=2 r$. The statement follows immediately. +Third solution. We shall prove a stronger statement: Let $A B C D$ be a convex quadrilateral inscribed in a circle $k$, and $k^{\prime}$ the circle that is tangent to segments $B O, A O$ at $K, L$ respectively (where $O=B D \cap A C$ ), and internally to $k$ at $M$. Then $K L$ contains the incenters $I, J$ of $\triangle A B C$ and $\triangle A B D$. +Let $K^{\prime}, K^{\prime \prime}, L^{\prime}, L^{\prime \prime}, N$ denote the midpoints of arcs $B C, B D, A C, A D, A B$ that don't contain $M ; X^{\prime}, X^{\prime \prime}$ the points on $k$ defined by $X^{\prime} N=N X^{\prime \prime}=$ $K^{\prime} K^{\prime \prime}=L^{\prime} L^{\prime \prime}$ (as oriented arcs); and set $S=A K^{\prime} \cap B L^{\prime \prime}, \bar{M}=N S \cap k$, $\bar{K}=K^{\prime \prime} M \cap B O, \bar{L}=L^{\prime} M \cap A O$. +It is clear that $I=A K^{\prime} \cap B L^{\prime}, J=A K^{\prime \prime} \cap B L^{\prime \prime}$. Furthermore, $X^{\prime} \bar{M}$ contains $I$ (to see this, use the fact that for $A, B, C, D, E, F$ on $k$, lines $A D, B E, C F$ are concurrent if and only if $A B \cdot C D \cdot E F=B C \cdot D E \cdot F A$, and then express $A \bar{M} / \bar{M} B$ by applying this rule to $A M B K^{\prime} N L^{\prime \prime}$ and show that $A K^{\prime}, \bar{M} X^{\prime}, B L^{\prime}$ are concurrent). +Analogously, $X^{\prime \prime} \bar{M}$ contains $J$. Now the points $B, \bar{K}, I, S, \bar{M}$ lie on a circle $(\angle B \overline{K M}=\angle B I \bar{M}=\angle B S \bar{M})$, and points $A, \bar{L}, J, S, \bar{M}$ do so as well. Lines $I \bar{K}, J \bar{L}$ are parallel to $K^{\prime \prime} L^{\prime}$ (because $\angle \overline{M K} I=\angle \bar{M} B I=$ $\left.\angle \bar{M} K^{\prime \prime} L^{\prime}\right)$. On the other hand, the quadrilateral $A B I J$ is cyclic, and simple calculation with angles shows that $I J$ is also parallel to $K^{\prime \prime} L^{\prime}$. Hence $\bar{K}, I, J, \bar{L}$ are collinear. +![](https://cdn.mathpix.com/cropped/2024_11_18_8e985d6b9c83aa3e9d0eg-578.jpg?height=496&width=523&top_left_y=967&top_left_x=827) + +Finally, $\bar{K} \equiv K, \bar{L} \equiv L$, and $\bar{M} \equiv M$ because the homothety centered at $M$ that maps $k^{\prime}$ to $k$ sends $K$ to $K^{\prime \prime}$ and $L$ to $L^{\prime}$ (thus $M, K, K^{\prime \prime}$, as well as $M, L, L^{\prime}$, must be collinear). As is seen now, the deciphered picture yields many other interesting properties. Thus, for example, $N, S, M$ are collinear, i.e., $\angle A M S=\angle B M S$. +Fourth solution. We give an alternative proof of the more general statement in the third solution. Let $W$ be the foot of the perpendicular from $B$ to $A C$. We define $q=C W, h=B W, t=O L=O K, x=A L$, $\theta=\measuredangle W B O(\theta$ is negative if $\mathcal{B}(O, W, A), \theta=0$ if $W=O)$, and as usual, $a=B C, b=A C, c=A B$. Let $\alpha=\measuredangle K L C$ and $\beta=\measuredangle I L C$ (both angles must be acute). Our goal is to prove $\alpha=\beta$. We note that $90^{\circ}-\theta=2 \alpha$. One easily gets + +$$ +\tan \alpha=\frac{\cos \theta}{1+\sin \theta}, \quad \tan \beta=\frac{\frac{2 S_{A B C}}{a+b+c}}{\frac{b+c-a}{2}-x} +$$ + +Applying Casey's theorem to $A, B, C, k^{\prime}$, we get $A C \cdot B K+A L \cdot B C=$ $A B \cdot C L$, i.e., $b\left(\frac{h}{\cos \theta}-t\right)+x a=c(b-x)$. Using that $t=b-x-q-h \tan \theta$ we get + +$$ +x=\frac{b(b+c-q)-b h\left(\frac{1}{\cos \theta}+\tan \theta\right)}{a+b+c} . +$$ + +Plugging (2) into the second equation of (1) and using $b h=2 S_{A B C}$ and $c^{2}=b^{2}+a^{2}-2 b q$, we obtain $\tan \alpha=\tan \beta$, i.e., $\alpha=\beta$, which completes our proof. +4. Let $h$ be the altitude from $A$ and $\varphi=\angle B A D$. We have $B M=\frac{1}{2}(B D+$ $A B-A D)$ and $M D=\frac{1}{2}(B D-A B+A D)$, so + +$$ +\begin{aligned} +\frac{1}{M B}+\frac{1}{M D} & =\frac{B D}{M B \cdot M D}=\frac{4 B D}{B D^{2}-A B^{2}-A D^{2}+2 A B \cdot A D} \\ +& =\frac{4 B D}{2 A B \cdot A D(1-\cos \varphi)}=\frac{2 B D \sin \varphi}{2 S_{A B D}(1-\cos \varphi)} \\ +& =\frac{2 B D \sin \varphi}{B D \cdot h(1-\cos \varphi)}=\frac{2}{h \tan \frac{\varphi}{2}} . +\end{aligned} +$$ + +It follows that $\frac{1}{M B}+\frac{1}{M D}$ depends only on $h$ and $\varphi$. Specially, $\frac{1}{N C}+\frac{1}{N E}=$ $\frac{2}{h \tan (\varphi / 2)}$ as well. +5. For $n=1$ the game is trivially over. If $n=2$, it can end, for example, in the following way: +![](https://cdn.mathpix.com/cropped/2024_11_18_8e985d6b9c83aa3e9d0eg-579.jpg?height=144&width=534&top_left_y=1229&top_left_x=510) + +Fig. 1 +The sequence of moves shown in Fig. 2 enables us to remove three pieces placed in a $1 \times 3$ rectangle, using one more piece and one more free cell. In that way, for any $n \geq 4$ we can reduce an $(n+3) \times(n+3)$ square to an $n \times n$ square (Fig. 3). Therefore the game can end for every $n$ that is not divisible by 3 . +![](https://cdn.mathpix.com/cropped/2024_11_18_8e985d6b9c83aa3e9d0eg-579.jpg?height=123&width=595&top_left_y=1716&top_left_x=265) + +Fig. 2 +![](https://cdn.mathpix.com/cropped/2024_11_18_8e985d6b9c83aa3e9d0eg-579.jpg?height=272&width=272&top_left_y=1642&top_left_x=1050) + +Fig. 3 + +Suppose now that one can play the game on a $3 k \times 3 k$ square so that at the end only one piece remains. Denote the cells by $(i, j), i, j \in\{1, \ldots, 3 k\}$, and let $S_{0}, S_{1}, S_{2}$ denote the numbers of pieces on those squares $(i, j)$ for +which $i+j$ gives remainder $0,1,2$ respectively upon division by 3 . Initially $S_{0}=S_{1}=S_{2}=3 k^{2}$. After each move, two of $S_{0}, S_{1}, S_{2}$ diminish and one increases by one. Thus each move reverses the parity of the $S_{i}$ 's, so that $S_{0}, S_{1}, S_{2}$ are always of the same parity. But in the final position one of the $S_{i}$ 's must be equal to 1 and the other two must be 0 , which is impossible. +6. Notice that for $\alpha=\frac{1+\sqrt{5}}{2}, \alpha^{2} n=\alpha n+n$ for all $n \in \mathbb{N}$. We shall show that $f(n)=\left[\alpha n+\frac{1}{2}\right]$ (the closest integer to $\alpha n$ ) satisfies the requirements. Observe that $f$ is strictly increasing and $f(1)=2$. By the definition of $f$, $|f(n)-\alpha n| \leq \frac{1}{2}$ and $f(f(n))-f(n)-n$ is an integer. On the other hand, + +$$ +\begin{aligned} +|f(f(n))-f(n)-n| & =\left|f(f(n))-f(n)-\alpha^{2} n+\alpha n\right| \\ +& =\left|f(f(n))-\alpha f(n)+\alpha f(n)-\alpha^{2} n-f(n)+\alpha n\right| \\ +& =|(\alpha-1)(f(n)-\alpha n)+(f(f(n))-\alpha f(n))| \\ +& \leq(\alpha-1)|f(n)-\alpha n|+|f(f(n))-\alpha f(n)| \\ +& \leq \frac{1}{2}(\alpha-1)+\frac{1}{2}=\frac{1}{2} \alpha<1, +\end{aligned} +$$ + +which implies that $f(f(n))-f(n)-n=0$. +7. Multiplying by $a$ and $c$ the equation + +$$ +a x^{2}+2 b x y+c y^{2}=P^{k} n +$$ + +gives $(a x+b y)^{2}+P y^{2}=a P^{k} n$ and $(b x+c y)^{2}+P x^{2}=c P^{k} n$. +It follows immediately that $M(n)$ is finite; moreover, $(a x+b y)^{2}$ and $(b x+$ $c y)^{2}$ are divisible by $P$, and consequently $a x+b y, b x+c y$ are divisible by $P$ because $P$ is not divisible by a square greater than 1 . Thus there exist integers $X, Y$ such that $b x+c y=P X, a x+b y=-P Y$. Then $x=-b X-c Y$ and $y=a X+b Y$. Introducing these values into (1) and simplifying the expression obtained we get + +$$ +a X^{2}+2 b X Y+c Y^{2}=P^{k-1} n +$$ + +Hence $(x, y) \mapsto(X, Y)$ is a bijective correspondence between integral solutions of (1) and (2), so that $M\left(P^{k} n\right)=M\left(P^{k-1} n\right)=\cdots=M(n)$. +8. Suppose that $f(n)=1$ for some $n>0$. Then $f(n+1)=n+2, f(n+$ $2)=2 n+4, f(n+3)=n+1, f(n+4)=2 n+5, f(n+5)=n$, and so by induction $f(n+2 k)=2 n+3+k, f(n+2 k-1)=n+3-k$ for $k=1,2, \ldots, n+2$. Particularly, $n^{\prime}=3 n+3$ is the smallest value greater than $n$ for which $f\left(n^{\prime}\right)=1$. It follows that all numbers $n$ with $f(n)=1$ are given by $n=b_{i}$, where $b_{0}=1, b_{n}=3 b_{n-1}+3$. Furthermore, $b_{n}=3+3 b_{n-1}=3+3^{2}+3^{2} b_{n-2}=\cdots=3+3^{2}+\cdots+3^{n}+3^{n}=$ $=\frac{1}{2}\left(5 \cdot 3^{n}-3\right)$. +It is seen from above that if $n \leq b_{i}$, then $f(n) \leq f\left(b_{i}-1\right)=b_{i}+1$. Hence if $f(n)=1993$, then $n \geq b_{i} \geq 1992$ for some $i$. The smallest such $b_{i}$ is +$b_{7}=5466$, and $f\left(b_{i}+2 k-1\right)=b_{i}+3-k=1993$ implies $k=3476$. Thus the least integer in $S$ is $n_{1}=5466+2 \cdot 3476-1=12417$. +All the elements of $S$ are given by $n_{i}=b_{i+6}+2 k-1$, where $b_{i+6}+3-k=$ 1993, i.e., $k=b_{i+6}-1990$. Therefore $n_{i}=3 b_{i+6}-3981=\frac{1}{2}\left(5 \cdot 3^{i+7}-7971\right)$. Clearly $S$ is infinite and $\lim _{i \rightarrow \infty} \frac{n_{i+1}}{n_{i}}=3$. +9. We shall first complete the "multiplication table" for the sets $A, B, C$. It is clear that this multiplication is commutative and associative, so that we have the following relations: + +$$ +\begin{aligned} +& A C=(A B) B=B B=C \\ +& A^{2}=A A=(A B) C=B C=A \\ +& C^{2}=C C=B(B C)=B A=B +\end{aligned} +$$ + +(a) Now put 1 in $A$ and distribute the primes arbitrarily in $A, B, C$. This distribution uniquely determines the partition of $\mathbb{Q}^{+}$with the stated property. Indeed, if an arbitrary rational number + +$$ +x=p_{1}^{\alpha_{1}} \cdots p_{k}^{\alpha_{k}} q_{1}^{\beta_{1}} \cdots q_{l}^{\beta_{l}} r_{1}^{\gamma_{1}} \cdots r_{m}^{\gamma_{m}} +$$ + +is given, where $p_{i} \in A, q_{i} \in B, r_{i} \in C$ are primes, it is easy to see that $x$ belongs to $A, B$, or $C$ according as $\beta_{1}+\cdots+\beta_{l}+2 \gamma_{1}+\cdots+2 \gamma_{m}$ is congruent to 0,1 , or $2(\bmod 3)$. +(b) In every such partition, cubes all belong to $A$. In fact, $A^{3}=A^{2} A=$ $A A=A, B^{3}=B^{2} B=C B=A, C^{3}=C^{2} C=B C=A$. +(c) By (b) we have $1,8,27 \in A$. Then $2 \notin A$, and since the problem is symmetric with respect to $B, C$, we can assume $2 \in B$ and consequently $4 \in C$. Also $7 \notin A$, and also $7 \notin B$ (otherwise, $28=4 \cdot 7 \in A$ and $27 \in A$ ), so $7 \in C, 14 \in A, 28 \in B$. Further, we see that $3 \notin A$ (since otherwise $9 \in A$ and $8 \in A$ ). Put 3 in $C$. Then $5 \notin B$ (otherwise $15 \in A$ and $14 \in A$ ), so let $5 \in C$ too. Consequently $6,10 \in A$. Also $13 \notin A$, and $13 \notin C$ because $26 \notin A$, so $13 \in B$. Now it is easy to distribute the remaining primes $11,17,19,23,29,31$ : one possibility is + +$$ +\begin{aligned} +& A=\{1,6,8,10,14,19,23,27,29,31,33, \ldots\}, \\ +& C=\{3,4,5,7,18,22,24,26,30,32,34, \ldots\} \\ +& B=\{2,9,11,12,13,15,16,17,20,21,25,28,35, \ldots\} +\end{aligned} +$$ + +Remark. It can be proved that $\min \{n \in \mathbb{N} \mid n \in A, n+1 \in A\} \leq 77$. +10. (a) Let $n=p$ be a prime and let $p \mid a^{p}-1$. By Fermat's theorem $p \mid$ $a^{p-1}-1$, so that $p \mid a^{\operatorname{gcd}(p, p-1)}-1=a-1$, i.e., $a \equiv 1(\bmod p)$. Since then $a^{i} \equiv 1(\bmod p)$, we obtain $p \mid a^{p-1}+\cdots+a+1$ and hence $p^{2} \mid a^{p}-1=(a-1)\left(a^{p-1}+\cdots+a+1\right)$. +(b) Let $n=p_{1} \cdots p_{k}$ be a product of distinct primes and let $n \mid a^{n}-1$. Then from $p_{i} \mid a^{n}-1=\left(a^{\left(n / p_{i}\right)}\right)^{p_{i}}-1$ and part (a) we conclude that $p_{i}^{2} \mid a^{n}-1$. Since this is true for all indices $i$, we also have $n^{2} \mid a^{n}-1$; hence $n$ has the property $P$. +11. Due to the extended Eisenstein criterion, $f$ must have an irreducible factor of degree not less than $n-1$. Since $f$ has no integral zeros, it must be irreducible. +Second solution. The proposer's solution was as follows. Suppose that $f(x)=g(x) h(x)$, where $g, h$ are nonconstant polynomials with integer coefficients. Since $|f(0)|=3$, either $|g(0)|=1$ or $|h(0)|=1$. We may assume $|g(0)|=1$ and that $g(x)=\left(x-\alpha_{1}\right) \cdots\left(x-\alpha_{k}\right)$. Then $\left|\alpha_{1} \cdots \alpha_{k}\right|=$ 1. Since $\alpha_{i}^{n-1}\left(\alpha_{i}+5\right)=-3$, taking the product over $i=1,2, \ldots, k$ yields $\left|\left(\alpha_{1}+5\right) \cdots\left(\alpha_{k}+5\right)\right|=|g(-5)|=3^{k}$. But $f(-5)=g(-5) h(-5)=3$, so the only possibility is $\operatorname{deg} g=k=1$. This is impossible, because $f$ has no integral zeros. +Remark. Generalizing this solution, it can be shown that if $a, m, n$ are positive integers and $p1$, let $S_{N}=\{(m, n) \in S \mid m+n=N\}$. If $f(m, n)=\left(m_{1}, n_{1}\right)$, then $m_{1}+n_{1}=m+n$ with $m_{1}$ odd and $m_{1} \leq \frac{n}{2}<$ $\frac{N}{2}n_{2}>\cdots>n_{s}$. +First, suppose that $n_{i} \equiv n_{j}(\bmod n), i \neq j$. Consider the number + +$$ +B=A-a_{i} b^{n_{i}}-a_{j} b^{n_{j}}+\left(a_{i}+a_{j}\right) b^{n_{j}+k n} +$$ + +with $k$ chosen large enough so that $n_{j}+k n>n_{1}$ : this number is divisible by $b^{n}-1$ as well. But if $a_{i}+a_{j}-\frac{c_{i}}{n}$ implies + +$$ +\sum_{i=1}^{n} q_{i}>-\sum_{i=1}^{n} \frac{c_{i}}{n} \geq-1 +$$ + +which leads to $\sum_{i=1}^{n} q_{i} \geq 0$. The proof is complete. +21. Assume that $S$ is a circle with center $O$ that cuts $S_{i}$ diametrically in points $P_{i}, Q_{i}, i \in\{A, B, C\}$, and denote by $r_{i}, r$ the radii of $S_{i}$ and $S$ respectively. Since $O A$ is perpendicular to $P_{A} Q_{A}$, it follows by Pythagoras's theorem that $O A^{2}+A P_{A}^{2}=O P_{A}^{2}$, i.e., $r_{A}^{2}+O A^{2}=r^{2}$. Analogously $r_{B}^{2}+O B^{2}=r^{2}$ and $r_{C}^{2}+O C^{2}=r^{2}$. Thus if $O_{A}, O_{B}, O_{C}$ are the feet of perpendiculars from $O$ to $B C, C A, A B$ respectively, then $O_{C} A^{2}-O_{C} B^{2}=r_{B}^{2}-r_{A}^{2}$. Since the left-hand side is a monotonic function of $O_{C} \in A B$, the point $O_{C}$ is uniquely determined by the imposed conditions. The same holds for $O_{A}$ and $O_{B}$. +If $A, B, C$ are not collinear, then the positions of $O_{A}, O_{B}, O_{C}$ uniquely determine the point $O$, and therefore the circle $S$ also. On the other hand, if $A, B, C$ are collinear, all one can deduce is that $O$ lies on the lines $l_{A}, l_{B}, l_{C}$ through $O_{A}, O_{B}, O_{C}$, perpendicular to $B C, C A, A B$ respectively. By this, $l_{A}, l_{B}, l_{C}$ are parallel, so $O$ can be either anywhere on the line if these lines coincide, or +![](https://cdn.mathpix.com/cropped/2024_11_18_8e985d6b9c83aa3e9d0eg-586.jpg?height=480&width=544&top_left_y=1409&top_left_x=810) +nowhere if they don't coincide. So if there exists more than one circle $S$, $A, B, C$ lie on a line and the foot $O^{\prime}$ of the perpendicular from $O$ to the line $A B C$ is fixed. If $X, Y$ are the intersection points of $S$ and the line $A B C$, then $r^{2}=O X^{2}=O A^{2}+r_{A}^{2}$ and consequently $O^{\prime} X^{2}=O^{\prime} A^{2}+r_{A}^{2}$, which implies that $X, Y$ are fixed. +22. Let $M$ be the point inside $\angle A D B$ that satisfies $D M=D B$ and $D M \perp$ +$D B$. Then $\angle A D M=\angle A C B$ and $A D / D M=A C / C B$. It follows that the triangles $A D M, A C B$ are similar; hence $\angle C A D=\angle B A M$ (because $\angle C A B=\angle D A M$ ) and $A B / A M=A C / A D$. Consequently the triangles $C A D, B A M$ are similar and therefore $\frac{A C}{A B}=\frac{C D}{B M}=$ $\frac{C D}{\sqrt{2} B D}$. Hence $\frac{A B \cdot C D}{A C \cdot B D}=\sqrt{2}$. +![](https://cdn.mathpix.com/cropped/2024_11_18_8e985d6b9c83aa3e9d0eg-587.jpg?height=380&width=391&top_left_y=239&top_left_x=882) + +Let $C T, C U$ be the tangents at $C$ to the circles $A C D, B C D$ respectively. Then (in oriented angles) $\angle T C U=\angle T C D+\angle D C U=\angle C A D+\angle C B D=$ $90^{\circ}$, as required. +Second solution to the first part. Denote by $E, F, G$ the feet of the perpendiculars from $D$ to $B C, C A, A B$. Consider the pedal triangle $E F G$. Since $F G=A D \sin \angle A$, from the sine theorem we have $F G: G E: E F=$ $(C D \cdot A B):(B D \cdot A C):(A D \cdot B C)$. Thus $E G=F G$. On the other hand, $\angle E G F=\angle E G D+\angle D G F=\angle C B D+\angle C A D=90^{\circ}$ implies that $E F: E G=\sqrt{2}: 1$; hence the required ratio is $\sqrt{2}$. +Third solution to the first part. Under inversion centered at $C$ and with power $r^{2}=C A \cdot C B$, the triangle $D A B$ maps into a right-angled isosceles triangle $D^{*} A^{*} B^{*}$, where + +$$ +D^{*} A^{*}=\frac{A D \cdot B C}{C D}, D^{*} B^{*}=\frac{A C \cdot B D}{C D}, A^{*} B^{*}=\frac{A B \cdot C D}{C D} . +$$ + +Thus $D^{*} B^{*}: A^{*} B^{*}=\sqrt{2}$, and this is the required ratio. +23. Let the given numbers be $a_{1}, \ldots, a_{n}$. Put $s=a_{1}+\cdots+a_{n}$ and $m=$ $\operatorname{lcm}\left(a_{1}, \ldots, a_{n}\right)$ and write $m=2^{k} r$ with $k \geq 0$ and $r$ odd. Let the binary expansion of $r$ be $r=2^{k_{0}}+2^{k_{1}}+\cdots+2^{k_{t}}$, with $0=k_{0}<\cdots1$ (since the case $a<-1$ is reduced to $a>1$ by taking $a^{\prime}=-a, x_{i}^{\prime}=-x_{i}$ ). Since the left sides of the equations are nonnegative, we have $x_{i} \geq-\frac{1}{a}>-1, i=1, \ldots, 1000$. Suppose w.l.o.g. that $x_{1}=\max \left\{x_{i}\right\}$. In particular, $x_{1} \geq x_{2}, x_{3}$. If $x_{1} \geq 0$, then we deduce that $x_{1000}^{2} \geq 1 \Rightarrow x_{1000} \geq 1$; further, from this we deduce that $x_{999}>1$ etc., so either $x_{i}>1$ for all $i$ or $x_{i}<0$ for all $i$. +(i) $x_{i}>1$ for every $i$. Then $x_{1} \geq x_{2}$ implies $x_{1}^{2} \geq x_{2}^{2}$, so $x_{2} \geq x_{3}$. Thus $x_{1} \geq x_{2} \geq \cdots \geq x_{1000} \geq x_{1}$, and consequently $x_{1}=\cdots=x_{1000}$. In this case the only solution is $x_{i}=\frac{1}{2}\left(a+\sqrt{a^{2}+4}\right)$ for all $i$. +(ii) $x_{i}<0$ for every $i$. Then $x_{1} \geq x_{3}$ implies $x_{1}^{2} \leq x_{3}^{2} \Rightarrow x_{2} \leq x_{4}$. Similarly, this leads to $x_{3} \geq x_{5}$, etc. Hence $x_{1} \geq x_{3} \geq x_{5} \geq \cdots \geq x_{999} \geq x_{1}$ and $x_{2} \leq x_{4} \leq \cdots \leq x_{2}$, so we deduce that $x_{1}=x_{3}=\cdots$ and $x_{2}=x_{4}=$ $\cdots$. Therefore the system is reduced to $x_{1}^{2}=a x_{2}+1, x_{2}^{2}=a x_{1}+1$. Subtracting these equations, one obtains $\left(x_{1}-x_{2}\right)\left(x_{1}+x_{2}+a\right)=0$. There are two possibilities: +(1) If $x_{1}=x_{2}$, then $x_{1}=x_{2}=\cdots=\frac{1}{2}\left(a-\sqrt{a^{2}+4}\right)$. +(2) $x_{1}+x_{2}+a=0$ is equivalent to $x_{1}^{2}+a x_{1}+\left(a^{2}-1\right)=0$. The discriminant of the last equation is $4-3 a^{2}$. Therefore if $a>\frac{2}{\sqrt{3}}$, this case yields no solutions, while if $a \leq \frac{2}{\sqrt{3}}$, we obtain $x_{1}=$ $\frac{1}{2}\left(-a-\sqrt{4-3 a^{2}}\right), x_{2}=\frac{1}{2}\left(-a+\sqrt{4-3 a^{2}}\right)$, or vice versa. +26. Set + +$$ +\begin{aligned} +f(a, b, c, d) & =a b c+b c d+c d a+d a b-\frac{176}{27} a b c d \\ +& =a b(c+d)+c d\left(a+b-\frac{176}{27} a b\right) . +\end{aligned} +$$ + +If $a+b-\frac{176}{a} b \leq 0$, by the arithmetic-geometric inequality we have $f(a, b, c, d) \leq a b(c+d) \leq \frac{1}{27}$. +On the other hand, if $a+b-\frac{176}{a} b>0$, the value of $f$ increases if $c, d$ are replaced by $\frac{c+d}{2}, \frac{c+d}{2}$. Consider now the following fourtuplets: + +$$ +\begin{gathered} +P_{0}(a, b, c, d), P_{1}\left(a, b, \frac{c+d}{2}, \frac{c+d}{2}\right), P_{2}\left(\frac{a+b}{2}, \frac{a+b}{2}, \frac{c+d}{2}, \frac{c+d}{2}\right), \\ +P_{3}\left(\frac{1}{4}, \frac{a+b}{2}, \frac{c+d}{2}, \frac{1}{4}\right), P_{4}\left(\frac{1}{4}, \frac{1}{4}, \frac{1}{4}, \frac{1}{4}\right) +\end{gathered} +$$ + +From the above considerations we deduce that for $i=0,1,2,3$ either $f\left(P_{i}\right) \leq f\left(P_{i+1}\right)$, or directly $f\left(P_{i}\right) \leq 1 / 27$. Since $f\left(P_{4}\right)=1 / 27$, in every case we are led to + +$$ +f(a, b, c, d)=f\left(P_{0}\right) \leq \frac{1}{27} +$$ + +Equality occurs only in the cases $(0,1 / 3,1 / 3,1 / 3)$ (with permutations) and ( $1 / 4,1 / 4,1 / 4,1 / 4)$. +Remark. Lagrange multipliers also work. On the boundary of the set one of the numbers $a, b, c, d$ is 0 , and the inequality immediately follows, while for an extremum point in the interior, among $a, b, c, d$ there are at most two distinct values, in which case one easily verifies the inequality. + +### 4.35 Solutions to the Shortlisted Problems of IMO 1994 + +1. Obviously $a_{0}>a_{1}>a_{2}>\cdots$. Since $a_{k}-a_{k+1}=1-\frac{1}{a_{k}+1}$, we have $a_{n}=a_{0}+\left(a_{1}-a_{0}\right)+\cdots+\left(a_{n}-a_{n-1}\right)=1994-n+\frac{1}{a_{0}+1}+\cdots+\frac{1}{a_{n-1}+1}>$ $1994-n$. Also, for $1 \leq n \leq 998$, + +$$ +\frac{1}{a_{0}+1}+\cdots+\frac{1}{a_{n-1}+1}<\frac{n}{a_{n-1}+1}<\frac{998}{a_{997}+1}<1 +$$ + +because as above, $a_{997}>997$. Hence $\left\lfloor a_{n}\right\rfloor=1994-n$. +2. We may assume that $a_{1}>a_{2}>\cdots>a_{m}$. We claim that for $i=1, \ldots, m$, $a_{i}+a_{m+1-i} \geq n+1$. Indeed, otherwise $a_{i}+a_{m+1-i}, \ldots, a_{i}+a_{m-1}, a_{i}+a_{m}$ are $i$ different elements of $A$ greater than $a_{i}$, which is impossible. Now by adding for $i=1, \ldots, m$ we obtain $2\left(a_{1}+\cdots+a_{m}\right) \geq m(n+1)$, and the result follows. +3. The last condition implies that $f(x)=x$ has at most one solution in $(-1,0)$ and at most one solution in $(0, \infty)$. Suppose that for $u \in(-1,0)$, $f(u)=u$. Then putting $x=y=u$ in the given functional equation yields $f\left(u^{2}+2 u\right)=u^{2}+2 u$. Since $u \in(-1,0) \Rightarrow u^{2}+2 u \in(-1,0)$, we deduce that $u^{2}+2 u=u$, i.e., $u=-1$ or $u=0$, which is impossible. Similarly, if $f(v)=v$ for $v \in(0, \infty)$, we are led to the same contradiction. +However, for all $x \in S, f(x+(1+x) f(x))=x+(1+x) f(x)$, so we must have $x+(1+x) f(x)=0$. Therefore $f(x)=-\frac{x}{1+x}$ for all $x \in S$. It is directly verified that this function satisfies all the conditions. +4. Suppose that $\alpha=\beta$. The given functional equation for $x=y$ yields $f(x / 2)=x^{-\alpha} f(x)^{2} / 2$; hence the functional equation can be written as + +$$ +f(x) f(y)=\frac{1}{2} x^{\alpha} y^{-\alpha} f(y)^{2}+\frac{1}{2} y^{\alpha} x^{-\alpha} f(x)^{2} +$$ + +i.e., + +$$ +\left((x / y)^{\alpha / 2} f(y)-(y / x)^{\alpha / 2} f(x)\right)^{2}=0 +$$ + +Hence $f(x) / x^{\alpha}=f(y) / y^{\alpha}$ for all $x, y \in \mathbb{R}^{+}$, so $f(x)=\lambda x^{\alpha}$ for some $\lambda$. Substituting into the functional equation we obtain that $\lambda=2^{1-\alpha}$ or $\lambda=0$. Thus either $f(x) \equiv 2^{1-\alpha} x^{\alpha}$ or $f(x) \equiv 0$. +Now let $\alpha \neq \beta$. Interchanging $x$ with $y$ in the given equation and subtracting these equalities from each other, we get $\left(x^{\alpha}-x^{\beta}\right) f(y / 2)=\left(y^{\alpha}-\right.$ $\left.y^{\beta}\right) f(x / 2)$, so for some constant $\lambda \geq 0$ and all $x \neq 1, f(x / 2)=\lambda\left(x^{\alpha}-x^{\beta}\right)$. Substituting this into the given equation, we obtain that only $\lambda=0$ is possible, i.e., $f(x) \equiv 0$. +5. If $f^{(n)}(x)=\frac{p_{n}(x)}{q_{n}(x)}$ for some positive integer $n$ and polynomials $p_{n}, q_{n}$, then + +$$ +f^{(n+1)}(x)=f\left(\frac{p_{n}(x)}{q_{n}(x)}\right)=\frac{p_{n}(x)^{2}+q_{n}(x)^{2}}{2 p_{n}(x) q_{n}(x)} +$$ + +Note that $f^{(0)}(x)=x / 1$. Thus $f^{(n)}(x)=\frac{p_{n}(x)}{q_{n}(x)}$, where the sequence of polynomials $p_{n}, q_{n}$ is defined recursively by + +$$ +\begin{gathered} +p_{0}(x)=x, \quad q_{0}(x)=1, \text { and } \\ +p_{n+1}(x)=p_{n}(x)^{2}+q_{n}(x)^{2}, q_{n+1}(x)=2 p_{n}(x) q_{n}(x) . +\end{gathered} +$$ + +Furthermore, $p_{0}(x) \pm q_{0}(x)=x \pm 1$ and $p_{n+1}(x) \pm q_{n+1}(x)=p_{n}(x)^{2}+$ $q_{n}(x)^{2} \pm 2 p_{n}(x) q_{n}(x)=\left(p_{n}(x) \pm q_{n}(x)\right)^{2}$, so $p_{n}(x) \pm q_{n}(x)=(x \pm 1)^{2^{n}}$ for all $n$. Hence + +$$ +p_{n}(x)=\frac{(x+1)^{2^{n}}+(x-1)^{2^{n}}}{2} \quad \text { and } \quad q_{n}(x)=\frac{(x+1)^{2^{n}}-(x-1)^{2^{n}}}{2} . +$$ + +Finally, + +$$ +\begin{aligned} +\frac{f^{(n)}(x)}{f^{(n+1)}(x)} & =\frac{p_{n}(x) q_{n+1}(x)}{q_{n}(x) p_{n+1}(x)}=\frac{2 p_{n}(x)^{2}}{p_{n+1}(x)}=\frac{\left((x+1)^{2^{n}}+(x-1)^{2^{n}}\right)^{2}}{(x+1)^{2^{n+1}}+(x-1)^{2^{n+1}}} \\ +& =1+\frac{2\left(\frac{x+1}{x-1}\right)^{2^{n}}}{1+\left(\frac{x+1}{x-1}\right)^{2^{n+1}}}=1+\frac{1}{f\left(\left(\frac{x+1}{x-1}\right)^{2^{n}}\right)} +\end{aligned} +$$ + +6. Call the first and second player $M$ and $N$ respectively. $N$ can keep $A \leq 6$. Indeed, let 10 dominoes be placed as shown in the picture, and whenever $M$ marks a 1 in a cell of some domino, let $N$ mark 0 in the other cell of that domino if it is still empty. Since any $3 \times 3$ square contains at least three complete domi- +![](https://cdn.mathpix.com/cropped/2024_11_18_8e985d6b9c83aa3e9d0eg-591.jpg?height=252&width=249&top_left_y=1193&top_left_x=937) +noes, there are at least three 0 's inside. Hence $A \leq 6$. +We now show that $M$ can make $A=6$. Let him start by marking 1 in $c 3$. By symmetry, we may assume that $N$ 's response is made in row 4 or 5 . Then $M$ marks 1 in $c 2$. If $N$ puts 0 in $c 1$, then $M$ can always mark two 1's in $b \times\{1,2,3\}$ as well as three 1's in $\{a, d\} \times\{1,2,3\}$. Thus either $\{a, b, c\} \times\{1,2,3\}$ or $\{b, c, d\} \times\{1,2,3\}$ will contain six 1's. However, if $N$ does not play his second move in $c 1$, then $M$ plays there, and thus he can easily achieve to have six 1's either in $\{a, b, c\} \times\{1,2,3\}$ or $\{c, d, e\} \times\{1,2,3\}$. +7. Let $a_{1}, a_{2}, \ldots, a_{m}$ be the ages of the male citizens $(m \geq 1)$. We claim that the age of each female citizen can be expressed in the form $c_{1} a_{1}+\cdots+c_{m} a_{m}$ for some constants $c_{i} \geq 0$, and we will prove this by induction on the number $n$ of female citizens. +The claim is clear if $n=1$. Suppose it holds for $n$ and consider the case of $n+1$ female citizens. Choose any of them, say $A$ of age $x$ who knows $k$ +citizens (at least one male). By the induction hypothesis, the age of each of the other $n$ females is expressible as $c_{1} a_{1}+\cdots+c_{m} a_{m}+c_{0} x$, where $c_{i} \geq 0$ and $c_{0}+c_{1}+\cdots+c_{m}=1$. Consequently, the sum of ages of the $k$ citizens who know $A$ is $k x=b_{1} a_{1}+\cdots+b_{m} a_{m}+b_{0} x$ for some constants $b_{i} \geq 0$ with sum $k$. But $A$ knows at least one male citizen (who does not contribute to the coefficient of $x)$, so $b_{0} \leq k-1$. Hence $x=\frac{b_{1} a_{1}+\cdots+b_{m} a_{m}}{k-b_{0}}$, and the claim follows. +8. (a) Let $a, b, c, a \leq b \leq c$ be the amounts of money in dollars in Peter's first, second, and third account, respectively. If $a=0$, then we are done, so suppose that $a>0$. Let Peter make transfers of money into the first account as follows. Write $b=a q+r$ with $0 \leq r1994$ is trivial. Suppose that $n=1994$. Label the girls $G_{1}$ to $G_{1994}$, and let $G_{1}$ initially hold all the cards. At any moment give to each card the value $i, i=1, \ldots, 1994$, if $G_{i}$ holds it. Define the characteristic $C$ of a position as the sum of all these values. Initially $C=1994$. In each move, if $G_{i}$ passes cards to $G_{i-1}$ and $G_{i+1}$ (where $G_{0}=G_{1994}$ and $G_{1995}=G_{1}$ ), $C$ changes for $\pm 1994$ or does not change, so that it remains divisible by 1994. But if the game ends, the characteristic of the final position will be $C=1+2+\cdots+1994=$ $997 \cdot 1995$, which is not divisible by 1994. +(b) Whenever a card is passed from one girl to another for the first time, let the girls sign their names on it. Thereafter, if one of them passes a card to her neighbor, we shall assume that the passed card is exactly the one signed by both of them. Thus each signed card is stuck between two neighboring girls, so if $n<1994$, there are two neighbors who never exchange cards. Consequently, there is a girl $G$ who played only a finite number of times. If her neighbor plays infinitely often, then after her last move, $G$ will continue to accumulate cards indefinitely, which is impossible. Hence every girl plays finitely many times. +11. Tile the table with dominoes and numbers as shown in the picture. The second player will not lose if whenever the first player plays in a cell of a domino, he plays in the other cell of the domino, and whenever the first player plays on a number, he plays on the same number that is diagonally adjacent. +![](https://cdn.mathpix.com/cropped/2024_11_18_8e985d6b9c83aa3e9d0eg-593.jpg?height=385&width=399&top_left_y=1025&top_left_x=851) +12. Define $S_{n}$ recursively as follows: Let $S_{2}=\{(0,0),(1,1)\}$ and $S_{n+1}=$ $S_{n} \cup T_{n}$, where $T_{n}=\left\{\left(x+2^{n-1}, y+M_{n}\right) \mid(x, y) \in S_{n}\right\}$, with $M_{n}$ chosen large enough so that the entire set $T_{n}$ lies above every line passing through two points of $S_{n}$. By definition, $S_{n}$ has exactly $2^{n-1}$ points and contains no three collinear points. We claim that no $2 n$ points of this set are the vertices of a convex $2 n$-gon. +Consider an arbitrary convex polygon $\mathcal{P}$ with vertices in $S_{n}$. Join by a diagonal $d$ the two vertices of $\mathcal{P}$ having the smallest and greatest $x$ coordinates. This diagonal divides $\mathcal{P}$ into two convex polygons $\mathcal{P}_{1}, \mathcal{P}_{2}$, the former lying above $d$. We shall show by induction that both $\mathcal{P}_{1}, \mathcal{P}_{2}$ have at most $n$ vertices. Assume to the contrary that $\mathcal{P}_{1}$ has at least $n+1$ vertices $A_{1}\left(x_{1}, y_{1}\right), \ldots, A_{n+1}\left(x_{n+1}, y_{n+1}\right)$ in $S_{n}$, with $x_{1}<\cdots\cdots>\frac{y_{n+1}-y_{n}}{x_{n+1}-x_{n}}$. By the induction hypothesis, not more than $n-1$ of these vertices belong to $S_{n-1}$ or $T_{n-1}$, so let $A_{k-1}, A_{k} \in S_{n-1}$, $A_{k+1} \in T_{n-1}$. But by the construction of $T_{n-1}, \frac{y_{k+1}-y_{k}}{x_{k+1}-x_{k}}>\frac{y_{k}-y_{k-1}}{x_{k}-x_{k-1}}$, which +gives a contradiction. Similarly, $\mathcal{P}_{2}$ has no more than $n$ vertices, and therefore $\mathcal{P}$ itself has at most $2 n-2$ vertices. +13. Extend $A D$ and $B C$ to meet at $P$, and let $Q$ be the foot of the perpendicular from $P$ to $A B$. Denote by $O$ the center of $\Gamma$. Since $\triangle P A Q \sim \triangle O A D$ and $\triangle P B Q \sim \triangle O B C$, we obtain $\frac{A Q}{A D}=\frac{P Q}{O D}=\frac{P Q}{O C}=\frac{B Q}{B C}$. Therefore $\frac{A Q}{Q B} \cdot \frac{B C}{C P} \cdot \frac{P D}{D A}=1$, so by the converse Ceva theorem, $A C, B D$, and $P Q$ are concurrent. It follows that $Q \equiv F$. Finally, since the points $O, C, P, D, F$ are concyclic, we have $\angle D F P=\angle D O P=\angle P O C=\angle P F C$. +14. Although it does not seem to have been noticed at the jury, the statement of the problem is false. For $A(0,0), B(0,4), C(1,4), D(7,0)$, we have $M(4,2), P(2,1), Q(2,3)$ and $N(9 / 2,1 / 2) \notin \triangle A B M$. +The official solution, if it can be called so, actually shows that $N$ lies inside $A B C D$ and goes as follows: The case $A D=B C$ is trivial, so let $A D>B C$. Let $L$ be the midpoint of $A B$. Complete the parallelograms $A D M X$ and $B C M Y$. Now $N=D X \cap C Y$, so let $C Y$ and $D X$ intersect $A B$ at $K$ and $H$ respectively. From $L X=L Y$ and + +$$ +\frac{H L}{L X}=\frac{H A}{A D}<\frac{L A}{A D}<\frac{K B}{A D}<\frac{K B}{B C}=\frac{K L}{L Y} +$$ + +we get $H Ln \geq 2$. Since $m^{3}+1 \equiv 1$ and +$m n-1 \equiv-1(\bmod n)$, we deduce $\frac{n^{3}+1}{m n-1}=k n-1$ for some integer $k>0$. On the other hand, $k n-1<\frac{n^{3}+1}{n^{2}-1}=n+\frac{1}{n-1} \leq 2 n-1$ gives that $k=1$, and therefore $n^{3}+1=(m n-1)(n-1)$. This yields $m=\frac{n^{2}+1}{n-1}=n+1+\frac{2}{n-1} \in \mathbb{N}$, so $n \in\{2,3\}$ and $m=5$. The solutions with $mz_{n}$ when $x_{n-1}$ is even and $2 y_{n}>z_{n}>y_{n}$ when $x_{n-1}$ is odd. In fact, $n=1$ is the trivial case, while if it holds for $n \geq 1$, then $y_{n+1}=2 y_{n}>z_{n}=z_{n+1}$ if $x_{n}$ is even, and $2 y_{n+1}=2 y_{n}>y_{n}+z_{n}=z_{n+1}$ if $x_{n}$ is odd (since then $x_{n-1}$ is even). +If $x_{1}=0$, then $x_{0}=3$ is good. Suppose $x_{n}=0$ for some $n \geq 2$. Then $x_{n-1}$ is odd and $x_{n-2}$ is even, so that $y_{n-1}>z_{n-1}$. We claim that a pair $\left(y_{n-1}, z_{n-1}\right)$, where $2^{k}=y_{n-1}>z_{n-1}>0$ and $z_{n-1} \equiv 1$ $(\bmod 4)$, uniquely determines $x_{0}=f\left(y_{n-1}, z_{n-1}\right)$. We see that $x_{n-1}=$ $\frac{1}{2} y_{n-1}+z_{n-1}$, and define $\left(x_{k}, y_{k}, z_{k}\right)$ backwards as follows, until we get $\left(y_{k}, z_{k}\right)=(4,1)$. If $y_{k}>z_{k}$, then $x_{k-1}$ must have been even, so we define $\left(x_{k-1}, y_{k-1}, z_{k-1}\right)=\left(2 x_{k}, y_{k} / 2, z_{k}\right)$; otherwise $x_{k-1}$ must have been odd, so we put $\left(x_{k-1}, y_{k-1}, z_{k-1}\right)=\left(x_{k}-y_{k} / 2+z_{k}, y_{k}, z_{k}-y_{k}\right)$. We eventually arrive at $\left(y_{0}, z_{0}\right)=(4,1)$ and a good integer $x_{0}=f\left(y_{n-1}, z_{n-1}\right)$, as claimed. Thus for example $\left(y_{n-1}, z_{n-1}\right)=(64,61)$ implies $x_{n-1}=93$, $\left(x_{n-2}, y_{n-2}, z_{n-2}\right)=(186,32,61)$ etc., and $x_{0}=1953$, while in the case of $\left(y_{n-1}, z_{n-1}\right)=(128,1)$ we get $x_{0}=2080$. +Note that $y^{\prime}>y \Rightarrow f\left(y^{\prime}, z^{\prime}\right)>f(y, z)$ and $z^{\prime}>z \Rightarrow f\left(y, z^{\prime}\right)>f(y, z)$. Therefore there are no $y, z$ for which $19531$, and let $x_{j} \equiv u_{j}(\bmod p)$ where $0 \leq u_{j} \leq p-1$ (particularly $u_{i} \equiv 0$ ). Then $u_{j+1} \equiv u_{j} u_{j-1}+$ $1(\bmod p)$. The number of possible pairs $\left(u_{j}, u_{j+1}\right)$ is finite, so $u_{j}$ is eventually periodic. We claim that for some $d_{p}>0, u_{i+d_{p}}=0$. Indeed, suppose the contrary and let $\left(u_{m}, u_{m+1}, \ldots, u_{m+d-1}\right)$ be the first period for $m \geq i$. Then $m \neq i$. By the assumption $u_{m-1} \not \equiv$ $u_{m+d-1}$, but $u_{m-1} u_{m} \equiv u_{m+1}-1 \equiv u_{m+d+1}-1 \equiv u_{m+d-1} u_{m+d} \equiv$ $u_{m+d-1} u_{m}(\bmod p)$, which is impossible if $p \nmid u_{m}$. Hence there is a $d_{p}$ with $u_{i}=u_{i+d_{p}}=0$ and moreover $u_{i+1}=u_{i+d_{p}+1}=1$, so the sequence $u_{j}$ is periodic with period $d_{p}$ starting from $u_{i}$. Let $m$ be the least common multiple of all $d_{p}$ 's, where $p$ goes through all prime divisors of $x_{i}$. Then the same primes divide every $x_{i+k m}, k=1,2, \ldots$, so for large enough $k$ and $j=i+k m, x_{i}^{i} \mid x_{j}^{j}$. +(b) If $i=1$, we cannot deduce that $x_{i+1} \equiv 1(\bmod p)$. The following example shows that the statement from (a) need not be true in this case. Take $x_{1}=22$ and $x_{2}=9$. Then $x_{n}$ is even if and only if $n \equiv 1(\bmod$ 3 ), but modulo 11 the sequence $\left\{x_{n}\right\}$ is $0,9,1,10,0,1,1,2,3,7,0, \ldots$, so $11 \mid x_{n}(n>1)$ if and only if $n \equiv 5(\bmod 6)$. Thus for no $n>1$ can we have $22 \mid x_{n}$. +24. A multiple of 10 does not divide any wobbly number. Also, if $25 \mid n$, then every multiple of $n$ ends with $25,50,75$, or 00 ; hence it is not wobbly. We now show that every other number $n$ divides some wobbly number. +(i) Let $n$ be odd and not divisible by 5 . For any $k \geq 1$ there exists $l$ such that $\left(10^{k}-1\right) n$ divides $10^{l}-1$, and thus also divides $10^{k l}-1$. Consequently, $v_{k}=\frac{10^{k l}-1}{10^{k}-1}$ is divisible by $n$, and it is wobbly when $k=2$ (indeed, $v_{2}=101 \ldots 01$ ). +If $n$ is divisible by 5 , one can simply take $5 v_{2}$ instead. +(ii) Let $n$ be a power of 2 . We prove by induction on $m$ that $2^{2 m+1}$ has a wobbly multiple $w_{m}$ with exactly $m$ nonzero digits. For $m=1$, take $w_{1}=8$. Suppose that for some $m \geq 1$ there is a wobbly $w_{m}=$ $2^{2 m+1} d_{m}$. Then the numbers $a \cdot 10^{2 m}+w_{m}$ are wobbly and divisible by $2^{2 m+1}$ when $a \in\{2,4,6,8\}$. Moreover, one of these numbers is divisible by $2^{2 m+3}$. Indeed, it suffices to choose $a$ such that $\frac{a}{2}+d_{m}$ is divisible by 4 . This proves the induction step. +(iii) Let $n=2^{m} r$, where $m \geq 1$ and $r$ is odd, $5 \nmid r$. Then $v_{2 m} w_{m}$ is wobbly and divisible by both $2^{m}$ and $r$ (using notation from (i), $r \mid v_{2 m}$ ). + +### 4.36 Solutions to the Shortlisted Problems of IMO 1995 + +1. Let $x=\frac{1}{a}, y=\frac{1}{b}, z=\frac{1}{c}$. Then $x y z=1$ and + +$$ +S=\frac{1}{a^{3}(b+c)}+\frac{1}{b^{3}(c+a)}+\frac{1}{c^{3}(a+b)}=\frac{x^{2}}{y+z}+\frac{y^{2}}{z+x}+\frac{z^{2}}{x+y} +$$ + +We must prove that $S \geq \frac{3}{2}$. From the Cauchy-Schwarz inequality, + +$$ +[(y+z)+(z+x)+(x+y)] \cdot S \geq(x+y+z)^{2} \Rightarrow S \geq \frac{x+y+z}{2} +$$ + +It follows from the A-G mean inequality that $\frac{x+y+z}{2} \geq \frac{3}{2} \sqrt[3]{x y z}=\frac{3}{2}$; hence the proof is complete. Equality holds if and only if $x=y=z=1$, i.e., $a=b=c=1$. +Remark. After reducing the problem to $\frac{x^{2}}{y+z}+\frac{y^{2}}{z+x}+\frac{z^{2}}{x+y} \geq \frac{3}{2}$, we can solve the problem using Jensen's inequality applied to the function $g(u, v)=$ $u^{2} / v$. The problem can also be solved using Muirhead's inequality. +2. We may assume $c \geq 0$ (otherwise, we may simply put $-y_{i}$ in the place of $\left.y_{i}\right)$. Also, we may assume $a \geq b$. If $b \geq c$, it is enough to take $n=a+b-c$, $x_{1}=\cdots=x_{a}=1, y_{1}=\cdots=y_{c}=y_{a+1}=\cdots=y_{a+b-c}=1$, and the other $x_{i}$ 's and $y_{i}$ 's equal to 0 , so we need only consider the case $a>c>b$. We proceed to prove the statement of the problem by induction on $a+b$. The case $a+b=1$ is trivial. Assume that the statement is true when $a+b \leq$ $N$, and let $a+b=N+1$. The triple $(a+b-2 c, b, c-b)$ satisfies the condition (since $(a+b-2 c) b-(c-b)^{2}=a b-c^{2}$ ), so by the induction hypothesis there are $n$-tuples $\left(x_{i}\right)_{i=1}^{n}$ and $\left(y_{i}\right)_{i=1}^{n}$ with the wanted property. It is easy to verify that $\left(x_{i}+y_{i}\right)_{i=1}^{n}$ and $\left(y_{i}\right)_{i=1}^{n}$ give a solution for $(a, b, c)$. +3. Write $A_{i}=\frac{a_{i}^{2}+a_{i+1}^{2}-a_{i+2}^{2}}{a_{i}+a_{i+1}-a_{i+2}}=a_{i}+a_{i+1}+a_{i+2}-\frac{2 a_{i} a_{i+1}}{a_{i}+a_{i+1}-a_{i+2}}$. Since $2 a_{i} a_{i+1} \geq$ $4\left(a_{i}+a_{i+1}-2\right)$ (which is equivalent to $\left.\left(a_{i}-2\right)\left(a_{i+1}-2\right) \geq 0\right)$, it follows that $A_{i} \leq a_{i}+a_{i+1}+a_{i+2}-4\left(1+\frac{a_{i+2}-2}{a_{i}+a_{i+1}-a_{i+2}}\right) \leq a_{i}+a_{i+1}+a_{i+2}-$ $4\left(1+\frac{a_{i+2}-2}{4}\right)$, because $1 \leq a_{i}+a_{i+1}-a_{i+2} \leq 4$. Therefore $A_{i} \leq a_{i}+$ $a_{i+1}-2$, so $\sum_{i=1}^{n} A_{i} \leq 2 s-2 n$ as required. +4. The second equation is equivalent to $\frac{a^{2}}{y z}+\frac{b^{2}}{z x}+\frac{c^{2}}{x y}+\frac{a b c}{x y z}=4$. Let $x_{1}=$ $\frac{a}{\sqrt{y z}}, y_{1}=\frac{b}{\sqrt{z x}}, z_{1}=\frac{c}{\sqrt{x y}}$. Then $x_{1}^{2}+y_{1}^{2}+z_{1}^{2}+x_{1} y_{1} z_{1}=4$, where $00$ is clearly impossible. On the other hand, if $u v w \leq 0$, then two of $u, v, w$ are nonnegative, say $u, v \geq 0$. Taking into account $w=-u-v$, the above equality reduces to $2\left[(a+c-2 v) u^{2}+(b+c-2 u) v^{2}+2 c u v\right]=0$, so $u=v=0$. + +Third solution. The fact that we are given two equations and three variables suggests that this is essentially a problem on inequalities. Setting $f(x, y, z)=4 x y z-a^{2} x-b^{2} y-c^{2} z$, we should show that $\max f(x, y, z)=$ $a b c$, for $0\frac{n-1}{4}$. Then $f(1 / x)=f\left(x+1 / x^{2}\right)-f(x)<1 / 4$, so $f(1 / x)>-1 / 2$. On the other hand, this implies $\left(\frac{n-1}{4}\right)^{2}0$. Since $\sum_{i=1}^{n-1} y_{i}=Y$ and $\sum_{i=1}^{n-1}(n-i)=\frac{n(n-1)}{2}$, we have $\sum z_{i}=0$. Note that $Y<\sum_{j=2}^{n}(j-1) x_{n}=\frac{n(n-1)}{2} x_{n}$, and consequently $z_{n-1}=$ $\frac{n(n-1)}{2} x_{n}-Y>0$. Furthermore, we have + +$$ +\frac{z_{i+1}}{n-i-1}-\frac{z_{i}}{n-i}=\frac{n(n-1)}{2}\left(\frac{y_{i+1}}{n-i-1}-\frac{y_{i}}{n-i}\right)>0 +$$ + +which means that $\frac{z_{1}}{n-1}<\frac{z_{2}}{n-2}<\cdots<\frac{z_{n-1}}{1}$. Therefore there is a $k$ for which $z_{1}, \ldots, z_{k} \leq 0$ and $z_{k+1}, \ldots, z_{n-1}>0$. But then $z_{i}\left(x_{i}-x_{k}\right) \geq 0$, i.e., $x_{i} z_{i} \geq x_{k} z_{i}$ for all $i$, so $\sum_{i=1}^{n-1} x_{i} z_{i}>\sum_{i=1}^{n-1} x_{k} z_{i}=0$ as required. + +Second solution. Set $X=\sum_{j=1}^{n-1}(n-j) x_{j}$ and $Y=\sum_{j=2}^{n}(j-1) x_{j}$. Since $4 X Y=(X+Y)^{2}-(X-Y)^{2}$, the RHS of the inequality becomes + +$$ +X Y=\frac{1}{4}\left[(n-1)^{2}\left(\sum_{i=1}^{n} x_{i}\right)^{2}-\left(\sum_{i=1}^{n}(2 i-1-n) x_{i}\right)^{2}\right] +$$ + +The LHS equals $\frac{1}{4}\left((n-1)^{2}\left(\sum_{i=1}^{n} x_{i}\right)^{2}-(n-1) \sum_{i(n-1) \sum_{i0$ (so, $x_{j}-x_{i}=d_{i}+d_{i+1}+\cdots+d_{j-1}$ ) and expanding the obtained expressions, we reduce this inequality to $\sum_{k} k^{2}(n-k)^{2} d_{k}^{2}+2 \sum_{k\sum_{k}(n-1) k(n-k) d_{k}^{2}+$ $2 \sum_{k3$. Then either $a_{k}^{2} \equiv-7\left(\bmod 2^{k+1}\right)$ or $a_{k}^{2} \equiv 2^{k}-7$ $\left(\bmod 2^{k+1}\right)$. In the former case, take $a_{k+1}=a_{k}$. In the latter case, set $a_{k+1}=a_{k}+2^{k-1}$. Then $a_{k+1}^{2}=a_{k}^{2}+2^{k} a_{k}+2^{2 k-2} \equiv a_{k}^{2}+2^{k} \equiv-7(\bmod$ $2^{k+1}$ ) because $a_{k}$ is odd. +16. If $A$ is odd, then every number in $M_{1}$ is of the form $x(x+A)+B \equiv B$ $(\bmod 2)$, while numbers in $M_{2}$ are congruent to $C$ modulo 2 . Thus it is enough to take $C \equiv B+1(\bmod 2)$. + +If $A$ is even, then all numbers in $M_{1}$ have the form $\left(X+\frac{A}{2}\right)^{2}+B-\frac{A^{2}}{4}$ and are congruent to $B-\frac{A^{2}}{4}$ or $B-\frac{A^{2}}{4}+1$ modulo 4 , while numbers in $M_{2}$ are congruent to $C$ modulo 4 . So one can choose any $C \equiv B-\frac{A^{2}}{4}+2$ $(\bmod 4)$. +17. For $n=4$, the vertices of a unit square $A_{1} A_{2} A_{3} A_{4}$ and $p_{1}=p_{2}=p_{3}=$ $p_{4}=\frac{1}{6}$ satisfy the conditions. We claim that there are no solutions for $n=5$ (and thus for any $n \geq 5$ ). +Suppose to the contrary that points $A_{i}$ and $p_{i}, i=1, \ldots, 5$, satisfy the conditions. Denote the area of $\triangle A_{i} A_{j} A_{k}$ by $S_{i j k}=p_{i}+p_{j}+p_{k}, 1 \leq i<$ $j2$. Therefore $c=\frac{b^{2}+z}{z^{2} b-1}<1$, a contradiction. +The only solutions for $(x, y)$ are $(4,2),(4,6),(5,2),(5,3)$. +19. For each two people let $n$ be the number of people exchanging greetings with both of them. To determine $n$ in terms of $k$, we shall count in two ways the number of triples $(A, B, C)$ of people such that $A$ exchanged greetings with both $B$ and $C$, but $B$ and $C$ mutually did not. +There are $12 k$ possibilities for $A$, and for each $A$ there are $(3 k+6)$ possibilities for $B$. Since there are $n$ people who exchanged greetings with both +$A$ and $B$, there are $3 k+5-n$ who did so with $A$ but not with $B$. Thus the number of triples $(A, B, C)$ is $12 k(3 k+6)(3 k+5-n)$. On the other hand, there are $12 k$ possible choices of $B$, and $12 k-1-(3 k+6)=9 k-7$ possible choices of $C$; for every $B, C, A$ can be chosen in $n$ ways, so the number of considered triples equals $12 k n(9 k-7)$. +Hence $(3 k+6)(3 k+5-n)=n(9 k-7)$, i.e., $n=\frac{3(k+2)(3 k+5)}{12 k-1}$. This gives us that $\frac{4 n}{3}=\frac{12 k^{2}+44 k+40}{12 k-1}=k+4-\frac{3 k-44}{12 k-1}$ is an integer too. It is directly verified that only $k=3$ gives an integer value for $n$, namely $n=6$. +Remark. The solution is complete under the assumption that such a $k$ exists. We give an example of such a party with 36 persons, $k=3$. Let the people sit in a $6 \times 6$ array $\left[P_{i j}\right]_{i, j=1}^{6}$, and suppose that two persons $P_{i j}, P_{k l}$ exchanged greetings if and only if $i=k$ or $j=l$ or $i-j \equiv k-l$ (mod 6). Thus each person exchanged greetings with exactly 15 others, and it is easily verified that this party satisfies the conditions. +20. We shall consider the set $M=\{0,1, \ldots, 2 p-1\}$ instead. Let $M_{1}=$ $\{0,1, \ldots, p-1\}$ and $M_{2}=\{p, p+1, \ldots, 2 p-1\}$. We shall denote by $|A|$ and $\sigma(A)$ the number of elements and the sum of elements of the set $A$; also, let $C_{p}$ be the family of all $p$-element subsets of $M$. Define the mapping $T: C_{p} \rightarrow C_{p}$ as $T(A)=\left\{x+1 \mid x \in A \cap M_{1}\right\} \cup\left\{A \cap M_{2}\right\}$, the addition being modulo $p$. There are exactly two fixed points of $T$ : these are $M_{1}$ and $M_{2}$. Now if $A$ is any subset from $C_{p}$ distinct from $M_{1}, M_{2}$, and $k=\left|A \cap M_{1}\right|$ with $1 \leq k \leq p-1$, then for $i=0,1, \ldots, p-1$, $\sigma\left(T^{i}(A)\right)=\sigma(A)+i k(\bmod p)$. Hence subsets $A, T(A), \ldots, T^{p-1}(A)$ are distinct, and exactly one of them has sum of elements divisible by $p$. Since $\sigma\left(M_{1}\right), \sigma\left(M_{2}\right)$ are divisible by $p$ and $C_{p} \backslash\left\{M_{1}, M_{2}\right\}$ decomposes into families of the form $\left\{A, T(A), \ldots, T^{p-1}(A)\right\}$, we conclude that the required number is $\frac{1}{p}\left(\left|C_{p}\right|-2\right)+2=\frac{1}{p}\left(\binom{2 p}{p}-2\right)+2$. +Second solution. Let $C_{k}$ be the family of all $k$-element subsets of $\{1,2, \ldots, 2 p\}$. Denote by $M_{k}(k=1,2, \ldots, p)$ the family of $p$-element multisets with $k$ distinct elements from $\{1,2, \ldots, 2 p\}$, exactly one of which appears more than once, that have sum of elements divisible by $p$. It is clear that every subset from $C_{k}, k0$, so $f(x)=$ $x p(x) / q(x)$ is a positive integer too. Let $\left\{p_{0}, p_{1}, p_{2}, \ldots\right\}$ be all prime numbers in increasing order. Since it easily follows by induction that all $x_{n}$ 's are square-free, we can assign to each of them a unique code according to which primes divide it: if $p_{m}$ is the largest prime dividing $x_{n}$, the code corresponding to $x_{n}$ will be $\ldots 0 s_{m} s_{m-1} \ldots s_{0}$, with $s_{i}=1$ if $p_{i} \mid x_{n}$ and $s_{i}=0$ otherwise. Let us investigate how $f$ acts on these codes. If the code of $x_{n}$ ends with 0 , then $x_{n}$ is odd, so the code of $f\left(x_{n}\right)=x_{n+1}$ is obtained from that of $x_{n}$ by replacing $s_{0}=0$ by $s_{0}=1$. Furthermore, if the code of +$x_{n}$ ends with $011 \ldots 1$, then the code of $x_{n+1}$ ends with $100 \ldots 0$ instead. Thus if we consider the codes as binary numbers, $f$ acts on them as an addition of 1 . Hence the code of $x_{n}$ is the binary representation of $n$ and thus $x_{n}$ uniquely determines $n$. +Specifically, if $x_{n}=1995=3 \cdot 5 \cdot 7 \cdot 19$, then its code is 10001110 and corresponds to $n=142$. +26. For $n=1$ the result is trivial, since $x_{1}=1$. Suppose now that $n \geq 2$ and let $f_{n}(x)=x^{n}-\sum_{i=0}^{n-1} x^{i}$. Note that $x_{n}$ is the unique positive real root of $f_{n}$, because $\frac{f_{n}(x)}{x^{n-1}}=x-1-\frac{1}{x}-\cdots-\frac{1}{x^{n-1}}$ is strictly increasing on $\mathbb{R}^{+}$. Consider $g_{n}(x)=(x-1) f_{n}(x)=(x-2) x^{n}+1$. Obviously $g_{n}(x)$ has no positive roots other than 1 and $x_{n}>1$. Observe that $\left(1-\frac{1}{2^{n}}\right)^{n}>$ $1-\frac{n}{2^{n}} \geq \frac{1}{2}$ for $n \geq 2$ (by Bernoulli's inequality). Since then + +$$ +g_{n}\left(2-\frac{1}{2^{n}}\right)=-\frac{1}{2^{n}}\left(2-\frac{1}{2^{n}}\right)^{n}+1=1-\left(1-\frac{1}{2^{n+1}}\right)^{n}>0 +$$ + +and + +$$ +g_{n}\left(2-\frac{1}{2^{n-1}}\right)=-\frac{1}{2^{n-1}}\left(2-\frac{1}{2^{n-1}}\right)^{n}+1=1-2\left(1-\frac{1}{2^{n}}\right)^{n}<0 +$$ + +we conclude that $x_{n}$ is between $2-\frac{1}{2^{n-1}}$ and $2-\frac{1}{2^{n}}$, as required. +Remark. Moreover, $\lim _{n \rightarrow \infty} 2^{n}\left(2-x_{n}\right)=1$. +27. Computing the first few values of $f(n)$, we observe the following pattern: + +$$ +\begin{aligned} +f(4 k) & =k, k \geq 3, & f(8) & =3 ; \\ +f(4 k+1) & =1, k \geq 4, & f(5) & =f(13)=2 ; \\ +f(4 k+2) & =k-3, k \geq 7, & f(2) & =1, f(6)=f(10)=2, \\ +& & f(14) & =f(18)=3, f(26)=4 ; \\ +f(4 k+3) & =2 . & & +\end{aligned} +$$ + +We shall prove these statements simultaneously by induction on $n$, having verified them for $k \leq 7$. +(i) Let $n=4 k$. Since $f(3)=f(7)=\cdots=f(4 k-1)=2$, we have $f(4 k) \geq k$. But $f(n) \leq \max _{m95$ for all $n$. If to the contrary $f(n) \leq 95$, we have $f(m)=n+f(m+95-f(n))$, so by induction $f(m)=k n+f(m+k(95-f(n))) \geq k n$ for all $k$, which is impossible. Now for $m>95$ we have $f(m+f(n)-95)=n+f(m)$, and again by induction $f(m+k(f(n)-95))=k n+f(m)$ for all $m, n, k$. It follows that with $n$ fixed, + +$$ +(\forall m) \lim _{k \rightarrow \infty} \frac{f(m+k(f(n)-95))}{m+k(f(n)-95)}=\frac{n}{f(n)-95} +$$ + +hence + +$$ +\lim _{s \rightarrow \infty} \frac{f(s)}{s}=\frac{n}{f(n)-95} +$$ + +Hence $\frac{n}{f(n)-95}$ does not depend on $n$, i.e., $f(n) \equiv c n+95$ for some constant $c$. It is easily checked that only $c=1$ is possible. + +### 4.37 Solutions to the Shortlisted Problems of IMO 1996 + +1. We have $a^{5}+b^{5}-a^{2} b^{2}(a+b)=\left(a^{3}-b^{3}\right)\left(a^{2}-b^{2}\right) \geq 0$, i.e. $a^{5}+b^{5} \geq$ $a^{2} b^{2}(a+b)$. Hence + +$$ +\frac{a b}{a^{5}+b^{5}+a b} \leq \frac{a b}{a^{2} b^{2}(a+b)+a b}=\frac{a b c^{2}}{a^{2} b^{2} c^{2}(a+b)+a b c^{2}}=\frac{c}{a+b+c} . +$$ + +Now, the left side of the inequality to be proved does not exceed $\frac{c}{a+b+c}+$ $\frac{a}{a+b+c}+\frac{b}{a+b+c}=1$. Equality holds if and only if $a=b=c$. +2. Clearly $a_{1}>0$, and if $p \neq a_{1}$, we must have $a_{n}<0,\left|a_{n}\right|>\left|a_{1}\right|$, and $p=-a_{n}$. But then for sufficiently large odd $k,-a_{n}^{k}=\left|a_{n}\right|^{k}>(n-1)\left|a_{1}\right|^{k}$, so that $a_{1}^{k}+\cdots+a_{n}^{k} \leq(n-1)\left|a_{1}\right|^{k}-\left|a_{n}\right|^{k}<0$, a contradiction. Hence $p=a_{1}$. +Now let $x>a_{1}$. From $a_{1}+\cdots+a_{n} \geq 0$ we deduce $\sum_{j=2}^{n}\left(x-a_{j}\right) \leq$ $(n-1)\left(x+\frac{a_{1}}{n-1}\right)$, so by the AM-GM inequality, + +$$ +\left(x-a_{2}\right) \cdots\left(x-a_{n}\right) \leq\left(x+\frac{a_{1}}{n-1}\right)^{n-1} \leq x^{n-1}+x^{n-2} a_{1}+\cdots+a_{1}^{n-1} +$$ + +The last inequality holds because $\binom{n-1}{r} \leq(n-1)^{r}$ for all $r \geq 0$. Multiplying (1) by $\left(x-a_{1}\right)$ yields the desired inequality. +3. Since $a_{1}>2$, it can be written as $a_{1}=b+b^{-1}$ for some $b>0$. Furthermore, $a_{1}^{2}-2=b^{2}+b^{-2}$ and hence $a_{2}=\left(b^{2}+b^{-2}\right)\left(b+b^{-1}\right)$. We prove that + +$$ +a_{n}=\left(b+b^{-1}\right)\left(b^{2}+b^{-2}\right)\left(b^{4}+b^{-4}\right) \cdots\left(b^{2^{n-1}}+b^{-2^{n-1}}\right) +$$ + +by induction. Indeed, $\frac{a_{n+1}}{a_{n}}=\left(\frac{a_{n}}{a_{n-1}}\right)^{2}-2=\left(b^{2^{n-1}}+b^{-2^{n-1}}\right)^{2}-2=$ $b^{2^{n}}+b^{-2^{n}}$. +Now we have + +$$ +\begin{aligned} +\sum_{i=1}^{n} \frac{1}{a_{i}}= & 1+\frac{b}{b^{2}+1}+\frac{b^{3}}{\left(b^{2}+1\right)\left(b^{4}+1\right)}+\cdots \\ +& \cdots+\frac{b^{2^{n}-1}}{\left(b^{2}+1\right)\left(b^{4}+1\right) \ldots\left(b^{2}+1\right)} +\end{aligned} +$$ + +Note that $\frac{1}{2}\left(a+2-\sqrt{a^{2}-4}\right)=1+\frac{1}{b}$; hence we must prove that the right side in (1) is less than $\frac{1}{b}$. This follows from the fact that + +$$ +\begin{aligned} +& \frac{b^{2^{k}}}{\left(b^{2}+1\right)\left(b^{4}+1\right) \cdots\left(b^{2^{k}}+1\right)} \\ +& \quad=\frac{1}{\left(b^{2}+1\right)\left(b^{4}+1\right) \cdots\left(b^{2^{k-1}}+1\right)}-\frac{1}{\left(b^{2}+1\right)\left(b^{4}+1\right) \cdots\left(b^{2^{k}}+1\right)} +\end{aligned} +$$ + +hence the right side in (1) equals $\frac{1}{b}\left(1-\frac{1}{\left(b^{2}+1\right)\left(b^{4}+1\right) \ldots\left(b^{2^{n}}+1\right)}\right)$, and this is clearly less than $1 / b$. +4. Consider the function + +$$ +f(x)=\frac{a_{1}}{x}+\frac{a_{2}}{x^{2}}+\cdots+\frac{a_{n}}{x^{n}} . +$$ + +Since $f$ is strictly decreasing from $+\infty$ to 0 on the interval $(0,+\infty)$, there exists exactly one $R>0$ for which $f(R)=1$. This $R$ is also the only positive real root of the given polynomial. +Since $\ln x$ is a concave function on $(0,+\infty)$, Jensen's inequality gives us + +$$ +\sum_{j=1}^{n} \frac{a_{j}}{A}\left(\ln \frac{A}{R^{j}}\right) \leq \ln \left(\sum_{j=1}^{n} \frac{a_{j}}{A} \cdot \frac{A}{R^{j}}\right)=\ln f(R)=0 . +$$ + +Therefore $\sum_{j=1}^{n} a_{j}(\ln A-j \ln R) \leq 0$, which is equivalent to $A \ln A \leq$ $B \ln R$, i.e., $A^{A} \leq R^{B}$. +5. Considering the polynomials $\pm P( \pm x)$ we may assume w.l.o.g. that $a, b \geq$ 0 . We have four cases: +(1) $c \geq 0, d \geq 0$. Then $|a|+|b|+|c|+|d|=a+b+c+d=P(1) \leq 1$. +(2) $c \geq 0, d<0$. Then $|a|+|b|+|c|+|d|=a+b+c-d=P(1)-2 P(0) \leq 3$. +(3) $c<0, d \geq 0$. Then + +$$ +\begin{aligned} +|a|+|b|+|c|+|d| & =a+b-c+d \\ +& =\frac{4}{3} P(1)-\frac{1}{3} P(-1)-\frac{8}{3} P(1 / 2)+\frac{8}{3} P(-1 / 2) \leq 7 . +\end{aligned} +$$ + +(4) $c<0, d<0$. Then + +$$ +\begin{aligned} +|a|+|b|+|c|+|d| & =a+b-c-d \\ +& =\frac{5}{3} P(1)-4 P(1 / 2)+\frac{4}{3} P(-1 / 2) \leq 7 +\end{aligned} +$$ + +Remark. It can be shown that the maximum of 7 is attained only for $P(x)= \pm\left(4 x^{3}-3 x\right)$. +6. Let $f(x), g(x)$ be polynomials with integer coefficients such that + +$$ +f(x)(x+1)^{n}+g(x)\left(x^{n}+1\right)=k_{0} +$$ + +Write $n=2^{r} m$ for $m$ odd and note that $x^{n}+1=\left(x^{2^{r}}+1\right) B(x)$, where $B(x)=x^{2^{r}(m-1)}-x^{2^{r}(m-2)}+\cdots-x^{2^{r}}+1$. Moreover, $B(-1)=1$; hence $B(x)-1=(x+1) c(x)$ and thus + +$$ +R(x) B(x)+1=(B(x)-1)^{n}=(x+1)^{n} c(x)^{n} +$$ + +for some polynomials $c(x)$ and $R(x)$. +The zeros of the polynomial $x^{2^{r}}+1$ are $\omega_{j}$, with $\omega_{1}=\cos \frac{\pi}{2^{r}}+i \sin \frac{\pi}{2^{r}}$, and $\omega_{j}=\omega^{2 j-1}$ for $1 \leq j \leq 2^{r}$. We have + +$$ +\left(\omega_{1}+1\right)\left(\omega_{2}+1\right) \cdots\left(\omega_{2^{r+1}}+1\right)=2 +$$ + +From $(*)$ we also get $f\left(\omega_{j}\right)\left(\omega_{j}+1\right)^{n}=k_{0}$ for $j=1,2, \ldots, 2^{r}$. Since $A=f\left(\omega_{1}\right) f\left(\omega_{2}\right) \cdots f\left(\omega_{2^{r}}\right)$ is a symmetric polynomial in $\omega_{1}, \ldots, \omega_{2^{r}}$ with integer coefficients, $A$ is an integer. Consequently, taking the product over $j=1,2, \ldots, 2^{r}$ and using (2) we deduce that $2^{n} A=k_{0}^{2^{r}}$ is divisible by $2^{n}=2^{2^{r} m}$. Hence $2^{m} \mid k_{0}$. +Furthermore, since $\omega_{j}+1=\left(\omega_{1}+1\right) p_{j}\left(\omega_{1}\right)$ for some polynomial $p_{j}$ with integer coefficients, (2) gives $\left(\omega_{1}+1\right)^{2^{r}} p\left(\omega_{1}\right)=2$, where $p(x)=$ $p_{2}(x) \cdots p_{2^{r}}(x)$ has integer coefficients. But then the polynomial $(x+$ $1)^{2^{2}} p(x)-2$ has a zero $x=\omega_{1}$, so it is divisible by its minimal polynomial $x^{2^{r}}+1$. Therefore + +$$ +(x+1)^{2^{r}} p(x)=2+\left(x^{2^{r}}+1\right) q(x) +$$ + +for some polynomial $q(x)$. Raising (3) to the $m$ th power we get $(x+$ $1)^{n} p(x)^{n}=2^{m}+\left(x^{2^{r}}+1\right) Q(x)$ for some polynomial $Q(x)$ with integer coefficients. Now using (1) we obtain + +$$ +\begin{aligned} +(x+1)^{n} c(x)^{n}\left(x^{2^{r}}+1\right) Q(x) & =\left(x^{2^{r}}+1\right) Q(x)+\left(x^{2^{r}}+1\right) Q(x) B(x) R(x) \\ +& =(x+1)^{n} p(x)^{n}-2^{m}+\left(x^{n}+1\right) Q(X) R(x) . +\end{aligned} +$$ + +Therefore $(x+1)^{n} f(x)+\left(x^{n}+1\right) g(x)=2^{m}$ for some polynomials $f(x), g(x)$ with integer coefficients, and $k_{0}=2^{m}$. +7. We are given that $f(x+a+b)-f(x+a)=f(x+b)-f(x)$, where $a=1 / 6$ and $b=1 / 7$. Summing up these equations for $x, x+b, \ldots, x+6 b$ we obtain $f(x+a+1)-f(x+a)=f(x+1)-f(x)$. Summing up the new equations for $x, x+a, \ldots, x+5 a$ we obtain that + +$$ +f(x+2)-f(x+1)=f(x+1)-f(x) . +$$ + +It follows by induction that $f(x+n)-f(x)=n[f(x+1)-f(x)]$. If $f(x+1) \neq f(x)$, then $f(x+n)-f(x)$ will exceed in absolute value an arbitrarily large number for a sufficiently large $n$, contradicting the assumption that $f$ is bounded. Hence $f(x+1)=f(x)$ for all $x$. +8. Putting $m=n=0$ we obtain $f(0)=0$ and consequently $f(f(n))=f(n)$ for all $n$. Thus the given functional equation is equivalent to + +$$ +f(m+f(n))=f(m)+f(n), \quad f(0)=0 +$$ + +Clearly one solution is $(\forall x) f(x)=0$. Suppose $f$ is not the zero function. We observe that $f$ has nonzero fixed points (for example, any $f(n)$ is a fixed point). Let $a$ be the smallest nonzero fixed point of $f$. By induction, each $k a(k \in \mathbb{N})$ is a fixed point too. We claim that all fixed points of $f$ are of this form. Indeed, suppose that $b=k a+i$ is a fixed point, where $i45^{\circ}$, then $\angle F H P=\angle A$. If $\angle A=45^{\circ}$, the point $P$ escapes to infinity. If $\angle A<45^{\circ}$, the point $P$ appears on the extension of $A C$ over $C$, and $\angle F H P=180^{\circ}-\angle A$. +13. By the law of cosines applied to $\triangle C A_{1} B_{1}$, we obtain + +$$ +A_{1} B_{1}^{2}=A_{1} C^{2}+B_{1} C^{2}-A_{1} C \cdot B_{1} C \geq A_{1} C \cdot B_{1} C +$$ + +Analogously, $B_{1} C_{1}^{2} \geq B_{1} A \cdot C_{1} A$ and $C_{1} A_{1}^{2} \geq C_{1} B \cdot A_{1} B$, so that multiplying these inequalities yields + +$$ +A_{1} B_{1}^{2} \cdot B_{1} C_{1}^{2} \cdot C_{1} A_{1}^{2} \geq A_{1} B \cdot A_{1} C \cdot B_{1} A \cdot B_{1} C \cdot C_{1} A \cdot C_{1} B +$$ + +Now, the lines $A A_{1}, B B_{1}, C C_{1}$ concur, so by Ceva's theorem, $A_{1} B \cdot B_{1} C$. $C_{1} A=A B_{1} \cdot B C_{1} \cdot C A_{1}$, which together with (1) gives the desired inequality. Equality holds if and only if $C A_{1}=C B_{1}$, etc. +14. Let $a, b, c, d, e$, and $f$ denote the lengths of the sides $A B, B C, C D, D E$, $E F$, and $F A$ respectively. +Note that $\angle A=\angle D, \angle B=\angle E$, and $\angle C=\angle F$. Draw the lines $P Q$ and $R S$ through $A$ and $D$ perpendicular to $B C$ and $E F$ respectively $(P, R \in B C, Q, S \in E F)$. Then $B F \geq P Q=R S$. Therefore $2 B F \geq$ $P Q+R S$, or +![](https://cdn.mathpix.com/cropped/2024_11_18_8e985d6b9c83aa3e9d0eg-617.jpg?height=322&width=389&top_left_y=327&top_left_x=885) + +$$ +\begin{array}{ll} +& 2 B F \geq(a \sin B+f \sin C)+(c \sin C+d \sin B), \\ +\text { and similarly, } & 2 B D \geq(c \sin A+b \sin B)+(e \sin B+f \sin A), \\ +& 2 D F \geq(e \sin C+d \sin A)+(a \sin A+b \sin C) +\end{array} +$$ + +Next, we have the following formulas for the considered circumradii: + +$$ +R_{A}=\frac{B F}{2 \sin A}, \quad R_{C}=\frac{B D}{2 \sin C}, \quad R_{E}=\frac{D F}{2 \sin E} +$$ + +It follows from (1) that + +$$ +\begin{aligned} +R_{A}+R_{C}+R_{E} & \geq \frac{1}{4} a\left(\frac{\sin B}{\sin A}+\frac{\sin A}{\sin B}\right)+\frac{1}{4} b\left(\frac{\sin C}{\sin B}+\frac{\sin B}{\sin C}\right)+\cdots \\ +& \geq \frac{1}{2}(a+b+\cdots)=\frac{P}{2} +\end{aligned} +$$ + +with equality if and only if $\angle A=\angle B=\angle C=120^{\circ}$ and $F B \perp B C$ etc., i.e., if and only if the hexagon is regular. + +Second solution. Let us construct points $A^{\prime \prime}, C^{\prime \prime}, E^{\prime \prime}$ such that $A B A^{\prime \prime} F$, $C D C^{\prime \prime} B$, and $E F E^{\prime \prime} D$ are parallelograms. It follows that $A^{\prime \prime}, C^{\prime \prime}, B$ are collinear and also $C^{\prime \prime}, E^{\prime \prime}, B$ and $E^{\prime \prime}, A^{\prime \prime}, F$. Furthermore, let $A^{\prime}$ be the intersection of the perpendiculars through $F$ and $B$ to $F A^{\prime \prime}$ and $B A^{\prime \prime}$, respectively, and let $C^{\prime}$ and $E^{\prime}$ be analogously defined. Since $A^{\prime} F A^{\prime \prime} B$ is cyclic with the diameter being $A^{\prime} A^{\prime \prime}$ and since $\triangle F A^{\prime \prime} B \cong$ $\triangle B A F$, it follows that $2 R_{A}=$ +![](https://cdn.mathpix.com/cropped/2024_11_18_8e985d6b9c83aa3e9d0eg-617.jpg?height=372&width=504&top_left_y=1470&top_left_x=830) +$A^{\prime} A^{\prime \prime}=x$. +Similarly, $2 R_{C}=C^{\prime} C^{\prime \prime}=y$ and $2 R_{E}=E^{\prime} E^{\prime \prime}=z$. We also have $A B=$ $F A^{\prime \prime}=y_{a}, A F=A^{\prime \prime} B=z_{a}, C D=C^{\prime \prime} B=z_{c}, C B=C^{\prime \prime} D=x_{c}$, $E F=E^{\prime \prime} D=x_{e}$, and $E D=E^{\prime \prime} F=y_{e}$. The original inequality we must prove now becomes + +$$ +x+y+z \geq y_{a}+z_{a}+z_{c}+x_{c}+x_{e}+y_{e} . +$$ + +We now follow and generalize the standard proof of the Erdős-Mordell inequality (for the triangle $A^{\prime} C^{\prime} E^{\prime}$ ), which is what (1) is equivalent to when $A^{\prime \prime}=C^{\prime \prime}=E^{\prime \prime}$. +We set $C^{\prime} E^{\prime}=a, A^{\prime} E^{\prime}=c$ and $A^{\prime} C^{\prime}=e$. Let $A_{1}$ be the point symmetric to $A^{\prime \prime}$ with respect to the bisector of $\angle E^{\prime} A^{\prime} C^{\prime}$. Let $F_{1}$ and $B_{1}$ be the feet of the perpendiculars from $A_{1}$ to $A^{\prime} C^{\prime}$ and $A^{\prime} E^{\prime}$, respectively. In that case, $A_{1} F_{1}=A^{\prime \prime} F=y_{a}$ and $A_{1} B_{1}=A^{\prime \prime} B=z_{a}$. We have + +$$ +\begin{aligned} +a x=A^{\prime} A_{1} \cdot E^{\prime} C^{\prime} \geq 2 S_{A^{\prime} E^{\prime} A_{1} C^{\prime}} & =2 S_{A^{\prime} E^{\prime} A_{1}}+2 S_{A^{\prime} C^{\prime} A_{1}} \\ +& =c z_{a}+e y_{a} . +\end{aligned} +$$ + +Similarly, $c y \geq e x_{c}+a z_{c}$ and $e z \geq a y_{e}+c x_{e}$. Thus + +$$ +\begin{aligned} +x+y+z & \geq \frac{c}{a} z_{a}+\frac{a}{c} z_{c}+\frac{e}{c} x_{c}+\frac{c}{e} x_{e}+\frac{a}{e} y_{e}+\frac{e}{a} y_{a} \\ +& =\left(\frac{c}{a}+\frac{a}{c}\right)\left(\frac{z_{a}+z_{c}}{2}\right)+\left(\frac{c}{a}-\frac{a}{c}\right)\left(\frac{z_{a}-z_{c}}{2}\right)+\cdots . +\end{aligned} +$$ + +Let us set $a_{1}=\frac{x_{c}-x_{e}}{2}, c_{1}=\frac{y_{e}-y_{a}}{2}, e_{1}=\frac{z_{a}-z_{c}}{2}$. We note that $\triangle A^{\prime \prime} C^{\prime \prime} E^{\prime \prime} \sim$ $\triangle A^{\prime} C^{\prime} E^{\prime}$ and hence $a_{1} / a=c_{1} / c=e_{1} / e=k$. Thus $\left(\frac{c}{a}-\frac{a}{c}\right) e_{1}+$ $\left(\frac{e}{c}-\frac{c}{e}\right) a_{1}+\left(\frac{a}{e}-\frac{e}{a}\right) c_{1}=k\left(\frac{c e}{a}-\frac{a e}{c}+\frac{e a}{c}-\frac{c a}{e}+\frac{a c}{e}-\frac{e c}{a}\right)=0$. Equation (2) reduces to + +$$ +\begin{aligned} +x+y+z \geq & \left(\frac{c}{a}+\frac{a}{c}\right)\left(\frac{z_{a}+z_{c}}{2}\right)+\left(\frac{e}{c}+\frac{c}{e}\right)\left(\frac{x_{e}+x_{c}}{2}\right) \\ +& +\left(\frac{a}{e}+\frac{e}{a}\right)\left(\frac{y_{a}+y_{e}}{2}\right) . +\end{aligned} +$$ + +Using $c / a+a / c, e / c+c / e, a / e+e / a \geq 2$ we finally get $x+y+z \geq$ $y_{a}+z_{a}+z_{c}+x_{c}+x_{e}+y_{e}$. +Equality holds if and only if $a=c=e$ and $A^{\prime \prime}=C^{\prime \prime}=E^{\prime \prime}=$ center of $\triangle A^{\prime} C^{\prime} E^{\prime}$, i.e., if and only if $A B C D E F$ is regular. +Remark. From the second proof it is evident that the Erdős-Mordell inequality is a special case of the problem. if $P_{a}, P_{b}, P_{c}$ are the feet of the perpendiculars from a point $P$ inside $\triangle A B C$ to the sides $B C, C A, A B$, and $P_{a} P P_{b} P_{c}^{\prime}, P_{b} P P_{c} P_{a}^{\prime}, P_{c} P P_{a} P_{b}^{\prime}$ parallelograms, we can apply the problem to the hexagon $P_{a} P_{c}^{\prime} P_{b} P_{a}^{\prime} P_{c} P_{b}^{\prime}$ to prove the Erdős-Mordell inequality for $\triangle A B C$ and point $P$. +15. Denote by $A B C D$ and $E F G H$ the two rectangles, where $A B=a, B C=$ $b, E F=c$, and $F G=d$. Obviously, the first rectangle can be placed within the second one with the angle $\alpha$ between $A B$ and $E F$ if and only if + +$$ +a \cos \alpha+b \sin \alpha \leq c, \quad a \sin \alpha+b \cos \alpha \leq d +$$ + +Hence $A B C D$ can be placed within $E F G H$ if and only if there is an $\alpha \in[0, \pi / 2]$ for which (1) holds. + +The lines $l_{1}(a x+b y=c)$ and $l_{2}(b x+a y=d)$ and the axes $x$ and $y$ bound a region $\mathcal{R}$. By (1), the desired placement of the rectangles is possible if and only if $\mathcal{R}$ contains some point $(\cos \alpha, \sin \alpha)$ of the unit circle centered at the origin $(0,0)$. This in turn holds if and only if the intersection point $L$ of $l_{1}$ and $l_{2}$ lies outside the unit circle. It is easily computed that $L$ has coordinates $\left(\frac{b d-a c}{b^{2}-a^{2}}, \frac{b c-a d}{b^{2}-a^{2}}\right)$. Now $L$ being outside the unit circle is exactly equivalent to the inequality we want to prove. +Remark. If equality holds, there is exactly one way of placing. This happens, for example, when $(a, b)=(5,20)$ and $(c, d)=(16,19)$. +Second remark. This problem is essentially very similar to (SL89-2). +16. Let $A_{1}$ be the point of intersection of $O A^{\prime}$ and $B C$; similarly define $B_{1}$ and $C_{1}$. From the similarity of triangles $O B A_{1}$ and $O A^{\prime} B$ we obtain $O A_{1}$. $O A^{\prime}=R^{2}$. Now it is enough to show that $8 O A_{1} \cdot O B^{\prime} \cdot O C^{\prime} \leq R^{3}$. Thus we must prove that + +$$ +\lambda \mu \nu \leq \frac{1}{8}, \quad \text { where } \quad \frac{O A_{1}}{O A}=\lambda, \quad \frac{O B_{1}}{O B}=\mu, \quad \frac{O C_{1}}{O C}=\nu +$$ + +On the other hand, we have + +$$ +\frac{\lambda}{1+\lambda}+\frac{\mu}{1+\mu}+\frac{\nu}{1+\nu}=\frac{S_{O B C}}{S_{A B C}}+\frac{S_{A O C}}{S_{A B C}}+\frac{S_{A B O}}{S_{A B C}}=1 . +$$ + +Simplifying this relation, we get + +$$ +1=\lambda \mu+\mu \nu+\nu \lambda+2 \lambda \mu \nu \geq 3(\lambda \mu \nu)^{2 / 3}+2 \lambda \mu \nu +$$ + +which cannot hold if $\lambda \mu \nu>\frac{1}{8}$. Hence $\lambda \mu \nu \leq \frac{1}{8}$, with equality if and only if $\lambda=\mu=\nu=\frac{1}{2}$. This implies that $O$ is the centroid of $A B C$, and consequently, that the triangle is equilateral. + +Second solution. In the official solution, the inequality to be proved is transformed into + +$$ +\cos (A-B) \cos (B-C) \cos (C-A) \geq 8 \cos A \cos B \cos C +$$ + +Since $\frac{\cos (B-C)}{\cos A}=-\frac{\cos (B-C)}{\cos (B+C)}=\frac{\tan B \tan C+1}{\tan B \tan C-1}$, the last inequality becomes $(x y+1)(y z+1)(z x+1) \geq 8(x y-1)(y z-1)(z x-1)$, where we write $x, y, z$ for $\tan A, \tan B, \tan C$. Using the relation $x+y+z=x y z$, we can reduce this inequality to + +$$ +(2 x+y+z)(x+2 y+z)(x+y+2 z) \geq 8(x+y)(y+z)(z+x) +$$ + +This follows from the AM-GM inequality: $2 x+y+z=(x+y)+(x+z) \geq$ $2 \sqrt{(x+y)(x+z)}$, etc. +17. Let the diagonals $A C$ and $B D$ meet in $X$. Either $\angle A X B$ or $\angle A X D$ is geater than or equal to $90^{\circ}$, so we assume w.l.o.g. that $\angle A X B \geq 90^{\circ}$. Let $\alpha, \beta, \alpha^{\prime}, \beta^{\prime}$ denote $\angle C A B, \angle A B D, \angle B D C, \angle D C A$. These angles are all acute and satisfy $\alpha+\beta=\alpha^{\prime}+\beta^{\prime}$. Furthermore, + +$$ +R_{A}=\frac{A D}{2 \sin \beta}, \quad R_{B}=\frac{B C}{2 \sin \alpha}, \quad R_{C}=\frac{B C}{2 \sin \alpha^{\prime}}, \quad R_{D}=\frac{A D}{2 \sin \beta^{\prime}} +$$ + +Let $\angle B+\angle D=180^{\circ}$. Then $A, B, C, D$ are concyclic and trivially $R_{A}+$ $R_{C}=R_{B}+R_{D}$. +Let $\angle B+\angle D>180^{\circ}$. Then $D$ lies within the circumcircle of $A B C$, which implies that $\beta>\beta^{\prime}$. Similarly $\alpha<\alpha^{\prime}$, so we obtain $R_{A}R_{D}$ and $R_{C}>R_{B}$, so $R_{A}+R_{C}>R_{B}+R_{D}$. +18. We first prove the result in the simplest case. Given a 2 -gon $A B A$ and a point $O$, let $a, b, c, h$ denote $O A, O B, A B$, and the distance of $O$ from $A B$. Then $D=a+b, P=2 c$, and $H=2 h$, so we should show that + +$$ +(a+b)^{2} \geq 4 h^{2}+c^{2} +$$ + +Indeed, let $l$ be the line through $O$ parallel to $A B$, and $D$ the point symmetric to $B$ with respect to $l$. Then $(a+b)^{2}=(O A+O B)^{2}=(O A+$ $O D)^{2} \geq A D^{2}=c^{2}+4 h^{2}$. +Now we pass to the general case. Let $A_{1} A_{2} \ldots A_{n}$ be the polygon $\mathcal{F}$ and denote by $d_{i}, p_{i}$, and $h_{i}$ respectively $O A_{i}, A_{i} A_{i+1}$, and the distance of $O$ from $A_{i} A_{i+1}$ (where $A_{n+1}=A_{1}$ ). By the case proved above, we have for each $i, d_{i}+d_{i+1} \geq \sqrt{4 h_{i}^{2}+p_{i}^{2}}$. Summing these inequalities for $i=1, \ldots, n$ and squaring, we obtain + +$$ +4 D^{2} \geq\left(\sum_{i=1}^{n} \sqrt{4 h_{i}^{2}+p_{i}^{2}}\right)^{2} +$$ + +It remains only to prove that $\sum_{i=1}^{n} \sqrt{4 h_{i}^{2}+p_{i}^{2}} \geq \sqrt{\sum_{i=1}^{n}\left(4 h_{i}^{2}+p_{i}^{2}\right)}=$ $\sqrt{4 H^{2}+D^{2}}$. But this follows immediately from the Minkowski inequality. Equality holds if and only if it holds in (1) and in the Minkowski inequality, i.e., if and only if $d_{1}=\cdots=d_{n}$ and $h_{1} / p_{1}=\cdots=h_{n} / p_{n}$. This means that $\mathcal{F}$ is inscribed in a circle with center at $O$ and $p_{1}=\cdots=p_{n}$, so $\mathcal{F}$ is a regular polygon and $O$ its center. +19. It is easy to check that after 4 steps we will have all $a, b, c, d$ even. Thus $|a b-c d|,|a c-b d|,|a d-b c|$ remain divisible by 4 , and clearly are not prime. The answer is no. +Second solution. After one step we have $a+b+c+d=0$. Then $a c-b d=$ $a c+b(a+b+c)=(a+b)(b+c)$ etc., so + +$$ +|a b-c d| \cdot|a c-b d| \cdot|a d-b c|=(a+b)^{2}(a+c)^{2}(b+c)^{2} +$$ + +However, the product of three primes cannot be a square, hence the answer is $n o$. +20. Let $15 a+16 b=x^{2}$ and $16 a-15 b=y^{2}$, where $x, y \in \mathbb{N}$. Then we obtain $x^{4}+y^{4}=(15 a+16 b)^{2}+(16 a-15 b)^{2}=\left(15^{2}+16^{2}\right)\left(a^{2}+b^{2}\right)=481\left(a^{2}+b^{2}\right)$. In particular, $481=13 \cdot 37 \mid x^{4}+y^{4}$. We have the following lemma. +Lemma. Suppose that $p \mid x^{4}+y^{4}$, where $x, y \in \mathbb{Z}$ and $p$ is an odd prime, where $p \not \equiv 1(\bmod 8)$. Then $p \mid x$ and $p \mid y$. +Proof. Since $p \mid x^{8}-y^{8}$ and by Fermat's theorem $p \mid x^{p-1}-y^{p-1}$, we deduce that $p \mid x^{d}-y^{d}$, where $d=(p-1,8)$. But $d \neq 8$, so $d \mid 4$. Thus $p \mid x^{4}-y^{4}$, which implies that $p \mid 2 y^{4}$, i.e., $p \mid y$ and $p \mid x$. +In particular, we can conclude that $13 \mid x, y$ and $37 \mid x, y$. Hence $x$ and $y$ are divisible by 481 . Thus each of them is at least 481. +On the other hand, $x=y=481$ is possible. It is sufficient to take $a=$ $31 \cdot 481$ and $b=481$. + +Second solution. Note that $15 x^{2}+16 y^{2}=481 a^{2}$. It can be directly verified that the divisibility of $15 x^{2}+16 y^{2}$ by 13 and by 37 implies that both $x$ and $y$ are divisible by both primes. Thus $481 \mid x, y$. +21. (a) It clearly suffices to show that for every integer $c$ there exists a quadratic sequence with $a_{0}=0$ and $a_{n}=c$, i.e., that $c$ can be expressed as $\pm 1^{2} \pm 2^{2} \pm \cdots \pm n^{2}$. Since + +$$ +(n+1)^{2}-(n+2)^{2}-(n+3)^{2}+(n+4)^{2}=4 +$$ + +we observe that if our claim is true for $c$, then it is also true for $c \pm 4$. Thus it remains only to prove the claim for $c=0,1,2,3$. But one immediately finds $1=1^{2}, 2=-1^{2}-2^{2}-3^{2}+4^{2}$, and $3=-1^{2}+2^{2}$, while the case $c=0$ is trivial. +(b) We have $a_{0}=0$ and $a_{n}=1996$. Since $a_{n} \leq 1^{2}+2^{2}+\cdots+n^{2}=$ $\frac{1}{6} n(n+1)(2 n+1)$, we get $a_{17} \leq 1785$, so $n \geq 18$. On the other hand, $a_{18}$ is of the same parity as $1^{2}+2^{2}+\cdots+18^{2}=2109$, so it cannot be equal to 1996. Therefore we must have $n \geq 19$. To construct a required sequence with $n=19$, we note that $1^{2}+2^{2}+\cdots+19^{2}=$ $2470=1996+2 \cdot 237$; hence it is enough to write 237 as a sum of distinct squares. Since $237=14^{2}+5^{2}+4^{2}$, we finally obtain + +$$ +1996=1^{2}+2^{2}+3^{2}-4^{2}-5^{2}+6^{2}+\cdots+13^{2}-14^{2}+15^{2}+\cdots+19^{2} +$$ + +22. Let $a, b \in \mathbb{N}$ satisfy the given equation. It is not possible that $a=b$ (since it leads to $a^{2}+2=2 a$ ), so we assume w.l.o.g. that $a>b$. Next, for $a>b=1$ the equation becomes $a^{2}=2 a$, and one obtains a solution $(a, b)=(2,1)$. +Let $b>1$. If $\left[\frac{a^{2}}{b}\right]=\alpha$ and $\left[\frac{b^{2}}{a}\right]=\beta$, then we trivially have $a b \geq$ $\alpha \beta$. Since also $\frac{a^{2}+b^{2}}{a b} \geq 2$, we obtain $\alpha+\beta \geq \alpha \beta+2$, or equivalently +$(\alpha-1)(\beta-1) \leq-1$. But $\alpha \geq 1$, and therefore $\beta=0$. It follows that $a>b^{2}$, i.e., $a=b^{2}+c$ for some $c>0$. Now the given equation becomes $b^{3}+2 b c+\left[\frac{c^{2}}{b}\right]=\left[\frac{b^{4}+2 b^{2} c+b^{2}+c^{2}}{b^{3}+b c}\right]+b^{3}+b c$, which reduces to + +$$ +(c-1) b+\left[\frac{c^{2}}{b}\right]=\left[\frac{b^{2}(c+1)+c^{2}}{b^{3}+b c}\right] +$$ + +If $c=1$, then (1) always holds, since both sides are 0 . We obtain a family of solutions $(a, b)=\left(n, n^{2}+1\right)$ or $(a, b)=\left(n^{2}+1, n\right)$. Note that the solution $(1,2)$ found earlier is obtained for $n=1$. +If $c>1$, then $(1)$ implies that $\frac{b^{2}(c+1)+c^{2}}{b^{3}+b c} \geq(c-1) b$. This simplifies to + +$$ +c^{2}\left(b^{2}-1\right)+b^{2}\left(c\left(b^{2}-2\right)-\left(b^{2}+1\right)\right) \leq 0 +$$ + +Since $c \geq 2$ and $b^{2}-2 \geq 0$, the only possibility is $b=2$. But then (2) becomes $3 c^{2}+8 c-20 \leq 0$, which does not hold for $c \geq 2$. +Hence the only solutions are $\left(n, n^{2}+1\right)$ and $\left(n^{2}+1, n\right), n \in \mathbb{N}$. +23. We first observe that the given functional equation is equivalent to + +$$ +4 f\left(\frac{(3 m+1)(3 n+1)-1}{3}\right)+1=(4 f(m)+1)(4 f(n)+1) . +$$ + +This gives us the idea of introducing a function $g: 3 \mathbb{N}_{0}+1 \rightarrow 4 \mathbb{N}_{0}+1$ defined as $g(x)=4 f\left(\frac{x-1}{3}\right)+1$. By the above equality, $g$ will be multiplicative, i.e., + +$$ +g(x y)=g(x) g(y) \quad \text { for all } x, y \in 3 \mathbb{N}_{0}+1 +$$ + +Conversely, any multiplicative bijection $g$ from $3 \mathbb{N}_{0}+1$ onto $4 \mathbb{N}_{0}+1$ gives us a function $f$ with the required property: $f(x)=\frac{g(3 x+1)-1}{4}$. +It remains to give an example of such a function $g$. Let $P_{1}, P_{2}, Q_{1}, Q_{2}$ be the sets of primes of the forms $3 k+1,3 k+2,4 k+1$, and $4 k+3$, respectively. It is well known that these sets are infinite. Take any bijection $h$ from $P_{1} \cup P_{2}$ onto $Q_{1} \cup Q_{2}$ that maps $P_{1}$ bijectively onto $Q_{1}$ and $P_{2}$ bijectively onto $Q_{2}$. Now define $g$ as follows: $g(1)=1$, and for $n=p_{1} p_{2} \cdots p_{m}\left(p_{i}\right.$ 's need not be different) define $g(n)=h\left(p_{1}\right) h\left(p_{2}\right) \cdots h\left(p_{m}\right)$. Note that $g$ is well-defined. Indeed, among the $p_{i}$ 's an even number are of the form $3 k+2$, and consequently an even number of $h\left(p_{i}\right) \mathrm{s}$ are of the form $4 k+3$. Hence the product of the $h\left(p_{i}\right)$ 's is of the form $4 k+1$. Also, it is obvious that $g$ is multiplicative. Thus, the defined $g$ satisfies all the required properties. +24. We shall work on the array of lattice points defined by $\mathcal{A}=\left\{(x, y) \in \mathbb{Z}^{2} \mid\right.$ $0 \leq x \leq 19,0 \leq y \leq 11\}$. Our task is to move from $(0,0)$ to $(19,0)$ via the points of $\mathcal{A}$ so that each move has the form $(x, y) \rightarrow(x+a, y+b)$, where $a, b \in \mathbb{Z}$ and $a^{2}+b^{2}=r$. +(a) If $r$ is even, then $a+b$ is even whenever $a^{2}+b^{2}=r(a, b \in \mathbb{Z})$. Thus the parity of $x+y$ does not change after each move, so we cannot reach $(19,0)$ from $(0,0)$. +If $3 \mid r$, then both $a$ and $b$ are divisible by 3 , so if a point $(x, y)$ can be reached from $(0,0)$, we must have $3 \mid x$. Since $3 \nmid 19$, we cannot get to $(19,0)$. +(b) We have $r=73=8^{2}+3^{2}$, so each move is either $(x, y) \rightarrow(x \pm 8, y \pm 3)$ or $(x, y) \rightarrow(x \pm 3, y \pm 8)$. One possible solution is shown in Fig. 1. +(c) We have $97=9^{2}+4^{2}$. Let us partition $\mathcal{A}$ as $\mathcal{B} \cup \mathcal{C}$, where $\mathcal{B}=$ $\{(x, y) \in \mathcal{A} \mid 4 \leq y \leq 7\}$. It is easily seen that moves of the type $(x, y) \rightarrow(x \pm 9, y \pm 4)$ always take us from the set $\mathcal{B}$ to $\mathcal{C}$ and vice versa, while the moves $(x, y) \rightarrow(x \pm 4, y \pm 9)$ always take us from $\mathcal{C}$ to $\mathcal{C}$. Furthermore, each move of the type $(x, y) \rightarrow(x \pm 9, y \pm 4)$ changes the parity of $x$, so to get from $(0,0)$ to $(19,0)$ we must have an odd number of such moves. On the other hand, with an odd number of such moves, starting from $\mathcal{C}$ we can end up only in $\mathcal{B}$, although the point $(19,0)$ is not in $\mathcal{B}$. Hence, the answer is no. +Remark. Part (c) can also be solved by examining all cells that can be reached from $(0,0)$. All these cells are marked in Fig. 2. +![](https://cdn.mathpix.com/cropped/2024_11_18_8e985d6b9c83aa3e9d0eg-623.jpg?height=259&width=420&top_left_y=1081&top_left_x=289) + +Fig. 1 +![](https://cdn.mathpix.com/cropped/2024_11_18_8e985d6b9c83aa3e9d0eg-623.jpg?height=256&width=414&top_left_y=1083&top_left_x=886) + +Fig. 2 +25. Let the vertices in the bottom row be assigned an arbitrary coloring, and suppose that some two adjacent vertices receive the same color. The number of such colorings equals $2^{n}-2$. It is easy to see that then the colors of the remaining vertices get fixed uniquely in order to satisfy the requirement. So in this case there are $2^{n}-2$ possible colorings. +Next, suppose that the vertices in the bottom row are colored alternately red and blue. There are two such colorings. In this case, the same must hold for every row, and thus we get $2^{n}$ possible colorings. +It follows that the total number of considered colorings is $\left(2^{n}-2\right)+2^{n}=$ $2^{n+1}-2$. +26. Denote the required maximum size by $M_{k}(m, n)$. If $m<\frac{n(n+1)}{2}$, then trivially $M=k$, so from now on we assume that $m \geq \frac{n(n+1)}{2}$. +First we give a lower bound for $M$. Let $r=r_{k}(m, n)$ be the largest integer such that $r+(r+1)+\cdots+(r+n-1) \leq m$. This is equivalent to $n r \leq m-\frac{n(n-1)}{2} \leq n(r+1)$, so $r=\left[\frac{m}{n}-\frac{n-1}{2}\right]$. Clearly no $n$ elements from $\{r+1, r+2, \ldots, k\}$ add up to $m$, so + +$$ +M \geq k-r_{k}(m, n)=k-\left[\frac{m}{n}-\frac{n-1}{2}\right] . +$$ + +We claim that $M$ is actually equal to $k-r_{k}(m, n)$. To show this, we shall prove by induction on $n$ that if no $n$ elements of a set $S \subseteq\{1,2, \ldots, k\}$ add up to $m$, then $|S| \leq k-r_{k}(m, n)$. +For $n=2$ the claim is true, because then for each $i=1, \ldots, r_{k}(m, 2)=$ $\left[\frac{m-1}{2}\right]$ at least one of $i$ and $m-i$ must be excluded from $S$. Now let us assume that $n>2$ and that the result holds for $n-1$. Suppose that $S \subseteq\{1,2, \ldots, k\}$ does not contain $n$ distinct elements with the sum $m$, and let $x$ be the smallest element of $S$. We may assume that $x \leq r_{k}(m, n)$, because otherwise the statement is clear. Consider the set $S^{\prime}=\{y-x \mid$ $y \in S, y \neq x\}$. Then $S^{\prime}$ is a subset of $\{1,2, \ldots, k-x\}$ no $n-1$ elements of which have the sum $m-n x$. Also, it is easily checked that $n-1 \leq$ $m-n x-1 \leq k-x$, so we may apply the induction hypothesis, which yields that + +$$ +|S| \leq 1+k-x-r_{k}(m-n x, n-1)=k-\left[\frac{m-x}{n-1}-\frac{n}{2}\right] . +$$ + +On the other hand, $\left(\frac{m-x}{n-1}-\frac{n}{2}\right)-r_{k}(m, n)=\frac{m-n x-\frac{n(n-1)}{2}}{n(n-1)} \geq 0$ because $x \leq r_{k}(m, n)$; hence (2) implies $|S| \leq k-r_{k}(m, n)$ as claimed. +27. Suppose that such sets of points $\mathcal{A}, \mathcal{B}$ exist. + +First, we observe that there exist five points $A, B, C, D, E$ in $\mathcal{A}$ such that their convex hull does not contain any other point of $\mathcal{A}$. Indeed, take any point $A \in \mathcal{A}$. Since any two points of $\mathcal{A}$ are at distance at least 1 , the number of points $X \in \mathcal{A}$ with $X A \leq r$ is finite for every $r>0$. Thus it is enough to choose four points $B, C, D, E$ of $\mathcal{A}$ that are closest to $A$. Now consider the convex hull $\mathcal{C}$ of $A, B, C, D, E$. +Suppose that $\mathcal{C}$ is a pentagon, say $A B C D E$. Then each of the disjoint triangles $A B C, A C D, A D E$ contains a point of $\mathcal{B}$. Denote these points by $P, Q, R$. Then $\triangle P Q R$ contains some point $F \in \mathcal{A}$, so $F$ is inside $A B C D E$, a contradiction. +Suppose that $\mathcal{C}$ is a quadrilateral, say $A B C D$, with $E$ lying within $A B C D$. Then the triangles $A B E, B C E, C D E, D A E$ contain some points $P, Q, R, S$ of $\mathcal{B}$ that form two disjoint triangles. It follows that there are two points of $\mathcal{A}$ inside $A B C D$, which is a contradiction. +Finally, suppose that $\mathcal{C}$ is a triangle with two points of $\mathcal{A}$ inside. Then $\mathcal{C}$ is the union of five disjoint triangles with vertices in $\mathcal{A}$, so there are at least five points of $\mathcal{B}$ inside $\mathcal{C}$. These five points make at least three disjoint triangles containing three points of $\mathcal{A}$. This is again a contradiction. It follows that no such sets $\mathcal{A}, \mathcal{B}$ exist. +28. Note that w.l.o.g., we can assume that $p$ and $q$ are coprime. Indeed, otherwise it suffices to consider the problem in which all $x_{i}$ 's and $p, q$ are divided by $\operatorname{gcd}(p, q)$. + +Let $k, l$ be the number of indices $i$ with $x_{i+1}-x_{i}=p$ and the number of those $i$ with $x_{i+1}-x_{i}=-q(0 \leq i1, k=q t, l=p t$, and $n=(p+q) t$. +Consider the sequence $y_{i}=x_{i+p+q}-x_{i}, i=0, \ldots, n-p-q$. We claim that at least one of the $y_{i}$ 's equals zero. We begin by noting that each $y_{i}$ is of the form $u p-v q$, where $u+v=p+q$; therefore $y_{i}=(u+v) p-$ $v(p+q)=(p-v)(p+q)$ is always divisible by $p+q$. Moreover, $y_{i+1}-y_{i}=$ $\left(x_{i+p+q+1}-x_{i+p+q}\right)-\left(x_{i+1}-x_{i}\right)$ is 0 or $\pm(p+q)$. We conclude that if no $y_{i}$ is 0 then all $y_{i}$ 's are of the same sign. But this is in contradiction with the relation $y_{0}+y_{p+q}+\cdots+y_{n-p-q}=x_{n}-x_{0}=0$. Consequently some $y_{i}$ is zero, as claimed. + +Second solution. As before we assume $(p, q)=1$. Let us define a sequence of points $A_{i}\left(y_{i}, z_{i}\right)(i=0,1, \ldots, n)$ in $\mathbb{N}_{0}^{2}$ inductively as follows. Set $A_{0}=$ $(0,0)$ and define $\left(y_{i+1}, z_{i+1}\right)$ as $\left(y_{i}, z_{i}+1\right)$ if $x_{i+1}=x_{i}+p$ and $\left(y_{i}+1, z_{i}\right)$ otherwise. The points $A_{i}$ form a trajectory $L$ in $\mathbb{N}_{0}^{2}$ continuously moving upwards and rightwards by steps of length 1 . Clearly, $x_{i}=p z_{i}-q y_{i}$ for all $i$. Since $x_{n}=0$, it follows that $\left(z_{n}, y_{n}\right)=(k q, k p), k \in \mathbb{N}$. Since $y_{n}+z_{n}=n>p+q$, it follows that $k>1$. We observe that $x_{i}=x_{j}$ if and only if $A_{i} A_{j} \| A_{0} A_{n}$. We shall show that such $i, j$ with $i1$. Such an index $j$ exists, since otherwise the game is over. Then one must make at least one move in the $j$ th cell, which implies that $x_{j}, x_{j}^{\prime} \geq 1$. However, then the sequences $\left\{x_{i}\right\}$ and $\left\{x_{i}^{\prime}\right\}$ with $x_{j}$ and $x_{j}^{\prime}$ decreased by 1 also satisfy (1) for a sequence $\left\{b_{i}\right\}$ where $b_{j-1}, b_{j}, b_{j+1}$ is replaced with $b_{j-1}+1, b_{j}-2, b_{j+1}+1$. This contradicts the assumption of minimal $\min \left\{\sum_{i \in \mathbb{Z}} x_{i}, \sum_{i \in \mathbb{Z}} x_{i}^{\prime}\right\}$ for the initial $\left\{b_{i}\right\}$. +30. For convenience, we shall write $f^{2}, f g, \ldots$ for the functions $f \circ f, f \circ g, \ldots$ We need two lemmas. +Lemma 1. If $f(x) \in S$ and $g(x) \in T$, then $x \in S \cap T$. +Proof. The given condition means that $f^{3}(x)=g^{2} f(x)$ and $g f g(x)=$ $f g^{2}(x)$. Since $x \in S \cup T=U$, we have two cases: +$x \in S$. Then $f^{2}(x)=g^{2}(x)$, which also implies $f^{3}(x)=f g^{2}(x)$. Therefore $g f g(x)=f g^{2}(x)=f^{3}(x)=g^{2} f(x)$, and since $g$ is a bijection, we obtain $f g(x)=g f(x)$, i.e., $x \in T$. +$x \in T$. Then $f g(x)=g f(x)$, so $g^{2} f(x)=g f g(x)$. It follows that $f^{3}(x)=g^{2} f(x)=g f g(x)=f g^{2}(x)$, and since $f$ is a bijection, we obtain $x \in S$. +Hence $x \in S \cap T$ in both cases. Similarly, $f(x) \in T$ and $g(x) \in S$ again imply $x \in S \cap T$. +Lemma 2. $f(S \cap T)=g(S \cap T)=S \cap T$. +Proof. By symmetry, it is enough to prove $f(S \cap T)=S \cap T$, or in other words that $f^{-1}(S \cap T)=S \cap T$. Since $S \cap T$ is finite, this is equivalent to $f(S \cap T) \subseteq S \cap T$. +Let $f(x) \in S \cap T$. Then if $g(x) \in S$ (since $f(x) \in T$ ), Lemma 1 gives $x \in S \cap T$; similarly, if $g(x) \in T$, then by Lemma $1, x \in S \cap T$. +Now we return to the problem. Assume that $f(x) \in S$. If $g(x) \notin S$, then $g(x) \in T$, so from Lemma 1 we deduce that $x \in S \cap T$. Then Lemma 2 claims that $g(x) \in S \cap T$ too, a contradiction. Analogously, from $g(x) \in S$ we are led to $f(x) \in S$. This finishes the proof. + +### 4.38 Solutions to the Shortlisted Problems of IMO 1997 + +1. Let $A B C$ be the given triangle, with $\angle B=90^{\circ}$ and $A B=m, B C=n$. For an arbitrary polygon $\mathcal{P}$ we denote by $w(\mathcal{P})$ and $b(\mathcal{P})$ respectively the total areas of the white and black parts of $\mathcal{P}$. +(a) Let $D$ be the fourth vertex of the rectangle $A B C D$. When $m$ and $n$ are of the same parity, the coloring of the rectangle $A B C D$ is centrally symmetric with respect to the midpoint of $A C$. It follows that $w(A B C)=\frac{1}{2} w(A B C D)$ and $b(A B C)=\frac{1}{2} b(A B C D)$; thus $f(m, n)=\frac{1}{2}|w(A B C D)-b(A B C D)|$. Hence $f(m, n)$ equals $\frac{1}{2}$ if $m$ and $n$ are both odd, and 0 otherwise. +(b) The result when $m, n$ are of the same parity follows from (a). Suppose that $m>n$, where $m$ and $n$ are of different parity. Choose a point $E$ on $A B$ such that $A E=1$. Since by (a) $|w(E B C)-b(E B C)|=$ $f(m-1, n) \leq \frac{1}{2}$, we have $f(m, n) \leq \frac{1}{2}+|w(E A C)-b(E A C)| \leq$ $\frac{1}{2}+S(E A C)=\frac{1}{2}+\frac{n-1}{2}=\frac{n}{2}$. Therefore $f(m, n) \leq \frac{1}{2} \min (m, n)$. +(c) Let us calculate $f(m, n)$ for $m=2 k+1, n=2 k, k \in \mathbb{N}$. With $E$ defined as in (b), we have $B E=B C=2 k$. If the square at $B$ is w.l.o.g. white, $C E$ passes only through black squares. The white part of $\triangle E A C$ then consists of $2 k$ similar triangles with areas $\frac{1}{2} \frac{i}{2 k} \frac{i}{2 k+1}=\frac{i^{2}}{4 k(2 k+1)}$, where $i=1,2, \ldots, 2 k$. The total white area of $E A C$ is + +$$ +\frac{1}{4 k(2 k+1)}\left(1^{2}+2^{2}+\cdots+(2 k)^{2}\right)=\frac{4 k+1}{12} . +$$ + +Therefore the black area is $(8 k-1) / 12$, and $f(2 k+1,2 k)=(2 k-1) / 6$, which is not bounded. +2. For any sequence $X=\left(x_{1}, x_{2}, \ldots, x_{n}\right)$ let us define + +$$ +\bar{X}=\left(1,2, \ldots, x_{1}, 1,2, \ldots, x_{2}, \ldots, 1,2, \ldots, x_{n}\right) +$$ + +Also, for any two sequences $A, B$ we denote their concatenation by $A B$. It clearly holds that $\overline{A B}=\bar{A} \bar{B}$. The sequences $R_{1}, R_{2}, \ldots$ are given by $R_{1}=(1)$ and $R_{n}=\overline{R_{n-1}}(n)$ for $n>1$. +We consider the family of sequences $Q_{n i}$ for $n, i \in \mathbb{N}, i \leq n$, defined as follows: +$Q_{n 1}=(1), \quad Q_{n n}=(n), \quad$ and $\quad Q_{n i}=Q_{n-1, i-1} Q_{n-1, i} \quad$ if $11$. Let $x \in\{1,2, \ldots, 2 n-1\}$ be a fixed number that does not appear on the fixed diagonal of $A$. Such an element must exist, since the diagonal can contain at most $n$ different numbers. Let us call the union of the $i$ th row and the $i$ th column the $i$ th cross. There are $n$ crosses, and each of them contains exactly one $x$. On the other hand, each entry $x$ of $A$ is contained in exactly two crosses. Hence $n$ must be even. However, 1997 is an odd number; hence no coveralls matrix exists for $n=1997$. +(b) For $n=2, A_{2}=\left[\begin{array}{ll}1 & 2 \\ 3 & 1\end{array}\right]$ is a coveralls matrix. For $n=4$, one such matrix is, for example, + +$$ +A_{4}=\left[\begin{array}{llll} +1 & 2 & 5 & 6 \\ +3 & 1 & 7 & 5 \\ +4 & 6 & 1 & 2 \\ +7 & 4 & 3 & 1 +\end{array}\right] +$$ + +This construction can be generalized. Suppose that we are given an $n \times n$ coveralls matrix $A_{n}$. Let $B_{n}$ be the matrix obtained from $A_{n}$ by adding $2 n$ to each entry, and $C_{n}$ the matrix obtained from $B_{n}$ by replacing each diagonal entry (equal to $2 n+1$ by induction) with $2 n$. Then the matrix + +$$ +A_{2 n}=\left[\begin{array}{ll} +A_{n} & B_{n} \\ +C_{n} & A_{n} +\end{array}\right] +$$ + +is coveralls. To show this, suppose that $i \leq n$ (the case $i>n$ is similar). The $i$ th cross is composed of the $i$ th cross of $A_{n}$, the $i$ th row of $B_{n}$, and the $i$ th column of $C_{n}$. The $i$ th cross of $A_{i}$ covers $1,2, \ldots, 2 n-1$. The $i$ th row of $B_{n}$ covers all numbers of the form $2 n+j$, where $j$ is covered by the $i$ th row of $A_{n}$ (including $j=1$ ). Similarly, the $i$ th column of $C_{n}$ covers $2 n$ and all numbers of the form $2 n+k$, where $k>1$ is covered by the $i$ th column of $A_{n}$. Thus we see that all numbers are accounted for in the $i$ th cross of $A_{2 n}$, and hence $A_{2 n}$ is a desired coveralls matrix. It follows that we can find a coveralls matrix whenever $n$ is a power of 2 . +Second solution for part $b$. We construct a coveralls matrix explicitly for $n=2^{k}$. We consider the coordinates/cells of the matrix elements modulo $n$ throughout the solution. We define the $i$-diagonal $(0 \leq i<$ $n$ ) to be the set of cells of the form $(j, j+i)$, for all $j$. We note that each cross contains exactly one cell from the 0 -diagonal (the main diagonal) and two cells from each $i$-diagonal. For two cells within an $i$ diagonal, $x$ and $y$, we define $x$ and $y$ to be related if there exists a cross containing both $x$ and $y$. Evidently, for every cell $x$ not on the 0 -diagonal there are exactly two other cells related to it. The relation thus breaks up each $i$-diagonal $(i>0)$ into cycles of length larger than 1 . Due to the diagonal translational symmetry (modulo $n$ ), all the cycles within a given $i$-diagonal must be of equal length and thus of an even length, since $n=2^{k}$. +The construction of a coveralls matrix is now obvious. We select a number, say 1, to place on all the cells of the 0-diagonal. We pair up the remaining numbers and assign each pair to an $i$-diagonal, say $(2 i, 2 i+1)$. Going along each cycle within the $i$-diagonal we alternately assign values of $2 i$ and $2 i+1$. Since the cycle has an even length, a cell will be related only to a cell of a different number, and hence each cross will contain both $2 i$ and $2 i+1$. +5. We shall prove first the 2-dimensional analogue: + +Lemma. Given an equilateral triangle $A B C$ and two points $M, N$ on the sides $A B$ and $A C$ respectively, there exists a triangle with sides $C M, B N, M N$. +Proof. Consider a regular tetrahedron $A B C D$. Since $C M=D M$ and $B N=D N$, one such triangle is $D M N$. + +Now, to solve the problem for a regular tetrahedron $A B C D$, we consider a 4-dimensional polytope $A B C D E$ whose faces $A B C D, A B C E, A B D E$, $A C D E, B C D E$ are regular tetrahedra. We don't know what it looks like, but it yields a desired triangle: for $M \in A B C$ and $N \in A D C$, we have $D M=E M$ and $B N=E N$; hence the desired triangle is $E M N$. +Remark. A solution that avoids embedding in $\mathbb{R}^{4}$ is possible, but no longer so short. +6. (a) One solution is + +$$ +x=2^{n^{2}} 3^{n+1}, \quad y=2^{n^{2}-n} 3^{n}, \quad z=2^{n^{2}-2 n+2} 3^{n-1} . +$$ + +(b) Suppose w.l.o.g. that $\operatorname{gcd}(c, a)=1$. We look for a solution of the form + +$$ +x=p^{m}, \quad y=p^{n}, \quad z=q p^{r}, \quad p, q, m, n, r \in \mathbb{N} . +$$ + +Then $x^{a}+y^{b}=p^{m a}+p^{n b}$ and $z^{c}=q^{c} p^{r c}$, and we see that it is enough to assume $m a-1=n b=r c$ (there are infinitely many such triples $(m, n, r))$ and $q^{c}=p+1$. +7. Let us set $A C=a, C E=b, E A=c$. Applying Ptolemy's inequality for the quadrilateral $A C E F$ we get + +$$ +A C \cdot E F+C E \cdot A F \geq A E \cdot C F +$$ + +Since $E F=A F$, this implies $\frac{F A}{F C} \geq \frac{c}{a+b}$. Similarly $\frac{B C}{B E} \geq \frac{a}{b+c}$ and $\frac{D E}{D A} \geq$ $\frac{b}{c+a}$. Now, + +$$ +\frac{B C}{B E}+\frac{D E}{D A}+\frac{F A}{F C} \geq \frac{a}{b+c}+\frac{b}{c+a}+\frac{c}{a+b} +$$ + +Hence it is enough to prove that + +$$ +\frac{a}{b+c}+\frac{b}{c+a}+\frac{c}{a+b} \geq \frac{3}{2} +$$ + +If we now substitute $x=b+c, y=c+a, z=a+b$ and $S=a+b+c$ the inequality (1) becomes equivalent to $S(1 / x+1 / y+1 / y)-3 \geq 3 / 2$ which follows immediately form $1 / x+1 / y+1 / z \geq 9 /(x+y+z)=9 /(2 S)$. Equality occurs if it holds in Ptolemy's inequalities and also $a=b=c$. The former happens if and only if the hexagon is cyclic. Hence the only case of equality is when $A B C D E F$ is regular. +8. (a) Denote by $b$ and $c$ the perpendicular bisectors of $A B$ and $A C$ respectively. If w.l.o.g. $b$ and $A D$ do not intersect (are parallel), then $\angle B C D=\angle B A D=90^{\circ}$, a contradiction. Hence $V, W$ are well-defined. Now, $\angle D W B=2 \angle D A B$ and $\angle D V C=2 \angle D A C$ as oriented angles, and therefore $\angle(W B, V C)=2(\angle D V C-\angle D W B)=2 \angle B A C=$ $2 \angle B C D$ is not equal to 0 . Consequently $C V$ and $B W$ meet at some $T$ with $\angle B T C=2 \angle B A C$. +(b) Let $B^{\prime}$ be the second point of intersection of $B W$ with $\Gamma$. Clearly $A D=B B^{\prime}$. But we also have $\angle B T C=2 \angle B A C=2 \angle B B^{\prime} C$, which implies that $C T=T B^{\prime}$. It follows that $A D=B B^{\prime}=\left|B T \pm T B^{\prime}\right|=$ $|B T \pm C T|$. +Remark. This problem is also solved easily using trigonometry. +9. For $i=1,2,3$ (all indices in this problem will be modulo 3 ) we denote by $O_{i}$ the center of $C_{i}$ and by $M_{i}$ the midpoint of the $\operatorname{arc} A_{i+1} A_{i+2}$ that does not contain $A_{i}$. First we have that $O_{i+1} O_{i+2}$ is the perpendicular bisector of $I B_{i}$, and thus it contains the circumcenter $R_{i}$ of $A_{i} B_{i} I$. Additionally, it is easy to show that $T_{i+1} A_{i}=T_{i+1} I$ and $T_{i+2} A_{i}=$ $T_{i+2} I$, which implies that $R_{i}$ lies on the line $T_{i+1} T_{i+2}$. Therefore $R_{i}=$ $O_{i+1} O_{i+2} \cap T_{i+1} T_{i+2}$. +![](https://cdn.mathpix.com/cropped/2024_11_18_8e985d6b9c83aa3e9d0eg-631.jpg?height=360&width=548&top_left_y=579&top_left_x=806) + +Now, the lines $T_{1} O_{1}, T_{2} O_{2}, T_{3} O_{3}$ are concurrent at $I$. By Desargues's theorem, the points of intersection of $O_{i+1} O_{i+2}$ and $T_{i+1} T_{i+2}$, i.e., the $R_{i}$ 's, lie on a line for $i=1,2,3$. + +Second solution. The centers of three circles passing through the same point $I$ and not touching each other are collinear if and only if they have another common point. Hence it is enough to show that the circles $A_{i} B_{i} I$ have a common point other than $I$. +Now apply inversion at center $I$ and with an arbitrary power. We shall denote by $X^{\prime}$ the image of $X$ under this inversion. In our case, the image of the circle $C_{i}$ is the line $B_{i+1}^{\prime} B_{i+2}^{\prime}$ while the image of the line $A_{i+1} A_{i+2}$ is the circle $I A_{i+1}^{\prime} A_{i+2}^{\prime}$ that is tangent to $B_{i}^{\prime} B_{i+2}^{\prime}$, and $B_{i}^{\prime} B_{i+2}^{\prime}$. These three circles have equal radii, so their centers $P_{1}, P_{2}, P_{3}$ form a triangle also homothetic to $\triangle B_{1}^{\prime} B_{2}^{\prime} B_{3}^{\prime}$. Consequently, points $A_{1}^{\prime}, A_{2}^{\prime}, A_{3}^{\prime}$, that are the reflections of $I$ across the sides of $P_{1} P_{2} P_{3}$, are vertices of a triangle also homothetic to $B_{1}^{\prime} B_{2}^{\prime} B_{3}^{\prime}$. It follows that $A_{1}^{\prime} B_{1}^{\prime}, A_{2}^{\prime} B_{2}^{\prime}, A_{3}^{\prime} B_{3}^{\prime}$ are concurrent at some point $J^{\prime}$, i.e., that the circles $A_{i} B_{i} I$ all pass through $J$. +10. Suppose that $k \geq 4$. Consider any polynomial $F(x)$ with integer coefficients such that $0 \leq F(x) \leq k$ for $x=0,1, \ldots, k+1$. Since $F(k+1)-F(0)$ is divisible by $k+1$, we must have $F(k+1)=F(0)$. Hence + +$$ +F(x)-F(0)=x(x-k-1) Q(x) +$$ + +for some polynomial $Q(x)$ with integer coefficients. In particular, $F(x)-$ $F(0)$ is divisible by $x(k+1-x)>k+1$ for every $x=2,3, \ldots, k-1$, so $F(x)=F(0)$ must hold for any $x=2,3, \ldots, k-1$. It follows that + +$$ +F(x)-F(0)=x(x-2)(x-3) \cdots(x-k+1)(x-k-1) R(x) +$$ + +for some polynomial $R(x)$ with integer coefficients. Thus $k \geq \mid F(1)-$ $F(0)|=k(k-2)!| R(1) \mid$, although $k(k-2)!>k$ for $k \geq 4$. In this case we have $F(1)=F(0)$ and similarly $F(k)=F(0)$. Hence, the statement is true for $k \geq 4$. +It is easy to find counterexamples for $k \leq 3$. These are, for example, + +$$ +F(x)= \begin{cases}x(2-x) & \text { for } k=1 \\ x(3-x) & \text { for } k=2 \\ x(2-x)^{2}(4-x) & \text { for } k=3\end{cases} +$$ + +11. All real roots of $P(x)$ (if any) are negative: say $-a_{1},-a_{2}, \ldots,-a_{k}$. Then $P(x)$ can be factored as + +$$ +P(x)=C\left(x+a_{1}\right) \cdots\left(x+a_{k}\right)\left(x^{2}-b_{1} x+c_{1}\right) \cdots\left(x^{2}-b_{m} x+c_{m}\right) +$$ + +where $x^{2}-b_{i} x+c_{i}$ are quadratic polynomials without real roots. +Since the product of polynomials with positive coefficients is again a polynomial with positive coefficients, it will be sufficient to prove the result for each of the factors in (1). The case of $x+a_{j}$ is trivial. It remains only to prove the claim for every polynomial $x^{2}-b x+c$ with $b^{2}<4 c$. +From the binomial formula, we have for any $n \in \mathbb{N}$, + +$$ +(1+x)^{n}\left(x^{2}-b x+c\right)=\sum_{i=0}^{n+2}\left[\binom{n}{i-2}-b\binom{n}{i-1}+c\binom{n}{i}\right] x^{i}=\sum_{i=0}^{n+2} C_{i} x^{i} +$$ + +where + +$$ +C_{i}=\frac{n!\left((b+c+1) i^{2}-((b+2 c) n+(2 b+3 c+1)) i+c\left(n^{2}+3 n+2\right)\right) x^{i}}{i!(n-i+2)!} +$$ + +The coefficients $C_{i}$ of $x^{i}$ appear in the form of a quadratic polynomial in $i$ depending on $n$. We claim that for large enough $n$ this polynomial has negative discriminant, and is thus positive for every $i$. Indeed, this discriminant equals $D=((b+2 c) n+(2 b+3 c+1))^{2}-4(b+c+1) c\left(n^{2}+\right.$ $3 n+2)=\left(b^{2}-4 c\right) n^{2}-2 U n+V$, where $U=2 b^{2}+b c+b-4 c$ and $V=(2 b+c+1)^{2}-4 c$, and since $b^{2}-4 c<0$, for large $n$ it clearly holds that $D<0$. +12. Lemma. For any polynomial $P$ of degree at most $n$, the following equality holds: + +$$ +\sum_{i=0}^{n+1}(-1)^{i}\binom{n+1}{i} P(i)=0 +$$ + +Proof. See (SL81-13). +Suppose to the contrary that the degree of $f$ is at most $p-2$. Then it follows from the lemma that + +$$ +0=\sum_{i=0}^{p-1}(-1)^{i}\binom{p-1}{i} f(i) \equiv \sum_{i=0}^{p-1} f(i)(\bmod p) +$$ + +since $\binom{p-1}{i}=\frac{(p-1)(p-2) \cdots(p-i)}{i!} \equiv(-1)^{i}(\bmod p)$. But this is clearly impossible if $f(i)$ equals 0 or 1 modulo $p$ and $f(0)=0, f(1)=1$. +Remark. In proving the essential relation $\sum_{i=0}^{p-1} f(i) \equiv 0(\bmod p)$, it is clearly enough to show that $S_{k}=1^{k}+2^{k}+\cdots+(p-1)^{k}$ is divisible by $p$ for every $k \leq p-2$. This can be shown in two other ways. +(1) By induction. Assume that $S_{0} \equiv \cdots \equiv S_{k-1}(\bmod p)$. By the binomial formula we have + +$$ +0 \equiv \sum_{n=0}^{p-1}\left[(n+1)^{k+1}-n^{k+1}\right] \equiv(k+1) S_{k}+\sum_{i=0}^{k-1}\binom{k+1}{i} S_{i}(\bmod p), +$$ + +and the inductive step follows. +(2) Using the primitive root $g$ modulo $p$. Then + +$$ +S_{k} \equiv 1+g^{k}+\cdots+g^{k(p-2)}=\frac{g^{k(p-1)}-1}{g^{k}-1} \equiv 0(\bmod p) . +$$ + +13. Denote $A(r)$ and $B(r)$ by $A(n, r)$ and $B(n, r)$ respectively. The numbers $A(n, r)$ can be found directly: one can choose $r$ girls and $r$ boys in $\binom{n}{r}^{2}$ ways, and pair them in $r$ ! ways. Hence + +$$ +A(n, r)=\binom{n}{r}^{2} \cdot r!=\frac{n!^{2}}{(n-r)!^{2} r!} +$$ + +Now we establish a recurrence relation between the $B(n, r)$ 's. Let $n \geq 2$ and $2 \leq r \leq n$. There are two cases for a desired selection of $r$ pairs of girls and boys: +(i) One of the girls dancing is $g_{n}$. Then the other $r-1$ girls can choose their partners in $B(n-1, r-1)$ ways and $g_{n}$ can choose any of the remaining $2 n-r$ boys. Thus, the total number of choices in this case is $(2 n-r) B(n-1, r-1)$. +(ii) $g_{n}$ is not dancing. Then there are exactly $B(n-1, r)$ possible choices. Therefore, for every $n \geq 2$ it holds that + +$$ +B(n, r)=(2 n-r) B(n-1, r-1)+B(n-1, r) \quad \text { for } r=2, \ldots, n +$$ + +Here we assume that $B(n, r)=0$ for $r>n$, while $B(n, 1)=1+3+\cdots+$ $(2 n-1)=n^{2}$ 。 +It is directly verified that the numbers $A(n, r)$ satisfy the same initial conditions and recurrence relations, from which it follows that $A(n, r)=$ $B(n, r)$ for all $n$ and $r \leq n$. +14. We use the following nonstandard notation: ( $1^{\circ}$ ) for $x, y \in \mathbb{N}, x \sim y$ means that $x$ and $y$ have the same prime divisors; $\left(2^{\circ}\right)$ for a prime $p$ and integers $r \geq 0$ and $x>0, p^{r} \| x$ means that $x$ is divisible by $p^{r}$, but not by $p^{r+1}$. First, $b^{m}-1 \sim b^{n}-1$ is obviously equivalent to $b^{m}-1 \sim \operatorname{gcd}\left(b^{m}-1, b^{n}-\right.$ $1)=b^{d}-1$, where $d=\operatorname{gcd}(m, n)$. Setting $b^{d}=a$ and $m=k d$, we reduce +the condition of the problem to $a^{k}-1 \sim a-1$. We are going to show that this implies that $a+1$ is a power of 2 . This will imply that $d$ is odd (for even $d$, $a+1=b^{d}+1$ cannot be divisible by 4 , and consequently $b+1$, as a divisor of $a+1$, is also a power of 2 . But before that, we need the following important lemma (Theorem 2.126). +Lemma. Let $a, k$ be positive integers and $p$ an odd prime. If $\alpha \geq 1$ and $\beta \geq 0$ are such that $p^{\alpha} \| a-1$ and $p^{\beta} \| k$, then $p^{\alpha+\beta} \| a^{k}-1$. +Proof. We use induction on $\beta$. If $\beta=0$, then $\frac{a^{k}-1}{a-1}=a^{k-1}+\cdots+a+1 \equiv k$ $(\bmod p)($ because $a \equiv 1$ ), and it is not divisible by $p$. +Suppose that the lemma is true for some $\beta \geq 0$, and let $k=p^{\beta+1} t$ where $p \nmid t$. By the induction hypothesis, $a^{k / p}=a^{p^{\beta} t}=m p^{\alpha+\beta}+1$ for some $m$ not divisible by $p$. Furthermore, + +$$ +a^{k}-1=\left(m p^{\alpha+\beta}+1\right)^{p}-1=\left(m p^{\alpha+\beta}\right)^{p}+\cdots+\binom{p}{2}\left(m p^{\alpha+\beta}\right)^{2}+m p^{\alpha+\beta+1} +$$ + +Since $p \left\lvert\,\binom{ p}{2}=\frac{p(p-1)}{2}\right.$, all summands except for the last one are divisible by $p^{\alpha+\beta+2}$. Hence $p^{\alpha+\beta+1} \| a^{k}-1$, completing the induction. Now let $a^{k}-1 \sim a-1$ for some $a, k>1$. Suppose that $p$ is an odd prime divisor of $k$, with $p^{\beta} \| k$. Then putting $X=a^{p^{\beta}-1}+\cdots+a+1$ we also have $(a-1) X=a^{p^{\beta}}-1 \sim a-1$; hence each prime divisor $q$ of $X$ must also divide $a-1$. But then $a^{i} \equiv 1(\bmod q)$ for each $i \in \mathbb{N}_{0}$, which gives us $X \equiv p^{\beta}(\bmod q)$. Therefore $q \mid p^{\beta}$, i.e., $q=p$; hence $X$ is a power of $p$. On the other hand, since $p \mid a-1$, we put $p^{\alpha} \| a-1$. From the lemma we obtain $p^{\alpha+\beta} \| a^{p^{\beta}}-1$, and deduce that $p^{\beta} \| X$. But $X$ has no prime divisors other than $p$, so we must have $X=p^{\beta}$. This is clearly impossible, because $X>p^{\beta}$ for $a>1$. Thus our assumption that $k$ has an odd prime divisor leads to a contradiction: in other words, $k$ must be a power of 2 . Now $a^{k}-1 \sim a-1$ implies $a-1 \sim a^{2}-1=(a-1)(a+1)$, and thus every prime divisor $q$ of $a+1$ must also divide $a-1$. Consequently $q=2$, so it follows that $a+1$ is a power of 2 . As we explained above, this gives that $b+1$ is also a power of 2 . +Remark. In fact, one can continue and show that $k$ must be equal to 2 . It is not possible for $a^{4}-1 \sim a^{2}-1$ to hold. Similarly, we must have $d=1$. Therefore all possible triples $(b, m, n)$ with $m>n$ are $\left(2^{s}-1,2,1\right)$. +15. Let $a+b t, t=0,1,2, \ldots$, be a given arithmetic progression that contains a square and a cube $(a, b>0)$. We use induction on the progression step $b$ to prove that the progression contains a sixth power. +(i) $b=1$ : this case is trivial. +(ii) $b=p^{m}$ for some prime $p$ and $m>0$. The case $p^{m} \mid a$ trivially reduces to the previous case, so let us have $p^{m} \nmid a$. +Suppose that $\operatorname{gcd}(a, p)=1$. If $x, y$ are integers such that $x^{2} \equiv y^{3} \equiv a$ (here all the congruences will be $\bmod p^{m}$ ), then $x^{6} \equiv a^{3}$ and $y^{6} \equiv a^{2}$. Consider an integer $y_{1}$ such that $y y_{1} \equiv 1$. It satisfies $a^{2}\left(x y_{1}\right)^{6} \equiv$ +$x^{6} y^{6} y_{1}^{6} \equiv x^{6} \equiv a^{3}$, and consequently $\left(x y_{1}\right)^{6} \equiv a$. Hence a sixth power exists in the progression. +If $\operatorname{gcd}(a, p)>1$, we can write $a=p^{k} c$, where $k1$ and $\operatorname{gcd}\left(b_{1}, b_{2}\right)=1$. It is given that progressions $a+b_{1} t$ and $a+b_{2} t$ both contain a square and a cube, and therefore by the inductive hypothesis they both contain sixth powers: say $z_{1}^{6}$ and $z_{2}^{6}$, respectively. By the Chinese remainder theorem, there exists $z \in \mathbb{N}$ such that $z \equiv z_{1}\left(\bmod b_{1}\right)$ and $z \equiv z_{2}\left(\bmod b_{2}\right)$. But then $z^{6}$ belongs to both of the progressions $a+b_{1} t$ and $a+b_{2} t$. Hence $z^{6}$ is a member of the progression $a+b t$. +16. Let $d_{a}(X), d_{b}(X), d_{c}(X)$ denote the distances of a point $X$ interior to $\triangle A B C$ from the lines $B C, C A, A B$ respectively. We claim that $X \in P Q$ if and only if $d_{a}(X)+d_{b}(X)=d_{c}(X)$. Indeed, if $X \in P Q$ and $P X=$ $k P Q$ then $d_{a}(X)=k d_{a}(Q), d_{b}(X)=(1-k) d_{b}(P)$, and $d_{c}(X)=(1-$ $k) d_{c}(P)+k d_{c}(Q)$, and simple substitution yields $d_{a}(X)+d_{b}(X)=d_{c}(X)$. The converse follows easily. In particular, $O \in P Q$ if and only if $d_{a}(O)+$ $d_{b}(O)=d_{c}(O)$, i.e., $\cos \alpha+\cos \beta=\cos \gamma$. +We shall now show that $I \in D E$ if and only if $A E+B D=D E$. Let $K$ be the point on the segment $D E$ such that $A E=E K$. Then $\angle E K A=$ $\frac{1}{2} \angle D E C=\frac{1}{2} \angle C B A=\angle I B A$; hence the points $A, B, I, K$ are concyclic. The point $I$ lies on $D E$ if and only if $\angle B K D=\angle B A I=\frac{1}{2} \angle B A C=$ $\frac{1}{2} \angle C D E=\angle D B K$, which is equivalent to $K D=B D$, i.e., to $A E+B D=$ $D E$. But since $A E=A B \cos \alpha, B D=A B \cos \beta$, and $D E=A B \cos \gamma$, we have that $I \in D E \Leftrightarrow \cos \alpha+\cos \beta=\cos \gamma$. The conditions for $O \in P Q$ and $I \in D E$ are thus equivalent. + +Second solution. We know that three points $X, Y, Z$ are collinear if and only if for some $\lambda, \mu \in \mathbb{R}$ with sum 1 , we have $\lambda \overrightarrow{C X}+\mu \overrightarrow{C Y}=\overrightarrow{C Z}$. Specially, if $\overrightarrow{C X}=p \overrightarrow{C A}$ and $\overrightarrow{C Y}=q \overrightarrow{C B}$ for some $p, q$, and $\overrightarrow{C Z}=k \overrightarrow{C A}+$ $l \overrightarrow{C B}$, then $Z$ lies on $X Y$ if and only if $k q+l p=p q$. +Using known relations in a triangle we directly obtain + +$$ +\begin{array}{rlrl} +\overrightarrow{C P} & =\frac{\sin \beta}{\sin \beta+\sin \gamma} \overrightarrow{C B}, & \overrightarrow{C Q}=\frac{\sin \alpha}{\sin \alpha+\sin \gamma} \overrightarrow{C A} \\ +\overrightarrow{C O} & =\frac{\sin 2 \alpha \cdot \overrightarrow{C A}+\sin 2 \beta \cdot \overrightarrow{C B}}{\sin 2 \alpha+\sin 2 \beta+\sin 2 \gamma} ; & \overrightarrow{C D}=\frac{\tan \beta}{\tan \beta+\tan \gamma} \overrightarrow{C B} \\ +\overrightarrow{C E} & =\frac{\tan \beta}{\tan \beta+\tan \gamma} \overrightarrow{C A}, & \overrightarrow{C I} & =\frac{\sin \alpha \cdot \overrightarrow{C A}+\sin \beta \cdot \overrightarrow{C B}}{\sin \alpha+\sin \beta+\sin \gamma} +\end{array} +$$ + +Now by the above considerations we get that the conditions (1) $P, Q, O$ are collinear and (2) $D, E, I$ are collinear are both equivalent to $\cos \alpha+\cos \beta=$ $\cos \gamma$. +17. We note first that $x$ and $y$ must be powers of the same positive integer. Indeed, if $x=p_{1}^{\alpha_{1}} \cdots p_{k}^{\alpha_{k}}$ and $y=p_{1}^{\beta_{1}} \cdots p_{k}^{\beta_{k}}$ (some of $\alpha_{i}$ and $\beta_{i}$ may be 0 , but not both for the same index $i$ ), then $x^{y^{2}}=y^{x}$ implies $\frac{\alpha_{i}}{\beta_{i}}=\frac{x}{y^{2}}=\frac{p}{q}$ for some $p, q>0$ with $\operatorname{gcd}(p, q)=1$, so for $a=p_{1}^{\alpha_{1} / p} \cdots p_{k}^{\alpha_{k} / p}$ we can take $x=a^{p}$ and $y=a^{q}$. +If $a=1$, then $(x, y)=(1,1)$ is the trivial solution. Let $a>1$. The given equation becomes $a^{p a^{2 q}}=a^{q a^{p}}$, which reduces to $p a^{2 q}=q a^{p}$. Hence $p \neq q$, so we distinguish two cases: +(i) $p>q$. Then from $a^{2 q}2 q$. We can rewrite the equation as $p=a^{p-2 q} q$, and putting $p=2 q+d, d>0$, we obtain $d=q\left(a^{d}-2\right)$. By induction, $2^{d}-2>d$ for each $d>2$, so we must have $d \leq 2$. For $d=1$ we get $q=1$ and $a=p=3$, and therefore $(x, y)=(27,3)$, which is indeed a solution. For $d=2$ we get $q=1$, $a=2$, and $p=4$, so $(x, y)=(16,2)$, which is another solution. +(ii) $p0$, this is transformed to $a^{d}=a^{\left(2 a^{d}-1\right) p}$, or equivalently to $d=\left(2 a^{d}-1\right) p$. However, this equality cannot hold, because $2 a^{d}-1>d$ for each $a \geq 2$, $d \geq 1$. +The only solutions are thus $(1,1),(16,2)$, and $(27,3)$. +18. By symmetry, assume that $A B>A C$. The point $D$ lies between $M$ and $P$ as well as between $Q$ and $R$, and if we show that $D M \cdot D P=D Q \cdot D R$, it will imply that $M, P, Q, R$ lie on a circle. +Since the triangles $A B C, A E F, A Q R$ are similar, the points $B, C, Q, R$ lie on a circle. Hence $D B \cdot D C=D Q \cdot D R$, and it remains to prove that + +$$ +D B \cdot D C=D M \cdot D P +$$ + +However, the points $B, C, E, F$ are concyclic, but so are the points $E, F, D, M$ (they lie on the nine-point circle), and we obtain $P B \cdot P C=$ $P E \cdot P F=P D \cdot P M$. Set $P B=x$ and $P C=y$. We have $P M=\frac{x+y}{2}$ and hence $P D=\frac{2 x y}{x+y}$. It follows that $D B=P B-P D=\frac{x(x-y)}{x+y}$, $D C=\frac{y(x-y)}{x+y}$, and $D M=\frac{(x-y)^{2}}{2(x+y)}$, from which we immediately obtain $D B \cdot D C=D M \cdot D P=\frac{x y(x-y)^{2}}{(x+y)^{2}}$, as needed. +19. Using that $a_{n+1}=0$ we can transform the desired inequality into + +$$ +\begin{aligned} +& \sqrt{a_{1}+}+a_{2}+\cdots+a_{n+1} \\ +& \quad \leq \sqrt{1} \sqrt{a_{1}}+(\sqrt{2}-\sqrt{1}) \sqrt{a_{2}}+\cdots+(\sqrt{n+1}-\sqrt{n}) \sqrt{a_{n+1}} +\end{aligned} +$$ + +We shall prove by induction on $n$ that (1) holds for any $a_{1} \geq a_{2} \geq \cdots \geq$ $a_{n+1} \geq 0$, i.e., not only when $a_{n+1}=0$. For $n=0$ the inequality is +obvious. For the inductive step from $n-1$ to $n$, where $n \geq 1$, we need to prove the inequality + +$$ +\sqrt{a_{1}+\cdots+a_{n+1}}-\sqrt{a_{1}+\cdots+a_{n}} \leq(\sqrt{n+1}-\sqrt{n}) \sqrt{a_{n+1}} +$$ + +Putting $S=a_{1}+a_{2}+\cdots+a_{n}$, this simplifies to $\sqrt{S+a_{n+1}}-\sqrt{S} \leq$ $\sqrt{n a_{n+1}+a_{n+1}}-\sqrt{n a_{n+1}}$. For $a_{n+1}=0$ the inequality is obvious. For $a_{n+1}>0$ we have that the function $f(x)=\sqrt{x+a_{n+1}}-\sqrt{x}=$ $\frac{a_{n+1}}{\sqrt{x+a_{n+1}}+\sqrt{x}}$ is strictly decreasing on $\mathbb{R}^{+}$; hence (2) will follow if we show that $S \geq n a_{n+1}$. However, this last is true because $a_{1}, \ldots, a_{n} \geq a_{n+1}$. +Equality holds if and only if $a_{1}=a_{2}=\cdots=a_{k}$ and $a_{k+1}=\cdots=a_{n+1}=$ 0 for some $k$. + +Second solution. Setting $b_{k}=\sqrt{a_{k}}-\sqrt{a_{k+1}}$ for $k=1, \ldots, n$ we have $a_{i}=\left(b_{i}+\cdots+b_{n}\right)^{2}$, so the desired inequality after squaring becomes + +$$ +\sum_{k=1}^{n} k b_{k}^{2}+2 \sum_{1 \leq k\frac{n+1}{2}$ for any $\pi$. +Further, we note that if $\pi^{\prime}$ is obtained from $\pi$ by interchanging two neighboring elements, say $y_{k}$ and $y_{k+1}$, then $S(\pi)$ and $S\left(\pi^{\prime}\right)$ differ by $\left|y_{k}+y_{k+1}\right| \leq n+1$, and consequently they must be of the same sign. +Now consider the identity permutation $\pi_{0}=\left(x_{1}, \ldots, x_{n}\right)$ and the reverse permutation $\overline{\pi_{0}}=\left(x_{n}, \ldots, x_{1}\right)$. There is a sequence of permutations $\pi_{0}, \pi_{1}, \ldots, \pi_{m}=\overline{\pi_{0}}$ such that for each $i, \pi_{i+1}$ is obtained from $\pi_{i}$ by interchanging two neighboring elements. Indeed, by successive interchanges we can put $x_{n}$ in the first place, then $x_{n-1}$ in the second place, etc. Hence all $S\left(\pi_{0}\right), \ldots, S\left(\pi_{m}\right)$ are of the same sign. However, since $\left|S\left(\pi_{0}\right)+S\left(\pi_{m}\right)\right|=(n+1)\left|x_{1}+\cdots+x_{n}\right|=n+1$, this implies that one of +$S\left(\pi_{0}\right)$ and $S\left(\overline{\pi_{0}}\right)$ is smaller than $\frac{n+1}{2}$ in absolute value, contradicting the initial assumption. +22. (a) Suppose that $f$ and $g$ are such functions. From $g(f(x))=x^{3}$ we have $f\left(x_{1}\right) \neq f\left(x_{2}\right)$ whenever $x_{1} \neq x_{2}$. In particular, $f(-1), f(0)$, and $f(1)$ are three distinct numbers. However, since $f(x)^{2}=f(g(f(x)))=$ $f\left(x^{3}\right)$, each of the numbers $f(-1), f(0), f(1)$ is equal to its square, and so must be either 0 or 1 . This contradiction shows that no such $f, g$ exist. +(b) The answer is yes. We begin with constructing functions $F, G:(1, \infty)$ $\rightarrow(1, \infty)$ with the property $F(G(x))=x^{2}$ and $G(F(x))=x^{4}$ for $x>$ 1. Define the functions $\varphi, \psi$ by $F\left(2^{2^{t}}\right)=2^{2^{\varphi(t)}}$ and $G\left(2^{2^{t}}\right)=2^{2^{\psi(t)}}$. These functions determine $F$ and $G$ on the entire interval $(1, \infty)$, and satisfy $\varphi(\psi(t))=t+1$ and $\psi(\varphi(t))=t+2$. It is easy to find examples of $\varphi$ and $\psi$ : for example, $\varphi(t)=\frac{1}{2} t+1, \psi(t)=2 t$. Thus we also arrive at an example for $F, G$ : + +$$ +F(x)=2^{2^{\frac{1}{2} \log _{2} \log _{2} x+1}}=2^{2 \sqrt{\log _{2} x}}, \quad G(x)=2^{2^{2 \log _{2} \log _{2} x}}=2^{\log _{2}^{2} x} +$$ + +It remains only to extend these functions to the whole of $\mathbb{R}$. This can be done as follows: + +$$ +\widetilde{f}(x)= \begin{cases}F(x) & \text { for } x>1 \\ +1 / F(1 / x) & \text { for } 0 1 } \\ +{ x } & { \text { for } x \in \{ 0 , 1 \} ; } +\end{array} \left\{\begin{array}{cl} +1 / G(1 / x) & \text { for } 0\beta_{4}$. Then point $A$ lies inside the circle $B C D$, which is further equivalent to $\beta_{1}>\alpha_{2}$. On the other hand, from $\alpha_{1}+\beta_{2}=\alpha_{3}+\beta_{4}$ we deduce $\alpha_{3}>\beta_{2}$, and similarly $\beta_{3}>\alpha_{4}$. Therefore, +since the cosine is strictly decreasing on $(0, \pi)$, the left side of (1) is strictly negative, yielding a contradiction. +24. There is a bijective correspondence between representations in the given form of $2 k$ and $2 k+1$ for $k=0,1, \ldots$, since adding 1 to every representation of $2 k$, we obtain a representation of $2 k+1$, and conversely, every representation of $2 k+1$ contains at least one 1 , which can be removed. Hence, $f(2 k+1)=f(2 k)$. +Consider all representations of $2 k$. The number of those that contain at least one 1 equals $f(2 k-1)=f(2 k-2)$, while the number of those not containing a 1 equals $f(k)$ (the correspondence is given by division of summands by 2). Therefore + +$$ +f(2 k)=f(2 k-2)+f(k) . +$$ + +Summing these equalities over $k=1, \ldots, n$, we obtain + +$$ +f(2 n)=f(0)+f(1)+\cdots+f(n) +$$ + +We first prove the right-hand inequality. Since $f$ is increasing, and $f(0)+$ $f(1)=f(2)$, (2) yields $f(2 n) \leq n f(n)$ for $n \geq 2$. Now $f\left(2^{3}\right)=f(0)+$ $\cdots+f(4)=10<2^{3^{2} / 2}$, and one can easily conclude by induction that $f\left(2^{n+1}\right) \leq 2^{n} f\left(2^{n}\right)<2^{n} \cdot 2^{n^{2} / 2}<2^{(n+1)^{2} / 2}$ for each $n \geq 3$. +We now derive the lower estimate. It follows from (1) that $f(x+2)-f(x)$ is increasing. Consequently, for each $m$ and $k(2 m-1) f(2 m)$, which together with the above inequality gives + +$$ +f(8 m)=f(0)+f(1)+\cdots+f(4 m)>4 m f(2 m) . +$$ + +Finally, we have that the inequality $f\left(2^{n}\right)>2^{n^{2} / 4}$ holds for $n=2$ and $n=3$, while for larger $n$ we have by induction $f\left(2^{n}\right)>2^{n-1} f\left(2^{n-2}\right)>$ $2^{n-1+(n-2)^{2} / 4}=2^{n^{2} / 4}$. This completes the proof. +Remark. Despite the fact that the lower estimate is more difficult, it is much weaker than the upper estimate. It can be shown that $f\left(2^{n}\right)$ eventually (for large $n$ ) exceeds $2^{c n^{2}}$ for any $c<\frac{1}{2}$. +25. Let $M R$ meet the circumcircle of triangle $A B C$ again at a point $X$. We claim that $X$ is the common point of the lines $K P, L Q, M R$. By symmetry, it will be enough to show that $X$ lies on $K P$. It is easy to see that $X$ and $P$ lie on the same side of $A B$ as $K$. Let $I_{a}=A K \cap B P$ be the excenter of $\triangle A B C$ corresponding to $A$. It is easy to calculate that $\angle A I_{a} B=\gamma / 2$, from which we get $\angle R P B=\angle A I_{a} B=\angle M C B=\angle R X B$. Therefore $R, B, P, X$ are concyclic. Now if $P$ and $K$ are on distinct sides of $B X$ (the +other case is similar), we have $\angle R X P=180^{\circ}-\angle R B P=90^{\circ}-$ $\beta / 2=\angle M A K=180^{\circ}-\angle R X K$, from which it follows that $K, X, P$ are collinear, as claimed. +Remark. It is not essential for the statement of the problem that $R$ be an internal point of $A B$. Work with cases can be avoided using oriented +![](https://cdn.mathpix.com/cropped/2024_11_18_8e985d6b9c83aa3e9d0eg-640.jpg?height=399&width=439&top_left_y=248&top_left_x=858) +angles. +26. Let us first examine the case that all the inequalities in the problem are actually equalities. Then $a_{n-2}=a_{n-1}+a_{n}, a_{n-3}=2 a_{n-1}+a_{n}, \ldots, a_{0}=$ $F_{n} a_{n-1}+F_{n-1} a_{n}=1$, where $F_{n}$ is the $n$th Fibonacci number. Then it is easy to see (from $F_{1}+F_{2}+\cdots+F_{k}=F_{k+2}$ ) that $a_{0}+\cdots+a_{n}=$ $\left(F_{n+2}-1\right) a_{n-1}+F_{n+1} a_{n}=\frac{F_{n+2}-1}{F_{n}}+\left(F_{n+1}-\frac{F_{n-1}\left(F_{n+2}-1\right)}{F_{n}}\right) a_{n}$. Since $\frac{F_{n-1}\left(F_{n+2}-1\right)}{F_{n}} \leq F_{n+1}$, it follows that $a_{0}+a_{1}+\cdots+a_{n} \geq \frac{F_{n+2}-1}{F_{n}}$, with equality holding if and only if $a_{n}=0$ and $a_{n-1}=\frac{1}{F_{n}}$. +We denote by $M_{n}$ the required minimum in the general case. We shall prove by induction that $M_{n}=\frac{F_{n+2}-1}{F_{n}}$. For $M_{1}=1$ and $M_{2}=2$ it is easy to show that the formula holds; hence the inductive basis is true. Suppose that $n>2$. The sequences $1, \frac{a_{2}}{a_{1}}, \ldots, \frac{a_{n}}{a_{1}}$ and $1, \frac{a_{3}}{a_{2}}, \ldots, \frac{a_{n}}{a_{2}}$ also satisfy the conditions of the problem. Hence we have + +$$ +a_{0}+\cdots+a_{n}=a_{0}+a_{1}\left(1+\frac{a_{2}}{a_{1}}+\cdots+\frac{a_{n}}{a_{1}}\right) \geq 1+a_{1} M_{n-1} +$$ + +and + +$$ +a_{0}+\cdots+a_{n}=a_{0}+a_{1}+a_{2}\left(1+\frac{a_{3}}{a_{2}}+\cdots+\frac{a_{n}}{a_{2}}\right) \geq 1+a_{1}+a_{2} M_{n-2} +$$ + +Multiplying the first inequality by $M_{n-2}-1$ and the second one by $M_{n-1}$, adding the inequalities and using that $a_{1}+a_{2} \geq 1$, we obtain ( $M_{n-1}+$ $\left.M_{n-2}+1\right)\left(a_{0}+\cdots+a_{n}\right) \geq M_{n-1} M_{n-2}+M_{n-1}+M_{n-2}+1$, so + +$$ +M_{n} \geq \frac{M_{n-1} M_{n-2}+M_{n-1}+M_{n-2}+1}{M_{n-1}+M_{n-2}+1} +$$ + +Since $M_{n-1}=\frac{F_{n+1}-1}{F_{n-1}}$ and $M_{n-2}=\frac{F_{n}-1}{F_{n-2}}$, the above inequality easily yields $M_{n} \geq \frac{F_{n+2}-1}{F_{n}}$. However, we have shown above that equality can occur; hence $\frac{F_{n+2}-1}{F_{n}}$ is indeed the required minimum. + +### 4.39 Solutions to the Shortlisted Problems of IMO 1998 + +1. We begin with the following observation: Suppose that $P$ lies in $\triangle A E B$, where $E$ is the intersection of $A C$ and $B D$ (the other cases are similar). Let $M, N$ be the feet of the perpendiculars from $P$ to $A C$ and $B D$ respectively. We have $S_{A B P}=S_{A B E}-S_{A E P}-S_{B E P}=\frac{1}{2}(A E \cdot B E-A E \cdot E N-B E$. $E M)=\frac{1}{2}(A M \cdot B N-E M \cdot E N)$. Similarly, $S_{C D P}=\frac{1}{2}(C M \cdot D N-E M$. $E N)$. Therefore, we obtain + +$$ +S_{A B P}-S_{C D P}=\frac{A M \cdot B N-C M \cdot D N}{2} +$$ + +Now suppose that $A B C D$ is cyclic. Then $P$ is the circumcenter of $A B C D$; hence $M$ and $N$ are the midpoints of $A C$ and $B D$. Hence $A M=C M$ and $B N=D N$; thus (1) gives us $S_{A B P}=S_{C D P}$. On the other hand, suppose that +![](https://cdn.mathpix.com/cropped/2024_11_18_8e985d6b9c83aa3e9d0eg-641.jpg?height=311&width=410&top_left_y=703&top_left_x=875) +$A B C D$ is not cyclic and let w.l.o.g. $P A=P B>P C=P D$. Then we must have $A M>C M$ and $B N>$ $D N$, and consequently by (1), $S_{A B P}>S_{C D P}$. This proves the other implication. + +Second solution. Let $F$ and $G$ denote the midpoints of $A B$ and $C D$, and assume that $P$ is on the same side of $F G$ as $B$ and $C$. Since $P F \perp A B$, $P G \perp C D$, and $\angle F E B=\angle A B E, \angle G E C=\angle D C E$, a direct computation yields $\angle F P G=\angle F E G=90^{\circ}+\angle A B E+\angle D C E$. +Taking into account that $S_{A B P}=\frac{1}{2} A B \cdot F P=F E \cdot F P$, we note that $S_{A B P}=S_{C D P}$ is equivalent to $F E \cdot F P=G E \cdot G P$, i.e., to $F E / E G=$ $G P / P F$. But this last is equivalent to triangles $E F G$ and $P G F$ being similar, which holds if and only if $E F P G$ is a parallelogram. This last is equivalent to $\angle E F P=\angle E G P$, or $2 \angle A B E=2 \angle D C E$. Thus $S_{A B P}=$ $S_{C D P}$ is equivalent to $A B C D$ being cyclic. +Remark. The problems also allows an analytic solution, for example putting the $x$ and $y$ axes along the diagonals $A C$ and $B D$. +2. If $A D$ and $B C$ are parallel, then $A B C D$ is an isosceles trapezoid with $A B=C D$, so $P$ is the midpoint of $E F$. Let $M$ and $N$ be the midpoints of $A B$ and $C D$. Then $M N \| B C$, and the distance $d(E, M N)$ equals the distance $d(F, M N)$ because $B$ and $D$ are the same distance from $M N$ and $E M / B M=F N / D N$. It follows that the midpoint $P$ of $E F$ lies on $M N$, and consequently $S_{A P D}: S_{B P C}=A D: B C$. +If $A D$ and $B C$ are not parallel, then they meet at some point $Q$. It is plain that $\triangle Q A B \sim \triangle Q C D$, and since $A E / A B=C F / C D$, we also deduce that $\triangle Q A E \sim \triangle Q C F$. Therefore $\angle A Q E=\angle C Q F$. Further, from these similarities we obtain $Q E / Q F=Q A / Q C=A B / C D=P E / P F$, +which in turn means that $Q P$ is the internal bisector of $\angle E Q F$. But since $\angle A Q E=\angle C Q F$, this is also the internal bisector of $\angle A Q B$. Hence $P$ is at equal distances from $A D$ and $B C$, so again $S_{A P D}: S_{B P C}=A D: B C$. Remark. The part $A B \| C D$ could also be regarded as a limiting case of the other part. +Second solution. Denote $\lambda=\frac{A E}{A B}, A B=a, B C=b, C D=c, D A=d$, $\angle D A B=\alpha, \angle A B C=\beta$. Since $d(P, A D)=\frac{c \cdot d(E, A D)+a \cdot d(F, A D)}{a+c}$, we have $S_{A P D}=\frac{c S_{E A D}+a S_{F A D}}{a+c}=\frac{\lambda c S_{A B D}+(1-\lambda) a S_{A C D}}{a+c}$. Since $S_{A B D}=\frac{1}{2} a d \sin \alpha$ and $S_{A C D}=\frac{1}{2} c d \sin \beta$, we are led to $S_{A P D}=\frac{a c d}{a+c}[\lambda \sin \alpha+(1-\lambda) \sin \beta]$, and analogously $S_{B P C}=\frac{a b c}{a+c}[\lambda \sin \alpha+(1-\lambda) \sin \beta]$. Thus we obtain $S_{A P D}: S_{B P C}=d: b$. +3. Lemma. If $U, W, V$ are three points on a line $l$ in this order, and $X$ a point in the plane with $X W \perp U V$, then $\angle U X V<90^{\circ}$ if and only if $X W^{2}>U W \cdot V W$. +Proof. Let $X W^{2}>U W \cdot V W$, and let $X_{0}$ be a point on the segment $X W$ such that $X_{0} W^{2} \geq U W \cdot V W$. Then $X_{0} W / U W=V W / X_{0} W$, so that triangles $X_{0} W U$ and $V W X_{0}$ are similar. Thus $\angle U X_{0} V=\angle U X_{0} W+$ $\angle W U X_{0}=90^{\circ}$, which immediately implies that $\angle U X V<90^{\circ}$. +Similarly, if $X W^{2} \leq U W \cdot V W$, then $\angle U X V \geq 90^{\circ}$. +Since $B I \perp R S$, it will be enough by the lemma to show that $B I^{2}>$ $B R \cdot B S$. Note that $\triangle B K R \sim \triangle B S L$ : in fact, we have $\angle K B R=\angle S B L=$ $90^{\circ}-\beta / 2$ and $\angle B K R=\angle A K M=\angle K L M=\angle B S L=90^{\circ}-\alpha / 2$. In particular, we obtain $B R / B K=B L / B S=B K / B S$, so that $B R \cdot B S=$ $B K^{2}B K$ and $B M>B L$. We conclude that $\angle M B E<\frac{1}{2} \angle M B K$ and $\angle M B F<\frac{1}{2} \angle M B L$. Adding these two inequalities gives $\angle E B F<$ $\beta / 2$. Therefore $\angle R I S<90^{\circ}$. +Remark. It can be shown (using vectors) that the statement remains true for an arbitrary line $t$ passing through $B$. +4. Let $K$ be the point on the ray $B N$ with $\angle B C K=\angle B M A$. Since $\angle K B C=\angle A B M$, we get $\triangle B C K \sim \triangle B M A$. It follows that $B C / B M=$ $B K / B A$, which implies that also $\triangle B A K \sim \triangle B M C$. The quadrilateral $A N C K$ is cyclic, because $\angle B K C=\angle B A M=\angle N A C$. Then by Ptolemy's theorem we obtain + +$$ +A C \cdot B K=A C \cdot B N+A N \cdot C K+C N \cdot A K +$$ + +On the other hand, from the similarities noted above we get + +$$ +C K=\frac{B C \cdot A M}{B M}, A K=\frac{A B \cdot C M}{B M} \text { and } B K=\frac{A B \cdot B C}{B M} . +$$ + +After substitution of these values, the equality (1) becomes + +$$ +\frac{A B \cdot B C \cdot A C}{B M}=A C \cdot B N+\frac{B C \cdot A M \cdot A N}{B M}+\frac{A B \cdot C M \cdot C N}{B M}, +$$ + +which is exactly the equality we must prove multiplied by $\frac{A B \cdot B C \cdot C A}{B M}$. +5. Let $G$ be the centroid of $\triangle A B C$ and $\mathcal{H}$ the homothety with center $G$ and ratio $-\frac{1}{2}$. It is well-known that $\mathcal{H}$ maps $H$ into $O$. For every other point $X$, let us denote by $X^{\prime}$ its image under $\mathcal{H}$. Also, let $A_{2} B_{2} C_{2}$ be the triangle in which $A, B, C$ are the midpoints of $B_{2} C_{2}, C_{2} A_{2}$, and $A_{2} B_{2}$, respectively. +It is clear that $A^{\prime}, B^{\prime}, C^{\prime}$ are the midpoints of sides $B C, C A, A B$ respectively. Furthermore, $D^{\prime}$ is the reflection of $A^{\prime}$ across $B^{\prime} C^{\prime}$. Thus $D^{\prime}$ must lie on $B_{2} C_{2}$ and $A^{\prime} D^{\prime} \perp$ +![](https://cdn.mathpix.com/cropped/2024_11_18_8e985d6b9c83aa3e9d0eg-643.jpg?height=484&width=552&top_left_y=612&top_left_x=806) +$B_{2} C_{2}$. However, it also holds that $O A^{\prime} \perp B_{2} C_{2}$, so we conclude that $O, D^{\prime}, A^{\prime}$ are collinear and $D^{\prime}$ is the projection of $O$ on $B_{2} C_{2}$. Analogously, $E^{\prime}, F^{\prime}$ are the projections of $O$ on $C_{2} A_{2}$ and $A_{2} B_{2}$. +Now we apply Simson's theorem. It claims that $D^{\prime}, E^{\prime}, F^{\prime}$ are collinear (which is equivalent to $D, E, F$ being collinear) if and only if $O$ lies on the circumcircle of $A_{2} B_{2} C_{2}$. However, this circumcircle is centered at $H$ with radius $2 R$, so the last condition is equivalent to $H O=2 R$. +6. Let $P$ be the point such that $\triangle C D P$ and $\triangle C B A$ are similar and equally oriented. Since then $\angle D C P=\angle B C A$ and $\frac{B C}{C A}=\frac{D C}{C P}$, it follows that $\angle A C P=\angle B C D$ and $\frac{A C}{C P}=\frac{B C}{C D}$, so $\triangle A C P \sim \triangle B C D$. In particular, $\frac{B C}{C A}=\frac{D B}{P A}$. +Furthermore, by the conditions of the problem we have $\angle E D P=360^{\circ}-$ $\angle B-\angle D=\angle F$ and $\frac{P D}{D E}=\frac{P D}{C D} \cdot \frac{C D}{D E}=\frac{A B}{B C} \cdot \frac{C D}{D E}=\frac{A F}{F E}$. Therefore $\triangle E D P \sim \triangle E F A$ as well, so that similarly as above we conclude that $\triangle A E P \sim \triangle F E D$ and consequently $\frac{A E}{E F}=\frac{P A}{F D}$. +Finally, $\frac{B C}{C A} \cdot \frac{A E}{E F} \cdot \frac{F D}{D B}=\frac{D B}{P A} \cdot \frac{P A}{F D} \cdot \frac{F D}{D B}=1$. +Second solution. Let $a, b, c, d, e, f$ be the complex coordinates of $A, B$, $C, D, E, F$, respectively. The condition of the problem implies that $\frac{a-b}{b-c}$. $\frac{c-d}{d-e} \cdot \frac{e-f}{f-a}=-1$. +On the other hand, since $(a-b)(c-d)(e-f)+(b-c)(d-e)(f-a)=$ $(b-c)(a-e)(f-d)+(c-a)(e-f)(d-b)$ holds identically, we immediately deduce that $\frac{b-c}{c-a} \cdot \frac{a-e}{e-f} \cdot \frac{f-d}{d-b}=-1$. Taking absolute values gives $\frac{B C}{C A} \cdot \frac{A E}{E F}$. $\frac{F D}{D B}=1$. +7. We shall use the following result. + +Lemma. In a triangle $A B C$ with $B C=a, C A=b$, and $A B=c$, +i. $\angle C=2 \angle B$ if and only if $c^{2}=b^{2}+a b$; +ii. $\angle C+180^{\circ}=2 \angle B$ if and only if $c^{2}=b^{2}-a b$. + +Proof. +i. Take a point $D$ on the extension of $B C$ over $C$ such that $C D=b$. The condition $\angle C=2 \angle B$ is equivalent to $\angle A D C=\frac{1}{2} \angle C=\angle B$, and thus to $A D=A B=c$. This is further equivalent to triangles $C A D$ and $A B D$ being similar, so $C A / A D=A B / B D$, i.e., $c^{2}=$ $b(a+b)$. +ii. Take a point $E$ on the ray $C B$ such that $C E=b$. As above, $\angle C+180^{\circ}=2 \angle B$ if and only if $\triangle C A E \sim \triangle A B E$, which is equivalent to $E B / B A=E A / A C$, or $c^{2}=b(b-a)$. +Let $F, G$ be points on the ray $C B$ such that $C F=\frac{1}{3} a$ and $C G=\frac{4}{3} a$. Set $B C=a, C A=b, A B=c, E C=b_{1}$, and $E B=c_{1}$. By the lemma it follows that $c^{2}=b^{2}+a b$. Also $b_{1}=A G$ and $c_{1}=A F$, so Stewart's theorem gives us $c_{1}^{2}=\frac{2}{3} b^{2}+\frac{1}{3} c^{2}-\frac{2}{9} a^{2}=b^{2}+\frac{1}{3} a b-\frac{2}{9} a^{2}$ and $b_{1}^{2}=$ $-\frac{1}{3} b^{2}+\frac{4}{3} c^{2}+\frac{4}{9} a^{2}=b^{2}+\frac{4}{3} a b+\frac{4}{9} a^{2}$. It follows that $b_{1}=\frac{2}{3} a+b$ and $c_{1}^{2}=b_{1}^{2}-\left(a b+\frac{2}{3} a^{2}\right)=b_{1}^{2}-a b_{1}$. The statement of the problem follows immediately by the lemma. +8. Let $M$ be the point of intersection of $A E$ and $B C$, and let $N$ be the point on $\omega$ diametrically opposite $A$. +Since $\angle B<\angle C$, points $N$ and $B$ are on the same side of $A E$. Furthermore, $\angle N A E=\angle B A X=$ $90^{\circ}-\angle A B E$; hence the triangles $N A E$ and $B A X$ are similar. Consequently, $\triangle B A Y$ and $\triangle N A M$ are also similar, since $M$ is the midpoint +![](https://cdn.mathpix.com/cropped/2024_11_18_8e985d6b9c83aa3e9d0eg-644.jpg?height=279&width=555&top_left_y=1171&top_left_x=802) +of $A E$. Thus $\angle A N Z=\angle A B Z=\angle A B Y=\angle A N M$, implying that $N, M, Z$ are collinear. Now we have $\angle Z M D=90^{\circ}-\angle Z M A=\angle E A Z=$ $\angle Z E D$ (the last equality because $E D$ is tangent to $\omega$ ); hence $Z M E D$ is a cyclic quadrilateral. It follows that $\angle Z D M=\angle Z E A=\angle Z A D$, which is enough to conclude that $M D$ is tangent to the circumcircle of $A Z D$. +Remark. The statement remains valid if $\angle B \geq \angle C$. +9. Set $a_{n+1}=1-\left(a_{1}+\cdots+a_{n}\right)$. Then $a_{n+1}>0$, and the desired inequality becomes + +$$ +\frac{a_{1} a_{2} \cdots a_{n+1}}{\left(1-a_{1}\right)\left(1-a_{2}\right) \cdots\left(1-a_{n+1}\right)} \leq \frac{1}{n^{n+1}} +$$ + +To prove it, we observe that +$1-a_{i}=a_{1}+\cdots+a_{i-1}+a_{i+1}+\cdots+a_{n+1} \geq n \sqrt[n]{a_{1} \cdots a_{i-1} a_{i+1} \cdots a_{n+1}}$. +Multiplying these inequalities for $i=1,2, \ldots, n+1$, we get exactly the inequality we need. +10. We shall first prove the inequality for $n$ of the form $2^{k}, k=0,1,2, \ldots$ The case $k=0$ is clear. For $k=1$, we have + +$$ +\frac{1}{r_{1}+1}+\frac{1}{r_{2}+1}-\frac{2}{\sqrt{r_{1} r_{2}}+1}=\frac{\left(\sqrt{r_{1} r_{2}}-1\right)\left(\sqrt{r_{1}}-\sqrt{r_{2}}\right)^{2}}{\left(r_{1}+1\right)\left(r_{2}+1\right)\left(\sqrt{r_{1} r_{2}}+1\right)} \geq 0 +$$ + +For the inductive step it suffices to show that the claim for $k$ and 2 implies that for $k+1$. Indeed, + +$$ +\begin{aligned} +\sum_{i=1}^{2^{k+1}} \frac{1}{r_{i}+1} & \geq \frac{2^{k}}{\sqrt[2^{k}]{r_{1} r_{2} \cdots r_{2^{k}}}+1}+\frac{2^{k}}{\sqrt[2^{k}]{r_{2^{k}+1^{\prime} r_{2^{k}}+2^{\cdots r_{2^{k+1}}}+1}}} \\ +& \geq \frac{2^{k+1}}{\sqrt[2^{k+1}]{r_{1} r_{2} \cdots r_{2^{k+1}}}+1} +\end{aligned} +$$ + +and the induction is complete. +We now show that if the statement holds for $2^{k}$, then it holds for every $n<2^{k}$ as well. Put $r_{n+1}=r_{n+2}=\cdots=r_{2^{k}}=\sqrt[n]{r_{1} r_{2} \ldots r_{n}}$. Then (1) becomes + +$$ +\frac{1}{r_{1}+1}+\cdots+\frac{1}{r_{n}+1}+\frac{2^{k}-n}{\sqrt[n]{r_{1} \cdots r_{n}}+1} \geq \frac{2^{k}}{\sqrt[n]{r_{1} \cdots r_{n}}+1} +$$ + +This proves the claim. +Second solution. Define $r_{i}=e^{x_{i}}$, where $x_{i}>0$. The function $f(x)=\frac{1}{1+e^{x}}$ is convex for $x>0$ : indeed, $f^{\prime \prime}(x)=\frac{e^{x}\left(e^{x}-1\right)}{\left(e^{x}+1\right)^{3}}>0$. Thus by Jensen's inequality applied to $f\left(x_{1}\right), \ldots, f\left(x_{n}\right)$, we get $\frac{1}{r_{1}+1}+\cdots+\frac{1}{r_{n}+1} \geq \frac{n}{\sqrt[n]{r_{1} \cdots r_{n}}+1}$. +11. The given inequality is equivalent to $x^{3}(x+1)+y^{3}(y+1)+z^{3}(z+1) \geq$ $\frac{3}{4}(x+1)(y+1)(z+1)$. By the A-G mean inequality, it will be enough to prove a stronger inequality: + +$$ +x^{4}+x^{3}+y^{4}+y^{3}+z^{4}+z^{3} \geq \frac{1}{4}\left[(x+1)^{3}+(y+1)^{3}+(z+1)^{3}\right] . +$$ + +If we set $S_{k}=x^{k}+y^{k}+z^{k}$, (1) takes the form $S_{4}+S_{3} \geq \frac{1}{4} S_{3}+\frac{3}{4} S_{2}+\frac{3}{4} S_{1}+\frac{3}{4}$. Note that by the A-G mean inequality, $S_{1}=x+y+z \geq 3$. Thus it suffices to prove the following: + +$$ +\text { If } S_{1} \geq 3 \text { and } m>n \text { are positive integers, then } S_{m} \geq S_{n} \text {. } +$$ + +This can be shown in many ways. For example, by Hölder's inequality, + +$$ +\left(x^{m}+y^{m}+z^{m}\right)^{n / m}(1+1+1)^{(m-n) / m} \geq x^{n}+y^{n}+z^{n} . +$$ + +(Another way is using the Chebyshev inequality: if $x \geq y \geq z$ then $x^{k-1} \geq$ $y^{k-1} \geq z^{k-1}$; hence $S_{k}=x \cdot x^{k-1}+y \cdot y^{k-1}+z \cdot z^{k-1} \geq \frac{1}{3} S_{1} S_{k-1}$, and the claim follows by induction.) + +Second solution. Assume that $x \geq y \geq z$. Then also $\frac{1}{(y+1)(z+1)} \geq$ $\frac{1}{(x+1)(z+1)} \geq \frac{1}{(x+1)(y+1)}$. Hence Chebyshev's inequality gives that + +$$ +\begin{aligned} +& \frac{x^{3}}{(1+y)(1+z)}+\frac{y^{3}}{(1+x)(1+z)}+\frac{z^{3}}{(1+x)(1+y)} \\ +\geq & \frac{1}{3} \frac{\left(x^{3}+y^{3}+z^{3}\right) \cdot(3+x+y+z)}{(1+x)(1+y)(1+z)} +\end{aligned} +$$ + +Now if we put $x+y+z=3 S$, we have $x^{3}+y^{3}+z^{3} \geq 3 S$ and $(1+$ $x)(1+y)(1+z) \leq(1+a)^{3}$ by the A-G mean inequality. Thus the needed inequality reduces to $\frac{6 S^{3}}{(1+S)^{3}} \geq \frac{3}{4}$, which is obviously true because $S \geq 1$. +Remark. Both these solutions use only that $x+y+z \geq 3$. +12. The assertion is clear for $n=0$. We shall prove the general case by induction on $n$. Suppose that $c(m, i)=c(m, m-i)$ for all $i$ and $m \leq n$. Then by the induction hypothesis and the recurrence formula we have $c(n+1, k)=2^{k} c(n, k)+c(n, k-1)$ and $c(n+1, n+1-k)=$ $2^{n+1-k} c(n, n+1-k)+c(n, n-k)=2^{n+1-k} c(n, k-1)+c(n, k)$. Thus it remains only to show that + +$$ +\left(2^{k}-1\right) c(n, k)=\left(2^{n+1-k}-1\right) c(n, k-1) +$$ + +We prove this also by induction on $n$. By the induction hypothesis, + +$$ +c(n-1, k)=\frac{2^{n-k}-1}{2^{k}-1} c(n-1, k-1) +$$ + +and + +$$ +c(n-1, k-2)=\frac{2^{k-1}-1}{2^{n+1-k}-1} c(n-1, k-1) +$$ + +Using these formulas and the recurrence formula we obtain $\left(2^{k}-1\right) c(n, k)-$ $\left(2^{n+1-k}-1\right) c(n, k-1)=\left(2^{2 k}-2^{k}\right) c(n-1, k)-\left(2^{n}-3 \cdot 2^{k-1}+1\right) c(n-$ $1, k-1)-\left(2^{n+1-k}-1\right) c(n-1, k-2)=\left(2^{n}-2^{k}\right) c(n-1, k-1)-\left(2^{n}-\right.$ $\left.3 \cdot 2^{k-1}+1\right) c(n-1, k-1)-\left(2^{k-1}-1\right) c(n-1, k-1)=0$. This completes the proof. +Second solution. The given recurrence formula resembles that of binomial coefficients, so it is natural to search for an explicit formula of the form $c(n, k)=\frac{F(n)}{F(k) F(n-k)}$, where $F(m)=f(1) f(2) \cdots f(m)($ with $F(0)=1$ ) and $f$ is a certain function from the natural numbers to the real numbers. If there is such an $f$, then $c(n, k)=c(n, n-k)$ follows immediately. +After substitution of the above relation, the recurrence equivalently reduces to $f(n+1)=2^{k} f(n-k+1)+f(k)$. It is easy to see that $f(m)=2^{m}-1$ satisfies this relation. +Remark. If we introduce the polynomial $P_{n}(x)=\sum_{k=0}^{n} c(n, k) x^{k}$, the recurrence relation gives $P_{0}(x)=1$ and $P_{n+1}(x)=x P_{n}(x)+P_{n}(2 x)$. As a consequence of the problem, all polynomials in this sequence are symmetric, i.e., $P_{n}(x)=x^{n} P_{n}\left(x^{-1}\right)$. +13. Denote by $\mathcal{F}$ the set of functions considered. Let $f \in \mathcal{F}$, and let $f(1)=a$. Putting $n=1$ and $m=1$ we obtain $f(f(z))=a^{2} z$ and $f\left(a z^{2}\right)=f(z)^{2}$ for all $z \in \mathbb{N}$. These equations, together with the original one, imply $f(x)^{2} f(y)^{2}=f(x)^{2} f\left(a y^{2}\right)=f\left(x^{2} f\left(f\left(a y^{2}\right)\right)\right)=f\left(x^{2} a^{3} y^{2}\right)=$ $f\left(a(a x y)^{2}\right)=f(a x y)^{2}$, or $f(a x y)=f(x) f(y)$ for all $x, y \in \mathbb{N}$. Thus $f(a x)=a f(x)$, and we conclude that + +$$ +a f(x y)=f(x) f(y) \quad \text { for all } x, y \in \mathbb{N} +$$ + +We now prove that $f(x)$ is divisible by $a$ for each $x \in \mathbb{N}$. In fact, we inductively get that $f(x)^{k}=a^{k-1} f\left(x^{k}\right)$ is divisible by $a^{k-1}$ for every $k$. If $p^{\alpha}$ and $p^{\beta}$ are the exact powers of a prime $p$ that divide $f(x)$ and $a$ respectively, we deduce that $k \alpha \geq(k-1) \beta$ for all $k$, so we must have $\alpha \geq \beta$ for any $p$. Therefore $a \mid f(x)$. +Now we consider the function on natural numbers $g(x)=f(x) / a$. The above relations imply + +$$ +g(1)=1, \quad g(x y)=g(x) g(y), \quad g(g(x))=x \quad \text { for all } x, y \in \mathbb{N} +$$ + +Since $g \in \mathcal{F}$ and $g(x) \leq f(x)$ for all $x$, we may restrict attention to the functions $g$ only. +Clearly $g$ is bijective. We observe that $g$ maps a prime to a prime. Assume to the contrary that $g(p)=u v, u, v>1$. Then $g(u v)=p$, so either $g(u)=1$ and $g(v)=1$. Thus either $g(1)=u$ or $g(1)=v$, which is impossible. +We return to the problem of determining the least possible value of $g(1998)$. Since $g(1998)=g\left(2 \cdot 3^{3} \cdot 37\right)=g(2) \cdot g(3)^{3} \cdot g(37)$, and $g(2)$, $g(3), g(37)$ are distinct primes, $g(1998)$ is not smaller than $2^{3} \cdot 3 \cdot 5=120$. On the other hand, the value of 120 is attained for any function $g$ satisfying (2) and $g(2)=3, g(3)=2, g(5)=37, g(37)=5$. Hence the answer is 120 . +14. If $x^{2} y+x+y$ is divisible by $x y^{2}+y+7$, then so is the number $y\left(x^{2} y+\right.$ $x+y)-x\left(x y^{2}+y+7\right)=y^{2}-7 x$. +If $y^{2}-7 x \geq 0$, then since $y^{2}-7 x0$ is divisible by $x y^{2}+y+7$. But then $x y^{2}+y+7 \leq 7 x-y^{2}<7 x$, from which we obtain $y \leq 2$. For $y=1$, we are led to $x+8 \mid 7 x-1$, and hence $x+8 \mid 7(x+8)-(7 x-1)=57$. Thus the only possibilities are $x=11$ and $x=49$, and the obtained pairs $(11,1),(49,1)$ are indeed solutions. For $y=2$, we have $4 x+9 \mid 7 x-4$, so that $7(4 x+9)-4(7 x-4)=79$ is divisible by $4 x+9$. We do not get any new solutions in this case. +Therefore all required pairs $(x, y)$ are $\left(7 t^{2}, 7 t\right)(t \in \mathbb{N}),(11,1)$, and $(49,1)$. +15. The condition is obviously satisfied if $a=0$ or $b=0$ or $a=b$ or $a, b$ are both integers. We claim that these are the only solutions. + +Suppose that $a, b$ belong to none of the above categories. The quotient $a / b=\lfloor a\rfloor /\lfloor b\rfloor$ is a nonzero rational number: let $a / b=p / q$, where $p$ and $q$ are coprime nonzero integers. +Suppose that $p \notin\{-1,1\}$. Then $p$ divides $\lfloor a n\rfloor$ for all $n$, so in particular $p$ divides $\lfloor a\rfloor$ and thus $a=k p+\varepsilon$ for some $k \in \mathbb{N}$ and $0 \leq \varepsilon<1$. Note that $\varepsilon \neq 0$, since otherwise $b=k q$ would also be an integer. It follows that there exists an $n \in \mathbb{N}$ such that $1 \leq n \varepsilon<2$. But then $\lfloor n a\rfloor=\lfloor k n p+n \varepsilon\rfloor=k n p+1$ is not divisible by $p$, a contradiction. Similarly, $q \notin\{-1,1\}$ is not possible. Therefore we must have $p, q= \pm 1$, and since $a \neq b$, the only possibility is $b=-a$. However, this leads to $\lfloor-a\rfloor=-\lfloor a\rfloor$, which is not valid if $a$ is not an integer. +16. Let $S$ be a set of integers such that for no four distinct elements $a, b, c, d \in$ $S$, it holds that $20 \mid a+b-c-d$. It is easily seen that there cannot exist distinct elements $a, b, c, d$ with $a \equiv b$ and $c \equiv d(\bmod 20)$. Consequently, if the elements of $S$ give $k$ different residues modulo 20, then $S$ itself has at most $k+2$ elements. +Next, consider these $k$ elements of $S$ with different residues modulo 20. They give $\frac{k(k-1)}{2}$ different sums of two elements. For $k \geq 7$ there are at least 21 such sums, and two of them, say $a+b$ and $c+d$, are equal modulo 20 ; it is easy to see that $a, b, c, d$ are discinct. It follows that $k$ cannot exceed 6 , and consequently $S$ has at most 8 elements. +An example of a set $S$ with 8 elements is $\{0,20,40,1,2,4,7,12\}$. Hence the answer is $n=9$. +17. Initially, we determine that the first few values for $a_{n}$ are $1,3,4,7,10$, $12,13,16,19,21,22,25$. Since these are exactly the numbers of the forms $3 k+1$ and $9 k+3$, we conjecture that this is the general pattern. In fact, it is easy to see that the equation $x+y=3 z$ has no solution in the set $K=\{3 k+1,9 k+3 \mid k \in \mathbb{N}\}$. We shall prove that the sequence $\left\{a_{n}\right\}$ is actually this set ordered increasingly. +Suppose $a_{n}>25$ is the first member of the sequence not belonging to $K$. We have several cases: +(i) $a_{n}=3 r+2, r \in \mathbb{N}$. By the assumption, one of $r+1, r+2, r+3$ is of the form $3 k+1$ (and smaller than $a_{n}$ ), and therefore is a member $a_{i}$ of the sequence. Then $3 a_{i}$ equals $a_{n}+1$, $a_{n}+4$, or $a_{n}+7$, which is a contradiction because $1,4,7$ are in the sequence. +(ii) $a_{n}=9 r, r \in \mathbb{N}$. Then $a_{n}+a_{2}=3(3 r+1)$, although $3 r+1$ is in the sequence, a contradiction. +(iii) $a_{n}=9 r+6, r \in \mathbb{N}$. Then one of the numbers $3 r+3,3 r+6,3 r+9$ is a member $a_{j}$ of the sequence, and thus $3 a_{j}$ is equal to $a_{n}+3$, $a_{n}+12$, or $a_{n}+21$, where $3,12,21$ are members of the sequence, again a contradiction. +Once we have revealed the structure of the sequence, it is easy to compute $a_{1998}$. We have $1998=4 \cdot 499+2$, which implies $a_{1998}=9 \cdot 499+a_{2}=4494$. +18. We claim that, if $2^{n}-1$ divides $m^{2}+9$ for some $m \in \mathbb{N}$, then $n$ must be a power of 2 . Suppose otherwise that $n$ has an odd divisor $d>1$. Then $2^{d}-1 \mid 2^{n}-1$ is also a divisor of $m^{2}+9=m^{2}+3^{2}$. However, $2^{d}-1$ has some prime divisor $p$ of the form $4 k-1$, and by a well-known fact, $p$ divides both $m$ and 3 . Hence $p=3$ divides $2^{d}-1$, which is impossible, because for $d$ odd, $2^{d} \equiv 2(\bmod 3)$. Hence $n=2^{r}$ for some $r \in \mathbb{N}$. +Now let $n=2^{r}$. We prove the existence of $m$ by induction on $r$. The case $r=1$ is trivial. Now for any $r>1$ note that $2^{2^{r}}-1=\left(2^{2^{r-1}}-1\right)\left(2^{2^{r-1}}+\right.$ 1). The induction hypothesis claims that there exists an $m_{1}$ such that $2^{2^{r-1}}-1 \mid m_{1}^{2}+9$. We also observe that $2^{2^{r-1}}+1 \mid m_{2}^{2}+9$ for simple $m_{2}=3 \cdot 2^{2^{r-2}}$. By the Chinese remainder theorem, there is an $m \in \mathbb{N}$ that satisfies $m \equiv m_{1}\left(\bmod 2^{2^{r-1}}-1\right)$ and $m \equiv m_{2}\left(\bmod 2^{2^{r-1}}+1\right)$. It is easy to see that this $m^{2}+9$ will be divisible by both $2^{2^{r-1}}-1$ and $2^{2^{r-1}}+1$, i.e., that $2^{2^{r}}-1 \mid m^{2}+9$. This completes the induction. +19. For $n=p_{1}^{\alpha_{1}} p_{2}^{\alpha_{2}} \cdots p_{r}^{\alpha_{r}}$, where $p_{i}$ are distinct primes and $\alpha_{i}$ natural numbers, we have $\tau(n)=\left(\alpha_{1}+1\right) \cdots\left(\alpha_{r}+1\right)$ and $\tau\left(n^{2}\right)=\left(2 \alpha_{1}+1\right) \ldots\left(2 \alpha_{r}+1\right)$. Putting $k_{i}=\alpha_{i}+1$, the problem reduces to determining all natural values of $m$ that can be represented as + +$$ +m=\frac{2 k_{1}-1}{k_{1}} \cdot \frac{2 k_{2}-1}{k_{2}} \cdots \frac{2 k_{r}-1}{k_{r}} +$$ + +Since the numerator $\tau\left(n^{2}\right)$ is odd, $m$ must be odd too. We claim that every odd $m$ has a representation of the form (1). The proof will be done by induction. +This is clear for $m=1$. Now for every $m=2 k-1$ with $k$ odd the result follows easily, since $m=\frac{2 k-1}{k} \cdot k$, and $k$ can be written as (1). We cannot do the same if $k$ is even; however, in the case $m=4 k-1$ with $k$ odd, we can write it as $m=\frac{12 k-3}{6 k-1} \cdot \frac{6 k-1}{3 k} \cdot k$, and this works. +In general, suppose that $m=2^{t} k-1$, with $k$ odd. Following the same pattern, we can write $m$ as + +$$ +m=\frac{2^{t}\left(2^{t}-1\right) k-\left(2^{t}-1\right)}{2^{t-1}\left(2^{t}-1\right) k-\left(2^{t-1}-1\right)} \cdots \frac{4\left(2^{t}-1\right) k-3}{2\left(2^{t}-1\right) k-1} \cdot \frac{2\left(2^{t}-1\right) k-1}{\left(2^{t}-1\right) k} \cdot k +$$ + +The induction is finished. Hence $m$ can be represented as $\frac{\tau\left(n^{2}\right)}{\tau(n)}$ if and only if it is odd. +20. We first consider the special case $n=3^{r}$. Then the simplest choice $\frac{10^{n}-1}{9}=$ $11 \ldots 1$ ( $n$ digits) works. This can be shown by induction: it is true for $r=$ 1, while the inductive step follows from $10^{3^{r}}-1=\left(10^{3^{r-1}}-1\right)\left(10^{2 \cdot 3^{r-1}}+\right.$ $10^{3^{r-1}}+1$ ), because the second factor is divisible by 3 . +In the general case, let $k \geq n / 2$ be a positive integer and $a_{1}, \ldots, a_{n-k}$ be nonzero digits. We have + +$$ +\begin{aligned} +A & =\left(10^{k}-1\right) \overline{a_{1} a_{2} \ldots a_{n-k}} \\ +& =\overline{a_{1} a_{2} \ldots a_{n-k-1} a_{n-k}^{\prime} \underbrace{99 \ldots 99}_{2 k-n}} b_{1} b_{2} \ldots b_{n-k-1} b_{n-k}^{\prime} +\end{aligned} +$$ + +where $a_{n-k}^{\prime}=a_{n-k}-1, b_{i}=9-a_{i}$, and $b_{n-k}^{\prime}=9-a_{n-k}^{\prime}$. The sum of digits of $A$ equals $9 k$ independently of the choice of digits $a_{1}, \ldots, a_{n-k}$. Thus we need only choose $k \geq \frac{n}{2}$ and digits $a_{1}, \ldots, a_{n-k-1} \notin\{0,9\}$ and $a_{n-k} \in\{0,1\}$ in order for the conditions to be fulfilled. Let us choose + +$$ +k=\left\{\begin{array}{l} +3^{r}, \quad \text { if } 3^{r}1$ we have $N_{s} \equiv 1(\bmod 4)$. Now we prove $3 \mid d\left(p_{s}\right)$. We have $p_{s}\left|N_{s}\right| 10^{3 s}-1$ and hence $d\left(p_{s}\right) \mid 3 s$. We cannot have $d\left(p_{s}\right) \mid s$, for otherwise $p_{s}\left|10^{s}-1 \Rightarrow p_{s}\right|\left(10^{2 s}+\right.$ $\left.10^{s}+1,10^{s}-1\right)=3$; and we cannot have $d\left(p_{s}\right) \mid 3$, for otherwise $p_{s} \mid 10^{3}-1=999=3^{3} \cdot 37$, both of which contradict $p_{s} \neq 3,37$. It follows that $d\left(p_{s}\right)=3 s$. Hence for every prime $s$ there exists a prime $p_{s}$ such that $d\left(p_{s}\right)=3 s$. It follows that the cardinality of $S$ is infinite. +(b) Let $r=r(s)$ be the fundamental period of $p \in S$. Then $p \mid 10^{3 r}-1$, $p \nmid 10^{r}-1 \Rightarrow p \mid 10^{2 r}+10^{r}+1$. Let $x_{j}=\frac{10^{j-1}}{p}$ and $y_{j}=\left\{x_{j}\right\}=$ $0 . a_{j} a_{j+1} a_{j+2} \ldots$. Then $a_{j}<10 y_{j}$, and hence + +$$ +f(k, p)=a_{k}+a_{k+r}+a_{k+2 r}<10\left(y_{k}+y_{k+r}+y_{k+2 r}\right) . +$$ + +We note that $x_{k}+x_{k+s(p)}+x_{k+2 s(p)}=\frac{10^{k-1} N_{p}}{p}$ is an integer, from which it follows that $y_{k}+y_{k+s(p)}+y_{k+2 s(p)} \in \mathbb{N}$. Hence $y_{k}+y_{k+s(p)}+$ $y_{k+2 s(p)} \leq 2$. It follows that $f(k, p)<20$. We note that $f(2,7)=$ $4+8+7=19$. Hence 19 is the greatest possible value of $f(k, p)$. +5. Since one can arbitrarily add zeros at the end of $m$, which increases divisibility by 2 and 5 to an arbitrary exponent, it suffices to assume $2,5 \nmid n$. If $(n, 10)=1$, there exists an integer $w \geq 2$ such that $10^{w} \equiv 1(\bmod n)$. We also note that $10^{i w} \equiv 1(\bmod n)$ and $10^{j w+1} \equiv 10(\bmod n)$ for all integers $i$ and $j$. Let us assume that $m$ is of the form $m=\sum_{i=1}^{u} 10^{i w}+\sum_{j=1}^{v} 10^{j w+1}$ for integers $u, v \geq 0$ (where if $u$ or $v$ is 0 , the corresponding sum is 0 ). Obviously, the sum of the digits of $m$ is equal to $u+v$, and also $m \equiv u+10 v(\bmod n)$. Hence our problem reduces to finding integers $u, v \geq 0$ such that $u+v=k$ and $n \mid u+10 v=k+9 v$. Since $(n, 9)=1$, it follows that there exists some $v_{0}$ such that $0 \leq v_{0}b)$ are respectively their smallest elements. +Lemma 1. Numbers $x, y$, and $x+y$ cannot belong to three different sets. Proof. The number $f(x, x+y)=f(y, x+y)$ must belong to both the set containing $y$ and the set containing $x$, a contradiction. +Lemma 2. The subset $C$ contains a multiple of $b$. Moreover, if $k b$ is the smallest such multiple, then $(k-1) b \in B$ and $(k-1) b+1, k b+1 \in A$. +Proof. Let $r$ be the residue of $c$ modulo $b$. If $r=0$, the first statement automatically holds. Let $02$ that for all $s$ we have $f(n, s)=\frac{1}{s}\binom{n-1}{s-1}\binom{n}{s-1}$. We shall prove that the given formula holds also for all $f(n+1, s)$, where $s \geq 2$. +We say that an $(n+1, s)$ - or $(n+1, s+1)$-path is related to a given $(n, s)$ path if it is obtained from the given path by inserting a step $E N$ between two moves or at the beginning or the end of the path. We note that by inserting the step between two moves that form a step one obtains an $(n+1, s)$-path; in all other cases one obtains an $(n+1, s+1)$-path. For each $(n, s)$-path there are exactly $2 n+1-s$ related $(n+1, s+1)$-paths, and for each $(n, s+1)$-path there are $s+1$ related $(n+1, s+1)$-paths. Also, each $(n+1, s+1)$-path is related to exactly $s+1$ different $(n, s)$ - or $(n, s+1)$-paths. Thus: + +$$ +\begin{aligned} +(s+1) f(n+1, s+1) & =(2 n+1-s) f(n, s)+(s+1) f(n, s+1) \\ +& =\frac{2 n+1-s}{s}\binom{n-1}{s-1}\binom{n}{s-1}+\binom{n-1}{s}\binom{n}{s} \\ +& =\binom{n}{s}\binom{n+1}{s}, +\end{aligned} +$$ + +i.e., $f(n+1, s+1)=\frac{1}{s+1}\binom{n}{s}\binom{n+1}{s}$. This completes the proof. +22. (a) Color the first, third, and fifth row red, and the remaining squares white. There in total $n$ pieces and $3 n$ red squares. Since each piece can cover at most three red squares, it follows that each piece colors exactly three red squares. Then it follows that the two white squares it covers must be on the same row; otherwise, the piece has to cover +at least three. Hence, each white row can be partitioned into pairs of squares belonging to the same piece. Thus it follows that the number of white squares in a row, which is $n$, must be even. +(b) Let $a_{k}$ denote the number of different tilings of a $5 \times 2 k$ rectangle. Let $b_{k}$ be the number of tilings that cannot be partitioned into two smaller tilings along a vertical line (without cutting any pieces). It is easy to see that $a_{1}=b_{1}=2, b_{2}=2, a_{2}=6=2 \cdot 3, b_{3}=4$, and subsequently, by induction, $b_{3 k} \geq 4, b_{3 k+1} \geq 2$, and $b_{3 k+2} \geq 2$. We also have $a_{k}=b_{k}+\sum_{i=1}^{k-1} b_{i} a_{k-i}$. For $k \geq 3$ we now have inductively + +$$ +a_{k}>2+\sum_{i=1}^{k-1} 2 a_{k-i} \geq 2 \cdot 3^{k-1}+2 a_{k-1} \geq 2 \cdot 3^{k} +$$ + +23. Let $r(m)$ denote the rest period before the $m$ th catch, $t(m)$ the number of minutes before the $m$ th catch, and $f(n)$ as the number of flies caught in $n$ minutes. We have $r(1)=1, r(2 m)=r(m)$, and $r(2 m+1)=f(m)+1$. We then have by induction that $r(m)$ is the number of ones in the binary representation of $m$. We also have $t(m)=\sum_{i=1}^{m} r(i)$ and $f(t(m))=m$. From the recursive relations for $r$ we easily derive $t(2 m+1)=2 t(m)+m+1$ and consequently $t(2 m)=2 t(m)+m-r(m)$. We then have, by induction on $p, t\left(2^{p} m\right)=2^{p} t(m)+p \cdot m \cdot 2^{p-1}-\left(2^{p}-1\right) r(m)$. +(a) We must find the smallest number $m$ such that $r(m+1)=9$. The smallest number with nine binary digits is $\overline{111111111}_{2}=511$; hence the required $m$ is 510 . +(b) We must calculate $t(98)$. Using the recursive formulas we have $t(98)=$ $2 t(49)+49-r(49), t(49)=2 t(24)+25$, and $t(24)=8 t(3)+36-7 r(3)$. Since we have $t(3)=4, r(3)=2$ and $r(49)=r\left(\overline{110001}_{2}\right)=3$, it follows $t(24)=54 \Rightarrow t(49)=133 \Rightarrow t(98)=312$. +(c) We must find $m_{c}$ such that $t\left(m_{c}\right) \leq 1999e>2$, it follows that $\left|\bigcup_{j=1}^{N} A_{i_{j}}\right| \geq \frac{1}{2}|S|$; hence the chosen $i_{1}<\cdots0$. Hence $F_{B}(w) \geq 1$. + +It remains to show that $F_{B}(w)=1$ if and only if $p=5$. We have the formula $(x-w)\left(x-w^{2}\right) \cdots\left(x-w^{p-1}\right)=x^{p-1}+x^{p-2}+\cdots+x+1=\frac{x^{p}-1}{x-1}$. Let $f_{B}(x)=x^{3}+x+1=(x-\lambda)(x-\mu)(x-\nu)$, where $\lambda$, $\mu$, and $\nu$ are the three zeros of the polynomial $f_{B}(x)$. It follows that +$F_{B}(w)=\left(\frac{\lambda^{p}-1}{\lambda-1}\right)\left(\frac{\mu^{p}-1}{\mu-1}\right)\left(\frac{\nu^{p}-1}{\nu-1}\right)=-\frac{1}{3}\left(\lambda^{p}-1\right)\left(\mu^{p}-1\right)\left(\nu^{p}-1\right)$, +since $(\lambda-1)(\mu-1)(\nu-1)=-f_{B}(1)=-3$. We also have $\lambda+\mu+\nu=0$, $\lambda \mu \nu=-1, \lambda \mu+\lambda \nu+\mu \nu=1$, and $\lambda^{2}+\mu^{2}+\nu^{2}=(\lambda+\mu+\nu)^{2}-2(\lambda \mu+$ $\lambda \nu+\mu \nu)=-2$. By induction (using that $\left(\lambda^{r}+\mu^{r}+\nu^{r}\right)+\left(\lambda^{r-2}+\mu^{r-2}+\right.$ $\left.\nu^{r-2}\right)+\left(\lambda^{r-3}+\mu^{r-3}+\nu^{r-3}\right)=0$ ), it follows that $\lambda^{r}+\mu^{r}+\nu^{r}$ is an integer for all $r \in \mathbb{N}$. +Let us assume $F_{B}(x)=1$. It follows that $\left(\lambda^{p}-1\right)\left(\mu^{p}-1\right)\left(\nu^{p}-1\right)=-3$. Hence $\lambda^{p}, \mu^{p}, \nu^{p}$ are roots of the polynomial $p(x)=x^{3}-q x^{2}+(1+q) x+1$, where $q=\lambda^{p}+\mu^{p}+\nu^{p}$. Since $f_{B}(x)$ is an increasing function in real numbers, it follows that it has only one real root (w.l.o.g.) $\lambda$, the other two roots being complex conjugates. From $f_{B}(-1)<00$; otherwise, $q\left(x^{2}-x\right)$ intersects $x^{3}+x+1$ at a value smaller than $\lambda$. Additionally, as $p$ increases, $\lambda^{p}$ approaches 0 , and hence $q$ must increase. +For $p=5$ we have $1+w+w^{3}=-w^{2}\left(1+w^{2}\right)$ and hence $G(w)=\prod_{i=1}^{p-1}(1+$ $\left.w^{2 j}\right)=1$. For a zero of $f_{B}(x)$ we have $x^{5}=-x^{3}-x^{2}=-x^{2}+x+1$ and hence $q=\lambda^{5}+\mu^{5}+\nu^{5}=-\left(\lambda^{2}+\mu^{2}+\nu^{2}\right)+(\lambda+\mu+\nu)+3=5$. +For $p>5$ we also have $q \geq 6$. Assuming again $F_{B}(x)=1$ and defining $p(x)$ as before, we have $p(-1)<0, p(0)>0, p(2)<0$, and $p(x)>0$ for a sufficiently large $x>2$. It follows that $p(x)$ must have three distinct real roots. However, since $\mu^{p}, \nu^{p} \in \mathbb{R} \Rightarrow \nu^{p}=\overline{\mu^{p}}=\mu^{p}$, it follows that $p(x)$ has at most two real roots, which is a contradiction. Hence, it follows that $F_{B}(x)>1$ for $p>5$ and thus $E(A) \leq E(B)$, where equality holds only for $p=5$. + +### 4.41 Solutions to the Shortlisted Problems of IMO 2000 + +1. In order for the trick to work, whenever $x+y=z+t$ and the cards $x, y$ are placed in different boxes, either $z, t$ are in these boxes as well or they are both in the remaining box. +Case 1. The cards $i, i+1, i+2$ are in different boxes for some $i$. Since $i+(i+3)=(i+1)+(i+2)$, the cards $i$ and $i+3$ must be in the same box; moreover, $i-1$ must be in the same box as $i+2$, etc. Hence the cards $1,4,7, \ldots, 100$ are placed in one box, the cards $2,5, \ldots, 98$ are in the second, while $3,6, \ldots, 99$ are in the third box. The number of different arrangements of the cards is 6 in this case. +Case 2. No three successive cards are all placed in different boxes. Suppose that 1 is in the blue box, and denote by $w$ and $r$ the smallest numbers on cards lying in the white and red boxes; assume w.l.o.g. that $w$ $w+1$. Now suppose that $r<100$. Since $w+r=(w-1)+(r+1), r+1$ must be in the blue box. But then $(r+1)+w=r+(w+1)$ implies that $w+1$ must be red, which is a contradiction. Hence the red box contains only the card 100 . Since $99+w=100+(w-1)$, we deduce that the card 99 is in the white box. Moreover, if any of the cards $k$, $2 \leq k \leq 99$, were in the blue box, then since $k+99=(k-1)+100$, the card $k-1$ should be in the red box, which is impossible. Hence the blue box contains only the card 1 , whereas the cards $2,3, \ldots, 99$ are all in the white box. +In general, one box contains 1 , another box only 100 , while the remaining contains all the other cards. There are exactly 6 such arrangements, and the trick works in each of them. +Therefore the answer is 12. +2. Since the volume of each brick is 12 , the side of any such cube must be divisible by 6 . +Suppose that a cube of side $n=6 k$ can be built using $\frac{n^{3}}{12}=18 k^{3}$ bricks. Set a coordinate system in which the cube is given as $[0, n] \times[0, n] \times[0, n]$ and color in black each unit cube $[2 p, 2 p+1] \times[2 q, 2 q+1] \times[2 r, 2 r+1]$. There are exactly $\frac{n^{3}}{9}=27 k^{3}$ black cubes. Each brick covers either one or three black cubes, which is in any case an odd number. It follows that the total number of black cubes must be even, which implies that $k$ is even. Hence $12 \mid n$. +On the other hand, two bricks can be fitted together to give a $2 \times 3 \times 4$ box. Using such boxes one can easily build a cube of side 12 , and consequently any cube of side divisible by 12 . +3. Clearly $m(S)$ is the number of pairs of point and triangle $\left(P_{t}, P_{i} P_{j} P_{k}\right)$ such that $P_{t}$ lies inside the circle $P_{i} P_{j} P_{k}$. Consider any four-element set $S_{i j k l}=\left\{P_{i}, P_{j}, P_{k}, P_{l}\right\}$. If the convex hull of $S_{i j k l}$ is the triangle $P_{i} P_{j} P_{k}$, then we have $a_{i}=a_{j}=a_{k}=0, a_{l}=1$. Suppose that the convex hull is +the quadrilateral $P_{i} P_{j} P_{k} P_{l}$. Since this quadrilateral is not cyclic, we may suppose that $\angle P_{i}+\angle P_{k}<180^{\circ}<\angle P_{j}+\angle P_{l}$. In this case $a_{i}=a_{k}=0$ and $a_{j}=a_{l}=1$. Therefore $m\left(S_{i j k l}\right)$ is 2 if $P_{i}, P_{j}, P_{k}, P_{l}$ are vertices of a convex quadrilateral, and 1 otherwise. +There are $\binom{n}{4}$ four-element subsets $S_{i j k l}$. If $a(S)$ is the number of such subsets whose points determine a convex quadrilateral, we have $m(S)=$ $2 a(S)+\left(\binom{n}{4}-a(S)\right)=\binom{n}{4}+a(S) \leq 2\binom{n}{4}$. Equality holds if and only if every four distinct points of $S$ determine a convex quadrilateral, i.e. if and only if the points of $S$ determine a convex polygon. Hence $f(n)=2\binom{n}{4}$ has the desired property. +4. By a good placement of pawns we mean the placement in which there is no block of $k$ adjacent unoccupied squares in a row or column. +We can make a good placement as follows: Label the rows and columns with $0,1, \ldots, n-1$ and place a pawn on a square $(i, j)$ if and only if $k$ divides $i+j+1$. This is obviously a good placement in which the pawns are placed on three lines with $k, 2 n-2 k$, and $2 n-3 k$ squares, which adds up to $4 n-4 k$ pawns in total. +Now we shall prove that a good placement must contain at least $4 n-4 k$ pawns. Suppose we have a good placement of $m$ pawns. Partition the board into nine rectangular regions as shown in the picture. Let $a, b, \ldots, h$ be the numbers of pawns in the rectangles $A, B, \ldots, H$ respectively. Note that each row that +![](https://cdn.mathpix.com/cropped/2024_11_18_8e985d6b9c83aa3e9d0eg-671.jpg?height=394&width=450&top_left_y=962&top_left_x=850) +passes through $A, B$, and $C$ either +contains a pawn inside $B$, or contains a pawn in both $A$ and $C$. It follows that $a+c+2 b \geq 2(n-k)$. We similarly obtain that $c+e+2 d, e+g+2 f$, and $g+a+2 h$ are all at least $2(n-k)$. Adding and dividing by 2 yields $a+b+\cdots+h \geq 4(n-k)$, which proves the statement. +5. We say that a vertex of a nice region is convex if the angle of the region at that vertex equals $90^{\circ}$; otherwise (if the angle is $270^{\circ}$ ), we say that a vertex is concave. +For a simple broken line $C$ contained in the boundary of a nice region $R$ we call the pair $(R, C)$ a boundary pair. Such a pair is called outer if the region $R$ is inside the broken line $C$, and inner otherwise. Let $\mathcal{B}_{i}, \mathcal{B}_{o}$ be the sets of inner and outer boundary pairs of nice regions respectively, and let $\mathcal{B}=\mathcal{B}_{i} \cup \mathcal{B}_{o}$. For a boundary pair $b=(R, C)$ denote by $c_{b}$ and $v_{b}$ respectively the number of convex and concave vertices of $R$ that belong to $C$. We have the following facts: +(1) Each vertex of a rectangle corresponds to one concave angle of a nice region and vice versa. This correspondence is bijective, so $\sum_{b \in \mathcal{B}} v_{b}=$ $4 n$. +(2) For a boundary pair $b=(R, C)$ the sum of angles of $R$ that are on $C$ equals $\left(c_{b}+v_{b}-2\right) 180^{\circ}$ if $b$ is outer, and $\left(c_{b}+v_{b}+2\right) 180^{\circ}$ if $b$ is inner. On the other hand the sum of angles is obviously equal to $c_{b} \cdot 90^{\circ}+v_{b} \cdot 270^{\circ}$. It immediately follows that $c_{b}-v_{b}=\left\{\begin{array}{r}4 \text { if } b \in \mathcal{B}_{o}, \\ -4 \text { if } b \in \mathcal{B}_{i} .\end{array}\right.$ +(3) Since every vertex of a rectangle appears in exactly two boundary pairs and each boundary pair contains at least one vertex of a rectangle, the number $K$ of boundary pairs is less than or equal to $8 n$. +(4) The set $\mathcal{B}_{i}$ is nonempty, because every boundary of the infinite region is inner. +Consequently, the sum of the numbers of the vertices of all nice regions is equal to + +$$ +\sum_{b \in \mathcal{B}}\left(c_{b}+v_{b}\right)=\sum_{b \in \mathcal{B}}\left(2 v_{b}+\left(c_{b}-v_{b}\right)\right) \leq 2 \cdot 4 n+4(K-1)-4 \leq 40 n-8 +$$ + +6. Every integer $z$ has a unique representation $z=p x+q y$, where $x, y \in \mathbb{Z}$, $0 \leq x \leq q-1$. Consider the region $T$ in the $x y$-plane defined by the last inequality and $p x+q y \geq 0$. There is a bijective correspondence between lattice points of this region and nonnegative integers given by $(x, y) \mapsto$ $z=p x+q y$. Let us mark all lattice points of $T$ whose corresponding integers belong to $S$ and color in black the unit squares whose left-bottom vertices are at marked points. Due to the condition for $S$, this coloring has the property that all points lying on the right or above a colored point are colored as well. In particular, since the point $(0,0)$ is colored, all points above or on the line $y=0$ are colored. What we need is the number of such colorings of $T$. +The border of the colored subregion $C$ of $T$ determines a path from $(0,0)$ to $(q,-p)$ consisting of consecutive unit moves either to the right or downwards. There are $\binom{p+q}{p}$ such paths in total. We must find the number of such paths not going below the line $l: p x+q y=0$. +Consider any path $\gamma=A_{0} A_{1} \ldots A_{p+q}$ from $A_{0}=(0,0)$ to $A_{p+q}=(q,-p)$. We shall see the path $\gamma$ as a sequence $G_{1} G_{2} \ldots G_{p+q}$ of moves to the right $(R)$ or downwards ( $D$ ) with exactly $p D$ 's and $q R$ 's. +Two paths are said to be equivalent if one is obtained from the other by a circular shift of the corresponding sequence $G_{1} G_{2} \ldots G_{p+q}$. We note that all the $p+q$ circular shifts of a path are distinct. Indeed, $G_{1} \ldots G_{p+q} \equiv$ $G_{i+1} \ldots G_{i+p+q}$ would imply $G_{1}=G_{i+1}=G_{2 i+1}=\cdots$ (where $G_{j+p+q}=$ $G_{j}$ ), so $G_{1}=\cdots=G_{p+q}$, which is impossible. Hence each equivalence class contains exactly $p+q$ paths. +Let $l_{i}, 0 \leq i0$, this proves the result. +12. Since $D(A)=D(B)$, we can define $f(i)>g(i) \geq 0$ that satisfy $b_{i}-b_{i-1}=$ $a_{f(i)}-a_{g(i)}$ for all $i$. +The number $b_{i+1}-b_{i-1} \in D(B)=D(A)$ can be written in the form $a_{u}-a_{v}, u>v \geq 0$. Then $b_{i+1}-b_{i-1}=b_{i+1}-b_{i}+b_{i}-b_{i-1}$ implies +$a_{f(i+1)}+a_{f(i)}+a_{v}=a_{g(i+1)}+a_{g(i)}+a_{u}$, so the $B_{3}$ property of $A$ implies that $(f(i+1), f(i), v)$ and $(g(i+1), g(i), u)$ coincide up to a permutation. It follows that either $f(i+1)=g(i)$ or $f(i)=g(i+1)$. Hence if we define $R=\left\{i \in \mathbb{N}_{0} \mid f(i+1)=g(i)\right\}$ and $S=\left\{i \in \mathbb{N}_{0} \mid f(i)=g(i+1)\right\}$ it holds that $R \cup S=\mathbb{N}_{0}$. +Lemma. If $i \in R$, then also $i+1 \in R$. +Proof. Suppose to the contrary that $i \in R$ and $i+1 \in S$, i.e., $g(i)=$ $f(i+1)=g(i+2)$. There are integers $x$ and $y$ such that $b_{i+2}-b_{i-1}=$ $a_{x}-a_{y}$. Then $a_{x}-a_{y}=a_{f(i+2)}-a_{g(i+2)}+a_{f(i+1)}-a_{g(i+1)}+a_{f(i)}-$ $a_{g(i)}=a_{f(i+2)}+a_{f(i)}-a_{g(i+1)}-a_{g(i)}$, so by the $B_{3}$ property $(x, g(i+$ $1), g(i))$ and $(y, f(i+2), f(i))$ coincide up to a permutation. But this is impossible, since $f(i+2), f(i)>g(i+2)=g(i)=f(i+1)>g(i+1)$. This proves the lemma. +Therefore if $i \in R \neq \emptyset$, then it follows that every $j>i$ belongs to $R$. Consequently $g(i)=f(i+1)>g(i+1)=f(i+2)>g(i+2)=f(i+3)>$ $\cdots$ is an infinite decreasing sequence of nonnegative integers, which is impossible. Hence $S=\mathbb{N}_{0}$, i.e., + +$$ +b_{i+1}-b_{i}=a_{f(i+1)}-a_{f(i)} \quad \text { for all } i \in \mathbb{N}_{0} +$$ + +Thus $f(0)=g(1)0$. Furthermore, by the induction hypothesis the polynomials of the form $Q_{k}(x)$ take at least $2^{k-2}$ values at $x=n$. Hence the total number of values of $Q(n)$ for $Q \in M(P)$ is at least $1+1+2+2^{2}+\cdots+2^{m-1}=2^{m}$. Now we return to the main result. Suppose that $P(x)=a_{2000} x^{2000}$ $+a_{1999} x^{1999}+a_{0}$ is an $n$-independent polynomial. Since $P_{2}(x)=a_{2000} x^{2000}$ $+a_{1998} x^{1998}+\cdots+a_{2} x^{2}+a_{0}$ is a polynomial in $t=x^{2}$ of degree 1000 , by the lemma it takes at least $2^{1000}$ distinct values at $x=n$. Hence $\{Q(n) \mid Q \in$ $M(P)\}$ contains at least $2^{1000}$ elements. On the other hand, interchanging the coefficients $b_{i}$ and $b_{j}$ in a polynomial $Q(x)=b_{2000} x^{2000}+\cdots+b_{0}$ modifies the value of $Q$ at $x=n$ by $\left(b_{i}-b_{j}\right)\left(n^{i}-n^{j}\right)=\left(a_{k}-a_{l}\right)\left(n^{i}-n^{j}\right)$ for some $k, l$. Hence there are fewer than $2001^{4}$ possible modifications of the value at $n$. Since $2001^{4}<2^{1000}$, we have arrived at a contradiction. +14. The given condition is obviously equivalent to $a^{2} \equiv 1(\bmod n)$ for all integers $a$ coprime to $n$. Let $n=p_{1}^{\alpha_{1}} p_{2}^{\alpha_{2}} \cdots p_{k}^{\alpha_{k}}$ be the factorization of $n$ onto primes. Since by the Chinese remainder theorem the numbers coprime to $n$ can give any remainder modulo $p_{i}^{\alpha_{i}}$ except 0 , our condition is equivalent to $a^{2} \equiv 1\left(\bmod p_{i}^{\alpha_{i}}\right)$ for all $i$ and integers $a$ coprime to $p_{i}$. +Now if $p_{i} \geq 3$, we have $2^{2} \equiv 1\left(\bmod p_{i}^{\alpha_{i}}\right)$, so $p_{i}=3$ and $\alpha_{i}=2$. If $p_{j}=2$, then $3^{2} \equiv 1\left(\bmod 2^{\alpha_{j}}\right)$ implies $\alpha_{j} \leq 3$. Hence $n$ is a divisor of $2^{3} \cdot 3=24$. Conversely, each $n \mid 24$ has the desired property. +15. Let $n=p_{1}^{\alpha_{1}} p_{2}^{\alpha_{2}} \cdots p_{k}^{\alpha_{k}}$ be the factorization of $n$ onto primes $\left(p_{1}5$ or $\beta_{i}>1$, then $\frac{p_{i}^{\beta_{i}}}{3 \beta_{i}+1}>\frac{5}{4}$, which is impossible. We conclude that $p_{2}=5$ and $k=2$, so $n \stackrel{2000}{=}$. +Hence the solutions for $n$ are 2, 128, and 2000. +16. More generally, we will prove by induction on $k$ that for each $k \in \mathbb{N}$ there exists $n_{k} \in \mathbb{N}$ that has exactly $k$ distinct prime divisors such that $n_{k} \mid 2^{n_{k}}+1$ and $3 \mid n_{k}$. +For $k=1, n_{1}=3$ satisfies the given conditions. Now assume that $k \geq 1$ and $n_{k}=3^{\alpha} m$ where $3 \nmid m$, so that $m$ has exactly $k-1$ prime divisors. Then the number $3 n_{k}=3^{\alpha+1} m$ has exactly $k$ prime divisors and $2^{3 n_{k}}+1=$ $\left(2^{n_{k}}+1\right)\left(2^{2 n_{k}}-2^{n_{k}}+1\right)$ is divisible by $3 n_{k}$, since $3 \mid 2^{2 n_{k}}-2^{n_{k}}+1$. We shall find a prime $p$ not dividing $n_{k}$ such that $n_{k+1}=3 p n_{k}$. It is enough to find $p$ such that $p \mid 2^{3 n_{k}}+1$ and $p \nmid 2^{n_{k}}+1$. +Moreover, we shall show that for every integer $a>2$ there exists a prime number $p$ that divides $a^{3}+1=(a+1)\left(a^{2}-a+1\right)$ but not $a+1$. To prove this we observe that $\operatorname{gcd}\left(a^{2}-a+1, a+1\right)=\operatorname{gcd}(3, a+1)$. Now if $3 \nmid a+1$, we can simply take $p=3$; otherwise, if $a=3 b-1$, then $a^{2}-a+1=9 b^{2}-9 b+3$ is not divisible by $3^{2}$; hence we can take for $p$ any prime divisor of $\frac{a^{2}-a+1}{3}$. +17. Trivially all triples $(a, 1, n)$ and $(1, m, n)$ are solutions. Assume now that $a>1$ and $m>1$. +If $m$ is even, then $a^{m}+1 \equiv(-1)^{m}+1 \equiv 2(\bmod a+1)$, which implies that $a^{m}+1=2^{t}$. In particular, $a$ is odd. But this is impossible, since $2p$ for $p>3$, so we must have $P=p=3$ and $b=2$. Since $b=a^{m_{1}}$, we obtain $a=2$ and $m=3$. The triple $(2,3, n)$ is indeed a solution if $n \geq 2$. +Hence the set of solutions is $\{(a, 1, n),(1, m, n) \mid a, m, n \in \mathbb{N}\} \cup\{(2,3, n) \mid$ $n \geq 2\}$. +Remark. This problem is very similar to (SL97-14). +18. It is known that the area of the triangle is $S=p r=p^{2} / n$ and $S=$ $\sqrt{p(p-a)(p-b)(p-c)}$. It follows that $p^{3}=n^{2}(p-a)(p-b)(p-c)$, which by putting $x=p-a, y=p-b$, and $z=p-c$ transforms into + +$$ +(x+y+z)^{3}=n^{2} x y z . +$$ + +We will be done if we show that (1) has a solution in positive integers for infinitely many natural numbers $n$. Let us assume that $z=k(x+y)$ for an integer $k>0$. Then (1) becomes $(k+1)^{3}(x+y)^{2}=k n^{2} x y$. Further, by setting $n=3(k+1)$ this equation reduces to + +$$ +(k+1)(x+y)^{2}=9 k x y +$$ + +Set $t=x / y$. Then (2) has solutions in positive integers if and only if $(k+$ 1) $(t+1)^{2}=9 k t$ has a rational solution, i.e., if and only if its discriminant $D=k(5 k-4)$ is a perfect square. Setting $k=u^{2}$, we are led to show that $5 u^{2}-4=v^{2}$ has infinitely many integer solutions. But this is a classic Pell-type equation, whose solution is every Fibonacci number $u=F_{2 i+1}$. This completes the proof. +19. Suppose that a natural number $N$ satisfies $N=a_{1}^{2}+\cdots+a_{k}^{2}, 2 N=$ $b_{1}^{2}+\cdots+b_{l}^{2}$, where $a_{i}, b_{j}$ are natural numbers such that none of the ratios $a_{i} / a_{j}, b_{i} / b_{j}, a_{i} / b_{j}, b_{j} / a_{i}$ is a power of 2. +We claim that every natural number $n>\sum_{i=0}^{4 N-2}(2 i N+1)^{2}$ can be represented as a sum of distinct squares. Suppose $n=4 q N+r, 0 \leq r<4 N$. Then + +$$ +n=4 N s+\sum_{i=0}^{r-1}(2 i N+1)^{2} +$$ + +for some positive integer $s$, so it is enough to show that $4 N s$ is a sum of distinct even squares. Let $s=\sum_{c=1}^{C} 2^{2 u_{c}}+\sum_{d=1}^{D} 2^{2 v_{d}+1}$ be the binary expansion of $s$. Then + +$$ +4 N s=\sum_{c=1}^{C} \sum_{i=1}^{k}\left(2^{u_{c}+1} a_{i}\right)^{2}+\sum_{d=1}^{D} \sum_{j=1}^{l}\left(2^{u_{d}+1} b_{j}\right)^{2} +$$ + +where all the summands are distinct by the condition on $a_{i}, b_{j}$. + +It remains to choose an appropriate $N$ : for example $N=29$, because $29=5^{2}+2^{2}$ and $58=7^{2}+3^{2}$. +Second solution. It can be directly checked that every odd integer $67<$ $n \leq 211$ can be represented as a sum of distinct squares. For any $n>211$ we can choose an integer $m$ such that $m^{2}>\frac{n}{2}$ and $n-m^{2}$ is odd and greater than 67 , and therefore by the induction hypothesis can be written as a sum of distinct squares. Hence $n$ is also a sum of distinct squares. +20. Denote by $k_{1}, k_{2}$ the given circles and by $k_{3}$ the circle through $A, B, C, D$. We shall consider the case that $k_{3}$ is inside $k_{1}$ and $k_{2}$, since the other case is analogous. +Let $A C$ and $A D$ meet $k_{1}$ at points $P$ and $R$, and $B C$ and $B D$ meet $k_{2}$ at $Q$ and $S$ respectively. We claim that $P Q$ and $R S$ are the common tangents to $k_{1}$ and $k_{2}$, and therefore $P, Q, R, S$ are the desired points. The circles $k_{1}$ and $k_{3}$ are tangent to each other, so we have $D C \| R P$. Since +![](https://cdn.mathpix.com/cropped/2024_11_18_8e985d6b9c83aa3e9d0eg-680.jpg?height=374&width=468&top_left_y=669&top_left_x=848) + +$$ +A C \cdot C P=X C \cdot C Y=B C \cdot C Q +$$ + +the quadrilateral $A B Q P$ is cyclic, implying that $\angle A P Q=\angle A B Q=$ $\angle A D C=\angle A R P$. It follows that $P Q$ is tangent to $k_{1}$. Similarly, $P Q$ is tangent to $k_{2}$. +21. Let $K$ be the intersection point of the lines $M N$ and $A B$. Since $K A^{2}=K M \cdot K N=K B^{2}$, it follows that $K$ is the midpoint of the segment $A B$, and consequently $M$ is the midpoint of $A B$. Thus it will be enough to show that $E M \perp$ $P Q$, or equivalently that $E M \perp$ $A B$. However, since $A B$ is tangent to the circle $G_{1}$ we have $\angle B A M=$ +![](https://cdn.mathpix.com/cropped/2024_11_18_8e985d6b9c83aa3e9d0eg-680.jpg?height=356&width=509&top_left_y=1295&top_left_x=812) +$\angle A C M=\angle E A B$, and similarly $\angle A B M=\angle E B A$. This implies that the triangles $E A B$ and $M A B$ are congruent. Hence $E$ and $M$ are symmetric with respect to $A B$; hence $E M \perp A B$. +Remark. The proposer has suggested an alternative version of the problem: to prove that $E N$ bisects the angle $C N D$. This can be proved by noting that $E A N B$ is cyclic. +22. Let $L$ be the point symmetric to $H$ with respect to $B C$. It is well known that $L$ lies on the circumcircle $k$ of $\triangle A B C$. Let $D$ be the intersection point of $O L$ and $B C$. We similarly define $E$ and $F$. Then + +$$ +O D+D H=O D+D L=O L=O E+E H=O F+F H +$$ + +We shall prove that $A D, B E$, and $C F$ are concurrent. Let line $A O$ meet $B C$ at $D^{\prime}$. It is easy to see that $\angle O D^{\prime} D=\angle O D D^{\prime}$; hence the perpendicular bisector of $B C$ bisects $D D^{\prime}$ as well. Hence $B D=C D^{\prime}$. If we define $E^{\prime}$ and $F^{\prime}$ analogously, we have $C E=A E^{\prime}$ and $A F=B F^{\prime}$. Since the lines $A D^{\prime}, B E^{\prime}, C F^{\prime}$ meet at $O$, it follows that $\frac{B D}{D C} \cdot \frac{C E}{E A} \cdot \frac{A F}{F B}=$ +![](https://cdn.mathpix.com/cropped/2024_11_18_8e985d6b9c83aa3e9d0eg-681.jpg?height=399&width=430&top_left_y=252&top_left_x=867) +$\frac{B D^{\prime}}{D^{\prime} C} \cdot \frac{C E^{\prime}}{E^{\prime} A} \cdot \frac{A F^{\prime}}{F^{\prime} B}=1$. This proves our claim by Ceva's theorem. +23. First, suppose that there are numbers $\left(b_{i}, c_{i}\right)$ assigned to the vertices of the polygon such that + +$$ +A_{i} A_{j}=b_{j} c_{i}-b_{i} c_{j} \quad \text { for all } i, j \text { with } 1 \leq i \leq j \leq n +$$ + +In order to show that the polygon is cyclic, it is enough to prove that $A_{1}, A_{2}, A_{3}, A_{i}$ lie on a circle for each $i, 4 \leq i \leq n$, or equivalently, by Ptolemy's theorem, that $A_{1} A_{2} \cdot A_{3} A_{i}+A_{2} A_{3} \cdot A_{i} A_{1}=A_{1} A_{3} \cdot A_{2} A_{i}$. But this is straightforward with regard to (1). +Now suppose that $A_{1} A_{2} \ldots A_{n}$ is a cyclic quadrilateral. By Ptolemy's theorem we have $A_{i} A_{j}=A_{2} A_{j} \cdot \frac{A_{1} A_{i}}{A_{1} A_{2}}-A_{2} A_{i} \cdot \frac{A_{1} A_{j}}{A_{1} A_{2}}$ for all $i, j$. This suggests taking $b_{1}=-A_{1} A_{2}, b_{i}=A_{2} A_{i}$ for $i \geq 2$ and $c_{i}=\frac{A_{1} A_{i}}{A_{1} A_{2}}$ for all $i$. Indeed, using Ptolemy's theorem, one easily verifies (1). +24. Since $\angle A B T=180^{\circ}-\gamma$ and $\angle A C T=180^{\circ}-\beta$, the law of sines gives $\frac{B P}{P C}=\frac{S_{A B T}}{S_{A C T}}=\frac{A B \cdot B T \cdot \sin \gamma}{A B \cdot B T \cdot \sin \beta}=\frac{A B \sin \gamma}{A C \sin \beta}=\frac{c^{2}}{b^{2}}$, which implies $B P=\frac{c^{2} a}{b^{2}+c^{2}}$. Denote by $M$ and $N$ the feet of perpendiculars from $P$ and $Q$ on $A B$. We have $\cot \angle A B Q=\frac{B N}{N Q}=\frac{2 B N}{P M}=\frac{B A+B M}{B P \sin \beta}=\frac{c+B P \cos \beta}{B P \sin \beta}=\frac{b^{2}+c^{2}+a c \cos \beta}{c a \sin \beta}=$ $\frac{2\left(b^{2}+c^{2}\right)+a^{2}+c^{2}-b^{2}}{2 c a \sin \beta}=\frac{a^{2}+b^{2}+3 c^{2}}{4 S_{A B C}}=2 \cot \alpha+2 \cot \beta+\cot \gamma$. Similarly, $\cot \angle B A S=2 \cot \alpha+2 \cot \beta+\cot \gamma$; hence $\angle A B Q=\angle B A S$. +Now put $p=\cot \alpha$ and $q=\cot \beta$. Since $p+q \geq 0$, the A-G mean inequality gives us $\cot \angle A B Q=2 p+2 q+\frac{1-p q}{p+q} \geq 2 p+2 q+\frac{1-(p+q)^{2} / 4}{p+q}=\frac{7}{4}(p+q)+$ $\frac{1}{p+q} \geq 2 \sqrt{\frac{7}{4}}=\sqrt{7}$. Hence $\angle A B Q \leq \arctan \frac{1}{\sqrt{7}}$. Equality holds if and only if $\cot \alpha=\cot \beta=\frac{1}{\sqrt{7}}$, i.e., when $a: b: c=1: 1: \frac{1}{\sqrt{2}}$. +25. By the condition of the problem, $\triangle A D X$ and $\triangle B C X$ are similar. Then there exist points $Y^{\prime}$ and $Z^{\prime}$ on the perpendicular bisector of $A B$ such that $\triangle A Y^{\prime} Z^{\prime}$ is similar and oriented the same as $\triangle A D X$, and $\triangle B Y^{\prime} Z^{\prime}$ is (being congruent to $\triangle A Y^{\prime} Z^{\prime}$ ) similar and oriented the same as $\triangle B C X$. Since then $A D / A Y^{\prime}=A X / A Z^{\prime}$ and $\angle D A Y^{\prime}=\angle X A Z^{\prime}, \triangle A D Y^{\prime}$ and $\triangle A X Z^{\prime}$ are also similar, implying $\frac{A D}{A X}=\frac{D Y^{\prime}}{X Z^{\prime}}$. Analogously, $\frac{B C}{B X}=\frac{C Y^{\prime}}{X Z^{\prime}}$. It follows from $\frac{A D}{A X}=\frac{B C}{B X}$ that $C Y^{\prime}=D Y^{\prime}$, which means that $Y^{\prime}$ lies on the perpendicular bisector of $C D$. Hence $Y^{\prime} \equiv Y$. + +Now $\angle A Y B=2 \angle A Y Z^{\prime}=2 \angle A D X$, as desired. +26. The problem can be reformulated in the following way: Given a set $S$ of ten points in the plane such that the distances between them are all distinct, for each point $P \in S$ we mark the point $Q \in S \backslash\{P\}$ nearest to $P$. Find the least possible number of marked points. +Observe that each point $A \in S$ is the nearest to at most five other points. Indeed, for any six points $P_{1}, \ldots, P_{6}$ one of the angles $P_{i} A P_{j}$ is at most $60^{\circ}$, in which case $P_{i} P_{j}$ is smaller than one of the distances $A P_{i}, A P_{j}$. It follows that at least two points are marked. +Now suppose that exactly two points, say $A$ and $B$, are marked. Then $A B$ is the minimal distance of the points from $S$, so by the previous observation the rest of the set $S$ splits into two subsets of four points according to whether the nearest point is $A$ or $B$. Let these subsets be $\left\{A_{1}, A_{2}, A_{3}, A_{4}\right\}$ and $\left\{B_{1}, B_{2}, B_{3}, B_{4}\right\}$ respectively. Assume that the points are labelled so that the angles $A_{i} A A_{i+1}$ are successively adjacent as well as the angles $B_{i} B B_{i+1}$, and that $A_{1}, B_{1}$ lie on one side of $A B$, and $A_{4}, B_{4}$ lie on the other side. Since all the angles $A_{i} A A_{i+1}$ and $B_{i} B B_{i+1}$ are greater than $60^{\circ}$, it follows that + +$$ +\angle A_{1} A B+\angle B A A_{4}+\angle B_{1} B A+\angle A B B_{4}<360^{\circ} . +$$ + +Therefore $\angle A_{1} A B+\angle B_{1} B A<180^{\circ}$ or $\angle A_{4} A B+\angle B_{4} B A<180^{\circ}$. Without loss of generality, let us assume the first inequality. +On the other hand, note that the quadrilateral $A B B_{1} A_{1}$ is convex because $A_{1}$ and $B_{1}$ are on different sides of the perpendicular bisector of $A B$. From $A_{1} B_{1}>A_{1} A$ and $B B_{1}>A B$ we obtain $\angle A_{1} A B_{1}>\angle A_{1} B_{1} A$ and $\angle B A B_{1}>\angle A B_{1} B$. Adding these relations yields $\angle A_{1} A B>\angle A_{1} B_{1} B$. Similarly, $\angle B_{1} B A>\angle B_{1} A_{1} A$. Adding these two inequalities, we get + +$$ +180^{\circ}>\angle A_{1} A B+\angle B_{1} B A>\angle A_{1} B_{1} B+\angle B_{1} A_{1} A +$$ + +hence the sum of the angles of the quadrilateral $A B B_{1} A_{1}$ is less than $360^{\circ}$, which is a contradiction. Thus at least 3 points are marked. +An example of a configuration in which exactly 3 gangsters are killed is shown below. +![](https://cdn.mathpix.com/cropped/2024_11_18_8e985d6b9c83aa3e9d0eg-682.jpg?height=250&width=441&top_left_y=1700&top_left_x=579) +27. Denote by $\alpha_{1}, \alpha_{2}, \alpha_{3}$ the angles of $\triangle A_{1} A_{2} A_{3}$ at vertices $A_{1}, A_{2}, A_{3}$ respectively. Let $T_{1}, T_{2}, T_{3}$ be the points symmetric to $L_{1}, L_{2}, L_{3}$ with respect to $A_{1} I, A_{2} I$, and $A_{3} I$ respectively. We claim that $T_{1} T_{2} T_{3}$ is the desired triangle. + +Denote by $S_{1}$ and $R_{1}$ the points symmetric to $K_{1}$ and $K_{3}$ with respect to $L_{1} L_{3}$. It is enough to show that $T_{1}$ and $T_{3}$ lie on the line $R_{1} S_{1}$. To prove this, we shall prove that $\angle K_{1} S_{1} T_{1}=\angle K^{\prime} K_{1} S_{1}$ for a point $K^{\prime}$ on the line $K_{1} K_{3}$ such that $K_{3}$ and $K^{\prime}$ lie on different sides of $K_{1}$. We show first that $S_{1} \in A_{1} I$. Let $X$ be the point of intersection of lines $A_{1} I$ and $L_{1} L_{3}$. We see from the triangle $A_{1} L_{3} X$ that $\angle L_{1} X I=$ $\alpha_{3} / 2=\angle L_{1} A_{3} I$, which implies that +![](https://cdn.mathpix.com/cropped/2024_11_18_8e985d6b9c83aa3e9d0eg-683.jpg?height=489&width=559&top_left_y=289&top_left_x=805) +$L_{1} X A_{3} I$ is cyclic. +We now have $\angle A_{1} X A_{3}=90^{\circ}=\angle A_{1} K_{1} A_{3}$; hence $A_{1} K_{1} X A_{3}$ is also cyclic. It follows that $\angle K_{1} X I=\angle K_{1} A_{3} A_{1}=\alpha_{3}=2 \angle L_{1} X I$; hence $X_{1} L_{1}$ bisects the angle $K_{1} X_{1} I$. Hence $S_{1} \in X I$ as claimed. Now we have $\angle K_{1} S_{1} T_{1}=\angle K_{1} S_{1} L_{1}+2 \angle L_{1} S_{1} X=\angle S_{1} K_{1} L_{1}+2 \angle L_{1} K_{1} X$. It remains to prove that $K_{1} X$ bisects $\angle A_{3} K_{1} K^{\prime}$. From the cyclic quadrilateral $A_{1} K_{1} X A_{3}$ we see that $\angle X K_{1} A_{3}=\alpha_{1} / 2$. Since $A_{1} K_{3} K_{1} A_{3}$ is cyclic, we also have $\angle K^{\prime} K_{1} A_{3}=\alpha_{1}=2 \angle X K_{1} A_{3}$, which proves the claim. + +### 4.42 Solutions to the Shortlisted Problems of IMO 2001 + +1. First, let us show that such a function is at most unique. Suppose that $f_{1}$ and $f_{2}$ are two such functions, and consider $g=f_{1}-f_{2}$. Then $g$ is zero on the boundary and satisfies + +$$ +g(p, q, r)=\frac{1}{6}[g(p+1, q-1, r)+\cdots+g(p, q-1, r+1)] +$$ + +i.e., $g(p, q, r)$ is equal to the average of the values of $g$ at six points $(p+$ $1, q-1, r), \ldots$ that lie in the plane $\pi$ given by $x+y+z=p+q+r$. Suppose that $(p, q, r)$ is the point at which $g$ attains its maximum in absolute value on $\pi \cap T$. The averaging property of $g$ implies that the values of $g$ at $(p+1, q-1, r)$ etc. are all equal to $g(p, q, r)$. Repeating this argument we obtain that $g$ is constant on the whole of $\pi \cap T$, and hence it equals 0 everywhere. Therefore $f_{1} \equiv f_{2}$. +It remains to guess $f$. It is natural to try $\bar{f}(p, q, r)=p q r$ first: it satisfies $\bar{f}(p, q, r)=\frac{1}{6}[\bar{f}(p+1, q-1, r)+\cdots+\bar{f}(p, q-1, r+1)]+\frac{p+q+r}{3}$. Thus we simply take + +$$ +f(p, q, r)=\frac{3}{p+q+r} \bar{f}(p, q, r)=\frac{3 p q r}{p+q+r} +$$ + +and directly check that it satisfies the required property. Hence this is the unique solution. +2. It follows from Bernoulli's inequality that for each $n \in \mathbb{N},\left(1+\frac{1}{n}\right)^{n} \geq 2$, or $\sqrt[n]{2} \leq 1+\frac{1}{n}$. Consequently, it will be enough to show that $1+a_{n}>$ $\left(1+\frac{1}{n}\right) a_{n-1}$. Assume the opposite. Then there exists $N$ such that for each $n \geq N$, + +$$ +1+a_{n} \leq\left(1+\frac{1}{n}\right) a_{n-1}, \quad \text { i.e., } \quad \frac{1}{n+1}+\frac{a_{n}}{n+1} \leq \frac{a_{n-1}}{n} +$$ + +Summing for $n=N, \ldots, m$ yields $\frac{a_{m}}{m+1} \leq \frac{a_{N-1}}{N}-\left(\frac{1}{N+1}+\cdots+\frac{1}{m+1}\right)$. However, it is well known that the sum $\frac{1}{N+1}+\cdots+\frac{1}{m+1}$ can be arbitrarily large for $m$ large enough, so that $\frac{a_{m}^{N+1}}{m+1}$ is eventually negative. This contradiction yields the result. +Second solution. Suppose that $1+a_{n} \leq \sqrt[n]{2} a_{n-1}$ for all $n \geq N$. Set $b_{n}=2^{-(1+1 / 2+\cdots+1 / n)}$ and multiply both sides of the above inequality to obtain $b_{n}+b_{n} a_{n} \leq b_{n-1} a_{n-1}$. Thus + +$$ +b_{N} a_{N}>b_{N} a_{N}-b_{n} a_{n} \geq b_{N}+b_{N+1}+\cdots+b_{n} +$$ + +However, it can be shown that $\sum_{n>N} b_{N}$ diverges: in fact, since $1+\frac{1}{2}+$ $\cdots+\frac{1}{n}<1+\ln n$, we have $b_{n}>2^{-1-\ln n}=\frac{1}{2} n^{-\ln 2}>\frac{1}{2 n}$, and we already know that $\sum_{n>N} \frac{1}{2 n}$ diverges. +Remark. As can be seen from both solutions, the value 2 in the problem can be increased to $e$. +3. By the arithmetic-quadratic mean inequality, it suffices to prove that + +$$ +\frac{x_{1}^{2}}{\left(1+x_{1}^{2}\right)^{2}}+\frac{x_{2}^{2}}{\left(1+x_{1}^{2}+x_{2}^{2}\right)^{2}}+\cdots+\frac{x_{n}^{2}}{\left(1+x_{1}^{2}+\cdots+x_{n}^{2}\right)^{2}}<1 . +$$ + +Observe that for $k \geq 2$ the following holds: + +$$ +\begin{aligned} +\frac{x_{k}^{2}}{\left(1+x_{1}^{2}+\cdots+x_{k}^{2}\right)^{2}} & \leq \frac{x_{k}^{2}}{\left(1+\cdots+x_{k-1}^{2}\right)\left(1+\cdots+x_{k}^{2}\right)} \\ +& =\frac{1}{1+x_{1}^{2}+\cdots+x_{k-1}^{2}}-\frac{1}{1+x_{1}^{2}+\cdots+x_{k}^{2}} +\end{aligned} +$$ + +For $k=1$ we have $\frac{x_{1}^{2}}{\left(1+x_{1}\right)^{2}} \leq 1-\frac{1}{1+x_{1}^{2}}$. Summing these inequalities, we obtain + +$$ +\frac{x_{1}^{2}}{\left(1+x_{1}^{2}\right)^{2}}+\cdots+\frac{x_{n}^{2}}{\left(1+x_{1}^{2}+\cdots+x_{n}^{2}\right)^{2}} \leq 1-\frac{1}{1+x_{1}^{2}+\cdots+x_{n}^{2}}<1 +$$ + +Second solution. Let $a_{n}(k)=\sup \left(\frac{x_{1}}{k^{2}+x_{1}^{2}}+\cdots+\frac{x_{n}}{k^{2}+x_{1}^{2}+\cdots+x_{n}^{2}}\right)$ and $a_{n}=$ $a_{n}(1)$. We must show that $a_{n}<\sqrt{n}$. Replacing $x_{i}$ by $k x_{i}$ shows that $a_{n}(k)=a_{n} / k$. Hence + +$$ +a_{n}=\sup _{x_{1}}\left(\frac{x_{1}}{1+x_{1}^{2}}+\frac{a_{n-1}}{\sqrt{1+x_{1}^{2}}}\right)=\sup _{\theta}\left(\sin \theta \cos \theta+a_{n-1} \cos \theta\right), +$$ + +where $\tan \theta=x_{1}$. The above supremum can be computed explicitly: + +$$ +a_{n}=\frac{1}{8 \sqrt{2}}\left(3 a_{n-1}+\sqrt{a_{n-1}^{2}+8}\right) \sqrt{4-a_{n-1}^{2}+a_{n-1} \sqrt{a_{n-1}^{2}+8}} . +$$ + +However, the required inequality is weaker and can be proved more easily: if $a_{n-1}<\sqrt{n-1}$, then by (1) $a_{n}<\sin \theta+\sqrt{n-1} \cos \theta=\sqrt{n} \sin (\theta+\alpha) \leq$ $\sqrt{n}$, for $\alpha \in(0, \pi / 2)$ with $\tan \alpha=\sqrt{n}$. +4. Let $(*)$ denote the given functional equation. Substituting $y=1$ we get $f(x)^{2}=x f(x) f(1)$. If $f(1)=0$, then $f(x)=0$ for all $x$, which is the trivial solution. Suppose $f(1)=C \neq 0$. Let $G=\{y \in \mathbb{R} \mid f(y) \neq 0\}$. Then + +$$ +f(x)=\left\{\begin{array}{cl} +C x & \text { if } x \in G \\ +0 & \text { otherwise } +\end{array}\right. +$$ + +We must determine the structure of $G$ so that the function defined by (1) satisfies (*). +(1) Clearly $1 \in G$, because $f(1) \neq 0$. +(2) If $x \in G, y \notin G$, then by $(*)$ it holds $f(x y) f(x)=0$, so $x y \notin G$. +(3) If $x, y \in G$, then $x / y \in G$ (otherwise by $2^{\circ}, y(x / y)=x \notin G$ ). +(4) If $x, y \in G$, then by $2^{\circ}$ we have $x^{-1} \in G$, so $x y=y / x^{-1} \in G$. + +Hence $G$ is a set that contains 1 , does not contain 0 , and is closed under multiplication and division. Conversely, it is easy to verify that every such $G$ in (1) gives a function satisfying (*). +5. Let $a_{1}, a_{2}, \ldots, a_{n}$ satisfy the conditions of the problem. Then $a_{k}>a_{k-1}$, and hence $a_{k} \geq 2$ for $k=1, \ldots, n$. The inequality $\left(a_{k+1}-1\right) a_{k-1} \geq$ $a_{k}^{2}\left(a_{k}-1\right)$ can be rewritten as + +$$ +\frac{a_{k-1}}{a_{k}}+\frac{a_{k}}{a_{k+1}-1} \leq \frac{a_{k-1}}{a_{k}-1} . +$$ + +Summing these inequalities for $k=i+1, \ldots, n-1$ and using the obvious inequality $\frac{a_{n-1}}{a_{n}}<\frac{a_{n-1}}{a_{n}-1}$, we obtain $\frac{a_{i}}{a_{i+1}}+\cdots+\frac{a_{n-1}}{a_{n}}<\frac{a_{i}}{a_{i+1}-1}$. Therefore + +$$ +\frac{a_{i}}{a_{i+1}} \leq \frac{99}{100}-\frac{a_{0}}{a_{1}}-\cdots-\frac{a_{i-1}}{a_{i}}<\frac{a_{i}}{a_{i+1}-1} \quad \text { for } i=1,2, \ldots, n-1 +$$ + +Consequently, given $a_{0}, a_{1}, \ldots, a_{i}$, there is at most one possibility for $a_{i+1}$. In our case, (1) yields $a_{1}=2, a_{2}=5, a_{3}=56, a_{4}=280^{2}=78400$. These values satisfy the conditions of the problem, so that this is a unique solution. +6. We shall determine a constant $k>0$ such that + +$$ +\frac{a}{\sqrt{a^{2}+8 b c}} \geq \frac{a^{k}}{a^{k}+b^{k}+c^{k}} \quad \text { for all } a, b, c>0 +$$ + +This inequality is equivalent to $\left(a^{k}+b^{k}+c^{k}\right)^{2} \geq a^{2 k-2}\left(a^{2}+8 b c\right)$, which further reduces to + +$$ +\left(a^{k}+b^{k}+c^{k}\right)^{2}-a^{2 k} \geq 8 a^{2 k-2} b c +$$ + +On the other hand, the AM-GM inequality yields + +$$ +\left(a^{k}+b^{k}+c^{k}\right)^{2}-a^{2 k}=\left(b^{k}+c^{k}\right)\left(2 a^{k}+b^{k}+c^{k}\right) \geq 8 a^{k / 2} b^{3 k / 4} c^{3 k / 4} +$$ + +and therefore $k=4 / 3$ is a good choice. Now we have + +$$ +\begin{aligned} +& \frac{a}{\sqrt{a^{2}+8 b c}}+\frac{b}{\sqrt{b^{2}+8 c a}}+\frac{c}{\sqrt{c^{2}+8 a b}} \\ +& \geq \frac{a^{4 / 3}}{a^{4 / 3}+b^{4 / 3}+c^{4 / 3}}+\frac{b^{4 / 3}}{a^{4 / 3}+b^{4 / 3}+c^{4 / 3}}+\frac{c^{4 / 3}}{a^{4 / 3}+b^{4 / 3}+c^{4 / 3}}=1 +\end{aligned} +$$ + +Second solution. The numbers $x=\frac{a}{\sqrt{a^{2}+8 b c}}, y=\frac{b}{\sqrt{b^{2}+8 c a}}$ and $z=\frac{c}{\sqrt{c^{2}+8 a b}}$ satisfy + +$$ +f(x, y, z)=\left(\frac{1}{x^{2}}-1\right)\left(\frac{1}{y^{2}}-1\right)\left(\frac{1}{z^{2}}-1\right)=8^{3} . +$$ + +Our task is to prove $x+y+z \geq 1$. +Since $f$ is decreasing on each of the variables $x, y, z$, this is the same as proving that $x, y, z>0, x+y+z=1$ implies $f(x, y, z) \geq 8^{3}$. However, since $\frac{1}{x^{2}}-1=\frac{(x+y+z)^{2}-x^{2}}{x^{2}}=\frac{(2 x+y+z)(y+z)}{x^{2}}$, the inequality $f(x, y, z) \geq 8^{3}$ becomes + +$$ +\frac{(2 x+y+z)(x+2 y+z)(x+y+2 z)(y+z)(z+x)(x+y)}{x^{2} y^{2} z^{2}} \geq 8^{3} +$$ + +which follows immediately by the AM-GM inequality. +Third solution. We shall prove a more general fact: the inequality $\frac{a}{\sqrt{a^{2}+k b c}}+\frac{b}{\sqrt{b^{2}+k c a}}+\frac{c}{\sqrt{c^{2}+k a b}} \geq \frac{3}{\sqrt{1+k}}$ is true for all $a, b, c>0$ if and only if $k \geq 8$. +Firstly suppose that $k \geq 8$. Setting $x=b c / a^{2}, y=c a / b^{2}, z=a b / c^{2}$, we reduce the desired inequality to + +$$ +F(x, y, z)=f(x)+f(y)+f(z) \geq \frac{3}{\sqrt{1+k}}, \quad \text { where } f(t)=\frac{1}{\sqrt{1+k t}} +$$ + +for $x, y, z>0$ such that $x y z=1$. We shall prove (2) using the method of Lagrange multipliers. +The boundary of the set $D=\left\{(x, y, z) \in \mathbb{R}_{+}^{3} \mid x y z=1\right\}$ consists of points $(x, y, z)$ with one of $x, y, z$ being 0 and another one being $+\infty$. If w.l.o.g. $x=0$, then $F(x, y, z) \geq f(x)=1 \geq 3 / \sqrt{1+k}$. +Suppose now that $(x, y, z)$ is a point of local minimum of $F$ on $D$. There exists $\lambda \in \mathbb{R}$ such that $(x, y, z)$ is stationary point of the function $F(x, y, z)+\lambda x y z$. Then $(x, y, z, \lambda)$ is a solution to the system $f^{\prime}(x)+\lambda y z=$ $f^{\prime}(y)+\lambda x z=f^{\prime}(z)+\lambda x y=0, x y z=1$. Eliminating $\lambda$ gives us + +$$ +x f^{\prime}(x)=y f^{\prime}(y)=z f^{\prime}(z), \quad x y z=1 +$$ + +The function $t f^{\prime}(t)=\frac{-k t}{2(1+k t)^{3 / 2}}$ decreases on the interval $(0,2 / k]$ and increases on $[2 / k,+\infty)$ because $\left(t f^{\prime}(t)\right)^{\prime}=\frac{k(k t-2)}{4(1+k t)^{5 / 2}}$. It follows that two of the numbers $x, y, z$ are equal. If $x=y=z$, then $(1,1,1)$ is the only solution to (3). Suppose that $x=y \neq z$. Since $\left(y f^{\prime}(y)\right)^{2}-\left(z f^{\prime}(z)\right)^{2}=$ $\frac{k^{2}(z-y)\left(k^{3} y^{2} z^{2}-3 k y z-y-z\right)}{4(1+k y)^{3}(1+k z)^{3}},(3)$ gives us $y^{2} z=1$ and $k^{3} y^{2} z^{2}-3 k y z-y-z=$ 0 . Eliminating $z$ we obtain an equation in $y, k^{3} / y^{2}-3 k / y-y-1 / y^{2}=0$, whose only real solution is $y=k-1$. Thus $\left(k-1, k-1,1 /(k-1)^{2}\right)$ and the cyclic permutations are the only solutions to (3) with $x, y, z$ being not all equal. Since $F\left(k-1, k-1,1 /(k-1)^{2}\right)=(k+1) / \sqrt{k^{2}-k+1}>$ $F(1,1,1)=1$, the inequality (2) follows. +For $0\frac{a}{\sqrt{a^{2}+8 b c}}+$ $\frac{b}{\sqrt{b^{2}+8 c a}}+\frac{c}{\sqrt{c^{2}+8 a b}} \geq 1$. If we fix $c$ and let $a, b$ tend to 0 , the first two summands will tend to 0 while the third will tend to 1 . Hence the inequality cannot be improved. +7. It is evident that arranging of $A$ in increasing order does not diminish $m$. Thus we can assume that $A$ is nondecreasing. Assume w.l.o.g. that $a_{1}=1$, and let $b_{i}$ be the number of elements of $A$ that are equal to $i$ $\left(1 \leq i \leq n=a_{2001}\right)$. Then we have $b_{1}+b_{2}+\cdots+b_{n}=2001$ and + +$$ +m=b_{1} b_{2} b_{3}+b_{2} b_{3} b_{4}+\cdots+b_{n-2} b_{n-1} b_{n} +$$ + +Now if $b_{i}, b_{j}(i0$. Let $m$ be the number of positive terms among $x_{1}, \ldots, x_{n}$. Since $x_{i}$ counts the terms equal to $i$, the sum $x_{1}+\cdots+x_{n}$ counts the total number of positive terms in the sequence, which is known to be $m+1$. Therefore among $x_{1}, \ldots, x_{n}$ exactly $m-1$ terms are equal to 1 , one is equal to 2 , and the others are 0 . Only $x_{0}$ can exceed 2 , and consequently at most one of $x_{3}, x_{4}, \ldots$ can be positive. It follows that $m \leq 3$. +(i) $m=1$ : Then $x_{2}=2$ (since $x_{1}=2$ is impossible), so $x_{0}=2$. The resulting sequence is $(2,0,2,0)$. +(ii) $m=2$ : Either $x_{1}=2$ or $x_{2}=2$. These cases yield $(1,2,1,0)$ and $(2,1,2,0,0)$ respectively. +(iii) $m=3$ : This means that $x_{k}>0$ for some $k>2$. Hence $x_{0}=k$ and $x_{k}=1$. Further, $x_{1}=1$ is impossible, so $x_{1}=2$ and $x_{2}=1$; there are no more positive terms in the sequence. The resulting sequence is $(p, 2,1, \underbrace{0, \ldots, 0}_{p-3}, 1,0,0,0)$. +12. For each balanced sequence $a=\left(a_{1}, a_{2}, \ldots, a_{2 n}\right)$ denote by $f(a)$ the sum of $j$ 's for which $a_{j}=1$ (for example, $f(100101)=1+4+6=11$ ). Partition the $\binom{2 n}{n}$ balanced sequences into $n+1$ classes according to the residue of $f$ modulo $n+1$. Now take $S$ to be a class of minimum size: obviously $|S| \leq \frac{1}{n+1}\binom{2 n}{n}$. We claim that every balanced sequence $a$ is either a member of $S$ or a neighbor of a member of $S$. We consider two cases. +(i) Let $a_{1}$ be 1 . It is easy to see that moving this 1 just to the right of the $k$ th 0 , we obtain a neighboring balanced sequence $b$ with $f(b)=$ $f(a)+k$. Thus if $a \notin S$, taking a suitable $k \in\{1,2, \ldots, n\}$ we can achieve that $b \in S$. +(ii) Let $a_{1}$ be 0 . Taking this 0 just to the right of the $k$ th 1 gives a neighbor $b$ with $f(b)=f(a)-k$, and the conclusion is similar to that of (i). +This justifies our claim. +13. At any moment, let $p_{i}$ be the number of pebbles in the $i$ th column, $i=$ $1,2, \ldots$ The final configuration has obvious properties $p_{1} \geq p_{2} \geq \cdots$ and $p_{i+1} \in\left\{p_{i}, p_{i}-1\right\}$. We claim that $p_{i+1}=p_{i}>0$ is possible for at most one $i$. +Assume the opposite. Then the final configuration has the property that for some $r$ and $s>r$ we have $p_{r+1}=p_{r}, p_{s+1}=p_{s}>0$ and $p_{r+k}=$ $p_{r+1}-k+1$ for all $k=1, \ldots, s-r$. Consider the earliest configuration, say $C$, with this property. What was the last move before $C$ ? The only possibilities are moving a pebble either from the $r$ th or from the $s$ th column; however, in both cases the configuration preceding this last move had the same property, contradicting the assumption that $C$ is the earliest. Therefore the final configuration looks as follows: $p_{1}=a \in \mathbb{N}$, and for some $r, p_{i}$ equals $a-(i-1)$ if $i \leq r$, and $a-(i-2)$ otherwise. It is easy to determine $a, r$ : since $n=p_{1}+p_{2}+\cdots=\frac{(a+1)(a+2)}{2}-r$, we get $\frac{a(a+1)}{2} \leq n<\frac{(a+1)(a+2)}{2}$, from which we uniquely find $a$ and then $r$ as well. + +The final configuration for $n=13$ : +![](https://cdn.mathpix.com/cropped/2024_11_18_8e985d6b9c83aa3e9d0eg-690.jpg?height=162&width=155&top_left_y=922&top_left_x=1027) +14. We say that a problem is difficult for boys if at most two boys solved it, and difficult for girls if at most two girls solved it. +Let us estimate the number of pairs boy-girl both of whom solved some problem difficult for boys. Consider any girl. By the condition (ii), among the six problems she solved, at least one was solved by at least 3 boys, and hence at most 5 were difficult for boys. Since each of these problems was solved by at most 2 boys and there are 21 girls, the considered number of pairs does not exceed $5 \cdot 2 \cdot 21=210$. +Similarly, there are at most 210 pairs boy-girl both of whom solved some problem difficult for girls. On the other hand, there are $21^{2}>2 \cdot 210$ pairs boy-girl, and each of them solved one problem in common. Thus some problems were difficult neither for girls nor for boys, as claimed. +Remark. The statement can be generalized: if $2(m-1)(n-1)+1$ boys and as many girls participated, and nobody solved more than $m$ problems, then some problem was solved by at least $n$ boys and $n$ girls. +15. Let $M N P Q$ be the square inscribed in $\triangle A B C$ with $M \in A B, N \in A C$, $P, Q \in B C$, and let $A A_{1}$ meet $M N, P Q$ at $K, X$ respectively. Put $M K=$ $P X=m, N K=Q X=n$, and $M N=d$. Then + +$$ +\frac{B X}{X C}=\frac{m}{n}=\frac{B X+m}{X C+n}=\frac{B P}{C Q}=\frac{d \cot \beta+d}{d \cot \gamma+d}=\frac{\cot \beta+1}{\cot \gamma+1} . +$$ + +Similarly, if $B B_{1}$ and $C C_{1}$ meet $A C$ and $B C$ at $Y, Z$ respectively then $\frac{C Y}{Y A}=\frac{\cot \gamma+1}{\cot \alpha+1}$ and $\frac{A Z}{Z B}=\frac{\cot \alpha+1}{\cot \beta+1}$. Therefore $\frac{B X}{X C} \frac{C Y}{Y A} \frac{A Z}{Z B}=1$, so by Ceva's theorem, $A X, B Y, C Z$ have a common point. + +Second solution. Let $A_{2}$ be the center of the square constructed over $B C$ outside $\triangle A B C$. Since this square and the inscribed square corresponding to the side $B C$ are homothetic, $A, A_{1}$, and $A_{2}$ are collinear. Points $B_{2}, C_{2}$ are analogously defined. Denote the angles $B A A_{2}, A_{2} A C, C B B_{2}$, $B_{2} B A, A C C_{2}, C_{2} C B$ by $\alpha_{1}, \alpha_{2}, \beta_{1}, \beta_{2}, \gamma_{1}, \gamma_{2}$. By the law of sines we have + +$$ +\frac{\sin \alpha_{1}}{\sin \alpha_{2}}=\frac{\sin \left(\beta+45^{\circ}\right)}{\sin \left(\gamma+45^{\circ}\right)}, \frac{\sin \beta_{1}}{\sin \beta_{2}}=\frac{\sin \left(\gamma+45^{\circ}\right)}{\sin \left(\alpha+45^{\circ}\right)}, \frac{\sin \gamma_{1}}{\sin \gamma_{2}}=\frac{\sin \left(\alpha+45^{\circ}\right)}{\sin \left(\beta+45^{\circ}\right)} +$$ + +Since the product of these ratios is 1 , by the trigonometric Ceva's theorem $A A_{2}, B B_{2}, C C_{2}$ are concurrent. +16. Since $\angle O C P=90^{\circ}-\angle A$, we are led to showing that $\angle O C P>\angle C O P$, i.e., $O P>C P$. By the triangle inequality it suffices to prove $C P<\frac{1}{2} C O$. Let $C O=R$. The law of sines yields $C P=A C \cos \gamma=2 R \sin \beta \cos \gamma<$ $2 R \sin \beta \cos \left(\beta+30^{\circ}\right)$. Finally, we have + +$$ +2 \sin \beta \cos \left(\beta+30^{\circ}\right)=\sin \left(2 \beta+30^{\circ}\right)-\sin 30^{\circ} \leq \frac{1}{2} +$$ + +which completes the proof. +17. Let us investigate a more general problem, in which $G$ is any point of the plane such that $A G, B G, C G$ are sides of a triangle. +Let $F$ be the point in the plane such that $B C: C F: F B=A G: B G: C G$ and $F, A$ lie on different sides of $B C$. Then by Ptolemy's inequality, on $B P C F$ we have $A G \cdot A P+B G \cdot B P+C G \cdot C P=A G \cdot A P+\frac{A G}{B C}(C F$. $B P+B F \cdot C P) \geq A G \cdot A P+\frac{A G}{B C} B C \cdot P F$. Hence + +$$ +A G \cdot A P+B G \cdot B P+C G \cdot C P \geq A G \cdot A F +$$ + +where equality holds if and only if $P$ lies on the segment $A F$ and on the circle $B C F$. Now we return to the case of $G$ the centroid of $\triangle A B C$. +We claim that $F$ is then the point $\widehat{G}$ in which the line $A G$ meets again the circumcircle of $\triangle B G C$. Indeed, if $M$ is the midpoint of $A B$, by the law of sines we have $\frac{B C}{C \widehat{G}}=$ $\frac{\sin \angle B \widehat{G} C}{\sin \angle C B \widehat{G}}=\frac{\sin \angle B G M}{\sin \angle A G M}=\frac{A G}{B G}$, and similarly $\frac{B C}{B \widehat{G}}=\frac{A G}{C G}$. Thus (1) implies +![](https://cdn.mathpix.com/cropped/2024_11_18_8e985d6b9c83aa3e9d0eg-691.jpg?height=342&width=507&top_left_y=1514&top_left_x=831) + +$$ +A G \cdot A P+B G \cdot B P+C G \cdot C P \geq A G \cdot A \widehat{G} +$$ + +It is easily seen from the above considerations that equality holds if and only if $P \equiv G$, and then the (minimum) value of $A G \cdot A P+B G \cdot B P+$ $C G \cdot C P$ equals + +$$ +A G^{2}+B G^{2}+C G^{2}=\frac{a^{2}+b^{2}+c^{2}}{3} +$$ + +Second solution. Notice that $A G \cdot A P \geq \overrightarrow{A G} \cdot \overrightarrow{A P}=\overrightarrow{A G} \cdot(\overrightarrow{A G}+\overrightarrow{P G})$. Summing this inequality with analogous inequalities for $B G \cdot B P$ and $C G \cdot C P$ gives us $A G \cdot A P+B G \cdot B P+C G \cdot C P \geq A G^{2}+B G^{2}+C G^{2}+$ $(\overrightarrow{A G}+\overrightarrow{B G}+\overrightarrow{C G}) \cdot \overrightarrow{P G}=A G^{2}+B G^{2}+C G^{2}=\frac{a^{2}+b^{2}+c^{2}}{3}$. Equality holds if and only if $P \equiv Q$. +18. Let $\alpha_{1}, \beta_{1}, \gamma_{1}, \alpha_{2}, \beta_{2}, \gamma_{2}$ denote the angles $\angle M A B, \angle M B C, \angle M C A$, $\angle M A C, \angle M B A, \angle M C B$ respectively. Then $\frac{M B^{\prime} \cdot M C^{\prime}}{M A^{2}}=\sin \alpha_{1} \sin \alpha_{2}$, $\frac{M C^{\prime} \cdot M A^{\prime}}{M B^{2}}=\sin \beta_{1} \sin \beta_{2}, \frac{M A^{\prime} \cdot M B^{\prime}}{M C^{2}}=\sin \gamma_{1} \sin \gamma_{2}$; hence + +$$ +p(M)^{2}=\sin \alpha_{1} \sin \alpha_{2} \sin \beta_{1} \sin \beta_{2} \sin \gamma_{1} \sin \gamma_{2} +$$ + +Since + +$$ +\sin \alpha_{1} \sin \alpha_{2}=\frac{1}{2}\left(\cos \left(\alpha_{1}-\alpha_{2}\right)-\cos \left(\alpha_{1}+\alpha_{2}\right) \leq \frac{1}{2}(1-\cos \alpha)=\sin ^{2} \frac{\alpha}{2}\right. +$$ + +we conclude that + +$$ +p(M) \leq \sin \frac{\alpha}{2} \sin \frac{\beta}{2} \sin \frac{\gamma}{2} +$$ + +Equality occurs when $\alpha_{1}=\alpha_{2}, \beta_{1}=\beta_{2}$, and $\gamma_{1}=\gamma_{2}$, that is, when $M$ is the incenter of $\triangle A B C$. +It is well known that $\mu(A B C)=\sin \frac{\alpha}{2} \sin \frac{\beta}{2} \sin \frac{\gamma}{2}$ is maximal when $\triangle A B C$ is equilateral (it follows, for example, from Jensen's inequality applied to $\ln \sin x)$. Hence $\max \mu(A B C)=\frac{1}{8}$. +19. It is easy to see that the hexagon $A E B F C D$ is convex and $\angle A E B+$ $\angle B F C+\angle C D A=360^{\circ}$. Using this relation we obtain that the circles $\omega_{1}, \omega_{2}, \omega_{3}$ with centers at $D, E, F$ and radii $D A, E B, F C$ respectively all pass through a common point $O$. Indeed, if $\omega_{1} \cap \omega_{2}=\{O\}$, then $\angle A O B=180^{\circ}-\angle A E B / 2$ and $\angle B O C=180^{\circ}-\angle B F C / 2$; hence $\angle C O A=180^{\circ}-\angle C D A / 2$ as well, i.e., $O \in \omega_{3}$. The point $O$ is the re- +![](https://cdn.mathpix.com/cropped/2024_11_18_8e985d6b9c83aa3e9d0eg-692.jpg?height=491&width=419&top_left_y=1318&top_left_x=870) +flection of $A$ with respect to $D E$. Similarly, it is also the reflection of $B$ with respect to $E F$, and that of $C$ with respect to $F D$. Hence + +$$ +\frac{D B}{D D^{\prime}}=1+\frac{D^{\prime} B}{D D^{\prime}}=1+\frac{S_{E B F}}{S_{E D F}}=1+\frac{S_{O E F}}{S_{D E F}} +$$ + +Analogously $\frac{E C}{E E^{\prime}}=1+\frac{S_{O D F}}{S_{D E F}}$ and $\frac{F A}{F F^{\prime}}=1+\frac{S_{O D E}}{S_{D E F}}$. Adding these relations gives us + +$$ +\frac{D B}{D D^{\prime}}+\frac{E C}{E E^{\prime}}+\frac{F A}{F F^{\prime}}=3+\frac{S_{O E F}+S_{O D F}+S_{O D E}}{S_{D E F}}=4 . +$$ + +20. By Ceva's theorem, we can choose real numbers $x, y, z$ such that + +$$ +\frac{\overrightarrow{B D}}{\overrightarrow{D C}}=\frac{z}{y}, \frac{\overrightarrow{C E}}{\overrightarrow{E A}}=\frac{x}{z}, \text { and } \frac{\overrightarrow{A F}}{\overrightarrow{F B}}=\frac{y}{x} +$$ + +The point $P$ lies outside the triangle $A B C$ if and only if $x, y, z$ are not all of the same sign. In what follows, $S_{X}$ will denote the signed area of a figure $X$. +Let us assume that the area $S_{A B C}$ of $\triangle A B C$ is 1 . Since $S_{P B C}: S_{P C A}$ : $S_{P A B}=x: y: z$ and $S_{P B D}: S_{P D C}=z: y$, it follows that $S_{P B D}=\frac{z}{y+z} \frac{x}{x+y+z}$. Hence $S_{P B D}=\frac{1}{y(y+z)} \frac{x y z}{x+y+z}, S_{P C E}=\frac{1}{z(z+x)} \frac{x y z}{x+y+z}$, $S_{P A F}=\frac{1}{x(x+y)} \frac{x y z}{x+y+z}$. By the condition of the problem we have $\left|S_{P B D}\right|=$ $\left|S_{P C E}\right|=\left|S_{P A F}\right|$, or + +$$ +|x(x+y)|=|y(y+z)|=|z(z+x)| . +$$ + +Obviously $x, y, z$ are nonzero, so that we can put w.l.o.g. $z=1$. At least two of the numbers $x(x+y), y(y+1), 1(1+x)$ are equal, so we can assume that $x(x+y)=y(y+1)$. We distinguish two cases: +(i) $x(x+y)=y(y+1)=1+x$. Then $x=y^{2}+y-1$, from which we obtain $\left(y^{2}+y-1\right)\left(y^{2}+2 y-1\right)=y(y+1)$. Simplification gives $y^{4}+3 y^{3}-y^{2}-4 y+1=0$, or + +$$ +(y-1)\left(y^{3}+4 y^{2}+3 y-1\right)=0 +$$ + +If $y=1$, then also $z=x=1$, so $P$ is the centroid of $\triangle A B C$, which is not an exterior point. Hence $y^{3}+4 y^{2}+3 y-1=0$. Now the signed area of each of the triangles $P B D, P C E, P A F$ equals + +$$ +\begin{aligned} +S_{P A F} & =\frac{y z}{(x+y)(x+y+z)} \\ +& =\frac{y}{\left(y^{2}+2 y-1\right)\left(y^{2}+2 y\right)}=\frac{1}{y^{3}+4 y^{2}+3 y-2}=-1 . +\end{aligned} +$$ + +It is easy to check that not both of $x, y$ are positive, implying that $P$ is indeed outside $\triangle A B C$. This is the desired result. +(ii) $x(x+y)=y(y+1)=-1-x$. In this case we are led to + +$$ +f(y)=y^{4}+3 y^{3}+y^{2}-2 y+1=0 . +$$ + +We claim that this equation has no real solutions. In fact, assume that $y_{0}$ is a real root of $f(y)$. We must have $y_{0}<0$, and hence $u=-y_{0}>0$ satisfies $3 u^{3}-u^{4}=(u+1)^{2}$. On the other hand, + +$$ +\begin{aligned} +3 u^{3}-u^{4} & =u^{3}(3-u)=4 u\left(\frac{u}{2}\right)\left(\frac{u}{2}\right)(3-u) \\ +& \leq 4 u\left(\frac{u / 2+u / 2+3-u}{3}\right)^{3}=4 u \\ +& \leq(u+1)^{2} +\end{aligned} +$$ + +where at least one of the inequalities is strict, a contradiction. +Remark. The official solution was incomplete, missing the case (ii). +21. We denote by $p(X Y Z)$ the perimeter of a triangle $X Y Z$. + +If $O$ is the circumcenter of $\triangle A B C$, then $A_{1}, B_{1}, C_{1}$ are the midpoints of the corresponding sides of the triangle, and hence $p\left(A_{1} B_{1} C_{1}\right)=$ $p\left(A B_{1} C_{1}\right)=p\left(A_{1} B C_{1}\right)=p\left(A_{1} B_{1} C\right)$. +Conversely, suppose that $p\left(A_{1} B_{1} C_{1}\right) \geq p\left(A B_{1} C_{1}\right), p\left(A_{1} B C_{1}\right), p\left(A_{1} B_{1} C\right)$. Let $\alpha_{1}, \alpha_{2}, \beta_{1}, \beta_{2}, \gamma_{1}, \gamma_{2}$ denote $\angle B_{1} A_{1} C, \angle C_{1} A_{1} B, \angle C_{1} B_{1} A, \angle A_{1} B_{1} C$, $\angle A_{1} C_{1} B, \angle B_{1} C_{1} A$. +Suppose that $\gamma_{1}, \beta_{2} \geq \alpha$. If $A_{2}$ is the fourth vertex of the parallelogram $B_{1} A C_{1} A_{2}$, then these conditions imply that $A_{1}$ is in the interior or on the border of $\triangle B_{1} C_{1} A_{2}$, and therefore $p\left(A_{1} B_{1} C_{1}\right) \leq p\left(A_{2} B_{1} C_{1}\right)=$ $p\left(A B_{1} C_{1}\right)$. Moreover, if one of the inequalities $\gamma_{1} \geq \alpha, \beta_{2} \geq \alpha$ is strict, +![](https://cdn.mathpix.com/cropped/2024_11_18_8e985d6b9c83aa3e9d0eg-694.jpg?height=346&width=543&top_left_y=902&top_left_x=804) +then $p\left(A_{1} B_{1} C_{1}\right)$ is strictly less than $p\left(A B_{1} C_{1}\right)$, contrary to the assumption. Hence + +$$ +\begin{aligned} +& \beta_{2} \geq \alpha \Longrightarrow \gamma_{1} \leq \alpha \\ +& \gamma_{2} \geq \beta \Longrightarrow \alpha_{1} \leq \beta \\ +& \alpha_{2} \geq \gamma \Longrightarrow \beta_{1} \leq \gamma +\end{aligned} +$$ + +the last two inequalities being obtained analogously to the first one. Because of the symmetry, there is no loss of generality in assuming that $\gamma_{1} \leq \alpha$. Then since $\gamma_{1}+\alpha_{2}=180^{\circ}-\beta=\alpha+\gamma$, it follows that $\alpha_{2} \geq \gamma$. From (1) we deduce $\beta_{1} \leq \gamma$, which further implies $\gamma_{2} \geq \beta$. Similarly, this leads to $\alpha_{1} \leq \beta$ and $\beta_{2} \geq \alpha$. To sum up, + +$$ +\gamma_{1} \leq \alpha \leq \beta_{2}, \quad \alpha_{1} \leq \beta \leq \gamma_{2}, \quad \beta_{1} \leq \gamma \leq \alpha_{2} +$$ + +Since $O A_{1} B C_{1}$ and $O B_{1} C A_{1}$ are cyclic, we have $\angle A_{1} O B=\gamma_{1}$ and $\angle A_{1} O C=\beta_{2}$. Hence $B O: C O=\cos \beta_{2}: \cos \gamma_{1}$, hence $B O \leq C O$. Analogously, $C O \leq A O$ and $A O \leq B O$. Therefore $A O=B O=C O$, i.e., $O$ is the circumcenter of $A B C$. +22. Let $S$ and $T$ respectively be the points on the extensions of $A B$ and $A Q$ over $B$ and $Q$ such that $B S=B P$ and $Q T=Q B$. It is given that $A S=$ $A B+B P=A Q+Q B=A T$. Since $\angle P A S=\angle P A T$, the triangles $A P S$ +and $A P T$ are congruent, from which we deduce that $\angle A T P=\angle A S P=$ $\beta / 2=\angle Q B P$. Hence $\angle Q T P=\angle Q B P$. +If $P$ does not lie on $B T$, then the last equality implies that $\triangle Q B P$ and $\triangle Q T P$ are congruent, so $P$ lies on the internal bisector of $\angle B Q T$. But $P$ also lies on the internal bisector of $\angle Q A B$; consequently, $P$ is an excenter of $\triangle Q A B$, thus lying on the internal bisector of $\angle Q B S$ as well. It follows that $\angle P B Q=\beta / 2=\angle P B S=180^{\circ}-\beta$, so $\beta=120^{\circ}$, which is impossible. Therefore $P \in B T$, which means that $T \equiv C$. Now from $Q C=Q B$ we conclude that $120^{\circ}-\beta=\gamma=\beta / 2$, i.e., $\beta=80^{\circ}$ and $\gamma=40^{\circ}$. +23. For each positive integer $x$, define $\alpha(x)=x / 10^{r}$ if $r$ is the positive integer satisfying $10^{r} \leq x<10^{r+1}$. Observe that if $\alpha(x) \alpha(y)<10$ for some $x, y \in \mathbb{N}$, then $\alpha(x y)=\alpha(x) \alpha(y)$. If, as usual, $[t]$ means the integer part of $t$, then $[\alpha(x)]$ is actually the leftmost digit of $x$. +Now suppose that $n$ is a positive integer such that $k \leq \alpha((n+k)!)\alpha(n+k)$ (the opposite can hold only if $\alpha(n+k) \geq 9)$. Therefore + +$$ +1<\alpha(n+2)<\cdots<\alpha(n+9) \leq \frac{5}{4} . +$$ + +On the other hand, this implies that $\alpha((n+4)!)=\alpha((n+1)!) \alpha(n+2) \alpha(n+$ 3) $\alpha(n+4)<(5 / 4)^{3} \alpha((n+1)$ ! $)<4$, contradicting the assumption that the leftmost digit of $(n+4)$ ! is 4 . +24. We shall find the general solution to the system. Squaring both sides of the first equation and subtracting twice the second equation we obtain $(x-y)^{2}=z^{2}+u^{2}$. Thus $(z, u, x-y)$ is a Pythagorean triple. Then it is well known that there are positive integers $t, a, b$ such that $z=t\left(a^{2}-b^{2}\right)$, $u=2 t a b$ (or vice versa), and $x-y=t\left(a^{2}+b^{2}\right)$. Using that $x+y=z+u$ we come to the general solution: + +$$ +x=t\left(a^{2}+a b\right), \quad y=t\left(a b-b^{2}\right) ; \quad z=t\left(a^{2}-b^{2}\right), \quad u=2 t a b . +$$ + +Putting $a / b=k$ we obtain + +$$ +\frac{x}{y}=\frac{k^{2}+k}{k-1}=3+(k-1)+\frac{2}{k-1} \geq 3+2 \sqrt{2} +$$ + +with equality for $k-1=\sqrt{2}$. On the other hand, $k$ can be arbitrarily close to $1+\sqrt{2}$, and so $x / y$ can be arbitrarily close to $3+2 \sqrt{2}$. Hence $m=3+2 \sqrt{2}$. +Remark. There are several other techniques for solving the given system. The exact lower bound of $m$ itself can be obtained as follows: by the $\operatorname{system}\left(\frac{x}{y}\right)^{2}-6 \frac{x}{y}+1=\left(\frac{z-u}{y}\right)^{2} \geq 0$, so $x / y \geq 3+2 \sqrt{2}$. +25. Define $b_{n}=\left|a_{n+1}-a_{n}\right|$ for $n \geq 1$. From the equalities $a_{n+1}=b_{n-1}+b_{n-2}$, from $a_{n}=b_{n-2}+b_{n-3}$ we obtain $b_{n}=\left|b_{n-1}-b_{n-3}\right|$. From this relation we deduce that $b_{m} \leq \max \left(b_{n}, b_{n+1}, b_{n+2}\right)$ for all $m \geq n$, and consequently $b_{n}$ is bounded. +Lemma. If $\max \left(b_{n}, b_{n+1}, b_{n+2}\right)=M \geq 2$, then $\max \left(b_{n+6}, b_{n+7}, b_{n+8}\right) \leq$ $M-1$. +Proof. Assume the opposite. Suppose that $b_{j}=M, j \in\{n, n+1, n+2\}$, and let $b_{j+1}=x$ and $b_{j+2}=y$. Thus $b_{j+3}=M-y$. If $x, y, M-y$ are all less than $M$, then the contradiction is immediate. The remaining cases are these: +(i) $x=M$. Then the sequence has the form $M, M, y, M-y, y, \ldots$, and since $\max (y, M-y, y)=M$, we must have $y=0$ or $y=M$. +(ii) $y=M$. Then the sequence has the form $M, x, M, 0, x, M-x, \ldots$, and since $\max (0, x, M-x)=M$, we must have $x=0$ or $x=M$. +(iii) $y=0$. Then the sequence is $M, x, 0, M, M-x, M-x, x, \ldots$, and since $\max (M-x, x, x)=M$, we have $x=0$ or $x=M$. +In every case $M$ divides both $x$ and $y$. From the recurrence formula $M$ also divides $b_{i}$ for every $ib>c>d$ that + +$$ +a b+c d>a c+b d>a d+b c +$$ + +hence $a c+b d$ is relatively prime with $a b+c d$. But then (1) implies that $a c+b d$ divides $a d+b c$, which is impossible by (2). +Remark. Alternatively, (1) could be obtained by applying the law of cosines and Ptolemy's theorem on a quadrilateral $X Y Z T$ with $X Y=a$, $Y Z=c, Z T=b, T X=d$ and $\angle Y=60^{\circ}, \angle T=120^{\circ}$. +28. Yes. The desired result is an immediate consequence of the following fact applied on $p=101$. +Lemma. For any odd prime number $p$, there exist $p$ nonnegative integers less than $2 p^{2}$ with all pairwise sums mutually distinct. +Proof. We claim that the numbers $a_{n}=2 n p+\left(n^{2}\right)$ have the desired property, where $(x)$ denotes the remainder of $x$ upon division by $p$. Suppose that $a_{k}+a_{l}=a_{m}+a_{n}$. By the construction of $a_{i}$, we have $2 p(k+l) \leq a_{k}+a_{l}<2 p(k+l+1)$. Hence we must have $k+l=m+n$, and therefore also $\left(k^{2}\right)+\left(l^{2}\right)=\left(m^{2}\right)+\left(n^{2}\right)$. Thus + +$$ +k+l \equiv m+n \quad \text { and } \quad k^{2}+l^{2} \equiv m^{2}+n^{2} \quad(\bmod p) . +$$ + +But then it holds that $(k-l)^{2}=2\left(k^{2}+l^{2}\right)-(k+l)^{2} \equiv(m-n)^{2}(\bmod$ $p)$, so $k-l \equiv \pm(m-n)$, which leads to $(k, l)=(m, n)$. This proves the lemma. + +### 4.43 Solutions to the Shortlisted Problems of IMO 2002 + +1. Consider the given equation modulo 9 . Since each cube is congruent to either $-1,0$ or 1 , whereas $2002^{2002} \equiv 4^{2002}=4 \cdot 64^{667} \equiv 4(\bmod 9)$, we conclude that $t \geq 4$. +On the other hand, $2002^{2002}=2002 \cdot\left(2002^{667}\right)^{3}=\left(10^{3}+10^{3}+1^{3}+\right.$ $\left.1^{3}\right)\left(2002^{667}\right)^{3}$ is a solution with $t=4$. Hence the answer is 4. +2. Set $S=d_{1} d_{2}+\cdots+d_{k-1} d_{k}$. Since $d_{i} / n=1 / d_{k+1-i}$, we have $\frac{S}{n^{2}}=$ $\frac{1}{d_{k} d_{k-1}}+\cdots+\frac{1}{d_{2} d_{1}}$. Hence + +$$ +\frac{1}{d_{2} d_{1}} \leq \frac{S}{n^{2}} \leq\left(\frac{1}{d_{k-1}}-\frac{1}{d_{k}}\right)+\cdots+\left(\frac{1}{d_{1}}-\frac{1}{d_{2}}\right)=1-\frac{1}{d_{k}}<1 +$$ + +or $\left(\right.$ since $\left.d_{1}=1\right) 1<\frac{n^{2}}{S} \leq d_{2}$. This shows that $S2^{a b}>2^{2 a} 2^{2 b}>Q^{2}$ if $a, b \geq 5$ ). Then $N$ has at least twice as many divisors as $Q$ (because for every $d \mid Q$ both $d$ and $N / d$ are divisors of $N$ ), which counts up to $4^{n}$ divisors, as required. +Remark. With some knowledge of cyclotomic polynomials, one can show that $2^{p_{1} \cdots p_{n}}+1$ has at least $2^{2^{n-1}}$ divisors, far exceeding $4^{n}$. +4. For $a=b=c=1$ we obtain $m=12$. We claim that the given equation has infinitely many solutions in positive integers $a, b, c$ for this value of $m$. After multiplication by $a b c(a+b+c)$ the equation $\frac{1}{a}+\frac{1}{b}+\frac{1}{c}+\frac{1}{a b c}-\frac{12}{a+b+c}=0$ becomes + +$$ +a^{2}(b+c)+b^{2}(c+a)+c^{2}(a+b)+a+b+c-9 a b c=0 +$$ + +We must show that this equation has infinitely many solutions in positive integers. Suppose that $(a, b, c)$ is one such solution with $a1$. +We show that all $a_{n}$ 's are integers. This procedure is fairly standard. The above relation for $n$ and $n-1$ gives $a_{n+1} a_{n-2}=a_{n} a_{n-1}+1$ and $a_{n-1} a_{n-2}+1=a_{n} a_{n-3}$, so that adding yields $a_{n-2}\left(a_{n-1}+a_{n+1}\right)=$ $a_{n}\left(a_{n-1}+a_{n-3}\right)$. Therefore $\frac{a_{n+1}+a_{n-1}}{a_{n}}=\frac{a_{n-1}+a_{n-3}}{a_{n-2}}=\cdots$, from which it follows that + +$$ +\frac{a_{n+1}+a_{n-1}}{a_{n}}=\left\{\begin{array}{l} +\frac{a_{2}+a_{0}}{a_{1}}=2 \text { for } n \text { odd; } \\ +\frac{a_{3}+a_{1}}{a_{2}}=3 \text { for } n \text { even. } +\end{array}\right. +$$ + +It is now an immediate consequence that every $a_{n}$ is integral. Also, the above consideration implies that $\left(a_{n-1}, a_{n}, a_{n+1}\right)$ is a solution of (1) for each $n \geq 1$. Since $a_{n}$ is strictly increasing, this gives an infinity of solutions in integers. +Remark. There are infinitely many values of $m \in \mathbb{N}$ for which the given equation has at least one solution in integers, and each of those values admits an infinity of solutions. +5. Consider all possible sums $c_{1} a_{1}+c_{2} a_{2}+\cdots+c_{n} a_{n}$, where each $c_{i}$ is an integer with $0 \leq c_{i}$ $\alpha^{m}+\alpha-1=0$; hence $m<2 n$. +Now we have $F(x) / G(x)=x^{m-n}-\left(x^{m-n+2}-x^{m-n}-x+1\right) / G(x)$, so $H(x)=x^{m-n+2}-x^{m-n}-x+1$ is also divisible by $G(x)$; but $\operatorname{deg} H(x)=$ $m-n+2 \leq n+1=\operatorname{deg} G(x)+1$, from which we deduce that either $H(x)=G(x)$ or $H(x)=(x-a) G(x)$ for some $a \in \mathbb{Z}$. The former case is impossible. In the latter case we must have $m=2 n-1$, and thus $H(x)=x^{n+1}-x^{n-1}-x+1$; on the other hand, putting $x=1$ gives $a=1$, so $H(x)=(x-1)\left(x^{n}+x^{2}-1\right)=x^{n+1}-x^{n}+x^{3}-x^{2}-x+1$. This is possible only if $n=3$ and $m=5$. +Remark. It is an old (though difficult) result that the polynomial $x^{n} \pm$ $x^{k} \pm 1$ is either irreducible or equals $x^{2} \pm x+1$ times an irreducible factor. +7. To avoid working with cases, we use oriented angles modulo $180^{\circ}$. Let $K$ be the circumcenter of $\triangle B C D$, and $X$ any point on the common tangent to the circles at $D$. Since the tangents at the ends of a chord are equally inclined to the chord, we have $\angle B A C=\angle A B D+\angle B D C+\angle D C A=$ $\angle B D X+\angle B D C+\angle X D C=2 \angle B D C=\angle B K C$. It follows that $B, C, A, K$ are concyclic, as required. +8. Construct equilateral triangles $A C P$ and $A B Q$ outside the triangle $A B C$. Since $\angle A P C+\angle A F C=60^{\circ}+120^{\circ}=180^{\circ}$, the points $A, C, F, P$ lie on a circle; hence $\angle A F P=\angle A C P=60^{\circ}=\angle A F D$, so $D$ lies on the segment $F P$; similarly, $E$ lies on $F Q$. Further, note that + +$$ +\frac{F P}{F D}=1+\frac{D P}{F D}=1+\frac{S_{A P C}}{S_{A F C}} \geq 4 +$$ + +with equality if $F$ is the midpoint of the smaller $\operatorname{arc} A C$ : hence $F D \leq \frac{1}{4} F P$ and $F E \leq \frac{1}{4} F Q$. Then by the law of cosines, + +$$ +\begin{aligned} +D E & =\sqrt{F D^{2}+F E^{2}+F D \cdot F E} \\ +& \leq \frac{1}{4} \sqrt{F P^{2}+F Q^{2}+F P \cdot F Q}=\frac{1}{4} P Q \leq A P+A Q=A B+A C . +\end{aligned} +$$ + +Equality holds if and only if $\triangle A B C$ is equilateral. +9. Since $\angle B C A=\frac{1}{2} \angle B O A=\angle B O D$, the lines $C A$ and $O D$ are parallel, so that $O D A I$ is a parallelogram. It follows that $A I=O D=O E=A E=$ $A F$. Hence +$\angle I F E=\angle I F A-\angle E F A=\angle A I F-\angle E C A=\angle A I F-\angle A C F=\angle C F I$. +Also, from $A E=A F$ we get that $C I$ bisects $\angle E C F$. Therefore $I$ is the incenter of $\triangle C E F$. +10. Let $O$ be the circumcenter of $A_{1} A_{2} C$, and $O_{1}, O_{2}$ the centers of $S_{1}, S_{2}$ respectively. +First, from $\angle A_{1} Q A_{2}=180^{\circ}-\angle P A_{1} Q-\angle Q A_{2} P=\frac{1}{2}\left(360^{\circ}-\angle P O_{1} Q-\right.$ $\left.\angle Q O_{2} P\right)=\angle O_{1} Q O_{2}$ we obtain $\angle A_{1} Q A_{2}=\angle B_{1} Q B_{2}=\angle O_{1} Q O_{2}$. +Therefore $\angle A_{1} Q A_{2}=\angle B_{1} Q P+$ $\angle P Q B_{2}=\angle C A_{1} P+\angle C A_{2} P=$ $180^{\circ}-\angle A_{1} C A_{2}$, from which we conclude that $Q$ lies on the circumcircle of $\triangle A_{1} A_{2} C$. Hence $O A_{1}=$ $O Q$. However, we also have $O_{1} A_{2}=$ $O_{1} Q$. Consequently, $O, O_{1}$ both lie on the perpendicular bisector of $A_{1} Q$, so $O O_{1} \perp A_{1} Q$. Similarly, $O O_{2} \perp A_{2} Q$, leading to $\angle O_{2} O O_{1}=$ +![](https://cdn.mathpix.com/cropped/2024_11_18_8e985d6b9c83aa3e9d0eg-701.jpg?height=412&width=518&top_left_y=598&top_left_x=825) +$180^{\circ}-\angle A_{1} Q A_{2}=180^{\circ}-\angle O_{1} Q O_{2}$. Hence, $O$ lies on the circle through $O_{1}, O_{2}, Q$, which is fixed. +11. When $S$ is the set of vertices of a regular pentagon, then it is easily verified that $\frac{M(S)}{m(S)}=\frac{1+\sqrt{5}}{2}=\alpha$. We claim that this is the best possible. Let $A, B, C, D, E$ be five arbitrary points, and assume that $\triangle A B C$ has the area $M(S)$. We claim that some triangle has area less than or equal to $M(S) / \alpha$. +Construct a larger triangle $A^{\prime} B^{\prime} C^{\prime}$ with $C \in A^{\prime} B^{\prime}\left\|A B, A \in B^{\prime} C^{\prime}\right\| B C$, $B \in C^{\prime} A^{\prime} \| C A$. The point $D$, as well as $E$, must lie on the same side of $B^{\prime} C^{\prime}$ as $B C$, for otherwise $\triangle D B C$ would have greater area than $\triangle A B C$. A similar result holds for the other edges, and therefore $D, E$ lie inside the triangle $A^{\prime} B^{\prime} C^{\prime}$ or on its boundary. Moreover, at least one of the triangles $A^{\prime} B C, A B^{\prime} C, A B C^{\prime}$, say $A B C^{\prime}$, contains neither $D$ nor $E$. Hence we can assume that $D, E$ are contained inside the quadrilateral $A^{\prime} B^{\prime} A B$. +An affine linear transformation does not change the ratios between areas. Thus if we apply such an affine transformation mapping $A, B, C$ into the vertices $A B M C N$ of a regular pentagon, we won't change $M(S) / m(S)$. If now $D$ or $E$ lies inside $A B M C N$, then we are done. Suppose that both $D$ and $E$ are inside the triangles $C M A^{\prime}, C N B^{\prime}$. Then $C D, C E \leq C M$ (because $C M=C N=C A^{\prime}=C B^{\prime}$ ) and $\angle D C E$ is either less than or equal to $36^{\circ}$ or greater than or equal to $108^{\circ}$, from which we obtain that the area of $\triangle C D E$ cannot exceed the area of $\triangle C M N=M(S) / \alpha$. This completes the proof. +12. Let $l(M N)$ denote the length of the shorter $\operatorname{arc} M N$ of a given circle. + +Lemma. Let $P R, Q S$ be two chords of a circle $k$ of radius $r$ that meet each other at a point $X$, and let $\angle P X Q=\angle R X S=2 \alpha$. Then $l(P Q)+$ $l(R S)=4 \alpha r$. +Proof. Let $O$ be the center of the circle. Then $l(P Q)+l(R S)=\angle P O Q$. $r+\angle R O S \cdot r=2(\angle Q S P+\angle R P S) r=2 \angle Q X P \cdot r=4 \alpha r$. +Consider a circle $k$, sufficiently large, whose interior contains all the given circles. For any two circles $C_{i}, C_{j}$, let their exterior common tangents $P R, Q S(P, Q, R, S \in k)$ form an angle $2 \alpha$. Then $O_{i} O_{j}=\frac{2}{\sin \alpha}$, so $\alpha>$ $\sin \alpha=\frac{2}{O_{i} O_{j}}$. By the lemma we have $l(P Q)+l(R S)=4 \alpha r \geq \frac{8 r}{O_{i} O_{j}}$, and hence + +$$ +\frac{1}{O_{i} O_{j}} \leq \frac{l(P Q)+l(R S)}{8 r} +$$ + +Now sum all these inequalities for $iA B$. As usual, $R, r, \alpha, \beta, \gamma$ denote the circumradius and the inradius and the angles of $\triangle A B C$, respectively. +We have $\tan \angle B K M=D M / D K$. Straightforward calculation gives $D M=\frac{1}{2} A D=R \sin \beta \sin \gamma$ and $D K=\frac{D C-D B}{2}-\frac{K C-K B}{2}=R \sin (\beta-$ $\gamma)-R(\sin \beta-\sin \gamma)=4 R \sin \frac{\beta-\gamma}{2} \sin \frac{\beta}{2} \sin \frac{\gamma}{2}$, so we obtain + +$$ +\tan \angle B K M=\frac{\sin \beta \sin \gamma}{4 \sin \frac{\beta-\gamma}{2} \sin \frac{\beta}{2} \sin \frac{\gamma}{2}}=\frac{\cos \frac{\beta}{2} \cos \frac{\gamma}{2}}{\sin \frac{\beta-\gamma}{2}} +$$ + +To calculate the angle $B K N^{\prime}$, we apply the inversion $\psi$ with center at $K$ and power $B K \cdot C K$. For each object $X$, we denote by $\widehat{X}$ its image under $\psi$. The incircle $\Omega$ maps to a +![](https://cdn.mathpix.com/cropped/2024_11_18_8e985d6b9c83aa3e9d0eg-702.jpg?height=215&width=545&top_left_y=1844&top_left_x=807) +line $\widehat{\Omega}$ parallel to $\widehat{B} \widehat{C}$, at distance $\frac{B K \cdot C K}{2 r}$ from $\widehat{B} \widehat{C}$. Thus the point $\widehat{N^{\prime}}$ is the projection of the midpoint $\widehat{U}$ of $\widehat{B} \widehat{C}$ onto $\widehat{\Omega}$. Hence + +$$ +\tan \angle B K N^{\prime}=\tan \angle \widehat{B} K \widehat{N^{\prime}}=\frac{\widehat{U} \widehat{N^{\prime}}}{\widehat{U} K}=\frac{B K \cdot C K}{r(C K-B K)} . +$$ + +Again, one easily checks that $K B \cdot K C=b c \sin ^{2} \frac{\alpha}{2}$ and $r=4 R \sin \frac{\alpha}{2}$. $\sin \frac{\beta}{2} \cdot \sin \frac{\gamma}{2}$, which implies + +$$ +\begin{aligned} +\tan \angle B K N^{\prime} & =\frac{b c \sin ^{2} \frac{\alpha}{2}}{r(b-c)} \\ +& =\frac{4 R^{2} \sin \beta \sin \gamma \sin ^{2} \frac{\alpha}{2}}{4 R \sin \frac{\alpha}{2} \sin \frac{\beta}{2} \sin \frac{\gamma}{2} \cdot 2 R(\sin \beta-\sin \gamma)}=\frac{\cos \frac{\beta}{2} \cos \frac{\gamma}{2}}{\sin \frac{\beta-\gamma}{2}} . +\end{aligned} +$$ + +Hence $\angle B K M=\angle B K N^{\prime}$, which implies that $K, M, N^{\prime}$ are indeed collinear; thus $N^{\prime} \equiv N$. +14. Let $G$ be the other point of intersection of the line $F K$ with the arc $B A D$. Since $B N / N C=D K / K B$ and $\angle C E B=\angle B G D$ the triangles $C E B$ and $B G D$ are similar. Thus $B N / N E=D K / K G=F K / K B$. From $B N=M K$ and $B K=$ $M N$ it follows that $M N / N E=$ $F K / K M$. But we also have that $\angle M N E=90^{\circ}+\angle M N B=90^{\circ}+$ +![](https://cdn.mathpix.com/cropped/2024_11_18_8e985d6b9c83aa3e9d0eg-703.jpg?height=327&width=539&top_left_y=1011&top_left_x=817) +$\angle M K B=\angle F K M$, and hence $\triangle M N E \sim \triangle F K M$. +Now $\angle E M F=\angle N M K-\angle N M E-\angle K M F=\angle N M K-\angle N M E-$ $\angle N E M=\angle N M K-90^{\circ}+\angle B N M=90^{\circ}$ as claimed. +15. We observe first that $f$ is surjective. Indeed, setting $y=-f(x)$ gives $f(f(-f(x))-x)=f(0)-2 x$, where the right-hand expression can take any real value. +In particular, there exists $x_{0}$ for which $f\left(x_{0}\right)=0$. Now setting $x=x_{0}$ in the functional equation yields $f(y)=2 x_{0}+f\left(f(y)-x_{0}\right)$, so we obtain + +$$ +f(z)=z-x_{0} \quad \text { for } z=f(y)-x_{0} +$$ + +Since $f$ is surjective, $z$ takes all real values. Hence for all $z, f(z)=z+c$ for some constant $c$, and this is indeed a solution. +16. For $n \geq 2$, let $\left(k_{1}, k_{2}, \ldots, k_{n}\right)$ be the permutation of $\{1,2, \ldots, n\}$ with $a_{k_{1}} \leq a_{k_{2}} \leq \cdots \leq a_{k_{n}}$. Then from the condition of the problem, using the Cauchy-Schwarz inequality, we obtain + +$$ +\begin{aligned} +c & \geq a_{k_{n}}-a_{k_{1}}=\left|a_{k_{n}}-a_{k_{n-1}}\right|+\cdots+\left|a_{k_{3}}-a_{k_{2}}\right|+\left|a_{k_{2}}-a_{k_{1}}\right| \\ +& \geq \frac{1}{k_{1}+k_{2}}+\frac{1}{k_{2}+k_{3}}+\cdots+\frac{1}{k_{n-1}+k_{n}} \\ +& \geq \frac{(n-1)^{2}}{\left(k_{1}+k_{2}\right)+\left(k_{2}+k_{3}\right)+\cdots+\left(k_{n-1}+k_{n}\right)} \\ +& =\frac{(n-1)^{2}}{2\left(k_{1}+k_{2}+\cdots+k_{n}\right)-k_{1}-k_{n}} \geq \frac{(n-1)^{2}}{n^{2}+n-3} \geq \frac{n-1}{n+2} . +\end{aligned} +$$ + +Therefore $c \geq 1-\frac{3}{n+2}$ for every positive integer $n$. But if $c<1$, this inequality is obviously false for all $n>\frac{3}{1-c}-2$. We conclude that $c \geq 1$. Remark. The least value of $c$ is not greater than $2 \ln 2$. An example of a sequence $\left\{a_{n}\right\}$ with $0 \leq a_{n} \leq 2 \ln 2$ can be constructed inductively as follows: Given $a_{1}, a_{2}, \ldots, a_{n-1}$, then $a_{n}$ can be any number from $[0,2 \ln 2]$ that does not belong to any of the intervals $\left(a_{i}-\frac{1}{i+n}, a_{i}+\frac{1}{i+n}\right)(i=$ $1,2, \ldots, n-1)$, and the total length of these intervals is always less than or equal to + +$$ +\frac{2}{n+1}+\frac{2}{n+2}+\cdots+\frac{2}{2 n-1}<2 \ln 2 +$$ + +17. Let $x, y$ be distinct integers satisfying $x P(x)=y P(y)$; this is equivalent to $a\left(x^{4}-y^{4}\right)+b\left(x^{3}-y^{3}\right)+c\left(x^{2}-y^{2}\right)+d(x-y)=0$. Dividing by $x-y$ we obtain + +$$ +a\left(x^{3}+x^{2} y+x y^{2}+y^{3}\right)+b\left(x^{2}+x y+y^{2}\right)+c(x+y)+d=0 . +$$ + +Putting $x+y=p, x^{2}+y^{2}=q$ leads to $x^{2}+x y+y^{2}=\frac{p^{2}+q}{2}$, so the above equality becomes + +$$ +a p q+\frac{b}{2}\left(p^{2}+q\right)+c p+d=0, \quad \text { i.e. } \quad(2 a p+b) q=-\left(b p^{2}+2 c p+2 d\right) +$$ + +Since $q \geq p^{2} / 2$, it follows that $p^{2}|2 a p+b| \leq 2\left|b p^{2}+2 c p+2 d\right|$, which is possible only for finitely many values of $p$, although there are infinitely many pairs $(x, y)$ with $x P(x)=y P(y)$. Hence there exists $p$ such that $x P(x)=(p-x) P(p-x)$ for infinitely many $x$, and therefore for all $x$. If $p \neq 0$, then $p$ is a root of $P(x)$. If $p=0$, the above relation gives $P(x)=-P(-x)$. This forces $b=d=0$, so $P(x)=x\left(a x^{2}+c\right)$. Thus 0 is a root of $P(x)$. +18. Putting $x=z=0$ and $t=y$ into the given equation gives $4 f(0) f(y)=$ $2 f(0)$ for all $y$. If $f(0) \neq 0$, then we deduce $f(y)=\frac{1}{2}$, i.e., $f$ is identically equal to $\frac{1}{2}$. +Now we suppose that $f(0)=0$. Setting $z=t=0$ we obtain + +$$ +f(x y)=f(x) f(y) \quad \text { for all } x, y \in \mathbb{R} +$$ + +Thus if $f(y)=0$ for some $y \neq 0$, then $f$ is identically zero. So, assume $f(y) \neq 0$ whenever $y \neq 0$. + +Next, we observe that $f$ is strictly increasing on the set of positive reals. Actually, it follows from (1) that $f(x)=f(\sqrt{x})^{2} \geq 0$ for all $x \geq 0$, so that the given equation for $t=x$ and $z=y$ yields $f\left(x^{2}+y^{2}\right)=(f(x)+f(y))^{2} \geq$ $f\left(x^{2}\right)$ for all $x, y \geq 0$. +Using (1) it is easy to get $f(1)=1$. Now plugging $t=y$ into the given equation, we are led to + +$$ +2[f(x)+f(z)]=f(x-z)+f(x+z) \quad \text { for all } x, z +$$ + +In particular, $f(z)=f(-z)$. Further, it is easy to get by induction from (2) that $f(n x)=n^{2} f(x)$ for all integers $n$ (and consequently for all rational numbers as well). Therefore $f(q)=f(-q)=q^{2}$ for all $q \in \mathbb{Q}$. But $f$ is increasing for $x>0$, so we must have $f(x)=x^{2}$ for all $x$. It is easy to verify that $f(x)=0, f(x)=\frac{1}{2}$ and $f(x)=x^{2}$ are indeed solutions. +19. Write $m=[\sqrt[3]{n}]$. To simplify the calculation, we shall assume that $[b]=1$. Then $a=\sqrt[3]{n}, b=\frac{1}{\sqrt[3]{n}-m}=\frac{1}{n-m^{3}}\left(m^{2}+m \sqrt[3]{n}+\sqrt[3]{n^{2}}\right), c=\frac{1}{b-1}=$ $u+v \sqrt[3]{n}+w \sqrt[3]{n^{2}}$ for certain rational numbers $u, v, w$. Obviously, integers $r, s, t$ with $r a+s b+t c=0$ exist if (and only if) $u=m^{2} w$, i.e., if ( $b-$ 1) $\left(m^{2} w+v \sqrt[3]{n}+w \sqrt[3]{n^{2}}\right)=1$ for some rational $v, w$. + +When the last equality is expanded and simplified, comparing the coefficients at $1, \sqrt[3]{n}, \sqrt[3]{n^{2}}$ one obtains + +$$ +\begin{array}{rlrl} +1: & v+\left(\left(m^{2}+m^{3}-n\right) m^{2}+m\right) w & =n-m^{3}, \\ +\sqrt[3]{n}: & \left(m^{2}+m^{3}-n\right) v+ & \left(m^{3}+n\right) w & =0, \\ +\sqrt[3]{n^{2}}: & m v+ & \left(2 m^{2}+m^{3}-n\right) w & =0 . +\end{array} +$$ + +In order for the system (1) to have a solution $v, w$, we must have $\left(2 m^{2}+\right.$ $\left.m^{3}-n\right)\left(m^{2}+m^{3}-n\right)=m\left(m^{3}+n\right)$. This quadratic equation has solutions $n=m^{3}$ and $n=m^{3}+3 m^{2}+m$. The former is not possible, but the latter gives $a-[a]>\frac{1}{2}$, so $[b]=1$, and the system (1) in $v, w$ is solvable. Hence every number $n=m^{3}+3 m^{2}+m, m \in \mathbb{N}$, satisfies the condition of the problem. +20. Assume to the contrary that $\frac{1}{b_{1}}+\cdots+\frac{1}{b_{n}}>1$. Certainly $n \geq 2$ and $A$ is infinite. Define $f_{i}: A \rightarrow A$ as $f_{i}(x)=b_{i} x+c_{i}$ for each $i$. By condition (ii), $f_{i}(x)=f_{j}(y)$ implies $i=j$ and $x=y$; iterating this argument, we deduce that $f_{i_{1}}\left(\ldots f_{i_{m}}(x) \ldots\right)=f_{j_{1}}\left(\ldots f_{j_{m}}(x) \ldots\right)$ implies $i_{1}=j_{1}, \ldots, i_{m}=j_{m}$ and $x=y$. +As an illustration, we shall consider the case $b_{1}=b_{2}=b_{3}=2$ first. If $a$ is large enough, then for any $i_{1}, \ldots, i_{m} \in\{1,2,3\}$ we have $f_{i_{1}} \circ \cdots \circ f_{i_{m}}(a) \leq$ $2.1^{m} a$. However, we obtain $3^{m}$ values in this way, so they cannot be all distinct if $m$ is sufficiently large, a contradiction. +In the general case, let real numbers $d_{i}>b_{i}, i=1,2 \ldots, n$, be chosen such that $\frac{1}{d_{1}}+\cdots+\frac{1}{d_{n}}>1$ : for $a$ large enough, $f_{i}(x)0$ be arbitrary rational numbers with sum 1 ; denote by $N_{0}$ the least common multiple of their denominators. +Let $N$ be a fixed multiple of $N_{0}$, so that each $k_{j} N$ is an integer. Consider all combinations $f_{i_{1}} \circ \cdots \circ f_{i_{N}}$ of $N$ functions, among which each $f_{i}$ appears exactly $k_{i} N$ times. There are $F_{N}=\frac{N!}{\left(k_{1} N\right)!\cdots\left(k_{n} N\right)!}$ such combinations, so they give $F_{N}$ distinct values when applied to $a$. On the other hand, $f_{i_{1}} \circ \cdots \circ f_{i_{N}}(a) \leq\left(d_{1}^{k_{1}} \cdots d_{n}^{k_{n}}\right)^{N} a$. Therefore + +$$ +\left(d_{1}^{k_{1}} \cdots d_{n}^{k_{n}}\right)^{N} a \geq F_{N} \quad \text { for all } N, N_{0} \mid N +$$ + +It remains to find a lower estimate for $F_{N}$. In fact, it is straightforward to verify that $F_{N+N_{0}} / F_{N}$ tends to $Q^{N_{0}}$, where $Q=1 /\left(k_{1}^{k_{1}} \cdots k_{n}^{k_{n}}\right)$. Consequently, for every $q0$ such that $F_{N}>p q^{N}$. Then (1) implies that + +$$ +\left(\frac{d_{1}^{k_{1}} \cdots d_{n}^{k_{n}}}{q}\right)^{N}>\frac{p}{a} \text { for every multiple } N \text { of } N_{0} +$$ + +and hence $d_{1}^{k_{1}} \cdots d_{n}^{k_{n}} / q \geq 1$. This must hold for every $q7$ this is possible as well: it follows by induction from Figure 2. +![](https://cdn.mathpix.com/cropped/2024_11_18_8e985d6b9c83aa3e9d0eg-707.jpg?height=186&width=195&top_left_y=302&top_left_x=402) + +Fig. 1 +![](https://cdn.mathpix.com/cropped/2024_11_18_8e985d6b9c83aa3e9d0eg-707.jpg?height=239&width=249&top_left_y=251&top_left_x=903) + +Fig. 2 +23. We claim that there are $n$ ! full sequences. To show this, we construct a bijection with the set of permutations of $\{1,2, \ldots, n\}$. +Consider a full sequence $\left(a_{1}, a_{2}, \ldots, a_{n}\right)$, and let $m$ be the greatest of the numbers $a_{1}, \ldots, a_{n}$. Let $S_{k}, 1 \leq k \leq m$, be the set of those indices $i$ for which $a_{i}=k$. Then $S_{1}, \ldots S_{m}$ are nonempty and form a partition of the set $\{1,2, \ldots, n\}$. Now we write down the elements of $S_{1}$ in descending order, then the elements of $S_{2}$ in descending order and so on. This maps the full sequence to a permutation of $\{1,2, \ldots, n\}$. Moreover, this map is reversible, since each permutation uniquely breaks apart into decreasing sequences $S_{1}^{\prime}, S_{2}^{\prime}, \ldots, S_{m}^{\prime}$, so that $\max S_{i}^{\prime}>\min S_{i-1}^{\prime}$. Therefore the full sequences are in bijection with the permutations of $\{1,2, \ldots, n\}$. +Second solution. Let there be given a full sequence of length $n$. Removing from it the first occurrence of the highest number, we obtain a full sequence of length $n-1$. On the other hand, each full sequence of length $n-1$ can be obtained from exactly $n$ full sequences of length $n$. Therefore, if $x_{n}$ is the number of full sequences of length $n$, we deduce $x_{n}=n x_{n-1}$. +24. Two moves are not sufficient. Indeed, the answer to each move is an even number between 0 and 54 , so the answer takes at most 28 distinct values. Consequently, two moves give at most $28^{2}=784$ distinct outcomes, which is less than $10^{3}=1000$. +We now show that three moves are sufficient. With the first move $(0,0,0)$, we get the reply $2(x+y+z)$, so we now know the value of $s=x+y+z$. Now there are several cases: +(i) $s \leq 9$. Then we ask $(9,0,0)$ as the second move and get $(9-x-y)+$ $(9-x-z)+(y+z)=18-2 x$, so we come to know $x$. Asking $(0,9,0)$ we obtain $y$, which is enough, since $z=s-x-y$. +(ii) $10 \leq s \leq 17$. In this case the second move is $(9, s-9,0)$. The answer is $z+(9-x)+|x+z-9|=2 k$, where $k=z$ if $x+z \geq 9$, or $k=9-x$ if $x+z<9$. In both cases we have $z \leq k \leq y+z \leq s$. +Let $s-k \leq 9$. Then in the third move we ask $(s-k, 0, k)$ and obtain $|z-k|+|k-y-z|+y$, which is actually $(k-z)+(y+z-k)+y=2 y$. Thus we also find out $x+z$, and thus deduce whether $k$ is $z$ or $9-x$. Consequently we determine both $x$ and $z$. +Let $s-k>9$. In this case, the third move is $(9, s-k-9, k)$. The answer is $|s-k-x-y|+|s-9-y-z|+|k+9-z-x|=$ $(k-z)+(9-x)+(9-x+k-z)=18+2 k-2(x+z)$, from which we find out again whether $k$ is $z$ or $9-x$. Now we are easily done. +(iii) $18 \leq s \leq 27$. Then as in the first case, asking $(0,9,9)$ and $(9,0,9)$ we obtain $x$ and $y$. +25. Assume to the contrary that no set of size less than $r$ meets all sets in $\mathcal{F}$. Consider any set $A$ of size less than $r$ that is contained in infinitely many sets of $\mathcal{F}$. By the assumption, $A$ is disjoint from some set $B \in \mathcal{F}$. Then of the infinitely many sets that contain $A$, each must meet $B$, so some element $b$ of $B$ belongs to infinitely many of them. But then the set $A \cup\{b\}$ is contained in infinitely many sets of $\mathcal{F}$ as well. +Such a set $A$ exists: for example, the empty set. Now taking for $A$ the largest such set we come to a contradiction. +26. Write $n=2 m$. We shall define a directed graph $G$ with vertices $1, \ldots, m$ and edges labelled $1,2, \ldots, 2 m$ in such a way that the edges issuing from $i$ are labelled $2 i-1$ and $2 i$, and those entering it are labelled $i$ and $i+m$. What we need is an Euler circuit in $G$, namely a closed path that passes each edge exactly once. Indeed, if $x_{i}$ is the $i$ th edge in such a circuit, then $x_{i}$ enters some vertex $j$ and $x_{i+1}$ leaves it, so $x_{i} \equiv j(\bmod m)$ and $x_{i+1}=2 j-1$ or $2 j$. Hence $2 x_{i} \equiv 2 j$ and $x_{i+1} \equiv 2 x_{i}$ or $2 x_{i}-1(\bmod n)$, as required. +The graph $G$ is connected: by induction on $k$ there is a path from 1 to $k$, since 1 is connected to $j$ with $2 j=k$ or $2 j-1=k$, and there is an edge from $j$ to $k$. Also, the in-degree and out-degree of each vertex of $G$ are equal (to 2), and thus by a known result, $G$ contains an Euler circuit. +27. For a graph $G$ on 120 vertices (i.e., people at the party), write $q(G)$ for the number of weak quartets in $G$. Our solution will consist of three parts. First, we prove that some graph $G$ with maximal $q(G)$ breaks up as a disjoint union of complete graphs. This will follow if we show that any two adjacent vertices $x, y$ have the same neighbors (apart from themselves). Let $G_{x}$ be the graph obtained from $G$ by "copying" $x$ to $y$ (i.e., for each $z \neq x, y$, we add the edge $z y$ if $z x$ is an edge, and delete $z y$ if $z x$ is not an edge). Similarly $G_{y}$ is the graph obtained from $G$ by copying $y$ to $x$. We claim that $2 q(G) \leq q\left(G_{x}\right)+q\left(G_{y}\right)$. Indeed, the number of weak quartets containing neither $x$ nor $y$ is the same in $G, G_{x}$, and $G_{y}$, while the number of those containing both $x$ and $y$ is not less in $G_{x}$ and $G_{y}$ than in $G$. Also, the number containing exactly one of $x$ and $y$ in $G_{x}$ is at least twice the number in $G$ containing $x$ but not $y$, while the number containing exactly one of $x$ and $y$ in $G_{y}$ is at least twice the number in $G$ containing $y$ but not $x$. This justifies our claim by adding. It follows that for an extremal graph $G$ we must have $q(G)=q\left(G_{x}\right)=q\left(G_{y}\right)$. Repeating the copying operation pair by pair ( $y$ to $x$, then their common neighbor $z$ to both $x, y$, etc.) we eventually obtain an extremal graph consisting of disjoint complete graphs. +Second, suppose the complete graphs in $G$ have sizes $a_{1}, a_{2}, \ldots, a_{n}$. Then + +$$ +q(G)=\sum_{i=1}^{n}\binom{a_{i}}{2} \sum_{\substack{j0$ with $\left.c_{1}+c_{2}+c_{3}=1\right)$. +Let $P^{\prime}\left(a_{11}, a_{21}, 0\right), Q^{\prime}\left(a_{12}, a_{22}, 0\right)$, and $R^{\prime}\left(a_{13}, a_{23}, 0\right)$ be the projections of $P, Q$, and $R$ onto the $O x y$ plane. We see that $P^{\prime}, Q^{\prime}, R^{\prime}$ lie in the fourth, second, and third quadrants, respectively. We have the following two cases: +(i) $O$ is in the exterior of $\triangle P^{\prime} Q^{\prime} R^{\prime}$. Set $S^{\prime}=O R^{\prime} \cap P^{\prime} Q^{\prime}$ and let $S$ be the point of the segment $P Q$ that projects to $S^{\prime}$. The point $S$ has its $z$ coordinate negative (because the $z$ coordinates of $P$ and $Q$ are negative). Thus any point +![](https://cdn.mathpix.com/cropped/2024_11_18_8e985d6b9c83aa3e9d0eg-710.jpg?height=290&width=446&top_left_y=702&top_left_x=875) +of the segment $S R$ sufficiently close to $S$ has all coordinates negative. +(ii) $O$ is in the interior or on the boundary of $\triangle P^{\prime} Q^{\prime} R^{\prime}$. + +Let $T$ be the point in the plane $P Q R$ whose projection is $O$. If $T=O$, then all coordinates of $T$ are zero, and we are done. Otherwise $O$ is interior to $\triangle P^{\prime} Q^{\prime} R^{\prime}$. Suppose that the $z$ coordinate of $T$ is positive (negative). Since $x$ and $y$ coordinates of $T$ are equal to 0 , there is a point $U$ inside $P Q R$ close to $T$ with both $x$ and $y$ coordinates positive (respectively negative), and this point $U$ has all coordinates of the same sign. +2. We can rewrite (ii) as $-(f(a)-1)(f(b)-1)=f(-(a-1)(b-1)+1)-1$. So putting $g(x)=f(x+1)-1$, this equation becomes $-g(a-1) g(b-1)=$ $g(-(a-1)(b-1))$ for $a<10$, and therefore +(1) reduces to $g(1) g(y z)=g(y) g(z)$ for all $y, z>0$. We have two cases: +(i) $g(1)=0$. By (1) we have $g(z)=0$ for all $z>0$. Then any nondecreasing function $g: \mathbb{R} \rightarrow \mathbb{R}$ with $g(-1)=-1$ and $g(z)=0$ for $z \geq 0$ satisfies (1) and gives a solution: $f$ is nondecreasing, $f(0)=0$ and $f(x)=1$ for every $x \geq 1$ +(ii) $g(1) \neq 0$. Then the function $h(x)=\frac{g(x)}{g(1)}$ is nondecreasing and satisfies $h(0)=0, h(1)=1$, and $h(x y)=h(x) h(y)$. Fix $a>0$, and let $h(a)=$ $b=a^{k}$ for some $k \in \mathbb{R}$. It follows by induction that $h\left(a^{q}\right)=h(a)^{q}=$ +$\left(a^{q}\right)^{k}$ for every rational number $q$. But $h$ is nondecreasing, so $k \geq 0$, and since the set $\left\{a^{q} \mid q \in \mathbb{Q}\right\}$ is dense in $\mathbb{R}^{+}$, we conclude that $h(x)=x^{k}$ for every $x>0$. Finally, putting $g(1)=c$, we obtain $g(x)=c x^{k}$ for all $x>0$. Then $g(-x)=-x^{k}$ for all $x>0$. This $g$ obviously satisfies (1). Hence + +$$ +f(x)=\left\{\begin{array}{ll} +c(x-1)^{k}, & \text { if } x>1 ; \\ +1, & \text { if } x=1 ; \\ +1-(1-x)^{k}, & \text { if } x<1 +\end{array} \quad \text { where } c>0 \text { and } k \geq 0\right. +$$ + +3. (a) Given any sequence $c_{n}$ (in particular, such that $C_{n}$ converges), we shall construct $a_{n}$ and $b_{n}$ such that $A_{n}$ and $B_{n}$ diverge. +First, choose $n_{1}$ such that $n_{1} c_{1}>1$ and set $a_{1}=a_{2}=\cdots=a_{n_{1}}=$ $c_{1}$ : this uniquely determines $b_{2}=c_{2}, \ldots, b_{n_{1}}=c_{n_{1}}$. Next, choose $n_{2}$ such that $\left(n_{2}-n_{1}\right) c_{n_{1}+1}>1$ and set $b_{n_{1}+1}=\cdots=b_{n_{2}}=c_{n_{1}+1}$; again $a_{n_{1}+1}, \ldots, a_{n_{2}}$ is hereby determined. Then choose $n_{3}$ with $\left(n_{3}-\right.$ $\left.n_{2}\right) c_{n_{2}+1}>1$ and set $a_{n_{2}+1}=\cdots=a_{n_{3}}=c_{n_{2}+1}$, and so on. It is plain that in this way we construct decreasing sequences $a_{n}, b_{n}$ such that $\sum a_{n}$ and $\sum b_{n}$ diverge, since they contain an infinity of subsums that exceed 1 ; on the other hand, $c_{n}=\min \left(a_{n}, b_{n}\right)$ and $C_{n}$ is convergent. +(b) The answer changes in this situation. Suppose to the contrary that there is such a pair of sequences $\left(a_{n}\right)$ and $\left(b_{n}\right)$. There are infinitely many indices $i$ such that $c_{i}=b_{i}$ (otherwise all but finitely many terms of the sequence $\left(c_{n}\right)$ would be equal to the terms of the sequence $\left(a_{n}\right)$, which has an unbounded sum). Thus for any $n_{0} \in \mathbb{N}$ there is $j \geq 2 n_{0}$ such that $c_{j}=b_{j}$. Then we have + +$$ +\sum_{k=n_{0}}^{j} c_{k} \geq \sum_{k=n_{0}}^{j} c_{j}=\left(j-n_{0}\right) \frac{1}{j} \geq \frac{1}{2} +$$ + +Hence the sequence ( $C_{n}$ ) is unbounded, a contradiction. +4. By the Cauchy-Schwarz inequality we have + +$$ +\left(\sum_{i, j=1}^{n}(i-j)^{2}\right)\left(\sum_{i, j=1}^{n}\left(x_{i}-x_{j}\right)^{2}\right) \geq\left(\sum_{i, j=1}^{n}|i-j| \cdot\left|x_{i}-x_{j}\right|\right)^{2} . +$$ + +On the other hand, it is easy to prove (for example by induction) that + +$$ +\sum_{i, j=1}^{n}(i-j)^{2}=(2 n-2) \cdot 1^{2}+(2 n-4) \cdot 2^{2}+\cdots+2 \cdot(n-1)^{2}=\frac{n^{2}\left(n^{2}-1\right)}{6} +$$ + +and that + +$$ +\sum_{i, j=1}^{n}|i-j| \cdot\left|x_{i}-x_{j}\right|=\frac{n}{2} \sum_{i, j=1}^{n}\left|x_{i}-x_{j}\right| +$$ + +Thus the inequality (1) becomes + +$$ +\frac{n^{2}\left(n^{2}-1\right)}{6}\left(\sum_{i, j=1}^{n}\left(x_{i}-x_{j}\right)^{2}\right) \geq \frac{n^{2}}{4}\left(\sum_{i, j=1}^{n}\left|x_{i}-x_{j}\right|\right)^{2} +$$ + +which is equivalent to the required one. +5. Placing $x=y=z=1$ in (i) leads to $4 f(1)=f(1)^{3}$, so by the condition $f(1)>0$ we get $f(1)=2$. Also putting $x=t s, y=\frac{t}{s}, z=\frac{s}{t}$ in (i) gives + +$$ +f(t) f(s)=f(t s)+f(t / s) +$$ + +In particular, for $s=1$ the last equality yields $f(t)=f(1 / t)$; hence $f(t) \geq f(1)=2$ for each $t$. It follows that there exists $g(t) \geq 1$ such that $\bar{f}(t)=g(t)+\frac{1}{g(t)}$. Now it follows by induction from (1) that $g\left(t^{n}\right)=$ $g(t)^{n}$ for every integer $n$, and therefore $g\left(t^{q}\right)=g(t)^{q}$ for every rational $q$. Consequently, if $t>1$ is fixed, we have $f\left(t^{q}\right)=a^{q}+a^{-q}$, where $a=g(t)$. But since the set of $a^{q}(q \in \mathbb{Q})$ is dense in $\mathbb{R}^{+}$and $f$ is monotone on $(0,1]$ and $[1, \infty)$, it follows that $f\left(t^{r}\right)=a^{r}+a^{-r}$ for every real $r$. Therefore, if $k$ is such that $t^{k}=a$, we have + +$$ +f(x)=x^{k}+x^{-k} \quad \text { for every } x \in \mathbb{R} +$$ + +6. Set $X=\max \left\{x_{1}, \ldots, x_{n}\right\}$ and $Y=\max \left\{y_{1}, \ldots, y_{n}\right\}$. By replacing $x_{i}$ by $x_{i}^{\prime}=\frac{x_{i}}{X}, y_{i}$ by $y_{i}^{\prime}=\frac{y_{i}}{Y}$ and $z_{i}$ by $z_{i}^{\prime}=\frac{z_{i}}{\sqrt{X Y}}$, we may assume that $X=Y=1$. It is sufficient to prove that + +$$ +M+z_{2}+\cdots+z_{2 n} \geq x_{1}+\cdots+x_{n}+y_{1}+\cdots+y_{n} +$$ + +because this implies the result by the A-G mean inequality. +To prove (1) it is enough to prove that for any $r$, the number of terms greater than $r$ on the left-hand side of (1) is at least that number on the right-hand side of (1). +If $r \geq 1$, then there are no terms on the right-hand side greater than $r$. Suppose that $r<1$ and consider the sets $A=\left\{i \mid 1 \leq i \leq n, x_{i}>r\right\}$ and $B=\left\{i \mid 1 \leq i \leq n, y_{i}>r\right\}$. Set $a=|A|$ and $b=|B|$. If $x_{i}>r$ and $y_{j}>r$, then $z_{i+j} \geq \sqrt{x_{i} y_{j}}>r$; hence + +$$ +C=\left\{k \mid 2 \leq k \leq 2 n, z_{k}>r\right\} \supseteq A+B=\{\alpha+\beta \mid \alpha \in A, \beta \in B\} +$$ + +It is easy to verify that $|A+B| \geq|A|+|B|-1$. It follows that the number of $z_{k}$ 's greater than $r$ is $\geq a+b-1$. But in that case $M>r$, implying that at least $a+b$ elements of the left-hand side of (1) is greater than $r$, which completes the proof. +7. Consider the set $D=\{x-y \mid x, y \in A\}$. Obviously, the number of elements of the set $D$ is less than or equal to $101 \cdot 100+1$. The sets $A+t_{i}$ and $A+t_{j}$ +are disjoint if and only if $t_{i}-t_{j} \notin D$. Now we shall choose inductively 100 elements $t_{1}, \ldots, t_{100}$. +Let $t_{1}$ be any element of the set $S \backslash D$ (such an element exists, since the number of elements of $S$ is greater than the number of elements of $D$ ). Suppose now that we have chosen $k(k \leq 99)$ elements $t_{1}, \ldots, t_{k}$ from $D$ such that the difference of any two of the chosen elements does not belong to $D$. We can select $t_{k+1}$ to be an element of $S$ that does not belong to any of the sets $t_{1}+D, t_{2}+D, \ldots, t_{k}+D$ (this is possible to do, since each of the previous sets has at most $101 \cdot 100+1$ elements; hence their union has at most $99(101 \cdot 100+1)=999999<1000000$ elements $)$. +8. Let $S$ be the disk with the smallest radius, say $s$, and $O$ the center of that disk. Divide the plane into 7 regions: one bounded by disk $s$ and 6 regions $T_{1}, \ldots, T_{6}$ shown in the figure. +Any of the disks different from $S$, say $D_{k}$, has its center in one of the seven regions. If its center is inside $S$ then $D_{k}$ contains point $O$. Hence the number of disks different from $S$ having their centers in $S$ is at most 2002. + +Consider a disk $D_{k}$ that intersects $S$ and whose center is in the region $T_{i}$. Let $P_{i}$ be the point such that $O P_{i}$ bisects the region $T_{i}$ and +![](https://cdn.mathpix.com/cropped/2024_11_18_8e985d6b9c83aa3e9d0eg-713.jpg?height=453&width=503&top_left_y=799&top_left_x=828) +$O P_{i}=s \sqrt{3}$. +We claim that $D_{k}$ contains $P_{i}$. Divide the region $T_{i}$ by a line $l_{i}$ through $P_{i}$ perpendicular to $O P_{i}$ into two regions $U_{i}$ and $V_{i}$, where $O$ and $U_{i}$ are on the same side of $l_{i}$. Let $K$ be the center of $D_{k}$. Consider two cases: +(i) $K \in U_{i}$. Since the disk with the center $P_{i}$ and radius $s$ contains $U_{i}$, we see that $K P_{i} \leq s$. Hence $D_{k}$ contains $P_{i}$. +(ii) $K \in V_{i}$. Denote by $L$ the intersection point of the segment $K O$ with the circle $s$. +We want to prove that $K L>K P_{i}$. It is enough to prove that $\angle K P_{i} L>\angle K L P_{i}$. However, it is obvious that $\angle L P_{i} O \leq 30^{\circ}$ and $\angle L O P_{i} \leq 30^{\circ}$, hence $\angle K L P_{i} \leq 60^{\circ}$, while $\angle N P_{i} L=90^{\circ}-\angle L P_{i} O \geq$ $60^{\circ}$. This implies that $\angle K P_{i} L \geq \angle N P_{i} L \geq 60^{\circ} \geq \angle K L P_{i}$ ( $N$ is the point on the edge of $T_{i}$ as shown in the figure). Our claim is thus proved. +Now we see that the number of disks with centers in $T_{i}$ that intersect $S$ is less than or equal to 2003, and the total number of disks that intersect $S$ is not greater than $2002+6 \cdot 2003=7 \cdot 2003-1$. +9. Suppose that $k$ of the angles of an $n$-gon are right. Since the other $n-k$ angles are less than $360^{\circ}$ and the sum of the angles is $(n-2) 180^{\circ}$, we have +the inequality $k \cdot 90^{\circ}+(n-k) 360^{\circ}>(n-2) 180^{\circ}$, which is equivalent to $k<\frac{2 n+4}{3}$. Since $n$ and $k$ are integers, it follows that $k \leq\left[\frac{2 n}{3}\right]+1$. +If $n=5$, then $\left[\frac{2 n}{3}\right]+1=4$, but if a pentagon has four right angles, the other angle is equal to $180^{\circ}$, which is impossible. Hence for $n=5$, $k \leq 3$. It is easy to construct a pentagon with 3 right angles, e.g., as in the picture below. +Now we shall show by induction that for $n \geq 6$ there is an $n$-gon with $\left[\frac{2 n}{3}\right]+1$ internal right angles. For $n=6,7,8$ examples are presented in the picture. Assume that there is a $(n-3)$ gon with $\left[\frac{2(n-3)}{3}\right]+1=\left[\frac{2 n}{3}\right]-1$ internal right angles. Then one of the internal angles, say $\angle B A C$, is not convex. Interchange the vertex $A$ with four new vertices $A_{1}, A_{2}, A_{3}, A_{4}$ as shown in the picture such that $\angle B A_{1} A_{2}=\angle A_{3} A_{4} C=90^{\circ}$. +![](https://cdn.mathpix.com/cropped/2024_11_18_8e985d6b9c83aa3e9d0eg-714.jpg?height=195&width=1024&top_left_y=781&top_left_x=292) +10. Denote by $b_{i j}$ the entries of the matrix $B$. Suppose the contrary, i.e., that there is a pair $\left(i_{0}, j_{0}\right)$ such that $a_{i_{0}, j_{0}} \neq b_{i_{0}, j_{0}}$. We may assume without loss of generality that $a_{i_{0}, j_{0}}=0$ and $b_{i_{0}, j_{0}}=1$. +Since the sums of elements in the $i_{0}$ th rows of the matrices $A$ and $B$ are equal, there is some $j_{1}$ for which $a_{i_{0}, j_{1}}=1$ and $b_{i_{0}, j_{1}}=0$. Similarly, from the fact that the sums in the $j_{1}$ th columns of the matrices $A$ and $B$ are equal, we conclude that there exists $i_{1}$ such that $a_{i_{1}, j_{1}}=0$ and $b_{i_{1}, j_{1}}=1$. Continuing this procedure, we construct two sequences $i_{k}, j_{k}$ such that $a_{i_{k}, j_{k}}=0, b_{i_{k}, j_{k}}=1, a_{i_{k}, j_{k+1}}=1, b_{i_{k}, j_{k+1}}=0$. Since the set of the pairs $\left(i_{k}, j_{k}\right)$ is finite, there are two different numbers $t, s$ such that $\left(i_{t}, j_{t}\right)=\left(i_{s}, j_{s}\right)$. From the given condition we have that $x_{i_{k}}+y_{i_{k}}<0$ and $x_{i_{k+1}}+y_{i_{k+1}} \geq 0$. But $j_{t}=j_{s}$, and hence $0 \leq \sum_{k=s}^{t-1}\left(x_{i_{k}}+y_{j_{k+1}}\right)=$ $\sum_{k=s}^{t-1}\left(x_{i_{k}}+y_{j_{k}}\right)<0$, a contradiction. +11. (a) By the pigeonhole principle there are two different integers $x_{1}, x_{2}$, $x_{1}>x_{2}$, such that $\left|\left\{x_{1} \sqrt{3}\right\}-\left\{x_{2} \sqrt{3}\right\}\right|<0.001$. Set $a=x_{1}-x_{2}$. Consider the equilateral triangle with vertices $(0,0),(2 a, 0),(a, a \sqrt{3})$. The points $(0,0)$ and $(2 a, 0)$ are lattice points, and we claim that the point $(a, a \sqrt{3})$ is at distance less than 0.001 from a lattice point. Indeed, since $0.001>\left|\left\{x_{1} \sqrt{3}\right\}-\left\{x_{2} \sqrt{3}\right\}\right|=\left|a \sqrt{3}-\left(\left[x_{1} \sqrt{3}\right]-\left[x_{2} \sqrt{3}\right]\right)\right|$, we see that the distance between the points $(a, a \sqrt{3})$ and $\left(a,\left[x_{1} \sqrt{3}\right]-\right.$ $\left.\left[x_{2} \sqrt{3}\right]\right)$ is less than 0.001 , and the point $\left(a,\left[x_{1} \sqrt{3}\right]-\left[x_{2} \sqrt{3}\right]\right)$ is with integer coefficients. +(b) Suppose that $P^{\prime} Q^{\prime} R^{\prime}$ is an equilateral triangle with side length $l \leq 96$ such that each of its vertices $P^{\prime}, Q^{\prime}, R^{\prime}$ lies in a disk of radius 0.001 centered at a lattice point. Denote by $P, Q, R$ the centers of these disks. Then we have $l-0.002 \leq P Q, Q R, R P \leq l+0.002$. Since $P Q R$ is not an equilateral triangle, two of its sides are different, say +$P Q \neq Q R$. On the other hand, $P Q^{2}, Q R^{2}$ are integers, so we have $1 \leq\left|P Q^{2}-Q R^{2}\right|=(P Q+Q R)|P Q-Q R| \leq 0.004(P Q+Q R) \leq$ $(2 l+0.004) \cdot 0.004 \leq 2 \cdot 96.002 \cdot 0.004<1$, which is a contradiction. +12. Denote by $\overline{a_{k-1} a_{k-2} \ldots a_{0}}$ the decimal representation of a number whose digits are $a_{k-1}, \ldots, a_{0}$. We will use the following well-known fact: + +$$ +\overline{a_{k-1} a_{k-2} \ldots a_{0}} \equiv i(\bmod 11) \Longleftrightarrow \sum_{l=0}^{k-1}(-1)^{l} a_{l} \equiv i(\bmod 11) . +$$ + +Let $m$ be a positive integer. Define $A$ as the set of integers $n(0 \leq n<$ $10^{2 m}$ ) whose right $2 m-1$ digits can be so permuted to yield an integer divisible by 11 , and $B$ as the set of integers $n\left(0 \leq n<10^{2 m-1}\right)$ whose digits can be permuted resulting in an integer divisible by 11. Suppose that $a=\overline{a_{2 m-1} \ldots a_{0}} \in A$. Then there that satisfies + +$$ +\sum_{l=0}^{2 m-1}(-1)^{l} b_{l} \equiv 0(\bmod 11) +$$ + +The $2 m$-tuple $\left(b_{2 m-1}, \ldots, b_{0}\right)$ satisfies (1) if and only if the $2 m$-tuple $\left(k b_{2 m-1}+l, \ldots, k b_{0}+l\right)$ satisfies ( 1 ), where $k, l \in \mathbb{Z}, 11 \nmid k$. +Since $a_{0}+1 \not \equiv 0(\bmod 11)$, we can choose $k$ from the set $\{1, \ldots, 10\}$ such that $\left(a_{0}+1\right) k \equiv 1(\bmod 11)$. Thus there is a permutation of the $2 m$-tuple $\left(\left(a_{2 m-1}+1\right) k-1, \ldots,\left(a_{1}+1\right) k-1,0\right)$ satisfying (1). Interchanging odd and even positions if necessary, we may assume that this permutation keeps the 0 at the last position. Since $\left(a_{i}+1\right) k$ is not divisible by 11 for any $i$, there is a unique $b_{i} \in\{0,1, \ldots, 9\}$ such that $b_{i} \equiv\left(a_{i}+1\right) k-1(\bmod 11)$. Hence the number $\overline{b_{2 m-1} \ldots b_{1}}$ belongs to $B$. +Thus for fixed $a_{0} \in\{0,1,2, \ldots, 9\}$, to each $a \in A$ such that the last digit of $a$ is $a_{0}$ we associate a unique $b \in B$. Conversely, having $a_{0} \in$ $\{0,1,2, \ldots, 9\}$ fixed, from any number $\overline{b_{2 m-1} \ldots b_{1}} \in B$ we can reconstruct $\overline{a_{2 m-1} \ldots a_{1} a_{0}} \in A$. Hence $|A|=10|B|$, i.e., $f(2 m)=10 f(2 m-1)$. +13. Denote by $K$ and $L$ the intersections of the bisectors of $\angle A B C$ and $\angle A D C$ with the line $A C$, respectively. Since $A B: B C=A K: K C$ and $A D: D C=A L: L C$, we have to prove that + +$$ +P Q=Q R \Leftrightarrow \frac{A B}{B C}=\frac{A D}{D C} . +$$ + +Since the quadrilaterals $A Q D R$ and $Q P C D$ are cyclic, we see that +![](https://cdn.mathpix.com/cropped/2024_11_18_8e985d6b9c83aa3e9d0eg-715.jpg?height=403&width=399&top_left_y=1567&top_left_x=862) +$\angle R D Q=\angle B A C$ and $\angle Q D P=\angle A C B$. By the law of sines it follows that $\frac{A B}{B C}=\frac{\sin (\angle A C B)}{\sin (\angle B A C)}$ and that $Q R=A D \sin (\angle R D Q), Q P=$ $C D \sin (\angle Q D P)$. Now we have + +$$ +\frac{A B}{B C}=\frac{\sin (\angle A C B)}{\sin (\angle B A C)}=\frac{\sin (\angle Q D P)}{\sin (\angle R D Q)}=\frac{A D \cdot Q P}{Q R \cdot C D} +$$ + +The statement (1) follows directly. +14. Denote by $R$ the intersection point of the bisector of $\angle A Q C$ and the line $A C$. From $\triangle A C Q$ we get + +$$ +\frac{A R}{R C}=\frac{A Q}{Q C}=\frac{\sin \angle Q C A}{\sin \angle Q A C} +$$ + +By the sine version of Ceva's theorem we have $\frac{\sin \angle A P B}{\sin \angle B P C} \cdot \frac{\sin \angle Q A C}{\sin \angle P A Q}$. $\frac{\sin \angle Q C P}{\sin \angle Q C A}=1$, which is equivalent to + +$$ +\frac{\sin \angle A P B}{\sin \angle B P C}=\left(\frac{\sin \angle Q C A}{\sin \angle Q A C}\right)^{2} +$$ + +because $\angle Q C A=\angle P A Q$ and $\angle Q A C=\angle Q C P$. Denote by $S(X Y Z)$ the area of a triangle $X Y Z$. Then + +$$ +\frac{\sin \angle A P B}{\sin \angle B P C}=\frac{A P \cdot B P \cdot \sin \angle A P B}{B P \cdot C P \cdot \sin \angle B P C}=\frac{S(\Delta A B P)}{S(\Delta B C P)}=\frac{A B}{B C}, +$$ + +which implies that $\left(\frac{A R}{R C}\right)^{2}=\frac{A B}{B C}$. Hence $R$ does not depend on $\Gamma$. +15. From the given equality we see that $0=\left(B P^{2}+P E^{2}\right)-\left(C P^{2}+P F^{2}\right)=$ $B F^{2}-C E^{2}$, so $B F=C E=x$ for some $x$. Similarly, there are $y$ and $z$ such that $C D=A F=y$ and $B D=A E=z$. It is easy to verify that $D$, $E$, and $F$ must lie on the segments $B C, C A, A B$. +Denote by $a, b, c$ the length of the segments $B C, C A, A B$. It follows that $a=z+y, b=z+x, c=x+y$, so $D, E, F$ are the points where the excircles touch the sides of $\triangle A B C$. Hence $P, D$, and $I_{A}$ are collinear and + +$$ +\angle P I_{A} C=\angle D I_{A} C=90^{\circ}-\frac{180^{\circ}-\angle A C B}{2}=\frac{\angle A C B}{2} +$$ + +In the same way we obtain that $\angle P I_{B} C=\frac{\angle A C B}{2}$ and $P I_{B}=P I_{A}$. Analogously, we get $P I_{C}=P I_{B}$, which implies that $P$ is the circumcenter of the triangle $I_{A} I_{B} I_{C}$. +16. Apply an inversion with center at $P$ and radius $r$; let $\widehat{X}$ denote the image of $X$. The circles $\Gamma_{1}, \Gamma_{2}, \Gamma_{3}, \Gamma_{4}$ are transformed into lines $\widehat{\Gamma_{1}}, \widehat{\Gamma_{2}}, \widehat{\Gamma_{3}}, \widehat{\Gamma}_{4}$, where $\widehat{\Gamma_{1}} \| \widehat{\Gamma_{3}}$ and $\widehat{\Gamma_{2}} \| \widehat{\Gamma_{4}}$, and therefore $\widehat{A} \widehat{B} \widehat{C} \widehat{D}$ is a parallelogram. Further, we have $A B=\frac{r^{2}}{P \widehat{A} \cdot P \widehat{B}} \widehat{A} \widehat{B}, B C=\frac{r^{2}}{P \widehat{B} \cdot P \widehat{C}} \widehat{B} \widehat{C}, C D=\frac{r^{2}}{P \widehat{C} \cdot P \widehat{D}} \widehat{C} \widehat{D}$, $D A=\frac{r^{2}}{P \widehat{D} \cdot P \widehat{A}} \widehat{D} \widehat{A}$ and $P B=\frac{r^{2}}{P \widehat{B}}, P D=\frac{r^{2}}{P \widehat{D}}$. The equality to be proven +becomes + +$$ +\frac{P \widehat{D}^{2}}{P \widehat{B}^{2}} \cdot \frac{\widehat{A} \widehat{B} \cdot \widehat{B} \widehat{C}}{\widehat{A} \cdot \widehat{D} \widehat{C}}=\frac{P \widehat{D}^{2}}{P \widehat{B}^{2}} +$$ + +which holds because $\widehat{A} \widehat{B}=\widehat{C} \widehat{D}$ and $\widehat{B} \widehat{C}=\widehat{D} \widehat{A}$. +17. The triangles $P D E$ and $C F G$ are homothetic; hence lines $F D, G E$, and $C P$ intersect at one point. Let $Q$ be the intersection point of the line $C P$ and the circumcircle of $\triangle A B C$. The required statement will follow if we show that $Q$ lies on the lines $G E$ and $F D$. +Since $\angle C F G=\angle C B A=\angle C Q A$, the quadrilateral $A Q P F$ is cyclic. Analogously, $B Q P G$ is cyclic. However, the isosceles trapezoid $B D P G$ is also cyclic; it follows that $B, Q, D, P, G$ lie on a circle. Therefore we get + +$$ +\angle P Q F=\angle P A C, \angle P Q D=\angle P B A . +$$ + +Since $I$ is the incenter of $\triangle A B C$, we have $\angle C A I=\frac{1}{2} \angle C A B=$ $\frac{1}{2} \angle C B A=\angle I B A$; hence $C A$ is the tangent at $A$ to the circumcircle of $\triangle A B I$. This implies that $\angle P A C=$ $\angle P B A$, and it follows from (1) that $\angle P Q F=\angle P Q D$, i.e., that $F, D, Q$ are also collinear. Similarly, $G, E, Q$ are collinear and the claim is thus proved. +![](https://cdn.mathpix.com/cropped/2024_11_18_8e985d6b9c83aa3e9d0eg-717.jpg?height=412&width=358&top_left_y=632&top_left_x=903) +18. Let $A B C D E F$ be the given hexagon. We shall use the following lemma. Lemma. If $\angle X Z Y \geq 60^{\circ}$ and if $M$ is the midpoint of $X Y$, then $M Z \leq$ $\frac{\sqrt{3}}{2} X Y$, with equality if and only if $\triangle X Y Z$ is equilateral. +Proof. Let $Z^{\prime}$ be the point such that $\triangle X Y Z^{\prime}$ is equilateral. Then $Z$ is inside the circle circumscribed about $\triangle X Y Z^{\prime}$. Consequently $M Z \leq$ $M Z^{\prime}=\frac{\sqrt{3}}{2} X Y$, with equality if and only if $Z=Z^{\prime}$. Set $A D \cap B E=P, B E \cap C F=Q$, and $C F \cap A D=R$. Suppose $\angle A P B=$ $\angle D P E>60^{\circ}$, and let $K, L$ be the midpoints of the segments $A B$ and $D E$ respectively. Then by the lemma, + +$$ +\frac{\sqrt{3}}{2}(A B+D E)=K L \leq P K+P L<\frac{\sqrt{3}}{2}(A B+D E), +$$ + +which is impossible. Therefore $\angle A P B \leq 60^{\circ}$ and similarly $\angle B Q C \leq 60^{\circ}$, $\angle C R D \leq 60^{\circ}$. But the sum of the angles $A P B, B Q C, C R D$ is $180^{\circ}$, from which we conclude that these angles are all equal to $60^{\circ}$, and moreover that the triangles $A P B, B Q C, C R D$ are equilateral. Thus $\angle A B C=\angle A B P+$ $\angle Q B C=120^{\circ}$, and in the same way all angles of the hexagon are equal to $120^{\circ}$. +19. Let $D, E, F$ be the midpoints of $B C, C A, A B$, respectively. We construct smaller semicircles $\Gamma_{d}, \Gamma_{e}, \Gamma_{f}$ inside $\triangle A B C$ with centers $D, E, F$ and radii $d=\frac{s-a}{2}, e=\frac{s-b}{2}, f=\frac{s-c}{2}$ respectively. Since $D E=d+e, D F=d+f$, and $E F=e+f$, we deduce that $\Gamma_{d}, \Gamma_{e}$, and $\Gamma_{f}$ touch each other at the points $D_{1}, E_{1}, F_{1}$ of tangency of the incircle $\gamma$ of $\triangle D E F$ with its sides ( $D_{1} \in E F$, etc.). Consider the circle $\Gamma_{g}$ with center $O$ and radius $g$ that lies inside $\triangle D E F$ and tangents $\Gamma_{d}, \Gamma_{e}, \Gamma_{f}$. + +Now let $O D, O E, O F$ meet the semicircles $\Gamma_{d}, \Gamma_{e}, \Gamma_{f}$ at $D^{\prime}, E^{\prime}, F^{\prime}$ respectively. We have $O D^{\prime}=O D+$ $D D^{\prime}=g+d+\frac{a}{2}=g+\frac{s}{2}$ and similarly $O E^{\prime}=O F^{\prime}=g+\frac{s}{2}$. It follows that the circle with center $O$ and radius $g+\frac{s}{2}$ touches all three semicircles, and consequently $t=$ $g+\frac{s}{2}>\frac{s}{2}$. Now set the coordinate system such that we have the points $D_{1}(0,0), E(-e, 0), F(f, 0)$ and such that the $y$ coordinate of $D$ is positive. +![](https://cdn.mathpix.com/cropped/2024_11_18_8e985d6b9c83aa3e9d0eg-718.jpg?height=528&width=529&top_left_y=233&top_left_x=831) + +Apply the inversion with center $D_{1}$ and unit radius. This inversion maps the circles $\Gamma_{e}$ and $\Gamma_{f}$ to the lines $\widehat{\Gamma_{e}}\left[x=-\frac{1}{2 e}\right]$ and $\widehat{\Gamma_{e}}\left[x=\frac{1}{2 f}\right]$ respectively, and the circle $\gamma$ goes to the line $\widehat{\gamma}\left[y=\frac{1}{r}\right]$. The images $\widehat{\Gamma_{d}}$ and $\widehat{\Gamma_{g}}$ of $\Gamma_{d}, \Gamma_{g}$ are the circles that touch the lines $\widehat{\Gamma_{e}}$ and $\widehat{\Gamma_{f}}$. Since $\widehat{\Gamma_{d}}, \widehat{\Gamma_{g}}$ are perpendicular to $\gamma$, they have radii equal to $R=\frac{1}{4 e}+\frac{1}{4 f}$ and centers at $\left(-\frac{1}{4 e}+\frac{1}{4 f}, \frac{1}{r}\right)$ and $\left(-\frac{1}{4 e}+\frac{1}{4 f}, \frac{1}{r}+2 R\right)$ respectively. Let $p$ and $P$ be the distances from $D_{1}(0,0)$ to the centers of $\Gamma_{g}$ and $\widehat{\Gamma_{g}}$ respectively. We have that $P^{2}=\left(\frac{1}{4 e}-\frac{1}{4 f}\right)^{2}+\left(\frac{1}{r}+2 R\right)^{2}$, and that the circles $\Gamma_{g}$ and $\widehat{\Gamma_{g}}$ are homothetic with center of homothety $D_{1}$; hence $p / P=g / R$. On the other hand, $\widehat{\Gamma_{g}}$ is the image of $\Gamma_{g}$ under inversion; hence the product of the tangents from $D_{1}$ to these two circles is equal to 1 . In other words, we obtain $\sqrt{p^{2}-g^{2}} \cdot \sqrt{P^{2}-R^{2}}=1$. Using the relation $p / P=g / R$ we get $g=\frac{R}{P^{2}-R^{2}}$. +The inequality we have to prove is equivalent to $(4+2 \sqrt{3}) g \leq r$. This can be proved as follows: + +$$ +\begin{aligned} +r-(4+2 \sqrt{3}) g & =\frac{r\left(P^{2}-R^{2}-(4+2 \sqrt{3}) R / r\right)}{P^{2}-R^{2}} \\ +& =\frac{r\left(\left(\frac{1}{r}+2 R\right)^{2}+\left(\frac{1}{4 e}-\frac{1}{4 f}\right)^{2}-R^{2}-(4+2 \sqrt{3}) \frac{R}{r}\right)}{P^{2}-R^{2}} \\ +& =\frac{r}{P^{2}-R^{2}}\left(\left(R \sqrt{3}-\frac{1}{r}\right)^{2}+\left(\frac{1}{4 e}-\frac{1}{4 f}\right)^{2}\right) \geq 0 +\end{aligned} +$$ + +Remark. One can obtain a symmetric formula for $g$ : + +$$ +\frac{1}{2 g}=\frac{1}{s-a}+\frac{1}{s-b}+\frac{1}{s-c}+\frac{2}{r} +$$ + +20. Let $r_{i}$ be the remainder when $x_{i}$ is divided by $m$. Since there are at most $m^{m}$ types of $m$-consecutive blocks in the sequence $\left(r_{i}\right)$, some type will +repeat at least twice. Then since the entire sequence is determined by one $m$-consecutive block, the entire sequence will be periodic. +The formula works both forward and backward; hence using the rule $x_{i}=$ $x_{i+m}-\sum_{j=1}^{m-1} x_{i+j}$ we can define $x_{-1}, x_{-2}, \ldots$. Thus we obtain that + +$$ +\left(r_{-m}, \ldots, r_{-1}\right)=(0,0, \ldots, 0,1) +$$ + +Hence there are $m-1$ consecutive terms in the sequence $\left(x_{i}\right)$ that are divisible by $m$. +If there were $m$ consecutive terms in the sequence $\left(x_{i}\right)$ divisible by $m$, then by the recurrence relation all the terms of $\left(x_{i}\right)$ would be divisible by $m$, which is impossible. +21. Let $a$ be a positive integer for which $d(a)=a^{2}$. Suppose that $a$ has $n+1$ digits, $n \geq 0$. Denote by $s$ the last digit of $a$ and by $f$ the first digit of $c$. Then $a=\overline{* \ldots * s}$, where $*$ stands for a digit that is not important to us at the moment. We have $\overline{\ldots * s^{2}}=a^{2}=d=\overline{* \ldots * f}$ and $b^{2}=\overline{s * \ldots *}^{2}=$ $c=\overline{f * \ldots *}$. +We cannot have $s=0$, since otherwise $c$ would have at most $2 n$ digits, while $a^{2}$ has either $2 n+1$ or $2 n+2$ digits. The following table gives all possibilities for $s$ and $f$ : + +| $s$ | 1 | 2 | 3 | 4 | 5 | 6 | 7 | 8 | 9 | +| :---: | :---: | :---: | :---: | :---: | :---: | :---: | :---: | :---: | :---: | +| $f=$ last digit of $\overline{* \ldots * s}^{2}$ | 1 | 4 | 9 | 6 | 5 | 6 | 9 | 4 | 1 | +| $f=$ first digit of $\overline{s * \ldots *}^{2}$ | $1,2,3$ | $4-8$ | 9,1 | 1,2 | 2,3 | 3,4 | $4,5,6$ | $6,7,8$ | 8,9 | + +We obtain from the table that $s \in\{1,2,3\}$ and $f=s^{2}$, and consequently $c=b^{2}$ and $d$ have exactly $2 n+1$ digits each. Put $a=10 x+s$, where $x<10^{n}$. Then $b=10^{n} s+x, c=10^{2 n} s^{2}+2 \cdot 10^{n} s x+x^{2}$, and $d=$ $2 \cdot 10^{n+1} s x+10 x^{2}+s^{2}$, so from $d=a^{2}$ it follows that $x=2 s \cdot \frac{10^{n}-1}{9}$. Thus $a=\underbrace{6 \ldots 6}_{n} 3, a=\underbrace{4 \ldots 4}_{n} 2$ or $a=\underbrace{2 \ldots 2}_{n} 1$. For $n \geq 1$ we see that $a$ cannot be $a=6 \ldots 63$ or $a=4 \ldots 42$ (otherwise $a^{2}$ would have $2 n+2$ digits). Therefore $a$ equals $1,2,3$ or $\underbrace{2 \ldots 2}_{n} 1$ for $n \geq 0$. It is easy to verify that these numbers have the required property. +22. Let $a$ and $b$ be positive integers for which $\frac{a^{2}}{2 a b^{2}-b^{3}+1}=k$ is a positive integer. Since $k>0$, it follows that $2 a b^{2} \geq b^{3}$, so $2 a \geq b$. If $2 a>b$, then from $2 a b^{2}-b^{3}+1>0$ we see that $a^{2}>b^{2}(2 a-b)+1>b^{2}$, i.e. $a>b$. Therefore, if $a \leq b$, then $a=b / 2$. +We can rewrite the given equation as a quadratic equation in $a, a^{2}-$ $2 k b^{2} a+k\left(b^{3}-1\right)=0$, which has two solutions, say $a_{1}$ and $a_{2}$, one of which is in $\mathbb{N}_{0}$. From $a_{1}+a_{2}=2 k b^{2}$ and $a_{1} a_{2}=k\left(b^{3}-1\right)$ it follows that the other solution is also in $\mathbb{N}_{0}$. Suppose w.l.o.g. that $a_{1} \geq a_{2}$. Then $a_{1} \geq k b^{2}$ and + +$$ +0 \leq a_{2}=\frac{k\left(b^{3}-1\right)}{a_{1}} \leq \frac{k\left(b^{3}-1\right)}{k b^{2}}M$. Also, straightforward calculation implies + +$$ +\left(b^{2 n+1}+\frac{b^{n+2}+b^{n+1}}{2}-b^{3}\right)^{2}M$ there is an integer $a_{n}$ such that $\left|a_{n}\right|3$ we get $a_{n}= \pm(3 b-5)$. +If $a_{n}=3 b-5$, then substituting in (1) yields $\frac{1}{4} b^{2 n}\left(b^{4}-14 b^{3}+45 b^{2}-\right.$ $52 b+20)=0$, with the unique positive integer solution $b=10$. Also, if $a_{n}=-3 b+5$, we similarly obtain $\frac{1}{4} b^{2 n}\left(b^{4}-14 b^{3}-3 b^{2}+28 b+20\right)-$ $2 b^{n+1}\left(3 b^{2}-2 b-5\right)=0$ for each $n$, which is impossible. +For $b=10$ it is easy to show that $x_{n}=\left(\frac{10^{n}+5}{3}\right)^{2}$ for all $n$. This proves the statement. +Second solution. In problems of this type, computing $z_{n}=\sqrt{x_{n}}$ asymptotically usually works. +From $\lim _{n \rightarrow \infty} \frac{b^{2 n}}{(b-1) x_{n}}=1$ we infer that $\lim _{n \rightarrow \infty} \frac{b^{n}}{z_{n}}=\sqrt{b-1}$. Furthermore, from $\left(b z_{n}+z_{n+1}\right)\left(b z_{n}-z_{n+1}\right)=b^{2} x_{n}-x_{n+1}=b^{n+2}+3 b^{2}-2 b-5$ we obtain + +$$ +\lim _{n \rightarrow \infty}\left(b z_{n}-z_{n+1}\right)=\frac{b \sqrt{b-1}}{2} +$$ + +Since the $z_{n}$ 's are integers for all $n \geq M$, we conclude that $b z_{n}-z_{n+1}=$ $\frac{b \sqrt{b-1}}{2}$ for all $n$ sufficiently large. Hence $b-1$ is a perfect square, and moreover $b$ divides $2 z_{n+1}$ for all large $n$. It follows that $b \mid 10$; hence the only possibility is $b=10$. +24. Suppose that $m=u+v+w$ where $u, v, w$ are good integers whose product is a perfect square of an odd integer. Since $u v w$ is an odd perfect square, we have that $u v w \equiv 1(\bmod 4)$. Thus either two or none of the numbers +$u, v, w$ are congruent to 3 modulo 4 . In both cases $u+v+w \equiv 3(\bmod 4)$. Hence $m \equiv 3(\bmod 4)$. +Now we shall prove the converse: every $m \equiv 3(\bmod 4)$ has infinitely many representations of the desired type. Let $m=4 k+3$. We shall represent $m$ in the form + +$$ +4 k+3=x y+y z+z x, \quad \text { for } x, y, z \text { odd. } +$$ + +The product of the summands is an odd square. Set $x=1+2 l$ and $y=1-2 l$. In order to satisfy (1), $z$ must satisfy $z=2 l^{2}+2 k+1$. The summands $x y, y z, z x$ are distinct except for finitely many $l$, so it remains only to prove that for infinitely many integers $l,|x y|,|y z|$, and $|z x|$ are not perfect squares. First, observe that $|x y|=4 l^{2}-1$ is not a perfect square for any $l \neq 0$. +Let $p, q>m$ be fixed different prime numbers. The system of congruences $1+2 l \equiv p\left(\bmod p^{2}\right)$ and $1-2 l \equiv q\left(\bmod q^{2}\right)$ has infinitely many solutions $l$ by the Chinese remainder theorem. For any such $l$, the number $z=$ $2 l^{2}+2 k+1$ is divisible by neither $p$ nor $q$, and hence $|x z|$ (respectively $|y z|)$ is divisible by $p$, but not by $p^{2}$ (respectively by $q$, but not by $q^{2}$ ). Thus $x z$ and $y z$ are also good numbers. +25. Suppose that for every prime $q$, there exists an $n$ for which $n^{p} \equiv p(\bmod$ $q$ ). Assume that $q=k p+1$. By Fermat's theorem we deduce that $p^{k} \equiv$ $n^{k p}=n^{q-1} \equiv 1(\bmod q)$, so $q \mid p^{k}-1$. +It is known that any prime $q$ such that $q \left\lvert\, \frac{p^{p}-1}{p-1}\right.$ must satisfy $q \equiv 1(\bmod$ $p$ ). Indeed, from $q \mid p^{q-1}-1$ it follows that $q \mid p^{\operatorname{gcd}(p, q-1)}-1$; but $q \nmid p-1$ because $\frac{p^{p}-1}{p-1} \equiv 1(\bmod p-1)$, so $\operatorname{gcd}(p, q-1) \neq 1$. Hence $\operatorname{gcd}(p, q-1)=p$. Now suppose $q$ is any prime divisor of $\frac{p^{p}-1}{p-1}$. Then $q \mid \operatorname{gcd}\left(p^{k}-1, p^{p}-1\right)=$ $p^{\operatorname{gcd}(p, k)}-1$, which implies that $\operatorname{gcd}(p, k)>1$, so $p \mid k$. Consequently $q \equiv 1$ $\left(\bmod p^{2}\right)$. However, the number $\frac{p^{p}-1}{p-1}=p^{p-1}+\cdots+p+1$ must have at least one prime divisor that is not congruent to 1 modulo $p^{2}$. Thus we arrived at a contradiction. +Remark. Taking $q \equiv 1(\bmod p)$ is natural, because for every other $q, n^{p}$ takes all possible residues modulo $q$ (including $p$ too). Indeed, if $p \nmid q-1$, then there is an $r \in \mathbb{N}$ satisfying $p r \equiv 1(\bmod q-1)$; hence for any $a$ the congruence $n^{p} \equiv a(\bmod q)$ has the solution $n \equiv a^{r}(\bmod q)$. +The statement of the problem itself is a special case of the Chebotarev's theorem. +26. Define the sequence $x_{k}$ of positive reals by $a_{k}=\cosh x_{k}$ ( $\cosh$ is the hyperbolic cosine defined by $\left.\cosh t=\frac{e^{t}+e^{-t}}{2}\right)$. Since $\cosh \left(2 x_{k}\right)=2 a_{k}^{2}-1=$ $\cosh x_{k+1}$, it follows that $x_{k+1}=2 x_{k}$ and thus $x_{k}=\lambda \cdot 2^{k}$ for some $\lambda>0$. From the condition $a_{0}=2$ we obtain $\lambda=\log (2+\sqrt{3})$. Therefore + +$$ +a_{n}=\frac{(2+\sqrt{3})^{2^{n}}+(2-\sqrt{3})^{2^{n}}}{2} +$$ + +Let $p$ be a prime number such that $p \mid a_{n}$. We distinguish the following two cases: +(i) There exists an $m \in \mathbb{Z}$ such that $m^{2} \equiv 3(\bmod p)$. Then we have + +$$ +(2+m)^{2^{n}}+(2-m)^{2^{n}} \equiv 0(\bmod p) +$$ + +Since $(2+m)(2-m)=4-m^{2} \equiv 1(\bmod p)$, multiplying both sides of $(1)$ by $(2+m)^{2^{n}}$ gives $(2+m)^{2^{n+1}} \equiv-1(\bmod p)$. It follows that the multiplicative order of $(2+m)$ modulo $p$ is $2^{n+2}$, or $2^{n+2} \mid p-1$, which implies that $2^{n+3} \mid(p-1)(p+1)=p^{2}-1$. +(ii) $m^{2} \equiv 3(\bmod p)$ has no integer solutions. We will work in the algebraic extension $\mathbb{Z}_{p}(\sqrt{3})$ of the field $\mathbb{Z}_{p}$. In this field $\sqrt{3}$ plays the role of $m$, so as in the previous case we obtain $(2+\sqrt{3})^{2^{n+1}}=-1$; i.e., the order of $2+\sqrt{3}$ in the multiplicative group $\mathbb{Z}_{p}(\sqrt{3})^{*}$ is $2^{n+2}$. We cannot finish the proof as in the previous case: in fact, we would conclude only that $2^{n+2}$ divides the order $p^{2}-1$ of the group. However, it will be enough to find a $u \in \mathbb{Z}_{p}(\sqrt{3})$ such that $u^{2}=2+\sqrt{3}$, since then the order of $u$ is equal to $2^{n+3}$. +Note that $(1+\sqrt{3})^{2}=2(2+\sqrt{3})$. Thus it is sufficient to prove that $\frac{1}{2}$ is a perfect square in $\mathbb{Z}_{p}(\sqrt{3})$. But we know that in this field $a_{n}=$ $0=2 a_{n-1}^{2}-1$, and hence $2 a_{n-1}^{2}=1$ which implies $\frac{1}{2}=a_{n-1}^{2}$. This completes the proof. +27. Let $p_{1}, p_{2}, \ldots, p_{r}$ be distinct primes, where $r=p-1$. Consider the sets $B_{i}=\left\{p_{i}, p_{i}^{p+1}, \ldots, p_{i}^{(r-1) p+1}\right\}$ and $B=\bigcup_{i=1}^{r} B_{i}$. Then $B$ has $(p-1)^{2}$ elements and satisfies (i) and (ii). +Now suppose that $|A| \geq r^{2}+1$ and that $A$ satisfies (i) and (ii), and let $\left\{t_{1}, \ldots, t_{r^{2}+1}\right\}$ be distinct elements of $A$, where $t_{j}=p_{1}^{\alpha_{j_{1}}} \cdot p_{2}^{\alpha_{j_{2}}} \cdots p_{r}^{\alpha_{j_{r}}}$. We shall show that the product of some elements of $A$ is a perfect $p$ th power, i.e., that there exist $\tau_{j} \in\{0,1\}\left(1 \leq j \leq r^{2}+1\right)$, not all equal to 0 , such that $T=t_{1}^{\tau_{1}} \cdot t_{2}^{\tau_{2}} \cdots t_{r^{2}+1}^{\tau_{r^{2}+1}}$ is a $p$ th power. This is equivalent to the condition that + +$$ +\sum_{j=1}^{r^{2}+1} \alpha_{i j} \tau_{j} \equiv 0(\bmod p) +$$ + +holds for all $i=1, \ldots, r$. +By Fermat's theorem it is sufficient to find integers $x_{1}, \ldots, x_{r^{2}+1}$, not all zero, such that the relation + +$$ +\sum_{j=1}^{r^{2}+1} \alpha_{i j} x_{j}^{r} \equiv 0(\bmod p) +$$ + +is satisfied for all $i \in\{1, \ldots, r\}$. Set $F_{i}=\sum_{j=1}^{r^{2}+1} \alpha_{i j} x_{j}^{r}$. We want to find $x_{1}, \ldots, x_{r}$ such that $F_{1} \equiv F_{2} \equiv \cdots \equiv F_{r} \equiv 0(\bmod p)$, which is by Fermat's theorem equivalent to + +$$ +F\left(x_{1}, \ldots, x_{r}\right)=F_{1}^{r}+F_{2}^{r}+\cdots+F_{r}^{r} \equiv 0(\bmod p) . +$$ + +Of course, one solution of (1) is $(0, \ldots, 0)$ : we are not satisfied with it because it generates the empty subset of $A$, but it tells us that (1) has at least one solution. +We shall prove that the number of solutions of (1) is divisible by $p$, which will imply the existence of a nontrivial solution and thus complete the proof. To do this, consider the sum $\sum F\left(x_{1}, \ldots, x_{r^{2}+1}\right)^{r}$ taken over all vectors $\left(x_{1}, \ldots, x_{r^{2}+1}\right)$ in the vector space $\mathbb{Z}_{p}^{r^{2}+1}$. Our statement is equivalent to + +$$ +\sum F\left(x_{1}, \ldots, x_{r^{2}+1}\right)^{r} \equiv 0(\bmod p) +$$ + +Since the degree of $F^{r}$ is $r^{2}$, in each monomial in $F^{r}$ at least one of the variables is missing. Consider any of these monomials, say $b x_{i_{1}}^{a_{1}} x_{i_{2}}^{a_{2}} \cdots x_{i_{k}}^{a_{k}}$. Then the sum $\sum b x_{i_{1}}^{a_{1}} x_{i_{2}}^{a_{2}} \cdots x_{i_{k}}^{a_{k}}$, taken over the set of all vectors $\left(x_{1}, \ldots, x_{r^{2}+1}\right) \in \mathbb{Z}_{p}^{r^{2}+1}$, is equal to + +$$ +p^{r^{2}+1-u} \cdot \sum_{\left(x_{i_{1}}, \ldots, x_{i_{k}}\right) \in \mathbb{Z}_{p}^{k}} b x_{i_{1}}^{a_{1}} x_{i_{2}}^{a_{2}} \cdots x_{i_{k}}^{a_{k}}, +$$ + +which is divisible by $p$, so that (2) is proved. Thus the answer is $(p-1)^{2}$. + +### 4.45 Solutions to the Shortlisted Problems of IMO 2004 + +1. By symmetry, it is enough to prove that $t_{1}+t_{2}>t_{3}$. We have + +$$ +\left(\sum_{i=1}^{n} t_{i}\right)\left(\sum_{i=1}^{n} \frac{1}{t}_{i}\right)=n^{2}+\sum_{i0$ and $a_{n+1}=a_{n}+a_{n-1}$ or $a_{n}-a_{n-1}$. In particular, if $a_{n}\max \left\{a_{n}, a_{n-1}\right\}$. +Let us remove all $a_{n}$ such that $a_{n}a_{n+1}$. We distinguish two cases: +(i) If $a_{n+1}>a_{n}$, we have $b_{m}=a_{n+1}$ and $b_{m-1} \geq a_{n-1}$ (since $b_{m-1}$ is either $a_{n-1}$ or $a_{n}$ ). Then $b_{m+1}-b_{m}=a_{n+2}-a_{n+1}=a_{n}=a_{n+1}-$ $a_{n-1}=b_{m}-a_{n-1} \geq b_{m}-b_{m-1}$. +(ii) If $a_{n+1}0$ can be uniquely expressed as a continued fraction of the form $a_{0}+1 /\left(a_{1}+1 /\left(a_{2}+1 /\left(\cdots+1 / a_{n}\right)\right)\right)$ (where $a_{0} \in \mathbb{N}_{0}, a_{1}, \ldots, a_{n} \in \mathbb{N}$ ). Then we write $x=\left[a_{0} ; a_{1}, a_{2}, \ldots, a_{n}\right]$. Since $n$ depends only on $x$, the function $s(x)=(-1)^{n}$ is well-defined. For $x<0$ we define $s(x)=-s(-x)$, and set $s(0)=1$. We claim that this $s(x)$ satisfies the requirements of the problem. +The equality $s(x) s(y)=-1$ trivially holds if $x+y=0$. +Suppose that $x y=1$. We may assume w.l.o.g. that $x>y>0$. Then $x>1$, so if $x=\left[a_{0} ; a_{1}, a_{2}, \ldots, a_{n}\right]$, then $a_{0} \geq 1$ and $y=0+1 / x=$ $\left[0 ; a_{0}, a_{1}, a_{2}, \ldots, a_{n}\right]$. It follows that $s(x)=(-1)^{n}, s(y)=(-1)^{n+1}$, and hence $s(x) s(y)=-1$. +Finally, suppose that $x+y=1$. We consider two cases: +(i) Let $x, y>0$. We may assume w.l.o.g. that $x>1 / 2$. Then there exist natural numbers $a_{2}, \ldots, a_{n}$ such that $x=\left[0 ; 1, a_{2}, \ldots, a_{n}\right]=$ $1 /(1+1 / t)$, where $t=\left[a_{2}, \ldots, a_{n}\right]$. Since $y=1-x=1 /(1+t)=$ +$\left[0 ; 1+a_{2}, a_{3}, \ldots, a_{n}\right]$, we have $s(x)=(-1)^{n}$ and $s(y)=(-1)^{n-1}$, giving us $s(x) s(y)=-1$. +(ii) Let $x>0>y$. If $a_{0}, \ldots, a_{n} \in \mathbb{N}$ are such that $-y=\left[a_{0} ; a_{1}, \ldots, a_{n}\right]$, then $x=\left[1+a_{0} ; a_{1}, \ldots, a_{n}\right]$. Thus $s(y)=-s(-y)=-(-1)^{n}$ and $s(x)=(-1)^{n}$, so again $s(x) s(y)=-1$. +4. Let $P(x)=a_{0}+a_{1} x+\cdots+a_{n} x^{n}$. For every $x \in \mathbb{R}$ the triple $(a, b, c)=$ $(6 x, 3 x,-2 x)$ satisfies the condition $a b+b c+c a=0$. Then the condition on $P$ gives us $P(3 x)+P(5 x)+P(-8 x)=2 P(7 x)$ for all $x$, implying that for all $i=0,1,2, \ldots, n$ the following equality holds: + +$$ +\left(3^{i}+5^{i}+(-8)^{i}-2 \cdot 7^{i}\right) a_{i}=0 +$$ + +Suppose that $a_{i} \neq 0$. Then $K(i)=3^{i}+5^{i}+(-8)^{i}-2 \cdot 7^{i}=0$. But $K(i)$ is negative for $i$ odd and positive for $i=0$ or $i \geq 6$ even. Only for $i=2$ and $i=4$ do we have $K(i)=0$. It follows that $P(x)=a_{2} x^{2}+a_{4} x^{4}$ for some real numbers $a_{2}, a_{4}$. +It is easily verified that all such $P(x)$ satisfy the required condition. +5. By the general mean inequality $\left(M_{1} \leq M_{3}\right)$, the LHS of the inequality to be proved does not exceed + +$$ +E=\frac{3}{\sqrt[3]{3}} \sqrt[3]{\frac{1}{a}+\frac{1}{b}+\frac{1}{c}+6(a+b+c)} +$$ + +From $a b+b c+c a=1$ we obtain that $3 a b c(a+b+c)=3(a b \cdot a c+$ $a b \cdot b c+a c \cdot b c) \leq(a b+a c+b c)^{2}=1$; hence $6(a+b+c) \leq \frac{2}{a b c}$. Since $\frac{1}{a}+\frac{1}{b}+\frac{1}{c}=\frac{a b+b c+c a}{a b c}=\frac{1}{a b c}$, it follows that + +$$ +E \leq \frac{3}{\sqrt[3]{3}} \sqrt[3]{\frac{3}{a b c}} \leq \frac{1}{a b c} +$$ + +where the last inequality follows from the AM-GM inequality $1=a b+b c+$ $c a \geq 3 \sqrt[3]{(a b c)^{2}}$, i.e., $a b c \leq 1 /(3 \sqrt{3})$. The desired inequality now follows. Equality holds if and only if $a=b=c=1 / \sqrt{3}$. +6. Let us make the substitution $z=x+y, t=x y$. Given $z, t \in \mathbb{R}, x, y$ are real if and only if $4 t \leq z^{2}$. Define $g(x)=2(f(x)-x)$. Now the given functional equation transforms into + +$$ +f\left(z^{2}+g(t)\right)=(f(z))^{2} \text { for all } t, z \in \mathbb{R} \text { with } z^{2} \geq 4 t +$$ + +Let us set $c=g(0)=2 f(0)$. Substituting $t=0$ into (1) gives us + +$$ +f\left(z^{2}+c\right)=(f(z))^{2} \text { for all } z \in \mathbb{R} +$$ + +If $c<0$, then taking $z$ such that $z^{2}+c=0$, we obtain from (2) that $f(z)^{2}=c / 2$, which is impossible; hence $c \geq 0$. We also observe that + +$$ +x>c \quad \text { implies } \quad f(x) \geq 0 +$$ + +If $g$ is a constant function, we easily find that $c=0$ and therefore $f(x)=x$, which is indeed a solution. +Suppose $g$ is nonconstant, and let $a, b \in \mathbb{R}$ be such that $g(a)-g(b)=d>0$. For some sufficiently large $K$ and each $u, v \geq K$ with $v^{2}-u^{2}=d$ the equality $u^{2}+g(a)=v^{2}+g(b)$ by (1) and (3) implies $f(u)=f(v)$. This further leads to $g(u)-g(v)=2(v-u)=\frac{d}{u+\sqrt{u^{2}+d}}$. Therefore every value from some suitably chosen segment $[\delta, 2 \delta]$ can be expressed as $g(u)-g(v)$, with $u$ and $v$ bounded from above by some $M$. +Consider any $x, y$ with $y>x \geq 2 \sqrt{M}$ and $\deltaN$ in (2) we obtain $k^{2}=k$, so $k=0$ or $k=1$ 。 +By (2) we have $f(-z)= \pm f(z)$, and thus $|f(z)| \leq 1$ for all $z \leq-N$. Hence $g(u)=2 f(u)-2 u \geq-2-2 u$ for $u \leq-N$, which implies that $g$ is unbounded. Hence for each $z$ there exists $t$ such that $z^{2}+g(t)>N$, and consequently $f(z)^{2}=f\left(z^{2}+g(t)\right)=k=k^{2}$. Therefore $f(z)= \pm k$ for each $z$. +If $k=0$, then $f(x) \equiv 0$, which is clearly a solution. Assume $k=1$. Then $c=2 f(0)=2$ (because $c \geq 0$ ), which together with (3) implies $f(x)=1$ for all $x \geq 2$. Suppose that $f(t)=-1$ for some $t<2$. Then $t-g(t)=3 t+2>4 t$. If also $t-g(t) \geq 0$, then for some $z \in \mathbb{R}$ we have $z^{2}=t-g(t)>4 t$, which by (1) leads to $f(z)^{2}=f\left(z^{2}+g(t)\right)=f(t)=-1$, which is impossible. Hence $t-g(t)<0$, giving us $t<-2 / 3$. On the other hand, if $X$ is any subset of $(-\infty,-2 / 3)$, the function $f$ defined by $f(x)=-1$ for $x \in X$ and $f(x)=1$ satisfies the requirements of the problem. +To sum up, the solutions are $f(x)=x, f(x)=0$ and all functions of the form + +$$ +f(x)= \begin{cases}1, & x \notin X \\ -1, & x \in X\end{cases} +$$ + +where $X \subset(-\infty,-2 / 3)$. +7. Let us set $c_{k}=A_{k-1} / A_{k}$ for $k=1,2, \ldots, n$, where we define $A_{0}=0$. We observe that $a_{k} / A_{k}=\left(k A_{k}-(k-1) A_{k-1}\right) / A_{k}=k-(k-1) c_{k}$. Now we can write the LHS of the inequality to be proved in terms of $c_{k}$, as follows: + +$$ +\sqrt[n]{\frac{G_{n}}{A_{n}}}=\sqrt[n^{2}]{c_{2} c_{3}^{2} \cdots c_{n}^{n-1}} \text { and } \frac{g_{n}}{G_{n}}=\sqrt[n]{\prod_{k=1}^{n}\left(k-(k-1) c_{k}\right)} +$$ + +By the $A M-G M$ inequality we have + +$$ +\begin{aligned} +n \sqrt[n^{2}]{1^{n(n+1) / 2} c_{2} c_{3}^{2} \ldots c_{n}^{n-1}} & \leq \frac{1}{n}\left(\frac{n(n+1)}{2}+\sum_{k=2}^{n}(k-1) c_{k}\right) \\ +& =\frac{n+1}{2}+\frac{1}{n} \sum_{k=1}^{n}(k-1) c_{k} . +\end{aligned} +$$ + +Also by the AM-GM inequality, we have + +$$ +\sqrt[n]{\prod_{k=1}^{n}\left(k-(k-1) c_{k}\right)} \leq \frac{n+1}{2}-\frac{1}{n} \sum_{k=1}^{n}(k-1) c_{k} +$$ + +Adding (1) and (2), we obtain the desired inequality. Equality holds if and only if $a_{1}=a_{2}=\cdots=a_{n}$. +8. Let us write $n=10001$. Denote by $\mathcal{T}$ the set of ordered triples $(a, C, \mathcal{S})$, where $a$ is a student, $C$ a club, and $\mathcal{S}$ a society such that $a \in C$ and $C \in \mathcal{S}$. We shall count $|\mathcal{T}|$ in two different ways. +Fix a student $a$ and a society $\mathcal{S}$. By (ii), there is a unique club $C$ such that $(a, C, \mathcal{S}) \in \mathcal{T}$. Since the ordered pair $(a, \mathcal{S})$ can be chosen in $n k$ ways, we have that $|\mathcal{T}|=n k$. +Now fix a club $C$. By (iii), $C$ is in exactly $(|C|-1) / 2$ societies, so there are $|C|(|C|-1) / 2$ triples from $\mathcal{T}$ with second coordinate $C$. If $\mathcal{C}$ is the set of all clubs, we obtain $|\mathcal{T}|=\sum_{C \in \mathcal{C}} \frac{|C|(|C|-1)}{2}$. But we also conclude from (i) that + +$$ +\sum_{C \in \mathcal{C}} \frac{|C|(|C|-1)}{2}=\frac{n(n-1)}{2} +$$ + +Therefore $n(n-1) / 2=n k$, i.e., $k=(n-1) / 2=5000$. +On the other hand, for $k=(n-1) / 2$ there is a desired configuration with only one club $C$ that contains all students and $k$ identical societies with only one element (the club $C$ ). It is easy to verify that (i)-(iii) hold. +9. Obviously we must have $2 \leq k \leq n$. We shall prove that the possible values for $k$ and $n$ are $2 \leq k \leq n \leq 3$ and $3 \leq k \leq n$. Denote all colors and circles by $1, \ldots, n$. Let $F(i, j)$ be the set of colors of the common points of circles $i$ and $j$. +Suppose that $k=2i$. +We now prove by induction on $k$ that a desired coloring exists for each $n \geq k \geq 3$. Let there be given $n$ circles. By the inductive hypothesis, circles $1,2, \ldots, n-1$ can be colored in $n-1$ colors, $k$ of which appear on each circle, such that color $i$ appears on circle $i$. Then we set $F(i, n)=\{i, n\}$ for $i=1, \ldots, k$ and $F(i, n)=\{n\}$ for $i>n$. We thus obtain a coloring of the $n$ circles in $n$ colors, such that $k+1$ colors (including color $i$ ) appear on each circle $i$. +10. The least number of edges of such a graph is $n$. + +We note that deleting edge $A B$ of a 4-cycle $A B C D$ from a connected and nonbipartite graph $G$ yields a connected and nonbipartite graph, say $H$. Indeed, the connectedness is obvious; also, if $H$ were bipartite with partition of the set of vertices into $P_{1}$ and $P_{2}$, then w.l.o.g. $A, C \in P_{1}$ and $B, D \in P_{2}$, so $G=H \cup\{A B\}$ would also be bipartite with the same partition, a contradiction. +Any graph that can be obtained from the complete $n$-graph in the described way is connected and has at least one cycle (otherwise it would be bipartite); hence it must have at least $n$ edges. +Now consider a complete graph with vertices $V_{1}, V_{2}, \ldots, V_{n}$. Let us remove every edge $V_{i} V_{j}$ with $3 \leq in / 2$, and 0 otherwise. This matrix satisfies the conditions from the problem and all row sums and column sums are equal to $\pm n / 2$. Hence $C \geq n / 2$. +Let us show that $C=n / 2$. Assume to the contrary that there is a matrix $B=\left(b_{i j}\right)_{i, j=1}^{n}$ all of whose row sums and column sums are either greater than $n / 2$ or smaller than $-n / 2$. We may assume w.l.o.g. that at least $n / 2$ row sums are positive and, permuting rows if necessary, that the first $n / 2$ rows have positive sums. The sum of entries in the $n / 2 \times n$ submatrix $B^{\prime}$ consisting of first $n / 2$ rows is greater than $n^{2} / 4$, and since each column of $B^{\prime}$ has sum at most $n / 2$, it follows that more than $n / 2$ column sums of $B^{\prime}$, and therefore also of $B$, are positive. Again, suppose w.l.o.g. that the first $n / 2$ column sums are positive. Thus the sums $R^{+}$and $C^{+}$of entries in the first $n / 2$ rows and in the first $n / 2$ columns respectively are greater than $n^{2} / 4$. Now the sum of all entries of $B$ can be written as + +$$ +\sum a_{i j}=R^{+}+C^{+}+\sum_{\substack{i>n / 2 \\ j>n / 2}} a_{i j}-\sum_{\substack{i \leq n / 2 \\ j \leq n / 2}} a_{i j}>\frac{n^{2}}{2}-\frac{n^{2}}{4}-\frac{n^{2}}{4}=0 +$$ + +a contradiction. Hence $C=n / 2$, as claimed. +12. We say that a number $n \in\{1,2, \ldots, N\}$ is winning if the player who is on turn has a winning strategy, and losing otherwise. The game is of type $A$ if and only if 1 is a losing number. +Let us define $n_{0}=N, n_{i+1}=\left[n_{i} / 2\right]$ for $i=0,1, \ldots$ and let $k$ be such that $n_{k}=1$. Consider the sets $A_{i}=\left\{n_{i+1}+1, \ldots, n_{i}\right\}$. We call a set $A_{i}$ all-winning if all numbers from $A_{i}$ are winning, even-winning if even numbers are winning and odd are losing, and odd-winning if odd numbers are winning and even are losing. +(i) Suppose $A_{i}$ is even-winning and consider $A_{i+1}$. Multiplying any number from $A_{i+1}$ by 2 yields an even number from $A_{i}$, which is a losing number. Thus $x \in A_{i+1}$ is winning if and only if $x+1$ is losing, i.e., if and only if it is even. Hence $A_{i+1}$ is also even-winning. +(ii) Suppose $A_{i}$ is odd-winning. Then each $k \in A_{i+1}$ is winning, since $2 k$ is losing. Hence $A_{i+1}$ is all-winning. +(iii) Suppose $A_{i}$ is all-winning. Multiplying $x \in A_{i+1}$ by two is then a losing move, so $x$ is winning if and only if $x+1$ is losing. Since $n_{i+1}$ is losing, $A_{i+1}$ is odd-winning if $n_{i+1}$ is even and even-winning otherwise. We observe that $A_{0}$ is even-winning if $N$ is odd and odd-winning otherwise. Also, if some $A_{i}$ is even-winning, then all $A_{i+1}, A_{i+2}, \ldots$ are evenwinning and thus 1 is losing; i.e., the game is of type $A$. The game is of type $B$ if and only if the sets $A_{0}, A_{1}, \ldots$ are alternately odd-winning and allwinning with $A_{0}$ odd-winning, which is equivalent to $N=n_{0}, n_{2}, n_{4}, \ldots$ all being even. Thus $N$ is of type $B$ if and only if all digits at the odd positions in the binary representation of $N$ are zeros. +Since $2004=\overline{11111010100}$ in the binary system, 2004 is of type $A$. The least $N>2004$ that is of type $B$ is $\overline{100000000000}=2^{11}=2048$. Thus the answer to part (b) is 2048. +13. Since $X_{i}, Y_{i}, i=1, \ldots, 2004$, are 4008 distinct subsets of the set $S_{n}=$ $\{1,2, \ldots, n\}$, it follows that $2^{n} \geq 4008$, i.e. $n \geq 12$. +Suppose $n=12$. Let $\mathcal{X}=\left\{X_{1}, \ldots, X_{2004}\right\}, \mathcal{Y}=\left\{Y_{1}, \ldots, Y_{2004}\right\}, \mathcal{A}=$ $\mathcal{X} \cup \mathcal{Y}$. Exactly $2^{12}-4008=88$ subsets of $S_{n}$ do not occur in $\mathcal{A}$. +Since each row intersects each column, we have $X_{i} \cap Y_{j} \neq \emptyset$ for all $i, j$. Suppose $\left|X_{i}\right|,\left|Y_{j}\right| \leq 3$ for some indices $i, j$. Since then $\left|X_{i} \cup Y_{j}\right| \leq 5$, any of at least $2^{7}>88$ subsets of $S_{n} \backslash\left(X_{i} \cap Y_{j}\right)$ can occur in neither $\mathcal{X}$ nor $\mathcal{Y}$, which is impossible. Hence either in $\mathcal{X}$ or in $\mathcal{Y}$ all subsets are of size at least 4. Suppose w.l.o.g. that $k=\left|X_{l}\right|=\min _{i}\left|X_{i}\right| \geq 4$. There are + +$$ +n_{k}=\binom{12-k}{0}+\binom{12-k}{1}+\cdots+\binom{12-k}{k-1} +$$ + +subsets of $S \backslash X_{l}$ with fewer than $k$ elements, and none of them can be either in $\mathcal{X}$ (because $\left|X_{l}\right|$ is minimal in $\mathcal{X}$ ) or in $\mathcal{Y}$. Hence we must have $n_{k} \leq 88$. Since $n_{4}=93$ and $n_{5}=99$, it follows that $k \geq 6$. But then none of the $\binom{12}{0}+\cdots+\binom{12}{5}=1586$ subsets of $S_{n}$ is in $\mathcal{X}$, hence at least $1586-88=1498$ of them are in $\mathcal{Y}$. The 1498 complements of these subsets +also do not occur in $\mathcal{X}$, which adds to 3084 subsets of $S_{n}$ not occurring in $\mathcal{X}$. This is clearly a contradiction. +Now we construct a golden matrix for $n=13$. Let + +$$ +A_{1}=\left[\begin{array}{ll} +1 & 1 \\ +2 & 3 +\end{array}\right] \quad \text { and } \quad A_{m}=\left[\begin{array}{ll} +A_{m-1} & A_{m-1} \\ +A_{m-1} & B_{m-1} +\end{array}\right] \text { for } m=2,3, \ldots +$$ + +where $B_{m-1}$ is the $2^{m-1} \times 2^{m-1}$ matrix with all entries equal to $m+2$. It can be easily proved by induction that each of the matrices $A_{m}$ is golden. Moreover, every upper-left square submatrix of $A_{m}$ of size greater than $2^{m-1}$ is also golden. Since $2^{10}<2004<2^{11}$, we thus obtain a golden matrix of size 2004 with entries in $S_{13}$. +14. Suppose that an $m \times n$ rectangle can be covered by "hooks". For any hook $H$ there is a unique hook $K$ that covers its " inside" square. Then also $H$ covers the inside square of $K$, so the set of hooks can be partitioned into pairs of type $\{H, K\}$, each of which forms one of the following two figures consisting of 12 squares: +![](https://cdn.mathpix.com/cropped/2024_11_18_8e985d6b9c83aa3e9d0eg-730.jpg?height=252&width=979&top_left_y=972&top_left_x=306) + +Thus the $m \times n$ rectangle is covered by these tiles. It immediately follows that $12 \mid \mathrm{mn}$. +Suppose one of $m, n$ is divisible by 4 . Let w.l.o.g. $4 \mid m$. If $3 \mid n$, one can easily cover the rectangle by $3 \times 4$ rectangles and therefore by hooks. Also, if $12 \mid m$ and $n \notin\{1,2,5\}$, then there exist $k, l \in \mathbb{N}_{0}$ such that $n=3 k+4 l$, and thus the rectangle $m \times n$ can be partitioned into $3 \times 12$ and $4 \times 12$ rectangles all of which can be covered by hooks. If $12 \mid m$ and $n=1,2$, or 5 , then it is easy to see that covering by hooks is not possible. +Now suppose that $4 \nmid m$ and $4 \nmid n$. Then $m, n$ are even and the number of tiles is odd. Assume that the total number of tiles of types $A_{1}$ and $B_{1}$ is odd (otherwise the total number of tiles of types $A_{2}$ and $B_{2}$ is odd, which is analogous). If we color in black all columns whose indices are divisible by 4 , we see that each tile of type $A_{1}$ or $B_{1}$ covers three black squares, which yields an odd number in total. Hence the total number of black squares covered by the tiles of types $A_{2}$ and $B_{2}$ must be odd. This is impossible, since each such tile covers two or four black squares. +15. Denote by $V_{1}, \ldots, V_{n}$ the vertices of a graph $G$ and by $E$ the set of its edges. For each $i=1, \ldots, n$, let $A_{i}$ be the set of vertices connected to $V_{i}$ by an edge, $G_{i}$ the subgraph of $G$ whose set of vertices is $A_{i}$, and $E_{i}$ the set of edges of $G_{i}$. Also, let $v_{i}, e_{i}$, and $t_{i}=f\left(G_{i}\right)$ be the numbers of vertices, edges, and triangles in $G_{i}$ respectively. + +The numbers of tetrahedra and triangles one of whose vertices is $V_{i}$ are respectively equal to $t_{i}$ and $e_{i}$. Hence + +$$ +\sum_{i=1}^{n} v_{i}=2|E|, \quad \sum_{i=1}^{n} e_{i}=3 f(G) \quad \text { and } \quad \sum_{i=1}^{n} t_{i}=4 g(G) . +$$ + +Since $e_{i} \leq v_{i}\left(v_{i}-1\right) / 2 \leq v_{i}^{2} / 2$ and $e_{i} \leq|E|$, we obtain $e_{i}^{2} \leq v_{i}^{2}|E| / 2$, i.e., $e_{i} \leq v_{i} \sqrt{|E| / 2}$. Summing over all $i$ yields $3 f(G) \leq 2|E| \sqrt{|E| / 2}$, or equivalently $f(G)^{2} \leq 2|E|^{3} / 9$. Since this relation holds for each graph $G_{i}$, it follows that + +$$ +t_{i}=f\left(G_{i}\right)=f\left(G_{i}\right)^{1 / 3} f\left(G_{i}\right)^{2 / 3} \leq\left(\frac{2}{9}\right)^{1 / 3} f(G)^{1 / 3} e_{i} +$$ + +Summing the last inequality for $i=1, \ldots, n$ gives us + +$$ +4 g(G) \leq 3\left(\frac{2}{9}\right)^{1 / 3} f(G)^{1 / 3} \cdot f(G), \quad \text { i.e. } \quad g(G)^{3} \leq \frac{3}{32} f(G)^{4} +$$ + +The constant $c=3 / 32$ is the best possible. Indeed, in a complete graph $C_{n}$ it holds that $g\left(K_{n}\right)^{3} / f\left(K_{n}\right)^{4}=\binom{n}{4}^{3}\binom{n}{3}^{-4} \rightarrow \frac{3}{32}$ as $n \rightarrow \infty$. +Remark. Let $N_{k}$ be the number of complete $k$-subgraphs in a finite graph $G$. Continuing inductively, one can prove that $N_{k+1}^{k} \leq \frac{k!}{(k+1)^{k}} N_{k}^{k+1}$. +16. Note that $\triangle A N M \sim \triangle A B C$ and consequently $A M \neq A N$. Since $O M=$ $O N$, it follows that $O R$ is a perpendicular bisector of $M N$. Thus, $R$ is the common point of the median of $M N$ and the bisector of $\angle M A N$. Then it follows from a well-known fact that $R$ lies on the circumcircle of $\triangle A M N$. Let $K$ be the intersection of $A R$ and $B C$. We then have $\angle M R A=$ $\angle M N A=\angle A B K$ and $\angle N R A=\angle N M A=\angle A C K$, from which we conclude that $R M B K$ and $R N C K$ are cyclic. Thus $K$ is the desired intersection of the circumcircles of $\triangle B M R$ and $\triangle C N R$ and it indeed lies on $B C$. +17. Let $H$ be the reflection of $G$ about $A B(G H \| \ell)$. Let $M$ be the intersection of $A B$ and $\ell$. Since $\angle F E A=\angle F M A=90^{\circ}$, it follows that $A E M F$ is cyclic and hence $\angle D F E=\angle B A E=\angle D E F$. The last equality holds because $D E$ is tangent to $\Gamma$. It follows that $D E=$ $D F$ and hence $D F^{2}=D E^{2}=$ +![](https://cdn.mathpix.com/cropped/2024_11_18_8e985d6b9c83aa3e9d0eg-731.jpg?height=396&width=459&top_left_y=1591&top_left_x=841) +$D C \cdot D A$ (the power of $D$ with respect to $\Gamma$ ). It then follows that $\angle D C F=\angle D F A=\angle H G A=\angle H C A$. Thus it follows that $H$ lies on $C F$ as desired. +18. It is important to note that since $\beta<\gamma, \angle A D C=90^{\circ}-\gamma+\beta$ is acute. It is elementary that $\angle C A O=90^{\circ}-\beta$. Let $X$ and $Y$ respectively be the intersections of $F E$ and $G H$ with $A D$. We trivially get $X \in E F \perp A D$ and $\triangle A G H \cong \triangle A C B$. Consequently, $\angle G A Y=\angle O A B=90^{\circ}-\gamma=$ $90^{\circ}-\angle A G Y$. Hence, $G H \perp A D$ and thus $G H \| F E$. That $E F G H$ is a rectangle is now equivalent to $F X=G Y$ and $E X=H Y$. +We have that $G Y=A G \sin \gamma=A C \sin \gamma$ and $F X=A F \sin \gamma$ (since $\angle A F X=\gamma$ ). Thus, + +$$ +F X=G Y \Leftrightarrow C F=A F=A C \Leftrightarrow \angle A F C=60^{\circ} \Leftrightarrow \angle A D C=30^{\circ} . +$$ + +Since $\angle A D C=180^{\circ}-\angle D C A-\angle D A C=180^{\circ}-\gamma-\left(90^{\circ}-\beta\right)$, it immediately follows that $F X=G Y \Leftrightarrow \gamma-\beta=60^{\circ}$. We similarly obtain $E X=H Y \Leftrightarrow \gamma-\beta=60^{\circ}$, proving the statement of the problem. +19. Assume first that the points $A, B, C, D$ are concyclic. Let the lines $B P$ and $D P$ meet the circumcircle of $A B C D$ again at $E$ and $F$, respectively. Then it follows from the given conditions that $\widehat{A B}=\widehat{C F}$ and $\widehat{A D}=\widehat{C E}$; hence $B F \| A C$ and $D E \| A C$. Therefore $B F E D$ and $B F A C$ are isosceles trapezoids and thus $P=B E \cap D F$ lies on the common bisector of segments $B F, E D, A C$. Hence $A P=C P$. +Assume in turn that $A P=C P$. Let $P$ w.l.o.g. lie in the triangles $A C D$ and $B C D$. Let $B P$ and $D P$ meet $A C$ at $K$ and $L$, respectively. The points $A$ and $C$ are isogonal conjugates with respect to $\triangle B D P$, which implies that $\angle A P K=\angle C P L$. Since $A P=C P$, we infer that $K$ and $L$ are symmetric with respect to the perpendicular bisector $p$ of $A C$. Let $E$ be the reflection of $D$ in $p$. Then $E$ lies on the line $B P$, and the triangles $A P D$ and $C P E$ are congruent. Thus $\angle B D C=\angle A D P=\angle B E C$, which means that the points $B, C, E, D$ are concyclic. Moreover, $A, C, E, D$ are also concyclic. Hence, $A B C D$ is a cyclic quadrilateral. +20. We first establish the following lemma. + +Lemma. Let $A B C D$ be an isosceles trapezoid with bases $A B$ and $C D$. The diagonals $A C$ and $B D$ intersect at $S$. Let $M$ be the midpoint of $B C$, and let the bisector of the angle $B S C$ intersect $B C$ at $N$. Then $\angle A M D=\angle A N D$. +Proof. It suffices to show that the points $A, D, M, N$ are concyclic. The statement is trivial for $A D \| B C$. Let us now assume that $A D$ and $B C$ meet at $X$, and let $X A=X B=a, X C=X D=b$. Since $S N$ is the bisector of $\angle C S B$, we have + +$$ +\frac{a-X N}{X N-b}=\frac{B N}{C N}=\frac{B S}{C S}=\frac{A B}{C D}=\frac{a}{b} +$$ + +and an easy computation yields $X N=\frac{2 a b}{a+b}$. We also have $X M=\frac{a+b}{2}$; hence $X M \cdot X N=X A \cdot X D$. Therefore $A, D, M, N$ are concyclic, as needed. + +Denote by $C_{i}$ the midpoint of the side $A_{i} A_{i+1}, i=1, \ldots, n-1$. By definition $C_{1}=B_{1}$ and $C_{n-1}=B_{n-1}$. Since $A_{1} A_{i} A_{i+1} A_{n}$ is an isosceles trapezoid with $A_{1} A_{i} \| A_{i+1} A_{n}$ for $i=2, \ldots, n-2$, it follows from the lemma that $\angle A_{1} B_{i} A_{n}=\angle A_{1} C_{i} A_{n}$ for all $i$. +The sum in consideration thus equals $\angle A_{1} C_{1} A_{n}+\angle A_{1} C_{2} A_{n}+\cdots+$ $\angle A_{1} C_{n-1} A_{n}$. Moreover, the triangles $A_{1} C_{i} A_{n}$ and $A_{n+2-i} C_{1} A_{n+1-i}$ are congruent (a rotation about the center of the $n$-gon carries the first one to the second), and consequently + +$$ +\angle A_{1} C_{i} A_{n}=\angle A_{n+2-i} C_{1} A_{n+1-i} +$$ + +![](https://cdn.mathpix.com/cropped/2024_11_18_8e985d6b9c83aa3e9d0eg-733.jpg?height=417&width=470&top_left_y=415&top_left_x=847) +for $i=2, \ldots, n-1$. +Hence $\Sigma=\angle A_{1} C_{1} A_{n}+\angle A_{n} C_{1} A_{n-1}+\cdots+\angle A_{3} C_{1} A_{2}=\angle A_{1} C_{1} A_{2}=180^{\circ}$. +21. Let $A B C$ be the triangle of maximum area $S$ contained in $\mathcal{P}$ (it exists because of compactness of $\mathcal{P}$ ). Draw parallels to $B C, C A, A B$ through $A, B, C$, respectively, and denote the triangle thus obtained by $A_{1} B_{1} C_{1}$ ( $A \in B_{1} C_{1}$, etc.). Since each triangle with vertices in $\mathcal{P}$ has area at most $S$, the entire polygon $\mathcal{P}$ is contained in $A_{1} B_{1} C_{1}$. +Next, draw lines of support of $\mathcal{P}$ parallel to $B C, C A, A B$ and not intersecting the triangle $A B C$. They determine a convex hexagon $U_{a} V_{a} U_{b} V_{b} U_{c} V_{c}$ containing $\mathcal{P}$, with $V_{b}, U_{c} \in B_{1} C_{1}, V_{c}, U_{a} \in C_{1} A_{1}, V_{a}, U_{b} \in A_{1} B_{1}$. Each of the line segments $U_{a} V_{a}, U_{b} V_{b}, U_{c} V_{c}$ contains points of $\mathcal{P}$. Choose such points $A_{0}, B_{0}, C_{0}$ on $U_{a} V_{a}, U_{b} V_{b}, U_{c} V_{c}$, respectively. The convex hexagon $A C_{0} B A_{0} C B_{0}$ is contained in $\mathcal{P}$, because the latter is convex. We prove that $A C_{0} B A_{0} C B_{0}$ has area at least $3 / 4$ the area of $\mathcal{P}$. +Let $x, y, z$ denote the areas of triangles $U_{a} B C, U_{b} C A$, and $U_{c} A B$. Then $S_{1}=S_{A C_{0} B A_{0} C B_{0}}=S+x+y+z$. On the other hand, the triangle $A_{1} U_{a} V_{a}$ is similar to $\triangle A_{1} B C$ with similitude $\tau=(S-x) / S$, and hence its area is $\tau^{2} S=(S-x)^{2} / S$. Thus the area of quadrilateral $U_{a} V_{a} C B$ is $S-(S-x)^{2} / S=2 z-z^{2} / S$. Analogous formulas hold for quadrilaterals $U_{b} V_{b} A C$ and $U_{c} V_{c} B A$. Therefore + +$$ +\begin{aligned} +S_{\mathcal{P}} & \leq S_{U_{a} V_{a} U_{b} V_{b} U_{c} V_{c}}=S+S_{U_{a} V_{a} C B}+S_{U_{b} V_{b} A C}+S_{U_{c} V_{c} B A} \\ +& =S+2(x+y+z)-\frac{x^{2}+y^{2}+z^{2}}{S} \\ +& \leq S+2(x+y+z)-\frac{(x+y+z)^{2}}{3 S} . +\end{aligned} +$$ + +Now $4 S_{1}-3 S_{\mathcal{P}} \geq=S-2(x+y+z)+(x+y+z)^{2} / S=(S-x-y-z)^{2} / S \geq 0$; i.e., $S_{1} \geq 3 S_{\mathcal{P}} / 4$, as claimed. +22. The proof uses the following observation: + +Lemma. In a triangle $A B C$, let $K, L$ be the midpoints of the sides $A C, A B$, respectively, and let the incircle of the triangle touch $B C, C A$ at $D, E$, respectively. Then the lines $K L$ and $D E$ intersect on the bisector of the angle $A B C$. +Proof. Let the bisector $\ell_{b}$ of $\angle A B C$ meet $D E$ at $T$. One can assume that $A B \neq B C$, or else $T \equiv K \in K L$. Note that the incenter $I$ of $\triangle A B C$ is between $B$ and $T$, and also $T \neq E$. From the triangles $B D T$ and $D E C$ we obtain $\angle I T D=\alpha / 2=\angle I A E$, which implies that $A, I, T, E$ are concyclic. Then $\angle A T B=\angle A E I=90^{\circ}$. Thus $L$ is the circumcenter of $\triangle A T B$ from which $\angle L T B=\angle L B T=\angle T B C \Rightarrow L T \| B C \Rightarrow T \in$ $K L$, which is what we were supposed to prove. +Let the incircles of $\triangle A B X$ and $\triangle A C X$ touch $B X$ at $D$ and $F$, respectively, and let them touch $A X$ at $E$ and $G$, respectively. Clearly, $D E$ and $F G$ are parallel. If the line $P Q$ intersects $B X$ and $A X$ at $M$ and $N$, respectively, then $M D^{2}=M P \cdot M Q=M F^{2}$, i.e., $M D=M F$ and analogously $N E=N G$. It follows that $P Q$ is parallel to $D E$ and $F G$ and equidistant from them. +The midpoints of $A B, A C$, and $A X$ lie on the same line $m$, parallel to $B C$. Applying the lemma to $\triangle A B X$, we conclude that $D E$ passes through the common point $U$ of $m$ and the bisector of $\angle A B X$. Analogously, $F G$ passes through the common point $V$ of $m$ and the bisector of $\angle A C X$. Therefore $P Q$ passes through the midpoint $W$ of the line segment $U V$. Since $U, V$ do not depend on $X$, neither does $W$. +23. To start with, note that point $N$ is uniquely determined by the imposed properties. Indeed, $f(X)=A X / B X$ is a monotone function on both arcs $A B$ of the circumcircle of $\triangle A B M$. Denote by $P$ and $Q$ respectively the second points of intersection of the line $E F$ with the circumcircles of $\triangle A B E$ and $\triangle A B F$. The problem is equivalent to showing that $N \in P Q$. In fact, we shall prove that $N$ coincides with the midpoint $\bar{N}$ of segment $P Q$. +The cyclic quadrilaterals $A P B E$, $A Q B F$, and $A B C D$ yield $\angle A P Q=$ $180^{\circ}-\angle A P E=180^{\circ}-\angle A B E=$ $\angle A D C$ and $\angle A Q P=\angle A Q F=$ $\angle A B F=\angle A C D$. It follows that $\triangle A P Q \sim \triangle A D C$, and consequently $\triangle A \bar{N} P \sim \triangle A M D$. Analo- +![](https://cdn.mathpix.com/cropped/2024_11_18_8e985d6b9c83aa3e9d0eg-734.jpg?height=604&width=446&top_left_y=1342&top_left_x=857) +gously $\triangle B \bar{N} P \sim \triangle B M C$. Therefore $A \bar{N} / A M=P Q / D C=B \bar{N} / B M$, i.e., $A \bar{N} / B \bar{N}=A M / B M$. Moreover, $\angle A \bar{N} B=\angle A \bar{N} P+\angle P \bar{N} B=$ $\angle A M D+\angle B M C=180^{\circ}-\angle A M B$, which means that point $\bar{N}$ lies on +the circumcircle of $\triangle A M B$. By the uniqueness of $N$, we conclude that $\bar{N} \equiv N$, which completes the solution. +24. Setting $m=a n$ we reduce the given equation to $m / \tau(m)=a$. + +Let us show that for $a=p^{p-1}$ the above equation has no solutions in $\mathbb{N}$ if $p>3$ is a prime. Assume to the contrary that $m \in \mathbb{N}$ is such that $m=p^{p-1} \tau(m)$. Then $p^{p-1} \mid m$, so we may set $m=p^{\alpha} k$, where $\alpha, k \in \mathbb{N}$, $\alpha \geq p-1$, and $p \nmid k$. Let $k=p_{1}^{\alpha_{1}} \cdots p_{r}^{\alpha_{r}}$ be the decomposition of $k$ into primes. Then $\tau(k)=\left(\alpha_{1}+1\right) \cdots\left(\alpha_{r}+1\right)$ and $\tau(m)=(\alpha+1) \tau(k)$. Our equation becomes + +$$ +p^{\alpha-p+1} k=(\alpha+1) \tau(k) . +$$ + +We observe that $\alpha \neq p-1$ : otherwise the RHS would be divisible by $p$ and the LHS would not be so. It follows that $\alpha \geq p$, which also easily implies that $p^{\alpha-p+1} \geq \frac{p}{p+1}(\alpha+1)$. +Furthermore, since $\alpha+1$ cannot be divisible by $p^{\alpha-p+1}$ for any $\alpha \geq p$, it follows that $p \mid \tau(k)$. Thus if $p \mid \tau(k)$, then at least one $\alpha_{i}+1$ is divisible by $p$ and consequently $\alpha_{i} \geq p-1$ for some $i$. Hence $k \geq \frac{p_{i}^{\alpha_{i}}}{\alpha_{i}+1} \tau(k) \geq \frac{2^{p-1}}{p} \tau(k)$. But then we have + +$$ +p^{\alpha-p+1} k \geq \frac{p}{p+1}(\alpha+1) \cdot \frac{2^{p-1}}{p} \tau(k)>(\alpha+1) \tau(k), +$$ + +contradicting (1). Therefore (1) has no solutions in $\mathbb{N}$. +Remark. There are many other values of $a$ for which the considered equation has no solutions in $\mathbb{N}$ : for example, $a=6 p$ for a prime $p \geq 5$. +25. Let $n$ be a natural number. For each $k=1,2, \ldots, n$, the number $(k, n)$ is a divisor of $n$. Consider any divisor $d$ of $n$. If $(k, n)=n / d$, then $k=n l / d$ for some $l \in \mathbb{N}$, and $(k, n)=(l, d) n / d$, which implies that $l$ is coprime to $d$ and $l \leq d$. It follows that $(k, n)$ is equal to $n / d$ for exactly $\varphi(d)$ natural numbers $k \leq n$. Therefore + +$$ +\psi(n)=\sum_{k=1}^{n}(k, n)=\sum_{d \mid n} \varphi(d) \frac{n}{d}=n \sum_{d \mid n} \frac{\varphi(d)}{d} +$$ + +(a) Let $n, m$ be coprime. Then each divisor $f$ of $m n$ can be uniquely expressed as $f=d e$, where $d \mid n$ and $e \mid m$. We now have by (1) + +$$ +\begin{aligned} +\psi(m n) & =m n \sum_{f \mid m n} \frac{\varphi(f)}{f}=m n \sum_{d|n, e| m} \frac{\varphi(d e)}{d e} \\ +& =m n \sum_{d|n, e| m} \frac{\varphi(d)}{d} \frac{\varphi(e)}{e}=\left(n \sum_{d \mid n} \frac{\varphi(d)}{d}\right)\left(m \sum_{e \mid m} \frac{\varphi(e)}{e}\right) \\ +& =\psi(m) \psi(n) . +\end{aligned} +$$ + +(b) Let $n=p^{k}$, where $p$ is a prime and $k$ a positive integer. According to (1), + +$$ +\frac{\psi(n)}{n}=\sum_{i=0}^{k} \frac{\varphi\left(p^{i}\right)}{p^{i}}=1+\frac{k(p-1)}{p} +$$ + +Setting $p=2$ and $k=2(a-1)$ we obtain $\psi(n)=a n$ for $n=2^{2(a-1)}$. +(c) We note that $\psi\left(p^{p}\right)=p^{p+1}$ if $p$ is a prime. Hence, if $a$ has an odd prime factor $p$ and $a_{1}=a / p$, then $x=p^{p} 2^{2 a_{1}-2}$ is a solution of $\psi(x)=a x$ different from $x=2^{2 a-2}$. +Now assume that $a=2^{k}$ for some $k \in \mathbb{N}$. Suppose $x=2^{\alpha} y$ is a positive integer such that $\psi(x)=2^{k} x$. Then $2^{\alpha+k} y=\psi(x)=\psi\left(2^{\alpha}\right) \psi(y)=$ $(\alpha+2) 2^{\alpha-1} \psi(y)$, i.e., $2^{k+1} y=(\alpha+2) \psi(y)$. We notice that for each odd $y, \psi(y)$ is (by definition) the sum of an odd number of odd summands and therefore odd. It follows that $\psi(y) \mid y$. On the other hand, $\psi(y)>$ $y$ for $y>1$, so we must have $y=1$. Consequently $\alpha=2^{k+1}-2=2 a-2$, giving us the unique solution $x=2^{2 a-2}$. +Thus $\psi(x)=a x$ has a unique solution if and only if $a$ is a power of 2 . +26. For $m=n=1$ we obtain that $f(1)^{2}+f(1)$ divides $\left(1^{2}+1\right)^{2}=4$, from which we find that $f(1)=1$. +Next, we show that $f(p-1)=p-1$ for each prime $p$. By the hypothesis for $m=1$ and $n=p-1, f(p-1)+1$ divides $p^{2}$, so $f(p-1)$ equals either $p-1$ or $p^{2}-1$. If $f(p-1)=p^{2}-1$, then $f(1)+f(p-1)^{2}=p^{4}-2 p^{2}+2$ divides $\left(1+(p-1)^{2}\right)^{2}4$. +We note that if $x \in S_{n}$, then also $n-x \in S_{n}$ and $(x, n)=1$. Thus $S_{n}$ splits into pairs $\{x, n-x\}$, where $x \in S_{n}$ and $x \leq n / 2$. In each of these pairs the product of elements gives remainder -1 upon division by $n$. Therefore $P_{n} \equiv(-1)^{m}$, where $S_{n}$ has $2 m$ elements. It remains to find the parity of $m$. +Suppose first that $n>4$ is divisible by 4 . Whenever $x \in S_{n}$, the numbers $|n / 2-x|, n-x, n-|n / 2-x|$ also belong to $S_{n}$ (indeed, $n \mid(n / 2-x)^{2}-1=$ $n^{2} / 4-n x+x^{2}-1$ because $n \mid n^{2} / 4$, etc.). In this way the set $S_{n}$ splits into four-element subsets $\{x, n / 2-x, n / 2+x, n-x\}$, where $x \in S_{n}$ and $x4\right)$. Therefore $m=\left|S_{n}\right| / 2$ is even and $P_{n} \equiv 1$ $(\bmod m)$. +Now let $n$ be odd. If $n \mid x^{2}-1=(x-1)(x+1)$, then there exist natural numbers $a, b$ such that $a b=n, a|x-1, b| x+1$. Obviously $a$ and $b$ are coprime. Conversely, given any odd $a, b \in \mathbb{N}$ such that $(a, b)=1$ and $a b=n$, by the Chinese remainder theorem there exists $x \in\{1,2, \ldots, n-1\}$ such that $a \mid x-1$ and $b \mid x+1$. This gives a bijection between all ordered pairs $(a, b)$ with $a b=n$ and $(a, b)=1$ and the elements of $S_{n}$. Now if $n=p_{1}^{\alpha_{1}} \cdots p_{k}^{\alpha_{k}}$ is the decomposition of $n$ into primes, the number of pairs $(a, b)$ is equal to $2^{k}$ (since for every $i$, either $p_{i}^{\alpha_{i}} \mid a$ or $p_{i}^{\alpha_{i}} \mid b$ ), and hence +$m=2^{k-1}$. Thus $P_{n} \equiv-1(\bmod n)$ if $n$ is a power of an odd prime, and $P_{n} \equiv 1$ otherwise. +Finally, let $n$ be even but not divisible by 4 . Then $x \in S_{n}$ if and only if $x$ or $n-x$ belongs to $S_{n / 2}$ and $x$ is odd. Since $n / 2$ is odd, for each $x \in S_{n / 2}$ either $x$ or $x+n / 2$ belongs to $S_{n}$, and by the case of $n$ odd we have $S_{n} \equiv \pm 1(\bmod n / 2)$, depending on whether or not $n / 2$ is a power of a prime. Since $S_{n}$ is odd, it follows that $P_{n} \equiv-1(\bmod n)$ if $n / 2$ is a power of a prime, and $P_{n} \equiv 1$ otherwise. +Second solution. Obviously $S_{n}$ is closed under multiplication modulo $n$. This implies that $S_{n}$ with multiplication modulo $n$ is a subgroup of $\mathbb{Z}_{n}$, and therefore there exist elements $a_{1}=-1, a_{2}, \ldots, a_{k} \in S_{n}$ that generate $S_{n}$. In other words, since the $a_{i}$ are of order two, $S_{n}$ consists of products $\prod_{i \in A} a_{i}$, where $A$ runs over all subsets of $\{1,2, \ldots, k\}$. Thus $S_{n}$ has $2^{k}$ elements, and the product of these elements equals $P_{n} \equiv\left(a_{1} a_{2} \cdots a_{k}\right)^{2^{k-1}}$ $(\bmod n)$. Since $a_{i}^{2} \equiv 1(\bmod n)$, it follows that $P_{n} \equiv 1$ if $k \geq 2$, i.e., if $\left|S_{n}\right|>2$. Otherwise $P_{n} \equiv-1(\bmod n)$. +We note that $\left|S_{n}\right|>2$ is equivalent to the existence of $a \in S_{n}$ with $1n$ and black otherwise, and thus obtain a graph $G$. The degree of a point $X$ will be the number of red segments with an endpoint in $X$. We distinguish three cases: +(i) There is a point, say $A_{8}$, whose degree is at most 1 . We may suppose w.l.o.g. that $A_{8} A_{7}$ is red and $A_{8} A_{1}, \ldots, A_{8} A_{6}$ black. By a well-known fact, the segments joining vertices $A_{1}, A_{2}, \ldots, A_{6}$ determine either a red triangle, in which case there is nothing to prove, or a black triangle, say $A_{1} A_{2} A_{3}$. But in the latter case the four points $A_{1}, A_{2}, A_{3}, A_{8}$ do not determine any red segment, a contradiction to Lemma 2 . +(ii) All points have degree 2. Then the set of red segments partitions into cycles. If one of these cycles has length 3 , then the proof is complete. If all the cycles have length at least 4 , then we have two possibilities: two 4 -cycles, say $A_{1} A_{2} A_{3} A_{4}$ and $A_{5} A_{6} A_{7} A_{8}$, or one 8-cycle, $A_{1} A_{2} \ldots A_{8}$. In both cases, the four points $A_{1}, A_{3}, A_{5}, A_{7}$ do not determine any red segment, a contradiction. +(iii) There is a point of degree at least 3 , say $A_{1}$. Suppose that $A_{1} A_{2}$, $A_{1} A_{3}$, and $A_{1} A_{4}$ are red. We claim that $A_{2}, A_{3}, A_{4}$ determine at least one red segment, which will complete the solution. If not, by Lemma $1, \mu\left(A_{2} A_{3}\right), \mu\left(A_{3} A_{4}\right), \mu\left(A_{4} A_{2}\right)$ are $n, n, 0$ in some order. Assuming w.l.o.g. that $\mu\left(A_{2} A_{3}\right)=0$, denote by $S$ the area of triangle $A_{1} A_{2} A_{3}$. Now by formula (1), $2 S$ is not divisible by $p$. On the other hand, since $\mu\left(A_{1} A_{2}\right) \geq n+1$ and $\mu\left(A_{1} A_{3}\right) \geq n+1$, it follows from (2) that $2 S$ is divisible by $p$, a contradiction. + +## Notation and Abbreviations + +## A. 1 Notation + +We assume familiarity with standard elementary notation of set theory, algebra, logic, geometry (including vectors), analysis, number theory (including divisibility and congruences), and combinatorics. We use this notation liberally. +We assume familiarity with the basic elements of the game of chess (the movement of pieces and the coloring of the board). +The following is notation that deserves additional clarification. + +- $\mathcal{B}(A, B, C), A-B-C$ : indicates the relation of betweenness, i.e., that $B$ is between $A$ and $C$ (this automatically means that $A, B, C$ are different collinear points). +- $A=l_{1} \cap l_{2}$ : indicates that $A$ is the intersection point of the lines $l_{1}$ and $l_{2}$. +- $A B$ : line through $A$ and $B$, segment $A B$, length of segment $A B$ (depending on context). +- $[A B$ : ray starting in $A$ and containing $B$. +- ( $A B$ : ray starting in $A$ and containing $B$, but without the point $A$. +- $(A B)$ : open interval $A B$, set of points between $A$ and $B$. +- $[A B]$ : closed interval $A B$, segment $A B,(A B) \cup\{A, B\}$. +- $(A B]$ : semiopen interval $A B$, closed at $B$ and open at $A,(A B) \cup\{B\}$. The same bracket notation is applied to real numbers, e.g., $[a, b)=\{x \mid$ $a \leq x0\},\{x \mid x \in X, x<0\}$, $\{a x+b \mid x \in X\},\{a x+b y \mid x \in X, y \in Y\}$ (respectively) for $X, Y \subseteq \mathbb{R}$, $a, b \in \mathbb{R}$. +- $[x],\lfloor x\rfloor$ : the greatest integer smaller than or equal to $x$. +- $\lceil x\rceil$ : the smallest integer greater than or equal to $x$. + +The following is notation simultaneously used in different concepts (depending on context). + +- $|A B|,|x|,|S|$ : the distance between two points $A B$, the absolute value of the number $x$, the number of elements of the set $S$ (respectively). +- $(x, y),(m, n),(a, b)$ : (ordered) pair $x$ and $y$, the greatest common divisor of integers $m$ and $n$, the open interval between real numbers $a$ and $b$ (respectively). + + +## A. 2 Abbreviations + +We tried to avoid using nonstandard notations and abbreviations as much as possible. However, one nonstandard abbreviation stood out as particularly convenient: + +- w.l.o.g.: without loss of generality. + +Other abbreviations include: + +- RHS: right-hand side (of a given equation). +- LHS: left-hand side (of a given equation). +- QM, AM, GM, HM: the quadratic mean, the arithmetic mean, the geometric mean, the harmonic mean (respectively). +- gcd, lcm: greatest common divisor, least common multiple (respectively). +- i.e.: in other words. +- e.g.: for example. + + +## B + +## Codes of the Countries of Origin + +| ARG | Argentina | GRE | Greece | PHI | Philippines | +| :--- | :--- | :--- | :--- | :--- | :--- | +| ARM | Armenia | HKG | Hong Kong | POL | Poland | +| AUS | Australia | HUN | Hungary | POR | Portugal | +| AUT | Austria | ICE | Iceland | PRK | Korea, North | +| BEL | Belgium | INA | Indonesia | PUR | Puerto Rico | +| BLR | Belarus | IND | India | ROM | Romania | +| BRA | Brazil | IRE | Ireland | RUS | Russia | +| BUL | Bulgaria | IRN | Iran | SAF | South Africa | +| CAN | Canada | ISR | Israel | SIN | Singapore | +| CHN | China | ITA | Italy | SLO | Slovenia | +| COL | Colombia | JAP | Japan | SMN | Serbia and Montenegro | +| CUB | Cuba | KAZ | Kazakhstan | SPA | Spain | +| CYP | Cyprus | KOR | Korea, South | SWE | Sweden | +| CZE | Czech Republic | KUW | Kuwait | THA | Thailand | +| CZS | Czechoslovakia | LAT | Latvia | TUN | Tunisia | +| EST | Estonia | LIT | Lithuania | TUR | Turkey | +| FIN | Finland | LUX | Luxembourg | TWN | Taiwan | +| FRA | France | MCD | Macedonia | UKR | Ukraine | +| FRG | Germany, FR | MEX | Mexico | USA | United States | +| GBR | United Kingdom | MON | Mongolia | USS | Soviet Union | +| GDR | Germany, DR | MOR | Morocco | UZB | Uzbekistan | +| GEO | Georgia | NET | Netherlands | VIE | Vietnam | +| GER | Germany | NOR | Norway | YUG | Yugoslavia | +| | | NZL | New Zealand | | | + +## References + +1. M. Aassila, 300 Défis Mathématiques, Ellipses, 2001. +2. M. Aigner, G.M. Ziegler, Proofs from THE BOOK, Springer; 3rd edition, 2003. +3. G.L. Alexanderson, L.F. Klosinski, L.C. Larson, The William Lowell Putnam Mathematical Competition, Problems and Solutions: 1965-1984, The Mathematical Association of America, 1985. +4. T. Andreescu, D. Andrica, 360 Problems for Mathematical Contests, GIL Publishing House, Zalău, 2003. +5. T. Andreescu, D. Andrica, An Introduction to the Diophantine Equations, GIL Publishing House, 2002. +6. T. Andreescu, D. Andrica, Complex Numbers from A to ... Z, Birkhäuser, Boston, 2005. +7. T. Andreescu, B. Enescu, Mathematical Treasures, Birkhäuser, Boston, 2003. +8. T. Andreescu, Z. Feng, 102 Combinatorial Problems, Birkhäuser Boston, 2002. +9. T. Andreescu, Z. Feng, 103 Trigonometry Problems: From the Training of the USA IMO Team, Birkhäuser Boston, 2004. +10. T. Andreescu, Z. Feng, Mathematical Olympiads 1998-1999, Problems and Solutions from Around the World, The Mathematical Association of America, 2000. +11. T. Andreescu, Z. Feng, Mathematical Olympiads 1999-2000, Problems and Solutions from Around the World, The Mathematical Association of America, 2002. +12. T. Andreescu, Z. Feng, Mathematical Olympiads 2000-2001, Problems and Solutions from Around the World, The Mathematical Association of America, 2003. +13. T. Andreescu, R. Gelca, Mathematical Olympiad Challenges, Birkhäuser, Boston, 2000. +14. T. Andreescu, K. Kedlaya, Mathematical Contests 1996-1997, Olympiads Problems and Solutions from Around the World, American Mathematics Competitions, 1998. +15. T. Andreescu, K. Kedlaya, Mathematical Contests 1997-1998, Olympiads Problems and Solutions from Around the World, American Mathematics Competitions, 1999. +16. T. Andreescu, V. Cartoaje, G. Dospinescu, M. Lascu, Old and New Inequalities, GIL Publishing House, 2004. +17. M. Arsenović, V. Dragović, Functional Equations (in Serbian), Mathematical Society of Serbia, Beograd, 1999. +18. M. Ašić et al., International Mathematical Olympiads (in Serbian), Mathematical Society of Serbia, Beograd, 1986. +19. M. Ašić et al., 60 Problems for XIX IMO (in Serbian), Society of Mathematicians, Physicists, and Astronomers, Beograd, 1979. +20. A. Baker, A Concise Introduction to the Theory of Numbers, Cambridge University Press, Cambridge, 1984. +21. E.J. Barbeau, Polynomials, Springer, 2003. +22. E.J. Barbeau, M.S. Klamkin, W.O.J. Moser, Five Hundred Mathematical Challenges, The Mathematical Association of America, 1995. +23. E.J. Barbeau, Pell's Equation, Springer-Verlag, 2003. +24. M. Becheanu, International Mathematical Olympiads 1959-2000. Problems. Solutions. Results, Academic Distribution Center, Freeland, USA, 2001. +25. E.L. Berlekamp, J.H. Conway, R.K. Guy, Winning Ways for Your Mathematical Plays, AK Peters, Ltd., 2nd edition, 2001. +26. G. Berzsenyi, S.B. Maurer, The Contest Problem Book V, The Mathematical Association of America, 1997. +27. P.S. Bullen, D.S. Mitrinović , M. Vasić, Means and Their Inequalities, Springer, 1989. +28. H.S.M. Coxeter, Introduction to Geometry, John Willey \& Sons, New York, 1969 +29. H.S.M. Coxeter, S.L. Greitzer, Geometry Revisited, Random House, New York, 1967. +30. I. Cuculescu, International Mathematical Olympiads for Students (in Romanian), Editura Tehnica, Bucharest, 1984. +31. A. Engel, Problem Solving Strategies, Springer, 1999. +32. A.A. Fomin, G.M. Kuznetsova, International Mathematical Olympiads (in Russian), Drofa, Moskva, 1998. +33. A.M. Gleason, R.E. Greenwood, L.M. Kelly, The William Lowell Putnam Mathematical Competition, Problems and Solutions: 1938-1964, The Mathematical Association of America, 1980. +34. R.K. Guy, Unsolved Problems in Number Theory, Springer, 3rd edition, 2004. +35. S.L. Greitzer, International Mathematical Olympiads 1959-1977, M.A.A., Washington, D. C., 1978. +36. R.L. Graham, D.E. Knuth, O. Patashnik, Concrete Mathematics, 2nd Edition, Addison-Wesley, 1989. +37. L. Hahn, Complex Numbers $\mathcal{E}^{2}$ Geometry, New York, 1960. +38. G.H. Hardy, J.E. Littlewood, G. Pólya, Inequalities, Cambridge University Press; 2nd edition, 1998. +39. G.H. Hardy, E.M. Wright, An Introduction to the Theory of Numbers, Oxford University Press; 5th edition, 1980. +40. V. Janković, Z. Kadelburg, P. Mladenović, International and Balkan Mathematical Olympiads 1984-1995 (in Serbian), Mathematical Society of Serbia, Beograd, 1996. +41. V. Janković, V. Mićić, IX $\mathcal{E}^{-1}$ XIX International Mathematical Olympiads, Mathematical Society of Serbia, Beograd, 1997. +42. R.A. Johnson, Advanced Euclidean Geometry, Dover Publications, 1960. +43. Z. Kadelburg, D. Djukić, M. Lukić, I. Matić, Inequalities (in Serbian), Mathematical Society of Serbia, Beograd, 2003. +44. N.D. Kazarinoff, Geometric Inequalities, Mathematical Association of America (MAA), 1975. +45. K.S. Kedlaya, B. Poonen, R. Vakil, The William Lowell Putnam Mathematical Competition 1985-2000 Problems, Solutions and Commentary, The Mathematical Association of America, 2002. +46. M.S. Klamkin, International Mathematical Olympiads 1979-1985 and Forty Supplementary Problems, M.A.A., Washington, D.C., 1986. +47. M.S. Klamkin, International Mathematical Olympiads 1979-1986, M.A.A., Washington, D.C., 1988. +48. A. Kupetov, A. Rubanov, Problems in Geometry, MIR, Moscow, 1975. +49. H.-H. Langmann, 30th International Mathematical Olympiad, Braunschweig 1989, Bildung und Begabung e.V., Bonn 2, 1990. +50. L.C. Larson, Problem Solving Through Problems, Springer, 1983. +51. E. Lozansky, C. Rousseau, Winning Solutions, Springer-Verlag, New York, 1996. +52. V. Mićić, Z. Kadelburg, D. Djukić, Introduction to Number Theory (in Serbian), 4th edition, Mathematical Society of Serbia, Beograd, 2004. +53. D.S. Mitrinović, J. Pečarić, A.M Fink, Classical and New Inequalities in Analysis, Springer, 1992. +54. D.S. Mitrinović, J.E. Pečarić, V. Volenec, Recent Advances in Geometric Inequalities, Kluwer Academic Publishers, 1989. +55. P. Mladenović, Combinatorics, 3rd edition, Mathematical Society of Serbia, Beograd, 2001. +56. P.S. Modenov, Problems in Geometry, MIR, Moscow, 1981. +57. P.S. Modenov, A.S. Parhomenko, Geometric Transformations, Academic Press, New York, 1965. +58. L. Moisotte, 1850 exercices de mathémathique, Bordas, Paris, 1978. +59. L.J. Mordell, Diophantine Equations, Academic Press, London and New York, 1969. +60. E.A. Morozova, I.S. Petrakov, V.A. Skvortsov, International Mathematical Olympiads (in Russian), Prosveshchenie, Moscow, 1976. +61. I. Nagell, Introduction to Number Theoury, John Wiley \& Sons, Inc., New York, Stockholm, 1951. +62. I. Niven, H.S. Zuckerman, H.L. Montgomery An Introduction to the Theory of Numbers, John Wiley and Sons, Inc., 1991. +63. C.R. Pranesachar, Shailesh A. Shirali, B.J. Venkatachala, C.S. Yogananda, Mathematical Challenges from Olympiads, Interline Publishing Pvt. Ltd., Bangalore, 1995. +64. V.V. Prasolov, Problems of Plane Geometry, Volumes 1 and 2, Nauka, Moscow, 1986. +65. I. Reiman, J. Pataki, A. Stipsitz International Mathematical Olympiad: 19591999, Anthem Press, London, 2002. +66. I.F. Sharygin, Problems in Plane Geometry, Imported Pubn, 1988. +67. W. Sierpinski, Elementary Theory of Numbers, Polski Academic Nauk, Warsaw, 1964. +68. W. Sierpinski, 250 Problems in Elementary Number Theory, American Elsevier Publishing Company, Inc., New York, PWN, Warsaw, 1970. +69. R.P. Stanley, Enumerative Combinatorics, Volumes 1 and 2, Cambridge University Press; New Ed edition, 2001. +70. D. Stevanović, M. Milošević, V. Baltić, Discrete Mathematics: Problem Book in Elementary Combinatorics and Graph Theory (in Serbian), Mathematical Society of Serbia, Beograd, 2004. +71. I. Tomescu, R.A. Melter, Problems in Combinatorics and Graph Theory, John Wiley \& Sons, 1985. +72. I. Tomescu et al., Balkan Mathematical Olympiads 1984-1994 (in Romanian), Gil, Zalău, 1996. +73. J.H. van Lint, R.M. Wilson, A Course in Combinatorics, second edition, Cambridge University Press, 2001. +74. I.M. Vinogradov, Elements of Number Theory, Dover Publications, 2003. +75. H.S. Wilf, Generatingfunctionology, Academic Press, Inc.; 2nd edition, 1994. +76. A.M. Yaglom, I.M. Yaglom, Challenging Mathematical Problems with Elementary Solutions, Dover Publications, 1987. +77. I.M. Yaglom, Geometric Transformations, Vols. I, II, III, The Mathematical Association of America (MAA), 1962, 1968, 1973. +78. P. Zeitz, The Art and Craft of Problem Solving, Wiley; International Student edition, 1999. + +[^0]: ${ }^{1}$ The statement so formulated is false. It would be trivially true under the additional assumption that the polygonal line is closed. However, from the offered solution, which is not clear, it does not seem that the proposer had this in mind. + +[^1]: ${ }^{2}$ This problem is not elementary. The solution offered by the proposer, which is not quite clear and complete, only shows that if such a $\beta$ exists, then $\beta \geq \frac{1}{2(1-\alpha)}$. + +[^2]: ${ }^{3}$ The problem is unclear. Presumably $n, i, j$ and the $i$ th digit are fixed. + ${ }^{4}$ The problem is unclear. The correct formulation could be the following: + Given $k$ parallel lines $l_{1}, \ldots, l_{k}$ and $n_{i}$ points on the line $l_{i}, i=1,2, \ldots, k$, find the maximum possible number of triangles with vertices at these points. + +[^3]: ${ }^{5}$ The numbers in the problem are not necessarily in base 10. + +[^4]: ${ }^{6}$ The problem in this formulation is senseless. The correct formulation could be, "Find $\ldots$ such that $\sum_{m=1}^{\infty}\left(\left[\frac{n}{m}\right]-\left[\frac{n-1}{m}\right]\right)=1992 \ldots$." + +[^5]: ${ }^{7}$ The statement of the problem is obviously wrong, and the authors couldn't determine a suitable alteration of the formulation which would make the problem correct. We put it here only for completeness of the problem set. + +[^6]: ${ }^{8}$ This problem is false. However, it is true if "not outside $A B M$ " is replaced by "not outside $A B C D$ ". + diff --git a/IMO/raw/en-IMO2006SL.pdf b/IMO/raw/en-IMO2006SL.pdf new file mode 100644 index 0000000000000000000000000000000000000000..bcaaddd34d66823cf3c3c6d32da8eec332401a4d --- /dev/null +++ b/IMO/raw/en-IMO2006SL.pdf @@ -0,0 +1,3 @@ +version https://git-lfs.github.com/spec/v1 +oid sha256:df9f97f299b7ae8ad3cf0614a50804c3f4709c9c78215bd5bc1618e7a5e447fe +size 523498 diff --git a/IMO/raw/en-IMO2007SL.pdf b/IMO/raw/en-IMO2007SL.pdf new file mode 100644 index 0000000000000000000000000000000000000000..af008f83fe3f57eeb0173dbb72b62f593c26bb33 --- /dev/null +++ b/IMO/raw/en-IMO2007SL.pdf @@ -0,0 +1,3 @@ +version https://git-lfs.github.com/spec/v1 +oid sha256:a874eb812eacc3d2732fb9e8f803ab45f8fd2d3a05aec7e665ae0fee6adbaf79 +size 550183 diff --git a/IMO/raw/en-IMO2008SL.pdf b/IMO/raw/en-IMO2008SL.pdf new file mode 100644 index 0000000000000000000000000000000000000000..449cfa14023e6f996bb53e646671bcace864b48a --- /dev/null +++ b/IMO/raw/en-IMO2008SL.pdf @@ -0,0 +1,3 @@ +version https://git-lfs.github.com/spec/v1 +oid sha256:ef610b4fead4865d6cadbcea7ff7586553d1e1c4ba074b0c9f67a5694820b4fc +size 525713 diff --git a/IMO/raw/en-IMO2009SL.pdf b/IMO/raw/en-IMO2009SL.pdf new file mode 100644 index 0000000000000000000000000000000000000000..98a0fe669eda19c0456fd936f9f992a6c542276b --- /dev/null +++ b/IMO/raw/en-IMO2009SL.pdf @@ -0,0 +1,3 @@ +version https://git-lfs.github.com/spec/v1 +oid sha256:bfdee720406220b62aa274de2fa3149bc08afe1dd6affc6f91cf29fb2b44d726 +size 1954803 diff --git a/IMO/raw/en-IMO2010SL.pdf b/IMO/raw/en-IMO2010SL.pdf new file mode 100644 index 0000000000000000000000000000000000000000..6647abf73cf5951ec563a7d4f80b4c5519f6c69b --- /dev/null +++ b/IMO/raw/en-IMO2010SL.pdf @@ -0,0 +1,3 @@ +version https://git-lfs.github.com/spec/v1 +oid sha256:8b22e58d9559699baa1adcd2d2ddbadb3b5f3a4fc8ef75d5d0e2d4864597783d +size 1683818 diff --git a/IMO/raw/en-IMO2011SL.pdf b/IMO/raw/en-IMO2011SL.pdf new file mode 100644 index 0000000000000000000000000000000000000000..9cc97136ccb3363428b429318d42f713da5253eb --- /dev/null +++ b/IMO/raw/en-IMO2011SL.pdf @@ -0,0 +1,3 @@ +version https://git-lfs.github.com/spec/v1 +oid sha256:1f7fae0cc74dba95dc556b5f78a5b429a8ecd70acbf7a5ffeb3018ea92331ab8 +size 713524 diff --git a/IMO/raw/en-IMO2012SL.pdf b/IMO/raw/en-IMO2012SL.pdf new file mode 100644 index 0000000000000000000000000000000000000000..fc78824823d67699d0cc6c1b5d8a9e6b406b50c7 --- /dev/null +++ b/IMO/raw/en-IMO2012SL.pdf @@ -0,0 +1,3 @@ +version https://git-lfs.github.com/spec/v1 +oid sha256:22b018a851a4b17a628563964a02649db15052d523d5371e0dbe42b537ef119d +size 411332 diff --git a/IMO/raw/en-IMO2013SL.pdf b/IMO/raw/en-IMO2013SL.pdf new file mode 100644 index 0000000000000000000000000000000000000000..c4cf030707c7a3d9c11f08c9a63d0900c79f2a2c --- /dev/null +++ b/IMO/raw/en-IMO2013SL.pdf @@ -0,0 +1,3 @@ +version https://git-lfs.github.com/spec/v1 +oid sha256:4ea75d998c6d67962c59011b8f8c81da253b0090dfef2d003e80cf2fc0e5468b +size 538224 diff --git a/IMO/raw/en-IMO2014SL.pdf b/IMO/raw/en-IMO2014SL.pdf new file mode 100644 index 0000000000000000000000000000000000000000..50affb18716c4dd4f173319ef757e2b69ae6ae2c --- /dev/null +++ b/IMO/raw/en-IMO2014SL.pdf @@ -0,0 +1,3 @@ +version https://git-lfs.github.com/spec/v1 +oid sha256:74c54060408e2598482e6db44afa1a3c5226a74383113f9add5a053667986495 +size 1196532 diff --git a/IMO/raw/en-IMO2015SL.pdf b/IMO/raw/en-IMO2015SL.pdf new file mode 100644 index 0000000000000000000000000000000000000000..e177814162b4acc9c6b916b43d61d0e737e9ca19 --- /dev/null +++ b/IMO/raw/en-IMO2015SL.pdf @@ -0,0 +1,3 @@ +version https://git-lfs.github.com/spec/v1 +oid sha256:bf6cdda0ff82cee99b2eed34d213f699aa9ef5c22be33b0b0a51eb8bd9e18716 +size 1628652 diff --git a/IMO/raw/en-IMO2016SL.pdf b/IMO/raw/en-IMO2016SL.pdf new file mode 100644 index 0000000000000000000000000000000000000000..1b665f0594703a2daabfec0903d3a614907aac35 --- /dev/null +++ b/IMO/raw/en-IMO2016SL.pdf @@ -0,0 +1,3 @@ +version https://git-lfs.github.com/spec/v1 +oid sha256:881095ca836abdd044373fbce409e0ef2696de7d815bbc32783397cad2c7874f +size 1012241 diff --git a/IMO/raw/en-IMO2017SL.pdf b/IMO/raw/en-IMO2017SL.pdf new file mode 100644 index 0000000000000000000000000000000000000000..19979092111eb787000048a958bf77b59913f955 --- /dev/null +++ b/IMO/raw/en-IMO2017SL.pdf @@ -0,0 +1,3 @@ +version https://git-lfs.github.com/spec/v1 +oid sha256:1ba525cc74014f0b74d5bd018374dc96dbce467eaaa67fe4a79cf005043e781d +size 1605265 diff --git a/IMO/raw/en-IMO2018SL.pdf b/IMO/raw/en-IMO2018SL.pdf new file mode 100644 index 0000000000000000000000000000000000000000..682c2dadef1458c3130f0caf5faeb36d7408756c --- /dev/null +++ b/IMO/raw/en-IMO2018SL.pdf @@ -0,0 +1,3 @@ +version https://git-lfs.github.com/spec/v1 +oid sha256:92b6a2fde1ba8cc85ec5d6bc5c0adbd4d2b576653a4c9c257df85a2c31599332 +size 981255 diff --git a/IMO/raw/en-IMO2019SL.pdf b/IMO/raw/en-IMO2019SL.pdf new file mode 100644 index 0000000000000000000000000000000000000000..2e7416da20e145d4a22dd1cab1274d01b741b388 --- /dev/null +++ b/IMO/raw/en-IMO2019SL.pdf @@ -0,0 +1,3 @@ +version https://git-lfs.github.com/spec/v1 +oid sha256:f112cbaa7abf1a4139dbba9e29e8fa1e4d73258708ba558388ae144db222e725 +size 3107272 diff --git a/IMO/raw/en-IMO2020SL.pdf b/IMO/raw/en-IMO2020SL.pdf new file mode 100644 index 0000000000000000000000000000000000000000..9e21e138ae83d5e7a86908676d63856087594607 --- /dev/null +++ b/IMO/raw/en-IMO2020SL.pdf @@ -0,0 +1,3 @@ +version https://git-lfs.github.com/spec/v1 +oid sha256:a42248a040d7d011b1c1ad58351207350af9dd8bd8f2ac347ba4e216323081f4 +size 2406391 diff --git a/IMO/raw/en-IMO2021SL.pdf b/IMO/raw/en-IMO2021SL.pdf new file mode 100644 index 0000000000000000000000000000000000000000..9f53734ee4de7eede76666aa2e43c416aa56e1fe --- /dev/null +++ b/IMO/raw/en-IMO2021SL.pdf @@ -0,0 +1,3 @@ +version https://git-lfs.github.com/spec/v1 +oid sha256:c02d9f0d68fd38fb3b3136b19004d09fb8ee421a0f4da0c325e28b42a086ff45 +size 2052151 diff --git a/IMO/raw/en-IMO2022SL.pdf b/IMO/raw/en-IMO2022SL.pdf new file mode 100644 index 0000000000000000000000000000000000000000..2936a29f080fe97365476f77361013cc1072f03f --- /dev/null +++ b/IMO/raw/en-IMO2022SL.pdf @@ -0,0 +1,3 @@ +version https://git-lfs.github.com/spec/v1 +oid sha256:2ed31c9e5efecfcc796940719da86ec47edca41b7f7df75811d2269453dc86f5 +size 4760328 diff --git a/IMO/raw/en-compendium.pdf b/IMO/raw/en-compendium.pdf new file mode 100644 index 0000000000000000000000000000000000000000..bf901ebcb6d0ca494cdcffac912b78dcacabb66e --- /dev/null +++ b/IMO/raw/en-compendium.pdf @@ -0,0 +1,3 @@ +version https://git-lfs.github.com/spec/v1 +oid sha256:c69aa00c85ffc60d815081d1eb6ca65b8e27ed26700c0c8741a04e40755d42d1 +size 5946928 diff --git a/IMO/segment_script/fixtures/test-case-1.md b/IMO/segment_script/fixtures/test-case-1.md new file mode 100644 index 0000000000000000000000000000000000000000..20ae4063059c2053c3d181a8fcd356d389dbc80a --- /dev/null +++ b/IMO/segment_script/fixtures/test-case-1.md @@ -0,0 +1,17 @@ +## Algebra + +A1. Find the roots of the quadratic equation \(x^2 - 4x + 4 = 0\). + +Solution. The equation simplifies to \((x - 2)^2 = 0\), so the root is \(x = 2\). + +A2. Determine the value of \(y\) in the equation \(2y + 3 = 7\). + +Solution 1. Subtracting 3 from both sides gives \(2y = 4\). + +Solution 2. Dividing both sides by 2 gives \(y = 2\). + +## Combinatorics + +C1. How many ways can 3 objects be arranged in a line? + +Solution. The number of permutations of 3 objects is \(3! = 6\). \ No newline at end of file diff --git a/IMO/segment_script/fixtures/test-case-2.md b/IMO/segment_script/fixtures/test-case-2.md new file mode 100644 index 0000000000000000000000000000000000000000..9a0fe32c56919d960b55142aee97af8127ff5f23 --- /dev/null +++ b/IMO/segment_script/fixtures/test-case-2.md @@ -0,0 +1,35 @@ +## Algebra + +## A1 + +Find the roots of the quadratic equation \(x^2 - 4x + 4 = 0\). + +## A2 + +Determine the value of \(y\) in the equation \(2y + 3 = 7\). + +## Combinatorics + +## C1 + +How many ways can 3 objects be arranged in a line? + +## A1 + +Find the roots of the quadratic equation \(x^2 - 4x + 4 = 0\). + +Solution. The equation simplifies to \((x - 2)^2 = 0\), so the root is \(x = 2\). + +## A2 + +Determine the value of \(y\) in the equation \(2y + 3 = 7\). + +Solution 1. Subtracting 3 from both sides gives \(2y = 4\). + +Solution 2. Dividing both sides by 2 gives \(y = 2\). + +## C1 + +How many ways can 3 objects be arranged in a line? + +Solution. The number of permutations of 3 objects is \(3! = 6\). \ No newline at end of file diff --git a/IMO/segment_script/fixtures/test-case-3.md b/IMO/segment_script/fixtures/test-case-3.md new file mode 100644 index 0000000000000000000000000000000000000000..56cdd8aa799b6e3c8826cae0869d59e9f2d3d414 --- /dev/null +++ b/IMO/segment_script/fixtures/test-case-3.md @@ -0,0 +1,35 @@ +## Algebra + +## A1 + +Find the roots of the quadratic equation \(x^2 - 4x + 4 = 0\). + +## A2 + +Determine the value of \(y\) in the equation \(2y + 3 = 7\). + +## Combinatorics + +## C1 + +How many ways can 3 objects be arranged in a line? + +## A 1 + +Find the roots of the quadratic equation \(x^2 - 4x + 4 = 0\). + +Solution. The equation simplifies to \((x - 2)^2 = 0\), so the root is \(x = 2\). + +## A2 + +Determine the value of \(y\) in the equation \(2y + 3 = 7\). + +Solution 1. Subtracting 3 from both sides gives \(2y = 4\). + +Solution 2. Dividing both sides by 2 gives \(y = 2\). + +## C1 + +How many ways can 3 objects be arranged in a line? + +Solution. The number of permutations of 3 objects is \(3! = 6\). \ No newline at end of file diff --git a/IMO/segment_script/fixtures/test-case-4.md b/IMO/segment_script/fixtures/test-case-4.md new file mode 100644 index 0000000000000000000000000000000000000000..27eac1ef262c664e4098a5b25f3282fca5fb726c --- /dev/null +++ b/IMO/segment_script/fixtures/test-case-4.md @@ -0,0 +1,44 @@ +# $5^{\text {nd }}$ International Mathematical Olympiad + +12 - 24 July 2011 Amsterdam The Netherlands + +![](https://cdn.mathpix.com/cropped/2024_04_17_481ddb3576adc2313a2eg-01.jpg?height=2070&width=1902&top_left_y=744&top_left_x=-3) + +## 52nd International
Mathematical Olympiad
12-24 July 2011
Amsterdam
The Netherlands + +Problem shortlist with solutions + +## IMO regulation:
these shortlist problems have to be kept strictly confidential until IMO 2012. + +The problem selection committee + +Bart de Smit (chairman), Ilya Bogdanov, Johan Bosman, + +Andries Brouwer, Gabriele Dalla Torre, Géza Kós, + +Hendrik Lenstra, Charles Leytem, Ronald van Luijk, + +Christian Reiher, Eckard Specht, Hans Sterk, Lenny Taelman + +The committee gratefully acknowledges the receipt of 142 problem proposals by the following 46 countries: + +Armenia, Australia, Austria, Belarus, Belgium, Bosnia and Herzegovina, Brazil, Bulgaria, Canada, Colombia, Cyprus, Denmark, Estonia, Finland, France, Germany, Greece, Hong Kong, Hungary, India, Islamic Republic of Iran, Ireland, Israel, Japan, Kazakhstan, Republic of Korea, Luxembourg, Malaysia, Mexico, Mongolia, Montenegro, Pakistan, Poland, Romania, Russian Federation, Saudi Arabia, Serbia, Slovakia, Slovenia, Sweden, Taiwan, Thailand, Turkey, Ukraine, United Kingdom, United States of America + + +## Algebra + +A1. Find the roots of the quadratic equation \(x^2 - 4x + 4 = 0\). + +Solution. The equation simplifies to \((x - 2)^2 = 0\), so the root is \(x = 2\). + +A2. Determine the value of \(y\) in the equation \(2y + 3 = 7\). + +Solution 1. Subtracting 3 from both sides gives \(2y = 4\). + +Solution 2. Dividing both sides by 2 gives \(y = 2\). + +## Combinatorics + +C1. How many ways can 3 objects be arranged in a line? + +Solution. The number of permutations of 3 objects is \(3! = 6\). \ No newline at end of file diff --git a/IMO/segment_script/fixtures/test-case-5.md b/IMO/segment_script/fixtures/test-case-5.md new file mode 100644 index 0000000000000000000000000000000000000000..48d61ac493c9e863702d638178e5798f6cf3d22e --- /dev/null +++ b/IMO/segment_script/fixtures/test-case-5.md @@ -0,0 +1,37 @@ +## Algebra + +## A1 + +Find the roots of the quadratic equation \(x^2 - 4x + 4 = 0\). + +## A2 + +Determine the value of \(y\) in the equation \(2y + 3 = 7\). + +## Combinatorics + +## C1 + +How many ways can 3 objects be arranged in a line? + +## A1 + +Find the roots of the quadratic equation \(x^2 - 4x + 4 = 0\). + +Solution. The equation simplifies to \((x - 2)^2 = 0\), so the root is: + +## $\(x = 2\)$ + +## A2 + +Determine the value of \(y\) in the equation \(2y + 3 = 7\). + +Solution 1. Subtracting 3 from both sides gives \(2y = 4\). + +Solution 2. Dividing both sides by 2 gives \(y = 2\). + +## C1 + +How many ways can 3 objects be arranged in a line? + +Solution. The number of permutations of 3 objects is \(3! = 6\). \ No newline at end of file diff --git a/IMO/segment_script/fixtures/test-out-1234.jsonl b/IMO/segment_script/fixtures/test-out-1234.jsonl new file mode 100644 index 0000000000000000000000000000000000000000..11de5e2a7aa5cf5906da3653d95622f3944f5168 --- /dev/null +++ b/IMO/segment_script/fixtures/test-out-1234.jsonl @@ -0,0 +1,4 @@ +{"problem_type":"Algebra","problem_label":"A1","problem":"Find the roots of the quadratic equation \\(x^2 - 4x + 4 = 0\\).","solution":"The equation simplifies to \\((x - 2)^2 = 0\\), so the root is \\(x = 2\\).","tier":0} +{"problem_type":"Algebra","problem_label":"A2","problem":"Determine the value of \\(y\\) in the equation \\(2y + 3 = 7\\).","solution":"Subtracting 3 from both sides gives \\(2y = 4\\).","tier":0} +{"problem_type":"Algebra","problem_label":"A2","problem":"Determine the value of \\(y\\) in the equation \\(2y + 3 = 7\\).","solution":"Dividing both sides by 2 gives \\(y = 2\\).","tier":0} +{"problem_type":"Combinatorics","problem_label":"C1","problem":"How many ways can 3 objects be arranged in a line?","solution":"The number of permutations of 3 objects is \\(3! = 6\\).","tier":0} diff --git a/IMO/segment_script/fixtures/test-out-5.jsonl b/IMO/segment_script/fixtures/test-out-5.jsonl new file mode 100644 index 0000000000000000000000000000000000000000..64e75c51e7810899ea3e836869a4f630512674da --- /dev/null +++ b/IMO/segment_script/fixtures/test-out-5.jsonl @@ -0,0 +1,4 @@ +{"problem_type":"Algebra","problem_label":"A1","problem":"Find the roots of the quadratic equation \\(x^2 - 4x + 4 = 0\\).","solution":"The equation simplifies to \\((x - 2)^2 = 0\\), so the root is: ## $\\(x = 2\\)$","tier":0} +{"problem_type":"Algebra","problem_label":"A2","problem":"Determine the value of \\(y\\) in the equation \\(2y + 3 = 7\\).","solution":"Subtracting 3 from both sides gives \\(2y = 4\\).","tier":0} +{"problem_type":"Algebra","problem_label":"A2","problem":"Determine the value of \\(y\\) in the equation \\(2y + 3 = 7\\).","solution":"Dividing both sides by 2 gives \\(y = 2\\).","tier":0} +{"problem_type":"Combinatorics","problem_label":"C1","problem":"How many ways can 3 objects be arranged in a line?","solution":"The number of permutations of 3 objects is \\(3! = 6\\).","tier":0} diff --git a/IMO/segment_script/segment.py b/IMO/segment_script/segment.py new file mode 100644 index 0000000000000000000000000000000000000000..031cc5b36253cef28ad7625766c375f4aa3830bf --- /dev/null +++ b/IMO/segment_script/segment.py @@ -0,0 +1,96 @@ +# ----------------------------------------------------------------------------- +# Author: Marina +# Date: 2024-11-15 +# ----------------------------------------------------------------------------- +''' Script to segment IMO shortlist md files using regex. It takes as input +files in en-shortlist and outputs en-shortlist-seg +To run: +`python segment_script/segment.py` +To debug (or see covered use cases listed in fixtures/): +`pytest test_segment` +''' + +from collections import defaultdict +import os +import re +import pandas as pd +import json + + +base = 'md' +seg_base = 'segmented' + +section_re = re.compile(r'##\s+([A-Za-z]\w.*)') +problem_re = re.compile( + r'^(?:##\s*)?((?:[AGNC]\s*\d+))\.*\s*(.*?)(?:\((.*?)\))?$', + re.MULTILINE +) +solution_re = re.compile( + r'^(?:##\s*)?(Solution(?: \d+)?\.)\s*(.*?)(?=(?:Solution|Comment|A\d+|G\d+|N\d+|C\d+|##|$))', + re.MULTILINE | re.DOTALL +) + +def add_content(section, label, text_class, text, problems, solutions): + text_str = " ".join(text).strip() + if text_class == "problem": + # print(f"ADD PROBLEM {section} {label} ") + problems.append({"section": section, "label": label, "problem": text_str}) + elif text_class == "solution": + # print(f"ADD SOLUTION {section} {label}") + solutions.append({"label": label, "solution": text_str}) + +def parse(file): + with open(file, 'r') as file: + content = file.read() + problems, solutions = [], [] + current_section, current_label, current_class = None, None, None + current_lines = [] + for line in content.splitlines(): + if match := problem_re.match(line): + label, text, country = match.groups() + label = label.replace(" ", "") # clean the label + add_content(current_section, current_label, current_class, current_lines, problems, solutions) + current_class = "problem" + current_label = label + current_lines = [text] + elif match := solution_re.match(line): + label, text = match.groups() + add_content(current_section, current_label, current_class, current_lines, problems, solutions) + current_class = "solution" + current_lines = [text] + elif match := section_re.match(line): + add_content(current_section, current_label, current_class, current_lines, problems, solutions) + current_class = "section" + text, = match.groups() + current_section = text + else: + current_lines.append(line) + add_content(current_section, current_label, current_class, current_lines, problems, solutions) + problems_df = pd.DataFrame(problems).drop_duplicates(subset=["label", "problem"]) + solutions_df = pd.DataFrame(solutions) + return problems_df, solutions_df + +def join(problems_df, solutions_df): + pairs_df = problems_df.merge(solutions_df, on=["label"], how="left") + return pairs_df + +def add_metadata(pairs_df): + pairs_df.rename(columns={"section": "problem_type", "label": "problem_label"}, inplace=True) + pairs_df['tier'] = 0 # according to omnimath + return pairs_df + +def write_pairs(filename, pairs_df): + pairs_df.to_json(filename, orient="records", lines=True) + + +os.makedirs(seg_base, exist_ok=True) +for name in os.listdir(base): + if "compendium" not in name: # en-compendium is segmented in segment_compendium.py + print(name) + problems, solutions = parse(os.path.join(base, name)) + pairs_df = join(problems, solutions) + pairs_df = add_metadata(pairs_df) + print(pairs_df) + basename = os.path.splitext(name)[0] + print(f"{seg_base}/{basename}.jsonl") + write_pairs(f"{seg_base}/{basename}.jsonl", pairs_df) \ No newline at end of file diff --git a/IMO/segment_script/segment_compendium.py b/IMO/segment_script/segment_compendium.py new file mode 100644 index 0000000000000000000000000000000000000000..55ac9e9bad8bc994a8af8d68d09a02896adb2840 --- /dev/null +++ b/IMO/segment_script/segment_compendium.py @@ -0,0 +1,155 @@ +# ----------------------------------------------------------------------------- +# Author: Marina +# Date: 2024-11-15 +# ----------------------------------------------------------------------------- +''' Script to segment IMO shortlist md files using regex. It takes as input +the file en-compendium.md in en-shortlist and outputs the segmentation +(problem/solution pairs) in en-shortlist-seg +To run: +`python segment_compendium.py` +To debug (or see covered use cases by regex): +`pytest test_segment_compendium` +''' + +import os +import re +import pandas as pd + + +base = 'en-shortlist' +seg_base = 'en-shortlist-seg' +basename = 'en-compendium' + + +level1_re = re.compile(r"^##\s+(Problems|Solutions|Notation and Abbreviations)$") +year_re = re.compile(r"^[^=]*,\s+(\d{4})\s*$") +problem_section_re = re.compile(r"^###\s+(\d+\.\d+\.\d+)\s+(.+)$") +solution_section_re = re.compile(r"^###\s+(\d+\.\d+)\s+([\w\s]+)\s+(\d{4})$") +problem_or_solution_re = re.compile(r"^(?:\[.*?\])?\s*(\d+)\s*\.\s*(.+)$") + + +def add_content(current_dict): + required_keys = ["year", "category", "section_label", "label", "lines"] + if not all(current_dict[key] for key in required_keys): + return + text_str = " ".join(current_dict["lines"]).strip() + entry = { + "year": current_dict["year"], + "category": current_dict["category"], + "section": current_dict["section_label"], + "label": current_dict["label"], + } + if current_dict["class"] == "problem": + entry["problem"] = text_str + current_dict["problems"].append(entry) + elif current_dict["class"] == "solution": + entry["solution"] = text_str + current_dict["solutions"].append(entry) + + +def get_category(s:str): + cat = None + if 'contest' in s.lower(): + cat = 'contest' + elif 'shortlisted' in s.lower(): + cat = 'shortlisted' + elif 'longlisted' in s.lower(): + cat = 'longlisted' + return cat + + +def get_matching_section_label(s:str): + """ + extracts the section number to be used a a join key to pair a problem and solution + for problems: 3.44.1 -> 44 + for solutions: 4.20 -> 20 + """ + return s.split('.')[1] + + +def parse(file): + with open(file, 'r') as file: + content = file.read() + # problems, solutions = [], [] + current = { + "year": None, + "category": None, + "section_label": None, + "label": None, + "class": None, + "lines": [], + "problems": [], + "solutions": [] + } + for line in content.splitlines(): + if match := level1_re.match(line): + add_content(current) + title, = match.groups() + current["class"] = { + "Problems": "problem", + "Solutions": "solution", + }.get(title, "other") + current["lines"] = [] + elif match := year_re.match(line): + add_content(current) + current["year"] = match.group(1) + current["lines"] = [] + elif match := problem_section_re.match(line): + add_content(current) + number, title = match.groups() + current["section_label"] = get_matching_section_label(number) + current["category"] = get_category(title) + current["lines"] = [] + elif match := solution_section_re.match(line): + add_content(current) + number, title, year = match.groups() + current["section_label"] = get_matching_section_label(number) + current["category"] = get_category(title) + current["year"] = year + current["lines"] = [] + elif match := problem_or_solution_re.match(line): + add_content(current) + current["label"] = match.group(1) + current["lines"] = [line] + else: + if current["lines"]: + current["lines"].append(line) + problems_df = pd.DataFrame(current["problems"]) + solutions_df = pd.DataFrame(current["solutions"]) + return problems_df, solutions_df + +def join(problems_df, solutions_df): + pairs_df = problems_df.merge(solutions_df, on=["year", "category", "section", "label"], how="outer") + return pairs_df + +def add_metadata(pairs_df): + problem_type_mapping = { + "A": "Algebra", + "C": "Combinatorics", + "G": "Geometry", + "N": "Number Theory" + } + pairs_df['problem_type'] = pairs_df['problem'].str.extract(r'^\d+\.\s*([ACGN])\d*')[0] + pairs_df['problem_type'] = pairs_df['problem_type'] .map(problem_type_mapping) + pairs_df['tier'] = 0 # according to omnimath + pairs_df.rename(columns={"category": "problem_phase"}, inplace=True) + pairs_df = pairs_df.drop(columns=['section', 'label']) + return pairs_df + +def write_pairs(filename, pairs_df): + pairs_df.to_json(filename, orient="records", lines=True) + + +problems, solutions = parse(f"{base}/{basename}.md") +pairs_df = join(problems, solutions) +pairs_df = pairs_df[pairs_df.notnull().all(axis=1)] +pairs_df = add_metadata(pairs_df) +write_pairs(f"{seg_base}/{basename}.jsonl", pairs_df) + +# problems contains duplicate problems (since problem in Shortlist appears in Contest, and problem in Longlist appeasr in Shortlist) +# >>>print(len(problems)) +# 2460 +# >>>print(len(solutions)) +# 961 +# print(len(pairs_df)) +# 960 diff --git a/IMO/segmented/en-IMO2006SL.jsonl b/IMO/segmented/en-IMO2006SL.jsonl new file mode 100644 index 0000000000000000000000000000000000000000..4385aecdac4b54f8a6c55b14638218bc59dc92c1 --- /dev/null +++ b/IMO/segmented/en-IMO2006SL.jsonl @@ -0,0 +1,40 @@ +{"problem_type":"Algebra","problem_label":"A1","problem":"A sequence of real numbers $a_{0}, a_{1}, a_{2}, \\ldots$ is defined by the formula $$ a_{i+1}=\\left\\lfloor a_{i}\\right\\rfloor \\cdot\\left\\langle a_{i}\\right\\rangle \\quad \\text { for } \\quad i \\geq 0 \\text {; } $$ here $a_{0}$ is an arbitrary real number, $\\left\\lfloor a_{i}\\right\\rfloor$ denotes the greatest integer not exceeding $a_{i}$, and $\\left\\langle a_{i}\\right\\rangle=a_{i}-\\left\\lfloor a_{i}\\right\\rfloor$. Prove that $a_{i}=a_{i+2}$ for $i$ sufficiently large. (Estonia)","solution":"First note that if $a_{0} \\geq 0$, then all $a_{i} \\geq 0$. For $a_{i} \\geq 1$ we have (in view of $\\left\\langle a_{i}\\right\\rangle<1$ and $\\left\\lfloor a_{i}\\right\\rfloor>0$ ) $$ \\left\\lfloor a_{i+1}\\right\\rfloor \\leq a_{i+1}=\\left\\lfloor a_{i}\\right\\rfloor \\cdot\\left\\langle a_{i}\\right\\rangle<\\left\\lfloor a_{i}\\right\\rfloor $$ the sequence $\\left\\lfloor a_{i}\\right\\rfloor$ is strictly decreasing as long as its terms are in $[1, \\infty)$. Eventually there appears a number from the interval $[0,1)$ and all subsequent terms are 0 . Now pass to the more interesting situation where $a_{0}<0$; then all $a_{i} \\leq 0$. Suppose the sequence never hits 0 . Then we have $\\left\\lfloor a_{i}\\right\\rfloor \\leq-1$ for all $i$, and so $$ 1+\\left\\lfloor a_{i+1}\\right\\rfloor>a_{i+1}=\\left\\lfloor a_{i}\\right\\rfloor \\cdot\\left\\langle a_{i}\\right\\rangle>\\left\\lfloor a_{i}\\right\\rfloor $$ this means that the sequence $\\left\\lfloor a_{i}\\right\\rfloor$ is nondecreasing. And since all its terms are integers from $(-\\infty,-1]$, this sequence must be constant from some term on: $$ \\left\\lfloor a_{i}\\right\\rfloor=c \\quad \\text { for } \\quad i \\geq i_{0} ; \\quad c \\text { a negative integer. } $$ The defining formula becomes $$ a_{i+1}=c \\cdot\\left\\langle a_{i}\\right\\rangle=c\\left(a_{i}-c\\right)=c a_{i}-c^{2} . $$ Consider the sequence $$ b_{i}=a_{i}-\\frac{c^{2}}{c-1} $$ It satisfies the recursion rule $$ b_{i+1}=a_{i+1}-\\frac{c^{2}}{c-1}=c a_{i}-c^{2}-\\frac{c^{2}}{c-1}=c b_{i} $$ implying $$ b_{i}=c^{i-i_{0}} b_{i_{0}} \\quad \\text { for } \\quad i \\geq i_{0} . $$ Since all the numbers $a_{i}$ (for $i \\geq i_{0}$ ) lie in $\\left[c, c+1\\right.$ ), the sequence $\\left(b_{i}\\right)$ is bounded. The equation (2) can be satisfied only if either $b_{i_{0}}=0$ or $|c|=1$, i.e., $c=-1$. In the first case, $b_{i}=0$ for all $i \\geq i_{0}$, so that $$ a_{i}=\\frac{c^{2}}{c-1} \\quad \\text { for } \\quad i \\geq i_{0} $$ In the second case, $c=-1$, equations (1) and (2) say that $$ a_{i}=-\\frac{1}{2}+(-1)^{i-i_{0}} b_{i_{0}}= \\begin{cases}a_{i_{0}} & \\text { for } i=i_{0}, i_{0}+2, i_{0}+4, \\ldots \\\\ 1-a_{i_{0}} & \\text { for } i=i_{0}+1, i_{0}+3, i_{0}+5, \\ldots\\end{cases} $$ Summarising, we see that (from some point on) the sequence $\\left(a_{i}\\right)$ either is constant or takes alternately two values from the interval $(-1,0)$. The result follows. Comment. There is nothing mysterious in introducing the sequence $\\left(b_{i}\\right)$. The sequence $\\left(a_{i}\\right)$ arises by iterating the function $x \\mapsto c x-c^{2}$ whose unique fixed point is $c^{2} \/(c-1)$.","tier":0} +{"problem_type":"Algebra","problem_label":"A2","problem":"The sequence of real numbers $a_{0}, a_{1}, a_{2}, \\ldots$ is defined recursively by $$ a_{0}=-1, \\quad \\sum_{k=0}^{n} \\frac{a_{n-k}}{k+1}=0 \\quad \\text { for } \\quad n \\geq 1 $$ Show that $a_{n}>0$ for $n \\geq 1$. (Poland)","solution":"The proof goes by induction. For $n=1$ the formula yields $a_{1}=1 \/ 2$. Take $n \\geq 1$, assume $a_{1}, \\ldots, a_{n}>0$ and write the recurrence formula for $n$ and $n+1$, respectively as $$ \\sum_{k=0}^{n} \\frac{a_{k}}{n-k+1}=0 \\quad \\text { and } \\quad \\sum_{k=0}^{n+1} \\frac{a_{k}}{n-k+2}=0 $$ Subtraction yields $$ \\begin{aligned} 0=(n+2) \\sum_{k=0}^{n+1} & \\frac{a_{k}}{n-k+2}-(n+1) \\sum_{k=0}^{n} \\frac{a_{k}}{n-k+1} \\\\ = & (n+2) a_{n+1}+\\sum_{k=0}^{n}\\left(\\frac{n+2}{n-k+2}-\\frac{n+1}{n-k+1}\\right) a_{k} . \\end{aligned} $$ The coefficient of $a_{0}$ vanishes, so $$ a_{n+1}=\\frac{1}{n+2} \\sum_{k=1}^{n}\\left(\\frac{n+1}{n-k+1}-\\frac{n+2}{n-k+2}\\right) a_{k}=\\frac{1}{n+2} \\sum_{k=1}^{n} \\frac{k}{(n-k+1)(n-k+2)} a_{k} . $$ The coefficients of $a_{1}, \\ldots, a_{n}$ are all positive. Therefore, $a_{1}, \\ldots, a_{n}>0$ implies $a_{n+1}>0$. Comment. Students familiar with the technique of generating functions will immediately recognise $\\sum a_{n} x^{n}$ as the power series expansion of $x \/ \\ln (1-x)$ (with value -1 at 0 ). But this can be a trap; attempts along these lines lead to unpleasant differential equations and integrals hard to handle. Using only tools from real analysis (e.g. computing the coefficients from the derivatives) seems very difficult. On the other hand, the coefficients can be approached applying complex contour integrals and some other techniques from complex analysis and an attractive formula can be obtained for the coefficients: $$ a_{n}=\\int_{1}^{\\infty} \\frac{\\mathrm{d} x}{x^{n}\\left(\\pi^{2}+\\log ^{2}(x-1)\\right)} \\quad(n \\geq 1) $$ which is evidently positive.","tier":0} +{"problem_type":"Algebra","problem_label":"A3","problem":"The sequence $c_{0}, c_{1}, \\ldots, c_{n}, \\ldots$ is defined by $c_{0}=1, c_{1}=0$ and $c_{n+2}=c_{n+1}+c_{n}$ for $n \\geq 0$. Consider the set $S$ of ordered pairs $(x, y)$ for which there is a finite set $J$ of positive integers such that $x=\\sum_{j \\in J} c_{j}, y=\\sum_{j \\in J} c_{j-1}$. Prove that there exist real numbers $\\alpha, \\beta$ and $m, M$ with the following property: An ordered pair of nonnegative integers $(x, y)$ satisfies the inequality $$ m<\\alpha x+\\beta y1$ and $-1<\\psi<0$, this implies $\\alpha \\varphi+\\beta=0$. To satisfy $\\alpha \\varphi+\\beta=0$, one can set for instance $\\alpha=\\psi, \\beta=1$. We now find the required $m$ and $M$ for this choice of $\\alpha$ and $\\beta$. Note first that the above displayed equation gives $c_{n} \\psi+c_{n-1}=\\psi^{n-1}, n \\geq 1$. In the sequel, we denote the pairs in $S$ by $\\left(a_{J}, b_{J}\\right)$, where $J$ is a finite subset of the set $\\mathbb{N}$ of positive integers and $a_{J}=\\sum_{j \\in J} c_{j}, b_{J}=\\sum_{j \\in J} c_{j-1}$. Since $\\psi a_{J}+b_{J}=\\sum_{j \\in J}\\left(c_{j} \\psi+c_{j-1}\\right)$, we obtain $$ \\psi a_{J}+b_{J}=\\sum_{j \\in J} \\psi^{j-1} \\quad \\text { for each }\\left(a_{J}, b_{J}\\right) \\in S $$ On the other hand, in view of $-1<\\psi<0$, $$ -1=\\frac{\\psi}{1-\\psi^{2}}=\\sum_{j=0}^{\\infty} \\psi^{2 j+1}<\\sum_{j \\in J} \\psi^{j-1}<\\sum_{j=0}^{\\infty} \\psi^{2 j}=\\frac{1}{1-\\psi^{2}}=1-\\psi=\\varphi $$ Therefore, according to (1), $$ -1<\\psi a_{J}+b_{J}<\\varphi \\quad \\text { for each }\\left(a_{J}, b_{J}\\right) \\in S $$ Thus $m=-1$ and $M=\\varphi$ is an appropriate choice. Conversely, we prove that if an ordered pair of nonnegative integers $(x, y)$ satisfies the inequality $-1<\\psi x+y<\\varphi$ then $(x, y) \\in S$. Lemma. Let $x, y$ be nonnegative integers such that $-1<\\psi x+y<\\varphi$. Then there exists a subset $J$ of $\\mathbb{N}$ such that $$ \\psi x+y=\\sum_{j \\in J} \\psi^{j-1} $$ Proof. For $x=y=0$ it suffices to choose the empty subset of $\\mathbb{N}$ as $J$, so let at least one of $x, y$ be nonzero. There exist representations of $\\psi x+y$ of the form $$ \\psi x+y=\\psi^{i_{1}}+\\cdots+\\psi^{i_{k}} $$ where $i_{1} \\leq \\cdots \\leq i_{k}$ is a sequence of nonnegative integers, not necessarily distinct. For instance, we can take $x$ summands $\\psi^{1}=\\psi$ and $y$ summands $\\psi^{0}=1$. Consider all such representations of minimum length $k$ and focus on the ones for which $i_{1}$ has the minimum possible value $j_{1}$. Among them, consider the representations where $i_{2}$ has the minimum possible value $j_{2}$. Upon choosing $j_{3}, \\ldots, j_{k}$ analogously, we obtain a sequence $j_{1} \\leq \\cdots \\leq j_{k}$ which clearly satisfies $\\psi x+y=\\sum_{r=1}^{k} \\psi^{j_{r}}$. To prove the lemma, it suffices to show that $j_{1}, \\ldots, j_{k}$ are pairwise distinct. Suppose on the contrary that $j_{r}=j_{r+1}$ for some $r=1, \\ldots, k-1$. Let us consider the case $j_{r} \\geq 2$ first. Observing that $2 \\psi^{2}=1+\\psi^{3}$, we replace $j_{r}$ and $j_{r+1}$ by $j_{r}-2$ and $j_{r}+1$, respectively. Since $$ \\psi^{j_{r}}+\\psi^{j_{r+1}}=2 \\psi^{j_{r}}=\\psi^{j_{r}-2}\\left(1+\\psi^{3}\\right)=\\psi^{j_{r}-2}+\\psi^{j_{r}+1} $$ the new sequence also represents $\\psi x+y$ as needed, and the value of $i_{r}$ in it contradicts the minimum choice of $j_{r}$. Let $j_{r}=j_{r+1}=0$. Then the sum $\\psi x+y=\\sum_{r=1}^{k} \\psi^{j_{r}}$ contains at least two summands equal to $\\psi^{0}=1$. On the other hand $j_{s} \\neq 1$ for all $s$, because the equality $1+\\psi=\\psi^{2}$ implies that a representation of minimum length cannot contain consecutive $i_{r}$ 's. It follows that $$ \\psi x+y=\\sum_{r=1}^{k} \\psi^{j_{r}}>2+\\psi^{3}+\\psi^{5}+\\psi^{7}+\\cdots=2-\\psi^{2}=\\varphi $$ contradicting the condition of the lemma. Let $j_{r}=j_{r+1}=1$; then $\\sum_{r=1}^{k} \\psi^{j_{r}}$ contains at least two summands equal to $\\psi^{1}=\\psi$. Like in the case $j_{r}=j_{r+1}=0$, we also infer that $j_{s} \\neq 0$ and $j_{s} \\neq 2$ for all $s$. Therefore $$ \\psi x+y=\\sum_{r=1}^{k} \\psi^{j_{r}}<2 \\psi+\\psi^{4}+\\psi^{6}+\\psi^{8}+\\cdots=2 \\psi-\\psi^{3}=-1 $$ which is a contradiction again. The conclusion follows. Now let the ordered pair $(x, y)$ satisfy $-1<\\psi x+y<\\varphi$; hence the lemma applies to $(x, y)$. Let $J \\subset \\mathbb{N}$ be such that (2) holds. Comparing (1) and (2), we conclude that $\\psi x+y=\\psi a_{J}+b_{J}$. Now, $x, y, a_{J}$ and $b_{J}$ are integers, and $\\psi$ is irrational. So the last equality implies $x=a_{J}$ and $y=b_{J}$. This shows that the numbers $\\alpha=\\psi, \\beta=1, m=-1, M=\\varphi$ meet the requirements. Comment. We present another way to prove the lemma, constructing the set $J$ inductively. For $x=y=0$, choose $J=\\emptyset$. We induct on $n=3 x+2 y$. Suppose that an appropriate set $J$ exists when $3 x+2 y0$. The current set $J$ should be $$ \\text { either } 1 \\leq j_{1}\\psi>-1$; therefore $x^{\\prime}, y^{\\prime} \\geq 0$. Moreover, we have $3 x^{\\prime}+2 y^{\\prime}=2 x+y \\leq \\frac{2}{3} n$; therefore, if (3) holds then the induction applies: the numbers $x^{\\prime}, y^{\\prime}$ are represented in the form as needed, hence $x, y$ also. Now consider $\\frac{\\psi x+y-1}{\\psi}$. Since $$ \\frac{\\psi x+y-1}{\\psi}=x+(\\psi-1)(y-1)=\\psi(y-1)+(x-y+1) $$ we set $x^{\\prime}=y-1$ and $y^{\\prime}=x-y+1$. Again we require that $\\frac{\\psi x+y-1}{\\psi} \\in(-1, \\varphi)$, i.e. $$ \\psi x+y \\in(\\varphi \\cdot \\psi+1,(-1) \\cdot \\psi+1)=(0, \\varphi) . $$ If (4) holds then $y-1 \\geq \\psi x+y-1>-1$ and $x-y+1 \\geq-\\psi x-y+1>-\\varphi+1>-1$, therefore $x^{\\prime}, y^{\\prime} \\geq 0$. Moreover, $3 x^{\\prime}+2 y^{\\prime}=2 x+y-1<\\frac{2}{3} n$ and the induction works. Finally, $(-1,-\\psi) \\cup(0, \\varphi)=(-1, \\varphi)$ so at least one of (3) and (4) holds and the induction step is justified.","tier":0} +{"problem_type":"Algebra","problem_label":"A4","problem":"Prove the inequality $$ \\sum_{i0$ (which can jointly occur as values of these symmetric forms). Suppose that among the numbers $a_{i}$ there are some three, say $a_{k}, a_{l}, a_{m}$ such that $a_{k}\\sqrt{a+b}>\\sqrt{c}$. Let $x=\\sqrt{b}+\\sqrt{c}-\\sqrt{a}, y=\\sqrt{c}+\\sqrt{a}-\\sqrt{b}$ and $z=\\sqrt{a}+\\sqrt{b}-\\sqrt{c}$. Then $b+c-a=\\left(\\frac{z+x}{2}\\right)^{2}+\\left(\\frac{x+y}{2}\\right)^{2}-\\left(\\frac{y+z}{2}\\right)^{2}=\\frac{x^{2}+x y+x z-y z}{2}=x^{2}-\\frac{1}{2}(x-y)(x-z)$ and $$ \\frac{\\sqrt{b+c-a}}{\\sqrt{b}+\\sqrt{c}-\\sqrt{a}}=\\sqrt{1-\\frac{(x-y)(x-z)}{2 x^{2}}} \\leq 1-\\frac{(x-y)(x-z)}{4 x^{2}} $$ applying $\\sqrt{1+2 u} \\leq 1+u$ in the last step. Similarly we obtain $$ \\frac{\\sqrt{c+a-b}}{\\sqrt{c}+\\sqrt{a}-\\sqrt{b}} \\leq 1-\\frac{(z-x)(z-y)}{4 z^{2}} \\quad \\text { and } \\quad \\frac{\\sqrt{a+b-c}}{\\sqrt{a}+\\sqrt{b}-\\sqrt{c}} \\leq 1-\\frac{(y-z)(y-x)}{4 y^{2}} $$ Substituting these quantities into the statement, it is sufficient to prove that $$ \\frac{(x-y)(x-z)}{x^{2}}+\\frac{(y-z)(y-x)}{y^{2}}+\\frac{(z-x)(z-y)}{z^{2}} \\geq 0 . $$ By symmetry we can assume $x \\leq y \\leq z$. Then $$ \\begin{gathered} \\frac{(x-y)(x-z)}{x^{2}}=\\frac{(y-x)(z-x)}{x^{2}} \\geq \\frac{(y-x)(z-y)}{y^{2}}=-\\frac{(y-z)(y-x)}{y^{2}} \\\\ \\frac{(z-x)(z-y)}{z^{2}} \\geq 0 \\end{gathered} $$ and (1) follows. Comment 1. Inequality (1) is a special case of the well-known inequality $$ x^{t}(x-y)(x-z)+y^{t}(y-z)(y-x)+z^{t}(z-x)(z-y) \\geq 0 $$ which holds for all positive numbers $x, y, z$ and real $t$; in our case $t=-2$. Case $t>0$ is called Schur's inequality. More generally, if $x \\leq y \\leq z$ are real numbers and $p, q, r$ are nonnegative numbers such that $q \\leq p$ or $q \\leq r$ then $$ p(x-y)(x-z)+q(y-z)(y-x)+r(z-x)(z-y) \\geq 0 \\text {. } $$ Comment 2. One might also start using Cauchy-Schwarz' inequality (or the root mean square vs. arithmetic mean inequality) to the effect that $$ \\left(\\sum \\frac{\\sqrt{b+c-a}}{\\sqrt{b}+\\sqrt{c}-\\sqrt{a}}\\right)^{2} \\leq 3 \\cdot \\sum \\frac{b+c-a}{(\\sqrt{b}+\\sqrt{c}-\\sqrt{a})^{2}} $$ in cyclic sum notation. There are several ways to prove that the right-hand side of (2) never exceeds 9 (and this is just what we need). One of them is to introduce new variables $x, y, z$, as in Solution 1, which upon some manipulation brings the problem again to inequality (1). Alternatively, the claim that right-hand side of (2) is not greater than 9 can be expressed in terms of the symmetric forms $\\sigma_{1}=\\sum x, \\sigma_{2}=\\sum x y, \\sigma_{3}=x y z$ equivalently as $$ 4 \\sigma_{1} \\sigma_{2} \\sigma_{3} \\leq \\sigma_{2}^{3}+9 \\sigma_{3}^{2} $$ which is a known inequality. A yet different method to deal with the right-hand expression in (2) is to consider $\\sqrt{a}, \\sqrt{b}, \\sqrt{c}$ as sides of a triangle. Through standard trigonometric formulas the problem comes down to showing that $$ p^{2} \\leq 4 R^{2}+4 R r+3 r^{2} $$ $p, R$ and $r$ standing for the semiperimeter, the circumradius and the inradius of that triangle. Again, (4) is another known inequality. Note that the inequalities (1), (3), (4) are equivalent statements about the same mathematical situation.","tier":0} +{"problem_type":"Algebra","problem_label":"A5","problem":"Let $a, b, c$ be the sides of a triangle. Prove that $$ \\frac{\\sqrt{b+c-a}}{\\sqrt{b}+\\sqrt{c}-\\sqrt{a}}+\\frac{\\sqrt{c+a-b}}{\\sqrt{c}+\\sqrt{a}-\\sqrt{b}}+\\frac{\\sqrt{a+b-c}}{\\sqrt{a}+\\sqrt{b}-\\sqrt{c}} \\leq 3 . $$ (Korea)","solution":"Due to the symmetry of variables, it can be assumed that $a \\geq b \\geq c$. We claim that $$ \\frac{\\sqrt{a+b-c}}{\\sqrt{a}+\\sqrt{b}-\\sqrt{c}} \\leq 1 \\quad \\text { and } \\quad \\frac{\\sqrt{b+c-a}}{\\sqrt{b}+\\sqrt{c}-\\sqrt{a}}+\\frac{\\sqrt{c+a-b}}{\\sqrt{c}+\\sqrt{a}-\\sqrt{b}} \\leq 2 . $$ The first inequality follows from $$ \\sqrt{a+b-c}-\\sqrt{a}=\\frac{(a+b-c)-a}{\\sqrt{a+b-c}+\\sqrt{a}} \\leq \\frac{b-c}{\\sqrt{b}+\\sqrt{c}}=\\sqrt{b}-\\sqrt{c} . $$ For proving the second inequality, let $p=\\sqrt{a}+\\sqrt{b}$ and $q=\\sqrt{a}-\\sqrt{b}$. Then $a-b=p q$ and the inequality becomes $$ \\frac{\\sqrt{c-p q}}{\\sqrt{c}-q}+\\frac{\\sqrt{c+p q}}{\\sqrt{c}+q} \\leq 2 $$ From $a \\geq b \\geq c$ we have $p \\geq 2 \\sqrt{c}$. Applying the Cauchy-Schwarz inequality, $$ \\begin{gathered} \\left(\\frac{\\sqrt{c-p q}}{\\sqrt{c}-q}+\\frac{\\sqrt{c+p q}}{\\sqrt{c}+q}\\right)^{2} \\leq\\left(\\frac{c-p q}{\\sqrt{c}-q}+\\frac{c+p q}{\\sqrt{c}+q}\\right)\\left(\\frac{1}{\\sqrt{c}-q}+\\frac{1}{\\sqrt{c}+q}\\right) \\\\ =\\frac{2\\left(c \\sqrt{c}-p q^{2}\\right)}{c-q^{2}} \\cdot \\frac{2 \\sqrt{c}}{c-q^{2}}=4 \\cdot \\frac{c^{2}-\\sqrt{c} p q^{2}}{\\left(c-q^{2}\\right)^{2}} \\leq 4 \\cdot \\frac{c^{2}-2 c q^{2}}{\\left(c-q^{2}\\right)^{2}} \\leq 4 . \\end{gathered} $$","tier":0} +{"problem_type":"Algebra","problem_label":"A6","problem":"Determine the smallest number $M$ such that the inequality $$ \\left|a b\\left(a^{2}-b^{2}\\right)+b c\\left(b^{2}-c^{2}\\right)+c a\\left(c^{2}-a^{2}\\right)\\right| \\leq M\\left(a^{2}+b^{2}+c^{2}\\right)^{2} $$ holds for all real numbers $a, b, c$. (Ireland)","solution":"We first consider the cubic polynomial $$ P(t)=t b\\left(t^{2}-b^{2}\\right)+b c\\left(b^{2}-c^{2}\\right)+c t\\left(c^{2}-t^{2}\\right) . $$ It is easy to check that $P(b)=P(c)=P(-b-c)=0$, and therefore $$ P(t)=(b-c)(t-b)(t-c)(t+b+c), $$ since the cubic coefficient is $b-c$. The left-hand side of the proposed inequality can therefore be written in the form $$ \\left|a b\\left(a^{2}-b^{2}\\right)+b c\\left(b^{2}-c^{2}\\right)+c a\\left(c^{2}-a^{2}\\right)\\right|=|P(a)|=|(b-c)(a-b)(a-c)(a+b+c)| . $$ The problem comes down to finding the smallest number $M$ that satisfies the inequality $$ |(b-c)(a-b)(a-c)(a+b+c)| \\leq M \\cdot\\left(a^{2}+b^{2}+c^{2}\\right)^{2} \\text {. } $$ Note that this expression is symmetric, and we can therefore assume $a \\leq b \\leq c$ without loss of generality. With this assumption, $$ |(a-b)(b-c)|=(b-a)(c-b) \\leq\\left(\\frac{(b-a)+(c-b)}{2}\\right)^{2}=\\frac{(c-a)^{2}}{4} $$ with equality if and only if $b-a=c-b$, i.e. $2 b=a+c$. Also $$ \\left(\\frac{(c-b)+(b-a)}{2}\\right)^{2} \\leq \\frac{(c-b)^{2}+(b-a)^{2}}{2} $$ or equivalently, $$ 3(c-a)^{2} \\leq 2 \\cdot\\left[(b-a)^{2}+(c-b)^{2}+(c-a)^{2}\\right], $$ again with equality only for $2 b=a+c$. From (2) and (3) we get $$ \\begin{aligned} & |(b-c)(a-b)(a-c)(a+b+c)| \\\\ \\leq & \\frac{1}{4} \\cdot\\left|(c-a)^{3}(a+b+c)\\right| \\\\ = & \\frac{1}{4} \\cdot \\sqrt{(c-a)^{6}(a+b+c)^{2}} \\\\ \\leq & \\frac{1}{4} \\cdot \\sqrt{\\left(\\frac{2 \\cdot\\left[(b-a)^{2}+(c-b)^{2}+(c-a)^{2}\\right]}{3}\\right)^{3} \\cdot(a+b+c)^{2}} \\\\ = & \\frac{\\sqrt{2}}{2} \\cdot\\left(\\sqrt[4]{\\left(\\frac{(b-a)^{2}+(c-b)^{2}+(c-a)^{2}}{3}\\right)^{3} \\cdot(a+b+c)^{2}}\\right)^{2} . \\end{aligned} $$ By the weighted AM-GM inequality this estimate continues as follows: $$ \\begin{aligned} & |(b-c)(a-b)(a-c)(a+b+c)| \\\\ \\leq & \\frac{\\sqrt{2}}{2} \\cdot\\left(\\frac{(b-a)^{2}+(c-b)^{2}+(c-a)^{2}+(a+b+c)^{2}}{4}\\right)^{2} \\\\ = & \\frac{9 \\sqrt{2}}{32} \\cdot\\left(a^{2}+b^{2}+c^{2}\\right)^{2} . \\end{aligned} $$ We see that the inequality (1) is satisfied for $M=\\frac{9}{32} \\sqrt{2}$, with equality if and only if $2 b=a+c$ and $$ \\frac{(b-a)^{2}+(c-b)^{2}+(c-a)^{2}}{3}=(a+b+c)^{2} $$ Plugging $b=(a+c) \/ 2$ into the last equation, we bring it to the equivalent form $$ 2(c-a)^{2}=9(a+c)^{2} . $$ The conditions for equality can now be restated as $$ 2 b=a+c \\quad \\text { and } \\quad(c-a)^{2}=18 b^{2} \\text {. } $$ Setting $b=1$ yields $a=1-\\frac{3}{2} \\sqrt{2}$ and $c=1+\\frac{3}{2} \\sqrt{2}$. We see that $M=\\frac{9}{32} \\sqrt{2}$ is indeed the smallest constant satisfying the inequality, with equality for any triple $(a, b, c)$ proportional to $\\left(1-\\frac{3}{2} \\sqrt{2}, 1,1+\\frac{3}{2} \\sqrt{2}\\right)$, up to permutation. Comment. With the notation $x=b-a, y=c-b, z=a-c, s=a+b+c$ and $r^{2}=a^{2}+b^{2}+c^{2}$, the inequality (1) becomes just $|s x y z| \\leq M r^{4}$ (with suitable constraints on $s$ and $r$ ). The original asymmetric inequality turns into a standard symmetric one; from this point on the solution can be completed in many ways. One can e.g. use the fact that, for fixed values of $\\sum x$ and $\\sum x^{2}$, the product $x y z$ is a maximum\/minimum only if some of $x, y, z$ are equal, thus reducing one degree of freedom, etc. As observed by the proposer, a specific attraction of the problem is that the maximum is attained at a point $(a, b, c)$ with all coordinates distinct.","tier":0} +{"problem_type":"Combinatorics","problem_label":"C1","problem":"We have $n \\geq 2$ lamps $L_{1}, \\ldots, L_{n}$ in a row, each of them being either on or off. Every second we simultaneously modify the state of each lamp as follows: - if the lamp $L_{i}$ and its neighbours (only one neighbour for $i=1$ or $i=n$, two neighbours for other $i$ ) are in the same state, then $L_{i}$ is switched off; - otherwise, $L_{i}$ is switched on. Initially all the lamps are off except the leftmost one which is on. (a) Prove that there are infinitely many integers $n$ for which all the lamps will eventually be off. (b) Prove that there are infinitely many integers $n$ for which the lamps will never be all off. (France)","solution":"(a) Experiments with small $n$ lead to the guess that every $n$ of the form $2^{k}$ should be good. This is indeed the case, and more precisely: let $A_{k}$ be the $2^{k} \\times 2^{k}$ matrix whose rows represent the evolution of the system, with entries 0,1 (for off and on respectively). The top row shows the initial state $[1,0,0, \\ldots, 0]$; the bottom row shows the state after $2^{k}-1$ steps. The claim is that: $$ \\text { The bottom row of } A_{k} \\text { is }[1,1,1, \\ldots, 1] \\text {. } $$ This will of course suffice because one more move then produces $[0,0,0, \\ldots, 0]$, as required. The proof is by induction on $k$. The base $k=1$ is obvious. Assume the claim to be true for a $k \\geq 1$ and write the matrix $A_{k+1}$ in the block form $\\left(\\begin{array}{ll}A_{k} & O_{k} \\\\ B_{k} & C_{k}\\end{array}\\right)$ with four $2^{k} \\times 2^{k}$ matrices. After $m$ steps, the last 1 in a row is at position $m+1$. Therefore $O_{k}$ is the zero matrix. According to the induction hypothesis, the bottom row of $\\left[A_{k} O_{k}\\right]$ is $[1, \\ldots, 1,0, \\ldots, 0]$, with $2^{k}$ ones and $2^{k}$ zeros. The next row is thus $$ [\\underbrace{0, \\ldots, 0}_{2^{k}-1}, 1,1, \\underbrace{0, \\ldots, 0}_{2^{k}-1}] $$ It is symmetric about its midpoint, and this symmetry is preserved in all subsequent rows because the procedure described in the problem statement is left\/right symmetric. Thus $B_{k}$ is the mirror image of $C_{k}$. In particular, the rightmost column of $B_{k}$ is identical with the leftmost column of $C_{k}$. Imagine the matrix $C_{k}$ in isolation from the rest of $A_{k+1}$. Suppose it is subject to evolution as defined in the problem: the first (leftmost) term in a row depends only on the two first terms in the preceding row, according as they are equal or not. Now embed $C_{k}$ again in $A_{k}$. The 'leftmost' terms in the rows of $C_{k}$ now have neighbours on their left side- but these neighbours are their exact copies. Consequently the actual evolution within $C_{k}$ is the same, whether or not $C_{k}$ is considered as a piece of $A_{k+1}$ or in isolation. And since the top row of $C_{k}$ is $[1,0, \\ldots, 0]$, it follows that $C_{k}$ is identical with $A_{k}$. The bottom row of $A_{k}$ is $[1,1, \\ldots, 1]$; the same is the bottom row of $C_{k}$, hence also of $B_{k}$, which mirrors $C_{k}$. So the bottom row of $A_{k+1}$ consists of ones only and the induction is complete. (b) There are many ways to produce an infinite sequence of those $n$ for which the state $[0,0, \\ldots, 0]$ will never be achieved. As an example, consider $n=2^{k}+1$ (for $k \\geq 1$ ). The evolution of the system can be represented by a matrix $\\mathcal{A}$ of width $2^{k}+1$ with infinitely many rows. The top $2^{k}$ rows form the matrix $A_{k}$ discussed above, with one column of zeros attached at its right. In the next row we then have the vector $[0,0, \\ldots, 0,1,1]$. But this is just the second row of $\\mathcal{A}$ reversed. Subsequent rows will be mirror copies of the foregoing ones, starting from the second one. So the configuration $[1,1,0, \\ldots, 0,0]$, i.e. the second row of $\\mathcal{A}$, will reappear. Further rows will periodically repeat this pattern and there will be no row of zeros.","tier":0} +{"problem_type":"Combinatorics","problem_label":"C2","problem":"A diagonal of a regular 2006-gon is called odd if its endpoints divide the boundary into two parts, each composed of an odd number of sides. Sides are also regarded as odd diagonals. Suppose the 2006-gon has been dissected into triangles by 2003 nonintersecting diagonals. Find the maximum possible number of isosceles triangles with two odd sides. (Serbia)","solution":"Call an isosceles triangle odd if it has two odd sides. Suppose we are given a dissection as in the problem statement. A triangle in the dissection which is odd and isosceles will be called iso-odd for brevity. Lemma. Let $A B$ be one of dissecting diagonals and let $\\mathcal{L}$ be the shorter part of the boundary of the 2006-gon with endpoints $A, B$. Suppose that $\\mathcal{L}$ consists of $n$ segments. Then the number of iso-odd triangles with vertices on $\\mathcal{L}$ does not exceed $n \/ 2$. Proof. This is obvious for $n=2$. Take $n$ with $20$ and assume the statement for less than $n$ points. Take a set $S$ of $n$ points. Let $C$ be the set of vertices of the convex hull of $S$, let $m=|C|$. Let $X \\subset C$ be an arbitrary nonempty set. For any convex polygon $P$ with vertices in the set $S \\backslash X$, we have $b(P)$ points of $S$ outside $P$. Excluding the points of $X-$ all outside $P$ \u2014 the set $S \\backslash X$ contains exactly $b(P)-|X|$ of them. Writing $1-x=y$, by the induction hypothesis $$ \\sum_{P \\subset S \\backslash X} x^{a(P)} y^{b(P)-|X|}=1 $$ (where $P \\subset S \\backslash X$ means that the vertices of $P$ belong to the set $S \\backslash X$ ). Therefore $$ \\sum_{P \\subset S \\backslash X} x^{a(P)} y^{b(P)}=y^{|X|} $$ All convex polygons appear at least once, except the convex hull $C$ itself. The convex hull adds $x^{m}$. We can use the inclusion-exclusion principle to compute the sum of the other terms: $$ \\begin{gathered} \\sum_{P \\neq C} x^{a(P)} y^{b(P)}=\\sum_{k=1}^{m}(-1)^{k-1} \\sum_{|X|=k} \\sum_{P \\subset S \\backslash X} x^{a(P)} y^{b(P)}=\\sum_{k=1}^{m}(-1)^{k-1} \\sum_{|X|=k} y^{k} \\\\ =\\sum_{k=1}^{m}(-1)^{k-1}\\left(\\begin{array}{c} m \\\\ k \\end{array}\\right) y^{k}=-\\left((1-y)^{m}-1\\right)=1-x^{m} \\end{gathered} $$ and then $$ \\sum_{P} x^{a(P)} y^{b(P)}=\\sum_{P=C}+\\sum_{P \\neq C}=x^{m}+\\left(1-x^{m}\\right)=1 $$","tier":0} +{"problem_type":"Combinatorics","problem_label":"C4","problem":"A cake has the form of an $n \\times n$ square composed of $n^{2}$ unit squares. Strawberries lie on some of the unit squares so that each row or column contains exactly one strawberry; call this arrangement $\\mathcal{A}$. Let $\\mathcal{B}$ be another such arrangement. Suppose that every grid rectangle with one vertex at the top left corner of the cake contains no fewer strawberries of arrangement $\\mathcal{B}$ than of arrangement $\\mathcal{A}$. Prove that arrangement $\\mathcal{B}$ can be obtained from $\\mathcal{A}$ by performing a number of switches, defined as follows: A switch consists in selecting a grid rectangle with only two strawberries, situated at its top right corner and bottom left corner, and moving these two strawberries to the other two corners of that rectangle. (Taiwan)","solution":"We use capital letters to denote unit squares; $O$ is the top left corner square. For any two squares $X$ and $Y$ let $[X Y]$ be the smallest grid rectangle containing these two squares. Strawberries lie on some squares in arrangement $\\mathcal{A}$. Put a plum on each square of the target configuration $\\mathcal{B}$. For a square $X$ denote by $a(X)$ and $b(X)$ respectively the number of strawberries and the number of plums in $[O X]$. By hypothesis $a(X) \\leq b(X)$ for each $X$, with strict inequality for some $X$ (otherwise the two arrangements coincide and there is nothing to prove). The idea is to show that by a legitimate switch one can obtain an arrangement $\\mathcal{A}^{\\prime}$ such that $$ a(X) \\leq a^{\\prime}(X) \\leq b(X) \\quad \\text { for each } X ; \\quad \\sum_{X} a(X)<\\sum_{X} a^{\\prime}(X) $$ (with $a^{\\prime}(X)$ defined analogously to $a(X)$; the sums range over all unit squares $X$ ). This will be enough because the same reasoning then applies to $\\mathcal{A}^{\\prime}$, giving rise to a new arrangement $\\mathcal{A}^{\\prime \\prime}$, and so on (induction). Since $\\sum a(X)<\\sum a^{\\prime}(X)<\\sum a^{\\prime \\prime}(X)<\\ldots$ and all these sums do not exceed $\\sum b(X)$, we eventually obtain a sum with all summands equal to the respective $b(X) \\mathrm{s}$; all strawberries will meet with plums. Consider the uppermost row in which the plum and the strawberry lie on different squares $P$ and $S$ (respectively); clearly $P$ must be situated left to $S$. In the column passing through $P$, let $T$ be the top square and $B$ the bottom square. The strawberry in that column lies below the plum (because there is no plum in that column above $P$, and the positions of strawberries and plums coincide everywhere above the row of $P$ ). Hence there is at least one strawberry in the region $[B S]$ below $[P S]$. Let $V$ be the position of the uppermost strawberry in that region. ![](https:\/\/cdn.mathpix.com\/cropped\/2024_04_17_e55ccc62f43b046023cag-26.jpg?height=708&width=711&top_left_y=2010&top_left_x=678) Denote by $W$ the square at the intersection of the row through $V$ with the column through $S$ and let $R$ be the square vertex-adjacent to $W$ up-left. We claim that $$ a(X)C D$. Points $K$ and $L$ lie on the line segments $A B$ and $C D$, respectively, so that $A K \/ K B=D L \/ L C$. Suppose that there are points $P$ and $Q$ on the line segment $K L$ satisfying $$ \\angle A P B=\\angle B C D \\quad \\text { and } \\quad \\angle C Q D=\\angle A B C \\text {. } $$ Prove that the points $P, Q, B$ and $C$ are concyclic. (Ukraine)","solution":"Because $A B \\| C D$, the relation $A K \/ K B=D L \/ L C$ readily implies that the lines $A D, B C$ and $K L$ have a common point $S$. ![](https:\/\/cdn.mathpix.com\/cropped\/2024_04_17_e55ccc62f43b046023cag-37.jpg?height=1136&width=1073&top_left_y=851&top_left_x=500) Consider the second intersection points $X$ and $Y$ of the line $S K$ with the circles $(A B P)$ and $(C D Q)$, respectively. Since $A P B X$ is a cyclic quadrilateral and $A B \\| C D$, one has $$ \\angle A X B=180^{\\circ}-\\angle A P B=180^{\\circ}-\\angle B C D=\\angle A B C . $$ This shows that $B C$ is tangent to the circle $(A B P)$ at $B$. Likewise, $B C$ is tangent to the circle $(C D Q)$ at $C$. Therefore $S P \\cdot S X=S B^{2}$ and $S Q \\cdot S Y=S C^{2}$. Let $h$ be the homothety with centre $S$ and ratio $S C \/ S B$. Since $h(B)=C$, the above conclusion about tangency implies that $h$ takes circle $(A B P)$ to circle $(C D Q)$. Also, $h$ takes $A B$ to $C D$, and it easily follows that $h(P)=Y, h(X)=Q$, yielding $S P \/ S Y=S B \/ S C=S X \/ S Q$. Equalities $S P \\cdot S X=S B^{2}$ and $S Q \/ S X=S C \/ S B$ imply $S P \\cdot S Q=S B \\cdot S C$, which is equivalent to $P, Q, B$ and $C$ being concyclic.","tier":0} +{"problem_type":"Geometry","problem_label":"G2","problem":"Let $A B C D$ be a trapezoid with parallel sides $A B>C D$. Points $K$ and $L$ lie on the line segments $A B$ and $C D$, respectively, so that $A K \/ K B=D L \/ L C$. Suppose that there are points $P$ and $Q$ on the line segment $K L$ satisfying $$ \\angle A P B=\\angle B C D \\quad \\text { and } \\quad \\angle C Q D=\\angle A B C \\text {. } $$ Prove that the points $P, Q, B$ and $C$ are concyclic. (Ukraine)","solution":"The case where $P=Q$ is trivial. Thus assume that $P$ and $Q$ are two distinct points. As in the first solution, notice that the lines $A D, B C$ and $K L$ concur at a point $S$. ![](https:\/\/cdn.mathpix.com\/cropped\/2024_04_17_e55ccc62f43b046023cag-38.jpg?height=762&width=799&top_left_y=430&top_left_x=640) Let the lines $A P$ and $D Q$ meet at $E$, and let $B P$ and $C Q$ meet at $F$. Then $\\angle E P F=\\angle B C D$ and $\\angle F Q E=\\angle A B C$ by the condition of the problem. Since the angles $B C D$ and $A B C$ add up to $180^{\\circ}$, it follows that $P E Q F$ is a cyclic quadrilateral. Applying Menelaus' theorem, first to triangle $A S P$ and line $D Q$ and then to triangle $B S P$ and line $C Q$, we have $$ \\frac{A D}{D S} \\cdot \\frac{S Q}{Q P} \\cdot \\frac{P E}{E A}=1 \\text { and } \\frac{B C}{C S} \\cdot \\frac{S Q}{Q P} \\cdot \\frac{P F}{F B}=1 $$ The first factors in these equations are equal, as $A B \\| C D$. Thus the last factors are also equal, which implies that $E F$ is parallel to $A B$ and $C D$. Using this and the cyclicity of $P E Q F$, we obtain $$ \\angle B C D=\\angle B C F+\\angle F C D=\\angle B C Q+\\angle E F Q=\\angle B C Q+\\angle E P Q $$ On the other hand, $$ \\angle B C D=\\angle A P B=\\angle E P F=\\angle E P Q+\\angle Q P F, $$ and consequently $\\angle B C Q=\\angle Q P F$. The latter angle either coincides with $\\angle Q P B$ or is supplementary to $\\angle Q P B$, depending on whether $Q$ lies between $K$ and $P$ or not. In either case it follows that $P, Q, B$ and $C$ are concyclic.","tier":0} +{"problem_type":"Geometry","problem_label":"G3","problem":"Let $A B C D E$ be a convex pentagon such that $$ \\angle B A C=\\angle C A D=\\angle D A E \\quad \\text { and } \\quad \\angle A B C=\\angle A C D=\\angle A D E \\text {. } $$ The diagonals $B D$ and $C E$ meet at $P$. Prove that the line $A P$ bisects the side $C D$.","solution":"Let the diagonals $A C$ and $B D$ meet at $Q$, the diagonals $A D$ and $C E$ meet at $R$, and let the ray $A P$ meet the side $C D$ at $M$. We want to prove that $C M=M D$ holds. ![](https:\/\/cdn.mathpix.com\/cropped\/2024_04_17_e55ccc62f43b046023cag-39.jpg?height=457&width=669&top_left_y=751&top_left_x=702) The idea is to show that $Q$ and $R$ divide $A C$ and $A D$ in the same ratio, or more precisely $$ \\frac{A Q}{Q C}=\\frac{A R}{R D} $$ (which is equivalent to $Q R \\| C D$ ). The given angle equalities imply that the triangles $A B C$, $A C D$ and $A D E$ are similar. We therefore have $$ \\frac{A B}{A C}=\\frac{A C}{A D}=\\frac{A D}{A E} $$ Since $\\angle B A D=\\angle B A C+\\angle C A D=\\angle C A D+\\angle D A E=\\angle C A E$, it follows from $A B \/ A C=$ $A D \/ A E$ that the triangles $A B D$ and $A C E$ are also similar. Their angle bisectors in $A$ are $A Q$ and $A R$, respectively, so that $$ \\frac{A B}{A C}=\\frac{A Q}{A R} $$ Because $A B \/ A C=A C \/ A D$, we obtain $A Q \/ A R=A C \/ A D$, which is equivalent to (1). Now Ceva's theorem for the triangle $A C D$ yields $$ \\frac{A Q}{Q C} \\cdot \\frac{C M}{M D} \\cdot \\frac{D R}{R A}=1 $$ In view of (1), this reduces to $C M=M D$, which completes the proof. Comment. Relation (1) immediately follows from the fact that quadrilaterals $A B C D$ and $A C D E$ are similar.","tier":0} +{"problem_type":"Geometry","problem_label":"G4","problem":"A point $D$ is chosen on the side $A C$ of a triangle $A B C$ with $\\angle C<\\angle A<90^{\\circ}$ in such a way that $B D=B A$. The incircle of $A B C$ is tangent to $A B$ and $A C$ at points $K$ and $L$, respectively. Let $J$ be the incentre of triangle $B C D$. Prove that the line $K L$ intersects the line segment $A J$ at its midpoint. (Russia)","solution":"Denote by $P$ be the common point of $A J$ and $K L$. Let the parallel to $K L$ through $J$ meet $A C$ at $M$. Then $P$ is the midpoint of $A J$ if and only if $A M=2 \\cdot A L$, which we are about to show. ![](https:\/\/cdn.mathpix.com\/cropped\/2024_04_17_e55ccc62f43b046023cag-40.jpg?height=574&width=937&top_left_y=775&top_left_x=568) Denoting $\\angle B A C=2 \\alpha$, the equalities $B A=B D$ and $A K=A L$ imply $\\angle A D B=2 \\alpha$ and $\\angle A L K=90^{\\circ}-\\alpha$. Since $D J$ bisects $\\angle B D C$, we obtain $\\angle C D J=\\frac{1}{2} \\cdot\\left(180^{\\circ}-\\angle A D B\\right)=90^{\\circ}-\\alpha$. Also $\\angle D M J=\\angle A L K=90^{\\circ}-\\alpha$ since $J M \\| K L$. It follows that $J D=J M$. Let the incircle of triangle $B C D$ touch its side $C D$ at $T$. Then $J T \\perp C D$, meaning that $J T$ is the altitude to the base $D M$ of the isosceles triangle $D M J$. It now follows that $D T=M T$, and we have $$ D M=2 \\cdot D T=B D+C D-B C \\text {. } $$ Therefore $$ \\begin{aligned} A M & =A D+(B D+C D-B C) \\\\ & =A D+A B+D C-B C \\\\ & =A C+A B-B C \\\\ & =2 \\cdot A L, \\end{aligned} $$ which completes the proof.","tier":0} +{"problem_type":"Geometry","problem_label":"G5","problem":"In triangle $A B C$, let $J$ be the centre of the excircle tangent to side $B C$ at $A_{1}$ and to the extensions of sides $A C$ and $A B$ at $B_{1}$ and $C_{1}$, respectively. Suppose that the lines $A_{1} B_{1}$ and $A B$ are perpendicular and intersect at $D$. Let $E$ be the foot of the perpendicular from $C_{1}$ to line $D J$. Determine the angles $\\angle B E A_{1}$ and $\\angle A E B_{1}$. (Greece)","solution":"Let $K$ be the intersection point of lines $J C$ and $A_{1} B_{1}$. Obviously $J C \\perp A_{1} B_{1}$ and since $A_{1} B_{1} \\perp A B$, the lines $J K$ and $C_{1} D$ are parallel and equal. From the right triangle $B_{1} C J$ we obtain $J C_{1}^{2}=J B_{1}^{2}=J C \\cdot J K=J C \\cdot C_{1} D$ from which we infer that $D C_{1} \/ C_{1} J=C_{1} J \/ J C$ and the right triangles $D C_{1} J$ and $C_{1} J C$ are similar. Hence $\\angle C_{1} D J=\\angle J C_{1} C$, which implies that the lines $D J$ and $C_{1} C$ are perpendicular, i.e. the points $C_{1}, E, C$ are collinear. ![](https:\/\/cdn.mathpix.com\/cropped\/2024_04_17_e55ccc62f43b046023cag-41.jpg?height=625&width=874&top_left_y=924&top_left_x=594) Since $\\angle C A_{1} J=\\angle C B_{1} J=\\angle C E J=90^{\\circ}$, points $A_{1}, B_{1}$ and $E$ lie on the circle of diameter $C J$. Then $\\angle D B A_{1}=\\angle A_{1} C J=\\angle D E A_{1}$, which implies that quadrilateral $B E A_{1} D$ is cyclic; therefore $\\angle A_{1} E B=90^{\\circ}$. Quadrilateral $A D E B_{1}$ is also cyclic because $\\angle E B_{1} A=\\angle E J C=\\angle E D C_{1}$, therefore we obtain $\\angle A E B_{1}=\\angle A D B=90^{\\circ}$. ![](https:\/\/cdn.mathpix.com\/cropped\/2024_04_17_e55ccc62f43b046023cag-41.jpg?height=691&width=943&top_left_y=1962&top_left_x=565)","tier":0} +{"problem_type":"Geometry","problem_label":"G5","problem":"In triangle $A B C$, let $J$ be the centre of the excircle tangent to side $B C$ at $A_{1}$ and to the extensions of sides $A C$ and $A B$ at $B_{1}$ and $C_{1}$, respectively. Suppose that the lines $A_{1} B_{1}$ and $A B$ are perpendicular and intersect at $D$. Let $E$ be the foot of the perpendicular from $C_{1}$ to line $D J$. Determine the angles $\\angle B E A_{1}$ and $\\angle A E B_{1}$. (Greece)","solution":"Consider the circles $\\omega_{1}, \\omega_{2}$ and $\\omega_{3}$ of diameters $C_{1} D, A_{1} B$ and $A B_{1}$, respectively. Line segments $J C_{1}, J B_{1}$ and $J A_{1}$ are tangents to those circles and, due to the right angle at $D$, $\\omega_{2}$ and $\\omega_{3}$ pass through point $D$. Since $\\angle C_{1} E D$ is a right angle, point $E$ lies on circle $\\omega_{1}$, therefore $$ J C_{1}^{2}=J D \\cdot J E . $$ Since $J A_{1}=J B_{1}=J C_{1}$ are all radii of the excircle, we also have $$ J A_{1}^{2}=J D \\cdot J E \\quad \\text { and } \\quad J B_{1}^{2}=J D \\cdot J E \\text {. } $$ These equalities show that $E$ lies on circles $\\omega_{2}$ and $\\omega_{3}$ as well, so $\\angle B E A_{1}=\\angle A E B_{1}=90^{\\circ}$.","tier":0} +{"problem_type":"Geometry","problem_label":"G5","problem":"In triangle $A B C$, let $J$ be the centre of the excircle tangent to side $B C$ at $A_{1}$ and to the extensions of sides $A C$ and $A B$ at $B_{1}$ and $C_{1}$, respectively. Suppose that the lines $A_{1} B_{1}$ and $A B$ are perpendicular and intersect at $D$. Let $E$ be the foot of the perpendicular from $C_{1}$ to line $D J$. Determine the angles $\\angle B E A_{1}$ and $\\angle A E B_{1}$. (Greece)","solution":"First note that $A_{1} B_{1}$ is perpendicular to the external angle bisector $C J$ of $\\angle B C A$ and parallel to the internal angle bisector of that angle. Therefore, $A_{1} B_{1}$ is perpendicular to $A B$ if and only if triangle $A B C$ is isosceles, $A C=B C$. In that case the external bisector $C J$ is parallel to $A B$. Triangles $A B C$ and $B_{1} A_{1} J$ are similar, as their corresponding sides are perpendicular. In particular, we have $\\angle D A_{1} J=\\angle C_{1} B A_{1}$; moreover, from cyclic deltoid $J A_{1} B C_{1}$, $$ \\angle C_{1} A_{1} J=\\angle C_{1} B J=\\frac{1}{2} \\angle C_{1} B A_{1}=\\frac{1}{2} \\angle D A_{1} J $$ Therefore, $A_{1} C_{1}$ bisects angle $\\angle D A_{1} J$. ![](https:\/\/cdn.mathpix.com\/cropped\/2024_04_17_e55ccc62f43b046023cag-42.jpg?height=637&width=870&top_left_y=1229&top_left_x=593) In triangle $B_{1} A_{1} J$, line $J C_{1}$ is the external bisector at vertex $J$. The point $C_{1}$ is the intersection of two external angle bisectors (at $A_{1}$ and $J$ ) so $C_{1}$ is the centre of the excircle $\\omega$, tangent to side $A_{1} J$, and to the extension of $B_{1} A_{1}$ at point $D$. Now consider the similarity transform $\\varphi$ which moves $B_{1}$ to $A, A_{1}$ to $B$ and $J$ to $C$. This similarity can be decomposed into a rotation by $90^{\\circ}$ around a certain point $O$ and a homothety from the same centre. This similarity moves point $C_{1}$ (the centre of excircle $\\omega$ ) to $J$ and moves $D$ (the point of tangency) to $C_{1}$. Since the rotation angle is $90^{\\circ}$, we have $\\angle X O \\varphi(X)=90^{\\circ}$ for an arbitrary point $X \\neq O$. For $X=D$ and $X=C_{1}$ we obtain $\\angle D O C_{1}=\\angle C_{1} O J=90^{\\circ}$. Therefore $O$ lies on line segment $D J$ and $C_{1} O$ is perpendicular to $D J$. This means that $O=E$. For $X=A_{1}$ and $X=B_{1}$ we obtain $\\angle A_{1} O B=\\angle B_{1} O A=90^{\\circ}$, i.e. $$ \\angle B E A_{1}=\\angle A E B_{1}=90^{\\circ} . $$ Comment. Choosing $X=J$, it also follows that $\\angle J E C=90^{\\circ}$ which proves that lines $D J$ and $C C_{1}$ intersect at point $E$. However, this is true more generally, without the assumption that $A_{1} B_{1}$ and $A B$ are perpendicular, because points $C$ and $D$ are conjugates with respect to the excircle. The last observation could replace the first paragraph of Solution 1.","tier":0} +{"problem_type":"Geometry","problem_label":"G6","problem":"Circles $\\omega_{1}$ and $\\omega_{2}$ with centres $O_{1}$ and $O_{2}$ are externally tangent at point $D$ and internally tangent to a circle $\\omega$ at points $E$ and $F$, respectively. Line $t$ is the common tangent of $\\omega_{1}$ and $\\omega_{2}$ at $D$. Let $A B$ be the diameter of $\\omega$ perpendicular to $t$, so that $A, E$ and $O_{1}$ are on the same side of $t$. Prove that lines $A O_{1}, B O_{2}, E F$ and $t$ are concurrent. (Brasil)","solution":"Point $E$ is the centre of a homothety $h$ which takes circle $\\omega_{1}$ to circle $\\omega$. The radii $O_{1} D$ and $O B$ of these circles are parallel as both are perpendicular to line t. Also, $O_{1} D$ and $O B$ are on the same side of line $E O$, hence $h$ takes $O_{1} D$ to $O B$. Consequently, points $E$, $D$ and $B$ are collinear. Likewise, points $F, D$ and $A$ are collinear as well. Let lines $A E$ and $B F$ intersect at $C$. Since $A F$ and $B E$ are altitudes in triangle $A B C$, their common point $D$ is the orthocentre of this triangle. So $C D$ is perpendicular to $A B$, implying that $C$ lies on line $t$. Note that triangle $A B C$ is acute-angled. We mention the well-known fact that triangles $F E C$ and $A B C$ are similar in ratio $\\cos \\gamma$, where $\\gamma=\\angle A C B$. In addition, points $C, E, D$ and $F$ lie on the circle with diameter $C D$. ![](https:\/\/cdn.mathpix.com\/cropped\/2024_04_17_e55ccc62f43b046023cag-43.jpg?height=771&width=748&top_left_y=1071&top_left_x=654) Let $P$ be the common point of lines $E F$ and $t$. We are going to prove that $P$ lies on line $A O_{1}$. Denote by $N$ the second common point of circle $\\omega_{1}$ and $A C$; this is the point of $\\omega_{1}$ diametrically opposite to $D$. By Menelaus' theorem for triangle $D C N$, points $A, O_{1}$ and $P$ are collinear if and only if $$ \\frac{C A}{A N} \\cdot \\frac{N O_{1}}{O_{1} D} \\cdot \\frac{D P}{P C}=1 $$ Because $N O_{1}=O_{1} D$, this reduces to $C A \/ A N=C P \/ P D$. Let line $t$ meet $A B$ at $K$. Then $C A \/ A N=C K \/ K D$, so it suffices to show that $$ \\frac{C P}{P D}=\\frac{C K}{K D} $$ To verify (1), consider the circumcircle $\\Omega$ of triangle $A B C$. Draw its diameter $C U$ through $C$, and let $C U$ meet $A B$ at $V$. Extend $C K$ to meet $\\Omega$ at $L$. Since $A B$ is parallel to $U L$, we have $\\angle A C U=\\angle B C L$. On the other hand $\\angle E F C=\\angle B A C, \\angle F E C=\\angle A B C$ and $E F \/ A B=\\cos \\gamma$, as stated above. So reflection in the bisector of $\\angle A C B$ followed by a homothety with centre $C$ and ratio $1 \/ \\cos \\gamma$ takes triangle $F E C$ to triangle $A B C$. Consequently, this transformation takes $C D$ to $C U$, which implies $C P \/ P D=C V \/ V U$. Next, we have $K L=K D$, because $D$ is the orthocentre of triangle $A B C$. Hence $C K \/ K D=C K \/ K L$. Finally, $C V \/ V U=C K \/ K L$ because $A B$ is parallel to $U L$. Relation (1) follows, proving that $P$ lies on line $A O_{1}$. By symmetry, $P$ also lies on line $A O_{2}$ which completes the solution.","tier":0} +{"problem_type":"Geometry","problem_label":"G6","problem":"Circles $\\omega_{1}$ and $\\omega_{2}$ with centres $O_{1}$ and $O_{2}$ are externally tangent at point $D$ and internally tangent to a circle $\\omega$ at points $E$ and $F$, respectively. Line $t$ is the common tangent of $\\omega_{1}$ and $\\omega_{2}$ at $D$. Let $A B$ be the diameter of $\\omega$ perpendicular to $t$, so that $A, E$ and $O_{1}$ are on the same side of $t$. Prove that lines $A O_{1}, B O_{2}, E F$ and $t$ are concurrent. (Brasil)","solution":"We proceed as in the first solution to define a triangle $A B C$ with orthocentre $D$, in which $A F$ and $B E$ are altitudes. Denote by $M$ the midpoint of $C D$. The quadrilateral $C E D F$ is inscribed in a circle with centre $M$, hence $M C=M E=M D=M F$. ![](https:\/\/cdn.mathpix.com\/cropped\/2024_04_17_e55ccc62f43b046023cag-44.jpg?height=723&width=849&top_left_y=752&top_left_x=612) Consider triangles $A B C$ and $O_{1} O_{2} M$. Lines $O_{1} O_{2}$ and $A B$ are parallel, both of them being perpendicular to line $t$. Next, $M O_{1}$ is the line of centres of circles $(C E F)$ and $\\omega_{1}$ whose common chord is $D E$. Hence $M O_{1}$ bisects $\\angle D M E$ which is the external angle at $M$ in the isosceles triangle $C E M$. It follows that $\\angle D M O_{1}=\\angle D C A$, so that $M O_{1}$ is parallel to $A C$. Likewise, $M O_{2}$ is parallel to $B C$. Thus the respective sides of triangles $A B C$ and $O_{1} O_{2} M$ are parallel; in addition, these triangles are not congruent. Hence there is a homothety taking $A B C$ to $O_{1} O_{2} M$. The lines $A O_{1}$, $B O_{2}$ and $C M=t$ are concurrent at the centre $Q$ of this homothety. Finally, apply Pappus' theorem to the triples of collinear points $A, O, B$ and $O_{2}, D, O_{1}$. The theorem implies that the points $A D \\cap O O_{2}=F, A O_{1} \\cap B O_{2}=Q$ and $O O_{1} \\cap B D=E$ are collinear. In other words, line $E F$ passes through the common point $Q$ of $A O_{1}, B O_{2}$ and $t$. Comment. Relation (1) from Solution 1 expresses the well-known fact that points $P$ and $K$ are harmonic conjugates with respect to points $C$ and $D$. It is also easy to justify it by direct computation. Denoting $\\angle C A B=\\alpha, \\angle A B C=\\beta$, it is straightforward to obtain $C P \/ P D=C K \/ K D=\\tan \\alpha \\tan \\beta$.","tier":0} +{"problem_type":"Geometry","problem_label":"G7","problem":"In a triangle $A B C$, let $M_{a}, M_{b}, M_{c}$ be respectively the midpoints of the sides $B C, C A$, $A B$ and $T_{a}, T_{b}, T_{c}$ be the midpoints of the $\\operatorname{arcs} B C, C A, A B$ of the circumcircle of $A B C$, not containing the opposite vertices. For $i \\in\\{a, b, c\\}$, let $\\omega_{i}$ be the circle with $M_{i} T_{i}$ as diameter. Let $p_{i}$ be the common external tangent to $\\omega_{j}, \\omega_{k}(\\{i, j, k\\}=\\{a, b, c\\})$ such that $\\omega_{i}$ lies on the opposite side of $p_{i}$ than $\\omega_{j}, \\omega_{k}$ do. Prove that the lines $p_{a}, p_{b}, p_{c}$ form a triangle similar to $A B C$ and find the ratio of similitude. (Slovakia)","solution":"Let $T_{a} T_{b}$ intersect circle $\\omega_{b}$ at $T_{b}$ and $U$, and let $T_{a} T_{c}$ intersect circle $\\omega_{c}$ at $T_{c}$ and $V$. Further, let $U X$ be the tangent to $\\omega_{b}$ at $U$, with $X$ on $A C$, and let $V Y$ be the tangent to $\\omega_{c}$ at $V$, with $Y$ on $A B$. The homothety with centre $T_{b}$ and ratio $T_{b} T_{a} \/ T_{b} U$ maps the circle $\\omega_{b}$ onto the circumcircle of $A B C$ and the line $U X$ onto the line tangent to the circumcircle at $T_{a}$, which is parallel to $B C$; thus $U X \\| B C$. The same is true of $V Y$, so that $U X\\|B C\\| V Y$. Let $T_{a} T_{b}$ cut $A C$ at $P$ and let $T_{a} T_{c}$ cut $A B$ at $Q$. The point $X$ lies on the hypotenuse $P M_{b}$ of the right triangle $P U M_{b}$ and is equidistant from $U$ and $M_{b}$. So $X$ is the midpoint of $M_{b} P$. Similarly $Y$ is the midpoint of $M_{c} Q$. Denote the incentre of triangle $A B C$ as usual by $I$. It is a known fact that $T_{a} I=T_{a} B$ and $T_{c} I=T_{c} B$. Therefore the points $B$ and $I$ are symmetric across $T_{a} T_{c}$, and consequently $\\angle Q I B=\\angle Q B I=\\angle I B C$. This implies that $B C$ is parallel to the line $I Q$, and likewise, to $I P$. In other words, $P Q$ is the line parallel to $B C$ passing through $I$. ![](https:\/\/cdn.mathpix.com\/cropped\/2024_04_17_e55ccc62f43b046023cag-46.jpg?height=868&width=874&top_left_y=1328&top_left_x=594) Clearly $M_{b} M_{c} \\| B C$. So $P M_{b} M_{c} Q$ is a trapezoid and the segment $X Y$ connects the midpoints of its nonparallel sides; hence $X Y \\| B C$. This combined with the previously established relations $U X\\|B C\\| V Y$ shows that all the four points $U, X, Y, V$ lie on a line which is the common tangent to circles $\\omega_{b}, \\omega_{c}$. Since it leaves these two circles on one side and the circle $\\omega_{a}$ on the other, this line is just the line $p_{a}$ from the problem statement. Line $p_{a}$ runs midway between $I$ and $M_{b} M_{c}$. Analogous conclusions hold for the lines $p_{b}$ and $p_{c}$. So these three lines form a triangle homothetic from centre $I$ to triangle $M_{a} M_{b} M_{c}$ in ratio $1 \/ 2$, hence similar to $A B C$ in ratio $1 \/ 4$.","tier":0} +{"problem_type":"Geometry","problem_label":"G8","problem":"Let $A B C D$ be a convex quadrilateral. A circle passing through the points $A$ and $D$ and a circle passing through the points $B$ and $C$ are externally tangent at a point $P$ inside the quadrilateral. Suppose that $$ \\angle P A B+\\angle P D C \\leq 90^{\\circ} \\quad \\text { and } \\quad \\angle P B A+\\angle P C D \\leq 90^{\\circ} \\text {. } $$ Prove that $A B+C D \\geq B C+A D$. (Poland)","solution":"We start with a preliminary observation. Let $T$ be a point inside the quadrilateral $A B C D$. Then: $$ \\begin{aligned} & \\text { Circles }(B C T) \\text { and }(D A T) \\text { are tangent at } T \\\\ & \\text { if and only if } \\quad \\angle A D T+\\angle B C T=\\angle A T B \\text {. } \\end{aligned} $$ Indeed, if the two circles touch each other then their common tangent at $T$ intersects the segment $A B$ at a point $Z$, and so $\\angle A D T=\\angle A T Z, \\angle B C T=\\angle B T Z$, by the tangent-chord theorem. Thus $\\angle A D T+\\angle B C T=\\angle A T Z+\\angle B T Z=\\angle A T B$. And conversely, if $\\angle A D T+\\angle B C T=\\angle A T B$ then one can draw from $T$ a ray $T Z$ with $Z$ on $A B$ so that $\\angle A D T=\\angle A T Z, \\angle B C T=\\angle B T Z$. The first of these equalities implies that $T Z$ is tangent to the circle $(D A T)$; by the second equality, $T Z$ is tangent to the circle $(B C T)$, so the two circles are tangent at $T$. ![](https:\/\/cdn.mathpix.com\/cropped\/2024_04_17_e55ccc62f43b046023cag-47.jpg?height=480&width=805&top_left_y=1319&top_left_x=634) So the equivalence (1) is settled. It will be used later on. Now pass to the actual solution. Its key idea is to introduce the circumcircles of triangles $A B P$ and $C D P$ and to consider their second intersection $Q$ (assume for the moment that they indeed meet at two distinct points $P$ and $Q$ ). Since the point $A$ lies outside the circle $(B C P)$, we have $\\angle B C P+\\angle B A P<180^{\\circ}$. Therefore the point $C$ lies outside the circle $(A B P)$. Analogously, $D$ also lies outside that circle. It follows that $P$ and $Q$ lie on the same $\\operatorname{arc} C D$ of the circle $(B C P)$. ![](https:\/\/cdn.mathpix.com\/cropped\/2024_04_17_e55ccc62f43b046023cag-47.jpg?height=466&width=606&top_left_y=2257&top_left_x=728) By symmetry, $P$ and $Q$ lie on the same arc $A B$ of the circle $(A B P)$. Thus the point $Q$ lies either inside the angle $B P C$ or inside the angle $A P D$. Without loss of generality assume that $Q$ lies inside the angle $B P C$. Then $$ \\angle A Q D=\\angle P Q A+\\angle P Q D=\\angle P B A+\\angle P C D \\leq 90^{\\circ} \\text {, } $$ by the condition of the problem. In the cyclic quadrilaterals $A P Q B$ and $D P Q C$, the angles at vertices $A$ and $D$ are acute. So their angles at $Q$ are obtuse. This implies that $Q$ lies not only inside the angle $B P C$ but in fact inside the triangle $B P C$, hence also inside the quadrilateral $A B C D$. Now an argument similar to that used in deriving (2) shows that $$ \\angle B Q C=\\angle P A B+\\angle P D C \\leq 90^{\\circ} . $$ Moreover, since $\\angle P C Q=\\angle P D Q$, we get $$ \\angle A D Q+\\angle B C Q=\\angle A D P+\\angle P D Q+\\angle B C P-\\angle P C Q=\\angle A D P+\\angle B C P . $$ The last sum is equal to $\\angle A P B$, according to the observation (1) applied to $T=P$. And because $\\angle A P B=\\angle A Q B$, we obtain $$ \\angle A D Q+\\angle B C Q=\\angle A Q B \\text {. } $$ Applying now (1) to $T=Q$ we conclude that the circles $(B C Q)$ and $(D A Q)$ are externally tangent at $Q$. (We have assumed $P \\neq Q$; but if $P=Q$ then the last conclusion holds trivially.) Finally consider the halfdiscs with diameters $B C$ and $D A$ constructed inwardly to the quadrilateral $A B C D$. They have centres at $M$ and $N$, the midpoints of $B C$ and $D A$ respectively. In view of (2) and (3), these two halfdiscs lie entirely inside the circles $(B Q C)$ and $(A Q D)$; and since these circles are tangent, the two halfdiscs cannot overlap. Hence $M N \\geq \\frac{1}{2} B C+\\frac{1}{2} D A$. On the other hand, since $\\overrightarrow{M N}=\\frac{1}{2}(\\overrightarrow{B A}+\\overrightarrow{C D})$, we have $M N \\leq \\frac{1}{2}(A B+C D)$. Thus indeed $A B+C D \\geq B C+D A$, as claimed.","tier":0} +{"problem_type":"Geometry","problem_label":"G9","problem":"Points $A_{1}, B_{1}, C_{1}$ are chosen on the sides $B C, C A, A B$ of a triangle $A B C$, respectively. The circumcircles of triangles $A B_{1} C_{1}, B C_{1} A_{1}, C A_{1} B_{1}$ intersect the circumcircle of triangle $A B C$ again at points $A_{2}, B_{2}, C_{2}$, respectively $\\left(A_{2} \\neq A, B_{2} \\neq B, C_{2} \\neq C\\right)$. Points $A_{3}, B_{3}, C_{3}$ are symmetric to $A_{1}, B_{1}, C_{1}$ with respect to the midpoints of the sides $B C, C A, A B$ respectively. Prove that the triangles $A_{2} B_{2} C_{2}$ and $A_{3} B_{3} C_{3}$ are similar. (Russia)","solution":"We will work with oriented angles between lines. For two straight lines $\\ell, m$ in the plane, $\\angle(\\ell, m)$ denotes the angle of counterclockwise rotation which transforms line $\\ell$ into a line parallel to $m$ (the choice of the rotation centre is irrelevant). This is a signed quantity; values differing by a multiple of $\\pi$ are identified, so that $$ \\angle(\\ell, m)=-\\angle(m, \\ell), \\quad \\angle(\\ell, m)+\\angle(m, n)=\\angle(\\ell, n) . $$ If $\\ell$ is the line through points $K, L$ and $m$ is the line through $M, N$, one writes $\\angle(K L, M N)$ for $\\angle(\\ell, m)$; the characters $K, L$ are freely interchangeable; and so are $M, N$. The counterpart of the classical theorem about cyclic quadrilaterals is the following: If $K, L, M, N$ are four noncollinear points in the plane then $$ K, L, M, N \\text { are concyclic if and only if } \\angle(K M, L M)=\\angle(K N, L N) \\text {. } $$ Passing to the solution proper, we first show that the three circles $\\left(A B_{1} C_{1}\\right),\\left(B C_{1} A_{1}\\right)$, $\\left(C A_{1} B_{1}\\right)$ have a common point. So, let $\\left(A B_{1} C_{1}\\right)$ and $\\left(B C_{1} A_{1}\\right)$ intersect at the points $C_{1}$ and $P$. Then by (1) $$ \\begin{gathered} \\angle\\left(P A_{1}, C A_{1}\\right)=\\angle\\left(P A_{1}, B A_{1}\\right)=\\angle\\left(P C_{1}, B C_{1}\\right) \\\\ =\\angle\\left(P C_{1}, A C_{1}\\right)=\\angle\\left(P B_{1}, A B_{1}\\right)=\\angle\\left(P B_{1}, C B_{1}\\right) \\end{gathered} $$ Denote this angle by $\\varphi$. The equality between the outer terms shows, again by (1), that the points $A_{1}, B_{1}, P, C$ are concyclic. Thus $P$ is the common point of the three mentioned circles. From now on the basic property (1) will be used without explicit reference. We have $$ \\varphi=\\angle\\left(P A_{1}, B C\\right)=\\angle\\left(P B_{1}, C A\\right)=\\angle\\left(P C_{1}, A B\\right) . $$ ![](https:\/\/cdn.mathpix.com\/cropped\/2024_04_17_e55ccc62f43b046023cag-49.jpg?height=778&width=1620&top_left_y=1938&top_left_x=226) Let lines $A_{2} P, B_{2} P, C_{2} P$ meet the circle $(A B C)$ again at $A_{4}, B_{4}, C_{4}$, respectively. As $$ \\angle\\left(A_{4} A_{2}, A A_{2}\\right)=\\angle\\left(P A_{2}, A A_{2}\\right)=\\angle\\left(P C_{1}, A C_{1}\\right)=\\angle\\left(P C_{1}, A B\\right)=\\varphi \\text {, } $$ we see that line $A_{2} A$ is the image of line $A_{2} A_{4}$ under rotation about $A_{2}$ by the angle $\\varphi$. Hence the point $A$ is the image of $A_{4}$ under rotation by $2 \\varphi$ about $O$, the centre of $(A B C)$. The same rotation sends $B_{4}$ to $B$ and $C_{4}$ to $C$. Triangle $A B C$ is the image of $A_{4} B_{4} C_{4}$ in this map. Thus $$ \\angle\\left(A_{4} B_{4}, A B\\right)=\\angle\\left(B_{4} C_{4}, B C\\right)=\\angle\\left(C_{4} A_{4}, C A\\right)=2 \\varphi \\text {. } $$ Since the rotation by $2 \\varphi$ about $O$ takes $B_{4}$ to $B$, we have $\\angle\\left(A B_{4}, A B\\right)=\\varphi$. Hence by (2) $$ \\angle\\left(A B_{4}, P C_{1}\\right)=\\angle\\left(A B_{4}, A B\\right)+\\angle\\left(A B, P C_{1}\\right)=\\varphi+(-\\varphi)=0 $$ which means that $A B_{4} \\| P C_{1}$. ![](https:\/\/cdn.mathpix.com\/cropped\/2024_04_17_e55ccc62f43b046023cag-50.jpg?height=784&width=1682&top_left_y=978&top_left_x=197) Let $C_{5}$ be the intersection of lines $P C_{1}$ and $A_{4} B_{4}$; define $A_{5}, B_{5}$ analogously. So $A B_{4} \\| C_{1} C_{5}$ and, by (3) and (2), $$ \\angle\\left(A_{4} B_{4}, P C_{1}\\right)=\\angle\\left(A_{4} B_{4}, A B\\right)+\\angle\\left(A B, P C_{1}\\right)=2 \\varphi+(-\\varphi)=\\varphi $$ i.e., $\\angle\\left(B_{4} C_{5}, C_{5} C_{1}\\right)=\\varphi$. This combined with $\\angle\\left(C_{5} C_{1}, C_{1} A\\right)=\\angle\\left(P C_{1}, A B\\right)=\\varphi$ (see (2)) proves that the quadrilateral $A B_{4} C_{5} C_{1}$ is an isosceles trapezoid with $A C_{1}=B_{4} C_{5}$. Interchanging the roles of $A$ and $B$ we infer that also $B C_{1}=A_{4} C_{5}$. And since $A C_{1}+B C_{1}=$ $A B=A_{4} B_{4}$, it follows that the point $C_{5}$ lies on the line segment $A_{4} B_{4}$ and partitions it into segments $A_{4} C_{5}, B_{4} C_{5}$ of lengths $B C_{1}\\left(=A C_{3}\\right)$ and $A C_{1}\\left(=B C_{3}\\right)$. In other words, the rotation which maps triangle $A_{4} B_{4} C_{4}$ onto $A B C$ carries $C_{5}$ onto $C_{3}$. Likewise, it sends $A_{5}$ to $A_{3}$ and $B_{5}$ to $B_{3}$. So the triangles $A_{3} B_{3} C_{3}$ and $A_{5} B_{5} C_{5}$ are congruent. It now suffices to show that the latter is similar to $A_{2} B_{2} C_{2}$. Lines $B_{4} C_{5}$ and $P C_{5}$ coincide respectively with $A_{4} B_{4}$ and $P C_{1}$. Thus by (4) $$ \\angle\\left(B_{4} C_{5}, P C_{5}\\right)=\\varphi $$ Analogously (by cyclic shift) $\\varphi=\\angle\\left(C_{4} A_{5}, P A_{5}\\right)$, which rewrites as $$ \\varphi=\\angle\\left(B_{4} A_{5}, P A_{5}\\right) $$ These relations imply that the points $P, B_{4}, C_{5}, A_{5}$ are concyclic. Analogously, $P, C_{4}, A_{5}, B_{5}$ and $P, A_{4}, B_{5}, C_{5}$ are concyclic quadruples. Therefore $$ \\angle\\left(A_{5} B_{5}, C_{5} B_{5}\\right)=\\angle\\left(A_{5} B_{5}, P B_{5}\\right)+\\angle\\left(P B_{5}, C_{5} B_{5}\\right)=\\angle\\left(A_{5} C_{4}, P C_{4}\\right)+\\angle\\left(P A_{4}, C_{5} A_{4}\\right) . $$ On the other hand, since the points $A_{2}, B_{2}, C_{2}, A_{4}, B_{4}, C_{4}$ all lie on the circle $(A B C)$, we have $$ \\angle\\left(A_{2} B_{2}, C_{2} B_{2}\\right)=\\angle\\left(A_{2} B_{2}, B_{4} B_{2}\\right)+\\angle\\left(B_{4} B_{2}, C_{2} B_{2}\\right)=\\angle\\left(A_{2} A_{4}, B_{4} A_{4}\\right)+\\angle\\left(B_{4} C_{4}, C_{2} C_{4}\\right) . $$ But the lines $A_{2} A_{4}, B_{4} A_{4}, B_{4} C_{4}, C_{2} C_{4}$ coincide respectively with $P A_{4}, C_{5} A_{4}, A_{5} C_{4}, P C_{4}$. So the sums on the right-hand sides of (5) and (6) are equal, leading to equality between their left-hand sides: $\\angle\\left(A_{5} B_{5}, C_{5} B_{5}\\right)=\\angle\\left(A_{2} B_{2}, C_{2} B_{2}\\right)$. Hence (by cyclic shift, once more) also $\\angle\\left(B_{5} C_{5}, A_{5} C_{5}\\right)=\\angle\\left(B_{2} C_{2}, A_{2} C_{2}\\right)$ and $\\angle\\left(C_{5} A_{5}, B_{5} A_{5}\\right)=\\angle\\left(C_{2} A_{2}, B_{2} A_{2}\\right)$. This means that the triangles $A_{5} B_{5} C_{5}$ and $A_{2} B_{2} C_{2}$ have their corresponding angles equal, and consequently they are similar. Comment 1. This is the way in which the proof has been presented by the proposer. Trying to work it out in the language of classical geometry, so as to avoid oriented angles, one is led to difficulties due to the fact that the reasoning becomes heavily case-dependent. Disposition of relevant points can vary in many respects. Angles which are equal in one case become supplementary in another. Although it seems not hard to translate all formulas from the shapes they have in one situation to the one they have in another, the real trouble is to identify all cases possible and rigorously verify that the key conclusions retain validity in each case. The use of oriented angles is a very efficient method to omit this trouble. It seems to be the most appropriate environment in which the solution can be elaborated. Comment 2. Actually, the fact that the circles $\\left(A B_{1} C_{1}\\right),\\left(B C_{1} A_{1}\\right)$ and $\\left(C A_{1} B_{1}\\right)$ have a common point does not require a proof; it is known as Miquel's theorem.","tier":0} +{"problem_type":"Geometry","problem_label":"G10","problem":"To each side $a$ of a convex polygon we assign the maximum area of a triangle contained in the polygon and having $a$ as one of its sides. Show that the sum of the areas assigned to all sides of the polygon is not less than twice the area of the polygon. (Serbia)","solution":"Lemma. Every convex $(2 n)$-gon, of area $S$, has a side and a vertex that jointly span a triangle of area not less than $S \/ n$. Proof. By main diagonals of the $(2 n)$-gon we shall mean those which partition the $(2 n)$-gon into two polygons with equally many sides. For any side $b$ of the $(2 n)$-gon denote by $\\Delta_{b}$ the triangle $A B P$ where $A, B$ are the endpoints of $b$ and $P$ is the intersection point of the main diagonals $A A^{\\prime}, B B^{\\prime}$. We claim that the union of triangles $\\Delta_{b}$, taken over all sides, covers the whole polygon. To show this, choose any side $A B$ and consider the main diagonal $A A^{\\prime}$ as a directed segment. Let $X$ be any point in the polygon, not on any main diagonal. For definiteness, let $X$ lie on the left side of the ray $A A^{\\prime}$. Consider the sequence of main diagonals $A A^{\\prime}, B B^{\\prime}, C C^{\\prime}, \\ldots$, where $A, B, C, \\ldots$ are consecutive vertices, situated right to $A A^{\\prime}$. The $n$-th item in this sequence is the diagonal $A^{\\prime} A$ (i.e. $A A^{\\prime}$ reversed), having $X$ on its right side. So there are two successive vertices $K, L$ in the sequence $A, B, C, \\ldots$ before $A^{\\prime}$ such that $X$ still lies to the left of $K K^{\\prime}$ but to the right of $L L^{\\prime}$. And this means that $X$ is in the triangle $\\Delta_{\\ell^{\\prime}}, \\ell^{\\prime}=K^{\\prime} L^{\\prime}$. Analogous reasoning applies to points $X$ on the right of $A A^{\\prime}$ (points lying on main diagonals can be safely ignored). Thus indeed the triangles $\\Delta_{b}$ jointly cover the whole polygon. The sum of their areas is no less than $S$. So we can find two opposite sides, say $b=A B$ and $b^{\\prime}=A^{\\prime} B^{\\prime}$ (with $A A^{\\prime}, B B^{\\prime}$ main diagonals) such that $\\left[\\Delta_{b}\\right]+\\left[\\Delta_{b^{\\prime}}\\right] \\geq S \/ n$, where $[\\cdots]$ stands for the area of a region. Let $A A^{\\prime}, B B^{\\prime}$ intersect at $P$; assume without loss of generality that $P B \\geq P B^{\\prime}$. Then $$ \\left[A B A^{\\prime}\\right]=[A B P]+\\left[P B A^{\\prime}\\right] \\geq[A B P]+\\left[P A^{\\prime} B^{\\prime}\\right]=\\left[\\Delta_{b}\\right]+\\left[\\Delta_{b^{\\prime}}\\right] \\geq S \/ n $$ proving the lemma. Now, let $\\mathcal{P}$ be any convex polygon, of area $S$, with $m$ sides $a_{1}, \\ldots, a_{m}$. Let $S_{i}$ be the area of the greatest triangle in $\\mathcal{P}$ with side $a_{i}$. Suppose, contrary to the assertion, that $$ \\sum_{i=1}^{m} \\frac{S_{i}}{S}<2 $$ Then there exist rational numbers $q_{1}, \\ldots, q_{m}$ such that $\\sum q_{i}=2$ and $q_{i}>S_{i} \/ S$ for each $i$. Let $n$ be a common denominator of the $m$ fractions $q_{1}, \\ldots, q_{m}$. Write $q_{i}=k_{i} \/ n$; so $\\sum k_{i}=2 n$. Partition each side $a_{i}$ of $\\mathcal{P}$ into $k_{i}$ equal segments, creating a convex $(2 n)$-gon of area $S$ (with some angles of size $180^{\\circ}$ ), to which we apply the lemma. Accordingly, this refined polygon has a side $b$ and a vertex $H$ spanning a triangle $T$ of area $[T] \\geq S \/ n$. If $b$ is a piece of a side $a_{i}$ of $\\mathcal{P}$, then the triangle $W$ with base $a_{i}$ and summit $H$ has area $$ [W]=k_{i} \\cdot[T] \\geq k_{i} \\cdot S \/ n=q_{i} \\cdot S>S_{i}, $$ in contradiction with the definition of $S_{i}$. This ends the proof.","tier":0} +{"problem_type":"Geometry","problem_label":"G10","problem":"To each side $a$ of a convex polygon we assign the maximum area of a triangle contained in the polygon and having $a$ as one of its sides. Show that the sum of the areas assigned to all sides of the polygon is not less than twice the area of the polygon. (Serbia)","solution":"As in the first solution, we allow again angles of size $180^{\\circ}$ at some vertices of the convex polygons considered. To each convex $n$-gon $\\mathcal{P}=A_{1} A_{2} \\ldots A_{n}$ we assign a centrally symmetric convex $(2 n)$-gon $\\mathcal{Q}$ with side vectors $\\pm \\overrightarrow{A_{i} A_{i+1}}, 1 \\leq i \\leq n$. The construction is as follows. Attach the $2 n$ vectors $\\pm \\overrightarrow{A_{i} A_{i+1}}$ at a common origin and label them $\\overrightarrow{\\mathbf{b}_{1}}, \\overrightarrow{\\mathbf{b}_{2}}, \\ldots, \\overrightarrow{\\mathbf{b}_{2 n}}$ in counterclockwise direction; the choice of the first vector $\\overrightarrow{\\mathbf{b}_{1}}$ is irrelevant. The order of labelling is well-defined if $\\mathcal{P}$ has neither parallel sides nor angles equal to $180^{\\circ}$. Otherwise several collinear vectors with the same direction are labelled consecutively $\\overrightarrow{\\mathbf{b}_{j}}, \\overrightarrow{\\mathbf{b}_{j+1}}, \\ldots, \\overrightarrow{\\mathbf{b}_{j+r}}$. One can assume that in such cases the respective opposite vectors occur in the order $-\\overrightarrow{\\mathbf{b}_{j}},-\\overrightarrow{\\mathbf{b}_{j+1}}, \\ldots,-\\overrightarrow{\\mathbf{b}_{j+r}}$, ensuring that $\\overrightarrow{\\mathbf{b}_{j+n}}=-\\overrightarrow{\\mathbf{b}_{j}}$ for $j=1, \\ldots, 2 n$. Indices are taken cyclically here and in similar situations below. Choose points $B_{1}, B_{2}, \\ldots, B_{2 n}$ satisfying $\\overrightarrow{B_{j} B_{j+1}}=\\overrightarrow{\\mathbf{b}_{j}}$ for $j=1, \\ldots, 2 n$. The polygonal line $\\mathcal{Q}=B_{1} B_{2} \\ldots B_{2 n}$ is closed, since $\\sum_{j=1}^{2 n} \\overrightarrow{\\mathbf{b}_{j}}=\\overrightarrow{0}$. Moreover, $\\mathcal{Q}$ is a convex $(2 n)$-gon due to the arrangement of the vectors $\\overrightarrow{\\mathbf{b}_{j}}$, possibly with $180^{\\circ}$-angles. The side vectors of $\\mathcal{Q}$ are $\\pm \\overrightarrow{A_{i} A_{i+1}}$, $1 \\leq i \\leq n$. So in particular $\\mathcal{Q}$ is centrally symmetric, because it contains as side vectors $\\overrightarrow{A_{i} A_{i+1}}$ and $-\\overrightarrow{A_{i} A_{i+1}}$ for each $i=1, \\ldots, n$. Note that $B_{j} B_{j+1}$ and $B_{j+n} B_{j+n+1}$ are opposite sides of $\\mathcal{Q}$, $1 \\leq j \\leq n$. We call $\\mathcal{Q}$ the associate of $\\mathcal{P}$. Let $S_{i}$ be the maximum area of a triangle with side $A_{i} A_{i+1}$ in $\\mathcal{P}, 1 \\leq i \\leq n$. We prove that $$ \\left[B_{1} B_{2} \\ldots B_{2 n}\\right]=2 \\sum_{i=1}^{n} S_{i} $$ and $$ \\left[B_{1} B_{2} \\ldots B_{2 n}\\right] \\geq 4\\left[A_{1} A_{2} \\ldots A_{n}\\right] $$ It is clear that (1) and (2) imply the conclusion of the original problem. Lemma. For a side $A_{i} A_{i+1}$ of $\\mathcal{P}$, let $h_{i}$ be the maximum distance from a point of $\\mathcal{P}$ to line $A_{i} A_{i+1}$, $i=1, \\ldots, n$. Denote by $B_{j} B_{j+1}$ the side of $\\mathcal{Q}$ such that $\\overrightarrow{A_{i} A_{i+1}}=\\overrightarrow{B_{j} B_{j+1}}$. Then the distance between $B_{j} B_{j+1}$ and its opposite side in $\\mathcal{Q}$ is equal to $2 h_{i}$. Proof. Choose a vertex $A_{k}$ of $\\mathcal{P}$ at distance $h_{i}$ from line $A_{i} A_{i+1}$. Let $\\mathbf{u}$ be the unit vector perpendicular to $A_{i} A_{i+1}$ and pointing inside $\\mathcal{P}$. Denoting by $\\mathbf{x} \\cdot \\mathbf{y}$ the dot product of vectors $\\mathbf{x}$ and $\\mathbf{y}$, we have $$ h=\\mathbf{u} \\cdot \\overrightarrow{A_{i} A_{k}}=\\mathbf{u} \\cdot\\left(\\overrightarrow{A_{i} A_{i+1}}+\\cdots+\\overrightarrow{A_{k-1} A_{k}}\\right)=\\mathbf{u} \\cdot\\left(\\overrightarrow{A_{i} A_{i-1}}+\\cdots+\\overrightarrow{A_{k+1} A_{k}}\\right) . $$ In $\\mathcal{Q}$, the distance $H_{i}$ between the opposite sides $B_{j} B_{j+1}$ and $B_{j+n} B_{j+n+1}$ is given by $$ H_{i}=\\mathbf{u} \\cdot\\left(\\overrightarrow{B_{j} B_{j+1}}+\\cdots+\\overrightarrow{B_{j+n-1} B_{j+n}}\\right)=\\mathbf{u} \\cdot\\left(\\overrightarrow{\\mathbf{b}_{j}}+\\overrightarrow{\\mathbf{b}_{j+1}}+\\cdots+\\overrightarrow{\\mathbf{b}_{j+n-1}}\\right) . $$ The choice of vertex $A_{k}$ implies that the $n$ consecutive vectors $\\overrightarrow{\\mathbf{b}_{j}}, \\overrightarrow{\\mathbf{b}_{j+1}}, \\ldots, \\overrightarrow{\\mathbf{b}_{j+n-1}}$ are precisely $\\overrightarrow{A_{i} A_{i+1}}, \\ldots, \\overrightarrow{A_{k-1} A_{k}}$ and $\\overrightarrow{A_{i} A_{i-1}}, \\ldots, \\overrightarrow{A_{k+1} A_{k}}$, taken in some order. This implies $H_{i}=2 h_{i}$. For a proof of (1), apply the lemma to each side of $\\mathcal{P}$. If $O$ the centre of $\\mathcal{Q}$ then, using the notation of the lemma, $$ \\left[B_{j} B_{j+1} O\\right]=\\left[B_{j+n} B_{j+n+1} O\\right]=\\left[A_{i} A_{i+1} A_{k}\\right]=S_{i} $$ Summation over all sides of $\\mathcal{P}$ yields (1). Set $d(\\mathcal{P})=[\\mathcal{Q}]-4[\\mathcal{P}]$ for a convex polygon $\\mathcal{P}$ with associate $\\mathcal{Q}$. Inequality $(2)$ means that $d(\\mathcal{P}) \\geq 0$ for each convex polygon $\\mathcal{P}$. The last inequality will be proved by induction on the number $\\ell$ of side directions of $\\mathcal{P}$, i. e. the number of pairwise nonparallel lines each containing a side of $\\mathcal{P}$. We choose to start the induction with $\\ell=1$ as a base case, meaning that certain degenerate polygons are allowed. More exactly, we regard as degenerate convex polygons all closed polygonal lines of the form $X_{1} X_{2} \\ldots X_{k} Y_{1} Y_{2} \\ldots Y_{m} X_{1}$, where $X_{1}, X_{2}, \\ldots, X_{k}$ are points in this order on a line segment $X_{1} Y_{1}$, and so are $Y_{m}, Y_{m-1}, \\ldots, Y_{1}$. The initial construction applies to degenerate polygons; their associates are also degenerate, and the value of $d$ is zero. For the inductive step, consider a convex polygon $\\mathcal{P}$ which determines $\\ell$ side directions, assuming that $d(\\mathcal{P}) \\geq 0$ for polygons with smaller values of $\\ell$. Suppose first that $\\mathcal{P}$ has a pair of parallel sides, i. e. sides on distinct parallel lines. Let $A_{i} A_{i+1}$ and $A_{j} A_{j+1}$ be such a pair, and let $A_{i} A_{i+1} \\leq A_{j} A_{j+1}$. Remove from $\\mathcal{P}$ the parallelogram $R$ determined by vectors $\\overrightarrow{A_{i} A_{i+1}}$ and $\\overrightarrow{A_{i} A_{j+1}}$. Two polygons are obtained in this way. Translating one of them by vector $\\overrightarrow{A_{i} A_{i+1}}$ yields a new convex polygon $\\mathcal{P}^{\\prime}$, of area $[\\mathcal{P}]-[R]$ and with value of $\\ell$ not exceeding the one of $\\mathcal{P}$. The construction just described will be called operation A. ![](https:\/\/cdn.mathpix.com\/cropped\/2024_04_17_e55ccc62f43b046023cag-54.jpg?height=464&width=1440&top_left_y=1098&top_left_x=316) The associate of $\\mathcal{P}^{\\prime}$ is obtained from $\\mathcal{Q}$ upon decreasing the lengths of two opposite sides by an amount of $2 A_{i} A_{i+1}$. By the lemma, the distance between these opposite sides is twice the distance between $A_{i} A_{i+1}$ and $A_{j} A_{j+1}$. Thus operation $\\mathbf{A}$ decreases $[\\mathcal{Q}]$ by the area of a parallelogram with base and respective altitude twice the ones of $R$, i. e. by $4[R]$. Hence $\\mathbf{A}$ leaves the difference $d(\\mathcal{P})=[\\mathcal{Q}]-4[\\mathcal{P}]$ unchanged. Now, if $\\mathcal{P}^{\\prime}$ also has a pair of parallel sides, apply operation $\\mathbf{A}$ to it. Keep doing so with the subsequent polygons obtained for as long as possible. Now, A decreases the number $p$ of pairs of parallel sides in $\\mathcal{P}$. Hence its repeated applications gradually reduce $p$ to 0 , and further applications of $\\mathbf{A}$ will be impossible after several steps. For clarity, let us denote by $\\mathcal{P}$ again the polygon obtained at that stage. The inductive step is complete if $\\mathcal{P}$ is degenerate. Otherwise $\\ell>1$ and $p=0$, i. e. there are no parallel sides in $\\mathcal{P}$. Observe that then $\\ell \\geq 3$. Indeed, $\\ell=2$ means that the vertices of $\\mathcal{P}$ all lie on the boundary of a parallelogram, implying $p>0$. Furthermore, since $\\mathcal{P}$ has no parallel sides, consecutive collinear vectors in the sequence $\\left(\\overrightarrow{\\mathbf{b}_{k}}\\right)$ (if any) correspond to consecutive $180^{\\circ}$-angles in $\\mathcal{P}$. Removing the vertices of such angles, we obtain a convex polygon with the same value of $d(\\mathcal{P})$. In summary, if operation $\\mathbf{A}$ is impossible for a nondegenerate polygon $\\mathcal{P}$, then $\\ell \\geq 3$. In addition, one may assume that $\\mathcal{P}$ has no angles of size $180^{\\circ}$. The last two conditions then also hold for the associate $\\mathcal{Q}$ of $\\mathcal{P}$, and we perform the following construction. Since $\\ell \\geq 3$, there is a side $B_{j} B_{j+1}$ of $\\mathcal{Q}$ such that the sum of the angles at $B_{j}$ and $B_{j+1}$ is greater than $180^{\\circ}$. (Such a side exists in each convex $k$-gon for $k>4$.) Naturally, $B_{j+n} B_{j+n+1}$ is a side with the same property. Extend the pairs of sides $B_{j-1} B_{j}, B_{j+1} B_{j+2}$ and $B_{j+n-1} B_{j+n}, B_{j+n+1} B_{j+n+2}$ to meet at $U$ and $V$, respectively. Let $\\mathcal{Q}^{\\prime}$ be the centrally symmetric convex $2(n+1)$-gon obtained from $\\mathcal{Q}$ by inserting $U$ and $V$ into the sequence $B_{1}, \\ldots, B_{2 n}$ as new vertices between $B_{j}, B_{j+1}$ and $B_{j+n}, B_{j+n+1}$, respectively. Informally, we adjoin to $\\mathcal{Q}$ the congruent triangles $B_{j} B_{j+1} U$ and $B_{j+n} B_{j+n+1} V$. Note that $B_{j}, B_{j+1}, B_{j+n}$ and $B_{j+n+1}$ are kept as vertices of $\\mathcal{Q}^{\\prime}$, although $B_{j} B_{j+1}$ and $B_{j+n} B_{j+n+1}$ are no longer its sides. Let $A_{i} A_{i+1}$ be the side of $\\mathcal{P}$ such that $\\overrightarrow{A_{i} A_{i+1}}=\\overrightarrow{B_{j} B_{j+1}}=\\overrightarrow{\\mathbf{b}_{j}}$. Consider the point $W$ such that triangle $A_{i} A_{i+1} W$ is congruent to triangle $B_{j} B_{j+1} U$ and exterior to $\\mathcal{P}$. Insert $W$ into the sequence $A_{1}, A_{2}, \\ldots, A_{n}$ as a new vertex between $A_{i}$ and $A_{i+1}$ to obtain an $(n+1)$-gon $\\mathcal{P}^{\\prime}$. We claim that $\\mathcal{P}^{\\prime}$ is convex and its associate is $\\mathcal{Q}^{\\prime}$. ![](https:\/\/cdn.mathpix.com\/cropped\/2024_04_17_e55ccc62f43b046023cag-55.jpg?height=532&width=1388&top_left_y=796&top_left_x=344) Vectors $\\overrightarrow{A_{i} W}$ and $\\overrightarrow{\\mathbf{b}_{j-1}}$ are collinear and have the same direction, as well as vectors $\\overrightarrow{W A_{i+1}}$ and $\\overrightarrow{\\mathbf{b}_{j+1}}$. Since $\\overrightarrow{\\mathbf{b}_{j-1}}, \\overrightarrow{\\mathbf{b}_{j}}, \\overrightarrow{\\mathbf{b}_{j+1}}$ are consecutive terms in the sequence $\\left(\\overrightarrow{\\mathbf{b}_{k}}\\right)$, the angle inequalities $\\angle\\left(\\overrightarrow{\\mathbf{b}_{j-1}}, \\overrightarrow{\\mathbf{b}_{j}}\\right) \\leq \\angle\\left(\\overrightarrow{A_{i-1} A_{i}}, \\overrightarrow{\\mathbf{b}_{j}}\\right)$ and $\\angle\\left(\\overrightarrow{\\mathbf{b}_{j}}, \\overrightarrow{\\mathbf{b}_{j+1}}\\right) \\leq \\angle\\left(\\overrightarrow{\\mathbf{b}_{j}}, \\overrightarrow{A_{i+1} A_{i+2}}\\right)$ hold true. They show that $\\mathcal{P}^{\\prime}$ is a convex polygon. To construct its associate, vectors $\\pm \\overrightarrow{A_{i} A_{i+1}}= \\pm \\overrightarrow{\\mathbf{b}_{j}}$ must be deleted from the defining sequence $\\left(\\overrightarrow{\\mathbf{b}_{k}}\\right.$ ) of $\\mathcal{Q}$, and the vectors $\\pm \\overrightarrow{A_{i} W}, \\pm \\overrightarrow{W A_{i+1}}$ must be inserted appropriately into it. The latter can be done as follows: $$ \\ldots, \\overrightarrow{\\mathbf{b}_{j-1}}, \\overrightarrow{A_{i} W}, \\overrightarrow{W A_{i+1}}, \\overrightarrow{\\mathbf{b}_{j+1}}, \\ldots,-\\overrightarrow{\\mathbf{b}_{j-1}},-\\overrightarrow{A_{i} W},-\\overrightarrow{W A_{i+1}},-\\overrightarrow{\\mathbf{b}_{j+1}}, \\ldots $$ This updated sequence produces $\\mathcal{Q}^{\\prime}$ as the associate of $\\mathcal{P}^{\\prime}$. It follows from the construction that $\\left[\\mathcal{P}^{\\prime}\\right]=[\\mathcal{P}]+\\left[A_{i} A_{i+1} W\\right]$ and $\\left[\\mathcal{Q}^{\\prime}\\right]=[\\mathcal{Q}]+2\\left[A_{i} A_{i+1} W\\right]$. Therefore $d\\left(\\mathcal{P}^{\\prime}\\right)=d(\\mathcal{P})-2\\left[A_{i} A_{i+1} W\\right]0$; without loss of generality confine attention to $y>0$. The equation rewritten as $$ 2^{x}\\left(1+2^{x+1}\\right)=(y-1)(y+1) $$ shows that the factors $y-1$ and $y+1$ are even, exactly one of them divisible by 4 . Hence $x \\geq 3$ and one of these factors is divisible by $2^{x-1}$ but not by $2^{x}$. So $$ y=2^{x-1} m+\\epsilon, \\quad m \\text { odd }, \\quad \\epsilon= \\pm 1 $$ Plugging this into the original equation we obtain $$ 2^{x}\\left(1+2^{x+1}\\right)=\\left(2^{x-1} m+\\epsilon\\right)^{2}-1=2^{2 x-2} m^{2}+2^{x} m \\epsilon, $$ or, equivalently $$ 1+2^{x+1}=2^{x-2} m^{2}+m \\epsilon . $$ Therefore $$ 1-\\epsilon m=2^{x-2}\\left(m^{2}-8\\right) . $$ For $\\epsilon=1$ this yields $m^{2}-8 \\leq 0$, i.e., $m=1$, which fails to satisfy (2). For $\\epsilon=-1$ equation (2) gives us $$ 1+m=2^{x-2}\\left(m^{2}-8\\right) \\geq 2\\left(m^{2}-8\\right), $$ implying $2 m^{2}-m-17 \\leq 0$. Hence $m \\leq 3$; on the other hand $m$ cannot be 1 by (2). Because $m$ is odd, we obtain $m=3$, leading to $x=4$. From (1) we get $y=23$. These values indeed satisfy the given equation. Recall that then $y=-23$ is also good. Thus we have the complete list of solutions $(x, y):(0,2),(0,-2),(4,23),(4,-23)$.","tier":0} +{"problem_type":"Number Theory","problem_label":"N2","problem":"For $x \\in(0,1)$ let $y \\in(0,1)$ be the number whose $n$th digit after the decimal point is the $\\left(2^{n}\\right)$ th digit after the decimal point of $x$. Show that if $x$ is rational then so is $y$. (Canada)","solution":"Since $x$ is rational, its digits repeat periodically starting at some point. We wish to show that this is also true for the digits of $y$, implying that $y$ is rational. Let $d$ be the length of the period of $x$ and let $d=2^{u} \\cdot v$, where $v$ is odd. There is a positive integer $w$ such that $$ 2^{w} \\equiv 1 \\quad(\\bmod v) $$ (For instance, one can choose $w$ to be $\\varphi(v)$, the value of Euler's function at $v$.) Therefore $$ 2^{n+w}=2^{n} \\cdot 2^{w} \\equiv 2^{n} \\quad(\\bmod v) $$ for each $n$. Also, for $n \\geq u$ we have $$ 2^{n+w} \\equiv 2^{n} \\equiv 0 \\quad\\left(\\bmod 2^{u}\\right) $$ It follows that, for all $n \\geq u$, the relation $$ 2^{n+w} \\equiv 2^{n} \\quad(\\bmod d) $$ holds. Thus, for $n$ sufficiently large, the $2^{n+w}$ th digit of $x$ is in the same spot in the cycle of $x$ as its $2^{n}$ th digit, and so these digits are equal. Hence the $(n+w)$ th digit of $y$ is equal to its $n$th digit. This means that the digits of $y$ repeat periodically with period $w$ from some point on, as required.","tier":0} +{"problem_type":"Number Theory","problem_label":"N3","problem":"The sequence $f(1), f(2), f(3), \\ldots$ is defined by $$ f(n)=\\frac{1}{n}\\left(\\left\\lfloor\\frac{n}{1}\\right\\rfloor+\\left\\lfloor\\frac{n}{2}\\right\\rfloor+\\cdots+\\left\\lfloor\\frac{n}{n}\\right\\rfloor\\right), $$ where $\\lfloor x\\rfloor$ denotes the integer part of $x$. (a) Prove that $f(n+1)>f(n)$ infinitely often. (b) Prove that $f(n+1)f(n)$ and $d(n+1)1, d(n) \\geq 2$ holds, with equality if and only if $n$ is prime. Since $f(6)=7 \/ 3>2$, it follows that $f(n)>2$ holds for all $n \\geq 6$. Since there are infinitely many primes, $d(n+1)=2$ holds for infinitely many values of $n$, and for each such $n \\geq 6$ we have $d(n+1)=2\\max \\{d(1), d(2), \\ldots, d(n)\\}$ for infinitely many $n$. For all such $n$, we have $d(n+1)>f(n)$. This completes the solution.","tier":0} +{"problem_type":"Number Theory","problem_label":"N4","problem":"Let $P$ be a polynomial of degree $n>1$ with integer coefficients and let $k$ be any positive integer. Consider the polynomial $Q(x)=P(P(\\ldots P(P(x)) \\ldots))$, with $k$ pairs of parentheses. Prove that $Q$ has no more than $n$ integer fixed points, i.e. integers satisfying the equation $Q(x)=x$. (Romania)","solution":"The claim is obvious if every integer fixed point of $Q$ is a fixed point of $P$ itself. For the sequel assume that this is not the case. Take any integer $x_{0}$ such that $Q\\left(x_{0}\\right)=x_{0}$, $P\\left(x_{0}\\right) \\neq x_{0}$ and define inductively $x_{i+1}=P\\left(x_{i}\\right)$ for $i=0,1,2, \\ldots$; then $x_{k}=x_{0}$. It is evident that $$ P(u)-P(v) \\text { is divisible by } u-v \\text { for distinct integers } u, v \\text {. } $$ (Indeed, if $P(x)=\\sum a_{i} x^{i}$ then each $a_{i}\\left(u^{i}-v^{i}\\right)$ is divisible by $u-v$.) Therefore each term in the chain of (nonzero) differences $$ x_{0}-x_{1}, \\quad x_{1}-x_{2}, \\quad \\ldots, \\quad x_{k-1}-x_{k}, \\quad x_{k}-x_{k+1} $$ is a divisor of the next one; and since $x_{k}-x_{k+1}=x_{0}-x_{1}$, all these differences have equal absolute values. For $x_{m}=\\min \\left(x_{1}, \\ldots, x_{k}\\right)$ this means that $x_{m-1}-x_{m}=-\\left(x_{m}-x_{m+1}\\right)$. Thus $x_{m-1}=x_{m+1}\\left(\\neq x_{m}\\right)$. It follows that consecutive differences in the sequence (2) have opposite signs. Consequently, $x_{0}, x_{1}, x_{2}, \\ldots$ is an alternating sequence of two distinct values. In other words, every integer fixed point of $Q$ is a fixed point of the polynomial $P(P(x))$. Our task is to prove that there are at most $n$ such points. Let $a$ be one of them so that $b=P(a) \\neq a$ (we have assumed that such an $a$ exists); then $a=P(b)$. Take any other integer fixed point $\\alpha$ of $P(P(x))$ and let $P(\\alpha)=\\beta$, so that $P(\\beta)=\\alpha$; the numbers $\\alpha$ and $\\beta$ need not be distinct ( $\\alpha$ can be a fixed point of $P$ ), but each of $\\alpha, \\beta$ is different from each of $a, b$. Applying property (1) to the four pairs of integers $(\\alpha, a),(\\beta, b)$, $(\\alpha, b),(\\beta, a)$ we get that the numbers $\\alpha-a$ and $\\beta-b$ divide each other, and also $\\alpha-b$ and $\\beta-a$ divide each other. Consequently $$ \\alpha-b= \\pm(\\beta-a), \\quad \\alpha-a= \\pm(\\beta-b) . $$ Suppose we have a plus in both instances: $\\alpha-b=\\beta-a$ and $\\alpha-a=\\beta-b$. Subtraction yields $a-b=b-a$, a contradiction, as $a \\neq b$. Therefore at least one equality in (3) holds with a minus sign. For each of them this means that $\\alpha+\\beta=a+b$; equivalently $a+b-\\alpha-P(\\alpha)=0$. Denote $a+b$ by $C$. We have shown that every integer fixed point of $Q$ other that $a$ and $b$ is a root of the polynomial $F(x)=C-x-P(x)$. This is of course true for $a$ and $b$ as well. And since $P$ has degree $n>1$, the polynomial $F$ has the same degree, so it cannot have more than $n$ roots. Hence the result. Comment. The first part of the solution, showing that integer fixed points of any iterate of $P$ are in fact fixed points of the second iterate $P \\circ P$ is standard; moreover, this fact has already appeared in contests. We however do not consider this as a major drawback to the problem because the only tricky moment comes up only at the next stage of the reasoning - to apply the divisibility property (1) to points from distinct 2-orbits of $P$. Yet maybe it would be more appropriate to state the problem in a version involving $k=2$ only.","tier":0} +{"problem_type":"Number Theory","problem_label":"N5","problem":"Find all integer solutions of the equation $$ \\frac{x^{7}-1}{x-1}=y^{5}-1 $$ (Russia)","solution":"The equation has no integer solutions. To show this, we first prove a lemma. Lemma. If $x$ is an integer and $p$ is a prime divisor of $\\frac{x^{7}-1}{x-1}$ then either $p \\equiv 1(\\bmod 7)$ or $p=7$. Proof. Both $x^{7}-1$ and $x^{p-1}-1$ are divisible by $p$, by hypothesis and by Fermat's little theorem, respectively. Suppose that 7 does not divide $p-1$. Then $\\operatorname{gcd}(p-1,7)=1$, so there exist integers $k$ and $m$ such that $7 k+(p-1) m=1$. We therefore have $$ x \\equiv x^{7 k+(p-1) m} \\equiv\\left(x^{7}\\right)^{k} \\cdot\\left(x^{p-1}\\right)^{m} \\equiv 1 \\quad(\\bmod p) $$ and so $$ \\frac{x^{7}-1}{x-1}=1+x+\\cdots+x^{6} \\equiv 7 \\quad(\\bmod p) $$ It follows that $p$ divides 7 , hence $p=7$ must hold if $p \\equiv 1(\\bmod 7)$ does not, as stated. The lemma shows that each positive divisor $d$ of $\\frac{x^{7}-1}{x-1}$ satisfies either $d \\equiv 0(\\bmod 7)$ or $d \\equiv 1(\\bmod 7)$. Now assume that $(x, y)$ is an integer solution of the original equation. Notice that $y-1>0$, because $\\frac{x^{7}-1}{x-1}>0$ for all $x \\neq 1$. Since $y-1$ divides $\\frac{x^{7}-1}{x-1}=y^{5}-1$, we have $y \\equiv 1(\\bmod 7)$ or $y \\equiv 2(\\bmod 7)$ by the previous paragraph. In the first case, $1+y+y^{2}+y^{3}+y^{4} \\equiv 5(\\bmod 7)$, and in the second $1+y+y^{2}+y^{3}+y^{4} \\equiv 3(\\bmod 7)$. Both possibilities contradict the fact that the positive divisor $1+y+y^{2}+y^{3}+y^{4}$ of $\\frac{x^{7}-1}{x-1}$ is congruent to 0 or 1 modulo 7 . So the given equation has no integer solutions.","tier":0} +{"problem_type":"Number Theory","problem_label":"N6","problem":"Let $a>b>1$ be relatively prime positive integers. Define the weight of an integer $c$, denoted by $w(c)$, to be the minimal possible value of $|x|+|y|$ taken over all pairs of integers $x$ and $y$ such that $$ a x+b y=c . $$ An integer $c$ is called a local champion if $w(c) \\geq w(c \\pm a)$ and $w(c) \\geq w(c \\pm b)$. Find all local champions and determine their number.","solution":"Call the pair of integers $(x, y)$ a representation of $c$ if $a x+b y=c$ and $|x|+|y|$ has the smallest possible value, i.e. $|x|+|y|=w(c)$. We characterise the local champions by the following three observations. Lemma 1. If $(x, y)$ a representation of a local champion $c$ then $x y<0$. Proof. Suppose indirectly that $x \\geq 0$ and $y \\geq 0$ and consider the values $w(c)$ and $w(c+a)$. All representations of the numbers $c$ and $c+a$ in the form $a u+b v$ can be written as $$ c=a(x-k b)+b(y+k a), \\quad c+a=a(x+1-k b)+b(y+k a) $$ where $k$ is an arbitrary integer. Since $|x|+|y|$ is minimal, we have $$ x+y=|x|+|y| \\leq|x-k b|+|y+k a| $$ for all $k$. On the other hand, $w(c+a) \\leq w(c)$, so there exists a $k$ for which $$ |x+1-k b|+|y+k a| \\leq|x|+|y|=x+y . $$ Then $$ (x+1-k b)+(y+k a) \\leq|x+1-k b|+|y+k a| \\leq x+y \\leq|x-k b|+|y+k a| . $$ Comparing the first and the third expressions, we find $k(a-b)+1 \\leq 0$ implying $k<0$. Comparing the second and fourth expressions, we get $|x+1-k b| \\leq|x-k b|$, therefore $k b>x$; this is a contradiction. If $x, y \\leq 0$ then we can switch to $-c,-x$ and $-y$. From this point, write $c=a x-b y$ instead of $c=a x+b y$ and consider only those cases where $x$ and $y$ are nonzero and have the same sign. By Lemma 1, there is no loss of generality in doing so. Lemma 2. Let $c=a x-b y$ where $|x|+|y|$ is minimal and $x, y$ have the same sign. The number $c$ is a local champion if and only if $|x|0$. The numbers $c-a$ and $c+b$ can be written as $$ c-a=a(x-1)-b y \\quad \\text { and } \\quad c+b=a x-b(y-1) $$ and trivially $w(c-a) \\leq(x-1)+y0$. We prove that we can choose $k=1$. Consider the function $f(t)=|x+1-b t|+|y-a t|-(x+y)$. This is a convex function and we have $f(0)=1$ and $f(k) \\leq 0$. By Jensen's inequality, $f(1) \\leq\\left(1-\\frac{1}{k}\\right) f(0)+\\frac{1}{k} f(k)<1$. But $f(1)$ is an integer. Therefore $f(1) \\leq 0$ and $$ |x+1-b|+|y-a| \\leq x+y . $$ Knowing $c=a(x-b)-b(y-a)$, we also have $$ x+y \\leq|x-b|+|y-a| . $$ Combining the two inequalities yields $|x+1-b| \\leq|x-b|$ which is equivalent to $xb$, we also have $0N$ and $$ 2^{b_{i}}+b_{i} \\equiv i \\quad(\\bmod d) . $$ This yields the claim for $m=b_{0}$. The base case $d=1$ is trivial. Take an $a>1$ and assume that the statement holds for all $d\\max \\left(2^{M}, N\\right)$ and $$ 2^{b_{i}}+b_{i} \\equiv i \\quad(\\bmod d) \\quad \\text { for } \\quad i=0,1,2, \\ldots, d-1 $$ For each $i=0,1, \\ldots, d-1$ consider the sequence $$ 2^{b_{i}}+b_{i}, \\quad 2^{b_{i}+k}+\\left(b_{i}+k\\right), \\ldots, \\quad 2^{b_{i}+\\left(a^{\\prime}-1\\right) k}+\\left(b_{i}+\\left(a^{\\prime}-1\\right) k\\right) . $$ Modulo $a$, these numbers are congruent to $$ 2^{b_{i}}+b_{i}, \\quad 2^{b_{i}}+\\left(b_{i}+k\\right), \\ldots, \\quad 2^{b_{i}}+\\left(b_{i}+\\left(a^{\\prime}-1\\right) k\\right), $$ respectively. The $d$ sequences contain $a^{\\prime} d=a$ numbers altogether. We shall now prove that no two of these numbers are congruent modulo $a$. Suppose that $$ 2^{b_{i}}+\\left(b_{i}+m k\\right) \\equiv 2^{b_{j}}+\\left(b_{j}+n k\\right) \\quad(\\bmod a) $$ for some values of $i, j \\in\\{0,1, \\ldots, d-1\\}$ and $m, n \\in\\left\\{0,1, \\ldots, a^{\\prime}-1\\right\\}$. Since $d$ is a divisor of $a$, we also have $$ 2^{b_{i}}+\\left(b_{i}+m k\\right) \\equiv 2^{b_{j}}+\\left(b_{j}+n k\\right) \\quad(\\bmod d) . $$ Because $d$ is a divisor of $k$ and in view of $(1)$, we obtain $i \\equiv j(\\bmod d)$. As $i, j \\in\\{0,1, \\ldots, d-1\\}$, this just means that $i=j$. Substituting this into (3) yields $m k \\equiv n k(\\bmod a)$. Therefore $m k^{\\prime} \\equiv n k^{\\prime}\\left(\\bmod a^{\\prime}\\right)$; and since $a^{\\prime}$ and $k^{\\prime}$ are coprime, we get $m \\equiv n\\left(\\bmod a^{\\prime}\\right)$. Hence also $m=n$. It follows that the $a$ numbers that make up the $d$ sequences (2) satisfy all the requirements; they are certainly all greater than $N$ because we chose each $b_{i}>\\max \\left(2^{M}, N\\right)$. So the statement holds for $a$, completing the induction.","tier":0} diff --git a/IMO/segmented/en-IMO2007SL.jsonl b/IMO/segmented/en-IMO2007SL.jsonl new file mode 100644 index 0000000000000000000000000000000000000000..982f22d9e80ece275ffccabdef2b31c98e0a4514 --- /dev/null +++ b/IMO/segmented/en-IMO2007SL.jsonl @@ -0,0 +1,42 @@ +{"problem_type":"Algebra","problem_label":"A1","problem":"Given a sequence $a_{1}, a_{2}, \\ldots, a_{n}$ of real numbers. For each $i(1 \\leq i \\leq n)$ define $$ d_{i}=\\max \\left\\{a_{j}: 1 \\leq j \\leq i\\right\\}-\\min \\left\\{a_{j}: i \\leq j \\leq n\\right\\} $$ and let $$ d=\\max \\left\\{d_{i}: 1 \\leq i \\leq n\\right\\} $$ (a) Prove that for arbitrary real numbers $x_{1} \\leq x_{2} \\leq \\ldots \\leq x_{n}$, $$ \\max \\left\\{\\left|x_{i}-a_{i}\\right|: 1 \\leq i \\leq n\\right\\} \\geq \\frac{d}{2} $$ (b) Show that there exists a sequence $x_{1} \\leq x_{2} \\leq \\ldots \\leq x_{n}$ of real numbers such that we have equality in (1). (New Zealand)","solution":"(a) Let $1 \\leq p \\leq q \\leq r \\leq n$ be indices for which $$ d=d_{q}, \\quad a_{p}=\\max \\left\\{a_{j}: 1 \\leq j \\leq q\\right\\}, \\quad a_{r}=\\min \\left\\{a_{j}: q \\leq j \\leq n\\right\\} $$ and thus $d=a_{p}-a_{r}$. (These indices are not necessarily unique.) ![](https:\/\/cdn.mathpix.com\/cropped\/2024_04_17_b8b307b9422db9068910g-08.jpg?height=363&width=797&top_left_y=1669&top_left_x=595) For arbitrary real numbers $x_{1} \\leq x_{2} \\leq \\ldots \\leq x_{n}$, consider just the two quantities $\\left|x_{p}-a_{p}\\right|$ and $\\left|x_{r}-a_{r}\\right|$. Since $$ \\left(a_{p}-x_{p}\\right)+\\left(x_{r}-a_{r}\\right)=\\left(a_{p}-a_{r}\\right)+\\left(x_{r}-x_{p}\\right) \\geq a_{p}-a_{r}=d, $$ we have either $a_{p}-x_{p} \\geq \\frac{d}{2}$ or $x_{r}-a_{r} \\geq \\frac{d}{2}$. Hence, $$ \\max \\left\\{\\left|x_{i}-a_{i}\\right|: 1 \\leq i \\leq n\\right\\} \\geq \\max \\left\\{\\left|x_{p}-a_{p}\\right|,\\left|x_{r}-a_{r}\\right|\\right\\} \\geq \\max \\left\\{a_{p}-x_{p}, x_{r}-a_{r}\\right\\} \\geq \\frac{d}{2} $$ (b) Define the sequence $\\left(x_{k}\\right)$ as $$ x_{1}=a_{1}-\\frac{d}{2}, \\quad x_{k}=\\max \\left\\{x_{k-1}, a_{k}-\\frac{d}{2}\\right\\} \\quad \\text { for } 2 \\leq k \\leq n $$ We show that we have equality in (1) for this sequence. By the definition, sequence $\\left(x_{k}\\right)$ is non-decreasing and $x_{k}-a_{k} \\geq-\\frac{d}{2}$ for all $1 \\leq k \\leq n$. Next we prove that $$ x_{k}-a_{k} \\leq \\frac{d}{2} \\quad \\text { for all } 1 \\leq k \\leq n $$ Consider an arbitrary index $1 \\leq k \\leq n$. Let $\\ell \\leq k$ be the smallest index such that $x_{k}=x_{\\ell}$. We have either $\\ell=1$, or $\\ell \\geq 2$ and $x_{\\ell}>x_{\\ell-1}$. In both cases, $$ x_{k}=x_{\\ell}=a_{\\ell}-\\frac{d}{2} $$ Since $$ a_{\\ell}-a_{k} \\leq \\max \\left\\{a_{j}: 1 \\leq j \\leq k\\right\\}-\\min \\left\\{a_{j}: k \\leq j \\leq n\\right\\}=d_{k} \\leq d $$ equality (3) implies $$ x_{k}-a_{k}=a_{\\ell}-a_{k}-\\frac{d}{2} \\leq d-\\frac{d}{2}=\\frac{d}{2} $$ We obtained that $-\\frac{d}{2} \\leq x_{k}-a_{k} \\leq \\frac{d}{2}$ for all $1 \\leq k \\leq n$, so $$ \\max \\left\\{\\left|x_{i}-a_{i}\\right|: 1 \\leq i \\leq n\\right\\} \\leq \\frac{d}{2} $$ We have equality because $\\left|x_{1}-a_{1}\\right|=\\frac{d}{2}$.","tier":0} +{"problem_type":"Algebra","problem_label":"A1","problem":"Given a sequence $a_{1}, a_{2}, \\ldots, a_{n}$ of real numbers. For each $i(1 \\leq i \\leq n)$ define $$ d_{i}=\\max \\left\\{a_{j}: 1 \\leq j \\leq i\\right\\}-\\min \\left\\{a_{j}: i \\leq j \\leq n\\right\\} $$ and let $$ d=\\max \\left\\{d_{i}: 1 \\leq i \\leq n\\right\\} $$ (a) Prove that for arbitrary real numbers $x_{1} \\leq x_{2} \\leq \\ldots \\leq x_{n}$, $$ \\max \\left\\{\\left|x_{i}-a_{i}\\right|: 1 \\leq i \\leq n\\right\\} \\geq \\frac{d}{2} $$ (b) Show that there exists a sequence $x_{1} \\leq x_{2} \\leq \\ldots \\leq x_{n}$ of real numbers such that we have equality in (1). (New Zealand)","solution":"We present another construction of a sequence $\\left(x_{i}\\right)$ for part (b). For each $1 \\leq i \\leq n$, let $$ M_{i}=\\max \\left\\{a_{j}: 1 \\leq j \\leq i\\right\\} \\quad \\text { and } \\quad m_{i}=\\min \\left\\{a_{j}: i \\leq j \\leq n\\right\\} $$ For all $1 \\leq in$, by (1) we have $$ f(m)=f(n+(m-n)) \\geq f(n)+f(f(m-n))-1 \\geq f(n), $$ so $f$ is nondecreasing. Function $f \\equiv 1$ is an obvious solution. To find other solutions, assume that $f \\not \\equiv 1$ and take the smallest $a \\in \\mathbb{N}$ such that $f(a)>1$. Then $f(b) \\geq f(a)>1$ for all integer $b \\geq a$. Suppose that $f(n)>n$ for some $n \\in \\mathbb{N}$. Then we have $$ f(f(n))=f((f(n)-n)+n) \\geq f(f(n)-n)+f(f(n))-1 $$ so $f(f(n)-n) \\leq 1$ and hence $f(n)-n2007 ; \\end{array} \\quad h_{j}(n)=\\max \\left\\{1,\\left\\lfloor\\frac{j n}{2007}\\right\\rfloor\\right\\}\\right. $$ Also the example for $j=2008$ can be generalized. In particular, choosing a divisor $d>1$ of 2007 , one can set $$ f_{2008, d}(n)= \\begin{cases}n, & d \\nless\\{n \\\\ n+1, & d \\mid n\\end{cases} $$","tier":0} +{"problem_type":"Algebra","problem_label":"A3","problem":"Let $n$ be a positive integer, and let $x$ and $y$ be positive real numbers such that $x^{n}+y^{n}=1$. Prove that $$ \\left(\\sum_{k=1}^{n} \\frac{1+x^{2 k}}{1+x^{4 k}}\\right)\\left(\\sum_{k=1}^{n} \\frac{1+y^{2 k}}{1+y^{4 k}}\\right)<\\frac{1}{(1-x)(1-y)} $$ (Estonia)","solution":"For each real $t \\in(0,1)$, $$ \\frac{1+t^{2}}{1+t^{4}}=\\frac{1}{t}-\\frac{(1-t)\\left(1-t^{3}\\right)}{t\\left(1+t^{4}\\right)}<\\frac{1}{t} $$ Substituting $t=x^{k}$ and $t=y^{k}$, $$ 0<\\sum_{k=1}^{n} \\frac{1+x^{2 k}}{1+x^{4 k}}<\\sum_{k=1}^{n} \\frac{1}{x^{k}}=\\frac{1-x^{n}}{x^{n}(1-x)} \\quad \\text { and } \\quad 0<\\sum_{k=1}^{n} \\frac{1+y^{2 k}}{1+y^{4 k}}<\\sum_{k=1}^{n} \\frac{1}{y^{k}}=\\frac{1-y^{n}}{y^{n}(1-y)} $$ Since $1-y^{n}=x^{n}$ and $1-x^{n}=y^{n}$, $$ \\frac{1-x^{n}}{x^{n}(1-x)}=\\frac{y^{n}}{x^{n}(1-x)}, \\quad \\frac{1-y^{n}}{y^{n}(1-y)}=\\frac{x^{n}}{y^{n}(1-y)} $$ and therefore $$ \\left(\\sum_{k=1}^{n} \\frac{1+x^{2 k}}{1+x^{4 k}}\\right)\\left(\\sum_{k=1}^{n} \\frac{1+y^{2 k}}{1+y^{4 k}}\\right)<\\frac{y^{n}}{x^{n}(1-x)} \\cdot \\frac{x^{n}}{y^{n}(1-y)}=\\frac{1}{(1-x)(1-y)} . $$","tier":0} +{"problem_type":"Algebra","problem_label":"A3","problem":"Let $n$ be a positive integer, and let $x$ and $y$ be positive real numbers such that $x^{n}+y^{n}=1$. Prove that $$ \\left(\\sum_{k=1}^{n} \\frac{1+x^{2 k}}{1+x^{4 k}}\\right)\\left(\\sum_{k=1}^{n} \\frac{1+y^{2 k}}{1+y^{4 k}}\\right)<\\frac{1}{(1-x)(1-y)} $$ (Estonia)","solution":"We prove $$ \\left(\\sum_{k=1}^{n} \\frac{1+x^{2 k}}{1+x^{4 k}}\\right)\\left(\\sum_{k=1}^{n} \\frac{1+y^{2 k}}{1+y^{4 k}}\\right)<\\frac{\\left(\\frac{1+\\sqrt{2}}{2} \\ln 2\\right)^{2}}{(1-x)(1-y)}<\\frac{0.7001}{(1-x)(1-y)} $$ The idea is to estimate each term on the left-hand side with the same constant. To find the upper bound for the expression $\\frac{1+x^{2 k}}{1+x^{4 k}}$, consider the function $f(t)=\\frac{1+t}{1+t^{2}}$ in interval $(0,1)$. Since $$ f^{\\prime}(t)=\\frac{1-2 t-t^{2}}{\\left(1+t^{2}\\right)^{2}}=\\frac{(\\sqrt{2}+1+t)(\\sqrt{2}-1-t)}{\\left(1+t^{2}\\right)^{2}} $$ the function increases in interval $(0, \\sqrt{2}-1]$ and decreases in $[\\sqrt{2}-1,1)$. Therefore the maximum is at point $t_{0}=\\sqrt{2}-1$ and $$ f(t)=\\frac{1+t}{1+t^{2}} \\leq f\\left(t_{0}\\right)=\\frac{1+\\sqrt{2}}{2}=\\alpha . $$ Applying this to each term on the left-hand side of (1), we obtain $$ \\left(\\sum_{k=1}^{n} \\frac{1+x^{2 k}}{1+x^{4 k}}\\right)\\left(\\sum_{k=1}^{n} \\frac{1+y^{2 k}}{1+y^{4 k}}\\right) \\leq n \\alpha \\cdot n \\alpha=(n \\alpha)^{2} $$ To estimate $(1-x)(1-y)$ on the right-hand side, consider the function $$ g(t)=\\ln \\left(1-t^{1 \/ n}\\right)+\\ln \\left(1-(1-t)^{1 \/ n}\\right) . $$ Substituting $s$ for $1-t$, we have $$ -n g^{\\prime}(t)=\\frac{t^{1 \/ n-1}}{1-t^{1 \/ n}}-\\frac{s^{1 \/ n-1}}{1-s^{1 \/ n}}=\\frac{1}{s t}\\left(\\frac{(1-t) t^{1 \/ n}}{1-t^{1 \/ n}}-\\frac{(1-s) s^{1 \/ n}}{1-s^{1 \/ n}}\\right)=\\frac{h(t)-h(s)}{s t} . $$ The function $$ h(t)=t^{1 \/ n} \\frac{1-t}{1-t^{1 \/ n}}=\\sum_{i=1}^{n} t^{i \/ n} $$ is obviously increasing for $t \\in(0,1)$, hence for these values of $t$ we have $$ g^{\\prime}(t)>0 \\Longleftrightarrow h(t)y$ for all $y \\in \\mathbb{R}^{+}$. Functional equation (1) yields $f(x+f(y))>f(x+y)$ and hence $f(y) \\neq y$ immediately. If $f(y)f(y) $$ contradiction. Therefore $f(y)>y$ for all $y \\in \\mathbb{R}^{+}$. For $x \\in \\mathbb{R}^{+}$define $g(x)=f(x)-x$; then $f(x)=g(x)+x$ and, as we have seen, $g(x)>0$. Transforming (1) for function $g(x)$ and setting $t=x+y$, $$ \\begin{aligned} f(t+g(y)) & =f(t)+f(y) \\\\ g(t+g(y))+t+g(y) & =(g(t)+t)+(g(y)+y) \\end{aligned} $$ and therefore $$ g(t+g(y))=g(t)+y \\quad \\text { for all } t>y>0 $$ Next we prove that function $g(x)$ is injective. Suppose that $g\\left(y_{1}\\right)=g\\left(y_{2}\\right)$ for some numbers $y_{1}, y_{2} \\in \\mathbb{R}^{+}$. Then by $(2)$, $$ g(t)+y_{1}=g\\left(t+g\\left(y_{1}\\right)\\right)=g\\left(t+g\\left(y_{2}\\right)\\right)=g(t)+y_{2} $$ for all $t>\\max \\left\\{y_{1}, y_{2}\\right\\}$. Hence, $g\\left(y_{1}\\right)=g\\left(y_{2}\\right)$ is possible only if $y_{1}=y_{2}$. Now let $u, v$ be arbitrary positive numbers and $t>u+v$. Applying (2) three times, $$ g(t+g(u)+g(v))=g(t+g(u))+v=g(t)+u+v=g(t+g(u+v)) \\text {. } $$ By the injective property we conclude that $t+g(u)+g(v)=t+g(u+v)$, hence $$ g(u)+g(v)=g(u+v) . $$ Since function $g(v)$ is positive, equation (3) also shows that $g$ is an increasing function. Finally we prove that $g(x)=x$. Combining (2) and (3), we obtain $$ g(t)+y=g(t+g(y))=g(t)+g(g(y)) $$ and hence $$ g(g(y))=y $$ Suppose that there exists an $x \\in \\mathbb{R}^{+}$such that $g(x) \\neq x$. By the monotonicity of $g$, if $x>g(x)$ then $g(x)>g(g(x))=x$. Similarly, if $x0$ ) imply $g(x)=c x$. So, after proving (3), it is sufficient to test functions $f(x)=(c+1) x$.","tier":0} +{"problem_type":"Algebra","problem_label":"A4","problem":"Find all functions $f: \\mathbb{R}^{+} \\rightarrow \\mathbb{R}^{+}$such that $$ f(x+f(y))=f(x+y)+f(y) $$ for all $x, y \\in \\mathbb{R}^{+}$. (Symbol $\\mathbb{R}^{+}$denotes the set of all positive real numbers.) (Thaliand) Answer. $f(x)=2 x$.","solution":"We prove that $f(y)>y$ and introduce function $g(x)=f(x)-x>0$ in the same way as in For arbitrary $t>y>0$, substitute $x=t-y$ into (1) to obtain $$ f(t+g(y))=f(t)+f(y) $$ which, by induction, implies $$ f(t+n g(y))=f(t)+n f(y) \\quad \\text { for all } t>y>0, n \\in \\mathbb{N} \\text {. } $$ Take two arbitrary positive reals $y$ and $z$ and a third fixed number $t>\\max \\{y, z\\}$. For each positive integer $k$, let $\\ell_{k}=\\left\\lfloor k \\frac{g(y)}{g(z)}\\right\\rfloor$. Then $t+k g(y)-\\ell_{k} g(z) \\geq t>z$ and, applying (4) twice, $$ \\begin{aligned} f\\left(t+k g(y)-\\ell_{k} g(z)\\right)+\\ell_{k} f(z) & =f(t+k g(y))=f(t)+k f(y) \\\\ 0<\\frac{1}{k} f\\left(t+k g(y)-\\ell_{k} g(z)\\right) & =\\frac{f(t)}{k}+f(y)-\\frac{\\ell_{k}}{k} f(z) . \\end{aligned} $$ As $k \\rightarrow \\infty$ we get $$ 0 \\leq \\lim _{k \\rightarrow \\infty}\\left(\\frac{f(t)}{k}+f(y)-\\frac{\\ell_{k}}{k} f(z)\\right)=f(y)-\\frac{g(y)}{g(z)} f(z)=f(y)-\\frac{f(y)-y}{f(z)-z} f(z) $$ and therefore $$ \\frac{f(y)}{y} \\leq \\frac{f(z)}{z} $$ Exchanging variables $y$ and $z$, we obtain the reverse inequality. Hence, $\\frac{f(y)}{y}=\\frac{f(z)}{z}$ for arbitrary $y$ and $z$; so function $\\frac{f(x)}{x}$ is constant, $f(x)=c x$. Substituting back into (1), we find that $f(x)=c x$ is a solution if and only if $c=2$. So the only solution for the problem is $f(x)=2 x$.","tier":0} +{"problem_type":"Algebra","problem_label":"A5","problem":"Let $c>2$, and let $a(1), a(2), \\ldots$ be a sequence of nonnegative real numbers such that $$ a(m+n) \\leq 2 a(m)+2 a(n) \\text { for all } m, n \\geq 1 \\text {, } $$ and $$ a\\left(2^{k}\\right) \\leq \\frac{1}{(k+1)^{c}} \\quad \\text { for all } k \\geq 0 $$ Prove that the sequence $a(n)$ is bounded. (Croatia)","solution":"For convenience, define $a(0)=0$; then condition (1) persists for all pairs of nonnegative indices. Lemma 1. For arbitrary nonnegative indices $n_{1}, \\ldots, n_{k}$, we have $$ a\\left(\\sum_{i=1}^{k} n_{i}\\right) \\leq \\sum_{i=1}^{k} 2^{i} a\\left(n_{i}\\right) $$ and $$ a\\left(\\sum_{i=1}^{k} n_{i}\\right) \\leq 2 k \\sum_{i=1}^{k} a\\left(n_{i}\\right) $$ Proof. Inequality (3) is proved by induction on $k$. The base case $k=1$ is trivial, while the induction step is provided by $a\\left(\\sum_{i=1}^{k+1} n_{i}\\right)=a\\left(n_{1}+\\sum_{i=2}^{k+1} n_{i}\\right) \\leq 2 a\\left(n_{1}\\right)+2 a\\left(\\sum_{i=1}^{k} n_{i+1}\\right) \\leq 2 a\\left(n_{1}\\right)+2 \\sum_{i=1}^{k} 2^{i} a\\left(n_{i+1}\\right)=\\sum_{i=1}^{k+1} 2^{i} a\\left(n_{i}\\right)$. To establish (4), first the inequality $$ a\\left(\\sum_{i=1}^{2^{d}} n_{i}\\right) \\leq 2^{d} \\sum_{i=1}^{2^{d}} a\\left(n_{i}\\right) $$ can be proved by an obvious induction on $d$. Then, turning to (4), we find an integer $d$ such that $2^{d-1}d$, and take some positive integer $f$ such that $M_{f}>d$. Applying (3), we get $$ a(n)=a\\left(\\sum_{k=1}^{f} \\sum_{M_{k-1} \\leq i2$, and let $a(1), a(2), \\ldots$ be a sequence of nonnegative real numbers such that $$ a(m+n) \\leq 2 a(m)+2 a(n) \\text { for all } m, n \\geq 1 \\text {, } $$ and $$ a\\left(2^{k}\\right) \\leq \\frac{1}{(k+1)^{c}} \\quad \\text { for all } k \\geq 0 $$ Prove that the sequence $a(n)$ is bounded. (Croatia)","solution":"Lemma 2. Suppose that $s_{1}, \\ldots, s_{k}$ are positive integers such that $$ \\sum_{i=1}^{k} 2^{-s_{i}} \\leq 1 $$ Then for arbitrary positive integers $n_{1}, \\ldots, n_{k}$ we have $$ a\\left(\\sum_{i=1}^{k} n_{i}\\right) \\leq \\sum_{i=1}^{k} 2^{s_{i}} a\\left(n_{i}\\right) $$ Proof. Apply an induction on $k$. The base cases are $k=1$ (trivial) and $k=2$ (follows from the condition (1)). Suppose that $k>2$. We can assume that $s_{1} \\leq s_{2} \\leq \\cdots \\leq s_{k}$. Note that $$ \\sum_{i=1}^{k-1} 2^{-s_{i}} \\leq 1-2^{-s_{k-1}} $$ since the left-hand side is a fraction with the denominator $2^{s_{k-1}}$, and this fraction is less than 1. Define $s_{k-1}^{\\prime}=s_{k-1}-1$ and $n_{k-1}^{\\prime}=n_{k-1}+n_{k}$; then we have $$ \\sum_{i=1}^{k-2} 2^{-s_{i}}+2^{-s_{k-1}^{\\prime}} \\leq\\left(1-2 \\cdot 2^{-s_{k-1}}\\right)+2^{1-s_{k-1}}=1 . $$ Now, the induction hypothesis can be applied to achieve $$ \\begin{aligned} a\\left(\\sum_{i=1}^{k} n_{i}\\right)=a\\left(\\sum_{i=1}^{k-2} n_{i}+n_{k-1}^{\\prime}\\right) & \\leq \\sum_{i=1}^{k-2} 2^{s_{i}} a\\left(n_{i}\\right)+2^{s_{k-1}^{\\prime}} a\\left(n_{k-1}^{\\prime}\\right) \\\\ & \\leq \\sum_{i=1}^{k-2} 2^{s_{i}} a\\left(n_{i}\\right)+2^{s_{k-1}-1} \\cdot 2\\left(a\\left(n_{k-1}\\right)+a\\left(n_{k}\\right)\\right) \\\\ & \\leq \\sum_{i=1}^{k-2} 2^{s_{i}} a\\left(n_{i}\\right)+2^{s_{k-1}} a\\left(n_{k-1}\\right)+2^{s_{k}} a\\left(n_{k}\\right) . \\end{aligned} $$ Let $q=c \/ 2>1$. Take an arbitrary positive integer $n$ and write $$ n=\\sum_{i=1}^{k} 2^{u_{i}}, \\quad 0 \\leq u_{1}1$. Comment 1. In fact, Lemma 2 (applied to the case $n_{i}=2^{u_{i}}$ only) provides a sharp bound for any $a(n)$. Actually, let $b(k)=\\frac{1}{(k+1)^{c}}$ and consider the sequence $$ a(n)=\\min \\left\\{\\sum_{i=1}^{k} 2^{s_{i}} b\\left(u_{i}\\right) \\mid k \\in \\mathbb{N}, \\quad \\sum_{i=1}^{k} 2^{-s_{i}} \\leq 1, \\quad \\sum_{i=1}^{k} 2^{u_{i}}=n\\right\\} $$ We show that this sequence satisfies the conditions of the problem. Take two arbitrary indices $m$ and $n$. Let $$ \\begin{aligned} & a(m)=\\sum_{i=1}^{k} 2^{s_{i}} b\\left(u_{i}\\right), \\quad \\sum_{i=1}^{k} 2^{-s_{i}} \\leq 1, \\quad \\sum_{i=1}^{k} 2^{u_{i}}=m ; \\\\ & a(n)=\\sum_{i=1}^{l} 2^{r_{i}} b\\left(w_{i}\\right), \\quad \\sum_{i=1}^{l} 2^{-r_{i}} \\leq 1, \\quad \\sum_{i=1}^{l} 2^{w_{i}}=n . \\end{aligned} $$ Then we have $$ \\sum_{i=1}^{k} 2^{-1-s_{i}}+\\sum_{i=1}^{l} 2^{-1-r_{i}} \\leq \\frac{1}{2}+\\frac{1}{2}=1, \\quad \\sum_{i=1}^{k} 2^{u_{i}}+\\sum_{i=1}^{l} 2^{w_{i}}=m+n $$ so by (5) we obtain $$ a(n+m) \\leq \\sum_{i=1}^{k} 2^{1+s_{i}} b\\left(u_{i}\\right)+\\sum_{i=1}^{l} 2^{1+r_{i}} b\\left(w_{i}\\right)=2 a(m)+2 a(n) . $$ Comment 2. The condition $c>2$ is sharp; we show that the sequence (5) is not bounded if $c \\leq 2$. First, we prove that for an arbitrary $n$ the minimum in (5) is attained with a sequence $\\left(u_{i}\\right)$ consisting of distinct numbers. To the contrary, assume that $u_{k-1}=u_{k}$. Replace $u_{k-1}$ and $u_{k}$ by a single number $u_{k-1}^{\\prime}=u_{k}+1$, and $s_{k-1}$ and $s_{k}$ by $s_{k-1}^{\\prime}=\\min \\left\\{s_{k-1}, s_{k}\\right\\}$. The modified sequences provide a better bound since $$ 2^{s_{k-1}^{\\prime}} b\\left(u_{k-1}^{\\prime}\\right)=2^{s_{k-1}^{\\prime}} b\\left(u_{k}+1\\right)<2^{s_{k-1}} b\\left(u_{k-1}\\right)+2^{s_{k}} b\\left(u_{k}\\right) $$ (we used the fact that $b(k)$ is decreasing). This is impossible. Hence, the claim is proved, and we can assume that the minimum is attained with $u_{1}<\\cdots0.4513$. Comment 3. The solution can be improved in several ways to give somewhat better bounds for $c_{n}$. Here we show a variant which proves $c_{n}<0.4589$ for $n \\geq 5$. The value of $c_{n}$ does not change if negative values are also allowed in (3). So the problem is equivalent to maximizing $$ f\\left(a_{1}, a_{2}, \\ldots, a_{n}\\right)=a_{1}^{2} a_{2}+a_{2}^{2} a_{3}+\\ldots+a_{n}^{2} a_{1} $$ on the unit sphere $a_{1}^{2}+a_{2}^{2}+\\ldots+a_{n}^{2}=1$ in $\\mathbb{R}^{n}$. Since the unit sphere is compact, the function has a maximum and we can apply the Lagrange multiplier method; for each maximum point there exists a real number $\\lambda$ such that $$ a_{k-1}^{2}+2 a_{k} a_{k+1}=\\lambda \\cdot 2 a_{k} \\quad \\text { for all } k=1,2, \\ldots, n $$ Then $$ 3 S=\\sum_{k=1}^{n}\\left(a_{k-1}^{2} a_{k}+2 a_{k}^{2} a_{k+1}\\right)=\\sum_{k=1}^{n} 2 \\lambda a_{k}^{2}=2 \\lambda $$ and therefore $$ a_{k-1}^{2}+2 a_{k} a_{k+1}=3 S a_{k} \\quad \\text { for all } k=1,2, \\ldots, n \\text {. } $$ From (4) we can derive $$ 9 S^{2}=\\sum_{k=1}^{n}\\left(3 S a_{k}\\right)^{2}=\\sum_{k=1}^{n}\\left(a_{k-1}^{2}+2 a_{k} a_{k+1}\\right)^{2}=\\sum_{k=1}^{n} a_{k}^{4}+4 \\sum_{k=1}^{n} a_{k}^{2} a_{k+1}^{2}+4 \\sum_{k=1}^{n} a_{k}^{2} a_{k+1} a_{k+2} $$ and $$ 3 S^{2}=\\sum_{k=1}^{n} 3 S a_{k-1}^{2} a_{k}=\\sum_{k=1}^{n} a_{k-1}^{2}\\left(a_{k-1}^{2}+2 a_{k} a_{k+1}\\right)=\\sum_{k=1}^{n} a_{k}^{4}+2 \\sum_{k=1}^{n} a_{k}^{2} a_{k+1} a_{k+2} . $$ Let $p$ be a positive number. Combining (5) and (6) and applying the AM-GM inequality, $$ \\begin{aligned} (9+3 p) S^{2} & =(1+p) \\sum_{k=1}^{n} a_{k}^{4}+4 \\sum_{k=1}^{n} a_{k}^{2} a_{k+1}^{2}+(4+2 p) \\sum_{k=1}^{n} a_{k}^{2} a_{k+1} a_{k+2} \\\\ & \\leq(1+p) \\sum_{k=1}^{n} a_{k}^{4}+4 \\sum_{k=1}^{n} a_{k}^{2} a_{k+1}^{2}+\\sum_{k=1}^{n}\\left(2(1+p) a_{k}^{2} a_{k+2}^{2}+\\frac{(2+p)^{2}}{2(1+p)} a_{k}^{2} a_{k+1}^{2}\\right) \\\\ & =(1+p) \\sum_{k=1}^{n}\\left(a_{k}^{4}+2 a_{k}^{2} a_{k+1}^{2}+2 a_{k}^{2} a_{k+2}^{2}\\right)+\\left(4+\\frac{(2+p)^{2}}{2(1+p)}-2(1+p)\\right) \\sum_{k=1}^{n} a_{k}^{2} a_{k+1}^{2} \\\\ & \\leq(1+p)\\left(\\sum_{k=1}^{n} a_{k}^{2}\\right)^{2}+\\frac{8+4 p-3 p^{2}}{2(1+p)} \\sum_{k=1}^{n} a_{k}^{2} a_{k+1}^{2} \\\\ & =(1+p)+\\frac{8+4 p-3 p^{2}}{2(1+p)} \\sum_{k=1}^{n} a_{k}^{2} a_{k+1}^{2} . \\end{aligned} $$ Setting $p=\\frac{2+2 \\sqrt{7}}{3}$ which is the positive root of $8+4 p-3 p^{2}=0$, we obtain $$ S \\leq \\sqrt{\\frac{1+p}{9+3 p}}=\\sqrt{\\frac{5+2 \\sqrt{7}}{33+6 \\sqrt{7}}} \\approx 0.458879 . $$","tier":0} +{"problem_type":"Algebra","problem_label":"A7","problem":"Let $n>1$ be an integer. In the space, consider the set $$ S=\\{(x, y, z) \\mid x, y, z \\in\\{0,1, \\ldots, n\\}, x+y+z>0\\} $$ Find the smallest number of planes that jointly contain all $(n+1)^{3}-1$ points of $S$ but none of them passes through the origin. (Netherlands) Answer. $3 n$ planes.","solution":"It is easy to find $3 n$ such planes. For example, planes $x=i, y=i$ or $z=i$ $(i=1,2, \\ldots, n)$ cover the set $S$ but none of them contains the origin. Another such collection consists of all planes $x+y+z=k$ for $k=1,2, \\ldots, 3 n$. We show that $3 n$ is the smallest possible number. Lemma 1. Consider a nonzero polynomial $P\\left(x_{1}, \\ldots, x_{k}\\right)$ in $k$ variables. Suppose that $P$ vanishes at all points $\\left(x_{1}, \\ldots, x_{k}\\right)$ such that $x_{1}, \\ldots, x_{k} \\in\\{0,1, \\ldots, n\\}$ and $x_{1}+\\cdots+x_{k}>0$, while $P(0,0, \\ldots, 0) \\neq 0$. Then $\\operatorname{deg} P \\geq k n$. Proof. We use induction on $k$. The base case $k=0$ is clear since $P \\neq 0$. Denote for clarity $y=x_{k}$. Let $R\\left(x_{1}, \\ldots, x_{k-1}, y\\right)$ be the residue of $P$ modulo $Q(y)=y(y-1) \\ldots(y-n)$. Polynomial $Q(y)$ vanishes at each $y=0,1, \\ldots, n$, hence $P\\left(x_{1}, \\ldots, x_{k-1}, y\\right)=R\\left(x_{1}, \\ldots, x_{k-1}, y\\right)$ for all $x_{1}, \\ldots, x_{k-1}, y \\in\\{0,1, \\ldots, n\\}$. Therefore, $R$ also satisfies the condition of the Lemma; moreover, $\\operatorname{deg}_{y} R \\leq n$. Clearly, $\\operatorname{deg} R \\leq \\operatorname{deg} P$, so it suffices to prove that $\\operatorname{deg} R \\geq n k$. Now, expand polynomial $R$ in the powers of $y$ : $$ R\\left(x_{1}, \\ldots, x_{k-1}, y\\right)=R_{n}\\left(x_{1}, \\ldots, x_{k-1}\\right) y^{n}+R_{n-1}\\left(x_{1}, \\ldots, x_{k-1}\\right) y^{n-1}+\\cdots+R_{0}\\left(x_{1}, \\ldots, x_{k-1}\\right) $$ We show that polynomial $R_{n}\\left(x_{1}, \\ldots, x_{k-1}\\right)$ satisfies the condition of the induction hypothesis. Consider the polynomial $T(y)=R(0, \\ldots, 0, y)$ of degree $\\leq n$. This polynomial has $n$ roots $y=1, \\ldots, n$; on the other hand, $T(y) \\not \\equiv 0$ since $T(0) \\neq 0$. Hence $\\operatorname{deg} T=n$, and its leading coefficient is $R_{n}(0,0, \\ldots, 0) \\neq 0$. In particular, in the case $k=1$ we obtain that coefficient $R_{n}$ is nonzero. Similarly, take any numbers $a_{1}, \\ldots, a_{k-1} \\in\\{0,1, \\ldots, n\\}$ with $a_{1}+\\cdots+a_{k-1}>0$. Substituting $x_{i}=a_{i}$ into $R\\left(x_{1}, \\ldots, x_{k-1}, y\\right)$, we get a polynomial in $y$ which vanishes at all points $y=0, \\ldots, n$ and has degree $\\leq n$. Therefore, this polynomial is null, hence $R_{i}\\left(a_{1}, \\ldots, a_{k-1}\\right)=0$ for all $i=0,1, \\ldots, n$. In particular, $R_{n}\\left(a_{1}, \\ldots, a_{k-1}\\right)=0$. Thus, the polynomial $R_{n}\\left(x_{1}, \\ldots, x_{k-1}\\right)$ satisfies the condition of the induction hypothesis. So, we have $\\operatorname{deg} R_{n} \\geq(k-1) n$ and $\\operatorname{deg} P \\geq \\operatorname{deg} R \\geq \\operatorname{deg} R_{n}+n \\geq k n$. Now we can finish the solution. Suppose that there are $N$ planes covering all the points of $S$ but not containing the origin. Let their equations be $a_{i} x+b_{i} y+c_{i} z+d_{i}=0$. Consider the polynomial $$ P(x, y, z)=\\prod_{i=1}^{N}\\left(a_{i} x+b_{i} y+c_{i} z+d_{i}\\right) $$ It has total degree $N$. This polynomial has the property that $P\\left(x_{0}, y_{0}, z_{0}\\right)=0$ for any $\\left(x_{0}, y_{0}, z_{0}\\right) \\in S$, while $P(0,0,0) \\neq 0$. Hence by Lemma 1 we get $N=\\operatorname{deg} P \\geq 3 n$, as desired. Comment 1. There are many other collections of $3 n$ planes covering the set $S$ but not covering the origin.","tier":0} +{"problem_type":"Algebra","problem_label":"A7","problem":"Let $n>1$ be an integer. In the space, consider the set $$ S=\\{(x, y, z) \\mid x, y, z \\in\\{0,1, \\ldots, n\\}, x+y+z>0\\} $$ Find the smallest number of planes that jointly contain all $(n+1)^{3}-1$ points of $S$ but none of them passes through the origin. (Netherlands) Answer. $3 n$ planes.","solution":"We present a different proof of the main Lemma 1. Here we confine ourselves to the case $k=3$, which is applied in the solution, and denote the variables by $x, y$ and $z$. (The same proof works for the general statement as well.) The following fact is known with various proofs; we provide one possible proof for the completeness. Lemma 2. For arbitrary integers $0 \\leq m1$ be an integer. Find all sequences $a_{1}, a_{2}, \\ldots, a_{n^{2}+n}$ satisfying the following conditions: (a) $a_{i} \\in\\{0,1\\}$ for all $1 \\leq i \\leq n^{2}+n$; (b) $a_{i+1}+a_{i+2}+\\ldots+a_{i+n}0$. By (2), it contains some \"ones\". Let the first \"one\" in this block be at the $u$ th position (that is, $\\left.a_{u+v n}=1\\right)$. By the induction hypothesis, the $(v-1)$ th and $v$ th blocks of $\\left(a_{i}\\right)$ have the form $$ (\\underbrace{0 \\ldots 0 \\ldots 0}_{n-v+1} \\underbrace{1 \\ldots 1}_{v-1})(\\underbrace{0 \\ldots 0}_{u-1} 1 * \\ldots *), $$ where each star can appear to be any binary digit. Observe that $u \\leq n-v+1$, since the sum in this block is $v$. Then, the fragment of length $n$ bracketed above has exactly $(v-1)+1$ ones, i. e. $S(u+(v-1) n, u+v n]=v$. Hence, $$ v=S(u+(v-1) n, u+v n]1$ be an integer. Find all sequences $a_{1}, a_{2}, \\ldots, a_{n^{2}+n}$ satisfying the following conditions: (a) $a_{i} \\in\\{0,1\\}$ for all $1 \\leq i \\leq n^{2}+n$; (b) $a_{i+1}+a_{i+2}+\\ldots+a_{i+n}1$ rectangles such that their sides are parallel to the sides of the square. Any line, parallel to a side of the square and intersecting its interior, also intersects the interior of some rectangle. Prove that in this dissection, there exists a rectangle having no point on the boundary of the square. (Japan)","solution":"Call the directions of the sides of the square horizontal and vertical. A horizontal or vertical line, which intersects the interior of the square but does not intersect the interior of any rectangle, will be called a splitting line. A rectangle having no point on the boundary of the square will be called an interior rectangle. Suppose, to the contrary, that there exists a dissection of the square into more than one rectangle, such that no interior rectangle and no splitting line appear. Consider such a dissection with the least possible number of rectangles. Notice that this number of rectangles is greater than 2, otherwise their common side provides a splitting line. If there exist two rectangles having a common side, then we can replace them by their union (see Figure 1). The number of rectangles was greater than 2, so in a new dissection it is greater than 1. Clearly, in the new dissection, there is also no splitting line as well as no interior rectangle. This contradicts the choice of the original dissection. Denote the initial square by $A B C D$, with $A$ and $B$ being respectively the lower left and lower right vertices. Consider those two rectangles $a$ and $b$ containing vertices $A$ and $B$, respectively. (Note that $a \\neq b$, otherwise its top side provides a splitting line.) We can assume that the height of $a$ is not greater than that of $b$. Then consider the rectangle $c$ neighboring to the lower right corner of $a$ (it may happen that $c=b$ ). By aforementioned, the heights of $a$ and $c$ are distinct. Then two cases are possible. ![](https:\/\/cdn.mathpix.com\/cropped\/2024_04_17_b8b307b9422db9068910g-29.jpg?height=214&width=323&top_left_y=1549&top_left_x=250) Figure 1 ![](https:\/\/cdn.mathpix.com\/cropped\/2024_04_17_b8b307b9422db9068910g-29.jpg?height=339&width=580&top_left_y=1481&top_left_x=658) Figure 2 ![](https:\/\/cdn.mathpix.com\/cropped\/2024_04_17_b8b307b9422db9068910g-29.jpg?height=339&width=583&top_left_y=1481&top_left_x=1322) Figure 3 Case 1. The height of $c$ is less than that of $a$. Consider the rectangle $d$ which is adjacent to both $a$ and $c$, i. e. the one containing the angle marked in Figure 2. This rectangle has no common point with $B C$ (since $a$ is not higher than $b$ ), as well as no common point with $A B$ or with $A D$ (obviously). Then $d$ has a common point with $C D$, and its left side provides a splitting line. Contradiction. Case 2. The height of $c$ is greater than that of $a$. Analogously, consider the rectangle $d$ containing the angle marked on Figure 3. It has no common point with $A D$ (otherwise it has a common side with $a$ ), as well as no common point with $A B$ or with $B C$ (obviously). Then $d$ has a common point with $C D$. Hence its right side provides a splitting line, and we get the contradiction again.","tier":0} +{"problem_type":"Combinatorics","problem_label":"C2","problem":"A unit square is dissected into $n>1$ rectangles such that their sides are parallel to the sides of the square. Any line, parallel to a side of the square and intersecting its interior, also intersects the interior of some rectangle. Prove that in this dissection, there exists a rectangle having no point on the boundary of the square. (Japan)","solution":"Again, we suppose the contrary. Consider an arbitrary counterexample. Then we know that each rectangle is attached to at least one side of the square. Observe that a rectangle cannot be attached to two opposite sides, otherwise one of its sides lies on a splitting line. We say that two rectangles are opposite if they are attached to opposite sides of $A B C D$. We claim that there exist two opposite rectangles having a common point. Consider the union $L$ of all rectangles attached to the left. Assume, to the contrary, that $L$ has no common point with the rectangles attached to the right. Take a polygonal line $p$ connecting the top and the bottom sides of the square and passing close from the right to the boundary of $L$ (see Figure 4). Then all its points belong to the rectangles attached either to the top or to the bottom. Moreover, the upper end-point of $p$ belongs to a rectangle attached to the top, and the lower one belongs to an other rectangle attached to the bottom. Hence, there is a point on $p$ where some rectangles attached to the top and to the bottom meet each other. So, there always exists a pair of neighboring opposite rectangles. ![](https:\/\/cdn.mathpix.com\/cropped\/2024_04_17_b8b307b9422db9068910g-30.jpg?height=417&width=414&top_left_y=1025&top_left_x=198) Figure 4 ![](https:\/\/cdn.mathpix.com\/cropped\/2024_04_17_b8b307b9422db9068910g-30.jpg?height=419&width=534&top_left_y=1024&top_left_x=675) Figure 5 ![](https:\/\/cdn.mathpix.com\/cropped\/2024_04_17_b8b307b9422db9068910g-30.jpg?height=546&width=576&top_left_y=955&top_left_x=1274) Figure 6 Now, take two opposite neighboring rectangles $a$ and $b$. We can assume that $a$ is attached to the left and $b$ is attached to the right. Let $X$ be their common point. If $X$ belongs to their horizontal sides (in particular, $X$ may appear to be a common vertex of $a$ and $b$ ), then these sides provide a splitting line (see Figure 5). Otherwise, $X$ lies on the vertical sides. Let $\\ell$ be the line containing these sides. Since $\\ell$ is not a splitting line, it intersects the interior of some rectangle. Let $c$ be such a rectangle, closest to $X$; we can assume that $c$ lies above $X$. Let $Y$ be the common point of $\\ell$ and the bottom side of $c$ (see Figure 6). Then $Y$ is also a vertex of two rectangles lying below $c$. So, let $Y$ be the upper-right and upper-left corners of the rectangles $a^{\\prime}$ and $b^{\\prime}$, respectively. Then $a^{\\prime}$ and $b^{\\prime}$ are situated not lower than $a$ and $b$, respectively (it may happen that $a=a^{\\prime}$ or $b=b^{\\prime}$ ). We claim that $a^{\\prime}$ is attached to the left. If $a=a^{\\prime}$ then of course it is. If $a \\neq a^{\\prime}$ then $a^{\\prime}$ is above $a$, below $c$ and to the left from $b^{\\prime}$. Hence, it can be attached to the left only. Analogously, $b^{\\prime}$ is attached to the right. Now, the top sides of these two rectangles pass through $Y$, hence they provide a splitting line again. This last contradiction completes the proof.","tier":0} +{"problem_type":"Combinatorics","problem_label":"C3","problem":"Find all positive integers $n$, for which the numbers in the set $S=\\{1,2, \\ldots, n\\}$ can be colored red and blue, with the following condition being satisfied: the set $S \\times S \\times S$ contains exactly 2007 ordered triples $(x, y, z)$ such that (i) $x, y, z$ are of the same color and (ii) $x+y+z$ is divisible by $n$. (Netherlands) Answer. $n=69$ and $n=84$.","solution":"Suppose that the numbers $1,2, \\ldots, n$ are colored red and blue. Denote by $R$ and $B$ the sets of red and blue numbers, respectively; let $|R|=r$ and $|B|=b=n-r$. Call a triple $(x, y, z) \\in S \\times S \\times S$ monochromatic if $x, y, z$ have the same color, and bichromatic otherwise. Call a triple $(x, y, z)$ divisible if $x+y+z$ is divisible by $n$. We claim that there are exactly $r^{2}-r b+b^{2}$ divisible monochromatic triples. For any pair $(x, y) \\in S \\times S$ there exists a unique $z_{x, y} \\in S$ such that the triple $\\left(x, y, z_{x, y}\\right)$ is divisible; so there are exactly $n^{2}$ divisible triples. Furthermore, if a divisible triple $(x, y, z)$ is bichromatic, then among $x, y, z$ there are either one blue and two red numbers, or vice versa. In both cases, exactly one of the pairs $(x, y),(y, z)$ and $(z, x)$ belongs to the set $R \\times B$. Assign such pair to the triple $(x, y, z)$. Conversely, consider any pair $(x, y) \\in R \\times B$, and denote $z=z_{x, y}$. Since $x \\neq y$, the triples $(x, y, z),(y, z, x)$ and $(z, x, y)$ are distinct, and $(x, y)$ is assigned to each of them. On the other hand, if $(x, y)$ is assigned to some triple, then this triple is clearly one of those mentioned above. So each pair in $R \\times B$ is assigned exactly three times. Thus, the number of bichromatic divisible triples is three times the number of elements in $R \\times B$, and the number of monochromatic ones is $n^{2}-3 r b=(r+b)^{2}-3 r b=r^{2}-r b+b^{2}$, as claimed. So, to find all values of $n$ for which the desired coloring is possible, we have to find all $n$, for which there exists a decomposition $n=r+b$ with $r^{2}-r b+b^{2}=2007$. Therefore, $9 \\mid r^{2}-r b+b^{2}=(r+b)^{2}-3 r b$. From this it consequently follows that $3|r+b, 3| r b$, and then $3|r, 3| b$. Set $r=3 s, b=3 c$. We can assume that $s \\geq c$. We have $s^{2}-s c+c^{2}=223$. Furthermore, $$ 892=4\\left(s^{2}-s c+c^{2}\\right)=(2 c-s)^{2}+3 s^{2} \\geq 3 s^{2} \\geq 3 s^{2}-3 c(s-c)=3\\left(s^{2}-s c+c^{2}\\right)=669 $$ so $297 \\geq s^{2} \\geq 223$ and $17 \\geq s \\geq 15$. If $s=15$ then $$ c(15-c)=c(s-c)=s^{2}-\\left(s^{2}-s c+c^{2}\\right)=15^{2}-223=2 $$ which is impossible for an integer $c$. In a similar way, if $s=16$ then $c(16-c)=33$, which is also impossible. Finally, if $s=17$ then $c(17-c)=66$, and the solutions are $c=6$ and $c=11$. Hence, $(r, b)=(51,18)$ or $(r, b)=(51,33)$, and the possible values of $n$ are $n=51+18=69$ and $n=51+33=84$. Comment. After the formula for the number of monochromatic divisible triples is found, the solution can be finished in various ways. The one presented is aimed to decrease the number of considered cases.","tier":0} +{"problem_type":"Combinatorics","problem_label":"C4","problem":"Let $A_{0}=\\left(a_{1}, \\ldots, a_{n}\\right)$ be a finite sequence of real numbers. For each $k \\geq 0$, from the sequence $A_{k}=\\left(x_{1}, \\ldots, x_{n}\\right)$ we construct a new sequence $A_{k+1}$ in the following way. 1. We choose a partition $\\{1, \\ldots, n\\}=I \\cup J$, where $I$ and $J$ are two disjoint sets, such that the expression $$ \\left|\\sum_{i \\in I} x_{i}-\\sum_{j \\in J} x_{j}\\right| $$ attains the smallest possible value. (We allow the sets $I$ or $J$ to be empty; in this case the corresponding sum is 0 .) If there are several such partitions, one is chosen arbitrarily. 2. We set $A_{k+1}=\\left(y_{1}, \\ldots, y_{n}\\right)$, where $y_{i}=x_{i}+1$ if $i \\in I$, and $y_{i}=x_{i}-1$ if $i \\in J$. Prove that for some $k$, the sequence $A_{k}$ contains an element $x$ such that $|x| \\geq n \/ 2$. (Iran)","solution":"Lemma. Suppose that all terms of the sequence $\\left(x_{1}, \\ldots, x_{n}\\right)$ satisfy the inequality $\\left|x_{i}\\right|1)$. By the induction hypothesis there exists a splitting $\\{1, \\ldots, n-1\\}=I^{\\prime} \\cup J^{\\prime}$ such that $$ \\left|\\sum_{i \\in I^{\\prime}} x_{i}-\\sum_{j \\in J^{\\prime}} x_{j}\\right|
n-2 \\cdot \\frac{n}{2}=0$. Thus we obtain $S_{q}>S_{q-1}>\\cdots>S_{p}$. This is impossible since $A_{p}=A_{q}$ and hence $S_{p}=S_{q}$.","tier":0} +{"problem_type":"Combinatorics","problem_label":"C5","problem":"In the Cartesian coordinate plane define the strip $S_{n}=\\{(x, y) \\mid n \\leq xb$. Then $a_{1}>b_{1} \\geq 1$, so $a_{1} \\geq 3$. Choose integers $k$ and $\\ell$ such that $k a_{1}-\\ell b_{1}=1$ and therefore $k a-\\ell b=d$. Since $a_{1}$ and $b_{1}$ are odd, $k+\\ell$ is odd as well. Hence, for every integer $n$, strips $S_{n}$ and $S_{n+k a-\\ell b}=S_{n+d}$ have opposite colors. This also implies that the coloring is periodic with period $2 d$, i.e. strips $S_{n}$ and $S_{n+2 d}$ have the same color for every $n$. ![](https:\/\/cdn.mathpix.com\/cropped\/2024_04_17_b8b307b9422db9068910g-33.jpg?height=573&width=671&top_left_y=1324&top_left_x=727) Figure 1 We will construct the desired rectangle $A B C D$ with $A B=C D=a$ and $B C=A D=b$ in a position such that vertex $A$ lies on the $x$-axis, and the projection of side $A B$ onto the $x$-axis is of length $2 d$ (see Figure 1). This is possible since $a=a_{1} d>2 d$. The coordinates of the vertices will have the forms $$ A=(t, 0), \\quad B=\\left(t+2 d, y_{1}\\right), \\quad C=\\left(u+2 d, y_{2}\\right), \\quad D=\\left(u, y_{3}\\right) . $$ Let $\\varphi=\\sqrt{a_{1}^{2}-4}$. By Pythagoras' theorem, $$ y_{1}=B B_{0}=\\sqrt{a^{2}-4 d^{2}}=d \\sqrt{a_{1}^{2}-4}=d \\varphi \\text {. } $$ So, by the similar triangles $A D D_{0}$ and $B A B_{0}$, we have the constraint $$ u-t=A D_{0}=\\frac{A D}{A B} \\cdot B B_{0}=\\frac{b d}{a} \\varphi $$ for numbers $t$ and $u$. Computing the numbers $y_{2}$ and $y_{3}$ is not required since they have no effect to the colors. Observe that the number $\\varphi$ is irrational, because $\\varphi^{2}$ is an integer, but $\\varphi$ is not: $a_{1}>\\varphi \\geq$ $\\sqrt{a_{1}^{2}-2 a_{1}+2}>a_{1}-1$. By the periodicity, points $A$ and $B$ have the same color; similarly, points $C$ and $D$ have the same color. Furthermore, these colors depend only on the values of $t$ and $u$. So it is sufficient to choose numbers $t$ and $u$ such that vertices $A$ and $D$ have the same color. Let $w$ be the largest positive integer such that there exist $w$ consecutive strips $S_{n_{0}}, S_{n_{0}+1}, \\ldots$, $S_{n_{0}+w-1}$ with the same color, say red. (Since $S_{n_{0}+d}$ must be blue, we have $w \\leq d$.) We will choose $t$ from the interval $\\left(n_{0}, n_{0}+w\\right)$. ![](https:\/\/cdn.mathpix.com\/cropped\/2024_04_17_b8b307b9422db9068910g-34.jpg?height=285&width=751&top_left_y=834&top_left_x=641) Figure 2 Consider the interval $I=\\left(n_{0}+\\frac{b d}{a} \\varphi, n_{0}+\\frac{b d}{a} \\varphi+w\\right)$ on the $x$-axis (see Figure 2). Its length is $w$, and the end-points are irrational. Therefore, this interval intersects $w+1$ consecutive strips. Since at most $w$ consecutive strips may have the same color, interval $I$ must contain both red and blue points. Choose $u \\in I$ such that the line $x=u$ is red and set $t=u-\\frac{b d}{a} \\varphi$, according to the constraint (1). Then $t \\in\\left(n_{0}, n_{0}+w\\right)$ and $A=(t, 0)$ is red as well as $D=\\left(u, y_{3}\\right)$. Hence, variables $u$ and $t$ can be set such that they provide a rectangle with four red vertices. Comment. The statement is false for squares, i.e. in the case $a=b$. If strips $S_{2 k a}, S_{2 k a+1}, \\ldots$, $S_{(2 k+1) a-1}$ are red, and strips $S_{(2 k+1) a}, S_{(2 k+1) a+1}, \\ldots, S_{(2 k+2) a-1}$ are blue for every integer $k$, then each square of size $a \\times a$ has at least one red and at least one blue vertex as well.","tier":0} +{"problem_type":"Combinatorics","problem_label":"C6","problem":"In a mathematical competition some competitors are friends; friendship is always mutual. Call a group of competitors a clique if each two of them are friends. The number of members in a clique is called its size. It is known that the largest size of cliques is even. Prove that the competitors can be arranged in two rooms such that the largest size of cliques in one room is the same as the largest size of cliques in the other room. (Russia)","solution":"We present an algorithm to arrange the competitors. Let the two rooms be Room $A$ and Room B. We start with an initial arrangement, and then we modify it several times by sending one person to the other room. At any state of the algorithm, $A$ and $B$ denote the sets of the competitors in the rooms, and $c(A)$ and $c(B)$ denote the largest sizes of cliques in the rooms, respectively. Step 1. Let $M$ be one of the cliques of largest size, $|M|=2 m$. Send all members of $M$ to Room $A$ and all other competitors to Room B. Since $M$ is a clique of the largest size, we have $c(A)=|M| \\geq c(B)$. Step 2. While $c(A)>c(B)$, send one person from Room $A$ to Room $B$. ![](https:\/\/cdn.mathpix.com\/cropped\/2024_04_17_b8b307b9422db9068910g-35.jpg?height=408&width=1028&top_left_y=1138&top_left_x=545) Note that $c(A)>c(B)$ implies that Room $A$ is not empty. In each step, $c(A)$ decreases by one and $c(B)$ increases by at most one. So at the end we have $c(A) \\leq c(B) \\leq c(A)+1$. We also have $c(A)=|A| \\geq m$ at the end. Otherwise we would have at least $m+1$ members of $M$ in Room $B$ and at most $m-1$ in Room $A$, implying $c(B)-c(A) \\geq(m+1)-(m-1)=2$. Step 3. Let $k=c(A)$. If $c(B)=k$ then $S T O P$. If we reached $c(A)=c(B)=k$ then we have found the desired arrangement. In all other cases we have $c(B)=k+1$. From the estimate above we also know that $k=|A|=|A \\cap M| \\geq m$ and $|B \\cap M| \\leq m$. Step 4. If there exists a competitor $x \\in B \\cap M$ and a clique $C \\subset B$ such that $|C|=k+1$ and $x \\notin C$, then move $x$ to Room $A$ and $S T O P$. ![](https:\/\/cdn.mathpix.com\/cropped\/2024_04_17_b8b307b9422db9068910g-35.jpg?height=408&width=1002&top_left_y=2143&top_left_x=561) After moving $x$ back to Room $A$, we will have $k+1$ members of $M$ in Room $A$, thus $c(A)=k+1$. Due to $x \\notin C, c(B)=|C|$ is not decreased, and after this step we have $c(A)=c(B)=k+1$. If there is no such competitor $x$, then in Room $B$, all cliques of size $k+1$ contain $B \\cap M$ as a subset. Step 5. While $c(B)=k+1$, choose a clique $C \\subset B$ such that $|C|=k+1$ and move one member of $C \\backslash M$ to Room $A$. ![](https:\/\/cdn.mathpix.com\/cropped\/2024_04_17_b8b307b9422db9068910g-36.jpg?height=411&width=1017&top_left_y=497&top_left_x=494) Note that $|C|=k+1>m \\geq|B \\cap M|$, so $C \\backslash M$ cannot be empty. Every time we move a single person from Room $B$ to Room $A$, so $c(B)$ decreases by at most 1. Hence, at the end of this loop we have $c(B)=k$. In Room $A$ we have the clique $A \\cap M$ with size $|A \\cap M|=k$ thus $c(A) \\geq k$. We prove that there is no clique of larger size there. Let $Q \\subset A$ be an arbitrary clique. We show that $|Q| \\leq k$. ![](https:\/\/cdn.mathpix.com\/cropped\/2024_04_17_b8b307b9422db9068910g-36.jpg?height=411&width=1017&top_left_y=1185&top_left_x=494) In Room $A$, and specially in set $Q$, there can be two types of competitors: - Some members of $M$. Since $M$ is a clique, they are friends with all members of $B \\cap M$. - Competitors which were moved to Room $A$ in Step 5. Each of them has been in a clique with $B \\cap M$ so they are also friends with all members of $B \\cap M$. Hence, all members of $Q$ are friends with all members of $B \\cap M$. Sets $Q$ and $B \\cap M$ are cliques themselves, so $Q \\cup(B \\cap M)$ is also a clique. Since $M$ is a clique of the largest size, $$ |M| \\geq|Q \\cup(B \\cap M)|=|Q|+|B \\cap M|=|Q|+|M|-|A \\cap M| $$ therefore $$ |Q| \\leq|A \\cap M|=k $$ Finally, after Step 5 we have $c(A)=c(B)=k$. Comment. Obviously, the statement is false without the assumption that the largest clique size is even.","tier":0} +{"problem_type":"Combinatorics","problem_label":"C7","problem":"Let $\\alpha<\\frac{3-\\sqrt{5}}{2}$ be a positive real number. Prove that there exist positive integers $n$ and $p>\\alpha \\cdot 2^{n}$ for which one can select $2 p$ pairwise distinct subsets $S_{1}, \\ldots, S_{p}, T_{1}, \\ldots, T_{p}$ of the set $\\{1,2, \\ldots, n\\}$ such that $S_{i} \\cap T_{j} \\neq \\varnothing$ for all $1 \\leq i, j \\leq p$. (Austria)","solution":"Let $k$ and $m$ be positive integers (to be determined later) and set $n=k m$. Decompose the set $\\{1,2, \\ldots, n\\}$ into $k$ disjoint subsets, each of size $m$; denote these subsets by $A_{1}, \\ldots, A_{k}$. Define the following families of sets: $$ \\begin{aligned} \\mathcal{S} & =\\left\\{S \\subset\\{1,2, \\ldots, n\\}: \\forall i S \\cap A_{i} \\neq \\varnothing\\right\\} \\\\ \\mathcal{T}_{1} & =\\left\\{T \\subset\\{1,2, \\ldots, n\\}: \\quad \\exists i A_{i} \\subset T\\right\\}, \\quad \\mathcal{T}=\\mathcal{T}_{1} \\backslash \\mathcal{S} . \\end{aligned} $$ For each set $T \\in \\mathcal{T} \\subset \\mathcal{T}_{1}$, there exists an index $1 \\leq i \\leq k$ such that $A_{i} \\subset T$. Then for all $S \\in \\mathcal{S}$, $S \\cap T \\supset S \\cap A_{i} \\neq \\varnothing$. Hence, each $S \\in \\mathcal{S}$ and each $T \\in \\mathcal{T}$ have at least one common element. Below we show that the numbers $m$ and $k$ can be chosen such that $|\\mathcal{S}|,|\\mathcal{T}|>\\alpha \\cdot 2^{n}$. Then, choosing $p=\\min \\{|\\mathcal{S}|,|\\mathcal{T}|\\}$, one can select the desired $2 p$ sets $S_{1}, \\ldots, S_{p}$ and $T_{1}, \\ldots, T_{p}$ from families $\\mathcal{S}$ and $\\mathcal{T}$, respectively. Since families $\\mathcal{S}$ and $\\mathcal{T}$ are disjoint, sets $S_{i}$ and $T_{j}$ will be pairwise distinct. To count the sets $S \\in \\mathcal{S}$, observe that each $A_{i}$ has $2^{m}-1$ nonempty subsets so we have $2^{m}-1$ choices for $S \\cap A_{i}$. These intersections uniquely determine set $S$, so $$ |\\mathcal{S}|=\\left(2^{m}-1\\right)^{k} $$ Similarly, if a set $H \\subset\\{1,2, \\ldots, n\\}$ does not contain a certain set $A_{i}$ then we have $2^{m}-1$ choices for $H \\cap A_{i}$ : all subsets of $A_{i}$, except $A_{i}$ itself. Therefore, the complement of $\\mathcal{T}_{1}$ contains $\\left(2^{m}-1\\right)^{k}$ sets and $$ \\left|\\mathcal{T}_{1}\\right|=2^{k m}-\\left(2^{m}-1\\right)^{k} . $$ Next consider the family $\\mathcal{S} \\backslash \\mathcal{T}_{1}$. If a set $S$ intersects all $A_{i}$ but does not contain any of them, then there exists $2^{m}-2$ possible values for each $S \\cap A_{i}$ : all subsets of $A_{i}$ except $\\varnothing$ and $A_{i}$. Therefore the number of such sets $S$ is $\\left(2^{m}-2\\right)^{k}$, so $$ \\left|\\mathcal{S} \\backslash \\mathcal{T}_{1}\\right|=\\left(2^{m}-2\\right)^{k} $$ From (1), (2), and (3) we obtain $$ |\\mathcal{T}|=\\left|\\mathcal{T}_{1}\\right|-\\left|\\mathcal{S} \\cap \\mathcal{T}_{1}\\right|=\\left|\\mathcal{T}_{1}\\right|-\\left(|\\mathcal{S}|-\\left|\\mathcal{S} \\backslash \\mathcal{T}_{1}\\right|\\right)=2^{k m}-2\\left(2^{m}-1\\right)^{k}+\\left(2^{m}-2\\right)^{k} $$ Let $\\delta=\\frac{3-\\sqrt{5}}{2}$ and $k=k(m)=\\left[2^{m} \\log \\frac{1}{\\delta}\\right]$. Then $$ \\lim _{m \\rightarrow \\infty} \\frac{|\\mathcal{S}|}{2^{k m}}=\\lim _{m \\rightarrow \\infty}\\left(1-\\frac{1}{2^{m}}\\right)^{k}=\\exp \\left(-\\lim _{m \\rightarrow \\infty} \\frac{k}{2^{m}}\\right)=\\delta $$ and similarly $$ \\lim _{m \\rightarrow \\infty} \\frac{|\\mathcal{T}|}{2^{k m}}=1-2 \\lim _{m \\rightarrow \\infty}\\left(1-\\frac{1}{2^{m}}\\right)^{k}+\\lim _{m \\rightarrow \\infty}\\left(1-\\frac{2}{2^{m}}\\right)^{k}=1-2 \\delta+\\delta^{2}=\\delta $$ Hence, if $m$ is sufficiently large then $\\frac{|\\mathcal{S}|}{2^{m k}}$ and $\\frac{|\\mathcal{T}|}{2^{m k}}$ are greater than $\\alpha$ (since $\\alpha<\\delta$ ). So $|\\mathcal{S}|,|\\mathcal{T}|>\\alpha \\cdot 2^{m k}=\\alpha \\cdot 2^{n}$. Comment. It can be proved that the constant $\\frac{3-\\sqrt{5}}{2}$ is sharp. Actually, if $S_{1}, \\ldots, S_{p}, T_{1}, \\ldots, T_{p}$ are distinct subsets of $\\{1,2, \\ldots, n\\}$ such that each $S_{i}$ intersects each $T_{j}$, then $p<\\frac{3-\\sqrt{5}}{2} \\cdot 2^{n}$.","tier":0} +{"problem_type":"Combinatorics","problem_label":"C8","problem":"Given a convex $n$-gon $P$ in the plane. For every three vertices of $P$, consider the triangle determined by them. Call such a triangle good if all its sides are of unit length. Prove that there are not more than $\\frac{2}{3} n$ good triangles. (Ukraine)","solution":"Consider all good triangles containing a certain vertex $A$. The other two vertices of any such triangle lie on the circle $\\omega_{A}$ with unit radius and center $A$. Since $P$ is convex, all these vertices lie on an arc of angle less than $180^{\\circ}$. Let $L_{A} R_{A}$ be the shortest such arc, oriented clockwise (see Figure 1). Each of segments $A L_{A}$ and $A R_{A}$ belongs to a unique good triangle. We say that the good triangle with side $A L_{A}$ is assigned counterclockwise to $A$, and the second one, with side $A R_{A}$, is assigned clockwise to $A$. In those cases when there is a single good triangle containing vertex $A$, this triangle is assigned to $A$ twice. There are at most two assignments to each vertex of the polygon. (Vertices which do not belong to any good triangle have no assignment.) So the number of assignments is at most $2 n$. Consider an arbitrary good triangle $A B C$, with vertices arranged clockwise. We prove that $A B C$ is assigned to its vertices at least three times. Then, denoting the number of good triangles by $t$, we obtain that the number $K$ of all assignments is at most $2 n$, while it is not less than $3 t$. Then $3 t \\leq K \\leq 2 n$, as required. Actually, we prove that triangle $A B C$ is assigned either counterclockwise to $C$ or clockwise to $B$. Then, by the cyclic symmetry of the vertices, we obtain that triangle $A B C$ is assigned either counterclockwise to $A$ or clockwise to $C$, and either counterclockwise to $B$ or clockwise to $A$, providing the claim. ![](https:\/\/cdn.mathpix.com\/cropped\/2024_04_17_b8b307b9422db9068910g-38.jpg?height=586&width=678&top_left_y=1369&top_left_x=290) Figure 1 ![](https:\/\/cdn.mathpix.com\/cropped\/2024_04_17_b8b307b9422db9068910g-38.jpg?height=549&width=617&top_left_y=1393&top_left_x=1119) Figure 2 Assume, to the contrary, that $L_{C} \\neq A$ and $R_{B} \\neq A$. Denote by $A^{\\prime}, B^{\\prime}, C^{\\prime}$ the intersection points of circles $\\omega_{A}, \\omega_{B}$ and $\\omega_{C}$, distinct from $A, B, C$ (see Figure 2). Let $C L_{C} L_{C}^{\\prime}$ be the good triangle containing $C L_{C}$. Observe that the angle of $\\operatorname{arc} L_{C} A$ is less than $120^{\\circ}$. Then one of the points $L_{C}$ and $L_{C}^{\\prime}$ belongs to $\\operatorname{arc} B^{\\prime} A$ of $\\omega_{C}$; let this point be $X$. In the case when $L_{C}=B^{\\prime}$ and $L_{C}^{\\prime}=A$, choose $X=B^{\\prime}$. Analogously, considering the good triangle $B R_{B}^{\\prime} R_{B}$ which contains $B R_{B}$ as an edge, we see that one of the points $R_{B}$ and $R_{B}^{\\prime}$ lies on arc $A C^{\\prime}$ of $\\omega_{B}$. Denote this point by $Y, Y \\neq A$. Then angles $X A Y, Y A B, B A C$ and $C A X$ (oriented clockwise) are not greater than $180^{\\circ}$. Hence, point $A$ lies in quadrilateral $X Y B C$ (either in its interior or on segment $X Y$ ). This is impossible, since all these five points are vertices of $P$. Hence, each good triangle has at least three assignments, and the statement is proved. Comment 1. Considering a diameter $A B$ of the polygon, one can prove that every good triangle containing either $A$ or $B$ has at least four assignments. This observation leads to $t \\leq\\left\\lfloor\\frac{2}{3}(n-1)\\right\\rfloor$. Comment 2. The result $t \\leq\\left\\lfloor\\frac{2}{3}(n-1)\\right\\rfloor$ is sharp. To construct a polygon with $n=3 k+1$ vertices and $t=2 k$ triangles, take a rhombus $A B_{1} C_{1} D_{1}$ with unit side length and $\\angle B_{1}=60^{\\circ}$. Then rotate it around $A$ by small angles obtaining rhombi $A B_{2} C_{2} D_{2}, \\ldots, A B_{k} C_{k} D_{k}$ (see Figure 3). The polygon $A B_{1} \\ldots B_{k} C_{1} \\ldots C_{k} D_{1} \\ldots D_{k}$ has $3 k+1$ vertices and contains $2 k$ good triangles. The construction for $n=3 k$ and $n=3 k-1$ can be obtained by deleting vertices $D_{n}$ and $D_{n-1}$. ![](https:\/\/cdn.mathpix.com\/cropped\/2024_04_17_b8b307b9422db9068910g-39.jpg?height=331&width=602&top_left_y=311&top_left_x=1298) Figure 3","tier":0} +{"problem_type":"Geometry","problem_label":"G1","problem":"In triangle $A B C$, the angle bisector at vertex $C$ intersects the circumcircle and the perpendicular bisectors of sides $B C$ and $C A$ at points $R, P$, and $Q$, respectively. The midpoints of $B C$ and $C A$ are $S$ and $T$, respectively. Prove that triangles $R Q T$ and $R P S$ have the same area. (Czech Republic)","solution":"If $A C=B C$ then triangle $A B C$ is isosceles, triangles $R Q T$ and $R P S$ are symmetric about the bisector $C R$ and the statement is trivial. If $A C \\neq B C$ then it can be assumed without loss of generality that $A C\\sqrt{E C^{2}-L C^{2}}=L E . $$ Since quadrilateral $B C E D$ is cyclic, we have $\\angle E D C=\\angle E B C$, so the right triangles $B E L$ and $D E K$ are similar. Then $K E>L E$ implies $D K>B L$, and hence $$ D F=D K-K F>B L-L C=B C=A D \\text {. } $$ But triangles $A D F$ and $G C F$ are similar, so we have $1>\\frac{A D}{D F}=\\frac{G C}{C F}$; this contradicts our assumption. The case $C F>G C$ is completely similar. We consequently obtain the converse inequalities $K F>L C, K E1$; also, lines $A_{1} C^{\\prime}$ and $C A$ intersect at a point $Z$ on the extension of $C A$ beyond point $A$, hence $\\frac{\\left[A_{1} B_{1} C^{\\prime}\\right]}{\\left[A_{1} B^{\\prime} C^{\\prime}\\right]}=\\frac{B_{1} Z}{B^{\\prime} Z}>1$. Finally, since $A_{1} A^{\\prime} \\| B^{\\prime} C^{\\prime}$, we have $\\left[A_{1} B_{1} C_{1}\\right]>\\left[A_{1} B_{1} C^{\\prime}\\right]>\\left[A_{1} B^{\\prime} C^{\\prime}\\right]=\\left[A^{\\prime} B^{\\prime} C^{\\prime}\\right]=\\frac{1}{4}[A B C]$. Now, from $\\left[A_{1} B_{1} C_{1}\\right]+\\left[A C_{1} B_{1}\\right]+\\left[B A_{1} C_{1}\\right]+\\left[C B_{1} A_{1}\\right]=[A B C]$ we obtain that one of the remaining triangles $A C_{1} B_{1}, B A_{1} C_{1}, C B_{1} A_{1}$ has an area less than $\\frac{1}{4}[A B C]$, so it is less than $\\left[A_{1} B_{1} C_{1}\\right]$. Now we return to the problem. We say that triangle $A_{1} B_{1} C_{1}$ is small if $\\left[A_{1} B_{1} C_{1}\\right]$ is less than each of $\\left[B B_{1} A_{1}\\right]$ and $\\left[C C_{1} B_{1}\\right]$; otherwise this triangle is big (the similar notion is introduced for triangles $B_{1} C_{1} D_{1}, C_{1} D_{1} A_{1}, D_{1} A_{1} B_{1}$ ). If both triangles $A_{1} B_{1} C_{1}$ and $C_{1} D_{1} A_{1}$ are big, then $\\left[A_{1} B_{1} C_{1}\\right]$ is not less than the area of some border triangle, and $\\left[C_{1} D_{1} A_{1}\\right]$ is not less than the area of another one; hence, $S_{1}=\\left[A_{1} B_{1} C_{1}\\right]+\\left[C_{1} D_{1} A_{1}\\right] \\geq S$. The same is valid for the pair of $B_{1} C_{1} D_{1}$ and $D_{1} A_{1} B_{1}$. So it is sufficient to prove that in one of these pairs both triangles are big. Suppose the contrary. Then there is a small triangle in each pair. Without loss of generality, assume that triangles $A_{1} B_{1} C_{1}$ and $D_{1} A_{1} B_{1}$ are small. We can assume also that $\\left[A_{1} B_{1} C_{1}\\right] \\leq$ $\\left[D_{1} A_{1} B_{1}\\right]$. Note that in this case ray $D_{1} C_{1}$ intersects line $B C$. Consider two cases. ![](https:\/\/cdn.mathpix.com\/cropped\/2024_04_17_b8b307b9422db9068910g-48.jpg?height=448&width=648&top_left_y=987&top_left_x=224) Figure 4 ![](https:\/\/cdn.mathpix.com\/cropped\/2024_04_17_b8b307b9422db9068910g-48.jpg?height=420&width=886&top_left_y=1001&top_left_x=899) Figure 5 Case 1. Ray $C_{1} D_{1}$ intersects line $A B$ at some point $K$. Let ray $D_{1} C_{1}$ intersect line $B C$ at point $L$ (see Figure 4). Then we have $\\left[A_{1} B_{1} C_{1}\\right]<\\left[C C_{1} B_{1}\\right]<\\left[L C_{1} B_{1}\\right],\\left[A_{1} B_{1} C_{1}\\right]<\\left[B B_{1} A_{1}\\right]$ (both - since $\\left[A_{1} B_{1} C_{1}\\right]$ is small), and $\\left[A_{1} B_{1} C_{1}\\right] \\leq\\left[D_{1} A_{1} B_{1}\\right]<\\left[A A_{1} D_{1}\\right]<\\left[K A_{1} D_{1}\\right]<\\left[K A_{1} C_{1}\\right]$ (since triangle $D_{1} A_{1} B_{1}$ is small). This contradicts the Lemma, applied for triangle $A_{1} B_{1} C_{1}$ inside $L K B$. Case 2. Ray $C_{1} D_{1}$ does not intersect $A B$. Then choose a \"sufficiently far\" point $K$ on ray $B A$ such that $\\left[K A_{1} C_{1}\\right]>\\left[A_{1} B_{1} C_{1}\\right]$, and that ray $K C_{1}$ intersects line $B C$ at some point $L$ (see Figure 5). Since ray $C_{1} D_{1}$ does not intersect line $A B$, the points $A$ and $D_{1}$ are on different sides of $K L$; then $A$ and $D$ are also on different sides, and $C$ is on the same side as $A$ and $B$. Then analogously we have $\\left[A_{1} B_{1} C_{1}\\right]<\\left[C C_{1} B_{1}\\right]<\\left[L C_{1} B_{1}\\right]$ and $\\left[A_{1} B_{1} C_{1}\\right]<\\left[B B_{1} A_{1}\\right]$ since triangle $A_{1} B_{1} C_{1}$ is small. This (together with $\\left[A_{1} B_{1} C_{1}\\right]<\\left[K A_{1} C_{1}\\right]$ ) contradicts the Lemma again.","tier":0} +{"problem_type":"Geometry","problem_label":"G7","problem":"Given an acute triangle $A B C$ with angles $\\alpha, \\beta$ and $\\gamma$ at vertices $A, B$ and $C$, respectively, such that $\\beta>\\gamma$. Point $I$ is the incenter, and $R$ is the circumradius. Point $D$ is the foot of the altitude from vertex $A$. Point $K$ lies on line $A D$ such that $A K=2 R$, and $D$ separates $A$ and $K$. Finally, lines $D I$ and $K I$ meet sides $A C$ and $B C$ at $E$ and $F$, respectively. Prove that if $I E=I F$ then $\\beta \\leq 3 \\gamma$. (Iran)","solution":"We first prove that $$ \\angle K I D=\\frac{\\beta-\\gamma}{2} $$ even without the assumption that $I E=I F$. Then we will show that the statement of the problem is a consequence of this fact. Denote the circumcenter by $O$. On the circumcircle, let $P$ be the point opposite to $A$, and let the angle bisector $A I$ intersect the circle again at $M$. Since $A K=A P=2 R$, triangle $A K P$ is isosceles. It is known that $\\angle B A D=\\angle C A O$, hence $\\angle D A I=\\angle B A I-\\angle B A D=\\angle C A I-$ $\\angle C A O=\\angle O A I$, and $A M$ is the bisector line in triangle $A K P$. Therefore, points $K$ and $P$ are symmetrical about $A M$, and $\\angle A M K=\\angle A M P=90^{\\circ}$. Thus, $M$ is the midpoint of $K P$, and $A M$ is the perpendicular bisector of $K P$. ![](https:\/\/cdn.mathpix.com\/cropped\/2024_04_17_b8b307b9422db9068910g-50.jpg?height=1082&width=962&top_left_y=1252&top_left_x=524) Denote the perpendicular feet of incenter $I$ on lines $B C, A C$, and $A D$ by $A_{1}, B_{1}$, and $T$, respectively. Quadrilateral $D A_{1} I T$ is a rectangle, hence $T D=I A_{1}=I B_{1}$. Due to the right angles at $T$ and $B_{1}$, quadrilateral $A B_{1} I T$ is cyclic. Hence $\\angle B_{1} T I=$ $\\angle B_{1} A I=\\angle C A M=\\angle B A M=\\angle B P M$ and $\\angle I B_{1} T=\\angle I A T=\\angle M A K=\\angle M A P=$ $\\angle M B P$. Therefore, triangles $B_{1} T I$ and $B P M$ are similar and $\\frac{I T}{I B_{1}}=\\frac{M P}{M B}$. It is well-known that $M B=M C=M I$. Then right triangles $I T D$ and $K M I$ are also similar, because $\\frac{I T}{T D}=\\frac{I T}{I B_{1}}=\\frac{M P}{M B}=\\frac{K M}{M I}$. Hence, $\\angle K I M=\\angle I D T=\\angle I D A$, and $$ \\angle K I D=\\angle M I D-\\angle K I M=(\\angle I A D+\\angle I D A)-\\angle I D A=\\angle I A D . $$ Finally, from the right triangle $A D B$ we can compute $$ \\angle K I D=\\angle I A D=\\angle I A B-\\angle D A B=\\frac{\\alpha}{2}-\\left(90^{\\circ}-\\beta\\right)=\\frac{\\alpha}{2}-\\frac{\\alpha+\\beta+\\gamma}{2}+\\beta=\\frac{\\beta-\\gamma}{2} . $$ Now let us turn to the statement and suppose that $I E=I F$. Since $I A_{1}=I B_{1}$, the right triangles $I E B_{1}$ and $I F A_{1}$ are congruent and $\\angle I E B_{1}=\\angle I F A_{1}$. Since $\\beta>\\gamma, A_{1}$ lies in the interior of segment $C D$ and $F$ lies in the interior of $A_{1} D$. Hence, $\\angle I F C$ is acute. Then two cases are possible depending on the order of points $A, C, B_{1}$ and $E$. ![](https:\/\/cdn.mathpix.com\/cropped\/2024_04_17_b8b307b9422db9068910g-51.jpg?height=818&width=1486&top_left_y=907&top_left_x=317) If point $E$ lies between $C$ and $B_{1}$ then $\\angle I F C=\\angle I E A$, hence quadrilateral $C E I F$ is cyclic and $\\angle F C E=180^{\\circ}-\\angle E I F=\\angle K I D$. By (1), in this case we obtain $\\angle F C E=\\gamma=\\angle K I D=$ $\\frac{\\beta-\\gamma}{2}$ and $\\beta=3 \\gamma$. Otherwise, if point $E$ lies between $A$ and $B_{1}$, quadrilateral $C E I F$ is a deltoid such that $\\angle I E C=\\angle I F C<90^{\\circ}$. Then we have $\\angle F C E>180^{\\circ}-\\angle E I F=\\angle K I D$. Therefore, $\\angle F C E=\\gamma>\\angle K I D=\\frac{\\beta-\\gamma}{2}$ and $\\beta<3 \\gamma$. Comment 1. In the case when quadrilateral $C E I F$ is a deltoid, one can prove the desired inequality without using (1). Actually, from $\\angle I E C=\\angle I F C<90^{\\circ}$ it follows that $\\angle A D I=90^{\\circ}-\\angle E D C<$ $\\angle A E D-\\angle E D C=\\gamma$. Since the incircle lies inside triangle $A B C$, we have $A D>2 r$ (here $r$ is the inradius), which implies $D T\\gamma$. Point $I$ is the incenter, and $R$ is the circumradius. Point $D$ is the foot of the altitude from vertex $A$. Point $K$ lies on line $A D$ such that $A K=2 R$, and $D$ separates $A$ and $K$. Finally, lines $D I$ and $K I$ meet sides $A C$ and $B C$ at $E$ and $F$, respectively. Prove that if $I E=I F$ then $\\beta \\leq 3 \\gamma$. (Iran)","solution":"We give a different proof for (1). Then the solution can be finished in the same way as above. Define points $M$ and $P$ again; it can be proved in the same way that $A M$ is the perpendicular bisector of $K P$. Let $J$ be the center of the excircle touching side $B C$. It is well-known that points $B, C, I, J$ lie on a circle with center $M$; denote this circle by $\\omega_{1}$. Let $B^{\\prime}$ be the reflection of point $B$ about the angle bisector $A M$. By the symmetry, $B^{\\prime}$ is the second intersection point of circle $\\omega_{1}$ and line $A C$. Triangles $P B A$ and $K B^{\\prime} A$ are symmetrical with respect to line $A M$, therefore $\\angle K B^{\\prime} A=\\angle P B A=90^{\\circ}$. By the right angles at $D$ and $B^{\\prime}$, points $K, D, B^{\\prime}, C$ are concyclic and $$ A D \\cdot A K=A B^{\\prime} \\cdot A C . $$ From the cyclic quadrilateral $I J C B^{\\prime}$ we obtain $A B^{\\prime} \\cdot A C=A I \\cdot A J$ as well, therefore $$ A D \\cdot A K=A B^{\\prime} \\cdot A C=A I \\cdot A J $$ and points $I, J, K, D$ are also concyclic. Denote circle $I D K J$ by $\\omega_{2}$. ![](https:\/\/cdn.mathpix.com\/cropped\/2024_04_17_b8b307b9422db9068910g-52.jpg?height=997&width=1145&top_left_y=798&top_left_x=433) Let $N$ be the point on circle $\\omega_{2}$ which is opposite to $K$. Since $\\angle N D K=90^{\\circ}=\\angle C D K$, point $N$ lies on line $B C$. Point $M$, being the center of circle $\\omega_{1}$, is the midpoint of segment $I J$, and $K M$ is perpendicular to $I J$. Therefore, line $K M$ is the perpendicular bisector of $I J$ and hence it passes through $N$. From the cyclic quadrilateral $I D K N$ we obtain $$ \\angle K I D=\\angle K N D=90^{\\circ}-\\angle D K N=90^{\\circ}-\\angle A K M=\\angle M A K=\\frac{\\beta-\\gamma}{2} $$ Comment 2. The main difficulty in the solution is finding (1). If someone can guess this fact, he or she can compute it in a relatively short way. One possible way is finding and applying the relation $A I^{2}=2 R\\left(h_{a}-2 r\\right)$, where $h_{a}=A D$ is the length of the altitude. Using this fact, one can see that triangles $A K I$ and $A I D^{\\prime}$ are similar (here $D^{\\prime}$ is the point symmetrical to $D$ about $T$ ). Hence, $\\angle M I K=\\angle D D^{\\prime} I=\\angle I D D^{\\prime}$. The proof can be finished as in Solution 1.","tier":0} +{"problem_type":"Geometry","problem_label":"G8","problem":"Point $P$ lies on side $A B$ of a convex quadrilateral $A B C D$. Let $\\omega$ be the incircle of triangle $C P D$, and let $I$ be its incenter. Suppose that $\\omega$ is tangent to the incircles of triangles $A P D$ and $B P C$ at points $K$ and $L$, respectively. Let lines $A C$ and $B D$ meet at $E$, and let lines $A K$ and $B L$ meet at $F$. Prove that points $E, I$, and $F$ are collinear. (Poland)","solution":"Let $\\Omega$ be the circle tangent to segment $A B$ and to rays $A D$ and $B C$; let $J$ be its center. We prove that points $E$ and $F$ lie on line $I J$. ![](https:\/\/cdn.mathpix.com\/cropped\/2024_04_17_b8b307b9422db9068910g-53.jpg?height=682&width=920&top_left_y=647&top_left_x=611) Denote the incircles of triangles $A D P$ and $B C P$ by $\\omega_{A}$ and $\\omega_{B}$. Let $h_{1}$ be the homothety with a negative scale taking $\\omega$ to $\\Omega$. Consider this homothety as the composition of two homotheties: one taking $\\omega$ to $\\omega_{A}$ (with a negative scale and center $K$ ), and another one taking $\\omega_{A}$ to $\\Omega$ (with a positive scale and center $A$ ). It is known that in such a case the three centers of homothety are collinear (this theorem is also referred to as the theorem on the three similitude centers). Hence, the center of $h_{1}$ lies on line $A K$. Analogously, it also lies on $B L$, so this center is $F$. Hence, $F$ lies on the line of centers of $\\omega$ and $\\Omega$, i. e. on $I J$ (if $I=J$, then $F=I$ as well, and the claim is obvious). Consider quadrilateral $A P C D$ and mark the equal segments of tangents to $\\omega$ and $\\omega_{A}$ (see the figure below to the left). Since circles $\\omega$ and $\\omega_{A}$ have a common point of tangency with $P D$, one can easily see that $A D+P C=A P+C D$. So, quadrilateral $A P C D$ is circumscribed; analogously, circumscribed is also quadrilateral $B C D P$. Let $\\Omega_{A}$ and $\\Omega_{B}$ respectively be their incircles. ![](https:\/\/cdn.mathpix.com\/cropped\/2024_04_17_b8b307b9422db9068910g-53.jpg?height=754&width=1714&top_left_y=1962&top_left_x=223) Consider the homothety $h_{2}$ with a positive scale taking $\\omega$ to $\\Omega$. Consider $h_{2}$ as the composition of two homotheties: taking $\\omega$ to $\\Omega_{A}$ (with a positive scale and center $C$ ), and taking $\\Omega_{A}$ to $\\Omega$ (with a positive scale and center $A$ ), respectively. So the center of $h_{2}$ lies on line $A C$. By analogous reasons, it lies also on $B D$, hence this center is $E$. Thus, $E$ also lies on the line of centers $I J$, and the claim is proved. Comment. In both main steps of the solution, there can be several different reasonings for the same claims. For instance, one can mostly use Desargues' theorem instead of the three homotheties theorem. Namely, if $I_{A}$ and $I_{B}$ are the centers of $\\omega_{A}$ and $\\omega_{B}$, then lines $I_{A} I_{B}, K L$ and $A B$ are concurrent (by the theorem on three similitude centers applied to $\\omega, \\omega_{A}$ and $\\omega_{B}$ ). Then Desargues' theorem, applied to triangles $A I_{A} K$ and $B I_{B} L$, yields that the points $J=A I_{A} \\cap B I_{B}, I=I_{A} K \\cap I_{B} L$ and $F=A K \\cap B L$ are collinear. For the second step, let $J_{A}$ and $J_{B}$ be the centers of $\\Omega_{A}$ and $\\Omega_{B}$. Then lines $J_{A} J_{B}, A B$ and $C D$ are concurrent, since they appear to be the two common tangents and the line of centers of $\\Omega_{A}$ and $\\Omega_{B}$. Applying Desargues' theorem to triangles $A J_{A} C$ and $B J_{B} D$, we obtain that the points $J=A J_{A} \\cap B J_{B}$, $I=C J_{A} \\cap D J_{B}$ and $E=A C \\cap B D$ are collinear.","tier":0} +{"problem_type":"Number Theory","problem_label":"N1","problem":"Find all pairs $(k, n)$ of positive integers for which $7^{k}-3^{n}$ divides $k^{4}+n^{2}$. (Austria) Answer. $(2,4)$.","solution":"Suppose that a pair $(k, n)$ satisfies the condition of the problem. Since $7^{k}-3^{n}$ is even, $k^{4}+n^{2}$ is also even, hence $k$ and $n$ have the same parity. If $k$ and $n$ are odd, then $k^{4}+n^{2} \\equiv 1+1=2(\\bmod 4)$, while $7^{k}-3^{n} \\equiv 7-3 \\equiv 0(\\bmod 4)$, so $k^{4}+n^{2}$ cannot be divisible by $7^{k}-3^{n}$. Hence, both $k$ and $n$ must be even. Write $k=2 a, n=2 b$. Then $7^{k}-3^{n}=7^{2 a}-3^{2 b}=\\frac{7^{a}-3^{b}}{2} \\cdot 2\\left(7^{a}+3^{b}\\right)$, and both factors are integers. So $2\\left(7^{a}+3^{b}\\right) \\mid 7^{k}-3^{n}$ and $7^{k}-3^{n} \\mid k^{4}+n^{2}=2\\left(8 a^{4}+2 b^{2}\\right)$, hence $$ 7^{a}+3^{b} \\leq 8 a^{4}+2 b^{2} . $$ We prove by induction that $8 a^{4}<7^{a}$ for $a \\geq 4,2 b^{2}<3^{b}$ for $b \\geq 1$ and $2 b^{2}+9 \\leq 3^{b}$ for $b \\geq 3$. In the initial cases $a=4, b=1, b=2$ and $b=3$ we have $8 \\cdot 4^{4}=2048<7^{4}=2401,2<3$, $2 \\cdot 2^{2}=8<3^{2}=9$ and $2 \\cdot 3^{2}+9=3^{3}=27$, respectively. If $8 a^{4}<7^{a}(a \\geq 4)$ and $2 b^{2}+9 \\leq 3^{b}(b \\geq 3)$, then $$ \\begin{aligned} 8(a+1)^{4} & =8 a^{4}\\left(\\frac{a+1}{a}\\right)^{4}<7^{a}\\left(\\frac{5}{4}\\right)^{4}=7^{a} \\frac{625}{256}<7^{a+1} \\quad \\text { and } \\\\ 2(b+1)^{2}+9 & <\\left(2 b^{2}+9\\right)\\left(\\frac{b+1}{b}\\right)^{2} \\leq 3^{b}\\left(\\frac{4}{3}\\right)^{2}=3^{b} \\frac{16}{9}<3^{b+1}, \\end{aligned} $$ as desired. For $a \\geq 4$ we obtain $7^{a}+3^{b}>8 a^{4}+2 b^{2}$ and inequality (1) cannot hold. Hence $a \\leq 3$, and three cases are possible. Case 1: $a=1$. Then $k=2$ and $8+2 b^{2} \\geq 7+3^{b}$, thus $2 b^{2}+1 \\geq 3^{b}$. This is possible only if $b \\leq 2$. If $b=1$ then $n=2$ and $\\frac{k^{4}+n^{2}}{7^{k}-3^{n}}=\\frac{2^{4}+2^{2}}{7^{2}-3^{2}}=\\frac{1}{2}$, which is not an integer. If $b=2$ then $n=4$ and $\\frac{k^{4}+n^{2}}{7^{k}-3^{n}}=\\frac{2^{4}+4^{2}}{7^{2}-3^{4}}=-1$, so $(k, n)=(2,4)$ is a solution. Case 2: $a=2$. Then $k=4$ and $k^{4}+n^{2}=256+4 b^{2} \\geq\\left|7^{4}-3^{n}\\right|=\\left|49-3^{b}\\right| \\cdot\\left(49+3^{b}\\right)$. The smallest value of the first factor is 22 , attained at $b=3$, so $128+2 b^{2} \\geq 11\\left(49+3^{b}\\right)$, which is impossible since $3^{b}>2 b^{2}$. Case 3: $a=3$. Then $k=6$ and $k^{4}+n^{2}=1296+4 b^{2} \\geq\\left|7^{6}-3^{n}\\right|=\\left|343-3^{b}\\right| \\cdot\\left(343+3^{b}\\right)$. Analogously, $\\left|343-3^{b}\\right| \\geq 100$ and we have $324+b^{2} \\geq 25\\left(343+3^{b}\\right)$, which is impossible again. We find that there exists a unique solution $(k, n)=(2,4)$.","tier":0} +{"problem_type":"Number Theory","problem_label":"N2","problem":"Let $b, n>1$ be integers. Suppose that for each $k>1$ there exists an integer $a_{k}$ such that $b-a_{k}^{n}$ is divisible by $k$. Prove that $b=A^{n}$ for some integer $A$. (Canada)","solution":"Let the prime factorization of $b$ be $b=p_{1}^{\\alpha_{1}} \\ldots p_{s}^{\\alpha_{s}}$, where $p_{1}, \\ldots, p_{s}$ are distinct primes. Our goal is to show that all exponents $\\alpha_{i}$ are divisible by $n$, then we can set $A=p_{1}^{\\alpha_{1} \/ n} \\ldots p_{s}^{\\alpha_{s} \/ n}$. Apply the condition for $k=b^{2}$. The number $b-a_{k}^{n}$ is divisible by $b^{2}$ and hence, for each $1 \\leq i \\leq s$, it is divisible by $p_{i}^{2 \\alpha_{i}}>p_{i}^{\\alpha_{i}}$ as well. Therefore $$ a_{k}^{n} \\equiv b \\equiv 0 \\quad\\left(\\bmod p_{i}^{\\alpha_{i}}\\right) $$ and $$ a_{k}^{n} \\equiv b \\not \\equiv 0 \\quad\\left(\\bmod p_{i}^{\\alpha_{i}+1}\\right) $$ which implies that the largest power of $p_{i}$ dividing $a_{k}^{n}$ is $p_{i}^{\\alpha_{i}}$. Since $a_{k}^{n}$ is a complete $n$th power, this implies that $\\alpha_{i}$ is divisible by $n$. Comment. If $n=8$ and $b=16$, then for each prime $p$ there exists an integer $a_{p}$ such that $b-a_{p}^{n}$ is divisible by $p$. Actually, the congruency $x^{8}-16 \\equiv 0(\\bmod p)$ expands as $$ \\left(x^{2}-2\\right)\\left(x^{2}+2\\right)\\left(x^{2}-2 x+2\\right)\\left(x^{2}+2 x+2\\right) \\equiv 0 \\quad(\\bmod p) $$ Hence, if -1 is a quadratic residue modulo $p$, then congruency $x^{2}+2 x+2=(x+1)^{2}+1 \\equiv 0$ has a solution. Otherwise, one of congruencies $x^{2} \\equiv 2$ and $x^{2} \\equiv-2$ has a solution. Thus, the solution cannot work using only prime values of $k$.","tier":0} +{"problem_type":"Number Theory","problem_label":"N3","problem":"Let $X$ be a set of 10000 integers, none of them is divisible by 47 . Prove that there exists a 2007-element subset $Y$ of $X$ such that $a-b+c-d+e$ is not divisible by 47 for any $a, b, c, d, e \\in Y$. (Netherlands)","solution":"Call a set $M$ of integers good if $47 \\nmid a-b+c-d+e$ for any $a, b, c, d, e \\in M$. Consider the set $J=\\{-9,-7,-5,-3,-1,1,3,5,7,9\\}$. We claim that $J$ is good. Actually, for any $a, b, c, d, e \\in J$ the number $a-b+c-d+e$ is odd and $$ -45=(-9)-9+(-9)-9+(-9) \\leq a-b+c-d+e \\leq 9-(-9)+9-(-9)+9=45 $$ But there is no odd number divisible by 47 between -45 and 45 . For any $k=1, \\ldots, 46$ consider the set $$ A_{k}=\\{x \\in X \\mid \\exists j \\in J: \\quad k x \\equiv j(\\bmod 47)\\} . $$ If $A_{k}$ is not good, then $47 \\mid a-b+c-d+e$ for some $a, b, c, d, e \\in A_{k}$, hence $47 \\mid k a-k b+$ $k c-k d+k e$. But set $J$ contains numbers with the same residues modulo 47 , so $J$ also is not good. This is a contradiction; therefore each $A_{k}$ is a good subset of $X$. Then it suffices to prove that there exists a number $k$ such that $\\left|A_{k}\\right| \\geq 2007$. Note that each $x \\in X$ is contained in exactly 10 sets $A_{k}$. Then $$ \\sum_{k=1}^{46}\\left|A_{k}\\right|=10|X|=100000 $$ hence for some value of $k$ we have $$ \\left|A_{k}\\right| \\geq \\frac{100000}{46}>2173>2007 . $$ This completes the proof. Comment. For the solution, it is essential to find a good set consisting of 10 different residues. Actually, consider a set $X$ containing almost uniform distribution of the nonzero residues (i. e. each residue occurs 217 or 218 times). Let $Y \\subset X$ be a good subset containing 2007 elements. Then the set $K$ of all residues appearing in $Y$ contains not less than 10 residues, and obviously this set is good. On the other hand, there is no good set $K$ consisting of 11 different residues. The CauchyDavenport theorem claims that for any sets $A, B$ of residues modulo a prime $p$, $$ |A+B| \\geq \\min \\{p,|A|+|B|-1\\} . $$ Hence, if $|K| \\geq 11$, then $|K+K| \\geq 21,|K+K+K| \\geq 31>47-|K+K|$, hence $\\mid K+K+K+$ $(-K)+(-K) \\mid=47$, and $0 \\equiv a+c+e-b-d(\\bmod 47)$ for some $a, b, c, d, e \\in K$. From the same reasoning, one can see that a good set $K$ containing 10 residues should satisfy equalities $|K+K|=19=2|K|-1$ and $|K+K+K|=28=|K+K|+|K|-1$. It can be proved that in this case set $K$ consists of 10 residues forming an arithmetic progression. As an easy consequence, one obtains that set $K$ has the form $a J$ for some nonzero residue $a$.","tier":0} +{"problem_type":"Number Theory","problem_label":"N4","problem":"For every integer $k \\geq 2$, prove that $2^{3 k}$ divides the number $$ \\left(\\begin{array}{c} 2^{k+1} \\\\ 2^{k} \\end{array}\\right)-\\left(\\begin{array}{c} 2^{k} \\\\ 2^{k-1} \\end{array}\\right) $$ but $2^{3 k+1}$ does not. (Poland)","solution":"We use the notation $(2 n-1) ! !=1 \\cdot 3 \\cdots(2 n-1)$ and $(2 n) ! !=2 \\cdot 4 \\cdots(2 n)=2^{n} n$ ! for any positive integer $n$. Observe that $(2 n) !=(2 n) ! !(2 n-1) ! !=2^{n} n !(2 n-1) !$ !. For any positive integer $n$ we have $$ \\begin{aligned} & \\left(\\begin{array}{l} 4 n \\\\ 2 n \\end{array}\\right)=\\frac{(4 n) !}{(2 n) !^{2}}=\\frac{2^{2 n}(2 n) !(4 n-1) ! !}{(2 n) !^{2}}=\\frac{2^{2 n}}{(2 n) !}(4 n-1) ! ! \\\\ & \\left(\\begin{array}{c} 2 n \\\\ n \\end{array}\\right)=\\frac{1}{(2 n) !}\\left(\\frac{(2 n) !}{n !}\\right)^{2}=\\frac{1}{(2 n) !}\\left(2^{n}(2 n-1) ! !\\right)^{2}=\\frac{2^{2 n}}{(2 n) !}(2 n-1) ! !^{2} . \\end{aligned} $$ Then expression (1) can be rewritten as follows: $$ \\begin{aligned} \\left(\\begin{array}{c} 2^{k+1} \\\\ 2^{k} \\end{array}\\right) & -\\left(\\begin{array}{c} 2^{k} \\\\ 2^{k-1} \\end{array}\\right)=\\frac{2^{2^{k}}}{\\left(2^{k}\\right) !}\\left(2^{k+1}-1\\right) ! !-\\frac{2^{2^{k}}}{\\left(2^{k}\\right) !}\\left(2^{k}-1\\right) ! !^{2} \\\\ & =\\frac{2^{2^{k}}\\left(2^{k}-1\\right) ! !}{\\left(2^{k}\\right) !} \\cdot\\left(\\left(2^{k}+1\\right)\\left(2^{k}+3\\right) \\ldots\\left(2^{k}+2^{k}-1\\right)-\\left(2^{k}-1\\right)\\left(2^{k}-3\\right) \\ldots\\left(2^{k}-2^{k}+1\\right)\\right) . \\end{aligned} $$ We compute the exponent of 2 in the prime decomposition of each factor (the first one is a rational number but not necessarily an integer; it is not important). First, we show by induction on $n$ that the exponent of 2 in $\\left(2^{n}\\right)$ ! is $2^{n}-1$. The base case $n=1$ is trivial. Suppose that $\\left(2^{n}\\right) !=2^{2^{n}-1}(2 d+1)$ for some integer $d$. Then we have $$ \\left(2^{n+1}\\right) !=2^{2^{n}}\\left(2^{n}\\right) !\\left(2^{n+1}-1\\right) ! !=2^{2^{n}} 2^{2^{n}-1} \\cdot(2 d+1)\\left(2^{n+1}-1\\right) ! !=2^{2^{n+1}-1} \\cdot(2 q+1) $$ for some integer $q$. This finishes the induction step. Hence, the exponent of 2 in the first factor in $(2)$ is $2^{k}-\\left(2^{k}-1\\right)=1$. The second factor in (2) can be considered as the value of the polynomial $$ P(x)=(x+1)(x+3) \\ldots\\left(x+2^{k}-1\\right)-(x-1)(x-3) \\ldots\\left(x-2^{k}+1\\right) . $$ at $x=2^{k}$. Now we collect some information about $P(x)$. Observe that $P(-x)=-P(x)$, since $k \\geq 2$. So $P(x)$ is an odd function, and it has nonzero coefficients only at odd powers of $x$. Hence $P(x)=x^{3} Q(x)+c x$, where $Q(x)$ is a polynomial with integer coefficients. Compute the exponent of 2 in $c$. We have $$ \\begin{aligned} c & =2\\left(2^{k}-1\\right) ! ! \\sum_{i=1}^{2^{k-1}} \\frac{1}{2 i-1}=\\left(2^{k}-1\\right) ! ! \\sum_{i=1}^{2^{k-1}}\\left(\\frac{1}{2 i-1}+\\frac{1}{2^{k}-2 i+1}\\right) \\\\ & =\\left(2^{k}-1\\right) ! ! \\sum_{i=1}^{2^{k-1}} \\frac{2^{k}}{(2 i-1)\\left(2^{k}-2 i+1\\right)}=2^{k} \\sum_{i=1}^{2^{k-1}} \\frac{\\left(2^{k}-1\\right) ! !}{(2 i-1)\\left(2^{k}-2 i+1\\right)}=2^{k} S \\end{aligned} $$ For any integer $i=1, \\ldots, 2^{k-1}$, denote by $a_{2 i-1}$ the residue inverse to $2 i-1$ modulo $2^{k}$. Clearly, when $2 i-1$ runs through all odd residues, so does $a_{2 i-1}$, hence $$ \\begin{aligned} & S=\\sum_{i=1}^{2^{k-1}} \\frac{\\left(2^{k}-1\\right) ! !}{(2 i-1)\\left(2^{k}-2 i+1\\right)} \\equiv-\\sum_{i=1}^{2^{k-1}} \\frac{\\left(2^{k}-1\\right) ! !}{(2 i-1)^{2}} \\equiv-\\sum_{i=1}^{2^{k-1}}\\left(2^{k}-1\\right) ! ! a_{2 i-1}^{2} \\\\ &=-\\left(2^{k}-1\\right) ! ! \\sum_{i=1}^{2^{k-1}}(2 i-1)^{2}=-\\left(2^{k}-1\\right) ! ! \\frac{2^{k-1}\\left(2^{2 k}-1\\right)}{3} \\quad\\left(\\bmod 2^{k}\\right) . \\end{aligned} $$ Therefore, the exponent of 2 in $S$ is $k-1$, so $c=2^{k} S=2^{2 k-1}(2 t+1)$ for some integer $t$. Finally we obtain that $$ P\\left(2^{k}\\right)=2^{3 k} Q\\left(2^{k}\\right)+2^{k} c=2^{3 k} Q\\left(2^{k}\\right)+2^{3 k-1}(2 t+1), $$ which is divisible exactly by $2^{3 k-1}$. Thus, the exponent of 2 in $(2)$ is $1+(3 k-1)=3 k$. Comment. The fact that (1) is divisible by $2^{2 k}$ is known; but it does not help in solving this problem.","tier":0} +{"problem_type":"Number Theory","problem_label":"N5","problem":"Find all surjective functions $f: \\mathbb{N} \\rightarrow \\mathbb{N}$ such that for every $m, n \\in \\mathbb{N}$ and every prime $p$, the number $f(m+n)$ is divisible by $p$ if and only if $f(m)+f(n)$ is divisible by $p$. ( $\\mathbb{N}$ is the set of all positive integers.) (Iran) Answer. $f(n)=n$.","solution":"Suppose that function $f: \\mathbb{N} \\rightarrow \\mathbb{N}$ satisfies the problem conditions. Lemma. For any prime $p$ and any $x, y \\in \\mathbb{N}$, we have $x \\equiv y(\\bmod p)$ if and only if $f(x) \\equiv f(y)$ $(\\bmod p)$. Moreover, $p \\mid f(x)$ if and only if $p \\mid x$. Proof. Consider an arbitrary prime $p$. Since $f$ is surjective, there exists some $x \\in \\mathbb{N}$ such that $p \\mid f(x)$. Let $$ d=\\min \\{x \\in \\mathbb{N}: p \\mid f(x)\\} . $$ By induction on $k$, we obtain that $p \\mid f(k d)$ for all $k \\in \\mathbb{N}$. The base is true since $p \\mid f(d)$. Moreover, if $p \\mid f(k d)$ and $p \\mid f(d)$ then, by the problem condition, $p \\mid f(k d+d)=f((k+1) d)$ as required. Suppose that there exists an $x \\in \\mathbb{N}$ such that $d \\not x$ but $p \\mid f(x)$. Let $$ y=\\min \\{x \\in \\mathbb{N}: d \\not\\{x, p \\mid f(x)\\} . $$ By the choice of $d$, we have $y>d$, and $y-d$ is a positive integer not divisible by $d$. Then $p \\nmid\\{(y-d)$, while $p \\mid f(d)$ and $p \\mid f(d+(y-d))=f(y)$. This contradicts the problem condition. Hence, there is no such $x$, and $$ p|f(x) \\Longleftrightarrow d| x . $$ Take arbitrary $x, y \\in \\mathbb{N}$ such that $x \\equiv y(\\bmod d)$. We have $p \\mid f(x+(2 x d-x))=f(2 x d)$; moreover, since $d \\mid 2 x d+(y-x)=y+(2 x d-x)$, we get $p \\mid f(y+(2 x d-x))$. Then by the problem condition $p|f(x)+f(2 x d-x), p| f(y)+f(2 x d-x)$, and hence $f(x) \\equiv-f(2 x d-x) \\equiv f(y)$ $(\\bmod p)$. On the other hand, assume that $f(x) \\equiv f(y)(\\bmod p)$. Again we have $p \\mid f(x)+f(2 x d-x)$ which by our assumption implies that $p \\mid f(x)+f(2 x d-x)+(f(y)-f(x))=f(y)+f(2 x d-x)$. Hence by the problem condition $p \\mid f(y+(2 x d-x))$. Using (1) we get $0 \\equiv y+(2 x d-x) \\equiv y-x$ $(\\bmod d)$. Thus, we have proved that $$ x \\equiv y \\quad(\\bmod d) \\Longleftrightarrow f(x) \\equiv f(y) \\quad(\\bmod p) $$ We are left to show that $p=d$ : in this case (1) and (2) provide the desired statements. The numbers $1,2, \\ldots, d$ have distinct residues modulo $d$. By (2), numbers $f(1), f(2), \\ldots$, $f(d)$ have distinct residues modulo $p$; hence there are at least $d$ distinct residues, and $p \\geq d$. On the other hand, by the surjectivity of $f$, there exist $x_{1}, \\ldots, x_{p} \\in \\mathbb{N}$ such that $f\\left(x_{i}\\right)=i$ for any $i=1,2, \\ldots, p$. By (2), all these $x_{i}$ 's have distinct residues modulo $d$. For the same reasons, $d \\geq p$. Hence, $d=p$. Now we prove that $f(n)=n$ by induction on $n$. If $n=1$ then, by the Lemma, $p \\not \\nmid f(1)$ for any prime $p$, so $f(1)=1$, and the base is established. Suppose that $n>1$ and denote $k=f(n)$. Note that there exists a prime $q \\mid n$, so by the Lemma $q \\mid k$ and $k>1$. If $k>n$ then $k-n+1>1$, and there exists a prime $p \\mid k-n+1$; we have $k \\equiv n-1$ $(\\bmod p)$. By the induction hypothesis we have $f(n-1)=n-1 \\equiv k=f(n)(\\bmod p)$. Now, by the Lemma we obtain $n-1 \\equiv n(\\bmod p)$ which cannot be true. Analogously, if $k1$, so there exists a prime $p \\mid n-k+1$ and $n \\equiv k-1(\\bmod p)$. By the Lemma again, $k=f(n) \\equiv$ $f(k-1)=k-1(\\bmod p)$, which is also false. The only remaining case is $k=n$, so $f(n)=n$. Finally, the function $f(n)=n$ obviously satisfies the condition.","tier":0} +{"problem_type":"Number Theory","problem_label":"N6","problem":"Let $k$ be a positive integer. Prove that the number $\\left(4 k^{2}-1\\right)^{2}$ has a positive divisor of the form $8 k n-1$ if and only if $k$ is even. (United Kingdom)","solution":"The statement follows from the following fact. Lemma. For arbitrary positive integers $x$ and $y$, the number $4 x y-1$ divides $\\left(4 x^{2}-1\\right)^{2}$ if and only if $x=y$. Proof. If $x=y$ then $4 x y-1=4 x^{2}-1$ obviously divides $\\left(4 x^{2}-1\\right)^{2}$ so it is sufficient to consider the opposite direction. Call a pair $(x, y)$ of positive integers bad if $4 x y-1$ divides $\\left(4 x^{2}-1\\right)^{2}$ but $x \\neq y$. In order to prove that bad pairs do not exist, we present two properties of them which provide an infinite descent. Property (i). If $(x, y)$ is a bad pair and $x1$ and $a x y-1$ divides $\\left(a x^{2}-1\\right)^{2}$ then $x=y$.","tier":0} +{"problem_type":"Number Theory","problem_label":"N7","problem":"For a prime $p$ and a positive integer $n$, denote by $\\nu_{p}(n)$ the exponent of $p$ in the prime factorization of $n$ !. Given a positive integer $d$ and a finite set $\\left\\{p_{1}, \\ldots, p_{k}\\right\\}$ of primes. Show that there are infinitely many positive integers $n$ such that $d \\mid \\nu_{p_{i}}(n)$ for all $1 \\leq i \\leq k$. (India)","solution":"For arbitrary prime $p$ and positive integer $n$, denote by $\\operatorname{ord}_{p}(n)$ the exponent of $p$ in $n$. Thus, $$ \\nu_{p}(n)=\\operatorname{ord}_{p}(n !)=\\sum_{i=1}^{n} \\operatorname{ord}_{p}(i) $$ Lemma. Let $p$ be a prime number, $q$ be a positive integer, $k$ and $r$ be positive integers such that $p^{k}>r$. Then $\\nu_{p}\\left(q p^{k}+r\\right)=\\nu_{p}\\left(q p^{k}\\right)+\\nu_{p}(r)$. Proof. We claim that $\\operatorname{ord}_{p}\\left(q p^{k}+i\\right)=\\operatorname{ord}_{p}(i)$ for all $01$. By the construction of the sequence, $p_{i}^{n_{\\ell_{1}}}$ divides $n_{\\ell_{2}}+\\ldots+n_{\\ell_{m}}$; clearly, $p_{i}^{n_{\\ell_{1}}}>n_{\\ell_{1}}$ for all $1 \\leq i \\leq k$. Therefore the Lemma can be applied for $p=p_{i}, k=r=n_{\\ell_{1}}$ and $q p^{k}=n_{\\ell_{2}}+\\ldots+n_{\\ell_{m}}$ to obtain $$ f_{i}\\left(n_{\\ell_{1}}+n_{\\ell_{2}}+\\ldots+n_{\\ell_{m}}\\right)=f_{i}\\left(n_{\\ell_{1}}\\right)+f_{i}\\left(n_{\\ell_{2}}+\\ldots+n_{\\ell_{m}}\\right) \\quad \\text { for all } 1 \\leq i \\leq k $$ and hence $$ f\\left(n_{\\ell_{1}}+n_{\\ell_{2}}+\\ldots+n_{\\ell_{m}}\\right)=f\\left(n_{\\ell_{1}}\\right)+f\\left(n_{\\ell_{2}}+\\ldots+n_{\\ell_{m}}\\right)=f\\left(n_{\\ell_{1}}\\right)+f\\left(n_{\\ell_{2}}\\right)+\\ldots+f\\left(n_{\\ell_{m}}\\right) $$ by the induction hypothesis. Now consider the values $f\\left(n_{1}\\right), f\\left(n_{2}\\right), \\ldots$ There exist finitely many possible values of $f$. Hence, there exists an infinite sequence of indices $\\ell_{1}<\\ell_{2}<\\ldots$ such that $f\\left(n_{\\ell_{1}}\\right)=f\\left(n_{\\ell_{2}}\\right)=\\ldots$ and thus $$ f\\left(n_{\\ell_{m+1}}+n_{\\ell_{m+2}}+\\ldots+n_{\\ell_{m+d}}\\right)=f\\left(n_{\\ell_{m+1}}\\right)+\\ldots+f\\left(n_{\\ell_{m+d}}\\right)=d \\cdot f\\left(n_{\\ell_{1}}\\right)=(\\overline{0}, \\ldots, \\overline{0}) $$ for all $m$. We have found infinitely many suitable numbers.","tier":0} +{"problem_type":"Number Theory","problem_label":"N7","problem":"For a prime $p$ and a positive integer $n$, denote by $\\nu_{p}(n)$ the exponent of $p$ in the prime factorization of $n$ !. Given a positive integer $d$ and a finite set $\\left\\{p_{1}, \\ldots, p_{k}\\right\\}$ of primes. Show that there are infinitely many positive integers $n$ such that $d \\mid \\nu_{p_{i}}(n)$ for all $1 \\leq i \\leq k$. (India)","solution":"We use the same Lemma and definition of the function $f$. Let $S=\\{f(n): n \\in \\mathbb{N}\\}$. Obviously, set $S$ is finite. For every $s \\in S$ choose the minimal $n_{s}$ such that $f\\left(n_{s}\\right)=s$. Denote $N=\\max _{s \\in S} n_{s}$. Moreover, let $g$ be an integer such that $p_{i}^{g}>N$ for each $i=1,2, \\ldots, k$. Let $P=\\left(p_{1} p_{2} \\ldots p_{k}\\right)^{g}$. We claim that $$ \\{f(n) \\mid n \\in[m P, m P+N]\\}=S $$ for every positive integer $m$. In particular, since $(\\overline{0}, \\ldots, \\overline{0})=f(1) \\in S$, it follows that for an arbitrary $m$ there exists $n \\in[m P, m P+N]$ such that $f(n)=(\\overline{0}, \\ldots, \\overline{0})$. So there are infinitely many suitable numbers. To prove (1), let $a_{i}=f_{i}(m P)$. Consider all numbers of the form $n_{m, s}=m P+n_{s}$ with $s=\\left(s_{1}, \\ldots, s_{k}\\right) \\in S$ (clearly, all $n_{m, s}$ belong to $[m P, m P+N]$ ). Since $n_{s} \\leq N0$ with $p q=r s$.","solution":"Let $f$ satisfy the given condition. Setting $p=q=r=s=1$ yields $f(1)^{2}=f(1)$ and hence $f(1)=1$. Now take any $x>0$ and set $p=x, q=1, r=s=\\sqrt{x}$ to obtain $$ \\frac{f(x)^{2}+1}{2 f(x)}=\\frac{x^{2}+1}{2 x} . $$ This recasts into $$ \\begin{gathered} x f(x)^{2}+x=x^{2} f(x)+f(x), \\\\ (x f(x)-1)(f(x)-x)=0 . \\end{gathered} $$ And thus, $$ \\text { for every } x>0, \\text { either } f(x)=x \\text { or } f(x)=\\frac{1}{x} \\text {. } $$ Obviously, if $$ f(x)=x \\quad \\text { for all } x>0 \\quad \\text { or } \\quad f(x)=\\frac{1}{x} \\quad \\text { for all } x>0 $$ then the condition of the problem is satisfied. We show that actually these two functions are the only solutions. So let us assume that there exists a function $f$ satisfying the requirement, other than those in (2). Then $f(a) \\neq a$ and $f(b) \\neq 1 \/ b$ for some $a, b>0$. By (1), these values must be $f(a)=1 \/ a, f(b)=b$. Applying now the equation with $p=a, q=b, r=s=\\sqrt{a b}$ we obtain $\\left(a^{-2}+b^{2}\\right) \/ 2 f(a b)=\\left(a^{2}+b^{2}\\right) \/ 2 a b ;$ equivalently, $$ f(a b)=\\frac{a b\\left(a^{-2}+b^{2}\\right)}{a^{2}+b^{2}} . $$ We know however (see (1)) that $f(a b)$ must be either $a b$ or $1 \/ a b$. If $f(a b)=a b$ then by (3) $a^{-2}+b^{2}=a^{2}+b^{2}$, so that $a=1$. But, as $f(1)=1$, this contradicts the relation $f(a) \\neq a$. Likewise, if $f(a b)=1 \/ a b$ then (3) gives $a^{2} b^{2}\\left(a^{-2}+b^{2}\\right)=a^{2}+b^{2}$, whence $b=1$, in contradiction to $f(b) \\neq 1 \/ b$. Thus indeed the functions listed in (2) are the only two solutions. Comment. The equation has as many as four variables with only one constraint $p q=r s$, leaving three degrees of freedom and providing a lot of information. Various substitutions force various useful properties of the function searched. We sketch one more method to reach conclusion (1); certainly there are many others. Noticing that $f(1)=1$ and setting, first, $p=q=1, r=\\sqrt{x}, s=1 \/ \\sqrt{x}$, and then $p=x, q=1 \/ x$, $r=s=1$, we obtain two relations, holding for every $x>0$, $$ f(x)+f\\left(\\frac{1}{x}\\right)=x+\\frac{1}{x} \\quad \\text { and } \\quad f(x)^{2}+f\\left(\\frac{1}{x}\\right)^{2}=x^{2}+\\frac{1}{x^{2}} . $$ Squaring the first and subtracting the second gives $2 f(x) f(1 \/ x)=2$. Subtracting this from the second relation of (4) leads to $$ \\left(f(x)-f\\left(\\frac{1}{x}\\right)\\right)^{2}=\\left(x-\\frac{1}{x}\\right)^{2} \\quad \\text { or } \\quad f(x)-f\\left(\\frac{1}{x}\\right)= \\pm\\left(x-\\frac{1}{x}\\right) . $$ The last two alternatives combined with the first equation of (4) imply the two alternatives of (1).","tier":0} +{"problem_type":"Algebra","problem_label":"A2","problem":"(a) Prove the inequality $$ \\frac{x^{2}}{(x-1)^{2}}+\\frac{y^{2}}{(y-1)^{2}}+\\frac{z^{2}}{(z-1)^{2}} \\geq 1 $$ for real numbers $x, y, z \\neq 1$ satisfying the condition $x y z=1$. (b) Show that there are infinitely many triples of rational numbers $x, y, z$ for which this inequality turns into equality.","solution":"(a) We start with the substitution $$ \\frac{x}{x-1}=a, \\quad \\frac{y}{y-1}=b, \\quad \\frac{z}{z-1}=c, \\quad \\text { i.e., } \\quad x=\\frac{a}{a-1}, \\quad y=\\frac{b}{b-1}, \\quad z=\\frac{c}{c-1} \\text {. } $$ The inequality to be proved reads $a^{2}+b^{2}+c^{2} \\geq 1$. The new variables are subject to the constraints $a, b, c \\neq 1$ and the following one coming from the condition $x y z=1$, $$ (a-1)(b-1)(c-1)=a b c . $$ This is successively equivalent to $$ \\begin{aligned} a+b+c-1 & =a b+b c+c a, \\\\ 2(a+b+c-1) & =(a+b+c)^{2}-\\left(a^{2}+b^{2}+c^{2}\\right), \\\\ a^{2}+b^{2}+c^{2}-2 & =(a+b+c)^{2}-2(a+b+c), \\\\ a^{2}+b^{2}+c^{2}-1 & =(a+b+c-1)^{2} . \\end{aligned} $$ Thus indeed $a^{2}+b^{2}+c^{2} \\geq 1$, as desired. (b) From the equation $a^{2}+b^{2}+c^{2}-1=(a+b+c-1)^{2}$ we see that the proposed inequality becomes an equality if and only if both sums $a^{2}+b^{2}+c^{2}$ and $a+b+c$ have value 1 . The first of them is equal to $(a+b+c)^{2}-2(a b+b c+c a)$. So the instances of equality are described by the system of two equations $$ a+b+c=1, \\quad a b+b c+c a=0 $$ plus the constraint $a, b, c \\neq 1$. Elimination of $c$ leads to $a^{2}+a b+b^{2}=a+b$, which we regard as a quadratic equation in $b$, $$ b^{2}+(a-1) b+a(a-1)=0, $$ with discriminant $$ \\Delta=(a-1)^{2}-4 a(a-1)=(1-a)(1+3 a) . $$ We are looking for rational triples $(a, b, c)$; it will suffice to have $a$ rational such that $1-a$ and $1+3 a$ are both squares of rational numbers (then $\\Delta$ will be so too). Set $a=k \/ m$. We want $m-k$ and $m+3 k$ to be squares of integers. This is achieved for instance by taking $m=k^{2}-k+1$ (clearly nonzero); then $m-k=(k-1)^{2}, m+3 k=(k+1)^{2}$. Note that distinct integers $k$ yield distinct values of $a=k \/ m$. And thus, if $k$ is any integer and $m=k^{2}-k+1, a=k \/ m$ then $\\Delta=\\left(k^{2}-1\\right)^{2} \/ m^{2}$ and the quadratic equation has rational roots $b=\\left(m-k \\pm k^{2} \\mp 1\\right) \/(2 m)$. Choose e.g. the larger root, $$ b=\\frac{m-k+k^{2}-1}{2 m}=\\frac{m+(m-2)}{2 m}=\\frac{m-1}{m} . $$ Computing $c$ from $a+b+c=1$ then gives $c=(1-k) \/ m$. The condition $a, b, c \\neq 1$ eliminates only $k=0$ and $k=1$. Thus, as $k$ varies over integers greater than 1 , we obtain an infinite family of rational triples $(a, b, c)$ - and coming back to the original variables $(x=a \/(a-1)$ etc. $)$-an infinite family of rational triples $(x, y, z)$ with the needed property. (A short calculation shows that the resulting triples are $x=-k \/(k-1)^{2}, y=k-k^{2}, z=(k-1) \/ k^{2}$; but the proof was complete without listing them.) Comment 1. There are many possible variations in handling the equation system $a^{2}+b^{2}+c^{2}=1$, $a+b+c=1(a, b, c \\neq 1)$ which of course describes a circle in the $(a, b, c)$-space (with three points excluded), and finding infinitely many rational points on it. Also the initial substitution $x=a \/(a-1)$ (etc.) can be successfully replaced by other similar substitutions, e.g. $x=1-1 \/ \\alpha$ (etc.); or $x=x^{\\prime}-1$ (etc.); or $1-y z=u$ (etc.) - eventually reducing the inequality to $(\\cdots)^{2} \\geq 0$, the expression in the parentheses depending on the actual substitution. Depending on the method chosen, one arrives at various sequences of rational triples $(x, y, z)$ as needed; let us produce just one more such example: $x=(2 r-2) \/(r+1)^{2}, y=(2 r+2) \/(r-1)^{2}$, $z=\\left(r^{2}-1\\right) \/ 4$ where $r$ can be any rational number different from 1 or -1 . Solution 2 (an outline). (a) Without changing variables, just setting $z=1 \/ x y$ and clearing fractions, the proposed inequality takes the form $$ (x y-1)^{2}\\left(x^{2}(y-1)^{2}+y^{2}(x-1)^{2}\\right)+(x-1)^{2}(y-1)^{2} \\geq(x-1)^{2}(y-1)^{2}(x y-1)^{2} . $$ With the notation $p=x+y, q=x y$ this becomes, after lengthy routine manipulation and a lot of cancellation $$ q^{4}-6 q^{3}+2 p q^{2}+9 q^{2}-6 p q+p^{2} \\geq 0 \\text {. } $$ It is not hard to notice that the expression on the left is just $\\left(q^{2}-3 q+p\\right)^{2}$, hence nonnegative. (Without introducing $p$ and $q$, one is of course led with some more work to the same expression, just written in terms of $x$ and $y$; but then it is not that easy to see that it is a square.) (b) To have equality, one needs $q^{2}-3 q+p=0$. Note that $x$ and $y$ are the roots of the quadratic trinomial (in a formal variable $t$ ): $t^{2}-p t+q$. When $q^{2}-3 q+p=0$, the discriminant equals $$ \\delta=p^{2}-4 q=\\left(3 q-q^{2}\\right)^{2}-4 q=q(q-1)^{2}(q-4) . $$ Now it suffices to have both $q$ and $q-4$ squares of rational numbers (then $p=3 q-q^{2}$ and $\\sqrt{\\delta}$ are also rational, and so are the roots of the trinomial). On setting $q=(n \/ m)^{2}=4+(l \/ m)^{2}$ the requirement becomes $4 m^{2}+l^{2}=n^{2}$ (with $l, m, n$ being integers). This is just the Pythagorean equation, known to have infinitely many integer solutions. Comment 2. Part (a) alone might also be considered as a possible contest problem (in the category of easy problems).","tier":0} +{"problem_type":"Algebra","problem_label":"A3","problem":"Let $S \\subseteq \\mathbb{R}$ be a set of real numbers. We say that a pair $(f, g)$ of functions from $S$ into $S$ is a Spanish Couple on $S$, if they satisfy the following conditions: (i) Both functions are strictly increasing, i.e. $f(x)0, C>2$ and $B=1$ produce a Spanish couple (in the example above, $A=1, C=3$ ). The proposer's example results from taking $h(a)=a+1, G(a, b)=3^{a}+b$.","tier":0} +{"problem_type":"Algebra","problem_label":"A4","problem":"For an integer $m$, denote by $t(m)$ the unique number in $\\{1,2,3\\}$ such that $m+t(m)$ is a multiple of 3. A function $f: \\mathbb{Z} \\rightarrow \\mathbb{Z}$ satisfies $f(-1)=0, f(0)=1, f(1)=-1$ and $$ f\\left(2^{n}+m\\right)=f\\left(2^{n}-t(m)\\right)-f(m) \\quad \\text { for all integers } m, n \\geq 0 \\text { with } 2^{n}>m \\text {. } $$ Prove that $f(3 p) \\geq 0$ holds for all integers $p \\geq 0$.","solution":"The given conditions determine $f$ uniquely on the positive integers. The signs of $f(1), f(2), \\ldots$ seem to change quite erratically. However values of the form $f\\left(2^{n}-t(m)\\right)$ are sufficient to compute directly any functional value. Indeed, let $n>0$ have base 2 representation $n=2^{a_{0}}+2^{a_{1}}+\\cdots+2^{a_{k}}, a_{0}>a_{1}>\\cdots>a_{k} \\geq 0$, and let $n_{j}=2^{a_{j}}+2^{a_{j-1}}+\\cdots+2^{a_{k}}, j=0, \\ldots, k$. Repeated applications of the recurrence show that $f(n)$ is an alternating sum of the quantities $f\\left(2^{a_{j}}-t\\left(n_{j+1}\\right)\\right)$ plus $(-1)^{k+1}$. (The exact formula is not needed for our proof.) So we focus attention on the values $f\\left(2^{n}-1\\right), f\\left(2^{n}-2\\right)$ and $f\\left(2^{n}-3\\right)$. Six cases arise; more specifically, $t\\left(2^{2 k}-3\\right)=2, t\\left(2^{2 k}-2\\right)=1, t\\left(2^{2 k}-1\\right)=3, t\\left(2^{2 k+1}-3\\right)=1, t\\left(2^{2 k+1}-2\\right)=3, t\\left(2^{2 k+1}-1\\right)=2$. Claim. For all integers $k \\geq 0$ the following equalities hold: $$ \\begin{array}{lll} f\\left(2^{2 k+1}-3\\right)=0, & f\\left(2^{2 k+1}-2\\right)=3^{k}, & f\\left(2^{2 k+1}-1\\right)=-3^{k}, \\\\ f\\left(2^{2 k+2}-3\\right)=-3^{k}, & f\\left(2^{2 k+2}-2\\right)=-3^{k}, & f\\left(2^{2 k+2}-1\\right)=2 \\cdot 3^{k} . \\end{array} $$ Proof. By induction on $k$. The base $k=0$ comes down to checking that $f(2)=-1$ and $f(3)=2$; the given values $f(-1)=0, f(0)=1, f(1)=-1$ are also needed. Suppose the claim holds for $k-1$. For $f\\left(2^{2 k+1}-t(m)\\right)$, the recurrence formula and the induction hypothesis yield $$ \\begin{aligned} & f\\left(2^{2 k+1}-3\\right)=f\\left(2^{2 k}+\\left(2^{2 k}-3\\right)\\right)=f\\left(2^{2 k}-2\\right)-f\\left(2^{2 k}-3\\right)=-3^{k-1}+3^{k-1}=0, \\\\ & f\\left(2^{2 k+1}-2\\right)=f\\left(2^{2 k}+\\left(2^{2 k}-2\\right)\\right)=f\\left(2^{2 k}-1\\right)-f\\left(2^{2 k}-2\\right)=2 \\cdot 3^{k-1}+3^{k-1}=3^{k}, \\\\ & f\\left(2^{2 k+1}-1\\right)=f\\left(2^{2 k}+\\left(2^{2 k}-1\\right)\\right)=f\\left(2^{2 k}-3\\right)-f\\left(2^{2 k}-1\\right)=-3^{k-1}-2 \\cdot 3^{k-1}=-3^{k} . \\end{aligned} $$ For $f\\left(2^{2 k+2}-t(m)\\right)$ we use the three equalities just established: $$ \\begin{aligned} & f\\left(2^{2 k+2}-3\\right)=f\\left(2^{2 k+1}+\\left(2^{2 k+1}-3\\right)\\right)=f\\left(2^{2 k+1}-1\\right)-f\\left(2^{2 k+1}-3\\right)=-3^{k}-0=-3^{k}, \\\\ & f\\left(2^{2 k+2}-2\\right)=f\\left(2^{2 k+1}+\\left(2^{2 k+1}-2\\right)\\right)=f\\left(2^{2 k+1}-3\\right)-f\\left(2^{2 k}-2\\right)=0-3^{k}=-3^{k}, \\\\ & f\\left(2^{2 k+2}-1\\right)=f\\left(2^{2 k+1}+\\left(2^{2 k+1}-1\\right)\\right)=f\\left(2^{2 k+1}-2\\right)-f\\left(2^{2 k+1}-1\\right)=3^{k}+3^{k}=2 \\cdot 3^{k} . \\end{aligned} $$ The claim follows. A closer look at the six cases shows that $f\\left(2^{n}-t(m)\\right) \\geq 3^{(n-1) \/ 2}$ if $2^{n}-t(m)$ is divisible by 3 , and $f\\left(2^{n}-t(m)\\right) \\leq 0$ otherwise. On the other hand, note that $2^{n}-t(m)$ is divisible by 3 if and only if $2^{n}+m$ is. Therefore, for all nonnegative integers $m$ and $n$, (i) $f\\left(2^{n}-t(m)\\right) \\geq 3^{(n-1) \/ 2}$ if $2^{n}+m$ is divisible by 3 ; (ii) $f\\left(2^{n}-t(m)\\right) \\leq 0$ if $2^{n}+m$ is not divisible by 3 . One more (direct) consequence of the claim is that $\\left|f\\left(2^{n}-t(m)\\right)\\right| \\leq \\frac{2}{3} \\cdot 3^{n \/ 2}$ for all $m, n \\geq 0$. The last inequality enables us to find an upper bound for $|f(m)|$ for $m$ less than a given power of 2 . We prove by induction on $n$ that $|f(m)| \\leq 3^{n \/ 2}$ holds true for all integers $m, n \\geq 0$ with $2^{n}>m$. The base $n=0$ is clear as $f(0)=1$. For the inductive step from $n$ to $n+1$, let $m$ and $n$ satisfy $2^{n+1}>m$. If $m<2^{n}$, we are done by the inductive hypothesis. If $m \\geq 2^{n}$ then $m=2^{n}+k$ where $2^{n}>k \\geq 0$. Now, by $\\left|f\\left(2^{n}-t(k)\\right)\\right| \\leq \\frac{2}{3} \\cdot 3^{n \/ 2}$ and the inductive assumption, $$ |f(m)|=\\left|f\\left(2^{n}-t(k)\\right)-f(k)\\right| \\leq\\left|f\\left(2^{n}-t(k)\\right)\\right|+|f(k)| \\leq \\frac{2}{3} \\cdot 3^{n \/ 2}+3^{n \/ 2}<3^{(n+1) \/ 2} . $$ The induction is complete. We proceed to prove that $f(3 p) \\geq 0$ for all integers $p \\geq 0$. Since $3 p$ is not a power of 2 , its binary expansion contains at least two summands. Hence one can write $3 p=2^{a}+2^{b}+c$ where $a>b$ and $2^{b}>c \\geq 0$. Applying the recurrence formula twice yields $$ f(3 p)=f\\left(2^{a}+2^{b}+c\\right)=f\\left(2^{a}-t\\left(2^{b}+c\\right)\\right)-f\\left(2^{b}-t(c)\\right)+f(c) . $$ Since $2^{a}+2^{b}+c$ is divisible by 3 , we have $f\\left(2^{a}-t\\left(2^{b}+c\\right)\\right) \\geq 3^{(a-1) \/ 2}$ by (i). Since $2^{b}+c$ is not divisible by 3 , we have $f\\left(2^{b}-t(c)\\right) \\leq 0$ by (ii). Finally $|f(c)| \\leq 3^{b \/ 2}$ as $2^{b}>c \\geq 0$, so that $f(c) \\geq-3^{b \/ 2}$. Therefore $f(3 p) \\geq 3^{(a-1) \/ 2}-3^{b \/ 2}$ which is nonnegative because $a>b$.","tier":0} +{"problem_type":"Algebra","problem_label":"A5","problem":"Let $a, b, c, d$ be positive real numbers such that $$ a b c d=1 \\quad \\text { and } \\quad a+b+c+d>\\frac{a}{b}+\\frac{b}{c}+\\frac{c}{d}+\\frac{d}{a} $$ Prove that $$ a+b+c+d<\\frac{b}{a}+\\frac{c}{b}+\\frac{d}{c}+\\frac{a}{d} $$","solution":"We show that if $a b c d=1$, the sum $a+b+c+d$ cannot exceed a certain weighted mean of the expressions $\\frac{a}{b}+\\frac{b}{c}+\\frac{c}{d}+\\frac{d}{a}$ and $\\frac{b}{a}+\\frac{c}{b}+\\frac{d}{c}+\\frac{a}{d}$. By applying the AM-GM inequality to the numbers $\\frac{a}{b}, \\frac{a}{b}, \\frac{b}{c}$ and $\\frac{a}{d}$, we obtain $$ a=\\sqrt[4]{\\frac{a^{4}}{a b c d}}=\\sqrt[4]{\\frac{a}{b} \\cdot \\frac{a}{b} \\cdot \\frac{b}{c} \\cdot \\frac{a}{d}} \\leq \\frac{1}{4}\\left(\\frac{a}{b}+\\frac{a}{b}+\\frac{b}{c}+\\frac{a}{d}\\right) $$ Analogously, $$ b \\leq \\frac{1}{4}\\left(\\frac{b}{c}+\\frac{b}{c}+\\frac{c}{d}+\\frac{b}{a}\\right), \\quad c \\leq \\frac{1}{4}\\left(\\frac{c}{d}+\\frac{c}{d}+\\frac{d}{a}+\\frac{c}{b}\\right) \\quad \\text { and } \\quad d \\leq \\frac{1}{4}\\left(\\frac{d}{a}+\\frac{d}{a}+\\frac{a}{b}+\\frac{d}{c}\\right) . $$ Summing up these estimates yields $$ a+b+c+d \\leq \\frac{3}{4}\\left(\\frac{a}{b}+\\frac{b}{c}+\\frac{c}{d}+\\frac{d}{a}\\right)+\\frac{1}{4}\\left(\\frac{b}{a}+\\frac{c}{b}+\\frac{d}{c}+\\frac{a}{d}\\right) . $$ In particular, if $a+b+c+d>\\frac{a}{b}+\\frac{b}{c}+\\frac{c}{d}+\\frac{d}{a}$ then $a+b+c+d<\\frac{b}{a}+\\frac{c}{b}+\\frac{d}{c}+\\frac{a}{d}$. Comment. The estimate in the above solution was obtained by applying the AM-GM inequality to each column of the $4 \\times 4$ array $$ \\begin{array}{llll} a \/ b & b \/ c & c \/ d & d \/ a \\\\ a \/ b & b \/ c & c \/ d & d \/ a \\\\ b \/ c & c \/ d & d \/ a & a \/ b \\\\ a \/ d & b \/ a & c \/ b & d \/ c \\end{array} $$ and adding up the resulting inequalities. The same table yields a stronger bound: If $a, b, c, d>0$ and $a b c d=1$ then $$ \\left(\\frac{a}{b}+\\frac{b}{c}+\\frac{c}{d}+\\frac{d}{a}\\right)^{3}\\left(\\frac{b}{a}+\\frac{c}{b}+\\frac{d}{c}+\\frac{a}{d}\\right) \\geq(a+b+c+d)^{4} $$ It suffices to apply H\u00f6lder's inequality to the sequences in the four rows, with weights $1 \/ 4$ : $$ \\begin{gathered} \\left(\\frac{a}{b}+\\frac{b}{c}+\\frac{c}{d}+\\frac{d}{a}\\right)^{1 \/ 4}\\left(\\frac{a}{b}+\\frac{b}{c}+\\frac{c}{d}+\\frac{d}{a}\\right)^{1 \/ 4}\\left(\\frac{b}{c}+\\frac{c}{d}+\\frac{d}{a}+\\frac{a}{b}\\right)^{1 \/ 4}\\left(\\frac{a}{d}+\\frac{b}{a}+\\frac{c}{b}+\\frac{d}{c}\\right)^{1 \/ 4} \\\\ \\geq\\left(\\frac{a a b a}{b b c d}\\right)^{1 \/ 4}+\\left(\\frac{b b c b}{c c d a}\\right)^{1 \/ 4}+\\left(\\frac{c c d c}{d d a b}\\right)^{1 \/ 4}+\\left(\\frac{d d a d}{a a b c}\\right)^{1 \/ 4}=a+b+c+d \\end{gathered} $$","tier":0} +{"problem_type":"Algebra","problem_label":"A6","problem":"Let $f: \\mathbb{R} \\rightarrow \\mathbb{N}$ be a function which satisfies $$ f\\left(x+\\frac{1}{f(y)}\\right)=f\\left(y+\\frac{1}{f(x)}\\right) \\quad \\text { for all } x, y \\in \\mathbb{R} . $$ Prove that there is a positive integer which is not a value of $f$.","solution":"Suppose that the statement is false and $f(\\mathbb{R})=\\mathbb{N}$. We prove several properties of the function $f$ in order to reach a contradiction. To start with, observe that one can assume $f(0)=1$. Indeed, let $a \\in \\mathbb{R}$ be such that $f(a)=1$, and consider the function $g(x)=f(x+a)$. By substituting $x+a$ and $y+a$ for $x$ and $y$ in (1), we have $$ g\\left(x+\\frac{1}{g(y)}\\right)=f\\left(x+a+\\frac{1}{f(y+a)}\\right)=f\\left(y+a+\\frac{1}{f(x+a)}\\right)=g\\left(y+\\frac{1}{g(x)}\\right) . $$ So $g$ satisfies the functional equation (1), with the additional property $g(0)=1$. Also, $g$ and $f$ have the same set of values: $g(\\mathbb{R})=f(\\mathbb{R})=\\mathbb{N}$. Henceforth we assume $f(0)=1$. Claim 1. For an arbitrary fixed $c \\in \\mathbb{R}$ we have $\\left\\{f\\left(c+\\frac{1}{n}\\right): n \\in \\mathbb{N}\\right\\}=\\mathbb{N}$. Proof. Equation (1) and $f(\\mathbb{R})=\\mathbb{N}$ imply $f(\\mathbb{R})=\\left\\{f\\left(x+\\frac{1}{f(c)}\\right): x \\in \\mathbb{R}\\right\\}=\\left\\{f\\left(c+\\frac{1}{f(x)}\\right): x \\in \\mathbb{R}\\right\\} \\subset\\left\\{f\\left(c+\\frac{1}{n}\\right): n \\in \\mathbb{N}\\right\\} \\subset f(\\mathbb{R})$. The claim follows. We will use Claim 1 in the special cases $c=0$ and $c=1 \/ 3$ : $$ \\left\\{f\\left(\\frac{1}{n}\\right): n \\in \\mathbb{N}\\right\\}=\\left\\{f\\left(\\frac{1}{3}+\\frac{1}{n}\\right): n \\in \\mathbb{N}\\right\\}=\\mathbb{N} $$ Claim 2. If $f(u)=f(v)$ for some $u, v \\in \\mathbb{R}$ then $f(u+q)=f(v+q)$ for all nonnegative rational $q$. Furthermore, if $f(q)=1$ for some nonnegative rational $q$ then $f(k q)=1$ for all $k \\in \\mathbb{N}$. Proof. For all $x \\in \\mathbb{R}$ we have by (1) $$ f\\left(u+\\frac{1}{f(x)}\\right)=f\\left(x+\\frac{1}{f(u)}\\right)=f\\left(x+\\frac{1}{f(v)}\\right)=f\\left(v+\\frac{1}{f(x)}\\right) $$ Since $f(x)$ attains all positive integer values, this yields $f(u+1 \/ n)=f(v+1 \/ n)$ for all $n \\in \\mathbb{N}$. Let $q=k \/ n$ be a positive rational number. Then $k$ repetitions of the last step yield $$ f(u+q)=f\\left(u+\\frac{k}{n}\\right)=f\\left(v+\\frac{k}{n}\\right)=f(v+q) $$ Now let $f(q)=1$ for some nonnegative rational $q$, and let $k \\in \\mathbb{N}$. As $f(0)=1$, the previous conclusion yields successively $f(q)=f(2 q), f(2 q)=f(3 q), \\ldots, f((k-1) q)=f(k q)$, as needed. Claim 3. The equality $f(q)=f(q+1)$ holds for all nonnegative rational $q$. Proof. Let $m$ be a positive integer such that $f(1 \/ m)=1$. Such an $m$ exists by (2). Applying the second statement of Claim 2 with $q=1 \/ m$ and $k=m$ yields $f(1)=1$. Given that $f(0)=f(1)=1$, the first statement of Claim 2 implies $f(q)=f(q+1)$ for all nonnegative rational $q$. Claim 4. The equality $f\\left(\\frac{1}{n}\\right)=n$ holds for every $n \\in \\mathbb{N}$. Proof. For a nonnegative rational $q$ we set $x=q, y=0$ in (1) and use Claim 3 to obtain $$ f\\left(\\frac{1}{f(q)}\\right)=f\\left(q+\\frac{1}{f(0)}\\right)=f(q+1)=f(q) . $$ By (2), for each $n \\in \\mathbb{N}$ there exists a $k \\in \\mathbb{N}$ such that $f(1 \/ k)=n$. Applying the last equation with $q=1 \/ k$, we have $$ n=f\\left(\\frac{1}{k}\\right)=f\\left(\\frac{1}{f(1 \/ k)}\\right)=f\\left(\\frac{1}{n}\\right) $$ Now we are ready to obtain a contradiction. Let $n \\in \\mathbb{N}$ be such that $f(1 \/ 3+1 \/ n)=1$. Such an $n$ exists by (2). Let $1 \/ 3+1 \/ n=s \/ t$, where $s, t \\in \\mathbb{N}$ are coprime. Observe that $t>1$ as $1 \/ 3+1 \/ n$ is not an integer. Choose $k, l \\in \\mathbb{N}$ so that that $k s-l t=1$. Because $f(0)=f(s \/ t)=1$, Claim 2 implies $f(k s \/ t)=1$. Now $f(k s \/ t)=f(1 \/ t+l)$; on the other hand $f(1 \/ t+l)=f(1 \/ t)$ by $l$ successive applications of Claim 3 . Finally, $f(1 \/ t)=t$ by Claim 4, leading to the impossible $t=1$. The solution is complete.","tier":0} +{"problem_type":"Algebra","problem_label":"A7","problem":"Prove that for any four positive real numbers $a, b, c, d$ the inequality $$ \\frac{(a-b)(a-c)}{a+b+c}+\\frac{(b-c)(b-d)}{b+c+d}+\\frac{(c-d)(c-a)}{c+d+a}+\\frac{(d-a)(d-b)}{d+a+b} \\geq 0 $$ holds. Determine all cases of equality.","solution":"Denote the four terms by $$ A=\\frac{(a-b)(a-c)}{a+b+c}, \\quad B=\\frac{(b-c)(b-d)}{b+c+d}, \\quad C=\\frac{(c-d)(c-a)}{c+d+a}, \\quad D=\\frac{(d-a)(d-b)}{d+a+b} . $$ The expression $2 A$ splits into two summands as follows, $$ 2 A=A^{\\prime}+A^{\\prime \\prime} \\quad \\text { where } \\quad A^{\\prime}=\\frac{(a-c)^{2}}{a+b+c}, \\quad A^{\\prime \\prime}=\\frac{(a-c)(a-2 b+c)}{a+b+c} ; $$ this is easily verified. We analogously represent $2 B=B^{\\prime}+B^{\\prime \\prime}, 2 C=C^{\\prime}+C^{\\prime \\prime}, 2 B=D^{\\prime}+D^{\\prime \\prime}$ and examine each of the sums $A^{\\prime}+B^{\\prime}+C^{\\prime}+D^{\\prime}$ and $A^{\\prime \\prime}+B^{\\prime \\prime}+C^{\\prime \\prime}+D^{\\prime \\prime}$ separately. Write $s=a+b+c+d$; the denominators become $s-d, s-a, s-b, s-c$. By the CauchySchwarz inequality, $$ \\begin{aligned} & \\left(\\frac{|a-c|}{\\sqrt{s-d}} \\cdot \\sqrt{s-d}+\\frac{|b-d|}{\\sqrt{s-a}} \\cdot \\sqrt{s-a}+\\frac{|c-a|}{\\sqrt{s-b}} \\cdot \\sqrt{s-b}+\\frac{|d-b|}{\\sqrt{s-c}} \\cdot \\sqrt{s-c}\\right)^{2} \\\\ & \\quad \\leq\\left(\\frac{(a-c)^{2}}{s-d}+\\frac{(b-d)^{2}}{s-a}+\\frac{(c-a)^{2}}{s-b}+\\frac{(d-b)^{2}}{s-c}\\right)(4 s-s)=3 s\\left(A^{\\prime}+B^{\\prime}+C^{\\prime}+D^{\\prime}\\right) . \\end{aligned} $$ Hence $$ A^{\\prime}+B^{\\prime}+C^{\\prime}+D^{\\prime} \\geq \\frac{(2|a-c|+2|b-d|)^{2}}{3 s} \\geq \\frac{16 \\cdot|a-c| \\cdot|b-d|}{3 s} . $$ Next we estimate the absolute value of the other sum. We couple $A^{\\prime \\prime}$ with $C^{\\prime \\prime}$ to obtain $$ \\begin{aligned} A^{\\prime \\prime}+C^{\\prime \\prime} & =\\frac{(a-c)(a+c-2 b)}{s-d}+\\frac{(c-a)(c+a-2 d)}{s-b} \\\\ & =\\frac{(a-c)(a+c-2 b)(s-b)+(c-a)(c+a-2 d)(s-d)}{(s-d)(s-b)} \\\\ & =\\frac{(a-c)(-2 b(s-b)-b(a+c)+2 d(s-d)+d(a+c))}{s(a+c)+b d} \\\\ & =\\frac{3(a-c)(d-b)(a+c)}{M}, \\quad \\text { with } \\quad M=s(a+c)+b d . \\end{aligned} $$ Hence by cyclic shift $$ B^{\\prime \\prime}+D^{\\prime \\prime}=\\frac{3(b-d)(a-c)(b+d)}{N}, \\quad \\text { with } \\quad N=s(b+d)+c a $$ Thus $$ A^{\\prime \\prime}+B^{\\prime \\prime}+C^{\\prime \\prime}+D^{\\prime \\prime}=3(a-c)(b-d)\\left(\\frac{b+d}{N}-\\frac{a+c}{M}\\right)=\\frac{3(a-c)(b-d) W}{M N} $$ where $$ W=(b+d) M-(a+c) N=b d(b+d)-a c(a+c) . $$ Note that $$ M N>(a c(a+c)+b d(b+d)) s \\geq|W| \\cdot s . $$ Now (2) and (4) yield $$ \\left|A^{\\prime \\prime}+B^{\\prime \\prime}+C^{\\prime \\prime}+D^{\\prime \\prime}\\right| \\leq \\frac{3 \\cdot|a-c| \\cdot|b-d|}{s} $$ Combined with (1) this results in $$ \\begin{aligned} 2(A+B & +C+D)=\\left(A^{\\prime}+B^{\\prime}+C^{\\prime}+D^{\\prime}\\right)+\\left(A^{\\prime \\prime}+B^{\\prime \\prime}+C^{\\prime \\prime}+D^{\\prime \\prime}\\right) \\\\ & \\geq \\frac{16 \\cdot|a-c| \\cdot|b-d|}{3 s}-\\frac{3 \\cdot|a-c| \\cdot|b-d|}{s}=\\frac{7 \\cdot|a-c| \\cdot|b-d|}{3(a+b+c+d)} \\geq 0 \\end{aligned} $$ This is the required inequality. From the last line we see that equality can be achieved only if either $a=c$ or $b=d$. Since we also need equality in (1), this implies that actually $a=c$ and $b=d$ must hold simultaneously, which is obviously also a sufficient condition.","tier":0} +{"problem_type":"Algebra","problem_label":"A7","problem":"Prove that for any four positive real numbers $a, b, c, d$ the inequality $$ \\frac{(a-b)(a-c)}{a+b+c}+\\frac{(b-c)(b-d)}{b+c+d}+\\frac{(c-d)(c-a)}{c+d+a}+\\frac{(d-a)(d-b)}{d+a+b} \\geq 0 $$ holds. Determine all cases of equality.","solution":"We keep the notations $A, B, C, D, s$, and also $M, N, W$ from the preceding solution; the definitions of $M, N, W$ and relations (3), (4) in that solution did not depend on the foregoing considerations. Starting from $$ 2 A=\\frac{(a-c)^{2}+3(a+c)(a-c)}{a+b+c}-2 a+2 c $$ we get $$ \\begin{aligned} 2(A & +C)=(a-c)^{2}\\left(\\frac{1}{s-d}+\\frac{1}{s-b}\\right)+3(a+c)(a-c)\\left(\\frac{1}{s-d}-\\frac{1}{s-b}\\right) \\\\ & =(a-c)^{2} \\frac{2 s-b-d}{M}+3(a+c)(a-c) \\cdot \\frac{d-b}{M}=\\frac{p(a-c)^{2}-3(a+c)(a-c)(b-d)}{M} \\end{aligned} $$ where $p=2 s-b-d=s+a+c$. Similarly, writing $q=s+b+d$ we have $$ 2(B+D)=\\frac{q(b-d)^{2}-3(b+d)(b-d)(c-a)}{N} ; $$ specific grouping of terms in the numerators has its aim. Note that $p q>2 s^{2}$. By adding the fractions expressing $2(A+C)$ and $2(B+D)$, $$ 2(A+B+C+D)=\\frac{p(a-c)^{2}}{M}+\\frac{3(a-c)(b-d) W}{M N}+\\frac{q(b-d)^{2}}{N} $$ with $W$ defined by (3). Substitution $x=(a-c) \/ M, y=(b-d) \/ N$ brings the required inequality to the form $$ 2(A+B+C+D)=M p x^{2}+3 W x y+N q y^{2} \\geq 0 . $$ It will be enough to verify that the discriminant $\\Delta=9 W^{2}-4 M N p q$ of the quadratic trinomial $M p t^{2}+3 W t+N q$ is negative; on setting $t=x \/ y$ one then gets (6). The first inequality in (4) together with $p q>2 s^{2}$ imply $4 M N p q>8 s^{3}(a c(a+c)+b d(b+d))$. Since $$ (a+c) s^{3}>(a+c)^{4} \\geq 4 a c(a+c)^{2} \\quad \\text { and likewise } \\quad(b+d) s^{3}>4 b d(b+d)^{2} $$ the estimate continues as follows, $$ 4 M N p q>8\\left(4(a c)^{2}(a+c)^{2}+4(b d)^{2}(b+d)^{2}\\right)>32(b d(b+d)-a c(a+c))^{2}=32 W^{2} \\geq 9 W^{2} $$ Thus indeed $\\Delta<0$. The desired inequality (6) hence results. It becomes an equality if and only if $x=y=0$; equivalently, if and only if $a=c$ and simultaneously $b=d$. Comment. The two solutions presented above do not differ significantly; large portions overlap. The properties of the number $W$ turn out to be crucial in both approaches. The Cauchy-Schwarz inequality, applied in the first solution, is avoided in the second, which requires no knowledge beyond quadratic trinomials. The estimates in the proof of $\\Delta<0$ in the second solution seem to be very wasteful. However, they come close to sharp when the terms in one of the pairs $(a, c),(b, d)$ are equal and much bigger than those in the other pair. In attempts to prove the inequality by just considering the six cases of arrangement of the numbers $a, b, c, d$ on the real line, one soon discovers that the cases which create real trouble are precisely those in which $a$ and $c$ are both greater or both smaller than $b$ and $d$.","tier":0} +{"problem_type":"Algebra","problem_label":"A7","problem":"Prove that for any four positive real numbers $a, b, c, d$ the inequality $$ \\frac{(a-b)(a-c)}{a+b+c}+\\frac{(b-c)(b-d)}{b+c+d}+\\frac{(c-d)(c-a)}{c+d+a}+\\frac{(d-a)(d-b)}{d+a+b} \\geq 0 $$ holds. Determine all cases of equality.","solution":"$$ \\begin{gathered} (a-b)(a-c)(a+b+d)(a+c+d)(b+c+d)= \\\\ =((a-b)(a+b+d))((a-c)(a+c+d))(b+c+d)= \\\\ =\\left(a^{2}+a d-b^{2}-b d\\right)\\left(a^{2}+a d-c^{2}-c d\\right)(b+c+d)= \\\\ =\\left(a^{4}+2 a^{3} d-a^{2} b^{2}-a^{2} b d-a^{2} c^{2}-a^{2} c d+a^{2} d^{2}-a b^{2} d-a b d^{2}-a c^{2} d-a c d^{2}+b^{2} c^{2}+b^{2} c d+b c^{2} d+b c d^{2}\\right)(b+c+d)= \\\\ =a^{4} b+a^{4} c+a^{4} d+\\left(b^{3} c^{2}+a^{2} d^{3}\\right)-a^{2} c^{3}+\\left(2 a^{3} d^{2}-b^{3} a^{2}+c^{3} b^{2}\\right)+ \\\\ +\\left(b^{3} c d-c^{3} d a-d^{3} a b\\right)+\\left(2 a^{3} b d+c^{3} d b-d^{3} a c\\right)+\\left(2 a^{3} c d-b^{3} d a+d^{3} b c\\right) \\\\ +\\left(-a^{2} b^{2} c+3 b^{2} c^{2} d-2 a c^{2} d^{2}\\right)+\\left(-2 a^{2} b^{2} d+2 b c^{2} d^{2}\\right)+\\left(-a^{2} b c^{2}-2 a^{2} c^{2} d-2 a b^{2} d^{2}+2 b^{2} c d^{2}\\right)+ \\\\ +\\left(-2 a^{2} b c d-a b^{2} c d-a b c^{2} d-2 a b c d^{2}\\right) \\\\ \\text { Introducing the notation } S_{x y z w}=\\sum_{c y c} a^{x} b^{y} c^{z} d^{w}, \\text { one can write } \\\\ \\sum_{c y c}(a-b)(a-c)(a+b+d)(a+c+d)(b+c+d)= \\\\ =S_{4100}+S_{4010}+S_{4001}+2 S_{3200}-S_{3020}+2 S_{3002}-S_{3110}+2 S_{3101}+2 S_{3011}-3 S_{2120}-6 S_{2111}= \\\\ +\\left(S_{4100}+S_{4001}+\\frac{1}{2} S_{3110}+\\frac{1}{2} S_{3011}-3 S_{2120}\\right)+ \\\\ +\\left(S_{4010}-S_{3020}-\\frac{3}{2} S_{3110}+\\frac{3}{2} S_{3011}+\\frac{9}{16} S_{2210}+\\frac{9}{16} S_{2201}-\\frac{9}{8} S_{2111}\\right)+ \\\\ +\\frac{9}{16}\\left(S_{3200}-S_{2210}-S_{2201}+S_{3002}\\right)+\\frac{23}{16}\\left(S_{3200}-2 S_{3101}+S_{3002}\\right)+\\frac{39}{8}\\left(S_{3101}-S_{2111}\\right), \\end{gathered} $$ where the expressions $$ \\begin{gathered} S_{4100}+S_{4001}+\\frac{1}{2} S_{3110}+\\frac{1}{2} S_{3011}-3 S_{2120}=\\sum_{c y c}\\left(a^{4} b+b c^{4}+\\frac{1}{2} a^{3} b c+\\frac{1}{2} a b c^{3}-3 a^{2} b c^{2}\\right), \\\\ S_{4010}-S_{3020}-\\frac{3}{2} S_{3110}+\\frac{3}{2} S_{3011}+\\frac{9}{16} S_{2210}+\\frac{9}{16} S_{2201}-\\frac{9}{8} S_{2111}=\\sum_{c y c} a^{2} c\\left(a-c-\\frac{3}{4} b+\\frac{3}{4} d\\right)^{2}, \\\\ S_{3200}-S_{2210}-S_{2201}+S_{3002}=\\sum_{c y c} b^{2}\\left(a^{3}-a^{2} c-a c^{2}+c^{3}\\right)=\\sum_{c y c} b^{2}(a+c)(a-c)^{2}, \\\\ S_{3200}-2 S_{3101}+S_{3002}=\\sum_{c y c} a^{3}(b-d)^{2} \\quad \\text { and } \\quad S_{3101}-S_{2111}=\\frac{1}{3} \\sum_{c y c} b d\\left(2 a^{3}+c^{3}-3 a^{2} c\\right) \\end{gathered} $$ are all nonnegative.","tier":0} +{"problem_type":"Combinatorics","problem_label":"C1","problem":"In the plane we consider rectangles whose sides are parallel to the coordinate axes and have positive length. Such a rectangle will be called a box. Two boxes intersect if they have a common point in their interior or on their boundary. Find the largest $n$ for which there exist $n$ boxes $B_{1}, \\ldots, B_{n}$ such that $B_{i}$ and $B_{j}$ intersect if and only if $i \\not \\equiv j \\pm 1(\\bmod n)$.","solution":"The maximum number of such boxes is 6 . One example is shown in the figure. ![](https:\/\/cdn.mathpix.com\/cropped\/2024_04_17_e32cef0f3c5b05a0a6b1g-22.jpg?height=437&width=457&top_left_y=1135&top_left_x=777) Now we show that 6 is the maximum. Suppose that boxes $B_{1}, \\ldots, B_{n}$ satisfy the condition. Let the closed intervals $I_{k}$ and $J_{k}$ be the projections of $B_{k}$ onto the $x$ - and $y$-axis, for $1 \\leq k \\leq n$. If $B_{i}$ and $B_{j}$ intersect, with a common point $(x, y)$, then $x \\in I_{i} \\cap I_{j}$ and $y \\in J_{i} \\cap J_{j}$. So the intersections $I_{i} \\cap I_{j}$ and $J_{i} \\cap J_{j}$ are nonempty. Conversely, if $x \\in I_{i} \\cap I_{j}$ and $y \\in J_{i} \\cap J_{j}$ for some real numbers $x, y$, then $(x, y)$ is a common point of $B_{i}$ and $B_{j}$. Putting it around, $B_{i}$ and $B_{j}$ are disjoint if and only if their projections on at least one coordinate axis are disjoint. For brevity we call two boxes or intervals adjacent if their indices differ by 1 modulo $n$, and nonadjacent otherwise. The adjacent boxes $B_{k}$ and $B_{k+1}$ do not intersect for each $k=1, \\ldots, n$. Hence $\\left(I_{k}, I_{k+1}\\right)$ or $\\left(J_{k}, J_{k+1}\\right)$ is a pair of disjoint intervals, $1 \\leq k \\leq n$. So there are at least $n$ pairs of disjoint intervals among $\\left(I_{1}, I_{2}\\right), \\ldots,\\left(I_{n-1}, I_{n}\\right),\\left(I_{n}, I_{1}\\right) ;\\left(J_{1}, J_{2}\\right), \\ldots,\\left(J_{n-1}, J_{n}\\right),\\left(J_{n}, J_{1}\\right)$. Next, every two nonadjacent boxes intersect, hence their projections on both axes intersect, too. Then the claim below shows that at most 3 pairs among $\\left(I_{1}, I_{2}\\right), \\ldots,\\left(I_{n-1}, I_{n}\\right),\\left(I_{n}, I_{1}\\right)$ are disjoint, and the same holds for $\\left(J_{1}, J_{2}\\right) \\ldots,\\left(J_{n-1}, J_{n}\\right),\\left(J_{n}, J_{1}\\right)$. Consequently $n \\leq 3+3=6$, as stated. Thus we are left with the claim and its justification. Claim. Let $\\Delta_{1}, \\Delta_{2}, \\ldots, \\Delta_{n}$ be intervals on a straight line such that every two nonadjacent intervals intersect. Then $\\Delta_{k}$ and $\\Delta_{k+1}$ are disjoint for at most three values of $k=1, \\ldots, n$. Proof. Denote $\\Delta_{k}=\\left[a_{k}, b_{k}\\right], 1 \\leq k \\leq n$. Let $\\alpha=\\max \\left(a_{1}, \\ldots, a_{n}\\right)$ be the rightmost among the left endpoints of $\\Delta_{1}, \\ldots, \\Delta_{n}$, and let $\\beta=\\min \\left(b_{1}, \\ldots, b_{n}\\right)$ be the leftmost among their right endpoints. Assume that $\\alpha=a_{2}$ without loss of generality. If $\\alpha \\leq \\beta$ then $a_{i} \\leq \\alpha \\leq \\beta \\leq b_{i}$ for all $i$. Every $\\Delta_{i}$ contains $\\alpha$, and thus no disjoint pair $\\left(\\Delta_{i}, \\Delta_{i+1}\\right)$ exists. If $\\beta<\\alpha$ then $\\beta=b_{i}$ for some $i$ such that $a_{i}3$ and consider any nice permutation $\\left(a_{1}, a_{2}, \\ldots, a_{n}\\right)$ of $\\{1,2, \\ldots, n\\}$. Then $n-1$ must be a divisor of the number $$ \\begin{aligned} & 2\\left(a_{1}+a_{2}+\\cdots+a_{n-1}\\right)=2\\left((1+2+\\cdots+n)-a_{n}\\right) \\\\ & \\quad=n(n+1)-2 a_{n}=(n+2)(n-1)+\\left(2-2 a_{n}\\right) . \\end{aligned} $$ So $2 a_{n}-2$ must be divisible by $n-1$, hence equal to 0 or $n-1$ or $2 n-2$. This means that $$ a_{n}=1 \\quad \\text { or } \\quad a_{n}=\\frac{n+1}{2} \\quad \\text { or } \\quad a_{n}=n \\text {. } $$ Suppose that $a_{n}=(n+1) \/ 2$. Since the permutation is nice, taking $k=n-2$ we get that $n-2$ has to be a divisor of $$ \\begin{aligned} 2\\left(a_{1}+a_{2}+\\cdots+a_{n-2}\\right) & =2\\left((1+2+\\cdots+n)-a_{n}-a_{n-1}\\right) \\\\ & =n(n+1)-(n+1)-2 a_{n-1}=(n+2)(n-2)+\\left(3-2 a_{n-1}\\right) . \\end{aligned} $$ So $2 a_{n-1}-3$ should be divisible by $n-2$, hence equal to 0 or $n-2$ or $2 n-4$. Obviously 0 and $2 n-4$ are excluded because $2 a_{n-1}-3$ is odd. The remaining possibility $\\left(2 a_{n-1}-3=n-2\\right)$ leads to $a_{n-1}=(n+1) \/ 2=a_{n}$, which also cannot hold. This eliminates $(n+1) \/ 2$ as a possible value of $a_{n}$. Consequently $a_{n}=1$ or $a_{n}=n$. If $a_{n}=n$ then $\\left(a_{1}, a_{2}, \\ldots, a_{n-1}\\right)$ is a nice permutation of $\\{1,2, \\ldots, n-1\\}$. There are $F_{n-1}$ such permutations. Attaching $n$ to any one of them at the end creates a nice permutation of $\\{1,2, \\ldots, n\\}$. If $a_{n}=1$ then $\\left(a_{1}-1, a_{2}-1, \\ldots, a_{n-1}-1\\right)$ is a permutation of $\\{1,2, \\ldots, n-1\\}$. It is also nice because the number $$ 2\\left(\\left(a_{1}-1\\right)+\\cdots+\\left(a_{k}-1\\right)\\right)=2\\left(a_{1}+\\cdots+a_{k}\\right)-2 k $$ is divisible by $k$, for any $k \\leq n-1$. And again, any one of the $F_{n-1}$ nice permutations $\\left(b_{1}, b_{2}, \\ldots, b_{n-1}\\right)$ of $\\{1,2, \\ldots, n-1\\}$ gives rise to a nice permutation of $\\{1,2, \\ldots, n\\}$ whose last term is 1 , namely $\\left(b_{1}+1, b_{2}+1, \\ldots, b_{n-1}+1,1\\right)$. The bijective correspondences established in both cases show that there are $F_{n-1}$ nice permutations of $\\{1,2, \\ldots, n\\}$ with the last term 1 and also $F_{n-1}$ nice permutations of $\\{1,2, \\ldots, n\\}$ with the last term $n$. Hence follows the recurrence $F_{n}=2 F_{n-1}$. With the base value $F_{3}=6$ this gives the outcome formula $F_{n}=3 \\cdot 2^{n-2}$ for $n \\geq 3$.","tier":0} +{"problem_type":"Combinatorics","problem_label":"C3","problem":"In the coordinate plane consider the set $S$ of all points with integer coordinates. For a positive integer $k$, two distinct points $A, B \\in S$ will be called $k$-friends if there is a point $C \\in S$ such that the area of the triangle $A B C$ is equal to $k$. A set $T \\subset S$ will be called a $k$-clique if every two points in $T$ are $k$-friends. Find the least positive integer $k$ for which there exists a $k$-clique with more than 200 elements.","solution":"To begin, let us describe those points $B \\in S$ which are $k$-friends of the point $(0,0)$. By definition, $B=(u, v)$ satisfies this condition if and only if there is a point $C=(x, y) \\in S$ such that $\\frac{1}{2}|u y-v x|=k$. (This is a well-known formula expressing the area of triangle $A B C$ when $A$ is the origin.) To say that there exist integers $x, y$ for which $|u y-v x|=2 k$, is equivalent to saying that the greatest common divisor of $u$ and $v$ is also a divisor of $2 k$. Summing up, a point $B=(u, v) \\in S$ is a $k$-friend of $(0,0)$ if and only if $\\operatorname{gcd}(u, v)$ divides $2 k$. Translation by a vector with integer coordinates does not affect $k$-friendship; if two points are $k$-friends, so are their translates. It follows that two points $A, B \\in S, A=(s, t), B=(u, v)$, are $k$-friends if and only if the point $(u-s, v-t)$ is a $k$-friend of $(0,0)$; i.e., if $\\operatorname{gcd}(u-s, v-t) \\mid 2 k$. Let $n$ be a positive integer which does not divide $2 k$. We claim that a $k$-clique cannot have more than $n^{2}$ elements. Indeed, all points $(x, y) \\in S$ can be divided into $n^{2}$ classes determined by the remainders that $x$ and $y$ leave in division by $n$. If a set $T$ has more than $n^{2}$ elements, some two points $A, B \\in T, A=(s, t), B=(u, v)$, necessarily fall into the same class. This means that $n \\mid u-s$ and $n \\mid v-t$. Hence $n \\mid d$ where $d=\\operatorname{gcd}(u-s, v-t)$. And since $n$ does not divide $2 k$, also $d$ does not divide $2 k$. Thus $A$ and $B$ are not $k$-friends and the set $T$ is not a $k$-clique. Now let $M(k)$ be the least positive integer which does not divide $2 k$. Write $M(k)=m$ for the moment and consider the set $T$ of all points $(x, y)$ with $0 \\leq x, y200$. By the definition of $M(k), 2 k$ is divisible by the numbers $1,2, \\ldots, M(k)-1$, but not by $M(k)$ itself. If $M(k)^{2}>200$ then $M(k) \\geq 15$. Trying to hit $M(k)=15$ we get a contradiction immediately ( $2 k$ would have to be divisible by 3 and 5 , but not by 15 ). So let us try $M(k)=16$. Then $2 k$ is divisible by the numbers $1,2, \\ldots, 15$, hence also by their least common multiple $L$, but not by 16 . And since $L$ is not a multiple of 16 , we infer that $k=L \/ 2$ is the least $k$ with $M(k)=16$. Finally, observe that if $M(k) \\geq 17$ then $2 k$ must be divisible by the least common multiple of $1,2, \\ldots, 16$, which is equal to $2 L$. Then $2 k \\geq 2 L$, yielding $k>L \/ 2$. In conclusion, the least $k$ with the required property is equal to $L \/ 2=180180$.","tier":0} +{"problem_type":"Combinatorics","problem_label":"C4","problem":"Let $n$ and $k$ be fixed positive integers of the same parity, $k \\geq n$. We are given $2 n$ lamps numbered 1 through $2 n$; each of them can be on or off. At the beginning all lamps are off. We consider sequences of $k$ steps. At each step one of the lamps is switched (from off to on or from on to off). Let $N$ be the number of $k$-step sequences ending in the state: lamps $1, \\ldots, n$ on, lamps $n+1, \\ldots, 2 n$ off. Let $M$ be the number of $k$-step sequences leading to the same state and not touching lamps $n+1, \\ldots, 2 n$ at all. Find the ratio $N \/ M$.","solution":"A sequence of $k$ switches ending in the state as described in the problem statement (lamps $1, \\ldots, n$ on, lamps $n+1, \\ldots, 2 n$ off) will be called an admissible process. If, moreover, the process does not touch the lamps $n+1, \\ldots, 2 n$, it will be called restricted. So there are $N$ admissible processes, among which $M$ are restricted. In every admissible process, restricted or not, each one of the lamps $1, \\ldots, n$ goes from off to on, so it is switched an odd number of times; and each one of the lamps $n+1, \\ldots, 2 n$ goes from off to off, so it is switched an even number of times. Notice that $M>0$; i.e., restricted admissible processes do exist (it suffices to switch each one of the lamps $1, \\ldots, n$ just once and then choose one of them and switch it $k-n$ times, which by hypothesis is an even number). Consider any restricted admissible process $\\mathbf{p}$. Take any lamp $\\ell, 1 \\leq \\ell \\leq n$, and suppose that it was switched $k_{\\ell}$ times. As noticed, $k_{\\ell}$ must be odd. Select arbitrarily an even number of these $k_{\\ell}$ switches and replace each of them by the switch of lamp $n+\\ell$. This can be done in $2^{k_{\\ell}-1}$ ways (because a $k_{\\ell}$-element set has $2^{k_{\\ell}-1}$ subsets of even cardinality). Notice that $k_{1}+\\cdots+k_{n}=k$. These actions are independent, in the sense that the action involving lamp $\\ell$ does not affect the action involving any other lamp. So there are $2^{k_{1}-1} \\cdot 2^{k_{2}-1} \\cdots 2^{k_{n}-1}=2^{k-n}$ ways of combining these actions. In any of these combinations, each one of the lamps $n+1, \\ldots, 2 n$ gets switched an even number of times and each one of the lamps $1, \\ldots, n$ remains switched an odd number of times, so the final state is the same as that resulting from the original process $\\mathbf{p}$. This shows that every restricted admissible process $\\mathbf{p}$ can be modified in $2^{k-n}$ ways, giving rise to $2^{k-n}$ distinct admissible processes (with all lamps allowed). Now we show that every admissible process $\\mathbf{q}$ can be achieved in that way. Indeed, it is enough to replace every switch of a lamp with a label $\\ell>n$ that occurs in $\\mathbf{q}$ by the switch of the corresponding lamp $\\ell-n$; in the resulting process $\\mathbf{p}$ the lamps $n+1, \\ldots, 2 n$ are not involved. Switches of each lamp with a label $\\ell>n$ had occurred in $\\mathbf{q}$ an even number of times. So the performed replacements have affected each lamp with a label $\\ell \\leq n$ also an even number of times; hence in the overall effect the final state of each lamp has remained the same. This means that the resulting process $\\mathbf{p}$ is admissible - and clearly restricted, as the lamps $n+1, \\ldots, 2 n$ are not involved in it any more. If we now take process $\\mathbf{p}$ and reverse all these replacements, then we obtain process $\\mathbf{q}$. These reversed replacements are nothing else than the modifications described in the foregoing paragraphs. Thus there is a one-to $-\\left(2^{k-n}\\right)$ correspondence between the $M$ restricted admissible processes and the total of $N$ admissible processes. Therefore $N \/ M=2^{k-n}$.","tier":0} +{"problem_type":"Combinatorics","problem_label":"C5","problem":"Let $S=\\left\\{x_{1}, x_{2}, \\ldots, x_{k+\\ell}\\right\\}$ be a $(k+\\ell)$-element set of real numbers contained in the interval $[0,1] ; k$ and $\\ell$ are positive integers. A $k$-element subset $A \\subset S$ is called nice if $$ \\left|\\frac{1}{k} \\sum_{x_{i} \\in A} x_{i}-\\frac{1}{\\ell} \\sum_{x_{j} \\in S \\backslash A} x_{j}\\right| \\leq \\frac{k+\\ell}{2 k \\ell} . $$ Prove that the number of nice subsets is at least $\\frac{2}{k+\\ell}\\left(\\begin{array}{c}k+\\ell \\\\ k\\end{array}\\right)$.","solution":"For a $k$-element subset $A \\subset S$, let $f(A)=\\frac{1}{k} \\sum_{x_{i} \\in A} x_{i}-\\frac{1}{\\ell} \\sum_{x_{j} \\in S \\backslash A} x_{j}$. Denote $\\frac{k+\\ell}{2 k \\ell}=d$. By definition a subset $A$ is nice if $|f(A)| \\leq d$. To each permutation $\\left(y_{1}, y_{2}, \\ldots, y_{k+\\ell}\\right)$ of the set $S=\\left\\{x_{1}, x_{2}, \\ldots, x_{k+\\ell}\\right\\}$ we assign $k+\\ell$ subsets of $S$ with $k$ elements each, namely $A_{i}=\\left\\{y_{i}, y_{i+1}, \\ldots, y_{i+k-1}\\right\\}, i=1,2, \\ldots, k+\\ell$. Indices are taken modulo $k+\\ell$ here and henceforth. In other words, if $y_{1}, y_{2}, \\ldots, y_{k+\\ell}$ are arranged around a circle in this order, the sets in question are all possible blocks of $k$ consecutive elements. Claim. At least two nice sets are assigned to every permutation of $S$. Proof. Adjacent sets $A_{i}$ and $A_{i+1}$ differ only by the elements $y_{i}$ and $y_{i+k}, i=1, \\ldots, k+\\ell$. By the definition of $f$, and because $y_{i}, y_{i+k} \\in[0,1]$, $$ \\left|f\\left(A_{i+1}\\right)-f\\left(A_{i}\\right)\\right|=\\left|\\left(\\frac{1}{k}+\\frac{1}{\\ell}\\right)\\left(y_{i+k}-y_{i}\\right)\\right| \\leq \\frac{1}{k}+\\frac{1}{\\ell}=2 d \\text {. } $$ Each element $y_{i} \\in S$ belongs to exactly $k$ of the sets $A_{1}, \\ldots, A_{k+\\ell}$. Hence in $k$ of the expressions $f\\left(A_{1}\\right), \\ldots, f\\left(A_{k+\\ell}\\right)$ the coefficient of $y_{i}$ is $1 \/ k$; in the remaining $\\ell$ expressions, its coefficient is $-1 \/ \\ell$. So the contribution of $y_{i}$ to the sum of all $f\\left(A_{i}\\right)$ equals $k \\cdot 1 \/ k-\\ell \\cdot 1 \/ \\ell=0$. Since this holds for all $i$, it follows that $f\\left(A_{1}\\right)+\\cdots+f\\left(A_{k+\\ell}\\right)=0$. If $f\\left(A_{p}\\right)=\\min f\\left(A_{i}\\right), f\\left(A_{q}\\right)=\\max f\\left(A_{i}\\right)$, we obtain in particular $f\\left(A_{p}\\right) \\leq 0, f\\left(A_{q}\\right) \\geq 0$. Let $pq$ is analogous; and the claim is true for $p=q$ as $f\\left(A_{i}\\right)=0$ for all $i$ ). We are ready to prove that at least two of the sets $A_{1}, \\ldots, A_{k+\\ell}$ are nice. The interval $[-d, d]$ has length $2 d$, and we saw that adjacent numbers in the circular arrangement $f\\left(A_{1}\\right), \\ldots, f\\left(A_{k+\\ell}\\right)$ differ by at most $2 d$. Suppose that $f\\left(A_{p}\\right)<-d$ and $f\\left(A_{q}\\right)>d$. Then one of the numbers $f\\left(A_{p+1}\\right), \\ldots, f\\left(A_{q-1}\\right)$ lies in $[-d, d]$, and also one of the numbers $f\\left(A_{q+1}\\right), \\ldots, f\\left(A_{p-1}\\right)$ lies there. Consequently, one of the sets $A_{p+1}, \\ldots, A_{q-1}$ is nice, as well as one of the sets $A_{q+1}, \\ldots, A_{p-1}$. If $-d \\leq f\\left(A_{p}\\right)$ and $f\\left(A_{q}\\right) \\leq d$ then $A_{p}$ and $A_{q}$ are nice. Let now $f\\left(A_{p}\\right)<-d$ and $f\\left(A_{q}\\right) \\leq d$. Then $f\\left(A_{p}\\right)+f\\left(A_{q}\\right)<0$, and since $\\sum f\\left(A_{i}\\right)=0$, there is an $r \\neq q$ such that $f\\left(A_{r}\\right)>0$. We have $02\\left(u_{m-1}-u_{1}\\right)$ for all $m \\geq 3$. Indeed, assume that $u_{m}-u_{1} \\leq 2\\left(u_{m-1}-u_{1}\\right)$ holds for some $m \\geq 3$. This inequality can be written as $2\\left(u_{m}-u_{m-1}\\right) \\leq u_{m}-u_{1}$. Take the unique $k$ such that $2^{k} \\leq u_{m}-u_{1}<2^{k+1}$. Then $2\\left(u_{m}-u_{m-1}\\right) \\leq u_{m}-u_{1}<2^{k+1}$ yields $u_{m}-u_{m-1}<2^{k}$. However the elements $z=u_{m}, x=u_{1}$, $y=u_{m-1}$ of $S_{a}$ then satisfy $z-y<2^{k}$ and $z-x \\geq 2^{k}$, so that $z=u_{m}$ is $k$-good to $S_{a}$. Thus each term of the sequence $u_{2}-u_{1}, u_{3}-u_{1}, \\ldots, u_{p}-u_{1}$ is more than twice the previous one. Hence $u_{p}-u_{1}>2^{p-1}\\left(u_{2}-u_{1}\\right) \\geq 2^{p-1}$. But $u_{p} \\in\\left\\{1,2,3, \\ldots, 2^{n+1}\\right\\}$, so that $u_{p} \\leq 2^{n+1}$. This yields $p-1 \\leq n$, i. e. $p \\leq n+1$. In other words, each set $S_{a}$ contains at most $n+1$ elements that are not good to it. To summarize the conclusions, mark with red all elements in the sets $S_{a}$ that are good to the respective set, and with blue the ones that are not good. Then the total number of red elements, counting multiplicities, is at most $n \\cdot 2^{n+1}$ (each $z \\in A$ can be marked red in at most $n$ sets). The total number of blue elements is at most $(n+1) 2^{n}$ (each set $S_{a}$ contains at most $n+1$ blue elements). Therefore the sum of cardinalities of $S_{1}, S_{2}, \\ldots, S_{2^{n}}$ does not exceed $(3 n+1) 2^{n}$. By averaging, the smallest set has at most $3 n+1$ elements.","tier":0} +{"problem_type":"Combinatorics","problem_label":"C6","problem":"For $n \\geq 2$, let $S_{1}, S_{2}, \\ldots, S_{2^{n}}$ be $2^{n}$ subsets of $A=\\left\\{1,2,3, \\ldots, 2^{n+1}\\right\\}$ that satisfy the following property: There do not exist indices $a$ and $b$ with $a2^{n}$, which implies $z=v_{a}$. It follows that if the element $v_{i}$ is removed from each $V_{i}$, a family of pairwise disjoint sets $W_{i}=V_{i} \\backslash\\left\\{v_{i}\\right\\}$ is obtained, $i \\in I$ (we assume $W_{i}=\\emptyset$ if $V_{i}=\\emptyset$ ). As $W_{i} \\subseteq\\left\\{2^{n}+1, \\ldots, 2^{n+1}\\right\\}$ for all $i$, we infer that $\\sum_{i \\in I}\\left|W_{i}\\right| \\leq 2^{n}$. Therefore $\\sum_{i \\in I}\\left(\\left|V_{i}\\right|-1\\right) \\leq \\sum_{i \\in I}\\left|W_{i}\\right| \\leq 2^{n}$. On the other hand, the induction hypothesis applies directly to the sets $U_{i}, i \\in I$, so that $\\sum_{i \\in \\mathcal{I}}\\left(\\left|U_{i}\\right|-n\\right) \\leq(2 n-1) 2^{n-2}$. In summary, $$ \\sum_{i \\in I}\\left(\\left|S_{i}\\right|-(n+1)\\right)=\\sum_{i \\in I}\\left(\\left|U_{i}\\right|-n\\right)+\\sum_{i \\in I}\\left(\\left|V_{i}\\right|-1\\right) \\leq(2 n-1) 2^{n-2}+2^{n} $$ The estimates (1) and (2) are sufficient to complete the inductive step: $$ \\begin{aligned} \\sum_{i=1}^{k}\\left(\\left|S_{i}\\right|-(n+1)\\right) & =\\sum_{i \\in I}\\left(\\left|S_{i}\\right|-(n+1)\\right)+\\sum_{j \\in J}\\left(\\left|S_{j}\\right|-(n+1)\\right) \\\\ & \\leq(2 n-1) 2^{n-2}+2^{n}+(2 n-1) 2^{n-2}=(2 n+1) 2^{n-1} \\end{aligned} $$ Returning to the problem, consider $k=2^{n}$ subsets $S_{1}, S_{2}, \\ldots, S_{2^{n}}$ of $\\left\\{1,2,3, \\ldots, 2^{n+1}\\right\\}$. If they satisfy the given condition, the claim implies $\\sum_{i=1}^{2^{n}}\\left(\\left|S_{i}\\right|-(n+1)\\right) \\leq(2 n+1) 2^{n-1}$. By averaging again, we see that the smallest set has at most $2 n+1$ elements. Comment. It can happen that each set $S_{i}$ has cardinality at least $n+1$. Here is an example by the proposer. For $i=1, \\ldots, 2^{n}$, let $S_{i}=\\left\\{i+2^{k} \\mid 0 \\leq k \\leq n\\right\\}$. Then $\\left|S_{i}\\right|=n+1$ for all $i$. Suppose that there exist $al$. Since $y \\in S_{a}$ and $y180^{\\circ}, \\end{gathered} $$ point $P$ lies in triangle $C D Q$, and hence in $A B C D$. The proof is complete. ![](https:\/\/cdn.mathpix.com\/cropped\/2024_04_17_e32cef0f3c5b05a0a6b1g-40.jpg?height=757&width=1008&top_left_y=1489&top_left_x=501)","tier":0} +{"problem_type":"Geometry","problem_label":"G7","problem":"Let $A B C D$ be a convex quadrilateral with $A B \\neq B C$. Denote by $\\omega_{1}$ and $\\omega_{2}$ the incircles of triangles $A B C$ and $A D C$. Suppose that there exists a circle $\\omega$ inscribed in angle $A B C$, tangent to the extensions of line segments $A D$ and $C D$. Prove that the common external tangents of $\\omega_{1}$ and $\\omega_{2}$ intersect on $\\omega$.","solution":"The proof below is based on two known facts. Lemma 1. Given a convex quadrilateral $A B C D$, suppose that there exists a circle which is inscribed in angle $A B C$ and tangent to the extensions of line segments $A D$ and $C D$. Then $A B+A D=C B+C D$. Proof. The circle in question is tangent to each of the lines $A B, B C, C D, D A$, and the respective points of tangency $K, L, M, N$ are located as with circle $\\omega$ in the figure. Then $$ A B+A D=(B K-A K)+(A N-D N), \\quad C B+C D=(B L-C L)+(C M-D M) . $$ Also $B K=B L, D N=D M, A K=A N, C L=C M$ by equalities of tangents. It follows that $A B+A D=C B+C D$. ![](https:\/\/cdn.mathpix.com\/cropped\/2024_04_17_e32cef0f3c5b05a0a6b1g-41.jpg?height=1068&width=1079&top_left_y=1156&top_left_x=523) For brevity, in the sequel we write \"excircle $A C$ \" for the excircle of a triangle with side $A C$ which is tangent to line segment $A C$ and the extensions of the other two sides. Lemma 2. The incircle of triangle $A B C$ is tangent to its side $A C$ at $P$. Let $P P^{\\prime}$ be the diameter of the incircle through $P$, and let line $B P^{\\prime}$ intersect $A C$ at $Q$. Then $Q$ is the point of tangency of side $A C$ and excircle $A C$. Proof. Let the tangent at $P^{\\prime}$ to the incircle $\\omega_{1}$ meet $B A$ and $B C$ at $A^{\\prime}$ and $C^{\\prime}$. Now $\\omega_{1}$ is the excircle $A^{\\prime} C^{\\prime}$ of triangle $A^{\\prime} B C^{\\prime}$, and it touches side $A^{\\prime} C^{\\prime}$ at $P^{\\prime}$. Since $A^{\\prime} C^{\\prime} \\| A C$, the homothety with centre $B$ and ratio $B Q \/ B P^{\\prime}$ takes $\\omega_{1}$ to the excircle $A C$ of triangle $A B C$. Because this homothety takes $P^{\\prime}$ to $Q$, the lemma follows. Recall also that if the incircle of a triangle touches its side $A C$ at $P$, then the tangency point $Q$ of the same side and excircle $A C$ is the unique point on line segment $A C$ such that $A P=C Q$. We pass on to the main proof. Let $\\omega_{1}$ and $\\omega_{2}$ touch $A C$ at $P$ and $Q$, respectively; then $A P=(A C+A B-B C) \/ 2, C Q=(C A+C D-A D) \/ 2$. Since $A B-B C=C D-A D$ by Lemma 1, we obtain $A P=C Q$. It follows that in triangle $A B C$ side $A C$ and excircle $A C$ are tangent at $Q$. Likewise, in triangle $A D C$ side $A C$ and excircle $A C$ are tangent at $P$. Note that $P \\neq Q$ as $A B \\neq B C$. Let $P P^{\\prime}$ and $Q Q^{\\prime}$ be the diameters perpendicular to $A C$ of $\\omega_{1}$ and $\\omega_{2}$, respectively. Then Lemma 2 shows that points $B, P^{\\prime}$ and $Q$ are collinear, and so are points $D, Q^{\\prime}$ and $P$. Consider the diameter of $\\omega$ perpendicular to $A C$ and denote by $T$ its endpoint that is closer to $A C$. The homothety with centre $B$ and ratio $B T \/ B P^{\\prime}$ takes $\\omega_{1}$ to $\\omega$. Hence $B, P^{\\prime}$ and $T$ are collinear. Similarly, $D, Q^{\\prime}$ and $T$ are collinear since the homothety with centre $D$ and ratio $-D T \/ D Q^{\\prime}$ takes $\\omega_{2}$ to $\\omega$. We infer that points $T, P^{\\prime}$ and $Q$ are collinear, as well as $T, Q^{\\prime}$ and $P$. Since $P P^{\\prime} \\| Q Q^{\\prime}$, line segments $P P^{\\prime}$ and $Q Q^{\\prime}$ are then homothetic with centre $T$. The same holds true for circles $\\omega_{1}$ and $\\omega_{2}$ because they have $P P^{\\prime}$ and $Q Q^{\\prime}$ as diameters. Moreover, it is immediate that $T$ lies on the same side of line $P P^{\\prime}$ as $Q$ and $Q^{\\prime}$, hence the ratio of homothety is positive. In particular $\\omega_{1}$ and $\\omega_{2}$ are not congruent. In summary, $T$ is the centre of a homothety with positive ratio that takes circle $\\omega_{1}$ to circle $\\omega_{2}$. This completes the solution, since the only point with the mentioned property is the intersection of the the common external tangents of $\\omega_{1}$ and $\\omega_{2}$.","tier":0} +{"problem_type":"Number Theory","problem_label":"N1","problem":"Let $n$ be a positive integer and let $p$ be a prime number. Prove that if $a, b, c$ are integers (not necessarily positive) satisfying the equations $$ a^{n}+p b=b^{n}+p c=c^{n}+p a, $$ then $a=b=c$.","solution":"If two of $a, b, c$ are equal, it is immediate that all the three are equal. So we may assume that $a \\neq b \\neq c \\neq a$. Subtracting the equations we get $a^{n}-b^{n}=-p(b-c)$ and two cyclic copies of this equation, which upon multiplication yield $$ \\frac{a^{n}-b^{n}}{a-b} \\cdot \\frac{b^{n}-c^{n}}{b-c} \\cdot \\frac{c^{n}-a^{n}}{c-a}=-p^{3} . $$ If $n$ is odd then the differences $a^{n}-b^{n}$ and $a-b$ have the same sign and the product on the left is positive, while $-p^{3}$ is negative. So $n$ must be even. Let $d$ be the greatest common divisor of the three differences $a-b, b-c, c-a$, so that $a-b=d u, b-c=d v, \\quad c-a=d w ; \\operatorname{gcd}(u, v, w)=1, u+v+w=0$. From $a^{n}-b^{n}=-p(b-c)$ we see that $(a-b) \\mid p(b-c)$, i.e., $u \\mid p v$; and cyclically $v|p w, w| p u$. As $\\operatorname{gcd}(u, v, w)=1$ and $u+v+w=0$, at most one of $u, v, w$ can be divisible by $p$. Supposing that the prime $p$ does not divide any one of them, we get $u|v, v| w, w \\mid u$, whence $|u|=|v|=|w|=1$; but this quarrels with $u+v+w=0$. Thus $p$ must divide exactly one of these numbers. Let e.g. $p \\mid u$ and write $u=p u_{1}$. Now we obtain, similarly as before, $u_{1}|v, v| w, w \\mid u_{1}$ so that $\\left|u_{1}\\right|=|v|=|w|=1$. The equation $p u_{1}+v+w=0$ forces that the prime $p$ must be even; i.e. $p=2$. Hence $v+w=-2 u_{1}= \\pm 2$, implying $v=w(= \\pm 1)$ and $u=-2 v$. Consequently $a-b=-2(b-c)$. Knowing that $n$ is even, say $n=2 k$, we rewrite the equation $a^{n}-b^{n}=-p(b-c)$ with $p=2$ in the form $$ \\left(a^{k}+b^{k}\\right)\\left(a^{k}-b^{k}\\right)=-2(b-c)=a-b . $$ The second factor on the left is divisible by $a-b$, so the first factor $\\left(a^{k}+b^{k}\\right)$ must be $\\pm 1$. Then exactly one of $a$ and $b$ must be odd; yet $a-b=-2(b-c)$ is even. Contradiction ends the proof.","tier":0} +{"problem_type":"Number Theory","problem_label":"N1","problem":"Let $n$ be a positive integer and let $p$ be a prime number. Prove that if $a, b, c$ are integers (not necessarily positive) satisfying the equations $$ a^{n}+p b=b^{n}+p c=c^{n}+p a, $$ then $a=b=c$.","solution":"The beginning is as in the first solution. Assuming that $a, b, c$ are not all equal, hence are all distinct, we derive equation (1) with the conclusion that $n$ is even. Write $n=2 k$. Suppose that $p$ is odd. Then the integer $$ \\frac{a^{n}-b^{n}}{a-b}=a^{n-1}+a^{n-2} b+\\cdots+b^{n-1} $$ which is a factor in (1), must be odd as well. This sum of $n=2 k$ summands is odd only if $a$ and $b$ have different parities. The same conclusion holding for $b, c$ and for $c$, $a$, we get that $a, b, c, a$ alternate in their parities, which is clearly impossible. Thus $p=2$. The original system shows that $a, b, c$ must be of the same parity. So we may divide (1) by $p^{3}$, i.e. $2^{3}$, to obtain the following product of six integer factors: $$ \\frac{a^{k}+b^{k}}{2} \\cdot \\frac{a^{k}-b^{k}}{a-b} \\cdot \\frac{b^{k}+c^{k}}{2} \\cdot \\frac{b^{k}-c^{k}}{b-c} \\cdot \\frac{c^{k}+a^{k}}{2} \\cdot \\frac{c^{k}-a^{k}}{c-a}=-1 . $$ Each one of the factors must be equal to $\\pm 1$. In particular, $a^{k}+b^{k}= \\pm 2$. If $k$ is even, this becomes $a^{k}+b^{k}=2$ and yields $|a|=|b|=1$, whence $a^{k}-b^{k}=0$, contradicting (2). Let now $k$ be odd. Then the sum $a^{k}+b^{k}$, with value $\\pm 2$, has $a+b$ as a factor. Since $a$ and $b$ are of the same parity, this means that $a+b= \\pm 2$; and cyclically, $b+c= \\pm 2, c+a= \\pm 2$. In some two of these equations the signs must coincide, hence some two of $a, b, c$ are equal. This is the desired contradiction. Comment. Having arrived at the equation (1) one is tempted to write down all possible decompositions of $-p^{3}$ (cube of a prime) into a product of three integers. This leads to cumbersome examination of many cases, some of which are unpleasant to handle. One may do that just for $p=2$, having earlier in some way eliminated odd primes from consideration. However, the second solution shows that the condition of $p$ being a prime is far too strong. What is actually being used in that solution, is that $p$ is either a positive odd integer or $p=2$.","tier":0} +{"problem_type":"Number Theory","problem_label":"N2","problem":"Let $a_{1}, a_{2}, \\ldots, a_{n}$ be distinct positive integers, $n \\geq 3$. Prove that there exist distinct indices $i$ and $j$ such that $a_{i}+a_{j}$ does not divide any of the numbers $3 a_{1}, 3 a_{2}, \\ldots, 3 a_{n}$.","solution":"Without loss of generality, let $0a_{n-1}\\right)$. Thus $3 a_{n-1}=a_{n}+a_{n-1}$, i. e. $a_{n}=2 a_{n-1}$. Similarly, if $j=n$ then $3 a_{n}=k\\left(a_{n}+a_{n-1}\\right)$ for some integer $k$, and only $k=2$ is possible. Hence $a_{n}=2 a_{n-1}$ holds true in both cases remaining, $j=n-1$ and $j=n$. Now $a_{n}=2 a_{n-1}$ implies that the sum $a_{n-1}+a_{1}$ is strictly between $a_{n} \/ 2$ and $a_{n}$. But $a_{n-1}$ and $a_{1}$ are distinct as $n \\geq 3$, so it follows from the above that $a_{n-1}+a_{1}$ divides $a_{n}$. This provides the desired contradiction.","tier":0} +{"problem_type":"Number Theory","problem_label":"N3","problem":"Let $a_{0}, a_{1}, a_{2}, \\ldots$ be a sequence of positive integers such that the greatest common divisor of any two consecutive terms is greater than the preceding term; in symbols, $\\operatorname{gcd}\\left(a_{i}, a_{i+1}\\right)>a_{i-1}$. Prove that $a_{n} \\geq 2^{n}$ for all $n \\geq 0$.","solution":"Since $a_{i} \\geq \\operatorname{gcd}\\left(a_{i}, a_{i+1}\\right)>a_{i-1}$, the sequence is strictly increasing. In particular $a_{0} \\geq 1, a_{1} \\geq 2$. For each $i \\geq 1$ we also have $a_{i+1}-a_{i} \\geq \\operatorname{gcd}\\left(a_{i}, a_{i+1}\\right)>a_{i-1}$, and consequently $a_{i+1} \\geq a_{i}+a_{i-1}+1$. Hence $a_{2} \\geq 4$ and $a_{3} \\geq 7$. The equality $a_{3}=7$ would force equalities in the previous estimates, leading to $\\operatorname{gcd}\\left(a_{2}, a_{3}\\right)=\\operatorname{gcd}(4,7)>a_{1}=2$, which is false. Thus $a_{3} \\geq 8$; the result is valid for $n=0,1,2,3$. These are the base cases for a proof by induction. Take an $n \\geq 3$ and assume that $a_{i} \\geq 2^{i}$ for $i=0,1, \\ldots, n$. We must show that $a_{n+1} \\geq 2^{n+1}$. Let $\\operatorname{gcd}\\left(a_{n}, a_{n+1}\\right)=d$. We know that $d>a_{n-1}$. The induction claim is reached immediately in the following cases: $$ \\begin{array}{ll} \\text { if } \\quad a_{n+1} \\geq 4 d & \\text { then } a_{n+1}>4 a_{n-1} \\geq 4 \\cdot 2^{n-1}=2^{n+1} \\\\ \\text { if } \\quad a_{n} \\geq 3 d & \\text { then } a_{n+1} \\geq a_{n}+d \\geq 4 d>4 a_{n-1} \\geq 4 \\cdot 2^{n-1}=2^{n+1} \\\\ \\text { if } \\quad a_{n}=d & \\text { then } a_{n+1} \\geq a_{n}+d=2 a_{n} \\geq 2 \\cdot 2^{n}=2^{n+1} \\end{array} $$ The only remaining possibility is that $a_{n}=2 d$ and $a_{n+1}=3 d$, which we assume for the sequel. So $a_{n+1}=\\frac{3}{2} a_{n}$. Let now $\\operatorname{gcd}\\left(a_{n-1}, a_{n}\\right)=d^{\\prime}$; then $d^{\\prime}>a_{n-2}$. Write $a_{n}=m d^{\\prime} \\quad(m$ an integer $)$. Keeping in mind that $d^{\\prime} \\leq a_{n-1}9 a_{n-2} \\geq 9 \\cdot 2^{n-2}>2^{n+1} \\\\ & \\text { if } 3 \\leq m \\leq 4 \\text { then } a_{n-1}<\\frac{1}{2} \\cdot 4 d^{\\prime} \\text {, and hence } a_{n-1}=d^{\\prime} \\\\ & \\qquad a_{n+1}=\\frac{3}{2} m a_{n-1} \\geq \\frac{3}{2} \\cdot 3 a_{n-1} \\geq \\frac{9}{2} \\cdot 2^{n-1}>2^{n+1} . \\end{aligned} $$ So we are left with the case $m=5$, which means that $a_{n}=5 d^{\\prime}, a_{n+1}=\\frac{15}{2} d^{\\prime}, a_{n-1}a_{n-3}$. Because $d^{\\prime \\prime}$ is a divisor of $a_{n-1}$, hence also of $2 d^{\\prime}$, we may write $2 d^{\\prime}=m^{\\prime} d^{\\prime \\prime}$ ( $m^{\\prime}$ an integer). Since $d^{\\prime \\prime} \\leq a_{n-2}\\frac{75}{4} a_{n-3} \\geq \\frac{75}{4} \\cdot 2^{n-3}>2^{n+1} \\\\ & \\text { if } 3 \\leq m^{\\prime} \\leq 4 \\text { then } a_{n-2}<\\frac{1}{2} \\cdot 4 d^{\\prime \\prime} \\text {, and hence } a_{n-2}=d^{\\prime \\prime} \\\\ & \\qquad a_{n+1}=\\frac{15}{4} m^{\\prime} a_{n-2} \\geq \\frac{15}{4} \\cdot 3 a_{n-2} \\geq \\frac{45}{4} \\cdot 2^{n-2}>2^{n+1} . \\end{aligned} $$ Both of them have produced the induction claim. But now there are no cases left. Induction is complete; the inequality $a_{n} \\geq 2^{n}$ holds for all $n$.","tier":0} +{"problem_type":"Number Theory","problem_label":"N4","problem":"Let $n$ be a positive integer. Show that the numbers $$ \\left(\\begin{array}{c} 2^{n}-1 \\\\ 0 \\end{array}\\right), \\quad\\left(\\begin{array}{c} 2^{n}-1 \\\\ 1 \\end{array}\\right), \\quad\\left(\\begin{array}{c} 2^{n}-1 \\\\ 2 \\end{array}\\right), \\quad \\ldots, \\quad\\left(\\begin{array}{c} 2^{n}-1 \\\\ 2^{n-1}-1 \\end{array}\\right) $$ are congruent modulo $2^{n}$ to $1,3,5, \\ldots, 2^{n}-1$ in some order.","solution":"It is well-known that all these numbers are odd. So the assertion that their remainders $\\left(\\bmod 2^{n}\\right)$ make up a permutation of $\\left\\{1,3, \\ldots, 2^{n}-1\\right\\}$ is equivalent just to saying that these remainders are all distinct. We begin by showing that $$ \\left(\\begin{array}{c} 2^{n}-1 \\\\ 2 k \\end{array}\\right)+\\left(\\begin{array}{c} 2^{n}-1 \\\\ 2 k+1 \\end{array}\\right) \\equiv 0 \\quad\\left(\\bmod 2^{n}\\right) \\quad \\text { and } \\quad\\left(\\begin{array}{c} 2^{n}-1 \\\\ 2 k \\end{array}\\right) \\equiv(-1)^{k}\\left(\\begin{array}{c} 2^{n-1}-1 \\\\ k \\end{array}\\right) \\quad\\left(\\bmod 2^{n}\\right) $$ The first relation is immediate, as the sum on the left is equal to $\\left(\\begin{array}{c}2^{n} \\\\ 2 k+1\\end{array}\\right)=\\frac{2^{n}}{2 k+1}\\left(\\begin{array}{c}2^{n}-1 \\\\ 2 k\\end{array}\\right)$, hence is divisible by $2^{n}$. The second relation: $$ \\left(\\begin{array}{c} 2^{n}-1 \\\\ 2 k \\end{array}\\right)=\\prod_{j=1}^{2 k} \\frac{2^{n}-j}{j}=\\prod_{i=1}^{k} \\frac{2^{n}-(2 i-1)}{2 i-1} \\cdot \\prod_{i=1}^{k} \\frac{2^{n-1}-i}{i} \\equiv(-1)^{k}\\left(\\begin{array}{c} 2^{n-1}-1 \\\\ k \\end{array}\\right) \\quad\\left(\\bmod 2^{n}\\right) . $$ This prepares ground for a proof of the required result by induction on $n$. The base case $n=1$ is obvious. Assume the assertion is true for $n-1$ and pass to $n$, denoting $a_{k}=\\left(\\begin{array}{c}2^{n-1}-1 \\\\ k\\end{array}\\right)$, $b_{m}=\\left(\\begin{array}{c}2^{n}-1 \\\\ m\\end{array}\\right)$. The induction hypothesis is that all the numbers $a_{k}\\left(0 \\leq k<2^{n-2}\\right)$ are distinct $\\left(\\bmod 2^{n-1}\\right)$; the claim is that all the numbers $b_{m}\\left(0 \\leq m<2^{n-1}\\right)$ are distinct $\\left(\\bmod 2^{n}\\right)$. The congruence relations (1) are restated as $$ b_{2 k} \\equiv(-1)^{k} a_{k} \\equiv-b_{2 k+1} \\quad\\left(\\bmod 2^{n}\\right) $$ Shifting the exponent in the first relation of (1) from $n$ to $n-1$ we also have the congruence $a_{2 i+1} \\equiv-a_{2 i}\\left(\\bmod 2^{n-1}\\right)$. We hence conclude: If, for some $j, k<2^{n-2}, a_{k} \\equiv-a_{j}\\left(\\bmod 2^{n-1}\\right)$, then $\\{j, k\\}=\\{2 i, 2 i+1\\}$ for some $i$. This is so because in the sequence $\\left(a_{k}: k<2^{n-2}\\right)$ each term $a_{j}$ is complemented to $0\\left(\\bmod 2^{n-1}\\right)$ by only one other term $a_{k}$, according to the induction hypothesis. From (2) we see that $b_{4 i} \\equiv a_{2 i}$ and $b_{4 i+3} \\equiv a_{2 i+1}\\left(\\bmod 2^{n}\\right)$. Let $$ M=\\left\\{m: 0 \\leq m<2^{n-1}, m \\equiv 0 \\text { or } 3(\\bmod 4)\\right\\}, \\quad L=\\left\\{l: 0 \\leq l<2^{n-1}, l \\equiv 1 \\text { or } 2(\\bmod 4)\\right\\} $$ The last two congruences take on the unified form $$ b_{m} \\equiv a_{\\lfloor m \/ 2\\rfloor} \\quad\\left(\\bmod 2^{n}\\right) \\quad \\text { for all } \\quad m \\in M $$ Thus all the numbers $b_{m}$ for $m \\in M$ are distinct $\\left(\\bmod 2^{n}\\right)$ because so are the numbers $a_{k}$ (they are distinct $\\left(\\bmod 2^{n-1}\\right)$, hence also $\\left(\\bmod 2^{n}\\right)$ ). Every $l \\in L$ is paired with a unique $m \\in M$ into a pair of the form $\\{2 k, 2 k+1\\}$. So (2) implies that also all the $b_{l}$ for $l \\in L$ are distinct $\\left(\\bmod 2^{n}\\right)$. It remains to eliminate the possibility that $b_{m} \\equiv b_{l}\\left(\\bmod 2^{n}\\right)$ for some $m \\in M, l \\in L$. Suppose that such a situation occurs. Let $m^{\\prime} \\in M$ be such that $\\left\\{m^{\\prime}, l\\right\\}$ is a pair of the form $\\{2 k, 2 k+1\\}$, so that $($ see $(2)) b_{m^{\\prime}} \\equiv-b_{l}\\left(\\bmod 2^{n}\\right)$. Hence $b_{m^{\\prime}} \\equiv-b_{m}\\left(\\bmod 2^{n}\\right)$. Since both $m^{\\prime}$ and $m$ are in $M$, we have by (4) $b_{m^{\\prime}} \\equiv a_{j}, b_{m} \\equiv a_{k}\\left(\\bmod 2^{n}\\right)$ for $j=\\left\\lfloor m^{\\prime} \/ 2\\right\\rfloor, k=\\lfloor m \/ 2\\rfloor$. Then $a_{j} \\equiv-a_{k}\\left(\\bmod 2^{n}\\right)$. Thus, according to $(3), j=2 i, k=2 i+1$ for some $i$ (or vice versa). The equality $a_{2 i+1} \\equiv-a_{2 i}\\left(\\bmod 2^{n}\\right)$ now means that $\\left(\\begin{array}{c}2^{n-1}-1 \\\\ 2 i\\end{array}\\right)+\\left(\\begin{array}{c}2^{n-1}-1 \\\\ 2 i+1\\end{array}\\right) \\equiv 0\\left(\\bmod 2^{n}\\right)$. However, the sum on the left is equal to $\\left(\\begin{array}{c}2^{n-1} \\\\ 2 i+1\\end{array}\\right)$. A number of this form cannot be divisible by $2^{n}$. This is a contradiction which concludes the induction step and proves the result.","tier":0} +{"problem_type":"Number Theory","problem_label":"N4","problem":"Let $n$ be a positive integer. Show that the numbers $$ \\left(\\begin{array}{c} 2^{n}-1 \\\\ 0 \\end{array}\\right), \\quad\\left(\\begin{array}{c} 2^{n}-1 \\\\ 1 \\end{array}\\right), \\quad\\left(\\begin{array}{c} 2^{n}-1 \\\\ 2 \\end{array}\\right), \\quad \\ldots, \\quad\\left(\\begin{array}{c} 2^{n}-1 \\\\ 2^{n-1}-1 \\end{array}\\right) $$ are congruent modulo $2^{n}$ to $1,3,5, \\ldots, 2^{n}-1$ in some order.","solution":"We again proceed by induction, writing for brevity $N=2^{n-1}$ and keeping notation $a_{k}=\\left(\\begin{array}{c}N-1 \\\\ k\\end{array}\\right), b_{m}=\\left(\\begin{array}{c}2 N-1 \\\\ m\\end{array}\\right)$. Assume that the result holds for the sequence $\\left(a_{0}, a_{1}, a_{2}, \\ldots, a_{N \/ 2-1}\\right)$. In view of the symmetry $a_{N-1-k}=a_{k}$ this sequence is a permutation of $\\left(a_{0}, a_{2}, a_{4}, \\ldots, a_{N-2}\\right)$. So the induction hypothesis says that this latter sequence, taken $(\\bmod N)$, is a permutation of $(1,3,5, \\ldots, N-1)$. Similarly, the induction claim is that $\\left(b_{0}, b_{2}, b_{4}, \\ldots, b_{2 N-2}\\right)$, taken $(\\bmod 2 N)$, is a permutation of $(1,3,5, \\ldots, 2 N-1)$. In place of the congruence relations (2) we now use the following ones, $$ b_{4 i} \\equiv a_{2 i} \\quad(\\bmod N) \\quad \\text { and } \\quad b_{4 i+2} \\equiv b_{4 i}+N \\quad(\\bmod 2 N) . $$ Given this, the conclusion is immediate: the first formula of (5) together with the induction hypothesis tells us that $\\left(b_{0}, b_{4}, b_{8}, \\ldots, b_{2 N-4}\\right)(\\bmod N)$ is a permutation of $(1,3,5, \\ldots, N-1)$. Then the second formula of $(5)$ shows that $\\left(b_{2}, b_{6}, b_{10}, \\ldots, b_{2 N-2}\\right)(\\bmod N)$ is exactly the same permutation; moreover, this formula distinguishes $(\\bmod 2 N)$ each $b_{4 i}$ from $b_{4 i+2}$. Consequently, these two sequences combined represent $(\\bmod 2 N)$ a permutation of the sequence $(1,3,5, \\ldots, N-1, N+1, N+3, N+5, \\ldots, N+N-1)$, and this is precisely the induction claim. Now we prove formulas (5); we begin with the second one. Since $b_{m+1}=b_{m} \\cdot \\frac{2 N-m-1}{m+1}$, $$ b_{4 i+2}=b_{4 i} \\cdot \\frac{2 N-4 i-1}{4 i+1} \\cdot \\frac{2 N-4 i-2}{4 i+2}=b_{4 i} \\cdot \\frac{2 N-4 i-1}{4 i+1} \\cdot \\frac{N-2 i-1}{2 i+1} . $$ The desired congruence $b_{4 i+2} \\equiv b_{4 i}+N$ may be multiplied by the odd number $(4 i+1)(2 i+1)$, giving rise to a chain of successively equivalent congruences: $$ \\begin{aligned} b_{4 i}(2 N-4 i-1)(N-2 i-1) & \\equiv\\left(b_{4 i}+N\\right)(4 i+1)(2 i+1) & & (\\bmod 2 N), \\\\ b_{4 i}(2 i+1-N) & \\equiv\\left(b_{4 i}+N\\right)(2 i+1) & & (\\bmod 2 N), \\\\ \\left(b_{4 i}+2 i+1\\right) N & \\equiv 0 & & (\\bmod 2 N) ; \\end{aligned} $$ and the last one is satisfied, as $b_{4 i}$ is odd. This settles the second relation in (5). The first one is proved by induction on $i$. It holds for $i=0$. Assume $b_{4 i} \\equiv a_{2 i}(\\bmod 2 N)$ and consider $i+1$ : $$ b_{4 i+4}=b_{4 i+2} \\cdot \\frac{2 N-4 i-3}{4 i+3} \\cdot \\frac{2 N-4 i-4}{4 i+4} ; \\quad a_{2 i+2}=a_{2 i} \\cdot \\frac{N-2 i-1}{2 i+1} \\cdot \\frac{N-2 i-2}{2 i+2} . $$ Both expressions have the fraction $\\frac{N-2 i-2}{2 i+2}$ as the last factor. Since $2 i+21$. Direct verification shows that this function meets the requirements. Conversely, let $f: \\mathbb{N} \\rightarrow \\mathbb{N}$ satisfy (i) and (ii). Applying (i) for $x=1$ gives $d(f(1))=1$, so $f(1)=1$. In the sequel we prove that (1) holds for all $n>1$. Notice that $f(m)=f(n)$ implies $m=n$ in view of (i). The formula $d\\left(p_{1}^{b_{1}} \\cdots p_{k}^{b_{k}}\\right)=\\left(b_{1}+1\\right) \\cdots\\left(b_{k}+1\\right)$ will be used throughout. Let $p$ be a prime. Since $d(f(p))=p$, the formula just mentioned yields $f(p)=q^{p-1}$ for some prime $q$; in particular $f(2)=q^{2-1}=q$ is a prime. We prove that $f(p)=p^{p-1}$ for all primes $p$. Suppose that $p$ is odd and $f(p)=q^{p-1}$ for a prime $q$. Applying (ii) first with $x=2$, $y=p$ and then with $x=p, y=2$ shows that $f(2 p)$ divides both $(2-1) p^{2 p-1} f(2)=p^{2 p-1} f(2)$ and $(p-1) 2^{2 p-1} f(p)=(p-1) 2^{2 p-1} q^{p-1}$. If $q \\neq p$ then the odd prime $p$ does not divide $(p-1) 2^{2 p-1} q^{p-1}$, hence the greatest common divisor of $p^{2 p-1} f(2)$ and $(p-1) 2^{2 p-1} q^{p-1}$ is a divisor of $f(2)$. Thus $f(2 p)$ divides $f(2)$ which is a prime. As $f(2 p)>1$, we obtain $f(2 p)=f(2)$ which is impossible. So $q=p$, i. e. $f(p)=p^{p-1}$. For $p=2$ the same argument with $x=2, y=3$ and $x=3, y=2$ shows that $f(6)$ divides both $3^{5} f(2)$ and $2^{6} f(3)=2^{6} 3^{2}$. If the prime $f(2)$ is odd then $f(6)$ divides $3^{2}=9$, so $f(6) \\in\\{1,3,9\\}$. However then $6=d(f(6)) \\in\\{d(1), d(3), d(9)\\}=\\{1,2,3\\}$ which is false. In conclusion $f(2)=2$. Next, for each $n>1$ the prime divisors of $f(n)$ are among the ones of $n$. Indeed, let $p$ be the least prime divisor of $n$. Apply (ii) with $x=p$ and $y=n \/ p$ to obtain that $f(n)$ divides $(p-1) y^{n-1} f(p)=(p-1) y^{n-1} p^{p-1}$. Write $f(n)=\\ell P$ where $\\ell$ is coprime to $n$ and $P$ is a product of primes dividing $n$. Since $\\ell$ divides $(p-1) y^{n-1} p^{p-1}$ and is coprime to $y^{n-1} p^{p-1}$, it divides $p-1$; hence $d(\\ell) \\leq \\ell1$ ). So $f\\left(p^{a}\\right)=p^{b}$ for some $b \\geq 1$, and (i) yields $p^{a}=d\\left(f\\left(p^{a}\\right)\\right)=d\\left(p^{b}\\right)=b+1$. Hence $f\\left(p^{a}\\right)=p^{p^{a}-1}$, as needed. Let us finally show that (1) is true for a general $n>1$ with prime factorization $n=p_{1}^{a_{1}} \\cdots p_{k}^{a_{k}}$. We saw that the prime factorization of $f(n)$ has the form $f(n)=p_{1}^{b_{1}} \\cdots p_{k}^{b_{k}}$. For $i=1, \\ldots, k$, set $x=p_{i}^{a_{i}}$ and $y=n \/ x$ in (ii) to infer that $f(n)$ divides $\\left(p_{i}^{a_{i}}-1\\right) y^{n-1} f\\left(p_{i}^{a_{i}}\\right)$. Hence $p_{i}^{b_{i}}$ divides $\\left(p_{i}^{a_{i}}-1\\right) y^{n-1} f\\left(p_{i}^{a_{i}}\\right)$, and because $p_{i}^{b_{i}}$ is coprime to $\\left(p_{i}^{a_{i}}-1\\right) y^{n-1}$, it follows that $p_{i}^{b_{i}}$ divides $f\\left(p_{i}^{a_{i}}\\right)=p_{i}^{p_{i}^{a_{i}}-1}$. So $b_{i} \\leq p_{i}^{a_{i}}-1$ for all $i=1, \\ldots, k$. Combined with (i), these conclusions imply $$ p_{1}^{a_{1}} \\cdots p_{k}^{a_{k}}=n=d(f(n))=d\\left(p_{1}^{b_{1}} \\cdots p_{k}^{b_{k}}\\right)=\\left(b_{1}+1\\right) \\cdots\\left(b_{k}+1\\right) \\leq p_{1}^{a_{1}} \\cdots p_{k}^{a_{k}} . $$ Hence all inequalities $b_{i} \\leq p_{i}^{a_{i}}-1$ must be equalities, $i=1, \\ldots, k$, implying that (1) holds true. The proof is complete.","tier":0} +{"problem_type":"Number Theory","problem_label":"N6","problem":"Prove that there exist infinitely many positive integers $n$ such that $n^{2}+1$ has a prime divisor greater than $2 n+\\sqrt{2 n}$.","solution":"Let $p \\equiv 1(\\bmod 8)$ be a prime. The congruence $x^{2} \\equiv-1(\\bmod p)$ has two solutions in $[1, p-1]$ whose sum is $p$. If $n$ is the smaller one of them then $p$ divides $n^{2}+1$ and $n \\leq(p-1) \/ 2$. We show that $p>2 n+\\sqrt{10 n}$. Let $n=(p-1) \/ 2-\\ell$ where $\\ell \\geq 0$. Then $n^{2} \\equiv-1(\\bmod p)$ gives $$ \\left(\\frac{p-1}{2}-\\ell\\right)^{2} \\equiv-1 \\quad(\\bmod p) \\quad \\text { or } \\quad(2 \\ell+1)^{2}+4 \\equiv 0 \\quad(\\bmod p) $$ Thus $(2 \\ell+1)^{2}+4=r p$ for some $r \\geq 0$. As $(2 \\ell+1)^{2} \\equiv 1 \\equiv p(\\bmod 8)$, we have $r \\equiv 5(\\bmod 8)$, so that $r \\geq 5$. Hence $(2 \\ell+1)^{2}+4 \\geq 5 p$, implying $\\ell \\geq(\\sqrt{5 p-4}-1) \/ 2$. Set $\\sqrt{5 p-4}=u$ for clarity; then $\\ell \\geq(u-1) \/ 2$. Therefore $$ n=\\frac{p-1}{2}-\\ell \\leq \\frac{1}{2}(p-u) $$ Combined with $p=\\left(u^{2}+4\\right) \/ 5$, this leads to $u^{2}-5 u-10 n+4 \\geq 0$. Solving this quadratic inequality with respect to $u \\geq 0$ gives $u \\geq(5+\\sqrt{40 n+9}) \/ 2$. So the estimate $n \\leq(p-u) \/ 2$ leads to $$ p \\geq 2 n+u \\geq 2 n+\\frac{1}{2}(5+\\sqrt{40 n+9})>2 n+\\sqrt{10 n} $$ Since there are infinitely many primes of the form $8 k+1$, it follows easily that there are also infinitely many $n$ with the stated property. Comment. By considering the prime factorization of the product $\\prod_{n=1}^{N}\\left(n^{2}+1\\right)$, it can be obtained that its greatest prime divisor is at least $c N \\log N$. This could improve the statement as $p>n \\log n$. However, the proof applies some advanced information about the distribution of the primes of the form $4 k+1$, which is inappropriate for high schools contests.","tier":0} diff --git a/IMO/segmented/en-IMO2009SL.jsonl b/IMO/segmented/en-IMO2009SL.jsonl new file mode 100644 index 0000000000000000000000000000000000000000..0f8756b0f10b6fd18052984accf9c4a804077114 --- /dev/null +++ b/IMO/segmented/en-IMO2009SL.jsonl @@ -0,0 +1,60 @@ +{"problem_type":"Algebra","problem_label":"A1","problem":"CZE Find the largest possible integer $k$, such that the following statement is true: Let 2009 arbitrary non-degenerated triangles be given. In every triangle the three sides are colored, such that one is blue, one is red and one is white. Now, for every color separately, let us sort the lengths of the sides. We obtain $$ \\begin{aligned} & b_{1} \\leq b_{2} \\\\ & \\leq \\ldots \\leq b_{2009} \\quad \\text { the lengths of the blue sides, } \\\\ r_{1} & \\leq r_{2} \\leq \\ldots \\leq r_{2009} \\quad \\text { the lengths of the red sides, } \\\\ \\text { and } \\quad w_{1} & \\leq w_{2} \\leq \\ldots \\leq w_{2009} \\quad \\text { the lengths of the white sides. } \\end{aligned} $$ Then there exist $k$ indices $j$ such that we can form a non-degenerated triangle with side lengths $b_{j}, r_{j}, w_{j}$ \u3002","solution":"We will prove that the largest possible number $k$ of indices satisfying the given condition is one. Firstly we prove that $b_{2009}, r_{2009}, w_{2009}$ are always lengths of the sides of a triangle. Without loss of generality we may assume that $w_{2009} \\geq r_{2009} \\geq b_{2009}$. We show that the inequality $b_{2009}+r_{2009}>w_{2009}$ holds. Evidently, there exists a triangle with side lengths $w, b, r$ for the white, blue and red side, respectively, such that $w_{2009}=w$. By the conditions of the problem we have $b+r>w, b_{2009} \\geq b$ and $r_{2009} \\geq r$. From these inequalities it follows $$ b_{2009}+r_{2009} \\geq b+r>w=w_{2009} $$ Secondly we will describe a sequence of triangles for which $w_{j}, b_{j}, r_{j}$ with $j<2009$ are not the lengths of the sides of a triangle. Let us define the sequence $\\Delta_{j}, j=1,2, \\ldots, 2009$, of triangles, where $\\Delta_{j}$ has a blue side of length $2 j$, a red side of length $j$ for all $j \\leq 2008$ and 4018 for $j=2009$, and a white side of length $j+1$ for all $j \\leq 2007$, 4018 for $j=2008$ and 1 for $j=2009$. Since $$ \\begin{array}{rrrl} (j+1)+j>2 j & \\geq j+1>j, & \\text { if } & j \\leq 2007 \\\\ 2 j+j>4018>2 j \\quad>j, & \\text { if } & j=2008 \\\\ 4018+1>2 j & =4018>1, & \\text { if } & j=2009 \\end{array} $$ such a sequence of triangles exists. Moreover, $w_{j}=j, r_{j}=j$ and $b_{j}=2 j$ for $1 \\leq j \\leq 2008$. Then $$ w_{j}+r_{j}=j+j=2 j=b_{j} $$ i.e., $b_{j}, r_{j}$ and $w_{j}$ are not the lengths of the sides of a triangle for $1 \\leq j \\leq 2008$.","tier":0} +{"problem_type":"Algebra","problem_label":"A2","problem":"EST Let $a, b, c$ be positive real numbers such that $\\frac{1}{a}+\\frac{1}{b}+\\frac{1}{c}=a+b+c$. Prove that $$ \\frac{1}{(2 a+b+c)^{2}}+\\frac{1}{(2 b+c+a)^{2}}+\\frac{1}{(2 c+a+b)^{2}} \\leq \\frac{3}{16} $$","solution":"For positive real numbers $x, y, z$, from the arithmetic-geometric-mean inequality, $$ 2 x+y+z=(x+y)+(x+z) \\geq 2 \\sqrt{(x+y)(x+z)} $$ we obtain $$ \\frac{1}{(2 x+y+z)^{2}} \\leq \\frac{1}{4(x+y)(x+z)} $$ Applying this to the left-hand side terms of the inequality to prove, we get $$ \\begin{aligned} \\frac{1}{(2 a+b+c)^{2}} & +\\frac{1}{(2 b+c+a)^{2}}+\\frac{1}{(2 c+a+b)^{2}} \\\\ & \\leq \\frac{1}{4(a+b)(a+c)}+\\frac{1}{4(b+c)(b+a)}+\\frac{1}{4(c+a)(c+b)} \\\\ & =\\frac{(b+c)+(c+a)+(a+b)}{4(a+b)(b+c)(c+a)}=\\frac{a+b+c}{2(a+b)(b+c)(c+a)} . \\end{aligned} $$ A second application of the inequality of the arithmetic-geometric mean yields $$ a^{2} b+a^{2} c+b^{2} a+b^{2} c+c^{2} a+c^{2} b \\geq 6 a b c $$ or, equivalently, $$ 9(a+b)(b+c)(c+a) \\geq 8(a+b+c)(a b+b c+c a) $$ The supposition $\\frac{1}{a}+\\frac{1}{b}+\\frac{1}{c}=a+b+c$ can be written as $$ a b+b c+c a=a b c(a+b+c) . $$ Applying the arithmetic-geometric-mean inequality $x^{2} y^{2}+x^{2} z^{2} \\geq 2 x^{2} y z$ thrice, we get $$ a^{2} b^{2}+b^{2} c^{2}+c^{2} a^{2} \\geq a^{2} b c+a b^{2} c+a b c^{2} $$ which is equivalent to $$ (a b+b c+c a)^{2} \\geq 3 a b c(a+b+c) $$ Combining (1), (2), (3), and (4), we will finish the proof: $$ \\begin{aligned} \\frac{a+b+c}{2(a+b)(b+c)(c+a)} & =\\frac{(a+b+c)(a b+b c+c a)}{2(a+b)(b+c)(c+a)} \\cdot \\frac{a b+b c+c a}{a b c(a+b+c)} \\cdot \\frac{a b c(a+b+c)}{(a b+b c+c a)^{2}} \\\\ & \\leq \\frac{9}{2 \\cdot 8} \\cdot 1 \\cdot \\frac{1}{3}=\\frac{3}{16} \\end{aligned} $$","tier":0} +{"problem_type":"Algebra","problem_label":"A2","problem":"EST Let $a, b, c$ be positive real numbers such that $\\frac{1}{a}+\\frac{1}{b}+\\frac{1}{c}=a+b+c$. Prove that $$ \\frac{1}{(2 a+b+c)^{2}}+\\frac{1}{(2 b+c+a)^{2}}+\\frac{1}{(2 c+a+b)^{2}} \\leq \\frac{3}{16} $$","solution":"Equivalently, we prove the homogenized inequality $$ \\frac{(a+b+c)^{2}}{(2 a+b+c)^{2}}+\\frac{(a+b+c)^{2}}{(a+2 b+c)^{2}}+\\frac{(a+b+c)^{2}}{(a+b+2 c)^{2}} \\leq \\frac{3}{16}(a+b+c)\\left(\\frac{1}{a}+\\frac{1}{b}+\\frac{1}{c}\\right) $$ for all positive real numbers $a, b, c$. Without loss of generality we choose $a+b+c=1$. Thus, the problem is equivalent to prove for all $a, b, c>0$, fulfilling this condition, the inequality $$ \\frac{1}{(1+a)^{2}}+\\frac{1}{(1+b)^{2}}+\\frac{1}{(1+c)^{2}} \\leq \\frac{3}{16}\\left(\\frac{1}{a}+\\frac{1}{b}+\\frac{1}{c}\\right) . $$ Applying Jensen's inequality to the function $f(x)=\\frac{x}{(1+x)^{2}}$, which is concave for $0 \\leq x \\leq 2$ and increasing for $0 \\leq x \\leq 1$, we obtain $$ \\alpha \\frac{a}{(1+a)^{2}}+\\beta \\frac{b}{(1+b)^{2}}+\\gamma \\frac{c}{(1+c)^{2}} \\leq(\\alpha+\\beta+\\gamma) \\frac{A}{(1+A)^{2}}, \\quad \\text { where } \\quad A=\\frac{\\alpha a+\\beta b+\\gamma c}{\\alpha+\\beta+\\gamma} $$ Choosing $\\alpha=\\frac{1}{a}, \\beta=\\frac{1}{b}$, and $\\gamma=\\frac{1}{c}$, we can apply the harmonic-arithmetic-mean inequality $$ A=\\frac{3}{\\frac{1}{a}+\\frac{1}{b}+\\frac{1}{c}} \\leq \\frac{a+b+c}{3}=\\frac{1}{3}<1 $$ Finally we prove (5): $$ \\begin{aligned} \\frac{1}{(1+a)^{2}}+\\frac{1}{(1+b)^{2}}+\\frac{1}{(1+c)^{2}} & \\leq\\left(\\frac{1}{a}+\\frac{1}{b}+\\frac{1}{c}\\right) \\frac{A}{(1+A)^{2}} \\\\ & \\leq\\left(\\frac{1}{a}+\\frac{1}{b}+\\frac{1}{c}\\right) \\frac{\\frac{1}{3}}{\\left(1+\\frac{1}{3}\\right)^{2}}=\\frac{3}{16}\\left(\\frac{1}{a}+\\frac{1}{b}+\\frac{1}{c}\\right) . \\end{aligned} $$","tier":0} +{"problem_type":"Algebra","problem_label":"A3","problem":"FRA Determine all functions $f$ from the set of positive integers into the set of positive integers such that for all $x$ and $y$ there exists a non degenerated triangle with sides of lengths $$ x, \\quad f(y) \\quad \\text { and } \\quad f(y+f(x)-1) $$","solution":"The identity function $f(x)=x$ is the only solution of the problem. If $f(x)=x$ for all positive integers $x$, the given three lengths are $x, y=f(y)$ and $z=$ $f(y+f(x)-1)=x+y-1$. Because of $x \\geq 1, y \\geq 1$ we have $z \\geq \\max \\{x, y\\}>|x-y|$ and $z1$ we would conclude $f(y)=f(y+m)$ for all $y$ considering the triangle with the side lengths $1, f(y)$ and $f(y+m)$. Thus, $f$ would be $m$-periodic and, consequently, bounded. Let $B$ be a bound, $f(x) \\leq B$. If we choose $x>2 B$ we obtain the contradiction $x>2 B \\geq f(y)+f(y+f(x)-1)$. Step 2. For all positive integers $z$, we have $f(f(z))=z$. Setting $x=z$ and $y=1$ this follows immediately from Step 1. Step 3. For all integers $z \\geq 1$, we have $f(z) \\leq z$. Let us show, that the contrary leads to a contradiction. Assume $w+1=f(z)>z$ for some $z$. From Step 1 we know that $w \\geq z \\geq 2$. Let $M=\\max \\{f(1), f(2), \\ldots, f(w)\\}$ be the largest value of $f$ for the first $w$ integers. First we show, that no positive integer $t$ exists with $$ f(t)>\\frac{z-1}{w} \\cdot t+M $$ otherwise we decompose the smallest value $t$ as $t=w r+s$ where $r$ is an integer and $1 \\leq s \\leq w$. Because of the definition of $M$, we have $t>w$. Setting $x=z$ and $y=t-w$ we get from the triangle inequality $$ z+f(t-w)>f((t-w)+f(z)-1)=f(t-w+w)=f(t) $$ Hence, $$ f(t-w) \\geq f(t)-(z-1)>\\frac{z-1}{w}(t-w)+M $$ a contradiction to the minimality of $t$. Therefore the inequality (1) fails for all $t \\geq 1$, we have proven $$ f(t) \\leq \\frac{z-1}{w} \\cdot t+M $$ instead. Now, using (2), we finish the proof of Step 3. Because of $z \\leq w$ we have $\\frac{z-1}{w}<1$ and we can choose an integer $t$ sufficiently large to fulfill the condition $$ \\left(\\frac{z-1}{w}\\right)^{2} t+\\left(\\frac{z-1}{w}+1\\right) My f(x)+x $$","solution":"Assume that $$ f(x-f(y)) \\leq y f(x)+x \\quad \\text { for all real } x, y $$ Let $a=f(0)$. Setting $y=0$ in (1) gives $f(x-a) \\leq x$ for all real $x$ and, equivalently, $$ f(y) \\leq y+a \\quad \\text { for all real } y $$ Setting $x=f(y)$ in (1) yields in view of (2) $$ a=f(0) \\leq y f(f(y))+f(y) \\leq y f(f(y))+y+a . $$ This implies $0 \\leq y(f(f(y))+1)$ and thus $$ f(f(y)) \\geq-1 \\quad \\text { for all } y>0 $$ From (2) and (3) we obtain $-1 \\leq f(f(y)) \\leq f(y)+a$ for all $y>0$, so $$ f(y) \\geq-a-1 \\quad \\text { for all } y>0 $$ Now we show that $$ f(x) \\leq 0 \\quad \\text { for all real } x $$ Assume the contrary, i.e. there is some $x$ such that $f(x)>0$. Take any $y$ such that $$ y0 $$ and with (1) and (4) we obtain $$ y f(x)+x \\geq f(x-f(y)) \\geq-a-1, $$ whence $$ y \\geq \\frac{-a-x-1}{f(x)} $$ contrary to our choice of $y$. Thereby, we have established (5). Setting $x=0$ in (5) leads to $a=f(0) \\leq 0$ and (2) then yields $$ f(x) \\leq x \\quad \\text { for all real } x $$ Now choose $y$ such that $y>0$ and $y>-f(-1)-1$ and set $x=f(y)-1$. From (11), (5) and (6) we obtain $$ f(-1)=f(x-f(y)) \\leq y f(x)+x=y f(f(y)-1)+f(y)-1 \\leq y(f(y)-1)-1 \\leq-y-1, $$ i.e. $y \\leq-f(-1)-1$, a contradiction to the choice of $y$.","tier":0} +{"problem_type":"Algebra","problem_label":"A5","problem":"BLR Let $f$ be any function that maps the set of real numbers into the set of real numbers. Prove that there exist real numbers $x$ and $y$ such that $$ f(x-f(y))>y f(x)+x $$","solution":"Assume that $$ f(x-f(y)) \\leq y f(x)+x \\quad \\text { for all real } x, y $$ Let $a=f(0)$. Setting $y=0$ in (7) gives $f(x-a) \\leq x$ for all real $x$ and, equivalently, $$ f(y) \\leq y+a \\quad \\text { for all real } y $$ Now we show that $$ f(z) \\geq 0 \\quad \\text { for all } z \\geq 1 $$ Let $z \\geq 1$ be fixed, set $b=f(z)$ and assume that $b<0$. Setting $x=w+b$ and $y=z$ in (7) gives $$ f(w)-z f(w+b) \\leq w+b \\quad \\text { for all real } w $$ Applying (10) to $w, w+b, \\ldots, w+(n-1) b$, where $n=1,2, \\ldots$, leads to $$ \\begin{gathered} f(w)-z^{n} f(w+n b)=(f(w)-z f(w+b))+z(f(w+b)-z f(w+2 b)) \\\\ +\\cdots+z^{n-1}(f(w+(n-1) b)-z f(w+n b)) \\\\ \\leq(w+b)+z(w+2 b)+\\cdots+z^{n-1}(w+n b) \\end{gathered} $$ From (8) we obtain $$ f(w+n b) \\leq w+n b+a $$ and, thus, we have for all positive integers $n$ $$ f(w) \\leq\\left(1+z+\\cdots+z^{n-1}+z^{n}\\right) w+\\left(1+2 z+\\cdots+n z^{n-1}+n z^{n}\\right) b+z^{n} a . $$ With $w=0$ we get $$ a \\leq\\left(1+2 z+\\cdots+n z^{n-1}+n z^{n}\\right) b+a z^{n} . $$ In view of the assumption $b<0$ we find some $n$ such that $$ a>(n b+a) z^{n} $$ because the right hand side tends to $-\\infty$ as $n \\rightarrow \\infty$. Now (12) and (13) give the desired contradiction and (9) is established. In addition, we have for $z=1$ the strict inequality $$ f(1)>0 \\text {. } $$ Indeed, assume that $f(1)=0$. Then setting $w=-1$ and $z=1$ in (11) leads to $$ f(-1) \\leq-(n+1)+a $$ which is false if $n$ is sufficiently large. To complete the proof we set $t=\\min \\{-a,-2 \/ f(1)\\}$. Setting $x=1$ and $y=t$ in (7) gives $$ f(1-f(t)) \\leq t f(1)+1 \\leq-2+1=-1 . $$ On the other hand, by (8) and the choice of $t$ we have $f(t) \\leq t+a \\leq 0$ and hence $1-f(t) \\geq 1$. The inequality (9) yields $$ f(1-f(t)) \\geq 0 $$ which contradicts (15).","tier":0} +{"problem_type":"Algebra","problem_label":"A6","problem":"USA Suppose that $s_{1}, s_{2}, s_{3}, \\ldots$ is a strictly increasing sequence of positive integers such that the subsequences $$ s_{s_{1}}, s_{s_{2}}, s_{s_{3}}, \\ldots \\quad \\text { and } \\quad s_{s_{1}+1}, s_{s_{2}+1}, s_{s_{3}+1}, \\ldots $$ are both arithmetic progressions. Prove that $s_{1}, s_{2}, s_{3}, \\ldots$ is itself an arithmetic progression.","solution":"Let $D$ be the common difference of the progression $s_{s_{1}}, s_{s_{2}}, \\ldots$. Let for $n=$ 1, 2, ... $$ d_{n}=s_{n+1}-s_{n} $$ We have to prove that $d_{n}$ is constant. First we show that the numbers $d_{n}$ are bounded. Indeed, by supposition $d_{n} \\geq 1$ for all $n$. Thus, we have for all $n$ $$ d_{n}=s_{n+1}-s_{n} \\leq d_{s_{n}}+d_{s_{n}+1}+\\cdots+d_{s_{n+1}-1}=s_{s_{n+1}}-s_{s_{n}}=D . $$ The boundedness implies that there exist $$ m=\\min \\left\\{d_{n}: n=1,2, \\ldots\\right\\} \\quad \\text { and } \\quad M=\\max \\left\\{d_{n}: n=1,2, \\ldots\\right\\} $$ It suffices to show that $m=M$. Assume that $mn$ if $d_{n}=n$, because in the case $s_{n}=n$ we would have $m=d_{n}=d_{s_{n}}=M$ in contradiction to the assumption $mn$. Consequently, there is a strictly increasing sequence $n_{1}, n_{2}, \\ldots$ such that $$ d_{s_{n_{1}}}=M, \\quad d_{s_{n_{2}}}=m, \\quad d_{s_{n_{3}}}=M, \\quad d_{s_{n_{4}}}=m, \\quad \\ldots $$ The sequence $d_{s_{1}}, d_{s_{2}}, \\ldots$ is the sequence of pairwise differences of $s_{s_{1}+1}, s_{s_{2}+1}, \\ldots$ and $s_{s_{1}}, s_{s_{2}}, \\ldots$, hence also an arithmetic progression. Thus $m=M$.","tier":0} +{"problem_type":"Algebra","problem_label":"A6","problem":"USA Suppose that $s_{1}, s_{2}, s_{3}, \\ldots$ is a strictly increasing sequence of positive integers such that the subsequences $$ s_{s_{1}}, s_{s_{2}}, s_{s_{3}}, \\ldots \\quad \\text { and } \\quad s_{s_{1}+1}, s_{s_{2}+1}, s_{s_{3}+1}, \\ldots $$ are both arithmetic progressions. Prove that $s_{1}, s_{2}, s_{3}, \\ldots$ is itself an arithmetic progression.","solution":"Let the integers $D$ and $E$ be the common differences of the progressions $s_{s_{1}}, s_{s_{2}}, \\ldots$ and $s_{s_{1}+1}, s_{s_{2}+1}, \\ldots$, respectively. Let briefly $A=s_{s_{1}}-D$ and $B=s_{s_{1}+1}-E$. Then, for all positive integers $n$, $$ s_{s_{n}}=A+n D, \\quad s_{s_{n}+1}=B+n E $$ Since the sequence $s_{1}, s_{2}, \\ldots$ is strictly increasing, we have for all positive integers $n$ $$ s_{s_{n}}s_{n}+m$. Then $\\left.m(m+1) \\leq m\\left(s_{n+1}-s_{n}\\right) \\leq s_{s_{n+1}}-s_{s_{n}}=(A+(n+1) D)-(A+n D)\\right)=D=m(B-A)=m^{2}$, a contradiction. Hence $s_{1}, s_{2}, \\ldots$ is an arithmetic progression with common difference $m$.","tier":0} +{"problem_type":"Algebra","problem_label":"A7","problem":"JPN Find all functions $f$ from the set of real numbers into the set of real numbers which satisfy for all real $x, y$ the identity $$ f(x f(x+y))=f(y f(x))+x^{2} $$","solution":"It is no hard to see that the two functions given by $f(x)=x$ and $f(x)=-x$ for all real $x$ respectively solve the functional equation. In the sequel, we prove that there are no further solutions. Let $f$ be a function satisfying the given equation. It is clear that $f$ cannot be a constant. Let us first show that $f(0)=0$. Suppose that $f(0) \\neq 0$. For any real $t$, substituting $(x, y)=\\left(0, \\frac{t}{f(0)}\\right)$ into the given functional equation, we obtain $$ f(0)=f(t) $$ contradicting the fact that $f$ is not a constant function. Therefore, $f(0)=0$. Next for any $t$, substituting $(x, y)=(t, 0)$ and $(x, y)=(t,-t)$ into the given equation, we get $$ f(t f(t))=f(0)+t^{2}=t^{2} $$ and $$ f(t f(0))=f(-t f(t))+t^{2} $$ respectively. Therefore, we conclude that $$ f(t f(t))=t^{2}, \\quad f(-t f(t))=-t^{2}, \\quad \\text { for every real } t $$ Consequently, for every real $v$, there exists a real $u$, such that $f(u)=v$. We also see that if $f(t)=0$, then $0=f(t f(t))=t^{2}$ so that $t=0$, and thus 0 is the only real number satisfying $f(t)=0$. We next show that for any real number $s$, $$ f(-s)=-f(s) $$ This is clear if $f(s)=0$. Suppose now $f(s)<0$, then we can find a number $t$ for which $f(s)=-t^{2}$. As $t \\neq 0$ implies $f(t) \\neq 0$, we can also find number $a$ such that $a f(t)=s$. Substituting $(x, y)=(t, a)$ into the given equation, we get $$ f(t f(t+a))=f(a f(t))+t^{2}=f(s)+t^{2}=0 $$ and therefore, $t f(t+a)=0$, which implies $t+a=0$, and hence $s=-t f(t)$. Consequently, $f(-s)=f(t f(t))=t^{2}=-\\left(-t^{2}\\right)=-f(s)$ holds in this case. Finally, suppose $f(s)>0$ holds. Then there exists a real number $t \\neq 0$ for which $f(s)=t^{2}$. Choose a number $a$ such that $t f(a)=s$. Substituting $(x, y)=(t, a-t)$ into the given equation, we get $f(s)=f(t f(a))=f((a-t) f(t))+t^{2}=f((a-t) f(t))+f(s)$. So we have $f((a-t) f(t))=0$, from which we conclude that $(a-t) f(t)=0$. Since $f(t) \\neq 0$, we get $a=t$ so that $s=t f(t)$ and thus we see $f(-s)=f(-t f(t))=-t^{2}=-f(s)$ holds in this case also. This observation finishes the proof of (3). By substituting $(x, y)=(s, t),(x, y)=(t,-s-t)$ and $(x, y)=(-s-t, s)$ into the given equation, we obtain $$ \\begin{array}{r} f(s f(s+t)))=f(t f(s))+s^{2} \\\\ f(t f(-s))=f((-s-t) f(t))+t^{2} \\end{array} $$ and $$ f((-s-t) f(-t))=f(s f(-s-t))+(s+t)^{2} $$ respectively. Using the fact that $f(-x)=-f(x)$ holds for all $x$ to rewrite the second and the third equation, and rearranging the terms, we obtain $$ \\begin{aligned} f(t f(s))-f(s f(s+t)) & =-s^{2} \\\\ f(t f(s))-f((s+t) f(t)) & =-t^{2} \\\\ f((s+t) f(t))+f(s f(s+t)) & =(s+t)^{2} \\end{aligned} $$ Adding up these three equations now yields $2 f(t f(s))=2 t s$, and therefore, we conclude that $f(t f(s))=t s$ holds for every pair of real numbers $s, t$. By fixing $s$ so that $f(s)=1$, we obtain $f(x)=s x$. In view of the given equation, we see that $s= \\pm 1$. It is easy to check that both functions $f(x)=x$ and $f(x)=-x$ satisfy the given functional equation, so these are the desired solutions.","tier":0} +{"problem_type":"Algebra","problem_label":"A7","problem":"JPN Find all functions $f$ from the set of real numbers into the set of real numbers which satisfy for all real $x, y$ the identity $$ f(x f(x+y))=f(y f(x))+x^{2} $$","solution":"As in Now we prove that $f$ is injective. For this purpose, let us assume that $f(r)=f(s)$ for some $r \\neq s$. Then, by (2) $$ r^{2}=f(r f(r))=f(r f(s))=f((s-r) f(r))+r^{2} $$ where the last statement follows from the given functional equation with $x=r$ and $y=s-r$. Hence, $h=(s-r) f(r)$ satisfies $f(h)=0$ which implies $h^{2}=f(h f(h))=f(0)=0$, i.e., $h=0$. Then, by $s \\neq r$ we have $f(r)=0$ which implies $r=0$, and finally $f(s)=f(r)=f(0)=0$. Analogously, it follows that $s=0$ which gives the contradiction $r=s$. To prove $|f(1)|=1$ we apply (2) with $t=1$ and also with $t=f(1)$ and obtain $f(f(1))=1$ and $(f(1))^{2}=f(f(1) \\cdot f(f(1)))=f(f(1))=1$. Now we choose $\\eta \\in\\{-1,1\\}$ with $f(1)=\\eta$. Using that $f$ is odd and the given equation with $x=1, y=z$ (second equality) and with $x=-1, y=z+2$ (fourth equality) we obtain $$ \\begin{aligned} & f(z)+2 \\eta=\\eta(f(z \\eta)+2)=\\eta(f(f(z+1))+1)=\\eta(-f(-f(z+1))+1) \\\\ & =-\\eta f((z+2) f(-1))=-\\eta f((z+2)(-\\eta))=\\eta f((z+2) \\eta)=f(z+2) \\end{aligned} $$ Hence, $$ f(z+2 \\eta)=\\eta f(\\eta z+2)=\\eta(f(\\eta z)+2 \\eta)=f(z)+2 $$ Using this argument twice we obtain $$ f(z+4 \\eta)=f(z+2 \\eta)+2=f(z)+4 $$ Substituting $z=2 f(x)$ we have $$ f(2 f(x))+4=f(2 f(x)+4 \\eta)=f(2 f(x+2)) $$ where the last equality follows from (4). Applying the given functional equation we proceed to $$ f(2 f(x+2))=f(x f(2))+4=f(2 \\eta x)+4 $$ where the last equality follows again from (4) with $z=0$, i.e., $f(2)=2 \\eta$. Finally, $f(2 f(x))=$ $f(2 \\eta x)$ and by injectivity of $f$ we get $2 f(x)=2 \\eta x$ and hence the two solutions.","tier":0} +{"problem_type":"Combinatorics","problem_label":"C1","problem":"NZL Consider 2009 cards, each having one gold side and one black side, lying in parallel on a long table. Initially all cards show their gold sides. Two players, standing by the same long side of the table, play a game with alternating moves. Each move consists of choosing a block of 50 consecutive cards, the leftmost of which is showing gold, and turning them all over, so those which showed gold now show black and vice versa. The last player who can make a legal move wins. (a) Does the game necessarily end? (b) Does there exist a winning strategy for the starting player?","solution":"(a) We interpret a card showing black as the digit 0 and a card showing gold as the digit 1. Thus each position of the 2009 cards, read from left to right, corresponds bijectively to a nonnegative integer written in binary notation of 2009 digits, where leading zeros are allowed. Each move decreases this integer, so the game must end. (b) We show that there is no winning strategy for the starting player. We label the cards from right to left by $1, \\ldots, 2009$ and consider the set $S$ of cards with labels $50 i, i=1,2, \\ldots, 40$. Let $g_{n}$ be the number of cards from $S$ showing gold after $n$ moves. Obviously, $g_{0}=40$. Moreover, $\\left|g_{n}-g_{n+1}\\right|=1$ as long as the play goes on. Thus, after an odd number of moves, the nonstarting player finds a card from $S$ showing gold and hence can make a move. Consequently, this player always wins.","tier":0} +{"problem_type":"Combinatorics","problem_label":"C2","problem":"ROU For any integer $n \\geq 2$, let $N(n)$ be the maximal number of triples $\\left(a_{i}, b_{i}, c_{i}\\right), i=1, \\ldots, N(n)$, consisting of nonnegative integers $a_{i}, b_{i}$ and $c_{i}$ such that the following two conditions are satisfied: (1) $a_{i}+b_{i}+c_{i}=n$ for all $i=1, \\ldots, N(n)$, (2) If $i \\neq j$, then $a_{i} \\neq a_{j}, b_{i} \\neq b_{j}$ and $c_{i} \\neq c_{j}$. Determine $N(n)$ for all $n \\geq 2$. Comment. The original problem was formulated for $m$-tuples instead for triples. The numbers $N(m, n)$ are then defined similarly to $N(n)$ in the case $m=3$. The numbers $N(3, n)$ and $N(n, n)$ should be determined. The case $m=3$ is the same as in the present problem. The upper bound for $N(n, n)$ can be proved by a simple generalization. The construction of a set of triples attaining the bound can be easily done by induction from $n$ to $n+2$.","solution":"Let $n \\geq 2$ be an integer and let $\\left\\{T_{1}, \\ldots, T_{N}\\right\\}$ be any set of triples of nonnegative integers satisfying the conditions (1) and (2). Since the $a$-coordinates are pairwise distinct we have $$ \\sum_{i=1}^{N} a_{i} \\geq \\sum_{i=1}^{N}(i-1)=\\frac{N(N-1)}{2} $$ Analogously, $$ \\sum_{i=1}^{N} b_{i} \\geq \\frac{N(N-1)}{2} \\quad \\text { and } \\quad \\sum_{i=1}^{N} c_{i} \\geq \\frac{N(N-1)}{2} $$ Summing these three inequalities and applying (1) yields $$ 3 \\frac{N(N-1)}{2} \\leq \\sum_{i=1}^{N} a_{i}+\\sum_{i=1}^{N} b_{i}+\\sum_{i=1}^{N} c_{i}=\\sum_{i=1}^{N}\\left(a_{i}+b_{i}+c_{i}\\right)=n N, $$ hence $3 \\frac{N-1}{2} \\leq n$ and, consequently, $$ N \\leq\\left\\lfloor\\frac{2 n}{3}\\right\\rfloor+1 $$ By constructing examples, we show that this upper bound can be attained, so $N(n)=\\left\\lfloor\\frac{2 n}{3}\\right\\rfloor+1$. We distinguish the cases $n=3 k-1, n=3 k$ and $n=3 k+1$ for $k \\geq 1$ and present the extremal examples in form of a table. | $n=3 k-1$ | | | | :---: | :---: | :---: | | $\\left\\lfloor\\frac{2 n}{3}\\right\\rfloor+1=2 k$ | | | | $a_{i}$ | $b_{i}$ | $c_{i}$ | | 0 | $k+1$ | $2 k-2$ | | 1 | $k+2$ | $2 k-4$ | | $\\vdots$ | $\\vdots$ | $\\vdots$ | | $k-1$ | $2 k$ | 0 | | $k$ | 0 | $2 k-1$ | | $k+1$ | 1 | $2 k-3$ | | $\\vdots$ | $\\vdots$ | $\\vdots$ | | $2 k-1$ | $k-1$ | 1 | | $n=3 k$ | | | | :---: | :---: | :---: | | $\\rfloor+1=2 k+1$ | | | | $a_{i}$ | $b_{i}$ | $c_{i}$ | | 0 | $k$ | $2 k$ | | 1 | $k+1$ | $2 k-2$ | | $\\vdots$ | $\\vdots$ | $\\vdots$ | | $k$ | $2 k$ | 0 | | $k+1$ | 0 | $2 k-1$ | | $k+2$ | 1 | $2 k-3$ | | $\\vdots$ | $\\vdots$ | $\\vdots$ | | $2 k$ | $k-1$ | 1 | | $n=3 k+1$ | | | | :---: | :---: | :---: | | $\\left\\lfloor\\left\\lfloor\\frac{2 n}{3}\\right\\rfloor+1=2 k+1\\right.$ | | | | $a_{i}$ | $b_{i}$ | $c_{i}$ | | 0 | $k$ | $2 k+1$ | | 1 | $k+1$ | $2 k-1$ | | $\\vdots$ | $\\vdots$ | $\\vdots$ | | $k$ | $2 k$ | 1 | | $k+1$ | 0 | $2 k$ | | $k+2$ | 1 | $2 k-2$ | | $\\vdots$ | $\\vdots$ | $\\vdots$ | | $2 k$ | $k-1$ | 2 | It can be easily seen that the conditions (1) and (2) are satisfied and that we indeed have $\\left\\lfloor\\frac{2 n}{3}\\right\\rfloor+1$ triples in each case. Comment. A cute combinatorial model is given by an equilateral triangle, partitioned into $n^{2}$ congruent equilateral triangles by $n-1$ equidistant parallels to each of its three sides. Two chess-like bishops placed at any two vertices of the small triangles are said to menace one another if they lie on a same parallel. The problem is to determine the largest number of bishops that can be placed so that none menaces another. A bishop may be assigned three coordinates $a, b, c$, namely the numbers of sides of small triangles they are off each of the sides of the big triangle. It is readily seen that the sum of these coordinates is always $n$, therefore fulfilling the requirements.","tier":0} +{"problem_type":"Combinatorics","problem_label":"C3","problem":"RUS Let $n$ be a positive integer. Given a sequence $\\varepsilon_{1}, \\ldots, \\varepsilon_{n-1}$ with $\\varepsilon_{i}=0$ or $\\varepsilon_{i}=1$ for each $i=1, \\ldots, n-1$, the sequences $a_{0}, \\ldots, a_{n}$ and $b_{0}, \\ldots, b_{n}$ are constructed by the following rules: $$ \\begin{gathered} a_{0}=b_{0}=1, \\quad a_{1}=b_{1}=7, \\\\ a_{i+1}=\\left\\{\\begin{array}{ll} 2 a_{i-1}+3 a_{i}, & \\text { if } \\varepsilon_{i}=0, \\\\ 3 a_{i-1}+a_{i}, & \\text { if } \\varepsilon_{i}=1, \\end{array} \\text { for each } i=1, \\ldots, n-1,\\right. \\\\ b_{i+1}=\\left\\{\\begin{array}{ll} 2 b_{i-1}+3 b_{i}, & \\text { if } \\varepsilon_{n-i}=0, \\\\ 3 b_{i-1}+b_{i}, & \\text { if } \\varepsilon_{n-i}=1, \\end{array} \\text { for each } i=1, \\ldots, n-1 .\\right. \\end{gathered} $$ Prove that $a_{n}=b_{n}$.","solution":"For a binary word $w=\\sigma_{1} \\ldots \\sigma_{n}$ of length $n$ and a letter $\\sigma \\in\\{0,1\\}$ let $w \\sigma=$ $\\sigma_{1} \\ldots \\sigma_{n} \\sigma$ and $\\sigma w=\\sigma \\sigma_{1} \\ldots \\sigma_{n}$. Moreover let $\\bar{w}=\\sigma_{n} \\ldots \\sigma_{1}$ and let $\\emptyset$ be the empty word (of length 0 and with $\\bar{\\emptyset}=\\emptyset$ ). Let $(u, v)$ be a pair of two real numbers. For binary words $w$ we define recursively the numbers $(u, v)^{w}$ as follows: $$ \\begin{gathered} (u, v)^{\\emptyset}=v, \\quad(u, v)^{0}=2 u+3 v, \\quad(u, v)^{1}=3 u+v, \\\\ (u, v)^{w \\sigma \\varepsilon}= \\begin{cases}3(u, v)^{w}+3(u, v)^{w \\sigma}, & \\text { if } \\varepsilon=0, \\\\ 3(u, v)^{w}+(u, v)^{w \\sigma}, & \\text { if } \\varepsilon=1\\end{cases} \\end{gathered} $$ It easily follows by induction on the length of $w$ that for all real numbers $u_{1}, v_{1}, u_{2}, v_{2}, \\lambda_{1}$ and $\\lambda_{2}$ $$ \\left(\\lambda_{1} u_{1}+\\lambda_{2} u_{2}, \\lambda_{1} v_{1}+\\lambda_{2} v_{2}\\right)^{w}=\\lambda_{1}\\left(u_{1}, v_{1}\\right)^{w}+\\lambda_{2}\\left(u_{2}, v_{2}\\right)^{w} $$ and that for $\\varepsilon \\in\\{0,1\\}$ $$ (u, v)^{\\varepsilon w}=\\left(v,(u, v)^{\\varepsilon}\\right)^{w} . $$ Obviously, for $n \\geq 1$ and $w=\\varepsilon_{1} \\ldots \\varepsilon_{n-1}$, we have $a_{n}=(1,7)^{w}$ and $b_{n}=(1,7)^{\\bar{w}}$. Thus it is sufficient to prove that $$ (1,7)^{w}=(1,7)^{\\bar{w}} $$ for each binary word $w$. We proceed by induction on the length of $w$. The assertion is obvious if $w$ has length 0 or 1 . Now let $w \\sigma \\varepsilon$ be a binary word of length $n \\geq 2$ and suppose that the assertion is true for all binary words of length at most $n-1$. Note that $(2,1)^{\\sigma}=7=(1,7)^{\\emptyset}$ for $\\sigma \\in\\{0,1\\},(1,7)^{0}=23$, and $(1,7)^{1}=10$. First let $\\varepsilon=0$. Then in view of the induction hypothesis and the equalities (1) and (2), we obtain $$ \\begin{aligned} &(1,7)^{w \\sigma 0}=2(1,7)^{w}+3(1,7)^{w \\sigma}=2(1,7)^{\\bar{w}}+3(1,7)^{\\sigma \\bar{w}}=2(2,1)^{\\sigma \\bar{w}}+3(1,7)^{\\sigma \\bar{w}} \\\\ &=(7,23)^{\\sigma \\bar{w}}=(1,7)^{0 \\sigma \\bar{w}} \\end{aligned} $$ Now let $\\varepsilon=1$. Analogously, we obtain $$ \\begin{aligned} &(1,7)^{w \\sigma 1}=3(1,7)^{w}+(1,7)^{w \\sigma}=3(1,7)^{\\bar{w}}+(1,7)^{\\sigma \\bar{w}}=3(2,1)^{\\sigma \\bar{w}}+(1,7)^{\\sigma \\bar{w}} \\\\ &=(7,10)^{\\sigma \\bar{w}}=(1,7)^{1 \\sigma \\bar{w}} \\end{aligned} $$ Thus the induction step is complete, (3) and hence also $a_{n}=b_{n}$ are proved. Comment. The original solution uses the relation $$ (1,7)^{\\alpha \\beta w}=\\left((1,7)^{w},(1,7)^{\\beta w}\\right)^{\\alpha}, \\quad \\alpha, \\beta \\in\\{0,1\\} $$ which can be proved by induction on the length of $w$. Then (3) also follows by induction on the length of $w$ : $$ (1,7)^{\\alpha \\beta w}=\\left((1,7)^{w},(1,7)^{\\beta w}\\right)^{\\alpha}=\\left((1,7)^{\\bar{w}},(1,7)^{\\bar{w} \\beta}\\right)^{\\alpha}=(1,7)^{\\bar{w} \\beta \\alpha} . $$ Here $w$ may be the empty word.","tier":0} +{"problem_type":"Combinatorics","problem_label":"C4","problem":"NLD For an integer $m \\geq 1$, we consider partitions of a $2^{m} \\times 2^{m}$ chessboard into rectangles consisting of cells of the chessboard, in which each of the $2^{m}$ cells along one diagonal forms a separate rectangle of side length 1 . Determine the smallest possible sum of rectangle perimeters in such a partition.","solution":"For a $k \\times k$ chessboard, we introduce in a standard way coordinates of the vertices of the cells and assume that the cell $C_{i j}$ in row $i$ and column $j$ has vertices $(i-1, j-1),(i-$ $1, j),(i, j-1),(i, j)$, where $i, j \\in\\{1, \\ldots, k\\}$. Without loss of generality assume that the cells $C_{i i}$, $i=1, \\ldots, k$, form a separate rectangle. Then we may consider the boards $B_{k}=\\bigcup_{1 \\leq i0$, JEnSEN's inequality immediately shows that the minimum of the right hand sight of (1) is attained for $i=k \/ 2$. Hence the total perimeter of the optimal partition of $B_{k}$ is at least $2 k+2 k \/ 2 \\log _{2} k \/ 2+2(k \/ 2) \\log _{2}(k \/ 2)=D_{k}$.","tier":0} +{"problem_type":"Combinatorics","problem_label":"C4","problem":"NLD For an integer $m \\geq 1$, we consider partitions of a $2^{m} \\times 2^{m}$ chessboard into rectangles consisting of cells of the chessboard, in which each of the $2^{m}$ cells along one diagonal forms a separate rectangle of side length 1 . Determine the smallest possible sum of rectangle perimeters in such a partition.","solution":"We start as in $$ p=2\\left(\\sum_{i=1}^{M} r_{i}+\\sum_{i=1}^{M} c_{i}\\right) . $$ No rectangle can simultaneously cover cells from row $i$ and from column $i$ since otherwise it would also cover the cell $C_{i i}$. We classify subsets $S$ of rectangles of the partition as follows. We say that $S$ is of type $i, 1 \\leq i \\leq M$, if $S$ contains all $r_{i}$ rectangles that cover some cell from row $i$, but none of the $c_{i}$ rectangles that cover some cell from column $i$. Altogether there are $2^{n-r_{i}-c_{i}}$ subsets of type $i$. Now we show that no subset $S$ can be simultaneously of type $i$ and of type $j$ if $i \\neq j$. Assume the contrary and let without loss of generality $i1$. By condition $\\left(2^{\\prime}\\right)$, after the Stepmother has distributed her water we have $y_{0}+y_{1}+y_{2}+y_{3}+y_{4} \\leq \\frac{5}{2}$. Therefore, $$ \\left(y_{0}+y_{2}\\right)+\\left(y_{1}+y_{3}\\right)+\\left(y_{2}+y_{4}\\right)+\\left(y_{3}+y_{0}\\right)+\\left(y_{4}+y_{1}\\right)=2\\left(y_{0}+y_{1}+y_{2}+y_{3}+y_{4}\\right) \\leq 5 $$ and hence there is a pair of non-neighboring buckets which is not critical, say $\\left(B_{0}, B_{2}\\right)$. Now, if both of the pairs $\\left(B_{3}, B_{0}\\right)$ and $\\left(B_{2}, B_{4}\\right)$ are critical, we must have $y_{1}<\\frac{1}{2}$ and Cinderella can empty the buckets $B_{3}$ and $B_{4}$. This clearly leaves no critical pair of buckets and the total contents of all the buckets is then $y_{1}+\\left(y_{0}+y_{2}\\right) \\leq \\frac{3}{2}$. Therefore, conditions $\\left(1^{\\prime}\\right)$ and $\\left(2^{\\prime}\\right)$ are fulfilled. Now suppose that without loss of generality the pair $\\left(B_{3}, B_{0}\\right)$ is not critical. If in this case $y_{0} \\leq \\frac{1}{2}$, then one of the inequalities $y_{0}+y_{1}+y_{2} \\leq \\frac{3}{2}$ and $y_{0}+y_{3}+y_{4} \\leq \\frac{3}{2}$ must hold. But then Cinderella can empty $B_{3}$ and $B_{4}$ or $B_{1}$ and $B_{2}$, respectively and clearly fulfill the conditions. Finally consider the case $y_{0}>\\frac{1}{2}$. By $y_{0}+y_{1}+y_{2}+y_{3}+y_{4} \\leq \\frac{5}{2}$, at least one of the pairs $\\left(B_{1}, B_{3}\\right)$ and $\\left(B_{2}, B_{4}\\right)$ is not critical. Without loss of generality let this be the pair $\\left(B_{1}, B_{3}\\right)$. Since the pair $\\left(B_{3}, B_{0}\\right)$ is not critical and $y_{0}>\\frac{1}{2}$, we must have $y_{3} \\leq \\frac{1}{2}$. But then, as before, Cinderella can maintain the two conditions at the beginning of the next round by either emptying $B_{1}$ and $B_{2}$ or $B_{4}$ and $B_{0}$. Comments on GREEDY approaches. A natural approach for Cinderella would be a GREEDY strategy as for example: Always remove as much water as possible from the system. It is straightforward to prove that GREEDY can avoid buckets of capacity $\\frac{5}{2}$ from overflowing: If before the Stepmothers move one has $x_{0}+x_{1}+x_{2}+x_{3}+x_{4} \\leq \\frac{3}{2}$ then after her move the inequality $Y=y_{0}+y_{1}+y_{2}+y_{3}+y_{4} \\leq \\frac{5}{2}$ holds. If now Cinderella removes the two adjacent buckets with maximum total contents she removes at least $\\frac{2 Y}{5}$ and thus the remaining buckets contain at most $\\frac{3}{5} \\cdot Y \\leq \\frac{3}{2}$. But GREEDY is in general not strong enough to settle this problem as can be seen in the following example: - In an initial phase, the Stepmother brings all the buckets (after her move) to contents of at least $\\frac{1}{2}-2 \\epsilon$, where $\\epsilon$ is an arbitrary small positive number. This can be done by always splitting the 1 liter she has to distribute so that all buckets have the same contents. After her $r$-th move the total contents of each of the buckets is then $c_{r}$ with $c_{1}=1$ and $c_{r+1}=1+\\frac{3}{5} \\cdot c_{r}$ and hence $c_{r}=\\frac{5}{2}-\\frac{3}{2} \\cdot\\left(\\frac{3}{5}\\right)^{r-1}$. So the contents of each single bucket indeed approaches $\\frac{1}{2}$ (from below). In particular, any two adjacent buckets have total contents strictly less than 1 which enables the Stepmother to always refill the buckets that Cinderella just emptied and then distribute the remaining water evenly over all buckets. - After that phase GREEDY faces a situation like this ( $\\left.\\frac{1}{2}-2 \\epsilon, \\frac{1}{2}-2 \\epsilon, \\frac{1}{2}-2 \\epsilon, \\frac{1}{2}-2 \\epsilon, \\frac{1}{2}-2 \\epsilon\\right)$ and leaves a situation of the form $\\left(x_{0}, x_{1}, x_{2}, x_{3}, x_{4}\\right)=\\left(\\frac{1}{2}-2 \\epsilon, \\frac{1}{2}-2 \\epsilon, \\frac{1}{2}-2 \\epsilon, 0,0\\right)$. - Then the Stepmother can add the amounts $\\left(0, \\frac{1}{4}+\\epsilon, \\epsilon, \\frac{3}{4}-2 \\epsilon, 0\\right)$ to achieve a situation like this: $\\left(y_{0}, y_{1}, y_{2}, y_{3}, y_{4}\\right)=\\left(\\frac{1}{2}-2 \\epsilon, \\frac{3}{4}-\\epsilon, \\frac{1}{2}-\\epsilon, \\frac{3}{4}-2 \\epsilon, 0\\right)$. - Now $B_{1}$ and $B_{2}$ are the adjacent buckets with the maximum total contents and thus GREEDY empties them to yield $\\left(x_{0}, x_{1}, x_{2}, x_{3}, x_{4}\\right)=\\left(\\frac{1}{2}-2 \\epsilon, 0,0, \\frac{3}{4}-2 \\epsilon, 0\\right)$. - Then the Stepmother adds $\\left(\\frac{5}{8}, 0,0, \\frac{3}{8}, 0\\right)$, which yields $\\left(\\frac{9}{8}-2 \\epsilon, 0,0, \\frac{9}{8}-2 \\epsilon, 0\\right)$. - Now GREEDY can only empty one of the two nonempty buckets and in the next step the Stepmother adds her liter to the other bucket and brings it to $\\frac{17}{8}-2 \\epsilon$, i.e. an overflow. A harder variant. Five identical empty buckets of capacity $b$ stand at the vertices of a regular pentagon. Cinderella and her wicked Stepmother go through a sequence of rounds: At the beginning of every round, the Stepmother takes one liter of water from the nearby river and distributes it arbitrarily over the five buckets. Then Cinderella chooses a pair of neighboring buckets, empties them into the river, and puts them back. Then the next round begins. The Stepmother's goal is to make one of these buckets overflow. Cinderella's goal is to prevent this. Determine all bucket capacities $b$ for which the Stepmother can enforce a bucket to overflow. Solution to the harder variant. The answer is $b<2$. The previous proof shows that for all $b \\geq 2$ the Stepmother cannot enforce overflowing. Now if $b<2$, let $R$ be a positive integer such that $b<2-2^{1-R}$. In the first $R$ rounds the Stepmother now ensures that at least one of the (nonadjacent) buckets $B_{1}$ and $B_{3}$ have contents of at least $1-2^{1-r}$ at the beginning of round $r(r=1,2, \\ldots, R)$. This is trivial for $r=1$ and if it holds at the beginning of round $r$, she can fill the bucket which contains at least $1-2^{1-r}$ liters with another $2^{-r}$ liters and put the rest of her water $-1-2^{-r}$ liters - in the other bucket. As Cinderella now can remove the water of at most one of the two buckets, the other bucket carries its contents into the next round. At the beginning of the $R$-th round there are $1-2^{1-R}$ liters in $B_{1}$ or $B_{3}$. The Stepmother puts the entire liter into that bucket and produces an overflow since $b<2-2^{1-R}$.","tier":0} +{"problem_type":"Combinatorics","problem_label":"C6","problem":"BGR On a $999 \\times 999$ board a limp rook can move in the following way: From any square it can move to any of its adjacent squares, i.e. a square having a common side with it, and every move must be a turn, i.e. the directions of any two consecutive moves must be perpendicular. A nonintersecting route of the limp rook consists of a sequence of pairwise different squares that the limp rook can visit in that order by an admissible sequence of moves. Such a non-intersecting route is called cyclic, if the limp rook can, after reaching the last square of the route, move directly to the first square of the route and start over. How many squares does the longest possible cyclic, non-intersecting route of a limp rook visit?","solution":"The answer is $998^{2}-4=4 \\cdot\\left(499^{2}-1\\right)$ squares. First we show that this number is an upper bound for the number of cells a limp rook can visit. To do this we color the cells with four colors $A, B, C$ and $D$ in the following way: for $(i, j) \\equiv(0,0) \\bmod 2$ use $A$, for $(i, j) \\equiv(0,1) \\bmod 2$ use $B$, for $(i, j) \\equiv(1,0) \\bmod 2$ use $C$ and for $(i, j) \\equiv(1,1) \\bmod 2$ use $D$. From an $A$-cell the rook has to move to a $B$-cell or a $C$-cell. In the first case, the order of the colors of the cells visited is given by $A, B, D, C, A, B, D, C, A, \\ldots$, in the second case it is $A, C, D, B, A, C, D, B, A, \\ldots$ Since the route is closed it must contain the same number of cells of each color. There are only $499^{2} A$-cells. In the following we will show that the rook cannot visit all the $A$-cells on its route and hence the maximum possible number of cells in a route is $4 \\cdot\\left(499^{2}-1\\right)$. Assume that the route passes through every single $A$-cell. Color the $A$-cells in black and white in a chessboard manner, i.e. color any two $A$-cells at distance 2 in different color. Since the number of $A$-cells is odd the rook cannot always alternate between visiting black and white $A$-cells along its route. Hence there are two $A$-cells of the same color which are four rook-steps apart that are visited directly one after the other. Let these two $A$-cells have row and column numbers $(a, b)$ and $(a+2, b+2)$ respectively. ![](https:\/\/cdn.mathpix.com\/cropped\/2024_11_18_b191c9407e8cb3d672cbg-38.jpg?height=449&width=524&top_left_y=1883&top_left_x=766) There is up to reflection only one way the rook can take from $(a, b)$ to $(a+2, b+2)$. Let this way be $(a, b) \\rightarrow(a, b+1) \\rightarrow(a+1, b+1) \\rightarrow(a+1, b+2) \\rightarrow(a+2, b+2)$. Also let without loss of generality the color of the cell $(a, b+1)$ be $B$ (otherwise change the roles of columns and rows). Now consider the $A$-cell $(a, b+2)$. The only way the rook can pass through it is via $(a-1, b+2) \\rightarrow$ $(a, b+2) \\rightarrow(a, b+3)$ in this order, since according to our assumption after every $A$-cell the rook passes through a $B$-cell. Hence, to connect these two parts of the path, there must be a path connecting the cell $(a, b+3)$ and $(a, b)$ and also a path connecting $(a+2, b+2)$ and $(a-1, b+2)$. But these four cells are opposite vertices of a convex quadrilateral and the paths are outside of that quadrilateral and hence they must intersect. This is due to the following fact: The path from $(a, b)$ to $(a, b+3)$ together with the line segment joining these two cells form a closed loop that has one of the cells $(a-1, b+2)$ and $(a+2, b+2)$ in its inside and the other one on the outside. Thus the path between these two points must cross the previous path. But an intersection is only possible if a cell is visited twice. This is a contradiction. Hence the number of cells visited is at most $4 \\cdot\\left(499^{2}-1\\right)$. The following picture indicates a recursive construction for all $n \\times n$-chessboards with $n \\equiv 3$ $\\bmod 4$ which clearly yields a path that misses exactly one $A$-cell (marked with a dot, the center cell of the $15 \\times 15$-chessboard) and hence, in the case of $n=999$ crosses exactly $4 \\cdot\\left(499^{2}-1\\right)$ cells. ![](https:\/\/cdn.mathpix.com\/cropped\/2024_11_18_b191c9407e8cb3d672cbg-39.jpg?height=686&width=715&top_left_y=979&top_left_x=676)","tier":0} +{"problem_type":"Combinatorics","problem_label":"C7","problem":"RUS Variant 1. A grasshopper jumps along the real axis. He starts at point 0 and makes 2009 jumps to the right with lengths $1,2, \\ldots, 2009$ in an arbitrary order. Let $M$ be a set of 2008 positive integers less than 1005 - 2009. Prove that the grasshopper can arrange his jumps in such a way that he never lands on a point from $M$. Variant 2. Let $n$ be a nonnegative integer. A grasshopper jumps along the real axis. He starts at point 0 and makes $n+1$ jumps to the right with pairwise different positive integral lengths $a_{1}, a_{2}, \\ldots, a_{n+1}$ in an arbitrary order. Let $M$ be a set of $n$ positive integers in the interval $(0, s)$, where $s=a_{1}+a_{2}+\\cdots+a_{n+1}$. Prove that the grasshopper can arrange his jumps in such a way that he never lands on a point from $M$.","solution":null,"tier":0} +{"problem_type":"Combinatorics","problem_label":"C8","problem":"AUT For any integer $n \\geq 2$, we compute the integer $h(n)$ by applying the following procedure to its decimal representation. Let $r$ be the rightmost digit of $n$. (1) If $r=0$, then the decimal representation of $h(n)$ results from the decimal representation of $n$ by removing this rightmost digit 0 . (2) If $1 \\leq r \\leq 9$ we split the decimal representation of $n$ into a maximal right part $R$ that solely consists of digits not less than $r$ and into a left part $L$ that either is empty or ends with a digit strictly smaller than $r$. Then the decimal representation of $h(n)$ consists of the decimal representation of $L$, followed by two copies of the decimal representation of $R-1$. For instance, for the number $n=17,151,345,543$, we will have $L=17,151, R=345,543$ and $h(n)=17,151,345,542,345,542$. Prove that, starting with an arbitrary integer $n \\geq 2$, iterated application of $h$ produces the integer 1 after finitely many steps.","solution":"We identify integers $n \\geq 2$ with the digit-strings, briefly strings, of their decimal representation and extend the definition of $h$ to all non-empty strings with digits from 0 to 9. We recursively define ten functions $f_{0}, \\ldots, f_{9}$ that map some strings into integers for $k=$ $9,8, \\ldots, 1,0$. The function $f_{9}$ is only defined on strings $x$ (including the empty string $\\varepsilon$ ) that entirely consist of nines. If $x$ consists of $m$ nines, then $f_{9}(x)=m+1, m=0,1, \\ldots$ For $k \\leq 8$, the domain of $f_{k}(x)$ is the set of all strings consisting only of digits that are $\\geq k$. We write $x$ in the form $x_{0} k x_{1} k x_{2} k \\ldots x_{m-1} k x_{m}$ where the strings $x_{s}$ only consist of digits $\\geq k+1$. Note that some of these strings might equal the empty string $\\varepsilon$ and that $m=0$ is possible, i.e. the digit $k$ does not appear in $x$. Then we define $$ f_{k}(x)=\\sum_{s=0}^{m} 4^{f_{k+1}\\left(x_{s}\\right)} $$ We will use the following obvious fact: Fact 1. If $x$ does not contain digits smaller than $k$, then $f_{i}(x)=4^{f_{i+1}(x)}$ for all $i=0, \\ldots, k-1$. In particular, $f_{i}(\\varepsilon)=4^{9-i}$ for all $i=0,1, \\ldots, 9$. Moreover, by induction on $k=9,8, \\ldots, 0$ it follows easily: Fact 2. If the nonempty string $x$ does not contain digits smaller than $k$, then $f_{i}(x)>f_{i}(\\varepsilon)$ for all $i=0, \\ldots, k$. We will show the essential fact: Fact 3. $f_{0}(n)>f_{0}(h(n))$. Then the empty string will necessarily be reached after a finite number of applications of $h$. But starting from a string without leading zeros, $\\varepsilon$ can only be reached via the strings $1 \\rightarrow 00 \\rightarrow 0 \\rightarrow \\varepsilon$. Hence also the number 1 will appear after a finite number of applications of $h$. Proof of Fact 3. If the last digit $r$ of $n$ is 0 , then we write $n=x_{0} 0 \\ldots 0 x_{m-1} 0 \\varepsilon$ where the $x_{i}$ do not contain the digit 0 . Then $h(n)=x_{0} 0 \\ldots 0 x_{m-1}$ and $f_{0}(n)-f_{0}(h(n))=f_{0}(\\varepsilon)>0$. So let the last digit $r$ of $n$ be at least 1 . Let $L=y k$ and $R=z r$ be the corresponding left and right parts where $y$ is some string, $k \\leq r-1$ and the string $z$ consists only of digits not less than $r$. Then $n=y k z r$ and $h(n)=y k z(r-1) z(r-1)$. Let $d(y)$ be the smallest digit of $y$. We consider two cases which do not exclude each other. Case 1. $d(y) \\geq k$. Then $$ f_{k}(n)-f_{k}(h(n))=f_{k}(z r)-f_{k}(z(r-1) z(r-1)) . $$ In view of Fact 1 this difference is positive if and only if $$ f_{r-1}(z r)-f_{r-1}(z(r-1) z(r-1))>0 $$ We have, using Fact 2, $$ f_{r-1}(z r)=4^{f_{r}(z r)}=4^{f_{r}(z)+4^{f_{r+1}(z)}} \\geq 4 \\cdot 4^{f_{r}(z)}>4^{f_{r}(z)}+4^{f_{r}(z)}+4^{f_{r}(z)}=f_{r-1}(z(r-1) z(r-1)) . $$ Here we use the additional definition $f_{10}(\\varepsilon)=0$ if $r=9$. Consequently, $f_{k}(n)-f_{k}(h(n))>0$ and according to Fact $1, f_{0}(n)-f_{0}(h(n))>0$. Case 2. $d(y) \\leq k$. We prove by induction on $d(y)=k, k-1, \\ldots, 0$ that $f_{i}(n)-f_{i}(h(n))>0$ for all $i=0, \\ldots, d(y)$. By Fact 1, it suffices to do so for $i=d(y)$. The initialization $d(y)=k$ was already treated in Case 1. Let $t=d(y)0 $$ Thus the inequality $f_{d(y)}(n)-f_{d(y)}(h(n))>0$ is established and from Fact 1 it follows that $f_{0}(n)-f_{0}(h(n))>0$.","tier":0} +{"problem_type":"Combinatorics","problem_label":"C8","problem":"AUT For any integer $n \\geq 2$, we compute the integer $h(n)$ by applying the following procedure to its decimal representation. Let $r$ be the rightmost digit of $n$. (1) If $r=0$, then the decimal representation of $h(n)$ results from the decimal representation of $n$ by removing this rightmost digit 0 . (2) If $1 \\leq r \\leq 9$ we split the decimal representation of $n$ into a maximal right part $R$ that solely consists of digits not less than $r$ and into a left part $L$ that either is empty or ends with a digit strictly smaller than $r$. Then the decimal representation of $h(n)$ consists of the decimal representation of $L$, followed by two copies of the decimal representation of $R-1$. For instance, for the number $n=17,151,345,543$, we will have $L=17,151, R=345,543$ and $h(n)=17,151,345,542,345,542$. Prove that, starting with an arbitrary integer $n \\geq 2$, iterated application of $h$ produces the integer 1 after finitely many steps.","solution":"We identify integers $n \\geq 2$ with the digit-strings, briefly strings, of their decimal representation and extend the definition of $h$ to all non-empty strings with digits from 0 to 9. Moreover, let us define that the empty string, $\\varepsilon$, is being mapped to the empty string. In the following all functions map the set of strings into the set of strings. For two functions $f$ and $g$ let $g \\circ f$ be defined by $(g \\circ f)(x)=g(f(x))$ for all strings $x$ and let, for non-negative integers $n$, $f^{n}$ denote the $n$-fold application of $f$. For any string $x$ let $s(x)$ be the smallest digit of $x$, and for the empty string let $s(\\varepsilon)=\\infty$. We define nine functions $g_{1}, \\ldots, g_{9}$ as follows: Let $k \\in\\{1, \\ldots, 9\\}$ and let $x$ be a string. If $x=\\varepsilon$ then $g_{k}(x)=\\varepsilon$. Otherwise, write $x$ in the form $x=y z r$ where $y$ is either the empty string or ends with a digit smaller than $k, s(z) \\geq k$ and $r$ is the rightmost digit of $x$. Then $g_{k}(x)=z r$. Lemma 1. We have $g_{k} \\circ h=g_{k} \\circ h \\circ g_{k}$ for all $k=1, \\ldots, 9$. Proof of Lemma 1. Let $x=y z r$ be as in the definition of $g_{k}$. If $y=\\varepsilon$, then $g_{k}(x)=x$, whence $$ g_{k}(h(x))=g_{k}\\left(h\\left(g_{k}(x)\\right) .\\right. $$ So let $y \\neq \\varepsilon$. Case 1. $z$ contains a digit smaller than $r$. Let $z=u a v$ where $a0\\end{cases} $$ and $$ h\\left(g_{k}(x)\\right)=h(z r)=h(\\text { uavr })= \\begin{cases}\\operatorname{uav} & \\text { if } r=0 \\\\ \\operatorname{uav}(r-1) v(r-1) & \\text { if } r>0\\end{cases} $$ Since $y$ ends with a digit smaller than $k$, (1) is obviously true. Case 2. $z$ does not contain a digit smaller than $r$. Let $y=u v$ where $u$ is either the empty string or ends with a digit smaller than $r$ and $s(v) \\geq r$. We have $$ h(x)= \\begin{cases}u v z & \\text { if } r=0 \\\\ u v z(r-1) v z(r-1) & \\text { if } r>0\\end{cases} $$ and $$ h\\left(g_{k}(x)\\right)=h(z r)= \\begin{cases}z & \\text { if } r=0 \\\\ z(r-1) z(r-1) & \\text { if } r>0\\end{cases} $$ Recall that $y$ and hence $v$ ends with a digit smaller than $k$, but all digits of $v$ are at least $r$. Now if $r>k$, then $v=\\varepsilon$, whence the terminal digit of $u$ is smaller than $k$, which entails $$ g_{k}(h(x))=z(r-1) z(r-1)=g_{k}\\left(h\\left(g_{k}(x)\\right)\\right) . $$ If $r \\leq k$, then $$ g_{k}(h(x))=z(r-1)=g_{k}\\left(h\\left(g_{k}(x)\\right)\\right), $$ so that in both cases (1) is true. Thus Lemma 1 is proved. Lemma 2. Let $k \\in\\{1, \\ldots, 9\\}$, let $x$ be a non-empty string and let $n$ be a positive integer. If $h^{n}(x)=\\varepsilon$ then $\\left(g_{k} \\circ h\\right)^{n}(x)=\\varepsilon$. Proof of Lemma 2. We proceed by induction on $n$. If $n=1$ we have $$ \\varepsilon=h(x)=g_{k}(h(x))=\\left(g_{k} \\circ h\\right)(x) . $$ Now consider the step from $n-1$ to $n$ where $n \\geq 2$. Let $h^{n}(x)=\\varepsilon$ and let $y=h(x)$. Then $h^{n-1}(y)=\\varepsilon$ and by the induction hypothesis $\\left(g_{k} \\circ h\\right)^{n-1}(y)=\\varepsilon$. In view of Lemma 1 , $$ \\begin{aligned} & \\varepsilon=\\left(g_{k} \\circ h\\right)^{n-2}\\left(\\left(g_{k} \\circ h\\right)(y)\\right)=\\left(g_{k} \\circ h\\right)^{n-2}\\left(g_{k}(h(y))\\right. \\\\ &=\\left(g_{k} \\circ h\\right)^{n-2}\\left(g_{k}\\left(h\\left(g_{k}(y)\\right)\\right)=\\left(g_{k} \\circ h\\right)^{n-2}\\left(g_{k}\\left(h\\left(g_{k}(h(x))\\right)\\right)=\\left(g_{k} \\circ h\\right)^{n}(x)\\right.\\right. \\end{aligned} $$ Thus the induction step is complete and Lemma 2 is proved. We say that the non-empty string $x$ terminates if $h^{n}(x)=\\varepsilon$ for some non-negative integer $n$. Lemma 3. Let $x=y z r$ where $s(y) \\geq k, s(z) \\geq k, y$ ends with the digit $k$ and $z$ is possibly empty. If $y$ and $z r$ terminate then also $x$ terminates. Proof of Lemma 3. Suppose that $y$ and $z r$ terminate. We proceed by induction on $k$. Let $k=0$. Obviously, $h(y w)=y h(w)$ for any non-empty string $w$. Let $h^{n}(z r)=\\epsilon$. It follows easily by induction on $m$ that $h^{m}(y z r)=y h^{m}(z r)$ for $m=1, \\ldots, n$. Consequently, $h^{n}(y z r)=y$. Since $y$ terminates, also $x=y z r$ terminates. Now let the assertion be true for all nonnegative integers less than $k$ and let us prove it for $k$ where $k \\geq 1$. It turns out that it is sufficient to prove that $y g_{k}(h(z r))$ terminates. Indeed: Case 1. $r=0$. Then $h(y z r)=y z=y g_{k}(h(z r))$. Case 2. $0k$. Then $h(y z r)=y h(z r)=y g_{k}(h(z r))$. Note that $y g_{k}(h(z r))$ has the form $y z^{\\prime} r^{\\prime}$ where $s\\left(z^{\\prime}\\right) \\geq k$. By the same arguments it is sufficient to prove that $y g_{k}\\left(h\\left(z^{\\prime} r^{\\prime}\\right)\\right)=y\\left(g_{k} \\circ h\\right)^{2}(z r)$ terminates and, by induction, that $y\\left(g_{k} \\circ h\\right)^{m}(z r)$ terminates for some positive integer $m$. In view of Lemma 2 there is some $m$ such that $\\left(g_{k} \\circ\\right.$ $h)^{m}(z r)=\\epsilon$, so $x=y z r$ terminates if $y$ terminates. Thus Lemma 3 is proved. Now assume that there is some string $x$ that does not terminate. We choose $x$ minimal. If $x \\geq 10$, we can write $x$ in the form $x=y z r$ of Lemma 3 and by this lemma $x$ terminates since $y$ and $z r$ are smaller than $x$. If $x \\leq 9$, then $h(x)=(x-1)(x-1)$ and $h(x)$ terminates again by Lemma 3 and the minimal choice of $x$.","tier":0} +{"problem_type":"Combinatorics","problem_label":"C8","problem":"AUT For any integer $n \\geq 2$, we compute the integer $h(n)$ by applying the following procedure to its decimal representation. Let $r$ be the rightmost digit of $n$. (1) If $r=0$, then the decimal representation of $h(n)$ results from the decimal representation of $n$ by removing this rightmost digit 0 . (2) If $1 \\leq r \\leq 9$ we split the decimal representation of $n$ into a maximal right part $R$ that solely consists of digits not less than $r$ and into a left part $L$ that either is empty or ends with a digit strictly smaller than $r$. Then the decimal representation of $h(n)$ consists of the decimal representation of $L$, followed by two copies of the decimal representation of $R-1$. For instance, for the number $n=17,151,345,543$, we will have $L=17,151, R=345,543$ and $h(n)=17,151,345,542,345,542$. Prove that, starting with an arbitrary integer $n \\geq 2$, iterated application of $h$ produces the integer 1 after finitely many steps.","solution":"We commence by introducing some terminology. Instead of integers, we will consider the set $S$ of all strings consisting of the digits $0,1, \\ldots, 9$, including the empty string $\\epsilon$. If $\\left(a_{1}, a_{2}, \\ldots, a_{n}\\right)$ is a nonempty string, we let $\\rho(a)=a_{n}$ denote the terminal digit of $a$ and $\\lambda(a)$ be the string with the last digit removed. We also define $\\lambda(\\epsilon)=\\epsilon$ and denote the set of non-negative integers by $\\mathbb{N}_{0}$. Now let $k \\in\\{0,1,2, \\ldots, 9\\}$ denote any digit. We define a function $f_{k}: S \\longrightarrow S$ on the set of strings: First, if the terminal digit of $n$ belongs to $\\{0,1, \\ldots, k\\}$, then $f_{k}(n)$ is obtained from $n$ by deleting this terminal digit, i.e $f_{k}(n)=\\lambda(n)$. Secondly, if the terminal digit of $n$ belongs to $\\{k+1, \\ldots, 9\\}$, then $f_{k}(n)$ is obtained from $n$ by the process described in the problem. We also define $f_{k}(\\epsilon)=\\epsilon$. Note that up to the definition for integers $n \\leq 1$, the function $f_{0}$ coincides with the function $h$ in the problem, through interpreting integers as digit strings. The argument will be roughly as follows. We begin by introducing a straightforward generalization of our claim about $f_{0}$. Then it will be easy to see that $f_{9}$ has all these stronger properties, which means that is suffices to show for $k \\in\\{0,1, \\ldots, 8\\}$ that $f_{k}$ possesses these properties provided that $f_{k+1}$ does. We continue to use $k$ to denote any digit. The operation $f_{k}$ is said to be separating, if the followings holds: Whenever $a$ is an initial segment of $b$, there is some $N \\in \\mathbb{N}_{0}$ such that $f_{k}^{N}(b)=a$. The following two notions only apply to the case where $f_{k}$ is indeed separating, otherwise they remain undefined. For every $a \\in S$ we denote the least $N \\in \\mathbb{N}_{0}$ for which $f_{k}^{N}(a)=\\epsilon$ occurs by $g_{k}(a)$ (because $\\epsilon$ is an initial segment of $a$, such an $N$ exists if $f_{k}$ is separating). If for every two strings $a$ and $b$ such that $a$ is a terminal segment of $b$ one has $g_{k}(a) \\leq g_{k}(b)$, we say that $f_{k}$ is coherent. In case that $f_{k}$ is separating and coherent we call the digit $k$ seductive. As $f_{9}(a)=\\lambda(a)$ for all $a$, it is obvious that 9 is seductive. Hence in order to show that 0 is seductive, which clearly implies the statement of the problem, it suffices to take any $k \\in\\{0,1, \\ldots, 8\\}$ such that $k+1$ is seductive and to prove that $k$ has to be seductive as well. Note that in doing so, we have the function $g_{k+1}$ at our disposal. We have to establish two things and we begin with Step 1. $f_{k}$ is separating. Before embarking on the proof of this, we record a useful observation which is easily proved by induction on $M$. Claim 1. For any strings $A, B$ and any positive integer $M$ such that $f_{k}^{M-1}(B) \\neq \\epsilon$, we have $$ f_{k}^{M}(A k B)=A k f_{k}^{M}(B) $$ Now we call a pair $(a, b)$ of strings wicked provided that $a$ is an initial segment of $b$, but there is no $N \\in \\mathbb{N}_{0}$ such that $f_{k}^{N}(b)=a$. We need to show that there are none, so assume that there were such pairs. Choose a wicked pair $(a, b)$ for which $g_{k+1}(b)$ attains its minimal possible value. Obviously $b \\neq \\epsilon$ for any wicked pair $(a, b)$. Let $z$ denote the terminal digit of $b$. Observe that $a \\neq b$, which means that $a$ is also an initial segment of $\\lambda(b)$. To facilitate the construction of the eventual contradiction, we prove Claim 2. There cannot be an $N \\in \\mathbb{N}_{0}$ such that $$ f_{k}^{N}(b)=\\lambda(b) $$ Proof of Claim 2. For suppose that such an $N$ existed. Because $g_{k+1}(\\lambda(b))k+1$ is impossible: Set $B=f_{k}(b)$. Then also $f_{k+1}(b)=B$, but $g_{k+1}(B)g_{k}(b)$. Observe that if $f_{k}$ was incoherent, which we shall assume from now on, then such pairs existed. Now among all aggressive pairs we choose one, say $(a, b)$, for which $g_{k}(b)$ attains its least possible value. Obviously $f_{k}(a)$ cannot be injectible into $f_{k}(b)$, for otherwise the pair $\\left(f_{k}(a), f_{k}(b)\\right)$ was aggressive and contradicted our choice of $(a, b)$. Let $\\left(A_{1}, A_{2}, \\ldots, A_{m}\\right)$ and $\\left(B_{1}, B_{2}, \\ldots, B_{n}\\right)$ be the decompositions of $a$ and $b$ and take a function $H:\\{1,2, \\ldots, m\\} \\longrightarrow\\{1,2, \\ldots, n\\}$ exemplifying that $a$ is indeed injectible into $b$. If we had $H(m)|O A B|$, which is contradictory to the choice of $A$ and $B$. If it is one of the lines $b, a^{\\prime}$ or $b^{\\prime}$ almost identical arguments lead to a similar contradiction. Let $R_{2}$ be the parallelogram $A B A^{\\prime} B^{\\prime}$. Since $A$ and $B$ are points of $P$, segment $A B \\subset P$ and so $R_{2} \\subset R_{1}$. Since $A, B, A^{\\prime}$ and $B^{\\prime}$ are midpoints of the sides of $R_{1}$, an easy argument yields $$ \\left|R_{1}\\right|=2 \\cdot\\left|R_{2}\\right| $$ Let $R_{3}$ be the smallest parallelogram enclosing $P$ defined by lines parallel to $A B$ and $B A^{\\prime}$. Obviously $R_{2} \\subset R_{3}$ and every side of $R_{3}$ contains at least one point of the boundary of $P$. Denote by $C$ the intersection point of $a$ and $b$, by $X$ the intersection point of $A B$ and $O C$, and by $X^{\\prime}$ the intersection point of $X C$ and the boundary of $R_{3}$. In a similar way denote by $D$ the intersection point of $b$ and $a^{\\prime}$, by $Y$ the intersection point of $A^{\\prime} B$ and $O D$, and by $Y^{\\prime}$ the intersection point of $Y D$ and the boundary of $R_{3}$. Note that $O C=2 \\cdot O X$ and $O D=2 \\cdot O Y$, so there exist real numbers $x$ and $y$ with $1 \\leq x, y \\leq 2$ and $O X^{\\prime}=x \\cdot O X$ and $O Y^{\\prime}=y \\cdot O Y$. Corresponding sides of $R_{3}$ and $R_{2}$ are parallel which yields $$ \\left|R_{3}\\right|=x y \\cdot\\left|R_{2}\\right| $$ The side of $R_{3}$ containing $X^{\\prime}$ contains at least one point $X^{*}$ of $P$; due to the convexity of $P$ we have $A X^{*} B \\subset P$. Since this side of the parallelogram $R_{3}$ is parallel to $A B$ we have $\\left|A X^{*} B\\right|=\\left|A X^{\\prime} B\\right|$, so $\\left|O A X^{\\prime} B\\right|$ does not exceed the area of $P$ confined to the sector defined by the rays $O B$ and $O A$. In a similar way we conclude that $\\left|O B^{\\prime} Y^{\\prime} A^{\\prime}\\right|$ does not exceed the area of $P$ confined to the sector defined by the rays $O B$ and $O A^{\\prime}$. Putting things together we have $\\left|O A X^{\\prime} B\\right|=x \\cdot|O A B|,\\left|O B D A^{\\prime}\\right|=y \\cdot\\left|O B A^{\\prime}\\right|$. Since $|O A B|=\\left|O B A^{\\prime}\\right|$, we conclude that $|P| \\geq 2 \\cdot\\left|A X^{\\prime} B Y^{\\prime} A^{\\prime}\\right|=2 \\cdot\\left(x \\cdot|O A B|+y \\cdot\\left|O B A^{\\prime}\\right|\\right)=4 \\cdot \\frac{x+y}{2} \\cdot|O A B|=\\frac{x+y}{2} \\cdot R_{2}$; this is in short $$ \\frac{x+y}{2} \\cdot\\left|R_{2}\\right| \\leq|P| $$ Since all numbers concerned are positive, we can combine (1)-(3). Using the arithmetic-geometric-mean inequality we obtain $$ \\left|R_{1}\\right| \\cdot\\left|R_{3}\\right|=2 \\cdot\\left|R_{2}\\right| \\cdot x y \\cdot\\left|R_{2}\\right| \\leq 2 \\cdot\\left|R_{2}\\right|^{2}\\left(\\frac{x+y}{2}\\right)^{2} \\leq 2 \\cdot|P|^{2} $$ This implies immediately the desired result $\\left|R_{1}\\right| \\leq \\sqrt{2} \\cdot|P|$ or $\\left|R_{3}\\right| \\leq \\sqrt{2} \\cdot|P|$.","tier":0} +{"problem_type":"Geometry","problem_label":"G5","problem":"POL Let $P$ be a polygon that is convex and symmetric to some point $O$. Prove that for some parallelogram $R$ satisfying $P \\subset R$ we have $$ \\frac{|R|}{|P|} \\leq \\sqrt{2} $$ where $|R|$ and $|P|$ denote the area of the sets $R$ and $P$, respectively.","solution":"We construct the parallelograms $R_{1}, R_{2}$ and $R_{3}$ in the same way as in ![](https:\/\/cdn.mathpix.com\/cropped\/2024_11_18_b191c9407e8cb3d672cbg-60.jpg?height=569&width=580&top_left_y=1720&top_left_x=738) Figure 2 Recall that affine one-to-one maps of the plane preserve the ratio of areas of subsets of the plane. On the other hand, every parallelogram can be transformed with an affine map onto a square. It follows that without loss of generality we may assume that $R_{1}$ is a square (see Figure 2). Then $R_{2}$, whose vertices are the midpoints of the sides of $R_{1}$, is a square too, and $R_{3}$, whose sides are parallel to the diagonals of $R_{1}$, is a rectangle. Let $a>0, b \\geq 0$ and $c \\geq 0$ be the distances introduced in Figure 2. Then $\\left|R_{1}\\right|=2 a^{2}$ and $\\left|R_{3}\\right|=(a+2 b)(a+2 c)$. Points $A, A^{\\prime}, B$ and $B^{\\prime}$ are in the convex polygon $P$. Hence the square $A B A^{\\prime} B^{\\prime}$ is a subset of $P$. Moreover, each of the sides of the rectangle $R_{3}$ contains a point of $P$, otherwise $R_{3}$ would not be minimal. It follows that $$ |P| \\geq a^{2}+2 \\cdot \\frac{a b}{2}+2 \\cdot \\frac{a c}{2}=a(a+b+c) $$ Now assume that both $\\frac{\\left|R_{1}\\right|}{|P|}>\\sqrt{2}$ and $\\frac{\\left|R_{3}\\right|}{|P|}>\\sqrt{2}$, then $$ 2 a^{2}=\\left|R_{1}\\right|>\\sqrt{2} \\cdot|P| \\geq \\sqrt{2} \\cdot a(a+b+c) $$ and $$ (a+2 b)(a+2 c)=\\left|R_{3}\\right|>\\sqrt{2} \\cdot|P| \\geq \\sqrt{2} \\cdot a(a+b+c) $$ All numbers concerned are positive, so after multiplying these inequalities we get $$ 2 a^{2}(a+2 b)(a+2 c)>2 a^{2}(a+b+c)^{2} $$ But the arithmetic-geometric-mean inequality implies the contradictory result $$ 2 a^{2}(a+2 b)(a+2 c) \\leq 2 a^{2}\\left(\\frac{(a+2 b)+(a+2 c)}{2}\\right)^{2}=2 a^{2}(a+b+c)^{2} $$ Hence $\\frac{\\left|R_{1}\\right|}{|P|} \\leq \\sqrt{2}$ or $\\frac{\\left|R_{3}\\right|}{|P|} \\leq \\sqrt{2}$, as desired.","tier":0} +{"problem_type":"Geometry","problem_label":"G6","problem":"UKR Let the sides $A D$ and $B C$ of the quadrilateral $A B C D$ (such that $A B$ is not parallel to $C D$ ) intersect at point $P$. Points $O_{1}$ and $O_{2}$ are the circumcenters and points $H_{1}$ and $H_{2}$ are the orthocenters of triangles $A B P$ and $D C P$, respectively. Denote the midpoints of segments $O_{1} H_{1}$ and $O_{2} H_{2}$ by $E_{1}$ and $E_{2}$, respectively. Prove that the perpendicular from $E_{1}$ on $C D$, the perpendicular from $E_{2}$ on $A B$ and the line $H_{1} H_{2}$ are concurrent.","solution":"We keep triangle $A B P$ fixed and move the line $C D$ parallel to itself uniformly, i.e. linearly dependent on a single parameter $\\lambda$ (see Figure 1). Then the points $C$ and $D$ also move uniformly. Hence, the points $O_{2}, H_{2}$ and $E_{2}$ move uniformly, too. Therefore also the perpendicular from $E_{2}$ on $A B$ moves uniformly. Obviously, the points $O_{1}, H_{1}, E_{1}$ and the perpendicular from $E_{1}$ on $C D$ do not move at all. Hence, the intersection point $S$ of these two perpendiculars moves uniformly. Since $H_{1}$ does not move, while $H_{2}$ and $S$ move uniformly along parallel lines (both are perpendicular to $C D$ ), it is sufficient to prove their collinearity for two different positions of $C D$. ![](https:\/\/cdn.mathpix.com\/cropped\/2024_11_18_b191c9407e8cb3d672cbg-62.jpg?height=897&width=895&top_left_y=1162&top_left_x=592) Figure 1 Let $C D$ pass through either point $A$ or point $B$. Note that by hypothesis these two cases are different. We will consider the case $A \\in C D$, i.e. $A=D$. So we have to show that the perpendiculars from $E_{1}$ on $A C$ and from $E_{2}$ on $A B$ intersect on the altitude $A H$ of triangle $A B C$ (see Figure 2). ![](https:\/\/cdn.mathpix.com\/cropped\/2024_11_18_b191c9407e8cb3d672cbg-63.jpg?height=971&width=749&top_left_y=197&top_left_x=659) Figure 2 To this end, we consider the midpoints $A_{1}, B_{1}, C_{1}$ of $B C, C A, A B$, respectively. As $E_{1}$ is the center of Feuerbach's circle (nine-point circle) of $\\triangle A B P$, we have $E_{1} C_{1}=E_{1} H$. Similarly, $E_{2} B_{1}=E_{2} H$. Note further that a point $X$ lies on the perpendicular from $E_{1}$ on $A_{1} C_{1}$ if and only if $$ X C_{1}^{2}-X A_{1}^{2}=E_{1} C_{1}^{2}-E_{1} A_{1}^{2} $$ Similarly, the perpendicular from $E_{2}$ on $A_{1} B_{1}$ is characterized by $$ X A_{1}^{2}-X B_{1}^{2}=E_{2} A_{1}^{2}-E_{2} B_{1}^{2} $$ The line $H_{1} H_{2}$, which is perpendicular to $B_{1} C_{1}$ and contains $A$, is given by $$ X B_{1}^{2}-X C_{1}^{2}=A B_{1}^{2}-A C_{1}^{2} $$ The three lines are concurrent if and only if $$ \\begin{aligned} 0 & =X C_{1}^{2}-X A_{1}^{2}+X A_{1}^{2}-X B_{1}^{2}+X B_{1}^{2}-X C_{1}^{2} \\\\ & =E_{1} C_{1}^{2}-E_{1} A_{1}^{2}+E_{2} A_{1}^{2}-E_{2} B_{1}^{2}+A B_{1}^{2}-A C_{1}^{2} \\\\ & =-E_{1} A_{1}^{2}+E_{2} A_{1}^{2}+E_{1} H^{2}-E_{2} H^{2}+A B_{1}^{2}-A C_{1}^{2} \\end{aligned} $$ i.e. it suffices to show that $$ E_{1} A_{1}^{2}-E_{2} A_{1}^{2}-E_{1} H^{2}+E_{2} H^{2}=\\frac{A C^{2}-A B^{2}}{4} $$ We have $$ \\frac{A C^{2}-A B^{2}}{4}=\\frac{H C^{2}-H B^{2}}{4}=\\frac{(H C+H B)(H C-H B)}{4}=\\frac{H A_{1} \\cdot B C}{2} $$ Let $F_{1}, F_{2}$ be the projections of $E_{1}, E_{2}$ on $B C$. Obviously, these are the midpoints of $H P_{1}$, $H P_{2}$, where $P_{1}, P_{2}$ are the midpoints of $P B$ and $P C$ respectively. Then $$ \\begin{aligned} & E_{1} A_{1}^{2}-E_{2} A_{1}^{2}-E_{1} H^{2}+E_{2} H^{2} \\\\ & =F_{1} A_{1}^{2}-F_{1} H^{2}-F_{2} A_{1}^{2}+F_{2} H^{2} \\\\ & =\\left(F_{1} A_{1}-F_{1} H\\right)\\left(F_{1} A_{1}+F_{1} H\\right)-\\left(F_{2} A_{1}-F_{2} H\\right)\\left(F_{2} A_{1}+F_{2} H\\right) \\\\ & =A_{1} H \\cdot\\left(A_{1} P_{1}-A_{1} P_{2}\\right) \\\\ & =\\frac{A_{1} H \\cdot B C}{2} \\\\ & =\\frac{A C^{2}-A B^{2}}{4} \\end{aligned} $$ which proves the claim.","tier":0} +{"problem_type":"Geometry","problem_label":"G6","problem":"UKR Let the sides $A D$ and $B C$ of the quadrilateral $A B C D$ (such that $A B$ is not parallel to $C D$ ) intersect at point $P$. Points $O_{1}$ and $O_{2}$ are the circumcenters and points $H_{1}$ and $H_{2}$ are the orthocenters of triangles $A B P$ and $D C P$, respectively. Denote the midpoints of segments $O_{1} H_{1}$ and $O_{2} H_{2}$ by $E_{1}$ and $E_{2}$, respectively. Prove that the perpendicular from $E_{1}$ on $C D$, the perpendicular from $E_{2}$ on $A B$ and the line $H_{1} H_{2}$ are concurrent.","solution":"Let the perpendicular from $E_{1}$ on $C D$ meet $P H_{1}$ at $X$, and the perpendicular from $E_{2}$ on $A B$ meet $P H_{2}$ at $Y$ (see Figure 3). Let $\\varphi$ be the intersection angle of $A B$ and $C D$. Denote by $M, N$ the midpoints of $P H_{1}, P H_{2}$ respectively. ![](https:\/\/cdn.mathpix.com\/cropped\/2024_11_18_b191c9407e8cb3d672cbg-64.jpg?height=754&width=1069&top_left_y=1185&top_left_x=499) Figure 3 We will prove now that triangles $E_{1} X M$ and $E_{2} Y N$ have equal angles at $E_{1}, E_{2}$, and supplementary angles at $X, Y$. In the following, angles are understood as oriented, and equalities of angles modulo $180^{\\circ}$. Let $\\alpha=\\angle H_{2} P D, \\psi=\\angle D P C, \\beta=\\angle C P H_{1}$. Then $\\alpha+\\psi+\\beta=\\varphi, \\angle E_{1} X H_{1}=\\angle H_{2} Y E_{2}=\\varphi$, thus $\\angle M X E_{1}+\\angle N Y E_{2}=180^{\\circ}$. By considering the Feuerbach circle of $\\triangle A B P$ whose center is $E_{1}$ and which goes through $M$, we have $\\angle E_{1} M H_{1}=\\psi+2 \\beta$. Analogous considerations with the Feuerbach circle of $\\triangle D C P$ yield $\\angle H_{2} N E_{2}=\\psi+2 \\alpha$. Hence indeed $\\angle X E_{1} M=\\varphi-(\\psi+2 \\beta)=(\\psi+2 \\alpha)-\\varphi=\\angle Y E_{2} N$. It follows now that $$ \\frac{X M}{M E_{1}}=\\frac{Y N}{N E_{2}} $$ Furthermore, $M E_{1}$ is half the circumradius of $\\triangle A B P$, while $P H_{1}$ is the distance of $P$ to the orthocenter of that triangle, which is twice the circumradius times the cosine of $\\psi$. Together with analogous reasoning for $\\triangle D C P$ we have $$ \\frac{M E_{1}}{P H_{1}}=\\frac{1}{4 \\cos \\psi}=\\frac{N E_{2}}{P H_{2}} $$ By multiplication, $$ \\frac{X M}{P H_{1}}=\\frac{Y N}{P H_{2}} $$ and therefore $$ \\frac{P X}{X H_{1}}=\\frac{H_{2} Y}{Y P} $$ Let $E_{1} X, E_{2} Y$ meet $H_{1} H_{2}$ in $R, S$ respectively. Applying the intercept theorem to the parallels $E_{1} X, P H_{2}$ and center $H_{1}$ gives $$ \\frac{H_{2} R}{R H_{1}}=\\frac{P X}{X H_{1}} $$ while with parallels $E_{2} Y, P H_{1}$ and center $H_{2}$ we obtain $$ \\frac{H_{2} S}{S H_{1}}=\\frac{H_{2} Y}{Y P} $$ Combination of the last three equalities yields that $R$ and $S$ coincide.","tier":0} +{"problem_type":"Geometry","problem_label":"G7","problem":"IRN Let $A B C$ be a triangle with incenter $I$ and let $X, Y$ and $Z$ be the incenters of the triangles $B I C, C I A$ and $A I B$, respectively. Let the triangle $X Y Z$ be equilateral. Prove that $A B C$ is equilateral too.","solution":"$A Z, A I$ and $A Y$ divide $\\angle B A C$ into four equal angles; denote them by $\\alpha$. In the same way we have four equal angles $\\beta$ at $B$ and four equal angles $\\gamma$ at $C$. Obviously $\\alpha+\\beta+\\gamma=\\frac{180^{\\circ}}{4}=45^{\\circ}$; and $0^{\\circ}<\\alpha, \\beta, \\gamma<45^{\\circ}$. ![](https:\/\/cdn.mathpix.com\/cropped\/2024_11_18_b191c9407e8cb3d672cbg-66.jpg?height=629&width=938&top_left_y=802&top_left_x=562) Easy calculations in various triangles yield $\\angle B I C=180^{\\circ}-2 \\beta-2 \\gamma=180^{\\circ}-\\left(90^{\\circ}-2 \\alpha\\right)=$ $90^{\\circ}+2 \\alpha$, hence (for $X$ is the incenter of triangle $B C I$, so $I X$ bisects $\\angle B I C$ ) we have $\\angle X I C=$ $\\angle B I X=\\frac{1}{2} \\angle B I C=45^{\\circ}+\\alpha$ and with similar aguments $\\angle C I Y=\\angle Y I A=45^{\\circ}+\\beta$ and $\\angle A I Z=\\angle Z I B=45^{\\circ}+\\gamma$. Furthermore, we have $\\angle X I Y=\\angle X I C+\\angle C I Y=\\left(45^{\\circ}+\\alpha\\right)+$ $\\left(45^{\\circ}+\\beta\\right)=135^{\\circ}-\\gamma, \\angle Y I Z=135^{\\circ}-\\alpha$, and $\\angle Z I X=135^{\\circ}-\\beta$. Now we calculate the lengths of $I X, I Y$ and $I Z$ in terms of $\\alpha, \\beta$ and $\\gamma$. The perpendicular from $I$ on $C X$ has length $I X \\cdot \\sin \\angle C X I=I X \\cdot \\sin \\left(90^{\\circ}+\\beta\\right)=I X \\cdot \\cos \\beta$. But $C I$ bisects $\\angle Y C X$, so the perpendicular from $I$ on $C Y$ has the same length, and we conclude $$ I X \\cdot \\cos \\beta=I Y \\cdot \\cos \\alpha $$ To make calculations easier we choose a length unit that makes $I X=\\cos \\alpha$. Then $I Y=\\cos \\beta$ and with similar arguments $I Z=\\cos \\gamma$. Since $X Y Z$ is equilateral we have $Z X=Z Y$. The law of Cosines in triangles $X Y I, Y Z I$ yields $$ \\begin{aligned} & Z X^{2}=Z Y^{2} \\\\ \\Longrightarrow & I Z^{2}+I X^{2}-2 \\cdot I Z \\cdot I X \\cdot \\cos \\angle Z I X=I Z^{2}+I Y^{2}-2 \\cdot I Z \\cdot I Y \\cdot \\cos \\angle Y I Z \\\\ \\Longrightarrow & I X^{2}-I Y^{2}=2 \\cdot I Z \\cdot(I X \\cdot \\cos \\angle Z I X-I Y \\cdot \\cos \\angle Y I Z) \\\\ \\Longrightarrow & \\underbrace{\\cos ^{2} \\alpha-\\cos ^{2} \\beta}_{\\text {L.H.S. }}=\\underbrace{2 \\cdot \\cos \\gamma \\cdot\\left(\\cos \\alpha \\cdot \\cos \\left(135^{\\circ}-\\beta\\right)-\\cos \\beta \\cdot \\cos \\left(135^{\\circ}-\\alpha\\right)\\right)}_{\\text {R.H.S. }} . \\end{aligned} $$ A transformation of the left-hand side (L.H.S.) yields $$ \\begin{aligned} \\text { L.H.S. } & =\\cos ^{2} \\alpha \\cdot\\left(\\sin ^{2} \\beta+\\cos ^{2} \\beta\\right)-\\cos ^{2} \\beta \\cdot\\left(\\sin ^{2} \\alpha+\\cos ^{2} \\alpha\\right) \\\\ & =\\cos ^{2} \\alpha \\cdot \\sin ^{2} \\beta-\\cos ^{2} \\beta \\cdot \\sin ^{2} \\alpha \\end{aligned} $$ $$ \\begin{aligned} & =(\\cos \\alpha \\cdot \\sin \\beta+\\cos \\beta \\cdot \\sin \\alpha) \\cdot(\\cos \\alpha \\cdot \\sin \\beta-\\cos \\beta \\cdot \\sin \\alpha) \\\\ & =\\sin (\\beta+\\alpha) \\cdot \\sin (\\beta-\\alpha)=\\sin \\left(45^{\\circ}-\\gamma\\right) \\cdot \\sin (\\beta-\\alpha) \\end{aligned} $$ whereas a transformation of the right-hand side (R.H.S.) leads to $$ \\begin{aligned} \\text { R.H.S. } & =2 \\cdot \\cos \\gamma \\cdot\\left(\\cos \\alpha \\cdot\\left(-\\cos \\left(45^{\\circ}+\\beta\\right)\\right)-\\cos \\beta \\cdot\\left(-\\cos \\left(45^{\\circ}+\\alpha\\right)\\right)\\right) \\\\ & =2 \\cdot \\frac{\\sqrt{2}}{2} \\cdot \\cos \\gamma \\cdot(\\cos \\alpha \\cdot(\\sin \\beta-\\cos \\beta)+\\cos \\beta \\cdot(\\cos \\alpha-\\sin \\alpha)) \\\\ & =\\sqrt{2} \\cdot \\cos \\gamma \\cdot(\\cos \\alpha \\cdot \\sin \\beta-\\cos \\beta \\cdot \\sin \\alpha) \\\\ & =\\sqrt{2} \\cdot \\cos \\gamma \\cdot \\sin (\\beta-\\alpha) \\end{aligned} $$ Equating L.H.S. and R.H.S. we obtain $$ \\begin{aligned} & \\sin \\left(45^{\\circ}-\\gamma\\right) \\cdot \\sin (\\beta-\\alpha)=\\sqrt{2} \\cdot \\cos \\gamma \\cdot \\sin (\\beta-\\alpha) \\\\ \\Longrightarrow & \\sin (\\beta-\\alpha) \\cdot\\left(\\sqrt{2} \\cdot \\cos \\gamma-\\sin \\left(45^{\\circ}-\\gamma\\right)\\right)=0 \\\\ \\Longrightarrow & \\alpha=\\beta \\quad \\text { or } \\quad \\sqrt{2} \\cdot \\cos \\gamma=\\sin \\left(45^{\\circ}-\\gamma\\right) . \\end{aligned} $$ But $\\gamma<45^{\\circ}$; so $\\sqrt{2} \\cdot \\cos \\gamma>\\cos \\gamma>\\cos 45^{\\circ}=\\sin 45^{\\circ}>\\sin \\left(45^{\\circ}-\\gamma\\right)$. This leaves $\\alpha=\\beta$. With similar reasoning we have $\\alpha=\\gamma$, which means triangle $A B C$ must be equilateral.","tier":0} +{"problem_type":"Geometry","problem_label":"G8","problem":"BGR Let $A B C D$ be a circumscribed quadrilateral. Let $g$ be a line through $A$ which meets the segment $B C$ in $M$ and the line $C D$ in $N$. Denote by $I_{1}, I_{2}$, and $I_{3}$ the incenters of $\\triangle A B M$, $\\triangle M N C$, and $\\triangle N D A$, respectively. Show that the orthocenter of $\\triangle I_{1} I_{2} I_{3}$ lies on $g$.","solution":"Let $k_{1}, k_{2}$ and $k_{3}$ be the incircles of triangles $A B M, M N C$, and $N D A$, respectively (see Figure 1). We shall show that the tangent $h$ from $C$ to $k_{1}$ which is different from $C B$ is also tangent to $k_{3}$. ![](https:\/\/cdn.mathpix.com\/cropped\/2024_11_18_b191c9407e8cb3d672cbg-68.jpg?height=926&width=1063&top_left_y=842&top_left_x=499) Figure 1 To this end, let $X$ denote the point of intersection of $g$ and $h$. Then $A B C X$ and $A B C D$ are circumscribed quadrilaterals, whence $$ C D-C X=(A B+C D)-(A B+C X)=(B C+A D)-(B C+A X)=A D-A X $$ i.e. $$ A X+C D=C X+A D $$ which in turn reveals that the quadrilateral $A X C D$ is also circumscribed. Thus $h$ touches indeed the circle $k_{3}$. Moreover, we find that $\\angle I_{3} C I_{1}=\\angle I_{3} C X+\\angle X C I_{1}=\\frac{1}{2}(\\angle D C X+\\angle X C B)=\\frac{1}{2} \\angle D C B=$ $\\frac{1}{2}\\left(180^{\\circ}-\\angle M C N\\right)=180^{\\circ}-\\angle M I_{2} N=\\angle I_{3} I_{2} I_{1}$, from which we conclude that $C, I_{1}, I_{2}, I_{3}$ are concyclic. Let now $L_{1}$ and $L_{3}$ be the reflection points of $C$ with respect to the lines $I_{2} I_{3}$ and $I_{1} I_{2}$ respectively. Since $I_{1} I_{2}$ is the angle bisector of $\\angle N M C$, it follows that $L_{3}$ lies on $g$. By analogous reasoning, $L_{1}$ lies on $g$. Let $H$ be the orthocenter of $\\triangle I_{1} I_{2} I_{3}$. We have $\\angle I_{2} L_{3} I_{1}=\\angle I_{1} C I_{2}=\\angle I_{1} I_{3} I_{2}=180^{\\circ}-\\angle I_{1} H I_{2}$, which entails that the quadrilateral $I_{2} H I_{1} L_{3}$ is cyclic. Analogously, $I_{3} H L_{1} I_{2}$ is cyclic. Then, working with oriented angles modulo $180^{\\circ}$, we have $$ \\angle L_{3} H I_{2}=\\angle L_{3} I_{1} I_{2}=\\angle I_{2} I_{1} C=\\angle I_{2} I_{3} C=\\angle L_{1} I_{3} I_{2}=\\angle L_{1} H I_{2}, $$ whence $L_{1}, L_{3}$, and $H$ are collinear. By $L_{1} \\neq L_{3}$, the claim follows. Comment. The last part of the argument essentially reproves the following fact: The Simson line of a point $P$ lying on the circumcircle of a triangle $A B C$ with respect to that triangle bisects the line segment connecting $P$ with the orthocenter of $A B C$.","tier":0} +{"problem_type":"Geometry","problem_label":"G8","problem":"BGR Let $A B C D$ be a circumscribed quadrilateral. Let $g$ be a line through $A$ which meets the segment $B C$ in $M$ and the line $C D$ in $N$. Denote by $I_{1}, I_{2}$, and $I_{3}$ the incenters of $\\triangle A B M$, $\\triangle M N C$, and $\\triangle N D A$, respectively. Show that the orthocenter of $\\triangle I_{1} I_{2} I_{3}$ lies on $g$.","solution":"We start by proving that $C, I_{1}, I_{2}$, and $I_{3}$ are concyclic. ![](https:\/\/cdn.mathpix.com\/cropped\/2024_11_18_b191c9407e8cb3d672cbg-69.jpg?height=797&width=1015&top_left_y=818&top_left_x=526) Figure 2 To this end, notice first that $I_{2}, M, I_{1}$ are collinear, as are $N, I_{2}, I_{3}$ (see Figure 2). Denote by $\\alpha, \\beta, \\gamma, \\delta$ the internal angles of $A B C D$. By considerations in triangle $C M N$, it follows that $\\angle I_{3} I_{2} I_{1}=\\frac{\\gamma}{2}$. We will show that $\\angle I_{3} C I_{1}=\\frac{\\gamma}{2}$, too. Denote by $I$ the incenter of $A B C D$. Clearly, $I_{1} \\in B I, I_{3} \\in D I, \\angle I_{1} A I_{3}=\\frac{\\alpha}{2}$. Using the abbreviation $[X, Y Z]$ for the distance from point $X$ to the line $Y Z$, we have because of $\\angle B A I_{1}=\\angle I A I_{3}$ and $\\angle I_{1} A I=\\angle I_{3} A D$ that $$ \\frac{\\left[I_{1}, A B\\right]}{\\left[I_{1}, A I\\right]}=\\frac{\\left[I_{3}, A I\\right]}{\\left[I_{3}, A D\\right]} $$ Furthermore, consideration of the angle sums in $A I B, B I C, C I D$ and $D I A$ implies $\\angle A I B+$ $\\angle C I D=\\angle B I C+\\angle D I A=180^{\\circ}$, from which we see $$ \\frac{\\left[I_{1}, A I\\right]}{\\left[I_{3}, C I\\right]}=\\frac{I_{1} I}{I_{3} I}=\\frac{\\left[I_{1}, C I\\right]}{\\left[I_{3}, A I\\right]} . $$ Because of $\\left[I_{1}, A B\\right]=\\left[I_{1}, B C\\right],\\left[I_{3}, A D\\right]=\\left[I_{3}, C D\\right]$, multiplication yields $$ \\frac{\\left[I_{1}, B C\\right]}{\\left[I_{3}, C I\\right]}=\\frac{\\left[I_{1}, C I\\right]}{\\left[I_{3}, C D\\right]} . $$ By $\\angle D C I=\\angle I C B=\\gamma \/ 2$ it follows that $\\angle I_{1} C B=\\angle I_{3} C I$ which concludes the proof of the above statement. Let the perpendicular from $I_{1}$ on $I_{2} I_{3}$ intersect $g$ at $Z$. Then $\\angle M I_{1} Z=90^{\\circ}-\\angle I_{3} I_{2} I_{1}=$ $90^{\\circ}-\\gamma \/ 2=\\angle M C I_{2}$. Since we have also $\\angle Z M I_{1}=\\angle I_{2} M C$, triangles $M Z I_{1}$ and $M I_{2} C$ are similar. From this one easily proves that also $M I_{2} Z$ and $M C I_{1}$ are similar. Because $C, I_{1}, I_{2}$, and $I_{3}$ are concyclic, $\\angle M Z I_{2}=\\angle M I_{1} C=\\angle N I_{3} C$, thus $N I_{2} Z$ and $N C I_{3}$ are similar, hence $N C I_{2}$ and $N I_{3} Z$ are similar. We conclude $\\angle Z I_{3} I_{2}=\\angle I_{2} C N=90^{\\circ}-\\gamma \/ 2$, hence $I_{1} I_{2} \\perp Z I_{3}$. This completes the proof.","tier":0} +{"problem_type":"Number Theory","problem_label":"N1","problem":"AUS A social club has $n$ members. They have the membership numbers $1,2, \\ldots, n$, respectively. From time to time members send presents to other members, including items they have already received as presents from other members. In order to avoid the embarrassing situation that a member might receive a present that he or she has sent to other members, the club adds the following rule to its statutes at one of its annual general meetings: \"A member with membership number $a$ is permitted to send a present to a member with membership number $b$ if and only if $a(b-1)$ is a multiple of $n$.\" Prove that, if each member follows this rule, none will receive a present from another member that he or she has already sent to other members. Alternative formulation: Let $G$ be a directed graph with $n$ vertices $v_{1}, v_{2}, \\ldots, v_{n}$, such that there is an edge going from $v_{a}$ to $v_{b}$ if and only if $a$ and $b$ are distinct and $a(b-1)$ is a multiple of $n$. Prove that this graph does not contain a directed cycle.","solution":"Suppose there is an edge from $v_{i}$ to $v_{j}$. Then $i(j-1)=i j-i=k n$ for some integer $k$, which implies $i=i j-k n$. If $\\operatorname{gcd}(i, n)=d$ and $\\operatorname{gcd}(j, n)=e$, then $e$ divides $i j-k n=i$ and thus $e$ also divides $d$. Hence, if there is an edge from $v_{i}$ to $v_{j}$, then $\\operatorname{gcd}(j, n) \\mid \\operatorname{gcd}(i, n)$. If there is a cycle in $G$, say $v_{i_{1}} \\rightarrow v_{i_{2}} \\rightarrow \\cdots \\rightarrow v_{i_{r}} \\rightarrow v_{i_{1}}$, then we have $$ \\operatorname{gcd}\\left(i_{1}, n\\right)\\left|\\operatorname{gcd}\\left(i_{r}, n\\right)\\right| \\operatorname{gcd}\\left(i_{r-1}, n\\right)|\\ldots| \\operatorname{gcd}\\left(i_{2}, n\\right) \\mid \\operatorname{gcd}\\left(i_{1}, n\\right) $$ which implies that all these greatest common divisors must be equal, say be equal to $t$. Now we pick any of the $i_{k}$, without loss of generality let it be $i_{1}$. Then $i_{r}\\left(i_{1}-1\\right)$ is a multiple of $n$ and hence also (by dividing by $t$ ), $i_{1}-1$ is a multiple of $\\frac{n}{t}$. Since $i_{1}$ and $i_{1}-1$ are relatively prime, also $t$ and $\\frac{n}{t}$ are relatively prime. So, by the Chinese remainder theorem, the value of $i_{1}$ is uniquely determined modulo $n=t \\cdot \\frac{n}{t}$ by the value of $t$. But, as $i_{1}$ was chosen arbitrarily among the $i_{k}$, this implies that all the $i_{k}$ have to be equal, a contradiction.","tier":0} +{"problem_type":"Number Theory","problem_label":"N1","problem":"AUS A social club has $n$ members. They have the membership numbers $1,2, \\ldots, n$, respectively. From time to time members send presents to other members, including items they have already received as presents from other members. In order to avoid the embarrassing situation that a member might receive a present that he or she has sent to other members, the club adds the following rule to its statutes at one of its annual general meetings: \"A member with membership number $a$ is permitted to send a present to a member with membership number $b$ if and only if $a(b-1)$ is a multiple of $n$.\" Prove that, if each member follows this rule, none will receive a present from another member that he or she has already sent to other members. Alternative formulation: Let $G$ be a directed graph with $n$ vertices $v_{1}, v_{2}, \\ldots, v_{n}$, such that there is an edge going from $v_{a}$ to $v_{b}$ if and only if $a$ and $b$ are distinct and $a(b-1)$ is a multiple of $n$. Prove that this graph does not contain a directed cycle.","solution":"If $a, b, c$ are integers such that $a b-a$ and $b c-b$ are multiples of $n$, then also $a c-a=a(b c-b)+(a b-a)-(a b-a) c$ is a multiple of $n$. This implies that if there is an edge from $v_{a}$ to $v_{b}$ and an edge from $v_{b}$ to $v_{c}$, then there also must be an edge from $v_{a}$ to $v_{c}$. Therefore, if there are any cycles at all, the smallest cycle must have length 2. But suppose the vertices $v_{a}$ and $v_{b}$ form such a cycle, i. e., $a b-a$ and $a b-b$ are both multiples of $n$. Then $a-b$ is also a multiple of $n$, which can only happen if $a=b$, which is impossible.","tier":0} +{"problem_type":"Number Theory","problem_label":"N1","problem":"AUS A social club has $n$ members. They have the membership numbers $1,2, \\ldots, n$, respectively. From time to time members send presents to other members, including items they have already received as presents from other members. In order to avoid the embarrassing situation that a member might receive a present that he or she has sent to other members, the club adds the following rule to its statutes at one of its annual general meetings: \"A member with membership number $a$ is permitted to send a present to a member with membership number $b$ if and only if $a(b-1)$ is a multiple of $n$.\" Prove that, if each member follows this rule, none will receive a present from another member that he or she has already sent to other members. Alternative formulation: Let $G$ be a directed graph with $n$ vertices $v_{1}, v_{2}, \\ldots, v_{n}$, such that there is an edge going from $v_{a}$ to $v_{b}$ if and only if $a$ and $b$ are distinct and $a(b-1)$ is a multiple of $n$. Prove that this graph does not contain a directed cycle.","solution":"Suppose there was a cycle $v_{i_{1}} \\rightarrow v_{i_{2}} \\rightarrow \\cdots \\rightarrow v_{i_{r}} \\rightarrow v_{i_{1}}$. Then $i_{1}\\left(i_{2}-1\\right)$ is a multiple of $n$, i.e., $i_{1} \\equiv i_{1} i_{2} \\bmod n$. Continuing in this manner, we get $i_{1} \\equiv i_{1} i_{2} \\equiv$ $i_{1} i_{2} i_{3} \\equiv i_{1} i_{2} i_{3} \\ldots i_{r} \\bmod n$. But the same holds for all $i_{k}$, i. e., $i_{k} \\equiv i_{1} i_{2} i_{3} \\ldots i_{r} \\bmod n$. Hence $i_{1} \\equiv i_{2} \\equiv \\cdots \\equiv i_{r} \\bmod n$, which means $i_{1}=i_{2}=\\cdots=i_{r}$, a contradiction.","tier":0} +{"problem_type":"Number Theory","problem_label":"N1","problem":"AUS A social club has $n$ members. They have the membership numbers $1,2, \\ldots, n$, respectively. From time to time members send presents to other members, including items they have already received as presents from other members. In order to avoid the embarrassing situation that a member might receive a present that he or she has sent to other members, the club adds the following rule to its statutes at one of its annual general meetings: \"A member with membership number $a$ is permitted to send a present to a member with membership number $b$ if and only if $a(b-1)$ is a multiple of $n$.\" Prove that, if each member follows this rule, none will receive a present from another member that he or she has already sent to other members. Alternative formulation: Let $G$ be a directed graph with $n$ vertices $v_{1}, v_{2}, \\ldots, v_{n}$, such that there is an edge going from $v_{a}$ to $v_{b}$ if and only if $a$ and $b$ are distinct and $a(b-1)$ is a multiple of $n$. Prove that this graph does not contain a directed cycle.","solution":"Let $n=k$ be the smallest value of $n$ for which the corresponding graph has a cycle. We show that $k$ is a prime power. If $k$ is not a prime power, it can be written as a product $k=d e$ of relatively prime integers greater than 1. Reducing all the numbers modulo $d$ yields a single vertex or a cycle in the corresponding graph on $d$ vertices, because if $a(b-1) \\equiv 0 \\bmod k$ then this equation also holds modulo $d$. But since the graph on $d$ vertices has no cycles, by the minimality of $k$, we must have that all the indices of the cycle are congruent modulo $d$. The same holds modulo $e$ and hence also modulo $k=d e$. But then all the indices are equal, which is a contradiction. Thus $k$ must be a prime power $k=p^{m}$. There are no edges ending at $v_{k}$, so $v_{k}$ is not contained in any cycle. All edges not starting at $v_{k}$ end at a vertex belonging to a non-multiple of $p$, and all edges starting at a non-multiple of $p$ must end at $v_{1}$. But there is no edge starting at $v_{1}$. Hence there is no cycle.","tier":0} +{"problem_type":"Number Theory","problem_label":"N1","problem":"AUS A social club has $n$ members. They have the membership numbers $1,2, \\ldots, n$, respectively. From time to time members send presents to other members, including items they have already received as presents from other members. In order to avoid the embarrassing situation that a member might receive a present that he or she has sent to other members, the club adds the following rule to its statutes at one of its annual general meetings: \"A member with membership number $a$ is permitted to send a present to a member with membership number $b$ if and only if $a(b-1)$ is a multiple of $n$.\" Prove that, if each member follows this rule, none will receive a present from another member that he or she has already sent to other members. Alternative formulation: Let $G$ be a directed graph with $n$ vertices $v_{1}, v_{2}, \\ldots, v_{n}$, such that there is an edge going from $v_{a}$ to $v_{b}$ if and only if $a$ and $b$ are distinct and $a(b-1)$ is a multiple of $n$. Prove that this graph does not contain a directed cycle.","solution":"Suppose there was a cycle $v_{i_{1}} \\rightarrow v_{i_{2}} \\rightarrow \\cdots \\rightarrow v_{i_{r}} \\rightarrow v_{i_{1}}$. Let $q=p^{m}$ be a prime power dividing $n$. We claim that either $i_{1} \\equiv i_{2} \\equiv \\cdots \\equiv i_{r} \\equiv 0 \\bmod q$ or $i_{1} \\equiv i_{2} \\equiv \\cdots \\equiv i_{r} \\equiv$ $1 \\bmod q$. Suppose that there is an $i_{s}$ not divisible by $q$. Then, as $i_{s}\\left(i_{s+1}-1\\right)$ is a multiple of $q, i_{s+1} \\equiv$ $1 \\bmod p$. Similarly, we conclude $i_{s+2} \\equiv 1 \\bmod p$ and so on. So none of the labels is divisible by $p$, but since $i_{s}\\left(i_{s+1}-1\\right)$ is a multiple of $q=p^{m}$ for all $s$, all $i_{s+1}$ are congruent to 1 modulo $q$. This proves the claim. Now, as all the labels are congruent modulo all the prime powers dividing $n$, they must all be equal by the Chinese remainder theorem. This is a contradiction.","tier":0} +{"problem_type":"Number Theory","problem_label":"N2","problem":"PER A positive integer $N$ is called balanced, if $N=1$ or if $N$ can be written as a product of an even number of not necessarily distinct primes. Given positive integers $a$ and $b$, consider the polynomial $P$ defined by $P(x)=(x+a)(x+b)$. (a) Prove that there exist distinct positive integers $a$ and $b$ such that all the numbers $P(1), P(2)$, $\\ldots, P(50)$ are balanced. (b) Prove that if $P(n)$ is balanced for all positive integers $n$, then $a=b$.","solution":"Define a function $f$ on the set of positive integers by $f(n)=0$ if $n$ is balanced and $f(n)=1$ otherwise. Clearly, $f(n m) \\equiv f(n)+f(m) \\bmod 2$ for all positive integers $n, m$. (a) Now for each positive integer $n$ consider the binary sequence $(f(n+1), f(n+2), \\ldots, f(n+$ $50)$ ). As there are only $2^{50}$ different such sequences there are two different positive integers $a$ and $b$ such that $$ (f(a+1), f(a+2), \\ldots, f(a+50))=(f(b+1), f(b+2), \\ldots, f(b+50)) $$ But this implies that for the polynomial $P(x)=(x+a)(x+b)$ all the numbers $P(1), P(2)$, $\\ldots, P(50)$ are balanced, since for all $1 \\leq k \\leq 50$ we have $f(P(k)) \\equiv f(a+k)+f(b+k) \\equiv$ $2 f(a+k) \\equiv 0 \\bmod 2$. (b) Now suppose $P(n)$ is balanced for all positive integers $n$ and $av_{p_{i}}(f(1))$ for all $i=1,2, \\ldots, m$, e.g. $a=\\left(p_{1} p_{2} \\ldots p_{m}\\right)^{\\alpha}$ with $\\alpha$ sufficiently large. Pick any such $a$. The condition of the problem then yields $a \\mid(f(a+1)-f(1))$. Assume $f(a+1) \\neq f(1)$. Then we must have $v_{p_{i}}(f(a+1)) \\neq$ $v_{p_{i}}(f(1))$ for at least one $i$. This yields $v_{p_{i}}(f(a+1)-f(1))=\\min \\left\\{v_{p_{i}}(f(a+1)), v_{p_{i}}(f(1))\\right\\} \\leq$ $v_{p_{1}}(f(1))p_{1}^{\\alpha_{1}+1} p_{2}^{\\alpha_{2}+1} \\ldots p_{m}^{\\alpha_{m}+1} \\cdot(f(r)+r)-r \\\\ & \\geq p_{1}^{\\alpha_{1}+1} p_{2}^{\\alpha_{2}+1} \\ldots p_{m}^{\\alpha_{m}+1}+(f(r)+r)-r \\\\ & >p_{1}^{\\alpha_{1}} p_{2}^{\\alpha_{2}} \\ldots p_{m}^{\\alpha_{m}}+f(r) \\\\ & \\geq|f(M)-f(r)| . \\end{aligned} $$ But since $M-r$ divides $f(M)-f(r)$ this can only be true if $f(r)=f(M)=f(1)$, which contradicts the choice of $r$. Comment. In the case that $f$ is a polynomial with integer coefficients the result is well-known, see e.g. W. Schwarz, Einf\u00fchrung in die Methoden der Primzahltheorie, 1969.","tier":0} +{"problem_type":"Number Theory","problem_label":"N4","problem":"PRK Find all positive integers $n$ such that there exists a sequence of positive integers $a_{1}, a_{2}, \\ldots, a_{n}$ satisfying $$ a_{k+1}=\\frac{a_{k}^{2}+1}{a_{k-1}+1}-1 $$ for every $k$ with $2 \\leq k \\leq n-1$.","solution":"Such a sequence exists for $n=1,2,3,4$ and no other $n$. Since the existence of such a sequence for some $n$ implies the existence of such a sequence for all smaller $n$, it suffices to prove that $n=5$ is not possible and $n=4$ is possible. Assume first that for $n=5$ there exists a sequence of positive integers $a_{1}, a_{2}, \\ldots, a_{5}$ satisfying the conditions $$ \\begin{aligned} & a_{2}^{2}+1=\\left(a_{1}+1\\right)\\left(a_{3}+1\\right), \\\\ & a_{3}^{2}+1=\\left(a_{2}+1\\right)\\left(a_{4}+1\\right), \\\\ & a_{4}^{2}+1=\\left(a_{3}+1\\right)\\left(a_{5}+1\\right) . \\end{aligned} $$ Assume $a_{1}$ is odd, then $a_{2}$ has to be odd as well and as then $a_{2}^{2}+1 \\equiv 2 \\bmod 4, a_{3}$ has to be even. But this is a contradiction, since then the even number $a_{2}+1$ cannot divide the odd number $a_{3}^{2}+1$. Hence $a_{1}$ is even. If $a_{2}$ is odd, $a_{3}^{2}+1$ is even (as a multiple of $a_{2}+1$ ) and hence $a_{3}$ is odd, too. Similarly we must have $a_{4}$ odd as well. But then $a_{3}^{2}+1$ is a product of two even numbers $\\left(a_{2}+1\\right)\\left(a_{4}+1\\right)$ and thus is divisible by 4 , which is a contradiction as for odd $a_{3}$ we have $a_{3}^{2}+1 \\equiv 2 \\bmod 4$. Hence $a_{2}$ is even. Furthermore $a_{3}+1$ divides the odd number $a_{2}^{2}+1$ and so $a_{3}$ is even. Similarly, $a_{4}$ and $a_{5}$ are even as well. Now set $x=a_{2}$ and $y=a_{3}$. From the given condition we get $(x+1) \\mid\\left(y^{2}+1\\right)$ and $(y+1) \\mid\\left(x^{2}+1\\right)$. We will prove that there is no pair of positive even numbers $(x, y)$ satisfying these two conditions, thus yielding a contradiction to the assumption. Assume there exists a pair $\\left(x_{0}, y_{0}\\right)$ of positive even numbers satisfying the two conditions $\\left(x_{0}+1\\right) \\mid\\left(y_{0}^{2}+1\\right)$ and $\\left(y_{0}+1\\right) \\mid\\left(x_{0}^{2}+1\\right)$. Then one has $\\left(x_{0}+1\\right) \\mid\\left(y_{0}^{2}+1+x_{0}^{2}-1\\right)$, i.e., $\\left(x_{0}+1\\right) \\mid\\left(x_{0}^{2}+y_{0}^{2}\\right)$, and similarly $\\left(y_{0}+1\\right) \\mid\\left(x_{0}^{2}+y_{0}^{2}\\right)$. Any common divisor $d$ of $x_{0}+1$ and $y_{0}+1$ must hence also divide the number $\\left(x_{0}^{2}+1\\right)+\\left(y_{0}^{2}+1\\right)-\\left(x_{0}^{2}+y_{0}^{2}\\right)=2$. But as $x_{0}+1$ and $y_{0}+1$ are both odd, we must have $d=1$. Thus $x_{0}+1$ and $y_{0}+1$ are relatively prime and therefore there exists a positive integer $k$ such that $$ k(x+1)(y+1)=x^{2}+y^{2} $$ has the solution $\\left(x_{0}, y_{0}\\right)$. We will show that the latter equation has no solution $(x, y)$ in positive even numbers. Assume there is a solution. Pick the solution $\\left(x_{1}, y_{1}\\right)$ with the smallest sum $x_{1}+y_{1}$ and assume $x_{1} \\geq y_{1}$. Then $x_{1}$ is a solution to the quadratic equation $$ x^{2}-k\\left(y_{1}+1\\right) x+y_{1}^{2}-k\\left(y_{1}+1\\right)=0 . $$ Let $x_{2}$ be the second solution, which by Vieta's theorem fulfills $x_{1}+x_{2}=k\\left(y_{1}+1\\right)$ and $x_{1} x_{2}=y_{1}^{2}-k\\left(y_{1}+1\\right)$. If $x_{2}=0$, the second equation implies $y_{1}^{2}=k\\left(y_{1}+1\\right)$, which is impossible, as $y_{1}+1>1$ cannot divide the relatively prime number $y_{1}^{2}$. Therefore $x_{2} \\neq 0$. Also we get $\\left(x_{1}+1\\right)\\left(x_{2}+1\\right)=x_{1} x_{2}+x_{1}+x_{2}+1=y_{1}^{2}+1$ which is odd, and hence $x_{2}$ must be even and positive. Also we have $x_{2}+1=\\frac{y_{1}^{2}+1}{x_{1}+1} \\leq \\frac{y_{1}^{2}+1}{y_{1}+1} \\leq y_{1} \\leq x_{1}$. But this means that the pair $\\left(x^{\\prime}, y^{\\prime}\\right)$ with $x^{\\prime}=y_{1}$ and $y^{\\prime}=x_{2}$ is another solution of $k(x+1)(y+1)=x^{2}+y^{2}$ in even positive numbers with $x^{\\prime}+y^{\\prime}y $$ and similarly in the other case. Now, if $a_{3}$ was odd, then $\\left(a_{2}+1\\right)\\left(a_{4}+1\\right)=a_{3}^{2}+1 \\equiv 2 \\bmod 4$ would imply that one of $a_{2}$ or $a_{4}$ is even, this contradicts (1). Thus $a_{3}$ and hence also $a_{1}, a_{2}, a_{4}$ and $a_{5}$ are even. According to (1), one has $a_{1}a_{2}>a_{3}>a_{4}>a_{5}$ but due to the minimality of $a_{1}$ the first series of inequalities must hold. Consider the identity $\\left(a_{3}+1\\right)\\left(a_{1}+a_{3}\\right)=a_{3}^{2}-1+\\left(a_{1}+1\\right)\\left(a_{3}+1\\right)=a_{2}^{2}+a_{3}^{2}=a_{2}^{2}-1+\\left(a_{2}+1\\right)\\left(a_{4}+1\\right)=\\left(a_{2}+1\\right)\\left(a_{2}+a_{4}\\right)$. Any common divisor of the two odd numbers $a_{2}+1$ and $a_{3}+1$ must also divide $\\left(a_{2}+1\\right)\\left(a_{4}+\\right.$ $1)-\\left(a_{3}+1\\right)\\left(a_{3}-1\\right)=2$, so these numbers are relatively prime. Hence the last identity shows that $a_{1}+a_{3}$ must be a multiple of $a_{2}+1$, i.e. there is an integer $k$ such that $$ a_{1}+a_{3}=k\\left(a_{2}+1\\right) $$ Now set $a_{0}=k\\left(a_{1}+1\\right)-a_{2}$. This is an integer and we have $$ \\begin{aligned} \\left(a_{0}+1\\right)\\left(a_{2}+1\\right) & =k\\left(a_{1}+1\\right)\\left(a_{2}+1\\right)-\\left(a_{2}-1\\right)\\left(a_{2}+1\\right) \\\\ & =\\left(a_{1}+1\\right)\\left(a_{1}+a_{3}\\right)-\\left(a_{1}+1\\right)\\left(a_{3}+1\\right)+2 \\\\ & =\\left(a_{1}+1\\right)\\left(a_{1}-1\\right)+2=a_{1}^{2}+1 \\end{aligned} $$ Thus $a_{0} \\geq 0$. If $a_{0}>0$, then by (1) we would have $a_{0}1$ is relatively prime to $a_{1}^{2}$ and thus cannot be a divisior of this number. Hence $n \\geq 5$ is not possible. Comment 1. Finding the example for $n=4$ is not trivial and requires a tedious calculation, but it can be reduced to checking a few cases. The equations $\\left(a_{1}+1\\right)\\left(a_{3}+1\\right)=a_{2}^{2}+1$ and $\\left(a_{2}+1\\right)\\left(a_{4}+1\\right)=a_{3}^{2}+1$ imply, as seen in the proof, that $a_{1}$ is even and $a_{2}, a_{3}, a_{4}$ are odd. The case $a_{1}=2$ yields $a_{2}^{2} \\equiv-1 \\bmod 3$ which is impossible. Hence $a_{1}=4$ is the smallest possibility. In this case $a_{2}^{2} \\equiv-1 \\bmod 5$ and $a_{2}$ is odd, which implies $a_{2} \\equiv 3$ or $a_{2} \\equiv 7 \\bmod 10$. Hence we have to start checking $a_{2}=7,13,17,23,27,33$ and in the last case we succeed. Comment 2. The choice of $a_{0}=k\\left(a_{1}+1\\right)-a_{2}$ in the second solution appears more natural if one considers that by the previous calculations one has $a_{1}=k\\left(a_{2}+1\\right)-a_{3}$ and $a_{2}=k\\left(a_{3}+1\\right)-a_{4}$. Alternatively, one can solve the equation (2) for $a_{3}$ and use $a_{2}^{2}+1=\\left(a_{1}+1\\right)\\left(a_{3}+1\\right)$ to get $a_{2}^{2}-k\\left(a_{1}+1\\right) a_{2}+a_{1}^{2}-k\\left(a_{1}+1\\right)=0$. Now $a_{0}$ is the second solution to this quadratic equation in $a_{2}$ (Vieta jumping).","tier":0} +{"problem_type":"Number Theory","problem_label":"N5","problem":"HUN Let $P(x)$ be a non-constant polynomial with integer coefficients. Prove that there is no function $T$ from the set of integers into the set of integers such that the number of integers $x$ with $T^{n}(x)=x$ is equal to $P(n)$ for every $n \\geq 1$, where $T^{n}$ denotes the $n$-fold application of $T$.","solution":"Assume there is a polynomial $P$ of degree at least 1 with the desired property for a given function $T$. Let $A(n)$ denote the set of all $x \\in \\mathbb{Z}$ such that $T^{n}(x)=x$ and let $B(n)$ denote the set of all $x \\in \\mathbb{Z}$ for which $T^{n}(x)=x$ and $T^{k}(x) \\neq x$ for all $1 \\leq k\\sqrt{a b}, b^{\\ell_{0}}>\\sqrt{a b}$, we define the polynomial $$ P(x)=\\prod_{k=0, \\ell=0}^{k_{0}-1, \\ell_{0}-1}\\left(a^{k} b^{\\ell} x-\\sqrt{a b}\\right)=: \\sum_{i=0}^{k_{0} \\cdot \\ell_{0}} d_{i} x^{i} $$ with integer coefficients $d_{i}$. By our assumption, the zeros $$ \\frac{\\sqrt{a b}}{a^{k} b^{\\ell}}, \\quad k=0, \\ldots, k_{0}-1, \\quad \\ell=0, \\ldots, \\ell_{0}-1 $$ of $P$ are pairwise distinct. Furthermore, we consider the integer sequence $$ y_{n}=\\sum_{i=0}^{k_{0} \\cdot \\ell_{0}} d_{i} x_{n+i}, \\quad n=1,2, \\ldots $$ By the theory of linear recursions, we obtain $$ y_{n}=\\sum_{\\substack{k, \\ell \\geq 0 \\\\ k \\geq k_{0} \\text { or } \\ell \\geq \\ell_{0}}} e_{k, \\ell}\\left(\\frac{\\sqrt{a b}}{a^{k} b^{\\ell}}\\right)^{n}, \\quad n=1,2, \\ldots $$ with real numbers $e_{k, \\ell}$. We have $$ \\left|y_{n}\\right| \\leq \\sum_{\\substack{k, \\ell \\geq 0 \\\\ k \\geq k_{0} \\text { or } \\ell \\geq \\ell_{0}}}\\left|e_{k, \\ell}\\right|\\left(\\frac{\\sqrt{a b}}{a^{k} b^{\\ell}}\\right)^{n}=: M_{n} . $$ Because the series in (4) is obtained by a finite linear combination of the absolutely convergent series (1), we conclude that in particular $M_{1}<\\infty$. Since $$ \\frac{\\sqrt{a b}}{a^{k} b^{\\ell}} \\leq \\lambda:=\\max \\left\\{\\frac{\\sqrt{a b}}{a^{k_{0}}}, \\frac{\\sqrt{a b}}{b^{\\ell_{0}}}\\right\\} \\quad \\text { for all } k, \\ell \\geq 0 \\text { such that } k \\geq k_{0} \\text { or } \\ell \\geq \\ell_{0} $$ we get the estimates $M_{n+1} \\leq \\lambda M_{n}, n=1,2, \\ldots$ Our choice of $k_{0}$ and $\\ell_{0}$ ensures $\\lambda<1$, which implies $M_{n} \\rightarrow 0$ and consequently $y_{n} \\rightarrow 0$ as $n \\rightarrow \\infty$. It follows that $y_{n}=0$ for all sufficiently large $n$. So, equation (3) reduces to $\\sum_{i=0}^{k_{0} \\cdot \\ell_{0}} d_{i} x_{n+i}=0$. Using the theory of linear recursions again, for sufficiently large $n$ we have $$ x_{n}=\\sum_{k=0, \\ell=0}^{k_{0}-1, \\ell_{0}-1} f_{k, \\ell}\\left(\\frac{\\sqrt{a b}}{a^{k} b^{\\ell}}\\right)^{n} $$ for certain real numbers $f_{k, \\ell}$. Comparing with (2), we see that $f_{k, \\ell}=c_{k, \\ell}$ for all $k, \\ell \\geq 0$ with $kj_{0}$. But this means that $$ \\left(1-x^{\\mu}\\right)^{\\frac{1}{2}}\\left(1-x^{\\nu}\\right)^{\\frac{1}{2}}=\\sum_{j=0}^{j_{0}} g_{j} x^{j} $$ for all real numbers $x \\in(0,1)$. Squaring, we see that $$ \\left(1-x^{\\mu}\\right)\\left(1-x^{\\nu}\\right) $$ is the square of a polynomial in $x$. In particular, all its zeros are of order at least 2 , which implies $\\mu=\\nu$ by looking at roots of unity. So we obtain $\\mu=\\nu=1$, i. e., $a=b$, a contradiction.","tier":0} +{"problem_type":"Number Theory","problem_label":"N7","problem":"MNG Let $a$ and $b$ be distinct integers greater than 1 . Prove that there exists a positive integer $n$ such that $\\left(a^{n}-1\\right)\\left(b^{n}-1\\right)$ is not a perfect square.","solution":"We set $a^{2}=A, b^{2}=B$, and $z_{n}=\\sqrt{\\left(A^{n}-1\\right)\\left(B^{n}-1\\right)}$. Let us assume that $z_{n}$ is an integer for $n=1,2, \\ldots$. Without loss of generality, we may suppose that $b0$ as $z_{n}>0$. As before, one obtains $$ \\begin{aligned} & A^{n} B^{n}-A^{n}-B^{n}+1=z_{n}^{2} \\\\ & =\\left\\{\\delta_{0}(a b)^{n}-\\delta_{1}\\left(\\frac{a}{b}\\right)^{n}-\\delta_{2}\\left(\\frac{a}{b^{3}}\\right)^{n}-\\cdots-\\delta_{k}\\left(\\frac{a}{b^{2 k-1}}\\right)^{n}\\right\\}^{2} \\\\ & =\\delta_{0}^{2} A^{n} B^{n}-2 \\delta_{0} \\delta_{1} A^{n}-\\sum_{i=2}^{i=k}\\left(2 \\delta_{0} \\delta_{i}-\\sum_{j=1}^{j=i-1} \\delta_{j} \\delta_{i-j}\\right)\\left(\\frac{A}{B^{i-1}}\\right)^{n}+O\\left(\\frac{A}{B^{k}}\\right)^{n} \\end{aligned} $$ Easy asymptotic calculations yield $\\delta_{0}=1, \\delta_{1}=\\frac{1}{2}, \\delta_{i}=\\frac{1}{2} \\sum_{j=1}^{j=i-1} \\delta_{j} \\delta_{i-j}$ for $i=2,3, \\ldots, k-2$, and then $a=b^{k-1}$. It follows that $k>2$ and there is some $P \\in \\mathbb{Q}[X]$ for which $(X-1)\\left(X^{k-1}-1\\right)=$ $P(X)^{2}$. But this cannot occur, for instance as $X^{k-1}-1$ has no double zeros. Thus our assumption that $z_{n}$ was an integer for $n=1,2, \\ldots$ turned out to be wrong, which solves the problem. Original formulation of the problem. $a, b$ are positive integers such that $a \\cdot b$ is not a square of an integer. Prove that there exists a (infinitely many) positive integer $n$ such that ( $\\left.a^{n}-1\\right)\\left(b^{n}-1\\right)$ is not a square of an integer.","tier":0} +{"problem_type":"Number Theory","problem_label":"N7","problem":"MNG Let $a$ and $b$ be distinct integers greater than 1 . Prove that there exists a positive integer $n$ such that $\\left(a^{n}-1\\right)\\left(b^{n}-1\\right)$ is not a perfect square.","solution":"Lemma. Let $c$ be a positive integer, which is not a perfect square. Then there exists an odd prime $p$ such that $c$ is not a quadratic residue modulo $p$. Proof. Denoting the square-free part of $c$ by $c^{\\prime}$, we have the equality $\\left(\\frac{c^{\\prime}}{p}\\right)=\\left(\\frac{c}{p}\\right)$ of the corresponding LegEndre symbols. Suppose that $c^{\\prime}=q_{1} \\cdots q_{m}$, where $q_{1}<\\cdotsw_{2009}$ holds. Evidently, there exists a triangle with side lengths $w, b, r$ for the white, blue and red side, respectively, such that $w_{2009}=w$. By the conditions of the problem we have $b+r>w, b_{2009} \\geq b$ and $r_{2009} \\geq r$. From these inequalities it follows $$ b_{2009}+r_{2009} \\geq b+r>w=w_{2009} $$ Secondly we will describe a sequence of triangles for which $w_{j}, b_{j}, r_{j}$ with $j<2009$ are not the lengths of the sides of a triangle. Let us define the sequence $\\Delta_{j}, j=1,2, \\ldots, 2009$, of triangles, where $\\Delta_{j}$ has a blue side of length $2 j$, a red side of length $j$ for all $j \\leq 2008$ and 4018 for $j=2009$, and a white side of length $j+1$ for all $j \\leq 2007$, 4018 for $j=2008$ and 1 for $j=2009$. Since $$ \\begin{array}{rrrl} (j+1)+j>2 j & \\geq j+1>j, & \\text { if } & j \\leq 2007 \\\\ 2 j+j>4018>2 j \\quad>j, & \\text { if } & j=2008 \\\\ 4018+1>2 j & =4018>1, & \\text { if } & j=2009 \\end{array} $$ such a sequence of triangles exists. Moreover, $w_{j}=j, r_{j}=j$ and $b_{j}=2 j$ for $1 \\leq j \\leq 2008$. Then $$ w_{j}+r_{j}=j+j=2 j=b_{j} $$ i.e., $b_{j}, r_{j}$ and $w_{j}$ are not the lengths of the sides of a triangle for $1 \\leq j \\leq 2008$.","tier":0} +{"problem_type":"Algebra","problem_label":"A3","problem":"FRA Determine all functions $f$ from the set of positive integers into the set of positive integers such that for all $x$ and $y$ there exists a non degenerated triangle with sides of lengths $$ x, \\quad f(y) \\quad \\text { and } \\quad f(y+f(x)-1) . $$","solution":"The identity function $f(x)=x$ is the only solution of the problem. If $f(x)=x$ for all positive integers $x$, the given three lengths are $x, y=f(y)$ and $z=$ $f(y+f(x)-1)=x+y-1$. Because of $x \\geq 1, y \\geq 1$ we have $z \\geq \\max \\{x, y\\}>|x-y|$ and $z1$ we would conclude $f(y)=f(y+m)$ for all $y$ considering the triangle with the side lengths $1, f(y)$ and $f(y+m)$. Thus, $f$ would be $m$-periodic and, consequently, bounded. Let $B$ be a bound, $f(x) \\leq B$. If we choose $x>2 B$ we obtain the contradiction $x>2 B \\geq f(y)+f(y+f(x)-1)$. Step 2. For all positive integers $z$, we have $f(f(z))=z$. Setting $x=z$ and $y=1$ this follows immediately from Step 1. Step 3. For all integers $z \\geq 1$, we have $f(z) \\leq z$. Let us show, that the contrary leads to a contradiction. Assume $w+1=f(z)>z$ for some $z$. From Step 1 we know that $w \\geq z \\geq 2$. Let $M=\\max \\{f(1), f(2), \\ldots, f(w)\\}$ be the largest value of $f$ for the first $w$ integers. First we show, that no positive integer $t$ exists with $$ f(t)>\\frac{z-1}{w} \\cdot t+M $$ otherwise we decompose the smallest value $t$ as $t=w r+s$ where $r$ is an integer and $1 \\leq s \\leq w$. Because of the definition of $M$, we have $t>w$. Setting $x=z$ and $y=t-w$ we get from the triangle inequality $$ z+f(t-w)>f((t-w)+f(z)-1)=f(t-w+w)=f(t) $$ Hence, $$ f(t-w) \\geq f(t)-(z-1)>\\frac{z-1}{w}(t-w)+M $$ a contradiction to the minimality of $t$. Therefore the inequality (1) fails for all $t \\geq 1$, we have proven $$ f(t) \\leq \\frac{z-1}{w} \\cdot t+M $$ instead. Now, using (2), we finish the proof of Step 3. Because of $z \\leq w$ we have $\\frac{z-1}{w}<1$ and we can choose an integer $t$ sufficiently large to fulfill the condition $$ \\left(\\frac{z-1}{w}\\right)^{2} t+\\left(\\frac{z-1}{w}+1\\right) M0$ and let the assertion be true for all nonnegative integers less than $n$. Moreover let $a_{1}, a_{2}, \\ldots, a_{n+1}, s$ and $M$ be given as in the problem. Without loss of generality we may assume that $a_{n+1}1$. If $T_{\\bar{k}-1} \\in M$, then (4.) follows immediately by the minimality of $\\bar{k}$. If $T_{\\bar{k}-1} \\notin M$, by the smoothness of $\\bar{k}-1$, we obtain a situation as in Claim 1 with $m=\\bar{k}-1$ provided that $\\mid M \\cap\\left(0, T_{\\bar{k}-1}| | \\geq \\bar{k}-1\\right.$. Hence, we may even restrict ourselves to $\\mid M \\cap\\left(0, T_{\\bar{k}-1} \\mid \\leq \\bar{k}-2\\right.$ in this case and Claim 3 is proved. Choose an integer $v \\geq 0$ with $\\left|M \\cap\\left(0, T_{\\bar{k}}\\right)\\right|=\\bar{k}+v$. Let $r_{1}>r_{2}>\\cdots>r_{l}$ be exactly those indices $r$ from $\\{\\bar{k}+1, \\bar{k}+2, \\ldots, n+1\\}$ for which $T_{\\bar{k}}+a_{r} \\notin M$. Then $$ n=|M|=\\left|M \\cap\\left(0, T_{\\bar{k}}\\right)\\right|+1+\\left|M \\cap\\left(T_{\\bar{k}}, s\\right)\\right| \\geq \\bar{k}+v+1+(n+1-\\bar{k}-l) $$ and consequently $l \\geq v+2$. Note that $T_{\\bar{k}}+a_{r_{1}}-a_{1}1$ ); so, the expression $\\frac{4}{3}(3-d)(d-1)\\left(d^{2}-3\\right)$ in the right-hand part of $(2)$ is nonnegative, and the desired inequality is proved. Comment. The rightmost inequality is easier than the leftmost one. In particular, Solutions 2 and 3 show that only the condition $a^{2}+b^{2}+c^{2}+d^{2}=12$ is needed for the former one.","tier":0} +{"problem_type":"Algebra","problem_label":"A3","problem":"Let $x_{1}, \\ldots, x_{100}$ be nonnegative real numbers such that $x_{i}+x_{i+1}+x_{i+2} \\leq 1$ for all $i=1, \\ldots, 100$ (we put $x_{101}=x_{1}, x_{102}=x_{2}$ ). Find the maximal possible value of the sum $$ S=\\sum_{i=1}^{100} x_{i} x_{i+2} $$ (Russia) Answer. $\\frac{25}{2}$.","solution":"Let $x_{2 i}=0, x_{2 i-1}=\\frac{1}{2}$ for all $i=1, \\ldots, 50$. Then we have $S=50 \\cdot\\left(\\frac{1}{2}\\right)^{2}=\\frac{25}{2}$. So, we are left to show that $S \\leq \\frac{25}{2}$ for all values of $x_{i}$ 's satisfying the problem conditions. Consider any $1 \\leq i \\leq 50$. By the problem condition, we get $x_{2 i-1} \\leq 1-x_{2 i}-x_{2 i+1}$ and $x_{2 i+2} \\leq 1-x_{2 i}-x_{2 i+1}$. Hence by the AM-GM inequality we get $$ \\begin{aligned} x_{2 i-1} x_{2 i+1} & +x_{2 i} x_{2 i+2} \\leq\\left(1-x_{2 i}-x_{2 i+1}\\right) x_{2 i+1}+x_{2 i}\\left(1-x_{2 i}-x_{2 i+1}\\right) \\\\ & =\\left(x_{2 i}+x_{2 i+1}\\right)\\left(1-x_{2 i}-x_{2 i+1}\\right) \\leq\\left(\\frac{\\left(x_{2 i}+x_{2 i+1}\\right)+\\left(1-x_{2 i}-x_{2 i+1}\\right)}{2}\\right)^{2}=\\frac{1}{4} \\end{aligned} $$ Summing up these inequalities for $i=1,2, \\ldots, 50$, we get the desired inequality $$ \\sum_{i=1}^{50}\\left(x_{2 i-1} x_{2 i+1}+x_{2 i} x_{2 i+2}\\right) \\leq 50 \\cdot \\frac{1}{4}=\\frac{25}{2} $$ Comment. This solution shows that a bit more general fact holds. Namely, consider $2 n$ nonnegative numbers $x_{1}, \\ldots, x_{2 n}$ in a row (with no cyclic notation) and suppose that $x_{i}+x_{i+1}+x_{i+2} \\leq 1$ for all $i=1,2, \\ldots, 2 n-2$. Then $\\sum_{i=1}^{2 n-2} x_{i} x_{i+2} \\leq \\frac{n-1}{4}$. The proof is the same as above, though if might be easier to find it (for instance, applying induction). The original estimate can be obtained from this version by considering the sequence $x_{1}, x_{2}, \\ldots, x_{100}, x_{1}, x_{2}$.","tier":0} +{"problem_type":"Algebra","problem_label":"A3","problem":"Let $x_{1}, \\ldots, x_{100}$ be nonnegative real numbers such that $x_{i}+x_{i+1}+x_{i+2} \\leq 1$ for all $i=1, \\ldots, 100$ (we put $x_{101}=x_{1}, x_{102}=x_{2}$ ). Find the maximal possible value of the sum $$ S=\\sum_{i=1}^{100} x_{i} x_{i+2} $$ (Russia) Answer. $\\frac{25}{2}$.","solution":"We present another proof of the estimate. From the problem condition, we get $$ \\begin{aligned} S=\\sum_{i=1}^{100} x_{i} x_{i+2} \\leq \\sum_{i=1}^{100} x_{i}\\left(1-x_{i}-x_{i+1}\\right) & =\\sum_{i=1}^{100} x_{i}-\\sum_{i=1}^{100} x_{i}^{2}-\\sum_{i=1}^{100} x_{i} x_{i+1} \\\\ & =\\sum_{i=1}^{100} x_{i}-\\frac{1}{2} \\sum_{i=1}^{100}\\left(x_{i}+x_{i+1}\\right)^{2} \\end{aligned} $$ By the AM-QM inequality, we have $\\sum\\left(x_{i}+x_{i+1}\\right)^{2} \\geq \\frac{1}{100}\\left(\\sum\\left(x_{i}+x_{i+1}\\right)\\right)^{2}$, so $$ \\begin{aligned} S \\leq \\sum_{i=1}^{100} x_{i}-\\frac{1}{200}\\left(\\sum_{i=1}^{100}\\left(x_{i}+x_{i+1}\\right)\\right)^{2} & =\\sum_{i=1}^{100} x_{i}-\\frac{2}{100}\\left(\\sum_{i=1}^{100} x_{i}\\right)^{2} \\\\ & =\\frac{2}{100}\\left(\\sum_{i=1}^{100} x_{i}\\right)\\left(\\frac{100}{2}-\\sum_{i=1}^{100} x_{i}\\right) \\end{aligned} $$ And finally, by the AM-GM inequality $$ S \\leq \\frac{2}{100} \\cdot\\left(\\frac{1}{2}\\left(\\sum_{i=1}^{100} x_{i}+\\frac{100}{2}-\\sum_{i=1}^{100} x_{i}\\right)\\right)^{2}=\\frac{2}{100} \\cdot\\left(\\frac{100}{4}\\right)^{2}=\\frac{25}{2} $$ Comment. These solutions are not as easy as they may seem at the first sight. There are two different optimal configurations in which the variables have different values, and not all of sums of three consecutive numbers equal 1. Although it is easy to find the value $\\frac{25}{2}$, the estimates must be done with care to preserve equality in the optimal configurations.","tier":0} +{"problem_type":"Algebra","problem_label":"A4","problem":"A sequence $x_{1}, x_{2}, \\ldots$ is defined by $x_{1}=1$ and $x_{2 k}=-x_{k}, x_{2 k-1}=(-1)^{k+1} x_{k}$ for all $k \\geq 1$. Prove that $x_{1}+x_{2}+\\cdots+x_{n} \\geq 0$ for all $n \\geq 1$. (Austria)","solution":"We start with some observations. First, from the definition of $x_{i}$ it follows that for each positive integer $k$ we have $$ x_{4 k-3}=x_{2 k-1}=-x_{4 k-2} \\quad \\text { and } \\quad x_{4 k-1}=x_{4 k}=-x_{2 k}=x_{k} $$ Hence, denoting $S_{n}=\\sum_{i=1}^{n} x_{i}$, we have $$ \\begin{gathered} S_{4 k}=\\sum_{i=1}^{k}\\left(\\left(x_{4 k-3}+x_{4 k-2}\\right)+\\left(x_{4 k-1}+x_{4 k}\\right)\\right)=\\sum_{i=1}^{k}\\left(0+2 x_{k}\\right)=2 S_{k}, \\\\ S_{4 k+2}=S_{4 k}+\\left(x_{4 k+1}+x_{4 k+2}\\right)=S_{4 k} . \\end{gathered} $$ Observe also that $S_{n}=\\sum_{i=1}^{n} x_{i} \\equiv \\sum_{i=1}^{n} 1=n(\\bmod 2)$. Now we prove by induction on $k$ that $S_{i} \\geq 0$ for all $i \\leq 4 k$. The base case is valid since $x_{1}=x_{3}=x_{4}=1, x_{2}=-1$. For the induction step, assume that $S_{i} \\geq 0$ for all $i \\leq 4 k$. Using the relations (1)-(3), we obtain $$ S_{4 k+4}=2 S_{k+1} \\geq 0, \\quad S_{4 k+2}=S_{4 k} \\geq 0, \\quad S_{4 k+3}=S_{4 k+2}+x_{4 k+3}=\\frac{S_{4 k+2}+S_{4 k+4}}{2} \\geq 0 $$ So, we are left to prove that $S_{4 k+1} \\geq 0$. If $k$ is odd, then $S_{4 k}=2 S_{k} \\geq 0$; since $k$ is odd, $S_{k}$ is odd as well, so we have $S_{4 k} \\geq 2$ and hence $S_{4 k+1}=S_{4 k}+x_{4 k+1} \\geq 1$. Conversely, if $k$ is even, then we have $x_{4 k+1}=x_{2 k+1}=x_{k+1}$, hence $S_{4 k+1}=S_{4 k}+x_{4 k+1}=$ $2 S_{k}+x_{k+1}=S_{k}+S_{k+1} \\geq 0$. The step is proved.","tier":0} +{"problem_type":"Algebra","problem_label":"A4","problem":"A sequence $x_{1}, x_{2}, \\ldots$ is defined by $x_{1}=1$ and $x_{2 k}=-x_{k}, x_{2 k-1}=(-1)^{k+1} x_{k}$ for all $k \\geq 1$. Prove that $x_{1}+x_{2}+\\cdots+x_{n} \\geq 0$ for all $n \\geq 1$. (Austria)","solution":"We will use the notation of $S_{n}$ and the relations (1)-(3) from the previous solution. Assume the contrary and consider the minimal $n$ such that $S_{n+1}<0$; surely $n \\geq 1$, and from $S_{n} \\geq 0$ we get $S_{n}=0, x_{n+1}=-1$. Hence, we are especially interested in the set $M=\\left\\{n: S_{n}=0\\right\\}$; our aim is to prove that $x_{n+1}=1$ whenever $n \\in M$ thus coming to a contradiction. For this purpose, we first describe the set $M$ inductively. We claim that (i) $M$ consists only of even numbers, (ii) $2 \\in M$, and (iii) for every even $n \\geq 4$ we have $n \\in M \\Longleftrightarrow[n \/ 4] \\in M$. Actually, (i) holds since $S_{n} \\equiv n(\\bmod 2)$, (ii) is straightforward, while (iii) follows from the relations $S_{4 k+2}=S_{4 k}=2 S_{k}$. Now, we are left to prove that $x_{n+1}=1$ if $n \\in M$. We use the induction on $n$. The base case is $n=2$, that is, the minimal element of $M$; here we have $x_{3}=1$, as desired. For the induction step, consider some $4 \\leq n \\in M$ and let $m=[n \/ 4] \\in M$; then $m$ is even, and $x_{m+1}=1$ by the induction hypothesis. We prove that $x_{n+1}=x_{m+1}=1$. If $n=4 m$ then we have $x_{n+1}=x_{2 m+1}=x_{m+1}$ since $m$ is even; otherwise, $n=4 m+2$, and $x_{n+1}=-x_{2 m+2}=x_{m+1}$, as desired. The proof is complete. Comment. Using the inductive definition of set $M$, one can describe it explicitly. Namely, $M$ consists exactly of all positive integers not containing digits 1 and 3 in their 4 -base representation.","tier":0} +{"problem_type":"Algebra","problem_label":"A5","problem":"Denote by $\\mathbb{Q}^{+}$the set of all positive rational numbers. Determine all functions $f: \\mathbb{Q}^{+} \\rightarrow \\mathbb{Q}^{+}$ which satisfy the following equation for all $x, y \\in \\mathbb{Q}^{+}$: $$ f\\left(f(x)^{2} y\\right)=x^{3} f(x y) $$ (Switzerland) Answer. The only such function is $f(x)=\\frac{1}{x}$.","solution":"By substituting $y=1$, we get $$ f\\left(f(x)^{2}\\right)=x^{3} f(x) $$ Then, whenever $f(x)=f(y)$, we have $$ x^{3}=\\frac{f\\left(f(x)^{2}\\right)}{f(x)}=\\frac{f\\left(f(y)^{2}\\right)}{f(y)}=y^{3} $$ which implies $x=y$, so the function $f$ is injective. Now replace $x$ by $x y$ in (2), and apply (1) twice, second time to $\\left(y, f(x)^{2}\\right)$ instead of $(x, y)$ : $$ f\\left(f(x y)^{2}\\right)=(x y)^{3} f(x y)=y^{3} f\\left(f(x)^{2} y\\right)=f\\left(f(x)^{2} f(y)^{2}\\right) $$ Since $f$ is injective, we get $$ \\begin{aligned} f(x y)^{2} & =f(x)^{2} f(y)^{2} \\\\ f(x y) & =f(x) f(y) . \\end{aligned} $$ Therefore, $f$ is multiplicative. This also implies $f(1)=1$ and $f\\left(x^{n}\\right)=f(x)^{n}$ for all integers $n$. Then the function equation (1) can be re-written as $$ \\begin{aligned} f(f(x))^{2} f(y) & =x^{3} f(x) f(y), \\\\ f(f(x)) & =\\sqrt{x^{3} f(x)} . \\end{aligned} $$ Let $g(x)=x f(x)$. Then, by (3), we have $$ \\begin{aligned} g(g(x)) & =g(x f(x))=x f(x) \\cdot f(x f(x))=x f(x)^{2} f(f(x))= \\\\ & =x f(x)^{2} \\sqrt{x^{3} f(x)}=(x f(x))^{5 \/ 2}=(g(x))^{5 \/ 2} \\end{aligned} $$ and, by induction, for every positive integer $n$. Consider (4) for a fixed $x$. The left-hand side is always rational, so $(g(x))^{(5 \/ 2)^{n}}$ must be rational for every $n$. We show that this is possible only if $g(x)=1$. Suppose that $g(x) \\neq 1$, and let the prime factorization of $g(x)$ be $g(x)=p_{1}^{\\alpha_{1}} \\ldots p_{k}^{\\alpha_{k}}$ where $p_{1}, \\ldots, p_{k}$ are distinct primes and $\\alpha_{1}, \\ldots, \\alpha_{k}$ are nonzero integers. Then the unique prime factorization of (4) is $$ \\underbrace{g(g(\\ldots g}_{n+1}(x) \\ldots))=(g(x))^{(5 \/ 2)^{n}}=p_{1}^{(5 \/ 2)^{n} \\alpha_{1}} \\cdots p_{k}^{(5 \/ 2)^{n} \\alpha_{k}} $$ where the exponents should be integers. But this is not true for large values of $n$, for example $\\left(\\frac{5}{2}\\right)^{n} \\alpha_{1}$ cannot be a integer number when $2^{n} \\nmid \\alpha_{1}$. Therefore, $g(x) \\neq 1$ is impossible. Hence, $g(x)=1$ and thus $f(x)=\\frac{1}{x}$ for all $x$. The function $f(x)=\\frac{1}{x}$ satisfies the equation (1): $$ f\\left(f(x)^{2} y\\right)=\\frac{1}{f(x)^{2} y}=\\frac{1}{\\left(\\frac{1}{x}\\right)^{2} y}=\\frac{x^{3}}{x y}=x^{3} f(x y) $$ Comment. Among $\\mathbb{R}^{+} \\rightarrow \\mathbb{R}^{+}$functions, $f(x)=\\frac{1}{x}$ is not the only solution. Another solution is $f_{1}(x)=x^{3 \/ 2}$. Using transfinite tools, infinitely many other solutions can be constructed.","tier":0} +{"problem_type":"Algebra","problem_label":"A6","problem":"Suppose that $f$ and $g$ are two functions defined on the set of positive integers and taking positive integer values. Suppose also that the equations $f(g(n))=f(n)+1$ and $g(f(n))=$ $g(n)+1$ hold for all positive integers. Prove that $f(n)=g(n)$ for all positive integer $n$. (Germany)","solution":"Throughout the solution, by $\\mathbb{N}$ we denote the set of all positive integers. For any function $h: \\mathbb{N} \\rightarrow \\mathbb{N}$ and for any positive integer $k$, define $h^{k}(x)=\\underbrace{h(h(\\ldots h}_{k}(x) \\ldots)$ ) (in particular, $\\left.h^{0}(x)=x\\right)$. Observe that $f\\left(g^{k}(x)\\right)=f\\left(g^{k-1}(x)\\right)+1=\\cdots=f(x)+k$ for any positive integer $k$, and similarly $g\\left(f^{k}(x)\\right)=g(x)+k$. Now let $a$ and $b$ are the minimal values attained by $f$ and $g$, respectively; say $f\\left(n_{f}\\right)=a, g\\left(n_{g}\\right)=b$. Then we have $f\\left(g^{k}\\left(n_{f}\\right)\\right)=a+k, g\\left(f^{k}\\left(n_{g}\\right)\\right)=b+k$, so the function $f$ attains all values from the set $N_{f}=\\{a, a+1, \\ldots\\}$, while $g$ attains all the values from the set $N_{g}=\\{b, b+1, \\ldots\\}$. Next, note that $f(x)=f(y)$ implies $g(x)=g(f(x))-1=g(f(y))-1=g(y)$; surely, the converse implication also holds. Now, we say that $x$ and $y$ are similar (and write $x \\sim y$ ) if $f(x)=f(y)$ (equivalently, $g(x)=g(y)$ ). For every $x \\in \\mathbb{N}$, we define $[x]=\\{y \\in \\mathbb{N}: x \\sim y\\}$; surely, $y_{1} \\sim y_{2}$ for all $y_{1}, y_{2} \\in[x]$, so $[x]=[y]$ whenever $y \\in[x]$. Now we investigate the structure of the sets $[x]$. Claim 1. Suppose that $f(x) \\sim f(y)$; then $x \\sim y$, that is, $f(x)=f(y)$. Consequently, each class $[x]$ contains at most one element from $N_{f}$, as well as at most one element from $N_{g}$. Proof. If $f(x) \\sim f(y)$, then we have $g(x)=g(f(x))-1=g(f(y))-1=g(y)$, so $x \\sim y$. The second statement follows now from the sets of values of $f$ and $g$. Next, we clarify which classes do not contain large elements. Claim 2. For any $x \\in \\mathbb{N}$, we have $[x] \\subseteq\\{1,2, \\ldots, b-1\\}$ if and only if $f(x)=a$. Analogously, $[x] \\subseteq\\{1,2, \\ldots, a-1\\}$ if and only if $g(x)=b$. Proof. We will prove that $[x] \\nsubseteq\\{1,2, \\ldots, b-1\\} \\Longleftrightarrow f(x)>a$; the proof of the second statement is similar. Note that $f(x)>a$ implies that there exists some $y$ satisfying $f(y)=f(x)-1$, so $f(g(y))=$ $f(y)+1=f(x)$, and hence $x \\sim g(y) \\geq b$. Conversely, if $b \\leq c \\sim x$ then $c=g(y)$ for some $y \\in \\mathbb{N}$, which in turn follows $f(x)=f(g(y))=f(y)+1 \\geq a+1$, and hence $f(x)>a$. Claim 2 implies that there exists exactly one class contained in $\\{1, \\ldots, a-1\\}$ (that is, the class $\\left[n_{g}\\right]$ ), as well as exactly one class contained in $\\{1, \\ldots, b-1\\}$ (the class $\\left[n_{f}\\right]$ ). Assume for a moment that $a \\leq b$; then $\\left[n_{g}\\right]$ is contained in $\\{1, \\ldots, b-1\\}$ as well, hence it coincides with $\\left[n_{g}\\right]$. So, we get that $$ f(x)=a \\Longleftrightarrow g(x)=b \\Longleftrightarrow x \\sim n_{f} \\sim n_{g} . $$ Claim 3. $a=b$. Proof. By Claim 2, we have $[a] \\neq\\left[n_{f}\\right]$, so $[a]$ should contain some element $a^{\\prime} \\geq b$ by Claim 2 again. If $a \\neq a^{\\prime}$, then $[a]$ contains two elements $\\geq a$ which is impossible by Claim 1. Therefore, $a=a^{\\prime} \\geq b$. Similarly, $b \\geq a$. Now we are ready to prove the problem statement. First, we establish the following Claim 4. For every integer $d \\geq 0, f^{d+1}\\left(n_{f}\\right)=g^{d+1}\\left(n_{f}\\right)=a+d$. Proof. Induction on $d$. For $d=0$, the statement follows from (1) and Claim 3. Next, for $d>1$ from the induction hypothesis we have $f^{d+1}\\left(n_{f}\\right)=f\\left(f^{d}\\left(n_{f}\\right)\\right)=f\\left(g^{d}\\left(n_{f}\\right)\\right)=f\\left(n_{f}\\right)+d=a+d$. The equality $g^{d+1}\\left(n_{f}\\right)=a+d$ is analogous. Finally, for each $x \\in \\mathbb{N}$, we have $f(x)=a+d$ for some $d \\geq 0$, so $f(x)=f\\left(g^{d}\\left(n_{f}\\right)\\right)$ and hence $x \\sim g^{d}\\left(n_{f}\\right)$. It follows that $g(x)=g\\left(g^{d}\\left(n_{f}\\right)\\right)=g^{d+1}\\left(n_{f}\\right)=a+d=f(x)$ by Claim 4 .","tier":0} +{"problem_type":"Algebra","problem_label":"A6","problem":"Suppose that $f$ and $g$ are two functions defined on the set of positive integers and taking positive integer values. Suppose also that the equations $f(g(n))=f(n)+1$ and $g(f(n))=$ $g(n)+1$ hold for all positive integers. Prove that $f(n)=g(n)$ for all positive integer $n$. (Germany)","solution":"We start with the same observations, introducing the relation $\\sim$ and proving Claim 1 from the previous solution. Note that $f(a)>a$ since otherwise we have $f(a)=a$ and hence $g(a)=g(f(a))=g(a)+1$, which is false. Claim 2'. $a=b$. Proof. We can assume that $a \\leq b$. Since $f(a) \\geq a+1$, there exists some $x \\in \\mathbb{N}$ such that $f(a)=f(x)+1$, which is equivalent to $f(a)=f(g(x))$ and $a \\sim g(x)$. Since $g(x) \\geq b \\geq a$, by Claim 1 we have $a=g(x) \\geq b$, which together with $a \\leq b$ proves the Claim. Now, almost the same method allows to find the values $f(a)$ and $g(a)$. Claim 3'. $f(a)=g(a)=a+1$. Proof. Assume the contrary; then $f(a) \\geq a+2$, hence there exist some $x, y \\in \\mathbb{N}$ such that $f(x)=f(a)-2$ and $f(y)=g(x)($ as $g(x) \\geq a=b)$. Now we get $f(a)=f(x)+2=f\\left(g^{2}(x)\\right)$, so $a \\sim g^{2}(x) \\geq a$, and by Claim 1 we get $a=g^{2}(x)=g(f(y))=1+g(y) \\geq 1+a$; this is impossible. The equality $g(a)=a+1$ is similar. Now, we are prepared for the proof of the problem statement. First, we prove it for $n \\geq a$. Claim 4'. For each integer $x \\geq a$, we have $f(x)=g(x)=x+1$. Proof. Induction on $x$. The base case $x=a$ is provided by Claim $3^{\\prime}$, while the induction step follows from $f(x+1)=f(g(x))=f(x)+1=(x+1)+1$ and the similar computation for $g(x+1)$. Finally, for an arbitrary $n \\in \\mathbb{N}$ we have $g(n) \\geq a$, so by Claim $4^{\\prime}$ we have $f(n)+1=$ $f(g(n))=g(n)+1$, hence $f(n)=g(n)$. Comment. It is not hard now to describe all the functions $f: \\mathbb{N} \\rightarrow \\mathbb{N}$ satisfying the property $f(f(n))=$ $f(n)+1$. For each such function, there exists $n_{0} \\in \\mathbb{N}$ such that $f(n)=n+1$ for all $n \\geq n_{0}$, while for each $nr$, we inductively define $$ a_{n}=\\max _{1 \\leq k \\leq n-1}\\left(a_{k}+a_{n-k}\\right) $$ Prove that there exist positive integers $\\ell \\leq r$ and $N$ such that $a_{n}=a_{n-\\ell}+a_{\\ell}$ for all $n \\geq N$. (Iran)","solution":"First, from the problem conditions we have that each $a_{n}(n>r)$ can be expressed as $a_{n}=a_{j_{1}}+a_{j_{2}}$ with $j_{1}, j_{2}r$ then we can proceed in the same way with $a_{j_{1}}$, and so on. Finally, we represent $a_{n}$ in a form $$ \\begin{gathered} a_{n}=a_{i_{1}}+\\cdots+a_{i_{k}}, \\\\ 1 \\leq i_{j} \\leq r, \\quad i_{1}+\\cdots+i_{k}=n . \\end{gathered} $$ Moreover, if $a_{i_{1}}$ and $a_{i_{2}}$ are the numbers in (2) obtained on the last step, then $i_{1}+i_{2}>r$. Hence we can adjust (3) as $$ 1 \\leq i_{j} \\leq r, \\quad i_{1}+\\cdots+i_{k}=n, \\quad i_{1}+i_{2}>r . $$ On the other hand, suppose that the indices $i_{1}, \\ldots, i_{k}$ satisfy the conditions (4). Then, denoting $s_{j}=i_{1}+\\cdots+i_{j}$, from (1) we have $$ a_{n}=a_{s_{k}} \\geq a_{s_{k-1}}+a_{i_{k}} \\geq a_{s_{k-2}}+a_{i_{k-1}}+a_{i_{k}} \\geq \\cdots \\geq a_{i_{1}}+\\cdots+a_{i_{k}} $$ Summarizing these observations we get the following Claim. For every $n>r$, we have $$ a_{n}=\\max \\left\\{a_{i_{1}}+\\cdots+a_{i_{k}}: \\text { the collection }\\left(i_{1}, \\ldots, i_{k}\\right) \\text { satisfies }(4)\\right\\} $$ Now we denote $$ s=\\max _{1 \\leq i \\leq r} \\frac{a_{i}}{i} $$ and fix some index $\\ell \\leq r$ such that $s=\\frac{a_{\\ell}}{\\ell}$. Consider some $n \\geq r^{2} \\ell+2 r$ and choose an expansion of $a_{n}$ in the form (2), (4). Then we have $n=i_{1}+\\cdots+i_{k} \\leq r k$, so $k \\geq n \/ r \\geq r \\ell+2$. Suppose that none of the numbers $i_{3}, \\ldots, i_{k}$ equals $\\ell$. Then by the pigeonhole principle there is an index $1 \\leq j \\leq r$ which appears among $i_{3}, \\ldots, i_{k}$ at least $\\ell$ times, and surely $j \\neq \\ell$. Let us delete these $\\ell$ occurrences of $j$ from $\\left(i_{1}, \\ldots, i_{k}\\right)$, and add $j$ occurrences of $\\ell$ instead, obtaining a sequence $\\left(i_{1}, i_{2}, i_{3}^{\\prime}, \\ldots, i_{k^{\\prime}}^{\\prime}\\right)$ also satisfying (4). By Claim, we have $$ a_{i_{1}}+\\cdots+a_{i_{k}}=a_{n} \\geq a_{i_{1}}+a_{i_{2}}+a_{i_{3}^{\\prime}}+\\cdots+a_{i_{k^{\\prime}}^{\\prime}} $$ or, after removing the coinciding terms, $\\ell a_{j} \\geq j a_{\\ell}$, so $\\frac{a_{\\ell}}{\\ell} \\leq \\frac{a_{j}}{j}$. By the definition of $\\ell$, this means that $\\ell a_{j}=j a_{\\ell}$, hence $$ a_{n}=a_{i_{1}}+a_{i_{2}}+a_{i_{3}^{\\prime}}+\\cdots+a_{i_{k^{\\prime}}^{\\prime}} $$ Thus, for every $n \\geq r^{2} \\ell+2 r$ we have found a representation of the form (2), (4) with $i_{j}=\\ell$ for some $j \\geq 3$. Rearranging the indices we may assume that $i_{k}=\\ell$. Finally, observe that in this representation, the indices $\\left(i_{1}, \\ldots, i_{k-1}\\right)$ satisfy the conditions (4) with $n$ replaced by $n-\\ell$. Thus, from the Claim we get $$ a_{n-\\ell}+a_{\\ell} \\geq\\left(a_{i_{1}}+\\cdots+a_{i_{k-1}}\\right)+a_{\\ell}=a_{n} $$ which by (1) implies $$ a_{n}=a_{n-\\ell}+a_{\\ell} \\quad \\text { for each } n \\geq r^{2} \\ell+2 r $$ as desired.","tier":0} +{"problem_type":"Algebra","problem_label":"A7","problem":"Let $a_{1}, \\ldots, a_{r}$ be positive real numbers. For $n>r$, we inductively define $$ a_{n}=\\max _{1 \\leq k \\leq n-1}\\left(a_{k}+a_{n-k}\\right) $$ Prove that there exist positive integers $\\ell \\leq r$ and $N$ such that $a_{n}=a_{n-\\ell}+a_{\\ell}$ for all $n \\geq N$. (Iran)","solution":"As in the previous solution, we involve the expansion (2), (3), and we fix some index $1 \\leq \\ell \\leq r$ such that $$ \\frac{a_{\\ell}}{\\ell}=s=\\max _{1 \\leq i \\leq r} \\frac{a_{i}}{i} $$ Now, we introduce the sequence $\\left(b_{n}\\right)$ as $b_{n}=a_{n}-s n$; then $b_{\\ell}=0$. We prove by induction on $n$ that $b_{n} \\leq 0$, and $\\left(b_{n}\\right)$ satisfies the same recurrence relation as $\\left(a_{n}\\right)$. The base cases $n \\leq r$ follow from the definition of $s$. Now, for $n>r$ from the induction hypothesis we have $$ b_{n}=\\max _{1 \\leq k \\leq n-1}\\left(a_{k}+a_{n-k}\\right)-n s=\\max _{1 \\leq k \\leq n-1}\\left(b_{k}+b_{n-k}+n s\\right)-n s=\\max _{1 \\leq k \\leq n-1}\\left(b_{k}+b_{n-k}\\right) \\leq 0 $$ as required. Now, if $b_{k}=0$ for all $1 \\leq k \\leq r$, then $b_{n}=0$ for all $n$, hence $a_{n}=s n$, and the statement is trivial. Otherwise, define $$ M=\\max _{1 \\leq i \\leq r}\\left|b_{i}\\right|, \\quad \\varepsilon=\\min \\left\\{\\left|b_{i}\\right|: 1 \\leq i \\leq r, b_{i}<0\\right\\} $$ Then for $n>r$ we obtain $$ b_{n}=\\max _{1 \\leq k \\leq n-1}\\left(b_{k}+b_{n-k}\\right) \\geq b_{\\ell}+b_{n-\\ell}=b_{n-\\ell} $$ so $$ 0 \\geq b_{n} \\geq b_{n-\\ell} \\geq b_{n-2 \\ell} \\geq \\cdots \\geq-M $$ Thus, in view of the expansion (2), (3) applied to the sequence $\\left(b_{n}\\right)$, we get that each $b_{n}$ is contained in a set $$ T=\\left\\{b_{i_{1}}+b_{i_{2}}+\\cdots+b_{i_{k}}: i_{1}, \\ldots, i_{k} \\leq r\\right\\} \\cap[-M, 0] $$ We claim that this set is finite. Actually, for any $x \\in T$, let $x=b_{i_{1}}+\\cdots+b_{i_{k}}\\left(i_{1}, \\ldots, i_{k} \\leq r\\right)$. Then among $b_{i_{j}}$ 's there are at most $\\frac{M}{\\varepsilon}$ nonzero terms (otherwise $x<\\frac{M}{\\varepsilon} \\cdot(-\\varepsilon)<-M$ ). Thus $x$ can be expressed in the same way with $k \\leq \\frac{M}{\\varepsilon}$, and there is only a finite number of such sums. Finally, for every $t=1,2, \\ldots, \\ell$ we get that the sequence $$ b_{r+t}, b_{r+t+\\ell}, b_{r+t+2 \\ell}, \\ldots $$ is non-decreasing and attains the finite number of values; therefore it is constant from some index. Thus, the sequence $\\left(b_{n}\\right)$ is periodic with period $\\ell$ from some index $N$, which means that $$ b_{n}=b_{n-\\ell}=b_{n-\\ell}+b_{\\ell} \\quad \\text { for all } n>N+\\ell $$ and hence $$ a_{n}=b_{n}+n s=\\left(b_{n-\\ell}+(n-\\ell) s\\right)+\\left(b_{\\ell}+\\ell s\\right)=a_{n-\\ell}+a_{\\ell} \\quad \\text { for all } n>N+\\ell \\text {, } $$ as desired.","tier":0} +{"problem_type":"Algebra","problem_label":"A8","problem":"Given six positive numbers $a, b, c, d, e, f$ such that $a\\sqrt{3(S+T)(S(b d+b f+d f)+T(a c+a e+c e))} $$ (South Korea)","solution":"We define also $\\sigma=a c+c e+a e, \\tau=b d+b f+d f$. The idea of the solution is to interpret (1) as a natural inequality on the roots of an appropriate polynomial. Actually, consider the polynomial $$ \\begin{aligned} & P(x)=(b+d+f)(x-a)(x-c)(x-e)+(a+c+e)(x-b)(x-d)(x-f) \\\\ &=T\\left(x^{3}-S x^{2}+\\sigma x-a c e\\right)+S\\left(x^{3}-T x^{2}+\\tau x-b d f\\right) \\end{aligned} $$ Surely, $P$ is cubic with leading coefficient $S+T>0$. Moreover, we have $$ \\begin{array}{ll} P(a)=S(a-b)(a-d)(a-f)<0, & P(c)=S(c-b)(c-d)(c-f)>0 \\\\ P(e)=S(e-b)(e-d)(e-f)<0, & P(f)=T(f-a)(f-c)(f-e)>0 \\end{array} $$ Hence, each of the intervals $(a, c),(c, e),(e, f)$ contains at least one root of $P(x)$. Since there are at most three roots at all, we obtain that there is exactly one root in each interval (denote them by $\\alpha \\in(a, c), \\beta \\in(c, e), \\gamma \\in(e, f))$. Moreover, the polynomial $P$ can be factorized as $$ P(x)=(T+S)(x-\\alpha)(x-\\beta)(x-\\gamma) $$ Equating the coefficients in the two representations (2) and (3) of $P(x)$ provides $$ \\alpha+\\beta+\\gamma=\\frac{2 T S}{T+S}, \\quad \\alpha \\beta+\\alpha \\gamma+\\beta \\gamma=\\frac{S \\tau+T \\sigma}{T+S} $$ Now, since the numbers $\\alpha, \\beta, \\gamma$ are distinct, we have $$ 0<(\\alpha-\\beta)^{2}+(\\alpha-\\gamma)^{2}+(\\beta-\\gamma)^{2}=2(\\alpha+\\beta+\\gamma)^{2}-6(\\alpha \\beta+\\alpha \\gamma+\\beta \\gamma) $$ which implies $$ \\frac{4 S^{2} T^{2}}{(T+S)^{2}}=(\\alpha+\\beta+\\gamma)^{2}>3(\\alpha \\beta+\\alpha \\gamma+\\beta \\gamma)=\\frac{3(S \\tau+T \\sigma)}{T+S} $$ or $$ 4 S^{2} T^{2}>3(T+S)(T \\sigma+S \\tau) $$ which is exactly what we need. Comment 1. In fact, one can locate the roots of $P(x)$ more narrowly: they should lie in the intervals $(a, b),(c, d),(e, f)$. Surely, if we change all inequality signs in the problem statement to non-strict ones, the (non-strict) inequality will also hold by continuity. One can also find when the equality is achieved. This happens in that case when $P(x)$ is a perfect cube, which immediately implies that $b=c=d=e(=\\alpha=\\beta=\\gamma)$, together with the additional condition that $P^{\\prime \\prime}(b)=0$. Algebraically, $$ \\begin{array}{rlr} 6(T+S) b-4 T S=0 & \\Longleftrightarrow & 3 b(a+4 b+f)=2(a+2 b)(2 b+f) \\\\ & \\Longleftrightarrow & f=\\frac{b(4 b-a)}{2 a+b}=b\\left(1+\\frac{3(b-a)}{2 a+b}\\right)>b \\end{array} $$ This means that for every pair of numbers $a, b$ such that $0b$ such that the point $(a, b, b, b, b, f)$ is a point of equality.","tier":0} +{"problem_type":"Algebra","problem_label":"A8","problem":"Given six positive numbers $a, b, c, d, e, f$ such that $a\\sqrt{3(S+T)(S(b d+b f+d f)+T(a c+a e+c e))} $$ (South Korea)","solution":"Let $$ U=\\frac{1}{2}\\left((e-a)^{2}+(c-a)^{2}+(e-c)^{2}\\right)=S^{2}-3(a c+a e+c e) $$ and $$ V=\\frac{1}{2}\\left((f-b)^{2}+(f-d)^{2}+(d-b)^{2}\\right)=T^{2}-3(b d+b f+d f) $$ Then $$ \\begin{aligned} & \\text { (L.H.S. })^{2}-(\\text { R.H.S. })^{2}=(2 S T)^{2}-(S+T)(S \\cdot 3(b d+b f+d f)+T \\cdot 3(a c+a e+c e))= \\\\ & \\quad=4 S^{2} T^{2}-(S+T)\\left(S\\left(T^{2}-V\\right)+T\\left(S^{2}-U\\right)\\right)=(S+T)(S V+T U)-S T(T-S)^{2} \\end{aligned} $$ and the statement is equivalent with $$ (S+T)(S V+T U)>S T(T-S)^{2} $$ By the Cauchy-Schwarz inequality, $$ (S+T)(T U+S V) \\geq(\\sqrt{S \\cdot T U}+\\sqrt{T \\cdot S V})^{2}=S T(\\sqrt{U}+\\sqrt{V})^{2} $$ Estimate the quantities $\\sqrt{U}$ and $\\sqrt{V}$ by the QM-AM inequality with the positive terms $(e-c)^{2}$ and $(d-b)^{2}$ being omitted: $$ \\begin{aligned} \\sqrt{U}+\\sqrt{V} & >\\sqrt{\\frac{(e-a)^{2}+(c-a)^{2}}{2}}+\\sqrt{\\frac{(f-b)^{2}+(f-d)^{2}}{2}} \\\\ & >\\frac{(e-a)+(c-a)}{2}+\\frac{(f-b)+(f-d)}{2}=\\left(f-\\frac{d}{2}-\\frac{b}{2}\\right)+\\left(\\frac{e}{2}+\\frac{c}{2}-a\\right) \\\\ & =(T-S)+\\frac{3}{2}(e-d)+\\frac{3}{2}(c-b)>T-S . \\end{aligned} $$ The estimates (5) and (6) prove (4) and hence the statement.","tier":0} +{"problem_type":"Algebra","problem_label":"A8","problem":"Given six positive numbers $a, b, c, d, e, f$ such that $a\\sqrt{3(S+T)(S(b d+b f+d f)+T(a c+a e+c e))} $$ (South Korea)","solution":"We keep using the notations $\\sigma$ and $\\tau$ from $$ (c-b)(c-d)+(e-f)(e-d)+(e-f)(c-b)<0 $$ since each summand is negative. This rewrites as $$ \\begin{gathered} (b d+b f+d f)-(a c+c e+a e)<(c+e)(b+d+f-a-c-e), \\text { or } \\\\ \\tau-\\sigma\\sqrt{3(S+T)(S(b d+b f+d f)+T(a c+a e+c e))} $$ Comment 2. The expression (7) can be found by considering the sum of the roots of the quadratic polynomial $q(x)=(x-b)(x-d)(x-f)-(x-a)(x-c)(x-e)$.","tier":0} +{"problem_type":"Algebra","problem_label":"A8","problem":"Given six positive numbers $a, b, c, d, e, f$ such that $a\\sqrt{3(S+T)(S(b d+b f+d f)+T(a c+a e+c e))} $$ (South Korea)","solution":"We introduce the expressions $\\sigma$ and $\\tau$ as in the previous solutions. The idea of the solution is to change the values of variables $a, \\ldots, f$ keeping the left-hand side unchanged and increasing the right-hand side; it will lead to a simpler inequality which can be proved in a direct way. Namely, we change the variables (i) keeping the (non-strict) inequalities $a \\leq b \\leq c \\leq d \\leq$ $e \\leq f$; (ii) keeping the values of sums $S$ and $T$ unchanged; and finally (iii) increasing the values of $\\sigma$ and $\\tau$. Then the left-hand side of (1) remains unchanged, while the right-hand side increases. Hence, the inequality (1) (and even a non-strict version of (1)) for the changed values would imply the same (strict) inequality for the original values. First, we find the sufficient conditions for (ii) and (iii) to be satisfied. Lemma. Let $x, y, z>0$; denote $U(x, y, z)=x+y+z, v(x, y, z)=x y+x z+y z$. Suppose that $x^{\\prime}+y^{\\prime}=x+y$ but $|x-y| \\geq\\left|x^{\\prime}-y^{\\prime}\\right| ;$ then we have $U\\left(x^{\\prime}, y^{\\prime}, z\\right)=U(x, y, z)$ and $v\\left(x^{\\prime}, y^{\\prime}, z\\right) \\geq$ $v(x, y, z)$ with equality achieved only when $|x-y|=\\left|x^{\\prime}-y^{\\prime}\\right|$. Proof. The first equality is obvious. For the second, we have $$ \\begin{aligned} v\\left(x^{\\prime}, y^{\\prime}, z\\right)=z\\left(x^{\\prime}+y^{\\prime}\\right)+x^{\\prime} y^{\\prime} & =z\\left(x^{\\prime}+y^{\\prime}\\right)+\\frac{\\left(x^{\\prime}+y^{\\prime}\\right)^{2}-\\left(x^{\\prime}-y^{\\prime}\\right)^{2}}{4} \\\\ & \\geq z(x+y)+\\frac{(x+y)^{2}-(x-y)^{2}}{4}=v(x, y, z) \\end{aligned} $$ with the equality achieved only for $\\left(x^{\\prime}-y^{\\prime}\\right)^{2}=(x-y)^{2} \\Longleftrightarrow\\left|x^{\\prime}-y^{\\prime}\\right|=|x-y|$, as desired. Now, we apply Lemma several times making the following changes. For each change, we denote the new values by the same letters to avoid cumbersome notations. 1. Let $k=\\frac{d-c}{2}$. Replace $(b, c, d, e)$ by $(b+k, c+k, d-k, e-k)$. After the change we have $a2^{N-2}$. Consider the collection of all $2^{N-2}$ flags having yellow first squares and blue second ones. Obviously, both colors appear on the diagonal of each $N \\times N$ square formed by these flags. We are left to show that $M_{N} \\leq 2^{N-2}+1$, thus obtaining the desired answer. We start with establishing this statement for $N=4$. Suppose that we have 5 flags of length 4 . We decompose each flag into two parts of 2 squares each; thereby, we denote each flag as $L R$, where the $2 \\times 1$ flags $L, R \\in \\mathcal{S}=\\{\\mathrm{BB}, \\mathrm{BY}, \\mathrm{YB}, \\mathrm{YY}\\}$ are its left and right parts, respectively. First, we make two easy observations on the flags $2 \\times 1$ which can be checked manually. (i) For each $A \\in \\mathcal{S}$, there exists only one $2 \\times 1$ flag $C \\in \\mathcal{S}$ (possibly $C=A$ ) such that $A$ and $C$ cannot form a $2 \\times 2$ square with monochrome diagonal (for part BB, that is YY, and for BY that is YB). (ii) Let $A_{1}, A_{2}, A_{3} \\in \\mathcal{S}$ be three distinct elements; then two of them can form a $2 \\times 2$ square with yellow diagonal, and two of them can form a $2 \\times 2$ square with blue diagonal (for all parts but BB , a pair $(\\mathrm{BY}, \\mathrm{YB})$ fits for both statements, while for all parts but BY, these pairs are $(\\mathrm{YB}, \\mathrm{YY})$ and (BB, YB)). Now, let $\\ell$ and $r$ be the numbers of distinct left and right parts of our 5 flags, respectively. The total number of flags is $5 \\leq r \\ell$, hence one of the factors (say, $r$ ) should be at least 3 . On the other hand, $\\ell, r \\leq 4$, so there are two flags with coinciding right part; let them be $L_{1} R_{1}$ and $L_{2} R_{1}\\left(L_{1} \\neq L_{2}\\right)$. Next, since $r \\geq 3$, there exist some flags $L_{3} R_{3}$ and $L_{4} R_{4}$ such that $R_{1}, R_{3}, R_{4}$ are distinct. Let $L^{\\prime} R^{\\prime}$ be the remaining flag. By (i), one of the pairs $\\left(L^{\\prime}, L_{1}\\right)$ and $\\left(L^{\\prime}, L_{2}\\right)$ can form a $2 \\times 2$ square with monochrome diagonal; we can assume that $L^{\\prime}, L_{2}$ form a square with a blue diagonal. Finally, the right parts of two of the flags $L_{1} R_{1}, L_{3} R_{3}, L_{4} R_{4}$ can also form a $2 \\times 2$ square with a blue diagonal by (ii). Putting these $2 \\times 2$ squares on the diagonal of a $4 \\times 4$ square, we find a desired arrangement of four flags. We are ready to prove the problem statement by induction on $N$; actually, above we have proved the base case $N=4$. For the induction step, assume that $N>4$, consider any $2^{N-2}+1$ flags of length $N$, and arrange them into a large flag of size $\\left(2^{N-2}+1\\right) \\times N$. This flag contains a non-monochrome column since the flags are distinct; we may assume that this column is the first one. By the pigeonhole principle, this column contains at least $\\left\\lceil\\frac{2^{N-2}+1}{2}\\right\\rceil=2^{N-3}+1$ squares of one color (say, blue). We call the flags with a blue first square good. Consider all the good flags and remove the first square from each of them. We obtain at least $2^{N-3}+1 \\geq M_{N-1}$ flags of length $N-1$; by the induction hypothesis, $N-1$ of them can form a square $Q$ with the monochrome diagonal. Now, returning the removed squares, we obtain a rectangle $(N-1) \\times N$, and our aim is to supplement it on the top by one more flag. If $Q$ has a yellow diagonal, then we can take each flag with a yellow first square (it exists by a choice of the first column; moreover, it is not used in $Q$ ). Conversely, if the diagonal of $Q$ is blue then we can take any of the $\\geq 2^{N-3}+1-(N-1)>0$ remaining good flags. So, in both cases we get a desired $N \\times N$ square.","tier":0} +{"problem_type":"Combinatorics","problem_label":"C2","problem":"On some planet, there are $2^{N}$ countries $(N \\geq 4)$. Each country has a flag $N$ units wide and one unit high composed of $N$ fields of size $1 \\times 1$, each field being either yellow or blue. No two countries have the same flag. We say that a set of $N$ flags is diverse if these flags can be arranged into an $N \\times N$ square so that all $N$ fields on its main diagonal will have the same color. Determine the smallest positive integer $M$ such that among any $M$ distinct flags, there exist $N$ flags forming a diverse set. (Croatia) Answer. $M=2^{N-2}+1$.","solution":"We present a different proof of the estimate $M_{N} \\leq 2^{N-2}+1$. We do not use the induction, involving Hall's lemma on matchings instead. Consider arbitrary $2^{N-2}+1$ distinct flags and arrange them into a large $\\left(2^{N-2}+1\\right) \\times N$ flag. Construct two bipartite graphs $G_{\\mathrm{y}}=\\left(V \\cup V^{\\prime}, E_{\\mathrm{y}}\\right)$ and $G_{\\mathrm{b}}=\\left(V \\cup V^{\\prime}, E_{\\mathrm{b}}\\right)$ with the common set of vertices as follows. Let $V$ and $V^{\\prime}$ be the set of columns and the set of flags under consideration, respectively. Next, let the edge $(c, f)$ appear in $E_{\\mathrm{y}}$ if the intersection of column $c$ and flag $f$ is yellow, and $(c, f) \\in E_{\\mathrm{b}}$ otherwise. Then we have to prove exactly that one of the graphs $G_{\\mathrm{y}}$ and $G_{\\mathrm{b}}$ contains a matching with all the vertices of $V$ involved. Assume that these matchings do not exist. By Hall's lemma, it means that there exist two sets of columns $S_{\\mathrm{y}}, S_{\\mathrm{b}} \\subset V$ such that $\\left|E_{\\mathrm{y}}\\left(S_{\\mathrm{y}}\\right)\\right| \\leq\\left|S_{\\mathrm{y}}\\right|-1$ and $\\left|E_{\\mathrm{b}}\\left(S_{\\mathrm{b}}\\right)\\right| \\leq\\left|S_{\\mathrm{b}}\\right|-1$ (in the left-hand sides, $E_{\\mathrm{y}}\\left(S_{\\mathrm{y}}\\right)$ and $E_{\\mathrm{b}}\\left(S_{\\mathrm{b}}\\right)$ denote respectively the sets of all vertices connected to $S_{\\mathrm{y}}$ and $S_{\\mathrm{b}}$ in the corresponding graphs). Our aim is to prove that this is impossible. Note that $S_{\\mathrm{y}}, S_{\\mathrm{b}} \\neq V$ since $N \\leq 2^{N-2}+1$. First, suppose that $S_{\\mathrm{y}} \\cap S_{\\mathrm{b}} \\neq \\varnothing$, so there exists some $c \\in S_{\\mathrm{y}} \\cap S_{\\mathrm{b}}$. Note that each flag is connected to $c$ either in $G_{\\mathrm{y}}$ or in $G_{\\mathrm{b}}$, hence $E_{\\mathrm{y}}\\left(S_{\\mathrm{y}}\\right) \\cup E_{\\mathrm{b}}\\left(S_{\\mathrm{b}}\\right)=V^{\\prime}$. Hence we have $2^{N-2}+1=\\left|V^{\\prime}\\right| \\leq\\left|E_{\\mathrm{y}}\\left(S_{\\mathrm{y}}\\right)\\right|+\\left|E_{\\mathrm{b}}\\left(S_{\\mathrm{b}}\\right)\\right| \\leq\\left|S_{\\mathrm{y}}\\right|+\\left|S_{\\mathrm{b}}\\right|-2 \\leq 2 N-4$; this is impossible for $N \\geq 4$. So, we have $S_{\\mathrm{y}} \\cap S_{\\mathrm{b}}=\\varnothing$. Let $y=\\left|S_{\\mathrm{y}}\\right|, b=\\left|S_{\\mathrm{b}}\\right|$. From the construction of our graph, we have that all the flags in the set $V^{\\prime \\prime}=V^{\\prime} \\backslash\\left(E_{\\mathrm{y}}\\left(S_{\\mathrm{y}}\\right) \\cup E_{\\mathrm{b}}\\left(S_{\\mathrm{b}}\\right)\\right)$ have blue squares in the columns of $S_{\\mathrm{y}}$ and yellow squares in the columns of $S_{\\mathrm{b}}$. Hence the only undetermined positions in these flags are the remaining $N-y-b$ ones, so $2^{N-y-b} \\geq\\left|V^{\\prime \\prime}\\right| \\geq\\left|V^{\\prime}\\right|-\\left|E_{\\mathrm{y}}\\left(S_{\\mathrm{y}}\\right)\\right|-\\left|E_{\\mathrm{b}}\\left(S_{\\mathrm{b}}\\right)\\right| \\geq$ $2^{N-2}+1-(y-1)-(b-1)$, or, denoting $c=y+b, 2^{N-c}+c>2^{N-2}+2$. This is impossible since $N \\geq c \\geq 2$.","tier":0} +{"problem_type":"Combinatorics","problem_label":"C3","problem":"2500 chess kings have to be placed on a $100 \\times 100$ chessboard so that (i) no king can capture any other one (i.e. no two kings are placed in two squares sharing a common vertex); (ii) each row and each column contains exactly 25 kings. Find the number of such arrangements. (Two arrangements differing by rotation or symmetry are supposed to be different.) (Russia) Answer. There are two such arrangements.","solution":"Suppose that we have an arrangement satisfying the problem conditions. Divide the board into $2 \\times 2$ pieces; we call these pieces blocks. Each block can contain not more than one king (otherwise these two kings would attack each other); hence, by the pigeonhole principle each block must contain exactly one king. Now assign to each block a letter T or B if a king is placed in its top or bottom half, respectively. Similarly, assign to each block a letter L or R if a king stands in its left or right half. So we define $T$-blocks, $B$-blocks, $L$-blocks, and $R$-blocks. We also combine the letters; we call a block a TL-block if it is simultaneously T-block and L-block. Similarly we define TR-blocks, $B L$-blocks, and BR-blocks. The arrangement of blocks determines uniquely the arrangement of kings; so in the rest of the solution we consider the $50 \\times 50$ system of blocks (see Fig. 1). We identify the blocks by their coordinate pairs; the pair $(i, j)$, where $1 \\leq i, j \\leq 50$, refers to the $j$ th block in the $i$ th row (or the $i$ th block in the $j$ th column). The upper-left block is $(1,1)$. The system of blocks has the following properties.. ( $\\left.\\mathrm{i}^{\\prime}\\right)$ If $(i, j)$ is a B-block then $(i+1, j)$ is a B-block: otherwise the kings in these two blocks can take each other. Similarly: if $(i, j)$ is a T-block then $(i-1, j)$ is a T-block; if $(i, j)$ is an L-block then $(i, j-1)$ is an L-block; if $(i, j)$ is an R-block then $(i, j+1)$ is an R-block. (ii') Each column contains exactly 25 L-blocks and 25 R-blocks, and each row contains exactly 25 T -blocks and 25 B-blocks. In particular, the total number of L-blocks (or R-blocks, or T-blocks, or B-blocks) is equal to $25 \\cdot 50=1250$. Consider any B-block of the form $(1, j)$. By ( $\\mathrm{i}^{\\prime}$ ), all blocks in the $j$ th column are B-blocks; so we call such a column $B$-column. By (ii'), we have 25 B-blocks in the first row, so we obtain 25 B-columns. These 25 B-columns contain 1250 B-blocks, hence all blocks in the remaining columns are T-blocks, and we obtain 25 T-columns. Similarly, there are exactly $25 L$-rows and exactly $25 R$-rows. Now consider an arbitrary pair of a T-column and a neighboring B-column (columns with numbers $j$ and $j+1$ ). ![](https:\/\/cdn.mathpix.com\/cropped\/2024_11_18_ba86565be3364967ae6fg-29.jpg?height=343&width=778&top_left_y=2127&top_left_x=436) Fig. 1 ![](https:\/\/cdn.mathpix.com\/cropped\/2024_11_18_ba86565be3364967ae6fg-29.jpg?height=320&width=361&top_left_y=2150&top_left_x=1330) Fig. 2 Case 1. Suppose that the $j$ th column is a T-column, and the $(j+1)$ th column is a Bcolumn. Consider some index $i$ such that the $i$ th row is an L-row; then $(i, j+1)$ is a BL-block. Therefore, $(i+1, j)$ cannot be a TR-block (see Fig. 2), hence $(i+1, j)$ is a TL-block, thus the $(i+1)$ th row is an L-row. Now, choosing the $i$ th row to be the topmost L-row, we successively obtain that all rows from the $i$ th to the 50 th are L-rows. Since we have exactly 25 L-rows, it follows that the rows from the 1st to the 25 th are R-rows, and the rows from the 26 th to the 50th are L-rows. Now consider the neighboring R-row and L-row (that are the rows with numbers 25 and 26). Replacing in the previous reasoning rows by columns and vice versa, the columns from the 1 st to the 25 th are T-columns, and the columns from the 26 th to the 50 th are B-columns. So we have a unique arrangement of blocks that leads to the arrangement of kings satisfying the condition of the problem (see Fig. 3). ![](https:\/\/cdn.mathpix.com\/cropped\/2024_11_18_ba86565be3364967ae6fg-30.jpg?height=521&width=1020&top_left_y=796&top_left_x=204) Fig. 3 ![](https:\/\/cdn.mathpix.com\/cropped\/2024_11_18_ba86565be3364967ae6fg-30.jpg?height=524&width=509&top_left_y=792&top_left_x=1296) Fig. 4 Case 2. Suppose that the $j$ th column is a B-column, and the $(j+1)$ th column is a T-column. Repeating the arguments from Case 1, we obtain that the rows from the 1st to the 25th are L-rows (and all other rows are R-rows), the columns from the 1st to the 25 th are B-columns (and all other columns are T-columns), so we find exactly one more arrangement of kings (see Fig. 4).","tier":0} +{"problem_type":"Combinatorics","problem_label":"C4","problem":"Six stacks $S_{1}, \\ldots, S_{6}$ of coins are standing in a row. In the beginning every stack contains a single coin. There are two types of allowed moves: Move 1: If stack $S_{k}$ with $1 \\leq k \\leq 5$ contains at least one coin, you may remove one coin from $S_{k}$ and add two coins to $S_{k+1}$. Move 2: If stack $S_{k}$ with $1 \\leq k \\leq 4$ contains at least one coin, then you may remove one coin from $S_{k}$ and exchange stacks $S_{k+1}$ and $S_{k+2}$. Decide whether it is possible to achieve by a sequence of such moves that the first five stacks are empty, whereas the sixth stack $S_{6}$ contains exactly $2010^{2010^{2010}}$ coins.","solution":"Denote by $\\left(a_{1}, a_{2}, \\ldots, a_{n}\\right) \\rightarrow\\left(a_{1}^{\\prime}, a_{2}^{\\prime}, \\ldots, a_{n}^{\\prime}\\right)$ the following: if some consecutive stacks contain $a_{1}, \\ldots, a_{n}$ coins, then it is possible to perform several allowed moves such that the stacks contain $a_{1}^{\\prime}, \\ldots, a_{n}^{\\prime}$ coins respectively, whereas the contents of the other stacks remain unchanged. Let $A=2010^{2010}$ or $A=2010^{2010^{2010}}$, respectively. Our goal is to show that $$ (1,1,1,1,1,1) \\rightarrow(0,0,0,0,0, A) $$ First we prove two auxiliary observations. Lemma 1. $(a, 0,0) \\rightarrow\\left(0,2^{a}, 0\\right)$ for every $a \\geq 1$. Proof. We prove by induction that $(a, 0,0) \\rightarrow\\left(a-k, 2^{k}, 0\\right)$ for every $1 \\leq k \\leq a$. For $k=1$, apply Move 1 to the first stack: $$ (a, 0,0) \\rightarrow(a-1,2,0)=\\left(a-1,2^{1}, 0\\right) $$ Now assume that $k0$, while in the latter one $S(T)=1^{3}+1^{3}+(-1)^{3}+(-1)^{3}=0$, as desired. Now we turn to the general problem. Consider a tournament $T$ with no bad companies and enumerate the players by the numbers from 1 to $n$. For every 4 players $i_{1}, i_{2}, i_{3}, i_{4}$ consider a \"sub-tournament\" $T_{i_{1} i_{2} i_{3} i_{4}}$ consisting of only these players and the games which they performed with each other. By the abovementioned, we have $S\\left(T_{i_{1} i_{2} i_{3} i_{4}}\\right) \\geq 0$. Our aim is to prove that $$ S(T)=\\sum_{i_{1}, i_{2}, i_{3}, i_{4}} S\\left(T_{i_{1} i_{2} i_{3} i_{4}}\\right) $$ where the sum is taken over all 4 -tuples of distinct numbers from the set $\\{1, \\ldots, n\\}$. This way the problem statement will be established. We interpret the number $\\left(w_{i}-\\ell_{i}\\right)^{3}$ as following. For $i \\neq j$, let $\\varepsilon_{i j}=1$ if the $i$ th player wins against the $j$ th one, and $\\varepsilon_{i j}=-1$ otherwise. Then $$ \\left(w_{i}-\\ell_{i}\\right)^{3}=\\left(\\sum_{j \\neq i} \\varepsilon_{i j}\\right)^{3}=\\sum_{j_{1}, j_{2}, j_{3} \\neq i} \\varepsilon_{i j_{1}} \\varepsilon_{i j_{2}} \\varepsilon_{i j_{3}} . $$ Hence, $$ S(T)=\\sum_{i \\notin\\left\\{j_{1}, j_{2}, j_{3}\\right\\}} \\varepsilon_{i j_{1}} \\varepsilon_{i j_{2}} \\varepsilon_{i j_{3}} $$ To simplify this expression, consider all the terms in this sum where two indices are equal. If, for instance, $j_{1}=j_{2}$, then the term contains $\\varepsilon_{i j_{1}}^{2}=1$, so we can replace this term by $\\varepsilon_{i j_{3}}$. Make such replacements for each such term; obviously, after this change each term of the form $\\varepsilon_{i j_{3}}$ will appear $P(T)$ times, hence $$ S(T)=\\sum_{\\left|\\left\\{i, j_{1}, j_{2}, j_{3}\\right\\}\\right|=4} \\varepsilon_{i j_{1}} \\varepsilon_{i j_{2}} \\varepsilon_{i j_{3}}+P(T) \\sum_{i \\neq j} \\varepsilon_{i j}=S_{1}(T)+P(T) S_{2}(T) $$ We show that $S_{2}(T)=0$ and hence $S(T)=S_{1}(T)$ for each tournament. Actually, note that $\\varepsilon_{i j}=-\\varepsilon_{j i}$, and the whole sum can be split into such pairs. Since the sum in each pair is 0 , so is $S_{2}(T)$. Thus the desired equality (2) rewrites as $$ S_{1}(T)=\\sum_{i_{1}, i_{2}, i_{3}, i_{4}} S_{1}\\left(T_{i_{1} i_{2} i_{3} i_{4}}\\right) $$ Now, if all the numbers $j_{1}, j_{2}, j_{3}$ are distinct, then the set $\\left\\{i, j_{1}, j_{2}, j_{3}\\right\\}$ is contained in exactly one 4 -tuple, hence the term $\\varepsilon_{i j_{1}} \\varepsilon_{i j_{2}} \\varepsilon_{i j_{3}}$ appears in the right-hand part of (3) exactly once, as well as in the left-hand part. Clearly, there are no other terms in both parts, so the equality is established.","tier":0} +{"problem_type":"Combinatorics","problem_label":"C5","problem":"$n \\geq 4$ players participated in a tennis tournament. Any two players have played exactly one game, and there was no tie game. We call a company of four players bad if one player was defeated by the other three players, and each of these three players won a game and lost another game among themselves. Suppose that there is no bad company in this tournament. Let $w_{i}$ and $\\ell_{i}$ be respectively the number of wins and losses of the $i$ th player. Prove that $$ \\sum_{i=1}^{n}\\left(w_{i}-\\ell_{i}\\right)^{3} \\geq 0 $$ (South Korea)","solution":"Similarly to the first solution, we call the subsets of players as companies, and the $k$-element subsets will be called as $k$-companies. In any company of the players, call a player the local champion of the company if he defeated all other members of the company. Similarly, if a player lost all his games against the others in the company then call him the local loser of the company. Obviously every company has at most one local champion and at most one local loser. By the condition of the problem, whenever a 4-company has a local loser, then this company has a local champion as well. Suppose that $k$ is some positive integer, and let us count all cases when a player is the local champion of some $k$-company. The $i$ th player won against $w_{i}$ other player. To be the local champion of a $k$-company, he must be a member of the company, and the other $k-1$ members must be chosen from those whom he defeated. Therefore, the $i$ th player is the local champion of $\\binom{w_{i}}{k-1} k$-companies. Hence, the total number of local champions of all $k$-companies is $\\sum_{i=1}^{n}\\binom{w_{i}}{k-1}$. Similarly, the total number of local losers of the $k$-companies is $\\sum_{i=1}^{n}\\binom{\\ell_{i}}{k-1}$. Now apply this for $k=2,3$ and 4. Since every game has a winner and a loser, we have $\\sum_{i=1}^{n} w_{i}=\\sum_{i=1}^{n} \\ell_{i}=\\binom{n}{2}$, and hence $$ \\sum_{i=1}^{n}\\left(w_{i}-\\ell_{i}\\right)=0 $$ In every 3-company, either the players defeated one another in a cycle or the company has both a local champion and a local loser. Therefore, the total number of local champions and local losers in the 3-companies is the same, $\\sum_{i=1}^{n}\\binom{w_{i}}{2}=\\sum_{i=1}^{n}\\binom{\\ell_{i}}{2}$. So we have $$ \\sum_{i=1}^{n}\\left(\\binom{w_{i}}{2}-\\binom{\\ell_{i}}{2}\\right)=0 $$ In every 4-company, by the problem's condition, the number of local losers is less than or equal to the number of local champions. Then the same holds for the total numbers of local champions and local losers in all 4-companies, so $\\sum_{i=1}^{n}\\binom{w_{i}}{3} \\geq \\sum_{i=1}^{n}\\binom{\\ell_{i}}{3}$. Hence, $$ \\sum_{i=1}^{n}\\left(\\binom{w_{i}}{3}-\\binom{\\ell_{i}}{3}\\right) \\geq 0 $$ Now we establish the problem statement (1) as a linear combination of (4), (5) and (6). It is easy check that $$ (x-y)^{3}=24\\left(\\binom{x}{3}-\\binom{y}{3}\\right)+24\\left(\\binom{x}{2}-\\binom{y}{2}\\right)-\\left(3(x+y)^{2}-4\\right)(x-y) $$ Apply this identity to $x=w_{1}$ and $y=\\ell_{i}$. Since every player played $n-1$ games, we have $w_{i}+\\ell_{i}=n-1$, and thus $$ \\left(w_{i}-\\ell_{i}\\right)^{3}=24\\left(\\binom{w_{i}}{3}-\\binom{\\ell_{i}}{3}\\right)+24\\left(\\binom{w_{i}}{2}-\\binom{\\ell_{i}}{2}\\right)-\\left(3(n-1)^{2}-4\\right)\\left(w_{i}-\\ell_{i}\\right) $$ Then $$ \\sum_{i=1}^{n}\\left(w_{i}-\\ell_{i}\\right)^{3}=24 \\underbrace{\\sum_{i=1}^{n}\\left(\\binom{w_{i}}{3}-\\binom{\\ell_{i}}{3}\\right)}_{\\geq 0}+24 \\underbrace{\\sum_{i=1}^{n}\\left(\\binom{w_{i}}{2}-\\binom{\\ell_{i}}{2}\\right)}_{0}-\\left(3(n-1)^{2}-4\\right) \\underbrace{\\sum_{i=1}^{n}\\left(w_{i}-\\ell_{i}\\right)}_{0} \\geq 0 $$","tier":0} +{"problem_type":"Combinatorics","problem_label":"C6","problem":"Given a positive integer $k$ and other two integers $b>w>1$. There are two strings of pearls, a string of $b$ black pearls and a string of $w$ white pearls. The length of a string is the number of pearls on it. One cuts these strings in some steps by the following rules. In each step: (i) The strings are ordered by their lengths in a non-increasing order. If there are some strings of equal lengths, then the white ones precede the black ones. Then $k$ first ones (if they consist of more than one pearl) are chosen; if there are less than $k$ strings longer than 1 , then one chooses all of them. (ii) Next, one cuts each chosen string into two parts differing in length by at most one. (For instance, if there are strings of $5,4,4,2$ black pearls, strings of $8,4,3$ white pearls and $k=4$, then the strings of 8 white, 5 black, 4 white and 4 black pearls are cut into the parts $(4,4),(3,2),(2,2)$ and $(2,2)$, respectively.) The process stops immediately after the step when a first isolated white pearl appears. Prove that at this stage, there will still exist a string of at least two black pearls. (Canada)","solution":"Denote the situation after the $i$ th step by $A_{i}$; hence $A_{0}$ is the initial situation, and $A_{i-1} \\rightarrow A_{i}$ is the $i$ th step. We call a string containing $m$ pearls an $m$-string; it is an $m$-w-string or a $m$-b-string if it is white or black, respectively. We continue the process until every string consists of a single pearl. We will focus on three moments of the process: (a) the first stage $A_{s}$ when the first 1 -string (no matter black or white) appears; (b) the first stage $A_{t}$ where the total number of strings is greater than $k$ (if such moment does not appear then we put $t=\\infty$ ); and (c) the first stage $A_{f}$ when all black pearls are isolated. It is sufficient to prove that in $A_{f-1}$ (or earlier), a 1-w-string appears. We start with some easy properties of the situations under consideration. Obviously, we have $s \\leq f$. Moreover, all b-strings from $A_{f-1}$ become single pearls in the $f$ th step, hence all of them are 1- or 2-b-strings. Next, observe that in each step $A_{i} \\rightarrow A_{i+1}$ with $i \\leq t-1$, all $(>1)$-strings were cut since there are not more than $k$ strings at all; if, in addition, $i1$ as $s-1 \\leq \\min \\{s, t\\}$. Now, if $s=f$, then in $A_{s-1}$, there is no 1 -w-string as well as no ( $>2$ )-b-string. That is, $2=B_{s-1} \\geq W_{s-1} \\geq b_{s-1} \\geq w_{s-1}>1$, hence all these numbers equal 2 . This means that in $A_{s-1}$, all strings contain 2 pearls, and there are $2^{s-1}$ black and $2^{s-1}$ white strings, which means $b=2 \\cdot 2^{s-1}=w$. This contradicts the problem conditions. Hence we have $s \\leq f-1$ and thus $s \\leq t$. Therefore, in the $s$ th step each string is cut into two parts. Now, if a 1-b-string appears in this step, then from $w_{s-1} \\leq b_{s-1}$ we see that a 1-w-string appears as well; so, in each case in the sth step a 1-w-string appears, while not all black pearls become single, as desired. Case 2. Now assume that $t+1 \\leq s$ and $t+2 \\leq f$. Then in $A_{t}$ we have exactly $2^{t}$ white and $2^{t}$ black strings, all being larger than 1 , and $2^{t+1}>k \\geq 2^{t}$ (the latter holds since $2^{t}$ is the total number of strings in $A_{t-1}$ ). Now, in the $(t+1)$ st step, exactly $k$ strings are cut, not more than $2^{t}$ of them being black; so the number of w-strings in $A_{t+1}$ is at least $2^{t}+\\left(k-2^{t}\\right)=k$. Since the number of w-strings does not decrease in our process, in $A_{f-1}$ we have at least $k$ white strings as well. Finally, in $A_{f-1}$, all b-strings are not larger than 2, and at least one 2-b-string is cut in the $f$ th step. Therefore, at most $k-1$ white strings are cut in this step, hence there exists a w-string $\\mathcal{W}$ which is not cut in the $f$ th step. On the other hand, since a 2 -b-string is cut, all $(\\geq 2)$-w-strings should also be cut in the $f$ th step; hence $\\mathcal{W}$ should be a single pearl. This is exactly what we needed. Comment. In this solution, we used the condition $b \\neq w$ only to avoid the case $b=w=2^{t}$. Hence, if a number $b=w$ is not a power of 2 , then the problem statement is also valid.","tier":0} +{"problem_type":"Combinatorics","problem_label":"C6","problem":"Given a positive integer $k$ and other two integers $b>w>1$. There are two strings of pearls, a string of $b$ black pearls and a string of $w$ white pearls. The length of a string is the number of pearls on it. One cuts these strings in some steps by the following rules. In each step: (i) The strings are ordered by their lengths in a non-increasing order. If there are some strings of equal lengths, then the white ones precede the black ones. Then $k$ first ones (if they consist of more than one pearl) are chosen; if there are less than $k$ strings longer than 1 , then one chooses all of them. (ii) Next, one cuts each chosen string into two parts differing in length by at most one. (For instance, if there are strings of $5,4,4,2$ black pearls, strings of $8,4,3$ white pearls and $k=4$, then the strings of 8 white, 5 black, 4 white and 4 black pearls are cut into the parts $(4,4),(3,2),(2,2)$ and $(2,2)$, respectively.) The process stops immediately after the step when a first isolated white pearl appears. Prove that at this stage, there will still exist a string of at least two black pearls. (Canada)","solution":"We use the same notations as introduced in the first paragraph of the previous solution. We claim that at every stage, there exist a $u$-b-string and a $v$-w-string such that either (i) $u>v \\geq 1$, or (ii) $2 \\leq u \\leq v<2 u$, and there also exist $k-1$ of ( $>v \/ 2$ )-strings other than considered above. First, we notice that this statement implies the problem statement. Actually, in both cases (i) and (ii) we have $u>1$, so at each stage there exists a ( $\\geq 2$ )-b-string, and for the last stage it is exactly what we need. Now, we prove the claim by induction on the number of the stage. Obviously, for $A_{0}$ the condition (i) holds since $b>w$. Further, we suppose that the statement holds for $A_{i}$, and prove it for $A_{i+1}$. Two cases are possible. Case 1. Assume that in $A_{i}$, there are a $u$-b-string and a $v$-w-string with $u>v$. We can assume that $v$ is the length of the shortest w-string in $A_{i}$; since we are not at the final stage, we have $v \\geq 2$. Now, in the $(i+1)$ st step, two subcases may occur. Subcase 1a. Suppose that either no $u$-b-string is cut, or both some $u$-b-string and some $v$-w-string are cut. Then in $A_{i+1}$, we have either a $u$-b-string and a $(\\leq v)$-w-string (and (i) is valid), or we have a $\\lceil u \/ 2\\rceil$-b-string and a $\\lfloor v \/ 2\\rfloor$-w-string. In the latter case, from $u>v$ we get $\\lceil u \/ 2\\rceil>\\lfloor v \/ 2\\rfloor$, and (i) is valid again. Subcase $1 b$. Now, some $u$-b-string is cut, and no $v$-w-string is cut (and hence all the strings which are cut are longer than $v$ ). If $u^{\\prime}=\\lceil u \/ 2\\rceil>v$, then the condition (i) is satisfied since we have a $u^{\\prime}$-b-string and a $v$-w-string in $A_{i+1}$. Otherwise, notice that the inequality $u>v \\geq 2$ implies $u^{\\prime} \\geq 2$. Furthermore, besides a fixed $u$-b-string, other $k-1$ of $(\\geq v+1)$-strings should be cut in the $(i+1)$ st step, hence providing at least $k-1$ of $(\\geq\\lceil(v+1) \/ 2\\rceil)$-strings, and $\\lceil(v+1) \/ 2\\rceil>v \/ 2$. So, we can put $v^{\\prime}=v$, and we have $u^{\\prime} \\leq vv$, so each one results in a ( $>v \/ 2$ )string. Hence in $A_{i+1}$, there exist $k \\geq k-1$ of $(>v \/ 2)$-strings other than the considered $u$ - and $v$-strings, and the condition (ii) is satisfied. Subcase 2c. In the remaining case, all $u$-b-strings are cut. This means that all $(\\geq u)$-strings are cut as well, hence our $v$-w-string is cut. Therefore in $A_{i+1}$ there exists a $\\lceil u \/ 2\\rceil$-b-string together with a $\\lfloor v \/ 2\\rfloor$-w-string. Now, if $u^{\\prime}=\\lceil u \/ 2\\rceil>\\lfloor v \/ 2\\rfloor=v^{\\prime}$ then the condition (i) is fulfilled. Otherwise, we have $u^{\\prime} \\leq v^{\\prime}$ $|f(W)|$; if there is no such set then we set $W=\\varnothing$. Denote $W^{\\prime}=f(W), U=V \\backslash W, U^{\\prime}=V^{\\prime} \\backslash W^{\\prime}$. By our assumption and the Lemma condition, $|f(V)|=\\left|V^{\\prime}\\right| \\geq|V|$, hence $W \\neq V$ and $U \\neq \\varnothing$. Permuting the coordinates, we can assume that $U^{\\prime}=\\left\\{v_{i j}: 1 \\leq i \\leq \\ell\\right\\}, W^{\\prime}=\\left\\{v_{i j}: \\ell+1 \\leq i \\leq k\\right\\}$. Consider the induced subgraph $G^{\\prime}$ of $G$ on the vertices $U \\cup U^{\\prime}$. We claim that for every $X \\subset U$, we get $\\left|f(X) \\cap U^{\\prime}\\right| \\geq|X|$ (so $G^{\\prime}$ satisfies the conditions of Hall's lemma). Actually, we have $|W| \\geq|f(W)|$, so if $|X|>\\left|f(X) \\cap U^{\\prime}\\right|$ for some $X \\subset U$, then we have $$ |W \\cup X|=|W|+|X|>|f(W)|+\\left|f(X) \\cap U^{\\prime}\\right|=\\left|f(W) \\cup\\left(f(X) \\cap U^{\\prime}\\right)\\right|=|f(W \\cup X)| $$ This contradicts the maximality of $|W|$. Thus, applying Hall's lemma, we can assign to each $L \\in U$ some vertex $v_{i j} \\in U^{\\prime}$ so that to distinct elements of $U$, distinct vertices of $U^{\\prime}$ are assigned. In this situation, we say that $L \\in U$ corresponds to the $i$ th axis, and write $g(L)=i$. Since there are $n_{i}-1$ vertices of the form $v_{i j}$, we get that for each $1 \\leq i \\leq \\ell$, not more than $n_{i}-1$ subgrids correspond to the $i$ th axis. Finally, we are ready to present the desired point. Since $W \\neq V$, there exists a point $b=\\left(b_{1}, b_{2}, \\ldots, b_{k}\\right) \\in N \\backslash\\left(\\cup_{L \\in W} L\\right)$. On the other hand, for every $1 \\leq i \\leq \\ell$, consider any subgrid $L \\in U$ with $g(L)=i$. This means exactly that $L$ is orthogonal to the $i$ th axis, and hence all its elements have the same $i$ th coordinate $c_{L}$. Since there are at most $n_{i}-1$ such subgrids, there exists a number $0 \\leq a_{i} \\leq n_{i}-1$ which is not contained in a set $\\left\\{c_{L}: g(L)=i\\right\\}$. Choose such number for every $1 \\leq i \\leq \\ell$. Now we claim that point $a=\\left(a_{1}, \\ldots, a_{\\ell}, b_{\\ell+1}, \\ldots, b_{k}\\right)$ is not covered, hence contradicting the Lemma condition. Surely, point $a$ cannot lie in some $L \\in U$, since all the points in $L$ have $g(L)$ th coordinate $c_{L} \\neq a_{g(L)}$. On the other hand, suppose that $a \\in L$ for some $L \\in W$; recall that $b \\notin L$. But the points $a$ and $b$ differ only at first $\\ell$ coordinates, so $L$ should be orthogonal to at least one of the first $\\ell$ axes, and hence our graph contains some edge $\\left(L, v_{i j}\\right)$ for $i \\leq \\ell$. It contradicts the definition of $W^{\\prime}$. The Lemma is proved. Now we turn to the problem. Let $d_{j}$ be the step of the progression $P_{j}$. Note that since $n=$ l.c.m. $\\left(d_{1}, \\ldots, d_{s}\\right)$, for each $1 \\leq i \\leq k$ there exists an index $j(i)$ such that $p_{i}^{\\alpha_{i}} \\mid d_{j(i)}$. We assume that $n>1$; otherwise the problem statement is trivial. For each $0 \\leq m \\leq n-1$ and $1 \\leq i \\leq k$, let $m_{i}$ be the residue of $m$ modulo $p_{i}^{\\alpha_{i}}$, and let $m_{i}=\\overline{r_{i \\alpha_{i}} \\ldots r_{i 1}}$ be the base $p_{i}$ representation of $m_{i}$ (possibly, with some leading zeroes). Now, we put into correspondence to $m$ the sequence $r(m)=\\left(r_{11}, \\ldots, r_{1 \\alpha_{1}}, r_{21}, \\ldots, r_{k \\alpha_{k}}\\right)$. Hence $r(m)$ lies in a $\\underbrace{p_{1} \\times \\cdots \\times p_{1}}_{\\alpha_{1} \\text { times }} \\times \\cdots \\times \\underbrace{p_{k} \\times \\cdots \\times p_{k}}_{\\alpha_{k} \\text { times }}$ grid $N$. Surely, if $r(m)=r\\left(m^{\\prime}\\right)$ then $p_{i}^{\\alpha_{i}} \\mid m_{i}-m_{i}^{\\prime}$, which follows $p_{i}^{\\alpha_{i}} \\mid m-m^{\\prime}$ for all $1 \\leq i \\leq k$; consequently, $n \\mid m-m^{\\prime}$. So, when $m$ runs over the set $\\{0, \\ldots, n-1\\}$, the sequences $r(m)$ do not repeat; since $|N|=n$, this means that $r$ is a bijection between $\\{0, \\ldots, n-1\\}$ and $N$. Now we will show that for each $1 \\leq i \\leq s$, the set $L_{i}=\\left\\{r(m): m \\in P_{i}\\right\\}$ is a subgrid, and that for each axis there exists a subgrid orthogonal to this axis. Obviously, these subgrids cover $N$, and the condition (ii') follows directly from (ii). Hence the Lemma provides exactly the estimate we need. Consider some $1 \\leq j \\leq s$ and let $d_{j}=p_{1}^{\\gamma_{1}} \\ldots p_{k}^{\\gamma_{k}}$. Consider some $q \\in P_{j}$ and let $r(q)=$ $\\left(r_{11}, \\ldots, r_{k \\alpha_{k}}\\right)$. Then for an arbitrary $q^{\\prime}$, setting $r\\left(q^{\\prime}\\right)=\\left(r_{11}^{\\prime}, \\ldots, r_{k \\alpha_{k}}^{\\prime}\\right)$ we have $$ q^{\\prime} \\in P_{j} \\quad \\Longleftrightarrow \\quad p_{i}^{\\gamma_{i}} \\mid q-q^{\\prime} \\text { for each } 1 \\leq i \\leq k \\quad \\Longleftrightarrow \\quad r_{i, t}=r_{i, t}^{\\prime} \\text { for all } t \\leq \\gamma_{i} $$ Hence $L_{j}=\\left\\{\\left(r_{11}^{\\prime}, \\ldots, r_{k \\alpha_{k}}^{\\prime}\\right) \\in N: r_{i, t}=r_{i, t}^{\\prime}\\right.$ for all $\\left.t \\leq \\gamma_{i}\\right\\}$ which means that $L_{j}$ is a subgrid containing $r(q)$. Moreover, in $L_{j(i)}$, all the coordinates corresponding to $p_{i}$ are fixed, so it is orthogonal to all of their axes, as desired. Comment 1. The estimate in the problem is sharp for every $n$. One of the possible examples is the following one. For each $1 \\leq i \\leq k, 0 \\leq j \\leq \\alpha_{i}-1,1 \\leq k \\leq p-1$, let $$ P_{i, j, k}=k p_{i}^{j}+p_{i}^{j+1} \\mathbb{Z} $$ and add the progression $P_{0}=n \\mathbb{Z}$. One can easily check that this set satisfies all the problem conditions. There also exist other examples. On the other hand, the estimate can be adjusted in the following sense. For every $1 \\leq i \\leq k$, let $0=\\alpha_{i 0}, \\alpha_{i 1}, \\ldots, \\alpha_{i h_{i}}$ be all the numbers of the form $\\operatorname{ord}_{p_{i}}\\left(d_{j}\\right)$ in an increasing order (we delete the repeating occurences of a number, and add a number $0=\\alpha_{i 0}$ if it does not occur). Then, repeating the arguments from the solution one can obtain that $$ s \\geq 1+\\sum_{i=1}^{k} \\sum_{j=1}^{h_{i}}\\left(p^{\\alpha_{j}-\\alpha_{j-1}}-1\\right) $$ Note that $p^{\\alpha}-1 \\geq \\alpha(p-1)$, and the equality is achieved only for $\\alpha=1$. Hence, for reaching the minimal number of the progressions, one should have $\\alpha_{i, j}=j$ for all $i, j$. In other words, for each $1 \\leq j \\leq \\alpha_{i}$, there should be an index $t$ such that $\\operatorname{ord}_{p_{i}}\\left(d_{t}\\right)=j$.","tier":0} +{"problem_type":"Combinatorics","problem_label":"C7","problem":"Let $P_{1}, \\ldots, P_{s}$ be arithmetic progressions of integers, the following conditions being satisfied: (i) each integer belongs to at least one of them; (ii) each progression contains a number which does not belong to other progressions. Denote by $n$ the least common multiple of steps of these progressions; let $n=p_{1}^{\\alpha_{1}} \\ldots p_{k}^{\\alpha_{k}}$ be its prime factorization. Prove that $$ s \\geq 1+\\sum_{i=1}^{k} \\alpha_{i}\\left(p_{i}-1\\right) $$ (Germany)","solution":"We start with introducing some notation. For positive integer $r$, we denote $[r]=\\{1,2, \\ldots, r\\}$. Next, we say that a set of progressions $\\mathcal{P}=\\left\\{P_{1}, \\ldots, P_{s}\\right\\}$ cover $\\mathbb{Z}$ if each integer belongs to some of them; we say that this covering is minimal if no proper subset of $\\mathcal{P}$ covers $\\mathbb{Z}$. Obviously, each covering contains a minimal subcovering. Next, for a minimal covering $\\left\\{P_{1}, \\ldots, P_{s}\\right\\}$ and for every $1 \\leq i \\leq s$, let $d_{i}$ be the step of progression $P_{i}$, and $h_{i}$ be some number which is contained in $P_{i}$ but in none of the other progressions. We assume that $n>1$, otherwise the problem is trivial. This implies $d_{i}>1$, otherwise the progression $P_{i}$ covers all the numbers, and $n=1$. We will prove a more general statement, namely the following Claim. Assume that the progressions $P_{1}, \\ldots, P_{s}$ and number $n=p_{1}^{\\alpha_{1}} \\ldots p_{k}^{\\alpha_{k}}>1$ are chosen as in the problem statement. Moreover, choose some nonempty set of indices $I=\\left\\{i_{1}, \\ldots, i_{t}\\right\\} \\subseteq[k]$ and some positive integer $\\beta_{i} \\leq \\alpha_{i}$ for every $i \\in I$. Consider the set of indices $$ T=\\left\\{j: 1 \\leq j \\leq s, \\text { and } p_{i}^{\\alpha_{i}-\\beta_{i}+1} \\mid d_{j} \\text { for some } i \\in I\\right\\} $$ Then $$ |T| \\geq 1+\\sum_{i \\in I} \\beta_{i}\\left(p_{i}-1\\right) $$ Observe that the Claim for $I=[k]$ and $\\beta_{i}=\\alpha_{i}$ implies the problem statement, since the left-hand side in (2) is not greater than $s$. Hence, it suffices to prove the Claim. 1. First, we prove the Claim assuming that all $d_{j}$ 's are prime numbers. If for some $1 \\leq i \\leq k$ we have at least $p_{i}$ progressions with the step $p_{i}$, then they do not intersect and hence cover all the integers; it means that there are no other progressions, and $n=p_{i}$; the Claim is trivial in this case. Now assume that for every $1 \\leq i \\leq k$, there are not more than $p_{i}-1$ progressions with step $p_{i}$; each such progression covers the numbers with a fixed residue modulo $p_{i}$, therefore there exists a residue $q_{i} \\bmod p_{i}$ which is not touched by these progressions. By the Chinese Remainder Theorem, there exists a number $q$ such that $q \\equiv q_{i}\\left(\\bmod p_{i}\\right)$ for all $1 \\leq i \\leq k$; this number cannot be covered by any progression with step $p_{i}$, hence it is not covered at all. A contradiction. 2. Now, we assume that the general Claim is not valid, and hence we consider a counterexample $\\left\\{P_{1}, \\ldots, P_{s}\\right\\}$ for the Claim; we can choose it to be minimal in the following sense: - the number $n$ is minimal possible among all the counterexamples; - the sum $\\sum_{i} d_{i}$ is minimal possible among all the counterexamples having the chosen value of $n$. As was mentioned above, not all numbers $d_{i}$ are primes; hence we can assume that $d_{1}$ is composite, say $p_{1} \\mid d_{1}$ and $d_{1}^{\\prime}=\\frac{d_{1}}{p_{1}}>1$. Consider a progression $P_{1}^{\\prime}$ having the step $d_{1}^{\\prime}$, and containing $P_{1}$. We will focus on two coverings constructed as follows. (i) Surely, the progressions $P_{1}^{\\prime}, P_{2}, \\ldots, P_{s}$ cover $\\mathbb{Z}$, though this covering in not necessarily minimal. So, choose some minimal subcovering $\\mathcal{P}^{\\prime}$ in it; surely $P_{1}^{\\prime} \\in \\mathcal{P}^{\\prime}$ since $h_{1}$ is not covered by $P_{2}, \\ldots, P_{s}$, so we may assume that $\\mathcal{P}^{\\prime}=\\left\\{P_{1}^{\\prime}, P_{2}, \\ldots, P_{s^{\\prime}}\\right\\}$ for some $s^{\\prime} \\leq s$. Furthermore, the period of the covering $\\mathcal{P}^{\\prime}$ can appear to be less than $n$; so we denote this period by $$ n^{\\prime}=p_{1}^{\\alpha_{1}-\\sigma_{1}} \\ldots p_{k}^{\\alpha_{k}-\\sigma_{k}}=\\text { l.c.m. }\\left(d_{1}^{\\prime}, d_{2}, \\ldots, d_{s^{\\prime}}\\right) $$ Observe that for each $P_{j} \\notin \\mathcal{P}^{\\prime}$, we have $h_{j} \\in P_{1}^{\\prime}$, otherwise $h_{j}$ would not be covered by $\\mathcal{P}$. (ii) On the other hand, each nonempty set of the form $R_{i}=P_{i} \\cap P_{1}^{\\prime}(1 \\leq i \\leq s)$ is also a progression with a step $r_{i}=$ l.c.m. $\\left(d_{i}, d_{1}^{\\prime}\\right)$, and such sets cover $P_{1}^{\\prime}$. Scaling these progressions with the ratio $1 \/ d_{1}^{\\prime}$, we obtain the progressions $Q_{i}$ with steps $q_{i}=r_{i} \/ d_{1}^{\\prime}$ which cover $\\mathbb{Z}$. Now we choose a minimal subcovering $\\mathcal{Q}$ of this covering; again we should have $Q_{1} \\in \\mathcal{Q}$ by the reasons of $h_{1}$. Now, denote the period of $\\mathcal{Q}$ by $$ n^{\\prime \\prime}=\\text { l.c.m. }\\left\\{q_{i}: Q_{i} \\in \\mathcal{Q}\\right\\}=\\frac{\\text { l.c.m. }\\left\\{r_{i}: Q_{i} \\in \\mathcal{Q}\\right\\}}{d_{1}^{\\prime}}=\\frac{p_{1}^{\\gamma_{1}} \\ldots p_{k}^{\\gamma_{k}}}{d_{1}^{\\prime}} $$ Note that if $h_{j} \\in P_{1}^{\\prime}$, then the image of $h_{j}$ under the scaling can be covered by $Q_{j}$ only; so, in this case we have $Q_{j} \\in \\mathcal{Q}$. Our aim is to find the desired number of progressions in coverings $\\mathcal{P}$ and $\\mathcal{Q}$. First, we have $n \\geq n^{\\prime}$, and the sum of the steps in $\\mathcal{P}^{\\prime}$ is less than that in $\\mathcal{P}$; hence the Claim is valid for $\\mathcal{P}^{\\prime}$. We apply it to the set of indices $I^{\\prime}=\\left\\{i \\in I: \\beta_{i}>\\sigma_{i}\\right\\}$ and the exponents $\\beta_{i}^{\\prime}=\\beta_{i}-\\sigma_{i}$; hence the set under consideration is $$ T^{\\prime}=\\left\\{j: 1 \\leq j \\leq s^{\\prime}, \\text { and } p_{i}^{\\left(\\alpha_{i}-\\sigma_{i}\\right)-\\beta_{i}^{\\prime}+1}=p_{i}^{\\alpha_{i}-\\beta_{i}+1} \\mid d_{j} \\text { for some } i \\in I^{\\prime}\\right\\} \\subseteq T \\cap\\left[s^{\\prime}\\right], $$ and we obtain that $$ \\left|T \\cap\\left[s^{\\prime}\\right]\\right| \\geq\\left|T^{\\prime}\\right| \\geq 1+\\sum_{i \\in I^{\\prime}}\\left(\\beta_{i}-\\sigma_{i}\\right)\\left(p_{i}-1\\right)=1+\\sum_{i \\in I}\\left(\\beta_{i}-\\sigma_{i}\\right)_{+}\\left(p_{i}-1\\right) $$ where $(x)_{+}=\\max \\{x, 0\\}$; the latter equality holds as for $i \\notin I^{\\prime}$ we have $\\beta_{i} \\leq \\sigma_{i}$. Observe that $x=(x-y)_{+}+\\min \\{x, y\\}$ for all $x, y$. So, if we find at least $$ G=\\sum_{i \\in I} \\min \\left\\{\\beta_{i}, \\sigma_{i}\\right\\}\\left(p_{i}-1\\right) $$ indices in $T \\cap\\left\\{s^{\\prime}+1, \\ldots, s\\right\\}$, then we would have $$ |T|=\\left|T \\cap\\left[s^{\\prime}\\right]\\right|+\\left|T \\cap\\left\\{s^{\\prime}+1, \\ldots, s\\right\\}\\right| \\geq 1+\\sum_{i \\in I}\\left(\\left(\\beta_{i}-\\sigma_{i}\\right)_{+}+\\min \\left\\{\\beta_{i}, \\sigma_{i}\\right\\}\\right)\\left(p_{i}-1\\right)=1+\\sum_{i \\in I} \\beta_{i}\\left(p_{i}-1\\right) $$ thus leading to a contradiction with the choice of $\\mathcal{P}$. We will find those indices among the indices of progressions in $\\mathcal{Q}$. 3. Now denote $I^{\\prime \\prime}=\\left\\{i \\in I: \\sigma_{i}>0\\right\\}$ and consider some $i \\in I^{\\prime \\prime}$; then $p_{i}^{\\alpha_{i}} \\nmid n^{\\prime}$. On the other hand, there exists an index $j(i)$ such that $p_{i}^{\\alpha_{i}} \\mid d_{j(i)}$; this means that $d_{j(i)} \\backslash n^{\\prime}$ and hence $P_{j(i)}$ cannot appear in $\\mathcal{P}^{\\prime}$, so $j(i)>s^{\\prime}$. Moreover, we have observed before that in this case $h_{j(i)} \\in P_{1}^{\\prime}$, hence $Q_{j(i)} \\in \\mathcal{Q}$. This means that $q_{j(i)} \\mid n^{\\prime \\prime}$, therefore $\\gamma_{i}=\\alpha_{i}$ for each $i \\in I^{\\prime \\prime}$ (recall here that $q_{i}=r_{i} \/ d_{1}^{\\prime}$ and hence $\\left.d_{j(i)}\\left|r_{j(i)}\\right| d_{1}^{\\prime} n^{\\prime \\prime}\\right)$. Let $d_{1}^{\\prime}=p_{1}^{\\tau_{1}} \\ldots p_{k}^{\\tau_{k}}$. Then $n^{\\prime \\prime}=p_{1}^{\\gamma_{1}-\\tau_{1}} \\ldots p_{k}^{\\gamma_{i}-\\tau_{i}}$. Now, if $i \\in I^{\\prime \\prime}$, then for every $\\beta$ the condition $p_{i}^{\\left(\\gamma_{i}-\\tau_{i}\\right)-\\beta+1} \\mid q_{j}$ is equivalent to $p_{i}^{\\alpha_{i}-\\beta+1} \\mid r_{j}$. Note that $n^{\\prime \\prime} \\leq n \/ d_{1}^{\\prime}0$. So, the set under consideration is $$ \\begin{aligned} T^{\\prime \\prime} & =\\left\\{j: Q_{j} \\in \\mathcal{Q}, \\text { and } p_{i}^{\\left(\\gamma_{i}-\\tau_{i}\\right)-\\min \\left\\{\\beta_{i}, \\sigma_{i}\\right\\}+1} \\mid q_{j} \\text { for some } i \\in I^{\\prime \\prime}\\right\\} \\\\ & =\\left\\{j: Q_{j} \\in \\mathcal{Q}, \\text { and } p_{i}^{\\alpha_{i}-\\min \\left\\{\\beta_{i}, \\sigma_{i}\\right\\}+1} \\mid r_{j} \\text { for some } i \\in I^{\\prime \\prime}\\right\\}, \\end{aligned} $$ and we obtain $\\left|T^{\\prime \\prime}\\right| \\geq 1+G$. Finally, we claim that $T^{\\prime \\prime} \\subseteq T \\cap\\left(\\{1\\} \\cup\\left\\{s^{\\prime}+1, \\ldots, s\\right\\}\\right)$; then we will obtain $\\left|T \\cap\\left\\{s^{\\prime}+1, \\ldots, s\\right\\}\\right| \\geq G$, which is exactly what we need. To prove this, consider any $j \\in T^{\\prime \\prime}$. Observe first that $\\alpha_{i}-\\min \\left\\{\\beta_{i}, \\sigma_{i}\\right\\}+1>\\alpha_{i}-\\sigma_{i} \\geq \\tau_{i}$, hence from $p_{i}^{\\alpha_{i}-\\min \\left\\{\\beta_{i}, \\sigma_{i}\\right\\}+1} \\mid r_{j}=$ l.c.m. $\\left(d_{1}^{\\prime}, d_{j}\\right)$ we have $p_{i}^{\\alpha_{i}-\\min \\left\\{\\beta_{i}, \\sigma_{i}\\right\\}+1} \\mid d_{j}$, which means that $j \\in T$. Next, the exponent of $p_{i}$ in $d_{j}$ is greater than that in $n^{\\prime}$, which means that $P_{j} \\notin \\mathcal{P}^{\\prime}$. This may appear only if $j=1$ or $j>s^{\\prime}$, as desired. This completes the proof. Comment 2. A grid analogue of the Claim is also valid. It reads as following. Claim. Assume that the grid $N$ is covered by subgrids $L_{1}, L_{2}, \\ldots, L_{s}$ so that (ii') each subgrid contains a point which is not covered by other subgrids; (iii) for each coordinate axis, there exists a subgrid $L_{i}$ orthogonal to this axis. Choose some set of indices $I=\\left\\{i_{1}, \\ldots, i_{t}\\right\\} \\subset[k]$, and consider the set of indices $$ T=\\left\\{j: 1 \\leq j \\leq s \\text {, and } L_{j} \\text { is orthogonal to the } i \\text { th axis for some } i \\in I\\right\\} $$ Then $$ |T| \\geq 1+\\sum_{i \\in I}\\left(n_{i}-1\\right) $$ This Claim may be proved almost in the same way as in Solution 1.","tier":0} +{"problem_type":"Geometry","problem_label":"G1","problem":"Let $A B C$ be an acute triangle with $D, E, F$ the feet of the altitudes lying on $B C, C A, A B$ respectively. One of the intersection points of the line $E F$ and the circumcircle is $P$. The lines $B P$ and $D F$ meet at point $Q$. Prove that $A P=A Q$. (United Kingdom)","solution":"The line $E F$ intersects the circumcircle at two points. Depending on the choice of $P$, there are two different cases to consider. Case 1: The point $P$ lies on the ray $E F$ (see Fig. 1). Let $\\angle C A B=\\alpha, \\angle A B C=\\beta$ and $\\angle B C A=\\gamma$. The quadrilaterals $B C E F$ and $C A F D$ are cyclic due to the right angles at $D, E$ and $F$. So, $$ \\begin{aligned} & \\angle B D F=180^{\\circ}-\\angle F D C=\\angle C A F=\\alpha, \\\\ & \\angle A F E=180^{\\circ}-\\angle E F B=\\angle B C E=\\gamma, \\\\ & \\angle D F B=180^{\\circ}-\\angle A F D=\\angle D C A=\\gamma . \\end{aligned} $$ Since $P$ lies on the arc $A B$ of the circumcircle, $\\angle P B A<\\angle B C A=\\gamma$. Hence, we have $$ \\angle P B D+\\angle B D F=\\angle P B A+\\angle A B D+\\angle B D F<\\gamma+\\beta+\\alpha=180^{\\circ}, $$ and the point $Q$ must lie on the extensions of $B P$ and $D F$ beyond the points $P$ and $F$, respectively. From the cyclic quadrilateral $A P B C$ we get $$ \\angle Q P A=180^{\\circ}-\\angle A P B=\\angle B C A=\\gamma=\\angle D F B=\\angle Q F A . $$ Hence, the quadrilateral $A Q P F$ is cyclic. Then $\\angle A Q P=180^{\\circ}-\\angle P F A=\\angle A F E=\\gamma$. We obtained that $\\angle A Q P=\\angle Q P A=\\gamma$, so the triangle $A Q P$ is isosceles, $A P=A Q$. ![](https:\/\/cdn.mathpix.com\/cropped\/2024_11_18_ba86565be3364967ae6fg-45.jpg?height=680&width=763&top_left_y=2007&top_left_x=361) Fig. 1 ![](https:\/\/cdn.mathpix.com\/cropped\/2024_11_18_ba86565be3364967ae6fg-45.jpg?height=549&width=638&top_left_y=2127&top_left_x=1143) Fig. 2 Case 2: The point $P$ lies on the ray $F E$ (see Fig. 2). In this case the point $Q$ lies inside the segment $F D$. Similarly to the first case, we have $$ \\angle Q P A=\\angle B C A=\\gamma=\\angle D F B=180^{\\circ}-\\angle A F Q $$ Hence, the quadrilateral $A F Q P$ is cyclic. Then $\\angle A Q P=\\angle A F P=\\angle A F E=\\gamma=\\angle Q P A$. The triangle $A Q P$ is isosceles again, $\\angle A Q P=\\angle Q P A$ and thus $A P=A Q$. Comment. Using signed angles, the two possible configurations can be handled simultaneously, without investigating the possible locations of $P$ and $Q$.","tier":0} +{"problem_type":"Geometry","problem_label":"G1","problem":"Let $A B C$ be an acute triangle with $D, E, F$ the feet of the altitudes lying on $B C, C A, A B$ respectively. One of the intersection points of the line $E F$ and the circumcircle is $P$. The lines $B P$ and $D F$ meet at point $Q$. Prove that $A P=A Q$. (United Kingdom)","solution":"For arbitrary points $X, Y$ on the circumcircle, denote by $\\widehat{X Y}$ the central angle of the arc $X Y$. Let $P$ and $P^{\\prime}$ be the two points where the line $E F$ meets the circumcircle; let $P$ lie on the $\\operatorname{arc} A B$ and let $P^{\\prime}$ lie on the $\\operatorname{arc} C A$. Let $B P$ and $B P^{\\prime}$ meet the line $D F$ and $Q$ and $Q^{\\prime}$, respectively (see Fig. 3). We will prove that $A P=A P^{\\prime}=A Q=A Q^{\\prime}$. ![](https:\/\/cdn.mathpix.com\/cropped\/2024_11_18_ba86565be3364967ae6fg-46.jpg?height=667&width=760&top_left_y=1197&top_left_x=628) Fig. 3 Like in the first solution, we have $\\angle A F E=\\angle B F P=\\angle D F B=\\angle B C A=\\gamma$ from the cyclic quadrilaterals $B C E F$ and $C A F D$. By $\\overparen{P B}+\\overparen{P^{\\prime} A}=2 \\angle A F P^{\\prime}=2 \\gamma=2 \\angle B C A=\\overparen{A P}+\\overparen{P B}$, we have $$ \\overline{A P}=\\widetilde{P^{\\prime} A}, \\quad \\angle P B A=\\angle A B P^{\\prime} \\quad \\text { and } \\quad A P=A P^{\\prime} $$ Due to $\\overparen{A P}=\\widehat{P^{\\prime}} A$, the lines $B P$ and $B Q^{\\prime}$ are symmetrical about line $A B$. Similarly, by $\\angle B F P=\\angle Q^{\\prime} F B$, the lines $F P$ and $F Q^{\\prime}$ are symmetrical about $A B$. It follows that also the points $P$ and $P^{\\prime}$ are symmetrical to $Q^{\\prime}$ and $Q$, respectively. Therefore, $$ A P=A Q^{\\prime} \\quad \\text { and } \\quad A P^{\\prime}=A Q $$ The relations (1) and (2) together prove $A P=A P^{\\prime}=A Q=A Q^{\\prime}$.","tier":0} +{"problem_type":"Geometry","problem_label":"G2","problem":"Point $P$ lies inside triangle $A B C$. Lines $A P, B P, C P$ meet the circumcircle of $A B C$ again at points $K, L, M$, respectively. The tangent to the circumcircle at $C$ meets line $A B$ at $S$. Prove that $S C=S P$ if and only if $M K=M L$.","solution":"We assume that $C A>C B$, so point $S$ lies on the ray $A B$. From the similar triangles $\\triangle P K M \\sim \\triangle P C A$ and $\\triangle P L M \\sim \\triangle P C B$ we get $\\frac{P M}{K M}=\\frac{P A}{C A}$ and $\\frac{L M}{P M}=\\frac{C B}{P B}$. Multiplying these two equalities, we get $$ \\frac{L M}{K M}=\\frac{C B}{C A} \\cdot \\frac{P A}{P B} $$ Hence, the relation $M K=M L$ is equivalent to $\\frac{C B}{C A}=\\frac{P B}{P A}$. Denote by $E$ the foot of the bisector of angle $B$ in triangle $A B C$. Recall that the locus of points $X$ for which $\\frac{X A}{X B}=\\frac{C A}{C B}$ is the Apollonius circle $\\Omega$ with the center $Q$ on the line $A B$, and this circle passes through $C$ and $E$. Hence, we have $M K=M L$ if and only if $P$ lies on $\\Omega$, that is $Q P=Q C$. ![](https:\/\/cdn.mathpix.com\/cropped\/2024_11_18_ba86565be3364967ae6fg-47.jpg?height=692&width=1086&top_left_y=1387&top_left_x=522) Fig. 1 Now we prove that $S=Q$, thus establishing the problem statement. We have $\\angle C E S=$ $\\angle C A E+\\angle A C E=\\angle B C S+\\angle E C B=\\angle E C S$, so $S C=S E$. Hence, the point $S$ lies on $A B$ as well as on the perpendicular bisector of $C E$ and therefore coincides with $Q$.","tier":0} +{"problem_type":"Geometry","problem_label":"G2","problem":"Point $P$ lies inside triangle $A B C$. Lines $A P, B P, C P$ meet the circumcircle of $A B C$ again at points $K, L, M$, respectively. The tangent to the circumcircle at $C$ meets line $A B$ at $S$. Prove that $S C=S P$ if and only if $M K=M L$.","solution":"As in the previous solution, we assume that $S$ lies on the ray $A B$. 1. Let $P$ be an arbitrary point inside both the circumcircle $\\omega$ of the triangle $A B C$ and the angle $A S C$, the points $K, L, M$ defined as in the problem. We claim that $S P=S C$ implies $M K=M L$. Let $E$ and $F$ be the points of intersection of the line $S P$ with $\\omega$, point $E$ lying on the segment $S P$ (see Fig. 2). ![](https:\/\/cdn.mathpix.com\/cropped\/2024_11_18_ba86565be3364967ae6fg-48.jpg?height=641&width=658&top_left_y=288&top_left_x=679) Fig. 2 We have $S P^{2}=S C^{2}=S A \\cdot S B$, so $\\frac{S P}{S B}=\\frac{S A}{S P}$, and hence $\\triangle P S A \\sim \\triangle B S P$. Then $\\angle B P S=\\angle S A P$. Since $2 \\angle B P S=\\overparen{B E}+\\overparen{L F}$ and $2 \\angle S A P=\\overparen{B E}+\\overparen{E K}$ we have $$ \\overparen{L F}=\\overparen{E K} $$ On the other hand, from $\\angle S P C=\\angle S C P$ we have $\\widehat{E C}+\\widehat{M F}=\\widehat{E C}+\\widehat{E M}$, or $$ \\widetilde{M F}=\\overparen{E M} . $$ From (1) and (2) we get $\\widehat{M F L}=\\widehat{M F}+\\widehat{F L}=\\widehat{M E}+\\widehat{E K}=\\widehat{M E K}$ and hence $M K=M L$. The claim is proved. 2. We are left to prove the converse. So, assume that $M K=M L$, and introduce the points $E$ and $F$ as above. We have $S C^{2}=S E \\cdot S F$; hence, there exists a point $P^{\\prime}$ lying on the segment $E F$ such that $S P^{\\prime}=S C$ (see Fig. 3). ![](https:\/\/cdn.mathpix.com\/cropped\/2024_11_18_ba86565be3364967ae6fg-48.jpg?height=743&width=1154&top_left_y=1933&top_left_x=431) Fig. 3 Assume that $P \\neq P^{\\prime}$. Let the lines $A P^{\\prime}, B P^{\\prime}, C P^{\\prime}$ meet $\\omega$ again at points $K^{\\prime}, L^{\\prime}, M^{\\prime}$ respectively. Now, if $P^{\\prime}$ lies on the segment $P F$ then by the first part of the solution we have $\\widehat{M^{\\prime} F L^{\\prime}}=\\widehat{M^{\\prime} E K^{\\prime}}$. On the other hand, we have $\\widehat{M F L}>\\widehat{M^{\\prime} F L^{\\prime}}=\\widehat{M^{\\prime} E K^{\\prime}}>\\widehat{M E K}$, therefore $\\widehat{M F L}>\\widehat{M E K}$ which contradicts $M K=M L$. Similarly, if point $P^{\\prime}$ lies on the segment $E P$ then we get $\\widehat{M F L}<\\widehat{M E K}$ which is impossible. Therefore, the points $P$ and $P^{\\prime}$ coincide and hence $S P=S P^{\\prime}=S C$.","tier":0} +{"problem_type":"Geometry","problem_label":"G2","problem":"Point $P$ lies inside triangle $A B C$. Lines $A P, B P, C P$ meet the circumcircle of $A B C$ again at points $K, L, M$, respectively. The tangent to the circumcircle at $C$ meets line $A B$ at $S$. Prove that $S C=S P$ if and only if $M K=M L$.","solution":"We present a different proof of the converse direction, that is, $M K=M L \\Rightarrow$ $S P=S C$. As in the previous solutions we assume that $C A>C B$, and the line $S P$ meets $\\omega$ at $E$ and $F$. From $M L=M K$ we get $\\widehat{M E K}=\\widehat{M F L}$. Now we claim that $\\widehat{M E}=\\widehat{M F}$ and $\\widehat{E K}=\\widehat{F L}$. To the contrary, suppose first that $\\widehat{M E}>\\widehat{M F}$; then $\\widehat{E K}=\\widehat{M E K}-\\widehat{M E}<\\widehat{M F L}-\\widehat{M F}=$ $\\overparen{F L}$. Now, the inequality $\\overparen{M E}>\\overparen{M F}$ implies $2 \\angle S C M=\\overparen{E C}+\\overparen{M E}>\\overparen{E C}+\\overparen{M F}=2 \\angle S P C$ and hence $S P>S C$. On the other hand, the inequality $\\overparen{E K}<\\overparen{F L}$ implies $2 \\angle S P K=$ $\\overparen{E K}+\\widetilde{A F}<\\overparen{F L}+\\widetilde{A F}=2 \\angle A B L$, hence $$ \\angle S P A=180^{\\circ}-\\angle S P K>180^{\\circ}-\\angle A B L=\\angle S B P . $$ ![](https:\/\/cdn.mathpix.com\/cropped\/2024_11_18_ba86565be3364967ae6fg-49.jpg?height=712&width=1098&top_left_y=1226&top_left_x=519) Fig. 4 Consider the point $A^{\\prime}$ on the ray $S A$ for which $\\angle S P A^{\\prime}=\\angle S B P$; in our case, this point lies on the segment $S A$ (see Fig. 4). Then $\\triangle S B P \\sim \\triangle S P A^{\\prime}$ and $S P^{2}=S B \\cdot S A^{\\prime}S C$. Similarly, one can prove that the inequality $\\widehat{M E}<\\widehat{M F}$ is also impossible. So, we get $\\widehat{M E}=\\widehat{M F}$ and therefore $2 \\angle S C M=\\widehat{E C}+\\widehat{M E}=\\widehat{E C}+\\widehat{M F}=2 \\angle S P C$, which implies $S C=S P$.","tier":0} +{"problem_type":"Geometry","problem_label":"G3","problem":"Let $A_{1} A_{2} \\ldots A_{n}$ be a convex polygon. Point $P$ inside this polygon is chosen so that its projections $P_{1}, \\ldots, P_{n}$ onto lines $A_{1} A_{2}, \\ldots, A_{n} A_{1}$ respectively lie on the sides of the polygon. Prove that for arbitrary points $X_{1}, \\ldots, X_{n}$ on sides $A_{1} A_{2}, \\ldots, A_{n} A_{1}$ respectively, $$ \\max \\left\\{\\frac{X_{1} X_{2}}{P_{1} P_{2}}, \\ldots, \\frac{X_{n} X_{1}}{P_{n} P_{1}}\\right\\} \\geq 1 $$ (Armenia)","solution":"Denote $P_{n+1}=P_{1}, X_{n+1}=X_{1}, A_{n+1}=A_{1}$. Lemma. Let point $Q$ lies inside $A_{1} A_{2} \\ldots A_{n}$. Then it is contained in at least one of the circumcircles of triangles $X_{1} A_{2} X_{2}, \\ldots, X_{n} A_{1} X_{1}$. Proof. If $Q$ lies in one of the triangles $X_{1} A_{2} X_{2}, \\ldots, X_{n} A_{1} X_{1}$, the claim is obvious. Otherwise $Q$ lies inside the polygon $X_{1} X_{2} \\ldots X_{n}$ (see Fig. 1). Then we have $$ \\begin{aligned} & \\left(\\angle X_{1} A_{2} X_{2}+\\angle X_{1} Q X_{2}\\right)+\\cdots+\\left(\\angle X_{n} A_{1} X_{1}+\\angle X_{n} Q X_{1}\\right) \\\\ & \\quad=\\left(\\angle X_{1} A_{1} X_{2}+\\cdots+\\angle X_{n} A_{1} X_{1}\\right)+\\left(\\angle X_{1} Q X_{2}+\\cdots+\\angle X_{n} Q X_{1}\\right)=(n-2) \\pi+2 \\pi=n \\pi \\end{aligned} $$ hence there exists an index $i$ such that $\\angle X_{i} A_{i+1} X_{i+1}+\\angle X_{i} Q X_{i+1} \\geq \\frac{\\pi n}{n}=\\pi$. Since the quadrilateral $Q X_{i} A_{i+1} X_{i+1}$ is convex, this means exactly that $Q$ is contained the circumcircle of $\\triangle X_{i} A_{i+1} X_{i+1}$, as desired. Now we turn to the solution. Applying lemma, we get that $P$ lies inside the circumcircle of triangle $X_{i} A_{i+1} X_{i+1}$ for some $i$. Consider the circumcircles $\\omega$ and $\\Omega$ of triangles $P_{i} A_{i+1} P_{i+1}$ and $X_{i} A_{i+1} X_{i+1}$ respectively (see Fig. 2); let $r$ and $R$ be their radii. Then we get $2 r=A_{i+1} P \\leq 2 R$ (since $P$ lies inside $\\Omega$ ), hence $$ P_{i} P_{i+1}=2 r \\sin \\angle P_{i} A_{i+1} P_{i+1} \\leq 2 R \\sin \\angle X_{i} A_{i+1} X_{i+1}=X_{i} X_{i+1} $$ QED. ![](https:\/\/cdn.mathpix.com\/cropped\/2024_11_18_ba86565be3364967ae6fg-51.jpg?height=547&width=566&top_left_y=2011&top_left_x=508) Fig. 1 ![](https:\/\/cdn.mathpix.com\/cropped\/2024_11_18_ba86565be3364967ae6fg-51.jpg?height=458&width=527&top_left_y=2098&top_left_x=1093) Fig. 2","tier":0} +{"problem_type":"Geometry","problem_label":"G3","problem":"Let $A_{1} A_{2} \\ldots A_{n}$ be a convex polygon. Point $P$ inside this polygon is chosen so that its projections $P_{1}, \\ldots, P_{n}$ onto lines $A_{1} A_{2}, \\ldots, A_{n} A_{1}$ respectively lie on the sides of the polygon. Prove that for arbitrary points $X_{1}, \\ldots, X_{n}$ on sides $A_{1} A_{2}, \\ldots, A_{n} A_{1}$ respectively, $$ \\max \\left\\{\\frac{X_{1} X_{2}}{P_{1} P_{2}}, \\ldots, \\frac{X_{n} X_{1}}{P_{n} P_{1}}\\right\\} \\geq 1 $$ (Armenia)","solution":"As in We will prove a bit stronger inequality, namely $$ \\max \\left\\{\\frac{X_{1} X_{2}}{P_{1} P_{2}} \\cos \\alpha_{1}, \\ldots, \\frac{X_{n} X_{1}}{P_{n} P_{1}} \\cos \\alpha_{n}\\right\\} \\geq 1 $$ where $\\alpha_{i}(1 \\leq i \\leq n)$ is the angle between lines $X_{i} X_{i+1}$ and $P_{i} P_{i+1}$. We denote $\\beta_{i}=\\angle A_{i} P_{i} P_{i-1}$ and $\\gamma_{i}=\\angle A_{i+1} P_{i} P_{i+1}$ for all $1 \\leq i \\leq n$. Suppose that for some $1 \\leq i \\leq n$, point $X_{i}$ lies on the segment $A_{i} P_{i}$, while point $X_{i+1}$ lies on the segment $P_{i+1} A_{i+2}$. Then the projection of the segment $X_{i} X_{i+1}$ onto the line $P_{i} P_{i+1}$ contains segment $P_{i} P_{i+1}$, since $\\gamma_{i}$ and $\\beta_{i+1}$ are acute angles (see Fig. 3). Therefore, $X_{i} X_{i+1} \\cos \\alpha_{i} \\geq$ $P_{i} P_{i+1}$, and in this case the statement is proved. So, the only case left is when point $X_{i}$ lies on segment $P_{i} A_{i+1}$ for all $1 \\leq i \\leq n$ (the case when each $X_{i}$ lies on segment $A_{i} P_{i}$ is completely analogous). Now, assume to the contrary that the inequality $$ X_{i} X_{i+1} \\cos \\alpha_{i}P_{i+1} Y_{i+1}^{\\prime}$ (again since $\\gamma_{i}$ and $\\beta_{i+1}$ are acute; see Fig. 4). Hence, we have $$ X_{i} P_{i} \\cos \\gamma_{i}>X_{i+1} P_{i+1} \\cos \\beta_{i+1}, \\quad 1 \\leq i \\leq n $$ Multiplying these inequalities, we get $$ \\cos \\gamma_{1} \\cos \\gamma_{2} \\cdots \\cos \\gamma_{n}>\\cos \\beta_{1} \\cos \\beta_{2} \\cdots \\cos \\beta_{n} $$ On the other hand, the sines theorem applied to triangle $P P_{i} P_{i+1}$ provides $$ \\frac{P P_{i}}{P P_{i+1}}=\\frac{\\sin \\left(\\frac{\\pi}{2}-\\beta_{i+1}\\right)}{\\sin \\left(\\frac{\\pi}{2}-\\gamma_{i}\\right)}=\\frac{\\cos \\beta_{i+1}}{\\cos \\gamma_{i}} $$ Multiplying these equalities we get $$ 1=\\frac{\\cos \\beta_{2}}{\\cos \\gamma_{1}} \\cdot \\frac{\\cos \\beta_{3}}{\\cos \\gamma_{2}} \\cdots \\frac{\\cos \\beta_{1}}{\\cos \\gamma_{n}} $$ which contradicts (2). ![](https:\/\/cdn.mathpix.com\/cropped\/2024_11_18_ba86565be3364967ae6fg-52.jpg?height=817&width=1297&top_left_y=1873&top_left_x=351) Fig. 3 Fig. 4","tier":0} +{"problem_type":"Geometry","problem_label":"G4","problem":"Let $I$ be the incenter of a triangle $A B C$ and $\\Gamma$ be its circumcircle. Let the line $A I$ intersect $\\Gamma$ at a point $D \\neq A$. Let $F$ and $E$ be points on side $B C$ and $\\operatorname{arc} B D C$ respectively such that $\\angle B A F=\\angle C A E<\\frac{1}{2} \\angle B A C$. Finally, let $G$ be the midpoint of the segment $I F$. Prove that the lines $D G$ and $E I$ intersect on $\\Gamma$. (Hong Kong)","solution":"Let $X$ be the second point of intersection of line $E I$ with $\\Gamma$, and $L$ be the foot of the bisector of angle $B A C$. Let $G^{\\prime}$ and $T$ be the points of intersection of segment $D X$ with lines $I F$ and $A F$, respectively. We are to prove that $G=G^{\\prime}$, or $I G^{\\prime}=G^{\\prime} F$. By the Menelaus theorem applied to triangle $A I F$ and line $D X$, it means that we need the relation $$ 1=\\frac{G^{\\prime} F}{I G^{\\prime}}=\\frac{T F}{A T} \\cdot \\frac{A D}{I D}, \\quad \\text { or } \\quad \\frac{T F}{A T}=\\frac{I D}{A D} $$ Let the line $A F$ intersect $\\Gamma$ at point $K \\neq A$ (see Fig. 1); since $\\angle B A K=\\angle C A E$ we have $\\widehat{B K}=\\widehat{C E}$, hence $K E \\| B C$. Notice that $\\angle I A T=\\angle D A K=\\angle E A D=\\angle E X D=\\angle I X T$, so the points $I, A, X, T$ are concyclic. Hence we have $\\angle I T A=\\angle I X A=\\angle E X A=\\angle E K A$, so $I T\\|K E\\| B C$. Therefore we obtain $\\frac{T F}{A T}=\\frac{I L}{A I}$. Since $C I$ is the bisector of $\\angle A C L$, we get $\\frac{I L}{A I}=\\frac{C L}{A C}$. Furthermore, $\\angle D C L=\\angle D C B=$ $\\angle D A B=\\angle C A D=\\frac{1}{2} \\angle B A C$, hence the triangles $D C L$ and $D A C$ are similar; therefore we get $\\frac{C L}{A C}=\\frac{D C}{A D}$. Finally, it is known that the midpoint $D$ of $\\operatorname{arc} B C$ is equidistant from points $I$, $B, C$, hence $\\frac{D C}{A D}=\\frac{I D}{A D}$. Summarizing all these equalities, we get $$ \\frac{T F}{A T}=\\frac{I L}{A I}=\\frac{C L}{A C}=\\frac{D C}{A D}=\\frac{I D}{A D} $$ as desired. ![](https:\/\/cdn.mathpix.com\/cropped\/2024_11_18_ba86565be3364967ae6fg-53.jpg?height=767&width=715&top_left_y=1838&top_left_x=336) Fig. 1 ![](https:\/\/cdn.mathpix.com\/cropped\/2024_11_18_ba86565be3364967ae6fg-53.jpg?height=775&width=530&top_left_y=1828&top_left_x=1254) Fig. 2 Comment. The equality $\\frac{A I}{I L}=\\frac{A D}{D I}$ is known and can be obtained in many different ways. For instance, one can consider the inversion with center $D$ and radius $D C=D I$. This inversion takes $\\widehat{B A C}$ to the segment $B C$, so point $A$ goes to $L$. Hence $\\frac{I L}{D I}=\\frac{A I}{A D}$, which is the desired equality.","tier":0} +{"problem_type":"Geometry","problem_label":"G4","problem":"Let $I$ be the incenter of a triangle $A B C$ and $\\Gamma$ be its circumcircle. Let the line $A I$ intersect $\\Gamma$ at a point $D \\neq A$. Let $F$ and $E$ be points on side $B C$ and $\\operatorname{arc} B D C$ respectively such that $\\angle B A F=\\angle C A E<\\frac{1}{2} \\angle B A C$. Finally, let $G$ be the midpoint of the segment $I F$. Prove that the lines $D G$ and $E I$ intersect on $\\Gamma$. (Hong Kong)","solution":"As in the previous solution, we introduce the points $X, T$ and $K$ and note that it suffice to prove the equality $$ \\frac{T F}{A T}=\\frac{D I}{A D} \\quad \\Longleftrightarrow \\quad \\frac{T F+A T}{A T}=\\frac{D I+A D}{A D} \\quad \\Longleftrightarrow \\quad \\frac{A T}{A D}=\\frac{A F}{D I+A D} $$ Since $\\angle F A D=\\angle E A I$ and $\\angle T D A=\\angle X D A=\\angle X E A=\\angle I E A$, we get that the triangles $A T D$ and $A I E$ are similar, therefore $\\frac{A T}{A D}=\\frac{A I}{A E}$. Next, we also use the relation $D B=D C=D I$. Let $J$ be the point on the extension of segment $A D$ over point $D$ such that $D J=D I=D C$ (see Fig. 2). Then $\\angle D J C=$ $\\angle J C D=\\frac{1}{2}(\\pi-\\angle J D C)=\\frac{1}{2} \\angle A D C=\\frac{1}{2} \\angle A B C=\\angle A B I$. Moreover, $\\angle B A I=\\angle J A C$, hence triangles $A B I$ and $A J C$ are similar, so $\\frac{A B}{A J}=\\frac{A I}{A C}$, or $A B \\cdot A C=A J \\cdot A I=(D I+A D) \\cdot A I$. On the other hand, we get $\\angle A B F=\\angle A B C=\\angle A E C$ and $\\angle B A F=\\angle C A E$, so triangles $A B F$ and $A E C$ are also similar, which implies $\\frac{A F}{A C}=\\frac{A B}{A E}$, or $A B \\cdot A C=A F \\cdot A E$. Summarizing we get $$ (D I+A D) \\cdot A I=A B \\cdot A C=A F \\cdot A E \\quad \\Rightarrow \\quad \\frac{A I}{A E}=\\frac{A F}{A D+D I} \\quad \\Rightarrow \\quad \\frac{A T}{A D}=\\frac{A F}{A D+D I} $$ as desired. Comment. In fact, point $J$ is an excenter of triangle $A B C$.","tier":0} +{"problem_type":"Geometry","problem_label":"G5","problem":"Let $A B C D E$ be a convex pentagon such that $B C \\| A E, A B=B C+A E$, and $\\angle A B C=$ $\\angle C D E$. Let $M$ be the midpoint of $C E$, and let $O$ be the circumcenter of triangle $B C D$. Given that $\\angle D M O=90^{\\circ}$, prove that $2 \\angle B D A=\\angle C D E$. (Ukraine)","solution":"Choose point $T$ on ray $A E$ such that $A T=A B$; then from $A E \\| B C$ we have $\\angle C B T=\\angle A T B=\\angle A B T$, so $B T$ is the bisector of $\\angle A B C$. On the other hand, we have $E T=A T-A E=A B-A E=B C$, hence quadrilateral $B C T E$ is a parallelogram, and the midpoint $M$ of its diagonal $C E$ is also the midpoint of the other diagonal $B T$. Next, let point $K$ be symmetrical to $D$ with respect to $M$. Then $O M$ is the perpendicular bisector of segment $D K$, and hence $O D=O K$, which means that point $K$ lies on the circumcircle of triangle $B C D$. Hence we have $\\angle B D C=\\angle B K C$. On the other hand, the angles $B K C$ and $T D E$ are symmetrical with respect to $M$, so $\\angle T D E=\\angle B K C=\\angle B D C$. Therefore, $\\angle B D T=\\angle B D E+\\angle E D T=\\angle B D E+\\angle B D C=\\angle C D E=\\angle A B C=180^{\\circ}-$ $\\angle B A T$. This means that the points $A, B, D, T$ are concyclic, and hence $\\angle A D B=\\angle A T B=$ $\\frac{1}{2} \\angle A B C=\\frac{1}{2} \\angle C D E$, as desired. ![](https:\/\/cdn.mathpix.com\/cropped\/2024_11_18_ba86565be3364967ae6fg-55.jpg?height=621&width=1276&top_left_y=1140&top_left_x=425)","tier":0} +{"problem_type":"Geometry","problem_label":"G5","problem":"Let $A B C D E$ be a convex pentagon such that $B C \\| A E, A B=B C+A E$, and $\\angle A B C=$ $\\angle C D E$. Let $M$ be the midpoint of $C E$, and let $O$ be the circumcenter of triangle $B C D$. Given that $\\angle D M O=90^{\\circ}$, prove that $2 \\angle B D A=\\angle C D E$. (Ukraine)","solution":"Let $\\angle C B D=\\alpha, \\angle B D C=\\beta, \\angle A D E=\\gamma$, and $\\angle A B C=\\angle C D E=2 \\varphi$. Then we have $\\angle A D B=2 \\varphi-\\beta-\\gamma, \\angle B C D=180^{\\circ}-\\alpha-\\beta, \\angle A E D=360^{\\circ}-\\angle B C D-\\angle C D E=$ $180^{\\circ}-2 \\varphi+\\alpha+\\beta$, and finally $\\angle D A E=180^{\\circ}-\\angle A D E-\\angle A E D=2 \\varphi-\\alpha-\\beta-\\gamma$. ![](https:\/\/cdn.mathpix.com\/cropped\/2024_11_18_ba86565be3364967ae6fg-55.jpg?height=530&width=1039&top_left_y=2008&top_left_x=546) Let $N$ be the midpoint of $C D$; then $\\angle D N O=90^{\\circ}=\\angle D M O$, hence points $M, N$ lie on the circle with diameter $O D$. Now, if points $O$ and $M$ lie on the same side of $C D$, we have $\\angle D M N=\\angle D O N=\\frac{1}{2} \\angle D O C=\\alpha$; in the other case, we have $\\angle D M N=180^{\\circ}-\\angle D O N=\\alpha ;$ so, in both cases $\\angle D M N=\\alpha$ (see Figures). Next, since $M N$ is a midline in triangle $C D E$, we have $\\angle M D E=\\angle D M N=\\alpha$ and $\\angle N D M=2 \\varphi-\\alpha$. Now we apply the sine rule to the triangles $A B D, A D E$ (twice), $B C D$ and $M N D$ obtaining $$ \\begin{gathered} \\frac{A B}{A D}=\\frac{\\sin (2 \\varphi-\\beta-\\gamma)}{\\sin (2 \\varphi-\\alpha)}, \\quad \\frac{A E}{A D}=\\frac{\\sin \\gamma}{\\sin (2 \\varphi-\\alpha-\\beta)}, \\quad \\frac{D E}{A D}=\\frac{\\sin (2 \\varphi-\\alpha-\\beta-\\gamma)}{\\sin (2 \\varphi-\\alpha-\\beta)} \\\\ \\frac{B C}{C D}=\\frac{\\sin \\beta}{\\sin \\alpha}, \\quad \\frac{C D}{D E}=\\frac{C D \/ 2}{D E \/ 2}=\\frac{N D}{N M}=\\frac{\\sin \\alpha}{\\sin (2 \\varphi-\\alpha)} \\end{gathered} $$ which implies $$ \\frac{B C}{A D}=\\frac{B C}{C D} \\cdot \\frac{C D}{D E} \\cdot \\frac{D E}{A D}=\\frac{\\sin \\beta \\cdot \\sin (2 \\varphi-\\alpha-\\beta-\\gamma)}{\\sin (2 \\varphi-\\alpha) \\cdot \\sin (2 \\varphi-\\alpha-\\beta)} $$ Hence, the condition $A B=A E+B C$, or equivalently $\\frac{A B}{A D}=\\frac{A E+B C}{A D}$, after multiplying by the common denominator rewrites as $$ \\begin{gathered} \\sin (2 \\varphi-\\alpha-\\beta) \\cdot \\sin (2 \\varphi-\\beta-\\gamma)=\\sin \\gamma \\cdot \\sin (2 \\varphi-\\alpha)+\\sin \\beta \\cdot \\sin (2 \\varphi-\\alpha-\\beta-\\gamma) \\\\ \\Longleftrightarrow \\cos (\\gamma-\\alpha)-\\cos (4 \\varphi-2 \\beta-\\alpha-\\gamma)=\\cos (2 \\varphi-\\alpha-2 \\beta-\\gamma)-\\cos (2 \\varphi+\\gamma-\\alpha) \\\\ \\Longleftrightarrow \\cos (\\gamma-\\alpha)+\\cos (2 \\varphi+\\gamma-\\alpha)=\\cos (2 \\varphi-\\alpha-2 \\beta-\\gamma)+\\cos (4 \\varphi-2 \\beta-\\alpha-\\gamma) \\\\ \\Longleftrightarrow \\cos \\varphi \\cdot \\cos (\\varphi+\\gamma-\\alpha)=\\cos \\varphi \\cdot \\cos (3 \\varphi-2 \\beta-\\alpha-\\gamma) \\\\ \\Longleftrightarrow \\cos \\varphi \\cdot(\\cos (\\varphi+\\gamma-\\alpha)-\\cos (3 \\varphi-2 \\beta-\\alpha-\\gamma))=0 \\\\ \\Longleftrightarrow \\cos \\varphi \\cdot \\sin (2 \\varphi-\\beta-\\alpha) \\cdot \\sin (\\varphi-\\beta-\\gamma)=0 \\end{gathered} $$ Since $2 \\varphi-\\beta-\\alpha=180^{\\circ}-\\angle A E D<180^{\\circ}$ and $\\varphi=\\frac{1}{2} \\angle A B C<90^{\\circ}$, it follows that $\\varphi=\\beta+\\gamma$, hence $\\angle B D A=2 \\varphi-\\beta-\\gamma=\\varphi=\\frac{1}{2} \\angle C D E$, as desired.","tier":0} +{"problem_type":"Geometry","problem_label":"G6","problem":"The vertices $X, Y, Z$ of an equilateral triangle $X Y Z$ lie respectively on the sides $B C$, $C A, A B$ of an acute-angled triangle $A B C$. Prove that the incenter of triangle $A B C$ lies inside triangle $X Y Z$.","solution":null,"tier":0} +{"problem_type":"Geometry","problem_label":"G6","problem":"${ }^{\\prime}$. The vertices $X, Y, Z$ of an equilateral triangle $X Y Z$ lie respectively on the sides $B C, C A, A B$ of a triangle $A B C$. Prove that if the incenter of triangle $A B C$ lies outside triangle $X Y Z$, then one of the angles of triangle $A B C$ is greater than $120^{\\circ}$. (Bulgaria) Solution 1 for G6. We will prove a stronger fact; namely, we will show that the incenter $I$ of triangle $A B C$ lies inside the incircle of triangle $X Y Z$ (and hence surely inside triangle $X Y Z$ itself). We denote by $d(U, V W)$ the distance between point $U$ and line $V W$. Denote by $O$ the incenter of $\\triangle X Y Z$ and by $r, r^{\\prime}$ and $R^{\\prime}$ the inradii of triangles $A B C, X Y Z$ and the circumradius of $X Y Z$, respectively. Then we have $R^{\\prime}=2 r^{\\prime}$, and the desired inequality is $O I \\leq r^{\\prime}$. We assume that $O \\neq I$; otherwise the claim is trivial. Let the incircle of $\\triangle A B C$ touch its sides $B C, A C, A B$ at points $A_{1}, B_{1}, C_{1}$ respectively. The lines $I A_{1}, I B_{1}, I C_{1}$ cut the plane into 6 acute angles, each one containing one of the points $A_{1}, B_{1}, C_{1}$ on its border. We may assume that $O$ lies in an angle defined by lines $I A_{1}$, $I C_{1}$ and containing point $C_{1}$ (see Fig. 1). Let $A^{\\prime}$ and $C^{\\prime}$ be the projections of $O$ onto lines $I A_{1}$ and $I C_{1}$, respectively. Since $O X=R^{\\prime}$, we have $d(O, B C) \\leq R^{\\prime}$. Since $O A^{\\prime} \\| B C$, it follows that $d\\left(A^{\\prime}, B C\\right)=$ $A^{\\prime} I+r \\leq R^{\\prime}$, or $A^{\\prime} I \\leq R^{\\prime}-r$. On the other hand, the incircle of $\\triangle X Y Z$ lies inside $\\triangle A B C$, hence $d(O, A B) \\geq r^{\\prime}$, and analogously we get $d(O, A B)=C^{\\prime} C_{1}=r-I C^{\\prime} \\geq r^{\\prime}$, or $I C^{\\prime} \\leq r-r^{\\prime}$. ![](https:\/\/cdn.mathpix.com\/cropped\/2024_11_18_ba86565be3364967ae6fg-57.jpg?height=475&width=815&top_left_y=1536&top_left_x=455) Fig. 1 ![](https:\/\/cdn.mathpix.com\/cropped\/2024_11_18_ba86565be3364967ae6fg-57.jpg?height=346&width=389&top_left_y=1666&top_left_x=1282) Fig. 2 Finally, the quadrilateral $I A^{\\prime} O C^{\\prime}$ is circumscribed due to the right angles at $A^{\\prime}$ and $C^{\\prime}$ (see Fig. 2). On its circumcircle, we have $\\widehat{A^{\\prime} O C^{\\prime}}=2 \\angle A^{\\prime} I C^{\\prime}<180^{\\circ}=\\widetilde{O C^{\\prime} I}$, hence $180^{\\circ} \\geq$ $\\widetilde{I C^{\\prime}}>\\widetilde{A^{\\prime} O}$. This means that $I C^{\\prime}>A^{\\prime} O$. Finally, we have $O I \\leq I A^{\\prime}+A^{\\prime} O90^{\\circ}$ thus leading to a contradiction. Note that $\\omega$ intersects each of the segments $X Y$ and $Y Z$ at two points; let $U, U^{\\prime}$ and $V$, $V^{\\prime}$ be the points of intersection of $\\omega$ with $X Y$ and $Y Z$, respectively $\\left(U Y>U^{\\prime} Y, V Y>V^{\\prime} Y\\right.$; see Figs. 3 and 4). Note that $60^{\\circ}=\\angle X Y Z=\\frac{1}{2}\\left(\\overparen{U V}-\\widetilde{U^{\\prime} V^{\\prime}}\\right) \\leq \\frac{1}{2} \\overparen{U V}$, hence $\\overparen{U V} \\geq 120^{\\circ}$. On the other hand, since $I$ lies in $\\triangle A Y Z$, we get $\\sqrt{U V^{\\prime}}<180^{\\circ}$, hence $\\sqrt{U A_{1} U^{\\prime}} \\leq \\sqrt{U A_{1} V^{\\prime}}<$ $180^{\\circ}-\\widehat{U V} \\leq 60^{\\circ}$. Now, two cases are possible due to the order of points $Y, B_{1}$ on segment $A C$. ![](https:\/\/cdn.mathpix.com\/cropped\/2024_11_18_ba86565be3364967ae6fg-58.jpg?height=495&width=690&top_left_y=529&top_left_x=249) Fig. 3 ![](https:\/\/cdn.mathpix.com\/cropped\/2024_11_18_ba86565be3364967ae6fg-58.jpg?height=415&width=801&top_left_y=609&top_left_x=953) Fig. 4 Case 1. Let point $Y$ lie on the segment $A B_{1}$ (see Fig. 3). Then we have $\\angle Y X C=$ $\\frac{1}{2}\\left(\\widehat{A_{1} U^{\\prime}}-\\widehat{A_{1} U}\\right) \\leq \\frac{1}{2} \\widehat{U A_{1} U^{\\prime}}<30^{\\circ}$; analogously, we get $\\angle X Y C \\leq \\frac{1}{2} \\widehat{U A_{1} U^{\\prime}}<30^{\\circ}$. Therefore, $\\angle Y C X=180^{\\circ}-\\angle Y X C-\\angle X Y C>120^{\\circ}$, as desired. Case 2. Now let point $Y$ lie on the segment $C B_{1}$ (see Fig. 4). Analogously, we obtain $\\angle Y X C<30^{\\circ}$. Next, $\\angle I Y X>\\angle Z Y X=60^{\\circ}$, but $\\angle I Y X<\\angle I Y B_{1}$, since $Y B_{1}$ is a tangent and $Y X$ is a secant line to circle $\\omega$ from point $Y$. Hence, we get $120^{\\circ}<\\angle I Y B_{1}+\\angle I Y X=$ $\\angle B_{1} Y X=\\angle Y X C+\\angle Y C X<30^{\\circ}+\\angle Y C X$, hence $\\angle Y C X>120^{\\circ}-30^{\\circ}=90^{\\circ}$, as desired. Comment. In the same way, one can prove a more general Claim. Let the vertices $X, Y, Z$ of a triangle $X Y Z$ lie respectively on the sides $B C, C A, A B$ of a triangle $A B C$. Suppose that the incenter of triangle $A B C$ lies outside triangle $X Y Z$, and $\\alpha$ is the least angle of $\\triangle X Y Z$. Then one of the angles of triangle $A B C$ is greater than $3 \\alpha-90^{\\circ}$. Solution for G6'. Assume the contrary. As in Solution 2, we assume that the incenter $I$ of $\\triangle A B C$ lies in $\\triangle A Y Z$, and the tangency point $A_{1}$ of $\\omega$ and $B C$ lies on segment $C X$. Surely, $\\angle Y Z A \\leq 180^{\\circ}-\\angle Y Z X=120^{\\circ}$, hence points $I$ and $Y$ lie on one side of the perpendicular bisector to $X Y$; therefore $I X>I Y$. Moreover, $\\omega$ intersects segment $X Y$ at two points, and therefore the projection $M$ of $I$ onto $X Y$ lies on the segment $X Y$. In this case, we will prove that $\\angle C>120^{\\circ}$. Let $Y K, Y L$ be two tangents from point $Y$ to $\\omega$ (points $K$ and $A_{1}$ lie on one side of $X Y$; if $Y$ lies on $\\omega$, we say $K=L=Y$ ); one of the points $K$ and $L$ is in fact a tangency point $B_{1}$ of $\\omega$ and $A C$. From symmetry, we have $\\angle Y I K=\\angle Y I L$. On the other hand, since $I X>I Y$, we get $X M120^{\\circ}$, as desired. ![](https:\/\/cdn.mathpix.com\/cropped\/2024_11_18_ba86565be3364967ae6fg-59.jpg?height=366&width=715&top_left_y=408&top_left_x=271) Fig. 5 ![](https:\/\/cdn.mathpix.com\/cropped\/2024_11_18_ba86565be3364967ae6fg-59.jpg?height=515&width=869&top_left_y=276&top_left_x=982) Fig. 6 Comment 1. The estimate claimed in $\\mathrm{G}^{\\prime}$ is sharp. Actually, if $\\angle B A C>120^{\\circ}$, one can consider an equilateral triangle $X Y Z$ with $Z=A, Y \\in A C, X \\in B C$ (such triangle exists since $\\angle A C B<60^{\\circ}$ ). It intersects with the angle bisector of $\\angle B A C$ only at point $A$, hence it does not contain $I$. Comment 2. As in the previous solution, there is a generalization for an arbitrary triangle $X Y Z$, but here we need some additional condition. The statement reads as follows. Claim. Let the vertices $X, Y, Z$ of a triangle $X Y Z$ lie respectively on the sides $B C, C A, A B$ of a triangle $A B C$. Suppose that the incenter of triangle $A B C$ lies outside triangle $X Y Z, \\alpha$ is the least angle of $\\triangle X Y Z$, and all sides of triangle $X Y Z$ are greater than $2 r \\cot \\alpha$, where $r$ is the inradius of $\\triangle A B C$. Then one of the angles of triangle $A B C$ is greater than $2 \\alpha$. The additional condition is needed to verify that $X M>Y M$ since it cannot be shown in the original way. Actually, we have $\\angle M Y I>\\alpha, I M2 r \\cot \\alpha$, then surely $X M>Y M$. On the other hand, this additional condition follows easily from the conditions of the original problem. Actually, if $I \\in \\triangle A Y Z$, then the diameter of $\\omega$ parallel to $Y Z$ is contained in $\\triangle A Y Z$ and is thus shorter than $Y Z$. Hence $Y Z>2 r>2 r \\cot 60^{\\circ}$.","solution":null,"tier":0} +{"problem_type":"Geometry","problem_label":"G7","problem":"Three circular arcs $\\gamma_{1}, \\gamma_{2}$, and $\\gamma_{3}$ connect the points $A$ and $C$. These arcs lie in the same half-plane defined by line $A C$ in such a way that arc $\\gamma_{2}$ lies between the arcs $\\gamma_{1}$ and $\\gamma_{3}$. Point $B$ lies on the segment $A C$. Let $h_{1}, h_{2}$, and $h_{3}$ be three rays starting at $B$, lying in the same half-plane, $h_{2}$ being between $h_{1}$ and $h_{3}$. For $i, j=1,2,3$, denote by $V_{i j}$ the point of intersection of $h_{i}$ and $\\gamma_{j}$ (see the Figure below). Denote by $\\widehat{V_{i j} V_{k j}} \\widehat{V_{k \\ell} V_{i \\ell}}$ the curved quadrilateral, whose sides are the segments $V_{i j} V_{i \\ell}, V_{k j} V_{k \\ell}$ and $\\operatorname{arcs} V_{i j} V_{k j}$ and $V_{i \\ell} V_{k \\ell}$. We say that this quadrilateral is circumscribed if there exists a circle touching these two segments and two arcs. Prove that if the curved quadrilaterals $\\sqrt{V_{11} V_{21}} \\sqrt{22} V_{12}, \\sqrt{V_{12} V_{22}} \\sqrt{23} V_{13}, \\sqrt{V_{21} V_{31}} \\sqrt{V_{32} V_{22}}$ are circumscribed, then the curved quadrilateral $\\sqrt{V_{22} V_{32}} \\sqrt{33} V_{23}$ is circumscribed, too. ![](https:\/\/cdn.mathpix.com\/cropped\/2024_11_18_ba86565be3364967ae6fg-61.jpg?height=495&width=815&top_left_y=798&top_left_x=655) Fig. 1 (Hungary)","solution":"Denote by $O_{i}$ and $R_{i}$ the center and the radius of $\\gamma_{i}$, respectively. Denote also by $H$ the half-plane defined by $A C$ which contains the whole configuration. For every point $P$ in the half-plane $H$, denote by $d(P)$ the distance between $P$ and line $A C$. Furthermore, for any $r>0$, denote by $\\Omega(P, r)$ the circle with center $P$ and radius $r$. Lemma 1. For every $1 \\leq iR_{i}$ and $O_{j} P0$; then the ![](https:\/\/cdn.mathpix.com\/cropped\/2024_11_18_ba86565be3364967ae6fg-62.jpg?height=787&width=1338&top_left_y=269&top_left_x=336) circle $\\Omega(P, r)$ touches $\\gamma_{i}$ externally and touches $\\gamma_{j}$ internally, so $P$ belongs to the locus under investigation. (b) Let $\\vec{\\rho}=\\overrightarrow{A P}, \\vec{\\rho}_{i}=\\overrightarrow{A O_{i}}$, and $\\vec{\\rho}_{j}=\\overrightarrow{A O_{j}}$; let $d_{i j}=O_{i} O_{j}$, and let $\\vec{v}$ be a unit vector orthogonal to $A C$ and directed toward $H$. Then we have $\\left|\\vec{\\rho}_{i}\\right|=R_{i},\\left|\\vec{\\rho}_{j}\\right|=R_{j},\\left|\\overrightarrow{O_{i} P}\\right|=$ $\\left|\\vec{\\rho}-\\vec{\\rho}_{i}\\right|=R_{i}+r,\\left|\\overrightarrow{O_{j} P}\\right|=\\left|\\vec{\\rho}-\\vec{\\rho}_{j}\\right|=R_{j}-r$, hence $$ \\begin{gathered} \\left(\\vec{\\rho}-\\vec{\\rho}_{i}\\right)^{2}-\\left(\\vec{\\rho}-\\vec{\\rho}_{j}\\right)^{2}=\\left(R_{i}+r\\right)^{2}-\\left(R_{j}-r\\right)^{2}, \\\\ \\left(\\vec{\\rho}_{i}^{2}-\\vec{\\rho}_{j}^{2}\\right)+2 \\vec{\\rho} \\cdot\\left(\\vec{\\rho}_{j}-\\vec{\\rho}_{i}\\right)=\\left(R_{i}^{2}-R_{j}^{2}\\right)+2 r\\left(R_{i}+R_{j}\\right), \\\\ d_{i j} \\cdot d(P)=d_{i j} \\vec{v} \\cdot \\vec{\\rho}=\\left(\\vec{\\rho}_{j}-\\vec{\\rho}_{i}\\right) \\cdot \\vec{\\rho}=r\\left(R_{i}+R_{j}\\right) . \\end{gathered} $$ Therefore, $$ r=\\frac{d_{i j}}{R_{i}+R_{j}} \\cdot d(P) $$ and the value $v_{i j}=\\frac{d_{i j}}{R_{i}+R_{j}}$ does not depend on $P$. Lemma 3. The curved quadrilateral $\\mathcal{Q}_{i j}=\\sqrt{i, j V_{i+1, j}} V_{i+1, j+1} V_{i, j+1}$ is circumscribed if and only if $u_{i, i+1}=v_{j, j+1}$. Proof. First suppose that the curved quadrilateral $\\mathcal{Q}_{i j}$ is circumscribed and $\\Omega(P, r)$ is its inscribed circle. By Lemma 1 and Lemma 2 we have $r=u_{i, i+1} \\cdot d(P)$ and $r=v_{j, j+1} \\cdot d(P)$ as well. Hence, $u_{i, i+1}=v_{j, j+1}$. To prove the opposite direction, suppose $u_{i, i+1}=v_{j, j+1}$. Let $P$ be the intersection of the angle bisector $\\beta_{i, i+1}$ and the ellipse arc $\\varepsilon_{j, j+1}$. Choose $r=u_{i, i+1} \\cdot d(P)=v_{j, j+1} \\cdot d(P)$. Then the circle $\\Omega(P, r)$ is tangent to the half lines $h_{i}$ and $h_{i+1}$ by Lemma 1 , and it is tangent to the $\\operatorname{arcs} \\gamma_{j}$ and $\\gamma_{j+1}$ by Lemma 2. Hence, the curved quadrilateral $\\mathcal{Q}_{i j}$ is circumscribed. By Lemma 3, the statement of the problem can be reformulated to an obvious fact: If the equalities $u_{12}=v_{12}, u_{12}=v_{23}$, and $u_{23}=v_{12}$ hold, then $u_{23}=v_{23}$ holds as well. Comment 1. Lemma 2(b) (together with the easy Lemma 1(b)) is the key tool in this solution. If one finds this fact, then the solution can be finished in many ways. That is, one can find a circle touching three of $h_{2}, h_{3}, \\gamma_{2}$, and $\\gamma_{3}$, and then prove that it is tangent to the fourth one in either synthetic or analytical way. Both approaches can be successful. Here we present some discussion about this key Lemma. 1. In the solution above we chose an analytic proof for Lemma 2(b) because we expect that most students will use coordinates or vectors to examine the locus of the centers, and these approaches are less case-sensitive. Here we outline a synthetic proof. We consider only the case when $P$ does not lie in the line $O_{i} O_{j}$. The other case can be obtained as a limit case, or computed in a direct way. Let $S$ be the internal homothety center between the circles of $\\gamma_{i}$ and $\\gamma_{j}$, lying on $O_{i} O_{j}$; this point does not depend on $P$. Let $U$ and $V$ be the points of tangency of circle $\\sigma=\\Omega(P, r)$ with $\\gamma_{i}$ and $\\gamma_{j}$, respectively (then $r=P U=P V$ ); in other words, points $U$ and $V$ are the intersection points of rays $O_{i} P, O_{j} P$ with arcs $\\gamma_{i}, \\gamma_{j}$ respectively (see Fig. 4). Due to the theorem on three homothety centers (or just to the Menelaus theorem applied to triangle $O_{i} O_{j} P$ ), the points $U, V$ and $S$ are collinear. Let $T$ be the intersection point of line $A C$ and the common tangent to $\\sigma$ and $\\gamma_{i}$ at $U$; then $T$ is the radical center of $\\sigma, \\gamma_{i}$ and $\\gamma_{j}$, hence $T V$ is the common tangent to $\\sigma$ and $\\gamma_{j}$. Let $Q$ be the projection of $P$ onto the line $A C$. By the right angles, the points $U, V$ and $Q$ lie on the circle with diameter $P T$. From this fact and the equality $P U=P V$ we get $\\angle U Q P=\\angle U V P=$ $\\angle V U P=\\angle S U O_{i}$. Since $O_{i} S \\| P Q$, we have $\\angle S O_{i} U=\\angle Q P U$. Hence, the triangles $S O_{i} U$ and $U P Q$ are similar and thus $\\frac{r}{d(P)}=\\frac{P U}{P Q}=\\frac{O_{i} S}{O_{i} U}=\\frac{O_{i} S}{R_{i}}$; the last expression is constant since $S$ is a constant point. ![](https:\/\/cdn.mathpix.com\/cropped\/2024_11_18_ba86565be3364967ae6fg-63.jpg?height=595&width=797&top_left_y=1570&top_left_x=361) Fig. 4 ![](https:\/\/cdn.mathpix.com\/cropped\/2024_11_18_ba86565be3364967ae6fg-63.jpg?height=647&width=546&top_left_y=1524&top_left_x=1212) Fig. 5 2. Using some known facts about conics, the same statement can be proved in a very short way. Denote by $\\ell$ the directrix of ellipse of $\\varepsilon_{i j}$ related to the focus $O_{j}$; since $\\varepsilon_{i j}$ is symmetrical about $O_{i} O_{j}$, we have $\\ell \\| A C$. Recall that for each point $P \\in \\varepsilon_{i j}$, we have $P O_{j}=\\epsilon \\cdot d_{\\ell}(P)$, where $d_{\\ell}(P)$ is the distance from $P$ to $\\ell$, and $\\epsilon$ is the eccentricity of $\\varepsilon_{i j}$ (see Fig. 5). Now we have $$ r=R_{j}-\\left(R_{j}-r\\right)=A O_{j}-P O_{j}=\\epsilon\\left(d_{\\ell}(A)-d_{\\ell}(P)\\right)=\\epsilon(d(P)-d(A))=\\epsilon \\cdot d(P), $$ and $\\epsilon$ does not depend on $P$. Comment 2. One can find a spatial interpretations of the problem and the solution. For every point $(x, y)$ and radius $r>0$, represent the circle $\\Omega((x, y), r)$ by the point $(x, y, r)$ in space. This point is the apex of the cone with base circle $\\Omega((x, y), r)$ and height $r$. According to Lemma 1 , the circles which are tangent to $h_{i}$ and $h_{j}$ correspond to the points of a half line $\\beta_{i j}^{\\prime}$, starting at $B$. Now we translate Lemma 2. Take some $1 \\leq i1$ since otherwise $1-\\frac{1}{s_{1}}=0$. So we have $2 \\leq s_{1} \\leq s_{2}-1 \\leq \\cdots \\leq s_{n}-(n-1)$, hence $s_{i} \\geq i+1$ for each $i=1, \\ldots, n$. Therefore $$ \\begin{aligned} \\frac{51}{2010} & =\\left(1-\\frac{1}{s_{1}}\\right)\\left(1-\\frac{1}{s_{2}}\\right) \\ldots\\left(1-\\frac{1}{s_{n}}\\right) \\\\ & \\geq\\left(1-\\frac{1}{2}\\right)\\left(1-\\frac{1}{3}\\right) \\ldots\\left(1-\\frac{1}{n+1}\\right)=\\frac{1}{2} \\cdot \\frac{2}{3} \\cdots \\frac{n}{n+1}=\\frac{1}{n+1} \\end{aligned} $$ which implies $$ n+1 \\geq \\frac{2010}{51}=\\frac{670}{17}>39 $$ so $n \\geq 39$. Now we are left to show that $n=39$ fits. Consider the set $\\{2,3, \\ldots, 33,35,36, \\ldots, 40,67\\}$ which contains exactly 39 numbers. We have $$ \\frac{1}{2} \\cdot \\frac{2}{3} \\cdots \\frac{32}{33} \\cdot \\frac{34}{35} \\cdots \\frac{39}{40} \\cdot \\frac{66}{67}=\\frac{1}{33} \\cdot \\frac{34}{40} \\cdot \\frac{66}{67}=\\frac{17}{670}=\\frac{51}{2010} $$ hence for $n=39$ there exists a desired example. Comment. One can show that the example (1) is unique. Answer for Problem N1' ${ }^{\\prime} n=48$. Solution for Problem N1'. Suppose that for some $n$ there exist the desired numbers. In the same way we obtain that $s_{i} \\geq i+1$. Moreover, since the denominator of the fraction $\\frac{42}{2010}=\\frac{7}{335}$ is divisible by 67 , some of $s_{i}$ 's should be divisible by 67 , so $s_{n} \\geq s_{i} \\geq 67$. This means that $$ \\frac{42}{2010} \\geq \\frac{1}{2} \\cdot \\frac{2}{3} \\cdots \\frac{n-1}{n} \\cdot\\left(1-\\frac{1}{67}\\right)=\\frac{66}{67 n} $$ which implies $$ n \\geq \\frac{2010 \\cdot 66}{42 \\cdot 67}=\\frac{330}{7}>47 $$ so $n \\geq 48$. Now we are left to show that $n=48$ fits. Consider the set $\\{2,3, \\ldots, 33,36,37, \\ldots, 50,67\\}$ which contains exactly 48 numbers. We have $$ \\frac{1}{2} \\cdot \\frac{2}{3} \\cdots \\frac{32}{33} \\cdot \\frac{35}{36} \\cdots \\frac{49}{50} \\cdot \\frac{66}{67}=\\frac{1}{33} \\cdot \\frac{35}{50} \\cdot \\frac{66}{67}=\\frac{7}{335}=\\frac{42}{2010} $$ hence for $n=48$ there exists a desired example. Comment 1. In this version of the problem, the estimate needs one more step, hence it is a bit harder. On the other hand, the example in this version is not unique. Another example is $$ \\frac{1}{2} \\cdot \\frac{2}{3} \\cdots \\frac{46}{47} \\cdot \\frac{66}{67} \\cdot \\frac{329}{330}=\\frac{1}{67} \\cdot \\frac{66}{330} \\cdot \\frac{329}{47}=\\frac{7}{67 \\cdot 5}=\\frac{42}{2010} . $$ Comment 2. N1' was the Proposer's formulation of the problem. We propose N1 according to the number of current IMO.","solution":null,"tier":0} +{"problem_type":"Number Theory","problem_label":"N2","problem":"Find all pairs $(m, n)$ of nonnegative integers for which $$ m^{2}+2 \\cdot 3^{n}=m\\left(2^{n+1}-1\\right) $$ (Australia) Answer. $(6,3),(9,3),(9,5),(54,5)$.","solution":"For fixed values of $n$, the equation (1) is a simple quadratic equation in $m$. For $n \\leq 5$ the solutions are listed in the following table. | case | equation | discriminant | integer roots | | :--- | :--- | :--- | :--- | | $n=0$ | $m^{2}-m+2=0$ | -7 | none | | $n=1$ | $m^{2}-3 m+6=0$ | -15 | none | | $n=2$ | $m^{2}-7 m+18=0$ | -23 | none | | $n=3$ | $m^{2}-15 m+54=0$ | 9 | $m=6$ and $m=9$ | | $n=4$ | $m^{2}-31 m+162=0$ | 313 | none | | $n=5$ | $m^{2}-63 m+486=0$ | $2025=45^{2}$ | $m=9$ and $m=54$ | We prove that there is no solution for $n \\geq 6$. Suppose that $(m, n)$ satisfies (1) and $n \\geq 6$. Since $m \\mid 2 \\cdot 3^{n}=m\\left(2^{n+1}-1\\right)-m^{2}$, we have $m=3^{p}$ with some $0 \\leq p \\leq n$ or $m=2 \\cdot 3^{q}$ with some $0 \\leq q \\leq n$. In the first case, let $q=n-p$; then $$ 2^{n+1}-1=m+\\frac{2 \\cdot 3^{n}}{m}=3^{p}+2 \\cdot 3^{q} $$ In the second case let $p=n-q$. Then $$ 2^{n+1}-1=m+\\frac{2 \\cdot 3^{n}}{m}=2 \\cdot 3^{q}+3^{p} $$ Hence, in both cases we need to find the nonnegative integer solutions of $$ 3^{p}+2 \\cdot 3^{q}=2^{n+1}-1, \\quad p+q=n $$ Next, we prove bounds for $p, q$. From (2) we get $$ 3^{p}<2^{n+1}=8^{\\frac{n+1}{3}}<9^{\\frac{n+1}{3}}=3^{\\frac{2(n+1)}{3}} $$ and $$ 2 \\cdot 3^{q}<2^{n+1}=2 \\cdot 8^{\\frac{n}{3}}<2 \\cdot 9^{\\frac{n}{3}}=2 \\cdot 3^{\\frac{2 n}{3}}<2 \\cdot 3^{\\frac{2(n+1)}{3}} $$ so $p, q<\\frac{2(n+1)}{3}$. Combining these inequalities with $p+q=n$, we obtain $$ \\frac{n-2}{3}\\frac{n-2}{3}$; in particular, we have $h>1$. On the left-hand side of (2), both terms are divisible by $3^{h}$, therefore $9\\left|3^{h}\\right| 2^{n+1}-1$. It is easy check that $\\operatorname{ord}_{9}(2)=6$, so $9 \\mid 2^{n+1}-1$ if and only if $6 \\mid n+1$. Therefore, $n+1=6 r$ for some positive integer $r$, and we can write $$ 2^{n+1}-1=4^{3 r}-1=\\left(4^{2 r}+4^{r}+1\\right)\\left(2^{r}-1\\right)\\left(2^{r}+1\\right) $$ Notice that the factor $4^{2 r}+4^{r}+1=\\left(4^{r}-1\\right)^{2}+3 \\cdot 4^{r}$ is divisible by 3 , but it is never divisible by 9 . The other two factors in (4), $2^{r}-1$ and $2^{r}+1$ are coprime: both are odd and their difference is 2 . Since the whole product is divisible by $3^{h}$, we have either $3^{h-1} \\mid 2^{r}-1$ or $3^{h-1} \\mid 2^{r}+1$. In any case, we have $3^{h-1} \\leq 2^{r}+1$. Then $$ \\begin{gathered} 3^{h-1} \\leq 2^{r}+1 \\leq 3^{r}=3^{\\frac{n+1}{6}} \\\\ \\frac{n-2}{3}-10$ of the system of equations (i) $\\sum_{i=1}^{4} x_{i}^{2}=8 m^{2}$, (ii) $\\sum_{i=1}^{4} y_{i}^{2}=8 m^{2}$, (iii) $\\sum_{i=1}^{4} x_{i} y_{i}=-6 m^{2}$. We will show that such a solution does not exist. Assume the contrary and consider a solution with minimal $m$. Note that if an integer $x$ is odd then $x^{2} \\equiv 1(\\bmod 8)$. Otherwise (i.e., if $x$ is even) we have $x^{2} \\equiv 0(\\bmod 8)$ or $x^{2} \\equiv 4$ $(\\bmod 8)$. Hence, by (i), we get that $x_{1}, x_{2}, x_{3}$ and $x_{4}$ are even. Similarly, by (ii), we get that $y_{1}, y_{2}, y_{3}$ and $y_{4}$ are even. Thus the LHS of (iii) is divisible by 4 and $m$ is also even. It follows that $\\left(\\frac{x_{1}}{2}, \\frac{y_{1}}{2}, \\frac{x_{2}}{2}, \\frac{y_{2}}{2}, \\frac{x_{3}}{2}, \\frac{y_{3}}{2}, \\frac{x_{4}}{2}, \\frac{y_{4}}{2}, \\frac{m}{2}\\right)$ is a solution of the system of equations (i), (ii) and (iii), which contradicts the minimality of $m$.","tier":0} +{"problem_type":"Number Theory","problem_label":"N3","problem":"Find the smallest number $n$ such that there exist polynomials $f_{1}, f_{2}, \\ldots, f_{n}$ with rational coefficients satisfying $$ x^{2}+7=f_{1}(x)^{2}+f_{2}(x)^{2}+\\cdots+f_{n}(x)^{2} . $$ (Poland) Answer. The smallest $n$ is 5 .","solution":"We prove that $n \\leq 4$ is impossible. Define the numbers $a_{i}, b_{i}$ for $i=1,2,3,4$ as in the previous solution. By Euler's identity we have $$ \\begin{aligned} \\left(a_{1}^{2}+a_{2}^{2}+a_{3}^{2}+a_{4}^{2}\\right)\\left(b_{1}^{2}+b_{2}^{2}+b_{3}^{2}+b_{4}^{2}\\right) & =\\left(a_{1} b_{1}+a_{2} b_{2}+a_{3} b_{3}+a_{4} b_{4}\\right)^{2}+\\left(a_{1} b_{2}-a_{2} b_{1}+a_{3} b_{4}-a_{4} b_{3}\\right)^{2} \\\\ & +\\left(a_{1} b_{3}-a_{3} b_{1}+a_{4} b_{2}-a_{2} b_{4}\\right)^{2}+\\left(a_{1} b_{4}-a_{4} b_{1}+a_{2} b_{3}-a_{3} b_{2}\\right)^{2} . \\end{aligned} $$ So, using the relations (1) from the Solution 1 we get that $$ 7=\\left(\\frac{m_{1}}{m}\\right)^{2}+\\left(\\frac{m_{2}}{m}\\right)^{2}+\\left(\\frac{m_{3}}{m}\\right)^{2} $$ where $$ \\begin{aligned} & \\frac{m_{1}}{m}=a_{1} b_{2}-a_{2} b_{1}+a_{3} b_{4}-a_{4} b_{3} \\\\ & \\frac{m_{2}}{m}=a_{1} b_{3}-a_{3} b_{1}+a_{4} b_{2}-a_{2} b_{4} \\\\ & \\frac{m_{3}}{m}=a_{1} b_{4}-a_{4} b_{1}+a_{2} b_{3}-a_{3} b_{2} \\end{aligned} $$ and $m_{1}, m_{2}, m_{3} \\in \\mathbb{Z}, m \\in \\mathbb{N}$. Let $m$ be a minimum positive integer number for which (2) holds. Then $$ 8 m^{2}=m_{1}^{2}+m_{2}^{2}+m_{3}^{2}+m^{2} $$ As in the previous solution, we get that $m_{1}, m_{2}, m_{3}, m$ are all even numbers. Then $\\left(\\frac{m_{1}}{2}, \\frac{m_{2}}{2}, \\frac{m_{3}}{2}, \\frac{m}{2}\\right)$ is also a solution of ( 2 ) which contradicts the minimality of $m$. So, we have $n \\geq 5$. The example with $n=5$ is already shown in Solution 1 .","tier":0} +{"problem_type":"Number Theory","problem_label":"N4","problem":"Let $a, b$ be integers, and let $P(x)=a x^{3}+b x$. For any positive integer $n$ we say that the pair $(a, b)$ is $n$-good if $n \\mid P(m)-P(k)$ implies $n \\mid m-k$ for all integers $m, k$. We say that $(a, b)$ is very good if $(a, b)$ is $n$-good for infinitely many positive integers $n$. (a) Find a pair $(a, b)$ which is 51 -good, but not very good. (b) Show that all 2010-good pairs are very good. (Turkey)","solution":"(a) We show that the pair $\\left(1,-51^{2}\\right)$ is good but not very good. Let $P(x)=x^{3}-51^{2} x$. Since $P(51)=P(0)$, the pair $\\left(1,-51^{2}\\right)$ is not $n$-good for any positive integer that does not divide 51 . Therefore, $\\left(1,-51^{2}\\right)$ is not very good. On the other hand, if $P(m) \\equiv P(k)(\\bmod 51)$, then $m^{3} \\equiv k^{3}(\\bmod 51)$. By Fermat's theorem, from this we obtain $$ m \\equiv m^{3} \\equiv k^{3} \\equiv k \\quad(\\bmod 3) \\quad \\text { and } \\quad m \\equiv m^{33} \\equiv k^{33} \\equiv k \\quad(\\bmod 17) . $$ Hence we have $m \\equiv k(\\bmod 51)$. Therefore $\\left(1,-51^{2}\\right)$ is 51 -good. (b) We will show that if a pair $(a, b)$ is 2010-good then $(a, b)$ is $67^{i}$-good for all positive integer $i$. Claim 1. If $(a, b)$ is 2010 -good then $(a, b)$ is 67 -good. Proof. Assume that $P(m)=P(k)(\\bmod 67)$. Since 67 and 30 are coprime, there exist integers $m^{\\prime}$ and $k^{\\prime}$ such that $k^{\\prime} \\equiv k(\\bmod 67), k^{\\prime} \\equiv 0(\\bmod 30)$, and $m^{\\prime} \\equiv m(\\bmod 67), m^{\\prime} \\equiv 0$ $(\\bmod 30)$. Then we have $P\\left(m^{\\prime}\\right) \\equiv P(0) \\equiv P\\left(k^{\\prime}\\right)(\\bmod 30)$ and $P\\left(m^{\\prime}\\right) \\equiv P(m) \\equiv P(k) \\equiv P\\left(k^{\\prime}\\right)$ $(\\bmod 67)$, hence $P\\left(m^{\\prime}\\right) \\equiv P\\left(k^{\\prime}\\right)(\\bmod 2010)$. This implies $m^{\\prime} \\equiv k^{\\prime}(\\bmod 2010)$ as $(a, b)$ is 2010 -good. It follows that $m \\equiv m^{\\prime} \\equiv k^{\\prime} \\equiv k(\\bmod 67)$. Therefore, $(a, b)$ is 67 -good. Claim 2. If $(a, b)$ is 67 -good then $67 \\mid a$. Proof. Suppose that $67 \\nmid a$. Consider the sets $\\left\\{a t^{2}(\\bmod 67): 0 \\leq t \\leq 33\\right\\}$ and $\\left\\{-3 a s^{2}-b\\right.$ $\\bmod 67: 0 \\leq s \\leq 33\\}$. Since $a \\not \\equiv 0(\\bmod 67)$, each of these sets has 34 elements. Hence they have at least one element in common. If $a t^{2} \\equiv-3 a s^{2}-b(\\bmod 67)$ then for $m=t \\pm s, k=\\mp 2 s$ we have $$ \\begin{aligned} P(m)-P(k)=a\\left(m^{3}-k^{3}\\right)+b(m-k) & =(m-k)\\left(a\\left(m^{2}+m k+k^{2}\\right)+b\\right) \\\\ & =(t \\pm 3 s)\\left(a t^{2}+3 a s^{2}+b\\right) \\equiv 0 \\quad(\\bmod 67) \\end{aligned} $$ Since $(a, b)$ is 67 -good, we must have $m \\equiv k(\\bmod 67)$ in both cases, that is, $t \\equiv 3 s(\\bmod 67)$ and $t \\equiv-3 s(\\bmod 67)$. This means $t \\equiv s \\equiv 0(\\bmod 67)$ and $b \\equiv-3 a s^{2}-a t^{2} \\equiv 0(\\bmod 67)$. But then $67 \\mid P(7)-P(2)=67 \\cdot 5 a+5 b$ and $67 \\not 7-2$, contradicting that $(a, b)$ is 67 -good. Claim 3. If $(a, b)$ is 2010 -good then $(a, b)$ is $67^{i}$-good all $i \\geq 1$. Proof. By Claim 2, we have $67 \\mid a$. If $67 \\mid b$, then $P(x) \\equiv P(0)(\\bmod 67)$ for all $x$, contradicting that $(a, b)$ is 67 -good. Hence, $67 \\nmid b$. Suppose that $67^{i} \\mid P(m)-P(k)=(m-k)\\left(a\\left(m^{2}+m k+k^{2}\\right)+b\\right)$. Since $67 \\mid a$ and $67 \\nmid b$, the second factor $a\\left(m^{2}+m k+k^{2}\\right)+b$ is coprime to 67 and hence $67^{i} \\mid m-k$. Therefore, $(a, b)$ is $67^{i}$-good. Comment 1. In the proof of Claim 2, the following reasoning can also be used. Since 3 is not a quadratic residue modulo 67 , either $a u^{2} \\equiv-b(\\bmod 67)$ or $3 a v^{2} \\equiv-b(\\bmod 67)$ has a solution. The settings $(m, k)=(u, 0)$ in the first case and $(m, k)=(v,-2 v)$ in the second case lead to $b \\equiv 0$ $(\\bmod 67)$. Comment 2. The pair $(67,30)$ is $n$-good if and only if $n=d \\cdot 67^{i}$, where $d \\mid 30$ and $i \\geq 0$. It shows that in part (b), one should deal with the large powers of 67 to reach the solution. The key property of number 67 is that it has the form $3 k+1$, so there exists a nontrivial cubic root of unity modulo 67 .","tier":0} +{"problem_type":"Number Theory","problem_label":"N5","problem":"Let $\\mathbb{N}$ be the set of all positive integers. Find all functions $f: \\mathbb{N} \\rightarrow \\mathbb{N}$ such that the number $(f(m)+n)(m+f(n))$ is a square for all $m, n \\in \\mathbb{N}$. Answer. All functions of the form $f(n)=n+c$, where $c \\in \\mathbb{N} \\cup\\{0\\}$.","solution":"First, it is clear that all functions of the form $f(n)=n+c$ with a constant nonnegative integer $c$ satisfy the problem conditions since $(f(m)+n)(f(n)+m)=(n+m+c)^{2}$ is a square. We are left to prove that there are no other functions. We start with the following Lemma. Suppose that $p \\mid f(k)-f(\\ell)$ for some prime $p$ and positive integers $k, \\ell$. Then $p \\mid k-\\ell$. Proof. Suppose first that $p^{2} \\mid f(k)-f(\\ell)$, so $f(\\ell)=f(k)+p^{2} a$ for some integer $a$. Take some positive integer $D>\\max \\{f(k), f(\\ell)\\}$ which is not divisible by $p$ and set $n=p D-f(k)$. Then the positive numbers $n+f(k)=p D$ and $n+f(\\ell)=p D+(f(\\ell)-f(k))=p(D+p a)$ are both divisible by $p$ but not by $p^{2}$. Now, applying the problem conditions, we get that both the numbers $(f(k)+n)(f(n)+k)$ and $(f(\\ell)+n)(f(n)+\\ell)$ are squares divisible by $p$ (and thus by $p^{2}$ ); this means that the multipliers $f(n)+k$ and $f(n)+\\ell$ are also divisible by $p$, therefore $p \\mid(f(n)+k)-(f(n)+\\ell)=k-\\ell$ as well. On the other hand, if $f(k)-f(\\ell)$ is divisible by $p$ but not by $p^{2}$, then choose the same number $D$ and set $n=p^{3} D-f(k)$. Then the positive numbers $f(k)+n=p^{3} D$ and $f(\\ell)+n=$ $p^{3} D+(f(\\ell)-f(k))$ are respectively divisible by $p^{3}$ (but not by $p^{4}$ ) and by $p$ (but not by $p^{2}$ ). Hence in analogous way we obtain that the numbers $f(n)+k$ and $f(n)+\\ell$ are divisible by $p$, therefore $p \\mid(f(n)+k)-(f(n)+\\ell)=k-\\ell$. We turn to the problem. First, suppose that $f(k)=f(\\ell)$ for some $k, \\ell \\in \\mathbb{N}$. Then by Lemma we have that $k-\\ell$ is divisible by every prime number, so $k-\\ell=0$, or $k=\\ell$. Therefore, the function $f$ is injective. Next, consider the numbers $f(k)$ and $f(k+1)$. Since the number $(k+1)-k=1$ has no prime divisors, by Lemma the same holds for $f(k+1)-f(k)$; thus $|f(k+1)-f(k)|=1$. Now, let $f(2)-f(1)=q,|q|=1$. Then we prove by induction that $f(n)=f(1)+q(n-1)$. The base for $n=1,2$ holds by the definition of $q$. For the step, if $n>1$ we have $f(n+1)=$ $f(n) \\pm q=f(1)+q(n-1) \\pm q$. Since $f(n) \\neq f(n-2)=f(1)+q(n-2)$, we get $f(n)=f(1)+q n$, as desired. Finally, we have $f(n)=f(1)+q(n-1)$. Then $q$ cannot be -1 since otherwise for $n \\geq f(1)+1$ we have $f(n) \\leq 0$ which is impossible. Hence $q=1$ and $f(n)=(f(1)-1)+n$ for each $n \\in \\mathbb{N}$, and $f(1)-1 \\geq 0$, as desired.","tier":0} +{"problem_type":"Number Theory","problem_label":"N6","problem":"The rows and columns of a $2^{n} \\times 2^{n}$ table are numbered from 0 to $2^{n}-1$. The cells of the table have been colored with the following property being satisfied: for each $0 \\leq i, j \\leq 2^{n}-1$, the $j$ th cell in the $i$ th row and the $ Prove that the maximal possible number of colors is $2^{n}$.","solution":"Throughout the solution we denote the cells of the table by coordinate pairs; $(i, j)$ refers to the $j$ th cell in the $i$ th row. Consider the directed graph, whose vertices are the cells of the board, and the edges are the arrows $(i, j) \\rightarrow(j, i+j)$ for all $0 \\leq i, j \\leq 2^{n}-1$. From each vertex $(i, j)$, exactly one edge passes $\\left(\\right.$ to $\\left(j, i+j \\bmod 2^{n}\\right)$ ); conversely, to each cell $(j, k)$ exactly one edge is directed (from the cell $\\left.\\left(k-j \\bmod 2^{n}, j\\right)\\right)$. Hence, the graph splits into cycles. Now, in any coloring considered, the vertices of each cycle should have the same color by the problem condition. On the other hand, if each cycle has its own color, the obtained coloring obviously satisfies the problem conditions. Thus, the maximal possible number of colors is the same as the number of cycles, and we have to prove that this number is $2^{n}$. Next, consider any cycle $\\left(i_{1}, j_{1}\\right),\\left(i_{2}, j_{2}\\right), \\ldots$; we will describe it in other terms. Define a sequence $\\left(a_{0}, a_{1}, \\ldots\\right)$ by the relations $a_{0}=i_{1}, a_{1}=j_{1}, a_{n+1}=a_{n}+a_{n-1}$ for all $n \\geq 1$ (we say that such a sequence is a Fibonacci-type sequence). Then an obvious induction shows that $i_{k} \\equiv a_{k-1}\\left(\\bmod 2^{n}\\right), j_{k} \\equiv a_{k}\\left(\\bmod 2^{n}\\right)$. Hence we need to investigate the behavior of Fibonacci-type sequences modulo $2^{n}$. Denote by $F_{0}, F_{1}, \\ldots$ the Fibonacci numbers defined by $F_{0}=0, F_{1}=1$, and $F_{n+2}=$ $F_{n+1}+F_{n}$ for $n \\geq 0$. We also set $F_{-1}=1$ according to the recurrence relation. For every positive integer $m$, denote by $\\nu(m)$ the exponent of 2 in the prime factorization of $m$, i.e. for which $2^{\\nu(m)} \\mid m$ but $2^{\\nu(m)+1} \\backslash m$. Lemma 1. For every Fibonacci-type sequence $a_{0}, a_{1}, a_{2}, \\ldots$, and every $k \\geq 0$, we have $a_{k}=$ $F_{k-1} a_{0}+F_{k} a_{1}$. Proof. Apply induction on $k$. The base cases $k=0,1$ are trivial. For the step, from the induction hypothesis we get $$ a_{k+1}=a_{k}+a_{k-1}=\\left(F_{k-1} a_{0}+F_{k} a_{1}\\right)+\\left(F_{k-2} a_{0}+F_{k-1} a_{1}\\right)=F_{k} a_{0}+F_{k+1} a_{1} $$ Lemma 2. For every $m \\geq 3$, (a) we have $\\nu\\left(F_{3 \\cdot 2^{m-2}}\\right)=m$; (b) $d=3 \\cdot 2^{m-2}$ is the least positive index for which $2^{m} \\mid F_{d}$; (c) $F_{3 \\cdot 2^{m-2}+1} \\equiv 1+2^{m-1}\\left(\\bmod 2^{m}\\right)$. Proof. Apply induction on $m$. In the base case $m=3$ we have $\\nu\\left(F_{3 \\cdot 2^{m-2}}\\right)=F_{6}=8$, so $\\nu\\left(F_{3 \\cdot 2^{m-2}}\\right)=\\nu(8)=3$, the preceding Fibonacci-numbers are not divisible by 8 , and indeed $F_{3 \\cdot 2^{m-2}+1}=F_{7}=13 \\equiv 1+4(\\bmod 8)$. Now suppose that $m>3$ and let $k=3 \\cdot 2^{m-3}$. By applying Lemma 1 to the Fibonacci-type sequence $F_{k}, F_{k+1}, \\ldots$ we get $$ \\begin{gathered} F_{2 k}=F_{k-1} F_{k}+F_{k} F_{k+1}=\\left(F_{k+1}-F_{k}\\right) F_{k}+F_{k+1} F_{k}=2 F_{k+1} F_{k}-F_{k}^{2} \\\\ F_{2 k+1}=F_{k} \\cdot F_{k}+F_{k+1} \\cdot F_{k+1}=F_{k}^{2}+F_{k+1}^{2} \\end{gathered} $$ By the induction hypothesis, $\\nu\\left(F_{k}\\right)=m-1$, and $F_{k+1}$ is odd. Therefore we get $\\nu\\left(F_{k}^{2}\\right)=$ $2(m-1)>(m-1)+1=\\nu\\left(2 F_{k} F_{k+1}\\right)$, which implies $\\nu\\left(F_{2 k}\\right)=m$, establishing statement (a). Moreover, since $F_{k+1}=1+2^{m-2}+a 2^{m-1}$ for some integer $a$, we get $$ F_{2 k+1}=F_{k}^{2}+F_{k+1}^{2} \\equiv 0+\\left(1+2^{m-2}+a 2^{m-1}\\right)^{2} \\equiv 1+2^{m-1} \\quad\\left(\\bmod 2^{m}\\right) $$ as desired in statement (c). We are left to prove that $2^{m} \\backslash F_{\\ell}$ for $\\ell<2 k$. Assume the contrary. Since $2^{m-1} \\mid F_{\\ell}$, from the induction hypothesis it follows that $\\ell>k$. But then we have $F_{\\ell}=F_{k-1} F_{\\ell-k}+F_{k} F_{\\ell-k+1}$, where the second summand is divisible by $2^{m-1}$ but the first one is not (since $F_{k-1}$ is odd and $\\ell-k0$ such that $a_{k+p} \\equiv a_{k}\\left(\\bmod 2^{n}\\right)$ for all $k \\geq 0$. Lemma 3. Let $A=\\left(a_{0}, a_{1}, \\ldots\\right)$ be a Fibonacci-type sequence such that $\\mu(A)=ka_{1}+a_{3}$ and $a_{3}+a_{4}>a_{1}+a_{2}$. Hence $a_{2}+a_{4}$ and $a_{3}+a_{4}$ do not divide $s_{A}$. This proves $p_{A} \\leq 4$. Now suppose $p_{A}=4$. By the previous argument we have $$ \\begin{array}{lll} a_{1}+a_{4} \\mid a_{2}+a_{3} & \\text { and } & a_{2}+a_{3} \\mid a_{1}+a_{4}, \\\\ a_{1}+a_{2} \\mid a_{3}+a_{4} & \\text { and } & a_{3}+a_{4} \\not a_{1}+a_{2}, \\\\ a_{1}+a_{3} \\mid a_{2}+a_{4} & \\text { and } & a_{2}+a_{4} \\not a_{1}+a_{3} . \\end{array} $$ Hence, there exist positive integers $m$ and $n$ with $m>n \\geq 2$ such that $$ \\left\\{\\begin{array}{l} a_{1}+a_{4}=a_{2}+a_{3} \\\\ m\\left(a_{1}+a_{2}\\right)=a_{3}+a_{4} \\\\ n\\left(a_{1}+a_{3}\\right)=a_{2}+a_{4} \\end{array}\\right. $$ Adding up the first equation and the third one, we get $n\\left(a_{1}+a_{3}\\right)=2 a_{2}+a_{3}-a_{1}$. If $n \\geq 3$, then $n\\left(a_{1}+a_{3}\\right)>3 a_{3}>2 a_{2}+a_{3}>2 a_{2}+a_{3}-a_{1}$. This is a contradiction. Therefore $n=2$. If we multiply by 2 the sum of the first equation and the third one, we obtain $$ 6 a_{1}+2 a_{3}=4 a_{2} $$ while the sum of the first one and the second one is $$ (m+1) a_{1}+(m-1) a_{2}=2 a_{3} . $$ Adding up the last two equations we get $$ (m+7) a_{1}=(5-m) a_{2} . $$ It follows that $5-m \\geq 1$, because the left-hand side of the last equation and $a_{2}$ are positive. Since we have $m>n=2$, the integer $m$ can be equal only to either 3 or 4 . Substituting $(3,2)$ and $(4,2)$ for $(m, n)$ and solving the previous system of equations, we find the families of solutions $\\{d, 5 d, 7 d, 11 d\\}$ and $\\{d, 11 d, 19 d, 29 d\\}$, where $d$ is any positive integer.","tier":0} +{"problem_type":"Algebra","problem_label":"A2","problem":"Determine all sequences $\\left(x_{1}, x_{2}, \\ldots, x_{2011}\\right)$ of positive integers such that for every positive integer $n$ there is an integer $a$ with $$ x_{1}^{n}+2 x_{2}^{n}+\\cdots+2011 x_{2011}^{n}=a^{n+1}+1 . $$","solution":"Throughout this solution, the set of positive integers will be denoted by $\\mathbb{Z}_{+}$. Put $k=2+3+\\cdots+2011=2023065$. We have $$ 1^{n}+2 k^{n}+\\cdots 2011 k^{n}=1+k \\cdot k^{n}=k^{n+1}+1 $$ for all $n$, so $(1, k, \\ldots, k)$ is a valid sequence. We shall prove that it is the only one. Let a valid sequence $\\left(x_{1}, \\ldots, x_{2011}\\right)$ be given. For each $n \\in \\mathbb{Z}_{+}$we have some $y_{n} \\in \\mathbb{Z}_{+}$with $$ x_{1}^{n}+2 x_{2}^{n}+\\cdots+2011 x_{2011}^{n}=y_{n}^{n+1}+1 . $$ Note that $x_{1}^{n}+2 x_{2}^{n}+\\cdots+2011 x_{2011}^{n}<\\left(x_{1}+2 x_{2}+\\cdots+2011 x_{2011}\\right)^{n+1}$, which implies that the sequence $\\left(y_{n}\\right)$ is bounded. In particular, there is some $y \\in \\mathbb{Z}_{+}$with $y_{n}=y$ for infinitely many $n$. Let $m$ be the maximum of all the $x_{i}$. Grouping terms with equal $x_{i}$ together, the sum $x_{1}^{n}+$ $2 x_{2}^{n}+\\cdots+2011 x_{2011}^{n}$ can be written as $$ x_{1}^{n}+2 x_{2}^{n}+\\cdots+x_{2011}^{n}=a_{m} m^{n}+a_{m-1}(m-1)^{n}+\\cdots+a_{1} $$ with $a_{i} \\geq 0$ for all $i$ and $a_{1}+\\cdots+a_{m}=1+2+\\cdots+2011$. So there exist arbitrarily large values of $n$, for which $$ a_{m} m^{n}+\\cdots+a_{1}-1-y \\cdot y^{n}=0 \\text {. } $$ The following lemma will help us to determine the $a_{i}$ and $y$ : Lemma. Let integers $b_{1}, \\ldots, b_{N}$ be given and assume that there are arbitrarily large positive integers $n$ with $b_{1}+b_{2} 2^{n}+\\cdots+b_{N} N^{n}=0$. Then $b_{i}=0$ for all $i$. Proof. Suppose that not all $b_{i}$ are zero. We may assume without loss of generality that $b_{N} \\neq 0$. Dividing through by $N^{n}$ gives $$ \\left|b_{N}\\right|=\\left|b_{N-1}\\left(\\frac{N-1}{N}\\right)^{n}+\\cdots+b_{1}\\left(\\frac{1}{N}\\right)^{n}\\right| \\leq\\left(\\left|b_{N-1}\\right|+\\cdots+\\left|b_{1}\\right|\\right)\\left(\\frac{N-1}{N}\\right)^{n} . $$ The expression $\\left(\\frac{N-1}{N}\\right)^{n}$ can be made arbitrarily small for $n$ large enough, contradicting the assumption that $b_{N}$ be non-zero. We obviously have $y>1$. Applying the lemma to (1) we see that $a_{m}=y=m, a_{1}=1$, and all the other $a_{i}$ are zero. This implies $\\left(x_{1}, \\ldots, x_{2011}\\right)=(1, m, \\ldots, m)$. But we also have $1+m=a_{1}+\\cdots+a_{m}=1+\\cdots+2011=1+k$ so $m=k$, which is what we wanted to show.","tier":0} +{"problem_type":"Algebra","problem_label":"A3","problem":"Determine all pairs $(f, g)$ of functions from the set of real numbers to itself that satisfy $$ g(f(x+y))=f(x)+(2 x+y) g(y) $$ for all real numbers $x$ and $y$.","solution":"Clearly all these pairs of functions satisfy the functional equation in question, so it suffices to verify that there cannot be any further ones. Substituting $-2 x$ for $y$ in the given functional equation we obtain $$ g(f(-x))=f(x) . $$ Using this equation for $-x-y$ in place of $x$ we obtain $$ f(-x-y)=g(f(x+y))=f(x)+(2 x+y) g(y) . $$ Now for any two real numbers $a$ and $b$, setting $x=-b$ and $y=a+b$ we get $$ f(-a)=f(-b)+(a-b) g(a+b) . $$ If $c$ denotes another arbitrary real number we have similarly $$ f(-b)=f(-c)+(b-c) g(b+c) $$ as well as $$ f(-c)=f(-a)+(c-a) g(c+a) . $$ Adding all these equations up, we obtain $$ ((a+c)-(b+c)) g(a+b)+((a+b)-(a+c)) g(b+c)+((b+c)-(a+b)) g(a+c)=0 \\text {. } $$ Now given any three real numbers $x, y$, and $z$ one may determine three reals $a, b$, and $c$ such that $x=b+c, y=c+a$, and $z=a+b$, so that we get $$ (y-x) g(z)+(z-y) g(x)+(x-z) g(y)=0 . $$ This implies that the three points $(x, g(x)),(y, g(y))$, and $(z, g(z))$ from the graph of $g$ are collinear. Hence that graph is a line, i.e., $g$ is either a constant or a linear function. Let us write $g(x)=A x+B$, where $A$ and $B$ are two real numbers. Substituting $(0,-y)$ for $(x, y)$ in (2) and denoting $C=f(0)$, we have $f(y)=A y^{2}-B y+C$. Now, comparing the coefficients of $x^{2}$ in (1I) we see that $A^{2}=A$, so $A=0$ or $A=1$. If $A=0$, then (1) becomes $B=-B x+C$ and thus $B=C=0$, which provides the first of the two solutions mentioned above. Now suppose $A=1$. Then (1) becomes $x^{2}-B x+C+B=x^{2}-B x+C$, so $B=0$. Thus, $g(x)=x$ and $f(x)=x^{2}+C$, which is the second solution from above. Comment. Another way to show that $g(x)$ is either a constant or a linear function is the following. If we interchange $x$ and $y$ in the given functional equation and subtract this new equation from the given one, we obtain $$ f(x)-f(y)=(2 y+x) g(x)-(2 x+y) g(y) . $$ Substituting $(x, 0),(1, x)$, and $(0,1)$ for $(x, y)$, we get $$ \\begin{aligned} & f(x)-f(0)=x g(x)-2 x g(0), \\\\ & f(1)-f(x)=(2 x+1) g(1)-(x+2) g(x), \\\\ & f(0)-f(1)=2 g(0)-g(1) . \\end{aligned} $$ Taking the sum of these three equations and dividing by 2 , we obtain $$ g(x)=x(g(1)-g(0))+g(0) . $$ This proves that $g(x)$ is either a constant of a linear function.","tier":0} +{"problem_type":"Algebra","problem_label":"A4","problem":"Determine all pairs $(f, g)$ of functions from the set of positive integers to itself that satisfy $$ f^{g(n)+1}(n)+g^{f(n)}(n)=f(n+1)-g(n+1)+1 $$ for every positive integer $n$. Here, $f^{k}(n)$ means $\\underbrace{f(f(\\ldots f}_{k}(n) \\ldots))$.","solution":"The given relation implies $$ f\\left(f^{g(n)}(n)\\right)1$, then (1) reads $f\\left(f^{g(x-1)}(x-1)\\right)n$. Substituting $x-1$ into (1) we have $f\\left(f^{g(x-1)}(x-1)\\right)n$. So $b=n$, and hence $y_{n}=n$, which proves (ii) ${ }_{n}$. Next, from (i) ${ }_{n}$ we now get $f(k)=n \\Longleftrightarrow k=n$, so removing all the iterations of $f$ in (3) we obtain $x-1=b=n$, which proves (i) ${ }_{n+1}$. So, all the statements in (2) are valid and hence $f(n)=n$ for all $n$. The given relation between $f$ and $g$ now reads $n+g^{n}(n)=n+1-g(n+1)+1$ or $g^{n}(n)+g(n+1)=2$, from which it immediately follows that we have $g(n)=1$ for all $n$. Comment. Several variations of the above solution are possible. For instance, one may first prove by induction that the smallest $n$ values of $f$ are exactly $f(1)<\\cdotsn$, then $f(x)>x$ for all $x \\geq n$. From this we conclude $f^{g(n)+1}(n)>f^{g(n)}(n)>\\cdots>f(n)$. But we also have $f^{g(n)+1}(c-b)(c+b)>a^{2}$. Now we turn to the induction step. Let $n>1$ and put $t=\\lfloor n \/ 2\\rfloor\\frac{n}{2} \\cdot \\frac{9 n}{2} \\geq(n+1)^{2} \\geq i^{2}$, so this triangle is obtuse. The proof is completed.","tier":0} +{"problem_type":"Algebra","problem_label":"A6","problem":"Let $f$ be a function from the set of real numbers to itself that satisfies $$ f(x+y) \\leq y f(x)+f(f(x)) $$ for all real numbers $x$ and $y$. Prove that $f(x)=0$ for all $x \\leq 0$.","solution":"Substituting $y=t-x$, we rewrite (II) as $$ f(t) \\leq t f(x)-x f(x)+f(f(x)) . $$ Consider now some real numbers $a, b$ and use (2) with $t=f(a), x=b$ as well as with $t=f(b)$, $x=a$. We get $$ \\begin{aligned} & f(f(a))-f(f(b)) \\leq f(a) f(b)-b f(b), \\\\ & f(f(b))-f(f(a)) \\leq f(a) f(b)-a f(a) . \\end{aligned} $$ Adding these two inequalities yields $$ 2 f(a) f(b) \\geq a f(a)+b f(b) . $$ Now, substitute $b=2 f(a)$ to obtain $2 f(a) f(b) \\geq a f(a)+2 f(a) f(b)$, or $a f(a) \\leq 0$. So, we get $$ f(a) \\geq 0 \\text { for all } a<0 \\text {. } $$ Now suppose $f(x)>0$ for some real number $x$. From (21) we immediately get that for every $t<\\frac{x f(x)-f(f(x))}{f(x)}$ we have $f(t)<0$. This contradicts (3); therefore $$ f(x) \\leq 0 \\quad \\text { for all real } x $$ and by (3) again we get $f(x)=0$ for all $x<0$. We are left to find $f(0)$. Setting $t=x<0$ in (2) we get $$ 0 \\leq 0-0+f(0), $$ so $f(0) \\geq 0$. Combining this with (4) we obtain $f(0)=0$.","tier":0} +{"problem_type":"Algebra","problem_label":"A6","problem":"Let $f$ be a function from the set of real numbers to itself that satisfies $$ f(x+y) \\leq y f(x)+f(f(x)) $$ for all real numbers $x$ and $y$. Prove that $f(x)=0$ for all $x \\leq 0$.","solution":"We will also use the condition of the problem in form (2)). For clarity we divide the argument into four steps. Step 1. We begin by proving that $f$ attains nonpositive values only. Assume that there exist some real number $z$ with $f(z)>0$. Substituting $x=z$ into (2) and setting $A=f(z)$, $B=-z f(z)-f(f(z))$ we get $f(t) \\leq A t+B$ for all real $t$. Hence, if for any positive real number $t$ we substitute $x=-t, y=t$ into (11), we get $$ \\begin{aligned} f(0) & \\leq t f(-t)+f(f(-t)) \\leq t(-A t+B)+A f(-t)+B \\\\ & \\leq-t(A t-B)+A(-A t+B)+B=-A t^{2}-\\left(A^{2}-B\\right) t+(A+1) B . \\end{aligned} $$ But surely this cannot be true if we take $t$ to be large enough. This contradiction proves that we have indeed $f(x) \\leq 0$ for all real numbers $x$. Note that for this reason (11) entails $$ f(x+y) \\leq y f(x) $$ for all real numbers $x$ and $y$. Step 2. We proceed by proving that $f$ has at least one zero. If $f(0)=0$, we are done. Otherwise, in view of Step 1 we get $f(0)<0$. Observe that (5) tells us now $f(y) \\leq y f(0)$ for all real numbers $y$. Thus we can specify a positive real number $a$ that is so large that $f(a)^{2}>-f(0)$. Put $b=f(a)$ and substitute $x=b$ and $y=-b$ into (5); we learn $-b^{2}0 \\\\ 0 & \\text { if } x \\leq 0\\end{cases} $$ automatically satisfies (11). Indeed, we have $f(x) \\leq 0$ and hence also $f(f(x))=0$ for all real numbers $x$. So (11) reduces to (5); moreover, this inequality is nontrivial only if $x$ and $y$ are positive. In this last case it is provided by (6). Now it is not hard to come up with a nonzero function $g$ obeying (6). E.g. $g(z)=C e^{z}$ (where $C$ is a positive constant) fits since the inequality $e^{y}>y$ holds for all (positive) real numbers $y$. One may also consider the function $g(z)=e^{z}-1$; in this case, we even have that $f$ is continuous.","tier":0} +{"problem_type":"Algebra","problem_label":"A7","problem":"Let $a, b$, and $c$ be positive real numbers satisfying $\\min (a+b, b+c, c+a)>\\sqrt{2}$ and $a^{2}+b^{2}+c^{2}=3$. Prove that $$ \\frac{a}{(b+c-a)^{2}}+\\frac{b}{(c+a-b)^{2}}+\\frac{c}{(a+b-c)^{2}} \\geq \\frac{3}{(a b c)^{2}} $$","solution":"The condition $b+c>\\sqrt{2}$ implies $b^{2}+c^{2}>1$, so $a^{2}=3-\\left(b^{2}+c^{2}\\right)<2$, i.e. $a<\\sqrt{2}0$, and also $c+a-b>0$ and $a+b-c>0$ for similar reasons. We will use the variant of H\u00d6LDER's inequality $$ \\frac{x_{1}^{p+1}}{y_{1}^{p}}+\\frac{x_{1}^{p+1}}{y_{1}^{p}}+\\ldots+\\frac{x_{n}^{p+1}}{y_{n}^{p}} \\geq \\frac{\\left(x_{1}+x_{2}+\\ldots+x_{n}\\right)^{p+1}}{\\left(y_{1}+y_{2}+\\ldots+y_{n}\\right)^{p}} $$ which holds for all positive real numbers $p, x_{1}, x_{2}, \\ldots, x_{n}, y_{1}, y_{2}, \\ldots, y_{n}$. Applying it to the left-hand side of (11) with $p=2$ and $n=3$, we get $$ \\sum \\frac{a}{(b+c-a)^{2}}=\\sum \\frac{\\left(a^{2}\\right)^{3}}{a^{5}(b+c-a)^{2}} \\geq \\frac{\\left(a^{2}+b^{2}+c^{2}\\right)^{3}}{\\left(\\sum a^{5 \/ 2}(b+c-a)\\right)^{2}}=\\frac{27}{\\left(\\sum a^{5 \/ 2}(b+c-a)\\right)^{2}} . $$ To estimate the denominator of the right-hand part, we use an instance of SCHUR's inequality, namely $$ \\sum a^{3 \/ 2}(a-b)(a-c) \\geq 0 $$ which can be rewritten as $$ \\sum a^{5 \/ 2}(b+c-a) \\leq a b c(\\sqrt{a}+\\sqrt{b}+\\sqrt{c}) . $$ Moreover, by the inequality between the arithmetic mean and the fourth power mean we also have $$ \\left(\\frac{\\sqrt{a}+\\sqrt{b}+\\sqrt{c}}{3}\\right)^{4} \\leq \\frac{a^{2}+b^{2}+c^{2}}{3}=1 $$ i.e., $\\sqrt{a}+\\sqrt{b}+\\sqrt{c} \\leq 3$. Hence, (2) yields $$ \\sum \\frac{a}{(b+c-a)^{2}} \\geq \\frac{27}{(a b c(\\sqrt{a}+\\sqrt{b}+\\sqrt{c}))^{2}} \\geq \\frac{3}{a^{2} b^{2} c^{2}} $$ thus solving the problem. Comment. In this solution, one may also start from the following version of H\u00d6LDER's inequality $$ \\left(\\sum_{i=1}^{n} a_{i}^{3}\\right)\\left(\\sum_{i=1}^{n} b_{i}^{3}\\right)\\left(\\sum_{i=1}^{n} c_{i}^{3}\\right) \\geq\\left(\\sum_{i=1}^{n} a_{i} b_{i} c_{i}\\right)^{3} $$ applied as $$ \\sum \\frac{a}{(b+c-a)^{2}} \\cdot \\sum a^{3}(b+c-a) \\cdot \\sum a^{2}(b+c-a) \\geq 27 $$ After doing that, one only needs the slightly better known instances $$ \\sum a^{3}(b+c-a) \\leq(a+b+c) a b c \\quad \\text { and } \\quad \\sum a^{2}(b+c-a) \\leq 3 a b c $$ of Schur's Inequality.","tier":0} +{"problem_type":"Algebra","problem_label":"A7","problem":"Let $a, b$, and $c$ be positive real numbers satisfying $\\min (a+b, b+c, c+a)>\\sqrt{2}$ and $a^{2}+b^{2}+c^{2}=3$. Prove that $$ \\frac{a}{(b+c-a)^{2}}+\\frac{b}{(c+a-b)^{2}}+\\frac{c}{(a+b-c)^{2}} \\geq \\frac{3}{(a b c)^{2}} $$","solution":"As in $$ a^{5}+b^{5}+c^{5} \\geq 3 $$ which is weaker than the given one. Due to the symmetry we may assume that $a \\geq b \\geq c$. In view of (3)), it suffices to prove the inequality $$ \\sum \\frac{a^{3} b^{2} c^{2}}{(b+c-a)^{2}} \\geq \\sum a^{5} $$ or, moving all the terms into the left-hand part, $$ \\sum \\frac{a^{3}}{(b+c-a)^{2}}\\left((b c)^{2}-(a(b+c-a))^{2}\\right) \\geq 0 $$ Note that the signs of the expressions $(y z)^{2}-(x(y+z-x))^{2}$ and $y z-x(y+z-x)=(x-y)(x-z)$ are the same for every positive $x, y, z$ satisfying the triangle inequality. So the terms in (4) corresponding to $a$ and $c$ are nonnegative, and hence it is sufficient to prove that the sum of the terms corresponding to $a$ and $b$ is nonnegative. Equivalently, we need the relation $$ \\frac{a^{3}}{(b+c-a)^{2}}(a-b)(a-c)(b c+a(b+c-a)) \\geq \\frac{b^{3}}{(a+c-b)^{2}}(a-b)(b-c)(a c+b(a+c-b)) $$ Obviously, we have $$ a^{3} \\geq b^{3} \\geq 0, \\quad 0(a-b)^{2} $$ which holds since $c>a-b \\geq 0$ and $a+b>a-b \\geq 0$.","tier":0} +{"problem_type":"Combinatorics","problem_label":"C1","problem":"Let $n>0$ be an integer. We are given a balance and $n$ weights of weight $2^{0}, 2^{1}, \\ldots, 2^{n-1}$. In a sequence of $n$ moves we place all weights on the balance. In the first move we choose a weight and put it on the left pan. In each of the following moves we choose one of the remaining weights and we add it either to the left or to the right pan. Compute the number of ways in which we can perform these $n$ moves in such a way that the right pan is never heavier than the left pan. ## $\\mathrm{C} 2$ Suppose that 1000 students are standing in a circle. Prove that there exists an integer $k$ with $100 \\leq k \\leq 300$ such that in this circle there exists a contiguous group of $2 k$ students, for which the first half contains the same number of girls as the second half.","solution":"Assume $n \\geq 2$. We claim $$ f(n)=(2 n-1) f(n-1) . $$ Firstly, note that after the first move the left pan is always at least 1 heavier than the right one. Hence, any valid way of placing the $n$ weights on the scale gives rise, by not considering weight 1 , to a valid way of placing the weights $2,2^{2}, \\ldots, 2^{n-1}$. If we divide the weight of each weight by 2 , the answer does not change. So these $n-1$ weights can be placed on the scale in $f(n-1)$ valid ways. Now we look at weight 1 . If it is put on the scale in the first move, then it has to be placed on the left side, otherwise it can be placed either on the left or on the right side, because after the first move the difference between the weights on the left pan and the weights on the right pan is at least 2 . Hence, there are exactly $2 n-1$ different ways of inserting weight 1 in each of the $f(n-1)$ valid sequences for the $n-1$ weights in order to get a valid sequence for the $n$ weights. This proves the claim. Since $f(1)=1$, by induction we obtain for all positive integers $n$ $$ f(n)=(2 n-1) ! !=1 \\cdot 3 \\cdot 5 \\cdot \\ldots \\cdot(2 n-1) $$ Comment 1. The word \"compute\" in the statement of the problem is probably too vague. An alternative but more artificial question might ask for the smallest $n$ for which the number of valid ways is divisible by 2011. In this case the answer would be 1006 . Comment 2. It is useful to remark that the answer is the same for any set of weights where each weight is heavier than the sum of the lighter ones. Indeed, in such cases the given condition is equivalent to asking that during the process the heaviest weight on the balance is always on the left pan. Comment 3. Instead of considering the lightest weight, one may also consider the last weight put on the balance. If this weight is $2^{n-1}$ then it should be put on the left pan. Otherwise it may be put on any pan; the inequality would not be violated since at this moment the heaviest weight is already put onto the left pan. In view of the previous comment, in each of these $2 n-1$ cases the number of ways to place the previous weights is exactly $f(n-1)$, which yields (1).","tier":0} +{"problem_type":"Combinatorics","problem_label":"C1","problem":"Let $n>0$ be an integer. We are given a balance and $n$ weights of weight $2^{0}, 2^{1}, \\ldots, 2^{n-1}$. In a sequence of $n$ moves we place all weights on the balance. In the first move we choose a weight and put it on the left pan. In each of the following moves we choose one of the remaining weights and we add it either to the left or to the right pan. Compute the number of ways in which we can perform these $n$ moves in such a way that the right pan is never heavier than the left pan. ## $\\mathrm{C} 2$ Suppose that 1000 students are standing in a circle. Prove that there exists an integer $k$ with $100 \\leq k \\leq 300$ such that in this circle there exists a contiguous group of $2 k$ students, for which the first half contains the same number of girls as the second half.","solution":"We present a different way of obtaining (1). Set $f(0)=1$. Firstly, we find a recurrent formula for $f(n)$. Assume $n \\geq 1$. Suppose that weight $2^{n-1}$ is placed on the balance in the $i$-th move with $1 \\leq i \\leq n$. This weight has to be put on the left pan. For the previous moves we have $\\left(\\begin{array}{c}n-1 \\\\ i-1\\end{array}\\right)$ choices of the weights and from Comment 2 there are $f(i-1)$ valid ways of placing them on the balance. For later moves there is no restriction on the way in which the weights are to be put on the pans. Therefore, all $(n-i) ! 2^{n-i}$ ways are possible. This gives $$ f(n)=\\sum_{i=1}^{n}\\left(\\begin{array}{c} n-1 \\\\ i-1 \\end{array}\\right) f(i-1)(n-i) ! 2^{n-i}=\\sum_{i=1}^{n} \\frac{(n-1) ! f(i-1) 2^{n-i}}{(i-1) !} $$ Now we are ready to prove (11). Using $n-1$ instead of $n$ in (2) we get $$ f(n-1)=\\sum_{i=1}^{n-1} \\frac{(n-2) ! f(i-1) 2^{n-1-i}}{(i-1) !} . $$ Hence, again from (2) we get $$ \\begin{aligned} f(n)=2(n-1) \\sum_{i=1}^{n-1} & \\frac{(n-2) ! f(i-1) 2^{n-1-i}}{(i-1) !}+f(n-1) \\\\ & =(2 n-2) f(n-1)+f(n-1)=(2 n-1) f(n-1), \\end{aligned} $$ QED. Comment. There exist different ways of obtaining the formula (2). Here we show one of them. Suppose that in the first move we use weight $2^{n-i+1}$. Then the lighter $n-i$ weights may be put on the balance at any moment and on either pan. This gives $2^{n-i} \\cdot(n-1) ! \/(i-1)$ ! choices for the moves (moments and choices of pan) with the lighter weights. The remaining $i-1$ moves give a valid sequence for the $i-1$ heavier weights and this is the only requirement for these moves, so there are $f(i-1)$ such sequences. Summing over all $i=1,2, \\ldots, n$ we again come to (2).","tier":0} +{"problem_type":"Combinatorics","problem_label":"C3","problem":"Let $\\mathcal{S}$ be a finite set of at least two points in the plane. Assume that no three points of $\\mathcal{S}$ are collinear. By a windmill we mean a process as follows. Start with a line $\\ell$ going through a point $P \\in \\mathcal{S}$. Rotate $\\ell$ clockwise around the pivot $P$ until the line contains another point $Q$ of $\\mathcal{S}$. The point $Q$ now takes over as the new pivot. This process continues indefinitely, with the pivot always being a point from $\\mathcal{S}$. Show that for a suitable $P \\in \\mathcal{S}$ and a suitable starting line $\\ell$ containing $P$, the resulting windmill will visit each point of $\\mathcal{S}$ as a pivot infinitely often. ## $\\mathrm{C} 4$ Determine the greatest positive integer $k$ that satisfies the following property: The set of positive integers can be partitioned into $k$ subsets $A_{1}, A_{2}, \\ldots, A_{k}$ such that for all integers $n \\geq 15$ and all $i \\in\\{1,2, \\ldots, k\\}$ there exist two distinct elements of $A_{i}$ whose sum is $n$.","solution":"Give the rotating line an orientation and distinguish its sides as the oranje side and the blue side. Notice that whenever the pivot changes from some point $T$ to another point $U$, after the change, $T$ is on the same side as $U$ was before. Therefore, the number of elements of $\\mathcal{S}$ on the oranje side and the number of those on the blue side remain the same throughout the whole process (except for those moments when the line contains two points). ![](https:\/\/cdn.mathpix.com\/cropped\/2024_04_17_481ddb3576adc2313a2eg-32.jpg?height=422&width=1108&top_left_y=1198&top_left_x=474) First consider the case that $|\\mathcal{S}|=2 n+1$ is odd. We claim that through any point $T \\in \\mathcal{S}$, there is a line that has $n$ points on each side. To see this, choose an oriented line through $T$ containing no other point of $\\mathcal{S}$ and suppose that it has $n+r$ points on its oranje side. If $r=0$ then we have established the claim, so we may assume that $r \\neq 0$. As the line rotates through $180^{\\circ}$ around $T$, the number of points of $\\mathcal{S}$ on its oranje side changes by 1 whenever the line passes through a point; after $180^{\\circ}$, the number of points on the oranje side is $n-r$. Therefore there is an intermediate stage at which the oranje side, and thus also the blue side, contains $n$ points. Now select the point $P$ arbitrarily, and choose a line through $P$ that has $n$ points of $\\mathcal{S}$ on each side to be the initial state of the windmill. We will show that during a rotation over $180^{\\circ}$, the line of the windmill visits each point of $\\mathcal{S}$ as a pivot. To see this, select any point $T$ of $\\mathcal{S}$ and select a line $\\ell$ through $T$ that separates $\\mathcal{S}$ into equal halves. The point $T$ is the unique point of $\\mathcal{S}$ through which a line in this direction can separate the points of $\\mathcal{S}$ into equal halves (parallel translation would disturb the balance). Therefore, when the windmill line is parallel to $\\ell$, it must be $\\ell$ itself, and so pass through $T$. Next suppose that $|\\mathcal{S}|=2 n$. Similarly to the odd case, for every $T \\in \\mathcal{S}$ there is an oriented line through $T$ with $n-1$ points on its oranje side and $n$ points on its blue side. Select such an oriented line through an arbitrary $P$ to be the initial state of the windmill. We will now show that during a rotation over $360^{\\circ}$, the line of the windmill visits each point of $\\mathcal{S}$ as a pivot. To see this, select any point $T$ of $\\mathcal{S}$ and an oriented line $\\ell$ through $T$ that separates $\\mathcal{S}$ into two subsets with $n-1$ points on its oranje and $n$ points on its blue side. Again, parallel translation would change the numbers of points on the two sides, so when the windmill line is parallel to $\\ell$ with the same orientation, the windmill line must pass through $T$. Comment. One may shorten this solution in the following way. Suppose that $|\\mathcal{S}|=2 n+1$. Consider any line $\\ell$ that separates $\\mathcal{S}$ into equal halves; this line is unique given its direction and contains some point $T \\in \\mathcal{S}$. Consider the windmill starting from this line. When the line has made a rotation of $180^{\\circ}$, it returns to the same location but the oranje side becomes blue and vice versa. So, for each point there should have been a moment when it appeared as pivot, as this is the only way for a point to pass from on side to the other. Now suppose that $|\\mathcal{S}|=2 n$. Consider a line having $n-1$ and $n$ points on the two sides; it contains some point $T$. Consider the windmill starting from this line. After having made a rotation of $180^{\\circ}$, the windmill line contains some different point $R$, and each point different from $T$ and $R$ has changed the color of its side. So, the windmill should have passed through all the points. ## $\\mathrm{C} 4$ Determine the greatest positive integer $k$ that satisfies the following property: The set of positive integers can be partitioned into $k$ subsets $A_{1}, A_{2}, \\ldots, A_{k}$ such that for all integers $n \\geq 15$ and all $i \\in\\{1,2, \\ldots, k\\}$ there exist two distinct elements of $A_{i}$ whose sum is $n$. Answer. The greatest such number $k$ is 3 .","tier":0} +{"problem_type":"Combinatorics","problem_label":"C3","problem":"Let $\\mathcal{S}$ be a finite set of at least two points in the plane. Assume that no three points of $\\mathcal{S}$ are collinear. By a windmill we mean a process as follows. Start with a line $\\ell$ going through a point $P \\in \\mathcal{S}$. Rotate $\\ell$ clockwise around the pivot $P$ until the line contains another point $Q$ of $\\mathcal{S}$. The point $Q$ now takes over as the new pivot. This process continues indefinitely, with the pivot always being a point from $\\mathcal{S}$. Show that for a suitable $P \\in \\mathcal{S}$ and a suitable starting line $\\ell$ containing $P$, the resulting windmill will visit each point of $\\mathcal{S}$ as a pivot infinitely often. ## $\\mathrm{C} 4$ Determine the greatest positive integer $k$ that satisfies the following property: The set of positive integers can be partitioned into $k$ subsets $A_{1}, A_{2}, \\ldots, A_{k}$ such that for all integers $n \\geq 15$ and all $i \\in\\{1,2, \\ldots, k\\}$ there exist two distinct elements of $A_{i}$ whose sum is $n$.","solution":"There are various examples showing that $k=3$ does indeed have the property under consideration. E.g. one can take $$ \\begin{gathered} A_{1}=\\{1,2,3\\} \\cup\\{3 m \\mid m \\geq 4\\}, \\\\ A_{2}=\\{4,5,6\\} \\cup\\{3 m-1 \\mid m \\geq 4\\}, \\\\ A_{3}=\\{7,8,9\\} \\cup\\{3 m-2 \\mid m \\geq 4\\} \\end{gathered} $$ To check that this partition fits, we notice first that the sums of two distinct elements of $A_{i}$ obviously represent all numbers $n \\geq 1+12=13$ for $i=1$, all numbers $n \\geq 4+11=15$ for $i=2$, and all numbers $n \\geq 7+10=17$ for $i=3$. So, we are left to find representations of the numbers 15 and 16 as sums of two distinct elements of $A_{3}$. These are $15=7+8$ and $16=7+9$. Let us now suppose that for some $k \\geq 4$ there exist sets $A_{1}, A_{2}, \\ldots, A_{k}$ satisfying the given property. Obviously, the sets $A_{1}, A_{2}, A_{3}, A_{4} \\cup \\cdots \\cup A_{k}$ also satisfy the same property, so one may assume $k=4$. Put $B_{i}=A_{i} \\cap\\{1,2, \\ldots, 23\\}$ for $i=1,2,3,4$. Now for any index $i$ each of the ten numbers $15,16, \\ldots, 24$ can be written as sum of two distinct elements of $B_{i}$. Therefore this set needs to contain at least five elements. As we also have $\\left|B_{1}\\right|+\\left|B_{2}\\right|+\\left|B_{3}\\right|+\\left|B_{4}\\right|=23$, there has to be some index $j$ for which $\\left|B_{j}\\right|=5$. Let $B_{j}=\\left\\{x_{1}, x_{2}, x_{3}, x_{4}, x_{5}\\right\\}$. Finally, now the sums of two distinct elements of $A_{j}$ representing the numbers $15,16, \\ldots, 24$ should be exactly all the pairwise sums of the elements of $B_{j}$. Calculating the sum of these numbers in two different ways, we reach $$ 4\\left(x_{1}+x_{2}+x_{3}+x_{4}+x_{5}\\right)=15+16+\\ldots+24=195 . $$ Thus the number 195 should be divisible by 4 , which is false. This contradiction completes our solution. Comment. There are several variation of the proof that $k$ should not exceed 3. E.g., one may consider the sets $C_{i}=A_{i} \\cap\\{1,2, \\ldots, 19\\}$ for $i=1,2,3,4$. As in the previous solution one can show that for some index $j$ one has $\\left|C_{j}\\right|=4$, and the six pairwise sums of the elements of $C_{j}$ should represent all numbers $15,16, \\ldots, 20$. Let $C_{j}=\\left\\{y_{1}, y_{2}, y_{3}, y_{4}\\right\\}$ with $y_{1}1$. In the sequel, the word collision will be used to denote meeting of exactly two ants, moving in opposite directions. If at the beginning we place an ant on the southwest corner square facing east and an ant on the southeast corner square facing west, then they will meet in the middle of the bottom row at time $\\frac{m-1}{2}$. After the collision, the ant that moves to the north will stay on the board for another $m-\\frac{1}{2}$ time units and thus we have established an example in which the last ant falls off at time $\\frac{m-1}{2}+m-\\frac{1}{2}=\\frac{3 m}{2}-1$. So, we are left to prove that this is the latest possible moment. Consider any collision of two ants $a$ and $a^{\\prime}$. Let us change the rule for this collision, and enforce these two ants to turn anticlockwise. Then the succeeding behavior of all the ants does not change; the only difference is that $a$ and $a^{\\prime}$ swap their positions. These arguments may be applied to any collision separately, so we may assume that at any collision, either both ants rotate clockwise or both of them rotate anticlockwise by our own choice. For instance, we may assume that there are only two types of ants, depending on their initial direction: $N E$-ants, which move only north or east, and $S W$-ants, moving only south and west. Then we immediately obtain that all ants will have fallen off the board after $2 m-1$ time units. However, we can get a better bound by considering the last moment at which a given ant collides with another ant. Choose a coordinate system such that the corners of the checkerboard are $(0,0),(m, 0),(m, m)$ and $(0, m)$. At time $t$, there will be no NE-ants in the region $\\{(x, y): x+y2 m-t-1\\}$. So if two ants collide at $(x, y)$ at time $t$, we have $$ t+1 \\leq x+y \\leq 2 m-t-1 $$ Analogously, we may change the rules so that each ant would move either alternatingly north and west, or alternatingly south and east. By doing so, we find that apart from (1) we also have $|x-y| \\leq m-t-1$ for each collision at point $(x, y)$ and time $t$. To visualize this, put $$ B(t)=\\left\\{(x, y) \\in[0, m]^{2}: t+1 \\leq x+y \\leq 2 m-t-1 \\text { and }|x-y| \\leq m-t-1\\right\\} . $$ An ant can thus only collide with another ant at time $t$ if it happens to be in the region $B(t)$. The following figure displays $B(t)$ for $t=\\frac{1}{2}$ and $t=\\frac{7}{2}$ in the case $m=6$ : ![](https:\/\/cdn.mathpix.com\/cropped\/2024_04_17_481ddb3576adc2313a2eg-37.jpg?height=622&width=623&top_left_y=771&top_left_x=722) Now suppose that an NE-ant has its last collision at time $t$ and that it does so at the point $(x, y)$ (if the ant does not collide at all, it will fall off the board within $m-\\frac{1}{2}<\\frac{3 m}{2}-1$ time units, so this case can be ignored). Then we have $(x, y) \\in B(t)$ and thus $x+y \\geq t+1$ and $x-y \\geq-(m-t-1)$. So we get $$ x \\geq \\frac{(t+1)-(m-t-1)}{2}=t+1-\\frac{m}{2} $$ By symmetry we also have $y \\geq t+1-\\frac{m}{2}$, and hence $\\min \\{x, y\\} \\geq t+1-\\frac{m}{2}$. After this collision, the ant will move directly to an edge, which will take at $\\operatorname{most} m-\\min \\{x, y\\}$ units of time. In sum, the total amount of time the ant stays on the board is at most $$ t+(m-\\min \\{x, y\\}) \\leq t+m-\\left(t+1-\\frac{m}{2}\\right)=\\frac{3 m}{2}-1 $$ By symmetry, the same bound holds for $\\mathrm{SW}$-ants as well.","tier":0} +{"problem_type":"Combinatorics","problem_label":"C6","problem":"Let $n$ be a positive integer and let $W=\\ldots x_{-1} x_{0} x_{1} x_{2} \\ldots$ be an infinite periodic word consisting of the letters $a$ and $b$. Suppose that the minimal period $N$ of $W$ is greater than $2^{n}$. A finite nonempty word $U$ is said to appear in $W$ if there exist indices $k \\leq \\ell$ such that $U=x_{k} x_{k+1} \\ldots x_{\\ell}$. A finite word $U$ is called ubiquitous if the four words $U a, U b, a U$, and $b U$ all appear in $W$. Prove that there are at least $n$ ubiquitous finite nonempty words.","solution":"Throughout the solution, all the words are nonempty. For any word $R$ of length $m$, we call the number of indices $i \\in\\{1,2, \\ldots, N\\}$ for which $R$ coincides with the subword $x_{i+1} x_{i+2} \\ldots x_{i+m}$ of $W$ the multiplicity of $R$ and denote it by $\\mu(R)$. Thus a word $R$ appears in $W$ if and only if $\\mu(R)>0$. Since each occurrence of a word in $W$ is both succeeded by either the letter $a$ or the letter $b$ and similarly preceded by one of those two letters, we have $$ \\mu(R)=\\mu(R a)+\\mu(R b)=\\mu(a R)+\\mu(b R) $$ for all words $R$. We claim that the condition that $N$ is in fact the minimal period of $W$ guarantees that each word of length $N$ has multiplicity 1 or 0 depending on whether it appears or not. Indeed, if the words $x_{i+1} x_{i+2} \\ldots x_{i+N}$ and $x_{j+1} \\ldots x_{j+N}$ are equal for some $1 \\leq i2^{n}$, at least one of the two words $a$ and $b$ has a multiplicity that is strictly larger than $2^{n-1}$. For each $k=0,1, \\ldots, n-1$, let $U_{k}$ be a subword of $W$ whose multiplicity is strictly larger than $2^{k}$ and whose length is maximal subject to this property. Note that such a word exists in view of the two observations made in the two previous paragraphs. Fix some index $k \\in\\{0,1, \\ldots, n-1\\}$. Since the word $U_{k} b$ is longer than $U_{k}$, its multiplicity can be at most $2^{k}$, so in particular $\\mu\\left(U_{k} b\\right)<\\mu\\left(U_{k}\\right)$. Therefore, the word $U_{k} a$ has to appear by (11). For a similar reason, the words $U_{k} b, a U_{k}$, and $b U_{k}$ have to appear as well. Hence, the word $U_{k}$ is ubiquitous. Moreover, if the multiplicity of $U_{k}$ were strictly greater than $2^{k+1}$, then by (1) at least one of the two words $U_{k} a$ and $U_{k} b$ would have multiplicity greater than $2^{k}$ and would thus violate the maximality condition imposed on $U_{k}$. So we have $\\mu\\left(U_{0}\\right) \\leq 2<\\mu\\left(U_{1}\\right) \\leq 4<\\ldots \\leq 2^{n-1}<\\mu\\left(U_{n-1}\\right)$, which implies in particular that the words $U_{0}, U_{1}, \\ldots, U_{n-1}$ have to be distinct. As they have been proved to be ubiquitous as well, the problem is solved. Comment 1. There is an easy construction for obtaining ubiquitous words from appearing words whose multiplicity is at least two. Starting with any such word $U$ we may simply extend one of its occurrences in $W$ forwards and backwards as long as its multiplicity remains fixed, thus arriving at a word that one might call the ubiquitous prolongation $p(U)$ of $U$. There are several variants of the argument in the second half of the solution using the concept of prolongation. For instance, one may just take all ubiquitous words $U_{1}, U_{2}, \\ldots, U_{\\ell}$ ordered by increasing multiplicity and then prove for $i \\in\\{1,2, \\ldots, \\ell\\}$ that $\\mu\\left(U_{i}\\right) \\leq 2^{i}$. Indeed, assume that $i$ is a minimal counterexample to this statement; then by the arguments similar to those presented above, the ubiquitous prolongation of one of the words $U_{i} a, U_{i} b, a U_{i}$ or $b U_{i}$ violates the definition of $U_{i}$. Now the multiplicity of one of the two letters $a$ and $b$ is strictly greater than $2^{n-1}$, so passing to ubiquitous prolongations once more we obtain $2^{n-1}<\\mu\\left(U_{\\ell}\\right) \\leq 2^{\\ell}$, which entails $\\ell \\geq n$, as needed. Comment 2. The bound $n$ for the number of ubiquitous subwords in the problem statement is not optimal, but it is close to an optimal one in the following sense. There is a universal constant $C>0$ such that for each positive integer $n$ there exists an infinite periodic word $W$ whose minimal period is greater than $2^{n}$ but for which there exist fewer than $C n$ ubiquitous words.","tier":0} +{"problem_type":"Combinatorics","problem_label":"C7","problem":"On a square table of 2011 by 2011 cells we place a finite number of napkins that each cover a square of 52 by 52 cells. In each cell we write the number of napkins covering it, and we record the maximal number $k$ of cells that all contain the same nonzero number. Considering all possible napkin configurations, what is the largest value of $k$ ?","solution":"Let $m=39$, then $2011=52 m-17$. We begin with an example showing that there can exist 3986729 cells carrying the same positive number. ![](https:\/\/cdn.mathpix.com\/cropped\/2024_04_17_481ddb3576adc2313a2eg-40.jpg?height=577&width=580&top_left_y=1022&top_left_x=744) To describe it, we number the columns from the left to the right and the rows from the bottom to the top by $1,2, \\ldots, 2011$. We will denote each napkin by the coordinates of its lowerleft cell. There are four kinds of napkins: first, we take all napkins $(52 i+36,52 j+1)$ with $0 \\leq j \\leq i \\leq m-2$; second, we use all napkins $(52 i+1,52 j+36)$ with $0 \\leq i \\leq j \\leq m-2$; third, we use all napkins $(52 i+36,52 i+36)$ with $0 \\leq i \\leq m-2$; and finally the napkin $(1,1)$. Different groups of napkins are shown by different types of hatchings in the picture. Now except for those squares that carry two or more different hatchings, all squares have the number 1 written into them. The number of these exceptional cells is easily computed to be $\\left(52^{2}-35^{2}\\right) m-17^{2}=57392$. We are left to prove that 3986729 is an upper bound for the number of cells containing the same number. Consider any configuration of napkins and any positive integer $M$. Suppose there are $g$ cells with a number different from $M$. Then it suffices to show $g \\geq 57392$. Throughout the solution, a line will mean either a row or a column. Consider any line $\\ell$. Let $a_{1}, \\ldots, a_{52 m-17}$ be the numbers written into its consecutive cells. For $i=1,2, \\ldots, 52$, let $s_{i}=\\sum_{t \\equiv i(\\bmod 52)} a_{t}$. Note that $s_{1}, \\ldots, s_{35}$ have $m$ terms each, while $s_{36}, \\ldots, s_{52}$ have $m-1$ terms each. Every napkin intersecting $\\ell$ contributes exactly 1 to each $s_{i}$; hence the number $s$ of all those napkins satisfies $s_{1}=\\cdots=s_{52}=s$. Call the line $\\ell$ rich if $s>(m-1) M$ and poor otherwise. Suppose now that $\\ell$ is rich. Then in each of the sums $s_{36}, \\ldots, s_{52}$ there exists a term greater than $M$; consider all these terms and call the corresponding cells the rich bad cells for this line. So, each rich line contains at least 17 cells that are bad for this line. If, on the other hand, $\\ell$ is poor, then certainly $s2 H^{2}$, then the answer and the example are the same as in the previous case; otherwise the answer is $(2 m-1) S^{2}+2 S H(m-1)$, and the example is provided simply by $(m-1)^{2}$ nonintersecting napkins. Now we sketch the proof of both estimates for Case 2. We introduce a more appropriate notation based on that from Solution 2. Denote by $a_{-}$and $a_{+}$the number of cells of class $A$ that contain the number which is strictly less than $M$ and strictly greater than $M$, respectively. The numbers $b_{ \\pm}, c_{ \\pm}$, and $d_{ \\pm}$are defined in a similar way. One may notice that the proofs of Claim 1 and Claims 2, 3 lead in fact to the inequalities $$ m-1 \\leq \\frac{b_{-}+c_{-}}{2 S H}+\\frac{d_{+}}{H^{2}} \\quad \\text { and } \\quad 2 m-1 \\leq \\frac{a}{S^{2}}+\\frac{b_{+}+c_{+}}{2 S H}+\\frac{d_{+}}{H^{2}} $$ (to obtain the first one, one needs to look at the big lines instead of the small ones). Combining these inequalities, one may obtain the desired estimates. These estimates can also be proved in some different ways, e.g. without distinguishing rich and poor cells.","tier":0} +{"problem_type":"Geometry","problem_label":"G1","problem":"Let $A B C$ be an acute triangle. Let $\\omega$ be a circle whose center $L$ lies on the side $B C$. Suppose that $\\omega$ is tangent to $A B$ at $B^{\\prime}$ and to $A C$ at $C^{\\prime}$. Suppose also that the circumcenter $O$ of the triangle $A B C$ lies on the shorter $\\operatorname{arc} B^{\\prime} C^{\\prime}$ of $\\omega$. Prove that the circumcircle of $A B C$ and $\\omega$ meet at two points.","solution":"The point $B^{\\prime}$, being the perpendicular foot of $L$, is an interior point of side $A B$. Analogously, $C^{\\prime}$ lies in the interior of $A C$. The point $O$ is located inside the triangle $A B^{\\prime} C^{\\prime}$, hence $\\angle C O B<\\angle C^{\\prime} O B^{\\prime}$. ![](https:\/\/cdn.mathpix.com\/cropped\/2024_04_17_481ddb3576adc2313a2eg-45.jpg?height=837&width=674&top_left_y=895&top_left_x=697) Let $\\alpha=\\angle C A B$. The angles $\\angle C A B$ and $\\angle C^{\\prime} O B^{\\prime}$ are inscribed into the two circles with centers $O$ and $L$, respectively, so $\\angle C O B=2 \\angle C A B=2 \\alpha$ and $2 \\angle C^{\\prime} O B^{\\prime}=360^{\\circ}-\\angle C^{\\prime} L B^{\\prime}$. From the kite $A B^{\\prime} L C^{\\prime}$ we have $\\angle C^{\\prime} L B^{\\prime}=180^{\\circ}-\\angle C^{\\prime} A B^{\\prime}=180^{\\circ}-\\alpha$. Combining these, we get $$ 2 \\alpha=\\angle C O B<\\angle C^{\\prime} O B^{\\prime}=\\frac{360^{\\circ}-\\angle C^{\\prime} L B^{\\prime}}{2}=\\frac{360^{\\circ}-\\left(180^{\\circ}-\\alpha\\right)}{2}=90^{\\circ}+\\frac{\\alpha}{2}, $$ So $$ \\alpha<60^{\\circ} \\text {. } $$ Let $O^{\\prime}$ be the reflection of $O$ in the line $B C$. In the quadrilateral $A B O^{\\prime} C$ we have $$ \\angle C O^{\\prime} B+\\angle C A B=\\angle C O B+\\angle C A B=2 \\alpha+\\alpha<180^{\\circ}, $$ so the point $O^{\\prime}$ is outside the circle $A B C$. Hence, $O$ and $O^{\\prime}$ are two points of $\\omega$ such that one of them lies inside the circumcircle, while the other one is located outside. Therefore, the two circles intersect. Comment. There are different ways of reducing the statement of the problem to the case $\\alpha<60^{\\circ}$. E.g., since the point $O$ lies in the interior of the isosceles triangle $A B^{\\prime} C^{\\prime}$, we have $O A2 L B^{\\prime}$, and this condition implies $\\angle C A B=2 \\angle B^{\\prime} A L<2 \\cdot 30^{\\circ}=60^{\\circ}$.","tier":0} +{"problem_type":"Geometry","problem_label":"G2","problem":"Let $A_{1} A_{2} A_{3} A_{4}$ be a non-cyclic quadrilateral. Let $O_{1}$ and $r_{1}$ be the circumcenter and the circumradius of the triangle $A_{2} A_{3} A_{4}$. Define $O_{2}, O_{3}, O_{4}$ and $r_{2}, r_{3}, r_{4}$ in a similar way. Prove that $$ \\frac{1}{O_{1} A_{1}^{2}-r_{1}^{2}}+\\frac{1}{O_{2} A_{2}^{2}-r_{2}^{2}}+\\frac{1}{O_{3} A_{3}^{2}-r_{3}^{2}}+\\frac{1}{O_{4} A_{4}^{2}-r_{4}^{2}}=0 $$","solution":"Let $M$ be the point of intersection of the diagonals $A_{1} A_{3}$ and $A_{2} A_{4}$. On each diagonal choose a direction and let $x, y, z$, and $w$ be the signed distances from $M$ to the points $A_{1}, A_{2}, A_{3}$, and $A_{4}$, respectively. Let $\\omega_{1}$ be the circumcircle of the triangle $A_{2} A_{3} A_{4}$ and let $B_{1}$ be the second intersection point of $\\omega_{1}$ and $A_{1} A_{3}$ (thus, $B_{1}=A_{3}$ if and only if $A_{1} A_{3}$ is tangent to $\\omega_{1}$ ). Since the expression $O_{1} A_{1}^{2}-r_{1}^{2}$ is the power of the point $A_{1}$ with respect to $\\omega_{1}$, we get $$ O_{1} A_{1}^{2}-r_{1}^{2}=A_{1} B_{1} \\cdot A_{1} A_{3} $$ On the other hand, from the equality $M B_{1} \\cdot M A_{3}=M A_{2} \\cdot M A_{4}$ we obtain $M B_{1}=y w \/ z$. Hence, we have $$ O_{1} A_{1}^{2}-r_{1}^{2}=\\left(\\frac{y w}{z}-x\\right)(z-x)=\\frac{z-x}{z}(y w-x z) . $$ Substituting the analogous expressions into the sought sum we get $$ \\sum_{i=1}^{4} \\frac{1}{O_{i} A_{i}^{2}-r_{i}^{2}}=\\frac{1}{y w-x z}\\left(\\frac{z}{z-x}-\\frac{w}{w-y}+\\frac{x}{x-z}-\\frac{y}{y-w}\\right)=0 $$ as desired. Comment. One might reformulate the problem by assuming that the quadrilateral $A_{1} A_{2} A_{3} A_{4}$ is convex. This should not really change the difficulty, but proofs that distinguish several cases may become shorter.","tier":0} +{"problem_type":"Geometry","problem_label":"G2","problem":"Let $A_{1} A_{2} A_{3} A_{4}$ be a non-cyclic quadrilateral. Let $O_{1}$ and $r_{1}$ be the circumcenter and the circumradius of the triangle $A_{2} A_{3} A_{4}$. Define $O_{2}, O_{3}, O_{4}$ and $r_{2}, r_{3}, r_{4}$ in a similar way. Prove that $$ \\frac{1}{O_{1} A_{1}^{2}-r_{1}^{2}}+\\frac{1}{O_{2} A_{2}^{2}-r_{2}^{2}}+\\frac{1}{O_{3} A_{3}^{2}-r_{3}^{2}}+\\frac{1}{O_{4} A_{4}^{2}-r_{4}^{2}}=0 $$","solution":"Introduce a Cartesian coordinate system in the plane. Every circle has an equation of the form $p(x, y)=x^{2}+y^{2}+l(x, y)=0$, where $l(x, y)$ is a polynomial of degree at most 1 . For any point $A=\\left(x_{A}, y_{A}\\right)$ we have $p\\left(x_{A}, y_{A}\\right)=d^{2}-r^{2}$, where $d$ is the distance from $A$ to the center of the circle and $r$ is the radius of the circle. For each $i$ in $\\{1,2,3,4\\}$ let $p_{i}(x, y)=x^{2}+y^{2}+l_{i}(x, y)=0$ be the equation of the circle with center $O_{i}$ and radius $r_{i}$ and let $d_{i}$ be the distance from $A_{i}$ to $O_{i}$. Consider the equation $$ \\sum_{i=1}^{4} \\frac{p_{i}(x, y)}{d_{i}^{2}-r_{i}^{2}}=1 $$ Since the coordinates of the points $A_{1}, A_{2}, A_{3}$, and $A_{4}$ satisfy (1) but these four points do not lie on a circle or on an line, equation (1) defines neither a circle, nor a line. Hence, the equation is an identity and the coefficient of the quadratic term $x^{2}+y^{2}$ also has to be zero, i.e. $$ \\sum_{i=1}^{4} \\frac{1}{d_{i}^{2}-r_{i}^{2}}=0 $$ Comment. Using the determinant form of the equation of the circle through three given points, the same solution can be formulated as follows. For $i=1,2,3,4$ let $\\left(u_{i}, v_{i}\\right)$ be the coordinates of $A_{i}$ and define $$ \\Delta=\\left|\\begin{array}{llll} u_{1}^{2}+v_{1}^{2} & u_{1} & v_{1} & 1 \\\\ u_{2}^{2}+v_{2}^{2} & u_{2} & v_{2} & 1 \\\\ u_{3}^{2}+v_{3}^{2} & u_{3} & v_{3} & 1 \\\\ u_{4}^{2}+v_{4}^{2} & u_{4} & v_{4} & 1 \\end{array}\\right| \\quad \\text { and } \\quad \\Delta_{i}=\\left|\\begin{array}{lll} u_{i+1} & v_{i+1} & 1 \\\\ u_{i+2} & v_{i+2} & 1 \\\\ u_{i+3} & v_{i+3} & 1 \\end{array}\\right| $$ where $i+1, i+2$, and $i+3$ have to be read modulo 4 as integers in the set $\\{1,2,3,4\\}$. Expanding $\\left|\\begin{array}{llll}u_{1} & v_{1} & 1 & 1 \\\\ u_{2} & v_{2} & 1 & 1 \\\\ u_{3} & v_{3} & 1 & 1 \\\\ u_{4} & v_{4} & 1 & 1\\end{array}\\right|=0$ along the third column, we get $\\Delta_{1}-\\Delta_{2}+\\Delta_{3}-\\Delta_{4}=0$. The circle through $A_{i+1}, A_{i+2}$, and $A_{i+3}$ is given by the equation $$ \\frac{1}{\\Delta_{i}}\\left|\\begin{array}{cccc} x^{2}+y^{2} & x & y & 1 \\\\ u_{i+1}^{2}+v_{i+1}^{2} & u_{i+1} & v_{i+1} & 1 \\\\ u_{i+2}^{2}+v_{i+2}^{2} & u_{i+2} & v_{i+2} & 1 \\\\ u_{i+3}^{2}+v_{i+3}^{2} & u_{i+3} & v_{i+3} & 1 \\end{array}\\right|=0 $$ On the left-hand side, the coefficient of $x^{2}+y^{2}$ is equal to 1 . Substituting $\\left(u_{i}, v_{i}\\right)$ for $(x, y)$ in (2) we obtain the power of point $A_{i}$ with respect to the circle through $A_{i+1}, A_{i+2}$, and $A_{i+3}$ : $$ d_{i}^{2}-r_{i}^{2}=\\frac{1}{\\Delta_{i}}\\left|\\begin{array}{cccc} u_{i}^{2}+v_{i}^{2} & u_{i} & v_{i} & 1 \\\\ u_{i+1}^{2}+v_{i+1}^{2} & u_{i+1} & v_{i+1} & 1 \\\\ u_{i+2}^{2}+v_{i+2}^{2} & u_{i+2} & v_{i+2} & 1 \\\\ u_{i+3}^{2}+v_{i+3}^{2} & u_{i+3} & v_{i+3} & 1 \\end{array}\\right|=(-1)^{i+1} \\frac{\\Delta}{\\Delta_{i}} $$ Thus, we have $$ \\sum_{i=1}^{4} \\frac{1}{d_{i}^{2}-r_{i}^{2}}=\\frac{\\Delta_{1}-\\Delta_{2}+\\Delta_{3}-\\Delta_{4}}{\\Delta}=0 $$","tier":0} +{"problem_type":"Geometry","problem_label":"G3","problem":"Let $A B C D$ be a convex quadrilateral whose sides $A D$ and $B C$ are not parallel. Suppose that the circles with diameters $A B$ and $C D$ meet at points $E$ and $F$ inside the quadrilateral. Let $\\omega_{E}$ be the circle through the feet of the perpendiculars from $E$ to the lines $A B, B C$, and $C D$. Let $\\omega_{F}$ be the circle through the feet of the perpendiculars from $F$ to the lines $C D, D A$, and $A B$. Prove that the midpoint of the segment $E F$ lies on the line through the two intersection points of $\\omega_{E}$ and $\\omega_{F}$.","solution":"Denote by $P, Q, R$, and $S$ the projections of $E$ on the lines $D A, A B, B C$, and $C D$ respectively. The points $P$ and $Q$ lie on the circle with diameter $A E$, so $\\angle Q P E=\\angle Q A E$; analogously, $\\angle Q R E=\\angle Q B E$. So $\\angle Q P E+\\angle Q R E=\\angle Q A E+\\angle Q B E=90^{\\circ}$. By similar reasons, we have $\\angle S P E+\\angle S R E=90^{\\circ}$, hence we get $\\angle Q P S+\\angle Q R S=90^{\\circ}+90^{\\circ}=180^{\\circ}$, and the quadrilateral $P Q R S$ is inscribed in $\\omega_{E}$. Analogously, all four projections of $F$ onto the sides of $A B C D$ lie on $\\omega_{F}$. Denote by $K$ the meeting point of the lines $A D$ and $B C$. Due to the arguments above, there is no loss of generality in assuming that $A$ lies on segment $D K$. Suppose that $\\angle C K D \\geq 90^{\\circ}$; then the circle with diameter $C D$ covers the whole quadrilateral $A B C D$, so the points $E, F$ cannot lie inside this quadrilateral. Hence our assumption is wrong. Therefore, the lines EP and $B C$ intersect at some point $P^{\\prime}$, while the lines $E R$ and $A D$ intersect at some point $R^{\\prime}$. ![](https:\/\/cdn.mathpix.com\/cropped\/2024_04_17_481ddb3576adc2313a2eg-49.jpg?height=794&width=1376&top_left_y=1522&top_left_x=340) Figure 1 We claim that the points $P^{\\prime}$ and $R^{\\prime}$ also belong to $\\omega_{E}$. Since the points $R, E, Q, B$ are concyclic, $\\angle Q R K=\\angle Q E B=90^{\\circ}-\\angle Q B E=\\angle Q A E=\\angle Q P E$. So $\\angle Q R K=\\angle Q P P^{\\prime}$, which means that the point $P^{\\prime}$ lies on $\\omega_{E}$. Analogously, $R^{\\prime}$ also lies on $\\omega_{E}$. In the same manner, denote by $M$ and $N$ the projections of $F$ on the lines $A D$ and $B C$ respectively, and let $M^{\\prime}=F M \\cap B C, N^{\\prime}=F N \\cap A D$. By the same arguments, we obtain that the points $M^{\\prime}$ and $N^{\\prime}$ belong to $\\omega_{F}$. ![](https:\/\/cdn.mathpix.com\/cropped\/2024_04_17_481ddb3576adc2313a2eg-50.jpg?height=782&width=1291&top_left_y=340&top_left_x=385) Figure 2 Now we concentrate on Figure 2, where all unnecessary details are removed. Let $U=N N^{\\prime} \\cap$ $P P^{\\prime}, V=M M^{\\prime} \\cap R R^{\\prime}$. Due to the right angles at $N$ and $P$, the points $N, N^{\\prime}, P, P^{\\prime}$ are concyclic, so $U N \\cdot U N^{\\prime}=U P \\cdot U P^{\\prime}$ which means that $U$ belongs to the radical axis $g$ of the circles $\\omega_{E}$ and $\\omega_{F}$. Analogously, $V$ also belongs to $g$. Finally, since $E U F V$ is a parallelogram, the radical axis $U V$ of $\\omega_{E}$ and $\\omega_{F}$ bisects $E F$.","tier":0} +{"problem_type":"Geometry","problem_label":"G4","problem":"Let $A B C$ be an acute triangle with circumcircle $\\Omega$. Let $B_{0}$ be the midpoint of $A C$ and let $C_{0}$ be the midpoint of $A B$. Let $D$ be the foot of the altitude from $A$, and let $G$ be the centroid of the triangle $A B C$. Let $\\omega$ be a circle through $B_{0}$ and $C_{0}$ that is tangent to the circle $\\Omega$ at a point $X \\neq A$. Prove that the points $D, G$, and $X$ are collinear.","solution":"If $A B=A C$, then the statement is trivial. So without loss of generality we may assume $A B0$, let $f(d)$ be the smallest positive integer that has exactly $d$ positive divisors (so for example we have $f(1)=1, f(5)=16$, and $f(6)=12$ ). Prove that for every integer $k \\geq 0$ the number $f\\left(2^{k}\\right)$ divides $f\\left(2^{k+1}\\right)$.","solution":"For any positive integer $n$, let $d(n)$ be the number of positive divisors of $n$. Let $n=\\prod_{p} p^{a(p)}$ be the prime factorization of $n$ where $p$ ranges over the prime numbers, the integers $a(p)$ are nonnegative and all but finitely many $a(p)$ are zero. Then we have $d(n)=\\prod_{p}(a(p)+1)$. Thus, $d(n)$ is a power of 2 if and only if for every prime $p$ there is a nonnegative integer $b(p)$ with $a(p)=2^{b(p)}-1=1+2+2^{2}+\\cdots+2^{b(p)-1}$. We then have $$ n=\\prod_{p} \\prod_{i=0}^{b(p)-1} p^{2^{i}}, \\quad \\text { and } \\quad d(n)=2^{k} \\quad \\text { with } \\quad k=\\sum_{p} b(p) $$ Let $\\mathcal{S}$ be the set of all numbers of the form $p^{2^{r}}$ with $p$ prime and $r$ a nonnegative integer. Then we deduce that $d(n)$ is a power of 2 if and only if $n$ is the product of the elements of some finite subset $\\mathcal{T}$ of $\\mathcal{S}$ that satisfies the following condition: for all $t \\in \\mathcal{T}$ and $s \\in \\mathcal{S}$ with $s \\mid t$ we have $s \\in \\mathcal{T}$. Moreover, if $d(n)=2^{k}$ then the corresponding set $\\mathcal{T}$ has $k$ elements. Note that the set $\\mathcal{T}_{k}$ consisting of the smallest $k$ elements from $\\mathcal{S}$ obviously satisfies the condition above. Thus, given $k$, the smallest $n$ with $d(n)=2^{k}$ is the product of the elements of $\\mathcal{T}_{k}$. This $n$ is $f\\left(2^{k}\\right)$. Since obviously $\\mathcal{T}_{k} \\subset \\mathcal{T}_{k+1}$, it follows that $f\\left(2^{k}\\right) \\mid f\\left(2^{k+1}\\right)$.","tier":0} +{"problem_type":"Number Theory","problem_label":"N1","problem":"For any integer $d>0$, let $f(d)$ be the smallest positive integer that has exactly $d$ positive divisors (so for example we have $f(1)=1, f(5)=16$, and $f(6)=12$ ). Prove that for every integer $k \\geq 0$ the number $f\\left(2^{k}\\right)$ divides $f\\left(2^{k+1}\\right)$.","solution":"This is an alternative to the second part of the $$ m=p^{2^{b(p)}}>q^{2^{b(q)-1}}=\\ell . $$ To see this, note first that $\\ell$ divides $f\\left(2^{k}\\right)$. With the first part of Solution 1 one can see that the integer $n=f\\left(2^{k}\\right) m \/ \\ell$ also satisfies $d(n)=2^{k}$. By the definition of $f\\left(2^{k}\\right)$ this implies that $n \\geq f\\left(2^{k}\\right)$ so $m \\geq \\ell$. Since $p \\neq q$ the inequality (1) follows. Let the prime factorization of $f\\left(2^{k+1}\\right)$ be given by $f\\left(2^{k+1}\\right)=\\prod_{p} p^{r(p)}$ with $r(p)=2^{s(p)}-1$. Since we have $\\sum_{p} s(p)=k+1>k=\\sum_{p} b(p)$ there is a prime $p$ with $s(p)>b(p)$. For any prime $q \\neq p$ with $b(q)>0$ we apply inequality (1) twice and get $$ q^{2^{s(q)}}>p^{2^{s(p)-1}} \\geq p^{2^{b(p)}}>q^{2^{b(q)-1}} $$ which implies $s(q) \\geq b(q)$. It follows that $s(q) \\geq b(q)$ for all primes $q$, so $f\\left(2^{k}\\right) \\mid f\\left(2^{k+1}\\right)$.","tier":0} +{"problem_type":"Number Theory","problem_label":"N2","problem":"Consider a polynomial $P(x)=\\left(x+d_{1}\\right)\\left(x+d_{2}\\right) \\cdot \\ldots \\cdot\\left(x+d_{9}\\right)$, where $d_{1}, d_{2}, \\ldots, d_{9}$ are nine distinct integers. Prove that there exists an integer $N$ such that for all integers $x \\geq N$ the number $P(x)$ is divisible by a prime number greater than 20 .","solution":"Note that the statement of the problem is invariant under translations of $x$; hence without loss of generality we may suppose that the numbers $d_{1}, d_{2}, \\ldots, d_{9}$ are positive. The key observation is that there are only eight primes below 20, while $P(x)$ involves more than eight factors. We shall prove that $N=d^{8}$ satisfies the desired property, where $d=\\max \\left\\{d_{1}, d_{2}, \\ldots, d_{9}\\right\\}$. Suppose for the sake of contradiction that there is some integer $x \\geq N$ such that $P(x)$ is composed of primes below 20 only. Then for every index $i \\in\\{1,2, \\ldots, 9\\}$ the number $x+d_{i}$ can be expressed as product of powers of the first 8 primes. Since $x+d_{i}>x \\geq d^{8}$ there is some prime power $f_{i}>d$ that divides $x+d_{i}$. Invoking the pigeonhole principle we see that there are two distinct indices $i$ and $j$ such that $f_{i}$ and $f_{j}$ are powers of the same prime number. For reasons of symmetry, we may suppose that $f_{i} \\leq f_{j}$. Now both of the numbers $x+d_{i}$ and $x+d_{j}$ are divisible by $f_{i}$ and hence so is their difference $d_{i}-d_{j}$. But as $$ 0<\\left|d_{i}-d_{j}\\right| \\leq \\max \\left(d_{i}, d_{j}\\right) \\leq dD_{i}$ the numerator of the fraction we thereby get cannot be 1 , and hence it has to be divisible by some prime number $p_{i}<20$. By the pigeonhole principle, there are a prime number $p$ and two distinct indices $i$ and $j$ such that $p_{i}=p_{j}=p$. Let $p^{\\alpha_{i}}$ and $p^{\\alpha_{j}}$ be the greatest powers of $p$ dividing $x+d_{i}$ and $x+d_{j}$, respectively. Due to symmetry we may suppose $\\alpha_{i} \\leq \\alpha_{j}$. But now $p^{\\alpha_{i}}$ divides $d_{i}-d_{j}$ and hence also $D_{i}$, which means that all occurrences of $p$ in the numerator of the fraction $\\left(x+d_{i}\\right) \/ D_{i}$ cancel out, contrary to the choice of $p=p_{i}$. This contradiction proves our claim.","tier":0} +{"problem_type":"Number Theory","problem_label":"N2","problem":"Consider a polynomial $P(x)=\\left(x+d_{1}\\right)\\left(x+d_{2}\\right) \\cdot \\ldots \\cdot\\left(x+d_{9}\\right)$, where $d_{1}, d_{2}, \\ldots, d_{9}$ are nine distinct integers. Prove that there exists an integer $N$ such that for all integers $x \\geq N$ the number $P(x)$ is divisible by a prime number greater than 20 .","solution":"Given a nonzero integer $N$ as well as a prime number $p$ we write $v_{p}(N)$ for the exponent with which $p$ occurs in the prime factorization of $|N|$. Evidently, if the statement of the problem were not true, then there would exist an infinite sequence $\\left(x_{n}\\right)$ of positive integers tending to infinity such that for each $n \\in \\mathbb{Z}_{+}$the integer $P\\left(x_{n}\\right)$ is not divisible by any prime number $>20$. Observe that the numbers $-d_{1},-d_{2}, \\ldots,-d_{9}$ do not appear in this sequence. Now clearly there exists a prime $p_{1}<20$ for which the sequence $v_{p_{1}}\\left(x_{n}+d_{1}\\right)$ is not bounded; thinning out the sequence $\\left(x_{n}\\right)$ if necessary we may even suppose that $$ v_{p_{1}}\\left(x_{n}+d_{1}\\right) \\longrightarrow \\infty . $$ Repeating this argument eight more times we may similarly choose primes $p_{2}, \\ldots, p_{9}<20$ and suppose that our sequence $\\left(x_{n}\\right)$ has been thinned out to such an extent that $v_{p_{i}}\\left(x_{n}+d_{i}\\right) \\longrightarrow \\infty$ holds for $i=2, \\ldots, 9$ as well. In view of the pigeonhole principle, there are distinct indices $i$ and $j$ as well as a prime $p<20$ such that $p_{i}=p_{j}=p$. Setting $k=v_{p}\\left(d_{i}-d_{j}\\right)$ there now has to be some $n$ for which both $v_{p}\\left(x_{n}+d_{i}\\right)$ and $v_{p}\\left(x_{n}+d_{j}\\right)$ are greater than $k$. But now the numbers $x_{n}+d_{i}$ and $x_{n}+d_{j}$ are divisible by $p^{k+1}$ whilst their difference $d_{i}-d_{j}$ is not - a contradiction. Comment. This problem is supposed to be a relatively easy one, so one might consider adding the hypothesis that the numbers $d_{1}, d_{2}, \\ldots, d_{9}$ be positive. Then certain merely technical issues are not going to arise while the main ideas required to solve the problems remain the same. Let $n \\geq 1$ be an odd integer. Determine all functions $f$ from the set of integers to itself such that for all integers $x$ and $y$ the difference $f(x)-f(y)$ divides $x^{n}-y^{n}$. Answer. All functions $f$ of the form $f(x)=\\varepsilon x^{d}+c$, where $\\varepsilon$ is in $\\{1,-1\\}$, the integer $d$ is a positive divisor of $n$, and $c$ is an integer.","tier":0} +{"problem_type":"Number Theory","problem_label":"N2","problem":"Consider a polynomial $P(x)=\\left(x+d_{1}\\right)\\left(x+d_{2}\\right) \\cdot \\ldots \\cdot\\left(x+d_{9}\\right)$, where $d_{1}, d_{2}, \\ldots, d_{9}$ are nine distinct integers. Prove that there exists an integer $N$ such that for all integers $x \\geq N$ the number $P(x)$ is divisible by a prime number greater than 20 .","solution":"Obviously, all functions in the answer satisfy the condition of the problem. We will show that there are no other functions satisfying that condition. Let $f$ be a function satisfying the given condition. For each integer $n$, the function $g$ defined by $g(x)=f(x)+n$ also satisfies the same condition. Therefore, by subtracting $f(0)$ from $f(x)$ we may assume that $f(0)=0$. For any prime $p$, the condition on $f$ with $(x, y)=(p, 0)$ states that $f(p)$ divides $p^{n}$. Since the set of primes is infinite, there exist integers $d$ and $\\varepsilon$ with $0 \\leq d \\leq n$ and $\\varepsilon \\in\\{1,-1\\}$ such that for infinitely many primes $p$ we have $f(p)=\\varepsilon p^{d}$. Denote the set of these primes by $P$. Since a function $g$ satisfies the given condition if and only if $-g$ satisfies the same condition, we may suppose $\\varepsilon=1$. The case $d=0$ is easily ruled out, because 0 does not divide any nonzero integer. Suppose $d \\geq 1$ and write $n$ as $m d+r$, where $m$ and $r$ are integers such that $m \\geq 1$ and $0 \\leq r \\leq d-1$. Let $x$ be an arbitrary integer. For each prime $p$ in $P$, the difference $f(p)-f(x)$ divides $p^{n}-x^{n}$. Using the equality $f(p)=p^{d}$, we get $$ p^{n}-x^{n}=p^{r}\\left(p^{d}\\right)^{m}-x^{n} \\equiv p^{r} f(x)^{m}-x^{n} \\equiv 0 \\quad\\left(\\bmod p^{d}-f(x)\\right) $$ Since we have $r8$. For each positive integer $n$, there exists an $i \\in\\{0,1, \\ldots, a-5\\}$ such that $n+i=2 d$ for some odd $d$. We get $$ t(n+i)=d \\not \\equiv d+2=t(n+i+4) \\quad(\\bmod 4) $$ and $$ t(n+a+i)=n+a+i \\equiv n+a+i+4=t(n+a+i+4) \\quad(\\bmod 4) $$ Therefore, the integers $t(n+a+i)-t(n+i)$ and $t(n+a+i+4)-t(n+i+4)$ cannot be both divisible by 4 , and therefore there are no winning pairs in this case. Case 3: $a=7$. For each positive integer $n$, there exists an $i \\in\\{0,1, \\ldots, 6\\}$ such that $n+i$ is either of the form $8 k+3$ or of the form $8 k+6$, where $k$ is a nonnegative integer. But we have $$ t(8 k+3) \\equiv 3 \\not \\equiv 1 \\equiv 4 k+5=t(8 k+3+7) \\quad(\\bmod 4) $$ and $$ t(8 k+6)=4 k+3 \\equiv 3 \\not \\equiv 1 \\equiv t(8 k+6+7) \\quad(\\bmod 4) . $$ Hence, there are no winning pairs of the form $(7, n)$.","tier":0} +{"problem_type":"Number Theory","problem_label":"N5","problem":"Let $f$ be a function from the set of integers to the set of positive integers. Suppose that for any two integers $m$ and $n$, the difference $f(m)-f(n)$ is divisible by $f(m-n)$. Prove that for all integers $m, n$ with $f(m) \\leq f(n)$ the number $f(n)$ is divisible by $f(m)$.","solution":"Suppose that $x$ and $y$ are two integers with $f(x)0 $$ so $f(x-y) \\leq f(y)-f(x)f(1)$. Note that such a number exists due to the symmetry of $f$ obtained in Claim 2. Claim 3. $f(n) \\neq f(1)$ if and only if $a \\mid n$. Proof. Since $f(1)=\\cdots=f(a-1)0$, so $f(n+a) \\leq$ $f(a)-f(n)b_{2}>\\cdots>b_{k}$ be all these values. One may show (essentially in the same way as in Claim 3) that the set $S_{i}=\\left\\{n: f(n) \\geq b_{i}\\right\\}$ consists exactly of all numbers divisible by some integer $a_{i} \\geq 0$. One obviously has $a_{i} \\mid a_{i-1}$, which implies $f\\left(a_{i}\\right) \\mid f\\left(a_{i-1}\\right)$ by Claim 1. So, $b_{k}\\left|b_{k-1}\\right| \\cdots \\mid b_{1}$, thus proving the problem statement. Moreover, now it is easy to describe all functions satisfying the conditions of the problem. Namely, all these functions can be constructed as follows. Consider a sequence of nonnegative integers $a_{1}, a_{2}, \\ldots, a_{k}$ and another sequence of positive integers $b_{1}, b_{2}, \\ldots, b_{k}$ such that $\\left|a_{k}\\right|=1, a_{i} \\neq a_{j}$ and $b_{i} \\neq b_{j}$ for all $1 \\leq i0$ this implies $M \\leq 3^{P(m+a x-b y)}-1$. But $P(m+a x-b y)$ is listed among $P(1), P(2), \\ldots, P(d)$, so $$ M<3^{P(m+a x-b y)} \\leq 3^{\\max \\{P(1), P(2), \\ldots, P(d)\\}} $$ which contradicts (1). Comment. We present another variant of the solution above. Denote the degree of $P$ by $k$ and its leading coefficient by $p$. Consider any positive integer $n$ and let $a=Q(n)$. Again, denote by $b$ the multiplicative order of 3 modulo $2^{a}-1$. Since $2^{a}-1 \\mid 3^{P(n)}-1$, we have $b \\mid P(n)$. Moreover, since $2^{Q(n+a t)}-1 \\mid 3^{P(n+a t)}-1$ and $a=Q(n) \\mid Q(n+a t)$ for each positive integer $t$, we have $2^{a}-1 \\mid 3^{P(n+a t)}-1$, hence $b \\mid P(n+a t)$ as well. Therefore, $b$ divides $\\operatorname{gcd}\\{P(n+a t): t \\geq 0\\}$; hence it also divides the number $$ \\sum_{i=0}^{k}(-1)^{k-i}\\left(\\begin{array}{c} k \\\\ i \\end{array}\\right) P(n+a i)=p \\cdot k ! \\cdot a^{k} $$ Finally, we get $b \\mid \\operatorname{gcd}\\left(P(n), k ! \\cdot p \\cdot Q(n)^{k}\\right)$, which is bounded by the same arguments as in the beginning of the solution. So $3^{b}-1$ is bounded, and hence $2^{Q(n)}-1$ is bounded as well.","tier":0} +{"problem_type":"Number Theory","problem_label":"N7","problem":"Let $p$ be an odd prime number. For every integer $a$, define the number $$ S_{a}=\\frac{a}{1}+\\frac{a^{2}}{2}+\\cdots+\\frac{a^{p-1}}{p-1} $$ Let $m$ and $n$ be integers such that $$ S_{3}+S_{4}-3 S_{2}=\\frac{m}{n} $$ Prove that $p$ divides $m$.","solution":"For rational numbers $p_{1} \/ q_{1}$ and $p_{2} \/ q_{2}$ with the denominators $q_{1}, q_{2}$ not divisible by $p$, we write $p_{1} \/ q_{1} \\equiv p_{2} \/ q_{2}(\\bmod p)$ if the numerator $p_{1} q_{2}-p_{2} q_{1}$ of their difference is divisible by $p$. We start with finding an explicit formula for the residue of $S_{a}$ modulo $p$. Note first that for every $k=1, \\ldots, p-1$ the number $\\left(\\begin{array}{l}p \\\\ k\\end{array}\\right)$ is divisible by $p$, and $$ \\frac{1}{p}\\left(\\begin{array}{l} p \\\\ k \\end{array}\\right)=\\frac{(p-1)(p-2) \\cdots(p-k+1)}{k !} \\equiv \\frac{(-1) \\cdot(-2) \\cdots(-k+1)}{k !}=\\frac{(-1)^{k-1}}{k} \\quad(\\bmod p) $$ Therefore, we have $$ S_{a}=-\\sum_{k=1}^{p-1} \\frac{(-a)^{k}(-1)^{k-1}}{k} \\equiv-\\sum_{k=1}^{p-1}(-a)^{k} \\cdot \\frac{1}{p}\\left(\\begin{array}{l} p \\\\ k \\end{array}\\right) \\quad(\\bmod p) $$ The number on the right-hand side is integer. Using the binomial formula we express it as $$ -\\sum_{k=1}^{p-1}(-a)^{k} \\cdot \\frac{1}{p}\\left(\\begin{array}{l} p \\\\ k \\end{array}\\right)=-\\frac{1}{p}\\left(-1-(-a)^{p}+\\sum_{k=0}^{p}(-a)^{k}\\left(\\begin{array}{l} p \\\\ k \\end{array}\\right)\\right)=\\frac{(a-1)^{p}-a^{p}+1}{p} $$ since $p$ is odd. So, we have $$ S_{a} \\equiv \\frac{(a-1)^{p}-a^{p}+1}{p} \\quad(\\bmod p) $$ Finally, using the obtained formula we get $$ \\begin{aligned} S_{3}+S_{4}-3 S_{2} & \\equiv \\frac{\\left(2^{p}-3^{p}+1\\right)+\\left(3^{p}-4^{p}+1\\right)-3\\left(1^{p}-2^{p}+1\\right)}{p} \\\\ & =\\frac{4 \\cdot 2^{p}-4^{p}-4}{p}=-\\frac{\\left(2^{p}-2\\right)^{2}}{p} \\quad(\\bmod p) . \\end{aligned} $$ By Fermat's theorem, $p \\mid 2^{p}-2$, so $p^{2} \\mid\\left(2^{p}-2\\right)^{2}$ and hence $S_{3}+S_{4}-3 S_{2} \\equiv 0(\\bmod p)$.","tier":0} +{"problem_type":"Number Theory","problem_label":"N7","problem":"Let $p$ be an odd prime number. For every integer $a$, define the number $$ S_{a}=\\frac{a}{1}+\\frac{a^{2}}{2}+\\cdots+\\frac{a^{p-1}}{p-1} $$ Let $m$ and $n$ be integers such that $$ S_{3}+S_{4}-3 S_{2}=\\frac{m}{n} $$ Prove that $p$ divides $m$.","solution":"One may solve the problem without finding an explicit formula for $S_{a}$. It is enough to find the following property. Lemma. For every integer $a$, we have $S_{a+1} \\equiv S_{-a}(\\bmod p)$. Proof. We expand $S_{a+1}$ using the binomial formula as $$ S_{a+1}=\\sum_{k=1}^{p-1} \\frac{1}{k} \\sum_{j=0}^{k}\\left(\\begin{array}{l} k \\\\ j \\end{array}\\right) a^{j}=\\sum_{k=1}^{p-1}\\left(\\frac{1}{k}+\\sum_{j=1}^{k} a^{j} \\cdot \\frac{1}{k}\\left(\\begin{array}{l} k \\\\ j \\end{array}\\right)\\right)=\\sum_{k=1}^{p-1} \\frac{1}{k}+\\sum_{j=1}^{p-1} a^{j} \\sum_{k=j}^{p-1} \\frac{1}{k}\\left(\\begin{array}{l} k \\\\ j \\end{array}\\right) a^{k} . $$ Note that $\\frac{1}{k}+\\frac{1}{p-k}=\\frac{p}{k(p-k)} \\equiv 0(\\bmod p)$ for all $1 \\leq k \\leq p-1$; hence the first sum vanishes modulo $p$. For the second sum, we use the relation $\\frac{1}{k}\\left(\\begin{array}{c}k \\\\ j\\end{array}\\right)=\\frac{1}{j}\\left(\\begin{array}{c}k-1 \\\\ j-1\\end{array}\\right)$ to obtain $$ S_{a+1} \\equiv \\sum_{j=1}^{p-1} \\frac{a^{j}}{j} \\sum_{k=1}^{p-1}\\left(\\begin{array}{c} k-1 \\\\ j-1 \\end{array}\\right) \\quad(\\bmod p) $$ Finally, from the relation $$ \\sum_{k=1}^{p-1}\\left(\\begin{array}{l} k-1 \\\\ j-1 \\end{array}\\right)=\\left(\\begin{array}{c} p-1 \\\\ j \\end{array}\\right)=\\frac{(p-1)(p-2) \\ldots(p-j)}{j !} \\equiv(-1)^{j} \\quad(\\bmod p) $$ we obtain $$ S_{a+1} \\equiv \\sum_{j=1}^{p-1} \\frac{a^{j}(-1)^{j}}{j !}=S_{-a} $$ Now we turn to the problem. Using the lemma we get $$ S_{3}-3 S_{2} \\equiv S_{-2}-3 S_{2}=\\sum_{\\substack{1 \\leq k \\leq p-1 \\\\ k \\text { is even }}} \\frac{-2 \\cdot 2^{k}}{k}+\\sum_{\\substack{1 \\leq k \\leq p-1 \\\\ k \\text { is odd }}} \\frac{-4 \\cdot 2^{k}}{k}(\\bmod p) . $$ The first sum in (11) expands as $$ \\sum_{\\ell=1}^{(p-1) \/ 2} \\frac{-2 \\cdot 2^{2 \\ell}}{2 \\ell}=-\\sum_{\\ell=1}^{(p-1) \/ 2} \\frac{4^{\\ell}}{\\ell} $$ Next, using Fermat's theorem, we expand the second sum in (11) as $$ -\\sum_{\\ell=1}^{(p-1) \/ 2} \\frac{2^{2 \\ell+1}}{2 \\ell-1} \\equiv-\\sum_{\\ell=1}^{(p-1) \/ 2} \\frac{2^{p+2 \\ell}}{p+2 \\ell-1}=-\\sum_{m=(p+1) \/ 2}^{p-1} \\frac{2 \\cdot 4^{m}}{2 m}=-\\sum_{m=(p+1) \/ 2}^{p-1} \\frac{4^{m}}{m}(\\bmod p) $$ (here we set $m=\\ell+\\frac{p-1}{2}$ ). Hence, $$ S_{3}-3 S_{2} \\equiv-\\sum_{\\ell=1}^{(p-1) \/ 2} \\frac{4^{\\ell}}{\\ell}-\\sum_{m=(p+1) \/ 2}^{p-1} \\frac{4^{m}}{m}=-S_{4} \\quad(\\bmod p) $$ Let $k$ be a positive integer and set $n=2^{k}+1$. Prove that $n$ is a prime number if and only if the following holds: there is a permutation $a_{1}, \\ldots, a_{n-1}$ of the numbers $1,2, \\ldots, n-1$ and a sequence of integers $g_{1}, g_{2}, \\ldots, g_{n-1}$ such that $n$ divides $g_{i}^{a_{i}}-a_{i+1}$ for every $i \\in\\{1,2, \\ldots, n-1\\}$, where we set $a_{n}=a_{1}$.","tier":0} +{"problem_type":"Number Theory","problem_label":"N7","problem":"Let $p$ be an odd prime number. For every integer $a$, define the number $$ S_{a}=\\frac{a}{1}+\\frac{a^{2}}{2}+\\cdots+\\frac{a^{p-1}}{p-1} $$ Let $m$ and $n$ be integers such that $$ S_{3}+S_{4}-3 S_{2}=\\frac{m}{n} $$ Prove that $p$ divides $m$.","solution":"Let $N=\\{1,2, \\ldots, n-1\\}$. For $a, b \\in N$, we say that $b$ follows $a$ if there exists an integer $g$ such that $b \\equiv g^{a}(\\bmod n)$ and denote this property as $a \\rightarrow b$. This way we have a directed graph with $N$ as set of vertices. If $a_{1}, \\ldots, a_{n-1}$ is a permutation of $1,2, \\ldots, n-1$ such that $a_{1} \\rightarrow a_{2} \\rightarrow \\ldots \\rightarrow a_{n-1} \\rightarrow a_{1}$ then this is a Hamiltonian cycle in the graph. Step I. First consider the case when $n$ is composite. Let $n=p_{1}^{\\alpha_{1}} \\ldots p_{s}^{\\alpha_{s}}$ be its prime factorization. All primes $p_{i}$ are odd. Suppose that $\\alpha_{i}>1$ for some $i$. For all integers $a, g$ with $a \\geq 2$, we have $g^{a} \\not \\equiv p_{i}\\left(\\bmod p_{i}^{2}\\right)$, because $g^{a}$ is either divisible by $p_{i}^{2}$ or it is not divisible by $p_{i}$. It follows that in any Hamiltonian cycle $p_{i}$ comes immediately after 1 . The same argument shows that $2 p_{i}$ also should come immediately after 1, which is impossible. Hence, there is no Hamiltonian cycle in the graph. Now suppose that $n$ is square-free. We have $n=p_{1} p_{2} \\ldots p_{s}>9$ and $s \\geq 2$. Assume that there exists a Hamiltonian cycle. There are $\\frac{n-1}{2}$ even numbers in this cycle, and each number which follows one of them should be a quadratic residue modulo $n$. So, there should be at least $\\frac{n-1}{2}$ nonzero quadratic residues modulo $n$. On the other hand, for each $p_{i}$ there exist exactly $\\frac{p_{i}+1}{2}$ quadratic residues modulo $p_{i}$; by the Chinese Remainder Theorem, the number of quadratic residues modulo $n$ is exactly $\\frac{p_{1}+1}{2} \\cdot \\frac{p_{2}+1}{2} \\cdot \\ldots \\cdot \\frac{p_{s}+1}{2}$, including 0 . Then we have a contradiction by $$ \\frac{p_{1}+1}{2} \\cdot \\frac{p_{2}+1}{2} \\cdot \\ldots \\cdot \\frac{p_{s}+1}{2} \\leq \\frac{2 p_{1}}{3} \\cdot \\frac{2 p_{2}}{3} \\cdot \\ldots \\cdot \\frac{2 p_{s}}{3}=\\left(\\frac{2}{3}\\right)^{s} n \\leq \\frac{4 n}{9}<\\frac{n-1}{2} $$ This proves the \"if\"-part of the problem. Step II. Now suppose that $n$ is prime. For any $a \\in N$, denote by $\\nu_{2}(a)$ the exponent of 2 in the prime factorization of $a$, and let $\\mu(a)=\\max \\left\\{t \\in[0, k] \\mid 2^{t} \\rightarrow a\\right\\}$. Lemma. For any $a, b \\in N$, we have $a \\rightarrow b$ if and only if $\\nu_{2}(a) \\leq \\mu(b)$. Proof. Let $\\ell=\\nu_{2}(a)$ and $m=\\mu(b)$. Suppose $\\ell \\leq m$. Since $b$ follows $2^{m}$, there exists some $g_{0}$ such that $b \\equiv g_{0}^{2^{m}}(\\bmod n)$. By $\\operatorname{gcd}(a, n-1)=2^{\\ell}$ there exist some integers $p$ and $q$ such that $p a-q(n-1)=2^{\\ell}$. Choosing $g=g_{0}^{2^{m-\\ell} p}$ we have $g^{a}=g_{0}^{2^{m-\\ell} p a}=g_{0}^{2^{m}+2^{m-\\ell} q(n-1)} \\equiv g_{0}^{2^{m}} \\equiv b(\\bmod n)$ by FERMAT's theorem. Hence, $a \\rightarrow b$. To prove the reverse statement, suppose that $a \\rightarrow b$, so $b \\equiv g^{a}(\\bmod n)$ with some $g$. Then $b \\equiv\\left(g^{a \/ 2^{\\ell}}\\right)^{2^{\\ell}}$, and therefore $2^{\\ell} \\rightarrow b$. By the definition of $\\mu(b)$, we have $\\mu(b) \\geq \\ell$. The lemma is proved. Now for every $i$ with $0 \\leq i \\leq k$, let $$ \\begin{aligned} A_{i} & =\\left\\{a \\in N \\mid \\nu_{2}(a)=i\\right\\}, \\\\ B_{i} & =\\{a \\in N \\mid \\mu(a)=i\\}, \\\\ \\text { and } C_{i} & =\\{a \\in N \\mid \\mu(a) \\geq i\\}=B_{i} \\cup B_{i+1} \\cup \\ldots \\cup B_{k} . \\end{aligned} $$ We claim that $\\left|A_{i}\\right|=\\left|B_{i}\\right|$ for all $0 \\leq i \\leq k$. Obviously we have $\\left|A_{i}\\right|=2^{k-i-1}$ for all $i=$ $0, \\ldots, k-1$, and $\\left|A_{k}\\right|=1$. Now we determine $\\left|C_{i}\\right|$. We have $\\left|C_{0}\\right|=n-1$ and by Fermat's theorem we also have $C_{k}=\\{1\\}$, so $\\left|C_{k}\\right|=1$. Next, notice that $C_{i+1}=\\left\\{x^{2} \\bmod n \\mid x \\in C_{i}\\right\\}$. For every $a \\in N$, the relation $x^{2} \\equiv a(\\bmod n)$ has at most two solutions in $N$. Therefore we have $2\\left|C_{i+1}\\right| \\leq\\left|C_{i}\\right|$, with the equality achieved only if for every $y \\in C_{i+1}$, there exist distinct elements $x, x^{\\prime} \\in C_{i}$ such that $x^{2} \\equiv x^{\\prime 2} \\equiv y(\\bmod n)$ (this implies $\\left.x+x^{\\prime}=n\\right)$. Now, since $2^{k}\\left|C_{k}\\right|=\\left|C_{0}\\right|$, we obtain that this equality should be achieved in each step. Hence $\\left|C_{i}\\right|=2^{k-i}$ for $0 \\leq i \\leq k$, and therefore $\\left|B_{i}\\right|=2^{k-i-1}$ for $0 \\leq i \\leq k-1$ and $\\left|B_{k}\\right|=1$. From the previous arguments we can see that for each $z \\in C_{i}(0 \\leq i0$. Finally, if $\\lambda=k-1$, then $C$ contains $2^{k-1}$ which is the only element of $A_{k-1}$. Since $B_{k-1}=\\left\\{2^{k}\\right\\}=A_{k}$ and $B_{k}=\\{1\\}$, the cycle $C$ contains the path $2^{k-1} \\rightarrow 2^{k} \\rightarrow 1$ and it contains an odd number again. This completes the proof of the \"only if\"-part of the problem. Comment 1. The lemma and the fact $\\left|A_{i}\\right|=\\left|B_{i}\\right|$ together show that for every edge $a \\rightarrow b$ of the Hamiltonian cycle, $\\nu_{2}(a)=\\mu(b)$ must hold. After this observation, the Hamiltonian cycle can be built in many ways. For instance, it is possible to select edges from $A_{i}$ to $B_{i}$ for $i=k, k-1, \\ldots, 1$ in such a way that they form disjoint paths; at the end all these paths will have odd endpoints. In the final step, the paths can be closed to form a unique cycle. Comment 2. Step II is an easy consequence of some basic facts about the multiplicative group modulo the prime $n=2^{k}+1$. The Lemma follows by noting that this group has order $2^{k}$, so the $a$-th powers are exactly the $2^{\\nu_{2}(a)}$-th powers. Using the existence of a primitive root $g$ modulo $n$ one sees that the map from $\\{1,2, \\ldots, n-1\\}$ to itself that sends $a$ to $g^{a} \\bmod n$ is a bijection that sends $A_{i}$ to $B_{i}$ for each $i \\in\\{0, \\ldots, k\\}$.","tier":0} +{"problem_type":"Number Theory","problem_label":"N8","problem":"Let $k$ be a positive integer and set $n=2^{k}+1$. Prove that $n$ is a prime number if and only if the following holds: there is a permutation $a_{1}, \\ldots, a_{n-1}$ of the numbers $1,2, \\ldots, n-1$ and a sequence of integers $g_{1}, g_{2}, \\ldots, g_{n-1}$ such that $n$ divides $g_{i}^{a_{i}}-a_{i+1}$ for every $i \\in\\{1,2, \\ldots, n-1\\}$, where we set $a_{n}=a_{1}$.","solution":null,"tier":0} +{"problem_type":"Number Theory","problem_label":"A1","problem":"For any set $A=\\left\\{a_{1}, a_{2}, a_{3}, a_{4}\\right\\}$ of four distinct positive integers with sum $s_{A}=a_{1}+a_{2}+a_{3}+a_{4}$, let $p_{A}$ denote the number of pairs $(i, j)$ with $1 \\leq ia_{1}+a_{3}$ and $a_{3}+a_{4}>a_{1}+a_{2}$. Hence $a_{2}+a_{4}$ and $a_{3}+a_{4}$ do not divide $s_{A}$. This proves $p_{A} \\leq 4$. Now suppose $p_{A}=4$. By the previous argument we have $$ \\begin{array}{lll} a_{1}+a_{4} \\mid a_{2}+a_{3} & \\text { and } & a_{2}+a_{3} \\mid a_{1}+a_{4}, \\\\ a_{1}+a_{2} \\mid a_{3}+a_{4} & \\text { and } & a_{3}+a_{4} \\not a_{1}+a_{2}, \\\\ a_{1}+a_{3} \\mid a_{2}+a_{4} & \\text { and } & a_{2}+a_{4} \\not a_{1}+a_{3} . \\end{array} $$ Hence, there exist positive integers $m$ and $n$ with $m>n \\geq 2$ such that $$ \\left\\{\\begin{array}{l} a_{1}+a_{4}=a_{2}+a_{3} \\\\ m\\left(a_{1}+a_{2}\\right)=a_{3}+a_{4} \\\\ n\\left(a_{1}+a_{3}\\right)=a_{2}+a_{4} \\end{array}\\right. $$ Adding up the first equation and the third one, we get $n\\left(a_{1}+a_{3}\\right)=2 a_{2}+a_{3}-a_{1}$. If $n \\geq 3$, then $n\\left(a_{1}+a_{3}\\right)>3 a_{3}>2 a_{2}+a_{3}>2 a_{2}+a_{3}-a_{1}$. This is a contradiction. Therefore $n=2$. If we multiply by 2 the sum of the first equation and the third one, we obtain $$ 6 a_{1}+2 a_{3}=4 a_{2} $$ while the sum of the first one and the second one is $$ (m+1) a_{1}+(m-1) a_{2}=2 a_{3} . $$ Adding up the last two equations we get $$ (m+7) a_{1}=(5-m) a_{2} . $$ It follows that $5-m \\geq 1$, because the left-hand side of the last equation and $a_{2}$ are positive. Since we have $m>n=2$, the integer $m$ can be equal only to either 3 or 4 . Substituting $(3,2)$ and $(4,2)$ for $(m, n)$ and solving the previous system of equations, we find the families of solutions $\\{d, 5 d, 7 d, 11 d\\}$ and $\\{d, 11 d, 19 d, 29 d\\}$, where $d$ is any positive integer.","tier":0} +{"problem_type":"Number Theory","problem_label":"A2","problem":"Determine all sequences $\\left(x_{1}, x_{2}, \\ldots, x_{2011}\\right)$ of positive integers such that for every positive integer $n$ there is an integer $a$ with $$ x_{1}^{n}+2 x_{2}^{n}+\\cdots+2011 x_{2011}^{n}=a^{n+1}+1 . $$ Answer. The only sequence that satisfies the condition is $$ \\left(x_{1}, \\ldots, x_{2011}\\right)=(1, k, \\ldots, k) \\quad \\text { with } k=2+3+\\cdots+2011=2023065 $$","solution":"Throughout this solution, the set of positive integers will be denoted by $\\mathbb{Z}_{+}$. Put $k=2+3+\\cdots+2011=2023065$. We have $$ 1^{n}+2 k^{n}+\\cdots 2011 k^{n}=1+k \\cdot k^{n}=k^{n+1}+1 $$ for all $n$, so $(1, k, \\ldots, k)$ is a valid sequence. We shall prove that it is the only one. Let a valid sequence $\\left(x_{1}, \\ldots, x_{2011}\\right)$ be given. For each $n \\in \\mathbb{Z}_{+}$we have some $y_{n} \\in \\mathbb{Z}_{+}$with $$ x_{1}^{n}+2 x_{2}^{n}+\\cdots+2011 x_{2011}^{n}=y_{n}^{n+1}+1 . $$ Note that $x_{1}^{n}+2 x_{2}^{n}+\\cdots+2011 x_{2011}^{n}<\\left(x_{1}+2 x_{2}+\\cdots+2011 x_{2011}\\right)^{n+1}$, which implies that the sequence $\\left(y_{n}\\right)$ is bounded. In particular, there is some $y \\in \\mathbb{Z}_{+}$with $y_{n}=y$ for infinitely many $n$. Let $m$ be the maximum of all the $x_{i}$. Grouping terms with equal $x_{i}$ together, the sum $x_{1}^{n}+$ $2 x_{2}^{n}+\\cdots+2011 x_{2011}^{n}$ can be written as $$ x_{1}^{n}+2 x_{2}^{n}+\\cdots+x_{2011}^{n}=a_{m} m^{n}+a_{m-1}(m-1)^{n}+\\cdots+a_{1} $$ with $a_{i} \\geq 0$ for all $i$ and $a_{1}+\\cdots+a_{m}=1+2+\\cdots+2011$. So there exist arbitrarily large values of $n$, for which $$ a_{m} m^{n}+\\cdots+a_{1}-1-y \\cdot y^{n}=0 \\text {. } $$ The following lemma will help us to determine the $a_{i}$ and $y$ : Lemma. Let integers $b_{1}, \\ldots, b_{N}$ be given and assume that there are arbitrarily large positive integers $n$ with $b_{1}+b_{2} 2^{n}+\\cdots+b_{N} N^{n}=0$. Then $b_{i}=0$ for all $i$. Proof. Suppose that not all $b_{i}$ are zero. We may assume without loss of generality that $b_{N} \\neq 0$. Dividing through by $N^{n}$ gives $$ \\left|b_{N}\\right|=\\left|b_{N-1}\\left(\\frac{N-1}{N}\\right)^{n}+\\cdots+b_{1}\\left(\\frac{1}{N}\\right)^{n}\\right| \\leq\\left(\\left|b_{N-1}\\right|+\\cdots+\\left|b_{1}\\right|\\right)\\left(\\frac{N-1}{N}\\right)^{n} . $$ The expression $\\left(\\frac{N-1}{N}\\right)^{n}$ can be made arbitrarily small for $n$ large enough, contradicting the assumption that $b_{N}$ be non-zero. We obviously have $y>1$. Applying the lemma to (1) we see that $a_{m}=y=m, a_{1}=1$, and all the other $a_{i}$ are zero. This implies $\\left(x_{1}, \\ldots, x_{2011}\\right)=(1, m, \\ldots, m)$. But we also have $1+m=a_{1}+\\cdots+a_{m}=1+\\cdots+2011=1+k$ so $m=k$, which is what we wanted to show.","tier":0} +{"problem_type":"Number Theory","problem_label":"A3","problem":"Determine all pairs $(f, g)$ of functions from the set of real numbers to itself that satisfy $$ g(f(x+y))=f(x)+(2 x+y) g(y) $$ for all real numbers $x$ and $y$. Answer. Either both $f$ and $g$ vanish identically, or there exists a real number $C$ such that $f(x)=x^{2}+C$ and $g(x)=x$ for all real numbers $x$.","solution":"Clearly all these pairs of functions satisfy the functional equation in question, so it suffices to verify that there cannot be any further ones. Substituting $-2 x$ for $y$ in the given functional equation we obtain $$ g(f(-x))=f(x) . $$ Using this equation for $-x-y$ in place of $x$ we obtain $$ f(-x-y)=g(f(x+y))=f(x)+(2 x+y) g(y) . $$ Now for any two real numbers $a$ and $b$, setting $x=-b$ and $y=a+b$ we get $$ f(-a)=f(-b)+(a-b) g(a+b) . $$ If $c$ denotes another arbitrary real number we have similarly $$ f(-b)=f(-c)+(b-c) g(b+c) $$ as well as $$ f(-c)=f(-a)+(c-a) g(c+a) . $$ Adding all these equations up, we obtain $$ ((a+c)-(b+c)) g(a+b)+((a+b)-(a+c)) g(b+c)+((b+c)-(a+b)) g(a+c)=0 \\text {. } $$ Now given any three real numbers $x, y$, and $z$ one may determine three reals $a, b$, and $c$ such that $x=b+c, y=c+a$, and $z=a+b$, so that we get $$ (y-x) g(z)+(z-y) g(x)+(x-z) g(y)=0 . $$ This implies that the three points $(x, g(x)),(y, g(y))$, and $(z, g(z))$ from the graph of $g$ are collinear. Hence that graph is a line, i.e., $g$ is either a constant or a linear function. Let us write $g(x)=A x+B$, where $A$ and $B$ are two real numbers. Substituting $(0,-y)$ for $(x, y)$ in (2) and denoting $C=f(0)$, we have $f(y)=A y^{2}-B y+C$. Now, comparing the coefficients of $x^{2}$ in (1I) we see that $A^{2}=A$, so $A=0$ or $A=1$. If $A=0$, then (1) becomes $B=-B x+C$ and thus $B=C=0$, which provides the first of the two solutions mentioned above. Now suppose $A=1$. Then (1) becomes $x^{2}-B x+C+B=x^{2}-B x+C$, so $B=0$. Thus, $g(x)=x$ and $f(x)=x^{2}+C$, which is the second solution from above. Comment. Another way to show that $g(x)$ is either a constant or a linear function is the following. If we interchange $x$ and $y$ in the given functional equation and subtract this new equation from the given one, we obtain $$ f(x)-f(y)=(2 y+x) g(x)-(2 x+y) g(y) . $$ Substituting $(x, 0),(1, x)$, and $(0,1)$ for $(x, y)$, we get $$ \\begin{aligned} & f(x)-f(0)=x g(x)-2 x g(0), \\\\ & f(1)-f(x)=(2 x+1) g(1)-(x+2) g(x), \\\\ & f(0)-f(1)=2 g(0)-g(1) . \\end{aligned} $$ Taking the sum of these three equations and dividing by 2 , we obtain $$ g(x)=x(g(1)-g(0))+g(0) . $$ This proves that $g(x)$ is either a constant of a linear function.","tier":0} +{"problem_type":"Number Theory","problem_label":"A4","problem":"Determine all pairs $(f, g)$ of functions from the set of positive integers to itself that satisfy $$ f^{g(n)+1}(n)+g^{f(n)}(n)=f(n+1)-g(n+1)+1 $$ for every positive integer $n$. Here, $f^{k}(n)$ means $\\underbrace{f(f(\\ldots f}_{k}(n) \\ldots))$. Answer. The only pair $(f, g)$ of functions that satisfies the equation is given by $f(n)=n$ and $g(n)=1$ for all $n$.","solution":"The given relation implies $$ f\\left(f^{g(n)}(n)\\right)1$, then (1) reads $f\\left(f^{g(x-1)}(x-1)\\right)n$. Substituting $x-1$ into (1) we have $f\\left(f^{g(x-1)}(x-1)\\right)n$. So $b=n$, and hence $y_{n}=n$, which proves (ii) ${ }_{n}$. Next, from (i) ${ }_{n}$ we now get $f(k)=n \\Longleftrightarrow k=n$, so removing all the iterations of $f$ in (3) we obtain $x-1=b=n$, which proves (i) ${ }_{n+1}$. So, all the statements in (2) are valid and hence $f(n)=n$ for all $n$. The given relation between $f$ and $g$ now reads $n+g^{n}(n)=n+1-g(n+1)+1$ or $g^{n}(n)+g(n+1)=2$, from which it immediately follows that we have $g(n)=1$ for all $n$. Comment. Several variations of the above solution are possible. For instance, one may first prove by induction that the smallest $n$ values of $f$ are exactly $f(1)<\\cdotsn$, then $f(x)>x$ for all $x \\geq n$. From this we conclude $f^{g(n)+1}(n)>f^{g(n)}(n)>\\cdots>f(n)$. But we also have $f^{g(n)+1}\\sqrt{2}$ and $a^{2}+b^{2}+c^{2}=3$. Prove that $$ \\frac{a}{(b+c-a)^{2}}+\\frac{b}{(c+a-b)^{2}}+\\frac{c}{(a+b-c)^{2}} \\geq \\frac{3}{(a b c)^{2}} $$ Throughout both solutions, we denote the sums of the form $f(a, b, c)+f(b, c, a)+f(c, a, b)$ by $\\sum f(a, b, c)$.","solution":"The condition $b+c>\\sqrt{2}$ implies $b^{2}+c^{2}>1$, so $a^{2}=3-\\left(b^{2}+c^{2}\\right)<2$, i.e. $a<\\sqrt{2}0$, and also $c+a-b>0$ and $a+b-c>0$ for similar reasons. We will use the variant of H\u00d6LDER's inequality $$ \\frac{x_{1}^{p+1}}{y_{1}^{p}}+\\frac{x_{1}^{p+1}}{y_{1}^{p}}+\\ldots+\\frac{x_{n}^{p+1}}{y_{n}^{p}} \\geq \\frac{\\left(x_{1}+x_{2}+\\ldots+x_{n}\\right)^{p+1}}{\\left(y_{1}+y_{2}+\\ldots+y_{n}\\right)^{p}} $$ which holds for all positive real numbers $p, x_{1}, x_{2}, \\ldots, x_{n}, y_{1}, y_{2}, \\ldots, y_{n}$. Applying it to the left-hand side of (11) with $p=2$ and $n=3$, we get $$ \\sum \\frac{a}{(b+c-a)^{2}}=\\sum \\frac{\\left(a^{2}\\right)^{3}}{a^{5}(b+c-a)^{2}} \\geq \\frac{\\left(a^{2}+b^{2}+c^{2}\\right)^{3}}{\\left(\\sum a^{5 \/ 2}(b+c-a)\\right)^{2}}=\\frac{27}{\\left(\\sum a^{5 \/ 2}(b+c-a)\\right)^{2}} . $$ To estimate the denominator of the right-hand part, we use an instance of SCHUR's inequality, namely $$ \\sum a^{3 \/ 2}(a-b)(a-c) \\geq 0 $$ which can be rewritten as $$ \\sum a^{5 \/ 2}(b+c-a) \\leq a b c(\\sqrt{a}+\\sqrt{b}+\\sqrt{c}) . $$ Moreover, by the inequality between the arithmetic mean and the fourth power mean we also have $$ \\left(\\frac{\\sqrt{a}+\\sqrt{b}+\\sqrt{c}}{3}\\right)^{4} \\leq \\frac{a^{2}+b^{2}+c^{2}}{3}=1 $$ i.e., $\\sqrt{a}+\\sqrt{b}+\\sqrt{c} \\leq 3$. Hence, (2) yields $$ \\sum \\frac{a}{(b+c-a)^{2}} \\geq \\frac{27}{(a b c(\\sqrt{a}+\\sqrt{b}+\\sqrt{c}))^{2}} \\geq \\frac{3}{a^{2} b^{2} c^{2}} $$ thus solving the problem. Comment. In this solution, one may also start from the following version of H\u00d6LDER's inequality $$ \\left(\\sum_{i=1}^{n} a_{i}^{3}\\right)\\left(\\sum_{i=1}^{n} b_{i}^{3}\\right)\\left(\\sum_{i=1}^{n} c_{i}^{3}\\right) \\geq\\left(\\sum_{i=1}^{n} a_{i} b_{i} c_{i}\\right)^{3} $$ applied as $$ \\sum \\frac{a}{(b+c-a)^{2}} \\cdot \\sum a^{3}(b+c-a) \\cdot \\sum a^{2}(b+c-a) \\geq 27 $$ After doing that, one only needs the slightly better known instances $$ \\sum a^{3}(b+c-a) \\leq(a+b+c) a b c \\quad \\text { and } \\quad \\sum a^{2}(b+c-a) \\leq 3 a b c $$ of Schur's Inequality.","tier":0} +{"problem_type":"Number Theory","problem_label":"A7","problem":"Let $a, b$, and $c$ be positive real numbers satisfying $\\min (a+b, b+c, c+a)>\\sqrt{2}$ and $a^{2}+b^{2}+c^{2}=3$. Prove that $$ \\frac{a}{(b+c-a)^{2}}+\\frac{b}{(c+a-b)^{2}}+\\frac{c}{(a+b-c)^{2}} \\geq \\frac{3}{(a b c)^{2}} $$ Throughout both solutions, we denote the sums of the form $f(a, b, c)+f(b, c, a)+f(c, a, b)$ by $\\sum f(a, b, c)$.","solution":"As in $$ a^{5}+b^{5}+c^{5} \\geq 3 $$ which is weaker than the given one. Due to the symmetry we may assume that $a \\geq b \\geq c$. In view of (3)), it suffices to prove the inequality $$ \\sum \\frac{a^{3} b^{2} c^{2}}{(b+c-a)^{2}} \\geq \\sum a^{5} $$ or, moving all the terms into the left-hand part, $$ \\sum \\frac{a^{3}}{(b+c-a)^{2}}\\left((b c)^{2}-(a(b+c-a))^{2}\\right) \\geq 0 $$ Note that the signs of the expressions $(y z)^{2}-(x(y+z-x))^{2}$ and $y z-x(y+z-x)=(x-y)(x-z)$ are the same for every positive $x, y, z$ satisfying the triangle inequality. So the terms in (4) corresponding to $a$ and $c$ are nonnegative, and hence it is sufficient to prove that the sum of the terms corresponding to $a$ and $b$ is nonnegative. Equivalently, we need the relation $$ \\frac{a^{3}}{(b+c-a)^{2}}(a-b)(a-c)(b c+a(b+c-a)) \\geq \\frac{b^{3}}{(a+c-b)^{2}}(a-b)(b-c)(a c+b(a+c-b)) $$ Obviously, we have $$ a^{3} \\geq b^{3} \\geq 0, \\quad 0(a-b)^{2} $$ which holds since $c>a-b \\geq 0$ and $a+b>a-b \\geq 0$.","tier":0} +{"problem_type":"Number Theory","problem_label":"C1","problem":"Let $n>0$ be an integer. We are given a balance and $n$ weights of weight $2^{0}, 2^{1}, \\ldots, 2^{n-1}$. In a sequence of $n$ moves we place all weights on the balance. In the first move we choose a weight and put it on the left pan. In each of the following moves we choose one of the remaining weights and we add it either to the left or to the right pan. Compute the number of ways in which we can perform these $n$ moves in such a way that the right pan is never heavier than the left pan. Answer. The number $f(n)$ of ways of placing the $n$ weights is equal to the product of all odd positive integers less than or equal to $2 n-1$, i.e. $f(n)=(2 n-1) ! !=1 \\cdot 3 \\cdot 5 \\cdot \\ldots \\cdot(2 n-1)$.","solution":"Assume $n \\geq 2$. We claim $$ f(n)=(2 n-1) f(n-1) . $$ Firstly, note that after the first move the left pan is always at least 1 heavier than the right one. Hence, any valid way of placing the $n$ weights on the scale gives rise, by not considering weight 1 , to a valid way of placing the weights $2,2^{2}, \\ldots, 2^{n-1}$. If we divide the weight of each weight by 2 , the answer does not change. So these $n-1$ weights can be placed on the scale in $f(n-1)$ valid ways. Now we look at weight 1 . If it is put on the scale in the first move, then it has to be placed on the left side, otherwise it can be placed either on the left or on the right side, because after the first move the difference between the weights on the left pan and the weights on the right pan is at least 2 . Hence, there are exactly $2 n-1$ different ways of inserting weight 1 in each of the $f(n-1)$ valid sequences for the $n-1$ weights in order to get a valid sequence for the $n$ weights. This proves the claim. Since $f(1)=1$, by induction we obtain for all positive integers $n$ $$ f(n)=(2 n-1) ! !=1 \\cdot 3 \\cdot 5 \\cdot \\ldots \\cdot(2 n-1) $$ Comment 1. The word \"compute\" in the statement of the problem is probably too vague. An alternative but more artificial question might ask for the smallest $n$ for which the number of valid ways is divisible by 2011. In this case the answer would be 1006 . Comment 2. It is useful to remark that the answer is the same for any set of weights where each weight is heavier than the sum of the lighter ones. Indeed, in such cases the given condition is equivalent to asking that during the process the heaviest weight on the balance is always on the left pan. Comment 3. Instead of considering the lightest weight, one may also consider the last weight put on the balance. If this weight is $2^{n-1}$ then it should be put on the left pan. Otherwise it may be put on any pan; the inequality would not be violated since at this moment the heaviest weight is already put onto the left pan. In view of the previous comment, in each of these $2 n-1$ cases the number of ways to place the previous weights is exactly $f(n-1)$, which yields (1).","tier":0} +{"problem_type":"Number Theory","problem_label":"C1","problem":"Let $n>0$ be an integer. We are given a balance and $n$ weights of weight $2^{0}, 2^{1}, \\ldots, 2^{n-1}$. In a sequence of $n$ moves we place all weights on the balance. In the first move we choose a weight and put it on the left pan. In each of the following moves we choose one of the remaining weights and we add it either to the left or to the right pan. Compute the number of ways in which we can perform these $n$ moves in such a way that the right pan is never heavier than the left pan. Answer. The number $f(n)$ of ways of placing the $n$ weights is equal to the product of all odd positive integers less than or equal to $2 n-1$, i.e. $f(n)=(2 n-1) ! !=1 \\cdot 3 \\cdot 5 \\cdot \\ldots \\cdot(2 n-1)$.","solution":"We present a different way of obtaining (1). Set $f(0)=1$. Firstly, we find a recurrent formula for $f(n)$. Assume $n \\geq 1$. Suppose that weight $2^{n-1}$ is placed on the balance in the $i$-th move with $1 \\leq i \\leq n$. This weight has to be put on the left pan. For the previous moves we have $\\left(\\begin{array}{c}n-1 \\\\ i-1\\end{array}\\right)$ choices of the weights and from Comment 2 there are $f(i-1)$ valid ways of placing them on the balance. For later moves there is no restriction on the way in which the weights are to be put on the pans. Therefore, all $(n-i) ! 2^{n-i}$ ways are possible. This gives $$ f(n)=\\sum_{i=1}^{n}\\left(\\begin{array}{c} n-1 \\\\ i-1 \\end{array}\\right) f(i-1)(n-i) ! 2^{n-i}=\\sum_{i=1}^{n} \\frac{(n-1) ! f(i-1) 2^{n-i}}{(i-1) !} $$ Now we are ready to prove (11). Using $n-1$ instead of $n$ in (2) we get $$ f(n-1)=\\sum_{i=1}^{n-1} \\frac{(n-2) ! f(i-1) 2^{n-1-i}}{(i-1) !} . $$ Hence, again from (2) we get $$ \\begin{aligned} f(n)=2(n-1) \\sum_{i=1}^{n-1} & \\frac{(n-2) ! f(i-1) 2^{n-1-i}}{(i-1) !}+f(n-1) \\\\ & =(2 n-2) f(n-1)+f(n-1)=(2 n-1) f(n-1), \\end{aligned} $$ QED. Comment. There exist different ways of obtaining the formula (2). Here we show one of them. Suppose that in the first move we use weight $2^{n-i+1}$. Then the lighter $n-i$ weights may be put on the balance at any moment and on either pan. This gives $2^{n-i} \\cdot(n-1) ! \/(i-1)$ ! choices for the moves (moments and choices of pan) with the lighter weights. The remaining $i-1$ moves give a valid sequence for the $i-1$ heavier weights and this is the only requirement for these moves, so there are $f(i-1)$ such sequences. Summing over all $i=1,2, \\ldots, n$ we again come to (2).","tier":0} +{"problem_type":"Number Theory","problem_label":"C2","problem":"Suppose that 1000 students are standing in a circle. Prove that there exists an integer $k$ with $100 \\leq k \\leq 300$ such that in this circle there exists a contiguous group of $2 k$ students, for which the first half contains the same number of girls as the second half.","solution":"Number the students consecutively from 1 to 1000. Let $a_{i}=1$ if the $i$ th student is a girl, and $a_{i}=0$ otherwise. We expand this notion for all integers $i$ by setting $a_{i+1000}=$ $a_{i-1000}=a_{i}$. Next, let $$ S_{k}(i)=a_{i}+a_{i+1}+\\cdots+a_{i+k-1} $$ Now the statement of the problem can be reformulated as follows: There exist an integer $k$ with $100 \\leq k \\leq 300$ and an index $i$ such that $S_{k}(i)=S_{k}(i+k)$. Assume now that this statement is false. Choose an index $i$ such that $S_{100}(i)$ attains the maximal possible value. In particular, we have $S_{100}(i-100)-S_{100}(i)<0$ and $S_{100}(i)-S_{100}(i+100)>0$, for if we had an equality, then the statement would hold. This means that the function $S(j)-$ $S(j+100)$ changes sign somewhere on the segment $[i-100, i]$, so there exists some index $j \\in$ $[i-100, i-1]$ such that $$ S_{100}(j) \\leq S_{100}(j+100)-1, \\quad \\text { but } \\quad S_{100}(j+1) \\geq S_{100}(j+101)+1 $$ Subtracting the first inequality from the second one, we get $a_{j+100}-a_{j} \\geq a_{j+200}-a_{j+100}+2$, so $$ a_{j}=0, \\quad a_{j+100}=1, \\quad a_{j+200}=0 $$ Substituting this into the inequalities of (1), we also obtain $S_{99}(j+1) \\leq S_{99}(j+101) \\leq S_{99}(j+1)$, which implies $$ S_{99}(j+1)=S_{99}(j+101) . $$ Now let $k$ and $\\ell$ be the least positive integers such that $a_{j-k}=1$ and $a_{j+200+\\ell}=1$. By symmetry, we may assume that $k \\geq \\ell$. If $k \\geq 200$ then we have $a_{j}=a_{j-1}=\\cdots=a_{j-199}=0$, so $S_{100}(j-199)=S_{100}(j-99)=0$, which contradicts the initial assumption. Hence $\\ell \\leq k \\leq 199$. Finally, we have $$ \\begin{gathered} S_{100+\\ell}(j-\\ell+1)=\\left(a_{j-\\ell+1}+\\cdots+a_{j}\\right)+S_{99}(j+1)+a_{j+100}=S_{99}(j+1)+1, \\\\ S_{100+\\ell}(j+101)=S_{99}(j+101)+\\left(a_{j+200}+\\cdots+a_{j+200+\\ell-1}\\right)+a_{j+200+\\ell}=S_{99}(j+101)+1 . \\end{gathered} $$ Comparing with (2) we get $S_{100+\\ell}(j-\\ell+1)=S_{100+\\ell}(j+101)$ and $100+\\ell \\leq 299$, which again contradicts our assumption. Comment. It may be seen from the solution that the number 300 from the problem statement can be replaced by 299. Here we consider some improvements of this result. Namely, we investigate which interval can be put instead of $[100,300]$ in order to keep the problem statement valid. First of all, the two examples ![](https:\/\/cdn.mathpix.com\/cropped\/2024_04_17_481ddb3576adc2313a2eg-31.jpg?height=109&width=1054&top_left_y=454&top_left_x=501) and $$ \\underbrace{1,1, \\ldots, 1}_{249}, \\underbrace{0,0, \\ldots, 0}_{251}, \\underbrace{1,1, \\ldots, 1}_{249}, \\underbrace{0,0, \\ldots, 0}_{251} $$ show that the interval can be changed neither to $[84,248]$ nor to $[126,374]$. On the other hand, we claim that this interval can be changed to $[125,250]$. Note that this statement is invariant under replacing all 1's by 0's and vice versa. Assume, to the contrary, that there is no admissible $k \\in[125,250]$. The arguments from the solution easily yield the following lemma. Lemma. Under our assumption, suppose that for some indices $i\\left(i_{3}^{\\prime}-i_{1}\\right)+\\left(i_{6}^{\\prime}-i_{4}\\right) \\geq 500+500$. This final contradiction shows that our claim holds. One may even show that the interval in the statement of the problem may be replaced by $[125,249]$ (both these numbers cannot be improved due to the examples above). But a proof of this fact is a bit messy, and we do not present it here.","tier":0} +{"problem_type":"Number Theory","problem_label":"C3","problem":"Let $\\mathcal{S}$ be a finite set of at least two points in the plane. Assume that no three points of $\\mathcal{S}$ are collinear. By a windmill we mean a process as follows. Start with a line $\\ell$ going through a point $P \\in \\mathcal{S}$. Rotate $\\ell$ clockwise around the pivot $P$ until the line contains another point $Q$ of $\\mathcal{S}$. The point $Q$ now takes over as the new pivot. This process continues indefinitely, with the pivot always being a point from $\\mathcal{S}$. Show that for a suitable $P \\in \\mathcal{S}$ and a suitable starting line $\\ell$ containing $P$, the resulting windmill will visit each point of $\\mathcal{S}$ as a pivot infinitely often.","solution":"Give the rotating line an orientation and distinguish its sides as the oranje side and the blue side. Notice that whenever the pivot changes from some point $T$ to another point $U$, after the change, $T$ is on the same side as $U$ was before. Therefore, the number of elements of $\\mathcal{S}$ on the oranje side and the number of those on the blue side remain the same throughout the whole process (except for those moments when the line contains two points). ![](https:\/\/cdn.mathpix.com\/cropped\/2024_04_17_481ddb3576adc2313a2eg-32.jpg?height=422&width=1108&top_left_y=1198&top_left_x=474) First consider the case that $|\\mathcal{S}|=2 n+1$ is odd. We claim that through any point $T \\in \\mathcal{S}$, there is a line that has $n$ points on each side. To see this, choose an oriented line through $T$ containing no other point of $\\mathcal{S}$ and suppose that it has $n+r$ points on its oranje side. If $r=0$ then we have established the claim, so we may assume that $r \\neq 0$. As the line rotates through $180^{\\circ}$ around $T$, the number of points of $\\mathcal{S}$ on its oranje side changes by 1 whenever the line passes through a point; after $180^{\\circ}$, the number of points on the oranje side is $n-r$. Therefore there is an intermediate stage at which the oranje side, and thus also the blue side, contains $n$ points. Now select the point $P$ arbitrarily, and choose a line through $P$ that has $n$ points of $\\mathcal{S}$ on each side to be the initial state of the windmill. We will show that during a rotation over $180^{\\circ}$, the line of the windmill visits each point of $\\mathcal{S}$ as a pivot. To see this, select any point $T$ of $\\mathcal{S}$ and select a line $\\ell$ through $T$ that separates $\\mathcal{S}$ into equal halves. The point $T$ is the unique point of $\\mathcal{S}$ through which a line in this direction can separate the points of $\\mathcal{S}$ into equal halves (parallel translation would disturb the balance). Therefore, when the windmill line is parallel to $\\ell$, it must be $\\ell$ itself, and so pass through $T$. Next suppose that $|\\mathcal{S}|=2 n$. Similarly to the odd case, for every $T \\in \\mathcal{S}$ there is an oriented line through $T$ with $n-1$ points on its oranje side and $n$ points on its blue side. Select such an oriented line through an arbitrary $P$ to be the initial state of the windmill. We will now show that during a rotation over $360^{\\circ}$, the line of the windmill visits each point of $\\mathcal{S}$ as a pivot. To see this, select any point $T$ of $\\mathcal{S}$ and an oriented line $\\ell$ through $T$ that separates $\\mathcal{S}$ into two subsets with $n-1$ points on its oranje and $n$ points on its blue side. Again, parallel translation would change the numbers of points on the two sides, so when the windmill line is parallel to $\\ell$ with the same orientation, the windmill line must pass through $T$. Comment. One may shorten this solution in the following way. Suppose that $|\\mathcal{S}|=2 n+1$. Consider any line $\\ell$ that separates $\\mathcal{S}$ into equal halves; this line is unique given its direction and contains some point $T \\in \\mathcal{S}$. Consider the windmill starting from this line. When the line has made a rotation of $180^{\\circ}$, it returns to the same location but the oranje side becomes blue and vice versa. So, for each point there should have been a moment when it appeared as pivot, as this is the only way for a point to pass from on side to the other. Now suppose that $|\\mathcal{S}|=2 n$. Consider a line having $n-1$ and $n$ points on the two sides; it contains some point $T$. Consider the windmill starting from this line. After having made a rotation of $180^{\\circ}$, the windmill line contains some different point $R$, and each point different from $T$ and $R$ has changed the color of its side. So, the windmill should have passed through all the points. ## $\\mathrm{C} 4$ Determine the greatest positive integer $k$ that satisfies the following property: The set of positive integers can be partitioned into $k$ subsets $A_{1}, A_{2}, \\ldots, A_{k}$ such that for all integers $n \\geq 15$ and all $i \\in\\{1,2, \\ldots, k\\}$ there exist two distinct elements of $A_{i}$ whose sum is $n$. Answer. The greatest such number $k$ is 3 .","tier":0} +{"problem_type":"Number Theory","problem_label":"C3","problem":"Let $\\mathcal{S}$ be a finite set of at least two points in the plane. Assume that no three points of $\\mathcal{S}$ are collinear. By a windmill we mean a process as follows. Start with a line $\\ell$ going through a point $P \\in \\mathcal{S}$. Rotate $\\ell$ clockwise around the pivot $P$ until the line contains another point $Q$ of $\\mathcal{S}$. The point $Q$ now takes over as the new pivot. This process continues indefinitely, with the pivot always being a point from $\\mathcal{S}$. Show that for a suitable $P \\in \\mathcal{S}$ and a suitable starting line $\\ell$ containing $P$, the resulting windmill will visit each point of $\\mathcal{S}$ as a pivot infinitely often.","solution":"There are various examples showing that $k=3$ does indeed have the property under consideration. E.g. one can take $$ \\begin{gathered} A_{1}=\\{1,2,3\\} \\cup\\{3 m \\mid m \\geq 4\\}, \\\\ A_{2}=\\{4,5,6\\} \\cup\\{3 m-1 \\mid m \\geq 4\\}, \\\\ A_{3}=\\{7,8,9\\} \\cup\\{3 m-2 \\mid m \\geq 4\\} \\end{gathered} $$ To check that this partition fits, we notice first that the sums of two distinct elements of $A_{i}$ obviously represent all numbers $n \\geq 1+12=13$ for $i=1$, all numbers $n \\geq 4+11=15$ for $i=2$, and all numbers $n \\geq 7+10=17$ for $i=3$. So, we are left to find representations of the numbers 15 and 16 as sums of two distinct elements of $A_{3}$. These are $15=7+8$ and $16=7+9$. Let us now suppose that for some $k \\geq 4$ there exist sets $A_{1}, A_{2}, \\ldots, A_{k}$ satisfying the given property. Obviously, the sets $A_{1}, A_{2}, A_{3}, A_{4} \\cup \\cdots \\cup A_{k}$ also satisfy the same property, so one may assume $k=4$. Put $B_{i}=A_{i} \\cap\\{1,2, \\ldots, 23\\}$ for $i=1,2,3,4$. Now for any index $i$ each of the ten numbers $15,16, \\ldots, 24$ can be written as sum of two distinct elements of $B_{i}$. Therefore this set needs to contain at least five elements. As we also have $\\left|B_{1}\\right|+\\left|B_{2}\\right|+\\left|B_{3}\\right|+\\left|B_{4}\\right|=23$, there has to be some index $j$ for which $\\left|B_{j}\\right|=5$. Let $B_{j}=\\left\\{x_{1}, x_{2}, x_{3}, x_{4}, x_{5}\\right\\}$. Finally, now the sums of two distinct elements of $A_{j}$ representing the numbers $15,16, \\ldots, 24$ should be exactly all the pairwise sums of the elements of $B_{j}$. Calculating the sum of these numbers in two different ways, we reach $$ 4\\left(x_{1}+x_{2}+x_{3}+x_{4}+x_{5}\\right)=15+16+\\ldots+24=195 . $$ Thus the number 195 should be divisible by 4 , which is false. This contradiction completes our solution. Comment. There are several variation of the proof that $k$ should not exceed 3. E.g., one may consider the sets $C_{i}=A_{i} \\cap\\{1,2, \\ldots, 19\\}$ for $i=1,2,3,4$. As in the previous solution one can show that for some index $j$ one has $\\left|C_{j}\\right|=4$, and the six pairwise sums of the elements of $C_{j}$ should represent all numbers $15,16, \\ldots, 20$. Let $C_{j}=\\left\\{y_{1}, y_{2}, y_{3}, y_{4}\\right\\}$ with $y_{1}1$. In the sequel, the word collision will be used to denote meeting of exactly two ants, moving in opposite directions. If at the beginning we place an ant on the southwest corner square facing east and an ant on the southeast corner square facing west, then they will meet in the middle of the bottom row at time $\\frac{m-1}{2}$. After the collision, the ant that moves to the north will stay on the board for another $m-\\frac{1}{2}$ time units and thus we have established an example in which the last ant falls off at time $\\frac{m-1}{2}+m-\\frac{1}{2}=\\frac{3 m}{2}-1$. So, we are left to prove that this is the latest possible moment. Consider any collision of two ants $a$ and $a^{\\prime}$. Let us change the rule for this collision, and enforce these two ants to turn anticlockwise. Then the succeeding behavior of all the ants does not change; the only difference is that $a$ and $a^{\\prime}$ swap their positions. These arguments may be applied to any collision separately, so we may assume that at any collision, either both ants rotate clockwise or both of them rotate anticlockwise by our own choice. For instance, we may assume that there are only two types of ants, depending on their initial direction: $N E$-ants, which move only north or east, and $S W$-ants, moving only south and west. Then we immediately obtain that all ants will have fallen off the board after $2 m-1$ time units. However, we can get a better bound by considering the last moment at which a given ant collides with another ant. Choose a coordinate system such that the corners of the checkerboard are $(0,0),(m, 0),(m, m)$ and $(0, m)$. At time $t$, there will be no NE-ants in the region $\\{(x, y): x+y2 m-t-1\\}$. So if two ants collide at $(x, y)$ at time $t$, we have $$ t+1 \\leq x+y \\leq 2 m-t-1 $$ Analogously, we may change the rules so that each ant would move either alternatingly north and west, or alternatingly south and east. By doing so, we find that apart from (1) we also have $|x-y| \\leq m-t-1$ for each collision at point $(x, y)$ and time $t$. To visualize this, put $$ B(t)=\\left\\{(x, y) \\in[0, m]^{2}: t+1 \\leq x+y \\leq 2 m-t-1 \\text { and }|x-y| \\leq m-t-1\\right\\} . $$ An ant can thus only collide with another ant at time $t$ if it happens to be in the region $B(t)$. The following figure displays $B(t)$ for $t=\\frac{1}{2}$ and $t=\\frac{7}{2}$ in the case $m=6$ : ![](https:\/\/cdn.mathpix.com\/cropped\/2024_04_17_481ddb3576adc2313a2eg-37.jpg?height=622&width=623&top_left_y=771&top_left_x=722) Now suppose that an NE-ant has its last collision at time $t$ and that it does so at the point $(x, y)$ (if the ant does not collide at all, it will fall off the board within $m-\\frac{1}{2}<\\frac{3 m}{2}-1$ time units, so this case can be ignored). Then we have $(x, y) \\in B(t)$ and thus $x+y \\geq t+1$ and $x-y \\geq-(m-t-1)$. So we get $$ x \\geq \\frac{(t+1)-(m-t-1)}{2}=t+1-\\frac{m}{2} $$ By symmetry we also have $y \\geq t+1-\\frac{m}{2}$, and hence $\\min \\{x, y\\} \\geq t+1-\\frac{m}{2}$. After this collision, the ant will move directly to an edge, which will take at $\\operatorname{most} m-\\min \\{x, y\\}$ units of time. In sum, the total amount of time the ant stays on the board is at most $$ t+(m-\\min \\{x, y\\}) \\leq t+m-\\left(t+1-\\frac{m}{2}\\right)=\\frac{3 m}{2}-1 $$ By symmetry, the same bound holds for $\\mathrm{SW}$-ants as well.","tier":0} +{"problem_type":"Number Theory","problem_label":"C7","problem":"On a square table of 2011 by 2011 cells we place a finite number of napkins that each cover a square of 52 by 52 cells. In each cell we write the number of napkins covering it, and we record the maximal number $k$ of cells that all contain the same nonzero number. Considering all possible napkin configurations, what is the largest value of $k$ ? Answer. $2011^{2}-\\left(\\left(52^{2}-35^{2}\\right) \\cdot 39-17^{2}\\right)=4044121-57392=3986729$.","solution":"Let $m=39$, then $2011=52 m-17$. We begin with an example showing that there can exist 3986729 cells carrying the same positive number. ![](https:\/\/cdn.mathpix.com\/cropped\/2024_04_17_481ddb3576adc2313a2eg-40.jpg?height=577&width=580&top_left_y=1022&top_left_x=744) To describe it, we number the columns from the left to the right and the rows from the bottom to the top by $1,2, \\ldots, 2011$. We will denote each napkin by the coordinates of its lowerleft cell. There are four kinds of napkins: first, we take all napkins $(52 i+36,52 j+1)$ with $0 \\leq j \\leq i \\leq m-2$; second, we use all napkins $(52 i+1,52 j+36)$ with $0 \\leq i \\leq j \\leq m-2$; third, we use all napkins $(52 i+36,52 i+36)$ with $0 \\leq i \\leq m-2$; and finally the napkin $(1,1)$. Different groups of napkins are shown by different types of hatchings in the picture. Now except for those squares that carry two or more different hatchings, all squares have the number 1 written into them. The number of these exceptional cells is easily computed to be $\\left(52^{2}-35^{2}\\right) m-17^{2}=57392$. We are left to prove that 3986729 is an upper bound for the number of cells containing the same number. Consider any configuration of napkins and any positive integer $M$. Suppose there are $g$ cells with a number different from $M$. Then it suffices to show $g \\geq 57392$. Throughout the solution, a line will mean either a row or a column. Consider any line $\\ell$. Let $a_{1}, \\ldots, a_{52 m-17}$ be the numbers written into its consecutive cells. For $i=1,2, \\ldots, 52$, let $s_{i}=\\sum_{t \\equiv i(\\bmod 52)} a_{t}$. Note that $s_{1}, \\ldots, s_{35}$ have $m$ terms each, while $s_{36}, \\ldots, s_{52}$ have $m-1$ terms each. Every napkin intersecting $\\ell$ contributes exactly 1 to each $s_{i}$; hence the number $s$ of all those napkins satisfies $s_{1}=\\cdots=s_{52}=s$. Call the line $\\ell$ rich if $s>(m-1) M$ and poor otherwise. Suppose now that $\\ell$ is rich. Then in each of the sums $s_{36}, \\ldots, s_{52}$ there exists a term greater than $M$; consider all these terms and call the corresponding cells the rich bad cells for this line. So, each rich line contains at least 17 cells that are bad for this line. If, on the other hand, $\\ell$ is poor, then certainly $s2 H^{2}$, then the answer and the example are the same as in the previous case; otherwise the answer is $(2 m-1) S^{2}+2 S H(m-1)$, and the example is provided simply by $(m-1)^{2}$ nonintersecting napkins. Now we sketch the proof of both estimates for Case 2. We introduce a more appropriate notation based on that from Solution 2. Denote by $a_{-}$and $a_{+}$the number of cells of class $A$ that contain the number which is strictly less than $M$ and strictly greater than $M$, respectively. The numbers $b_{ \\pm}, c_{ \\pm}$, and $d_{ \\pm}$are defined in a similar way. One may notice that the proofs of Claim 1 and Claims 2, 3 lead in fact to the inequalities $$ m-1 \\leq \\frac{b_{-}+c_{-}}{2 S H}+\\frac{d_{+}}{H^{2}} \\quad \\text { and } \\quad 2 m-1 \\leq \\frac{a}{S^{2}}+\\frac{b_{+}+c_{+}}{2 S H}+\\frac{d_{+}}{H^{2}} $$ (to obtain the first one, one needs to look at the big lines instead of the small ones). Combining these inequalities, one may obtain the desired estimates. These estimates can also be proved in some different ways, e.g. without distinguishing rich and poor cells.","tier":0} +{"problem_type":"Number Theory","problem_label":"G1","problem":"Let $A B C$ be an acute triangle. Let $\\omega$ be a circle whose center $L$ lies on the side $B C$. Suppose that $\\omega$ is tangent to $A B$ at $B^{\\prime}$ and to $A C$ at $C^{\\prime}$. Suppose also that the circumcenter $O$ of the triangle $A B C$ lies on the shorter arc $B^{\\prime} C^{\\prime}$ of $\\omega$. Prove that the circumcircle of $A B C$ and $\\omega$ meet at two points.","solution":"The point $B^{\\prime}$, being the perpendicular foot of $L$, is an interior point of side $A B$. Analogously, $C^{\\prime}$ lies in the interior of $A C$. The point $O$ is located inside the triangle $A B^{\\prime} C^{\\prime}$, hence $\\angle C O B<\\angle C^{\\prime} O B^{\\prime}$. ![](https:\/\/cdn.mathpix.com\/cropped\/2024_04_17_481ddb3576adc2313a2eg-45.jpg?height=837&width=674&top_left_y=895&top_left_x=697) Let $\\alpha=\\angle C A B$. The angles $\\angle C A B$ and $\\angle C^{\\prime} O B^{\\prime}$ are inscribed into the two circles with centers $O$ and $L$, respectively, so $\\angle C O B=2 \\angle C A B=2 \\alpha$ and $2 \\angle C^{\\prime} O B^{\\prime}=360^{\\circ}-\\angle C^{\\prime} L B^{\\prime}$. From the kite $A B^{\\prime} L C^{\\prime}$ we have $\\angle C^{\\prime} L B^{\\prime}=180^{\\circ}-\\angle C^{\\prime} A B^{\\prime}=180^{\\circ}-\\alpha$. Combining these, we get $$ 2 \\alpha=\\angle C O B<\\angle C^{\\prime} O B^{\\prime}=\\frac{360^{\\circ}-\\angle C^{\\prime} L B^{\\prime}}{2}=\\frac{360^{\\circ}-\\left(180^{\\circ}-\\alpha\\right)}{2}=90^{\\circ}+\\frac{\\alpha}{2}, $$ So $$ \\alpha<60^{\\circ} \\text {. } $$ Let $O^{\\prime}$ be the reflection of $O$ in the line $B C$. In the quadrilateral $A B O^{\\prime} C$ we have $$ \\angle C O^{\\prime} B+\\angle C A B=\\angle C O B+\\angle C A B=2 \\alpha+\\alpha<180^{\\circ}, $$ so the point $O^{\\prime}$ is outside the circle $A B C$. Hence, $O$ and $O^{\\prime}$ are two points of $\\omega$ such that one of them lies inside the circumcircle, while the other one is located outside. Therefore, the two circles intersect. Comment. There are different ways of reducing the statement of the problem to the case $\\alpha<60^{\\circ}$. E.g., since the point $O$ lies in the interior of the isosceles triangle $A B^{\\prime} C^{\\prime}$, we have $O A2 L B^{\\prime}$, and this condition implies $\\angle C A B=2 \\angle B^{\\prime} A L<2 \\cdot 30^{\\circ}=60^{\\circ}$.","tier":0} +{"problem_type":"Number Theory","problem_label":"G8","problem":"Let $A B C$ be an acute triangle with circumcircle $\\omega$. Let $t$ be a tangent line to $\\omega$. Let $t_{a}, t_{b}$, and $t_{c}$ be the lines obtained by reflecting $t$ in the lines $B C, C A$, and $A B$, respectively. Show that the circumcircle of the triangle determined by the lines $t_{a}, t_{b}$, and $t_{c}$ is tangent to the circle $\\omega$. To avoid a large case distinction, we will use the notion of oriented angles. Namely, for two lines $\\ell$ and $m$, we denote by $\\angle(\\ell, m)$ the angle by which one may rotate $\\ell$ anticlockwise to obtain a line parallel to $m$. Thus, all oriented angles are considered modulo $180^{\\circ}$. ![](https:\/\/cdn.mathpix.com\/cropped\/2024_04_17_481ddb3576adc2313a2eg-61.jpg?height=1090&width=1676&top_left_y=820&top_left_x=196)","solution":"Denote by $T$ the point of tangency of $t$ and $\\omega$. Let $A^{\\prime}=t_{b} \\cap t_{c}, B^{\\prime}=t_{a} \\cap t_{c}$, $C^{\\prime}=t_{a} \\cap t_{b}$. Introduce the point $A^{\\prime \\prime}$ on $\\omega$ such that $T A=A A^{\\prime \\prime}\\left(A^{\\prime \\prime} \\neq T\\right.$ unless $T A$ is a diameter). Define the points $B^{\\prime \\prime}$ and $C^{\\prime \\prime}$ in a similar way. Since the points $C$ and $B$ are the midpoints of arcs $T C^{\\prime \\prime}$ and $T B^{\\prime \\prime}$, respectively, we have $$ \\begin{aligned} \\angle\\left(t, B^{\\prime \\prime} C^{\\prime \\prime}\\right) & =\\angle\\left(t, T C^{\\prime \\prime}\\right)+\\angle\\left(T C^{\\prime \\prime}, B^{\\prime \\prime} C^{\\prime \\prime}\\right)=2 \\angle(t, T C)+2 \\angle\\left(T C^{\\prime \\prime}, B C^{\\prime \\prime}\\right) \\\\ & =2(\\angle(t, T C)+\\angle(T C, B C))=2 \\angle(t, B C)=\\angle\\left(t, t_{a}\\right) . \\end{aligned} $$ It follows that $t_{a}$ and $B^{\\prime \\prime} C^{\\prime \\prime}$ are parallel. Similarly, $t_{b} \\| A^{\\prime \\prime} C^{\\prime \\prime}$ and $t_{c} \\| A^{\\prime \\prime} B^{\\prime \\prime}$. Thus, either the triangles $A^{\\prime} B^{\\prime} C^{\\prime}$ and $A^{\\prime \\prime} B^{\\prime \\prime} C^{\\prime \\prime}$ are homothetic, or they are translates of each other. Now we will prove that they are in fact homothetic, and that the center $K$ of the homothety belongs to $\\omega$. It would then follow that their circumcircles are also homothetic with respect to $K$ and are therefore tangent at this point, as desired. We need the two following claims. Claim 1. The point of intersection $X$ of the lines $B^{\\prime \\prime} C$ and $B C^{\\prime \\prime}$ lies on $t_{a}$. Proof. Actually, the points $X$ and $T$ are symmetric about the line $B C$, since the lines $C T$ and $C B^{\\prime \\prime}$ are symmetric about this line, as are the lines $B T$ and $B C^{\\prime \\prime}$. Claim 2. The point of intersection $I$ of the lines $B B^{\\prime}$ and $C C^{\\prime}$ lies on the circle $\\omega$. Proof. We consider the case that $t$ is not parallel to the sides of $A B C$; the other cases may be regarded as limit cases. Let $D=t \\cap B C, E=t \\cap A C$, and $F=t \\cap A B$. Due to symmetry, the line $D B$ is one of the angle bisectors of the lines $B^{\\prime} D$ and $F D$; analogously, the line $F B$ is one of the angle bisectors of the lines $B^{\\prime} F$ and $D F$. So $B$ is either the incenter or one of the excenters of the triangle $B^{\\prime} D F$. In any case we have $\\angle(B D, D F)+\\angle(D F, F B)+$ $\\angle\\left(B^{\\prime} B, B^{\\prime} D\\right)=90^{\\circ}$, so $$ \\angle\\left(B^{\\prime} B, B^{\\prime} C^{\\prime}\\right)=\\angle\\left(B^{\\prime} B, B^{\\prime} D\\right)=90^{\\circ}-\\angle(B C, D F)-\\angle(D F, B A)=90^{\\circ}-\\angle(B C, A B) \\text {. } $$ Analogously, we get $\\angle\\left(C^{\\prime} C, B^{\\prime} C^{\\prime}\\right)=90^{\\circ}-\\angle(B C, A C)$. Hence, $$ \\angle(B I, C I)=\\angle\\left(B^{\\prime} B, B^{\\prime} C^{\\prime}\\right)+\\angle\\left(B^{\\prime} C^{\\prime}, C^{\\prime} C\\right)=\\angle(B C, A C)-\\angle(B C, A B)=\\angle(A B, A C), $$ which means exactly that the points $A, B, I, C$ are concyclic. Now we can complete the proof. Let $K$ be the second intersection point of $B^{\\prime} B^{\\prime \\prime}$ and $\\omega$. Applying PASCAL's theorem to hexagon $K B^{\\prime \\prime} C I B C^{\\prime \\prime}$ we get that the points $B^{\\prime}=K B^{\\prime \\prime} \\cap I B$ and $X=B^{\\prime \\prime} C \\cap B C^{\\prime \\prime}$ are collinear with the intersection point $S$ of $C I$ and $C^{\\prime \\prime} K$. So $S=$ $C I \\cap B^{\\prime} X=C^{\\prime}$, and the points $C^{\\prime}, C^{\\prime \\prime}, K$ are collinear. Thus $K$ is the intersection point of $B^{\\prime} B^{\\prime \\prime}$ and $C^{\\prime} C^{\\prime \\prime}$ which implies that $K$ is the center of the homothety mapping $A^{\\prime} B^{\\prime} C^{\\prime}$ to $A^{\\prime \\prime} B^{\\prime \\prime} C^{\\prime \\prime}$, and it belongs to $\\omega$.","tier":0} +{"problem_type":"Number Theory","problem_label":"G8","problem":"Let $A B C$ be an acute triangle with circumcircle $\\omega$. Let $t$ be a tangent line to $\\omega$. Let $t_{a}, t_{b}$, and $t_{c}$ be the lines obtained by reflecting $t$ in the lines $B C, C A$, and $A B$, respectively. Show that the circumcircle of the triangle determined by the lines $t_{a}, t_{b}$, and $t_{c}$ is tangent to the circle $\\omega$. To avoid a large case distinction, we will use the notion of oriented angles. Namely, for two lines $\\ell$ and $m$, we denote by $\\angle(\\ell, m)$ the angle by which one may rotate $\\ell$ anticlockwise to obtain a line parallel to $m$. Thus, all oriented angles are considered modulo $180^{\\circ}$. ![](https:\/\/cdn.mathpix.com\/cropped\/2024_04_17_481ddb3576adc2313a2eg-61.jpg?height=1090&width=1676&top_left_y=820&top_left_x=196)","solution":"Define the points $T, A^{\\prime}, B^{\\prime}$, and $C^{\\prime}$ in the same way as in the previous solution. Let $X, Y$, and $Z$ be the symmetric images of $T$ about the lines $B C, C A$, and $A B$, respectively. Note that the projections of $T$ on these lines form a Simson line of $T$ with respect to $A B C$, therefore the points $X, Y, Z$ are also collinear. Moreover, we have $X \\in B^{\\prime} C^{\\prime}, Y \\in C^{\\prime} A^{\\prime}$, $Z \\in A^{\\prime} B^{\\prime}$. Denote $\\alpha=\\angle(t, T C)=\\angle(B T, B C)$. Using the symmetry in the lines $A C$ and $B C$, we get $$ \\angle(B C, B X)=\\angle(B T, B C)=\\alpha \\quad \\text { and } \\quad \\angle\\left(X C, X C^{\\prime}\\right)=\\angle(t, T C)=\\angle\\left(Y C, Y C^{\\prime}\\right)=\\alpha . $$ Since $\\angle\\left(X C, X C^{\\prime}\\right)=\\angle\\left(Y C, Y C^{\\prime}\\right)$, the points $X, Y, C, C^{\\prime}$ lie on some circle $\\omega_{c}$. Define the circles $\\omega_{a}$ and $\\omega_{b}$ analogously. Let $\\omega^{\\prime}$ be the circumcircle of triangle $A^{\\prime} B^{\\prime} C^{\\prime}$. Now, applying Miquel's theorem to the four lines $A^{\\prime} B^{\\prime}, A^{\\prime} C^{\\prime}, B^{\\prime} C^{\\prime}$, and $X Y$, we obtain that the circles $\\omega^{\\prime}, \\omega_{a}, \\omega_{b}, \\omega_{c}$ intersect at some point $K$. We will show that $K$ lies on $\\omega$, and that the tangent lines to $\\omega$ and $\\omega^{\\prime}$ at this point coincide; this implies the problem statement. Due to symmetry, we have $X B=T B=Z B$, so the point $B$ is the midpoint of one of the $\\operatorname{arcs} X Z$ of circle $\\omega_{b}$. Therefore $\\angle(K B, K X)=\\angle(X Z, X B)$. Analogously, $\\angle(K X, K C)=$ $\\angle(X C, X Y)$. Adding these equalities and using the symmetry in the line $B C$ we get $$ \\angle(K B, K C)=\\angle(X Z, X B)+\\angle(X C, X Z)=\\angle(X C, X B)=\\angle(T B, T C) . $$ Therefore, $K$ lies on $\\omega$. Next, let $k$ be the tangent line to $\\omega$ at $K$. We have $$ \\begin{aligned} \\angle\\left(k, K C^{\\prime}\\right) & =\\angle(k, K C)+\\angle\\left(K C, K C^{\\prime}\\right)=\\angle(K B, B C)+\\angle\\left(X C, X C^{\\prime}\\right) \\\\ & =(\\angle(K B, B X)-\\angle(B C, B X))+\\alpha=\\angle\\left(K B^{\\prime}, B^{\\prime} X\\right)-\\alpha+\\alpha=\\angle\\left(K B^{\\prime}, B^{\\prime} C^{\\prime}\\right), \\end{aligned} $$ which means exactly that $k$ is tangent to $\\omega^{\\prime}$. ![](https:\/\/cdn.mathpix.com\/cropped\/2024_04_17_481ddb3576adc2313a2eg-63.jpg?height=1165&width=1219&top_left_y=1185&top_left_x=424) Comment. There exist various solutions combining the ideas from the two solutions presented above. For instance, one may define the point $X$ as the reflection of $T$ with respect to the line $B C$, and then introduce the point $K$ as the second intersection point of the circumcircles of $B B^{\\prime} X$ and $C C^{\\prime} X$. Using the fact that $B B^{\\prime}$ and $C C^{\\prime}$ are the bisectors of $\\angle\\left(A^{\\prime} B^{\\prime}, B^{\\prime} C^{\\prime}\\right)$ and $\\angle\\left(A^{\\prime} C^{\\prime}, B^{\\prime} C^{\\prime}\\right)$ one can show successively that $K \\in \\omega, K \\in \\omega^{\\prime}$, and that the tangents to $\\omega$ and $\\omega^{\\prime}$ at $K$ coincide.","tier":0} +{"problem_type":"Number Theory","problem_label":"N4","problem":"For each positive integer $k$, let $t(k)$ be the largest odd divisor of $k$. Determine all positive integers $a$ for which there exists a positive integer $n$ such that all the differences $$ t(n+a)-t(n), \\quad t(n+a+1)-t(n+1), \\quad \\ldots, \\quad t(n+2 a-1)-t(n+a-1) $$ are divisible by 4 . Answer. $a=1,3$, or 5 .","solution":"A pair $(a, n)$ satisfying the condition of the problem will be called a winning pair. It is straightforward to check that the pairs $(1,1),(3,1)$, and $(5,4)$ are winning pairs. Now suppose that $a$ is a positive integer not equal to 1,3 , and 5 . We will show that there are no winning pairs $(a, n)$ by distinguishing three cases. Case 1: $a$ is even. In this case we have $a=2^{\\alpha} d$ for some positive integer $\\alpha$ and some odd $d$. Since $a \\geq 2^{\\alpha}$, for each positive integer $n$ there exists an $i \\in\\{0,1, \\ldots, a-1\\}$ such that $n+i=2^{\\alpha-1} e$, where $e$ is some odd integer. Then we have $t(n+i)=t\\left(2^{\\alpha-1} e\\right)=e$ and $$ t(n+a+i)=t\\left(2^{\\alpha} d+2^{\\alpha-1} e\\right)=2 d+e \\equiv e+2 \\quad(\\bmod 4) . $$ So we get $t(n+i)-t(n+a+i) \\equiv 2(\\bmod 4)$, and $(a, n)$ is not a winning pair. Case 2: $a$ is odd and $a>8$. For each positive integer $n$, there exists an $i \\in\\{0,1, \\ldots, a-5\\}$ such that $n+i=2 d$ for some odd $d$. We get $$ t(n+i)=d \\not \\equiv d+2=t(n+i+4) \\quad(\\bmod 4) $$ and $$ t(n+a+i)=n+a+i \\equiv n+a+i+4=t(n+a+i+4) \\quad(\\bmod 4) $$ Therefore, the integers $t(n+a+i)-t(n+i)$ and $t(n+a+i+4)-t(n+i+4)$ cannot be both divisible by 4 , and therefore there are no winning pairs in this case. Case 3: $a=7$. For each positive integer $n$, there exists an $i \\in\\{0,1, \\ldots, 6\\}$ such that $n+i$ is either of the form $8 k+3$ or of the form $8 k+6$, where $k$ is a nonnegative integer. But we have $$ t(8 k+3) \\equiv 3 \\not \\equiv 1 \\equiv 4 k+5=t(8 k+3+7) \\quad(\\bmod 4) $$ and $$ t(8 k+6)=4 k+3 \\equiv 3 \\not \\equiv 1 \\equiv t(8 k+6+7) \\quad(\\bmod 4) . $$ Hence, there are no winning pairs of the form $(7, n)$.","tier":0} diff --git a/IMO/segmented/en-IMO2012SL.jsonl b/IMO/segmented/en-IMO2012SL.jsonl new file mode 100644 index 0000000000000000000000000000000000000000..6254bdb63b8ddcc568bcc74e3a50aaf0b6e0f5dd --- /dev/null +++ b/IMO/segmented/en-IMO2012SL.jsonl @@ -0,0 +1,41 @@ +{"problem_type":"Algebra","problem_label":"A1","problem":"Find all the functions $f: \\mathbb{Z} \\rightarrow \\mathbb{Z}$ such that $$ f(a)^{2}+f(b)^{2}+f(c)^{2}=2 f(a) f(b)+2 f(b) f(c)+2 f(c) f(a) $$ for all integers $a, b, c$ satisfying $a+b+c=0$.","solution":"The substitution $a=b=c=0$ gives $3 f(0)^{2}=6 f(0)^{2}$, hence $$ f(0)=0 \\text {. } $$ The substitution $b=-a$ and $c=0$ gives $\\left((f(a)-f(-a))^{2}=0\\right.$. Hence $f$ is an even function: $$ f(a)=f(-a) \\quad \\text { for all } a \\in \\mathbb{Z} $$ Now set $b=a$ and $c=-2 a$ to obtain $2 f(a)^{2}+f(2 a)^{2}=2 f(a)^{2}+4 f(a) f(2 a)$. Hence $$ f(2 a)=0 \\text { or } f(2 a)=4 f(a) \\text { for all } a \\in \\mathbb{Z} $$ If $f(r)=0$ for some $r \\geq 1$ then the substitution $b=r$ and $c=-a-r$ gives $(f(a+r)-f(a))^{2}=0$. So $f$ is periodic with period $r$, i. e. $$ f(a+r)=f(a) \\text { for all } a \\in \\mathbb{Z} $$ In particular, if $f(1)=0$ then $f$ is constant, thus $f(a)=0$ for all $a \\in \\mathbb{Z}$. This function clearly satisfies the functional equation. For the rest of the analysis, we assume $f(1)=k \\neq 0$. By (3) we have $f(2)=0$ or $f(2)=4 k$. If $f(2)=0$ then $f$ is periodic of period 2 , thus $f($ even $)=0$ and $f($ odd $)=k$. This function is a solution for every $k$. We postpone the verification; for the sequel assume $f(2)=4 k \\neq 0$. By (3) again, we have $f(4)=0$ or $f(4)=16 k$. In the first case $f$ is periodic of period 4 , and $f(3)=f(-1)=f(1)=k$, so we have $f(4 n)=0, f(4 n+1)=f(4 n+3)=k$, and $f(4 n+2)=4 k$ for all $n \\in \\mathbb{Z}$. This function is a solution too, which we justify later. For the rest of the analysis, we assume $f(4)=16 k \\neq 0$. We show now that $f(3)=9 k$. In order to do so, we need two substitutions: $$ \\begin{gathered} a=1, b=2, c=-3 \\Longrightarrow f(3)^{2}-10 k f(3)+9 k^{2}=0 \\Longrightarrow f(3) \\in\\{k, 9 k\\}, \\\\ a=1, b=3, c=-4 \\Longrightarrow f(3)^{2}-34 k f(3)+225 k^{2}=0 \\Longrightarrow f(3) \\in\\{9 k, 25 k\\} . \\end{gathered} $$ Therefore $f(3)=9 k$, as claimed. Now we prove inductively that the only remaining function is $f(x)=k x^{2}, x \\in \\mathbb{Z}$. We proved this for $x=0,1,2,3,4$. Assume that $n \\geq 4$ and that $f(x)=k x^{2}$ holds for all integers $x \\in[0, n]$. Then the substitutions $a=n, b=1, c=-n-1$ and $a=n-1$, $b=2, c=-n-1$ lead respectively to $$ f(n+1) \\in\\left\\{k(n+1)^{2}, k(n-1)^{2}\\right\\} \\quad \\text { and } \\quad f(n+1) \\in\\left\\{k(n+1)^{2}, k(n-3)^{2}\\right\\} \\text {. } $$ Since $k(n-1)^{2} \\neq k(n-3)^{2}$ for $n \\neq 2$, the only possibility is $f(n+1)=k(n+1)^{2}$. This completes the induction, so $f(x)=k x^{2}$ for all $x \\geq 0$. The same expression is valid for negative values of $x$ since $f$ is even. To verify that $f(x)=k x^{2}$ is actually a solution, we need to check the identity $a^{4}+b^{4}+(a+b)^{4}=2 a^{2} b^{2}+2 a^{2}(a+b)^{2}+2 b^{2}(a+b)^{2}$, which follows directly by expanding both sides. Therefore the only possible solutions of the functional equation are the constant function $f_{1}(x)=0$ and the following functions: $$ f_{2}(x)=k x^{2} \\quad f_{3}(x)=\\left\\{\\begin{array}{cc} 0 & x \\text { even } \\\\ k & x \\text { odd } \\end{array} \\quad f_{4}(x)=\\left\\{\\begin{array}{ccc} 0 & x \\equiv 0 & (\\bmod 4) \\\\ k & x \\equiv 1 & (\\bmod 2) \\\\ 4 k & x \\equiv 2 & (\\bmod 4) \\end{array}\\right.\\right. $$ for any non-zero integer $k$. The verification that they are indeed solutions was done for the first two. For $f_{3}$ note that if $a+b+c=0$ then either $a, b, c$ are all even, in which case $f(a)=f(b)=f(c)=0$, or one of them is even and the other two are odd, so both sides of the equation equal $2 k^{2}$. For $f_{4}$ we use similar parity considerations and the symmetry of the equation, which reduces the verification to the triples $(0, k, k),(4 k, k, k),(0,0,0),(0,4 k, 4 k)$. They all satisfy the equation. Comment. We used several times the same fact: For any $a, b \\in \\mathbb{Z}$ the functional equation is a quadratic equation in $f(a+b)$ whose coefficients depend on $f(a)$ and $f(b)$ : $$ f(a+b)^{2}-2(f(a)+f(b)) f(a+b)+(f(a)-f(b))^{2}=0 $$ Its discriminant is $16 f(a) f(b)$. Since this value has to be non-negative for any $a, b \\in \\mathbb{Z}$, we conclude that either $f$ or $-f$ is always non-negative. Also, if $f$ is a solution of the functional equation, then $-f$ is also a solution. Therefore we can assume $f(x) \\geq 0$ for all $x \\in \\mathbb{Z}$. Now, the two solutions of the quadratic equation are $$ f(a+b) \\in\\left\\{(\\sqrt{f(a)}+\\sqrt{f(b)})^{2},(\\sqrt{f(a)}-\\sqrt{f(b)})^{2}\\right\\} \\quad \\text { for all } a, b \\in \\mathbb{Z} $$ The computation of $f(3)$ from $f(1), f(2)$ and $f(4)$ that we did above follows immediately by setting $(a, b)=(1,2)$ and $(a, b)=(1,-4)$. The inductive step, where $f(n+1)$ is derived from $f(n), f(n-1)$, $f(2)$ and $f(1)$, follows immediately using $(a, b)=(n, 1)$ and $(a, b)=(n-1,2)$.","tier":0} +{"problem_type":"Algebra","problem_label":"A2","problem":"Let $\\mathbb{Z}$ and $\\mathbb{Q}$ be the sets of integers and rationals respectively. a) Does there exist a partition of $\\mathbb{Z}$ into three non-empty subsets $A, B, C$ such that the sets $A+B, B+C, C+A$ are disjoint? b) Does there exist a partition of $\\mathbb{Q}$ into three non-empty subsets $A, B, C$ such that the sets $A+B, B+C, C+A$ are disjoint? Here $X+Y$ denotes the set $\\{x+y \\mid x \\in X, y \\in Y\\}$, for $X, Y \\subseteq \\mathbb{Z}$ and $X, Y \\subseteq \\mathbb{Q}$.","solution":"a) The residue classes modulo 3 yield such a partition: $$ A=\\{3 k \\mid k \\in \\mathbb{Z}\\}, \\quad B=\\{3 k+1 \\mid k \\in \\mathbb{Z}\\}, \\quad C=\\{3 k+2 \\mid k \\in \\mathbb{Z}\\} $$ b) The answer is no. Suppose that $\\mathbb{Q}$ can be partitioned into non-empty subsets $A, B, C$ as stated. Note that for all $a \\in A, b \\in B, c \\in C$ one has $$ a+b-c \\in C, \\quad b+c-a \\in A, \\quad c+a-b \\in B $$ Indeed $a+b-c \\notin A$ as $(A+B) \\cap(A+C)=\\emptyset$, and similarly $a+b-c \\notin B$, hence $a+b-c \\in C$. The other two relations follow by symmetry. Hence $A+B \\subset C+C, B+C \\subset A+A, C+A \\subset B+B$. The opposite inclusions also hold. Let $a, a^{\\prime} \\in A$ and $b \\in B, c \\in C$ be arbitrary. By (1) $a^{\\prime}+c-b \\in B$, and since $a \\in A, c \\in C$, we use (1) again to obtain $$ a+a^{\\prime}-b=a+\\left(a^{\\prime}+c-b\\right)-c \\in C . $$ So $A+A \\subset B+C$ and likewise $B+B \\subset C+A, C+C \\subset A+B$. In summary $$ B+C=A+A, \\quad C+A=B+B, \\quad A+B=C+C . $$ Furthermore suppose that $0 \\in A$ without loss of generality. Then $B=\\{0\\}+B \\subset A+B$ and $C=\\{0\\}+C \\subset A+C$. So, since $B+C$ is disjoint with $A+B$ and $A+C$, it is also disjoint with $B$ and $C$. Hence $B+C$ is contained in $\\mathbb{Z} \\backslash(B \\cup C)=A$. Because $B+C=A+A$, we obtain $A+A \\subset A$. On the other hand $A=\\{0\\}+A \\subset A+A$, implying $A=A+A=B+C$. Therefore $A+B+C=A+A+A=A$, and now $B+B=C+A$ and $C+C=A+B$ yield $B+B+B=A+B+C=A, C+C+C=A+B+C=A$. In particular if $r \\in \\mathbb{Q}=A \\cup B \\cup C$ is arbitrary then $3 r \\in A$. However such a conclusion is impossible. Take any $b \\in B(B \\neq \\emptyset)$ and let $r=b \/ 3 \\in \\mathbb{Q}$. Then $b=3 r \\in A$ which is a contradiction.","tier":0} +{"problem_type":"Algebra","problem_label":"A2","problem":"Let $\\mathbb{Z}$ and $\\mathbb{Q}$ be the sets of integers and rationals respectively. a) Does there exist a partition of $\\mathbb{Z}$ into three non-empty subsets $A, B, C$ such that the sets $A+B, B+C, C+A$ are disjoint? b) Does there exist a partition of $\\mathbb{Q}$ into three non-empty subsets $A, B, C$ such that the sets $A+B, B+C, C+A$ are disjoint? Here $X+Y$ denotes the set $\\{x+y \\mid x \\in X, y \\in Y\\}$, for $X, Y \\subseteq \\mathbb{Z}$ and $X, Y \\subseteq \\mathbb{Q}$.","solution":"We prove that the example for $\\mathbb{Z}$ from the first solution is unique, and then use this fact to solve part b). Let $\\mathbb{Z}=A \\cup B \\cup C$ be a partition of $\\mathbb{Z}$ with $A, B, C \\neq \\emptyset$ and $A+B, B+C, C+A$ disjoint. We need the relations (1) which clearly hold for $\\mathbb{Z}$. Fix two consecutive integers from different sets, say $b \\in B$ and $c=b+1 \\in C$. For every $a \\in A$ we have, in view of (1), $a-1=a+b-c \\in C$ and $a+1=a+c-b \\in B$. So every $a \\in A$ is preceded by a number from $C$ and followed by a number from $B$. In particular there are pairs of the form $c, c+1$ with $c \\in C, c+1 \\in A$. For such a pair and any $b \\in B$ analogous reasoning shows that each $b \\in B$ is preceded by a number from $A$ and followed by a number from $C$. There are also pairs $b, b-1$ with $b \\in B, b-1 \\in A$. We use them in a similar way to prove that each $c \\in C$ is preceded by a number from $B$ and followed by a number from $A$. By putting the observations together we infer that $A, B, C$ are the three congruence classes modulo 3. Observe that all multiples of 3 are in the set of the partition that contains 0 . Now we turn to part b). Suppose that there is a partition of $\\mathbb{Q}$ with the given properties. Choose three rationals $r_{i}=p_{i} \/ q_{i}$ from the three sets $A, B, C, i=1,2,3$, and set $N=3 q_{1} q_{2} q_{3}$. Let $S \\subset \\mathbb{Q}$ be the set of fractions with denominators $N$ (irreducible or not). It is obtained through multiplication of every integer by the constant $1 \/ N$, hence closed under sums and differences. Moreover, if we identify each $k \\in \\mathbb{Z}$ with $k \/ N \\in S$ then $S$ is essentially the set $\\mathbb{Z}$ with respect to addition. The numbers $r_{i}$ belong to $S$ because $$ r_{1}=\\frac{3 p_{1} q_{2} q_{3}}{N}, \\quad r_{2}=\\frac{3 p_{2} q_{3} q_{1}}{N}, \\quad r_{3}=\\frac{3 p_{3} q_{1} q_{2}}{N} $$ The partition $\\mathbb{Q}=A \\cup B \\cup C$ of $\\mathbb{Q}$ induces a partition $S=A^{\\prime} \\cup B^{\\prime} \\cup C^{\\prime}$ of $S$, with $A^{\\prime}=A \\cap S$, $B^{\\prime}=B \\cap S, C^{\\prime}=C \\cap S$. Clearly $A^{\\prime}+B^{\\prime}, B^{\\prime}+C^{\\prime}, C^{\\prime}+A^{\\prime}$ are disjoint, so this partition has the properties we consider. By the uniqueness of the example for $\\mathbb{Z}$ the sets $A^{\\prime}, B^{\\prime}, C^{\\prime}$ are the congruence classes modulo 3 , multiplied by $1 \/ N$. Also all multiples of $3 \/ N$ are in the same set, $A^{\\prime}, B^{\\prime}$ or $C^{\\prime}$. This holds for $r_{1}, r_{2}, r_{3}$ in particular as they are all multiples of $3 \/ N$. However $r_{1}, r_{2}, r_{3}$ are in different sets $A^{\\prime}, B^{\\prime}, C^{\\prime}$ since they were chosen from different sets $A, B, C$. The contradiction ends the proof. Comment. The uniqueness of the example for $\\mathbb{Z}$ can also be deduced from the argument in the first solution.","tier":0} +{"problem_type":"Algebra","problem_label":"A3","problem":"Let $a_{2}, \\ldots, a_{n}$ be $n-1$ positive real numbers, where $n \\geq 3$, such that $a_{2} a_{3} \\cdots a_{n}=1$. Prove that $$ \\left(1+a_{2}\\right)^{2}\\left(1+a_{3}\\right)^{3} \\cdots\\left(1+a_{n}\\right)^{n}>n^{n} . $$","solution":"The substitution $a_{2}=\\frac{x_{2}}{x_{1}}, a_{3}=\\frac{x_{3}}{x_{2}}, \\ldots, a_{n}=\\frac{x_{1}}{x_{n-1}}$ transforms the original problem into the inequality $$ \\left(x_{1}+x_{2}\\right)^{2}\\left(x_{2}+x_{3}\\right)^{3} \\cdots\\left(x_{n-1}+x_{1}\\right)^{n}>n^{n} x_{1}^{2} x_{2}^{3} \\cdots x_{n-1}^{n} $$ for all $x_{1}, \\ldots, x_{n-1}>0$. To prove this, we use the AM-GM inequality for each factor of the left-hand side as follows: $$ \\begin{array}{rlcl} \\left(x_{1}+x_{2}\\right)^{2} & & & \\geq 2^{2} x_{1} x_{2} \\\\ \\left(x_{2}+x_{3}\\right)^{3} & = & \\left(2\\left(\\frac{x_{2}}{2}\\right)+x_{3}\\right)^{3} & \\geq 3^{3}\\left(\\frac{x_{2}}{2}\\right)^{2} x_{3} \\\\ \\left(x_{3}+x_{4}\\right)^{4} & = & \\left(3\\left(\\frac{x_{3}}{3}\\right)+x_{4}\\right)^{4} & \\geq 4^{4}\\left(\\frac{x_{3}}{3}\\right)^{3} x_{4} \\\\ & \\vdots & \\vdots & \\vdots \\end{array} $$ Multiplying these inequalities together gives $\\left({ }^{*}\\right)$, with inequality sign $\\geq$ instead of $>$. However for the equality to occur it is necessary that $x_{1}=x_{2}, x_{2}=2 x_{3}, \\ldots, x_{n-1}=(n-1) x_{1}$, implying $x_{1}=(n-1) ! x_{1}$. This is impossible since $x_{1}>0$ and $n \\geq 3$. Therefore the inequality is strict. Comment. One can avoid the substitution $a_{i}=x_{i} \/ x_{i-1}$. Apply the weighted AM-GM inequality to each factor $\\left(1+a_{k}\\right)^{k}$, with the same weights like above, to obtain $$ \\left(1+a_{k}\\right)^{k}=\\left((k-1) \\frac{1}{k-1}+a_{k}\\right)^{k} \\geq \\frac{k^{k}}{(k-1)^{k-1}} a_{k} $$ Multiplying all these inequalities together gives $$ \\left(1+a_{2}\\right)^{2}\\left(1+a_{3}\\right)^{3} \\cdots\\left(1+a_{n}\\right)^{n} \\geq n^{n} a_{2} a_{3} \\cdots a_{n}=n^{n} . $$ The same argument as in the proof above shows that the equality cannot be attained.","tier":0} +{"problem_type":"Algebra","problem_label":"A4","problem":"Let $f$ and $g$ be two nonzero polynomials with integer coefficients and $\\operatorname{deg} f>\\operatorname{deg} g$. Suppose that for infinitely many primes $p$ the polynomial $p f+g$ has a rational root. Prove that $f$ has a rational root.","solution":"Since $\\operatorname{deg} f>\\operatorname{deg} g$, we have $|g(x) \/ f(x)|<1$ for sufficiently large $x$; more precisely, there is a real number $R$ such that $|g(x) \/ f(x)|<1$ for all $x$ with $|x|>R$. Then for all such $x$ and all primes $p$ we have $$ |p f(x)+g(x)| \\geq|f(x)|\\left(p-\\frac{|g(x)|}{|f(x)|}\\right)>0 $$ Hence all real roots of the polynomials $p f+g$ lie in the interval $[-R, R]$. Let $f(x)=a_{n} x^{n}+a_{n-1} x^{n-1}+\\cdots+a_{0}$ and $g(x)=b_{m} x^{m}+b_{m-1} x^{m-1}+\\cdots+b_{0}$ where $n>m, a_{n} \\neq 0$ and $b_{m} \\neq 0$. Upon replacing $f(x)$ and $g(x)$ by $a_{n}^{n-1} f\\left(x \/ a_{n}\\right)$ and $a_{n}^{n-1} g\\left(x \/ a_{n}\\right)$ respectively, we reduce the problem to the case $a_{n}=1$. In other words one can assume that $f$ is monic. Then the leading coefficient of $p f+g$ is $p$, and if $r=u \/ v$ is a rational root of $p f+g$ with $(u, v)=1$ and $v>0$, then either $v=1$ or $v=p$. First consider the case when $v=1$ infinitely many times. If $v=1$ then $|u| \\leq R$, so there are only finitely many possibilities for the integer $u$. Therefore there exist distinct primes $p$ and $q$ for which we have the same value of $u$. Then the polynomials $p f+g$ and $q f+g$ share this root, implying $f(u)=g(u)=0$. So in this case $f$ and $g$ have an integer root in common. Now suppose that $v=p$ infinitely many times. By comparing the exponent of $p$ in the denominators of $p f(u \/ p)$ and $g(u \/ p)$ we get $m=n-1$ and $p f(u \/ p)+g(u \/ p)=0$ reduces to an equation of the form $$ \\left(u^{n}+a_{n-1} p u^{n-1}+\\ldots+a_{0} p^{n}\\right)+\\left(b_{n-1} u^{n-1}+b_{n-2} p u^{n-2}+\\ldots+b_{0} p^{n-1}\\right)=0 . $$ The equation above implies that $u^{n}+b_{n-1} u^{n-1}$ is divisible by $p$ and hence, since $(u, p)=1$, we have $u+b_{n-1}=p k$ with some integer $k$. On the other hand all roots of $p f+g$ lie in the interval $[-R, R]$, so that $$ \\begin{gathered} \\frac{\\left|p k-b_{n-1}\\right|}{p}=\\frac{|u|}{p}\\operatorname{deg} g$. Suppose that for infinitely many primes $p$ the polynomial $p f+g$ has a rational root. Prove that $f$ has a rational root.","solution":"Analogously to the first solution, there exists a real number $R$ such that the complex roots of all polynomials of the form $p f+g$ lie in the disk $|z| \\leq R$. For each prime $p$ such that $p f+g$ has a rational root, by GAUSs' lemma $p f+g$ is the product of two integer polynomials, one with degree 1 and the other with degree $\\operatorname{deg} f-1$. Since $p$ is a prime, the leading coefficient of one of these factors divides the leading coefficient of $f$. Denote that factor by $h_{p}$. By narrowing the set of the primes used we can assume that all polynomials $h_{p}$ have the same degree and the same leading coefficient. Their complex roots lie in the disk $|z| \\leq R$, hence VIETA's formulae imply that all coefficients of all polynomials $h_{p}$ form a bounded set. Since these coefficients are integers, there are only finitely many possible polynomials $h_{p}$. Hence there is a polynomial $h$ such that $h_{p}=h$ for infinitely many primes $p$. Finally, if $p$ and $q$ are distinct primes with $h_{p}=h_{q}=h$ then $h$ divides $(p-q) f$. Since $\\operatorname{deg} h=1$ or $\\operatorname{deg} h=\\operatorname{deg} f-1$, in both cases $f$ has a rational root. Comment. Clearly the polynomial $h$ is a common factor of $f$ and $g$. If $\\operatorname{deg} h=1$ then $f$ and $g$ share a rational root. Otherwise $\\operatorname{deg} h=\\operatorname{deg} f-1$ forces $\\operatorname{deg} g=\\operatorname{deg} f-1$ and $g$ divides $f$ over the rationals.","tier":0} +{"problem_type":"Algebra","problem_label":"A4","problem":"Let $f$ and $g$ be two nonzero polynomials with integer coefficients and $\\operatorname{deg} f>\\operatorname{deg} g$. Suppose that for infinitely many primes $p$ the polynomial $p f+g$ has a rational root. Prove that $f$ has a rational root.","solution":"Like in the first solution, there is a real number $R$ such that the real roots of all polynomials of the form $p f+g$ lie in the interval $[-R, R]$. Let $p_{1}5 \/ 4$ then $-x<5 \/ 4$ and so $g(2-x)-g(-x)=2$ by the above. On the other hand (3) gives $g(x)=2-g(2-x), g(x+2)=2-g(-x)$, so that $g(x+2)-g(x)=g(2-x)-g(-x)=2$. Thus (4) is true for all $x \\in \\mathbb{R}$. Now replace $x$ by $-x$ in (3) to obtain $g(-x)+g(2+x)=2$. In view of (4) this leads to $g(x)+g(-x)=0$, i. e. $g(-x)=-g(x)$ for all $x$. Taking this into account, we apply (1) to the pairs $(-x, y)$ and $(x,-y)$ : $g(1-x y)-g(-x+y)=(g(x)+1)(1-g(y)), \\quad g(1-x y)-g(x-y)=(1-g(x))(g(y)+1)$. Adding up yields $g(1-x y)=1-g(x) g(y)$. Then $g(1+x y)=1+g(x) g(y)$ by (3). Now the original equation (1) takes the form $g(x+y)=g(x)+g(y)$. Hence $g$ is additive. By additvity $g(1+x y)=g(1)+g(x y)=1+g(x y)$; since $g(1+x y)=1+g(x) g(y)$ was shown above, we also have $g(x y)=g(x) g(y)$ ( $g$ is multiplicative). In particular $y=x$ gives $g\\left(x^{2}\\right)=g(x)^{2} \\geq 0$ for all $x$, meaning that $g(x) \\geq 0$ for $x \\geq 0$. Since $g$ is additive and bounded from below on $[0,+\\infty)$, it is linear; more exactly $g(x)=g(1) x=x$ for all $x \\in \\mathbb{R}$. In summary $f(x)=x-1, x \\in \\mathbb{R}$. It is straightforward that this function satisfies the requirements. Comment. There are functions that satisfy the given equation but vanish at -1 , for instance the constant function 0 and $f(x)=x^{2}-1, x \\in \\mathbb{R}$.","tier":0} +{"problem_type":"Algebra","problem_label":"A6","problem":"Let $f: \\mathbb{N} \\rightarrow \\mathbb{N}$ be a function, and let $f^{m}$ be $f$ applied $m$ times. Suppose that for every $n \\in \\mathbb{N}$ there exists a $k \\in \\mathbb{N}$ such that $f^{2 k}(n)=n+k$, and let $k_{n}$ be the smallest such $k$. Prove that the sequence $k_{1}, k_{2}, \\ldots$ is unbounded.","solution":"We restrict attention to the set $$ S=\\left\\{1, f(1), f^{2}(1), \\ldots\\right\\} $$ Observe that $S$ is unbounded because for every number $n$ in $S$ there exists a $k>0$ such that $f^{2 k}(n)=n+k$ is in $S$. Clearly $f$ maps $S$ into itself; moreover $f$ is injective on $S$. Indeed if $f^{i}(1)=f^{j}(1)$ with $i \\neq j$ then the values $f^{m}(1)$ start repeating periodically from some point on, and $S$ would be finite. Define $g: S \\rightarrow S$ by $g(n)=f^{2 k_{n}}(n)=n+k_{n}$. We prove that $g$ is injective too. Suppose that $g(a)=g(b)$ with $ak_{b}$. So, since $f$ is injective on $S$, we obtain $$ f^{2\\left(k_{a}-k_{b}\\right)}(a)=b=a+\\left(k_{a}-k_{b}\\right) . $$ However this contradicts the minimality of $k_{a}$ as $0n$ for $n \\in S$, so $T$ is non-empty. For each $t \\in T$ denote $C_{t}=\\left\\{t, g(t), g^{2}(t), \\ldots\\right\\}$; call $C_{t}$ the chain starting at $t$. Observe that distinct chains are disjoint because $g$ is injective. Each $n \\in S \\backslash T$ has the form $n=g\\left(n^{\\prime}\\right)$ with $n^{\\prime}k$, i. e. $k_{n}>k$. In conclusion $k_{1}, k_{2}, \\ldots$ is unbounded.","tier":0} +{"problem_type":"Algebra","problem_label":"A7","problem":"We say that a function $f: \\mathbb{R}^{k} \\rightarrow \\mathbb{R}$ is a metapolynomial if, for some positive integers $m$ and $n$, it can be represented in the form $$ f\\left(x_{1}, \\ldots, x_{k}\\right)=\\max _{i=1, \\ldots, m} \\min _{j=1, \\ldots, n} P_{i, j}\\left(x_{1}, \\ldots, x_{k}\\right) $$ where $P_{i, j}$ are multivariate polynomials. Prove that the product of two metapolynomials is also a metapolynomial.","solution":"We use the notation $f(x)=f\\left(x_{1}, \\ldots, x_{k}\\right)$ for $x=\\left(x_{1}, \\ldots, x_{k}\\right)$ and $[m]=\\{1,2, \\ldots, m\\}$. Observe that if a metapolynomial $f(x)$ admits a representation like the one in the statement for certain positive integers $m$ and $n$, then they can be replaced by any $m^{\\prime} \\geq m$ and $n^{\\prime} \\geq n$. For instance, if we want to replace $m$ by $m+1$ then it is enough to define $P_{m+1, j}(x)=P_{m, j}(x)$ and note that repeating elements of a set do not change its maximum nor its minimum. So one can assume that any two metapolynomials are defined with the same $m$ and $n$. We reserve letters $P$ and $Q$ for polynomials, so every function called $P, P_{i, j}, Q, Q_{i, j}, \\ldots$ is a polynomial function. We start with a lemma that is useful to change expressions of the form $\\min \\max f_{i, j}$ to ones of the form $\\max \\min g_{i, j}$. Lemma. Let $\\left\\{a_{i, j}\\right\\}$ be real numbers, for all $i \\in[m]$ and $j \\in[n]$. Then $$ \\min _{i \\in[m]} \\max _{j \\in[n]} a_{i, j}=\\max _{j_{1}, \\ldots, j_{m} \\in[n]} \\min _{i \\in[m]} a_{i, j_{i}} $$ where the max in the right-hand side is over all vectors $\\left(j_{1}, \\ldots, j_{m}\\right)$ with $j_{1}, \\ldots, j_{m} \\in[n]$. Proof. We can assume for all $i$ that $a_{i, n}=\\max \\left\\{a_{i, 1}, \\ldots, a_{i, n}\\right\\}$ and $a_{m, n}=\\min \\left\\{a_{1, n}, \\ldots, a_{m, n}\\right\\}$. The left-hand side is $=a_{m, n}$ and hence we need to prove the same for the right-hand side. If $\\left(j_{1}, j_{2}, \\ldots, j_{m}\\right)=(n, n, \\ldots, n)$ then $\\min \\left\\{a_{1, j_{1}}, \\ldots, a_{m, j_{m}}\\right\\}=\\min \\left\\{a_{1, n}, \\ldots, a_{m, n}\\right\\}=a_{m, n}$ which implies that the right-hand side is $\\geq a_{m, n}$. It remains to prove the opposite inequality and this is equivalent to $\\min \\left\\{a_{1, j_{1}}, \\ldots, a_{m, j_{m}}\\right\\} \\leq a_{m, n}$ for all possible $\\left(j_{1}, j_{2}, \\ldots, j_{m}\\right)$. This is true because $\\min \\left\\{a_{1, j_{1}}, \\ldots, a_{m, j_{m}}\\right\\} \\leq a_{m, j_{m}} \\leq a_{m, n}$. We need to show that the family $\\mathcal{M}$ of metapolynomials is closed under multiplication, but it turns out easier to prove more: that it is also closed under addition, maxima and minima. First we prove the assertions about the maxima and the minima. If $f_{1}, \\ldots, f_{r}$ are metapolynomials, assume them defined with the same $m$ and $n$. Then $$ f=\\max \\left\\{f_{1}, \\ldots, f_{r}\\right\\}=\\max \\left\\{\\max _{i \\in[m]} \\min _{j \\in[n]} P_{i, j}^{1}, \\ldots, \\max _{i \\in[m]} \\min _{j \\in[n]} P_{i, j}^{r}\\right\\}=\\max _{s \\in[r], i \\in[m]} \\min _{j \\in[n]} P_{i, j}^{s} $$ It follows that $f=\\max \\left\\{f_{1}, \\ldots, f_{r}\\right\\}$ is a metapolynomial. The same argument works for the minima, but first we have to replace $\\min \\max$ by $\\max \\min$, and this is done via the lemma. Another property we need is that if $f=\\max \\min P_{i, j}$ is a metapolynomial then so is $-f$. Indeed, $-f=\\min \\left(-\\min P_{i, j}\\right)=\\min \\max P_{i, j}$. To prove $\\mathcal{M}$ is closed under addition let $f=\\max \\min P_{i, j}$ and $g=\\max \\min Q_{i, j}$. Then $$ \\begin{gathered} f(x)+g(x)=\\max _{i \\in[m]} \\min _{j \\in[n]} P_{i, j}(x)+\\max _{i \\in[m]} \\min _{j \\in[n]} Q_{i, j}(x) \\\\ =\\max _{i_{1}, i_{2} \\in[m]}\\left(\\min _{j \\in[n]} P_{i_{1}, j}(x)+\\min _{j \\in[n]} Q_{i_{2}, j}(x)\\right)=\\max _{i_{1}, i_{2} \\in[m]} \\min _{j_{1}, j_{2} \\in[n]}\\left(P_{i_{1}, j_{1}}(x)+Q_{i_{2}, j_{2}}(x)\\right), \\end{gathered} $$ and hence $f(x)+g(x)$ is a metapolynomial. We proved that $\\mathcal{M}$ is closed under sums, maxima and minima, in particular any function that can be expressed by sums, max, min, polynomials or even metapolynomials is in $\\mathcal{M}$. We would like to proceed with multiplication along the same lines like with addition, but there is an essential difference. In general the product of the maxima of two sets is not equal to the maximum of the product of the sets. We need to deal with the fact that $ay$ and $x$ is to the left of $y$, and replaces the pair $(x, y)$ by either $(y+1, x)$ or $(x-1, x)$. Prove that she can perform only finitely many such iterations.","solution":"Note first that the allowed operation does not change the maximum $M$ of the initial sequence. Let $a_{1}, a_{2}, \\ldots, a_{n}$ be the numbers obtained at some point of the process. Consider the sum $$ S=a_{1}+2 a_{2}+\\cdots+n a_{n} $$ We claim that $S$ increases by a positive integer amount with every operation. Let the operation replace the pair ( $\\left.a_{i}, a_{i+1}\\right)$ by a pair $\\left(c, a_{i}\\right)$, where $a_{i}>a_{i+1}$ and $c=a_{i+1}+1$ or $c=a_{i}-1$. Then the new and the old value of $S$ differ by $d=\\left(i c+(i+1) a_{i}\\right)-\\left(i a_{i}+(i+1) a_{i+1}\\right)=a_{i}-a_{i+1}+i\\left(c-a_{i+1}\\right)$. The integer $d$ is positive since $a_{i}-a_{i+1} \\geq 1$ and $c-a_{i+1} \\geq 0$. On the other hand $S \\leq(1+2+\\cdots+n) M$ as $a_{i} \\leq M$ for all $i=1, \\ldots, n$. Since $S$ increases by at least 1 at each step and never exceeds the constant $(1+2+\\cdots+n) M$, the process stops after a finite number of iterations.","tier":0} +{"problem_type":"Combinatorics","problem_label":"C1","problem":"Several positive integers are written in a row. Iteratively, Alice chooses two adjacent numbers $x$ and $y$ such that $x>y$ and $x$ is to the left of $y$, and replaces the pair $(x, y)$ by either $(y+1, x)$ or $(x-1, x)$. Prove that she can perform only finitely many such iterations.","solution":"Like in the first solution note that the operations do not change the maximum $M$ of the initial sequence. Now consider the reverse lexicographical order for $n$-tuples of integers. We say that $\\left(x_{1}, \\ldots, x_{n}\\right)<\\left(y_{1}, \\ldots, y_{n}\\right)$ if $x_{n}y$ and $x$ is to the left of $y$, and replaces the pair $(x, y)$ by either $(y+1, x)$ or $(x-1, x)$. Prove that she can perform only finitely many such iterations.","solution":"Let the current numbers be $a_{1}, a_{2}, \\ldots, a_{n}$. Define the score $s_{i}$ of $a_{i}$ as the number of $a_{j}$ 's that are less than $a_{i}$. Call the sequence $s_{1}, s_{2}, \\ldots, s_{n}$ the score sequence of $a_{1}, a_{2}, \\ldots, a_{n}$. Let us say that a sequence $x_{1}, \\ldots, x_{n}$ dominates a sequence $y_{1}, \\ldots, y_{n}$ if the first index $i$ with $x_{i} \\neq y_{i}$ is such that $x_{i}y$ and $y \\leq a \\leq x$, we see that $s_{i}$ decreases by at least 1 . This concludes the proof. Comment. All three proofs work if $x$ and $y$ are not necessarily adjacent, and if the pair $(x, y)$ is replaced by any pair $(a, x)$, with $a$ an integer satisfying $y \\leq a \\leq x$. There is nothing special about the \"weights\" $1,2, \\ldots, n$ in the definition of $S=\\sum_{i=1}^{n} i a_{i}$ from the first solution. For any sequence $w_{1}2^{k}$ we show how Ben can find a number $y \\in T$ that is different from $x$. By performing this step repeatedly he can reduce $T$ to be of size $2^{k} \\leq n$ and thus win. Since only the size $m>2^{k}$ of $T$ is relevant, assume that $T=\\left\\{0,1, \\ldots, 2^{k}, \\ldots, m-1\\right\\}$. Ben begins by asking repeatedly whether $x$ is $2^{k}$. If Amy answers no $k+1$ times in a row, one of these answers is truthful, and so $x \\neq 2^{k}$. Otherwise Ben stops asking about $2^{k}$ at the first answer yes. He then asks, for each $i=1, \\ldots, k$, if the binary representation of $x$ has a 0 in the $i$ th digit. Regardless of what the $k$ answers are, they are all inconsistent with a certain number $y \\in\\left\\{0,1, \\ldots, 2^{k}-1\\right\\}$. The preceding answer yes about $2^{k}$ is also inconsistent with $y$. Hence $y \\neq x$. Otherwise the last $k+1$ answers are not truthful, which is impossible. Either way, Ben finds a number in $T$ that is different from $x$, and the claim is proven. b) We prove that if $1<\\lambda<2$ and $n=\\left\\lfloor(2-\\lambda) \\lambda^{k+1}\\right\\rfloor-1$ then Ben cannot guarantee a win. To complete the proof, then it suffices to take $\\lambda$ such that $1.99<\\lambda<2$ and $k$ large enough so that $$ n=\\left\\lfloor(2-\\lambda) \\lambda^{k+1}\\right\\rfloor-1 \\geq 1.99^{k} $$ Consider the following strategy for Amy. First she chooses $N=n+1$ and $x \\in\\{1,2, \\ldots, n+1\\}$ arbitrarily. After every answer of hers Amy determines, for each $i=1,2, \\ldots, n+1$, the number $m_{i}$ of consecutive answers she has given by that point that are inconsistent with $i$. To decide on her next answer, she then uses the quantity $$ \\phi=\\sum_{i=1}^{n+1} \\lambda^{m_{i}} $$ No matter what Ben's next question is, Amy chooses the answer which minimizes $\\phi$. We claim that with this strategy $\\phi$ will always stay less than $\\lambda^{k+1}$. Consequently no exponent $m_{i}$ in $\\phi$ will ever exceed $k$, hence Amy will never give more than $k$ consecutive answers inconsistent with some $i$. In particular this applies to the target number $x$, so she will never lie more than $k$ times in a row. Thus, given the claim, Amy's strategy is legal. Since the strategy does not depend on $x$ in any way, Ben can make no deductions about $x$, and therefore he cannot guarantee a win. It remains to show that $\\phi<\\lambda^{k+1}$ at all times. Initially each $m_{i}$ is 0 , so this condition holds in the beginning due to $1<\\lambda<2$ and $n=\\left\\lfloor(2-\\lambda) \\lambda^{k+1}\\right\\rfloor-1$. Suppose that $\\phi<\\lambda^{k+1}$ at some point, and Ben has just asked if $x \\in S$ for some set $S$. According as Amy answers yes or no, the new value of $\\phi$ becomes $$ \\phi_{1}=\\sum_{i \\in S} 1+\\sum_{i \\notin S} \\lambda^{m_{i}+1} \\quad \\text { or } \\quad \\phi_{2}=\\sum_{i \\in S} \\lambda^{m_{i}+1}+\\sum_{i \\notin S} 1 $$ Since Amy chooses the option minimizing $\\phi$, the new $\\phi$ will equal $\\min \\left(\\phi_{1}, \\phi_{2}\\right)$. Now we have $$ \\min \\left(\\phi_{1}, \\phi_{2}\\right) \\leq \\frac{1}{2}\\left(\\phi_{1}+\\phi_{2}\\right)=\\frac{1}{2}\\left(\\sum_{i \\in S}\\left(1+\\lambda^{m_{i}+1}\\right)+\\sum_{i \\notin S}\\left(\\lambda^{m_{i}+1}+1\\right)\\right)=\\frac{1}{2}(\\lambda \\phi+n+1) $$ Because $\\phi<\\lambda^{k+1}$, the assumptions $\\lambda<2$ and $n=\\left\\lfloor(2-\\lambda) \\lambda^{k+1}\\right\\rfloor-1$ lead to $$ \\min \\left(\\phi_{1}, \\phi_{2}\\right)<\\frac{1}{2}\\left(\\lambda^{k+2}+(2-\\lambda) \\lambda^{k+1}\\right)=\\lambda^{k+1} $$ The claim follows, which completes the solution. Comment. Given a fixed $k$, let $f(k)$ denote the minimum value of $n$ for which Ben can guarantee a victory. The problem asks for a proof that for large $k$ $$ 1.99^{k} \\leq f(k) \\leq 2^{k} $$ A computer search shows that $f(k)=2,3,4,7,11,17$ for $k=1,2,3,4,5,6$.","tier":0} +{"problem_type":"Combinatorics","problem_label":"C7","problem":"There are given $2^{500}$ points on a circle labeled $1,2, \\ldots, 2^{500}$ in some order. Prove that one can choose 100 pairwise disjoint chords joining some of these points so that the 100 sums of the pairs of numbers at the endpoints of the chosen chords are equal.","solution":"The proof is based on the following general fact. Lemma. In a graph $G$ each vertex $v$ has degree $d_{v}$. Then $G$ contains an independent set $S$ of vertices such that $|S| \\geq f(G)$ where $$ f(G)=\\sum_{v \\in G} \\frac{1}{d_{v}+1} $$ Proof. Induction on $n=|G|$. The base $n=1$ is clear. For the inductive step choose a vertex $v_{0}$ in $G$ of minimum degree $d$. Delete $v_{0}$ and all of its neighbors $v_{1}, \\ldots, v_{d}$ and also all edges with endpoints $v_{0}, v_{1}, \\ldots, v_{d}$. This gives a new graph $G^{\\prime}$. By the inductive assumption $G^{\\prime}$ contains an independent set $S^{\\prime}$ of vertices such that $\\left|S^{\\prime}\\right| \\geq f\\left(G^{\\prime}\\right)$. Since no vertex in $S^{\\prime}$ is a neighbor of $v_{0}$ in $G$, the set $S=S^{\\prime} \\cup\\left\\{v_{0}\\right\\}$ is independent in $G$. Let $d_{v}^{\\prime}$ be the degree of a vertex $v$ in $G^{\\prime}$. Clearly $d_{v}^{\\prime} \\leq d_{v}$ for every such vertex $v$, and also $d_{v_{i}} \\geq d$ for all $i=0,1, \\ldots, d$ by the minimal choice of $v_{0}$. Therefore $$ f\\left(G^{\\prime}\\right)=\\sum_{v \\in G^{\\prime}} \\frac{1}{d_{v}^{\\prime}+1} \\geq \\sum_{v \\in G^{\\prime}} \\frac{1}{d_{v}+1}=f(G)-\\sum_{i=0}^{d} \\frac{1}{d_{v_{i}}+1} \\geq f(G)-\\frac{d+1}{d+1}=f(G)-1 . $$ Hence $|S|=\\left|S^{\\prime}\\right|+1 \\geq f\\left(G^{\\prime}\\right)+1 \\geq f(G)$, and the induction is complete. We pass on to our problem. For clarity denote $n=2^{499}$ and draw all chords determined by the given $2 n$ points. Color each chord with one of the colors $3,4, \\ldots, 4 n-1$ according to the sum of the numbers at its endpoints. Chords with a common endpoint have different colors. For each color $c$ consider the following graph $G_{c}$. Its vertices are the chords of color $c$, and two chords are neighbors in $G_{c}$ if they intersect. Let $f\\left(G_{c}\\right)$ have the same meaning as in the lemma for all graphs $G_{c}$. Every chord $\\ell$ divides the circle into two arcs, and one of them contains $m(\\ell) \\leq n-1$ given points. (In particular $m(\\ell)=0$ if $\\ell$ joins two consecutive points.) For each $i=0,1, \\ldots, n-2$ there are $2 n$ chords $\\ell$ with $m(\\ell)=i$. Such a chord has degree at most $i$ in the respective graph. Indeed let $A_{1}, \\ldots, A_{i}$ be all points on either arc determined by a chord $\\ell$ with $m(\\ell)=i$ and color $c$. Every $A_{j}$ is an endpoint of at most 1 chord colored $c, j=1, \\ldots, i$. Hence at most $i$ chords of color $c$ intersect $\\ell$. It follows that for each $i=0,1, \\ldots, n-2$ the $2 n$ chords $\\ell$ with $m(\\ell)=i$ contribute at least $\\frac{2 n}{i+1}$ to the sum $\\sum_{c} f\\left(G_{c}\\right)$. Summation over $i=0,1, \\ldots, n-2$ gives $$ \\sum_{c} f\\left(G_{c}\\right) \\geq 2 n \\sum_{i=1}^{n-1} \\frac{1}{i} $$ Because there are $4 n-3$ colors in all, averaging yields a color $c$ such that $$ f\\left(G_{c}\\right) \\geq \\frac{2 n}{4 n-3} \\sum_{i=1}^{n-1} \\frac{1}{i}>\\frac{1}{2} \\sum_{i=1}^{n-1} \\frac{1}{i} $$ By the lemma there are at least $\\frac{1}{2} \\sum_{i=1}^{n-1} \\frac{1}{i}$ pairwise disjoint chords of color $c$, i. e. with the same sum $c$ of the pairs of numbers at their endpoints. It remains to show that $\\frac{1}{2} \\sum_{i=1}^{n-1} \\frac{1}{i} \\geq 100$ for $n=2^{499}$. Indeed we have $$ \\sum_{i=1}^{n-1} \\frac{1}{i}>\\sum_{i=1}^{2^{400}} \\frac{1}{i}=1+\\sum_{k=1}^{400} \\sum_{i=2^{k-1+1}}^{2^{k}} \\frac{1}{i}>1+\\sum_{k=1}^{400} \\frac{2^{k-1}}{2^{k}}=201>200 $$ This completes the solution.","tier":0} +{"problem_type":"Geometry","problem_label":"G1","problem":"In the triangle $A B C$ the point $J$ is the center of the excircle opposite to $A$. This excircle is tangent to the side $B C$ at $M$, and to the lines $A B$ and $A C$ at $K$ and $L$ respectively. The lines $L M$ and $B J$ meet at $F$, and the lines $K M$ and $C J$ meet at $G$. Let $S$ be the point of intersection of the lines $A F$ and $B C$, and let $T$ be the point of intersection of the lines $A G$ and $B C$. Prove that $M$ is the midpoint of $S T$.","solution":"Let $\\alpha=\\angle C A B, \\beta=\\angle A B C$ and $\\gamma=\\angle B C A$. The line $A J$ is the bisector of $\\angle C A B$, so $\\angle J A K=\\angle J A L=\\frac{\\alpha}{2}$. By $\\angle A K J=\\angle A L J=90^{\\circ}$ the points $K$ and $L$ lie on the circle $\\omega$ with diameter $A J$. The triangle $K B M$ is isosceles as $B K$ and $B M$ are tangents to the excircle. Since $B J$ is the bisector of $\\angle K B M$, we have $\\angle M B J=90^{\\circ}-\\frac{\\beta}{2}$ and $\\angle B M K=\\frac{\\beta}{2}$. Likewise $\\angle M C J=90^{\\circ}-\\frac{\\gamma}{2}$ and $\\angle C M L=\\frac{\\gamma}{2}$. Also $\\angle B M F=\\angle C M L$, therefore $$ \\angle L F J=\\angle M B J-\\angle B M F=\\left(90^{\\circ}-\\frac{\\beta}{2}\\right)-\\frac{\\gamma}{2}=\\frac{\\alpha}{2}=\\angle L A J . $$ Hence $F$ lies on the circle $\\omega$. (By the angle computation, $F$ and $A$ are on the same side of $B C$.) Analogously, $G$ also lies on $\\omega$. Since $A J$ is a diameter of $\\omega$, we obtain $\\angle A F J=\\angle A G J=90^{\\circ}$. ![](https:\/\/cdn.mathpix.com\/cropped\/2024_04_17_93f7104904c3717a5058g-29.jpg?height=703&width=1242&top_left_y=1276&top_left_x=407) The lines $A B$ and $B C$ are symmetric with respect to the external bisector $B F$. Because $A F \\perp B F$ and $K M \\perp B F$, the segments $S M$ and $A K$ are symmetric with respect to $B F$, hence $S M=A K$. By symmetry $T M=A L$. Since $A K$ and $A L$ are equal as tangents to the excircle, it follows that $S M=T M$, and the proof is complete. Comment. After discovering the circle $A F K J L G$, there are many other ways to complete the solution. For instance, from the cyclic quadrilaterals $J M F S$ and $J M G T$ one can find $\\angle T S J=\\angle S T J=\\frac{\\alpha}{2}$. Another possibility is to use the fact that the lines $A S$ and $G M$ are parallel (both are perpendicular to the external angle bisector $B J$ ), so $\\frac{M S}{M T}=\\frac{A G}{G T}=1$.","tier":0} +{"problem_type":"Geometry","problem_label":"G2","problem":"Let $A B C D$ be a cyclic quadrilateral whose diagonals $A C$ and $B D$ meet at $E$. The extensions of the sides $A D$ and $B C$ beyond $A$ and $B$ meet at $F$. Let $G$ be the point such that $E C G D$ is a parallelogram, and let $H$ be the image of $E$ under reflection in $A D$. Prove that $D, H, F, G$ are concyclic.","solution":"We show first that the triangles $F D G$ and $F B E$ are similar. Since $A B C D$ is cyclic, the triangles $E A B$ and $E D C$ are similar, as well as $F A B$ and $F C D$. The parallelogram $E C G D$ yields $G D=E C$ and $\\angle C D G=\\angle D C E$; also $\\angle D C E=\\angle D C A=\\angle D B A$ by inscribed angles. Therefore $$ \\begin{gathered} \\angle F D G=\\angle F D C+\\angle C D G=\\angle F B A+\\angle A B D=\\angle F B E, \\\\ \\frac{G D}{E B}=\\frac{C E}{E B}=\\frac{C D}{A B}=\\frac{F D}{F B} . \\end{gathered} $$ It follows that $F D G$ and $F B E$ are similar, and so $\\angle F G D=\\angle F E B$. ![](https:\/\/cdn.mathpix.com\/cropped\/2024_04_17_93f7104904c3717a5058g-30.jpg?height=954&width=974&top_left_y=948&top_left_x=538) Since $H$ is the reflection of $E$ with respect to $F D$, we conclude that $$ \\angle F H D=\\angle F E D=180^{\\circ}-\\angle F E B=180^{\\circ}-\\angle F G D . $$ This proves that $D, H, F, G$ are concyclic. Comment. Points $E$ and $G$ are always in the half-plane determined by the line $F D$ that contains $B$ and $C$, but $H$ is always in the other half-plane. In particular, $D H F G$ is cyclic if and only if $\\angle F H D+\\angle F G D=180^{\\circ}$.","tier":0} +{"problem_type":"Geometry","problem_label":"G3","problem":"In an acute triangle $A B C$ the points $D, E$ and $F$ are the feet of the altitudes through $A$, $B$ and $C$ respectively. The incenters of the triangles $A E F$ and $B D F$ are $I_{1}$ and $I_{2}$ respectively; the circumcenters of the triangles $A C I_{1}$ and $B C I_{2}$ are $O_{1}$ and $O_{2}$ respectively. Prove that $I_{1} I_{2}$ and $O_{1} O_{2}$ are parallel.","solution":"Let $\\angle C A B=\\alpha, \\angle A B C=\\beta, \\angle B C A=\\gamma$. We start by showing that $A, B, I_{1}$ and $I_{2}$ are concyclic. Since $A I_{1}$ and $B I_{2}$ bisect $\\angle C A B$ and $\\angle A B C$, their extensions beyond $I_{1}$ and $I_{2}$ meet at the incenter $I$ of the triangle. The points $E$ and $F$ are on the circle with diameter $B C$, so $\\angle A E F=\\angle A B C$ and $\\angle A F E=\\angle A C B$. Hence the triangles $A E F$ and $A B C$ are similar with ratio of similitude $\\frac{A E}{A B}=\\cos \\alpha$. Because $I_{1}$ and $I$ are their incenters, we obtain $I_{1} A=I A \\cos \\alpha$ and $I I_{1}=I A-I_{1} A=2 I A \\sin ^{2} \\frac{\\alpha}{2}$. By symmetry $I I_{2}=2 I B \\sin ^{2} \\frac{\\beta}{2}$. The law of sines in the triangle $A B I$ gives $I A \\sin \\frac{\\alpha}{2}=I B \\sin \\frac{\\beta}{2}$. Hence $$ I I_{1} \\cdot I A=2\\left(I A \\sin \\frac{\\alpha}{2}\\right)^{2}=2\\left(I B \\sin \\frac{\\beta}{2}\\right)^{2}=I I_{2} \\cdot I B $$ Therefore $A, B, I_{1}$ and $I_{2}$ are concyclic, as claimed. ![](https:\/\/cdn.mathpix.com\/cropped\/2024_04_17_93f7104904c3717a5058g-31.jpg?height=848&width=1466&top_left_y=998&top_left_x=295) In addition $I I_{1} \\cdot I A=I I_{2} \\cdot I B$ implies that $I$ has the same power with respect to the circles $\\left(A C I_{1}\\right),\\left(B C I_{2}\\right)$ and $\\left(A B I_{1} I_{2}\\right)$. Then $C I$ is the radical axis of $\\left(A C I_{1}\\right)$ and $\\left(B C I_{2}\\right)$; in particular $C I$ is perpendicular to the line of centers $O_{1} O_{2}$. Now it suffices to prove that $C I \\perp I_{1} I_{2}$. Let $C I$ meet $I_{1} I_{2}$ at $Q$, then it is enough to check that $\\angle I I_{1} Q+\\angle I_{1} I Q=90^{\\circ}$. Since $\\angle I_{1} I Q$ is external for the triangle $A C I$, we have $$ \\angle I I_{1} Q+\\angle I_{1} I Q=\\angle I I_{1} Q+(\\angle A C I+\\angle C A I)=\\angle I I_{1} I_{2}+\\angle A C I+\\angle C A I . $$ It remains to note that $\\angle I I_{1} I_{2}=\\frac{\\beta}{2}$ from the cyclic quadrilateral $A B I_{1} I_{2}$, and $\\angle A C I=\\frac{\\gamma}{2}$, $\\angle C A I=\\frac{\\alpha}{2}$. Therefore $\\angle I I_{1} Q+\\angle I_{1} I Q=\\frac{\\alpha}{2}+\\frac{\\beta}{2}+\\frac{\\gamma}{2}=90^{\\circ}$, completing the proof. Comment. It follows from the first part of the solution that the common point $I_{3} \\neq C$ of the circles $\\left(A C I_{1}\\right)$ and $\\left(B C I_{2}\\right)$ is the incenter of the triangle $C D E$.","tier":0} +{"problem_type":"Geometry","problem_label":"G4","problem":"Let $A B C$ be a triangle with $A B \\neq A C$ and circumcenter $O$. The bisector of $\\angle B A C$ intersects $B C$ at $D$. Let $E$ be the reflection of $D$ with respect to the midpoint of $B C$. The lines through $D$ and $E$ perpendicular to $B C$ intersect the lines $A O$ and $A D$ at $X$ and $Y$ respectively. Prove that the quadrilateral $B X C Y$ is cyclic.","solution":"The bisector of $\\angle B A C$ and the perpendicular bisector of $B C$ meet at $P$, the midpoint of the minor arc $\\widehat{B C}$ (they are different lines as $A B \\neq A C$ ). In particular $O P$ is perpendicular to $B C$ and intersects it at $M$, the midpoint of $B C$. Denote by $Y^{\\prime}$ the reflexion of $Y$ with respect to $O P$. Since $\\angle B Y C=\\angle B Y^{\\prime} C$, it suffices to prove that $B X C Y^{\\prime}$ is cyclic. ![](https:\/\/cdn.mathpix.com\/cropped\/2024_04_17_93f7104904c3717a5058g-32.jpg?height=1039&width=826&top_left_y=720&top_left_x=615) We have $$ \\angle X A P=\\angle O P A=\\angle E Y P \\text {. } $$ The first equality holds because $O A=O P$, and the second one because $E Y$ and $O P$ are both perpendicular to $B C$ and hence parallel. But $\\left\\{Y, Y^{\\prime}\\right\\}$ and $\\{E, D\\}$ are pairs of symmetric points with respect to $O P$, it follows that $\\angle E Y P=\\angle D Y^{\\prime} P$ and hence $$ \\angle X A P=\\angle D Y^{\\prime} P=\\angle X Y^{\\prime} P . $$ The last equation implies that $X A Y^{\\prime} P$ is cyclic. By the powers of $D$ with respect to the circles $\\left(X A Y^{\\prime} P\\right)$ and $(A B P C)$ we obtain $$ X D \\cdot D Y^{\\prime}=A D \\cdot D P=B D \\cdot D C $$ It follows that $B X C Y^{\\prime}$ is cyclic, as desired.","tier":0} +{"problem_type":"Geometry","problem_label":"G5","problem":"Let $A B C$ be a triangle with $\\angle B C A=90^{\\circ}$, and let $C_{0}$ be the foot of the altitude from $C$. Choose a point $X$ in the interior of the segment $C C_{0}$, and let $K, L$ be the points on the segments $A X, B X$ for which $B K=B C$ and $A L=A C$ respectively. Denote by $M$ the intersection of $A L$ and $B K$. Show that $M K=M L$.","solution":"Let $C^{\\prime}$ be the reflection of $C$ in the line $A B$, and let $\\omega_{1}$ and $\\omega_{2}$ be the circles with centers $A$ and $B$, passing through $L$ and $K$ respectively. Since $A C^{\\prime}=A C=A L$ and $B C^{\\prime}=B C=B K$, both $\\omega_{1}$ and $\\omega_{2}$ pass through $C$ and $C^{\\prime}$. By $\\angle B C A=90^{\\circ}, A C$ is tangent to $\\omega_{2}$ at $C$, and $B C$ is tangent to $\\omega_{1}$ at $C$. Let $K_{1} \\neq K$ be the second intersection of $A X$ and $\\omega_{2}$, and let $L_{1} \\neq L$ be the second intersection of $B X$ and $\\omega_{1}$. ![](https:\/\/cdn.mathpix.com\/cropped\/2024_04_17_93f7104904c3717a5058g-33.jpg?height=1051&width=1016&top_left_y=688&top_left_x=520) By the powers of $X$ with respect to $\\omega_{2}$ and $\\omega_{1}$, $$ X K \\cdot X K_{1}=X C \\cdot X C^{\\prime}=X L \\cdot X L_{1}, $$ so the points $K_{1}, L, K, L_{1}$ lie on a circle $\\omega_{3}$. The power of $A$ with respect to $\\omega_{2}$ gives $$ A L^{2}=A C^{2}=A K \\cdot A K_{1}, $$ indicating that $A L$ is tangent to $\\omega_{3}$ at $L$. Analogously, $B K$ is tangent to $\\omega_{3}$ at $K$. Hence $M K$ and $M L$ are the two tangents from $M$ to $\\omega_{3}$ and therefore $M K=M L$.","tier":0} +{"problem_type":"Geometry","problem_label":"G6","problem":"Let $A B C$ be a triangle with circumcenter $O$ and incenter $I$. The points $D, E$ and $F$ on the sides $B C, C A$ and $A B$ respectively are such that $B D+B F=C A$ and $C D+C E=A B$. The circumcircles of the triangles $B F D$ and $C D E$ intersect at $P \\neq D$. Prove that $O P=O I$.","solution":"By Miquel's theorem the circles $(A E F)=\\omega_{A},(B F D)=\\omega_{B}$ and $(C D E)=\\omega_{C}$ have a common point, for arbitrary points $D, E$ and $F$ on $B C, C A$ and $A B$. So $\\omega_{A}$ passes through the common point $P \\neq D$ of $\\omega_{B}$ and $\\omega_{C}$. Let $\\omega_{A}, \\omega_{B}$ and $\\omega_{C}$ meet the bisectors $A I, B I$ and $C I$ at $A \\neq A^{\\prime}, B \\neq B^{\\prime}$ and $C \\neq C^{\\prime}$ respectively. The key observation is that $A^{\\prime}, B^{\\prime}$ and $C^{\\prime}$ do not depend on the particular choice of $D, E$ and $F$, provided that $B D+B F=C A, C D+C E=A B$ and $A E+A F=B C$ hold true (the last equality follows from the other two). For a proof we need the following fact. Lemma. Given is an angle with vertex $A$ and measure $\\alpha$. A circle $\\omega$ through $A$ intersects the angle bisector at $L$ and sides of the angle at $X$ and $Y$. Then $A X+A Y=2 A L \\cos \\frac{\\alpha}{2}$. Proof. Note that $L$ is the midpoint of $\\operatorname{arc} \\widehat{X L Y}$ in $\\omega$ and set $X L=Y L=u, X Y=v$. By PtOLEMY's theorem $A X \\cdot Y L+A Y \\cdot X L=A L \\cdot X Y$, which rewrites as $(A X+A Y) u=A L \\cdot v$. Since $\\angle L X Y=\\frac{\\alpha}{2}$ and $\\angle X L Y=180^{\\circ}-\\alpha$, we have $v=2 \\cos \\frac{\\alpha}{2} u$ by the law of sines, and the claim follows. ![](https:\/\/cdn.mathpix.com\/cropped\/2024_04_17_93f7104904c3717a5058g-34.jpg?height=551&width=671&top_left_y=1118&top_left_x=698) Apply the lemma to $\\angle B A C=\\alpha$ and the circle $\\omega=\\omega_{A}$, which intersects $A I$ at $A^{\\prime}$. This gives $2 A A^{\\prime} \\cos \\frac{\\alpha}{2}=A E+A F=B C$; by symmetry analogous relations hold for $B B^{\\prime}$ and $C C^{\\prime}$. It follows that $A^{\\prime}, B^{\\prime}$ and $C^{\\prime}$ are independent of the choice of $D, E$ and $F$, as stated. We use the lemma two more times with $\\angle B A C=\\alpha$. Let $\\omega$ be the circle with diameter $A I$. Then $X$ and $Y$ are the tangency points of the incircle of $A B C$ with $A B$ and $A C$, and hence $A X=A Y=\\frac{1}{2}(A B+A C-B C)$. So the lemma yields $2 A I \\cos \\frac{\\alpha}{2}=A B+A C-B C$. Next, if $\\omega$ is the circumcircle of $A B C$ and $A I$ intersects $\\omega$ at $M \\neq A$ then $\\{X, Y\\}=\\{B, C\\}$, and so $2 A M \\cos \\frac{\\alpha}{2}=A B+A C$ by the lemma. To summarize, $$ 2 A A^{\\prime} \\cos \\frac{\\alpha}{2}=B C, \\quad 2 A I \\cos \\frac{\\alpha}{2}=A B+A C-B C, \\quad 2 A M \\cos \\frac{\\alpha}{2}=A B+A C $$ These equalities imply $A A^{\\prime}+A I=A M$, hence the segments $A M$ and $I A^{\\prime}$ have a common midpoint. It follows that $I$ and $A^{\\prime}$ are equidistant from the circumcenter $O$. By symmetry $O I=O A^{\\prime}=O B^{\\prime}=O C^{\\prime}$, so $I, A^{\\prime}, B^{\\prime}, C^{\\prime}$ are on a circle centered at $O$. To prove $O P=O I$, now it suffices to show that $I, A^{\\prime}, B^{\\prime}, C^{\\prime}$ and $P$ are concyclic. Clearly one can assume $P \\neq I, A^{\\prime}, B^{\\prime}, C^{\\prime}$. We use oriented angles to avoid heavy case distinction. The oriented angle between the lines $l$ and $m$ is denoted by $\\angle(l, m)$. We have $\\angle(l, m)=-\\angle(m, l)$ and $\\angle(l, m)+\\angle(m, n)=\\angle(l, n)$ for arbitrary lines $l, m$ and $n$. Four distinct non-collinear points $U, V, X, Y$ are concyclic if and only if $\\angle(U X, V X)=\\angle(U Y, V Y)$. ![](https:\/\/cdn.mathpix.com\/cropped\/2024_04_17_93f7104904c3717a5058g-35.jpg?height=1048&width=1059&top_left_y=184&top_left_x=493) Suppose for the moment that $A^{\\prime}, B^{\\prime}, P, I$ are distinct and noncollinear; then it is enough to check the equality $\\angle\\left(A^{\\prime} P, B^{\\prime} P\\right)=\\angle\\left(A^{\\prime} I, B^{\\prime} I\\right)$. Because $A, F, P, A^{\\prime}$ are on the circle $\\omega_{A}$, we have $\\angle\\left(A^{\\prime} P, F P\\right)=\\angle\\left(A^{\\prime} A, F A\\right)=\\angle\\left(A^{\\prime} I, A B\\right)$. Likewise $\\angle\\left(B^{\\prime} P, F P\\right)=\\angle\\left(B^{\\prime} I, A B\\right)$. Therefore $$ \\angle\\left(A^{\\prime} P, B^{\\prime} P\\right)=\\angle\\left(A^{\\prime} P, F P\\right)+\\angle\\left(F P, B^{\\prime} P\\right)=\\angle\\left(A^{\\prime} I, A B\\right)-\\angle\\left(B^{\\prime} I, A B\\right)=\\angle\\left(A^{\\prime} I, B^{\\prime} I\\right) \\text {. } $$ Here we assumed that $P \\neq F$. If $P=F$ then $P \\neq D, E$ and the conclusion follows similarly (use $\\angle\\left(A^{\\prime} F, B^{\\prime} F\\right)=\\angle\\left(A^{\\prime} F, E F\\right)+\\angle(E F, D F)+\\angle\\left(D F, B^{\\prime} F\\right)$ and inscribed angles in $\\left.\\omega_{A}, \\omega_{B}, \\omega_{C}\\right)$. There is no loss of generality in assuming $A^{\\prime}, B^{\\prime}, P, I$ distinct and noncollinear. If $A B C$ is an equilateral triangle then the equalities $\\left(^{*}\\right)$ imply that $A^{\\prime}, B^{\\prime}, C^{\\prime}, I, O$ and $P$ coincide, so $O P=O I$. Otherwise at most one of $A^{\\prime}, B^{\\prime}, C^{\\prime}$ coincides with $I$. If say $C^{\\prime}=I$ then $O I \\perp C I$ by the previous reasoning. It follows that $A^{\\prime}, B^{\\prime} \\neq I$ and hence $A^{\\prime} \\neq B^{\\prime}$. Finally $A^{\\prime}, B^{\\prime}$ and $I$ are noncollinear because $I, A^{\\prime}, B^{\\prime}, C^{\\prime}$ are concyclic. Comment. The proposer remarks that the locus $\\gamma$ of the points $P$ is an arc of the circle $\\left(A^{\\prime} B^{\\prime} C^{\\prime} I\\right)$. The reflection $I^{\\prime}$ of $I$ in $O$ belongs to $\\gamma$; it is obtained by choosing $D, E$ and $F$ to be the tangency points of the three excircles with their respective sides. The rest of the circle $\\left(A^{\\prime} B^{\\prime} C^{\\prime} I\\right)$, except $I$, can be included in $\\gamma$ by letting $D, E$ and $F$ vary on the extensions of the sides and assuming signed lengths. For instance if $B$ is between $C$ and $D$ then the length $B D$ must be taken with a negative sign. The incenter $I$ corresponds to the limit case where $D$ tends to infinity.","tier":0} +{"problem_type":"Geometry","problem_label":"G7","problem":"Let $A B C D$ be a convex quadrilateral with non-parallel sides $B C$ and $A D$. Assume that there is a point $E$ on the side $B C$ such that the quadrilaterals $A B E D$ and $A E C D$ are circumscribed. Prove that there is a point $F$ on the side $A D$ such that the quadrilaterals $A B C F$ and $B C D F$ are circumscribed if and only if $A B$ is parallel to $C D$.","solution":"Let $\\omega_{1}$ and $\\omega_{2}$ be the incircles and $O_{1}$ and $O_{2}$ the incenters of the quadrilaterals $A B E D$ and $A E C D$ respectively. A point $F$ with the stated property exists only if $\\omega_{1}$ and $\\omega_{2}$ are also the incircles of the quadrilaterals $A B C F$ and $B C D F$. ![](https:\/\/cdn.mathpix.com\/cropped\/2024_04_17_93f7104904c3717a5058g-36.jpg?height=494&width=868&top_left_y=593&top_left_x=591) Let the tangents from $B$ to $\\omega_{2}$ and from $C$ to $\\omega_{1}$ (other than $B C$ ) meet $A D$ at $F_{1}$ and $F_{2}$ respectively. We need to prove that $F_{1}=F_{2}$ if and only if $A B \\| C D$. Lemma. The circles $\\omega_{1}$ and $\\omega_{2}$ with centers $O_{1}$ and $O_{2}$ are inscribed in an angle with vertex $O$. The points $P, S$ on one side of the angle and $Q, R$ on the other side are such that $\\omega_{1}$ is the incircle of the triangle $P Q O$, and $\\omega_{2}$ is the excircle of the triangle $R S O$ opposite to $O$. Denote $p=O O_{1} \\cdot O O_{2}$. Then exactly one of the following relations holds: $$ O P \\cdot O Rp>O Q \\cdot O S, \\quad O P \\cdot O R=p=O Q \\cdot O S $$ Proof. Denote $\\angle O P O_{1}=u, \\angle O Q O_{1}=v, \\angle O O_{2} R=x, \\angle O O_{2} S=y, \\angle P O Q=2 \\varphi$. Because $P O_{1}, Q O_{1}, R O_{2}, S O_{2}$ are internal or external bisectors in the triangles $P Q O$ and $R S O$, we have $$ u+v=x+y\\left(=90^{\\circ}-\\varphi\\right) . $$ By the law of sines ![](https:\/\/cdn.mathpix.com\/cropped\/2024_04_17_93f7104904c3717a5058g-36.jpg?height=417&width=880&top_left_y=1756&top_left_x=588) $$ \\frac{O P}{O O_{1}}=\\frac{\\sin (u+\\varphi)}{\\sin u} \\quad \\text { and } \\quad \\frac{O O_{2}}{O R}=\\frac{\\sin (x+\\varphi)}{\\sin x} $$ Therefore, since $x, u$ and $\\varphi$ are acute, $O P \\cdot O R \\geq p \\Leftrightarrow \\frac{O P}{O O_{1}} \\geq \\frac{O O_{2}}{O R} \\Leftrightarrow \\sin x \\sin (u+\\varphi) \\geq \\sin u \\sin (x+\\varphi) \\Leftrightarrow \\sin (x-u) \\geq 0 \\Leftrightarrow x \\geq u$. Thus $O P \\cdot O R \\geq p$ is equivalent to $x \\geq u$, with $O P \\cdot O R=p$ if and only if $x=u$. Analogously, $p \\geq O Q \\cdot O S$ is equivalent to $v \\geq y$, with $p=O Q \\cdot O S$ if and only if $v=y$. On the other hand $x \\geq u$ and $v \\geq y$ are equivalent by (1), with $x=u$ if and only if $v=y$. The conclusion of the lemma follows from here. Going back to the problem, apply the lemma to the quadruples $\\left\\{B, E, D, F_{1}\\right\\},\\{A, B, C, D\\}$ and $\\left\\{A, E, C, F_{2}\\right\\}$. Assuming $O E \\cdot O F_{1}>p$, we obtain $$ O E \\cdot O F_{1}>p \\Rightarrow O B \\cdot O D

p \\Rightarrow O E \\cdot O F_{2}

p$ implies $$ O B \\cdot O Dp>O E \\cdot O F_{2} \\text {. } $$ Similarly, $O E \\cdot O F_{1}p>O A \\cdot O C \\text { and } O E \\cdot O F_{1}s>O^{\\prime \\prime} Q+O^{\\prime} S, \\quad O^{\\prime} P+O^{\\prime \\prime} R=s=O^{\\prime \\prime} Q+O^{\\prime} S . $$ ![](https:\/\/cdn.mathpix.com\/cropped\/2024_04_17_93f7104904c3717a5058g-37.jpg?height=300&width=780&top_left_y=1403&top_left_x=638) Once this is established, the proof of the original statement for $B C \\| A D$ is analogous to the one in the intersecting case. One replaces products by sums of relevant segments.","tier":0} +{"problem_type":"Geometry","problem_label":"G8","problem":"Let $A B C$ be a triangle with circumcircle $\\omega$ and $\\ell$ a line without common points with $\\omega$. Denote by $P$ the foot of the perpendicular from the center of $\\omega$ to $\\ell$. The side-lines $B C, C A, A B$ intersect $\\ell$ at the points $X, Y, Z$ different from $P$. Prove that the circumcircles of the triangles $A X P, B Y P$ and $C Z P$ have a common point different from $P$ or are mutually tangent at $P$.","solution":"Let $\\omega_{A}, \\omega_{B}, \\omega_{C}$ and $\\omega$ be the circumcircles of triangles $A X P, B Y P, C Z P$ and $A B C$ respectively. The strategy of the proof is to construct a point $Q$ with the same power with respect to the four circles. Then each of $P$ and $Q$ has the same power with respect to $\\omega_{A}, \\omega_{B}, \\omega_{C}$ and hence the three circles are coaxial. In other words they have another common point $P^{\\prime}$ or the three of them are tangent at $P$. We first give a description of the point $Q$. Let $A^{\\prime} \\neq A$ be the second intersection of $\\omega$ and $\\omega_{A}$; define $B^{\\prime}$ and $C^{\\prime}$ analogously. We claim that $A A^{\\prime}, B B^{\\prime}$ and $C C^{\\prime}$ have a common point. Once this claim is established, the point just constructed will be on the radical axes of the three pairs of circles $\\left\\{\\omega, \\omega_{A}\\right\\},\\left\\{\\omega, \\omega_{B}\\right\\},\\left\\{\\omega, \\omega_{C}\\right\\}$. Hence it will have the same power with respect to $\\omega, \\omega_{A}, \\omega_{B}, \\omega_{C}$. ![](https:\/\/cdn.mathpix.com\/cropped\/2024_04_17_93f7104904c3717a5058g-38.jpg?height=783&width=1242&top_left_y=965&top_left_x=407) We proceed to prove that $A A^{\\prime}, B B^{\\prime}$ and $C C^{\\prime}$ intersect at one point. Let $r$ be the circumradius of triangle $A B C$. Define the points $X^{\\prime}, Y^{\\prime}, Z^{\\prime}$ as the intersections of $A A^{\\prime}, B B^{\\prime}, C C^{\\prime}$ with $\\ell$. Observe that $X^{\\prime}, Y^{\\prime}, Z^{\\prime}$ do exist. If $A A^{\\prime}$ is parallel to $\\ell$ then $\\omega_{A}$ is tangent to $\\ell$; hence $X=P$ which is a contradiction. Similarly, $B B^{\\prime}$ and $C C^{\\prime}$ are not parallel to $\\ell$. From the powers of the point $X^{\\prime}$ with respect to the circles $\\omega_{A}$ and $\\omega$ we get $$ X^{\\prime} P \\cdot\\left(X^{\\prime} P+P X\\right)=X^{\\prime} P \\cdot X^{\\prime} X=X^{\\prime} A^{\\prime} \\cdot X^{\\prime} A=X^{\\prime} O^{2}-r^{2} $$ hence $$ X^{\\prime} P \\cdot P X=X^{\\prime} O^{2}-r^{2}-X^{\\prime} P^{2}=O P^{2}-r^{2} . $$ We argue analogously for the points $Y^{\\prime}$ and $Z^{\\prime}$, obtaining $$ X^{\\prime} P \\cdot P X=Y^{\\prime} P \\cdot P Y=Z^{\\prime} P \\cdot P Z=O P^{2}-r^{2}=k^{2} $$ In these computations all segments are regarded as directed segments. We keep the same convention for the sequel. We prove that the lines $A A^{\\prime}, B B^{\\prime}, C C^{\\prime}$ intersect at one point by CEVA's theorem. To avoid distracting remarks we interpret everything projectively, i. e. whenever two lines are parallel they meet at a point on the line at infinity. Let $U, V, W$ be the intersections of $A A^{\\prime}, B B^{\\prime}, C C^{\\prime}$ with $B C, C A, A B$ respectively. The idea is that although it is difficult to calculate the ratio $\\frac{B U}{C U}$, it is easier to deal with the cross-ratio $\\frac{B U}{C U} \/ \\frac{B X}{C X}$ because we can send it to the line $\\ell$. With this in mind we apply MEnELaus' theorem to the triangle $A B C$ and obtain $\\frac{B X}{C X} \\cdot \\frac{C Y}{A Y} \\cdot \\frac{A Z}{B Z}=1$. Hence Ceva's ratio can be expressed as $$ \\frac{B U}{C U} \\cdot \\frac{C V}{A V} \\cdot \\frac{A W}{B W}=\\frac{B U}{C U} \/ \\frac{B X}{C X} \\cdot \\frac{C V}{A V} \/ \\frac{C Y}{A Y} \\cdot \\frac{A W}{B W} \/ \\frac{A Z}{B Z} $$ ![](https:\/\/cdn.mathpix.com\/cropped\/2024_04_17_93f7104904c3717a5058g-39.jpg?height=594&width=1145&top_left_y=565&top_left_x=453) Project the line $B C$ to $\\ell$ from $A$. The cross-ratio between $B C$ and $U X$ equals the cross-ratio between $Z Y$ and $X^{\\prime} X$. Repeating the same argument with the lines $C A$ and $A B$ gives $$ \\frac{B U}{C U} \\cdot \\frac{C V}{A V} \\cdot \\frac{A W}{B W}=\\frac{Z X^{\\prime}}{Y X^{\\prime}} \/ \\frac{Z X}{Y X} \\cdot \\frac{X Y^{\\prime}}{Z Y^{\\prime}} \/ \\frac{X Y}{Z Y} \\cdot \\frac{Y Z^{\\prime}}{X Z^{\\prime}} \/ \\frac{Y Z}{X Z} $$ and hence $$ \\frac{B U}{C U} \\cdot \\frac{C V}{A V} \\cdot \\frac{A W}{B W}=(-1) \\cdot \\frac{Z X^{\\prime}}{Y X^{\\prime}} \\cdot \\frac{X Y^{\\prime}}{Z Y^{\\prime}} \\cdot \\frac{Y Z^{\\prime}}{X Z^{\\prime}} $$ The equations (1) reduce the problem to a straightforward computation on the line $\\ell$. For instance, the transformation $t \\mapsto-k^{2} \/ t$ preserves cross-ratio and interchanges the points $X, Y, Z$ with the points $X^{\\prime}, Y^{\\prime}, Z^{\\prime}$. Then $$ \\frac{B U}{C U} \\cdot \\frac{C V}{A V} \\cdot \\frac{A W}{B W}=(-1) \\cdot \\frac{Z X^{\\prime}}{Y X^{\\prime}} \/ \\frac{Z Z^{\\prime}}{Y Z^{\\prime}} \\cdot \\frac{X Y^{\\prime}}{Z Y^{\\prime}} \/ \\frac{X Z^{\\prime}}{Z Z^{\\prime}}=-1 $$ We proved that CEvA's ratio equals -1 , so $A A^{\\prime}, B B^{\\prime}, C C^{\\prime}$ intersect at one point $Q$. Comment 1. There is a nice projective argument to prove that $A X^{\\prime}, B Y^{\\prime}, C Z^{\\prime}$ intersect at one point. Suppose that $\\ell$ and $\\omega$ intersect at a pair of complex conjugate points $D$ and $E$. Consider a projective transformation that takes $D$ and $E$ to $[i ; 1,0]$ and $[-i, 1,0]$. Then $\\ell$ is the line at infinity, and $\\omega$ is a conic through the special points $[i ; 1,0]$ and $[-i, 1,0]$, hence it is a circle. So one can assume that $A X, B Y, C Z$ are parallel to $B C, C A, A B$. The involution on $\\ell$ taking $X, Y, Z$ to $X^{\\prime}, Y^{\\prime}, Z^{\\prime}$ and leaving $D, E$ fixed is the involution changing each direction to its perpendicular one. Hence $A X, B Y, C Z$ are also perpendicular to $A X^{\\prime}, B Y^{\\prime}, C Z^{\\prime}$. It follows from the above that $A X^{\\prime}, B Y^{\\prime}, C Z^{\\prime}$ intersect at the orthocenter of triangle $A B C$. Comment 2. The restriction that the line $\\ell$ does not intersect the circumcricle $\\omega$ is unnecessary. The proof above works in general. In case $\\ell$ intersects $\\omega$ at $D$ and $E$ point $P$ is the midpoint of $D E$, and some equations can be interpreted differently. For instance $$ X^{\\prime} P \\cdot X^{\\prime} X=X^{\\prime} A^{\\prime} \\cdot X^{\\prime} A=X^{\\prime} D \\cdot X^{\\prime} E $$ and hence the pairs $X^{\\prime} X$ and $D E$ are harmonic conjugates. This means that $X^{\\prime}, Y^{\\prime}, Z^{\\prime}$ are the harmonic conjugates of $X, Y, Z$ with respect to the segment $D E$.","tier":0} +{"problem_type":"Geometry","problem_label":"G8","problem":"Let $A B C$ be a triangle with circumcircle $\\omega$ and $\\ell$ a line without common points with $\\omega$. Denote by $P$ the foot of the perpendicular from the center of $\\omega$ to $\\ell$. The side-lines $B C, C A, A B$ intersect $\\ell$ at the points $X, Y, Z$ different from $P$. Prove that the circumcircles of the triangles $A X P, B Y P$ and $C Z P$ have a common point different from $P$ or are mutually tangent at $P$.","solution":"First we prove that there is an inversion in space that takes $\\ell$ and $\\omega$ to parallel circles on a sphere. Let $Q R$ be the diameter of $\\omega$ whose extension beyond $Q$ passes through $P$. Let $\\Pi$ be the plane carrying our objects. In space, choose a point $O$ such that the line $Q O$ is perpendicular to $\\Pi$ and $\\angle P O R=90^{\\circ}$, and apply an inversion with pole $O$ (the radius of the inversion does not matter). For any object $\\mathcal{T}$ denote by $\\mathcal{T}^{\\prime}$ the image of $\\mathcal{T}$ under this inversion. The inversion takes the plane $\\Pi$ to a sphere $\\Pi^{\\prime}$. The lines in $\\Pi$ are taken to circles through $O$, and the circles in $\\Pi$ also are taken to circles on $\\Pi^{\\prime}$. ![](https:\/\/cdn.mathpix.com\/cropped\/2024_04_17_93f7104904c3717a5058g-40.jpg?height=491&width=1288&top_left_y=560&top_left_x=384) Since the line $\\ell$ and the circle $\\omega$ are perpendicular to the plane $O P Q$, the circles $\\ell^{\\prime}$ and $\\omega^{\\prime}$ also are perpendicular to this plane. Hence, the planes of the circles $\\ell^{\\prime}$ and $\\omega^{\\prime}$ are parallel. Now consider the circles $A^{\\prime} X^{\\prime} P^{\\prime}, B^{\\prime} Y^{\\prime} P^{\\prime}$ and $C^{\\prime} Z^{\\prime} P^{\\prime}$. We want to prove that either they have a common point (on $\\Pi^{\\prime}$ ), different from $P^{\\prime}$, or they are tangent to each other. ![](https:\/\/cdn.mathpix.com\/cropped\/2024_04_17_93f7104904c3717a5058g-40.jpg?height=819&width=870&top_left_y=1321&top_left_x=593) The point $X^{\\prime}$ is the second intersection of the circles $B^{\\prime} C^{\\prime} O$ and $\\ell^{\\prime}$, other than $O$. Hence, the lines $O X^{\\prime}$ and $B^{\\prime} C^{\\prime}$ are coplanar. Moreover, they lie in the parallel planes of $\\ell^{\\prime}$ and $\\omega^{\\prime}$. Therefore, $O X^{\\prime}$ and $B^{\\prime} C^{\\prime}$ are parallel. Analogously, $O Y^{\\prime}$ and $O Z^{\\prime}$ are parallel to $A^{\\prime} C^{\\prime}$ and $A^{\\prime} B^{\\prime}$. Let $A_{1}$ be the second intersection of the circles $A^{\\prime} X^{\\prime} P^{\\prime}$ and $\\omega^{\\prime}$, other than $A^{\\prime}$. The segments $A^{\\prime} A_{1}$ and $P^{\\prime} X^{\\prime}$ are coplanar, and therefore parallel. Now we know that $B^{\\prime} C^{\\prime}$ and $A^{\\prime} A_{1}$ are parallel to $O X^{\\prime}$ and $X^{\\prime} P^{\\prime}$ respectively, but these two segments are perpendicular because $O P^{\\prime}$ is a diameter in $\\ell^{\\prime}$. We found that $A^{\\prime} A_{1}$ and $B^{\\prime} C^{\\prime}$ are perpendicular, hence $A^{\\prime} A_{1}$ is the altitude in the triangle $A^{\\prime} B^{\\prime} C^{\\prime}$, starting from $A$. Analogously, let $B_{1}$ and $C_{1}$ be the second intersections of $\\omega^{\\prime}$ with the circles $B^{\\prime} P^{\\prime} Y^{\\prime}$ and $C^{\\prime} P^{\\prime} Z^{\\prime}$, other than $B^{\\prime}$ and $C^{\\prime}$ respectively. Then $B^{\\prime} B_{1}$ and $C^{\\prime} C_{1}$ are the other two altitudes in the triangle $A^{\\prime} B^{\\prime} C^{\\prime}$. Let $H$ be the orthocenter of the triangle $A^{\\prime} B^{\\prime} C^{\\prime}$. Let $W$ be the second intersection of the line $P^{\\prime} H$ with the sphere $\\Pi^{\\prime}$, other than $P^{\\prime}$. The point $W$ lies on the sphere $\\Pi^{\\prime}$, in the plane of the circle $A^{\\prime} P^{\\prime} X^{\\prime}$, so $W$ lies on the circle $A^{\\prime} P^{\\prime} X^{\\prime}$. Similarly, $W$ lies on the circles $B^{\\prime} P^{\\prime} Y^{\\prime}$ and $C^{\\prime} P^{\\prime} Z^{\\prime}$ as well; indeed $W$ is the second common point of the three circles. If the line $P^{\\prime} H$ is tangent to the sphere then $W$ coincides with $P^{\\prime}$, and $P^{\\prime} H$ is the common tangent of the three circles.","tier":0} +{"problem_type":"Number Theory","problem_label":"N1","problem":"Call admissible a set $A$ of integers that has the following property: $$ \\text { If } x, y \\in A \\text { (possibly } x=y \\text { ) then } x^{2}+k x y+y^{2} \\in A \\text { for every integer } k \\text {. } $$ Determine all pairs $m, n$ of nonzero integers such that the only admissible set containing both $m$ and $n$ is the set of all integers.","solution":"A pair of integers $m, n$ fulfills the condition if and only if $\\operatorname{gcd}(m, n)=1$. Suppose that $\\operatorname{gcd}(m, n)=d>1$. The set $$ A=\\{\\ldots,-2 d,-d, 0, d, 2 d, \\ldots\\} $$ is admissible, because if $d$ divides $x$ and $y$ then it divides $x^{2}+k x y+y^{2}$ for every integer $k$. Also $m, n \\in A$ and $A \\neq \\mathbb{Z}$. Now let $\\operatorname{gcd}(m, n)=1$, and let $A$ be an admissible set containing $m$ and $n$. We use the following observations to prove that $A=\\mathbb{Z}$ : (i) $k x^{2} \\in A$ for every $x \\in A$ and every integer $k$. (ii) $(x+y)^{2} \\in A$ for all $x, y \\in A$. To justify (i) let $y=x$ in the definition of an admissible set; to justify (ii) let $k=2$. Since $\\operatorname{gcd}(m, n)=1$, we also have $\\operatorname{gcd}\\left(m^{2}, n^{2}\\right)=1$. Hence one can find integers $a, b$ such that $a m^{2}+b n^{2}=1$. It follows from (i) that $a m^{2} \\in A$ and $b n^{2} \\in A$. Now we deduce from (ii) that $1=\\left(a m^{2}+b n^{2}\\right)^{2} \\in A$. But if $1 \\in A$ then (i) implies $k \\in A$ for every integer $k$.","tier":0} +{"problem_type":"Number Theory","problem_label":"N2","problem":"Find all triples $(x, y, z)$ of positive integers such that $x \\leq y \\leq z$ and $$ x^{3}\\left(y^{3}+z^{3}\\right)=2012(x y z+2) \\text {. } $$","solution":"First note that $x$ divides $2012 \\cdot 2=2^{3} \\cdot 503$. If $503 \\mid x$ then the right-hand side of the equation is divisible by $503^{3}$, and it follows that $503^{2} \\mid x y z+2$. This is false as $503 \\mid x$. Hence $x=2^{m}$ with $m \\in\\{0,1,2,3\\}$. If $m \\geq 2$ then $2^{6} \\mid 2012(x y z+2)$. However the highest powers of 2 dividing 2012 and $x y z+2=2^{m} y z+2$ are $2^{2}$ and $2^{1}$ respectively. So $x=1$ or $x=2$, yielding the two equations $$ y^{3}+z^{3}=2012(y z+2), \\quad \\text { and } \\quad y^{3}+z^{3}=503(y z+1) \\text {. } $$ In both cases the prime $503=3 \\cdot 167+2$ divides $y^{3}+z^{3}$. We claim that $503 \\mid y+z$. This is clear if $503 \\mid y$, so let $503 \\nmid y$ and $503 \\nmid z$. Then $y^{502} \\equiv z^{502}(\\bmod 503)$ by FeRmat's little theorem. On the other hand $y^{3} \\equiv-z^{3}(\\bmod 503)$ implies $y^{3 \\cdot 167} \\equiv-z^{3 \\cdot 167}(\\bmod 503)$, i. e. $y^{501} \\equiv-z^{501}(\\bmod 503)$. It follows that $y \\equiv-z(\\bmod 503)$ as claimed. Therefore $y+z=503 k$ with $k \\geq 1$. In view of $y^{3}+z^{3}=(y+z)\\left((y-z)^{2}+y z\\right)$ the two equations take the form $$ \\begin{aligned} & k(y-z)^{2}+(k-4) y z=8 \\\\ & k(y-z)^{2}+(k-1) y z=1 . \\end{aligned} $$ In (1) we have $(k-4) y z \\leq 8$, which implies $k \\leq 4$. Indeed if $k>4$ then $1 \\leq(k-4) y z \\leq 8$, so that $y \\leq 8$ and $z \\leq 8$. This is impossible as $y+z=503 k \\geq 503$. Note next that $y^{3}+z^{3}$ is even in the first equation. Hence $y+z=503 k$ is even too, meaning that $k$ is even. Thus $k=2$ or $k=4$. Clearly (1) has no integer solutions for $k=4$. If $k=2$ then (1) takes the form $(y+z)^{2}-5 y z=4$. Since $y+z=503 k=503 \\cdot 2$, this leads to $5 y z=503^{2} \\cdot 2^{2}-4$. However $503^{2} \\cdot 2^{2}-4$ is not a multiple of 5 . Therefore (1) has no integer solutions. Equation (2) implies $0 \\leq(k-1) y z \\leq 1$, so that $k=1$ or $k=2$. Also $0 \\leq k(y-z)^{2} \\leq 1$, hence $k=2$ only if $y=z$. However then $y=z=1$, which is false in view of $y+z \\geq 503$. Therefore $k=1$ and (2) takes the form $(y-z)^{2}=1$, yielding $z-y=|y-z|=1$. Combined with $k=1$ and $y+z=503 k$, this leads to $y=251, z=252$. In summary the triple $(2,251,252)$ is the only solution.","tier":0} +{"problem_type":"Number Theory","problem_label":"N3","problem":"Determine all integers $m \\geq 2$ such that every $n$ with $\\frac{m}{3} \\leq n \\leq \\frac{m}{2}$ divides the binomial coefficient $\\left(\\begin{array}{c}n \\\\ m-2 n\\end{array}\\right)$.","solution":"The integers in question are all prime numbers. First we check that all primes satisfy the condition, and even a stronger one. Namely, if $p$ is a prime then every $n$ with $1 \\leq n \\leq \\frac{p}{2}$ divides $\\left(\\begin{array}{c}n \\\\ p-2 n\\end{array}\\right)$. This is true for $p=2$ where $n=1$ is the only possibility. For an odd prime $p$ take $n \\in\\left[1, \\frac{p}{2}\\right]$ and consider the following identity of binomial coefficients: $$ (p-2 n) \\cdot\\left(\\begin{array}{c} n \\\\ p-2 n \\end{array}\\right)=n \\cdot\\left(\\begin{array}{c} n-1 \\\\ p-2 n-1 \\end{array}\\right) $$ Since $p \\geq 2 n$ and $p$ is odd, all factors are non-zero. If $d=\\operatorname{gcd}(p-2 n, n)$ then $d$ divides $p$, but $d \\leq n1$, pick $n=k$. Then $\\frac{m}{3} \\leq n \\leq \\frac{m}{2}$ but $\\left(\\begin{array}{c}n \\\\ m-2 n\\end{array}\\right)=\\left(\\begin{array}{l}k \\\\ 0\\end{array}\\right)=1$ is not divisible by $k>1$. - If $m$ is odd then there exist an odd prime $p$ and an integer $k \\geq 1$ with $m=p(2 k+1)$. Pick $n=p k$, then $\\frac{m}{3} \\leq n \\leq \\frac{m}{2}$ by $k \\geq 1$. However $$ \\frac{1}{n}\\left(\\begin{array}{c} n \\\\ m-2 n \\end{array}\\right)=\\frac{1}{p k}\\left(\\begin{array}{c} p k \\\\ p \\end{array}\\right)=\\frac{(p k-1)(p k-2) \\cdots(p k-(p-1))}{p !} $$ is not an integer, because $p$ divides the denominator but not the numerator.","tier":0} +{"problem_type":"Number Theory","problem_label":"N4","problem":"An integer $a$ is called friendly if the equation $\\left(m^{2}+n\\right)\\left(n^{2}+m\\right)=a(m-n)^{3}$ has a solution over the positive integers. a) Prove that there are at least 500 friendly integers in the set $\\{1,2, \\ldots, 2012\\}$. b) Decide whether $a=2$ is friendly.","solution":"a) Every $a$ of the form $a=4 k-3$ with $k \\geq 2$ is friendly. Indeed the numbers $m=2 k-1>0$ and $n=k-1>0$ satisfy the given equation with $a=4 k-3$ : $$ \\left(m^{2}+n\\right)\\left(n^{2}+m\\right)=\\left((2 k-1)^{2}+(k-1)\\right)\\left((k-1)^{2}+(2 k-1)\\right)=(4 k-3) k^{3}=a(m-n)^{3} \\text {. } $$ Hence $5,9, \\ldots, 2009$ are friendly and so $\\{1,2, \\ldots, 2012\\}$ contains at least 502 friendly numbers. b) We show that $a=2$ is not friendly. Consider the equation with $a=2$ and rewrite its left-hand side as a difference of squares: $$ \\frac{1}{4}\\left(\\left(m^{2}+n+n^{2}+m\\right)^{2}-\\left(m^{2}+n-n^{2}-m\\right)^{2}\\right)=2(m-n)^{3} $$ Since $m^{2}+n-n^{2}-m=(m-n)(m+n-1)$, we can further reformulate the equation as $$ \\left(m^{2}+n+n^{2}+m\\right)^{2}=(m-n)^{2}\\left(8(m-n)+(m+n-1)^{2}\\right) . $$ It follows that $8(m-n)+(m+n-1)^{2}$ is a perfect square. Clearly $m>n$, hence there is an integer $s \\geq 1$ such that $$ (m+n-1+2 s)^{2}=8(m-n)+(m+n-1)^{2} . $$ Subtracting the squares gives $s(m+n-1+s)=2(m-n)$. Since $m+n-1+s>m-n$, we conclude that $s<2$. Therefore the only possibility is $s=1$ and $m=3 n$. However then the left-hand side of the given equation (with $a=2$ ) is greater than $m^{3}=27 n^{3}$, whereas its right-hand side equals $16 n^{3}$. The contradiction proves that $a=2$ is not friendly. Comment. A computer search shows that there are 561 friendly numbers in $\\{1,2, \\ldots, 2012\\}$.","tier":0} +{"problem_type":"Number Theory","problem_label":"N5","problem":"For a nonnegative integer $n$ define $\\operatorname{rad}(n)=1$ if $n=0$ or $n=1$, and $\\operatorname{rad}(n)=p_{1} p_{2} \\cdots p_{k}$ where $p_{1}0$. Let $p$ be a prime number. The condition is that $f(n) \\equiv 0(\\bmod p)$ implies $$ f\\left(n^{\\operatorname{rad}(n)}\\right) \\equiv 0 \\quad(\\bmod p) $$ Since $\\operatorname{rad}\\left(n^{\\operatorname{rad}(n)^{k}}\\right)=\\operatorname{rad}(n)$ for all $k$, repeated applications of the preceding implication show that if $p$ divides $f(n)$ then $$ f\\left(n^{\\operatorname{rad}(n)^{k}}\\right) \\equiv 0 \\quad(\\bmod p) \\quad \\text { for all } k . $$ The idea is to construct a prime $p$ and a positive integer $n$ such that $p-1$ divides $n$ and $p$ divides $f(n)$. In this case, for $k$ large enough $p-1$ divides $\\operatorname{rad}(n)^{k}$. Hence if $(p, n)=1$ then $n^{r a d(n)^{k}} \\equiv 1(\\bmod p)$ by FERMAT's little theorem, so that $$ f(1) \\equiv f\\left(n^{\\operatorname{rad}(n)^{k}}\\right) \\equiv 0 \\quad(\\bmod p) . $$ Suppose that $f(x)=g(x) x^{m}$ with $g(0) \\neq 0$. Let $t$ be a positive integer, $p$ any prime factor of $g(-t)$ and $n=(p-1) t$. So $p-1$ divides $n$ and $f(n)=f((p-1) t) \\equiv f(-t) \\equiv 0(\\bmod p)$, hence either $(p, n)>1$ or $(2)$ holds. If $(p,(p-1) t)>1$ then $p$ divides $t$ and $g(0) \\equiv g(-t) \\equiv 0(\\bmod p)$, meaning that $p$ divides $g(0)$. In conclusion we proved that each prime factor of $g(-t)$ divides $g(0) f(1) \\neq 0$, and thus the set of prime factors of $g(-t)$ when $t$ ranges through the positive integers is finite. This is known to imply that $g(x)$ is a constant polynomial, and so $f(x)=a x^{m}$.","tier":0} +{"problem_type":"Number Theory","problem_label":"N5","problem":"For a nonnegative integer $n$ define $\\operatorname{rad}(n)=1$ if $n=0$ or $n=1$, and $\\operatorname{rad}(n)=p_{1} p_{2} \\cdots p_{k}$ where $p_{1}0$ and $\\xi=0$ is the only possible root. In either case $f(x)=a x^{m}$ with $a$ and $m$ nonnegative integers. To prove the claim let $\\xi$ be a root of $f(x)$, and let $g(x)$ be an irreducible factor of $f(x)$ such that $g(\\xi)=0$. If 0 or 1 are roots of $g(x)$ then either $\\xi=0$ or $\\xi=1$ (because $g(x)$ is irreducible) and we are done. So assume that $g(0), g(1) \\neq 0$. By decomposing $d$ as a product of prime numbers, it is enough to consider the case $d=p$ prime. We argue for $p=2$. Since $\\operatorname{rad}\\left(2^{k}\\right)=2$ for every $k$, we have $$ \\operatorname{rad}\\left(f\\left(2^{k}\\right)\\right) \\mid \\operatorname{rad}\\left(f\\left(2^{2 k}\\right)\\right) . $$ Now we prove that $g(x)$ divides $f\\left(x^{2}\\right)$. Suppose that this is not the case. Then, since $g(x)$ is irreducible, there are integer-coefficient polynomials $a(x), b(x)$ and an integer $N$ such that $$ a(x) g(x)+b(x) f\\left(x^{2}\\right)=N $$ Each prime factor $p$ of $g\\left(2^{k}\\right)$ divides $f\\left(2^{k}\\right)$, so by $\\operatorname{rad}\\left(f\\left(2^{k}\\right)\\right) \\mid \\operatorname{rad}\\left(f\\left(2^{2 k}\\right)\\right)$ it also divides $f\\left(2^{2 k}\\right)$. From the equation above with $x=2^{k}$ it follows that $p$ divides $N$. In summary, each prime divisor of $g\\left(2^{k}\\right)$ divides $N$, for all $k \\geq 0$. Let $p_{1}, \\ldots, p_{n}$ be the odd primes dividing $N$, and suppose that $$ g(1)=2^{\\alpha} p_{1}^{\\alpha_{1}} \\cdots p_{n}^{\\alpha_{n}} $$ If $k$ is divisible by $\\varphi\\left(p_{1}^{\\alpha_{1}+1} \\cdots p_{n}^{\\alpha_{n}+1}\\right)$ then $$ 2^{k} \\equiv 1 \\quad\\left(\\bmod p_{1}^{\\alpha_{1}+1} \\cdots p_{n}^{\\alpha_{n}+1}\\right) $$ yielding $$ g\\left(2^{k}\\right) \\equiv g(1) \\quad\\left(\\bmod p_{1}^{\\alpha_{1}+1} \\cdots p_{n}^{\\alpha_{n}+1}\\right) $$ It follows that for each $i$ the maximal power of $p_{i}$ dividing $g\\left(2^{k}\\right)$ and $g(1)$ is the same, namely $p_{i}^{\\alpha_{i}}$. On the other hand, for large enough $k$, the maximal power of 2 dividing $g\\left(2^{k}\\right)$ and $g(0) \\neq 0$ is the same. From the above, for $k$ divisible by $\\varphi\\left(p_{1}^{\\alpha_{1}+1} \\cdots p_{n}^{\\alpha_{n}+1}\\right)$ and large enough, we obtain that $g\\left(2^{k}\\right)$ divides $g(0) \\cdot g(1)$. This is impossible because $g(0), g(1) \\neq 0$ are fixed and $g\\left(2^{k}\\right)$ is arbitrarily large. In conclusion, $g(x)$ divides $f\\left(x^{2}\\right)$. Recall that $\\xi$ is a root of $f(x)$ such that $g(\\xi)=0$; then $f\\left(\\xi^{2}\\right)=0$, i. e. $\\xi^{2}$ is a root of $f(x)$. Likewise if $\\xi$ is a root of $f(x)$ and $p$ an arbitrary prime then $\\xi^{p}$ is a root too. The argument is completely analogous, in the proof above just replace 2 by $p$ and \"odd prime\" by \"prime different from $p . \"$ Comment. The claim in the second solution can be proved by varying $n(\\bmod p)$ in $(1)$. For instance, we obtain $$ f\\left(n^{\\operatorname{rad}(n+p k)}\\right) \\equiv 0 \\quad(\\bmod p) $$ for every positive integer $k$. One can prove that if $(n, p)=1$ then $\\operatorname{rad}(n+p k)$ runs through all residue classes $r(\\bmod p-1)$ with $(r, p-1)$ squarefree. Hence if $f(n) \\equiv 0(\\bmod p)$ then $f\\left(n^{r}\\right) \\equiv 0(\\bmod p)$ for all integers $r$. This implies the claim by an argument leading to the identity (3).","tier":0} +{"problem_type":"Number Theory","problem_label":"N6","problem":"Let $x$ and $y$ be positive integers. If $x^{2^{n}}-1$ is divisible by $2^{n} y+1$ for every positive integer $n$, prove that $x=1$.","solution":"First we prove the following fact: For every positive integer $y$ there exist infinitely many primes $p \\equiv 3(\\bmod 4)$ such that $p$ divides some number of the form $2^{n} y+1$. Clearly it is enough to consider the case $y$ odd. Let $$ 2 y+1=p_{1}^{e_{1}} \\cdots p_{r}^{e_{r}} $$ be the prime factorization of $2 y+1$. Suppose on the contrary that there are finitely many primes $p_{r+1}, \\ldots, p_{r+s} \\equiv 3(\\bmod 4)$ that divide some number of the form $2^{n} y+1$ but do not divide $2 y+1$. We want to find an $n$ such that $p_{i}^{e_{i}} \\| 2^{n} y+1$ for $1 \\leq i \\leq r$ and $p_{i} \\nmid 2^{n} y+1$ for $r+1 \\leq i \\leq r+s$. For this it suffices to take $$ n=1+\\varphi\\left(p_{1}^{e_{1}+1} \\cdots p_{r}^{e_{r}+1} p_{r+1}^{1} \\cdots p_{r+s}^{1}\\right) $$ because then $$ 2^{n} y+1 \\equiv 2 y+1 \\quad\\left(\\bmod p_{1}^{e_{1}+1} \\cdots p_{r}^{e_{r}+1} p_{r+1}^{1} \\cdots p_{r+s}^{1}\\right) $$ The last congruence means that $p_{1}^{e_{1}}, \\ldots, p_{r}^{e_{r}}$ divide exactly $2^{n} y+1$ and no prime $p_{r+1}, \\ldots, p_{r+s}$ divides $2^{n} y+1$. It follows that the prime factorization of $2^{n} y+1$ consists of the prime powers $p_{1}^{e_{1}}, \\ldots, p_{r}^{e_{r}}$ and powers of primes $\\equiv 1(\\bmod 4)$. Because $y$ is odd, we obtain $$ 2^{n} y+1 \\equiv p_{1}^{e_{1}} \\cdots p_{r}^{e_{r}} \\equiv 2 y+1 \\equiv 3 \\quad(\\bmod 4) $$ This is a contradiction since $n>1$, and so $2^{n} y+1 \\equiv 1(\\bmod 4)$. Now we proceed to the problem. If $p$ is a prime divisor of $2^{n} y+1$ the problem statement implies that $x^{d} \\equiv 1(\\bmod p)$ for $d=2^{n}$. By FERMAT's little theorem the same congruence holds for $d=p-1$, so it must also hold for $d=\\left(2^{n}, p-1\\right)$. For $p \\equiv 3(\\bmod 4)$ we have $\\left(2^{n}, p-1\\right)=2$, therefore in this case $x^{2} \\equiv 1(\\bmod p)$. In summary, we proved that every prime $p \\equiv 3(\\bmod 4)$ that divides some number of the form $2^{n} y+1$ also divides $x^{2}-1$. This is possible only if $x=1$, otherwise by the above $x^{2}-1$ would be a positive integer with infinitely many prime factors. Comment. For each $x$ and each odd prime $p$ the maximal power of $p$ dividing $x^{2^{n}}-1$ for some $n$ is bounded and hence the same must be true for the numbers $2^{n} y+1$. We infer that $p^{2}$ divides $2^{p-1}-1$ for each prime divisor $p$ of $2^{n} y+1$. However trying to reach a contradiction with this conclusion alone seems hopeless, since it is not even known if there are infinitely many primes $p$ without this property.","tier":0} +{"problem_type":"Number Theory","problem_label":"N7","problem":"Find all $n \\in \\mathbb{N}$ for which there exist nonnegative integers $a_{1}, a_{2}, \\ldots, a_{n}$ such that $$ \\frac{1}{2^{a_{1}}}+\\frac{1}{2^{a_{2}}}+\\cdots+\\frac{1}{2^{a_{n}}}=\\frac{1}{3^{a_{1}}}+\\frac{2}{3^{a_{2}}}+\\cdots+\\frac{n}{3^{a_{n}}}=1 . $$","solution":"Such numbers $a_{1}, a_{2}, \\ldots, a_{n}$ exist if and only if $n \\equiv 1(\\bmod 4)$ or $n \\equiv 2(\\bmod 4)$. Let $\\sum_{k=1}^{n} \\frac{k}{3^{a} k}=1$ with $a_{1}, a_{2}, \\ldots, a_{n}$ nonnegative integers. Then $1 \\cdot x_{1}+2 \\cdot x_{2}+\\cdots+n \\cdot x_{n}=3^{a}$ with $x_{1}, \\ldots, x_{n}$ powers of 3 and $a \\geq 0$. The right-hand side is odd, and the left-hand side has the same parity as $1+2+\\cdots+n$. Hence the latter sum is odd, which implies $n \\equiv 1,2(\\bmod 4)$. Now we prove the converse. Call feasible a sequence $b_{1}, b_{2}, \\ldots, b_{n}$ if there are nonnegative integers $a_{1}, a_{2}, \\ldots, a_{n}$ such that $$ \\frac{1}{2^{a_{1}}}+\\frac{1}{2^{a_{2}}}+\\cdots+\\frac{1}{2^{a_{n}}}=\\frac{b_{1}}{3^{a_{1}}}+\\frac{b_{2}}{3^{a_{2}}}+\\cdots+\\frac{b_{n}}{3^{a_{n}}}=1 $$ Let $b_{k}$ be a term of a feasible sequence $b_{1}, b_{2}, \\ldots, b_{n}$ with exponents $a_{1}, a_{2}, \\ldots, a_{n}$ like above, and let $u, v$ be nonnegative integers with sum $3 b_{k}$. Observe that $$ \\frac{1}{2^{a_{k}+1}}+\\frac{1}{2^{a_{k}+1}}=\\frac{1}{2^{a_{k}}} \\quad \\text { and } \\quad \\frac{u}{3^{a_{k}+1}}+\\frac{v}{3^{a_{k}+1}}=\\frac{b_{k}}{3^{a_{k}}} $$ It follows that the sequence $b_{1}, \\ldots, b_{k-1}, u, v, b_{k+1}, \\ldots, b_{n}$ is feasible. The exponents $a_{i}$ are the same for the unchanged terms $b_{i}, i \\neq k$; the new terms $u, v$ have exponents $a_{k}+1$. We state the conclusion in reverse. If two terms $u, v$ of a sequence are replaced by one term $\\frac{u+v}{3}$ and the obtained sequence is feasible, then the original sequence is feasible too. Denote by $\\alpha_{n}$ the sequence $1,2, \\ldots, n$. To show that $\\alpha_{n}$ is feasible for $n \\equiv 1,2(\\bmod 4)$, we transform it by $n-1$ replacements $\\{u, v\\} \\mapsto \\frac{u+v}{3}$ to the one-term sequence $\\alpha_{1}$. The latter is feasible, with $a_{1}=0$. Note that if $m$ and $2 m$ are terms of a sequence then $\\{m, 2 m\\} \\mapsto m$, so $2 m$ can be ignored if necessary. Let $n \\geq 16$. We prove that $\\alpha_{n}$ can be reduced to $\\alpha_{n-12}$ by 12 operations. Write $n=12 k+r$ where $k \\geq 1$ and $0 \\leq r \\leq 11$. If $0 \\leq r \\leq 5$ then the last 12 terms of $\\alpha_{n}$ can be partitioned into 2 singletons $\\{12 k-6\\},\\{12 k\\}$ and the following 5 pairs: $$ \\{12 k-6-i, 12 k-6+i\\}, i=1, \\ldots, 5-r ; \\quad\\{12 k-j, 12 k+j\\}, j=1, \\ldots, r . $$ (There is only one kind of pairs if $r \\in\\{0,5\\}$.) One can ignore $12 k-6$ and $12 k$ since $\\alpha_{n}$ contains $6 k-3$ and $6 k$. Furthermore the 5 operations $\\{12 k-6-i, 12 k-6+i\\} \\mapsto 8 k-4$ and $\\{12 k-j, 12 k+j\\} \\mapsto 8 k$ remove the 10 terms in the pairs and bring in 5 new terms equal to $8 k-4$ or $8 k$. All of these can be ignored too as $4 k-2$ and $4 k$ are still present in the sequence. Indeed $4 k \\leq n-12$ is equivalent to $8 k \\geq 12-r$, which is true for $r \\in\\{4,5\\}$. And if $r \\in\\{0,1,2,3\\}$ then $n \\geq 16$ implies $k \\geq 2$, so $8 k \\geq 12-r$ also holds. Thus $\\alpha_{n}$ reduces to $\\alpha_{n-12}$. The case $6 \\leq r \\leq 11$ is analogous. Consider the singletons $\\{12 k\\},\\{12 k+6\\}$ and the 5 pairs $$ \\{12 k-i, 12 k+i\\}, i=1, \\ldots, 11-r ; \\quad\\{12 k+6-j, 12 k+6+j\\}, j=1, \\ldots, r-6 $$ Ignore the singletons like before, then remove the pairs via operations $\\{12 k-i, 12 k+i\\} \\mapsto 8 k$ and $\\{12 k+6-j, 12 k+6+j\\} \\mapsto 8 k+4$. The 5 newly-appeared terms $8 k$ and $8 k+4$ can be ignored too since $4 k+2 \\leq n-12$ (this follows from $k \\geq 1$ and $r \\geq 6$ ). We obtain $\\alpha_{n-12}$ again. The problem reduces to $2 \\leq n \\leq 15$. In fact $n \\in\\{2,5,6,9,10,13,14\\}$ by $n \\equiv 1,2(\\bmod 4)$. The cases $n=2,6,10,14$ reduce to $n=1,5,9,13$ respectively because the last even term of $\\alpha_{n}$ can be ignored. For $n=5$ apply $\\{4,5\\} \\mapsto 3$, then $\\{3,3\\} \\mapsto 2$, then ignore the 2 occurrences of 2 . For $n=9$ ignore 6 first, then apply $\\{5,7\\} \\mapsto 4,\\{4,8\\} \\mapsto 4,\\{3,9\\} \\mapsto 4$. Now ignore the 3 occurrences of 4 , then ignore 2. Finally $n=13$ reduces to $n=10$ by $\\{11,13\\} \\mapsto 8$ and ignoring 8 and 12 . The proof is complete.","tier":0} +{"problem_type":"Number Theory","problem_label":"N8","problem":"Prove that for every prime $p>100$ and every integer $r$ there exist two integers $a$ and $b$ such that $p$ divides $a^{2}+b^{5}-r$.","solution":"Throughout the solution, all congruence relations are meant modulo $p$. Fix $p$, and let $\\mathcal{P}=\\{0,1, \\ldots, p-1\\}$ be the set of residue classes modulo $p$. For every $r \\in \\mathcal{P}$, let $S_{r}=\\left\\{(a, b) \\in \\mathcal{P} \\times \\mathcal{P}: a^{2}+b^{5} \\equiv r\\right\\}$, and let $s_{r}=\\left|S_{r}\\right|$. Our aim is to prove $s_{r}>0$ for all $r \\in \\mathcal{P}$. We will use the well-known fact that for every residue class $r \\in \\mathcal{P}$ and every positive integer $k$, there are at most $k$ values $x \\in \\mathcal{P}$ such that $x^{k} \\equiv r$. Lemma. Let $N$ be the number of quadruples $(a, b, c, d) \\in \\mathcal{P}^{4}$ for which $a^{2}+b^{5} \\equiv c^{2}+d^{5}$. Then $$ N=\\sum_{r \\in \\mathcal{P}} s_{r}^{2} $$ and $$ N \\leq p\\left(p^{2}+4 p-4\\right) $$ Proof. (a) For each residue class $r$ there exist exactly $s_{r}$ pairs $(a, b)$ with $a^{2}+b^{5} \\equiv r$ and $s_{r}$ pairs $(c, d)$ with $c^{2}+d^{5} \\equiv r$. So there are $s_{r}^{2}$ quadruples with $a^{2}+b^{5} \\equiv c^{2}+d^{5} \\equiv r$. Taking the sum over all $r \\in \\mathcal{P}$, the statement follows. (b) Choose an arbitrary pair $(b, d) \\in \\mathcal{P}$ and look for the possible values of $a, c$. 1. Suppose that $b^{5} \\equiv d^{5}$, and let $k$ be the number of such pairs $(b, d)$. The value $b$ can be chosen in $p$ different ways. For $b \\equiv 0$ only $d=0$ has this property; for the nonzero values of $b$ there are at most 5 possible values for $d$. So we have $k \\leq 1+5(p-1)=5 p-4$. The values $a$ and $c$ must satisfy $a^{2} \\equiv c^{2}$, so $a \\equiv \\pm c$, and there are exactly $2 p-1$ such pairs $(a, c)$. 2. Now suppose $b^{5} \\not \\equiv d^{5}$. In this case $a$ and $c$ must be distinct. By $(a-c)(a+c)=d^{5}-b^{5}$, the value of $a-c$ uniquely determines $a+c$ and thus $a$ and $c$ as well. Hence, there are $p-1$ suitable pairs $(a, c)$. Thus, for each of the $k$ pairs $(b, d)$ with $b^{5} \\equiv d^{5}$ there are $2 p-1$ pairs $(a, c)$, and for each of the other $p^{2}-k$ pairs $(b, d)$ there are $p-1$ pairs $(a, c)$. Hence, $$ N=k(2 p-1)+\\left(p^{2}-k\\right)(p-1)=p^{2}(p-1)+k p \\leq p^{2}(p-1)+(5 p-4) p=p\\left(p^{2}+4 p-4\\right) $$ To prove the statement of the problem, suppose that $S_{r}=\\emptyset$ for some $r \\in \\mathcal{P}$; obviously $r \\not \\equiv 0$. Let $T=\\left\\{x^{10}: x \\in \\mathcal{P} \\backslash\\{0\\}\\right\\}$ be the set of nonzero 10th powers modulo $p$. Since each residue class is the 10 th power of at most 10 elements in $\\mathcal{P}$, we have $|T| \\geq \\frac{p-1}{10} \\geq 4$ by $p>100$. For every $t \\in T$, we have $S_{t r}=\\emptyset$. Indeed, if $(x, y) \\in S_{t r}$ and $t \\equiv z^{10}$ then $$ \\left(z^{-5} x\\right)^{2}+\\left(z^{-2} y\\right)^{5} \\equiv t^{-1}\\left(x^{2}+y^{5}\\right) \\equiv r $$ so $\\left(z^{-5} x, z^{-2} y\\right) \\in S_{r}$. So, there are at least $\\frac{p-1}{10} \\geq 4$ empty sets among $S_{1}, \\ldots, S_{p-1}$, and there are at most $p-4$ nonzero values among $s_{0}, s_{2}, \\ldots, s_{p-1}$. Then by the AM-QM inequality we obtain $$ N=\\sum_{r \\in \\mathcal{P} \\backslash r T} s_{r}^{2} \\geq \\frac{1}{p-4}\\left(\\sum_{r \\in \\mathcal{P} \\backslash r T} s_{r}\\right)^{2}=\\frac{|\\mathcal{P} \\times \\mathcal{P}|^{2}}{p-4}=\\frac{p^{4}}{p-4}>p\\left(p^{2}+4 p-4\\right), $$ which is impossible by the lemma.","tier":0} +{"problem_type":"Number Theory","problem_label":"N8","problem":"Prove that for every prime $p>100$ and every integer $r$ there exist two integers $a$ and $b$ such that $p$ divides $a^{2}+b^{5}-r$.","solution":"If $5 \\nmid p-1$, then all modulo $p$ residue classes are complete fifth powers and the statement is trivial. So assume that $p=10 k+1$ where $k \\geq 10$. Let $g$ be a primitive root modulo $p$. We will use the following facts: (F1) If some residue class $x$ is not quadratic then $x^{(p-1) \/ 2} \\equiv-1(\\bmod p)$. (F2) For every integer $d$, as a simple corollary of the summation formula for geometric progressions, $$ \\sum_{i=0}^{2 k-1} g^{5 d i} \\equiv\\left\\{\\begin{array}{ll} 2 k & \\text { if } 2 k \\mid d \\\\ 0 & \\text { if } 2 k \\not \\nless d \\end{array} \\quad(\\bmod p)\\right. $$ Suppose that, contrary to the statement, some modulo $p$ residue class $r$ cannot be expressed as $a^{2}+b^{5}$. Of course $r \\not \\equiv 0(\\bmod p)$. By $(\\mathrm{F} 1)$ we have $\\left(r-b^{5}\\right)^{(p-1) \/ 2}=\\left(r-b^{5}\\right)^{5 k} \\equiv-1(\\bmod p)$ for all residue classes $b$. For $t=1,2 \\ldots, k-1$ consider the sums $$ S(t)=\\sum_{i=0}^{2 k-1}\\left(r-g^{5 i}\\right)^{5 k} g^{5 t i} $$ By the indirect assumption and (F2), $$ S(t)=\\sum_{i=0}^{2 k-1}\\left(r-\\left(g^{i}\\right)^{5}\\right)^{5 k} g^{5 t i} \\equiv \\sum_{i=0}^{2 k-1}(-1) g^{5 t i} \\equiv-\\sum_{i=0}^{2 k-1} g^{5 t i} \\equiv 0 \\quad(\\bmod p) $$ because $2 k$ cannot divide $t$. On the other hand, by the binomial theorem, $$ \\begin{aligned} S(t) & =\\sum_{i=0}^{2 k-1}\\left(\\sum_{j=0}^{5 k}\\left(\\begin{array}{c} 5 k \\\\ j \\end{array}\\right) r^{5 k-j}\\left(-g^{5 i}\\right)^{j}\\right) g^{5 t i}=\\sum_{j=0}^{5 k}(-1)^{j}\\left(\\begin{array}{c} 5 k \\\\ j \\end{array}\\right) r^{5 k-j}\\left(\\sum_{i=0}^{2 k-1} g^{5(j+t) i}\\right) \\equiv \\\\ & \\equiv \\sum_{j=0}^{5 k}(-1)^{j}\\left(\\begin{array}{c} 5 k \\\\ j \\end{array}\\right) r^{5 k-j}\\left\\{\\begin{array}{ll} 2 k & \\text { if } 2 k \\mid j+t \\\\ 0 & \\text { if } 2 k \\not j j+t \\end{array}(\\bmod p) .\\right. \\end{aligned} $$ Since $1 \\leq j+t<6 k$, the number $2 k$ divides $j+t$ only for $j=2 k-t$ and $j=4 k-t$. Hence, $$ \\begin{gathered} 0 \\equiv S(t) \\equiv(-1)^{t}\\left(\\left(\\begin{array}{c} 5 k \\\\ 2 k-t \\end{array}\\right) r^{3 k+t}+\\left(\\begin{array}{c} 5 k \\\\ 4 k-t \\end{array}\\right) r^{k+t}\\right) \\cdot 2 k \\quad(\\bmod p), \\\\ \\left(\\begin{array}{c} 5 k \\\\ 2 k-t \\end{array}\\right) r^{2 k}+\\left(\\begin{array}{c} 5 k \\\\ 4 k-t \\end{array}\\right) \\equiv 0 \\quad(\\bmod p) . \\end{gathered} $$ Taking this for $t=1,2$ and eliminating $r$, we get $$ \\begin{aligned} 0 & \\equiv\\left(\\begin{array}{c} 5 k \\\\ 2 k-2 \\end{array}\\right)\\left(\\left(\\begin{array}{c} 5 k \\\\ 2 k-1 \\end{array}\\right) r^{2 k}+\\left(\\begin{array}{c} 5 k \\\\ 4 k-1 \\end{array}\\right)\\right)-\\left(\\begin{array}{c} 5 k \\\\ 2 k-1 \\end{array}\\right)\\left(\\left(\\begin{array}{c} 5 k \\\\ 2 k-2 \\end{array}\\right) r^{2 k}+\\left(\\begin{array}{c} 5 k \\\\ 4 k-2 \\end{array}\\right)\\right) \\\\ & =\\left(\\begin{array}{c} 5 k \\\\ 2 k-2 \\end{array}\\right)\\left(\\begin{array}{c} 5 k \\\\ 4 k-1 \\end{array}\\right)-\\left(\\begin{array}{c} 5 k \\\\ 2 k-1 \\end{array}\\right)\\left(\\begin{array}{c} 5 k \\\\ 4 k-2 \\end{array}\\right) \\\\ & =\\frac{(5 k) !^{2}}{(2 k-1) !(3 k+2) !(4 k-1) !(k+2) !}((2 k-1)(k+2)-(3 k+2)(4 k-1)) \\\\ & =\\frac{-(5 k) !^{2} \\cdot 2 k(5 k+1)}{(2 k-1) !(3 k+2) !(4 k-1) !(k+2) !}(\\bmod p) . \\end{aligned} $$ But in the last expression none of the numbers is divisible by $p=10 k+1$, a contradiction. Comment 1. The argument in the second solution is valid whenever $k \\geq 3$, that is for all primes $p=10 k+1$ except $p=11$. This is an exceptional case when the statement is not true; $r=7$ cannot be expressed as desired. Comment 2. The statement is true in a more general setting: for every positive integer $n$, for all sufficiently large $p$, each residue class modulo $p$ can be expressed as $a^{2}+b^{n}$. Choosing $t=3$ would allow using the Cauchy-Davenport theorem (together with some analysis on the case of equality). In the literature more general results are known. For instance, the statement easily follows from the Hasse-Weil bound.","tier":0} +{"problem_type":"Combinatorics","problem_label":"C5","problem":"The columns and the rows of a $3 n \\times 3 n$ square board are numbered $1,2, \\ldots, 3 n$. Every square $(x, y)$ with $1 \\leq x, y \\leq 3 n$ is colored asparagus, byzantium or citrine according as the modulo 3 remainder of $x+y$ is 0 , 1 or 2 respectively. One token colored asparagus, byzantium or citrine is placed on each square, so that there are $3 n^{2}$ tokens of each color. Suppose that one can permute the tokens so that each token is moved to a distance of at most $d$ from its original position, each asparagus token replaces a byzantium token, each byzantium token replaces a citrine token, and each citrine token replaces an asparagus token. Prove that it is possible to permute the tokens so that each token is moved to a distance of at most $d+2$ from its original position, and each square contains a token with the same color as the square.","solution":"Without loss of generality it suffices to prove that the A-tokens can be moved to distinct $\\mathrm{A}$-squares in such a way that each $\\mathrm{A}$-token is moved to a distance at most $d+2$ from its original place. This means we need a perfect matching between the $3 n^{2} \\mathrm{~A}$-squares and the $3 n^{2}$ A-tokens such that the distance in each pair of the matching is at most $d+2$. To find the matching, we construct a bipartite graph. The A-squares will be the vertices in one class of the graph; the vertices in the other class will be the A-tokens. Split the board into $3 \\times 1$ horizontal triminos; then each trimino contains exactly one Asquare. Take a permutation $\\pi$ of the tokens which moves A-tokens to B-tokens, B-tokens to C-tokens, and C-tokens to A-tokens, in each case to a distance at most $d$. For each A-square $S$, and for each A-token $T$, connect $S$ and $T$ by an edge if $T, \\pi(T)$ or $\\pi^{-1}(T)$ is on the trimino containing $S$. We allow multiple edges; it is even possible that the same square and the same token are connected with three edges. Obviously the lengths of the edges in the graph do not exceed $d+2$. By length of an edge we mean the distance between the A-square and the A-token it connects. Each A-token $T$ is connected with the three A-squares whose triminos contain $T, \\pi(T)$ and $\\pi^{-1}(T)$. Therefore in the graph all tokens are of degree 3. We show that the same is true for the A-squares. Let $S$ be an arbitrary A-square, and let $T_{1}, T_{2}, T_{3}$ be the three tokens on the trimino containing $S$. For $i=1,2,3$, if $T_{i}$ is an A-token, then $S$ is connected with $T_{i}$; if $T_{i}$ is a B-token then $S$ is connected with $\\pi^{-1}\\left(T_{i}\\right)$; finally, if $T_{i}$ is a C-token then $S$ is connected with $\\pi\\left(T_{i}\\right)$. Hence in the graph the A-squares also are of degree 3. Since the A-squares are of degree 3 , from every set $\\mathcal{S}$ of A-squares exactly $3|\\mathcal{S}|$ edges start. These edges end in at least $|\\mathcal{S}|$ tokens because the A-tokens also are of degree 3. Hence every set $\\mathcal{S}$ of A-squares has at least $|\\mathcal{S}|$ neighbors among the A-tokens. Therefore, by HALL's marriage theorem, the graph contains a perfect matching between the two vertex classes. So there is a perfect matching between the A-squares and A-tokens with edges no longer than $d+2$. It follows that the tokens can be permuted as specified in the problem statement. Comment 1. In the original problem proposal the board was infinite and there were only two colors. Having $n$ colors for some positive integer $n$ was an option; we chose $n=3$. Moreover, we changed the board to a finite one to avoid dealing with infinite graphs (although Hall's theorem works in the infinite case as well). With only two colors Hall's theorem is not needed. In this case we split the board into $2 \\times 1$ dominos, and in the resulting graph all vertices are of degree 2. The graph consists of disjoint cycles with even length and infinite paths, so the existence of the matching is trivial. Having more than three colors would make the problem statement more complicated, because we need a matching between every two color classes of tokens. However, this would not mean a significant increase in difficulty. Comment 2. According to Wikipedia, the color asparagus (hexadecimal code \\#87A96B) is a tone of green that is named after the vegetable. Crayola created this color in 1993 as one of the 16 to be named in the Name The Color Contest. Byzantium (\\#702963) is a dark tone of purple. Its first recorded use as a color name in English was in 1926. Citrine (\\#E4D00A) is variously described as yellow, greenish-yellow, brownish-yellow or orange. The first known use of citrine as a color name in English was in the 14th century.","tier":0} +{"problem_type":"Number Theory","problem_label":"N5","problem":"For a nonnegative integer $n$ define $\\operatorname{rad}(n)=1$ if $n=0$ or $n=1$, and $\\operatorname{rad}(n)=p_{1} p_{2} \\cdots p_{k}$ where $p_{1}0$. Let $p$ be a prime number. The condition is that $f(n) \\equiv 0(\\bmod p)$ implies $$ f\\left(n^{\\operatorname{rad}(n)}\\right) \\equiv 0 \\quad(\\bmod p) $$ Since $\\operatorname{rad}\\left(n^{\\operatorname{rad}(n)^{k}}\\right)=\\operatorname{rad}(n)$ for all $k$, repeated applications of the preceding implication show that if $p$ divides $f(n)$ then $$ f\\left(n^{\\operatorname{rad}(n)^{k}}\\right) \\equiv 0 \\quad(\\bmod p) \\quad \\text { for all } k . $$ The idea is to construct a prime $p$ and a positive integer $n$ such that $p-1$ divides $n$ and $p$ divides $f(n)$. In this case, for $k$ large enough $p-1$ divides $\\operatorname{rad}(n)^{k}$. Hence if $(p, n)=1$ then $n^{r a d(n)^{k}} \\equiv 1(\\bmod p)$ by FERMAT's little theorem, so that $$ f(1) \\equiv f\\left(n^{\\operatorname{rad}(n)^{k}}\\right) \\equiv 0 \\quad(\\bmod p) . $$ Suppose that $f(x)=g(x) x^{m}$ with $g(0) \\neq 0$. Let $t$ be a positive integer, $p$ any prime factor of $g(-t)$ and $n=(p-1) t$. So $p-1$ divides $n$ and $f(n)=f((p-1) t) \\equiv f(-t) \\equiv 0(\\bmod p)$, hence either $(p, n)>1$ or $(2)$ holds. If $(p,(p-1) t)>1$ then $p$ divides $t$ and $g(0) \\equiv g(-t) \\equiv 0(\\bmod p)$, meaning that $p$ divides $g(0)$. In conclusion we proved that each prime factor of $g(-t)$ divides $g(0) f(1) \\neq 0$, and thus the set of prime factors of $g(-t)$ when $t$ ranges through the positive integers is finite. This is known to imply that $g(x)$ is a constant polynomial, and so $f(x)=a x^{m}$.","tier":0} +{"problem_type":"Number Theory","problem_label":"N5","problem":"For a nonnegative integer $n$ define $\\operatorname{rad}(n)=1$ if $n=0$ or $n=1$, and $\\operatorname{rad}(n)=p_{1} p_{2} \\cdots p_{k}$ where $p_{1}0$ and $\\xi=0$ is the only possible root. In either case $f(x)=a x^{m}$ with $a$ and $m$ nonnegative integers. To prove the claim let $\\xi$ be a root of $f(x)$, and let $g(x)$ be an irreducible factor of $f(x)$ such that $g(\\xi)=0$. If 0 or 1 are roots of $g(x)$ then either $\\xi=0$ or $\\xi=1$ (because $g(x)$ is irreducible) and we are done. So assume that $g(0), g(1) \\neq 0$. By decomposing $d$ as a product of prime numbers, it is enough to consider the case $d=p$ prime. We argue for $p=2$. Since $\\operatorname{rad}\\left(2^{k}\\right)=2$ for every $k$, we have $$ \\operatorname{rad}\\left(f\\left(2^{k}\\right)\\right) \\mid \\operatorname{rad}\\left(f\\left(2^{2 k}\\right)\\right) . $$ Now we prove that $g(x)$ divides $f\\left(x^{2}\\right)$. Suppose that this is not the case. Then, since $g(x)$ is irreducible, there are integer-coefficient polynomials $a(x), b(x)$ and an integer $N$ such that $$ a(x) g(x)+b(x) f\\left(x^{2}\\right)=N $$ Each prime factor $p$ of $g\\left(2^{k}\\right)$ divides $f\\left(2^{k}\\right)$, so by $\\operatorname{rad}\\left(f\\left(2^{k}\\right)\\right) \\mid \\operatorname{rad}\\left(f\\left(2^{2 k}\\right)\\right)$ it also divides $f\\left(2^{2 k}\\right)$. From the equation above with $x=2^{k}$ it follows that $p$ divides $N$. In summary, each prime divisor of $g\\left(2^{k}\\right)$ divides $N$, for all $k \\geq 0$. Let $p_{1}, \\ldots, p_{n}$ be the odd primes dividing $N$, and suppose that $$ g(1)=2^{\\alpha} p_{1}^{\\alpha_{1}} \\cdots p_{n}^{\\alpha_{n}} $$ If $k$ is divisible by $\\varphi\\left(p_{1}^{\\alpha_{1}+1} \\cdots p_{n}^{\\alpha_{n}+1}\\right)$ then $$ 2^{k} \\equiv 1 \\quad\\left(\\bmod p_{1}^{\\alpha_{1}+1} \\cdots p_{n}^{\\alpha_{n}+1}\\right) $$ yielding $$ g\\left(2^{k}\\right) \\equiv g(1) \\quad\\left(\\bmod p_{1}^{\\alpha_{1}+1} \\cdots p_{n}^{\\alpha_{n}+1}\\right) $$ It follows that for each $i$ the maximal power of $p_{i}$ dividing $g\\left(2^{k}\\right)$ and $g(1)$ is the same, namely $p_{i}^{\\alpha_{i}}$. On the other hand, for large enough $k$, the maximal power of 2 dividing $g\\left(2^{k}\\right)$ and $g(0) \\neq 0$ is the same. From the above, for $k$ divisible by $\\varphi\\left(p_{1}^{\\alpha_{1}+1} \\cdots p_{n}^{\\alpha_{n}+1}\\right)$ and large enough, we obtain that $g\\left(2^{k}\\right)$ divides $g(0) \\cdot g(1)$. This is impossible because $g(0), g(1) \\neq 0$ are fixed and $g\\left(2^{k}\\right)$ is arbitrarily large. In conclusion, $g(x)$ divides $f\\left(x^{2}\\right)$. Recall that $\\xi$ is a root of $f(x)$ such that $g(\\xi)=0$; then $f\\left(\\xi^{2}\\right)=0$, i. e. $\\xi^{2}$ is a root of $f(x)$. Likewise if $\\xi$ is a root of $f(x)$ and $p$ an arbitrary prime then $\\xi^{p}$ is a root too. The argument is completely analogous, in the proof above just replace 2 by $p$ and \"odd prime\" by \"prime different from $p . \"$ Comment. The claim in the second solution can be proved by varying $n(\\bmod p)$ in $(1)$. For instance, we obtain $$ f\\left(n^{\\operatorname{rad}(n+p k)}\\right) \\equiv 0 \\quad(\\bmod p) $$ for every positive integer $k$. One can prove that if $(n, p)=1$ then $\\operatorname{rad}(n+p k)$ runs through all residue classes $r(\\bmod p-1)$ with $(r, p-1)$ squarefree. Hence if $f(n) \\equiv 0(\\bmod p)$ then $f\\left(n^{r}\\right) \\equiv 0(\\bmod p)$ for all integers $r$. This implies the claim by an argument leading to the identity (3).","tier":0} diff --git a/IMO/segmented/en-IMO2013SL.jsonl b/IMO/segmented/en-IMO2013SL.jsonl new file mode 100644 index 0000000000000000000000000000000000000000..f321eaf56a38935eb1497b242c320cc17be552ae --- /dev/null +++ b/IMO/segmented/en-IMO2013SL.jsonl @@ -0,0 +1,74 @@ +{"problem_type":"Algebra","problem_label":"A1","problem":"Let $n$ be a positive integer and let $a_{1}, \\ldots, a_{n-1}$ be arbitrary real numbers. Define the sequences $u_{0}, \\ldots, u_{n}$ and $v_{0}, \\ldots, v_{n}$ inductively by $u_{0}=u_{1}=v_{0}=v_{1}=1$, and $$ u_{k+1}=u_{k}+a_{k} u_{k-1}, \\quad v_{k+1}=v_{k}+a_{n-k} v_{k-1} \\quad \\text { for } k=1, \\ldots, n-1 . $$ Prove that $u_{n}=v_{n}$. (France)","solution":"We prove by induction on $k$ that $$ u_{k}=\\sum_{\\substack{0i_{1}>\\ldots>i_{t}>n-k \\\\ i_{j}-i_{j+1} \\geqslant 2}} a_{i_{1}} \\ldots a_{i_{t}} $$ For $k=n$ the expressions (1) and (2) coincide, so indeed $u_{n}=v_{n}$.","tier":0} +{"problem_type":"Algebra","problem_label":"A1","problem":"Let $n$ be a positive integer and let $a_{1}, \\ldots, a_{n-1}$ be arbitrary real numbers. Define the sequences $u_{0}, \\ldots, u_{n}$ and $v_{0}, \\ldots, v_{n}$ inductively by $u_{0}=u_{1}=v_{0}=v_{1}=1$, and $$ u_{k+1}=u_{k}+a_{k} u_{k-1}, \\quad v_{k+1}=v_{k}+a_{n-k} v_{k-1} \\quad \\text { for } k=1, \\ldots, n-1 . $$ Prove that $u_{n}=v_{n}$. (France)","solution":"Define recursively a sequence of multivariate polynomials by $$ P_{0}=P_{1}=1, \\quad P_{k+1}\\left(x_{1}, \\ldots, x_{k}\\right)=P_{k}\\left(x_{1}, \\ldots, x_{k-1}\\right)+x_{k} P_{k-1}\\left(x_{1}, \\ldots, x_{k-2}\\right), $$ so $P_{n}$ is a polynomial in $n-1$ variables for each $n \\geqslant 1$. Two easy inductive arguments show that $$ u_{n}=P_{n}\\left(a_{1}, \\ldots, a_{n-1}\\right), \\quad v_{n}=P_{n}\\left(a_{n-1}, \\ldots, a_{1}\\right) $$ so we need to prove $P_{n}\\left(x_{1}, \\ldots, x_{n-1}\\right)=P_{n}\\left(x_{n-1}, \\ldots, x_{1}\\right)$ for every positive integer $n$. The cases $n=1,2$ are trivial, and the cases $n=3,4$ follow from $P_{3}(x, y)=1+x+y$ and $P_{4}(x, y, z)=$ $1+x+y+z+x z$. Now we proceed by induction, assuming that $n \\geqslant 5$ and the claim hold for all smaller cases. Using $F(a, b)$ as an abbreviation for $P_{|a-b|+1}\\left(x_{a}, \\ldots, x_{b}\\right)$ (where the indices $a, \\ldots, b$ can be either in increasing or decreasing order), $$ \\begin{aligned} F(n, 1) & =F(n, 2)+x_{1} F(n, 3)=F(2, n)+x_{1} F(3, n) \\\\ & =\\left(F(2, n-1)+x_{n} F(2, n-2)\\right)+x_{1}\\left(F(3, n-1)+x_{n} F(3, n-2)\\right) \\\\ & =\\left(F(n-1,2)+x_{1} F(n-1,3)\\right)+x_{n}\\left(F(n-2,2)+x_{1} F(n-2,3)\\right) \\\\ & =F(n-1,1)+x_{n} F(n-2,1)=F(1, n-1)+x_{n} F(1, n-2) \\\\ & =F(1, n), \\end{aligned} $$ as we wished to show.","tier":0} +{"problem_type":"Algebra","problem_label":"A1","problem":"Let $n$ be a positive integer and let $a_{1}, \\ldots, a_{n-1}$ be arbitrary real numbers. Define the sequences $u_{0}, \\ldots, u_{n}$ and $v_{0}, \\ldots, v_{n}$ inductively by $u_{0}=u_{1}=v_{0}=v_{1}=1$, and $$ u_{k+1}=u_{k}+a_{k} u_{k-1}, \\quad v_{k+1}=v_{k}+a_{n-k} v_{k-1} \\quad \\text { for } k=1, \\ldots, n-1 . $$ Prove that $u_{n}=v_{n}$. (France)","solution":"Using matrix notation, we can rewrite the recurrence relation as $$ \\left(\\begin{array}{c} u_{k+1} \\\\ u_{k+1}-u_{k} \\end{array}\\right)=\\left(\\begin{array}{c} u_{k}+a_{k} u_{k-1} \\\\ a_{k} u_{k-1} \\end{array}\\right)=\\left(\\begin{array}{cc} 1+a_{k} & -a_{k} \\\\ a_{k} & -a_{k} \\end{array}\\right)\\left(\\begin{array}{c} u_{k} \\\\ u_{k}-u_{k-1} \\end{array}\\right) $$ for $1 \\leqslant k \\leqslant n-1$, and similarly $$ \\left(v_{k+1} ; v_{k}-v_{k+1}\\right)=\\left(v_{k}+a_{n-k} v_{k-1} ;-a_{n-k} v_{k-1}\\right)=\\left(v_{k} ; v_{k-1}-v_{k}\\right)\\left(\\begin{array}{cc} 1+a_{n-k} & -a_{n-k} \\\\ a_{n-k} & -a_{n-k} \\end{array}\\right) $$ for $1 \\leqslant k \\leqslant n-1$. Hence, introducing the $2 \\times 2$ matrices $A_{k}=\\left(\\begin{array}{cc}1+a_{k} & -a_{k} \\\\ a_{k} & -a_{k}\\end{array}\\right)$ we have $$ \\left(\\begin{array}{c} u_{k+1} \\\\ u_{k+1}-u_{k} \\end{array}\\right)=A_{k}\\left(\\begin{array}{c} u_{k} \\\\ u_{k}-u_{k-1} \\end{array}\\right) \\quad \\text { and } \\quad\\left(v_{k+1} ; v_{k}-v_{k+1}\\right)=\\left(v_{k} ; v_{k-1}-v_{k}\\right) A_{n-k} . $$ for $1 \\leqslant k \\leqslant n-1$. Since $\\left(\\begin{array}{c}u_{1} \\\\ u_{1}-u_{0}\\end{array}\\right)=\\left(\\begin{array}{l}1 \\\\ 0\\end{array}\\right)$ and $\\left(v_{1} ; v_{0}-v_{1}\\right)=(1 ; 0)$, we get $$ \\left(\\begin{array}{c} u_{n} \\\\ u_{n}-u_{n-1} \\end{array}\\right)=A_{n-1} A_{n-2} \\cdots A_{1} \\cdot\\left(\\begin{array}{l} 1 \\\\ 0 \\end{array}\\right) \\quad \\text { and } \\quad\\left(v_{n} ; v_{n-1}-v_{n}\\right)=(1 ; 0) \\cdot A_{n-1} A_{n-2} \\cdots A_{1} \\text {. } $$ It follows that $$ \\left(u_{n}\\right)=(1 ; 0)\\left(\\begin{array}{c} u_{n} \\\\ u_{n}-u_{n-1} \\end{array}\\right)=(1 ; 0) \\cdot A_{n-1} A_{n-2} \\cdots A_{1} \\cdot\\left(\\begin{array}{l} 1 \\\\ 0 \\end{array}\\right)=\\left(v_{n} ; v_{n-1}-v_{n}\\right)\\left(\\begin{array}{l} 1 \\\\ 0 \\end{array}\\right)=\\left(v_{n}\\right) . $$ Comment 1. These sequences are related to the Fibonacci sequence; when $a_{1}=\\cdots=a_{n-1}=1$, we have $u_{k}=v_{k}=F_{k+1}$, the $(k+1)$ st Fibonacci number. Also, for every positive integer $k$, the polynomial $P_{k}\\left(x_{1}, \\ldots, x_{k-1}\\right)$ from Solution 2 is the sum of $F_{k+1}$ monomials. Comment 2. One may notice that the condition is equivalent to $$ \\frac{u_{k+1}}{u_{k}}=1+\\frac{a_{k}}{1+\\frac{a_{k-1}}{1+\\ldots+\\frac{a_{2}}{1+a_{1}}}} \\quad \\text { and } \\quad \\frac{v_{k+1}}{v_{k}}=1+\\frac{a_{n-k}}{1+\\frac{a_{n-k+1}}{1+\\ldots+\\frac{a_{n-2}}{1+a_{n-1}}}} $$ so the problem claims that the corresponding continued fractions for $u_{n} \/ u_{n-1}$ and $v_{n} \/ v_{n-1}$ have the same numerator. Comment 3. An alternative variant of the problem is the following. Let $n$ be a positive integer and let $a_{1}, \\ldots, a_{n-1}$ be arbitrary real numbers. Define the sequences $u_{0}, \\ldots, u_{n}$ and $v_{0}, \\ldots, v_{n}$ inductively by $u_{0}=v_{0}=0, u_{1}=v_{1}=1$, and $$ u_{k+1}=a_{k} u_{k}+u_{k-1}, \\quad v_{k+1}=a_{n-k} v_{k}+v_{k-1} \\quad \\text { for } k=1, \\ldots, n-1 $$ Prove that $u_{n}=v_{n}$. All three solutions above can be reformulated to prove this statement; one may prove $$ u_{n}=v_{n}=\\sum_{\\substack{0=i_{0}0 $$ or observe that $$ \\left(\\begin{array}{c} u_{k+1} \\\\ u_{k} \\end{array}\\right)=\\left(\\begin{array}{cc} a_{k} & 1 \\\\ 1 & 0 \\end{array}\\right)\\left(\\begin{array}{c} u_{k} \\\\ u_{k-1} \\end{array}\\right) \\quad \\text { and } \\quad\\left(v_{k+1} ; v_{k}\\right)=\\left(v_{k} ; v_{k-1}\\right)\\left(\\begin{array}{cc} a_{k} & 1 \\\\ 1 & 0 \\end{array}\\right) . $$ Here we have $$ \\frac{u_{k+1}}{u_{k}}=a_{k}+\\frac{1}{a_{k-1}+\\frac{1}{a_{k-2}+\\ldots+\\frac{1}{a_{1}}}}=\\left[a_{k} ; a_{k-1}, \\ldots, a_{1}\\right] $$ and $$ \\frac{v_{k+1}}{v_{k}}=a_{n-k}+\\frac{1}{a_{n-k+1}+\\frac{1}{a_{n-k+2}+\\ldots+\\frac{1}{a_{n-1}}}}=\\left[a_{n-k} ; a_{n-k+1}, \\ldots, a_{n-1}\\right] $$ so this alternative statement is equivalent to the known fact that the continued fractions $\\left[a_{n-1} ; a_{n-2}, \\ldots, a_{1}\\right]$ and $\\left[a_{1} ; a_{2}, \\ldots, a_{n-1}\\right]$ have the same numerator.","tier":0} +{"problem_type":"Algebra","problem_label":"A2","problem":"Prove that in any set of 2000 distinct real numbers there exist two pairs $a>b$ and $c>d$ with $a \\neq c$ or $b \\neq d$, such that $$ \\left|\\frac{a-b}{c-d}-1\\right|<\\frac{1}{100000} $$ (Lithuania)","solution":"For any set $S$ of $n=2000$ distinct real numbers, let $D_{1} \\leqslant D_{2} \\leqslant \\cdots \\leqslant D_{m}$ be the distances between them, displayed with their multiplicities. Here $m=n(n-1) \/ 2$. By rescaling the numbers, we may assume that the smallest distance $D_{1}$ between two elements of $S$ is $D_{1}=1$. Let $D_{1}=1=y-x$ for $x, y \\in S$. Evidently $D_{m}=v-u$ is the difference between the largest element $v$ and the smallest element $u$ of $S$. If $D_{i+1} \/ D_{i}<1+10^{-5}$ for some $i=1,2, \\ldots, m-1$ then the required inequality holds, because $0 \\leqslant D_{i+1} \/ D_{i}-1<10^{-5}$. Otherwise, the reverse inequality $$ \\frac{D_{i+1}}{D_{i}} \\geqslant 1+\\frac{1}{10^{5}} $$ holds for each $i=1,2, \\ldots, m-1$, and therefore $$ v-u=D_{m}=\\frac{D_{m}}{D_{1}}=\\frac{D_{m}}{D_{m-1}} \\cdots \\frac{D_{3}}{D_{2}} \\cdot \\frac{D_{2}}{D_{1}} \\geqslant\\left(1+\\frac{1}{10^{5}}\\right)^{m-1} . $$ From $m-1=n(n-1) \/ 2-1=1000 \\cdot 1999-1>19 \\cdot 10^{5}$, together with the fact that for all $n \\geqslant 1$, $\\left(1+\\frac{1}{n}\\right)^{n} \\geqslant 1+\\left(\\begin{array}{l}n \\\\ 1\\end{array}\\right) \\cdot \\frac{1}{n}=2$, we get $$ \\left(1+\\frac{1}{10^{5}}\\right)^{19 \\cdot 10^{5}}=\\left(\\left(1+\\frac{1}{10^{5}}\\right)^{10^{5}}\\right)^{19} \\geqslant 2^{19}=2^{9} \\cdot 2^{10}>500 \\cdot 1000>2 \\cdot 10^{5}, $$ and so $v-u=D_{m}>2 \\cdot 10^{5}$. Since the distance of $x$ to at least one of the numbers $u, v$ is at least $(u-v) \/ 2>10^{5}$, we have $$ |x-z|>10^{5} . $$ for some $z \\in\\{u, v\\}$. Since $y-x=1$, we have either $z>y>x$ (if $z=v$ ) or $y>x>z$ (if $z=u$ ). If $z>y>x$, selecting $a=z, b=y, c=z$ and $d=x$ (so that $b \\neq d$ ), we obtain $$ \\left|\\frac{a-b}{c-d}-1\\right|=\\left|\\frac{z-y}{z-x}-1\\right|=\\left|\\frac{x-y}{z-x}\\right|=\\frac{1}{z-x}<10^{-5} . $$ Otherwise, if $y>x>z$, we may choose $a=y, b=z, c=x$ and $d=z$ (so that $a \\neq c$ ), and obtain $$ \\left|\\frac{a-b}{c-d}-1\\right|=\\left|\\frac{y-z}{x-z}-1\\right|=\\left|\\frac{y-x}{x-z}\\right|=\\frac{1}{x-z}<10^{-5} $$ The desired result follows. Comment. As the solution shows, the numbers 2000 and $\\frac{1}{100000}$ appearing in the statement of the problem may be replaced by any $n \\in \\mathbb{Z}_{>0}$ and $\\delta>0$ satisfying $$ \\delta(1+\\delta)^{n(n-1) \/ 2-1}>2 $$","tier":0} +{"problem_type":"Algebra","problem_label":"A3","problem":"Let $\\mathbb{Q}_{>0}$ be the set of positive rational numbers. Let $f: \\mathbb{Q}_{>0} \\rightarrow \\mathbb{R}$ be a function satisfying the conditions $$ f(x) f(y) \\geqslant f(x y) \\text { and } f(x+y) \\geqslant f(x)+f(y) $$ for all $x, y \\in \\mathbb{Q}_{>0}$. Given that $f(a)=a$ for some rational $a>1$, prove that $f(x)=x$ for all $x \\in \\mathbb{Q}_{>0}$. (Bulgaria)","solution":"Denote by $\\mathbb{Z}_{>0}$ the set of positive integers. Plugging $x=1, y=a$ into (1) we get $f(1) \\geqslant 1$. Next, by an easy induction on $n$ we get from (2) that $$ f(n x) \\geqslant n f(x) \\text { for all } n \\in \\mathbb{Z}_{>0} \\text { and } x \\in \\mathbb{Q}_{>0} $$ In particular, we have $$ f(n) \\geqslant n f(1) \\geqslant n \\quad \\text { for all } n \\in \\mathbb{Z}_{>0} $$ From (1) again we have $f(m \/ n) f(n) \\geqslant f(m)$, so $f(q)>0$ for all $q \\in \\mathbb{Q}_{>0}$. Now, (2) implies that $f$ is strictly increasing; this fact together with (4) yields $$ f(x) \\geqslant f(\\lfloor x\\rfloor) \\geqslant\\lfloor x\\rfloor>x-1 \\quad \\text { for all } x \\geqslant 1 $$ By an easy induction we get from (1) that $f(x)^{n} \\geqslant f\\left(x^{n}\\right)$, so $$ f(x)^{n} \\geqslant f\\left(x^{n}\\right)>x^{n}-1 \\quad \\Longrightarrow \\quad f(x) \\geqslant \\sqrt[n]{x^{n}-1} \\text { for all } x>1 \\text { and } n \\in \\mathbb{Z}_{>0} $$ This yields $$ f(x) \\geqslant x \\text { for every } x>1 \\text {. } $$ (Indeed, if $x>y>1$ then $x^{n}-y^{n}=(x-y)\\left(x^{n-1}+x^{n-2} y+\\cdots+y^{n}\\right)>n(x-y)$, so for a large $n$ we have $x^{n}-1>y^{n}$ and thus $f(x)>y$.) Now, (1) and (5) give $a^{n}=f(a)^{n} \\geqslant f\\left(a^{n}\\right) \\geqslant a^{n}$, so $f\\left(a^{n}\\right)=a^{n}$. Now, for $x>1$ let us choose $n \\in \\mathbb{Z}_{>0}$ such that $a^{n}-x>1$. Then by (2) and (5) we get $$ a^{n}=f\\left(a^{n}\\right) \\geqslant f(x)+f\\left(a^{n}-x\\right) \\geqslant x+\\left(a^{n}-x\\right)=a^{n} $$ and therefore $f(x)=x$ for $x>1$. Finally, for every $x \\in \\mathbb{Q}_{>0}$ and every $n \\in \\mathbb{Z}_{>0}$, from (1) and (3) we get $$ n f(x)=f(n) f(x) \\geqslant f(n x) \\geqslant n f(x) $$ which gives $f(n x)=n f(x)$. Therefore $f(m \/ n)=f(m) \/ n=m \/ n$ for all $m, n \\in \\mathbb{Z}_{>0}$. Comment. The condition $f(a)=a>1$ is essential. Indeed, for $b \\geqslant 1$ the function $f(x)=b x^{2}$ satisfies (1) and (2) for all $x, y \\in \\mathbb{Q}_{>0}$, and it has a unique fixed point $1 \/ b \\leqslant 1$.","tier":0} +{"problem_type":"Algebra","problem_label":"A4","problem":"Let $n$ be a positive integer, and consider a sequence $a_{1}, a_{2}, \\ldots, a_{n}$ of positive integers. Extend it periodically to an infinite sequence $a_{1}, a_{2}, \\ldots$ by defining $a_{n+i}=a_{i}$ for all $i \\geqslant 1$. If $$ a_{1} \\leqslant a_{2} \\leqslant \\cdots \\leqslant a_{n} \\leqslant a_{1}+n $$ and $$ a_{a_{i}} \\leqslant n+i-1 \\quad \\text { for } i=1,2, \\ldots, n $$ prove that $$ a_{1}+\\cdots+a_{n} \\leqslant n^{2} . $$ (Germany)","solution":"First, we claim that $$ a_{i} \\leqslant n+i-1 \\quad \\text { for } i=1,2, \\ldots, n \\text {. } $$ Assume contrariwise that $i$ is the smallest counterexample. From $a_{n} \\geqslant a_{n-1} \\geqslant \\cdots \\geqslant a_{i} \\geqslant n+i$ and $a_{a_{i}} \\leqslant n+i-1$, taking into account the periodicity of our sequence, it follows that $$ a_{i} \\text { cannot be congruent to } i, i+1, \\ldots, n-1 \\text {, or } n(\\bmod n) \\text {. } $$ Thus our assumption that $a_{i} \\geqslant n+i$ implies the stronger statement that $a_{i} \\geqslant 2 n+1$, which by $a_{1}+n \\geqslant a_{n} \\geqslant a_{i}$ gives $a_{1} \\geqslant n+1$. The minimality of $i$ then yields $i=1$, and (4) becomes contradictory. This establishes our first claim. In particular we now know that $a_{1} \\leqslant n$. If $a_{n} \\leqslant n$, then $a_{1} \\leqslant \\cdots \\leqslant \\cdots a_{n} \\leqslant n$ and the desired inequality holds trivially. Otherwise, consider the number $t$ with $1 \\leqslant t \\leqslant n-1$ such that $$ a_{1} \\leqslant a_{2} \\leqslant \\ldots \\leqslant a_{t} \\leqslant na_{a_{i}}$ ) belongs to $\\left\\{a_{i}+1, \\ldots, n\\right\\}$, and for this reason $b_{i} \\leqslant n-a_{i}$. It follows from the definition of the $b_{i} \\mathrm{~s}$ and (5) that $$ a_{t+1}+\\ldots+a_{n} \\leqslant n(n-t)+b_{1}+\\ldots+b_{t} . $$ Adding $a_{1}+\\ldots+a_{t}$ to both sides and using that $a_{i}+b_{i} \\leqslant n$ for $1 \\leqslant i \\leqslant t$, we get $$ a_{1}+a_{2}+\\cdots+a_{n} \\leqslant n(n-t)+n t=n^{2} $$ as we wished to prove.","tier":0} +{"problem_type":"Algebra","problem_label":"A4","problem":"Let $n$ be a positive integer, and consider a sequence $a_{1}, a_{2}, \\ldots, a_{n}$ of positive integers. Extend it periodically to an infinite sequence $a_{1}, a_{2}, \\ldots$ by defining $a_{n+i}=a_{i}$ for all $i \\geqslant 1$. If $$ a_{1} \\leqslant a_{2} \\leqslant \\cdots \\leqslant a_{n} \\leqslant a_{1}+n $$ and $$ a_{a_{i}} \\leqslant n+i-1 \\quad \\text { for } i=1,2, \\ldots, n $$ prove that $$ a_{1}+\\cdots+a_{n} \\leqslant n^{2} . $$ (Germany)","solution":"In the first quadrant of an infinite grid, consider the increasing \"staircase\" obtained by shading in dark the bottom $a_{i}$ cells of the $i$ th column for $1 \\leqslant i \\leqslant n$. We will prove that there are at most $n^{2}$ dark cells. To do it, consider the $n \\times n$ square $S$ in the first quadrant with a vertex at the origin. Also consider the $n \\times n$ square directly to the left of $S$. Starting from its lower left corner, shade in light the leftmost $a_{j}$ cells of the $j$ th row for $1 \\leqslant j \\leqslant n$. Equivalently, the light shading is obtained by reflecting the dark shading across the line $x=y$ and translating it $n$ units to the left. The figure below illustrates this construction for the sequence $6,6,6,7,7,7,8,12,12,14$. ![](https:\/\/cdn.mathpix.com\/cropped\/2024_04_17_e0914cb4f8055c731538g-14.jpg?height=650&width=1052&top_left_y=667&top_left_x=534) We claim that there is no cell in $S$ which is both dark and light. Assume, contrariwise, that there is such a cell in column $i$. Consider the highest dark cell in column $i$ which is inside $S$. Since it is above a light cell and inside $S$, it must be light as well. There are two cases: Case 1. $a_{i} \\leqslant n$ If $a_{i} \\leqslant n$ then this dark and light cell is $\\left(i, a_{i}\\right)$, as highlighted in the figure. However, this is the $(n+i)$-th cell in row $a_{i}$, and we only shaded $a_{a_{i}}0$ such that $f(n)=b-1$; but then $f^{3}(n-1)=f(n)+1=b$, so $b \\in R_{3}$. This yields $$ 3 k=\\left|S_{1} \\cup S_{2} \\cup S_{3}\\right| \\leqslant 1+1+\\left|S_{1}\\right|=k+2, $$ or $k \\leqslant 1$. Therefore $k=1$, and the inequality above comes to equality. So we have $S_{1}=\\{a\\}$, $S_{2}=\\{f(a)\\}$, and $S_{3}=\\left\\{f^{2}(a)\\right\\}$ for some $a \\in \\mathbb{Z}_{\\geqslant 0}$, and each one of the three options (i), (ii), and (iii) should be realized exactly once, which means that $$ \\left\\{a, f(a), f^{2}(a)\\right\\}=\\{0, a+1, f(0)+1\\} . $$ III. From (3), we get $a+1 \\in\\left\\{f(a), f^{2}(a)\\right\\}$ (the case $a+1=a$ is impossible). If $a+1=f^{2}(a)$ then we have $f(a+1)=f^{3}(a)=f(a+1)+1$ which is absurd. Therefore $$ f(a)=a+1 $$ Next, again from (3) we have $0 \\in\\left\\{a, f^{2}(a)\\right\\}$. Let us consider these two cases separately. Case 1. Assume that $a=0$, then $f(0)=f(a)=a+1=1$. Also from (3) we get $f(1)=f^{2}(a)=$ $f(0)+1=2$. Now, let us show that $f(n)=n+1$ by induction on $n$; the base cases $n \\leqslant 1$ are established. Next, if $n \\geqslant 2$ then the induction hypothesis implies $$ n+1=f(n-1)+1=f^{3}(n-2)=f^{2}(n-1)=f(n), $$ establishing the step. In this case we have obtained the first of two answers; checking that is satisfies (*) is straightforward. Case 2. Assume now that $f^{2}(a)=0$; then by (3) we get $a=f(0)+1$. By (4) we get $f(a+1)=$ $f^{2}(a)=0$, then $f(0)=f^{3}(a)=f(a+1)+1=1$, hence $a=f(0)+1=2$ and $f(2)=3$ by (4). To summarize, $$ f(0)=1, \\quad f(2)=3, \\quad f(3)=0 . $$ Now let us prove by induction on $m$ that (1) holds for all $n=4 k, 4 k+2,4 k+3$ with $k \\leqslant m$ and for all $n=4 k+1$ with $k0$ and $B(1)=m+2>0$ since $n=2 m$. Therefore $B(x)=A(x+a+b)$. Writing $c=a+b \\geqslant 1$ we compute $$ 0=A(x+c)-B(x)=(3 c-2 m) x^{2}+c(3 c-2 m) x+c^{2}(c-m) . $$ Then we must have $3 c-2 m=c-m=0$, which gives $m=0$, a contradiction. We conclude that $f(x)=t x$ is the only solution.","tier":0} +{"problem_type":"Algebra","problem_label":"A6","problem":"Let $m \\neq 0$ be an integer. Find all polynomials $P(x)$ with real coefficients such that $$ \\left(x^{3}-m x^{2}+1\\right) P(x+1)+\\left(x^{3}+m x^{2}+1\\right) P(x-1)=2\\left(x^{3}-m x+1\\right) P(x) $$ for all real numbers $x$.","solution":"Multiplying (1) by $x$, we rewrite it as $$ x\\left(x^{3}-m x^{2}+1\\right) P(x+1)+x\\left(x^{3}+m x^{2}+1\\right) P(x-1)=[(x+1)+(x-1)]\\left(x^{3}-m x+1\\right) P(x) . $$ After regrouping, it becomes $$ \\left(x^{3}-m x^{2}+1\\right) Q(x)=\\left(x^{3}+m x^{2}+1\\right) Q(x-1) \\text {, } $$ where $Q(x)=x P(x+1)-(x+1) P(x)$. If $\\operatorname{deg} P \\geqslant 2$ then $\\operatorname{deg} Q=\\operatorname{deg} P$, so $Q(x)$ has a finite multiset of complex roots, which we denote $R_{Q}$. Each root is taken with its multiplicity. Then the multiset of complex roots of $Q(x-1)$ is $R_{Q}+1=\\left\\{z+1: z \\in R_{Q}\\right\\}$. Let $\\left\\{x_{1}, x_{2}, x_{3}\\right\\}$ and $\\left\\{y_{1}, y_{2}, y_{3}\\right\\}$ be the multisets of roots of the polynomials $A(x)=x^{3}-m x^{2}+1$ and $B(x)=x^{3}+m x^{2}+1$, respectively. From (2) we get the equality of multisets $$ \\left\\{x_{1}, x_{2}, x_{3}\\right\\} \\cup R_{Q}=\\left\\{y_{1}, y_{2}, y_{3}\\right\\} \\cup\\left(R_{Q}+1\\right) . $$ For every $r \\in R_{Q}$, since $r+1$ is in the set of the right hand side, we must have $r+1 \\in R_{Q}$ or $r+1=x_{i}$ for some $i$. Similarly, since $r$ is in the set of the left hand side, either $r-1 \\in R_{Q}$ or $r=y_{i}$ for some $i$. This implies that, possibly after relabelling $y_{1}, y_{2}, y_{3}$, all the roots of (2) may be partitioned into three chains of the form $\\left\\{y_{i}, y_{i}+1, \\ldots, y_{i}+k_{i}=x_{i}\\right\\}$ for $i=1,2,3$ and some integers $k_{1}, k_{2}, k_{3} \\geqslant 0$. Now we analyze the roots of the polynomial $A_{a}(x)=x^{3}+a x^{2}+1$. Using calculus or elementary methods, we find that the local extrema of $A_{a}(x)$ occur at $x=0$ and $x=-2 a \/ 3$; their values are $A_{a}(0)=1>0$ and $A_{a}(-2 a \/ 3)=1+4 a^{3} \/ 27$, which is positive for integers $a \\geqslant-1$ and negative for integers $a \\leqslant-2$. So when $a \\in \\mathbb{Z}, A_{a}$ has three real roots if $a \\leqslant-2$ and one if $a \\geqslant-1$. Now, since $y_{i}-x_{i} \\in \\mathbb{Z}$ for $i=1,2,3$, the cubics $A_{m}$ and $A_{-m}$ must have the same number of real roots. The previous analysis then implies that $m=1$ or $m=-1$. Therefore the real root $\\alpha$ of $A_{1}(x)=x^{3}+x^{2}+1$ and the real root $\\beta$ of $A_{-1}(x)=x^{3}-x^{2}+1$ must differ by an integer. But this is impossible, because $A_{1}\\left(-\\frac{3}{2}\\right)=-\\frac{1}{8}$ and $A_{1}(-1)=1$ so $-1.5<\\alpha<-1$, while $A_{-1}(-1)=-1$ and $A_{-1}\\left(-\\frac{1}{2}\\right)=\\frac{5}{8}$, so $-1<\\beta<-0.5$. It follows that $\\operatorname{deg} P \\leqslant 1$. Then, as shown in Solution 1, we conclude that the solutions are $P(x)=t x$ for all real numbers $t$.","tier":0} +{"problem_type":"Combinatorics","problem_label":"C1","problem":"Let $n$ be a positive integer. Find the smallest integer $k$ with the following property: Given any real numbers $a_{1}, \\ldots, a_{d}$ such that $a_{1}+a_{2}+\\cdots+a_{d}=n$ and $0 \\leqslant a_{i} \\leqslant 1$ for $i=1,2, \\ldots, d$, it is possible to partition these numbers into $k$ groups (some of which may be empty) such that the sum of the numbers in each group is at most 1 . (Poland)","solution":"If $d=2 n-1$ and $a_{1}=\\cdots=a_{2 n-1}=n \/(2 n-1)$, then each group in such a partition can contain at most one number, since $2 n \/(2 n-1)>1$. Therefore $k \\geqslant 2 n-1$. It remains to show that a suitable partition into $2 n-1$ groups always exists. We proceed by induction on $d$. For $d \\leqslant 2 n-1$ the result is trivial. If $d \\geqslant 2 n$, then since $$ \\left(a_{1}+a_{2}\\right)+\\ldots+\\left(a_{2 n-1}+a_{2 n}\\right) \\leqslant n $$ we may find two numbers $a_{i}, a_{i+1}$ such that $a_{i}+a_{i+1} \\leqslant 1$. We \"merge\" these two numbers into one new number $a_{i}+a_{i+1}$. By the induction hypothesis, a suitable partition exists for the $d-1$ numbers $a_{1}, \\ldots, a_{i-1}, a_{i}+a_{i+1}, a_{i+2}, \\ldots, a_{d}$. This induces a suitable partition for $a_{1}, \\ldots, a_{d}$.","tier":0} +{"problem_type":"Combinatorics","problem_label":"C1","problem":"Let $n$ be a positive integer. Find the smallest integer $k$ with the following property: Given any real numbers $a_{1}, \\ldots, a_{d}$ such that $a_{1}+a_{2}+\\cdots+a_{d}=n$ and $0 \\leqslant a_{i} \\leqslant 1$ for $i=1,2, \\ldots, d$, it is possible to partition these numbers into $k$ groups (some of which may be empty) such that the sum of the numbers in each group is at most 1 . (Poland)","solution":"We will show that it is even possible to split the sequence $a_{1}, \\ldots, a_{d}$ into $2 n-1$ contiguous groups so that the sum of the numbers in each groups does not exceed 1. Consider a segment $S$ of length $n$, and partition it into segments $S_{1}, \\ldots, S_{d}$ of lengths $a_{1}, \\ldots, a_{d}$, respectively, as shown below. Consider a second partition of $S$ into $n$ equal parts by $n-1$ \"empty dots\". | $a_{1}$ | $a_{2}$ | $a_{3}$ | $a_{4}$ | $a_{5}$ | $a_{6}$ | $a_{7}$ | $a_{8}, a_{9}, a_{10}$ | | :--- | :--- | :--- | :--- | :--- | :--- | :--- | :--- | :--- | | | | | | | | | | Assume that the $n-1$ empty dots are in segments $S_{i_{1}}, \\ldots, S_{i_{n-1}}$. (If a dot is on the boundary of two segments, we choose the right segment). These $n-1$ segments are distinct because they have length at most 1. Consider the partition: $$ \\left\\{a_{1}, \\ldots, a_{i_{1}-1}\\right\\},\\left\\{a_{i_{1}}\\right\\},\\left\\{a_{i_{1}+1}, \\ldots, a_{i_{2}-1}\\right\\},\\left\\{a_{i_{2}}\\right\\}, \\ldots\\left\\{a_{i_{n-1}}\\right\\},\\left\\{a_{i_{n-1}+1}, \\ldots, a_{d}\\right\\} $$ In the example above, this partition is $\\left\\{a_{1}, a_{2}\\right\\},\\left\\{a_{3}\\right\\},\\left\\{a_{4}, a_{5}\\right\\},\\left\\{a_{6}\\right\\}, \\varnothing,\\left\\{a_{7}\\right\\},\\left\\{a_{8}, a_{9}, a_{10}\\right\\}$. We claim that in this partition, the sum of the numbers in this group is at most 1 . For the sets $\\left\\{a_{i_{t}}\\right\\}$ this is obvious since $a_{i_{t}} \\leqslant 1$. For the sets $\\left\\{a_{i_{t}}+1, \\ldots, a_{i_{t+1}-1}\\right\\}$ this follows from the fact that the corresponding segments lie between two neighboring empty dots, or between an endpoint of $S$ and its nearest empty dot. Therefore the sum of their lengths cannot exceed 1.","tier":0} +{"problem_type":"Combinatorics","problem_label":"C1","problem":"Let $n$ be a positive integer. Find the smallest integer $k$ with the following property: Given any real numbers $a_{1}, \\ldots, a_{d}$ such that $a_{1}+a_{2}+\\cdots+a_{d}=n$ and $0 \\leqslant a_{i} \\leqslant 1$ for $i=1,2, \\ldots, d$, it is possible to partition these numbers into $k$ groups (some of which may be empty) such that the sum of the numbers in each group is at most 1 . (Poland)","solution":"First put all numbers greater than $\\frac{1}{2}$ in their own groups. Then, form the remaining groups as follows: For each group, add new $a_{i} \\mathrm{~S}$ one at a time until their sum exceeds $\\frac{1}{2}$. Since the last summand is at most $\\frac{1}{2}$, this group has sum at most 1 . Continue this procedure until we have used all the $a_{i}$ s. Notice that the last group may have sum less than $\\frac{1}{2}$. If the sum of the numbers in the last two groups is less than or equal to 1, we merge them into one group. In the end we are left with $m$ groups. If $m=1$ we are done. Otherwise the first $m-2$ have sums greater than $\\frac{1}{2}$ and the last two have total sum greater than 1 . Therefore $n>(m-2) \/ 2+1$ so $m \\leqslant 2 n-1$ as desired. Comment 1. The original proposal asked for the minimal value of $k$ when $n=2$. Comment 2. More generally, one may ask the same question for real numbers between 0 and 1 whose sum is a real number $r$. In this case the smallest value of $k$ is $k=\\lceil 2 r\\rceil-1$, as Solution 3 shows. Solutions 1 and 2 lead to the slightly weaker bound $k \\leqslant 2\\lceil r\\rceil-1$. This is actually the optimal bound for partitions into consecutive groups, which are the ones contemplated in these two solutions. To see this, assume that $r$ is not an integer and let $c=(r+1-\\lceil r\\rceil) \/(1+\\lceil r\\rceil)$. One easily checks that $0N$.","tier":0} +{"problem_type":"Combinatorics","problem_label":"C3","problem":"A crazy physicist discovered a new kind of particle which he called an imon, after some of them mysteriously appeared in his lab. Some pairs of imons in the lab can be entangled, and each imon can participate in many entanglement relations. The physicist has found a way to perform the following two kinds of operations with these particles, one operation at a time. (i) If some imon is entangled with an odd number of other imons in the lab, then the physicist can destroy it. (ii) At any moment, he may double the whole family of imons in his lab by creating a copy $I^{\\prime}$ of each imon $I$. During this procedure, the two copies $I^{\\prime}$ and $J^{\\prime}$ become entangled if and only if the original imons $I$ and $J$ are entangled, and each copy $I^{\\prime}$ becomes entangled with its original imon $I$; no other entanglements occur or disappear at this moment. Prove that the physicist may apply a sequence of such operations resulting in a family of imons, no two of which are entangled. (Japan)","solution":"Let us consider a graph with the imons as vertices, and two imons being connected if and only if they are entangled. Recall that a proper coloring of a graph $G$ is a coloring of its vertices in several colors so that every two connected vertices have different colors. Lemma. Assume that a graph $G$ admits a proper coloring in $n$ colors $(n>1)$. Then one may perform a sequence of operations resulting in a graph which admits a proper coloring in $n-1$ colors. Proof. Let us apply repeatedly operation $(i)$ to any appropriate vertices while it is possible. Since the number of vertices decreases, this process finally results in a graph where all the degrees are even. Surely this graph also admits a proper coloring in $n$ colors $1, \\ldots, n$; let us fix this coloring. Now apply the operation (ii) to this graph. A proper coloring of the resulting graph in $n$ colors still exists: one may preserve the colors of the original vertices and color the vertex $I^{\\prime}$ in a color $k+1(\\bmod n)$ if the vertex $I$ has color $k$. Then two connected original vertices still have different colors, and so do their two connected copies. On the other hand, the vertices $I$ and $I^{\\prime}$ have different colors since $n>1$. All the degrees of the vertices in the resulting graph are odd, so one may apply operation $(i)$ to delete consecutively all the vertices of color $n$ one by one; no two of them are connected by an edge, so their degrees do not change during the process. Thus, we obtain a graph admitting a proper coloring in $n-1$ colors, as required. The lemma is proved. Now, assume that a graph $G$ has $n$ vertices; then it admits a proper coloring in $n$ colors. Applying repeatedly the lemma we finally obtain a graph admitting a proper coloring in one color, that is - a graph with no edges, as required.","tier":0} +{"problem_type":"Combinatorics","problem_label":"C3","problem":"A crazy physicist discovered a new kind of particle which he called an imon, after some of them mysteriously appeared in his lab. Some pairs of imons in the lab can be entangled, and each imon can participate in many entanglement relations. The physicist has found a way to perform the following two kinds of operations with these particles, one operation at a time. (i) If some imon is entangled with an odd number of other imons in the lab, then the physicist can destroy it. (ii) At any moment, he may double the whole family of imons in his lab by creating a copy $I^{\\prime}$ of each imon $I$. During this procedure, the two copies $I^{\\prime}$ and $J^{\\prime}$ become entangled if and only if the original imons $I$ and $J$ are entangled, and each copy $I^{\\prime}$ becomes entangled with its original imon $I$; no other entanglements occur or disappear at this moment. Prove that the physicist may apply a sequence of such operations resulting in a family of imons, no two of which are entangled. (Japan)","solution":"Again, we will use the graph language. I. We start with the following observation. Lemma. Assume that a graph $G$ contains an isolated vertex $A$, and a graph $G^{\\circ}$ is obtained from $G$ by deleting this vertex. Then, if one can apply a sequence of operations which makes a graph with no edges from $G^{\\circ}$, then such a sequence also exists for $G$. Proof. Consider any operation applicable to $G^{\\circ}$ resulting in a graph $G_{1}^{\\circ}$; then there exists a sequence of operations applicable to $G$ and resulting in a graph $G_{1}$ differing from $G_{1}^{\\circ}$ by an addition of an isolated vertex $A$. Indeed, if this operation is of type $(i)$, then one may simply repeat it in $G$. Otherwise, the operation is of type (ii), and one may apply it to $G$ and then delete the vertex $A^{\\prime}$ (it will have degree 1 ). Thus one may change the process for $G^{\\circ}$ into a corresponding process for $G$ step by step. In view of this lemma, if at some moment a graph contains some isolated vertex, then we may simply delete it; let us call this operation (iii). II. Let $V=\\left\\{A_{1}^{0}, \\ldots, A_{n}^{0}\\right\\}$ be the vertices of the initial graph. Let us describe which graphs can appear during our operations. Assume that operation (ii) was applied $m$ times. If these were the only operations applied, then the resulting graph $G_{n}^{m}$ has the set of vertices which can be enumerated as $$ V_{n}^{m}=\\left\\{A_{i}^{j}: 1 \\leqslant i \\leqslant n, 0 \\leqslant j \\leqslant 2^{m}-1\\right\\} $$ where $A_{i}^{0}$ is the common \"ancestor\" of all the vertices $A_{i}^{j}$, and the binary expansion of $j$ (adjoined with some zeroes at the left to have $m$ digits) \"keeps the history\" of this vertex: the $d$ th digit from the right is 0 if at the $d$ th doubling the ancestor of $A_{i}^{j}$ was in the original part, and this digit is 1 if it was in the copy. Next, the two vertices $A_{i}^{j}$ and $A_{k}^{\\ell}$ in $G_{n}^{m}$ are connected with an edge exactly if either (1) $j=\\ell$ and there was an edge between $A_{i}^{0}$ and $A_{k}^{0}$ (so these vertices appeared at the same application of operation (ii)); or (2) $i=k$ and the binary expansions of $j$ and $\\ell$ differ in exactly one digit (so their ancestors became connected as a copy and the original vertex at some application of (ii)). Now, if some operations $(i)$ were applied during the process, then simply some vertices in $G_{n}^{m}$ disappeared. So, in any case the resulting graph is some induced subgraph of $G_{n}^{m}$. III. Finally, we will show that from each (not necessarily induced) subgraph of $G_{n}^{m}$ one can obtain a graph with no vertices by applying operations $(i),(i i)$ and $(i i i)$. We proceed by induction on $n$; the base case $n=0$ is trivial. For the induction step, let us show how to apply several operations so as to obtain a graph containing no vertices of the form $A_{n}^{j}$ for $j \\in \\mathbb{Z}$. We will do this in three steps. Step 1. We apply repeatedly operation (i) to any appropriate vertices while it is possible. In the resulting graph, all vertices have even degrees. Step 2. Apply operation (ii) obtaining a subgraph of $G_{n}^{m+1}$ with all degrees being odd. In this graph, we delete one by one all the vertices $A_{n}^{j}$ where the sum of the binary digits of $j$ is even; it is possible since there are no edges between such vertices, so all their degrees remain odd. After that, we delete all isolated vertices. Step 3. Finally, consider any remaining vertex $A_{n}^{j}$ (then the sum of digits of $j$ is odd). If its degree is odd, then we simply delete it. Otherwise, since $A_{n}^{j}$ is not isolated, we consider any vertex adjacent to it. It has the form $A_{k}^{j}$ for some $k\\sqrt[3]{6 n}$, we will prove that there exist subsets $X$ and $Y$ of $S$ such that $|X|<|Y|$ and $\\sum_{x \\in X} x=\\sum_{y \\in Y} y$. Then, deleting the elements of $Y$ from our partition and adding the elements of $X$ to it, we obtain an $A$-partition of $n$ into less than $k_{\\text {min }}$ parts, which is the desired contradiction. For each positive integer $k \\leqslant s$, we consider the $k$-element subset $$ S_{1,0}^{k}:=\\left\\{b_{1}, \\ldots, b_{k}\\right\\} $$ as well as the following $k$-element subsets $S_{i, j}^{k}$ of $S$ : $$ S_{i, j}^{k}:=\\left\\{b_{1}, \\ldots, b_{k-i}, b_{k-i+j+1}, b_{s-i+2}, \\ldots, b_{s}\\right\\}, \\quad i=1, \\ldots, k, \\quad j=1, \\ldots, s-k $$ Pictorially, if we represent the elements of $S$ by a sequence of dots in increasing order, and represent a subset of $S$ by shading in the appropriate dots, we have: ![](https:\/\/cdn.mathpix.com\/cropped\/2024_04_17_e0914cb4f8055c731538g-27.jpg?height=124&width=967&top_left_y=1388&top_left_x=579) Denote by $\\Sigma_{i, j}^{k}$ the sum of elements in $S_{i, j}^{k}$. Clearly, $\\Sigma_{1,0}^{k}$ is the minimum sum of a $k$-element subset of $S$. Next, for all appropriate indices $i$ and $j$ we have $$ \\Sigma_{i, j}^{k}=\\Sigma_{i, j+1}^{k}+b_{k-i+j+1}-b_{k-i+j+2}<\\Sigma_{i, j+1}^{k} \\quad \\text { and } \\quad \\sum_{i, s-k}^{k}=\\sum_{i+1,1}^{k}+b_{k-i}-b_{k-i+1}<\\Sigma_{i+1,1}^{k} \\text {. } $$ Therefore $$ 1 \\leqslant \\Sigma_{1,0}^{k}<\\Sigma_{1,1}^{k}<\\Sigma_{1,2}^{k}<\\cdots<\\Sigma_{1, s-k}^{k}<\\Sigma_{2,1}^{k}<\\cdots<\\Sigma_{2, s-k}^{k}<\\Sigma_{3,1}^{k}<\\cdots<\\Sigma_{k, s-k}^{k} \\leqslant n . $$ To see this in the picture, we start with the $k$ leftmost points marked. At each step, we look for the rightmost point which can move to the right, and move it one unit to the right. We continue until the $k$ rightmost points are marked. As we do this, the corresponding sums clearly increase. For each $k$ we have found $k(s-k)+1$ different integers of the form $\\Sigma_{i, j}^{k}$ between 1 and $n$. As we vary $k$, the total number of integers we are considering is $$ \\sum_{k=1}^{s}(k(s-k)+1)=s \\cdot \\frac{s(s+1)}{2}-\\frac{s(s+1)(2 s+1)}{6}+s=\\frac{s\\left(s^{2}+5\\right)}{6}>\\frac{s^{3}}{6}>n . $$ Since they are between 1 and $n$, at least two of these integers are equal. Consequently, there exist $1 \\leqslant k\\sqrt[3]{6 n}>1$. Without loss of generality we assume that $a_{k_{\\min }}=b_{s}$. Let us distinguish two cases. Case 1. $b_{s} \\geqslant \\frac{s(s-1)}{2}+1$. Consider the partition $n-b_{s}=a_{1}+\\cdots+a_{k_{\\min }-1}$, which is clearly a minimum $A$-partition of $n-b_{s}$ with at least $s-1 \\geqslant 1$ different parts. Now, from $n<\\frac{s^{3}}{6}$ we obtain $$ n-b_{s} \\leqslant n-\\frac{s(s-1)}{2}-1<\\frac{s^{3}}{6}-\\frac{s(s-1)}{2}-1<\\frac{(s-1)^{3}}{6} $$ so $s-1>\\sqrt[3]{6\\left(n-b_{s}\\right)}$, which contradicts the choice of $n$. Case 2. $b_{s} \\leqslant \\frac{s(s-1)}{2}$. Set $b_{0}=0, \\Sigma_{0,0}=0$, and $\\Sigma_{i, j}=b_{1}+\\cdots+b_{i-1}+b_{j}$ for $1 \\leqslant i \\leqslant jb_{s}$ such sums; so at least two of them, say $\\Sigma_{i, j}$ and $\\Sigma_{i^{\\prime}, j^{\\prime}}$, are congruent modulo $b_{s}$ (where $(i, j) \\neq\\left(i^{\\prime}, j^{\\prime}\\right)$ ). This means that $\\Sigma_{i, j}-\\Sigma_{i^{\\prime}, j^{\\prime}}=r b_{s}$ for some integer $r$. Notice that for $i \\leqslant ji^{\\prime}$. Next, we observe that $\\Sigma_{i, j}-\\Sigma_{i^{\\prime}, j^{\\prime}}=\\left(b_{i^{\\prime}}-b_{j^{\\prime}}\\right)+b_{j}+b_{i^{\\prime}+1}+\\cdots+b_{i-1}$ and $b_{i^{\\prime}} \\leqslant b_{j^{\\prime}}$ imply $$ -b_{s}<-b_{j^{\\prime}}<\\Sigma_{i, j}-\\Sigma_{i^{\\prime}, j^{\\prime}}<\\left(i-i^{\\prime}\\right) b_{s}, $$ so $0 \\leqslant r \\leqslant i-i^{\\prime}-1$. Thus, we may remove the $i$ terms of $\\Sigma_{i, j}$ in our $A$-partition, and replace them by the $i^{\\prime}$ terms of $\\Sigma_{i^{\\prime}, j^{\\prime}}$ and $r$ terms equal to $b_{s}$, for a total of $r+i^{\\prime}a_{k+1} \\geqslant 1$, which shows that $$ n=a_{1}+\\ldots+a_{k+1} $$ is an $A$-partition of $n$ into $k+1$ different parts. Since $k h3$. Finally, in the sequence $d(a, y), d\\left(a, b_{y}\\right), d\\left(a, c_{y}\\right), d\\left(a, d_{y}\\right)$ the neighboring terms differ by at most 1 , the first term is less than 3 , and the last one is greater than 3 ; thus there exists one which is equal to 3 , as required. Comment 1. The upper bound 2550 is sharp. This can be seen by means of various examples; one of them is the \"Roman Empire\": it has one capital, called \"Rome\", that is connected to 51 semicapitals by internally disjoint paths of length 3. Moreover, each of these semicapitals is connected to 50 rural cities by direct flights. Comment 2. Observe that, under the conditions of the problem, there exists no bound for the size of $S_{1}(x)$ or $S_{2}(x)$. Comment 3. The numbers 100 and 2550 appearing in the statement of the problem may be replaced by $n$ and $\\left\\lfloor\\frac{(n+1)^{2}}{4}\\right\\rfloor$ for any positive integer $n$. Still more generally, one can also replace the pair $(3,4)$ of distances under consideration by any pair $(r, s)$ of positive integers satisfying $rn$ is equivalent to the case $t0$ is very small. ![](https:\/\/cdn.mathpix.com\/cropped\/2024_04_17_e0914cb4f8055c731538g-35.jpg?height=585&width=607&top_left_y=797&top_left_x=756) Figure 3 If $0 \\leqslant a, b, c, d \\leqslant n$ satisfy $a+c=b+d$, then $a \\alpha+c \\alpha=b \\alpha+d \\alpha$, so the chord from $a$ to $c$ is parallel to the chord from $b$ to $d$ in $A(\\alpha)$. Hence in a cyclic arrangement all $k$-chords are parallel. In particular every cyclic arrangement is beautiful. Next we show that there are exactly $N+1$ distinct cyclic arrangements. To see this, let us see how $A(\\alpha)$ changes as we increase $\\alpha$ from 0 to 1 . The order of points $p$ and $q$ changes precisely when we cross a value $\\alpha=f$ such that $\\{p f\\}=\\{q f\\}$; this can only happen if $f$ is one of the $N$ fractions $f_{1}, \\ldots, f_{N}$. Therefore there are at most $N+1$ different cyclic arrangements. To show they are all distinct, recall that $f_{i}=a_{i} \/ b_{i}$ and let $\\epsilon>0$ be a very small number. In the arrangement $A\\left(f_{i}+\\epsilon\\right)$, point $k$ lands at $\\frac{k a_{i}\\left(\\bmod b_{i}\\right)}{b_{i}}+k \\epsilon$. Therefore the points are grouped into $b_{i}$ clusters next to the points $0, \\frac{1}{b_{i}}, \\ldots, \\frac{b_{i}-1}{b_{i}}$ of the circle. The cluster following $\\frac{k}{b_{i}}$ contains the numbers congruent to $k a_{i}^{-1}$ modulo $b_{i}$, listed clockwise in increasing order. It follows that the first number after 0 in $A\\left(f_{i}+\\epsilon\\right)$ is $b_{i}$, and the first number after 0 which is less than $b_{i}$ is $a_{i}^{-1}\\left(\\bmod b_{i}\\right)$, which uniquely determines $a_{i}$. In this way we can recover $f_{i}$ from the cyclic arrangement. Note also that $A\\left(f_{i}+\\epsilon\\right)$ is not the trivial arrangement where we list $0,1, \\ldots, n$ in order clockwise. It follows that the $N+1$ cyclic arrangements $A(\\epsilon), A\\left(f_{1}+\\epsilon\\right), \\ldots, A\\left(f_{N}+\\epsilon\\right)$ are distinct. Let us record an observation which will be useful later: $$ \\text { if } f_{i}<\\alpha1 \/ 2^{m}$. Next, any interval blackened by $B$ before the $r$ th move which intersects $\\left(x_{r}, x_{r+1}\\right)$ should be contained in $\\left[x_{r}, x_{r+1}\\right]$; by (ii), all such intervals have different lengths not exceeding $1 \/ 2^{m}$, so the total amount of ink used for them is less than $2 \/ 2^{m}$. Thus, the amount of ink used for the segment $\\left[0, x_{r+1}\\right]$ does not exceed the sum of $2 \/ 2^{m}, 3 x_{r}$ (used for $\\left[0, x_{r}\\right]$ ), and $1 \/ 2^{m}$ used for the segment $I_{0}^{r}$. In total it gives at most $3\\left(x_{r}+1 \/ 2^{m}\\right)<3\\left(x_{r}+\\alpha\\right)=3 x_{r+1}$. Thus condition $(i)$ is also verified in this case. The claim is proved. Finally, we can perform the desired estimation. Consider any situation in the game, say after the $(r-1)$ st move; assume that the segment $[0,1]$ is not completely black. By $(i i)$, in the segment $\\left[x_{r}, 1\\right]$ player $B$ has colored several segments of different lengths; all these lengths are negative powers of 2 not exceeding $1-x_{r}$; thus the total amount of ink used for this interval is at most $2\\left(1-x_{r}\\right)$. Using $(i)$, we obtain that the total amount of ink used is at most $3 x_{r}+2\\left(1-x_{r}\\right)<3$. Thus the pot is not empty, and therefore $A$ never wins. Comment 1. Notice that this strategy works even if the pot contains initially only 3 units of ink. Comment 2. There exist other strategies for $B$ allowing him to prevent emptying the pot before the whole interval is colored. On the other hand, let us mention some idea which does not work. Player $B$ could try a strategy in which the set of blackened points in each round is an interval of the type $[0, x]$. Such a strategy cannot work (even if there is more ink available). Indeed, under the assumption that $B$ uses such a strategy, let us prove by induction on $s$ the following statement: For any positive integer $s$, player $A$ has a strategy picking only positive integers $m \\leqslant s$ in which, if player $B$ ever paints a point $x \\geqslant 1-1 \/ 2^{s}$ then after some move, exactly the interval $\\left[0,1-1 \/ 2^{s}\\right]$ is blackened, and the amount of ink used up to this moment is at least s\/2. For the base case $s=1$, player $A$ just picks $m=1$ in the first round. If for some positive integer $k$ player $A$ has such a strategy, for $s+1$ he can first rescale his strategy to the interval $[0,1 \/ 2]$ (sending in each round half of the amount of ink he would give by the original strategy). Thus, after some round, the interval $\\left[0,1 \/ 2-1 \/ 2^{s+1}\\right]$ becomes blackened, and the amount of ink used is at least $s \/ 4$. Now player $A$ picks $m=1 \/ 2$, and player $B$ spends $1 \/ 2$ unit of ink to blacken the interval [0,1\/2]. After that, player $A$ again rescales his strategy to the interval $[1 \/ 2,1]$, and player $B$ spends at least $s \/ 4$ units of ink to blacken the interval $\\left[1 \/ 2,1-1 \/ 2^{s+1}\\right]$, so he spends in total at least $s \/ 4+1 \/ 2+s \/ 4=(s+1) \/ 2$ units of ink. Comment 3. In order to avoid finiteness issues, the statement could be replaced by the following one: Players $A$ and $B$ play a paintful game on the real numbers. Player $A$ has a paint pot with four units of black ink. A quantity $p$ of this ink suffices to blacken a (closed) real interval of length $p$. In the beginning of the game, player $A$ chooses (and announces) a positive integer $N$. In every round, player $A$ picks some positive integer $m \\leqslant N$ and provides $1 \/ 2^{m}$ units of ink from the pot. The player $B$ picks an integer $k$ and blackens the interval from $k \/ 2^{m}$ to $(k+1) \/ 2^{m}$ (some parts of this interval may happen to be blackened before). The goal of player $A$ is to reach a situation where the pot is empty and the interval $[0,1]$ is not completely blackened. Decide whether there exists a strategy for player A to win. However, the Problem Selection Committee believes that this version may turn out to be harder than the original one.","tier":0} +{"problem_type":"Geometry","problem_label":"G1","problem":"Let $A B C$ be an acute-angled triangle with orthocenter $H$, and let $W$ be a point on side $B C$. Denote by $M$ and $N$ the feet of the altitudes from $B$ and $C$, respectively. Denote by $\\omega_{1}$ the circumcircle of $B W N$, and let $X$ be the point on $\\omega_{1}$ which is diametrically opposite to $W$. Analogously, denote by $\\omega_{2}$ the circumcircle of $C W M$, and let $Y$ be the point on $\\omega_{2}$ which is diametrically opposite to $W$. Prove that $X, Y$ and $H$ are collinear. (Thaliand)","solution":"Let $L$ be the foot of the altitude from $A$, and let $Z$ be the second intersection point of circles $\\omega_{1}$ and $\\omega_{2}$, other than $W$. We show that $X, Y, Z$ and $H$ lie on the same line. Due to $\\angle B N C=\\angle B M C=90^{\\circ}$, the points $B, C, N$ and $M$ are concyclic; denote their circle by $\\omega_{3}$. Observe that the line $W Z$ is the radical axis of $\\omega_{1}$ and $\\omega_{2}$; similarly, $B N$ is the radical axis of $\\omega_{1}$ and $\\omega_{3}$, and $C M$ is the radical axis of $\\omega_{2}$ and $\\omega_{3}$. Hence $A=B N \\cap C M$ is the radical center of the three circles, and therefore $W Z$ passes through $A$. Since $W X$ and $W Y$ are diameters in $\\omega_{1}$ and $\\omega_{2}$, respectively, we have $\\angle W Z X=\\angle W Z Y=90^{\\circ}$, so the points $X$ and $Y$ lie on the line through $Z$, perpendicular to $W Z$. ![](https:\/\/cdn.mathpix.com\/cropped\/2024_04_17_e0914cb4f8055c731538g-39.jpg?height=770&width=1114&top_left_y=1103&top_left_x=511) The quadrilateral $B L H N$ is cyclic, because it has two opposite right angles. From the power of $A$ with respect to the circles $\\omega_{1}$ and $B L H N$ we find $A L \\cdot A H=A B \\cdot A N=A W \\cdot A Z$. If $H$ lies on the line $A W$ then this implies $H=Z$ immediately. Otherwise, by $\\frac{A Z}{A H}=\\frac{A L}{A W}$ the triangles $A H Z$ and $A W L$ are similar. Then $\\angle H Z A=\\angle W L A=90^{\\circ}$, so the point $H$ also lies on the line $X Y Z$. Comment. The original proposal also included a second statement: Let $P$ be the point on $\\omega_{1}$ such that $W P$ is parallel to $C N$, and let $Q$ be the point on $\\omega_{2}$ such that $W Q$ is parallel to $B M$. Prove that $P, Q$ and $H$ are collinear if and only if $B W=C W$ or $A W \\perp B C$. The Problem Selection Committee considered the first part more suitable for the competition.","tier":0} +{"problem_type":"Geometry","problem_label":"G2","problem":"Let $\\omega$ be the circumcircle of a triangle $A B C$. Denote by $M$ and $N$ the midpoints of the sides $A B$ and $A C$, respectively, and denote by $T$ the midpoint of the $\\operatorname{arc} B C$ of $\\omega$ not containing $A$. The circumcircles of the triangles $A M T$ and $A N T$ intersect the perpendicular bisectors of $A C$ and $A B$ at points $X$ and $Y$, respectively; assume that $X$ and $Y$ lie inside the triangle $A B C$. The lines $M N$ and $X Y$ intersect at $K$. Prove that $K A=K T$. (Iran)","solution":"Let $O$ be the center of $\\omega$, thus $O=M Y \\cap N X$. Let $\\ell$ be the perpendicular bisector of $A T$ (it also passes through $O$ ). Denote by $r$ the operation of reflection about $\\ell$. Since $A T$ is the angle bisector of $\\angle B A C$, the line $r(A B)$ is parallel to $A C$. Since $O M \\perp A B$ and $O N \\perp A C$, this means that the line $r(O M)$ is parallel to the line $O N$ and passes through $O$, so $r(O M)=O N$. Finally, the circumcircle $\\gamma$ of the triangle $A M T$ is symmetric about $\\ell$, so $r(\\gamma)=\\gamma$. Thus the point $M$ maps to the common point of $O N$ with the arc $A M T$ of $\\gamma$ - that is, $r(M)=X$. Similarly, $r(N)=Y$. Thus, we get $r(M N)=X Y$, and the common point $K$ of $M N$ nd $X Y$ lies on $\\ell$. This means exactly that $K A=K T$. ![](https:\/\/cdn.mathpix.com\/cropped\/2024_04_17_e0914cb4f8055c731538g-40.jpg?height=748&width=707&top_left_y=981&top_left_x=709)","tier":0} +{"problem_type":"Geometry","problem_label":"G2","problem":"Let $\\omega$ be the circumcircle of a triangle $A B C$. Denote by $M$ and $N$ the midpoints of the sides $A B$ and $A C$, respectively, and denote by $T$ the midpoint of the $\\operatorname{arc} B C$ of $\\omega$ not containing $A$. The circumcircles of the triangles $A M T$ and $A N T$ intersect the perpendicular bisectors of $A C$ and $A B$ at points $X$ and $Y$, respectively; assume that $X$ and $Y$ lie inside the triangle $A B C$. The lines $M N$ and $X Y$ intersect at $K$. Prove that $K A=K T$. (Iran)","solution":"Let $L$ be the second common point of the line $A C$ with the circumcircle $\\gamma$ of the triangle $A M T$. From the cyclic quadrilaterals $A B T C$ and $A M T L$ we get $\\angle B T C=180^{\\circ}-$ $\\angle B A C=\\angle M T L$, which implies $\\angle B T M=\\angle C T L$. Since $A T$ is an angle bisector in these quadrilaterals, we have $B T=T C$ and $M T=T L$. Thus the triangles $B T M$ and $C T L$ are congruent, so $C L=B M=A M$. Let $X^{\\prime}$ be the common point of the line $N X$ with the external bisector of $\\angle B A C$; notice that it lies outside the triangle $A B C$. Then we have $\\angle T A X^{\\prime}=90^{\\circ}$ and $X^{\\prime} A=X^{\\prime} C$, so we get $\\angle X^{\\prime} A M=90^{\\circ}+\\angle B A C \/ 2=180^{\\circ}-\\angle X^{\\prime} A C=180^{\\circ}-\\angle X^{\\prime} C A=\\angle X^{\\prime} C L$. Thus the triangles $X^{\\prime} A M$ and $X^{\\prime} C L$ are congruent, and therefore $$ \\angle M X^{\\prime} L=\\angle A X^{\\prime} C+\\left(\\angle C X^{\\prime} L-\\angle A X^{\\prime} M\\right)=\\angle A X^{\\prime} C=180^{\\circ}-2 \\angle X^{\\prime} A C=\\angle B A C=\\angle M A L . $$ This means that $X^{\\prime}$ lies on $\\gamma$. Thus we have $\\angle T X N=\\angle T X X^{\\prime}=\\angle T A X^{\\prime}=90^{\\circ}$, so $T X \\| A C$. Then $\\angle X T A=\\angle T A C=$ $\\angle T A M$, so the cyclic quadrilateral MATX is an isosceles trapezoid. Similarly, $N A T Y$ is an isosceles trapezoid, so again the lines $M N$ and $X Y$ are the reflections of each other about the perpendicular bisector of $A T$. Thus $K$ belongs to this perpendicular bisector. ![](https:\/\/cdn.mathpix.com\/cropped\/2024_04_17_e0914cb4f8055c731538g-41.jpg?height=797&width=1001&top_left_y=214&top_left_x=562) Comment. There are several different ways of showing that the points $X$ and $M$ are symmetrical with respect to $\\ell$. For instance, one can show that the quadrilaterals $A M O N$ and $T X O Y$ are congruent. We chose Solution 1 as a simple way of doing it. On the other hand, Solution 2 shows some other interesting properties of the configuration. Let us define $Y^{\\prime}$, analogously to $X^{\\prime}$, as the common point of $M Y$ and the external bisector of $\\angle B A C$. One may easily see that in general the lines $M N$ and $X^{\\prime} Y^{\\prime}$ (which is the external bisector of $\\angle B A C$ ) do not intersect on the perpendicular bisector of $A T$. Thus, any solution should involve some argument using the choice of the intersection points $X$ and $Y$.","tier":0} +{"problem_type":"Geometry","problem_label":"G3","problem":"In a triangle $A B C$, let $D$ and $E$ be the feet of the angle bisectors of angles $A$ and $B$, respectively. A rhombus is inscribed into the quadrilateral $A E D B$ (all vertices of the rhombus lie on different sides of $A E D B$ ). Let $\\varphi$ be the non-obtuse angle of the rhombus. Prove that $\\varphi \\leqslant \\max \\{\\angle B A C, \\angle A B C\\}$. (Serbia)","solution":"Let $K, L, M$, and $N$ be the vertices of the rhombus lying on the sides $A E, E D, D B$, and $B A$, respectively. Denote by $d(X, Y Z)$ the distance from a point $X$ to a line $Y Z$. Since $D$ and $E$ are the feet of the bisectors, we have $d(D, A B)=d(D, A C), d(E, A B)=d(E, B C)$, and $d(D, B C)=d(E, A C)=0$, which implies $$ d(D, A C)+d(D, B C)=d(D, A B) \\quad \\text { and } \\quad d(E, A C)+d(E, B C)=d(E, A B) $$ Since $L$ lies on the segment $D E$ and the relation $d(X, A C)+d(X, B C)=d(X, A B)$ is linear in $X$ inside the triangle, these two relations imply $$ d(L, A C)+d(L, B C)=d(L, A B) . $$ Denote the angles as in the figure below, and denote $a=K L$. Then we have $d(L, A C)=a \\sin \\mu$ and $d(L, B C)=a \\sin \\nu$. Since $K L M N$ is a parallelogram lying on one side of $A B$, we get $$ d(L, A B)=d(L, A B)+d(N, A B)=d(K, A B)+d(M, A B)=a(\\sin \\delta+\\sin \\varepsilon) $$ Thus the condition (1) reads $$ \\sin \\mu+\\sin \\nu=\\sin \\delta+\\sin \\varepsilon $$ ![](https:\/\/cdn.mathpix.com\/cropped\/2024_04_17_e0914cb4f8055c731538g-42.jpg?height=593&width=653&top_left_y=1384&top_left_x=736) If one of the angles $\\alpha$ and $\\beta$ is non-acute, then the desired inequality is trivial. So we assume that $\\alpha, \\beta<\\pi \/ 2$. It suffices to show then that $\\psi=\\angle N K L \\leqslant \\max \\{\\alpha, \\beta\\}$. Assume, to the contrary, that $\\psi>\\max \\{\\alpha, \\beta\\}$. Since $\\mu+\\psi=\\angle C K N=\\alpha+\\delta$, by our assumption we obtain $\\mu=(\\alpha-\\psi)+\\delta<\\delta$. Similarly, $\\nu<\\varepsilon$. Next, since $K N \\| M L$, we have $\\beta=\\delta+\\nu$, so $\\delta<\\beta<\\pi \/ 2$. Similarly, $\\varepsilon<\\pi \/ 2$. Finally, by $\\mu<\\delta<\\pi \/ 2$ and $\\nu<\\varepsilon<\\pi \/ 2$, we obtain $$ \\sin \\mu<\\sin \\delta \\quad \\text { and } \\quad \\sin \\nu<\\sin \\varepsilon $$ This contradicts (2). Comment. One can see that the equality is achieved if $\\alpha=\\beta$ for every rhombus inscribed into the quadrilateral $A E D B$.","tier":0} +{"problem_type":"Geometry","problem_label":"G4","problem":"Let $A B C$ be a triangle with $\\angle B>\\angle C$. Let $P$ and $Q$ be two different points on line $A C$ such that $\\angle P B A=\\angle Q B A=\\angle A C B$ and $A$ is located between $P$ and $C$. Suppose that there exists an interior point $D$ of segment $B Q$ for which $P D=P B$. Let the ray $A D$ intersect the circle $A B C$ at $R \\neq A$. Prove that $Q B=Q R$. (Georgia)","solution":"Denote by $\\omega$ the circumcircle of the triangle $A B C$, and let $\\angle A C B=\\gamma$. Note that the condition $\\gamma<\\angle C B A$ implies $\\gamma<90^{\\circ}$. Since $\\angle P B A=\\gamma$, the line $P B$ is tangent to $\\omega$, so $P A \\cdot P C=P B^{2}=P D^{2}$. By $\\frac{P A}{P D}=\\frac{P D}{P C}$ the triangles $P A D$ and $P D C$ are similar, and $\\angle A D P=\\angle D C P$. Next, since $\\angle A B Q=\\angle A C B$, the triangles $A B C$ and $A Q B$ are also similar. Then $\\angle A Q B=$ $\\angle A B C=\\angle A R C$, which means that the points $D, R, C$, and $Q$ are concyclic. Therefore $\\angle D R Q=$ $\\angle D C Q=\\angle A D P$. ![](https:\/\/cdn.mathpix.com\/cropped\/2024_04_17_e0914cb4f8055c731538g-43.jpg?height=594&width=959&top_left_y=923&top_left_x=583) Figure 1 Now from $\\angle A R B=\\angle A C B=\\gamma$ and $\\angle P D B=\\angle P B D=2 \\gamma$ we get $$ \\angle Q B R=\\angle A D B-\\angle A R B=\\angle A D P+\\angle P D B-\\angle A R B=\\angle D R Q+\\gamma=\\angle Q R B $$ so the triangle $Q R B$ is isosceles, which yields $Q B=Q R$.","tier":0} +{"problem_type":"Geometry","problem_label":"G4","problem":"Let $A B C$ be a triangle with $\\angle B>\\angle C$. Let $P$ and $Q$ be two different points on line $A C$ such that $\\angle P B A=\\angle Q B A=\\angle A C B$ and $A$ is located between $P$ and $C$. Suppose that there exists an interior point $D$ of segment $B Q$ for which $P D=P B$. Let the ray $A D$ intersect the circle $A B C$ at $R \\neq A$. Prove that $Q B=Q R$. (Georgia)","solution":"Again, denote by $\\omega$ the circumcircle of the triangle $A B C$. Denote $\\angle A C B=\\gamma$. Since $\\angle P B A=\\gamma$, the line $P B$ is tangent to $\\omega$. Let $E$ be the second intersection point of $B Q$ with $\\omega$. If $V^{\\prime}$ is any point on the ray $C E$ beyond $E$, then $\\angle B E V^{\\prime}=180^{\\circ}-\\angle B E C=180^{\\circ}-\\angle B A C=\\angle P A B$; together with $\\angle A B Q=$ $\\angle P B A$ this shows firstly, that the rays $B A$ and $C E$ intersect at some point $V$, and secondly that the triangle $V E B$ is similar to the triangle $P A B$. Thus we have $\\angle B V E=\\angle B P A$. Next, $\\angle A E V=\\angle B E V-\\gamma=\\angle P A B-\\angle A B Q=\\angle A Q B$; so the triangles $P B Q$ and $V A E$ are also similar. Let $P H$ be an altitude in the isosceles triangle $P B D$; then $B H=H D$. Let $G$ be the intersection point of $P H$ and $A B$. By the symmetry with respect to $P H$, we have $\\angle B D G=\\angle D B G=\\gamma=$ $\\angle B E A$; thus $D G \\| A E$ and hence $\\frac{B G}{G A}=\\frac{B D}{D E}$. Thus the points $G$ and $D$ correspond to each other in the similar triangles $P A B$ and $V E B$, so $\\angle D V B=\\angle G P B=90^{\\circ}-\\angle P B Q=90^{\\circ}-\\angle V A E$. Thus $V D \\perp A E$. Let $T$ be the common point of $V D$ and $A E$, and let $D S$ be an altitude in the triangle $B D R$. The points $S$ and $T$ are the feet of corresponding altitudes in the similar triangles $A D E$ and $B D R$, so $\\frac{B S}{S R}=\\frac{A T}{T E}$. On the other hand, the points $T$ and $H$ are feet of corresponding altitudes in the similar triangles $V A E$ and $P B Q$, so $\\frac{A T}{T E}=\\frac{B H}{H Q}$. Thus $\\frac{B S}{S R}=\\frac{A T}{T E}=\\frac{B H}{H Q}$, and the triangles $B H S$ and $B Q R$ are similar. Finally, $S H$ is a median in the right-angled triangle $S B D$; so $B H=H S$, and hence $B Q=Q R$. ![](https:\/\/cdn.mathpix.com\/cropped\/2024_04_17_e0914cb4f8055c731538g-44.jpg?height=848&width=875&top_left_y=584&top_left_x=625) Figure 2","tier":0} +{"problem_type":"Geometry","problem_label":"G4","problem":"Let $A B C$ be a triangle with $\\angle B>\\angle C$. Let $P$ and $Q$ be two different points on line $A C$ such that $\\angle P B A=\\angle Q B A=\\angle A C B$ and $A$ is located between $P$ and $C$. Suppose that there exists an interior point $D$ of segment $B Q$ for which $P D=P B$. Let the ray $A D$ intersect the circle $A B C$ at $R \\neq A$. Prove that $Q B=Q R$. (Georgia)","solution":"Denote by $\\omega$ and $O$ the circumcircle of the triangle $A B C$ and its center, respectively. From the condition $\\angle P B A=\\angle B C A$ we know that $B P$ is tangent to $\\omega$. Let $E$ be the second point of intersection of $\\omega$ and $B D$. Due to the isosceles triangle $B D P$, the tangent of $\\omega$ at $E$ is parallel to $D P$ and consequently it intersects $B P$ at some point $L$. Of course, $P D \\| L E$. Let $M$ be the midpoint of $B E$, and let $H$ be the midpoint of $B R$. Notice that $\\angle A E B=\\angle A C B=\\angle A B Q=\\angle A B E$, so $A$ lies on the perpendicular bisector of $B E$; thus the points $L, A, M$, and $O$ are collinear. Let $\\omega_{1}$ be the circle with diameter $B O$. Let $Q^{\\prime}=H O \\cap B E$; since $H O$ is the perpendicular bisector of $B R$, the statement of the problem is equivalent to $Q^{\\prime}=Q$. Consider the following sequence of projections (see Fig. 3). 1. Project the line $B E$ to the line $L B$ through the center $A$. (This maps $Q$ to $P$.) 2. Project the line $L B$ to $B E$ in parallel direction with $L E$. $(P \\mapsto D$.) 3. Project the line $B E$ to the circle $\\omega$ through its point $A .(D \\mapsto R$.) 4. Scale $\\omega$ by the ratio $\\frac{1}{2}$ from the point $B$ to the circle $\\omega_{1} .(R \\mapsto H$. 5. Project $\\omega_{1}$ to the line $B E$ through its point $O$. $\\left(H \\mapsto Q^{\\prime}\\right.$.) We prove that the composition of these transforms, which maps the line $B E$ to itself, is the identity. To achieve this, it suffices to show three fixed points. An obvious fixed point is $B$ which is fixed by all the transformations above. Another fixed point is $M$, its path being $M \\mapsto L \\mapsto$ $E \\mapsto E \\mapsto M \\mapsto M$. ![](https:\/\/cdn.mathpix.com\/cropped\/2024_04_17_e0914cb4f8055c731538g-45.jpg?height=800&width=786&top_left_y=218&top_left_x=279) Figure 3 ![](https:\/\/cdn.mathpix.com\/cropped\/2024_04_17_e0914cb4f8055c731538g-45.jpg?height=786&width=759&top_left_y=233&top_left_x=1125) Figure 4 In order to show a third fixed point, draw a line parallel with $L E$ through $A$; let that line intersect $B E, L B$ and $\\omega$ at $X, Y$ and $Z \\neq A$, respectively (see Fig. 4). We show that $X$ is a fixed point. The images of $X$ at the first three transformations are $X \\mapsto Y \\mapsto X \\mapsto Z$. From $\\angle X B Z=\\angle E A Z=\\angle A E L=\\angle L B A=\\angle B Z X$ we can see that the triangle $X B Z$ is isosceles. Let $U$ be the midpoint of $B Z$; then the last two transformations do $Z \\mapsto U \\mapsto X$, and the point $X$ is fixed. Comment. Verifying that the point $E$ is fixed seems more natural at first, but it appears to be less straightforward. Here we outline a possible proof. Let the images of $E$ at the first three transforms above be $F, G$ and $I$. After comparing the angles depicted in Fig. 5 (noticing that the quadrilateral $A F B G$ is cyclic) we can observe that the tangent $L E$ of $\\omega$ is parallel to $B I$. Then, similarly to the above reasons, the point $E$ is also fixed. ![](https:\/\/cdn.mathpix.com\/cropped\/2024_04_17_e0914cb4f8055c731538g-45.jpg?height=789&width=751&top_left_y=1687&top_left_x=687) Figure 5","tier":0} +{"problem_type":"Geometry","problem_label":"G5","problem":"Let $A B C D E F$ be a convex hexagon with $A B=D E, B C=E F, C D=F A$, and $\\angle A-\\angle D=\\angle C-\\angle F=\\angle E-\\angle B$. Prove that the diagonals $A D, B E$, and $C F$ are concurrent. (Ukraine)","solution":"Let $x=A B=D E, y=C D=F A, z=E F=B C$. Consider the points $P, Q$, and $R$ such that the quadrilaterals $C D E P, E F A Q$, and $A B C R$ are parallelograms. We compute $$ \\begin{aligned} \\angle P E Q & =\\angle F E Q+\\angle D E P-\\angle E=\\left(180^{\\circ}-\\angle F\\right)+\\left(180^{\\circ}-\\angle D\\right)-\\angle E \\\\ & =360^{\\circ}-\\angle D-\\angle E-\\angle F=\\frac{1}{2}(\\angle A+\\angle B+\\angle C-\\angle D-\\angle E-\\angle F)=\\theta \/ 2 \\end{aligned} $$ Similarly, $\\angle Q A R=\\angle R C P=\\theta \/ 2$. ![](https:\/\/cdn.mathpix.com\/cropped\/2024_04_17_e0914cb4f8055c731538g-46.jpg?height=491&width=827&top_left_y=947&top_left_x=649) If $\\theta=0$, since $\\triangle R C P$ is isosceles, $R=P$. Therefore $A B\\|R C=P C\\| E D$, so $A B D E$ is a parallelogram. Similarly, $B C E F$ and $C D F A$ are parallelograms. It follows that $A D, B E$ and $C F$ meet at their common midpoint. Now assume $\\theta>0$. Since $\\triangle P E Q, \\triangle Q A R$, and $\\triangle R C P$ are isosceles and have the same angle at the apex, we have $\\triangle P E Q \\sim \\triangle Q A R \\sim \\triangle R C P$ with ratios of similarity $y: z: x$. Thus $\\triangle P Q R$ is similar to the triangle with sidelengths $y, z$, and $x$. Next, notice that $$ \\frac{R Q}{Q P}=\\frac{z}{y}=\\frac{R A}{A F} $$ and, using directed angles between rays, $$ \\begin{aligned} \\not(R Q, Q P) & =\\Varangle(R Q, Q E)+\\Varangle(Q E, Q P) \\\\ & =\\Varangle(R Q, Q E)+\\Varangle(R A, R Q)=\\Varangle(R A, Q E)=\\Varangle(R A, A F) . \\end{aligned} $$ Thus $\\triangle P Q R \\sim \\triangle F A R$. Since $F A=y$ and $A R=z$, (1) then implies that $F R=x$. Similarly $F P=x$. Therefore $C R F P$ is a rhombus. We conclude that $C F$ is the perpendicular bisector of $P R$. Similarly, $B E$ is the perpendicular bisector of $P Q$ and $A D$ is the perpendicular bisector of $Q R$. It follows that $A D, B E$, and $C F$ are concurrent at the circumcenter of $P Q R$.","tier":0} +{"problem_type":"Geometry","problem_label":"G5","problem":"Let $A B C D E F$ be a convex hexagon with $A B=D E, B C=E F, C D=F A$, and $\\angle A-\\angle D=\\angle C-\\angle F=\\angle E-\\angle B$. Prove that the diagonals $A D, B E$, and $C F$ are concurrent. (Ukraine)","solution":"Let $X=C D \\cap E F, Y=E F \\cap A B, Z=A B \\cap C D, X^{\\prime}=F A \\cap B C, Y^{\\prime}=$ $B C \\cap D E$, and $Z^{\\prime}=D E \\cap F A$. From $\\angle A+\\angle B+\\angle C=360^{\\circ}+\\theta \/ 2$ we get $\\angle A+\\angle B>180^{\\circ}$ and $\\angle B+\\angle C>180^{\\circ}$, so $Z$ and $X^{\\prime}$ are respectively on the opposite sides of $B C$ and $A B$ from the hexagon. Similar conclusions hold for $X, Y, Y^{\\prime}$, and $Z^{\\prime}$. Then $$ \\angle Y Z X=\\angle B+\\angle C-180^{\\circ}=\\angle E+\\angle F-180^{\\circ}=\\angle Y^{\\prime} Z^{\\prime} X^{\\prime}, $$ and similarly $\\angle Z X Y=\\angle Z^{\\prime} X^{\\prime} Y^{\\prime}$ and $\\angle X Y Z=\\angle X^{\\prime} Y^{\\prime} Z^{\\prime}$, so $\\triangle X Y Z \\sim \\triangle X^{\\prime} Y^{\\prime} Z^{\\prime}$. Thus there is a rotation $R$ which sends $\\triangle X Y Z$ to a triangle with sides parallel to $\\triangle X^{\\prime} Y^{\\prime} Z^{\\prime}$. Since $A B=D E$ we have $R(\\overrightarrow{A B})=\\overrightarrow{D E}$. Similarly, $R(\\overrightarrow{C D})=\\overrightarrow{F A}$ and $R(\\overrightarrow{E F})=\\overrightarrow{B C}$. Therefore $$ \\overrightarrow{0}=\\overrightarrow{A B}+\\overrightarrow{B C}+\\overrightarrow{C D}+\\overrightarrow{D E}+\\overrightarrow{E F}+\\overrightarrow{F A}=(\\overrightarrow{A B}+\\overrightarrow{C D}+\\overrightarrow{E F})+R(\\overrightarrow{A B}+\\overrightarrow{C D}+\\overrightarrow{E F}) $$ If $R$ is a rotation by $180^{\\circ}$, then any two opposite sides of our hexagon are equal and parallel, so the three diagonals meet at their common midpoint. Otherwise, we must have $$ \\overrightarrow{A B}+\\overrightarrow{C D}+\\overrightarrow{E F}=\\overrightarrow{0} $$ or else we would have two vectors with different directions whose sum is $\\overrightarrow{0}$. ![](https:\/\/cdn.mathpix.com\/cropped\/2024_04_17_e0914cb4f8055c731538g-47.jpg?height=689&width=1333&top_left_y=1057&top_left_x=388) This allows us to consider a triangle $L M N$ with $\\overrightarrow{L M}=\\overrightarrow{E F}, \\overrightarrow{M N}=\\overrightarrow{A B}$, and $\\overrightarrow{N L}=\\overrightarrow{C D}$. Let $O$ be the circumcenter of $\\triangle L M N$ and consider the points $O_{1}, O_{2}, O_{3}$ such that $\\triangle A O_{1} B, \\triangle C O_{2} D$, and $\\triangle E O_{3} F$ are translations of $\\triangle M O N, \\triangle N O L$, and $\\triangle L O M$, respectively. Since $F O_{3}$ and $A O_{1}$ are translations of $M O$, quadrilateral $A F O_{3} O_{1}$ is a parallelogram and $O_{3} O_{1}=F A=C D=N L$. Similarly, $O_{1} O_{2}=L M$ and $O_{2} O_{3}=M N$. Therefore $\\triangle O_{1} O_{2} O_{3} \\cong \\triangle L M N$. Moreover, by means of the rotation $R$ one may check that these triangles have the same orientation. Let $T$ be the circumcenter of $\\triangle O_{1} O_{2} O_{3}$. We claim that $A D, B E$, and $C F$ meet at $T$. Let us show that $C, T$, and $F$ are collinear. Notice that $C O_{2}=O_{2} T=T O_{3}=O_{3} F$ since they are all equal to the circumradius of $\\triangle L M N$. Therefore $\\triangle T O_{3} F$ and $\\triangle C O_{2} T$ are isosceles. Using directed angles between rays again, we get $$ \\Varangle\\left(T F, T O_{3}\\right)=\\Varangle\\left(F O_{3}, F T\\right) \\quad \\text { and } \\quad \\Varangle\\left(T O_{2}, T C\\right)=\\Varangle\\left(C T, C O_{2}\\right) \\text {. } $$ Also, $T$ and $O$ are the circumcenters of the congruent triangles $\\triangle O_{1} O_{2} O_{3}$ and $\\triangle L M N$ so we have $\\Varangle\\left(T O_{3}, T O_{2}\\right)=\\Varangle(O N, O M)$. Since $C_{2}$ and $F O_{3}$ are translations of $N O$ and $M O$ respectively, this implies $$ \\Varangle\\left(T O_{3}, T O_{2}\\right)=\\Varangle\\left(C O_{2}, F O_{3}\\right) . $$ Adding the three equations in (2) and (3) gives $$ \\Varangle(T F, T C)=\\Varangle(C T, F T)=-\\not(T F, T C) $$ which implies that $T$ is on $C F$. Analogous arguments show that it is on $A D$ and $B E$ also. The desired result follows.","tier":0} +{"problem_type":"Geometry","problem_label":"G5","problem":"Let $A B C D E F$ be a convex hexagon with $A B=D E, B C=E F, C D=F A$, and $\\angle A-\\angle D=\\angle C-\\angle F=\\angle E-\\angle B$. Prove that the diagonals $A D, B E$, and $C F$ are concurrent. (Ukraine)","solution":"Place the hexagon on the complex plane, with $A$ at the origin and vertices labelled clockwise. Now $A, B, C, D, E, F$ represent the corresponding complex numbers. Also consider the complex numbers $a, b, c, a^{\\prime}, b^{\\prime}, c^{\\prime}$ given by $B-A=a, D-C=b, F-E=c, E-D=a^{\\prime}$, $A-F=b^{\\prime}$, and $C-B=c^{\\prime}$. Let $k=|a| \/|b|$. From $a \/ b^{\\prime}=-k e^{i \\angle A}$ and $a^{\\prime} \/ b=-k e^{i \\angle D}$ we get that $\\left(a^{\\prime} \/ a\\right)\\left(b^{\\prime} \/ b\\right)=e^{-i \\theta}$ and similarly $\\left(b^{\\prime} \/ b\\right)\\left(c^{\\prime} \/ c\\right)=e^{-i \\theta}$ and $\\left(c^{\\prime} \/ c\\right)\\left(a^{\\prime} \/ a\\right)=e^{-i \\theta}$. It follows that $a^{\\prime}=a r$, $b^{\\prime}=b r$, and $c^{\\prime}=c r$ for a complex number $r$ with $|r|=1$, as shown below. ![](https:\/\/cdn.mathpix.com\/cropped\/2024_04_17_e0914cb4f8055c731538g-48.jpg?height=512&width=1052&top_left_y=823&top_left_x=534) We have $$ 0=a+c r+b+a r+c+b r=(a+b+c)(1+r) . $$ If $r=-1$, then the hexagon is centrally symmetric and its diagonals intersect at its center of symmetry. Otherwise $$ a+b+c=0 \\text {. } $$ Therefore $$ A=0, \\quad B=a, \\quad C=a+c r, \\quad D=c(r-1), \\quad E=-b r-c, \\quad F=-b r . $$ Now consider a point $W$ on $A D$ given by the complex number $c(r-1) \\lambda$, where $\\lambda$ is a real number with $0<\\lambda<1$. Since $D \\neq A$, we have $r \\neq 1$, so we can define $s=1 \/(r-1)$. From $r \\bar{r}=|r|^{2}=1$ we get $$ 1+s=\\frac{r}{r-1}=\\frac{r}{r-r \\bar{r}}=\\frac{1}{1-\\bar{r}}=-\\bar{s} . $$ Now, $$ \\begin{aligned} W \\text { is on } B E & \\Longleftrightarrow c(r-1) \\lambda-a\\|a-(-b r-c)=b(r-1) \\Longleftrightarrow c \\lambda-a s\\| b \\\\ & \\Longleftrightarrow-a \\lambda-b \\lambda-a s\\|b \\Longleftrightarrow a(\\lambda+s)\\| b . \\end{aligned} $$ One easily checks that $r \\neq \\pm 1$ implies that $\\lambda+s \\neq 0$ since $s$ is not real. On the other hand, $$ \\begin{aligned} W \\text { on } C F & \\Longleftrightarrow c(r-1) \\lambda+b r\\|-b r-(a+c r)=a(r-1) \\Longleftrightarrow c \\lambda+b(1+s)\\| a \\\\ & \\Longleftrightarrow-a \\lambda-b \\lambda-b \\bar{s}\\|a \\Longleftrightarrow b(\\lambda+\\bar{s})\\| a \\Longleftrightarrow b \\| a(\\lambda+s), \\end{aligned} $$ where in the last step we use that $(\\lambda+s)(\\lambda+\\bar{s})=|\\lambda+s|^{2} \\in \\mathbb{R}_{>0}$. We conclude that $A D \\cap B E=$ $C F \\cap B E$, and the desired result follows.","tier":0} +{"problem_type":"Geometry","problem_label":"G6","problem":"Let the excircle of the triangle $A B C$ lying opposite to $A$ touch its side $B C$ at the point $A_{1}$. Define the points $B_{1}$ and $C_{1}$ analogously. Suppose that the circumcentre of the triangle $A_{1} B_{1} C_{1}$ lies on the circumcircle of the triangle $A B C$. Prove that the triangle $A B C$ is right-angled. (Russia)","solution":"Denote the circumcircles of the triangles $A B C$ and $A_{1} B_{1} C_{1}$ by $\\Omega$ and $\\Gamma$, respectively. Denote the midpoint of the arc $C B$ of $\\Omega$ containing $A$ by $A_{0}$, and define $B_{0}$ as well as $C_{0}$ analogously. By our hypothesis the centre $Q$ of $\\Gamma$ lies on $\\Omega$. Lemma. One has $A_{0} B_{1}=A_{0} C_{1}$. Moreover, the points $A, A_{0}, B_{1}$, and $C_{1}$ are concyclic. Finally, the points $A$ and $A_{0}$ lie on the same side of $B_{1} C_{1}$. Similar statements hold for $B$ and $C$. Proof. Let us consider the case $A=A_{0}$ first. Then the triangle $A B C$ is isosceles at $A$, which implies $A B_{1}=A C_{1}$ while the remaining assertions of the Lemma are obvious. So let us suppose $A \\neq A_{0}$ from now on. By the definition of $A_{0}$, we have $A_{0} B=A_{0} C$. It is also well known and easy to show that $B C_{1}=$ $C B_{1}$. Next, we have $\\angle C_{1} B A_{0}=\\angle A B A_{0}=\\angle A C A_{0}=\\angle B_{1} C A_{0}$. Hence the triangles $A_{0} B C_{1}$ and $A_{0} C B_{1}$ are congruent. This implies $A_{0} C_{1}=A_{0} B_{1}$, establishing the first part of the Lemma. It also follows that $\\angle A_{0} C_{1} A=\\angle A_{0} B_{1} A$, as these are exterior angles at the corresponding vertices $C_{1}$ and $B_{1}$ of the congruent triangles $A_{0} B C_{1}$ and $A_{0} C B_{1}$. For that reason the points $A, A_{0}, B_{1}$, and $C_{1}$ are indeed the vertices of some cyclic quadrilateral two opposite sides of which are $A A_{0}$ and $B_{1} C_{1}$. Now we turn to the solution. Evidently the points $A_{1}, B_{1}$, and $C_{1}$ lie interior to some semicircle arc of $\\Gamma$, so the triangle $A_{1} B_{1} C_{1}$ is obtuse-angled. Without loss of generality, we will assume that its angle at $B_{1}$ is obtuse. Thus $Q$ and $B_{1}$ lie on different sides of $A_{1} C_{1}$; obviously, the same holds for the points $B$ and $B_{1}$. So, the points $Q$ and $B$ are on the same side of $A_{1} C_{1}$. Notice that the perpendicular bisector of $A_{1} C_{1}$ intersects $\\Omega$ at two points lying on different sides of $A_{1} C_{1}$. By the first statement from the Lemma, both points $B_{0}$ and $Q$ are among these points of intersection; since they share the same side of $A_{1} C_{1}$, they coincide (see Figure 1). ![](https:\/\/cdn.mathpix.com\/cropped\/2024_04_17_e0914cb4f8055c731538g-49.jpg?height=781&width=827&top_left_y=1667&top_left_x=649) Figure 1 Now, by the first part of the Lemma again, the lines $Q A_{0}$ and $Q C_{0}$ are the perpendicular bisectors of $B_{1} C_{1}$ and $A_{1} B_{1}$, respectively. Thus $$ \\angle C_{1} B_{0} A_{1}=\\angle C_{1} B_{0} B_{1}+\\angle B_{1} B_{0} A_{1}=2 \\angle A_{0} B_{0} B_{1}+2 \\angle B_{1} B_{0} C_{0}=2 \\angle A_{0} B_{0} C_{0}=180^{\\circ}-\\angle A B C $$ recalling that $A_{0}$ and $C_{0}$ are the midpoints of the arcs $C B$ and $B A$, respectively. On the other hand, by the second part of the Lemma we have $$ \\angle C_{1} B_{0} A_{1}=\\angle C_{1} B A_{1}=\\angle A B C . $$ From the last two equalities, we get $\\angle A B C=90^{\\circ}$, whereby the problem is solved.","tier":0} +{"problem_type":"Geometry","problem_label":"G6","problem":"Let the excircle of the triangle $A B C$ lying opposite to $A$ touch its side $B C$ at the point $A_{1}$. Define the points $B_{1}$ and $C_{1}$ analogously. Suppose that the circumcentre of the triangle $A_{1} B_{1} C_{1}$ lies on the circumcircle of the triangle $A B C$. Prove that the triangle $A B C$ is right-angled. (Russia)","solution":"Let $Q$ again denote the centre of the circumcircle of the triangle $A_{1} B_{1} C_{1}$, that lies on the circumcircle $\\Omega$ of the triangle $A B C$. We first consider the case where $Q$ coincides with one of the vertices of $A B C$, say $Q=B$. Then $B C_{1}=B A_{1}$ and consequently the triangle $A B C$ is isosceles at $B$. Moreover we have $B C_{1}=B_{1} C$ in any triangle, and hence $B B_{1}=B C_{1}=B_{1} C$; similarly, $B B_{1}=B_{1} A$. It follows that $B_{1}$ is the centre of $\\Omega$ and that the triangle $A B C$ has a right angle at $B$. So from now on we may suppose $Q \\notin\\{A, B, C\\}$. We start with the following well known fact. Lemma. Let $X Y Z$ and $X^{\\prime} Y^{\\prime} Z^{\\prime}$ be two triangles with $X Y=X^{\\prime} Y^{\\prime}$ and $Y Z=Y^{\\prime} Z^{\\prime}$. (i) If $X Z \\neq X^{\\prime} Z^{\\prime}$ and $\\angle Y Z X=\\angle Y^{\\prime} Z^{\\prime} X^{\\prime}$, then $\\angle Z X Y+\\angle Z^{\\prime} X^{\\prime} Y^{\\prime}=180^{\\circ}$. (ii) If $\\angle Y Z X+\\angle X^{\\prime} Z^{\\prime} Y^{\\prime}=180^{\\circ}$, then $\\angle Z X Y=\\angle Y^{\\prime} X^{\\prime} Z^{\\prime}$. Proof. For both parts, we may move the triangle $X Y Z$ through the plane until $Y=Y^{\\prime}$ and $Z=Z^{\\prime}$. Possibly after reflecting one of the two triangles about $Y Z$, we may also suppose that $X$ and $X^{\\prime}$ lie on the same side of $Y Z$ if we are in case (i) and on different sides if we are in case (ii). In both cases, the points $X, Z$, and $X^{\\prime}$ are collinear due to the angle condition (see Fig. 2). Moreover we have $X \\neq X^{\\prime}$, because in case (i) we assumed $X Z \\neq X^{\\prime} Z^{\\prime}$ and in case (ii) these points even lie on different sides of $Y Z$. Thus the triangle $X X^{\\prime} Y$ is isosceles at $Y$. The claim now follows by considering the equal angles at its base. ![](https:\/\/cdn.mathpix.com\/cropped\/2024_04_17_e0914cb4f8055c731538g-50.jpg?height=304&width=460&top_left_y=1732&top_left_x=518) Figure 2(i) ![](https:\/\/cdn.mathpix.com\/cropped\/2024_04_17_e0914cb4f8055c731538g-50.jpg?height=304&width=464&top_left_y=1732&top_left_x=1123) Figure 2(ii) Relabeling the vertices of the triangle $A B C$ if necessary we may suppose that $Q$ lies in the interior of the arc $A B$ of $\\Omega$ not containing $C$. We will sometimes use tacitly that the six triangles $Q B A_{1}, Q A_{1} C, Q C B_{1}, Q B_{1} A, Q C_{1} A$, and $Q B C_{1}$ have the same orientation. As $Q$ cannot be the circumcentre of the triangle $A B C$, it is impossible that $Q A=Q B=Q C$ and thus we may also suppose that $Q C \\neq Q B$. Now the above Lemma $(i)$ is applicable to the triangles $Q B_{1} C$ and $Q C_{1} B$, since $Q B_{1}=Q C_{1}$ and $B_{1} C=C_{1} B$, while $\\angle B_{1} C Q=\\angle C_{1} B Q$ holds as both angles appear over the same side of the chord $Q A$ in $\\Omega$ (see Fig. 3). So we get $$ \\angle C Q B_{1}+\\angle B Q C_{1}=180^{\\circ} . $$ We claim that $Q C=Q A$. To see this, let us assume for the sake of a contradiction that $Q C \\neq Q A$. Then arguing similarly as before but now with the triangles $Q A_{1} C$ and $Q C_{1} A$ we get $$ \\angle A_{1} Q C+\\angle C_{1} Q A=180^{\\circ} \\text {. } $$ Adding this equation to (1), we get $\\angle A_{1} Q B_{1}+\\angle B Q A=360^{\\circ}$, which is absurd as both summands lie in the interval $\\left(0^{\\circ}, 180^{\\circ}\\right)$. This proves $Q C=Q A$; so the triangles $Q A_{1} C$ and $Q C_{1} A$ are congruent their sides being equal, which in turn yields $$ \\angle A_{1} Q C=\\angle C_{1} Q A \\text {. } $$ Finally our Lemma (ii) is applicable to the triangles $Q A_{1} B$ and $Q B_{1} A$. Indeed we have $Q A_{1}=Q B_{1}$ and $A_{1} B=B_{1} A$ as usual, and the angle condition $\\angle A_{1} B Q+\\angle Q A B_{1}=180^{\\circ}$ holds as $A$ and $B$ lie on different sides of the chord $Q C$ in $\\Omega$. Consequently we have $$ \\angle B Q A_{1}=\\angle B_{1} Q A \\text {. } $$ From (1) and (3) we get $$ \\left(\\angle B_{1} Q C+\\angle B_{1} Q A\\right)+\\left(\\angle C_{1} Q B-\\angle B Q A_{1}\\right)=180^{\\circ} \\text {, } $$ i.e. $\\angle C Q A+\\angle A_{1} Q C_{1}=180^{\\circ}$. In light of (2) this may be rewritten as $2 \\angle C Q A=180^{\\circ}$ and as $Q$ lies on $\\Omega$ this implies that the triangle $A B C$ has a right angle at $B$. ![](https:\/\/cdn.mathpix.com\/cropped\/2024_04_17_e0914cb4f8055c731538g-51.jpg?height=569&width=908&top_left_y=1404&top_left_x=606) Figure 3 Comment 1. One may also check that $Q$ is in the interior of $\\Omega$ if and only if the triangle $A B C$ is acute-angled. Comment 2. The original proposal asked to prove the converse statement as well: if the triangle $A B C$ is right-angled, then the point $Q$ lies on its circumcircle. The Problem Selection Committee thinks that the above simplified version is more suitable for the competition.","tier":0} +{"problem_type":"Number Theory","problem_label":"N1","problem":"Let $\\mathbb{Z}_{>0}$ be the set of positive integers. Find all functions $f: \\mathbb{Z}_{>0} \\rightarrow \\mathbb{Z}_{>0}$ such that $$ m^{2}+f(n) \\mid m f(m)+n $$ for all positive integers $m$ and $n$. (Malaysia)","solution":"Setting $m=n=2$ tells us that $4+f(2) \\mid 2 f(2)+2$. Since $2 f(2)+2<2(4+f(2))$, we must have $2 f(2)+2=4+f(2)$, so $f(2)=2$. Plugging in $m=2$ then tells us that $4+f(n) \\mid 4+n$, which implies that $f(n) \\leqslant n$ for all $n$. Setting $m=n$ gives $n^{2}+f(n) \\mid n f(n)+n$, so $n f(n)+n \\geqslant n^{2}+f(n)$ which we rewrite as $(n-1)(f(n)-n) \\geqslant 0$. Therefore $f(n) \\geqslant n$ for all $n \\geqslant 2$. This is trivially true for $n=1$ also. It follows that $f(n)=n$ for all $n$. This function obviously satisfies the desired property.","tier":0} +{"problem_type":"Number Theory","problem_label":"N1","problem":"Let $\\mathbb{Z}_{>0}$ be the set of positive integers. Find all functions $f: \\mathbb{Z}_{>0} \\rightarrow \\mathbb{Z}_{>0}$ such that $$ m^{2}+f(n) \\mid m f(m)+n $$ for all positive integers $m$ and $n$. (Malaysia)","solution":"Setting $m=f(n)$ we get $f(n)(f(n)+1) \\mid f(n) f(f(n))+n$. This implies that $f(n) \\mid n$ for all $n$. Now let $m$ be any positive integer, and let $p>2 m^{2}$ be a prime number. Note that $p>m f(m)$ also. Plugging in $n=p-m f(m)$ we learn that $m^{2}+f(n)$ divides $p$. Since $m^{2}+f(n)$ cannot equal 1, it must equal $p$. Therefore $p-m^{2}=f(n) \\mid n=p-m f(m)$. But $p-m f(m)0}$ be the set of positive integers. Find all functions $f: \\mathbb{Z}_{>0} \\rightarrow \\mathbb{Z}_{>0}$ such that $$ m^{2}+f(n) \\mid m f(m)+n $$ for all positive integers $m$ and $n$. (Malaysia)","solution":"Plugging $m=1$ we obtain $1+f(n) \\leqslant f(1)+n$, so $f(n) \\leqslant n+c$ for the constant $c=$ $f(1)-1$. Assume that $f(n) \\neq n$ for some fixed $n$. When $m$ is large enough (e.g. $m \\geqslant \\max (n, c+1)$ ) we have $$ m f(m)+n \\leqslant m(m+c)+n \\leqslant 2 m^{2}<2\\left(m^{2}+f(n)\\right), $$ so we must have $m f(m)+n=m^{2}+f(n)$. This implies that $$ 0 \\neq f(n)-n=m(f(m)-m) $$ which is impossible for $m>|f(n)-n|$. It follows that $f$ is the identity function.","tier":0} +{"problem_type":"Number Theory","problem_label":"N2","problem":"Prove that for any pair of positive integers $k$ and $n$ there exist $k$ positive integers $m_{1}, m_{2}, \\ldots, m_{k}$ such that $$ 1+\\frac{2^{k}-1}{n}=\\left(1+\\frac{1}{m_{1}}\\right)\\left(1+\\frac{1}{m_{2}}\\right) \\cdots\\left(1+\\frac{1}{m_{k}}\\right) . $$ (Japan)","solution":"We proceed by induction on $k$. For $k=1$ the statement is trivial. Assuming we have proved it for $k=j-1$, we now prove it for $k=j$. Case 1. $n=2 t-1$ for some positive integer $t$. Observe that $$ 1+\\frac{2^{j}-1}{2 t-1}=\\frac{2\\left(t+2^{j-1}-1\\right)}{2 t} \\cdot \\frac{2 t}{2 t-1}=\\left(1+\\frac{2^{j-1}-1}{t}\\right)\\left(1+\\frac{1}{2 t-1}\\right) . $$ By the induction hypothesis we can find $m_{1}, \\ldots, m_{j-1}$ such that $$ 1+\\frac{2^{j-1}-1}{t}=\\left(1+\\frac{1}{m_{1}}\\right)\\left(1+\\frac{1}{m_{2}}\\right) \\cdots\\left(1+\\frac{1}{m_{j-1}}\\right) $$ so setting $m_{j}=2 t-1$ gives the desired expression. Case 2. $n=2 t$ for some positive integer $t$. Now we have $$ 1+\\frac{2^{j}-1}{2 t}=\\frac{2 t+2^{j}-1}{2 t+2^{j}-2} \\cdot \\frac{2 t+2^{j}-2}{2 t}=\\left(1+\\frac{1}{2 t+2^{j}-2}\\right)\\left(1+\\frac{2^{j-1}-1}{t}\\right) $$ noting that $2 t+2^{j}-2>0$. Again, we use that $$ 1+\\frac{2^{j-1}-1}{t}=\\left(1+\\frac{1}{m_{1}}\\right)\\left(1+\\frac{1}{m_{2}}\\right) \\cdots\\left(1+\\frac{1}{m_{j-1}}\\right) . $$ Setting $m_{j}=2 t+2^{j}-2$ then gives the desired expression.","tier":0} +{"problem_type":"Number Theory","problem_label":"N2","problem":"Prove that for any pair of positive integers $k$ and $n$ there exist $k$ positive integers $m_{1}, m_{2}, \\ldots, m_{k}$ such that $$ 1+\\frac{2^{k}-1}{n}=\\left(1+\\frac{1}{m_{1}}\\right)\\left(1+\\frac{1}{m_{2}}\\right) \\cdots\\left(1+\\frac{1}{m_{k}}\\right) . $$ (Japan)","solution":"Consider the base 2 expansions of the residues of $n-1$ and $-n$ modulo $2^{k}$ : $$ \\begin{aligned} n-1 & \\equiv 2^{a_{1}}+2^{a_{2}}+\\cdots+2^{a_{r}}\\left(\\bmod 2^{k}\\right) & & \\text { where } 0 \\leqslant a_{1}q_{n-1} \\text { and } q_{n}>q_{n+1}\\right\\} $$ is infinite, since for each $n \\in S$ one has $$ p_{n}=\\max \\left\\{q_{n}, q_{n-1}\\right\\}=q_{n}=\\max \\left\\{q_{n}, q_{n+1}\\right\\}=p_{n+1} . $$ Suppose on the contrary that $S$ is finite. Since $q_{2}=7<13=q_{3}$ and $q_{3}=13>7=q_{4}$, the set $S$ is non-empty. Since it is finite, we can consider its largest element, say $m$. Note that it is impossible that $q_{m}>q_{m+1}>q_{m+2}>\\ldots$ because all these numbers are positive integers, so there exists a $k \\geqslant m$ such that $q_{k}q_{\\ell+1}$. By the minimality of $\\ell$ we have $q_{\\ell-1}k \\geqslant m$, this contradicts the maximality of $m$, and hence $S$ is indeed infinite. Comment. Once the factorization of $n^{4}+n^{2}+1$ is found and the set $S$ is introduced, the problem is mainly about ruling out the case that $$ q_{k}0}$. In the above solution, this is done by observing $q_{(k+1)^{2}}=\\max \\left(q_{k}, q_{k+1}\\right)$. Alternatively one may notice that (1) implies that $q_{j+2}-q_{j} \\geqslant 6$ for $j \\geqslant k+1$, since every prime greater than 3 is congruent to -1 or 1 modulo 6 . Then there is some integer $C \\geqslant 0$ such that $q_{n} \\geqslant 3 n-C$ for all $n \\geqslant k$. Now let the integer $t$ be sufficiently large (e.g. $t=\\max \\{k+1, C+3\\}$ ) and set $p=q_{t-1} \\geqslant 2 t$. Then $p \\mid(t-1)^{2}+(t-1)+1$ implies that $p \\mid(p-t)^{2}+(p-t)+1$, so $p$ and $q_{p-t}$ are prime divisors of $(p-t)^{2}+(p-t)+1$. But $p-t>t-1 \\geqslant k$, so $q_{p-t}>q_{t-1}=p$ and $p \\cdot q_{p-t}>p^{2}>(p-t)^{2}+(p-t)+1$, a contradiction.","tier":0} +{"problem_type":"Number Theory","problem_label":"N4","problem":"Determine whether there exists an infinite sequence of nonzero digits $a_{1}, a_{2}, a_{3}, \\ldots$ and a positive integer $N$ such that for every integer $k>N$, the number $\\overline{a_{k} a_{k-1} \\ldots a_{1}}$ is a perfect square. (Iran)","solution":"Assume that $a_{1}, a_{2}, a_{3}, \\ldots$ is such a sequence. For each positive integer $k$, let $y_{k}=$ $\\overline{a_{k} a_{k-1} \\ldots a_{1}}$. By the assumption, for each $k>N$ there exists a positive integer $x_{k}$ such that $y_{k}=x_{k}^{2}$. I. For every $n$, let $5^{\\gamma_{n}}$ be the greatest power of 5 dividing $x_{n}$. Let us show first that $2 \\gamma_{n} \\geqslant n$ for every positive integer $n>N$. Assume, to the contrary, that there exists a positive integer $n>N$ such that $2 \\gamma_{n}N$. II. Consider now any integer $k>\\max \\{N \/ 2,2\\}$. Since $2 \\gamma_{2 k+1} \\geqslant 2 k+1$ and $2 \\gamma_{2 k+2} \\geqslant 2 k+2$, we have $\\gamma_{2 k+1} \\geqslant k+1$ and $\\gamma_{2 k+2} \\geqslant k+1$. So, from $y_{2 k+2}=a_{2 k+2} \\cdot 10^{2 k+1}+y_{2 k+1}$ we obtain $5^{2 k+2} \\mid y_{2 k+2}-y_{2 k+1}=a_{2 k+2} \\cdot 10^{2 k+1}$ and thus $5 \\mid a_{2 k+2}$, which implies $a_{2 k+2}=5$. Therefore, $$ \\left(x_{2 k+2}-x_{2 k+1}\\right)\\left(x_{2 k+2}+x_{2 k+1}\\right)=x_{2 k+2}^{2}-x_{2 k+1}^{2}=y_{2 k+2}-y_{2 k+1}=5 \\cdot 10^{2 k+1}=2^{2 k+1} \\cdot 5^{2 k+2} . $$ Setting $A_{k}=x_{2 k+2} \/ 5^{k+1}$ and $B_{k}=x_{2 k+1} \/ 5^{k+1}$, which are integers, we obtain $$ \\left(A_{k}-B_{k}\\right)\\left(A_{k}+B_{k}\\right)=2^{2 k+1} . $$ Both $A_{k}$ and $B_{k}$ are odd, since otherwise $y_{2 k+2}$ or $y_{2 k+1}$ would be a multiple of 10 which is false by $a_{1} \\neq 0$; so one of the numbers $A_{k}-B_{k}$ and $A_{k}+B_{k}$ is not divisible by 4 . Therefore (1) yields $A_{k}-B_{k}=2$ and $A_{k}+B_{k}=2^{2 k}$, hence $A_{k}=2^{2 k-1}+1$ and thus $$ x_{2 k+2}=5^{k+1} A_{k}=10^{k+1} \\cdot 2^{k-2}+5^{k+1}>10^{k+1}, $$ since $k \\geqslant 2$. This implies that $y_{2 k+2}>10^{2 k+2}$ which contradicts the fact that $y_{2 k+2}$ contains $2 k+2$ digits. The desired result follows.","tier":0} +{"problem_type":"Number Theory","problem_label":"N4","problem":"Determine whether there exists an infinite sequence of nonzero digits $a_{1}, a_{2}, a_{3}, \\ldots$ and a positive integer $N$ such that for every integer $k>N$, the number $\\overline{a_{k} a_{k-1} \\ldots a_{1}}$ is a perfect square. (Iran)","solution":"Again, we assume that a sequence $a_{1}, a_{2}, a_{3}, \\ldots$ satisfies the problem conditions, introduce the numbers $x_{k}$ and $y_{k}$ as in the previous solution, and notice that $$ y_{k+1}-y_{k}=\\left(x_{k+1}-x_{k}\\right)\\left(x_{k+1}+x_{k}\\right)=10^{k} a_{k+1} $$ for all $k>N$. Consider any such $k$. Since $a_{1} \\neq 0$, the numbers $x_{k}$ and $x_{k+1}$ are not multiples of 10 , and therefore the numbers $p_{k}=x_{k+1}-x_{k}$ and $q_{k}=x_{k+1}+x_{k}$ cannot be simultaneously multiples of 20 , and hence one of them is not divisible either by 4 or by 5 . In view of (2), this means that the other one is divisible by either $5^{k}$ or by $2^{k-1}$. Notice also that $p_{k}$ and $q_{k}$ have the same parity, so both are even. On the other hand, we have $x_{k+1}^{2}=x_{k}^{2}+10^{k} a_{k+1} \\geqslant x_{k}^{2}+10^{k}>2 x_{k}^{2}$, so $x_{k+1} \/ x_{k}>\\sqrt{2}$, which implies that $$ 1<\\frac{q_{k}}{p_{k}}=1+\\frac{2}{x_{k+1} \/ x_{k}-1}<1+\\frac{2}{\\sqrt{2}-1}<6 . $$ Thus, if one of the numbers $p_{k}$ and $q_{k}$ is divisible by $5^{k}$, then we have $$ 10^{k+1}>10^{k} a_{k+1}=p_{k} q_{k} \\geqslant \\frac{\\left(5^{k}\\right)^{2}}{6} $$ and hence $(5 \/ 2)^{k}<60$ which is false for sufficiently large $k$. So, assuming that $k$ is large, we get that $2^{k-1}$ divides one of the numbers $p_{k}$ and $q_{k}$. Hence $$ \\left\\{p_{k}, q_{k}\\right\\}=\\left\\{2^{k-1} \\cdot 5^{r_{k}} b_{k}, 2 \\cdot 5^{k-r_{k}} c_{k}\\right\\} \\quad \\text { with nonnegative integers } b_{k}, c_{k}, r_{k} \\text { such that } b_{k} c_{k}=a_{k+1} \\text {. } $$ Moreover, from (3) we get $$ 6>\\frac{2^{k-1} \\cdot 5^{r_{k}} b_{k}}{2 \\cdot 5^{k-r_{k}} c_{k}} \\geqslant \\frac{1}{36} \\cdot\\left(\\frac{2}{5}\\right)^{k} \\cdot 5^{2 r_{k}} \\quad \\text { and } \\quad 6>\\frac{2 \\cdot 5^{k-r_{k}} c_{k}}{2^{k-1} \\cdot 5^{r_{k}} b_{k}} \\geqslant \\frac{4}{9} \\cdot\\left(\\frac{5}{2}\\right)^{k} \\cdot 5^{-2 r_{k}} $$ SO $$ \\alpha k+c_{1}c_{1} \\text {. } $$ Consequently, for $C=c_{2}-c_{1}+1-\\alpha>0$ we have $$ (k+1)-r_{k+1} \\leqslant k-r_{k}+C . $$ Next, we will use the following easy lemma. Lemma. Let $s$ be a positive integer. Then $5^{s+2^{s}} \\equiv 5^{s}\\left(\\bmod 10^{s}\\right)$. Proof. Euler's theorem gives $5^{2^{s}} \\equiv 1\\left(\\bmod 2^{s}\\right)$, so $5^{s+2^{s}}-5^{s}=5^{s}\\left(5^{2^{s}}-1\\right)$ is divisible by $2^{s}$ and $5^{s}$. Now, for every large $k$ we have $$ x_{k+1}=\\frac{p_{k}+q_{k}}{2}=5^{r_{k}} \\cdot 2^{k-2} b_{k}+5^{k-r_{k}} c_{k} \\equiv 5^{k-r_{k}} c_{k} \\quad\\left(\\bmod 10^{r_{k}}\\right) $$ since $r_{k} \\leqslant k-2$ by $(4)$; hence $y_{k+1} \\equiv 5^{2\\left(k-r_{k}\\right)} c_{k}^{2}\\left(\\bmod 10^{r_{k}}\\right)$. Let us consider some large integer $s$, and choose the minimal $k$ such that $2\\left(k-r_{k}\\right) \\geqslant s+2^{s}$; it exists by (4). Set $d=2\\left(k-r_{k}\\right)-\\left(s+2^{s}\\right)$. By (4) we have $2^{s}<2\\left(k-r_{k}\\right)<\\left(\\frac{2}{\\alpha}-2\\right) r_{k}-\\frac{2 c_{1}}{\\alpha}$; if $s$ is large this implies $r_{k}>s$, so (6) also holds modulo $10^{s}$. Then (6) and the lemma give $$ y_{k+1} \\equiv 5^{2\\left(k-r_{k}\\right)} c_{k}^{2}=5^{s+2^{s}} \\cdot 5^{d} c_{k}^{2} \\equiv 5^{s} \\cdot 5^{d} c_{k}^{2} \\quad\\left(\\bmod 10^{s}\\right) . $$ By (5) and the minimality of $k$ we have $d \\leqslant 2 C$, so $5^{d} c_{k}^{2} \\leqslant 5^{2 C} \\cdot 81=D$. Using $5^{4}<10^{3}$ we obtain $$ 5^{s} \\cdot 5^{d} c_{k}^{2}<10^{3 s \/ 4} D<10^{s-1} $$ for sufficiently large $s$. This, together with (7), shows that the sth digit from the right in $y_{k+1}$, which is $a_{s}$, is zero. This contradicts the problem condition.","tier":0} +{"problem_type":"Number Theory","problem_label":"N5","problem":"Fix an integer $k \\geqslant 2$. Two players, called Ana and Banana, play the following game of numbers: Initially, some integer $n \\geqslant k$ gets written on the blackboard. Then they take moves in turn, with Ana beginning. A player making a move erases the number $m$ just written on the blackboard and replaces it by some number $m^{\\prime}$ with $k \\leqslant m^{\\prime}m \\geqslant k$ and $\\operatorname{gcd}(m, n)=1$, then $n$ itself is bad, for Ana has the following winning strategy in the game with initial number $n$ : She proceeds by first playing $m$ and then using Banana's strategy for the game with starting number $m$. Otherwise, if some integer $n \\geqslant k$ has the property that every integer $m$ with $n>m \\geqslant k$ and $\\operatorname{gcd}(m, n)=1$ is bad, then $n$ is good. Indeed, if Ana can make a first move at all in the game with initial number $n$, then she leaves it in a position where the first player has a winning strategy, so that Banana can defeat her. In particular, this implies that any two good numbers have a non-trivial common divisor. Also, $k$ itself is good. For brevity, we say that $n \\longrightarrow x$ is a move if $n$ and $x$ are two coprime integers with $n>x \\geqslant k$. Claim 1. If $n$ is good and $n^{\\prime}$ is a multiple of $n$, then $n^{\\prime}$ is also good. Proof. If $n^{\\prime}$ were bad, there would have to be some move $n^{\\prime} \\longrightarrow x$, where $x$ is good. As $n^{\\prime}$ is a multiple of $n$ this implies that the two good numbers $n$ and $x$ are coprime, which is absurd. Claim 2. If $r$ and $s$ denote two positive integers for which $r s \\geqslant k$ is bad, then $r^{2} s$ is also bad. Proof. Since $r s$ is bad, there is a move $r s \\longrightarrow x$ for some good $x$. Evidently $x$ is coprime to $r^{2} s$ as well, and hence the move $r^{2} s \\longrightarrow x$ shows that $r^{2} s$ is indeed bad. Claim 3. If $p>k$ is prime and $n \\geqslant k$ is bad, then $n p$ is also bad. Proof. Otherwise we choose a counterexample with $n$ being as small as possible. In particular, $n p$ is good. Since $n$ is bad, there is a move $n \\longrightarrow x$ for some good $x$. Now $n p \\longrightarrow x$ cannot be a valid move, which tells us that $x$ has to be divisible by $p$. So we can write $x=p^{r} y$, where $r$ and $y$ denote some positive integers, the latter of which is not divisible by $p$. Note that $y=1$ is impossible, for then we would have $x=p^{r}$ and the move $x \\longrightarrow k$ would establish that $x$ is bad. In view of this, there is a least power $y^{\\alpha}$ of $y$ that is at least as large as $k$. Since the numbers $n p$ and $y^{\\alpha}$ are coprime and the former is good, the latter has to be bad. Moreover, the minimality of $\\alpha$ implies $y^{\\alpha}k$, but now we get the same contradiction using Claim 3 instead of Claim 2 . Thereby the problem is solved.","tier":0} +{"problem_type":"Number Theory","problem_label":"N5","problem":"Fix an integer $k \\geqslant 2$. Two players, called Ana and Banana, play the following game of numbers: Initially, some integer $n \\geqslant k$ gets written on the blackboard. Then they take moves in turn, with Ana beginning. A player making a move erases the number $m$ just written on the blackboard and replaces it by some number $m^{\\prime}$ with $k \\leqslant m^{\\prime}x$. This is clear in case $n=0$, so let us assume $n>0$ from now on. Then we have $x

b$. Applying Claim 4 to $b$ we get an integer $x$ with $b \\geqslant x \\geqslant k$ that is similar to $b$ and has no big prime divisors at all. By our assumption, $b^{\\prime}$ and $x$ are coprime, and as $b^{\\prime}$ is good this implies that $x$ is bad. Consequently there has to be some move $x \\longrightarrow b^{*}$ such that $b^{*}$ is good. But now all the small prime factors of $b$ also appear in $x$ and thus they cannot divide $b^{*}$. Therefore the pair $\\left(b^{*}, b\\right)$ contradicts the supposed minimality of $b^{\\prime}$. From that point, it is easy to complete the solution: assume that there are two similar integers $a$ and $b$ such that $a$ is bad and $b$ is good. Since $a$ is bad, there is a move $a \\longrightarrow b^{\\prime}$ for some good $b^{\\prime}$. By Claim 5, there is a small prime $p$ dividing $b$ and $b^{\\prime}$. Due to the similarity of $a$ and $b$, the prime $p$ has to divide $a$ as well, but this contradicts the fact that $a \\longrightarrow b^{\\prime}$ is a valid move. Thereby the problem is solved. Comment 2. There are infinitely many good numbers, e.g. all multiples of $k$. The increasing sequence $b_{0}, b_{1}, \\ldots$, of all good numbers may be constructed recursively as follows: - Start with $b_{0}=k$. - If $b_{n}$ has just been defined for some $n \\geqslant 0$, then $b_{n+1}$ is the smallest number $b>b_{n}$ that is coprime to none of $b_{0}, \\ldots, b_{n}$. This construction can be used to determine the set of good numbers for any specific $k$ as explained in the next comment. It is already clear that if $k=p^{\\alpha}$ is a prime power, then a number $b \\geqslant k$ is good if and only if it is divisible by $p$. Comment 3. Let $P>1$ denote the product of all small prime numbers. Then any two integers $a, b \\geqslant k$ that are congruent modulo $P$ are similar. Thus the infinite word $W_{k}=\\left(X_{k}, X_{k+1}, \\ldots\\right)$ defined by $$ X_{i}= \\begin{cases}A & \\text { if } i \\text { is bad } \\\\ B & \\text { if } i \\text { is good }\\end{cases} $$ for all $i \\geqslant k$ is periodic and the length of its period divides $P$. As the prime power example shows, the true period can sometimes be much smaller than $P$. On the other hand, there are cases where the period is rather large; e.g., if $k=15$, the sequence of good numbers begins with 15,18,20,24,30,36,40,42, 45 and the period of $W_{15}$ is 30 . Comment 4. The original proposal contained two questions about the game of numbers, namely $(a)$ to show that if two numbers have the same prime factors then either both are good or both are bad, and (b) to show that the word $W_{k}$ introduced in the previous comment is indeed periodic. The Problem Selection Committee thinks that the above version of the problem is somewhat easier, even though it demands to prove a stronger result.","tier":0} +{"problem_type":"Number Theory","problem_label":"N6","problem":"Determine all functions $f: \\mathbb{Q} \\longrightarrow \\mathbb{Z}$ satisfying $$ f\\left(\\frac{f(x)+a}{b}\\right)=f\\left(\\frac{x+a}{b}\\right) $$ for all $x \\in \\mathbb{Q}, a \\in \\mathbb{Z}$, and $b \\in \\mathbb{Z}_{>0}$. (Here, $\\mathbb{Z}_{>0}$ denotes the set of positive integers.) (Israel)","solution":"I. We start by verifying that these functions do indeed satisfy (1). This is clear for all constant functions. Now consider any triple $(x, a, b) \\in \\mathbb{Q} \\times \\mathbb{Z} \\times \\mathbb{Z}_{>0}$ and set $$ q=\\left\\lfloor\\frac{x+a}{b}\\right\\rfloor . $$ This means that $q$ is an integer and $b q \\leqslant x+a0}$. According to the behaviour of the restriction of $f$ to the integers we distinguish two cases. Case 1: There is some $m \\in \\mathbb{Z}$ such that $f(m) \\neq m$. Write $f(m)=C$ and let $\\eta \\in\\{-1,+1\\}$ and $b$ denote the sign and absolute value of $f(m)-m$, respectively. Given any integer $r$, we may plug the triple $(m, r b-C, b)$ into (1), thus getting $f(r)=f(r-\\eta)$. Starting with $m$ and using induction in both directions, we deduce from this that the equation $f(r)=C$ holds for all integers $r$. Now any rational number $y$ can be written in the form $y=\\frac{p}{q}$ with $(p, q) \\in \\mathbb{Z} \\times \\mathbb{Z}_{>0}$, and substituting $(C-p, p-C, q)$ into (1) we get $f(y)=f(0)=C$. Thus $f$ is the constant function whose value is always $C$. Case 2: One has $f(m)=m$ for all integers $m$. Note that now the special case $b=1$ of (1) takes a particularly simple form, namely $$ f(x)+a=f(x+a) \\quad \\text { for all }(x, a) \\in \\mathbb{Q} \\times \\mathbb{Z} $$ Defining $f\\left(\\frac{1}{2}\\right)=\\omega$ we proceed in three steps. Step $A$. We show that $\\omega \\in\\{0,1\\}$. If $\\omega \\leqslant 0$, we may plug $\\left(\\frac{1}{2},-\\omega, 1-2 \\omega\\right)$ into (1), obtaining $0=f(0)=f\\left(\\frac{1}{2}\\right)=\\omega$. In the contrary case $\\omega \\geqslant 1$ we argue similarly using the triple $\\left(\\frac{1}{2}, \\omega-1,2 \\omega-1\\right)$. Step B. We show that $f(x)=\\omega$ for all rational numbers $x$ with $0b \\geqslant k+\\omega$, which is absurd. Similarly, $m \\geqslant r$ leads to $r a-m b0} $$ Now suppose first that $x$ is not an integer but can be written in the form $\\frac{p}{q}$ with $p \\in \\mathbb{Z}$ and $q \\in \\mathbb{Z}_{>0}$ both being odd. Let $d$ denote the multiplicative order of 2 modulo $q$ and let $m$ be any large integer. Plugging $n=d m$ into (6) and using (2) we get $$ f(x)=\\left[\\frac{f\\left(2^{d m} x\\right)}{2^{d m}}\\right]=\\left[\\frac{f(x)+\\left(2^{d m}-1\\right) x}{2^{d m}}\\right]=\\left[x+\\frac{f(x)-x}{2^{d m}}\\right] . $$ Since $x$ is not an integer, the square bracket function is continuous at $x$; hence as $m$ tends to infinity the above fomula gives $f(x)=[x]$. To complete the argument we just need to observe that if some $y \\in \\mathbb{Q}$ satisfies $f(y)=[y]$, then (5) yields $f\\left(\\frac{y}{2}\\right)=f\\left(\\frac{[y]}{2}\\right)=\\left[\\frac{[y]}{2}\\right]=\\left[\\frac{y}{2}\\right]$.","tier":0} +{"problem_type":"Number Theory","problem_label":"N6","problem":"Determine all functions $f: \\mathbb{Q} \\longrightarrow \\mathbb{Z}$ satisfying $$ f\\left(\\frac{f(x)+a}{b}\\right)=f\\left(\\frac{x+a}{b}\\right) $$ for all $x \\in \\mathbb{Q}, a \\in \\mathbb{Z}$, and $b \\in \\mathbb{Z}_{>0}$. (Here, $\\mathbb{Z}_{>0}$ denotes the set of positive integers.) (Israel)","solution":"Here we just give another argument for the second case of the above solution. Again we use equation (2). It follows that the set $S$ of all zeros of $f$ contains for each $x \\in \\mathbb{Q}$ exactly one term from the infinite sequence $\\ldots, x-2, x-1, x, x+1, x+2, \\ldots$. Next we claim that $$ \\text { if }(p, q) \\in \\mathbb{Z} \\times \\mathbb{Z}_{>0} \\text { and } \\frac{p}{q} \\in S \\text {, then } \\frac{p}{q+1} \\in S \\text { holds as well. } $$ To see this we just plug $\\left(\\frac{p}{q}, p, q+1\\right)$ into (1), thus getting $f\\left(\\frac{p}{q+1}\\right)=f\\left(\\frac{p}{q}\\right)=0$. From this we get that $$ \\text { if } x, y \\in \\mathbb{Q}, x>y>0 \\text {, and } x \\in S \\text {, then } y \\in S \\text {. } $$ Indeed, if we write $x=\\frac{p}{q}$ and $y=\\frac{r}{s}$ with $p, q, r, s \\in \\mathbb{Z}_{>0}$, then $p s>q r$ and (7) tells us $$ 0=f\\left(\\frac{p}{q}\\right)=f\\left(\\frac{p r}{q r}\\right)=f\\left(\\frac{p r}{q r+1}\\right)=\\ldots=f\\left(\\frac{p r}{p s}\\right)=f\\left(\\frac{r}{s}\\right) $$ Essentially the same argument also establishes that $$ \\text { if } x, y \\in \\mathbb{Q}, x2 m$. Such a pair is necessarily a child of $(c, d-c)$, and thus a descendant of some pair $\\left(c, d^{\\prime}\\right)$ with $m\\{(b+k a) \\nu\\}=\\left\\{\\left(b+k_{0} a\\right) \\nu\\right\\}+\\left(k-k_{0}\\right)\\{a \\nu\\}$ for all $k>k_{0}$, which is absurd. Similarly, one can prove that $S_{m}$ contains finitely many pairs $(c, d)$ with $c>2 m$, thus finitely many elements at all. We are now prepared for proving the following crucial lemma. Lemma. Consider any pair $(a, b)$ with $f(a, b) \\neq m$. Then the number $g(a, b)$ of its $m$-excellent descendants is equal to the number $h(a, b)$ of ways to represent the number $t=m-f(a, b)$ as $t=k a+\\ell b$ with $k$ and $\\ell$ being some nonnegative integers. Proof. We proceed by induction on the number $N$ of descendants of $(a, b)$ in $S_{m}$. If $N=0$ then clearly $g(a, b)=0$. Assume that $h(a, b)>0$; without loss of generality, we have $a \\leqslant b$. Then, clearly, $m-f(a, b) \\geqslant a$, so $f(a, b+a) \\leqslant f(a, b)+a \\leqslant m$ and $a \\leqslant m$, hence $(a, b+a) \\in S_{m}$ which is impossible. Thus in the base case we have $g(a, b)=h(a, b)=0$, as desired. Now let $N>0$. Assume that $f(a+b, b)=f(a, b)+b$ and $f(a, b+a)=f(a, b)$ (the other case is similar). If $f(a, b)+b \\neq m$, then by the induction hypothesis we have $$ g(a, b)=g(a+b, b)+g(a, b+a)=h(a+b, b)+h(a, b+a) $$ Notice that both pairs $(a+b, b)$ and $(a, b+a)$ are descendants of $(a, b)$ and thus each of them has strictly less descendants in $S_{m}$ than $(a, b)$ does. Next, each one of the $h(a+b, b)$ representations of $m-f(a+b, b)=m-b-f(a, b)$ as the sum $k^{\\prime}(a+b)+\\ell^{\\prime} b$ provides the representation $m-f(a, b)=k a+\\ell b$ with $k=k^{\\prime}1} d=\\sum_{d \\mid m} d $$ as required. Comment. Let us present a sketch of an outline of a different solution. The plan is to check that the number of excellent pairs does not depend on the (irrational) number $\\nu$, and to find this number for some appropriate value of $\\nu$. For that, we first introduce some geometrical language. We deal only with the excellent pairs $(a, b)$ with $a \\neq b$. Part I. Given an irrational positive $\\nu$, for every positive integer $n$ we introduce two integral points $F_{\\nu}(n)=$ $(n,\\lfloor n \\nu\\rfloor)$ and $C_{\\nu}(n)=(n,\\lceil n \\nu\\rceil)$ on the coordinate plane $O x y$. Then $(*)$ reads as $\\left[O F_{\\nu}(a) C_{\\nu}(b)\\right]=m \/ 2$; here $[\\cdot]$ stands for the signed area. Next, we rewrite in these terms the condition on a pair $(a, b)$ to be excellent. Let $\\ell_{\\nu}, \\ell_{\\nu}^{+}$, and $\\ell_{\\nu}^{-}$be the lines determined by the equations $y=\\nu x, y=\\nu x+1$, and $y=\\nu x-1$, respectively. a). Firstly, we deal with all excellent pairs $(a, b)$ with $aa$ is excellent exactly when $p_{\\nu}(a)$ lies between $b-a$ and $b$, and the point of $f_{\\nu}(a)$ with abscissa $b$ is integral (which means that this point is $C_{\\nu}(b)$ ). Notice now that, if $p_{\\nu}(a)>a$, then the number of excellent pairs of the form $(a, b)$ (with $b>a$ ) is $\\operatorname{gcd}(a,\\lfloor a \\nu\\rfloor)$. ![](https:\/\/cdn.mathpix.com\/cropped\/2024_04_17_e0914cb4f8055c731538g-67.jpg?height=396&width=422&top_left_y=1274&top_left_x=556) Figure 1 ![](https:\/\/cdn.mathpix.com\/cropped\/2024_04_17_e0914cb4f8055c731538g-67.jpg?height=431&width=442&top_left_y=1254&top_left_x=1123) Figure 2 b). Analogously, considering the pairs $(a, b)$ with $a>b$, we fix the value of $b$, introduce the line $c_{\\nu}(b)$ containing all the points $F$ with $\\left[O F C_{\\nu}(b)\\right]=m \/ 2$, assume that this line contains an integral point (which means $\\operatorname{gcd}(b,\\lceil b \\nu\\rceil) \\mid m$ ), and denote the common point of $c_{\\nu}(b)$ and $\\ell_{\\nu}^{-}$by $Q_{\\nu}(b)$, its abscissa being $q_{\\nu}(b)$. Similarly to the previous case, we obtain that the pair $(a, b)$ is excellent exactly when $q_{\\nu}(a)$ lies between $a-b$ and $a$, and the point of $c_{\\nu}(b)$ with abscissa $a$ is integral (see Fig. 2). Again, if $q_{\\nu}(b)>b$, then the number of excellent pairs of the form $(a, b)$ (with $a>b)$ is $\\operatorname{gcd}(b,\\lceil b \\nu\\rceil)$. Part II, sketchy. Having obtained such a description, one may check how the number of excellent pairs changes as $\\nu$ grows. (Having done that, one may find this number for one appropriate value of $\\nu$; for instance, it is relatively easy to make this calculation for $\\nu \\in\\left(1,1+\\frac{1}{m}\\right)$.) Consider, for the initial value of $\\nu$, some excellent pair $(a, t)$ with $a>t$. As $\\nu$ grows, this pair eventually stops being excellent; this happens when the point $Q_{\\nu}(t)$ passes through $F_{\\nu}(a)$. At the same moment, the pair $(a+t, t)$ becomes excellent instead. This process halts when the point $Q_{\\nu}(t)$ eventually disappears, i.e. when $\\nu$ passes through the ratio of the coordinates of the point $T=C_{\\nu}(t)$. Hence, the point $T$ afterwards is regarded as $F_{\\nu}(t)$. Thus, all the old excellent pairs of the form $(a, t)$ with $a>t$ disappear; on the other hand, the same number of excellent pairs with the first element being $t$ just appear. Similarly, if some pair $(t, b)$ with $t0}$ be the set of positive rational numbers. Let $f: \\mathbb{Q}_{>0} \\rightarrow \\mathbb{R}$ be a function satisfying the conditions $$ \\begin{aligned} & f(x) f(y) \\geqslant f(x y) \\\\ & f(x+y) \\geqslant f(x)+f(y) \\end{aligned} $$ for all $x, y \\in \\mathbb{Q}_{>0}$. Given that $f(a)=a$ for some rational $a>1$, prove that $f(x)=x$ for all $x \\in \\mathbb{Q}_{>0}$. (Bulgaria)","solution":"Denote by $\\mathbb{Z}_{>0}$ the set of positive integers. Plugging $x=1, y=a$ into (1) we get $f(1) \\geqslant 1$. Next, by an easy induction on $n$ we get from (2) that $$ f(n x) \\geqslant n f(x) \\text { for all } n \\in \\mathbb{Z}_{>0} \\text { and } x \\in \\mathbb{Q}_{>0} $$ In particular, we have $$ f(n) \\geqslant n f(1) \\geqslant n \\quad \\text { for all } n \\in \\mathbb{Z}_{>0} $$ From (1) again we have $f(m \/ n) f(n) \\geqslant f(m)$, so $f(q)>0$ for all $q \\in \\mathbb{Q}_{>0}$. Now, (2) implies that $f$ is strictly increasing; this fact together with (4) yields $$ f(x) \\geqslant f(\\lfloor x\\rfloor) \\geqslant\\lfloor x\\rfloor>x-1 \\quad \\text { for all } x \\geqslant 1 $$ By an easy induction we get from (1) that $f(x)^{n} \\geqslant f\\left(x^{n}\\right)$, so $$ f(x)^{n} \\geqslant f\\left(x^{n}\\right)>x^{n}-1 \\quad \\Longrightarrow \\quad f(x) \\geqslant \\sqrt[n]{x^{n}-1} \\text { for all } x>1 \\text { and } n \\in \\mathbb{Z}_{>0} $$ This yields $$ f(x) \\geqslant x \\text { for every } x>1 \\text {. } $$ (Indeed, if $x>y>1$ then $x^{n}-y^{n}=(x-y)\\left(x^{n-1}+x^{n-2} y+\\cdots+y^{n}\\right)>n(x-y)$, so for a large $n$ we have $x^{n}-1>y^{n}$ and thus $f(x)>y$.) Now, (1) and (5) give $a^{n}=f(a)^{n} \\geqslant f\\left(a^{n}\\right) \\geqslant a^{n}$, so $f\\left(a^{n}\\right)=a^{n}$. Now, for $x>1$ let us choose $n \\in \\mathbb{Z}_{>0}$ such that $a^{n}-x>1$. Then by (2) and (5) we get $$ a^{n}=f\\left(a^{n}\\right) \\geqslant f(x)+f\\left(a^{n}-x\\right) \\geqslant x+\\left(a^{n}-x\\right)=a^{n} $$ and therefore $f(x)=x$ for $x>1$. Finally, for every $x \\in \\mathbb{Q}_{>0}$ and every $n \\in \\mathbb{Z}_{>0}$, from (1) and (3) we get $$ n f(x)=f(n) f(x) \\geqslant f(n x) \\geqslant n f(x) $$ which gives $f(n x)=n f(x)$. Therefore $f(m \/ n)=f(m) \/ n=m \/ n$ for all $m, n \\in \\mathbb{Z}_{>0}$. Comment. The condition $f(a)=a>1$ is essential. Indeed, for $b \\geqslant 1$ the function $f(x)=b x^{2}$ satisfies (1) and (2) for all $x, y \\in \\mathbb{Q}_{>0}$, and it has a unique fixed point $1 \/ b \\leqslant 1$.","tier":0} +{"problem_type":"Algebra","problem_label":"A5","problem":"Let $\\mathbb{Z}_{\\geqslant 0}$ be the set of all nonnegative integers. Find all the functions $f: \\mathbb{Z}_{\\geqslant 0} \\rightarrow \\mathbb{Z}_{\\geqslant 0}$ satisfying the relation $$ f(f(f(n)))=f(n+1)+1 $$ for all $n \\in \\mathbb{Z}_{\\geqslant 0}$. (Serbia) Answer. There are two such functions: $f(n)=n+1$ for all $n \\in \\mathbb{Z}_{\\geqslant 0}$, and $$ f(n)=\\left\\{\\begin{array}{ll} n+1, & n \\equiv 0(\\bmod 4) \\text { or } n \\equiv 2(\\bmod 4), \\\\ n+5, & n \\equiv 1(\\bmod 4), \\\\ n-3, & n \\equiv 3(\\bmod 4) \\end{array} \\quad \\text { for all } n \\in \\mathbb{Z}_{\\geqslant 0}\\right. $$ Throughout all the solutions, we write $h^{k}(x)$ to abbreviate the $k$ th iteration of function $h$, so $h^{0}$ is the identity function, and $h^{k}(x)=\\underbrace{h(\\ldots h}_{k \\text { times }}(x) \\ldots))$ for $k \\geqslant 1$.","solution":"To start, we get from (*) that $$ f^{4}(n)=f\\left(f^{3}(n)\\right)=f(f(n+1)+1) \\quad \\text { and } \\quad f^{4}(n+1)=f^{3}(f(n+1))=f(f(n+1)+1)+1 $$ thus $$ f^{4}(n)+1=f^{4}(n+1) . $$ I. Let us denote by $R_{i}$ the range of $f^{i}$; note that $R_{0}=\\mathbb{Z}_{\\geqslant 0}$ since $f^{0}$ is the identity function. Obviously, $R_{0} \\supseteq R_{1} \\supseteq \\ldots$ Next, from (2) we get that if $a \\in R_{4}$ then also $a+1 \\in R_{4}$. This implies that $\\mathbb{Z}_{\\geqslant 0} \\backslash R_{4}$ - and hence $\\mathbb{Z}_{\\geqslant 0} \\backslash R_{1}$ - is finite. In particular, $R_{1}$ is unbounded. Assume that $f(m)=f(n)$ for some distinct $m$ and $n$. Then from $(*)$ we obtain $f(m+1)=$ $f(n+1)$; by an easy induction we then get that $f(m+c)=f(n+c)$ for every $c \\geqslant 0$. So the function $f(k)$ is periodic with period $|m-n|$ for $k \\geqslant m$, and thus $R_{1}$ should be bounded, which is false. So, $f$ is injective. II. Denote now $S_{i}=R_{i-1} \\backslash R_{i}$; all these sets are finite for $i \\leqslant 4$. On the other hand, by the injectivity we have $n \\in S_{i} \\Longleftrightarrow f(n) \\in S_{i+1}$. By the injectivity again, $f$ implements a bijection between $S_{i}$ and $S_{i+1}$, thus $\\left|S_{1}\\right|=\\left|S_{2}\\right|=\\ldots$; denote this common cardinality by $k$. If $0 \\in R_{3}$ then $0=f(f(f(n)))$ for some $n$, thus from (*) we get $f(n+1)=-1$ which is impossible. Therefore $0 \\in R_{0} \\backslash R_{3}=S_{1} \\cup S_{2} \\cup S_{3}$, thus $k \\geqslant 1$. Next, let us describe the elements $b$ of $R_{0} \\backslash R_{3}=S_{1} \\cup S_{2} \\cup S_{3}$. We claim that each such element satisfies at least one of three conditions $(i) b=0,(i i) b=f(0)+1$, and (iii) $b-1 \\in S_{1}$. Otherwise $b-1 \\in \\mathbb{Z}_{\\geqslant 0}$, and there exists some $n>0$ such that $f(n)=b-1$; but then $f^{3}(n-1)=f(n)+1=b$, so $b \\in R_{3}$. This yields $$ 3 k=\\left|S_{1} \\cup S_{2} \\cup S_{3}\\right| \\leqslant 1+1+\\left|S_{1}\\right|=k+2, $$ or $k \\leqslant 1$. Therefore $k=1$, and the inequality above comes to equality. So we have $S_{1}=\\{a\\}$, $S_{2}=\\{f(a)\\}$, and $S_{3}=\\left\\{f^{2}(a)\\right\\}$ for some $a \\in \\mathbb{Z}_{\\geqslant 0}$, and each one of the three options (i), (ii), and (iii) should be realized exactly once, which means that $$ \\left\\{a, f(a), f^{2}(a)\\right\\}=\\{0, a+1, f(0)+1\\} . $$ III. From (3), we get $a+1 \\in\\left\\{f(a), f^{2}(a)\\right\\}$ (the case $a+1=a$ is impossible). If $a+1=f^{2}(a)$ then we have $f(a+1)=f^{3}(a)=f(a+1)+1$ which is absurd. Therefore $$ f(a)=a+1 $$ Next, again from (3) we have $0 \\in\\left\\{a, f^{2}(a)\\right\\}$. Let us consider these two cases separately. Case 1. Assume that $a=0$, then $f(0)=f(a)=a+1=1$. Also from (3) we get $f(1)=f^{2}(a)=$ $f(0)+1=2$. Now, let us show that $f(n)=n+1$ by induction on $n$; the base cases $n \\leqslant 1$ are established. Next, if $n \\geqslant 2$ then the induction hypothesis implies $$ n+1=f(n-1)+1=f^{3}(n-2)=f^{2}(n-1)=f(n), $$ establishing the step. In this case we have obtained the first of two answers; checking that is satisfies (*) is straightforward. Case 2. Assume now that $f^{2}(a)=0$; then by (3) we get $a=f(0)+1$. By (4) we get $f(a+1)=$ $f^{2}(a)=0$, then $f(0)=f^{3}(a)=f(a+1)+1=1$, hence $a=f(0)+1=2$ and $f(2)=3$ by (4). To summarize, $$ f(0)=1, \\quad f(2)=3, \\quad f(3)=0 . $$ Now let us prove by induction on $m$ that (1) holds for all $n=4 k, 4 k+2,4 k+3$ with $k \\leqslant m$ and for all $n=4 k+1$ with $k0$ and $B(1)=m+2>0$ since $n=2 m$. Therefore $B(x)=A(x+a+b)$. Writing $c=a+b \\geqslant 1$ we compute $$ 0=A(x+c)-B(x)=(3 c-2 m) x^{2}+c(3 c-2 m) x+c^{2}(c-m) . $$ Then we must have $3 c-2 m=c-m=0$, which gives $m=0$, a contradiction. We conclude that $f(x)=t x$ is the only solution.","tier":0} +{"problem_type":"Algebra","problem_label":"A6","problem":"Let $m \\neq 0$ be an integer. Find all polynomials $P(x)$ with real coefficients such that $$ \\left(x^{3}-m x^{2}+1\\right) P(x+1)+\\left(x^{3}+m x^{2}+1\\right) P(x-1)=2\\left(x^{3}-m x+1\\right) P(x) $$ for all real numbers $x$. (Serbia) Answer. $P(x)=t x$ for any real number $t$.","solution":"Multiplying (1) by $x$, we rewrite it as $$ x\\left(x^{3}-m x^{2}+1\\right) P(x+1)+x\\left(x^{3}+m x^{2}+1\\right) P(x-1)=[(x+1)+(x-1)]\\left(x^{3}-m x+1\\right) P(x) . $$ After regrouping, it becomes $$ \\left(x^{3}-m x^{2}+1\\right) Q(x)=\\left(x^{3}+m x^{2}+1\\right) Q(x-1) \\text {, } $$ where $Q(x)=x P(x+1)-(x+1) P(x)$. If $\\operatorname{deg} P \\geqslant 2$ then $\\operatorname{deg} Q=\\operatorname{deg} P$, so $Q(x)$ has a finite multiset of complex roots, which we denote $R_{Q}$. Each root is taken with its multiplicity. Then the multiset of complex roots of $Q(x-1)$ is $R_{Q}+1=\\left\\{z+1: z \\in R_{Q}\\right\\}$. Let $\\left\\{x_{1}, x_{2}, x_{3}\\right\\}$ and $\\left\\{y_{1}, y_{2}, y_{3}\\right\\}$ be the multisets of roots of the polynomials $A(x)=x^{3}-m x^{2}+1$ and $B(x)=x^{3}+m x^{2}+1$, respectively. From (2) we get the equality of multisets $$ \\left\\{x_{1}, x_{2}, x_{3}\\right\\} \\cup R_{Q}=\\left\\{y_{1}, y_{2}, y_{3}\\right\\} \\cup\\left(R_{Q}+1\\right) . $$ For every $r \\in R_{Q}$, since $r+1$ is in the set of the right hand side, we must have $r+1 \\in R_{Q}$ or $r+1=x_{i}$ for some $i$. Similarly, since $r$ is in the set of the left hand side, either $r-1 \\in R_{Q}$ or $r=y_{i}$ for some $i$. This implies that, possibly after relabelling $y_{1}, y_{2}, y_{3}$, all the roots of (2) may be partitioned into three chains of the form $\\left\\{y_{i}, y_{i}+1, \\ldots, y_{i}+k_{i}=x_{i}\\right\\}$ for $i=1,2,3$ and some integers $k_{1}, k_{2}, k_{3} \\geqslant 0$. Now we analyze the roots of the polynomial $A_{a}(x)=x^{3}+a x^{2}+1$. Using calculus or elementary methods, we find that the local extrema of $A_{a}(x)$ occur at $x=0$ and $x=-2 a \/ 3$; their values are $A_{a}(0)=1>0$ and $A_{a}(-2 a \/ 3)=1+4 a^{3} \/ 27$, which is positive for integers $a \\geqslant-1$ and negative for integers $a \\leqslant-2$. So when $a \\in \\mathbb{Z}, A_{a}$ has three real roots if $a \\leqslant-2$ and one if $a \\geqslant-1$. Now, since $y_{i}-x_{i} \\in \\mathbb{Z}$ for $i=1,2,3$, the cubics $A_{m}$ and $A_{-m}$ must have the same number of real roots. The previous analysis then implies that $m=1$ or $m=-1$. Therefore the real root $\\alpha$ of $A_{1}(x)=x^{3}+x^{2}+1$ and the real root $\\beta$ of $A_{-1}(x)=x^{3}-x^{2}+1$ must differ by an integer. But this is impossible, because $A_{1}\\left(-\\frac{3}{2}\\right)=-\\frac{1}{8}$ and $A_{1}(-1)=1$ so $-1.5<\\alpha<-1$, while $A_{-1}(-1)=-1$ and $A_{-1}\\left(-\\frac{1}{2}\\right)=\\frac{5}{8}$, so $-1<\\beta<-0.5$. It follows that $\\operatorname{deg} P \\leqslant 1$. Then, as shown in Solution 1, we conclude that the solutions are $P(x)=t x$ for all real numbers $t$.","tier":0} +{"problem_type":"Combinatorics","problem_label":"C1","problem":"Let $n$ be a positive integer. Find the smallest integer $k$ with the following property: Given any real numbers $a_{1}, \\ldots, a_{d}$ such that $a_{1}+a_{2}+\\cdots+a_{d}=n$ and $0 \\leqslant a_{i} \\leqslant 1$ for $i=1,2, \\ldots, d$, it is possible to partition these numbers into $k$ groups (some of which may be empty) such that the sum of the numbers in each group is at most 1 . (Poland) Answer. $k=2 n-1$.","solution":"If $d=2 n-1$ and $a_{1}=\\cdots=a_{2 n-1}=n \/(2 n-1)$, then each group in such a partition can contain at most one number, since $2 n \/(2 n-1)>1$. Therefore $k \\geqslant 2 n-1$. It remains to show that a suitable partition into $2 n-1$ groups always exists. We proceed by induction on $d$. For $d \\leqslant 2 n-1$ the result is trivial. If $d \\geqslant 2 n$, then since $$ \\left(a_{1}+a_{2}\\right)+\\ldots+\\left(a_{2 n-1}+a_{2 n}\\right) \\leqslant n $$ we may find two numbers $a_{i}, a_{i+1}$ such that $a_{i}+a_{i+1} \\leqslant 1$. We \"merge\" these two numbers into one new number $a_{i}+a_{i+1}$. By the induction hypothesis, a suitable partition exists for the $d-1$ numbers $a_{1}, \\ldots, a_{i-1}, a_{i}+a_{i+1}, a_{i+2}, \\ldots, a_{d}$. This induces a suitable partition for $a_{1}, \\ldots, a_{d}$.","tier":0} +{"problem_type":"Combinatorics","problem_label":"C1","problem":"Let $n$ be a positive integer. Find the smallest integer $k$ with the following property: Given any real numbers $a_{1}, \\ldots, a_{d}$ such that $a_{1}+a_{2}+\\cdots+a_{d}=n$ and $0 \\leqslant a_{i} \\leqslant 1$ for $i=1,2, \\ldots, d$, it is possible to partition these numbers into $k$ groups (some of which may be empty) such that the sum of the numbers in each group is at most 1 . (Poland) Answer. $k=2 n-1$.","solution":"We will show that it is even possible to split the sequence $a_{1}, \\ldots, a_{d}$ into $2 n-1$ contiguous groups so that the sum of the numbers in each groups does not exceed 1. Consider a segment $S$ of length $n$, and partition it into segments $S_{1}, \\ldots, S_{d}$ of lengths $a_{1}, \\ldots, a_{d}$, respectively, as shown below. Consider a second partition of $S$ into $n$ equal parts by $n-1$ \"empty dots\". | $a_{1}$ | $a_{2}$ | $a_{3}$ | $a_{4}$ | $a_{5}$ | $a_{6}$ | $a_{7}$ | $a_{8}, a_{9}, a_{10}$ | | :--- | :--- | :--- | :--- | :--- | :--- | :--- | :--- | :--- | | | | | | | | | | Assume that the $n-1$ empty dots are in segments $S_{i_{1}}, \\ldots, S_{i_{n-1}}$. (If a dot is on the boundary of two segments, we choose the right segment). These $n-1$ segments are distinct because they have length at most 1. Consider the partition: $$ \\left\\{a_{1}, \\ldots, a_{i_{1}-1}\\right\\},\\left\\{a_{i_{1}}\\right\\},\\left\\{a_{i_{1}+1}, \\ldots, a_{i_{2}-1}\\right\\},\\left\\{a_{i_{2}}\\right\\}, \\ldots\\left\\{a_{i_{n-1}}\\right\\},\\left\\{a_{i_{n-1}+1}, \\ldots, a_{d}\\right\\} $$ In the example above, this partition is $\\left\\{a_{1}, a_{2}\\right\\},\\left\\{a_{3}\\right\\},\\left\\{a_{4}, a_{5}\\right\\},\\left\\{a_{6}\\right\\}, \\varnothing,\\left\\{a_{7}\\right\\},\\left\\{a_{8}, a_{9}, a_{10}\\right\\}$. We claim that in this partition, the sum of the numbers in this group is at most 1 . For the sets $\\left\\{a_{i_{t}}\\right\\}$ this is obvious since $a_{i_{t}} \\leqslant 1$. For the sets $\\left\\{a_{i_{t}}+1, \\ldots, a_{i_{t+1}-1}\\right\\}$ this follows from the fact that the corresponding segments lie between two neighboring empty dots, or between an endpoint of $S$ and its nearest empty dot. Therefore the sum of their lengths cannot exceed 1.","tier":0} +{"problem_type":"Combinatorics","problem_label":"C1","problem":"Let $n$ be a positive integer. Find the smallest integer $k$ with the following property: Given any real numbers $a_{1}, \\ldots, a_{d}$ such that $a_{1}+a_{2}+\\cdots+a_{d}=n$ and $0 \\leqslant a_{i} \\leqslant 1$ for $i=1,2, \\ldots, d$, it is possible to partition these numbers into $k$ groups (some of which may be empty) such that the sum of the numbers in each group is at most 1 . (Poland) Answer. $k=2 n-1$.","solution":"First put all numbers greater than $\\frac{1}{2}$ in their own groups. Then, form the remaining groups as follows: For each group, add new $a_{i} \\mathrm{~S}$ one at a time until their sum exceeds $\\frac{1}{2}$. Since the last summand is at most $\\frac{1}{2}$, this group has sum at most 1 . Continue this procedure until we have used all the $a_{i}$ s. Notice that the last group may have sum less than $\\frac{1}{2}$. If the sum of the numbers in the last two groups is less than or equal to 1, we merge them into one group. In the end we are left with $m$ groups. If $m=1$ we are done. Otherwise the first $m-2$ have sums greater than $\\frac{1}{2}$ and the last two have total sum greater than 1 . Therefore $n>(m-2) \/ 2+1$ so $m \\leqslant 2 n-1$ as desired. Comment 1. The original proposal asked for the minimal value of $k$ when $n=2$. Comment 2. More generally, one may ask the same question for real numbers between 0 and 1 whose sum is a real number $r$. In this case the smallest value of $k$ is $k=\\lceil 2 r\\rceil-1$, as Solution 3 shows. Solutions 1 and 2 lead to the slightly weaker bound $k \\leqslant 2\\lceil r\\rceil-1$. This is actually the optimal bound for partitions into consecutive groups, which are the ones contemplated in these two solutions. To see this, assume that $r$ is not an integer and let $c=(r+1-\\lceil r\\rceil) \/(1+\\lceil r\\rceil)$. One easily checks that $0N$.","tier":0} +{"problem_type":"Combinatorics","problem_label":"C5","problem":"Let $r$ be a positive integer, and let $a_{0}, a_{1}, \\ldots$ be an infinite sequence of real numbers. Assume that for all nonnegative integers $m$ and $s$ there exists a positive integer $n \\in[m+1, m+r]$ such that $$ a_{m}+a_{m+1}+\\cdots+a_{m+s}=a_{n}+a_{n+1}+\\cdots+a_{n+s} $$ Prove that the sequence is periodic, i. e. there exists some $p \\geqslant 1$ such that $a_{n+p}=a_{n}$ for all $n \\geqslant 0$. (India)","solution":"For every indices $m \\leqslant n$ we will denote $S(m, n)=a_{m}+a_{m+1}+\\cdots+a_{n-1}$; thus $S(n, n)=0$. Let us start with the following lemma. Lemma. Let $b_{0}, b_{1}, \\ldots$ be an infinite sequence. Assume that for every nonnegative integer $m$ there exists a nonnegative integer $n \\in[m+1, m+r]$ such that $b_{m}=b_{n}$. Then for every indices $k \\leqslant \\ell$ there exists an index $t \\in[\\ell, \\ell+r-1]$ such that $b_{t}=b_{k}$. Moreover, there are at most $r$ distinct numbers among the terms of $\\left(b_{i}\\right)$. Proof. To prove the first claim, let us notice that there exists an infinite sequence of indices $k_{1}=k, k_{2}, k_{3}, \\ldots$ such that $b_{k_{1}}=b_{k_{2}}=\\cdots=b_{k}$ and $k_{i}1 \/ 2^{m}$. Next, any interval blackened by $B$ before the $r$ th move which intersects $\\left(x_{r}, x_{r+1}\\right)$ should be contained in $\\left[x_{r}, x_{r+1}\\right]$; by (ii), all such intervals have different lengths not exceeding $1 \/ 2^{m}$, so the total amount of ink used for them is less than $2 \/ 2^{m}$. Thus, the amount of ink used for the segment $\\left[0, x_{r+1}\\right]$ does not exceed the sum of $2 \/ 2^{m}, 3 x_{r}$ (used for $\\left[0, x_{r}\\right]$ ), and $1 \/ 2^{m}$ used for the segment $I_{0}^{r}$. In total it gives at most $3\\left(x_{r}+1 \/ 2^{m}\\right)<3\\left(x_{r}+\\alpha\\right)=3 x_{r+1}$. Thus condition $(i)$ is also verified in this case. The claim is proved. Finally, we can perform the desired estimation. Consider any situation in the game, say after the $(r-1)$ st move; assume that the segment $[0,1]$ is not completely black. By $(i i)$, in the segment $\\left[x_{r}, 1\\right]$ player $B$ has colored several segments of different lengths; all these lengths are negative powers of 2 not exceeding $1-x_{r}$; thus the total amount of ink used for this interval is at most $2\\left(1-x_{r}\\right)$. Using $(i)$, we obtain that the total amount of ink used is at most $3 x_{r}+2\\left(1-x_{r}\\right)<3$. Thus the pot is not empty, and therefore $A$ never wins. Comment 1. Notice that this strategy works even if the pot contains initially only 3 units of ink. Comment 2. There exist other strategies for $B$ allowing him to prevent emptying the pot before the whole interval is colored. On the other hand, let us mention some idea which does not work. Player $B$ could try a strategy in which the set of blackened points in each round is an interval of the type $[0, x]$. Such a strategy cannot work (even if there is more ink available). Indeed, under the assumption that $B$ uses such a strategy, let us prove by induction on $s$ the following statement: For any positive integer $s$, player $A$ has a strategy picking only positive integers $m \\leqslant s$ in which, if player $B$ ever paints a point $x \\geqslant 1-1 \/ 2^{s}$ then after some move, exactly the interval $\\left[0,1-1 \/ 2^{s}\\right]$ is blackened, and the amount of ink used up to this moment is at least s\/2. For the base case $s=1$, player $A$ just picks $m=1$ in the first round. If for some positive integer $k$ player $A$ has such a strategy, for $s+1$ he can first rescale his strategy to the interval $[0,1 \/ 2]$ (sending in each round half of the amount of ink he would give by the original strategy). Thus, after some round, the interval $\\left[0,1 \/ 2-1 \/ 2^{s+1}\\right]$ becomes blackened, and the amount of ink used is at least $s \/ 4$. Now player $A$ picks $m=1 \/ 2$, and player $B$ spends $1 \/ 2$ unit of ink to blacken the interval [0,1\/2]. After that, player $A$ again rescales his strategy to the interval $[1 \/ 2,1]$, and player $B$ spends at least $s \/ 4$ units of ink to blacken the interval $\\left[1 \/ 2,1-1 \/ 2^{s+1}\\right]$, so he spends in total at least $s \/ 4+1 \/ 2+s \/ 4=(s+1) \/ 2$ units of ink. Comment 3. In order to avoid finiteness issues, the statement could be replaced by the following one: Players $A$ and $B$ play a paintful game on the real numbers. Player $A$ has a paint pot with four units of black ink. A quantity $p$ of this ink suffices to blacken a (closed) real interval of length $p$. In the beginning of the game, player $A$ chooses (and announces) a positive integer $N$. In every round, player $A$ picks some positive integer $m \\leqslant N$ and provides $1 \/ 2^{m}$ units of ink from the pot. The player $B$ picks an integer $k$ and blackens the interval from $k \/ 2^{m}$ to $(k+1) \/ 2^{m}$ (some parts of this interval may happen to be blackened before). The goal of player $A$ is to reach a situation where the pot is empty and the interval $[0,1]$ is not completely blackened. Decide whether there exists a strategy for player A to win. However, the Problem Selection Committee believes that this version may turn out to be harder than the original one.","tier":0} +{"problem_type":"Geometry","problem_label":"G5","problem":"Let $A B C D E F$ be a convex hexagon with $A B=D E, B C=E F, C D=F A$, and $\\angle A-\\angle D=\\angle C-\\angle F=\\angle E-\\angle B$. Prove that the diagonals $A D, B E$, and $C F$ are concurrent. (Ukraine) In all three solutions, we denote $\\theta=\\angle A-\\angle D=\\angle C-\\angle F=\\angle E-\\angle B$ and assume without loss of generality that $\\theta \\geqslant 0$.","solution":"Let $x=A B=D E, y=C D=F A, z=E F=B C$. Consider the points $P, Q$, and $R$ such that the quadrilaterals $C D E P, E F A Q$, and $A B C R$ are parallelograms. We compute $$ \\begin{aligned} \\angle P E Q & =\\angle F E Q+\\angle D E P-\\angle E=\\left(180^{\\circ}-\\angle F\\right)+\\left(180^{\\circ}-\\angle D\\right)-\\angle E \\\\ & =360^{\\circ}-\\angle D-\\angle E-\\angle F=\\frac{1}{2}(\\angle A+\\angle B+\\angle C-\\angle D-\\angle E-\\angle F)=\\theta \/ 2 \\end{aligned} $$ Similarly, $\\angle Q A R=\\angle R C P=\\theta \/ 2$. ![](https:\/\/cdn.mathpix.com\/cropped\/2024_04_17_e0914cb4f8055c731538g-46.jpg?height=491&width=827&top_left_y=947&top_left_x=649) If $\\theta=0$, since $\\triangle R C P$ is isosceles, $R=P$. Therefore $A B\\|R C=P C\\| E D$, so $A B D E$ is a parallelogram. Similarly, $B C E F$ and $C D F A$ are parallelograms. It follows that $A D, B E$ and $C F$ meet at their common midpoint. Now assume $\\theta>0$. Since $\\triangle P E Q, \\triangle Q A R$, and $\\triangle R C P$ are isosceles and have the same angle at the apex, we have $\\triangle P E Q \\sim \\triangle Q A R \\sim \\triangle R C P$ with ratios of similarity $y: z: x$. Thus $\\triangle P Q R$ is similar to the triangle with sidelengths $y, z$, and $x$. Next, notice that $$ \\frac{R Q}{Q P}=\\frac{z}{y}=\\frac{R A}{A F} $$ and, using directed angles between rays, $$ \\begin{aligned} \\not(R Q, Q P) & =\\Varangle(R Q, Q E)+\\Varangle(Q E, Q P) \\\\ & =\\Varangle(R Q, Q E)+\\Varangle(R A, R Q)=\\Varangle(R A, Q E)=\\Varangle(R A, A F) . \\end{aligned} $$ Thus $\\triangle P Q R \\sim \\triangle F A R$. Since $F A=y$ and $A R=z$, (1) then implies that $F R=x$. Similarly $F P=x$. Therefore $C R F P$ is a rhombus. We conclude that $C F$ is the perpendicular bisector of $P R$. Similarly, $B E$ is the perpendicular bisector of $P Q$ and $A D$ is the perpendicular bisector of $Q R$. It follows that $A D, B E$, and $C F$ are concurrent at the circumcenter of $P Q R$.","tier":0} +{"problem_type":"Geometry","problem_label":"G5","problem":"Let $A B C D E F$ be a convex hexagon with $A B=D E, B C=E F, C D=F A$, and $\\angle A-\\angle D=\\angle C-\\angle F=\\angle E-\\angle B$. Prove that the diagonals $A D, B E$, and $C F$ are concurrent. (Ukraine) In all three solutions, we denote $\\theta=\\angle A-\\angle D=\\angle C-\\angle F=\\angle E-\\angle B$ and assume without loss of generality that $\\theta \\geqslant 0$.","solution":"Let $X=C D \\cap E F, Y=E F \\cap A B, Z=A B \\cap C D, X^{\\prime}=F A \\cap B C, Y^{\\prime}=$ $B C \\cap D E$, and $Z^{\\prime}=D E \\cap F A$. From $\\angle A+\\angle B+\\angle C=360^{\\circ}+\\theta \/ 2$ we get $\\angle A+\\angle B>180^{\\circ}$ and $\\angle B+\\angle C>180^{\\circ}$, so $Z$ and $X^{\\prime}$ are respectively on the opposite sides of $B C$ and $A B$ from the hexagon. Similar conclusions hold for $X, Y, Y^{\\prime}$, and $Z^{\\prime}$. Then $$ \\angle Y Z X=\\angle B+\\angle C-180^{\\circ}=\\angle E+\\angle F-180^{\\circ}=\\angle Y^{\\prime} Z^{\\prime} X^{\\prime}, $$ and similarly $\\angle Z X Y=\\angle Z^{\\prime} X^{\\prime} Y^{\\prime}$ and $\\angle X Y Z=\\angle X^{\\prime} Y^{\\prime} Z^{\\prime}$, so $\\triangle X Y Z \\sim \\triangle X^{\\prime} Y^{\\prime} Z^{\\prime}$. Thus there is a rotation $R$ which sends $\\triangle X Y Z$ to a triangle with sides parallel to $\\triangle X^{\\prime} Y^{\\prime} Z^{\\prime}$. Since $A B=D E$ we have $R(\\overrightarrow{A B})=\\overrightarrow{D E}$. Similarly, $R(\\overrightarrow{C D})=\\overrightarrow{F A}$ and $R(\\overrightarrow{E F})=\\overrightarrow{B C}$. Therefore $$ \\overrightarrow{0}=\\overrightarrow{A B}+\\overrightarrow{B C}+\\overrightarrow{C D}+\\overrightarrow{D E}+\\overrightarrow{E F}+\\overrightarrow{F A}=(\\overrightarrow{A B}+\\overrightarrow{C D}+\\overrightarrow{E F})+R(\\overrightarrow{A B}+\\overrightarrow{C D}+\\overrightarrow{E F}) $$ If $R$ is a rotation by $180^{\\circ}$, then any two opposite sides of our hexagon are equal and parallel, so the three diagonals meet at their common midpoint. Otherwise, we must have $$ \\overrightarrow{A B}+\\overrightarrow{C D}+\\overrightarrow{E F}=\\overrightarrow{0} $$ or else we would have two vectors with different directions whose sum is $\\overrightarrow{0}$. ![](https:\/\/cdn.mathpix.com\/cropped\/2024_04_17_e0914cb4f8055c731538g-47.jpg?height=689&width=1333&top_left_y=1057&top_left_x=388) This allows us to consider a triangle $L M N$ with $\\overrightarrow{L M}=\\overrightarrow{E F}, \\overrightarrow{M N}=\\overrightarrow{A B}$, and $\\overrightarrow{N L}=\\overrightarrow{C D}$. Let $O$ be the circumcenter of $\\triangle L M N$ and consider the points $O_{1}, O_{2}, O_{3}$ such that $\\triangle A O_{1} B, \\triangle C O_{2} D$, and $\\triangle E O_{3} F$ are translations of $\\triangle M O N, \\triangle N O L$, and $\\triangle L O M$, respectively. Since $F O_{3}$ and $A O_{1}$ are translations of $M O$, quadrilateral $A F O_{3} O_{1}$ is a parallelogram and $O_{3} O_{1}=F A=C D=N L$. Similarly, $O_{1} O_{2}=L M$ and $O_{2} O_{3}=M N$. Therefore $\\triangle O_{1} O_{2} O_{3} \\cong \\triangle L M N$. Moreover, by means of the rotation $R$ one may check that these triangles have the same orientation. Let $T$ be the circumcenter of $\\triangle O_{1} O_{2} O_{3}$. We claim that $A D, B E$, and $C F$ meet at $T$. Let us show that $C, T$, and $F$ are collinear. Notice that $C O_{2}=O_{2} T=T O_{3}=O_{3} F$ since they are all equal to the circumradius of $\\triangle L M N$. Therefore $\\triangle T O_{3} F$ and $\\triangle C O_{2} T$ are isosceles. Using directed angles between rays again, we get $$ \\Varangle\\left(T F, T O_{3}\\right)=\\Varangle\\left(F O_{3}, F T\\right) \\quad \\text { and } \\quad \\Varangle\\left(T O_{2}, T C\\right)=\\Varangle\\left(C T, C O_{2}\\right) \\text {. } $$ Also, $T$ and $O$ are the circumcenters of the congruent triangles $\\triangle O_{1} O_{2} O_{3}$ and $\\triangle L M N$ so we have $\\Varangle\\left(T O_{3}, T O_{2}\\right)=\\Varangle(O N, O M)$. Since $C_{2}$ and $F O_{3}$ are translations of $N O$ and $M O$ respectively, this implies $$ \\Varangle\\left(T O_{3}, T O_{2}\\right)=\\Varangle\\left(C O_{2}, F O_{3}\\right) . $$ Adding the three equations in (2) and (3) gives $$ \\Varangle(T F, T C)=\\Varangle(C T, F T)=-\\not(T F, T C) $$ which implies that $T$ is on $C F$. Analogous arguments show that it is on $A D$ and $B E$ also. The desired result follows.","tier":0} +{"problem_type":"Geometry","problem_label":"G5","problem":"Let $A B C D E F$ be a convex hexagon with $A B=D E, B C=E F, C D=F A$, and $\\angle A-\\angle D=\\angle C-\\angle F=\\angle E-\\angle B$. Prove that the diagonals $A D, B E$, and $C F$ are concurrent. (Ukraine) In all three solutions, we denote $\\theta=\\angle A-\\angle D=\\angle C-\\angle F=\\angle E-\\angle B$ and assume without loss of generality that $\\theta \\geqslant 0$.","solution":"Place the hexagon on the complex plane, with $A$ at the origin and vertices labelled clockwise. Now $A, B, C, D, E, F$ represent the corresponding complex numbers. Also consider the complex numbers $a, b, c, a^{\\prime}, b^{\\prime}, c^{\\prime}$ given by $B-A=a, D-C=b, F-E=c, E-D=a^{\\prime}$, $A-F=b^{\\prime}$, and $C-B=c^{\\prime}$. Let $k=|a| \/|b|$. From $a \/ b^{\\prime}=-k e^{i \\angle A}$ and $a^{\\prime} \/ b=-k e^{i \\angle D}$ we get that $\\left(a^{\\prime} \/ a\\right)\\left(b^{\\prime} \/ b\\right)=e^{-i \\theta}$ and similarly $\\left(b^{\\prime} \/ b\\right)\\left(c^{\\prime} \/ c\\right)=e^{-i \\theta}$ and $\\left(c^{\\prime} \/ c\\right)\\left(a^{\\prime} \/ a\\right)=e^{-i \\theta}$. It follows that $a^{\\prime}=a r$, $b^{\\prime}=b r$, and $c^{\\prime}=c r$ for a complex number $r$ with $|r|=1$, as shown below. ![](https:\/\/cdn.mathpix.com\/cropped\/2024_04_17_e0914cb4f8055c731538g-48.jpg?height=512&width=1052&top_left_y=823&top_left_x=534) We have $$ 0=a+c r+b+a r+c+b r=(a+b+c)(1+r) . $$ If $r=-1$, then the hexagon is centrally symmetric and its diagonals intersect at its center of symmetry. Otherwise $$ a+b+c=0 \\text {. } $$ Therefore $$ A=0, \\quad B=a, \\quad C=a+c r, \\quad D=c(r-1), \\quad E=-b r-c, \\quad F=-b r . $$ Now consider a point $W$ on $A D$ given by the complex number $c(r-1) \\lambda$, where $\\lambda$ is a real number with $0<\\lambda<1$. Since $D \\neq A$, we have $r \\neq 1$, so we can define $s=1 \/(r-1)$. From $r \\bar{r}=|r|^{2}=1$ we get $$ 1+s=\\frac{r}{r-1}=\\frac{r}{r-r \\bar{r}}=\\frac{1}{1-\\bar{r}}=-\\bar{s} . $$ Now, $$ \\begin{aligned} W \\text { is on } B E & \\Longleftrightarrow c(r-1) \\lambda-a\\|a-(-b r-c)=b(r-1) \\Longleftrightarrow c \\lambda-a s\\| b \\\\ & \\Longleftrightarrow-a \\lambda-b \\lambda-a s\\|b \\Longleftrightarrow a(\\lambda+s)\\| b . \\end{aligned} $$ One easily checks that $r \\neq \\pm 1$ implies that $\\lambda+s \\neq 0$ since $s$ is not real. On the other hand, $$ \\begin{aligned} W \\text { on } C F & \\Longleftrightarrow c(r-1) \\lambda+b r\\|-b r-(a+c r)=a(r-1) \\Longleftrightarrow c \\lambda+b(1+s)\\| a \\\\ & \\Longleftrightarrow-a \\lambda-b \\lambda-b \\bar{s}\\|a \\Longleftrightarrow b(\\lambda+\\bar{s})\\| a \\Longleftrightarrow b \\| a(\\lambda+s), \\end{aligned} $$ where in the last step we use that $(\\lambda+s)(\\lambda+\\bar{s})=|\\lambda+s|^{2} \\in \\mathbb{R}_{>0}$. We conclude that $A D \\cap B E=$ $C F \\cap B E$, and the desired result follows.","tier":0} +{"problem_type":"Number Theory","problem_label":"N1","problem":"Let $\\mathbb{Z}_{>0}$ be the set of positive integers. Find all functions $f: \\mathbb{Z}_{>0} \\rightarrow \\mathbb{Z}_{>0}$ such that $$ m^{2}+f(n) \\mid m f(m)+n $$ for all positive integers $m$ and $n$. (Malaysia) Answer. $f(n)=n$.","solution":"Setting $m=n=2$ tells us that $4+f(2) \\mid 2 f(2)+2$. Since $2 f(2)+2<2(4+f(2))$, we must have $2 f(2)+2=4+f(2)$, so $f(2)=2$. Plugging in $m=2$ then tells us that $4+f(n) \\mid 4+n$, which implies that $f(n) \\leqslant n$ for all $n$. Setting $m=n$ gives $n^{2}+f(n) \\mid n f(n)+n$, so $n f(n)+n \\geqslant n^{2}+f(n)$ which we rewrite as $(n-1)(f(n)-n) \\geqslant 0$. Therefore $f(n) \\geqslant n$ for all $n \\geqslant 2$. This is trivially true for $n=1$ also. It follows that $f(n)=n$ for all $n$. This function obviously satisfies the desired property.","tier":0} +{"problem_type":"Number Theory","problem_label":"N1","problem":"Let $\\mathbb{Z}_{>0}$ be the set of positive integers. Find all functions $f: \\mathbb{Z}_{>0} \\rightarrow \\mathbb{Z}_{>0}$ such that $$ m^{2}+f(n) \\mid m f(m)+n $$ for all positive integers $m$ and $n$. (Malaysia) Answer. $f(n)=n$.","solution":"Setting $m=f(n)$ we get $f(n)(f(n)+1) \\mid f(n) f(f(n))+n$. This implies that $f(n) \\mid n$ for all $n$. Now let $m$ be any positive integer, and let $p>2 m^{2}$ be a prime number. Note that $p>m f(m)$ also. Plugging in $n=p-m f(m)$ we learn that $m^{2}+f(n)$ divides $p$. Since $m^{2}+f(n)$ cannot equal 1, it must equal $p$. Therefore $p-m^{2}=f(n) \\mid n=p-m f(m)$. But $p-m f(m)0}$ be the set of positive integers. Find all functions $f: \\mathbb{Z}_{>0} \\rightarrow \\mathbb{Z}_{>0}$ such that $$ m^{2}+f(n) \\mid m f(m)+n $$ for all positive integers $m$ and $n$. (Malaysia) Answer. $f(n)=n$.","solution":"Plugging $m=1$ we obtain $1+f(n) \\leqslant f(1)+n$, so $f(n) \\leqslant n+c$ for the constant $c=$ $f(1)-1$. Assume that $f(n) \\neq n$ for some fixed $n$. When $m$ is large enough (e.g. $m \\geqslant \\max (n, c+1)$ ) we have $$ m f(m)+n \\leqslant m(m+c)+n \\leqslant 2 m^{2}<2\\left(m^{2}+f(n)\\right), $$ so we must have $m f(m)+n=m^{2}+f(n)$. This implies that $$ 0 \\neq f(n)-n=m(f(m)-m) $$ which is impossible for $m>|f(n)-n|$. It follows that $f$ is the identity function.","tier":0} +{"problem_type":"Number Theory","problem_label":"N4","problem":"Determine whether there exists an infinite sequence of nonzero digits $a_{1}, a_{2}, a_{3}, \\ldots$ and a positive integer $N$ such that for every integer $k>N$, the number $\\overline{a_{k} a_{k-1} \\ldots a_{1}}$ is a perfect square. (Iran) Answer. No.","solution":"Assume that $a_{1}, a_{2}, a_{3}, \\ldots$ is such a sequence. For each positive integer $k$, let $y_{k}=$ $\\overline{a_{k} a_{k-1} \\ldots a_{1}}$. By the assumption, for each $k>N$ there exists a positive integer $x_{k}$ such that $y_{k}=x_{k}^{2}$. I. For every $n$, let $5^{\\gamma_{n}}$ be the greatest power of 5 dividing $x_{n}$. Let us show first that $2 \\gamma_{n} \\geqslant n$ for every positive integer $n>N$. Assume, to the contrary, that there exists a positive integer $n>N$ such that $2 \\gamma_{n}N$. II. Consider now any integer $k>\\max \\{N \/ 2,2\\}$. Since $2 \\gamma_{2 k+1} \\geqslant 2 k+1$ and $2 \\gamma_{2 k+2} \\geqslant 2 k+2$, we have $\\gamma_{2 k+1} \\geqslant k+1$ and $\\gamma_{2 k+2} \\geqslant k+1$. So, from $y_{2 k+2}=a_{2 k+2} \\cdot 10^{2 k+1}+y_{2 k+1}$ we obtain $5^{2 k+2} \\mid y_{2 k+2}-y_{2 k+1}=a_{2 k+2} \\cdot 10^{2 k+1}$ and thus $5 \\mid a_{2 k+2}$, which implies $a_{2 k+2}=5$. Therefore, $$ \\left(x_{2 k+2}-x_{2 k+1}\\right)\\left(x_{2 k+2}+x_{2 k+1}\\right)=x_{2 k+2}^{2}-x_{2 k+1}^{2}=y_{2 k+2}-y_{2 k+1}=5 \\cdot 10^{2 k+1}=2^{2 k+1} \\cdot 5^{2 k+2} . $$ Setting $A_{k}=x_{2 k+2} \/ 5^{k+1}$ and $B_{k}=x_{2 k+1} \/ 5^{k+1}$, which are integers, we obtain $$ \\left(A_{k}-B_{k}\\right)\\left(A_{k}+B_{k}\\right)=2^{2 k+1} . $$ Both $A_{k}$ and $B_{k}$ are odd, since otherwise $y_{2 k+2}$ or $y_{2 k+1}$ would be a multiple of 10 which is false by $a_{1} \\neq 0$; so one of the numbers $A_{k}-B_{k}$ and $A_{k}+B_{k}$ is not divisible by 4 . Therefore (1) yields $A_{k}-B_{k}=2$ and $A_{k}+B_{k}=2^{2 k}$, hence $A_{k}=2^{2 k-1}+1$ and thus $$ x_{2 k+2}=5^{k+1} A_{k}=10^{k+1} \\cdot 2^{k-2}+5^{k+1}>10^{k+1}, $$ since $k \\geqslant 2$. This implies that $y_{2 k+2}>10^{2 k+2}$ which contradicts the fact that $y_{2 k+2}$ contains $2 k+2$ digits. The desired result follows.","tier":0} +{"problem_type":"Number Theory","problem_label":"N4","problem":"Determine whether there exists an infinite sequence of nonzero digits $a_{1}, a_{2}, a_{3}, \\ldots$ and a positive integer $N$ such that for every integer $k>N$, the number $\\overline{a_{k} a_{k-1} \\ldots a_{1}}$ is a perfect square. (Iran) Answer. No.","solution":"Again, we assume that a sequence $a_{1}, a_{2}, a_{3}, \\ldots$ satisfies the problem conditions, introduce the numbers $x_{k}$ and $y_{k}$ as in the previous solution, and notice that $$ y_{k+1}-y_{k}=\\left(x_{k+1}-x_{k}\\right)\\left(x_{k+1}+x_{k}\\right)=10^{k} a_{k+1} $$ for all $k>N$. Consider any such $k$. Since $a_{1} \\neq 0$, the numbers $x_{k}$ and $x_{k+1}$ are not multiples of 10 , and therefore the numbers $p_{k}=x_{k+1}-x_{k}$ and $q_{k}=x_{k+1}+x_{k}$ cannot be simultaneously multiples of 20 , and hence one of them is not divisible either by 4 or by 5 . In view of (2), this means that the other one is divisible by either $5^{k}$ or by $2^{k-1}$. Notice also that $p_{k}$ and $q_{k}$ have the same parity, so both are even. On the other hand, we have $x_{k+1}^{2}=x_{k}^{2}+10^{k} a_{k+1} \\geqslant x_{k}^{2}+10^{k}>2 x_{k}^{2}$, so $x_{k+1} \/ x_{k}>\\sqrt{2}$, which implies that $$ 1<\\frac{q_{k}}{p_{k}}=1+\\frac{2}{x_{k+1} \/ x_{k}-1}<1+\\frac{2}{\\sqrt{2}-1}<6 . $$ Thus, if one of the numbers $p_{k}$ and $q_{k}$ is divisible by $5^{k}$, then we have $$ 10^{k+1}>10^{k} a_{k+1}=p_{k} q_{k} \\geqslant \\frac{\\left(5^{k}\\right)^{2}}{6} $$ and hence $(5 \/ 2)^{k}<60$ which is false for sufficiently large $k$. So, assuming that $k$ is large, we get that $2^{k-1}$ divides one of the numbers $p_{k}$ and $q_{k}$. Hence $$ \\left\\{p_{k}, q_{k}\\right\\}=\\left\\{2^{k-1} \\cdot 5^{r_{k}} b_{k}, 2 \\cdot 5^{k-r_{k}} c_{k}\\right\\} \\quad \\text { with nonnegative integers } b_{k}, c_{k}, r_{k} \\text { such that } b_{k} c_{k}=a_{k+1} \\text {. } $$ Moreover, from (3) we get $$ 6>\\frac{2^{k-1} \\cdot 5^{r_{k}} b_{k}}{2 \\cdot 5^{k-r_{k}} c_{k}} \\geqslant \\frac{1}{36} \\cdot\\left(\\frac{2}{5}\\right)^{k} \\cdot 5^{2 r_{k}} \\quad \\text { and } \\quad 6>\\frac{2 \\cdot 5^{k-r_{k}} c_{k}}{2^{k-1} \\cdot 5^{r_{k}} b_{k}} \\geqslant \\frac{4}{9} \\cdot\\left(\\frac{5}{2}\\right)^{k} \\cdot 5^{-2 r_{k}} $$ SO $$ \\alpha k+c_{1}c_{1} \\text {. } $$ Consequently, for $C=c_{2}-c_{1}+1-\\alpha>0$ we have $$ (k+1)-r_{k+1} \\leqslant k-r_{k}+C . $$ Next, we will use the following easy lemma. Lemma. Let $s$ be a positive integer. Then $5^{s+2^{s}} \\equiv 5^{s}\\left(\\bmod 10^{s}\\right)$. Proof. Euler's theorem gives $5^{2^{s}} \\equiv 1\\left(\\bmod 2^{s}\\right)$, so $5^{s+2^{s}}-5^{s}=5^{s}\\left(5^{2^{s}}-1\\right)$ is divisible by $2^{s}$ and $5^{s}$. Now, for every large $k$ we have $$ x_{k+1}=\\frac{p_{k}+q_{k}}{2}=5^{r_{k}} \\cdot 2^{k-2} b_{k}+5^{k-r_{k}} c_{k} \\equiv 5^{k-r_{k}} c_{k} \\quad\\left(\\bmod 10^{r_{k}}\\right) $$ since $r_{k} \\leqslant k-2$ by $(4)$; hence $y_{k+1} \\equiv 5^{2\\left(k-r_{k}\\right)} c_{k}^{2}\\left(\\bmod 10^{r_{k}}\\right)$. Let us consider some large integer $s$, and choose the minimal $k$ such that $2\\left(k-r_{k}\\right) \\geqslant s+2^{s}$; it exists by (4). Set $d=2\\left(k-r_{k}\\right)-\\left(s+2^{s}\\right)$. By (4) we have $2^{s}<2\\left(k-r_{k}\\right)<\\left(\\frac{2}{\\alpha}-2\\right) r_{k}-\\frac{2 c_{1}}{\\alpha}$; if $s$ is large this implies $r_{k}>s$, so (6) also holds modulo $10^{s}$. Then (6) and the lemma give $$ y_{k+1} \\equiv 5^{2\\left(k-r_{k}\\right)} c_{k}^{2}=5^{s+2^{s}} \\cdot 5^{d} c_{k}^{2} \\equiv 5^{s} \\cdot 5^{d} c_{k}^{2} \\quad\\left(\\bmod 10^{s}\\right) . $$ By (5) and the minimality of $k$ we have $d \\leqslant 2 C$, so $5^{d} c_{k}^{2} \\leqslant 5^{2 C} \\cdot 81=D$. Using $5^{4}<10^{3}$ we obtain $$ 5^{s} \\cdot 5^{d} c_{k}^{2}<10^{3 s \/ 4} D<10^{s-1} $$ for sufficiently large $s$. This, together with (7), shows that the sth digit from the right in $y_{k+1}$, which is $a_{s}$, is zero. This contradicts the problem condition.","tier":0} +{"problem_type":"Number Theory","problem_label":"N6","problem":"Determine all functions $f: \\mathbb{Q} \\longrightarrow \\mathbb{Z}$ satisfying $$ f\\left(\\frac{f(x)+a}{b}\\right)=f\\left(\\frac{x+a}{b}\\right) $$ for all $x \\in \\mathbb{Q}, a \\in \\mathbb{Z}$, and $b \\in \\mathbb{Z}_{>0}$. (Here, $\\mathbb{Z}_{>0}$ denotes the set of positive integers.) (Israel) Answer. There are three kinds of such functions, which are: all constant functions, the floor function, and the ceiling function.","solution":"I. We start by verifying that these functions do indeed satisfy (1). This is clear for all constant functions. Now consider any triple $(x, a, b) \\in \\mathbb{Q} \\times \\mathbb{Z} \\times \\mathbb{Z}_{>0}$ and set $$ q=\\left\\lfloor\\frac{x+a}{b}\\right\\rfloor . $$ This means that $q$ is an integer and $b q \\leqslant x+a0}$. According to the behaviour of the restriction of $f$ to the integers we distinguish two cases. Case 1: There is some $m \\in \\mathbb{Z}$ such that $f(m) \\neq m$. Write $f(m)=C$ and let $\\eta \\in\\{-1,+1\\}$ and $b$ denote the sign and absolute value of $f(m)-m$, respectively. Given any integer $r$, we may plug the triple $(m, r b-C, b)$ into (1), thus getting $f(r)=f(r-\\eta)$. Starting with $m$ and using induction in both directions, we deduce from this that the equation $f(r)=C$ holds for all integers $r$. Now any rational number $y$ can be written in the form $y=\\frac{p}{q}$ with $(p, q) \\in \\mathbb{Z} \\times \\mathbb{Z}_{>0}$, and substituting $(C-p, p-C, q)$ into (1) we get $f(y)=f(0)=C$. Thus $f$ is the constant function whose value is always $C$. Case 2: One has $f(m)=m$ for all integers $m$. Note that now the special case $b=1$ of (1) takes a particularly simple form, namely $$ f(x)+a=f(x+a) \\quad \\text { for all }(x, a) \\in \\mathbb{Q} \\times \\mathbb{Z} $$ Defining $f\\left(\\frac{1}{2}\\right)=\\omega$ we proceed in three steps. Step $A$. We show that $\\omega \\in\\{0,1\\}$. If $\\omega \\leqslant 0$, we may plug $\\left(\\frac{1}{2},-\\omega, 1-2 \\omega\\right)$ into (1), obtaining $0=f(0)=f\\left(\\frac{1}{2}\\right)=\\omega$. In the contrary case $\\omega \\geqslant 1$ we argue similarly using the triple $\\left(\\frac{1}{2}, \\omega-1,2 \\omega-1\\right)$. Step B. We show that $f(x)=\\omega$ for all rational numbers $x$ with $0b \\geqslant k+\\omega$, which is absurd. Similarly, $m \\geqslant r$ leads to $r a-m b0} $$ Now suppose first that $x$ is not an integer but can be written in the form $\\frac{p}{q}$ with $p \\in \\mathbb{Z}$ and $q \\in \\mathbb{Z}_{>0}$ both being odd. Let $d$ denote the multiplicative order of 2 modulo $q$ and let $m$ be any large integer. Plugging $n=d m$ into (6) and using (2) we get $$ f(x)=\\left[\\frac{f\\left(2^{d m} x\\right)}{2^{d m}}\\right]=\\left[\\frac{f(x)+\\left(2^{d m}-1\\right) x}{2^{d m}}\\right]=\\left[x+\\frac{f(x)-x}{2^{d m}}\\right] . $$ Since $x$ is not an integer, the square bracket function is continuous at $x$; hence as $m$ tends to infinity the above fomula gives $f(x)=[x]$. To complete the argument we just need to observe that if some $y \\in \\mathbb{Q}$ satisfies $f(y)=[y]$, then (5) yields $f\\left(\\frac{y}{2}\\right)=f\\left(\\frac{[y]}{2}\\right)=\\left[\\frac{[y]}{2}\\right]=\\left[\\frac{y}{2}\\right]$.","tier":0} +{"problem_type":"Number Theory","problem_label":"N6","problem":"Determine all functions $f: \\mathbb{Q} \\longrightarrow \\mathbb{Z}$ satisfying $$ f\\left(\\frac{f(x)+a}{b}\\right)=f\\left(\\frac{x+a}{b}\\right) $$ for all $x \\in \\mathbb{Q}, a \\in \\mathbb{Z}$, and $b \\in \\mathbb{Z}_{>0}$. (Here, $\\mathbb{Z}_{>0}$ denotes the set of positive integers.) (Israel) Answer. There are three kinds of such functions, which are: all constant functions, the floor function, and the ceiling function.","solution":"Here we just give another argument for the second case of the above solution. Again we use equation (2). It follows that the set $S$ of all zeros of $f$ contains for each $x \\in \\mathbb{Q}$ exactly one term from the infinite sequence $\\ldots, x-2, x-1, x, x+1, x+2, \\ldots$. Next we claim that $$ \\text { if }(p, q) \\in \\mathbb{Z} \\times \\mathbb{Z}_{>0} \\text { and } \\frac{p}{q} \\in S \\text {, then } \\frac{p}{q+1} \\in S \\text { holds as well. } $$ To see this we just plug $\\left(\\frac{p}{q}, p, q+1\\right)$ into (1), thus getting $f\\left(\\frac{p}{q+1}\\right)=f\\left(\\frac{p}{q}\\right)=0$. From this we get that $$ \\text { if } x, y \\in \\mathbb{Q}, x>y>0 \\text {, and } x \\in S \\text {, then } y \\in S \\text {. } $$ Indeed, if we write $x=\\frac{p}{q}$ and $y=\\frac{r}{s}$ with $p, q, r, s \\in \\mathbb{Z}_{>0}$, then $p s>q r$ and (7) tells us $$ 0=f\\left(\\frac{p}{q}\\right)=f\\left(\\frac{p r}{q r}\\right)=f\\left(\\frac{p r}{q r+1}\\right)=\\ldots=f\\left(\\frac{p r}{p s}\\right)=f\\left(\\frac{r}{s}\\right) $$ Essentially the same argument also establishes that $$ \\text { if } x, y \\in \\mathbb{Q}, x2 m$. Such a pair is necessarily a child of $(c, d-c)$, and thus a descendant of some pair $\\left(c, d^{\\prime}\\right)$ with $m\\{(b+k a) \\nu\\}=\\left\\{\\left(b+k_{0} a\\right) \\nu\\right\\}+\\left(k-k_{0}\\right)\\{a \\nu\\}$ for all $k>k_{0}$, which is absurd. Similarly, one can prove that $S_{m}$ contains finitely many pairs $(c, d)$ with $c>2 m$, thus finitely many elements at all. We are now prepared for proving the following crucial lemma. Lemma. Consider any pair $(a, b)$ with $f(a, b) \\neq m$. Then the number $g(a, b)$ of its $m$-excellent descendants is equal to the number $h(a, b)$ of ways to represent the number $t=m-f(a, b)$ as $t=k a+\\ell b$ with $k$ and $\\ell$ being some nonnegative integers. Proof. We proceed by induction on the number $N$ of descendants of $(a, b)$ in $S_{m}$. If $N=0$ then clearly $g(a, b)=0$. Assume that $h(a, b)>0$; without loss of generality, we have $a \\leqslant b$. Then, clearly, $m-f(a, b) \\geqslant a$, so $f(a, b+a) \\leqslant f(a, b)+a \\leqslant m$ and $a \\leqslant m$, hence $(a, b+a) \\in S_{m}$ which is impossible. Thus in the base case we have $g(a, b)=h(a, b)=0$, as desired. Now let $N>0$. Assume that $f(a+b, b)=f(a, b)+b$ and $f(a, b+a)=f(a, b)$ (the other case is similar). If $f(a, b)+b \\neq m$, then by the induction hypothesis we have $$ g(a, b)=g(a+b, b)+g(a, b+a)=h(a+b, b)+h(a, b+a) $$ Notice that both pairs $(a+b, b)$ and $(a, b+a)$ are descendants of $(a, b)$ and thus each of them has strictly less descendants in $S_{m}$ than $(a, b)$ does. Next, each one of the $h(a+b, b)$ representations of $m-f(a+b, b)=m-b-f(a, b)$ as the sum $k^{\\prime}(a+b)+\\ell^{\\prime} b$ provides the representation $m-f(a, b)=k a+\\ell b$ with $k=k^{\\prime}1} d=\\sum_{d \\mid m} d $$ as required. Comment. Let us present a sketch of an outline of a different solution. The plan is to check that the number of excellent pairs does not depend on the (irrational) number $\\nu$, and to find this number for some appropriate value of $\\nu$. For that, we first introduce some geometrical language. We deal only with the excellent pairs $(a, b)$ with $a \\neq b$. Part I. Given an irrational positive $\\nu$, for every positive integer $n$ we introduce two integral points $F_{\\nu}(n)=$ $(n,\\lfloor n \\nu\\rfloor)$ and $C_{\\nu}(n)=(n,\\lceil n \\nu\\rceil)$ on the coordinate plane $O x y$. Then $(*)$ reads as $\\left[O F_{\\nu}(a) C_{\\nu}(b)\\right]=m \/ 2$; here $[\\cdot]$ stands for the signed area. Next, we rewrite in these terms the condition on a pair $(a, b)$ to be excellent. Let $\\ell_{\\nu}, \\ell_{\\nu}^{+}$, and $\\ell_{\\nu}^{-}$be the lines determined by the equations $y=\\nu x, y=\\nu x+1$, and $y=\\nu x-1$, respectively. a). Firstly, we deal with all excellent pairs $(a, b)$ with $aa$ is excellent exactly when $p_{\\nu}(a)$ lies between $b-a$ and $b$, and the point of $f_{\\nu}(a)$ with abscissa $b$ is integral (which means that this point is $C_{\\nu}(b)$ ). Notice now that, if $p_{\\nu}(a)>a$, then the number of excellent pairs of the form $(a, b)$ (with $b>a$ ) is $\\operatorname{gcd}(a,\\lfloor a \\nu\\rfloor)$. ![](https:\/\/cdn.mathpix.com\/cropped\/2024_04_17_e0914cb4f8055c731538g-67.jpg?height=396&width=422&top_left_y=1274&top_left_x=556) Figure 1 ![](https:\/\/cdn.mathpix.com\/cropped\/2024_04_17_e0914cb4f8055c731538g-67.jpg?height=431&width=442&top_left_y=1254&top_left_x=1123) Figure 2 b). Analogously, considering the pairs $(a, b)$ with $a>b$, we fix the value of $b$, introduce the line $c_{\\nu}(b)$ containing all the points $F$ with $\\left[O F C_{\\nu}(b)\\right]=m \/ 2$, assume that this line contains an integral point (which means $\\operatorname{gcd}(b,\\lceil b \\nu\\rceil) \\mid m$ ), and denote the common point of $c_{\\nu}(b)$ and $\\ell_{\\nu}^{-}$by $Q_{\\nu}(b)$, its abscissa being $q_{\\nu}(b)$. Similarly to the previous case, we obtain that the pair $(a, b)$ is excellent exactly when $q_{\\nu}(a)$ lies between $a-b$ and $a$, and the point of $c_{\\nu}(b)$ with abscissa $a$ is integral (see Fig. 2). Again, if $q_{\\nu}(b)>b$, then the number of excellent pairs of the form $(a, b)$ (with $a>b)$ is $\\operatorname{gcd}(b,\\lceil b \\nu\\rceil)$. Part II, sketchy. Having obtained such a description, one may check how the number of excellent pairs changes as $\\nu$ grows. (Having done that, one may find this number for one appropriate value of $\\nu$; for instance, it is relatively easy to make this calculation for $\\nu \\in\\left(1,1+\\frac{1}{m}\\right)$.) Consider, for the initial value of $\\nu$, some excellent pair $(a, t)$ with $a>t$. As $\\nu$ grows, this pair eventually stops being excellent; this happens when the point $Q_{\\nu}(t)$ passes through $F_{\\nu}(a)$. At the same moment, the pair $(a+t, t)$ becomes excellent instead. This process halts when the point $Q_{\\nu}(t)$ eventually disappears, i.e. when $\\nu$ passes through the ratio of the coordinates of the point $T=C_{\\nu}(t)$. Hence, the point $T$ afterwards is regarded as $F_{\\nu}(t)$. Thus, all the old excellent pairs of the form $(a, t)$ with $a>t$ disappear; on the other hand, the same number of excellent pairs with the first element being $t$ just appear. Similarly, if some pair $(t, b)$ with $t0$. Notice that $$ n z_{n+1}-\\left(z_{0}+z_{1}+\\cdots+z_{n}\\right)=(n+1) z_{n+1}-\\left(z_{0}+z_{1}+\\cdots+z_{n}+z_{n+1}\\right)=-d_{n+1} $$ so the second inequality in (1) is equivalent to $d_{n+1} \\leqslant 0$. Therefore, we have to prove that there is a unique index $n \\geqslant 1$ that satisfies $d_{n}>0 \\geqslant d_{n+1}$. By its definition the sequence $d_{1}, d_{2}, \\ldots$ consists of integers and we have $$ d_{1}=\\left(z_{0}+z_{1}\\right)-1 \\cdot z_{1}=z_{0}>0 $$ From $d_{n+1}-d_{n}=\\left(\\left(z_{0}+\\cdots+z_{n}+z_{n+1}\\right)-(n+1) z_{n+1}\\right)-\\left(\\left(z_{0}+\\cdots+z_{n}\\right)-n z_{n}\\right)=n\\left(z_{n}-z_{n+1}\\right)<0$ we can see that $d_{n+1}d_{2}>\\ldots$ of integers such that its first element $d_{1}$ is positive. The sequence must drop below 0 at some point, and thus there is a unique index $n$, that is the index of the last positive term, satisfying $d_{n}>0 \\geqslant d_{n+1}$. Comment. Omitting the assumption that $z_{1}, z_{2}, \\ldots$ are integers allows the numbers $d_{n}$ to be all positive. In such cases the desired $n$ does not exist. This happens for example if $z_{n}=2-\\frac{1}{2^{n}}$ for all integers $n \\geqslant 0$.","tier":0} +{"problem_type":"Algebra","problem_label":"A2","problem":"Define the function $f:(0,1) \\rightarrow(0,1)$ by $$ f(x)= \\begin{cases}x+\\frac{1}{2} & \\text { if } x<\\frac{1}{2} \\\\ x^{2} & \\text { if } x \\geqslant \\frac{1}{2}\\end{cases} $$ Let $a$ and $b$ be two real numbers such that $00$. Show that there exists a positive integer $n$ such that $$ \\left(a_{n}-a_{n-1}\\right)\\left(b_{n}-b_{n-1}\\right)<0 . $$ (Denmark)","solution":"Note that $$ f(x)-x=\\frac{1}{2}>0 $$ if $x<\\frac{1}{2}$ and $$ f(x)-x=x^{2}-x<0 $$ if $x \\geqslant \\frac{1}{2}$. So if we consider $(0,1)$ as being divided into the two subintervals $I_{1}=\\left(0, \\frac{1}{2}\\right)$ and $I_{2}=\\left[\\frac{1}{2}, 1\\right)$, the inequality $$ \\left(a_{n}-a_{n-1}\\right)\\left(b_{n}-b_{n-1}\\right)=\\left(f\\left(a_{n-1}\\right)-a_{n-1}\\right)\\left(f\\left(b_{n-1}\\right)-b_{n-1}\\right)<0 $$ holds if and only if $a_{n-1}$ and $b_{n-1}$ lie in distinct subintervals. Let us now assume, to the contrary, that $a_{k}$ and $b_{k}$ always lie in the same subinterval. Consider the distance $d_{k}=\\left|a_{k}-b_{k}\\right|$. If both $a_{k}$ and $b_{k}$ lie in $I_{1}$, then $$ d_{k+1}=\\left|a_{k+1}-b_{k+1}\\right|=\\left|a_{k}+\\frac{1}{2}-b_{k}-\\frac{1}{2}\\right|=d_{k} $$ If, on the other hand, $a_{k}$ and $b_{k}$ both lie in $I_{2}$, then $\\min \\left(a_{k}, b_{k}\\right) \\geqslant \\frac{1}{2}$ and $\\max \\left(a_{k}, b_{k}\\right)=$ $\\min \\left(a_{k}, b_{k}\\right)+d_{k} \\geqslant \\frac{1}{2}+d_{k}$, which implies $$ d_{k+1}=\\left|a_{k+1}-b_{k+1}\\right|=\\left|a_{k}^{2}-b_{k}^{2}\\right|=\\left|\\left(a_{k}-b_{k}\\right)\\left(a_{k}+b_{k}\\right)\\right| \\geqslant\\left|a_{k}-b_{k}\\right|\\left(\\frac{1}{2}+\\frac{1}{2}+d_{k}\\right)=d_{k}\\left(1+d_{k}\\right) \\geqslant d_{k} $$ This means that the difference $d_{k}$ is non-decreasing, and in particular $d_{k} \\geqslant d_{0}>0$ for all $k$. We can even say more. If $a_{k}$ and $b_{k}$ lie in $I_{2}$, then $$ d_{k+2} \\geqslant d_{k+1} \\geqslant d_{k}\\left(1+d_{k}\\right) \\geqslant d_{k}\\left(1+d_{0}\\right) $$ If $a_{k}$ and $b_{k}$ both lie in $I_{1}$, then $a_{k+1}$ and $b_{k+1}$ both lie in $I_{2}$, and so we have $$ d_{k+2} \\geqslant d_{k+1}\\left(1+d_{k+1}\\right) \\geqslant d_{k+1}\\left(1+d_{0}\\right)=d_{k}\\left(1+d_{0}\\right) $$ In either case, $d_{k+2} \\geqslant d_{k}\\left(1+d_{0}\\right)$, and inductively we get $$ d_{2 m} \\geqslant d_{0}\\left(1+d_{0}\\right)^{m} $$ For sufficiently large $m$, the right-hand side is greater than 1 , but since $a_{2 m}, b_{2 m}$ both lie in $(0,1)$, we must have $d_{2 m}<1$, a contradiction. Thus there must be a positive integer $n$ such that $a_{n-1}$ and $b_{n-1}$ do not lie in the same subinterval, which proves the desired statement.","tier":0} +{"problem_type":"Algebra","problem_label":"A3","problem":"For a sequence $x_{1}, x_{2}, \\ldots, x_{n}$ of real numbers, we define its price as $$ \\max _{1 \\leqslant i \\leqslant n}\\left|x_{1}+\\cdots+x_{i}\\right| $$ Given $n$ real numbers, Dave and George want to arrange them into a sequence with a low price. Diligent Dave checks all possible ways and finds the minimum possible price $D$. Greedy George, on the other hand, chooses $x_{1}$ such that $\\left|x_{1}\\right|$ is as small as possible; among the remaining numbers, he chooses $x_{2}$ such that $\\left|x_{1}+x_{2}\\right|$ is as small as possible, and so on. Thus, in the $i^{\\text {th }}$ step he chooses $x_{i}$ among the remaining numbers so as to minimise the value of $\\left|x_{1}+x_{2}+\\cdots+x_{i}\\right|$. In each step, if several numbers provide the same value, George chooses one at random. Finally he gets a sequence with price $G$. Find the least possible constant $c$ such that for every positive integer $n$, for every collection of $n$ real numbers, and for every possible sequence that George might obtain, the resulting values satisfy the inequality $G \\leqslant c D$. (Georgia)","solution":"If the initial numbers are $1,-1,2$, and -2 , then Dave may arrange them as $1,-2,2,-1$, while George may get the sequence $1,-1,2,-2$, resulting in $D=1$ and $G=2$. So we obtain $c \\geqslant 2$. Therefore, it remains to prove that $G \\leqslant 2 D$. Let $x_{1}, x_{2}, \\ldots, x_{n}$ be the numbers Dave and George have at their disposal. Assume that Dave and George arrange them into sequences $d_{1}, d_{2}, \\ldots, d_{n}$ and $g_{1}, g_{2}, \\ldots, g_{n}$, respectively. Put $$ M=\\max _{1 \\leqslant i \\leqslant n}\\left|x_{i}\\right|, \\quad S=\\left|x_{1}+\\cdots+x_{n}\\right|, \\quad \\text { and } \\quad N=\\max \\{M, S\\} $$ We claim that $$ \\begin{aligned} & D \\geqslant S, \\\\ & D \\geqslant \\frac{M}{2}, \\quad \\text { and } \\\\ & G \\leqslant N=\\max \\{M, S\\} \\end{aligned} $$ These inequalities yield the desired estimate, as $G \\leqslant \\max \\{M, S\\} \\leqslant \\max \\{M, 2 S\\} \\leqslant 2 D$. The inequality (1) is a direct consequence of the definition of the price. To prove (2), consider an index $i$ with $\\left|d_{i}\\right|=M$. Then we have $$ M=\\left|d_{i}\\right|=\\left|\\left(d_{1}+\\cdots+d_{i}\\right)-\\left(d_{1}+\\cdots+d_{i-1}\\right)\\right| \\leqslant\\left|d_{1}+\\cdots+d_{i}\\right|+\\left|d_{1}+\\cdots+d_{i-1}\\right| \\leqslant 2 D $$ as required. It remains to establish (3). Put $h_{i}=g_{1}+g_{2}+\\cdots+g_{i}$. We will prove by induction on $i$ that $\\left|h_{i}\\right| \\leqslant N$. The base case $i=1$ holds, since $\\left|h_{1}\\right|=\\left|g_{1}\\right| \\leqslant M \\leqslant N$. Notice also that $\\left|h_{n}\\right|=S \\leqslant N$. For the induction step, assume that $\\left|h_{i-1}\\right| \\leqslant N$. We distinguish two cases. Case 1. Assume that no two of the numbers $g_{i}, g_{i+1}, \\ldots, g_{n}$ have opposite signs. Without loss of generality, we may assume that they are all nonnegative. Then one has $h_{i-1} \\leqslant h_{i} \\leqslant \\cdots \\leqslant h_{n}$, thus $$ \\left|h_{i}\\right| \\leqslant \\max \\left\\{\\left|h_{i-1}\\right|,\\left|h_{n}\\right|\\right\\} \\leqslant N $$ Case 2. Among the numbers $g_{i}, g_{i+1}, \\ldots, g_{n}$ there are positive and negative ones. Then there exists some index $j \\geqslant i$ such that $h_{i-1} g_{j} \\leqslant 0$. By the definition of George's sequence we have $$ \\left|h_{i}\\right|=\\left|h_{i-1}+g_{i}\\right| \\leqslant\\left|h_{i-1}+g_{j}\\right| \\leqslant \\max \\left\\{\\left|h_{i-1}\\right|,\\left|g_{j}\\right|\\right\\} \\leqslant N $$ Thus, the induction step is established. Comment 1. One can establish the weaker inequalities $D \\geqslant \\frac{M}{2}$ and $G \\leqslant D+\\frac{M}{2}$ from which the result also follows. Comment 2. One may ask a more specific question to find the maximal suitable $c$ if the number $n$ is fixed. For $n=1$ or 2 , the answer is $c=1$. For $n=3$, the answer is $c=\\frac{3}{2}$, and it is reached e.g., for the collection $1,2,-4$. Finally, for $n \\geqslant 4$ the answer is $c=2$. In this case the arguments from the solution above apply, and the answer is reached e.g., for the same collection $1,-1,2,-2$, augmented by several zeroes.","tier":0} +{"problem_type":"Algebra","problem_label":"A4","problem":"Determine all functions $f: \\mathbb{Z} \\rightarrow \\mathbb{Z}$ satisfying $$ f(f(m)+n)+f(m)=f(n)+f(3 m)+2014 $$ for all integers $m$ and $n$.","solution":"Let $f$ be a function satisfying (1). Set $C=1007$ and define the function $g: \\mathbb{Z} \\rightarrow \\mathbb{Z}$ by $g(m)=f(3 m)-f(m)+2 C$ for all $m \\in \\mathbb{Z}$; in particular, $g(0)=2 C$. Now (1) rewrites as $$ f(f(m)+n)=g(m)+f(n) $$ for all $m, n \\in \\mathbb{Z}$. By induction in both directions it follows that $$ f(t f(m)+n)=t g(m)+f(n) $$ holds for all $m, n, t \\in \\mathbb{Z}$. Applying this, for any $r \\in \\mathbb{Z}$, to the triples $(r, 0, f(0))$ and $(0,0, f(r))$ in place of $(m, n, t)$ we obtain $$ f(0) g(r)=f(f(r) f(0))-f(0)=f(r) g(0) $$ Now if $f(0)$ vanished, then $g(0)=2 C>0$ would entail that $f$ vanishes identically, contrary to (1). Thus $f(0) \\neq 0$ and the previous equation yields $g(r)=\\alpha f(r)$, where $\\alpha=\\frac{g(0)}{f(0)}$ is some nonzero constant. So the definition of $g$ reveals $f(3 m)=(1+\\alpha) f(m)-2 C$, i.e., $$ f(3 m)-\\beta=(1+\\alpha)(f(m)-\\beta) $$ for all $m \\in \\mathbb{Z}$, where $\\beta=\\frac{2 C}{\\alpha}$. By induction on $k$ this implies $$ f\\left(3^{k} m\\right)-\\beta=(1+\\alpha)^{k}(f(m)-\\beta) $$ for all integers $k \\geqslant 0$ and $m$. Since $3 \\nmid 2014$, there exists by (1) some value $d=f(a)$ attained by $f$ that is not divisible by 3 . Now by (2) we have $f(n+t d)=f(n)+t g(a)=f(n)+\\alpha \\cdot t f(a)$, i.e., $$ f(n+t d)=f(n)+\\alpha \\cdot t d $$ for all $n, t \\in \\mathbb{Z}$. Let us fix any positive integer $k$ with $d \\mid\\left(3^{k}-1\\right)$, which is possible, since $\\operatorname{gcd}(3, d)=1$. E.g., by the Euler-Fermat theorem, we may take $k=\\varphi(|d|)$. Now for each $m \\in \\mathbb{Z}$ we get $$ f\\left(3^{k} m\\right)=f(m)+\\alpha\\left(3^{k}-1\\right) m $$ from (5), which in view of (4) yields $\\left((1+\\alpha)^{k}-1\\right)(f(m)-\\beta)=\\alpha\\left(3^{k}-1\\right) m$. Since $\\alpha \\neq 0$, the right hand side does not vanish for $m \\neq 0$, wherefore the first factor on the left hand side cannot vanish either. It follows that $$ f(m)=\\frac{\\alpha\\left(3^{k}-1\\right)}{(1+\\alpha)^{k}-1} \\cdot m+\\beta $$ So $f$ is a linear function, say $f(m)=A m+\\beta$ for all $m \\in \\mathbb{Z}$ with some constant $A \\in \\mathbb{Q}$. Plugging this into (1) one obtains $\\left(A^{2}-2 A\\right) m+(A \\beta-2 C)=0$ for all $m$, which is equivalent to the conjunction of $$ A^{2}=2 A \\quad \\text { and } \\quad A \\beta=2 C $$ The first equation is equivalent to $A \\in\\{0,2\\}$, and as $C \\neq 0$ the second one gives $$ A=2 \\quad \\text { and } \\quad \\beta=C $$ This shows that $f$ is indeed the function mentioned in the answer and as the numbers found in (7) do indeed satisfy the equations (6) this function is indeed as desired. Comment 1. One may see that $\\alpha=2$. A more pedestrian version of the above solution starts with a direct proof of this fact, that can be obtained by substituting some special values into (1), e.g., as follows. Set $D=f(0)$. Plugging $m=0$ into (1) and simplifying, we get $$ f(n+D)=f(n)+2 C $$ for all $n \\in \\mathbb{Z}$. In particular, for $n=0, D, 2 D$ we obtain $f(D)=2 C+D, f(2 D)=f(D)+2 C=4 C+D$, and $f(3 D)=f(2 D)+2 C=6 C+D$. So substituting $m=D$ and $n=r-D$ into (1) and applying (8) with $n=r-D$ afterwards we learn $$ f(r+2 C)+2 C+D=(f(r)-2 C)+(6 C+D)+2 C $$ i.e., $f(r+2 C)=f(r)+4 C$. By induction in both directions it follows that $$ f(n+2 C t)=f(n)+4 C t $$ holds for all $n, t \\in \\mathbb{Z}$. Claim. If $a$ and $b$ denote two integers with the property that $f(n+a)=f(n)+b$ holds for all $n \\in \\mathbb{Z}$, then $b=2 a$. Proof. Applying induction in both directions to the assumption we get $f(n+t a)=f(n)+t b$ for all $n, t \\in \\mathbb{Z}$. Plugging $(n, t)=(0,2 C)$ into this equation and $(n, t)=(0, a)$ into $(9)$ we get $f(2 a C)-f(0)=$ $2 b C=4 a C$, and, as $C \\neq 0$, the claim follows. Now by (1), for any $m \\in \\mathbb{Z}$, the numbers $a=f(m)$ and $b=f(3 m)-f(m)+2 C$ have the property mentioned in the claim, whence we have $$ f(3 m)-C=3(f(m)-C) . $$ In view of (3) this tells us indeed that $\\alpha=2$. Now the solution may be completed as above, but due to our knowledge of $\\alpha=2$ we get the desired formula $f(m)=2 m+C$ directly without having the need to go through all linear functions. Now it just remains to check that this function does indeed satisfy (1). Comment 2. It is natural to wonder what happens if one replaces the number 2014 appearing in the statement of the problem by some arbitrary integer $B$. If $B$ is odd, there is no such function, as can be seen by using the same ideas as in the above solution. If $B \\neq 0$ is even, however, then the only such function is given by $n \\longmapsto 2 n+B \/ 2$. In case $3 \\nmid B$ this was essentially proved above, but for the general case one more idea seems to be necessary. Writing $B=3^{\\nu} \\cdot k$ with some integers $\\nu$ and $k$ such that $3 \\nmid k$ one can obtain $f(n)=2 n+B \/ 2$ for all $n$ that are divisible by $3^{\\nu}$ in the same manner as usual; then one may use the formula $f(3 n)=3 f(n)-B$ to establish the remaining cases. Finally, in case $B=0$ there are more solutions than just the function $n \\longmapsto 2 n$. It can be shown that all these other functions are periodic; to mention just one kind of example, for any even integers $r$ and $s$ the function $$ f(n)= \\begin{cases}r & \\text { if } n \\text { is even, } \\\\ s & \\text { if } n \\text { is odd }\\end{cases} $$ also has the property under discussion.","tier":0} +{"problem_type":"Algebra","problem_label":"A5","problem":"Consider all polynomials $P(x)$ with real coefficients that have the following property: for any two real numbers $x$ and $y$ one has $$ \\left|y^{2}-P(x)\\right| \\leqslant 2|x| \\quad \\text { if and only if } \\quad\\left|x^{2}-P(y)\\right| \\leqslant 2|y| $$ Determine all possible values of $P(0)$.","solution":"Part I. We begin by verifying that these numbers are indeed possible values of $P(0)$. To see that each negative real number $-C$ can be $P(0)$, it suffices to check that for every $C>0$ the polynomial $P(x)=-\\left(\\frac{2 x^{2}}{C}+C\\right)$ has the property described in the statement of the problem. Due to symmetry it is enough for this purpose to prove $\\left|y^{2}-P(x)\\right|>2|x|$ for any two real numbers $x$ and $y$. In fact we have $$ \\left|y^{2}-P(x)\\right|=y^{2}+\\frac{x^{2}}{C}+\\frac{(|x|-C)^{2}}{C}+2|x| \\geqslant \\frac{x^{2}}{C}+2|x| \\geqslant 2|x|, $$ where in the first estimate equality can only hold if $|x|=C$, whilst in the second one it can only hold if $x=0$. As these two conditions cannot be met at the same time, we have indeed $\\left|y^{2}-P(x)\\right|>2|x|$. To show that $P(0)=1$ is possible as well, we verify that the polynomial $P(x)=x^{2}+1$ satisfies (1). Notice that for all real numbers $x$ and $y$ we have $$ \\begin{aligned} \\left|y^{2}-P(x)\\right| \\leqslant 2|x| & \\Longleftrightarrow\\left(y^{2}-x^{2}-1\\right)^{2} \\leqslant 4 x^{2} \\\\ & \\Longleftrightarrow 0 \\leqslant\\left(\\left(y^{2}-(x-1)^{2}\\right)\\left((x+1)^{2}-y^{2}\\right)\\right. \\\\ & \\Longleftrightarrow 0 \\leqslant(y-x+1)(y+x-1)(x+1-y)(x+1+y) \\\\ & \\Longleftrightarrow 0 \\leqslant\\left((x+y)^{2}-1\\right)\\left(1-(x-y)^{2}\\right) . \\end{aligned} $$ Since this inequality is symmetric in $x$ and $y$, we are done. Part II. Now we show that no values other than those mentioned in the answer are possible for $P(0)$. To reach this we let $P$ denote any polynomial satisfying (1) and $P(0) \\geqslant 0$; as we shall see, this implies $P(x)=x^{2}+1$ for all real $x$, which is actually more than what we want. First step: We prove that $P$ is even. By (1) we have $$ \\left|y^{2}-P(x)\\right| \\leqslant 2|x| \\Longleftrightarrow\\left|x^{2}-P(y)\\right| \\leqslant 2|y| \\Longleftrightarrow\\left|y^{2}-P(-x)\\right| \\leqslant 2|x| $$ for all real numbers $x$ and $y$. Considering just the equivalence of the first and third statement and taking into account that $y^{2}$ may vary through $\\mathbb{R}_{\\geqslant 0}$ we infer that $$ [P(x)-2|x|, P(x)+2|x|] \\cap \\mathbb{R}_{\\geqslant 0}=[P(-x)-2|x|, P(-x)+2|x|] \\cap \\mathbb{R}_{\\geqslant 0} $$ holds for all $x \\in \\mathbb{R}$. We claim that there are infinitely many real numbers $x$ such that $P(x)+2|x| \\geqslant 0$. This holds in fact for any real polynomial with $P(0) \\geqslant 0$; in order to see this, we may assume that the coefficient of $P$ appearing in front of $x$ is nonnegative. In this case the desired inequality holds for all sufficiently small positive real numbers. For such numbers $x$ satisfying $P(x)+2|x| \\geqslant 0$ we have $P(x)+2|x|=P(-x)+2|x|$ by the previous displayed formula, and hence also $P(x)=P(-x)$. Consequently the polynomial $P(x)-P(-x)$ has infinitely many zeros, wherefore it has to vanish identically. Thus $P$ is indeed even. Second step: We prove that $P(t)>0$ for all $t \\in \\mathbb{R}$. Let us assume for a moment that there exists a real number $t \\neq 0$ with $P(t)=0$. Then there is some open interval $I$ around $t$ such that $|P(y)| \\leqslant 2|y|$ holds for all $y \\in I$. Plugging $x=0$ into (1) we learn that $y^{2}=P(0)$ holds for all $y \\in I$, which is clearly absurd. We have thus shown $P(t) \\neq 0$ for all $t \\neq 0$. In combination with $P(0) \\geqslant 0$ this informs us that our claim could only fail if $P(0)=0$. In this case there is by our first step a polynomial $Q(x)$ such that $P(x)=x^{2} Q(x)$. Applying (1) to $x=0$ and an arbitrary $y \\neq 0$ we get $|y Q(y)|>2$, which is surely false when $y$ is sufficiently small. Third step: We prove that $P$ is a quadratic polynomial. Notice that $P$ cannot be constant, for otherwise if $x=\\sqrt{P(0)}$ and $y$ is sufficiently large, the first part of (1) is false whilst the second part is true. So the degree $n$ of $P$ has to be at least 1 . By our first step $n$ has to be even as well, whence in particular $n \\geqslant 2$. Now assume that $n \\geqslant 4$. Plugging $y=\\sqrt{P(x)}$ into (1) we get $\\left|x^{2}-P(\\sqrt{P(x)})\\right| \\leqslant 2 \\sqrt{P(x)}$ and hence $$ P(\\sqrt{P(x)}) \\leqslant x^{2}+2 \\sqrt{P(x)} $$ for all real $x$. Choose positive real numbers $x_{0}, a$, and $b$ such that if $x \\in\\left(x_{0}, \\infty\\right)$, then $a x^{n}<$ $P(x)0$ denotes the leading coefficient of $P$, then $\\lim _{x \\rightarrow \\infty} \\frac{P(x)}{x^{n}}=d$, whence for instance the numbers $a=\\frac{d}{2}$ and $b=2 d$ work provided that $x_{0}$ is chosen large enough. Now for all sufficiently large real numbers $x$ we have $$ a^{n \/ 2+1} x^{n^{2} \/ 2}0$ and $b$ such that $P(x)=$ $a x^{2}+b$. Now if $x$ is large enough and $y=\\sqrt{a} x$, the left part of (1) holds and the right part reads $\\left|\\left(1-a^{2}\\right) x^{2}-b\\right| \\leqslant 2 \\sqrt{a} x$. In view of the fact that $a>0$ this is only possible if $a=1$. Finally, substituting $y=x+1$ with $x>0$ into (1) we get $$ |2 x+1-b| \\leqslant 2 x \\Longleftrightarrow|2 x+1+b| \\leqslant 2 x+2 $$ i.e., $$ b \\in[1,4 x+1] \\Longleftrightarrow b \\in[-4 x-3,1] $$ for all $x>0$. Choosing $x$ large enough, we can achieve that at least one of these two statements holds; then both hold, which is only possible if $b=1$, as desired. Comment 1. There are some issues with this problem in that its most natural solutions seem to use some basic facts from analysis, such as the continuity of polynomials or the intermediate value theorem. Yet these facts are intuitively obvious and implicitly clear to the students competing at this level of difficulty, so that the Problem Selection Committee still thinks that the problem is suitable for the IMO. Comment 2. It seems that most solutions will in the main case, where $P(0)$ is nonnegative, contain an argument that is somewhat asymptotic in nature showing that $P$ is quadratic, and some part narrowing that case down to $P(x)=x^{2}+1$. Comment 3. It is also possible to skip the first step and start with the second step directly, but then one has to work a bit harder to rule out the case $P(0)=0$. Let us sketch one possibility of doing this: Take the auxiliary polynomial $Q(x)$ such that $P(x)=x Q(x)$. Applying (1) to $x=0$ and an arbitrary $y \\neq 0$ we get $|Q(y)|>2$. Hence we either have $Q(z) \\geqslant 2$ for all real $z$ or $Q(z) \\leqslant-2$ for all real $z$. In particular there is some $\\eta \\in\\{-1,+1\\}$ such that $P(\\eta) \\geqslant 2$ and $P(-\\eta) \\leqslant-2$. Substituting $x= \\pm \\eta$ into (1) we learn $$ \\left|y^{2}-P(\\eta)\\right| \\leqslant 2 \\Longleftrightarrow|1-P(y)| \\leqslant 2|y| \\Longleftrightarrow\\left|y^{2}-P(-\\eta)\\right| \\leqslant 2 $$ But for $y=\\sqrt{P(\\eta)}$ the first statement is true, whilst the third one is false. Also, if one has not obtained the evenness of $P$ before embarking on the fourth step, one needs to work a bit harder there, but not in a way that is likely to cause major difficulties. Comment 4. Truly curious people may wonder about the set of all polynomials having property (1). As explained in the solution above, $P(x)=x^{2}+1$ is the only one with $P(0)=1$. On the other hand, it is not hard to notice that for negative $P(0)$ there are more possibilities than those mentioned above. E.g., as remarked by the proposer, if $a$ and $b$ denote two positive real numbers with $a b>1$ and $Q$ denotes a polynomial attaining nonnegative values only, then $P(x)=-\\left(a x^{2}+b+Q(x)\\right)$ works. More generally, it may be proved that if $P(x)$ satisfies (1) and $P(0)<0$, then $-P(x)>2|x|$ holds for all $x \\in \\mathbb{R}$ so that one just considers the equivalence of two false statements. One may generate all such polynomials $P$ by going through all combinations of a solution of the polynomial equation $$ x=A(x) B(x)+C(x) D(x) $$ and a real $E>0$, and setting $$ P(x)=-\\left(A(x)^{2}+B(x)^{2}+C(x)^{2}+D(x)^{2}+E\\right) $$ for each of them.","tier":0} +{"problem_type":"Algebra","problem_label":"A6","problem":"Find all functions $f: \\mathbb{Z} \\rightarrow \\mathbb{Z}$ such that $$ n^{2}+4 f(n)=f(f(n))^{2} $$ for all $n \\in \\mathbb{Z}$.","solution":"Part I. Let us first check that each of the functions above really satisfies the given functional equation. If $f(n)=n+1$ for all $n$, then we have $$ n^{2}+4 f(n)=n^{2}+4 n+4=(n+2)^{2}=f(n+1)^{2}=f(f(n))^{2} . $$ If $f(n)=n+1$ for $n>-a$ and $f(n)=-n+1$ otherwise, then we have the same identity for $n>-a$ and $$ n^{2}+4 f(n)=n^{2}-4 n+4=(2-n)^{2}=f(1-n)^{2}=f(f(n))^{2} $$ otherwise. The same applies to the third solution (with $a=0$ ), where in addition one has $$ 0^{2}+4 f(0)=0=f(f(0))^{2} $$ Part II. It remains to prove that these are really the only functions that satisfy our functional equation. We do so in three steps: Step 1: We prove that $f(n)=n+1$ for $n>0$. Consider the sequence $\\left(a_{k}\\right)$ given by $a_{k}=f^{k}(1)$ for $k \\geqslant 0$. Setting $n=a_{k}$ in (1), we get $$ a_{k}^{2}+4 a_{k+1}=a_{k+2}^{2} $$ Of course, $a_{0}=1$ by definition. Since $a_{2}^{2}=1+4 a_{1}$ is odd, $a_{2}$ has to be odd as well, so we set $a_{2}=2 r+1$ for some $r \\in \\mathbb{Z}$. Then $a_{1}=r^{2}+r$ and consequently $$ a_{3}^{2}=a_{1}^{2}+4 a_{2}=\\left(r^{2}+r\\right)^{2}+8 r+4 $$ Since $8 r+4 \\neq 0, a_{3}^{2} \\neq\\left(r^{2}+r\\right)^{2}$, so the difference between $a_{3}^{2}$ and $\\left(r^{2}+r\\right)^{2}$ is at least the distance from $\\left(r^{2}+r\\right)^{2}$ to the nearest even square (since $8 r+4$ and $r^{2}+r$ are both even). This implies that $$ |8 r+4|=\\left|a_{3}^{2}-\\left(r^{2}+r\\right)^{2}\\right| \\geqslant\\left(r^{2}+r\\right)^{2}-\\left(r^{2}+r-2\\right)^{2}=4\\left(r^{2}+r-1\\right) $$ (for $r=0$ and $r=-1$, the estimate is trivial, but this does not matter). Therefore, we ave $$ 4 r^{2} \\leqslant|8 r+4|-4 r+4 $$ If $|r| \\geqslant 4$, then $$ 4 r^{2} \\geqslant 16|r| \\geqslant 12|r|+16>8|r|+4+4|r|+4 \\geqslant|8 r+4|-4 r+4 $$ a contradiction. Thus $|r|<4$. Checking all possible remaining values of $r$, we find that $\\left(r^{2}+r\\right)^{2}+8 r+4$ is only a square in three cases: $r=-3, r=0$ and $r=1$. Let us now distinguish these three cases: - $r=-3$, thus $a_{1}=6$ and $a_{2}=-5$. For each $k \\geqslant 1$, we have $$ a_{k+2}= \\pm \\sqrt{a_{k}^{2}+4 a_{k+1}} $$ and the sign needs to be chosen in such a way that $a_{k+1}^{2}+4 a_{k+2}$ is again a square. This yields $a_{3}=-4, a_{4}=-3, a_{5}=-2, a_{6}=-1, a_{7}=0, a_{8}=1, a_{9}=2$. At this point we have reached a contradiction, since $f(1)=f\\left(a_{0}\\right)=a_{1}=6$ and at the same time $f(1)=f\\left(a_{8}\\right)=a_{9}=2$. - $r=0$, thus $a_{1}=0$ and $a_{2}=1$. Then $a_{3}^{2}=a_{1}^{2}+4 a_{2}=4$, so $a_{3}= \\pm 2$. This, however, is a contradiction again, since it gives us $f(1)=f\\left(a_{0}\\right)=a_{1}=0$ and at the same time $f(1)=f\\left(a_{2}\\right)=a_{3}= \\pm 2$. - $r=1$, thus $a_{1}=2$ and $a_{2}=3$. We prove by induction that $a_{k}=k+1$ for all $k \\geqslant 0$ in this case, which we already know for $k \\leqslant 2$ now. For the induction step, assume that $a_{k-1}=k$ and $a_{k}=k+1$. Then $$ a_{k+1}^{2}=a_{k-1}^{2}+4 a_{k}=k^{2}+4 k+4=(k+2)^{2} $$ so $a_{k+1}= \\pm(k+2)$. If $a_{k+1}=-(k+2)$, then $$ a_{k+2}^{2}=a_{k}^{2}+4 a_{k+1}=(k+1)^{2}-4 k-8=k^{2}-2 k-7=(k-1)^{2}-8 $$ The latter can only be a square if $k=4$ (since 1 and 9 are the only two squares whose difference is 8 ). Then, however, $a_{4}=5, a_{5}=-6$ and $a_{6}= \\pm 1$, so $$ a_{7}^{2}=a_{5}^{2}+4 a_{6}=36 \\pm 4 $$ but neither 32 nor 40 is a perfect square. Thus $a_{k+1}=k+2$, which completes our induction. This also means that $f(n)=f\\left(a_{n-1}\\right)=a_{n}=n+1$ for all $n \\geqslant 1$. Step 2: We prove that either $f(0)=1$, or $f(0)=0$ and $f(n) \\neq 0$ for $n \\neq 0$. Set $n=0$ in (1) to get $$ 4 f(0)=f(f(0))^{2} $$ This means that $f(0) \\geqslant 0$. If $f(0)=0$, then $f(n) \\neq 0$ for all $n \\neq 0$, since we would otherwise have $$ n^{2}=n^{2}+4 f(n)=f(f(n))^{2}=f(0)^{2}=0 $$ If $f(0)>0$, then we know that $f(f(0))=f(0)+1$ from the first step, so $$ 4 f(0)=(f(0)+1)^{2} $$ which yields $f(0)=1$. Step 3: We discuss the values of $f(n)$ for $n<0$. Lemma. For every $n \\geqslant 1$, we have $f(-n)=-n+1$ or $f(-n)=n+1$. Moreover, if $f(-n)=$ $-n+1$ for some $n \\geqslant 1$, then also $f(-n+1)=-n+2$. Proof. We prove this statement by strong induction on $n$. For $n=1$, we get $$ 1+4 f(-1)=f(f(-1))^{2} $$ Thus $f(-1)$ needs to be nonnegative. If $f(-1)=0$, then $f(f(-1))=f(0)= \\pm 1$, so $f(0)=1$ (by our second step). Otherwise, we know that $f(f(-1))=f(-1)+1$, so $$ 1+4 f(-1)=(f(-1)+1)^{2} $$ which yields $f(-1)=2$ and thus establishes the base case. For the induction step, we consider two cases: - If $f(-n) \\leqslant-n$, then $$ f(f(-n))^{2}=(-n)^{2}+4 f(-n) \\leqslant n^{2}-4 n<(n-2)^{2} $$ so $|f(f(-n))| \\leqslant n-3$ (for $n=2$, this case cannot even occur). If $f(f(-n)) \\geqslant 0$, then we already know from the first two steps that $f(f(f(-n)))=f(f(-n))+1$, unless perhaps if $f(0)=0$ and $f(f(-n))=0$. However, the latter would imply $f(-n)=0$ (as shown in Step 2) and thus $n=0$, which is impossible. If $f(f(-n))<0$, we can apply the induction hypothesis to $f(f(-n))$. In either case, $f(f(f(-n)))= \\pm f(f(-n))+1$. Therefore, $$ f(-n)^{2}+4 f(f(-n))=f(f(f(-n)))^{2}=( \\pm f(f(-n))+1)^{2} $$ which gives us $$ \\begin{aligned} n^{2} & \\leqslant f(-n)^{2}=( \\pm f(f(-n))+1)^{2}-4 f(f(-n)) \\leqslant f(f(-n))^{2}+6|f(f(-n))|+1 \\\\ & \\leqslant(n-3)^{2}+6(n-3)+1=n^{2}-8 \\end{aligned} $$ a contradiction. - Thus, we are left with the case that $f(-n)>-n$. Now we argue as in the previous case: if $f(-n) \\geqslant 0$, then $f(f(-n))=f(-n)+1$ by the first two steps, since $f(0)=0$ and $f(-n)=0$ would imply $n=0$ (as seen in Step 2) and is thus impossible. If $f(-n)<0$, we can apply the induction hypothesis, so in any case we can infer that $f(f(-n))= \\pm f(-n)+1$. We obtain $$ (-n)^{2}+4 f(-n)=( \\pm f(-n)+1)^{2} $$ so either $$ n^{2}=f(-n)^{2}-2 f(-n)+1=(f(-n)-1)^{2} $$ which gives us $f(-n)= \\pm n+1$, or $$ n^{2}=f(-n)^{2}-6 f(-n)+1=(f(-n)-3)^{2}-8 $$ Since 1 and 9 are the only perfect squares whose difference is 8 , we must have $n=1$, which we have already considered. Finally, suppose that $f(-n)=-n+1$ for some $n \\geqslant 2$. Then $$ f(-n+1)^{2}=f(f(-n))^{2}=(-n)^{2}+4 f(-n)=(n-2)^{2} $$ so $f(-n+1)= \\pm(n-2)$. However, we already know that $f(-n+1)=-n+2$ or $f(-n+1)=n$, so $f(-n+1)=-n+2$. Combining everything we know, we find the solutions as stated in the answer: - One solution is given by $f(n)=n+1$ for all $n$. - If $f(n)$ is not always equal to $n+1$, then there is a largest integer $m$ (which cannot be positive) for which this is not the case. In view of the lemma that we proved, we must then have $f(n)=-n+1$ for any integer $n(|b|-4)^{2} $$ because $|b| \\geqslant|a|-1 \\geqslant 9$. Thus (3) can be refined to $$ |a|+3 \\geqslant|f(a)| \\geqslant|a|-1 \\quad \\text { for }|a| \\geqslant E $$ Now, from $c^{2}=a^{2}+4 b$ with $|b| \\in[|a|-1,|a|+3]$ we get $c^{2}=(a \\pm 2)^{2}+d$, where $d \\in\\{-16,-12,-8,-4,0,4,8\\}$. Since $|a \\pm 2| \\geqslant 8$, this can happen only if $c^{2}=(a \\pm 2)^{2}$, which in turn yields $b= \\pm a+1$. To summarise, $$ f(a)=1 \\pm a \\quad \\text { for }|a| \\geqslant E \\text {. } $$ We have shown that, with at most finitely many exceptions, $f(a)=1 \\pm a$. Thus it will be convenient for our second step to introduce the sets $$ Z_{+}=\\{a \\in \\mathbb{Z}: f(a)=a+1\\}, \\quad Z_{-}=\\{a \\in \\mathbb{Z}: f(a)=1-a\\}, \\quad \\text { and } \\quad Z_{0}=\\mathbb{Z} \\backslash\\left(Z_{+} \\cup Z_{-}\\right) $$ Step 2. Now we investigate the structure of the sets $Z_{+}, Z_{-}$, and $Z_{0}$. 4. Note that $f(E+1)=1 \\pm(E+1)$. If $f(E+1)=E+2$, then $E+1 \\in Z_{+}$. Otherwise we have $f(1+E)=-E$; then the original equation (1) with $n=E+1$ gives us $(E-1)^{2}=f(-E)^{2}$, so $f(-E)= \\pm(E-1)$. By (4) this may happen only if $f(-E)=1-E$, so in this case $-E \\in Z_{+}$. In any case we find that $Z_{+} \\neq \\varnothing$. 5. Now take any $a \\in Z_{+}$. We claim that every integer $x \\geqslant a$ also lies in $Z_{+}$. We proceed by induction on $x$, the base case $x=a$ being covered by our assumption. For the induction step, assume that $f(x-1)=x$ and plug $n=x-1$ into (1). We get $f(x)^{2}=(x+1)^{2}$, so either $f(x)=x+1$ or $f(x)=-(x+1)$. Assume that $f(x)=-(x+1)$ and $x \\neq-1$, since otherwise we already have $f(x)=x+1$. Plugging $n=x$ into (1), we obtain $f(-x-1)^{2}=(x-2)^{2}-8$, which may happen only if $x-2= \\pm 3$ and $f(-x-1)= \\pm 1$. Plugging $n=-x-1$ into (1), we get $f( \\pm 1)^{2}=(x+1)^{2} \\pm 4$, which in turn may happen only if $x+1 \\in\\{-2,0,2\\}$. Thus $x \\in\\{-1,5\\}$ and at the same time $x \\in\\{-3,-1,1\\}$, which gives us $x=-1$. Since this has already been excluded, we must have $f(x)=x+1$, which completes our induction. 6. Now we know that either $Z_{+}=\\mathbb{Z}$ (if $Z_{+}$is not bounded below), or $Z_{+}=\\left\\{a \\in \\mathbb{Z}: a \\geqslant a_{0}\\right\\}$, where $a_{0}$ is the smallest element of $Z_{+}$. In the former case, $f(n)=n+1$ for all $n \\in \\mathbb{Z}$, which is our first solution. So we assume in the following that $Z_{+}$is bounded below and has a smallest element $a_{0}$. If $Z_{0}=\\varnothing$, then we have $f(x)=x+1$ for $x \\geqslant a_{0}$ and $f(x)=1-x$ for $x1$. If none of the line segments that form the borders between the rectangles is horizontal, then we have $k-1$ vertical segments dividing $R$ into $k$ rectangles. On each of them, there can only be one of the $n$ points, so $n \\leqslant k-1$, which is exactly what we want to prove. Otherwise, consider the lowest horizontal line $h$ that contains one or more of these line segments. Let $R^{\\prime}$ be the rectangle that results when everything that lies below $h$ is removed from $R$ (see the example in the figure below). The rectangles that lie entirely below $h$ form blocks of rectangles separated by vertical line segments. Suppose there are $r$ blocks and $k_{i}$ rectangles in the $i^{\\text {th }}$ block. The left and right border of each block has to extend further upwards beyond $h$. Thus we can move any points that lie on these borders upwards, so that they now lie inside $R^{\\prime}$. This can be done without violating the conditions, one only needs to make sure that they do not get to lie on a common horizontal line with one of the other given points. All other borders between rectangles in the $i^{\\text {th }}$ block have to lie entirely below $h$. There are $k_{i}-1$ such line segments, each of which can contain at most one of the given points. Finally, there can be one point that lies on $h$. All other points have to lie in $R^{\\prime}$ (after moving some of them as explained in the previous paragraph). ![](https:\/\/cdn.mathpix.com\/cropped\/2024_11_18_62c85fb0b01b41aca29bg-28.jpg?height=320&width=512&top_left_y=1322&top_left_x=772) Figure 2: Illustration of the inductive argument We see that $R^{\\prime}$ is divided into $k-\\sum_{i=1}^{r} k_{i}$ rectangles. Applying the induction hypothesis to $R^{\\prime}$, we find that there are at most $$ \\left(k-\\sum_{i=1}^{r} k_{i}\\right)-1+\\sum_{i=1}^{r}\\left(k_{i}-1\\right)+1=k-r $$ points. Since $r \\geqslant 1$, this means that $n \\leqslant k-1$, which completes our induction.","tier":0} +{"problem_type":"Combinatorics","problem_label":"C2","problem":"We have $2^{m}$ sheets of paper, with the number 1 written on each of them. We perform the following operation. In every step we choose two distinct sheets; if the numbers on the two sheets are $a$ and $b$, then we erase these numbers and write the number $a+b$ on both sheets. Prove that after $m 2^{m-1}$ steps, the sum of the numbers on all the sheets is at least $4^{m}$. (Iran)","solution":"Let $P_{k}$ be the product of the numbers on the sheets after $k$ steps. Suppose that in the $(k+1)^{\\text {th }}$ step the numbers $a$ and $b$ are replaced by $a+b$. In the product, the number $a b$ is replaced by $(a+b)^{2}$, and the other factors do not change. Since $(a+b)^{2} \\geqslant 4 a b$, we see that $P_{k+1} \\geqslant 4 P_{k}$. Starting with $P_{0}=1$, a straightforward induction yields $$ P_{k} \\geqslant 4^{k} $$ for all integers $k \\geqslant 0$; in particular $$ P_{m \\cdot 2^{m-1}} \\geqslant 4^{m \\cdot 2^{m-1}}=\\left(2^{m}\\right)^{2^{m}} $$ so by the AM-GM inequality, the sum of the numbers written on the sheets after $m 2^{m-1}$ steps is at least $$ 2^{m} \\cdot \\sqrt[2^{m}]{P_{m \\cdot 2^{m-1}}} \\geqslant 2^{m} \\cdot 2^{m}=4^{m} $$ Comment 1. It is possible to achieve the sum $4^{m}$ in $m 2^{m-1}$ steps. For example, starting from $2^{m}$ equal numbers on the sheets, in $2^{m-1}$ consecutive steps we can double all numbers. After $m$ such doubling rounds we have the number $2^{m}$ on every sheet. Comment 2. There are several versions of the solution above. E.g., one may try to assign to each positive integer $n$ a weight $w_{n}$ in such a way that the sum of the weights of the numbers written on the sheets increases, say, by at least 2 in each step. For this purpose, one needs the inequality $$ 2 w_{a+b} \\geqslant w_{a}+w_{b}+2 $$ to be satisfied for all positive integers $a$ and $b$. Starting from $w_{1}=1$ and trying to choose the weights as small as possible, one may find that these weights can be defined as follows: For every positive integer $n$, one chooses $k$ to be the maximal integer such that $n \\geqslant 2^{k}$, and puts $$ w_{n}=k+\\frac{n}{2^{k}}=\\min _{d \\in \\mathbb{Z} \\geq 0}\\left(d+\\frac{n}{2^{d}}\\right) . $$ Now, in order to prove that these weights satisfy (1), one may take arbitrary positive integers $a$ and $b$, and choose an integer $d \\geqslant 0$ such that $w_{a+b}=d+\\frac{a+b}{2^{d}}$. Then one has $$ 2 w_{a+b}=2 d+2 \\cdot \\frac{a+b}{2^{d}}=\\left((d-1)+\\frac{a}{2^{d-1}}\\right)+\\left((d-1)+\\frac{b}{2^{d-1}}\\right)+2 \\geqslant w_{a}+w_{b}+2 $$ Since the initial sum of the weights was $2^{m}$, after $m 2^{m-1}$ steps the sum is at least $(m+1) 2^{m}$. To finish the solution, one may notice that by (2) for every positive integer $a$ one has $$ w_{a} \\leqslant m+\\frac{a}{2^{m}}, \\quad \\text { i.e., } \\quad a \\geqslant 2^{m}\\left(-m+w_{a}\\right) $$ So the sum of the numbers $a_{1}, a_{2}, \\ldots, a_{2^{m}}$ on the sheets can be estimated as $$ \\sum_{i=1}^{2^{m}} a_{i} \\geqslant \\sum_{i=1}^{2^{m}} 2^{m}\\left(-m+w_{a_{i}}\\right)=-m 2^{m} \\cdot 2^{m}+2^{m} \\sum_{i=1}^{2^{m}} w_{a_{i}} \\geqslant-m 4^{m}+(m+1) 4^{m}=4^{m} $$ as required. For establishing the inequalities (1) and (3), one may also use the convexity argument, instead of the second definition of $w_{n}$ in (2). One may check that $\\log _{2} n \\leqslant w_{n} \\leqslant \\log _{2} n+1$; thus, in some rough sense, this approach is obtained by \"taking the logarithm\" of the solution above. Comment 3. An intuitive strategy to minimise the sum of numbers is that in every step we choose the two smallest numbers. We may call this the greedy strategy. In the following paragraphs we prove that the greedy strategy indeed provides the least possible sum of numbers. Claim. Starting from any sequence $x_{1}, \\ldots, x_{N}$ of positive real numbers on $N$ sheets, for any number $k$ of steps, the greedy strategy achieves the lowest possible sum of numbers. Proof. We apply induction on $k$; for $k=1$ the statement is obvious. Let $k \\geqslant 2$, and assume that the claim is true for smaller values. Every sequence of $k$ steps can be encoded as $S=\\left(\\left(i_{1}, j_{1}\\right), \\ldots,\\left(i_{k}, j_{k}\\right)\\right)$, where, for $r=1,2, \\ldots, k$, the numbers $i_{r}$ and $j_{r}$ are the indices of the two sheets that are chosen in the $r^{\\text {th }}$ step. The resulting final sum will be some linear combination of $x_{1}, \\ldots, x_{N}$, say, $c_{1} x_{1}+\\cdots+c_{N} x_{N}$ with positive integers $c_{1}, \\ldots, c_{N}$ that depend on $S$ only. Call the numbers $\\left(c_{1}, \\ldots, c_{N}\\right)$ the characteristic vector of $S$. Choose a sequence $S_{0}=\\left(\\left(i_{1}, j_{1}\\right), \\ldots,\\left(i_{k}, j_{k}\\right)\\right)$ of steps that produces the minimal sum, starting from $x_{1}, \\ldots, x_{N}$, and let $\\left(c_{1}, \\ldots, c_{N}\\right)$ be the characteristic vector of $S$. We may assume that the sheets are indexed in such an order that $c_{1} \\geqslant c_{2} \\geqslant \\cdots \\geqslant c_{N}$. If the sheets (and the numbers) are permuted by a permutation $\\pi$ of the indices $(1,2, \\ldots, N)$ and then the same steps are performed, we can obtain the $\\operatorname{sum} \\sum_{t=1}^{N} c_{t} x_{\\pi(t)}$. By the rearrangement inequality, the smallest possible sum can be achieved when the numbers $\\left(x_{1}, \\ldots, x_{N}\\right)$ are in non-decreasing order. So we can assume that also $x_{1} \\leqslant x_{2} \\leqslant \\cdots \\leqslant x_{N}$. Let $\\ell$ be the largest index with $c_{1}=\\cdots=c_{\\ell}$, and let the $r^{\\text {th }}$ step be the first step for which $c_{i_{r}}=c_{1}$ or $c_{j_{r}}=c_{1}$. The role of $i_{r}$ and $j_{r}$ is symmetrical, so we can assume $c_{i_{r}}=c_{1}$ and thus $i_{r} \\leqslant \\ell$. We show that $c_{j_{r}}=c_{1}$ and $j_{r} \\leqslant \\ell$ hold, too. Before the $r^{\\text {th }}$ step, on the $i_{r}{ }^{\\text {th }}$ sheet we had the number $x_{i_{r}}$. On the $j_{r}{ }^{\\text {th }}$ sheet there was a linear combination that contains the number $x_{j_{r}}$ with a positive integer coefficient, and possibly some other terms. In the $r^{\\text {th }}$ step, the number $x_{i_{r}}$ joins that linear combination. From this point, each sheet contains a linear combination of $x_{1}, \\ldots, x_{N}$, with the coefficient of $x_{j_{r}}$ being not smaller than the coefficient of $x_{i_{r}}$. This is preserved to the end of the procedure, so we have $c_{j_{r}} \\geqslant c_{i_{r}}$. But $c_{i_{r}}=c_{1}$ is maximal among the coefficients, so we have $c_{j_{r}}=c_{i_{r}}=c_{1}$ and thus $j_{r} \\leqslant \\ell$. Either from $c_{j_{r}}=c_{i_{r}}=c_{1}$ or from the arguments in the previous paragraph we can see that none of the $i_{r}{ }^{\\text {th }}$ and the $j_{r}{ }^{\\text {th }}$ sheets were used before step $r$. Therefore, the final linear combination of the numbers does not change if the step $\\left(i_{r}, j_{r}\\right)$ is performed first: the sequence of steps $$ S_{1}=\\left(\\left(i_{r}, j_{r}\\right),\\left(i_{1}, j_{1}\\right), \\ldots,\\left(i_{r-1}, j_{r-1}\\right),\\left(i_{r+1}, j_{r+1}\\right), \\ldots,\\left(i_{N}, j_{N}\\right)\\right) $$ also produces the same minimal sum at the end. Therefore, we can replace $S_{0}$ by $S_{1}$ and we may assume that $r=1$ and $c_{i_{1}}=c_{j_{1}}=c_{1}$. As $i_{1} \\neq j_{1}$, we can see that $\\ell \\geqslant 2$ and $c_{1}=c_{2}=c_{i_{1}}=c_{j_{1}}$. Let $\\pi$ be such a permutation of the indices $(1,2, \\ldots, N)$ that exchanges 1,2 with $i_{r}, j_{r}$ and does not change the remaining indices. Let $$ S_{2}=\\left(\\left(\\pi\\left(i_{1}\\right), \\pi\\left(j_{1}\\right)\\right), \\ldots,\\left(\\pi\\left(i_{N}\\right), \\pi\\left(j_{N}\\right)\\right)\\right) $$ Since $c_{\\pi(i)}=c_{i}$ for all indices $i$, this sequence of steps produces the same, minimal sum. Moreover, in the first step we chose $x_{\\pi\\left(i_{1}\\right)}=x_{1}$ and $x_{\\pi\\left(j_{1}\\right)}=x_{2}$, the two smallest numbers. Hence, it is possible to achieve the optimal sum if we follow the greedy strategy in the first step. By the induction hypothesis, following the greedy strategy in the remaining steps we achieve the optimal sum.","tier":0} +{"problem_type":"Combinatorics","problem_label":"C3","problem":"Let $n \\geqslant 2$ be an integer. Consider an $n \\times n$ chessboard divided into $n^{2}$ unit squares. We call a configuration of $n$ rooks on this board happy if every row and every column contains exactly one rook. Find the greatest positive integer $k$ such that for every happy configuration of rooks, we can find a $k \\times k$ square without a rook on any of its $k^{2}$ unit squares. (Croatia)","solution":"Let $\\ell$ be a positive integer. We will show that (i) if $n>\\ell^{2}$ then each happy configuration contains an empty $\\ell \\times \\ell$ square, but (ii) if $n \\leqslant \\ell^{2}$ then there exists a happy configuration not containing such a square. These two statements together yield the answer. (i). Assume that $n>\\ell^{2}$. Consider any happy configuration. There exists a row $R$ containing a rook in its leftmost square. Take $\\ell$ consecutive rows with $R$ being one of them. Their union $U$ contains exactly $\\ell$ rooks. Now remove the $n-\\ell^{2} \\geqslant 1$ leftmost columns from $U$ (thus at least one rook is also removed). The remaining part is an $\\ell^{2} \\times \\ell$ rectangle, so it can be split into $\\ell$ squares of size $\\ell \\times \\ell$, and this part contains at most $\\ell-1$ rooks. Thus one of these squares is empty. (ii). Now we assume that $n \\leqslant \\ell^{2}$. Firstly, we will construct a happy configuration with no empty $\\ell \\times \\ell$ square for the case $n=\\ell^{2}$. After that we will modify it to work for smaller values of $n$. Let us enumerate the rows from bottom to top as well as the columns from left to right by the numbers $0,1, \\ldots, \\ell^{2}-1$. Every square will be denoted, as usual, by the pair $(r, c)$ of its row and column numbers. Now we put the rooks on all squares of the form $(i \\ell+j, j \\ell+i)$ with $i, j=0,1, \\ldots, \\ell-1$ (the picture below represents this arrangement for $\\ell=3$ ). Since each number from 0 to $\\ell^{2}-1$ has a unique representation of the form $i \\ell+j(0 \\leqslant i, j \\leqslant \\ell-1)$, each row and each column contains exactly one rook. ![](https:\/\/cdn.mathpix.com\/cropped\/2024_11_18_62c85fb0b01b41aca29bg-31.jpg?height=524&width=527&top_left_y=1554&top_left_x=773) Next, we show that each $\\ell \\times \\ell$ square $A$ on the board contains a rook. Consider such a square $A$, and consider $\\ell$ consecutive rows the union of which contains $A$. Let the lowest of these rows have number $p \\ell+q$ with $0 \\leqslant p, q \\leqslant \\ell-1$ (notice that $p \\ell+q \\leqslant \\ell^{2}-\\ell$ ). Then the rooks in this union are placed in the columns with numbers $q \\ell+p,(q+1) \\ell+p, \\ldots,(\\ell-1) \\ell+p$, $p+1, \\ell+(p+1), \\ldots,(q-1) \\ell+p+1$, or, putting these numbers in increasing order, $$ p+1, \\ell+(p+1), \\ldots,(q-1) \\ell+(p+1), q \\ell+p,(q+1) \\ell+p, \\ldots,(\\ell-1) \\ell+p $$ One readily checks that the first number in this list is at most $\\ell-1$ (if $p=\\ell-1$, then $q=0$, and the first listed number is $q \\ell+p=\\ell-1$ ), the last one is at least $(\\ell-1) \\ell$, and the difference between any two consecutive numbers is at most $\\ell$. Thus, one of the $\\ell$ consecutive columns intersecting $A$ contains a number listed above, and the rook in this column is inside $A$, as required. The construction for $n=\\ell^{2}$ is established. It remains to construct a happy configuration of rooks not containing an empty $\\ell \\times \\ell$ square for $n<\\ell^{2}$. In order to achieve this, take the construction for an $\\ell^{2} \\times \\ell^{2}$ square described above and remove the $\\ell^{2}-n$ bottom rows together with the $\\ell^{2}-n$ rightmost columns. We will have a rook arrangement with no empty $\\ell \\times \\ell$ square, but several rows and columns may happen to be empty. Clearly, the number of empty rows is equal to the number of empty columns, so one can find a bijection between them, and put a rook on any crossing of an empty row and an empty column corresponding to each other. Comment. Part (i) allows several different proofs. E.g., in the last paragraph of the solution, it suffices to deal only with the case $n=\\ell^{2}+1$. Notice now that among the four corner squares, at least one is empty. So the rooks in its row and in its column are distinct. Now, deleting this row and column we obtain an $\\ell^{2} \\times \\ell^{2}$ square with $\\ell^{2}-1$ rooks in it. This square can be partitioned into $\\ell^{2}$ squares of size $\\ell \\times \\ell$, so one of them is empty.","tier":0} +{"problem_type":"Combinatorics","problem_label":"C4","problem":"Construct a tetromino by attaching two $2 \\times 1$ dominoes along their longer sides such that the midpoint of the longer side of one domino is a corner of the other domino. This construction yields two kinds of tetrominoes with opposite orientations. Let us call them Sand Z-tetrominoes, respectively. ![](https:\/\/cdn.mathpix.com\/cropped\/2024_11_18_62c85fb0b01b41aca29bg-07.jpg?height=181&width=321&top_left_y=1349&top_left_x=636) S-tetrominoes ![](https:\/\/cdn.mathpix.com\/cropped\/2024_11_18_62c85fb0b01b41aca29bg-07.jpg?height=167&width=318&top_left_y=1364&top_left_x=1115) Z-tetrominoes Assume that a lattice polygon $P$ can be tiled with S-tetrominoes. Prove than no matter how we tile $P$ using only S- and Z-tetrominoes, we always use an even number of Z-tetrominoes. (Hungary)","solution":"We may assume that polygon $P$ is the union of some squares of an infinite chessboard. Colour the squares of the chessboard with two colours as the figure below illustrates. ![](https:\/\/cdn.mathpix.com\/cropped\/2024_11_18_62c85fb0b01b41aca29bg-33.jpg?height=663&width=866&top_left_y=979&top_left_x=601) Observe that no matter how we tile $P$, any S-tetromino covers an even number of black squares, whereas any Z-tetromino covers an odd number of them. As $P$ can be tiled exclusively by S-tetrominoes, it contains an even number of black squares. But if some S-tetrominoes and some Z-tetrominoes cover an even number of black squares, then the number of Z-tetrominoes must be even. Comment. An alternative approach makes use of the following two colourings, which are perhaps somewhat more natural: ![](https:\/\/cdn.mathpix.com\/cropped\/2024_11_18_62c85fb0b01b41aca29bg-33.jpg?height=371&width=1297&top_left_y=2071&top_left_x=385) Let $s_{1}$ and $s_{2}$ be the number of $S$-tetrominoes of the first and second type (as shown in the figure above) respectively that are used in a tiling of $P$. Likewise, let $z_{1}$ and $z_{2}$ be the number of $Z$-tetrominoes of the first and second type respectively. The first colouring shows that $s_{1}+z_{2}$ is invariant modulo 2 , the second colouring shows that $s_{1}+z_{1}$ is invariant modulo 2 . Adding these two conditions, we find that $z_{1}+z_{2}$ is invariant modulo 2 , which is what we have to prove. Indeed, the sum of the two colourings (regarding white as 0 and black as 1 and adding modulo 2) is the colouring shown in the solution.","tier":0} +{"problem_type":"Combinatorics","problem_label":"C4","problem":"Construct a tetromino by attaching two $2 \\times 1$ dominoes along their longer sides such that the midpoint of the longer side of one domino is a corner of the other domino. This construction yields two kinds of tetrominoes with opposite orientations. Let us call them Sand Z-tetrominoes, respectively. ![](https:\/\/cdn.mathpix.com\/cropped\/2024_11_18_62c85fb0b01b41aca29bg-07.jpg?height=181&width=321&top_left_y=1349&top_left_x=636) S-tetrominoes ![](https:\/\/cdn.mathpix.com\/cropped\/2024_11_18_62c85fb0b01b41aca29bg-07.jpg?height=167&width=318&top_left_y=1364&top_left_x=1115) Z-tetrominoes Assume that a lattice polygon $P$ can be tiled with S-tetrominoes. Prove than no matter how we tile $P$ using only S- and Z-tetrominoes, we always use an even number of Z-tetrominoes. (Hungary)","solution":"Let us assign coordinates to the squares of the infinite chessboard in such a way that the squares of $P$ have nonnegative coordinates only, and that the first coordinate increases as one moves to the right, while the second coordinate increases as one moves upwards. Write the integer $3^{i} \\cdot(-3)^{j}$ into the square with coordinates $(i, j)$, as in the following figure: | $\\vdots$ | | | | | | | :---: | :---: | :---: | :---: | :---: | :---: | | 81 | $\\vdots$ | | | | | | -27 | -81 | $\\vdots$ | | | | | 9 | 27 | 81 | $\\cdots$ | | | | -3 | -9 | -27 | -81 | $\\cdots$ | | | 1 | 3 | 9 | 27 | 81 | $\\cdots$ | The sum of the numbers written into four squares that can be covered by an $S$-tetromino is either of the form $$ 3^{i} \\cdot(-3)^{j} \\cdot\\left(1+3+3 \\cdot(-3)+3^{2} \\cdot(-3)\\right)=-32 \\cdot 3^{i} \\cdot(-3)^{j} $$ (for the first type of $S$-tetrominoes), or of the form $$ 3^{i} \\cdot(-3)^{j} \\cdot\\left(3+3 \\cdot(-3)+(-3)+(-3)^{2}\\right)=0 $$ and thus divisible by 32 . For this reason, the sum of the numbers written into the squares of $P$, and thus also the sum of the numbers covered by $Z$-tetrominoes in the second covering, is likewise divisible by 32 . Now the sum of the entries of a $Z$-tetromino is either of the form $$ 3^{i} \\cdot(-3)^{j} \\cdot\\left(3+3^{2}+(-3)+3 \\cdot(-3)\\right)=0 $$ (for the first type of $Z$-tetrominoes), or of the form $$ 3^{i} \\cdot(-3)^{j} \\cdot\\left(1+(-3)+3 \\cdot(-3)+3 \\cdot(-3)^{2}\\right)=16 \\cdot 3^{i} \\cdot(-3)^{j} $$ i.e., 16 times an odd number. Thus in order to obtain a total that is divisible by 32 , an even number of the latter kind of $Z$-tetrominoes needs to be used. Rotating everything by $90^{\\circ}$, we find that the number of $Z$-tetrominoes of the first kind is even as well. So we have even proven slightly more than necessary. Comment 1. In the second solution, 3 and -3 can be replaced by other combinations as well. For example, for any positive integer $a \\equiv 3(\\bmod 4)$, we can write $a^{i} \\cdot(-a)^{j}$ into the square with coordinates $(i, j)$ and apply the same argument. Comment 2. As the second solution shows, we even have the stronger result that the parity of the number of each of the four types of tetrominoes in a tiling of $P$ by S - and Z-tetrominoes is an invariant of $P$. This also remains true if there is no tiling of $P$ that uses only S-tetrominoes.","tier":0} +{"problem_type":"Combinatorics","problem_label":"C5","problem":"Consider $n \\geqslant 3$ lines in the plane such that no two lines are parallel and no three have a common point. These lines divide the plane into polygonal regions; let $\\mathcal{F}$ be the set of regions having finite area. Prove that it is possible to colour $\\lceil\\sqrt{n \/ 2}\\rceil$ of the lines blue in such a way that no region in $\\mathcal{F}$ has a completely blue boundary.","solution":"Let $L$ be the given set of lines. Choose a maximal (by inclusion) subset $B \\subseteq L$ such that when we colour the lines of $B$ blue, no region in $\\mathcal{F}$ has a completely blue boundary. Let $|B|=k$. We claim that $k \\geqslant\\lceil\\sqrt{n \/ 2}\\rceil$. Let us colour all the lines of $L \\backslash B$ red. Call a point blue if it is the intersection of two blue lines. Then there are $\\binom{k}{2}$ blue points. Now consider any red line $\\ell$. By the maximality of $B$, there exists at least one region $A \\in \\mathcal{F}$ whose only red side lies on $\\ell$. Since $A$ has at least three sides, it must have at least one blue vertex. Let us take one such vertex and associate it to $\\ell$. Since each blue point belongs to four regions (some of which may be unbounded), it is associated to at most four red lines. Thus the total number of red lines is at most $4\\binom{k}{2}$. On the other hand, this number is $n-k$, so $$ n-k \\leqslant 2 k(k-1), \\quad \\text { thus } \\quad n \\leqslant 2 k^{2}-k \\leqslant 2 k^{2} $$ and finally $k \\geqslant\\lceil\\sqrt{n \/ 2}\\rceil$, which gives the desired result. Comment 1. The constant factor in the estimate can be improved in different ways; we sketch two of them below. On the other hand, the Problem Selection Committee is not aware of any results showing that it is sometimes impossible to colour $k$ lines satisfying the desired condition for $k \\gg \\sqrt{n}$. In this situation we find it more suitable to keep the original formulation of the problem. 1. Firstly, we show that in the proof above one has in fact $k=|B| \\geqslant\\lceil\\sqrt{2 n \/ 3}\\rceil$. Let us make weighted associations as follows. Let a region $A$ whose only red side lies on $\\ell$ have $k$ vertices, so that $k-2$ of them are blue. We associate each of these blue vertices to $\\ell$, and put the weight $\\frac{1}{k-2}$ on each such association. So the sum of the weights of all the associations is exactly $n-k$. Now, one may check that among the four regions adjacent to a blue vertex $v$, at most two are triangles. This means that the sum of the weights of all associations involving $v$ is at most $1+1+\\frac{1}{2}+\\frac{1}{2}=3$. This leads to the estimate $$ n-k \\leqslant 3\\binom{k}{2} $$ or $$ 2 n \\leqslant 3 k^{2}-k<3 k^{2} $$ which yields $k \\geqslant\\lceil\\sqrt{2 n \/ 3}\\rceil$. 2. Next, we even show that $k=|B| \\geqslant\\lceil\\sqrt{n}\\rceil$. For this, we specify the process of associating points to red lines in one more different way. Call a point red if it lies on a red line as well as on a blue line. Consider any red line $\\ell$, and take an arbitrary region $A \\in \\mathcal{F}$ whose only red side lies on $\\ell$. Let $r^{\\prime}, r, b_{1}, \\ldots, b_{k}$ be its vertices in clockwise order with $r^{\\prime}, r \\in \\ell$; then the points $r^{\\prime}, r$ are red, while all the points $b_{1}, \\ldots, b_{k}$ are blue. Let us associate to $\\ell$ the red point $r$ and the blue point $b_{1}$. One may notice that to each pair of a red point $r$ and a blue point $b$, at most one red line can be associated, since there is at most one region $A$ having $r$ and $b$ as two clockwise consecutive vertices. We claim now that at most two red lines are associated to each blue point $b$; this leads to the desired bound $$ n-k \\leqslant 2\\binom{k}{2} \\quad \\Longleftrightarrow \\quad n \\leqslant k^{2} $$ Assume, to the contrary, that three red lines $\\ell_{1}, \\ell_{2}$, and $\\ell_{3}$ are associated to the same blue point $b$. Let $r_{1}, r_{2}$, and $r_{3}$ respectively be the red points associated to these lines; all these points are distinct. The point $b$ defines four blue rays, and each point $r_{i}$ is the red point closest to $b$ on one of these rays. So we may assume that the points $r_{2}$ and $r_{3}$ lie on one blue line passing through $b$, while $r_{1}$ lies on the other one. ![](https:\/\/cdn.mathpix.com\/cropped\/2024_11_18_62c85fb0b01b41aca29bg-36.jpg?height=432&width=532&top_left_y=475&top_left_x=768) Now consider the region $A$ used to associate $r_{1}$ and $b$ with $\\ell_{1}$. Three of its clockwise consecutive vertices are $r_{1}, b$, and either $r_{2}$ or $r_{3}$ (say, $r_{2}$ ). Since $A$ has only one red side, it can only be the triangle $r_{1} b r_{2}$; but then both $\\ell_{1}$ and $\\ell_{2}$ pass through $r_{2}$, as well as some blue line. This is impossible by the problem assumptions. Comment 2. The condition that the lines be non-parallel is essentially not used in the solution, nor in the previous comment; thus it may be omitted.","tier":0} +{"problem_type":"Combinatorics","problem_label":"C6","problem":"We are given an infinite deck of cards, each with a real number on it. For every real number $x$, there is exactly one card in the deck that has $x$ written on it. Now two players draw disjoint sets $A$ and $B$ of 100 cards each from this deck. We would like to define a rule that declares one of them a winner. This rule should satisfy the following conditions: 1. The winner only depends on the relative order of the 200 cards: if the cards are laid down in increasing order face down and we are told which card belongs to which player, but not what numbers are written on them, we can still decide the winner. 2. If we write the elements of both sets in increasing order as $A=\\left\\{a_{1}, a_{2}, \\ldots, a_{100}\\right\\}$ and $B=\\left\\{b_{1}, b_{2}, \\ldots, b_{100}\\right\\}$, and $a_{i}>b_{i}$ for all $i$, then $A$ beats $B$. 3. If three players draw three disjoint sets $A, B, C$ from the deck, $A$ beats $B$ and $B$ beats $C$, then $A$ also beats $C$. How many ways are there to define such a rule? Here, we consider two rules as different if there exist two sets $A$ and $B$ such that $A$ beats $B$ according to one rule, but $B$ beats $A$ according to the other. (Russia)","solution":"We prove a more general statement for sets of cardinality $n$ (the problem being the special case $n=100$, then the answer is $n$ ). In the following, we write $A>B$ or $Bb_{k}$. This rule clearly satisfies all three conditions, and the rules corresponding to different $k$ are all different. Thus there are at least $n$ different rules. Part II. Now we have to prove that there is no other way to define such a rule. Suppose that our rule satisfies the conditions, and let $k \\in\\{1,2, \\ldots, n\\}$ be minimal with the property that $$ A_{k}=\\{1,2, \\ldots, k, n+k+1, n+k+2, \\ldots, 2 n\\} \\prec B_{k}=\\{k+1, k+2, \\ldots, n+k\\} $$ Clearly, such a $k$ exists, since this holds for $k=n$ by assumption. Now consider two disjoint sets $X=\\left\\{x_{1}, x_{2}, \\ldots, x_{n}\\right\\}$ and $Y=\\left\\{y_{1}, y_{2}, \\ldots, y_{n}\\right\\}$, both in increasing order (i.e., $x_{1}B_{k-1}$ by our choice of $k$, we also have $U>W$ (if $k=1$, this is trivial). - The elements of $V \\cup W$ are ordered in the same way as those of $A_{k} \\cup B_{k}$, and since $A_{k} \\prec B_{k}$ by our choice of $k$, we also have $V \\prec W$. It follows that $$ X \\prec V \\prec W \\prec U \\prec Y $$ so $Xb_{i}$ for all $i$, then $A$ beats $B$. 3. If three players draw three disjoint sets $A, B, C$ from the deck, $A$ beats $B$ and $B$ beats $C$, then $A$ also beats $C$. How many ways are there to define such a rule? Here, we consider two rules as different if there exist two sets $A$ and $B$ such that $A$ beats $B$ according to one rule, but $B$ beats $A$ according to the other. (Russia)","solution":"Another possible approach to Part II of this problem is induction on $n$. For $n=1$, there is trivially only one rule in view of the second condition. In the following, we assume that our claim (namely, that there are no possible rules other than those given in Part I) holds for $n-1$ in place of $n$. We start with the following observation: Claim. At least one of the two relations $$ (\\{2\\} \\cup\\{2 i-1 \\mid 2 \\leqslant i \\leqslant n\\})<(\\{1\\} \\cup\\{2 i \\mid 2 \\leqslant i \\leqslant n\\}) $$ and $$ (\\{2 i-1 \\mid 1 \\leqslant i \\leqslant n-1\\} \\cup\\{2 n\\})<(\\{2 i \\mid 1 \\leqslant i \\leqslant n-1\\} \\cup\\{2 n-1\\}) $$ holds. Proof. Suppose that the first relation does not hold. Since our rule may only depend on the relative order, we must also have $$ (\\{2\\} \\cup\\{3 i-2 \\mid 2 \\leqslant i \\leqslant n-1\\} \\cup\\{3 n-2\\})>(\\{1\\} \\cup\\{3 i-1 \\mid 2 \\leqslant i \\leqslant n-1\\} \\cup\\{3 n\\}) . $$ Likewise, if the second relation does not hold, then we must also have $$ (\\{1\\} \\cup\\{3 i-1 \\mid 2 \\leqslant i \\leqslant n-1\\} \\cup\\{3 n\\})>(\\{3\\} \\cup\\{3 i \\mid 2 \\leqslant i \\leqslant n-1\\} \\cup\\{3 n-1\\}) $$ Now condition 3 implies that $$ (\\{2\\} \\cup\\{3 i-2 \\mid 2 \\leqslant i \\leqslant n-1\\} \\cup\\{3 n-2\\})>(\\{3\\} \\cup\\{3 i \\mid 2 \\leqslant i \\leqslant n-1\\} \\cup\\{3 n-1\\}) $$ which contradicts the second condition. Now we distinguish two cases, depending on which of the two relations actually holds: First case: $(\\{2\\} \\cup\\{2 i-1 \\mid 2 \\leqslant i \\leqslant n\\})<(\\{1\\} \\cup\\{2 i \\mid 2 \\leqslant i \\leqslant n\\})$. Let $A=\\left\\{a_{1}, a_{2}, \\ldots, a_{n}\\right\\}$ and $B=\\left\\{b_{1}, b_{2}, \\ldots, b_{n}\\right\\}$ be two disjoint sets, both in increasing order. We claim that the winner can be decided only from the values of $a_{2}, \\ldots, a_{n}$ and $b_{2}, \\ldots, b_{n}$, while $a_{1}$ and $b_{1}$ are actually irrelevant. Suppose that this was not the case, and assume without loss of generality that $a_{2}0$ be smaller than half the distance between any two of the numbers in $B_{x} \\cup B_{y} \\cup A$. For any set $M$, let $M \\pm \\varepsilon$ be the set obtained by adding\/subtracting $\\varepsilon$ to all elements of $M$. By our choice of $\\varepsilon$, the relative order of the elements of $\\left(B_{y}+\\varepsilon\\right) \\cup A$ is still the same as for $B_{y} \\cup A$, while the relative order of the elements of $\\left(B_{x}-\\varepsilon\\right) \\cup A$ is still the same as for $B_{x} \\cup A$. Thus $A \\prec B_{x}-\\varepsilon$, but $A>B_{y}+\\varepsilon$. Moreover, if $y>x$, then $B_{x}-\\varepsilon \\prec B_{y}+\\varepsilon$ by condition 2, while otherwise the relative order of the elements in $\\left(B_{x}-\\varepsilon\\right) \\cup\\left(B_{y}+\\varepsilon\\right)$ is the same as for the two sets $\\{2\\} \\cup\\{2 i-1 \\mid 2 \\leqslant i \\leqslant n\\}$ and $\\{1\\} \\cup\\{2 i \\mid 2 \\leqslant i \\leqslant n\\}$, so that $B_{x}-\\varepsilon_{\\sigma} B$ if and only if $\\left(a_{\\sigma(1)}, \\ldots, a_{\\sigma(n)}\\right)$ is lexicographically greater than $\\left(b_{\\sigma(1)}, \\ldots, b_{\\sigma(n)}\\right)$. It seems, however, that this formulation adds rather more technicalities to the problem than additional ideas. This page is intentionally left blank","tier":0} +{"problem_type":"Combinatorics","problem_label":"C7","problem":"Let $M$ be a set of $n \\geqslant 4$ points in the plane, no three of which are collinear. Initially these points are connected with $n$ segments so that each point in $M$ is the endpoint of exactly two segments. Then, at each step, one may choose two segments $A B$ and $C D$ sharing a common interior point and replace them by the segments $A C$ and $B D$ if none of them is present at this moment. Prove that it is impossible to perform $n^{3} \/ 4$ or more such moves. (Russia)","solution":"A line is said to be red if it contains two points of $M$. As no three points of $M$ are collinear, each red line determines a unique pair of points of $M$. Moreover, there are precisely $\\binom{n}{2}<\\frac{n^{2}}{2}$ red lines. By the value of a segment we mean the number of red lines intersecting it in its interior, and the value of a set of segments is defined to be the sum of the values of its elements. We will prove that $(i)$ the value of the initial set of segments is smaller than $n^{3} \/ 2$ and that (ii) each step decreases the value of the set of segments present by at least 2 . Since such a value can never be negative, these two assertions imply the statement of the problem. To show $(i)$ we just need to observe that each segment has a value that is smaller than $n^{2} \/ 2$. Thus the combined value of the $n$ initial segments is indeed below $n \\cdot n^{2} \/ 2=n^{3} \/ 2$. It remains to establish (ii). Suppose that at some moment we have two segments $A B$ and $C D$ sharing an interior point $S$, and that at the next moment we have the two segments $A C$ and $B D$ instead. Let $X_{A B}$ denote the set of red lines intersecting the segment $A B$ in its interior and let the sets $X_{A C}, X_{B D}$, and $X_{C D}$ be defined similarly. We are to prove that $\\left|X_{A C}\\right|+\\left|X_{B D}\\right|+2 \\leqslant\\left|X_{A B}\\right|+\\left|X_{C D}\\right|$. As a first step in this direction, we claim that $$ \\left|X_{A C} \\cup X_{B D}\\right|+2 \\leqslant\\left|X_{A B} \\cup X_{C D}\\right| . $$ Indeed, if $g$ is a red line intersecting, e.g. the segment $A C$ in its interior, then it has to intersect the triangle $A C S$ once again, either in the interior of its side $A S$, or in the interior of its side $C S$, or at $S$, meaning that it belongs to $X_{A B}$ or to $X_{C D}$ (see Figure 1). Moreover, the red lines $A B$ and $C D$ contribute to $X_{A B} \\cup X_{C D}$ but not to $X_{A C} \\cup X_{B D}$. Thereby (1) is proved. ![](https:\/\/cdn.mathpix.com\/cropped\/2024_11_18_62c85fb0b01b41aca29bg-41.jpg?height=404&width=390&top_left_y=1780&top_left_x=339) Figure 1 ![](https:\/\/cdn.mathpix.com\/cropped\/2024_11_18_62c85fb0b01b41aca29bg-41.jpg?height=452&width=360&top_left_y=1733&top_left_x=865) Figure 2 ![](https:\/\/cdn.mathpix.com\/cropped\/2024_11_18_62c85fb0b01b41aca29bg-41.jpg?height=448&width=366&top_left_y=1729&top_left_x=1356) Figure 3 Similarly but more easily one obtains $$ \\left|X_{A C} \\cap X_{B D}\\right| \\leqslant\\left|X_{A B} \\cap X_{C D}\\right| $$ Indeed, a red line $h$ appearing in $X_{A C} \\cap X_{B D}$ belongs, for similar reasons as above, also to $X_{A B} \\cap X_{C D}$. To make the argument precise, one may just distinguish the cases $S \\in h$ (see Figure 2) and $S \\notin h$ (see Figure 3). Thereby (2) is proved. Adding (1) and (2) we obtain the desired conclusion, thus completing the solution of this problem. Comment 1. There is a problem belonging to the folklore, in the solution of which one may use the same kind of operation: Given $n$ red and $n$ green points in the plane, prove that one may draw $n$ nonintersecting segments each of which connects a red point with a green point. A standard approach to this problem consists in taking $n$ arbitrary segments connecting the red points with the green points, and to perform the same operation as in the above proposal whenever an intersection occurs. Now each time one performs such a step, the total length of the segments that are present decreases due to the triangle inequality. So, as there are only finitely many possibilities for the set of segments present, the process must end at some stage. In the above proposal, however, considering the sum of the Euclidean lengths of the segment that are present does not seem to help much, for even though it shows that the process must necessarily terminate after some finite number of steps, it does not seem to easily yield any upper bound on the number of these steps that grows polynomially with $n$. One may regard the concept of the value of a segment introduced in the above solution as an appropriately discretised version of Euclidean length suitable for obtaining such a bound. The Problem Selection Committee still believes the problem to be sufficiently original for the competition. Comment 2. There are some other essentially equivalent ways of presenting the same solution. E.g., put $M=\\left\\{A_{1}, A_{2}, \\ldots, A_{n}\\right\\}$, denote the set of segments present at any moment by $\\left\\{e_{1}, e_{2}, \\ldots, e_{n}\\right\\}$, and called a triple $(i, j, k)$ of indices with $i \\neq j$ intersecting, if the line $A_{i} A_{j}$ intersects the segment $e_{k}$. It may then be shown that the number $S$ of intersecting triples satisfies $0 \\leqslant S0$ ). It would be interesting to say more about the gap between $c n^{2}$ and $c n^{3}$.","tier":0} +{"problem_type":"Combinatorics","problem_label":"C8","problem":"A card deck consists of 1024 cards. On each card, a set of distinct decimal digits is written in such a way that no two of these sets coincide (thus, one of the cards is empty). Two players alternately take cards from the deck, one card per turn. After the deck is empty, each player checks if he can throw out one of his cards so that each of the ten digits occurs on an even number of his remaining cards. If one player can do this but the other one cannot, the one who can is the winner; otherwise a draw is declared. Determine all possible first moves of the first player after which he has a winning strategy. (Russia)","solution":"Let us identify each card with the set of digits written on it. For any collection of cards $C_{1}, C_{2}, \\ldots, C_{k}$ denote by their sum the set $C_{1} \\triangle C_{2} \\triangle \\cdots \\triangle C_{k}$ consisting of all elements belonging to an odd number of the $C_{i}$ 's. Denote the first and the second player by $\\mathcal{F}$ and $\\mathcal{S}$, respectively. Since each digit is written on exactly 512 cards, the sum of all the cards is $\\varnothing$. Therefore, at the end of the game the sum of all the cards of $\\mathcal{F}$ will be the same as that of $\\mathcal{S}$; denote this sum by $C$. Then the player who took $C$ can throw it out and get the desired situation, while the other one cannot. Thus, the player getting card $C$ wins, and no draw is possible. Now, given a nonempty card $B$, one can easily see that all the cards can be split into 512 pairs of the form $(X, X \\triangle B)$ because $(X \\triangle B) \\triangle B=X$. The following lemma shows a property of such a partition that is important for the solution. Lemma. Let $B \\neq \\varnothing$ be some card. Let us choose 512 cards so that exactly one card is chosen from every pair $(X, X \\triangle B)$. Then the sum of all chosen cards is either $\\varnothing$ or $B$. Proof. Let $b$ be some element of $B$. Enumerate the pairs; let $X_{i}$ be the card not containing $b$ in the $i^{\\text {th }}$ pair, and let $Y_{i}$ be the other card in this pair. Then the sets $X_{i}$ are exactly all the sets not containing $b$, therefore each digit $a \\neq b$ is written on exactly 256 of these cards, so $X_{1} \\triangle X_{2} \\triangle \\cdots \\triangle X_{512}=\\varnothing$. Now, if we replace some summands in this sum by the other elements from their pairs, we will simply add $B$ several times to this sum, thus the sum will either remain unchanged or change by $B$, as required. Now we consider two cases. Case 1. Assume that $\\mathcal{F}$ takes the card $\\varnothing$ on his first move. In this case, we present a winning strategy for $\\mathcal{S}$. Let $\\mathcal{S}$ take an arbitrary card $A$. Assume that $\\mathcal{F}$ takes card $B$ after that; then $\\mathcal{S}$ takes $A \\triangle B$. Split all 1024 cards into 512 pairs of the form $(X, X \\triangle B)$; we call two cards in one pair partners. Then the four cards taken so far form two pairs $(\\varnothing, B)$ and $(A, A \\triangle B)$ belonging to $\\mathcal{F}$ and $\\mathcal{S}$, respectively. On each of the subsequent moves, when $\\mathcal{F}$ takes some card, $\\mathcal{S}$ should take the partner of this card in response. Consider the situation at the end of the game. Let us for a moment replace card $A$ belonging to $\\mathcal{S}$ by $\\varnothing$. Then he would have one card from each pair; by our lemma, the sum of all these cards would be either $\\varnothing$ or $B$. Now, replacing $\\varnothing$ back by $A$ we get that the actual sum of the cards of $\\mathcal{S}$ is either $A$ or $A \\triangle B$, and he has both these cards. Thus $\\mathcal{S}$ wins. Case 2. Now assume that $\\mathcal{F}$ takes some card $A \\neq \\varnothing$ on his first move. Let us present a winning strategy for $\\mathcal{F}$ in this case. Assume that $\\mathcal{S}$ takes some card $B \\neq \\varnothing$ on his first move; then $\\mathcal{F}$ takes $A \\triangle B$. Again, let us split all the cards into pairs of the form $(X, X \\triangle B)$; then the cards which have not been taken yet form several complete pairs and one extra element (card $\\varnothing$ has not been taken while its partner $B$ has). Now, on each of the subsequent moves, if $\\mathcal{S}$ takes some element from a complete pair, then $\\mathcal{F}$ takes its partner. If $\\mathcal{S}$ takes the extra element, then $\\mathcal{F}$ takes an arbitrary card $Y$, and the partner of $Y$ becomes the new extra element. Thus, on his last move $\\mathcal{S}$ is forced to take the extra element. After that player $\\mathcal{F}$ has cards $A$ and $A \\triangle B$, player $\\mathcal{S}$ has cards $B$ and $\\varnothing$, and $\\mathcal{F}$ has exactly one element from every other pair. Thus the situation is the same as in the previous case with roles reversed, and $\\mathcal{F}$ wins. Finally, if $\\mathcal{S}$ takes $\\varnothing$ on his first move then $\\mathcal{F}$ denotes any card which has not been taken yet by $B$ and takes $A \\triangle B$. After that, the same strategy as above is applicable. Comment 1. If one wants to avoid the unusual question about the first move, one may change the formulation as follows. (The difficulty of the problem would decrease somewhat.) A card deck consists of 1023 cards; on each card, a nonempty set of distinct decimal digits is written in such a way that no two of these sets coincide. Two players alternately take cards from the deck, one card per turn. When the deck is empty, each player checks if he can throw out one of his cards so that for each of the ten digits, he still holds an even number of cards with this digit. If one player can do this but the other one cannot, the one who can is the winner; otherwise a draw is declared. Determine which of the players (if any) has a winning strategy. The winner in this version is the first player. The analysis of the game from the first two paragraphs of the previous solution applies to this version as well, except for the case $C=\\varnothing$ in which the result is a draw. Then the strategy for $\\mathcal{S}$ in Case 1 works for $\\mathcal{F}$ in this version: the sum of all his cards at the end is either $A$ or $A \\triangle B$, thus nonempty in both cases. Comment 2. Notice that all the cards form a vector space over $\\mathbb{F}_{2}$, with $\\triangle$ the operation of addition. Due to the automorphisms of this space, all possibilities for $\\mathcal{F}$ 's first move except $\\varnothing$ are equivalent. The same holds for the response by $\\mathcal{S}$ if $\\mathcal{F}$ takes the card $\\varnothing$ on his first move. Comment 3. It is not that hard to show that in the initial game, $\\mathcal{F}$ has a winning move, by the idea of \"strategy stealing\". Namely, assume that $\\mathcal{S}$ has a winning strategy. Let us take two card decks and start two games, in which $\\mathcal{S}$ will act by his strategy. In the first game, $\\mathcal{F}$ takes an arbitrary card $A_{1}$; assume that $\\mathcal{S}$ takes some $B_{1}$ in response. Then $\\mathcal{F}$ takes the card $B_{1}$ at the second game; let the response by $\\mathcal{S}$ be $A_{2}$. Then $\\mathcal{F}$ takes $A_{2}$ in the first game and gets a response $B_{2}$, and so on. This process stops at some moment when in the second game $\\mathcal{S}$ takes $A_{i}=A_{1}$. At this moment the players hold the same sets of cards in both games, but with roles reversed. Now, if some cards remain in the decks, $\\mathcal{F}$ takes an arbitrary card from the first deck starting a similar cycle. At the end of the game, player $\\mathcal{F}$ 's cards in the first game are exactly player $\\mathcal{S}$ 's cards in the second game, and vice versa. Thus in one of the games $\\mathcal{F}$ will win, which is impossible by our assumption. One may notice that the strategy in Case 2 is constructed exactly in this way from the strategy in Case 1 . This is possible since every response by $\\mathcal{S}$ wins if $\\mathcal{F}$ takes the card $\\varnothing$ on his first move.","tier":0} +{"problem_type":"Combinatorics","problem_label":"C9","problem":"There are $n$ circles drawn on a piece of paper in such a way that any two circles intersect in two points, and no three circles pass through the same point. Turbo the snail slides along the circles in the following fashion. Initially he moves on one of the circles in clockwise direction. Turbo always keeps sliding along the current circle until he reaches an intersection with another circle. Then he continues his journey on this new circle and also changes the direction of moving, i.e. from clockwise to anticlockwise or vice versa. Suppose that Turbo's path entirely covers all circles. Prove that $n$ must be odd.","solution":"Replace every cross (i.e. intersection of two circles) by two small circle arcs that indicate the direction in which the snail should leave the cross (see Figure 1.1). Notice that the placement of the small arcs does not depend on the direction of moving on the curves; no matter which direction the snail is moving on the circle arcs, he will follow the same curves (see Figure 1.2). In this way we have a set of curves, that are the possible paths of the snail. Call these curves snail orbits or just orbits. Every snail orbit is a simple closed curve that has no intersection with any other orbit. ![](https:\/\/cdn.mathpix.com\/cropped\/2024_11_18_62c85fb0b01b41aca29bg-45.jpg?height=355&width=375&top_left_y=1025&top_left_x=406) Figure 1.1 ![](https:\/\/cdn.mathpix.com\/cropped\/2024_11_18_62c85fb0b01b41aca29bg-45.jpg?height=347&width=670&top_left_y=1020&top_left_x=930) Figure 1.2 We prove the following, more general statement. (*) In any configuration of $n$ circles such that no two of them are tangent, the number of snail orbits has the same parity as the number $n$. (Note that it is not assumed that all circle pairs intersect.) This immediately solves the problem. Let us introduce the following operation that will be called flipping a cross. At a cross, remove the two small arcs of the orbits, and replace them by the other two arcs. Hence, when the snail arrives at a flipped cross, he will continue on the other circle as before, but he will preserve the orientation in which he goes along the circle arcs (see Figure 2). ![](https:\/\/cdn.mathpix.com\/cropped\/2024_11_18_62c85fb0b01b41aca29bg-45.jpg?height=292&width=735&top_left_y=1964&top_left_x=663) Figure 2 Consider what happens to the number of orbits when a cross is flipped. Denote by $a, b, c$, and $d$ the four arcs that meet at the cross such that $a$ and $b$ belong to the same circle. Before the flipping $a$ and $b$ were connected to $c$ and $d$, respectively, and after the flipping $a$ and $b$ are connected to $d$ and $c$, respectively. The orbits passing through the cross are closed curves, so each of the $\\operatorname{arcs} a, b, c$, and $d$ is connected to another one by orbits outside the cross. We distinguish three cases. Case 1: $a$ is connected to $b$ and $c$ is connected to $d$ by the orbits outside the cross (see Figure 3.1). We show that this case is impossible. Remove the two small arcs at the cross, connect $a$ to $b$, and connect $c$ to $d$ at the cross. Let $\\gamma$ be the new closed curve containing $a$ and $b$, and let $\\delta$ be the new curve that connects $c$ and $d$. These two curves intersect at the cross. So one of $c$ and $d$ is inside $\\gamma$ and the other one is outside $\\gamma$. Then the two closed curves have to meet at least one more time, but this is a contradiction, since no orbit can intersect itself. ![](https:\/\/cdn.mathpix.com\/cropped\/2024_11_18_62c85fb0b01b41aca29bg-46.jpg?height=247&width=333&top_left_y=499&top_left_x=239) Figure 3.1 ![](https:\/\/cdn.mathpix.com\/cropped\/2024_11_18_62c85fb0b01b41aca29bg-46.jpg?height=232&width=552&top_left_y=501&top_left_x=632) Figure 3.2 ![](https:\/\/cdn.mathpix.com\/cropped\/2024_11_18_62c85fb0b01b41aca29bg-46.jpg?height=235&width=575&top_left_y=496&top_left_x=1249) Figure 3.3 Case 2: $a$ is connected to $c$ and $b$ is connected to $d$ (see Figure 3.2). Before the flipping $a$ and $c$ belong to one orbit and $b$ and $d$ belong to another orbit. Flipping the cross merges the two orbits into a single orbit. Hence, the number of orbits decreases by 1. Case 3: $a$ is connected to $d$ and $b$ is connected to $c$ (see Figure 3.3). Before the flipping the arcs $a, b, c$, and $d$ belong to a single orbit. Flipping the cross splits that orbit in two. The number of orbits increases by 1. As can be seen, every flipping decreases or increases the number of orbits by one, thus changes its parity. Now flip every cross, one by one. Since every pair of circles has 0 or 2 intersections, the number of crosses is even. Therefore, when all crosses have been flipped, the original parity of the number of orbits is restored. So it is sufficient to prove (*) for the new configuration, where all crosses are flipped. Of course also in this new configuration the (modified) orbits are simple closed curves not intersecting each other. Orient the orbits in such a way that the snail always moves anticlockwise along the circle arcs. Figure 4 shows the same circles as in Figure 1 after flipping all crosses and adding orientation. (Note that this orientation may be different from the orientation of the orbit as a planar curve; the orientation of every orbit may be negative as well as positive, like the middle orbit in Figure 4.) If the snail moves around an orbit, the total angle change in his moving direction, the total curvature, is either $+2 \\pi$ or $-2 \\pi$, depending on the orientation of the orbit. Let $P$ and $N$ be the number of orbits with positive and negative orientation, respectively. Then the total curvature of all orbits is $(P-N) \\cdot 2 \\pi$. ![](https:\/\/cdn.mathpix.com\/cropped\/2024_11_18_62c85fb0b01b41aca29bg-46.jpg?height=372&width=412&top_left_y=1961&top_left_x=499) Figure 4 ![](https:\/\/cdn.mathpix.com\/cropped\/2024_11_18_62c85fb0b01b41aca29bg-46.jpg?height=213&width=500&top_left_y=2115&top_left_x=1052) Figure 5 Double-count the total curvature of all orbits. Along every circle the total curvature is $2 \\pi$. At every cross, the two turnings make two changes with some angles having the same absolute value but opposite signs, as depicted in Figure 5. So the changes in the direction at the crosses cancel out. Hence, the total curvature is $n \\cdot 2 \\pi$. Now we have $(P-N) \\cdot 2 \\pi=n \\cdot 2 \\pi$, so $P-N=n$. The number of (modified) orbits is $P+N$, that has a same parity as $P-N=n$.","tier":0} +{"problem_type":"Combinatorics","problem_label":"C9","problem":"There are $n$ circles drawn on a piece of paper in such a way that any two circles intersect in two points, and no three circles pass through the same point. Turbo the snail slides along the circles in the following fashion. Initially he moves on one of the circles in clockwise direction. Turbo always keeps sliding along the current circle until he reaches an intersection with another circle. Then he continues his journey on this new circle and also changes the direction of moving, i.e. from clockwise to anticlockwise or vice versa. Suppose that Turbo's path entirely covers all circles. Prove that $n$ must be odd.","solution":"We present a different proof of (*). We perform a sequence of small modification steps on the configuration of the circles in such a way that at the end they have no intersection at all (see Figure 6.1). We use two kinds of local changes to the structure of the orbits (see Figure 6.2): - Type-1 step: An arc of a circle is moved over an arc of another circle; such a step creates or removes two intersections. - Type-2 step: An arc of a circle is moved through the intersection of two other circles. ![](https:\/\/cdn.mathpix.com\/cropped\/2024_11_18_62c85fb0b01b41aca29bg-47.jpg?height=313&width=701&top_left_y=803&top_left_x=295) Figure 6.1 ![](https:\/\/cdn.mathpix.com\/cropped\/2024_11_18_62c85fb0b01b41aca29bg-47.jpg?height=292&width=189&top_left_y=819&top_left_x=1139) Figure 6.2 We assume that in every step only one circle is moved, and that this circle is moved over at most one arc or intersection point of other circles. We will show that the parity of the number of orbits does not change in any step. As every circle becomes a separate orbit at the end of the procedure, this fact proves (*). Consider what happens to the number of orbits when a Type-1 step is performed. The two intersection points are created or removed in a small neighbourhood. Denote some points of the two circles where they enter or leave this neighbourhood by $a, b, c$, and $d$ in this order around the neighbourhood; let $a$ and $b$ belong to one circle and let $c$ and $d$ belong to the other circle. The two circle arcs may have the same or opposite orientations. Moreover, the four end-points of the two arcs are connected by the other parts of the orbits. This can happen in two ways without intersection: either $a$ is connected to $d$ and $b$ is connected to $c$, or $a$ is connected to $b$ and $c$ is connected to $d$. Altogether we have four cases, as shown in Figure 7. ![](https:\/\/cdn.mathpix.com\/cropped\/2024_11_18_62c85fb0b01b41aca29bg-47.jpg?height=276&width=1621&top_left_y=1873&top_left_x=221) Figure 7 We can see that the number of orbits is changed by -2 or +2 in the leftmost case when the arcs have the same orientation, $a$ is connected to $d$, and $b$ is connected to $c$. In the other three cases the number of orbits is not changed. Hence, Type-1 steps do not change the parity of the number of orbits. Now consider a Type-2 step. The three circles enclose a small, triangular region; by the step, this triangle is replaced by another triangle. Again, the modification of the orbits is done in some small neighbourhood; the structure does not change outside. Each side of the triangle shaped region can be convex or concave; the number of concave sides can be $0,1,2$ or 3 , so there are 4 possible arrangements of the orbits inside the neighbourhood, as shown in Figure 8. ![](https:\/\/cdn.mathpix.com\/cropped\/2024_11_18_62c85fb0b01b41aca29bg-48.jpg?height=312&width=301&top_left_y=221&top_left_x=295) all convex ![](https:\/\/cdn.mathpix.com\/cropped\/2024_11_18_62c85fb0b01b41aca29bg-48.jpg?height=345&width=1092&top_left_y=219&top_left_x=683) Figure 8 Denote the points where the three circles enter or leave the neighbourhood by $a, b, c, d$, $e$, and $f$ in this order around the neighbourhood. As can be seen in Figure 8, there are only two essentially different cases; either $a, c, e$ are connected to $b, d, f$, respectively, or $a, c, e$ are connected to $f, b, d$, respectively. The step either preserves the set of connections or switches to the other arrangement. Obviously, in the earlier case the number of orbits is not changed; therefore we have to consider only the latter case. The points $a, b, c, d, e$, and $f$ are connected by the orbits outside, without intersection. If $a$ was connected to $c$, say, then this orbit would isolate $b$, so this is impossible. Hence, each of $a, b, c, d, e$ and $f$ must be connected either to one of its neighbours or to the opposite point. If say $a$ is connected to $d$, then this orbit separates $b$ and $c$ from $e$ and $f$, therefore $b$ must be connected to $c$ and $e$ must be connected to $f$. Altogether there are only two cases and their reverses: either each point is connected to one of its neighbours or two opposite points are connected and the the remaining neigh boring pairs are connected to each other. See Figure 9. ![](https:\/\/cdn.mathpix.com\/cropped\/2024_11_18_62c85fb0b01b41aca29bg-48.jpg?height=295&width=1628&top_left_y=1299&top_left_x=216) Figure 9 We can see that if only neighbouring points are connected, then the number of orbits is changed by +2 or -2 . If two opposite points are connected ( $a$ and $d$ in the figure), then the orbits are re-arranged, but their number is unchanged. Hence, Type-2 steps also preserve the parity. This completes the proof of (*).","tier":0} +{"problem_type":"Combinatorics","problem_label":"C9","problem":"There are $n$ circles drawn on a piece of paper in such a way that any two circles intersect in two points, and no three circles pass through the same point. Turbo the snail slides along the circles in the following fashion. Initially he moves on one of the circles in clockwise direction. Turbo always keeps sliding along the current circle until he reaches an intersection with another circle. Then he continues his journey on this new circle and also changes the direction of moving, i.e. from clockwise to anticlockwise or vice versa. Suppose that Turbo's path entirely covers all circles. Prove that $n$ must be odd.","solution":"Like in the previous solutions, we do not need all circle pairs to intersect but we assume that the circles form a connected set. Denote by $\\mathcal{C}$ and $\\mathcal{P}$ the sets of circles and their intersection points, respectively. The circles divide the plane into several simply connected, bounded regions and one unbounded region. Denote the set of these regions by $\\mathcal{R}$. We say that an intersection point or a region is odd or even if it is contained inside an odd or even number of circles, respectively. Let $\\mathcal{P}_{\\text {odd }}$ and $\\mathcal{R}_{\\text {odd }}$ be the sets of odd intersection points and odd regions, respectively. Claim. $$ \\left|\\mathcal{R}_{\\text {odd }}\\right|-\\left|\\mathcal{P}_{\\text {odd }}\\right| \\equiv n \\quad(\\bmod 2) . $$ Proof. For each circle $c \\in \\mathcal{C}$, denote by $R_{c}, P_{c}$, and $X_{c}$ the number of regions inside $c$, the number of intersection points inside $c$, and the number of circles intersecting $c$, respectively. The circles divide each other into several arcs; denote by $A_{c}$ the number of such arcs inside $c$. By double counting the regions and intersection points inside the circles we get $$ \\left|\\mathcal{R}_{\\mathrm{odd}}\\right| \\equiv \\sum_{c \\in \\mathcal{C}} R_{c} \\quad(\\bmod 2) \\quad \\text { and } \\quad\\left|\\mathcal{P}_{\\mathrm{odd}}\\right| \\equiv \\sum_{c \\in \\mathcal{C}} P_{c} \\quad(\\bmod 2) . $$ For each circle $c$, apply EULER's polyhedron theorem to the (simply connected) regions in $c$. There are $2 X_{c}$ intersection points on $c$; they divide the circle into $2 X_{c}$ arcs. The polyhedron theorem yields $\\left(R_{c}+1\\right)+\\left(P_{c}+2 X_{c}\\right)=\\left(A_{c}+2 X_{c}\\right)+2$, considering the exterior of $c$ as a single region. Therefore, $$ R_{c}+P_{c}=A_{c}+1 $$ Moreover, we have four arcs starting from every interior points inside $c$ and a single arc starting into the interior from each intersection point on the circle. By double-counting the end-points of the interior arcs we get $2 A_{c}=4 P_{c}+2 X_{c}$, so $$ A_{c}=2 P_{c}+X_{c} $$ The relations (2) and (3) together yield $$ R_{c}-P_{c}=X_{c}+1 $$ By summing up (4) for all circles we obtain $$ \\sum_{c \\in \\mathcal{C}} R_{c}-\\sum_{c \\in \\mathcal{C}} P_{c}=\\sum_{c \\in \\mathcal{C}} X_{c}+|\\mathcal{C}| $$ which yields $$ \\left|\\mathcal{R}_{\\mathrm{odd}}\\right|-\\left|\\mathcal{P}_{\\mathrm{odd}}\\right| \\equiv \\sum_{c \\in \\mathcal{C}} X_{c}+n \\quad(\\bmod 2) $$ Notice that in $\\sum_{c \\in \\mathcal{C}} X_{c}$ each intersecting circle pair is counted twice, i.e., for both circles in the pair, so $$ \\sum_{c \\in \\mathcal{C}} X_{c} \\equiv 0 \\quad(\\bmod 2) $$ which finishes the proof of the Claim. Now insert the same small arcs at the intersections as in the first solution, and suppose that there is a single snail orbit $b$. First we show that the odd regions are inside the curve $b$, while the even regions are outside. Take a region $r \\in \\mathcal{R}$ and a point $x$ in its interior, and draw a ray $y$, starting from $x$, that does not pass through any intersection point of the circles and is neither tangent to any of the circles. As is well-known, $x$ is inside the curve $b$ if and only if $y$ intersects $b$ an odd number of times (see Figure 10). Notice that if an arbitrary circle $c$ contains $x$ in its interior, then $c$ intersects $y$ at a single point; otherwise, if $x$ is outside $c$, then $c$ has 2 or 0 intersections with $y$. Therefore, $y$ intersects $b$ an odd number of times if and only if $x$ is contained in an odd number of circles, so if and only if $r$ is odd. ![](https:\/\/cdn.mathpix.com\/cropped\/2024_11_18_62c85fb0b01b41aca29bg-49.jpg?height=321&width=626&top_left_y=2255&top_left_x=715) Figure 10 Now consider an intersection point $p$ of two circles $c_{1}$ and $c_{2}$ and a small neighbourhood around $p$. Suppose that $p$ is contained inside $k$ circles. We have four regions that meet at $p$. Let $r_{1}$ be the region that lies outside both $c_{1}$ and $c_{2}$, let $r_{2}$ be the region that lies inside both $c_{1}$ and $c_{2}$, and let $r_{3}$ and $r_{4}$ be the two remaining regions, each lying inside exactly one of $c_{1}$ and $c_{2}$. The region $r_{1}$ is contained inside the same $k$ circles as $p$; the region $r_{2}$ is contained also by $c_{1}$ and $c_{2}$, so by $k+2$ circles in total; each of the regions $r_{3}$ and $r_{4}$ is contained inside $k+1$ circles. After the small arcs have been inserted at $p$, the regions $r_{1}$ and $r_{2}$ get connected, and the regions $r_{3}$ and $r_{4}$ remain separated at $p$ (see Figure 11). If $p$ is an odd point, then $r_{1}$ and $r_{2}$ are odd, so two odd regions are connected at $p$. Otherwise, if $p$ is even, then we have two even regions connected at $p$. ![](https:\/\/cdn.mathpix.com\/cropped\/2024_11_18_62c85fb0b01b41aca29bg-50.jpg?height=261&width=298&top_left_y=658&top_left_x=571) Figure 11 ![](https:\/\/cdn.mathpix.com\/cropped\/2024_11_18_62c85fb0b01b41aca29bg-50.jpg?height=286&width=481&top_left_y=645&top_left_x=1007) Figure 12 Consider the system of odd regions and their connections at the odd points as a graph. In this graph the odd regions are the vertices, and each odd point establishes an edge that connects two vertices (see Figure 12). As $b$ is a single closed curve, this graph is connected and contains no cycle, so the graph is a tree. Then the number of vertices must be by one greater than the number of edges, so $$ \\left|\\mathcal{R}_{\\text {odd }}\\right|-\\left|\\mathcal{P}_{\\text {odd }}\\right|=1 . $$ The relations (1) and (9) together prove that $n$ must be odd. Comment. For every odd $n$ there exists at least one configuration of $n$ circles with a single snail orbit. Figure 13 shows a possible configuration with 5 circles. In general, if a circle is rotated by $k \\cdot \\frac{360^{\\circ}}{n}$ $(k=1,2, \\ldots, n-1)$ around an interior point other than the centre, the circle and its rotated copies together provide a single snail orbit. ![](https:\/\/cdn.mathpix.com\/cropped\/2024_11_18_62c85fb0b01b41aca29bg-50.jpg?height=584&width=595&top_left_y=1641&top_left_x=736) Figure 13","tier":0} +{"problem_type":"Geometry","problem_label":"G1","problem":"The points $P$ and $Q$ are chosen on the side $B C$ of an acute-angled triangle $A B C$ so that $\\angle P A B=\\angle A C B$ and $\\angle Q A C=\\angle C B A$. The points $M$ and $N$ are taken on the rays $A P$ and $A Q$, respectively, so that $A P=P M$ and $A Q=Q N$. Prove that the lines $B M$ and $C N$ intersect on the circumcircle of the triangle $A B C$. (Georgia)","solution":"Denote by $S$ the intersection point of the lines $B M$ and $C N$. Let moreover $\\beta=\\angle Q A C=\\angle C B A$ and $\\gamma=\\angle P A B=\\angle A C B$. From these equalities it follows that the triangles $A B P$ and $C A Q$ are similar (see Figure 1). Therefore we obtain $$ \\frac{B P}{P M}=\\frac{B P}{P A}=\\frac{A Q}{Q C}=\\frac{N Q}{Q C} $$ Moreover, $$ \\angle B P M=\\beta+\\gamma=\\angle C Q N $$ Hence the triangles $B P M$ and $N Q C$ are similar. This gives $\\angle B M P=\\angle N C Q$, so the triangles $B P M$ and $B S C$ are also similar. Thus we get $$ \\angle C S B=\\angle B P M=\\beta+\\gamma=180^{\\circ}-\\angle B A C, $$ which completes the solution. ![](https:\/\/cdn.mathpix.com\/cropped\/2024_11_18_62c85fb0b01b41aca29bg-51.jpg?height=618&width=595&top_left_y=1299&top_left_x=374) Figure 1 ![](https:\/\/cdn.mathpix.com\/cropped\/2024_11_18_62c85fb0b01b41aca29bg-51.jpg?height=643&width=624&top_left_y=1272&top_left_x=1090) Figure 2","tier":0} +{"problem_type":"Geometry","problem_label":"G1","problem":"The points $P$ and $Q$ are chosen on the side $B C$ of an acute-angled triangle $A B C$ so that $\\angle P A B=\\angle A C B$ and $\\angle Q A C=\\angle C B A$. The points $M$ and $N$ are taken on the rays $A P$ and $A Q$, respectively, so that $A P=P M$ and $A Q=Q N$. Prove that the lines $B M$ and $C N$ intersect on the circumcircle of the triangle $A B C$. (Georgia)","solution":"As in the previous solution, denote by $S$ the intersection point of the lines $B M$ and $N C$. Let moreover the circumcircle of the triangle $A B C$ intersect the lines $A P$ and $A Q$ again at $K$ and $L$, respectively (see Figure 2). Note that $\\angle L B C=\\angle L A C=\\angle C B A$ and similarly $\\angle K C B=\\angle K A B=\\angle B C A$. It implies that the lines $B L$ and $C K$ meet at a point $X$, being symmetric to the point $A$ with respect to the line $B C$. Since $A P=P M$ and $A Q=Q N$, it follows that $X$ lies on the line $M N$. Therefore, using Pascal's theorem for the hexagon $A L B S C K$, we infer that $S$ lies on the circumcircle of the triangle $A B C$, which finishes the proof. Comment. Both solutions can be modified to obtain a more general result, with the equalities $$ A P=P M \\quad \\text { and } \\quad A Q=Q N $$ replaced by $$ \\frac{A P}{P M}=\\frac{Q N}{A Q} $$","tier":0} +{"problem_type":"Geometry","problem_label":"G2","problem":"Let $A B C$ be a triangle. The points $K, L$, and $M$ lie on the segments $B C, C A$, and $A B$, respectively, such that the lines $A K, B L$, and $C M$ intersect in a common point. Prove that it is possible to choose two of the triangles $A L M, B M K$, and $C K L$ whose inradii sum up to at least the inradius of the triangle $A B C$. (Estonia)","solution":"Denote $$ a=\\frac{B K}{K C}, \\quad b=\\frac{C L}{L A}, \\quad c=\\frac{A M}{M B} . $$ By Ceva's theorem, $a b c=1$, so we may, without loss of generality, assume that $a \\geqslant 1$. Then at least one of the numbers $b$ or $c$ is not greater than 1 . Therefore at least one of the pairs $(a, b)$, $(b, c)$ has its first component not less than 1 and the second one not greater than 1 . Without loss of generality, assume that $1 \\leqslant a$ and $b \\leqslant 1$. Therefore, we obtain $b c \\leqslant 1$ and $1 \\leqslant c a$, or equivalently $$ \\frac{A M}{M B} \\leqslant \\frac{L A}{C L} \\quad \\text { and } \\quad \\frac{M B}{A M} \\leqslant \\frac{B K}{K C} . $$ The first inequality implies that the line passing through $M$ and parallel to $B C$ intersects the segment $A L$ at a point $X$ (see Figure 1). Therefore the inradius of the triangle $A L M$ is not less than the inradius $r_{1}$ of triangle $A M X$. Similarly, the line passing through $M$ and parallel to $A C$ intersects the segment $B K$ at a point $Y$, so the inradius of the triangle $B M K$ is not less than the inradius $r_{2}$ of the triangle $B M Y$. Thus, to complete our solution, it is enough to show that $r_{1}+r_{2} \\geqslant r$, where $r$ is the inradius of the triangle $A B C$. We prove that in fact $r_{1}+r_{2}=r$. ![](https:\/\/cdn.mathpix.com\/cropped\/2024_11_18_62c85fb0b01b41aca29bg-52.jpg?height=597&width=849&top_left_y=1369&top_left_x=621) Figure 1 Since $M X \\| B C$, the dilation with centre $A$ that takes $M$ to $B$ takes the incircle of the triangle $A M X$ to the incircle of the triangle $A B C$. Therefore $$ \\frac{r_{1}}{r}=\\frac{A M}{A B}, \\quad \\text { and similarly } \\quad \\frac{r_{2}}{r}=\\frac{M B}{A B} . $$ Adding these equalities gives $r_{1}+r_{2}=r$, as required. Comment. Alternatively, one can use Desargues' theorem instead of Ceva's theorem, as follows: The lines $A B, B C, C A$ dissect the plane into seven regions. One of them is bounded, and amongst the other six, three are two-sided and three are three-sided. Now define the points $P=B C \\cap L M$, $Q=C A \\cap M K$, and $R=A B \\cap K L$ (in the projective plane). By Desargues' theorem, the points $P$, $Q, R$ lie on a common line $\\ell$. This line intersects only unbounded regions. If we now assume (without loss of generality) that $P, Q$ and $R$ lie on $\\ell$ in that order, then one of the segments $P Q$ or $Q R$ lies inside a two-sided region. If, for example, this segment is $P Q$, then the triangles $A L M$ and $B M K$ will satisfy the statement of the problem for the same reason.","tier":0} +{"problem_type":"Geometry","problem_label":"G3","problem":"Let $\\Omega$ and $O$ be the circumcircle and the circumcentre of an acute-angled triangle $A B C$ with $A B>B C$. The angle bisector of $\\angle A B C$ intersects $\\Omega$ at $M \\neq B$. Let $\\Gamma$ be the circle with diameter $B M$. The angle bisectors of $\\angle A O B$ and $\\angle B O C$ intersect $\\Gamma$ at points $P$ and $Q$, respectively. The point $R$ is chosen on the line $P Q$ so that $B R=M R$. Prove that $B R \\| A C$. (Russia)","solution":"Let $K$ be the midpoint of $B M$, i.e., the centre of $\\Gamma$. Notice that $A B \\neq B C$ implies $K \\neq O$. Clearly, the lines $O M$ and $O K$ are the perpendicular bisectors of $A C$ and $B M$, respectively. Therefore, $R$ is the intersection point of $P Q$ and $O K$. Let $N$ be the second point of intersection of $\\Gamma$ with the line $O M$. Since $B M$ is a diameter of $\\Gamma$, the lines $B N$ and $A C$ are both perpendicular to $O M$. Hence $B N \\| A C$, and it suffices to prove that $B N$ passes through $R$. Our plan for doing this is to interpret the lines $B N, O K$, and $P Q$ as the radical axes of three appropriate circles. Let $\\omega$ be the circle with diameter $B O$. Since $\\angle B N O=\\angle B K O=90^{\\circ}$, the points $N$ and $K$ lie on $\\omega$. Next we show that the points $O, K, P$, and $Q$ are concyclic. To this end, let $D$ and $E$ be the midpoints of $B C$ and $A B$, respectively. Clearly, $D$ and $E$ lie on the rays $O Q$ and $O P$, respectively. By our assumptions about the triangle $A B C$, the points $B, E, O, K$, and $D$ lie in this order on $\\omega$. It follows that $\\angle E O R=\\angle E B K=\\angle K B D=\\angle K O D$, so the line $K O$ externally bisects the angle $P O Q$. Since the point $K$ is the centre of $\\Gamma$, it also lies on the perpendicular bisector of $P Q$. So $K$ coincides with the midpoint of the $\\operatorname{arc} P O Q$ of the circumcircle $\\gamma$ of triangle $P O Q$. Thus the lines $O K, B N$, and $P Q$ are pairwise radical axes of the circles $\\omega, \\gamma$, and $\\Gamma$. Hence they are concurrent at $R$, as required. ![](https:\/\/cdn.mathpix.com\/cropped\/2024_11_18_62c85fb0b01b41aca29bg-53.jpg?height=877&width=1429&top_left_y=1466&top_left_x=316)","tier":0} +{"problem_type":"Geometry","problem_label":"G4","problem":"Consider a fixed circle $\\Gamma$ with three fixed points $A, B$, and $C$ on it. Also, let us fix a real number $\\lambda \\in(0,1)$. For a variable point $P \\notin\\{A, B, C\\}$ on $\\Gamma$, let $M$ be the point on the segment $C P$ such that $C M=\\lambda \\cdot C P$. Let $Q$ be the second point of intersection of the circumcircles of the triangles $A M P$ and $B M C$. Prove that as $P$ varies, the point $Q$ lies on a fixed circle. (United Kingdom)","solution":"Throughout the solution, we denote by $\\Varangle(a, b)$ the directed angle between the lines $a$ and $b$. Let $D$ be the point on the segment $A B$ such that $B D=\\lambda \\cdot B A$. We will show that either $Q=D$, or $\\Varangle(D Q, Q B)=\\Varangle(A B, B C)$; this would mean that the point $Q$ varies over the constant circle through $D$ tangent to $B C$ at $B$, as required. Denote the circumcircles of the triangles $A M P$ and $B M C$ by $\\omega_{A}$ and $\\omega_{B}$, respectively. The lines $A P, B C$, and $M Q$ are pairwise radical axes of the circles $\\Gamma, \\omega_{A}$, and $\\omega_{B}$, thus either they are parallel, or they share a common point $X$. Assume that these lines are parallel (see Figure 1). Then the segments $A P, Q M$, and $B C$ have a common perpendicular bisector; the reflection in this bisector maps the segment $C P$ to $B A$, and maps $M$ to $Q$. Therefore, in this case $Q$ lies on $A B$, and $B Q \/ A B=C M \/ C P=$ $B D \/ A B$; so we have $Q=D$. ![](https:\/\/cdn.mathpix.com\/cropped\/2024_11_18_62c85fb0b01b41aca29bg-54.jpg?height=644&width=549&top_left_y=1300&top_left_x=225) Figure 1 ![](https:\/\/cdn.mathpix.com\/cropped\/2024_11_18_62c85fb0b01b41aca29bg-54.jpg?height=709&width=1040&top_left_y=1230&top_left_x=802) Figure 2 Now assume that the lines $A P, Q M$, and $B C$ are concurrent at some point $X$ (see Figure 2). Notice that the points $A, B, Q$, and $X$ lie on a common circle $\\Omega$ by Miquel's theorem applied to the triangle $X P C$. Let us denote by $Y$ the symmetric image of $X$ about the perpendicular bisector of $A B$. Clearly, $Y$ lies on $\\Omega$, and the triangles $Y A B$ and $\\triangle X B A$ are congruent. Moreover, the triangle $X P C$ is similar to the triangle $X B A$, so it is also similar to the triangle $Y A B$. Next, the points $D$ and $M$ correspond to each other in similar triangles $Y A B$ and $X P C$, since $B D \/ B A=C M \/ C P=\\lambda$. Moreover, the triangles $Y A B$ and $X P C$ are equi-oriented, so $\\Varangle(M X, X P)=\\Varangle(D Y, Y A)$. On the other hand, since the points $A, Q, X$, and $Y$ lie on $\\Omega$, we have $\\Varangle(Q Y, Y A)=\\Varangle(M X, X P)$. Therefore, $\\Varangle(Q Y, Y A)=\\Varangle(D Y, Y A)$, so the points $Y, D$, and $Q$ are collinear. Finally, we have $\\Varangle(D Q, Q B)=\\Varangle(Y Q, Q B)=\\Varangle(Y A, A B)=\\Varangle(A B, B X)=\\Varangle(A B, B C)$, as desired. Comment. In the original proposal, $\\lambda$ was supposed to be an arbitrary real number distinct from 0 and 1, and the point $M$ was defined by $\\overrightarrow{C M}=\\lambda \\cdot \\overrightarrow{C P}$. The Problem Selection Committee decided to add the restriction $\\lambda \\in(0,1)$ in order to avoid a large case distinction.","tier":0} +{"problem_type":"Geometry","problem_label":"G4","problem":"Consider a fixed circle $\\Gamma$ with three fixed points $A, B$, and $C$ on it. Also, let us fix a real number $\\lambda \\in(0,1)$. For a variable point $P \\notin\\{A, B, C\\}$ on $\\Gamma$, let $M$ be the point on the segment $C P$ such that $C M=\\lambda \\cdot C P$. Let $Q$ be the second point of intersection of the circumcircles of the triangles $A M P$ and $B M C$. Prove that as $P$ varies, the point $Q$ lies on a fixed circle. (United Kingdom)","solution":"As in the previous solution, we introduce the radical centre $X=A P \\cap B C \\cap M Q$ of the circles $\\omega_{A}, \\omega_{B}$, and $\\Gamma$. Next, we also notice that the points $A, Q, B$, and $X$ lie on a common circle $\\Omega$. If the point $P$ lies on the arc $B A C$ of $\\Gamma$, then the point $X$ is outside $\\Gamma$, thus the point $Q$ belongs to the ray $X M$, and therefore the points $P, A$, and $Q$ lie on the same side of $B C$. Otherwise, if $P$ lies on the arc $B C$ not containing $A$, then $X$ lies inside $\\Gamma$, so $M$ and $Q$ lie on different sides of $B C$; thus again $Q$ and $A$ lie on the same side of $B C$. So, in each case the points $Q$ and $A$ lie on the same side of $B C$. ![](https:\/\/cdn.mathpix.com\/cropped\/2024_11_18_62c85fb0b01b41aca29bg-55.jpg?height=715&width=1009&top_left_y=839&top_left_x=523) Figure 3 Now we prove that the ratio $$ \\frac{Q B}{\\sin \\angle Q B C}=\\frac{Q B}{Q X} \\cdot \\frac{Q X}{\\sin \\angle Q B X} $$ is constant. Since the points $A, Q, B$, and $X$ are concyclic, we have $$ \\frac{Q X}{\\sin \\angle Q B X}=\\frac{A X}{\\sin \\angle A B C} $$ Next, since the points $B, Q, M$, and $C$ are concyclic, the triangles $X B Q$ and $X M C$ are similar, so $$ \\frac{Q B}{Q X}=\\frac{C M}{C X}=\\lambda \\cdot \\frac{C P}{C X} $$ Analogously, the triangles $X C P$ and $X A B$ are also similar, so $$ \\frac{C P}{C X}=\\frac{A B}{A X} $$ Therefore, we obtain $$ \\frac{Q B}{\\sin \\angle Q B C}=\\lambda \\cdot \\frac{A B}{A X} \\cdot \\frac{A X}{\\sin \\angle A B C}=\\lambda \\cdot \\frac{A B}{\\sin \\angle A B C} $$ so this ratio is indeed constant. Thus the circle passing through $Q$ and tangent to $B C$ at $B$ is also constant, and $Q$ varies over this fixed circle. Comment. It is not hard to guess that the desired circle should be tangent to $B C$ at $B$. Indeed, the second paragraph of this solution shows that this circle lies on one side of $B C$; on the other hand, in the limit case $P=B$, the point $Q$ also coincides with $B$.","tier":0} +{"problem_type":"Geometry","problem_label":"G4","problem":"Consider a fixed circle $\\Gamma$ with three fixed points $A, B$, and $C$ on it. Also, let us fix a real number $\\lambda \\in(0,1)$. For a variable point $P \\notin\\{A, B, C\\}$ on $\\Gamma$, let $M$ be the point on the segment $C P$ such that $C M=\\lambda \\cdot C P$. Let $Q$ be the second point of intersection of the circumcircles of the triangles $A M P$ and $B M C$. Prove that as $P$ varies, the point $Q$ lies on a fixed circle. (United Kingdom)","solution":"Let us perform an inversion centred at $C$. Denote by $X^{\\prime}$ the image of a point $X$ under this inversion. The circle $\\Gamma$ maps to the line $\\Gamma^{\\prime}$ passing through the constant points $A^{\\prime}$ and $B^{\\prime}$, and containing the variable point $P^{\\prime}$. By the problem condition, the point $M$ varies over the circle $\\gamma$ which is the homothetic image of $\\Gamma$ with centre $C$ and coefficient $\\lambda$. Thus $M^{\\prime}$ varies over the constant line $\\gamma^{\\prime} \\| A^{\\prime} B^{\\prime}$ which is the homothetic image of $A^{\\prime} B^{\\prime}$ with centre $C$ and coefficient $1 \/ \\lambda$, and $M=\\gamma^{\\prime} \\cap C P^{\\prime}$. Next, the circumcircles $\\omega_{A}$ and $\\omega_{B}$ of the triangles $A M P$ and $B M C$ map to the circumcircle $\\omega_{A}^{\\prime}$ of the triangle $A^{\\prime} M^{\\prime} P^{\\prime}$ and to the line $B^{\\prime} M^{\\prime}$, respectively; the point $Q$ thus maps to the second point of intersection of $B^{\\prime} M^{\\prime}$ with $\\omega_{A}^{\\prime}$ (see Figure 4). ![](https:\/\/cdn.mathpix.com\/cropped\/2024_11_18_62c85fb0b01b41aca29bg-56.jpg?height=675&width=1029&top_left_y=913&top_left_x=519) Figure 4 Let $J$ be the (constant) common point of the lines $\\gamma^{\\prime}$ and $C A^{\\prime}$, and let $\\ell$ be the (constant) line through $J$ parallel to $C B^{\\prime}$. Let $V$ be the common point of the lines $\\ell$ and $B^{\\prime} M^{\\prime}$. Applying Pappus' theorem to the triples $\\left(C, J, A^{\\prime}\\right)$ and $\\left(V, B^{\\prime}, M^{\\prime}\\right)$ we get that the points $C B^{\\prime} \\cap J V$, $J M^{\\prime} \\cap A^{\\prime} B^{\\prime}$, and $C M^{\\prime} \\cap A^{\\prime} V$ are collinear. The first two of these points are ideal, hence so is the third, which means that $C M^{\\prime} \\| A^{\\prime} V$. Now we have $\\Varangle\\left(Q^{\\prime} A^{\\prime}, A^{\\prime} P^{\\prime}\\right)=\\Varangle\\left(Q^{\\prime} M^{\\prime}, M^{\\prime} P^{\\prime}\\right)=\\angle\\left(V M^{\\prime}, A^{\\prime} V\\right)$, which means that the triangles $B^{\\prime} Q^{\\prime} A^{\\prime}$ and $B^{\\prime} A^{\\prime} V$ are similar, and $\\left(B^{\\prime} A^{\\prime}\\right)^{2}=B^{\\prime} Q^{\\prime} \\cdot B^{\\prime} V$. Thus $Q^{\\prime}$ is the image of $V$ under the second (fixed) inversion with centre $B^{\\prime}$ and radius $B^{\\prime} A^{\\prime}$. Since $V$ varies over the constant line $\\ell, Q^{\\prime}$ varies over some constant circle $\\Theta$. Thus, applying the first inversion back we get that $Q$ also varies over some fixed circle. One should notice that this last circle is not a line; otherwise $\\Theta$ would contain $C$, and thus $\\ell$ would contain the image of $C$ under the second inversion. This is impossible, since $C B^{\\prime} \\| \\ell$.","tier":0} +{"problem_type":"Geometry","problem_label":"G5","problem":"Let $A B C D$ be a convex quadrilateral with $\\angle B=\\angle D=90^{\\circ}$. Point $H$ is the foot of the perpendicular from $A$ to $B D$. The points $S$ and $T$ are chosen on the sides $A B$ and $A D$, respectively, in such a way that $H$ lies inside triangle $S C T$ and $$ \\angle S H C-\\angle B S C=90^{\\circ}, \\quad \\angle T H C-\\angle D T C=90^{\\circ} . $$ Prove that the circumcircle of triangle $S H T$ is tangent to the line $B D$. (Iran)","solution":"Let the line passing through $C$ and perpendicular to the line $S C$ intersect the line $A B$ at $Q$ (see Figure 1). Then $$ \\angle S Q C=90^{\\circ}-\\angle B S C=180^{\\circ}-\\angle S H C, $$ which implies that the points $C, H, S$, and $Q$ lie on a common circle. Moreover, since $S Q$ is a diameter of this circle, we infer that the circumcentre $K$ of triangle $S H C$ lies on the line $A B$. Similarly, we prove that the circumcentre $L$ of triangle $C H T$ lies on the line $A D$. ![](https:\/\/cdn.mathpix.com\/cropped\/2024_11_18_62c85fb0b01b41aca29bg-57.jpg?height=795&width=1052&top_left_y=959&top_left_x=516) Figure 1 In order to prove that the circumcircle of triangle $S H T$ is tangent to $B D$, it suffices to show that the perpendicular bisectors of $H S$ and $H T$ intersect on the line $A H$. However, these two perpendicular bisectors coincide with the angle bisectors of angles $A K H$ and $A L H$. Therefore, in order to complete the solution, it is enough (by the bisector theorem) to show that $$ \\frac{A K}{K H}=\\frac{A L}{L H} $$ We present two proofs of this equality. First proof. Let the lines $K L$ and $H C$ intersect at $M$ (see Figure 2). Since $K H=K C$ and $L H=L C$, the points $H$ and $C$ are symmetric to each other with respect to the line $K L$. Therefore $M$ is the midpoint of $H C$. Denote by $O$ the circumcentre of quadrilateral $A B C D$. Then $O$ is the midpoint of $A C$. Therefore we have $O M \\| A H$ and hence $O M \\perp B D$. This together with the equality $O B=O D$ implies that $O M$ is the perpendicular bisector of $B D$ and therefore $B M=D M$. Since $C M \\perp K L$, the points $B, C, M$, and $K$ lie on a common circle with diameter $K C$. Similarly, the points $L, C, M$, and $D$ lie on a circle with diameter $L C$. Thus, using the sine law, we obtain $$ \\frac{A K}{A L}=\\frac{\\sin \\angle A L K}{\\sin \\angle A K L}=\\frac{D M}{C L} \\cdot \\frac{C K}{B M}=\\frac{C K}{C L}=\\frac{K H}{L H} $$ which finishes the proof of (1). ![](https:\/\/cdn.mathpix.com\/cropped\/2024_11_18_62c85fb0b01b41aca29bg-58.jpg?height=806&width=875&top_left_y=294&top_left_x=202) Figure 2 ![](https:\/\/cdn.mathpix.com\/cropped\/2024_11_18_62c85fb0b01b41aca29bg-58.jpg?height=744&width=752&top_left_y=359&top_left_x=1126) Figure 3 Second proof. If the points $A, H$, and $C$ are collinear, then $A K=A L$ and $K H=L H$, so the equality (1) follows. Assume therefore that the points $A, H$, and $C$ do not lie in a line and consider the circle $\\omega$ passing through them (see Figure 3). Since the quadrilateral $A B C D$ is cyclic, $$ \\angle B A C=\\angle B D C=90^{\\circ}-\\angle A D H=\\angle H A D . $$ Let $N \\neq A$ be the intersection point of the circle $\\omega$ and the angle bisector of $\\angle C A H$. Then $A N$ is also the angle bisector of $\\angle B A D$. Since $H$ and $C$ are symmetric to each other with respect to the line $K L$ and $H N=N C$, it follows that both $N$ and the centre of $\\omega$ lie on the line $K L$. This means that the circle $\\omega$ is an Apollonius circle of the points $K$ and $L$. This immediately yields (1). Comment. Either proof can be used to obtain the following generalised result: Let $A B C D$ be a convex quadrilateral and let $H$ be a point in its interior with $\\angle B A C=\\angle D A H$. The points $S$ and $T$ are chosen on the sides $A B$ and $A D$, respectively, in such a way that $H$ lies inside triangle SCT and $$ \\angle S H C-\\angle B S C=90^{\\circ}, \\quad \\angle T H C-\\angle D T C=90^{\\circ} . $$ Then the circumcentre of triangle SHT lies on the line AH (and moreover the circumcentre of triangle SCT lies on $A C$ ).","tier":0} +{"problem_type":"Geometry","problem_label":"G6","problem":"Let $A B C$ be a fixed acute-angled triangle. Consider some points $E$ and $F$ lying on the sides $A C$ and $A B$, respectively, and let $M$ be the midpoint of $E F$. Let the perpendicular bisector of $E F$ intersect the line $B C$ at $K$, and let the perpendicular bisector of $M K$ intersect the lines $A C$ and $A B$ at $S$ and $T$, respectively. We call the pair $(E, F)$ interesting, if the quadrilateral $K S A T$ is cyclic. Suppose that the pairs $\\left(E_{1}, F_{1}\\right)$ and $\\left(E_{2}, F_{2}\\right)$ are interesting. Prove that $$ \\frac{E_{1} E_{2}}{A B}=\\frac{F_{1} F_{2}}{A C} . $$","solution":"For any interesting pair $(E, F)$, we will say that the corresponding triangle $E F K$ is also interesting. Let $E F K$ be an interesting triangle. Firstly, we prove that $\\angle K E F=\\angle K F E=\\angle A$, which also means that the circumcircle $\\omega_{1}$ of the triangle $A E F$ is tangent to the lines $K E$ and $K F$. Denote by $\\omega$ the circle passing through the points $K, S, A$, and $T$. Let the line $A M$ intersect the line $S T$ and the circle $\\omega$ (for the second time) at $N$ and $L$, respectively (see Figure 1). Since $E F \\| T S$ and $M$ is the midpoint of $E F, N$ is the midpoint of $S T$. Moreover, since $K$ and $M$ are symmetric to each other with respect to the line $S T$, we have $\\angle K N S=\\angle M N S=$ $\\angle L N T$. Thus the points $K$ and $L$ are symmetric to each other with respect to the perpendicular bisector of $S T$. Therefore $K L \\| S T$. Let $G$ be the point symmetric to $K$ with respect to $N$. Then $G$ lies on the line $E F$, and we may assume that it lies on the ray $M F$. One has $$ \\angle K G E=\\angle K N S=\\angle S N M=\\angle K L A=180^{\\circ}-\\angle K S A $$ (if $K=L$, then the angle $K L A$ is understood to be the angle between $A L$ and the tangent to $\\omega$ at $L$ ). This means that the points $K, G, E$, and $S$ are concyclic. Now, since $K S G T$ is a parallelogram, we obtain $\\angle K E F=\\angle K S G=180^{\\circ}-\\angle T K S=\\angle A$. Since $K E=K F$, we also have $\\angle K F E=\\angle K E F=\\angle A$. After having proved this fact, one may finish the solution by different methods. ![](https:\/\/cdn.mathpix.com\/cropped\/2024_11_18_62c85fb0b01b41aca29bg-59.jpg?height=684&width=829&top_left_y=1874&top_left_x=245) Figure 1 ![](https:\/\/cdn.mathpix.com\/cropped\/2024_11_18_62c85fb0b01b41aca29bg-59.jpg?height=612&width=730&top_left_y=1944&top_left_x=1117) Figure 2 First method. We have just proved that all interesting triangles are similar to each other. This allows us to use the following lemma. Lemma. Let $A B C$ be an arbitrary triangle. Choose two points $E_{1}$ and $E_{2}$ on the side $A C$, two points $F_{1}$ and $F_{2}$ on the side $A B$, and two points $K_{1}$ and $K_{2}$ on the side $B C$, in a way that the triangles $E_{1} F_{1} K_{1}$ and $E_{2} F_{2} K_{2}$ are similar. Then the six circumcircles of the triangles $A E_{i} F_{i}$, $B F_{i} K_{i}$, and $C E_{i} K_{i}(i=1,2)$ meet at a common point $Z$. Moreover, $Z$ is the centre of the spiral similarity that takes the triangle $E_{1} F_{1} K_{1}$ to the triangle $E_{2} F_{2} K_{2}$. Proof. Firstly, notice that for each $i=1,2$, the circumcircles of the triangles $A E_{i} F_{i}, B F_{i} K_{i}$, and $C K_{i} E_{i}$ have a common point $Z_{i}$ by Miquel's theorem. Moreover, we have $\\Varangle\\left(Z_{i} F_{i}, Z_{i} E_{i}\\right)=\\Varangle(A B, C A), \\quad \\Varangle\\left(Z_{i} K_{i}, Z_{i} F_{i}\\right)=\\Varangle(B C, A B), \\quad \\Varangle\\left(Z_{i} E_{i}, Z_{i} K_{i}\\right)=\\Varangle(C A, B C)$. This yields that the points $Z_{1}$ and $Z_{2}$ correspond to each other in similar triangles $E_{1} F_{1} K_{1}$ and $E_{2} F_{2} K_{2}$. Thus, if they coincide, then this common point is indeed the desired centre of a spiral similarity. Finally, in order to show that $Z_{1}=Z_{2}$, one may notice that $\\Varangle\\left(A B, A Z_{1}\\right)=\\Varangle\\left(E_{1} F_{1}, E_{1} Z_{1}\\right)=$ $\\Varangle\\left(E_{2} F_{2}, E_{2} Z_{2}\\right)=\\Varangle\\left(A B, A Z_{2}\\right)$ (see Figure 2). Similarly, one has $\\Varangle\\left(B C, B Z_{1}\\right)=\\Varangle\\left(B C, B Z_{2}\\right)$ and $\\Varangle\\left(C A, C Z_{1}\\right)=\\Varangle\\left(C A, C Z_{2}\\right)$. This yields $Z_{1}=Z_{2}$. Now, let $P$ and $Q$ be the feet of the perpendiculars from $B$ and $C$ onto $A C$ and $A B$, respectively, and let $R$ be the midpoint of $B C$ (see Figure 3). Then $R$ is the circumcentre of the cyclic quadrilateral $B C P Q$. Thus we obtain $\\angle A P Q=\\angle B$ and $\\angle R P C=\\angle C$, which yields $\\angle Q P R=\\angle A$. Similarly, we show that $\\angle P Q R=\\angle A$. Thus, all interesting triangles are similar to the triangle $P Q R$. ![](https:\/\/cdn.mathpix.com\/cropped\/2024_11_18_62c85fb0b01b41aca29bg-60.jpg?height=615&width=715&top_left_y=1326&top_left_x=316) Figure 3 ![](https:\/\/cdn.mathpix.com\/cropped\/2024_11_18_62c85fb0b01b41aca29bg-60.jpg?height=615&width=692&top_left_y=1326&top_left_x=1090) Figure 4 Denote now by $Z$ the common point of the circumcircles of $A P Q, B Q R$, and $C P R$. Let $E_{1} F_{1} K_{1}$ and $E_{2} F_{2} K_{2}$ be two interesting triangles. By the lemma, $Z$ is the centre of any spiral similarity taking one of the triangles $E_{1} F_{1} K_{1}, E_{2} F_{2} K_{2}$, and $P Q R$ to some other of them. Therefore the triangles $Z E_{1} E_{2}$ and $Z F_{1} F_{2}$ are similar, as well as the triangles $Z E_{1} F_{1}$ and $Z P Q$. Hence $$ \\frac{E_{1} E_{2}}{F_{1} F_{2}}=\\frac{Z E_{1}}{Z F_{1}}=\\frac{Z P}{Z Q} $$ Moreover, the equalities $\\angle A Z Q=\\angle A P Q=\\angle A B C=180^{\\circ}-\\angle Q Z R$ show that the point $Z$ lies on the line $A R$ (see Figure 4). Therefore the triangles $A Z P$ and $A C R$ are similar, as well as the triangles $A Z Q$ and $A B R$. This yields $$ \\frac{Z P}{Z Q}=\\frac{Z P}{R C} \\cdot \\frac{R B}{Z Q}=\\frac{A Z}{A C} \\cdot \\frac{A B}{A Z}=\\frac{A B}{A C} $$ which completes the solution. Second method. Now we will start from the fact that $\\omega_{1}$ is tangent to the lines $K E$ and $K F$ (see Figure 5). We prove that if $(E, F)$ is an interesting pair, then $$ \\frac{A E}{A B}+\\frac{A F}{A C}=2 \\cos \\angle A $$ Let $Y$ be the intersection point of the segments $B E$ and $C F$. The points $B, K$, and $C$ are collinear, hence applying PASCAL's theorem to the degenerated hexagon AFFYEE, we infer that $Y$ lies on the circle $\\omega_{1}$. Denote by $Z$ the second intersection point of the circumcircle of the triangle $B F Y$ with the line $B C$ (see Figure 6). By Miquel's theorem, the points $C, Z, Y$, and $E$ are concyclic. Therefore we obtain $$ B F \\cdot A B+C E \\cdot A C=B Y \\cdot B E+C Y \\cdot C F=B Z \\cdot B C+C Z \\cdot B C=B C^{2} $$ On the other hand, $B C^{2}=A B^{2}+A C^{2}-2 A B \\cdot A C \\cos \\angle A$, by the cosine law. Hence $$ (A B-A F) \\cdot A B+(A C-A E) \\cdot A C=A B^{2}+A C^{2}-2 A B \\cdot A C \\cos \\angle A, $$ which simplifies to the desired equality (1). Let now $\\left(E_{1}, F_{1}\\right)$ and $\\left(E_{2}, F_{2}\\right)$ be two interesting pairs of points. Then we get $$ \\frac{A E_{1}}{A B}+\\frac{A F_{1}}{A C}=\\frac{A E_{2}}{A B}+\\frac{A F_{2}}{A C} $$ which gives the desired result. ![](https:\/\/cdn.mathpix.com\/cropped\/2024_11_18_62c85fb0b01b41aca29bg-61.jpg?height=709&width=915&top_left_y=1527&top_left_x=248) Figure 5 ![](https:\/\/cdn.mathpix.com\/cropped\/2024_11_18_62c85fb0b01b41aca29bg-61.jpg?height=569&width=615&top_left_y=1669&top_left_x=1229) Figure 6 Third method. Again, we make use of the fact that all interesting triangles are similar (and equi-oriented). Let us put the picture onto a complex plane such that $A$ is at the origin, and identify each point with the corresponding complex number. Let $E F K$ be any interesting triangle. The equalities $\\angle K E F=\\angle K F E=\\angle A$ yield that the ratio $\\nu=\\frac{K-E}{F-E}$ is the same for all interesting triangles. This in turn means that the numbers $E$, $F$, and $K$ satisfy the linear equation $$ K=\\mu E+\\nu F, \\quad \\text { where } \\quad \\mu=1-\\nu $$ Now let us choose the points $X$ and $Y$ on the rays $A B$ and $A C$, respectively, so that $\\angle C X A=\\angle A Y B=\\angle A=\\angle K E F$ (see Figure 7). Then each of the triangles $A X C$ and $Y A B$ is similar to any interesting triangle, which also means that $$ C=\\mu A+\\nu X=\\nu X \\quad \\text { and } \\quad B=\\mu Y+\\nu A=\\mu Y . $$ Moreover, one has $X \/ Y=\\overline{C \/ B}$. Since the points $E, F$, and $K$ lie on $A C, A B$, and $B C$, respectively, one gets $$ E=\\rho Y, \\quad F=\\sigma X, \\quad \\text { and } \\quad K=\\lambda B+(1-\\lambda) C $$ for some real $\\rho, \\sigma$, and $\\lambda$. In view of (3), the equation (2) now reads $\\lambda B+(1-\\lambda) C=K=$ $\\mu E+\\nu F=\\rho B+\\sigma C$, or $$ (\\lambda-\\rho) B=(\\sigma+\\lambda-1) C $$ Since the nonzero complex numbers $B$ and $C$ have different arguments, the coefficients in the brackets vanish, so $\\rho=\\lambda$ and $\\sigma=1-\\lambda$. Therefore, $$ \\frac{E}{Y}+\\frac{F}{X}=\\rho+\\sigma=1 $$ Now, if $\\left(E_{1}, F_{1}\\right)$ and $\\left(E_{2}, F_{2}\\right)$ are two distinct interesting pairs, one may apply (4) to both pairs. Subtracting, we get $$ \\frac{E_{1}-E_{2}}{Y}=\\frac{F_{2}-F_{1}}{X}, \\quad \\text { so } \\quad \\frac{E_{1}-E_{2}}{F_{2}-F_{1}}=\\frac{Y}{X}=\\frac{\\bar{B}}{\\bar{C}} $$ Taking absolute values provides the required result. ![](https:\/\/cdn.mathpix.com\/cropped\/2024_11_18_62c85fb0b01b41aca29bg-62.jpg?height=487&width=669&top_left_y=1664&top_left_x=696) Figure 7 Comment 1. One may notice that the triangle $P Q R$ is also interesting. Comment 2. In order to prove that $\\angle K E F=\\angle K F E=\\angle A$, one may also use the following well-known fact: Let $A E F$ be a triangle with $A E \\neq A F$, and let $K$ be the common point of the symmedian taken from $A$ and the perpendicular bisector of $E F$. Then the lines $K E$ and $K F$ are tangent to the circumcircle $\\omega_{1}$ of the triangle $A E F$. In this case, however, one needs to deal with the case $A E=A F$ separately.","tier":0} +{"problem_type":"Geometry","problem_label":"G6","problem":"Let $A B C$ be a fixed acute-angled triangle. Consider some points $E$ and $F$ lying on the sides $A C$ and $A B$, respectively, and let $M$ be the midpoint of $E F$. Let the perpendicular bisector of $E F$ intersect the line $B C$ at $K$, and let the perpendicular bisector of $M K$ intersect the lines $A C$ and $A B$ at $S$ and $T$, respectively. We call the pair $(E, F)$ interesting, if the quadrilateral $K S A T$ is cyclic. Suppose that the pairs $\\left(E_{1}, F_{1}\\right)$ and $\\left(E_{2}, F_{2}\\right)$ are interesting. Prove that $$ \\frac{E_{1} E_{2}}{A B}=\\frac{F_{1} F_{2}}{A C} . $$","solution":"Let $(E, F)$ be an interesting pair. This time we prove that $$ \\frac{A M}{A K}=\\cos \\angle A $$ As in Solution 1, we introduce the circle $\\omega$ passing through the points $K, S$, $A$, and $T$, together with the points $N$ and $L$ at which the line $A M$ intersect the line $S T$ and the circle $\\omega$ for the second time, respectively. Let moreover $O$ be the centre of $\\omega$ (see Figures 8 and 9). As in Solution 1, we note that $N$ is the midpoint of $S T$ and show that $K L \\| S T$, which implies $\\angle F A M=\\angle E A K$. ![](https:\/\/cdn.mathpix.com\/cropped\/2024_11_18_62c85fb0b01b41aca29bg-63.jpg?height=752&width=1009&top_left_y=749&top_left_x=266) Figure 8 ![](https:\/\/cdn.mathpix.com\/cropped\/2024_11_18_62c85fb0b01b41aca29bg-63.jpg?height=552&width=549&top_left_y=952&top_left_x=1279) Figure 9 Suppose now that $K \\neq L$ (see Figure 8). Then $K L \\| S T$, and consequently the lines $K M$ and $K L$ are perpendicular. It implies that the lines $L O$ and $K M$ meet at a point $X$ lying on the circle $\\omega$. Since the lines $O N$ and $X M$ are both perpendicular to the line $S T$, they are parallel to each other, and hence $\\angle L O N=\\angle L X K=\\angle M A K$. On the other hand, $\\angle O L N=\\angle M K A$, so we infer that triangles $N O L$ and $M A K$ are similar. This yields $$ \\frac{A M}{A K}=\\frac{O N}{O L}=\\frac{O N}{O T}=\\cos \\angle T O N=\\cos \\angle A $$ If, on the other hand, $K=L$, then the points $A, M, N$, and $K$ lie on a common line, and this line is the perpendicular bisector of $S T$ (see Figure 9). This implies that $A K$ is a diameter of $\\omega$, which yields $A M=2 O K-2 N K=2 O N$. So also in this case we obtain $$ \\frac{A M}{A K}=\\frac{2 O N}{2 O T}=\\cos \\angle T O N=\\cos \\angle A $$ Thus (5) is proved. Let $P$ and $Q$ be the feet of the perpendiculars from $B$ and $C$ onto $A C$ and $A B$, respectively (see Figure 10). We claim that the point $M$ lies on the line $P Q$. Consider now the composition of the dilatation with factor $\\cos \\angle A$ and centre $A$, and the reflection with respect to the angle bisector of $\\angle B A C$. This transformation is a similarity that takes $B, C$, and $K$ to $P, Q$, and $M$, respectively. Since $K$ lies on the line $B C$, the point $M$ lies on the line $P Q$. ![](https:\/\/cdn.mathpix.com\/cropped\/2024_11_18_62c85fb0b01b41aca29bg-64.jpg?height=715&width=918&top_left_y=219&top_left_x=586) Figure 10 Suppose that $E \\neq P$. Then also $F \\neq Q$, and by Menelaus' theorem, we obtain $$ \\frac{A Q}{F Q} \\cdot \\frac{F M}{E M} \\cdot \\frac{E P}{A P}=1 $$ Using the similarity of the triangles $A P Q$ and $A B C$, we infer that $$ \\frac{E P}{F Q}=\\frac{A P}{A Q}=\\frac{A B}{A C}, \\quad \\text { and hence } \\quad \\frac{E P}{A B}=\\frac{F Q}{A C} $$ The last equality holds obviously also in case $E=P$, because then $F=Q$. Moreover, since the line $P Q$ intersects the segment $E F$, we infer that the point $E$ lies on the segment $A P$ if and only if the point $F$ lies outside of the segment $A Q$. Let now $\\left(E_{1}, F_{1}\\right)$ and $\\left(E_{2}, F_{2}\\right)$ be two interesting pairs. Then we obtain $$ \\frac{E_{1} P}{A B}=\\frac{F_{1} Q}{A C} \\quad \\text { and } \\quad \\frac{E_{2} P}{A B}=\\frac{F_{2} Q}{A C} . $$ If $P$ lies between the points $E_{1}$ and $E_{2}$, we add the equalities above, otherwise we subtract them. In any case we obtain $$ \\frac{E_{1} E_{2}}{A B}=\\frac{F_{1} F_{2}}{A C} $$ which completes the solution.","tier":0} +{"problem_type":"Geometry","problem_label":"G7","problem":"Let $A B C$ be a triangle with circumcircle $\\Omega$ and incentre $I$. Let the line passing through $I$ and perpendicular to $C I$ intersect the segment $B C$ and the $\\operatorname{arc} B C($ not containing $A)$ of $\\Omega$ at points $U$ and $V$, respectively. Let the line passing through $U$ and parallel to $A I$ intersect $A V$ at $X$, and let the line passing through $V$ and parallel to $A I$ intersect $A B$ at $Y$. Let $W$ and $Z$ be the midpoints of $A X$ and $B C$, respectively. Prove that if the points $I, X$, and $Y$ are collinear, then the points $I, W$, and $Z$ are also collinear.","solution":"We start with some general observations. Set $\\alpha=\\angle A \/ 2, \\beta=\\angle B \/ 2, \\gamma=\\angle C \/ 2$. Then obviously $\\alpha+\\beta+\\gamma=90^{\\circ}$. Since $\\angle U I C=90^{\\circ}$, we obtain $\\angle I U C=\\alpha+\\beta$. Therefore $\\angle B I V=\\angle I U C-\\angle I B C=\\alpha=\\angle B A I=\\angle B Y V$, which implies that the points $B, Y, I$, and $V$ lie on a common circle (see Figure 1). Assume now that the points $I, X$ and $Y$ are collinear. We prove that $\\angle Y I A=90^{\\circ}$. Let the line $X U$ intersect $A B$ at $N$. Since the lines $A I, U X$, and $V Y$ are parallel, we get $$ \\frac{N X}{A I}=\\frac{Y N}{Y A}=\\frac{V U}{V I}=\\frac{X U}{A I} $$ implying $N X=X U$. Moreover, $\\angle B I U=\\alpha=\\angle B N U$. This implies that the quadrilateral BUIN is cyclic, and since $B I$ is the angle bisector of $\\angle U B N$, we infer that $N I=U I$. Thus in the isosceles triangle $N I U$, the point $X$ is the midpoint of the base $N U$. This gives $\\angle I X N=90^{\\circ}$, i.e., $\\angle Y I A=90^{\\circ}$. ![](https:\/\/cdn.mathpix.com\/cropped\/2024_11_18_62c85fb0b01b41aca29bg-65.jpg?height=880&width=966&top_left_y=1256&top_left_x=565) Figure 1 Let $S$ be the midpoint of the segment $V C$. Let moreover $T$ be the intersection point of the lines $A X$ and $S I$, and set $x=\\angle B A V=\\angle B C V$. Since $\\angle C I A=90^{\\circ}+\\beta$ and $S I=S C$, we obtain $$ \\angle T I A=180^{\\circ}-\\angle A I S=90^{\\circ}-\\beta-\\angle C I S=90^{\\circ}-\\beta-\\gamma-x=\\alpha-x=\\angle T A I, $$ which implies that $T I=T A$. Therefore, since $\\angle X I A=90^{\\circ}$, the point $T$ is the midpoint of $A X$, i.e., $T=W$. To complete our solution, it remains to show that the intersection point of the lines $I S$ and $B C$ coincide with the midpoint of the segment $B C$. But since $S$ is the midpoint of the segment $V C$, it suffices to show that the lines $B V$ and $I S$ are parallel. Since the quadrilateral $B Y I V$ is cyclic, $\\angle V B I=\\angle V Y I=\\angle Y I A=90^{\\circ}$. This implies that $B V$ is the external angle bisector of the angle $A B C$, which yields $\\angle V A C=\\angle V C A$. Therefore $2 \\alpha-x=2 \\gamma+x$, which gives $\\alpha=\\gamma+x$. Hence $\\angle S C I=\\alpha$, so $\\angle V S I=2 \\alpha$. On the other hand, $\\angle B V C=180^{\\circ}-\\angle B A C=180^{\\circ}-2 \\alpha$, which implies that the lines $B V$ and $I S$ are parallel. This completes the solution.","tier":0} +{"problem_type":"Geometry","problem_label":"G7","problem":"Let $A B C$ be a triangle with circumcircle $\\Omega$ and incentre $I$. Let the line passing through $I$ and perpendicular to $C I$ intersect the segment $B C$ and the $\\operatorname{arc} B C($ not containing $A)$ of $\\Omega$ at points $U$ and $V$, respectively. Let the line passing through $U$ and parallel to $A I$ intersect $A V$ at $X$, and let the line passing through $V$ and parallel to $A I$ intersect $A B$ at $Y$. Let $W$ and $Z$ be the midpoints of $A X$ and $B C$, respectively. Prove that if the points $I, X$, and $Y$ are collinear, then the points $I, W$, and $Z$ are also collinear.","solution":"As in $$ \\angle B A V=\\angle C A E $$ Proof. Let $\\rho$ be the composition of the inversion with centre $A$ and radius $\\sqrt{A B \\cdot A C}$, and the symmetry with respect to $A I$. Clearly, $\\rho$ interchanges $B$ and $C$. Let $J$ be the excentre of the triangle $A B C$ opposite to $A$ (see Figure 2). Then we have $\\angle J A C=\\angle B A I$ and $\\angle J C A=90^{\\circ}+\\gamma=\\angle B I A$, so the triangles $A C J$ and $A I B$ are similar, and therefore $A B \\cdot A C=A I \\cdot A J$. This means that $\\rho$ interchanges $I$ and $J$. Moreover, since $Y$ lies on $A B$ and $\\angle A I Y=90^{\\circ}$, the point $Y^{\\prime}=\\rho(Y)$ lies on $A C$, and $\\angle J Y^{\\prime} A=90^{\\circ}$. Thus $\\rho$ maps the circumcircle $\\gamma$ of the triangle $B Y I$ to a circle $\\gamma^{\\prime}$ with diameter $J C$. Finally, since $V$ lies on both $\\Gamma$ and $\\gamma$, the point $V^{\\prime}=\\rho(V)$ lies on the line $\\rho(\\Gamma)=A B$ as well as on $\\gamma^{\\prime}$, which in turn means that $V^{\\prime}=E$. This implies the desired result. ![](https:\/\/cdn.mathpix.com\/cropped\/2024_11_18_62c85fb0b01b41aca29bg-66.jpg?height=709&width=572&top_left_y=1713&top_left_x=248) Figure 2 ![](https:\/\/cdn.mathpix.com\/cropped\/2024_11_18_62c85fb0b01b41aca29bg-66.jpg?height=872&width=989&top_left_y=1549&top_left_x=822) Figure 3 Now we turn to the solution of the problem. Assume that the incircle $\\omega_{1}$ of the triangle $A B C$ is tangent to $B C$ at $D$, and let the excircle $\\omega_{2}$ of the triangle $A B C$ opposite to the vertex $A$ touch the side $B C$ at $E$ (see Figure 3). The homothety with centre $A$ that takes $\\omega_{2}$ to $\\omega_{1}$ takes the point $E$ to some point $F$, and the tangent to $\\omega_{1}$ at $F$ is parallel to $B C$. Therefore $D F$ is a diameter of $\\omega_{1}$. Moreover, $Z$ is the midpoint of $D E$. This implies that the lines $I Z$ and $F E$ are parallel. Let $K=Y I \\cap A E$. Since $\\angle Y I A=90^{\\circ}$, the lemma yields that $I$ is the midpoint of $X K$. This implies that the segments $I W$ and $A K$ are parallel. Therefore, the points $W, I$ and $Z$ are collinear. Comment 1. The properties $\\angle Y I A=90^{\\circ}$ and $V A=V C$ can be established in various ways. The main difficulty of the problem seems to find out how to use these properties in connection to the points $W$ and $Z$. In Solution 2 this principal part is more or less covered by the lemma, for which we have presented a direct proof. On the other hand, this lemma appears to be a combination of two well-known facts; let us formulate them in terms of the lemma statement. Let the line $I Y$ intersect $A C$ at $P$ (see Figure 4). The first fact states that the circumcircle $\\omega$ of the triangle $V Y P$ is tangent to the segments $A B$ and $A C$, as well as to the circle $\\Gamma$. The second fact states that for such a circle, the angles $B A V$ and $C A E$ are equal. The awareness of this lemma may help a lot in solving this problem; so the Jury might also consider a variation of the proposed problem, for which the lemma does not seem to be useful; see Comment 3. ![](https:\/\/cdn.mathpix.com\/cropped\/2024_11_18_62c85fb0b01b41aca29bg-67.jpg?height=946&width=1689&top_left_y=1149&top_left_x=189) Comment 2. The proposed problem stated the equivalence: the point $I$ lies on the line $X Y$ if and only if $I$ lies on the line $W Z$. Here we sketch the proof of the \"if\" part (see Figure 5). As in Solution 2 , let $B C$ touch the circles $\\omega_{1}$ and $\\omega_{2}$ at $D$ and $E$, respectively. Since $I Z \\| A E$ and $W$ lies on $I Z$, the line $D X$ is also parallel to $A E$. Therefore, the triangles $X U P$ and $A I Q$ are similar. Moreover, the line $D X$ is symmetric to $A E$ with respect to $I$, so $I P=I Q$, where $P=U V \\cap X D$ and $Q=U V \\cap A E$. Thus we obtain $$ \\frac{U V}{V I}=\\frac{U X}{I A}=\\frac{U P}{I Q}=\\frac{U P}{I P} $$ So the pairs $I U$ and $P V$ are harmonic conjugates, and since $\\angle U D I=90^{\\circ}$, we get $\\angle V D B=\\angle B D X=$ $\\angle B E A$. Therefore the point $V^{\\prime}$ symmetric to $V$ with respect to the perpendicular bisector of $B C$ lies on the line $A E$. So we obtain $\\angle B A V=\\angle C A E$. The rest can be obtained by simply reversing the arguments in Solution 2 . The points $B, V, I$, and $Y$ are concyclic. The lemma implies that $\\angle Y I A=90^{\\circ}$. Moreover, the points $B, U, I$, and $N$, where $N=U X \\cap A B$, lie on a common circle, so $I N=I U$. Since $I Y \\perp U N$, the point $X^{\\prime}=I Y \\cap U N$ is the midpoint of $U N$. But in the trapezoid $A Y V I$, the line $X U$ is parallel to the sides $A I$ and $Y V$, so $N X=U X^{\\prime}$. This yields $X=X^{\\prime}$. The reasoning presented in Solution 1 can also be reversed, but it requires a lot of technicalities. Therefore the Problem Selection Committee proposes to consider only the \"only if\" part of the original proposal, which is still challenging enough. Comment 3. The Jury might also consider the following variation of the proposed problem. Let $A B C$ be a triangle with circumcircle $\\Omega$ and incentre I. Let the line through I perpendicular to CI intersect the segment $B C$ and the arc $B C$ (not containing $A$ ) of $\\Omega$ at $U$ and $V$, respectively. Let the line through $U$ parallel to $A I$ intersect $A V$ at $X$. Prove that if the lines XI and AI are perpendicular, then the midpoint of the segment AC lies on the line XI (see Figure 6). ![](https:\/\/cdn.mathpix.com\/cropped\/2024_11_18_62c85fb0b01b41aca29bg-68.jpg?height=675&width=746&top_left_y=1273&top_left_x=184) Figure 6 ![](https:\/\/cdn.mathpix.com\/cropped\/2024_11_18_62c85fb0b01b41aca29bg-68.jpg?height=1012&width=980&top_left_y=930&top_left_x=892) Figure 7 Since the solution contains the arguments used above, we only sketch it. Let $N=X U \\cap A B$ (see Figure 7). Then $\\angle B N U=\\angle B A I=\\angle B I U$, so the points $B, U, I$, and $N$ lie on a common circle. Therefore $I U=I N$, and since $I X \\perp N U$, it follows that $N X=X U$. Now set $Y=X I \\cap A B$. The equality $N X=X U$ implies that $$ \\frac{V X}{V A}=\\frac{X U}{A I}=\\frac{N X}{A I}=\\frac{Y X}{Y I} $$ and therefore $Y V \\| A I$. Hence $\\angle B Y V=\\angle B A I=\\angle B I V$, so the points $B, V, I, Y$ are concyclic. Next we have $I Y \\perp Y V$, so $\\angle I B V=90^{\\circ}$. This implies that $B V$ is the external angle bisector of the angle $A B C$, which gives $\\angle V A C=\\angle V C A$. So in order to show that $M=X I \\cap A C$ is the midpoint of $A C$, it suffices to prove that $\\angle V M C=90^{\\circ}$. But this follows immediately from the observation that the points $V, C, M$, and $I$ are concyclic, as $\\angle M I V=\\angle Y B V=180^{\\circ}-\\angle A C V$. The converse statement is also true, but its proof requires some technicalities as well.","tier":0} +{"problem_type":"Number Theory","problem_label":"N1","problem":"Let $n \\geqslant 2$ be an integer, and let $A_{n}$ be the set $$ A_{n}=\\left\\{2^{n}-2^{k} \\mid k \\in \\mathbb{Z}, 0 \\leqslant k2$, and take an integer $m>(n-2) 2^{n}+1$. If $m$ is even, then consider $$ \\frac{m}{2} \\geqslant \\frac{(n-2) 2^{n}+2}{2}=(n-2) 2^{n-1}+1>(n-3) 2^{n-1}+1 $$ By the induction hypothesis, there is a representation of the form $$ \\frac{m}{2}=\\left(2^{n-1}-2^{k_{1}}\\right)+\\left(2^{n-1}-2^{k_{2}}\\right)+\\cdots+\\left(2^{n-1}-2^{k_{r}}\\right) $$ for some $k_{i}$ with $0 \\leqslant k_{i}\\frac{(n-2) 2^{n}+1-\\left(2^{n}-1\\right)}{2}=(n-3) 2^{n-1}+1 $$ By the induction hypothesis, there is a representation of the form $$ \\frac{m-\\left(2^{n}-1\\right)}{2}=\\left(2^{n-1}-2^{k_{1}}\\right)+\\left(2^{n-1}-2^{k_{2}}\\right)+\\cdots+\\left(2^{n-1}-2^{k_{r}}\\right) $$ for some $k_{i}$ with $0 \\leqslant k_{i}2$, assume that there exist integers $a, b$ with $a \\geqslant 0, b \\geqslant 1$ and $a+by$ due to symmetry. Then the integer $n=x-y$ is positive and (1) may be rewritten as $$ \\sqrt[3]{7(y+n)^{2}-13(y+n) y+7 y^{2}}=n+1 $$ Raising this to the third power and simplifying the result one obtains $$ y^{2}+y n=n^{3}-4 n^{2}+3 n+1 . $$ To complete the square on the left hand side, we multiply by 4 and add $n^{2}$, thus getting $$ (2 y+n)^{2}=4 n^{3}-15 n^{2}+12 n+4=(n-2)^{2}(4 n+1) $$ This shows that the cases $n=1$ and $n=2$ are impossible, whence $n>2$, and $4 n+1$ is the square of the rational number $\\frac{2 y+n}{n-2}$. Consequently, it has to be a perfect square, and, since it is odd as well, there has to exist some nonnegative integer $m$ such that $4 n+1=(2 m+1)^{2}$, i.e. $$ n=m^{2}+m $$ Notice that $n>2$ entails $m \\geqslant 2$. Substituting the value of $n$ just found into the previous displayed equation we arrive at $$ \\left(2 y+m^{2}+m\\right)^{2}=\\left(m^{2}+m-2\\right)^{2}(2 m+1)^{2}=\\left(2 m^{3}+3 m^{2}-3 m-2\\right)^{2} . $$ Extracting square roots and taking $2 m^{3}+3 m^{2}-3 m-2=(m-1)\\left(2 m^{2}+5 m+2\\right)>0$ into account we derive $2 y+m^{2}+m=2 m^{3}+3 m^{2}-3 m-2$, which in turn yields $$ y=m^{3}+m^{2}-2 m-1 $$ Notice that $m \\geqslant 2$ implies that $y=\\left(m^{3}-1\\right)+(m-2) m$ is indeed positive, as it should be. In view of $x=y+n=y+m^{2}+m$ it also follows that $$ x=m^{3}+2 m^{2}-m-1, $$ and that this integer is positive as well. Comment. Alternatively one could ask to find all pairs $(x, y)$ of - not necessarily positive - integers solving (1). The answer to that question is a bit nicer than the answer above: the set of solutions are now described by $$ \\{x, y\\}=\\left\\{m^{3}+m^{2}-2 m-1, m^{3}+2 m^{2}-m-1\\right\\} $$ where $m$ varies through $\\mathbb{Z}$. This may be shown using essentially the same arguments as above. We finally observe that the pair $(x, y)=(1,1)$, that appears to be sporadic above, corresponds to $m=-1$.","tier":0} +{"problem_type":"Number Theory","problem_label":"N3","problem":"A coin is called a Cape Town coin if its value is $1 \/ n$ for some positive integer $n$. Given a collection of Cape Town coins of total value at most $99+\\frac{1}{2}$, prove that it is possible to split this collection into at most 100 groups each of total value at most 1. (Luxembourg)","solution":"We will show that for every positive integer $N$ any collection of Cape Town coins of total value at most $N-\\frac{1}{2}$ can be split into $N$ groups each of total value at most 1 . The problem statement is a particular case for $N=100$. We start with some preparations. If several given coins together have a total value also of the form $\\frac{1}{k}$ for a positive integer $k$, then we may merge them into one new coin. Clearly, if the resulting collection can be split in the required way then the initial collection can also be split. After each such merging, the total number of coins decreases, thus at some moment we come to a situation when no more merging is possible. At this moment, for every even $k$ there is at most one coin of value $\\frac{1}{k}$ (otherwise two such coins may be merged), and for every odd $k>1$ there are at most $k-1$ coins of value $\\frac{1}{k}$ (otherwise $k$ such coins may also be merged). Now, clearly, each coin of value 1 should form a single group; if there are $d$ such coins then we may remove them from the collection and replace $N$ by $N-d$. So from now on we may assume that there are no coins of value 1. Finally, we may split all the coins in the following way. For each $k=1,2, \\ldots, N$ we put all the coins of values $\\frac{1}{2 k-1}$ and $\\frac{1}{2 k}$ into a group $G_{k}$; the total value of $G_{k}$ does not exceed $$ (2 k-2) \\cdot \\frac{1}{2 k-1}+\\frac{1}{2 k}<1 $$ It remains to distribute the \"small\" coins of values which are less than $\\frac{1}{2 N}$; we will add them one by one. In each step, take any remaining small coin. The total value of coins in the groups at this moment is at most $N-\\frac{1}{2}$, so there exists a group of total value at most $\\frac{1}{N}\\left(N-\\frac{1}{2}\\right)=1-\\frac{1}{2 N}$; thus it is possible to put our small coin into this group. Acting so, we will finally distribute all the coins. Comment 1. The algorithm may be modified, at least the step where one distributes the coins of values $\\geqslant \\frac{1}{2 N}$. One different way is to put into $G_{k}$ all the coins of values $\\frac{1}{(2 k-1) 2^{s}}$ for all integer $s \\geqslant 0$. One may easily see that their total value also does not exceed 1. Comment 2. The original proposal also contained another part, suggesting to show that a required splitting may be impossible if the total value of coins is at most 100 . There are many examples of such a collection, e.g. one may take 98 coins of value 1 , one coin of value $\\frac{1}{2}$, two coins of value $\\frac{1}{3}$, and four coins of value $\\frac{1}{5}$. The Problem Selection Committee thinks that this part is less suitable for the competition.","tier":0} +{"problem_type":"Number Theory","problem_label":"N4","problem":"Let $n>1$ be a given integer. Prove that infinitely many terms of the sequence $\\left(a_{k}\\right)_{k \\geqslant 1}$, defined by $$ a_{k}=\\left\\lfloor\\frac{n^{k}}{k}\\right\\rfloor $$ are odd. (For a real number $x,\\lfloor x\\rfloor$ denotes the largest integer not exceeding $x$.) (Hong Kong)","solution":"If $n$ is odd, let $k=n^{m}$ for $m=1,2, \\ldots$. Then $a_{k}=n^{n^{m}-m}$, which is odd for each $m$. Henceforth, assume that $n$ is even, say $n=2 t$ for some integer $t \\geqslant 1$. Then, for any $m \\geqslant 2$, the integer $n^{2^{m}}-2^{m}=2^{m}\\left(2^{2^{m}-m} \\cdot t^{2^{m}}-1\\right)$ has an odd prime divisor $p$, since $2^{m}-m>1$. Then, for $k=p \\cdot 2^{m}$, we have $$ n^{k}=\\left(n^{2^{m}}\\right)^{p} \\equiv\\left(2^{m}\\right)^{p}=\\left(2^{p}\\right)^{m} \\equiv 2^{m} $$ where the congruences are taken modulo $p\\left(\\right.$ recall that $2^{p} \\equiv 2(\\bmod p)$, by Fermat's little theorem). Also, from $n^{k}-2^{m}m$ ). Note that for different values of $m$, we get different values of $k$, due to the different powers of 2 in the prime factorisation of $k$.","tier":0} +{"problem_type":"Number Theory","problem_label":"N4","problem":"Let $n>1$ be a given integer. Prove that infinitely many terms of the sequence $\\left(a_{k}\\right)_{k \\geqslant 1}$, defined by $$ a_{k}=\\left\\lfloor\\frac{n^{k}}{k}\\right\\rfloor $$ are odd. (For a real number $x,\\lfloor x\\rfloor$ denotes the largest integer not exceeding $x$.) (Hong Kong)","solution":"Treat the (trivial) case when $n$ is odd as in Now assume that $n$ is even and $n>2$. Let $p$ be a prime divisor of $n-1$. Proceed by induction on $i$ to prove that $p^{i+1}$ is a divisor of $n^{p^{i}}-1$ for every $i \\geqslant 0$. The case $i=0$ is true by the way in which $p$ is chosen. Suppose the result is true for some $i \\geqslant 0$. The factorisation $$ n^{p^{i+1}}-1=\\left(n^{p^{i}}-1\\right)\\left[n^{p^{i}(p-1)}+n^{p^{i}(p-2)}+\\cdots+n^{p^{i}}+1\\right], $$ together with the fact that each of the $p$ terms between the square brackets is congruent to 1 modulo $p$, implies that the result is also true for $i+1$. Hence $\\left\\lfloor\\frac{n^{p^{i}}}{p^{i}}\\right\\rfloor=\\frac{n^{p^{i}}-1}{p^{i}}$, an odd integer for each $i \\geqslant 1$. Finally, we consider the case $n=2$. We observe that $3 \\cdot 4^{i}$ is a divisor of $2^{3 \\cdot 4^{i}}-4^{i}$ for every $i \\geqslant 1$ : Trivially, $4^{i}$ is a divisor of $2^{3 \\cdot 4^{i}}-4^{i}$, since $3 \\cdot 4^{i}>2 i$. Furthermore, since $2^{3 \\cdot 4^{i}}$ and $4^{i}$ are both congruent to 1 modulo 3, we have $3 \\mid 2^{3 \\cdot 4^{i}}-4^{i}$. Hence, $\\left\\lfloor\\frac{2^{3 \\cdot 4^{i}}}{3 \\cdot 4^{i}}\\right\\rfloor=\\frac{2^{3 \\cdot 4^{i}}-4^{i}}{3 \\cdot 4^{i}}=\\frac{2^{3 \\cdot 4^{i}-2 i}-1}{3}$, which is odd for every $i \\geqslant 1$. Comment. The case $n$ even and $n>2$ can also be solved by recursively defining the sequence $\\left(k_{i}\\right)_{i \\geqslant 1}$ by $k_{1}=1$ and $k_{i+1}=n^{k_{i}}-1$ for $i \\geqslant 1$. Then $\\left(k_{i}\\right)$ is strictly increasing and it follows (by induction on $i$ ) that $k_{i} \\mid n^{k_{i}}-1$ for all $i \\geqslant 1$, so the $k_{i}$ are as desired. The case $n=2$ can also be solved as follows: Let $i \\geqslant 2$. By Bertrand's postulate, there exists a prime number $p$ such that $2^{2^{i}-1}

1$ be a given integer. Prove that infinitely many terms of the sequence $\\left(a_{k}\\right)_{k \\geqslant 1}$, defined by $$ a_{k}=\\left\\lfloor\\frac{n^{k}}{k}\\right\\rfloor $$ are odd. (For a real number $x,\\lfloor x\\rfloor$ denotes the largest integer not exceeding $x$.) (Hong Kong)","solution":"Treat the (trivial) case when $n$ is odd as in Let $n$ be even, and let $p$ be a prime divisor of $n+1$. Define the sequence $\\left(a_{i}\\right)_{i \\geqslant 1}$ by $$ a_{i}=\\min \\left\\{a \\in \\mathbb{Z}_{>0}: 2^{i} \\text { divides } a p+1\\right\\} $$ Recall that there exists $a$ with $1 \\leqslant a<2^{i}$ such that $a p \\equiv-1\\left(\\bmod 2^{i}\\right)$, so each $a_{i}$ satisfies $1 \\leqslant a_{i}<2^{i}$. This implies that $a_{i} p+1

2$, and we let $a$ and $b$ be positive integers such that $x^{p-1}+y=p^{a}$ and $x+y^{p-1}=p^{b}$. Assume further, without loss of generality, that $x \\leqslant y$, so that $p^{a}=x^{p-1}+y \\leqslant x+y^{p-1}=p^{b}$, which means that $a \\leqslant b$ (and thus $\\left.p^{a} \\mid p^{b}\\right)$. Now we have $$ p^{b}=y^{p-1}+x=\\left(p^{a}-x^{p-1}\\right)^{p-1}+x . $$ We take this equation modulo $p^{a}$ and take into account that $p-1$ is even, which gives us $$ 0 \\equiv x^{(p-1)^{2}}+x \\quad\\left(\\bmod p^{a}\\right) $$ If $p \\mid x$, then $p^{a} \\mid x$, since $x^{(p-1)^{2}-1}+1$ is not divisible by $p$ in this case. However, this is impossible, since $x \\leqslant x^{p-1}2$. Thus $a=r+1$. Now since $p^{r} \\leqslant x+1$, we get $$ x=\\frac{x^{2}+x}{x+1} \\leqslant \\frac{x^{p-1}+y}{x+1}=\\frac{p^{a}}{x+1} \\leqslant \\frac{p^{a}}{p^{r}}=p, $$ so we must have $x=p-1$ for $p$ to divide $x+1$. It follows that $r=1$ and $a=2$. If $p \\geqslant 5$, we obtain $$ p^{a}=x^{p-1}+y>(p-1)^{4}=\\left(p^{2}-2 p+1\\right)^{2}>(3 p)^{2}>p^{2}=p^{a} $$ a contradiction. So the only case that remains is $p=3$, and indeed $x=2$ and $y=p^{a}-x^{p-1}=5$ satisfy the conditions. Comment 1. In this solution, we are implicitly using a special case of the following lemma known as \"lifting the exponent\": Lemma. Let $n$ be a positive integer, let $p$ be an odd prime, and let $v_{p}(m)$ denote the exponent of the highest power of $p$ that divides $m$. If $x$ and $y$ are integers not divisible by $p$ such that $p \\mid x-y$, then we have $$ v_{p}\\left(x^{n}-y^{n}\\right)=v_{p}(x-y)+v_{p}(n) $$ Likewise, if $x$ and $y$ are integers not divisible by $p$ such that $p \\mid x+y$, then we have $$ v_{p}\\left(x^{n}+y^{n}\\right)=v_{p}(x+y)+v_{p}(n) . $$ Comment 2. There exist various ways of solving the problem involving the \"lifting the exponent\" lemma. Let us sketch another one. The cases $x=y$ and $p \\mid x$ are ruled out easily, so we assume that $p>2, x2$. If $p \\mid x$, then also $p \\mid y$. In this case, let $p^{k}$ and $p^{\\ell}$ be the highest powers of $p$ that divide $x$ and $y$ respectively, and assume without loss of generality that $k \\leqslant \\ell$. Then $p^{k}$ divides $x+y^{p-1}$ while $p^{k+1}$ does not, but $p^{k}p$, so $x^{p-1}+y$ and $y^{p-1}+x$ are both at least equal to $p^{2}$. Now we have $$ x^{p-1} \\equiv-y \\quad\\left(\\bmod p^{2}\\right) \\quad \\text { and } \\quad y^{p-1} \\equiv-x \\quad\\left(\\bmod p^{2}\\right) $$ These two congruences, together with the Euler-Fermat theorem, give us $$ 1 \\equiv x^{p(p-1)} \\equiv(-y)^{p} \\equiv-y^{p} \\equiv x y \\quad\\left(\\bmod p^{2}\\right) $$ Since $x \\equiv y \\equiv-1(\\bmod p), x-y$ is divisible by $p$, so $(x-y)^{2}$ is divisible by $p^{2}$. This means that $$ (x+y)^{2}=(x-y)^{2}+4 x y \\equiv 4 \\quad\\left(\\bmod p^{2}\\right) $$ so $p^{2}$ divides $(x+y-2)(x+y+2)$. We already know that $x+y \\equiv-2(\\bmod p)$, so $x+y-2 \\equiv$ $-4 \\not \\equiv 0(\\bmod p)$. This means that $p^{2}$ divides $x+y+2$. Using the same notation as in the first solution, we subtract the two original equations to obtain $$ p^{b}-p^{a}=y^{p-1}-x^{p-1}+x-y=(y-x)\\left(y^{p-2}+y^{p-3} x+\\cdots+x^{p-2}-1\\right) $$ The second factor is symmetric in $x$ and $y$, so it can be written as a polynomial of the elementary symmetric polynomials $x+y$ and $x y$ with integer coefficients. In particular, its value modulo $p^{2}$ is characterised by the two congruences $x y \\equiv 1\\left(\\bmod p^{2}\\right)$ and $x+y \\equiv-2\\left(\\bmod p^{2}\\right)$. Since both congruences are satisfied when $x=y=-1$, we must have $$ y^{p-2}+y^{p-3} x+\\cdots+x^{p-2}-1 \\equiv(-1)^{p-2}+(-1)^{p-3}(-1)+\\cdots+(-1)^{p-2}-1 \\quad\\left(\\bmod p^{2}\\right), $$ which simplifies to $y^{p-2}+y^{p-3} x+\\cdots+x^{p-2}-1 \\equiv-p\\left(\\bmod p^{2}\\right)$. Thus the second factor in (1) is divisible by $p$, but not $p^{2}$. This means that $p^{a-1}$ has to divide the other factor $y-x$. It follows that $$ 0 \\equiv x^{p-1}+y \\equiv x^{p-1}+x \\equiv x(x+1)\\left(x^{p-3}-x^{p-4}+\\cdots+1\\right) \\quad\\left(\\bmod p^{a-1}\\right) . $$ Since $x \\equiv-1(\\bmod p)$, the last factor is $x^{p-3}-x^{p-4}+\\cdots+1 \\equiv p-2(\\bmod p)$ and in particular not divisible by $p$. We infer that $p^{a-1} \\mid x+1$ and continue as in the first solution. Comment. Instead of reasoning by means of elementary symmetric polynomials, it is possible to provide a more direct argument as well. For odd $r,(x+1)^{2}$ divides $\\left(x^{r}+1\\right)^{2}$, and since $p$ divides $x+1$, we deduce that $p^{2}$ divides $\\left(x^{r}+1\\right)^{2}$. Together with the fact that $x y \\equiv 1\\left(\\bmod p^{2}\\right)$, we obtain $$ 0 \\equiv y^{r}\\left(x^{r}+1\\right)^{2} \\equiv x^{2 r} y^{r}+2 x^{r} y^{r}+y^{r} \\equiv x^{r}+2+y^{r} \\quad\\left(\\bmod p^{2}\\right) . $$ We apply this congruence with $r=p-2-2 k$ (where $0 \\leqslant k<(p-2) \/ 2$ ) to find that $$ x^{k} y^{p-2-k}+x^{p-2-k} y^{k} \\equiv(x y)^{k}\\left(x^{p-2-2 k}+y^{p-2-2 k}\\right) \\equiv 1^{k} \\cdot(-2) \\equiv-2 \\quad\\left(\\bmod p^{2}\\right) . $$ Summing over all $k$ yields $$ y^{p-2}+y^{p-3} x+\\cdots+x^{p-2}-1 \\equiv \\frac{p-1}{2} \\cdot(-2)-1 \\equiv-p \\quad\\left(\\bmod p^{2}\\right) $$ once again.","tier":0} +{"problem_type":"Number Theory","problem_label":"N6","problem":"Let $a_{1}x_{n} $$ holds for all integers $n \\geqslant 0$, it is also strictly increasing. Since $x_{n+1}$ is by (1) coprime to $c$ for any $n \\geqslant 0$, it suffices to prove that for each $n \\geqslant 2$ there exists a prime number $p$ dividing $x_{n}$ but none of the numbers $x_{1}, \\ldots, x_{n-1}$. Let us begin by establishing three preliminary claims. Claim 1. If $i \\equiv j(\\bmod m)$ holds for some integers $i, j \\geqslant 0$ and $m \\geqslant 1$, then $x_{i} \\equiv x_{j}\\left(\\bmod x_{m}\\right)$ holds as well. Proof. Evidently, it suffices to show $x_{i+m} \\equiv x_{i}\\left(\\bmod x_{m}\\right)$ for all integers $i \\geqslant 0$ and $m \\geqslant 1$. For this purpose we may argue for fixed $m$ by induction on $i$ using $x_{0}=0$ in the base case $i=0$. Now, if we have $x_{i+m} \\equiv x_{i}\\left(\\bmod x_{m}\\right)$ for some integer $i$, then the recursive equation (1) yields $$ x_{i+m+1} \\equiv c^{2}\\left(x_{i+m}^{3}-4 x_{i+m}^{2}+5 x_{i+m}\\right)+1 \\equiv c^{2}\\left(x_{i}^{3}-4 x_{i}^{2}+5 x_{i}\\right)+1 \\equiv x_{i+1} \\quad\\left(\\bmod x_{m}\\right), $$ which completes the induction. Claim 2. If the integers $i, j \\geqslant 2$ and $m \\geqslant 1$ satisfy $i \\equiv j(\\bmod m)$, then $x_{i} \\equiv x_{j}\\left(\\bmod x_{m}^{2}\\right)$ holds as well. Proof. Again it suffices to prove $x_{i+m} \\equiv x_{i}\\left(\\bmod x_{m}^{2}\\right)$ for all integers $i \\geqslant 2$ and $m \\geqslant 1$. As above, we proceed for fixed $m$ by induction on $i$. The induction step is again easy using (1), but this time the base case $i=2$ requires some calculation. Set $L=5 c^{2}$. By (1) we have $x_{m+1} \\equiv L x_{m}+1\\left(\\bmod x_{m}^{2}\\right)$, and hence $$ \\begin{aligned} x_{m+1}^{3}-4 x_{m+1}^{2}+5 x_{m+1} & \\equiv\\left(L x_{m}+1\\right)^{3}-4\\left(L x_{m}+1\\right)^{2}+5\\left(L x_{m}+1\\right) \\\\ & \\equiv\\left(3 L x_{m}+1\\right)-4\\left(2 L x_{m}+1\\right)+5\\left(L x_{m}+1\\right) \\equiv 2 \\quad\\left(\\bmod x_{m}^{2}\\right) \\end{aligned} $$ which in turn gives indeed $x_{m+2} \\equiv 2 c^{2}+1 \\equiv x_{2}\\left(\\bmod x_{m}^{2}\\right)$. Claim 3. For each integer $n \\geqslant 2$, we have $x_{n}>x_{1} \\cdot x_{2} \\cdots x_{n-2}$. Proof. The cases $n=2$ and $n=3$ are clear. Arguing inductively, we assume now that the claim holds for some $n \\geqslant 3$. Recall that $x_{2} \\geqslant 3$, so by monotonicity and (2) we get $x_{n} \\geqslant x_{3} \\geqslant x_{2}\\left(x_{2}-2\\right)^{2}+x_{2}+1 \\geqslant 7$. It follows that $$ x_{n+1}>x_{n}^{3}-4 x_{n}^{2}+5 x_{n}>7 x_{n}^{2}-4 x_{n}^{2}>x_{n}^{2}>x_{n} x_{n-1} $$ which by the induction hypothesis yields $x_{n+1}>x_{1} \\cdot x_{2} \\cdots x_{n-1}$, as desired. Now we direct our attention to the problem itself: let any integer $n \\geqslant 2$ be given. By Claim 3 there exists a prime number $p$ appearing with a higher exponent in the prime factorisation of $x_{n}$ than in the prime factorisation of $x_{1} \\cdots x_{n-2}$. In particular, $p \\mid x_{n}$, and it suffices to prove that $p$ divides none of $x_{1}, \\ldots, x_{n-1}$. Otherwise let $k \\in\\{1, \\ldots, n-1\\}$ be minimal such that $p$ divides $x_{k}$. Since $x_{n-1}$ and $x_{n}$ are coprime by (1) and $x_{1}=1$, we actually have $2 \\leqslant k \\leqslant n-2$. Write $n=q k+r$ with some integers $q \\geqslant 0$ and $0 \\leqslant r1$, so there exists a prime $p$ with $v_{p}(N)>0$. Since $N$ is a fraction of two odd numbers, $p$ is odd. By our lemma, $$ 0\\frac{1}{2}, \\quad \\text { or } \\quad\\{x\\}>\\frac{1}{2}, \\quad\\{y\\}>\\frac{1}{2}, \\quad\\{x+y\\}<\\frac{1}{2}, $$ where $\\{x\\}$ denotes the fractional part of $x$. In the context of our problem, the first condition seems easier to deal with. Also, one may notice that $$ \\{x\\}<\\frac{1}{2} \\Longleftrightarrow \\varkappa(x)=0 \\quad \\text { and } \\quad\\{x\\} \\geqslant \\frac{1}{2} \\Longleftrightarrow \\varkappa(x)=1, $$ where $$ \\varkappa(x)=\\lfloor 2 x\\rfloor-2\\lfloor x\\rfloor . $$ Now it is natural to consider the number $$ M=\\frac{\\binom{2 a+2 b}{a+b}}{\\binom{2 a}{a}\\binom{2 b}{b}} $$ since $$ v_{p}(M)=\\sum_{k=1}^{\\infty}\\left(\\varkappa\\left(\\frac{2(a+b)}{p^{k}}\\right)-\\varkappa\\left(\\frac{2 a}{p^{k}}\\right)-\\varkappa\\left(\\frac{2 b}{p^{k}}\\right)\\right) . $$ One may see that $M>1$, and that $v_{2}(M) \\leqslant 0$. Thus, there exist an odd prime $p$ and a positive integer $k$ with $$ \\varkappa\\left(\\frac{2(a+b)}{p^{k}}\\right)-\\varkappa\\left(\\frac{2 a}{p^{k}}\\right)-\\varkappa\\left(\\frac{2 b}{p^{k}}\\right)>0 . $$ In view of (4), the last inequality yields $$ \\left\\{\\frac{a}{p^{k}}\\right\\}<\\frac{1}{2}, \\quad\\left\\{\\frac{b}{p^{k}}\\right\\}<\\frac{1}{2}, \\quad \\text { and } \\quad\\left\\{\\frac{a+b}{p^{k}}\\right\\}>\\frac{1}{2} $$ which is what we wanted to obtain. Comment 2. Once one tries to prove the existence of suitable $p$ and $k$ satisfying (5), it seems somehow natural to suppose that $a \\leqslant b$ and to add the restriction $p^{k}>a$. In this case the inequalities (5) can be rewritten as $$ 2 ak$. We would like to mention here that Sylvester's theorem itself does not seem to suffice for solving the problem.","tier":0} +{"problem_type":"Algebra","problem_label":"A1","problem":"Let $z_{0}0$. Notice that $$ n z_{n+1}-\\left(z_{0}+z_{1}+\\cdots+z_{n}\\right)=(n+1) z_{n+1}-\\left(z_{0}+z_{1}+\\cdots+z_{n}+z_{n+1}\\right)=-d_{n+1} $$ so the second inequality in (1) is equivalent to $d_{n+1} \\leqslant 0$. Therefore, we have to prove that there is a unique index $n \\geqslant 1$ that satisfies $d_{n}>0 \\geqslant d_{n+1}$. By its definition the sequence $d_{1}, d_{2}, \\ldots$ consists of integers and we have $$ d_{1}=\\left(z_{0}+z_{1}\\right)-1 \\cdot z_{1}=z_{0}>0 $$ From $d_{n+1}-d_{n}=\\left(\\left(z_{0}+\\cdots+z_{n}+z_{n+1}\\right)-(n+1) z_{n+1}\\right)-\\left(\\left(z_{0}+\\cdots+z_{n}\\right)-n z_{n}\\right)=n\\left(z_{n}-z_{n+1}\\right)<0$ we can see that $d_{n+1}d_{2}>\\ldots$ of integers such that its first element $d_{1}$ is positive. The sequence must drop below 0 at some point, and thus there is a unique index $n$, that is the index of the last positive term, satisfying $d_{n}>0 \\geqslant d_{n+1}$. Comment. Omitting the assumption that $z_{1}, z_{2}, \\ldots$ are integers allows the numbers $d_{n}$ to be all positive. In such cases the desired $n$ does not exist. This happens for example if $z_{n}=2-\\frac{1}{2^{n}}$ for all integers $n \\geqslant 0$.","tier":0} +{"problem_type":"Algebra","problem_label":"A3","problem":"For a sequence $x_{1}, x_{2}, \\ldots, x_{n}$ of real numbers, we define its price as $$ \\max _{1 \\leqslant i \\leqslant n}\\left|x_{1}+\\cdots+x_{i}\\right| $$ Given $n$ real numbers, Dave and George want to arrange them into a sequence with a low price. Diligent Dave checks all possible ways and finds the minimum possible price $D$. Greedy George, on the other hand, chooses $x_{1}$ such that $\\left|x_{1}\\right|$ is as small as possible; among the remaining numbers, he chooses $x_{2}$ such that $\\left|x_{1}+x_{2}\\right|$ is as small as possible, and so on. Thus, in the $i^{\\text {th }}$ step he chooses $x_{i}$ among the remaining numbers so as to minimise the value of $\\left|x_{1}+x_{2}+\\cdots+x_{i}\\right|$. In each step, if several numbers provide the same value, George chooses one at random. Finally he gets a sequence with price $G$. Find the least possible constant $c$ such that for every positive integer $n$, for every collection of $n$ real numbers, and for every possible sequence that George might obtain, the resulting values satisfy the inequality $G \\leqslant c D$. (Georgia) Answer. $c=2$.","solution":"If the initial numbers are $1,-1,2$, and -2 , then Dave may arrange them as $1,-2,2,-1$, while George may get the sequence $1,-1,2,-2$, resulting in $D=1$ and $G=2$. So we obtain $c \\geqslant 2$. Therefore, it remains to prove that $G \\leqslant 2 D$. Let $x_{1}, x_{2}, \\ldots, x_{n}$ be the numbers Dave and George have at their disposal. Assume that Dave and George arrange them into sequences $d_{1}, d_{2}, \\ldots, d_{n}$ and $g_{1}, g_{2}, \\ldots, g_{n}$, respectively. Put $$ M=\\max _{1 \\leqslant i \\leqslant n}\\left|x_{i}\\right|, \\quad S=\\left|x_{1}+\\cdots+x_{n}\\right|, \\quad \\text { and } \\quad N=\\max \\{M, S\\} $$ We claim that $$ \\begin{aligned} & D \\geqslant S, \\\\ & D \\geqslant \\frac{M}{2}, \\quad \\text { and } \\\\ & G \\leqslant N=\\max \\{M, S\\} \\end{aligned} $$ These inequalities yield the desired estimate, as $G \\leqslant \\max \\{M, S\\} \\leqslant \\max \\{M, 2 S\\} \\leqslant 2 D$. The inequality (1) is a direct consequence of the definition of the price. To prove (2), consider an index $i$ with $\\left|d_{i}\\right|=M$. Then we have $$ M=\\left|d_{i}\\right|=\\left|\\left(d_{1}+\\cdots+d_{i}\\right)-\\left(d_{1}+\\cdots+d_{i-1}\\right)\\right| \\leqslant\\left|d_{1}+\\cdots+d_{i}\\right|+\\left|d_{1}+\\cdots+d_{i-1}\\right| \\leqslant 2 D $$ as required. It remains to establish (3). Put $h_{i}=g_{1}+g_{2}+\\cdots+g_{i}$. We will prove by induction on $i$ that $\\left|h_{i}\\right| \\leqslant N$. The base case $i=1$ holds, since $\\left|h_{1}\\right|=\\left|g_{1}\\right| \\leqslant M \\leqslant N$. Notice also that $\\left|h_{n}\\right|=S \\leqslant N$. For the induction step, assume that $\\left|h_{i-1}\\right| \\leqslant N$. We distinguish two cases. Case 1. Assume that no two of the numbers $g_{i}, g_{i+1}, \\ldots, g_{n}$ have opposite signs. Without loss of generality, we may assume that they are all nonnegative. Then one has $h_{i-1} \\leqslant h_{i} \\leqslant \\cdots \\leqslant h_{n}$, thus $$ \\left|h_{i}\\right| \\leqslant \\max \\left\\{\\left|h_{i-1}\\right|,\\left|h_{n}\\right|\\right\\} \\leqslant N $$ Case 2. Among the numbers $g_{i}, g_{i+1}, \\ldots, g_{n}$ there are positive and negative ones. Then there exists some index $j \\geqslant i$ such that $h_{i-1} g_{j} \\leqslant 0$. By the definition of George's sequence we have $$ \\left|h_{i}\\right|=\\left|h_{i-1}+g_{i}\\right| \\leqslant\\left|h_{i-1}+g_{j}\\right| \\leqslant \\max \\left\\{\\left|h_{i-1}\\right|,\\left|g_{j}\\right|\\right\\} \\leqslant N $$ Thus, the induction step is established. Comment 1. One can establish the weaker inequalities $D \\geqslant \\frac{M}{2}$ and $G \\leqslant D+\\frac{M}{2}$ from which the result also follows. Comment 2. One may ask a more specific question to find the maximal suitable $c$ if the number $n$ is fixed. For $n=1$ or 2 , the answer is $c=1$. For $n=3$, the answer is $c=\\frac{3}{2}$, and it is reached e.g., for the collection $1,2,-4$. Finally, for $n \\geqslant 4$ the answer is $c=2$. In this case the arguments from the solution above apply, and the answer is reached e.g., for the same collection $1,-1,2,-2$, augmented by several zeroes.","tier":0} +{"problem_type":"Algebra","problem_label":"A4","problem":"Determine all functions $f: \\mathbb{Z} \\rightarrow \\mathbb{Z}$ satisfying $$ f(f(m)+n)+f(m)=f(n)+f(3 m)+2014 $$ for all integers $m$ and $n$. (Netherlands) Answer. There is only one such function, namely $n \\longmapsto 2 n+1007$.","solution":"Let $f$ be a function satisfying (1). Set $C=1007$ and define the function $g: \\mathbb{Z} \\rightarrow \\mathbb{Z}$ by $g(m)=f(3 m)-f(m)+2 C$ for all $m \\in \\mathbb{Z}$; in particular, $g(0)=2 C$. Now (1) rewrites as $$ f(f(m)+n)=g(m)+f(n) $$ for all $m, n \\in \\mathbb{Z}$. By induction in both directions it follows that $$ f(t f(m)+n)=t g(m)+f(n) $$ holds for all $m, n, t \\in \\mathbb{Z}$. Applying this, for any $r \\in \\mathbb{Z}$, to the triples $(r, 0, f(0))$ and $(0,0, f(r))$ in place of $(m, n, t)$ we obtain $$ f(0) g(r)=f(f(r) f(0))-f(0)=f(r) g(0) $$ Now if $f(0)$ vanished, then $g(0)=2 C>0$ would entail that $f$ vanishes identically, contrary to (1). Thus $f(0) \\neq 0$ and the previous equation yields $g(r)=\\alpha f(r)$, where $\\alpha=\\frac{g(0)}{f(0)}$ is some nonzero constant. So the definition of $g$ reveals $f(3 m)=(1+\\alpha) f(m)-2 C$, i.e., $$ f(3 m)-\\beta=(1+\\alpha)(f(m)-\\beta) $$ for all $m \\in \\mathbb{Z}$, where $\\beta=\\frac{2 C}{\\alpha}$. By induction on $k$ this implies $$ f\\left(3^{k} m\\right)-\\beta=(1+\\alpha)^{k}(f(m)-\\beta) $$ for all integers $k \\geqslant 0$ and $m$. Since $3 \\nmid 2014$, there exists by (1) some value $d=f(a)$ attained by $f$ that is not divisible by 3 . Now by (2) we have $f(n+t d)=f(n)+t g(a)=f(n)+\\alpha \\cdot t f(a)$, i.e., $$ f(n+t d)=f(n)+\\alpha \\cdot t d $$ for all $n, t \\in \\mathbb{Z}$. Let us fix any positive integer $k$ with $d \\mid\\left(3^{k}-1\\right)$, which is possible, since $\\operatorname{gcd}(3, d)=1$. E.g., by the Euler-Fermat theorem, we may take $k=\\varphi(|d|)$. Now for each $m \\in \\mathbb{Z}$ we get $$ f\\left(3^{k} m\\right)=f(m)+\\alpha\\left(3^{k}-1\\right) m $$ from (5), which in view of (4) yields $\\left((1+\\alpha)^{k}-1\\right)(f(m)-\\beta)=\\alpha\\left(3^{k}-1\\right) m$. Since $\\alpha \\neq 0$, the right hand side does not vanish for $m \\neq 0$, wherefore the first factor on the left hand side cannot vanish either. It follows that $$ f(m)=\\frac{\\alpha\\left(3^{k}-1\\right)}{(1+\\alpha)^{k}-1} \\cdot m+\\beta $$ So $f$ is a linear function, say $f(m)=A m+\\beta$ for all $m \\in \\mathbb{Z}$ with some constant $A \\in \\mathbb{Q}$. Plugging this into (1) one obtains $\\left(A^{2}-2 A\\right) m+(A \\beta-2 C)=0$ for all $m$, which is equivalent to the conjunction of $$ A^{2}=2 A \\quad \\text { and } \\quad A \\beta=2 C $$ The first equation is equivalent to $A \\in\\{0,2\\}$, and as $C \\neq 0$ the second one gives $$ A=2 \\quad \\text { and } \\quad \\beta=C $$ This shows that $f$ is indeed the function mentioned in the answer and as the numbers found in (7) do indeed satisfy the equations (6) this function is indeed as desired. Comment 1. One may see that $\\alpha=2$. A more pedestrian version of the above solution starts with a direct proof of this fact, that can be obtained by substituting some special values into (1), e.g., as follows. Set $D=f(0)$. Plugging $m=0$ into (1) and simplifying, we get $$ f(n+D)=f(n)+2 C $$ for all $n \\in \\mathbb{Z}$. In particular, for $n=0, D, 2 D$ we obtain $f(D)=2 C+D, f(2 D)=f(D)+2 C=4 C+D$, and $f(3 D)=f(2 D)+2 C=6 C+D$. So substituting $m=D$ and $n=r-D$ into (1) and applying (8) with $n=r-D$ afterwards we learn $$ f(r+2 C)+2 C+D=(f(r)-2 C)+(6 C+D)+2 C $$ i.e., $f(r+2 C)=f(r)+4 C$. By induction in both directions it follows that $$ f(n+2 C t)=f(n)+4 C t $$ holds for all $n, t \\in \\mathbb{Z}$. Claim. If $a$ and $b$ denote two integers with the property that $f(n+a)=f(n)+b$ holds for all $n \\in \\mathbb{Z}$, then $b=2 a$. Proof. Applying induction in both directions to the assumption we get $f(n+t a)=f(n)+t b$ for all $n, t \\in \\mathbb{Z}$. Plugging $(n, t)=(0,2 C)$ into this equation and $(n, t)=(0, a)$ into $(9)$ we get $f(2 a C)-f(0)=$ $2 b C=4 a C$, and, as $C \\neq 0$, the claim follows. Now by (1), for any $m \\in \\mathbb{Z}$, the numbers $a=f(m)$ and $b=f(3 m)-f(m)+2 C$ have the property mentioned in the claim, whence we have $$ f(3 m)-C=3(f(m)-C) . $$ In view of (3) this tells us indeed that $\\alpha=2$. Now the solution may be completed as above, but due to our knowledge of $\\alpha=2$ we get the desired formula $f(m)=2 m+C$ directly without having the need to go through all linear functions. Now it just remains to check that this function does indeed satisfy (1). Comment 2. It is natural to wonder what happens if one replaces the number 2014 appearing in the statement of the problem by some arbitrary integer $B$. If $B$ is odd, there is no such function, as can be seen by using the same ideas as in the above solution. If $B \\neq 0$ is even, however, then the only such function is given by $n \\longmapsto 2 n+B \/ 2$. In case $3 \\nmid B$ this was essentially proved above, but for the general case one more idea seems to be necessary. Writing $B=3^{\\nu} \\cdot k$ with some integers $\\nu$ and $k$ such that $3 \\nmid k$ one can obtain $f(n)=2 n+B \/ 2$ for all $n$ that are divisible by $3^{\\nu}$ in the same manner as usual; then one may use the formula $f(3 n)=3 f(n)-B$ to establish the remaining cases. Finally, in case $B=0$ there are more solutions than just the function $n \\longmapsto 2 n$. It can be shown that all these other functions are periodic; to mention just one kind of example, for any even integers $r$ and $s$ the function $$ f(n)= \\begin{cases}r & \\text { if } n \\text { is even, } \\\\ s & \\text { if } n \\text { is odd }\\end{cases} $$ also has the property under discussion.","tier":0} +{"problem_type":"Algebra","problem_label":"A5","problem":"Consider all polynomials $P(x)$ with real coefficients that have the following property: for any two real numbers $x$ and $y$ one has $$ \\left|y^{2}-P(x)\\right| \\leqslant 2|x| \\quad \\text { if and only if } \\quad\\left|x^{2}-P(y)\\right| \\leqslant 2|y| $$ Determine all possible values of $P(0)$. (Belgium) Answer. The set of possible values of $P(0)$ is $(-\\infty, 0) \\cup\\{1\\}$.","solution":"Part I. We begin by verifying that these numbers are indeed possible values of $P(0)$. To see that each negative real number $-C$ can be $P(0)$, it suffices to check that for every $C>0$ the polynomial $P(x)=-\\left(\\frac{2 x^{2}}{C}+C\\right)$ has the property described in the statement of the problem. Due to symmetry it is enough for this purpose to prove $\\left|y^{2}-P(x)\\right|>2|x|$ for any two real numbers $x$ and $y$. In fact we have $$ \\left|y^{2}-P(x)\\right|=y^{2}+\\frac{x^{2}}{C}+\\frac{(|x|-C)^{2}}{C}+2|x| \\geqslant \\frac{x^{2}}{C}+2|x| \\geqslant 2|x|, $$ where in the first estimate equality can only hold if $|x|=C$, whilst in the second one it can only hold if $x=0$. As these two conditions cannot be met at the same time, we have indeed $\\left|y^{2}-P(x)\\right|>2|x|$. To show that $P(0)=1$ is possible as well, we verify that the polynomial $P(x)=x^{2}+1$ satisfies (1). Notice that for all real numbers $x$ and $y$ we have $$ \\begin{aligned} \\left|y^{2}-P(x)\\right| \\leqslant 2|x| & \\Longleftrightarrow\\left(y^{2}-x^{2}-1\\right)^{2} \\leqslant 4 x^{2} \\\\ & \\Longleftrightarrow 0 \\leqslant\\left(\\left(y^{2}-(x-1)^{2}\\right)\\left((x+1)^{2}-y^{2}\\right)\\right. \\\\ & \\Longleftrightarrow 0 \\leqslant(y-x+1)(y+x-1)(x+1-y)(x+1+y) \\\\ & \\Longleftrightarrow 0 \\leqslant\\left((x+y)^{2}-1\\right)\\left(1-(x-y)^{2}\\right) . \\end{aligned} $$ Since this inequality is symmetric in $x$ and $y$, we are done. Part II. Now we show that no values other than those mentioned in the answer are possible for $P(0)$. To reach this we let $P$ denote any polynomial satisfying (1) and $P(0) \\geqslant 0$; as we shall see, this implies $P(x)=x^{2}+1$ for all real $x$, which is actually more than what we want. First step: We prove that $P$ is even. By (1) we have $$ \\left|y^{2}-P(x)\\right| \\leqslant 2|x| \\Longleftrightarrow\\left|x^{2}-P(y)\\right| \\leqslant 2|y| \\Longleftrightarrow\\left|y^{2}-P(-x)\\right| \\leqslant 2|x| $$ for all real numbers $x$ and $y$. Considering just the equivalence of the first and third statement and taking into account that $y^{2}$ may vary through $\\mathbb{R}_{\\geqslant 0}$ we infer that $$ [P(x)-2|x|, P(x)+2|x|] \\cap \\mathbb{R}_{\\geqslant 0}=[P(-x)-2|x|, P(-x)+2|x|] \\cap \\mathbb{R}_{\\geqslant 0} $$ holds for all $x \\in \\mathbb{R}$. We claim that there are infinitely many real numbers $x$ such that $P(x)+2|x| \\geqslant 0$. This holds in fact for any real polynomial with $P(0) \\geqslant 0$; in order to see this, we may assume that the coefficient of $P$ appearing in front of $x$ is nonnegative. In this case the desired inequality holds for all sufficiently small positive real numbers. For such numbers $x$ satisfying $P(x)+2|x| \\geqslant 0$ we have $P(x)+2|x|=P(-x)+2|x|$ by the previous displayed formula, and hence also $P(x)=P(-x)$. Consequently the polynomial $P(x)-P(-x)$ has infinitely many zeros, wherefore it has to vanish identically. Thus $P$ is indeed even. Second step: We prove that $P(t)>0$ for all $t \\in \\mathbb{R}$. Let us assume for a moment that there exists a real number $t \\neq 0$ with $P(t)=0$. Then there is some open interval $I$ around $t$ such that $|P(y)| \\leqslant 2|y|$ holds for all $y \\in I$. Plugging $x=0$ into (1) we learn that $y^{2}=P(0)$ holds for all $y \\in I$, which is clearly absurd. We have thus shown $P(t) \\neq 0$ for all $t \\neq 0$. In combination with $P(0) \\geqslant 0$ this informs us that our claim could only fail if $P(0)=0$. In this case there is by our first step a polynomial $Q(x)$ such that $P(x)=x^{2} Q(x)$. Applying (1) to $x=0$ and an arbitrary $y \\neq 0$ we get $|y Q(y)|>2$, which is surely false when $y$ is sufficiently small. Third step: We prove that $P$ is a quadratic polynomial. Notice that $P$ cannot be constant, for otherwise if $x=\\sqrt{P(0)}$ and $y$ is sufficiently large, the first part of (1) is false whilst the second part is true. So the degree $n$ of $P$ has to be at least 1 . By our first step $n$ has to be even as well, whence in particular $n \\geqslant 2$. Now assume that $n \\geqslant 4$. Plugging $y=\\sqrt{P(x)}$ into (1) we get $\\left|x^{2}-P(\\sqrt{P(x)})\\right| \\leqslant 2 \\sqrt{P(x)}$ and hence $$ P(\\sqrt{P(x)}) \\leqslant x^{2}+2 \\sqrt{P(x)} $$ for all real $x$. Choose positive real numbers $x_{0}, a$, and $b$ such that if $x \\in\\left(x_{0}, \\infty\\right)$, then $a x^{n}<$ $P(x)0$ denotes the leading coefficient of $P$, then $\\lim _{x \\rightarrow \\infty} \\frac{P(x)}{x^{n}}=d$, whence for instance the numbers $a=\\frac{d}{2}$ and $b=2 d$ work provided that $x_{0}$ is chosen large enough. Now for all sufficiently large real numbers $x$ we have $$ a^{n \/ 2+1} x^{n^{2} \/ 2}0$ and $b$ such that $P(x)=$ $a x^{2}+b$. Now if $x$ is large enough and $y=\\sqrt{a} x$, the left part of (1) holds and the right part reads $\\left|\\left(1-a^{2}\\right) x^{2}-b\\right| \\leqslant 2 \\sqrt{a} x$. In view of the fact that $a>0$ this is only possible if $a=1$. Finally, substituting $y=x+1$ with $x>0$ into (1) we get $$ |2 x+1-b| \\leqslant 2 x \\Longleftrightarrow|2 x+1+b| \\leqslant 2 x+2 $$ i.e., $$ b \\in[1,4 x+1] \\Longleftrightarrow b \\in[-4 x-3,1] $$ for all $x>0$. Choosing $x$ large enough, we can achieve that at least one of these two statements holds; then both hold, which is only possible if $b=1$, as desired. Comment 1. There are some issues with this problem in that its most natural solutions seem to use some basic facts from analysis, such as the continuity of polynomials or the intermediate value theorem. Yet these facts are intuitively obvious and implicitly clear to the students competing at this level of difficulty, so that the Problem Selection Committee still thinks that the problem is suitable for the IMO. Comment 2. It seems that most solutions will in the main case, where $P(0)$ is nonnegative, contain an argument that is somewhat asymptotic in nature showing that $P$ is quadratic, and some part narrowing that case down to $P(x)=x^{2}+1$. Comment 3. It is also possible to skip the first step and start with the second step directly, but then one has to work a bit harder to rule out the case $P(0)=0$. Let us sketch one possibility of doing this: Take the auxiliary polynomial $Q(x)$ such that $P(x)=x Q(x)$. Applying (1) to $x=0$ and an arbitrary $y \\neq 0$ we get $|Q(y)|>2$. Hence we either have $Q(z) \\geqslant 2$ for all real $z$ or $Q(z) \\leqslant-2$ for all real $z$. In particular there is some $\\eta \\in\\{-1,+1\\}$ such that $P(\\eta) \\geqslant 2$ and $P(-\\eta) \\leqslant-2$. Substituting $x= \\pm \\eta$ into (1) we learn $$ \\left|y^{2}-P(\\eta)\\right| \\leqslant 2 \\Longleftrightarrow|1-P(y)| \\leqslant 2|y| \\Longleftrightarrow\\left|y^{2}-P(-\\eta)\\right| \\leqslant 2 $$ But for $y=\\sqrt{P(\\eta)}$ the first statement is true, whilst the third one is false. Also, if one has not obtained the evenness of $P$ before embarking on the fourth step, one needs to work a bit harder there, but not in a way that is likely to cause major difficulties. Comment 4. Truly curious people may wonder about the set of all polynomials having property (1). As explained in the solution above, $P(x)=x^{2}+1$ is the only one with $P(0)=1$. On the other hand, it is not hard to notice that for negative $P(0)$ there are more possibilities than those mentioned above. E.g., as remarked by the proposer, if $a$ and $b$ denote two positive real numbers with $a b>1$ and $Q$ denotes a polynomial attaining nonnegative values only, then $P(x)=-\\left(a x^{2}+b+Q(x)\\right)$ works. More generally, it may be proved that if $P(x)$ satisfies (1) and $P(0)<0$, then $-P(x)>2|x|$ holds for all $x \\in \\mathbb{R}$ so that one just considers the equivalence of two false statements. One may generate all such polynomials $P$ by going through all combinations of a solution of the polynomial equation $$ x=A(x) B(x)+C(x) D(x) $$ and a real $E>0$, and setting $$ P(x)=-\\left(A(x)^{2}+B(x)^{2}+C(x)^{2}+D(x)^{2}+E\\right) $$ for each of them.","tier":0} +{"problem_type":"Algebra","problem_label":"A6","problem":"Find all functions $f: \\mathbb{Z} \\rightarrow \\mathbb{Z}$ such that $$ n^{2}+4 f(n)=f(f(n))^{2} $$ for all $n \\in \\mathbb{Z}$. (United Kingdom) Answer. The possibilities are: - $f(n)=n+1$ for all $n$; - or, for some $a \\geqslant 1, \\quad f(n)= \\begin{cases}n+1, & n>-a, \\\\ -n+1, & n \\leqslant-a ;\\end{cases}$ - or $f(n)= \\begin{cases}n+1, & n>0, \\\\ 0, & n=0, \\\\ -n+1, & n<0 .\\end{cases}$","solution":"Part I. Let us first check that each of the functions above really satisfies the given functional equation. If $f(n)=n+1$ for all $n$, then we have $$ n^{2}+4 f(n)=n^{2}+4 n+4=(n+2)^{2}=f(n+1)^{2}=f(f(n))^{2} . $$ If $f(n)=n+1$ for $n>-a$ and $f(n)=-n+1$ otherwise, then we have the same identity for $n>-a$ and $$ n^{2}+4 f(n)=n^{2}-4 n+4=(2-n)^{2}=f(1-n)^{2}=f(f(n))^{2} $$ otherwise. The same applies to the third solution (with $a=0$ ), where in addition one has $$ 0^{2}+4 f(0)=0=f(f(0))^{2} $$ Part II. It remains to prove that these are really the only functions that satisfy our functional equation. We do so in three steps: Step 1: We prove that $f(n)=n+1$ for $n>0$. Consider the sequence $\\left(a_{k}\\right)$ given by $a_{k}=f^{k}(1)$ for $k \\geqslant 0$. Setting $n=a_{k}$ in (1), we get $$ a_{k}^{2}+4 a_{k+1}=a_{k+2}^{2} $$ Of course, $a_{0}=1$ by definition. Since $a_{2}^{2}=1+4 a_{1}$ is odd, $a_{2}$ has to be odd as well, so we set $a_{2}=2 r+1$ for some $r \\in \\mathbb{Z}$. Then $a_{1}=r^{2}+r$ and consequently $$ a_{3}^{2}=a_{1}^{2}+4 a_{2}=\\left(r^{2}+r\\right)^{2}+8 r+4 $$ Since $8 r+4 \\neq 0, a_{3}^{2} \\neq\\left(r^{2}+r\\right)^{2}$, so the difference between $a_{3}^{2}$ and $\\left(r^{2}+r\\right)^{2}$ is at least the distance from $\\left(r^{2}+r\\right)^{2}$ to the nearest even square (since $8 r+4$ and $r^{2}+r$ are both even). This implies that $$ |8 r+4|=\\left|a_{3}^{2}-\\left(r^{2}+r\\right)^{2}\\right| \\geqslant\\left(r^{2}+r\\right)^{2}-\\left(r^{2}+r-2\\right)^{2}=4\\left(r^{2}+r-1\\right) $$ (for $r=0$ and $r=-1$, the estimate is trivial, but this does not matter). Therefore, we ave $$ 4 r^{2} \\leqslant|8 r+4|-4 r+4 $$ If $|r| \\geqslant 4$, then $$ 4 r^{2} \\geqslant 16|r| \\geqslant 12|r|+16>8|r|+4+4|r|+4 \\geqslant|8 r+4|-4 r+4 $$ a contradiction. Thus $|r|<4$. Checking all possible remaining values of $r$, we find that $\\left(r^{2}+r\\right)^{2}+8 r+4$ is only a square in three cases: $r=-3, r=0$ and $r=1$. Let us now distinguish these three cases: - $r=-3$, thus $a_{1}=6$ and $a_{2}=-5$. For each $k \\geqslant 1$, we have $$ a_{k+2}= \\pm \\sqrt{a_{k}^{2}+4 a_{k+1}} $$ and the sign needs to be chosen in such a way that $a_{k+1}^{2}+4 a_{k+2}$ is again a square. This yields $a_{3}=-4, a_{4}=-3, a_{5}=-2, a_{6}=-1, a_{7}=0, a_{8}=1, a_{9}=2$. At this point we have reached a contradiction, since $f(1)=f\\left(a_{0}\\right)=a_{1}=6$ and at the same time $f(1)=f\\left(a_{8}\\right)=a_{9}=2$. - $r=0$, thus $a_{1}=0$ and $a_{2}=1$. Then $a_{3}^{2}=a_{1}^{2}+4 a_{2}=4$, so $a_{3}= \\pm 2$. This, however, is a contradiction again, since it gives us $f(1)=f\\left(a_{0}\\right)=a_{1}=0$ and at the same time $f(1)=f\\left(a_{2}\\right)=a_{3}= \\pm 2$. - $r=1$, thus $a_{1}=2$ and $a_{2}=3$. We prove by induction that $a_{k}=k+1$ for all $k \\geqslant 0$ in this case, which we already know for $k \\leqslant 2$ now. For the induction step, assume that $a_{k-1}=k$ and $a_{k}=k+1$. Then $$ a_{k+1}^{2}=a_{k-1}^{2}+4 a_{k}=k^{2}+4 k+4=(k+2)^{2} $$ so $a_{k+1}= \\pm(k+2)$. If $a_{k+1}=-(k+2)$, then $$ a_{k+2}^{2}=a_{k}^{2}+4 a_{k+1}=(k+1)^{2}-4 k-8=k^{2}-2 k-7=(k-1)^{2}-8 $$ The latter can only be a square if $k=4$ (since 1 and 9 are the only two squares whose difference is 8 ). Then, however, $a_{4}=5, a_{5}=-6$ and $a_{6}= \\pm 1$, so $$ a_{7}^{2}=a_{5}^{2}+4 a_{6}=36 \\pm 4 $$ but neither 32 nor 40 is a perfect square. Thus $a_{k+1}=k+2$, which completes our induction. This also means that $f(n)=f\\left(a_{n-1}\\right)=a_{n}=n+1$ for all $n \\geqslant 1$. Step 2: We prove that either $f(0)=1$, or $f(0)=0$ and $f(n) \\neq 0$ for $n \\neq 0$. Set $n=0$ in (1) to get $$ 4 f(0)=f(f(0))^{2} $$ This means that $f(0) \\geqslant 0$. If $f(0)=0$, then $f(n) \\neq 0$ for all $n \\neq 0$, since we would otherwise have $$ n^{2}=n^{2}+4 f(n)=f(f(n))^{2}=f(0)^{2}=0 $$ If $f(0)>0$, then we know that $f(f(0))=f(0)+1$ from the first step, so $$ 4 f(0)=(f(0)+1)^{2} $$ which yields $f(0)=1$. Step 3: We discuss the values of $f(n)$ for $n<0$. Lemma. For every $n \\geqslant 1$, we have $f(-n)=-n+1$ or $f(-n)=n+1$. Moreover, if $f(-n)=$ $-n+1$ for some $n \\geqslant 1$, then also $f(-n+1)=-n+2$. Proof. We prove this statement by strong induction on $n$. For $n=1$, we get $$ 1+4 f(-1)=f(f(-1))^{2} $$ Thus $f(-1)$ needs to be nonnegative. If $f(-1)=0$, then $f(f(-1))=f(0)= \\pm 1$, so $f(0)=1$ (by our second step). Otherwise, we know that $f(f(-1))=f(-1)+1$, so $$ 1+4 f(-1)=(f(-1)+1)^{2} $$ which yields $f(-1)=2$ and thus establishes the base case. For the induction step, we consider two cases: - If $f(-n) \\leqslant-n$, then $$ f(f(-n))^{2}=(-n)^{2}+4 f(-n) \\leqslant n^{2}-4 n<(n-2)^{2} $$ so $|f(f(-n))| \\leqslant n-3$ (for $n=2$, this case cannot even occur). If $f(f(-n)) \\geqslant 0$, then we already know from the first two steps that $f(f(f(-n)))=f(f(-n))+1$, unless perhaps if $f(0)=0$ and $f(f(-n))=0$. However, the latter would imply $f(-n)=0$ (as shown in Step 2) and thus $n=0$, which is impossible. If $f(f(-n))<0$, we can apply the induction hypothesis to $f(f(-n))$. In either case, $f(f(f(-n)))= \\pm f(f(-n))+1$. Therefore, $$ f(-n)^{2}+4 f(f(-n))=f(f(f(-n)))^{2}=( \\pm f(f(-n))+1)^{2} $$ which gives us $$ \\begin{aligned} n^{2} & \\leqslant f(-n)^{2}=( \\pm f(f(-n))+1)^{2}-4 f(f(-n)) \\leqslant f(f(-n))^{2}+6|f(f(-n))|+1 \\\\ & \\leqslant(n-3)^{2}+6(n-3)+1=n^{2}-8 \\end{aligned} $$ a contradiction. - Thus, we are left with the case that $f(-n)>-n$. Now we argue as in the previous case: if $f(-n) \\geqslant 0$, then $f(f(-n))=f(-n)+1$ by the first two steps, since $f(0)=0$ and $f(-n)=0$ would imply $n=0$ (as seen in Step 2) and is thus impossible. If $f(-n)<0$, we can apply the induction hypothesis, so in any case we can infer that $f(f(-n))= \\pm f(-n)+1$. We obtain $$ (-n)^{2}+4 f(-n)=( \\pm f(-n)+1)^{2} $$ so either $$ n^{2}=f(-n)^{2}-2 f(-n)+1=(f(-n)-1)^{2} $$ which gives us $f(-n)= \\pm n+1$, or $$ n^{2}=f(-n)^{2}-6 f(-n)+1=(f(-n)-3)^{2}-8 $$ Since 1 and 9 are the only perfect squares whose difference is 8 , we must have $n=1$, which we have already considered. Finally, suppose that $f(-n)=-n+1$ for some $n \\geqslant 2$. Then $$ f(-n+1)^{2}=f(f(-n))^{2}=(-n)^{2}+4 f(-n)=(n-2)^{2} $$ so $f(-n+1)= \\pm(n-2)$. However, we already know that $f(-n+1)=-n+2$ or $f(-n+1)=n$, so $f(-n+1)=-n+2$. Combining everything we know, we find the solutions as stated in the answer: - One solution is given by $f(n)=n+1$ for all $n$. - If $f(n)$ is not always equal to $n+1$, then there is a largest integer $m$ (which cannot be positive) for which this is not the case. In view of the lemma that we proved, we must then have $f(n)=-n+1$ for any integer $n-a, \\\\ -n+1, & n \\leqslant-a ;\\end{cases}$ - or $f(n)= \\begin{cases}n+1, & n>0, \\\\ 0, & n=0, \\\\ -n+1, & n<0 .\\end{cases}$","solution":"Let us provide an alternative proof for Part II, which also proceeds in several steps. Step 1. Let $a$ be an arbitrary integer and $b=f(a)$. We first concentrate on the case where $|a|$ is sufficiently large. 1. If $b=0$, then (1) applied to $a$ yields $a^{2}=f(f(a))^{2}$, thus $$ f(a)=0 \\quad \\Rightarrow \\quad a= \\pm f(0) $$ From now on, we set $D=|f(0)|$. Throughout Step 1, we will assume that $a \\notin\\{-D, 0, D\\}$, thus $b \\neq 0$. 2. From (1), noticing that $f(f(a))$ and $a$ have the same parity, we get $$ 0 \\neq 4|b|=\\left|f(f(a))^{2}-a^{2}\\right| \\geqslant a^{2}-(|a|-2)^{2}=4|a|-4 $$ Hence we have $$ |b|=|f(a)| \\geqslant|a|-1 \\quad \\text { for } a \\notin\\{-D, 0, D\\} $$ For the rest of Step 1, we also assume that $|a| \\geqslant E=\\max \\{D+2,10\\}$. Then by (3) we have $|b| \\geqslant D+1$ and thus $|f(b)| \\geqslant D$. 3. Set $c=f(b)$; by (3), we have $|c| \\geqslant|b|-1$. Thus (1) yields $$ a^{2}+4 b=c^{2} \\geqslant(|b|-1)^{2}, $$ which implies $$ a^{2} \\geqslant(|b|-1)^{2}-4|b|=(|b|-3)^{2}-8>(|b|-4)^{2} $$ because $|b| \\geqslant|a|-1 \\geqslant 9$. Thus (3) can be refined to $$ |a|+3 \\geqslant|f(a)| \\geqslant|a|-1 \\quad \\text { for }|a| \\geqslant E $$ Now, from $c^{2}=a^{2}+4 b$ with $|b| \\in[|a|-1,|a|+3]$ we get $c^{2}=(a \\pm 2)^{2}+d$, where $d \\in\\{-16,-12,-8,-4,0,4,8\\}$. Since $|a \\pm 2| \\geqslant 8$, this can happen only if $c^{2}=(a \\pm 2)^{2}$, which in turn yields $b= \\pm a+1$. To summarise, $$ f(a)=1 \\pm a \\quad \\text { for }|a| \\geqslant E \\text {. } $$ We have shown that, with at most finitely many exceptions, $f(a)=1 \\pm a$. Thus it will be convenient for our second step to introduce the sets $$ Z_{+}=\\{a \\in \\mathbb{Z}: f(a)=a+1\\}, \\quad Z_{-}=\\{a \\in \\mathbb{Z}: f(a)=1-a\\}, \\quad \\text { and } \\quad Z_{0}=\\mathbb{Z} \\backslash\\left(Z_{+} \\cup Z_{-}\\right) $$ Step 2. Now we investigate the structure of the sets $Z_{+}, Z_{-}$, and $Z_{0}$. 4. Note that $f(E+1)=1 \\pm(E+1)$. If $f(E+1)=E+2$, then $E+1 \\in Z_{+}$. Otherwise we have $f(1+E)=-E$; then the original equation (1) with $n=E+1$ gives us $(E-1)^{2}=f(-E)^{2}$, so $f(-E)= \\pm(E-1)$. By (4) this may happen only if $f(-E)=1-E$, so in this case $-E \\in Z_{+}$. In any case we find that $Z_{+} \\neq \\varnothing$. 5. Now take any $a \\in Z_{+}$. We claim that every integer $x \\geqslant a$ also lies in $Z_{+}$. We proceed by induction on $x$, the base case $x=a$ being covered by our assumption. For the induction step, assume that $f(x-1)=x$ and plug $n=x-1$ into (1). We get $f(x)^{2}=(x+1)^{2}$, so either $f(x)=x+1$ or $f(x)=-(x+1)$. Assume that $f(x)=-(x+1)$ and $x \\neq-1$, since otherwise we already have $f(x)=x+1$. Plugging $n=x$ into (1), we obtain $f(-x-1)^{2}=(x-2)^{2}-8$, which may happen only if $x-2= \\pm 3$ and $f(-x-1)= \\pm 1$. Plugging $n=-x-1$ into (1), we get $f( \\pm 1)^{2}=(x+1)^{2} \\pm 4$, which in turn may happen only if $x+1 \\in\\{-2,0,2\\}$. Thus $x \\in\\{-1,5\\}$ and at the same time $x \\in\\{-3,-1,1\\}$, which gives us $x=-1$. Since this has already been excluded, we must have $f(x)=x+1$, which completes our induction. 6. Now we know that either $Z_{+}=\\mathbb{Z}$ (if $Z_{+}$is not bounded below), or $Z_{+}=\\left\\{a \\in \\mathbb{Z}: a \\geqslant a_{0}\\right\\}$, where $a_{0}$ is the smallest element of $Z_{+}$. In the former case, $f(n)=n+1$ for all $n \\in \\mathbb{Z}$, which is our first solution. So we assume in the following that $Z_{+}$is bounded below and has a smallest element $a_{0}$. If $Z_{0}=\\varnothing$, then we have $f(x)=x+1$ for $x \\geqslant a_{0}$ and $f(x)=1-x$ for $x\\ell^{2}$ then each happy configuration contains an empty $\\ell \\times \\ell$ square, but (ii) if $n \\leqslant \\ell^{2}$ then there exists a happy configuration not containing such a square. These two statements together yield the answer. (i). Assume that $n>\\ell^{2}$. Consider any happy configuration. There exists a row $R$ containing a rook in its leftmost square. Take $\\ell$ consecutive rows with $R$ being one of them. Their union $U$ contains exactly $\\ell$ rooks. Now remove the $n-\\ell^{2} \\geqslant 1$ leftmost columns from $U$ (thus at least one rook is also removed). The remaining part is an $\\ell^{2} \\times \\ell$ rectangle, so it can be split into $\\ell$ squares of size $\\ell \\times \\ell$, and this part contains at most $\\ell-1$ rooks. Thus one of these squares is empty. (ii). Now we assume that $n \\leqslant \\ell^{2}$. Firstly, we will construct a happy configuration with no empty $\\ell \\times \\ell$ square for the case $n=\\ell^{2}$. After that we will modify it to work for smaller values of $n$. Let us enumerate the rows from bottom to top as well as the columns from left to right by the numbers $0,1, \\ldots, \\ell^{2}-1$. Every square will be denoted, as usual, by the pair $(r, c)$ of its row and column numbers. Now we put the rooks on all squares of the form $(i \\ell+j, j \\ell+i)$ with $i, j=0,1, \\ldots, \\ell-1$ (the picture below represents this arrangement for $\\ell=3$ ). Since each number from 0 to $\\ell^{2}-1$ has a unique representation of the form $i \\ell+j(0 \\leqslant i, j \\leqslant \\ell-1)$, each row and each column contains exactly one rook. ![](https:\/\/cdn.mathpix.com\/cropped\/2024_11_18_62c85fb0b01b41aca29bg-31.jpg?height=524&width=527&top_left_y=1554&top_left_x=773) Next, we show that each $\\ell \\times \\ell$ square $A$ on the board contains a rook. Consider such a square $A$, and consider $\\ell$ consecutive rows the union of which contains $A$. Let the lowest of these rows have number $p \\ell+q$ with $0 \\leqslant p, q \\leqslant \\ell-1$ (notice that $p \\ell+q \\leqslant \\ell^{2}-\\ell$ ). Then the rooks in this union are placed in the columns with numbers $q \\ell+p,(q+1) \\ell+p, \\ldots,(\\ell-1) \\ell+p$, $p+1, \\ell+(p+1), \\ldots,(q-1) \\ell+p+1$, or, putting these numbers in increasing order, $$ p+1, \\ell+(p+1), \\ldots,(q-1) \\ell+(p+1), q \\ell+p,(q+1) \\ell+p, \\ldots,(\\ell-1) \\ell+p $$ One readily checks that the first number in this list is at most $\\ell-1$ (if $p=\\ell-1$, then $q=0$, and the first listed number is $q \\ell+p=\\ell-1$ ), the last one is at least $(\\ell-1) \\ell$, and the difference between any two consecutive numbers is at most $\\ell$. Thus, one of the $\\ell$ consecutive columns intersecting $A$ contains a number listed above, and the rook in this column is inside $A$, as required. The construction for $n=\\ell^{2}$ is established. It remains to construct a happy configuration of rooks not containing an empty $\\ell \\times \\ell$ square for $n<\\ell^{2}$. In order to achieve this, take the construction for an $\\ell^{2} \\times \\ell^{2}$ square described above and remove the $\\ell^{2}-n$ bottom rows together with the $\\ell^{2}-n$ rightmost columns. We will have a rook arrangement with no empty $\\ell \\times \\ell$ square, but several rows and columns may happen to be empty. Clearly, the number of empty rows is equal to the number of empty columns, so one can find a bijection between them, and put a rook on any crossing of an empty row and an empty column corresponding to each other. Comment. Part (i) allows several different proofs. E.g., in the last paragraph of the solution, it suffices to deal only with the case $n=\\ell^{2}+1$. Notice now that among the four corner squares, at least one is empty. So the rooks in its row and in its column are distinct. Now, deleting this row and column we obtain an $\\ell^{2} \\times \\ell^{2}$ square with $\\ell^{2}-1$ rooks in it. This square can be partitioned into $\\ell^{2}$ squares of size $\\ell \\times \\ell$, so one of them is empty.","tier":0} +{"problem_type":"Combinatorics","problem_label":"C4","problem":"Construct a tetromino by attaching two $2 \\times 1$ dominoes along their longer sides such that the midpoint of the longer side of one domino is a corner of the other domino. This construction yields two kinds of tetrominoes with opposite orientations. Let us call them Sand Z-tetrominoes, respectively. ![](https:\/\/cdn.mathpix.com\/cropped\/2024_11_18_62c85fb0b01b41aca29bg-33.jpg?height=238&width=809&top_left_y=446&top_left_x=629) Assume that a lattice polygon $P$ can be tiled with S-tetrominoes. Prove than no matter how we tile $P$ using only S - and Z -tetrominoes, we always use an even number of Z -tetrominoes. (Hungary)","solution":"We may assume that polygon $P$ is the union of some squares of an infinite chessboard. Colour the squares of the chessboard with two colours as the figure below illustrates. ![](https:\/\/cdn.mathpix.com\/cropped\/2024_11_18_62c85fb0b01b41aca29bg-33.jpg?height=663&width=866&top_left_y=979&top_left_x=601) Observe that no matter how we tile $P$, any S-tetromino covers an even number of black squares, whereas any Z-tetromino covers an odd number of them. As $P$ can be tiled exclusively by S-tetrominoes, it contains an even number of black squares. But if some S-tetrominoes and some Z-tetrominoes cover an even number of black squares, then the number of Z-tetrominoes must be even. Comment. An alternative approach makes use of the following two colourings, which are perhaps somewhat more natural: ![](https:\/\/cdn.mathpix.com\/cropped\/2024_11_18_62c85fb0b01b41aca29bg-33.jpg?height=371&width=1297&top_left_y=2071&top_left_x=385) Let $s_{1}$ and $s_{2}$ be the number of $S$-tetrominoes of the first and second type (as shown in the figure above) respectively that are used in a tiling of $P$. Likewise, let $z_{1}$ and $z_{2}$ be the number of $Z$-tetrominoes of the first and second type respectively. The first colouring shows that $s_{1}+z_{2}$ is invariant modulo 2 , the second colouring shows that $s_{1}+z_{1}$ is invariant modulo 2 . Adding these two conditions, we find that $z_{1}+z_{2}$ is invariant modulo 2 , which is what we have to prove. Indeed, the sum of the two colourings (regarding white as 0 and black as 1 and adding modulo 2) is the colouring shown in the solution.","tier":0} +{"problem_type":"Combinatorics","problem_label":"C4","problem":"Construct a tetromino by attaching two $2 \\times 1$ dominoes along their longer sides such that the midpoint of the longer side of one domino is a corner of the other domino. This construction yields two kinds of tetrominoes with opposite orientations. Let us call them Sand Z-tetrominoes, respectively. ![](https:\/\/cdn.mathpix.com\/cropped\/2024_11_18_62c85fb0b01b41aca29bg-33.jpg?height=238&width=809&top_left_y=446&top_left_x=629) Assume that a lattice polygon $P$ can be tiled with S-tetrominoes. Prove than no matter how we tile $P$ using only S - and Z -tetrominoes, we always use an even number of Z -tetrominoes. (Hungary)","solution":"Let us assign coordinates to the squares of the infinite chessboard in such a way that the squares of $P$ have nonnegative coordinates only, and that the first coordinate increases as one moves to the right, while the second coordinate increases as one moves upwards. Write the integer $3^{i} \\cdot(-3)^{j}$ into the square with coordinates $(i, j)$, as in the following figure: | $\\vdots$ | | | | | | | :---: | :---: | :---: | :---: | :---: | :---: | | 81 | $\\vdots$ | | | | | | -27 | -81 | $\\vdots$ | | | | | 9 | 27 | 81 | $\\cdots$ | | | | -3 | -9 | -27 | -81 | $\\cdots$ | | | 1 | 3 | 9 | 27 | 81 | $\\cdots$ | The sum of the numbers written into four squares that can be covered by an $S$-tetromino is either of the form $$ 3^{i} \\cdot(-3)^{j} \\cdot\\left(1+3+3 \\cdot(-3)+3^{2} \\cdot(-3)\\right)=-32 \\cdot 3^{i} \\cdot(-3)^{j} $$ (for the first type of $S$-tetrominoes), or of the form $$ 3^{i} \\cdot(-3)^{j} \\cdot\\left(3+3 \\cdot(-3)+(-3)+(-3)^{2}\\right)=0 $$ and thus divisible by 32 . For this reason, the sum of the numbers written into the squares of $P$, and thus also the sum of the numbers covered by $Z$-tetrominoes in the second covering, is likewise divisible by 32 . Now the sum of the entries of a $Z$-tetromino is either of the form $$ 3^{i} \\cdot(-3)^{j} \\cdot\\left(3+3^{2}+(-3)+3 \\cdot(-3)\\right)=0 $$ (for the first type of $Z$-tetrominoes), or of the form $$ 3^{i} \\cdot(-3)^{j} \\cdot\\left(1+(-3)+3 \\cdot(-3)+3 \\cdot(-3)^{2}\\right)=16 \\cdot 3^{i} \\cdot(-3)^{j} $$ i.e., 16 times an odd number. Thus in order to obtain a total that is divisible by 32 , an even number of the latter kind of $Z$-tetrominoes needs to be used. Rotating everything by $90^{\\circ}$, we find that the number of $Z$-tetrominoes of the first kind is even as well. So we have even proven slightly more than necessary. Comment 1. In the second solution, 3 and -3 can be replaced by other combinations as well. For example, for any positive integer $a \\equiv 3(\\bmod 4)$, we can write $a^{i} \\cdot(-a)^{j}$ into the square with coordinates $(i, j)$ and apply the same argument. Comment 2. As the second solution shows, we even have the stronger result that the parity of the number of each of the four types of tetrominoes in a tiling of $P$ by S - and Z-tetrominoes is an invariant of $P$. This also remains true if there is no tiling of $P$ that uses only S-tetrominoes.","tier":0} +{"problem_type":"Combinatorics","problem_label":"C5","problem":"Consider $n \\geqslant 3$ lines in the plane such that no two lines are parallel and no three have a common point. These lines divide the plane into polygonal regions; let $\\mathcal{F}$ be the set of regions having finite area. Prove that it is possible to colour $\\lceil\\sqrt{n \/ 2}\\rceil$ of the lines blue in such a way that no region in $\\mathcal{F}$ has a completely blue boundary. (Austria)","solution":"Let $L$ be the given set of lines. Choose a maximal (by inclusion) subset $B \\subseteq L$ such that when we colour the lines of $B$ blue, no region in $\\mathcal{F}$ has a completely blue boundary. Let $|B|=k$. We claim that $k \\geqslant\\lceil\\sqrt{n \/ 2}\\rceil$. Let us colour all the lines of $L \\backslash B$ red. Call a point blue if it is the intersection of two blue lines. Then there are $\\binom{k}{2}$ blue points. Now consider any red line $\\ell$. By the maximality of $B$, there exists at least one region $A \\in \\mathcal{F}$ whose only red side lies on $\\ell$. Since $A$ has at least three sides, it must have at least one blue vertex. Let us take one such vertex and associate it to $\\ell$. Since each blue point belongs to four regions (some of which may be unbounded), it is associated to at most four red lines. Thus the total number of red lines is at most $4\\binom{k}{2}$. On the other hand, this number is $n-k$, so $$ n-k \\leqslant 2 k(k-1), \\quad \\text { thus } \\quad n \\leqslant 2 k^{2}-k \\leqslant 2 k^{2} $$ and finally $k \\geqslant\\lceil\\sqrt{n \/ 2}\\rceil$, which gives the desired result. Comment 1. The constant factor in the estimate can be improved in different ways; we sketch two of them below. On the other hand, the Problem Selection Committee is not aware of any results showing that it is sometimes impossible to colour $k$ lines satisfying the desired condition for $k \\gg \\sqrt{n}$. In this situation we find it more suitable to keep the original formulation of the problem. 1. Firstly, we show that in the proof above one has in fact $k=|B| \\geqslant\\lceil\\sqrt{2 n \/ 3}\\rceil$. Let us make weighted associations as follows. Let a region $A$ whose only red side lies on $\\ell$ have $k$ vertices, so that $k-2$ of them are blue. We associate each of these blue vertices to $\\ell$, and put the weight $\\frac{1}{k-2}$ on each such association. So the sum of the weights of all the associations is exactly $n-k$. Now, one may check that among the four regions adjacent to a blue vertex $v$, at most two are triangles. This means that the sum of the weights of all associations involving $v$ is at most $1+1+\\frac{1}{2}+\\frac{1}{2}=3$. This leads to the estimate $$ n-k \\leqslant 3\\binom{k}{2} $$ or $$ 2 n \\leqslant 3 k^{2}-k<3 k^{2} $$ which yields $k \\geqslant\\lceil\\sqrt{2 n \/ 3}\\rceil$. 2. Next, we even show that $k=|B| \\geqslant\\lceil\\sqrt{n}\\rceil$. For this, we specify the process of associating points to red lines in one more different way. Call a point red if it lies on a red line as well as on a blue line. Consider any red line $\\ell$, and take an arbitrary region $A \\in \\mathcal{F}$ whose only red side lies on $\\ell$. Let $r^{\\prime}, r, b_{1}, \\ldots, b_{k}$ be its vertices in clockwise order with $r^{\\prime}, r \\in \\ell$; then the points $r^{\\prime}, r$ are red, while all the points $b_{1}, \\ldots, b_{k}$ are blue. Let us associate to $\\ell$ the red point $r$ and the blue point $b_{1}$. One may notice that to each pair of a red point $r$ and a blue point $b$, at most one red line can be associated, since there is at most one region $A$ having $r$ and $b$ as two clockwise consecutive vertices. We claim now that at most two red lines are associated to each blue point $b$; this leads to the desired bound $$ n-k \\leqslant 2\\binom{k}{2} \\quad \\Longleftrightarrow \\quad n \\leqslant k^{2} $$ Assume, to the contrary, that three red lines $\\ell_{1}, \\ell_{2}$, and $\\ell_{3}$ are associated to the same blue point $b$. Let $r_{1}, r_{2}$, and $r_{3}$ respectively be the red points associated to these lines; all these points are distinct. The point $b$ defines four blue rays, and each point $r_{i}$ is the red point closest to $b$ on one of these rays. So we may assume that the points $r_{2}$ and $r_{3}$ lie on one blue line passing through $b$, while $r_{1}$ lies on the other one. ![](https:\/\/cdn.mathpix.com\/cropped\/2024_11_18_62c85fb0b01b41aca29bg-36.jpg?height=432&width=532&top_left_y=475&top_left_x=768) Now consider the region $A$ used to associate $r_{1}$ and $b$ with $\\ell_{1}$. Three of its clockwise consecutive vertices are $r_{1}, b$, and either $r_{2}$ or $r_{3}$ (say, $r_{2}$ ). Since $A$ has only one red side, it can only be the triangle $r_{1} b r_{2}$; but then both $\\ell_{1}$ and $\\ell_{2}$ pass through $r_{2}$, as well as some blue line. This is impossible by the problem assumptions. Comment 2. The condition that the lines be non-parallel is essentially not used in the solution, nor in the previous comment; thus it may be omitted.","tier":0} +{"problem_type":"Combinatorics","problem_label":"C6","problem":"We are given an infinite deck of cards, each with a real number on it. For every real number $x$, there is exactly one card in the deck that has $x$ written on it. Now two players draw disjoint sets $A$ and $B$ of 100 cards each from this deck. We would like to define a rule that declares one of them a winner. This rule should satisfy the following conditions: 1. The winner only depends on the relative order of the 200 cards: if the cards are laid down in increasing order face down and we are told which card belongs to which player, but not what numbers are written on them, we can still decide the winner. 2. If we write the elements of both sets in increasing order as $A=\\left\\{a_{1}, a_{2}, \\ldots, a_{100}\\right\\}$ and $B=\\left\\{b_{1}, b_{2}, \\ldots, b_{100}\\right\\}$, and $a_{i}>b_{i}$ for all $i$, then $A$ beats $B$. 3. If three players draw three disjoint sets $A, B, C$ from the deck, $A$ beats $B$ and $B$ beats $C$, then $A$ also beats $C$. How many ways are there to define such a rule? Here, we consider two rules as different if there exist two sets $A$ and $B$ such that $A$ beats $B$ according to one rule, but $B$ beats $A$ according to the other. (Russia) Answer. 100.","solution":"We prove a more general statement for sets of cardinality $n$ (the problem being the special case $n=100$, then the answer is $n$ ). In the following, we write $A>B$ or $Bb_{k}$. This rule clearly satisfies all three conditions, and the rules corresponding to different $k$ are all different. Thus there are at least $n$ different rules. Part II. Now we have to prove that there is no other way to define such a rule. Suppose that our rule satisfies the conditions, and let $k \\in\\{1,2, \\ldots, n\\}$ be minimal with the property that $$ A_{k}=\\{1,2, \\ldots, k, n+k+1, n+k+2, \\ldots, 2 n\\} \\prec B_{k}=\\{k+1, k+2, \\ldots, n+k\\} $$ Clearly, such a $k$ exists, since this holds for $k=n$ by assumption. Now consider two disjoint sets $X=\\left\\{x_{1}, x_{2}, \\ldots, x_{n}\\right\\}$ and $Y=\\left\\{y_{1}, y_{2}, \\ldots, y_{n}\\right\\}$, both in increasing order (i.e., $x_{1}B_{k-1}$ by our choice of $k$, we also have $U>W$ (if $k=1$, this is trivial). - The elements of $V \\cup W$ are ordered in the same way as those of $A_{k} \\cup B_{k}$, and since $A_{k} \\prec B_{k}$ by our choice of $k$, we also have $V \\prec W$. It follows that $$ X \\prec V \\prec W \\prec U \\prec Y $$ so $Xb_{i}$ for all $i$, then $A$ beats $B$. 3. If three players draw three disjoint sets $A, B, C$ from the deck, $A$ beats $B$ and $B$ beats $C$, then $A$ also beats $C$. How many ways are there to define such a rule? Here, we consider two rules as different if there exist two sets $A$ and $B$ such that $A$ beats $B$ according to one rule, but $B$ beats $A$ according to the other. (Russia) Answer. 100.","solution":"Another possible approach to Part II of this problem is induction on $n$. For $n=1$, there is trivially only one rule in view of the second condition. In the following, we assume that our claim (namely, that there are no possible rules other than those given in Part I) holds for $n-1$ in place of $n$. We start with the following observation: Claim. At least one of the two relations $$ (\\{2\\} \\cup\\{2 i-1 \\mid 2 \\leqslant i \\leqslant n\\})<(\\{1\\} \\cup\\{2 i \\mid 2 \\leqslant i \\leqslant n\\}) $$ and $$ (\\{2 i-1 \\mid 1 \\leqslant i \\leqslant n-1\\} \\cup\\{2 n\\})<(\\{2 i \\mid 1 \\leqslant i \\leqslant n-1\\} \\cup\\{2 n-1\\}) $$ holds. Proof. Suppose that the first relation does not hold. Since our rule may only depend on the relative order, we must also have $$ (\\{2\\} \\cup\\{3 i-2 \\mid 2 \\leqslant i \\leqslant n-1\\} \\cup\\{3 n-2\\})>(\\{1\\} \\cup\\{3 i-1 \\mid 2 \\leqslant i \\leqslant n-1\\} \\cup\\{3 n\\}) . $$ Likewise, if the second relation does not hold, then we must also have $$ (\\{1\\} \\cup\\{3 i-1 \\mid 2 \\leqslant i \\leqslant n-1\\} \\cup\\{3 n\\})>(\\{3\\} \\cup\\{3 i \\mid 2 \\leqslant i \\leqslant n-1\\} \\cup\\{3 n-1\\}) $$ Now condition 3 implies that $$ (\\{2\\} \\cup\\{3 i-2 \\mid 2 \\leqslant i \\leqslant n-1\\} \\cup\\{3 n-2\\})>(\\{3\\} \\cup\\{3 i \\mid 2 \\leqslant i \\leqslant n-1\\} \\cup\\{3 n-1\\}) $$ which contradicts the second condition. Now we distinguish two cases, depending on which of the two relations actually holds: First case: $(\\{2\\} \\cup\\{2 i-1 \\mid 2 \\leqslant i \\leqslant n\\})<(\\{1\\} \\cup\\{2 i \\mid 2 \\leqslant i \\leqslant n\\})$. Let $A=\\left\\{a_{1}, a_{2}, \\ldots, a_{n}\\right\\}$ and $B=\\left\\{b_{1}, b_{2}, \\ldots, b_{n}\\right\\}$ be two disjoint sets, both in increasing order. We claim that the winner can be decided only from the values of $a_{2}, \\ldots, a_{n}$ and $b_{2}, \\ldots, b_{n}$, while $a_{1}$ and $b_{1}$ are actually irrelevant. Suppose that this was not the case, and assume without loss of generality that $a_{2}0$ be smaller than half the distance between any two of the numbers in $B_{x} \\cup B_{y} \\cup A$. For any set $M$, let $M \\pm \\varepsilon$ be the set obtained by adding\/subtracting $\\varepsilon$ to all elements of $M$. By our choice of $\\varepsilon$, the relative order of the elements of $\\left(B_{y}+\\varepsilon\\right) \\cup A$ is still the same as for $B_{y} \\cup A$, while the relative order of the elements of $\\left(B_{x}-\\varepsilon\\right) \\cup A$ is still the same as for $B_{x} \\cup A$. Thus $A \\prec B_{x}-\\varepsilon$, but $A>B_{y}+\\varepsilon$. Moreover, if $y>x$, then $B_{x}-\\varepsilon \\prec B_{y}+\\varepsilon$ by condition 2, while otherwise the relative order of the elements in $\\left(B_{x}-\\varepsilon\\right) \\cup\\left(B_{y}+\\varepsilon\\right)$ is the same as for the two sets $\\{2\\} \\cup\\{2 i-1 \\mid 2 \\leqslant i \\leqslant n\\}$ and $\\{1\\} \\cup\\{2 i \\mid 2 \\leqslant i \\leqslant n\\}$, so that $B_{x}-\\varepsilon_{\\sigma} B$ if and only if $\\left(a_{\\sigma(1)}, \\ldots, a_{\\sigma(n)}\\right)$ is lexicographically greater than $\\left(b_{\\sigma(1)}, \\ldots, b_{\\sigma(n)}\\right)$. It seems, however, that this formulation adds rather more technicalities to the problem than additional ideas. This page is intentionally left blank","tier":0} +{"problem_type":"Combinatorics","problem_label":"C8","problem":"A card deck consists of 1024 cards. On each card, a set of distinct decimal digits is written in such a way that no two of these sets coincide (thus, one of the cards is empty). Two players alternately take cards from the deck, one card per turn. After the deck is empty, each player checks if he can throw out one of his cards so that each of the ten digits occurs on an even number of his remaining cards. If one player can do this but the other one cannot, the one who can is the winner; otherwise a draw is declared. Determine all possible first moves of the first player after which he has a winning strategy. (Russia) Answer. All the moves except for taking the empty card.","solution":"Let us identify each card with the set of digits written on it. For any collection of cards $C_{1}, C_{2}, \\ldots, C_{k}$ denote by their sum the set $C_{1} \\triangle C_{2} \\triangle \\cdots \\triangle C_{k}$ consisting of all elements belonging to an odd number of the $C_{i}$ 's. Denote the first and the second player by $\\mathcal{F}$ and $\\mathcal{S}$, respectively. Since each digit is written on exactly 512 cards, the sum of all the cards is $\\varnothing$. Therefore, at the end of the game the sum of all the cards of $\\mathcal{F}$ will be the same as that of $\\mathcal{S}$; denote this sum by $C$. Then the player who took $C$ can throw it out and get the desired situation, while the other one cannot. Thus, the player getting card $C$ wins, and no draw is possible. Now, given a nonempty card $B$, one can easily see that all the cards can be split into 512 pairs of the form $(X, X \\triangle B)$ because $(X \\triangle B) \\triangle B=X$. The following lemma shows a property of such a partition that is important for the solution. Lemma. Let $B \\neq \\varnothing$ be some card. Let us choose 512 cards so that exactly one card is chosen from every pair $(X, X \\triangle B)$. Then the sum of all chosen cards is either $\\varnothing$ or $B$. Proof. Let $b$ be some element of $B$. Enumerate the pairs; let $X_{i}$ be the card not containing $b$ in the $i^{\\text {th }}$ pair, and let $Y_{i}$ be the other card in this pair. Then the sets $X_{i}$ are exactly all the sets not containing $b$, therefore each digit $a \\neq b$ is written on exactly 256 of these cards, so $X_{1} \\triangle X_{2} \\triangle \\cdots \\triangle X_{512}=\\varnothing$. Now, if we replace some summands in this sum by the other elements from their pairs, we will simply add $B$ several times to this sum, thus the sum will either remain unchanged or change by $B$, as required. Now we consider two cases. Case 1. Assume that $\\mathcal{F}$ takes the card $\\varnothing$ on his first move. In this case, we present a winning strategy for $\\mathcal{S}$. Let $\\mathcal{S}$ take an arbitrary card $A$. Assume that $\\mathcal{F}$ takes card $B$ after that; then $\\mathcal{S}$ takes $A \\triangle B$. Split all 1024 cards into 512 pairs of the form $(X, X \\triangle B)$; we call two cards in one pair partners. Then the four cards taken so far form two pairs $(\\varnothing, B)$ and $(A, A \\triangle B)$ belonging to $\\mathcal{F}$ and $\\mathcal{S}$, respectively. On each of the subsequent moves, when $\\mathcal{F}$ takes some card, $\\mathcal{S}$ should take the partner of this card in response. Consider the situation at the end of the game. Let us for a moment replace card $A$ belonging to $\\mathcal{S}$ by $\\varnothing$. Then he would have one card from each pair; by our lemma, the sum of all these cards would be either $\\varnothing$ or $B$. Now, replacing $\\varnothing$ back by $A$ we get that the actual sum of the cards of $\\mathcal{S}$ is either $A$ or $A \\triangle B$, and he has both these cards. Thus $\\mathcal{S}$ wins. Case 2. Now assume that $\\mathcal{F}$ takes some card $A \\neq \\varnothing$ on his first move. Let us present a winning strategy for $\\mathcal{F}$ in this case. Assume that $\\mathcal{S}$ takes some card $B \\neq \\varnothing$ on his first move; then $\\mathcal{F}$ takes $A \\triangle B$. Again, let us split all the cards into pairs of the form $(X, X \\triangle B)$; then the cards which have not been taken yet form several complete pairs and one extra element (card $\\varnothing$ has not been taken while its partner $B$ has). Now, on each of the subsequent moves, if $\\mathcal{S}$ takes some element from a complete pair, then $\\mathcal{F}$ takes its partner. If $\\mathcal{S}$ takes the extra element, then $\\mathcal{F}$ takes an arbitrary card $Y$, and the partner of $Y$ becomes the new extra element. Thus, on his last move $\\mathcal{S}$ is forced to take the extra element. After that player $\\mathcal{F}$ has cards $A$ and $A \\triangle B$, player $\\mathcal{S}$ has cards $B$ and $\\varnothing$, and $\\mathcal{F}$ has exactly one element from every other pair. Thus the situation is the same as in the previous case with roles reversed, and $\\mathcal{F}$ wins. Finally, if $\\mathcal{S}$ takes $\\varnothing$ on his first move then $\\mathcal{F}$ denotes any card which has not been taken yet by $B$ and takes $A \\triangle B$. After that, the same strategy as above is applicable. Comment 1. If one wants to avoid the unusual question about the first move, one may change the formulation as follows. (The difficulty of the problem would decrease somewhat.) A card deck consists of 1023 cards; on each card, a nonempty set of distinct decimal digits is written in such a way that no two of these sets coincide. Two players alternately take cards from the deck, one card per turn. When the deck is empty, each player checks if he can throw out one of his cards so that for each of the ten digits, he still holds an even number of cards with this digit. If one player can do this but the other one cannot, the one who can is the winner; otherwise a draw is declared. Determine which of the players (if any) has a winning strategy. The winner in this version is the first player. The analysis of the game from the first two paragraphs of the previous solution applies to this version as well, except for the case $C=\\varnothing$ in which the result is a draw. Then the strategy for $\\mathcal{S}$ in Case 1 works for $\\mathcal{F}$ in this version: the sum of all his cards at the end is either $A$ or $A \\triangle B$, thus nonempty in both cases. Comment 2. Notice that all the cards form a vector space over $\\mathbb{F}_{2}$, with $\\triangle$ the operation of addition. Due to the automorphisms of this space, all possibilities for $\\mathcal{F}$ 's first move except $\\varnothing$ are equivalent. The same holds for the response by $\\mathcal{S}$ if $\\mathcal{F}$ takes the card $\\varnothing$ on his first move. Comment 3. It is not that hard to show that in the initial game, $\\mathcal{F}$ has a winning move, by the idea of \"strategy stealing\". Namely, assume that $\\mathcal{S}$ has a winning strategy. Let us take two card decks and start two games, in which $\\mathcal{S}$ will act by his strategy. In the first game, $\\mathcal{F}$ takes an arbitrary card $A_{1}$; assume that $\\mathcal{S}$ takes some $B_{1}$ in response. Then $\\mathcal{F}$ takes the card $B_{1}$ at the second game; let the response by $\\mathcal{S}$ be $A_{2}$. Then $\\mathcal{F}$ takes $A_{2}$ in the first game and gets a response $B_{2}$, and so on. This process stops at some moment when in the second game $\\mathcal{S}$ takes $A_{i}=A_{1}$. At this moment the players hold the same sets of cards in both games, but with roles reversed. Now, if some cards remain in the decks, $\\mathcal{F}$ takes an arbitrary card from the first deck starting a similar cycle. At the end of the game, player $\\mathcal{F}$ 's cards in the first game are exactly player $\\mathcal{S}$ 's cards in the second game, and vice versa. Thus in one of the games $\\mathcal{F}$ will win, which is impossible by our assumption. One may notice that the strategy in Case 2 is constructed exactly in this way from the strategy in Case 1 . This is possible since every response by $\\mathcal{S}$ wins if $\\mathcal{F}$ takes the card $\\varnothing$ on his first move.","tier":0} +{"problem_type":"Combinatorics","problem_label":"C9","problem":"There are $n$ circles drawn on a piece of paper in such a way that any two circles intersect in two points, and no three circles pass through the same point. Turbo the snail slides along the circles in the following fashion. Initially he moves on one of the circles in clockwise direction. Turbo always keeps sliding along the current circle until he reaches an intersection with another circle. Then he continues his journey on this new circle and also changes the direction of moving, i.e. from clockwise to anticlockwise or vice versa. Suppose that Turbo's path entirely covers all circles. Prove that $n$ must be odd. (India)","solution":"Replace every cross (i.e. intersection of two circles) by two small circle arcs that indicate the direction in which the snail should leave the cross (see Figure 1.1). Notice that the placement of the small arcs does not depend on the direction of moving on the curves; no matter which direction the snail is moving on the circle arcs, he will follow the same curves (see Figure 1.2). In this way we have a set of curves, that are the possible paths of the snail. Call these curves snail orbits or just orbits. Every snail orbit is a simple closed curve that has no intersection with any other orbit. ![](https:\/\/cdn.mathpix.com\/cropped\/2024_11_18_62c85fb0b01b41aca29bg-45.jpg?height=355&width=375&top_left_y=1025&top_left_x=406) Figure 1.1 ![](https:\/\/cdn.mathpix.com\/cropped\/2024_11_18_62c85fb0b01b41aca29bg-45.jpg?height=347&width=670&top_left_y=1020&top_left_x=930) Figure 1.2 We prove the following, more general statement. (*) In any configuration of $n$ circles such that no two of them are tangent, the number of snail orbits has the same parity as the number $n$. (Note that it is not assumed that all circle pairs intersect.) This immediately solves the problem. Let us introduce the following operation that will be called flipping a cross. At a cross, remove the two small arcs of the orbits, and replace them by the other two arcs. Hence, when the snail arrives at a flipped cross, he will continue on the other circle as before, but he will preserve the orientation in which he goes along the circle arcs (see Figure 2). ![](https:\/\/cdn.mathpix.com\/cropped\/2024_11_18_62c85fb0b01b41aca29bg-45.jpg?height=292&width=735&top_left_y=1964&top_left_x=663) Figure 2 Consider what happens to the number of orbits when a cross is flipped. Denote by $a, b, c$, and $d$ the four arcs that meet at the cross such that $a$ and $b$ belong to the same circle. Before the flipping $a$ and $b$ were connected to $c$ and $d$, respectively, and after the flipping $a$ and $b$ are connected to $d$ and $c$, respectively. The orbits passing through the cross are closed curves, so each of the $\\operatorname{arcs} a, b, c$, and $d$ is connected to another one by orbits outside the cross. We distinguish three cases. Case 1: $a$ is connected to $b$ and $c$ is connected to $d$ by the orbits outside the cross (see Figure 3.1). We show that this case is impossible. Remove the two small arcs at the cross, connect $a$ to $b$, and connect $c$ to $d$ at the cross. Let $\\gamma$ be the new closed curve containing $a$ and $b$, and let $\\delta$ be the new curve that connects $c$ and $d$. These two curves intersect at the cross. So one of $c$ and $d$ is inside $\\gamma$ and the other one is outside $\\gamma$. Then the two closed curves have to meet at least one more time, but this is a contradiction, since no orbit can intersect itself. ![](https:\/\/cdn.mathpix.com\/cropped\/2024_11_18_62c85fb0b01b41aca29bg-46.jpg?height=247&width=333&top_left_y=499&top_left_x=239) Figure 3.1 ![](https:\/\/cdn.mathpix.com\/cropped\/2024_11_18_62c85fb0b01b41aca29bg-46.jpg?height=232&width=552&top_left_y=501&top_left_x=632) Figure 3.2 ![](https:\/\/cdn.mathpix.com\/cropped\/2024_11_18_62c85fb0b01b41aca29bg-46.jpg?height=235&width=575&top_left_y=496&top_left_x=1249) Figure 3.3 Case 2: $a$ is connected to $c$ and $b$ is connected to $d$ (see Figure 3.2). Before the flipping $a$ and $c$ belong to one orbit and $b$ and $d$ belong to another orbit. Flipping the cross merges the two orbits into a single orbit. Hence, the number of orbits decreases by 1. Case 3: $a$ is connected to $d$ and $b$ is connected to $c$ (see Figure 3.3). Before the flipping the arcs $a, b, c$, and $d$ belong to a single orbit. Flipping the cross splits that orbit in two. The number of orbits increases by 1. As can be seen, every flipping decreases or increases the number of orbits by one, thus changes its parity. Now flip every cross, one by one. Since every pair of circles has 0 or 2 intersections, the number of crosses is even. Therefore, when all crosses have been flipped, the original parity of the number of orbits is restored. So it is sufficient to prove (*) for the new configuration, where all crosses are flipped. Of course also in this new configuration the (modified) orbits are simple closed curves not intersecting each other. Orient the orbits in such a way that the snail always moves anticlockwise along the circle arcs. Figure 4 shows the same circles as in Figure 1 after flipping all crosses and adding orientation. (Note that this orientation may be different from the orientation of the orbit as a planar curve; the orientation of every orbit may be negative as well as positive, like the middle orbit in Figure 4.) If the snail moves around an orbit, the total angle change in his moving direction, the total curvature, is either $+2 \\pi$ or $-2 \\pi$, depending on the orientation of the orbit. Let $P$ and $N$ be the number of orbits with positive and negative orientation, respectively. Then the total curvature of all orbits is $(P-N) \\cdot 2 \\pi$. ![](https:\/\/cdn.mathpix.com\/cropped\/2024_11_18_62c85fb0b01b41aca29bg-46.jpg?height=372&width=412&top_left_y=1961&top_left_x=499) Figure 4 ![](https:\/\/cdn.mathpix.com\/cropped\/2024_11_18_62c85fb0b01b41aca29bg-46.jpg?height=213&width=500&top_left_y=2115&top_left_x=1052) Figure 5 Double-count the total curvature of all orbits. Along every circle the total curvature is $2 \\pi$. At every cross, the two turnings make two changes with some angles having the same absolute value but opposite signs, as depicted in Figure 5. So the changes in the direction at the crosses cancel out. Hence, the total curvature is $n \\cdot 2 \\pi$. Now we have $(P-N) \\cdot 2 \\pi=n \\cdot 2 \\pi$, so $P-N=n$. The number of (modified) orbits is $P+N$, that has a same parity as $P-N=n$.","tier":0} +{"problem_type":"Combinatorics","problem_label":"C9","problem":"There are $n$ circles drawn on a piece of paper in such a way that any two circles intersect in two points, and no three circles pass through the same point. Turbo the snail slides along the circles in the following fashion. Initially he moves on one of the circles in clockwise direction. Turbo always keeps sliding along the current circle until he reaches an intersection with another circle. Then he continues his journey on this new circle and also changes the direction of moving, i.e. from clockwise to anticlockwise or vice versa. Suppose that Turbo's path entirely covers all circles. Prove that $n$ must be odd. (India)","solution":"We present a different proof of (*). We perform a sequence of small modification steps on the configuration of the circles in such a way that at the end they have no intersection at all (see Figure 6.1). We use two kinds of local changes to the structure of the orbits (see Figure 6.2): - Type-1 step: An arc of a circle is moved over an arc of another circle; such a step creates or removes two intersections. - Type-2 step: An arc of a circle is moved through the intersection of two other circles. ![](https:\/\/cdn.mathpix.com\/cropped\/2024_11_18_62c85fb0b01b41aca29bg-47.jpg?height=313&width=701&top_left_y=803&top_left_x=295) Figure 6.1 ![](https:\/\/cdn.mathpix.com\/cropped\/2024_11_18_62c85fb0b01b41aca29bg-47.jpg?height=292&width=189&top_left_y=819&top_left_x=1139) Figure 6.2 We assume that in every step only one circle is moved, and that this circle is moved over at most one arc or intersection point of other circles. We will show that the parity of the number of orbits does not change in any step. As every circle becomes a separate orbit at the end of the procedure, this fact proves (*). Consider what happens to the number of orbits when a Type-1 step is performed. The two intersection points are created or removed in a small neighbourhood. Denote some points of the two circles where they enter or leave this neighbourhood by $a, b, c$, and $d$ in this order around the neighbourhood; let $a$ and $b$ belong to one circle and let $c$ and $d$ belong to the other circle. The two circle arcs may have the same or opposite orientations. Moreover, the four end-points of the two arcs are connected by the other parts of the orbits. This can happen in two ways without intersection: either $a$ is connected to $d$ and $b$ is connected to $c$, or $a$ is connected to $b$ and $c$ is connected to $d$. Altogether we have four cases, as shown in Figure 7. ![](https:\/\/cdn.mathpix.com\/cropped\/2024_11_18_62c85fb0b01b41aca29bg-47.jpg?height=276&width=1621&top_left_y=1873&top_left_x=221) Figure 7 We can see that the number of orbits is changed by -2 or +2 in the leftmost case when the arcs have the same orientation, $a$ is connected to $d$, and $b$ is connected to $c$. In the other three cases the number of orbits is not changed. Hence, Type-1 steps do not change the parity of the number of orbits. Now consider a Type-2 step. The three circles enclose a small, triangular region; by the step, this triangle is replaced by another triangle. Again, the modification of the orbits is done in some small neighbourhood; the structure does not change outside. Each side of the triangle shaped region can be convex or concave; the number of concave sides can be $0,1,2$ or 3 , so there are 4 possible arrangements of the orbits inside the neighbourhood, as shown in Figure 8. ![](https:\/\/cdn.mathpix.com\/cropped\/2024_11_18_62c85fb0b01b41aca29bg-48.jpg?height=312&width=301&top_left_y=221&top_left_x=295) all convex ![](https:\/\/cdn.mathpix.com\/cropped\/2024_11_18_62c85fb0b01b41aca29bg-48.jpg?height=345&width=1092&top_left_y=219&top_left_x=683) Figure 8 Denote the points where the three circles enter or leave the neighbourhood by $a, b, c, d$, $e$, and $f$ in this order around the neighbourhood. As can be seen in Figure 8, there are only two essentially different cases; either $a, c, e$ are connected to $b, d, f$, respectively, or $a, c, e$ are connected to $f, b, d$, respectively. The step either preserves the set of connections or switches to the other arrangement. Obviously, in the earlier case the number of orbits is not changed; therefore we have to consider only the latter case. The points $a, b, c, d, e$, and $f$ are connected by the orbits outside, without intersection. If $a$ was connected to $c$, say, then this orbit would isolate $b$, so this is impossible. Hence, each of $a, b, c, d, e$ and $f$ must be connected either to one of its neighbours or to the opposite point. If say $a$ is connected to $d$, then this orbit separates $b$ and $c$ from $e$ and $f$, therefore $b$ must be connected to $c$ and $e$ must be connected to $f$. Altogether there are only two cases and their reverses: either each point is connected to one of its neighbours or two opposite points are connected and the the remaining neigh boring pairs are connected to each other. See Figure 9. ![](https:\/\/cdn.mathpix.com\/cropped\/2024_11_18_62c85fb0b01b41aca29bg-48.jpg?height=295&width=1628&top_left_y=1299&top_left_x=216) Figure 9 We can see that if only neighbouring points are connected, then the number of orbits is changed by +2 or -2 . If two opposite points are connected ( $a$ and $d$ in the figure), then the orbits are re-arranged, but their number is unchanged. Hence, Type-2 steps also preserve the parity. This completes the proof of (*).","tier":0} +{"problem_type":"Combinatorics","problem_label":"C9","problem":"There are $n$ circles drawn on a piece of paper in such a way that any two circles intersect in two points, and no three circles pass through the same point. Turbo the snail slides along the circles in the following fashion. Initially he moves on one of the circles in clockwise direction. Turbo always keeps sliding along the current circle until he reaches an intersection with another circle. Then he continues his journey on this new circle and also changes the direction of moving, i.e. from clockwise to anticlockwise or vice versa. Suppose that Turbo's path entirely covers all circles. Prove that $n$ must be odd. (India)","solution":"Like in the previous solutions, we do not need all circle pairs to intersect but we assume that the circles form a connected set. Denote by $\\mathcal{C}$ and $\\mathcal{P}$ the sets of circles and their intersection points, respectively. The circles divide the plane into several simply connected, bounded regions and one unbounded region. Denote the set of these regions by $\\mathcal{R}$. We say that an intersection point or a region is odd or even if it is contained inside an odd or even number of circles, respectively. Let $\\mathcal{P}_{\\text {odd }}$ and $\\mathcal{R}_{\\text {odd }}$ be the sets of odd intersection points and odd regions, respectively. Claim. $$ \\left|\\mathcal{R}_{\\text {odd }}\\right|-\\left|\\mathcal{P}_{\\text {odd }}\\right| \\equiv n \\quad(\\bmod 2) . $$ Proof. For each circle $c \\in \\mathcal{C}$, denote by $R_{c}, P_{c}$, and $X_{c}$ the number of regions inside $c$, the number of intersection points inside $c$, and the number of circles intersecting $c$, respectively. The circles divide each other into several arcs; denote by $A_{c}$ the number of such arcs inside $c$. By double counting the regions and intersection points inside the circles we get $$ \\left|\\mathcal{R}_{\\mathrm{odd}}\\right| \\equiv \\sum_{c \\in \\mathcal{C}} R_{c} \\quad(\\bmod 2) \\quad \\text { and } \\quad\\left|\\mathcal{P}_{\\mathrm{odd}}\\right| \\equiv \\sum_{c \\in \\mathcal{C}} P_{c} \\quad(\\bmod 2) . $$ For each circle $c$, apply EULER's polyhedron theorem to the (simply connected) regions in $c$. There are $2 X_{c}$ intersection points on $c$; they divide the circle into $2 X_{c}$ arcs. The polyhedron theorem yields $\\left(R_{c}+1\\right)+\\left(P_{c}+2 X_{c}\\right)=\\left(A_{c}+2 X_{c}\\right)+2$, considering the exterior of $c$ as a single region. Therefore, $$ R_{c}+P_{c}=A_{c}+1 $$ Moreover, we have four arcs starting from every interior points inside $c$ and a single arc starting into the interior from each intersection point on the circle. By double-counting the end-points of the interior arcs we get $2 A_{c}=4 P_{c}+2 X_{c}$, so $$ A_{c}=2 P_{c}+X_{c} $$ The relations (2) and (3) together yield $$ R_{c}-P_{c}=X_{c}+1 $$ By summing up (4) for all circles we obtain $$ \\sum_{c \\in \\mathcal{C}} R_{c}-\\sum_{c \\in \\mathcal{C}} P_{c}=\\sum_{c \\in \\mathcal{C}} X_{c}+|\\mathcal{C}| $$ which yields $$ \\left|\\mathcal{R}_{\\mathrm{odd}}\\right|-\\left|\\mathcal{P}_{\\mathrm{odd}}\\right| \\equiv \\sum_{c \\in \\mathcal{C}} X_{c}+n \\quad(\\bmod 2) $$ Notice that in $\\sum_{c \\in \\mathcal{C}} X_{c}$ each intersecting circle pair is counted twice, i.e., for both circles in the pair, so $$ \\sum_{c \\in \\mathcal{C}} X_{c} \\equiv 0 \\quad(\\bmod 2) $$ which finishes the proof of the Claim. Now insert the same small arcs at the intersections as in the first solution, and suppose that there is a single snail orbit $b$. First we show that the odd regions are inside the curve $b$, while the even regions are outside. Take a region $r \\in \\mathcal{R}$ and a point $x$ in its interior, and draw a ray $y$, starting from $x$, that does not pass through any intersection point of the circles and is neither tangent to any of the circles. As is well-known, $x$ is inside the curve $b$ if and only if $y$ intersects $b$ an odd number of times (see Figure 10). Notice that if an arbitrary circle $c$ contains $x$ in its interior, then $c$ intersects $y$ at a single point; otherwise, if $x$ is outside $c$, then $c$ has 2 or 0 intersections with $y$. Therefore, $y$ intersects $b$ an odd number of times if and only if $x$ is contained in an odd number of circles, so if and only if $r$ is odd. ![](https:\/\/cdn.mathpix.com\/cropped\/2024_11_18_62c85fb0b01b41aca29bg-49.jpg?height=321&width=626&top_left_y=2255&top_left_x=715) Figure 10 Now consider an intersection point $p$ of two circles $c_{1}$ and $c_{2}$ and a small neighbourhood around $p$. Suppose that $p$ is contained inside $k$ circles. We have four regions that meet at $p$. Let $r_{1}$ be the region that lies outside both $c_{1}$ and $c_{2}$, let $r_{2}$ be the region that lies inside both $c_{1}$ and $c_{2}$, and let $r_{3}$ and $r_{4}$ be the two remaining regions, each lying inside exactly one of $c_{1}$ and $c_{2}$. The region $r_{1}$ is contained inside the same $k$ circles as $p$; the region $r_{2}$ is contained also by $c_{1}$ and $c_{2}$, so by $k+2$ circles in total; each of the regions $r_{3}$ and $r_{4}$ is contained inside $k+1$ circles. After the small arcs have been inserted at $p$, the regions $r_{1}$ and $r_{2}$ get connected, and the regions $r_{3}$ and $r_{4}$ remain separated at $p$ (see Figure 11). If $p$ is an odd point, then $r_{1}$ and $r_{2}$ are odd, so two odd regions are connected at $p$. Otherwise, if $p$ is even, then we have two even regions connected at $p$. ![](https:\/\/cdn.mathpix.com\/cropped\/2024_11_18_62c85fb0b01b41aca29bg-50.jpg?height=261&width=298&top_left_y=658&top_left_x=571) Figure 11 ![](https:\/\/cdn.mathpix.com\/cropped\/2024_11_18_62c85fb0b01b41aca29bg-50.jpg?height=286&width=481&top_left_y=645&top_left_x=1007) Figure 12 Consider the system of odd regions and their connections at the odd points as a graph. In this graph the odd regions are the vertices, and each odd point establishes an edge that connects two vertices (see Figure 12). As $b$ is a single closed curve, this graph is connected and contains no cycle, so the graph is a tree. Then the number of vertices must be by one greater than the number of edges, so $$ \\left|\\mathcal{R}_{\\text {odd }}\\right|-\\left|\\mathcal{P}_{\\text {odd }}\\right|=1 . $$ The relations (1) and (9) together prove that $n$ must be odd. Comment. For every odd $n$ there exists at least one configuration of $n$ circles with a single snail orbit. Figure 13 shows a possible configuration with 5 circles. In general, if a circle is rotated by $k \\cdot \\frac{360^{\\circ}}{n}$ $(k=1,2, \\ldots, n-1)$ around an interior point other than the centre, the circle and its rotated copies together provide a single snail orbit. ![](https:\/\/cdn.mathpix.com\/cropped\/2024_11_18_62c85fb0b01b41aca29bg-50.jpg?height=584&width=595&top_left_y=1641&top_left_x=736) Figure 13","tier":0} +{"problem_type":"Geometry","problem_label":"G6","problem":"Let $A B C$ be a fixed acute-angled triangle. Consider some points $E$ and $F$ lying on the sides $A C$ and $A B$, respectively, and let $M$ be the midpoint of $E F$. Let the perpendicular bisector of $E F$ intersect the line $B C$ at $K$, and let the perpendicular bisector of $M K$ intersect the lines $A C$ and $A B$ at $S$ and $T$, respectively. We call the pair $(E, F)$ interesting, if the quadrilateral $K S A T$ is cyclic. Suppose that the pairs $\\left(E_{1}, F_{1}\\right)$ and $\\left(E_{2}, F_{2}\\right)$ are interesting. Prove that $$ \\frac{E_{1} E_{2}}{A B}=\\frac{F_{1} F_{2}}{A C} $$ (Iran)","solution":"For any interesting pair $(E, F)$, we will say that the corresponding triangle $E F K$ is also interesting. Let $E F K$ be an interesting triangle. Firstly, we prove that $\\angle K E F=\\angle K F E=\\angle A$, which also means that the circumcircle $\\omega_{1}$ of the triangle $A E F$ is tangent to the lines $K E$ and $K F$. Denote by $\\omega$ the circle passing through the points $K, S, A$, and $T$. Let the line $A M$ intersect the line $S T$ and the circle $\\omega$ (for the second time) at $N$ and $L$, respectively (see Figure 1). Since $E F \\| T S$ and $M$ is the midpoint of $E F, N$ is the midpoint of $S T$. Moreover, since $K$ and $M$ are symmetric to each other with respect to the line $S T$, we have $\\angle K N S=\\angle M N S=$ $\\angle L N T$. Thus the points $K$ and $L$ are symmetric to each other with respect to the perpendicular bisector of $S T$. Therefore $K L \\| S T$. Let $G$ be the point symmetric to $K$ with respect to $N$. Then $G$ lies on the line $E F$, and we may assume that it lies on the ray $M F$. One has $$ \\angle K G E=\\angle K N S=\\angle S N M=\\angle K L A=180^{\\circ}-\\angle K S A $$ (if $K=L$, then the angle $K L A$ is understood to be the angle between $A L$ and the tangent to $\\omega$ at $L$ ). This means that the points $K, G, E$, and $S$ are concyclic. Now, since $K S G T$ is a parallelogram, we obtain $\\angle K E F=\\angle K S G=180^{\\circ}-\\angle T K S=\\angle A$. Since $K E=K F$, we also have $\\angle K F E=\\angle K E F=\\angle A$. After having proved this fact, one may finish the solution by different methods. ![](https:\/\/cdn.mathpix.com\/cropped\/2024_11_18_62c85fb0b01b41aca29bg-59.jpg?height=684&width=829&top_left_y=1874&top_left_x=245) Figure 1 ![](https:\/\/cdn.mathpix.com\/cropped\/2024_11_18_62c85fb0b01b41aca29bg-59.jpg?height=612&width=730&top_left_y=1944&top_left_x=1117) Figure 2 First method. We have just proved that all interesting triangles are similar to each other. This allows us to use the following lemma. Lemma. Let $A B C$ be an arbitrary triangle. Choose two points $E_{1}$ and $E_{2}$ on the side $A C$, two points $F_{1}$ and $F_{2}$ on the side $A B$, and two points $K_{1}$ and $K_{2}$ on the side $B C$, in a way that the triangles $E_{1} F_{1} K_{1}$ and $E_{2} F_{2} K_{2}$ are similar. Then the six circumcircles of the triangles $A E_{i} F_{i}$, $B F_{i} K_{i}$, and $C E_{i} K_{i}(i=1,2)$ meet at a common point $Z$. Moreover, $Z$ is the centre of the spiral similarity that takes the triangle $E_{1} F_{1} K_{1}$ to the triangle $E_{2} F_{2} K_{2}$. Proof. Firstly, notice that for each $i=1,2$, the circumcircles of the triangles $A E_{i} F_{i}, B F_{i} K_{i}$, and $C K_{i} E_{i}$ have a common point $Z_{i}$ by Miquel's theorem. Moreover, we have $\\Varangle\\left(Z_{i} F_{i}, Z_{i} E_{i}\\right)=\\Varangle(A B, C A), \\quad \\Varangle\\left(Z_{i} K_{i}, Z_{i} F_{i}\\right)=\\Varangle(B C, A B), \\quad \\Varangle\\left(Z_{i} E_{i}, Z_{i} K_{i}\\right)=\\Varangle(C A, B C)$. This yields that the points $Z_{1}$ and $Z_{2}$ correspond to each other in similar triangles $E_{1} F_{1} K_{1}$ and $E_{2} F_{2} K_{2}$. Thus, if they coincide, then this common point is indeed the desired centre of a spiral similarity. Finally, in order to show that $Z_{1}=Z_{2}$, one may notice that $\\Varangle\\left(A B, A Z_{1}\\right)=\\Varangle\\left(E_{1} F_{1}, E_{1} Z_{1}\\right)=$ $\\Varangle\\left(E_{2} F_{2}, E_{2} Z_{2}\\right)=\\Varangle\\left(A B, A Z_{2}\\right)$ (see Figure 2). Similarly, one has $\\Varangle\\left(B C, B Z_{1}\\right)=\\Varangle\\left(B C, B Z_{2}\\right)$ and $\\Varangle\\left(C A, C Z_{1}\\right)=\\Varangle\\left(C A, C Z_{2}\\right)$. This yields $Z_{1}=Z_{2}$. Now, let $P$ and $Q$ be the feet of the perpendiculars from $B$ and $C$ onto $A C$ and $A B$, respectively, and let $R$ be the midpoint of $B C$ (see Figure 3). Then $R$ is the circumcentre of the cyclic quadrilateral $B C P Q$. Thus we obtain $\\angle A P Q=\\angle B$ and $\\angle R P C=\\angle C$, which yields $\\angle Q P R=\\angle A$. Similarly, we show that $\\angle P Q R=\\angle A$. Thus, all interesting triangles are similar to the triangle $P Q R$. ![](https:\/\/cdn.mathpix.com\/cropped\/2024_11_18_62c85fb0b01b41aca29bg-60.jpg?height=615&width=715&top_left_y=1326&top_left_x=316) Figure 3 ![](https:\/\/cdn.mathpix.com\/cropped\/2024_11_18_62c85fb0b01b41aca29bg-60.jpg?height=615&width=692&top_left_y=1326&top_left_x=1090) Figure 4 Denote now by $Z$ the common point of the circumcircles of $A P Q, B Q R$, and $C P R$. Let $E_{1} F_{1} K_{1}$ and $E_{2} F_{2} K_{2}$ be two interesting triangles. By the lemma, $Z$ is the centre of any spiral similarity taking one of the triangles $E_{1} F_{1} K_{1}, E_{2} F_{2} K_{2}$, and $P Q R$ to some other of them. Therefore the triangles $Z E_{1} E_{2}$ and $Z F_{1} F_{2}$ are similar, as well as the triangles $Z E_{1} F_{1}$ and $Z P Q$. Hence $$ \\frac{E_{1} E_{2}}{F_{1} F_{2}}=\\frac{Z E_{1}}{Z F_{1}}=\\frac{Z P}{Z Q} $$ Moreover, the equalities $\\angle A Z Q=\\angle A P Q=\\angle A B C=180^{\\circ}-\\angle Q Z R$ show that the point $Z$ lies on the line $A R$ (see Figure 4). Therefore the triangles $A Z P$ and $A C R$ are similar, as well as the triangles $A Z Q$ and $A B R$. This yields $$ \\frac{Z P}{Z Q}=\\frac{Z P}{R C} \\cdot \\frac{R B}{Z Q}=\\frac{A Z}{A C} \\cdot \\frac{A B}{A Z}=\\frac{A B}{A C} $$ which completes the solution. Second method. Now we will start from the fact that $\\omega_{1}$ is tangent to the lines $K E$ and $K F$ (see Figure 5). We prove that if $(E, F)$ is an interesting pair, then $$ \\frac{A E}{A B}+\\frac{A F}{A C}=2 \\cos \\angle A $$ Let $Y$ be the intersection point of the segments $B E$ and $C F$. The points $B, K$, and $C$ are collinear, hence applying PASCAL's theorem to the degenerated hexagon AFFYEE, we infer that $Y$ lies on the circle $\\omega_{1}$. Denote by $Z$ the second intersection point of the circumcircle of the triangle $B F Y$ with the line $B C$ (see Figure 6). By Miquel's theorem, the points $C, Z, Y$, and $E$ are concyclic. Therefore we obtain $$ B F \\cdot A B+C E \\cdot A C=B Y \\cdot B E+C Y \\cdot C F=B Z \\cdot B C+C Z \\cdot B C=B C^{2} $$ On the other hand, $B C^{2}=A B^{2}+A C^{2}-2 A B \\cdot A C \\cos \\angle A$, by the cosine law. Hence $$ (A B-A F) \\cdot A B+(A C-A E) \\cdot A C=A B^{2}+A C^{2}-2 A B \\cdot A C \\cos \\angle A, $$ which simplifies to the desired equality (1). Let now $\\left(E_{1}, F_{1}\\right)$ and $\\left(E_{2}, F_{2}\\right)$ be two interesting pairs of points. Then we get $$ \\frac{A E_{1}}{A B}+\\frac{A F_{1}}{A C}=\\frac{A E_{2}}{A B}+\\frac{A F_{2}}{A C} $$ which gives the desired result. ![](https:\/\/cdn.mathpix.com\/cropped\/2024_11_18_62c85fb0b01b41aca29bg-61.jpg?height=709&width=915&top_left_y=1527&top_left_x=248) Figure 5 ![](https:\/\/cdn.mathpix.com\/cropped\/2024_11_18_62c85fb0b01b41aca29bg-61.jpg?height=569&width=615&top_left_y=1669&top_left_x=1229) Figure 6 Third method. Again, we make use of the fact that all interesting triangles are similar (and equi-oriented). Let us put the picture onto a complex plane such that $A$ is at the origin, and identify each point with the corresponding complex number. Let $E F K$ be any interesting triangle. The equalities $\\angle K E F=\\angle K F E=\\angle A$ yield that the ratio $\\nu=\\frac{K-E}{F-E}$ is the same for all interesting triangles. This in turn means that the numbers $E$, $F$, and $K$ satisfy the linear equation $$ K=\\mu E+\\nu F, \\quad \\text { where } \\quad \\mu=1-\\nu $$ Now let us choose the points $X$ and $Y$ on the rays $A B$ and $A C$, respectively, so that $\\angle C X A=\\angle A Y B=\\angle A=\\angle K E F$ (see Figure 7). Then each of the triangles $A X C$ and $Y A B$ is similar to any interesting triangle, which also means that $$ C=\\mu A+\\nu X=\\nu X \\quad \\text { and } \\quad B=\\mu Y+\\nu A=\\mu Y . $$ Moreover, one has $X \/ Y=\\overline{C \/ B}$. Since the points $E, F$, and $K$ lie on $A C, A B$, and $B C$, respectively, one gets $$ E=\\rho Y, \\quad F=\\sigma X, \\quad \\text { and } \\quad K=\\lambda B+(1-\\lambda) C $$ for some real $\\rho, \\sigma$, and $\\lambda$. In view of (3), the equation (2) now reads $\\lambda B+(1-\\lambda) C=K=$ $\\mu E+\\nu F=\\rho B+\\sigma C$, or $$ (\\lambda-\\rho) B=(\\sigma+\\lambda-1) C $$ Since the nonzero complex numbers $B$ and $C$ have different arguments, the coefficients in the brackets vanish, so $\\rho=\\lambda$ and $\\sigma=1-\\lambda$. Therefore, $$ \\frac{E}{Y}+\\frac{F}{X}=\\rho+\\sigma=1 $$ Now, if $\\left(E_{1}, F_{1}\\right)$ and $\\left(E_{2}, F_{2}\\right)$ are two distinct interesting pairs, one may apply (4) to both pairs. Subtracting, we get $$ \\frac{E_{1}-E_{2}}{Y}=\\frac{F_{2}-F_{1}}{X}, \\quad \\text { so } \\quad \\frac{E_{1}-E_{2}}{F_{2}-F_{1}}=\\frac{Y}{X}=\\frac{\\bar{B}}{\\bar{C}} $$ Taking absolute values provides the required result. ![](https:\/\/cdn.mathpix.com\/cropped\/2024_11_18_62c85fb0b01b41aca29bg-62.jpg?height=487&width=669&top_left_y=1664&top_left_x=696) Figure 7 Comment 1. One may notice that the triangle $P Q R$ is also interesting. Comment 2. In order to prove that $\\angle K E F=\\angle K F E=\\angle A$, one may also use the following well-known fact: Let $A E F$ be a triangle with $A E \\neq A F$, and let $K$ be the common point of the symmedian taken from $A$ and the perpendicular bisector of $E F$. Then the lines $K E$ and $K F$ are tangent to the circumcircle $\\omega_{1}$ of the triangle $A E F$. In this case, however, one needs to deal with the case $A E=A F$ separately.","tier":0} +{"problem_type":"Geometry","problem_label":"G6","problem":"Let $A B C$ be a fixed acute-angled triangle. Consider some points $E$ and $F$ lying on the sides $A C$ and $A B$, respectively, and let $M$ be the midpoint of $E F$. Let the perpendicular bisector of $E F$ intersect the line $B C$ at $K$, and let the perpendicular bisector of $M K$ intersect the lines $A C$ and $A B$ at $S$ and $T$, respectively. We call the pair $(E, F)$ interesting, if the quadrilateral $K S A T$ is cyclic. Suppose that the pairs $\\left(E_{1}, F_{1}\\right)$ and $\\left(E_{2}, F_{2}\\right)$ are interesting. Prove that $$ \\frac{E_{1} E_{2}}{A B}=\\frac{F_{1} F_{2}}{A C} $$ (Iran)","solution":"Let $(E, F)$ be an interesting pair. This time we prove that $$ \\frac{A M}{A K}=\\cos \\angle A $$ As in Solution 1, we introduce the circle $\\omega$ passing through the points $K, S$, $A$, and $T$, together with the points $N$ and $L$ at which the line $A M$ intersect the line $S T$ and the circle $\\omega$ for the second time, respectively. Let moreover $O$ be the centre of $\\omega$ (see Figures 8 and 9). As in Solution 1, we note that $N$ is the midpoint of $S T$ and show that $K L \\| S T$, which implies $\\angle F A M=\\angle E A K$. ![](https:\/\/cdn.mathpix.com\/cropped\/2024_11_18_62c85fb0b01b41aca29bg-63.jpg?height=752&width=1009&top_left_y=749&top_left_x=266) Figure 8 ![](https:\/\/cdn.mathpix.com\/cropped\/2024_11_18_62c85fb0b01b41aca29bg-63.jpg?height=552&width=549&top_left_y=952&top_left_x=1279) Figure 9 Suppose now that $K \\neq L$ (see Figure 8). Then $K L \\| S T$, and consequently the lines $K M$ and $K L$ are perpendicular. It implies that the lines $L O$ and $K M$ meet at a point $X$ lying on the circle $\\omega$. Since the lines $O N$ and $X M$ are both perpendicular to the line $S T$, they are parallel to each other, and hence $\\angle L O N=\\angle L X K=\\angle M A K$. On the other hand, $\\angle O L N=\\angle M K A$, so we infer that triangles $N O L$ and $M A K$ are similar. This yields $$ \\frac{A M}{A K}=\\frac{O N}{O L}=\\frac{O N}{O T}=\\cos \\angle T O N=\\cos \\angle A $$ If, on the other hand, $K=L$, then the points $A, M, N$, and $K$ lie on a common line, and this line is the perpendicular bisector of $S T$ (see Figure 9). This implies that $A K$ is a diameter of $\\omega$, which yields $A M=2 O K-2 N K=2 O N$. So also in this case we obtain $$ \\frac{A M}{A K}=\\frac{2 O N}{2 O T}=\\cos \\angle T O N=\\cos \\angle A $$ Thus (5) is proved. Let $P$ and $Q$ be the feet of the perpendiculars from $B$ and $C$ onto $A C$ and $A B$, respectively (see Figure 10). We claim that the point $M$ lies on the line $P Q$. Consider now the composition of the dilatation with factor $\\cos \\angle A$ and centre $A$, and the reflection with respect to the angle bisector of $\\angle B A C$. This transformation is a similarity that takes $B, C$, and $K$ to $P, Q$, and $M$, respectively. Since $K$ lies on the line $B C$, the point $M$ lies on the line $P Q$. ![](https:\/\/cdn.mathpix.com\/cropped\/2024_11_18_62c85fb0b01b41aca29bg-64.jpg?height=715&width=918&top_left_y=219&top_left_x=586) Figure 10 Suppose that $E \\neq P$. Then also $F \\neq Q$, and by Menelaus' theorem, we obtain $$ \\frac{A Q}{F Q} \\cdot \\frac{F M}{E M} \\cdot \\frac{E P}{A P}=1 $$ Using the similarity of the triangles $A P Q$ and $A B C$, we infer that $$ \\frac{E P}{F Q}=\\frac{A P}{A Q}=\\frac{A B}{A C}, \\quad \\text { and hence } \\quad \\frac{E P}{A B}=\\frac{F Q}{A C} $$ The last equality holds obviously also in case $E=P$, because then $F=Q$. Moreover, since the line $P Q$ intersects the segment $E F$, we infer that the point $E$ lies on the segment $A P$ if and only if the point $F$ lies outside of the segment $A Q$. Let now $\\left(E_{1}, F_{1}\\right)$ and $\\left(E_{2}, F_{2}\\right)$ be two interesting pairs. Then we obtain $$ \\frac{E_{1} P}{A B}=\\frac{F_{1} Q}{A C} \\quad \\text { and } \\quad \\frac{E_{2} P}{A B}=\\frac{F_{2} Q}{A C} . $$ If $P$ lies between the points $E_{1}$ and $E_{2}$, we add the equalities above, otherwise we subtract them. In any case we obtain $$ \\frac{E_{1} E_{2}}{A B}=\\frac{F_{1} F_{2}}{A C} $$ which completes the solution.","tier":0} +{"problem_type":"Geometry","problem_label":"G7","problem":"Let $A B C$ be a triangle with circumcircle $\\Omega$ and incentre $I$. Let the line passing through $I$ and perpendicular to $C I$ intersect the segment $B C$ and the $\\operatorname{arc} B C$ (not containing $A$ ) of $\\Omega$ at points $U$ and $V$, respectively. Let the line passing through $U$ and parallel to $A I$ intersect $A V$ at $X$, and let the line passing through $V$ and parallel to $A I$ intersect $A B$ at $Y$. Let $W$ and $Z$ be the midpoints of $A X$ and $B C$, respectively. Prove that if the points $I, X$, and $Y$ are collinear, then the points $I, W$, and $Z$ are also collinear.","solution":"We start with some general observations. Set $\\alpha=\\angle A \/ 2, \\beta=\\angle B \/ 2, \\gamma=\\angle C \/ 2$. Then obviously $\\alpha+\\beta+\\gamma=90^{\\circ}$. Since $\\angle U I C=90^{\\circ}$, we obtain $\\angle I U C=\\alpha+\\beta$. Therefore $\\angle B I V=\\angle I U C-\\angle I B C=\\alpha=\\angle B A I=\\angle B Y V$, which implies that the points $B, Y, I$, and $V$ lie on a common circle (see Figure 1). Assume now that the points $I, X$ and $Y$ are collinear. We prove that $\\angle Y I A=90^{\\circ}$. Let the line $X U$ intersect $A B$ at $N$. Since the lines $A I, U X$, and $V Y$ are parallel, we get $$ \\frac{N X}{A I}=\\frac{Y N}{Y A}=\\frac{V U}{V I}=\\frac{X U}{A I} $$ implying $N X=X U$. Moreover, $\\angle B I U=\\alpha=\\angle B N U$. This implies that the quadrilateral BUIN is cyclic, and since $B I$ is the angle bisector of $\\angle U B N$, we infer that $N I=U I$. Thus in the isosceles triangle $N I U$, the point $X$ is the midpoint of the base $N U$. This gives $\\angle I X N=90^{\\circ}$, i.e., $\\angle Y I A=90^{\\circ}$. ![](https:\/\/cdn.mathpix.com\/cropped\/2024_11_18_62c85fb0b01b41aca29bg-65.jpg?height=880&width=966&top_left_y=1256&top_left_x=565) Figure 1 Let $S$ be the midpoint of the segment $V C$. Let moreover $T$ be the intersection point of the lines $A X$ and $S I$, and set $x=\\angle B A V=\\angle B C V$. Since $\\angle C I A=90^{\\circ}+\\beta$ and $S I=S C$, we obtain $$ \\angle T I A=180^{\\circ}-\\angle A I S=90^{\\circ}-\\beta-\\angle C I S=90^{\\circ}-\\beta-\\gamma-x=\\alpha-x=\\angle T A I, $$ which implies that $T I=T A$. Therefore, since $\\angle X I A=90^{\\circ}$, the point $T$ is the midpoint of $A X$, i.e., $T=W$. To complete our solution, it remains to show that the intersection point of the lines $I S$ and $B C$ coincide with the midpoint of the segment $B C$. But since $S$ is the midpoint of the segment $V C$, it suffices to show that the lines $B V$ and $I S$ are parallel. Since the quadrilateral $B Y I V$ is cyclic, $\\angle V B I=\\angle V Y I=\\angle Y I A=90^{\\circ}$. This implies that $B V$ is the external angle bisector of the angle $A B C$, which yields $\\angle V A C=\\angle V C A$. Therefore $2 \\alpha-x=2 \\gamma+x$, which gives $\\alpha=\\gamma+x$. Hence $\\angle S C I=\\alpha$, so $\\angle V S I=2 \\alpha$. On the other hand, $\\angle B V C=180^{\\circ}-\\angle B A C=180^{\\circ}-2 \\alpha$, which implies that the lines $B V$ and $I S$ are parallel. This completes the solution.","tier":0} +{"problem_type":"Geometry","problem_label":"G7","problem":"Let $A B C$ be a triangle with circumcircle $\\Omega$ and incentre $I$. Let the line passing through $I$ and perpendicular to $C I$ intersect the segment $B C$ and the $\\operatorname{arc} B C$ (not containing $A$ ) of $\\Omega$ at points $U$ and $V$, respectively. Let the line passing through $U$ and parallel to $A I$ intersect $A V$ at $X$, and let the line passing through $V$ and parallel to $A I$ intersect $A B$ at $Y$. Let $W$ and $Z$ be the midpoints of $A X$ and $B C$, respectively. Prove that if the points $I, X$, and $Y$ are collinear, then the points $I, W$, and $Z$ are also collinear.","solution":"As in $$ \\angle B A V=\\angle C A E $$ Proof. Let $\\rho$ be the composition of the inversion with centre $A$ and radius $\\sqrt{A B \\cdot A C}$, and the symmetry with respect to $A I$. Clearly, $\\rho$ interchanges $B$ and $C$. Let $J$ be the excentre of the triangle $A B C$ opposite to $A$ (see Figure 2). Then we have $\\angle J A C=\\angle B A I$ and $\\angle J C A=90^{\\circ}+\\gamma=\\angle B I A$, so the triangles $A C J$ and $A I B$ are similar, and therefore $A B \\cdot A C=A I \\cdot A J$. This means that $\\rho$ interchanges $I$ and $J$. Moreover, since $Y$ lies on $A B$ and $\\angle A I Y=90^{\\circ}$, the point $Y^{\\prime}=\\rho(Y)$ lies on $A C$, and $\\angle J Y^{\\prime} A=90^{\\circ}$. Thus $\\rho$ maps the circumcircle $\\gamma$ of the triangle $B Y I$ to a circle $\\gamma^{\\prime}$ with diameter $J C$. Finally, since $V$ lies on both $\\Gamma$ and $\\gamma$, the point $V^{\\prime}=\\rho(V)$ lies on the line $\\rho(\\Gamma)=A B$ as well as on $\\gamma^{\\prime}$, which in turn means that $V^{\\prime}=E$. This implies the desired result. ![](https:\/\/cdn.mathpix.com\/cropped\/2024_11_18_62c85fb0b01b41aca29bg-66.jpg?height=709&width=572&top_left_y=1713&top_left_x=248) Figure 2 ![](https:\/\/cdn.mathpix.com\/cropped\/2024_11_18_62c85fb0b01b41aca29bg-66.jpg?height=872&width=989&top_left_y=1549&top_left_x=822) Figure 3 Now we turn to the solution of the problem. Assume that the incircle $\\omega_{1}$ of the triangle $A B C$ is tangent to $B C$ at $D$, and let the excircle $\\omega_{2}$ of the triangle $A B C$ opposite to the vertex $A$ touch the side $B C$ at $E$ (see Figure 3). The homothety with centre $A$ that takes $\\omega_{2}$ to $\\omega_{1}$ takes the point $E$ to some point $F$, and the tangent to $\\omega_{1}$ at $F$ is parallel to $B C$. Therefore $D F$ is a diameter of $\\omega_{1}$. Moreover, $Z$ is the midpoint of $D E$. This implies that the lines $I Z$ and $F E$ are parallel. Let $K=Y I \\cap A E$. Since $\\angle Y I A=90^{\\circ}$, the lemma yields that $I$ is the midpoint of $X K$. This implies that the segments $I W$ and $A K$ are parallel. Therefore, the points $W, I$ and $Z$ are collinear. Comment 1. The properties $\\angle Y I A=90^{\\circ}$ and $V A=V C$ can be established in various ways. The main difficulty of the problem seems to find out how to use these properties in connection to the points $W$ and $Z$. In Solution 2 this principal part is more or less covered by the lemma, for which we have presented a direct proof. On the other hand, this lemma appears to be a combination of two well-known facts; let us formulate them in terms of the lemma statement. Let the line $I Y$ intersect $A C$ at $P$ (see Figure 4). The first fact states that the circumcircle $\\omega$ of the triangle $V Y P$ is tangent to the segments $A B$ and $A C$, as well as to the circle $\\Gamma$. The second fact states that for such a circle, the angles $B A V$ and $C A E$ are equal. The awareness of this lemma may help a lot in solving this problem; so the Jury might also consider a variation of the proposed problem, for which the lemma does not seem to be useful; see Comment 3. ![](https:\/\/cdn.mathpix.com\/cropped\/2024_11_18_62c85fb0b01b41aca29bg-67.jpg?height=946&width=1689&top_left_y=1149&top_left_x=189) Comment 2. The proposed problem stated the equivalence: the point $I$ lies on the line $X Y$ if and only if $I$ lies on the line $W Z$. Here we sketch the proof of the \"if\" part (see Figure 5). As in Solution 2 , let $B C$ touch the circles $\\omega_{1}$ and $\\omega_{2}$ at $D$ and $E$, respectively. Since $I Z \\| A E$ and $W$ lies on $I Z$, the line $D X$ is also parallel to $A E$. Therefore, the triangles $X U P$ and $A I Q$ are similar. Moreover, the line $D X$ is symmetric to $A E$ with respect to $I$, so $I P=I Q$, where $P=U V \\cap X D$ and $Q=U V \\cap A E$. Thus we obtain $$ \\frac{U V}{V I}=\\frac{U X}{I A}=\\frac{U P}{I Q}=\\frac{U P}{I P} $$ So the pairs $I U$ and $P V$ are harmonic conjugates, and since $\\angle U D I=90^{\\circ}$, we get $\\angle V D B=\\angle B D X=$ $\\angle B E A$. Therefore the point $V^{\\prime}$ symmetric to $V$ with respect to the perpendicular bisector of $B C$ lies on the line $A E$. So we obtain $\\angle B A V=\\angle C A E$. The rest can be obtained by simply reversing the arguments in Solution 2 . The points $B, V, I$, and $Y$ are concyclic. The lemma implies that $\\angle Y I A=90^{\\circ}$. Moreover, the points $B, U, I$, and $N$, where $N=U X \\cap A B$, lie on a common circle, so $I N=I U$. Since $I Y \\perp U N$, the point $X^{\\prime}=I Y \\cap U N$ is the midpoint of $U N$. But in the trapezoid $A Y V I$, the line $X U$ is parallel to the sides $A I$ and $Y V$, so $N X=U X^{\\prime}$. This yields $X=X^{\\prime}$. The reasoning presented in Solution 1 can also be reversed, but it requires a lot of technicalities. Therefore the Problem Selection Committee proposes to consider only the \"only if\" part of the original proposal, which is still challenging enough. Comment 3. The Jury might also consider the following variation of the proposed problem. Let $A B C$ be a triangle with circumcircle $\\Omega$ and incentre I. Let the line through I perpendicular to CI intersect the segment $B C$ and the arc $B C$ (not containing $A$ ) of $\\Omega$ at $U$ and $V$, respectively. Let the line through $U$ parallel to $A I$ intersect $A V$ at $X$. Prove that if the lines XI and AI are perpendicular, then the midpoint of the segment AC lies on the line XI (see Figure 6). ![](https:\/\/cdn.mathpix.com\/cropped\/2024_11_18_62c85fb0b01b41aca29bg-68.jpg?height=675&width=746&top_left_y=1273&top_left_x=184) Figure 6 ![](https:\/\/cdn.mathpix.com\/cropped\/2024_11_18_62c85fb0b01b41aca29bg-68.jpg?height=1012&width=980&top_left_y=930&top_left_x=892) Figure 7 Since the solution contains the arguments used above, we only sketch it. Let $N=X U \\cap A B$ (see Figure 7). Then $\\angle B N U=\\angle B A I=\\angle B I U$, so the points $B, U, I$, and $N$ lie on a common circle. Therefore $I U=I N$, and since $I X \\perp N U$, it follows that $N X=X U$. Now set $Y=X I \\cap A B$. The equality $N X=X U$ implies that $$ \\frac{V X}{V A}=\\frac{X U}{A I}=\\frac{N X}{A I}=\\frac{Y X}{Y I} $$ and therefore $Y V \\| A I$. Hence $\\angle B Y V=\\angle B A I=\\angle B I V$, so the points $B, V, I, Y$ are concyclic. Next we have $I Y \\perp Y V$, so $\\angle I B V=90^{\\circ}$. This implies that $B V$ is the external angle bisector of the angle $A B C$, which gives $\\angle V A C=\\angle V C A$. So in order to show that $M=X I \\cap A C$ is the midpoint of $A C$, it suffices to prove that $\\angle V M C=90^{\\circ}$. But this follows immediately from the observation that the points $V, C, M$, and $I$ are concyclic, as $\\angle M I V=\\angle Y B V=180^{\\circ}-\\angle A C V$. The converse statement is also true, but its proof requires some technicalities as well.","tier":0} +{"problem_type":"Number Theory","problem_label":"N1","problem":"Let $n \\geqslant 2$ be an integer, and let $A_{n}$ be the set $$ A_{n}=\\left\\{2^{n}-2^{k} \\mid k \\in \\mathbb{Z}, 0 \\leqslant k2$, and take an integer $m>(n-2) 2^{n}+1$. If $m$ is even, then consider $$ \\frac{m}{2} \\geqslant \\frac{(n-2) 2^{n}+2}{2}=(n-2) 2^{n-1}+1>(n-3) 2^{n-1}+1 $$ By the induction hypothesis, there is a representation of the form $$ \\frac{m}{2}=\\left(2^{n-1}-2^{k_{1}}\\right)+\\left(2^{n-1}-2^{k_{2}}\\right)+\\cdots+\\left(2^{n-1}-2^{k_{r}}\\right) $$ for some $k_{i}$ with $0 \\leqslant k_{i}\\frac{(n-2) 2^{n}+1-\\left(2^{n}-1\\right)}{2}=(n-3) 2^{n-1}+1 $$ By the induction hypothesis, there is a representation of the form $$ \\frac{m-\\left(2^{n}-1\\right)}{2}=\\left(2^{n-1}-2^{k_{1}}\\right)+\\left(2^{n-1}-2^{k_{2}}\\right)+\\cdots+\\left(2^{n-1}-2^{k_{r}}\\right) $$ for some $k_{i}$ with $0 \\leqslant k_{i}2$, assume that there exist integers $a, b$ with $a \\geqslant 0, b \\geqslant 1$ and $a+by$ due to symmetry. Then the integer $n=x-y$ is positive and (1) may be rewritten as $$ \\sqrt[3]{7(y+n)^{2}-13(y+n) y+7 y^{2}}=n+1 $$ Raising this to the third power and simplifying the result one obtains $$ y^{2}+y n=n^{3}-4 n^{2}+3 n+1 . $$ To complete the square on the left hand side, we multiply by 4 and add $n^{2}$, thus getting $$ (2 y+n)^{2}=4 n^{3}-15 n^{2}+12 n+4=(n-2)^{2}(4 n+1) $$ This shows that the cases $n=1$ and $n=2$ are impossible, whence $n>2$, and $4 n+1$ is the square of the rational number $\\frac{2 y+n}{n-2}$. Consequently, it has to be a perfect square, and, since it is odd as well, there has to exist some nonnegative integer $m$ such that $4 n+1=(2 m+1)^{2}$, i.e. $$ n=m^{2}+m $$ Notice that $n>2$ entails $m \\geqslant 2$. Substituting the value of $n$ just found into the previous displayed equation we arrive at $$ \\left(2 y+m^{2}+m\\right)^{2}=\\left(m^{2}+m-2\\right)^{2}(2 m+1)^{2}=\\left(2 m^{3}+3 m^{2}-3 m-2\\right)^{2} . $$ Extracting square roots and taking $2 m^{3}+3 m^{2}-3 m-2=(m-1)\\left(2 m^{2}+5 m+2\\right)>0$ into account we derive $2 y+m^{2}+m=2 m^{3}+3 m^{2}-3 m-2$, which in turn yields $$ y=m^{3}+m^{2}-2 m-1 $$ Notice that $m \\geqslant 2$ implies that $y=\\left(m^{3}-1\\right)+(m-2) m$ is indeed positive, as it should be. In view of $x=y+n=y+m^{2}+m$ it also follows that $$ x=m^{3}+2 m^{2}-m-1, $$ and that this integer is positive as well. Comment. Alternatively one could ask to find all pairs $(x, y)$ of - not necessarily positive - integers solving (1). The answer to that question is a bit nicer than the answer above: the set of solutions are now described by $$ \\{x, y\\}=\\left\\{m^{3}+m^{2}-2 m-1, m^{3}+2 m^{2}-m-1\\right\\} $$ where $m$ varies through $\\mathbb{Z}$. This may be shown using essentially the same arguments as above. We finally observe that the pair $(x, y)=(1,1)$, that appears to be sporadic above, corresponds to $m=-1$.","tier":0} +{"problem_type":"Number Theory","problem_label":"N5","problem":"Find all triples $(p, x, y)$ consisting of a prime number $p$ and two positive integers $x$ and $y$ such that $x^{p-1}+y$ and $x+y^{p-1}$ are both powers of $p$. (Belgium) Answer. $(p, x, y) \\in\\{(3,2,5),(3,5,2)\\} \\cup\\left\\{\\left(2, n, 2^{k}-n\\right) \\mid 02$, and we let $a$ and $b$ be positive integers such that $x^{p-1}+y=p^{a}$ and $x+y^{p-1}=p^{b}$. Assume further, without loss of generality, that $x \\leqslant y$, so that $p^{a}=x^{p-1}+y \\leqslant x+y^{p-1}=p^{b}$, which means that $a \\leqslant b$ (and thus $\\left.p^{a} \\mid p^{b}\\right)$. Now we have $$ p^{b}=y^{p-1}+x=\\left(p^{a}-x^{p-1}\\right)^{p-1}+x . $$ We take this equation modulo $p^{a}$ and take into account that $p-1$ is even, which gives us $$ 0 \\equiv x^{(p-1)^{2}}+x \\quad\\left(\\bmod p^{a}\\right) $$ If $p \\mid x$, then $p^{a} \\mid x$, since $x^{(p-1)^{2}-1}+1$ is not divisible by $p$ in this case. However, this is impossible, since $x \\leqslant x^{p-1}2$. Thus $a=r+1$. Now since $p^{r} \\leqslant x+1$, we get $$ x=\\frac{x^{2}+x}{x+1} \\leqslant \\frac{x^{p-1}+y}{x+1}=\\frac{p^{a}}{x+1} \\leqslant \\frac{p^{a}}{p^{r}}=p, $$ so we must have $x=p-1$ for $p$ to divide $x+1$. It follows that $r=1$ and $a=2$. If $p \\geqslant 5$, we obtain $$ p^{a}=x^{p-1}+y>(p-1)^{4}=\\left(p^{2}-2 p+1\\right)^{2}>(3 p)^{2}>p^{2}=p^{a} $$ a contradiction. So the only case that remains is $p=3$, and indeed $x=2$ and $y=p^{a}-x^{p-1}=5$ satisfy the conditions. Comment 1. In this solution, we are implicitly using a special case of the following lemma known as \"lifting the exponent\": Lemma. Let $n$ be a positive integer, let $p$ be an odd prime, and let $v_{p}(m)$ denote the exponent of the highest power of $p$ that divides $m$. If $x$ and $y$ are integers not divisible by $p$ such that $p \\mid x-y$, then we have $$ v_{p}\\left(x^{n}-y^{n}\\right)=v_{p}(x-y)+v_{p}(n) $$ Likewise, if $x$ and $y$ are integers not divisible by $p$ such that $p \\mid x+y$, then we have $$ v_{p}\\left(x^{n}+y^{n}\\right)=v_{p}(x+y)+v_{p}(n) . $$ Comment 2. There exist various ways of solving the problem involving the \"lifting the exponent\" lemma. Let us sketch another one. The cases $x=y$ and $p \\mid x$ are ruled out easily, so we assume that $p>2, x2$. If $p \\mid x$, then also $p \\mid y$. In this case, let $p^{k}$ and $p^{\\ell}$ be the highest powers of $p$ that divide $x$ and $y$ respectively, and assume without loss of generality that $k \\leqslant \\ell$. Then $p^{k}$ divides $x+y^{p-1}$ while $p^{k+1}$ does not, but $p^{k}p$, so $x^{p-1}+y$ and $y^{p-1}+x$ are both at least equal to $p^{2}$. Now we have $$ x^{p-1} \\equiv-y \\quad\\left(\\bmod p^{2}\\right) \\quad \\text { and } \\quad y^{p-1} \\equiv-x \\quad\\left(\\bmod p^{2}\\right) $$ These two congruences, together with the Euler-Fermat theorem, give us $$ 1 \\equiv x^{p(p-1)} \\equiv(-y)^{p} \\equiv-y^{p} \\equiv x y \\quad\\left(\\bmod p^{2}\\right) $$ Since $x \\equiv y \\equiv-1(\\bmod p), x-y$ is divisible by $p$, so $(x-y)^{2}$ is divisible by $p^{2}$. This means that $$ (x+y)^{2}=(x-y)^{2}+4 x y \\equiv 4 \\quad\\left(\\bmod p^{2}\\right) $$ so $p^{2}$ divides $(x+y-2)(x+y+2)$. We already know that $x+y \\equiv-2(\\bmod p)$, so $x+y-2 \\equiv$ $-4 \\not \\equiv 0(\\bmod p)$. This means that $p^{2}$ divides $x+y+2$. Using the same notation as in the first solution, we subtract the two original equations to obtain $$ p^{b}-p^{a}=y^{p-1}-x^{p-1}+x-y=(y-x)\\left(y^{p-2}+y^{p-3} x+\\cdots+x^{p-2}-1\\right) $$ The second factor is symmetric in $x$ and $y$, so it can be written as a polynomial of the elementary symmetric polynomials $x+y$ and $x y$ with integer coefficients. In particular, its value modulo $p^{2}$ is characterised by the two congruences $x y \\equiv 1\\left(\\bmod p^{2}\\right)$ and $x+y \\equiv-2\\left(\\bmod p^{2}\\right)$. Since both congruences are satisfied when $x=y=-1$, we must have $$ y^{p-2}+y^{p-3} x+\\cdots+x^{p-2}-1 \\equiv(-1)^{p-2}+(-1)^{p-3}(-1)+\\cdots+(-1)^{p-2}-1 \\quad\\left(\\bmod p^{2}\\right), $$ which simplifies to $y^{p-2}+y^{p-3} x+\\cdots+x^{p-2}-1 \\equiv-p\\left(\\bmod p^{2}\\right)$. Thus the second factor in (1) is divisible by $p$, but not $p^{2}$. This means that $p^{a-1}$ has to divide the other factor $y-x$. It follows that $$ 0 \\equiv x^{p-1}+y \\equiv x^{p-1}+x \\equiv x(x+1)\\left(x^{p-3}-x^{p-4}+\\cdots+1\\right) \\quad\\left(\\bmod p^{a-1}\\right) . $$ Since $x \\equiv-1(\\bmod p)$, the last factor is $x^{p-3}-x^{p-4}+\\cdots+1 \\equiv p-2(\\bmod p)$ and in particular not divisible by $p$. We infer that $p^{a-1} \\mid x+1$ and continue as in the first solution. Comment. Instead of reasoning by means of elementary symmetric polynomials, it is possible to provide a more direct argument as well. For odd $r,(x+1)^{2}$ divides $\\left(x^{r}+1\\right)^{2}$, and since $p$ divides $x+1$, we deduce that $p^{2}$ divides $\\left(x^{r}+1\\right)^{2}$. Together with the fact that $x y \\equiv 1\\left(\\bmod p^{2}\\right)$, we obtain $$ 0 \\equiv y^{r}\\left(x^{r}+1\\right)^{2} \\equiv x^{2 r} y^{r}+2 x^{r} y^{r}+y^{r} \\equiv x^{r}+2+y^{r} \\quad\\left(\\bmod p^{2}\\right) . $$ We apply this congruence with $r=p-2-2 k$ (where $0 \\leqslant k<(p-2) \/ 2$ ) to find that $$ x^{k} y^{p-2-k}+x^{p-2-k} y^{k} \\equiv(x y)^{k}\\left(x^{p-2-2 k}+y^{p-2-2 k}\\right) \\equiv 1^{k} \\cdot(-2) \\equiv-2 \\quad\\left(\\bmod p^{2}\\right) . $$ Summing over all $k$ yields $$ y^{p-2}+y^{p-3} x+\\cdots+x^{p-2}-1 \\equiv \\frac{p-1}{2} \\cdot(-2)-1 \\equiv-p \\quad\\left(\\bmod p^{2}\\right) $$ once again.","tier":0} diff --git a/IMO/segmented/en-IMO2015SL.jsonl b/IMO/segmented/en-IMO2015SL.jsonl new file mode 100644 index 0000000000000000000000000000000000000000..a607d3c31a62123097fdba799a7cd8dcb9339181 --- /dev/null +++ b/IMO/segmented/en-IMO2015SL.jsonl @@ -0,0 +1,80 @@ +{"problem_type":"Algebra","problem_label":"A1","problem":"Suppose that a sequence $a_{1}, a_{2}, \\ldots$ of positive real numbers satisfies $$ a_{k+1} \\geqslant \\frac{k a_{k}}{a_{k}^{2}+(k-1)} $$ for every positive integer $k$. Prove that $a_{1}+a_{2}+\\cdots+a_{n} \\geqslant n$ for every $n \\geqslant 2$.","solution":"From the constraint (1), it can be seen that $$ \\frac{k}{a_{k+1}} \\leqslant \\frac{a_{k}^{2}+(k-1)}{a_{k}}=a_{k}+\\frac{k-1}{a_{k}} $$ and so $$ a_{k} \\geqslant \\frac{k}{a_{k+1}}-\\frac{k-1}{a_{k}} . $$ Summing up the above inequality for $k=1, \\ldots, m$, we obtain $$ a_{1}+a_{2}+\\cdots+a_{m} \\geqslant\\left(\\frac{1}{a_{2}}-\\frac{0}{a_{1}}\\right)+\\left(\\frac{2}{a_{3}}-\\frac{1}{a_{2}}\\right)+\\cdots+\\left(\\frac{m}{a_{m+1}}-\\frac{m-1}{a_{m}}\\right)=\\frac{m}{a_{m+1}} $$ Now we prove the problem statement by induction on $n$. The case $n=2$ can be done by applying (1) to $k=1$ : $$ a_{1}+a_{2} \\geqslant a_{1}+\\frac{1}{a_{1}} \\geqslant 2 $$ For the induction step, assume that the statement is true for some $n \\geqslant 2$. If $a_{n+1} \\geqslant 1$, then the induction hypothesis yields $$ \\left(a_{1}+\\cdots+a_{n}\\right)+a_{n+1} \\geqslant n+1 $$ Otherwise, if $a_{n+1}<1$ then apply (2) as $$ \\left(a_{1}+\\cdots+a_{n}\\right)+a_{n+1} \\geqslant \\frac{n}{a_{n+1}}+a_{n+1}=\\frac{n-1}{a_{n+1}}+\\left(\\frac{1}{a_{n+1}}+a_{n+1}\\right)>(n-1)+2 $$ That completes the solution. Comment 1. It can be seen easily that having equality in the statement requires $a_{1}=a_{2}=1$ in the base case $n=2$, and $a_{n+1}=1$ in (3). So the equality $a_{1}+\\cdots+a_{n}=n$ is possible only in the trivial case $a_{1}=\\cdots=a_{n}=1$. Comment 2. After obtaining (2), there are many ways to complete the solution. We outline three such possibilities. - With defining $s_{n}=a_{1}+\\cdots+a_{n}$, the induction step can be replaced by $$ s_{n+1}=s_{n}+a_{n+1} \\geqslant s_{n}+\\frac{n}{s_{n}} \\geqslant n+1 $$ because the function $x \\mapsto x+\\frac{n}{x}$ increases on $[n, \\infty)$. - By applying the AM-GM inequality to the numbers $a_{1}+\\cdots+a_{k}$ and $k a_{k+1}$, we can conclude $$ a_{1}+\\cdots+a_{k}+k a_{k+1} \\geqslant 2 k $$ and sum it up for $k=1, \\ldots, n-1$. - We can derive the symmetric estimate $$ \\sum_{1 \\leqslant ib$ with $f(a)=f(b)$. A straightforward induction using (2) in the induction step reveals that we have $f(a+n)=f(b+n)$ for all nonnegative integers $n$. Consequently, the sequence $\\gamma_{n}=f(b+n)$ is periodic and thus in particular bounded, which means that the numbers $$ \\varphi=\\min _{n \\geqslant 0} \\gamma_{n} \\quad \\text { and } \\quad \\psi=\\max _{n \\geqslant 0} \\gamma_{n} $$ exist. Let us pick any integer $y$ with $f(y)=\\varphi$ and then an integer $x \\geqslant a$ with $f(x-f(y))=\\varphi$. Due to the definition of $\\varphi$ and (3) we have $$ \\varphi \\leqslant f(x+1)=f(x-f(y))+f(y)+1=2 \\varphi+1 $$ whence $\\varphi \\geqslant-1$. The same reasoning applied to $\\psi$ yields $\\psi \\leqslant-1$. Since $\\varphi \\leqslant \\psi$ holds trivially, it follows that $\\varphi=\\psi=-1$, or in other words that we have $f(t)=-1$ for all integers $t \\geqslant a$. Finally, if any integer $y$ is given, we may find an integer $x$ which is so large that $x+1 \\geqslant a$ and $x-f(y) \\geqslant a$ hold. Due to (3) and the result from the previous paragraph we get $$ f(y)=f(x+1)-f(x-f(y))-1=(-1)-(-1)-1=-1 . $$ Thereby the problem is solved.","tier":0} +{"problem_type":"Algebra","problem_label":"A2","problem":"Determine all functions $f: \\mathbb{Z} \\rightarrow \\mathbb{Z}$ with the property that $$ f(x-f(y))=f(f(x))-f(y)-1 $$ holds for all $x, y \\in \\mathbb{Z}$. (Croatia)","solution":"Set $d=f(0)$. By plugging $x=f(y)$ into (1) we obtain $$ f^{3}(y)=f(y)+d+1 $$ for all $y \\in \\mathbb{Z}$, where the left-hand side abbreviates $f(f(f(y)))$. When we replace $x$ in (1) by $f(x)$ we obtain $f(f(x)-f(y))=f^{3}(x)-f(y)-1$ and as a consequence of (4) this simplifies to $$ f(f(x)-f(y))=f(x)-f(y)+d $$ Now we consider the set $$ E=\\{f(x)-d \\mid x \\in \\mathbb{Z}\\} $$ Given two integers $a$ and $b$ from $E$, we may pick some integers $x$ and $y$ with $f(x)=a+d$ and $f(y)=b+d$; now (5) tells us that $f(a-b)=(a-b)+d$, which means that $a-b$ itself exemplifies $a-b \\in E$. Thus, $$ E \\text { is closed under taking differences. } $$ Also, the definitions of $d$ and $E$ yield $0 \\in E$. If $E=\\{0\\}$, then $f$ is a constant function and (1) implies that the only value attained by $f$ is indeed -1 . So let us henceforth suppose that $E$ contains some number besides zero. It is known that in this case (6) entails $E$ to be the set of all integer multiples of some positive integer $k$. Indeed, this holds for $$ k=\\min \\{|x| \\mid x \\in E \\text { and } x \\neq 0\\} $$ as one may verify by an argument based on division with remainder. Thus we have $$ \\{f(x) \\mid x \\in \\mathbb{Z}\\}=\\{k \\cdot t+d \\mid t \\in \\mathbb{Z}\\} $$ Due to (5) and (7) we get $$ f(k \\cdot t)=k \\cdot t+d $$ for all $t \\in \\mathbb{Z}$, whence in particular $f(k)=k+d$. So by comparing the results of substituting $y=0$ and $y=k$ into (1) we learn that $$ f(z+k)=f(z)+k $$ holds for all integers $z$. In plain English, this means that on any residue class modulo $k$ the function $f$ is linear with slope 1. Now by (7) the set of all values attained by $f$ is such a residue class. Hence, there exists an absolute constant $c$ such that $f(f(x))=f(x)+c$ holds for all $x \\in \\mathbb{Z}$. Thereby (1) simplifies to $$ f(x-f(y))=f(x)-f(y)+c-1 $$ On the other hand, considering (1) modulo $k$ we obtain $d \\equiv-1(\\bmod k)$ because of (7). So by (7) again, $f$ attains the value -1 . Thus we may apply (9) to some integer $y$ with $f(y)=-1$, which gives $f(x+1)=f(x)+c$. So $f$ is a linear function with slope $c$. Hence, (8) leads to $c=1$, wherefore there is an absolute constant $d^{\\prime}$ with $f(x)=x+d^{\\prime}$ for all $x \\in \\mathbb{Z}$. Using this for $x=0$ we obtain $d^{\\prime}=d$ and finally (4) discloses $d=1$, meaning that $f$ is indeed the successor function.","tier":0} +{"problem_type":"Algebra","problem_label":"A3","problem":"Let $n$ be a fixed positive integer. Find the maximum possible value of $$ \\sum_{1 \\leqslant r1$ is odd. One can modify the arguments of the last part in order to work for every (not necessarily odd) sufficiently large value of $k$; namely, when $k$ is even, one may show that the sequence $P(1), P(2), \\ldots, P(k)$ has different numbers of positive and negative terms. On the other hand, the problem statement with $k$ replaced by 2 is false, since the polynomials $P(x)=T(x)-T(x-1)$ and $Q(x)=T(x-1)-T(x)$ are block-similar in this case, due to the fact that $P(2 i-1)=-P(2 i)=Q(2 i)=-Q(2 i-1)=T(2 i-1)$ for all $i=1,2, \\ldots, n$. Thus, every complete solution should use the relation $k>2$. One may easily see that the condition $n \\geqslant 2$ is also substantial, since the polynomials $x$ and $k+1-x$ become block-similar if we set $n=1$. It is easily seen from the solution that the result still holds if we assume that the polynomials have degree at most $n$.","tier":0} +{"problem_type":"Algebra","problem_label":"A6","problem":"Let $n$ be a fixed integer with $n \\geqslant 2$. We say that two polynomials $P$ and $Q$ with real coefficients are block-similar if for each $i \\in\\{1,2, \\ldots, n\\}$ the sequences $$ \\begin{aligned} & P(2015 i), P(2015 i-1), \\ldots, P(2015 i-2014) \\quad \\text { and } \\\\ & Q(2015 i), Q(2015 i-1), \\ldots, Q(2015 i-2014) \\end{aligned} $$ are permutations of each other. (a) Prove that there exist distinct block-similar polynomials of degree $n+1$. (b) Prove that there do not exist distinct block-similar polynomials of degree $n$.","solution":"We provide an alternative argument for part (b). Assume again that there exist two distinct block-similar polynomials $P(x)$ and $Q(x)$ of degree $n$. Let $R(x)=P(x)-Q(x)$ and $S(x)=P(x)+Q(x)$. For brevity, we also denote the segment $[(i-1) k+1, i k]$ by $I_{i}$, and the set $\\{(i-1) k+1,(i-1) k+2, \\ldots, i k\\}$ of all integer points in $I_{i}$ by $Z_{i}$. Step 1. We prove that $R(x)$ has exactly one root in each segment $I_{i}, i=1,2, \\ldots, n$, and all these roots are simple. Indeed, take any $i \\in\\{1,2, \\ldots, n\\}$ and choose some points $p^{-}, p^{+} \\in Z_{i}$ so that $$ P\\left(p^{-}\\right)=\\min _{x \\in Z_{i}} P(x) \\quad \\text { and } \\quad P\\left(p^{+}\\right)=\\max _{x \\in Z_{i}} P(x) $$ Since the sequences of values of $P$ and $Q$ in $Z_{i}$ are permutations of each other, we have $R\\left(p^{-}\\right)=P\\left(p^{-}\\right)-Q\\left(p^{-}\\right) \\leqslant 0$ and $R\\left(p^{+}\\right)=P\\left(p^{+}\\right)-Q\\left(p^{+}\\right) \\geqslant 0$. Since $R(x)$ is continuous, there exists at least one root of $R(x)$ between $p^{-}$and $p^{+}$- thus in $I_{i}$. So, $R(x)$ has at least one root in each of the $n$ disjoint segments $I_{i}$ with $i=1,2, \\ldots, n$. Since $R(x)$ is nonzero and its degree does not exceed $n$, it should have exactly one root in each of these segments, and all these roots are simple, as required.","tier":0} +{"problem_type":"Combinatorics","problem_label":"C1","problem":"In Lineland there are $n \\geqslant 1$ towns, arranged along a road running from left to right. Each town has a left bulldozer (put to the left of the town and facing left) and a right bulldozer (put to the right of the town and facing right). The sizes of the $2 n$ bulldozers are distinct. Every time when a right and a left bulldozer confront each other, the larger bulldozer pushes the smaller one off the road. On the other hand, the bulldozers are quite unprotected at their rears; so, if a bulldozer reaches the rear-end of another one, the first one pushes the second one off the road, regardless of their sizes. Let $A$ and $B$ be two towns, with $B$ being to the right of $A$. We say that town $A$ can sweep town $B$ away if the right bulldozer of $A$ can move over to $B$ pushing off all bulldozers it meets. Similarly, $B$ can sweep $A$ away if the left bulldozer of $B$ can move to $A$ pushing off all bulldozers of all towns on its way. Prove that there is exactly one town which cannot be swept away by any other one. (Estonia)","solution":"Let $T_{1}, T_{2}, \\ldots, T_{n}$ be the towns enumerated from left to right. Observe first that, if town $T_{i}$ can sweep away town $T_{j}$, then $T_{i}$ also can sweep away every town located between $T_{i}$ and $T_{j}$. We prove the problem statement by strong induction on $n$. The base case $n=1$ is trivial. For the induction step, we first observe that the left bulldozer in $T_{1}$ and the right bulldozer in $T_{n}$ are completely useless, so we may forget them forever. Among the other $2 n-2$ bulldozers, we choose the largest one. Without loss of generality, it is the right bulldozer of some town $T_{k}$ with $kk$ may sweep away some town $T_{j}$ with $jm$. As we have already observed, $p$ cannot be greater than $k$. On the other hand, $T_{m}$ cannot sweep $T_{p}$ away, so a fortiori it cannot sweep $T_{k}$ away. Claim 2. Any town $T_{m}$ with $m \\neq k$ can be swept away by some other town. Proof. If $mk$. Let $T_{p}$ be a town among $T_{k}, T_{k+1}, \\ldots, T_{m-1}$ having the largest right bulldozer. We claim that $T_{p}$ can sweep $T_{m}$ away. If this is not the case, then $r_{p}<\\ell_{q}$ for some $q$ with $p1$. Firstly, we find a town which can be swept away by each of its neighbors (each town has two neighbors, except for the bordering ones each of which has one); we call such town a loser. Such a town exists, because there are $n-1$ pairs of neighboring towns, and in each of them there is only one which can sweep the other away; so there exists a town which is a winner in none of these pairs. Notice that a loser can be swept away, but it cannot sweep any other town away (due to its neighbors' protection). Now we remove a loser, and suggest its left bulldozer to its right neighbor (if it exists), and its right bulldozer to a left one (if it exists). Surely, a town accepts a suggestion if a suggested bulldozer is larger than the town's one of the same orientation. Notice that suggested bulldozers are useless in attack (by the definition of a loser), but may serve for defensive purposes. Moreover, each suggested bulldozer's protection works for the same pairs of remaining towns as before the removal. By the induction hypothesis, the new configuration contains exactly one town which cannot be swept away. The arguments above show that the initial one also satisfies this property.","tier":0} +{"problem_type":"Combinatorics","problem_label":"C1","problem":"In Lineland there are $n \\geqslant 1$ towns, arranged along a road running from left to right. Each town has a left bulldozer (put to the left of the town and facing left) and a right bulldozer (put to the right of the town and facing right). The sizes of the $2 n$ bulldozers are distinct. Every time when a right and a left bulldozer confront each other, the larger bulldozer pushes the smaller one off the road. On the other hand, the bulldozers are quite unprotected at their rears; so, if a bulldozer reaches the rear-end of another one, the first one pushes the second one off the road, regardless of their sizes. Let $A$ and $B$ be two towns, with $B$ being to the right of $A$. We say that town $A$ can sweep town $B$ away if the right bulldozer of $A$ can move over to $B$ pushing off all bulldozers it meets. Similarly, $B$ can sweep $A$ away if the left bulldozer of $B$ can move to $A$ pushing off all bulldozers of all towns on its way. Prove that there is exactly one town which cannot be swept away by any other one. (Estonia)","solution":"We separately prove that $(i)$ there exists a town which cannot be swept away, and that (ii) there is at most one such town. We also make use of the two observations from the previous solutions. To prove ( $i$ ), assume contrariwise that every town can be swept away. Let $t_{1}$ be the leftmost town; next, for every $k=1,2, \\ldots$ we inductively choose $t_{k+1}$ to be some town which can sweep $t_{k}$ away. Now we claim that for every $k=1,2, \\ldots$, the town $t_{k+1}$ is to the right of $t_{k}$; this leads to the contradiction, since the number of towns is finite. Induction on $k$. The base case $k=1$ is clear due to the choice of $t_{1}$. Assume now that for all $j$ with $1 \\leqslant jj$ and $a_{i}\\frac{n}{2}$, then we have $O A_{i-n \/ 2+1}=A_{i} A_{i-n \/ 2+1}$. This completes the proof. An example of such a construction when $n=10$ is shown in Figure 1. ![](https:\/\/cdn.mathpix.com\/cropped\/2024_11_18_62dbd531c72d0ff8164cg-28.jpg?height=507&width=804&top_left_y=1574&top_left_x=272) Figure 1 ![](https:\/\/cdn.mathpix.com\/cropped\/2024_11_18_62dbd531c72d0ff8164cg-28.jpg?height=600&width=617&top_left_y=1479&top_left_x=1185) Figure 2 Comment (a). There are many ways to construct an example by placing equilateral triangles in a circle. Here we present one general method. Let $O$ be the center of a circle and let $A_{1}, B_{1}, \\ldots, A_{k}, B_{k}$ be distinct points on the circle such that the triangle $O A_{i} B_{i}$ is equilateral for each $i$. Then $\\mathcal{V}=\\left\\{O, A_{1}, B_{1}, \\ldots, A_{k}, B_{k}\\right\\}$ is balanced. To construct a set of even cardinality, put extra points $C, D, E$ on the circle such that triangles $O C D$ and $O D E$ are equilateral (see Figure 2). Then $\\mathcal{V}=\\left\\{O, A_{1}, B_{1}, \\ldots, A_{k}, B_{k}, C, D, E\\right\\}$ is balanced. Part (b). We now show that there exists a balanced, center-free set containing $n$ points for all odd $n \\geqslant 3$, and that one does not exist for any even $n \\geqslant 3$. If $n$ is odd, then let $\\mathcal{V}$ be the set of vertices of a regular $n$-gon. We have shown in part ( $a$ ) that $\\mathcal{V}$ is balanced. We claim that $\\mathcal{V}$ is also center-free. Indeed, if $P$ is a point such that $P A=P B=P C$ for some three distinct vertices $A, B$ and $C$, then $P$ is the circumcenter of the $n$-gon, which is not contained in $\\mathcal{V}$. Now suppose that $\\mathcal{V}$ is a balanced, center-free set of even cardinality $n$. We will derive a contradiction. For a pair of distinct points $A, B \\in \\mathcal{V}$, we say that a point $C \\in \\mathcal{V}$ is associated with the pair $\\{A, B\\}$ if $A C=B C$. Since there are $\\frac{n(n-1)}{2}$ pairs of points, there exists a point $P \\in \\mathcal{V}$ which is associated with at least $\\left\\lceil\\frac{n(n-1)}{2} \/ n\\right\\rceil=\\frac{n}{2}$ pairs. Note that none of these $\\frac{n}{2}$ pairs can contain $P$, so that the union of these $\\frac{n}{2}$ pairs consists of at most $n-1$ points. Hence there exist two such pairs that share a point. Let these two pairs be $\\{A, B\\}$ and $\\{A, C\\}$. Then $P A=P B=P C$, which is a contradiction. Comment (b). We can rephrase the argument in graph theoretic terms as follows. Let $\\mathcal{V}$ be a balanced, center-free set consisting of $n$ points. For any pair of distinct vertices $A, B \\in \\mathcal{V}$ and for any $C \\in \\mathcal{V}$ such that $A C=B C$, draw directed edges $A \\rightarrow C$ and $B \\rightarrow C$. Then all pairs of vertices generate altogether at least $n(n-1)$ directed edges; since the set is center-free, these edges are distinct. So we must obtain a graph in which any two vertices are connected in both directions. Now, each vertex has exactly $n-1$ incoming edges, which means that $n-1$ is even. Hence $n$ is odd.","tier":0} +{"problem_type":"Combinatorics","problem_label":"C3","problem":"For a finite set $A$ of positive integers, we call a partition of $A$ into two disjoint nonempty subsets $A_{1}$ and $A_{2}$ good if the least common multiple of the elements in $A_{1}$ is equal to the greatest common divisor of the elements in $A_{2}$. Determine the minimum value of $n$ such that there exists a set of $n$ positive integers with exactly 2015 good partitions. (Ukraine)","solution":"Let $A=\\left\\{a_{1}, a_{2}, \\ldots, a_{n}\\right\\}$, where $a_{1}1$ and $B$ can respond by choosing $a-1$ on the $k^{\\text {th }}$ move instead. We now give an alternative winning strategy in the case $n$ is even and $n \\geqslant 8$. We first present a winning strategy for the case when $A$ 's first pick is 1 . We consider two cases depending on $A$ 's second move. Case 1. A's second pick is 3 . Then $B$ chooses $n-3$ on the second move. On the $k^{\\text {th }}$ move, $B$ chooses the number exactly 1 less than $A^{\\prime}$ 's $k^{\\text {th }}$ pick except that $B$ chooses 2 if $A$ 's $k^{\\text {th }}$ pick is $n-2$ or $n-1$. Case 2. A's second pick is $a>3$. Then $B$ chooses $a-2$ on the second move. Afterwards on the $k^{\\text {th }}$ move, $B$ picks the number exactly 1 less than $A^{\\text {'s }} k^{\\text {th }}$ pick. One may easily see that this strategy guarantees $B$ 's victory, when $A$ 's first pick is 1 . The following claim shows how to extend the strategy to the general case. Claim. Assume that $B$ has an explicit strategy leading to a victory after $A$ picks 1 on the first move. Then $B$ also has an explicit strategy leading to a victory after any first moves of $A$. Proof. Let $S$ be an optimal strategy of $B$ after $A$ picks 1 on the first move. Assume that $A$ picks some number $a>1$ on this move; we show how $B$ can make use of $S$ in order to win in this case. In parallel to the real play, $B$ starts an imaginary play. The positions in these plays differ by flipping the segment $[1, a]$; so, if a player chooses some number $x$ in the real play, then the same player chooses a number $x$ or $a+1-x$ in the imaginary play, depending on whether $x>a$ or $x \\leqslant a$. Thus $A$ 's first pick in the imaginary play is 1. Clearly, a number is chosen in the real play exactly if the corresponding number is chosen in the imaginary one. Next, if an unchosen number is neighboring to one chosen by $A$ in the imaginary play, then the corresponding number also has this property in the real play, so $A$ also cannot choose it. One can easily see that a similar statement with real and imaginary plays interchanged holds for $B$ instead of $A$. Thus, when $A$ makes some move in the real play, $B$ may imagine the corresponding legal move in the imaginary one. Then $B$ chooses the response according to $S$ in the imaginary game and makes the corresponding legal move in the real one. Acting so, $B$ wins the imaginary game, thus $B$ will also win the real one. Hence, $B$ has a winning strategy for all even $n$ greater or equal to 8 . Notice that the claim can also be used to simplify the argument when $n$ is odd. Comment 2. One may also employ symmetry when $n$ is odd. In particular, $B$ could use a mirror strategy. However, additional ideas are required to modify the strategy after $A$ picks $\\frac{n+1}{2}$.","tier":0} +{"problem_type":"Combinatorics","problem_label":"C5","problem":"Consider an infinite sequence $a_{1}, a_{2}, \\ldots$ of positive integers with $a_{i} \\leqslant 2015$ for all $i \\geqslant 1$. Suppose that for any two distinct indices $i$ and $j$ we have $i+a_{i} \\neq j+a_{j}$. Prove that there exist two positive integers $b$ and $N$ such that $$ \\left|\\sum_{i=m+1}^{n}\\left(a_{i}-b\\right)\\right| \\leqslant 1007^{2} $$ whenever $n>m \\geqslant N$.","solution":"We visualize the set of positive integers as a sequence of points. For each $n$ we draw an arrow emerging from $n$ that points to $n+a_{n}$; so the length of this arrow is $a_{n}$. Due to the condition that $m+a_{m} \\neq n+a_{n}$ for $m \\neq n$, each positive integer receives at most one arrow. There are some positive integers, such as 1 , that receive no arrows; these will be referred to as starting points in the sequel. When one starts at any of the starting points and keeps following the arrows, one is led to an infinite path, called its ray, that visits a strictly increasing sequence of positive integers. Since the length of any arrow is at most 2015, such a ray, say with starting point $s$, meets every interval of the form $[n, n+2014]$ with $n \\geqslant s$ at least once. Suppose for the sake of contradiction that there would be at least 2016 starting points. Then we could take an integer $n$ that is larger than the first 2016 starting points. But now the interval $[n, n+2014]$ must be met by at least 2016 rays in distinct points, which is absurd. We have thereby shown that the number $b$ of starting points satisfies $1 \\leqslant b \\leqslant 2015$. Let $N$ denote any integer that is larger than all starting points. We contend that $b$ and $N$ are as required. To see this, let any two integers $m$ and $n$ with $n>m \\geqslant N$ be given. The sum $\\sum_{i=m+1}^{n} a_{i}$ gives the total length of the arrows emerging from $m+1, \\ldots, n$. Taken together, these arrows form $b$ subpaths of our rays, some of which may be empty. Now on each ray we look at the first number that is larger than $m$; let $x_{1}, \\ldots, x_{b}$ denote these numbers, and let $y_{1}, \\ldots, y_{b}$ enumerate in corresponding order the numbers defined similarly with respect to $n$. Then the list of differences $y_{1}-x_{1}, \\ldots, y_{b}-x_{b}$ consists of the lengths of these paths and possibly some zeros corresponding to empty paths. Consequently, we obtain $$ \\sum_{i=m+1}^{n} a_{i}=\\sum_{j=1}^{b}\\left(y_{j}-x_{j}\\right) $$ whence $$ \\sum_{i=m+1}^{n}\\left(a_{i}-b\\right)=\\sum_{j=1}^{b}\\left(y_{j}-n\\right)-\\sum_{j=1}^{b}\\left(x_{j}-m\\right) . $$ Now each of the $b$ rays meets the interval $[m+1, m+2015]$ at some point and thus $x_{1}-$ $m, \\ldots, x_{b}-m$ are $b$ distinct members of the set $\\{1,2, \\ldots, 2015\\}$. Moreover, since $m+1$ is not a starting point, it must belong to some ray; so 1 has to appear among these numbers, wherefore $$ 1+\\sum_{j=1}^{b-1}(j+1) \\leqslant \\sum_{j=1}^{b}\\left(x_{j}-m\\right) \\leqslant 1+\\sum_{j=1}^{b-1}(2016-b+j) $$ The same argument applied to $n$ and $y_{1}, \\ldots, y_{b}$ yields $$ 1+\\sum_{j=1}^{b-1}(j+1) \\leqslant \\sum_{j=1}^{b}\\left(y_{j}-n\\right) \\leqslant 1+\\sum_{j=1}^{b-1}(2016-b+j) $$ So altogether we get $$ \\begin{aligned} \\left|\\sum_{i=m+1}^{n}\\left(a_{i}-b\\right)\\right| & \\leqslant \\sum_{j=1}^{b-1}((2016-b+j)-(j+1))=(b-1)(2015-b) \\\\ & \\leqslant\\left(\\frac{(b-1)+(2015-b)}{2}\\right)^{2}=1007^{2}, \\end{aligned} $$ as desired.","tier":0} +{"problem_type":"Combinatorics","problem_label":"C5","problem":"Consider an infinite sequence $a_{1}, a_{2}, \\ldots$ of positive integers with $a_{i} \\leqslant 2015$ for all $i \\geqslant 1$. Suppose that for any two distinct indices $i$ and $j$ we have $i+a_{i} \\neq j+a_{j}$. Prove that there exist two positive integers $b$ and $N$ such that $$ \\left|\\sum_{i=m+1}^{n}\\left(a_{i}-b\\right)\\right| \\leqslant 1007^{2} $$ whenever $n>m \\geqslant N$.","solution":"Set $s_{n}=n+a_{n}$ for all positive integers $n$. By our assumptions, we have $$ n+1 \\leqslant s_{n} \\leqslant n+2015 $$ for all $n \\in \\mathbb{Z}_{>0}$. The members of the sequence $s_{1}, s_{2}, \\ldots$ are distinct. We shall investigate the set $$ M=\\mathbb{Z}_{>0} \\backslash\\left\\{s_{1}, s_{2}, \\ldots\\right\\} $$ Claim. At most 2015 numbers belong to $M$. Proof. Otherwise let $m_{1}m \\geqslant N$. Plugging $r=n$ and $r=m$ into (3) and subtracting the estimates that result, we deduce $$ \\sum_{i=m+1}^{n}\\left(a_{i}-b\\right)=\\sum C_{n}-\\sum C_{m} $$ Since $C_{n}$ and $C_{m}$ are subsets of $\\{1,2, \\ldots, 2014\\}$ with $\\left|C_{n}\\right|=\\left|C_{m}\\right|=b-1$, it is clear that the absolute value of the right-hand side of the above inequality attains its largest possible value if either $C_{m}=\\{1,2, \\ldots, b-1\\}$ and $C_{n}=\\{2016-b, \\ldots, 2014\\}$, or the other way around. In these two cases we have $$ \\left|\\sum C_{n}-\\sum C_{m}\\right|=(b-1)(2015-b) $$ so in the general case we find $$ \\left|\\sum_{i=m+1}^{n}\\left(a_{i}-b\\right)\\right| \\leqslant(b-1)(2015-b) \\leqslant\\left(\\frac{(b-1)+(2015-b)}{2}\\right)^{2}=1007^{2} $$ as desired. Comment. The sets $C_{n}$ may be visualized by means of the following process: Start with an empty blackboard. For $n \\geqslant 1$, the following happens during the $n^{\\text {th }}$ step. The number $a_{n}$ gets written on the blackboard, then all numbers currently on the blackboard are decreased by 1 , and finally all zeros that have arisen get swept away. It is not hard to see that the numbers present on the blackboard after $n$ steps are distinct and form the set $C_{n}$. Moreover, it is possible to complete a solution based on this idea.","tier":0} +{"problem_type":"Combinatorics","problem_label":"C6","problem":"Let $S$ be a nonempty set of positive integers. We say that a positive integer $n$ is clean if it has a unique representation as a sum of an odd number of distinct elements from $S$. Prove that there exist infinitely many positive integers that are not clean.","solution":"Define an odd (respectively, even) representation of $n$ to be a representation of $n$ as a sum of an odd (respectively, even) number of distinct elements of $S$. Let $\\mathbb{Z}_{>0}$ denote the set of all positive integers. Suppose, to the contrary, that there exist only finitely many positive integers that are not clean. Therefore, there exists a positive integer $N$ such that every integer $n>N$ has exactly one odd representation. Clearly, $S$ is infinite. We now claim the following properties of odd and even representations. $\\underline{P r o p e r t y}$ 1. Any positive integer $n$ has at most one odd and at most one even representation. Proof. We first show that every integer $n$ has at most one even representation. Since $S$ is infinite, there exists $x \\in S$ such that $x>\\max \\{n, N\\}$. Then, the number $n+x$ must be clean, and $x$ does not appear in any even representation of $n$. If $n$ has more than one even representation, then we obtain two distinct odd representations of $n+x$ by adding $x$ to the even representations of $n$, which is impossible. Therefore, $n$ can have at most one even representation. Similarly, there exist two distinct elements $y, z \\in S$ such that $y, z>\\max \\{n, N\\}$. If $n$ has more than one odd representation, then we obtain two distinct odd representations of $n+y+z$ by adding $y$ and $z$ to the odd representations of $n$. This is again a contradiction. Property 2. Fix $s \\in S$. Suppose that a number $n>N$ has no even representation. Then $n+2 a s$ has an even representation containing $s$ for all integers $a \\geqslant 1$. Proof. It is sufficient to prove the following statement: If $n$ has no even representation without $s$, then $n+2 s$ has an even representation containing $s$ (and hence no even representation without $s$ by Property 1). Notice that the odd representation of $n+s$ does not contain $s$; otherwise, we have an even representation of $n$ without $s$. Then, adding $s$ to this odd representation of $n+s$, we get that $n+2 s$ has an even representation containing $s$, as desired. Property 3. Every sufficiently large integer has an even representation. Proof. Fix any $s \\in S$, and let $r$ be an arbitrary element in $\\{1,2, \\ldots, 2 s\\}$. Then, Property 2 implies that the set $Z_{r}=\\{r+2 a s: a \\geqslant 0\\}$ contains at most one number exceeding $N$ with no even representation. Therefore, $Z_{r}$ contains finitely many positive integers with no even representation, and so does $\\mathbb{Z}_{>0}=\\bigcup_{r=1}^{2 s} Z_{r}$. In view of Properties 1 and 3 , we may assume that $N$ is chosen such that every $n>N$ has exactly one odd and exactly one even representation. In particular, each element $s>N$ of $S$ has an even representation. Property 4. For any $s, t \\in S$ with $NN$. Then, Property 4 implies that for every $i>k$ the even representation of $s_{i}$ contains all the numbers $s_{k}, s_{k+1}, \\ldots, s_{i-1}$. Therefore, $$ s_{i}=s_{k}+s_{k+1}+\\cdots+s_{i-1}+R_{i}=\\sigma_{i-1}-\\sigma_{k-1}+R_{i} $$ where $R_{i}$ is a sum of some of $s_{1}, \\ldots, s_{k-1}$. In particular, $0 \\leqslant R_{i} \\leqslant s_{1}+\\cdots+s_{k-1}=\\sigma_{k-1}$. Let $j_{0}$ be an integer satisfying $j_{0}>k$ and $\\sigma_{j_{0}}>2 \\sigma_{k-1}$. Then (1) shows that, for every $j>j_{0}$, $$ s_{j+1} \\geqslant \\sigma_{j}-\\sigma_{k-1}>\\sigma_{j} \/ 2 . $$ Next, let $p>j_{0}$ be an index such that $R_{p}=\\min _{i>j_{0}} R_{i}$. Then, $$ s_{p+1}=s_{k}+s_{k+1}+\\cdots+s_{p}+R_{p+1}=\\left(s_{p}-R_{p}\\right)+s_{p}+R_{p+1} \\geqslant 2 s_{p} $$ Therefore, there is no element of $S$ larger than $s_{p}$ but smaller than $2 s_{p}$. It follows that the even representation $\\tau$ of $2 s_{p}$ does not contain any element larger than $s_{p}$. On the other hand, inequality (2) yields $2 s_{p}>s_{1}+\\cdots+s_{p-1}$, so $\\tau$ must contain a term larger than $s_{p-1}$. Thus, it must contain $s_{p}$. After removing $s_{p}$ from $\\tau$, we have that $s_{p}$ has an odd representation not containing $s_{p}$, which contradicts Property 1 since $s_{p}$ itself also forms an odd representation of $s_{p}$.","tier":0} +{"problem_type":"Combinatorics","problem_label":"C6","problem":"Let $S$ be a nonempty set of positive integers. We say that a positive integer $n$ is clean if it has a unique representation as a sum of an odd number of distinct elements from $S$. Prove that there exist infinitely many positive integers that are not clean.","solution":"We will also use Property 1 from We first define some terminology and notations used in this solution. Let $\\mathbb{Z}_{\\geqslant 0}$ denote the set of all nonnegative integers. All sums mentioned are regarded as sums of distinct elements of $S$. Moreover, a sum is called even or odd depending on the parity of the number of terms in it. All closed or open intervals refer to sets of all integers inside them, e.g., $[a, b]=\\{x \\in \\mathbb{Z}: a \\leqslant x \\leqslant b\\}$. Again, let $s_{1}$ $2^{n-1}-1+n m \\geqslant \\sigma_{n}$ for every sufficiently large $n$. We now claim the following. Claim 1. $\\left(\\sigma_{n}-s_{n+1}, s_{n+2}-s_{n+1}\\right) \\subseteq E_{n}$ for every sufficiently large $n$. Proof. For sufficiently large $n$, all elements of $\\left(\\sigma_{n}, s_{n+2}\\right)$ are clean. Clearly, the elements of $\\left(\\sigma_{n}, s_{n+2}\\right)$ can be in neither $O_{n}$ nor $O \\backslash O_{n+1}$. So, $\\left(\\sigma_{n}, s_{n+2}\\right) \\subseteq O_{n+1} \\backslash O_{n}=s_{n+1}+E_{n}$, which yields the claim. Now, Claim 1 together with inequalities (3) implies that, for all sufficiently large $n$, $$ E \\supseteq E_{n} \\supseteq\\left(\\sigma_{n}-s_{n+1}, s_{n+2}-s_{n+1}\\right) \\supseteq\\left(2 n m, 2^{n-1}-(n+2) m\\right) . $$ This easily yields that $\\mathbb{Z}_{\\geqslant 0} \\backslash E$ is also finite. Since $\\mathbb{Z}_{\\geqslant 0} \\backslash O$ is also finite, by Property 1 , there exists a positive integer $N$ such that every integer $n>N$ has exactly one even and one odd representation. Step 3. We investigate the structures of $E_{n}$ and $O_{n}$. Suppose that $z \\in E_{2 n}$. Since $z$ can be represented as an even sum using $\\left\\{s_{1}, s_{2}, \\ldots, s_{2 n}\\right\\}$, so can its complement $\\sigma_{2 n}-z$. Thus, we get $E_{2 n}=\\sigma_{2 n}-E_{2 n}$. Similarly, we have $$ E_{2 n}=\\sigma_{2 n}-E_{2 n}, \\quad O_{2 n}=\\sigma_{2 n}-O_{2 n}, \\quad E_{2 n+1}=\\sigma_{2 n+1}-O_{2 n+1}, \\quad O_{2 n+1}=\\sigma_{2 n+1}-E_{2 n+1} $$ Claim 2. For every sufficiently large $n$, we have $$ \\left[0, \\sigma_{n}\\right] \\supseteq O_{n} \\supseteq\\left(N, \\sigma_{n}-N\\right) \\quad \\text { and } \\quad\\left[0, \\sigma_{n}\\right] \\supseteq E_{n} \\supseteq\\left(N, \\sigma_{n}-N\\right) $$ Proof. Clearly $O_{n}, E_{n} \\subseteq\\left[0, \\sigma_{n}\\right]$ for every positive integer $n$. We now prove $O_{n}, E_{n} \\supseteq\\left(N, \\sigma_{n}-N\\right)$. Taking $n$ sufficiently large, we may assume that $s_{n+1} \\geqslant 2^{n-1}+1-n m>\\frac{1}{2}\\left(2^{n-1}-1+n m\\right) \\geqslant \\sigma_{n} \/ 2$. Therefore, the odd representation of every element of ( $\\left.N, \\sigma_{n} \/ 2\\right]$ cannot contain a term larger than $s_{n}$. Thus, $\\left(N, \\sigma_{n} \/ 2\\right] \\subseteq O_{n}$. Similarly, since $s_{n+1}+s_{1}>\\sigma_{n} \/ 2$, we also have $\\left(N, \\sigma_{n} \/ 2\\right] \\subseteq E_{n}$. Equations (4) then yield that, for sufficiently large $n$, the interval $\\left(N, \\sigma_{n}-N\\right)$ is a subset of both $O_{n}$ and $E_{n}$, as desired. Step 4. We obtain a final contradiction. Notice that $0 \\in \\mathbb{Z}_{\\geqslant 0} \\backslash O$ and $1 \\in \\mathbb{Z}_{\\geqslant 0} \\backslash E$. Therefore, the sets $\\mathbb{Z}_{\\geqslant 0} \\backslash O$ and $\\mathbb{Z}_{\\geqslant 0} \\backslash E$ are nonempty. Denote $o=\\max \\left(\\mathbb{Z}_{\\geqslant 0} \\backslash O\\right)$ and $e=\\max \\left(\\mathbb{Z}_{\\geqslant 0} \\backslash E\\right)$. Observe also that $e, o \\leqslant N$. Taking $k$ sufficiently large, we may assume that $\\sigma_{2 k}>2 N$ and that Claim 2 holds for all $n \\geqslant 2 k$. Due to (4) and Claim 2, we have that $\\sigma_{2 k}-e$ is the minimal number greater than $N$ which is not in $E_{2 k}$, i.e., $\\sigma_{2 k}-e=s_{2 k+1}+s_{1}$. Similarly, $$ \\sigma_{2 k}-o=s_{2 k+1}, \\quad \\sigma_{2 k+1}-e=s_{2 k+2}, \\quad \\text { and } \\quad \\sigma_{2 k+1}-o=s_{2 k+2}+s_{1} $$ Therefore, we have $$ \\begin{aligned} s_{1} & =\\left(s_{2 k+1}+s_{1}\\right)-s_{2 k+1}=\\left(\\sigma_{2 k}-e\\right)-\\left(\\sigma_{2 k}-o\\right)=o-e \\\\ & =\\left(\\sigma_{2 k+1}-e\\right)-\\left(\\sigma_{2 k+1}-o\\right)=s_{2 k+2}-\\left(s_{2 k+2}+s_{1}\\right)=-s_{1} \\end{aligned} $$ which is impossible since $s_{1}>0$.","tier":0} +{"problem_type":"Combinatorics","problem_label":"C7","problem":"In a company of people some pairs are enemies. A group of people is called unsociable if the number of members in the group is odd and at least 3 , and it is possible to arrange all its members around a round table so that every two neighbors are enemies. Given that there are at most 2015 unsociable groups, prove that it is possible to partition the company into 11 parts so that no two enemies are in the same part.","solution":"Let $G=(V, E)$ be a graph where $V$ is the set of people in the company and $E$ is the set of the enemy pairs - the edges of the graph. In this language, partitioning into 11 disjoint enemy-free subsets means properly coloring the vertices of this graph with 11 colors. We will prove the following more general statement. Claim. Let $G$ be a graph with chromatic number $k \\geqslant 3$. Then $G$ contains at least $2^{k-1}-k$ unsociable groups. Recall that the chromatic number of $G$ is the least $k$ such that a proper coloring $$ V=V_{1} \\sqcup \\cdots \\sqcup V_{k} $$ exists. In view of $2^{11}-12>2015$, the claim implies the problem statement. Let $G$ be a graph with chromatic number $k$. We say that a proper coloring (1) of $G$ is leximinimal, if the $k$-tuple $\\left(\\left|V_{1}\\right|,\\left|V_{2}\\right|, \\ldots,\\left|V_{k}\\right|\\right)$ is lexicographically minimal; in other words, the following conditions are satisfied: the number $n_{1}=\\left|V_{1}\\right|$ is minimal; the number $n_{2}=\\left|V_{2}\\right|$ is minimal, subject to the previously chosen value of $n_{1} ; \\ldots$; the number $n_{k-1}=\\left|V_{k-1}\\right|$ is minimal, subject to the previously chosen values of $n_{1}, \\ldots, n_{k-2}$. The following lemma is the core of the proof. Lemma 1. Suppose that $G=(V, E)$ is a graph with odd chromatic number $k \\geqslant 3$, and let (1) be one of its leximinimal colorings. Then $G$ contains an odd cycle which visits all color classes $V_{1}, V_{2}, \\ldots, V_{k}$. Proof of Lemma 1. Let us call a cycle colorful if it visits all color classes. Due to the definition of the chromatic number, $V_{1}$ is nonempty. Choose an arbitrary vertex $v \\in V_{1}$. We construct a colorful odd cycle that has only one vertex in $V_{1}$, and this vertex is $v$. We draw a subgraph of $G$ as follows. Place $v$ in the center, and arrange the sets $V_{2}, V_{3}, \\ldots, V_{k}$ in counterclockwise circular order around it. For convenience, let $V_{k+1}=V_{2}$. We will draw arrows to add direction to some edges of $G$, and mark the vertices these arrows point to. First we draw arrows from $v$ to all its neighbors in $V_{2}$, and mark all those neighbors. If some vertex $u \\in V_{i}$ with $i \\in\\{2,3, \\ldots, k\\}$ is already marked, we draw arrows from $u$ to all its neighbors in $V_{i+1}$ which are not marked yet, and we mark all of them. We proceed doing this as long as it is possible. The process of marking is exemplified in Figure 1. Notice that by the rules of our process, in the final state, marked vertices in $V_{i}$ cannot have unmarked neighbors in $V_{i+1}$. Moreover, $v$ is connected to all marked vertices by directed paths. Now move each marked vertex to the next color class in circular order (see an example in Figure 3). In view of the arguments above, the obtained coloring $V_{1} \\sqcup W_{2} \\sqcup \\cdots \\sqcup W_{k}$ is proper. Notice that $v$ has a neighbor $w \\in W_{2}$, because otherwise $$ \\left(V_{1} \\backslash\\{v\\}\\right) \\sqcup\\left(W_{2} \\cup\\{v\\}\\right) \\sqcup W_{3} \\sqcup \\cdots \\sqcup W_{k} $$ would be a proper coloring lexicographically smaller than (1). If $w$ was unmarked, i.e., $w$ was an element of $V_{2}$, then it would be marked at the beginning of the process and thus moved to $V_{3}$, which did not happen. Therefore, $w$ is marked and $w \\in V_{k}$. ![](https:\/\/cdn.mathpix.com\/cropped\/2024_11_18_62dbd531c72d0ff8164cg-40.jpg?height=780&width=1658&top_left_y=181&top_left_x=219) Figure 1 Figure 2 Since $w$ is marked, there exists a directed path from $v$ to $w$. This path moves through the sets $V_{2}, \\ldots, V_{k}$ in circular order, so the number of edges in it is divisible by $k-1$ and thus even. Closing this path by the edge $w \\rightarrow v$, we get a colorful odd cycle, as required. Proof of the claim. Let us choose a leximinimal coloring (1) of $G$. For every set $C \\subseteq\\{1,2, \\ldots, k\\}$ such that $|C|$ is odd and greater than 1 , we will provide an odd cycle visiting exactly those color classes whose indices are listed in the set $C$. This property ensures that we have different cycles for different choices of $C$, and it proves the claim because there are $2^{k-1}-k$ choices for the set $C$. Let $V_{C}=\\bigcup_{c \\in C} V_{c}$, and let $G_{C}$ be the induced subgraph of $G$ on the vertex set $V_{C}$. We also have the induced coloring of $V_{C}$ with $|C|$ colors; this coloring is of course proper. Notice further that the induced coloring is leximinimal: if we had a lexicographically smaller coloring $\\left(W_{c}\\right)_{c \\in C}$ of $G_{C}$, then these classes, together the original color classes $V_{i}$ for $i \\notin C$, would provide a proper coloring which is lexicographically smaller than (1). Hence Lemma 1, applied to the subgraph $G_{C}$ and its leximinimal coloring $\\left(V_{c}\\right)_{c \\in C}$, provides an odd cycle that visits exactly those color classes that are listed in the set $C$.","tier":0} +{"problem_type":"Combinatorics","problem_label":"C7","problem":"In a company of people some pairs are enemies. A group of people is called unsociable if the number of members in the group is odd and at least 3 , and it is possible to arrange all its members around a round table so that every two neighbors are enemies. Given that there are at most 2015 unsociable groups, prove that it is possible to partition the company into 11 parts so that no two enemies are in the same part.","solution":"We provide a different proof of the claim from the previous solution. We say that a graph is critical if deleting any vertex from the graph decreases the graph's chromatic number. Obviously every graph contains a critical induced subgraph with the same chromatic number. Lemma 2. Suppose that $G=(V, E)$ is a critical graph with chromatic number $k \\geqslant 3$. Then every vertex $v$ of $G$ is contained in at least $2^{k-2}-1$ unsociable groups. Proof. For every set $X \\subseteq V$, denote by $n(X)$ the number of neighbors of $v$ in the set $X$. Since $G$ is critical, there exists a proper coloring of $G \\backslash\\{v\\}$ with $k-1$ colors, so there exists a proper coloring $V=V_{1} \\sqcup V_{2} \\sqcup \\cdots \\sqcup V_{k}$ of $G$ such that $V_{1}=\\{v\\}$. Among such colorings, take one for which the sequence $\\left(n\\left(V_{2}\\right), n\\left(V_{3}\\right), \\ldots, n\\left(V_{k}\\right)\\right)$ is lexicographically minimal. Clearly, $n\\left(V_{i}\\right)>0$ for every $i=2,3, \\ldots, k$; otherwise $V_{2} \\sqcup \\ldots \\sqcup V_{i-1} \\sqcup\\left(V_{i} \\cup V_{1}\\right) \\sqcup V_{i+1} \\sqcup \\ldots V_{k}$ would be a proper coloring of $G$ with $k-1$ colors. We claim that for every $C \\subseteq\\{2,3, \\ldots, k\\}$ with $|C| \\geqslant 2$ being even, $G$ contains an unsociable group so that the set of its members' colors is precisely $C \\cup\\{1\\}$. Since the number of such sets $C$ is $2^{k-2}-1$, this proves the lemma. Denote the elements of $C$ by $c_{1}, \\ldots, c_{2 \\ell}$ in increasing order. For brevity, let $U_{i}=V_{c_{i}}$. Denote by $N_{i}$ the set of neighbors of $v$ in $U_{i}$. We show that for every $i=1, \\ldots, 2 \\ell-1$ and $x \\in N_{i}$, the subgraph induced by $U_{i} \\cup U_{i+1}$ contains a path that connects $x$ with another point in $N_{i+1}$. For the sake of contradiction, suppose that no such path exists. Let $S$ be the set of vertices that lie in the connected component of $x$ in the subgraph induced by $U_{i} \\cup U_{i+1}$, and let $P=U_{i} \\cap S$, and $Q=U_{i+1} \\cap S$ (see Figure 3). Since $x$ is separated from $N_{i+1}$, the sets $Q$ and $N_{i+1}$ are disjoint. So, if we re-color $G$ by replacing $U_{i}$ and $U_{i+1}$ by $\\left(U_{i} \\cup Q\\right) \\backslash P$ and $\\left(U_{i+1} \\cup P\\right) \\backslash Q$, respectively, we obtain a proper coloring such that $n\\left(U_{i}\\right)=n\\left(V_{c_{i}}\\right)$ is decreased and only $n\\left(U_{i+1}\\right)=n\\left(V_{c_{i+1}}\\right)$ is increased. That contradicts the lexicographical minimality of $\\left(n\\left(V_{2}\\right), n\\left(V_{3}\\right), \\ldots, n\\left(V_{k}\\right)\\right)$. ![](https:\/\/cdn.mathpix.com\/cropped\/2024_11_18_62dbd531c72d0ff8164cg-41.jpg?height=438&width=851&top_left_y=632&top_left_x=611) Figure 3 Next, we build a path through $U_{1}, U_{2}, \\ldots, U_{2 \\ell}$ as follows. Let the starting point of the path be an arbitrary vertex $v_{1}$ in the set $N_{1}$. For $i \\leqslant 2 \\ell-1$, if the vertex $v_{i} \\in N_{i}$ is already defined, connect $v_{i}$ to some vertex in $N_{i+1}$ in the subgraph induced by $U_{i} \\cup U_{i+1}$, and add these edges to the path. Denote the new endpoint of the path by $v_{i+1}$; by the construction we have $v_{i+1} \\in N_{i+1}$ again, so the process can be continued. At the end we have a path that starts at $v_{1} \\in N_{1}$ and ends at some $v_{2 \\ell} \\in N_{2 \\ell}$. Moreover, all edges in this path connect vertices in neighboring classes: if a vertex of the path lies in $U_{i}$, then the next vertex lies in $U_{i+1}$ or $U_{i-1}$. Notice that the path is not necessary simple, so take a minimal subpath of it. The minimal subpath is simple and connects the same endpoints $v_{1}$ and $v_{2 \\ell}$. The property that every edge steps to a neighboring color class (i.e., from $U_{i}$ to $U_{i+1}$ or $U_{i-1}$ ) is preserved. So the resulting path also visits all of $U_{1}, \\ldots, U_{2 \\ell}$, and its length must be odd. Closing the path with the edges $v v_{1}$ and $v_{2 \\ell} v$ we obtain the desired odd cycle (see Figure 4). ![](https:\/\/cdn.mathpix.com\/cropped\/2024_11_18_62dbd531c72d0ff8164cg-41.jpg?height=518&width=1489&top_left_y=1803&top_left_x=286) Figure 4 Now we prove the claim by induction on $k \\geqslant 3$. The base case $k=3$ holds by applying Lemma 2 to a critical subgraph. For the induction step, let $G_{0}$ be a critical $k$-chromatic subgraph of $G$, and let $v$ be an arbitrary vertex of $G_{0}$. By Lemma 2, $G_{0}$ has at least $2^{k-2}-1$ unsociable groups containing $v$. On the other hand, the graph $G_{0} \\backslash\\{v\\}$ has chromatic number $k-1$, so it contains at least $2^{k-2}-(k-1)$ unsociable groups by the induction hypothesis. Altogether, this gives $2^{k-2}-1+2^{k-2}-(k-1)=2^{k-1}-k$ distinct unsociable groups in $G_{0}$ (and thus in $G$ ). Comment 1. The claim we proved is sharp. The complete graph with $k$ vertices has chromatic number $k$ and contains exactly $2^{k-1}-k$ unsociable groups. Comment 2. The proof of Lemma 2 works for odd values of $|C| \\geqslant 3$ as well. Hence, the second solution shows the analogous statement that the number of even sized unsociable groups is at least $2^{k}-1-\\binom{k}{2}$.","tier":0} +{"problem_type":"Geometry","problem_label":"G1","problem":"Let $A B C$ be an acute triangle with orthocenter $H$. Let $G$ be the point such that the quadrilateral $A B G H$ is a parallelogram. Let $I$ be the point on the line $G H$ such that $A C$ bisects $H I$. Suppose that the line $A C$ intersects the circumcircle of the triangle $G C I$ at $C$ and $J$. Prove that $I J=A H$. (Australia)","solution":"Since $H G \\| A B$ and $B G \\| A H$, we have $B G \\perp B C$ and $C H \\perp G H$. Therefore, the quadrilateral $B G C H$ is cyclic. Since $H$ is the orthocenter of the triangle $A B C$, we have $\\angle H A C=90^{\\circ}-\\angle A C B=\\angle C B H$. Using that $B G C H$ and $C G J I$ are cyclic quadrilaterals, we get $$ \\angle C J I=\\angle C G H=\\angle C B H=\\angle H A C . $$ Let $M$ be the intersection of $A C$ and $G H$, and let $D \\neq A$ be the point on the line $A C$ such that $A H=H D$. Then $\\angle M J I=\\angle H A C=\\angle M D H$. Since $\\angle M J I=\\angle M D H, \\angle I M J=\\angle H M D$, and $I M=M H$, the triangles $I M J$ and $H M D$ are congruent, and thus $I J=H D=A H$. ![](https:\/\/cdn.mathpix.com\/cropped\/2024_11_18_62dbd531c72d0ff8164cg-44.jpg?height=863&width=901&top_left_y=1062&top_left_x=583) Comment. Instead of introducing the point $D$, one can complete the solution by using the law of sines in the triangles $I J M$ and $A M H$, yielding $$ \\frac{I J}{I M}=\\frac{\\sin \\angle I M J}{\\sin \\angle M J I}=\\frac{\\sin \\angle A M H}{\\sin \\angle H A M}=\\frac{A H}{M H}=\\frac{A H}{I M} . $$","tier":0} +{"problem_type":"Geometry","problem_label":"G1","problem":"Let $A B C$ be an acute triangle with orthocenter $H$. Let $G$ be the point such that the quadrilateral $A B G H$ is a parallelogram. Let $I$ be the point on the line $G H$ such that $A C$ bisects $H I$. Suppose that the line $A C$ intersects the circumcircle of the triangle $G C I$ at $C$ and $J$. Prove that $I J=A H$. (Australia)","solution":"Obtain $\\angle C G H=\\angle H A C$ as in the previous solution. In the parallelogram $A B G H$ we have $\\angle B A H=\\angle H G B$. It follows that $$ \\angle H M C=\\angle B A C=\\angle B A H+\\angle H A C=\\angle H G B+\\angle C G H=\\angle C G B . $$ So the right triangles $C M H$ and $C G B$ are similar. Also, in the circumcircle of triangle $G C I$ we have similar triangles $M I J$ and $M C G$. Therefore, $$ \\frac{I J}{C G}=\\frac{M I}{M C}=\\frac{M H}{M C}=\\frac{G B}{G C}=\\frac{A H}{C G} $$ Hence $I J=A H$.","tier":0} +{"problem_type":"Geometry","problem_label":"G2","problem":"Let $A B C$ be a triangle inscribed into a circle $\\Omega$ with center $O$. A circle $\\Gamma$ with center $A$ meets the side $B C$ at points $D$ and $E$ such that $D$ lies between $B$ and $E$. Moreover, let $F$ and $G$ be the common points of $\\Gamma$ and $\\Omega$. We assume that $F$ lies on the arc $A B$ of $\\Omega$ not containing $C$, and $G$ lies on the arc $A C$ of $\\Omega$ not containing $B$. The circumcircles of the triangles $B D F$ and $C E G$ meet the sides $A B$ and $A C$ again at $K$ and $L$, respectively. Suppose that the lines $F K$ and $G L$ are distinct and intersect at $X$. Prove that the points $A, X$, and $O$ are collinear. (Greece)","solution":"It suffices to prove that the lines $F K$ and $G L$ are symmetric about $A O$. Now the segments $A F$ and $A G$, being chords of $\\Omega$ with the same length, are clearly symmetric with respect to $A O$. Hence it is enough to show $$ \\angle K F A=\\angle A G L . $$ Let us denote the circumcircles of $B D F$ and $C E G$ by $\\omega_{B}$ and $\\omega_{C}$, respectively. To prove (1), we start from $$ \\angle K F A=\\angle D F G+\\angle G F A-\\angle D F K $$ In view of the circles $\\omega_{B}, \\Gamma$, and $\\Omega$, this may be rewritten as $$ \\angle K F A=\\angle C E G+\\angle G B A-\\angle D B K=\\angle C E G-\\angle C B G . $$ Due to the circles $\\omega_{C}$ and $\\Omega$, we obtain $\\angle K F A=\\angle C L G-\\angle C A G=\\angle A G L$. Thereby the problem is solved. ![](https:\/\/cdn.mathpix.com\/cropped\/2024_11_18_62dbd531c72d0ff8164cg-45.jpg?height=729&width=1220&top_left_y=1389&top_left_x=418) Figure 1","tier":0} +{"problem_type":"Geometry","problem_label":"G2","problem":"Let $A B C$ be a triangle inscribed into a circle $\\Omega$ with center $O$. A circle $\\Gamma$ with center $A$ meets the side $B C$ at points $D$ and $E$ such that $D$ lies between $B$ and $E$. Moreover, let $F$ and $G$ be the common points of $\\Gamma$ and $\\Omega$. We assume that $F$ lies on the arc $A B$ of $\\Omega$ not containing $C$, and $G$ lies on the arc $A C$ of $\\Omega$ not containing $B$. The circumcircles of the triangles $B D F$ and $C E G$ meet the sides $A B$ and $A C$ again at $K$ and $L$, respectively. Suppose that the lines $F K$ and $G L$ are distinct and intersect at $X$. Prove that the points $A, X$, and $O$ are collinear. (Greece)","solution":"Again, we denote the circumcircle of $B D K F$ by $\\omega_{B}$. In addition, we set $\\alpha=$ $\\angle B A C, \\varphi=\\angle A B F$, and $\\psi=\\angle E D A=\\angle A E D$ (see Figure 2). Notice that $A F=A G$ entails $\\varphi=\\angle G C A$, so all three of $\\alpha, \\varphi$, and $\\psi$ respect the \"symmetry\" between $B$ and $C$ of our configuration. Again, we reduce our task to proving (1). This time, we start from $$ 2 \\angle K F A=2(\\angle D F A-\\angle D F K) . $$ Since the triangle $A F D$ is isosceles, we have $$ \\angle D F A=\\angle A D F=\\angle E D F-\\psi=\\angle B F D+\\angle E B F-\\psi . $$ Moreover, because of the circle $\\omega_{B}$ we have $\\angle D F K=\\angle C B A$. Altogether, this yields $$ 2 \\angle K F A=\\angle D F A+(\\angle B F D+\\angle E B F-\\psi)-2 \\angle C B A, $$ which simplifies to $$ 2 \\angle K F A=\\angle B F A+\\varphi-\\psi-\\angle C B A . $$ Now the quadrilateral $A F B C$ is cyclic, so this entails $2 \\angle K F A=\\alpha+\\varphi-\\psi$. Due to the \"symmetry\" between $B$ and $C$ alluded to above, this argument also shows that $2 \\angle A G L=\\alpha+\\varphi-\\psi$. This concludes the proof of (1). ![](https:\/\/cdn.mathpix.com\/cropped\/2024_11_18_62dbd531c72d0ff8164cg-46.jpg?height=547&width=926&top_left_y=726&top_left_x=571) Figure 2 Comment 1. As the first solution shows, the assumption that $A$ be the center of $\\Gamma$ may be weakened to the following one: The center of $\\Gamma$ lies on the line $O A$. The second solution may be modified to yield the same result. Comment 2. It might be interesting to remark that $\\angle G D K=90^{\\circ}$. To prove this, let $G^{\\prime}$ denote the point on $\\Gamma$ diametrically opposite to $G$. Because of $\\angle K D F=\\angle K B F=\\angle A G F=\\angle G^{\\prime} D F$, the points $D, K$, and $G^{\\prime}$ are collinear, which leads to the desired result. Notice that due to symmetry we also have $\\angle L E F=90^{\\circ}$. Moreover, a standard argument shows that the triangles $A G L$ and $B G E$ are similar. By symmetry again, also the triangles $A F K$ and $C D F$ are similar. There are several ways to derive a solution from these facts. For instance, one may argue that $$ \\begin{aligned} \\angle K F A & =\\angle B F A-\\angle B F K=\\angle B F A-\\angle E D G^{\\prime}=\\left(180^{\\circ}-\\angle A G B\\right)-\\left(180^{\\circ}-\\angle G^{\\prime} G E\\right) \\\\ & =\\angle A G E-\\angle A G B=\\angle B G E=\\angle A G L . \\end{aligned} $$ Comment 3. The original proposal did not contain the point $X$ in the assumption and asked instead to prove that the lines $F K, G L$, and $A O$ are concurrent. This differs from the version given above only insofar as it also requires to show that these lines cannot be parallel. The Problem Selection Committee removed this part from the problem intending to make it thus more suitable for the Olympiad. For the sake of completeness, we would still like to sketch one possibility for proving $F K \\nVdash A O$ here. As the points $K$ and $O$ lie in the angular region $\\angle F A G$, it suffices to check $\\angle K F A+\\angle F A O<180^{\\circ}$. Multiplying by 2 and making use of the formulae from the second solution, we see that this is equivalent to $(\\alpha+\\varphi-\\psi)+\\left(180^{\\circ}-2 \\varphi\\right)<360^{\\circ}$, which in turn is an easy consequence of $\\alpha<180^{\\circ}$.","tier":0} +{"problem_type":"Geometry","problem_label":"G3","problem":"Let $A B C$ be a triangle with $\\angle C=90^{\\circ}$, and let $H$ be the foot of the altitude from $C$. A point $D$ is chosen inside the triangle $C B H$ so that $C H$ bisects $A D$. Let $P$ be the intersection point of the lines $B D$ and $C H$. Let $\\omega$ be the semicircle with diameter $B D$ that meets the segment $C B$ at an interior point. A line through $P$ is tangent to $\\omega$ at $Q$. Prove that the lines $C Q$ and $A D$ meet on $\\omega$. (Georgia)","solution":"Let $K$ be the projection of $D$ onto $A B$; then $A H=H K$ (see Figure 1). Since $P H \\| D K$, we have $$ \\frac{P D}{P B}=\\frac{H K}{H B}=\\frac{A H}{H B} $$ Let $L$ be the projection of $Q$ onto $D B$. Since $P Q$ is tangent to $\\omega$ and $\\angle D Q B=\\angle B L Q=$ $90^{\\circ}$, we have $\\angle P Q D=\\angle Q B P=\\angle D Q L$. Therefore, $Q D$ and $Q B$ are respectively the internal and the external bisectors of $\\angle P Q L$. By the angle bisector theorem, we obtain $$ \\frac{P D}{D L}=\\frac{P Q}{Q L}=\\frac{P B}{B L} $$ The relations (1) and (2) yield $\\frac{A H}{H B}=\\frac{P D}{P B}=\\frac{D L}{L B}$. So, the spiral similarity $\\tau$ centered at $B$ and sending $A$ to $D$ maps $H$ to $L$. Moreover, $\\tau$ sends the semicircle with diameter $A B$ passing through $C$ to $\\omega$. Due to $C H \\perp A B$ and $Q L \\perp D B$, it follows that $\\tau(C)=Q$. Hence, the triangles $A B D$ and $C B Q$ are similar, so $\\angle A D B=\\angle C Q B$. This means that the lines $A D$ and $C Q$ meet at some point $T$, and this point satisfies $\\angle B D T=\\angle B Q T$. Therefore, $T$ lies on $\\omega$, as needed. ![](https:\/\/cdn.mathpix.com\/cropped\/2024_11_18_62dbd531c72d0ff8164cg-47.jpg?height=438&width=801&top_left_y=1397&top_left_x=225) Figure 1 ![](https:\/\/cdn.mathpix.com\/cropped\/2024_11_18_62dbd531c72d0ff8164cg-47.jpg?height=464&width=798&top_left_y=1367&top_left_x=1046) Figure 2 Comment 1. Since $\\angle B A D=\\angle B C Q$, the point $T$ lies also on the circumcircle of the triangle $A B C$.","tier":0} +{"problem_type":"Geometry","problem_label":"G3","problem":"Let $A B C$ be a triangle with $\\angle C=90^{\\circ}$, and let $H$ be the foot of the altitude from $C$. A point $D$ is chosen inside the triangle $C B H$ so that $C H$ bisects $A D$. Let $P$ be the intersection point of the lines $B D$ and $C H$. Let $\\omega$ be the semicircle with diameter $B D$ that meets the segment $C B$ at an interior point. A line through $P$ is tangent to $\\omega$ at $Q$. Prove that the lines $C Q$ and $A D$ meet on $\\omega$. (Georgia)","solution":"Let $\\Gamma$ be the circumcircle of $A B C$, and let $A D$ meet $\\omega$ at $T$. Then $\\angle A T B=$ $\\angle A C B=90^{\\circ}$, so $T$ lies on $\\Gamma$ as well. As in the previous solution, let $K$ be the projection of $D$ onto $A B$; then $A H=H K$ (see Figure 2). Our goal now is to prove that the points $C, Q$, and $T$ are collinear. Let $C T$ meet $\\omega$ again at $Q^{\\prime}$. Then, it suffices to show that $P Q^{\\prime}$ is tangent to $\\omega$, or that $\\angle P Q^{\\prime} D=\\angle Q^{\\prime} B D$. Since the quadrilateral $B D Q^{\\prime} T$ is cyclic and the triangles $A H C$ and $K H C$ are congruent, we have $\\angle Q^{\\prime} B D=\\angle Q^{\\prime} T D=\\angle C T A=\\angle C B A=\\angle A C H=\\angle H C K$. Hence, the right triangles $C H K$ and $B Q^{\\prime} D$ are similar. This implies that $\\frac{H K}{C K}=\\frac{Q^{\\prime} D}{B D}$, and thus $H K \\cdot B D=C K \\cdot Q^{\\prime} D$. Notice that $P H \\| D K$; therefore, we have $\\frac{P D}{B D}=\\frac{H K}{B K}$, and so $P D \\cdot B K=H K \\cdot B D$. Consequently, $P D \\cdot B K=H K \\cdot B D=C K \\cdot Q^{\\prime} D$, which yields $\\frac{P D}{Q^{\\prime} D}=\\frac{C K}{B K}$. Since $\\angle C K A=\\angle K A C=\\angle B D Q^{\\prime}$, the triangles $C K B$ and $P D Q^{\\prime}$ are similar, so $\\angle P Q^{\\prime} D=$ $\\angle C B A=\\angle Q^{\\prime} B D$, as required. Comment 2. There exist several other ways to prove that $P Q^{\\prime}$ is tangent to $\\omega$. For instance, one may compute $\\frac{P D}{P B}$ and $\\frac{P Q^{\\prime}}{P B}$ in terms of $A H$ and $H B$ to verify that $P Q^{\\prime 2}=P D \\cdot P B$, concluding that $P Q^{\\prime}$ is tangent to $\\omega$. Another possible approach is the following. As in Solution 2, we introduce the points $T$ and $Q^{\\prime}$ and mention that the triangles $A B C$ and $D B Q^{\\prime}$ are similar (see Figure 3). Let $M$ be the midpoint of $A D$, and let $L$ be the projection of $Q^{\\prime}$ onto $A B$. Construct $E$ on the line $A B$ so that $E P$ is parallel to $A D$. Projecting from $P$, we get $(A, B ; H, E)=(A, D ; M, \\infty)=-1$. Since $\\frac{E A}{A B}=\\frac{P D}{D B}$, the point $P$ is the image of $E$ under the similarity transform mapping $A B C$ to $D B Q^{\\prime}$. Therefore, we have $(D, B ; L, P)=(A, B ; H, E)=-1$, which means that $Q^{\\prime} D$ and $Q^{\\prime} B$ are respectively the internal and the external bisectors of $\\angle P Q^{\\prime} L$. This implies that $P Q^{\\prime}$ is tangent to $\\omega$, as required. ![](https:\/\/cdn.mathpix.com\/cropped\/2024_11_18_62dbd531c72d0ff8164cg-48.jpg?height=415&width=1026&top_left_y=820&top_left_x=521) Figure 3","tier":0} +{"problem_type":"Geometry","problem_label":"G3","problem":"Let $A B C$ be a triangle with $\\angle C=90^{\\circ}$, and let $H$ be the foot of the altitude from $C$. A point $D$ is chosen inside the triangle $C B H$ so that $C H$ bisects $A D$. Let $P$ be the intersection point of the lines $B D$ and $C H$. Let $\\omega$ be the semicircle with diameter $B D$ that meets the segment $C B$ at an interior point. A line through $P$ is tangent to $\\omega$ at $Q$. Prove that the lines $C Q$ and $A D$ meet on $\\omega$. (Georgia)","solution":"Introduce the points $T$ and $Q^{\\prime}$ as in the previous solution. Note that $T$ lies on the circumcircle of $A B C$. Here we present yet another proof that $P Q^{\\prime}$ is tangent to $\\omega$. Let $\\Omega$ be the circle completing the semicircle $\\omega$. Construct a point $F$ symmetric to $C$ with respect to $A B$. Let $S \\neq T$ be the second intersection point of $F T$ and $\\Omega$ (see Figure 4). ![](https:\/\/cdn.mathpix.com\/cropped\/2024_11_18_62dbd531c72d0ff8164cg-48.jpg?height=772&width=855&top_left_y=1553&top_left_x=606) Figure 4 Since $A C=A F$, we have $\\angle D K C=\\angle H C K=\\angle C B A=\\angle C T A=\\angle D T S=180^{\\circ}-$ $\\angle S K D$. Thus, the points $C, K$, and $S$ are collinear. Notice also that $\\angle Q^{\\prime} K D=\\angle Q^{\\prime} T D=$ $\\angle H C K=\\angle K F H=180^{\\circ}-\\angle D K F$. This implies that the points $F, K$, and $Q^{\\prime}$ are collinear. Applying PASCAL's theorem to the degenerate hexagon $K Q^{\\prime} Q^{\\prime} T S S$, we get that the tangents to $\\Omega$ passing through $Q^{\\prime}$ and $S$ intersect on $C F$. The relation $\\angle Q^{\\prime} T D=\\angle D T S$ yields that $Q^{\\prime}$ and $S$ are symmetric with respect to $B D$. Therefore, the two tangents also intersect on $B D$. Thus, the two tangents pass through $P$. Hence, $P Q^{\\prime}$ is tangent to $\\omega$, as needed.","tier":0} +{"problem_type":"Geometry","problem_label":"G4","problem":"Let $A B C$ be an acute triangle, and let $M$ be the midpoint of $A C$. A circle $\\omega$ passing through $B$ and $M$ meets the sides $A B$ and $B C$ again at $P$ and $Q$, respectively. Let $T$ be the point such that the quadrilateral $B P T Q$ is a parallelogram. Suppose that $T$ lies on the circumcircle of the triangle $A B C$. Determine all possible values of $B T \/ B M$. (Russia)","solution":"Let $S$ be the center of the parallelogram $B P T Q$, and let $B^{\\prime} \\neq B$ be the point on the ray $B M$ such that $B M=M B^{\\prime}$ (see Figure 1). It follows that $A B C B^{\\prime}$ is a parallelogram. Then, $\\angle A B B^{\\prime}=\\angle P Q M$ and $\\angle B B^{\\prime} A=\\angle B^{\\prime} B C=\\angle M P Q$, and so the triangles $A B B^{\\prime}$ and $M Q P$ are similar. It follows that $A M$ and $M S$ are corresponding medians in these triangles. Hence, $$ \\angle S M P=\\angle B^{\\prime} A M=\\angle B C A=\\angle B T A . $$ Since $\\angle A C T=\\angle P B T$ and $\\angle T A C=\\angle T B C=\\angle B T P$, the triangles $T C A$ and $P B T$ are similar. Again, as $T M$ and $P S$ are corresponding medians in these triangles, we have $$ \\angle M T A=\\angle T P S=\\angle B Q P=\\angle B M P . $$ Now we deal separately with two cases. Case 1. $\\quad S$ does not lie on $B M$. Since the configuration is symmetric between $A$ and $C$, we may assume that $S$ and $A$ lie on the same side with respect to the line $B M$. Applying (1) and (2), we get $$ \\angle B M S=\\angle B M P-\\angle S M P=\\angle M T A-\\angle B T A=\\angle M T B, $$ and so the triangles $B S M$ and $B M T$ are similar. We now have $B M^{2}=B S \\cdot B T=B T^{2} \/ 2$, so $B T=\\sqrt{2} B M$. Case 2. $\\quad S$ lies on $B M$. It follows from (2) that $\\angle B C A=\\angle M T A=\\angle B Q P=\\angle B M P$ (see Figure 2). Thus, $P Q \\| A C$ and $P M \\| A T$. Hence, $B S \/ B M=B P \/ B A=B M \/ B T$, so $B T^{2}=2 B M^{2}$ and $B T=\\sqrt{2} B M$. ![](https:\/\/cdn.mathpix.com\/cropped\/2024_11_18_62dbd531c72d0ff8164cg-49.jpg?height=960&width=730&top_left_y=1713&top_left_x=246) Figure 1 ![](https:\/\/cdn.mathpix.com\/cropped\/2024_11_18_62dbd531c72d0ff8164cg-49.jpg?height=749&width=729&top_left_y=1938&top_left_x=1092) Figure 2 Comment 1. Here is another way to show that the triangles $B S M$ and $B M T$ are similar. Denote by $\\Omega$ the circumcircle of the triangle $A B C$. Let $R$ be the second point of intersection of $\\omega$ and $\\Omega$, and let $\\tau$ be the spiral similarity centered at $R$ mapping $\\omega$ to $\\Omega$. Then, one may show that $\\tau$ maps each point $X$ on $\\omega$ to a point $Y$ on $\\Omega$ such that $B, X$, and $Y$ are collinear (see Figure 3). If we let $K$ and $L$ be the second points of intersection of $B M$ with $\\Omega$ and of $B T$ with $\\omega$, respectively, then it follows that the triangle $M K T$ is the image of $S M L$ under $\\tau$. We now obtain $\\angle B S M=\\angle T M B$, which implies the desired result. ![](https:\/\/cdn.mathpix.com\/cropped\/2024_11_18_62dbd531c72d0ff8164cg-50.jpg?height=758&width=757&top_left_y=752&top_left_x=204) Figure 3 ![](https:\/\/cdn.mathpix.com\/cropped\/2024_11_18_62dbd531c72d0ff8164cg-50.jpg?height=954&width=846&top_left_y=551&top_left_x=1002) Figure 4","tier":0} +{"problem_type":"Geometry","problem_label":"G4","problem":"Let $A B C$ be an acute triangle, and let $M$ be the midpoint of $A C$. A circle $\\omega$ passing through $B$ and $M$ meets the sides $A B$ and $B C$ again at $P$ and $Q$, respectively. Let $T$ be the point such that the quadrilateral $B P T Q$ is a parallelogram. Suppose that $T$ lies on the circumcircle of the triangle $A B C$. Determine all possible values of $B T \/ B M$. (Russia)","solution":"Again, we denote by $\\Omega$ the circumcircle of the triangle $A B C$. Choose the points $X$ and $Y$ on the rays $B A$ and $B C$ respectively, so that $\\angle M X B=\\angle M B C$ and $\\angle B Y M=\\angle A B M$ (see Figure 4). Then the triangles $B M X$ and $Y M B$ are similar. Since $\\angle X P M=\\angle B Q M$, the points $P$ and $Q$ correspond to each other in these triangles. So, if $\\overrightarrow{B P}=\\mu \\cdot \\overrightarrow{B X}$, then $\\overrightarrow{B Q}=(1-\\mu) \\cdot \\overrightarrow{B Y}$. Thus $$ \\overrightarrow{B T}=\\overrightarrow{B P}+\\overrightarrow{B Q}=\\overrightarrow{B Y}+\\mu \\cdot(\\overrightarrow{B X}-\\overrightarrow{B Y})=\\overrightarrow{B Y}+\\mu \\cdot \\overrightarrow{Y X} $$ which means that $T$ lies on the line $X Y$. Let $B^{\\prime} \\neq B$ be the point on the ray $B M$ such that $B M=M B^{\\prime}$. Then $\\angle M B^{\\prime} A=$ $\\angle M B C=\\angle M X B$ and $\\angle C B^{\\prime} M=\\angle A B M=\\angle B Y M$. This means that the triangles $B M X$, $B A B^{\\prime}, Y M B$, and $B^{\\prime} C B$ are all similar; hence $B A \\cdot B X=B M \\cdot B B^{\\prime}=B C \\cdot B Y$. Thus there exists an inversion centered at $B$ which swaps $A$ with $X, M$ with $B^{\\prime}$, and $C$ with $Y$. This inversion then swaps $\\Omega$ with the line $X Y$, and hence it preserves $T$. Therefore, we have $B T^{2}=B M \\cdot B B^{\\prime}=2 B M^{2}$, and $B T=\\sqrt{2} B M$.","tier":0} +{"problem_type":"Geometry","problem_label":"G4","problem":"Let $A B C$ be an acute triangle, and let $M$ be the midpoint of $A C$. A circle $\\omega$ passing through $B$ and $M$ meets the sides $A B$ and $B C$ again at $P$ and $Q$, respectively. Let $T$ be the point such that the quadrilateral $B P T Q$ is a parallelogram. Suppose that $T$ lies on the circumcircle of the triangle $A B C$. Determine all possible values of $B T \/ B M$. (Russia)","solution":"We begin with the following lemma. Lemma. Let $A B C T$ be a cyclic quadrilateral. Let $P$ and $Q$ be points on the sides $B A$ and $B C$ respectively, such that $B P T Q$ is a parallelogram. Then $B P \\cdot B A+B Q \\cdot B C=B T^{2}$. Proof. Let the circumcircle of the triangle $Q T C$ meet the line $B T$ again at $J$ (see Figure 5). The power of $B$ with respect to this circle yields $$ B Q \\cdot B C=B J \\cdot B T $$ We also have $\\angle T J Q=180^{\\circ}-\\angle Q C T=\\angle T A B$ and $\\angle Q T J=\\angle A B T$, and so the triangles $T J Q$ and $B A T$ are similar. We now have $T J \/ T Q=B A \/ B T$. Therefore, $$ T J \\cdot B T=T Q \\cdot B A=B P \\cdot B A $$ Combining (3) and (4) now yields the desired result. Let $X$ and $Y$ be the midpoints of $B A$ and $B C$ respectively (see Figure 6). Applying the lemma to the cyclic quadrilaterals $P B Q M$ and $A B C T$, we obtain $$ B X \\cdot B P+B Y \\cdot B Q=B M^{2} $$ and $$ B P \\cdot B A+B Q \\cdot B C=B T^{2} $$ Since $B A=2 B X$ and $B C=2 B Y$, we have $B T^{2}=2 B M^{2}$, and so $B T=\\sqrt{2} B M$. ![](https:\/\/cdn.mathpix.com\/cropped\/2024_11_18_62dbd531c72d0ff8164cg-51.jpg?height=789&width=1464&top_left_y=933&top_left_x=296) Comment 2. Here we give another proof of the lemma using Ptolemy's theorem. We readily have $$ T C \\cdot B A+T A \\cdot B C=A C \\cdot B T $$ The lemma now follows from $$ \\frac{B P}{T C}=\\frac{B Q}{T A}=\\frac{B T}{A C}=\\frac{\\sin \\angle B C T}{\\sin \\angle A B C} $$","tier":0} +{"problem_type":"Geometry","problem_label":"G5","problem":"Let $A B C$ be a triangle with $C A \\neq C B$. Let $D, F$, and $G$ be the midpoints of the sides $A B, A C$, and $B C$, respectively. A circle $\\Gamma$ passing through $C$ and tangent to $A B$ at $D$ meets the segments $A F$ and $B G$ at $H$ and $I$, respectively. The points $H^{\\prime}$ and $I^{\\prime}$ are symmetric to $H$ and $I$ about $F$ and $G$, respectively. The line $H^{\\prime} I^{\\prime}$ meets $C D$ and $F G$ at $Q$ and $M$, respectively. The line $C M$ meets $\\Gamma$ again at $P$. Prove that $C Q=Q P$. (El Salvador)","solution":"We may assume that $C A>C B$. Observe that $H^{\\prime}$ and $I^{\\prime}$ lie inside the segments $C F$ and $C G$, respectively. Therefore, $M$ lies outside $\\triangle A B C$ (see Figure 1). Due to the powers of points $A$ and $B$ with respect to the circle $\\Gamma$, we have $$ C H^{\\prime} \\cdot C A=A H \\cdot A C=A D^{2}=B D^{2}=B I \\cdot B C=C I^{\\prime} \\cdot C B $$ Therefore, $C H^{\\prime} \\cdot C F=C I^{\\prime} \\cdot C G$. Hence, the quadrilateral $H^{\\prime} I^{\\prime} G F$ is cyclic, and so $\\angle I^{\\prime} H^{\\prime} C=$ $\\angle C G F$. Let $D F$ and $D G$ meet $\\Gamma$ again at $R$ and $S$, respectively. We claim that the points $R$ and $S$ lie on the line $H^{\\prime} I^{\\prime}$. Observe that $F H^{\\prime} \\cdot F A=F H \\cdot F C=F R \\cdot F D$. Thus, the quadrilateral $A D H^{\\prime} R$ is cyclic, and hence $\\angle R H^{\\prime} F=\\angle F D A=\\angle C G F=\\angle I^{\\prime} H^{\\prime} C$. Therefore, the points $R, H^{\\prime}$, and $I^{\\prime}$ are collinear. Similarly, the points $S, H^{\\prime}$, and $I^{\\prime}$ are also collinear, and so all the points $R, H^{\\prime}, Q, I^{\\prime}, S$, and $M$ are all collinear. ![](https:\/\/cdn.mathpix.com\/cropped\/2024_11_18_62dbd531c72d0ff8164cg-52.jpg?height=689&width=803&top_left_y=1249&top_left_x=204) Figure 1 ![](https:\/\/cdn.mathpix.com\/cropped\/2024_11_18_62dbd531c72d0ff8164cg-52.jpg?height=684&width=826&top_left_y=1257&top_left_x=1023) Figure 2 Then, $\\angle R S D=\\angle R D A=\\angle D F G$. Hence, the quadrilateral $R S G F$ is cyclic (see Figure 2). Therefore, $M H^{\\prime} \\cdot M I^{\\prime}=M F \\cdot M G=M R \\cdot M S=M P \\cdot M C$. Thus, the quadrilateral $C P I^{\\prime} H^{\\prime}$ is also cyclic. Let $\\omega$ be its circumcircle. Notice that $\\angle H^{\\prime} C Q=\\angle S D C=\\angle S R C$ and $\\angle Q C I^{\\prime}=\\angle C D R=\\angle C S R$. Hence, $\\triangle C H^{\\prime} Q \\sim \\triangle R C Q$ and $\\triangle C I^{\\prime} Q \\sim \\triangle S C Q$, and therefore $Q H^{\\prime} \\cdot Q R=Q C^{2}=Q I^{\\prime} \\cdot Q S$. We apply the inversion with center $Q$ and radius $Q C$. Observe that the points $R, C$, and $S$ are mapped to $H^{\\prime}, C$, and $I^{\\prime}$, respectively. Therefore, the circumcircle $\\Gamma$ of $\\triangle R C S$ is mapped to the circumcircle $\\omega$ of $\\triangle H^{\\prime} C I^{\\prime}$. Since $P$ and $C$ belong to both circles and the point $C$ is preserved by the inversion, we have that $P$ is also mapped to itself. We then get $Q P^{2}=Q C^{2}$. Hence, $Q P=Q C$. Comment 1. The problem statement still holds when $\\Gamma$ intersects the sides $C A$ and $C B$ outside segments $A F$ and $B G$, respectively.","tier":0} +{"problem_type":"Geometry","problem_label":"G5","problem":"Let $A B C$ be a triangle with $C A \\neq C B$. Let $D, F$, and $G$ be the midpoints of the sides $A B, A C$, and $B C$, respectively. A circle $\\Gamma$ passing through $C$ and tangent to $A B$ at $D$ meets the segments $A F$ and $B G$ at $H$ and $I$, respectively. The points $H^{\\prime}$ and $I^{\\prime}$ are symmetric to $H$ and $I$ about $F$ and $G$, respectively. The line $H^{\\prime} I^{\\prime}$ meets $C D$ and $F G$ at $Q$ and $M$, respectively. The line $C M$ meets $\\Gamma$ again at $P$. Prove that $C Q=Q P$. (El Salvador)","solution":"Let $X=H I \\cap A B$, and let the tangent to $\\Gamma$ at $C$ meet $A B$ at $Y$. Let $X C$ meet $\\Gamma$ again at $X^{\\prime}$ (see Figure 3). Projecting from $C, X$, and $C$ again, we have $(X, A ; D, B)=$ $\\left(X^{\\prime}, H ; D, I\\right)=(C, I ; D, H)=(Y, B ; D, A)$. Since $A$ and $B$ are symmetric about $D$, it follows that $X$ and $Y$ are also symmetric about $D$. Now, Menelaus' theorem applied to $\\triangle A B C$ with the line $H I X$ yields $$ 1=\\frac{C H}{H A} \\cdot \\frac{B I}{I C} \\cdot \\frac{A X}{X B}=\\frac{A H^{\\prime}}{H^{\\prime} C} \\cdot \\frac{C I^{\\prime}}{I^{\\prime} B} \\cdot \\frac{B Y}{Y A} . $$ By the converse of Menelaus' theorem applied to $\\triangle A B C$ with points $H^{\\prime}, I^{\\prime}, Y$, we get that the points $H^{\\prime}, I^{\\prime}, Y$ are collinear. ![](https:\/\/cdn.mathpix.com\/cropped\/2024_11_18_62dbd531c72d0ff8164cg-53.jpg?height=568&width=1382&top_left_y=744&top_left_x=340) Figure 3 Let $T$ be the midpoint of $C D$, and let $O$ be the center of $\\Gamma$. Let $C M$ meet $T Y$ at $N$. To avoid confusion, we clean some superfluous details out of the picture (see Figure 4). Let $V=M T \\cap C Y$. Since $M T \\| Y D$ and $D T=T C$, we get $C V=V Y$. Then CevA's theorem applied to $\\triangle C T Y$ with the point $M$ yields $$ 1=\\frac{T Q}{Q C} \\cdot \\frac{C V}{V Y} \\cdot \\frac{Y N}{N T}=\\frac{T Q}{Q C} \\cdot \\frac{Y N}{N T} $$ Therefore, $\\frac{T Q}{Q C}=\\frac{T N}{N Y}$. So, $N Q \\| C Y$, and thus $N Q \\perp O C$. Note that the points $O, N, T$, and $Y$ are collinear. Therefore, $C Q \\perp O N$. So, $Q$ is the orthocenter of $\\triangle O C N$, and hence $O Q \\perp C P$. Thus, $Q$ lies on the perpendicular bisector of $C P$, and therefore $C Q=Q P$, as required. ![](https:\/\/cdn.mathpix.com\/cropped\/2024_11_18_62dbd531c72d0ff8164cg-53.jpg?height=652&width=866&top_left_y=2027&top_left_x=595) Figure 4 Comment 2. The second part of Solution 2 provides a proof of the following more general statement, which does not involve a specific choice of $Q$ on $C D$. Let $Y C$ and $Y D$ be two tangents to a circle $\\Gamma$ with center $O$ (see Figure 4). Let $\\ell$ be the midline of $\\triangle Y C D$ parallel to $Y D$. Let $Q$ and $M$ be two points on $C D$ and $\\ell$, respectively, such that the line $Q M$ passes through $Y$. Then $O Q \\perp C M$.","tier":0} +{"problem_type":"Geometry","problem_label":"G6","problem":"Let $A B C$ be an acute triangle with $A B>A C$, and let $\\Gamma$ be its circumcircle. Let $H$, $M$, and $F$ be the orthocenter of the triangle, the midpoint of $B C$, and the foot of the altitude from $A$, respectively. Let $Q$ and $K$ be the two points on $\\Gamma$ that satisfy $\\angle A Q H=90^{\\circ}$ and $\\angle Q K H=90^{\\circ}$. Prove that the circumcircles of the triangles $K Q H$ and $K F M$ are tangent to each other. (Ukraine)","solution":"Let $A^{\\prime}$ be the point diametrically opposite to $A$ on $\\Gamma$. Since $\\angle A Q A^{\\prime}=90^{\\circ}$ and $\\angle A Q H=90^{\\circ}$, the points $Q, H$, and $A^{\\prime}$ are collinear. Similarly, if $Q^{\\prime}$ denotes the point on $\\Gamma$ diametrically opposite to $Q$, then $K, H$, and $Q^{\\prime}$ are collinear. Let the line $A H F$ intersect $\\Gamma$ again at $E$; it is known that $M$ is the midpoint of the segment $H A^{\\prime}$ and that $F$ is the midpoint of $H E$. Let $J$ be the midpoint of $H Q^{\\prime}$. Consider any point $T$ such that $T K$ is tangent to the circle $K Q H$ at $K$ with $Q$ and $T$ lying on different sides of $K H$ (see Figure 1). Then $\\angle H K T=\\angle H Q K$ and we are to prove that $\\angle M K T=\\angle C F K$. Thus it remains to show that $\\angle H Q K=\\angle C F K+\\angle H K M$. Due to $\\angle H Q K=90^{\\circ}-\\angle Q^{\\prime} H A^{\\prime}$ and $\\angle C F K=90^{\\circ}-\\angle K F A$, this means the same as $\\angle Q^{\\prime} H A^{\\prime}=$ $\\angle K F A-\\angle H K M$. Now, since the triangles $K H E$ and $A H Q^{\\prime}$ are similar with $F$ and $J$ being the midpoints of corresponding sides, we have $\\angle K F A=\\angle H J A$, and analogously one may obtain $\\angle H K M=\\angle J Q H$. Thereby our task is reduced to verifying $$ \\angle Q^{\\prime} H A^{\\prime}=\\angle H J A-\\angle J Q H $$ ![](https:\/\/cdn.mathpix.com\/cropped\/2024_11_18_62dbd531c72d0ff8164cg-55.jpg?height=798&width=758&top_left_y=1257&top_left_x=289) Figure 1 ![](https:\/\/cdn.mathpix.com\/cropped\/2024_11_18_62dbd531c72d0ff8164cg-55.jpg?height=710&width=672&top_left_y=1344&top_left_x=1092) Figure 2 To avoid confusion, let us draw a new picture at this moment (see Figure 2). Owing to $\\angle Q^{\\prime} H A^{\\prime}=\\angle J Q H+\\angle H J Q$ and $\\angle H J A=\\angle Q J A+\\angle H J Q$, we just have to show that $2 \\angle J Q H=\\angle Q J A$. To this end, it suffices to remark that $A Q A^{\\prime} Q^{\\prime}$ is a rectangle and that $J$, being defined to be the midpoint of $H Q^{\\prime}$, has to lie on the mid parallel of $Q A^{\\prime}$ and $Q^{\\prime} A$.","tier":0} +{"problem_type":"Geometry","problem_label":"G6","problem":"Let $A B C$ be an acute triangle with $A B>A C$, and let $\\Gamma$ be its circumcircle. Let $H$, $M$, and $F$ be the orthocenter of the triangle, the midpoint of $B C$, and the foot of the altitude from $A$, respectively. Let $Q$ and $K$ be the two points on $\\Gamma$ that satisfy $\\angle A Q H=90^{\\circ}$ and $\\angle Q K H=90^{\\circ}$. Prove that the circumcircles of the triangles $K Q H$ and $K F M$ are tangent to each other. (Ukraine)","solution":"We define the points $A^{\\prime}$ and $E$ and prove that the ray $M H$ passes through $Q$ in the same way as in the first solution. Notice that the points $A^{\\prime}$ and $E$ can play analogous roles to the points $Q$ and $K$, respectively: point $A^{\\prime}$ is the second intersection of the line $M H$ with $\\Gamma$, and $E$ is the point on $\\Gamma$ with the property $\\angle H E A^{\\prime}=90^{\\circ}$ (see Figure 3). In the circles $K Q H$ and $E A^{\\prime} H$, the line segments $H Q$ and $H A^{\\prime}$ are diameters, respectively; so, these circles have a common tangent $t$ at $H$, perpendicular to $M H$. Let $R$ be the radical center of the circles $A B C, K Q H$ and $E A^{\\prime} H$. Their pairwise radical axes are the lines $Q K$, $A^{\\prime} E$ and the line $t$; they all pass through $R$. Let $S$ be the midpoint of $H R$; by $\\angle Q K H=$ ![](https:\/\/cdn.mathpix.com\/cropped\/2024_11_18_62dbd531c72d0ff8164cg-56.jpg?height=689&width=960&top_left_y=181&top_left_x=545) Figure 3 $\\angle H E A^{\\prime}=90^{\\circ}$, the quadrilateral $H E R K$ is cyclic and its circumcenter is $S$; hence we have $S K=S E=S H$. The line $B C$, being the perpendicular bisector of $H E$, passes through $S$. The circle $H M F$ also is tangent to $t$ at $H$; from the power of $S$ with respect to the circle $H M F$ we have $$ S M \\cdot S F=S H^{2}=S K^{2} $$ So, the power of $S$ with respect to the circles $K Q H$ and $K F M$ is $S K^{2}$. Therefore, the line segment $S K$ is tangent to both circles at $K$.","tier":0} +{"problem_type":"Geometry","problem_label":"G7","problem":"Let $A B C D$ be a convex quadrilateral, and let $P, Q, R$, and $S$ be points on the sides $A B, B C, C D$, and $D A$, respectively. Let the line segments $P R$ and $Q S$ meet at $O$. Suppose that each of the quadrilaterals $A P O S, B Q O P, C R O Q$, and $D S O R$ has an incircle. Prove that the lines $A C, P Q$, and $R S$ are either concurrent or parallel to each other. (Bulgaria)","solution":"Denote by $\\gamma_{A}, \\gamma_{B}, \\gamma_{C}$, and $\\gamma_{D}$ the incircles of the quadrilaterals $A P O S, B Q O P$, $C R O Q$, and $D S O R$, respectively. We start with proving that the quadrilateral $A B C D$ also has an incircle which will be referred to as $\\Omega$. Denote the points of tangency as in Figure 1. It is well-known that $Q Q_{1}=O O_{1}$ (if $B C \\| P R$, this is obvious; otherwise, one may regard the two circles involved as the incircle and an excircle of the triangle formed by the lines $O Q, P R$, and $B C$ ). Similarly, $O O_{1}=P P_{1}$. Hence we have $Q Q_{1}=P P_{1}$. The other equalities of segment lengths marked in Figure 1 can be proved analogously. These equalities, together with $A P_{1}=A S_{1}$ and similar ones, yield $A B+C D=A D+B C$, as required. ![](https:\/\/cdn.mathpix.com\/cropped\/2024_11_18_62dbd531c72d0ff8164cg-57.jpg?height=654&width=1377&top_left_y=958&top_left_x=342) Figure 1 Next, let us draw the lines parallel to $Q S$ through $P$ and $R$, and also draw the lines parallel to $P R$ through $Q$ and $S$. These lines form a parallelogram; denote its vertices by $A^{\\prime}, B^{\\prime}, C^{\\prime}$, and $D^{\\prime}$ as shown in Figure 2. Since the quadrilateral $A P O S$ has an incircle, we have $A P-A S=O P-O S=A^{\\prime} S-A^{\\prime} P$. It is well-known that in this case there also exists a circle $\\omega_{A}$ tangent to the four rays $A P$, $A S, A^{\\prime} P$, and $A^{\\prime} S$. It is worth mentioning here that in case when, say, the lines $A B$ and $A^{\\prime} B^{\\prime}$ coincide, the circle $\\omega_{A}$ is just tangent to $A B$ at $P$. We introduce the circles $\\omega_{B}, \\omega_{C}$, and $\\omega_{D}$ in a similar manner. Assume that the radii of the circles $\\omega_{A}$ and $\\omega_{C}$ are different. Let $X$ be the center of the homothety having a positive scale factor and mapping $\\omega_{A}$ to $\\omega_{C}$. Now, Monge's theorem applied to the circles $\\omega_{A}, \\Omega$, and $\\omega_{C}$ shows that the points $A, C$, and $X$ are collinear. Applying the same theorem to the circles $\\omega_{A}, \\omega_{B}$, and $\\omega_{C}$, we see that the points $P, Q$, and $X$ are also collinear. Similarly, the points $R, S$, and $X$ are collinear, as required. If the radii of $\\omega_{A}$ and $\\omega_{C}$ are equal but these circles do not coincide, then the degenerate version of the same theorem yields that the three lines $A C, P Q$, and $R S$ are parallel to the line of centers of $\\omega_{A}$ and $\\omega_{C}$. Finally, we need to say a few words about the case when $\\omega_{A}$ and $\\omega_{C}$ coincide (and thus they also coincide with $\\Omega, \\omega_{B}$, and $\\omega_{D}$ ). It may be regarded as the limit case in the following manner. ![](https:\/\/cdn.mathpix.com\/cropped\/2024_11_18_62dbd531c72d0ff8164cg-58.jpg?height=1012&width=1580&top_left_y=179&top_left_x=244) Figure 2 Let us fix the positions of $A, P, O$, and $S$ (thus we also fix the circles $\\omega_{A}, \\gamma_{A}, \\gamma_{B}$, and $\\gamma_{D}$ ). Now we vary the circle $\\gamma_{C}$ inscribed into $\\angle Q O R$; for each of its positions, one may reconstruct the lines $B C$ and $C D$ as the external common tangents to $\\gamma_{B}, \\gamma_{C}$ and $\\gamma_{C}, \\gamma_{D}$ different from $P R$ and $Q S$, respectively. After such variation, the circle $\\Omega$ changes, so the result obtained above may be applied.","tier":0} +{"problem_type":"Geometry","problem_label":"G7","problem":"Let $A B C D$ be a convex quadrilateral, and let $P, Q, R$, and $S$ be points on the sides $A B, B C, C D$, and $D A$, respectively. Let the line segments $P R$ and $Q S$ meet at $O$. Suppose that each of the quadrilaterals $A P O S, B Q O P, C R O Q$, and $D S O R$ has an incircle. Prove that the lines $A C, P Q$, and $R S$ are either concurrent or parallel to each other. (Bulgaria)","solution":"Applying Menelaus' theorem to $\\triangle A B C$ with the line $P Q$ and to $\\triangle A C D$ with the line $R S$, we see that the line $A C$ meets $P Q$ and $R S$ at the same point (possibly at infinity) if and only if $$ \\frac{A P}{P B} \\cdot \\frac{B Q}{Q C} \\cdot \\frac{C R}{R D} \\cdot \\frac{D S}{S A}=1 $$ So, it suffices to prove (1). We start with the following result. Lemma 1. Let $E F G H$ be a circumscribed quadrilateral, and let $M$ be its incenter. Then $$ \\frac{E F \\cdot F G}{G H \\cdot H E}=\\frac{F M^{2}}{H M^{2}} $$ Proof. Notice that $\\angle E M H+\\angle G M F=\\angle F M E+\\angle H M G=180^{\\circ}, \\angle F G M=\\angle M G H$, and $\\angle H E M=\\angle M E F$ (see Figure 3). By the law of sines, we get $$ \\frac{E F}{F M} \\cdot \\frac{F G}{F M}=\\frac{\\sin \\angle F M E \\cdot \\sin \\angle G M F}{\\sin \\angle M E F \\cdot \\sin \\angle F G M}=\\frac{\\sin \\angle H M G \\cdot \\sin \\angle E M H}{\\sin \\angle M G H \\cdot \\sin \\angle H E M}=\\frac{G H}{H M} \\cdot \\frac{H E}{H M} . $$ ![](https:\/\/cdn.mathpix.com\/cropped\/2024_11_18_62dbd531c72d0ff8164cg-59.jpg?height=569&width=646&top_left_y=452&top_left_x=225) Figure 3 ![](https:\/\/cdn.mathpix.com\/cropped\/2024_11_18_62dbd531c72d0ff8164cg-59.jpg?height=829&width=954&top_left_y=179&top_left_x=885) Figure 4 We denote by $I, J, K$, and $L$ the incenters of the quadrilaterals $A P O S, B Q O P, C R O Q$, and $D S O R$, respectively. Applying Lemma 1 to these four quadrilaterals we get $$ \\frac{A P \\cdot P O}{O S \\cdot S A} \\cdot \\frac{B Q \\cdot Q O}{O P \\cdot P B} \\cdot \\frac{C R \\cdot R O}{O Q \\cdot Q C} \\cdot \\frac{D S \\cdot S O}{O R \\cdot R D}=\\frac{P I^{2}}{S I^{2}} \\cdot \\frac{Q J^{2}}{P J^{2}} \\cdot \\frac{R K^{2}}{Q K^{2}} \\cdot \\frac{S L^{2}}{R L^{2}} $$ which reduces to $$ \\frac{A P}{P B} \\cdot \\frac{B Q}{Q C} \\cdot \\frac{C R}{R D} \\cdot \\frac{D S}{S A}=\\frac{P I^{2}}{P J^{2}} \\cdot \\frac{Q J^{2}}{Q K^{2}} \\cdot \\frac{R K^{2}}{R L^{2}} \\cdot \\frac{S L^{2}}{S I^{2}} $$ Next, we have $\\angle I P J=\\angle J O I=90^{\\circ}$, and the line $O P$ separates $I$ and $J$ (see Figure 4). This means that the quadrilateral $I P J O$ is cyclic. Similarly, we get that the quadrilateral $J Q K O$ is cyclic with $\\angle J Q K=90^{\\circ}$. Thus, $\\angle Q K J=\\angle Q O J=\\angle J O P=\\angle J I P$. Hence, the right triangles $I P J$ and $K Q J$ are similar. Therefore, $\\frac{P I}{P J}=\\frac{Q K}{Q J}$. Likewise, we obtain $\\frac{R K}{R L}=\\frac{S I}{S L}$. These two equations together with (2) yield (1). Comment. Instead of using the sine law, one may prove Lemma 1 by the following approach. ![](https:\/\/cdn.mathpix.com\/cropped\/2024_11_18_62dbd531c72d0ff8164cg-59.jpg?height=558&width=698&top_left_y=2028&top_left_x=685) Figure 5 Let $N$ be the point such that $\\triangle N H G \\sim \\triangle M E F$ and such that $N$ and $M$ lie on different sides of the line $G H$, as shown in Figure 5. Then $\\angle G N H+\\angle H M G=\\angle F M E+\\angle H M G=180^{\\circ}$. So, the quadrilateral $G N H M$ is cyclic. Thus, $\\angle M N H=\\angle M G H=\\angle F G M$ and $\\angle H M N=\\angle H G N=$ $\\angle E F M=\\angle M F G$. Hence, $\\triangle H M N \\sim \\triangle M F G$. Therefore, $\\frac{H M}{H G}=\\frac{H M}{H N} \\cdot \\frac{H N}{H G}=\\frac{M F}{M G} \\cdot \\frac{E M}{E F}$. Similarly, we obtain $\\frac{H M}{H E}=\\frac{M F}{M E} \\cdot \\frac{G M}{G F}$. By multiplying these two equations, we complete the proof.","tier":0} +{"problem_type":"Geometry","problem_label":"G7","problem":"Let $A B C D$ be a convex quadrilateral, and let $P, Q, R$, and $S$ be points on the sides $A B, B C, C D$, and $D A$, respectively. Let the line segments $P R$ and $Q S$ meet at $O$. Suppose that each of the quadrilaterals $A P O S, B Q O P, C R O Q$, and $D S O R$ has an incircle. Prove that the lines $A C, P Q$, and $R S$ are either concurrent or parallel to each other. (Bulgaria)","solution":"We present another approach for showing (1) from Lemma 2. Let $E F G H$ and $E^{\\prime} F^{\\prime} G^{\\prime} H^{\\prime}$ be circumscribed quadrilaterals such that $\\angle E+\\angle E^{\\prime}=$ $\\angle F+\\angle F^{\\prime}=\\angle G+\\angle G^{\\prime}=\\angle H+\\angle H^{\\prime}=180^{\\circ}$. Then $$ \\frac{E F \\cdot G H}{F G \\cdot H E}=\\frac{E^{\\prime} F^{\\prime} \\cdot G^{\\prime} H^{\\prime}}{F^{\\prime} G^{\\prime} \\cdot H^{\\prime} E^{\\prime}} $$ Proof. Let $M$ and $M^{\\prime}$ be the incenters of $E F G H$ and $E^{\\prime} F^{\\prime} G^{\\prime} H^{\\prime}$, respectively. We use the notation [XYZ] for the area of a triangle $X Y Z$. Taking into account the relation $\\angle F M E+\\angle F^{\\prime} M^{\\prime} E^{\\prime}=180^{\\circ}$ together with the analogous ones, we get $$ \\begin{aligned} \\frac{E F \\cdot G H}{F G \\cdot H E} & =\\frac{[M E F] \\cdot[M G H]}{[M F G] \\cdot[M H E]}=\\frac{M E \\cdot M F \\cdot \\sin \\angle F M E \\cdot M G \\cdot M H \\cdot \\sin \\angle H M G}{M F \\cdot M G \\cdot \\sin \\angle G M F \\cdot M H \\cdot M E \\cdot \\sin \\angle E M H} \\\\ & =\\frac{M^{\\prime} E^{\\prime} \\cdot M^{\\prime} F^{\\prime} \\cdot \\sin \\angle F^{\\prime} M^{\\prime} E^{\\prime} \\cdot M^{\\prime} G^{\\prime} \\cdot M^{\\prime} H^{\\prime} \\cdot \\sin \\angle H^{\\prime} M^{\\prime} G^{\\prime}}{M^{\\prime} F^{\\prime} \\cdot M^{\\prime} G^{\\prime} \\cdot \\sin \\angle G^{\\prime} M^{\\prime} F^{\\prime} \\cdot M^{\\prime} H^{\\prime} \\cdot M^{\\prime} E^{\\prime} \\cdot \\sin \\angle E^{\\prime} M^{\\prime} H^{\\prime}}=\\frac{E^{\\prime} F^{\\prime} \\cdot G^{\\prime} H^{\\prime}}{F^{\\prime} G^{\\prime} \\cdot H^{\\prime} E^{\\prime}} . \\end{aligned} $$ ![](https:\/\/cdn.mathpix.com\/cropped\/2024_11_18_62dbd531c72d0ff8164cg-60.jpg?height=849&width=1089&top_left_y=1300&top_left_x=489) Figure 6 Denote by $h$ the homothety centered at $O$ that maps the incircle of $C R O Q$ to the incircle of $A P O S$. Let $Q^{\\prime}=h(Q), C^{\\prime}=h(C), R^{\\prime}=h(R), O^{\\prime}=O, S^{\\prime}=S, A^{\\prime}=A$, and $P^{\\prime}=P$. Furthermore, define $B^{\\prime}=A^{\\prime} P^{\\prime} \\cap C^{\\prime} Q^{\\prime}$ and $D^{\\prime}=A^{\\prime} S^{\\prime} \\cap C^{\\prime} R^{\\prime}$ as shown in Figure 6. Then $$ \\frac{A P \\cdot O S}{P O \\cdot S A}=\\frac{A^{\\prime} P^{\\prime} \\cdot O^{\\prime} S^{\\prime}}{P^{\\prime} O^{\\prime} \\cdot S^{\\prime} A^{\\prime}} $$ holds trivially. We also have $$ \\frac{C R \\cdot O Q}{R O \\cdot Q C}=\\frac{C^{\\prime} R^{\\prime} \\cdot O^{\\prime} Q^{\\prime}}{R^{\\prime} O^{\\prime} \\cdot Q^{\\prime} C^{\\prime}} $$ by the similarity of the quadrilaterals $C R O Q$ and $C^{\\prime} R^{\\prime} O^{\\prime} Q^{\\prime}$. Next, consider the circumscribed quadrilaterals $B Q O P$ and $B^{\\prime} Q^{\\prime} O^{\\prime} P^{\\prime}$ whose incenters lie on different sides of the quadrilaterals' shared side line $O P=O^{\\prime} P^{\\prime}$. Observe that $B Q \\| B^{\\prime} Q^{\\prime}$ and that $B^{\\prime}$ and $Q^{\\prime}$ lie on the lines $B P$ and $Q O$, respectively. It is now easy to see that the two quadrilaterals satisfy the hypotheses of Lemma 2. Thus, we deduce $$ \\frac{B Q \\cdot O P}{Q O \\cdot P B}=\\frac{B^{\\prime} Q^{\\prime} \\cdot O^{\\prime} P^{\\prime}}{Q^{\\prime} O^{\\prime} \\cdot P^{\\prime} B^{\\prime}} $$ Similarly, we get $$ \\frac{D S \\cdot O R}{S O \\cdot R D}=\\frac{D^{\\prime} S^{\\prime} \\cdot O^{\\prime} R^{\\prime}}{S^{\\prime} O^{\\prime} \\cdot R^{\\prime} D^{\\prime}} $$ Multiplying these four equations, we obtain $$ \\frac{A P}{P B} \\cdot \\frac{B Q}{Q C} \\cdot \\frac{C R}{R D} \\cdot \\frac{D S}{S A}=\\frac{A^{\\prime} P^{\\prime}}{P^{\\prime} B^{\\prime}} \\cdot \\frac{B^{\\prime} Q^{\\prime}}{Q^{\\prime} C^{\\prime}} \\cdot \\frac{C^{\\prime} R^{\\prime}}{R^{\\prime} D^{\\prime}} \\cdot \\frac{D^{\\prime} S^{\\prime}}{S^{\\prime} A^{\\prime}} $$ Finally, we apply BRIANChon's theorem to the circumscribed hexagon $A^{\\prime} P^{\\prime} R^{\\prime} C^{\\prime} Q^{\\prime} S^{\\prime}$ and deduce that the lines $A^{\\prime} C^{\\prime}, P^{\\prime} Q^{\\prime}$, and $R^{\\prime} S^{\\prime}$ are either concurrent or parallel to each other. So, by Menelaus' theorem, we obtain $$ \\frac{A^{\\prime} P^{\\prime}}{P^{\\prime} B^{\\prime}} \\cdot \\frac{B^{\\prime} Q^{\\prime}}{Q^{\\prime} C^{\\prime}} \\cdot \\frac{C^{\\prime} R^{\\prime}}{R^{\\prime} D^{\\prime}} \\cdot \\frac{D^{\\prime} S^{\\prime}}{S^{\\prime} A^{\\prime}}=1 $$ This equation together with (3) yield (1).","tier":0} +{"problem_type":"Geometry","problem_label":"G8","problem":"A triangulation of a convex polygon $\\Pi$ is a partitioning of $\\Pi$ into triangles by diagonals having no common points other than the vertices of the polygon. We say that a triangulation is a Thaiangulation if all triangles in it have the same area. Prove that any two different Thaiangulations of a convex polygon $\\Pi$ differ by exactly two triangles. (In other words, prove that it is possible to replace one pair of triangles in the first Thaiangulation with a different pair of triangles so as to obtain the second Thaiangulation.) (Bulgaria)","solution":"We denote by [S] the area of a polygon $S$. Recall that each triangulation of a convex $n$-gon has exactly $n-2$ triangles. This means that all triangles in any two Thaiangulations of a convex polygon $\\Pi$ have the same area. Let $\\mathcal{T}$ be a triangulation of a convex polygon $\\Pi$. If four vertices $A, B, C$, and $D$ of $\\Pi$ form a parallelogram, and $\\mathcal{T}$ contains two triangles whose union is this parallelogram, then we say that $\\mathcal{T}$ contains parallelogram $A B C D$. Notice here that if two Thaiangulations $\\mathcal{T}_{1}$ and $\\mathcal{T}_{2}$ of $\\Pi$ differ by two triangles, then the union of these triangles is a quadrilateral each of whose diagonals bisects its area, i.e., a parallelogram. We start with proving two properties of triangulations. Lemma 1. A triangulation of a convex polygon $\\Pi$ cannot contain two parallelograms. Proof. Arguing indirectly, assume that $P_{1}$ and $P_{2}$ are two parallelograms contained in some triangulation $\\mathcal{T}$. If they have a common triangle in $\\mathcal{T}$, then we may assume that $P_{1}$ consists of triangles $A B C$ and $A D C$ of $\\mathcal{T}$, while $P_{2}$ consists of triangles $A D C$ and $C D E$ (see Figure 1). But then $B C\\|A D\\| C E$, so the three vertices $B, C$, and $E$ of $\\Pi$ are collinear, which is absurd. Assume now that $P_{1}$ and $P_{2}$ contain no common triangle. Let $P_{1}=A B C D$. The sides $A B$, $B C, C D$, and $D A$ partition $\\Pi$ into several parts, and $P_{2}$ is contained in one of them; we may assume that this part is cut off from $P_{1}$ by $A D$. Then one may label the vertices of $P_{2}$ by $X$, $Y, Z$, and $T$ so that the polygon $A B C D X Y Z T$ is convex (see Figure 2; it may happen that $D=X$ and\/or $T=A$, but still this polygon has at least six vertices). But the sum of the external angles of this polygon at $B, C, Y$, and $Z$ is already $360^{\\circ}$, which is impossible. A final contradiction. ![](https:\/\/cdn.mathpix.com\/cropped\/2024_11_18_62dbd531c72d0ff8164cg-62.jpg?height=458&width=269&top_left_y=1687&top_left_x=251) Figure 1 ![](https:\/\/cdn.mathpix.com\/cropped\/2024_11_18_62dbd531c72d0ff8164cg-62.jpg?height=341&width=601&top_left_y=1800&top_left_x=613) Figure 2 ![](https:\/\/cdn.mathpix.com\/cropped\/2024_11_18_62dbd531c72d0ff8164cg-62.jpg?height=406&width=484&top_left_y=1733&top_left_x=1323) Figure 3 Lemma 2. Every triangle in a Thaiangulation $\\mathcal{T}$ of $\\Pi$ contains a side of $\\Pi$. Proof. Let $A B C$ be a triangle in $\\mathcal{T}$. Apply an affine transform such that $A B C$ maps to an equilateral triangle; let $A^{\\prime} B^{\\prime} C^{\\prime}$ be the image of this triangle, and $\\Pi^{\\prime}$ be the image of $\\Pi$. Clearly, $\\mathcal{T}$ maps into a Thaiangulation $\\mathcal{T}^{\\prime}$ of $\\Pi^{\\prime}$. Assume that none of the sides of $\\triangle A^{\\prime} B^{\\prime} C^{\\prime}$ is a side of $\\Pi^{\\prime}$. Then $\\mathcal{T}^{\\prime}$ contains some other triangles with these sides, say, $A^{\\prime} B^{\\prime} Z, C^{\\prime} A^{\\prime} Y$, and $B^{\\prime} C^{\\prime} X$; notice that $A^{\\prime} Z B^{\\prime} X C^{\\prime} Y$ is a convex hexagon (see Figure 3). The sum of its external angles at $X, Y$, and $Z$ is less than $360^{\\circ}$. So one of these angles (say, at $Z$ ) is less than $120^{\\circ}$, hence $\\angle A^{\\prime} Z B^{\\prime}>60^{\\circ}$. Then $Z$ lies on a circular arc subtended by $A^{\\prime} B^{\\prime}$ and having angular measure less than $240^{\\circ}$; consequently, the altitude $Z H$ of $\\triangle A^{\\prime} B^{\\prime} Z$ is less than $\\sqrt{3} A^{\\prime} B^{\\prime} \/ 2$. Thus $\\left[A^{\\prime} B^{\\prime} Z\\right]<\\left[A^{\\prime} B^{\\prime} C^{\\prime}\\right]$, and $\\mathcal{T}^{\\prime}$ is not a Thaiangulation. A contradiction. Now we pass to the solution. We say that a triangle in a triangulation of $\\Pi$ is an ear if it contains two sides of $\\Pi$. Note that each triangulation of a polygon contains some ear. Arguing indirectly, we choose a convex polygon $\\Pi$ with the least possible number of sides such that some two Thaiangulations $\\mathcal{T}_{1}$ and $\\mathcal{T}_{2}$ of $\\Pi$ violate the problem statement (thus $\\Pi$ has at least five sides). Consider now any ear $A B C$ in $\\mathcal{T}_{1}$, with $A C$ being a diagonal of $\\Pi$. If $\\mathcal{T}_{2}$ also contains $\\triangle A B C$, then one may cut $\\triangle A B C$ off from $\\Pi$, getting a polygon with a smaller number of sides which also violates the problem statement. This is impossible; thus $\\mathcal{T}_{2}$ does not contain $\\triangle A B C$. Next, $\\mathcal{T}_{1}$ contains also another triangle with side $A C$, say $\\triangle A C D$. By Lemma 2 , this triangle contains a side of $\\Pi$, so $D$ is adjacent to either $A$ or $C$ on the boundary of $\\Pi$. We may assume that $D$ is adjacent to $C$. Assume that $\\mathcal{T}_{2}$ does not contain the triangle $B C D$. Then it contains two different triangles $B C X$ and $C D Y$ (possibly, with $X=Y$ ); since these triangles have no common interior points, the polygon $A B C D Y X$ is convex (see Figure 4). But, since $[A B C]=[B C X]=$ $[A C D]=[C D Y]$, we get $A X \\| B C$ and $A Y \\| C D$ which is impossible. Thus $\\mathcal{T}_{2}$ contains $\\triangle B C D$. Therefore, $[A B D]=[A B C]+[A C D]-[B C D]=[A B C]$, and $A B C D$ is a parallelogram contained in $\\mathcal{T}_{1}$. Let $\\mathcal{T}^{\\prime}$ be the Thaiangulation of $\\Pi$ obtained from $\\mathcal{T}_{1}$ by replacing the diagonal $A C$ with $B D$; then $\\mathcal{T}^{\\prime}$ is distinct from $\\mathcal{T}_{2}$ (otherwise $\\mathcal{T}_{1}$ and $\\mathcal{T}_{2}$ would differ by two triangles). Moreover, $\\mathcal{T}^{\\prime}$ shares a common ear $B C D$ with $\\mathcal{T}_{2}$. As above, cutting this ear away we obtain that $\\mathcal{T}_{2}$ and $\\mathcal{T}^{\\prime}$ differ by two triangles forming a parallelogram different from $A B C D$. Thus $\\mathcal{T}^{\\prime}$ contains two parallelograms, which contradicts Lemma 1. ![](https:\/\/cdn.mathpix.com\/cropped\/2024_11_18_62dbd531c72d0ff8164cg-63.jpg?height=455&width=1166&top_left_y=1372&top_left_x=199) Figure 4 ![](https:\/\/cdn.mathpix.com\/cropped\/2024_11_18_62dbd531c72d0ff8164cg-63.jpg?height=412&width=506&top_left_y=1416&top_left_x=1363) Figure 5 Comment 1. Lemma 2 is equivalent to the well-known Erd\u0151s-Debrunner inequality stating that for any triangle $P Q R$ and any points $A, B, C$ lying on the sides $Q R, R P$, and $P Q$, respectively, we have $$ [A B C] \\geqslant \\min \\{[A B R],[B C P],[C A Q]\\} $$ To derive this inequality from Lemma 2, one may assume that (1) does not hold, and choose some points $X, Y$, and $Z$ inside the triangles $B C P, C A Q$, and $A B R$, respectively, so that $[A B C]=$ $[A B Z]=[B C X]=[C A Y]$. Then a convex hexagon $A Z B X C Y$ has a Thaiangulation containing $\\triangle A B C$, which contradicts Lemma 2. Conversely, assume that a Thaiangulation $\\mathcal{T}$ of $\\Pi$ contains a triangle $A B C$ none of whose sides is a side of $\\Pi$, and let $A B Z, A Y C$, and $X B C$ be other triangles in $\\mathcal{T}$ containing the corresponding sides. Then $A Z B X C Y$ is a convex hexagon. Consider the lines through $A, B$, and $C$ parallel to $Y Z, Z X$, and $X Y$, respectively. They form a triangle $X^{\\prime} Y^{\\prime} Z^{\\prime}$ similar to $\\triangle X Y Z$ (see Figure 5). By (1) we have $$ [A B C] \\geqslant \\min \\left\\{\\left[A B Z^{\\prime}\\right],\\left[B C X^{\\prime}\\right],\\left[C A Y^{\\prime}\\right]\\right\\}>\\min \\{[A B Z],[B C X],[C A Y]\\}, $$ so $\\mathcal{T}$ is not a Thaiangulation.","tier":0} +{"problem_type":"Geometry","problem_label":"G8","problem":"A triangulation of a convex polygon $\\Pi$ is a partitioning of $\\Pi$ into triangles by diagonals having no common points other than the vertices of the polygon. We say that a triangulation is a Thaiangulation if all triangles in it have the same area. Prove that any two different Thaiangulations of a convex polygon $\\Pi$ differ by exactly two triangles. (In other words, prove that it is possible to replace one pair of triangles in the first Thaiangulation with a different pair of triangles so as to obtain the second Thaiangulation.) (Bulgaria)","solution":"We will make use of the preliminary observations from Arguing indirectly, we choose a convex polygon $\\Pi$ with the least possible number of sides such that some two Thaiangulations $\\mathcal{T}_{1}$ and $\\mathcal{T}_{2}$ of $\\Pi$ violate the statement (thus $\\Pi$ has at least five sides). Assume that $\\mathcal{T}_{1}$ and $\\mathcal{T}_{2}$ share a diagonal $d$ splitting $\\Pi$ into two smaller polygons $\\Pi_{1}$ and $\\Pi_{2}$. Since the problem statement holds for any of them, the induced Thaiangulations of each of $\\Pi_{i}$ differ by two triangles forming a parallelogram (the Thaiangulations induced on $\\Pi_{i}$ by $\\mathcal{T}_{1}$ and $T_{2}$ may not coincide, otherwise $\\mathcal{T}_{1}$ and $\\mathcal{T}_{2}$ would differ by at most two triangles). But both these parallelograms are contained in $\\mathcal{T}_{1}$; this contradicts Lemma 1. Therefore, $\\mathcal{T}_{1}$ and $\\mathcal{T}_{2}$ share no diagonal. Hence they also share no triangle. We consider two cases. Case 1. Assume that some vertex $B$ of $\\Pi$ is an endpoint of some diagonal in $\\mathcal{T}_{1}$, as well as an endpoint of some diagonal in $\\mathcal{T}_{2}$. Let $A$ and $C$ be the vertices of $\\Pi$ adjacent to $B$. Then $\\mathcal{T}_{1}$ contains some triangles $A B X$ and $B C Y$, while $\\mathcal{T}_{2}$ contains some triangles $A B X^{\\prime}$ and $B C Y^{\\prime}$. Here, some of the points $X$, $X^{\\prime}, Y$, and $Y^{\\prime}$ may coincide; however, in view of our assumption together with the fact that $\\mathcal{T}_{1}$ and $\\mathcal{T}_{2}$ share no triangle, all four triangles $A B X, B C Y, A B X^{\\prime}$, and $B C Y^{\\prime}$ are distinct. Since $[A B X]=[B C Y]=\\left[A B X^{\\prime}\\right]=\\left[B C Y^{\\prime}\\right]$, we have $X X^{\\prime} \\| A B$ and $Y Y^{\\prime} \\| B C$. Now, if $X=Y$, then $X^{\\prime}$ and $Y^{\\prime}$ lie on different lines passing through $X$ and are distinct from that point, so that $X^{\\prime} \\neq Y^{\\prime}$. In this case, we may switch the two Thaiangulations. So, hereafter we assume that $X \\neq Y$. In the convex pentagon $A B C Y X$ we have either $\\angle B A X+\\angle A X Y>180^{\\circ}$ or $\\angle X Y C+$ $\\angle Y C B>180^{\\circ}$ (or both); due to the symmetry, we may assume that the first inequality holds. Let $r$ be the ray emerging from $X$ and co-directed with $\\overrightarrow{A B}$; our inequality shows that $r$ points to the interior of the pentagon (and thus to the interior of $\\Pi$ ). Therefore, the ray opposite to $r$ points outside $\\Pi$, so $X^{\\prime}$ lies on $r$; moreover, $X^{\\prime}$ lies on the \"arc\" $C Y$ of $\\Pi$ not containing $X$. So the segments $X X^{\\prime}$ and $Y B$ intersect (see Figure 6). Let $O$ be the intersection point of the rays $r$ and $B C$. Since the triangles $A B X^{\\prime}$ and $B C Y^{\\prime}$ have no common interior points, $Y^{\\prime}$ must lie on the \"arc\" $C X^{\\prime}$ which is situated inside the triangle $X B O$. Therefore, the line $Y Y^{\\prime}$ meets two sides of $\\triangle X B O$, none of which may be $X B$ (otherwise the diagonals $X B$ and $Y Y^{\\prime}$ would share a common point). Thus $Y Y^{\\prime}$ intersects $B O$, which contradicts $Y Y^{\\prime} \\| B C$. ![](https:\/\/cdn.mathpix.com\/cropped\/2024_11_18_62dbd531c72d0ff8164cg-64.jpg?height=518&width=892&top_left_y=1928&top_left_x=588) Figure 6 Case 2. In the remaining case, each vertex of $\\Pi$ is an endpoint of a diagonal in at most one of $\\mathcal{T}_{1}$ and $\\mathcal{T}_{2}$. On the other hand, a triangulation cannot contain two consecutive vertices with no diagonals from each. Therefore, the vertices of $\\Pi$ alternatingly emerge diagonals in $\\mathcal{T}_{1}$ and in $\\mathcal{T}_{2}$. In particular, $\\Pi$ has an even number of sides. Next, we may choose five consecutive vertices $A, B, C, D$, and $E$ of $\\Pi$ in such a way that $$ \\angle A B C+\\angle B C D>180^{\\circ} \\text { and } \\angle B C D+\\angle C D E>180^{\\circ} . $$ In order to do this, it suffices to choose three consecutive vertices $B, C$, and $D$ of $\\Pi$ such that the sum of their external angles is at most $180^{\\circ}$. This is possible, since $\\Pi$ has at least six sides. ![](https:\/\/cdn.mathpix.com\/cropped\/2024_11_18_62dbd531c72d0ff8164cg-65.jpg?height=529&width=701&top_left_y=521&top_left_x=683) Figure 7 We may assume that $\\mathcal{T}_{1}$ has no diagonals from $B$ and $D$ (and thus contains the triangles $A B C$ and $C D E$ ), while $\\mathcal{T}_{2}$ has no diagonals from $A, C$, and $E$ (and thus contains the triangle $B C D)$. Now, since $[A B C]=[B C D]=[C D E]$, we have $A D \\| B C$ and $B E \\| C D$ (see Figure 7). By (2) this yields that $A D>B C$ and $B E>C D$. Let $X=A C \\cap B D$ and $Y=C E \\cap B D$; then the inequalities above imply that $A X>C X$ and $E Y>C Y$. Finally, $\\mathcal{T}_{2}$ must also contain some triangle $B D Z$ with $Z \\neq C$; then the ray $C Z$ lies in the angle $A C E$. Since $[B C D]=[B D Z]$, the diagonal $B D$ bisects $C Z$. Together with the inequalities above, this yields that $Z$ lies inside the triangle $A C E$ (but $Z$ is distinct from $A$ and $E$ ), which is impossible. The final contradiction. Comment 2. Case 2 may also be accomplished with the use of Lemma 2. Indeed, since each triangulation of an $n$-gon contains $n-2$ triangles neither of which may contain three sides of $\\Pi$, Lemma 2 yields that each Thaiangulation contains exactly two ears. But each vertex of $\\Pi$ is a vertex of an ear either in $\\mathcal{T}_{1}$ or in $\\mathcal{T}_{2}$, so $\\Pi$ cannot have more than four vertices.","tier":0} +{"problem_type":"Number Theory","problem_label":"N1","problem":"Determine all positive integers $M$ for which the sequence $a_{0}, a_{1}, a_{2}, \\ldots$, defined by $a_{0}=\\frac{2 M+1}{2}$ and $a_{k+1}=a_{k}\\left\\lfloor a_{k}\\right\\rfloor$ for $k=0,1,2, \\ldots$, contains at least one integer term. (Luxembourg)","solution":"Define $b_{k}=2 a_{k}$ for all $k \\geqslant 0$. Then $$ b_{k+1}=2 a_{k+1}=2 a_{k}\\left\\lfloor a_{k}\\right\\rfloor=b_{k}\\left\\lfloor\\frac{b_{k}}{2}\\right\\rfloor . $$ Since $b_{0}$ is an integer, it follows that $b_{k}$ is an integer for all $k \\geqslant 0$. Suppose that the sequence $a_{0}, a_{1}, a_{2}, \\ldots$ does not contain any integer term. Then $b_{k}$ must be an odd integer for all $k \\geqslant 0$, so that $$ b_{k+1}=b_{k}\\left\\lfloor\\frac{b_{k}}{2}\\right\\rfloor=\\frac{b_{k}\\left(b_{k}-1\\right)}{2} $$ Hence $$ b_{k+1}-3=\\frac{b_{k}\\left(b_{k}-1\\right)}{2}-3=\\frac{\\left(b_{k}-3\\right)\\left(b_{k}+2\\right)}{2} $$ for all $k \\geqslant 0$. Suppose that $b_{0}-3>0$. Then equation (2) yields $b_{k}-3>0$ for all $k \\geqslant 0$. For each $k \\geqslant 0$, define $c_{k}$ to be the highest power of 2 that divides $b_{k}-3$. Since $b_{k}-3$ is even for all $k \\geqslant 0$, the number $c_{k}$ is positive for every $k \\geqslant 0$. Note that $b_{k}+2$ is an odd integer. Therefore, from equation (2), we have that $c_{k+1}=c_{k}-1$. Thus, the sequence $c_{0}, c_{1}, c_{2}, \\ldots$ of positive integers is strictly decreasing, a contradiction. So, $b_{0}-3 \\leqslant 0$, which implies $M=1$. For $M=1$, we can check that the sequence is constant with $a_{k}=\\frac{3}{2}$ for all $k \\geqslant 0$. Therefore, the answer is $M \\geqslant 2$.","tier":0} +{"problem_type":"Number Theory","problem_label":"N1","problem":"Determine all positive integers $M$ for which the sequence $a_{0}, a_{1}, a_{2}, \\ldots$, defined by $a_{0}=\\frac{2 M+1}{2}$ and $a_{k+1}=a_{k}\\left\\lfloor a_{k}\\right\\rfloor$ for $k=0,1,2, \\ldots$, contains at least one integer term. (Luxembourg)","solution":"We provide an alternative way to show $M=1$ once equation (1) has been reached. We claim that $b_{k} \\equiv 3\\left(\\bmod 2^{m}\\right)$ for all $k \\geqslant 0$ and $m \\geqslant 1$. If this is true, then we would have $b_{k}=3$ for all $k \\geqslant 0$ and hence $M=1$. To establish our claim, we proceed by induction on $m$. The base case $b_{k} \\equiv 3(\\bmod 2)$ is true for all $k \\geqslant 0$ since $b_{k}$ is odd. Now suppose that $b_{k} \\equiv 3\\left(\\bmod 2^{m}\\right)$ for all $k \\geqslant 0$. Hence $b_{k}=2^{m} d_{k}+3$ for some integer $d_{k}$. We have $$ 3 \\equiv b_{k+1} \\equiv\\left(2^{m} d_{k}+3\\right)\\left(2^{m-1} d_{k}+1\\right) \\equiv 3 \\cdot 2^{m-1} d_{k}+3 \\quad\\left(\\bmod 2^{m}\\right) $$ so that $d_{k}$ must be even. This implies that $b_{k} \\equiv 3\\left(\\bmod 2^{m+1}\\right)$, as required. Comment. The reason the number 3 which appears in both solutions is important, is that it is a nontrivial fixed point of the recurrence relation for $b_{k}$.","tier":0} +{"problem_type":"Number Theory","problem_label":"N2","problem":"Let $a$ and $b$ be positive integers such that $a!b$ ! is a multiple of $a!+b!$. Prove that $3 a \\geqslant 2 b+2$. (United Kingdom)","solution":"If $a>b$, we immediately get $3 a \\geqslant 2 b+2$. In the case $a=b$, the required inequality is equivalent to $a \\geqslant 2$, which can be checked easily since $(a, b)=(1,1)$ does not satisfy $a!+b!\\mid a!b!$. We now assume $aa$ !, which is impossible. We observe that $c!\\mid M$ since $M$ is a product of $c$ consecutive integers. Thus $\\operatorname{gcd}(1+M, c!)=1$, which implies $$ 1+M \\left\\lvert\\, \\frac{a!}{c!}=(c+1)(c+2) \\cdots a\\right. $$ If $a \\leqslant 2 c$, then $\\frac{a!}{c!}$ is a product of $a-c \\leqslant c$ integers not exceeding $a$ whereas $M$ is a product of $c$ integers exceeding $a$. Therefore, $1+M>\\frac{a!}{c!}$, which is a contradiction. It remains to exclude the case $a=2 c+1$. Since $a+1=2(c+1)$, we have $c+1 \\mid M$. Hence, we can deduce from (1) that $1+M \\mid(c+2)(c+3) \\cdots a$. Now $(c+2)(c+3) \\cdots a$ is a product of $a-c-1=c$ integers not exceeding $a$; thus it is smaller than $1+M$. Again, we arrive at a contradiction. Comment 1. One may derive a weaker version of (1) and finish the problem as follows. After assuming $a \\leqslant 2 c+1$, we have $\\left\\lfloor\\frac{a}{2}\\right\\rfloor \\leqslant c$, so $\\left.\\left\\lfloor\\frac{a}{2}\\right\\rfloor!\\right\\rvert\\, M$. Therefore, $$ 1+M \\left\\lvert\\,\\left(\\left\\lfloor\\frac{a}{2}\\right\\rfloor+1\\right)\\left(\\left\\lfloor\\frac{a}{2}\\right\\rfloor+2\\right) \\cdots a\\right. $$ Observe that $\\left(\\left\\lfloor\\frac{a}{2}\\right\\rfloor+1\\right)\\left(\\left\\lfloor\\frac{a}{2}\\right\\rfloor+2\\right) \\cdots a$ is a product of $\\left\\lceil\\frac{a}{2}\\right\\rceil$ integers not exceeding $a$. This leads to a contradiction when $a$ is even since $\\left\\lceil\\frac{a}{2}\\right\\rceil=\\frac{a}{2} \\leqslant c$ and $M$ is a product of $c$ integers exceeding $a$. When $a$ is odd, we can further deduce that $1+M \\left\\lvert\\,\\left(\\frac{a+3}{2}\\right)\\left(\\frac{a+5}{2}\\right) \\cdots a\\right.$ since $\\left.\\left\\lfloor\\frac{a}{2}\\right\\rfloor+1=\\frac{a+1}{2} \\right\\rvert\\, a+1$. Now $\\left(\\frac{a+3}{2}\\right)\\left(\\frac{a+5}{2}\\right) \\cdots a$ is a product of $\\frac{a-1}{2} \\leqslant c$ numbers not exceeding $a$, and we get a contradiction.","tier":0} +{"problem_type":"Number Theory","problem_label":"N2","problem":"Let $a$ and $b$ be positive integers such that $a!b$ ! is a multiple of $a!+b!$. Prove that $3 a \\geqslant 2 b+2$. (United Kingdom)","solution":"As in $$ N=1+(a+1)(a+2) \\cdots(a+c) \\mid(a+c)! $$ which implies that all prime factors of $N$ are at most $a+c$. Let $p$ be a prime factor of $N$. If $p \\leqslant c$ or $p \\geqslant a+1$, then $p$ divides one of $a+1, \\ldots, a+c$ which is impossible. Hence $a \\geqslant p \\geqslant c+1$. Furthermore, we must have $2 p>a+c$; otherwise, $a+1 \\leqslant 2 c+2 \\leqslant 2 p \\leqslant a+c$ so $p \\mid N-1$, again impossible. Thus, we have $p \\in\\left(\\frac{a+c}{2}, a\\right]$, and $p^{2} \\nmid(a+c)$ ! since $2 p>a+c$. Therefore, $p^{2} \\nmid N$ as well. If $a \\leqslant c+2$, then the interval $\\left(\\frac{a+c}{2}, a\\right]$ contains at most one integer and hence at most one prime number, which has to be $a$. Since $p^{2} \\nmid N$, we must have $N=p=a$ or $N=1$, which is absurd since $N>a \\geqslant 1$. Thus, we have $a \\geqslant c+3$, and so $\\frac{a+c+1}{2} \\geqslant c+2$. It follows that $p$ lies in the interval $[c+2, a]$. Thus, every prime appearing in the prime factorization of $N$ lies in the interval $[c+2, a]$, and its exponent is exactly 1 . So we must have $N \\mid(c+2)(c+3) \\cdots a$. However, $(c+2)(c+3) \\cdots a$ is a product of $a-c-1 \\leqslant c$ numbers not exceeding $a$, so it is less than $N$. This is a contradiction. Comment 2. The original problem statement also asks to determine when the equality $3 a=2 b+2$ holds. It can be checked that the answer is $(a, b)=(2,2),(4,5)$.","tier":0} +{"problem_type":"Number Theory","problem_label":"N3","problem":"Let $m$ and $n$ be positive integers such that $m>n$. Define $x_{k}=(m+k) \/(n+k)$ for $k=$ $1,2, \\ldots, n+1$. Prove that if all the numbers $x_{1}, x_{2}, \\ldots, x_{n+1}$ are integers, then $x_{1} x_{2} \\cdots x_{n+1}-1$ is divisible by an odd prime. (Austria)","solution":"Assume that $x_{1}, x_{2}, \\ldots, x_{n+1}$ are integers. Define the integers $$ a_{k}=x_{k}-1=\\frac{m+k}{n+k}-1=\\frac{m-n}{n+k}>0 $$ for $k=1,2, \\ldots, n+1$. Let $P=x_{1} x_{2} \\cdots x_{n+1}-1$. We need to prove that $P$ is divisible by an odd prime, or in other words, that $P$ is not a power of 2 . To this end, we investigate the powers of 2 dividing the numbers $a_{k}$. Let $2^{d}$ be the largest power of 2 dividing $m-n$, and let $2^{c}$ be the largest power of 2 not exceeding $2 n+1$. Then $2 n+1 \\leqslant 2^{c+1}-1$, and so $n+1 \\leqslant 2^{c}$. We conclude that $2^{c}$ is one of the numbers $n+1, n+2, \\ldots, 2 n+1$, and that it is the only multiple of $2^{c}$ appearing among these numbers. Let $\\ell$ be such that $n+\\ell=2^{c}$. Since $\\frac{m-n}{n+\\ell}$ is an integer, we have $d \\geqslant c$. Therefore, $2^{d-c+1} \\nmid a_{\\ell}=\\frac{m-n}{n+\\ell}$, while $2^{d-c+1} \\mid a_{k}$ for all $k \\in\\{1, \\ldots, n+1\\} \\backslash\\{\\ell\\}$. Computing modulo $2^{d-c+1}$, we get $$ P=\\left(a_{1}+1\\right)\\left(a_{2}+1\\right) \\cdots\\left(a_{n+1}+1\\right)-1 \\equiv\\left(a_{\\ell}+1\\right) \\cdot 1^{n}-1 \\equiv a_{\\ell} \\not \\equiv 0 \\quad\\left(\\bmod 2^{d-c+1}\\right) $$ Therefore, $2^{d-c+1} \\nmid P$. On the other hand, for any $k \\in\\{1, \\ldots, n+1\\} \\backslash\\{\\ell\\}$, we have $2^{d-c+1} \\mid a_{k}$. So $P \\geqslant a_{k} \\geqslant 2^{d-c+1}$, and it follows that $P$ is not a power of 2 . Comment. Instead of attempting to show that $P$ is not a power of 2 , one may try to find an odd factor of $P$ (greater than 1) as follows: From $a_{k}=\\frac{m-n}{n+k} \\in \\mathbb{Z}_{>0}$, we get that $m-n$ is divisible by $n+1, n+2, \\ldots, 2 n+1$, and thus it is also divisible by their least common multiple $L$. So $m-n=q L$ for some positive integer $q$; hence $x_{k}=q \\cdot \\frac{L}{n+k}+1$. Then, since $n+1 \\leqslant 2^{c}=n+\\ell \\leqslant 2 n+1 \\leqslant 2^{c+1}-1$, we have $2^{c} \\mid L$, but $2^{c+1} \\nmid L$. So $\\frac{L}{n+\\ell}$ is odd, while $\\frac{L}{n+k}$ is even for $k \\neq \\ell$. Computing modulo $2 q$ yields $$ x_{1} x_{2} \\cdots x_{n+1}-1 \\equiv(q+1) \\cdot 1^{n}-1 \\equiv q \\quad(\\bmod 2 q) $$ Thus, $x_{1} x_{2} \\cdots x_{n+1}-1=2 q r+q=q(2 r+1)$ for some integer $r$. Since $x_{1} x_{2} \\cdots x_{n+1}-1 \\geqslant x_{1} x_{2}-1 \\geqslant(q+1)^{2}-1>q$, we have $r \\geqslant 1$. This implies that $x_{1} x_{2} \\cdots x_{n+1}-1$ is divisible by an odd prime.","tier":0} +{"problem_type":"Number Theory","problem_label":"N4","problem":"Suppose that $a_{0}, a_{1}, \\ldots$ and $b_{0}, b_{1}, \\ldots$ are two sequences of positive integers satisfying $a_{0}, b_{0} \\geqslant 2$ and $$ a_{n+1}=\\operatorname{gcd}\\left(a_{n}, b_{n}\\right)+1, \\quad b_{n+1}=\\operatorname{lcm}\\left(a_{n}, b_{n}\\right)-1 $$ for all $n \\geqslant 0$. Prove that the sequence $\\left(a_{n}\\right)$ is eventually periodic; in other words, there exist integers $N \\geqslant 0$ and $t>0$ such that $a_{n+t}=a_{n}$ for all $n \\geqslant N$. (France)","solution":"Let $s_{n}=a_{n}+b_{n}$. Notice that if $a_{n} \\mid b_{n}$, then $a_{n+1}=a_{n}+1, b_{n+1}=b_{n}-1$ and $s_{n+1}=s_{n}$. So, $a_{n}$ increases by 1 and $s_{n}$ does not change until the first index is reached with $a_{n} \\nmid s_{n}$. Define $$ W_{n}=\\left\\{m \\in \\mathbb{Z}_{>0}: m \\geqslant a_{n} \\text { and } m \\nmid s_{n}\\right\\} \\quad \\text { and } \\quad w_{n}=\\min W_{n} $$ Claim 1. The sequence $\\left(w_{n}\\right)$ is non-increasing. Proof. If $a_{n} \\mid b_{n}$ then $a_{n+1}=a_{n}+1$. Due to $a_{n} \\mid s_{n}$, we have $a_{n} \\notin W_{n}$. Moreover $s_{n+1}=s_{n}$; therefore, $W_{n+1}=W_{n}$ and $w_{n+1}=w_{n}$. Otherwise, if $a_{n} \\nmid b_{n}$, then $a_{n} \\nmid s_{n}$, so $a_{n} \\in W_{n}$ and thus $w_{n}=a_{n}$. We show that $a_{n} \\in W_{n+1}$; this implies $w_{n+1} \\leqslant a_{n}=w_{n}$. By the definition of $W_{n+1}$, we need that $a_{n} \\geqslant a_{n+1}$ and $a_{n} \\nmid s_{n+1}$. The first relation holds because of $\\operatorname{gcd}\\left(a_{n}, b_{n}\\right)0$ such that $a_{n+t}=a_{n}$ for all $n \\geqslant N$. (France)","solution":"By Claim 1 in the first solution, we have $a_{n} \\leqslant w_{n} \\leqslant w_{0}$, so the sequence $\\left(a_{n}\\right)$ is bounded, and hence it has only finitely many values. Let $M=\\operatorname{lcm}\\left(a_{1}, a_{2}, \\ldots\\right)$, and consider the sequence $b_{n}$ modulo $M$. Let $r_{n}$ be the remainder of $b_{n}$, divided by $M$. For every index $n$, since $a_{n}|M| b_{n}-r_{n}$, we have $\\operatorname{gcd}\\left(a_{n}, b_{n}\\right)=\\operatorname{gcd}\\left(a_{n}, r_{n}\\right)$, and therefore $$ a_{n+1}=\\operatorname{gcd}\\left(a_{n}, r_{n}\\right)+1 $$ Moreover, $$ \\begin{aligned} r_{n+1} & \\equiv b_{n+1}=\\operatorname{lcm}\\left(a_{n}, b_{n}\\right)-1=\\frac{a_{n}}{\\operatorname{gcd}\\left(a_{n}, b_{n}\\right)} b_{n}-1 \\\\ & =\\frac{a_{n}}{\\operatorname{gcd}\\left(a_{n}, r_{n}\\right)} b_{n}-1 \\equiv \\frac{a_{n}}{\\operatorname{gcd}\\left(a_{n}, r_{n}\\right)} r_{n}-1 \\quad(\\bmod M) \\end{aligned} $$ Hence, the pair $\\left(a_{n}, r_{n}\\right)$ uniquely determines the pair $\\left(a_{n+1}, r_{n+1}\\right)$. Since there are finitely many possible pairs, the sequence of pairs $\\left(a_{n}, r_{n}\\right)$ is eventually periodic; in particular, the sequence $\\left(a_{n}\\right)$ is eventually periodic. Comment. We show that there are only four possibilities for $g$ and $w$ (as defined in Solution 1), namely $$ (w, g) \\in\\{(2,1),(3,1),(4,2),(5,1)\\} $$ This means that the sequence $\\left(a_{n}\\right)$ eventually repeats one of the following cycles: $$ (2), \\quad(2,3), \\quad(3,4), \\quad \\text { or } \\quad(2,3,4,5) $$ Using the notation of Solution 1 , for $n \\geqslant N$ the sequence $\\left(a_{n}\\right)$ has a cycle $(g+1, g+2, \\ldots, w)$ such that $g=\\operatorname{gcd}\\left(w, s_{n}\\right)$. By the observations in the proof of Claim 2, the numbers $g+1, \\ldots, w-1$ all divide $s_{n}$; so the number $L=\\operatorname{lcm}(g+1, g+2, \\ldots, w-1)$ also divides $s_{n}$. Moreover, $g$ also divides $w$. Now choose any $n \\geqslant N$ such that $a_{n}=w$. By (1), we have $$ s_{n+1}=g+\\frac{w\\left(s_{n}-w\\right)}{g}=s_{n} \\cdot \\frac{w}{g}-\\frac{w^{2}-g^{2}}{g} . $$ Since $L$ divides both $s_{n}$ and $s_{n+1}$, it also divides the number $T=\\frac{w^{2}-g^{2}}{g}$. Suppose first that $w \\geqslant 6$, which yields $g+1 \\leqslant \\frac{w}{2}+1 \\leqslant w-2$. Then $(w-2)(w-1)|L| T$, so we have either $w^{2}-g^{2} \\geqslant 2(w-1)(w-2)$, or $g=1$ and $w^{2}-g^{2}=(w-1)(w-2)$. In the former case we get $(w-1)(w-5)+\\left(g^{2}-1\\right) \\leqslant 0$ which is false by our assumption. The latter equation rewrites as $3 w=3$, so $w=1$, which is also impossible. Now we are left with the cases when $w \\leqslant 5$ and $g \\mid w$. The case $(w, g)=(4,1)$ violates the condition $L \\left\\lvert\\, \\frac{w^{2}-g^{2}}{g}\\right.$; all other such pairs are listed in (2). In the table below, for each pair $(w, g)$, we provide possible sequences $\\left(a_{n}\\right)$ and $\\left(b_{n}\\right)$. That shows that the cycles shown in (3) are indeed possible. $$ \\begin{array}{llll} w=2 & g=1 & a_{n}=2 & b_{n}=2 \\cdot 2^{n}+1 \\\\ w=3 & g=1 & \\left(a_{2 k}, a_{2 k+1}\\right)=(2,3) & \\left(b_{2 k}, b_{2 k+1}\\right)=\\left(6 \\cdot 3^{k}+2,6 \\cdot 3^{k}+1\\right) \\\\ w=4 & g=2 & \\left(a_{2 k}, a_{2 k+1}\\right)=(3,4) & \\left(b_{2 k}, b_{2 k+1}\\right)=\\left(12 \\cdot 2^{k}+3,12 \\cdot 2^{k}+2\\right) \\\\ w=5 & g=1 & \\left(a_{4 k}, \\ldots, a_{4 k+3}\\right)=(2,3,4,5) & \\left(b_{4 k}, \\ldots, b_{4 k+3}\\right)=\\left(6 \\cdot 5^{k}+4, \\ldots, 6 \\cdot 5^{k}+1\\right) \\end{array} $$ $$ b_{n}=2 \\cdot 2^{n}+1 $$","tier":0} +{"problem_type":"Number Theory","problem_label":"N5","problem":"Determine all triples $(a, b, c)$ of positive integers for which $a b-c, b c-a$, and $c a-b$ are powers of 2 . Explanation: A power of 2 is an integer of the form $2^{n}$, where $n$ denotes some nonnegative integer. (Serbia)","solution":"It can easily be verified that these sixteen triples are as required. Now let ( $a, b, c$ ) be any triple with the desired property. If we would have $a=1$, then both $b-c$ and $c-b$ were powers of 2 , which is impossible since their sum is zero; because of symmetry, this argument shows $a, b, c \\geqslant 2$. Case 1. Among $a, b$, and $c$ there are at least two equal numbers. Without loss of generality we may suppose that $a=b$. Then $a^{2}-c$ and $a(c-1)$ are powers of 2 . The latter tells us that actually $a$ and $c-1$ are powers of 2 . So there are nonnegative integers $\\alpha$ and $\\gamma$ with $a=2^{\\alpha}$ and $c=2^{\\gamma}+1$. Since $a^{2}-c=2^{2 \\alpha}-2^{\\gamma}-1$ is a power of 2 and thus incongruent to -1 modulo 4 , we must have $\\gamma \\leqslant 1$. Moreover, each of the terms $2^{2 \\alpha}-2$ and $2^{2 \\alpha}-3$ can only be a power of 2 if $\\alpha=1$. It follows that the triple $(a, b, c)$ is either $(2,2,2)$ or $(2,2,3)$. Case 2. The numbers $a, b$, and $c$ are distinct. Due to symmetry we may suppose that $$ 2 \\leqslant a\\beta>\\gamma . $$ Depending on how large $a$ is, we divide the argument into two further cases. Case 2.1. $\\quad a=2$. We first prove that $\\gamma=0$. Assume for the sake of contradiction that $\\gamma>0$. Then $c$ is even by (4) and, similarly, $b$ is even by (5) and (3). So the left-hand side of (2) is congruent to 2 modulo 4 , which is only possible if $b c=4$. As this contradicts (1), we have thereby shown that $\\gamma=0$, i.e., that $c=2 b-1$. Now (3) yields $3 b-2=2^{\\beta}$. Due to $b>2$ this is only possible if $\\beta \\geqslant 4$. If $\\beta=4$, then we get $b=6$ and $c=2 \\cdot 6-1=11$, which is a solution. It remains to deal with the case $\\beta \\geqslant 5$. Now (2) implies $$ 9 \\cdot 2^{\\alpha}=9 b(2 b-1)-18=(3 b-2)(6 b+1)-16=2^{\\beta}\\left(2^{\\beta+1}+5\\right)-16 $$ and by $\\beta \\geqslant 5$ the right-hand side is not divisible by 32 . Thus $\\alpha \\leqslant 4$ and we get a contradiction to (5). Case 2.2. $\\quad a \\geqslant 3$. Pick an integer $\\vartheta \\in\\{-1,+1\\}$ such that $c-\\vartheta$ is not divisible by 4 . Now $$ 2^{\\alpha}+\\vartheta \\cdot 2^{\\beta}=\\left(b c-a \\vartheta^{2}\\right)+\\vartheta(c a-b)=(b+a \\vartheta)(c-\\vartheta) $$ is divisible by $2^{\\beta}$ and, consequently, $b+a \\vartheta$ is divisible by $2^{\\beta-1}$. On the other hand, $2^{\\beta}=a c-b>$ $(a-1) c \\geqslant 2 c$ implies in view of (1) that $a$ and $b$ are smaller than $2^{\\beta-1}$. All this is only possible if $\\vartheta=1$ and $a+b=2^{\\beta-1}$. Now (3) yields $$ a c-b=2(a+b) $$ whence $4 b>a+3 b=a(c-1) \\geqslant a b$, which in turn yields $a=3$. So (6) simplifies to $c=b+2$ and (2) tells us that $b(b+2)-3=(b-1)(b+3)$ is a power of 2 . Consequently, the factors $b-1$ and $b+3$ are powers of 2 themselves. Since their difference is 4 , this is only possible if $b=5$ and thus $c=7$. Thereby the solution is complete.","tier":0} +{"problem_type":"Number Theory","problem_label":"N5","problem":"Determine all triples $(a, b, c)$ of positive integers for which $a b-c, b c-a$, and $c a-b$ are powers of 2 . Explanation: A power of 2 is an integer of the form $2^{n}$, where $n$ denotes some nonnegative integer. (Serbia)","solution":"As in the beginning of the first solution, we observe that $a, b, c \\geqslant 2$. Depending on the parities of $a, b$, and $c$ we distinguish three cases. Case 1. The numbers $a, b$, and $c$ are even. Let $2^{A}, 2^{B}$, and $2^{C}$ be the largest powers of 2 dividing $a, b$, and $c$ respectively. We may assume without loss of generality that $1 \\leqslant A \\leqslant B \\leqslant C$. Now $2^{B}$ is the highest power of 2 dividing $a c-b$, whence $a c-b=2^{B} \\leqslant b$. Similarly, we deduce $b c-a=2^{A} \\leqslant a$. Adding both estimates we get $(a+b) c \\leqslant 2(a+b)$, whence $c \\leqslant 2$. So $c=2$ and thus $A=B=C=1$; moreover, we must have had equality throughout, i.e., $a=2^{A}=2$ and $b=2^{B}=2$. We have thereby found the solution $(a, b, c)=(2,2,2)$. Case 2. The numbers $a, b$, and $c$ are odd. If any two of these numbers are equal, say $a=b$, then $a c-b=a(c-1)$ has a nontrivial odd divisor and cannot be a power of 2 . Hence $a, b$, and $c$ are distinct. So we may assume without loss of generality that $a\\beta$, and thus $2^{\\beta}$ divides $$ a \\cdot 2^{\\alpha}-b \\cdot 2^{\\beta}=a(b c-a)-b(a c-b)=b^{2}-a^{2}=(b+a)(b-a) $$ Since $a$ is odd, it is not possible that both factors $b+a$ and $b-a$ are divisible by 4 . Consequently, one of them has to be a multiple of $2^{\\beta-1}$. Hence one of the numbers $2(b+a)$ and $2(b-a)$ is divisible by $2^{\\beta}$ and in either case we have $$ a c-b=2^{\\beta} \\leqslant 2(a+b) $$ This in turn yields $(a-1) b\\beta$ denote the integers satisfying $$ 2^{\\alpha}=b c-a \\quad \\text { and } \\quad 2^{\\beta}=a c-b $$ If $\\beta=0$ it would follow that $a c-b=a b-c=1$ and hence that $b=c=1$, which is absurd. So $\\beta$ and $\\alpha$ are positive and consequently $a$ and $b$ are even. Substituting $c=a b-1$ into (9) we obtain $$ \\begin{aligned} 2^{\\alpha} & =a b^{2}-(a+b), \\\\ \\text { and } \\quad 2^{\\beta} & =a^{2} b-(a+b) . \\end{aligned} $$ The addition of both equation yields $2^{\\alpha}+2^{\\beta}=(a b-2)(a+b)$. Now $a b-2$ is even but not divisible by 4 , so the highest power of 2 dividing $a+b$ is $2^{\\beta-1}$. For this reason, the equations (10) and (11) show that the highest powers of 2 dividing either of the numbers $a b^{2}$ and $a^{2} b$ is likewise $2^{\\beta-1}$. Thus there is an integer $\\tau \\geqslant 1$ together with odd integers $A, B$, and $C$ such that $a=2^{\\tau} A, b=2^{\\tau} B, a+b=2^{3 \\tau} C$, and $\\beta=1+3 \\tau$. Notice that $A+B=2^{2 \\tau} C \\geqslant 4 C$. Moreover, (11) entails $A^{2} B-C=2$. Thus $8=$ $4 A^{2} B-4 C \\geqslant 4 A^{2} B-A-B \\geqslant A^{2}(3 B-1)$. Since $A$ and $B$ are odd with $A0$, this may be weakened to $p^{2} q+p-q \\leqslant p+q$. Hence $p^{2} q \\leqslant 2 q$, which is only possible if $p=1$. Going back to (13), we get $$ \\left(d^{2} q-q-1\\right) \\mid d^{2}(q-1) $$ Now $2\\left(d^{2} q-q-1\\right) \\leqslant d^{2}(q-1)$ would entail $d^{2}(q+1) \\leqslant 2(q+1)$ and thus $d=1$. But this would tell us that $a=d p=1$, which is absurd. This argument proves $2\\left(d^{2} q-q-1\\right)>d^{2}(q-1)$ and in the light of (14) it follows that $d^{2} q-q-1=d^{2}(q-1)$, i.e., $q=d^{2}-1$. Plugging this together with $p=1$ into (12) we infer $2^{\\beta}=d^{3}\\left(d^{2}-2\\right)$. Hence $d$ and $d^{2}-2$ are powers of 2 . Consequently, $d=2, q=3$, $a=2, b=6$, and $c=11$, as desired.","tier":0} +{"problem_type":"Number Theory","problem_label":"N6","problem":"Let $\\mathbb{Z}_{>0}$ denote the set of positive integers. Consider a function $f: \\mathbb{Z}_{>0} \\rightarrow \\mathbb{Z}_{>0}$. For any $m, n \\in \\mathbb{Z}_{>0}$ we write $f^{n}(m)=\\underbrace{f(f(\\ldots f}_{n}(m) \\ldots))$. Suppose that $f$ has the following two properties: (i) If $m, n \\in \\mathbb{Z}_{>0}$, then $\\frac{f^{n}(m)-m}{n} \\in \\mathbb{Z}_{>0}$; (ii) The set $\\mathbb{Z}_{>0} \\backslash\\left\\{f(n) \\mid n \\in \\mathbb{Z}_{>0}\\right\\}$ is finite. Prove that the sequence $f(1)-1, f(2)-2, f(3)-3, \\ldots$ is periodic. (Singapore)","solution":"We split the solution into three steps. In the first of them, we show that the function $f$ is injective and explain how this leads to a useful visualization of $f$. Then comes the second step, in which most of the work happens: its goal is to show that for any $n \\in \\mathbb{Z}_{>0}$ the sequence $n, f(n), f^{2}(n), \\ldots$ is an arithmetic progression. Finally, in the third step we put everything together, thus solving the problem. $\\underline{\\text { Step 1. We commence by checking that } f \\text { is injective. For this purpose, we consider any }}$ $m, k \\in \\mathbb{Z}_{>0}$ with $f(m)=f(k)$. By $(i)$, every positive integer $n$ has the property that $$ \\frac{k-m}{n}=\\frac{f^{n}(m)-m}{n}-\\frac{f^{n}(k)-k}{n} $$ is a difference of two integers and thus integral as well. But for $n=|k-m|+1$ this is only possible if $k=m$. Thereby, the injectivity of $f$ is established. Now recall that due to condition (ii) there are finitely many positive integers $a_{1}, \\ldots, a_{k}$ such that $\\mathbb{Z}_{>0}$ is the disjoint union of $\\left\\{a_{1}, \\ldots, a_{k}\\right\\}$ and $\\left\\{f(n) \\mid n \\in \\mathbb{Z}_{>0}\\right\\}$. Notice that by plugging $n=1$ into condition $(i)$ we get $f(m)>m$ for all $m \\in \\mathbb{Z}_{>0}$. We contend that every positive integer $n$ may be expressed uniquely in the form $n=f^{j}\\left(a_{i}\\right)$ for some $j \\geqslant 0$ and $i \\in\\{1, \\ldots, k\\}$. The uniqueness follows from the injectivity of $f$. The existence can be proved by induction on $n$ in the following way. If $n \\in\\left\\{a_{1}, \\ldots, a_{k}\\right\\}$, then we may take $j=0$; otherwise there is some $n^{\\prime}0$; and $T=1$ and $A=0$ if $t=0$. For every integer $n \\geqslant A$, the interval $\\Delta_{n}=[n+1, n+T]$ contains exactly $T \/ T_{i}$ elements of the $i^{\\text {th }}$ row $(1 \\leqslant i \\leqslant t)$. Therefore, the number of elements from the last $(k-t)$ rows of the Table contained in $\\Delta_{n}$ does not depend on $n \\geqslant A$. It is not possible that none of these intervals $\\Delta_{n}$ contains an element from the $k-t$ last rows, because infinitely many numbers appear in these rows. It follows that for each $n \\geqslant A$ the interval $\\Delta_{n}$ contains at least one member from these rows. This yields that for every positive integer $d$, the interval $[A+1, A+(d+1)(k-t) T]$ contains at least $(d+1)(k-t)$ elements from the last $k-t$ rows; therefore, there exists an index $x$ with $t+1 \\leqslant x \\leqslant k$, possibly depending on $d$, such that our interval contains at least $d+1$ elements from the $x^{\\text {th }}$ row. In this situation we have $$ f^{d}\\left(a_{x}\\right) \\leqslant A+(d+1)(k-t) T . $$ Finally, since there are finitely many possibilities for $x$, there exists an index $x \\geqslant t+1$ such that the set $$ X=\\left\\{d \\in \\mathbb{Z}_{>0} \\mid f^{d}\\left(a_{x}\\right) \\leqslant A+(d+1)(k-t) T\\right\\} $$ is infinite. Thereby we have found the \"dense row\" promised above. By assumption $(i)$, for every $d \\in X$ the number $$ \\beta_{d}=\\frac{f^{d}\\left(a_{x}\\right)-a_{x}}{d} $$ is a positive integer not exceeding $$ \\frac{A+(d+1)(k-t) T}{d} \\leqslant \\frac{A d+2 d(k-t) T}{d}=A+2(k-t) T $$ This leaves us with finitely many choices for $\\beta_{d}$, which means that there exists a number $T_{x}$ such that the set $$ Y=\\left\\{d \\in X \\mid \\beta_{d}=T_{x}\\right\\} $$ is infinite. Notice that we have $f^{d}\\left(a_{x}\\right)=a_{x}+d \\cdot T_{x}$ for all $d \\in Y$. Now we are prepared to prove that the numbers in the $x^{\\text {th }}$ row form an arithmetic progression, thus coming to a contradiction with our assumption. Let us fix any positive integer $j$. Since the set $Y$ is infinite, we can choose a number $y \\in Y$ such that $y-j>\\left|f^{j}\\left(a_{x}\\right)-\\left(a_{x}+j T_{x}\\right)\\right|$. Notice that both numbers $$ f^{y}\\left(a_{x}\\right)-f^{j}\\left(a_{x}\\right)=f^{y-j}\\left(f^{j}\\left(a_{x}\\right)\\right)-f^{j}\\left(a_{x}\\right) \\quad \\text { and } \\quad f^{y}\\left(a_{x}\\right)-\\left(a_{x}+j T_{x}\\right)=(y-j) T_{x} $$ are divisible by $y-j$. Thus, the difference between these numbers is also divisible by $y-j$. Since the absolute value of this difference is less than $y-j$, it has to vanish, so we get $f^{j}\\left(a_{x}\\right)=$ $a_{x}+j \\cdot T_{x}$. Hence, it is indeed true that all rows of the Table are arithmetic progressions. Step 3. Keeping the above notation in force, we denote the step of the $i^{\\text {th }}$ row of the table by $T_{i}$. Now we claim that we have $f(n)-n=f(n+T)-(n+T)$ for all $n \\in \\mathbb{Z}_{>0}$, where $$ T=\\operatorname{lcm}\\left(T_{1}, \\ldots, T_{k}\\right) $$ To see this, let any $n \\in \\mathbb{Z}_{>0}$ be given and denote the index of the row in which it appears in the Table by $i$. Then we have $f^{j}(n)=n+j \\cdot T_{i}$ for all $j \\in \\mathbb{Z}_{>0}$, and thus indeed $$ f(n+T)-f(n)=f^{1+T \/ T_{i}}(n)-f(n)=\\left(n+T+T_{i}\\right)-\\left(n+T_{i}\\right)=T $$ This concludes the solution. Comment 1. There are some alternative ways to complete the second part once the index $x$ corresponding to a \"dense row\" is found. For instance, one may show that for some integer $T_{x}^{*}$ the set $$ Y^{*}=\\left\\{j \\in \\mathbb{Z}_{>0} \\mid f^{j+1}\\left(a_{x}\\right)-f^{j}\\left(a_{x}\\right)=T_{x}^{*}\\right\\} $$ is infinite, and then one may conclude with a similar divisibility argument. Comment 2. It may be checked that, conversely, any way to fill out the Table with finitely many arithmetic progressions so that each positive integer appears exactly once, gives rise to a function $f$ satisfying the two conditions mentioned in the problem. For example, we may arrange the positive integers as follows: | 2 | 4 | 6 | 8 | 10 | $\\ldots$ | | :---: | :---: | :---: | :---: | :---: | :---: | | 1 | 5 | 9 | 13 | 17 | $\\ldots$ | | 3 | 7 | 11 | 15 | 19 | $\\ldots$ | This corresponds to the function $$ f(n)= \\begin{cases}n+2 & \\text { if } n \\text { is even } \\\\ n+4 & \\text { if } n \\text { is odd }\\end{cases} $$ As this example shows, it is not true that the function $n \\mapsto f(n)-n$ has to be constant.","tier":0} +{"problem_type":"Number Theory","problem_label":"N7","problem":"Let $\\mathbb{Z}_{>0}$ denote the set of positive integers. For any positive integer $k$, a function $f: \\mathbb{Z}_{>0} \\rightarrow \\mathbb{Z}_{>0}$ is called $k$-good if $\\operatorname{gcd}(f(m)+n, f(n)+m) \\leqslant k$ for all $m \\neq n$. Find all $k$ such that there exists a $k$-good function. (Canada)","solution":"For any function $f: \\mathbb{Z}_{>0} \\rightarrow \\mathbb{Z}_{>0}$, let $G_{f}(m, n)=\\operatorname{gcd}(f(m)+n, f(n)+m)$. Note that a $k$-good function is also $(k+1)$-good for any positive integer $k$. Hence, it suffices to show that there does not exist a 1-good function and that there exists a 2-good function. We first show that there is no 1 -good function. Suppose that there exists a function $f$ such that $G_{f}(m, n)=1$ for all $m \\neq n$. Now, if there are two distinct even numbers $m$ and $n$ such that $f(m)$ and $f(n)$ are both even, then $2 \\mid G_{f}(m, n)$, a contradiction. A similar argument holds if there are two distinct odd numbers $m$ and $n$ such that $f(m)$ and $f(n)$ are both odd. Hence we can choose an even $m$ and an odd $n$ such that $f(m)$ is odd and $f(n)$ is even. This also implies that $2 \\mid G_{f}(m, n)$, a contradiction. We now construct a 2-good function. Define $f(n)=2^{g(n)+1}-n-1$, where $g$ is defined recursively by $g(1)=1$ and $g(n+1)=\\left(2^{g(n)+1}\\right)!$. For any positive integers $m>n$, set $$ A=f(m)+n=2^{g(m)+1}-m+n-1, \\quad B=f(n)+m=2^{g(n)+1}-n+m-1 $$ We need to show that $\\operatorname{gcd}(A, B) \\leqslant 2$. First, note that $A+B=2^{g(m)+1}+2^{g(n)+1}-2$ is not divisible by 4 , so that $4 \\nmid \\operatorname{gcd}(A, B)$. Now we suppose that there is an odd prime $p$ for which $p \\mid \\operatorname{gcd}(A, B)$ and derive a contradiction. We first claim that $2^{g(m-1)+1} \\geqslant B$. This is a rather weak bound; one way to prove it is as follows. Observe that $g(k+1)>g(k)$ and hence $2^{g(k+1)+1} \\geqslant 2^{g(k)+1}+1$ for every positive integer $k$. By repeatedly applying this inequality, we obtain $2^{g(m-1)+1} \\geqslant 2^{g(n)+1}+(m-1)-n=B$. Now, since $p \\mid B$, we have $p-10}$ denote the set of positive integers. For any positive integer $k$, a function $f: \\mathbb{Z}_{>0} \\rightarrow \\mathbb{Z}_{>0}$ is called $k$-good if $\\operatorname{gcd}(f(m)+n, f(n)+m) \\leqslant k$ for all $m \\neq n$. Find all $k$ such that there exists a $k$-good function. (Canada)","solution":"We provide an alternative construction of a 2 -good function $f$. Let $\\mathcal{P}$ be the set consisting of 4 and all odd primes. For every $p \\in \\mathcal{P}$, we say that a number $a \\in\\{0,1, \\ldots, p-1\\}$ is $p$-useful if $a \\not \\equiv-a(\\bmod p)$. Note that a residue modulo $p$ which is neither 0 nor 2 is $p$-useful (the latter is needed only when $p=4$ ). We will construct $f$ recursively; in some steps, we will also define a $p$-useful number $a_{p}$. After the $m^{\\text {th }}$ step, the construction will satisfy the following conditions: ( $i$ ) The values of $f(n)$ have already been defined for all $n \\leqslant m$, and $p$-useful numbers $a_{p}$ have already been defined for all $p \\leqslant m+2$; (ii) If $n \\leqslant m$ and $p \\leqslant m+2$, then $f(n)+n \\not \\equiv a_{p}(\\bmod p)$; (iii) $\\operatorname{gcd}\\left(f\\left(n_{1}\\right)+n_{2}, f\\left(n_{2}\\right)+n_{1}\\right) \\leqslant 2$ for all $n_{1}1$, introduce the set $X_{m}$ like in Solution 2 and define $f(m)$ so as to satisfy $$ \\begin{array}{rll} f(m) \\equiv f(m-p) & (\\bmod p) & \\text { for all } p \\in X_{m} \\text { with } p10^{100}} \\alpha_{i} . $$ That is, $\\mho(n)$ is the number of prime factors of $n$ greater than $10^{100}$, counted with multiplicity. Find all strictly increasing functions $f: \\mathbb{Z} \\rightarrow \\mathbb{Z}$ such that $$ \\mho(f(a)-f(b)) \\leqslant \\mho(a-b) \\quad \\text { for all integers } a \\text { and } b \\text { with } a>b . $$","solution":"A straightforward check shows that all the functions listed in the answer satisfy the problem condition. It remains to show the converse. Assume that $f$ is a function satisfying the problem condition. Notice that the function $g(x)=f(x)-f(0)$ also satisfies this condition. Replacing $f$ by $g$, we assume from now on that $f(0)=0$; then $f(n)>0$ for any positive integer $n$. Thus, we aim to prove that there exists a positive integer $a$ with $\\mho(a)=0$ such that $f(n)=a n$ for all $n \\in \\mathbb{Z}$. We start by introducing some notation. Set $N=10^{100}$. We say that a prime $p$ is large if $p>N$, and $p$ is small otherwise; let $\\mathcal{S}$ be the set of all small primes. Next, we say that a positive integer is large or small if all its prime factors are such (thus, the number 1 is the unique number which is both large and small). For a positive integer $k$, we denote the greatest large divisor of $k$ and the greatest small divisor of $k$ by $L(k)$ and $S(k)$, respectively; thus, $k=L(k) S(k)$. We split the proof into three steps. Step 1. We prove that for every large $k$, we have $k|f(a)-f(b) \\Longleftrightarrow k| a-b$. In other words, $L(f(a)-f(b))=L(a-b)$ for all integers $a$ and $b$ with $a>b$. We use induction on $k$. The base case $k=1$ is trivial. For the induction step, assume that $k_{0}$ is a large number, and that the statement holds for all large numbers $k$ with $k\\ell$. But then $$ \\mho(f(x)-f(y)) \\geqslant \\mho\\left(\\operatorname{lcm}\\left(k_{0}, \\ell\\right)\\right)>\\mho(\\ell)=\\mho(x-y), $$ which is impossible. Now we complete the induction step. By Claim 1, for every integer $a$ each of the sequences $$ f(a), f(a+1), \\ldots, f\\left(a+k_{0}-1\\right) \\quad \\text { and } \\quad f(a+1), f(a+2), \\ldots, f\\left(a+k_{0}\\right) $$ forms a complete residue system modulo $k_{0}$. This yields $f(a) \\equiv f\\left(a+k_{0}\\right)\\left(\\bmod k_{0}\\right)$. Thus, $f(a) \\equiv f(b)\\left(\\bmod k_{0}\\right)$ whenever $a \\equiv b\\left(\\bmod k_{0}\\right)$. Finally, if $a \\not \\equiv b\\left(\\bmod k_{0}\\right)$ then there exists an integer $b^{\\prime}$ such that $b^{\\prime} \\equiv b\\left(\\bmod k_{0}\\right)$ and $\\left|a-b^{\\prime}\\right|N$. Proof. Let $d$ be the product of all small primes, and let $\\alpha$ be a positive integer such that $2^{\\alpha}>f(N)$. Then, for every $p \\in \\mathcal{S}$ the numbers $f(0), f(1), \\ldots, f(N)$ are distinct modulo $p^{\\alpha}$. Set $P=d^{\\alpha}$ and $c=P+f(N)$. Choose any integer $t>N$. Due to the choice of $\\alpha$, for every $p \\in \\mathcal{S}$ there exists at most one nonnegative integer $i \\leqslant N$ with $p^{\\alpha} \\mid f(t)-f(i)$. Since $|\\mathcal{S}||f(x)-a x|$. Since $n-x \\equiv r-(r+1)=-1(\\bmod N!)$, the number $|n-x|$ is large. Therefore, by Step 1 we have $f(x) \\equiv f(n)=a n \\equiv a x(\\bmod n-x)$, so $n-x \\mid f(x)-a x$. Due to the choice of $n$, this yields $f(x)=a x$. To complete Step 3, notice that the set $\\mathcal{T}^{\\prime}$ found in Step 2 contains infinitely many elements of some residue class $R_{i}$. Applying Claim 3, we successively obtain that $f(x)=a x$ for all $x \\in R_{i+1}, R_{i+2}, \\ldots, R_{i+N!}=R_{i}$. This finishes the solution. Comment 1. As the proposer also mentions, one may also consider the version of the problem where the condition (1) is replaced by the condition that $L(f(a)-f(b))=L(a-b)$ for all integers $a$ and $b$ with $a>b$. This allows to remove of Step 1 from the solution. Comment 2. Step 2 is the main step of the solution. We sketch several different approaches allowing to perform this step using statements which are weaker than Claim 2. Approach 1. Let us again denote the product of all small primes by $d$. We focus on the values $f\\left(d^{i}\\right)$, $i \\geqslant 0$. In view of Step 1, we have $L\\left(f\\left(d^{i}\\right)-f\\left(d^{k}\\right)\\right)=L\\left(d^{i}-d^{k}\\right)=d^{i-k}-1$ for all $i>k \\geqslant 0$. Acting similarly to the beginning of the proof of Claim 2, one may choose a number $\\alpha \\geqslant 0$ such that the residues of the numbers $f\\left(d^{i}\\right), i=0,1, \\ldots, N$, are distinct modulo $p^{\\alpha}$ for each $p \\in \\mathcal{S}$. Then, for every $i>N$, there exists an exponent $k=k(i) \\leqslant N$ such that $S\\left(f\\left(d^{i}\\right)-f\\left(d^{k}\\right)\\right)N$ such that $k(i)$ attains the same value $k_{0}$ for all $i \\in I$, and such that, moreover, $S\\left(f\\left(d^{i}\\right)-f\\left(d^{k_{0}}\\right)\\right)$ attains the same value $s_{0}$ for all $i \\in I$. Therefore, for all such $i$ we have $$ f\\left(d^{i}\\right)=f\\left(d^{k_{0}}\\right)+L\\left(f\\left(d^{i}\\right)-f\\left(d^{k_{0}}\\right)\\right) \\cdot S\\left(f\\left(d^{i}\\right)-f\\left(d^{k_{0}}\\right)\\right)=f\\left(d^{k_{0}}\\right)+\\left(d^{i-k_{0}}-1\\right) s_{0}, $$ which means that $f$ is linear on the infinite set $\\left\\{d^{i}: i \\in I\\right\\}$ (although with rational coefficients). Finally, one may implement the relation $f\\left(d^{i}\\right) \\equiv f(1)\\left(\\bmod d^{i}-1\\right)$ in order to establish that in fact $f\\left(d^{i}\\right) \/ d^{i}$ is a (small and fixed) integer for all $i \\in I$. Approach 2. Alternatively, one may start with the following lemma. Lemma. There exists a positive constant $c$ such that $$ L\\left(\\prod_{i=1}^{3 N}(f(k)-f(i))\\right)=\\prod_{i=1}^{3 N} L(f(k)-f(i)) \\geqslant c(f(k))^{2 N} $$ for all $k>3 N$. Proof. Let $k$ be an integer with $k>3 N$. Set $\\Pi=\\prod_{i=1}^{3 N}(f(k)-f(i))$. Notice that for every prime $p \\in \\mathcal{S}$, at most one of the numbers in the set $$ \\mathcal{H}=\\{f(k)-f(i): 1 \\leqslant i \\leqslant 3 N\\} $$ is divisible by a power of $p$ which is greater than $f(3 N)$; we say that such elements of $\\mathcal{H}$ are bad. Now, for each element $h \\in \\mathcal{H}$ which is not bad we have $S(h) \\leqslant f(3 N)^{N}$, while the bad elements do not exceed $f(k)$. Moreover, there are less than $N$ bad elements in $\\mathcal{H}$. Therefore, $$ S(\\Pi)=\\prod_{h \\in \\mathcal{H}} S(h) \\leqslant(f(3 N))^{3 N^{2}} \\cdot(f(k))^{N} . $$ This easily yields the lemma statement in view of the fact that $L(\\Pi) S(\\Pi)=\\Pi \\geqslant \\mu(f(k))^{3 N}$ for some absolute constant $\\mu$. As a corollary of the lemma, one may get a weaker version of Claim 2 stating that there exists a positive constant $C$ such that $f(k) \\leqslant C k^{3 \/ 2}$ for all $k>3 N$. Indeed, from Step 1 we have $$ k^{3 N} \\geqslant \\prod_{i=1}^{3 N} L(k-i)=\\prod_{i=1}^{3 N} L(f(k)-f(i)) \\geqslant c(f(k))^{2 N} $$ so $f(k) \\leqslant c^{-1 \/(2 N)} k^{3 \/ 2}$. To complete Step 2 now, set $a=f(1)$. Due to the estimates above, we may choose a positive integer $n_{0}$ such that $|f(n)-a n|<\\frac{n(n-1)}{2}$ for all $n \\geqslant n_{0}$. Take any $n \\geqslant n_{0}$ with $n \\equiv 2(\\bmod N!)$. Then $L(f(n)-f(0))=L(n)=n \/ 2$ and $L(f(n)-f(1))=$ $L(n-1)=n-1$; these relations yield $f(n) \\equiv f(0)=0 \\equiv a n(\\bmod n \/ 2)$ and $f(n) \\equiv f(1)=a \\equiv a n$ $(\\bmod n-1)$, respectively. Thus, $\\left.\\frac{n(n-1)}{2} \\right\\rvert\\, f(n)-a n$, which shows that $f(n)=a n$ in view of the estimate above. Comment 3. In order to perform Step 3, it suffices to establish the equality $f(n)=a n$ for any infinite set of values of $n$. However, if this set has some good structure, then one may find easier ways to complete this step. For instance, after showing, as in Approach 2 , that $f(n)=$ an for all $n \\geqslant n_{0}$ with $n \\equiv 2(\\bmod N!)$, one may proceed as follows. Pick an arbitrary integer $x$ and take any large prime $p$ which is greater than $|f(x)-a x|$. By the Chinese Remainder Theorem, there exists a positive integer $n>\\max \\left(x, n_{0}\\right)$ such that $n \\equiv 2(\\bmod N!)$ and $n \\equiv x(\\bmod p)$. By Step 1 , we have $f(x) \\equiv f(n)=a n \\equiv a x(\\bmod p)$. Due to the choice of $p$, this is possible only if $f(x)=a x$.","tier":0} +{"problem_type":"Algebra","problem_label":"A2","problem":"Determine all functions $f: \\mathbb{Z} \\rightarrow \\mathbb{Z}$ with the property that $$ f(x-f(y))=f(f(x))-f(y)-1 $$ holds for all $x, y \\in \\mathbb{Z}$. (Croatia) Answer. There are two such functions, namely the constant function $x \\mapsto-1$ and the successor function $x \\mapsto x+1$.","solution":"It is immediately checked that both functions mentioned in the answer are as desired. Now let $f$ denote any function satisfying (1) for all $x, y \\in \\mathbb{Z}$. Substituting $x=0$ and $y=f(0)$ into (1) we learn that the number $z=-f(f(0))$ satisfies $f(z)=-1$. So by plugging $y=z$ into (1) we deduce that $$ f(x+1)=f(f(x)) $$ holds for all $x \\in \\mathbb{Z}$. Thereby (1) simplifies to $$ f(x-f(y))=f(x+1)-f(y)-1 $$ We now work towards showing that $f$ is linear by contemplating the difference $f(x+1)-f(x)$ for any $x \\in \\mathbb{Z}$. By applying (3) with $y=x$ and (2) in this order, we obtain $$ f(x+1)-f(x)=f(x-f(x))+1=f(f(x-1-f(x)))+1 $$ Since (3) shows $f(x-1-f(x))=f(x)-f(x)-1=-1$, this simplifies to $$ f(x+1)=f(x)+A, $$ where $A=f(-1)+1$ is some absolute constant. Now a standard induction in both directions reveals that $f$ is indeed linear and that in fact we have $f(x)=A x+B$ for all $x \\in \\mathbb{Z}$, where $B=f(0)$. Substituting this into (2) we obtain that $$ A x+(A+B)=A^{2} x+(A B+B) $$ holds for all $x \\in \\mathbb{Z}$; applying this to $x=0$ and $x=1$ we infer $A+B=A B+B$ and $A^{2}=A$. The second equation leads to $A=0$ or $A=1$. In case $A=1$, the first equation gives $B=1$, meaning that $f$ has to be the successor function. If $A=0$, then $f$ is constant and (1) shows that its constant value has to be -1 . Thereby the solution is complete. Comment. After (2) and (3) have been obtained, there are several other ways to combine them so as to obtain linearity properties of $f$. For instance, using (2) thrice in a row and then (3) with $x=f(y)$ one may deduce that $$ f(y+2)=f(f(y+1))=f(f(f(y)))=f(f(y)+1)=f(y)+f(0)+1 $$ holds for all $y \\in \\mathbb{Z}$. It follows that $f$ behaves linearly on the even numbers and on the odd numbers separately, and moreover that the slopes of these two linear functions coincide. From this point, one may complete the solution with some straightforward case analysis. A different approach using the equations (2) and (3) will be presented in Solution 2. To show that it is also possible to start in a completely different way, we will also present a third solution that avoids these equations entirely.","tier":0} +{"problem_type":"Algebra","problem_label":"A2","problem":"Determine all functions $f: \\mathbb{Z} \\rightarrow \\mathbb{Z}$ with the property that $$ f(x-f(y))=f(f(x))-f(y)-1 $$ holds for all $x, y \\in \\mathbb{Z}$. (Croatia) Answer. There are two such functions, namely the constant function $x \\mapsto-1$ and the successor function $x \\mapsto x+1$.","solution":"We commence by deriving (2) and (3) as in the first solution. Now provided that $f$ is injective, (2) tells us that $f$ is the successor function. Thus we may assume from now on that $f$ is not injective, i.e., that there are two integers $a>b$ with $f(a)=f(b)$. A straightforward induction using (2) in the induction step reveals that we have $f(a+n)=f(b+n)$ for all nonnegative integers $n$. Consequently, the sequence $\\gamma_{n}=f(b+n)$ is periodic and thus in particular bounded, which means that the numbers $$ \\varphi=\\min _{n \\geqslant 0} \\gamma_{n} \\quad \\text { and } \\quad \\psi=\\max _{n \\geqslant 0} \\gamma_{n} $$ exist. Let us pick any integer $y$ with $f(y)=\\varphi$ and then an integer $x \\geqslant a$ with $f(x-f(y))=\\varphi$. Due to the definition of $\\varphi$ and (3) we have $$ \\varphi \\leqslant f(x+1)=f(x-f(y))+f(y)+1=2 \\varphi+1 $$ whence $\\varphi \\geqslant-1$. The same reasoning applied to $\\psi$ yields $\\psi \\leqslant-1$. Since $\\varphi \\leqslant \\psi$ holds trivially, it follows that $\\varphi=\\psi=-1$, or in other words that we have $f(t)=-1$ for all integers $t \\geqslant a$. Finally, if any integer $y$ is given, we may find an integer $x$ which is so large that $x+1 \\geqslant a$ and $x-f(y) \\geqslant a$ hold. Due to (3) and the result from the previous paragraph we get $$ f(y)=f(x+1)-f(x-f(y))-1=(-1)-(-1)-1=-1 . $$ Thereby the problem is solved.","tier":0} +{"problem_type":"Algebra","problem_label":"A2","problem":"Determine all functions $f: \\mathbb{Z} \\rightarrow \\mathbb{Z}$ with the property that $$ f(x-f(y))=f(f(x))-f(y)-1 $$ holds for all $x, y \\in \\mathbb{Z}$. (Croatia) Answer. There are two such functions, namely the constant function $x \\mapsto-1$ and the successor function $x \\mapsto x+1$.","solution":"Set $d=f(0)$. By plugging $x=f(y)$ into (1) we obtain $$ f^{3}(y)=f(y)+d+1 $$ for all $y \\in \\mathbb{Z}$, where the left-hand side abbreviates $f(f(f(y)))$. When we replace $x$ in (1) by $f(x)$ we obtain $f(f(x)-f(y))=f^{3}(x)-f(y)-1$ and as a consequence of (4) this simplifies to $$ f(f(x)-f(y))=f(x)-f(y)+d $$ Now we consider the set $$ E=\\{f(x)-d \\mid x \\in \\mathbb{Z}\\} $$ Given two integers $a$ and $b$ from $E$, we may pick some integers $x$ and $y$ with $f(x)=a+d$ and $f(y)=b+d$; now (5) tells us that $f(a-b)=(a-b)+d$, which means that $a-b$ itself exemplifies $a-b \\in E$. Thus, $$ E \\text { is closed under taking differences. } $$ Also, the definitions of $d$ and $E$ yield $0 \\in E$. If $E=\\{0\\}$, then $f$ is a constant function and (1) implies that the only value attained by $f$ is indeed -1 . So let us henceforth suppose that $E$ contains some number besides zero. It is known that in this case (6) entails $E$ to be the set of all integer multiples of some positive integer $k$. Indeed, this holds for $$ k=\\min \\{|x| \\mid x \\in E \\text { and } x \\neq 0\\} $$ as one may verify by an argument based on division with remainder. Thus we have $$ \\{f(x) \\mid x \\in \\mathbb{Z}\\}=\\{k \\cdot t+d \\mid t \\in \\mathbb{Z}\\} $$ Due to (5) and (7) we get $$ f(k \\cdot t)=k \\cdot t+d $$ for all $t \\in \\mathbb{Z}$, whence in particular $f(k)=k+d$. So by comparing the results of substituting $y=0$ and $y=k$ into (1) we learn that $$ f(z+k)=f(z)+k $$ holds for all integers $z$. In plain English, this means that on any residue class modulo $k$ the function $f$ is linear with slope 1. Now by (7) the set of all values attained by $f$ is such a residue class. Hence, there exists an absolute constant $c$ such that $f(f(x))=f(x)+c$ holds for all $x \\in \\mathbb{Z}$. Thereby (1) simplifies to $$ f(x-f(y))=f(x)-f(y)+c-1 $$ On the other hand, considering (1) modulo $k$ we obtain $d \\equiv-1(\\bmod k)$ because of (7). So by (7) again, $f$ attains the value -1 . Thus we may apply (9) to some integer $y$ with $f(y)=-1$, which gives $f(x+1)=f(x)+c$. So $f$ is a linear function with slope $c$. Hence, (8) leads to $c=1$, wherefore there is an absolute constant $d^{\\prime}$ with $f(x)=x+d^{\\prime}$ for all $x \\in \\mathbb{Z}$. Using this for $x=0$ we obtain $d^{\\prime}=d$ and finally (4) discloses $d=1$, meaning that $f$ is indeed the successor function.","tier":0} +{"problem_type":"Algebra","problem_label":"A3","problem":"Let $n$ be a fixed positive integer. Find the maximum possible value of $$ \\sum_{1 \\leqslant r1$ is odd. One can modify the arguments of the last part in order to work for every (not necessarily odd) sufficiently large value of $k$; namely, when $k$ is even, one may show that the sequence $P(1), P(2), \\ldots, P(k)$ has different numbers of positive and negative terms. On the other hand, the problem statement with $k$ replaced by 2 is false, since the polynomials $P(x)=T(x)-T(x-1)$ and $Q(x)=T(x-1)-T(x)$ are block-similar in this case, due to the fact that $P(2 i-1)=-P(2 i)=Q(2 i)=-Q(2 i-1)=T(2 i-1)$ for all $i=1,2, \\ldots, n$. Thus, every complete solution should use the relation $k>2$. One may easily see that the condition $n \\geqslant 2$ is also substantial, since the polynomials $x$ and $k+1-x$ become block-similar if we set $n=1$. It is easily seen from the solution that the result still holds if we assume that the polynomials have degree at most $n$.","tier":0} +{"problem_type":"Step 3. We describe all functions $f$.","problem_label":"A6","problem":"Let $n$ be a fixed integer with $n \\geqslant 2$. We say that two polynomials $P$ and $Q$ with real coefficients are block-similar if for each $i \\in\\{1,2, \\ldots, n\\}$ the sequences $$ \\begin{aligned} & P(2015 i), P(2015 i-1), \\ldots, P(2015 i-2014) \\quad \\text { and } \\\\ & Q(2015 i), Q(2015 i-1), \\ldots, Q(2015 i-2014) \\end{aligned} $$ are permutations of each other. (a) Prove that there exist distinct block-similar polynomials of degree $n+1$. (b) Prove that there do not exist distinct block-similar polynomials of degree $n$. (Canada)","solution":"We provide an alternative argument for part (b). Assume again that there exist two distinct block-similar polynomials $P(x)$ and $Q(x)$ of degree $n$. Let $R(x)=P(x)-Q(x)$ and $S(x)=P(x)+Q(x)$. For brevity, we also denote the segment $[(i-1) k+1, i k]$ by $I_{i}$, and the set $\\{(i-1) k+1,(i-1) k+2, \\ldots, i k\\}$ of all integer points in $I_{i}$ by $Z_{i}$. Step 1. We prove that $R(x)$ has exactly one root in each segment $I_{i}, i=1,2, \\ldots, n$, and all these roots are simple. Indeed, take any $i \\in\\{1,2, \\ldots, n\\}$ and choose some points $p^{-}, p^{+} \\in Z_{i}$ so that $$ P\\left(p^{-}\\right)=\\min _{x \\in Z_{i}} P(x) \\quad \\text { and } \\quad P\\left(p^{+}\\right)=\\max _{x \\in Z_{i}} P(x) $$ Since the sequences of values of $P$ and $Q$ in $Z_{i}$ are permutations of each other, we have $R\\left(p^{-}\\right)=P\\left(p^{-}\\right)-Q\\left(p^{-}\\right) \\leqslant 0$ and $R\\left(p^{+}\\right)=P\\left(p^{+}\\right)-Q\\left(p^{+}\\right) \\geqslant 0$. Since $R(x)$ is continuous, there exists at least one root of $R(x)$ between $p^{-}$and $p^{+}$- thus in $I_{i}$. So, $R(x)$ has at least one root in each of the $n$ disjoint segments $I_{i}$ with $i=1,2, \\ldots, n$. Since $R(x)$ is nonzero and its degree does not exceed $n$, it should have exactly one root in each of these segments, and all these roots are simple, as required.","tier":0} +{"problem_type":"Combinatorics","problem_label":"C2","problem":"Let $\\mathcal{V}$ be a finite set of points in the plane. We say that $\\mathcal{V}$ is balanced if for any two distinct points $A, B \\in \\mathcal{V}$, there exists a point $C \\in \\mathcal{V}$ such that $A C=B C$. We say that $\\mathcal{V}$ is center-free if for any distinct points $A, B, C \\in \\mathcal{V}$, there does not exist a point $P \\in \\mathcal{V}$ such that $P A=P B=P C$. (a) Show that for all $n \\geqslant 3$, there exists a balanced set consisting of $n$ points. (b) For which $n \\geqslant 3$ does there exist a balanced, center-free set consisting of $n$ points? (Netherlands) Answer for part (b). All odd integers $n \\geqslant 3$.","solution":"Part ( $\\boldsymbol{a}$ ). Assume that $n$ is odd. Consider a regular $n$-gon. Label the vertices of the $n$-gon as $A_{1}, A_{2}, \\ldots, A_{n}$ in counter-clockwise order, and set $\\mathcal{V}=\\left\\{A_{1}, \\ldots, A_{n}\\right\\}$. We check that $\\mathcal{V}$ is balanced. For any two distinct vertices $A_{i}$ and $A_{j}$, let $k \\in\\{1,2, \\ldots, n\\}$ be the solution of $2 k \\equiv i+j(\\bmod n)$. Then, since $k-i \\equiv j-k(\\bmod n)$, we have $A_{i} A_{k}=A_{j} A_{k}$, as required. Now assume that $n$ is even. Consider a regular $(3 n-6)$-gon, and let $O$ be its circumcenter. Again, label its vertices as $A_{1}, \\ldots, A_{3 n-6}$ in counter-clockwise order, and choose $\\mathcal{V}=$ $\\left\\{O, A_{1}, A_{2}, \\ldots, A_{n-1}\\right\\}$. We check that $\\mathcal{V}$ is balanced. For any two distinct vertices $A_{i}$ and $A_{j}$, we always have $O A_{i}=O A_{j}$. We now consider the vertices $O$ and $A_{i}$. First note that the triangle $O A_{i} A_{n \/ 2-1+i}$ is equilateral for all $i \\leqslant \\frac{n}{2}$. Hence, if $i \\leqslant \\frac{n}{2}$, then we have $O A_{n \/ 2-1+i}=A_{i} A_{n \/ 2-1+i}$; otherwise, if $i>\\frac{n}{2}$, then we have $O A_{i-n \/ 2+1}=A_{i} A_{i-n \/ 2+1}$. This completes the proof. An example of such a construction when $n=10$ is shown in Figure 1. ![](https:\/\/cdn.mathpix.com\/cropped\/2024_11_18_62dbd531c72d0ff8164cg-28.jpg?height=507&width=804&top_left_y=1574&top_left_x=272) Figure 1 ![](https:\/\/cdn.mathpix.com\/cropped\/2024_11_18_62dbd531c72d0ff8164cg-28.jpg?height=600&width=617&top_left_y=1479&top_left_x=1185) Figure 2 Comment (a). There are many ways to construct an example by placing equilateral triangles in a circle. Here we present one general method. Let $O$ be the center of a circle and let $A_{1}, B_{1}, \\ldots, A_{k}, B_{k}$ be distinct points on the circle such that the triangle $O A_{i} B_{i}$ is equilateral for each $i$. Then $\\mathcal{V}=\\left\\{O, A_{1}, B_{1}, \\ldots, A_{k}, B_{k}\\right\\}$ is balanced. To construct a set of even cardinality, put extra points $C, D, E$ on the circle such that triangles $O C D$ and $O D E$ are equilateral (see Figure 2). Then $\\mathcal{V}=\\left\\{O, A_{1}, B_{1}, \\ldots, A_{k}, B_{k}, C, D, E\\right\\}$ is balanced. Part (b). We now show that there exists a balanced, center-free set containing $n$ points for all odd $n \\geqslant 3$, and that one does not exist for any even $n \\geqslant 3$. If $n$ is odd, then let $\\mathcal{V}$ be the set of vertices of a regular $n$-gon. We have shown in part ( $a$ ) that $\\mathcal{V}$ is balanced. We claim that $\\mathcal{V}$ is also center-free. Indeed, if $P$ is a point such that $P A=P B=P C$ for some three distinct vertices $A, B$ and $C$, then $P$ is the circumcenter of the $n$-gon, which is not contained in $\\mathcal{V}$. Now suppose that $\\mathcal{V}$ is a balanced, center-free set of even cardinality $n$. We will derive a contradiction. For a pair of distinct points $A, B \\in \\mathcal{V}$, we say that a point $C \\in \\mathcal{V}$ is associated with the pair $\\{A, B\\}$ if $A C=B C$. Since there are $\\frac{n(n-1)}{2}$ pairs of points, there exists a point $P \\in \\mathcal{V}$ which is associated with at least $\\left\\lceil\\frac{n(n-1)}{2} \/ n\\right\\rceil=\\frac{n}{2}$ pairs. Note that none of these $\\frac{n}{2}$ pairs can contain $P$, so that the union of these $\\frac{n}{2}$ pairs consists of at most $n-1$ points. Hence there exist two such pairs that share a point. Let these two pairs be $\\{A, B\\}$ and $\\{A, C\\}$. Then $P A=P B=P C$, which is a contradiction. Comment (b). We can rephrase the argument in graph theoretic terms as follows. Let $\\mathcal{V}$ be a balanced, center-free set consisting of $n$ points. For any pair of distinct vertices $A, B \\in \\mathcal{V}$ and for any $C \\in \\mathcal{V}$ such that $A C=B C$, draw directed edges $A \\rightarrow C$ and $B \\rightarrow C$. Then all pairs of vertices generate altogether at least $n(n-1)$ directed edges; since the set is center-free, these edges are distinct. So we must obtain a graph in which any two vertices are connected in both directions. Now, each vertex has exactly $n-1$ incoming edges, which means that $n-1$ is even. Hence $n$ is odd.","tier":0} +{"problem_type":"Combinatorics","problem_label":"C3","problem":"For a finite set $A$ of positive integers, we call a partition of $A$ into two disjoint nonempty subsets $A_{1}$ and $A_{2}$ good if the least common multiple of the elements in $A_{1}$ is equal to the greatest common divisor of the elements in $A_{2}$. Determine the minimum value of $n$ such that there exists a set of $n$ positive integers with exactly 2015 good partitions. (Ukraine) Answer. 3024.","solution":"Let $A=\\left\\{a_{1}, a_{2}, \\ldots, a_{n}\\right\\}$, where $a_{1}1$ and $B$ can respond by choosing $a-1$ on the $k^{\\text {th }}$ move instead. We now give an alternative winning strategy in the case $n$ is even and $n \\geqslant 8$. We first present a winning strategy for the case when $A$ 's first pick is 1 . We consider two cases depending on $A$ 's second move. Case 1. A's second pick is 3 . Then $B$ chooses $n-3$ on the second move. On the $k^{\\text {th }}$ move, $B$ chooses the number exactly 1 less than $A^{\\prime}$ 's $k^{\\text {th }}$ pick except that $B$ chooses 2 if $A$ 's $k^{\\text {th }}$ pick is $n-2$ or $n-1$. Case 2. A's second pick is $a>3$. Then $B$ chooses $a-2$ on the second move. Afterwards on the $k^{\\text {th }}$ move, $B$ picks the number exactly 1 less than $A^{\\text {'s }} k^{\\text {th }}$ pick. One may easily see that this strategy guarantees $B$ 's victory, when $A$ 's first pick is 1 . The following claim shows how to extend the strategy to the general case. Claim. Assume that $B$ has an explicit strategy leading to a victory after $A$ picks 1 on the first move. Then $B$ also has an explicit strategy leading to a victory after any first moves of $A$. Proof. Let $S$ be an optimal strategy of $B$ after $A$ picks 1 on the first move. Assume that $A$ picks some number $a>1$ on this move; we show how $B$ can make use of $S$ in order to win in this case. In parallel to the real play, $B$ starts an imaginary play. The positions in these plays differ by flipping the segment $[1, a]$; so, if a player chooses some number $x$ in the real play, then the same player chooses a number $x$ or $a+1-x$ in the imaginary play, depending on whether $x>a$ or $x \\leqslant a$. Thus $A$ 's first pick in the imaginary play is 1. Clearly, a number is chosen in the real play exactly if the corresponding number is chosen in the imaginary one. Next, if an unchosen number is neighboring to one chosen by $A$ in the imaginary play, then the corresponding number also has this property in the real play, so $A$ also cannot choose it. One can easily see that a similar statement with real and imaginary plays interchanged holds for $B$ instead of $A$. Thus, when $A$ makes some move in the real play, $B$ may imagine the corresponding legal move in the imaginary one. Then $B$ chooses the response according to $S$ in the imaginary game and makes the corresponding legal move in the real one. Acting so, $B$ wins the imaginary game, thus $B$ will also win the real one. Hence, $B$ has a winning strategy for all even $n$ greater or equal to 8 . Notice that the claim can also be used to simplify the argument when $n$ is odd. Comment 2. One may also employ symmetry when $n$ is odd. In particular, $B$ could use a mirror strategy. However, additional ideas are required to modify the strategy after $A$ picks $\\frac{n+1}{2}$.","tier":0} +{"problem_type":"Combinatorics","problem_label":"C5","problem":"Consider an infinite sequence $a_{1}, a_{2}, \\ldots$ of positive integers with $a_{i} \\leqslant 2015$ for all $i \\geqslant 1$. Suppose that for any two distinct indices $i$ and $j$ we have $i+a_{i} \\neq j+a_{j}$. Prove that there exist two positive integers $b$ and $N$ such that $$ \\left|\\sum_{i=m+1}^{n}\\left(a_{i}-b\\right)\\right| \\leqslant 1007^{2} $$ whenever $n>m \\geqslant N$. (Australia)","solution":"We visualize the set of positive integers as a sequence of points. For each $n$ we draw an arrow emerging from $n$ that points to $n+a_{n}$; so the length of this arrow is $a_{n}$. Due to the condition that $m+a_{m} \\neq n+a_{n}$ for $m \\neq n$, each positive integer receives at most one arrow. There are some positive integers, such as 1 , that receive no arrows; these will be referred to as starting points in the sequel. When one starts at any of the starting points and keeps following the arrows, one is led to an infinite path, called its ray, that visits a strictly increasing sequence of positive integers. Since the length of any arrow is at most 2015, such a ray, say with starting point $s$, meets every interval of the form $[n, n+2014]$ with $n \\geqslant s$ at least once. Suppose for the sake of contradiction that there would be at least 2016 starting points. Then we could take an integer $n$ that is larger than the first 2016 starting points. But now the interval $[n, n+2014]$ must be met by at least 2016 rays in distinct points, which is absurd. We have thereby shown that the number $b$ of starting points satisfies $1 \\leqslant b \\leqslant 2015$. Let $N$ denote any integer that is larger than all starting points. We contend that $b$ and $N$ are as required. To see this, let any two integers $m$ and $n$ with $n>m \\geqslant N$ be given. The sum $\\sum_{i=m+1}^{n} a_{i}$ gives the total length of the arrows emerging from $m+1, \\ldots, n$. Taken together, these arrows form $b$ subpaths of our rays, some of which may be empty. Now on each ray we look at the first number that is larger than $m$; let $x_{1}, \\ldots, x_{b}$ denote these numbers, and let $y_{1}, \\ldots, y_{b}$ enumerate in corresponding order the numbers defined similarly with respect to $n$. Then the list of differences $y_{1}-x_{1}, \\ldots, y_{b}-x_{b}$ consists of the lengths of these paths and possibly some zeros corresponding to empty paths. Consequently, we obtain $$ \\sum_{i=m+1}^{n} a_{i}=\\sum_{j=1}^{b}\\left(y_{j}-x_{j}\\right) $$ whence $$ \\sum_{i=m+1}^{n}\\left(a_{i}-b\\right)=\\sum_{j=1}^{b}\\left(y_{j}-n\\right)-\\sum_{j=1}^{b}\\left(x_{j}-m\\right) . $$ Now each of the $b$ rays meets the interval $[m+1, m+2015]$ at some point and thus $x_{1}-$ $m, \\ldots, x_{b}-m$ are $b$ distinct members of the set $\\{1,2, \\ldots, 2015\\}$. Moreover, since $m+1$ is not a starting point, it must belong to some ray; so 1 has to appear among these numbers, wherefore $$ 1+\\sum_{j=1}^{b-1}(j+1) \\leqslant \\sum_{j=1}^{b}\\left(x_{j}-m\\right) \\leqslant 1+\\sum_{j=1}^{b-1}(2016-b+j) $$ The same argument applied to $n$ and $y_{1}, \\ldots, y_{b}$ yields $$ 1+\\sum_{j=1}^{b-1}(j+1) \\leqslant \\sum_{j=1}^{b}\\left(y_{j}-n\\right) \\leqslant 1+\\sum_{j=1}^{b-1}(2016-b+j) $$ So altogether we get $$ \\begin{aligned} \\left|\\sum_{i=m+1}^{n}\\left(a_{i}-b\\right)\\right| & \\leqslant \\sum_{j=1}^{b-1}((2016-b+j)-(j+1))=(b-1)(2015-b) \\\\ & \\leqslant\\left(\\frac{(b-1)+(2015-b)}{2}\\right)^{2}=1007^{2}, \\end{aligned} $$ as desired.","tier":0} +{"problem_type":"Combinatorics","problem_label":"C5","problem":"Consider an infinite sequence $a_{1}, a_{2}, \\ldots$ of positive integers with $a_{i} \\leqslant 2015$ for all $i \\geqslant 1$. Suppose that for any two distinct indices $i$ and $j$ we have $i+a_{i} \\neq j+a_{j}$. Prove that there exist two positive integers $b$ and $N$ such that $$ \\left|\\sum_{i=m+1}^{n}\\left(a_{i}-b\\right)\\right| \\leqslant 1007^{2} $$ whenever $n>m \\geqslant N$. (Australia)","solution":"Set $s_{n}=n+a_{n}$ for all positive integers $n$. By our assumptions, we have $$ n+1 \\leqslant s_{n} \\leqslant n+2015 $$ for all $n \\in \\mathbb{Z}_{>0}$. The members of the sequence $s_{1}, s_{2}, \\ldots$ are distinct. We shall investigate the set $$ M=\\mathbb{Z}_{>0} \\backslash\\left\\{s_{1}, s_{2}, \\ldots\\right\\} $$ Claim. At most 2015 numbers belong to $M$. Proof. Otherwise let $m_{1}m \\geqslant N$. Plugging $r=n$ and $r=m$ into (3) and subtracting the estimates that result, we deduce $$ \\sum_{i=m+1}^{n}\\left(a_{i}-b\\right)=\\sum C_{n}-\\sum C_{m} $$ Since $C_{n}$ and $C_{m}$ are subsets of $\\{1,2, \\ldots, 2014\\}$ with $\\left|C_{n}\\right|=\\left|C_{m}\\right|=b-1$, it is clear that the absolute value of the right-hand side of the above inequality attains its largest possible value if either $C_{m}=\\{1,2, \\ldots, b-1\\}$ and $C_{n}=\\{2016-b, \\ldots, 2014\\}$, or the other way around. In these two cases we have $$ \\left|\\sum C_{n}-\\sum C_{m}\\right|=(b-1)(2015-b) $$ so in the general case we find $$ \\left|\\sum_{i=m+1}^{n}\\left(a_{i}-b\\right)\\right| \\leqslant(b-1)(2015-b) \\leqslant\\left(\\frac{(b-1)+(2015-b)}{2}\\right)^{2}=1007^{2} $$ as desired. Comment. The sets $C_{n}$ may be visualized by means of the following process: Start with an empty blackboard. For $n \\geqslant 1$, the following happens during the $n^{\\text {th }}$ step. The number $a_{n}$ gets written on the blackboard, then all numbers currently on the blackboard are decreased by 1 , and finally all zeros that have arisen get swept away. It is not hard to see that the numbers present on the blackboard after $n$ steps are distinct and form the set $C_{n}$. Moreover, it is possible to complete a solution based on this idea.","tier":0} +{"problem_type":"Combinatorics","problem_label":"C7","problem":"In a company of people some pairs are enemies. A group of people is called unsociable if the number of members in the group is odd and at least 3, and it is possible to arrange all its members around a round table so that every two neighbors are enemies. Given that there are at most 2015 unsociable groups, prove that it is possible to partition the company into 11 parts so that no two enemies are in the same part. (Russia)","solution":"Let $G=(V, E)$ be a graph where $V$ is the set of people in the company and $E$ is the set of the enemy pairs - the edges of the graph. In this language, partitioning into 11 disjoint enemy-free subsets means properly coloring the vertices of this graph with 11 colors. We will prove the following more general statement. Claim. Let $G$ be a graph with chromatic number $k \\geqslant 3$. Then $G$ contains at least $2^{k-1}-k$ unsociable groups. Recall that the chromatic number of $G$ is the least $k$ such that a proper coloring $$ V=V_{1} \\sqcup \\cdots \\sqcup V_{k} $$ exists. In view of $2^{11}-12>2015$, the claim implies the problem statement. Let $G$ be a graph with chromatic number $k$. We say that a proper coloring (1) of $G$ is leximinimal, if the $k$-tuple $\\left(\\left|V_{1}\\right|,\\left|V_{2}\\right|, \\ldots,\\left|V_{k}\\right|\\right)$ is lexicographically minimal; in other words, the following conditions are satisfied: the number $n_{1}=\\left|V_{1}\\right|$ is minimal; the number $n_{2}=\\left|V_{2}\\right|$ is minimal, subject to the previously chosen value of $n_{1} ; \\ldots$; the number $n_{k-1}=\\left|V_{k-1}\\right|$ is minimal, subject to the previously chosen values of $n_{1}, \\ldots, n_{k-2}$. The following lemma is the core of the proof. Lemma 1. Suppose that $G=(V, E)$ is a graph with odd chromatic number $k \\geqslant 3$, and let (1) be one of its leximinimal colorings. Then $G$ contains an odd cycle which visits all color classes $V_{1}, V_{2}, \\ldots, V_{k}$. Proof of Lemma 1. Let us call a cycle colorful if it visits all color classes. Due to the definition of the chromatic number, $V_{1}$ is nonempty. Choose an arbitrary vertex $v \\in V_{1}$. We construct a colorful odd cycle that has only one vertex in $V_{1}$, and this vertex is $v$. We draw a subgraph of $G$ as follows. Place $v$ in the center, and arrange the sets $V_{2}, V_{3}, \\ldots, V_{k}$ in counterclockwise circular order around it. For convenience, let $V_{k+1}=V_{2}$. We will draw arrows to add direction to some edges of $G$, and mark the vertices these arrows point to. First we draw arrows from $v$ to all its neighbors in $V_{2}$, and mark all those neighbors. If some vertex $u \\in V_{i}$ with $i \\in\\{2,3, \\ldots, k\\}$ is already marked, we draw arrows from $u$ to all its neighbors in $V_{i+1}$ which are not marked yet, and we mark all of them. We proceed doing this as long as it is possible. The process of marking is exemplified in Figure 1. Notice that by the rules of our process, in the final state, marked vertices in $V_{i}$ cannot have unmarked neighbors in $V_{i+1}$. Moreover, $v$ is connected to all marked vertices by directed paths. Now move each marked vertex to the next color class in circular order (see an example in Figure 3). In view of the arguments above, the obtained coloring $V_{1} \\sqcup W_{2} \\sqcup \\cdots \\sqcup W_{k}$ is proper. Notice that $v$ has a neighbor $w \\in W_{2}$, because otherwise $$ \\left(V_{1} \\backslash\\{v\\}\\right) \\sqcup\\left(W_{2} \\cup\\{v\\}\\right) \\sqcup W_{3} \\sqcup \\cdots \\sqcup W_{k} $$ would be a proper coloring lexicographically smaller than (1). If $w$ was unmarked, i.e., $w$ was an element of $V_{2}$, then it would be marked at the beginning of the process and thus moved to $V_{3}$, which did not happen. Therefore, $w$ is marked and $w \\in V_{k}$. ![](https:\/\/cdn.mathpix.com\/cropped\/2024_11_18_62dbd531c72d0ff8164cg-40.jpg?height=780&width=1658&top_left_y=181&top_left_x=219) Figure 1 Figure 2 Since $w$ is marked, there exists a directed path from $v$ to $w$. This path moves through the sets $V_{2}, \\ldots, V_{k}$ in circular order, so the number of edges in it is divisible by $k-1$ and thus even. Closing this path by the edge $w \\rightarrow v$, we get a colorful odd cycle, as required. Proof of the claim. Let us choose a leximinimal coloring (1) of $G$. For every set $C \\subseteq\\{1,2, \\ldots, k\\}$ such that $|C|$ is odd and greater than 1 , we will provide an odd cycle visiting exactly those color classes whose indices are listed in the set $C$. This property ensures that we have different cycles for different choices of $C$, and it proves the claim because there are $2^{k-1}-k$ choices for the set $C$. Let $V_{C}=\\bigcup_{c \\in C} V_{c}$, and let $G_{C}$ be the induced subgraph of $G$ on the vertex set $V_{C}$. We also have the induced coloring of $V_{C}$ with $|C|$ colors; this coloring is of course proper. Notice further that the induced coloring is leximinimal: if we had a lexicographically smaller coloring $\\left(W_{c}\\right)_{c \\in C}$ of $G_{C}$, then these classes, together the original color classes $V_{i}$ for $i \\notin C$, would provide a proper coloring which is lexicographically smaller than (1). Hence Lemma 1, applied to the subgraph $G_{C}$ and its leximinimal coloring $\\left(V_{c}\\right)_{c \\in C}$, provides an odd cycle that visits exactly those color classes that are listed in the set $C$.","tier":0} +{"problem_type":"Combinatorics","problem_label":"C7","problem":"In a company of people some pairs are enemies. A group of people is called unsociable if the number of members in the group is odd and at least 3, and it is possible to arrange all its members around a round table so that every two neighbors are enemies. Given that there are at most 2015 unsociable groups, prove that it is possible to partition the company into 11 parts so that no two enemies are in the same part. (Russia)","solution":"We provide a different proof of the claim from the previous solution. We say that a graph is critical if deleting any vertex from the graph decreases the graph's chromatic number. Obviously every graph contains a critical induced subgraph with the same chromatic number. Lemma 2. Suppose that $G=(V, E)$ is a critical graph with chromatic number $k \\geqslant 3$. Then every vertex $v$ of $G$ is contained in at least $2^{k-2}-1$ unsociable groups. Proof. For every set $X \\subseteq V$, denote by $n(X)$ the number of neighbors of $v$ in the set $X$. Since $G$ is critical, there exists a proper coloring of $G \\backslash\\{v\\}$ with $k-1$ colors, so there exists a proper coloring $V=V_{1} \\sqcup V_{2} \\sqcup \\cdots \\sqcup V_{k}$ of $G$ such that $V_{1}=\\{v\\}$. Among such colorings, take one for which the sequence $\\left(n\\left(V_{2}\\right), n\\left(V_{3}\\right), \\ldots, n\\left(V_{k}\\right)\\right)$ is lexicographically minimal. Clearly, $n\\left(V_{i}\\right)>0$ for every $i=2,3, \\ldots, k$; otherwise $V_{2} \\sqcup \\ldots \\sqcup V_{i-1} \\sqcup\\left(V_{i} \\cup V_{1}\\right) \\sqcup V_{i+1} \\sqcup \\ldots V_{k}$ would be a proper coloring of $G$ with $k-1$ colors. We claim that for every $C \\subseteq\\{2,3, \\ldots, k\\}$ with $|C| \\geqslant 2$ being even, $G$ contains an unsociable group so that the set of its members' colors is precisely $C \\cup\\{1\\}$. Since the number of such sets $C$ is $2^{k-2}-1$, this proves the lemma. Denote the elements of $C$ by $c_{1}, \\ldots, c_{2 \\ell}$ in increasing order. For brevity, let $U_{i}=V_{c_{i}}$. Denote by $N_{i}$ the set of neighbors of $v$ in $U_{i}$. We show that for every $i=1, \\ldots, 2 \\ell-1$ and $x \\in N_{i}$, the subgraph induced by $U_{i} \\cup U_{i+1}$ contains a path that connects $x$ with another point in $N_{i+1}$. For the sake of contradiction, suppose that no such path exists. Let $S$ be the set of vertices that lie in the connected component of $x$ in the subgraph induced by $U_{i} \\cup U_{i+1}$, and let $P=U_{i} \\cap S$, and $Q=U_{i+1} \\cap S$ (see Figure 3). Since $x$ is separated from $N_{i+1}$, the sets $Q$ and $N_{i+1}$ are disjoint. So, if we re-color $G$ by replacing $U_{i}$ and $U_{i+1}$ by $\\left(U_{i} \\cup Q\\right) \\backslash P$ and $\\left(U_{i+1} \\cup P\\right) \\backslash Q$, respectively, we obtain a proper coloring such that $n\\left(U_{i}\\right)=n\\left(V_{c_{i}}\\right)$ is decreased and only $n\\left(U_{i+1}\\right)=n\\left(V_{c_{i+1}}\\right)$ is increased. That contradicts the lexicographical minimality of $\\left(n\\left(V_{2}\\right), n\\left(V_{3}\\right), \\ldots, n\\left(V_{k}\\right)\\right)$. ![](https:\/\/cdn.mathpix.com\/cropped\/2024_11_18_62dbd531c72d0ff8164cg-41.jpg?height=438&width=851&top_left_y=632&top_left_x=611) Figure 3 Next, we build a path through $U_{1}, U_{2}, \\ldots, U_{2 \\ell}$ as follows. Let the starting point of the path be an arbitrary vertex $v_{1}$ in the set $N_{1}$. For $i \\leqslant 2 \\ell-1$, if the vertex $v_{i} \\in N_{i}$ is already defined, connect $v_{i}$ to some vertex in $N_{i+1}$ in the subgraph induced by $U_{i} \\cup U_{i+1}$, and add these edges to the path. Denote the new endpoint of the path by $v_{i+1}$; by the construction we have $v_{i+1} \\in N_{i+1}$ again, so the process can be continued. At the end we have a path that starts at $v_{1} \\in N_{1}$ and ends at some $v_{2 \\ell} \\in N_{2 \\ell}$. Moreover, all edges in this path connect vertices in neighboring classes: if a vertex of the path lies in $U_{i}$, then the next vertex lies in $U_{i+1}$ or $U_{i-1}$. Notice that the path is not necessary simple, so take a minimal subpath of it. The minimal subpath is simple and connects the same endpoints $v_{1}$ and $v_{2 \\ell}$. The property that every edge steps to a neighboring color class (i.e., from $U_{i}$ to $U_{i+1}$ or $U_{i-1}$ ) is preserved. So the resulting path also visits all of $U_{1}, \\ldots, U_{2 \\ell}$, and its length must be odd. Closing the path with the edges $v v_{1}$ and $v_{2 \\ell} v$ we obtain the desired odd cycle (see Figure 4). ![](https:\/\/cdn.mathpix.com\/cropped\/2024_11_18_62dbd531c72d0ff8164cg-41.jpg?height=518&width=1489&top_left_y=1803&top_left_x=286) Figure 4 Now we prove the claim by induction on $k \\geqslant 3$. The base case $k=3$ holds by applying Lemma 2 to a critical subgraph. For the induction step, let $G_{0}$ be a critical $k$-chromatic subgraph of $G$, and let $v$ be an arbitrary vertex of $G_{0}$. By Lemma 2, $G_{0}$ has at least $2^{k-2}-1$ unsociable groups containing $v$. On the other hand, the graph $G_{0} \\backslash\\{v\\}$ has chromatic number $k-1$, so it contains at least $2^{k-2}-(k-1)$ unsociable groups by the induction hypothesis. Altogether, this gives $2^{k-2}-1+2^{k-2}-(k-1)=2^{k-1}-k$ distinct unsociable groups in $G_{0}$ (and thus in $G$ ). Comment 1. The claim we proved is sharp. The complete graph with $k$ vertices has chromatic number $k$ and contains exactly $2^{k-1}-k$ unsociable groups. Comment 2. The proof of Lemma 2 works for odd values of $|C| \\geqslant 3$ as well. Hence, the second solution shows the analogous statement that the number of even sized unsociable groups is at least $2^{k}-1-\\binom{k}{2}$.","tier":0} +{"problem_type":"Geometry","problem_label":"G4","problem":"Let $A B C$ be an acute triangle, and let $M$ be the midpoint of $A C$. A circle $\\omega$ passing through $B$ and $M$ meets the sides $A B$ and $B C$ again at $P$ and $Q$, respectively. Let $T$ be the point such that the quadrilateral $B P T Q$ is a parallelogram. Suppose that $T$ lies on the circumcircle of the triangle $A B C$. Determine all possible values of $B T \/ B M$. (Russia) Answer. $\\sqrt{2}$.","solution":"Let $S$ be the center of the parallelogram $B P T Q$, and let $B^{\\prime} \\neq B$ be the point on the ray $B M$ such that $B M=M B^{\\prime}$ (see Figure 1). It follows that $A B C B^{\\prime}$ is a parallelogram. Then, $\\angle A B B^{\\prime}=\\angle P Q M$ and $\\angle B B^{\\prime} A=\\angle B^{\\prime} B C=\\angle M P Q$, and so the triangles $A B B^{\\prime}$ and $M Q P$ are similar. It follows that $A M$ and $M S$ are corresponding medians in these triangles. Hence, $$ \\angle S M P=\\angle B^{\\prime} A M=\\angle B C A=\\angle B T A . $$ Since $\\angle A C T=\\angle P B T$ and $\\angle T A C=\\angle T B C=\\angle B T P$, the triangles $T C A$ and $P B T$ are similar. Again, as $T M$ and $P S$ are corresponding medians in these triangles, we have $$ \\angle M T A=\\angle T P S=\\angle B Q P=\\angle B M P . $$ Now we deal separately with two cases. Case 1. $\\quad S$ does not lie on $B M$. Since the configuration is symmetric between $A$ and $C$, we may assume that $S$ and $A$ lie on the same side with respect to the line $B M$. Applying (1) and (2), we get $$ \\angle B M S=\\angle B M P-\\angle S M P=\\angle M T A-\\angle B T A=\\angle M T B, $$ and so the triangles $B S M$ and $B M T$ are similar. We now have $B M^{2}=B S \\cdot B T=B T^{2} \/ 2$, so $B T=\\sqrt{2} B M$. Case 2. $\\quad S$ lies on $B M$. It follows from (2) that $\\angle B C A=\\angle M T A=\\angle B Q P=\\angle B M P$ (see Figure 2). Thus, $P Q \\| A C$ and $P M \\| A T$. Hence, $B S \/ B M=B P \/ B A=B M \/ B T$, so $B T^{2}=2 B M^{2}$ and $B T=\\sqrt{2} B M$. ![](https:\/\/cdn.mathpix.com\/cropped\/2024_11_18_62dbd531c72d0ff8164cg-49.jpg?height=960&width=730&top_left_y=1713&top_left_x=246) Figure 1 ![](https:\/\/cdn.mathpix.com\/cropped\/2024_11_18_62dbd531c72d0ff8164cg-49.jpg?height=749&width=729&top_left_y=1938&top_left_x=1092) Figure 2 Comment 1. Here is another way to show that the triangles $B S M$ and $B M T$ are similar. Denote by $\\Omega$ the circumcircle of the triangle $A B C$. Let $R$ be the second point of intersection of $\\omega$ and $\\Omega$, and let $\\tau$ be the spiral similarity centered at $R$ mapping $\\omega$ to $\\Omega$. Then, one may show that $\\tau$ maps each point $X$ on $\\omega$ to a point $Y$ on $\\Omega$ such that $B, X$, and $Y$ are collinear (see Figure 3). If we let $K$ and $L$ be the second points of intersection of $B M$ with $\\Omega$ and of $B T$ with $\\omega$, respectively, then it follows that the triangle $M K T$ is the image of $S M L$ under $\\tau$. We now obtain $\\angle B S M=\\angle T M B$, which implies the desired result. ![](https:\/\/cdn.mathpix.com\/cropped\/2024_11_18_62dbd531c72d0ff8164cg-50.jpg?height=758&width=757&top_left_y=752&top_left_x=204) Figure 3 ![](https:\/\/cdn.mathpix.com\/cropped\/2024_11_18_62dbd531c72d0ff8164cg-50.jpg?height=954&width=846&top_left_y=551&top_left_x=1002) Figure 4","tier":0} +{"problem_type":"Geometry","problem_label":"G4","problem":"Let $A B C$ be an acute triangle, and let $M$ be the midpoint of $A C$. A circle $\\omega$ passing through $B$ and $M$ meets the sides $A B$ and $B C$ again at $P$ and $Q$, respectively. Let $T$ be the point such that the quadrilateral $B P T Q$ is a parallelogram. Suppose that $T$ lies on the circumcircle of the triangle $A B C$. Determine all possible values of $B T \/ B M$. (Russia) Answer. $\\sqrt{2}$.","solution":"Again, we denote by $\\Omega$ the circumcircle of the triangle $A B C$. Choose the points $X$ and $Y$ on the rays $B A$ and $B C$ respectively, so that $\\angle M X B=\\angle M B C$ and $\\angle B Y M=\\angle A B M$ (see Figure 4). Then the triangles $B M X$ and $Y M B$ are similar. Since $\\angle X P M=\\angle B Q M$, the points $P$ and $Q$ correspond to each other in these triangles. So, if $\\overrightarrow{B P}=\\mu \\cdot \\overrightarrow{B X}$, then $\\overrightarrow{B Q}=(1-\\mu) \\cdot \\overrightarrow{B Y}$. Thus $$ \\overrightarrow{B T}=\\overrightarrow{B P}+\\overrightarrow{B Q}=\\overrightarrow{B Y}+\\mu \\cdot(\\overrightarrow{B X}-\\overrightarrow{B Y})=\\overrightarrow{B Y}+\\mu \\cdot \\overrightarrow{Y X} $$ which means that $T$ lies on the line $X Y$. Let $B^{\\prime} \\neq B$ be the point on the ray $B M$ such that $B M=M B^{\\prime}$. Then $\\angle M B^{\\prime} A=$ $\\angle M B C=\\angle M X B$ and $\\angle C B^{\\prime} M=\\angle A B M=\\angle B Y M$. This means that the triangles $B M X$, $B A B^{\\prime}, Y M B$, and $B^{\\prime} C B$ are all similar; hence $B A \\cdot B X=B M \\cdot B B^{\\prime}=B C \\cdot B Y$. Thus there exists an inversion centered at $B$ which swaps $A$ with $X, M$ with $B^{\\prime}$, and $C$ with $Y$. This inversion then swaps $\\Omega$ with the line $X Y$, and hence it preserves $T$. Therefore, we have $B T^{2}=B M \\cdot B B^{\\prime}=2 B M^{2}$, and $B T=\\sqrt{2} B M$.","tier":0} +{"problem_type":"Geometry","problem_label":"G4","problem":"Let $A B C$ be an acute triangle, and let $M$ be the midpoint of $A C$. A circle $\\omega$ passing through $B$ and $M$ meets the sides $A B$ and $B C$ again at $P$ and $Q$, respectively. Let $T$ be the point such that the quadrilateral $B P T Q$ is a parallelogram. Suppose that $T$ lies on the circumcircle of the triangle $A B C$. Determine all possible values of $B T \/ B M$. (Russia) Answer. $\\sqrt{2}$.","solution":"We begin with the following lemma. Lemma. Let $A B C T$ be a cyclic quadrilateral. Let $P$ and $Q$ be points on the sides $B A$ and $B C$ respectively, such that $B P T Q$ is a parallelogram. Then $B P \\cdot B A+B Q \\cdot B C=B T^{2}$. Proof. Let the circumcircle of the triangle $Q T C$ meet the line $B T$ again at $J$ (see Figure 5). The power of $B$ with respect to this circle yields $$ B Q \\cdot B C=B J \\cdot B T $$ We also have $\\angle T J Q=180^{\\circ}-\\angle Q C T=\\angle T A B$ and $\\angle Q T J=\\angle A B T$, and so the triangles $T J Q$ and $B A T$ are similar. We now have $T J \/ T Q=B A \/ B T$. Therefore, $$ T J \\cdot B T=T Q \\cdot B A=B P \\cdot B A $$ Combining (3) and (4) now yields the desired result. Let $X$ and $Y$ be the midpoints of $B A$ and $B C$ respectively (see Figure 6). Applying the lemma to the cyclic quadrilaterals $P B Q M$ and $A B C T$, we obtain $$ B X \\cdot B P+B Y \\cdot B Q=B M^{2} $$ and $$ B P \\cdot B A+B Q \\cdot B C=B T^{2} $$ Since $B A=2 B X$ and $B C=2 B Y$, we have $B T^{2}=2 B M^{2}$, and so $B T=\\sqrt{2} B M$. ![](https:\/\/cdn.mathpix.com\/cropped\/2024_11_18_62dbd531c72d0ff8164cg-51.jpg?height=789&width=1464&top_left_y=933&top_left_x=296) Comment 2. Here we give another proof of the lemma using Ptolemy's theorem. We readily have $$ T C \\cdot B A+T A \\cdot B C=A C \\cdot B T $$ The lemma now follows from $$ \\frac{B P}{T C}=\\frac{B Q}{T A}=\\frac{B T}{A C}=\\frac{\\sin \\angle B C T}{\\sin \\angle A B C} $$","tier":0} +{"problem_type":"Number Theory","problem_label":"N1","problem":"Determine all positive integers $M$ for which the sequence $a_{0}, a_{1}, a_{2}, \\ldots$, defined by $a_{0}=\\frac{2 M+1}{2}$ and $a_{k+1}=a_{k}\\left\\lfloor a_{k}\\right\\rfloor$ for $k=0,1,2, \\ldots$, contains at least one integer term. (Luxembourg) Answer. All integers $M \\geqslant 2$.","solution":"Define $b_{k}=2 a_{k}$ for all $k \\geqslant 0$. Then $$ b_{k+1}=2 a_{k+1}=2 a_{k}\\left\\lfloor a_{k}\\right\\rfloor=b_{k}\\left\\lfloor\\frac{b_{k}}{2}\\right\\rfloor . $$ Since $b_{0}$ is an integer, it follows that $b_{k}$ is an integer for all $k \\geqslant 0$. Suppose that the sequence $a_{0}, a_{1}, a_{2}, \\ldots$ does not contain any integer term. Then $b_{k}$ must be an odd integer for all $k \\geqslant 0$, so that $$ b_{k+1}=b_{k}\\left\\lfloor\\frac{b_{k}}{2}\\right\\rfloor=\\frac{b_{k}\\left(b_{k}-1\\right)}{2} $$ Hence $$ b_{k+1}-3=\\frac{b_{k}\\left(b_{k}-1\\right)}{2}-3=\\frac{\\left(b_{k}-3\\right)\\left(b_{k}+2\\right)}{2} $$ for all $k \\geqslant 0$. Suppose that $b_{0}-3>0$. Then equation (2) yields $b_{k}-3>0$ for all $k \\geqslant 0$. For each $k \\geqslant 0$, define $c_{k}$ to be the highest power of 2 that divides $b_{k}-3$. Since $b_{k}-3$ is even for all $k \\geqslant 0$, the number $c_{k}$ is positive for every $k \\geqslant 0$. Note that $b_{k}+2$ is an odd integer. Therefore, from equation (2), we have that $c_{k+1}=c_{k}-1$. Thus, the sequence $c_{0}, c_{1}, c_{2}, \\ldots$ of positive integers is strictly decreasing, a contradiction. So, $b_{0}-3 \\leqslant 0$, which implies $M=1$. For $M=1$, we can check that the sequence is constant with $a_{k}=\\frac{3}{2}$ for all $k \\geqslant 0$. Therefore, the answer is $M \\geqslant 2$.","tier":0} +{"problem_type":"Number Theory","problem_label":"N1","problem":"Determine all positive integers $M$ for which the sequence $a_{0}, a_{1}, a_{2}, \\ldots$, defined by $a_{0}=\\frac{2 M+1}{2}$ and $a_{k+1}=a_{k}\\left\\lfloor a_{k}\\right\\rfloor$ for $k=0,1,2, \\ldots$, contains at least one integer term. (Luxembourg) Answer. All integers $M \\geqslant 2$.","solution":"We provide an alternative way to show $M=1$ once equation (1) has been reached. We claim that $b_{k} \\equiv 3\\left(\\bmod 2^{m}\\right)$ for all $k \\geqslant 0$ and $m \\geqslant 1$. If this is true, then we would have $b_{k}=3$ for all $k \\geqslant 0$ and hence $M=1$. To establish our claim, we proceed by induction on $m$. The base case $b_{k} \\equiv 3(\\bmod 2)$ is true for all $k \\geqslant 0$ since $b_{k}$ is odd. Now suppose that $b_{k} \\equiv 3\\left(\\bmod 2^{m}\\right)$ for all $k \\geqslant 0$. Hence $b_{k}=2^{m} d_{k}+3$ for some integer $d_{k}$. We have $$ 3 \\equiv b_{k+1} \\equiv\\left(2^{m} d_{k}+3\\right)\\left(2^{m-1} d_{k}+1\\right) \\equiv 3 \\cdot 2^{m-1} d_{k}+3 \\quad\\left(\\bmod 2^{m}\\right) $$ so that $d_{k}$ must be even. This implies that $b_{k} \\equiv 3\\left(\\bmod 2^{m+1}\\right)$, as required. Comment. The reason the number 3 which appears in both solutions is important, is that it is a nontrivial fixed point of the recurrence relation for $b_{k}$.","tier":0} +{"problem_type":"Number Theory","problem_label":"N5","problem":"Determine all triples $(a, b, c)$ of positive integers for which $a b-c, b c-a$, and $c a-b$ are powers of 2 . Explanation: A power of 2 is an integer of the form $2^{n}$, where $n$ denotes some nonnegative integer. (Serbia) Answer. There are sixteen such triples, namely $(2,2,2)$, the three permutations of $(2,2,3)$, and the six permutations of each of $(2,6,11)$ and $(3,5,7)$.","solution":"It can easily be verified that these sixteen triples are as required. Now let ( $a, b, c$ ) be any triple with the desired property. If we would have $a=1$, then both $b-c$ and $c-b$ were powers of 2 , which is impossible since their sum is zero; because of symmetry, this argument shows $a, b, c \\geqslant 2$. Case 1. Among $a, b$, and $c$ there are at least two equal numbers. Without loss of generality we may suppose that $a=b$. Then $a^{2}-c$ and $a(c-1)$ are powers of 2 . The latter tells us that actually $a$ and $c-1$ are powers of 2 . So there are nonnegative integers $\\alpha$ and $\\gamma$ with $a=2^{\\alpha}$ and $c=2^{\\gamma}+1$. Since $a^{2}-c=2^{2 \\alpha}-2^{\\gamma}-1$ is a power of 2 and thus incongruent to -1 modulo 4 , we must have $\\gamma \\leqslant 1$. Moreover, each of the terms $2^{2 \\alpha}-2$ and $2^{2 \\alpha}-3$ can only be a power of 2 if $\\alpha=1$. It follows that the triple $(a, b, c)$ is either $(2,2,2)$ or $(2,2,3)$. Case 2. The numbers $a, b$, and $c$ are distinct. Due to symmetry we may suppose that $$ 2 \\leqslant a\\beta>\\gamma . $$ Depending on how large $a$ is, we divide the argument into two further cases. Case 2.1. $\\quad a=2$. We first prove that $\\gamma=0$. Assume for the sake of contradiction that $\\gamma>0$. Then $c$ is even by (4) and, similarly, $b$ is even by (5) and (3). So the left-hand side of (2) is congruent to 2 modulo 4 , which is only possible if $b c=4$. As this contradicts (1), we have thereby shown that $\\gamma=0$, i.e., that $c=2 b-1$. Now (3) yields $3 b-2=2^{\\beta}$. Due to $b>2$ this is only possible if $\\beta \\geqslant 4$. If $\\beta=4$, then we get $b=6$ and $c=2 \\cdot 6-1=11$, which is a solution. It remains to deal with the case $\\beta \\geqslant 5$. Now (2) implies $$ 9 \\cdot 2^{\\alpha}=9 b(2 b-1)-18=(3 b-2)(6 b+1)-16=2^{\\beta}\\left(2^{\\beta+1}+5\\right)-16 $$ and by $\\beta \\geqslant 5$ the right-hand side is not divisible by 32 . Thus $\\alpha \\leqslant 4$ and we get a contradiction to (5). Case 2.2. $\\quad a \\geqslant 3$. Pick an integer $\\vartheta \\in\\{-1,+1\\}$ such that $c-\\vartheta$ is not divisible by 4 . Now $$ 2^{\\alpha}+\\vartheta \\cdot 2^{\\beta}=\\left(b c-a \\vartheta^{2}\\right)+\\vartheta(c a-b)=(b+a \\vartheta)(c-\\vartheta) $$ is divisible by $2^{\\beta}$ and, consequently, $b+a \\vartheta$ is divisible by $2^{\\beta-1}$. On the other hand, $2^{\\beta}=a c-b>$ $(a-1) c \\geqslant 2 c$ implies in view of (1) that $a$ and $b$ are smaller than $2^{\\beta-1}$. All this is only possible if $\\vartheta=1$ and $a+b=2^{\\beta-1}$. Now (3) yields $$ a c-b=2(a+b) $$ whence $4 b>a+3 b=a(c-1) \\geqslant a b$, which in turn yields $a=3$. So (6) simplifies to $c=b+2$ and (2) tells us that $b(b+2)-3=(b-1)(b+3)$ is a power of 2 . Consequently, the factors $b-1$ and $b+3$ are powers of 2 themselves. Since their difference is 4 , this is only possible if $b=5$ and thus $c=7$. Thereby the solution is complete.","tier":0} +{"problem_type":"Number Theory","problem_label":"N5","problem":"Determine all triples $(a, b, c)$ of positive integers for which $a b-c, b c-a$, and $c a-b$ are powers of 2 . Explanation: A power of 2 is an integer of the form $2^{n}$, where $n$ denotes some nonnegative integer. (Serbia) Answer. There are sixteen such triples, namely $(2,2,2)$, the three permutations of $(2,2,3)$, and the six permutations of each of $(2,6,11)$ and $(3,5,7)$.","solution":"As in the beginning of the first solution, we observe that $a, b, c \\geqslant 2$. Depending on the parities of $a, b$, and $c$ we distinguish three cases. Case 1. The numbers $a, b$, and $c$ are even. Let $2^{A}, 2^{B}$, and $2^{C}$ be the largest powers of 2 dividing $a, b$, and $c$ respectively. We may assume without loss of generality that $1 \\leqslant A \\leqslant B \\leqslant C$. Now $2^{B}$ is the highest power of 2 dividing $a c-b$, whence $a c-b=2^{B} \\leqslant b$. Similarly, we deduce $b c-a=2^{A} \\leqslant a$. Adding both estimates we get $(a+b) c \\leqslant 2(a+b)$, whence $c \\leqslant 2$. So $c=2$ and thus $A=B=C=1$; moreover, we must have had equality throughout, i.e., $a=2^{A}=2$ and $b=2^{B}=2$. We have thereby found the solution $(a, b, c)=(2,2,2)$. Case 2. The numbers $a, b$, and $c$ are odd. If any two of these numbers are equal, say $a=b$, then $a c-b=a(c-1)$ has a nontrivial odd divisor and cannot be a power of 2 . Hence $a, b$, and $c$ are distinct. So we may assume without loss of generality that $a\\beta$, and thus $2^{\\beta}$ divides $$ a \\cdot 2^{\\alpha}-b \\cdot 2^{\\beta}=a(b c-a)-b(a c-b)=b^{2}-a^{2}=(b+a)(b-a) $$ Since $a$ is odd, it is not possible that both factors $b+a$ and $b-a$ are divisible by 4 . Consequently, one of them has to be a multiple of $2^{\\beta-1}$. Hence one of the numbers $2(b+a)$ and $2(b-a)$ is divisible by $2^{\\beta}$ and in either case we have $$ a c-b=2^{\\beta} \\leqslant 2(a+b) $$ This in turn yields $(a-1) b\\beta$ denote the integers satisfying $$ 2^{\\alpha}=b c-a \\quad \\text { and } \\quad 2^{\\beta}=a c-b $$ If $\\beta=0$ it would follow that $a c-b=a b-c=1$ and hence that $b=c=1$, which is absurd. So $\\beta$ and $\\alpha$ are positive and consequently $a$ and $b$ are even. Substituting $c=a b-1$ into (9) we obtain $$ \\begin{aligned} 2^{\\alpha} & =a b^{2}-(a+b), \\\\ \\text { and } \\quad 2^{\\beta} & =a^{2} b-(a+b) . \\end{aligned} $$ The addition of both equation yields $2^{\\alpha}+2^{\\beta}=(a b-2)(a+b)$. Now $a b-2$ is even but not divisible by 4 , so the highest power of 2 dividing $a+b$ is $2^{\\beta-1}$. For this reason, the equations (10) and (11) show that the highest powers of 2 dividing either of the numbers $a b^{2}$ and $a^{2} b$ is likewise $2^{\\beta-1}$. Thus there is an integer $\\tau \\geqslant 1$ together with odd integers $A, B$, and $C$ such that $a=2^{\\tau} A, b=2^{\\tau} B, a+b=2^{3 \\tau} C$, and $\\beta=1+3 \\tau$. Notice that $A+B=2^{2 \\tau} C \\geqslant 4 C$. Moreover, (11) entails $A^{2} B-C=2$. Thus $8=$ $4 A^{2} B-4 C \\geqslant 4 A^{2} B-A-B \\geqslant A^{2}(3 B-1)$. Since $A$ and $B$ are odd with $A0$, this may be weakened to $p^{2} q+p-q \\leqslant p+q$. Hence $p^{2} q \\leqslant 2 q$, which is only possible if $p=1$. Going back to (13), we get $$ \\left(d^{2} q-q-1\\right) \\mid d^{2}(q-1) $$ Now $2\\left(d^{2} q-q-1\\right) \\leqslant d^{2}(q-1)$ would entail $d^{2}(q+1) \\leqslant 2(q+1)$ and thus $d=1$. But this would tell us that $a=d p=1$, which is absurd. This argument proves $2\\left(d^{2} q-q-1\\right)>d^{2}(q-1)$ and in the light of (14) it follows that $d^{2} q-q-1=d^{2}(q-1)$, i.e., $q=d^{2}-1$. Plugging this together with $p=1$ into (12) we infer $2^{\\beta}=d^{3}\\left(d^{2}-2\\right)$. Hence $d$ and $d^{2}-2$ are powers of 2 . Consequently, $d=2, q=3$, $a=2, b=6$, and $c=11$, as desired.","tier":0} +{"problem_type":"Number Theory","problem_label":"N7","problem":"Let $\\mathbb{Z}_{>0}$ denote the set of positive integers. For any positive integer $k$, a function $f: \\mathbb{Z}_{>0} \\rightarrow \\mathbb{Z}_{>0}$ is called $k$-good if $\\operatorname{gcd}(f(m)+n, f(n)+m) \\leqslant k$ for all $m \\neq n$. Find all $k$ such that there exists a $k$-good function. (Canada) Answer. $k \\geqslant 2$.","solution":"For any function $f: \\mathbb{Z}_{>0} \\rightarrow \\mathbb{Z}_{>0}$, let $G_{f}(m, n)=\\operatorname{gcd}(f(m)+n, f(n)+m)$. Note that a $k$-good function is also $(k+1)$-good for any positive integer $k$. Hence, it suffices to show that there does not exist a 1-good function and that there exists a 2-good function. We first show that there is no 1 -good function. Suppose that there exists a function $f$ such that $G_{f}(m, n)=1$ for all $m \\neq n$. Now, if there are two distinct even numbers $m$ and $n$ such that $f(m)$ and $f(n)$ are both even, then $2 \\mid G_{f}(m, n)$, a contradiction. A similar argument holds if there are two distinct odd numbers $m$ and $n$ such that $f(m)$ and $f(n)$ are both odd. Hence we can choose an even $m$ and an odd $n$ such that $f(m)$ is odd and $f(n)$ is even. This also implies that $2 \\mid G_{f}(m, n)$, a contradiction. We now construct a 2-good function. Define $f(n)=2^{g(n)+1}-n-1$, where $g$ is defined recursively by $g(1)=1$ and $g(n+1)=\\left(2^{g(n)+1}\\right)!$. For any positive integers $m>n$, set $$ A=f(m)+n=2^{g(m)+1}-m+n-1, \\quad B=f(n)+m=2^{g(n)+1}-n+m-1 $$ We need to show that $\\operatorname{gcd}(A, B) \\leqslant 2$. First, note that $A+B=2^{g(m)+1}+2^{g(n)+1}-2$ is not divisible by 4 , so that $4 \\nmid \\operatorname{gcd}(A, B)$. Now we suppose that there is an odd prime $p$ for which $p \\mid \\operatorname{gcd}(A, B)$ and derive a contradiction. We first claim that $2^{g(m-1)+1} \\geqslant B$. This is a rather weak bound; one way to prove it is as follows. Observe that $g(k+1)>g(k)$ and hence $2^{g(k+1)+1} \\geqslant 2^{g(k)+1}+1$ for every positive integer $k$. By repeatedly applying this inequality, we obtain $2^{g(m-1)+1} \\geqslant 2^{g(n)+1}+(m-1)-n=B$. Now, since $p \\mid B$, we have $p-10}$ denote the set of positive integers. For any positive integer $k$, a function $f: \\mathbb{Z}_{>0} \\rightarrow \\mathbb{Z}_{>0}$ is called $k$-good if $\\operatorname{gcd}(f(m)+n, f(n)+m) \\leqslant k$ for all $m \\neq n$. Find all $k$ such that there exists a $k$-good function. (Canada) Answer. $k \\geqslant 2$.","solution":"We provide an alternative construction of a 2 -good function $f$. Let $\\mathcal{P}$ be the set consisting of 4 and all odd primes. For every $p \\in \\mathcal{P}$, we say that a number $a \\in\\{0,1, \\ldots, p-1\\}$ is $p$-useful if $a \\not \\equiv-a(\\bmod p)$. Note that a residue modulo $p$ which is neither 0 nor 2 is $p$-useful (the latter is needed only when $p=4$ ). We will construct $f$ recursively; in some steps, we will also define a $p$-useful number $a_{p}$. After the $m^{\\text {th }}$ step, the construction will satisfy the following conditions: ( $i$ ) The values of $f(n)$ have already been defined for all $n \\leqslant m$, and $p$-useful numbers $a_{p}$ have already been defined for all $p \\leqslant m+2$; (ii) If $n \\leqslant m$ and $p \\leqslant m+2$, then $f(n)+n \\not \\equiv a_{p}(\\bmod p)$; (iii) $\\operatorname{gcd}\\left(f\\left(n_{1}\\right)+n_{2}, f\\left(n_{2}\\right)+n_{1}\\right) \\leqslant 2$ for all $n_{1}1$, introduce the set $X_{m}$ like in Solution 2 and define $f(m)$ so as to satisfy $$ \\begin{array}{rll} f(m) \\equiv f(m-p) & (\\bmod p) & \\text { for all } p \\in X_{m} \\text { with } p10^{100}} \\alpha_{i} . $$ That is, $\\mho(n)$ is the number of prime factors of $n$ greater than $10^{100}$, counted with multiplicity. Find all strictly increasing functions $f: \\mathbb{Z} \\rightarrow \\mathbb{Z}$ such that $$ \\mho(f(a)-f(b)) \\leqslant \\mho(a-b) \\quad \\text { for all integers } a \\text { and } b \\text { with } a>b . $$ (Brazil) Answer. $f(x)=a x+b$, where $b$ is an arbitrary integer, and $a$ is an arbitrary positive integer with $\\mho(a)=0$.","solution":"A straightforward check shows that all the functions listed in the answer satisfy the problem condition. It remains to show the converse. Assume that $f$ is a function satisfying the problem condition. Notice that the function $g(x)=f(x)-f(0)$ also satisfies this condition. Replacing $f$ by $g$, we assume from now on that $f(0)=0$; then $f(n)>0$ for any positive integer $n$. Thus, we aim to prove that there exists a positive integer $a$ with $\\mho(a)=0$ such that $f(n)=a n$ for all $n \\in \\mathbb{Z}$. We start by introducing some notation. Set $N=10^{100}$. We say that a prime $p$ is large if $p>N$, and $p$ is small otherwise; let $\\mathcal{S}$ be the set of all small primes. Next, we say that a positive integer is large or small if all its prime factors are such (thus, the number 1 is the unique number which is both large and small). For a positive integer $k$, we denote the greatest large divisor of $k$ and the greatest small divisor of $k$ by $L(k)$ and $S(k)$, respectively; thus, $k=L(k) S(k)$. We split the proof into three steps. Step 1. We prove that for every large $k$, we have $k|f(a)-f(b) \\Longleftrightarrow k| a-b$. In other words, $L(f(a)-f(b))=L(a-b)$ for all integers $a$ and $b$ with $a>b$. We use induction on $k$. The base case $k=1$ is trivial. For the induction step, assume that $k_{0}$ is a large number, and that the statement holds for all large numbers $k$ with $k\\ell$. But then $$ \\mho(f(x)-f(y)) \\geqslant \\mho\\left(\\operatorname{lcm}\\left(k_{0}, \\ell\\right)\\right)>\\mho(\\ell)=\\mho(x-y), $$ which is impossible. Now we complete the induction step. By Claim 1, for every integer $a$ each of the sequences $$ f(a), f(a+1), \\ldots, f\\left(a+k_{0}-1\\right) \\quad \\text { and } \\quad f(a+1), f(a+2), \\ldots, f\\left(a+k_{0}\\right) $$ forms a complete residue system modulo $k_{0}$. This yields $f(a) \\equiv f\\left(a+k_{0}\\right)\\left(\\bmod k_{0}\\right)$. Thus, $f(a) \\equiv f(b)\\left(\\bmod k_{0}\\right)$ whenever $a \\equiv b\\left(\\bmod k_{0}\\right)$. Finally, if $a \\not \\equiv b\\left(\\bmod k_{0}\\right)$ then there exists an integer $b^{\\prime}$ such that $b^{\\prime} \\equiv b\\left(\\bmod k_{0}\\right)$ and $\\left|a-b^{\\prime}\\right|N$. Proof. Let $d$ be the product of all small primes, and let $\\alpha$ be a positive integer such that $2^{\\alpha}>f(N)$. Then, for every $p \\in \\mathcal{S}$ the numbers $f(0), f(1), \\ldots, f(N)$ are distinct modulo $p^{\\alpha}$. Set $P=d^{\\alpha}$ and $c=P+f(N)$. Choose any integer $t>N$. Due to the choice of $\\alpha$, for every $p \\in \\mathcal{S}$ there exists at most one nonnegative integer $i \\leqslant N$ with $p^{\\alpha} \\mid f(t)-f(i)$. Since $|\\mathcal{S}||f(x)-a x|$. Since $n-x \\equiv r-(r+1)=-1(\\bmod N!)$, the number $|n-x|$ is large. Therefore, by Step 1 we have $f(x) \\equiv f(n)=a n \\equiv a x(\\bmod n-x)$, so $n-x \\mid f(x)-a x$. Due to the choice of $n$, this yields $f(x)=a x$. To complete Step 3, notice that the set $\\mathcal{T}^{\\prime}$ found in Step 2 contains infinitely many elements of some residue class $R_{i}$. Applying Claim 3, we successively obtain that $f(x)=a x$ for all $x \\in R_{i+1}, R_{i+2}, \\ldots, R_{i+N!}=R_{i}$. This finishes the solution. Comment 1. As the proposer also mentions, one may also consider the version of the problem where the condition (1) is replaced by the condition that $L(f(a)-f(b))=L(a-b)$ for all integers $a$ and $b$ with $a>b$. This allows to remove of Step 1 from the solution. Comment 2. Step 2 is the main step of the solution. We sketch several different approaches allowing to perform this step using statements which are weaker than Claim 2. Approach 1. Let us again denote the product of all small primes by $d$. We focus on the values $f\\left(d^{i}\\right)$, $i \\geqslant 0$. In view of Step 1, we have $L\\left(f\\left(d^{i}\\right)-f\\left(d^{k}\\right)\\right)=L\\left(d^{i}-d^{k}\\right)=d^{i-k}-1$ for all $i>k \\geqslant 0$. Acting similarly to the beginning of the proof of Claim 2, one may choose a number $\\alpha \\geqslant 0$ such that the residues of the numbers $f\\left(d^{i}\\right), i=0,1, \\ldots, N$, are distinct modulo $p^{\\alpha}$ for each $p \\in \\mathcal{S}$. Then, for every $i>N$, there exists an exponent $k=k(i) \\leqslant N$ such that $S\\left(f\\left(d^{i}\\right)-f\\left(d^{k}\\right)\\right)N$ such that $k(i)$ attains the same value $k_{0}$ for all $i \\in I$, and such that, moreover, $S\\left(f\\left(d^{i}\\right)-f\\left(d^{k_{0}}\\right)\\right)$ attains the same value $s_{0}$ for all $i \\in I$. Therefore, for all such $i$ we have $$ f\\left(d^{i}\\right)=f\\left(d^{k_{0}}\\right)+L\\left(f\\left(d^{i}\\right)-f\\left(d^{k_{0}}\\right)\\right) \\cdot S\\left(f\\left(d^{i}\\right)-f\\left(d^{k_{0}}\\right)\\right)=f\\left(d^{k_{0}}\\right)+\\left(d^{i-k_{0}}-1\\right) s_{0}, $$ which means that $f$ is linear on the infinite set $\\left\\{d^{i}: i \\in I\\right\\}$ (although with rational coefficients). Finally, one may implement the relation $f\\left(d^{i}\\right) \\equiv f(1)\\left(\\bmod d^{i}-1\\right)$ in order to establish that in fact $f\\left(d^{i}\\right) \/ d^{i}$ is a (small and fixed) integer for all $i \\in I$. Approach 2. Alternatively, one may start with the following lemma. Lemma. There exists a positive constant $c$ such that $$ L\\left(\\prod_{i=1}^{3 N}(f(k)-f(i))\\right)=\\prod_{i=1}^{3 N} L(f(k)-f(i)) \\geqslant c(f(k))^{2 N} $$ for all $k>3 N$. Proof. Let $k$ be an integer with $k>3 N$. Set $\\Pi=\\prod_{i=1}^{3 N}(f(k)-f(i))$. Notice that for every prime $p \\in \\mathcal{S}$, at most one of the numbers in the set $$ \\mathcal{H}=\\{f(k)-f(i): 1 \\leqslant i \\leqslant 3 N\\} $$ is divisible by a power of $p$ which is greater than $f(3 N)$; we say that such elements of $\\mathcal{H}$ are bad. Now, for each element $h \\in \\mathcal{H}$ which is not bad we have $S(h) \\leqslant f(3 N)^{N}$, while the bad elements do not exceed $f(k)$. Moreover, there are less than $N$ bad elements in $\\mathcal{H}$. Therefore, $$ S(\\Pi)=\\prod_{h \\in \\mathcal{H}} S(h) \\leqslant(f(3 N))^{3 N^{2}} \\cdot(f(k))^{N} . $$ This easily yields the lemma statement in view of the fact that $L(\\Pi) S(\\Pi)=\\Pi \\geqslant \\mu(f(k))^{3 N}$ for some absolute constant $\\mu$. As a corollary of the lemma, one may get a weaker version of Claim 2 stating that there exists a positive constant $C$ such that $f(k) \\leqslant C k^{3 \/ 2}$ for all $k>3 N$. Indeed, from Step 1 we have $$ k^{3 N} \\geqslant \\prod_{i=1}^{3 N} L(k-i)=\\prod_{i=1}^{3 N} L(f(k)-f(i)) \\geqslant c(f(k))^{2 N} $$ so $f(k) \\leqslant c^{-1 \/(2 N)} k^{3 \/ 2}$. To complete Step 2 now, set $a=f(1)$. Due to the estimates above, we may choose a positive integer $n_{0}$ such that $|f(n)-a n|<\\frac{n(n-1)}{2}$ for all $n \\geqslant n_{0}$. Take any $n \\geqslant n_{0}$ with $n \\equiv 2(\\bmod N!)$. Then $L(f(n)-f(0))=L(n)=n \/ 2$ and $L(f(n)-f(1))=$ $L(n-1)=n-1$; these relations yield $f(n) \\equiv f(0)=0 \\equiv a n(\\bmod n \/ 2)$ and $f(n) \\equiv f(1)=a \\equiv a n$ $(\\bmod n-1)$, respectively. Thus, $\\left.\\frac{n(n-1)}{2} \\right\\rvert\\, f(n)-a n$, which shows that $f(n)=a n$ in view of the estimate above. Comment 3. In order to perform Step 3, it suffices to establish the equality $f(n)=a n$ for any infinite set of values of $n$. However, if this set has some good structure, then one may find easier ways to complete this step. For instance, after showing, as in Approach 2 , that $f(n)=$ an for all $n \\geqslant n_{0}$ with $n \\equiv 2(\\bmod N!)$, one may proceed as follows. Pick an arbitrary integer $x$ and take any large prime $p$ which is greater than $|f(x)-a x|$. By the Chinese Remainder Theorem, there exists a positive integer $n>\\max \\left(x, n_{0}\\right)$ such that $n \\equiv 2(\\bmod N!)$ and $n \\equiv x(\\bmod p)$. By Step 1 , we have $f(x) \\equiv f(n)=a n \\equiv a x(\\bmod p)$. Due to the choice of $p$, this is possible only if $f(x)=a x$.","tier":0} diff --git a/IMO/segmented/en-IMO2016SL.jsonl b/IMO/segmented/en-IMO2016SL.jsonl new file mode 100644 index 0000000000000000000000000000000000000000..557a7f133ccf69ce720bd1aacd6a53e240f32353 --- /dev/null +++ b/IMO/segmented/en-IMO2016SL.jsonl @@ -0,0 +1,78 @@ +{"problem_type":"Algebra","problem_label":"A1","problem":"Let $a, b$ and $c$ be positive real numbers such that $\\min \\{a b, b c, c a\\} \\geqslant 1$. Prove that $$ \\sqrt[3]{\\left(a^{2}+1\\right)\\left(b^{2}+1\\right)\\left(c^{2}+1\\right)} \\leqslant\\left(\\frac{a+b+c}{3}\\right)^{2}+1 $$","solution":"We first show the following. - Claim. For any positive real numbers $x, y$ with $x y \\geqslant 1$, we have $$ \\left(x^{2}+1\\right)\\left(y^{2}+1\\right) \\leqslant\\left(\\left(\\frac{x+y}{2}\\right)^{2}+1\\right)^{2} $$ Proof. Note that $x y \\geqslant 1$ implies $\\left(\\frac{x+y}{2}\\right)^{2}-1 \\geqslant x y-1 \\geqslant 0$. We find that $\\left(x^{2}+1\\right)\\left(y^{2}+1\\right)=(x y-1)^{2}+(x+y)^{2} \\leqslant\\left(\\left(\\frac{x+y}{2}\\right)^{2}-1\\right)^{2}+(x+y)^{2}=\\left(\\left(\\frac{x+y}{2}\\right)^{2}+1\\right)^{2}$. Without loss of generality, assume $a \\geqslant b \\geqslant c$. This implies $a \\geqslant 1$. Let $d=\\frac{a+b+c}{3}$. Note that $$ a d=\\frac{a(a+b+c)}{3} \\geqslant \\frac{1+1+1}{3}=1 . $$ Then we can apply (2) to the pair $(a, d)$ and the pair $(b, c)$. We get $$ \\left(a^{2}+1\\right)\\left(d^{2}+1\\right)\\left(b^{2}+1\\right)\\left(c^{2}+1\\right) \\leqslant\\left(\\left(\\frac{a+d}{2}\\right)^{2}+1\\right)^{2}\\left(\\left(\\frac{b+c}{2}\\right)^{2}+1\\right)^{2} $$ Next, from $$ \\frac{a+d}{2} \\cdot \\frac{b+c}{2} \\geqslant \\sqrt{a d} \\cdot \\sqrt{b c} \\geqslant 1 $$ we can apply (2) again to the pair $\\left(\\frac{a+d}{2}, \\frac{b+c}{2}\\right)$. Together with (3), we have $$ \\left(a^{2}+1\\right)\\left(d^{2}+1\\right)\\left(b^{2}+1\\right)\\left(c^{2}+1\\right) \\leqslant\\left(\\left(\\frac{a+b+c+d}{4}\\right)^{2}+1\\right)^{4}=\\left(d^{2}+1\\right)^{4} $$ Therefore, $\\left(a^{2}+1\\right)\\left(b^{2}+1\\right)\\left(c^{2}+1\\right) \\leqslant\\left(d^{2}+1\\right)^{3}$, and (1) follows by taking cube root of both sides. Comment. After justifying the Claim, one may also obtain (1) by mixing variables. Indeed, the function involved is clearly continuous, and hence it suffices to check that the condition $x y \\geqslant 1$ is preserved under each mixing step. This is true since whenever $a b, b c, c a \\geqslant 1$, we have $$ \\frac{a+b}{2} \\cdot \\frac{a+b}{2} \\geqslant a b \\geqslant 1 \\quad \\text { and } \\quad \\frac{a+b}{2} \\cdot c \\geqslant \\frac{1+1}{2}=1 . $$","tier":0} +{"problem_type":"Algebra","problem_label":"A1","problem":"Let $a, b$ and $c$ be positive real numbers such that $\\min \\{a b, b c, c a\\} \\geqslant 1$. Prove that $$ \\sqrt[3]{\\left(a^{2}+1\\right)\\left(b^{2}+1\\right)\\left(c^{2}+1\\right)} \\leqslant\\left(\\frac{a+b+c}{3}\\right)^{2}+1 $$","solution":"Let $f(x)=\\ln \\left(1+x^{2}\\right)$. Then the inequality (1) to be shown is equivalent to $$ \\frac{f(a)+f(b)+f(c)}{3} \\leqslant f\\left(\\frac{a+b+c}{3}\\right), $$ while (2) becomes $$ \\frac{f(x)+f(y)}{2} \\leqslant f\\left(\\frac{x+y}{2}\\right) $$ for $x y \\geqslant 1$. Without loss of generality, assume $a \\geqslant b \\geqslant c$. From the Claim in Solution 1, we have $$ \\frac{f(a)+f(b)+f(c)}{3} \\leqslant \\frac{f(a)+2 f\\left(\\frac{b+c}{2}\\right)}{3} . $$ Note that $a \\geqslant 1$ and $\\frac{b+c}{2} \\geqslant \\sqrt{b c} \\geqslant 1$. Since $$ f^{\\prime \\prime}(x)=\\frac{2\\left(1-x^{2}\\right)}{\\left(1+x^{2}\\right)^{2}} $$ we know that $f$ is concave on $[1, \\infty)$. Then we can apply Jensen's Theorem to get $$ \\frac{f(a)+2 f\\left(\\frac{b+c}{2}\\right)}{3} \\leqslant f\\left(\\frac{a+2 \\cdot \\frac{b+c}{2}}{3}\\right)=f\\left(\\frac{a+b+c}{3}\\right) . $$ This completes the proof.","tier":0} +{"problem_type":"Algebra","problem_label":"A2","problem":"Find the smallest real constant $C$ such that for any positive real numbers $a_{1}, a_{2}, a_{3}, a_{4}$ and $a_{5}$ (not necessarily distinct), one can always choose distinct subscripts $i, j, k$ and $l$ such that $$ \\left|\\frac{a_{i}}{a_{j}}-\\frac{a_{k}}{a_{l}}\\right| \\leqslant C $$","solution":"We first show that $C \\leqslant \\frac{1}{2}$. For any positive real numbers $a_{1} \\leqslant a_{2} \\leqslant a_{3} \\leqslant a_{4} \\leqslant a_{5}$, consider the five fractions $$ \\frac{a_{1}}{a_{2}}, \\frac{a_{3}}{a_{4}}, \\frac{a_{1}}{a_{5}}, \\frac{a_{2}}{a_{3}}, \\frac{a_{4}}{a_{5}} . $$ Each of them lies in the interval $(0,1]$. Therefore, by the Pigeonhole Principle, at least three of them must lie in $\\left(0, \\frac{1}{2}\\right]$ or lie in $\\left(\\frac{1}{2}, 1\\right]$ simultaneously. In particular, there must be two consecutive terms in (2) which belong to an interval of length $\\frac{1}{2}$ (here, we regard $\\frac{a_{1}}{a_{2}}$ and $\\frac{a_{4}}{a_{5}}$ as consecutive). In other words, the difference of these two fractions is less than $\\frac{1}{2}$. As the indices involved in these two fractions are distinct, we can choose them to be $i, j, k, l$ and conclude that $C \\leqslant \\frac{1}{2}$. Next, we show that $C=\\frac{1}{2}$ is best possible. Consider the numbers $1,2,2,2, n$ where $n$ is a large real number. The fractions formed by two of these numbers in ascending order are $\\frac{1}{n}, \\frac{2}{n}, \\frac{1}{2}, \\frac{2}{2}, \\frac{2}{1}, \\frac{n}{2}, \\frac{n}{1}$. Since the indices $i, j, k, l$ are distinct, $\\frac{1}{n}$ and $\\frac{2}{n}$ cannot be chosen simultaneously. Therefore the minimum value of the left-hand side of (1) is $\\frac{1}{2}-\\frac{2}{n}$. When $n$ tends to infinity, this value approaches $\\frac{1}{2}$, and so $C$ cannot be less than $\\frac{1}{2}$. These conclude that $C=\\frac{1}{2}$ is the smallest possible choice. Comment. The conclusion still holds if $a_{1}, a_{2}, \\ldots, a_{5}$ are pairwise distinct, since in the construction, we may replace the 2's by real numbers sufficiently close to 2 . There are two possible simplifications for this problem: (i) the answer $C=\\frac{1}{2}$ is given to the contestants; or (ii) simply ask the contestants to prove the inequality (1) for $C=\\frac{1}{2}$.","tier":0} +{"problem_type":"Algebra","problem_label":"A3","problem":"Find all integers $n \\geqslant 3$ with the following property: for all real numbers $a_{1}, a_{2}, \\ldots, a_{n}$ and $b_{1}, b_{2}, \\ldots, b_{n}$ satisfying $\\left|a_{k}\\right|+\\left|b_{k}\\right|=1$ for $1 \\leqslant k \\leqslant n$, there exist $x_{1}, x_{2}, \\ldots, x_{n}$, each of which is either -1 or 1 , such that $$ \\left|\\sum_{k=1}^{n} x_{k} a_{k}\\right|+\\left|\\sum_{k=1}^{n} x_{k} b_{k}\\right| \\leqslant 1 $$","solution":"For any even integer $n \\geqslant 4$, we consider the case $$ a_{1}=a_{2}=\\cdots=a_{n-1}=b_{n}=0 \\quad \\text { and } \\quad b_{1}=b_{2}=\\cdots=b_{n-1}=a_{n}=1 $$ The condition $\\left|a_{k}\\right|+\\left|b_{k}\\right|=1$ is satisfied for each $1 \\leqslant k \\leqslant n$. No matter how we choose each $x_{k}$, both sums $\\sum_{k=1}^{n} x_{k} a_{k}$ and $\\sum_{k=1}^{n} x_{k} b_{k}$ are odd integers. This implies $\\left|\\sum_{k=1}^{n} x_{k} a_{k}\\right| \\geqslant 1$ and $\\left|\\sum_{k=1}^{n} x_{k} b_{k}\\right| \\geqslant 1$, which shows (1) cannot hold. For any odd integer $n \\geqslant 3$, we may assume without loss of generality $b_{k} \\geqslant 0$ for $1 \\leqslant k \\leqslant n$ (this can be done by flipping the pair $\\left(a_{k}, b_{k}\\right)$ to $\\left(-a_{k},-b_{k}\\right)$ and $x_{k}$ to $-x_{k}$ if necessary) and $a_{1} \\geqslant a_{2} \\geqslant \\cdots \\geqslant a_{m} \\geqslant 0>a_{m+1} \\geqslant \\cdots \\geqslant a_{n}$. We claim that the choice $x_{k}=(-1)^{k+1}$ for $1 \\leqslant k \\leqslant n$ will work. Define $$ s=\\sum_{k=1}^{m} x_{k} a_{k} \\quad \\text { and } \\quad t=-\\sum_{k=m+1}^{n} x_{k} a_{k} . $$ Note that $$ s=\\left(a_{1}-a_{2}\\right)+\\left(a_{3}-a_{4}\\right)+\\cdots \\geqslant 0 $$ by the assumption $a_{1} \\geqslant a_{2} \\geqslant \\cdots \\geqslant a_{m}$ (when $m$ is odd, there is a single term $a_{m}$ at the end, which is also positive). Next, we have $$ s=a_{1}-\\left(a_{2}-a_{3}\\right)-\\left(a_{4}-a_{5}\\right)-\\cdots \\leqslant a_{1} \\leqslant 1 $$ Similarly, $$ t=\\left(-a_{n}+a_{n-1}\\right)+\\left(-a_{n-2}+a_{n-3}\\right)+\\cdots \\geqslant 0 $$ and $$ t=-a_{n}+\\left(a_{n-1}-a_{n-2}\\right)+\\left(a_{n-3}-a_{n-4}\\right)+\\cdots \\leqslant-a_{n} \\leqslant 1 . $$ From the condition, we have $a_{k}+b_{k}=1$ for $1 \\leqslant k \\leqslant m$ and $-a_{k}+b_{k}=1$ for $m+1 \\leqslant k \\leqslant n$. It follows that $\\sum_{k=1}^{n} x_{k} a_{k}=s-t$ and $\\sum_{k=1}^{n} x_{k} b_{k}=1-s-t$. Hence it remains to prove $$ |s-t|+|1-s-t| \\leqslant 1 $$ under the constraint $0 \\leqslant s, t \\leqslant 1$. By symmetry, we may assume $s \\geqslant t$. If $1-s-t \\geqslant 0$, then we have $$ |s-t|+|1-s-t|=s-t+1-s-t=1-2 t \\leqslant 1 $$ If $1-s-t \\leqslant 0$, then we have $$ |s-t|+|1-s-t|=s-t-1+s+t=2 s-1 \\leqslant 1 $$ Hence, the inequality is true in both cases. These show $n$ can be any odd integer greater than or equal to 3 .","tier":0} +{"problem_type":"Algebra","problem_label":"A3","problem":"Find all integers $n \\geqslant 3$ with the following property: for all real numbers $a_{1}, a_{2}, \\ldots, a_{n}$ and $b_{1}, b_{2}, \\ldots, b_{n}$ satisfying $\\left|a_{k}\\right|+\\left|b_{k}\\right|=1$ for $1 \\leqslant k \\leqslant n$, there exist $x_{1}, x_{2}, \\ldots, x_{n}$, each of which is either -1 or 1 , such that $$ \\left|\\sum_{k=1}^{n} x_{k} a_{k}\\right|+\\left|\\sum_{k=1}^{n} x_{k} b_{k}\\right| \\leqslant 1 $$","solution":"The even case can be handled in the same way as Firstly, for $n=3$, we may assume without loss of generality $a_{1} \\geqslant a_{2} \\geqslant a_{3} \\geqslant 0$ and $b_{1}=a_{1}-1$ (if $b_{1}=1-a_{1}$, we may replace each $b_{k}$ by $-b_{k}$ ). - Case 1. $b_{2}=a_{2}-1$ and $b_{3}=a_{3}-1$, in which case we take $\\left(x_{1}, x_{2}, x_{3}\\right)=(1,-1,1)$. Let $c=a_{1}-a_{2}+a_{3}$ so that $0 \\leqslant c \\leqslant 1$. Then $\\left|b_{1}-b_{2}+b_{3}\\right|=\\left|a_{1}-a_{2}+a_{3}-1\\right|=1-c$ and hence $|c|+\\left|b_{1}-b_{2}+b_{3}\\right|=1$. - Case 2. $b_{2}=1-a_{2}$ and $b_{3}=1-a_{3}$, in which case we take $\\left(x_{1}, x_{2}, x_{3}\\right)=(1,-1,1)$. Let $c=a_{1}-a_{2}+a_{3}$ so that $0 \\leqslant c \\leqslant 1$. Since $a_{3} \\leqslant a_{2}$ and $a_{1} \\leqslant 1$, we have $$ c-1 \\leqslant b_{1}-b_{2}+b_{3}=a_{1}+a_{2}-a_{3}-1 \\leqslant 1-c $$ This gives $\\left|b_{1}-b_{2}+b_{3}\\right| \\leqslant 1-c$ and hence $|c|+\\left|b_{1}-b_{2}+b_{3}\\right| \\leqslant 1$. - Case 3. $b_{2}=a_{2}-1$ and $b_{3}=1-a_{3}$, in which case we take $\\left(x_{1}, x_{2}, x_{3}\\right)=(-1,1,1)$. Let $c=-a_{1}+a_{2}+a_{3}$. If $c \\geqslant 0$, then $a_{3} \\leqslant 1$ and $a_{2} \\leqslant a_{1}$ imply $$ c-1 \\leqslant-b_{1}+b_{2}+b_{3}=-a_{1}+a_{2}-a_{3}+1 \\leqslant 1-c $$ If $c<0$, then $a_{1} \\leqslant a_{2}+1$ and $a_{3} \\geqslant 0$ imply $$ -c-1 \\leqslant-b_{1}+b_{2}+b_{3}=-a_{1}+a_{2}-a_{3}+1 \\leqslant 1+c $$ In both cases, we get $\\left|-b_{1}+b_{2}+b_{3}\\right| \\leqslant 1-|c|$ and hence $|c|+\\left|-b_{1}+b_{2}+b_{3}\\right| \\leqslant 1$. - Case 4. $b_{2}=1-a_{2}$ and $b_{3}=a_{3}-1$, in which case we take $\\left(x_{1}, x_{2}, x_{3}\\right)=(-1,1,1)$. Let $c=-a_{1}+a_{2}+a_{3}$. If $c \\geqslant 0$, then $a_{2} \\leqslant 1$ and $a_{3} \\leqslant a_{1}$ imply $$ c-1 \\leqslant-b_{1}+b_{2}+b_{3}=-a_{1}-a_{2}+a_{3}+1 \\leqslant 1-c $$ If $c<0$, then $a_{1} \\leqslant a_{3}+1$ and $a_{2} \\geqslant 0$ imply $$ -c-1 \\leqslant-b_{1}+b_{2}+b_{3}=-a_{1}-a_{2}+a_{3}+1 \\leqslant 1+c $$ In both cases, we get $\\left|-b_{1}+b_{2}+b_{3}\\right| \\leqslant 1-|c|$ and hence $|c|+\\left|-b_{1}+b_{2}+b_{3}\\right| \\leqslant 1$. We have found $x_{1}, x_{2}, x_{3}$ satisfying (1) in each case for $n=3$. Now, let $n \\geqslant 5$ be odd and suppose the result holds for any smaller odd cases. Again we may assume $a_{k} \\geqslant 0$ for each $1 \\leqslant k \\leqslant n$. By the Pigeonhole Principle, there are at least three indices $k$ for which $b_{k}=a_{k}-1$ or $b_{k}=1-a_{k}$. Without loss of generality, suppose $b_{k}=a_{k}-1$ for $k=1,2,3$. Again by the Pigeonhole Principle, as $a_{1}, a_{2}, a_{3}$ lies between 0 and 1 , the difference of two of them is at most $\\frac{1}{2}$. By changing indices if necessary, we may assume $0 \\leqslant d=a_{1}-a_{2} \\leqslant \\frac{1}{2}$. By the inductive hypothesis, we can choose $x_{3}, x_{4}, \\ldots, x_{n}$ such that $a^{\\prime}=\\sum_{k=3}^{n} x_{k} a_{k}$ and $b^{\\prime}=\\sum_{k=3}^{n} x_{k} b_{k}$ satisfy $\\left|a^{\\prime}\\right|+\\left|b^{\\prime}\\right| \\leqslant 1$. We may further assume $a^{\\prime} \\geqslant 0$. - Case 1. $b^{\\prime} \\geqslant 0$, in which case we take $\\left(x_{1}, x_{2}\\right)=(-1,1)$. We have $\\left|-a_{1}+a_{2}+a^{\\prime}\\right|+\\left|-\\left(a_{1}-1\\right)+\\left(a_{2}-1\\right)+b^{\\prime}\\right|=\\left|-d+a^{\\prime}\\right|+\\left|-d+b^{\\prime}\\right| \\leqslant$ $\\max \\left\\{a^{\\prime}+b^{\\prime}-2 d, a^{\\prime}-b^{\\prime}, b^{\\prime}-a^{\\prime}, 2 d-a^{\\prime}-b^{\\prime}\\right\\} \\leqslant 1$ since $0 \\leqslant a^{\\prime}, b^{\\prime}, a^{\\prime}+b^{\\prime} \\leqslant 1$ and $0 \\leqslant d \\leqslant \\frac{1}{2}$. - Case 2. $0>b^{\\prime} \\geqslant-a^{\\prime}$, in which case we take $\\left(x_{1}, x_{2}\\right)=(-1,1)$. We have $\\left|-a_{1}+a_{2}+a^{\\prime}\\right|+\\left|-\\left(a_{1}-1\\right)+\\left(a_{2}-1\\right)+b^{\\prime}\\right|=\\left|-d+a^{\\prime}\\right|+\\left|-d+b^{\\prime}\\right|$. If $-d+a^{\\prime} \\geqslant 0$, this equals $a^{\\prime}-b^{\\prime}=\\left|a^{\\prime}\\right|+\\left|b^{\\prime}\\right| \\leqslant 1$. If $-d+a^{\\prime}<0$, this equals $2 d-a^{\\prime}-b^{\\prime} \\leqslant 2 d \\leqslant 1$. - Case 3. $b^{\\prime}<-a^{\\prime}$, in which case we take $\\left(x_{1}, x_{2}\\right)=(1,-1)$. We have $\\left|a_{1}-a_{2}+a^{\\prime}\\right|+\\left|\\left(a_{1}-1\\right)-\\left(a_{2}-1\\right)+b^{\\prime}\\right|=\\left|d+a^{\\prime}\\right|+\\left|d+b^{\\prime}\\right|$. If $d+b^{\\prime} \\geqslant 0$, this equals $2 d+a^{\\prime}+b^{\\prime}<2 d \\leqslant 1$. If $d+b^{\\prime}<0$, this equals $a^{\\prime}-b^{\\prime}=\\left|a^{\\prime}\\right|+\\left|b^{\\prime}\\right| \\leqslant 1$. Therefore, we have found $x_{1}, x_{2}, \\ldots, x_{n}$ satisfying (1) in each case. By induction, the property holds for all odd integers $n \\geqslant 3$.","tier":0} +{"problem_type":"Algebra","problem_label":"A4","problem":"Denote by $\\mathbb{R}^{+}$the set of all positive real numbers. Find all functions $f: \\mathbb{R}^{+} \\rightarrow \\mathbb{R}^{+}$such that $$ x f\\left(x^{2}\\right) f(f(y))+f(y f(x))=f(x y)\\left(f\\left(f\\left(x^{2}\\right)\\right)+f\\left(f\\left(y^{2}\\right)\\right)\\right) $$ for all positive real numbers $x$ and $y$.","solution":"Taking $x=y=1$ in (1), we get $f(1) f(f(1))+f(f(1))=2 f(1) f(f(1))$ and hence $f(1)=1$. Putting $x=1$ in (1), we have $f(f(y))+f(y)=f(y)\\left(1+f\\left(f\\left(y^{2}\\right)\\right)\\right)$ so that $$ f(f(y))=f(y) f\\left(f\\left(y^{2}\\right)\\right) $$ Putting $y=1$ in (1), we get $x f\\left(x^{2}\\right)+f(f(x))=f(x)\\left(f\\left(f\\left(x^{2}\\right)\\right)+1\\right)$. Using (9), this gives $$ x f\\left(x^{2}\\right)=f(x) $$ Replace $y$ by $\\frac{1}{x}$ in (1). Then we have $$ x f\\left(x^{2}\\right) f\\left(f\\left(\\frac{1}{x}\\right)\\right)+f\\left(\\frac{f(x)}{x}\\right)=f\\left(f\\left(x^{2}\\right)\\right)+f\\left(f\\left(\\frac{1}{x^{2}}\\right)\\right) . $$ The relation (10) shows $f\\left(\\frac{f(x)}{x}\\right)=f\\left(f\\left(x^{2}\\right)\\right)$. Also, using (9) with $y=\\frac{1}{x}$ and using (10) again, the last equation reduces to $$ f(x) f\\left(\\frac{1}{x}\\right)=1 $$ Replace $x$ by $\\frac{1}{x}$ and $y$ by $\\frac{1}{y}$ in (1) and apply (11). We get $$ \\frac{1}{x f\\left(x^{2}\\right) f(f(y))}+\\frac{1}{f(y f(x))}=\\frac{1}{f(x y)}\\left(\\frac{1}{f\\left(f\\left(x^{2}\\right)\\right)}+\\frac{1}{f\\left(f\\left(y^{2}\\right)\\right)}\\right) . $$ Clearing denominators, we can use (1) to simplify the numerators and obtain $$ f(x y)^{2} f\\left(f\\left(x^{2}\\right)\\right) f\\left(f\\left(y^{2}\\right)\\right)=x f\\left(x^{2}\\right) f(f(y)) f(y f(x)) $$ Using (9) and (10), this is the same as $$ f(x y)^{2} f(f(x))=f(x)^{2} f(y) f(y f(x)) $$ Substitute $y=f(x)$ in (12) and apply (10) (with $x$ replaced by $f(x)$ ). We have $$ f(x f(x))^{2}=f(x) f(f(x)) $$ Taking $y=x$ in (12), squaring both sides, and using (10) and (13), we find that $$ f(f(x))=x^{4} f(x)^{3} $$ Finally, we combine (9), (10) and (14) to get $$ y^{4} f(y)^{3} \\stackrel{(14)}{=} f(f(y)) \\stackrel{(9)}{=} f(y) f\\left(f\\left(y^{2}\\right)\\right) \\stackrel{(14)}{=} f(y) y^{8} f\\left(y^{2}\\right)^{3} \\stackrel{(10)}{=} y^{5} f(y)^{4}, $$ which implies $f(y)=\\frac{1}{y}$. This is a solution by the checking in Solution 1.","tier":0} +{"problem_type":"Algebra","problem_label":"A5","problem":"(a) Prove that for every positive integer $n$, there exists a fraction $\\frac{a}{b}$ where $a$ and $b$ are integers satisfying $02016$ or $4 k2$ for $0 \\leqslant j \\leqslant 503$ in this case. So each term in the product lies strictly between 0 and 1 , and the whole product must be less than 1 , which is impossible. - Case 4. $4 k+2\\frac{n+1}{2}$, we have $|l(x)|>|r(x)|$. If $n \\equiv 3(\\bmod 4)$, one may leave $l(x)=(x-1)(x-2) \\cdots\\left(x-\\frac{n+1}{2}\\right)$ on the left-hand side and $r(x)=\\left(x-\\frac{n+3}{2}\\right)\\left(x-\\frac{x+5}{2}\\right) \\cdots(x-n)$ on the right-hand side. For $x<1$ or $\\frac{n+1}{2}0>r(x)$. For $1\\frac{n+3}{2}$, we have $|l(x)|>|r(x)|$. If $n \\equiv 1(\\bmod 4)$, as the proposer mentioned, the situation is a bit more out of control. Since the construction for $n-1 \\equiv 0(\\bmod 4)$ works, the answer can be either $n$ or $n-1$. For $n=5$, we can leave the products $(x-1)(x-2)(x-3)(x-4)$ and $(x-5)$. For $n=9$, the only example that works is $l(x)=(x-1)(x-2)(x-9)$ and $r(x)=(x-3)(x-4) \\cdots(x-8)$, while there seems to be no such partition for $n=13$.","tier":0} +{"problem_type":"Algebra","problem_label":"A7","problem":"Denote by $\\mathbb{R}$ the set of all real numbers. Find all functions $f: \\mathbb{R} \\rightarrow \\mathbb{R}$ such that $f(0) \\neq 0$ and $$ f(x+y)^{2}=2 f(x) f(y)+\\max \\left\\{f\\left(x^{2}\\right)+f\\left(y^{2}\\right), f\\left(x^{2}+y^{2}\\right)\\right\\} $$ for all real numbers $x$ and $y$.","solution":"Taking $x=y=0$ in (1), we get $f(0)^{2}=2 f(0)^{2}+\\max \\{2 f(0), f(0)\\}$. If $f(0)>0$, then $f(0)^{2}+2 f(0)=0$ gives no positive solution. If $f(0)<0$, then $f(0)^{2}+f(0)=0$ gives $f(0)=-1$. Putting $y=0$ in (1), we have $f(x)^{2}=-2 f(x)+f\\left(x^{2}\\right)$, which is the same as $(f(x)+1)^{2}=f\\left(x^{2}\\right)+1$. Let $g(x)=f(x)+1$. Then for any $x \\in \\mathbb{R}$, we have $$ g\\left(x^{2}\\right)=g(x)^{2} \\geqslant 0 $$ From (1), we find that $f(x+y)^{2} \\geqslant 2 f(x) f(y)+f\\left(x^{2}\\right)+f\\left(y^{2}\\right)$. In terms of $g$, this becomes $(g(x+y)-1)^{2} \\geqslant 2(g(x)-1)(g(y)-1)+g\\left(x^{2}\\right)+g\\left(y^{2}\\right)-2$. Using (2), this means $$ (g(x+y)-1)^{2} \\geqslant(g(x)+g(y)-1)^{2}-1 $$ Putting $x=1$ in (2), we get $g(1)=0$ or 1 . The two cases are handled separately. - Case 1. $g(1)=0$, which is the same as $f(1)=-1$. We put $x=-1$ and $y=0$ in (1). This gives $f(-1)^{2}=-2 f(-1)-1$, which forces $f(-1)=-1$. Next, we take $x=-1$ and $y=1$ in (1) to get $1=2+\\max \\{-2, f(2)\\}$. This clearly implies $1=2+f(2)$ and hence $f(2)=-1$, that is, $g(2)=0$. From (2), we can prove inductively that $g\\left(2^{2^{n}}\\right)=g(2)^{2^{n}}=0$ for any $n \\in \\mathbb{N}$. Substitute $y=2^{2^{n}}-x$ in (3). We obtain $$ \\left(g(x)+g\\left(2^{2^{n}}-x\\right)-1\\right)^{2} \\leqslant\\left(g\\left(2^{2^{n}}\\right)-1\\right)^{2}+1=2 $$ For any fixed $x \\geqslant 0$, we consider $n$ to be sufficiently large so that $2^{2^{n}}-x>0$. From (2), this implies $g\\left(2^{2^{n}}-x\\right) \\geqslant 0$ so that $g(x) \\leqslant 1+\\sqrt{2}$. Using (2) again, we get $$ g(x)^{2^{n}}=g\\left(x^{2^{n}}\\right) \\leqslant 1+\\sqrt{2} $$ for any $n \\in \\mathbb{N}$. Therefore, $|g(x)| \\leqslant 1$ for any $x \\geqslant 0$. If there exists $a \\in \\mathbb{R}$ for which $g(a) \\neq 0$, then for sufficiently large $n$ we must have $g\\left(\\left(a^{2}\\right)^{\\frac{1}{2^{n}}}\\right)=g\\left(a^{2}\\right)^{\\frac{1}{2^{n}}}>\\frac{1}{2}$. By taking $x=-y=-\\left(a^{2}\\right)^{\\frac{1}{2^{n}}}$ in (1), we obtain $$ \\begin{aligned} 1 & =2 f(x) f(-x)+\\max \\left\\{2 f\\left(x^{2}\\right), f\\left(2 x^{2}\\right)\\right\\} \\\\ & =2(g(x)-1)(g(-x)-1)+\\max \\left\\{2\\left(g\\left(x^{2}\\right)-1\\right), g\\left(2 x^{2}\\right)-1\\right\\} \\\\ & \\leqslant 2\\left(-\\frac{1}{2}\\right)\\left(-\\frac{1}{2}\\right)+0=\\frac{1}{2} \\end{aligned} $$ since $|g(-x)|=|g(x)| \\in\\left(\\frac{1}{2}, 1\\right]$ by (2) and the choice of $x$, and since $g(z) \\leqslant 1$ for $z \\geqslant 0$. This yields a contradiction and hence $g(x)=0$ must hold for any $x$. This means $f(x)=-1$ for any $x \\in \\mathbb{R}$, which clearly satisfies (1). - Case 2. $g(1)=1$, which is the same as $f(1)=0$. We put $x=-1$ and $y=1$ in (1) to get $1=\\max \\{0, f(2)\\}$. This clearly implies $f(2)=1$ and hence $g(2)=2$. Setting $x=2 n$ and $y=2$ in (3), we have $$ (g(2 n+2)-1)^{2} \\geqslant(g(2 n)+1)^{2}-1 $$ By induction on $n$, it is easy to prove that $g(2 n) \\geqslant n+1$ for all $n \\in \\mathbb{N}$. For any real number $a>1$, we choose a large $n \\in \\mathbb{N}$ and take $k$ to be the positive integer such that $2 k \\leqslant a^{2^{n}}<2 k+2$. From (2) and (3), we have $$ \\left(g(a)^{2^{n}}-1\\right)^{2}+1=\\left(g\\left(a^{2^{n}}\\right)-1\\right)^{2}+1 \\geqslant\\left(g(2 k)+g\\left(a^{2^{n}}-2 k\\right)-1\\right)^{2} \\geqslant k^{2}>\\frac{1}{4}\\left(a^{2^{n}}-2\\right)^{2} $$ since $g\\left(a^{2^{n}}-2 k\\right) \\geqslant 0$. For large $n$, this clearly implies $g(a)^{2^{n}}>1$. Thus, $$ \\left(g(a)^{2^{n}}\\right)^{2}>\\left(g(a)^{2^{n}}-1\\right)^{2}+1>\\frac{1}{4}\\left(a^{2^{n}}-2\\right)^{2} $$ This yields $$ g(a)^{2^{n}}>\\frac{1}{2}\\left(a^{2^{n}}-2\\right) $$ Note that $$ \\frac{a^{2^{n}}}{a^{2^{n}}-2}=1+\\frac{2}{a^{2^{n}}-2} \\leqslant\\left(1+\\frac{2}{2^{n}\\left(a^{2^{n}}-2\\right)}\\right)^{2^{n}} $$ by binomial expansion. This can be rewritten as $$ \\left(a^{2^{n}}-2\\right)^{\\frac{1}{2^{n}}} \\geqslant \\frac{a}{1+\\frac{2}{2^{n}\\left(a^{2^{n}}-2\\right)}} $$ Together with (4), we conclude $g(a) \\geqslant a$ by taking $n$ sufficiently large. Consider $x=n a$ and $y=a>1$ in (3). This gives $(g((n+1) a)-1)^{2} \\geqslant(g(n a)+g(a)-1)^{2}-1$. By induction on $n$, it is easy to show $g(n a) \\geqslant(n-1)(g(a)-1)+a$ for any $n \\in \\mathbb{N}$. We choose a large $n \\in \\mathbb{N}$ and take $k$ to be the positive integer such that $k a \\leqslant 2^{2^{n}}<(k+1) a$. Using (2) and (3), we have $2^{2^{n+1}}>\\left(2^{2^{n}}-1\\right)^{2}+1=\\left(g\\left(2^{2^{n}}\\right)-1\\right)^{2}+1 \\geqslant\\left(g\\left(2^{2^{n}}-k a\\right)+g(k a)-1\\right)^{2} \\geqslant((k-1)(g(a)-1)+a-1)^{2}$, from which it follows that $$ 2^{2^{n}} \\geqslant(k-1)(g(a)-1)+a-1>\\frac{2^{2^{n}}}{a}(g(a)-1)-2(g(a)-1)+a-1 $$ holds for sufficiently large $n$. Hence, we must have $\\frac{g(a)-1}{a} \\leqslant 1$, which implies $g(a) \\leqslant a+1$ for any $a>1$. Then for large $n \\in \\mathbb{N}$, from (3) and (2) we have $$ 4 a^{2^{n+1}}=\\left(2 a^{2^{n}}\\right)^{2} \\geqslant\\left(g\\left(2 a^{2^{n}}\\right)-1\\right)^{2} \\geqslant\\left(2 g\\left(a^{2^{n}}\\right)-1\\right)^{2}-1=\\left(2 g(a)^{2^{n}}-1\\right)^{2}-1 $$ This implies $$ 2 a^{2^{n}}>\\frac{1}{2}\\left(1+\\sqrt{4 a^{2^{n+1}}+1}\\right) \\geqslant g(a)^{2^{n}} $$ When $n$ tends to infinity, this forces $g(a) \\leqslant a$. Together with $g(a) \\geqslant a$, we get $g(a)=a$ for all real numbers $a>1$, that is, $f(a)=a-1$ for all $a>1$. Finally, for any $x \\in \\mathbb{R}$, we choose $y$ sufficiently large in (1) so that $y, x+y>1$. This gives $(x+y-1)^{2}=2 f(x)(y-1)+\\max \\left\\{f\\left(x^{2}\\right)+y^{2}-1, x^{2}+y^{2}-1\\right\\}$, which can be rewritten as $$ 2(x-1-f(x)) y=-x^{2}+2 x-2-2 f(x)+\\max \\left\\{f\\left(x^{2}\\right), x^{2}\\right\\} $$ As the right-hand side is fixed, this can only hold for all large $y$ when $f(x)=x-1$. We now check that this function satisfies (1). Indeed, we have $$ \\begin{aligned} f(x+y)^{2} & =(x+y-1)^{2}=2(x-1)(y-1)+\\left(x^{2}+y^{2}-1\\right) \\\\ & =2 f(x) f(y)+\\max \\left\\{f\\left(x^{2}\\right)+f\\left(y^{2}\\right), f\\left(x^{2}+y^{2}\\right)\\right\\} . \\end{aligned} $$","tier":0} +{"problem_type":"Algebra","problem_label":"A7","problem":"Denote by $\\mathbb{R}$ the set of all real numbers. Find all functions $f: \\mathbb{R} \\rightarrow \\mathbb{R}$ such that $f(0) \\neq 0$ and $$ f(x+y)^{2}=2 f(x) f(y)+\\max \\left\\{f\\left(x^{2}\\right)+f\\left(y^{2}\\right), f\\left(x^{2}+y^{2}\\right)\\right\\} $$ for all real numbers $x$ and $y$.","solution":"Taking $x=y=0$ in (1), we get $f(0)^{2}=2 f(0)^{2}+\\max \\{2 f(0), f(0)\\}$. If $f(0)>0$, then $f(0)^{2}+2 f(0)=0$ gives no positive solution. If $f(0)<0$, then $f(0)^{2}+f(0)=0$ gives $f(0)=-1$. Putting $y=0$ in (1), we have $$ f(x)^{2}=-2 f(x)+f\\left(x^{2}\\right) $$ Replace $x$ by $-x$ in (5) and compare with (5) again. We get $f(x)^{2}+2 f(x)=f(-x)^{2}+2 f(-x)$, which implies $$ f(x)=f(-x) \\quad \\text { or } \\quad f(x)+f(-x)=-2 $$ Taking $x=y$ and $x=-y$ respectively in (1) and comparing the two equations obtained, we have $$ f(2 x)^{2}-2 f(x)^{2}=1-2 f(x) f(-x) . $$ Combining (6) and (7) to eliminate $f(-x)$, we find that $f(2 x)$ can be $\\pm 1$ (when $f(x)=f(-x)$ ) or $\\pm(2 f(x)+1)$ (when $f(x)+f(-x)=-2$ ). We prove the following. - Claim. $f(x)+f(-x)=-2$ for any $x \\in \\mathbb{R}$. Proof. Suppose there exists $a \\in \\mathbb{R}$ such that $f(a)+f(-a) \\neq-2$. Then $f(a)=f(-a) \\neq-1$ and we may assume $a>0$. We first show that $f(a) \\neq 1$. Suppose $f(a)=1$. Consider $y=a$ in (7). We get $f(2 a)^{2}=1$. Taking $x=y=a$ in (1), we have $1=2+\\max \\left\\{2 f\\left(a^{2}\\right), f\\left(2 a^{2}\\right)\\right\\}$. From (5), $f\\left(a^{2}\\right)=3$ so that $1 \\geqslant 2+6$. This is impossible, and thus $f(a) \\neq 1$. As $f(a) \\neq \\pm 1$, we have $f(a)= \\pm\\left(2 f\\left(\\frac{a}{2}\\right)+1\\right)$. Similarly, $f(-a)= \\pm\\left(2 f\\left(-\\frac{a}{2}\\right)+1\\right)$. These two expressions are equal since $f(a)=f(-a)$. If $f\\left(\\frac{a}{2}\\right)=f\\left(-\\frac{a}{2}\\right)$, then the above argument works when we replace $a$ by $\\frac{a}{2}$. In particular, we have $f(a)^{2}=f\\left(2 \\cdot \\frac{a}{2}\\right)^{2}=1$, which is a contradiction. Therefore, (6) forces $f\\left(\\frac{a}{2}\\right)+f\\left(-\\frac{a}{2}\\right)=-2$. Then we get $$ \\pm\\left(2 f\\left(\\frac{a}{2}\\right)+1\\right)= \\pm\\left(-2 f\\left(\\frac{a}{2}\\right)-3\\right) . $$ For any choices of the two signs, we either get a contradiction or $f\\left(\\frac{a}{2}\\right)=-1$, in which case $f\\left(\\frac{a}{2}\\right)=f\\left(-\\frac{a}{2}\\right)$ and hence $f(a)= \\pm 1$ again. Therefore, there is no such real number $a$ and the Claim follows. Replace $x$ and $y$ by $-x$ and $-y$ in (1) respectively and compare with (1). We get $$ f(x+y)^{2}-2 f(x) f(y)=f(-x-y)^{2}-2 f(-x) f(-y) $$ Using the Claim, this simplifies to $f(x+y)=f(x)+f(y)+1$. In addition, (5) can be rewritten as $(f(x)+1)^{2}=f\\left(x^{2}\\right)+1$. Therefore, the function $g$ defined by $g(x)=f(x)+1$ satisfies $g(x+y)=g(x)+g(y)$ and $g(x)^{2}=g\\left(x^{2}\\right)$. The latter relation shows $g(y)$ is nonnegative for $y \\geqslant 0$. For such a function satisfying the Cauchy Equation $g(x+y)=g(x)+g(y)$, it must be monotonic increasing and hence $g(x)=c x$ for some constant $c$. From $(c x)^{2}=g(x)^{2}=g\\left(x^{2}\\right)=c x^{2}$, we get $c=0$ or 1 , which corresponds to the two functions $f(x)=-1$ and $f(x)=x-1$ respectively, both of which are solutions to (1) as checked in Solution 1.","tier":0} +{"problem_type":"Algebra","problem_label":"A7","problem":"Denote by $\\mathbb{R}$ the set of all real numbers. Find all functions $f: \\mathbb{R} \\rightarrow \\mathbb{R}$ such that $f(0) \\neq 0$ and $$ f(x+y)^{2}=2 f(x) f(y)+\\max \\left\\{f\\left(x^{2}\\right)+f\\left(y^{2}\\right), f\\left(x^{2}+y^{2}\\right)\\right\\} $$ for all real numbers $x$ and $y$.","solution":"As in $$ (f(x)+1)^{2}=f\\left(x^{2}\\right)+1 $$ and $$ f(x)=f(-x) \\quad \\text { or } \\quad f(x)+f(-x)=-2 $$ for any $x \\in \\mathbb{R}$. We shall show that one of the statements in (9) holds for all $x \\in \\mathbb{R}$. Suppose $f(a)=f(-a)$ but $f(a)+f(-a) \\neq-2$, while $f(b) \\neq f(-b)$ but $f(b)+f(-b)=-2$. Clearly, $a, b \\neq 0$ and $f(a), f(b) \\neq-1$. Taking $y=a$ and $y=-a$ in (1) respectively and comparing the two equations obtained, we have $f(x+a)^{2}=f(x-a)^{2}$, that is, $f(x+a)= \\pm f(x-a)$. This implies $f(x+2 a)= \\pm f(x)$ for all $x \\in \\mathbb{R}$. Putting $x=b$ and $x=-2 a-b$ respectively, we find $f(2 a+b)= \\pm f(b)$ and $f(-2 a-b)= \\pm f(-b)= \\pm(-2-f(b))$. Since $f(b) \\neq-1$, the term $\\pm(-2-f(b))$ is distinct from $\\pm f(b)$ in any case. So $f(2 a+b) \\neq f(-2 a-b)$. From (9), we must have $f(2 a+b)+f(-2 a-b)=-2$. Note that we also have $f(b)+f(-b)=-2$ where $|f(b)|,|f(-b)|$ are equal to $|f(2 a+b)|,|f(-2 a-b)|$ respectively. The only possible case is $f(2 a+b)=f(b)$ and $f(-2 a-b)=f(-b)$. Applying the argument to $-a$ instead of $a$ and using induction, we have $f(2 k a+b)=f(b)$ and $f(2 k a-b)=f(-b)$ for any integer $k$. Note that $f(b)+f(-b)=-2$ and $f(b) \\neq-1$ imply one of $f(b), f(-b)$ is less than -1 . Without loss of generality, assume $f(b)<-1$. We consider $x=\\sqrt{2 k a+b}$ in (8) for sufficiently large $k$ so that $$ (f(x)+1)^{2}=f(2 k a+b)+1=f(b)+1<0 $$ yields a contradiction. Therefore, one of the statements in (9) must hold for all $x \\in \\mathbb{R}$. - Case 1. $f(x)=f(-x)$ for any $x \\in \\mathbb{R}$. For any $a \\in \\mathbb{R}$, setting $x=y=\\frac{a}{2}$ and $x=-y=\\frac{a}{2}$ in (1) respectively and comparing these, we obtain $f(a)^{2}=f(0)^{2}=1$, which means $f(a)= \\pm 1$ for all $a \\in \\mathbb{R}$. If $f(a)=1$ for some $a$, we may assume $a>0$ since $f(a)=f(-a)$. Taking $x=y=\\sqrt{a}$ in (1), we get $$ f(2 \\sqrt{a})^{2}=2 f(\\sqrt{a})^{2}+\\max \\{2, f(2 a)\\}=2 f(\\sqrt{a})^{2}+2 $$ Note that the left-hand side is $\\pm 1$ while the right-hand side is an even integer. This is a contradiction. Therefore, $f(x)=-1$ for all $x \\in \\mathbb{R}$, which is clearly a solution. - Case 2. $f(x)+f(-x)=-2$ for any $x \\in \\mathbb{R}$. This case can be handled in the same way as in Solution 2, which yields another solution $f(x)=x-1$.","tier":0} +{"problem_type":"Algebra","problem_label":"A8","problem":"Determine the largest real number $a$ such that for all $n \\geqslant 1$ and for all real numbers $x_{0}, x_{1}, \\ldots, x_{n}$ satisfying $0=x_{0}4 \\sum_{k=1}^{n} \\frac{k+1}{x_{k}} . $$ This shows (1) holds for $a=\\frac{4}{9}$. Next, we show that $a=\\frac{4}{9}$ is the optimal choice. Consider the sequence defined by $x_{0}=0$ and $x_{k}=x_{k-1}+k(k+1)$ for $k \\geqslant 1$, that is, $x_{k}=\\frac{1}{3} k(k+1)(k+2)$. Then the left-hand side of (1) equals $$ \\sum_{k=1}^{n} \\frac{1}{k(k+1)}=\\sum_{k=1}^{n}\\left(\\frac{1}{k}-\\frac{1}{k+1}\\right)=1-\\frac{1}{n+1} $$ while the right-hand side equals $$ a \\sum_{k=1}^{n} \\frac{k+1}{x_{k}}=3 a \\sum_{k=1}^{n} \\frac{1}{k(k+2)}=\\frac{3}{2} a \\sum_{k=1}^{n}\\left(\\frac{1}{k}-\\frac{1}{k+2}\\right)=\\frac{3}{2}\\left(1+\\frac{1}{2}-\\frac{1}{n+1}-\\frac{1}{n+2}\\right) a . $$ When $n$ tends to infinity, the left-hand side tends to 1 while the right-hand side tends to $\\frac{9}{4} a$. Therefore $a$ has to be at most $\\frac{4}{9}$. Hence the largest value of $a$ is $\\frac{4}{9}$.","tier":0} +{"problem_type":"Algebra","problem_label":"A8","problem":"Determine the largest real number $a$ such that for all $n \\geqslant 1$ and for all real numbers $x_{0}, x_{1}, \\ldots, x_{n}$ satisfying $0=x_{0}0$ for $1 \\leqslant k \\leqslant n$. By the Cauchy-Schwarz Inequality, for $1 \\leqslant k \\leqslant n$, we have $$ \\left(y_{1}+y_{2}+\\cdots+y_{k}\\right)\\left(\\sum_{j=1}^{k} \\frac{1}{y_{j}}\\binom{j+1}{2}^{2}\\right) \\geqslant\\left(\\binom{2}{2}+\\binom{3}{2}+\\cdots+\\binom{k+1}{2}\\right)^{2}=\\binom{k+2}{3}^{2} $$ This can be rewritten as $$ \\frac{k+1}{y_{1}+y_{2}+\\cdots+y_{k}} \\leqslant \\frac{36}{k^{2}(k+1)(k+2)^{2}}\\left(\\sum_{j=1}^{k} \\frac{1}{y_{j}}\\binom{j+1}{2}^{2}\\right) . $$ Summing (3) over $k=1,2, \\ldots, n$, we get $$ \\frac{2}{y_{1}}+\\frac{3}{y_{1}+y_{2}}+\\cdots+\\frac{n+1}{y_{1}+y_{2}+\\cdots+y_{n}} \\leqslant \\frac{c_{1}}{y_{1}}+\\frac{c_{2}}{y_{2}}+\\cdots+\\frac{c_{n}}{y_{n}} $$ where for $1 \\leqslant m \\leqslant n$, $$ \\begin{aligned} c_{m} & =36\\binom{m+1}{2}^{2} \\sum_{k=m}^{n} \\frac{1}{k^{2}(k+1)(k+2)^{2}} \\\\ & =\\frac{9 m^{2}(m+1)^{2}}{4} \\sum_{k=m}^{n}\\left(\\frac{1}{k^{2}(k+1)^{2}}-\\frac{1}{(k+1)^{2}(k+2)^{2}}\\right) \\\\ & =\\frac{9 m^{2}(m+1)^{2}}{4}\\left(\\frac{1}{m^{2}(m+1)^{2}}-\\frac{1}{(n+1)^{2}(n+2)^{2}}\\right)<\\frac{9}{4} . \\end{aligned} $$ From (4), the inequality (1) holds for $a=\\frac{4}{9}$. This is also the upper bound as can be verified in the same way as Solution 1.","tier":0} +{"problem_type":"Combinatorics","problem_label":"C1","problem":"The leader of an IMO team chooses positive integers $n$ and $k$ with $n>k$, and announces them to the deputy leader and a contestant. The leader then secretly tells the deputy leader an $n$-digit binary string, and the deputy leader writes down all $n$-digit binary strings which differ from the leader's in exactly $k$ positions. (For example, if $n=3$ and $k=1$, and if the leader chooses 101, the deputy leader would write down 001, 111 and 100.) The contestant is allowed to look at the strings written by the deputy leader and guess the leader's string. What is the minimum number of guesses (in terms of $n$ and $k$ ) needed to guarantee the correct answer?","solution":"Let $X$ be the binary string chosen by the leader and let $X^{\\prime}$ be the binary string of length $n$ every digit of which is different from that of $X$. The strings written by the deputy leader are the same as those in the case when the leader's string is $X^{\\prime}$ and $k$ is changed to $n-k$. In view of this, we may assume $k \\geqslant \\frac{n}{2}$. Also, for the particular case $k=\\frac{n}{2}$, this argument shows that the strings $X$ and $X^{\\prime}$ cannot be distinguished, and hence in that case the contestant has to guess at least twice. It remains to show that the number of guesses claimed suffices. Consider any string $Y$ which differs from $X$ in $m$ digits where $0k$, and announces them to the deputy leader and a contestant. The leader then secretly tells the deputy leader an $n$-digit binary string, and the deputy leader writes down all $n$-digit binary strings which differ from the leader's in exactly $k$ positions. (For example, if $n=3$ and $k=1$, and if the leader chooses 101, the deputy leader would write down 001, 111 and 100.) The contestant is allowed to look at the strings written by the deputy leader and guess the leader's string. What is the minimum number of guesses (in terms of $n$ and $k$ ) needed to guarantee the correct answer?","solution":"Firstly, assume $n \\neq 2 k$. Without loss of generality suppose the first digit of the leader's string is 1 . Then among the $\\binom{n}{k}$ strings written by the deputy leader, $\\binom{n-1}{k}$ will begin with 1 and $\\binom{n-1}{k-1}$ will begin with 0 . Since $n \\neq 2 k$, we have $k+(k-1) \\neq n-1$ and so $\\binom{n-1}{k} \\neq\\binom{ n-1}{k-1}$. Thus, by counting the number of strings written by the deputy leader that start with 0 and 1 , the contestant can tell the first digit of the leader's string. The same can be done on the other digits, so 1 guess suffices when $n \\neq 2 k$. Secondly, for the case $n=2$ and $k=1$, the answer is clearly 2 . For the remaining cases where $n=2 k>2$, the deputy leader would write down the same strings if the leader's string $X$ is replaced by $X^{\\prime}$ obtained by changing each digit of $X$. This shows at least 2 guesses are needed. We shall show that 2 guesses suffice in this case. Suppose the first two digits of the leader's string are the same. Then among the strings written by the deputy leader, the prefices 01 and 10 will occur $\\binom{2 k-2}{k-1}$ times each, while the prefices 00 and 11 will occur $\\binom{2 k-2}{k}$ times each. The two numbers are interchanged if the first two digits of the leader's string are different. Since $\\binom{2 k-2}{k-1} \\neq\\binom{ 2 k-2}{k}$, the contestant can tell whether the first two digits of the leader's string are the same or not. He can work out the relation of the first digit and the other digits in the same way and reduce the leader's string to only 2 possibilities. The proof is complete.","tier":0} +{"problem_type":"Combinatorics","problem_label":"C2","problem":"Find all positive integers $n$ for which all positive divisors of $n$ can be put into the cells of a rectangular table under the following constraints: - each cell contains a distinct divisor; - the sums of all rows are equal; and - the sums of all columns are equal.","solution":"Suppose all positive divisors of $n$ can be arranged into a rectangular table of size $k \\times l$ where the number of rows $k$ does not exceed the number of columns $l$. Let the sum of numbers in each column be $s$. Since $n$ belongs to one of the columns, we have $s \\geqslant n$, where equality holds only when $n=1$. For $j=1,2, \\ldots, l$, let $d_{j}$ be the largest number in the $j$-th column. Without loss of generality, assume $d_{1}>d_{2}>\\cdots>d_{l}$. Since these are divisors of $n$, we have $$ d_{l} \\leqslant \\frac{n}{l} $$ As $d_{l}$ is the maximum entry of the $l$-th column, we must have $$ d_{l} \\geqslant \\frac{s}{k} \\geqslant \\frac{n}{k} . $$ The relations (1) and (2) combine to give $\\frac{n}{l} \\geqslant \\frac{n}{k}$, that is, $k \\geqslant l$. Together with $k \\leqslant l$, we conclude that $k=l$. Then all inequalities in (1) and (2) are equalities. In particular, $s=n$ and so $n=1$, in which case the conditions are clearly satisfied.","tier":0} +{"problem_type":"Combinatorics","problem_label":"C2","problem":"Find all positive integers $n$ for which all positive divisors of $n$ can be put into the cells of a rectangular table under the following constraints: - each cell contains a distinct divisor; - the sums of all rows are equal; and - the sums of all columns are equal.","solution":"Clearly $n=1$ works. Then we assume $n>1$ and let its prime factorization be $n=p_{1}^{r_{1}} p_{2}^{r_{2}} \\cdots p_{t}^{r_{t}}$. Suppose the table has $k$ rows and $l$ columns with $1n$. Therefore, we have $$ \\begin{aligned} \\left(r_{1}+1\\right)\\left(r_{2}+1\\right) \\cdots\\left(r_{t}+1\\right) & =k l \\leqslant l^{2}<\\left(\\frac{\\sigma(n)}{n}\\right)^{2} \\\\ & =\\left(1+\\frac{1}{p_{1}}+\\cdots+\\frac{1}{p_{1}^{r_{1}}}\\right)^{2} \\cdots\\left(1+\\frac{1}{p_{t}}+\\cdots+\\frac{1}{p_{t}^{r_{t}}}\\right)^{2} \\end{aligned} $$ This can be rewritten as $$ f\\left(p_{1}, r_{1}\\right) f\\left(p_{2}, r_{2}\\right) \\cdots f\\left(p_{t}, r_{t}\\right)<1 $$ where $$ f(p, r)=\\frac{r+1}{\\left(1+\\frac{1}{p}+\\cdots+\\frac{1}{p^{r}}\\right)^{2}}=\\frac{(r+1)\\left(1-\\frac{1}{p}\\right)^{2}}{\\left(1-\\frac{1}{p^{r+1}}\\right)^{2}} $$ Direct computation yields $$ f(2,1)=\\frac{8}{9}, \\quad f(2,2)=\\frac{48}{49}, \\quad f(3,1)=\\frac{9}{8} . $$ Also, we find that $$ \\begin{aligned} & f(2, r) \\geqslant\\left(1-\\frac{1}{2^{r+1}}\\right)^{-2}>1 \\quad \\text { for } r \\geqslant 3 \\\\ & f(3, r) \\geqslant \\frac{4}{3}\\left(1-\\frac{1}{3^{r+1}}\\right)^{-2}>\\frac{4}{3}>\\frac{9}{8} \\quad \\text { for } r \\geqslant 2, \\text { and } \\\\ & f(p, r) \\geqslant \\frac{32}{25}\\left(1-\\frac{1}{p^{r+1}}\\right)^{-2}>\\frac{32}{25}>\\frac{9}{8} \\quad \\text { for } p \\geqslant 5 \\end{aligned} $$ From these values and bounds, it is clear that (3) holds only when $n=2$ or 4 . In both cases, it is easy to see that the conditions are not satisfied. Hence, the only possible $n$ is 1 .","tier":0} +{"problem_type":"Combinatorics","problem_label":"C3","problem":"Let $n$ be a positive integer relatively prime to 6 . We paint the vertices of a regular $n$-gon with three colours so that there is an odd number of vertices of each colour. Show that there exists an isosceles triangle whose three vertices are of different colours. $\\mathbf{C 4}$. Find all positive integers $n$ for which we can fill in the entries of an $n \\times n$ table with the following properties: - each entry can be one of $I, M$ and $O$; - in each row and each column, the letters $I, M$ and $O$ occur the same number of times; and - in any diagonal whose number of entries is a multiple of three, the letters $I, M$ and $O$ occur the same number of times.","solution":"For $k=1,2,3$, let $a_{k}$ be the number of isosceles triangles whose vertices contain exactly $k$ colours. Suppose on the contrary that $a_{3}=0$. Let $b, c, d$ be the number of vertices of the three different colours respectively. We now count the number of pairs $(\\triangle, E)$ where $\\triangle$ is an isosceles triangle and $E$ is a side of $\\triangle$ whose endpoints are of different colours. On the one hand, since we have assumed $a_{3}=0$, each triangle in the pair must contain exactly two colours, and hence each triangle contributes twice. Thus the number of pairs is $2 a_{2}$ \u3002 On the other hand, if we pick any two vertices $A, B$ of distinct colours, then there are three isosceles triangles having these as vertices, two when $A B$ is not the base and one when $A B$ is the base since $n$ is odd. Note that the three triangles are all distinct as $(n, 3)=1$. In this way, we count the number of pairs to be $3(b c+c d+d b)$. However, note that $2 a_{2}$ is even while $3(b c+c d+d b)$ is odd, as each of $b, c, d$ is. This yields a contradiction and hence $a_{3} \\geqslant 1$. Comment. A slightly stronger version of this problem is to replace the condition $(n, 6)=1$ by $n$ being odd (where equilateral triangles are regarded as isosceles triangles). In that case, the only difference in the proof is that by fixing any two vertices $A, B$, one can find exactly one or three isosceles triangles having these as vertices. But since only parity is concerned in the solution, the proof goes the same way. The condition that there is an odd number of vertices of each colour is necessary, as can be seen from the following example. Consider $n=25$ and we label the vertices $A_{0}, A_{1}, \\ldots, A_{24}$. Suppose colour 1 is used for $A_{0}$, colour 2 is used for $A_{5}, A_{10}, A_{15}, A_{20}$, while colour 3 is used for the remaining vertices. Then any isosceles triangle having colours 1 and 2 must contain $A_{0}$ and one of $A_{5}, A_{10}, A_{15}, A_{20}$. Clearly, the third vertex must have index which is a multiple of 5 so it is not of colour 3 .","tier":0} +{"problem_type":"Combinatorics","problem_label":"C5","problem":"Let $n \\geqslant 3$ be a positive integer. Find the maximum number of diagonals of a regular $n$-gon one can select, so that any two of them do not intersect in the interior or they are perpendicular to each other.","solution":"We consider two cases according to the parity of $n$. - Case 1. $n$ is odd. We first claim that no pair of diagonals is perpendicular. Suppose $A, B, C, D$ are vertices where $A B$ and $C D$ are perpendicular, and let $E$ be the vertex lying on the perpendicular bisector of $A B$. Let $E^{\\prime}$ be the opposite point of $E$ on the circumcircle of the regular polygon. Since $E C=E^{\\prime} D$ and $C, D, E$ are vertices of the regular polygon, $E^{\\prime}$ should also belong to the polygon. This contradicts the fact that a regular polygon with an odd number of vertices does not contain opposite points on the circumcircle. ![](https:\/\/cdn.mathpix.com\/cropped\/2024_11_18_e5e735288fb13d37ab0cg-39.jpg?height=513&width=1250&top_left_y=1061&top_left_x=493) Therefore in the odd case we can only select diagonals which do not intersect. In the maximal case these diagonals should divide the regular $n$-gon into $n-2$ triangles, so we can select at most $n-3$ diagonals. This can be done, for example, by selecting all diagonals emanated from a particular vertex. - Case 2. $n$ is even. If there is no intersection, then the proof in the odd case works. Suppose there are two perpendicular diagonals selected. We consider the set $S$ of all selected diagonals parallel to one of them which intersect with some selected diagonals. Suppose $S$ contains $k$ diagonals and the number of distinct endpoints of the $k$ diagonals is $l$. Firstly, consider the longest diagonal in one of the two directions in $S$. No other diagonal in $S$ can start from either endpoint of that diagonal, since otherwise it has to meet another longer diagonal in $S$. The same holds true for the other direction. Ignoring these two longest diagonals and their four endpoints, the remaining $k-2$ diagonals share $l-4$ endpoints where each endpoint can belong to at most two diagonals. This gives $2(l-4) \\geqslant 2(k-2)$, so that $k \\leqslant l-2$. ![](https:\/\/cdn.mathpix.com\/cropped\/2024_11_18_e5e735288fb13d37ab0cg-40.jpg?height=430&width=1442&top_left_y=289&top_left_x=294) Consider a group of consecutive vertices of the regular $n$-gon so that each of the two outermost vertices is an endpoint of a diagonal in $S$, while the interior points are not. There are $l$ such groups. We label these groups $P_{1}, P_{2}, \\ldots, P_{l}$ in this order. We claim that each selected diagonal outside $S$ must connect vertices of the same group $P_{i}$. Consider any diagonal $d$ joining vertices from distinct groups $P_{i}$ and $P_{j}$. Let $d_{1}$ and $d_{2}$ be two diagonals in $S$ each having one of the outermost points of $P_{i}$ as endpoint. Then $d$ must meet either $d_{1}, d_{2}$ or a diagonal in $S$ which is perpendicular to both $d_{1}$ and $d_{2}$. In any case $d$ should belong to $S$ by definition, which is a contradiction. Within the same group $P_{i}$, there are no perpendicular diagonals since the vertices belong to the same side of a diameter of the circumcircle. Hence there can be at most $\\left|P_{i}\\right|-2$ selected diagonals within $P_{i}$, including the one joining the two outermost points of $P_{i}$ when $\\left|P_{i}\\right|>2$. Therefore, the maximum number of diagonals selected is $$ \\sum_{i=1}^{l}\\left(\\left|P_{i}\\right|-2\\right)+k=\\sum_{i=1}^{l}\\left|P_{i}\\right|-2 l+k=(n+l)-2 l+k=n-l+k \\leqslant n-2 $$ This upper bound can be attained as follows. We take any vertex $A$ and let $A^{\\prime}$ be the vertex for which $A A^{\\prime}$ is a diameter of the circumcircle. If we select all diagonals emanated from $A$ together with the diagonal $d^{\\prime}$ joining the two neighbouring vertices of $A^{\\prime}$, then the only pair of diagonals that meet each other is $A A^{\\prime}$ and $d^{\\prime}$, which are perpendicular to each other. In total we can take $n-2$ diagonals. ![](https:\/\/cdn.mathpix.com\/cropped\/2024_11_18_e5e735288fb13d37ab0cg-40.jpg?height=459&width=421&top_left_y=1798&top_left_x=803)","tier":0} +{"problem_type":"Combinatorics","problem_label":"C5","problem":"Let $n \\geqslant 3$ be a positive integer. Find the maximum number of diagonals of a regular $n$-gon one can select, so that any two of them do not intersect in the interior or they are perpendicular to each other.","solution":"The constructions and the odd case are the same as The base case $n=3$ is trivial since there is no diagonal at all. Suppose the upper bound holds for any cyclic polygon with fewer than $n$ sides. For a cyclic $n$-gon, if there is a selected diagonal which does not intersect any other selected diagonal, then this diagonal divides the $n$-gon into an $m$-gon and an $l$-gon (with $m+l=n+2$ ) so that each selected diagonal belongs to one of them. Without loss of generality, we may assume the $m$-gon lies on the same side of a diameter of $\\Gamma$. Then no two selected diagonals of the $m$-gon can intersect, and hence we can select at most $m-3$ diagonals. Also, we can apply the inductive hypothesis to the $l$-gon. This shows the maximum number of selected diagonals is $(m-3)+(l-2)+1=n-2$. It remains to consider the case when all selected diagonals meet at least one other selected diagonal. Consider a pair of selected perpendicular diagonals $d_{1}, d_{2}$. They divide the circumference of $\\Gamma$ into four arcs, each of which lies on the same side of a diameter of $\\Gamma$. If there are two selected diagonals intersecting each other and neither is parallel to $d_{1}$ or $d_{2}$, then their endpoints must belong to the same arc determined by $d_{1}, d_{2}$, and hence they cannot be perpendicular. This violates the condition, and hence all selected diagonals must have the same direction as one of $d_{1}, d_{2}$. ![](https:\/\/cdn.mathpix.com\/cropped\/2024_11_18_e5e735288fb13d37ab0cg-41.jpg?height=435&width=1234&top_left_y=1036&top_left_x=494) Take the longest selected diagonal in one of the two directions. We argue as in Solution 1 that its endpoints do not belong to any other selected diagonal. The same holds true for the longest diagonal in the other direction. Apart from these four endpoints, each of the remaining $n-4$ vertices can belong to at most two selected diagonals. Thus we can select at most $\\frac{1}{2}(2(n-4)+4)=n-2$ diagonals. Then the proof follows by induction.","tier":0} +{"problem_type":"Combinatorics","problem_label":"C6","problem":"There are $n \\geqslant 3$ islands in a city. Initially, the ferry company offers some routes between some pairs of islands so that it is impossible to divide the islands into two groups such that no two islands in different groups are connected by a ferry route. After each year, the ferry company will close a ferry route between some two islands $X$ and $Y$. At the same time, in order to maintain its service, the company will open new routes according to the following rule: for any island which is connected by a ferry route to exactly one of $X$ and $Y$, a new route between this island and the other of $X$ and $Y$ is added. Suppose at any moment, if we partition all islands into two nonempty groups in any way, then it is known that the ferry company will close a certain route connecting two islands from the two groups after some years. Prove that after some years there will be an island which is connected to all other islands by ferry routes.","solution":"Initially, we pick any pair of islands $A$ and $B$ which are connected by a ferry route and put $A$ in set $\\mathcal{A}$ and $B$ in set $\\mathcal{B}$. From the condition, without loss of generality there must be another island which is connected to $A$. We put such an island $C$ in set $\\mathcal{B}$. We say that two sets of islands form a network if each island in one set is connected to each island in the other set. Next, we shall included all islands to $\\mathcal{A} \\cup \\mathcal{B}$ one by one. Suppose we have two sets $\\mathcal{A}$ and $\\mathcal{B}$ which form a network where $3 \\leqslant|\\mathcal{A} \\cup \\mathcal{B}|\\angle C B A$. This shows $S$ and $O_{1}$ lie on different sides of $X Y$. As $S^{\\prime}$ lies on ray $S O_{1}$, it follows that $S$ and $S^{\\prime}$ cannot lie on the same side of $X Y$. Therefore, $S$ and $S^{\\prime}$ are symmetric with respect to $X Y$. Let $d$ be the diameter of the circumcircle of triangle $X S Y$. As $S S^{\\prime}$ is twice the distance from $S$ to $X Y$ and $S X=S Y$, we have $S S^{\\prime}=2 \\frac{S X^{2}}{d}$. It follows from (1) that $d=2 S O_{1}$. As $S O_{1}$ is the perpendicular bisector of $X Y$, point $O_{1}^{d}$ is the circumcentre of triangle $X S Y$.","tier":0} +{"problem_type":"Geometry","problem_label":"G5","problem":"Let $D$ be the foot of perpendicular from $A$ to the Euler line (the line passing through the circumcentre and the orthocentre) of an acute scalene triangle $A B C$. A circle $\\omega$ with centre $S$ passes through $A$ and $D$, and it intersects sides $A B$ and $A C$ at $X$ and $Y$ respectively. Let $P$ be the foot of altitude from $A$ to $B C$, and let $M$ be the midpoint of $B C$. Prove that the circumcentre of triangle $X S Y$ is equidistant from $P$ and $M$.","solution":"Denote the orthocentre and circumcentre of triangle $A B C$ by $H$ and $O$ respectively. Let $O_{1}$ be the circumcentre of triangle $X S Y$. Consider two other possible positions of $S$. We name them $S^{\\prime}$ and $S^{\\prime \\prime}$ and define the analogous points $X^{\\prime}, Y^{\\prime}, O_{1}^{\\prime}, X^{\\prime \\prime}, Y^{\\prime \\prime} O_{1}^{\\prime \\prime}$ accordingly. Note that $S, S^{\\prime}, S^{\\prime \\prime}$ lie on the perpendicular bisector of $A D$. As $X X^{\\prime}$ and $Y Y^{\\prime}$ meet at $A$ and the circumcircles of triangles $A X Y$ and $A X^{\\prime} Y^{\\prime}$ meet at $D$, there is a spiral similarity with centre $D$ mapping $X Y$ to $X^{\\prime} Y^{\\prime}$. We find that $$ \\measuredangle S X Y=\\frac{\\pi}{2}-\\measuredangle Y A X=\\frac{\\pi}{2}-\\measuredangle Y^{\\prime} A X^{\\prime}=\\measuredangle S^{\\prime} X^{\\prime} Y^{\\prime} $$ and similarly $\\measuredangle S Y X=\\measuredangle S^{\\prime} Y^{\\prime} X^{\\prime}$. This shows triangles $S X Y$ and $S^{\\prime} X^{\\prime} Y^{\\prime}$ are directly similar. Then the spiral similarity with centre $D$ takes points $S, X, Y, O_{1}$ to $S^{\\prime}, X^{\\prime}, Y^{\\prime}, O_{1}^{\\prime}$. Similarly, there is a spiral similarity with centre $D$ mapping $S, X, Y, O_{1}$ to $S^{\\prime \\prime}, X^{\\prime \\prime}, Y^{\\prime \\prime}, O_{1}^{\\prime \\prime}$. From these, we see that there is a spiral similarity taking the corresponding points $S, S^{\\prime}, S^{\\prime \\prime}$ to points $O_{1}, O_{1}^{\\prime}, O_{1}^{\\prime \\prime}$. In particular, $O_{1}, O_{1}^{\\prime}, O_{1}^{\\prime \\prime}$ are collinear. ![](https:\/\/cdn.mathpix.com\/cropped\/2024_11_18_e5e735288fb13d37ab0cg-63.jpg?height=750&width=1071&top_left_y=251&top_left_x=592) It now suffices to show that $O_{1}$ lies on the perpendicular bisector of $P M$ for two special cases. Firstly, we take $S$ to be the midpoint of $A H$. Then $X$ and $Y$ are the feet of altitudes from $C$ and $B$ respectively. It is well-known that the circumcircle of triangle $X S Y$ is the nine-point circle of triangle $A B C$. Then $O_{1}$ is the nine-point centre and $O_{1} P=O_{1} M$. Indeed, $P$ and $M$ also lies on the nine-point circle. Secondly, we take $S^{\\prime}$ to be the midpoint of $A O$. Then $X^{\\prime}$ and $Y^{\\prime}$ are the midpoints of $A B$ and $A C$ respectively. Then $X^{\\prime} Y^{\\prime} \/ \/ B C$. Clearly, $S^{\\prime}$ lies on the perpendicular bisector of $P M$. This shows the perpendicular bisectors of $X^{\\prime} Y^{\\prime}$ and $P M$ coincide. Hence, we must have $O_{1}^{\\prime} P=O_{1}^{\\prime} M$. ![](https:\/\/cdn.mathpix.com\/cropped\/2024_11_18_e5e735288fb13d37ab0cg-63.jpg?height=548&width=1257&top_left_y=1610&top_left_x=481)","tier":0} +{"problem_type":"Geometry","problem_label":"G6","problem":"Let $A B C D$ be a convex quadrilateral with $\\angle A B C=\\angle A D C<90^{\\circ}$. The internal angle bisectors of $\\angle A B C$ and $\\angle A D C$ meet $A C$ at $E$ and $F$ respectively, and meet each other at point $P$. Let $M$ be the midpoint of $A C$ and let $\\omega$ be the circumcircle of triangle $B P D$. Segments $B M$ and $D M$ intersect $\\omega$ again at $X$ and $Y$ respectively. Denote by $Q$ the intersection point of lines $X E$ and $Y F$. Prove that $P Q \\perp A C$.","solution":"![](https:\/\/cdn.mathpix.com\/cropped\/2024_11_18_e5e735288fb13d37ab0cg-64.jpg?height=857&width=1492&top_left_y=604&top_left_x=273) Let $\\omega_{1}$ be the circumcircle of triangle $A B C$. We first prove that $Y$ lies on $\\omega_{1}$. Let $Y^{\\prime}$ be the point on ray $M D$ such that $M Y^{\\prime} \\cdot M D=M A^{2}$. Then triangles $M A Y^{\\prime}$ and $M D A$ are oppositely similar. Since $M C^{2}=M A^{2}=M Y^{\\prime} \\cdot M D$, triangles $M C Y^{\\prime}$ and $M D C$ are also oppositely similar. Therefore, using directed angles, we have $$ \\measuredangle A Y^{\\prime} C=\\measuredangle A Y^{\\prime} M+\\measuredangle M Y^{\\prime} C=\\measuredangle M A D+\\measuredangle D C M=\\measuredangle C D A=\\measuredangle A B C $$ so that $Y^{\\prime}$ lies on $\\omega_{1}$. Let $Z$ be the intersection point of lines $B C$ and $A D$. Since $\\measuredangle P D Z=\\measuredangle P B C=\\measuredangle P B Z$, point $Z$ lies on $\\omega$. In addition, from $\\measuredangle Y^{\\prime} B Z=\\measuredangle Y^{\\prime} B C=\\measuredangle Y^{\\prime} A C=\\measuredangle Y^{\\prime} A M=\\measuredangle Y^{\\prime} D Z$, we also know that $Y^{\\prime}$ lies on $\\omega$. Note that $\\angle A D C$ is acute implies $M A \\neq M D$ so $M Y^{\\prime} \\neq M D$. Therefore, $Y^{\\prime}$ is the second intersection of $D M$ and $\\omega$. Then $Y^{\\prime}=Y$ and hence $Y$ lies on $\\omega_{1}$. Next, by the Angle Bisector Theorem and the similar triangles, we have $$ \\frac{F A}{F C}=\\frac{A D}{C D}=\\frac{A D}{A M} \\cdot \\frac{C M}{C D}=\\frac{Y A}{Y M} \\cdot \\frac{Y M}{Y C}=\\frac{Y A}{Y C} $$ Hence, $F Y$ is the internal angle bisector of $\\angle A Y C$. Let $B^{\\prime}$ be the second intersection of the internal angle bisector of $\\angle C B A$ and $\\omega_{1}$. Then $B^{\\prime}$ is the midpoint of arc $A C$ not containing $B$. Therefore, $Y B^{\\prime}$ is the external angle bisector of $\\angle A Y C$, so that $B^{\\prime} Y \\perp F Y$. Denote by $l$ the line through $P$ parallel to $A C$. Suppose $l$ meets line $B^{\\prime} Y$ at $S$. From $$ \\begin{aligned} \\measuredangle P S Y & =\\measuredangle\\left(A C, B^{\\prime} Y\\right)=\\measuredangle A C Y+\\measuredangle C Y B^{\\prime}=\\measuredangle A C Y+\\measuredangle C A B^{\\prime}=\\measuredangle A C Y+\\measuredangle B^{\\prime} C A \\\\ & =\\measuredangle B^{\\prime} C Y=\\measuredangle B^{\\prime} B Y=\\measuredangle P B Y \\end{aligned} $$ the point $S$ lies on $\\omega$. Similarly, the line through $X$ perpendicular to $X E$ also passes through the second intersection of $l$ and $\\omega$, which is the point $S$. From $Q Y \\perp Y S$ and $Q X \\perp X S$, point $Q$ lies on $\\omega$ and $Q S$ is a diameter of $\\omega$. Therefore, $P Q \\perp P S$ so that $P Q \\perp A C$.","tier":0} +{"problem_type":"Geometry","problem_label":"G6","problem":"Let $A B C D$ be a convex quadrilateral with $\\angle A B C=\\angle A D C<90^{\\circ}$. The internal angle bisectors of $\\angle A B C$ and $\\angle A D C$ meet $A C$ at $E$ and $F$ respectively, and meet each other at point $P$. Let $M$ be the midpoint of $A C$ and let $\\omega$ be the circumcircle of triangle $B P D$. Segments $B M$ and $D M$ intersect $\\omega$ again at $X$ and $Y$ respectively. Denote by $Q$ the intersection point of lines $X E$ and $Y F$. Prove that $P Q \\perp A C$.","solution":"Denote by $\\omega_{1}$ and $\\omega_{2}$ the circumcircles of triangles $A B C$ and $A D C$ respectively. Since $\\angle A B C=\\angle A D C$, we know that $\\omega_{1}$ and $\\omega_{2}$ are symmetric with respect to the midpoint $M$ of $A C$. Firstly, we show that $X$ lies on $\\omega_{2}$. Let $X_{1}$ be the second intersection of ray $M B$ and $\\omega_{2}$ and $X^{\\prime}$ be its symmetric point with respect to $M$. Then $X^{\\prime}$ lies on $\\omega_{1}$ and $X^{\\prime} A X_{1} C$ is a parallelogram. Hence, we have $$ \\begin{aligned} \\measuredangle D X_{1} B & =\\measuredangle D X_{1} A+\\measuredangle A X_{1} B=\\measuredangle D C A+\\measuredangle A X_{1} X^{\\prime}=\\measuredangle D C A+\\measuredangle C X^{\\prime} X_{1} \\\\ & =\\measuredangle D C A+\\measuredangle C A B=\\measuredangle(C D, A B) . \\end{aligned} $$ ![](https:\/\/cdn.mathpix.com\/cropped\/2024_11_18_e5e735288fb13d37ab0cg-65.jpg?height=914&width=1302&top_left_y=1191&top_left_x=449) Also, we have $$ \\measuredangle D P B=\\measuredangle P D C+\\angle(C D, A B)+\\measuredangle A B P=\\angle(C D, A B) . $$ These yield $\\measuredangle D X_{1} B=\\measuredangle D P B$ and hence $X_{1}$ lies on $\\omega$. It follows that $X_{1}=X$ and $X$ lies on $\\omega_{2}$. Similarly, $Y$ lies on $\\omega_{1}$. Next, we prove that $Q$ lies on $\\omega$. Suppose the perpendicular bisector of $A C$ meet $\\omega_{1}$ at $B^{\\prime}$ and $M_{1}$ and meet $\\omega_{2}$ at $D^{\\prime}$ and $M_{2}$, so that $B, M_{1}$ and $D^{\\prime}$ lie on the same side of $A C$. Note that $B^{\\prime}$ lies on the angle bisector of $\\angle A B C$ and similarly $D^{\\prime}$ lies on $D P$. If we denote the area of $W_{1} W_{2} W_{3}$ by $\\left[W_{1} W_{2} W_{3}\\right]$, then $$ \\frac{B A \\cdot X^{\\prime} A}{B C \\cdot X^{\\prime} C}=\\frac{\\frac{1}{2} B A \\cdot X^{\\prime} A \\sin \\angle B A X^{\\prime}}{\\frac{1}{2} B C \\cdot X^{\\prime} C \\sin \\angle B C X^{\\prime}}=\\frac{\\left[B A X^{\\prime}\\right]}{\\left[B C X^{\\prime}\\right]}=\\frac{M A}{M C}=1 $$ As $B E$ is the angle bisector of $\\angle A B C$, we have $$ \\frac{E A}{E C}=\\frac{B A}{B C}=\\frac{X^{\\prime} C}{X^{\\prime} A}=\\frac{X A}{X C} $$ Therefore, $X E$ is the angle bisector of $\\angle A X C$, so that $M_{2}$ lies on the line joining $X, E, Q$. Analogously, $M_{1}, F, Q, Y$ are collinear. Thus, $$ \\begin{aligned} \\measuredangle X Q Y & =\\measuredangle M_{2} Q M_{1}=\\measuredangle Q M_{2} M_{1}+\\measuredangle M_{2} M_{1} Q=\\measuredangle X M_{2} D^{\\prime}+\\measuredangle B^{\\prime} M_{1} Y \\\\ & =\\measuredangle X D D^{\\prime}+\\measuredangle B^{\\prime} B Y=\\measuredangle X D P+\\measuredangle P B Y=\\measuredangle X B P+\\measuredangle P B Y=\\measuredangle X B Y, \\end{aligned} $$ which implies $Q$ lies on $\\omega$. Finally, as $M_{1}$ and $M_{2}$ are symmetric with respect to $M$, the quadrilateral $X^{\\prime} M_{2} X M_{1}$ is a parallelogram. Consequently, $$ \\measuredangle X Q P=\\measuredangle X B P=\\measuredangle X^{\\prime} B B^{\\prime}=\\measuredangle X^{\\prime} M_{1} B^{\\prime}=\\measuredangle X M_{2} M_{1} . $$ This shows $Q P \/ \/ M_{2} M_{1}$. As $M_{2} M_{1} \\perp A C$, we get $Q P \\perp A C$.","tier":0} +{"problem_type":"Geometry","problem_label":"G6","problem":"Let $A B C D$ be a convex quadrilateral with $\\angle A B C=\\angle A D C<90^{\\circ}$. The internal angle bisectors of $\\angle A B C$ and $\\angle A D C$ meet $A C$ at $E$ and $F$ respectively, and meet each other at point $P$. Let $M$ be the midpoint of $A C$ and let $\\omega$ be the circumcircle of triangle $B P D$. Segments $B M$ and $D M$ intersect $\\omega$ again at $X$ and $Y$ respectively. Denote by $Q$ the intersection point of lines $X E$ and $Y F$. Prove that $P Q \\perp A C$.","solution":"We first state two results which will be needed in our proof. - Claim 1. In $\\triangle X^{\\prime} Y^{\\prime} Z^{\\prime}$ with $X^{\\prime} Y^{\\prime} \\neq X^{\\prime} Z^{\\prime}$, let $N^{\\prime}$ be the midpoint of $Y^{\\prime} Z^{\\prime}$ and $W^{\\prime}$ be the foot of internal angle bisector from $X^{\\prime}$. Then $\\tan ^{2} \\measuredangle W^{\\prime} X^{\\prime} Z^{\\prime}=\\tan \\measuredangle N^{\\prime} X^{\\prime} W^{\\prime} \\tan \\measuredangle Z^{\\prime} W^{\\prime} X^{\\prime}$. Proof. ![](https:\/\/cdn.mathpix.com\/cropped\/2024_11_18_e5e735288fb13d37ab0cg-66.jpg?height=416&width=562&top_left_y=1684&top_left_x=733) Without loss of generality, assume $X^{\\prime} Y^{\\prime}>X^{\\prime} Z^{\\prime}$. Then $W^{\\prime}$ lies between $N^{\\prime}$ and $Z^{\\prime}$. The signs of both sides agree so it suffices to establish the relation for ordinary angles. Let $\\angle W^{\\prime} X^{\\prime} Z^{\\prime}=\\alpha, \\angle N^{\\prime} X^{\\prime} W^{\\prime}=\\beta$ and $\\angle Z^{\\prime} W^{\\prime} X^{\\prime}=\\gamma$. We have $$ \\frac{\\sin (\\gamma-\\alpha)}{\\sin (\\alpha-\\beta)}=\\frac{N^{\\prime} X^{\\prime}}{N^{\\prime} Y^{\\prime}}=\\frac{N^{\\prime} X^{\\prime}}{N^{\\prime} Z^{\\prime}}=\\frac{\\sin (\\gamma+\\alpha)}{\\sin (\\alpha+\\beta)} $$ This implies $$ \\frac{\\tan \\gamma-\\tan \\alpha}{\\tan \\gamma+\\tan \\alpha}=\\frac{\\sin \\gamma \\cos \\alpha-\\cos \\gamma \\sin \\alpha}{\\sin \\gamma \\cos \\alpha+\\cos \\gamma \\sin \\alpha}=\\frac{\\sin \\alpha \\cos \\beta-\\cos \\alpha \\sin \\beta}{\\sin \\alpha \\cos \\beta+\\cos \\alpha \\sin \\beta}=\\frac{\\tan \\alpha-\\tan \\beta}{\\tan \\alpha+\\tan \\beta} $$ Expanding and simplifying, we get the desired result $\\tan ^{2} \\alpha=\\tan \\beta \\tan \\gamma$. - Claim 2. Let $A^{\\prime} B^{\\prime} C^{\\prime} D^{\\prime}$ be a quadrilateral inscribed in circle $\\Gamma$. Let diagonals $A^{\\prime} C^{\\prime}$ and $B^{\\prime} D^{\\prime}$ meet at $E^{\\prime}$, and $F^{\\prime}$ be the intersection of lines $A^{\\prime} B^{\\prime}$ and $C^{\\prime} D^{\\prime}$. Let $M^{\\prime}$ be the midpoint of $E^{\\prime} F^{\\prime}$. Then the power of $M^{\\prime}$ with respect to $\\Gamma$ is equal to $\\left(M^{\\prime} E^{\\prime}\\right)^{2}$. Proof. ![](https:\/\/cdn.mathpix.com\/cropped\/2024_11_18_e5e735288fb13d37ab0cg-67.jpg?height=624&width=904&top_left_y=1000&top_left_x=678) Let $O^{\\prime}$ be the centre of $\\Gamma$ and let $\\Gamma^{\\prime}$ be the circle with centre $M^{\\prime}$ passing through $E^{\\prime}$. Let $F_{1}$ be the inversion image of $F^{\\prime}$ with respect to $\\Gamma$. It is well-known that $E^{\\prime}$ lies on the polar of $F^{\\prime}$ with respect to $\\Gamma$. This shows $E^{\\prime} F_{1} \\perp O^{\\prime} F^{\\prime}$ and hence $F_{1}$ lies on $\\Gamma^{\\prime}$. It follows that the inversion image of $\\Gamma^{\\prime}$ with respect to $\\Gamma$ is $\\Gamma^{\\prime}$ itself. This shows $\\Gamma^{\\prime}$ is orthogonal to $\\Gamma$, and thus the power of $M^{\\prime}$ with respect to $\\Gamma$ is the square of radius of $\\Gamma^{\\prime}$, which is $\\left(M^{\\prime} E^{\\prime}\\right)^{2}$. We return to the main problem. Let $Z$ be the intersection of lines $A D$ and $B C$, and $W$ be the intersection of lines $A B$ and $C D$. Since $\\measuredangle P D Z=\\measuredangle P B C=\\measuredangle P B Z$, point $Z$ lies on $\\omega$. Similarly, $W$ lies on $\\omega$. Applying Claim 2 to the cyclic quadrilateral $Z B D W$, we know that the power of $M$ with respect to $\\omega$ is $M A^{2}$. Hence, $M X \\cdot M B=M A^{2}$. Suppose the line through $B$ perpendicular to $B E$ meets line $A C$ at $T$. Then $B E$ and $B T$ are the angle bisectors of $\\angle C B A$. This shows $(T, E ; A, C)$ is harmonic. Thus, we have $M E \\cdot M T=M A^{2}=M X \\cdot M B$. It follows that $E, T, B, X$ are concyclic. ![](https:\/\/cdn.mathpix.com\/cropped\/2024_11_18_e5e735288fb13d37ab0cg-68.jpg?height=808&width=1424&top_left_y=252&top_left_x=285) The result is trivial for the special case $A D=C D$ since $P, Q$ lie on the perpendicular bisector of $A C$ in that case. Similarly, the case $A B=C B$ is trivial. It remains to consider the general cases where we can apply Claim 1 in the latter part of the proof. Let the projections from $P$ and $Q$ to $A C$ be $P^{\\prime}$ and $Q^{\\prime}$ respectively. Then $P Q \\perp A C$ if and only if $P^{\\prime}=Q^{\\prime}$ if and only if $\\frac{E P^{\\prime}}{F P^{\\prime}}=\\frac{E Q^{\\prime}}{F Q^{\\prime}}$ in terms of directed lengths. Note that $$ \\frac{E P^{\\prime}}{F P^{\\prime}}=\\frac{\\tan \\measuredangle E F P}{\\tan \\measuredangle F E P}=\\frac{\\tan \\measuredangle A F D}{\\tan \\measuredangle A E B} $$ Next, we have $\\frac{E Q^{\\prime}}{F Q^{\\prime}}=\\frac{\\tan \\measuredangle E F Q}{\\tan \\measuredangle F E Q}$ where $\\measuredangle F E Q=\\measuredangle T E X=\\measuredangle T B X=\\frac{\\pi}{2}+\\measuredangle E B M$ and by symmetry $\\measuredangle E F Q=\\frac{\\pi}{2}+\\measuredangle F D M$. Combining all these, it suffices to show $$ \\frac{\\tan \\measuredangle A F D}{\\tan \\measuredangle A E B}=\\frac{\\tan \\measuredangle M B E}{\\tan \\measuredangle M D F} $$ We now apply Claim 1 twice to get $$ \\tan \\measuredangle A F D \\tan \\measuredangle M D F=\\tan ^{2} \\measuredangle F D C=\\tan ^{2} \\measuredangle E B A=\\tan \\measuredangle M B E \\tan \\measuredangle A E B . $$ The result then follows.","tier":0} +{"problem_type":"Geometry","problem_label":"G7","problem":"Let $I$ be the incentre of a non-equilateral triangle $A B C, I_{A}$ be the $A$-excentre, $I_{A}^{\\prime}$ be the reflection of $I_{A}$ in $B C$, and $l_{A}$ be the reflection of line $A I_{A}^{\\prime}$ in $A I$. Define points $I_{B}, I_{B}^{\\prime}$ and line $l_{B}$ analogously. Let $P$ be the intersection point of $l_{A}$ and $l_{B}$. (a) Prove that $P$ lies on line $O I$ where $O$ is the circumcentre of triangle $A B C$. (b) Let one of the tangents from $P$ to the incircle of triangle $A B C$ meet the circumcircle at points $X$ and $Y$. Show that $\\angle X I Y=120^{\\circ}$.","solution":"(a) Let $A^{\\prime}$ be the reflection of $A$ in $B C$ and let $M$ be the second intersection of line $A I$ and the circumcircle $\\Gamma$ of triangle $A B C$. As triangles $A B A^{\\prime}$ and $A O C$ are isosceles with $\\angle A B A^{\\prime}=2 \\angle A B C=\\angle A O C$, they are similar to each other. Also, triangles $A B I_{A}$ and $A I C$ are similar. Therefore we have $$ \\frac{A A^{\\prime}}{A I_{A}}=\\frac{A A^{\\prime}}{A B} \\cdot \\frac{A B}{A I_{A}}=\\frac{A C}{A O} \\cdot \\frac{A I}{A C}=\\frac{A I}{A O} $$ Together with $\\angle A^{\\prime} A I_{A}=\\angle I A O$, we find that triangles $A A^{\\prime} I_{A}$ and $A I O$ are similar. ![](https:\/\/cdn.mathpix.com\/cropped\/2024_11_18_e5e735288fb13d37ab0cg-69.jpg?height=890&width=1025&top_left_y=1192&top_left_x=653) Denote by $P^{\\prime}$ the intersection of line $A P$ and line $O I$. Using directed angles, we have $$ \\begin{aligned} \\measuredangle M A P^{\\prime} & =\\measuredangle I_{A}^{\\prime} A I_{A}=\\measuredangle I_{A}^{\\prime} A A^{\\prime}-\\measuredangle I_{A} A A^{\\prime}=\\measuredangle A A^{\\prime} I_{A}-\\measuredangle(A M, O M) \\\\ & =\\measuredangle A I O-\\measuredangle A M O=\\measuredangle M O P^{\\prime} . \\end{aligned} $$ This shows $M, O, A, P^{\\prime}$ are concyclic. Denote by $R$ and $r$ the circumradius and inradius of triangle $A B C$. Then $$ I P^{\\prime}=\\frac{I A \\cdot I M}{I O}=\\frac{I O^{2}-R^{2}}{I O} $$ is independent of $A$. Hence, $B P$ also meets line $O I$ at the same point $P^{\\prime}$ so that $P^{\\prime}=P$, and $P$ lies on $O I$. (b) By Poncelet's Porism, the other tangents to the incircle of triangle $A B C$ from $X$ and $Y$ meet at a point $Z$ on $\\Gamma$. Let $T$ be the touching point of the incircle to $X Y$, and let $D$ be the midpoint of $X Y$. We have $$ \\begin{aligned} O D & =I T \\cdot \\frac{O P}{I P}=r\\left(1+\\frac{O I}{I P}\\right)=r\\left(1+\\frac{O I^{2}}{O I \\cdot I P}\\right)=r\\left(1+\\frac{R^{2}-2 R r}{R^{2}-I O^{2}}\\right) \\\\ & =r\\left(1+\\frac{R^{2}-2 R r}{2 R r}\\right)=\\frac{R}{2}=\\frac{O X}{2} \\end{aligned} $$ This shows $\\angle X Z Y=60^{\\circ}$ and hence $\\angle X I Y=120^{\\circ}$.","tier":0} +{"problem_type":"Geometry","problem_label":"G7","problem":"Let $I$ be the incentre of a non-equilateral triangle $A B C, I_{A}$ be the $A$-excentre, $I_{A}^{\\prime}$ be the reflection of $I_{A}$ in $B C$, and $l_{A}$ be the reflection of line $A I_{A}^{\\prime}$ in $A I$. Define points $I_{B}, I_{B}^{\\prime}$ and line $l_{B}$ analogously. Let $P$ be the intersection point of $l_{A}$ and $l_{B}$. (a) Prove that $P$ lies on line $O I$ where $O$ is the circumcentre of triangle $A B C$. (b) Let one of the tangents from $P$ to the incircle of triangle $A B C$ meet the circumcircle at points $X$ and $Y$. Show that $\\angle X I Y=120^{\\circ}$.","solution":"(a) Note that triangles $A I_{B} C$ and $I_{A} B C$ are similar since their corresponding interior angles are equal. Therefore, the four triangles $A I_{B}^{\\prime} C, A I_{B} C, I_{A} B C$ and $I_{A}^{\\prime} B C$ are all similar. From $\\triangle A I_{B}^{\\prime} C \\sim \\triangle I_{A}^{\\prime} B C$, we get $\\triangle A I_{A}^{\\prime} C \\sim \\triangle I_{B}^{\\prime} B C$. From $\\measuredangle A B P=\\measuredangle I_{B}^{\\prime} B C=\\measuredangle A I_{A}^{\\prime} C$ and $\\measuredangle B A P=\\measuredangle I_{A}^{\\prime} A C$, the triangles $A B P$ and $A I_{A}^{\\prime} C$ are directly similar. ![](https:\/\/cdn.mathpix.com\/cropped\/2024_11_18_e5e735288fb13d37ab0cg-70.jpg?height=820&width=1137&top_left_y=1455&top_left_x=494) Consider the inversion with centre $A$ and radius $\\sqrt{A B \\cdot A C}$ followed by the reflection in $A I$. Then $B$ and $C$ are mapped to each other, and $I$ and $I_{A}$ are mapped to each other. From the similar triangles obtained, we have $A P \\cdot A I_{A}^{\\prime}=A B \\cdot A C$ so that $P$ is mapped to $I_{A}^{\\prime}$ under the transformation. In addition, line $A O$ is mapped to the altitude from $A$, and hence $O$ is mapped to the reflection of $A$ in $B C$, which we call point $A^{\\prime}$. Note that $A A^{\\prime} I_{A} I_{A}^{\\prime}$ is an isosceles trapezoid, which shows it is inscribed in a circle. The preimage of this circle is a straight line, meaning that $O, I, P$ are collinear. (b) Denote by $R$ and $r$ the circumradius and inradius of triangle $A B C$. Note that by the above transformation, we have $\\triangle A P O \\sim \\triangle A A^{\\prime} I_{A}^{\\prime}$ and $\\triangle A A^{\\prime} I_{A} \\sim \\triangle A I O$. Therefore, we find that $$ P O=A^{\\prime} I_{A}^{\\prime} \\cdot \\frac{A O}{A I_{A}^{\\prime}}=A I_{A} \\cdot \\frac{A O}{A^{\\prime} I_{A}}=\\frac{A I_{A}}{A^{\\prime} I_{A}} \\cdot A O=\\frac{A O}{I O} \\cdot A O $$ This shows $P O \\cdot I O=R^{2}$, and it follows that $P$ and $I$ are mapped to each other under the inversion with respect to the circumcircle $\\Gamma$ of triangle $A B C$. Then $P X \\cdot P Y$, which is the power of $P$ with respect to $\\Gamma$, equals $P I \\cdot P O$. This yields $X, I, O, Y$ are concyclic. Let $T$ be the touching point of the incircle to $X Y$, and let $D$ be the midpoint of $X Y$. Then $$ O D=I T \\cdot \\frac{P O}{P I}=r \\cdot \\frac{P O}{P O-I O}=r \\cdot \\frac{R^{2}}{R^{2}-I O^{2}}=r \\cdot \\frac{R^{2}}{2 R r}=\\frac{R}{2} $$ This shows $\\angle D O X=60^{\\circ}$ and hence $\\angle X I Y=\\angle X O Y=120^{\\circ}$. Comment. A simplification of this problem is to ask part (a) only. Note that the question in part (b) implicitly requires $P$ to lie on $O I$, or otherwise the angle is not uniquely determined as we can find another tangent from $P$ to the incircle.","tier":0} +{"problem_type":"Geometry","problem_label":"G8","problem":"Let $A_{1}, B_{1}$ and $C_{1}$ be points on sides $B C, C A$ and $A B$ of an acute triangle $A B C$ respectively, such that $A A_{1}, B B_{1}$ and $C C_{1}$ are the internal angle bisectors of triangle $A B C$. Let $I$ be the incentre of triangle $A B C$, and $H$ be the orthocentre of triangle $A_{1} B_{1} C_{1}$. Show that $$ A H+B H+C H \\geqslant A I+B I+C I $$","solution":"Without loss of generality, assume $\\alpha=\\angle B A C \\leqslant \\beta=\\angle C B A \\leqslant \\gamma=\\angle A C B$. Denote by $a, b, c$ the lengths of $B C, C A, A B$ respectively. We first show that triangle $A_{1} B_{1} C_{1}$ is acute. Choose points $D$ and $E$ on side $B C$ such that $B_{1} D \/ \/ A B$ and $B_{1} E$ is the internal angle bisector of $\\angle B B_{1} C$. As $\\angle B_{1} D B=180^{\\circ}-\\beta$ is obtuse, we have $B B_{1}>B_{1} D$. Thus, $$ \\frac{B E}{E C}=\\frac{B B_{1}}{B_{1} C}>\\frac{D B_{1}}{B_{1} C}=\\frac{B A}{A C}=\\frac{B A_{1}}{A_{1} C} $$ Therefore, $B E>B A_{1}$ and $\\frac{1}{2} \\angle B B_{1} C=\\angle B B_{1} E>\\angle B B_{1} A_{1}$. Similarly, $\\frac{1}{2} \\angle B B_{1} A>\\angle B B_{1} C_{1}$. It follows that $$ \\angle A_{1} B_{1} C_{1}=\\angle B B_{1} A_{1}+\\angle B B_{1} C_{1}<\\frac{1}{2}\\left(\\angle B B_{1} C+\\angle B B_{1} A\\right)=90^{\\circ} $$ is acute. By symmetry, triangle $A_{1} B_{1} C_{1}$ is acute. Let $B B_{1}$ meet $A_{1} C_{1}$ at $F$. From $\\alpha \\leqslant \\gamma$, we get $a \\leqslant c$, which implies $$ B A_{1}=\\frac{c a}{b+c} \\leqslant \\frac{a c}{a+b}=B C_{1} $$ and hence $\\angle B C_{1} A_{1} \\leqslant \\angle B A_{1} C_{1}$. As $B F$ is the internal angle bisector of $\\angle A_{1} B C_{1}$, this shows $\\angle B_{1} F C_{1}=\\angle B F A_{1} \\leqslant 90^{\\circ}$. Hence, $H$ lies on the same side of $B B_{1}$ as $C_{1}$. This shows $H$ lies inside triangle $B B_{1} C_{1}$. Similarly, from $\\alpha \\leqslant \\beta$ and $\\beta \\leqslant \\gamma$, we know that $H$ lies inside triangles $C C_{1} B_{1}$ and $A A_{1} C_{1}$. ![](https:\/\/cdn.mathpix.com\/cropped\/2024_11_18_e5e735288fb13d37ab0cg-72.jpg?height=779&width=1010&top_left_y=1660&top_left_x=506) As $\\alpha \\leqslant \\beta \\leqslant \\gamma$, we have $\\alpha \\leqslant 60^{\\circ} \\leqslant \\gamma$. Then $\\angle B I C \\leqslant 120^{\\circ} \\leqslant \\angle A I B$. Firstly, suppose $\\angle A I C \\geqslant 120^{\\circ}$. Rotate points $B, I, H$ through $60^{\\circ}$ about $A$ to $B^{\\prime}, I^{\\prime}, H^{\\prime}$ so that $B^{\\prime}$ and $C$ lie on different sides of $A B$. Since triangle $A I^{\\prime} I$ is equilateral, we have $$ A I+B I+C I=I^{\\prime} I+B^{\\prime} I^{\\prime}+I C=B^{\\prime} I^{\\prime}+I^{\\prime} I+I C . $$ Similarly, $$ A H+B H+C H=H^{\\prime} H+B^{\\prime} H^{\\prime}+H C=B^{\\prime} H^{\\prime}+H^{\\prime} H+H C $$ As $\\angle A I I^{\\prime}=\\angle A I^{\\prime} I=60^{\\circ}, \\angle A I^{\\prime} B^{\\prime}=\\angle A I B \\geqslant 120^{\\circ}$ and $\\angle A I C \\geqslant 120^{\\circ}$, the quadrilateral $B^{\\prime} I^{\\prime} I C$ is convex and lies on the same side of $B^{\\prime} C$ as $A$. Next, since $H$ lies inside triangle $A C C_{1}, H$ lies outside $B^{\\prime} I^{\\prime} I C$. Also, $H$ lying inside triangle $A B I$ implies $H^{\\prime}$ lies inside triangle $A B^{\\prime} I^{\\prime}$. This shows $H^{\\prime}$ lies outside $B^{\\prime} I^{\\prime} I C$ and hence the convex quadrilateral $B^{\\prime} I^{\\prime} I C$ is contained inside the quadrilateral $B^{\\prime} H^{\\prime} H C$. It follows that the perimeter of $B^{\\prime} I^{\\prime} I C$ cannot exceed the perimeter of $B^{\\prime} H^{\\prime} H C$. From (1) and (2), we conclude that $$ A H+B H+C H \\geqslant A I+B I+C I $$ For the case $\\angle A I C<120^{\\circ}$, we can rotate $B, I, H$ through $60^{\\circ}$ about $C$ to $B^{\\prime}, I^{\\prime}, H^{\\prime}$ so that $B^{\\prime}$ and $A$ lie on different sides of $B C$. The proof is analogous to the previous case and we still get the desired inequality.","tier":0} +{"problem_type":"Number Theory","problem_label":"N1","problem":"For any positive integer $k$, denote the sum of digits of $k$ in its decimal representation by $S(k)$. Find all polynomials $P(x)$ with integer coefficients such that for any positive integer $n \\geqslant 2016$, the integer $P(n)$ is positive and $$ S(P(n))=P(S(n)) $$","solution":"We consider three cases according to the degree of $P$. - Case 1. $P(x)$ is a constant polynomial. Let $P(x)=c$ where $c$ is an integer constant. Then (1) becomes $S(c)=c$. This holds if and only if $1 \\leqslant c \\leqslant 9$. - Case 2. $\\operatorname{deg} P=1$. We have the following observation. For any positive integers $m, n$, we have $$ S(m+n) \\leqslant S(m)+S(n) $$ and equality holds if and only if there is no carry in the addition $m+n$. Let $P(x)=a x+b$ for some integers $a, b$ where $a \\neq 0$. As $P(n)$ is positive for large $n$, we must have $a \\geqslant 1$. The condition (1) becomes $S(a n+b)=a S(n)+b$ for all $n \\geqslant 2016$. Setting $n=2025$ and $n=2020$ respectively, we get $$ S(2025 a+b)-S(2020 a+b)=(a S(2025)+b)-(a S(2020)+b)=5 a $$ On the other hand, (2) implies $$ S(2025 a+b)=S((2020 a+b)+5 a) \\leqslant S(2020 a+b)+S(5 a) $$ These give $5 a \\leqslant S(5 a)$. As $a \\geqslant 1$, this holds only when $a=1$, in which case (1) reduces to $S(n+b)=S(n)+b$ for all $n \\geqslant 2016$. Then we find that $$ S(n+1+b)-S(n+b)=(S(n+1)+b)-(S(n)+b)=S(n+1)-S(n) $$ If $b>0$, we choose $n$ such that $n+1+b=10^{k}$ for some sufficiently large $k$. Note that all the digits of $n+b$ are 9 's, so that the left-hand side of (3) equals $1-9 k$. As $n$ is a positive integer less than $10^{k}-1$, we have $S(n)<9 k$. Therefore, the right-hand side of (3) is at least $1-(9 k-1)=2-9 k$, which is a contradiction. The case $b<0$ can be handled similarly by considering $n+1$ to be a large power of 10 . Therefore, we conclude that $P(x)=x$, in which case (1) is trivially satisfied. - Case 3. $\\operatorname{deg} P \\geqslant 2$. Suppose the leading term of $P$ is $a_{d} n^{d}$ where $a_{d} \\neq 0$. Clearly, we have $a_{d}>0$. Consider $n=10^{k}-1$ in (1). We get $S(P(n))=P(9 k)$. Note that $P(n)$ grows asymptotically as fast as $n^{d}$, so $S(P(n))$ grows asymptotically as no faster than a constant multiple of $k$. On the other hand, $P(9 k)$ grows asymptotically as fast as $k^{d}$. This shows the two sides of the last equation cannot be equal for sufficiently large $k$ since $d \\geqslant 2$. Therefore, we conclude that $P(x)=c$ where $1 \\leqslant c \\leqslant 9$ is an integer, or $P(x)=x$.","tier":0} +{"problem_type":"Number Theory","problem_label":"N1","problem":"For any positive integer $k$, denote the sum of digits of $k$ in its decimal representation by $S(k)$. Find all polynomials $P(x)$ with integer coefficients such that for any positive integer $n \\geqslant 2016$, the integer $P(n)$ is positive and $$ S(P(n))=P(S(n)) $$","solution":"Let $P(x)=a_{d} x^{d}+a_{d-1} x^{d-1}+\\cdots+a_{0}$. Clearly $a_{d}>0$. There exists an integer $m \\geqslant 1$ such that $\\left|a_{i}\\right|<10^{m}$ for all $0 \\leqslant i \\leqslant d$. Consider $n=9 \\times 10^{k}$ for a sufficiently large integer $k$ in (1). If there exists an index $0 \\leqslant i \\leqslant d-1$ such that $a_{i}<0$, then all digits of $P(n)$ in positions from $10^{i k+m+1}$ to $10^{(i+1) k-1}$ are all 9 's. Hence, we have $S(P(n)) \\geqslant 9(k-m-1)$. On the other hand, $P(S(n))=P(9)$ is a fixed constant. Therefore, (1) cannot hold for large $k$. This shows $a_{i} \\geqslant 0$ for all $0 \\leqslant i \\leqslant d-1$. Hence, $P(n)$ is an integer formed by the nonnegative integers $a_{d} \\times 9^{d}, a_{d-1} \\times 9^{d-1}, \\ldots, a_{0}$ by inserting some zeros in between. This yields $$ S(P(n))=S\\left(a_{d} \\times 9^{d}\\right)+S\\left(a_{d-1} \\times 9^{d-1}\\right)+\\cdots+S\\left(a_{0}\\right) . $$ Combining with (1), we have $$ S\\left(a_{d} \\times 9^{d}\\right)+S\\left(a_{d-1} \\times 9^{d-1}\\right)+\\cdots+S\\left(a_{0}\\right)=P(9)=a_{d} \\times 9^{d}+a_{d-1} \\times 9^{d-1}+\\cdots+a_{0} $$ As $S(m) \\leqslant m$ for any positive integer $m$, with equality when $1 \\leqslant m \\leqslant 9$, this forces each $a_{i} \\times 9^{i}$ to be a positive integer between 1 and 9 . In particular, this shows $a_{i}=0$ for $i \\geqslant 2$ and hence $d \\leqslant 1$. Also, we have $a_{1} \\leqslant 1$ and $a_{0} \\leqslant 9$. If $a_{1}=1$ and $1 \\leqslant a_{0} \\leqslant 9$, we take $n=10^{k}+\\left(10-a_{0}\\right)$ for sufficiently large $k$ in (1). This yields a contradiction since $$ S(P(n))=S\\left(10^{k}+10\\right)=2 \\neq 11=P\\left(11-a_{0}\\right)=P(S(n)) $$ The zero polynomial is also rejected since $P(n)$ is positive for large $n$. The remaining candidates are $P(x)=x$ or $P(x)=a_{0}$ where $1 \\leqslant a_{0} \\leqslant 9$, all of which satisfy (1), and hence are the only solutions.","tier":0} +{"problem_type":"Number Theory","problem_label":"N2","problem":"Let $\\tau(n)$ be the number of positive divisors of $n$. Let $\\tau_{1}(n)$ be the number of positive divisors of $n$ which have remainders 1 when divided by 3 . Find all possible integral values of the fraction $\\frac{\\tau(10 n)}{\\tau_{1}(10 n)}$.","solution":"In this solution, we always use $p_{i}$ to denote primes congruent to $1 \\bmod 3$, and use $q_{j}$ to denote primes congruent to $2 \\bmod 3$. When we express a positive integer $m$ using its prime factorization, we also include the special case $m=1$ by allowing the exponents to be zeros. We first compute $\\tau_{1}(m)$ for a positive integer $m$. - Claim. Let $m=3^{x} p_{1}^{a_{1}} p_{2}^{a_{2}} \\cdots p_{s}^{a_{s}} q_{1}^{b_{1}} q_{2}^{b_{2}} \\cdots q_{t}^{b_{t}}$ be the prime factorization of $m$. Then $$ \\tau_{1}(m)=\\prod_{i=1}^{s}\\left(a_{i}+1\\right)\\left\\lceil\\frac{1}{2} \\prod_{j=1}^{t}\\left(b_{j}+1\\right)\\right\\rceil . $$ Proof. To choose a divisor of $m$ congruent to $1 \\bmod 3$, it cannot have the prime divisor 3, while there is no restriction on choosing prime factors congruent to $1 \\bmod 3$. Also, we have to choose an even number of prime factors (counted with multiplicity) congruent to $2 \\bmod 3$. If $\\prod_{j=1}^{t}\\left(b_{j}+1\\right)$ is even, then we may assume without loss of generality $b_{1}+1$ is even. We can choose the prime factors $q_{2}, q_{3}, \\ldots, q_{t}$ freely in $\\prod_{j=2}^{t}\\left(b_{j}+1\\right)$ ways. Then the parity of the number of $q_{1}$ is uniquely determined, and hence there are $\\frac{1}{2}\\left(b_{1}+1\\right)$ ways to choose the exponent of $q_{1}$. Hence (1) is verified in this case. If $\\prod_{j=1}^{t}\\left(b_{j}+1\\right)$ is odd, we use induction on $t$ to count the number of choices. When $t=1$, there are $\\left\\lceil\\frac{b_{1}+1}{2}\\right\\rceil$ choices for which the exponent is even and $\\left\\lfloor\\frac{b_{1}+1}{2}\\right\\rfloor$ choices for which the exponent is odd. For the inductive step, we find that there are $$ \\left\\lceil\\frac{1}{2} \\prod_{j=1}^{t-1}\\left(b_{j}+1\\right)\\right\\rceil \\cdot\\left\\lceil\\frac{b_{t}+1}{2}\\right\\rceil+\\left\\lfloor\\frac{1}{2} \\prod_{j=1}^{t-1}\\left(b_{j}+1\\right)\\right\\rfloor \\cdot\\left\\lfloor\\frac{b_{t}+1}{2}\\right\\rfloor=\\left\\lceil\\frac{1}{2} \\prod_{j=1}^{t}\\left(b_{j}+1\\right)\\right\\rceil $$ choices with an even number of prime factors and hence $\\left\\lfloor\\frac{1}{2} \\prod_{j=1}^{t}\\left(b_{j}+1\\right)\\right\\rfloor$ choices with an odd number of prime factors. Hence (1) is also true in this case. Let $n=3^{x} 2^{y} 5^{z} p_{1}^{a_{1}} p_{2}^{a_{2}} \\cdots p_{s}^{a_{s}} q_{1}^{b_{1}} q_{2}^{b_{2}} \\cdots q_{t}^{b_{t}}$. Using the well-known formula for computing the divisor function, we get $$ \\tau(10 n)=(x+1)(y+2)(z+2) \\prod_{i=1}^{s}\\left(a_{i}+1\\right) \\prod_{j=1}^{t}\\left(b_{j}+1\\right) $$ By the Claim, we have $$ \\tau_{1}(10 n)=\\prod_{i=1}^{s}\\left(a_{i}+1\\right)\\left\\lceil\\frac{1}{2}(y+2)(z+2) \\prod_{j=1}^{t}\\left(b_{j}+1\\right)\\right\\rceil $$ If $c=(y+2)(z+2) \\prod_{j=1}^{t}\\left(b_{j}+1\\right)$ is even, then (2) and (3) imply $$ \\frac{\\tau(10 n)}{\\tau_{1}(10 n)}=2(x+1) $$ In this case $\\frac{\\tau(10 n)}{\\tau_{1}(10 n)}$ can be any even positive integer as $x$ runs through all nonnegative integers. If $c$ is odd, which means $y, z$ are odd and each $b_{j}$ is even, then (2) and (3) imply $$ \\frac{\\tau(10 n)}{\\tau_{1}(10 n)}=\\frac{2(x+1) c}{c+1} $$ For this to be an integer, we need $c+1$ divides $2(x+1)$ since $c$ and $c+1$ are relatively prime. Let $2(x+1)=k(c+1)$. Then (4) reduces to $$ \\frac{\\tau(10 n)}{\\tau_{1}(10 n)}=k c=k(y+2)(z+2) \\prod_{j=1}^{t}\\left(b_{j}+1\\right) $$ Noting that $y, z$ are odd, the integers $y+2$ and $z+2$ are at least 3 . This shows the integer in this case must be composite. On the other hand, for any odd composite number $a b$ with $a, b \\geqslant 3$, we may simply take $n=3^{\\frac{a b-1}{2}} \\cdot 2^{a-2} \\cdot 5^{b-2}$ so that $\\frac{\\tau(10 n)}{\\tau_{1}(10 n)}=a b$ from (5). We conclude that the fraction can be any even integer or any odd composite number. Equivalently, it can be 2 or any composite number.","tier":0} +{"problem_type":"Number Theory","problem_label":"N3","problem":"Define $P(n)=n^{2}+n+1$. For any positive integers $a$ and $b$, the set $$ \\{P(a), P(a+1), P(a+2), \\ldots, P(a+b)\\} $$ is said to be fragrant if none of its elements is relatively prime to the product of the other elements. Determine the smallest size of a fragrant set.","solution":"We have the following observations. (i) $(P(n), P(n+1))=1$ for any $n$. We have $(P(n), P(n+1))=\\left(n^{2}+n+1, n^{2}+3 n+3\\right)=\\left(n^{2}+n+1,2 n+2\\right)$. Noting that $n^{2}+n+1$ is odd and $\\left(n^{2}+n+1, n+1\\right)=(1, n+1)=1$, the claim follows. (ii) $(P(n), P(n+2))=1$ for $n \\not \\equiv 2(\\bmod 7)$ and $(P(n), P(n+2))=7$ for $n \\equiv 2(\\bmod 7)$. From $(2 n+7) P(n)-(2 n-1) P(n+2)=14$ and the fact that $P(n)$ is odd, $(P(n), P(n+2))$ must be a divisor of 7 . The claim follows by checking $n \\equiv 0,1, \\ldots, 6(\\bmod 7)$ directly. (iii) $(P(n), P(n+3))=1$ for $n \\not \\equiv 1(\\bmod 3)$ and $3 \\mid(P(n), P(n+3))$ for $n \\equiv 1(\\bmod 3)$. From $(n+5) P(n)-(n-1) P(n+3)=18$ and the fact that $P(n)$ is odd, $(P(n), P(n+3))$ must be a divisor of 9 . The claim follows by checking $n \\equiv 0,1,2(\\bmod 3)$ directly. Suppose there exists a fragrant set with at most 5 elements. We may assume it contains exactly 5 elements $P(a), P(a+1), \\ldots, P(a+4)$ since the following argument also works with fewer elements. Consider $P(a+2)$. From (i), it is relatively prime to $P(a+1)$ and $P(a+3)$. Without loss of generality, assume $(P(a), P(a+2))>1$. From (ii), we have $a \\equiv 2(\\bmod 7)$. The same observation implies $(P(a+1), P(a+3))=1$. In order that the set is fragrant, $(P(a), P(a+3))$ and $(P(a+1), P(a+4))$ must both be greater than 1. From (iii), this holds only when both $a$ and $a+1$ are congruent to $1 \\bmod 3$, which is a contradiction. It now suffices to construct a fragrant set of size 6. By the Chinese Remainder Theorem, we can take a positive integer $a$ such that $$ a \\equiv 7 \\quad(\\bmod 19), \\quad a+1 \\equiv 2 \\quad(\\bmod 7), \\quad a+2 \\equiv 1 \\quad(\\bmod 3) $$ For example, we may take $a=197$. From (ii), both $P(a+1)$ and $P(a+3)$ are divisible by 7. From (iii), both $P(a+2)$ and $P(a+5)$ are divisible by 3 . One also checks from $19 \\mid P(7)=57$ and $19 \\mid P(11)=133$ that $P(a)$ and $P(a+4)$ are divisible by 19. Therefore, the set $\\{P(a), P(a+1), \\ldots, P(a+5)\\}$ is fragrant. Therefore, the smallest size of a fragrant set is 6 . Comment. \"Fragrant Harbour\" is the English translation of \"Hong Kong\". A stronger version of this problem is to show that there exists a fragrant set of size $k$ for any $k \\geqslant 6$. We present a proof here. For each even positive integer $m$ which is not divisible by 3 , since $m^{2}+3 \\equiv 3(\\bmod 4)$, we can find a prime $p_{m} \\equiv 3(\\bmod 4)$ such that $p_{m} \\mid m^{2}+3$. Clearly, $p_{m}>3$. If $b=2 t \\geqslant 6$, we choose $a$ such that $3 \\mid 2(a+t)+1$ and $p_{m} \\mid 2(a+t)+1$ for each $1 \\leqslant m \\leqslant b$ with $m \\equiv 2,4(\\bmod 6)$. For $0 \\leqslant r \\leqslant t$ and $3 \\mid r$, we have $a+t \\pm r \\equiv 1(\\bmod 3)$ so that $3 \\mid P(a+t \\pm r)$. For $0 \\leqslant r \\leqslant t$ and $(r, 3)=1$, we have $$ 4 P(a+t \\pm r) \\equiv(-1 \\pm 2 r)^{2}+2(-1 \\pm 2 r)+4=4 r^{2}+3 \\equiv 0 \\quad\\left(\\bmod p_{2 r}\\right) . $$ Hence, $\\{P(a), P(a+1), \\ldots, P(a+b)\\}$ is fragrant. If $b=2 t+1 \\geqslant 7$ (the case $b=5$ has been done in the original problem), we choose $a$ such that $3 \\mid 2(a+t)+1$ and $p_{m} \\mid 2(a+t)+1$ for $1 \\leqslant m \\leqslant b$ with $m \\equiv 2,4(\\bmod 6)$, and that $a+b \\equiv 9(\\bmod 13)$. Note that $a$ exists by the Chinese Remainder Theorem since $p_{m} \\neq 13$ for all $m$. The even case shows that $\\{P(a), P(a+1), \\ldots, P(a+b-1)\\}$ is fragrant. Also, one checks from $13 \\mid P(9)=91$ and $13 \\mid P(3)=13$ that $P(a+b)$ and $P(a+b-6)$ are divisible by 13. The proof is thus complete.","tier":0} +{"problem_type":"Number Theory","problem_label":"N4","problem":"Let $n, m, k$ and $l$ be positive integers with $n \\neq 1$ such that $n^{k}+m n^{l}+1$ divides $n^{k+l}-1$. Prove that - $m=1$ and $l=2 k$; or - $l \\mid k$ and $m=\\frac{n^{k-l}-1}{n^{l}-1}$.","solution":"It is given that $$ n^{k}+m n^{l}+1 \\mid n^{k+l}-1 $$ This implies $$ n^{k}+m n^{l}+1 \\mid\\left(n^{k+l}-1\\right)+\\left(n^{k}+m n^{l}+1\\right)=n^{k+l}+n^{k}+m n^{l} . $$ We have two cases to discuss. - Case 1. $l \\geqslant k$. Since $\\left(n^{k}+m n^{l}+1, n\\right)=1$,(2) yields $$ n^{k}+m n^{l}+1 \\mid n^{l}+m n^{l-k}+1 . $$ In particular, we get $n^{k}+m n^{l}+1 \\leqslant n^{l}+m n^{l-k}+1$. As $n \\geqslant 2$ and $k \\geqslant 1,(m-1) n^{l}$ is at least $2(m-1) n^{l-k}$. It follows that the inequality cannot hold when $m \\geqslant 2$. For $m=1$, the above divisibility becomes $$ n^{k}+n^{l}+1 \\mid n^{l}+n^{l-k}+1 . $$ Note that $n^{l}+n^{l-k}+12 m n^{l}+1>n^{l}+m n^{l-k}+1$, it follows that $n^{k}+m n^{l}+1=n^{l}+m n^{l-k}+1$, that is, $$ m\\left(n^{l}-n^{l-k}\\right)=n^{l}-n^{k} . $$ If $m \\geqslant 2$, then $m\\left(n^{l}-n^{l-k}\\right) \\geqslant 2 n^{l}-2 n^{l-k} \\geqslant 2 n^{l}-n^{l}>n^{l}-n^{k}$ gives a contradiction. Hence $m=1$ and $l-k=k$, which means $m=1$ and $l=2 k$. - Case 2. $l2 n^{k}+m>n^{k}+n^{k-l}+m$, it follows that $n^{k}+m n^{l}+1=n^{k}+n^{k-l}+m$. This gives $m=\\frac{n^{k-l}-1}{n^{l}-1}$. Note that $n^{l}-1 \\mid n^{k-l}-1$ implies $l \\mid k-l$ and hence $l \\mid k$. The proof is thus complete. Comment. Another version of this problem is as follows: let $n, m, k$ and $l$ be positive integers with $n \\neq 1$ such that $k$ and $l$ do not divide each other. Show that $n^{k}+m n^{l}+1$ does not divide $n^{k+l}-1$.","tier":0} +{"problem_type":"Number Theory","problem_label":"N5","problem":"Let $a$ be a positive integer which is not a square number. Denote by $A$ the set of all positive integers $k$ such that $$ k=\\frac{x^{2}-a}{x^{2}-y^{2}} $$ for some integers $x$ and $y$ with $x>\\sqrt{a}$. Denote by $B$ the set of all positive integers $k$ such that (1) is satisfied for some integers $x$ and $y$ with $0 \\leqslant x<\\sqrt{a}$. Prove that $A=B$.","solution":"We first prove the following preliminary result. - Claim. For fixed $k$, let $x, y$ be integers satisfying (1). Then the numbers $x_{1}, y_{1}$ defined by $$ x_{1}=\\frac{1}{2}\\left(x-y+\\frac{(x-y)^{2}-4 a}{x+y}\\right), \\quad y_{1}=\\frac{1}{2}\\left(x-y-\\frac{(x-y)^{2}-4 a}{x+y}\\right) $$ are integers and satisfy (1) (with $x, y$ replaced by $x_{1}, y_{1}$ respectively). Proof. Since $x_{1}+y_{1}=x-y$ and $$ x_{1}=\\frac{x^{2}-x y-2 a}{x+y}=-x+\\frac{2\\left(x^{2}-a\\right)}{x+y}=-x+2 k(x-y), $$ both $x_{1}$ and $y_{1}$ are integers. Let $u=x+y$ and $v=x-y$. The relation (1) can be rewritten as $$ u^{2}-(4 k-2) u v+\\left(v^{2}-4 a\\right)=0 $$ By Vieta's Theorem, the number $z=\\frac{v^{2}-4 a}{u}$ satisfies $$ v^{2}-(4 k-2) v z+\\left(z^{2}-4 a\\right)=0 $$ Since $x_{1}$ and $y_{1}$ are defined so that $v=x_{1}+y_{1}$ and $z=x_{1}-y_{1}$, we can reverse the process and verify (1) for $x_{1}, y_{1}$. We first show that $B \\subset A$. Take any $k \\in B$ so that (1) is satisfied for some integers $x, y$ with $0 \\leqslant x<\\sqrt{a}$. Clearly, $y \\neq 0$ and we may assume $y$ is positive. Since $a$ is not a square, we have $k>1$. Hence, we get $0 \\leqslant x\\sqrt{a} $$ This implies $k \\in A$ and hence $B \\subset A$. Next, we shall show that $A \\subset B$. Take any $k \\in A$ so that (1) is satisfied for some integers $x, y$ with $x>\\sqrt{a}$. Again, we may assume $y$ is positive. Among all such representations of $k$, we choose the one with smallest $x+y$. Define $$ x_{1}=\\frac{1}{2}\\left|x-y+\\frac{(x-y)^{2}-4 a}{x+y}\\right|, \\quad y_{1}=\\frac{1}{2}\\left(x-y-\\frac{(x-y)^{2}-4 a}{x+y}\\right) . $$ By the Claim, $x_{1}, y_{1}$ are integers satisfying (1). Since $k>1$, we get $x>y>\\sqrt{a}$. Therefore, we have $y_{1}>\\frac{4 a}{x+y}>0$ and $\\frac{4 a}{x+y}\\sqrt{a}$, we get a contradiction due to the minimality of $x+y$. Therefore, we must have $0 \\leqslant x_{1}<\\sqrt{a}$, which means $k \\in B$ so that $A \\subset B$. The two subset relations combine to give $A=B$.","tier":0} +{"problem_type":"Number Theory","problem_label":"N5","problem":"Let $a$ be a positive integer which is not a square number. Denote by $A$ the set of all positive integers $k$ such that $$ k=\\frac{x^{2}-a}{x^{2}-y^{2}} $$ for some integers $x$ and $y$ with $x>\\sqrt{a}$. Denote by $B$ the set of all positive integers $k$ such that (1) is satisfied for some integers $x$ and $y$ with $0 \\leqslant x<\\sqrt{a}$. Prove that $A=B$.","solution":"The relation (1) is equivalent to $$ k y^{2}-(k-1) x^{2}=a $$ Motivated by Pell's Equation, we prove the following, which is essentially the same as the Claim in Solution 1. - Claim. If $\\left(x_{0}, y_{0}\\right)$ is a solution to $(2)$, then $\\left((2 k-1) x_{0} \\pm 2 k y_{0},(2 k-1) y_{0} \\pm 2(k-1) x_{0}\\right)$ is also a solution to (2). Proof. We check directly that $$ \\begin{aligned} & k\\left((2 k-1) y_{0} \\pm 2(k-1) x_{0}\\right)^{2}-(k-1)\\left((2 k-1) x_{0} \\pm 2 k y_{0}\\right)^{2} \\\\ = & \\left(k(2 k-1)^{2}-(k-1)(2 k)^{2}\\right) y_{0}^{2}+\\left(k(2(k-1))^{2}-(k-1)(2 k-1)^{2}\\right) x_{0}^{2} \\\\ = & k y_{0}^{2}-(k-1) x_{0}^{2}=a \\end{aligned} $$ If (2) is satisfied for some $0 \\leqslant x<\\sqrt{a}$ and nonnegative integer $y$, then clearly (1) implies $y>x$. Also, we have $k>1$ since $a$ is not a square number. By the Claim, consider another solution to (2) defined by $$ x_{1}=(2 k-1) x+2 k y, \\quad y_{1}=(2 k-1) y+2(k-1) x $$ It satisfies $x_{1} \\geqslant(2 k-1) x+2 k(x+1)=(4 k-1) x+2 k>x$. Then we can replace the old solution by a new one which has a larger value in $x$. After a finite number of replacements, we must get a solution with $x>\\sqrt{a}$. This shows $B \\subset A$. If (2) is satisfied for some $x>\\sqrt{a}$ and nonnegative integer $y$, by the Claim we consider another solution to (2) defined by $$ x_{1}=|(2 k-1) x-2 k y|, \\quad y_{1}=(2 k-1) y-2(k-1) x . $$ From (2), we get $\\sqrt{k} y>\\sqrt{k-1} x$. This implies $k y>\\sqrt{k(k-1)} x>(k-1) x$ and hence $(2 k-1) x-2 k yy$. Then it is clear that $(2 k-1) x-2 k y>-x$. These combine to give $x_{1}\\sqrt{a}$. Denote by $B$ the set of all positive integers $k$ such that (1) is satisfied for some integers $x$ and $y$ with $0 \\leqslant x<\\sqrt{a}$. Prove that $A=B$.","solution":"It suffices to show $A \\cup B$ is a subset of $A \\cap B$. We take any $k \\in A \\cup B$, which means there exist integers $x, y$ satisfying (1). Since $a$ is not a square, it follows that $k \\neq 1$. As in Without loss of generality, assume $x, y \\geqslant 0$. Let $u=x+y$ and $v=x-y$. Then $u \\geqslant v$ and (1) becomes $$ k=\\frac{(u+v)^{2}-4 a}{4 u v} $$ This is the same as $$ v^{2}+(2 u-4 k u) v+u^{2}-4 a=0 $$ Let $v_{1}=4 k u-2 u-v$. Then $u+v_{1}=4 k u-u-v \\geqslant 8 u-u-v>u+v$. By Vieta's Theorem, $v_{1}$ satisfies $$ v_{1}^{2}+(2 u-4 k u) v_{1}+u^{2}-4 a=0 $$ This gives $k=\\frac{\\left(u+v_{1}\\right)^{2}-4 a}{4 u v_{1}}$. As $k$ is an integer, $u+v_{1}$ must be even. Therefore, $x_{1}=\\frac{u+v_{1}}{2}$ and $y_{1}=\\frac{v_{1}-u}{2}$ are integers. By reversing the process, we can see that $\\left(x_{1}, y_{1}\\right)$ is a solution to (1), with $x_{1}=\\frac{u+v_{1}}{2}>\\frac{u+v}{2}=x \\geqslant 0$. This completes the first half of the proof. Suppose $x>\\sqrt{a}$. Then $u+v>2 \\sqrt{a}$ and (3) can be rewritten as $$ u^{2}+(2 v-4 k v) u+v^{2}-4 a=0 . $$ Let $u_{2}=4 k v-2 v-u$. By Vieta's Theorem, we have $u u_{2}=v^{2}-4 a$ and $$ u_{2}^{2}+(2 v-4 k v) u_{2}+v^{2}-4 a=0 $$ By $u>0, u+v>2 \\sqrt{a}$ and (3), we have $v>0$. If $u_{2} \\geqslant 0$, then $v u_{2} \\leqslant u u_{2}=v^{2}-4 a0$ and $u_{2}+v\\nu_{p}\\left(A_{1} A_{m+1}^{2}\\right)$ for $2 \\leqslant m \\leqslant k-1$. Proof. The case $m=2$ is obvious since $\\nu_{p}\\left(A_{1} A_{2}^{2}\\right) \\geqslant p^{t}>\\nu_{p}\\left(A_{1} A_{3}^{2}\\right)$ by the condition and the above assumption. Suppose $\\nu_{p}\\left(A_{1} A_{2}^{2}\\right)>\\nu_{p}\\left(A_{1} A_{3}^{2}\\right)>\\cdots>\\nu_{p}\\left(A_{1} A_{m}^{2}\\right)$ where $3 \\leqslant m \\leqslant k-1$. For the induction step, we apply Ptolemy's Theorem to the cyclic quadrilateral $A_{1} A_{m-1} A_{m} A_{m+1}$ to get $$ A_{1} A_{m+1} \\times A_{m-1} A_{m}+A_{1} A_{m-1} \\times A_{m} A_{m+1}=A_{1} A_{m} \\times A_{m-1} A_{m+1} $$ which can be rewritten as $$ \\begin{aligned} A_{1} A_{m+1}^{2} \\times A_{m-1} A_{m}^{2}= & A_{1} A_{m-1}^{2} \\times A_{m} A_{m+1}^{2}+A_{1} A_{m}^{2} \\times A_{m-1} A_{m+1}^{2} \\\\ & -2 A_{1} A_{m-1} \\times A_{m} A_{m+1} \\times A_{1} A_{m} \\times A_{m-1} A_{m+1} \\end{aligned} $$ From this, $2 A_{1} A_{m-1} \\times A_{m} A_{m+1} \\times A_{1} A_{m} \\times A_{m-1} A_{m+1}$ is an integer. We consider the component of $p$ of each term in (1). By the inductive hypothesis, we have $\\nu_{p}\\left(A_{1} A_{m-1}^{2}\\right)>\\nu_{p}\\left(A_{1} A_{m}^{2}\\right)$. Also, we have $\\nu_{p}\\left(A_{m} A_{m+1}^{2}\\right) \\geqslant p^{t}>\\nu_{p}\\left(A_{m-1} A_{m+1}^{2}\\right)$. These give $$ \\nu_{p}\\left(A_{1} A_{m-1}^{2} \\times A_{m} A_{m+1}^{2}\\right)>\\nu_{p}\\left(A_{1} A_{m}^{2} \\times A_{m-1} A_{m+1}^{2}\\right) $$ Next, we have $\\nu_{p}\\left(4 A_{1} A_{m-1}^{2} \\times A_{m} A_{m+1}^{2} \\times A_{1} A_{m}^{2} \\times A_{m-1} A_{m+1}^{2}\\right)=\\nu_{p}\\left(A_{1} A_{m-1}^{2} \\times A_{m} A_{m+1}^{2}\\right)+$ $\\nu_{p}\\left(A_{1} A_{m}^{2} \\times A_{m-1} A_{m+1}^{2}\\right)>2 \\nu_{p}\\left(A_{1} A_{m}^{2} \\times A_{m-1} A_{m+1}^{2}\\right)$ from (2). This implies $$ \\nu_{p}\\left(2 A_{1} A_{m-1} \\times A_{m} A_{m+1} \\times A_{1} A_{m} \\times A_{m-1} A_{m+1}\\right)>\\nu_{p}\\left(A_{1} A_{m}^{2} \\times A_{m-1} A_{m+1}^{2}\\right) $$ Combining (1), (2) and (3), we conclude that $$ \\nu_{p}\\left(A_{1} A_{m+1}^{2} \\times A_{m-1} A_{m}^{2}\\right)=\\nu_{p}\\left(A_{1} A_{m}^{2} \\times A_{m-1} A_{m+1}^{2}\\right) $$ By $\\nu_{p}\\left(A_{m-1} A_{m}^{2}\\right) \\geqslant p^{t}>\\nu_{p}\\left(A_{m-1} A_{m+1}^{2}\\right)$, we get $\\nu_{p}\\left(A_{1} A_{m+1}^{2}\\right)<\\nu_{p}\\left(A_{1} A_{m}^{2}\\right)$. The Claim follows by induction. From the Claim, we get a chain of inequalities $$ p^{t}>\\nu_{p}\\left(A_{1} A_{3}^{2}\\right)>\\nu_{p}\\left(A_{1} A_{4}^{2}\\right)>\\cdots>\\nu_{p}\\left(A_{1} A_{k}^{2}\\right) \\geqslant p^{t} $$ which yields a contradiction. Therefore, we can show by induction that $2 S$ is divisible by $n$. Comment. The condition that $P$ is cyclic is crucial. As a counterexample, consider the rhombus with vertices $(0,3),(4,0),(0,-3),(-4,0)$. Each of its squares of side lengths is divisible by 5 , while $2 S=48$ is not. The proposer also gives a proof for the case $n$ is even. One just needs an extra technical step for the case $p=2$.","tier":0} +{"problem_type":"Number Theory","problem_label":"N8","problem":"Find all polynomials $P(x)$ of odd degree $d$ and with integer coefficients satisfying the following property: for each positive integer $n$, there exist $n$ positive integers $x_{1}, x_{2}, \\ldots, x_{n}$ such that $\\frac{1}{2}<\\frac{P\\left(x_{i}\\right)}{P\\left(x_{j}\\right)}<2$ and $\\frac{P\\left(x_{i}\\right)}{P\\left(x_{j}\\right)}$ is the $d$-th power of a rational number for every pair of indices $i$ and $j$ with $1 \\leqslant i, j \\leqslant n$.","solution":"Let $P(x)=a_{d} x^{d}+a_{d-1} x^{d-1}+\\cdots+a_{0}$. Consider the substitution $y=d a_{d} x+a_{d-1}$. By defining $Q(y)=P(x)$, we find that $Q$ is a polynomial with rational coefficients without the term $y^{d-1}$. Let $Q(y)=b_{d} y^{d}+b_{d-2} y^{d-2}+b_{d-3} y^{d-3}+\\cdots+b_{0}$ and $B=\\max _{0 \\leqslant i \\leqslant d}\\left\\{\\left|b_{i}\\right|\\right\\}$ (where $b_{d-1}=0$ ). The condition shows that for each $n \\geqslant 1$, there exist integers $y_{1}, y_{2}, \\ldots, y_{n}$ such that $\\frac{1}{2}<\\frac{Q\\left(y_{i}\\right)}{Q\\left(y_{j}\\right)}<2$ and $\\frac{Q\\left(y_{i}\\right)}{Q\\left(y_{j}\\right)}$ is the $d$-th power of a rational number for $1 \\leqslant i, j \\leqslant n$. Since $n$ can be arbitrarily large, we may assume all $x_{i}$ 's and hence $y_{i}$ 's are integers larger than some absolute constant in the following. By Dirichlet's Theorem, since $d$ is odd, we can find a sufficiently large prime $p$ such that $p \\equiv 2(\\bmod d)$. In particular, we have $(p-1, d)=1$. For this fixed $p$, we choose $n$ to be sufficiently large. Then by the Pigeonhole Principle, there must be $d+1$ of $y_{1}, y_{2}, \\ldots, y_{n}$ which are congruent $\\bmod p$. Without loss of generality, assume $y_{i} \\equiv y_{j}(\\bmod p)$ for $1 \\leqslant i, j \\leqslant d+1$. We shall establish the following. - Claim. $\\frac{Q\\left(y_{i}\\right)}{Q\\left(y_{1}\\right)}=\\frac{y_{i}^{d}}{y_{1}^{d}}$ for $2 \\leqslant i \\leqslant d+1$. Proof. Let $\\frac{Q\\left(y_{i}\\right)}{Q\\left(y_{1}\\right)}=\\frac{l^{d}}{m^{d}}$ where $(l, m)=1$ and $l, m>0$. This can be rewritten in the expanded form $$ b_{d}\\left(m^{d} y_{i}^{d}-l^{d} y_{1}^{d}\\right)=-\\sum_{j=0}^{d-2} b_{j}\\left(m^{d} y_{i}^{j}-l^{d} y_{1}^{j}\\right) $$ Let $c$ be the common denominator of $Q$, so that $c Q(k)$ is an integer for any integer $k$. Note that $c$ depends only on $P$ and so we may assume $(p, c)=1$. Then $y_{1} \\equiv y_{i}(\\bmod p)$ implies $c Q\\left(y_{1}\\right) \\equiv c Q\\left(y_{i}\\right)(\\bmod p)$. - Case 1. $p \\mid c Q\\left(y_{1}\\right)$. In this case, there is a cancellation of $p$ in the numerator and denominator of $\\frac{c Q\\left(y_{i}\\right)}{c Q\\left(y_{1}\\right)}$, so that $m^{d} \\leqslant p^{-1}\\left|c Q\\left(y_{1}\\right)\\right|$. Noting $\\left|Q\\left(y_{1}\\right)\\right|<2 B y_{1}^{d}$ as $y_{1}$ is large, we get $$ m \\leqslant p^{-\\frac{1}{d}}(2 c B)^{\\frac{1}{d}} y_{1} $$ For large $y_{1}$ and $y_{i}$, the relation $\\frac{1}{2}<\\frac{Q\\left(y_{i}\\right)}{Q\\left(y_{1}\\right)}<2$ implies $$ \\frac{1}{3}<\\frac{y_{i}^{d}}{y_{1}^{d}}<3 $$ We also have $$ \\frac{1}{2}<\\frac{l^{d}}{m^{d}}<2 $$ Now, the left-hand side of (1) is $$ b_{d}\\left(m y_{i}-l y_{1}\\right)\\left(m^{d-1} y_{i}^{d-1}+m^{d-2} y_{i}^{d-2} l y_{1}+\\cdots+l^{d-1} y_{1}^{d-1}\\right) . $$ Suppose on the contrary that $m y_{i}-l y_{1} \\neq 0$. Then the absolute value of the above expression is at least $\\left|b_{d}\\right| m^{d-1} y_{i}^{d-1}$. On the other hand, the absolute value of the right-hand side of (1) is at most $$ \\begin{aligned} \\sum_{j=0}^{d-2} B\\left(m^{d} y_{i}^{j}+l^{d} y_{1}^{j}\\right) & \\leqslant(d-1) B\\left(m^{d} y_{i}^{d-2}+l^{d} y_{1}^{d-2}\\right) \\\\ & \\leqslant(d-1) B\\left(7 m^{d} y_{i}^{d-2}\\right) \\\\ & \\leqslant 7(d-1) B\\left(p^{-\\frac{1}{d}}(2 c B)^{\\frac{1}{d}} y_{1}\\right) m^{d-1} y_{i}^{d-2} \\\\ & \\leqslant 21(d-1) B p^{-\\frac{1}{d}}(2 c B)^{\\frac{1}{d}} m^{d-1} y_{i}^{d-1} \\end{aligned} $$ by using successively (3), (4), (2) and again (3). This shows $$ \\left|b_{d}\\right| m^{d-1} y_{i}^{d-1} \\leqslant 21(d-1) B p^{-\\frac{1}{d}}(2 c B)^{\\frac{1}{d}} m^{d-1} y_{i}^{d-1}, $$ which is a contradiction for large $p$ as $b_{d}, B, c, d$ depend only on the polynomial $P$. Therefore, we have $m y_{i}-l y_{1}=0$ in this case. - Case 2. $\\left(p, c Q\\left(y_{1}\\right)\\right)=1$. From $c Q\\left(y_{1}\\right) \\equiv c Q\\left(y_{i}\\right)(\\bmod p)$, we have $l^{d} \\equiv m^{d}(\\bmod p)$. Since $(p-1, d)=1$, we use Fermat Little Theorem to conclude $l \\equiv m(\\bmod p)$. Then $p \\mid m y_{i}-l y_{1}$. Suppose on the contrary that $m y_{i}-l y_{1} \\neq 0$. Then the left-hand side of (1) has absolute value at least $\\left|b_{d}\\right| p m^{d-1} y_{i}^{d-1}$. Similar to Case 1, the right-hand side of (1) has absolute value at most $$ 21(d-1) B(2 c B)^{\\frac{1}{d}} m^{d-1} y_{i}^{d-1} $$ which must be smaller than $\\left|b_{d}\\right| p m^{d-1} y_{i}^{d-1}$ for large $p$. Again this yields a contradiction and hence $m y_{i}-l y_{1}=0$. In both cases, we find that $\\frac{Q\\left(y_{i}\\right)}{Q\\left(y_{1}\\right)}=\\frac{l^{d}}{m^{d}}=\\frac{y_{i}^{d}}{y_{1}^{d}}$. From the Claim, the polynomial $Q\\left(y_{1}\\right) y^{d}-y_{1}^{d} Q(y)$ has roots $y=y_{1}, y_{2}, \\ldots, y_{d+1}$. Since its degree is at most $d$, this must be the zero polynomial. Hence, $Q(y)=b_{d} y^{d}$. This implies $P(x)=a_{d}\\left(x+\\frac{a_{d-1}}{d a_{d}}\\right)^{d}$. Let $\\frac{a_{d-1}}{d a_{d}}=\\frac{s}{r}$ with integers $r, s$ where $r \\geqslant 1$ and $(r, s)=1$. Since $P$ has integer coefficients, we need $r^{d} \\mid a_{d}$. Let $a_{d}=r^{d} a$. Then $P(x)=a(r x+s)^{d}$. It is obvious that such a polynomial satisfies the conditions. Comment. In the proof, the use of prime and Dirichlet's Theorem can be avoided. One can easily show that each $P\\left(x_{i}\\right)$ can be expressed in the form $u v_{i}^{d}$ where $u, v_{i}$ are integers and $u$ cannot be divisible by the $d$-th power of a prime (note that $u$ depends only on $P$ ). By fixing a large integer $q$ and by choosing a large $n$, we can apply the Pigeonhole Principle and assume $x_{1} \\equiv x_{2} \\equiv \\cdots \\equiv x_{d+1}(\\bmod q)$ and $v_{1} \\equiv v_{2} \\equiv \\cdots \\equiv v_{d+1}(\\bmod q)$. Then the remaining proof is similar to Case 2 of the Solution. Alternatively, we give another modification of the proof as follows. We take a sufficiently large $n$ and consider the corresponding positive integers $y_{1}, y_{2}, \\ldots, y_{n}$. For each $2 \\leqslant i \\leqslant n$, let $\\frac{Q\\left(y_{i}\\right)}{Q\\left(y_{1}\\right)}=\\frac{l_{i}^{d}}{m_{i}^{d}}$. As in Case 1, if there are $d$ indices $i$ such that the integers $\\frac{c\\left|Q\\left(y_{1}\\right)\\right|}{m_{i}^{d}}$ are bounded below by a constant depending only on $P$, we can establish the Claim using those $y_{i}$ 's and complete the proof. Similarly, as in Case 2, if there are $d$ indices $i$ such that the integers $\\left|m_{i} y_{i}-l_{i} y_{1}\\right|$ are bounded below, then the proof goes the same. So it suffices to consider the case where $\\frac{c\\left|Q\\left(y_{1}\\right)\\right|}{m_{i}^{d}} \\leqslant M$ and $\\left|m_{i} y_{i}-l_{i} y_{1}\\right| \\leqslant N$ for all $2 \\leqslant i \\leqslant n^{\\prime}$ where $M, N$ are fixed constants and $n^{\\prime}$ is large. Since there are only finitely many choices for $m_{i}$ and $m_{i} y_{i}-l_{i} y_{1}$, by the Pigeonhole Principle, we can assume without loss of generality $m_{i}=m$ and $m_{i} y_{i}-l_{i} y_{1}=t$ for $2 \\leqslant i \\leqslant d+2$. Then $$ \\frac{Q\\left(y_{i}\\right)}{Q\\left(y_{1}\\right)}=\\frac{l_{i}^{d}}{m^{d}}=\\frac{\\left(m y_{i}-t\\right)^{d}}{m^{d} y_{1}^{d}} $$ so that $Q\\left(y_{1}\\right)(m y-t)^{d}-m^{d} y_{1}^{d} Q(y)$ has roots $y=y_{2}, y_{3}, \\ldots, y_{d+2}$. Its degree is at most $d$ and hence it is the zero polynomial. Therefore, $Q(y)=\\frac{b_{d}}{m^{d}}(m y-t)^{d}$. Indeed, $Q$ does not have the term $y^{d-1}$, which means $t$ should be 0 . This gives the corresponding $P(x)$ of the desired form. The two modifications of the Solution work equally well when the degree $d$ is even.","tier":0} +{"problem_type":"Algebra","problem_label":"A2","problem":"Find the smallest real constant $C$ such that for any positive real numbers $a_{1}, a_{2}, a_{3}, a_{4}$ and $a_{5}$ (not necessarily distinct), one can always choose distinct subscripts $i, j, k$ and $l$ such that $$ \\left|\\frac{a_{i}}{a_{j}}-\\frac{a_{k}}{a_{l}}\\right| \\leqslant C . $$ Answer. The smallest $C$ is $\\frac{1}{2}$.","solution":"We first show that $C \\leqslant \\frac{1}{2}$. For any positive real numbers $a_{1} \\leqslant a_{2} \\leqslant a_{3} \\leqslant a_{4} \\leqslant a_{5}$, consider the five fractions $$ \\frac{a_{1}}{a_{2}}, \\frac{a_{3}}{a_{4}}, \\frac{a_{1}}{a_{5}}, \\frac{a_{2}}{a_{3}}, \\frac{a_{4}}{a_{5}} . $$ Each of them lies in the interval $(0,1]$. Therefore, by the Pigeonhole Principle, at least three of them must lie in $\\left(0, \\frac{1}{2}\\right]$ or lie in $\\left(\\frac{1}{2}, 1\\right]$ simultaneously. In particular, there must be two consecutive terms in (2) which belong to an interval of length $\\frac{1}{2}$ (here, we regard $\\frac{a_{1}}{a_{2}}$ and $\\frac{a_{4}}{a_{5}}$ as consecutive). In other words, the difference of these two fractions is less than $\\frac{1}{2}$. As the indices involved in these two fractions are distinct, we can choose them to be $i, j, k, l$ and conclude that $C \\leqslant \\frac{1}{2}$. Next, we show that $C=\\frac{1}{2}$ is best possible. Consider the numbers $1,2,2,2, n$ where $n$ is a large real number. The fractions formed by two of these numbers in ascending order are $\\frac{1}{n}, \\frac{2}{n}, \\frac{1}{2}, \\frac{2}{2}, \\frac{2}{1}, \\frac{n}{2}, \\frac{n}{1}$. Since the indices $i, j, k, l$ are distinct, $\\frac{1}{n}$ and $\\frac{2}{n}$ cannot be chosen simultaneously. Therefore the minimum value of the left-hand side of (1) is $\\frac{1}{2}-\\frac{2}{n}$. When $n$ tends to infinity, this value approaches $\\frac{1}{2}$, and so $C$ cannot be less than $\\frac{1}{2}$. These conclude that $C=\\frac{1}{2}$ is the smallest possible choice. Comment. The conclusion still holds if $a_{1}, a_{2}, \\ldots, a_{5}$ are pairwise distinct, since in the construction, we may replace the 2's by real numbers sufficiently close to 2 . There are two possible simplifications for this problem: (i) the answer $C=\\frac{1}{2}$ is given to the contestants; or (ii) simply ask the contestants to prove the inequality (1) for $C=\\frac{1}{2}$.","tier":0} +{"problem_type":"Algebra","problem_label":"A3","problem":"Find all integers $n \\geqslant 3$ with the following property: for all real numbers $a_{1}, a_{2}, \\ldots, a_{n}$ and $b_{1}, b_{2}, \\ldots, b_{n}$ satisfying $\\left|a_{k}\\right|+\\left|b_{k}\\right|=1$ for $1 \\leqslant k \\leqslant n$, there exist $x_{1}, x_{2}, \\ldots, x_{n}$, each of which is either -1 or 1 , such that $$ \\left|\\sum_{k=1}^{n} x_{k} a_{k}\\right|+\\left|\\sum_{k=1}^{n} x_{k} b_{k}\\right| \\leqslant 1 $$ Answer. $n$ can be any odd integer greater than or equal to 3 .","solution":"For any even integer $n \\geqslant 4$, we consider the case $$ a_{1}=a_{2}=\\cdots=a_{n-1}=b_{n}=0 \\quad \\text { and } \\quad b_{1}=b_{2}=\\cdots=b_{n-1}=a_{n}=1 $$ The condition $\\left|a_{k}\\right|+\\left|b_{k}\\right|=1$ is satisfied for each $1 \\leqslant k \\leqslant n$. No matter how we choose each $x_{k}$, both sums $\\sum_{k=1}^{n} x_{k} a_{k}$ and $\\sum_{k=1}^{n} x_{k} b_{k}$ are odd integers. This implies $\\left|\\sum_{k=1}^{n} x_{k} a_{k}\\right| \\geqslant 1$ and $\\left|\\sum_{k=1}^{n} x_{k} b_{k}\\right| \\geqslant 1$, which shows (1) cannot hold. For any odd integer $n \\geqslant 3$, we may assume without loss of generality $b_{k} \\geqslant 0$ for $1 \\leqslant k \\leqslant n$ (this can be done by flipping the pair $\\left(a_{k}, b_{k}\\right)$ to $\\left(-a_{k},-b_{k}\\right)$ and $x_{k}$ to $-x_{k}$ if necessary) and $a_{1} \\geqslant a_{2} \\geqslant \\cdots \\geqslant a_{m} \\geqslant 0>a_{m+1} \\geqslant \\cdots \\geqslant a_{n}$. We claim that the choice $x_{k}=(-1)^{k+1}$ for $1 \\leqslant k \\leqslant n$ will work. Define $$ s=\\sum_{k=1}^{m} x_{k} a_{k} \\quad \\text { and } \\quad t=-\\sum_{k=m+1}^{n} x_{k} a_{k} . $$ Note that $$ s=\\left(a_{1}-a_{2}\\right)+\\left(a_{3}-a_{4}\\right)+\\cdots \\geqslant 0 $$ by the assumption $a_{1} \\geqslant a_{2} \\geqslant \\cdots \\geqslant a_{m}$ (when $m$ is odd, there is a single term $a_{m}$ at the end, which is also positive). Next, we have $$ s=a_{1}-\\left(a_{2}-a_{3}\\right)-\\left(a_{4}-a_{5}\\right)-\\cdots \\leqslant a_{1} \\leqslant 1 $$ Similarly, $$ t=\\left(-a_{n}+a_{n-1}\\right)+\\left(-a_{n-2}+a_{n-3}\\right)+\\cdots \\geqslant 0 $$ and $$ t=-a_{n}+\\left(a_{n-1}-a_{n-2}\\right)+\\left(a_{n-3}-a_{n-4}\\right)+\\cdots \\leqslant-a_{n} \\leqslant 1 . $$ From the condition, we have $a_{k}+b_{k}=1$ for $1 \\leqslant k \\leqslant m$ and $-a_{k}+b_{k}=1$ for $m+1 \\leqslant k \\leqslant n$. It follows that $\\sum_{k=1}^{n} x_{k} a_{k}=s-t$ and $\\sum_{k=1}^{n} x_{k} b_{k}=1-s-t$. Hence it remains to prove $$ |s-t|+|1-s-t| \\leqslant 1 $$ under the constraint $0 \\leqslant s, t \\leqslant 1$. By symmetry, we may assume $s \\geqslant t$. If $1-s-t \\geqslant 0$, then we have $$ |s-t|+|1-s-t|=s-t+1-s-t=1-2 t \\leqslant 1 $$ If $1-s-t \\leqslant 0$, then we have $$ |s-t|+|1-s-t|=s-t-1+s+t=2 s-1 \\leqslant 1 $$ Hence, the inequality is true in both cases. These show $n$ can be any odd integer greater than or equal to 3 .","tier":0} +{"problem_type":"Algebra","problem_label":"A3","problem":"Find all integers $n \\geqslant 3$ with the following property: for all real numbers $a_{1}, a_{2}, \\ldots, a_{n}$ and $b_{1}, b_{2}, \\ldots, b_{n}$ satisfying $\\left|a_{k}\\right|+\\left|b_{k}\\right|=1$ for $1 \\leqslant k \\leqslant n$, there exist $x_{1}, x_{2}, \\ldots, x_{n}$, each of which is either -1 or 1 , such that $$ \\left|\\sum_{k=1}^{n} x_{k} a_{k}\\right|+\\left|\\sum_{k=1}^{n} x_{k} b_{k}\\right| \\leqslant 1 $$ Answer. $n$ can be any odd integer greater than or equal to 3 .","solution":"The even case can be handled in the same way as Firstly, for $n=3$, we may assume without loss of generality $a_{1} \\geqslant a_{2} \\geqslant a_{3} \\geqslant 0$ and $b_{1}=a_{1}-1$ (if $b_{1}=1-a_{1}$, we may replace each $b_{k}$ by $-b_{k}$ ). - Case 1. $b_{2}=a_{2}-1$ and $b_{3}=a_{3}-1$, in which case we take $\\left(x_{1}, x_{2}, x_{3}\\right)=(1,-1,1)$. Let $c=a_{1}-a_{2}+a_{3}$ so that $0 \\leqslant c \\leqslant 1$. Then $\\left|b_{1}-b_{2}+b_{3}\\right|=\\left|a_{1}-a_{2}+a_{3}-1\\right|=1-c$ and hence $|c|+\\left|b_{1}-b_{2}+b_{3}\\right|=1$. - Case 2. $b_{2}=1-a_{2}$ and $b_{3}=1-a_{3}$, in which case we take $\\left(x_{1}, x_{2}, x_{3}\\right)=(1,-1,1)$. Let $c=a_{1}-a_{2}+a_{3}$ so that $0 \\leqslant c \\leqslant 1$. Since $a_{3} \\leqslant a_{2}$ and $a_{1} \\leqslant 1$, we have $$ c-1 \\leqslant b_{1}-b_{2}+b_{3}=a_{1}+a_{2}-a_{3}-1 \\leqslant 1-c $$ This gives $\\left|b_{1}-b_{2}+b_{3}\\right| \\leqslant 1-c$ and hence $|c|+\\left|b_{1}-b_{2}+b_{3}\\right| \\leqslant 1$. - Case 3. $b_{2}=a_{2}-1$ and $b_{3}=1-a_{3}$, in which case we take $\\left(x_{1}, x_{2}, x_{3}\\right)=(-1,1,1)$. Let $c=-a_{1}+a_{2}+a_{3}$. If $c \\geqslant 0$, then $a_{3} \\leqslant 1$ and $a_{2} \\leqslant a_{1}$ imply $$ c-1 \\leqslant-b_{1}+b_{2}+b_{3}=-a_{1}+a_{2}-a_{3}+1 \\leqslant 1-c $$ If $c<0$, then $a_{1} \\leqslant a_{2}+1$ and $a_{3} \\geqslant 0$ imply $$ -c-1 \\leqslant-b_{1}+b_{2}+b_{3}=-a_{1}+a_{2}-a_{3}+1 \\leqslant 1+c $$ In both cases, we get $\\left|-b_{1}+b_{2}+b_{3}\\right| \\leqslant 1-|c|$ and hence $|c|+\\left|-b_{1}+b_{2}+b_{3}\\right| \\leqslant 1$. - Case 4. $b_{2}=1-a_{2}$ and $b_{3}=a_{3}-1$, in which case we take $\\left(x_{1}, x_{2}, x_{3}\\right)=(-1,1,1)$. Let $c=-a_{1}+a_{2}+a_{3}$. If $c \\geqslant 0$, then $a_{2} \\leqslant 1$ and $a_{3} \\leqslant a_{1}$ imply $$ c-1 \\leqslant-b_{1}+b_{2}+b_{3}=-a_{1}-a_{2}+a_{3}+1 \\leqslant 1-c $$ If $c<0$, then $a_{1} \\leqslant a_{3}+1$ and $a_{2} \\geqslant 0$ imply $$ -c-1 \\leqslant-b_{1}+b_{2}+b_{3}=-a_{1}-a_{2}+a_{3}+1 \\leqslant 1+c $$ In both cases, we get $\\left|-b_{1}+b_{2}+b_{3}\\right| \\leqslant 1-|c|$ and hence $|c|+\\left|-b_{1}+b_{2}+b_{3}\\right| \\leqslant 1$. We have found $x_{1}, x_{2}, x_{3}$ satisfying (1) in each case for $n=3$. Now, let $n \\geqslant 5$ be odd and suppose the result holds for any smaller odd cases. Again we may assume $a_{k} \\geqslant 0$ for each $1 \\leqslant k \\leqslant n$. By the Pigeonhole Principle, there are at least three indices $k$ for which $b_{k}=a_{k}-1$ or $b_{k}=1-a_{k}$. Without loss of generality, suppose $b_{k}=a_{k}-1$ for $k=1,2,3$. Again by the Pigeonhole Principle, as $a_{1}, a_{2}, a_{3}$ lies between 0 and 1 , the difference of two of them is at most $\\frac{1}{2}$. By changing indices if necessary, we may assume $0 \\leqslant d=a_{1}-a_{2} \\leqslant \\frac{1}{2}$. By the inductive hypothesis, we can choose $x_{3}, x_{4}, \\ldots, x_{n}$ such that $a^{\\prime}=\\sum_{k=3}^{n} x_{k} a_{k}$ and $b^{\\prime}=\\sum_{k=3}^{n} x_{k} b_{k}$ satisfy $\\left|a^{\\prime}\\right|+\\left|b^{\\prime}\\right| \\leqslant 1$. We may further assume $a^{\\prime} \\geqslant 0$. - Case 1. $b^{\\prime} \\geqslant 0$, in which case we take $\\left(x_{1}, x_{2}\\right)=(-1,1)$. We have $\\left|-a_{1}+a_{2}+a^{\\prime}\\right|+\\left|-\\left(a_{1}-1\\right)+\\left(a_{2}-1\\right)+b^{\\prime}\\right|=\\left|-d+a^{\\prime}\\right|+\\left|-d+b^{\\prime}\\right| \\leqslant$ $\\max \\left\\{a^{\\prime}+b^{\\prime}-2 d, a^{\\prime}-b^{\\prime}, b^{\\prime}-a^{\\prime}, 2 d-a^{\\prime}-b^{\\prime}\\right\\} \\leqslant 1$ since $0 \\leqslant a^{\\prime}, b^{\\prime}, a^{\\prime}+b^{\\prime} \\leqslant 1$ and $0 \\leqslant d \\leqslant \\frac{1}{2}$. - Case 2. $0>b^{\\prime} \\geqslant-a^{\\prime}$, in which case we take $\\left(x_{1}, x_{2}\\right)=(-1,1)$. We have $\\left|-a_{1}+a_{2}+a^{\\prime}\\right|+\\left|-\\left(a_{1}-1\\right)+\\left(a_{2}-1\\right)+b^{\\prime}\\right|=\\left|-d+a^{\\prime}\\right|+\\left|-d+b^{\\prime}\\right|$. If $-d+a^{\\prime} \\geqslant 0$, this equals $a^{\\prime}-b^{\\prime}=\\left|a^{\\prime}\\right|+\\left|b^{\\prime}\\right| \\leqslant 1$. If $-d+a^{\\prime}<0$, this equals $2 d-a^{\\prime}-b^{\\prime} \\leqslant 2 d \\leqslant 1$. - Case 3. $b^{\\prime}<-a^{\\prime}$, in which case we take $\\left(x_{1}, x_{2}\\right)=(1,-1)$. We have $\\left|a_{1}-a_{2}+a^{\\prime}\\right|+\\left|\\left(a_{1}-1\\right)-\\left(a_{2}-1\\right)+b^{\\prime}\\right|=\\left|d+a^{\\prime}\\right|+\\left|d+b^{\\prime}\\right|$. If $d+b^{\\prime} \\geqslant 0$, this equals $2 d+a^{\\prime}+b^{\\prime}<2 d \\leqslant 1$. If $d+b^{\\prime}<0$, this equals $a^{\\prime}-b^{\\prime}=\\left|a^{\\prime}\\right|+\\left|b^{\\prime}\\right| \\leqslant 1$. Therefore, we have found $x_{1}, x_{2}, \\ldots, x_{n}$ satisfying (1) in each case. By induction, the property holds for all odd integers $n \\geqslant 3$.","tier":0} +{"problem_type":"Algebra","problem_label":"A4","problem":"Denote by $\\mathbb{R}^{+}$the set of all positive real numbers. Find all functions $f: \\mathbb{R}^{+} \\rightarrow \\mathbb{R}^{+}$such that $$ x f\\left(x^{2}\\right) f(f(y))+f(y f(x))=f(x y)\\left(f\\left(f\\left(x^{2}\\right)\\right)+f\\left(f\\left(y^{2}\\right)\\right)\\right) $$ for all positive real numbers $x$ and $y$. Answer. $f(x)=\\frac{1}{x}$ for any $x \\in \\mathbb{R}^{+}$. ![](https:\/\/cdn.mathpix.com\/cropped\/2024_11_18_e5e735288fb13d37ab0cg-20.jpg?height=63&width=1663&top_left_y=565&top_left_x=177) hence $f(1)=1$. Swapping $x$ and $y$ in (1) and comparing with (1) again, we find $$ x f\\left(x^{2}\\right) f(f(y))+f(y f(x))=y f\\left(y^{2}\\right) f(f(x))+f(x f(y)) . $$ Taking $y=1$ in (2), we have $x f\\left(x^{2}\\right)+f(f(x))=f(f(x))+f(x)$, that is, $$ f\\left(x^{2}\\right)=\\frac{f(x)}{x} $$ Take $y=1$ in (1) and apply (3) to $x f\\left(x^{2}\\right)$. We get $f(x)+f(f(x))=f(x)\\left(f\\left(f\\left(x^{2}\\right)\\right)+1\\right)$, which implies $$ f\\left(f\\left(x^{2}\\right)\\right)=\\frac{f(f(x))}{f(x)} $$ For any $x \\in \\mathbb{R}^{+}$, we find that $$ f\\left(f(x)^{2}\\right) \\stackrel{(3)}{=} \\frac{f(f(x))}{f(x)} \\stackrel{(4)}{=} f\\left(f\\left(x^{2}\\right)\\right) \\stackrel{(3)}{=} f\\left(\\frac{f(x)}{x}\\right) $$ It remains to show the following key step. - Claim. The function $f$ is injective. Proof. Using (3) and (4), we rewrite (1) as $$ f(x) f(f(y))+f(y f(x))=f(x y)\\left(\\frac{f(f(x))}{f(x)}+\\frac{f(f(y))}{f(y)}\\right) . $$ Take $x=y$ in (6) and apply (3). This gives $f(x) f(f(x))+f(x f(x))=2 \\frac{f(f(x))}{x}$, which means $$ f(x f(x))=f(f(x))\\left(\\frac{2}{x}-f(x)\\right) $$ Using (3), equation (2) can be rewritten as $$ f(x) f(f(y))+f(y f(x))=f(y) f(f(x))+f(x f(y)) $$ Suppose $f(x)=f(y)$ for some $x, y \\in \\mathbb{R}^{+}$. Then (8) implies $$ f(y f(y))=f(y f(x))=f(x f(y))=f(x f(x)) $$ Using (7), this gives $$ f(f(y))\\left(\\frac{2}{y}-f(y)\\right)=f(f(x))\\left(\\frac{2}{x}-f(x)\\right) $$ Noting $f(x)=f(y)$, we find $x=y$. This establishes the injectivity. By the Claim and (5), we get the only possible solution $f(x)=\\frac{1}{x}$. It suffices to check that this is a solution. Indeed, the left-hand side of (1) becomes $$ x \\cdot \\frac{1}{x^{2}} \\cdot y+\\frac{x}{y}=\\frac{y}{x}+\\frac{x}{y}, $$ while the right-hand side becomes $$ \\frac{1}{x y}\\left(x^{2}+y^{2}\\right)=\\frac{x}{y}+\\frac{y}{x} . $$ The two sides agree with each other.","solution":"Taking $x=y=1$ in (1), we get $f(1) f(f(1))+f(f(1))=2 f(1) f(f(1))$ and hence $f(1)=1$. Putting $x=1$ in (1), we have $f(f(y))+f(y)=f(y)\\left(1+f\\left(f\\left(y^{2}\\right)\\right)\\right)$ so that $$ f(f(y))=f(y) f\\left(f\\left(y^{2}\\right)\\right) $$ Putting $y=1$ in (1), we get $x f\\left(x^{2}\\right)+f(f(x))=f(x)\\left(f\\left(f\\left(x^{2}\\right)\\right)+1\\right)$. Using (9), this gives $$ x f\\left(x^{2}\\right)=f(x) $$ Replace $y$ by $\\frac{1}{x}$ in (1). Then we have $$ x f\\left(x^{2}\\right) f\\left(f\\left(\\frac{1}{x}\\right)\\right)+f\\left(\\frac{f(x)}{x}\\right)=f\\left(f\\left(x^{2}\\right)\\right)+f\\left(f\\left(\\frac{1}{x^{2}}\\right)\\right) . $$ The relation (10) shows $f\\left(\\frac{f(x)}{x}\\right)=f\\left(f\\left(x^{2}\\right)\\right)$. Also, using (9) with $y=\\frac{1}{x}$ and using (10) again, the last equation reduces to $$ f(x) f\\left(\\frac{1}{x}\\right)=1 $$ Replace $x$ by $\\frac{1}{x}$ and $y$ by $\\frac{1}{y}$ in (1) and apply (11). We get $$ \\frac{1}{x f\\left(x^{2}\\right) f(f(y))}+\\frac{1}{f(y f(x))}=\\frac{1}{f(x y)}\\left(\\frac{1}{f\\left(f\\left(x^{2}\\right)\\right)}+\\frac{1}{f\\left(f\\left(y^{2}\\right)\\right)}\\right) . $$ Clearing denominators, we can use (1) to simplify the numerators and obtain $$ f(x y)^{2} f\\left(f\\left(x^{2}\\right)\\right) f\\left(f\\left(y^{2}\\right)\\right)=x f\\left(x^{2}\\right) f(f(y)) f(y f(x)) $$ Using (9) and (10), this is the same as $$ f(x y)^{2} f(f(x))=f(x)^{2} f(y) f(y f(x)) $$ Substitute $y=f(x)$ in (12) and apply (10) (with $x$ replaced by $f(x)$ ). We have $$ f(x f(x))^{2}=f(x) f(f(x)) $$ Taking $y=x$ in (12), squaring both sides, and using (10) and (13), we find that $$ f(f(x))=x^{4} f(x)^{3} $$ Finally, we combine (9), (10) and (14) to get $$ y^{4} f(y)^{3} \\stackrel{(14)}{=} f(f(y)) \\stackrel{(9)}{=} f(y) f\\left(f\\left(y^{2}\\right)\\right) \\stackrel{(14)}{=} f(y) y^{8} f\\left(y^{2}\\right)^{3} \\stackrel{(10)}{=} y^{5} f(y)^{4}, $$ which implies $f(y)=\\frac{1}{y}$. This is a solution by the checking in Solution 1.","tier":0} +{"problem_type":"Algebra","problem_label":"A6","problem":"The equation $$ (x-1)(x-2) \\cdots(x-2016)=(x-1)(x-2) \\cdots(x-2016) $$ is written on the board. One tries to erase some linear factors from both sides so that each side still has at least one factor, and the resulting equation has no real roots. Find the least number of linear factors one needs to erase to achieve this. Answer. 2016.","solution":"Since there are 2016 common linear factors on both sides, we need to erase at least 2016 factors. We claim that the equation has no real roots if we erase all factors $(x-k)$ on the left-hand side with $k \\equiv 2,3(\\bmod 4)$, and all factors $(x-m)$ on the right-hand side with $m \\equiv 0,1(\\bmod 4)$. Therefore, it suffices to show that no real number $x$ satisfies $$ \\prod_{j=0}^{503}(x-4 j-1)(x-4 j-4)=\\prod_{j=0}^{503}(x-4 j-2)(x-4 j-3) $$ - Case 1. $x=1,2, \\ldots, 2016$. In this case, one side of (1) is zero while the other side is not. This shows $x$ cannot satisfy (1). - Case 2. $4 k+12016$ or $4 k2$ for $0 \\leqslant j \\leqslant 503$ in this case. So each term in the product lies strictly between 0 and 1 , and the whole product must be less than 1 , which is impossible. - Case 4. $4 k+2\\frac{n+1}{2}$, we have $|l(x)|>|r(x)|$. If $n \\equiv 3(\\bmod 4)$, one may leave $l(x)=(x-1)(x-2) \\cdots\\left(x-\\frac{n+1}{2}\\right)$ on the left-hand side and $r(x)=\\left(x-\\frac{n+3}{2}\\right)\\left(x-\\frac{x+5}{2}\\right) \\cdots(x-n)$ on the right-hand side. For $x<1$ or $\\frac{n+1}{2}0>r(x)$. For $1\\frac{n+3}{2}$, we have $|l(x)|>|r(x)|$. If $n \\equiv 1(\\bmod 4)$, as the proposer mentioned, the situation is a bit more out of control. Since the construction for $n-1 \\equiv 0(\\bmod 4)$ works, the answer can be either $n$ or $n-1$. For $n=5$, we can leave the products $(x-1)(x-2)(x-3)(x-4)$ and $(x-5)$. For $n=9$, the only example that works is $l(x)=(x-1)(x-2)(x-9)$ and $r(x)=(x-3)(x-4) \\cdots(x-8)$, while there seems to be no such partition for $n=13$.","tier":0} +{"problem_type":"Answer.","problem_label":"A8","problem":"Determine the largest real number $a$ such that for all $n \\geqslant 1$ and for all real numbers $x_{0}, x_{1}, \\ldots, x_{n}$ satisfying $0=x_{0}4 \\sum_{k=1}^{n} \\frac{k+1}{x_{k}} . $$ This shows (1) holds for $a=\\frac{4}{9}$. Next, we show that $a=\\frac{4}{9}$ is the optimal choice. Consider the sequence defined by $x_{0}=0$ and $x_{k}=x_{k-1}+k(k+1)$ for $k \\geqslant 1$, that is, $x_{k}=\\frac{1}{3} k(k+1)(k+2)$. Then the left-hand side of (1) equals $$ \\sum_{k=1}^{n} \\frac{1}{k(k+1)}=\\sum_{k=1}^{n}\\left(\\frac{1}{k}-\\frac{1}{k+1}\\right)=1-\\frac{1}{n+1} $$ while the right-hand side equals $$ a \\sum_{k=1}^{n} \\frac{k+1}{x_{k}}=3 a \\sum_{k=1}^{n} \\frac{1}{k(k+2)}=\\frac{3}{2} a \\sum_{k=1}^{n}\\left(\\frac{1}{k}-\\frac{1}{k+2}\\right)=\\frac{3}{2}\\left(1+\\frac{1}{2}-\\frac{1}{n+1}-\\frac{1}{n+2}\\right) a . $$ When $n$ tends to infinity, the left-hand side tends to 1 while the right-hand side tends to $\\frac{9}{4} a$. Therefore $a$ has to be at most $\\frac{4}{9}$. Hence the largest value of $a$ is $\\frac{4}{9}$.","tier":0} +{"problem_type":"Answer.","problem_label":"A8","problem":"Determine the largest real number $a$ such that for all $n \\geqslant 1$ and for all real numbers $x_{0}, x_{1}, \\ldots, x_{n}$ satisfying $0=x_{0}0$ for $1 \\leqslant k \\leqslant n$. By the Cauchy-Schwarz Inequality, for $1 \\leqslant k \\leqslant n$, we have $$ \\left(y_{1}+y_{2}+\\cdots+y_{k}\\right)\\left(\\sum_{j=1}^{k} \\frac{1}{y_{j}}\\binom{j+1}{2}^{2}\\right) \\geqslant\\left(\\binom{2}{2}+\\binom{3}{2}+\\cdots+\\binom{k+1}{2}\\right)^{2}=\\binom{k+2}{3}^{2} $$ This can be rewritten as $$ \\frac{k+1}{y_{1}+y_{2}+\\cdots+y_{k}} \\leqslant \\frac{36}{k^{2}(k+1)(k+2)^{2}}\\left(\\sum_{j=1}^{k} \\frac{1}{y_{j}}\\binom{j+1}{2}^{2}\\right) . $$ Summing (3) over $k=1,2, \\ldots, n$, we get $$ \\frac{2}{y_{1}}+\\frac{3}{y_{1}+y_{2}}+\\cdots+\\frac{n+1}{y_{1}+y_{2}+\\cdots+y_{n}} \\leqslant \\frac{c_{1}}{y_{1}}+\\frac{c_{2}}{y_{2}}+\\cdots+\\frac{c_{n}}{y_{n}} $$ where for $1 \\leqslant m \\leqslant n$, $$ \\begin{aligned} c_{m} & =36\\binom{m+1}{2}^{2} \\sum_{k=m}^{n} \\frac{1}{k^{2}(k+1)(k+2)^{2}} \\\\ & =\\frac{9 m^{2}(m+1)^{2}}{4} \\sum_{k=m}^{n}\\left(\\frac{1}{k^{2}(k+1)^{2}}-\\frac{1}{(k+1)^{2}(k+2)^{2}}\\right) \\\\ & =\\frac{9 m^{2}(m+1)^{2}}{4}\\left(\\frac{1}{m^{2}(m+1)^{2}}-\\frac{1}{(n+1)^{2}(n+2)^{2}}\\right)<\\frac{9}{4} . \\end{aligned} $$ From (4), the inequality (1) holds for $a=\\frac{4}{9}$. This is also the upper bound as can be verified in the same way as Solution 1.","tier":0} +{"problem_type":"Combinatorics","problem_label":"C1","problem":"The leader of an IMO team chooses positive integers $n$ and $k$ with $n>k$, and announces them to the deputy leader and a contestant. The leader then secretly tells the deputy leader an $n$-digit binary string, and the deputy leader writes down all $n$-digit binary strings which differ from the leader's in exactly $k$ positions. (For example, if $n=3$ and $k=1$, and if the leader chooses 101, the deputy leader would write down 001, 111 and 100.) The contestant is allowed to look at the strings written by the deputy leader and guess the leader's string. What is the minimum number of guesses (in terms of $n$ and $k$ ) needed to guarantee the correct answer? Answer. The minimum number of guesses is 2 if $n=2 k$ and 1 if $n \\neq 2 k$.","solution":"Let $X$ be the binary string chosen by the leader and let $X^{\\prime}$ be the binary string of length $n$ every digit of which is different from that of $X$. The strings written by the deputy leader are the same as those in the case when the leader's string is $X^{\\prime}$ and $k$ is changed to $n-k$. In view of this, we may assume $k \\geqslant \\frac{n}{2}$. Also, for the particular case $k=\\frac{n}{2}$, this argument shows that the strings $X$ and $X^{\\prime}$ cannot be distinguished, and hence in that case the contestant has to guess at least twice. It remains to show that the number of guesses claimed suffices. Consider any string $Y$ which differs from $X$ in $m$ digits where $0k$, and announces them to the deputy leader and a contestant. The leader then secretly tells the deputy leader an $n$-digit binary string, and the deputy leader writes down all $n$-digit binary strings which differ from the leader's in exactly $k$ positions. (For example, if $n=3$ and $k=1$, and if the leader chooses 101, the deputy leader would write down 001, 111 and 100.) The contestant is allowed to look at the strings written by the deputy leader and guess the leader's string. What is the minimum number of guesses (in terms of $n$ and $k$ ) needed to guarantee the correct answer? Answer. The minimum number of guesses is 2 if $n=2 k$ and 1 if $n \\neq 2 k$.","solution":"Firstly, assume $n \\neq 2 k$. Without loss of generality suppose the first digit of the leader's string is 1 . Then among the $\\binom{n}{k}$ strings written by the deputy leader, $\\binom{n-1}{k}$ will begin with 1 and $\\binom{n-1}{k-1}$ will begin with 0 . Since $n \\neq 2 k$, we have $k+(k-1) \\neq n-1$ and so $\\binom{n-1}{k} \\neq\\binom{ n-1}{k-1}$. Thus, by counting the number of strings written by the deputy leader that start with 0 and 1 , the contestant can tell the first digit of the leader's string. The same can be done on the other digits, so 1 guess suffices when $n \\neq 2 k$. Secondly, for the case $n=2$ and $k=1$, the answer is clearly 2 . For the remaining cases where $n=2 k>2$, the deputy leader would write down the same strings if the leader's string $X$ is replaced by $X^{\\prime}$ obtained by changing each digit of $X$. This shows at least 2 guesses are needed. We shall show that 2 guesses suffice in this case. Suppose the first two digits of the leader's string are the same. Then among the strings written by the deputy leader, the prefices 01 and 10 will occur $\\binom{2 k-2}{k-1}$ times each, while the prefices 00 and 11 will occur $\\binom{2 k-2}{k}$ times each. The two numbers are interchanged if the first two digits of the leader's string are different. Since $\\binom{2 k-2}{k-1} \\neq\\binom{ 2 k-2}{k}$, the contestant can tell whether the first two digits of the leader's string are the same or not. He can work out the relation of the first digit and the other digits in the same way and reduce the leader's string to only 2 possibilities. The proof is complete.","tier":0} +{"problem_type":"Combinatorics","problem_label":"C2","problem":"Find all positive integers $n$ for which all positive divisors of $n$ can be put into the cells of a rectangular table under the following constraints: - each cell contains a distinct divisor; - the sums of all rows are equal; and - the sums of all columns are equal. Answer. 1.","solution":"Suppose all positive divisors of $n$ can be arranged into a rectangular table of size $k \\times l$ where the number of rows $k$ does not exceed the number of columns $l$. Let the sum of numbers in each column be $s$. Since $n$ belongs to one of the columns, we have $s \\geqslant n$, where equality holds only when $n=1$. For $j=1,2, \\ldots, l$, let $d_{j}$ be the largest number in the $j$-th column. Without loss of generality, assume $d_{1}>d_{2}>\\cdots>d_{l}$. Since these are divisors of $n$, we have $$ d_{l} \\leqslant \\frac{n}{l} $$ As $d_{l}$ is the maximum entry of the $l$-th column, we must have $$ d_{l} \\geqslant \\frac{s}{k} \\geqslant \\frac{n}{k} . $$ The relations (1) and (2) combine to give $\\frac{n}{l} \\geqslant \\frac{n}{k}$, that is, $k \\geqslant l$. Together with $k \\leqslant l$, we conclude that $k=l$. Then all inequalities in (1) and (2) are equalities. In particular, $s=n$ and so $n=1$, in which case the conditions are clearly satisfied.","tier":0} +{"problem_type":"Combinatorics","problem_label":"C2","problem":"Find all positive integers $n$ for which all positive divisors of $n$ can be put into the cells of a rectangular table under the following constraints: - each cell contains a distinct divisor; - the sums of all rows are equal; and - the sums of all columns are equal. Answer. 1.","solution":"Clearly $n=1$ works. Then we assume $n>1$ and let its prime factorization be $n=p_{1}^{r_{1}} p_{2}^{r_{2}} \\cdots p_{t}^{r_{t}}$. Suppose the table has $k$ rows and $l$ columns with $1n$. Therefore, we have $$ \\begin{aligned} \\left(r_{1}+1\\right)\\left(r_{2}+1\\right) \\cdots\\left(r_{t}+1\\right) & =k l \\leqslant l^{2}<\\left(\\frac{\\sigma(n)}{n}\\right)^{2} \\\\ & =\\left(1+\\frac{1}{p_{1}}+\\cdots+\\frac{1}{p_{1}^{r_{1}}}\\right)^{2} \\cdots\\left(1+\\frac{1}{p_{t}}+\\cdots+\\frac{1}{p_{t}^{r_{t}}}\\right)^{2} \\end{aligned} $$ This can be rewritten as $$ f\\left(p_{1}, r_{1}\\right) f\\left(p_{2}, r_{2}\\right) \\cdots f\\left(p_{t}, r_{t}\\right)<1 $$ where $$ f(p, r)=\\frac{r+1}{\\left(1+\\frac{1}{p}+\\cdots+\\frac{1}{p^{r}}\\right)^{2}}=\\frac{(r+1)\\left(1-\\frac{1}{p}\\right)^{2}}{\\left(1-\\frac{1}{p^{r+1}}\\right)^{2}} $$ Direct computation yields $$ f(2,1)=\\frac{8}{9}, \\quad f(2,2)=\\frac{48}{49}, \\quad f(3,1)=\\frac{9}{8} . $$ Also, we find that $$ \\begin{aligned} & f(2, r) \\geqslant\\left(1-\\frac{1}{2^{r+1}}\\right)^{-2}>1 \\quad \\text { for } r \\geqslant 3 \\\\ & f(3, r) \\geqslant \\frac{4}{3}\\left(1-\\frac{1}{3^{r+1}}\\right)^{-2}>\\frac{4}{3}>\\frac{9}{8} \\quad \\text { for } r \\geqslant 2, \\text { and } \\\\ & f(p, r) \\geqslant \\frac{32}{25}\\left(1-\\frac{1}{p^{r+1}}\\right)^{-2}>\\frac{32}{25}>\\frac{9}{8} \\quad \\text { for } p \\geqslant 5 \\end{aligned} $$ From these values and bounds, it is clear that (3) holds only when $n=2$ or 4 . In both cases, it is easy to see that the conditions are not satisfied. Hence, the only possible $n$ is 1 .","tier":0} +{"problem_type":"Combinatorics","problem_label":"C3","problem":"Let $n$ be a positive integer relatively prime to 6 . We paint the vertices of a regular $n$-gon with three colours so that there is an odd number of vertices of each colour. Show that there exists an isosceles triangle whose three vertices are of different colours.","solution":"For $k=1,2,3$, let $a_{k}$ be the number of isosceles triangles whose vertices contain exactly $k$ colours. Suppose on the contrary that $a_{3}=0$. Let $b, c, d$ be the number of vertices of the three different colours respectively. We now count the number of pairs $(\\triangle, E)$ where $\\triangle$ is an isosceles triangle and $E$ is a side of $\\triangle$ whose endpoints are of different colours. On the one hand, since we have assumed $a_{3}=0$, each triangle in the pair must contain exactly two colours, and hence each triangle contributes twice. Thus the number of pairs is $2 a_{2}$ \u3002 On the other hand, if we pick any two vertices $A, B$ of distinct colours, then there are three isosceles triangles having these as vertices, two when $A B$ is not the base and one when $A B$ is the base since $n$ is odd. Note that the three triangles are all distinct as $(n, 3)=1$. In this way, we count the number of pairs to be $3(b c+c d+d b)$. However, note that $2 a_{2}$ is even while $3(b c+c d+d b)$ is odd, as each of $b, c, d$ is. This yields a contradiction and hence $a_{3} \\geqslant 1$. Comment. A slightly stronger version of this problem is to replace the condition $(n, 6)=1$ by $n$ being odd (where equilateral triangles are regarded as isosceles triangles). In that case, the only difference in the proof is that by fixing any two vertices $A, B$, one can find exactly one or three isosceles triangles having these as vertices. But since only parity is concerned in the solution, the proof goes the same way. The condition that there is an odd number of vertices of each colour is necessary, as can be seen from the following example. Consider $n=25$ and we label the vertices $A_{0}, A_{1}, \\ldots, A_{24}$. Suppose colour 1 is used for $A_{0}$, colour 2 is used for $A_{5}, A_{10}, A_{15}, A_{20}$, while colour 3 is used for the remaining vertices. Then any isosceles triangle having colours 1 and 2 must contain $A_{0}$ and one of $A_{5}, A_{10}, A_{15}, A_{20}$. Clearly, the third vertex must have index which is a multiple of 5 so it is not of colour 3 .","tier":0} +{"problem_type":"Combinatorics","problem_label":"C4","problem":"Find all positive integers $n$ for which we can fill in the entries of an $n \\times n$ table with the following properties: - each entry can be one of $I, M$ and $O$; - in each row and each column, the letters $I, M$ and $O$ occur the same number of times; and - in any diagonal whose number of entries is a multiple of three, the letters $I, M$ and $O$ occur the same number of times. Answer. $n$ can be any multiple of 9 .","solution":"We first show that such a table exists when $n$ is a multiple of 9 . Consider the following $9 \\times 9$ table. $$ \\left(\\begin{array}{ccccccccc} I & I & I & M & M & M & O & O & O \\\\ M & M & M & O & O & O & I & I & I \\\\ O & O & O & I & I & I & M & M & M \\\\ I & I & I & M & M & M & O & O & O \\\\ M & M & M & O & O & O & I & I & I \\\\ O & O & O & I & I & I & M & M & M \\\\ I & I & I & M & M & M & O & O & O \\\\ M & M & M & O & O & O & I & I & I \\\\ O & O & O & I & I & I & M & M & M \\end{array}\\right) $$ It is a direct checking that the table (1) satisfies the requirements. For $n=9 k$ where $k$ is a positive integer, we form an $n \\times n$ table using $k \\times k$ copies of (1). For each row and each column of the table of size $n$, since there are three $I$ 's, three $M$ 's and three $O$ 's for any nine consecutive entries, the numbers of $I, M$ and $O$ are equal. In addition, every diagonal of the large table whose number of entries is divisible by 3 intersects each copy of (1) at a diagonal with number of entries divisible by 3 (possibly zero). Therefore, every such diagonal also contains the same number of $I, M$ and $O$. Next, consider any $n \\times n$ table for which the requirements can be met. As the number of entries of each row should be a multiple of 3 , we let $n=3 k$ where $k$ is a positive integer. We divide the whole table into $k \\times k$ copies of $3 \\times 3$ blocks. We call the entry at the centre of such a $3 \\times 3$ square a vital entry. We also call any row, column or diagonal that contains at least one vital entry a vital line. We compute the number of pairs $(l, c)$ where $l$ is a vital line and $c$ is an entry belonging to $l$ that contains the letter $M$. We let this number be $N$. On the one hand, since each vital line contains the same number of $I, M$ and $O$, it is obvious that each vital row and each vital column contain $k$ occurrences of $M$. For vital diagonals in either direction, we count there are exactly $$ 1+2+\\cdots+(k-1)+k+(k-1)+\\cdots+2+1=k^{2} $$ occurrences of $M$. Therefore, we have $N=4 k^{2}$. On the other hand, there are $3 k^{2}$ occurrences of $M$ in the whole table. Note that each entry belongs to exactly 1 or 4 vital lines. Therefore, $N$ must be congruent to $3 k^{2} \\bmod 3$. From the double counting, we get $4 k^{2} \\equiv 3 k^{2}(\\bmod 3)$, which forces $k$ to be a multiple of 3. Therefore, $n$ has to be a multiple of 9 and the proof is complete.","tier":0} +{"problem_type":"Combinatorics","problem_label":"C5","problem":"Let $n \\geqslant 3$ be a positive integer. Find the maximum number of diagonals of a regular $n$-gon one can select, so that any two of them do not intersect in the interior or they are perpendicular to each other. Answer. $n-2$ if $n$ is even and $n-3$ if $n$ is odd.","solution":"We consider two cases according to the parity of $n$. - Case 1. $n$ is odd. We first claim that no pair of diagonals is perpendicular. Suppose $A, B, C, D$ are vertices where $A B$ and $C D$ are perpendicular, and let $E$ be the vertex lying on the perpendicular bisector of $A B$. Let $E^{\\prime}$ be the opposite point of $E$ on the circumcircle of the regular polygon. Since $E C=E^{\\prime} D$ and $C, D, E$ are vertices of the regular polygon, $E^{\\prime}$ should also belong to the polygon. This contradicts the fact that a regular polygon with an odd number of vertices does not contain opposite points on the circumcircle. ![](https:\/\/cdn.mathpix.com\/cropped\/2024_11_18_e5e735288fb13d37ab0cg-39.jpg?height=513&width=1250&top_left_y=1061&top_left_x=493) Therefore in the odd case we can only select diagonals which do not intersect. In the maximal case these diagonals should divide the regular $n$-gon into $n-2$ triangles, so we can select at most $n-3$ diagonals. This can be done, for example, by selecting all diagonals emanated from a particular vertex. - Case 2. $n$ is even. If there is no intersection, then the proof in the odd case works. Suppose there are two perpendicular diagonals selected. We consider the set $S$ of all selected diagonals parallel to one of them which intersect with some selected diagonals. Suppose $S$ contains $k$ diagonals and the number of distinct endpoints of the $k$ diagonals is $l$. Firstly, consider the longest diagonal in one of the two directions in $S$. No other diagonal in $S$ can start from either endpoint of that diagonal, since otherwise it has to meet another longer diagonal in $S$. The same holds true for the other direction. Ignoring these two longest diagonals and their four endpoints, the remaining $k-2$ diagonals share $l-4$ endpoints where each endpoint can belong to at most two diagonals. This gives $2(l-4) \\geqslant 2(k-2)$, so that $k \\leqslant l-2$. ![](https:\/\/cdn.mathpix.com\/cropped\/2024_11_18_e5e735288fb13d37ab0cg-40.jpg?height=430&width=1442&top_left_y=289&top_left_x=294) Consider a group of consecutive vertices of the regular $n$-gon so that each of the two outermost vertices is an endpoint of a diagonal in $S$, while the interior points are not. There are $l$ such groups. We label these groups $P_{1}, P_{2}, \\ldots, P_{l}$ in this order. We claim that each selected diagonal outside $S$ must connect vertices of the same group $P_{i}$. Consider any diagonal $d$ joining vertices from distinct groups $P_{i}$ and $P_{j}$. Let $d_{1}$ and $d_{2}$ be two diagonals in $S$ each having one of the outermost points of $P_{i}$ as endpoint. Then $d$ must meet either $d_{1}, d_{2}$ or a diagonal in $S$ which is perpendicular to both $d_{1}$ and $d_{2}$. In any case $d$ should belong to $S$ by definition, which is a contradiction. Within the same group $P_{i}$, there are no perpendicular diagonals since the vertices belong to the same side of a diameter of the circumcircle. Hence there can be at most $\\left|P_{i}\\right|-2$ selected diagonals within $P_{i}$, including the one joining the two outermost points of $P_{i}$ when $\\left|P_{i}\\right|>2$. Therefore, the maximum number of diagonals selected is $$ \\sum_{i=1}^{l}\\left(\\left|P_{i}\\right|-2\\right)+k=\\sum_{i=1}^{l}\\left|P_{i}\\right|-2 l+k=(n+l)-2 l+k=n-l+k \\leqslant n-2 $$ This upper bound can be attained as follows. We take any vertex $A$ and let $A^{\\prime}$ be the vertex for which $A A^{\\prime}$ is a diameter of the circumcircle. If we select all diagonals emanated from $A$ together with the diagonal $d^{\\prime}$ joining the two neighbouring vertices of $A^{\\prime}$, then the only pair of diagonals that meet each other is $A A^{\\prime}$ and $d^{\\prime}$, which are perpendicular to each other. In total we can take $n-2$ diagonals. ![](https:\/\/cdn.mathpix.com\/cropped\/2024_11_18_e5e735288fb13d37ab0cg-40.jpg?height=459&width=421&top_left_y=1798&top_left_x=803)","tier":0} +{"problem_type":"Combinatorics","problem_label":"C5","problem":"Let $n \\geqslant 3$ be a positive integer. Find the maximum number of diagonals of a regular $n$-gon one can select, so that any two of them do not intersect in the interior or they are perpendicular to each other. Answer. $n-2$ if $n$ is even and $n-3$ if $n$ is odd.","solution":"The constructions and the odd case are the same as The base case $n=3$ is trivial since there is no diagonal at all. Suppose the upper bound holds for any cyclic polygon with fewer than $n$ sides. For a cyclic $n$-gon, if there is a selected diagonal which does not intersect any other selected diagonal, then this diagonal divides the $n$-gon into an $m$-gon and an $l$-gon (with $m+l=n+2$ ) so that each selected diagonal belongs to one of them. Without loss of generality, we may assume the $m$-gon lies on the same side of a diameter of $\\Gamma$. Then no two selected diagonals of the $m$-gon can intersect, and hence we can select at most $m-3$ diagonals. Also, we can apply the inductive hypothesis to the $l$-gon. This shows the maximum number of selected diagonals is $(m-3)+(l-2)+1=n-2$. It remains to consider the case when all selected diagonals meet at least one other selected diagonal. Consider a pair of selected perpendicular diagonals $d_{1}, d_{2}$. They divide the circumference of $\\Gamma$ into four arcs, each of which lies on the same side of a diameter of $\\Gamma$. If there are two selected diagonals intersecting each other and neither is parallel to $d_{1}$ or $d_{2}$, then their endpoints must belong to the same arc determined by $d_{1}, d_{2}$, and hence they cannot be perpendicular. This violates the condition, and hence all selected diagonals must have the same direction as one of $d_{1}, d_{2}$. ![](https:\/\/cdn.mathpix.com\/cropped\/2024_11_18_e5e735288fb13d37ab0cg-41.jpg?height=435&width=1234&top_left_y=1036&top_left_x=494) Take the longest selected diagonal in one of the two directions. We argue as in Solution 1 that its endpoints do not belong to any other selected diagonal. The same holds true for the longest diagonal in the other direction. Apart from these four endpoints, each of the remaining $n-4$ vertices can belong to at most two selected diagonals. Thus we can select at most $\\frac{1}{2}(2(n-4)+4)=n-2$ diagonals. Then the proof follows by induction.","tier":0} +{"problem_type":"Combinatorics","problem_label":"C8","problem":"Let $n$ be a positive integer. Determine the smallest positive integer $k$ with the following property: it is possible to mark $k$ cells on a $2 n \\times 2 n$ board so that there exists a unique partition of the board into $1 \\times 2$ and $2 \\times 1$ dominoes, none of which contains two marked cells. Answer. $2 n$.","solution":"We first construct an example of marking $2 n$ cells satisfying the requirement. Label the rows and columns $1,2, \\ldots, 2 n$ and label the cell in the $i$-th row and the $j$-th column $(i, j)$. For $i=1,2, \\ldots, n$, we mark the cells $(i, i)$ and $(i, i+1)$. We claim that the required partition exists and is unique. The two diagonals of the board divide the board into four regions. Note that the domino covering cell $(1,1)$ must be vertical. This in turn shows that each domino covering $(2,2),(3,3), \\ldots,(n, n)$ is vertical. By induction, the dominoes in the left region are all vertical. By rotational symmetry, the dominoes in the bottom region are horizontal, and so on. This shows that the partition exists and is unique. ![](https:\/\/cdn.mathpix.com\/cropped\/2024_11_18_e5e735288fb13d37ab0cg-45.jpg?height=323&width=820&top_left_y=1080&top_left_x=696) It remains to show that this value of $k$ is the smallest possible. Assume that only $k<2 n$ cells are marked, and there exists a partition $P$ satisfying the requirement. It suffices to show there exists another desirable partition distinct from $P$. Let $d$ be the main diagonal of the board. Construct the following graph with edges of two colours. Its vertices are the cells of the board. Connect two vertices with a red edge if they belong to the same domino of $P$. Connect two vertices with a blue edge if their reflections in $d$ are connected by a red edge. It is possible that two vertices are connected by edges of both colours. Clearly, each vertex has both red and blue degrees equal to 1 . Thus the graph splits into cycles where the colours of edges in each cycle alternate (a cycle may have length 2). Consider any cell $c$ lying on the diagonal $d$. Its two edges are symmetrical with respect to $d$. Thus they connect $c$ to different cells. This shows $c$ belongs to a cycle $C(c)$ of length at least 4. Consider a part of this cycle $c_{0}, c_{1}, \\ldots, c_{m}$ where $c_{0}=c$ and $m$ is the least positive integer such that $c_{m}$ lies on $d$. Clearly, $c_{m}$ is distinct from $c$. From the construction, the path symmetrical to this with respect to $d$ also lies in the graph, and so these paths together form $C(c)$. Hence, $C(c)$ contains exactly two cells from $d$. Then all $2 n$ cells in $d$ belong to $n$ cycles $C_{1}, C_{2}, \\ldots, C_{n}$, each has length at least 4. By the Pigeonhole Principle, there exists a cycle $C_{i}$ containing at most one of the $k$ marked cells. We modify $P$ as follows. We remove all dominoes containing the vertices of $C_{i}$, which correspond to the red edges of $C_{i}$. Then we put the dominoes corresponding to the blue edges of $C_{i}$. Since $C_{i}$ has at least 4 vertices, the resultant partition $P^{\\prime}$ is different from $P$. Clearly, no domino in $P^{\\prime}$ contains two marked cells as $C_{i}$ contains at most one marked cell. This shows the partition is not unique and hence $k$ cannot be less than $2 n$.","tier":0} +{"problem_type":"Number Theory","problem_label":"N1","problem":"For any positive integer $k$, denote the sum of digits of $k$ in its decimal representation by $S(k)$. Find all polynomials $P(x)$ with integer coefficients such that for any positive integer $n \\geqslant 2016$, the integer $P(n)$ is positive and $$ S(P(n))=P(S(n)) $$ Answer. - $P(x)=c$ where $1 \\leqslant c \\leqslant 9$ is an integer; or - $P(x)=x$.","solution":"We consider three cases according to the degree of $P$. - Case 1. $P(x)$ is a constant polynomial. Let $P(x)=c$ where $c$ is an integer constant. Then (1) becomes $S(c)=c$. This holds if and only if $1 \\leqslant c \\leqslant 9$. - Case 2. $\\operatorname{deg} P=1$. We have the following observation. For any positive integers $m, n$, we have $$ S(m+n) \\leqslant S(m)+S(n) $$ and equality holds if and only if there is no carry in the addition $m+n$. Let $P(x)=a x+b$ for some integers $a, b$ where $a \\neq 0$. As $P(n)$ is positive for large $n$, we must have $a \\geqslant 1$. The condition (1) becomes $S(a n+b)=a S(n)+b$ for all $n \\geqslant 2016$. Setting $n=2025$ and $n=2020$ respectively, we get $$ S(2025 a+b)-S(2020 a+b)=(a S(2025)+b)-(a S(2020)+b)=5 a $$ On the other hand, (2) implies $$ S(2025 a+b)=S((2020 a+b)+5 a) \\leqslant S(2020 a+b)+S(5 a) $$ These give $5 a \\leqslant S(5 a)$. As $a \\geqslant 1$, this holds only when $a=1$, in which case (1) reduces to $S(n+b)=S(n)+b$ for all $n \\geqslant 2016$. Then we find that $$ S(n+1+b)-S(n+b)=(S(n+1)+b)-(S(n)+b)=S(n+1)-S(n) $$ If $b>0$, we choose $n$ such that $n+1+b=10^{k}$ for some sufficiently large $k$. Note that all the digits of $n+b$ are 9 's, so that the left-hand side of (3) equals $1-9 k$. As $n$ is a positive integer less than $10^{k}-1$, we have $S(n)<9 k$. Therefore, the right-hand side of (3) is at least $1-(9 k-1)=2-9 k$, which is a contradiction. The case $b<0$ can be handled similarly by considering $n+1$ to be a large power of 10 . Therefore, we conclude that $P(x)=x$, in which case (1) is trivially satisfied. - Case 3. $\\operatorname{deg} P \\geqslant 2$. Suppose the leading term of $P$ is $a_{d} n^{d}$ where $a_{d} \\neq 0$. Clearly, we have $a_{d}>0$. Consider $n=10^{k}-1$ in (1). We get $S(P(n))=P(9 k)$. Note that $P(n)$ grows asymptotically as fast as $n^{d}$, so $S(P(n))$ grows asymptotically as no faster than a constant multiple of $k$. On the other hand, $P(9 k)$ grows asymptotically as fast as $k^{d}$. This shows the two sides of the last equation cannot be equal for sufficiently large $k$ since $d \\geqslant 2$. Therefore, we conclude that $P(x)=c$ where $1 \\leqslant c \\leqslant 9$ is an integer, or $P(x)=x$.","tier":0} +{"problem_type":"Number Theory","problem_label":"N1","problem":"For any positive integer $k$, denote the sum of digits of $k$ in its decimal representation by $S(k)$. Find all polynomials $P(x)$ with integer coefficients such that for any positive integer $n \\geqslant 2016$, the integer $P(n)$ is positive and $$ S(P(n))=P(S(n)) $$ Answer. - $P(x)=c$ where $1 \\leqslant c \\leqslant 9$ is an integer; or - $P(x)=x$.","solution":"Let $P(x)=a_{d} x^{d}+a_{d-1} x^{d-1}+\\cdots+a_{0}$. Clearly $a_{d}>0$. There exists an integer $m \\geqslant 1$ such that $\\left|a_{i}\\right|<10^{m}$ for all $0 \\leqslant i \\leqslant d$. Consider $n=9 \\times 10^{k}$ for a sufficiently large integer $k$ in (1). If there exists an index $0 \\leqslant i \\leqslant d-1$ such that $a_{i}<0$, then all digits of $P(n)$ in positions from $10^{i k+m+1}$ to $10^{(i+1) k-1}$ are all 9 's. Hence, we have $S(P(n)) \\geqslant 9(k-m-1)$. On the other hand, $P(S(n))=P(9)$ is a fixed constant. Therefore, (1) cannot hold for large $k$. This shows $a_{i} \\geqslant 0$ for all $0 \\leqslant i \\leqslant d-1$. Hence, $P(n)$ is an integer formed by the nonnegative integers $a_{d} \\times 9^{d}, a_{d-1} \\times 9^{d-1}, \\ldots, a_{0}$ by inserting some zeros in between. This yields $$ S(P(n))=S\\left(a_{d} \\times 9^{d}\\right)+S\\left(a_{d-1} \\times 9^{d-1}\\right)+\\cdots+S\\left(a_{0}\\right) . $$ Combining with (1), we have $$ S\\left(a_{d} \\times 9^{d}\\right)+S\\left(a_{d-1} \\times 9^{d-1}\\right)+\\cdots+S\\left(a_{0}\\right)=P(9)=a_{d} \\times 9^{d}+a_{d-1} \\times 9^{d-1}+\\cdots+a_{0} $$ As $S(m) \\leqslant m$ for any positive integer $m$, with equality when $1 \\leqslant m \\leqslant 9$, this forces each $a_{i} \\times 9^{i}$ to be a positive integer between 1 and 9 . In particular, this shows $a_{i}=0$ for $i \\geqslant 2$ and hence $d \\leqslant 1$. Also, we have $a_{1} \\leqslant 1$ and $a_{0} \\leqslant 9$. If $a_{1}=1$ and $1 \\leqslant a_{0} \\leqslant 9$, we take $n=10^{k}+\\left(10-a_{0}\\right)$ for sufficiently large $k$ in (1). This yields a contradiction since $$ S(P(n))=S\\left(10^{k}+10\\right)=2 \\neq 11=P\\left(11-a_{0}\\right)=P(S(n)) $$ The zero polynomial is also rejected since $P(n)$ is positive for large $n$. The remaining candidates are $P(x)=x$ or $P(x)=a_{0}$ where $1 \\leqslant a_{0} \\leqslant 9$, all of which satisfy (1), and hence are the only solutions.","tier":0} +{"problem_type":"Number Theory","problem_label":"N2","problem":"Let $\\tau(n)$ be the number of positive divisors of $n$. Let $\\tau_{1}(n)$ be the number of positive divisors of $n$ which have remainders 1 when divided by 3 . Find all possible integral values of the fraction $\\frac{\\tau(10 n)}{\\tau_{1}(10 n)}$. Answer. All composite numbers together with 2.","solution":"In this solution, we always use $p_{i}$ to denote primes congruent to $1 \\bmod 3$, and use $q_{j}$ to denote primes congruent to $2 \\bmod 3$. When we express a positive integer $m$ using its prime factorization, we also include the special case $m=1$ by allowing the exponents to be zeros. We first compute $\\tau_{1}(m)$ for a positive integer $m$. - Claim. Let $m=3^{x} p_{1}^{a_{1}} p_{2}^{a_{2}} \\cdots p_{s}^{a_{s}} q_{1}^{b_{1}} q_{2}^{b_{2}} \\cdots q_{t}^{b_{t}}$ be the prime factorization of $m$. Then $$ \\tau_{1}(m)=\\prod_{i=1}^{s}\\left(a_{i}+1\\right)\\left\\lceil\\frac{1}{2} \\prod_{j=1}^{t}\\left(b_{j}+1\\right)\\right\\rceil . $$ Proof. To choose a divisor of $m$ congruent to $1 \\bmod 3$, it cannot have the prime divisor 3, while there is no restriction on choosing prime factors congruent to $1 \\bmod 3$. Also, we have to choose an even number of prime factors (counted with multiplicity) congruent to $2 \\bmod 3$. If $\\prod_{j=1}^{t}\\left(b_{j}+1\\right)$ is even, then we may assume without loss of generality $b_{1}+1$ is even. We can choose the prime factors $q_{2}, q_{3}, \\ldots, q_{t}$ freely in $\\prod_{j=2}^{t}\\left(b_{j}+1\\right)$ ways. Then the parity of the number of $q_{1}$ is uniquely determined, and hence there are $\\frac{1}{2}\\left(b_{1}+1\\right)$ ways to choose the exponent of $q_{1}$. Hence (1) is verified in this case. If $\\prod_{j=1}^{t}\\left(b_{j}+1\\right)$ is odd, we use induction on $t$ to count the number of choices. When $t=1$, there are $\\left\\lceil\\frac{b_{1}+1}{2}\\right\\rceil$ choices for which the exponent is even and $\\left\\lfloor\\frac{b_{1}+1}{2}\\right\\rfloor$ choices for which the exponent is odd. For the inductive step, we find that there are $$ \\left\\lceil\\frac{1}{2} \\prod_{j=1}^{t-1}\\left(b_{j}+1\\right)\\right\\rceil \\cdot\\left\\lceil\\frac{b_{t}+1}{2}\\right\\rceil+\\left\\lfloor\\frac{1}{2} \\prod_{j=1}^{t-1}\\left(b_{j}+1\\right)\\right\\rfloor \\cdot\\left\\lfloor\\frac{b_{t}+1}{2}\\right\\rfloor=\\left\\lceil\\frac{1}{2} \\prod_{j=1}^{t}\\left(b_{j}+1\\right)\\right\\rceil $$ choices with an even number of prime factors and hence $\\left\\lfloor\\frac{1}{2} \\prod_{j=1}^{t}\\left(b_{j}+1\\right)\\right\\rfloor$ choices with an odd number of prime factors. Hence (1) is also true in this case. Let $n=3^{x} 2^{y} 5^{z} p_{1}^{a_{1}} p_{2}^{a_{2}} \\cdots p_{s}^{a_{s}} q_{1}^{b_{1}} q_{2}^{b_{2}} \\cdots q_{t}^{b_{t}}$. Using the well-known formula for computing the divisor function, we get $$ \\tau(10 n)=(x+1)(y+2)(z+2) \\prod_{i=1}^{s}\\left(a_{i}+1\\right) \\prod_{j=1}^{t}\\left(b_{j}+1\\right) $$ By the Claim, we have $$ \\tau_{1}(10 n)=\\prod_{i=1}^{s}\\left(a_{i}+1\\right)\\left\\lceil\\frac{1}{2}(y+2)(z+2) \\prod_{j=1}^{t}\\left(b_{j}+1\\right)\\right\\rceil $$ If $c=(y+2)(z+2) \\prod_{j=1}^{t}\\left(b_{j}+1\\right)$ is even, then (2) and (3) imply $$ \\frac{\\tau(10 n)}{\\tau_{1}(10 n)}=2(x+1) $$ In this case $\\frac{\\tau(10 n)}{\\tau_{1}(10 n)}$ can be any even positive integer as $x$ runs through all nonnegative integers. If $c$ is odd, which means $y, z$ are odd and each $b_{j}$ is even, then (2) and (3) imply $$ \\frac{\\tau(10 n)}{\\tau_{1}(10 n)}=\\frac{2(x+1) c}{c+1} $$ For this to be an integer, we need $c+1$ divides $2(x+1)$ since $c$ and $c+1$ are relatively prime. Let $2(x+1)=k(c+1)$. Then (4) reduces to $$ \\frac{\\tau(10 n)}{\\tau_{1}(10 n)}=k c=k(y+2)(z+2) \\prod_{j=1}^{t}\\left(b_{j}+1\\right) $$ Noting that $y, z$ are odd, the integers $y+2$ and $z+2$ are at least 3 . This shows the integer in this case must be composite. On the other hand, for any odd composite number $a b$ with $a, b \\geqslant 3$, we may simply take $n=3^{\\frac{a b-1}{2}} \\cdot 2^{a-2} \\cdot 5^{b-2}$ so that $\\frac{\\tau(10 n)}{\\tau_{1}(10 n)}=a b$ from (5). We conclude that the fraction can be any even integer or any odd composite number. Equivalently, it can be 2 or any composite number.","tier":0} +{"problem_type":"Number Theory","problem_label":"N3","problem":"Define $P(n)=n^{2}+n+1$. For any positive integers $a$ and $b$, the set $$ \\{P(a), P(a+1), P(a+2), \\ldots, P(a+b)\\} $$ is said to be fragrant if none of its elements is relatively prime to the product of the other elements. Determine the smallest size of a fragrant set. Answer. 6.","solution":"We have the following observations. (i) $(P(n), P(n+1))=1$ for any $n$. We have $(P(n), P(n+1))=\\left(n^{2}+n+1, n^{2}+3 n+3\\right)=\\left(n^{2}+n+1,2 n+2\\right)$. Noting that $n^{2}+n+1$ is odd and $\\left(n^{2}+n+1, n+1\\right)=(1, n+1)=1$, the claim follows. (ii) $(P(n), P(n+2))=1$ for $n \\not \\equiv 2(\\bmod 7)$ and $(P(n), P(n+2))=7$ for $n \\equiv 2(\\bmod 7)$. From $(2 n+7) P(n)-(2 n-1) P(n+2)=14$ and the fact that $P(n)$ is odd, $(P(n), P(n+2))$ must be a divisor of 7 . The claim follows by checking $n \\equiv 0,1, \\ldots, 6(\\bmod 7)$ directly. (iii) $(P(n), P(n+3))=1$ for $n \\not \\equiv 1(\\bmod 3)$ and $3 \\mid(P(n), P(n+3))$ for $n \\equiv 1(\\bmod 3)$. From $(n+5) P(n)-(n-1) P(n+3)=18$ and the fact that $P(n)$ is odd, $(P(n), P(n+3))$ must be a divisor of 9 . The claim follows by checking $n \\equiv 0,1,2(\\bmod 3)$ directly. Suppose there exists a fragrant set with at most 5 elements. We may assume it contains exactly 5 elements $P(a), P(a+1), \\ldots, P(a+4)$ since the following argument also works with fewer elements. Consider $P(a+2)$. From (i), it is relatively prime to $P(a+1)$ and $P(a+3)$. Without loss of generality, assume $(P(a), P(a+2))>1$. From (ii), we have $a \\equiv 2(\\bmod 7)$. The same observation implies $(P(a+1), P(a+3))=1$. In order that the set is fragrant, $(P(a), P(a+3))$ and $(P(a+1), P(a+4))$ must both be greater than 1. From (iii), this holds only when both $a$ and $a+1$ are congruent to $1 \\bmod 3$, which is a contradiction. It now suffices to construct a fragrant set of size 6. By the Chinese Remainder Theorem, we can take a positive integer $a$ such that $$ a \\equiv 7 \\quad(\\bmod 19), \\quad a+1 \\equiv 2 \\quad(\\bmod 7), \\quad a+2 \\equiv 1 \\quad(\\bmod 3) $$ For example, we may take $a=197$. From (ii), both $P(a+1)$ and $P(a+3)$ are divisible by 7. From (iii), both $P(a+2)$ and $P(a+5)$ are divisible by 3 . One also checks from $19 \\mid P(7)=57$ and $19 \\mid P(11)=133$ that $P(a)$ and $P(a+4)$ are divisible by 19. Therefore, the set $\\{P(a), P(a+1), \\ldots, P(a+5)\\}$ is fragrant. Therefore, the smallest size of a fragrant set is 6 . Comment. \"Fragrant Harbour\" is the English translation of \"Hong Kong\". A stronger version of this problem is to show that there exists a fragrant set of size $k$ for any $k \\geqslant 6$. We present a proof here. For each even positive integer $m$ which is not divisible by 3 , since $m^{2}+3 \\equiv 3(\\bmod 4)$, we can find a prime $p_{m} \\equiv 3(\\bmod 4)$ such that $p_{m} \\mid m^{2}+3$. Clearly, $p_{m}>3$. If $b=2 t \\geqslant 6$, we choose $a$ such that $3 \\mid 2(a+t)+1$ and $p_{m} \\mid 2(a+t)+1$ for each $1 \\leqslant m \\leqslant b$ with $m \\equiv 2,4(\\bmod 6)$. For $0 \\leqslant r \\leqslant t$ and $3 \\mid r$, we have $a+t \\pm r \\equiv 1(\\bmod 3)$ so that $3 \\mid P(a+t \\pm r)$. For $0 \\leqslant r \\leqslant t$ and $(r, 3)=1$, we have $$ 4 P(a+t \\pm r) \\equiv(-1 \\pm 2 r)^{2}+2(-1 \\pm 2 r)+4=4 r^{2}+3 \\equiv 0 \\quad\\left(\\bmod p_{2 r}\\right) . $$ Hence, $\\{P(a), P(a+1), \\ldots, P(a+b)\\}$ is fragrant. If $b=2 t+1 \\geqslant 7$ (the case $b=5$ has been done in the original problem), we choose $a$ such that $3 \\mid 2(a+t)+1$ and $p_{m} \\mid 2(a+t)+1$ for $1 \\leqslant m \\leqslant b$ with $m \\equiv 2,4(\\bmod 6)$, and that $a+b \\equiv 9(\\bmod 13)$. Note that $a$ exists by the Chinese Remainder Theorem since $p_{m} \\neq 13$ for all $m$. The even case shows that $\\{P(a), P(a+1), \\ldots, P(a+b-1)\\}$ is fragrant. Also, one checks from $13 \\mid P(9)=91$ and $13 \\mid P(3)=13$ that $P(a+b)$ and $P(a+b-6)$ are divisible by 13. The proof is thus complete.","tier":0} +{"problem_type":"Number Theory","problem_label":"N6","problem":"Denote by $\\mathbb{N}$ the set of all positive integers. Find all functions $f: \\mathbb{N} \\rightarrow \\mathbb{N}$ such that for all positive integers $m$ and $n$, the integer $f(m)+f(n)-m n$ is nonzero and divides $m f(m)+n f(n)$. Answer. $f(n)=n^{2}$ for any $n \\in \\mathbb{N}$.","solution":"It is given that $$ f(m)+f(n)-m n \\mid m f(m)+n f(n) $$ Taking $m=n=1$ in (1), we have $2 f(1)-1 \\mid 2 f(1)$. Then $2 f(1)-1 \\mid 2 f(1)-(2 f(1)-1)=1$ and hence $f(1)=1$. Let $p \\geqslant 7$ be a prime. Taking $m=p$ and $n=1$ in (1), we have $f(p)-p+1 \\mid p f(p)+1$ and hence $$ f(p)-p+1 \\mid p f(p)+1-p(f(p)-p+1)=p^{2}-p+1 $$ If $f(p)-p+1=p^{2}-p+1$, then $f(p)=p^{2}$. If $f(p)-p+1 \\neq p^{2}-p+1$, as $p^{2}-p+1$ is an odd positive integer, we have $p^{2}-p+1 \\geqslant 3(f(p)-p+1)$, that is, $$ f(p) \\leqslant \\frac{1}{3}\\left(p^{2}+2 p-2\\right) $$ Taking $m=n=p$ in (1), we have $2 f(p)-p^{2} \\mid 2 p f(p)$. This implies $$ 2 f(p)-p^{2} \\mid 2 p f(p)-p\\left(2 f(p)-p^{2}\\right)=p^{3} $$ By (2) and $f(p) \\geqslant 1$, we get $$ -p^{2}<2 f(p)-p^{2} \\leqslant \\frac{2}{3}\\left(p^{2}+2 p-2\\right)-p^{2}<-p $$ since $p \\geqslant 7$. This contradicts the fact that $2 f(p)-p^{2}$ is a factor of $p^{3}$. Thus we have proved that $f(p)=p^{2}$ for all primes $p \\geqslant 7$. Let $n$ be a fixed positive integer. Choose a sufficiently large prime $p$. Consider $m=p$ in (1). We obtain $$ f(p)+f(n)-p n \\mid p f(p)+n f(n)-n(f(p)+f(n)-p n)=p f(p)-n f(p)+p n^{2} $$ As $f(p)=p^{2}$, this implies $p^{2}-p n+f(n) \\mid p\\left(p^{2}-p n+n^{2}\\right)$. As $p$ is sufficiently large and $n$ is fixed, $p$ cannot divide $f(n)$, and so $\\left(p, p^{2}-p n+f(n)\\right)=1$. It follows that $p^{2}-p n+f(n) \\mid p^{2}-p n+n^{2}$ and hence $$ p^{2}-p n+f(n) \\mid\\left(p^{2}-p n+n^{2}\\right)-\\left(p^{2}-p n+f(n)\\right)=n^{2}-f(n) $$ Note that $n^{2}-f(n)$ is fixed while $p^{2}-p n+f(n)$ is chosen to be sufficiently large. Therefore, we must have $n^{2}-f(n)=0$ so that $f(n)=n^{2}$ for any positive integer $n$. Finally, we check that when $f(n)=n^{2}$ for any positive integer $n$, then $$ f(m)+f(n)-m n=m^{2}+n^{2}-m n $$ and $$ m f(m)+n f(n)=m^{3}+n^{3}=(m+n)\\left(m^{2}+n^{2}-m n\\right) $$ The latter expression is divisible by the former for any positive integers $m, n$. This shows $f(n)=n^{2}$ is the only solution.","tier":0} +{"problem_type":"Number Theory","problem_label":"N7","problem":"Let $n$ be an odd positive integer. In the Cartesian plane, a cyclic polygon $P$ with area $S$ is chosen. All its vertices have integral coordinates, and the squares of its side lengths are all divisible by $n$. Prove that $2 S$ is an integer divisible by $n$.","solution":"Let $P=A_{1} A_{2} \\ldots A_{k}$ and let $A_{k+i}=A_{i}$ for $i \\geqslant 1$. By the Shoelace Formula, the area of any convex polygon with integral coordinates is half an integer. Therefore, $2 S$ is an integer. We shall prove by induction on $k \\geqslant 3$ that $2 S$ is divisible by $n$. Clearly, it suffices to consider $n=p^{t}$ where $p$ is an odd prime and $t \\geqslant 1$. For the base case $k=3$, let the side lengths of $P$ be $\\sqrt{n a}, \\sqrt{n b}, \\sqrt{n c}$ where $a, b, c$ are positive integers. By Heron's Formula, $$ 16 S^{2}=n^{2}\\left(2 a b+2 b c+2 c a-a^{2}-b^{2}-c^{2}\\right) $$ This shows $16 S^{2}$ is divisible by $n^{2}$. Since $n$ is odd, $2 S$ is divisible by $n$. Assume $k \\geqslant 4$. If the square of length of one of the diagonals is divisible by $n$, then that diagonal divides $P$ into two smaller polygons, to which the induction hypothesis applies. Hence we may assume that none of the squares of diagonal lengths is divisible by $n$. As usual, we denote by $\\nu_{p}(r)$ the exponent of $p$ in the prime decomposition of $r$. We claim the following. - Claim. $\\nu_{p}\\left(A_{1} A_{m}^{2}\\right)>\\nu_{p}\\left(A_{1} A_{m+1}^{2}\\right)$ for $2 \\leqslant m \\leqslant k-1$. Proof. The case $m=2$ is obvious since $\\nu_{p}\\left(A_{1} A_{2}^{2}\\right) \\geqslant p^{t}>\\nu_{p}\\left(A_{1} A_{3}^{2}\\right)$ by the condition and the above assumption. Suppose $\\nu_{p}\\left(A_{1} A_{2}^{2}\\right)>\\nu_{p}\\left(A_{1} A_{3}^{2}\\right)>\\cdots>\\nu_{p}\\left(A_{1} A_{m}^{2}\\right)$ where $3 \\leqslant m \\leqslant k-1$. For the induction step, we apply Ptolemy's Theorem to the cyclic quadrilateral $A_{1} A_{m-1} A_{m} A_{m+1}$ to get $$ A_{1} A_{m+1} \\times A_{m-1} A_{m}+A_{1} A_{m-1} \\times A_{m} A_{m+1}=A_{1} A_{m} \\times A_{m-1} A_{m+1} $$ which can be rewritten as $$ \\begin{aligned} A_{1} A_{m+1}^{2} \\times A_{m-1} A_{m}^{2}= & A_{1} A_{m-1}^{2} \\times A_{m} A_{m+1}^{2}+A_{1} A_{m}^{2} \\times A_{m-1} A_{m+1}^{2} \\\\ & -2 A_{1} A_{m-1} \\times A_{m} A_{m+1} \\times A_{1} A_{m} \\times A_{m-1} A_{m+1} \\end{aligned} $$ From this, $2 A_{1} A_{m-1} \\times A_{m} A_{m+1} \\times A_{1} A_{m} \\times A_{m-1} A_{m+1}$ is an integer. We consider the component of $p$ of each term in (1). By the inductive hypothesis, we have $\\nu_{p}\\left(A_{1} A_{m-1}^{2}\\right)>\\nu_{p}\\left(A_{1} A_{m}^{2}\\right)$. Also, we have $\\nu_{p}\\left(A_{m} A_{m+1}^{2}\\right) \\geqslant p^{t}>\\nu_{p}\\left(A_{m-1} A_{m+1}^{2}\\right)$. These give $$ \\nu_{p}\\left(A_{1} A_{m-1}^{2} \\times A_{m} A_{m+1}^{2}\\right)>\\nu_{p}\\left(A_{1} A_{m}^{2} \\times A_{m-1} A_{m+1}^{2}\\right) $$ Next, we have $\\nu_{p}\\left(4 A_{1} A_{m-1}^{2} \\times A_{m} A_{m+1}^{2} \\times A_{1} A_{m}^{2} \\times A_{m-1} A_{m+1}^{2}\\right)=\\nu_{p}\\left(A_{1} A_{m-1}^{2} \\times A_{m} A_{m+1}^{2}\\right)+$ $\\nu_{p}\\left(A_{1} A_{m}^{2} \\times A_{m-1} A_{m+1}^{2}\\right)>2 \\nu_{p}\\left(A_{1} A_{m}^{2} \\times A_{m-1} A_{m+1}^{2}\\right)$ from (2). This implies $$ \\nu_{p}\\left(2 A_{1} A_{m-1} \\times A_{m} A_{m+1} \\times A_{1} A_{m} \\times A_{m-1} A_{m+1}\\right)>\\nu_{p}\\left(A_{1} A_{m}^{2} \\times A_{m-1} A_{m+1}^{2}\\right) $$ Combining (1), (2) and (3), we conclude that $$ \\nu_{p}\\left(A_{1} A_{m+1}^{2} \\times A_{m-1} A_{m}^{2}\\right)=\\nu_{p}\\left(A_{1} A_{m}^{2} \\times A_{m-1} A_{m+1}^{2}\\right) $$ By $\\nu_{p}\\left(A_{m-1} A_{m}^{2}\\right) \\geqslant p^{t}>\\nu_{p}\\left(A_{m-1} A_{m+1}^{2}\\right)$, we get $\\nu_{p}\\left(A_{1} A_{m+1}^{2}\\right)<\\nu_{p}\\left(A_{1} A_{m}^{2}\\right)$. The Claim follows by induction. From the Claim, we get a chain of inequalities $$ p^{t}>\\nu_{p}\\left(A_{1} A_{3}^{2}\\right)>\\nu_{p}\\left(A_{1} A_{4}^{2}\\right)>\\cdots>\\nu_{p}\\left(A_{1} A_{k}^{2}\\right) \\geqslant p^{t} $$ which yields a contradiction. Therefore, we can show by induction that $2 S$ is divisible by $n$. Comment. The condition that $P$ is cyclic is crucial. As a counterexample, consider the rhombus with vertices $(0,3),(4,0),(0,-3),(-4,0)$. Each of its squares of side lengths is divisible by 5 , while $2 S=48$ is not. The proposer also gives a proof for the case $n$ is even. One just needs an extra technical step for the case $p=2$.","tier":0} +{"problem_type":"Number Theory","problem_label":"N8","problem":"Find all polynomials $P(x)$ of odd degree $d$ and with integer coefficients satisfying the following property: for each positive integer $n$, there exist $n$ positive integers $x_{1}, x_{2}, \\ldots, x_{n}$ such that $\\frac{1}{2}<\\frac{P\\left(x_{i}\\right)}{P\\left(x_{j}\\right)}<2$ and $\\frac{P\\left(x_{i}\\right)}{P\\left(x_{j}\\right)}$ is the $d$-th power of a rational number for every pair of indices $i$ and $j$ with $1 \\leqslant i, j \\leqslant n$. Answer. $P(x)=a(r x+s)^{d}$ where $a, r, s$ are integers with $a \\neq 0, r \\geqslant 1$ and $(r, s)=1$.","solution":"Let $P(x)=a_{d} x^{d}+a_{d-1} x^{d-1}+\\cdots+a_{0}$. Consider the substitution $y=d a_{d} x+a_{d-1}$. By defining $Q(y)=P(x)$, we find that $Q$ is a polynomial with rational coefficients without the term $y^{d-1}$. Let $Q(y)=b_{d} y^{d}+b_{d-2} y^{d-2}+b_{d-3} y^{d-3}+\\cdots+b_{0}$ and $B=\\max _{0 \\leqslant i \\leqslant d}\\left\\{\\left|b_{i}\\right|\\right\\}$ (where $b_{d-1}=0$ ). The condition shows that for each $n \\geqslant 1$, there exist integers $y_{1}, y_{2}, \\ldots, y_{n}$ such that $\\frac{1}{2}<\\frac{Q\\left(y_{i}\\right)}{Q\\left(y_{j}\\right)}<2$ and $\\frac{Q\\left(y_{i}\\right)}{Q\\left(y_{j}\\right)}$ is the $d$-th power of a rational number for $1 \\leqslant i, j \\leqslant n$. Since $n$ can be arbitrarily large, we may assume all $x_{i}$ 's and hence $y_{i}$ 's are integers larger than some absolute constant in the following. By Dirichlet's Theorem, since $d$ is odd, we can find a sufficiently large prime $p$ such that $p \\equiv 2(\\bmod d)$. In particular, we have $(p-1, d)=1$. For this fixed $p$, we choose $n$ to be sufficiently large. Then by the Pigeonhole Principle, there must be $d+1$ of $y_{1}, y_{2}, \\ldots, y_{n}$ which are congruent $\\bmod p$. Without loss of generality, assume $y_{i} \\equiv y_{j}(\\bmod p)$ for $1 \\leqslant i, j \\leqslant d+1$. We shall establish the following. - Claim. $\\frac{Q\\left(y_{i}\\right)}{Q\\left(y_{1}\\right)}=\\frac{y_{i}^{d}}{y_{1}^{d}}$ for $2 \\leqslant i \\leqslant d+1$. Proof. Let $\\frac{Q\\left(y_{i}\\right)}{Q\\left(y_{1}\\right)}=\\frac{l^{d}}{m^{d}}$ where $(l, m)=1$ and $l, m>0$. This can be rewritten in the expanded form $$ b_{d}\\left(m^{d} y_{i}^{d}-l^{d} y_{1}^{d}\\right)=-\\sum_{j=0}^{d-2} b_{j}\\left(m^{d} y_{i}^{j}-l^{d} y_{1}^{j}\\right) $$ Let $c$ be the common denominator of $Q$, so that $c Q(k)$ is an integer for any integer $k$. Note that $c$ depends only on $P$ and so we may assume $(p, c)=1$. Then $y_{1} \\equiv y_{i}(\\bmod p)$ implies $c Q\\left(y_{1}\\right) \\equiv c Q\\left(y_{i}\\right)(\\bmod p)$. - Case 1. $p \\mid c Q\\left(y_{1}\\right)$. In this case, there is a cancellation of $p$ in the numerator and denominator of $\\frac{c Q\\left(y_{i}\\right)}{c Q\\left(y_{1}\\right)}$, so that $m^{d} \\leqslant p^{-1}\\left|c Q\\left(y_{1}\\right)\\right|$. Noting $\\left|Q\\left(y_{1}\\right)\\right|<2 B y_{1}^{d}$ as $y_{1}$ is large, we get $$ m \\leqslant p^{-\\frac{1}{d}}(2 c B)^{\\frac{1}{d}} y_{1} $$ For large $y_{1}$ and $y_{i}$, the relation $\\frac{1}{2}<\\frac{Q\\left(y_{i}\\right)}{Q\\left(y_{1}\\right)}<2$ implies $$ \\frac{1}{3}<\\frac{y_{i}^{d}}{y_{1}^{d}}<3 $$ We also have $$ \\frac{1}{2}<\\frac{l^{d}}{m^{d}}<2 $$ Now, the left-hand side of (1) is $$ b_{d}\\left(m y_{i}-l y_{1}\\right)\\left(m^{d-1} y_{i}^{d-1}+m^{d-2} y_{i}^{d-2} l y_{1}+\\cdots+l^{d-1} y_{1}^{d-1}\\right) . $$ Suppose on the contrary that $m y_{i}-l y_{1} \\neq 0$. Then the absolute value of the above expression is at least $\\left|b_{d}\\right| m^{d-1} y_{i}^{d-1}$. On the other hand, the absolute value of the right-hand side of (1) is at most $$ \\begin{aligned} \\sum_{j=0}^{d-2} B\\left(m^{d} y_{i}^{j}+l^{d} y_{1}^{j}\\right) & \\leqslant(d-1) B\\left(m^{d} y_{i}^{d-2}+l^{d} y_{1}^{d-2}\\right) \\\\ & \\leqslant(d-1) B\\left(7 m^{d} y_{i}^{d-2}\\right) \\\\ & \\leqslant 7(d-1) B\\left(p^{-\\frac{1}{d}}(2 c B)^{\\frac{1}{d}} y_{1}\\right) m^{d-1} y_{i}^{d-2} \\\\ & \\leqslant 21(d-1) B p^{-\\frac{1}{d}}(2 c B)^{\\frac{1}{d}} m^{d-1} y_{i}^{d-1} \\end{aligned} $$ by using successively (3), (4), (2) and again (3). This shows $$ \\left|b_{d}\\right| m^{d-1} y_{i}^{d-1} \\leqslant 21(d-1) B p^{-\\frac{1}{d}}(2 c B)^{\\frac{1}{d}} m^{d-1} y_{i}^{d-1}, $$ which is a contradiction for large $p$ as $b_{d}, B, c, d$ depend only on the polynomial $P$. Therefore, we have $m y_{i}-l y_{1}=0$ in this case. - Case 2. $\\left(p, c Q\\left(y_{1}\\right)\\right)=1$. From $c Q\\left(y_{1}\\right) \\equiv c Q\\left(y_{i}\\right)(\\bmod p)$, we have $l^{d} \\equiv m^{d}(\\bmod p)$. Since $(p-1, d)=1$, we use Fermat Little Theorem to conclude $l \\equiv m(\\bmod p)$. Then $p \\mid m y_{i}-l y_{1}$. Suppose on the contrary that $m y_{i}-l y_{1} \\neq 0$. Then the left-hand side of (1) has absolute value at least $\\left|b_{d}\\right| p m^{d-1} y_{i}^{d-1}$. Similar to Case 1, the right-hand side of (1) has absolute value at most $$ 21(d-1) B(2 c B)^{\\frac{1}{d}} m^{d-1} y_{i}^{d-1} $$ which must be smaller than $\\left|b_{d}\\right| p m^{d-1} y_{i}^{d-1}$ for large $p$. Again this yields a contradiction and hence $m y_{i}-l y_{1}=0$. In both cases, we find that $\\frac{Q\\left(y_{i}\\right)}{Q\\left(y_{1}\\right)}=\\frac{l^{d}}{m^{d}}=\\frac{y_{i}^{d}}{y_{1}^{d}}$. From the Claim, the polynomial $Q\\left(y_{1}\\right) y^{d}-y_{1}^{d} Q(y)$ has roots $y=y_{1}, y_{2}, \\ldots, y_{d+1}$. Since its degree is at most $d$, this must be the zero polynomial. Hence, $Q(y)=b_{d} y^{d}$. This implies $P(x)=a_{d}\\left(x+\\frac{a_{d-1}}{d a_{d}}\\right)^{d}$. Let $\\frac{a_{d-1}}{d a_{d}}=\\frac{s}{r}$ with integers $r, s$ where $r \\geqslant 1$ and $(r, s)=1$. Since $P$ has integer coefficients, we need $r^{d} \\mid a_{d}$. Let $a_{d}=r^{d} a$. Then $P(x)=a(r x+s)^{d}$. It is obvious that such a polynomial satisfies the conditions. Comment. In the proof, the use of prime and Dirichlet's Theorem can be avoided. One can easily show that each $P\\left(x_{i}\\right)$ can be expressed in the form $u v_{i}^{d}$ where $u, v_{i}$ are integers and $u$ cannot be divisible by the $d$-th power of a prime (note that $u$ depends only on $P$ ). By fixing a large integer $q$ and by choosing a large $n$, we can apply the Pigeonhole Principle and assume $x_{1} \\equiv x_{2} \\equiv \\cdots \\equiv x_{d+1}(\\bmod q)$ and $v_{1} \\equiv v_{2} \\equiv \\cdots \\equiv v_{d+1}(\\bmod q)$. Then the remaining proof is similar to Case 2 of the Solution. Alternatively, we give another modification of the proof as follows. We take a sufficiently large $n$ and consider the corresponding positive integers $y_{1}, y_{2}, \\ldots, y_{n}$. For each $2 \\leqslant i \\leqslant n$, let $\\frac{Q\\left(y_{i}\\right)}{Q\\left(y_{1}\\right)}=\\frac{l_{i}^{d}}{m_{i}^{d}}$. As in Case 1, if there are $d$ indices $i$ such that the integers $\\frac{c\\left|Q\\left(y_{1}\\right)\\right|}{m_{i}^{d}}$ are bounded below by a constant depending only on $P$, we can establish the Claim using those $y_{i}$ 's and complete the proof. Similarly, as in Case 2, if there are $d$ indices $i$ such that the integers $\\left|m_{i} y_{i}-l_{i} y_{1}\\right|$ are bounded below, then the proof goes the same. So it suffices to consider the case where $\\frac{c\\left|Q\\left(y_{1}\\right)\\right|}{m_{i}^{d}} \\leqslant M$ and $\\left|m_{i} y_{i}-l_{i} y_{1}\\right| \\leqslant N$ for all $2 \\leqslant i \\leqslant n^{\\prime}$ where $M, N$ are fixed constants and $n^{\\prime}$ is large. Since there are only finitely many choices for $m_{i}$ and $m_{i} y_{i}-l_{i} y_{1}$, by the Pigeonhole Principle, we can assume without loss of generality $m_{i}=m$ and $m_{i} y_{i}-l_{i} y_{1}=t$ for $2 \\leqslant i \\leqslant d+2$. Then $$ \\frac{Q\\left(y_{i}\\right)}{Q\\left(y_{1}\\right)}=\\frac{l_{i}^{d}}{m^{d}}=\\frac{\\left(m y_{i}-t\\right)^{d}}{m^{d} y_{1}^{d}} $$ so that $Q\\left(y_{1}\\right)(m y-t)^{d}-m^{d} y_{1}^{d} Q(y)$ has roots $y=y_{2}, y_{3}, \\ldots, y_{d+2}$. Its degree is at most $d$ and hence it is the zero polynomial. Therefore, $Q(y)=\\frac{b_{d}}{m^{d}}(m y-t)^{d}$. Indeed, $Q$ does not have the term $y^{d-1}$, which means $t$ should be 0 . This gives the corresponding $P(x)$ of the desired form. The two modifications of the Solution work equally well when the degree $d$ is even.","tier":0} diff --git a/IMO/segmented/en-IMO2017SL.jsonl b/IMO/segmented/en-IMO2017SL.jsonl new file mode 100644 index 0000000000000000000000000000000000000000..4e88f760f59cafc9af74d8d50baa8595b4932f8e --- /dev/null +++ b/IMO/segmented/en-IMO2017SL.jsonl @@ -0,0 +1,80 @@ +{"problem_type":"Algebra","problem_label":"A1","problem":"Let $a_{1}, a_{2}, \\ldots, a_{n}, k$, and $M$ be positive integers such that $$ \\frac{1}{a_{1}}+\\frac{1}{a_{2}}+\\cdots+\\frac{1}{a_{n}}=k \\quad \\text { and } \\quad a_{1} a_{2} \\ldots a_{n}=M $$ If $M>1$, prove that the polynomial $$ P(x)=M(x+1)^{k}-\\left(x+a_{1}\\right)\\left(x+a_{2}\\right) \\cdots\\left(x+a_{n}\\right) $$ has no positive roots. (Trinidad and Tobago)","solution":"We first prove that, for $x>0$, $$ a_{i}(x+1)^{1 \/ a_{i}} \\leqslant x+a_{i}, $$ with equality if and only if $a_{i}=1$. It is clear that equality occurs if $a_{i}=1$. If $a_{i}>1$, the AM-GM inequality applied to a single copy of $x+1$ and $a_{i}-1$ copies of 1 yields $$ \\frac{(x+1)+\\overbrace{1+1+\\cdots+1}^{a_{i}-1 \\text { ones }}}{a_{i}} \\geqslant \\sqrt[a_{i}]{(x+1) \\cdot 1^{a_{i}-1}} \\Longrightarrow a_{i}(x+1)^{1 \/ a_{i}} \\leqslant x+a_{i} $$ Since $x+1>1$, the inequality is strict for $a_{i}>1$. Multiplying the inequalities (1) for $i=1,2, \\ldots, n$ yields $$ \\prod_{i=1}^{n} a_{i}(x+1)^{1 \/ a_{i}} \\leqslant \\prod_{i=1}^{n}\\left(x+a_{i}\\right) \\Longleftrightarrow M(x+1)^{\\sum_{i=1}^{n} 1 \/ a_{i}}-\\prod_{i=1}^{n}\\left(x+a_{i}\\right) \\leqslant 0 \\Longleftrightarrow P(x) \\leqslant 0 $$ with equality iff $a_{i}=1$ for all $i \\in\\{1,2, \\ldots, n\\}$. But this implies $M=1$, which is not possible. Hence $P(x)<0$ for all $x \\in \\mathbb{R}^{+}$, and $P$ has no positive roots. Comment 1. Inequality (1) can be obtained in several ways. For instance, we may also use the binomial theorem: since $a_{i} \\geqslant 1$, $$ \\left(1+\\frac{x}{a_{i}}\\right)^{a_{i}}=\\sum_{j=0}^{a_{i}}\\binom{a_{i}}{j}\\left(\\frac{x}{a_{i}}\\right)^{j} \\geqslant\\binom{ a_{i}}{0}+\\binom{a_{i}}{1} \\cdot \\frac{x}{a_{i}}=1+x $$ Both proofs of (1) mimic proofs to Bernoulli's inequality for a positive integer exponent $a_{i}$; we can use this inequality directly: $$ \\left(1+\\frac{x}{a_{i}}\\right)^{a_{i}} \\geqslant 1+a_{i} \\cdot \\frac{x}{a_{i}}=1+x $$ and so $$ x+a_{i}=a_{i}\\left(1+\\frac{x}{a_{i}}\\right) \\geqslant a_{i}(1+x)^{1 \/ a_{i}} $$ or its (reversed) formulation, with exponent $1 \/ a_{i} \\leqslant 1$ : $$ (1+x)^{1 \/ a_{i}} \\leqslant 1+\\frac{1}{a_{i}} \\cdot x=\\frac{x+a_{i}}{a_{i}} \\Longrightarrow a_{i}(1+x)^{1 \/ a_{i}} \\leqslant x+a_{i} . $$","tier":0} +{"problem_type":"Algebra","problem_label":"A1","problem":"Let $a_{1}, a_{2}, \\ldots, a_{n}, k$, and $M$ be positive integers such that $$ \\frac{1}{a_{1}}+\\frac{1}{a_{2}}+\\cdots+\\frac{1}{a_{n}}=k \\quad \\text { and } \\quad a_{1} a_{2} \\ldots a_{n}=M $$ If $M>1$, prove that the polynomial $$ P(x)=M(x+1)^{k}-\\left(x+a_{1}\\right)\\left(x+a_{2}\\right) \\cdots\\left(x+a_{n}\\right) $$ has no positive roots. (Trinidad and Tobago)","solution":"We will prove that, in fact, all coefficients of the polynomial $P(x)$ are non-positive, and at least one of them is negative, which implies that $P(x)<0$ for $x>0$. Indeed, since $a_{j} \\geqslant 1$ for all $j$ and $a_{j}>1$ for some $j$ (since $a_{1} a_{2} \\ldots a_{n}=M>1$ ), we have $k=\\frac{1}{a_{1}}+\\frac{1}{a_{2}}+\\cdots+\\frac{1}{a_{n}}1$, if (2) is true for a given $r-1$ and $x \\neq 0$.","tier":0} +{"problem_type":"Algebra","problem_label":"A2","problem":"Let $q$ be a real number. Gugu has a napkin with ten distinct real numbers written on it, and he writes the following three lines of real numbers on the blackboard: - In the first line, Gugu writes down every number of the form $a-b$, where $a$ and $b$ are two (not necessarily distinct) numbers on his napkin. - In the second line, Gugu writes down every number of the form $q a b$, where $a$ and $b$ are two (not necessarily distinct) numbers from the first line. - In the third line, Gugu writes down every number of the form $a^{2}+b^{2}-c^{2}-d^{2}$, where $a, b, c, d$ are four (not necessarily distinct) numbers from the first line. Determine all values of $q$ such that, regardless of the numbers on Gugu's napkin, every number in the second line is also a number in the third line.","solution":"Call a number $q$ good if every number in the second line appears in the third line unconditionally. We first show that the numbers 0 and $\\pm 2$ are good. The third line necessarily contains 0 , so 0 is good. For any two numbers $a, b$ in the first line, write $a=x-y$ and $b=u-v$, where $x, y, u, v$ are (not necessarily distinct) numbers on the napkin. We may now write $$ 2 a b=2(x-y)(u-v)=(x-v)^{2}+(y-u)^{2}-(x-u)^{2}-(y-v)^{2} $$ which shows that 2 is good. By negating both sides of the above equation, we also see that -2 is good. We now show that $-2,0$, and 2 are the only good numbers. Assume for sake of contradiction that $q$ is a good number, where $q \\notin\\{-2,0,2\\}$. We now consider some particular choices of numbers on Gugu's napkin to arrive at a contradiction. Assume that the napkin contains the integers $1,2, \\ldots, 10$. Then, the first line contains the integers $-9,-8, \\ldots, 9$. The second line then contains $q$ and $81 q$, so the third line must also contain both of them. But the third line only contains integers, so $q$ must be an integer. Furthermore, the third line contains no number greater than $162=9^{2}+9^{2}-0^{2}-0^{2}$ or less than -162 , so we must have $-162 \\leqslant 81 q \\leqslant 162$. This shows that the only possibilities for $q$ are $\\pm 1$. Now assume that $q= \\pm 1$. Let the napkin contain $0,1,4,8,12,16,20,24,28,32$. The first line contains $\\pm 1$ and $\\pm 4$, so the second line contains $\\pm 4$. However, for every number $a$ in the first line, $a \\not \\equiv 2(\\bmod 4)$, so we may conclude that $a^{2} \\equiv 0,1(\\bmod 8)$. Consequently, every number in the third line must be congruent to $-2,-1,0,1,2(\\bmod 8)$; in particular, $\\pm 4$ cannot be in the third line, which is a contradiction.","tier":0} +{"problem_type":"Algebra","problem_label":"A2","problem":"Let $q$ be a real number. Gugu has a napkin with ten distinct real numbers written on it, and he writes the following three lines of real numbers on the blackboard: - In the first line, Gugu writes down every number of the form $a-b$, where $a$ and $b$ are two (not necessarily distinct) numbers on his napkin. - In the second line, Gugu writes down every number of the form $q a b$, where $a$ and $b$ are two (not necessarily distinct) numbers from the first line. - In the third line, Gugu writes down every number of the form $a^{2}+b^{2}-c^{2}-d^{2}$, where $a, b, c, d$ are four (not necessarily distinct) numbers from the first line. Determine all values of $q$ such that, regardless of the numbers on Gugu's napkin, every number in the second line is also a number in the third line.","solution":"Let $q$ be a good number, as defined in the first solution, and define the polynomial $P\\left(x_{1}, \\ldots, x_{10}\\right)$ as $$ \\prod_{i2017 $$ Prove that this sequence is bounded, i.e., there is a constant $M$ such that $\\left|a_{n}\\right| \\leqslant M$ for all positive integers $n$.","solution":"Set $D=2017$. Denote $$ M_{n}=\\max _{kD$; our first aim is to bound $a_{n}$ in terms of $m_{n}$ and $M_{n}$. (i) There exist indices $p$ and $q$ such that $a_{n}=-\\left(a_{p}+a_{q}\\right)$ and $p+q=n$. Since $a_{p}, a_{q} \\leqslant M_{n}$, we have $a_{n} \\geqslant-2 M_{n}$. (ii) On the other hand, choose an index $kD$ is lucky if $m_{n} \\leqslant 2 M_{n}$. Two cases are possible. Case 1. Assume that there exists a lucky index $n$. In this case, (1) yields $m_{n+1} \\leqslant 2 M_{n}$ and $M_{n} \\leqslant M_{n+1} \\leqslant M_{n}$. Therefore, $M_{n+1}=M_{n}$ and $m_{n+1} \\leqslant 2 M_{n}=2 M_{n+1}$. So, the index $n+1$ is also lucky, and $M_{n+1}=M_{n}$. Applying the same arguments repeatedly, we obtain that all indices $k>n$ are lucky (i.e., $m_{k} \\leqslant 2 M_{k}$ for all these indices), and $M_{k}=M_{n}$ for all such indices. Thus, all of the $m_{k}$ and $M_{k}$ are bounded by $2 M_{n}$. Case 2. Assume now that there is no lucky index, i.e., $2 M_{n}D$. Then (1) shows that for all $n>D$ we have $m_{n} \\leqslant m_{n+1} \\leqslant m_{n}$, so $m_{n}=m_{D+1}$ for all $n>D$. Since $M_{n}2017 $$ Prove that this sequence is bounded, i.e., there is a constant $M$ such that $\\left|a_{n}\\right| \\leqslant M$ for all positive integers $n$.","solution":"As in the previous solution, let $D=2017$. If the sequence is bounded above, say, by $Q$, then we have that $a_{n} \\geqslant \\min \\left\\{a_{1}, \\ldots, a_{D},-2 Q\\right\\}$ for all $n$, so the sequence is bounded. Assume for sake of contradiction that the sequence is not bounded above. Let $\\ell=\\min \\left\\{a_{1}, \\ldots, a_{D}\\right\\}$, and $L=\\max \\left\\{a_{1}, \\ldots, a_{D}\\right\\}$. Call an index $n$ good if the following criteria hold: $$ a_{n}>a_{i} \\text { for each } i-2 \\ell, \\quad \\text { and } \\quad n>D $$ We first show that there must be some good index $n$. By assumption, we may take an index $N$ such that $a_{N}>\\max \\{L,-2 \\ell\\}$. Choose $n$ minimally such that $a_{n}=\\max \\left\\{a_{1}, a_{2}, \\ldots, a_{N}\\right\\}$. Now, the first condition in (2) is satisfied because of the minimality of $n$, and the second and third conditions are satisfied because $a_{n} \\geqslant a_{N}>L,-2 \\ell$, and $L \\geqslant a_{i}$ for every $i$ such that $1 \\leqslant i \\leqslant D$. Let $n$ be a good index. We derive a contradiction. We have that $$ a_{n}+a_{u}+a_{v} \\leqslant 0 $$ whenever $u+v=n$. We define the index $u$ to maximize $a_{u}$ over $1 \\leqslant u \\leqslant n-1$, and let $v=n-u$. Then, we note that $a_{u} \\geqslant a_{v}$ by the maximality of $a_{u}$. Assume first that $v \\leqslant D$. Then, we have that $$ a_{N}+2 \\ell \\leqslant 0, $$ because $a_{u} \\geqslant a_{v} \\geqslant \\ell$. But this contradicts our assumption that $a_{n}>-2 \\ell$ in the second criteria of (2). Now assume that $v>D$. Then, there exist some indices $w_{1}, w_{2}$ summing up to $v$ such that $$ a_{v}+a_{w_{1}}+a_{w_{2}}=0 $$ But combining this with (3), we have $$ a_{n}+a_{u} \\leqslant a_{w_{1}}+a_{w_{2}} $$ Because $a_{n}>a_{u}$, we have that $\\max \\left\\{a_{w_{1}}, a_{w_{2}}\\right\\}>a_{u}$. But since each of the $w_{i}$ is less than $v$, this contradicts the maximality of $a_{u}$. Comment 1. We present two harder versions of this problem below. Version 1. Let $a_{1}, a_{2}, \\ldots$ be a sequence of numbers that satisfies the relation $$ a_{n}=-\\max _{i+j+k=n}\\left(a_{i}+a_{j}+a_{k}\\right) \\quad \\text { for all } n>2017 $$ Then, this sequence is bounded. Proof. Set $D=2017$. Denote $$ M_{n}=\\max _{k2 D$; our first aim is to bound $a_{n}$ in terms of $m_{i}$ and $M_{i}$. Set $k=\\lfloor n \/ 2\\rfloor$. (i) Choose indices $p, q$, and $r$ such that $a_{n}=-\\left(a_{p}+a_{q}+a_{r}\\right)$ and $p+q+r=n$. Without loss of generality, $p \\geqslant q \\geqslant r$. Assume that $p \\geqslant k+1(>D)$; then $p>q+r$. Hence $$ -a_{p}=\\max _{i_{1}+i_{2}+i_{3}=p}\\left(a_{i_{1}}+a_{i_{2}}+a_{i_{3}}\\right) \\geqslant a_{q}+a_{r}+a_{p-q-r} $$ and therefore $a_{n}=-\\left(a_{p}+a_{q}+a_{r}\\right) \\geqslant\\left(a_{q}+a_{r}+a_{p-q-r}\\right)-a_{q}-a_{r}=a_{p-q-r} \\geqslant-m_{n}$. Otherwise, we have $k \\geqslant p \\geqslant q \\geqslant r$. Since $n<3 k$, we have $r2 D$ is lucky if $m_{n} \\leqslant 2 M_{\\lfloor n \/ 2\\rfloor+1}+M_{\\lfloor n \/ 2]}$. Two cases are possible. Case 1. Assume that there exists a lucky index $n$; set $k=\\lfloor n \/ 2\\rfloor$. In this case, (4) yields $m_{n+1} \\leqslant$ $2 M_{k+1}+M_{k}$ and $M_{n} \\leqslant M_{n+1} \\leqslant M_{n}$ (the last relation holds, since $m_{n}-M_{k+1}-M_{k} \\leqslant\\left(2 M_{k+1}+\\right.$ $\\left.M_{k}\\right)-M_{k+1}-M_{k}=M_{k+1} \\leqslant M_{n}$ ). Therefore, $M_{n+1}=M_{n}$ and $m_{n+1} \\leqslant 2 M_{k+1}+M_{k}$; the last relation shows that the index $n+1$ is also lucky. Thus, all indices $N>n$ are lucky, and $M_{N}=M_{n} \\geqslant m_{N} \/ 3$, whence all the $m_{N}$ and $M_{N}$ are bounded by $3 M_{n}$. Case 2. Conversely, assume that there is no lucky index, i.e., $2 M_{\\lfloor n \/ 2\\rfloor+1}+M_{\\lfloor n \/ 2\\rfloor}2 D$. Then (4) shows that for all $n>2 D$ we have $m_{n} \\leqslant m_{n+1} \\leqslant m_{n}$, i.e., $m_{N}=m_{2 D+1}$ for all $N>2 D$. Since $M_{N}2017 $$ Then, this sequence is bounded. Proof. As in the solutions above, let $D=2017$. If the sequence is bounded above, say, by $Q$, then we have that $a_{n} \\geqslant \\min \\left\\{a_{1}, \\ldots, a_{D},-k Q\\right\\}$ for all $n$, so the sequence is bounded. Assume for sake of contradiction that the sequence is not bounded above. Let $\\ell=\\min \\left\\{a_{1}, \\ldots, a_{D}\\right\\}$, and $L=\\max \\left\\{a_{1}, \\ldots, a_{D}\\right\\}$. Call an index $n$ good if the following criteria hold: $$ a_{n}>a_{i} \\text { for each } i-k \\ell, \\quad \\text { and } \\quad n>D $$ We first show that there must be some good index $n$. By assumption, we may take an index $N$ such that $a_{N}>\\max \\{L,-k \\ell\\}$. Choose $n$ minimally such that $a_{n}=\\max \\left\\{a_{1}, a_{2}, \\ldots, a_{N}\\right\\}$. Now, the first condition is satisfied because of the minimality of $n$, and the second and third conditions are satisfied because $a_{n} \\geqslant a_{N}>L,-k \\ell$, and $L \\geqslant a_{i}$ for every $i$ such that $1 \\leqslant i \\leqslant D$. Let $n$ be a good index. We derive a contradiction. We have that $$ a_{n}+a_{v_{1}}+\\cdots+a_{v_{k}} \\leqslant 0 $$ whenever $v_{1}+\\cdots+v_{k}=n$. We define the sequence of indices $v_{1}, \\ldots, v_{k-1}$ to greedily maximize $a_{v_{1}}$, then $a_{v_{2}}$, and so forth, selecting only from indices such that the equation $v_{1}+\\cdots+v_{k}=n$ can be satisfied by positive integers $v_{1}, \\ldots, v_{k}$. More formally, we define them inductively so that the following criteria are satisfied by the $v_{i}$ : 1. $1 \\leqslant v_{i} \\leqslant n-(k-i)-\\left(v_{1}+\\cdots+v_{i-1}\\right)$. 2. $a_{v_{i}}$ is maximal among all choices of $v_{i}$ from the first criteria. First of all, we note that for each $i$, the first criteria is always satisfiable by some $v_{i}$, because we are guaranteed that $$ v_{i-1} \\leqslant n-(k-(i-1))-\\left(v_{1}+\\cdots+v_{i-2}\\right), $$ which implies $$ 1 \\leqslant n-(k-i)-\\left(v_{1}+\\cdots+v_{i-1}\\right) . $$ Secondly, the sum $v_{1}+\\cdots+v_{k-1}$ is at most $n-1$. Define $v_{k}=n-\\left(v_{1}+\\cdots+v_{k-1}\\right)$. Then, (6) is satisfied by the $v_{i}$. We also note that $a_{v_{i}} \\geqslant a_{v_{j}}$ for all $i-k \\ell$ in the second criteria of (5). Now assume that $v_{k}>D$, and then we must have some indices $w_{1}, \\ldots, w_{k}$ summing up to $v_{k}$ such that $$ a_{v_{k}}+a_{w_{1}}+\\cdots+a_{w_{k}}=0 $$ But combining this with (6), we have $$ a_{n}+a_{v_{1}}+\\cdots+a_{v_{k-1}} \\leqslant a_{w_{1}}+\\cdots+a_{w_{k}} . $$ Because $a_{n}>a_{v_{1}} \\geqslant \\cdots \\geqslant a_{v_{k-1}}$, we have that $\\max \\left\\{a_{w_{1}}, \\ldots, a_{w_{k}}\\right\\}>a_{v_{k-1}}$. But since each of the $w_{i}$ is less than $v_{k}$, in the definition of the $v_{k-1}$ we could have chosen one of the $w_{i}$ instead, which is a contradiction. Comment 2. It seems that each sequence satisfying the condition in Version 2 is eventually periodic, at least when its terms are integers. However, up to this moment, the Problem Selection Committee is not aware of a proof for this fact (even in the case $k=2$ ).","tier":0} +{"problem_type":"Algebra","problem_label":"A5","problem":"An integer $n \\geqslant 3$ is given. We call an $n$-tuple of real numbers $\\left(x_{1}, x_{2}, \\ldots, x_{n}\\right)$ Shiny if for each permutation $y_{1}, y_{2}, \\ldots, y_{n}$ of these numbers we have $$ \\sum_{i=1}^{n-1} y_{i} y_{i+1}=y_{1} y_{2}+y_{2} y_{3}+y_{3} y_{4}+\\cdots+y_{n-1} y_{n} \\geqslant-1 $$ Find the largest constant $K=K(n)$ such that $$ \\sum_{1 \\leqslant i\\ell$. Case 1: $k>\\ell$. Consider all permutations $\\phi:\\{1,2, \\ldots, n\\} \\rightarrow\\{1,2, \\ldots, n\\}$ such that $\\phi^{-1}(T)=\\{2,4, \\ldots, 2 \\ell\\}$. Note that there are $k!!$ ! such permutations $\\phi$. Define $$ f(\\phi)=\\sum_{i=1}^{n-1} x_{\\phi(i)} x_{\\phi(i+1)} $$ We know that $f(\\phi) \\geqslant-1$ for every permutation $\\phi$ with the above property. Averaging $f(\\phi)$ over all $\\phi$ gives $$ -1 \\leqslant \\frac{1}{k!\\ell!} \\sum_{\\phi} f(\\phi)=\\frac{2 \\ell}{k \\ell} M+\\frac{2(k-\\ell-1)}{k(k-1)} K $$ where the equality holds because there are $k \\ell$ products in $M$, of which $2 \\ell$ are selected for each $\\phi$, and there are $k(k-1) \/ 2$ products in $K$, of which $k-\\ell-1$ are selected for each $\\phi$. We now have $$ K+L+M \\geqslant K+L+\\left(-\\frac{k}{2}-\\frac{k-\\ell-1}{k-1} K\\right)=-\\frac{k}{2}+\\frac{\\ell}{k-1} K+L $$ Since $k \\leqslant n-1$ and $K, L \\geqslant 0$, we get the desired inequality. Case 2: $k=\\ell=n \/ 2$. We do a similar approach, considering all $\\phi:\\{1,2, \\ldots, n\\} \\rightarrow\\{1,2, \\ldots, n\\}$ such that $\\phi^{-1}(T)=$ $\\{2,4, \\ldots, 2 \\ell\\}$, and defining $f$ the same way. Analogously to Case 1 , we have $$ -1 \\leqslant \\frac{1}{k!\\ell!} \\sum_{\\phi} f(\\phi)=\\frac{2 \\ell-1}{k \\ell} M $$ because there are $k \\ell$ products in $M$, of which $2 \\ell-1$ are selected for each $\\phi$. Now, we have that $$ K+L+M \\geqslant M \\geqslant-\\frac{n^{2}}{4(n-1)} \\geqslant-\\frac{n-1}{2} $$ where the last inequality holds because $n \\geqslant 4$.","tier":0} +{"problem_type":"Algebra","problem_label":"A6","problem":"Find all functions $f: \\mathbb{R} \\rightarrow \\mathbb{R}$ such that $$ f(f(x) f(y))+f(x+y)=f(x y) $$ for all $x, y \\in \\mathbb{R}$. (Albania)","solution":"An easy check shows that all the 3 above mentioned functions indeed satisfy the original equation (*). In order to show that these are the only solutions, first observe that if $f(x)$ is a solution then $-f(x)$ is also a solution. Hence, without loss of generality we may (and will) assume that $f(0) \\leqslant 0$ from now on. We have to show that either $f$ is identically zero or $f(x)=x-1$ $(\\forall x \\in \\mathbb{R})$. Observe that, for a fixed $x \\neq 1$, we may choose $y \\in \\mathbb{R}$ so that $x+y=x y \\Longleftrightarrow y=\\frac{x}{x-1}$, and therefore from the original equation (*) we have $$ f\\left(f(x) \\cdot f\\left(\\frac{x}{x-1}\\right)\\right)=0 \\quad(x \\neq 1) $$ In particular, plugging in $x=0$ in (1), we conclude that $f$ has at least one zero, namely $(f(0))^{2}$ : $$ f\\left((f(0))^{2}\\right)=0 $$ We analyze two cases (recall that $f(0) \\leqslant 0$ ): Case 1: $f(0)=0$. Setting $y=0$ in the original equation we get the identically zero solution: $$ f(f(x) f(0))+f(x)=f(0) \\Longrightarrow f(x)=0 \\text { for all } x \\in \\mathbb{R} $$ From now on, we work on the main Case 2: $f(0)<0$. We begin with the following","tier":0} +{"problem_type":"Algebra","problem_label":"A7","problem":"Let $a_{0}, a_{1}, a_{2}, \\ldots$ be a sequence of integers and $b_{0}, b_{1}, b_{2}, \\ldots$ be a sequence of positive integers such that $a_{0}=0, a_{1}=1$, and $$ a_{n+1}=\\left\\{\\begin{array}{ll} a_{n} b_{n}+a_{n-1}, & \\text { if } b_{n-1}=1 \\\\ a_{n} b_{n}-a_{n-1}, & \\text { if } b_{n-1}>1 \\end{array} \\quad \\text { for } n=1,2, \\ldots\\right. $$ Prove that at least one of the two numbers $a_{2017}$ and $a_{2018}$ must be greater than or equal to 2017 . (Australia)","solution":"The value of $b_{0}$ is irrelevant since $a_{0}=0$, so we may assume that $b_{0}=1$. Lemma. We have $a_{n} \\geqslant 1$ for all $n \\geqslant 1$. Proof. Let us suppose otherwise in order to obtain a contradiction. Let $$ n \\geqslant 1 \\text { be the smallest integer with } a_{n} \\leqslant 0 \\text {. } $$ Note that $n \\geqslant 2$. It follows that $a_{n-1} \\geqslant 1$ and $a_{n-2} \\geqslant 0$. Thus we cannot have $a_{n}=$ $a_{n-1} b_{n-1}+a_{n-2}$, so we must have $a_{n}=a_{n-1} b_{n-1}-a_{n-2}$. Since $a_{n} \\leqslant 0$, we have $a_{n-1} \\leqslant a_{n-2}$. Thus we have $a_{n-2} \\geqslant a_{n-1} \\geqslant a_{n}$. Let $$ r \\text { be the smallest index with } a_{r} \\geqslant a_{r+1} \\geqslant a_{r+2} \\text {. } $$ Then $r \\leqslant n-2$ by the above, but also $r \\geqslant 2$ : if $b_{1}=1$, then $a_{2}=a_{1}=1$ and $a_{3}=a_{2} b_{2}+a_{1}>a_{2}$; if $b_{1}>1$, then $a_{2}=b_{1}>1=a_{1}$. By the minimal choice (2) of $r$, it follows that $a_{r-1}0$. In order to have $a_{r+1} \\geqslant a_{r+2}$, we must have $a_{r+2}=a_{r+1} b_{r+1}-a_{r}$ so that $b_{r} \\geqslant 2$. Putting everything together, we conclude that $$ a_{r+1}=a_{r} b_{r} \\pm a_{r-1} \\geqslant 2 a_{r}-a_{r-1}=a_{r}+\\left(a_{r}-a_{r-1}\\right)>a_{r} $$ which contradicts (2). To complete the problem, we prove that $\\max \\left\\{a_{n}, a_{n+1}\\right\\} \\geqslant n$ by induction. The cases $n=0,1$ are given. Assume it is true for all non-negative integers strictly less than $n$, where $n \\geqslant 2$. There are two cases: Case 1: $b_{n-1}=1$. Then $a_{n+1}=a_{n} b_{n}+a_{n-1}$. By the inductive assumption one of $a_{n-1}, a_{n}$ is at least $n-1$ and the other, by the lemma, is at least 1 . Hence $$ a_{n+1}=a_{n} b_{n}+a_{n-1} \\geqslant a_{n}+a_{n-1} \\geqslant(n-1)+1=n . $$ Thus $\\max \\left\\{a_{n}, a_{n+1}\\right\\} \\geqslant n$, as desired. Case 2: $b_{n-1}>1$. Since we defined $b_{0}=1$ there is an index $r$ with $1 \\leqslant r \\leqslant n-1$ such that $$ b_{n-1}, b_{n-2}, \\ldots, b_{r} \\geqslant 2 \\quad \\text { and } \\quad b_{r-1}=1 $$ We have $a_{r+1}=a_{r} b_{r}+a_{r-1} \\geqslant 2 a_{r}+a_{r-1}$. Thus $a_{r+1}-a_{r} \\geqslant a_{r}+a_{r-1}$. Now we claim that $a_{r}+a_{r-1} \\geqslant r$. Indeed, this holds by inspection for $r=1$; for $r \\geqslant 2$, one of $a_{r}, a_{r-1}$ is at least $r-1$ by the inductive assumption, while the other, by the lemma, is at least 1 . Hence $a_{r}+a_{r-1} \\geqslant r$, as claimed, and therefore $a_{r+1}-a_{r} \\geqslant r$ by the last inequality in the previous paragraph. Since $r \\geqslant 1$ and, by the lemma, $a_{r} \\geqslant 1$, from $a_{r+1}-a_{r} \\geqslant r$ we get the following two inequalities: $$ a_{r+1} \\geqslant r+1 \\quad \\text { and } \\quad a_{r+1}>a_{r} . $$ Now observe that $$ a_{m}>a_{m-1} \\Longrightarrow a_{m+1}>a_{m} \\text { for } m=r+1, r+2, \\ldots, n-1 \\text {, } $$ since $a_{m+1}=a_{m} b_{m}-a_{m-1} \\geqslant 2 a_{m}-a_{m-1}=a_{m}+\\left(a_{m}-a_{m-1}\\right)>a_{m}$. Thus $$ a_{n}>a_{n-1}>\\cdots>a_{r+1} \\geqslant r+1 \\Longrightarrow a_{n} \\geqslant n $$ So $\\max \\left\\{a_{n}, a_{n+1}\\right\\} \\geqslant n$, as desired.","tier":0} +{"problem_type":"Algebra","problem_label":"A7","problem":"Let $a_{0}, a_{1}, a_{2}, \\ldots$ be a sequence of integers and $b_{0}, b_{1}, b_{2}, \\ldots$ be a sequence of positive integers such that $a_{0}=0, a_{1}=1$, and $$ a_{n+1}=\\left\\{\\begin{array}{ll} a_{n} b_{n}+a_{n-1}, & \\text { if } b_{n-1}=1 \\\\ a_{n} b_{n}-a_{n-1}, & \\text { if } b_{n-1}>1 \\end{array} \\quad \\text { for } n=1,2, \\ldots\\right. $$ Prove that at least one of the two numbers $a_{2017}$ and $a_{2018}$ must be greater than or equal to 2017 . (Australia)","solution":"We say that an index $n>1$ is bad if $b_{n-1}=1$ and $b_{n-2}>1$; otherwise $n$ is good. The value of $b_{0}$ is irrelevant to the definition of $\\left(a_{n}\\right)$ since $a_{0}=0$; so we assume that $b_{0}>1$. Lemma 1. (a) $a_{n} \\geqslant 1$ for all $n>0$. (b) If $n>1$ is good, then $a_{n}>a_{n-1}$. Proof. Induction on $n$. In the base cases $n=1,2$ we have $a_{1}=1 \\geqslant 1, a_{2}=b_{1} a_{1} \\geqslant 1$, and finally $a_{2}>a_{1}$ if 2 is good, since in this case $b_{1}>1$. Now we assume that the lemma statement is proved for $n=1,2, \\ldots, k$ with $k \\geqslant 2$, and prove it for $n=k+1$. Recall that $a_{k}$ and $a_{k-1}$ are positive by the induction hypothesis. Case 1: $k$ is bad. We have $b_{k-1}=1$, so $a_{k+1}=b_{k} a_{k}+a_{k-1} \\geqslant a_{k}+a_{k-1}>a_{k} \\geqslant 1$, as required. Case 2: $k$ is good. We already have $a_{k}>a_{k-1} \\geqslant 1$ by the induction hypothesis. We consider three easy subcases. Subcase 2.1: $b_{k}>1$. Then $a_{k+1} \\geqslant b_{k} a_{k}-a_{k-1} \\geqslant a_{k}+\\left(a_{k}-a_{k-1}\\right)>a_{k} \\geqslant 1$. Subcase 2.2: $b_{k}=b_{k-1}=1$. Then $a_{k+1}=a_{k}+a_{k-1}>a_{k} \\geqslant 1$. Subcase 2.3: $b_{k}=1$ but $b_{k-1}>1$. Then $k+1$ is bad, and we need to prove only (a), which is trivial: $a_{k+1}=a_{k}-a_{k-1} \\geqslant 1$. So, in all three subcases we have verified the required relations. Lemma 2. Assume that $n>1$ is bad. Then there exists a $j \\in\\{1,2,3\\}$ such that $a_{n+j} \\geqslant$ $a_{n-1}+j+1$, and $a_{n+i} \\geqslant a_{n-1}+i$ for all $1 \\leqslant i0: b_{n+i-1}>1\\right\\} $$ (possibly $m=+\\infty$ ). We claim that $j=\\min \\{m, 3\\}$ works. Again, we distinguish several cases, according to the value of $m$; in each of them we use Lemma 1 without reference. Case 1: $m=1$, so $b_{n}>1$. Then $a_{n+1} \\geqslant 2 a_{n}+a_{n-1} \\geqslant a_{n-1}+2$, as required. Case 2: $m=2$, so $b_{n}=1$ and $b_{n+1}>1$. Then we successively get $$ \\begin{gathered} a_{n+1}=a_{n}+a_{n-1} \\geqslant a_{n-1}+1 \\\\ a_{n+2} \\geqslant 2 a_{n+1}+a_{n} \\geqslant 2\\left(a_{n-1}+1\\right)+a_{n}=a_{n-1}+\\left(a_{n-1}+a_{n}+2\\right) \\geqslant a_{n-1}+4 \\end{gathered} $$ which is even better than we need. Case 3: $m>2$, so $b_{n}=b_{n+1}=1$. Then we successively get $$ \\begin{gathered} a_{n+1}=a_{n}+a_{n-1} \\geqslant a_{n-1}+1, \\quad a_{n+2}=a_{n+1}+a_{n} \\geqslant a_{n-1}+1+a_{n} \\geqslant a_{n-1}+2, \\\\ a_{n+3} \\geqslant a_{n+2}+a_{n+1} \\geqslant\\left(a_{n-1}+1\\right)+\\left(a_{n-1}+2\\right) \\geqslant a_{n-1}+4 \\end{gathered} $$ as required. Lemmas 1 (b) and 2 provide enough information to prove that $\\max \\left\\{a_{n}, a_{n+1}\\right\\} \\geqslant n$ for all $n$ and, moreover, that $a_{n} \\geqslant n$ often enough. Indeed, assume that we have found some $n$ with $a_{n-1} \\geqslant n-1$. If $n$ is good, then by Lemma 1 (b) we have $a_{n} \\geqslant n$ as well. If $n$ is bad, then Lemma 2 yields $\\max \\left\\{a_{n+i}, a_{n+i+1}\\right\\} \\geqslant a_{n-1}+i+1 \\geqslant n+i$ for all $0 \\leqslant i0$, we have $f(x)+y=f(y)+x$. Prove that $f(x)+y \\leqslant f(y)+x$ whenever $x>y$. (Netherlands)","solution":"Define $g(x)=x-f(x)$. The condition on $f$ then rewrites as follows: For every $x, y \\in \\mathbb{R}$ such that $((x+y)-g(x))((x+y)-g(y))>0$, we have $g(x)=g(y)$. This condition may in turn be rewritten in the following form: If $g(x) \\neq g(y)$, then the number $x+y$ lies (non-strictly) between $g(x)$ and $g(y)$. Notice here that the function $g_{1}(x)=-g(-x)$ also satisfies $(*)$, since $$ \\begin{aligned} g_{1}(x) \\neq g_{1}(y) \\Longrightarrow & g(-x) \\neq g(-y) \\Longrightarrow \\quad-(x+y) \\text { lies between } g(-x) \\text { and } g(-y) \\\\ & \\Longrightarrow \\quad x+y \\text { lies between } g_{1}(x) \\text { and } g_{1}(y) \\end{aligned} $$ On the other hand, the relation we need to prove reads now as $$ g(x) \\leqslant g(y) \\quad \\text { whenever } xX$. Similarly, if $X>2 x$, then $g$ attains at most two values on $[x ; X-x)$ - namely, $X$ and, possibly, some $YX$ and hence by (*) we get $X \\leqslant a+x \\leqslant g(a)$. Now, for any $b \\in(X-x ; x)$ with $g(b) \\neq X$ we similarly get $b+x \\leqslant g(b)$. Therefore, the number $a+b$ (which is smaller than each of $a+x$ and $b+x$ ) cannot lie between $g(a)$ and $g(b)$, which by (*) implies that $g(a)=g(b)$. Hence $g$ may attain only two values on $(X-x ; x]$, namely $X$ and $g(a)>X$. To prove the second claim, notice that $g_{1}(-x)=-X<2 \\cdot(-x)$, so $g_{1}$ attains at most two values on $(-X+x,-x]$, i.e., $-X$ and, possibly, some $-Y>-X$. Passing back to $g$, we get what we need. Lemma 2. If $X<2 x$, then $g$ is constant on $(X-x ; x)$. Similarly, if $X>2 x$, then $g$ is constant on $(x ; X-x)$. Proof. Again, it suffices to prove the first claim only. Assume, for the sake of contradiction, that there exist $a, b \\in(X-x ; x)$ with $g(a) \\neq g(b)$; by Lemma 1, we may assume that $g(a)=X$ and $Y=g(b)>X$. Notice that $\\min \\{X-a, X-b\\}>X-x$, so there exists a $u \\in(X-x ; x)$ such that $u<\\min \\{X-a, X-b\\}$. By Lemma 1, we have either $g(u)=X$ or $g(u)=Y$. In the former case, by (*) we have $X \\leqslant u+b \\leqslant Y$ which contradicts $u2 x$, then $g(a)=X$ for all $a \\in(x ; X-x)$. Proof. Again, we only prove the first claim. By Lemmas 1 and 2, this claim may be violated only if $g$ takes on a constant value $Y>X$ on $(X-x, x)$. Choose any $a, b \\in(X-x ; x)$ with $a2 a$. Applying Lemma 2 to $a$ in place of $x$, we obtain that $g$ is constant on $(a, Y-a)$. By (2) again, we have $x \\leqslant Y-bg(y)$ for some $x0$, we have $f(x)+y=f(y)+x$. Prove that $f(x)+y \\leqslant f(y)+x$ whenever $x>y$. (Netherlands)","solution":"As in the previous solution, we pass to the function $g$ satisfying ( $*$ ) and notice that we need to prove the condition (1). We will also make use of the function $g_{1}$. If $g$ is constant, then (1) is clearly satisfied. So, in the sequel we assume that $g$ takes on at least two different values. Now we collect some information about the function $g$. Claim 1. For any $c \\in \\mathbb{R}$, all the solutions of $g(x)=c$ are bounded. Proof. Fix any $y \\in \\mathbb{R}$ with $g(y) \\neq c$. Assume first that $g(y)>c$. Now, for any $x$ with $g(x)=c$, by (*) we have $c \\leqslant x+y \\leqslant g(y)$, or $c-y \\leqslant x \\leqslant g(y)-y$. Since $c$ and $y$ are constant, we get what we need. If $g(y)-c$. By the above arguments, we obtain that all the solutions of $g_{1}(-x)=-c$ are bounded, which is equivalent to what we need. As an immediate consequence, the function $g$ takes on infinitely many values, which shows that the next claim is indeed widely applicable. Claim 2. If $g(x)g(y)$ for some $x0$ the equation $g(x)=-N x$ has at most one solution, and (ii) for every $N>1$ the equation $g(x)=N x$ has at least one solution. Claim ( $i$ ) is now trivial. Indeed, $g$ is proven to be non-decreasing, so $g(x)+N x$ is strictly increasing and thus has at most one zero. We proceed on claim $(i i)$. If $g(0)=0$, then the required root has been already found. Otherwise, we may assume that $g(0)>0$ and denote $c=g(0)$. We intend to prove that $x=c \/ N$ is the required root. Indeed, by monotonicity we have $g(c \/ N) \\geqslant g(0)=c$; if we had $g(c \/ N)>c$, then (*) would yield $c \\leqslant 0+c \/ N \\leqslant g(c \/ N)$ which is false. Thus, $g(x)=c=N x$. Comment 2. There are plenty of functions $g$ satisfying (*) (and hence of functions $f$ satisfying the problem conditions). One simple example is $g_{0}(x)=2 x$. Next, for any increasing sequence $A=\\left(\\ldots, a_{-1}, a_{0}, a_{1}, \\ldots\\right)$ which is unbounded in both directions (i.e., for every $N$ this sequence contains terms greater than $N$, as well as terms smaller than $-N$ ), the function $g_{A}$ defined by $$ g_{A}(x)=a_{i}+a_{i+1} \\quad \\text { whenever } x \\in\\left[a_{i} ; a_{i+1}\\right) $$ satisfies (*). Indeed, pick any $xx\\} & \\text { and } \\quad s_{A-}(x)=\\sup \\{a \\in A: aN_{t}(Y)$, and define $d^{*}(X, Y)$ to be the number of $(X, Y)$-inversions. Then for any two chameleons $Y$ and $Y^{\\prime}$ differing by a single swap, we have $\\left|d^{*}(X, Y)-d^{*}\\left(X, Y^{\\prime}\\right)\\right|=1$. Since $d^{*}(X, X)=0$, this yields $d(X, Y) \\geqslant d^{*}(X, Y)$ for any pair of chameleons $X$ and $Y$. The bound $d^{*}$ may also be used in both Solution 1 and Solution 2. Comment 3. In fact, one may prove that the distance $d^{*}$ defined in the previous comment coincides with $d$. Indeed, if $X \\neq Y$, then there exist an ( $X, Y$ )-inversion $(s, t)$. One can show that such $s$ and $t$ may be chosen to occupy consecutive positions in $Y$. Clearly, $s$ and $t$ correspond to different letters among $\\{a, b, c\\}$. So, swapping them in $Y$ we get another chameleon $Y^{\\prime}$ with $d^{*}\\left(X, Y^{\\prime}\\right)=d^{*}(X, Y)-1$. Proceeding in this manner, we may change $Y$ to $X$ in $d^{*}(X, Y)$ steps. Using this fact, one can show that the estimate in the problem statement is sharp for all $n \\geqslant 2$. (For $n=1$ it is not sharp, since any permutation of three letters can be changed to an opposite one in no less than three swaps.) We outline the proof below. For any $k \\geqslant 0$, define $$ X_{2 k}=\\underbrace{a b c a b c \\ldots a b c}_{3 k \\text { letters }} \\underbrace{c b a c b a \\ldots c b a}_{3 k \\text { letters }} \\text { and } \\quad X_{2 k+3}=\\underbrace{a b c a b c \\ldots a b c}_{3 k \\text { letters }} a b c b c a c a b \\underbrace{c b a c b a \\ldots c b a}_{3 k \\text { letters }} . $$ We claim that for every $n \\geqslant 2$ and every chameleon $Y$, we have $d^{*}\\left(X_{n}, Y\\right) \\leqslant\\left\\lceil 3 n^{2} \/ 2\\right\\rceil$. This will mean that for every $n \\geqslant 2$ the number $3 n^{2} \/ 2$ in the problem statement cannot be changed by any number larger than $\\left\\lceil 3 n^{2} \/ 2\\right\\rceil$. For any distinct letters $u, v \\in\\{a, b, c\\}$ and any two chameleons $X$ and $Y$, we define $d_{u, v}^{*}(X, Y)$ as the number of $(X, Y)$-inversions $(s, t)$ such that $s$ and $t$ are instances of $u$ and $v$ (in any of the two possible orders). Then $d^{*}(X, Y)=d_{a, b}^{*}(X, Y)+d_{b, c}^{*}(X, Y)+d_{c, a}^{*}(X, Y)$. We start with the case when $n=2 k$ is even; denote $X=X_{2 k}$. We show that $d_{a, b}^{*}(X, Y) \\leqslant 2 k^{2}$ for any chameleon $Y$; this yields the required estimate. Proceed by the induction on $k$ with the trivial base case $k=0$. To perform the induction step, notice that $d_{a, b}^{*}(X, Y)$ is indeed the minimal number of swaps needed to change $Y_{\\bar{c}}$ into $X_{\\bar{c}}$. One may show that moving $a_{1}$ and $a_{2 k}$ in $Y$ onto the first and the last positions in $Y$, respectively, takes at most $2 k$ swaps, and that subsequent moving $b_{1}$ and $b_{2 k}$ onto the second and the second last positions takes at most $2 k-2$ swaps. After performing that, one may delete these letters from both $X_{\\bar{c}}$ and $Y_{\\bar{c}}$ and apply the induction hypothesis; so $X_{\\bar{c}}$ can be obtained from $Y_{\\bar{c}}$ using at most $2(k-1)^{2}+2 k+(2 k-2)=2 k^{2}$ swaps, as required. If $n=2 k+3$ is odd, the proof is similar but more technically involved. Namely, we claim that $d_{a, b}^{*}\\left(X_{2 k+3}, Y\\right) \\leqslant 2 k^{2}+6 k+5$ for any chameleon $Y$, and that the equality is achieved only if $Y_{\\bar{c}}=$ $b b \\ldots b a a \\ldots a$. The proof proceeds by a similar induction, with some care taken of the base case, as well as of extracting the equality case. Similar estimates hold for $d_{b, c}^{*}$ and $d_{c, a}^{*}$. Summing three such estimates, we obtain $$ d^{*}\\left(X_{2 k+3}, Y\\right) \\leqslant 3\\left(2 k^{2}+6 k+5\\right)=\\left\\lceil\\frac{3 n^{2}}{2}\\right\\rceil+1 $$ which is by 1 more than we need. But the equality could be achieved only if $Y_{\\bar{c}}=b b \\ldots b a a \\ldots a$ and, similarly, $Y_{\\bar{b}}=a a \\ldots a c c \\ldots c$ and $Y_{\\bar{a}}=c c \\ldots c b b \\ldots b$. Since these three equalities cannot hold simultaneously, the proof is finished.","tier":0} +{"problem_type":"Combinatorics","problem_label":"C3","problem":"Sir Alex plays the following game on a row of 9 cells. Initially, all cells are empty. In each move, Sir Alex is allowed to perform exactly one of the following two operations: (1) Choose any number of the form $2^{j}$, where $j$ is a non-negative integer, and put it into an empty cell. (2) Choose two (not necessarily adjacent) cells with the same number in them; denote that number by $2^{j}$. Replace the number in one of the cells with $2^{j+1}$ and erase the number in the other cell. At the end of the game, one cell contains the number $2^{n}$, where $n$ is a given positive integer, while the other cells are empty. Determine the maximum number of moves that Sir Alex could have made, in terms of $n$. (Thailand)","solution":"We will solve a more general problem, replacing the row of 9 cells with a row of $k$ cells, where $k$ is a positive integer. Denote by $m(n, k)$ the maximum possible number of moves Sir Alex can make starting with a row of $k$ empty cells, and ending with one cell containing the number $2^{n}$ and all the other $k-1$ cells empty. Call an operation of type (1) an insertion, and an operation of type (2) a merge. Only one move is possible when $k=1$, so we have $m(n, 1)=1$. From now on we consider $k \\geqslant 2$, and we may assume Sir Alex's last move was a merge. Then, just before the last move, there were exactly two cells with the number $2^{n-1}$, and the other $k-2$ cells were empty. Paint one of those numbers $2^{n-1}$ blue, and the other one red. Now trace back Sir Alex's moves, always painting the numbers blue or red following this rule: if $a$ and $b$ merge into $c$, paint $a$ and $b$ with the same color as $c$. Notice that in this backward process new numbers are produced only by reversing merges, since reversing an insertion simply means deleting one of the numbers. Therefore, all numbers appearing in the whole process will receive one of the two colors. Sir Alex's first move is an insertion. Without loss of generality, assume this first number inserted is blue. Then, from this point on, until the last move, there is always at least one cell with a blue number. Besides the last move, there is no move involving a blue and a red number, since all merges involves numbers with the same color, and insertions involve only one number. Call an insertion of a blue number or merge of two blue numbers a blue move, and define a red move analogously. The whole sequence of blue moves could be repeated on another row of $k$ cells to produce one cell with the number $2^{n-1}$ and all the others empty, so there are at most $m(n-1, k)$ blue moves. Now we look at the red moves. Since every time we perform a red move there is at least one cell occupied with a blue number, the whole sequence of red moves could be repeated on a row of $k-1$ cells to produce one cell with the number $2^{n-1}$ and all the others empty, so there are at most $m(n-1, k-1)$ red moves. This proves that $$ m(n, k) \\leqslant m(n-1, k)+m(n-1, k-1)+1 . $$ On the other hand, we can start with an empty row of $k$ cells and perform $m(n-1, k)$ moves to produce one cell with the number $2^{n-1}$ and all the others empty, and after that perform $m(n-1, k-1)$ moves on those $k-1$ empty cells to produce the number $2^{n-1}$ in one of them, leaving $k-2$ empty. With one more merge we get one cell with $2^{n}$ and the others empty, proving that $$ m(n, k) \\geqslant m(n-1, k)+m(n-1, k-1)+1 . $$ It follows that $$ m(n, k)=m(n-1, k)+m(n-1, k-1)+1, $$ for $n \\geqslant 1$ and $k \\geqslant 2$. If $k=1$ or $n=0$, we must insert $2^{n}$ on our first move and immediately get the final configuration, so $m(0, k)=1$ and $m(n, 1)=1$, for $n \\geqslant 0$ and $k \\geqslant 1$. These initial values, together with the recurrence relation (1), determine $m(n, k)$ uniquely. Finally, we show that $$ m(n, k)=2 \\sum_{j=0}^{k-1}\\binom{n}{j}-1 $$ for all integers $n \\geqslant 0$ and $k \\geqslant 1$. We use induction on $n$. Since $m(0, k)=1$ for $k \\geqslant 1,(2)$ is true for the base case. We make the induction hypothesis that (2) is true for some fixed positive integer $n$ and all $k \\geqslant 1$. We have $m(n+1,1)=1=2\\binom{n+1}{0}-1$, and for $k \\geqslant 2$ the recurrence relation (1) and the induction hypothesis give us $$ \\begin{aligned} & m(n+1, k)=m(n, k)+m(n, k-1)+1=2 \\sum_{j=0}^{k-1}\\binom{n}{j}-1+2 \\sum_{j=0}^{k-2}\\binom{n}{j}-1+1 \\\\ & \\quad=2 \\sum_{j=0}^{k-1}\\binom{n}{j}+2 \\sum_{j=0}^{k-1}\\binom{n}{j-1}-1=2 \\sum_{j=0}^{k-1}\\left(\\binom{n}{j}+\\binom{n}{j-1}\\right)-1=2 \\sum_{j=0}^{k-1}\\binom{n+1}{j}-1, \\end{aligned} $$ which completes the proof. Comment 1. After deducing the recurrence relation (1), it may be convenient to homogenize the recurrence relation by defining $h(n, k)=m(n, k)+1$. We get the new relation $$ h(n, k)=h(n-1, k)+h(n-1, k), $$ for $n \\geqslant 1$ and $k \\geqslant 2$, with initial values $h(0, k)=h(n, 1)=2$, for $n \\geqslant 0$ and $k \\geqslant 1$. This may help one to guess the answer, and also with other approaches like the one we develop next. Comment 2. We can use a generating function to find the answer without guessing. We work with the homogenized recurrence relation (3). Define $h(n, 0)=0$ so that (3) is valid for $k=1$ as well. Now we set up the generating function $f(x, y)=\\sum_{n, k \\geqslant 0} h(n, k) x^{n} y^{k}$. Multiplying the recurrence relation (3) by $x^{n} y^{k}$ and summing over $n, k \\geqslant 1$, we get $$ \\sum_{n, k \\geqslant 1} h(n, k) x^{n} y^{k}=x \\sum_{n, k \\geqslant 1} h(n-1, k) x^{n-1} y^{k}+x y \\sum_{n, k \\geqslant 1} h(n-1, k-1) x^{n-1} y^{k-1} . $$ Completing the missing terms leads to the following equation on $f(x, y)$ : $$ f(x, y)-\\sum_{n \\geqslant 0} h(n, 0) x^{n}-\\sum_{k \\geqslant 1} h(0, k) y^{k}=x f(x, y)-x \\sum_{n \\geqslant 0} h(n, 0) x^{n}+x y f(x, y) . $$ Substituting the initial values, we obtain $$ f(x, y)=\\frac{2 y}{1-y} \\cdot \\frac{1}{1-x(1+y)} . $$ Developing as a power series, we get $$ f(x, y)=2 \\sum_{j \\geqslant 1} y^{j} \\cdot \\sum_{n \\geqslant 0}(1+y)^{n} x^{n} . $$ The coefficient of $x^{n}$ in this power series is $$ 2 \\sum_{j \\geqslant 1} y^{j} \\cdot(1+y)^{n}=2 \\sum_{j \\geqslant 1} y^{j} \\cdot \\sum_{i \\geqslant 0}\\binom{n}{i} y^{i} $$ and extracting the coefficient of $y^{k}$ in this last expression we finally obtain the value for $h(n, k)$, $$ h(n, k)=2 \\sum_{j=0}^{k-1}\\binom{n}{j} $$ This proves that $$ m(n, k)=2 \\sum_{j=0}^{k-1}\\binom{n}{j}-1 $$ The generating function approach also works if applied to the non-homogeneous recurrence relation (1), but the computations are less straightforward.","tier":0} +{"problem_type":"Combinatorics","problem_label":"C3","problem":"Sir Alex plays the following game on a row of 9 cells. Initially, all cells are empty. In each move, Sir Alex is allowed to perform exactly one of the following two operations: (1) Choose any number of the form $2^{j}$, where $j$ is a non-negative integer, and put it into an empty cell. (2) Choose two (not necessarily adjacent) cells with the same number in them; denote that number by $2^{j}$. Replace the number in one of the cells with $2^{j+1}$ and erase the number in the other cell. At the end of the game, one cell contains the number $2^{n}$, where $n$ is a given positive integer, while the other cells are empty. Determine the maximum number of moves that Sir Alex could have made, in terms of $n$. (Thailand)","solution":"Define merges and insertions as in We will need the following lemma. Lemma. If the binary representation of a positive integer $A$ has $d$ nonzero digits, then $A$ cannot be represented as a sum of fewer than $d$ powers of 2 . Moreover, any representation of $A$ as a sum of $d$ powers of 2 must coincide with its binary representation. Proof. Let $s$ be the minimum number of summands in all possible representations of $A$ as sum of powers of 2 . Suppose there is such a representation with $s$ summands, where two of the summands are equal to each other. Then, replacing those two summands with the result of their sum, we obtain a representation with fewer than $s$ summands, which is a contradiction. We deduce that in any representation with $s$ summands, the summands are all distinct, so any such representation must coincide with the unique binary representation of $A$, and $s=d$. Now we split the solution into a sequence of claims. Claim 1. After every move, the number $S$ is the sum of at most $k-1$ distinct powers of 2 . Proof. If $S$ is the sum of $k$ (or more) distinct powers of 2 , the Lemma implies that the $k$ cells are filled with these numbers. This is a contradiction since no more merges or insertions can be made. Let $A(n, k-1)$ denote the set of all positive integers not exceeding $2^{n}$ with at most $k-1$ nonzero digits in its base 2 representation. Since every insertion increases the value of $S$, by Claim 1, the total number of insertions is at most $|A(n, k-1)|$. We proceed to prove that it is possible to achieve this number of insertions. Claim 2. Let $A(n, k-1)=\\left\\{a_{1}, a_{2}, \\ldots, a_{m}\\right\\}$, with $a_{1}$ | $\\boldsymbol{x}_{\\boldsymbol{N}, \\boldsymbol{N}}$ | | $\\vee$ | $\\vee$ | $\\vee$ | | $\\vee$ | $\\vee$ | | $x_{N+1,1}$ | $x_{N+1,2}$ | $x_{N+1,3}$ | $\\cdots$ | $x_{N+1, N-1}$ | $\\boldsymbol{x}_{\\boldsymbol{N + 1 , N}}$ | Now we make the bold choice: from the original row of people, remove everyone but those with heights $$ x_{1,1}>x_{2,1}>x_{2,2}>x_{3,2}>\\cdots>x_{N, N-1}>x_{N, N}>x_{N+1, N} $$ Of course this height order (*) is not necessarily their spatial order in the new row. We now need to convince ourselves that each pair $\\left(x_{k, k} ; x_{k+1, k}\\right)$ remains spatially together in this new row. But $x_{k, k}$ and $x_{k+1, k}$ belong to the same column\/block of consecutive $N+1$ people; the only people that could possibly stand between them were also in this block, and they are all gone.","tier":0} +{"problem_type":"Combinatorics","problem_label":"C4","problem":"Let $N \\geqslant 2$ be an integer. $N(N+1)$ soccer players, no two of the same height, stand in a row in some order. Coach Ralph wants to remove $N(N-1)$ people from this row so that in the remaining row of $2 N$ players, no one stands between the two tallest ones, no one stands between the third and the fourth tallest ones, ..., and finally no one stands between the two shortest ones. Show that this is always possible. (Russia)","solution":"Split the people into $N$ groups by height: group $G_{1}$ has the $N+1$ tallest ones, group $G_{2}$ has the next $N+1$ tallest, and so on, up to group $G_{N}$ with the $N+1$ shortest people. Now scan the original row from left to right, stopping as soon as you have scanned two people (consecutively or not) from the same group, say, $G_{i}$. Since we have $N$ groups, this must happen before or at the $(N+1)^{\\text {th }}$ person of the row. Choose this pair of people, removing all the other people from the same group $G_{i}$ and also all people that have been scanned so far. The only people that could separate this pair's heights were in group $G_{i}$ (and they are gone); the only people that could separate this pair's positions were already scanned (and they are gone too). We are now left with $N-1$ groups (all except $G_{i}$ ). Since each of them lost at most one person, each one has at least $N$ unscanned people left in the row. Repeat the scanning process from left to right, choosing the next two people from the same group, removing this group and everyone scanned up to that point. Once again we end up with two people who are next to each other in the remaining row and whose heights cannot be separated by anyone else who remains (since the rest of their group is gone). After picking these 2 pairs, we still have $N-2$ groups with at least $N-1$ people each. If we repeat the scanning process a total of $N$ times, it is easy to check that we will end up with 2 people from each group, for a total of $2 N$ people remaining. The height order is guaranteed by the grouping, and the scanning construction from left to right guarantees that each pair from a group stand next to each other in the final row. We are done.","tier":0} +{"problem_type":"Combinatorics","problem_label":"C4","problem":"Let $N \\geqslant 2$ be an integer. $N(N+1)$ soccer players, no two of the same height, stand in a row in some order. Coach Ralph wants to remove $N(N-1)$ people from this row so that in the remaining row of $2 N$ players, no one stands between the two tallest ones, no one stands between the third and the fourth tallest ones, ..., and finally no one stands between the two shortest ones. Show that this is always possible. (Russia)","solution":"This is essentially the same as solution 1, but presented inductively. The essence of the argument is the following lemma. Lemma. Assume that we have $N$ disjoint groups of at least $N+1$ people in each, all people have distinct heights. Then one can choose two people from each group so that among the chosen people, the two tallest ones are in one group, the third and the fourth tallest ones are in one group, ..., and the two shortest ones are in one group. Proof. Induction on $N \\geqslant 1$; for $N=1$, the statement is trivial. Consider now $N$ groups $G_{1}, \\ldots, G_{N}$ with at least $N+1$ people in each for $N \\geqslant 2$. Enumerate the people by $1,2, \\ldots, N(N+1)$ according to their height, say, from tallest to shortest. Find the least $s$ such that two people among $1,2, \\ldots, s$ are in one group (without loss of generality, say this group is $G_{N}$ ). By the minimality of $s$, the two mentioned people in $G_{N}$ are $s$ and some $i\\frac{1}{400} $$ In particular, $\\varepsilon^{2}+1=400 \\varepsilon$, so $$ y^{2}=d_{n}^{2}-2 \\varepsilon d_{n}+\\varepsilon^{2}+1=d_{n}^{2}+\\varepsilon\\left(400-2 d_{n}\\right) $$ Since $\\varepsilon>\\frac{1}{400}$ and we assumed $d_{n}<100$, this shows that $y^{2}>d_{n}^{2}+\\frac{1}{2}$. So, as we claimed, with this list of radar pings, no matter what the hunter does, the rabbit might achieve $d_{n+200}^{2}>d_{n}^{2}+\\frac{1}{2}$. The wabbit wins. Comment 1. Many different versions of the solution above can be found by replacing 200 with some other number $N$ for the number of hops the rabbit takes between reveals. If this is done, we have: $$ \\varepsilon=N-\\sqrt{N^{2}-1}>\\frac{1}{N+\\sqrt{N^{2}-1}}>\\frac{1}{2 N} $$ and $$ \\varepsilon^{2}+1=2 N \\varepsilon $$ so, as long as $N>d_{n}$, we would find $$ y^{2}=d_{n}^{2}+\\varepsilon\\left(2 N-2 d_{n}\\right)>d_{n}^{2}+\\frac{N-d_{n}}{N} $$ For example, taking $N=101$ is already enough-the squared distance increases by at least $\\frac{1}{101}$ every 101 rounds, and $101^{2} \\cdot 10^{4}=1.0201 \\cdot 10^{8}<10^{9}$ rounds are enough for the rabbit. If the statement is made sharper, some such versions might not work any longer. Comment 2. The original statement asked whether the distance could be kept under $10^{10}$ in $10^{100}$ rounds.","tier":0} +{"problem_type":"Combinatorics","problem_label":"C6","problem":"Let $n>1$ be an integer. An $n \\times n \\times n$ cube is composed of $n^{3}$ unit cubes. Each unit cube is painted with one color. For each $n \\times n \\times 1$ box consisting of $n^{2}$ unit cubes (of any of the three possible orientations), we consider the set of the colors present in that box (each color is listed only once). This way, we get $3 n$ sets of colors, split into three groups according to the orientation. It happens that for every set in any group, the same set appears in both of the other groups. Determine, in terms of $n$, the maximal possible number of colors that are present. (Russia)","solution":"Call a $n \\times n \\times 1$ box an $x$-box, a $y$-box, or a $z$-box, according to the direction of its short side. Let $C$ be the number of colors in a valid configuration. We start with the upper bound for $C$. Let $\\mathcal{C}_{1}, \\mathcal{C}_{2}$, and $\\mathcal{C}_{3}$ be the sets of colors which appear in the big cube exactly once, exactly twice, and at least thrice, respectively. Let $M_{i}$ be the set of unit cubes whose colors are in $\\mathcal{C}_{i}$, and denote $n_{i}=\\left|M_{i}\\right|$. Consider any $x$-box $X$, and let $Y$ and $Z$ be a $y$ - and a $z$-box containing the same set of colors as $X$ does. Claim. $4\\left|X \\cap M_{1}\\right|+\\left|X \\cap M_{2}\\right| \\leqslant 3 n+1$. Proof. We distinguish two cases. Case 1: $X \\cap M_{1} \\neq \\varnothing$. A cube from $X \\cap M_{1}$ should appear in all three boxes $X, Y$, and $Z$, so it should lie in $X \\cap Y \\cap Z$. Thus $X \\cap M_{1}=X \\cap Y \\cap Z$ and $\\left|X \\cap M_{1}\\right|=1$. Consider now the cubes in $X \\cap M_{2}$. There are at most $2(n-1)$ of them lying in $X \\cap Y$ or $X \\cap Z$ (because the cube from $X \\cap Y \\cap Z$ is in $M_{1}$ ). Let $a$ be some other cube from $X \\cap M_{2}$. Recall that there is just one other cube $a^{\\prime}$ sharing a color with $a$. But both $Y$ and $Z$ should contain such cube, so $a^{\\prime} \\in Y \\cap Z$ (but $a^{\\prime} \\notin X \\cap Y \\cap Z$ ). The map $a \\mapsto a^{\\prime}$ is clearly injective, so the number of cubes $a$ we are interested in does not exceed $|(Y \\cap Z) \\backslash X|=n-1$. Thus $\\left|X \\cap M_{2}\\right| \\leqslant 2(n-1)+(n-1)=3(n-1)$, and hence $4\\left|X \\cap M_{1}\\right|+\\left|X \\cap M_{2}\\right| \\leqslant 4+3(n-1)=3 n+1$. Case 2: $X \\cap M_{1}=\\varnothing$. In this case, the same argument applies with several changes. Indeed, $X \\cap M_{2}$ contains at most $2 n-1$ cubes from $X \\cap Y$ or $X \\cap Z$. Any other cube $a$ in $X \\cap M_{2}$ corresponds to some $a^{\\prime} \\in Y \\cap Z$ (possibly with $a^{\\prime} \\in X$ ), so there are at most $n$ of them. All this results in $\\left|X \\cap M_{2}\\right| \\leqslant(2 n-1)+n=3 n-1$, which is even better than we need (by the assumptions of our case). Summing up the inequalities from the Claim over all $x$-boxes $X$, we obtain $$ 4 n_{1}+n_{2} \\leqslant n(3 n+1) $$ Obviously, we also have $n_{1}+n_{2}+n_{3}=n^{3}$. Now we are prepared to estimate $C$. Due to the definition of the $M_{i}$, we have $n_{i} \\geqslant i\\left|\\mathcal{C}_{i}\\right|$, so $$ C \\leqslant n_{1}+\\frac{n_{2}}{2}+\\frac{n_{3}}{3}=\\frac{n_{1}+n_{2}+n_{3}}{3}+\\frac{4 n_{1}+n_{2}}{6} \\leqslant \\frac{n^{3}}{3}+\\frac{3 n^{2}+n}{6}=\\frac{n(n+1)(2 n+1)}{6} $$ It remains to present an example of an appropriate coloring in the above-mentioned number of colors. For each color, we present the set of all cubes of this color. These sets are: 1. $n$ singletons of the form $S_{i}=\\{(i, i, i)\\}$ (with $1 \\leqslant i \\leqslant n$ ); 2. $3\\binom{n}{2}$ doubletons of the forms $D_{i, j}^{1}=\\{(i, j, j),(j, i, i)\\}, D_{i, j}^{2}=\\{(j, i, j),(i, j, i)\\}$, and $D_{i, j}^{3}=$ $\\{(j, j, i),(i, i, j)\\}$ (with $1 \\leqslant i1$ be an integer. An $n \\times n \\times n$ cube is composed of $n^{3}$ unit cubes. Each unit cube is painted with one color. For each $n \\times n \\times 1$ box consisting of $n^{2}$ unit cubes (of any of the three possible orientations), we consider the set of the colors present in that box (each color is listed only once). This way, we get $3 n$ sets of colors, split into three groups according to the orientation. It happens that for every set in any group, the same set appears in both of the other groups. Determine, in terms of $n$, the maximal possible number of colors that are present. (Russia)","solution":"We will approach a new version of the original problem. In this new version, each cube may have a color, or be invisible (not both). Now we make sets of colors for each $n \\times n \\times 1$ box as before (where \"invisible\" is not considered a color) and group them by orientation, also as before. Finally, we require that, for every non-empty set in any group, the same set must appear in the other 2 groups. What is the maximum number of colors present with these new requirements? Let us call strange a big $n \\times n \\times n$ cube whose painting scheme satisfies the new requirements, and let $D$ be the number of colors in a strange cube. Note that any cube that satisfies the original requirements is also strange, so $\\max (D)$ is an upper bound for the original answer. Claim. $D \\leqslant \\frac{n(n+1)(2 n+1)}{6}$. Proof. The proof is by induction on $n$. If $n=1$, we must paint the cube with at most 1 color. Now, pick a $n \\times n \\times n$ strange cube $A$, where $n \\geqslant 2$. If $A$ is completely invisible, $D=0$ and we are done. Otherwise, pick a non-empty set of colors $\\mathcal{S}$ which corresponds to, say, the boxes $X, Y$ and $Z$ of different orientations. Now find all cubes in $A$ whose colors are in $\\mathcal{S}$ and make them invisible. Since $X, Y$ and $Z$ are now completely invisible, we can throw them away and focus on the remaining $(n-1) \\times(n-1) \\times(n-1)$ cube $B$. The sets of colors in all the groups for $B$ are the same as the sets for $A$, removing exactly the colors in $\\mathcal{S}$, and no others! Therefore, every nonempty set that appears in one group for $B$ still shows up in all possible orientations (it is possible that an empty set of colors in $B$ only matched $X, Y$ or $Z$ before these were thrown away, but remember we do not require empty sets to match anyway). In summary, $B$ is also strange. By the induction hypothesis, we may assume that $B$ has at most $\\frac{(n-1) n(2 n-1)}{6}$ colors. Since there were at most $n^{2}$ different colors in $\\mathcal{S}$, we have that $A$ has at most $\\frac{(n-1) n(2 n-1)}{6}+n^{2}=$ $\\frac{n(n+1)(2 n+1)}{6}$ colors. Finally, the construction in the previous solution shows a painting scheme (with no invisible cubes) that reaches this maximum, so we are done.","tier":0} +{"problem_type":"Combinatorics","problem_label":"C7","problem":"For any finite sets $X$ and $Y$ of positive integers, denote by $f_{X}(k)$ the $k^{\\text {th }}$ smallest positive integer not in $X$, and let $$ X * Y=X \\cup\\left\\{f_{X}(y): y \\in Y\\right\\} $$ Let $A$ be a set of $a>0$ positive integers, and let $B$ be a set of $b>0$ positive integers. Prove that if $A * B=B * A$, then $$ \\underbrace{A *(A * \\cdots *(A *(A * A)) \\ldots)}_{A \\text { appears } b \\text { times }}=\\underbrace{B *(B * \\cdots *(B *(B * B)) \\ldots)}_{B \\text { appears } a \\text { times }} . $$","solution":"For any function $g: \\mathbb{Z}_{>0} \\rightarrow \\mathbb{Z}_{>0}$ and any subset $X \\subset \\mathbb{Z}_{>0}$, we define $g(X)=$ $\\{g(x): x \\in X\\}$. We have that the image of $f_{X}$ is $f_{X}\\left(\\mathbb{Z}_{>0}\\right)=\\mathbb{Z}_{>0} \\backslash X$. We now show a general lemma about the operation *, with the goal of showing that $*$ is associative. Lemma 1. Let $X$ and $Y$ be finite sets of positive integers. The functions $f_{X * Y}$ and $f_{X} \\circ f_{Y}$ are equal. Proof. We have $f_{X * Y}\\left(\\mathbb{Z}_{>0}\\right)=\\mathbb{Z}_{>0} \\backslash(X * Y)=\\left(\\mathbb{Z}_{>0} \\backslash X\\right) \\backslash f_{X}(Y)=f_{X}\\left(\\mathbb{Z}_{>0}\\right) \\backslash f_{X}(Y)=f_{X}\\left(\\mathbb{Z}_{>0} \\backslash Y\\right)=f_{X}\\left(f_{Y}\\left(\\mathbb{Z}_{>0}\\right)\\right)$. Thus, the functions $f_{X * Y}$ and $f_{X} \\circ f_{Y}$ are strictly increasing functions with the same range. Because a strictly function is uniquely defined by its range, we have $f_{X * Y}=f_{X} \\circ f_{Y}$. Lemma 1 implies that * is associative, in the sense that $(A * B) * C=A *(B * C)$ for any finite sets $A, B$, and $C$ of positive integers. We prove the associativity by noting $$ \\begin{gathered} \\mathbb{Z}_{>0} \\backslash((A * B) * C)=f_{(A * B) * C}\\left(\\mathbb{Z}_{>0}\\right)=f_{A * B}\\left(f_{C}\\left(\\mathbb{Z}_{>0}\\right)\\right)=f_{A}\\left(f_{B}\\left(f_{C}\\left(\\mathbb{Z}_{>0}\\right)\\right)\\right) \\\\ =f_{A}\\left(f_{B * C}\\left(\\mathbb{Z}_{>0}\\right)=f_{A *(B * C)}\\left(\\mathbb{Z}_{>0}\\right)=\\mathbb{Z}_{>0} \\backslash(A *(B * C))\\right. \\end{gathered} $$ In light of the associativity of *, we may drop the parentheses when we write expressions like $A *(B * C)$. We also introduce the notation $$ X^{* k}=\\underbrace{X *(X * \\cdots *(X *(X * X)) \\ldots)}_{X \\text { appears } k \\text { times }} $$ Our goal is then to show that $A * B=B * A$ implies $A^{* b}=B^{* a}$. We will do so via the following general lemma. Lemma 2. Suppose that $X$ and $Y$ are finite sets of positive integers satisfying $X * Y=Y * X$ and $|X|=|Y|$. Then, we must have $X=Y$. Proof. Assume that $X$ and $Y$ are not equal. Let $s$ be the largest number in exactly one of $X$ and $Y$. Without loss of generality, say that $s \\in X \\backslash Y$. The number $f_{X}(s)$ counts the $s^{t h}$ number not in $X$, which implies that $$ f_{X}(s)=s+\\left|X \\cap\\left\\{1,2, \\ldots, f_{X}(s)\\right\\}\\right| $$ Since $f_{X}(s) \\geqslant s$, we have that $$ \\left\\{f_{X}(s)+1, f_{X}(s)+2, \\ldots\\right\\} \\cap X=\\left\\{f_{X}(s)+1, f_{X}(s)+2, \\ldots\\right\\} \\cap Y $$ which, together with the assumption that $|X|=|Y|$, gives $$ \\left|X \\cap\\left\\{1,2, \\ldots, f_{X}(s)\\right\\}\\right|=\\left|Y \\cap\\left\\{1,2, \\ldots, f_{X}(s)\\right\\}\\right| $$ Now consider the equation $$ t-|Y \\cap\\{1,2, \\ldots, t\\}|=s $$ This equation is satisfied only when $t \\in\\left[f_{Y}(s), f_{Y}(s+1)\\right)$, because the left hand side counts the number of elements up to $t$ that are not in $Y$. We have that the value $t=f_{X}(s)$ satisfies the above equation because of (1) and (2). Furthermore, since $f_{X}(s) \\notin X$ and $f_{X}(s) \\geqslant s$, we have that $f_{X}(s) \\notin Y$ due to the maximality of $s$. Thus, by the above discussion, we must have $f_{X}(s)=f_{Y}(s)$. Finally, we arrive at a contradiction. The value $f_{X}(s)$ is neither in $X$ nor in $f_{X}(Y)$, because $s$ is not in $Y$ by assumption. Thus, $f_{X}(s) \\notin X * Y$. However, since $s \\in X$, we have $f_{Y}(s) \\in Y * X$, a contradiction. We are now ready to finish the proof. Note first of all that $\\left|A^{* b}\\right|=a b=\\left|B^{* a}\\right|$. Moreover, since $A * B=B * A$, and $*$ is associative, it follows that $A^{* b} * B^{* a}=B^{* a} * A^{* b}$. Thus, by Lemma 2, we have $A^{* b}=B^{* a}$, as desired. Comment 1. Taking $A=X^{* k}$ and $B=X^{* l}$ generates many non-trivial examples where $A * B=B * A$. There are also other examples not of this form. For example, if $A=\\{1,2,4\\}$ and $B=\\{1,3\\}$, then $A * B=\\{1,2,3,4,6\\}=B * A$.","tier":0} +{"problem_type":"Combinatorics","problem_label":"C7","problem":"For any finite sets $X$ and $Y$ of positive integers, denote by $f_{X}(k)$ the $k^{\\text {th }}$ smallest positive integer not in $X$, and let $$ X * Y=X \\cup\\left\\{f_{X}(y): y \\in Y\\right\\} $$ Let $A$ be a set of $a>0$ positive integers, and let $B$ be a set of $b>0$ positive integers. Prove that if $A * B=B * A$, then $$ \\underbrace{A *(A * \\cdots *(A *(A * A)) \\ldots)}_{A \\text { appears } b \\text { times }}=\\underbrace{B *(B * \\cdots *(B *(B * B)) \\ldots)}_{B \\text { appears } a \\text { times }} . $$","solution":"We will use Lemma 1 from $$ f_{X}=f_{Y} \\Longleftrightarrow f_{X}\\left(\\mathbb{Z}_{>0}\\right)=f_{Y}\\left(\\mathbb{Z}_{>0}\\right) \\Longleftrightarrow\\left(\\mathbb{Z}_{>0} \\backslash X\\right)=\\left(\\mathbb{Z}_{>0} \\backslash Y\\right) \\Longleftrightarrow X=Y, $$ where the first equivalence is because $f_{X}$ and $f_{Y}$ are strictly increasing functions, and the second equivalence is because $f_{X}\\left(\\mathbb{Z}_{>0}\\right)=\\mathbb{Z}_{>0} \\backslash X$ and $f_{Y}\\left(\\mathbb{Z}_{>0}\\right)=\\mathbb{Z}_{>0} \\backslash Y$. Denote $g=f_{A}$ and $h=f_{B}$. The given relation $A * B=B * A$ is equivalent to $f_{A * B}=f_{B * A}$ because of (3), and by Lemma 1 of the first solution, this is equivalent to $g \\circ h=h \\circ g$. Similarly, the required relation $A^{* b}=B^{* a}$ is equivalent to $g^{b}=h^{a}$. We will show that $$ g^{b}(n)=h^{a}(n) $$ for all $n \\in \\mathbb{Z}_{>0}$, which suffices to solve the problem. To start, we claim that (4) holds for all sufficiently large $n$. Indeed, let $p$ and $q$ be the maximal elements of $A$ and $B$, respectively; we may assume that $p \\geqslant q$. Then, for every $n \\geqslant p$ we have $g(n)=n+a$ and $h(n)=n+b$, whence $g^{b}(n)=n+a b=h^{a}(n)$, as was claimed. In view of this claim, if (4) is not identically true, then there exists a maximal $s$ with $g^{b}(s) \\neq$ $h^{a}(s)$. Without loss of generality, we may assume that $g(s) \\neq s$, for if we had $g(s)=h(s)=s$, then $s$ would satisfy (4). As $g$ is increasing, we then have $g(s)>s$, so (4) holds for $n=g(s)$. But then we have $$ g\\left(g^{b}(s)\\right)=g^{b+1}(s)=g^{b}(n)=h^{a}(n)=h^{a}(g(s))=g\\left(h^{a}(s)\\right), $$ where the last equality holds in view of $g \\circ h=h \\circ g$. By the injectivity of $g$, the above equality yields $g^{b}(s)=h^{a}(s)$, which contradicts the choice of $s$. Thus, we have proved that (4) is identically true on $\\mathbb{Z}_{>0}$, as desired. Comment 2. We present another proof of Lemma 2 of the first solution. Let $x=|X|=|Y|$. Say that $u$ is the smallest number in $X$ and $v$ is the smallest number in $Y$; assume without loss of generality that $u \\leqslant v$. Let $T$ be any finite set of positive integers, and define $t=|T|$. Enumerate the elements of $X$ as $x_{1}m_{i}$, we have that $s_{m, i}=t+m n-c_{i}$. Furthermore, the $c_{i}$ do not depend on the choice of $T$. First, we show that this claim implies Lemma 2. We may choose $T=X$ and $T=Y$. Then, there is some $m^{\\prime}$ such that for all $m \\geqslant m^{\\prime}$, we have $$ f_{X * m}(X)=f_{(Y * X *(m-1))}(X) $$ Because $u$ is the minimum element of $X, v$ is the minimum element of $Y$, and $u \\leqslant v$, we have that $$ \\left(\\bigcup_{m=m^{\\prime}}^{\\infty} f_{X * m}(X)\\right) \\cup X^{* m^{\\prime}}=\\left(\\bigcup_{m=m^{\\prime}}^{\\infty} f_{\\left(Y * X^{*(m-1)}\\right)}(X)\\right) \\cup\\left(Y * X^{*\\left(m^{\\prime}-1\\right)}\\right)=\\{u, u+1, \\ldots\\}, $$ and in both the first and second expressions, the unions are of pairwise distinct sets. By (5), we obtain $X^{* m^{\\prime}}=Y * X^{*\\left(m^{\\prime}-1\\right)}$. Now, because $X$ and $Y$ commute, we get $X^{* m^{\\prime}}=X^{*\\left(m^{\\prime}-1\\right)} * Y$, and so $X=Y$. We now prove the claim. Proof of the claim. We induct downwards on $i$, first proving the statement for $i=n$, and so on. Assume that $m$ is chosen so that all elements of $S_{m}$ are greater than all elements of $T$ (which is possible because $T$ is finite). For $i=n$, we have that $s_{m, n}>s_{k, n}$ for every $ki$ and $pi$ eventually do not depend on $T$, the sequence $s_{m, i}-t$ eventually does not depend on $T$ either, so the inductive step is complete. This proves the claim and thus Lemma 2.","tier":0} +{"problem_type":"Combinatorics","problem_label":"C8","problem":"Let $n$ be a given positive integer. In the Cartesian plane, each lattice point with nonnegative coordinates initially contains a butterfly, and there are no other butterflies. The neighborhood of a lattice point $c$ consists of all lattice points within the axis-aligned $(2 n+1) \\times(2 n+1)$ square centered at $c$, apart from $c$ itself. We call a butterfly lonely, crowded, or comfortable, depending on whether the number of butterflies in its neighborhood $N$ is respectively less than, greater than, or equal to half of the number of lattice points in $N$. Every minute, all lonely butterflies fly away simultaneously. This process goes on for as long as there are any lonely butterflies. Assuming that the process eventually stops, determine the number of comfortable butterflies at the final state.","solution":"We always identify a butterfly with the lattice point it is situated at. For two points $p$ and $q$, we write $p \\geqslant q$ if each coordinate of $p$ is at least the corresponding coordinate of $q$. Let $O$ be the origin, and let $\\mathcal{Q}$ be the set of initially occupied points, i.e., of all lattice points with nonnegative coordinates. Let $\\mathcal{R}_{\\mathrm{H}}=\\{(x, 0): x \\geqslant 0\\}$ and $\\mathcal{R}_{\\mathrm{V}}=\\{(0, y): y \\geqslant 0\\}$ be the sets of the lattice points lying on the horizontal and vertical boundary rays of $\\mathcal{Q}$. Denote by $N(a)$ the neighborhood of a lattice point $a$. 1. Initial observations. We call a set of lattice points up-right closed if its points stay in the set after being shifted by any lattice vector $(i, j)$ with $i, j \\geqslant 0$. Whenever the butterflies form a up-right closed set $\\mathcal{S}$, we have $|N(p) \\cap \\mathcal{S}| \\geqslant|N(q) \\cap \\mathcal{S}|$ for any two points $p, q \\in \\mathcal{S}$ with $p \\geqslant q$. So, since $\\mathcal{Q}$ is up-right closed, the set of butterflies at any moment also preserves this property. We assume all forthcoming sets of lattice points to be up-right closed. When speaking of some set $\\mathcal{S}$ of lattice points, we call its points lonely, comfortable, or crowded with respect to this set (i.e., as if the butterflies were exactly at all points of $\\mathcal{S}$ ). We call a set $\\mathcal{S} \\subset \\mathcal{Q}$ stable if it contains no lonely points. In what follows, we are interested only in those stable sets whose complements in $\\mathcal{Q}$ are finite, because one can easily see that only a finite number of butterflies can fly away on each minute. If the initial set $\\mathcal{Q}$ of butterflies contains some stable set $\\mathcal{S}$, then, clearly no butterfly of this set will fly away. On the other hand, the set $\\mathcal{F}$ of all butterflies in the end of the process is stable. This means that $\\mathcal{F}$ is the largest (with respect to inclusion) stable set within $\\mathcal{Q}$, and we are about to describe this set. 2. A description of a final set. The following notion will be useful. Let $\\mathcal{U}=\\left\\{\\vec{u}_{1}, \\vec{u}_{2}, \\ldots, \\vec{u}_{d}\\right\\}$ be a set of $d$ pairwise non-parallel lattice vectors, each having a positive $x$ - and a negative $y$-coordinate. Assume that they are numbered in increasing order according to slope. We now define a $\\mathcal{U}$-curve to be the broken line $p_{0} p_{1} \\ldots p_{d}$ such that $p_{0} \\in \\mathcal{R}_{\\mathrm{V}}, p_{d} \\in \\mathcal{R}_{\\mathrm{H}}$, and $\\overrightarrow{p_{i-1} p_{i}}=\\vec{u}_{i}$ for all $i=1,2, \\ldots, m$ (see the Figure below to the left). ![](https:\/\/cdn.mathpix.com\/cropped\/2024_11_18_ff08e3c077fa2f8c16b2g-53.jpg?height=546&width=766&top_left_y=2194&top_left_x=191) Construction of $\\mathcal{U}$-curve ![](https:\/\/cdn.mathpix.com\/cropped\/2024_11_18_ff08e3c077fa2f8c16b2g-53.jpg?height=415&width=803&top_left_y=2322&top_left_x=1066) Construction of $\\mathcal{D}$ Now, let $\\mathcal{K}_{n}=\\{(i, j): 1 \\leqslant i \\leqslant n,-n \\leqslant j \\leqslant-1\\}$. Consider all the rays emerging at $O$ and passing through a point from $\\mathcal{K}_{n}$; number them as $r_{1}, \\ldots, r_{m}$ in increasing order according to slope. Let $A_{i}$ be the farthest from $O$ lattice point in $r_{i} \\cap \\mathcal{K}_{n}$, set $k_{i}=\\left|r_{i} \\cap \\mathcal{K}_{n}\\right|$, let $\\vec{v}_{i}=\\overrightarrow{O A_{i}}$, and finally denote $\\mathcal{V}=\\left\\{\\vec{v}_{i}: 1 \\leqslant i \\leqslant m\\right\\}$; see the Figure above to the right. We will concentrate on the $\\mathcal{V}$-curve $d_{0} d_{1} \\ldots d_{m}$; let $\\mathcal{D}$ be the set of all lattice points $p$ such that $p \\geqslant p^{\\prime}$ for some (not necessarily lattice) point $p^{\\prime}$ on the $\\mathcal{V}$-curve. In fact, we will show that $\\mathcal{D}=\\mathcal{F}$. Clearly, the $\\mathcal{V}$-curve is symmetric in the line $y=x$. Denote by $D$ the convex hull of $\\mathcal{D}$. 3. We prove that the set $\\mathcal{D}$ contains all stable sets. Let $\\mathcal{S} \\subset \\mathcal{Q}$ be a stable set (recall that it is assumed to be up-right closed and to have a finite complement in $\\mathcal{Q}$ ). Denote by $S$ its convex hull; clearly, the vertices of $S$ are lattice points. The boundary of $S$ consists of two rays (horizontal and vertical ones) along with some $\\mathcal{V}_{*}$-curve for some set of lattice vectors $\\mathcal{V}_{*}$. Claim 1. For every $\\vec{v}_{i} \\in \\mathcal{V}$, there is a $\\vec{v}_{i}^{*} \\in \\mathcal{V}_{*}$ co-directed with $\\vec{v}$ with $\\left|\\vec{v}_{i}^{*}\\right| \\geqslant|\\vec{v}|$. Proof. Let $\\ell$ be the supporting line of $S$ parallel to $\\vec{v}_{i}$ (i.e., $\\ell$ contains some point of $S$, and the set $S$ lies on one side of $\\ell$ ). Take any point $b \\in \\ell \\cap \\mathcal{S}$ and consider $N(b)$. The line $\\ell$ splits the set $N(b) \\backslash \\ell$ into two congruent parts, one having an empty intersection with $\\mathcal{S}$. Hence, in order for $b$ not to be lonely, at least half of the set $\\ell \\cap N(b)$ (which contains $2 k_{i}$ points) should lie in $S$. Thus, the boundary of $S$ contains a segment $\\ell \\cap S$ with at least $k_{i}+1$ lattice points (including $b$ ) on it; this segment corresponds to the required vector $\\vec{v}_{i}^{*} \\in \\mathcal{V}_{*}$. ![](https:\/\/cdn.mathpix.com\/cropped\/2024_11_18_ff08e3c077fa2f8c16b2g-54.jpg?height=600&width=1626&top_left_y=1199&top_left_x=221) Claim 2. Each stable set $\\mathcal{S} \\subseteq \\mathcal{Q}$ lies in $\\mathcal{D}$. Proof. To show this, it suffices to prove that the $\\mathcal{V}_{*}$-curve lies in $D$, i.e., that all its vertices do so. Let $p^{\\prime}$ be an arbitrary vertex of the $\\mathcal{V}_{*^{-}}$-curve; $p^{\\prime}$ partitions this curve into two parts, $\\mathcal{X}$ (being down-right of $p$ ) and $\\mathcal{Y}$ (being up-left of $p$ ). The set $\\mathcal{V}$ is split now into two parts: $\\mathcal{V}_{\\mathcal{X}}$ consisting of those $\\vec{v}_{i} \\in \\mathcal{V}$ for which $\\vec{v}_{i}^{*}$ corresponds to a segment in $\\mathcal{X}$, and a similar part $\\mathcal{V}_{\\mathcal{Y}}$. Notice that the $\\mathcal{V}$-curve consists of several segments corresponding to $\\mathcal{V}_{\\mathcal{X}}$, followed by those corresponding to $\\mathcal{V}_{\\mathcal{Y}}$. Hence there is a vertex $p$ of the $\\mathcal{V}$-curve separating $\\mathcal{V}_{\\mathcal{X}}$ from $\\mathcal{V}_{\\mathcal{Y}}$. Claim 1 now yields that $p^{\\prime} \\geqslant p$, so $p^{\\prime} \\in \\mathcal{D}$, as required. Claim 2 implies that the final set $\\mathcal{F}$ is contained in $\\mathcal{D}$. 4. $\\mathcal{D}$ is stable, and its comfortable points are known. Recall the definitions of $r_{i}$; let $r_{i}^{\\prime}$ be the ray complementary to $r_{i}$. By our definitions, the set $N(O)$ contains no points between the rays $r_{i}$ and $r_{i+1}$, as well as between $r_{i}^{\\prime}$ and $r_{i+1}^{\\prime}$. Claim 3. In the set $\\mathcal{D}$, all lattice points of the $\\mathcal{V}$-curve are comfortable. Proof. Let $p$ be any lattice point of the $\\mathcal{V}$-curve, belonging to some segment $d_{i} d_{i+1}$. Draw the line $\\ell$ containing this segment. Then $\\ell \\cap \\mathcal{D}$ contains exactly $k_{i}+1$ lattice points, all of which lie in $N(p)$ except for $p$. Thus, exactly half of the points in $N(p) \\cap \\ell$ lie in $\\mathcal{D}$. It remains to show that all points of $N(p)$ above $\\ell$ lie in $\\mathcal{D}$ (recall that all the points below $\\ell$ lack this property). Notice that each vector in $\\mathcal{V}$ has one coordinate greater than $n \/ 2$; thus the neighborhood of $p$ contains parts of at most two segments of the $\\mathcal{V}$-curve succeeding $d_{i} d_{i+1}$, as well as at most two of those preceding it. The angles formed by these consecutive segments are obtained from those formed by $r_{j}$ and $r_{j-1}^{\\prime}$ (with $i-1 \\leqslant j \\leqslant i+2$ ) by shifts; see the Figure below. All the points in $N(p)$ above $\\ell$ which could lie outside $\\mathcal{D}$ lie in shifted angles between $r_{j}, r_{j+1}$ or $r_{j}^{\\prime}, r_{j-1}^{\\prime}$. But those angles, restricted to $N(p)$, have no lattice points due to the above remark. The claim is proved. ![](https:\/\/cdn.mathpix.com\/cropped\/2024_11_18_ff08e3c077fa2f8c16b2g-55.jpg?height=404&width=1052&top_left_y=592&top_left_x=503) Proof of Claim 3 Claim 4. All the points of $\\mathcal{D}$ which are not on the boundary of $D$ are crowded. Proof. Let $p \\in \\mathcal{D}$ be such a point. If it is to the up-right of some point $p^{\\prime}$ on the curve, then the claim is easy: the shift of $N\\left(p^{\\prime}\\right) \\cap \\mathcal{D}$ by $\\overrightarrow{p^{\\prime} p}$ is still in $\\mathcal{D}$, and $N(p)$ contains at least one more point of $\\mathcal{D}$ - either below or to the left of $p$. So, we may assume that $p$ lies in a right triangle constructed on some hypothenuse $d_{i} d_{i+1}$. Notice here that $d_{i}, d_{i+1} \\in N(p)$. Draw a line $\\ell \\| d_{i} d_{i+1}$ through $p$, and draw a vertical line $h$ through $d_{i}$; see Figure below. Let $\\mathcal{D}_{\\mathrm{L}}$ and $\\mathcal{D}_{\\mathrm{R}}$ be the parts of $\\mathcal{D}$ lying to the left and to the right of $h$, respectively (points of $\\mathcal{D} \\cap h$ lie in both parts). ![](https:\/\/cdn.mathpix.com\/cropped\/2024_11_18_ff08e3c077fa2f8c16b2g-55.jpg?height=538&width=1120&top_left_y=1556&top_left_x=468) Notice that the vectors $\\overrightarrow{d_{i} p}, \\overrightarrow{d_{i+1} d_{i+2}}, \\overrightarrow{d_{i} d_{i+1}}, \\overrightarrow{d_{i-1} d_{i}}$, and $\\overrightarrow{p d_{i+1}}$ are arranged in non-increasing order by slope. This means that $\\mathcal{D}_{\\mathrm{L}}$ shifted by $\\overrightarrow{d_{i} p}$ still lies in $\\mathcal{D}$, as well as $\\mathcal{D}_{\\mathrm{R}}$ shifted by $\\overrightarrow{d_{i+1} p}$. As we have seen in the proof of Claim 3, these two shifts cover all points of $N(p)$ above $\\ell$, along with those on $\\ell$ to the left of $p$. Since $N(p)$ contains also $d_{i}$ and $d_{i+1}$, the point $p$ is crowded. Thus, we have proved that $\\mathcal{D}=\\mathcal{F}$, and have shown that the lattice points on the $\\mathcal{V}$-curve are exactly the comfortable points of $\\mathcal{D}$. It remains to find their number. Recall the definition of $\\mathcal{K}_{n}$ (see Figure on the first page of the solution). Each segment $d_{i} d_{i+1}$ contains $k_{i}$ lattice points different from $d_{i}$. Taken over all $i$, these points exhaust all the lattice points in the $\\mathcal{V}$-curve, except for $d_{1}$, and thus the number of lattice points on the $\\mathcal{V}$-curve is $1+\\sum_{i=1}^{m} k_{i}$. On the other hand, $\\sum_{i=1}^{m} k_{i}$ is just the number of points in $\\mathcal{K}_{n}$, so it equals $n^{2}$. Hence the answer to the problem is $n^{2}+1$. Comment 1. The assumption that the process eventually stops is unnecessary for the problem, as one can see that, in fact, the process stops for every $n \\geqslant 1$. Indeed, the proof of Claims 3 and 4 do not rely essentially on this assumption, and they together yield that the set $\\mathcal{D}$ is stable. So, only butterflies that are not in $\\mathcal{D}$ may fly away, and this takes only a finite time. This assumption has been inserted into the problem statement in order to avoid several technical details regarding finiteness issues. It may also simplify several other arguments. Comment 2. The description of the final set $\\mathcal{F}(=\\mathcal{D})$ seems to be crucial for the solution; the Problem Selection Committee is not aware of any solution that completely avoids such a description. On the other hand, after the set $\\mathcal{D}$ has been defined, the further steps may be performed in several ways. For example, in order to prove that all butterflies outside $\\mathcal{D}$ will fly away, one may argue as follows. (Here we will also make use of the assumption that the process eventually stops.) First of all, notice that the process can be modified in the following manner: Each minute, exactly one of the lonely butterflies flies away, until there are no more lonely butterflies. The modified process necessarily stops at the same state as the initial one. Indeed, one may observe, as in solution above, that the (unique) largest stable set is still the final set for the modified process. Thus, in order to prove our claim, it suffices to indicate an order in which the butterflies should fly away in the new process; if we are able to exhaust the whole set $\\mathcal{Q} \\backslash \\mathcal{D}$, we are done. Let $\\mathcal{C}_{0}=d_{0} d_{1} \\ldots d_{m}$ be the $\\mathcal{V}$-curve. Take its copy $\\mathcal{C}$ and shift it downwards so that $d_{0}$ comes to some point below the origin $O$. Now we start moving $\\mathcal{C}$ upwards continuously, until it comes back to its initial position $\\mathcal{C}_{0}$. At each moment when $\\mathcal{C}$ meets some lattice points, we convince all the butterflies at those points to fly away in a certain order. We will now show that we always have enough arguments for butterflies to do so, which will finish our argument for the claim.. Let $\\mathcal{C}^{\\prime}=d_{0}^{\\prime} d_{1}^{\\prime} \\ldots d_{m}^{\\prime}$ be a position of $\\mathcal{C}$ when it meets some butterflies. We assume that all butterflies under this current position of $\\mathcal{C}$ were already convinced enough and flied away. Consider the lowest butterfly $b$ on $\\mathcal{C}^{\\prime}$. Let $d_{i}^{\\prime} d_{i+1}^{\\prime}$ be the segment it lies on; we choose $i$ so that $b \\neq d_{i+1}^{\\prime}$ (this is possible because $\\mathcal{C}$ as not yet reached $\\mathcal{C}_{0}$ ). Draw a line $\\ell$ containing the segment $d_{i}^{\\prime} d_{i+1}^{\\prime}$. Then all the butterflies in $N(b)$ are situated on or above $\\ell$; moreover, those on $\\ell$ all lie on the segment $d_{i} d_{i+1}$. But this segment now contains at most $k_{i}$ butterflies (including $b$ ), since otherwise some butterfly had to occupy $d_{i+1}^{\\prime}$ which is impossible by the choice of $b$. Thus, $b$ is lonely and hence may be convinced to fly away. After $b$ has flied away, we switch to the lowest of the remaining butterflies on $\\mathcal{C}^{\\prime}$, and so on. Claims 3 and 4 also allow some different proofs which are not presented here. This page is intentionally left blank","tier":0} +{"problem_type":"Geometry","problem_label":"G1","problem":"Let $A B C D E$ be a convex pentagon such that $A B=B C=C D, \\angle E A B=\\angle B C D$, and $\\angle E D C=\\angle C B A$. Prove that the perpendicular line from $E$ to $B C$ and the line segments $A C$ and $B D$ are concurrent. (Italy)","solution":"Throughout the solution, we refer to $\\angle A, \\angle B, \\angle C, \\angle D$, and $\\angle E$ as internal angles of the pentagon $A B C D E$. Let the perpendicular bisectors of $A C$ and $B D$, which pass respectively through $B$ and $C$, meet at point $I$. Then $B D \\perp C I$ and, similarly, $A C \\perp B I$. Hence $A C$ and $B D$ meet at the orthocenter $H$ of the triangle $B I C$, and $I H \\perp B C$. It remains to prove that $E$ lies on the line $I H$ or, equivalently, $E I \\perp B C$. Lines $I B$ and $I C$ bisect $\\angle B$ and $\\angle C$, respectively. Since $I A=I C, I B=I D$, and $A B=$ $B C=C D$, the triangles $I A B, I C B$ and $I C D$ are congruent. Hence $\\angle I A B=\\angle I C B=$ $\\angle C \/ 2=\\angle A \/ 2$, so the line $I A$ bisects $\\angle A$. Similarly, the line $I D$ bisects $\\angle D$. Finally, the line $I E$ bisects $\\angle E$ because $I$ lies on all the other four internal bisectors of the angles of the pentagon. The sum of the internal angles in a pentagon is $540^{\\circ}$, so $$ \\angle E=540^{\\circ}-2 \\angle A+2 \\angle B . $$ In quadrilateral $A B I E$, $$ \\begin{aligned} \\angle B I E & =360^{\\circ}-\\angle E A B-\\angle A B I-\\angle A E I=360^{\\circ}-\\angle A-\\frac{1}{2} \\angle B-\\frac{1}{2} \\angle E \\\\ & =360^{\\circ}-\\angle A-\\frac{1}{2} \\angle B-\\left(270^{\\circ}-\\angle A-\\angle B\\right) \\\\ & =90^{\\circ}+\\frac{1}{2} \\angle B=90^{\\circ}+\\angle I B C, \\end{aligned} $$ which means that $E I \\perp B C$, completing the proof. ![](https:\/\/cdn.mathpix.com\/cropped\/2024_11_18_ff08e3c077fa2f8c16b2g-58.jpg?height=618&width=730&top_left_y=1801&top_left_x=663)","tier":0} +{"problem_type":"Geometry","problem_label":"G1","problem":"Let $A B C D E$ be a convex pentagon such that $A B=B C=C D, \\angle E A B=\\angle B C D$, and $\\angle E D C=\\angle C B A$. Prove that the perpendicular line from $E$ to $B C$ and the line segments $A C$ and $B D$ are concurrent. (Italy)","solution":"We present another proof of the fact that $E$ lies on line $I H$. Since all five internal bisectors of $A B C D E$ meet at $I$, this pentagon has an inscribed circle with center $I$. Let this circle touch side $B C$ at $T$. Applying Brianchon's theorem to the (degenerate) hexagon $A B T C D E$ we conclude that $A C, B D$ and $E T$ are concurrent, so point $E$ also lies on line $I H T$, completing the proof.","tier":0} +{"problem_type":"Geometry","problem_label":"G1","problem":"Let $A B C D E$ be a convex pentagon such that $A B=B C=C D, \\angle E A B=\\angle B C D$, and $\\angle E D C=\\angle C B A$. Prove that the perpendicular line from $E$ to $B C$ and the line segments $A C$ and $B D$ are concurrent. (Italy)","solution":"We present yet another proof that $E I \\perp B C$. In pentagon $A B C D E, \\angle E<$ $180^{\\circ} \\Longleftrightarrow \\angle A+\\angle B+\\angle C+\\angle D>360^{\\circ}$. Then $\\angle A+\\angle B=\\angle C+\\angle D>180^{\\circ}$, so rays $E A$ and $C B$ meet at a point $P$, and rays $B C$ and $E D$ meet at a point $Q$. Now, $$ \\angle P B A=180^{\\circ}-\\angle B=180^{\\circ}-\\angle D=\\angle Q D C $$ and, similarly, $\\angle P A B=\\angle Q C D$. Since $A B=C D$, the triangles $P A B$ and $Q C D$ are congruent with the same orientation. Moreover, $P Q E$ is isosceles with $E P=E Q$. ![](https:\/\/cdn.mathpix.com\/cropped\/2024_11_18_ff08e3c077fa2f8c16b2g-59.jpg?height=524&width=1546&top_left_y=612&top_left_x=255) In Solution 1 we have proved that triangles $I A B$ and $I C D$ are also congruent with the same orientation. Then we conclude that quadrilaterals $P B I A$ and $Q D I C$ are congruent, which implies $I P=I Q$. Then $E I$ is the perpendicular bisector of $P Q$ and, therefore, $E I \\perp$ $P Q \\Longleftrightarrow E I \\perp B C$. Comment. Even though all three solutions used the point $I$, there are solutions that do not need it. We present an outline of such a solution: if $J$ is the incenter of $\\triangle Q C D$ (with $P$ and $Q$ as defined in Solution 3), then a simple angle chasing shows that triangles $C J D$ and $B H C$ are congruent. Then if $S$ is the projection of $J$ onto side $C D$ and $T$ is the orthogonal projection of $H$ onto side $B C$, one can verify that $$ Q T=Q C+C T=Q C+D S=Q C+\\frac{C D+D Q-Q C}{2}=\\frac{P B+B C+Q C}{2}=\\frac{P Q}{2}, $$ so $T$ is the midpoint of $P Q$, and $E, H$ and $T$ all lie on the perpendicular bisector of $P Q$.","tier":0} +{"problem_type":"Geometry","problem_label":"G2","problem":"Let $R$ and $S$ be distinct points on circle $\\Omega$, and let $t$ denote the tangent line to $\\Omega$ at $R$. Point $R^{\\prime}$ is the reflection of $R$ with respect to $S$. A point $I$ is chosen on the smaller arc $R S$ of $\\Omega$ so that the circumcircle $\\Gamma$ of triangle $I S R^{\\prime}$ intersects $t$ at two different points. Denote by $A$ the common point of $\\Gamma$ and $t$ that is closest to $R$. Line $A I$ meets $\\Omega$ again at $J$. Show that $J R^{\\prime}$ is tangent to $\\Gamma$. (Luxembourg)","solution":"In the circles $\\Omega$ and $\\Gamma$ we have $\\angle J R S=\\angle J I S=\\angle A R^{\\prime} S$. On the other hand, since $R A$ is tangent to $\\Omega$, we get $\\angle S J R=\\angle S R A$. So the triangles $A R R^{\\prime}$ and $S J R$ are similar, and $$ \\frac{R^{\\prime} R}{R J}=\\frac{A R^{\\prime}}{S R}=\\frac{A R^{\\prime}}{S R^{\\prime}} $$ The last relation, together with $\\angle A R^{\\prime} S=\\angle J R R^{\\prime}$, yields $\\triangle A S R^{\\prime} \\sim \\triangle R^{\\prime} J R$, hence $\\angle S A R^{\\prime}=\\angle R R^{\\prime} J$. It follows that $J R^{\\prime}$ is tangent to $\\Gamma$ at $R^{\\prime}$. ![](https:\/\/cdn.mathpix.com\/cropped\/2024_11_18_ff08e3c077fa2f8c16b2g-60.jpg?height=880&width=757&top_left_y=1082&top_left_x=244) Solution 1 ![](https:\/\/cdn.mathpix.com\/cropped\/2024_11_18_ff08e3c077fa2f8c16b2g-60.jpg?height=1029&width=758&top_left_y=933&top_left_x=1066) Solution 2","tier":0} +{"problem_type":"Geometry","problem_label":"G2","problem":"Let $R$ and $S$ be distinct points on circle $\\Omega$, and let $t$ denote the tangent line to $\\Omega$ at $R$. Point $R^{\\prime}$ is the reflection of $R$ with respect to $S$. A point $I$ is chosen on the smaller arc $R S$ of $\\Omega$ so that the circumcircle $\\Gamma$ of triangle $I S R^{\\prime}$ intersects $t$ at two different points. Denote by $A$ the common point of $\\Gamma$ and $t$ that is closest to $R$. Line $A I$ meets $\\Omega$ again at $J$. Show that $J R^{\\prime}$ is tangent to $\\Gamma$. (Luxembourg)","solution":"As in Let $A^{\\prime}$ be the reflection of $A$ about $S$; then $A R A^{\\prime} R^{\\prime}$ is a parallelogram with center $S$, and hence the point $J$ lies on the line $R A^{\\prime}$. From $\\angle S R^{\\prime} A^{\\prime}=\\angle S R A=\\angle S J R$ we get that the points $S, J, A^{\\prime}, R^{\\prime}$ are concyclic. This proves that $\\angle S R^{\\prime} J=\\angle S A^{\\prime} J=\\angle S A^{\\prime} R=\\angle S A R^{\\prime}$, so $J R^{\\prime}$ is tangent to $\\Gamma$ at $R^{\\prime}$.","tier":0} +{"problem_type":"Geometry","problem_label":"G3","problem":"Let $O$ be the circumcenter of an acute scalene triangle $A B C$. Line $O A$ intersects the altitudes of $A B C$ through $B$ and $C$ at $P$ and $Q$, respectively. The altitudes meet at $H$. Prove that the circumcenter of triangle $P Q H$ lies on a median of triangle $A B C$. (Ukraine)","solution":"Suppose, without loss of generality, that $A B0$. Denote by $h$ the homothety mapping $\\mathcal{C}$ to $\\mathcal{A}$. We need now to prove that the vertices of $\\mathcal{C}$ which stay in $\\mathcal{B}$ after applying $h$ are consecutive. If $X \\in \\mathcal{B}$, the claim is easy. Indeed, if $k<1$, then the vertices of $\\mathcal{A}$ lie on the segments of the form $X C$ ( $C$ being a vertex of $\\mathcal{C}$ ) which lie in $\\mathcal{B}$. If $k>1$, then the vertices of $\\mathcal{A}$ lie on the extensions of such segments $X C$ beyond $C$, and almost all these extensions lie outside $\\mathcal{B}$. The exceptions may occur only in case when $X$ lies on the boundary of $\\mathcal{B}$, and they may cause one or two vertices of $\\mathcal{A}$ stay on the boundary of $\\mathcal{B}$. But even in this case those vertices are still consecutive. So, from now on we assume that $X \\notin \\mathcal{B}$. Now, there are two vertices $B_{\\mathrm{T}}$ and $\\mathcal{B}_{\\mathrm{B}}$ of $\\mathcal{B}$ such that $\\mathcal{B}$ is contained in the angle $\\angle B_{\\mathrm{T}} X B_{\\mathrm{B}}$; if there are several options, say, for $B_{\\mathrm{T}}$, then we choose the farthest one from $X$ if $k>1$, and the nearest one if $k<1$. For the visualization purposes, we refer the plane to Cartesian coordinates so that the $y$-axis is co-directional with $\\overrightarrow{B_{\\mathrm{B}} B_{\\mathrm{T}}}$, and $X$ lies to the left of the line $B_{\\mathrm{T}} B_{\\mathrm{B}}$. Again, the perimeter of $\\mathcal{B}$ is split by $B_{\\mathrm{T}}$ and $B_{\\mathrm{B}}$ into the right part $\\mathcal{B}_{\\mathrm{R}}$ and the left part $\\mathcal{B}_{\\mathrm{L}}$, and the set of vertices of $\\mathcal{C}$ is split into two subsets $\\mathcal{C}_{\\mathrm{R}} \\subset \\mathcal{B}_{\\mathrm{R}}$ and $\\mathcal{C}_{\\mathrm{L}} \\subset \\mathcal{B}_{\\mathrm{L}}$. ![](https:\/\/cdn.mathpix.com\/cropped\/2024_11_18_ff08e3c077fa2f8c16b2g-67.jpg?height=572&width=546&top_left_y=959&top_left_x=281) Case 2, $X$ inside $\\mathcal{B}$ ![](https:\/\/cdn.mathpix.com\/cropped\/2024_11_18_ff08e3c077fa2f8c16b2g-67.jpg?height=623&width=906&top_left_y=908&top_left_x=866) Subcase 2.1: $k>1$ Subcase 2.1: $k>1$. In this subcase, all points from $\\mathcal{B}_{\\mathrm{R}}$ (and hence from $\\mathcal{C}_{\\mathrm{R}}$ ) move out from $\\mathcal{B}$ under $h$, because they are the farthest points of $\\mathcal{B}$ on the corresponding rays emanated from $X$. It remains to prove that the vertices of $\\mathcal{C}_{\\mathrm{L}}$ which stay in $\\mathcal{B}$ under $h$ are consecutive. Again, let $C_{1}, C_{2}, C_{3}$ be three vertices in $\\mathcal{C}_{\\mathrm{L}}$ such that $C_{2}$ is between $C_{1}$ and $C_{3}$, and $h\\left(C_{1}\\right)$ and $h\\left(C_{3}\\right)$ lie in $\\mathcal{B}$. Let $A_{i}=h\\left(C_{i}\\right)$. Then the ray $X C_{2}$ crosses the segment $C_{1} C_{3}$ beyond $C_{2}$, so this ray crosses $A_{1} A_{3}$ beyond $A_{2}$; this implies that $A_{2}$ lies in the triangle $A_{1} C_{2} A_{3}$, which is contained in $\\mathcal{B}$. ![](https:\/\/cdn.mathpix.com\/cropped\/2024_11_18_ff08e3c077fa2f8c16b2g-67.jpg?height=515&width=909&top_left_y=2021&top_left_x=576) Subcase 2.2: $k<1$ Subcase 2.2: $k<1$. This case is completely similar to the previous one. All points from $\\mathcal{B}_{\\mathrm{L}}$ (and hence from $\\mathcal{C}_{\\mathrm{L}}$ move out from $\\mathcal{B}$ under $h$, because they are the nearest points of $\\mathcal{B}$ on the corresponding rays emanated from $X$. Assume that $C_{1}, C_{2}$, and $C_{3}$ are three vertices in $\\mathcal{C}_{\\mathrm{R}}$ such that $C_{2}$ lies between $C_{1}$ and $C_{3}$, and $h\\left(C_{1}\\right)$ and $h\\left(C_{3}\\right)$ lie in $\\mathcal{B}$; let $A_{i}=h\\left(C_{i}\\right)$. Then $A_{2}$ lies on the segment $X C_{2}$, and the segments $X A_{2}$ and $A_{1} A_{3}$ cross each other. Thus $A_{2}$ lies in the triangle $A_{1} C_{2} A_{3}$, which is contained in $\\mathcal{B}$. Comment 1. In fact, Case 1 can be reduced to Case 2 via the following argument. Assume that $\\mathcal{A}$ and $\\mathcal{C}$ are congruent. Apply to $\\mathcal{A}$ a homothety centered at $O_{B}$ with a factor slightly smaller than 1 to obtain a polygon $\\mathcal{A}^{\\prime}$. With appropriately chosen factor, the vertices of $\\mathcal{A}$ which were outside $\/$ inside $\\mathcal{B}$ stay outside\/inside it, so it suffices to prove our claim for $\\mathcal{A}^{\\prime}$ instead of $\\mathcal{A}$. And now, the polygon $\\mathcal{A}^{\\prime}$ is a homothetic image of $\\mathcal{C}$, so the arguments from Case 2 apply. Comment 2. After the polygon $\\mathcal{C}$ has been found, the rest of the solution uses only the convexity of the polygons, instead of regularity. Thus, it proves a more general statement: Assume that $\\mathcal{A}, \\mathcal{B}$, and $\\mathcal{C}$ are three convex polygons in the plane such that $\\mathcal{C}$ is inscribed into $\\mathcal{B}$, and $\\mathcal{A}$ can be obtained from it via either translation or positive homothety. Then the vertices of $\\mathcal{A}$ that lie inside $\\mathcal{B}$ or on its boundary are consecutive.","tier":0} +{"problem_type":"Geometry","problem_label":"G6","problem":"Let $n \\geqslant 3$ be an integer. Two regular $n$-gons $\\mathcal{A}$ and $\\mathcal{B}$ are given in the plane. Prove that the vertices of $\\mathcal{A}$ that lie inside $\\mathcal{B}$ or on its boundary are consecutive. (That is, prove that there exists a line separating those vertices of $\\mathcal{A}$ that lie inside $\\mathcal{B}$ or on its boundary from the other vertices of $\\mathcal{A}$.) (Czech Republic)","solution":"Let $O_{A}$ and $O_{B}$ be the centers of $\\mathcal{A}$ and $\\mathcal{B}$, respectively. Denote $[n]=\\{1,2, \\ldots, n\\}$. We start with introducing appropriate enumerations and notations. Enumerate the sidelines of $\\mathcal{B}$ clockwise as $\\ell_{1}, \\ell_{2}, \\ldots, \\ell_{n}$. Denote by $\\mathcal{H}_{i}$ the half-plane of $\\ell_{i}$ that contains $\\mathcal{B}$ ( $\\mathcal{H}_{i}$ is assumed to contain $\\left.\\ell_{i}\\right)$; by $B_{i}$ the midpoint of the side belonging to $\\ell_{i}$; and finally denote $\\overrightarrow{b_{i}}=\\overrightarrow{B_{i} O_{B}}$. (As usual, the numbering is cyclic modulo $n$, so $\\ell_{n+i}=\\ell_{i}$ etc.) Now, choose a vertex $A_{1}$ of $\\mathcal{A}$ such that the vector $\\overrightarrow{O_{A} A_{1}}$ points \"mostly outside $\\mathcal{H}_{1}$ \"; strictly speaking, this means that the scalar product $\\left\\langle\\overrightarrow{O_{A} A_{1}}, \\overrightarrow{b_{1}}\\right\\rangle$ is minimal. Starting from $A_{1}$, enumerate the vertices of $\\mathcal{A}$ clockwise as $A_{1}, A_{2}, \\ldots, A_{n}$; by the rotational symmetry, the choice of $A_{1}$ yields that the vector $\\overrightarrow{O_{A} A_{i}}$ points \"mostly outside $\\mathcal{H}_{i}$ \", i.e., $$ \\left\\langle\\overrightarrow{O_{A} A_{i}}, \\overrightarrow{b_{i}}\\right\\rangle=\\min _{j \\in[n]}\\left\\langle\\overrightarrow{O_{A} A_{j}}, \\overrightarrow{b_{i}}\\right\\rangle $$ ![](https:\/\/cdn.mathpix.com\/cropped\/2024_11_18_ff08e3c077fa2f8c16b2g-68.jpg?height=560&width=898&top_left_y=1516&top_left_x=579) Enumerations and notations We intend to reformulate the problem in more combinatorial terms, for which purpose we introduce the following notion. Say that a subset $I \\subseteq[n]$ is connected if the elements of this set are consecutive in the cyclic order (in other words, if we join each $i$ with $i+1 \\bmod n$ by an edge, this subset is connected in the usual graph sense). Clearly, the union of two connected subsets sharing at least one element is connected too. Next, for any half-plane $\\mathcal{H}$ the indices of vertices of, say, $\\mathcal{A}$ that lie in $\\mathcal{H}$ form a connected set. To access the problem, we denote $$ M=\\left\\{j \\in[n]: A_{j} \\notin \\mathcal{B}\\right\\}, \\quad M_{i}=\\left\\{j \\in[n]: A_{j} \\notin \\mathcal{H}_{i}\\right\\} \\quad \\text { for } i \\in[n] $$ We need to prove that $[n] \\backslash M$ is connected, which is equivalent to $M$ being connected. On the other hand, since $\\mathcal{B}=\\bigcap_{i \\in[n]} \\mathcal{H}_{i}$, we have $M=\\bigcup_{i \\in[n]} M_{i}$, where the sets $M_{i}$ are easier to investigate. We will utilize the following properties of these sets; the first one holds by the definition of $M_{i}$, along with the above remark. ![](https:\/\/cdn.mathpix.com\/cropped\/2024_11_18_ff08e3c077fa2f8c16b2g-69.jpg?height=529&width=1183&top_left_y=175&top_left_x=439) The sets $M_{i}$ Property 1: Each set $M_{i}$ is connected. Property 2: If $M_{i}$ is nonempty, then $i \\in M_{i}$. Proof. Indeed, we have $$ j \\in M_{i} \\Longleftrightarrow A_{j} \\notin \\mathcal{H}_{i} \\Longleftrightarrow\\left\\langle\\overrightarrow{B_{i} A_{j}}, \\overrightarrow{b_{i}}\\right\\rangle<0 \\Longleftrightarrow\\left\\langle\\overrightarrow{O_{A} A_{j}}, \\overrightarrow{b_{i}}\\right\\rangle<\\left\\langle\\overrightarrow{O_{A} B_{i}}, \\overrightarrow{b_{i}}\\right\\rangle $$ The right-hand part of the last inequality does not depend on $j$. Therefore, if some $j$ lies in $M_{i}$, then by (1) so does $i$. In view of Property 2, it is useful to define the set $$ M^{\\prime}=\\left\\{i \\in[n]: i \\in M_{i}\\right\\}=\\left\\{i \\in[n]: M_{i} \\neq \\varnothing\\right\\} . $$ Property 3: The set $M^{\\prime}$ is connected. Proof. To prove this property, we proceed on with the investigation started in (2) to write $$ i \\in M^{\\prime} \\Longleftrightarrow A_{i} \\in M_{i} \\Longleftrightarrow\\left\\langle\\overrightarrow{B_{i}} \\overrightarrow{A_{i}}, \\overrightarrow{b_{i}}\\right\\rangle<0 \\Longleftrightarrow\\left\\langle\\overrightarrow{O_{B} O_{A}}, \\overrightarrow{b_{i}}\\right\\rangle<\\left\\langle\\overrightarrow{O_{B} B_{i}}, \\overrightarrow{b_{i}}\\right\\rangle+\\left\\langle\\overrightarrow{A_{i} O_{A}}, \\overrightarrow{b_{i}}\\right\\rangle $$ The right-hand part of the obtained inequality does not depend on $i$, due to the rotational symmetry; denote its constant value by $\\mu$. Thus, $i \\in M^{\\prime}$ if and only if $\\left\\langle\\overrightarrow{O_{B} O_{A}}, \\overrightarrow{b_{i}}\\right\\rangle<\\mu$. This condition is in turn equivalent to the fact that $B_{i}$ lies in a certain (open) half-plane whose boundary line is orthogonal to $O_{B} O_{A}$; thus, it defines a connected set. Now we can finish the solution. Since $M^{\\prime} \\subseteq M$, we have $$ M=\\bigcup_{i \\in[n]} M_{i}=M^{\\prime} \\cup \\bigcup_{i \\in[n]} M_{i}, $$ so $M$ can be obtained from $M^{\\prime}$ by adding all the sets $M_{i}$ one by one. All these sets are connected, and each nonempty $M_{i}$ contains an element of $M^{\\prime}$ (namely, $i$ ). Thus their union is also connected. Comment 3. Here we present a way in which one can come up with a solution like the one above. Assume, for sake of simplicity, that $O_{A}$ lies inside $\\mathcal{B}$. Let us first put onto the plane a very small regular $n$-gon $\\mathcal{A}^{\\prime}$ centered at $O_{A}$ and aligned with $\\mathcal{A}$; all its vertices lie inside $\\mathcal{B}$. Now we start blowing it up, looking at the order in which the vertices leave $\\mathcal{B}$. To go out of $\\mathcal{B}$, a vertex should cross a certain side of $\\mathcal{B}$ (which is hard to describe), or, equivalently, to cross at least one sideline of $\\mathcal{B}$ - and this event is easier to describe. Indeed, the first vertex of $\\mathcal{A}^{\\prime}$ to cross $\\ell_{i}$ is the vertex $A_{i}^{\\prime}$ (corresponding to $A_{i}$ in $\\mathcal{A}$ ); more generally, the vertices $A_{j}^{\\prime}$ cross $\\ell_{i}$ in such an order that the scalar product $\\left\\langle\\overrightarrow{O_{A}} \\overrightarrow{A_{j}}, \\overrightarrow{b_{i}}\\right\\rangle$ does not increase. For different indices $i$, these orders are just cyclic shifts of each other; and this provides some intuition for the notions and claims from Solution 2.","tier":0} +{"problem_type":"Geometry","problem_label":"G7","problem":"A convex quadrilateral $A B C D$ has an inscribed circle with center $I$. Let $I_{a}, I_{b}, I_{c}$, and $I_{d}$ be the incenters of the triangles $D A B, A B C, B C D$, and $C D A$, respectively. Suppose that the common external tangents of the circles $A I_{b} I_{d}$ and $C I_{b} I_{d}$ meet at $X$, and the common external tangents of the circles $B I_{a} I_{c}$ and $D I_{a} I_{c}$ meet at $Y$. Prove that $\\angle X I Y=90^{\\circ}$. (Kazakhstan)","solution":"Denote by $\\omega_{a}, \\omega_{b}, \\omega_{c}$ and $\\omega_{d}$ the circles $A I_{b} I_{d}, B I_{a} I_{c}, C I_{b} I_{d}$, and $D I_{a} I_{c}$, let their centers be $O_{a}, O_{b}, O_{c}$ and $O_{d}$, and let their radii be $r_{a}, r_{b}, r_{c}$ and $r_{d}$, respectively. Claim 1. $I_{b} I_{d} \\perp A C$ and $I_{a} I_{c} \\perp B D$. Proof. Let the incircles of triangles $A B C$ and $A C D$ be tangent to the line $A C$ at $T$ and $T^{\\prime}$, respectively. (See the figure to the left.) We have $A T=\\frac{A B+A C-B C}{2}$ in triangle $A B C, A T^{\\prime}=$ $\\frac{A D+A C-C D}{2}$ in triangle $A C D$, and $A B-B C=A D-C D$ in quadrilateral $A B C D$, so $$ A T=\\frac{A C+A B-B C}{2}=\\frac{A C+A D-C D}{2}=A T^{\\prime} $$ This shows $T=T^{\\prime}$. As an immediate consequence, $I_{b} I_{d} \\perp A C$. The second statement can be shown analogously. ![](https:\/\/cdn.mathpix.com\/cropped\/2024_11_18_ff08e3c077fa2f8c16b2g-70.jpg?height=726&width=1729&top_left_y=1076&top_left_x=168) Claim 2. The points $O_{a}, O_{b}, O_{c}$ and $O_{d}$ lie on the lines $A I, B I, C I$ and $D I$, respectively. Proof. By symmetry it suffices to prove the claim for $O_{a}$. (See the figure to the right above.) Notice first that the incircles of triangles $A B C$ and $A C D$ can be obtained from the incircle of the quadrilateral $A B C D$ with homothety centers $B$ and $D$, respectively, and homothety factors less than 1 , therefore the points $I_{b}$ and $I_{d}$ lie on the line segments $B I$ and $D I$, respectively. As is well-known, in every triangle the altitude and the diameter of the circumcircle starting from the same vertex are symmetric about the angle bisector. By Claim 1, in triangle $A I_{d} I_{b}$, the segment $A T$ is the altitude starting from $A$. Since the foot $T$ lies inside the segment $I_{b} I_{d}$, the circumcenter $O_{a}$ of triangle $A I_{d} I_{b}$ lies in the angle domain $I_{b} A I_{d}$ in such a way that $\\angle I_{b} A T=\\angle O_{a} A I_{d}$. The points $I_{b}$ and $I_{d}$ are the incenters of triangles $A B C$ and $A C D$, so the lines $A I_{b}$ and $A I_{d}$ bisect the angles $\\angle B A C$ and $\\angle C A D$, respectively. Then $$ \\angle O_{a} A D=\\angle O_{a} A I_{d}+\\angle I_{d} A D=\\angle I_{b} A T+\\angle I_{d} A D=\\frac{1}{2} \\angle B A C+\\frac{1}{2} \\angle C A D=\\frac{1}{2} \\angle B A D, $$ so $O_{a}$ lies on the angle bisector of $\\angle B A D$, that is, on line $A I$. The point $X$ is the external similitude center of $\\omega_{a}$ and $\\omega_{c}$; let $U$ be their internal similitude center. The points $O_{a}$ and $O_{c}$ lie on the perpendicular bisector of the common chord $I_{b} I_{d}$ of $\\omega_{a}$ and $\\omega_{c}$, and the two similitude centers $X$ and $U$ lie on the same line; by Claim 2, that line is parallel to $A C$. ![](https:\/\/cdn.mathpix.com\/cropped\/2024_11_18_ff08e3c077fa2f8c16b2g-71.jpg?height=1149&width=1529&top_left_y=179&top_left_x=269) From the similarity of the circles $\\omega_{a}$ and $\\omega_{c}$, from $O_{a} I_{b}=O_{a} I_{d}=O_{a} A=r_{a}$ and $O_{c} I_{b}=$ $O_{c} I_{d}=O_{c} C=r_{c}$, and from $A C \\| O_{a} O_{c}$ we can see that $$ \\frac{O_{a} X}{O_{c} X}=\\frac{O_{a} U}{O_{c} U}=\\frac{r_{a}}{r_{c}}=\\frac{O_{a} I_{b}}{O_{c} I_{b}}=\\frac{O_{a} I_{d}}{O_{c} I_{d}}=\\frac{O_{a} A}{O_{c} C}=\\frac{O_{a} I}{O_{c} I} $$ So the points $X, U, I_{b}, I_{d}, I$ lie on the Apollonius circle of the points $O_{a}, O_{c}$ with ratio $r_{a}: r_{c}$. In this Apollonius circle $X U$ is a diameter, and the lines $I U$ and $I X$ are respectively the internal and external bisectors of $\\angle O_{a} I O_{c}=\\angle A I C$, according to the angle bisector theorem. Moreover, in the Apollonius circle the diameter $U X$ is the perpendicular bisector of $I_{b} I_{d}$, so the lines $I X$ and $I U$ are the internal and external bisectors of $\\angle I_{b} I I_{d}=\\angle B I D$, respectively. Repeating the same argument for the points $B, D$ instead of $A, C$, we get that the line $I Y$ is the internal bisector of $\\angle A I C$ and the external bisector of $\\angle B I D$. Therefore, the lines $I X$ and $I Y$ respectively are the internal and external bisectors of $\\angle B I D$, so they are perpendicular. Comment. In fact the points $O_{a}, O_{b}, O_{c}$ and $O_{d}$ lie on the line segments $A I, B I, C I$ and $D I$, respectively. For the point $O_{a}$ this can be shown for example by $\\angle I_{d} O_{a} A+\\angle A O_{a} I_{b}=\\left(180^{\\circ}-\\right.$ $\\left.2 \\angle O_{a} A I_{d}\\right)+\\left(180^{\\circ}-2 \\angle I_{b} A O_{a}\\right)=360^{\\circ}-\\angle B A D=\\angle A D I+\\angle D I A+\\angle A I B+\\angle I B A>\\angle I_{d} I A+\\angle A I I_{b}$. The solution also shows that the line $I Y$ passes through the point $U$, and analogously, $I X$ passes through the internal similitude center of $\\omega_{b}$ and $\\omega_{d}$.","tier":0} +{"problem_type":"Geometry","problem_label":"G8","problem":"There are 2017 mutually external circles drawn on a blackboard, such that no two are tangent and no three share a common tangent. A tangent segment is a line segment that is a common tangent to two circles, starting at one tangent point and ending at the other one. Luciano is drawing tangent segments on the blackboard, one at a time, so that no tangent segment intersects any other circles or previously drawn tangent segments. Luciano keeps drawing tangent segments until no more can be drawn. Find all possible numbers of tangent segments when he stops drawing. (Australia)","solution":"First, consider a particular arrangement of circles $C_{1}, C_{2}, \\ldots, C_{n}$ where all the centers are aligned and each $C_{i}$ is eclipsed from the other circles by its neighbors - for example, taking $C_{i}$ with center $\\left(i^{2}, 0\\right)$ and radius $i \/ 2$ works. Then the only tangent segments that can be drawn are between adjacent circles $C_{i}$ and $C_{i+1}$, and exactly three segments can be drawn for each pair. So Luciano will draw exactly $3(n-1)$ segments in this case. ![](https:\/\/cdn.mathpix.com\/cropped\/2024_11_18_ff08e3c077fa2f8c16b2g-72.jpg?height=258&width=1335&top_left_y=1059&top_left_x=355) For the general case, start from a final configuration (that is, an arrangement of circles and segments in which no further segments can be drawn). The idea of the solution is to continuously resize and move the circles around the plane, one by one (in particular, making sure we never have 4 circles with a common tangent line), and show that the number of segments drawn remains constant as the picture changes. This way, we can reduce any circle\/segment configuration to the particular one mentioned above, and the final number of segments must remain at $3 n-3$. Some preliminary considerations: look at all possible tangent segments joining any two circles. A segment that is tangent to a circle $A$ can do so in two possible orientations - it may come out of $A$ in clockwise or counterclockwise orientation. Two segments touching the same circle with the same orientation will never intersect each other. Each pair $(A, B)$ of circles has 4 choices of tangent segments, which can be identified by their orientations - for example, $(A+, B-)$ would be the segment which comes out of $A$ in clockwise orientation and comes out of $B$ in counterclockwise orientation. In total, we have $2 n(n-1)$ possible segments, disregarding intersections. Now we pick a circle $C$ and start to continuously move and resize it, maintaining all existing tangent segments according to their identifications, including those involving $C$. We can keep our choice of tangent segments until the configuration reaches a transition. We lose nothing if we assume that $C$ is kept at least $\\varepsilon$ units away from any other circle, where $\\varepsilon$ is a positive, fixed constant; therefore at a transition either: (1) a currently drawn tangent segment $t$ suddenly becomes obstructed; or (2) a currently absent tangent segment $t$ suddenly becomes unobstructed and available. Claim. A transition can only occur when three circles $C_{1}, C_{2}, C_{3}$ are tangent to a common line $\\ell$ containing $t$, in a way such that the three tangent segments lying on $\\ell$ (joining the three circles pairwise) are not obstructed by any other circles or tangent segments (other than $C_{1}, C_{2}, C_{3}$ ). Proof. Since (2) is effectively the reverse of (1), it suffices to prove the claim for (1). Suppose $t$ has suddenly become obstructed, and let us consider two cases. Case 1: $t$ becomes obstructed by a circle ![](https:\/\/cdn.mathpix.com\/cropped\/2024_11_18_ff08e3c077fa2f8c16b2g-73.jpg?height=318&width=1437&top_left_y=292&top_left_x=315) Then the new circle becomes the third circle tangent to $\\ell$, and no other circles or tangent segments are obstructing $t$.","tier":0} +{"problem_type":"Geometry","problem_label":"G8","problem":"There are 2017 mutually external circles drawn on a blackboard, such that no two are tangent and no three share a common tangent. A tangent segment is a line segment that is a common tangent to two circles, starting at one tangent point and ending at the other one. Luciano is drawing tangent segments on the blackboard, one at a time, so that no tangent segment intersects any other circles or previously drawn tangent segments. Luciano keeps drawing tangent segments until no more can be drawn. Find all possible numbers of tangent segments when he stops drawing. (Australia)","solution":"First note that all tangent segments lying on the boundary of the convex hull of the circles are always drawn since they do not intersect anything else. Now in the final picture, aside from the $n$ circles, the blackboard is divided into regions. We can consider the picture as a plane (multi-)graph $G$ in which the circles are the vertices and the tangent segments are the edges. The idea of this solution is to find a relation between the number of edges and the number of regions in $G$; then, once we prove that $G$ is connected, we can use Euler's formula to finish the problem. The boundary of each region consists of 1 or more (for now) simple closed curves, each made of arcs and tangent segments. The segment and the arc might meet smoothly (as in $S_{i}$, $i=1,2, \\ldots, 6$ in the figure below) or not (as in $P_{1}, P_{2}, P_{3}, P_{4}$; call such points sharp corners of the boundary). In other words, if a person walks along the border, her direction would suddenly turn an angle of $\\pi$ at a sharp corner. ![](https:\/\/cdn.mathpix.com\/cropped\/2024_11_18_ff08e3c077fa2f8c16b2g-75.jpg?height=626&width=577&top_left_y=178&top_left_x=748) Claim 1. The outer boundary $B_{1}$ of any internal region has at least 3 sharp corners. Proof. Let a person walk one lap along $B_{1}$ in the counterclockwise orientation. As she does so, she will turn clockwise as she moves along the circle arcs, and not turn at all when moving along the lines. On the other hand, her total rotation after one lap is $2 \\pi$ in the counterclockwise direction! Where could she be turning counterclockwise? She can only do so at sharp corners, and, even then, she turns only an angle of $\\pi$ there. But two sharp corners are not enough, since at least one arc must be present-so she must have gone through at least 3 sharp corners. Claim 2. Each internal region is simply connected, that is, has only one boundary curve. Proof. Suppose, by contradiction, that some region has an outer boundary $B_{1}$ and inner boundaries $B_{2}, B_{3}, \\ldots, B_{m}(m \\geqslant 2)$. Let $P_{1}$ be one of the sharp corners of $B_{1}$. Now consider a car starting at $P_{1}$ and traveling counterclockwise along $B_{1}$. It starts in reverse, i.e., it is initially facing the corner $P_{1}$. Due to the tangent conditions, the car may travel in a way so that its orientation only changes when it is moving along an arc. In particular, this means the car will sometimes travel forward. For example, if the car approaches a sharp corner when driving in reverse, it would continue travel forward after the corner, instead of making an immediate half-turn. This way, the orientation of the car only changes in a clockwise direction since the car always travels clockwise around each arc. Now imagine there is a laser pointer at the front of the car, pointing directly ahead. Initially, the laser endpoint hits $P_{1}$, but, as soon as the car hits an arc, the endpoint moves clockwise around $B_{1}$. In fact, the laser endpoint must move continuously along $B_{1}$ ! Indeed, if the endpoint ever jumped (within $B_{1}$, or from $B_{1}$ to one of the inner boundaries), at the moment of the jump the interrupted laser would be a drawable tangent segment that Luciano missed (see figure below for an example). ![](https:\/\/cdn.mathpix.com\/cropped\/2024_11_18_ff08e3c077fa2f8c16b2g-75.jpg?height=637&width=587&top_left_y=2140&top_left_x=740) Now, let $P_{2}$ and $P_{3}$ be the next two sharp corners the car goes through, after $P_{1}$ (the previous lemma assures their existence). At $P_{2}$ the car starts moving forward, and at $P_{3}$ it will start to move in reverse again. So, at $P_{3}$, the laser endpoint is at $P_{3}$ itself. So while the car moved counterclockwise between $P_{1}$ and $P_{3}$, the laser endpoint moved clockwise between $P_{1}$ and $P_{3}$. That means the laser beam itself scanned the whole region within $B_{1}$, and it should have crossed some of the inner boundaries. Claim 3. Each region has exactly 3 sharp corners. Proof. Consider again the car of the previous claim, with its laser still firmly attached to its front, traveling the same way as before and going through the same consecutive sharp corners $P_{1}, P_{2}$ and $P_{3}$. As we have seen, as the car goes counterclockwise from $P_{1}$ to $P_{3}$, the laser endpoint goes clockwise from $P_{1}$ to $P_{3}$, so together they cover the whole boundary. If there were a fourth sharp corner $P_{4}$, at some moment the laser endpoint would pass through it. But, since $P_{4}$ is a sharp corner, this means the car must be on the extension of a tangent segment going through $P_{4}$. Since the car is not on that segment itself (the car never goes through $P_{4}$ ), we would have 3 circles with a common tangent line, which is not allowed. ![](https:\/\/cdn.mathpix.com\/cropped\/2024_11_18_ff08e3c077fa2f8c16b2g-76.jpg?height=538&width=792&top_left_y=1007&top_left_x=643) We are now ready to finish the solution. Let $r$ be the number of internal regions, and $s$ be the number of tangent segments. Since each tangent segment contributes exactly 2 sharp corners to the diagram, and each region has exactly 3 sharp corners, we must have $2 s=3 r$. Since the graph corresponding to the diagram is connected, we can use Euler's formula $n-s+r=1$ and find $s=3 n-3$ and $r=2 n-2$.","tier":0} +{"problem_type":"Number Theory","problem_label":"N1","problem":"The sequence $a_{0}, a_{1}, a_{2}, \\ldots$ of positive integers satisfies $$ a_{n+1}=\\left\\{\\begin{array}{ll} \\sqrt{a_{n}}, & \\text { if } \\sqrt{a_{n}} \\text { is an integer } \\\\ a_{n}+3, & \\text { otherwise } \\end{array} \\quad \\text { for every } n \\geqslant 0\\right. $$ Determine all values of $a_{0}>1$ for which there is at least one number $a$ such that $a_{n}=a$ for infinitely many values of $n$. (South Africa)","solution":"Since the value of $a_{n+1}$ only depends on the value of $a_{n}$, if $a_{n}=a_{m}$ for two different indices $n$ and $m$, then the sequence is eventually periodic. So we look for the values of $a_{0}$ for which the sequence is eventually periodic. Claim 1. If $a_{n} \\equiv-1(\\bmod 3)$, then, for all $m>n, a_{m}$ is not a perfect square. It follows that the sequence is eventually strictly increasing, so it is not eventually periodic. Proof. A square cannot be congruent to -1 modulo 3 , so $a_{n} \\equiv-1(\\bmod 3)$ implies that $a_{n}$ is not a square, therefore $a_{n+1}=a_{n}+3>a_{n}$. As a consequence, $a_{n+1} \\equiv a_{n} \\equiv-1(\\bmod 3)$, so $a_{n+1}$ is not a square either. By repeating the argument, we prove that, from $a_{n}$ on, all terms of the sequence are not perfect squares and are greater than their predecessors, which completes the proof. Claim 2. If $a_{n} \\not \\equiv-1(\\bmod 3)$ and $a_{n}>9$ then there is an index $m>n$ such that $a_{m}9, t$ is at least 3. The first square in the sequence $a_{n}, a_{n}+3, a_{n}+6, \\ldots$ will be $(t+1)^{2},(t+2)^{2}$ or $(t+3)^{2}$, therefore there is an index $m>n$ such that $a_{m} \\leqslant t+3n$ such that $a_{m}=3$. Proof. First we notice that, by the definition of the sequence, a multiple of 3 is always followed by another multiple of 3 . If $a_{n} \\in\\{3,6,9\\}$ the sequence will eventually follow the periodic pattern $3,6,9,3,6,9, \\ldots$ If $a_{n}>9$, let $j$ be an index such that $a_{j}$ is equal to the minimum value of the set $\\left\\{a_{n+1}, a_{n+2}, \\ldots\\right\\}$. We must have $a_{j} \\leqslant 9$, otherwise we could apply Claim 2 to $a_{j}$ and get a contradiction on the minimality hypothesis. It follows that $a_{j} \\in\\{3,6,9\\}$, and the proof is complete. Claim 4. If $a_{n} \\equiv 1(\\bmod 3)$, then there is an index $m>n$ such that $a_{m} \\equiv-1(\\bmod 3)$. Proof. In the sequence, 4 is always followed by $2 \\equiv-1(\\bmod 3)$, so the claim is true for $a_{n}=4$. If $a_{n}=7$, the next terms will be $10,13,16,4,2, \\ldots$ and the claim is also true. For $a_{n} \\geqslant 10$, we again take an index $j>n$ such that $a_{j}$ is equal to the minimum value of the set $\\left\\{a_{n+1}, a_{n+2}, \\ldots\\right\\}$, which by the definition of the sequence consists of non-multiples of 3 . Suppose $a_{j} \\equiv 1(\\bmod 3)$. Then we must have $a_{j} \\leqslant 9$ by Claim 2 and the minimality of $a_{j}$. It follows that $a_{j} \\in\\{4,7\\}$, so $a_{m}=2j$, contradicting the minimality of $a_{j}$. Therefore, we must have $a_{j} \\equiv-1(\\bmod 3)$. It follows from the previous claims that if $a_{0}$ is a multiple of 3 the sequence will eventually reach the periodic pattern $3,6,9,3,6,9, \\ldots$; if $a_{0} \\equiv-1(\\bmod 3)$ the sequence will be strictly increasing; and if $a_{0} \\equiv 1(\\bmod 3)$ the sequence will be eventually strictly increasing. So the sequence will be eventually periodic if, and only if, $a_{0}$ is a multiple of 3 .","tier":0} +{"problem_type":"Number Theory","problem_label":"N2","problem":"Let $p \\geqslant 2$ be a prime number. Eduardo and Fernando play the following game making moves alternately: in each move, the current player chooses an index $i$ in the set $\\{0,1, \\ldots, p-1\\}$ that was not chosen before by either of the two players and then chooses an element $a_{i}$ of the set $\\{0,1,2,3,4,5,6,7,8,9\\}$. Eduardo has the first move. The game ends after all the indices $i \\in\\{0,1, \\ldots, p-1\\}$ have been chosen. Then the following number is computed: $$ M=a_{0}+10 \\cdot a_{1}+\\cdots+10^{p-1} \\cdot a_{p-1}=\\sum_{j=0}^{p-1} a_{j} \\cdot 10^{j} $$ The goal of Eduardo is to make the number $M$ divisible by $p$, and the goal of Fernando is to prevent this. Prove that Eduardo has a winning strategy. (Morocco)","solution":"We say that a player makes the move $\\left(i, a_{i}\\right)$ if he chooses the index $i$ and then the element $a_{i}$ of the set $\\{0,1,2,3,4,5,6,7,8,9\\}$ in this move. If $p=2$ or $p=5$ then Eduardo chooses $i=0$ and $a_{0}=0$ in the first move, and wins, since, independently of the next moves, $M$ will be a multiple of 10 . Now assume that the prime number $p$ does not belong to $\\{2,5\\}$. Eduardo chooses $i=p-1$ and $a_{p-1}=0$ in the first move. By Fermat's Little Theorem, $\\left(10^{(p-1) \/ 2}\\right)^{2}=10^{p-1} \\equiv 1(\\bmod p)$, so $p \\mid\\left(10^{(p-1) \/ 2}\\right)^{2}-1=\\left(10^{(p-1) \/ 2}+1\\right)\\left(10^{(p-1) \/ 2}-1\\right)$. Since $p$ is prime, either $p \\mid 10^{(p-1) \/ 2}+1$ or $p \\mid 10^{(p-1) \/ 2}-1$. Thus we have two cases: Case a: $10^{(p-1) \/ 2} \\equiv-1(\\bmod p)$ In this case, for each move $\\left(i, a_{i}\\right)$ of Fernando, Eduardo immediately makes the move $\\left(j, a_{j}\\right)=$ $\\left(i+\\frac{p-1}{2}, a_{i}\\right)$, if $0 \\leqslant i \\leqslant \\frac{p-3}{2}$, or $\\left(j, a_{j}\\right)=\\left(i-\\frac{p-1}{2}, a_{i}\\right)$, if $\\frac{p-1}{2} \\leqslant i \\leqslant p-2$. We will have $10^{j} \\equiv-10^{i}$ $(\\bmod p)$, and so $a_{j} \\cdot 10^{j}=a_{i} \\cdot 10^{j} \\equiv-a_{i} \\cdot 10^{i}(\\bmod p)$. Notice that this move by Eduardo is always possible. Indeed, immediately before a move by Fernando, for any set of the type $\\{r, r+(p-1) \/ 2\\}$ with $0 \\leqslant r \\leqslant(p-3) \/ 2$, either no element of this set was chosen as an index by the players in the previous moves or else both elements of this set were chosen as indices by the players in the previous moves. Therefore, after each of his moves, Eduardo always makes the sum of the numbers $a_{k} \\cdot 10^{k}$ corresponding to the already chosen pairs $\\left(k, a_{k}\\right)$ divisible by $p$, and thus wins the game. Case b: $10^{(p-1) \/ 2} \\equiv 1(\\bmod p)$ In this case, for each move $\\left(i, a_{i}\\right)$ of Fernando, Eduardo immediately makes the move $\\left(j, a_{j}\\right)=$ $\\left(i+\\frac{p-1}{2}, 9-a_{i}\\right)$, if $0 \\leqslant i \\leqslant \\frac{p-3}{2}$, or $\\left(j, a_{j}\\right)=\\left(i-\\frac{p-1}{2}, 9-a_{i}\\right)$, if $\\frac{p-1}{2} \\leqslant i \\leqslant p-2$. The same argument as above shows that Eduardo can always make such move. We will have $10^{j} \\equiv 10^{i}$ $(\\bmod p)$, and so $a_{j} \\cdot 10^{j}+a_{i} \\cdot 10^{i} \\equiv\\left(a_{i}+a_{j}\\right) \\cdot 10^{i}=9 \\cdot 10^{i}(\\bmod p)$. Therefore, at the end of the game, the sum of all terms $a_{k} \\cdot 10^{k}$ will be congruent to $$ \\sum_{i=0}^{\\frac{p-3}{2}} 9 \\cdot 10^{i}=10^{(p-1) \/ 2}-1 \\equiv 0 \\quad(\\bmod p) $$ and Eduardo wins the game.","tier":0} +{"problem_type":"Number Theory","problem_label":"N3","problem":"Determine all integers $n \\geqslant 2$ with the following property: for any integers $a_{1}, a_{2}, \\ldots, a_{n}$ whose sum is not divisible by $n$, there exists an index $1 \\leqslant i \\leqslant n$ such that none of the numbers $$ a_{i}, a_{i}+a_{i+1}, \\ldots, a_{i}+a_{i+1}+\\cdots+a_{i+n-1} $$ is divisible by $n$. (We let $a_{i}=a_{i-n}$ when $i>n$.) (Thailand)","solution":"Let us first show that, if $n=a b$, with $a, b \\geqslant 2$ integers, then the property in the statement of the problem does not hold. Indeed, in this case, let $a_{k}=a$ for $1 \\leqslant k \\leqslant n-1$ and $a_{n}=0$. The sum $a_{1}+a_{2}+\\cdots+a_{n}=a \\cdot(n-1)$ is not divisible by $n$. Let $i$ with $1 \\leqslant i \\leqslant n$ be an arbitrary index. Taking $j=b$ if $1 \\leqslant i \\leqslant n-b$, and $j=b+1$ if $n-b0$ such that $10^{t} \\equiv 1$ $(\\bmod \\mathrm{cm})$ for some integer $c \\in C$, that is, the set of orders of 10 modulo cm . In other words, $$ S(m)=\\left\\{\\operatorname{ord}_{c m}(10): c \\in C\\right\\} $$ Since there are $4 \\cdot 201+3=807$ numbers $c$ with $1 \\leqslant c \\leqslant 2017$ and $\\operatorname{gcd}(c, 10)=1$, namely those such that $c \\equiv 1,3,7,9(\\bmod 10)$, $$ |S(m)| \\leqslant|C|=807 $$ Now we find $m$ such that $|S(m)|=807$. Let $$ P=\\{1

0}$. We will show that $k=c$. Denote by $\\nu_{p}(n)$ the number of prime factors $p$ in $n$, that is, the maximum exponent $\\beta$ for which $p^{\\beta} \\mid n$. For every $\\ell \\geqslant 1$ and $p \\in P$, the Lifting the Exponent Lemma provides $$ \\nu_{p}\\left(10^{\\ell \\alpha}-1\\right)=\\nu_{p}\\left(\\left(10^{\\alpha}\\right)^{\\ell}-1\\right)=\\nu_{p}\\left(10^{\\alpha}-1\\right)+\\nu_{p}(\\ell)=\\nu_{p}(m)+\\nu_{p}(\\ell) $$ so $$ \\begin{aligned} c m \\mid 10^{k \\alpha}-1 & \\Longleftrightarrow \\forall p \\in P ; \\nu_{p}(c m) \\leqslant \\nu_{p}\\left(10^{k \\alpha}-1\\right) \\\\ & \\Longleftrightarrow \\forall p \\in P ; \\nu_{p}(m)+\\nu_{p}(c) \\leqslant \\nu_{p}(m)+\\nu_{p}(k) \\\\ & \\Longleftrightarrow \\forall p \\in P ; \\nu_{p}(c) \\leqslant \\nu_{p}(k) \\\\ & \\Longleftrightarrow c \\mid k . \\end{aligned} $$ The first such $k$ is $k=c$, so $\\operatorname{ord}_{c m}(10)=c \\alpha$. Comment. The Lifting the Exponent Lemma states that, for any odd prime $p$, any integers $a, b$ coprime with $p$ such that $p \\mid a-b$, and any positive integer exponent $n$, $$ \\nu_{p}\\left(a^{n}-b^{n}\\right)=\\nu_{p}(a-b)+\\nu_{p}(n), $$ and, for $p=2$, $$ \\nu_{2}\\left(a^{n}-b^{n}\\right)=\\nu_{2}\\left(a^{2}-b^{2}\\right)+\\nu_{p}(n)-1 . $$ Both claims can be proved by induction on $n$.","tier":0} +{"problem_type":"Number Theory","problem_label":"N5","problem":"Find all pairs $(p, q)$ of prime numbers with $p>q$ for which the number $$ \\frac{(p+q)^{p+q}(p-q)^{p-q}-1}{(p+q)^{p-q}(p-q)^{p+q}-1} $$ is an integer.","solution":"Let $M=(p+q)^{p-q}(p-q)^{p+q}-1$, which is relatively prime with both $p+q$ and $p-q$. Denote by $(p-q)^{-1}$ the multiplicative inverse of $(p-q)$ modulo $M$. By eliminating the term -1 in the numerator, $$ \\begin{aligned} (p+q)^{p+q}(p-q)^{p-q}-1 & \\equiv(p+q)^{p-q}(p-q)^{p+q}-1 \\quad(\\bmod M) \\\\ (p+q)^{2 q} & \\equiv(p-q)^{2 q} \\quad(\\bmod M) \\\\ \\left((p+q) \\cdot(p-q)^{-1}\\right)^{2 q} & \\equiv 1 \\quad(\\bmod M) \\end{aligned} $$ Case 1: $q \\geqslant 5$. Consider an arbitrary prime divisor $r$ of $M$. Notice that $M$ is odd, so $r \\geqslant 3$. By (2), the multiplicative order of $\\left((p+q) \\cdot(p-q)^{-1}\\right)$ modulo $r$ is a divisor of the exponent $2 q$ in (2), so it can be $1,2, q$ or $2 q$. By Fermat's theorem, the order divides $r-1$. So, if the order is $q$ or $2 q$ then $r \\equiv 1(\\bmod q)$. If the order is 1 or 2 then $r \\mid(p+q)^{2}-(p-q)^{2}=4 p q$, so $r=p$ or $r=q$. The case $r=p$ is not possible, because, by applying Fermat's theorem, $M=(p+q)^{p-q}(p-q)^{p+q}-1 \\equiv q^{p-q}(-q)^{p+q}-1=\\left(q^{2}\\right)^{p}-1 \\equiv q^{2}-1=(q+1)(q-1) \\quad(\\bmod p)$ and the last factors $q-1$ and $q+1$ are less than $p$ and thus $p \\nmid M$. Hence, all prime divisors of $M$ are either $q$ or of the form $k q+1$; it follows that all positive divisors of $M$ are congruent to 0 or 1 modulo $q$. Now notice that $$ M=\\left((p+q)^{\\frac{p-q}{2}}(p-q)^{\\frac{p+q}{2}}-1\\right)\\left((p+q)^{\\frac{p-q}{2}}(p-q)^{\\frac{p+q}{2}}+1\\right) $$ is the product of two consecutive positive odd numbers; both should be congruent to 0 or 1 modulo $q$. But this is impossible by the assumption $q \\geqslant 5$. So, there is no solution in Case 1 . Case 2: $q=2$. By (1), we have $M \\mid(p+q)^{2 q}-(p-q)^{2 q}=(p+2)^{4}-(p-2)^{4}$, so $$ \\begin{gathered} (p+2)^{p-2}(p-2)^{p+2}-1=M \\leqslant(p+2)^{4}-(p-2)^{4} \\leqslant(p+2)^{4}-1, \\\\ (p+2)^{p-6}(p-2)^{p+2} \\leqslant 1 . \\end{gathered} $$ If $p \\geqslant 7$ then the left-hand side is obviously greater than 1 . For $p=5$ we have $(p+2)^{p-6}(p-2)^{p+2}=7^{-1} \\cdot 3^{7}$ which is also too large. There remains only one candidate, $p=3$, which provides a solution: $$ \\frac{(p+q)^{p+q}(p-q)^{p-q}-1}{(p+q)^{p-q}(p-q)^{p+q}-1}=\\frac{5^{5} \\cdot 1^{1}-1}{5^{1} \\cdot 1^{5}-1}=\\frac{3124}{4}=781 . $$ So in Case 2 the only solution is $(p, q)=(3,2)$. Case 3: $q=3$. Similarly to Case 2, we have $$ M \\left\\lvert\\,(p+q)^{2 q}-(p-q)^{2 q}=64 \\cdot\\left(\\left(\\frac{p+3}{2}\\right)^{6}-\\left(\\frac{p-3}{2}\\right)^{6}\\right)\\right. $$ Since $M$ is odd, we conclude that $$ M \\left\\lvert\\,\\left(\\frac{p+3}{2}\\right)^{6}-\\left(\\frac{p-3}{2}\\right)^{6}\\right. $$ and $$ \\begin{gathered} (p+3)^{p-3}(p-3)^{p+3}-1=M \\leqslant\\left(\\frac{p+3}{2}\\right)^{6}-\\left(\\frac{p-3}{2}\\right)^{6} \\leqslant\\left(\\frac{p+3}{2}\\right)^{6}-1 \\\\ 64(p+3)^{p-9}(p-3)^{p+3} \\leqslant 1 \\end{gathered} $$ If $p \\geqslant 11$ then the left-hand side is obviously greater than 1 . If $p=7$ then the left-hand side is $64 \\cdot 10^{-2} \\cdot 4^{10}>1$. If $p=5$ then the left-hand side is $64 \\cdot 8^{-4} \\cdot 2^{8}=2^{2}>1$. Therefore, there is no solution in Case 3.","tier":0} +{"problem_type":"Number Theory","problem_label":"N6","problem":"Find the smallest positive integer $n$, or show that no such $n$ exists, with the following property: there are infinitely many distinct $n$-tuples of positive rational numbers ( $a_{1}, a_{2}, \\ldots, a_{n}$ ) such that both $$ a_{1}+a_{2}+\\cdots+a_{n} \\quad \\text { and } \\quad \\frac{1}{a_{1}}+\\frac{1}{a_{2}}+\\cdots+\\frac{1}{a_{n}} $$ are integers. (Singapore)","solution":"For $n=1, a_{1} \\in \\mathbb{Z}_{>0}$ and $\\frac{1}{a_{1}} \\in \\mathbb{Z}_{>0}$ if and only if $a_{1}=1$. Next we show that (i) There are finitely many $(x, y) \\in \\mathbb{Q}_{>0}^{2}$ satisfying $x+y \\in \\mathbb{Z}$ and $\\frac{1}{x}+\\frac{1}{y} \\in \\mathbb{Z}$ Write $x=\\frac{a}{b}$ and $y=\\frac{c}{d}$ with $a, b, c, d \\in \\mathbb{Z}_{>0}$ and $\\operatorname{gcd}(a, b)=\\operatorname{gcd}(c, d)=1$. Then $x+y \\in \\mathbb{Z}$ and $\\frac{1}{x}+\\frac{1}{y} \\in \\mathbb{Z}$ is equivalent to the two divisibility conditions $$ b d \\mid a d+b c \\quad(1) \\quad \\text { and } \\quad a c \\mid a d+b c $$ Condition (1) implies that $d|a d+b c \\Longleftrightarrow d| b c \\Longleftrightarrow d \\mid b$ since $\\operatorname{gcd}(c, d)=1$. Still from (1) we get $b|a d+b c \\Longleftrightarrow b| a d \\Longleftrightarrow b \\mid d$ since $\\operatorname{gcd}(a, b)=1$. From $b \\mid d$ and $d \\mid b$ we have $b=d$. An analogous reasoning with condition (2) shows that $a=c$. Hence $x=\\frac{a}{b}=\\frac{c}{d}=y$, i.e., the problem amounts to finding all $x \\in \\mathbb{Q}_{>0}$ such that $2 x \\in \\mathbb{Z}_{>0}$ and $\\frac{2}{x} \\in \\mathbb{Z}_{>0}$. Letting $n=2 x \\in \\mathbb{Z}_{>0}$, we have that $\\frac{2}{x} \\in \\mathbb{Z}_{>0} \\Longleftrightarrow \\frac{4}{n} \\in \\mathbb{Z}_{>0} \\Longleftrightarrow n=1,2$ or 4 , and there are finitely many solutions, namely $(x, y)=\\left(\\frac{1}{2}, \\frac{1}{2}\\right),(1,1)$ or $(2,2)$. (ii) There are infinitely many triples $(x, y, z) \\in \\mathbb{Q}_{>0}^{2}$ such that $x+y+z \\in \\mathbb{Z}$ and $\\frac{1}{x}+\\frac{1}{y}+\\frac{1}{z} \\in \\mathbb{Z}$. We will look for triples such that $x+y+z=1$, so we may write them in the form $$ (x, y, z)=\\left(\\frac{a}{a+b+c}, \\frac{b}{a+b+c}, \\frac{c}{a+b+c}\\right) \\quad \\text { with } a, b, c \\in \\mathbb{Z}_{>0} $$ We want these to satisfy $$ \\frac{1}{x}+\\frac{1}{y}+\\frac{1}{z}=\\frac{a+b+c}{a}+\\frac{a+b+c}{b}+\\frac{a+b+c}{c} \\in \\mathbb{Z} \\Longleftrightarrow \\frac{b+c}{a}+\\frac{a+c}{b}+\\frac{a+b}{c} \\in \\mathbb{Z} $$ Fixing $a=1$, it suffices to find infinitely many pairs $(b, c) \\in \\mathbb{Z}_{>0}^{2}$ such that $$ \\frac{1}{b}+\\frac{1}{c}+\\frac{c}{b}+\\frac{b}{c}=3 \\Longleftrightarrow b^{2}+c^{2}-3 b c+b+c=0 $$ To show that equation (*) has infinitely many solutions, we use Vieta jumping (also known as root flipping): starting with $b=2, c=3$, the following algorithm generates infinitely many solutions. Let $c \\geqslant b$, and view (*) as a quadratic equation in $b$ for $c$ fixed: $$ b^{2}-(3 c-1) \\cdot b+\\left(c^{2}+c\\right)=0 $$ Then there exists another root $b_{0} \\in \\mathbb{Z}$ of ( $\\left.* *\\right)$ which satisfies $b+b_{0}=3 c-1$ and $b \\cdot b_{0}=c^{2}+c$. Since $c \\geqslant b$ by assumption, $$ b_{0}=\\frac{c^{2}+c}{b} \\geqslant \\frac{c^{2}+c}{c}>c $$ Hence from the solution $(b, c)$ we obtain another one $\\left(c, b_{0}\\right)$ with $b_{0}>c$, and we can then \"jump\" again, this time with $c$ as the \"variable\" in the quadratic (*). This algorithm will generate an infinite sequence of distinct solutions, whose first terms are $(2,3),(3,6),(6,14),(14,35),(35,90),(90,234),(234,611),(611,1598),(1598,4182), \\ldots$ Comment. Although not needed for solving this problem, we may also explicitly solve the recursion given by the Vieta jumping. Define the sequence $\\left(x_{n}\\right)$ as follows: $$ x_{0}=2, \\quad x_{1}=3 \\quad \\text { and } \\quad x_{n+2}=3 x_{n+1}-x_{n}-1 \\text { for } n \\geqslant 0 $$ Then the triple $$ (x, y, z)=\\left(\\frac{1}{1+x_{n}+x_{n+1}}, \\frac{x_{n}}{1+x_{n}+x_{n+1}}, \\frac{x_{n+1}}{1+x_{n}+x_{n+1}}\\right) $$ satisfies the problem conditions for all $n \\in \\mathbb{N}$. It is easy to show that $x_{n}=F_{2 n+1}+1$, where $F_{n}$ denotes the $n$-th term of the Fibonacci sequence ( $F_{0}=0, F_{1}=1$, and $F_{n+2}=F_{n+1}+F_{n}$ for $n \\geqslant 0$ ).","tier":0} +{"problem_type":"Number Theory","problem_label":"N6","problem":"Find the smallest positive integer $n$, or show that no such $n$ exists, with the following property: there are infinitely many distinct $n$-tuples of positive rational numbers ( $a_{1}, a_{2}, \\ldots, a_{n}$ ) such that both $$ a_{1}+a_{2}+\\cdots+a_{n} \\quad \\text { and } \\quad \\frac{1}{a_{1}}+\\frac{1}{a_{2}}+\\cdots+\\frac{1}{a_{n}} $$ are integers. (Singapore)","solution":"Call the $n$-tuples $\\left(a_{1}, a_{2}, \\ldots, a_{n}\\right) \\in \\mathbb{Q}_{>0}^{n}$ satisfying the conditions of the problem statement good, and those for which $$ f\\left(a_{1}, \\ldots, a_{n}\\right) \\stackrel{\\text { def }}{=}\\left(a_{1}+a_{2}+\\cdots+a_{n}\\right)\\left(\\frac{1}{a_{1}}+\\frac{1}{a_{2}}+\\cdots+\\frac{1}{a_{n}}\\right) $$ is an integer pretty. Then good $n$-tuples are pretty, and if $\\left(b_{1}, \\ldots, b_{n}\\right)$ is pretty then $$ \\left(\\frac{b_{1}}{b_{1}+b_{2}+\\cdots+b_{n}}, \\frac{b_{2}}{b_{1}+b_{2}+\\cdots+b_{n}}, \\ldots, \\frac{b_{n}}{b_{1}+b_{2}+\\cdots+b_{n}}\\right) $$ is good since the sum of its components is 1 , and the sum of the reciprocals of its components equals $f\\left(b_{1}, \\ldots, b_{n}\\right)$. We declare pretty $n$-tuples proportional to each other equivalent since they are precisely those which give rise to the same good $n$-tuple. Clearly, each such equivalence class contains exactly one $n$-tuple of positive integers having no common prime divisors. Call such $n$-tuple a primitive pretty tuple. Our task is to find infinitely many primitive pretty $n$-tuples. For $n=1$, there is clearly a single primitive 1 -tuple. For $n=2$, we have $f(a, b)=\\frac{(a+b)^{2}}{a b}$, which can be integral (for coprime $a, b \\in \\mathbb{Z}_{>0}$ ) only if $a=b=1$ (see for instance (i) in the first solution). Now we construct infinitely many primitive pretty triples for $n=3$. Fix $b, c, k \\in \\mathbb{Z}_{>0}$; we will try to find sufficient conditions for the existence of an $a \\in \\mathbb{Q}_{>0}$ such that $f(a, b, c)=k$. Write $\\sigma=b+c, \\tau=b c$. From $f(a, b, c)=k$, we have that $a$ should satisfy the quadratic equation $$ a^{2} \\cdot \\sigma+a \\cdot\\left(\\sigma^{2}-(k-1) \\tau\\right)+\\sigma \\tau=0 $$ whose discriminant is $$ \\Delta=\\left(\\sigma^{2}-(k-1) \\tau\\right)^{2}-4 \\sigma^{2} \\tau=\\left((k+1) \\tau-\\sigma^{2}\\right)^{2}-4 k \\tau^{2} $$ We need it to be a square of an integer, say, $\\Delta=M^{2}$ for some $M \\in \\mathbb{Z}$, i.e., we want $$ \\left((k+1) \\tau-\\sigma^{2}\\right)^{2}-M^{2}=2 k \\cdot 2 \\tau^{2} $$ so that it suffices to set $$ (k+1) \\tau-\\sigma^{2}=\\tau^{2}+k, \\quad M=\\tau^{2}-k . $$ The first relation reads $\\sigma^{2}=(\\tau-1)(k-\\tau)$, so if $b$ and $c$ satisfy $$ \\tau-1 \\mid \\sigma^{2} \\quad \\text { i.e. } \\quad b c-1 \\mid(b+c)^{2} $$ then $k=\\frac{\\sigma^{2}}{\\tau-1}+\\tau$ will be integral, and we find rational solutions to (1), namely $$ a=\\frac{\\sigma}{\\tau-1}=\\frac{b+c}{b c-1} \\quad \\text { or } \\quad a=\\frac{\\tau^{2}-\\tau}{\\sigma}=\\frac{b c \\cdot(b c-1)}{b+c} $$ We can now find infinitely many pairs ( $b, c$ ) satisfying (2) by Vieta jumping. For example, if we impose $$ (b+c)^{2}=5 \\cdot(b c-1) $$ then all pairs $(b, c)=\\left(v_{i}, v_{i+1}\\right)$ satisfy the above condition, where $$ v_{1}=2, v_{2}=3, \\quad v_{i+2}=3 v_{i+1}-v_{i} \\quad \\text { for } i \\geqslant 0 $$ For $(b, c)=\\left(v_{i}, v_{i+1}\\right)$, one of the solutions to (1) will be $a=(b+c) \/(b c-1)=5 \/(b+c)=$ $5 \/\\left(v_{i}+v_{i+1}\\right)$. Then the pretty triple $(a, b, c)$ will be equivalent to the integral pretty triple $$ \\left(5, v_{i}\\left(v_{i}+v_{i+1}\\right), v_{i+1}\\left(v_{i}+v_{i+1}\\right)\\right) $$ After possibly dividing by 5 , we obtain infinitely many primitive pretty triples, as required. Comment. There are many other infinite series of $(b, c)=\\left(v_{i}, v_{i+1}\\right)$ with $b c-1 \\mid(b+c)^{2}$. Some of them are: $$ \\begin{array}{llll} v_{1}=1, & v_{2}=3, & v_{i+1}=6 v_{i}-v_{i-1}, & \\left(v_{i}+v_{i+1}\\right)^{2}=8 \\cdot\\left(v_{i} v_{i+1}-1\\right) ; \\\\ v_{1}=1, & v_{2}=2, & v_{i+1}=7 v_{i}-v_{i-1}, & \\left(v_{i}+v_{i+1}\\right)^{2}=9 \\cdot\\left(v_{i} v_{i+1}-1\\right) ; \\\\ v_{1}=1, & v_{2}=5, & v_{i+1}=7 v_{i}-v_{i-1}, & \\left(v_{i}+v_{i+1}\\right)^{2}=9 \\cdot\\left(v_{i} v_{i+1}-1\\right) \\end{array} $$ (the last two are in fact one sequence prolonged in two possible directions).","tier":0} +{"problem_type":"Number Theory","problem_label":"N7","problem":"Say that an ordered pair $(x, y)$ of integers is an irreducible lattice point if $x$ and $y$ are relatively prime. For any finite set $S$ of irreducible lattice points, show that there is a homogenous polynomial in two variables, $f(x, y)$, with integer coefficients, of degree at least 1 , such that $f(x, y)=1$ for each $(x, y)$ in the set $S$. Note: A homogenous polynomial of degree $n$ is any nonzero polynomial of the form $$ f(x, y)=a_{0} x^{n}+a_{1} x^{n-1} y+a_{2} x^{n-2} y^{2}+\\cdots+a_{n-1} x y^{n-1}+a_{n} y^{n} . $$","solution":"First of all, we note that finding a homogenous polynomial $f(x, y)$ such that $f(x, y)= \\pm 1$ is enough, because we then have $f^{2}(x, y)=1$. Label the irreducible lattice points $\\left(x_{1}, y_{1}\\right)$ through $\\left(x_{n}, y_{n}\\right)$. If any two of these lattice points $\\left(x_{i}, y_{i}\\right)$ and $\\left(x_{j}, y_{j}\\right)$ lie on the same line through the origin, then $\\left(x_{j}, y_{j}\\right)=\\left(-x_{i},-y_{i}\\right)$ because both of the points are irreducible. We then have $f\\left(x_{j}, y_{j}\\right)= \\pm f\\left(x_{i}, y_{i}\\right)$ whenever $f$ is homogenous, so we can assume that no two of the lattice points are collinear with the origin by ignoring the extra lattice points. Consider the homogenous polynomials $\\ell_{i}(x, y)=y_{i} x-x_{i} y$ and define $$ g_{i}(x, y)=\\prod_{j \\neq i} \\ell_{j}(x, y) $$ Then $\\ell_{i}\\left(x_{j}, y_{j}\\right)=0$ if and only if $j=i$, because there is only one lattice point on each line through the origin. Thus, $g_{i}\\left(x_{j}, y_{j}\\right)=0$ for all $j \\neq i$. Define $a_{i}=g_{i}\\left(x_{i}, y_{i}\\right)$, and note that $a_{i} \\neq 0$. Note that $g_{i}(x, y)$ is a degree $n-1$ polynomial with the following two properties: 1. $g_{i}\\left(x_{j}, y_{j}\\right)=0$ if $j \\neq i$. 2. $g_{i}\\left(x_{i}, y_{i}\\right)=a_{i}$. For any $N \\geqslant n-1$, there also exists a polynomial of degree $N$ with the same two properties. Specifically, let $I_{i}(x, y)$ be a degree 1 homogenous polynomial such that $I_{i}\\left(x_{i}, y_{i}\\right)=1$, which exists since $\\left(x_{i}, y_{i}\\right)$ is irreducible. Then $I_{i}(x, y)^{N-(n-1)} g_{i}(x, y)$ satisfies both of the above properties and has degree $N$. We may now reduce the problem to the following claim: Claim: For each positive integer $a$, there is a homogenous polynomial $f_{a}(x, y)$, with integer coefficients, of degree at least 1 , such that $f_{a}(x, y) \\equiv 1(\\bmod a)$ for all relatively prime $(x, y)$. To see that this claim solves the problem, take $a$ to be the least common multiple of the numbers $a_{i}(1 \\leqslant i \\leqslant n)$. Take $f_{a}$ given by the claim, choose some power $f_{a}(x, y)^{k}$ that has degree at least $n-1$, and subtract appropriate multiples of the $g_{i}$ constructed above to obtain the desired polynomial. We prove the claim by factoring $a$. First, if $a$ is a power of a prime ( $a=p^{k}$ ), then we may choose either: - $f_{a}(x, y)=\\left(x^{p-1}+y^{p-1}\\right)^{\\phi(a)}$ if $p$ is odd; - $f_{a}(x, y)=\\left(x^{2}+x y+y^{2}\\right)^{\\phi(a)}$ if $p=2$. Now suppose $a$ is any positive integer, and let $a=q_{1} q_{2} \\cdots q_{k}$, where the $q_{i}$ are prime powers, pairwise relatively prime. Let $f_{q_{i}}$ be the polynomials just constructed, and let $F_{q_{i}}$ be powers of these that all have the same degree. Note that $$ \\frac{a}{q_{i}} F_{q_{i}}(x, y) \\equiv \\frac{a}{q_{i}} \\quad(\\bmod a) $$ for any relatively prime $x, y$. By B\u00e9zout's lemma, there is an integer linear combination of the $\\frac{a}{q_{i}}$ that equals 1 . Thus, there is a linear combination of the $F_{q_{i}}$ such that $F_{q_{i}}(x, y) \\equiv 1$ $(\\bmod a)$ for any relatively prime $(x, y)$; and this polynomial is homogenous because all the $F_{q_{i}}$ have the same degree.","tier":0} +{"problem_type":"Number Theory","problem_label":"N7","problem":"Say that an ordered pair $(x, y)$ of integers is an irreducible lattice point if $x$ and $y$ are relatively prime. For any finite set $S$ of irreducible lattice points, show that there is a homogenous polynomial in two variables, $f(x, y)$, with integer coefficients, of degree at least 1 , such that $f(x, y)=1$ for each $(x, y)$ in the set $S$. Note: A homogenous polynomial of degree $n$ is any nonzero polynomial of the form $$ f(x, y)=a_{0} x^{n}+a_{1} x^{n-1} y+a_{2} x^{n-2} y^{2}+\\cdots+a_{n-1} x y^{n-1}+a_{n} y^{n} . $$","solution":"As in the previous solution, label the irreducible lattice points $\\left(x_{1}, y_{1}\\right), \\ldots,\\left(x_{n}, y_{n}\\right)$ and assume without loss of generality that no two of the points are collinear with the origin. We induct on $n$ to construct a homogenous polynomial $f(x, y)$ such that $f\\left(x_{i}, y_{i}\\right)=1$ for all $1 \\leqslant i \\leqslant n$. If $n=1$ : Since $x_{1}$ and $y_{1}$ are relatively prime, there exist some integers $c, d$ such that $c x_{1}+d y_{1}=1$. Then $f(x, y)=c x+d y$ is suitable. If $n \\geqslant 2$ : By the induction hypothesis we already have a homogeneous polynomial $g(x, y)$ with $g\\left(x_{1}, y_{1}\\right)=\\ldots=g\\left(x_{n-1}, y_{n-1}\\right)=1$. Let $j=\\operatorname{deg} g$, $$ g_{n}(x, y)=\\prod_{k=1}^{n-1}\\left(y_{k} x-x_{k} y\\right) $$ and $a_{n}=g_{n}\\left(x_{n}, y_{n}\\right)$. By assumption, $a_{n} \\neq 0$. Take some integers $c, d$ such that $c x_{n}+d y_{n}=1$. We will construct $f(x, y)$ in the form $$ f(x, y)=g(x, y)^{K}-C \\cdot g_{n}(x, y) \\cdot(c x+d y)^{L} $$ where $K$ and $L$ are some positive integers and $C$ is some integer. We assume that $L=K j-n+1$ so that $f$ is homogenous. Due to $g\\left(x_{1}, y_{1}\\right)=\\ldots=g\\left(x_{n-1}, y_{n-1}\\right)=1$ and $g_{n}\\left(x_{1}, y_{1}\\right)=\\ldots=g_{n}\\left(x_{n-1}, y_{n-1}\\right)=0$, the property $f\\left(x_{1}, y_{1}\\right)=\\ldots=f\\left(x_{n-1}, y_{n-1}\\right)=1$ is automatically satisfied with any choice of $K, L$, and $C$. Furthermore, $$ f\\left(x_{n}, y_{n}\\right)=g\\left(x_{n}, y_{n}\\right)^{K}-C \\cdot g_{n}\\left(x_{n}, y_{n}\\right) \\cdot\\left(c x_{n}+d y_{n}\\right)^{L}=g\\left(x_{n}, y_{n}\\right)^{K}-C a_{n} $$ If we have an exponent $K$ such that $g\\left(x_{n}, y_{n}\\right)^{K} \\equiv 1\\left(\\bmod a_{n}\\right)$, then we may choose $C$ such that $f\\left(x_{n}, y_{n}\\right)=1$. We now choose such a $K$. Consider an arbitrary prime divisor $p$ of $a_{n}$. By $$ p \\mid a_{n}=g_{n}\\left(x_{n}, y_{n}\\right)=\\prod_{k=1}^{n-1}\\left(y_{k} x_{n}-x_{k} y_{n}\\right) $$ there is some $1 \\leqslant k0$.) Comment. It is possible to show that there is no constant $C$ for which, given any two irreducible lattice points, there is some homogenous polynomial $f$ of degree at most $C$ with integer coefficients that takes the value 1 on the two points. Indeed, if one of the points is $(1,0)$ and the other is $(a, b)$, the polynomial $f(x, y)=a_{0} x^{n}+a_{1} x^{n-1} y+\\cdots+a_{n} y^{n}$ should satisfy $a_{0}=1$, and so $a^{n} \\equiv 1(\\bmod b)$. If $a=3$ and $b=2^{k}$ with $k \\geqslant 3$, then $n \\geqslant 2^{k-2}$. If we choose $2^{k-2}>C$, this gives a contradiction.","tier":0} +{"problem_type":"Number Theory","problem_label":"N8","problem":"Let $p$ be an odd prime number and $\\mathbb{Z}_{>0}$ be the set of positive integers. Suppose that a function $f: \\mathbb{Z}_{>0} \\times \\mathbb{Z}_{>0} \\rightarrow\\{0,1\\}$ satisfies the following properties: - $f(1,1)=0$; - $f(a, b)+f(b, a)=1$ for any pair of relatively prime positive integers $(a, b)$ not both equal to 1 ; - $f(a+b, b)=f(a, b)$ for any pair of relatively prime positive integers $(a, b)$. Prove that $$ \\sum_{n=1}^{p-1} f\\left(n^{2}, p\\right) \\geqslant \\sqrt{2 p}-2 $$","solution":"Denote by $\\mathbb{A}$ the set of all pairs of coprime positive integers. Notice that for every $(a, b) \\in \\mathbb{A}$ there exists a pair $(u, v) \\in \\mathbb{Z}^{2}$ with $u a+v b=1$. Moreover, if $\\left(u_{0}, v_{0}\\right)$ is one such pair, then all such pairs are of the form $(u, v)=\\left(u_{0}+k b, v_{0}-k a\\right)$, where $k \\in \\mathbb{Z}$. So there exists a unique such pair $(u, v)$ with $-b \/ 20$. Proof. We induct on $a+b$. The base case is $a+b=2$. In this case, we have that $a=b=1$, $g(a, b)=g(1,1)=(0,1)$ and $f(1,1)=0$, so the claim holds. Assume now that $a+b>2$, and so $a \\neq b$, since $a$ and $b$ are coprime. Two cases are possible. Case 1: $a>b$. Notice that $g(a-b, b)=(u, v+u)$, since $u(a-b)+(v+u) b=1$ and $u \\in(-b \/ 2, b \/ 2]$. Thus $f(a, b)=1 \\Longleftrightarrow f(a-b, b)=1 \\Longleftrightarrow u>0$ by the induction hypothesis. Case 2: $av b \\geqslant 1-\\frac{a b}{2}, \\quad \\text { so } \\quad \\frac{1+a}{2} \\geqslant \\frac{1}{b}+\\frac{a}{2}>v \\geqslant \\frac{1}{b}-\\frac{a}{2}>-\\frac{a}{2} . $$ Thus $1+a>2 v>-a$, so $a \\geqslant 2 v>-a$, hence $a \/ 2 \\geqslant v>-a \/ 2$, and thus $g(b, a)=(v, u)$. Observe that $f(a, b)=1 \\Longleftrightarrow f(b, a)=0 \\Longleftrightarrow f(b-a, a)=0$. We know from Case 1 that $g(b-a, a)=(v, u+v)$. We have $f(b-a, a)=0 \\Longleftrightarrow v \\leqslant 0$ by the inductive hypothesis. Then, since $b>a \\geqslant 1$ and $u a+v b=1$, we have $v \\leqslant 0 \\Longleftrightarrow u>0$, and we are done. The Lemma proves that, for all $(a, b) \\in \\mathbb{A}, f(a, b)=1$ if and only if the inverse of $a$ modulo $b$, taken in $\\{1,2, \\ldots, b-1\\}$, is at most $b \/ 2$. Then, for any odd prime $p$ and integer $n$ such that $n \\not \\equiv 0(\\bmod p), f\\left(n^{2}, p\\right)=1$ iff the inverse of $n^{2} \\bmod p$ is less than $p \/ 2$. Since $\\left\\{n^{2} \\bmod p: 1 \\leqslant n \\leqslant p-1\\right\\}=\\left\\{n^{-2} \\bmod p: 1 \\leqslant n \\leqslant p-1\\right\\}$, including multiplicities (two for each quadratic residue in each set), we conclude that the desired sum is twice the number of quadratic residues that are less than $p \/ 2$, i.e., $$ \\left.\\sum_{n=1}^{p-1} f\\left(n^{2}, p\\right)=2 \\left\\lvert\\,\\left\\{k: 1 \\leqslant k \\leqslant \\frac{p-1}{2} \\text { and } k^{2} \\bmod p<\\frac{p}{2}\\right\\}\\right. \\right\\rvert\\, . $$ Since the number of perfect squares in the interval $[1, p \/ 2)$ is $\\lfloor\\sqrt{p \/ 2}\\rfloor>\\sqrt{p \/ 2}-1$, we conclude that $$ \\sum_{n=1}^{p-1} f\\left(n^{2}, p\\right)>2\\left(\\sqrt{\\frac{p}{2}}-1\\right)=\\sqrt{2 p}-2 $$","tier":0} +{"problem_type":"Number Theory","problem_label":"N8","problem":"Let $p$ be an odd prime number and $\\mathbb{Z}_{>0}$ be the set of positive integers. Suppose that a function $f: \\mathbb{Z}_{>0} \\times \\mathbb{Z}_{>0} \\rightarrow\\{0,1\\}$ satisfies the following properties: - $f(1,1)=0$; - $f(a, b)+f(b, a)=1$ for any pair of relatively prime positive integers $(a, b)$ not both equal to 1 ; - $f(a+b, b)=f(a, b)$ for any pair of relatively prime positive integers $(a, b)$. Prove that $$ \\sum_{n=1}^{p-1} f\\left(n^{2}, p\\right) \\geqslant \\sqrt{2 p}-2 $$","solution":"We provide a different proof for the Lemma. For this purpose, we use continued fractions to find $g(a, b)=(u, v)$ explicitly. The function $f$ is completely determined on $\\mathbb{A}$ by the following Claim. Represent $a \/ b$ as a continued fraction; that is, let $a_{0}$ be an integer and $a_{1}, \\ldots, a_{k}$ be positive integers such that $a_{k} \\geqslant 2$ and $$ \\frac{a}{b}=a_{0}+\\frac{1}{a_{1}+\\frac{1}{a_{2}+\\frac{1}{\\cdots+\\frac{1}{a_{k}}}}}=\\left[a_{0} ; a_{1}, a_{2}, \\ldots, a_{k}\\right] . $$ Then $f(a, b)=0 \\Longleftrightarrow k$ is even. Proof. We induct on $b$. If $b=1$, then $a \/ b=[a]$ and $k=0$. Then, for $a \\geqslant 1$, an easy induction shows that $f(a, 1)=f(1,1)=0$. Now consider the case $b>1$. Perform the Euclidean division $a=q b+r$, with $0 \\leqslant r0$ and define $q_{-1}=0$ if necessary. Then - $q_{k}=a_{k} q_{k-1}+q_{k-2}, \\quad$ and - $a q_{k-1}-b p_{k-1}=p_{k} q_{k-1}-q_{k} p_{k-1}=(-1)^{k-1}$. Assume that $k>0$. Then $a_{k} \\geqslant 2$, and $$ b=q_{k}=a_{k} q_{k-1}+q_{k-2} \\geqslant a_{k} q_{k-1} \\geqslant 2 q_{k-1} \\Longrightarrow q_{k-1} \\leqslant \\frac{b}{2} $$ with strict inequality for $k>1$, and $$ (-1)^{k-1} q_{k-1} a+(-1)^{k} p_{k-1} b=1 $$ Now we finish the proof of the Lemma. It is immediate for $k=0$. If $k=1$, then $(-1)^{k-1}=1$, so $$ -b \/ 2<0 \\leqslant(-1)^{k-1} q_{k-1} \\leqslant b \/ 2 $$ If $k>1$, we have $q_{k-1}0$, we find that $g(a, b)=\\left((-1)^{k-1} q_{k-1},(-1)^{k} p_{k-1}\\right)$, and so $$ f(a, b)=1 \\Longleftrightarrow k \\text { is odd } \\Longleftrightarrow u=(-1)^{k-1} q_{k-1}>0 $$ Comment 1. The Lemma can also be established by observing that $f$ is uniquely defined on $\\mathbb{A}$, defining $f_{1}(a, b)=1$ if $u>0$ in $g(a, b)=(u, v)$ and $f_{1}(a, b)=0$ otherwise, and verifying that $f_{1}$ satisfies all the conditions from the statement. It seems that the main difficulty of the problem is in conjecturing the Lemma. Comment 2. The case $p \\equiv 1(\\bmod 4)$ is, in fact, easier than the original problem. We have, in general, for $1 \\leqslant a \\leqslant p-1$, $f(a, p)=1-f(p, a)=1-f(p-a, a)=f(a, p-a)=f(a+(p-a), p-a)=f(p, p-a)=1-f(p-a, p)$. If $p \\equiv 1(\\bmod 4)$, then $a$ is a quadratic residue modulo $p$ if and only if $p-a$ is a quadratic residue modulo $p$. Therefore, denoting by $r_{k}$ (with $1 \\leqslant r_{k} \\leqslant p-1$ ) the remainder of the division of $k^{2}$ by $p$, we get $$ \\sum_{n=1}^{p-1} f\\left(n^{2}, p\\right)=\\sum_{n=1}^{p-1} f\\left(r_{n}, p\\right)=\\frac{1}{2} \\sum_{n=1}^{p-1}\\left(f\\left(r_{n}, p\\right)+f\\left(p-r_{n}, p\\right)\\right)=\\frac{p-1}{2} $$ Comment 3. The estimate for the sum $\\sum_{n=1}^{p} f\\left(n^{2}, p\\right)$ can be improved by refining the final argument in Solution 1. In fact, one can prove that $$ \\sum_{n=1}^{p-1} f\\left(n^{2}, p\\right) \\geqslant \\frac{p-1}{16} $$ By counting the number of perfect squares in the intervals $[k p,(k+1 \/ 2) p)$, we find that $$ \\sum_{n=1}^{p-1} f\\left(n^{2}, p\\right)=\\sum_{k=0}^{p-1}\\left(\\left\\lfloor\\sqrt{\\left(k+\\frac{1}{2}\\right) p}\\right\\rfloor-\\lfloor\\sqrt{k p}\\rfloor\\right) . $$ Each summand of (2) is non-negative. We now estimate the number of positive summands. Suppose that a summand is zero, i.e., $$ \\left\\lfloor\\sqrt{\\left(k+\\frac{1}{2}\\right) p}\\right\\rfloor=\\lfloor\\sqrt{k p}\\rfloor=: q . $$ Then both of the numbers $k p$ and $k p+p \/ 2$ lie within the interval $\\left[q^{2},(q+1)^{2}\\right)$. Hence $$ \\frac{p}{2}<(q+1)^{2}-q^{2} $$ which implies $$ q \\geqslant \\frac{p-1}{4} $$ Since $q \\leqslant \\sqrt{k p}$, if the $k^{\\text {th }}$ summand of (2) is zero, then $$ k \\geqslant \\frac{q^{2}}{p} \\geqslant \\frac{(p-1)^{2}}{16 p}>\\frac{p-2}{16} \\Longrightarrow k \\geqslant \\frac{p-1}{16} $$ So at least the first $\\left\\lceil\\frac{p-1}{16}\\right\\rceil$ summands (from $k=0$ to $k=\\left\\lceil\\frac{p-1}{16}\\right\\rceil-1$ ) are positive, and the result follows. Comment 4. The bound can be further improved by using different methods. In fact, we prove that $$ \\sum_{n=1}^{p-1} f\\left(n^{2}, p\\right) \\geqslant \\frac{p-3}{4} $$ To that end, we use the Legendre symbol $$ \\left(\\frac{a}{p}\\right)= \\begin{cases}0 & \\text { if } p \\mid a \\\\ 1 & \\text { if } a \\text { is a nonzero quadratic residue } \\bmod p \\\\ -1 & \\text { otherwise. }\\end{cases} $$ We start with the following Claim, which tells us that there are not too many consecutive quadratic residues or consecutive quadratic non-residues. Claim. $\\sum_{n=1}^{p-1}\\left(\\frac{n}{p}\\right)\\left(\\frac{n+1}{p}\\right)=-1$. Proof. We have $\\left(\\frac{n}{p}\\right)\\left(\\frac{n+1}{p}\\right)=\\left(\\frac{n(n+1)}{p}\\right)$. For $1 \\leqslant n \\leqslant p-1$, we get that $n(n+1) \\equiv n^{2}\\left(1+n^{-1}\\right)(\\bmod p)$, hence $\\left(\\frac{n(n+1)}{p}\\right)=\\left(\\frac{1+n^{-1}}{p}\\right)$. Since $\\left\\{1+n^{-1} \\bmod p: 1 \\leqslant n \\leqslant p-1\\right\\}=\\{0,2,3, \\ldots, p-1 \\bmod p\\}$, we find $$ \\sum_{n=1}^{p-1}\\left(\\frac{n}{p}\\right)\\left(\\frac{n+1}{p}\\right)=\\sum_{n=1}^{p-1}\\left(\\frac{1+n^{-1}}{p}\\right)=\\sum_{n=1}^{p-1}\\left(\\frac{n}{p}\\right)-1=-1 $$ because $\\sum_{n=1}^{p}\\left(\\frac{n}{p}\\right)=0$. Observe that (1) becomes $$ \\sum_{n=1}^{p-1} f\\left(n^{2}, p\\right)=2|S|, \\quad S=\\left\\{r: 1 \\leqslant r \\leqslant \\frac{p-1}{2} \\text { and }\\left(\\frac{r}{p}\\right)=1\\right\\} $$ We connect $S$ with the sum from the claim by pairing quadratic residues and quadratic non-residues. To that end, define $$ \\begin{aligned} S^{\\prime} & =\\left\\{r: 1 \\leqslant r \\leqslant \\frac{p-1}{2} \\text { and }\\left(\\frac{r}{p}\\right)=-1\\right\\} \\\\ T & =\\left\\{r: \\frac{p+1}{2} \\leqslant r \\leqslant p-1 \\text { and }\\left(\\frac{r}{p}\\right)=1\\right\\} \\\\ T^{\\prime} & =\\left\\{r: \\frac{p+1}{2} \\leqslant r \\leqslant p-1 \\text { and }\\left(\\frac{r}{p}\\right)=-1\\right\\} \\end{aligned} $$ Since there are exactly $(p-1) \/ 2$ nonzero quadratic residues modulo $p,|S|+|T|=(p-1) \/ 2$. Also we obviously have $|T|+\\left|T^{\\prime}\\right|=(p-1) \/ 2$. Then $|S|=\\left|T^{\\prime}\\right|$. For the sake of brevity, define $t=|S|=\\left|T^{\\prime}\\right|$. If $\\left(\\frac{n}{p}\\right)\\left(\\frac{n+1}{p}\\right)=-1$, then exactly of one the numbers $\\left(\\frac{n}{p}\\right)$ and $\\left(\\frac{n+1}{p}\\right)$ is equal to 1 , so $$ \\left\\lvert\\,\\left\\{n: 1 \\leqslant n \\leqslant \\frac{p-3}{2} \\text { and }\\left(\\frac{n}{p}\\right)\\left(\\frac{n+1}{p}\\right)=-1\\right\\}|\\leqslant|S|+|S-1|=2 t\\right. $$ On the other hand, if $\\left(\\frac{n}{p}\\right)\\left(\\frac{n+1}{p}\\right)=-1$, then exactly one of $\\left(\\frac{n}{p}\\right)$ and $\\left(\\frac{n+1}{p}\\right)$ is equal to -1 , and $$ \\left\\lvert\\,\\left\\{n: \\frac{p+1}{2} \\leqslant n \\leqslant p-2 \\text { and }\\left(\\frac{n}{p}\\right)\\left(\\frac{n+1}{p}\\right)=-1\\right\\}\\left|\\leqslant\\left|T^{\\prime}\\right|+\\left|T^{\\prime}-1\\right|=2 t .\\right.\\right. $$ Thus, taking into account that the middle term $\\left(\\frac{(p-1) \/ 2}{p}\\right)\\left(\\frac{(p+1) \/ 2}{p}\\right)$ may happen to be -1 , $$ \\left.\\left\\lvert\\,\\left\\{n: 1 \\leqslant n \\leqslant p-2 \\text { and }\\left(\\frac{n}{p}\\right)\\left(\\frac{n+1}{p}\\right)=-1\\right\\}\\right. \\right\\rvert\\, \\leqslant 4 t+1 $$ This implies that $$ \\left.\\left\\lvert\\,\\left\\{n: 1 \\leqslant n \\leqslant p-2 \\text { and }\\left(\\frac{n}{p}\\right)\\left(\\frac{n+1}{p}\\right)=1\\right\\}\\right. \\right\\rvert\\, \\geqslant(p-2)-(4 t+1)=p-4 t-3 $$ and so $$ -1=\\sum_{n=1}^{p-1}\\left(\\frac{n}{p}\\right)\\left(\\frac{n+1}{p}\\right) \\geqslant p-4 t-3-(4 t+1)=p-8 t-4 $$ which implies $8 t \\geqslant p-3$, and thus $$ \\sum_{n=1}^{p-1} f\\left(n^{2}, p\\right)=2 t \\geqslant \\frac{p-3}{4} $$ Comment 5. It is possible to prove that $$ \\sum_{n=1}^{p-1} f\\left(n^{2}, p\\right) \\geqslant \\frac{p-1}{2} $$ The case $p \\equiv 1(\\bmod 4)$ was already mentioned, and it is the equality case. If $p \\equiv 3(\\bmod 4)$, then, by a theorem of Dirichlet, we have $$ \\left.\\left\\lvert\\,\\left\\{r: 1 \\leqslant r \\leqslant \\frac{p-1}{2} \\text { and }\\left(\\frac{r}{p}\\right)=1\\right\\}\\right. \\right\\rvert\\,>\\frac{p-1}{4} $$ which implies the result. See https:\/\/en.wikipedia.org\/wiki\/Quadratic_residue\\#Dirichlet.27s_formulas for the full statement of the theorem. It seems that no elementary proof of it is known; a proof using complex analysis is available, for instance, in Chapter 7 of the book Quadratic Residues and Non-Residues: Selected Topics, by Steve Wright, available in https:\/\/arxiv.org\/abs\/1408.0235. ![](https:\/\/cdn.mathpix.com\/cropped\/2024_11_18_ff08e3c077fa2f8c16b2g-96.jpg?height=138&width=204&top_left_y=2241&top_left_x=315) BI\u00caNIO DA MATEMATICA BRASIL ![](https:\/\/cdn.mathpix.com\/cropped\/2024_11_18_ff08e3c077fa2f8c16b2g-96.jpg?height=249&width=683&top_left_y=2251&top_left_x=1052) ![](https:\/\/cdn.mathpix.com\/cropped\/2024_11_18_ff08e3c077fa2f8c16b2g-96.jpg?height=132&width=338&top_left_y=2607&top_left_x=1436) [^0]: *The name Dirichlet interval is chosen for the reason that $g$ theoretically might act similarly to the Dirichlet function on this interval.","tier":0} +{"problem_type":"Algebra","problem_label":"A2","problem":"Let $q$ be a real number. Gugu has a napkin with ten distinct real numbers written on it, and he writes the following three lines of real numbers on the blackboard: - In the first line, Gugu writes down every number of the form $a-b$, where $a$ and $b$ are two (not necessarily distinct) numbers on his napkin. - In the second line, Gugu writes down every number of the form qab, where $a$ and $b$ are two (not necessarily distinct) numbers from the first line. - In the third line, Gugu writes down every number of the form $a^{2}+b^{2}-c^{2}-d^{2}$, where $a, b, c, d$ are four (not necessarily distinct) numbers from the first line. Determine all values of $q$ such that, regardless of the numbers on Gugu's napkin, every number in the second line is also a number in the third line. (Austria) Answer: -2, 0,2 .","solution":"Call a number $q$ good if every number in the second line appears in the third line unconditionally. We first show that the numbers 0 and $\\pm 2$ are good. The third line necessarily contains 0 , so 0 is good. For any two numbers $a, b$ in the first line, write $a=x-y$ and $b=u-v$, where $x, y, u, v$ are (not necessarily distinct) numbers on the napkin. We may now write $$ 2 a b=2(x-y)(u-v)=(x-v)^{2}+(y-u)^{2}-(x-u)^{2}-(y-v)^{2} $$ which shows that 2 is good. By negating both sides of the above equation, we also see that -2 is good. We now show that $-2,0$, and 2 are the only good numbers. Assume for sake of contradiction that $q$ is a good number, where $q \\notin\\{-2,0,2\\}$. We now consider some particular choices of numbers on Gugu's napkin to arrive at a contradiction. Assume that the napkin contains the integers $1,2, \\ldots, 10$. Then, the first line contains the integers $-9,-8, \\ldots, 9$. The second line then contains $q$ and $81 q$, so the third line must also contain both of them. But the third line only contains integers, so $q$ must be an integer. Furthermore, the third line contains no number greater than $162=9^{2}+9^{2}-0^{2}-0^{2}$ or less than -162 , so we must have $-162 \\leqslant 81 q \\leqslant 162$. This shows that the only possibilities for $q$ are $\\pm 1$. Now assume that $q= \\pm 1$. Let the napkin contain $0,1,4,8,12,16,20,24,28,32$. The first line contains $\\pm 1$ and $\\pm 4$, so the second line contains $\\pm 4$. However, for every number $a$ in the first line, $a \\not \\equiv 2(\\bmod 4)$, so we may conclude that $a^{2} \\equiv 0,1(\\bmod 8)$. Consequently, every number in the third line must be congruent to $-2,-1,0,1,2(\\bmod 8)$; in particular, $\\pm 4$ cannot be in the third line, which is a contradiction.","tier":0} +{"problem_type":"Algebra","problem_label":"A2","problem":"Let $q$ be a real number. Gugu has a napkin with ten distinct real numbers written on it, and he writes the following three lines of real numbers on the blackboard: - In the first line, Gugu writes down every number of the form $a-b$, where $a$ and $b$ are two (not necessarily distinct) numbers on his napkin. - In the second line, Gugu writes down every number of the form qab, where $a$ and $b$ are two (not necessarily distinct) numbers from the first line. - In the third line, Gugu writes down every number of the form $a^{2}+b^{2}-c^{2}-d^{2}$, where $a, b, c, d$ are four (not necessarily distinct) numbers from the first line. Determine all values of $q$ such that, regardless of the numbers on Gugu's napkin, every number in the second line is also a number in the third line. (Austria) Answer: -2, 0,2 .","solution":"Let $q$ be a good number, as defined in the first solution, and define the polynomial $P\\left(x_{1}, \\ldots, x_{10}\\right)$ as $$ \\prod_{i2017 $$ Prove that this sequence is bounded, i.e., there is a constant $M$ such that $\\left|a_{n}\\right| \\leqslant M$ for all positive integers $n$. (Russia)","solution":"Set $D=2017$. Denote $$ M_{n}=\\max _{kD$; our first aim is to bound $a_{n}$ in terms of $m_{n}$ and $M_{n}$. (i) There exist indices $p$ and $q$ such that $a_{n}=-\\left(a_{p}+a_{q}\\right)$ and $p+q=n$. Since $a_{p}, a_{q} \\leqslant M_{n}$, we have $a_{n} \\geqslant-2 M_{n}$. (ii) On the other hand, choose an index $kD$ is lucky if $m_{n} \\leqslant 2 M_{n}$. Two cases are possible. Case 1. Assume that there exists a lucky index $n$. In this case, (1) yields $m_{n+1} \\leqslant 2 M_{n}$ and $M_{n} \\leqslant M_{n+1} \\leqslant M_{n}$. Therefore, $M_{n+1}=M_{n}$ and $m_{n+1} \\leqslant 2 M_{n}=2 M_{n+1}$. So, the index $n+1$ is also lucky, and $M_{n+1}=M_{n}$. Applying the same arguments repeatedly, we obtain that all indices $k>n$ are lucky (i.e., $m_{k} \\leqslant 2 M_{k}$ for all these indices), and $M_{k}=M_{n}$ for all such indices. Thus, all of the $m_{k}$ and $M_{k}$ are bounded by $2 M_{n}$. Case 2. Assume now that there is no lucky index, i.e., $2 M_{n}D$. Then (1) shows that for all $n>D$ we have $m_{n} \\leqslant m_{n+1} \\leqslant m_{n}$, so $m_{n}=m_{D+1}$ for all $n>D$. Since $M_{n}2017 $$ Prove that this sequence is bounded, i.e., there is a constant $M$ such that $\\left|a_{n}\\right| \\leqslant M$ for all positive integers $n$. (Russia)","solution":"As in the previous solution, let $D=2017$. If the sequence is bounded above, say, by $Q$, then we have that $a_{n} \\geqslant \\min \\left\\{a_{1}, \\ldots, a_{D},-2 Q\\right\\}$ for all $n$, so the sequence is bounded. Assume for sake of contradiction that the sequence is not bounded above. Let $\\ell=\\min \\left\\{a_{1}, \\ldots, a_{D}\\right\\}$, and $L=\\max \\left\\{a_{1}, \\ldots, a_{D}\\right\\}$. Call an index $n$ good if the following criteria hold: $$ a_{n}>a_{i} \\text { for each } i-2 \\ell, \\quad \\text { and } \\quad n>D $$ We first show that there must be some good index $n$. By assumption, we may take an index $N$ such that $a_{N}>\\max \\{L,-2 \\ell\\}$. Choose $n$ minimally such that $a_{n}=\\max \\left\\{a_{1}, a_{2}, \\ldots, a_{N}\\right\\}$. Now, the first condition in (2) is satisfied because of the minimality of $n$, and the second and third conditions are satisfied because $a_{n} \\geqslant a_{N}>L,-2 \\ell$, and $L \\geqslant a_{i}$ for every $i$ such that $1 \\leqslant i \\leqslant D$. Let $n$ be a good index. We derive a contradiction. We have that $$ a_{n}+a_{u}+a_{v} \\leqslant 0 $$ whenever $u+v=n$. We define the index $u$ to maximize $a_{u}$ over $1 \\leqslant u \\leqslant n-1$, and let $v=n-u$. Then, we note that $a_{u} \\geqslant a_{v}$ by the maximality of $a_{u}$. Assume first that $v \\leqslant D$. Then, we have that $$ a_{N}+2 \\ell \\leqslant 0, $$ because $a_{u} \\geqslant a_{v} \\geqslant \\ell$. But this contradicts our assumption that $a_{n}>-2 \\ell$ in the second criteria of (2). Now assume that $v>D$. Then, there exist some indices $w_{1}, w_{2}$ summing up to $v$ such that $$ a_{v}+a_{w_{1}}+a_{w_{2}}=0 $$ But combining this with (3), we have $$ a_{n}+a_{u} \\leqslant a_{w_{1}}+a_{w_{2}} $$ Because $a_{n}>a_{u}$, we have that $\\max \\left\\{a_{w_{1}}, a_{w_{2}}\\right\\}>a_{u}$. But since each of the $w_{i}$ is less than $v$, this contradicts the maximality of $a_{u}$. Comment 1. We present two harder versions of this problem below. Version 1. Let $a_{1}, a_{2}, \\ldots$ be a sequence of numbers that satisfies the relation $$ a_{n}=-\\max _{i+j+k=n}\\left(a_{i}+a_{j}+a_{k}\\right) \\quad \\text { for all } n>2017 $$ Then, this sequence is bounded. Proof. Set $D=2017$. Denote $$ M_{n}=\\max _{k2 D$; our first aim is to bound $a_{n}$ in terms of $m_{i}$ and $M_{i}$. Set $k=\\lfloor n \/ 2\\rfloor$. (i) Choose indices $p, q$, and $r$ such that $a_{n}=-\\left(a_{p}+a_{q}+a_{r}\\right)$ and $p+q+r=n$. Without loss of generality, $p \\geqslant q \\geqslant r$. Assume that $p \\geqslant k+1(>D)$; then $p>q+r$. Hence $$ -a_{p}=\\max _{i_{1}+i_{2}+i_{3}=p}\\left(a_{i_{1}}+a_{i_{2}}+a_{i_{3}}\\right) \\geqslant a_{q}+a_{r}+a_{p-q-r} $$ and therefore $a_{n}=-\\left(a_{p}+a_{q}+a_{r}\\right) \\geqslant\\left(a_{q}+a_{r}+a_{p-q-r}\\right)-a_{q}-a_{r}=a_{p-q-r} \\geqslant-m_{n}$. Otherwise, we have $k \\geqslant p \\geqslant q \\geqslant r$. Since $n<3 k$, we have $r2 D$ is lucky if $m_{n} \\leqslant 2 M_{\\lfloor n \/ 2\\rfloor+1}+M_{\\lfloor n \/ 2]}$. Two cases are possible. Case 1. Assume that there exists a lucky index $n$; set $k=\\lfloor n \/ 2\\rfloor$. In this case, (4) yields $m_{n+1} \\leqslant$ $2 M_{k+1}+M_{k}$ and $M_{n} \\leqslant M_{n+1} \\leqslant M_{n}$ (the last relation holds, since $m_{n}-M_{k+1}-M_{k} \\leqslant\\left(2 M_{k+1}+\\right.$ $\\left.M_{k}\\right)-M_{k+1}-M_{k}=M_{k+1} \\leqslant M_{n}$ ). Therefore, $M_{n+1}=M_{n}$ and $m_{n+1} \\leqslant 2 M_{k+1}+M_{k}$; the last relation shows that the index $n+1$ is also lucky. Thus, all indices $N>n$ are lucky, and $M_{N}=M_{n} \\geqslant m_{N} \/ 3$, whence all the $m_{N}$ and $M_{N}$ are bounded by $3 M_{n}$. Case 2. Conversely, assume that there is no lucky index, i.e., $2 M_{\\lfloor n \/ 2\\rfloor+1}+M_{\\lfloor n \/ 2\\rfloor}2 D$. Then (4) shows that for all $n>2 D$ we have $m_{n} \\leqslant m_{n+1} \\leqslant m_{n}$, i.e., $m_{N}=m_{2 D+1}$ for all $N>2 D$. Since $M_{N}2017 $$ Then, this sequence is bounded. Proof. As in the solutions above, let $D=2017$. If the sequence is bounded above, say, by $Q$, then we have that $a_{n} \\geqslant \\min \\left\\{a_{1}, \\ldots, a_{D},-k Q\\right\\}$ for all $n$, so the sequence is bounded. Assume for sake of contradiction that the sequence is not bounded above. Let $\\ell=\\min \\left\\{a_{1}, \\ldots, a_{D}\\right\\}$, and $L=\\max \\left\\{a_{1}, \\ldots, a_{D}\\right\\}$. Call an index $n$ good if the following criteria hold: $$ a_{n}>a_{i} \\text { for each } i-k \\ell, \\quad \\text { and } \\quad n>D $$ We first show that there must be some good index $n$. By assumption, we may take an index $N$ such that $a_{N}>\\max \\{L,-k \\ell\\}$. Choose $n$ minimally such that $a_{n}=\\max \\left\\{a_{1}, a_{2}, \\ldots, a_{N}\\right\\}$. Now, the first condition is satisfied because of the minimality of $n$, and the second and third conditions are satisfied because $a_{n} \\geqslant a_{N}>L,-k \\ell$, and $L \\geqslant a_{i}$ for every $i$ such that $1 \\leqslant i \\leqslant D$. Let $n$ be a good index. We derive a contradiction. We have that $$ a_{n}+a_{v_{1}}+\\cdots+a_{v_{k}} \\leqslant 0 $$ whenever $v_{1}+\\cdots+v_{k}=n$. We define the sequence of indices $v_{1}, \\ldots, v_{k-1}$ to greedily maximize $a_{v_{1}}$, then $a_{v_{2}}$, and so forth, selecting only from indices such that the equation $v_{1}+\\cdots+v_{k}=n$ can be satisfied by positive integers $v_{1}, \\ldots, v_{k}$. More formally, we define them inductively so that the following criteria are satisfied by the $v_{i}$ : 1. $1 \\leqslant v_{i} \\leqslant n-(k-i)-\\left(v_{1}+\\cdots+v_{i-1}\\right)$. 2. $a_{v_{i}}$ is maximal among all choices of $v_{i}$ from the first criteria. First of all, we note that for each $i$, the first criteria is always satisfiable by some $v_{i}$, because we are guaranteed that $$ v_{i-1} \\leqslant n-(k-(i-1))-\\left(v_{1}+\\cdots+v_{i-2}\\right), $$ which implies $$ 1 \\leqslant n-(k-i)-\\left(v_{1}+\\cdots+v_{i-1}\\right) . $$ Secondly, the sum $v_{1}+\\cdots+v_{k-1}$ is at most $n-1$. Define $v_{k}=n-\\left(v_{1}+\\cdots+v_{k-1}\\right)$. Then, (6) is satisfied by the $v_{i}$. We also note that $a_{v_{i}} \\geqslant a_{v_{j}}$ for all $i-k \\ell$ in the second criteria of (5). Now assume that $v_{k}>D$, and then we must have some indices $w_{1}, \\ldots, w_{k}$ summing up to $v_{k}$ such that $$ a_{v_{k}}+a_{w_{1}}+\\cdots+a_{w_{k}}=0 $$ But combining this with (6), we have $$ a_{n}+a_{v_{1}}+\\cdots+a_{v_{k-1}} \\leqslant a_{w_{1}}+\\cdots+a_{w_{k}} . $$ Because $a_{n}>a_{v_{1}} \\geqslant \\cdots \\geqslant a_{v_{k-1}}$, we have that $\\max \\left\\{a_{w_{1}}, \\ldots, a_{w_{k}}\\right\\}>a_{v_{k-1}}$. But since each of the $w_{i}$ is less than $v_{k}$, in the definition of the $v_{k-1}$ we could have chosen one of the $w_{i}$ instead, which is a contradiction. Comment 2. It seems that each sequence satisfying the condition in Version 2 is eventually periodic, at least when its terms are integers. However, up to this moment, the Problem Selection Committee is not aware of a proof for this fact (even in the case $k=2$ ).","tier":0} +{"problem_type":"Algebra","problem_label":"A5","problem":"An integer $n \\geqslant 3$ is given. We call an $n$-tuple of real numbers $\\left(x_{1}, x_{2}, \\ldots, x_{n}\\right)$ Shiny if for each permutation $y_{1}, y_{2}, \\ldots, y_{n}$ of these numbers we have $$ \\sum_{i=1}^{n-1} y_{i} y_{i+1}=y_{1} y_{2}+y_{2} y_{3}+y_{3} y_{4}+\\cdots+y_{n-1} y_{n} \\geqslant-1 $$ Find the largest constant $K=K(n)$ such that $$ \\sum_{1 \\leqslant i\\ell$. Case 1: $k>\\ell$. Consider all permutations $\\phi:\\{1,2, \\ldots, n\\} \\rightarrow\\{1,2, \\ldots, n\\}$ such that $\\phi^{-1}(T)=\\{2,4, \\ldots, 2 \\ell\\}$. Note that there are $k!!$ ! such permutations $\\phi$. Define $$ f(\\phi)=\\sum_{i=1}^{n-1} x_{\\phi(i)} x_{\\phi(i+1)} $$ We know that $f(\\phi) \\geqslant-1$ for every permutation $\\phi$ with the above property. Averaging $f(\\phi)$ over all $\\phi$ gives $$ -1 \\leqslant \\frac{1}{k!\\ell!} \\sum_{\\phi} f(\\phi)=\\frac{2 \\ell}{k \\ell} M+\\frac{2(k-\\ell-1)}{k(k-1)} K $$ where the equality holds because there are $k \\ell$ products in $M$, of which $2 \\ell$ are selected for each $\\phi$, and there are $k(k-1) \/ 2$ products in $K$, of which $k-\\ell-1$ are selected for each $\\phi$. We now have $$ K+L+M \\geqslant K+L+\\left(-\\frac{k}{2}-\\frac{k-\\ell-1}{k-1} K\\right)=-\\frac{k}{2}+\\frac{\\ell}{k-1} K+L $$ Since $k \\leqslant n-1$ and $K, L \\geqslant 0$, we get the desired inequality. Case 2: $k=\\ell=n \/ 2$. We do a similar approach, considering all $\\phi:\\{1,2, \\ldots, n\\} \\rightarrow\\{1,2, \\ldots, n\\}$ such that $\\phi^{-1}(T)=$ $\\{2,4, \\ldots, 2 \\ell\\}$, and defining $f$ the same way. Analogously to Case 1 , we have $$ -1 \\leqslant \\frac{1}{k!\\ell!} \\sum_{\\phi} f(\\phi)=\\frac{2 \\ell-1}{k \\ell} M $$ because there are $k \\ell$ products in $M$, of which $2 \\ell-1$ are selected for each $\\phi$. Now, we have that $$ K+L+M \\geqslant M \\geqslant-\\frac{n^{2}}{4(n-1)} \\geqslant-\\frac{n-1}{2} $$ where the last inequality holds because $n \\geqslant 4$.","tier":0} +{"problem_type":"Algebra","problem_label":"A6","problem":"Find all functions $f: \\mathbb{R} \\rightarrow \\mathbb{R}$ such that $$ f(f(x) f(y))+f(x+y)=f(x y) $$ for all $x, y \\in \\mathbb{R}$. (Albania) Answer: There are 3 solutions: $$ x \\mapsto 0 \\quad \\text { or } \\quad x \\mapsto x-1 \\quad \\text { or } \\quad x \\mapsto 1-x \\quad(x \\in \\mathbb{R}) . $$","solution":"An easy check shows that all the 3 above mentioned functions indeed satisfy the original equation (*). In order to show that these are the only solutions, first observe that if $f(x)$ is a solution then $-f(x)$ is also a solution. Hence, without loss of generality we may (and will) assume that $f(0) \\leqslant 0$ from now on. We have to show that either $f$ is identically zero or $f(x)=x-1$ $(\\forall x \\in \\mathbb{R})$. Observe that, for a fixed $x \\neq 1$, we may choose $y \\in \\mathbb{R}$ so that $x+y=x y \\Longleftrightarrow y=\\frac{x}{x-1}$, and therefore from the original equation (*) we have $$ f\\left(f(x) \\cdot f\\left(\\frac{x}{x-1}\\right)\\right)=0 \\quad(x \\neq 1) $$ In particular, plugging in $x=0$ in (1), we conclude that $f$ has at least one zero, namely $(f(0))^{2}$ : $$ f\\left((f(0))^{2}\\right)=0 $$ We analyze two cases (recall that $f(0) \\leqslant 0$ ): Case 1: $f(0)=0$. Setting $y=0$ in the original equation we get the identically zero solution: $$ f(f(x) f(0))+f(x)=f(0) \\Longrightarrow f(x)=0 \\text { for all } x \\in \\mathbb{R} $$ From now on, we work on the main Case 2: $f(0)<0$. We begin with the following","tier":0} +{"problem_type":"Combinatorics","problem_label":"C3","problem":"Sir Alex plays the following game on a row of 9 cells. Initially, all cells are empty. In each move, Sir Alex is allowed to perform exactly one of the following two operations: (1) Choose any number of the form $2^{j}$, where $j$ is a non-negative integer, and put it into an empty cell. (2) Choose two (not necessarily adjacent) cells with the same number in them; denote that number by $2^{j}$. Replace the number in one of the cells with $2^{j+1}$ and erase the number in the other cell. At the end of the game, one cell contains the number $2^{n}$, where $n$ is a given positive integer, while the other cells are empty. Determine the maximum number of moves that Sir Alex could have made, in terms of $n$. (Thailand) Answer: $2 \\sum_{j=0}^{8}\\binom{n}{j}-1$.","solution":"We will solve a more general problem, replacing the row of 9 cells with a row of $k$ cells, where $k$ is a positive integer. Denote by $m(n, k)$ the maximum possible number of moves Sir Alex can make starting with a row of $k$ empty cells, and ending with one cell containing the number $2^{n}$ and all the other $k-1$ cells empty. Call an operation of type (1) an insertion, and an operation of type (2) a merge. Only one move is possible when $k=1$, so we have $m(n, 1)=1$. From now on we consider $k \\geqslant 2$, and we may assume Sir Alex's last move was a merge. Then, just before the last move, there were exactly two cells with the number $2^{n-1}$, and the other $k-2$ cells were empty. Paint one of those numbers $2^{n-1}$ blue, and the other one red. Now trace back Sir Alex's moves, always painting the numbers blue or red following this rule: if $a$ and $b$ merge into $c$, paint $a$ and $b$ with the same color as $c$. Notice that in this backward process new numbers are produced only by reversing merges, since reversing an insertion simply means deleting one of the numbers. Therefore, all numbers appearing in the whole process will receive one of the two colors. Sir Alex's first move is an insertion. Without loss of generality, assume this first number inserted is blue. Then, from this point on, until the last move, there is always at least one cell with a blue number. Besides the last move, there is no move involving a blue and a red number, since all merges involves numbers with the same color, and insertions involve only one number. Call an insertion of a blue number or merge of two blue numbers a blue move, and define a red move analogously. The whole sequence of blue moves could be repeated on another row of $k$ cells to produce one cell with the number $2^{n-1}$ and all the others empty, so there are at most $m(n-1, k)$ blue moves. Now we look at the red moves. Since every time we perform a red move there is at least one cell occupied with a blue number, the whole sequence of red moves could be repeated on a row of $k-1$ cells to produce one cell with the number $2^{n-1}$ and all the others empty, so there are at most $m(n-1, k-1)$ red moves. This proves that $$ m(n, k) \\leqslant m(n-1, k)+m(n-1, k-1)+1 . $$ On the other hand, we can start with an empty row of $k$ cells and perform $m(n-1, k)$ moves to produce one cell with the number $2^{n-1}$ and all the others empty, and after that perform $m(n-1, k-1)$ moves on those $k-1$ empty cells to produce the number $2^{n-1}$ in one of them, leaving $k-2$ empty. With one more merge we get one cell with $2^{n}$ and the others empty, proving that $$ m(n, k) \\geqslant m(n-1, k)+m(n-1, k-1)+1 . $$ It follows that $$ m(n, k)=m(n-1, k)+m(n-1, k-1)+1, $$ for $n \\geqslant 1$ and $k \\geqslant 2$. If $k=1$ or $n=0$, we must insert $2^{n}$ on our first move and immediately get the final configuration, so $m(0, k)=1$ and $m(n, 1)=1$, for $n \\geqslant 0$ and $k \\geqslant 1$. These initial values, together with the recurrence relation (1), determine $m(n, k)$ uniquely. Finally, we show that $$ m(n, k)=2 \\sum_{j=0}^{k-1}\\binom{n}{j}-1 $$ for all integers $n \\geqslant 0$ and $k \\geqslant 1$. We use induction on $n$. Since $m(0, k)=1$ for $k \\geqslant 1,(2)$ is true for the base case. We make the induction hypothesis that (2) is true for some fixed positive integer $n$ and all $k \\geqslant 1$. We have $m(n+1,1)=1=2\\binom{n+1}{0}-1$, and for $k \\geqslant 2$ the recurrence relation (1) and the induction hypothesis give us $$ \\begin{aligned} & m(n+1, k)=m(n, k)+m(n, k-1)+1=2 \\sum_{j=0}^{k-1}\\binom{n}{j}-1+2 \\sum_{j=0}^{k-2}\\binom{n}{j}-1+1 \\\\ & \\quad=2 \\sum_{j=0}^{k-1}\\binom{n}{j}+2 \\sum_{j=0}^{k-1}\\binom{n}{j-1}-1=2 \\sum_{j=0}^{k-1}\\left(\\binom{n}{j}+\\binom{n}{j-1}\\right)-1=2 \\sum_{j=0}^{k-1}\\binom{n+1}{j}-1, \\end{aligned} $$ which completes the proof. Comment 1. After deducing the recurrence relation (1), it may be convenient to homogenize the recurrence relation by defining $h(n, k)=m(n, k)+1$. We get the new relation $$ h(n, k)=h(n-1, k)+h(n-1, k), $$ for $n \\geqslant 1$ and $k \\geqslant 2$, with initial values $h(0, k)=h(n, 1)=2$, for $n \\geqslant 0$ and $k \\geqslant 1$. This may help one to guess the answer, and also with other approaches like the one we develop next. Comment 2. We can use a generating function to find the answer without guessing. We work with the homogenized recurrence relation (3). Define $h(n, 0)=0$ so that (3) is valid for $k=1$ as well. Now we set up the generating function $f(x, y)=\\sum_{n, k \\geqslant 0} h(n, k) x^{n} y^{k}$. Multiplying the recurrence relation (3) by $x^{n} y^{k}$ and summing over $n, k \\geqslant 1$, we get $$ \\sum_{n, k \\geqslant 1} h(n, k) x^{n} y^{k}=x \\sum_{n, k \\geqslant 1} h(n-1, k) x^{n-1} y^{k}+x y \\sum_{n, k \\geqslant 1} h(n-1, k-1) x^{n-1} y^{k-1} . $$ Completing the missing terms leads to the following equation on $f(x, y)$ : $$ f(x, y)-\\sum_{n \\geqslant 0} h(n, 0) x^{n}-\\sum_{k \\geqslant 1} h(0, k) y^{k}=x f(x, y)-x \\sum_{n \\geqslant 0} h(n, 0) x^{n}+x y f(x, y) . $$ Substituting the initial values, we obtain $$ f(x, y)=\\frac{2 y}{1-y} \\cdot \\frac{1}{1-x(1+y)} . $$ Developing as a power series, we get $$ f(x, y)=2 \\sum_{j \\geqslant 1} y^{j} \\cdot \\sum_{n \\geqslant 0}(1+y)^{n} x^{n} . $$ The coefficient of $x^{n}$ in this power series is $$ 2 \\sum_{j \\geqslant 1} y^{j} \\cdot(1+y)^{n}=2 \\sum_{j \\geqslant 1} y^{j} \\cdot \\sum_{i \\geqslant 0}\\binom{n}{i} y^{i} $$ and extracting the coefficient of $y^{k}$ in this last expression we finally obtain the value for $h(n, k)$, $$ h(n, k)=2 \\sum_{j=0}^{k-1}\\binom{n}{j} $$ This proves that $$ m(n, k)=2 \\sum_{j=0}^{k-1}\\binom{n}{j}-1 $$ The generating function approach also works if applied to the non-homogeneous recurrence relation (1), but the computations are less straightforward.","tier":0} +{"problem_type":"Combinatorics","problem_label":"C3","problem":"Sir Alex plays the following game on a row of 9 cells. Initially, all cells are empty. In each move, Sir Alex is allowed to perform exactly one of the following two operations: (1) Choose any number of the form $2^{j}$, where $j$ is a non-negative integer, and put it into an empty cell. (2) Choose two (not necessarily adjacent) cells with the same number in them; denote that number by $2^{j}$. Replace the number in one of the cells with $2^{j+1}$ and erase the number in the other cell. At the end of the game, one cell contains the number $2^{n}$, where $n$ is a given positive integer, while the other cells are empty. Determine the maximum number of moves that Sir Alex could have made, in terms of $n$. (Thailand) Answer: $2 \\sum_{j=0}^{8}\\binom{n}{j}-1$.","solution":"Define merges and insertions as in We will need the following lemma. Lemma. If the binary representation of a positive integer $A$ has $d$ nonzero digits, then $A$ cannot be represented as a sum of fewer than $d$ powers of 2 . Moreover, any representation of $A$ as a sum of $d$ powers of 2 must coincide with its binary representation. Proof. Let $s$ be the minimum number of summands in all possible representations of $A$ as sum of powers of 2 . Suppose there is such a representation with $s$ summands, where two of the summands are equal to each other. Then, replacing those two summands with the result of their sum, we obtain a representation with fewer than $s$ summands, which is a contradiction. We deduce that in any representation with $s$ summands, the summands are all distinct, so any such representation must coincide with the unique binary representation of $A$, and $s=d$. Now we split the solution into a sequence of claims. Claim 1. After every move, the number $S$ is the sum of at most $k-1$ distinct powers of 2 . Proof. If $S$ is the sum of $k$ (or more) distinct powers of 2 , the Lemma implies that the $k$ cells are filled with these numbers. This is a contradiction since no more merges or insertions can be made. Let $A(n, k-1)$ denote the set of all positive integers not exceeding $2^{n}$ with at most $k-1$ nonzero digits in its base 2 representation. Since every insertion increases the value of $S$, by Claim 1, the total number of insertions is at most $|A(n, k-1)|$. We proceed to prove that it is possible to achieve this number of insertions. Claim 2. Let $A(n, k-1)=\\left\\{a_{1}, a_{2}, \\ldots, a_{m}\\right\\}$, with $a_{1}\\frac{1}{400} $$ In particular, $\\varepsilon^{2}+1=400 \\varepsilon$, so $$ y^{2}=d_{n}^{2}-2 \\varepsilon d_{n}+\\varepsilon^{2}+1=d_{n}^{2}+\\varepsilon\\left(400-2 d_{n}\\right) $$ Since $\\varepsilon>\\frac{1}{400}$ and we assumed $d_{n}<100$, this shows that $y^{2}>d_{n}^{2}+\\frac{1}{2}$. So, as we claimed, with this list of radar pings, no matter what the hunter does, the rabbit might achieve $d_{n+200}^{2}>d_{n}^{2}+\\frac{1}{2}$. The wabbit wins. Comment 1. Many different versions of the solution above can be found by replacing 200 with some other number $N$ for the number of hops the rabbit takes between reveals. If this is done, we have: $$ \\varepsilon=N-\\sqrt{N^{2}-1}>\\frac{1}{N+\\sqrt{N^{2}-1}}>\\frac{1}{2 N} $$ and $$ \\varepsilon^{2}+1=2 N \\varepsilon $$ so, as long as $N>d_{n}$, we would find $$ y^{2}=d_{n}^{2}+\\varepsilon\\left(2 N-2 d_{n}\\right)>d_{n}^{2}+\\frac{N-d_{n}}{N} $$ For example, taking $N=101$ is already enough-the squared distance increases by at least $\\frac{1}{101}$ every 101 rounds, and $101^{2} \\cdot 10^{4}=1.0201 \\cdot 10^{8}<10^{9}$ rounds are enough for the rabbit. If the statement is made sharper, some such versions might not work any longer. Comment 2. The original statement asked whether the distance could be kept under $10^{10}$ in $10^{100}$ rounds.","tier":0} +{"problem_type":"Combinatorics","problem_label":"C6","problem":"Let $n>1$ be an integer. An $n \\times n \\times n$ cube is composed of $n^{3}$ unit cubes. Each unit cube is painted with one color. For each $n \\times n \\times 1$ box consisting of $n^{2}$ unit cubes (of any of the three possible orientations), we consider the set of the colors present in that box (each color is listed only once). This way, we get $3 n$ sets of colors, split into three groups according to the orientation. It happens that for every set in any group, the same set appears in both of the other groups. Determine, in terms of $n$, the maximal possible number of colors that are present. (Russia) Answer: The maximal number is $\\frac{n(n+1)(2 n+1)}{6}$.","solution":"Call a $n \\times n \\times 1$ box an $x$-box, a $y$-box, or a $z$-box, according to the direction of its short side. Let $C$ be the number of colors in a valid configuration. We start with the upper bound for $C$. Let $\\mathcal{C}_{1}, \\mathcal{C}_{2}$, and $\\mathcal{C}_{3}$ be the sets of colors which appear in the big cube exactly once, exactly twice, and at least thrice, respectively. Let $M_{i}$ be the set of unit cubes whose colors are in $\\mathcal{C}_{i}$, and denote $n_{i}=\\left|M_{i}\\right|$. Consider any $x$-box $X$, and let $Y$ and $Z$ be a $y$ - and a $z$-box containing the same set of colors as $X$ does. Claim. $4\\left|X \\cap M_{1}\\right|+\\left|X \\cap M_{2}\\right| \\leqslant 3 n+1$. Proof. We distinguish two cases. Case 1: $X \\cap M_{1} \\neq \\varnothing$. A cube from $X \\cap M_{1}$ should appear in all three boxes $X, Y$, and $Z$, so it should lie in $X \\cap Y \\cap Z$. Thus $X \\cap M_{1}=X \\cap Y \\cap Z$ and $\\left|X \\cap M_{1}\\right|=1$. Consider now the cubes in $X \\cap M_{2}$. There are at most $2(n-1)$ of them lying in $X \\cap Y$ or $X \\cap Z$ (because the cube from $X \\cap Y \\cap Z$ is in $M_{1}$ ). Let $a$ be some other cube from $X \\cap M_{2}$. Recall that there is just one other cube $a^{\\prime}$ sharing a color with $a$. But both $Y$ and $Z$ should contain such cube, so $a^{\\prime} \\in Y \\cap Z$ (but $a^{\\prime} \\notin X \\cap Y \\cap Z$ ). The map $a \\mapsto a^{\\prime}$ is clearly injective, so the number of cubes $a$ we are interested in does not exceed $|(Y \\cap Z) \\backslash X|=n-1$. Thus $\\left|X \\cap M_{2}\\right| \\leqslant 2(n-1)+(n-1)=3(n-1)$, and hence $4\\left|X \\cap M_{1}\\right|+\\left|X \\cap M_{2}\\right| \\leqslant 4+3(n-1)=3 n+1$. Case 2: $X \\cap M_{1}=\\varnothing$. In this case, the same argument applies with several changes. Indeed, $X \\cap M_{2}$ contains at most $2 n-1$ cubes from $X \\cap Y$ or $X \\cap Z$. Any other cube $a$ in $X \\cap M_{2}$ corresponds to some $a^{\\prime} \\in Y \\cap Z$ (possibly with $a^{\\prime} \\in X$ ), so there are at most $n$ of them. All this results in $\\left|X \\cap M_{2}\\right| \\leqslant(2 n-1)+n=3 n-1$, which is even better than we need (by the assumptions of our case). Summing up the inequalities from the Claim over all $x$-boxes $X$, we obtain $$ 4 n_{1}+n_{2} \\leqslant n(3 n+1) $$ Obviously, we also have $n_{1}+n_{2}+n_{3}=n^{3}$. Now we are prepared to estimate $C$. Due to the definition of the $M_{i}$, we have $n_{i} \\geqslant i\\left|\\mathcal{C}_{i}\\right|$, so $$ C \\leqslant n_{1}+\\frac{n_{2}}{2}+\\frac{n_{3}}{3}=\\frac{n_{1}+n_{2}+n_{3}}{3}+\\frac{4 n_{1}+n_{2}}{6} \\leqslant \\frac{n^{3}}{3}+\\frac{3 n^{2}+n}{6}=\\frac{n(n+1)(2 n+1)}{6} $$ It remains to present an example of an appropriate coloring in the above-mentioned number of colors. For each color, we present the set of all cubes of this color. These sets are: 1. $n$ singletons of the form $S_{i}=\\{(i, i, i)\\}$ (with $1 \\leqslant i \\leqslant n$ ); 2. $3\\binom{n}{2}$ doubletons of the forms $D_{i, j}^{1}=\\{(i, j, j),(j, i, i)\\}, D_{i, j}^{2}=\\{(j, i, j),(i, j, i)\\}$, and $D_{i, j}^{3}=$ $\\{(j, j, i),(i, i, j)\\}$ (with $1 \\leqslant i1$ be an integer. An $n \\times n \\times n$ cube is composed of $n^{3}$ unit cubes. Each unit cube is painted with one color. For each $n \\times n \\times 1$ box consisting of $n^{2}$ unit cubes (of any of the three possible orientations), we consider the set of the colors present in that box (each color is listed only once). This way, we get $3 n$ sets of colors, split into three groups according to the orientation. It happens that for every set in any group, the same set appears in both of the other groups. Determine, in terms of $n$, the maximal possible number of colors that are present. (Russia) Answer: The maximal number is $\\frac{n(n+1)(2 n+1)}{6}$.","solution":"We will approach a new version of the original problem. In this new version, each cube may have a color, or be invisible (not both). Now we make sets of colors for each $n \\times n \\times 1$ box as before (where \"invisible\" is not considered a color) and group them by orientation, also as before. Finally, we require that, for every non-empty set in any group, the same set must appear in the other 2 groups. What is the maximum number of colors present with these new requirements? Let us call strange a big $n \\times n \\times n$ cube whose painting scheme satisfies the new requirements, and let $D$ be the number of colors in a strange cube. Note that any cube that satisfies the original requirements is also strange, so $\\max (D)$ is an upper bound for the original answer. Claim. $D \\leqslant \\frac{n(n+1)(2 n+1)}{6}$. Proof. The proof is by induction on $n$. If $n=1$, we must paint the cube with at most 1 color. Now, pick a $n \\times n \\times n$ strange cube $A$, where $n \\geqslant 2$. If $A$ is completely invisible, $D=0$ and we are done. Otherwise, pick a non-empty set of colors $\\mathcal{S}$ which corresponds to, say, the boxes $X, Y$ and $Z$ of different orientations. Now find all cubes in $A$ whose colors are in $\\mathcal{S}$ and make them invisible. Since $X, Y$ and $Z$ are now completely invisible, we can throw them away and focus on the remaining $(n-1) \\times(n-1) \\times(n-1)$ cube $B$. The sets of colors in all the groups for $B$ are the same as the sets for $A$, removing exactly the colors in $\\mathcal{S}$, and no others! Therefore, every nonempty set that appears in one group for $B$ still shows up in all possible orientations (it is possible that an empty set of colors in $B$ only matched $X, Y$ or $Z$ before these were thrown away, but remember we do not require empty sets to match anyway). In summary, $B$ is also strange. By the induction hypothesis, we may assume that $B$ has at most $\\frac{(n-1) n(2 n-1)}{6}$ colors. Since there were at most $n^{2}$ different colors in $\\mathcal{S}$, we have that $A$ has at most $\\frac{(n-1) n(2 n-1)}{6}+n^{2}=$ $\\frac{n(n+1)(2 n+1)}{6}$ colors. Finally, the construction in the previous solution shows a painting scheme (with no invisible cubes) that reaches this maximum, so we are done.","tier":0} +{"problem_type":"Combinatorics","problem_label":"C7","problem":"For any finite sets $X$ and $Y$ of positive integers, denote by $f_{X}(k)$ the $k^{\\mathrm{th}}$ smallest positive integer not in $X$, and let $$ X * Y=X \\cup\\left\\{f_{X}(y): y \\in Y\\right\\} $$ Let $A$ be a set of $a>0$ positive integers, and let $B$ be a set of $b>0$ positive integers. Prove that if $A * B=B * A$, then $$ \\underbrace{A *(A * \\cdots *(A *(A * A)) \\ldots)}_{A \\text { appears } b \\text { times }}=\\underbrace{B *(B * \\cdots *(B *(B * B)) \\ldots)}_{B \\text { appears } a \\text { times }} . $$","solution":"For any function $g: \\mathbb{Z}_{>0} \\rightarrow \\mathbb{Z}_{>0}$ and any subset $X \\subset \\mathbb{Z}_{>0}$, we define $g(X)=$ $\\{g(x): x \\in X\\}$. We have that the image of $f_{X}$ is $f_{X}\\left(\\mathbb{Z}_{>0}\\right)=\\mathbb{Z}_{>0} \\backslash X$. We now show a general lemma about the operation *, with the goal of showing that $*$ is associative. Lemma 1. Let $X$ and $Y$ be finite sets of positive integers. The functions $f_{X * Y}$ and $f_{X} \\circ f_{Y}$ are equal. Proof. We have $f_{X * Y}\\left(\\mathbb{Z}_{>0}\\right)=\\mathbb{Z}_{>0} \\backslash(X * Y)=\\left(\\mathbb{Z}_{>0} \\backslash X\\right) \\backslash f_{X}(Y)=f_{X}\\left(\\mathbb{Z}_{>0}\\right) \\backslash f_{X}(Y)=f_{X}\\left(\\mathbb{Z}_{>0} \\backslash Y\\right)=f_{X}\\left(f_{Y}\\left(\\mathbb{Z}_{>0}\\right)\\right)$. Thus, the functions $f_{X * Y}$ and $f_{X} \\circ f_{Y}$ are strictly increasing functions with the same range. Because a strictly function is uniquely defined by its range, we have $f_{X * Y}=f_{X} \\circ f_{Y}$. Lemma 1 implies that * is associative, in the sense that $(A * B) * C=A *(B * C)$ for any finite sets $A, B$, and $C$ of positive integers. We prove the associativity by noting $$ \\begin{gathered} \\mathbb{Z}_{>0} \\backslash((A * B) * C)=f_{(A * B) * C}\\left(\\mathbb{Z}_{>0}\\right)=f_{A * B}\\left(f_{C}\\left(\\mathbb{Z}_{>0}\\right)\\right)=f_{A}\\left(f_{B}\\left(f_{C}\\left(\\mathbb{Z}_{>0}\\right)\\right)\\right) \\\\ =f_{A}\\left(f_{B * C}\\left(\\mathbb{Z}_{>0}\\right)=f_{A *(B * C)}\\left(\\mathbb{Z}_{>0}\\right)=\\mathbb{Z}_{>0} \\backslash(A *(B * C))\\right. \\end{gathered} $$ In light of the associativity of *, we may drop the parentheses when we write expressions like $A *(B * C)$. We also introduce the notation $$ X^{* k}=\\underbrace{X *(X * \\cdots *(X *(X * X)) \\ldots)}_{X \\text { appears } k \\text { times }} $$ Our goal is then to show that $A * B=B * A$ implies $A^{* b}=B^{* a}$. We will do so via the following general lemma. Lemma 2. Suppose that $X$ and $Y$ are finite sets of positive integers satisfying $X * Y=Y * X$ and $|X|=|Y|$. Then, we must have $X=Y$. Proof. Assume that $X$ and $Y$ are not equal. Let $s$ be the largest number in exactly one of $X$ and $Y$. Without loss of generality, say that $s \\in X \\backslash Y$. The number $f_{X}(s)$ counts the $s^{t h}$ number not in $X$, which implies that $$ f_{X}(s)=s+\\left|X \\cap\\left\\{1,2, \\ldots, f_{X}(s)\\right\\}\\right| $$ Since $f_{X}(s) \\geqslant s$, we have that $$ \\left\\{f_{X}(s)+1, f_{X}(s)+2, \\ldots\\right\\} \\cap X=\\left\\{f_{X}(s)+1, f_{X}(s)+2, \\ldots\\right\\} \\cap Y $$ which, together with the assumption that $|X|=|Y|$, gives $$ \\left|X \\cap\\left\\{1,2, \\ldots, f_{X}(s)\\right\\}\\right|=\\left|Y \\cap\\left\\{1,2, \\ldots, f_{X}(s)\\right\\}\\right| $$ Now consider the equation $$ t-|Y \\cap\\{1,2, \\ldots, t\\}|=s $$ This equation is satisfied only when $t \\in\\left[f_{Y}(s), f_{Y}(s+1)\\right)$, because the left hand side counts the number of elements up to $t$ that are not in $Y$. We have that the value $t=f_{X}(s)$ satisfies the above equation because of (1) and (2). Furthermore, since $f_{X}(s) \\notin X$ and $f_{X}(s) \\geqslant s$, we have that $f_{X}(s) \\notin Y$ due to the maximality of $s$. Thus, by the above discussion, we must have $f_{X}(s)=f_{Y}(s)$. Finally, we arrive at a contradiction. The value $f_{X}(s)$ is neither in $X$ nor in $f_{X}(Y)$, because $s$ is not in $Y$ by assumption. Thus, $f_{X}(s) \\notin X * Y$. However, since $s \\in X$, we have $f_{Y}(s) \\in Y * X$, a contradiction. We are now ready to finish the proof. Note first of all that $\\left|A^{* b}\\right|=a b=\\left|B^{* a}\\right|$. Moreover, since $A * B=B * A$, and $*$ is associative, it follows that $A^{* b} * B^{* a}=B^{* a} * A^{* b}$. Thus, by Lemma 2, we have $A^{* b}=B^{* a}$, as desired. Comment 1. Taking $A=X^{* k}$ and $B=X^{* l}$ generates many non-trivial examples where $A * B=B * A$. There are also other examples not of this form. For example, if $A=\\{1,2,4\\}$ and $B=\\{1,3\\}$, then $A * B=\\{1,2,3,4,6\\}=B * A$.","tier":0} +{"problem_type":"Combinatorics","problem_label":"C7","problem":"For any finite sets $X$ and $Y$ of positive integers, denote by $f_{X}(k)$ the $k^{\\mathrm{th}}$ smallest positive integer not in $X$, and let $$ X * Y=X \\cup\\left\\{f_{X}(y): y \\in Y\\right\\} $$ Let $A$ be a set of $a>0$ positive integers, and let $B$ be a set of $b>0$ positive integers. Prove that if $A * B=B * A$, then $$ \\underbrace{A *(A * \\cdots *(A *(A * A)) \\ldots)}_{A \\text { appears } b \\text { times }}=\\underbrace{B *(B * \\cdots *(B *(B * B)) \\ldots)}_{B \\text { appears } a \\text { times }} . $$","solution":"We will use Lemma 1 from $$ f_{X}=f_{Y} \\Longleftrightarrow f_{X}\\left(\\mathbb{Z}_{>0}\\right)=f_{Y}\\left(\\mathbb{Z}_{>0}\\right) \\Longleftrightarrow\\left(\\mathbb{Z}_{>0} \\backslash X\\right)=\\left(\\mathbb{Z}_{>0} \\backslash Y\\right) \\Longleftrightarrow X=Y, $$ where the first equivalence is because $f_{X}$ and $f_{Y}$ are strictly increasing functions, and the second equivalence is because $f_{X}\\left(\\mathbb{Z}_{>0}\\right)=\\mathbb{Z}_{>0} \\backslash X$ and $f_{Y}\\left(\\mathbb{Z}_{>0}\\right)=\\mathbb{Z}_{>0} \\backslash Y$. Denote $g=f_{A}$ and $h=f_{B}$. The given relation $A * B=B * A$ is equivalent to $f_{A * B}=f_{B * A}$ because of (3), and by Lemma 1 of the first solution, this is equivalent to $g \\circ h=h \\circ g$. Similarly, the required relation $A^{* b}=B^{* a}$ is equivalent to $g^{b}=h^{a}$. We will show that $$ g^{b}(n)=h^{a}(n) $$ for all $n \\in \\mathbb{Z}_{>0}$, which suffices to solve the problem. To start, we claim that (4) holds for all sufficiently large $n$. Indeed, let $p$ and $q$ be the maximal elements of $A$ and $B$, respectively; we may assume that $p \\geqslant q$. Then, for every $n \\geqslant p$ we have $g(n)=n+a$ and $h(n)=n+b$, whence $g^{b}(n)=n+a b=h^{a}(n)$, as was claimed. In view of this claim, if (4) is not identically true, then there exists a maximal $s$ with $g^{b}(s) \\neq$ $h^{a}(s)$. Without loss of generality, we may assume that $g(s) \\neq s$, for if we had $g(s)=h(s)=s$, then $s$ would satisfy (4). As $g$ is increasing, we then have $g(s)>s$, so (4) holds for $n=g(s)$. But then we have $$ g\\left(g^{b}(s)\\right)=g^{b+1}(s)=g^{b}(n)=h^{a}(n)=h^{a}(g(s))=g\\left(h^{a}(s)\\right), $$ where the last equality holds in view of $g \\circ h=h \\circ g$. By the injectivity of $g$, the above equality yields $g^{b}(s)=h^{a}(s)$, which contradicts the choice of $s$. Thus, we have proved that (4) is identically true on $\\mathbb{Z}_{>0}$, as desired. Comment 2. We present another proof of Lemma 2 of the first solution. Let $x=|X|=|Y|$. Say that $u$ is the smallest number in $X$ and $v$ is the smallest number in $Y$; assume without loss of generality that $u \\leqslant v$. Let $T$ be any finite set of positive integers, and define $t=|T|$. Enumerate the elements of $X$ as $x_{1}m_{i}$, we have that $s_{m, i}=t+m n-c_{i}$. Furthermore, the $c_{i}$ do not depend on the choice of $T$. First, we show that this claim implies Lemma 2. We may choose $T=X$ and $T=Y$. Then, there is some $m^{\\prime}$ such that for all $m \\geqslant m^{\\prime}$, we have $$ f_{X * m}(X)=f_{(Y * X *(m-1))}(X) $$ Because $u$ is the minimum element of $X, v$ is the minimum element of $Y$, and $u \\leqslant v$, we have that $$ \\left(\\bigcup_{m=m^{\\prime}}^{\\infty} f_{X * m}(X)\\right) \\cup X^{* m^{\\prime}}=\\left(\\bigcup_{m=m^{\\prime}}^{\\infty} f_{\\left(Y * X^{*(m-1)}\\right)}(X)\\right) \\cup\\left(Y * X^{*\\left(m^{\\prime}-1\\right)}\\right)=\\{u, u+1, \\ldots\\}, $$ and in both the first and second expressions, the unions are of pairwise distinct sets. By (5), we obtain $X^{* m^{\\prime}}=Y * X^{*\\left(m^{\\prime}-1\\right)}$. Now, because $X$ and $Y$ commute, we get $X^{* m^{\\prime}}=X^{*\\left(m^{\\prime}-1\\right)} * Y$, and so $X=Y$. We now prove the claim. Proof of the claim. We induct downwards on $i$, first proving the statement for $i=n$, and so on. Assume that $m$ is chosen so that all elements of $S_{m}$ are greater than all elements of $T$ (which is possible because $T$ is finite). For $i=n$, we have that $s_{m, n}>s_{k, n}$ for every $ki$ and $pi$ eventually do not depend on $T$, the sequence $s_{m, i}-t$ eventually does not depend on $T$ either, so the inductive step is complete. This proves the claim and thus Lemma 2.","tier":0} +{"problem_type":"Combinatorics","problem_label":"C8","problem":"Let $n$ be a given positive integer. In the Cartesian plane, each lattice point with nonnegative coordinates initially contains a butterfly, and there are no other butterflies. The neighborhood of a lattice point $c$ consists of all lattice points within the axis-aligned $(2 n+1) \\times$ $(2 n+1)$ square centered at $c$, apart from $c$ itself. We call a butterfly lonely, crowded, or comfortable, depending on whether the number of butterflies in its neighborhood $N$ is respectively less than, greater than, or equal to half of the number of lattice points in $N$. Every minute, all lonely butterflies fly away simultaneously. This process goes on for as long as there are any lonely butterflies. Assuming that the process eventually stops, determine the number of comfortable butterflies at the final state. (Bulgaria) Answer: $n^{2}+1$.","solution":"We always identify a butterfly with the lattice point it is situated at. For two points $p$ and $q$, we write $p \\geqslant q$ if each coordinate of $p$ is at least the corresponding coordinate of $q$. Let $O$ be the origin, and let $\\mathcal{Q}$ be the set of initially occupied points, i.e., of all lattice points with nonnegative coordinates. Let $\\mathcal{R}_{\\mathrm{H}}=\\{(x, 0): x \\geqslant 0\\}$ and $\\mathcal{R}_{\\mathrm{V}}=\\{(0, y): y \\geqslant 0\\}$ be the sets of the lattice points lying on the horizontal and vertical boundary rays of $\\mathcal{Q}$. Denote by $N(a)$ the neighborhood of a lattice point $a$. 1. Initial observations. We call a set of lattice points up-right closed if its points stay in the set after being shifted by any lattice vector $(i, j)$ with $i, j \\geqslant 0$. Whenever the butterflies form a up-right closed set $\\mathcal{S}$, we have $|N(p) \\cap \\mathcal{S}| \\geqslant|N(q) \\cap \\mathcal{S}|$ for any two points $p, q \\in \\mathcal{S}$ with $p \\geqslant q$. So, since $\\mathcal{Q}$ is up-right closed, the set of butterflies at any moment also preserves this property. We assume all forthcoming sets of lattice points to be up-right closed. When speaking of some set $\\mathcal{S}$ of lattice points, we call its points lonely, comfortable, or crowded with respect to this set (i.e., as if the butterflies were exactly at all points of $\\mathcal{S}$ ). We call a set $\\mathcal{S} \\subset \\mathcal{Q}$ stable if it contains no lonely points. In what follows, we are interested only in those stable sets whose complements in $\\mathcal{Q}$ are finite, because one can easily see that only a finite number of butterflies can fly away on each minute. If the initial set $\\mathcal{Q}$ of butterflies contains some stable set $\\mathcal{S}$, then, clearly no butterfly of this set will fly away. On the other hand, the set $\\mathcal{F}$ of all butterflies in the end of the process is stable. This means that $\\mathcal{F}$ is the largest (with respect to inclusion) stable set within $\\mathcal{Q}$, and we are about to describe this set. 2. A description of a final set. The following notion will be useful. Let $\\mathcal{U}=\\left\\{\\vec{u}_{1}, \\vec{u}_{2}, \\ldots, \\vec{u}_{d}\\right\\}$ be a set of $d$ pairwise non-parallel lattice vectors, each having a positive $x$ - and a negative $y$-coordinate. Assume that they are numbered in increasing order according to slope. We now define a $\\mathcal{U}$-curve to be the broken line $p_{0} p_{1} \\ldots p_{d}$ such that $p_{0} \\in \\mathcal{R}_{\\mathrm{V}}, p_{d} \\in \\mathcal{R}_{\\mathrm{H}}$, and $\\overrightarrow{p_{i-1} p_{i}}=\\vec{u}_{i}$ for all $i=1,2, \\ldots, m$ (see the Figure below to the left). ![](https:\/\/cdn.mathpix.com\/cropped\/2024_11_18_ff08e3c077fa2f8c16b2g-53.jpg?height=546&width=766&top_left_y=2194&top_left_x=191) Construction of $\\mathcal{U}$-curve ![](https:\/\/cdn.mathpix.com\/cropped\/2024_11_18_ff08e3c077fa2f8c16b2g-53.jpg?height=415&width=803&top_left_y=2322&top_left_x=1066) Construction of $\\mathcal{D}$ Now, let $\\mathcal{K}_{n}=\\{(i, j): 1 \\leqslant i \\leqslant n,-n \\leqslant j \\leqslant-1\\}$. Consider all the rays emerging at $O$ and passing through a point from $\\mathcal{K}_{n}$; number them as $r_{1}, \\ldots, r_{m}$ in increasing order according to slope. Let $A_{i}$ be the farthest from $O$ lattice point in $r_{i} \\cap \\mathcal{K}_{n}$, set $k_{i}=\\left|r_{i} \\cap \\mathcal{K}_{n}\\right|$, let $\\vec{v}_{i}=\\overrightarrow{O A_{i}}$, and finally denote $\\mathcal{V}=\\left\\{\\vec{v}_{i}: 1 \\leqslant i \\leqslant m\\right\\}$; see the Figure above to the right. We will concentrate on the $\\mathcal{V}$-curve $d_{0} d_{1} \\ldots d_{m}$; let $\\mathcal{D}$ be the set of all lattice points $p$ such that $p \\geqslant p^{\\prime}$ for some (not necessarily lattice) point $p^{\\prime}$ on the $\\mathcal{V}$-curve. In fact, we will show that $\\mathcal{D}=\\mathcal{F}$. Clearly, the $\\mathcal{V}$-curve is symmetric in the line $y=x$. Denote by $D$ the convex hull of $\\mathcal{D}$. 3. We prove that the set $\\mathcal{D}$ contains all stable sets. Let $\\mathcal{S} \\subset \\mathcal{Q}$ be a stable set (recall that it is assumed to be up-right closed and to have a finite complement in $\\mathcal{Q}$ ). Denote by $S$ its convex hull; clearly, the vertices of $S$ are lattice points. The boundary of $S$ consists of two rays (horizontal and vertical ones) along with some $\\mathcal{V}_{*}$-curve for some set of lattice vectors $\\mathcal{V}_{*}$. Claim 1. For every $\\vec{v}_{i} \\in \\mathcal{V}$, there is a $\\vec{v}_{i}^{*} \\in \\mathcal{V}_{*}$ co-directed with $\\vec{v}$ with $\\left|\\vec{v}_{i}^{*}\\right| \\geqslant|\\vec{v}|$. Proof. Let $\\ell$ be the supporting line of $S$ parallel to $\\vec{v}_{i}$ (i.e., $\\ell$ contains some point of $S$, and the set $S$ lies on one side of $\\ell$ ). Take any point $b \\in \\ell \\cap \\mathcal{S}$ and consider $N(b)$. The line $\\ell$ splits the set $N(b) \\backslash \\ell$ into two congruent parts, one having an empty intersection with $\\mathcal{S}$. Hence, in order for $b$ not to be lonely, at least half of the set $\\ell \\cap N(b)$ (which contains $2 k_{i}$ points) should lie in $S$. Thus, the boundary of $S$ contains a segment $\\ell \\cap S$ with at least $k_{i}+1$ lattice points (including $b$ ) on it; this segment corresponds to the required vector $\\vec{v}_{i}^{*} \\in \\mathcal{V}_{*}$. ![](https:\/\/cdn.mathpix.com\/cropped\/2024_11_18_ff08e3c077fa2f8c16b2g-54.jpg?height=600&width=1626&top_left_y=1199&top_left_x=221) Claim 2. Each stable set $\\mathcal{S} \\subseteq \\mathcal{Q}$ lies in $\\mathcal{D}$. Proof. To show this, it suffices to prove that the $\\mathcal{V}_{*}$-curve lies in $D$, i.e., that all its vertices do so. Let $p^{\\prime}$ be an arbitrary vertex of the $\\mathcal{V}_{*^{-}}$-curve; $p^{\\prime}$ partitions this curve into two parts, $\\mathcal{X}$ (being down-right of $p$ ) and $\\mathcal{Y}$ (being up-left of $p$ ). The set $\\mathcal{V}$ is split now into two parts: $\\mathcal{V}_{\\mathcal{X}}$ consisting of those $\\vec{v}_{i} \\in \\mathcal{V}$ for which $\\vec{v}_{i}^{*}$ corresponds to a segment in $\\mathcal{X}$, and a similar part $\\mathcal{V}_{\\mathcal{Y}}$. Notice that the $\\mathcal{V}$-curve consists of several segments corresponding to $\\mathcal{V}_{\\mathcal{X}}$, followed by those corresponding to $\\mathcal{V}_{\\mathcal{Y}}$. Hence there is a vertex $p$ of the $\\mathcal{V}$-curve separating $\\mathcal{V}_{\\mathcal{X}}$ from $\\mathcal{V}_{\\mathcal{Y}}$. Claim 1 now yields that $p^{\\prime} \\geqslant p$, so $p^{\\prime} \\in \\mathcal{D}$, as required. Claim 2 implies that the final set $\\mathcal{F}$ is contained in $\\mathcal{D}$. 4. $\\mathcal{D}$ is stable, and its comfortable points are known. Recall the definitions of $r_{i}$; let $r_{i}^{\\prime}$ be the ray complementary to $r_{i}$. By our definitions, the set $N(O)$ contains no points between the rays $r_{i}$ and $r_{i+1}$, as well as between $r_{i}^{\\prime}$ and $r_{i+1}^{\\prime}$. Claim 3. In the set $\\mathcal{D}$, all lattice points of the $\\mathcal{V}$-curve are comfortable. Proof. Let $p$ be any lattice point of the $\\mathcal{V}$-curve, belonging to some segment $d_{i} d_{i+1}$. Draw the line $\\ell$ containing this segment. Then $\\ell \\cap \\mathcal{D}$ contains exactly $k_{i}+1$ lattice points, all of which lie in $N(p)$ except for $p$. Thus, exactly half of the points in $N(p) \\cap \\ell$ lie in $\\mathcal{D}$. It remains to show that all points of $N(p)$ above $\\ell$ lie in $\\mathcal{D}$ (recall that all the points below $\\ell$ lack this property). Notice that each vector in $\\mathcal{V}$ has one coordinate greater than $n \/ 2$; thus the neighborhood of $p$ contains parts of at most two segments of the $\\mathcal{V}$-curve succeeding $d_{i} d_{i+1}$, as well as at most two of those preceding it. The angles formed by these consecutive segments are obtained from those formed by $r_{j}$ and $r_{j-1}^{\\prime}$ (with $i-1 \\leqslant j \\leqslant i+2$ ) by shifts; see the Figure below. All the points in $N(p)$ above $\\ell$ which could lie outside $\\mathcal{D}$ lie in shifted angles between $r_{j}, r_{j+1}$ or $r_{j}^{\\prime}, r_{j-1}^{\\prime}$. But those angles, restricted to $N(p)$, have no lattice points due to the above remark. The claim is proved. ![](https:\/\/cdn.mathpix.com\/cropped\/2024_11_18_ff08e3c077fa2f8c16b2g-55.jpg?height=404&width=1052&top_left_y=592&top_left_x=503) Proof of Claim 3 Claim 4. All the points of $\\mathcal{D}$ which are not on the boundary of $D$ are crowded. Proof. Let $p \\in \\mathcal{D}$ be such a point. If it is to the up-right of some point $p^{\\prime}$ on the curve, then the claim is easy: the shift of $N\\left(p^{\\prime}\\right) \\cap \\mathcal{D}$ by $\\overrightarrow{p^{\\prime} p}$ is still in $\\mathcal{D}$, and $N(p)$ contains at least one more point of $\\mathcal{D}$ - either below or to the left of $p$. So, we may assume that $p$ lies in a right triangle constructed on some hypothenuse $d_{i} d_{i+1}$. Notice here that $d_{i}, d_{i+1} \\in N(p)$. Draw a line $\\ell \\| d_{i} d_{i+1}$ through $p$, and draw a vertical line $h$ through $d_{i}$; see Figure below. Let $\\mathcal{D}_{\\mathrm{L}}$ and $\\mathcal{D}_{\\mathrm{R}}$ be the parts of $\\mathcal{D}$ lying to the left and to the right of $h$, respectively (points of $\\mathcal{D} \\cap h$ lie in both parts). ![](https:\/\/cdn.mathpix.com\/cropped\/2024_11_18_ff08e3c077fa2f8c16b2g-55.jpg?height=538&width=1120&top_left_y=1556&top_left_x=468) Notice that the vectors $\\overrightarrow{d_{i} p}, \\overrightarrow{d_{i+1} d_{i+2}}, \\overrightarrow{d_{i} d_{i+1}}, \\overrightarrow{d_{i-1} d_{i}}$, and $\\overrightarrow{p d_{i+1}}$ are arranged in non-increasing order by slope. This means that $\\mathcal{D}_{\\mathrm{L}}$ shifted by $\\overrightarrow{d_{i} p}$ still lies in $\\mathcal{D}$, as well as $\\mathcal{D}_{\\mathrm{R}}$ shifted by $\\overrightarrow{d_{i+1} p}$. As we have seen in the proof of Claim 3, these two shifts cover all points of $N(p)$ above $\\ell$, along with those on $\\ell$ to the left of $p$. Since $N(p)$ contains also $d_{i}$ and $d_{i+1}$, the point $p$ is crowded. Thus, we have proved that $\\mathcal{D}=\\mathcal{F}$, and have shown that the lattice points on the $\\mathcal{V}$-curve are exactly the comfortable points of $\\mathcal{D}$. It remains to find their number. Recall the definition of $\\mathcal{K}_{n}$ (see Figure on the first page of the solution). Each segment $d_{i} d_{i+1}$ contains $k_{i}$ lattice points different from $d_{i}$. Taken over all $i$, these points exhaust all the lattice points in the $\\mathcal{V}$-curve, except for $d_{1}$, and thus the number of lattice points on the $\\mathcal{V}$-curve is $1+\\sum_{i=1}^{m} k_{i}$. On the other hand, $\\sum_{i=1}^{m} k_{i}$ is just the number of points in $\\mathcal{K}_{n}$, so it equals $n^{2}$. Hence the answer to the problem is $n^{2}+1$. Comment 1. The assumption that the process eventually stops is unnecessary for the problem, as one can see that, in fact, the process stops for every $n \\geqslant 1$. Indeed, the proof of Claims 3 and 4 do not rely essentially on this assumption, and they together yield that the set $\\mathcal{D}$ is stable. So, only butterflies that are not in $\\mathcal{D}$ may fly away, and this takes only a finite time. This assumption has been inserted into the problem statement in order to avoid several technical details regarding finiteness issues. It may also simplify several other arguments. Comment 2. The description of the final set $\\mathcal{F}(=\\mathcal{D})$ seems to be crucial for the solution; the Problem Selection Committee is not aware of any solution that completely avoids such a description. On the other hand, after the set $\\mathcal{D}$ has been defined, the further steps may be performed in several ways. For example, in order to prove that all butterflies outside $\\mathcal{D}$ will fly away, one may argue as follows. (Here we will also make use of the assumption that the process eventually stops.) First of all, notice that the process can be modified in the following manner: Each minute, exactly one of the lonely butterflies flies away, until there are no more lonely butterflies. The modified process necessarily stops at the same state as the initial one. Indeed, one may observe, as in solution above, that the (unique) largest stable set is still the final set for the modified process. Thus, in order to prove our claim, it suffices to indicate an order in which the butterflies should fly away in the new process; if we are able to exhaust the whole set $\\mathcal{Q} \\backslash \\mathcal{D}$, we are done. Let $\\mathcal{C}_{0}=d_{0} d_{1} \\ldots d_{m}$ be the $\\mathcal{V}$-curve. Take its copy $\\mathcal{C}$ and shift it downwards so that $d_{0}$ comes to some point below the origin $O$. Now we start moving $\\mathcal{C}$ upwards continuously, until it comes back to its initial position $\\mathcal{C}_{0}$. At each moment when $\\mathcal{C}$ meets some lattice points, we convince all the butterflies at those points to fly away in a certain order. We will now show that we always have enough arguments for butterflies to do so, which will finish our argument for the claim.. Let $\\mathcal{C}^{\\prime}=d_{0}^{\\prime} d_{1}^{\\prime} \\ldots d_{m}^{\\prime}$ be a position of $\\mathcal{C}$ when it meets some butterflies. We assume that all butterflies under this current position of $\\mathcal{C}$ were already convinced enough and flied away. Consider the lowest butterfly $b$ on $\\mathcal{C}^{\\prime}$. Let $d_{i}^{\\prime} d_{i+1}^{\\prime}$ be the segment it lies on; we choose $i$ so that $b \\neq d_{i+1}^{\\prime}$ (this is possible because $\\mathcal{C}$ as not yet reached $\\mathcal{C}_{0}$ ). Draw a line $\\ell$ containing the segment $d_{i}^{\\prime} d_{i+1}^{\\prime}$. Then all the butterflies in $N(b)$ are situated on or above $\\ell$; moreover, those on $\\ell$ all lie on the segment $d_{i} d_{i+1}$. But this segment now contains at most $k_{i}$ butterflies (including $b$ ), since otherwise some butterfly had to occupy $d_{i+1}^{\\prime}$ which is impossible by the choice of $b$. Thus, $b$ is lonely and hence may be convinced to fly away. After $b$ has flied away, we switch to the lowest of the remaining butterflies on $\\mathcal{C}^{\\prime}$, and so on. Claims 3 and 4 also allow some different proofs which are not presented here. This page is intentionally left blank","tier":0} +{"problem_type":"Geometry","problem_label":"G8","problem":"There are 2017 mutually external circles drawn on a blackboard, such that no two are tangent and no three share a common tangent. A tangent segment is a line segment that is a common tangent to two circles, starting at one tangent point and ending at the other one. Luciano is drawing tangent segments on the blackboard, one at a time, so that no tangent segment intersects any other circles or previously drawn tangent segments. Luciano keeps drawing tangent segments until no more can be drawn. Find all possible numbers of tangent segments when he stops drawing. (Australia) Answer: If there were $n$ circles, there would always be exactly $3(n-1)$ segments; so the only possible answer is $3 \\cdot 2017-3=6048$.","solution":"First, consider a particular arrangement of circles $C_{1}, C_{2}, \\ldots, C_{n}$ where all the centers are aligned and each $C_{i}$ is eclipsed from the other circles by its neighbors - for example, taking $C_{i}$ with center $\\left(i^{2}, 0\\right)$ and radius $i \/ 2$ works. Then the only tangent segments that can be drawn are between adjacent circles $C_{i}$ and $C_{i+1}$, and exactly three segments can be drawn for each pair. So Luciano will draw exactly $3(n-1)$ segments in this case. ![](https:\/\/cdn.mathpix.com\/cropped\/2024_11_18_ff08e3c077fa2f8c16b2g-72.jpg?height=258&width=1335&top_left_y=1059&top_left_x=355) For the general case, start from a final configuration (that is, an arrangement of circles and segments in which no further segments can be drawn). The idea of the solution is to continuously resize and move the circles around the plane, one by one (in particular, making sure we never have 4 circles with a common tangent line), and show that the number of segments drawn remains constant as the picture changes. This way, we can reduce any circle\/segment configuration to the particular one mentioned above, and the final number of segments must remain at $3 n-3$. Some preliminary considerations: look at all possible tangent segments joining any two circles. A segment that is tangent to a circle $A$ can do so in two possible orientations - it may come out of $A$ in clockwise or counterclockwise orientation. Two segments touching the same circle with the same orientation will never intersect each other. Each pair $(A, B)$ of circles has 4 choices of tangent segments, which can be identified by their orientations - for example, $(A+, B-)$ would be the segment which comes out of $A$ in clockwise orientation and comes out of $B$ in counterclockwise orientation. In total, we have $2 n(n-1)$ possible segments, disregarding intersections. Now we pick a circle $C$ and start to continuously move and resize it, maintaining all existing tangent segments according to their identifications, including those involving $C$. We can keep our choice of tangent segments until the configuration reaches a transition. We lose nothing if we assume that $C$ is kept at least $\\varepsilon$ units away from any other circle, where $\\varepsilon$ is a positive, fixed constant; therefore at a transition either: (1) a currently drawn tangent segment $t$ suddenly becomes obstructed; or (2) a currently absent tangent segment $t$ suddenly becomes unobstructed and available. Claim. A transition can only occur when three circles $C_{1}, C_{2}, C_{3}$ are tangent to a common line $\\ell$ containing $t$, in a way such that the three tangent segments lying on $\\ell$ (joining the three circles pairwise) are not obstructed by any other circles or tangent segments (other than $C_{1}, C_{2}, C_{3}$ ). Proof. Since (2) is effectively the reverse of (1), it suffices to prove the claim for (1). Suppose $t$ has suddenly become obstructed, and let us consider two cases. Case 1: $t$ becomes obstructed by a circle ![](https:\/\/cdn.mathpix.com\/cropped\/2024_11_18_ff08e3c077fa2f8c16b2g-73.jpg?height=318&width=1437&top_left_y=292&top_left_x=315) Then the new circle becomes the third circle tangent to $\\ell$, and no other circles or tangent segments are obstructing $t$.","tier":0} +{"problem_type":"Geometry","problem_label":"G8","problem":"There are 2017 mutually external circles drawn on a blackboard, such that no two are tangent and no three share a common tangent. A tangent segment is a line segment that is a common tangent to two circles, starting at one tangent point and ending at the other one. Luciano is drawing tangent segments on the blackboard, one at a time, so that no tangent segment intersects any other circles or previously drawn tangent segments. Luciano keeps drawing tangent segments until no more can be drawn. Find all possible numbers of tangent segments when he stops drawing. (Australia) Answer: If there were $n$ circles, there would always be exactly $3(n-1)$ segments; so the only possible answer is $3 \\cdot 2017-3=6048$.","solution":"First note that all tangent segments lying on the boundary of the convex hull of the circles are always drawn since they do not intersect anything else. Now in the final picture, aside from the $n$ circles, the blackboard is divided into regions. We can consider the picture as a plane (multi-)graph $G$ in which the circles are the vertices and the tangent segments are the edges. The idea of this solution is to find a relation between the number of edges and the number of regions in $G$; then, once we prove that $G$ is connected, we can use Euler's formula to finish the problem. The boundary of each region consists of 1 or more (for now) simple closed curves, each made of arcs and tangent segments. The segment and the arc might meet smoothly (as in $S_{i}$, $i=1,2, \\ldots, 6$ in the figure below) or not (as in $P_{1}, P_{2}, P_{3}, P_{4}$; call such points sharp corners of the boundary). In other words, if a person walks along the border, her direction would suddenly turn an angle of $\\pi$ at a sharp corner. ![](https:\/\/cdn.mathpix.com\/cropped\/2024_11_18_ff08e3c077fa2f8c16b2g-75.jpg?height=626&width=577&top_left_y=178&top_left_x=748) Claim 1. The outer boundary $B_{1}$ of any internal region has at least 3 sharp corners. Proof. Let a person walk one lap along $B_{1}$ in the counterclockwise orientation. As she does so, she will turn clockwise as she moves along the circle arcs, and not turn at all when moving along the lines. On the other hand, her total rotation after one lap is $2 \\pi$ in the counterclockwise direction! Where could she be turning counterclockwise? She can only do so at sharp corners, and, even then, she turns only an angle of $\\pi$ there. But two sharp corners are not enough, since at least one arc must be present-so she must have gone through at least 3 sharp corners. Claim 2. Each internal region is simply connected, that is, has only one boundary curve. Proof. Suppose, by contradiction, that some region has an outer boundary $B_{1}$ and inner boundaries $B_{2}, B_{3}, \\ldots, B_{m}(m \\geqslant 2)$. Let $P_{1}$ be one of the sharp corners of $B_{1}$. Now consider a car starting at $P_{1}$ and traveling counterclockwise along $B_{1}$. It starts in reverse, i.e., it is initially facing the corner $P_{1}$. Due to the tangent conditions, the car may travel in a way so that its orientation only changes when it is moving along an arc. In particular, this means the car will sometimes travel forward. For example, if the car approaches a sharp corner when driving in reverse, it would continue travel forward after the corner, instead of making an immediate half-turn. This way, the orientation of the car only changes in a clockwise direction since the car always travels clockwise around each arc. Now imagine there is a laser pointer at the front of the car, pointing directly ahead. Initially, the laser endpoint hits $P_{1}$, but, as soon as the car hits an arc, the endpoint moves clockwise around $B_{1}$. In fact, the laser endpoint must move continuously along $B_{1}$ ! Indeed, if the endpoint ever jumped (within $B_{1}$, or from $B_{1}$ to one of the inner boundaries), at the moment of the jump the interrupted laser would be a drawable tangent segment that Luciano missed (see figure below for an example). ![](https:\/\/cdn.mathpix.com\/cropped\/2024_11_18_ff08e3c077fa2f8c16b2g-75.jpg?height=637&width=587&top_left_y=2140&top_left_x=740) Now, let $P_{2}$ and $P_{3}$ be the next two sharp corners the car goes through, after $P_{1}$ (the previous lemma assures their existence). At $P_{2}$ the car starts moving forward, and at $P_{3}$ it will start to move in reverse again. So, at $P_{3}$, the laser endpoint is at $P_{3}$ itself. So while the car moved counterclockwise between $P_{1}$ and $P_{3}$, the laser endpoint moved clockwise between $P_{1}$ and $P_{3}$. That means the laser beam itself scanned the whole region within $B_{1}$, and it should have crossed some of the inner boundaries. Claim 3. Each region has exactly 3 sharp corners. Proof. Consider again the car of the previous claim, with its laser still firmly attached to its front, traveling the same way as before and going through the same consecutive sharp corners $P_{1}, P_{2}$ and $P_{3}$. As we have seen, as the car goes counterclockwise from $P_{1}$ to $P_{3}$, the laser endpoint goes clockwise from $P_{1}$ to $P_{3}$, so together they cover the whole boundary. If there were a fourth sharp corner $P_{4}$, at some moment the laser endpoint would pass through it. But, since $P_{4}$ is a sharp corner, this means the car must be on the extension of a tangent segment going through $P_{4}$. Since the car is not on that segment itself (the car never goes through $P_{4}$ ), we would have 3 circles with a common tangent line, which is not allowed. ![](https:\/\/cdn.mathpix.com\/cropped\/2024_11_18_ff08e3c077fa2f8c16b2g-76.jpg?height=538&width=792&top_left_y=1007&top_left_x=643) We are now ready to finish the solution. Let $r$ be the number of internal regions, and $s$ be the number of tangent segments. Since each tangent segment contributes exactly 2 sharp corners to the diagram, and each region has exactly 3 sharp corners, we must have $2 s=3 r$. Since the graph corresponding to the diagram is connected, we can use Euler's formula $n-s+r=1$ and find $s=3 n-3$ and $r=2 n-2$.","tier":0} +{"problem_type":"Number Theory","problem_label":"N1","problem":"The sequence $a_{0}, a_{1}, a_{2}, \\ldots$ of positive integers satisfies $$ a_{n+1}=\\left\\{\\begin{array}{ll} \\sqrt{a_{n}}, & \\text { if } \\sqrt{a_{n}} \\text { is an integer } \\\\ a_{n}+3, & \\text { otherwise } \\end{array} \\quad \\text { for every } n \\geqslant 0\\right. $$ Determine all values of $a_{0}>1$ for which there is at least one number $a$ such that $a_{n}=a$ for infinitely many values of $n$. (South Africa) Answer: All positive multiples of 3.","solution":"Since the value of $a_{n+1}$ only depends on the value of $a_{n}$, if $a_{n}=a_{m}$ for two different indices $n$ and $m$, then the sequence is eventually periodic. So we look for the values of $a_{0}$ for which the sequence is eventually periodic. Claim 1. If $a_{n} \\equiv-1(\\bmod 3)$, then, for all $m>n, a_{m}$ is not a perfect square. It follows that the sequence is eventually strictly increasing, so it is not eventually periodic. Proof. A square cannot be congruent to -1 modulo 3 , so $a_{n} \\equiv-1(\\bmod 3)$ implies that $a_{n}$ is not a square, therefore $a_{n+1}=a_{n}+3>a_{n}$. As a consequence, $a_{n+1} \\equiv a_{n} \\equiv-1(\\bmod 3)$, so $a_{n+1}$ is not a square either. By repeating the argument, we prove that, from $a_{n}$ on, all terms of the sequence are not perfect squares and are greater than their predecessors, which completes the proof. Claim 2. If $a_{n} \\not \\equiv-1(\\bmod 3)$ and $a_{n}>9$ then there is an index $m>n$ such that $a_{m}9, t$ is at least 3. The first square in the sequence $a_{n}, a_{n}+3, a_{n}+6, \\ldots$ will be $(t+1)^{2},(t+2)^{2}$ or $(t+3)^{2}$, therefore there is an index $m>n$ such that $a_{m} \\leqslant t+3n$ such that $a_{m}=3$. Proof. First we notice that, by the definition of the sequence, a multiple of 3 is always followed by another multiple of 3 . If $a_{n} \\in\\{3,6,9\\}$ the sequence will eventually follow the periodic pattern $3,6,9,3,6,9, \\ldots$ If $a_{n}>9$, let $j$ be an index such that $a_{j}$ is equal to the minimum value of the set $\\left\\{a_{n+1}, a_{n+2}, \\ldots\\right\\}$. We must have $a_{j} \\leqslant 9$, otherwise we could apply Claim 2 to $a_{j}$ and get a contradiction on the minimality hypothesis. It follows that $a_{j} \\in\\{3,6,9\\}$, and the proof is complete. Claim 4. If $a_{n} \\equiv 1(\\bmod 3)$, then there is an index $m>n$ such that $a_{m} \\equiv-1(\\bmod 3)$. Proof. In the sequence, 4 is always followed by $2 \\equiv-1(\\bmod 3)$, so the claim is true for $a_{n}=4$. If $a_{n}=7$, the next terms will be $10,13,16,4,2, \\ldots$ and the claim is also true. For $a_{n} \\geqslant 10$, we again take an index $j>n$ such that $a_{j}$ is equal to the minimum value of the set $\\left\\{a_{n+1}, a_{n+2}, \\ldots\\right\\}$, which by the definition of the sequence consists of non-multiples of 3 . Suppose $a_{j} \\equiv 1(\\bmod 3)$. Then we must have $a_{j} \\leqslant 9$ by Claim 2 and the minimality of $a_{j}$. It follows that $a_{j} \\in\\{4,7\\}$, so $a_{m}=2j$, contradicting the minimality of $a_{j}$. Therefore, we must have $a_{j} \\equiv-1(\\bmod 3)$. It follows from the previous claims that if $a_{0}$ is a multiple of 3 the sequence will eventually reach the periodic pattern $3,6,9,3,6,9, \\ldots$; if $a_{0} \\equiv-1(\\bmod 3)$ the sequence will be strictly increasing; and if $a_{0} \\equiv 1(\\bmod 3)$ the sequence will be eventually strictly increasing. So the sequence will be eventually periodic if, and only if, $a_{0}$ is a multiple of 3 .","tier":0} +{"problem_type":"Number Theory","problem_label":"N3","problem":"Determine all integers $n \\geqslant 2$ with the following property: for any integers $a_{1}, a_{2}, \\ldots, a_{n}$ whose sum is not divisible by $n$, there exists an index $1 \\leqslant i \\leqslant n$ such that none of the numbers $$ a_{i}, a_{i}+a_{i+1}, \\ldots, a_{i}+a_{i+1}+\\cdots+a_{i+n-1} $$ is divisible by $n$. (We let $a_{i}=a_{i-n}$ when $i>n$.) (Thailand) Answer: These integers are exactly the prime numbers.","solution":"Let us first show that, if $n=a b$, with $a, b \\geqslant 2$ integers, then the property in the statement of the problem does not hold. Indeed, in this case, let $a_{k}=a$ for $1 \\leqslant k \\leqslant n-1$ and $a_{n}=0$. The sum $a_{1}+a_{2}+\\cdots+a_{n}=a \\cdot(n-1)$ is not divisible by $n$. Let $i$ with $1 \\leqslant i \\leqslant n$ be an arbitrary index. Taking $j=b$ if $1 \\leqslant i \\leqslant n-b$, and $j=b+1$ if $n-b0$ such that $10^{t} \\equiv 1$ $(\\bmod \\mathrm{cm})$ for some integer $c \\in C$, that is, the set of orders of 10 modulo cm . In other words, $$ S(m)=\\left\\{\\operatorname{ord}_{c m}(10): c \\in C\\right\\} $$ Since there are $4 \\cdot 201+3=807$ numbers $c$ with $1 \\leqslant c \\leqslant 2017$ and $\\operatorname{gcd}(c, 10)=1$, namely those such that $c \\equiv 1,3,7,9(\\bmod 10)$, $$ |S(m)| \\leqslant|C|=807 $$ Now we find $m$ such that $|S(m)|=807$. Let $$ P=\\{1

0}$. We will show that $k=c$. Denote by $\\nu_{p}(n)$ the number of prime factors $p$ in $n$, that is, the maximum exponent $\\beta$ for which $p^{\\beta} \\mid n$. For every $\\ell \\geqslant 1$ and $p \\in P$, the Lifting the Exponent Lemma provides $$ \\nu_{p}\\left(10^{\\ell \\alpha}-1\\right)=\\nu_{p}\\left(\\left(10^{\\alpha}\\right)^{\\ell}-1\\right)=\\nu_{p}\\left(10^{\\alpha}-1\\right)+\\nu_{p}(\\ell)=\\nu_{p}(m)+\\nu_{p}(\\ell) $$ so $$ \\begin{aligned} c m \\mid 10^{k \\alpha}-1 & \\Longleftrightarrow \\forall p \\in P ; \\nu_{p}(c m) \\leqslant \\nu_{p}\\left(10^{k \\alpha}-1\\right) \\\\ & \\Longleftrightarrow \\forall p \\in P ; \\nu_{p}(m)+\\nu_{p}(c) \\leqslant \\nu_{p}(m)+\\nu_{p}(k) \\\\ & \\Longleftrightarrow \\forall p \\in P ; \\nu_{p}(c) \\leqslant \\nu_{p}(k) \\\\ & \\Longleftrightarrow c \\mid k . \\end{aligned} $$ The first such $k$ is $k=c$, so $\\operatorname{ord}_{c m}(10)=c \\alpha$. Comment. The Lifting the Exponent Lemma states that, for any odd prime $p$, any integers $a, b$ coprime with $p$ such that $p \\mid a-b$, and any positive integer exponent $n$, $$ \\nu_{p}\\left(a^{n}-b^{n}\\right)=\\nu_{p}(a-b)+\\nu_{p}(n), $$ and, for $p=2$, $$ \\nu_{2}\\left(a^{n}-b^{n}\\right)=\\nu_{2}\\left(a^{2}-b^{2}\\right)+\\nu_{p}(n)-1 . $$ Both claims can be proved by induction on $n$.","tier":0} +{"problem_type":"Number Theory","problem_label":"N5","problem":"Find all pairs $(p, q)$ of prime numbers with $p>q$ for which the number $$ \\frac{(p+q)^{p+q}(p-q)^{p-q}-1}{(p+q)^{p-q}(p-q)^{p+q}-1} $$ is an integer. (Japan) Answer: The only such pair is $(3,2)$.","solution":"Let $M=(p+q)^{p-q}(p-q)^{p+q}-1$, which is relatively prime with both $p+q$ and $p-q$. Denote by $(p-q)^{-1}$ the multiplicative inverse of $(p-q)$ modulo $M$. By eliminating the term -1 in the numerator, $$ \\begin{aligned} (p+q)^{p+q}(p-q)^{p-q}-1 & \\equiv(p+q)^{p-q}(p-q)^{p+q}-1 \\quad(\\bmod M) \\\\ (p+q)^{2 q} & \\equiv(p-q)^{2 q} \\quad(\\bmod M) \\\\ \\left((p+q) \\cdot(p-q)^{-1}\\right)^{2 q} & \\equiv 1 \\quad(\\bmod M) \\end{aligned} $$ Case 1: $q \\geqslant 5$. Consider an arbitrary prime divisor $r$ of $M$. Notice that $M$ is odd, so $r \\geqslant 3$. By (2), the multiplicative order of $\\left((p+q) \\cdot(p-q)^{-1}\\right)$ modulo $r$ is a divisor of the exponent $2 q$ in (2), so it can be $1,2, q$ or $2 q$. By Fermat's theorem, the order divides $r-1$. So, if the order is $q$ or $2 q$ then $r \\equiv 1(\\bmod q)$. If the order is 1 or 2 then $r \\mid(p+q)^{2}-(p-q)^{2}=4 p q$, so $r=p$ or $r=q$. The case $r=p$ is not possible, because, by applying Fermat's theorem, $M=(p+q)^{p-q}(p-q)^{p+q}-1 \\equiv q^{p-q}(-q)^{p+q}-1=\\left(q^{2}\\right)^{p}-1 \\equiv q^{2}-1=(q+1)(q-1) \\quad(\\bmod p)$ and the last factors $q-1$ and $q+1$ are less than $p$ and thus $p \\nmid M$. Hence, all prime divisors of $M$ are either $q$ or of the form $k q+1$; it follows that all positive divisors of $M$ are congruent to 0 or 1 modulo $q$. Now notice that $$ M=\\left((p+q)^{\\frac{p-q}{2}}(p-q)^{\\frac{p+q}{2}}-1\\right)\\left((p+q)^{\\frac{p-q}{2}}(p-q)^{\\frac{p+q}{2}}+1\\right) $$ is the product of two consecutive positive odd numbers; both should be congruent to 0 or 1 modulo $q$. But this is impossible by the assumption $q \\geqslant 5$. So, there is no solution in Case 1 . Case 2: $q=2$. By (1), we have $M \\mid(p+q)^{2 q}-(p-q)^{2 q}=(p+2)^{4}-(p-2)^{4}$, so $$ \\begin{gathered} (p+2)^{p-2}(p-2)^{p+2}-1=M \\leqslant(p+2)^{4}-(p-2)^{4} \\leqslant(p+2)^{4}-1, \\\\ (p+2)^{p-6}(p-2)^{p+2} \\leqslant 1 . \\end{gathered} $$ If $p \\geqslant 7$ then the left-hand side is obviously greater than 1 . For $p=5$ we have $(p+2)^{p-6}(p-2)^{p+2}=7^{-1} \\cdot 3^{7}$ which is also too large. There remains only one candidate, $p=3$, which provides a solution: $$ \\frac{(p+q)^{p+q}(p-q)^{p-q}-1}{(p+q)^{p-q}(p-q)^{p+q}-1}=\\frac{5^{5} \\cdot 1^{1}-1}{5^{1} \\cdot 1^{5}-1}=\\frac{3124}{4}=781 . $$ So in Case 2 the only solution is $(p, q)=(3,2)$. Case 3: $q=3$. Similarly to Case 2, we have $$ M \\left\\lvert\\,(p+q)^{2 q}-(p-q)^{2 q}=64 \\cdot\\left(\\left(\\frac{p+3}{2}\\right)^{6}-\\left(\\frac{p-3}{2}\\right)^{6}\\right)\\right. $$ Since $M$ is odd, we conclude that $$ M \\left\\lvert\\,\\left(\\frac{p+3}{2}\\right)^{6}-\\left(\\frac{p-3}{2}\\right)^{6}\\right. $$ and $$ \\begin{gathered} (p+3)^{p-3}(p-3)^{p+3}-1=M \\leqslant\\left(\\frac{p+3}{2}\\right)^{6}-\\left(\\frac{p-3}{2}\\right)^{6} \\leqslant\\left(\\frac{p+3}{2}\\right)^{6}-1 \\\\ 64(p+3)^{p-9}(p-3)^{p+3} \\leqslant 1 \\end{gathered} $$ If $p \\geqslant 11$ then the left-hand side is obviously greater than 1 . If $p=7$ then the left-hand side is $64 \\cdot 10^{-2} \\cdot 4^{10}>1$. If $p=5$ then the left-hand side is $64 \\cdot 8^{-4} \\cdot 2^{8}=2^{2}>1$. Therefore, there is no solution in Case 3.","tier":0} +{"problem_type":"Number Theory","problem_label":"N6","problem":"Find the smallest positive integer $n$, or show that no such $n$ exists, with the following property: there are infinitely many distinct $n$-tuples of positive rational numbers $\\left(a_{1}, a_{2}, \\ldots, a_{n}\\right)$ such that both $$ a_{1}+a_{2}+\\cdots+a_{n} \\quad \\text { and } \\quad \\frac{1}{a_{1}}+\\frac{1}{a_{2}}+\\cdots+\\frac{1}{a_{n}} $$ are integers. (Singapore) Answer: $n=3$.","solution":"For $n=1, a_{1} \\in \\mathbb{Z}_{>0}$ and $\\frac{1}{a_{1}} \\in \\mathbb{Z}_{>0}$ if and only if $a_{1}=1$. Next we show that (i) There are finitely many $(x, y) \\in \\mathbb{Q}_{>0}^{2}$ satisfying $x+y \\in \\mathbb{Z}$ and $\\frac{1}{x}+\\frac{1}{y} \\in \\mathbb{Z}$ Write $x=\\frac{a}{b}$ and $y=\\frac{c}{d}$ with $a, b, c, d \\in \\mathbb{Z}_{>0}$ and $\\operatorname{gcd}(a, b)=\\operatorname{gcd}(c, d)=1$. Then $x+y \\in \\mathbb{Z}$ and $\\frac{1}{x}+\\frac{1}{y} \\in \\mathbb{Z}$ is equivalent to the two divisibility conditions $$ b d \\mid a d+b c \\quad(1) \\quad \\text { and } \\quad a c \\mid a d+b c $$ Condition (1) implies that $d|a d+b c \\Longleftrightarrow d| b c \\Longleftrightarrow d \\mid b$ since $\\operatorname{gcd}(c, d)=1$. Still from (1) we get $b|a d+b c \\Longleftrightarrow b| a d \\Longleftrightarrow b \\mid d$ since $\\operatorname{gcd}(a, b)=1$. From $b \\mid d$ and $d \\mid b$ we have $b=d$. An analogous reasoning with condition (2) shows that $a=c$. Hence $x=\\frac{a}{b}=\\frac{c}{d}=y$, i.e., the problem amounts to finding all $x \\in \\mathbb{Q}_{>0}$ such that $2 x \\in \\mathbb{Z}_{>0}$ and $\\frac{2}{x} \\in \\mathbb{Z}_{>0}$. Letting $n=2 x \\in \\mathbb{Z}_{>0}$, we have that $\\frac{2}{x} \\in \\mathbb{Z}_{>0} \\Longleftrightarrow \\frac{4}{n} \\in \\mathbb{Z}_{>0} \\Longleftrightarrow n=1,2$ or 4 , and there are finitely many solutions, namely $(x, y)=\\left(\\frac{1}{2}, \\frac{1}{2}\\right),(1,1)$ or $(2,2)$. (ii) There are infinitely many triples $(x, y, z) \\in \\mathbb{Q}_{>0}^{2}$ such that $x+y+z \\in \\mathbb{Z}$ and $\\frac{1}{x}+\\frac{1}{y}+\\frac{1}{z} \\in \\mathbb{Z}$. We will look for triples such that $x+y+z=1$, so we may write them in the form $$ (x, y, z)=\\left(\\frac{a}{a+b+c}, \\frac{b}{a+b+c}, \\frac{c}{a+b+c}\\right) \\quad \\text { with } a, b, c \\in \\mathbb{Z}_{>0} $$ We want these to satisfy $$ \\frac{1}{x}+\\frac{1}{y}+\\frac{1}{z}=\\frac{a+b+c}{a}+\\frac{a+b+c}{b}+\\frac{a+b+c}{c} \\in \\mathbb{Z} \\Longleftrightarrow \\frac{b+c}{a}+\\frac{a+c}{b}+\\frac{a+b}{c} \\in \\mathbb{Z} $$ Fixing $a=1$, it suffices to find infinitely many pairs $(b, c) \\in \\mathbb{Z}_{>0}^{2}$ such that $$ \\frac{1}{b}+\\frac{1}{c}+\\frac{c}{b}+\\frac{b}{c}=3 \\Longleftrightarrow b^{2}+c^{2}-3 b c+b+c=0 $$ To show that equation (*) has infinitely many solutions, we use Vieta jumping (also known as root flipping): starting with $b=2, c=3$, the following algorithm generates infinitely many solutions. Let $c \\geqslant b$, and view (*) as a quadratic equation in $b$ for $c$ fixed: $$ b^{2}-(3 c-1) \\cdot b+\\left(c^{2}+c\\right)=0 $$ Then there exists another root $b_{0} \\in \\mathbb{Z}$ of ( $\\left.* *\\right)$ which satisfies $b+b_{0}=3 c-1$ and $b \\cdot b_{0}=c^{2}+c$. Since $c \\geqslant b$ by assumption, $$ b_{0}=\\frac{c^{2}+c}{b} \\geqslant \\frac{c^{2}+c}{c}>c $$ Hence from the solution $(b, c)$ we obtain another one $\\left(c, b_{0}\\right)$ with $b_{0}>c$, and we can then \"jump\" again, this time with $c$ as the \"variable\" in the quadratic (*). This algorithm will generate an infinite sequence of distinct solutions, whose first terms are $(2,3),(3,6),(6,14),(14,35),(35,90),(90,234),(234,611),(611,1598),(1598,4182), \\ldots$ Comment. Although not needed for solving this problem, we may also explicitly solve the recursion given by the Vieta jumping. Define the sequence $\\left(x_{n}\\right)$ as follows: $$ x_{0}=2, \\quad x_{1}=3 \\quad \\text { and } \\quad x_{n+2}=3 x_{n+1}-x_{n}-1 \\text { for } n \\geqslant 0 $$ Then the triple $$ (x, y, z)=\\left(\\frac{1}{1+x_{n}+x_{n+1}}, \\frac{x_{n}}{1+x_{n}+x_{n+1}}, \\frac{x_{n+1}}{1+x_{n}+x_{n+1}}\\right) $$ satisfies the problem conditions for all $n \\in \\mathbb{N}$. It is easy to show that $x_{n}=F_{2 n+1}+1$, where $F_{n}$ denotes the $n$-th term of the Fibonacci sequence ( $F_{0}=0, F_{1}=1$, and $F_{n+2}=F_{n+1}+F_{n}$ for $n \\geqslant 0$ ).","tier":0} +{"problem_type":"Number Theory","problem_label":"N6","problem":"Find the smallest positive integer $n$, or show that no such $n$ exists, with the following property: there are infinitely many distinct $n$-tuples of positive rational numbers $\\left(a_{1}, a_{2}, \\ldots, a_{n}\\right)$ such that both $$ a_{1}+a_{2}+\\cdots+a_{n} \\quad \\text { and } \\quad \\frac{1}{a_{1}}+\\frac{1}{a_{2}}+\\cdots+\\frac{1}{a_{n}} $$ are integers. (Singapore) Answer: $n=3$.","solution":"Call the $n$-tuples $\\left(a_{1}, a_{2}, \\ldots, a_{n}\\right) \\in \\mathbb{Q}_{>0}^{n}$ satisfying the conditions of the problem statement good, and those for which $$ f\\left(a_{1}, \\ldots, a_{n}\\right) \\stackrel{\\text { def }}{=}\\left(a_{1}+a_{2}+\\cdots+a_{n}\\right)\\left(\\frac{1}{a_{1}}+\\frac{1}{a_{2}}+\\cdots+\\frac{1}{a_{n}}\\right) $$ is an integer pretty. Then good $n$-tuples are pretty, and if $\\left(b_{1}, \\ldots, b_{n}\\right)$ is pretty then $$ \\left(\\frac{b_{1}}{b_{1}+b_{2}+\\cdots+b_{n}}, \\frac{b_{2}}{b_{1}+b_{2}+\\cdots+b_{n}}, \\ldots, \\frac{b_{n}}{b_{1}+b_{2}+\\cdots+b_{n}}\\right) $$ is good since the sum of its components is 1 , and the sum of the reciprocals of its components equals $f\\left(b_{1}, \\ldots, b_{n}\\right)$. We declare pretty $n$-tuples proportional to each other equivalent since they are precisely those which give rise to the same good $n$-tuple. Clearly, each such equivalence class contains exactly one $n$-tuple of positive integers having no common prime divisors. Call such $n$-tuple a primitive pretty tuple. Our task is to find infinitely many primitive pretty $n$-tuples. For $n=1$, there is clearly a single primitive 1 -tuple. For $n=2$, we have $f(a, b)=\\frac{(a+b)^{2}}{a b}$, which can be integral (for coprime $a, b \\in \\mathbb{Z}_{>0}$ ) only if $a=b=1$ (see for instance (i) in the first solution). Now we construct infinitely many primitive pretty triples for $n=3$. Fix $b, c, k \\in \\mathbb{Z}_{>0}$; we will try to find sufficient conditions for the existence of an $a \\in \\mathbb{Q}_{>0}$ such that $f(a, b, c)=k$. Write $\\sigma=b+c, \\tau=b c$. From $f(a, b, c)=k$, we have that $a$ should satisfy the quadratic equation $$ a^{2} \\cdot \\sigma+a \\cdot\\left(\\sigma^{2}-(k-1) \\tau\\right)+\\sigma \\tau=0 $$ whose discriminant is $$ \\Delta=\\left(\\sigma^{2}-(k-1) \\tau\\right)^{2}-4 \\sigma^{2} \\tau=\\left((k+1) \\tau-\\sigma^{2}\\right)^{2}-4 k \\tau^{2} $$ We need it to be a square of an integer, say, $\\Delta=M^{2}$ for some $M \\in \\mathbb{Z}$, i.e., we want $$ \\left((k+1) \\tau-\\sigma^{2}\\right)^{2}-M^{2}=2 k \\cdot 2 \\tau^{2} $$ so that it suffices to set $$ (k+1) \\tau-\\sigma^{2}=\\tau^{2}+k, \\quad M=\\tau^{2}-k . $$ The first relation reads $\\sigma^{2}=(\\tau-1)(k-\\tau)$, so if $b$ and $c$ satisfy $$ \\tau-1 \\mid \\sigma^{2} \\quad \\text { i.e. } \\quad b c-1 \\mid(b+c)^{2} $$ then $k=\\frac{\\sigma^{2}}{\\tau-1}+\\tau$ will be integral, and we find rational solutions to (1), namely $$ a=\\frac{\\sigma}{\\tau-1}=\\frac{b+c}{b c-1} \\quad \\text { or } \\quad a=\\frac{\\tau^{2}-\\tau}{\\sigma}=\\frac{b c \\cdot(b c-1)}{b+c} $$ We can now find infinitely many pairs ( $b, c$ ) satisfying (2) by Vieta jumping. For example, if we impose $$ (b+c)^{2}=5 \\cdot(b c-1) $$ then all pairs $(b, c)=\\left(v_{i}, v_{i+1}\\right)$ satisfy the above condition, where $$ v_{1}=2, v_{2}=3, \\quad v_{i+2}=3 v_{i+1}-v_{i} \\quad \\text { for } i \\geqslant 0 $$ For $(b, c)=\\left(v_{i}, v_{i+1}\\right)$, one of the solutions to (1) will be $a=(b+c) \/(b c-1)=5 \/(b+c)=$ $5 \/\\left(v_{i}+v_{i+1}\\right)$. Then the pretty triple $(a, b, c)$ will be equivalent to the integral pretty triple $$ \\left(5, v_{i}\\left(v_{i}+v_{i+1}\\right), v_{i+1}\\left(v_{i}+v_{i+1}\\right)\\right) $$ After possibly dividing by 5 , we obtain infinitely many primitive pretty triples, as required. Comment. There are many other infinite series of $(b, c)=\\left(v_{i}, v_{i+1}\\right)$ with $b c-1 \\mid(b+c)^{2}$. Some of them are: $$ \\begin{array}{llll} v_{1}=1, & v_{2}=3, & v_{i+1}=6 v_{i}-v_{i-1}, & \\left(v_{i}+v_{i+1}\\right)^{2}=8 \\cdot\\left(v_{i} v_{i+1}-1\\right) ; \\\\ v_{1}=1, & v_{2}=2, & v_{i+1}=7 v_{i}-v_{i-1}, & \\left(v_{i}+v_{i+1}\\right)^{2}=9 \\cdot\\left(v_{i} v_{i+1}-1\\right) ; \\\\ v_{1}=1, & v_{2}=5, & v_{i+1}=7 v_{i}-v_{i-1}, & \\left(v_{i}+v_{i+1}\\right)^{2}=9 \\cdot\\left(v_{i} v_{i+1}-1\\right) \\end{array} $$ (the last two are in fact one sequence prolonged in two possible directions).","tier":0} +{"problem_type":"Number Theory","problem_label":"N8","problem":"Let $p$ be an odd prime number and $\\mathbb{Z}_{>0}$ be the set of positive integers. Suppose that a function $f: \\mathbb{Z}_{>0} \\times \\mathbb{Z}_{>0} \\rightarrow\\{0,1\\}$ satisfies the following properties: - $f(1,1)=0$; - $f(a, b)+f(b, a)=1$ for any pair of relatively prime positive integers $(a, b)$ not both equal to 1 ; - $f(a+b, b)=f(a, b)$ for any pair of relatively prime positive integers $(a, b)$. Prove that $$ \\sum_{n=1}^{p-1} f\\left(n^{2}, p\\right) \\geqslant \\sqrt{2 p}-2 $$ (Italy)","solution":"Denote by $\\mathbb{A}$ the set of all pairs of coprime positive integers. Notice that for every $(a, b) \\in \\mathbb{A}$ there exists a pair $(u, v) \\in \\mathbb{Z}^{2}$ with $u a+v b=1$. Moreover, if $\\left(u_{0}, v_{0}\\right)$ is one such pair, then all such pairs are of the form $(u, v)=\\left(u_{0}+k b, v_{0}-k a\\right)$, where $k \\in \\mathbb{Z}$. So there exists a unique such pair $(u, v)$ with $-b \/ 20$. Proof. We induct on $a+b$. The base case is $a+b=2$. In this case, we have that $a=b=1$, $g(a, b)=g(1,1)=(0,1)$ and $f(1,1)=0$, so the claim holds. Assume now that $a+b>2$, and so $a \\neq b$, since $a$ and $b$ are coprime. Two cases are possible. Case 1: $a>b$. Notice that $g(a-b, b)=(u, v+u)$, since $u(a-b)+(v+u) b=1$ and $u \\in(-b \/ 2, b \/ 2]$. Thus $f(a, b)=1 \\Longleftrightarrow f(a-b, b)=1 \\Longleftrightarrow u>0$ by the induction hypothesis. Case 2: $av b \\geqslant 1-\\frac{a b}{2}, \\quad \\text { so } \\quad \\frac{1+a}{2} \\geqslant \\frac{1}{b}+\\frac{a}{2}>v \\geqslant \\frac{1}{b}-\\frac{a}{2}>-\\frac{a}{2} . $$ Thus $1+a>2 v>-a$, so $a \\geqslant 2 v>-a$, hence $a \/ 2 \\geqslant v>-a \/ 2$, and thus $g(b, a)=(v, u)$. Observe that $f(a, b)=1 \\Longleftrightarrow f(b, a)=0 \\Longleftrightarrow f(b-a, a)=0$. We know from Case 1 that $g(b-a, a)=(v, u+v)$. We have $f(b-a, a)=0 \\Longleftrightarrow v \\leqslant 0$ by the inductive hypothesis. Then, since $b>a \\geqslant 1$ and $u a+v b=1$, we have $v \\leqslant 0 \\Longleftrightarrow u>0$, and we are done. The Lemma proves that, for all $(a, b) \\in \\mathbb{A}, f(a, b)=1$ if and only if the inverse of $a$ modulo $b$, taken in $\\{1,2, \\ldots, b-1\\}$, is at most $b \/ 2$. Then, for any odd prime $p$ and integer $n$ such that $n \\not \\equiv 0(\\bmod p), f\\left(n^{2}, p\\right)=1$ iff the inverse of $n^{2} \\bmod p$ is less than $p \/ 2$. Since $\\left\\{n^{2} \\bmod p: 1 \\leqslant n \\leqslant p-1\\right\\}=\\left\\{n^{-2} \\bmod p: 1 \\leqslant n \\leqslant p-1\\right\\}$, including multiplicities (two for each quadratic residue in each set), we conclude that the desired sum is twice the number of quadratic residues that are less than $p \/ 2$, i.e., $$ \\left.\\sum_{n=1}^{p-1} f\\left(n^{2}, p\\right)=2 \\left\\lvert\\,\\left\\{k: 1 \\leqslant k \\leqslant \\frac{p-1}{2} \\text { and } k^{2} \\bmod p<\\frac{p}{2}\\right\\}\\right. \\right\\rvert\\, . $$ Since the number of perfect squares in the interval $[1, p \/ 2)$ is $\\lfloor\\sqrt{p \/ 2}\\rfloor>\\sqrt{p \/ 2}-1$, we conclude that $$ \\sum_{n=1}^{p-1} f\\left(n^{2}, p\\right)>2\\left(\\sqrt{\\frac{p}{2}}-1\\right)=\\sqrt{2 p}-2 $$","tier":0} +{"problem_type":"Number Theory","problem_label":"N8","problem":"Let $p$ be an odd prime number and $\\mathbb{Z}_{>0}$ be the set of positive integers. Suppose that a function $f: \\mathbb{Z}_{>0} \\times \\mathbb{Z}_{>0} \\rightarrow\\{0,1\\}$ satisfies the following properties: - $f(1,1)=0$; - $f(a, b)+f(b, a)=1$ for any pair of relatively prime positive integers $(a, b)$ not both equal to 1 ; - $f(a+b, b)=f(a, b)$ for any pair of relatively prime positive integers $(a, b)$. Prove that $$ \\sum_{n=1}^{p-1} f\\left(n^{2}, p\\right) \\geqslant \\sqrt{2 p}-2 $$ (Italy)","solution":"We provide a different proof for the Lemma. For this purpose, we use continued fractions to find $g(a, b)=(u, v)$ explicitly. The function $f$ is completely determined on $\\mathbb{A}$ by the following Claim. Represent $a \/ b$ as a continued fraction; that is, let $a_{0}$ be an integer and $a_{1}, \\ldots, a_{k}$ be positive integers such that $a_{k} \\geqslant 2$ and $$ \\frac{a}{b}=a_{0}+\\frac{1}{a_{1}+\\frac{1}{a_{2}+\\frac{1}{\\cdots+\\frac{1}{a_{k}}}}}=\\left[a_{0} ; a_{1}, a_{2}, \\ldots, a_{k}\\right] . $$ Then $f(a, b)=0 \\Longleftrightarrow k$ is even. Proof. We induct on $b$. If $b=1$, then $a \/ b=[a]$ and $k=0$. Then, for $a \\geqslant 1$, an easy induction shows that $f(a, 1)=f(1,1)=0$. Now consider the case $b>1$. Perform the Euclidean division $a=q b+r$, with $0 \\leqslant r0$ and define $q_{-1}=0$ if necessary. Then - $q_{k}=a_{k} q_{k-1}+q_{k-2}, \\quad$ and - $a q_{k-1}-b p_{k-1}=p_{k} q_{k-1}-q_{k} p_{k-1}=(-1)^{k-1}$. Assume that $k>0$. Then $a_{k} \\geqslant 2$, and $$ b=q_{k}=a_{k} q_{k-1}+q_{k-2} \\geqslant a_{k} q_{k-1} \\geqslant 2 q_{k-1} \\Longrightarrow q_{k-1} \\leqslant \\frac{b}{2} $$ with strict inequality for $k>1$, and $$ (-1)^{k-1} q_{k-1} a+(-1)^{k} p_{k-1} b=1 $$ Now we finish the proof of the Lemma. It is immediate for $k=0$. If $k=1$, then $(-1)^{k-1}=1$, so $$ -b \/ 2<0 \\leqslant(-1)^{k-1} q_{k-1} \\leqslant b \/ 2 $$ If $k>1$, we have $q_{k-1}0$, we find that $g(a, b)=\\left((-1)^{k-1} q_{k-1},(-1)^{k} p_{k-1}\\right)$, and so $$ f(a, b)=1 \\Longleftrightarrow k \\text { is odd } \\Longleftrightarrow u=(-1)^{k-1} q_{k-1}>0 $$ Comment 1. The Lemma can also be established by observing that $f$ is uniquely defined on $\\mathbb{A}$, defining $f_{1}(a, b)=1$ if $u>0$ in $g(a, b)=(u, v)$ and $f_{1}(a, b)=0$ otherwise, and verifying that $f_{1}$ satisfies all the conditions from the statement. It seems that the main difficulty of the problem is in conjecturing the Lemma. Comment 2. The case $p \\equiv 1(\\bmod 4)$ is, in fact, easier than the original problem. We have, in general, for $1 \\leqslant a \\leqslant p-1$, $f(a, p)=1-f(p, a)=1-f(p-a, a)=f(a, p-a)=f(a+(p-a), p-a)=f(p, p-a)=1-f(p-a, p)$. If $p \\equiv 1(\\bmod 4)$, then $a$ is a quadratic residue modulo $p$ if and only if $p-a$ is a quadratic residue modulo $p$. Therefore, denoting by $r_{k}$ (with $1 \\leqslant r_{k} \\leqslant p-1$ ) the remainder of the division of $k^{2}$ by $p$, we get $$ \\sum_{n=1}^{p-1} f\\left(n^{2}, p\\right)=\\sum_{n=1}^{p-1} f\\left(r_{n}, p\\right)=\\frac{1}{2} \\sum_{n=1}^{p-1}\\left(f\\left(r_{n}, p\\right)+f\\left(p-r_{n}, p\\right)\\right)=\\frac{p-1}{2} $$ Comment 3. The estimate for the sum $\\sum_{n=1}^{p} f\\left(n^{2}, p\\right)$ can be improved by refining the final argument in Solution 1. In fact, one can prove that $$ \\sum_{n=1}^{p-1} f\\left(n^{2}, p\\right) \\geqslant \\frac{p-1}{16} $$ By counting the number of perfect squares in the intervals $[k p,(k+1 \/ 2) p)$, we find that $$ \\sum_{n=1}^{p-1} f\\left(n^{2}, p\\right)=\\sum_{k=0}^{p-1}\\left(\\left\\lfloor\\sqrt{\\left(k+\\frac{1}{2}\\right) p}\\right\\rfloor-\\lfloor\\sqrt{k p}\\rfloor\\right) . $$ Each summand of (2) is non-negative. We now estimate the number of positive summands. Suppose that a summand is zero, i.e., $$ \\left\\lfloor\\sqrt{\\left(k+\\frac{1}{2}\\right) p}\\right\\rfloor=\\lfloor\\sqrt{k p}\\rfloor=: q . $$ Then both of the numbers $k p$ and $k p+p \/ 2$ lie within the interval $\\left[q^{2},(q+1)^{2}\\right)$. Hence $$ \\frac{p}{2}<(q+1)^{2}-q^{2} $$ which implies $$ q \\geqslant \\frac{p-1}{4} $$ Since $q \\leqslant \\sqrt{k p}$, if the $k^{\\text {th }}$ summand of (2) is zero, then $$ k \\geqslant \\frac{q^{2}}{p} \\geqslant \\frac{(p-1)^{2}}{16 p}>\\frac{p-2}{16} \\Longrightarrow k \\geqslant \\frac{p-1}{16} $$ So at least the first $\\left\\lceil\\frac{p-1}{16}\\right\\rceil$ summands (from $k=0$ to $k=\\left\\lceil\\frac{p-1}{16}\\right\\rceil-1$ ) are positive, and the result follows. Comment 4. The bound can be further improved by using different methods. In fact, we prove that $$ \\sum_{n=1}^{p-1} f\\left(n^{2}, p\\right) \\geqslant \\frac{p-3}{4} $$ To that end, we use the Legendre symbol $$ \\left(\\frac{a}{p}\\right)= \\begin{cases}0 & \\text { if } p \\mid a \\\\ 1 & \\text { if } a \\text { is a nonzero quadratic residue } \\bmod p \\\\ -1 & \\text { otherwise. }\\end{cases} $$ We start with the following Claim, which tells us that there are not too many consecutive quadratic residues or consecutive quadratic non-residues. Claim. $\\sum_{n=1}^{p-1}\\left(\\frac{n}{p}\\right)\\left(\\frac{n+1}{p}\\right)=-1$. Proof. We have $\\left(\\frac{n}{p}\\right)\\left(\\frac{n+1}{p}\\right)=\\left(\\frac{n(n+1)}{p}\\right)$. For $1 \\leqslant n \\leqslant p-1$, we get that $n(n+1) \\equiv n^{2}\\left(1+n^{-1}\\right)(\\bmod p)$, hence $\\left(\\frac{n(n+1)}{p}\\right)=\\left(\\frac{1+n^{-1}}{p}\\right)$. Since $\\left\\{1+n^{-1} \\bmod p: 1 \\leqslant n \\leqslant p-1\\right\\}=\\{0,2,3, \\ldots, p-1 \\bmod p\\}$, we find $$ \\sum_{n=1}^{p-1}\\left(\\frac{n}{p}\\right)\\left(\\frac{n+1}{p}\\right)=\\sum_{n=1}^{p-1}\\left(\\frac{1+n^{-1}}{p}\\right)=\\sum_{n=1}^{p-1}\\left(\\frac{n}{p}\\right)-1=-1 $$ because $\\sum_{n=1}^{p}\\left(\\frac{n}{p}\\right)=0$. Observe that (1) becomes $$ \\sum_{n=1}^{p-1} f\\left(n^{2}, p\\right)=2|S|, \\quad S=\\left\\{r: 1 \\leqslant r \\leqslant \\frac{p-1}{2} \\text { and }\\left(\\frac{r}{p}\\right)=1\\right\\} $$ We connect $S$ with the sum from the claim by pairing quadratic residues and quadratic non-residues. To that end, define $$ \\begin{aligned} S^{\\prime} & =\\left\\{r: 1 \\leqslant r \\leqslant \\frac{p-1}{2} \\text { and }\\left(\\frac{r}{p}\\right)=-1\\right\\} \\\\ T & =\\left\\{r: \\frac{p+1}{2} \\leqslant r \\leqslant p-1 \\text { and }\\left(\\frac{r}{p}\\right)=1\\right\\} \\\\ T^{\\prime} & =\\left\\{r: \\frac{p+1}{2} \\leqslant r \\leqslant p-1 \\text { and }\\left(\\frac{r}{p}\\right)=-1\\right\\} \\end{aligned} $$ Since there are exactly $(p-1) \/ 2$ nonzero quadratic residues modulo $p,|S|+|T|=(p-1) \/ 2$. Also we obviously have $|T|+\\left|T^{\\prime}\\right|=(p-1) \/ 2$. Then $|S|=\\left|T^{\\prime}\\right|$. For the sake of brevity, define $t=|S|=\\left|T^{\\prime}\\right|$. If $\\left(\\frac{n}{p}\\right)\\left(\\frac{n+1}{p}\\right)=-1$, then exactly of one the numbers $\\left(\\frac{n}{p}\\right)$ and $\\left(\\frac{n+1}{p}\\right)$ is equal to 1 , so $$ \\left\\lvert\\,\\left\\{n: 1 \\leqslant n \\leqslant \\frac{p-3}{2} \\text { and }\\left(\\frac{n}{p}\\right)\\left(\\frac{n+1}{p}\\right)=-1\\right\\}|\\leqslant|S|+|S-1|=2 t\\right. $$ On the other hand, if $\\left(\\frac{n}{p}\\right)\\left(\\frac{n+1}{p}\\right)=-1$, then exactly one of $\\left(\\frac{n}{p}\\right)$ and $\\left(\\frac{n+1}{p}\\right)$ is equal to -1 , and $$ \\left\\lvert\\,\\left\\{n: \\frac{p+1}{2} \\leqslant n \\leqslant p-2 \\text { and }\\left(\\frac{n}{p}\\right)\\left(\\frac{n+1}{p}\\right)=-1\\right\\}\\left|\\leqslant\\left|T^{\\prime}\\right|+\\left|T^{\\prime}-1\\right|=2 t .\\right.\\right. $$ Thus, taking into account that the middle term $\\left(\\frac{(p-1) \/ 2}{p}\\right)\\left(\\frac{(p+1) \/ 2}{p}\\right)$ may happen to be -1 , $$ \\left.\\left\\lvert\\,\\left\\{n: 1 \\leqslant n \\leqslant p-2 \\text { and }\\left(\\frac{n}{p}\\right)\\left(\\frac{n+1}{p}\\right)=-1\\right\\}\\right. \\right\\rvert\\, \\leqslant 4 t+1 $$ This implies that $$ \\left.\\left\\lvert\\,\\left\\{n: 1 \\leqslant n \\leqslant p-2 \\text { and }\\left(\\frac{n}{p}\\right)\\left(\\frac{n+1}{p}\\right)=1\\right\\}\\right. \\right\\rvert\\, \\geqslant(p-2)-(4 t+1)=p-4 t-3 $$ and so $$ -1=\\sum_{n=1}^{p-1}\\left(\\frac{n}{p}\\right)\\left(\\frac{n+1}{p}\\right) \\geqslant p-4 t-3-(4 t+1)=p-8 t-4 $$ which implies $8 t \\geqslant p-3$, and thus $$ \\sum_{n=1}^{p-1} f\\left(n^{2}, p\\right)=2 t \\geqslant \\frac{p-3}{4} $$ Comment 5. It is possible to prove that $$ \\sum_{n=1}^{p-1} f\\left(n^{2}, p\\right) \\geqslant \\frac{p-1}{2} $$ The case $p \\equiv 1(\\bmod 4)$ was already mentioned, and it is the equality case. If $p \\equiv 3(\\bmod 4)$, then, by a theorem of Dirichlet, we have $$ \\left.\\left\\lvert\\,\\left\\{r: 1 \\leqslant r \\leqslant \\frac{p-1}{2} \\text { and }\\left(\\frac{r}{p}\\right)=1\\right\\}\\right. \\right\\rvert\\,>\\frac{p-1}{4} $$ which implies the result. See https:\/\/en.wikipedia.org\/wiki\/Quadratic_residue\\#Dirichlet.27s_formulas for the full statement of the theorem. It seems that no elementary proof of it is known; a proof using complex analysis is available, for instance, in Chapter 7 of the book Quadratic Residues and Non-Residues: Selected Topics, by Steve Wright, available in https:\/\/arxiv.org\/abs\/1408.0235. ![](https:\/\/cdn.mathpix.com\/cropped\/2024_11_18_ff08e3c077fa2f8c16b2g-96.jpg?height=138&width=204&top_left_y=2241&top_left_x=315) BI\u00caNIO DA MATEMATICA BRASIL ![](https:\/\/cdn.mathpix.com\/cropped\/2024_11_18_ff08e3c077fa2f8c16b2g-96.jpg?height=249&width=683&top_left_y=2251&top_left_x=1052) ![](https:\/\/cdn.mathpix.com\/cropped\/2024_11_18_ff08e3c077fa2f8c16b2g-96.jpg?height=132&width=338&top_left_y=2607&top_left_x=1436) [^0]: *The name Dirichlet interval is chosen for the reason that $g$ theoretically might act similarly to the Dirichlet function on this interval.","tier":0} diff --git a/IMO/segmented/en-IMO2018SL.jsonl b/IMO/segmented/en-IMO2018SL.jsonl new file mode 100644 index 0000000000000000000000000000000000000000..e5fec13cb61e65d1c5760195ddde9ae3107d3bec --- /dev/null +++ b/IMO/segmented/en-IMO2018SL.jsonl @@ -0,0 +1,69 @@ +{"problem_type":"Algebra","problem_label":"A1","problem":"Let $\\mathbb{Q}_{>0}$ denote the set of all positive rational numbers. Determine all functions $f: \\mathbb{Q}_{>0} \\rightarrow \\mathbb{Q}_{>0}$ satisfying for all $x, y \\in \\mathbb{Q}_{>0}$. $$ f\\left(x^{2} f(y)^{2}\\right)=f(x)^{2} f(y) $$ (Switzerland)","solution":"Take any $a, b \\in \\mathbb{Q}_{>0}$. By substituting $x=f(a), y=b$ and $x=f(b), y=a$ into $(*)$ we get $$ f(f(a))^{2} f(b)=f\\left(f(a)^{2} f(b)^{2}\\right)=f(f(b))^{2} f(a) $$ which yields $$ \\frac{f(f(a))^{2}}{f(a)}=\\frac{f(f(b))^{2}}{f(b)} \\quad \\text { for all } a, b \\in \\mathbb{Q}_{>0} $$ In other words, this shows that there exists a constant $C \\in \\mathbb{Q}_{>0}$ such that $f(f(a))^{2}=C f(a)$, or $$ \\left(\\frac{f(f(a))}{C}\\right)^{2}=\\frac{f(a)}{C} \\quad \\text { for all } a \\in \\mathbb{Q}_{>0} \\text {. } $$ Denote by $f^{n}(x)=\\underbrace{f(f(\\ldots(f}_{n}(x)) \\ldots))$ the $n^{\\text {th }}$ iteration of $f$. Equality (1) yields $$ \\frac{f(a)}{C}=\\left(\\frac{f^{2}(a)}{C}\\right)^{2}=\\left(\\frac{f^{3}(a)}{C}\\right)^{4}=\\cdots=\\left(\\frac{f^{n+1}(a)}{C}\\right)^{2^{n}} $$ for all positive integer $n$. So, $f(a) \/ C$ is the $2^{n}$-th power of a rational number for all positive integer $n$. This is impossible unless $f(a) \/ C=1$, since otherwise the exponent of some prime in the prime decomposition of $f(a) \/ C$ is not divisible by sufficiently large powers of 2 . Therefore, $f(a)=C$ for all $a \\in \\mathbb{Q}_{>0}$. Finally, after substituting $f \\equiv C$ into (*) we get $C=C^{3}$, whence $C=1$. So $f(x) \\equiv 1$ is the unique function satisfying $(*)$. Comment 1. There are several variations of the solution above. For instance, one may start with finding $f(1)=1$. To do this, let $d=f(1)$. By substituting $x=y=1$ and $x=d^{2}, y=1$ into (*) we get $f\\left(d^{2}\\right)=d^{3}$ and $f\\left(d^{6}\\right)=f\\left(d^{2}\\right)^{2} \\cdot d=d^{7}$. By substituting now $x=1, y=d^{2}$ we obtain $f\\left(d^{6}\\right)=d^{2} \\cdot d^{3}=d^{5}$. Therefore, $d^{7}=f\\left(d^{6}\\right)=d^{5}$, whence $d=1$. After that, the rest of the solution simplifies a bit, since we already know that $C=\\frac{f(f(1))^{2}}{f(1)}=1$. Hence equation (1) becomes merely $f(f(a))^{2}=f(a)$, which yields $f(a)=1$ in a similar manner. Comment 2. There exist nonconstant functions $f: \\mathbb{R}^{+} \\rightarrow \\mathbb{R}^{+}$satisfying $(*)$ for all real $x, y>0-$ e.g., $f(x)=\\sqrt{x}$.","tier":0} +{"problem_type":"Algebra","problem_label":"A2","problem":"Find all positive integers $n \\geqslant 3$ for which there exist real numbers $a_{1}, a_{2}, \\ldots, a_{n}$, $a_{n+1}=a_{1}, a_{n+2}=a_{2}$ such that $$ a_{i} a_{i+1}+1=a_{i+2} $$ for all $i=1,2, \\ldots, n$. (Slovakia)","solution":"For the sake of convenience, extend the sequence $a_{1}, \\ldots, a_{n+2}$ to an infinite periodic sequence with period $n$. ( $n$ is not necessarily the shortest period.) If $n$ is divisible by 3 , then $\\left(a_{1}, a_{2}, \\ldots\\right)=(-1,-1,2,-1,-1,2, \\ldots)$ is an obvious solution. We will show that in every periodic sequence satisfying the recurrence, each positive term is followed by two negative values, and after them the next number is positive again. From this, it follows that $n$ is divisible by 3 . If the sequence contains two consecutive positive numbers $a_{i}, a_{i+1}$, then $a_{i+2}=a_{i} a_{i+1}+1>1$, so the next value is positive as well; by induction, all numbers are positive and greater than 1 . But then $a_{i+2}=a_{i} a_{i+1}+1 \\geqslant 1 \\cdot a_{i+1}+1>a_{i+1}$ for every index $i$, which is impossible: our sequence is periodic, so it cannot increase everywhere. If the number 0 occurs in the sequence, $a_{i}=0$ for some index $i$, then it follows that $a_{i+1}=a_{i-1} a_{i}+1$ and $a_{i+2}=a_{i} a_{i+1}+1$ are two consecutive positive elements in the sequences and we get the same contradiction again. Notice that after any two consecutive negative numbers the next one must be positive: if $a_{i}<0$ and $a_{i+1}<0$, then $a_{i+2}=a_{1} a_{i+1}+1>1>0$. Hence, the positive and negative numbers follow each other in such a way that each positive term is followed by one or two negative values and then comes the next positive term. Consider the case when the positive and negative values alternate. So, if $a_{i}$ is a negative value then $a_{i+1}$ is positive, $a_{i+2}$ is negative and $a_{i+3}$ is positive again. Notice that $a_{i} a_{i+1}+1=a_{i+2}<00$ we conclude $a_{i}1$. The number $a_{i+3}$ must be negative. We show that $a_{i+4}$ also must be negative. Notice that $a_{i+3}$ is negative and $a_{i+4}=a_{i+2} a_{i+3}+1<10, $$ therefore $a_{i+5}>a_{i+4}$. Since at most one of $a_{i+4}$ and $a_{i+5}$ can be positive, that means that $a_{i+4}$ must be negative. Now $a_{i+3}$ and $a_{i+4}$ are negative and $a_{i+5}$ is positive; so after two negative and a positive terms, the next three terms repeat the same pattern. That completes the solution.","tier":0} +{"problem_type":"Algebra","problem_label":"A2","problem":"Find all positive integers $n \\geqslant 3$ for which there exist real numbers $a_{1}, a_{2}, \\ldots, a_{n}$, $a_{n+1}=a_{1}, a_{n+2}=a_{2}$ such that $$ a_{i} a_{i+1}+1=a_{i+2} $$ for all $i=1,2, \\ldots, n$. (Slovakia)","solution":"We prove that the shortest period of the sequence must be 3 . Then it follows that $n$ must be divisible by 3 . Notice that the equation $x^{2}+1=x$ has no real root, so the numbers $a_{1}, \\ldots, a_{n}$ cannot be all equal, hence the shortest period of the sequence cannot be 1 . By applying the recurrence relation for $i$ and $i+1$, $$ \\begin{gathered} \\left(a_{i+2}-1\\right) a_{i+2}=a_{i} a_{i+1} a_{i+2}=a_{i}\\left(a_{i+3}-1\\right), \\quad \\text { so } \\\\ a_{i+2}^{2}-a_{i} a_{i+3}=a_{i+2}-a_{i} . \\end{gathered} $$ By summing over $i=1,2, \\ldots, n$, we get $$ \\sum_{i=1}^{n}\\left(a_{i}-a_{i+3}\\right)^{2}=0 $$ That proves that $a_{i}=a_{i+3}$ for every index $i$, so the sequence $a_{1}, a_{2}, \\ldots$ is indeed periodic with period 3. The shortest period cannot be 1 , so it must be 3 ; therefore, $n$ is divisible by 3 . Comment. By solving the system of equations $a b+1=c, \\quad b c+1=a, \\quad c a+1=b$, it can be seen that the pattern $(-1,-1,2)$ is repeated in all sequences satisfying the problem conditions.","tier":0} +{"problem_type":"Algebra","problem_label":"A3","problem":"Given any set $S$ of positive integers, show that at least one of the following two assertions holds: (1) There exist distinct finite subsets $F$ and $G$ of $S$ such that $\\sum_{x \\in F} 1 \/ x=\\sum_{x \\in G} 1 \/ x$; (2) There exists a positive rational number $r<1$ such that $\\sum_{x \\in F} 1 \/ x \\neq r$ for all finite subsets $F$ of $S$. (Luxembourg)","solution":"Argue indirectly. Agree, as usual, that the empty sum is 0 to consider rationals in $[0,1)$; adjoining 0 causes no harm, since $\\sum_{x \\in F} 1 \/ x=0$ for no nonempty finite subset $F$ of $S$. For every rational $r$ in $[0,1)$, let $F_{r}$ be the unique finite subset of $S$ such that $\\sum_{x \\in F_{r}} 1 \/ x=r$. The argument hinges on the lemma below. Lemma. If $x$ is a member of $S$ and $q$ and $r$ are rationals in $[0,1)$ such that $q-r=1 \/ x$, then $x$ is a member of $F_{q}$ if and only if it is not one of $F_{r}$. Proof. If $x$ is a member of $F_{q}$, then $$ \\sum_{y \\in F_{q} \\backslash\\{x\\}} \\frac{1}{y}=\\sum_{y \\in F_{q}} \\frac{1}{y}-\\frac{1}{x}=q-\\frac{1}{x}=r=\\sum_{y \\in F_{r}} \\frac{1}{y} $$ so $F_{r}=F_{q} \\backslash\\{x\\}$, and $x$ is not a member of $F_{r}$. Conversely, if $x$ is not a member of $F_{r}$, then $$ \\sum_{y \\in F_{r} \\cup\\{x\\}} \\frac{1}{y}=\\sum_{y \\in F_{r}} \\frac{1}{y}+\\frac{1}{x}=r+\\frac{1}{x}=q=\\sum_{y \\in F_{q}} \\frac{1}{y} $$ so $F_{q}=F_{r} \\cup\\{x\\}$, and $x$ is a member of $F_{q}$. Consider now an element $x$ of $S$ and a positive rational $r<1$. Let $n=\\lfloor r x\\rfloor$ and consider the sets $F_{r-k \/ x}, k=0, \\ldots, n$. Since $0 \\leqslant r-n \/ x<1 \/ x$, the set $F_{r-n \/ x}$ does not contain $x$, and a repeated application of the lemma shows that the $F_{r-(n-2 k) \/ x}$ do not contain $x$, whereas the $F_{r-(n-2 k-1) \/ x}$ do. Consequently, $x$ is a member of $F_{r}$ if and only if $n$ is odd. Finally, consider $F_{2 \/ 3}$. By the preceding, $\\lfloor 2 x \/ 3\\rfloor$ is odd for each $x$ in $F_{2 \/ 3}$, so $2 x \/ 3$ is not integral. Since $F_{2 \/ 3}$ is finite, there exists a positive rational $\\varepsilon$ such that $\\lfloor(2 \/ 3-\\varepsilon) x\\rfloor=\\lfloor 2 x \/ 3\\rfloor$ for all $x$ in $F_{2 \/ 3}$. This implies that $F_{2 \/ 3}$ is a subset of $F_{2 \/ 3-\\varepsilon}$ which is impossible. Comment. The solution above can be adapted to show that the problem statement still holds, if the condition $r<1$ in (2) is replaced with $r<\\delta$, for an arbitrary positive $\\delta$. This yields that, if $S$ does not satisfy (1), then there exist infinitely many positive rational numbers $r<1$ such that $\\sum_{x \\in F} 1 \/ x \\neq r$ for all finite subsets $F$ of $S$.","tier":0} +{"problem_type":"Algebra","problem_label":"A3","problem":"Given any set $S$ of positive integers, show that at least one of the following two assertions holds: (1) There exist distinct finite subsets $F$ and $G$ of $S$ such that $\\sum_{x \\in F} 1 \/ x=\\sum_{x \\in G} 1 \/ x$; (2) There exists a positive rational number $r<1$ such that $\\sum_{x \\in F} 1 \/ x \\neq r$ for all finite subsets $F$ of $S$. (Luxembourg)","solution":"A finite $S$ clearly satisfies (2), so let $S$ be infinite. If $S$ fails both conditions, so does $S \\backslash\\{1\\}$. We may and will therefore assume that $S$ consists of integers greater than 1 . Label the elements of $S$ increasingly $x_{1}2 x_{n}$ for some $n$, then $\\sum_{x \\in F} 1 \/ x2$, we have $\\Delta_{n} \\leqslant \\frac{n-1}{n} \\Delta_{n-1}$. Proof. Choose positive integers $k, \\ell \\leqslant n$ such that $M_{n}=S(n, k) \/ k$ and $m_{n}=S(n, \\ell) \/ \\ell$. We have $S(n, k)=a_{n-1}+S(n-1, k-1)$, so $$ k\\left(M_{n}-a_{n-1}\\right)=S(n, k)-k a_{n-1}=S(n-1, k-1)-(k-1) a_{n-1} \\leqslant(k-1)\\left(M_{n-1}-a_{n-1}\\right), $$ since $S(n-1, k-1) \\leqslant(k-1) M_{n-1}$. Similarly, we get $$ \\ell\\left(a_{n-1}-m_{n}\\right)=(\\ell-1) a_{n-1}-S(n-1, \\ell-1) \\leqslant(\\ell-1)\\left(a_{n-1}-m_{n-1}\\right) . $$ Since $m_{n-1} \\leqslant a_{n-1} \\leqslant M_{n-1}$ and $k, \\ell \\leqslant n$, the obtained inequalities yield $$ \\begin{array}{ll} M_{n}-a_{n-1} \\leqslant \\frac{k-1}{k}\\left(M_{n-1}-a_{n-1}\\right) \\leqslant \\frac{n-1}{n}\\left(M_{n-1}-a_{n-1}\\right) \\quad \\text { and } \\\\ a_{n-1}-m_{n} \\leqslant \\frac{\\ell-1}{\\ell}\\left(a_{n-1}-m_{n-1}\\right) \\leqslant \\frac{n-1}{n}\\left(a_{n-1}-m_{n-1}\\right) . \\end{array} $$ Therefore, $$ \\Delta_{n}=\\left(M_{n}-a_{n-1}\\right)+\\left(a_{n-1}-m_{n}\\right) \\leqslant \\frac{n-1}{n}\\left(\\left(M_{n-1}-a_{n-1}\\right)+\\left(a_{n-1}-m_{n-1}\\right)\\right)=\\frac{n-1}{n} \\Delta_{n-1} . $$ Back to the problem, if $a_{n}=1$ for all $n \\leqslant 2017$, then $a_{2018} \\leqslant 1$ and hence $a_{2018}-a_{2017} \\leqslant 0$. Otherwise, let $2 \\leqslant q \\leqslant 2017$ be the minimal index with $a_{q}<1$. We have $S(q, i)=i$ for all $i=1,2, \\ldots, q-1$, while $S(q, q)=q-1$. Therefore, $a_{q}<1$ yields $a_{q}=S(q, q) \/ q=1-\\frac{1}{q}$. Now we have $S(q+1, i)=i-\\frac{1}{q}$ for $i=1,2, \\ldots, q$, and $S(q+1, q+1)=q-\\frac{1}{q}$. This gives us $$ m_{q+1}=\\frac{S(q+1,1)}{1}=\\frac{S(q+1, q+1)}{q+1}=\\frac{q-1}{q} \\quad \\text { and } \\quad M_{q+1}=\\frac{S(q+1, q)}{q}=\\frac{q^{2}-1}{q^{2}} $$ so $\\Delta_{q+1}=M_{q+1}-m_{q+1}=(q-1) \/ q^{2}$. Denoting $N=2017 \\geqslant q$ and using Claim 1 for $n=q+2, q+3, \\ldots, N+1$ we finally obtain $$ \\Delta_{N+1} \\leqslant \\frac{q-1}{q^{2}} \\cdot \\frac{q+1}{q+2} \\cdot \\frac{q+2}{q+3} \\cdots \\frac{N}{N+1}=\\frac{1}{N+1}\\left(1-\\frac{1}{q^{2}}\\right) \\leqslant \\frac{1}{N+1}\\left(1-\\frac{1}{N^{2}}\\right)=\\frac{N-1}{N^{2}} $$ as required. Comment 1. One may check that the maximal value of $a_{2018}-a_{2017}$ is attained at the unique sequence, which is presented in the solution above. Comment 2. An easier question would be to determine the maximal value of $\\left|a_{2018}-a_{2017}\\right|$. In this version, the answer $\\frac{1}{2018}$ is achieved at $$ a_{1}=a_{2}=\\cdots=a_{2017}=1, \\quad a_{2018}=\\frac{a_{2017}+\\cdots+a_{0}}{2018}=1-\\frac{1}{2018} . $$ To prove that this value is optimal, it suffices to notice that $\\Delta_{2}=\\frac{1}{2}$ and to apply Claim 1 obtaining $$ \\left|a_{2018}-a_{2017}\\right| \\leqslant \\Delta_{2018} \\leqslant \\frac{1}{2} \\cdot \\frac{2}{3} \\cdots \\frac{2017}{2018}=\\frac{1}{2018} . $$","tier":0} +{"problem_type":"Algebra","problem_label":"A4","problem":"Let $a_{0}, a_{1}, a_{2}, \\ldots$ be a sequence of real numbers such that $a_{0}=0, a_{1}=1$, and for every $n \\geqslant 2$ there exists $1 \\leqslant k \\leqslant n$ satisfying $$ a_{n}=\\frac{a_{n-1}+\\cdots+a_{n-k}}{k} $$ Find the maximal possible value of $a_{2018}-a_{2017}$. (Belgium)","solution":"We present a different proof of the estimate $a_{2018}-a_{2017} \\leqslant \\frac{2016}{2017^{2}}$. We keep the same notations of $S(n, k), m_{n}$ and $M_{n}$ from the previous solution. Notice that $S(n, n)=S(n, n-1)$, as $a_{0}=0$. Also notice that for $0 \\leqslant k \\leqslant \\ell \\leqslant n$ we have $S(n, \\ell)=S(n, k)+S(n-k, \\ell-k)$. Claim 2. For every positive integer $n$, we have $m_{n} \\leqslant m_{n+1}$ and $M_{n+1} \\leqslant M_{n}$, so the segment $\\left[m_{n+1}, M_{n+1}\\right]$ is contained in $\\left[m_{n}, M_{n}\\right]$. Proof. Choose a positive integer $k \\leqslant n+1$ such that $m_{n+1}=S(n+1, k) \/ k$. Then we have $$ k m_{n+1}=S(n+1, k)=a_{n}+S(n, k-1) \\geqslant m_{n}+(k-1) m_{n}=k m_{n}, $$ which establishes the first inequality in the Claim. The proof of the second inequality is similar. Claim 3. For every positive integers $k \\geqslant n$, we have $m_{n} \\leqslant a_{k} \\leqslant M_{n}$. Proof. By Claim 2, we have $\\left[m_{k}, M_{k}\\right] \\subseteq\\left[m_{k-1}, M_{k-1}\\right] \\subseteq \\cdots \\subseteq\\left[m_{n}, M_{n}\\right]$. Since $a_{k} \\in\\left[m_{k}, M_{k}\\right]$, the claim follows. Claim 4. For every integer $n \\geqslant 2$, we have $M_{n}=S(n, n-1) \/(n-1)$ and $m_{n}=S(n, n) \/ n$. Proof. We use induction on $n$. The base case $n=2$ is routine. To perform the induction step, we need to prove the inequalities $$ \\frac{S(n, n)}{n} \\leqslant \\frac{S(n, k)}{k} \\quad \\text { and } \\quad \\frac{S(n, k)}{k} \\leqslant \\frac{S(n, n-1)}{n-1} $$ for every positive integer $k \\leqslant n$. Clearly, these inequalities hold for $k=n$ and $k=n-1$, as $S(n, n)=S(n, n-1)>0$. In the sequel, we assume that $k0$. (South Korea)","solution":"Fix a real number $a>1$, and take a new variable $t$. For the values $f(t), f\\left(t^{2}\\right)$, $f(a t)$ and $f\\left(a^{2} t^{2}\\right)$, the relation (1) provides a system of linear equations: $$ \\begin{array}{llll} x=y=t: & \\left(t+\\frac{1}{t}\\right) f(t) & =f\\left(t^{2}\\right)+f(1) \\\\ x=\\frac{t}{a}, y=a t: & \\left(\\frac{t}{a}+\\frac{a}{t}\\right) f(a t) & =f\\left(t^{2}\\right)+f\\left(a^{2}\\right) \\\\ x=a^{2} t, y=t: & \\left(a^{2} t+\\frac{1}{a^{2} t}\\right) f(t) & =f\\left(a^{2} t^{2}\\right)+f\\left(\\frac{1}{a^{2}}\\right) \\\\ x=y=a t: & \\left(a t+\\frac{1}{a t}\\right) f(a t) & =f\\left(a^{2} t^{2}\\right)+f(1) \\end{array} $$ In order to eliminate $f\\left(t^{2}\\right)$, take the difference of (2a) and (2b); from (2c) and (2d) eliminate $f\\left(a^{2} t^{2}\\right)$; then by taking a linear combination, eliminate $f(a t)$ as well: $$ \\begin{gathered} \\left(t+\\frac{1}{t}\\right) f(t)-\\left(\\frac{t}{a}+\\frac{a}{t}\\right) f(a t)=f(1)-f\\left(a^{2}\\right) \\text { and } \\\\ \\left(a^{2} t+\\frac{1}{a^{2} t}\\right) f(t)-\\left(a t+\\frac{1}{a t}\\right) f(a t)=f\\left(1 \/ a^{2}\\right)-f(1), \\text { so } \\\\ \\left(\\left(a t+\\frac{1}{a t}\\right)\\left(t+\\frac{1}{t}\\right)-\\left(\\frac{t}{a}+\\frac{a}{t}\\right)\\left(a^{2} t+\\frac{1}{a^{2} t}\\right)\\right) f(t) \\\\ =\\left(a t+\\frac{1}{a t}\\right)\\left(f(1)-f\\left(a^{2}\\right)\\right)-\\left(\\frac{t}{a}+\\frac{a}{t}\\right)\\left(f\\left(1 \/ a^{2}\\right)-f(1)\\right) . \\end{gathered} $$ Notice that on the left-hand side, the coefficient of $f(t)$ is nonzero and does not depend on $t$ : $$ \\left(a t+\\frac{1}{a t}\\right)\\left(t+\\frac{1}{t}\\right)-\\left(\\frac{t}{a}+\\frac{a}{t}\\right)\\left(a^{2} t+\\frac{1}{a^{2} t}\\right)=a+\\frac{1}{a}-\\left(a^{3}+\\frac{1}{a^{3}}\\right)<0 . $$ After dividing by this fixed number, we get $$ f(t)=C_{1} t+\\frac{C_{2}}{t} $$ where the numbers $C_{1}$ and $C_{2}$ are expressed in terms of $a, f(1), f\\left(a^{2}\\right)$ and $f\\left(1 \/ a^{2}\\right)$, and they do not depend on $t$. The functions of the form (3) satisfy the equation: $$ \\left(x+\\frac{1}{x}\\right) f(y)=\\left(x+\\frac{1}{x}\\right)\\left(C_{1} y+\\frac{C_{2}}{y}\\right)=\\left(C_{1} x y+\\frac{C_{2}}{x y}\\right)+\\left(C_{1} \\frac{y}{x}+C_{2} \\frac{x}{y}\\right)=f(x y)+f\\left(\\frac{y}{x}\\right) . $$","tier":0} +{"problem_type":"Algebra","problem_label":"A5","problem":"Determine all functions $f:(0, \\infty) \\rightarrow \\mathbb{R}$ satisfying $$ \\left(x+\\frac{1}{x}\\right) f(y)=f(x y)+f\\left(\\frac{y}{x}\\right) $$ for all $x, y>0$. (South Korea)","solution":"We start with an observation. If we substitute $x=a \\neq 1$ and $y=a^{n}$ in (1), we obtain $$ f\\left(a^{n+1}\\right)-\\left(a+\\frac{1}{a}\\right) f\\left(a^{n}\\right)+f\\left(a^{n-1}\\right)=0 . $$ For the sequence $z_{n}=a^{n}$, this is a homogeneous linear recurrence of the second order, and its characteristic polynomial is $t^{2}-\\left(a+\\frac{1}{a}\\right) t+1=(t-a)\\left(t-\\frac{1}{a}\\right)$ with two distinct nonzero roots, namely $a$ and $1 \/ a$. As is well-known, the general solution is $z_{n}=C_{1} a^{n}+C_{2}(1 \/ a)^{n}$ where the index $n$ can be as well positive as negative. Of course, the numbers $C_{1}$ and $C_{2}$ may depend of the choice of $a$, so in fact we have two functions, $C_{1}$ and $C_{2}$, such that $$ f\\left(a^{n}\\right)=C_{1}(a) \\cdot a^{n}+\\frac{C_{2}(a)}{a^{n}} \\quad \\text { for every } a \\neq 1 \\text { and every integer } n \\text {. } $$ The relation (4) can be easily extended to rational values of $n$, so we may conjecture that $C_{1}$ and $C_{2}$ are constants, and whence $f(t)=C_{1} t+\\frac{C_{2}}{t}$. As it was seen in the previous solution, such functions indeed satisfy (1). The equation (1) is linear in $f$; so if some functions $f_{1}$ and $f_{2}$ satisfy (1) and $c_{1}, c_{2}$ are real numbers, then $c_{1} f_{1}(x)+c_{2} f_{2}(x)$ is also a solution of (1). In order to make our formulas simpler, define $$ f_{0}(x)=f(x)-f(1) \\cdot x \\text {. } $$ This function is another one satisfying (1) and the extra constraint $f_{0}(1)=0$. Repeating the same argument on linear recurrences, we can write $f_{0}(a)=K(a) a^{n}+\\frac{L(a)}{a^{n}}$ with some functions $K$ and $L$. By substituting $n=0$, we can see that $K(a)+L(a)=f_{0}(1)=0$ for every $a$. Hence, $$ f_{0}\\left(a^{n}\\right)=K(a)\\left(a^{n}-\\frac{1}{a^{n}}\\right) $$ Now take two numbers $a>b>1$ arbitrarily and substitute $x=(a \/ b)^{n}$ and $y=(a b)^{n}$ in (1): $$ \\begin{aligned} \\left(\\frac{a^{n}}{b^{n}}+\\frac{b^{n}}{a^{n}}\\right) f_{0}\\left((a b)^{n}\\right) & =f_{0}\\left(a^{2 n}\\right)+f_{0}\\left(b^{2 n}\\right), \\quad \\text { so } \\\\ \\left(\\frac{a^{n}}{b^{n}}+\\frac{b^{n}}{a^{n}}\\right) K(a b)\\left((a b)^{n}-\\frac{1}{(a b)^{n}}\\right) & =K(a)\\left(a^{2 n}-\\frac{1}{a^{2 n}}\\right)+K(b)\\left(b^{2 n}-\\frac{1}{b^{2 n}}\\right), \\quad \\text { or equivalently } \\\\ K(a b)\\left(a^{2 n}-\\frac{1}{a^{2 n}}+b^{2 n}-\\frac{1}{b^{2 n}}\\right) & =K(a)\\left(a^{2 n}-\\frac{1}{a^{2 n}}\\right)+K(b)\\left(b^{2 n}-\\frac{1}{b^{2 n}}\\right) . \\end{aligned} $$ By dividing (5) by $a^{2 n}$ and then taking limit with $n \\rightarrow+\\infty$ we get $K(a b)=K(a)$. Then (5) reduces to $K(a)=K(b)$. Hence, $K(a)=K(b)$ for all $a>b>1$. Fix $a>1$. For every $x>0$ there is some $b$ and an integer $n$ such that $10$, at least one of $a_{1}, \\ldots, a_{n}$ is positive; without loss of generality suppose $a_{1} \\geqslant 1$. Consider the polynomials $F_{1}=\\Delta_{1} F$ and $G_{1}=\\Delta G$. On the grid $\\left\\{0, \\ldots, a_{1}-1\\right\\} \\times\\left\\{0, \\ldots, a_{2}\\right\\} \\times$ $\\ldots \\times\\left\\{0, \\ldots, a_{n}\\right\\}$ we have $$ \\begin{aligned} F_{1}\\left(x_{1}, \\ldots, x_{n}\\right) & =F\\left(x_{1}+1, x_{2}, \\ldots, x_{n}\\right)-F\\left(x_{1}, x_{2}, \\ldots, x_{n}\\right)= \\\\ & =G\\left(x_{1}+\\ldots+x_{n}+1\\right)-G\\left(x_{1}+\\ldots+x_{n}\\right)=G_{1}\\left(x_{1}+\\ldots+x_{n}\\right) . \\end{aligned} $$ Since $G$ is nonconstant, we have $\\operatorname{deg} G_{1}=\\operatorname{deg} G-1 \\leqslant\\left(a_{1}-1\\right)+a_{2}+\\ldots+a_{n}$. Therefore we can apply the induction hypothesis to $F_{1}$ and $G_{1}$ and conclude that $F_{1}$ is not the zero polynomial and $\\operatorname{deg} F_{1} \\geqslant \\operatorname{deg} G_{1}$. Hence, $\\operatorname{deg} F \\geqslant \\operatorname{deg} F_{1}+1 \\geqslant \\operatorname{deg} G_{1}+1=\\operatorname{deg} G$. That finishes the proof. To prove the problem statement, take the unique polynomial $g(x)$ so that $g(x)=\\left\\lfloor\\frac{x}{m}\\right\\rfloor$ for $x \\in\\{0,1, \\ldots, n(m-1)\\}$ and $\\operatorname{deg} g \\leqslant n(m-1)$. Notice that precisely $n(m-1)+1$ values of $g$ are prescribed, so $g(x)$ indeed exists and is unique. Notice further that the constraints $g(0)=g(1)=0$ and $g(m)=1$ together enforce $\\operatorname{deg} g \\geqslant 2$. By applying the lemma to $a_{1}=\\ldots=a_{n}=m-1$ and the polynomials $f$ and $g$, we achieve $\\operatorname{deg} f \\geqslant \\operatorname{deg} g$. Hence we just need a suitable lower bound on $\\operatorname{deg} g$. Consider the polynomial $h(x)=g(x+m)-g(x)-1$. The degree of $g(x+m)-g(x)$ is $\\operatorname{deg} g-1 \\geqslant 1$, so $\\operatorname{deg} h=\\operatorname{deg} g-1 \\geqslant 1$, and therefore $h$ cannot be the zero polynomial. On the other hand, $h$ vanishes at the points $0,1, \\ldots, n(m-1)-m$, so $h$ has at least $(n-1)(m-1)$ roots. Hence, $$ \\operatorname{deg} f \\geqslant \\operatorname{deg} g=\\operatorname{deg} h+1 \\geqslant(n-1)(m-1)+1 \\geqslant n $$ Comment 1. In the lemma we have equality for the choice $F\\left(x_{1}, \\ldots, x_{n}\\right)=G\\left(x_{1}+\\ldots+x_{n}\\right)$, so it indeed transforms the problem to an equivalent single-variable question. Comment 2. If $m \\geqslant 3$, the polynomial $h(x)$ can be replaced by $\\Delta g$. Notice that $$ (\\Delta g)(x)= \\begin{cases}1 & \\text { if } x \\equiv-1 \\quad(\\bmod m) \\quad \\text { for } x=0,1, \\ldots, n(m-1)-1 \\\\ 0 & \\text { otherwise }\\end{cases} $$ Hence, $\\Delta g$ vanishes at all integers $x$ with $0 \\leqslant x0$ and $2\\left(k^{2}+1\\right)$, respectively; in this case, the answer becomes $$ 2 \\sqrt[3]{\\frac{(k+1)^{2}}{k}} $$ Even further, a linear substitution allows to extend the solutions to a version with 7 and 100 being replaced with arbitrary positive real numbers $p$ and $q$ satisfying $q \\geqslant 4 p$.","tier":0} +{"problem_type":"Combinatorics","problem_label":"C1","problem":"Let $n \\geqslant 3$ be an integer. Prove that there exists a set $S$ of $2 n$ positive integers satisfying the following property: For every $m=2,3, \\ldots, n$ the set $S$ can be partitioned into two subsets with equal sums of elements, with one of subsets of cardinality $m$. (Iceland)","solution":"We show that one of possible examples is the set $$ S=\\left\\{1 \\cdot 3^{k}, 2 \\cdot 3^{k}: k=1,2, \\ldots, n-1\\right\\} \\cup\\left\\{1, \\frac{3^{n}+9}{2}-1\\right\\} $$ It is readily verified that all the numbers listed above are distinct (notice that the last two are not divisible by 3 ). The sum of elements in $S$ is $$ \\Sigma=1+\\left(\\frac{3^{n}+9}{2}-1\\right)+\\sum_{k=1}^{n-1}\\left(1 \\cdot 3^{k}+2 \\cdot 3^{k}\\right)=\\frac{3^{n}+9}{2}+\\sum_{k=1}^{n-1} 3^{k+1}=\\frac{3^{n}+9}{2}+\\frac{3^{n+1}-9}{2}=2 \\cdot 3^{n} $$ Hence, in order to show that this set satisfies the problem requirements, it suffices to present, for every $m=2,3, \\ldots, n$, an $m$-element subset $A_{m} \\subset S$ whose sum of elements equals $3^{n}$. Such a subset is $$ A_{m}=\\left\\{2 \\cdot 3^{k}: k=n-m+1, n-m+2, \\ldots, n-1\\right\\} \\cup\\left\\{1 \\cdot 3^{n-m+1}\\right\\} . $$ Clearly, $\\left|A_{m}\\right|=m$. The sum of elements in $A_{m}$ is $$ 3^{n-m+1}+\\sum_{k=n-m+1}^{n-1} 2 \\cdot 3^{k}=3^{n-m+1}+\\frac{2 \\cdot 3^{n}-2 \\cdot 3^{n-m+1}}{2}=3^{n} $$ as required. Comment. Let us present a more general construction. Let $s_{1}, s_{2}, \\ldots, s_{2 n-1}$ be a sequence of pairwise distinct positive integers satisfying $s_{2 i+1}=s_{2 i}+s_{2 i-1}$ for all $i=2,3, \\ldots, n-1$. Set $s_{2 n}=s_{1}+s_{2}+$ $\\cdots+s_{2 n-4}$. Assume that $s_{2 n}$ is distinct from the other terms of the sequence. Then the set $S=\\left\\{s_{1}, s_{2}, \\ldots, s_{2 n}\\right\\}$ satisfies the problem requirements. Indeed, the sum of its elements is $$ \\Sigma=\\sum_{i=1}^{2 n-4} s_{i}+\\left(s_{2 n-3}+s_{2 n-2}\\right)+s_{2 n-1}+s_{2 n}=s_{2 n}+s_{2 n-1}+s_{2 n-1}+s_{2 n}=2 s_{2 n}+2 s_{2 n-1} $$ Therefore, we have $$ \\frac{\\Sigma}{2}=s_{2 n}+s_{2 n-1}=s_{2 n}+s_{2 n-2}+s_{2 n-3}=s_{2 n}+s_{2 n-2}+s_{2 n-4}+s_{2 n-5}=\\ldots, $$ which shows that the required sets $A_{m}$ can be chosen as $$ A_{m}=\\left\\{s_{2 n}, s_{2 n-2}, \\ldots, s_{2 n-2 m+4}, s_{2 n-2 m+3}\\right\\} $$ So, the only condition to be satisfied is $s_{2 n} \\notin\\left\\{s_{1}, s_{2}, \\ldots, s_{2 n-1}\\right\\}$, which can be achieved in many different ways (e.g., by choosing properly the number $s_{1}$ after specifying $s_{2}, s_{3}, \\ldots, s_{2 n-1}$ ). The solution above is an instance of this general construction. Another instance, for $n>3$, is the set $$ \\left\\{F_{1}, F_{2}, \\ldots, F_{2 n-1}, F_{1}+\\cdots+F_{2 n-4}\\right\\}, $$ where $F_{1}=1, F_{2}=2, F_{n+1}=F_{n}+F_{n-1}$ is the usual Fibonacci sequence.","tier":0} +{"problem_type":"Combinatorics","problem_label":"C2","problem":"Queenie and Horst play a game on a $20 \\times 20$ chessboard. In the beginning the board is empty. In every turn, Horst places a black knight on an empty square in such a way that his new knight does not attack any previous knights. Then Queenie places a white queen on an empty square. The game gets finished when somebody cannot move. Find the maximal positive $K$ such that, regardless of the strategy of Queenie, Horst can put at least $K$ knights on the board. (Armenia)","solution":"We show two strategies, one for Horst to place at least 100 knights, and another strategy for Queenie that prevents Horst from putting more than 100 knights on the board. A strategy for Horst: Put knights only on black squares, until all black squares get occupied. Colour the squares of the board black and white in the usual way, such that the white and black squares alternate, and let Horst put his knights on black squares as long as it is possible. Two knights on squares of the same colour never attack each other. The number of black squares is $20^{2} \/ 2=200$. The two players occupy the squares in turn, so Horst will surely find empty black squares in his first 100 steps. A strategy for Queenie: Group the squares into cycles of length 4, and after each step of Horst, occupy the opposite square in the same cycle. Consider the squares of the board as vertices of a graph; let two squares be connected if two knights on those squares would attack each other. Notice that in a $4 \\times 4$ board the squares can be grouped into 4 cycles of length 4 , as shown in Figure 1. Divide the board into parts of size $4 \\times 4$, and perform the same grouping in every part; this way we arrange the 400 squares of the board into 100 cycles (Figure 2). ![](https:\/\/cdn.mathpix.com\/cropped\/2024_04_17_6da854f21230b13cd732g-27.jpg?height=360&width=351&top_left_y=1579&top_left_x=310) Figure 1 ![](https:\/\/cdn.mathpix.com\/cropped\/2024_04_17_6da854f21230b13cd732g-27.jpg?height=362&width=420&top_left_y=1578&top_left_x=772) Figure 2 ![](https:\/\/cdn.mathpix.com\/cropped\/2024_04_17_6da854f21230b13cd732g-27.jpg?height=357&width=463&top_left_y=1578&top_left_x=1296) Figure 3 The strategy of Queenie can be as follows: Whenever Horst puts a new knight to a certain square $A$, which is part of some cycle $A-B-C-D-A$, let Queenie put her queen on the opposite square $C$ in that cycle (Figure 3). From this point, Horst cannot put any knight on $A$ or $C$ because those squares are already occupied, neither on $B$ or $D$ because those squares are attacked by the knight standing on $A$. Hence, Horst can put at most one knight on each cycle, that is at most 100 knights in total. Comment 1. Queenie's strategy can be prescribed by a simple rule: divide the board into $4 \\times 4$ parts; whenever Horst puts a knight in a part $P$, Queenie reflects that square about the centre of $P$ and puts her queen on the reflected square. Comment 2. The result remains the same if Queenie moves first. In the first turn, she may put her first queen arbitrarily. Later, if she has to put her next queen on a square that already contains a queen, she may move arbitrarily again.","tier":0} +{"problem_type":"Combinatorics","problem_label":"C3","problem":"Let $n$ be a given positive integer. Sisyphus performs a sequence of turns on a board consisting of $n+1$ squares in a row, numbered 0 to $n$ from left to right. Initially, $n$ stones are put into square 0, and the other squares are empty. At every turn, Sisyphus chooses any nonempty square, say with $k$ stones, takes one of those stones and moves it to the right by at most $k$ squares (the stone should stay within the board). Sisyphus' aim is to move all $n$ stones to square $n$. Prove that Sisyphus cannot reach the aim in less than $$ \\left\\lceil\\frac{n}{1}\\right\\rceil+\\left\\lceil\\frac{n}{2}\\right\\rceil+\\left\\lceil\\frac{n}{3}\\right\\rceil+\\cdots+\\left\\lceil\\frac{n}{n}\\right\\rceil $$ turns. (As usual, $\\lceil x\\rceil$ stands for the least integer not smaller than $x$.) (Netherlands)","solution":"The stones are indistinguishable, and all have the same origin and the same final position. So, at any turn we can prescribe which stone from the chosen square to move. We do it in the following manner. Number the stones from 1 to $n$. At any turn, after choosing a square, Sisyphus moves the stone with the largest number from this square. This way, when stone $k$ is moved from some square, that square contains not more than $k$ stones (since all their numbers are at most $k$ ). Therefore, stone $k$ is moved by at most $k$ squares at each turn. Since the total shift of the stone is exactly $n$, at least $\\lceil n \/ k\\rceil$ moves of stone $k$ should have been made, for every $k=1,2, \\ldots, n$. By summing up over all $k=1,2, \\ldots, n$, we get the required estimate. Comment. The original submission contained the second part, asking for which values of $n$ the equality can be achieved. The answer is $n=1,2,3,4,5,7$. The Problem Selection Committee considered this part to be less suitable for the competition, due to technicalities.","tier":0} +{"problem_type":"Combinatorics","problem_label":"C4","problem":"An anti-Pascal pyramid is a finite set of numbers, placed in a triangle-shaped array so that the first row of the array contains one number, the second row contains two numbers, the third row contains three numbers and so on; and, except for the numbers in the bottom row, each number equals the absolute value of the difference of the two numbers below it. For instance, the triangle below is an anti-Pascal pyramid with four rows, in which every integer from 1 to $1+2+3+4=10$ occurs exactly once: $$ \\begin{array}{cccc} & & 4 \\\\ & 2 & 6 & \\\\ & 5 \\quad 7 \\quad 1 \\\\ 8 & 3 & 10 & 9 . \\end{array} $$ Is it possible to form an anti-Pascal pyramid with 2018 rows, using every integer from 1 to $1+2+\\cdots+2018$ exactly once? (Iran)","solution":"Let $T$ be an anti-Pascal pyramid with $n$ rows, containing every integer from 1 to $1+2+\\cdots+n$, and let $a_{1}$ be the topmost number in $T$ (Figure 1). The two numbers below $a_{1}$ are some $a_{2}$ and $b_{2}=a_{1}+a_{2}$, the two numbers below $b_{2}$ are some $a_{3}$ and $b_{3}=a_{1}+a_{2}+a_{3}$, and so on and so forth all the way down to the bottom row, where some $a_{n}$ and $b_{n}=a_{1}+a_{2}+\\cdots+a_{n}$ are the two neighbours below $b_{n-1}=a_{1}+a_{2}+\\cdots+a_{n-1}$. Since the $a_{k}$ are $n$ pairwise distinct positive integers whose sum does not exceed the largest number in $T$, which is $1+2+\\cdots+n$, it follows that they form a permutation of $1,2, \\ldots, n$. ![](https:\/\/cdn.mathpix.com\/cropped\/2024_04_17_6da854f21230b13cd732g-29.jpg?height=391&width=465&top_left_y=1415&top_left_x=430) Figure 1 ![](https:\/\/cdn.mathpix.com\/cropped\/2024_04_17_6da854f21230b13cd732g-29.jpg?height=394&width=463&top_left_y=1411&top_left_x=1162) Figure 2 Consider now (Figure 2) the two 'equilateral' subtriangles of $T$ whose bottom rows contain the numbers to the left, respectively right, of the pair $a_{n}, b_{n}$. (One of these subtriangles may very well be empty.) At least one of these subtriangles, say $T^{\\prime}$, has side length $\\ell \\geqslant\\lceil(n-2) \/ 2\\rceil$. Since $T^{\\prime}$ obeys the anti-Pascal rule, it contains $\\ell$ pairwise distinct positive integers $a_{1}^{\\prime}, a_{2}^{\\prime}, \\ldots, a_{\\ell}^{\\prime}$, where $a_{1}^{\\prime}$ is at the apex, and $a_{k}^{\\prime}$ and $b_{k}^{\\prime}=a_{1}^{\\prime}+a_{2}^{\\prime}+\\cdots+a_{k}^{\\prime}$ are the two neighbours below $b_{k-1}^{\\prime}$ for each $k=2,3 \\ldots, \\ell$. Since the $a_{k}$ all lie outside $T^{\\prime}$, and they form a permutation of $1,2, \\ldots, n$, the $a_{k}^{\\prime}$ are all greater than $n$. Consequently, $$ \\begin{array}{r} b_{\\ell}^{\\prime} \\geqslant(n+1)+(n+2)+\\cdots+(n+\\ell)=\\frac{\\ell(2 n+\\ell+1)}{2} \\\\ \\geqslant \\frac{1}{2} \\cdot \\frac{n-2}{2}\\left(2 n+\\frac{n-2}{2}+1\\right)=\\frac{5 n(n-2)}{8}, \\end{array} $$ which is greater than $1+2+\\cdots+n=n(n+1) \/ 2$ for $n=2018$. A contradiction. Comment. The above estimate may be slightly improved by noticing that $b_{\\ell}^{\\prime} \\neq b_{n}$. This implies $n(n+1) \/ 2=b_{n}>b_{\\ell}^{\\prime} \\geqslant\\lceil(n-2) \/ 2\\rceil(2 n+\\lceil(n-2) \/ 2\\rceil+1) \/ 2$, so $n \\leqslant 7$ if $n$ is odd, and $n \\leqslant 12$ if $n$ is even. It seems that the largest anti-Pascal pyramid whose entries are a permutation of the integers from 1 to $1+2+\\cdots+n$ has 5 rows.","tier":0} +{"problem_type":"Combinatorics","problem_label":"C5","problem":"Let $k$ be a positive integer. The organising committee of a tennis tournament is to schedule the matches for $2 k$ players so that every two players play once, each day exactly one match is played, and each player arrives to the tournament site the day of his first match, and departs the day of his last match. For every day a player is present on the tournament, the committee has to pay 1 coin to the hotel. The organisers want to design the schedule so as to minimise the total cost of all players' stays. Determine this minimum cost. (Russia)","solution":"Enumerate the days of the tournament $1,2, \\ldots,\\left(\\begin{array}{c}2 k \\\\ 2\\end{array}\\right)$. Let $b_{1} \\leqslant b_{2} \\leqslant \\cdots \\leqslant b_{2 k}$ be the days the players arrive to the tournament, arranged in nondecreasing order; similarly, let $e_{1} \\geqslant \\cdots \\geqslant e_{2 k}$ be the days they depart arranged in nonincreasing order (it may happen that a player arrives on day $b_{i}$ and departs on day $e_{j}$, where $i \\neq j$ ). If a player arrives on day $b$ and departs on day $e$, then his stay cost is $e-b+1$. Therefore, the total stay cost is $$ \\Sigma=\\sum_{i=1}^{2 k} e_{i}-\\sum_{i=1}^{2 k} b_{i}+n=\\sum_{i=1}^{2 k}\\left(e_{i}-b_{i}+1\\right) $$ Bounding the total cost from below. To this end, estimate $e_{i+1}-b_{i+1}+1$. Before day $b_{i+1}$, only $i$ players were present, so at most $\\left(\\begin{array}{l}i \\\\ 2\\end{array}\\right)$ matches could be played. Therefore, $b_{i+1} \\leqslant\\left(\\begin{array}{l}i \\\\ 2\\end{array}\\right)+1$. Similarly, at most $\\left(\\begin{array}{l}i \\\\ 2\\end{array}\\right)$ matches could be played after day $e_{i+1}$, so $e_{i} \\geqslant\\left(\\begin{array}{c}2 k \\\\ 2\\end{array}\\right)-\\left(\\begin{array}{l}i \\\\ 2\\end{array}\\right)$. Thus, $$ e_{i+1}-b_{i+1}+1 \\geqslant\\left(\\begin{array}{c} 2 k \\\\ 2 \\end{array}\\right)-2\\left(\\begin{array}{l} i \\\\ 2 \\end{array}\\right)=k(2 k-1)-i(i-1) $$ This lower bound can be improved for $i>k$ : List the $i$ players who arrived first, and the $i$ players who departed last; at least $2 i-2 k$ players appear in both lists. The matches between these players were counted twice, though the players in each pair have played only once. Therefore, if $i>k$, then $$ e_{i+1}-b_{i+1}+1 \\geqslant\\left(\\begin{array}{c} 2 k \\\\ 2 \\end{array}\\right)-2\\left(\\begin{array}{c} i \\\\ 2 \\end{array}\\right)+\\left(\\begin{array}{c} 2 i-2 k \\\\ 2 \\end{array}\\right)=(2 k-i)^{2} $$ An optimal tournament, We now describe a schedule in which the lower bounds above are all achieved simultaneously. Split players into two groups $X$ and $Y$, each of cardinality $k$. Next, partition the schedule into three parts. During the first part, the players from $X$ arrive one by one, and each newly arrived player immediately plays with everyone already present. During the third part (after all players from $X$ have already departed) the players from $Y$ depart one by one, each playing with everyone still present just before departing. In the middle part, everyone from $X$ should play with everyone from $Y$. Let $S_{1}, S_{2}, \\ldots, S_{k}$ be the players in $X$, and let $T_{1}, T_{2}, \\ldots, T_{k}$ be the players in $Y$. Let $T_{1}, T_{2}, \\ldots, T_{k}$ arrive in this order; after $T_{j}$ arrives, he immediately plays with all the $S_{i}, i>j$. Afterwards, players $S_{k}$, $S_{k-1}, \\ldots, S_{1}$ depart in this order; each $S_{i}$ plays with all the $T_{j}, i \\leqslant j$, just before his departure, and $S_{k}$ departs the day $T_{k}$ arrives. For $0 \\leqslant s \\leqslant k-1$, the number of matches played between $T_{k-s}$ 's arrival and $S_{k-s}$ 's departure is $$ \\sum_{j=k-s}^{k-1}(k-j)+1+\\sum_{j=k-s}^{k-1}(k-j+1)=\\frac{1}{2} s(s+1)+1+\\frac{1}{2} s(s+3)=(s+1)^{2} $$ Thus, if $i>k$, then the number of matches that have been played between $T_{i-k+1}$ 's arrival, which is $b_{i+1}$, and $S_{i-k+1}$ 's departure, which is $e_{i+1}$, is $(2 k-i)^{2}$; that is, $e_{i+1}-b_{i+1}+1=(2 k-i)^{2}$, showing the second lower bound achieved for all $i>k$. If $i \\leqslant k$, then the matches between the $i$ players present before $b_{i+1}$ all fall in the first part of the schedule, so there are $\\left(\\begin{array}{l}i \\\\ 2\\end{array}\\right)$ such, and $b_{i+1}=\\left(\\begin{array}{l}i \\\\ 2\\end{array}\\right)+1$. Similarly, after $e_{i+1}$, there are $i$ players left, all $\\left(\\begin{array}{l}i \\\\ 2\\end{array}\\right)$ matches now fall in the third part of the schedule, and $e_{i+1}=\\left(\\begin{array}{c}2 k \\\\ 2\\end{array}\\right)-\\left(\\begin{array}{c}i \\\\ 2\\end{array}\\right)$. The first lower bound is therefore also achieved for all $i \\leqslant k$. Consequently, all lower bounds are achieved simultaneously, and the schedule is indeed optimal. Evaluation. Finally, evaluate the total cost for the optimal schedule: $$ \\begin{aligned} \\Sigma & =\\sum_{i=0}^{k}(k(2 k-1)-i(i-1))+\\sum_{i=k+1}^{2 k-1}(2 k-i)^{2}=(k+1) k(2 k-1)-\\sum_{i=0}^{k} i(i-1)+\\sum_{j=1}^{k-1} j^{2} \\\\ & =k(k+1)(2 k-1)-k^{2}+\\frac{1}{2} k(k+1)=\\frac{1}{2} k\\left(4 k^{2}+k-1\\right) . \\end{aligned} $$","tier":0} +{"problem_type":"Combinatorics","problem_label":"C5","problem":"Let $k$ be a positive integer. The organising committee of a tennis tournament is to schedule the matches for $2 k$ players so that every two players play once, each day exactly one match is played, and each player arrives to the tournament site the day of his first match, and departs the day of his last match. For every day a player is present on the tournament, the committee has to pay 1 coin to the hotel. The organisers want to design the schedule so as to minimise the total cost of all players' stays. Determine this minimum cost. (Russia)","solution":"Consider any tournament schedule. Label players $P_{1}, P_{2}, \\ldots, P_{2 k}$ in order of their arrival, and label them again $Q_{2 k}, Q_{2 k-1}, \\ldots, Q_{1}$ in order of their departure, to define a permutation $a_{1}, a_{2}, \\ldots, a_{2 k}$ of $1,2, \\ldots, 2 k$ by $P_{i}=Q_{a_{i}}$. We first describe an optimal tournament for any given permutation $a_{1}, a_{2}, \\ldots, a_{2 k}$ of the indices $1,2, \\ldots, 2 k$. Next, we find an optimal permutation and an optimal tournament. Optimisation for a fixed $a_{1}, \\ldots, a_{2 k}$. We say that the cost of the match between $P_{i}$ and $P_{j}$ is the number of players present at the tournament when this match is played. Clearly, the Committee pays for each day the cost of the match of that day. Hence, we are to minimise the total cost of all matches. Notice that $Q_{2 k}$ 's departure does not precede $P_{2 k}$ 's arrival. Hence, the number of players at the tournament monotonically increases (non-strictly) until it reaches $2 k$, and then monotonically decreases (non-strictly). So, the best time to schedule the match between $P_{i}$ and $P_{j}$ is either when $P_{\\max (i, j)}$ arrives, or when $Q_{\\max \\left(a_{i}, a_{j}\\right)}$ departs, in which case the cost is $\\min \\left(\\max (i, j), \\max \\left(a_{i}, a_{j}\\right)\\right)$. Conversely, assuming that $i>j$, if this match is scheduled between the arrivals of $P_{i}$ and $P_{i+1}$, then its cost will be exactly $i=\\max (i, j)$. Similarly, one can make it cost $\\max \\left(a_{i}, a_{j}\\right)$. Obviously, these conditions can all be simultaneously satisfied, so the minimal cost for a fixed sequence $a_{1}, a_{2}, \\ldots, a_{2 k}$ is $$ \\Sigma\\left(a_{1}, \\ldots, a_{2 k}\\right)=\\sum_{1 \\leqslant ia b-a-b$ is expressible in the form $x=s a+t b$, with integer $s, t \\geqslant 0$. We will prove by induction on $s+t$ that if $x=s a+b t$, with $s, t$ nonnegative integers, then $$ f(x)>\\frac{f(0)}{2^{s+t}}-2 . $$ The base case $s+t=0$ is trivial. Assume now that (3) is true for $s+t=v$. Then, if $s+t=v+1$ and $x=s a+t b$, at least one of the numbers $s$ and $t$ - say $s$ - is positive, hence by (2), $$ f(x)=f(s a+t b) \\geqslant \\frac{f((s-1) a+t b)}{2}-1>\\frac{1}{2}\\left(\\frac{f(0)}{2^{s+t-1}}-2\\right)-1=\\frac{f(0)}{2^{s+t}}-2 . $$ Assume now that we must perform moves of type (ii) ad infinitum. Take $n=a b-a-b$ and suppose $b>a$. Since each of the numbers $n+1, n+2, \\ldots, n+b$ can be expressed in the form $s a+t b$, with $0 \\leqslant s \\leqslant b$ and $0 \\leqslant t \\leqslant a$, after moves of type (ii) have been performed $2^{a+b+1}$ times and we have to add a new pair of zeros, each $f(n+k), k=1,2, \\ldots, b$, is at least 2 . In this case (1) yields inductively $f(n+k) \\geqslant 2$ for all $k \\geqslant 1$. But this is absurd: after a finite number of moves, $f$ cannot attain nonzero values at infinitely many points.","tier":0} +{"problem_type":"Combinatorics","problem_label":"C6","problem":"Let $a$ and $b$ be distinct positive integers. The following infinite process takes place on an initially empty board. (i) If there is at least a pair of equal numbers on the board, we choose such a pair and increase one of its components by $a$ and the other by $b$. (ii) If no such pair exists, we write down two times the number 0 . Prove that, no matter how we make the choices in $(i)$, operation (ii) will be performed only finitely many times. (Serbia)","solution":"We start by showing that the result of the process in the problem does not depend on the way the operations are performed. For that purpose, it is convenient to modify the process a bit. Claim 1. Suppose that the board initially contains a finite number of nonnegative integers, and one starts performing type $(i)$ moves only. Assume that one had applied $k$ moves which led to a final arrangement where no more type (i) moves are possible. Then, if one starts from the same initial arrangement, performing type (i) moves in an arbitrary fashion, then the process will necessarily stop at the same final arrangement Proof. Throughout this proof, all moves are supposed to be of type (i). Induct on $k$; the base case $k=0$ is trivial, since no moves are possible. Assume now that $k \\geqslant 1$. Fix some canonical process, consisting of $k$ moves $M_{1}, M_{2}, \\ldots, M_{k}$, and reaching the final arrangement $A$. Consider any sample process $m_{1}, m_{2}, \\ldots$ starting with the same initial arrangement and proceeding as long as possible; clearly, it contains at least one move. We need to show that this process stops at $A$. Let move $m_{1}$ consist in replacing two copies of $x$ with $x+a$ and $x+b$. If move $M_{1}$ does the same, we may apply the induction hypothesis to the arrangement appearing after $m_{1}$. Otherwise, the canonical process should still contain at least one move consisting in replacing $(x, x) \\mapsto(x+a, x+b)$, because the initial arrangement contains at least two copies of $x$, while the final one contains at most one such. Let $M_{i}$ be the first such move. Since the copies of $x$ are indistinguishable and no other copy of $x$ disappeared before $M_{i}$ in the canonical process, the moves in this process can be permuted as $M_{i}, M_{1}, \\ldots, M_{i-1}, M_{i+1}, \\ldots, M_{k}$, without affecting the final arrangement. Now it suffices to perform the move $m_{1}=M_{i}$ and apply the induction hypothesis as above. Claim 2. Consider any process starting from the empty board, which involved exactly $n$ moves of type (ii) and led to a final arrangement where all the numbers are distinct. Assume that one starts with the board containing $2 n$ zeroes (as if $n$ moves of type (ii) were made in the beginning), applying type ( $i$ ) moves in an arbitrary way. Then this process will reach the same final arrangement. Proof. Starting with the board with $2 n$ zeros, one may indeed model the first process mentioned in the statement of the claim, omitting the type (ii) moves. This way, one reaches the same final arrangement. Now, Claim 1 yields that this final arrangement will be obtained when type (i) moves are applied arbitrarily. Claim 2 allows now to reformulate the problem statement as follows: There exists an integer $n$ such that, starting from $2 n$ zeroes, one may apply type (i) moves indefinitely. In order to prove this, we start with an obvious induction on $s+t=k \\geqslant 1$ to show that if we start with $2^{s+t}$ zeros, then we can get simultaneously on the board, at some point, each of the numbers $s a+t b$, with $s+t=k$. Suppose now that $a4$, one can choose a point $A_{n+1}$ on the small arc $\\widehat{C A_{n}}$, close enough to $C$, so that $A_{n} B+A_{n} A_{n+1}+B A_{n+1}$ is still greater than 4 . Thus each of these new triangles $B A_{n} A_{n+1}$ and $B A_{n+1} C$ has perimeter greater than 4, which completes the induction step. ![](https:\/\/cdn.mathpix.com\/cropped\/2024_04_17_6da854f21230b13cd732g-42.jpg?height=366&width=691&top_left_y=1353&top_left_x=682) We proceed by showing that no $t>4$ satisfies the conditions of the problem. To this end, we assume that there exists a good collection $T$ of $n$ triangles, each of perimeter greater than $t$, and then bound $n$ from above. Take $\\varepsilon>0$ such that $t=4+2 \\varepsilon$. Claim. There exists a positive constant $\\sigma=\\sigma(\\varepsilon)$ such that any triangle $\\Delta$ with perimeter $2 s \\geqslant 4+2 \\varepsilon$, inscribed in $\\omega$, has area $S(\\Delta)$ at least $\\sigma$. Proof. Let $a, b, c$ be the side lengths of $\\Delta$. Since $\\Delta$ is inscribed in $\\omega$, each side has length at most 2. Therefore, $s-a \\geqslant(2+\\varepsilon)-2=\\varepsilon$. Similarly, $s-b \\geqslant \\varepsilon$ and $s-c \\geqslant \\varepsilon$. By Heron's formula, $S(\\Delta)=\\sqrt{s(s-a)(s-b)(s-c)} \\geqslant \\sqrt{(2+\\varepsilon) \\varepsilon^{3}}$. Thus we can set $\\sigma(\\varepsilon)=\\sqrt{(2+\\varepsilon) \\varepsilon^{3}}$. Now we see that the total area $S$ of all triangles from $T$ is at least $n \\sigma(\\varepsilon)$. On the other hand, $S$ does not exceed the area of the disk bounded by $\\omega$. Thus $n \\sigma(\\varepsilon) \\leqslant \\pi$, which means that $n$ is bounded from above. Comment 1. One may prove the Claim using the formula $S=\\frac{a b c}{4 R}$ instead of Heron's formula. Comment 2. In the statement of the problem condition $(i)$ could be replaced by a weaker one: each triangle from $T$ lies within $\\omega$. This does not affect the solution above, but reduces the number of ways to prove the Claim. This page is intentionally left blank","tier":0} +{"problem_type":"Geometry","problem_label":"G4","problem":"A point $T$ is chosen inside a triangle $A B C$. Let $A_{1}, B_{1}$, and $C_{1}$ be the reflections of $T$ in $B C, C A$, and $A B$, respectively. Let $\\Omega$ be the circumcircle of the triangle $A_{1} B_{1} C_{1}$. The lines $A_{1} T, B_{1} T$, and $C_{1} T$ meet $\\Omega$ again at $A_{2}, B_{2}$, and $C_{2}$, respectively. Prove that the lines $A A_{2}, B B_{2}$, and $C C_{2}$ are concurrent on $\\Omega$. (Mongolia)","solution":"By $\\Varangle(\\ell, n)$ we always mean the directed angle of the lines $\\ell$ and $n$, taken modulo $180^{\\circ}$. Let $C C_{2}$ meet $\\Omega$ again at $K$ (as usual, if $C C_{2}$ is tangent to $\\Omega$, we set $T=C_{2}$ ). We show that the line $B B_{2}$ contains $K$; similarly, $A A_{2}$ will also pass through $K$. For this purpose, it suffices to prove that $$ \\Varangle\\left(C_{2} C, C_{2} A_{1}\\right)=\\Varangle\\left(B_{2} B, B_{2} A_{1}\\right) . $$ By the problem condition, $C B$ and $C A$ are the perpendicular bisectors of $T A_{1}$ and $T B_{1}$, respectively. Hence, $C$ is the circumcentre of the triangle $A_{1} T B_{1}$. Therefore, $$ \\Varangle\\left(C A_{1}, C B\\right)=\\Varangle(C B, C T)=\\Varangle\\left(B_{1} A_{1}, B_{1} T\\right)=\\Varangle\\left(B_{1} A_{1}, B_{1} B_{2}\\right) . $$ In circle $\\Omega$ we have $\\Varangle\\left(B_{1} A_{1}, B_{1} B_{2}\\right)=\\Varangle\\left(C_{2} A_{1}, C_{2} B_{2}\\right)$. Thus, $$ \\Varangle\\left(C A_{1}, C B\\right)=\\Varangle\\left(B_{1} A_{1}, B_{1} B_{2}\\right)=\\Varangle\\left(C_{2} A_{1}, C_{2} B_{2}\\right) . $$ Similarly, we get $$ \\Varangle\\left(B A_{1}, B C\\right)=\\Varangle\\left(C_{1} A_{1}, C_{1} C_{2}\\right)=\\Varangle\\left(B_{2} A_{1}, B_{2} C_{2}\\right) . $$ The two obtained relations yield that the triangles $A_{1} B C$ and $A_{1} B_{2} C_{2}$ are similar and equioriented, hence $$ \\frac{A_{1} B_{2}}{A_{1} B}=\\frac{A_{1} C_{2}}{A_{1} C} \\quad \\text { and } \\quad \\Varangle\\left(A_{1} B, A_{1} C\\right)=\\Varangle\\left(A_{1} B_{2}, A_{1} C_{2}\\right) $$ The second equality may be rewritten as $\\Varangle\\left(A_{1} B, A_{1} B_{2}\\right)=\\Varangle\\left(A_{1} C, A_{1} C_{2}\\right)$, so the triangles $A_{1} B B_{2}$ and $A_{1} C C_{2}$ are also similar and equioriented. This establishes (1). ![](https:\/\/cdn.mathpix.com\/cropped\/2024_04_17_6da854f21230b13cd732g-44.jpg?height=839&width=1171&top_left_y=1782&top_left_x=448) Comment 1. In fact, the triangle $A_{1} B C$ is an image of $A_{1} B_{2} C_{2}$ under a spiral similarity centred at $A_{1}$; in this case, the triangles $A B B_{2}$ and $A C C_{2}$ are also spirally similar with the same centre. Comment 2. After obtaining (2) and (3), one can finish the solution in different ways. For instance, introducing the point $X=B C \\cap B_{2} C_{2}$, one gets from these relations that the 4-tuples $\\left(A_{1}, B, B_{2}, X\\right)$ and $\\left(A_{1}, C, C_{2}, X\\right)$ are both cyclic. Therefore, $K$ is the Miquel point of the lines $B B_{2}$, $C C_{2}, B C$, and $B_{2} C_{2}$; this yields that the meeting point of $B B_{2}$ and $C C_{2}$ lies on $\\Omega$. Yet another way is to show that the points $A_{1}, B, C$, and $K$ are concyclic, as $$ \\Varangle\\left(K C, K A_{1}\\right)=\\Varangle\\left(B_{2} C_{2}, B_{2} A_{1}\\right)=\\Varangle\\left(B C, B A_{1}\\right) . $$ By symmetry, the second point $K^{\\prime}$ of intersection of $B B_{2}$ with $\\Omega$ is also concyclic to $A_{1}, B$, and $C$, hence $K^{\\prime}=K$. ![](https:\/\/cdn.mathpix.com\/cropped\/2024_04_17_6da854f21230b13cd732g-45.jpg?height=877&width=1188&top_left_y=681&top_left_x=434) Comment 3. The requirement that the common point of the lines $A A_{2}, B B_{2}$, and $C C_{2}$ should lie on $\\Omega$ may seem to make the problem easier, since it suggests some approaches. On the other hand, there are also different ways of showing that the lines $A A_{2}, B B_{2}$, and $C C_{2}$ are just concurrent. In particular, the problem conditions yield that the lines $A_{2} T, B_{2} T$, and $C_{2} T$ are perpendicular to the corresponding sides of the triangle $A B C$. One may show that the lines $A T, B T$, and $C T$ are also perpendicular to the corresponding sides of the triangle $A_{2} B_{2} C_{2}$, i.e., the triangles $A B C$ and $A_{2} B_{2} C_{2}$ are orthologic, and their orthology centres coincide. It is known that such triangles are also perspective, i.e. the lines $A A_{2}, B B_{2}$, and $C C_{2}$ are concurrent (in projective sense). To show this mutual orthology, one may again apply angle chasing, but there are also other methods. Let $A^{\\prime}, B^{\\prime}$, and $C^{\\prime}$ be the projections of $T$ onto the sides of the triangle $A B C$. Then $A_{2} T \\cdot T A^{\\prime}=$ $B_{2} T \\cdot T B^{\\prime}=C_{2} T \\cdot T C^{\\prime}$, since all three products equal (minus) half the power of $T$ with respect to $\\Omega$. This means that $A_{2}, B_{2}$, and $C_{2}$ are the poles of the sidelines of the triangle $A B C$ with respect to some circle centred at $T$ and having pure imaginary radius (in other words, the reflections of $A_{2}, B_{2}$, and $C_{2}$ in $T$ are the poles of those sidelines with respect to some regular circle centred at $T$ ). Hence, dually, the vertices of the triangle $A B C$ are also the poles of the sidelines of the triangle $A_{2} B_{2} C_{2}$.","tier":0} +{"problem_type":"Geometry","problem_label":"G5","problem":"Let $A B C$ be a triangle with circumcircle $\\omega$ and incentre $I$. A line $\\ell$ intersects the lines $A I, B I$, and $C I$ at points $D, E$, and $F$, respectively, distinct from the points $A, B, C$, and $I$. The perpendicular bisectors $x, y$, and $z$ of the segments $A D, B E$, and $C F$, respectively determine a triangle $\\Theta$. Show that the circumcircle of the triangle $\\Theta$ is tangent to $\\omega$. (Denmark)","solution":"Claim 1. The reflections $\\ell_{a}, \\ell_{b}$ and $\\ell_{c}$ of the line $\\ell$ in the lines $x, y$, and $z$, respectively, are concurrent at a point $T$ which belongs to $\\omega$. ![](https:\/\/cdn.mathpix.com\/cropped\/2024_04_17_6da854f21230b13cd732g-46.jpg?height=831&width=916&top_left_y=1338&top_left_x=570) Proof. Notice that $\\Varangle\\left(\\ell_{b}, \\ell_{c}\\right)=\\Varangle\\left(\\ell_{b}, \\ell\\right)+\\Varangle\\left(\\ell, \\ell_{c}\\right)=2 \\Varangle(y, \\ell)+2 \\Varangle(\\ell, z)=2 \\Varangle(y, z)$. But $y \\perp B I$ and $z \\perp C I$ implies $\\Varangle(y, z)=\\Varangle(B I, I C)$, so, since $2 \\Varangle(B I, I C)=\\Varangle(B A, A C)$, we obtain $$ \\Varangle\\left(\\ell_{b}, \\ell_{c}\\right)=\\Varangle(B A, A C) . $$ Since $A$ is the reflection of $D$ in $x$, $A$ belongs to $\\ell_{a}$; similarly, $B$ belongs to $\\ell_{b}$. Then (1) shows that the common point $T^{\\prime}$ of $\\ell_{a}$ and $\\ell_{b}$ lies on $\\omega$; similarly, the common point $T^{\\prime \\prime}$ of $\\ell_{c}$ and $\\ell_{b}$ lies on $\\omega$. If $B \\notin \\ell_{a}$ and $B \\notin \\ell_{c}$, then $T^{\\prime}$ and $T^{\\prime \\prime}$ are the second point of intersection of $\\ell_{b}$ and $\\omega$, hence they coincide. Otherwise, if, say, $B \\in \\ell_{c}$, then $\\ell_{c}=B C$, so $\\Varangle(B A, A C)=\\Varangle\\left(\\ell_{b}, \\ell_{c}\\right)=\\Varangle\\left(\\ell_{b}, B C\\right)$, which shows that $\\ell_{b}$ is tangent at $B$ to $\\omega$ and $T^{\\prime}=T^{\\prime \\prime}=B$. So $T^{\\prime}$ and $T^{\\prime \\prime}$ coincide in all the cases, and the conclusion of the claim follows. Now we prove that $X, X_{0}, T$ are collinear. Denote by $D_{b}$ and $D_{c}$ the reflections of the point $D$ in the lines $y$ and $z$, respectively. Then $D_{b}$ lies on $\\ell_{b}, D_{c}$ lies on $\\ell_{c}$, and $$ \\begin{aligned} \\Varangle\\left(D_{b} X, X D_{c}\\right) & =\\Varangle\\left(D_{b} X, D X\\right)+\\Varangle\\left(D X, X D_{c}\\right)=2 \\Varangle(y, D X)+2 \\Varangle(D X, z)=2 \\Varangle(y, z) \\\\ & =\\Varangle(B A, A C)=\\Varangle(B T, T C), \\end{aligned} $$ hence the quadrilateral $X D_{b} T D_{c}$ is cyclic. Notice also that since $X D_{b}=X D=X D_{c}$, the points $D, D_{b}, D_{c}$ lie on a circle with centre $X$. Using in this circle the diameter $D_{c} D_{c}^{\\prime}$ yields $\\Varangle\\left(D_{b} D_{c}, D_{c} X\\right)=90^{\\circ}+\\Varangle\\left(D_{b} D_{c}^{\\prime}, D_{c}^{\\prime} X\\right)=90^{\\circ}+\\Varangle\\left(D_{b} D, D D_{c}\\right)$. Therefore, $$ \\begin{gathered} \\Varangle\\left(\\ell_{b}, X T\\right)=\\Varangle\\left(D_{b} T, X T\\right)=\\Varangle\\left(D_{b} D_{c}, D_{c} X\\right)=90^{\\circ}+\\Varangle\\left(D_{b} D, D D_{c}\\right) \\\\ =90^{\\circ}+\\Varangle(B I, I C)=\\Varangle(B A, A I)=\\Varangle\\left(B A, A X_{0}\\right)=\\Varangle\\left(B T, T X_{0}\\right)=\\Varangle\\left(\\ell_{b}, X_{0} T\\right) \\end{gathered} $$ so the points $X, X_{0}, T$ are collinear. By a similar argument, $Y, Y_{0}, T$ and $Z, Z_{0}, T$ are collinear. As mentioned in the preamble, the statement of the problem follows. Comment 1. After proving Claim 1 one may proceed in another way. As it was shown, the reflections of $\\ell$ in the sidelines of $X Y Z$ are concurrent at $T$. Thus $\\ell$ is the Steiner line of $T$ with respect to $\\triangle X Y Z$ (that is the line containing the reflections $T_{a}, T_{b}, T_{c}$ of $T$ in the sidelines of $X Y Z$ ). The properties of the Steiner line imply that $T$ lies on $\\Omega$, and $\\ell$ passes through the orthocentre $H$ of the triangle $X Y Z$. ![](https:\/\/cdn.mathpix.com\/cropped\/2024_04_17_6da854f21230b13cd732g-47.jpg?height=811&width=1311&top_left_y=1248&top_left_x=378) Let $H_{a}, H_{b}$, and $H_{c}$ be the reflections of the point $H$ in the lines $x, y$, and $z$, respectively. Then the triangle $H_{a} H_{b} H_{c}$ is inscribed in $\\Omega$ and homothetic to $A B C$ (by an easy angle chasing). Since $H_{a} \\in \\ell_{a}, H_{b} \\in \\ell_{b}$, and $H_{c} \\in \\ell_{c}$, the triangles $H_{a} H_{b} H_{c}$ and $A B C$ form a required pair of triangles $\\Delta$ and $\\delta$ mentioned in the preamble. Comment 2. The following observation shows how one may guess the description of the tangency point $T$ from Solution 1. Let us fix a direction and move the line $\\ell$ parallel to this direction with constant speed. Then the points $D, E$, and $F$ are moving with constant speeds along the lines $A I, B I$, and $C I$, respectively. In this case $x, y$, and $z$ are moving with constant speeds, defining a family of homothetic triangles $X Y Z$ with a common centre of homothety $T$. Notice that the triangle $X_{0} Y_{0} Z_{0}$ belongs to this family (for $\\ell$ passing through $I$ ). We may specify the location of $T$ considering the degenerate case when $x, y$, and $z$ are concurrent. In this degenerate case all the lines $x, y, z, \\ell, \\ell_{a}, \\ell_{b}, \\ell_{c}$ have a common point. Note that the lines $\\ell_{a}, \\ell_{b}, \\ell_{c}$ remain constant as $\\ell$ is moving (keeping its direction). Thus $T$ should be the common point of $\\ell_{a}, \\ell_{b}$, and $\\ell_{c}$, lying on $\\omega$.","tier":0} +{"problem_type":"Geometry","problem_label":"G5","problem":"Let $A B C$ be a triangle with circumcircle $\\omega$ and incentre $I$. A line $\\ell$ intersects the lines $A I, B I$, and $C I$ at points $D, E$, and $F$, respectively, distinct from the points $A, B, C$, and $I$. The perpendicular bisectors $x, y$, and $z$ of the segments $A D, B E$, and $C F$, respectively determine a triangle $\\Theta$. Show that the circumcircle of the triangle $\\Theta$ is tangent to $\\omega$. (Denmark)","solution":"As mentioned in the preamble, it is sufficient to prove that the centre $T$ of the homothety taking $X Y Z$ to $X_{0} Y_{0} Z_{0}$ belongs to $\\omega$. Thus, it suffices to prove that $\\Varangle\\left(T X_{0}, T Y_{0}\\right)=$ $\\Varangle\\left(Z_{0} X_{0}, Z_{0} Y_{0}\\right)$, or, equivalently, $\\Varangle\\left(X X_{0}, Y Y_{0}\\right)=\\Varangle\\left(Z_{0} X_{0}, Z_{0} Y_{0}\\right)$. Recall that $Y Z$ and $Y_{0} Z_{0}$ are the perpendicular bisectors of $A D$ and $A I$, respectively. Then, the vector $\\vec{x}$ perpendicular to $Y Z$ and shifting the line $Y_{0} Z_{0}$ to $Y Z$ is equal to $\\frac{1}{2} \\overrightarrow{I D}$. Define the shifting vectors $\\vec{y}=\\frac{1}{2} \\overrightarrow{I E}, \\vec{z}=\\frac{1}{2} \\overrightarrow{I F}$ similarly. Consider now the triangle $U V W$ formed by the perpendiculars to $A I, B I$, and $C I$ through $D, E$, and $F$, respectively (see figure below). This is another triangle whose sides are parallel to the corresponding sides of $X Y Z$. Claim 2. $\\overrightarrow{I U}=2 \\overrightarrow{X_{0} X}, \\overrightarrow{I V}=2 \\overrightarrow{Y_{0} Y}, \\overrightarrow{I W}=2 \\overrightarrow{Z_{0} Z}$. Proof. We prove one of the relations, the other proofs being similar. To prove the equality of two vectors it suffices to project them onto two non-parallel axes and check that their projections are equal. The projection of $\\overrightarrow{X_{0} X}$ onto $I B$ equals $\\vec{y}$, while the projection of $\\overrightarrow{I U}$ onto $I B$ is $\\overrightarrow{I E}=2 \\vec{y}$. The projections onto the other axis $I C$ are $\\vec{z}$ and $\\overrightarrow{I F}=2 \\vec{z}$. Then $\\overrightarrow{I U}=2 \\overrightarrow{X_{0} X}$ follows. Notice that the line $\\ell$ is the Simson line of the point $I$ with respect to the triangle $U V W$; thus $U, V, W$, and $I$ are concyclic. It follows from Claim 2 that $\\Varangle\\left(X X_{0}, Y Y_{0}\\right)=\\Varangle(I U, I V)=$ $\\Varangle(W U, W V)=\\Varangle\\left(Z_{0} X_{0}, Z_{0} Y_{0}\\right)$, and we are done. ![](https:\/\/cdn.mathpix.com\/cropped\/2024_04_17_6da854f21230b13cd732g-48.jpg?height=888&width=1106&top_left_y=1161&top_left_x=475)","tier":0} +{"problem_type":"Geometry","problem_label":"G5","problem":"Let $A B C$ be a triangle with circumcircle $\\omega$ and incentre $I$. A line $\\ell$ intersects the lines $A I, B I$, and $C I$ at points $D, E$, and $F$, respectively, distinct from the points $A, B, C$, and $I$. The perpendicular bisectors $x, y$, and $z$ of the segments $A D, B E$, and $C F$, respectively determine a triangle $\\Theta$. Show that the circumcircle of the triangle $\\Theta$ is tangent to $\\omega$. (Denmark)","solution":"Let $I_{a}, I_{b}$, and $I_{c}$ be the excentres of triangle $A B C$ corresponding to $A, B$, and $C$, respectively. Also, let $u, v$, and $w$ be the lines through $D, E$, and $F$ which are perpendicular to $A I, B I$, and $C I$, respectively, and let $U V W$ be the triangle determined by these lines, where $u=V W, v=U W$ and $w=U V$ (see figure above). Notice that the line $u$ is the reflection of $I_{b} I_{c}$ in the line $x$, because $u, x$, and $I_{b} I_{c}$ are perpendicular to $A D$ and $x$ is the perpendicular bisector of $A D$. Likewise, $v$ and $I_{a} I_{c}$ are reflections of each other in $y$, while $w$ and $I_{a} I_{b}$ are reflections of each other in $z$. It follows that $X, Y$, and $Z$ are the midpoints of $U I_{a}, V I_{b}$ and $W I_{c}$, respectively, and that the triangles $U V W$, $X Y Z$ and $I_{a} I_{b} I_{c}$ are either translates of each other or homothetic with a common homothety centre. Construct the points $T$ and $S$ such that the quadrilaterals $U V I W, X Y T Z$ and $I_{a} I_{b} S I_{c}$ are homothetic. Then $T$ is the midpoint of $I S$. Moreover, note that $\\ell$ is the Simson line of the point $I$ with respect to the triangle $U V W$, hence $I$ belongs to the circumcircle of the triangle $U V W$, therefore $T$ belongs to $\\Omega$. Consider now the homothety or translation $h_{1}$ that maps $X Y Z T$ to $I_{a} I_{b} I_{c} S$ and the homothety $h_{2}$ with centre $I$ and factor $\\frac{1}{2}$. Furthermore, let $h=h_{2} \\circ h_{1}$. The transform $h$ can be a homothety or a translation, and $$ h(T)=h_{2}\\left(h_{1}(T)\\right)=h_{2}(S)=T $$ hence $T$ is a fixed point of $h$. So, $h$ is a homothety with centre $T$. Note that $h_{2}$ maps the excentres $I_{a}, I_{b}, I_{c}$ to $X_{0}, Y_{0}, Z_{0}$ defined in the preamble. Thus the centre $T$ of the homothety taking $X Y Z$ to $X_{0} Y_{0} Z_{0}$ belongs to $\\Omega$, and this completes the proof.","tier":0} +{"problem_type":"Geometry","problem_label":"G6","problem":"A convex quadrilateral $A B C D$ satisfies $A B \\cdot C D=B C \\cdot D A$. A point $X$ is chosen inside the quadrilateral so that $\\angle X A B=\\angle X C D$ and $\\angle X B C=\\angle X D A$. Prove that $\\angle A X B+$ $\\angle C X D=180^{\\circ}$. (Poland)","solution":"Let $B^{\\prime}$ be the reflection of $B$ in the internal angle bisector of $\\angle A X C$, so that $\\angle A X B^{\\prime}=\\angle C X B$ and $\\angle C X B^{\\prime}=\\angle A X B$. If $X, D$, and $B^{\\prime}$ are collinear, then we are done. Now assume the contrary. On the ray $X B^{\\prime}$ take a point $E$ such that $X E \\cdot X B=X A \\cdot X C$, so that $\\triangle A X E \\sim$ $\\triangle B X C$ and $\\triangle C X E \\sim \\triangle B X A$. We have $\\angle X C E+\\angle X C D=\\angle X B A+\\angle X A B<180^{\\circ}$ and $\\angle X A E+\\angle X A D=\\angle X D A+\\angle X A D<180^{\\circ}$, which proves that $X$ lies inside the angles $\\angle E C D$ and $\\angle E A D$ of the quadrilateral $E A D C$. Moreover, $X$ lies in the interior of exactly one of the two triangles $E A D, E C D$ (and in the exterior of the other). ![](https:\/\/cdn.mathpix.com\/cropped\/2024_04_17_6da854f21230b13cd732g-50.jpg?height=491&width=600&top_left_y=891&top_left_x=728) The similarities mentioned above imply $X A \\cdot B C=X B \\cdot A E$ and $X B \\cdot C E=X C \\cdot A B$. Multiplying these equalities with the given equality $A B \\cdot C D=B C \\cdot D A$, we obtain $X A \\cdot C D$. $C E=X C \\cdot A D \\cdot A E$, or, equivalently, $$ \\frac{X A \\cdot D E}{A D \\cdot A E}=\\frac{X C \\cdot D E}{C D \\cdot C E} $$ Lemma. Let $P Q R$ be a triangle, and let $X$ be a point in the interior of the angle $Q P R$ such that $\\angle Q P X=\\angle P R X$. Then $\\frac{P X \\cdot Q R}{P Q \\cdot P R}<1$ if and only if $X$ lies in the interior of the triangle $P Q R$. Proof. The locus of points $X$ with $\\angle Q P X=\\angle P R X$ lying inside the angle $Q P R$ is an arc $\\alpha$ of the circle $\\gamma$ through $R$ tangent to $P Q$ at $P$. Let $\\gamma$ intersect the line $Q R$ again at $Y$ (if $\\gamma$ is tangent to $Q R$, then set $Y=R)$. The similarity $\\triangle Q P Y \\sim \\triangle Q R P$ yields $P Y=\\frac{P Q \\cdot P R}{Q R}$. Now it suffices to show that $P X

X T_{1}$. Let segments $X Z$ and $Z_{1} T_{1}$ intersect at $U$. We have $$ \\frac{T_{1} X}{T_{1} Z_{1}}<\\frac{T_{1} X}{T_{1} U}=\\frac{T X}{Z T}=\\frac{X Y}{Y Z}<\\frac{X Y}{Y Z_{1}} $$ thus $T_{1} X \\cdot Y Z_{1}\\beta$ be nonnegative integers. Then, for every integer $M \\geqslant \\beta+1$, there exists a nonnegative integer $\\gamma$ such that $$ \\frac{\\alpha+\\gamma+1}{\\beta+\\gamma+1}=1+\\frac{1}{M}=\\frac{M+1}{M} . $$ Proof. $$ \\frac{\\alpha+\\gamma+1}{\\beta+\\gamma+1}=1+\\frac{1}{M} \\Longleftrightarrow \\frac{\\alpha-\\beta}{\\beta+\\gamma+1}=\\frac{1}{M} \\Longleftrightarrow \\gamma=M(\\alpha-\\beta)-(\\beta+1) \\geqslant 0 . $$ Now we can finish the solution. Without loss of generality, there exists an index $u$ such that $\\alpha_{i}>\\beta_{i}$ for $i=1,2, \\ldots, u$, and $\\alpha_{i}<\\beta_{i}$ for $i=u+1, \\ldots, t$. The conditions $n \\nmid k$ and $k \\nmid n$ mean that $1 \\leqslant u \\leqslant t-1$. Choose an integer $X$ greater than all the $\\alpha_{i}$ and $\\beta_{i}$. By the lemma, we can define the numbers $\\gamma_{i}$ so as to satisfy $$ \\begin{array}{ll} \\frac{\\alpha_{i}+\\gamma_{i}+1}{\\beta_{i}+\\gamma_{i}+1}=\\frac{u X+i}{u X+i-1} & \\text { for } i=1,2, \\ldots, u, \\text { and } \\\\ \\frac{\\beta_{u+i}+\\gamma_{u+i}+1}{\\alpha_{u+i}+\\gamma_{u+i}+1}=\\frac{(t-u) X+i}{(t-u) X+i-1} & \\text { for } i=1,2, \\ldots, t-u . \\end{array} $$ Then we will have $$ \\frac{d(s n)}{d(s k)}=\\prod_{i=1}^{u} \\frac{u X+i}{u X+i-1} \\cdot \\prod_{i=1}^{t-u} \\frac{(t-u) X+i-1}{(t-u) X+i}=\\frac{u(X+1)}{u X} \\cdot \\frac{(t-u) X}{(t-u)(X+1)}=1, $$ as required. Comment. The lemma can be used in various ways, in order to provide a suitable value of $s$. In particular, one may apply induction on the number $t$ of prime factors, using identities like $$ \\frac{n}{n-1}=\\frac{n^{2}}{n^{2}-1} \\cdot \\frac{n+1}{n} $$","tier":0} +{"problem_type":"Number Theory","problem_label":"N2","problem":"Let $n>1$ be a positive integer. Each cell of an $n \\times n$ table contains an integer. Suppose that the following conditions are satisfied: (i) Each number in the table is congruent to 1 modulo $n$; (ii) The sum of numbers in any row, as well as the sum of numbers in any column, is congruent to $n$ modulo $n^{2}$. Let $R_{i}$ be the product of the numbers in the $i^{\\text {th }}$ row, and $C_{j}$ be the product of the numbers in the $j^{\\text {th }}$ column. Prove that the sums $R_{1}+\\cdots+R_{n}$ and $C_{1}+\\cdots+C_{n}$ are congruent modulo $n^{4}$. (Indonesia)","solution":"Let $A_{i, j}$ be the entry in the $i^{\\text {th }}$ row and the $j^{\\text {th }}$ column; let $P$ be the product of all $n^{2}$ entries. For convenience, denote $a_{i, j}=A_{i, j}-1$ and $r_{i}=R_{i}-1$. We show that $$ \\sum_{i=1}^{n} R_{i} \\equiv(n-1)+P \\quad\\left(\\bmod n^{4}\\right) $$ Due to symmetry of the problem conditions, the sum of all the $C_{j}$ is also congruent to $(n-1)+P$ modulo $n^{4}$, whence the conclusion. By condition $(i)$, the number $n$ divides $a_{i, j}$ for all $i$ and $j$. So, every product of at least two of the $a_{i, j}$ is divisible by $n^{2}$, hence $R_{i}=\\prod_{j=1}^{n}\\left(1+a_{i, j}\\right)=1+\\sum_{j=1}^{n} a_{i, j}+\\sum_{1 \\leqslant j_{1}1$ be a positive integer. Each cell of an $n \\times n$ table contains an integer. Suppose that the following conditions are satisfied: (i) Each number in the table is congruent to 1 modulo $n$; (ii) The sum of numbers in any row, as well as the sum of numbers in any column, is congruent to $n$ modulo $n^{2}$. Let $R_{i}$ be the product of the numbers in the $i^{\\text {th }}$ row, and $C_{j}$ be the product of the numbers in the $j^{\\text {th }}$ column. Prove that the sums $R_{1}+\\cdots+R_{n}$ and $C_{1}+\\cdots+C_{n}$ are congruent modulo $n^{4}$. (Indonesia)","solution":"We present a more straightforward (though lengthier) way to establish (1). We also use the notation of $a_{i, j}$. By condition (i), all the $a_{i, j}$ are divisible by $n$. Therefore, we have $$ \\begin{aligned} P=\\prod_{i=1}^{n} \\prod_{j=1}^{n}\\left(1+a_{i, j}\\right) \\equiv 1+\\sum_{(i, j)} a_{i, j} & +\\sum_{\\left(i_{1}, j_{1}\\right),\\left(i_{2}, j_{2}\\right)} a_{i_{1}, j_{1}} a_{i_{2}, j_{2}} \\\\ & +\\sum_{\\left(i_{1}, j_{1}\\right),\\left(i_{2}, j_{2}\\right),\\left(i_{3}, j_{3}\\right)} a_{i_{1}, j_{1}} a_{i_{2}, j_{2}} a_{i_{3}, j_{3}}\\left(\\bmod n^{4}\\right), \\end{aligned} $$ where the last two sums are taken over all unordered pairs\/triples of pairwise different pairs $(i, j)$; such conventions are applied throughout the solution. Similarly, $$ \\sum_{i=1}^{n} R_{i}=\\sum_{i=1}^{n} \\prod_{j=1}^{n}\\left(1+a_{i, j}\\right) \\equiv n+\\sum_{i} \\sum_{j} a_{i, j}+\\sum_{i} \\sum_{j_{1}, j_{2}} a_{i, j_{1}} a_{i, j_{2}}+\\sum_{i} \\sum_{j_{1}, j_{2}, j_{3}} a_{i, j_{1}} a_{i, j_{2}} a_{i, j_{3}} \\quad\\left(\\bmod n^{4}\\right) $$ Therefore, $$ \\begin{aligned} P+(n-1)-\\sum_{i} R_{i} \\equiv \\sum_{\\substack{\\left(i_{1}, j_{1}\\right),\\left(i_{2}, j_{2}\\right) \\\\ i_{1} \\neq i_{2}}} a_{i_{1}, j_{1}} a_{i_{2}, j_{2}} & +\\sum_{\\substack{\\left(i_{1}, j_{1}\\right),\\left(i_{2}, j_{2}\\right),\\left(i_{3}, j_{3}\\right) \\\\ i_{1} \\neq i_{2} \\neq i_{3} \\neq i_{1}}} a_{i_{1}, j_{1}} a_{i_{2}, j_{2}} a_{i_{3}, j_{3}} \\\\ & +\\sum_{\\substack{\\left(i_{1}, j_{1}\\right),\\left(i_{2}, j_{2}\\right),\\left(i_{3}, j_{3}\\right) \\\\ i_{1} \\neq i_{2}=i_{3}}} a_{i_{1}, j_{1}} a_{i_{2}, j_{2}} a_{i_{3}, j_{3}}\\left(\\bmod n^{4}\\right) . \\end{aligned} $$ We show that in fact each of the three sums appearing in the right-hand part of this congruence is divisible by $n^{4}$; this yields (1). Denote those three sums by $\\Sigma_{1}, \\Sigma_{2}$, and $\\Sigma_{3}$ in order of appearance. Recall that by condition (ii) we have $$ \\sum_{j} a_{i, j} \\equiv 0 \\quad\\left(\\bmod n^{2}\\right) \\quad \\text { for all indices } i $$ For every two indices $i_{1}3$, so $$ S_{n-1}=2^{n}+2^{\\lceil n \/ 2\\rceil}+2^{\\lfloor n \/ 2\\rfloor}-3>2^{n}+2^{\\lfloor n \/ 2\\rfloor}=a_{n} . $$ Also notice that $S_{n-1}-a_{n}=2^{[n \/ 2]}-3a_{n}$. Denote $c=S_{n-1}-b$; then $S_{n-1}-a_{n}2^{t}-3$. Proof. The inequality follows from $t \\geqslant 3$. In order to prove the equivalence, we apply Claim 1 twice in the following manner. First, since $S_{2 t-3}-a_{2 t-2}=2^{t-1}-3<2^{t}-3<2^{2 t-2}+2^{t-1}=a_{2 t-2}$, by Claim 1 we have $2^{t}-3 \\sim S_{2 t-3}-\\left(2^{t}-3\\right)=2^{2 t-2}$. Second, since $S_{4 t-7}-a_{4 t-6}=2^{2 t-3}-3<2^{2 t-2}<2^{4 t-6}+2^{2 t-3}=a_{4 t-6}$, by Claim 1 we have $2^{2 t-2} \\sim S_{4 t-7}-2^{2 t-2}=2^{4 t-6}-3$. Therefore, $2^{t}-3 \\sim 2^{2 t-2} \\sim 2^{4 t-6}-3$, as required. Now it is easy to find the required numbers. Indeed, the number $2^{3}-3=5=a_{0}+a_{1}$ is representable, so Claim 3 provides an infinite sequence of representable numbers $$ 2^{3}-3 \\sim 2^{6}-3 \\sim 2^{18}-3 \\sim \\cdots \\sim 2^{t}-3 \\sim 2^{4 t-6}-3 \\sim \\cdots \\text {. } $$ On the other hand, the number $2^{7}-3=125$ is non-representable (since by Claim 1 we have $125 \\sim S_{6}-125=24 \\sim S_{4}-24=17 \\sim S_{3}-17=4$ which is clearly non-representable). So Claim 3 provides an infinite sequence of non-representable numbers $$ 2^{7}-3 \\sim 2^{22}-3 \\sim 2^{82}-3 \\sim \\cdots \\sim 2^{t}-3 \\sim 2^{4 t-6}-3 \\sim \\cdots . $$","tier":0} +{"problem_type":"Number Theory","problem_label":"N3","problem":"Define the sequence $a_{0}, a_{1}, a_{2}, \\ldots$ by $a_{n}=2^{n}+2^{\\lfloor n \/ 2\\rfloor}$. Prove that there are infinitely many terms of the sequence which can be expressed as a sum of (two or more) distinct terms of the sequence, as well as infinitely many of those which cannot be expressed in such a way. (Serbia)","solution":"We keep the notion of representability and the notation $S_{n}$ from the previous solution. We say that an index $n$ is good if $a_{n}$ writes as a sum of smaller terms from the sequence $a_{0}, a_{1}, \\ldots$. Otherwise we say it is bad. We must prove that there are infinitely many good indices, as well as infinitely many bad ones. Lemma 1. If $m \\geqslant 0$ is an integer, then $4^{m}$ is representable if and only if either of $2 m+1$ and $2 m+2$ is good. Proof. The case $m=0$ is obvious, so we may assume that $m \\geqslant 1$. Let $n=2 m+1$ or $2 m+2$. Then $n \\geqslant 3$. We notice that $$ S_{n-1}4^{s}$. Proof. We have $2^{4 k-2}s$. Now $4^{2}=a_{2}+a_{3}$ is representable, whereas $4^{6}=4096$ is not. Indeed, note that $4^{6}=2^{12}v_{p}\\left(a_{n}\\right)$, then $v_{p}\\left(a_{n} \/ a_{n+1}\\right)<0$, while $v_{p}\\left(\\left(a_{n+1}-a_{n}\\right) \/ a_{1}\\right) \\geqslant 0$, so $(*)$ is not integer again. Thus, $v_{p}\\left(a_{1}\\right) \\leqslant v_{p}\\left(a_{n+1}\\right) \\leqslant v_{p}\\left(a_{n}\\right)$. The above arguments can now be applied successively to indices $n+1, n+2, \\ldots$, showing that all the indices greater than $n$ are large, and the sequence $v_{p}\\left(a_{n}\\right), v_{p}\\left(a_{n+1}\\right), v_{p}\\left(a_{n+2}\\right), \\ldots$ is nonincreasing \u2014 hence eventually constant. Case 2: There is no large index. We have $v_{p}\\left(a_{1}\\right)>v_{p}\\left(a_{n}\\right)$ for all $n \\geqslant k$. If we had $v_{p}\\left(a_{n+1}\\right)0$. Suppose that the number $x+y=z+t$ is odd. Then $x$ and $y$ have opposite parity, as well as $z$ and $t$. This means that both $x y$ and $z t$ are even, as well as $x y-z t=x+y$; a contradiction. Thus, $x+y$ is even, so the number $s=\\frac{x+y}{2}=\\frac{z+t}{2}$ is a positive integer. Next, we set $b=\\frac{|x-y|}{2}, d=\\frac{|z-t|}{2}$. Now the problem conditions yield $$ s^{2}=a^{2}+b^{2}=c^{2}+d^{2} $$ and $$ 2 s=a^{2}-c^{2}=d^{2}-b^{2} $$ (the last equality in (2) follows from (1)). We readily get from (2) that $a, d>0$. In the sequel we will use only the relations (1) and (2), along with the fact that $a, d, s$ are positive integers, while $b$ and $c$ are nonnegative integers, at most one of which may be zero. Since both relations are symmetric with respect to the simultaneous swappings $a \\leftrightarrow d$ and $b \\leftrightarrow c$, we assume, without loss of generality, that $b \\geqslant c$ (and hence $b>0$ ). Therefore, $d^{2}=2 s+b^{2}>c^{2}$, whence $$ d^{2}>\\frac{c^{2}+d^{2}}{2}=\\frac{s^{2}}{2} $$ On the other hand, since $d^{2}-b^{2}$ is even by (2), the numbers $b$ and $d$ have the same parity, so $00$ imply $b=c=0$, which is impossible.","tier":0} +{"problem_type":"Number Theory","problem_label":"N5","problem":"Four positive integers $x, y, z$, and $t$ satisfy the relations $$ x y-z t=x+y=z+t $$ Is it possible that both $x y$ and $z t$ are perfect squares? (Russia)","solution":"We start with a complete description of all 4-tuples $(x, y, z, t)$ of positive integers satisfying (*). As in the solution above, we notice that the numbers $$ s=\\frac{x+y}{2}=\\frac{z+t}{2}, \\quad p=\\frac{x-y}{2}, \\quad \\text { and } \\quad q=\\frac{z-t}{2} $$ are integers (we may, and will, assume that $p, q \\geqslant 0$ ). We have $$ 2 s=x y-z t=(s+p)(s-p)-(s+q)(s-q)=q^{2}-p^{2} $$ so $p$ and $q$ have the same parity, and $q>p$. Set now $k=\\frac{q-p}{2}, \\ell=\\frac{q+p}{2}$. Then we have $s=\\frac{q^{2}-p^{2}}{2}=2 k \\ell$ and hence $$ \\begin{array}{rlrl} x & =s+p=2 k \\ell-k+\\ell, & y & =s-p=2 k \\ell+k-\\ell, \\\\ z & =s+q=2 k \\ell+k+\\ell, & t=s-q=2 k \\ell-k-\\ell . \\end{array} $$ Recall here that $\\ell \\geqslant k>0$ and, moreover, $(k, \\ell) \\neq(1,1)$, since otherwise $t=0$. Assume now that both $x y$ and $z t$ are squares. Then $x y z t$ is also a square. On the other hand, we have $$ \\begin{aligned} x y z t=(2 k \\ell-k+\\ell) & (2 k \\ell+k-\\ell)(2 k \\ell+k+\\ell)(2 k \\ell-k-\\ell) \\\\ & =\\left(4 k^{2} \\ell^{2}-(k-\\ell)^{2}\\right)\\left(4 k^{2} \\ell^{2}-(k+\\ell)^{2}\\right)=\\left(4 k^{2} \\ell^{2}-k^{2}-\\ell^{2}\\right)^{2}-4 k^{2} \\ell^{2} \\end{aligned} $$ Denote $D=4 k^{2} \\ell^{2}-k^{2}-\\ell^{2}>0$. From (6) we get $D^{2}>x y z t$. On the other hand, $$ \\begin{array}{r} (D-1)^{2}=D^{2}-2\\left(4 k^{2} \\ell^{2}-k^{2}-\\ell^{2}\\right)+1=\\left(D^{2}-4 k^{2} \\ell^{2}\\right)-\\left(2 k^{2}-1\\right)\\left(2 \\ell^{2}-1\\right)+2 \\\\ =x y z t-\\left(2 k^{2}-1\\right)\\left(2 \\ell^{2}-1\\right)+20$ and $\\ell \\geqslant 2$. The converse is also true: every pair of positive integers $\\ell \\geqslant k>0$, except for the pair $k=\\ell=1$, generates via (5) a 4-tuple of positive integers satisfying $(*)$.","tier":0} +{"problem_type":"Number Theory","problem_label":"N6","problem":"Let $f:\\{1,2,3, \\ldots\\} \\rightarrow\\{2,3, \\ldots\\}$ be a function such that $f(m+n) \\mid f(m)+f(n)$ for all pairs $m, n$ of positive integers. Prove that there exists a positive integer $c>1$ which divides all values of $f$. (Mexico)","solution":"For every positive integer $m$, define $S_{m}=\\{n: m \\mid f(n)\\}$. Lemma. If the set $S_{m}$ is infinite, then $S_{m}=\\{d, 2 d, 3 d, \\ldots\\}=d \\cdot \\mathbb{Z}_{>0}$ for some positive integer $d$. Proof. Let $d=\\min S_{m}$; the definition of $S_{m}$ yields $m \\mid f(d)$. Whenever $n \\in S_{m}$ and $n>d$, we have $m|f(n)| f(n-d)+f(d)$, so $m \\mid f(n-d)$ and therefore $n-d \\in S_{m}$. Let $r \\leqslant d$ be the least positive integer with $n \\equiv r(\\bmod d)$; repeating the same step, we can see that $n-d, n-2 d, \\ldots, r \\in S_{m}$. By the minimality of $d$, this shows $r=d$ and therefore $d \\mid n$. Starting from an arbitrarily large element of $S_{m}$, the process above reaches all multiples of $d$; so they all are elements of $S_{m}$. The solution for the problem will be split into two cases. Case 1: The function $f$ is bounded. Call a prime $p$ frequent if the set $S_{p}$ is infinite, i.e., if $p$ divides $f(n)$ for infinitely many positive integers $n$; otherwise call $p$ sporadic. Since the function $f$ is bounded, there are only a finite number of primes that divide at least one $f(n)$; so altogether there are finitely many numbers $n$ such that $f(n)$ has a sporadic prime divisor. Let $N$ be a positive integer, greater than all those numbers $n$. Let $p_{1}, \\ldots, p_{k}$ be the frequent primes. By the lemma we have $S_{p_{i}}=d_{i} \\cdot \\mathbb{Z}_{>0}$ for some $d_{i}$. Consider the number $$ n=N d_{1} d_{2} \\cdots d_{k}+1 $$ Due to $n>N$, all prime divisors of $f(n)$ are frequent primes. Let $p_{i}$ be any frequent prime divisor of $f(n)$. Then $n \\in S_{p_{i}}$, and therefore $d_{i} \\mid n$. But $n \\equiv 1\\left(\\bmod d_{i}\\right)$, which means $d_{i}=1$. Hence $S_{p_{i}}=1 \\cdot \\mathbb{Z}_{>0}=\\mathbb{Z}_{>0}$ and therefore $p_{i}$ is a common divisor of all values $f(n)$. Case 2: $f$ is unbounded. We prove that $f(1)$ divides all $f(n)$. Let $a=f(1)$. Since $1 \\in S_{a}$, by the lemma it suffices to prove that $S_{a}$ is an infinite set. Call a positive integer $p$ a peak if $f(p)>\\max (f(1), \\ldots, f(p-1))$. Since $f$ is not bounded, there are infinitely many peaks. Let $1=p_{1}n+2$ and $h=f(p)>f(n)+2 a$. By (1) we have $f(p-1)=$ $f(p)-f(1)=h-a$ and $f(n+1)=f(p)-f(p-n-1)=h-f(p-n-1)$. From $h-a=f(p-1) \\mid$ $f(n)+f(p-n-1)2018\\end{cases}\\right. $$","tier":0} +{"problem_type":"Number Theory","problem_label":"N6","problem":"Let $f:\\{1,2,3, \\ldots\\} \\rightarrow\\{2,3, \\ldots\\}$ be a function such that $f(m+n) \\mid f(m)+f(n)$ for all pairs $m, n$ of positive integers. Prove that there exists a positive integer $c>1$ which divides all values of $f$. (Mexico)","solution":"Let $d_{n}=\\operatorname{gcd}(f(n), f(1))$. From $d_{n+1} \\mid f(1)$ and $d_{n+1}|f(n+1)| f(n)+f(1)$, we can see that $d_{n+1} \\mid f(n)$; then $d_{n+1} \\mid \\operatorname{gcd}(f(n), f(1))=d_{n}$. So the sequence $d_{1}, d_{2}, \\ldots$ is nonincreasing in the sense that every element is a divisor of the previous elements. Let $d=\\min \\left(d_{1}, d_{2}, \\ldots\\right)=\\operatorname{gcd}\\left(d_{1} \\cdot d_{2}, \\ldots\\right)=\\operatorname{gcd}(f(1), f(2), \\ldots)$; we have to prove $d \\geqslant 2$. For the sake of contradiction, suppose that the statement is wrong, so $d=1$; that means there is some index $n_{0}$ such that $d_{n}=1$ for every $n \\geqslant n_{0}$, i.e., $f(n)$ is coprime with $f(1)$. Claim 1. If $2^{k} \\geqslant n_{0}$ then $f\\left(2^{k}\\right) \\leqslant 2^{k}$. Proof. By the condition, $f(2 n) \\mid 2 f(n)$; a trivial induction yields $f\\left(2^{k}\\right) \\mid 2^{k} f(1)$. If $2^{k} \\geqslant n_{0}$ then $f\\left(2^{k}\\right)$ is coprime with $f(1)$, so $f\\left(2^{k}\\right)$ is a divisor of $2^{k}$. Claim 2. There is a constant $C$ such that $f(n)0}$ are coprime then $\\operatorname{gcd}(f(a), f(b)) \\mid f(1)$. In particular, if $a, b \\geqslant n_{0}$ are coprime then $f(a)$ and $f(b)$ are coprime. Proof. Let $d=\\operatorname{gcd}(f(a), f(b))$. We can replicate Euclid's algorithm. Formally, apply induction on $a+b$. If $a=1$ or $b=1$ then we already have $d \\mid f(1)$. Without loss of generality, suppose $1C$ (that is possible, because there are arbitrarily long gaps between the primes). Then we establish a contradiction $$ p_{N+1} \\leqslant \\max \\left(f(1), f\\left(q_{1}\\right), \\ldots, f\\left(q_{N}\\right)\\right)<\\max \\left(1+C, q_{1}+C, \\ldots, q_{N}+C\\right)=p_{N}+C0$ and $c, d$ are coprime. We will show that too many denominators $b_{i}$ should be divisible by $d$. To this end, for any $1 \\leqslant i \\leqslant n$ and any prime divisor $p$ of $d$, say that the index $i$ is $p$-wrong, if $v_{p}\\left(b_{i}\\right)5 n, $$ a contradiction. Claim 3. For every $0 \\leqslant k \\leqslant n-30$, among the denominators $b_{k+1}, b_{k+2}, \\ldots, b_{k+30}$, at least $\\varphi(30)=8$ are divisible by $d$. Proof. By Claim 1, the 2 -wrong, 3 -wrong and 5 -wrong indices can be covered by three arithmetic progressions with differences 2,3 and 5 . By a simple inclusion-exclusion, $(2-1) \\cdot(3-1) \\cdot(5-1)=8$ indices are not covered; by Claim 2, we have $d \\mid b_{i}$ for every uncovered index $i$. Claim 4. $|\\Delta|<\\frac{20}{n-2}$ and $d>\\frac{n-2}{20}$. Proof. From the sequence (1), remove all fractions with $b_{n}<\\frac{n}{2}$, There remain at least $\\frac{n}{2}$ fractions, and they cannot exceed $\\frac{5 n}{n \/ 2}=10$. So we have at least $\\frac{n}{2}$ elements of the arithmetic progression (1) in the interval $(0,10]$, hence the difference must be below $\\frac{10}{n \/ 2-1}=\\frac{20}{n-2}$. The second inequality follows from $\\frac{1}{d} \\leqslant \\frac{|c|}{d}=|\\Delta|$. Now we have everything to get the final contradiction. By Claim 3, we have $d \\mid b_{i}$ for at least $\\left\\lfloor\\frac{n}{30}\\right\\rfloor \\cdot 8$ indices $i$. By Claim 4 , we have $d \\geqslant \\frac{n-2}{20}$. Therefore, $$ 5 n \\geqslant \\max \\left\\{b_{i}: d \\mid b_{i}\\right\\} \\geqslant\\left(\\left\\lfloor\\frac{n}{30}\\right\\rfloor \\cdot 8\\right) \\cdot d>\\left(\\frac{n}{30}-1\\right) \\cdot 8 \\cdot \\frac{n-2}{20}>5 n . $$ Comment 1. It is possible that all terms in (1) are equal, for example with $a_{i}=2 i-1$ and $b_{i}=4 i-2$ we have $\\frac{a_{i}}{b_{i}}=\\frac{1}{2}$. Comment 2. The bound $5 n$ in the statement is far from sharp; the solution above can be modified to work for $9 n$. For large $n$, the bound $5 n$ can be replaced by $n^{\\frac{3}{2}-\\varepsilon}$. The activities of the Problem Selection Committee were supported by DEDEMAN LA FANTANA vine la tine","tier":0} +{"problem_type":"Algebra","problem_label":"A1","problem":"Let $\\mathbb{Q}_{>0}$ denote the set of all positive rational numbers. Determine all functions $f: \\mathbb{Q}_{>0} \\rightarrow \\mathbb{Q}_{>0}$ satisfying $$ f\\left(x^{2} f(y)^{2}\\right)=f(x)^{2} f(y) $$ for all $x, y \\in \\mathbb{Q}_{>0}$. (Switzerland) Answer: $f(x)=1$ for all $x \\in \\mathbb{Q}_{>0}$.","solution":"Take any $a, b \\in \\mathbb{Q}_{>0}$. By substituting $x=f(a), y=b$ and $x=f(b), y=a$ into $(*)$ we get $$ f(f(a))^{2} f(b)=f\\left(f(a)^{2} f(b)^{2}\\right)=f(f(b))^{2} f(a) $$ which yields $$ \\frac{f(f(a))^{2}}{f(a)}=\\frac{f(f(b))^{2}}{f(b)} \\quad \\text { for all } a, b \\in \\mathbb{Q}_{>0} $$ In other words, this shows that there exists a constant $C \\in \\mathbb{Q}_{>0}$ such that $f(f(a))^{2}=C f(a)$, or $$ \\left(\\frac{f(f(a))}{C}\\right)^{2}=\\frac{f(a)}{C} \\quad \\text { for all } a \\in \\mathbb{Q}_{>0} \\text {. } $$ Denote by $f^{n}(x)=\\underbrace{f(f(\\ldots(f}_{n}(x)) \\ldots))$ the $n^{\\text {th }}$ iteration of $f$. Equality (1) yields $$ \\frac{f(a)}{C}=\\left(\\frac{f^{2}(a)}{C}\\right)^{2}=\\left(\\frac{f^{3}(a)}{C}\\right)^{4}=\\cdots=\\left(\\frac{f^{n+1}(a)}{C}\\right)^{2^{n}} $$ for all positive integer $n$. So, $f(a) \/ C$ is the $2^{n}$-th power of a rational number for all positive integer $n$. This is impossible unless $f(a) \/ C=1$, since otherwise the exponent of some prime in the prime decomposition of $f(a) \/ C$ is not divisible by sufficiently large powers of 2 . Therefore, $f(a)=C$ for all $a \\in \\mathbb{Q}_{>0}$. Finally, after substituting $f \\equiv C$ into (*) we get $C=C^{3}$, whence $C=1$. So $f(x) \\equiv 1$ is the unique function satisfying $(*)$. Comment 1. There are several variations of the solution above. For instance, one may start with finding $f(1)=1$. To do this, let $d=f(1)$. By substituting $x=y=1$ and $x=d^{2}, y=1$ into (*) we get $f\\left(d^{2}\\right)=d^{3}$ and $f\\left(d^{6}\\right)=f\\left(d^{2}\\right)^{2} \\cdot d=d^{7}$. By substituting now $x=1, y=d^{2}$ we obtain $f\\left(d^{6}\\right)=d^{2} \\cdot d^{3}=d^{5}$. Therefore, $d^{7}=f\\left(d^{6}\\right)=d^{5}$, whence $d=1$. After that, the rest of the solution simplifies a bit, since we already know that $C=\\frac{f(f(1))^{2}}{f(1)}=1$. Hence equation (1) becomes merely $f(f(a))^{2}=f(a)$, which yields $f(a)=1$ in a similar manner. Comment 2. There exist nonconstant functions $f: \\mathbb{R}^{+} \\rightarrow \\mathbb{R}^{+}$satisfying $(*)$ for all real $x, y>0-$ e.g., $f(x)=\\sqrt{x}$.","tier":0} +{"problem_type":"Algebra","problem_label":"A2","problem":"Find all positive integers $n \\geqslant 3$ for which there exist real numbers $a_{1}, a_{2}, \\ldots, a_{n}$, $a_{n+1}=a_{1}, a_{n+2}=a_{2}$ such that $$ a_{i} a_{i+1}+1=a_{i+2} $$ for all $i=1,2, \\ldots, n$. (Slovakia) Answer: $n$ can be any multiple of 3 .","solution":"For the sake of convenience, extend the sequence $a_{1}, \\ldots, a_{n+2}$ to an infinite periodic sequence with period $n$. ( $n$ is not necessarily the shortest period.) If $n$ is divisible by 3 , then $\\left(a_{1}, a_{2}, \\ldots\\right)=(-1,-1,2,-1,-1,2, \\ldots)$ is an obvious solution. We will show that in every periodic sequence satisfying the recurrence, each positive term is followed by two negative values, and after them the next number is positive again. From this, it follows that $n$ is divisible by 3 . If the sequence contains two consecutive positive numbers $a_{i}, a_{i+1}$, then $a_{i+2}=a_{i} a_{i+1}+1>1$, so the next value is positive as well; by induction, all numbers are positive and greater than 1 . But then $a_{i+2}=a_{i} a_{i+1}+1 \\geqslant 1 \\cdot a_{i+1}+1>a_{i+1}$ for every index $i$, which is impossible: our sequence is periodic, so it cannot increase everywhere. If the number 0 occurs in the sequence, $a_{i}=0$ for some index $i$, then it follows that $a_{i+1}=a_{i-1} a_{i}+1$ and $a_{i+2}=a_{i} a_{i+1}+1$ are two consecutive positive elements in the sequences and we get the same contradiction again. Notice that after any two consecutive negative numbers the next one must be positive: if $a_{i}<0$ and $a_{i+1}<0$, then $a_{i+2}=a_{1} a_{i+1}+1>1>0$. Hence, the positive and negative numbers follow each other in such a way that each positive term is followed by one or two negative values and then comes the next positive term. Consider the case when the positive and negative values alternate. So, if $a_{i}$ is a negative value then $a_{i+1}$ is positive, $a_{i+2}$ is negative and $a_{i+3}$ is positive again. Notice that $a_{i} a_{i+1}+1=a_{i+2}<00$ we conclude $a_{i}1$. The number $a_{i+3}$ must be negative. We show that $a_{i+4}$ also must be negative. Notice that $a_{i+3}$ is negative and $a_{i+4}=a_{i+2} a_{i+3}+1<10, $$ therefore $a_{i+5}>a_{i+4}$. Since at most one of $a_{i+4}$ and $a_{i+5}$ can be positive, that means that $a_{i+4}$ must be negative. Now $a_{i+3}$ and $a_{i+4}$ are negative and $a_{i+5}$ is positive; so after two negative and a positive terms, the next three terms repeat the same pattern. That completes the solution.","tier":0} +{"problem_type":"Algebra","problem_label":"A2","problem":"Find all positive integers $n \\geqslant 3$ for which there exist real numbers $a_{1}, a_{2}, \\ldots, a_{n}$, $a_{n+1}=a_{1}, a_{n+2}=a_{2}$ such that $$ a_{i} a_{i+1}+1=a_{i+2} $$ for all $i=1,2, \\ldots, n$. (Slovakia) Answer: $n$ can be any multiple of 3 .","solution":"We prove that the shortest period of the sequence must be 3 . Then it follows that $n$ must be divisible by 3 . Notice that the equation $x^{2}+1=x$ has no real root, so the numbers $a_{1}, \\ldots, a_{n}$ cannot be all equal, hence the shortest period of the sequence cannot be 1 . By applying the recurrence relation for $i$ and $i+1$, $$ \\begin{gathered} \\left(a_{i+2}-1\\right) a_{i+2}=a_{i} a_{i+1} a_{i+2}=a_{i}\\left(a_{i+3}-1\\right), \\quad \\text { so } \\\\ a_{i+2}^{2}-a_{i} a_{i+3}=a_{i+2}-a_{i} . \\end{gathered} $$ By summing over $i=1,2, \\ldots, n$, we get $$ \\sum_{i=1}^{n}\\left(a_{i}-a_{i+3}\\right)^{2}=0 $$ That proves that $a_{i}=a_{i+3}$ for every index $i$, so the sequence $a_{1}, a_{2}, \\ldots$ is indeed periodic with period 3. The shortest period cannot be 1 , so it must be 3 ; therefore, $n$ is divisible by 3 . Comment. By solving the system of equations $a b+1=c, \\quad b c+1=a, \\quad c a+1=b$, it can be seen that the pattern $(-1,-1,2)$ is repeated in all sequences satisfying the problem conditions.","tier":0} +{"problem_type":"Algebra","problem_label":"A3","problem":"Given any set $S$ of positive integers, show that at least one of the following two assertions holds: (1) There exist distinct finite subsets $F$ and $G$ of $S$ such that $\\sum_{x \\in F} 1 \/ x=\\sum_{x \\in G} 1 \/ x$; (2) There exists a positive rational number $r<1$ such that $\\sum_{x \\in F} 1 \/ x \\neq r$ for all finite subsets $F$ of $S$. ## (Luxembourg)","solution":"Argue indirectly. Agree, as usual, that the empty sum is 0 to consider rationals in $[0,1)$; adjoining 0 causes no harm, since $\\sum_{x \\in F} 1 \/ x=0$ for no nonempty finite subset $F$ of $S$. For every rational $r$ in $[0,1)$, let $F_{r}$ be the unique finite subset of $S$ such that $\\sum_{x \\in F_{r}} 1 \/ x=r$. The argument hinges on the lemma below. Lemma. If $x$ is a member of $S$ and $q$ and $r$ are rationals in $[0,1)$ such that $q-r=1 \/ x$, then $x$ is a member of $F_{q}$ if and only if it is not one of $F_{r}$. Proof. If $x$ is a member of $F_{q}$, then $$ \\sum_{y \\in F_{q} \\backslash\\{x\\}} \\frac{1}{y}=\\sum_{y \\in F_{q}} \\frac{1}{y}-\\frac{1}{x}=q-\\frac{1}{x}=r=\\sum_{y \\in F_{r}} \\frac{1}{y} $$ so $F_{r}=F_{q} \\backslash\\{x\\}$, and $x$ is not a member of $F_{r}$. Conversely, if $x$ is not a member of $F_{r}$, then $$ \\sum_{y \\in F_{r} \\cup\\{x\\}} \\frac{1}{y}=\\sum_{y \\in F_{r}} \\frac{1}{y}+\\frac{1}{x}=r+\\frac{1}{x}=q=\\sum_{y \\in F_{q}} \\frac{1}{y} $$ so $F_{q}=F_{r} \\cup\\{x\\}$, and $x$ is a member of $F_{q}$. Consider now an element $x$ of $S$ and a positive rational $r<1$. Let $n=\\lfloor r x\\rfloor$ and consider the sets $F_{r-k \/ x}, k=0, \\ldots, n$. Since $0 \\leqslant r-n \/ x<1 \/ x$, the set $F_{r-n \/ x}$ does not contain $x$, and a repeated application of the lemma shows that the $F_{r-(n-2 k) \/ x}$ do not contain $x$, whereas the $F_{r-(n-2 k-1) \/ x}$ do. Consequently, $x$ is a member of $F_{r}$ if and only if $n$ is odd. Finally, consider $F_{2 \/ 3}$. By the preceding, $\\lfloor 2 x \/ 3\\rfloor$ is odd for each $x$ in $F_{2 \/ 3}$, so $2 x \/ 3$ is not integral. Since $F_{2 \/ 3}$ is finite, there exists a positive rational $\\varepsilon$ such that $\\lfloor(2 \/ 3-\\varepsilon) x\\rfloor=\\lfloor 2 x \/ 3\\rfloor$ for all $x$ in $F_{2 \/ 3}$. This implies that $F_{2 \/ 3}$ is a subset of $F_{2 \/ 3-\\varepsilon}$ which is impossible. Comment. The solution above can be adapted to show that the problem statement still holds, if the condition $r<1$ in (2) is replaced with $r<\\delta$, for an arbitrary positive $\\delta$. This yields that, if $S$ does not satisfy (1), then there exist infinitely many positive rational numbers $r<1$ such that $\\sum_{x \\in F} 1 \/ x \\neq r$ for all finite subsets $F$ of $S$.","tier":0} +{"problem_type":"Algebra","problem_label":"A3","problem":"Given any set $S$ of positive integers, show that at least one of the following two assertions holds: (1) There exist distinct finite subsets $F$ and $G$ of $S$ such that $\\sum_{x \\in F} 1 \/ x=\\sum_{x \\in G} 1 \/ x$; (2) There exists a positive rational number $r<1$ such that $\\sum_{x \\in F} 1 \/ x \\neq r$ for all finite subsets $F$ of $S$. ## (Luxembourg)","solution":"A finite $S$ clearly satisfies (2), so let $S$ be infinite. If $S$ fails both conditions, so does $S \\backslash\\{1\\}$. We may and will therefore assume that $S$ consists of integers greater than 1 . Label the elements of $S$ increasingly $x_{1}2 x_{n}$ for some $n$, then $\\sum_{x \\in F} 1 \/ x2$, we have $\\Delta_{n} \\leqslant \\frac{n-1}{n} \\Delta_{n-1}$. Proof. Choose positive integers $k, \\ell \\leqslant n$ such that $M_{n}=S(n, k) \/ k$ and $m_{n}=S(n, \\ell) \/ \\ell$. We have $S(n, k)=a_{n-1}+S(n-1, k-1)$, so $$ k\\left(M_{n}-a_{n-1}\\right)=S(n, k)-k a_{n-1}=S(n-1, k-1)-(k-1) a_{n-1} \\leqslant(k-1)\\left(M_{n-1}-a_{n-1}\\right), $$ since $S(n-1, k-1) \\leqslant(k-1) M_{n-1}$. Similarly, we get $$ \\ell\\left(a_{n-1}-m_{n}\\right)=(\\ell-1) a_{n-1}-S(n-1, \\ell-1) \\leqslant(\\ell-1)\\left(a_{n-1}-m_{n-1}\\right) . $$ Since $m_{n-1} \\leqslant a_{n-1} \\leqslant M_{n-1}$ and $k, \\ell \\leqslant n$, the obtained inequalities yield $$ \\begin{array}{ll} M_{n}-a_{n-1} \\leqslant \\frac{k-1}{k}\\left(M_{n-1}-a_{n-1}\\right) \\leqslant \\frac{n-1}{n}\\left(M_{n-1}-a_{n-1}\\right) \\quad \\text { and } \\\\ a_{n-1}-m_{n} \\leqslant \\frac{\\ell-1}{\\ell}\\left(a_{n-1}-m_{n-1}\\right) \\leqslant \\frac{n-1}{n}\\left(a_{n-1}-m_{n-1}\\right) . \\end{array} $$ Therefore, $$ \\Delta_{n}=\\left(M_{n}-a_{n-1}\\right)+\\left(a_{n-1}-m_{n}\\right) \\leqslant \\frac{n-1}{n}\\left(\\left(M_{n-1}-a_{n-1}\\right)+\\left(a_{n-1}-m_{n-1}\\right)\\right)=\\frac{n-1}{n} \\Delta_{n-1} . $$ Back to the problem, if $a_{n}=1$ for all $n \\leqslant 2017$, then $a_{2018} \\leqslant 1$ and hence $a_{2018}-a_{2017} \\leqslant 0$. Otherwise, let $2 \\leqslant q \\leqslant 2017$ be the minimal index with $a_{q}<1$. We have $S(q, i)=i$ for all $i=1,2, \\ldots, q-1$, while $S(q, q)=q-1$. Therefore, $a_{q}<1$ yields $a_{q}=S(q, q) \/ q=1-\\frac{1}{q}$. Now we have $S(q+1, i)=i-\\frac{1}{q}$ for $i=1,2, \\ldots, q$, and $S(q+1, q+1)=q-\\frac{1}{q}$. This gives us $$ m_{q+1}=\\frac{S(q+1,1)}{1}=\\frac{S(q+1, q+1)}{q+1}=\\frac{q-1}{q} \\quad \\text { and } \\quad M_{q+1}=\\frac{S(q+1, q)}{q}=\\frac{q^{2}-1}{q^{2}} $$ so $\\Delta_{q+1}=M_{q+1}-m_{q+1}=(q-1) \/ q^{2}$. Denoting $N=2017 \\geqslant q$ and using Claim 1 for $n=q+2, q+3, \\ldots, N+1$ we finally obtain $$ \\Delta_{N+1} \\leqslant \\frac{q-1}{q^{2}} \\cdot \\frac{q+1}{q+2} \\cdot \\frac{q+2}{q+3} \\cdots \\frac{N}{N+1}=\\frac{1}{N+1}\\left(1-\\frac{1}{q^{2}}\\right) \\leqslant \\frac{1}{N+1}\\left(1-\\frac{1}{N^{2}}\\right)=\\frac{N-1}{N^{2}} $$ as required. Comment 1. One may check that the maximal value of $a_{2018}-a_{2017}$ is attained at the unique sequence, which is presented in the solution above. Comment 2. An easier question would be to determine the maximal value of $\\left|a_{2018}-a_{2017}\\right|$. In this version, the answer $\\frac{1}{2018}$ is achieved at $$ a_{1}=a_{2}=\\cdots=a_{2017}=1, \\quad a_{2018}=\\frac{a_{2017}+\\cdots+a_{0}}{2018}=1-\\frac{1}{2018} . $$ To prove that this value is optimal, it suffices to notice that $\\Delta_{2}=\\frac{1}{2}$ and to apply Claim 1 obtaining $$ \\left|a_{2018}-a_{2017}\\right| \\leqslant \\Delta_{2018} \\leqslant \\frac{1}{2} \\cdot \\frac{2}{3} \\cdots \\frac{2017}{2018}=\\frac{1}{2018} . $$","tier":0} +{"problem_type":"Algebra","problem_label":"A4","problem":"Let $a_{0}, a_{1}, a_{2}, \\ldots$ be a sequence of real numbers such that $a_{0}=0, a_{1}=1$, and for every $n \\geqslant 2$ there exists $1 \\leqslant k \\leqslant n$ satisfying $$ a_{n}=\\frac{a_{n-1}+\\cdots+a_{n-k}}{k} $$ Find the maximal possible value of $a_{2018}-a_{2017}$. (Belgium) Answer: The maximal value is $\\frac{2016}{2017^{2}}$.","solution":"We present a different proof of the estimate $a_{2018}-a_{2017} \\leqslant \\frac{2016}{2017^{2}}$. We keep the same notations of $S(n, k), m_{n}$ and $M_{n}$ from the previous solution. Notice that $S(n, n)=S(n, n-1)$, as $a_{0}=0$. Also notice that for $0 \\leqslant k \\leqslant \\ell \\leqslant n$ we have $S(n, \\ell)=S(n, k)+S(n-k, \\ell-k)$. Claim 2. For every positive integer $n$, we have $m_{n} \\leqslant m_{n+1}$ and $M_{n+1} \\leqslant M_{n}$, so the segment $\\left[m_{n+1}, M_{n+1}\\right]$ is contained in $\\left[m_{n}, M_{n}\\right]$. Proof. Choose a positive integer $k \\leqslant n+1$ such that $m_{n+1}=S(n+1, k) \/ k$. Then we have $$ k m_{n+1}=S(n+1, k)=a_{n}+S(n, k-1) \\geqslant m_{n}+(k-1) m_{n}=k m_{n}, $$ which establishes the first inequality in the Claim. The proof of the second inequality is similar. Claim 3. For every positive integers $k \\geqslant n$, we have $m_{n} \\leqslant a_{k} \\leqslant M_{n}$. Proof. By Claim 2, we have $\\left[m_{k}, M_{k}\\right] \\subseteq\\left[m_{k-1}, M_{k-1}\\right] \\subseteq \\cdots \\subseteq\\left[m_{n}, M_{n}\\right]$. Since $a_{k} \\in\\left[m_{k}, M_{k}\\right]$, the claim follows. Claim 4. For every integer $n \\geqslant 2$, we have $M_{n}=S(n, n-1) \/(n-1)$ and $m_{n}=S(n, n) \/ n$. Proof. We use induction on $n$. The base case $n=2$ is routine. To perform the induction step, we need to prove the inequalities $$ \\frac{S(n, n)}{n} \\leqslant \\frac{S(n, k)}{k} \\quad \\text { and } \\quad \\frac{S(n, k)}{k} \\leqslant \\frac{S(n, n-1)}{n-1} $$ for every positive integer $k \\leqslant n$. Clearly, these inequalities hold for $k=n$ and $k=n-1$, as $S(n, n)=S(n, n-1)>0$. In the sequel, we assume that $k0$. (South Korea) Answer: $f(x)=C_{1} x+\\frac{C_{2}}{x}$ with arbitrary constants $C_{1}$ and $C_{2}$.","solution":"Fix a real number $a>1$, and take a new variable $t$. For the values $f(t), f\\left(t^{2}\\right)$, $f(a t)$ and $f\\left(a^{2} t^{2}\\right)$, the relation (1) provides a system of linear equations: $$ \\begin{array}{llll} x=y=t: & \\left(t+\\frac{1}{t}\\right) f(t) & =f\\left(t^{2}\\right)+f(1) \\\\ x=\\frac{t}{a}, y=a t: & \\left(\\frac{t}{a}+\\frac{a}{t}\\right) f(a t) & =f\\left(t^{2}\\right)+f\\left(a^{2}\\right) \\\\ x=a^{2} t, y=t: & \\left(a^{2} t+\\frac{1}{a^{2} t}\\right) f(t) & =f\\left(a^{2} t^{2}\\right)+f\\left(\\frac{1}{a^{2}}\\right) \\\\ x=y=a t: & \\left(a t+\\frac{1}{a t}\\right) f(a t) & =f\\left(a^{2} t^{2}\\right)+f(1) \\end{array} $$ In order to eliminate $f\\left(t^{2}\\right)$, take the difference of (2a) and (2b); from (2c) and (2d) eliminate $f\\left(a^{2} t^{2}\\right)$; then by taking a linear combination, eliminate $f(a t)$ as well: $$ \\begin{gathered} \\left(t+\\frac{1}{t}\\right) f(t)-\\left(\\frac{t}{a}+\\frac{a}{t}\\right) f(a t)=f(1)-f\\left(a^{2}\\right) \\text { and } \\\\ \\left(a^{2} t+\\frac{1}{a^{2} t}\\right) f(t)-\\left(a t+\\frac{1}{a t}\\right) f(a t)=f\\left(1 \/ a^{2}\\right)-f(1), \\text { so } \\\\ \\left(\\left(a t+\\frac{1}{a t}\\right)\\left(t+\\frac{1}{t}\\right)-\\left(\\frac{t}{a}+\\frac{a}{t}\\right)\\left(a^{2} t+\\frac{1}{a^{2} t}\\right)\\right) f(t) \\\\ =\\left(a t+\\frac{1}{a t}\\right)\\left(f(1)-f\\left(a^{2}\\right)\\right)-\\left(\\frac{t}{a}+\\frac{a}{t}\\right)\\left(f\\left(1 \/ a^{2}\\right)-f(1)\\right) . \\end{gathered} $$ Notice that on the left-hand side, the coefficient of $f(t)$ is nonzero and does not depend on $t$ : $$ \\left(a t+\\frac{1}{a t}\\right)\\left(t+\\frac{1}{t}\\right)-\\left(\\frac{t}{a}+\\frac{a}{t}\\right)\\left(a^{2} t+\\frac{1}{a^{2} t}\\right)=a+\\frac{1}{a}-\\left(a^{3}+\\frac{1}{a^{3}}\\right)<0 . $$ After dividing by this fixed number, we get $$ f(t)=C_{1} t+\\frac{C_{2}}{t} $$ where the numbers $C_{1}$ and $C_{2}$ are expressed in terms of $a, f(1), f\\left(a^{2}\\right)$ and $f\\left(1 \/ a^{2}\\right)$, and they do not depend on $t$. The functions of the form (3) satisfy the equation: $$ \\left(x+\\frac{1}{x}\\right) f(y)=\\left(x+\\frac{1}{x}\\right)\\left(C_{1} y+\\frac{C_{2}}{y}\\right)=\\left(C_{1} x y+\\frac{C_{2}}{x y}\\right)+\\left(C_{1} \\frac{y}{x}+C_{2} \\frac{x}{y}\\right)=f(x y)+f\\left(\\frac{y}{x}\\right) . $$","tier":0} +{"problem_type":"Algebra","problem_label":"A5","problem":"Determine all functions $f:(0, \\infty) \\rightarrow \\mathbb{R}$ satisfying $$ \\left(x+\\frac{1}{x}\\right) f(y)=f(x y)+f\\left(\\frac{y}{x}\\right) $$ for all $x, y>0$. (South Korea) Answer: $f(x)=C_{1} x+\\frac{C_{2}}{x}$ with arbitrary constants $C_{1}$ and $C_{2}$.","solution":"We start with an observation. If we substitute $x=a \\neq 1$ and $y=a^{n}$ in (1), we obtain $$ f\\left(a^{n+1}\\right)-\\left(a+\\frac{1}{a}\\right) f\\left(a^{n}\\right)+f\\left(a^{n-1}\\right)=0 . $$ For the sequence $z_{n}=a^{n}$, this is a homogeneous linear recurrence of the second order, and its characteristic polynomial is $t^{2}-\\left(a+\\frac{1}{a}\\right) t+1=(t-a)\\left(t-\\frac{1}{a}\\right)$ with two distinct nonzero roots, namely $a$ and $1 \/ a$. As is well-known, the general solution is $z_{n}=C_{1} a^{n}+C_{2}(1 \/ a)^{n}$ where the index $n$ can be as well positive as negative. Of course, the numbers $C_{1}$ and $C_{2}$ may depend of the choice of $a$, so in fact we have two functions, $C_{1}$ and $C_{2}$, such that $$ f\\left(a^{n}\\right)=C_{1}(a) \\cdot a^{n}+\\frac{C_{2}(a)}{a^{n}} \\quad \\text { for every } a \\neq 1 \\text { and every integer } n \\text {. } $$ The relation (4) can be easily extended to rational values of $n$, so we may conjecture that $C_{1}$ and $C_{2}$ are constants, and whence $f(t)=C_{1} t+\\frac{C_{2}}{t}$. As it was seen in the previous solution, such functions indeed satisfy (1). The equation (1) is linear in $f$; so if some functions $f_{1}$ and $f_{2}$ satisfy (1) and $c_{1}, c_{2}$ are real numbers, then $c_{1} f_{1}(x)+c_{2} f_{2}(x)$ is also a solution of (1). In order to make our formulas simpler, define $$ f_{0}(x)=f(x)-f(1) \\cdot x \\text {. } $$ This function is another one satisfying (1) and the extra constraint $f_{0}(1)=0$. Repeating the same argument on linear recurrences, we can write $f_{0}(a)=K(a) a^{n}+\\frac{L(a)}{a^{n}}$ with some functions $K$ and $L$. By substituting $n=0$, we can see that $K(a)+L(a)=f_{0}(1)=0$ for every $a$. Hence, $$ f_{0}\\left(a^{n}\\right)=K(a)\\left(a^{n}-\\frac{1}{a^{n}}\\right) $$ Now take two numbers $a>b>1$ arbitrarily and substitute $x=(a \/ b)^{n}$ and $y=(a b)^{n}$ in (1): $$ \\begin{aligned} \\left(\\frac{a^{n}}{b^{n}}+\\frac{b^{n}}{a^{n}}\\right) f_{0}\\left((a b)^{n}\\right) & =f_{0}\\left(a^{2 n}\\right)+f_{0}\\left(b^{2 n}\\right), \\quad \\text { so } \\\\ \\left(\\frac{a^{n}}{b^{n}}+\\frac{b^{n}}{a^{n}}\\right) K(a b)\\left((a b)^{n}-\\frac{1}{(a b)^{n}}\\right) & =K(a)\\left(a^{2 n}-\\frac{1}{a^{2 n}}\\right)+K(b)\\left(b^{2 n}-\\frac{1}{b^{2 n}}\\right), \\quad \\text { or equivalently } \\\\ K(a b)\\left(a^{2 n}-\\frac{1}{a^{2 n}}+b^{2 n}-\\frac{1}{b^{2 n}}\\right) & =K(a)\\left(a^{2 n}-\\frac{1}{a^{2 n}}\\right)+K(b)\\left(b^{2 n}-\\frac{1}{b^{2 n}}\\right) . \\end{aligned} $$ By dividing (5) by $a^{2 n}$ and then taking limit with $n \\rightarrow+\\infty$ we get $K(a b)=K(a)$. Then (5) reduces to $K(a)=K(b)$. Hence, $K(a)=K(b)$ for all $a>b>1$. Fix $a>1$. For every $x>0$ there is some $b$ and an integer $n$ such that $10$ and $2\\left(k^{2}+1\\right)$, respectively; in this case, the answer becomes $$ 2 \\sqrt[3]{\\frac{(k+1)^{2}}{k}} $$ Even further, a linear substitution allows to extend the solutions to a version with 7 and 100 being replaced with arbitrary positive real numbers $p$ and $q$ satisfying $q \\geqslant 4 p$.","tier":0} +{"problem_type":"Combinatorics","problem_label":"C2","problem":"Queenie and Horst play a game on a $20 \\times 20$ chessboard. In the beginning the board is empty. In every turn, Horst places a black knight on an empty square in such a way that his new knight does not attack any previous knights. Then Queenie places a white queen on an empty square. The game gets finished when somebody cannot move. Find the maximal positive $K$ such that, regardless of the strategy of Queenie, Horst can put at least $K$ knights on the board. (Armenia) Answer: $K=20^{2} \/ 4=100$. In case of a $4 N \\times 4 M$ board, the answer is $K=4 N M$.","solution":"We show two strategies, one for Horst to place at least 100 knights, and another strategy for Queenie that prevents Horst from putting more than 100 knights on the board. A strategy for Horst: Put knights only on black squares, until all black squares get occupied. Colour the squares of the board black and white in the usual way, such that the white and black squares alternate, and let Horst put his knights on black squares as long as it is possible. Two knights on squares of the same colour never attack each other. The number of black squares is $20^{2} \/ 2=200$. The two players occupy the squares in turn, so Horst will surely find empty black squares in his first 100 steps. A strategy for Queenie: Group the squares into cycles of length 4, and after each step of Horst, occupy the opposite square in the same cycle. Consider the squares of the board as vertices of a graph; let two squares be connected if two knights on those squares would attack each other. Notice that in a $4 \\times 4$ board the squares can be grouped into 4 cycles of length 4 , as shown in Figure 1. Divide the board into parts of size $4 \\times 4$, and perform the same grouping in every part; this way we arrange the 400 squares of the board into 100 cycles (Figure 2). ![](https:\/\/cdn.mathpix.com\/cropped\/2024_04_17_6da854f21230b13cd732g-27.jpg?height=360&width=351&top_left_y=1579&top_left_x=310) Figure 1 ![](https:\/\/cdn.mathpix.com\/cropped\/2024_04_17_6da854f21230b13cd732g-27.jpg?height=362&width=420&top_left_y=1578&top_left_x=772) Figure 2 ![](https:\/\/cdn.mathpix.com\/cropped\/2024_04_17_6da854f21230b13cd732g-27.jpg?height=357&width=463&top_left_y=1578&top_left_x=1296) Figure 3 The strategy of Queenie can be as follows: Whenever Horst puts a new knight to a certain square $A$, which is part of some cycle $A-B-C-D-A$, let Queenie put her queen on the opposite square $C$ in that cycle (Figure 3). From this point, Horst cannot put any knight on $A$ or $C$ because those squares are already occupied, neither on $B$ or $D$ because those squares are attacked by the knight standing on $A$. Hence, Horst can put at most one knight on each cycle, that is at most 100 knights in total. Comment 1. Queenie's strategy can be prescribed by a simple rule: divide the board into $4 \\times 4$ parts; whenever Horst puts a knight in a part $P$, Queenie reflects that square about the centre of $P$ and puts her queen on the reflected square. Comment 2. The result remains the same if Queenie moves first. In the first turn, she may put her first queen arbitrarily. Later, if she has to put her next queen on a square that already contains a queen, she may move arbitrarily again.","tier":0} +{"problem_type":"Combinatorics","problem_label":"C4","problem":"An anti-Pascal pyramid is a finite set of numbers, placed in a triangle-shaped array so that the first row of the array contains one number, the second row contains two numbers, the third row contains three numbers and so on; and, except for the numbers in the bottom row, each number equals the absolute value of the difference of the two numbers below it. For instance, the triangle below is an anti-Pascal pyramid with four rows, in which every integer from 1 to $1+2+3+4=10$ occurs exactly once: $$ \\begin{array}{ccccc} & & & \\\\ & 2 & 6 & \\\\ & 5 & 7 & \\\\ 8 & 3 & 10 & 9 . \\end{array} $$ Is it possible to form an anti-Pascal pyramid with 2018 rows, using every integer from 1 to $1+2+\\cdots+2018$ exactly once? (Iran) Answer: No, it is not possible.","solution":"Let $T$ be an anti-Pascal pyramid with $n$ rows, containing every integer from 1 to $1+2+\\cdots+n$, and let $a_{1}$ be the topmost number in $T$ (Figure 1). The two numbers below $a_{1}$ are some $a_{2}$ and $b_{2}=a_{1}+a_{2}$, the two numbers below $b_{2}$ are some $a_{3}$ and $b_{3}=a_{1}+a_{2}+a_{3}$, and so on and so forth all the way down to the bottom row, where some $a_{n}$ and $b_{n}=a_{1}+a_{2}+\\cdots+a_{n}$ are the two neighbours below $b_{n-1}=a_{1}+a_{2}+\\cdots+a_{n-1}$. Since the $a_{k}$ are $n$ pairwise distinct positive integers whose sum does not exceed the largest number in $T$, which is $1+2+\\cdots+n$, it follows that they form a permutation of $1,2, \\ldots, n$. ![](https:\/\/cdn.mathpix.com\/cropped\/2024_04_17_6da854f21230b13cd732g-29.jpg?height=391&width=465&top_left_y=1415&top_left_x=430) Figure 1 ![](https:\/\/cdn.mathpix.com\/cropped\/2024_04_17_6da854f21230b13cd732g-29.jpg?height=394&width=463&top_left_y=1411&top_left_x=1162) Figure 2 Consider now (Figure 2) the two 'equilateral' subtriangles of $T$ whose bottom rows contain the numbers to the left, respectively right, of the pair $a_{n}, b_{n}$. (One of these subtriangles may very well be empty.) At least one of these subtriangles, say $T^{\\prime}$, has side length $\\ell \\geqslant\\lceil(n-2) \/ 2\\rceil$. Since $T^{\\prime}$ obeys the anti-Pascal rule, it contains $\\ell$ pairwise distinct positive integers $a_{1}^{\\prime}, a_{2}^{\\prime}, \\ldots, a_{\\ell}^{\\prime}$, where $a_{1}^{\\prime}$ is at the apex, and $a_{k}^{\\prime}$ and $b_{k}^{\\prime}=a_{1}^{\\prime}+a_{2}^{\\prime}+\\cdots+a_{k}^{\\prime}$ are the two neighbours below $b_{k-1}^{\\prime}$ for each $k=2,3 \\ldots, \\ell$. Since the $a_{k}$ all lie outside $T^{\\prime}$, and they form a permutation of $1,2, \\ldots, n$, the $a_{k}^{\\prime}$ are all greater than $n$. Consequently, $$ \\begin{array}{r} b_{\\ell}^{\\prime} \\geqslant(n+1)+(n+2)+\\cdots+(n+\\ell)=\\frac{\\ell(2 n+\\ell+1)}{2} \\\\ \\geqslant \\frac{1}{2} \\cdot \\frac{n-2}{2}\\left(2 n+\\frac{n-2}{2}+1\\right)=\\frac{5 n(n-2)}{8}, \\end{array} $$ which is greater than $1+2+\\cdots+n=n(n+1) \/ 2$ for $n=2018$. A contradiction. Comment. The above estimate may be slightly improved by noticing that $b_{\\ell}^{\\prime} \\neq b_{n}$. This implies $n(n+1) \/ 2=b_{n}>b_{\\ell}^{\\prime} \\geqslant\\lceil(n-2) \/ 2\\rceil(2 n+\\lceil(n-2) \/ 2\\rceil+1) \/ 2$, so $n \\leqslant 7$ if $n$ is odd, and $n \\leqslant 12$ if $n$ is even. It seems that the largest anti-Pascal pyramid whose entries are a permutation of the integers from 1 to $1+2+\\cdots+n$ has 5 rows.","tier":0} +{"problem_type":"Combinatorics","problem_label":"C5","problem":"Let $k$ be a positive integer. The organising committee of a tennis tournament is to schedule the matches for $2 k$ players so that every two players play once, each day exactly one match is played, and each player arrives to the tournament site the day of his first match, and departs the day of his last match. For every day a player is present on the tournament, the committee has to pay 1 coin to the hotel. The organisers want to design the schedule so as to minimise the total cost of all players' stays. Determine this minimum cost. (Russia) Answer: The required minimum is $k\\left(4 k^{2}+k-1\\right) \/ 2$.","solution":"Enumerate the days of the tournament $1,2, \\ldots,\\left(\\begin{array}{c}2 k \\\\ 2\\end{array}\\right)$. Let $b_{1} \\leqslant b_{2} \\leqslant \\cdots \\leqslant b_{2 k}$ be the days the players arrive to the tournament, arranged in nondecreasing order; similarly, let $e_{1} \\geqslant \\cdots \\geqslant e_{2 k}$ be the days they depart arranged in nonincreasing order (it may happen that a player arrives on day $b_{i}$ and departs on day $e_{j}$, where $i \\neq j$ ). If a player arrives on day $b$ and departs on day $e$, then his stay cost is $e-b+1$. Therefore, the total stay cost is $$ \\Sigma=\\sum_{i=1}^{2 k} e_{i}-\\sum_{i=1}^{2 k} b_{i}+n=\\sum_{i=1}^{2 k}\\left(e_{i}-b_{i}+1\\right) $$ Bounding the total cost from below. To this end, estimate $e_{i+1}-b_{i+1}+1$. Before day $b_{i+1}$, only $i$ players were present, so at most $\\left(\\begin{array}{l}i \\\\ 2\\end{array}\\right)$ matches could be played. Therefore, $b_{i+1} \\leqslant\\left(\\begin{array}{l}i \\\\ 2\\end{array}\\right)+1$. Similarly, at most $\\left(\\begin{array}{l}i \\\\ 2\\end{array}\\right)$ matches could be played after day $e_{i+1}$, so $e_{i} \\geqslant\\left(\\begin{array}{c}2 k \\\\ 2\\end{array}\\right)-\\left(\\begin{array}{l}i \\\\ 2\\end{array}\\right)$. Thus, $$ e_{i+1}-b_{i+1}+1 \\geqslant\\left(\\begin{array}{c} 2 k \\\\ 2 \\end{array}\\right)-2\\left(\\begin{array}{l} i \\\\ 2 \\end{array}\\right)=k(2 k-1)-i(i-1) $$ This lower bound can be improved for $i>k$ : List the $i$ players who arrived first, and the $i$ players who departed last; at least $2 i-2 k$ players appear in both lists. The matches between these players were counted twice, though the players in each pair have played only once. Therefore, if $i>k$, then $$ e_{i+1}-b_{i+1}+1 \\geqslant\\left(\\begin{array}{c} 2 k \\\\ 2 \\end{array}\\right)-2\\left(\\begin{array}{c} i \\\\ 2 \\end{array}\\right)+\\left(\\begin{array}{c} 2 i-2 k \\\\ 2 \\end{array}\\right)=(2 k-i)^{2} $$ An optimal tournament, We now describe a schedule in which the lower bounds above are all achieved simultaneously. Split players into two groups $X$ and $Y$, each of cardinality $k$. Next, partition the schedule into three parts. During the first part, the players from $X$ arrive one by one, and each newly arrived player immediately plays with everyone already present. During the third part (after all players from $X$ have already departed) the players from $Y$ depart one by one, each playing with everyone still present just before departing. In the middle part, everyone from $X$ should play with everyone from $Y$. Let $S_{1}, S_{2}, \\ldots, S_{k}$ be the players in $X$, and let $T_{1}, T_{2}, \\ldots, T_{k}$ be the players in $Y$. Let $T_{1}, T_{2}, \\ldots, T_{k}$ arrive in this order; after $T_{j}$ arrives, he immediately plays with all the $S_{i}, i>j$. Afterwards, players $S_{k}$, $S_{k-1}, \\ldots, S_{1}$ depart in this order; each $S_{i}$ plays with all the $T_{j}, i \\leqslant j$, just before his departure, and $S_{k}$ departs the day $T_{k}$ arrives. For $0 \\leqslant s \\leqslant k-1$, the number of matches played between $T_{k-s}$ 's arrival and $S_{k-s}$ 's departure is $$ \\sum_{j=k-s}^{k-1}(k-j)+1+\\sum_{j=k-s}^{k-1}(k-j+1)=\\frac{1}{2} s(s+1)+1+\\frac{1}{2} s(s+3)=(s+1)^{2} $$ Thus, if $i>k$, then the number of matches that have been played between $T_{i-k+1}$ 's arrival, which is $b_{i+1}$, and $S_{i-k+1}$ 's departure, which is $e_{i+1}$, is $(2 k-i)^{2}$; that is, $e_{i+1}-b_{i+1}+1=(2 k-i)^{2}$, showing the second lower bound achieved for all $i>k$. If $i \\leqslant k$, then the matches between the $i$ players present before $b_{i+1}$ all fall in the first part of the schedule, so there are $\\left(\\begin{array}{l}i \\\\ 2\\end{array}\\right)$ such, and $b_{i+1}=\\left(\\begin{array}{l}i \\\\ 2\\end{array}\\right)+1$. Similarly, after $e_{i+1}$, there are $i$ players left, all $\\left(\\begin{array}{l}i \\\\ 2\\end{array}\\right)$ matches now fall in the third part of the schedule, and $e_{i+1}=\\left(\\begin{array}{c}2 k \\\\ 2\\end{array}\\right)-\\left(\\begin{array}{c}i \\\\ 2\\end{array}\\right)$. The first lower bound is therefore also achieved for all $i \\leqslant k$. Consequently, all lower bounds are achieved simultaneously, and the schedule is indeed optimal. Evaluation. Finally, evaluate the total cost for the optimal schedule: $$ \\begin{aligned} \\Sigma & =\\sum_{i=0}^{k}(k(2 k-1)-i(i-1))+\\sum_{i=k+1}^{2 k-1}(2 k-i)^{2}=(k+1) k(2 k-1)-\\sum_{i=0}^{k} i(i-1)+\\sum_{j=1}^{k-1} j^{2} \\\\ & =k(k+1)(2 k-1)-k^{2}+\\frac{1}{2} k(k+1)=\\frac{1}{2} k\\left(4 k^{2}+k-1\\right) . \\end{aligned} $$","tier":0} +{"problem_type":"Combinatorics","problem_label":"C5","problem":"Let $k$ be a positive integer. The organising committee of a tennis tournament is to schedule the matches for $2 k$ players so that every two players play once, each day exactly one match is played, and each player arrives to the tournament site the day of his first match, and departs the day of his last match. For every day a player is present on the tournament, the committee has to pay 1 coin to the hotel. The organisers want to design the schedule so as to minimise the total cost of all players' stays. Determine this minimum cost. (Russia) Answer: The required minimum is $k\\left(4 k^{2}+k-1\\right) \/ 2$.","solution":"Consider any tournament schedule. Label players $P_{1}, P_{2}, \\ldots, P_{2 k}$ in order of their arrival, and label them again $Q_{2 k}, Q_{2 k-1}, \\ldots, Q_{1}$ in order of their departure, to define a permutation $a_{1}, a_{2}, \\ldots, a_{2 k}$ of $1,2, \\ldots, 2 k$ by $P_{i}=Q_{a_{i}}$. We first describe an optimal tournament for any given permutation $a_{1}, a_{2}, \\ldots, a_{2 k}$ of the indices $1,2, \\ldots, 2 k$. Next, we find an optimal permutation and an optimal tournament. Optimisation for a fixed $a_{1}, \\ldots, a_{2 k}$. We say that the cost of the match between $P_{i}$ and $P_{j}$ is the number of players present at the tournament when this match is played. Clearly, the Committee pays for each day the cost of the match of that day. Hence, we are to minimise the total cost of all matches. Notice that $Q_{2 k}$ 's departure does not precede $P_{2 k}$ 's arrival. Hence, the number of players at the tournament monotonically increases (non-strictly) until it reaches $2 k$, and then monotonically decreases (non-strictly). So, the best time to schedule the match between $P_{i}$ and $P_{j}$ is either when $P_{\\max (i, j)}$ arrives, or when $Q_{\\max \\left(a_{i}, a_{j}\\right)}$ departs, in which case the cost is $\\min \\left(\\max (i, j), \\max \\left(a_{i}, a_{j}\\right)\\right)$. Conversely, assuming that $i>j$, if this match is scheduled between the arrivals of $P_{i}$ and $P_{i+1}$, then its cost will be exactly $i=\\max (i, j)$. Similarly, one can make it cost $\\max \\left(a_{i}, a_{j}\\right)$. Obviously, these conditions can all be simultaneously satisfied, so the minimal cost for a fixed sequence $a_{1}, a_{2}, \\ldots, a_{2 k}$ is $$ \\Sigma\\left(a_{1}, \\ldots, a_{2 k}\\right)=\\sum_{1 \\leqslant i4$, one can choose a point $A_{n+1}$ on the small arc $\\widehat{C A_{n}}$, close enough to $C$, so that $A_{n} B+A_{n} A_{n+1}+B A_{n+1}$ is still greater than 4 . Thus each of these new triangles $B A_{n} A_{n+1}$ and $B A_{n+1} C$ has perimeter greater than 4, which completes the induction step. ![](https:\/\/cdn.mathpix.com\/cropped\/2024_04_17_6da854f21230b13cd732g-42.jpg?height=366&width=691&top_left_y=1353&top_left_x=682) We proceed by showing that no $t>4$ satisfies the conditions of the problem. To this end, we assume that there exists a good collection $T$ of $n$ triangles, each of perimeter greater than $t$, and then bound $n$ from above. Take $\\varepsilon>0$ such that $t=4+2 \\varepsilon$. Claim. There exists a positive constant $\\sigma=\\sigma(\\varepsilon)$ such that any triangle $\\Delta$ with perimeter $2 s \\geqslant 4+2 \\varepsilon$, inscribed in $\\omega$, has area $S(\\Delta)$ at least $\\sigma$. Proof. Let $a, b, c$ be the side lengths of $\\Delta$. Since $\\Delta$ is inscribed in $\\omega$, each side has length at most 2. Therefore, $s-a \\geqslant(2+\\varepsilon)-2=\\varepsilon$. Similarly, $s-b \\geqslant \\varepsilon$ and $s-c \\geqslant \\varepsilon$. By Heron's formula, $S(\\Delta)=\\sqrt{s(s-a)(s-b)(s-c)} \\geqslant \\sqrt{(2+\\varepsilon) \\varepsilon^{3}}$. Thus we can set $\\sigma(\\varepsilon)=\\sqrt{(2+\\varepsilon) \\varepsilon^{3}}$. Now we see that the total area $S$ of all triangles from $T$ is at least $n \\sigma(\\varepsilon)$. On the other hand, $S$ does not exceed the area of the disk bounded by $\\omega$. Thus $n \\sigma(\\varepsilon) \\leqslant \\pi$, which means that $n$ is bounded from above. Comment 1. One may prove the Claim using the formula $S=\\frac{a b c}{4 R}$ instead of Heron's formula. Comment 2. In the statement of the problem condition $(i)$ could be replaced by a weaker one: each triangle from $T$ lies within $\\omega$. This does not affect the solution above, but reduces the number of ways to prove the Claim. This page is intentionally left blank","tier":0} +{"problem_type":"Geometry","problem_label":"G5","problem":"Let $A B C$ be a triangle with circumcircle $\\omega$ and incentre $I$. A line $\\ell$ intersects the lines $A I, B I$, and $C I$ at points $D, E$, and $F$, respectively, distinct from the points $A, B, C$, and $I$. The perpendicular bisectors $x, y$, and $z$ of the segments $A D, B E$, and $C F$, respectively determine a triangle $\\Theta$. Show that the circumcircle of the triangle $\\Theta$ is tangent to $\\omega$. (Denmark) Preamble. Let $X=y \\cap z, Y=x \\cap z, Z=x \\cap y$ and let $\\Omega$ denote the circumcircle of the triangle $X Y Z$. Denote by $X_{0}, Y_{0}$, and $Z_{0}$ the second intersection points of $A I, B I$ and $C I$, respectively, with $\\omega$. It is known that $Y_{0} Z_{0}$ is the perpendicular bisector of $A I, Z_{0} X_{0}$ is the perpendicular bisector of $B I$, and $X_{0} Y_{0}$ is the perpendicular bisector of $C I$. In particular, the triangles $X Y Z$ and $X_{0} Y_{0} Z_{0}$ are homothetic, because their corresponding sides are parallel. The solutions below mostly exploit the following approach. Consider the triangles $X Y Z$ and $X_{0} Y_{0} Z_{0}$, or some other pair of homothetic triangles $\\Delta$ and $\\delta$ inscribed into $\\Omega$ and $\\omega$, respectively. In order to prove that $\\Omega$ and $\\omega$ are tangent, it suffices to show that the centre $T$ of the homothety taking $\\Delta$ to $\\delta$ lies on $\\omega$ (or $\\Omega$ ), or, in other words, to show that $\\Delta$ and $\\delta$ are perspective (i.e., the lines joining corresponding vertices are concurrent), with their perspector lying on $\\omega$ (or $\\Omega$ ). We use directed angles throughout all the solutions.","solution":"Claim 1. The reflections $\\ell_{a}, \\ell_{b}$ and $\\ell_{c}$ of the line $\\ell$ in the lines $x, y$, and $z$, respectively, are concurrent at a point $T$ which belongs to $\\omega$. ![](https:\/\/cdn.mathpix.com\/cropped\/2024_04_17_6da854f21230b13cd732g-46.jpg?height=831&width=916&top_left_y=1338&top_left_x=570) Proof. Notice that $\\Varangle\\left(\\ell_{b}, \\ell_{c}\\right)=\\Varangle\\left(\\ell_{b}, \\ell\\right)+\\Varangle\\left(\\ell, \\ell_{c}\\right)=2 \\Varangle(y, \\ell)+2 \\Varangle(\\ell, z)=2 \\Varangle(y, z)$. But $y \\perp B I$ and $z \\perp C I$ implies $\\Varangle(y, z)=\\Varangle(B I, I C)$, so, since $2 \\Varangle(B I, I C)=\\Varangle(B A, A C)$, we obtain $$ \\Varangle\\left(\\ell_{b}, \\ell_{c}\\right)=\\Varangle(B A, A C) . $$ Since $A$ is the reflection of $D$ in $x$, $A$ belongs to $\\ell_{a}$; similarly, $B$ belongs to $\\ell_{b}$. Then (1) shows that the common point $T^{\\prime}$ of $\\ell_{a}$ and $\\ell_{b}$ lies on $\\omega$; similarly, the common point $T^{\\prime \\prime}$ of $\\ell_{c}$ and $\\ell_{b}$ lies on $\\omega$. If $B \\notin \\ell_{a}$ and $B \\notin \\ell_{c}$, then $T^{\\prime}$ and $T^{\\prime \\prime}$ are the second point of intersection of $\\ell_{b}$ and $\\omega$, hence they coincide. Otherwise, if, say, $B \\in \\ell_{c}$, then $\\ell_{c}=B C$, so $\\Varangle(B A, A C)=\\Varangle\\left(\\ell_{b}, \\ell_{c}\\right)=\\Varangle\\left(\\ell_{b}, B C\\right)$, which shows that $\\ell_{b}$ is tangent at $B$ to $\\omega$ and $T^{\\prime}=T^{\\prime \\prime}=B$. So $T^{\\prime}$ and $T^{\\prime \\prime}$ coincide in all the cases, and the conclusion of the claim follows. Now we prove that $X, X_{0}, T$ are collinear. Denote by $D_{b}$ and $D_{c}$ the reflections of the point $D$ in the lines $y$ and $z$, respectively. Then $D_{b}$ lies on $\\ell_{b}, D_{c}$ lies on $\\ell_{c}$, and $$ \\begin{aligned} \\Varangle\\left(D_{b} X, X D_{c}\\right) & =\\Varangle\\left(D_{b} X, D X\\right)+\\Varangle\\left(D X, X D_{c}\\right)=2 \\Varangle(y, D X)+2 \\Varangle(D X, z)=2 \\Varangle(y, z) \\\\ & =\\Varangle(B A, A C)=\\Varangle(B T, T C), \\end{aligned} $$ hence the quadrilateral $X D_{b} T D_{c}$ is cyclic. Notice also that since $X D_{b}=X D=X D_{c}$, the points $D, D_{b}, D_{c}$ lie on a circle with centre $X$. Using in this circle the diameter $D_{c} D_{c}^{\\prime}$ yields $\\Varangle\\left(D_{b} D_{c}, D_{c} X\\right)=90^{\\circ}+\\Varangle\\left(D_{b} D_{c}^{\\prime}, D_{c}^{\\prime} X\\right)=90^{\\circ}+\\Varangle\\left(D_{b} D, D D_{c}\\right)$. Therefore, $$ \\begin{gathered} \\Varangle\\left(\\ell_{b}, X T\\right)=\\Varangle\\left(D_{b} T, X T\\right)=\\Varangle\\left(D_{b} D_{c}, D_{c} X\\right)=90^{\\circ}+\\Varangle\\left(D_{b} D, D D_{c}\\right) \\\\ =90^{\\circ}+\\Varangle(B I, I C)=\\Varangle(B A, A I)=\\Varangle\\left(B A, A X_{0}\\right)=\\Varangle\\left(B T, T X_{0}\\right)=\\Varangle\\left(\\ell_{b}, X_{0} T\\right) \\end{gathered} $$ so the points $X, X_{0}, T$ are collinear. By a similar argument, $Y, Y_{0}, T$ and $Z, Z_{0}, T$ are collinear. As mentioned in the preamble, the statement of the problem follows. Comment 1. After proving Claim 1 one may proceed in another way. As it was shown, the reflections of $\\ell$ in the sidelines of $X Y Z$ are concurrent at $T$. Thus $\\ell$ is the Steiner line of $T$ with respect to $\\triangle X Y Z$ (that is the line containing the reflections $T_{a}, T_{b}, T_{c}$ of $T$ in the sidelines of $X Y Z$ ). The properties of the Steiner line imply that $T$ lies on $\\Omega$, and $\\ell$ passes through the orthocentre $H$ of the triangle $X Y Z$. ![](https:\/\/cdn.mathpix.com\/cropped\/2024_04_17_6da854f21230b13cd732g-47.jpg?height=811&width=1311&top_left_y=1248&top_left_x=378) Let $H_{a}, H_{b}$, and $H_{c}$ be the reflections of the point $H$ in the lines $x, y$, and $z$, respectively. Then the triangle $H_{a} H_{b} H_{c}$ is inscribed in $\\Omega$ and homothetic to $A B C$ (by an easy angle chasing). Since $H_{a} \\in \\ell_{a}, H_{b} \\in \\ell_{b}$, and $H_{c} \\in \\ell_{c}$, the triangles $H_{a} H_{b} H_{c}$ and $A B C$ form a required pair of triangles $\\Delta$ and $\\delta$ mentioned in the preamble. Comment 2. The following observation shows how one may guess the description of the tangency point $T$ from Solution 1. Let us fix a direction and move the line $\\ell$ parallel to this direction with constant speed. Then the points $D, E$, and $F$ are moving with constant speeds along the lines $A I, B I$, and $C I$, respectively. In this case $x, y$, and $z$ are moving with constant speeds, defining a family of homothetic triangles $X Y Z$ with a common centre of homothety $T$. Notice that the triangle $X_{0} Y_{0} Z_{0}$ belongs to this family (for $\\ell$ passing through $I$ ). We may specify the location of $T$ considering the degenerate case when $x, y$, and $z$ are concurrent. In this degenerate case all the lines $x, y, z, \\ell, \\ell_{a}, \\ell_{b}, \\ell_{c}$ have a common point. Note that the lines $\\ell_{a}, \\ell_{b}, \\ell_{c}$ remain constant as $\\ell$ is moving (keeping its direction). Thus $T$ should be the common point of $\\ell_{a}, \\ell_{b}$, and $\\ell_{c}$, lying on $\\omega$.","tier":0} +{"problem_type":"Geometry","problem_label":"G5","problem":"Let $A B C$ be a triangle with circumcircle $\\omega$ and incentre $I$. A line $\\ell$ intersects the lines $A I, B I$, and $C I$ at points $D, E$, and $F$, respectively, distinct from the points $A, B, C$, and $I$. The perpendicular bisectors $x, y$, and $z$ of the segments $A D, B E$, and $C F$, respectively determine a triangle $\\Theta$. Show that the circumcircle of the triangle $\\Theta$ is tangent to $\\omega$. (Denmark) Preamble. Let $X=y \\cap z, Y=x \\cap z, Z=x \\cap y$ and let $\\Omega$ denote the circumcircle of the triangle $X Y Z$. Denote by $X_{0}, Y_{0}$, and $Z_{0}$ the second intersection points of $A I, B I$ and $C I$, respectively, with $\\omega$. It is known that $Y_{0} Z_{0}$ is the perpendicular bisector of $A I, Z_{0} X_{0}$ is the perpendicular bisector of $B I$, and $X_{0} Y_{0}$ is the perpendicular bisector of $C I$. In particular, the triangles $X Y Z$ and $X_{0} Y_{0} Z_{0}$ are homothetic, because their corresponding sides are parallel. The solutions below mostly exploit the following approach. Consider the triangles $X Y Z$ and $X_{0} Y_{0} Z_{0}$, or some other pair of homothetic triangles $\\Delta$ and $\\delta$ inscribed into $\\Omega$ and $\\omega$, respectively. In order to prove that $\\Omega$ and $\\omega$ are tangent, it suffices to show that the centre $T$ of the homothety taking $\\Delta$ to $\\delta$ lies on $\\omega$ (or $\\Omega$ ), or, in other words, to show that $\\Delta$ and $\\delta$ are perspective (i.e., the lines joining corresponding vertices are concurrent), with their perspector lying on $\\omega$ (or $\\Omega$ ). We use directed angles throughout all the solutions.","solution":"As mentioned in the preamble, it is sufficient to prove that the centre $T$ of the homothety taking $X Y Z$ to $X_{0} Y_{0} Z_{0}$ belongs to $\\omega$. Thus, it suffices to prove that $\\Varangle\\left(T X_{0}, T Y_{0}\\right)=$ $\\Varangle\\left(Z_{0} X_{0}, Z_{0} Y_{0}\\right)$, or, equivalently, $\\Varangle\\left(X X_{0}, Y Y_{0}\\right)=\\Varangle\\left(Z_{0} X_{0}, Z_{0} Y_{0}\\right)$. Recall that $Y Z$ and $Y_{0} Z_{0}$ are the perpendicular bisectors of $A D$ and $A I$, respectively. Then, the vector $\\vec{x}$ perpendicular to $Y Z$ and shifting the line $Y_{0} Z_{0}$ to $Y Z$ is equal to $\\frac{1}{2} \\overrightarrow{I D}$. Define the shifting vectors $\\vec{y}=\\frac{1}{2} \\overrightarrow{I E}, \\vec{z}=\\frac{1}{2} \\overrightarrow{I F}$ similarly. Consider now the triangle $U V W$ formed by the perpendiculars to $A I, B I$, and $C I$ through $D, E$, and $F$, respectively (see figure below). This is another triangle whose sides are parallel to the corresponding sides of $X Y Z$. Claim 2. $\\overrightarrow{I U}=2 \\overrightarrow{X_{0} X}, \\overrightarrow{I V}=2 \\overrightarrow{Y_{0} Y}, \\overrightarrow{I W}=2 \\overrightarrow{Z_{0} Z}$. Proof. We prove one of the relations, the other proofs being similar. To prove the equality of two vectors it suffices to project them onto two non-parallel axes and check that their projections are equal. The projection of $\\overrightarrow{X_{0} X}$ onto $I B$ equals $\\vec{y}$, while the projection of $\\overrightarrow{I U}$ onto $I B$ is $\\overrightarrow{I E}=2 \\vec{y}$. The projections onto the other axis $I C$ are $\\vec{z}$ and $\\overrightarrow{I F}=2 \\vec{z}$. Then $\\overrightarrow{I U}=2 \\overrightarrow{X_{0} X}$ follows. Notice that the line $\\ell$ is the Simson line of the point $I$ with respect to the triangle $U V W$; thus $U, V, W$, and $I$ are concyclic. It follows from Claim 2 that $\\Varangle\\left(X X_{0}, Y Y_{0}\\right)=\\Varangle(I U, I V)=$ $\\Varangle(W U, W V)=\\Varangle\\left(Z_{0} X_{0}, Z_{0} Y_{0}\\right)$, and we are done. ![](https:\/\/cdn.mathpix.com\/cropped\/2024_04_17_6da854f21230b13cd732g-48.jpg?height=888&width=1106&top_left_y=1161&top_left_x=475)","tier":0} +{"problem_type":"Geometry","problem_label":"G5","problem":"Let $A B C$ be a triangle with circumcircle $\\omega$ and incentre $I$. A line $\\ell$ intersects the lines $A I, B I$, and $C I$ at points $D, E$, and $F$, respectively, distinct from the points $A, B, C$, and $I$. The perpendicular bisectors $x, y$, and $z$ of the segments $A D, B E$, and $C F$, respectively determine a triangle $\\Theta$. Show that the circumcircle of the triangle $\\Theta$ is tangent to $\\omega$. (Denmark) Preamble. Let $X=y \\cap z, Y=x \\cap z, Z=x \\cap y$ and let $\\Omega$ denote the circumcircle of the triangle $X Y Z$. Denote by $X_{0}, Y_{0}$, and $Z_{0}$ the second intersection points of $A I, B I$ and $C I$, respectively, with $\\omega$. It is known that $Y_{0} Z_{0}$ is the perpendicular bisector of $A I, Z_{0} X_{0}$ is the perpendicular bisector of $B I$, and $X_{0} Y_{0}$ is the perpendicular bisector of $C I$. In particular, the triangles $X Y Z$ and $X_{0} Y_{0} Z_{0}$ are homothetic, because their corresponding sides are parallel. The solutions below mostly exploit the following approach. Consider the triangles $X Y Z$ and $X_{0} Y_{0} Z_{0}$, or some other pair of homothetic triangles $\\Delta$ and $\\delta$ inscribed into $\\Omega$ and $\\omega$, respectively. In order to prove that $\\Omega$ and $\\omega$ are tangent, it suffices to show that the centre $T$ of the homothety taking $\\Delta$ to $\\delta$ lies on $\\omega$ (or $\\Omega$ ), or, in other words, to show that $\\Delta$ and $\\delta$ are perspective (i.e., the lines joining corresponding vertices are concurrent), with their perspector lying on $\\omega$ (or $\\Omega$ ). We use directed angles throughout all the solutions.","solution":"Let $I_{a}, I_{b}$, and $I_{c}$ be the excentres of triangle $A B C$ corresponding to $A, B$, and $C$, respectively. Also, let $u, v$, and $w$ be the lines through $D, E$, and $F$ which are perpendicular to $A I, B I$, and $C I$, respectively, and let $U V W$ be the triangle determined by these lines, where $u=V W, v=U W$ and $w=U V$ (see figure above). Notice that the line $u$ is the reflection of $I_{b} I_{c}$ in the line $x$, because $u, x$, and $I_{b} I_{c}$ are perpendicular to $A D$ and $x$ is the perpendicular bisector of $A D$. Likewise, $v$ and $I_{a} I_{c}$ are reflections of each other in $y$, while $w$ and $I_{a} I_{b}$ are reflections of each other in $z$. It follows that $X, Y$, and $Z$ are the midpoints of $U I_{a}, V I_{b}$ and $W I_{c}$, respectively, and that the triangles $U V W$, $X Y Z$ and $I_{a} I_{b} I_{c}$ are either translates of each other or homothetic with a common homothety centre. Construct the points $T$ and $S$ such that the quadrilaterals $U V I W, X Y T Z$ and $I_{a} I_{b} S I_{c}$ are homothetic. Then $T$ is the midpoint of $I S$. Moreover, note that $\\ell$ is the Simson line of the point $I$ with respect to the triangle $U V W$, hence $I$ belongs to the circumcircle of the triangle $U V W$, therefore $T$ belongs to $\\Omega$. Consider now the homothety or translation $h_{1}$ that maps $X Y Z T$ to $I_{a} I_{b} I_{c} S$ and the homothety $h_{2}$ with centre $I$ and factor $\\frac{1}{2}$. Furthermore, let $h=h_{2} \\circ h_{1}$. The transform $h$ can be a homothety or a translation, and $$ h(T)=h_{2}\\left(h_{1}(T)\\right)=h_{2}(S)=T $$ hence $T$ is a fixed point of $h$. So, $h$ is a homothety with centre $T$. Note that $h_{2}$ maps the excentres $I_{a}, I_{b}, I_{c}$ to $X_{0}, Y_{0}, Z_{0}$ defined in the preamble. Thus the centre $T$ of the homothety taking $X Y Z$ to $X_{0} Y_{0} Z_{0}$ belongs to $\\Omega$, and this completes the proof.","tier":0} +{"problem_type":"Number Theory","problem_label":"N1","problem":"Determine all pairs $(n, k)$ of distinct positive integers such that there exists a positive integer $s$ for which the numbers of divisors of $s n$ and of $s k$ are equal. (Ukraine) Answer: All pairs $(n, k)$ such that $n \\nmid k$ and $k \\nmid n$.","solution":"As usual, the number of divisors of a positive integer $n$ is denoted by $d(n)$. If $n=\\prod_{i} p_{i}^{\\alpha_{i}}$ is the prime factorisation of $n$, then $d(n)=\\prod_{i}\\left(\\alpha_{i}+1\\right)$. We start by showing that one cannot find any suitable number $s$ if $k \\mid n$ or $n \\mid k$ (and $k \\neq n$ ). Suppose that $n \\mid k$, and choose any positive integer $s$. Then the set of divisors of $s n$ is a proper subset of that of $s k$, hence $d(s n)\\beta$ be nonnegative integers. Then, for every integer $M \\geqslant \\beta+1$, there exists a nonnegative integer $\\gamma$ such that $$ \\frac{\\alpha+\\gamma+1}{\\beta+\\gamma+1}=1+\\frac{1}{M}=\\frac{M+1}{M} . $$ Proof. $$ \\frac{\\alpha+\\gamma+1}{\\beta+\\gamma+1}=1+\\frac{1}{M} \\Longleftrightarrow \\frac{\\alpha-\\beta}{\\beta+\\gamma+1}=\\frac{1}{M} \\Longleftrightarrow \\gamma=M(\\alpha-\\beta)-(\\beta+1) \\geqslant 0 . $$ Now we can finish the solution. Without loss of generality, there exists an index $u$ such that $\\alpha_{i}>\\beta_{i}$ for $i=1,2, \\ldots, u$, and $\\alpha_{i}<\\beta_{i}$ for $i=u+1, \\ldots, t$. The conditions $n \\nmid k$ and $k \\nmid n$ mean that $1 \\leqslant u \\leqslant t-1$. Choose an integer $X$ greater than all the $\\alpha_{i}$ and $\\beta_{i}$. By the lemma, we can define the numbers $\\gamma_{i}$ so as to satisfy $$ \\begin{array}{ll} \\frac{\\alpha_{i}+\\gamma_{i}+1}{\\beta_{i}+\\gamma_{i}+1}=\\frac{u X+i}{u X+i-1} & \\text { for } i=1,2, \\ldots, u, \\text { and } \\\\ \\frac{\\beta_{u+i}+\\gamma_{u+i}+1}{\\alpha_{u+i}+\\gamma_{u+i}+1}=\\frac{(t-u) X+i}{(t-u) X+i-1} & \\text { for } i=1,2, \\ldots, t-u . \\end{array} $$ Then we will have $$ \\frac{d(s n)}{d(s k)}=\\prod_{i=1}^{u} \\frac{u X+i}{u X+i-1} \\cdot \\prod_{i=1}^{t-u} \\frac{(t-u) X+i-1}{(t-u) X+i}=\\frac{u(X+1)}{u X} \\cdot \\frac{(t-u) X}{(t-u)(X+1)}=1, $$ as required. Comment. The lemma can be used in various ways, in order to provide a suitable value of $s$. In particular, one may apply induction on the number $t$ of prime factors, using identities like $$ \\frac{n}{n-1}=\\frac{n^{2}}{n^{2}-1} \\cdot \\frac{n+1}{n} $$","tier":0} +{"problem_type":"Number Theory","problem_label":"N5","problem":"Four positive integers $x, y, z$, and $t$ satisfy the relations $$ x y-z t=x+y=z+t . $$ Is it possible that both $x y$ and $z t$ are perfect squares? (Russia) Answer: No.","solution":"Arguing indirectly, assume that $x y=a^{2}$ and $z t=c^{2}$ with $a, c>0$. Suppose that the number $x+y=z+t$ is odd. Then $x$ and $y$ have opposite parity, as well as $z$ and $t$. This means that both $x y$ and $z t$ are even, as well as $x y-z t=x+y$; a contradiction. Thus, $x+y$ is even, so the number $s=\\frac{x+y}{2}=\\frac{z+t}{2}$ is a positive integer. Next, we set $b=\\frac{|x-y|}{2}, d=\\frac{|z-t|}{2}$. Now the problem conditions yield $$ s^{2}=a^{2}+b^{2}=c^{2}+d^{2} $$ and $$ 2 s=a^{2}-c^{2}=d^{2}-b^{2} $$ (the last equality in (2) follows from (1)). We readily get from (2) that $a, d>0$. In the sequel we will use only the relations (1) and (2), along with the fact that $a, d, s$ are positive integers, while $b$ and $c$ are nonnegative integers, at most one of which may be zero. Since both relations are symmetric with respect to the simultaneous swappings $a \\leftrightarrow d$ and $b \\leftrightarrow c$, we assume, without loss of generality, that $b \\geqslant c$ (and hence $b>0$ ). Therefore, $d^{2}=2 s+b^{2}>c^{2}$, whence $$ d^{2}>\\frac{c^{2}+d^{2}}{2}=\\frac{s^{2}}{2} $$ On the other hand, since $d^{2}-b^{2}$ is even by (2), the numbers $b$ and $d$ have the same parity, so $00$ imply $b=c=0$, which is impossible.","tier":0} +{"problem_type":"Number Theory","problem_label":"N5","problem":"Four positive integers $x, y, z$, and $t$ satisfy the relations $$ x y-z t=x+y=z+t . $$ Is it possible that both $x y$ and $z t$ are perfect squares? (Russia) Answer: No.","solution":"We start with a complete description of all 4-tuples $(x, y, z, t)$ of positive integers satisfying (*). As in the solution above, we notice that the numbers $$ s=\\frac{x+y}{2}=\\frac{z+t}{2}, \\quad p=\\frac{x-y}{2}, \\quad \\text { and } \\quad q=\\frac{z-t}{2} $$ are integers (we may, and will, assume that $p, q \\geqslant 0$ ). We have $$ 2 s=x y-z t=(s+p)(s-p)-(s+q)(s-q)=q^{2}-p^{2} $$ so $p$ and $q$ have the same parity, and $q>p$. Set now $k=\\frac{q-p}{2}, \\ell=\\frac{q+p}{2}$. Then we have $s=\\frac{q^{2}-p^{2}}{2}=2 k \\ell$ and hence $$ \\begin{array}{rlrl} x & =s+p=2 k \\ell-k+\\ell, & y & =s-p=2 k \\ell+k-\\ell, \\\\ z & =s+q=2 k \\ell+k+\\ell, & t=s-q=2 k \\ell-k-\\ell . \\end{array} $$ Recall here that $\\ell \\geqslant k>0$ and, moreover, $(k, \\ell) \\neq(1,1)$, since otherwise $t=0$. Assume now that both $x y$ and $z t$ are squares. Then $x y z t$ is also a square. On the other hand, we have $$ \\begin{aligned} x y z t=(2 k \\ell-k+\\ell) & (2 k \\ell+k-\\ell)(2 k \\ell+k+\\ell)(2 k \\ell-k-\\ell) \\\\ & =\\left(4 k^{2} \\ell^{2}-(k-\\ell)^{2}\\right)\\left(4 k^{2} \\ell^{2}-(k+\\ell)^{2}\\right)=\\left(4 k^{2} \\ell^{2}-k^{2}-\\ell^{2}\\right)^{2}-4 k^{2} \\ell^{2} \\end{aligned} $$ Denote $D=4 k^{2} \\ell^{2}-k^{2}-\\ell^{2}>0$. From (6) we get $D^{2}>x y z t$. On the other hand, $$ \\begin{array}{r} (D-1)^{2}=D^{2}-2\\left(4 k^{2} \\ell^{2}-k^{2}-\\ell^{2}\\right)+1=\\left(D^{2}-4 k^{2} \\ell^{2}\\right)-\\left(2 k^{2}-1\\right)\\left(2 \\ell^{2}-1\\right)+2 \\\\ =x y z t-\\left(2 k^{2}-1\\right)\\left(2 \\ell^{2}-1\\right)+20$ and $\\ell \\geqslant 2$. The converse is also true: every pair of positive integers $\\ell \\geqslant k>0$, except for the pair $k=\\ell=1$, generates via (5) a 4-tuple of positive integers satisfying $(*)$.","tier":0} +{"problem_type":"Number Theory","problem_label":"N7","problem":"Let $n \\geqslant 2018$ be an integer, and let $a_{1}, a_{2}, \\ldots, a_{n}, b_{1}, b_{2}, \\ldots, b_{n}$ be pairwise distinct positive integers not exceeding $5 n$. Suppose that the sequence $$ \\frac{a_{1}}{b_{1}}, \\frac{a_{2}}{b_{2}}, \\ldots, \\frac{a_{n}}{b_{n}} $$ forms an arithmetic progression. Prove that the terms of the sequence are equal. (Thailand)","solution":"Suppose that (1) is an arithmetic progression with nonzero difference. Let the difference be $\\Delta=\\frac{c}{d}$, where $d>0$ and $c, d$ are coprime. We will show that too many denominators $b_{i}$ should be divisible by $d$. To this end, for any $1 \\leqslant i \\leqslant n$ and any prime divisor $p$ of $d$, say that the index $i$ is $p$-wrong, if $v_{p}\\left(b_{i}\\right)5 n, $$ a contradiction. Claim 3. For every $0 \\leqslant k \\leqslant n-30$, among the denominators $b_{k+1}, b_{k+2}, \\ldots, b_{k+30}$, at least $\\varphi(30)=8$ are divisible by $d$. Proof. By Claim 1, the 2 -wrong, 3 -wrong and 5 -wrong indices can be covered by three arithmetic progressions with differences 2,3 and 5 . By a simple inclusion-exclusion, $(2-1) \\cdot(3-1) \\cdot(5-1)=8$ indices are not covered; by Claim 2, we have $d \\mid b_{i}$ for every uncovered index $i$. Claim 4. $|\\Delta|<\\frac{20}{n-2}$ and $d>\\frac{n-2}{20}$. Proof. From the sequence (1), remove all fractions with $b_{n}<\\frac{n}{2}$, There remain at least $\\frac{n}{2}$ fractions, and they cannot exceed $\\frac{5 n}{n \/ 2}=10$. So we have at least $\\frac{n}{2}$ elements of the arithmetic progression (1) in the interval $(0,10]$, hence the difference must be below $\\frac{10}{n \/ 2-1}=\\frac{20}{n-2}$. The second inequality follows from $\\frac{1}{d} \\leqslant \\frac{|c|}{d}=|\\Delta|$. Now we have everything to get the final contradiction. By Claim 3, we have $d \\mid b_{i}$ for at least $\\left\\lfloor\\frac{n}{30}\\right\\rfloor \\cdot 8$ indices $i$. By Claim 4 , we have $d \\geqslant \\frac{n-2}{20}$. Therefore, $$ 5 n \\geqslant \\max \\left\\{b_{i}: d \\mid b_{i}\\right\\} \\geqslant\\left(\\left\\lfloor\\frac{n}{30}\\right\\rfloor \\cdot 8\\right) \\cdot d>\\left(\\frac{n}{30}-1\\right) \\cdot 8 \\cdot \\frac{n-2}{20}>5 n . $$ Comment 1. It is possible that all terms in (1) are equal, for example with $a_{i}=2 i-1$ and $b_{i}=4 i-2$ we have $\\frac{a_{i}}{b_{i}}=\\frac{1}{2}$. Comment 2. The bound $5 n$ in the statement is far from sharp; the solution above can be modified to work for $9 n$. For large $n$, the bound $5 n$ can be replaced by $n^{\\frac{3}{2}-\\varepsilon}$. The activities of the Problem Selection Committee were supported by DEDEMAN LA FANTANA vine la tine","tier":0} diff --git a/IMO/segmented/en-IMO2019SL.jsonl b/IMO/segmented/en-IMO2019SL.jsonl new file mode 100644 index 0000000000000000000000000000000000000000..a3fc5f7b842a09564f9c6379ab6cb450a46b88d1 --- /dev/null +++ b/IMO/segmented/en-IMO2019SL.jsonl @@ -0,0 +1,136 @@ +{"problem_type":"Algebra","problem_label":"A1","problem":"Let $\\mathbb{Z}$ be the set of integers. Determine all functions $f: \\mathbb{Z} \\rightarrow \\mathbb{Z}$ such that, for all integers $a$ and $b$, $$ f(2 a)+2 f(b)=f(f(a+b)) $$ (South Africa)","solution":"Substituting $a=0, b=n+1$ gives $f(f(n+1))=f(0)+2 f(n+1)$. Substituting $a=1, b=n$ gives $f(f(n+1))=f(2)+2 f(n)$. In particular, $f(0)+2 f(n+1)=f(2)+2 f(n)$, and so $f(n+1)-f(n)=\\frac{1}{2}(f(2)-f(0))$. Thus $f(n+1)-f(n)$ must be constant. Since $f$ is defined only on $\\mathbb{Z}$, this tells us that $f$ must be a linear function; write $f(n)=M n+K$ for arbitrary constants $M$ and $K$, and we need only determine which choices of $M$ and $K$ work. Now, (1) becomes $$ 2 M a+K+2(M b+K)=M(M(a+b)+K)+K $$ which we may rearrange to form $$ (M-2)(M(a+b)+K)=0 $$ Thus, either $M=2$, or $M(a+b)+K=0$ for all values of $a+b$. In particular, the only possible solutions are $f(n)=0$ and $f(n)=2 n+K$ for any constant $K \\in \\mathbb{Z}$, and these are easily seen to work.","tier":0} +{"problem_type":"Algebra","problem_label":"A1","problem":"Let $\\mathbb{Z}$ be the set of integers. Determine all functions $f: \\mathbb{Z} \\rightarrow \\mathbb{Z}$ such that, for all integers $a$ and $b$, $$ f(2 a)+2 f(b)=f(f(a+b)) $$ (South Africa)","solution":"Let $K=f(0)$. First, put $a=0$ in (1); this gives $$ f(f(b))=2 f(b)+K $$ for all $b \\in \\mathbb{Z}$. Now put $b=0$ in (1); this gives $$ f(2 a)+2 K=f(f(a))=2 f(a)+K $$ where the second equality follows from (2). Consequently, $$ f(2 a)=2 f(a)-K $$ for all $a \\in \\mathbb{Z}$. Substituting (2) and (3) into (1), we obtain $$ \\begin{aligned} f(2 a)+2 f(b) & =f(f(a+b)) \\\\ 2 f(a)-K+2 f(b) & =2 f(a+b)+K \\\\ f(a)+f(b) & =f(a+b)+K \\end{aligned} $$ Thus, if we set $g(n)=f(n)-K$ we see that $g$ satisfies the Cauchy equation $g(a+b)=$ $g(a)+g(b)$. The solution to the Cauchy equation over $\\mathbb{Z}$ is well-known; indeed, it may be proven by an easy induction that $g(n)=M n$ for each $n \\in \\mathbb{Z}$, where $M=g(1)$ is a constant. Therefore, $f(n)=M n+K$, and we may proceed as in Solution 1 . Comment 1. Instead of deriving (3) by substituting $b=0$ into (1), we could instead have observed that the right hand side of (1) is symmetric in $a$ and $b$, and thus $$ f(2 a)+2 f(b)=f(2 b)+2 f(a) $$ Thus, $f(2 a)-2 f(a)=f(2 b)-2 f(b)$ for any $a, b \\in \\mathbb{Z}$, and in particular $f(2 a)-2 f(a)$ is constant. Setting $a=0$ shows that this constant is equal to $-K$, and so we obtain (3). Comment 2. Some solutions initially prove that $f(f(n))$ is linear (sometimes via proving that $f(f(n))-3 K$ satisfies the Cauchy equation). However, one can immediately prove that $f$ is linear by substituting something of the form $f(f(n))=M^{\\prime} n+K^{\\prime}$ into (2).","tier":0} +{"problem_type":"Algebra","problem_label":"A2","problem":"Let $u_{1}, u_{2}, \\ldots, u_{2019}$ be real numbers satisfying $$ u_{1}+u_{2}+\\cdots+u_{2019}=0 \\quad \\text { and } \\quad u_{1}^{2}+u_{2}^{2}+\\cdots+u_{2019}^{2}=1 . $$ Let $a=\\min \\left(u_{1}, u_{2}, \\ldots, u_{2019}\\right)$ and $b=\\max \\left(u_{1}, u_{2}, \\ldots, u_{2019}\\right)$. Prove that $$ a b \\leqslant-\\frac{1}{2019} $$ (Germany)","solution":"Notice first that $b>0$ and $a<0$. Indeed, since $\\sum_{i=1}^{2019} u_{i}^{2}=1$, the variables $u_{i}$ cannot be all zero, and, since $\\sum_{i=1}^{2019} u_{i}=0$, the nonzero elements cannot be all positive or all negative. Let $P=\\left\\{i: u_{i}>0\\right\\}$ and $N=\\left\\{i: u_{i} \\leqslant 0\\right\\}$ be the indices of positive and nonpositive elements in the sequence, and let $p=|P|$ and $n=|N|$ be the sizes of these sets; then $p+n=2019$. By the condition $\\sum_{i=1}^{2019} u_{i}=0$ we have $0=\\sum_{i=1}^{2019} u_{i}=\\sum_{i \\in P} u_{i}-\\sum_{i \\in N}\\left|u_{i}\\right|$, so $$ \\sum_{i \\in P} u_{i}=\\sum_{i \\in N}\\left|u_{i}\\right| . $$ After this preparation, estimate the sum of squares of the positive and nonpositive elements as follows: $$ \\begin{aligned} & \\sum_{i \\in P} u_{i}^{2} \\leqslant \\sum_{i \\in P} b u_{i}=b \\sum_{i \\in P} u_{i}=b \\sum_{i \\in N}\\left|u_{i}\\right| \\leqslant b \\sum_{i \\in N}|a|=-n a b ; \\\\ & \\sum_{i \\in N} u_{i}^{2} \\leqslant \\sum_{i \\in N}|a| \\cdot\\left|u_{i}\\right|=|a| \\sum_{i \\in N}\\left|u_{i}\\right|=|a| \\sum_{i \\in P} u_{i} \\leqslant|a| \\sum_{i \\in P} b=-p a b . \\end{aligned} $$ The sum of these estimates is $$ 1=\\sum_{i=1}^{2019} u_{i}^{2}=\\sum_{i \\in P} u_{i}^{2}+\\sum_{i \\in N} u_{i}^{2} \\leqslant-(p+n) a b=-2019 a b ; $$ that proves $a b \\leqslant \\frac{-1}{2019}$. Comment 1. After observing $\\sum_{i \\in P} u_{i}^{2} \\leqslant b \\sum_{i \\in P} u_{i}$ and $\\sum_{i \\in N} u_{i}^{2} \\leqslant|a| \\sum_{i \\in P}\\left|u_{i}\\right|$, instead of $(2,3)$ an alternative continuation is $$ |a b| \\geqslant \\frac{\\sum_{i \\in P} u_{i}^{2}}{\\sum_{i \\in P} u_{i}} \\cdot \\frac{\\sum_{i \\in N} u_{i}^{2}}{\\sum_{i \\in N}\\left|u_{i}\\right|}=\\frac{\\sum_{i \\in P} u_{i}^{2}}{\\left(\\sum_{i \\in P} u_{i}\\right)^{2}} \\sum_{i \\in N} u_{i}^{2} \\geqslant \\frac{1}{p} \\sum_{i \\in N} u_{i}^{2} $$ (by the AM-QM or the Cauchy-Schwarz inequality) and similarly $|a b| \\geqslant \\frac{1}{n} \\sum_{i \\in P} u_{i}^{2}$.","tier":0} +{"problem_type":"Algebra","problem_label":"A2","problem":"Let $u_{1}, u_{2}, \\ldots, u_{2019}$ be real numbers satisfying $$ u_{1}+u_{2}+\\cdots+u_{2019}=0 \\quad \\text { and } \\quad u_{1}^{2}+u_{2}^{2}+\\cdots+u_{2019}^{2}=1 . $$ Let $a=\\min \\left(u_{1}, u_{2}, \\ldots, u_{2019}\\right)$ and $b=\\max \\left(u_{1}, u_{2}, \\ldots, u_{2019}\\right)$. Prove that $$ a b \\leqslant-\\frac{1}{2019} $$ (Germany)","solution":"As in the previous solution we conclude that $a<0$ and $b>0$. For every index $i$, the number $u_{i}$ is a convex combination of $a$ and $b$, so $$ u_{i}=x_{i} a+y_{i} b \\quad \\text { with some weights } 0 \\leqslant x_{i}, y_{i} \\leqslant 1, \\text { with } x_{i}+y_{i}=1 \\text {. } $$ Let $X=\\sum_{i=1}^{2019} x_{i}$ and $Y=\\sum_{i=1}^{2019} y_{i}$. From $0=\\sum_{i=1}^{2019} u_{i}=\\sum_{i=1}^{2019}\\left(x_{i} a+y_{i} b\\right)=-|a| X+b Y$, we get $$ |a| X=b Y $$ From $\\sum_{i=1}^{2019}\\left(x_{i}+y_{i}\\right)=2019$ we have $$ X+Y=2019 $$ The system of linear equations $(4,5)$ has a unique solution: $$ X=\\frac{2019 b}{|a|+b}, \\quad Y=\\frac{2019|a|}{|a|+b} $$ Now apply the following estimate to every $u_{i}^{2}$ in their sum: $$ u_{i}^{2}=x_{i}^{2} a^{2}+2 x_{i} y_{i} a b+y_{i}^{2} b^{2} \\leqslant x_{i} a^{2}+y_{i} b^{2} $$ we obtain that $$ 1=\\sum_{i=1}^{2019} u_{i}^{2} \\leqslant \\sum_{i=1}^{2019}\\left(x_{i} a^{2}+y_{i} b^{2}\\right)=X a^{2}+Y b^{2}=\\frac{2019 b}{|a|+b}|a|^{2}+\\frac{2019|a|}{|a|+b} b^{2}=2019|a| b=-2019 a b . $$ Hence, $a b \\leqslant \\frac{-1}{2019}$. Comment 2. The idea behind Solution 2 is the following thought. Suppose we fix $a<0$ and $b>0$, fix $\\sum u_{i}=0$ and vary the $u_{i}$ to achieve the maximum value of $\\sum u_{i}^{2}$. Considering varying any two of the $u_{i}$ while preserving their sum: the maximum value of $\\sum u_{i}^{2}$ is achieved when those two are as far apart as possible, so all but at most one of the $u_{i}$ are equal to $a$ or $b$. Considering a weighted version of the problem, we see the maximum (with fractional numbers of $u_{i}$ having each value) is achieved when $\\frac{2019 b}{|a|+b}$ of them are $a$ and $\\frac{2019|a|}{|a|+b}$ are $b$. In fact, this happens in the solution: the number $u_{i}$ is replaced by $x_{i}$ copies of $a$ and $y_{i}$ copies of $b$.","tier":0} +{"problem_type":"Algebra","problem_label":"A3","problem":"Let $n \\geqslant 3$ be a positive integer and let $\\left(a_{1}, a_{2}, \\ldots, a_{n}\\right)$ be a strictly increasing sequence of $n$ positive real numbers with sum equal to 2 . Let $X$ be a subset of $\\{1,2, \\ldots, n\\}$ such that the value of $$ \\left|1-\\sum_{i \\in X} a_{i}\\right| $$ is minimised. Prove that there exists a strictly increasing sequence of $n$ positive real numbers $\\left(b_{1}, b_{2}, \\ldots, b_{n}\\right)$ with sum equal to 2 such that $$ \\sum_{i \\in X} b_{i}=1 $$ (New Zealand)","solution":"Without loss of generality, assume $\\sum_{i \\in X} a_{i} \\leqslant 1$, and we may assume strict inequality as otherwise $b_{i}=a_{i}$ works. Also, $X$ clearly cannot be empty. If $n \\in X$, add $\\Delta$ to $a_{n}$, producing a sequence of $c_{i}$ with $\\sum_{i \\in X} c_{i}=\\sum_{i \\in X^{c}} c_{i}$, and then scale as described above to make the sum equal to 2 . Otherwise, there is some $k$ with $k \\in X$ and $k+1 \\in X^{c}$. Let $\\delta=a_{k+1}-a_{k}$. - If $\\delta>\\Delta$, add $\\Delta$ to $a_{k}$ and then scale. - If $\\delta<\\Delta$, then considering $X \\cup\\{k+1\\} \\backslash\\{k\\}$ contradicts $X$ being $\\left(a_{i}\\right)$-minimising. - If $\\delta=\\Delta$, choose any $j \\neq k, k+1$ (possible since $n \\geqslant 3$ ), and any $\\epsilon$ less than the least of $a_{1}$ and all the differences $a_{i+1}-a_{i}$. If $j \\in X$ then add $\\Delta-\\epsilon$ to $a_{k}$ and $\\epsilon$ to $a_{j}$, then scale; otherwise, add $\\Delta$ to $a_{k}$ and $\\epsilon \/ 2$ to $a_{k+1}$, and subtract $\\epsilon \/ 2$ from $a_{j}$, then scale.","tier":0} +{"problem_type":"Algebra","problem_label":"A3","problem":"Let $n \\geqslant 3$ be a positive integer and let $\\left(a_{1}, a_{2}, \\ldots, a_{n}\\right)$ be a strictly increasing sequence of $n$ positive real numbers with sum equal to 2 . Let $X$ be a subset of $\\{1,2, \\ldots, n\\}$ such that the value of $$ \\left|1-\\sum_{i \\in X} a_{i}\\right| $$ is minimised. Prove that there exists a strictly increasing sequence of $n$ positive real numbers $\\left(b_{1}, b_{2}, \\ldots, b_{n}\\right)$ with sum equal to 2 such that $$ \\sum_{i \\in X} b_{i}=1 $$ (New Zealand)","solution":"This is similar to Suppose there exists $1 \\leqslant j \\leqslant n-1$ such that $j \\in X$ but $j+1 \\in X^{c}$. Then $a_{j+1}-a_{j} \\geqslant \\Delta$, because otherwise considering $X \\cup\\{j+1\\} \\backslash\\{j\\}$ contradicts $X$ being $\\left(a_{i}\\right)$-minimising. If $a_{j+1}-a_{j}>\\Delta$, put $$ b_{i}= \\begin{cases}a_{j}+\\Delta \/ 2, & \\text { if } i=j \\\\ a_{j+1}-\\Delta \/ 2, & \\text { if } i=j+1 \\\\ a_{i}, & \\text { otherwise }\\end{cases} $$ If $a_{j+1}-a_{j}=\\Delta$, choose any $\\epsilon$ less than the least of $\\Delta \/ 2, a_{1}$ and all the differences $a_{i+1}-a_{i}$. If $|X| \\geqslant 2$, choose $k \\in X$ with $k \\neq j$, and put $$ b_{i}= \\begin{cases}a_{j}+\\Delta \/ 2-\\epsilon, & \\text { if } i=j \\\\ a_{j+1}-\\Delta \/ 2, & \\text { if } i=j+1 \\\\ a_{k}+\\epsilon, & \\text { if } i=k \\\\ a_{i}, & \\text { otherwise }\\end{cases} $$ Otherwise, $\\left|X^{c}\\right| \\geqslant 2$, so choose $k \\in X^{c}$ with $k \\neq j+1$, and put $$ b_{i}= \\begin{cases}a_{j}+\\Delta \/ 2, & \\text { if } i=j \\\\ a_{j+1}-\\Delta \/ 2+\\epsilon, & \\text { if } i=j+1 \\\\ a_{k}-\\epsilon, & \\text { if } i=k \\\\ a_{i}, & \\text { otherwise }\\end{cases} $$ If there is no $1 \\leqslant j \\leqslant n$ such that $j \\in X$ but $j+1 \\in X^{c}$, there must be some $1\\Delta$, as otherwise considering $X \\cup\\{1\\}$ contradicts $X$ being $\\left(a_{i}\\right)$-minimising. Now put $$ b_{i}= \\begin{cases}a_{1}-\\Delta \/ 2, & \\text { if } i=1 \\\\ a_{n}+\\Delta \/ 2, & \\text { if } i=n \\\\ a_{i}, & \\text { otherwise }\\end{cases} $$","tier":0} +{"problem_type":"Algebra","problem_label":"A3","problem":"Let $n \\geqslant 3$ be a positive integer and let $\\left(a_{1}, a_{2}, \\ldots, a_{n}\\right)$ be a strictly increasing sequence of $n$ positive real numbers with sum equal to 2 . Let $X$ be a subset of $\\{1,2, \\ldots, n\\}$ such that the value of $$ \\left|1-\\sum_{i \\in X} a_{i}\\right| $$ is minimised. Prove that there exists a strictly increasing sequence of $n$ positive real numbers $\\left(b_{1}, b_{2}, \\ldots, b_{n}\\right)$ with sum equal to 2 such that $$ \\sum_{i \\in X} b_{i}=1 $$ (New Zealand)","solution":"Without loss of generality, assume $\\sum_{i \\in X} a_{i} \\leqslant 1$, so $\\Delta \\geqslant 0$. If $\\Delta=0$ we can take $b_{i}=a_{i}$, so now assume that $\\Delta>0$. Suppose that there is some $k \\leqslant n$ such that $|X \\cap[k, n]|>\\left|X^{c} \\cap[k, n]\\right|$. If we choose the largest such $k$ then $|X \\cap[k, n]|-\\left|X^{c} \\cap[k, n]\\right|=1$. We can now find the required sequence $\\left(b_{i}\\right)$ by starting with $c_{i}=a_{i}$ for $i\\frac{n-k+1}{2}$ and $|X \\cap[\\ell, n]|<\\frac{n-\\ell+1}{2}$. We now construct our sequence $\\left(b_{i}\\right)$ using this claim. Let $k$ and $\\ell$ be the greatest values satisfying the claim, and without loss of generality suppose $k=n$ and $\\ell\\sum_{i \\in Y} a_{i}>1 $$ contradicting $X$ being $\\left(a_{i}\\right)$-minimising. Otherwise, we always have equality, meaning that $X=Y$. But now consider $Z=Y \\cup\\{n-1\\} \\backslash\\{n\\}$. Since $n \\geqslant 3$, we have $$ \\sum_{i \\in Y} a_{i}>\\sum_{i \\in Z} a_{i}>\\sum_{i \\in Y^{c}} a_{i}=2-\\sum_{i \\in Y} a_{i} $$ and so $Z$ contradicts $X$ being $\\left(a_{i}\\right)$-minimising.","tier":0} +{"problem_type":"Algebra","problem_label":"A4","problem":"Let $n \\geqslant 2$ be a positive integer and $a_{1}, a_{2}, \\ldots, a_{n}$ be real numbers such that $$ a_{1}+a_{2}+\\cdots+a_{n}=0 $$ Define the set $A$ by $$ A=\\left\\{(i, j)\\left|1 \\leqslant i0 $$ Partition the indices into sets $P, Q, R$, and $S$ such that $$ \\begin{aligned} P & =\\left\\{i \\mid a_{i} \\leqslant-1\\right\\} & R & =\\left\\{i \\mid 01$ ). Therefore, $$ t_{+}+t_{-} \\leqslant \\frac{p^{2}+s^{2}}{2}+p q+r s+p r+p s+q s=\\frac{(p+q+r+s)^{2}}{2}-\\frac{(q+r)^{2}}{2}=-\\frac{(q+r)^{2}}{2} \\leqslant 0 $$ If $A$ is not empty and $p=s=0$, then there must exist $i \\in Q, j \\in R$ with $\\left|a_{i}-a_{j}\\right|>1$, and hence the earlier equality conditions cannot both occur. Comment. The RHS of the original inequality cannot be replaced with any constant $c<0$ (independent of $n$ ). Indeed, take $$ a_{1}=-\\frac{n}{n+2}, a_{2}=\\cdots=a_{n-1}=\\frac{1}{n+2}, a_{n}=\\frac{2}{n+2} . $$ Then $\\sum_{(i, j) \\in A} a_{i} a_{j}=-\\frac{2 n}{(n+2)^{2}}$, which converges to zero as $n \\rightarrow \\infty$. This page is intentionally left blank","tier":0} +{"problem_type":"Algebra","problem_label":"A5","problem":"Let $x_{1}, x_{2}, \\ldots, x_{n}$ be different real numbers. Prove that $$ \\sum_{1 \\leqslant i \\leqslant n} \\prod_{j \\neq i} \\frac{1-x_{i} x_{j}}{x_{i}-x_{j}}= \\begin{cases}0, & \\text { if } n \\text { is even } \\\\ 1, & \\text { if } n \\text { is odd }\\end{cases} $$","solution":null,"tier":0} +{"problem_type":"Algebra","problem_label":"A6","problem":"A polynomial $P(x, y, z)$ in three variables with real coefficients satisfies the identities $$ P(x, y, z)=P(x, y, x y-z)=P(x, z x-y, z)=P(y z-x, y, z) . $$ Prove that there exists a polynomial $F(t)$ in one variable such that $$ P(x, y, z)=F\\left(x^{2}+y^{2}+z^{2}-x y z\\right) . $$","solution":"In the first two steps, we deal with any polynomial $P(x, y, z)$ satisfying $P(x, y, z)=$ $P(x, y, x y-z)$. Call such a polynomial weakly symmetric, and call a polynomial satisfying the full conditions in the problem symmetric. Step 1. We start with the description of weakly symmetric polynomials. We claim that they are exactly the polynomials in $x, y$, and $z(x y-z)$. Clearly, all such polynomials are weakly symmetric. For the converse statement, consider $P_{1}(x, y, z):=P\\left(x, y, z+\\frac{1}{2} x y\\right)$, which satisfies $P_{1}(x, y, z)=P_{1}(x, y,-z)$ and is therefore a polynomial in $x, y$, and $z^{2}$. This means that $P$ is a polynomial in $x, y$, and $\\left(z-\\frac{1}{2} x y\\right)^{2}=-z(x y-z)+\\frac{1}{4} x^{2} y^{2}$, and therefore a polynomial in $x, y$, and $z(x y-z)$. Step 2. Suppose that $P$ is weakly symmetric. Consider the monomials in $P(x, y, z)$ of highest total degree. Our aim is to show that in each such monomial $\\mu x^{a} y^{b} z^{c}$ we have $a, b \\geqslant c$. Consider the expansion $$ P(x, y, z)=\\sum_{i, j, k} \\mu_{i j k} x^{i} y^{j}(z(x y-z))^{k} $$ The maximal total degree of a summand in (1.1) is $m=\\max _{i, j, k: \\mu_{i j k} \\neq 0}(i+j+3 k)$. Now, for any $i, j, k$ satisfying $i+j+3 k=m$ the summand $\\mu_{i, j, k} x^{i} y^{j}(z(x y-z))^{k}$ has leading term of the form $\\mu x^{i+k} y^{j+k} z^{k}$. No other nonzero summand in (1.1) may have a term of this form in its expansion, hence this term does not cancel in the whole sum. Therefore, $\\operatorname{deg} P=m$, and the leading component of $P$ is exactly $$ \\sum_{i+j+3 k=m} \\mu_{i, j, k} x^{i+k} y^{j+k} z^{k} $$ and each summand in this sum satisfies the condition claimed above. Step 3. We now prove the problem statement by induction on $m=\\operatorname{deg} P$. For $m=0$ the claim is trivial. Consider now a symmetric polynomial $P$ with $\\operatorname{deg} P>0$. By Step 2, each of its monomials $\\mu x^{a} y^{b} z^{c}$ of the highest total degree satisfies $a, b \\geqslant c$. Applying other weak symmetries, we obtain $a, c \\geqslant b$ and $b, c \\geqslant a$; therefore, $P$ has a unique leading monomial of the form $\\mu(x y z)^{c}$. The polynomial $P_{0}(x, y, z)=P(x, y, z)-\\mu\\left(x y z-x^{2}-y^{2}-z^{2}\\right)^{c}$ has smaller total degree. Since $P_{0}$ is symmetric, it is representable as a polynomial function of $x y z-x^{2}-y^{2}-z^{2}$. Then $P$ is also of this form, completing the inductive step. Comment. We could alternatively carry out Step 1 by an induction on $n=\\operatorname{deg}_{z} P$, in a manner similar to Step 3. If $n=0$, the statement holds. Assume that $n>0$ and check the leading component of $P$ with respect to $z$ : $$ P(x, y, z)=Q_{n}(x, y) z^{n}+R(x, y, z) $$ where $\\operatorname{deg}_{z} Rv(x)$ and $v(x) \\geqslant v(y)$. Hence this $f$ satisfies the functional equation and 0 is an $f$-rare integer. Comment 3. In fact, if $v$ is an $f$-rare integer for an $f$ satisfying the functional equation, then its fibre $X_{v}=\\{v\\}$ must be a singleton. We may assume without loss of generality that $v=0$. We've already seen in Solution 1 that 0 is either the greatest or least element of $X_{0}$; replacing $f$ with the function $x \\mapsto-f(-x)$ if necessary, we may assume that 0 is the least element of $X_{0}$. We write $b$ for the largest element of $X_{0}$, supposing for contradiction that $b>0$, and write $N=(2 b)$ !. It now follows from (*) that we have $$ f(f(N b)+b)=f(f(0)+b)=f(b)=0 $$ from which we see that $f(N b)+b \\in X_{0} \\subseteq[0, b]$. It follows that $f(N b) \\in[-b, 0)$, since by construction $N b \\notin X_{v}$. Now it follows that $(f(N b)-0) \\cdot(f(N b)-b)$ is a divisor of $N$, so from ( $\\dagger$ ) we see that $f(N b)=f(0)=0$. This yields the desired contradiction.","tier":0} +{"problem_type":"Combinatorics","problem_label":"C1","problem":"The infinite sequence $a_{0}, a_{1}, a_{2}, \\ldots$ of (not necessarily different) integers has the following properties: $0 \\leqslant a_{i} \\leqslant i$ for all integers $i \\geqslant 0$, and $$ \\binom{k}{a_{0}}+\\binom{k}{a_{1}}+\\cdots+\\binom{k}{a_{k}}=2^{k} $$ for all integers $k \\geqslant 0$. Prove that all integers $N \\geqslant 0$ occur in the sequence (that is, for all $N \\geqslant 0$, there exists $i \\geqslant 0$ with $a_{i}=N$ ). ## (Netherlands)","solution":"We prove by induction on $k$ that every initial segment of the sequence, $a_{0}, a_{1}, \\ldots, a_{k}$, consists of the following elements (counted with multiplicity, and not necessarily in order), for some $\\ell \\geqslant 0$ with $2 \\ell \\leqslant k+1$ : $$ 0,1, \\ldots, \\ell-1, \\quad 0,1, \\ldots, k-\\ell $$ For $k=0$ we have $a_{0}=0$, which is of this form. Now suppose that for $k=m$ the elements $a_{0}, a_{1}, \\ldots, a_{m}$ are $0,0,1,1,2,2, \\ldots, \\ell-1, \\ell-1, \\ell, \\ell+1, \\ldots, m-\\ell-1, m-\\ell$ for some $\\ell$ with $0 \\leqslant 2 \\ell \\leqslant m+1$. It is given that $$ \\binom{m+1}{a_{0}}+\\binom{m+1}{a_{1}}+\\cdots+\\binom{m+1}{a_{m}}+\\binom{m+1}{a_{m+1}}=2^{m+1} $$ which becomes $$ \\begin{aligned} \\left(\\binom{m+1}{0}+\\binom{m+1}{1}\\right. & \\left.+\\cdots+\\binom{m+1}{\\ell-1}\\right) \\\\ & +\\left(\\binom{m+1}{0}+\\binom{m+1}{1}+\\cdots+\\binom{m+1}{m-\\ell}\\right)+\\binom{m+1}{a_{m+1}}=2^{m+1} \\end{aligned} $$ or, using $\\binom{m+1}{i}=\\binom{m+1}{m+1-i}$, that $$ \\begin{aligned} \\left(\\binom{m+1}{0}+\\binom{m+1}{1}\\right. & \\left.+\\cdots+\\binom{m+1}{\\ell-1}\\right) \\\\ & +\\left(\\binom{m+1}{m+1}+\\binom{m+1}{m}+\\cdots+\\binom{m+1}{\\ell+1}\\right)+\\binom{m+1}{a_{m+1}}=2^{m+1} \\end{aligned} $$ On the other hand, it is well known that $$ \\binom{m+1}{0}+\\binom{m+1}{1}+\\cdots+\\binom{m+1}{m+1}=2^{m+1} $$ and so, by subtracting, we get $$ \\binom{m+1}{a_{m+1}}=\\binom{m+1}{\\ell} $$ From this, using the fact that the binomial coefficients $\\binom{m+1}{i}$ are increasing for $i \\leqslant \\frac{m+1}{2}$ and decreasing for $i \\geqslant \\frac{m+1}{2}$, we conclude that either $a_{m+1}=\\ell$ or $a_{m+1}=m+1-\\ell$. In either case, $a_{0}, a_{1}, \\ldots, a_{m+1}$ is again of the claimed form, which concludes the induction. As a result of this description, any integer $N \\geqslant 0$ appears as a term of the sequence $a_{i}$ for some $0 \\leqslant i \\leqslant 2 N$.","tier":0} +{"problem_type":"Combinatorics","problem_label":"C2","problem":"You are given a set of $n$ blocks, each weighing at least 1 ; their total weight is $2 n$. Prove that for every real number $r$ with $0 \\leqslant r \\leqslant 2 n-2$ you can choose a subset of the blocks whose total weight is at least $r$ but at most $r+2$. (Thailand)","solution":"We prove the following more general statement by induction on $n$. Claim. Suppose that you have $n$ blocks, each of weight at least 1 , and of total weight $s \\leqslant 2 n$. Then for every $r$ with $-2 \\leqslant r \\leqslant s$, you can choose some of the blocks whose total weight is at least $r$ but at most $r+2$. Proof. The base case $n=1$ is trivial. To prove the inductive step, let $x$ be the largest block weight. Clearly, $x \\geqslant s \/ n$, so $s-x \\leqslant \\frac{n-1}{n} s \\leqslant 2(n-1)$. Hence, if we exclude a block of weight $x$, we can apply the inductive hypothesis to show the claim holds (for this smaller set) for any $-2 \\leqslant r \\leqslant s-x$. Adding the excluded block to each of those combinations, we see that the claim also holds when $x-2 \\leqslant r \\leqslant s$. So if $x-2 \\leqslant s-x$, then we have covered the whole interval $[-2, s]$. But each block weight is at least 1 , so we have $x-2 \\leqslant(s-(n-1))-2=s-(2 n-(n-1)) \\leqslant s-(s-(n-1)) \\leqslant s-x$, as desired. Comment. Instead of inducting on sets of blocks with total weight $s \\leqslant 2 n$, we could instead prove the result only for $s=2 n$. We would then need to modify the inductive step to scale up the block weights before applying the induction hypothesis.","tier":0} +{"problem_type":"Combinatorics","problem_label":"C2","problem":"You are given a set of $n$ blocks, each weighing at least 1 ; their total weight is $2 n$. Prove that for every real number $r$ with $0 \\leqslant r \\leqslant 2 n-2$ you can choose a subset of the blocks whose total weight is at least $r$ but at most $r+2$. (Thailand)","solution":"Let $x_{1}, \\ldots, x_{n}$ be the weights of the blocks in weakly increasing order. Consider the set $S$ of sums of the form $\\sum_{j \\in J} x_{j}$ for a subset $J \\subseteq\\{1,2, \\ldots, n\\}$. We want to prove that the mesh of $S$ - i.e. the largest distance between two adjacent elements - is at most 2. For $0 \\leqslant k \\leqslant n$, let $S_{k}$ denote the set of sums of the form $\\sum_{i \\in J} x_{i}$ for a subset $J \\subseteq\\{1,2, \\ldots, k\\}$. We will show by induction on $k$ that the mesh of $S_{k}$ is at most 2 . The base case $k=0$ is trivial (as $S_{0}=\\{0\\}$ ). For $k>0$ we have $$ S_{k}=S_{k-1} \\cup\\left(x_{k}+S_{k-1}\\right) $$ (where $\\left(x_{k}+S_{k-1}\\right)$ denotes $\\left\\{x_{k}+s: s \\in S_{k-1}\\right\\}$ ), so it suffices to prove that $x_{k} \\leqslant \\sum_{j\\sum_{j(n+1-k)(k+1)+k-1 $$ This rearranges to $n>k(n+1-k)$, which is false for $1 \\leqslant k \\leqslant n$, giving the desired contradiction.","tier":0} +{"problem_type":"Combinatorics","problem_label":"C3","problem":"Let $n$ be a positive integer. Harry has $n$ coins lined up on his desk, each showing heads or tails. He repeatedly does the following operation: if there are $k$ coins showing heads and $k>0$, then he flips the $k^{\\text {th }}$ coin over; otherwise he stops the process. (For example, the process starting with THT would be THT $\\rightarrow H H T \\rightarrow H T T \\rightarrow T T T$, which takes three steps.) Letting $C$ denote the initial configuration (a sequence of $n H$ 's and $T$ 's), write $\\ell(C)$ for the number of steps needed before all coins show $T$. Show that this number $\\ell(C)$ is finite, and determine its average value over all $2^{n}$ possible initial configurations $C$.","solution":"We represent the problem using a directed graph $G_{n}$ whose vertices are the length- $n$ strings of $H$ 's and $T$ 's. The graph features an edge from each string to its successor (except for $T T \\cdots T T$, which has no successor). We will also write $\\bar{H}=T$ and $\\bar{T}=H$. The graph $G_{0}$ consists of a single vertex: the empty string. The main claim is that $G_{n}$ can be described explicitly in terms of $G_{n-1}$ : - We take two copies, $X$ and $Y$, of $G_{n-1}$. - In $X$, we take each string of $n-1$ coins and just append a $T$ to it. In symbols, we replace $s_{1} \\cdots s_{n-1}$ with $s_{1} \\cdots s_{n-1} T$. - In $Y$, we take each string of $n-1$ coins, flip every coin, reverse the order, and append an $H$ to it. In symbols, we replace $s_{1} \\cdots s_{n-1}$ with $\\bar{s}_{n-1} \\bar{s}_{n-2} \\cdots \\bar{s}_{1} H$. - Finally, we add one new edge from $Y$ to $X$, namely $H H \\cdots H H H \\rightarrow H H \\cdots H H T$. We depict $G_{4}$ below, in a way which indicates this recursive construction: ![](https:\/\/cdn.mathpix.com\/cropped\/2024_11_18_9ee8c2ac2c2f5002ff14g-034.jpg?height=515&width=1135&top_left_y=1850&top_left_x=466) We prove the claim inductively. Firstly, $X$ is correct as a subgraph of $G_{n}$, as the operation on coins is unchanged by an extra $T$ at the end: if $s_{1} \\cdots s_{n-1}$ is sent to $t_{1} \\cdots t_{n-1}$, then $s_{1} \\cdots s_{n-1} T$ is sent to $t_{1} \\cdots t_{n-1} T$. Next, $Y$ is also correct as a subgraph of $G_{n}$, as if $s_{1} \\cdots s_{n-1}$ has $k$ occurrences of $H$, then $\\bar{s}_{n-1} \\cdots \\bar{s}_{1} H$ has $(n-1-k)+1=n-k$ occurrences of $H$, and thus (provided that $k>0$ ), if $s_{1} \\cdots s_{n-1}$ is sent to $t_{1} \\cdots t_{n-1}$, then $\\bar{s}_{n-1} \\cdots \\bar{s}_{1} H$ is sent to $\\bar{t}_{n-1} \\cdots \\bar{t}_{1} H$. Finally, the one edge from $Y$ to $X$ is correct, as the operation does send $H H \\cdots H H H$ to HH $\\cdots$ HHT. To finish, note that the sequences in $X$ take an average of $E(n-1)$ steps to terminate, whereas the sequences in $Y$ take an average of $E(n-1)$ steps to reach $H H \\cdots H$ and then an additional $n$ steps to terminate. Therefore, we have $$ E(n)=\\frac{1}{2}(E(n-1)+(E(n-1)+n))=E(n-1)+\\frac{n}{2} $$ We have $E(0)=0$ from our description of $G_{0}$. Thus, by induction, we have $E(n)=\\frac{1}{2}(1+\\cdots+$ $n)=\\frac{1}{4} n(n+1)$, which in particular is finite.","tier":0} +{"problem_type":"Combinatorics","problem_label":"C3","problem":"Let $n$ be a positive integer. Harry has $n$ coins lined up on his desk, each showing heads or tails. He repeatedly does the following operation: if there are $k$ coins showing heads and $k>0$, then he flips the $k^{\\text {th }}$ coin over; otherwise he stops the process. (For example, the process starting with THT would be THT $\\rightarrow H H T \\rightarrow H T T \\rightarrow T T T$, which takes three steps.) Letting $C$ denote the initial configuration (a sequence of $n H$ 's and $T$ 's), write $\\ell(C)$ for the number of steps needed before all coins show $T$. Show that this number $\\ell(C)$ is finite, and determine its average value over all $2^{n}$ possible initial configurations $C$.","solution":"We consider what happens with configurations depending on the coins they start and end with. - If a configuration starts with $H$, the last $n-1$ coins follow the given rules, as if they were all the coins, until they are all $T$, then the first coin is turned over. - If a configuration ends with $T$, the last coin will never be turned over, and the first $n-1$ coins follow the given rules, as if they were all the coins. - If a configuration starts with $T$ and ends with $H$, the middle $n-2$ coins follow the given rules, as if they were all the coins, until they are all $T$. After that, there are $2 n-1$ more steps: first coins $1,2, \\ldots, n-1$ are turned over in that order, then coins $n, n-1, \\ldots, 1$ are turned over in that order. As this covers all configurations, and the number of steps is clearly finite for 0 or 1 coins, it follows by induction on $n$ that the number of steps is always finite. We define $E_{A B}(n)$, where $A$ and $B$ are each one of $H, T$ or $*$, to be the average number of steps over configurations of length $n$ restricted to those that start with $A$, if $A$ is not $*$, and that end with $B$, if $B$ is not * (so * represents \"either $H$ or $T$ \"). The above observations tell us that, for $n \\geqslant 2$ : - $E_{H *}(n)=E(n-1)+1$. - $E_{* T}(n)=E(n-1)$. - $E_{H T}(n)=E(n-2)+1$ (by using both the observations for $H *$ and for $\\left.* T\\right)$. - $E_{T H}(n)=E(n-2)+2 n-1$. Now $E_{H *}(n)=\\frac{1}{2}\\left(E_{H H}(n)+E_{H T}(n)\\right)$, so $E_{H H}(n)=2 E(n-1)-E(n-2)+1$. Similarly, $E_{T T}(n)=2 E(n-1)-E(n-2)-1$. So $$ E(n)=\\frac{1}{4}\\left(E_{H T}(n)+E_{H H}(n)+E_{T T}(n)+E_{T H}(n)\\right)=E(n-1)+\\frac{n}{2} $$ We have $E(0)=0$ and $E(1)=\\frac{1}{2}$, so by induction on $n$ we have $E(n)=\\frac{1}{4} n(n+1)$.","tier":0} +{"problem_type":"Combinatorics","problem_label":"C3","problem":"Let $n$ be a positive integer. Harry has $n$ coins lined up on his desk, each showing heads or tails. He repeatedly does the following operation: if there are $k$ coins showing heads and $k>0$, then he flips the $k^{\\text {th }}$ coin over; otherwise he stops the process. (For example, the process starting with THT would be THT $\\rightarrow H H T \\rightarrow H T T \\rightarrow T T T$, which takes three steps.) Letting $C$ denote the initial configuration (a sequence of $n H$ 's and $T$ 's), write $\\ell(C)$ for the number of steps needed before all coins show $T$. Show that this number $\\ell(C)$ is finite, and determine its average value over all $2^{n}$ possible initial configurations $C$.","solution":"Let $H_{i}$ be the number of heads in positions 1 to $i$ inclusive (so $H_{n}$ is the total number of heads), and let $I_{i}$ be 1 if the $i^{\\text {th }}$ coin is a head, 0 otherwise. Consider the function $$ t(i)=I_{i}+2\\left(\\min \\left\\{i, H_{n}\\right\\}-H_{i}\\right) $$ We claim that $t(i)$ is the total number of times coin $i$ is turned over (which implies that the process terminates). Certainly $t(i)=0$ when all coins are tails, and $t(i)$ is always a nonnegative integer, so it suffices to show that when the $k^{\\text {th }}$ coin is turned over (where $k=H_{n}$ ), $t(k)$ goes down by 1 and all the other $t(i)$ are unchanged. We show this by splitting into cases: - If $ik, \\min \\left\\{i, H_{n}\\right\\}=H_{n}$ both before and after the coin flip, and both $H_{n}$ and $H_{i}$ change by the same amount, so $t(i)$ is unchanged. - If $i=k$ and the coin is heads, $I_{i}$ goes down by 1 , as do both $\\min \\left\\{i, H_{n}\\right\\}=H_{n}$ and $H_{i}$; so $t(i)$ goes down by 1 . - If $i=k$ and the coin is tails, $I_{i}$ goes up by $1, \\min \\left\\{i, H_{n}\\right\\}=i$ is unchanged and $H_{i}$ goes up by 1 ; so $t(i)$ goes down by 1 . We now need to compute the average value of $$ \\sum_{i=1}^{n} t(i)=\\sum_{i=1}^{n} I_{i}+2 \\sum_{i=1}^{n} \\min \\left\\{i, H_{n}\\right\\}-2 \\sum_{i=1}^{n} H_{i} . $$ The average value of the first term is $\\frac{1}{2} n$, and that of the third term is $-\\frac{1}{2} n(n+1)$. To compute the second term, we sum over choices for the total number of heads, and then over the possible values of $i$, getting $$ 2^{1-n} \\sum_{j=0}^{n}\\binom{n}{j} \\sum_{i=1}^{n} \\min \\{i, j\\}=2^{1-n} \\sum_{j=0}^{n}\\binom{n}{j}\\left(n j-\\binom{j}{2}\\right) . $$ Now, in terms of trinomial coefficients, $$ \\sum_{j=0}^{n} j\\binom{n}{j}=\\sum_{j=1}^{n}\\binom{n}{n-j, j-1,1}=n \\sum_{j=0}^{n-1}\\binom{n-1}{j}=2^{n-1} n $$ and $$ \\sum_{j=0}^{n}\\binom{j}{2}\\binom{n}{j}=\\sum_{j=2}^{n}\\binom{n}{n-j, j-2,2}=\\binom{n}{2} \\sum_{j=0}^{n-2}\\binom{n-2}{j}=2^{n-2}\\binom{n}{2} $$ So the second term above is $$ 2^{1-n}\\left(2^{n-1} n^{2}-2^{n-2}\\binom{n}{2}\\right)=n^{2}-\\frac{n(n-1)}{4} $$ and the required average is $$ E(n)=\\frac{1}{2} n+n^{2}-\\frac{n(n-1)}{4}-\\frac{1}{2} n(n+1)=\\frac{n(n+1)}{4} . $$","tier":0} +{"problem_type":"Combinatorics","problem_label":"C3","problem":"Let $n$ be a positive integer. Harry has $n$ coins lined up on his desk, each showing heads or tails. He repeatedly does the following operation: if there are $k$ coins showing heads and $k>0$, then he flips the $k^{\\text {th }}$ coin over; otherwise he stops the process. (For example, the process starting with THT would be THT $\\rightarrow H H T \\rightarrow H T T \\rightarrow T T T$, which takes three steps.) Letting $C$ denote the initial configuration (a sequence of $n H$ 's and $T$ 's), write $\\ell(C)$ for the number of steps needed before all coins show $T$. Show that this number $\\ell(C)$ is finite, and determine its average value over all $2^{n}$ possible initial configurations $C$.","solution":"Harry has built a Turing machine to flip the coins for him. The machine is initially positioned at the $k^{\\text {th }}$ coin, where there are $k$ heads (and the position before the first coin is considered to be the $0^{\\text {th }}$ coin). The machine then moves according to the following rules, stopping when it reaches the position before the first coin: if the coin at its current position is $H$, it flips the coin and moves to the previous coin, while if the coin at its current position is $T$, it flips the coin and moves to the next position. Consider the maximal sequences of consecutive moves in the same direction. Suppose the machine has $a$ consecutive moves to the next coin, before a move to the previous coin. After those $a$ moves, the $a$ coins flipped in those moves are all heads, as is the coin the machine is now at, so at least the next $a+1$ moves will all be moves to the previous coin. Similarly, $a$ consecutive moves to the previous coin are followed by at least $a+1$ consecutive moves to the next coin. There cannot be more than $n$ consecutive moves in the same direction, so this proves that the process terminates (with a move from the first coin to the position before the first coin). Thus we have a (possibly empty) sequence $a_{1}<\\cdotsj$, meaning that, as we follow the moves backwards, each coin is always in the correct state when flipped to result in a move in the required direction. (Alternatively, since there are $2^{n}$ possible configurations of coins and $2^{n}$ possible such ascending sequences, the fact that the sequence of moves determines at most one configuration of coins, and thus that there is an injection from configurations of coins to such ascending sequences, is sufficient for it to be a bijection, without needing to show that coins are in the right state as we move backwards.)","tier":0} +{"problem_type":"Combinatorics","problem_label":"C3","problem":"Let $n$ be a positive integer. Harry has $n$ coins lined up on his desk, each showing heads or tails. He repeatedly does the following operation: if there are $k$ coins showing heads and $k>0$, then he flips the $k^{\\text {th }}$ coin over; otherwise he stops the process. (For example, the process starting with THT would be THT $\\rightarrow H H T \\rightarrow H T T \\rightarrow T T T$, which takes three steps.) Letting $C$ denote the initial configuration (a sequence of $n H$ 's and $T$ 's), write $\\ell(C)$ for the number of steps needed before all coins show $T$. Show that this number $\\ell(C)$ is finite, and determine its average value over all $2^{n}$ possible initial configurations $C$.","solution":"We explicitly describe what happens with an arbitrary sequence $C$ of $n$ coins. Suppose that $C$ contain $k$ heads at positions $1 \\leqslant c_{1}1$ be an integer. Suppose we are given $2 n$ points in a plane such that no three of them are collinear. The points are to be labelled $A_{1}, A_{2}, \\ldots, A_{2 n}$ in some order. We then consider the $2 n$ angles $\\angle A_{1} A_{2} A_{3}, \\angle A_{2} A_{3} A_{4}, \\ldots, \\angle A_{2 n-2} A_{2 n-1} A_{2 n}, \\angle A_{2 n-1} A_{2 n} A_{1}$, $\\angle A_{2 n} A_{1} A_{2}$. We measure each angle in the way that gives the smallest positive value (i.e. between $0^{\\circ}$ and $180^{\\circ}$ ). Prove that there exists an ordering of the given points such that the resulting $2 n$ angles can be separated into two groups with the sum of one group of angles equal to the sum of the other group.","solution":"Let $\\ell$ be a line separating the points into two groups ( $L$ and $R$ ) with $n$ points in each. Label the points $A_{1}, A_{2}, \\ldots, A_{2 n}$ so that $L=\\left\\{A_{1}, A_{3}, \\ldots, A_{2 n-1}\\right\\}$. We claim that this labelling works. Take a line $s=A_{2 n} A_{1}$. (a) Rotate $s$ around $A_{1}$ until it passes through $A_{2}$; the rotation is performed in a direction such that $s$ is never parallel to $\\ell$. (b) Then rotate the new $s$ around $A_{2}$ until it passes through $A_{3}$ in a similar manner. (c) Perform $2 n-2$ more such steps, after which $s$ returns to its initial position. The total (directed) rotation angle $\\Theta$ of $s$ is clearly a multiple of $180^{\\circ}$. On the other hand, $s$ was never parallel to $\\ell$, which is possible only if $\\Theta=0$. Now it remains to partition all the $2 n$ angles into those where $s$ is rotated anticlockwise, and the others.","tier":0} +{"problem_type":"Combinatorics","problem_label":"C6","problem":"Let $n>1$ be an integer. Suppose we are given $2 n$ points in a plane such that no three of them are collinear. The points are to be labelled $A_{1}, A_{2}, \\ldots, A_{2 n}$ in some order. We then consider the $2 n$ angles $\\angle A_{1} A_{2} A_{3}, \\angle A_{2} A_{3} A_{4}, \\ldots, \\angle A_{2 n-2} A_{2 n-1} A_{2 n}, \\angle A_{2 n-1} A_{2 n} A_{1}$, $\\angle A_{2 n} A_{1} A_{2}$. We measure each angle in the way that gives the smallest positive value (i.e. between $0^{\\circ}$ and $180^{\\circ}$ ). Prove that there exists an ordering of the given points such that the resulting $2 n$ angles can be separated into two groups with the sum of one group of angles equal to the sum of the other group.","solution":"When tracing a cyclic path through the $A_{i}$ in order, with straight line segments between consecutive points, let $\\theta_{i}$ be the exterior angle at $A_{i}$, with a sign convention that it is positive if the path turns left and negative if the path turns right. Then $\\sum_{i=1}^{2 n} \\theta_{i}=360 k^{\\circ}$ for some integer $k$. Let $\\phi_{i}=\\angle A_{i-1} A_{i} A_{i+1}($ indices $\\bmod 2 n)$, defined as in the problem; thus $\\phi_{i}=180^{\\circ}-\\left|\\theta_{i}\\right|$. Let $L$ be the set of $i$ for which the path turns left at $A_{i}$ and let $R$ be the set for which it turns right. Then $S=\\sum_{i \\in L} \\phi_{i}-\\sum_{i \\in R} \\phi_{i}=(180(|L|-|R|)-360 k)^{\\circ}$, which is a multiple of $360^{\\circ}$ since the number of points is even. We will show that the points can be labelled such that $S=0$, in which case $L$ and $R$ satisfy the required condition of the problem. Note that the value of $S$ is defined for a slightly larger class of configurations: it is OK for two points to coincide, as long as they are not consecutive, and OK for three points to be collinear, as long as $A_{i}, A_{i+1}$ and $A_{i+2}$ do not appear on a line in that order. In what follows it will be convenient, although not strictly necessary, to consider such configurations. Consider how $S$ changes if a single one of the $A_{i}$ is moved along some straight-line path (not passing through any $A_{j}$ and not lying on any line $A_{j} A_{k}$, but possibly crossing such lines). Because $S$ is a multiple of $360^{\\circ}$, and the angles change continuously, $S$ can only change when a point moves between $R$ and $L$. Furthermore, if $\\phi_{j}=0$ when $A_{j}$ moves between $R$ and $L, S$ is unchanged; it only changes if $\\phi_{j}=180^{\\circ}$ when $A_{j}$ moves between those sets. For any starting choice of points, we will now construct a new configuration, with labels such that $S=0$, that can be perturbed into the original one without any $\\phi_{i}$ passing through $180^{\\circ}$, so that $S=0$ for the original configuration with those labels as well. Take some line such that there are $n$ points on each side of that line. The new configuration has $n$ copies of a single point on each side of the line, and a path that alternates between sides of the line; all angles are 0 , so this configuration has $S=0$. Perturbing the points into their original positions, while keeping each point on its side of the line, no angle $\\phi_{i}$ can pass through $180^{\\circ}$, because no straight line can go from one side of the line to the other and back. So the perturbation process leaves $S=0$. Comment. More complicated variants of this solution are also possible; for example, a path defined using four quadrants of the plane rather than just two half-planes.","tier":0} +{"problem_type":"Combinatorics","problem_label":"C6","problem":"Let $n>1$ be an integer. Suppose we are given $2 n$ points in a plane such that no three of them are collinear. The points are to be labelled $A_{1}, A_{2}, \\ldots, A_{2 n}$ in some order. We then consider the $2 n$ angles $\\angle A_{1} A_{2} A_{3}, \\angle A_{2} A_{3} A_{4}, \\ldots, \\angle A_{2 n-2} A_{2 n-1} A_{2 n}, \\angle A_{2 n-1} A_{2 n} A_{1}$, $\\angle A_{2 n} A_{1} A_{2}$. We measure each angle in the way that gives the smallest positive value (i.e. between $0^{\\circ}$ and $180^{\\circ}$ ). Prove that there exists an ordering of the given points such that the resulting $2 n$ angles can be separated into two groups with the sum of one group of angles equal to the sum of the other group.","solution":"First, let $\\ell$ be a line in the plane such that there are $n$ points on one side and the other $n$ points on the other side. For convenience, assume $\\ell$ is horizontal (otherwise, we can rotate the plane). Then we can use the terms \"above\", \"below\", \"left\" and \"right\" in the usual way. We denote the $n$ points above the line in an arbitrary order as $P_{1}, P_{2}, \\ldots, P_{n}$, and the $n$ points below the line as $Q_{1}, Q_{2}, \\ldots, Q_{n}$. If we connect $P_{i}$ and $Q_{j}$ with a line segment, the line segment will intersect with the line $\\ell$. Denote the intersection as $I_{i j}$. If $P_{i}$ is connected to $Q_{j}$ and $Q_{k}$, where $jk$, then the sign of $\\angle Q_{j} P_{i} Q_{k}$ is taken to be the same as for $\\angle Q_{k} P_{i} Q_{j}$. Similarly, we can define the sign of $\\angle P_{j} Q_{i} P_{k}$ with $j1$ be an integer. Suppose we are given $2 n$ points in a plane such that no three of them are collinear. The points are to be labelled $A_{1}, A_{2}, \\ldots, A_{2 n}$ in some order. We then consider the $2 n$ angles $\\angle A_{1} A_{2} A_{3}, \\angle A_{2} A_{3} A_{4}, \\ldots, \\angle A_{2 n-2} A_{2 n-1} A_{2 n}, \\angle A_{2 n-1} A_{2 n} A_{1}$, $\\angle A_{2 n} A_{1} A_{2}$. We measure each angle in the way that gives the smallest positive value (i.e. between $0^{\\circ}$ and $180^{\\circ}$ ). Prove that there exists an ordering of the given points such that the resulting $2 n$ angles can be separated into two groups with the sum of one group of angles equal to the sum of the other group.","solution":"We shall think instead of the problem as asking us to assign a weight $\\pm 1$ to each angle, such that the weighted sum of all the angles is zero. Given an ordering $A_{1}, \\ldots, A_{2 n}$ of the points, we shall assign weights according to the following recipe: walk in order from point to point, and assign the left turns +1 and the right turns -1 . This is the same weighting as in Solution 3, and as in that solution, the weighted sum is a multiple of $360^{\\circ}$. We now aim to show the following: Lemma. Transposing any two consecutive points in the ordering changes the weighted sum by $\\pm 360^{\\circ}$ or 0 . Knowing that, we can conclude quickly: if the ordering $A_{1}, \\ldots, A_{2 n}$ has weighted angle sum $360 k^{\\circ}$, then the ordering $A_{2 n}, \\ldots, A_{1}$ has weighted angle sum $-360 k^{\\circ}$ (since the angles are the same, but left turns and right turns are exchanged). We can reverse the ordering of $A_{1}$, $\\ldots, A_{2 n}$ by a sequence of transpositions of consecutive points, and in doing so the weighted angle sum must become zero somewhere along the way. We now prove that lemma: Proof. Transposing two points amounts to taking a section $A_{k} A_{k+1} A_{k+2} A_{k+3}$ as depicted, reversing the central line segment $A_{k+1} A_{k+2}$, and replacing its two neighbours with the dotted lines. ![](https:\/\/cdn.mathpix.com\/cropped\/2024_11_18_9ee8c2ac2c2f5002ff14g-048.jpg?height=424&width=1357&top_left_y=1156&top_left_x=354) Figure 1: Transposing two consecutive vertices: before (left) and afterwards (right) In each triangle, we alter the sum by $\\pm 180^{\\circ}$. Indeed, using (anticlockwise) directed angles modulo $360^{\\circ}$, we either add or subtract all three angles of each triangle. Hence both triangles together alter the sum by $\\pm 180 \\pm 180^{\\circ}$, which is $\\pm 360^{\\circ}$ or 0 .","tier":0} +{"problem_type":"Combinatorics","problem_label":"C7","problem":"There are 60 empty boxes $B_{1}, \\ldots, B_{60}$ in a row on a table and an unlimited supply of pebbles. Given a positive integer $n$, Alice and Bob play the following game. In the first round, Alice takes $n$ pebbles and distributes them into the 60 boxes as she wishes. Each subsequent round consists of two steps: (a) Bob chooses an integer $k$ with $1 \\leqslant k \\leqslant 59$ and splits the boxes into the two groups $B_{1}, \\ldots, B_{k}$ and $B_{k+1}, \\ldots, B_{60}$. (b) Alice picks one of these two groups, adds one pebble to each box in that group, and removes one pebble from each box in the other group. Bob wins if, at the end of any round, some box contains no pebbles. Find the smallest $n$ such that Alice can prevent Bob from winning. (Czech Republic)","solution":null,"tier":0} +{"problem_type":"Combinatorics","problem_label":"C8","problem":"Alice has a map of Wonderland, a country consisting of $n \\geqslant 2$ towns. For every pair of towns, there is a narrow road going from one town to the other. One day, all the roads are declared to be \"one way\" only. Alice has no information on the direction of the roads, but the King of Hearts has offered to help her. She is allowed to ask him a number of questions. For each question in turn, Alice chooses a pair of towns and the King of Hearts tells her the direction of the road connecting those two towns. Alice wants to know whether there is at least one town in Wonderland with at most one outgoing road. Prove that she can always find out by asking at most $4 n$ questions. Comment. This problem could be posed with an explicit statement about points being awarded for weaker bounds $c n$ for some $c>4$, in the style of IMO 2014 Problem 6. (Thailand)","solution":"We will show Alice needs to ask at most $4 n-7$ questions. Her strategy has the following phases. In what follows, $S$ is the set of towns that Alice, so far, does not know to have more than one outgoing road (so initially $|S|=n$ ). Phase 1. Alice chooses any two towns, say $A$ and $B$. Without loss of generality, suppose that the King of Hearts' answer is that the road goes from $A$ to $B$. At the end of this phase, Alice has asked 1 question. Phase 2. During this phase there is a single (variable) town $T$ that is known to have at least one incoming road but not yet known to have any outgoing roads. Initially, $T$ is $B$. Alice does the following $n-2$ times: she picks a town $X$ she has not asked about before, and asks the direction of the road between $T$ and $X$. If it is from $X$ to $T, T$ is unchanged; if it is from $T$ to $X, X$ becomes the new choice of town $T$, as the previous $T$ is now known to have an outgoing road. At the end of this phase, Alice has asked a total of $n-1$ questions. The final town $T$ is not yet known to have any outgoing roads, while every other town has exactly one outgoing road known. The undirected graph of roads whose directions are known is a tree. Phase 3. During this phase, Alice asks about the directions of all roads between $T$ and another town she has not previously asked about, stopping if she finds two outgoing roads from $T$. This phase involves at most $n-2$ questions. If she does not find two outgoing roads from $T$, she has answered her original question with at most $2 n-3 \\leqslant 4 n-7$ questions, so in what follows we suppose that she does find two outgoing roads, asking a total of $k$ questions in this phase, where $2 \\leqslant k \\leqslant n-2$ (and thus $n \\geqslant 4$ for what follows). For every question where the road goes towards $T$, the town at the other end is removed from $S$ (as it already had one outgoing road known), while the last question resulted in $T$ being removed from $S$. So at the end of this phase, $|S|=n-k+1$, while a total of $n+k-1$ questions have been asked. Furthermore, the undirected graph of roads within $S$ whose directions are known contains no cycles (as $T$ is no longer a member of $S$, all questions asked in this phase involved $T$ and the graph was a tree before this phase started). Every town in $S$ has exactly one outgoing road known (not necessarily to another town in $S$ ). Phase 4. During this phase, Alice repeatedly picks any pair of towns in $S$ for which she does not know the direction of the road between them. Because every town in $S$ has exactly one outgoing road known, this always results in the removal of one of those two towns from $S$. Because there are no cycles in the graph of roads of known direction within $S$, this can continue until there are at most 2 towns left in $S$. If it ends with $t$ towns left, $n-k+1-t$ questions were asked in this phase, so a total of $2 n-t$ questions have been asked. Phase 5. During this phase, Alice asks about all the roads from the remaining towns in $S$ that she has not previously asked about. She has definitely already asked about any road between those towns (if $t=2$ ). She must also have asked in one of the first two phases about at least one other road involving one of those towns (as those phases resulted in a tree with $n>2$ vertices). So she asks at most $t(n-t)-1$ questions in this phase. At the end of this phase, Alice knows whether any town has at most one outgoing road. If $t=1$, at most $3 n-3 \\leqslant 4 n-7$ questions were needed in total, while if $t=2$, at most $4 n-7$ questions were needed in total. Comment 1. The version of this problem originally submitted asked only for an upper bound of $5 n$, which is much simpler to prove. The Problem Selection Committee preferred a version with an asymptotically optimal constant. In the following comment, we will show that the constant is optimal. Comment 2. We will show that Alice cannot always find out by asking at most $4 n-3\\left(\\log _{2} n\\right)-$ 15 questions, if $n \\geqslant 8$. To show this, we suppose the King of Hearts is choosing the directions as he goes along, only picking the direction of a road when Alice asks about it for the first time. We provide a strategy for the King of Hearts that ensures that, after the given number of questions, the map is still consistent both with the existence of a town with at most one outgoing road, and with the nonexistence of such a town. His strategy has the following phases. When describing how the King of Hearts' answer to a question is determined below, we always assume he is being asked about a road for the first time (otherwise, he just repeats his previous answer for that road). This strategy is described throughout in graph-theoretic terms (vertices and edges rather than towns and roads). Phase 1. In this phase, we consider the undirected graph formed by edges whose directions are known. The phase terminates when there are exactly 8 connected components whose undirected graphs are trees. The following invariant is maintained: in a component with $k$ vertices whose undirected graph is a tree, every vertex has at most $\\left[\\log _{2} k\\right\\rfloor$ edges into it. - If the King of Hearts is asked about an edge between two vertices in the same component, or about an edge between two components at least one of which is not a tree, he chooses any direction for that edge arbitrarily. - If he is asked about an edge between a vertex in component $A$ that has $a$ vertices and is a tree and a vertex in component $B$ that has $b$ vertices and is a tree, suppose without loss of generality that $a \\geqslant b$. He then chooses the edge to go from $A$ to $B$. In this case, the new number of edges into any vertex is at most $\\max \\left\\{\\left\\lfloor\\log _{2} a\\right\\rfloor,\\left\\lfloor\\log _{2} b\\right\\rfloor+1\\right\\} \\leqslant\\left\\lfloor\\log _{2}(a+b)\\right\\rfloor$. In all cases, the invariant is preserved, and the number of tree components either remains unchanged or goes down by 1. Assuming Alice does not repeat questions, the process must eventually terminate with 8 tree components, and at least $n-8$ questions having been asked. Note that each tree component contains at least one vertex with no outgoing edges. Colour one such vertex in each tree component red. Phase 2. Let $V_{1}, V_{2}$ and $V_{3}$ be the three of the red vertices whose components are smallest (so their components together have at most $\\left\\lfloor\\frac{3}{8} n\\right\\rfloor$ vertices, with each component having at most $\\left\\lfloor\\frac{3}{8} n-2\\right\\rfloor$ vertices). Let sets $C_{1}, C_{2}, \\ldots$ be the connected components after removing the $V_{j}$. By construction, there are no edges with known direction between $C_{i}$ and $C_{j}$ for $i \\neq j$, and there are at least five such components. If at any point during this phase, the King of Hearts is asked about an edge within one of the $C_{i}$, he chooses an arbitrary direction. If he is asked about an edge between $C_{i}$ and $C_{j}$ for $i \\neq j$, he answers so that all edges go from $C_{i}$ to $C_{i+1}$ and $C_{i+2}$, with indices taken modulo the number of components, and chooses arbitrarily for other pairs. This ensures that all vertices other than the $V_{j}$ will have more than one outgoing edge. For edges involving one of the $V_{j}$ he answers as follows, so as to remain consistent for as long as possible with both possibilities for whether one of those vertices has at most one outgoing edge. Note that as they were red vertices, they have no outgoing edges at the start of this phase. For edges between two of the $V_{j}$, he answers that the edges go from $V_{1}$ to $V_{2}$, from $V_{2}$ to $V_{3}$ and from $V_{3}$ to $V_{1}$. For edges between $V_{j}$ and some other vertex, he always answers that the edge goes into $V_{j}$, except for the last such edge for which he is asked the question for any given $V_{j}$, for which he answers that the edge goes out of $V_{j}$. Thus, as long as at least one of the $V_{j}$ has not had the question answered for all the vertices that are not among the $V_{j}$, his answers are still compatible both with all vertices having more than one outgoing edge, and with that $V_{j}$ having only one outgoing edge. At the start of this phase, each of the $V_{j}$ has at most $\\left\\lfloor\\log _{2}\\left\\lfloor\\frac{3}{8} n-2\\right\\rfloor\\right\\rfloor<\\left(\\log _{2} n\\right)-1$ incoming edges. Thus, Alice cannot determine whether some vertex has only one outgoing edge within $3(n-$ $\\left.3-\\left(\\left(\\log _{2} n\\right)-1\\right)\\right)-1$ questions in this phase; that is, $4 n-3\\left(\\log _{2} n\\right)-15$ questions total. Comment 3. We can also improve the upper bound slightly, to $4 n-2\\left(\\log _{2} n\\right)+1$. (We do not know where the precise minimum number of questions lies between $4 n-3\\left(\\log _{2} n\\right)+O(1)$ and $4 n-2\\left(\\log _{2} n\\right)+$ $O(1)$.) Suppose $n \\geqslant 5$ (otherwise no questions are required at all). To do this, we replace Phases 1 and 2 of the given solution with a different strategy that also results in a spanning tree where one vertex $V$ is not known to have any outgoing edges, and all other vertices have exactly one outgoing edge known, but where there is more control over the numbers of incoming edges. In Phases 3 and 4 we then take more care about the order in which pairs of towns are chosen, to ensure that each of the remaining towns has already had a question asked about at least $\\log _{2} n+O(1)$ edges. Define trees $T_{m}$ with $2^{m}$ vertices, exactly one of which (the root) has no outgoing edges and the rest of which have exactly one outgoing edge, as follows: $T_{0}$ is a single vertex, while $T_{m}$ is constructed by joining the roots of two copies of $T_{m-1}$ with an edge in either direction. If $n=2^{m}$ we can readily ask $n-1$ questions, resulting in a tree $T_{m}$ for the edges with known direction: first ask about $2^{m-1}$ disjoint pairs of vertices, then about $2^{m-2}$ disjoint pairs of the roots of the resulting $T_{1}$ trees, and so on. For the general case, where $n$ is not a power of 2 , after $k$ stages of this process we have $\\left\\lfloor n \/ 2^{k}\\right\\rfloor$ trees, each of which is like $T_{k}$ but may have some extra vertices (but, however, a unique root). If there are an even number of trees, then ask about pairs of their roots. If there are an odd number (greater than 1) of trees, when a single $T_{k}$ is left over, ask about its root together with that of one of the $T_{k+1}$ trees. Say $m=\\left\\lfloor\\log _{2} n\\right\\rfloor$. The result of that process is a single $T_{m}$ tree, possibly with some extra vertices but still a unique root $V$. That root has at least $m$ incoming edges, and we may list vertices $V_{0}$, $\\ldots, V_{m-1}$ with edges to $V$, such that, for all $0 \\leqslant i1$ we have $D\\left(x_{i}, x_{j}\\right)>d$, since $$ \\left|x_{i}-x_{j}\\right|=\\left|x_{i+1}-x_{i}\\right|+\\left|x_{j}-x_{i+1}\\right| \\geqslant 2^{d}+2^{d}=2^{d+1} . $$ Now choose the minimal $i \\leqslant t$ and the maximal $j \\geqslant t+1$ such that $D\\left(x_{i}, x_{i+1}\\right)=$ $D\\left(x_{i+1}, x_{i+2}\\right)=\\cdots=D\\left(x_{j-1}, x_{j}\\right)=d$. Let $E$ be the set of all the $x_{s}$ with even indices $i \\leqslant s \\leqslant j, O$ be the set of those with odd indices $i \\leqslant s \\leqslant j$, and $R$ be the rest of the elements (so that $\\mathcal{S}$ is the disjoint union of $E, O$ and $R$ ). Set $\\mathcal{S}_{O}=R \\cup O$ and $\\mathcal{S}_{E}=R \\cup E$; we have $\\left|\\mathcal{S}_{O}\\right|<|\\mathcal{S}|$ and $\\left|\\mathcal{S}_{E}\\right|<|\\mathcal{S}|$, so $w\\left(\\mathcal{S}_{O}\\right), w\\left(\\mathcal{S}_{E}\\right) \\leqslant 1$ by the inductive hypothesis. Clearly, $r_{\\mathcal{S}_{O}}(x) \\leqslant r_{\\mathcal{S}}(x)$ and $r_{\\mathcal{S}_{E}}(x) \\leqslant r_{\\mathcal{S}}(x)$ for any $x \\in R$, and thus $$ \\begin{aligned} \\sum_{x \\in R} 2^{-r_{\\mathcal{S}}(x)} & =\\frac{1}{2} \\sum_{x \\in R}\\left(2^{-r_{\\mathcal{S}}(x)}+2^{-r_{\\mathcal{S}}(x)}\\right) \\\\ & \\leqslant \\frac{1}{2} \\sum_{x \\in R}\\left(2^{-r_{\\mathcal{S}_{O}}(x)}+2^{-r_{\\mathcal{S}_{E}}(x)}\\right) \\end{aligned} $$ On the other hand, for every $x \\in O$, there is no $y \\in \\mathcal{S}_{O}$ such that $D_{\\mathcal{S}_{O}}(x, y)=d$ (as all candidates from $\\mathcal{S}$ were in $E$ ). Hence, we have $r_{\\mathcal{S}_{O}}(x) \\leqslant r_{\\mathcal{S}}(x)-1$, and thus $$ \\sum_{x \\in O} 2^{-r \\mathcal{S}_{\\mathcal{S}}(x)} \\leqslant \\frac{1}{2} \\sum_{x \\in O} 2^{-r s_{O}(x)} $$ Similarly, for every $x \\in E$, we have $$ \\sum_{x \\in E} 2^{-r_{\\mathcal{S}}(x)} \\leqslant \\frac{1}{2} \\sum_{x \\in E} 2^{-r_{\\mathcal{S}_{E}}(x)} $$ We can then combine these to give $$ \\begin{aligned} w(S) & =\\sum_{x \\in R} 2^{-r_{\\mathcal{S}}(x)}+\\sum_{x \\in O} 2^{-r_{\\mathcal{S}}(x)}+\\sum_{x \\in E} 2^{-r_{\\mathcal{S}}(x)} \\\\ & \\leqslant \\frac{1}{2} \\sum_{x \\in R}\\left(2^{-r_{\\mathcal{S}_{O}}(x)}+2^{-r_{\\mathcal{S}_{E}}(x)}\\right)+\\frac{1}{2} \\sum_{x \\in O} 2^{-r_{\\mathcal{S}_{O}}(x)}+\\frac{1}{2} \\sum_{x \\in E} 2^{-r_{\\mathcal{S}_{E}}(x)} \\\\ & \\left.=\\frac{1}{2}\\left(\\sum_{x \\in \\mathcal{S}_{O}} 2^{-r_{S_{O}}(x)}+\\sum_{x \\in \\mathcal{S}_{E}} 2^{-r s_{S_{E}}(x)}\\right) \\quad \\text { (since } \\mathcal{S}_{O}=O \\cup R \\text { and } \\mathcal{S}_{E}=E \\cup R\\right) \\\\ & \\left.\\left.=\\frac{1}{2}\\left(w\\left(\\mathcal{S}_{O}\\right)+w\\left(\\mathcal{S}_{E}\\right)\\right)\\right) \\quad \\text { (by definition of } w(\\cdot)\\right) \\\\ & \\leqslant 1 \\quad \\text { (by the inductive hypothesis) } \\end{aligned} $$ which completes the induction. Comment 1. The sets $O$ and $E$ above are not the only ones we could have chosen. Indeed, we could instead have used the following definitions: Let $d$ be the maximal scale between two distinct elements of $\\mathcal{S}$; that is, $d=D\\left(x_{1}, x_{n}\\right)$. Let $O=\\left\\{x \\in \\mathcal{S}: D\\left(x, x_{n}\\right)=d\\right\\}$ (a 'left' part of the set) and let $E=\\left\\{x \\in \\mathcal{S}: D\\left(x_{1}, x\\right)=d\\right\\}$ (a 'right' part of the set). Note that these two sets are disjoint, and nonempty (since they contain $x_{1}$ and $x_{n}$ respectively). The rest of the proof is then the same as in Solution 1. Comment 2. Another possible set $\\mathcal{F}$ containing $2^{k}$ members could arise from considering a binary tree of height $k$, allocating a real number to each leaf, and trying to make the scale between the values of two leaves dependent only on the (graph) distance between them. The following construction makes this more precise. We build up sets $\\mathcal{F}_{k}$ recursively. Let $\\mathcal{F}_{0}=\\{0\\}$, and then let $\\mathcal{F}_{k+1}=\\mathcal{F}_{k} \\cup\\left\\{x+3 \\cdot 4^{k}: x \\in \\mathcal{F}_{k}\\right\\}$ (i.e. each half of $\\mathcal{F}_{k+1}$ is a copy of $F_{k}$ ). We have that $\\mathcal{F}_{k}$ is contained in the interval $\\left[0,4^{k+1}\\right.$ ), and so it follows by induction on $k$ that every member of $F_{k+1}$ has $k$ different scales in its own half of $F_{k+1}$ (by the inductive hypothesis), and only the single scale $2 k+1$ in the other half of $F_{k+1}$. Both of the constructions presented here have the property that every member of $\\mathcal{F}$ has exactly $k$ different scales in $\\mathcal{F}$. Indeed, it can be seen that this must hold (up to a slight perturbation) for any such maximal set. Suppose there were some element $x$ with only $k-1$ different scales in $\\mathcal{F}$ (and every other element had at most $k$ different scales). Then we take some positive real $\\epsilon$, and construct a new set $\\mathcal{F}^{\\prime}=\\{y: y \\in \\mathcal{F}, y \\leqslant x\\} \\cup\\{y+\\epsilon: y \\in \\mathcal{F}, y \\geqslant x\\}$. We have $\\left|\\mathcal{F}^{\\prime}\\right|=|\\mathcal{F}|+1$, and if $\\epsilon$ is sufficiently small then $\\mathcal{F}^{\\prime}$ will also satisfy the property that no member has more than $k$ different scales in $\\mathcal{F}^{\\prime}$. This observation might be used to motivate the idea of weighting members of an arbitrary set $\\mathcal{S}$ of reals according to how many different scales they have in $\\mathcal{S}$.","tier":0} +{"problem_type":"Geometry","problem_label":"G1","problem":"Let $A B C$ be a triangle. Circle $\\Gamma$ passes through $A$, meets segments $A B$ and $A C$ again at points $D$ and $E$ respectively, and intersects segment $B C$ at $F$ and $G$ such that $F$ lies between $B$ and $G$. The tangent to circle $B D F$ at $F$ and the tangent to circle $C E G$ at $G$ meet at point $T$. Suppose that points $A$ and $T$ are distinct. Prove that line $A T$ is parallel to $B C$. (Nigeria)","solution":"Notice that $\\angle T F B=\\angle F D A$ because $F T$ is tangent to circle $B D F$, and moreover $\\angle F D A=\\angle C G A$ because quadrilateral $A D F G$ is cyclic. Similarly, $\\angle T G B=\\angle G E C$ because $G T$ is tangent to circle $C E G$, and $\\angle G E C=\\angle C F A$. Hence, $$ \\angle T F B=\\angle C G A \\text { and } \\quad \\angle T G B=\\angle C F A $$ ![](https:\/\/cdn.mathpix.com\/cropped\/2024_11_18_9ee8c2ac2c2f5002ff14g-058.jpg?height=624&width=1326&top_left_y=930&top_left_x=365) Triangles $F G A$ and $G F T$ have a common side $F G$, and by (1) their angles at $F, G$ are the same. So, these triangles are congruent. So, their altitudes starting from $A$ and $T$, respectively, are equal and hence $A T$ is parallel to line $B F G C$. Comment. Alternatively, we can prove first that $T$ lies on $\\Gamma$. For example, this can be done by showing that $\\angle A F T=\\angle A G T$ using (1). Then the statement follows as $\\angle T A F=\\angle T G F=\\angle G F A$.","tier":0} +{"problem_type":"Geometry","problem_label":"G2","problem":"Let $A B C$ be an acute-angled triangle and let $D, E$, and $F$ be the feet of altitudes from $A, B$, and $C$ to sides $B C, C A$, and $A B$, respectively. Denote by $\\omega_{B}$ and $\\omega_{C}$ the incircles of triangles $B D F$ and $C D E$, and let these circles be tangent to segments $D F$ and $D E$ at $M$ and $N$, respectively. Let line $M N$ meet circles $\\omega_{B}$ and $\\omega_{C}$ again at $P \\neq M$ and $Q \\neq N$, respectively. Prove that $M P=N Q$. (Vietnam)","solution":"Denote the centres of $\\omega_{B}$ and $\\omega_{C}$ by $O_{B}$ and $O_{C}$, let their radii be $r_{B}$ and $r_{C}$, and let $B C$ be tangent to the two circles at $T$ and $U$, respectively. ![](https:\/\/cdn.mathpix.com\/cropped\/2024_11_18_9ee8c2ac2c2f5002ff14g-059.jpg?height=801&width=1014&top_left_y=659&top_left_x=521) From the cyclic quadrilaterals $A F D C$ and $A B D E$ we have $$ \\angle M D O_{B}=\\frac{1}{2} \\angle F D B=\\frac{1}{2} \\angle B A C=\\frac{1}{2} \\angle C D E=\\angle O_{C} D N, $$ so the right-angled triangles $D M O_{B}$ and $D N O_{C}$ are similar. The ratio of similarity between the two triangles is $$ \\frac{D N}{D M}=\\frac{O_{C} N}{O_{B} M}=\\frac{r_{C}}{r_{B}} . $$ Let $\\varphi=\\angle D M N$ and $\\psi=\\angle M N D$. The lines $F M$ and $E N$ are tangent to $\\omega_{B}$ and $\\omega_{C}$, respectively, so $$ \\angle M T P=\\angle F M P=\\angle D M N=\\varphi \\quad \\text { and } \\quad \\angle Q U N=\\angle Q N E=\\angle M N D=\\psi $$ (It is possible that $P$ or $Q$ coincides with $T$ or $U$, or lie inside triangles $D M T$ or $D U N$, respectively. To reduce case-sensitivity, we may use directed angles or simply ignore angles $M T P$ and $Q U N$.) In the circles $\\omega_{B}$ and $\\omega_{C}$ the lengths of chords $M P$ and $N Q$ are $$ M P=2 r_{B} \\cdot \\sin \\angle M T P=2 r_{B} \\cdot \\sin \\varphi \\quad \\text { and } \\quad N Q=2 r_{C} \\cdot \\sin \\angle Q U N=2 r_{C} \\cdot \\sin \\psi $$ By applying the sine rule to triangle $D N M$ we get $$ \\frac{D N}{D M}=\\frac{\\sin \\angle D M N}{\\sin \\angle M N D}=\\frac{\\sin \\varphi}{\\sin \\psi} $$ Finally, putting the above observations together, we get $$ \\frac{M P}{N Q}=\\frac{2 r_{B} \\sin \\varphi}{2 r_{C} \\sin \\psi}=\\frac{r_{B}}{r_{C}} \\cdot \\frac{\\sin \\varphi}{\\sin \\psi}=\\frac{D M}{D N} \\cdot \\frac{\\sin \\varphi}{\\sin \\psi}=\\frac{\\sin \\psi}{\\sin \\varphi} \\cdot \\frac{\\sin \\varphi}{\\sin \\psi}=1, $$ so $M P=N Q$ as required.","tier":0} +{"problem_type":"Geometry","problem_label":"G3","problem":"In triangle $A B C$, let $A_{1}$ and $B_{1}$ be two points on sides $B C$ and $A C$, and let $P$ and $Q$ be two points on segments $A A_{1}$ and $B B_{1}$, respectively, so that line $P Q$ is parallel to $A B$. On ray $P B_{1}$, beyond $B_{1}$, let $P_{1}$ be a point so that $\\angle P P_{1} C=\\angle B A C$. Similarly, on ray $Q A_{1}$, beyond $A_{1}$, let $Q_{1}$ be a point so that $\\angle C Q_{1} Q=\\angle C B A$. Show that points $P, Q, P_{1}$, and $Q_{1}$ are concyclic. (Ukraine)","solution":"Throughout the solution we use oriented angles. Let rays $A A_{1}$ and $B B_{1}$ intersect the circumcircle of $\\triangle A C B$ at $A_{2}$ and $B_{2}$, respectively. By $$ \\angle Q P A_{2}=\\angle B A A_{2}=\\angle B B_{2} A_{2}=\\angle Q B_{2} A_{2} $$ points $P, Q, A_{2}, B_{2}$ are concyclic; denote the circle passing through these points by $\\omega$. We shall prove that $P_{1}$ and $Q_{1}$ also lie on $\\omega$. ![](https:\/\/cdn.mathpix.com\/cropped\/2024_11_18_9ee8c2ac2c2f5002ff14g-060.jpg?height=615&width=858&top_left_y=909&top_left_x=602) By $$ \\angle C A_{2} A_{1}=\\angle C A_{2} A=\\angle C B A=\\angle C Q_{1} Q=\\angle C Q_{1} A_{1}, $$ points $C, Q_{1}, A_{2}, A_{1}$ are also concyclic. From that we get $$ \\angle Q Q_{1} A_{2}=\\angle A_{1} Q_{1} A_{2}=\\angle A_{1} C A_{2}=\\angle B C A_{2}=\\angle B A A_{2}=\\angle Q P A_{2} $$ so $Q_{1}$ lies on $\\omega$. It follows similarly that $P_{1}$ lies on $\\omega$.","tier":0} +{"problem_type":"Geometry","problem_label":"G3","problem":"In triangle $A B C$, let $A_{1}$ and $B_{1}$ be two points on sides $B C$ and $A C$, and let $P$ and $Q$ be two points on segments $A A_{1}$ and $B B_{1}$, respectively, so that line $P Q$ is parallel to $A B$. On ray $P B_{1}$, beyond $B_{1}$, let $P_{1}$ be a point so that $\\angle P P_{1} C=\\angle B A C$. Similarly, on ray $Q A_{1}$, beyond $A_{1}$, let $Q_{1}$ be a point so that $\\angle C Q_{1} Q=\\angle C B A$. Show that points $P, Q, P_{1}$, and $Q_{1}$ are concyclic. (Ukraine)","solution":"First consider the case when lines $P P_{1}$ and $Q Q_{1}$ intersect each other at some point $R$. Let line $P Q$ meet the sides $A C$ and $B C$ at $E$ and $F$, respectively. Then $$ \\angle P P_{1} C=\\angle B A C=\\angle P E C $$ so points $C, E, P, P_{1}$ lie on a circle; denote that circle by $\\omega_{P}$. It follows analogously that points $C, F, Q, Q_{1}$ lie on another circle; denote it by $\\omega_{Q}$. Let $A Q$ and $B P$ intersect at $T$. Applying Pappus' theorem to the lines $A A_{1} P$ and $B B_{1} Q$ provides that points $C=A B_{1} \\cap B A_{1}, R=A_{1} Q \\cap B_{1} P$ and $T=A Q \\cap B P$ are collinear. Let line $R C T$ meet $P Q$ and $A B$ at $S$ and $U$, respectively. From $A B \\| P Q$ we obtain $$ \\frac{S P}{S Q}=\\frac{U B}{U A}=\\frac{S F}{S E} $$ so $$ S P \\cdot S E=S Q \\cdot S F $$ ![](https:\/\/cdn.mathpix.com\/cropped\/2024_11_18_9ee8c2ac2c2f5002ff14g-061.jpg?height=792&width=1072&top_left_y=204&top_left_x=498) So, point $S$ has equal powers with respect to $\\omega_{P}$ and $\\omega_{Q}$, hence line $R C S$ is their radical axis; then $R$ also has equal powers to the circles, so $R P \\cdot R P_{1}=R Q \\cdot R Q_{1}$, proving that points $P, P_{1}, Q, Q_{1}$ are indeed concyclic. Now consider the case when $P P_{1}$ and $Q Q_{1}$ are parallel. Like in the previous case, let $A Q$ and $B P$ intersect at $T$. Applying Pappus' theorem again to the lines $A A_{1} P$ and $B B_{1} Q$, in this limit case it shows that line $C T$ is parallel to $P P_{1}$ and $Q Q_{1}$. Let line $C T$ meet $P Q$ and $A B$ at $S$ and $U$, as before. The same calculation as in the previous case shows that $S P \\cdot S E=S Q \\cdot S F$, so $S$ lies on the radical axis between $\\omega_{P}$ and $\\omega_{Q}$. ![](https:\/\/cdn.mathpix.com\/cropped\/2024_11_18_9ee8c2ac2c2f5002ff14g-061.jpg?height=800&width=1043&top_left_y=1413&top_left_x=512) Line $C S T$, that is the radical axis between $\\omega_{P}$ and $\\omega_{Q}$, is perpendicular to the line $\\ell$ of centres of $\\omega_{P}$ and $\\omega_{Q}$. Hence, the chords $P P_{1}$ and $Q Q_{1}$ are perpendicular to $\\ell$. So the quadrilateral $P P_{1} Q_{1} Q$ is an isosceles trapezium with symmetry axis $\\ell$, and hence is cyclic. Comment. There are several ways of solving the problem involving Pappus' theorem. For example, one may consider the points $K=P B_{1} \\cap B C$ and $L=Q A_{1} \\cap A C$. Applying Pappus' theorem to the lines $A A_{1} P$ and $Q B_{1} B$ we get that $K, L$, and $P Q \\cap A B$ are collinear, i.e. that $K L \\| A B$. Therefore, cyclicity of $P, Q, P_{1}$, and $Q_{1}$ is equivalent to that of $K, L, P_{1}$, and $Q_{1}$. The latter is easy after noticing that $C$ also lies on that circle. Indeed, e.g. $\\angle(L K, L C)=\\angle(A B, A C)=\\angle\\left(P_{1} K, P_{1} C\\right)$ shows that $K$ lies on circle $K L C$. This approach also has some possible degeneracy, as the points $K$ and $L$ may happen to be ideal.","tier":0} +{"problem_type":"Geometry","problem_label":"G4","problem":"Let $P$ be a point inside triangle $A B C$. Let $A P$ meet $B C$ at $A_{1}$, let $B P$ meet $C A$ at $B_{1}$, and let $C P$ meet $A B$ at $C_{1}$. Let $A_{2}$ be the point such that $A_{1}$ is the midpoint of $P A_{2}$, let $B_{2}$ be the point such that $B_{1}$ is the midpoint of $P B_{2}$, and let $C_{2}$ be the point such that $C_{1}$ is the midpoint of $P C_{2}$. Prove that points $A_{2}, B_{2}$, and $C_{2}$ cannot all lie strictly inside the circumcircle of triangle $A B C$. (Australia)","solution":"Since $$ \\angle A P B+\\angle B P C+\\angle C P A=2 \\pi=(\\pi-\\angle A C B)+(\\pi-\\angle B A C)+(\\pi-\\angle C B A), $$ at least one of the following inequalities holds: $$ \\angle A P B \\geqslant \\pi-\\angle A C B, \\quad \\angle B P C \\geqslant \\pi-\\angle B A C, \\quad \\angle C P A \\geqslant \\pi-\\angle C B A . $$ Without loss of generality, we assume that $\\angle B P C \\geqslant \\pi-\\angle B A C$. We have $\\angle B P C>\\angle B A C$ because $P$ is inside $\\triangle A B C$. So $\\angle B P C \\geqslant \\max (\\angle B A C, \\pi-\\angle B A C)$ and hence $$ \\sin \\angle B P C \\leqslant \\sin \\angle B A C . $$ Let the rays $A P, B P$, and $C P$ cross the circumcircle $\\Omega$ again at $A_{3}, B_{3}$, and $C_{3}$, respectively. We will prove that at least one of the ratios $\\frac{P B_{1}}{B_{1} B_{3}}$ and $\\frac{P C_{1}}{C_{1} C_{3}}$ is at least 1 , which yields that one of the points $B_{2}$ and $C_{2}$ does not lie strictly inside $\\Omega$. Because $A, B, C, B_{3}$ lie on a circle, the triangles $C B_{1} B_{3}$ and $B B_{1} A$ are similar, so $$ \\frac{C B_{1}}{B_{1} B_{3}}=\\frac{B B_{1}}{B_{1} A} $$ Applying the sine rule we obtain $$ \\frac{P B_{1}}{B_{1} B_{3}}=\\frac{P B_{1}}{C B_{1}} \\cdot \\frac{C B_{1}}{B_{1} B_{3}}=\\frac{P B_{1}}{C B_{1}} \\cdot \\frac{B B_{1}}{B_{1} A}=\\frac{\\sin \\angle A C P}{\\sin \\angle B P C} \\cdot \\frac{\\sin \\angle B A C}{\\sin \\angle P B A} . $$ Similarly, $$ \\frac{P C_{1}}{C_{1} C_{3}}=\\frac{\\sin \\angle P B A}{\\sin \\angle B P C} \\cdot \\frac{\\sin \\angle B A C}{\\sin \\angle A C P} $$ Multiplying these two equations we get $$ \\frac{P B_{1}}{B_{1} B_{3}} \\cdot \\frac{P C_{1}}{C_{1} C_{3}}=\\frac{\\sin ^{2} \\angle B A C}{\\sin ^{2} \\angle B P C} \\geqslant 1 $$ using (*), which yields the desired conclusion. Comment. It also cannot happen that all three points $A_{2}, B_{2}$, and $C_{2}$ lie strictly outside $\\Omega$. The same proof works almost literally, starting by assuming without loss of generality that $\\angle B P C \\leqslant \\pi-\\angle B A C$ and using $\\angle B P C>\\angle B A C$ to deduce that $\\sin \\angle B P C \\geqslant \\sin \\angle B A C$. It is possible for $A_{2}, B_{2}$, and $C_{2}$ all to lie on the circumcircle; from the above solution we may derive that this happens if and only if $P$ is the orthocentre of the triangle $A B C$, (which lies strictly inside $A B C$ if and only if $A B C$ is acute).","tier":0} +{"problem_type":"Geometry","problem_label":"G4","problem":"Let $P$ be a point inside triangle $A B C$. Let $A P$ meet $B C$ at $A_{1}$, let $B P$ meet $C A$ at $B_{1}$, and let $C P$ meet $A B$ at $C_{1}$. Let $A_{2}$ be the point such that $A_{1}$ is the midpoint of $P A_{2}$, let $B_{2}$ be the point such that $B_{1}$ is the midpoint of $P B_{2}$, and let $C_{2}$ be the point such that $C_{1}$ is the midpoint of $P C_{2}$. Prove that points $A_{2}, B_{2}$, and $C_{2}$ cannot all lie strictly inside the circumcircle of triangle $A B C$. (Australia)","solution":"Define points $A_{3}, B_{3}$, and $C_{3}$ as in Observe that $\\triangle P B C_{3} \\sim \\triangle P C B_{3}$. Let $X$ be the point on side $P B_{3}$ that corresponds to point $C_{1}$ on side $P C_{3}$ under this similarity. In other words, $X$ lies on segment $P B_{3}$ and satisfies $P X: X B_{3}=P C_{1}: C_{1} C_{3}$. It follows that $$ \\angle X C P=\\angle P B C_{1}=\\angle B_{3} B A=\\angle B_{3} C B_{1} . $$ Hence lines $C X$ and $C B_{1}$ are isogonal conjugates in $\\triangle P C B_{3}$. ![](https:\/\/cdn.mathpix.com\/cropped\/2024_11_18_9ee8c2ac2c2f5002ff14g-063.jpg?height=812&width=807&top_left_y=1207&top_left_x=533) Let $Y$ be the foot of the bisector of $\\angle B_{3} C P$ in $\\triangle P C B_{3}$. Since $P C_{1}0$ and $\\alpha+\\beta+\\gamma=1$. Then $$ A_{1}=\\frac{\\beta B+\\gamma C}{\\beta+\\gamma}=\\frac{1}{1-\\alpha} P-\\frac{\\alpha}{1-\\alpha} A, $$ so $$ A_{2}=2 A_{1}-P=\\frac{1+\\alpha}{1-\\alpha} P-\\frac{2 \\alpha}{1-\\alpha} A $$ Hence $$ \\left|A_{2}\\right|^{2}=\\left(\\frac{1+\\alpha}{1-\\alpha}\\right)^{2}|P|^{2}+\\left(\\frac{2 \\alpha}{1-\\alpha}\\right)^{2}|A|^{2}-\\frac{4 \\alpha(1+\\alpha)}{(1-\\alpha)^{2}} A \\cdot P $$ Using $|A|^{2}=1$ we obtain $$ \\frac{(1-\\alpha)^{2}}{2(1+\\alpha)}\\left|A_{2}\\right|^{2}=\\frac{1+\\alpha}{2}|P|^{2}+\\frac{2 \\alpha^{2}}{1+\\alpha}-2 \\alpha A \\cdot P $$ Likewise $$ \\frac{(1-\\beta)^{2}}{2(1+\\beta)}\\left|B_{2}\\right|^{2}=\\frac{1+\\beta}{2}|P|^{2}+\\frac{2 \\beta^{2}}{1+\\beta}-2 \\beta B \\cdot P $$ and $$ \\frac{(1-\\gamma)^{2}}{2(1+\\gamma)}\\left|C_{2}\\right|^{2}=\\frac{1+\\gamma}{2}|P|^{2}+\\frac{2 \\gamma^{2}}{1+\\gamma}-2 \\gamma C \\cdot P $$ Summing (1), (2) and (3) we obtain on the LHS the positive linear combination $$ \\text { LHS }=\\frac{(1-\\alpha)^{2}}{2(1+\\alpha)}\\left|A_{2}\\right|^{2}+\\frac{(1-\\beta)^{2}}{2(1+\\beta)}\\left|B_{2}\\right|^{2}+\\frac{(1-\\gamma)^{2}}{2(1+\\gamma)}\\left|C_{2}\\right|^{2} $$ and on the RHS the quantity $$ \\left(\\frac{1+\\alpha}{2}+\\frac{1+\\beta}{2}+\\frac{1+\\gamma}{2}\\right)|P|^{2}+\\left(\\frac{2 \\alpha^{2}}{1+\\alpha}+\\frac{2 \\beta^{2}}{1+\\beta}+\\frac{2 \\gamma^{2}}{1+\\gamma}\\right)-2(\\alpha A \\cdot P+\\beta B \\cdot P+\\gamma C \\cdot P) . $$ The first term is $2|P|^{2}$ and the last term is $-2 P \\cdot P$, so $$ \\begin{aligned} \\text { RHS } & =\\left(\\frac{2 \\alpha^{2}}{1+\\alpha}+\\frac{2 \\beta^{2}}{1+\\beta}+\\frac{2 \\gamma^{2}}{1+\\gamma}\\right) \\\\ & =\\frac{3 \\alpha-1}{2}+\\frac{(1-\\alpha)^{2}}{2(1+\\alpha)}+\\frac{3 \\beta-1}{2}+\\frac{(1-\\beta)^{2}}{2(1+\\beta)}+\\frac{3 \\gamma-1}{2}+\\frac{(1-\\gamma)^{2}}{2(1+\\gamma)} \\\\ & =\\frac{(1-\\alpha)^{2}}{2(1+\\alpha)}+\\frac{(1-\\beta)^{2}}{2(1+\\beta)}+\\frac{(1-\\gamma)^{2}}{2(1+\\gamma)} \\end{aligned} $$ Here we used the fact that $$ \\frac{3 \\alpha-1}{2}+\\frac{3 \\beta-1}{2}+\\frac{3 \\gamma-1}{2}=0 . $$ We have shown that a linear combination of $\\left|A_{1}\\right|^{2},\\left|B_{1}\\right|^{2}$, and $\\left|C_{1}\\right|^{2}$ with positive coefficients is equal to the sum of the coefficients. Therefore at least one of $\\left|A_{1}\\right|^{2},\\left|B_{1}\\right|^{2}$, and $\\left|C_{1}\\right|^{2}$ must be at least 1 , as required. Comment. This proof also works when $P$ is any point for which $\\alpha, \\beta, \\gamma>-1, \\alpha+\\beta+\\gamma=1$, and $\\alpha, \\beta, \\gamma \\neq 1$. (In any cases where $\\alpha=1$ or $\\beta=1$ or $\\gamma=1$, some points in the construction are not defined.) This page is intentionally left blank","tier":0} +{"problem_type":"Geometry","problem_label":"G5","problem":"Let $A B C D E$ be a convex pentagon with $C D=D E$ and $\\angle E D C \\neq 2 \\cdot \\angle A D B$. Suppose that a point $P$ is located in the interior of the pentagon such that $A P=A E$ and $B P=B C$. Prove that $P$ lies on the diagonal $C E$ if and only if $\\operatorname{area}(B C D)+\\operatorname{area}(A D E)=$ $\\operatorname{area}(A B D)+\\operatorname{area}(A B P)$. (Hungary)","solution":"Let $P^{\\prime}$ be the reflection of $P$ across line $A B$, and let $M$ and $N$ be the midpoints of $P^{\\prime} E$ and $P^{\\prime} C$ respectively. Convexity ensures that $P^{\\prime}$ is distinct from both $E$ and $C$, and hence from both $M$ and $N$. We claim that both the area condition and the collinearity condition in the problem are equivalent to the condition that the (possibly degenerate) right-angled triangles $A P^{\\prime} M$ and $B P^{\\prime} N$ are directly similar (equivalently, $A P^{\\prime} E$ and $B P^{\\prime} C$ are directly similar). ![](https:\/\/cdn.mathpix.com\/cropped\/2024_11_18_9ee8c2ac2c2f5002ff14g-066.jpg?height=535&width=549&top_left_y=806&top_left_x=756) For the equivalence with the collinearity condition, let $F$ denote the foot of the perpendicular from $P^{\\prime}$ to $A B$, so that $F$ is the midpoint of $P P^{\\prime}$. We have that $P$ lies on $C E$ if and only if $F$ lies on $M N$, which occurs if and only if we have the equality $\\angle A F M=\\angle B F N$ of signed angles modulo $\\pi$. By concyclicity of $A P^{\\prime} F M$ and $B F P^{\\prime} N$, this is equivalent to $\\angle A P^{\\prime} M=\\angle B P^{\\prime} N$, which occurs if and only if $A P^{\\prime} M$ and $B P^{\\prime} N$ are directly similar. ![](https:\/\/cdn.mathpix.com\/cropped\/2024_11_18_9ee8c2ac2c2f5002ff14g-066.jpg?height=306&width=441&top_left_y=1663&top_left_x=813) For the other equivalence with the area condition, we have the equality of signed areas $\\operatorname{area}(A B D)+\\operatorname{area}(A B P)=\\operatorname{area}\\left(A P^{\\prime} B D\\right)=\\operatorname{area}\\left(A P^{\\prime} D\\right)+\\operatorname{area}\\left(B D P^{\\prime}\\right)$. Using the identity $\\operatorname{area}(A D E)-\\operatorname{area}\\left(A P^{\\prime} D\\right)=\\operatorname{area}(A D E)+\\operatorname{area}\\left(A D P^{\\prime}\\right)=2$ area $(A D M)$, and similarly for $B$, we find that the area condition is equivalent to the equality $$ \\operatorname{area}(D A M)=\\operatorname{area}(D B N) . $$ Now note that $A$ and $B$ lie on the perpendicular bisectors of $P^{\\prime} E$ and $P^{\\prime} C$, respectively. If we write $G$ and $H$ for the feet of the perpendiculars from $D$ to these perpendicular bisectors respectively, then this area condition can be rewritten as $$ M A \\cdot G D=N B \\cdot H D $$ (In this condition, we interpret all lengths as signed lengths according to suitable conventions: for instance, we orient $P^{\\prime} E$ from $P^{\\prime}$ to $E$, orient the parallel line $D H$ in the same direction, and orient the perpendicular bisector of $P^{\\prime} E$ at an angle $\\pi \/ 2$ clockwise from the oriented segment $P^{\\prime} E$ - we adopt the analogous conventions at $B$.) ![](https:\/\/cdn.mathpix.com\/cropped\/2024_11_18_9ee8c2ac2c2f5002ff14g-067.jpg?height=529&width=575&top_left_y=204&top_left_x=758) To relate the signed lengths $G D$ and $H D$ to the triangles $A P^{\\prime} M$ and $B P^{\\prime} N$, we use the following calculation. Claim. Let $\\Gamma$ denote the circle centred on $D$ with both $E$ and $C$ on the circumference, and $h$ the power of $P^{\\prime}$ with respect to $\\Gamma$. Then we have the equality $$ G D \\cdot P^{\\prime} M=H D \\cdot P^{\\prime} N=\\frac{1}{4} h \\neq 0 . $$ Proof. Firstly, we have $h \\neq 0$, since otherwise $P^{\\prime}$ would lie on $\\Gamma$, and hence the internal angle bisectors of $\\angle E D P^{\\prime}$ and $\\angle P^{\\prime} D C$ would pass through $A$ and $B$ respectively. This would violate the angle inequality $\\angle E D C \\neq 2 \\cdot \\angle A D B$ given in the question. Next, let $E^{\\prime}$ denote the second point of intersection of $P^{\\prime} E$ with $\\Gamma$, and let $E^{\\prime \\prime}$ denote the point on $\\Gamma$ diametrically opposite $E^{\\prime}$, so that $E^{\\prime \\prime} E$ is perpendicular to $P^{\\prime} E$. The point $G$ lies on the perpendicular bisectors of the sides $P^{\\prime} E$ and $E E^{\\prime \\prime}$ of the right-angled triangle $P^{\\prime} E E^{\\prime \\prime}$; it follows that $G$ is the midpoint of $P^{\\prime} E^{\\prime \\prime}$. Since $D$ is the midpoint of $E^{\\prime} E^{\\prime \\prime}$, we have that $G D=\\frac{1}{2} P^{\\prime} E^{\\prime}$. Since $P^{\\prime} M=\\frac{1}{2} P^{\\prime} E$, we have $G D \\cdot P^{\\prime} M=\\frac{1}{4} P^{\\prime} E^{\\prime} \\cdot P^{\\prime} E=\\frac{1}{4} h$. The other equality $H D \\cdot P^{\\prime} N$ follows by exactly the same argument. ![](https:\/\/cdn.mathpix.com\/cropped\/2024_11_18_9ee8c2ac2c2f5002ff14g-067.jpg?height=727&width=672&top_left_y=1664&top_left_x=726) From this claim, we see that the area condition is equivalent to the equality $$ \\left(M A: P^{\\prime} M\\right)=\\left(N B: P^{\\prime} N\\right) $$ of ratios of signed lengths, which is equivalent to direct similarity of $A P^{\\prime} M$ and $B P^{\\prime} N$, as desired.","tier":0} +{"problem_type":"Geometry","problem_label":"G5","problem":"Let $A B C D E$ be a convex pentagon with $C D=D E$ and $\\angle E D C \\neq 2 \\cdot \\angle A D B$. Suppose that a point $P$ is located in the interior of the pentagon such that $A P=A E$ and $B P=B C$. Prove that $P$ lies on the diagonal $C E$ if and only if $\\operatorname{area}(B C D)+\\operatorname{area}(A D E)=$ $\\operatorname{area}(A B D)+\\operatorname{area}(A B P)$. (Hungary)","solution":"Along the perpendicular bisector of $C E$, define the linear function $$ f(X)=\\operatorname{area}(B C X)+\\operatorname{area}(A X E)-\\operatorname{area}(A B X)-\\operatorname{area}(A B P), $$ where, from now on, we always use signed areas. Thus, we want to show that $C, P, E$ are collinear if and only if $f(D)=0$. ![](https:\/\/cdn.mathpix.com\/cropped\/2024_11_18_9ee8c2ac2c2f5002ff14g-068.jpg?height=433&width=512&top_left_y=503&top_left_x=772) Let $P^{\\prime}$ be the reflection of $P$ across line $A B$. The point $P^{\\prime}$ does not lie on the line $C E$. To see this, we let $A^{\\prime \\prime}$ and $B^{\\prime \\prime}$ be the points obtained from $A$ and $B$ by dilating with scale factor 2 about $P^{\\prime}$, so that $P$ is the orthogonal projection of $P^{\\prime}$ onto $A^{\\prime \\prime} B^{\\prime \\prime}$. Since $A$ lies on the perpendicular bisector of $P^{\\prime} E$, the triangle $A^{\\prime \\prime} E P^{\\prime}$ is right-angled at $E$ (and $B^{\\prime \\prime} C P^{\\prime}$ similarly). If $P^{\\prime}$ were to lie on $C E$, then the lines $A^{\\prime \\prime} E$ and $B^{\\prime \\prime} C$ would be perpendicular to $C E$ and $A^{\\prime \\prime}$ and $B^{\\prime \\prime}$ would lie on the opposite side of $C E$ to $D$. It follows that the line $A^{\\prime \\prime} B^{\\prime \\prime}$ does not meet triangle $C D E$, and hence point $P$ does not lie inside $C D E$. But then $P$ must lie inside $A B C E$, and it is clear that such a point cannot reflect to a point $P^{\\prime}$ on $C E$. We thus let $O$ be the centre of the circle $C E P^{\\prime}$. The lines $A O$ and $B O$ are the perpendicular bisectors of $E P^{\\prime}$ and $C P^{\\prime}$, respectively, so $$ \\begin{aligned} \\operatorname{area}(B C O)+\\operatorname{area}(A O E) & =\\operatorname{area}\\left(O P^{\\prime} B\\right)+\\operatorname{area}\\left(P^{\\prime} O A\\right)=\\operatorname{area}\\left(P^{\\prime} B O A\\right) \\\\ & =\\operatorname{area}(A B O)+\\operatorname{area}\\left(B A P^{\\prime}\\right)=\\operatorname{area}(A B O)+\\operatorname{area}(A B P), \\end{aligned} $$ and hence $f(O)=0$. Notice that if point $O$ coincides with $D$ then points $A, B$ lie in angle domain $C D E$ and $\\angle E O C=2 \\cdot \\angle A O B$, which is not allowed. So, $O$ and $D$ must be distinct. Since $f$ is linear and vanishes at $O$, it follows that $f(D)=0$ if and only if $f$ is constant zero - we want to show this occurs if and only if $C, P, E$ are collinear. ![](https:\/\/cdn.mathpix.com\/cropped\/2024_11_18_9ee8c2ac2c2f5002ff14g-068.jpg?height=556&width=1254&top_left_y=1906&top_left_x=408) In the one direction, suppose firstly that $C, P, E$ are not collinear, and let $T$ be the centre of the circle $C E P$. The same calculation as above provides $$ \\operatorname{area}(B C T)+\\operatorname{area}(A T E)=\\operatorname{area}(P B T A)=\\operatorname{area}(A B T)-\\operatorname{area}(A B P) $$ $$ f(T)=-2 \\operatorname{area}(A B P) \\neq 0 $$ Hence, the linear function $f$ is nonconstant with its zero is at $O$, so that $f(D) \\neq 0$. In the other direction, suppose that the points $C, P, E$ are collinear. We will show that $f$ is constant zero by finding a second point (other than $O$ ) at which it vanishes. ![](https:\/\/cdn.mathpix.com\/cropped\/2024_11_18_9ee8c2ac2c2f5002ff14g-069.jpg?height=410&width=1098&top_left_y=366&top_left_x=479) Let $Q$ be the reflection of $P$ across the midpoint of $A B$, so $P A Q B$ is a parallelogram. It is easy to see that $Q$ is on the perpendicular bisector of $C E$; for instance if $A^{\\prime}$ and $B^{\\prime}$ are the points produced from $A$ and $B$ by dilating about $P$ with scale factor 2, then the projection of $Q$ to $C E$ is the midpoint of the projections of $A^{\\prime}$ and $B^{\\prime}$, which are $E$ and $C$ respectively. The triangles $B C Q$ and $A Q E$ are indirectly congruent, so $$ f(Q)=(\\operatorname{area}(B C Q)+\\operatorname{area}(A Q E))-(\\operatorname{area}(A B Q)-\\operatorname{area}(B A P))=0-0=0 $$ The points $O$ and $Q$ are distinct. To see this, consider the circle $\\omega$ centred on $Q$ with $P^{\\prime}$ on the circumference; since triangle $P P^{\\prime} Q$ is right-angled at $P^{\\prime}$, it follows that $P$ lies outside $\\omega$. On the other hand, $P$ lies between $C$ and $E$ on the line $C P E$. It follows that $C$ and $E$ cannot both lie on $\\omega$, so that $\\omega$ is not the circle $C E P^{\\prime}$ and $Q \\neq O$. Since $O$ and $Q$ are distinct zeroes of the linear function $f$, we have $f(D)=0$ as desired. Comment 1. The condition $\\angle E D C \\neq 2 \\cdot \\angle A D B$ cannot be omitted. If $D$ is the centre of circle $C E P^{\\prime}$, then the condition on triangle areas is satisfied automatically, without having $P$ on line $C E$. Comment 2. The \"only if\" part of this problem is easier than the \"if\" part. For example, in the second part of Solution 2, the triangles $E A Q$ and $Q B C$ are indirectly congruent, so the sum of their areas is 0 , and $D C Q E$ is a kite. Now one can easily see that $\\angle(A Q, D E)=\\angle(C D, C B)$ and $\\angle(B Q, D C)=\\angle(E D, E A)$, whence area $(B C D)=\\operatorname{area}(A Q D)+\\operatorname{area}(E Q A)$ and $\\operatorname{area}(A D E)=$ $\\operatorname{area}(B D Q)+\\operatorname{area}(B Q C)$, which yields the result. Comment 3. The origin of the problem is the following observation. Let $A B D H$ be a tetrahedron and consider the sphere $\\mathcal{S}$ that is tangent to the four face planes, internally to planes $A D H$ and $B D H$ and externally to $A B D$ and $A B H$ (or vice versa). It is known that the sphere $\\mathcal{S}$ exists if and only if area $(A D H)+\\operatorname{area}(B D H) \\neq \\operatorname{area}(A B H)+\\operatorname{area}(A B D)$; this relation comes from the usual formula for the volume of the tetrahedron. Let $T, T_{a}, T_{b}, T_{d}$ be the points of tangency between the sphere and the four planes, as shown in the picture. Rotate the triangle $A B H$ inward, the triangles $B D H$ and $A D H$ outward, into the triangles $A B P, B D C$ and $A D E$, respectively, in the plane $A B D$. Notice that the points $T_{d}, T_{a}, T_{b}$ are rotated to $T$, so we have $H T_{a}=H T_{b}=H T_{d}=P T=C T=E T$. Therefore, the point $T$ is the centre of the circle $C E P$. Hence, if the sphere exists then $C, E, P$ cannot be collinear. If the condition $\\angle E D C \\neq 2 \\cdot \\angle A D B$ is replaced by the constraint that the angles $\\angle E D A, \\angle A D B$ and $\\angle B D C$ satisfy the triangle inequality, it enables reconstructing the argument with the tetrahedron and the tangent sphere. ![](https:\/\/cdn.mathpix.com\/cropped\/2024_11_18_9ee8c2ac2c2f5002ff14g-070.jpg?height=1069&width=1306&top_left_y=825&top_left_x=381)","tier":0} +{"problem_type":"Geometry","problem_label":"G6","problem":"Let $I$ be the incentre of acute-angled triangle $A B C$. Let the incircle meet $B C, C A$, and $A B$ at $D, E$, and $F$, respectively. Let line $E F$ intersect the circumcircle of the triangle at $P$ and $Q$, such that $F$ lies between $E$ and $P$. Prove that $\\angle D P A+\\angle A Q D=\\angle Q I P$. (Slovakia)","solution":"Let $N$ and $M$ be the midpoints of the arcs $\\widehat{B C}$ of the circumcircle, containing and opposite vertex $A$, respectively. By $\\angle F A E=\\angle B A C=\\angle B N C$, the right-angled kites $A F I E$ and $N B M C$ are similar. Consider the spiral similarity $\\varphi$ (dilation in case of $A B=A C$ ) that moves $A F I E$ to $N B M C$. The directed angle in which $\\varphi$ changes directions is $\\angle(A F, N B)$, same as $\\angle(A P, N P)$ and $\\angle(A Q, N Q)$; so lines $A P$ and $A Q$ are mapped to lines $N P$ and $N Q$, respectively. Line $E F$ is mapped to $B C$; we can see that the intersection points $P=E F \\cap A P$ and $Q=E F \\cap A Q$ are mapped to points $B C \\cap N P$ and $B C \\cap N Q$, respectively. Denote these points by $P^{\\prime}$ and $Q^{\\prime}$, respectively. ![](https:\/\/cdn.mathpix.com\/cropped\/2024_11_18_9ee8c2ac2c2f5002ff14g-071.jpg?height=689&width=1512&top_left_y=883&top_left_x=272) Let $L$ be the midpoint of $B C$. We claim that points $P, Q, D, L$ are concyclic (if $D=L$ then line $B C$ is tangent to circle $P Q D$ ). Let $P Q$ and $B C$ meet at $Z$. By applying Menelaus' theorem to triangle $A B C$ and line $E F Z$, we have $$ \\frac{B D}{D C}=\\frac{B F}{F A} \\cdot \\frac{A E}{E C}=-\\frac{B Z}{Z C}, $$ so the pairs $B, C$ and $D, Z$ are harmonic. It is well-known that this implies $Z B \\cdot Z C=Z D \\cdot Z L$. (The inversion with pole $Z$ that swaps $B$ and $C$ sends $Z$ to infinity and $D$ to the midpoint of $B C$, because the cross-ratio is preserved.) Hence, $Z D \\cdot Z L=Z B \\cdot Z C=Z P \\cdot Z Q$ by the power of $Z$ with respect to the circumcircle; this proves our claim. By $\\angle M P P^{\\prime}=\\angle M Q Q^{\\prime}=\\angle M L P^{\\prime}=\\angle M L Q^{\\prime}=90^{\\circ}$, the quadrilaterals $M L P P^{\\prime}$ and $M L Q Q^{\\prime}$ are cyclic. Then the problem statement follows by $$ \\begin{aligned} \\angle D P A+\\angle A Q D & =360^{\\circ}-\\angle P A Q-\\angle Q D P=360^{\\circ}-\\angle P N Q-\\angle Q L P \\\\ & =\\angle L P N+\\angle N Q L=\\angle P^{\\prime} M L+\\angle L M Q^{\\prime}=\\angle P^{\\prime} M Q^{\\prime}=\\angle P I Q . \\end{aligned} $$","tier":0} +{"problem_type":"Geometry","problem_label":"G6","problem":"Let $I$ be the incentre of acute-angled triangle $A B C$. Let the incircle meet $B C, C A$, and $A B$ at $D, E$, and $F$, respectively. Let line $E F$ intersect the circumcircle of the triangle at $P$ and $Q$, such that $F$ lies between $E$ and $P$. Prove that $\\angle D P A+\\angle A Q D=\\angle Q I P$. (Slovakia)","solution":"Define the point $M$ and the same spiral similarity $\\varphi$ as in the previous solution. (The point $N$ is not necessary.) It is well-known that the centre of the spiral similarity that maps $F, E$ to $B, C$ is the Miquel point of the lines $F E, B C, B F$ and $C E$; that is, the second intersection of circles $A B C$ and $A E F$. Denote that point by $S$. By $\\varphi(F)=B$ and $\\varphi(E)=C$ the triangles $S B F$ and $S C E$ are similar, so we have $$ \\frac{S B}{S C}=\\frac{B F}{C E}=\\frac{B D}{C D} $$ By the converse of the angle bisector theorem, that indicates that line $S D$ bisects $\\angle B S C$ and hence passes through $M$. Let $K$ be the intersection point of lines $E F$ and $S I$. Notice that $\\varphi$ sends points $S, F, E, I$ to $S, B, C, M$, so $\\varphi(K)=\\varphi(F E \\cap S I)=B C \\cap S M=D$. By $\\varphi(I)=M$, we have $K D \\| I M$. ![](https:\/\/cdn.mathpix.com\/cropped\/2024_11_18_9ee8c2ac2c2f5002ff14g-072.jpg?height=669&width=706&top_left_y=839&top_left_x=675) We claim that triangles $S P I$ and $S D Q$ are similar, and so are triangles $S P D$ and $S I Q$. Let ray $S I$ meet the circumcircle again at $L$. Note that the segment $E F$ is perpendicular to the angle bisector $A M$. Then by $\\angle A M L=\\angle A S L=\\angle A S I=90^{\\circ}$, we have $M L \\| P Q$. Hence, $\\widetilde{P L}=\\widetilde{M Q}$ and therefore $\\angle P S L=\\angle M S Q=\\angle D S Q$. By $\\angle Q P S=\\angle Q M S$, the triangles $S P K$ and $S M Q$ are similar. Finally, $$ \\frac{S P}{S I}=\\frac{S P}{S K} \\cdot \\frac{S K}{S I}=\\frac{S M}{S Q} \\cdot \\frac{S D}{S M}=\\frac{S D}{S Q} $$ shows that triangles $S P I$ and $S D Q$ are similar. The second part of the claim can be proved analogously. Now the problem statement can be proved by $$ \\angle D P A+\\angle A Q D=\\angle D P S+\\angle S Q D=\\angle Q I S+\\angle S I P=\\angle Q I P . $$","tier":0} +{"problem_type":"Geometry","problem_label":"G6","problem":"Let $I$ be the incentre of acute-angled triangle $A B C$. Let the incircle meet $B C, C A$, and $A B$ at $D, E$, and $F$, respectively. Let line $E F$ intersect the circumcircle of the triangle at $P$ and $Q$, such that $F$ lies between $E$ and $P$. Prove that $\\angle D P A+\\angle A Q D=\\angle Q I P$. (Slovakia)","solution":"Denote the circumcircle of triangle $A B C$ by $\\Gamma$, and let rays $P D$ and $Q D$ meet $\\Gamma$ again at $V$ and $U$, respectively. We will show that $A U \\perp I P$ and $A V \\perp I Q$. Then the problem statement will follow as $$ \\angle D P A+\\angle A Q D=\\angle V U A+\\angle A V U=180^{\\circ}-\\angle U A V=\\angle Q I P . $$ Let $M$ be the midpoint of arc $\\widehat{B U V C}$ and let $N$ be the midpoint of arc $\\widehat{C A B}$; the lines $A I M$ and $A N$ being the internal and external bisectors of angle $B A C$, respectively, are perpendicular. Let the tangents drawn to $\\Gamma$ at $B$ and $C$ meet at $R$; let line $P Q$ meet $A U, A I, A V$ and $B C$ at $X, T, Y$ and $Z$, respectively. As in Solution 1, we observe that the pairs $B, C$ and $D, Z$ are harmonic. Projecting these points from $Q$ onto the circumcircle, we can see that $B, C$ and $U, P$ are also harmonic. Analogously, the pair $V, Q$ is harmonic with $B, C$. Consider the inversion about the circle with centre $R$, passing through $B$ and $C$. Points $B$ and $C$ are fixed points, so this inversion exchanges every point of $\\Gamma$ by its harmonic pair with respect to $B, C$. In particular, the inversion maps points $B, C, N, U, V$ to points $B, C, M, P, Q$, respectively. Combine the inversion with projecting $\\Gamma$ from $A$ to line $P Q$; the points $B, C, M, P, Q$ are projected to $F, E, T, P, Q$, respectively. ![](https:\/\/cdn.mathpix.com\/cropped\/2024_11_18_9ee8c2ac2c2f5002ff14g-073.jpg?height=1006&width=1384&top_left_y=1082&top_left_x=339) The combination of these two transformations is projective map from the lines $A B, A C$, $A N, A U, A V$ to $I F, I E, I T, I P, I Q$, respectively. On the other hand, we have $A B \\perp I F$, $A C \\perp I E$ and $A N \\perp A T$, so the corresponding lines in these two pencils are perpendicular. This proves $A U \\perp I P$ and $A V \\perp I Q$, and hence completes the solution.","tier":0} +{"problem_type":"Geometry","problem_label":"G7","problem":"The incircle $\\omega$ of acute-angled scalene triangle $A B C$ has centre $I$ and meets sides $B C$, $C A$, and $A B$ at $D, E$, and $F$, respectively. The line through $D$ perpendicular to $E F$ meets $\\omega$ again at $R$. Line $A R$ meets $\\omega$ again at $P$. The circumcircles of triangles $P C E$ and $P B F$ meet again at $Q \\neq P$. Prove that lines $D I$ and $P Q$ meet on the external bisector of angle $B A C$. (India)","solution":"Step 1. The external bisector of $\\angle B A C$ is the line through $A$ perpendicular to $I A$. Let $D I$ meet this line at $L$ and let $D I$ meet $\\omega$ at $K$. Let $N$ be the midpoint of $E F$, which lies on $I A$ and is the pole of line $A L$ with respect to $\\omega$. Since $A N \\cdot A I=A E^{2}=A R \\cdot A P$, the points $R$, $N, I$, and $P$ are concyclic. As $I R=I P$, the line $N I$ is the external bisector of $\\angle P N R$, so $P N$ meets $\\omega$ again at the point symmetric to $R$ with respect to $A N-i . e$ at $K$. Let $D N$ cross $\\omega$ again at $S$. Opposite sides of any quadrilateral inscribed in the circle $\\omega$ meet on the polar line of the intersection of the diagonals with respect to $\\omega$. Since $L$ lies on the polar line $A L$ of $N$ with respect to $\\omega$, the line $P S$ must pass through $L$. Thus it suffices to prove that the points $S, Q$, and $P$ are collinear. ![](https:\/\/cdn.mathpix.com\/cropped\/2024_11_18_9ee8c2ac2c2f5002ff14g-074.jpg?height=875&width=1350&top_left_y=1176&top_left_x=361) Step 2. Let $\\Gamma$ be the circumcircle of $\\triangle B I C$. Notice that $$ \\begin{aligned} & \\angle(B Q, Q C)=\\angle(B Q, Q P)+\\angle(P Q, Q C)=\\angle(B F, F P)+\\angle(P E, E C) \\\\ &=\\angle(E F, E P)+\\angle(F P, F E)=\\angle(F P, E P)=\\angle(D F, D E)=\\angle(B I, I C) \\end{aligned} $$ so $Q$ lies on $\\Gamma$. Let $Q P$ meet $\\Gamma$ again at $T$. It will now suffice to prove that $S, P$, and $T$ are collinear. Notice that $\\angle(B I, I T)=\\angle(B Q, Q T)=\\angle(B F, F P)=\\angle(F K, K P)$. Note $F D \\perp F K$ and $F D \\perp B I$ so $F K \\| B I$ and hence $I T$ is parallel to the line $K N P$. Since $D I=I K$, the line $I T$ crosses $D N$ at its midpoint $M$. Step 3. Let $F^{\\prime}$ and $E^{\\prime}$ be the midpoints of $D E$ and $D F$, respectively. Since $D E^{\\prime} \\cdot E^{\\prime} F=D E^{\\prime 2}=$ $B E^{\\prime} \\cdot E^{\\prime} I$, the point $E^{\\prime}$ lies on the radical axis of $\\omega$ and $\\Gamma$; the same holds for $F^{\\prime}$. Therefore, this radical axis is $E^{\\prime} F^{\\prime}$, and it passes through $M$. Thus $I M \\cdot M T=D M \\cdot M S$, so $S, I, D$, and $T$ are concyclic. This shows $\\angle(D S, S T)=\\angle(D I, I T)=\\angle(D K, K P)=\\angle(D S, S P)$, whence the points $S, P$, and $T$ are collinear, as desired. ![](https:\/\/cdn.mathpix.com\/cropped\/2024_11_18_9ee8c2ac2c2f5002ff14g-075.jpg?height=1089&width=992&top_left_y=198&top_left_x=532) Comment. Here is a longer alternative proof in step 1 that $P, S$, and $L$ are collinear, using a circular inversion instead of the fact that opposite sides of a quadrilateral inscribed in a circle $\\omega$ meet on the polar line with respect to $\\omega$ of the intersection of the diagonals. Let $G$ be the foot of the altitude from $N$ to the line $D I K L$. Observe that $N, G, K, S$ are concyclic (opposite right angles) so $$ \\angle D I P=2 \\angle D K P=\\angle G K N+\\angle D S P=\\angle G S N+\\angle N S P=\\angle G S P, $$ hence $I, G, S, P$ are concyclic. We have $I G \\cdot I L=I N \\cdot I A=r^{2}$ since $\\triangle I G N \\sim \\triangle I A L$. Inverting the circle $I G S P$ in circle $\\omega$, points $P$ and $S$ are fixed and $G$ is taken to $L$ so we find that $P, S$, and $L$ are collinear.","tier":0} +{"problem_type":"Geometry","problem_label":"G7","problem":"The incircle $\\omega$ of acute-angled scalene triangle $A B C$ has centre $I$ and meets sides $B C$, $C A$, and $A B$ at $D, E$, and $F$, respectively. The line through $D$ perpendicular to $E F$ meets $\\omega$ again at $R$. Line $A R$ meets $\\omega$ again at $P$. The circumcircles of triangles $P C E$ and $P B F$ meet again at $Q \\neq P$. Prove that lines $D I$ and $P Q$ meet on the external bisector of angle $B A C$. (India)","solution":"We start as in Step 1. Let $A R$ meet the circumcircle $\\Omega$ of $A B C$ again at $X$. The lines $A R$ and $A K$ are isogonal in the angle $B A C$; it is well known that in this case $X$ is the tangency point of $\\Omega$ with the $A$-mixtilinear circle. It is also well known that for this point $X$, the line $X I$ crosses $\\Omega$ again at the midpoint $M^{\\prime}$ of arc $B A C$. Step 2. Denote the circles $B F P$ and $C E P$ by $\\Omega_{B}$ and $\\Omega_{C}$, respectively. Let $\\Omega_{B}$ cross $A R$ and $E F$ again at $U$ and $Y$, respectively. We have $$ \\angle(U B, B F)=\\angle(U P, P F)=\\angle(R P, P F)=\\angle(R F, F A) $$ so $U B \\| R F$. ![](https:\/\/cdn.mathpix.com\/cropped\/2024_11_18_9ee8c2ac2c2f5002ff14g-076.jpg?height=1009&width=1474&top_left_y=883&top_left_x=248) Next, we show that the points $B, I, U$, and $X$ are concyclic. Since $$ \\angle(U B, U X)=\\angle(R F, R X)=\\angle(A F, A R)+\\angle(F R, F A)=\\angle\\left(M^{\\prime} B, M^{\\prime} X\\right)+\\angle(D R, D F) $$ it suffices to prove $\\angle(I B, I X)=\\angle\\left(M^{\\prime} B, M^{\\prime} X\\right)+\\angle(D R, D F)$, or $\\angle\\left(I B, M^{\\prime} B\\right)=\\angle(D R, D F)$. But both angles equal $\\angle(C I, C B)$, as desired. (This is where we used the fact that $M^{\\prime}$ is the midpoint of $\\operatorname{arc} B A C$ of $\\Omega$.) It follows now from circles $B U I X$ and $B P U F Y$ that $$ \\begin{aligned} \\angle(I U, U B)=\\angle(I X, B X)=\\angle\\left(M^{\\prime} X, B X\\right)= & \\frac{\\pi-\\angle A}{2} \\\\ & =\\angle(E F, A F)=\\angle(Y F, B F)=\\angle(Y U, B U) \\end{aligned} $$ so the points $Y, U$, and $I$ are collinear. Let $E F$ meet $B C$ at $W$. We have $$ \\angle(I Y, Y W)=\\angle(U Y, F Y)=\\angle(U B, F B)=\\angle(R F, A F)=\\angle(C I, C W) $$ so the points $W, Y, I$, and $C$ are concyclic. Similarly, if $V$ and $Z$ are the second meeting points of $\\Omega_{C}$ with $A R$ and $E F$, we get that the 4-tuples $(C, V, I, X)$ and $(B, I, Z, W)$ are both concyclic. Step 3. Let $Q^{\\prime}=C Y \\cap B Z$. We will show that $Q^{\\prime}=Q$. First of all, we have $$ \\begin{aligned} & \\angle\\left(Q^{\\prime} Y, Q^{\\prime} B\\right)=\\angle(C Y, Z B)=\\angle(C Y, Z Y)+\\angle(Z Y, B Z) \\\\ & =\\angle(C I, I W)+\\angle(I W, I B)=\\angle(C I, I B)=\\frac{\\pi-\\angle A}{2}=\\angle(F Y, F B), \\end{aligned} $$ so $Q^{\\prime} \\in \\Omega_{B}$. Similarly, $Q^{\\prime} \\in \\Omega_{C}$. Thus $Q^{\\prime} \\in \\Omega_{B} \\cap \\Omega_{C}=\\{P, Q\\}$ and it remains to prove that $Q^{\\prime} \\neq P$. If we had $Q^{\\prime}=P$, we would have $\\angle(P Y, P Z)=\\angle\\left(Q^{\\prime} Y, Q^{\\prime} Z\\right)=\\angle(I C, I B)$. This would imply $$ \\angle(P Y, Y F)+\\angle(E Z, Z P)=\\angle(P Y, P Z)=\\angle(I C, I B)=\\angle(P E, P F), $$ so circles $\\Omega_{B}$ and $\\Omega_{C}$ would be tangent at $P$. That is excluded in the problem conditions, so $Q^{\\prime}=Q$. ![](https:\/\/cdn.mathpix.com\/cropped\/2024_11_18_9ee8c2ac2c2f5002ff14g-077.jpg?height=966&width=1468&top_left_y=1070&top_left_x=251) Step 4. Now we are ready to show that $P, Q$, and $S$ are collinear. Notice that $A$ and $D$ are the poles of $E W$ and $D W$ with respect to $\\omega$, so $W$ is the pole of $A D$. Hence, $W I \\perp A D$. Since $C I \\perp D E$, this yields $\\angle(I C, W I)=\\angle(D E, D A)$. On the other hand, $D A$ is a symmedian in $\\triangle D E F$, so $\\angle(D E, D A)=\\angle(D N, D F)=\\angle(D S, D F)$. Therefore, $$ \\begin{aligned} \\angle(P S, P F)=\\angle(D S, D F)=\\angle(D E, D A)= & \\angle(I C, I W) \\\\ & =\\angle(Y C, Y W)=\\angle(Y Q, Y F)=\\angle(P Q, P F), \\end{aligned} $$ which yields the desired collinearity.","tier":0} +{"problem_type":"Geometry","problem_label":"G8","problem":"Let $\\mathcal{L}$ be the set of all lines in the plane and let $f$ be a function that assigns to each line $\\ell \\in \\mathcal{L}$ a point $f(\\ell)$ on $\\ell$. Suppose that for any point $X$, and for any three lines $\\ell_{1}, \\ell_{2}, \\ell_{3}$ passing through $X$, the points $f\\left(\\ell_{1}\\right), f\\left(\\ell_{2}\\right), f\\left(\\ell_{3}\\right)$ and $X$ lie on a circle. Prove that there is a unique point $P$ such that $f(\\ell)=P$ for any line $\\ell$ passing through $P$. (Australia)","solution":"We provide a complete characterisation of the functions satisfying the given condition. Write $\\angle\\left(\\ell_{1}, \\ell_{2}\\right)$ for the directed angle modulo $180^{\\circ}$ between the lines $\\ell_{1}$ and $\\ell_{2}$. Given a point $P$ and an angle $\\alpha \\in\\left(0,180^{\\circ}\\right)$, for each line $\\ell$, let $\\ell^{\\prime}$ be the line through $P$ satisfying $\\angle\\left(\\ell^{\\prime}, \\ell\\right)=\\alpha$, and let $h_{P, \\alpha}(\\ell)$ be the intersection point of $\\ell$ and $\\ell^{\\prime}$. We will prove that there is some pair $(P, \\alpha)$ such that $f$ and $h_{P, \\alpha}$ are the same function. Then $P$ is the unique point in the problem statement. Given an angle $\\alpha$ and a point $P$, let a line $\\ell$ be called $(P, \\alpha)$-good if $f(\\ell)=h_{P, \\alpha}(\\ell)$. Let a point $X \\neq P$ be called $(P, \\alpha)$-good if the circle $g(X)$ passes through $P$ and some point $Y \\neq P, X$ on $g(X)$ satisfies $\\angle(P Y, Y X)=\\alpha$. It follows from this definition that if $X$ is $(P, \\alpha)$ good then every point $Y \\neq P, X$ of $g(X)$ satisfies this angle condition, so $h_{P, \\alpha}(X Y)=Y$ for every $Y \\in g(X)$. Equivalently, $f(\\ell) \\in\\left\\{X, h_{P, \\alpha}(\\ell)\\right\\}$ for each line $\\ell$ passing through $X$. This shows the following lemma. Lemma 1. If $X$ is $(P, \\alpha)$-good and $\\ell$ is a line passing through $X$ then either $f(\\ell)=X$ or $\\ell$ is $(P, \\alpha)$-good. Lemma 2. If $X$ and $Y$ are different $(P, \\alpha)$-good points, then line $X Y$ is $(P, \\alpha)$-good. Proof. If $X Y$ is not $(P, \\alpha)$-good then by the previous Lemma, $f(X Y)=X$ and similarly $f(X Y)=Y$, but clearly this is impossible as $X \\neq Y$. Lemma 3. If $\\ell_{1}$ and $\\ell_{2}$ are different $(P, \\alpha)$-good lines which intersect at $X \\neq P$, then either $f\\left(\\ell_{1}\\right)=X$ or $f\\left(\\ell_{2}\\right)=X$ or $X$ is $(P, \\alpha)$-good. Proof. If $f\\left(\\ell_{1}\\right), f\\left(\\ell_{2}\\right) \\neq X$, then $g(X)$ is the circumcircle of $X, f\\left(\\ell_{1}\\right)$ and $f\\left(\\ell_{2}\\right)$. Since $\\ell_{1}$ and $\\ell_{2}$ are $(P, \\alpha)$-good lines, the angles $$ \\angle\\left(P f\\left(\\ell_{1}\\right), f\\left(\\ell_{1}\\right) X\\right)=\\angle\\left(P f\\left(\\ell_{2}\\right), f\\left(\\ell_{2}\\right) X\\right)=\\alpha $$ so $P$ lies on $g(X)$. Hence, $X$ is $(P, \\alpha)$-good. Lemma 4. If $\\ell_{1}, \\ell_{2}$ and $\\ell_{3}$ are different $(P, \\alpha)$ good lines which intersect at $X \\neq P$, then $X$ is $(P, \\alpha)$-good. Proof. This follows from the previous Lemma since at most one of the three lines $\\ell_{i}$ can satisfy $f\\left(\\ell_{i}\\right)=X$ as the three lines are all $(P, \\alpha)$-good. Lemma 5. If $A B C$ is a triangle such that $A, B, C, f(A B), f(A C)$ and $f(B C)$ are all different points, then there is some point $P$ and some angle $\\alpha$ such that $A, B$ and $C$ are $(P, \\alpha)$-good points and $A B, B C$ and $C A$ are $(P, \\alpha)-\\operatorname{good}$ lines. ![](https:\/\/cdn.mathpix.com\/cropped\/2024_11_18_9ee8c2ac2c2f5002ff14g-079.jpg?height=415&width=612&top_left_y=207&top_left_x=728) Proof. Let $D, E, F$ denote the points $f(B C), f(A C), f(A B)$, respectively. Then $g(A)$, $g(B)$ and $g(C)$ are the circumcircles of $A E F, B D F$ and $C D E$, respectively. Let $P \\neq F$ be the second intersection of circles $g(A)$ and $g(B)$ (or, if these circles are tangent at $F$, then $P=F$ ). By Miquel's theorem (or an easy angle chase), $g(C)$ also passes through $P$. Then by the cyclic quadrilaterals, the directed angles $$ \\angle(P D, D C)=\\angle(P F, F B)=\\angle(P E, E A)=\\alpha $$ for some angle $\\alpha$. Hence, lines $A B, B C$ and $C A$ are all $(P, \\alpha)$-good, so by Lemma $3, A, B$ and $C$ are $(P, \\alpha)$-good. (In the case where $P=D$, the line $P D$ in the equation above denotes the line which is tangent to $g(B)$ at $P=D$. Similar definitions are used for $P E$ and $P F$ in the cases where $P=E$ or $P=F$.) Consider the set $\\Omega$ of all points $(x, y)$ with integer coordinates $1 \\leqslant x, y \\leqslant 1000$, and consider the set $L_{\\Omega}$ of all horizontal, vertical and diagonal lines passing through at least one point in $\\Omega$. A simple counting argument shows that there are 5998 lines in $L_{\\Omega}$. For each line $\\ell$ in $L_{\\Omega}$ we colour the point $f(\\ell)$ red. Then there are at most 5998 red points. Now we partition the points in $\\Omega$ into 10000 ten by ten squares. Since there are at most 5998 red points, at least one of these squares $\\Omega_{10}$ contains no red points. Let $(m, n)$ be the bottom left point in $\\Omega_{10}$. Then the triangle with vertices $(m, n),(m+1, n)$ and $(m, n+1)$ satisfies the condition of Lemma 5 , so these three vertices are all $(P, \\alpha)$-good for some point $P$ and angle $\\alpha$, as are the lines joining them. From this point on, we will simply call a point or line good if it is $(P, \\alpha)$-good for this particular pair $(P, \\alpha)$. Now by Lemma 1, the line $x=m+1$ is good, as is the line $y=n+1$. Then Lemma 3 implies that $(m+1, n+1)$ is good. By applying these two lemmas repeatedly, we can prove that the line $x+y=m+n+2$ is good, then the points $(m, n+2)$ and $(m+2, n)$ then the lines $x=m+2$ and $y=n+2$, then the points $(m+2, n+1),(m+1, n+2)$ and $(m+2, n+2)$ and so on until we have prove that all points in $\\Omega_{10}$ are good. Now we will use this to prove that every point $S \\neq P$ is good. Since $g(S)$ is a circle, it passes through at most two points of $\\Omega_{10}$ on any vertical line, so at most 20 points in total. Moreover, any line $\\ell$ through $S$ intersects at most 10 points in $\\Omega_{10}$. Hence, there are at least eight lines $\\ell$ through $S$ which contain a point $Q$ in $\\Omega_{10}$ which is not on $g(S)$. Since $Q$ is not on $g(S)$, the point $f(\\ell) \\neq Q$. Hence, by Lemma 1 , the line $\\ell$ is good. Hence, at least eight good lines pass through $S$, so by Lemma 4, the point $S$ is good. Hence, every point $S \\neq P$ is good, so by Lemma 2 , every line is good. In particular, every line $\\ell$ passing through $P$ is good, and therefore satisfies $f(\\ell)=P$, as required.","tier":0} +{"problem_type":"Geometry","problem_label":"G8","problem":"Let $\\mathcal{L}$ be the set of all lines in the plane and let $f$ be a function that assigns to each line $\\ell \\in \\mathcal{L}$ a point $f(\\ell)$ on $\\ell$. Suppose that for any point $X$, and for any three lines $\\ell_{1}, \\ell_{2}, \\ell_{3}$ passing through $X$, the points $f\\left(\\ell_{1}\\right), f\\left(\\ell_{2}\\right), f\\left(\\ell_{3}\\right)$ and $X$ lie on a circle. Prove that there is a unique point $P$ such that $f(\\ell)=P$ for any line $\\ell$ passing through $P$. (Australia)","solution":"Note that for any distinct points $X, Y$, the circles $g(X)$ and $g(Y)$ meet on $X Y$ at the point $f(X Y) \\in g(X) \\cap g(Y) \\cap(X Y)$. We write $s(X, Y)$ for the second intersection point of circles $g(X)$ and $g(Y)$. Lemma 1. Suppose that $X, Y$ and $Z$ are not collinear, and that $f(X Y) \\notin\\{X, Y\\}$ and similarly for $Y Z$ and $Z X$. Then $s(X, Y)=s(Y, Z)=s(Z, X)$. Proof. The circles $g(X), g(Y)$ and $g(Z)$ through the vertices of triangle $X Y Z$ meet pairwise on the corresponding edges (produced). By Miquel's theorem, the second points of intersection of any two of the circles coincide. (See the diagram for Lemma 5 of Solution 1.) Now pick any line $\\ell$ and any six different points $Y_{1}, \\ldots, Y_{6}$ on $\\ell \\backslash\\{f(\\ell)\\}$. Pick a point $X$ not on $\\ell$ or any of the circles $g\\left(Y_{i}\\right)$. Reordering the indices if necessary, we may suppose that $Y_{1}, \\ldots, Y_{4}$ do not lie on $g(X)$, so that $f\\left(X Y_{i}\\right) \\notin\\left\\{X, Y_{i}\\right\\}$ for $1 \\leqslant i \\leqslant 4$. By applying the above lemma to triangles $X Y_{i} Y_{j}$ for $1 \\leqslant i0$, there is a point $P_{\\epsilon}$ with $g\\left(P_{\\epsilon}\\right)$ of radius at most $\\epsilon$. Then there is a point $P$ with the given property. Proof. Consider a sequence $\\epsilon_{i}=2^{-i}$ and corresponding points $P_{\\epsilon_{i}}$. Because the two circles $g\\left(P_{\\epsilon_{i}}\\right)$ and $g\\left(P_{\\epsilon_{j}}\\right)$ meet, the distance between $P_{\\epsilon_{i}}$ and $P_{\\epsilon_{j}}$ is at most $2^{1-i}+2^{1-j}$. As $\\sum_{i} \\epsilon_{i}$ converges, these points converge to some point $P$. For all $\\epsilon>0$, the point $P$ has distance at most $2 \\epsilon$ from $P_{\\epsilon}$, and all circles $g(X)$ pass through a point with distance at most $2 \\epsilon$ from $P_{\\epsilon}$, so distance at most $4 \\epsilon$ from $P$. A circle that passes distance at most $4 \\epsilon$ from $P$ for all $\\epsilon>0$ must pass through $P$, so by Lemma 1 the point $P$ has the given property. Lemma 3. Suppose no two of the circles $g(X)$ cut. Then there is a point $P$ with the given property. Proof. Consider a circle $g(X)$ with centre $Y$. The circle $g(Y)$ must meet $g(X)$ without cutting it, so has half the radius of $g(X)$. Repeating this argument, there are circles with arbitrarily small radius and the result follows by Lemma 2. Lemma 4. Suppose there are six different points $A, B_{1}, B_{2}, B_{3}, B_{4}, B_{5}$ such that no three are collinear, no four are concyclic, and all the circles $g\\left(B_{i}\\right)$ cut pairwise at $A$. Then there is a point $P$ with the given property. Proof. Consider some line $\\ell$ through $A$ that does not pass through any of the $B_{i}$ and is not tangent to any of the $g\\left(B_{i}\\right)$. Fix some direction along that line, and let $X_{\\epsilon}$ be the point on $\\ell$ that has distance $\\epsilon$ from $A$ in that direction. In what follows we consider only those $\\epsilon$ for which $X_{\\epsilon}$ does not lie on any $g\\left(B_{i}\\right)$ (this restriction excludes only finitely many possible values of $\\epsilon$ ). Consider the circle $g\\left(X_{\\epsilon}\\right)$. Because no four of the $B_{i}$ are concyclic, at most three of them lie on this circle, so at least two of them do not. There must be some sequence of $\\epsilon \\rightarrow 0$ such that it is the same two of the $B_{i}$ for all $\\epsilon$ in that sequence, so now restrict attention to that sequence, and suppose without loss of generality that $B_{1}$ and $B_{2}$ do not lie on $g\\left(X_{\\epsilon}\\right)$ for any $\\epsilon$ in that sequence. Then $f\\left(X_{\\epsilon} B_{1}\\right)$ is not $B_{1}$, so must be the other point of intersection of $X_{\\epsilon} B_{1}$ with $g\\left(B_{1}\\right)$, and the same applies with $B_{2}$. Now consider the three points $X_{\\epsilon}, f\\left(X_{\\epsilon} B_{1}\\right)$ and $f\\left(X_{\\epsilon} B_{2}\\right)$. As $\\epsilon \\rightarrow 0$, the angle at $X_{\\epsilon}$ tends to $\\angle B_{1} A B_{2}$ or $180^{\\circ}-\\angle B_{1} A B_{2}$, which is not 0 or $180^{\\circ}$ because no three of the points were collinear. All three distances between those points are bounded above by constant multiples of $\\epsilon$ (in fact, if the triangle is scaled by a factor of $1 \/ \\epsilon$, it tends to a fixed triangle). Thus the circumradius of those three points, which is the radius of $g\\left(X_{\\epsilon}\\right)$, is also bounded above by a constant multiple of $\\epsilon$, and so the result follows by Lemma 2 . Lemma 5. Suppose there are two points $A$ and $B$ such that $g(A)$ and $g(B)$ cut. Then there is a point $P$ with the given property. Proof. Suppose that $g(A)$ and $g(B)$ cut at $C$ and $D$. One of those points, without loss of generality $C$, must be $f(A B)$, and so lie on the line $A B$. We now consider two cases, according to whether $D$ also lies on that line.","tier":0} +{"problem_type":"Geometry","problem_label":"G8","problem":"Let $\\mathcal{L}$ be the set of all lines in the plane and let $f$ be a function that assigns to each line $\\ell \\in \\mathcal{L}$ a point $f(\\ell)$ on $\\ell$. Suppose that for any point $X$, and for any three lines $\\ell_{1}, \\ell_{2}, \\ell_{3}$ passing through $X$, the points $f\\left(\\ell_{1}\\right), f\\left(\\ell_{2}\\right), f\\left(\\ell_{3}\\right)$ and $X$ lie on a circle. Prove that there is a unique point $P$ such that $f(\\ell)=P$ for any line $\\ell$ passing through $P$. (Australia)","solution":"For any point $X$, denote by $t(X)$ the line tangent to $g(X)$ at $X$; notice that $f(t(X))=X$, so $f$ is surjective. Step 1: We find a point $P$ for which there are at least two different lines $p_{1}$ and $p_{2}$ such that $f\\left(p_{i}\\right)=P$. Choose any point $X$. If $X$ does not have this property, take any $Y \\in g(X) \\backslash\\{X\\}$; then $f(X Y)=Y$. If $Y$ does not have the property, $t(Y)=X Y$, and the circles $g(X)$ and $g(Y)$ meet again at some point $Z$. Then $f(X Z)=Z=f(Y Z)$, so $Z$ has the required property. We will show that $P$ is the desired point. From now on, we fix two different lines $p_{1}$ and $p_{2}$ with $f\\left(p_{1}\\right)=f\\left(p_{2}\\right)=P$. Assume for contradiction that $f(\\ell)=Q \\neq P$ for some line $\\ell$ through $P$. We fix $\\ell$, and note that $Q \\in g(P)$. Step 2: We prove that $P \\in g(Q)$. Take an arbitrary point $X \\in \\ell \\backslash\\{P, Q\\}$. Two cases are possible for the position of $t(X)$ in relation to the $p_{i}$; we will show that each case (and subcase) occurs for only finitely many positions of $X$, yielding a contradiction. Case 2.1: $t(X)$ is parallel to one of the $p_{i}$; say, to $p_{1}$. Let $t(X) \\operatorname{cross} p_{2}$ at $R$. Then $g(R)$ is the circle $(P R X)$, as $f(R P)=P$ and $f(R X)=X$. Let $R Q$ cross $g(R)$ again at $S$. Then $f(R Q) \\in\\{R, S\\} \\cap g(Q)$, so $g(Q)$ contains one of the points $R$ and $S$. If $R \\in g(Q)$, then $R$ is one of finitely many points in the intersection $g(Q) \\cap p_{2}$, and each of them corresponds to a unique position of $X$, since $R X$ is parallel to $p_{1}$. If $S \\in g(Q)$, then $\\angle(Q S, S P)=\\angle(R S, S P)=\\angle(R X, X P)=\\angle\\left(p_{1}, \\ell\\right)$, so $\\angle(Q S, S P)$ is constant for all such points $X$, and all points $S$ obtained in such a way lie on one circle $\\gamma$ passing through $P$ and $Q$. Since $g(Q)$ does not contain $P$, it is different from $\\gamma$, so there are only finitely many points $S$. Each of them uniquely determines $R$ and thus $X$. ![](https:\/\/cdn.mathpix.com\/cropped\/2024_11_18_9ee8c2ac2c2f5002ff14g-083.jpg?height=843&width=909&top_left_y=207&top_left_x=568) So, Case 2.1 can occur for only finitely many points $X$. Case 2.2: $t(X)$ crosses $p_{1}$ and $p_{2}$ at $R_{1}$ and $R_{2}$, respectively. Clearly, $R_{1} \\neq R_{2}$, as $t(X)$ is the tangent to $g(X)$ at $X$, and $g(X)$ meets $\\ell$ only at $X$ and $Q$. Notice that $g\\left(R_{i}\\right)$ is the circle $\\left(P X R_{i}\\right)$. Let $R_{i} Q$ meet $g\\left(R_{i}\\right)$ again at $S_{i}$; then $S_{i} \\neq Q$, as $g\\left(R_{i}\\right)$ meets $\\ell$ only at $P$ and $X$. Then $f\\left(R_{i} Q\\right) \\in\\left\\{R_{i}, S_{i}\\right\\}$, and we distinguish several subcases. ![](https:\/\/cdn.mathpix.com\/cropped\/2024_11_18_9ee8c2ac2c2f5002ff14g-083.jpg?height=824&width=866&top_left_y=1501&top_left_x=595) Subcase 2.2.1: $f\\left(R_{1} Q\\right)=S_{1}, f\\left(R_{2} Q\\right)=S_{2}$; so $S_{1}, S_{2} \\in g(Q)$. In this case we have $0=\\angle\\left(R_{1} X, X P\\right)+\\angle\\left(X P, R_{2} X\\right)=\\angle\\left(R_{1} S_{1}, S_{1} P\\right)+\\angle\\left(S_{2} P, S_{2} R_{2}\\right)=$ $\\angle\\left(Q S_{1}, S_{1} P\\right)+\\angle\\left(S_{2} P, S_{2} Q\\right)$, which shows $P \\in g(Q)$. Subcase 2.2.2: $f\\left(R_{1} Q\\right)=R_{1}, f\\left(R_{2} Q\\right)=R_{2}$; so $R_{1}, R_{2} \\in g(Q)$. This can happen for at most four positions of $X$ - namely, at the intersections of $\\ell$ with a line of the form $K_{1} K_{2}$, where $K_{i} \\in g(Q) \\cap p_{i}$. Subcase 2.2.3: $f\\left(R_{1} Q\\right)=S_{1}, f\\left(R_{2} Q\\right)=R_{2}$ (the case $f\\left(R_{1} Q\\right)=R_{1}, f\\left(R_{2} Q\\right)=S_{2}$ is similar). In this case, there are at most two possible positions for $R_{2}$ - namely, the meeting points of $g(Q)$ with $p_{2}$. Consider one of them. Let $X$ vary on $\\ell$. Then $R_{1}$ is the projection of $X$ to $p_{1}$ via $R_{2}, S_{1}$ is the projection of $R_{1}$ to $g(Q)$ via $Q$. Finally, $\\angle\\left(Q S_{1}, S_{1} X\\right)=\\angle\\left(R_{1} S_{1}, S_{1} X\\right)=$ $\\angle\\left(R_{1} P, P X\\right)=\\angle\\left(p_{1}, \\ell\\right) \\neq 0$, so $X$ is obtained by a fixed projective transform $g(Q) \\rightarrow \\ell$ from $S_{1}$. So, if there were three points $X$ satisfying the conditions of this subcase, the composition of the three projective transforms would be the identity. But, if we apply it to $X=Q$, we successively get some point $R_{1}^{\\prime}$, then $R_{2}$, and then some point different from $Q$, a contradiction. Thus Case 2.2 also occurs for only finitely many points $X$, as desired. Step 3: We show that $f(P Q)=P$, as desired. The argument is similar to that in Step 2, with the roles of $Q$ and $X$ swapped. Again, we show that there are only finitely many possible positions for a point $X \\in \\ell \\backslash\\{P, Q\\}$, which is absurd. Case 3.1: $t(Q)$ is parallel to one of the $p_{i}$; say, to $p_{1}$. Let $t(Q)$ cross $p_{2}$ at $R$; then $g(R)$ is the circle $(P R Q)$. Let $R X \\operatorname{cross} g(R)$ again at $S$. Then $f(R X) \\in\\{R, S\\} \\cap g(X)$, so $g(X)$ contains one of the points $R$ and $S$. ![](https:\/\/cdn.mathpix.com\/cropped\/2024_11_18_9ee8c2ac2c2f5002ff14g-084.jpg?height=729&width=909&top_left_y=1126&top_left_x=568) Subcase 3.1.1: $S=f(R X) \\in g(X)$. We have $\\angle(t(X), Q X)=\\angle(S X, S Q)=\\angle(S R, S Q)=\\angle(P R, P Q)=\\angle\\left(p_{2}, \\ell\\right)$. Hence $t(X) \\| p_{2}$. Now we recall Case 2.1: we let $t(X)$ cross $p_{1}$ at $R^{\\prime}$, so $g\\left(R^{\\prime}\\right)=\\left(P R^{\\prime} X\\right)$, and let $R^{\\prime} Q$ meet $g\\left(R^{\\prime}\\right)$ again at $S^{\\prime}$; notice that $S^{\\prime} \\neq Q$. Excluding one position of $X$, we may assume that $R^{\\prime} \\notin g(Q)$, so $R^{\\prime} \\neq f\\left(R^{\\prime} Q\\right)$. Therefore, $S^{\\prime}=f\\left(R^{\\prime} Q\\right) \\in g(Q)$. But then, as in Case 2.1, we get $\\angle(t(Q), P Q)=\\angle\\left(Q S^{\\prime}, S^{\\prime} P\\right)=\\angle\\left(R^{\\prime} X, X P\\right)=\\angle\\left(p_{2}, \\ell\\right)$. This means that $t(Q)$ is parallel to $p_{2}$, which is impossible. Subcase 3.1.2: $R=f(R X) \\in g(X)$. In this case, we have $\\angle(t(X), \\ell)=\\angle(R X, R Q)=\\angle\\left(R X, p_{1}\\right)$. Again, let $R^{\\prime}=t(X) \\cap p_{1}$; this point exists for all but at most one position of $X$. Then $g\\left(R^{\\prime}\\right)=\\left(R^{\\prime} X P\\right)$; let $R^{\\prime} Q$ meet $g\\left(R^{\\prime}\\right)$ again at $S^{\\prime}$. Due to $\\angle\\left(R^{\\prime} X, X R\\right)=\\angle(Q X, Q R)=\\angle\\left(\\ell, p_{1}\\right), R^{\\prime}$ determines $X$ in at most two ways, so for all but finitely many positions of $X$ we have $R^{\\prime} \\notin g(Q)$. Therefore, for those positions we have $S^{\\prime}=f\\left(R^{\\prime} Q\\right) \\in g(Q)$. But then $\\angle\\left(R X, p_{1}\\right)=\\angle\\left(R^{\\prime} X, X P\\right)=\\angle\\left(R^{\\prime} S^{\\prime}, S^{\\prime} P\\right)=$ $\\angle\\left(Q S^{\\prime}, S^{\\prime} P\\right)=\\angle(t(Q), Q P)$ is fixed, so this case can hold only for one specific position of $X$ as well. Thus, in Case 3.1, there are only finitely many possible positions of $X$, yielding a contradiction. Case 3.2: $t(Q)$ crosses $p_{1}$ and $p_{2}$ at $R_{1}$ and $R_{2}$, respectively. By Step $2, R_{1} \\neq R_{2}$. Notice that $g\\left(R_{i}\\right)$ is the circle $\\left(P Q R_{i}\\right)$. Let $R_{i} X$ meet $g\\left(R_{i}\\right)$ at $S_{i}$; then $S_{i} \\neq X$. Then $f\\left(R_{i} X\\right) \\in\\left\\{R_{i}, S_{i}\\right\\}$, and we distinguish several subcases. ![](https:\/\/cdn.mathpix.com\/cropped\/2024_11_18_9ee8c2ac2c2f5002ff14g-085.jpg?height=854&width=866&top_left_y=424&top_left_x=589) Subcase 3.2.1: $f\\left(R_{1} X\\right)=S_{1}$ and $f\\left(R_{2} X\\right)=S_{2}$, so $S_{1}, S_{2} \\in g(X)$. As in Subcase 2.2.1, we have $0=\\angle\\left(R_{1} Q, Q P\\right)+\\angle\\left(Q P, R_{2} Q\\right)=\\angle\\left(X S_{1}, S_{1} P\\right)+\\angle\\left(S_{2} P, S_{2} X\\right)$, which shows $P \\in g(X)$. But $X, Q \\in g(X)$ as well, so $g(X)$ meets $\\ell$ at three distinct points, which is absurd. Subcase 3.2.2: $f\\left(R_{1} X\\right)=R_{1}, f\\left(R_{2} X\\right)=R_{2}$, so $R_{1}, R_{2} \\in g(X)$. Now three distinct collinear points $R_{1}, R_{2}$, and $Q$ belong to $g(X)$, which is impossible. Subcase 3.2.3: $f\\left(R_{1} X\\right)=S_{1}, f\\left(R_{2} X\\right)=R_{2}$ (the case $f\\left(R_{1} X\\right)=R_{1}, f\\left(R_{2} X\\right)=S_{2}$ is similar). We have $\\angle\\left(X R_{2}, R_{2} Q\\right)=\\angle\\left(X S_{1}, S_{1} Q\\right)=\\angle\\left(R_{1} S_{1}, S_{1} Q\\right)=\\angle\\left(R_{1} P, P Q\\right)=\\angle\\left(p_{1}, \\ell\\right)$, so this case can occur for a unique position of $X$. Thus, in Case 3.2 , there is only a unique position of $X$, again yielding the required contradiction.","tier":0} +{"problem_type":"Geometry","problem_label":"G8","problem":"Let $\\mathcal{L}$ be the set of all lines in the plane and let $f$ be a function that assigns to each line $\\ell \\in \\mathcal{L}$ a point $f(\\ell)$ on $\\ell$. Suppose that for any point $X$, and for any three lines $\\ell_{1}, \\ell_{2}, \\ell_{3}$ passing through $X$, the points $f\\left(\\ell_{1}\\right), f\\left(\\ell_{2}\\right), f\\left(\\ell_{3}\\right)$ and $X$ lie on a circle. Prove that there is a unique point $P$ such that $f(\\ell)=P$ for any line $\\ell$ passing through $P$. (Australia)","solution":"We will get an upper bound on $n$ from the speed at which $v_{2}\\left(L_{n}\\right)$ grows. From $$ L_{n}=\\left(2^{n}-1\\right)\\left(2^{n}-2\\right) \\cdots\\left(2^{n}-2^{n-1}\\right)=2^{1+2+\\cdots+(n-1)}\\left(2^{n}-1\\right)\\left(2^{n-1}-1\\right) \\cdots\\left(2^{1}-1\\right) $$ we read $$ v_{2}\\left(L_{n}\\right)=1+2+\\cdots+(n-1)=\\frac{n(n-1)}{2} . $$ On the other hand, $v_{2}(m!)$ is expressed by the Legendre formula as $$ v_{2}(m!)=\\sum_{i=1}^{\\infty}\\left\\lfloor\\frac{m}{2^{i}}\\right\\rfloor . $$ As usual, by omitting the floor functions, $$ v_{2}(m!)<\\sum_{i=1}^{\\infty} \\frac{m}{2^{i}}=m $$ Thus, $L_{n}=m$ ! implies the inequality $$ \\frac{n(n-1)}{2}1.3 \\cdot 10^{12}$. For $n \\geqslant 7$ we prove (3) by the following inequalities: $$ \\begin{aligned} \\left(\\frac{n(n-1)}{2}\\right)! & =15!\\cdot 16 \\cdot 17 \\cdots \\frac{n(n-1)}{2}>2^{36} \\cdot 16^{\\frac{n(n-1)}{2}-15} \\\\ & =2^{2 n(n-1)-24}=2^{n^{2}} \\cdot 2^{n(n-2)-24}>2^{n^{2}} . \\end{aligned} $$ Putting together (2) and (3), for $n \\geqslant 6$ we get a contradiction, since $$ L_{n}<2^{n^{2}}<\\left(\\frac{n(n-1)}{2}\\right)!a^{3} . $$ So $3 a^{3} \\geqslant(a b c)^{2}>a^{3}$ and hence $3 a \\geqslant b^{2} c^{2}>a$. Now $b^{3}+c^{3}=a^{2}\\left(b^{2} c^{2}-a\\right) \\geqslant a^{2}$, and so $$ 18 b^{3} \\geqslant 9\\left(b^{3}+c^{3}\\right) \\geqslant 9 a^{2} \\geqslant b^{4} c^{4} \\geqslant b^{3} c^{5} $$ so $18 \\geqslant c^{5}$ which yields $c=1$. Now, note that we must have $a>b$, as otherwise we would have $2 b^{3}+1=b^{4}$ which has no positive integer solutions. So $$ a^{3}-b^{3} \\geqslant(b+1)^{3}-b^{3}>1 $$ and $$ 2 a^{3}>1+a^{3}+b^{3}>a^{3} $$ which implies $2 a^{3}>a^{2} b^{2}>a^{3}$ and so $2 a>b^{2}>a$. Therefore $$ 4\\left(1+b^{3}\\right)=4 a^{2}\\left(b^{2}-a\\right) \\geqslant 4 a^{2}>b^{4} $$ so $4>b^{3}(b-4)$; that is, $b \\leqslant 4$. Now, for each possible value of $b$ with $2 \\leqslant b \\leqslant 4$ we obtain a cubic equation for $a$ with constant coefficients. These are as follows: $$ \\begin{array}{rlrl} b=2: & & & a^{3}-4 a^{2}+9=0 \\\\ b=3: & & a^{3}-9 a^{2}+28=0 \\\\ b=4: & & a^{3}-16 a^{2}+65=0 . \\end{array} $$ The only case with an integer solution for $a$ with $b \\leqslant a$ is $b=2$, leading to $(a, b, c)=(3,2,1)$. Comment 1.1. Instead of writing down each cubic equation explicitly, we could have just observed that $a^{2} \\mid b^{3}+1$, and for each choice of $b$ checked each square factor of $b^{3}+1$ for $a^{2}$. We could also have observed that, with $c=1$, the relation $18 b^{3} \\geqslant b^{4} c^{4}$ becomes $b \\leqslant 18$, and we can simply check all possibilities for $b$ (instead of working to prove that $b \\leqslant 4$ ). This check becomes easier after using the factorisation $b^{3}+1=(b+1)\\left(b^{2}-b+1\\right)$ and observing that no prime besides 3 can divide both of the factors. Comment 1.2. Another approach to finish the problem after establishing that $c \\leqslant 1$ is to set $k=b^{2} c^{2}-a$, which is clearly an integer and must be positive as it is equal to $\\left(b^{3}+c^{3}\\right) \/ a^{2}$. Then we divide into cases based on whether $k=1$ or $k \\geqslant 2$; in the first case, we have $b^{3}+1=a^{2}=\\left(b^{2}-1\\right)^{2}$ whose only positive root is $b=2$, and in the second case we have $b^{2} \\leqslant 3 a$, and so $$ b^{4} \\leqslant(3 a)^{2} \\leqslant \\frac{9}{2}\\left(k a^{2}\\right)=\\frac{9}{2}\\left(b^{3}+1\\right), $$ which implies that $b \\leqslant 4$.","tier":0} +{"problem_type":"Number Theory","problem_label":"N2","problem":"Find all triples $(a, b, c)$ of positive integers such that $a^{3}+b^{3}+c^{3}=(a b c)^{2}$. (Nigeria)","solution":"Again, we will start by proving that $c=1$. Suppose otherwise that $c \\geqslant 2$. We have $a^{3}+b^{3}+c^{3} \\leqslant 3 a^{3}$, so $b^{2} c^{2} \\leqslant 3 a$. Since $c \\geqslant 2$, this tells us that $b \\leqslant \\sqrt{3 a \/ 4}$. As the right-hand side of the original equation is a multiple of $a^{2}$, we have $a^{2} \\leqslant 2 b^{3} \\leqslant 2(3 a \/ 4)^{3 \/ 2}$. In other words, $a \\leqslant \\frac{27}{16}<2$, which contradicts the assertion that $a \\geqslant c \\geqslant 2$. So there are no solutions in this case, and so we must have $c=1$. Now, the original equation becomes $a^{3}+b^{3}+1=a^{2} b^{2}$. Observe that $a \\geqslant 2$, since otherwise $a=b=1$ as $a \\geqslant b$. The right-hand side is a multiple of $a^{2}$, so the left-hand side must be as well. Thus, $b^{3}+1 \\geqslant$ $a^{2}$. Since $a \\geqslant b$, we also have $$ b^{2}=a+\\frac{b^{3}+1}{a^{2}} \\leqslant 2 a+\\frac{1}{a^{2}} $$ and so $b^{2} \\leqslant 2 a$ since $b^{2}$ is an integer. Thus $(2 a)^{3 \/ 2}+1 \\geqslant b^{3}+1 \\geqslant a^{2}$, from which we deduce $a \\leqslant 8$. Now, for each possible value of $a$ with $2 \\leqslant a \\leqslant 8$ we obtain a cubic equation for $b$ with constant coefficients. These are as follows: $$ \\begin{array}{ll} a=2: & \\\\ b^{3}-4 b^{2}+9=0 \\\\ a=3: & \\\\ b^{3}-9 b^{2}+28=0 \\\\ a=4: & \\\\ a=5: 16 b^{2}+65=0 \\\\ a=6: & \\\\ b^{3}-25 b^{2}+126=0 \\\\ a=7: & \\\\ b^{3}-36 b^{2}+217=0 \\\\ a=8: & \\\\ b^{3}-49 b^{2}+344=0 \\\\ b^{3}-64 b^{2}+513=0 . \\end{array} $$ The only case with an integer solution for $b$ with $a \\geqslant b$ is $a=3$, leading to $(a, b, c)=(3,2,1)$. Comment 2.1. As in Solution 1, instead of writing down each cubic equation explicitly, we could have just observed that $b^{2} \\mid a^{3}+1$, and for each choice of $a$ checked each square factor of $a^{3}+1$ for $b^{2}$. Comment 2.2. This solution does not require initially proving that $c=1$, in which case the bound would become $a \\leqslant 108$. The resulting cases could, in principle, be checked by a particularly industrious student.","tier":0} +{"problem_type":"Number Theory","problem_label":"N2","problem":"Find all triples $(a, b, c)$ of positive integers such that $a^{3}+b^{3}+c^{3}=(a b c)^{2}$. (Nigeria)","solution":"Set $k=\\left(b^{3}+c^{3}\\right) \/ a^{2} \\leqslant 2 a$, and rewrite the original equation as $a+k=(b c)^{2}$. Since $b^{3}$ and $c^{3}$ are positive integers, we have $(b c)^{3} \\geqslant b^{3}+c^{3}-1=k a^{2}-1$, so $$ a+k \\geqslant\\left(k a^{2}-1\\right)^{2 \/ 3} $$ As in Comment 1.2, $k$ is a positive integer; for each value of $k \\geqslant 1$, this gives us a polynomial inequality satisfied by $a$ : $$ k^{2} a^{4}-a^{3}-5 k a^{2}-3 k^{2} a-\\left(k^{3}-1\\right) \\leqslant 0 . $$ We now prove that $a \\leqslant 3$. Indeed, $$ 0 \\geqslant \\frac{k^{2} a^{4}-a^{3}-5 k a^{2}-3 k^{2} a-\\left(k^{3}-1\\right)}{k^{2}} \\geqslant a^{4}-a^{3}-5 a^{2}-3 a-k \\geqslant a^{4}-a^{3}-5 a^{2}-5 a $$ which fails when $a \\geqslant 4$. This leaves ten triples with $3 \\geqslant a \\geqslant b \\geqslant c \\geqslant 1$, which may be checked manually to give $(a, b, c)=(3,2,1)$.","tier":0} +{"problem_type":"Number Theory","problem_label":"N2","problem":"Find all triples $(a, b, c)$ of positive integers such that $a^{3}+b^{3}+c^{3}=(a b c)^{2}$. (Nigeria)","solution":"Again, observe that $b^{3}+c^{3}=a^{2}\\left(b^{2} c^{2}-a\\right)$, so $b \\leqslant a \\leqslant b^{2} c^{2}-1$. We consider the function $f(x)=x^{2}\\left(b^{2} c^{2}-x\\right)$. It can be seen that that on the interval $\\left[0, b^{2} c^{2}-1\\right]$ the function $f$ is increasing if $x<\\frac{2}{3} b^{2} c^{2}$ and decreasing if $x>\\frac{2}{3} b^{2} c^{2}$. Consequently, it must be the case that $$ b^{3}+c^{3}=f(a) \\geqslant \\min \\left(f(b), f\\left(b^{2} c^{2}-1\\right)\\right) $$ First, suppose that $b^{3}+c^{3} \\geqslant f\\left(b^{2} c^{2}-1\\right)$. This may be written $b^{3}+c^{3} \\geqslant\\left(b^{2} c^{2}-1\\right)^{2}$, and so $$ 2 b^{3} \\geqslant b^{3}+c^{3} \\geqslant\\left(b^{2} c^{2}-1\\right)^{2}>b^{4} c^{4}-2 b^{2} c^{2} \\geqslant b^{4} c^{4}-2 b^{3} c^{4} $$ Thus, $(b-2) c^{4}<2$, and the only solutions to this inequality have $(b, c)=(2,2)$ or $b \\leqslant 3$ and $c=1$. It is easy to verify that the only case giving a solution for $a \\geqslant b$ is $(a, b, c)=(3,2,1)$. Otherwise, suppose that $b^{3}+c^{3}=f(a) \\geqslant f(b)$. Then, we have $$ 2 b^{3} \\geqslant b^{3}+c^{3}=a^{2}\\left(b^{2} c^{2}-a\\right) \\geqslant b^{2}\\left(b^{2} c^{2}-b\\right) . $$ Consequently $b c^{2} \\leqslant 3$, with strict inequality in the case that $b \\neq c$. Hence $c=1$ and $b \\leqslant 2$. Both of these cases have been considered already, so we are done. Comment 4.1. Instead of considering which of $f(b)$ and $f\\left(b^{2} c^{2}-1\\right)$ is less than $f(a)$, we may also proceed by explicitly dividing into cases based on whether $a \\geqslant \\frac{2}{3} b^{2} c^{2}$ or $a<\\frac{2}{3} b^{2} c^{2}$. The first case may now be dealt with as follows. We have $b^{3} c^{3}+1 \\geqslant b^{3}+c^{3}$ as $b^{3}$ and $c^{3}$ are positive integers, so we have $$ b^{3} c^{3}+1 \\geqslant b^{3}+c^{3} \\geqslant a^{2} \\geqslant \\frac{4}{9} b^{4} c^{4} $$ This implies $b c \\leqslant 2$, and hence $c=1$ and $b \\leqslant 2$.","tier":0} +{"problem_type":"Number Theory","problem_label":"N3","problem":"We say that a set $S$ of integers is rootiful if, for any positive integer $n$ and any $a_{0}, a_{1}, \\ldots, a_{n} \\in S$, all integer roots of the polynomial $a_{0}+a_{1} x+\\cdots+a_{n} x^{n}$ are also in $S$. Find all rootiful sets of integers that contain all numbers of the form $2^{a}-2^{b}$ for positive integers $a$ and $b$. (Czech Republic)","solution":"The set $\\mathbb{Z}$ of all integers is clearly rootiful. We shall prove that any rootiful set $S$ containing all the numbers of the form $2^{a}-2^{b}$ for $a, b \\in \\mathbb{Z}_{>0}$ must be all of $\\mathbb{Z}$. First, note that $0=2^{1}-2^{1} \\in S$ and $2=2^{2}-2^{1} \\in S$. Now, $-1 \\in S$, since it is a root of $2 x+2$, and $1 \\in S$, since it is a root of $2 x^{2}-x-1$. Also, if $n \\in S$ then $-n$ is a root of $x+n$, so it suffices to prove that all positive integers must be in $S$. Now, we claim that any positive integer $n$ has a multiple in $S$. Indeed, suppose that $n=2^{\\alpha} \\cdot t$ for $\\alpha \\in \\mathbb{Z}_{\\geqslant 0}$ and $t$ odd. Then $t \\mid 2^{\\phi(t)}-1$, so $n \\mid 2^{\\alpha+\\phi(t)+1}-2^{\\alpha+1}$. Moreover, $2^{\\alpha+\\phi(t)+1}-2^{\\alpha+1} \\in S$, and so $S$ contains a multiple of every positive integer $n$. We will now prove by induction that all positive integers are in $S$. Suppose that $0,1, \\ldots, n-$ $1 \\in S$; furthermore, let $N$ be a multiple of $n$ in $S$. Consider the base- $n$ expansion of $N$, say $N=a_{k} n^{k}+a_{k-1} n^{k-1}+\\cdots+a_{1} n+a_{0}$. Since $0 \\leqslant a_{i}0}$. We show that, in fact, every integer $k$ with $|k|>2$ can be expressed as a root of a polynomial whose coefficients are of the form $2^{a}-2^{b}$. Observe that it suffices to consider the case where $k$ is positive, as if $k$ is a root of $a_{n} x^{n}+\\cdots+a_{1} x+a_{0}=0$, then $-k$ is a root of $(-1)^{n} a_{n} x^{n}+\\cdots-$ $a_{1} x+a_{0}=0$. Note that $$ \\left(2^{a_{n}}-2^{b_{n}}\\right) k^{n}+\\cdots+\\left(2^{a_{0}}-2^{b_{0}}\\right)=0 $$ is equivalent to $$ 2^{a_{n}} k^{n}+\\cdots+2^{a_{0}}=2^{b_{n}} k^{n}+\\cdots+2^{b_{0}} $$ Hence our aim is to show that two numbers of the form $2^{a_{n}} k^{n}+\\cdots+2^{a_{0}}$ are equal, for a fixed value of $n$. We consider such polynomials where every term $2^{a_{i}} k^{i}$ is at most $2 k^{n}$; in other words, where $2 \\leqslant 2^{a_{i}} \\leqslant 2 k^{n-i}$, or, equivalently, $1 \\leqslant a_{i} \\leqslant 1+(n-i) \\log _{2} k$. Therefore, there must be $1+\\left\\lfloor(n-i) \\log _{2} k\\right\\rfloor$ possible choices for $a_{i}$ satisfying these constraints. The number of possible polynomials is then $$ \\prod_{i=0}^{n}\\left(1+\\left\\lfloor(n-i) \\log _{2} k\\right\\rfloor\\right) \\geqslant \\prod_{i=0}^{n-1}(n-i) \\log _{2} k=n!\\left(\\log _{2} k\\right)^{n} $$ where the inequality holds as $1+\\lfloor x\\rfloor \\geqslant x$. As there are $(n+1)$ such terms in the polynomial, each at most $2 k^{n}$, such a polynomial must have value at most $2 k^{n}(n+1)$. However, for large $n$, we have $n!\\left(\\log _{2} k\\right)^{n}>2 k^{n}(n+1)$. Therefore there are more polynomials than possible values, so some two must be equal, as required.","tier":0} +{"problem_type":"Number Theory","problem_label":"N4","problem":"Let $\\mathbb{Z}_{>0}$ be the set of positive integers. A positive integer constant $C$ is given. Find all functions $f: \\mathbb{Z}_{>0} \\rightarrow \\mathbb{Z}_{>0}$ such that, for all positive integers $a$ and $b$ satisfying $a+b>C$, $$ a+f(b) \\mid a^{2}+b f(a) $$ (Croatia)","solution":"First, we show that $b \\mid f(b)^{2}$ for all $b$. To do this, we choose a large positive integer $n$ so that $n b-f(b) \\geqslant C$. Setting $a=n b-f(b)$ in (*) then shows that $$ n b \\mid(n b-f(b))^{2}+b f(n b-f(b)) $$ so that $b \\mid f(b)^{2}$ as claimed. Now in particular we have that $p \\mid f(p)$ for every prime $p$. If we write $f(p)=k(p) \\cdot p$, then the bound $f(p) \\leqslant f(1) \\cdot p$ (valid for $p$ sufficiently large) shows that some value $k$ of $k(p)$ must be attained for infinitely many $p$. We will show that $f(a)=k a$ for all positive integers $a$. To do this, we substitute $b=p$ in (*), where $p$ is any sufficiently large prime for which $k(p)=k$, obtaining $$ a+k p \\mid\\left(a^{2}+p f(a)\\right)-a(a+k p)=p f(a)-p k a . $$ For suitably large $p$ we have $\\operatorname{gcd}(a+k p, p)=1$, and hence we have $$ a+k p \\mid f(a)-k a $$ But the only way this can hold for arbitrarily large $p$ is if $f(a)-k a=0$. This concludes the proof. Comment. There are other ways to obtain the divisibility $p \\mid f(p)$ for primes $p$, which is all that is needed in this proof. For instance, if $f(p)$ were not divisible by $p$ then the arithmetic progression $p^{2}+b f(p)$ would attain prime values for infinitely many $b$ by Dirichlet's Theorem: hence, for these pairs p , b, we would have $p+f(b)=p^{2}+b f(p)$. Substituting $a \\mapsto b$ and $b \\mapsto p$ in $(*)$ then shows that $\\left(f(p)^{2}-p^{2}\\right)(p-1)$ is divisible by $b+f(p)$ and hence vanishes, which is impossible since $p \\nmid f(p)$ by assumption.","tier":0} +{"problem_type":"Number Theory","problem_label":"N4","problem":"Let $\\mathbb{Z}_{>0}$ be the set of positive integers. A positive integer constant $C$ is given. Find all functions $f: \\mathbb{Z}_{>0} \\rightarrow \\mathbb{Z}_{>0}$ such that, for all positive integers $a$ and $b$ satisfying $a+b>C$, $$ a+f(b) \\mid a^{2}+b f(a) $$ (Croatia)","solution":"First, we substitute $b=1$ in (*) and rearrange to find that $$ \\frac{f(a)+f(1)^{2}}{a+f(1)}=f(1)-a+\\frac{a^{2}+f(a)}{a+f(1)} $$ is a positive integer for sufficiently large $a$. Since $f(a) \\leqslant a f(1)$, for all sufficiently large $a$, it follows that $\\frac{f(a)+f(1)^{2}}{a+f(1)} \\leqslant f(1)$ also and hence there is a positive integer $k$ such that $\\frac{f(a)+f(1)^{2}}{a+f(1)}=k$ for infinitely many values of $a$. In other words, $$ f(a)=k a+f(1) \\cdot(k-f(1)) $$ for infinitely many $a$. Fixing an arbitrary choice of $a$ in (*), we have that $$ \\frac{a^{2}+b f(a)}{a+k b+f(1) \\cdot(k-f(1))} $$ is an integer for infinitely many $b$ (the same $b$ as above, maybe with finitely many exceptions). On the other hand, for $b$ taken sufficiently large, this quantity becomes arbitrarily close to $\\frac{f(a)}{k}$; this is only possible if $\\frac{f(a)}{k}$ is an integer and $$ \\frac{a^{2}+b f(a)}{a+k b+f(1) \\cdot(k-f(1))}=\\frac{f(a)}{k} $$ for infinitely many $b$. This rearranges to $$ \\frac{f(a)}{k} \\cdot(a+f(1) \\cdot(k-f(1)))=a^{2} $$ Hence $a^{2}$ is divisible by $a+f(1) \\cdot(k-f(1))$, and hence so is $f(1)^{2}(k-f(1))^{2}$. The only way this can occur for all $a$ is if $k=f(1)$, in which case $(* *)$ provides that $f(a)=k a$ for all $a$, as desired.","tier":0} +{"problem_type":"Number Theory","problem_label":"N4","problem":"Let $\\mathbb{Z}_{>0}$ be the set of positive integers. A positive integer constant $C$ is given. Find all functions $f: \\mathbb{Z}_{>0} \\rightarrow \\mathbb{Z}_{>0}$ such that, for all positive integers $a$ and $b$ satisfying $a+b>C$, $$ a+f(b) \\mid a^{2}+b f(a) $$ (Croatia)","solution":"Fix any two distinct positive integers $a$ and $b$. From (*) it follows that the two integers $$ \\left(a^{2}+c f(a)\\right) \\cdot(b+f(c)) \\text { and }\\left(b^{2}+c f(b)\\right) \\cdot(a+f(c)) $$ are both multiples of $(a+f(c)) \\cdot(b+f(c))$ for all sufficiently large $c$. Taking an appropriate linear combination to eliminate the $c f(c)$ term, we find after expanding out that the integer $$ \\left[a^{2} f(b)-b^{2} f(a)\\right] \\cdot f(c)+[(b-a) f(a) f(b)] \\cdot c+[a b(a f(b)-b f(a))] $$ is also a multiple of $(a+f(c)) \\cdot(b+f(c))$. But as $c$ varies, $(\\dagger)$ is bounded above by a positive multiple of $c$ while $(a+f(c)) \\cdot(b+f(c))$ is bounded below by a positive multiple of $c^{2}$. The only way that such a divisibility can hold is if in fact $$ \\left[a^{2} f(b)-b^{2} f(a)\\right] \\cdot f(c)+[(b-a) f(a) f(b)] \\cdot c+[a b(a f(b)-b f(a))]=0 $$ for sufficiently large $c$. Since the coefficient of $c$ in this linear relation is nonzero, it follows that there are constants $k, \\ell$ such that $f(c)=k c+\\ell$ for all sufficiently large $c$; the constants $k$ and $\\ell$ are necessarily integers. The value of $\\ell$ satisfies $$ \\left[a^{2} f(b)-b^{2} f(a)\\right] \\cdot \\ell+[a b(a f(b)-b f(a))]=0 $$ and hence $b \\mid \\ell a^{2} f(b)$ for all $a$ and $b$. Taking $b$ sufficiently large so that $f(b)=k b+\\ell$, we thus have that $b \\mid \\ell^{2} a^{2}$ for all sufficiently large $b$; this implies that $\\ell=0$. From ( $\\dagger \\dagger \\dagger$ ) it then follows that $\\frac{f(a)}{a}=\\frac{f(b)}{b}$ for all $a \\neq b$, so that there is a constant $k$ such that $f(a)=k a$ for all $a$ ( $k$ is equal to the constant defined earlier).","tier":0} +{"problem_type":"Number Theory","problem_label":"N4","problem":"Let $\\mathbb{Z}_{>0}$ be the set of positive integers. A positive integer constant $C$ is given. Find all functions $f: \\mathbb{Z}_{>0} \\rightarrow \\mathbb{Z}_{>0}$ such that, for all positive integers $a$ and $b$ satisfying $a+b>C$, $$ a+f(b) \\mid a^{2}+b f(a) $$ (Croatia)","solution":"Let $\\Gamma$ denote the set of all points $(a, f(a))$, so that $\\Gamma$ is an infinite subset of the upper-right quadrant of the plane. For a point $A=(a, f(a))$ in $\\Gamma$, we define a point $A^{\\prime}=\\left(-f(a),-f(a)^{2} \/ a\\right)$ in the lower-left quadrant of the plane, and let $\\Gamma^{\\prime}$ denote the set of all such points $A^{\\prime}$. ![](https:\/\/cdn.mathpix.com\/cropped\/2024_11_18_9ee8c2ac2c2f5002ff14g-095.jpg?height=552&width=652&top_left_y=449&top_left_x=702) Claim. For any point $A \\in \\Gamma$, the set $\\Gamma$ is contained in finitely many lines through the point $A^{\\prime}$. Proof. Let $A=(a, f(a))$. The functional equation (with $a$ and $b$ interchanged) can be rewritten as $b+f(a) \\mid a f(b)-b f(a)$, so that all but finitely many points in $\\Gamma$ are contained in one of the lines with equation $$ a y-f(a) x=m(x+f(a)) $$ for $m$ an integer. Geometrically, these are the lines through $A^{\\prime}=\\left(-f(a),-f(a)^{2} \/ a\\right)$ with gradient $\\frac{f(a)+m}{a}$. Since $\\Gamma$ is contained, with finitely many exceptions, in the region $0 \\leqslant y \\leqslant$ $f(1) \\cdot x$ and the point $A^{\\prime}$ lies strictly in the lower-left quadrant of the plane, there are only finitely many values of $m$ for which this line meets $\\Gamma$. This concludes the proof of the claim. Now consider any distinct points $A, B \\in \\Gamma$. It is clear that $A^{\\prime}$ and $B^{\\prime}$ are distinct. A line through $A^{\\prime}$ and a line through $B^{\\prime}$ only meet in more than one point if these two lines are equal to the line $A^{\\prime} B^{\\prime}$. It then follows from the above claim that the line $A^{\\prime} B^{\\prime}$ must contain all but finitely many points of $\\Gamma$. If $C$ is another point of $\\Gamma$, then the line $A^{\\prime} C^{\\prime}$ also passes through all but finitely many points of $\\Gamma$, which is only possible if $A^{\\prime} C^{\\prime}=A^{\\prime} B^{\\prime}$. We have thus seen that there is a line $\\ell$ passing through all points of $\\Gamma^{\\prime}$ and through all but finitely many points of $\\Gamma$. We claim that this line passes through the origin $O$ and passes through every point of $\\Gamma$. To see this, note that by construction $A, O, A^{\\prime}$ are collinear for every point $A \\in \\Gamma$. Since $\\ell=A A^{\\prime}$ for all but finitely many points $A \\in \\Gamma$, it thus follows that $O \\in \\ell$. Thus any $A \\in \\Gamma$ lies on the line $\\ell=A^{\\prime} O$. Since $\\Gamma$ is contained in a line through $O$, it follows that there is a real constant $k$ (the gradient of $\\ell$ ) such that $f(a)=k a$ for all $a$. The number $k$ is, of course, a positive integer. Comment. Without the $a+b>C$ condition, this problem is approachable by much more naive methods. For instance, using the given divisibility for $a, b \\in\\{1,2,3\\}$ one can prove by a somewhat tedious case-check that $f(2)=2 f(1)$ and $f(3)=3 f(1)$; this then forms the basis of an induction establishing that $f(n)=n f(1)$ for all $n$.","tier":0} +{"problem_type":"Number Theory","problem_label":"N5","problem":"Let $a$ be a positive integer. We say that a positive integer $b$ is $a$-good if $\\binom{a n}{b}-1$ is divisible by $a n+1$ for all positive integers $n$ with $a n \\geqslant b$. Suppose $b$ is a positive integer such that $b$ is $a$-good, but $b+2$ is not $a$-good. Prove that $b+1$ is prime. (Netherlands)","solution":"For $p$ a prime and $n$ a nonzero integer, we write $v_{p}(n)$ for the $p$-adic valuation of $n$ : the largest integer $t$ such that $p^{t} \\mid n$. We first show that $b$ is $a$-good if and only if $b$ is even, and $p \\mid a$ for all primes $p \\leqslant b$. To start with, the condition that $a n+1 \\left\\lvert\\,\\binom{ a n}{b}-1\\right.$ can be rewritten as saying that $$ \\frac{a n(a n-1) \\cdots(a n-b+1)}{b!} \\equiv 1 \\quad(\\bmod a n+1) $$ Suppose, on the one hand, there is a prime $p \\leqslant b$ with $p \\nmid a$. Take $t=v_{p}(b!)$. Then there exist positive integers $c$ such that $a c \\equiv 1\\left(\\bmod p^{t+1}\\right)$. If we take $c$ big enough, and then take $n=(p-1) c$, then $a n=a(p-1) c \\equiv p-1\\left(\\bmod p^{t+1}\\right)$ and $a n \\geqslant b$. Since $p \\leqslant b$, one of the terms of the numerator an $(a n-1) \\cdots(a n-b+1)$ is $a n-p+1$, which is divisible by $p^{t+1}$. Hence the $p$-adic valuation of the numerator is at least $t+1$, but that of the denominator is exactly $t$. This means that $p \\left\\lvert\\,\\binom{ a n}{b}\\right.$, so $p \\nmid\\binom{a n}{b}-1$. As $p \\mid a n+1$, we get that $a n+1 \\nmid\\binom{a n}{b}$, so $b$ is not $a$-good. On the other hand, if for all primes $p \\leqslant b$ we have $p \\mid a$, then every factor of $b$ ! is coprime to $a n+1$, and hence invertible modulo $a n+1$ : hence $b$ ! is also invertible modulo $a n+1$. Then equation (1) reduces to: $$ a n(a n-1) \\cdots(a n-b+1) \\equiv b!\\quad(\\bmod a n+1) $$ However, we can rewrite the left-hand side as follows: $$ a n(a n-1) \\cdots(a n-b+1) \\equiv(-1)(-2) \\cdots(-b) \\equiv(-1)^{b} b!\\quad(\\bmod a n+1) $$ Provided that $a n>1$, if $b$ is even we deduce $(-1)^{b} b!\\equiv b$ ! as needed. On the other hand, if $b$ is odd, and we take $a n+1>2(b!)$, then we will not have $(-1)^{b} b!\\equiv b$ !, so $b$ is not $a$-good. This completes the claim. To conclude from here, suppose that $b$ is $a$-good, but $b+2$ is not. Then $b$ is even, and $p \\mid a$ for all primes $p \\leqslant b$, but there is a prime $q \\leqslant b+2$ for which $q \\nmid a$ : so $q=b+1$ or $q=b+2$. We cannot have $q=b+2$, as that is even too, so we have $q=b+1$ : in other words, $b+1$ is prime.","tier":0} +{"problem_type":"Number Theory","problem_label":"N5","problem":"Let $a$ be a positive integer. We say that a positive integer $b$ is $a$-good if $\\binom{a n}{b}-1$ is divisible by $a n+1$ for all positive integers $n$ with $a n \\geqslant b$. Suppose $b$ is a positive integer such that $b$ is $a$-good, but $b+2$ is not $a$-good. Prove that $b+1$ is prime. (Netherlands)","solution":"We show only half of the claim of the previous solution: we show that if $b$ is $a$-good, then $p \\mid a$ for all primes $p \\leqslant b$. We do this with Lucas' theorem. Suppose that we have $p \\leqslant b$ with $p \\nmid a$. Then consider the expansion of $b$ in base $p$; there will be some digit (not the final digit) which is nonzero, because $p \\leqslant b$. Suppose it is the $p^{t}$ digit for $t \\geqslant 1$. Now, as $n$ varies over the integers, an +1 runs over all residue classes modulo $p^{t+1}$; in particular, there is a choice of $n$ (with $a n>b$ ) such that the $p^{0}$ digit of $a n$ is $p-1$ (so $p \\mid a n+1$ ) and the $p^{t}$ digit of $a n$ is 0 . Consequently, $p \\mid a n+1$ but $p \\left\\lvert\\,\\binom{ a n}{b}\\right.$ (by Lucas' theorem) so $p \\nmid\\binom{a n}{b}-1$. Thus $b$ is not $a$-good. Now we show directly that if $b$ is $a$-good but $b+2$ fails to be so, then there must be a prime dividing $a n+1$ for some $n$, which also divides $(b+1)(b+2)$. Indeed, the ratio between $\\binom{a n}{b+2}$ and $\\binom{a n}{b}$ is $(b+1)(b+2) \/(a n-b)(a n-b-1)$. We know that there must be a choice of $a n+1$ such that the former binomial coefficient is 1 modulo $a n+1$ but the latter is not, which means that the given ratio must not be $1 \\bmod a n+1$. If $b+1$ and $b+2$ are both coprime to $a n+1$ then the ratio is 1 , so that must not be the case. In particular, as any prime less than $b$ divides $a$, it must be the case that either $b+1$ or $b+2$ is prime. However, we can observe that $b$ must be even by insisting that $a n+1$ is prime (which is possible by Dirichlet's theorem) and hence $\\binom{a n}{b} \\equiv(-1)^{b}=1$. Thus $b+2$ cannot be prime, so $b+1$ must be prime.","tier":0} +{"problem_type":"Number Theory","problem_label":"N6","problem":"Let $H=\\left\\{\\lfloor i \\sqrt{2}\\rfloor: i \\in \\mathbb{Z}_{>0}\\right\\}=\\{1,2,4,5,7, \\ldots\\}$, and let $n$ be a positive integer. Prove that there exists a constant $C$ such that, if $A \\subset\\{1,2, \\ldots, n\\}$ satisfies $|A| \\geqslant C \\sqrt{n}$, then there exist $a, b \\in A$ such that $a-b \\in H$. (Brazil)","solution":"First, observe that if $n$ is a positive integer, then $n \\in H$ exactly when $$ \\left\\{\\frac{n}{\\sqrt{2}}\\right\\}>1-\\frac{1}{\\sqrt{2}} . $$ To see why, observe that $n \\in H$ if and only if $00}$. In other words, $01$ since $a_{i}-a_{1} \\notin H$. Furthermore, we must have $\\left\\{a_{i} \/ \\sqrt{2}\\right\\}<\\left\\{a_{j} \/ \\sqrt{2}\\right\\}$ whenever $i1 \/ \\sqrt{2}>$ $1-1 \/ \\sqrt{2}$, contradicting (1). Now, we have a sequence $0=a_{1}\\frac{1}{2 d \\sqrt{2}} $$ To see why this is the case, let $h=\\lfloor d \/ \\sqrt{2}\\rfloor$, so $\\{d \/ \\sqrt{2}\\}=d \/ \\sqrt{2}-h$. Then $$ \\left\\{\\frac{d}{\\sqrt{2}}\\right\\}\\left(\\frac{d}{\\sqrt{2}}+h\\right)=\\frac{d^{2}-2 h^{2}}{2} \\geqslant \\frac{1}{2} $$ since the numerator is a positive integer. Because $d \/ \\sqrt{2}+h<2 d \/ \\sqrt{2}$, inequality (2) follows. Let $d_{i}=a_{i+1}-a_{i}$, for $1 \\leqslant i\\sum_{i}\\left\\{\\frac{d_{i}}{\\sqrt{2}}\\right\\}>\\frac{1}{2 \\sqrt{2}} \\sum_{i} \\frac{1}{d_{i}} \\geqslant \\frac{(k-1)^{2}}{2 \\sqrt{2}} \\frac{1}{\\sum_{i} d_{i}}>\\frac{(k-1)^{2}}{2 \\sqrt{2}} \\cdot \\frac{1}{n} . $$ Here, the first inequality holds because $\\left\\{a_{k} \/ \\sqrt{2}\\right\\}<1-1 \/ \\sqrt{2}$, the second follows from (2), the third follows from an easy application of the AM-HM inequality (or Cauchy-Schwarz), and the fourth follows from the fact that $\\sum_{i} d_{i}=a_{k}k-1 $$ which provides the required bound on $k$.","tier":0} +{"problem_type":"Number Theory","problem_label":"N6","problem":"Let $H=\\left\\{\\lfloor i \\sqrt{2}\\rfloor: i \\in \\mathbb{Z}_{>0}\\right\\}=\\{1,2,4,5,7, \\ldots\\}$, and let $n$ be a positive integer. Prove that there exists a constant $C$ such that, if $A \\subset\\{1,2, \\ldots, n\\}$ satisfies $|A| \\geqslant C \\sqrt{n}$, then there exist $a, b \\in A$ such that $a-b \\in H$. (Brazil)","solution":"Let $\\alpha=2+\\sqrt{2}$, so $(1 \/ \\alpha)+(1 \/ \\sqrt{2})=1$. Thus, $J=\\left\\{\\lfloor i \\alpha\\rfloor: i \\in \\mathbb{Z}_{>0}\\right\\}$ is the complementary Beatty sequence to $H$ (in other words, $H$ and $J$ are disjoint with $H \\cup J=\\mathbb{Z}_{>0}$ ). Write $A=\\left\\{a_{1}0}$. For any $j>i$, we have $a_{j}-a_{i}=\\left\\lfloor\\alpha b_{j}\\right\\rfloor-\\left\\lfloor\\alpha b_{i}\\right\\rfloor$. Because $a_{j}-a_{i} \\in J$, we also have $a_{j}-a_{i}=\\lfloor\\alpha t\\rfloor$ for some positive integer $t$. Thus, $\\lfloor\\alpha t\\rfloor=\\left\\lfloor\\alpha b_{j}\\right\\rfloor-\\left\\lfloor\\alpha b_{i}\\right\\rfloor$. The right hand side must equal either $\\left\\lfloor\\alpha\\left(b_{j}-b_{i}\\right)\\right\\rfloor$ or $\\left\\lfloor\\alpha\\left(b_{j}-b_{i}\\right)\\right\\rfloor-1$, the latter of which is not a member of $J$ as $\\alpha>2$. Therefore, $t=b_{j}-b_{i}$ and so we have $\\left\\lfloor\\alpha b_{j}\\right\\rfloor-\\left\\lfloor\\alpha b_{i}\\right\\rfloor=\\left\\lfloor\\alpha\\left(b_{j}-b_{i}\\right)\\right\\rfloor$. For $1 \\leqslant i1 \/(2 d \\sqrt{2})$ for positive integers $d)$ proves that $1>\\left((k-1)^{2} \/(2 \\sqrt{2})\\right)(\\alpha \/ n)$, which again rearranges to give $$ \\sqrt{2 \\sqrt{2}-2} \\cdot \\sqrt{n}>k-1 $$ Comment. The use of Beatty sequences in Solution 2 is essentially a way to bypass (1). Both Solutions 1 and 2 use the fact that $\\sqrt{2}<2$; the statement in the question would still be true if $\\sqrt{2}$ did not have this property (for instance, if it were replaced with $\\alpha$ ), but any argument along the lines of Solutions 1 or 2 would be more complicated.","tier":0} +{"problem_type":"Number Theory","problem_label":"N6","problem":"Let $H=\\left\\{\\lfloor i \\sqrt{2}\\rfloor: i \\in \\mathbb{Z}_{>0}\\right\\}=\\{1,2,4,5,7, \\ldots\\}$, and let $n$ be a positive integer. Prove that there exists a constant $C$ such that, if $A \\subset\\{1,2, \\ldots, n\\}$ satisfies $|A| \\geqslant C \\sqrt{n}$, then there exist $a, b \\in A$ such that $a-b \\in H$. (Brazil)","solution":"Again, define $J=\\mathbb{Z}_{>0} \\backslash H$, so all differences between elements of $A$ are in $J$. We start by making the following observation. Suppose we have a set $B \\subseteq\\{1,2, \\ldots, n\\}$ such that all of the differences between elements of $B$ are in $H$. Then $|A| \\cdot|B| \\leqslant 2 n$. To see why, observe that any two sums of the form $a+b$ with $a \\in A, b \\in B$ are different; otherwise, we would have $a_{1}+b_{1}=a_{2}+b_{2}$, and so $\\left|a_{1}-a_{2}\\right|=\\left|b_{2}-b_{1}\\right|$. However, then the left hand side is in $J$ whereas the right hand side is in $H$. Thus, $\\{a+b: a \\in A, b \\in B\\}$ is a set of size $|A| \\cdot|B|$ all of whose elements are no greater than $2 n$, yielding the claimed inequality. With this in mind, it suffices to construct a set $B$, all of whose differences are in $H$ and whose size is at least $C^{\\prime} \\sqrt{n}$ for some constant $C^{\\prime}>0$. To do so, we will use well-known facts about the negative Pell equation $X^{2}-2 Y^{2}=-1$; in particular, that there are infinitely many solutions and the values of $X$ are given by the recurrence $X_{1}=1, X_{2}=7$ and $X_{m}=6 X_{m-1}-X_{m-2}$. Therefore, we may choose $X$ to be a solution with $\\sqrt{n} \/ 6\\left(\\frac{X}{\\sqrt{2}}-Y\\right) \\geqslant \\frac{-3}{\\sqrt{2 n}}, $$ from which it follows that $\\{X \/ \\sqrt{2}\\}>1-(3 \/ \\sqrt{2 n})$. Combined with (1), this shows that all differences between elements of $B$ are in $H$. Comment. Some of the ideas behind Solution 3 may be used to prove that the constant $C=\\sqrt{2 \\sqrt{2}-2}$ (from Solutions 1 and 2) is optimal, in the sense that there are arbitrarily large values of $n$ and sets $A_{n} \\subseteq\\{1,2, \\ldots, n\\}$ of size roughly $C \\sqrt{n}$, all of whose differences are contained in $J$. To see why, choose $X$ to come from a sufficiently large solution to the Pell equation $X^{2}-2 Y^{2}=1$, so $\\{X \/ \\sqrt{2}\\} \\approx 1 \/(2 X \\sqrt{2})$. In particular, $\\{X\\},\\{2 X\\}, \\ldots,\\{[2 X \\sqrt{2}(1-1 \/ \\sqrt{2})\\rfloor X\\}$ are all less than $1-1 \/ \\sqrt{2}$. Thus, by (1) any positive integer of the form $i X$ for $1 \\leqslant i \\leqslant\\lfloor 2 X \\sqrt{2}(1-1 \/ \\sqrt{2})\\rfloor$ lies in $J$. Set $n \\approx 2 X^{2} \\sqrt{2}(1-1 \/ \\sqrt{2})$. We now have a set $A=\\{i X: i \\leqslant\\lfloor 2 X \\sqrt{2}(1-1 \/ \\sqrt{2})\\rfloor\\}$ containing roughly $2 X \\sqrt{2}(1-1 \/ \\sqrt{2})$ elements less than or equal to $n$ such that all of the differences lie in $J$, and we can see that $|A| \\approx C \\sqrt{n}$ with $C=\\sqrt{2 \\sqrt{2}-2}$.","tier":0} +{"problem_type":"Number Theory","problem_label":"N6","problem":"Let $H=\\left\\{\\lfloor i \\sqrt{2}\\rfloor: i \\in \\mathbb{Z}_{>0}\\right\\}=\\{1,2,4,5,7, \\ldots\\}$, and let $n$ be a positive integer. Prove that there exists a constant $C$ such that, if $A \\subset\\{1,2, \\ldots, n\\}$ satisfies $|A| \\geqslant C \\sqrt{n}$, then there exist $a, b \\in A$ such that $a-b \\in H$. (Brazil)","solution":"As in Choose $Y$ to be a solution to the Pell-like equation $X^{2}-2 Y^{2}= \\pm 1$; such solutions are given by the recurrence $Y_{1}=1, Y_{2}=2$ and $Y_{m}=2 Y_{m-1}+Y_{m-2}$, and so we can choose $Y$ such that $n \/(3 \\sqrt{2})\\sqrt{Y}$ elements; if that subsequence is decreasing, we may reflect (using (4) or (5)) to ensure that it is increasing. Call the subsequence $\\left\\{y_{1} \\sqrt{2}\\right\\},\\left\\{y_{2} \\sqrt{2}\\right\\}, \\ldots,\\left\\{y_{t} \\sqrt{2}\\right\\}$ for $t>\\sqrt{Y}$. Now, set $B=\\left\\{\\left\\lfloor y_{i} \\sqrt{2}\\right\\rfloor: 1 \\leqslant i \\leqslant t\\right\\}$. We have $\\left\\lfloor y_{j} \\sqrt{2}\\right\\rfloor-\\left\\lfloor y_{i} \\sqrt{2}\\right\\rfloor=\\left\\lfloor\\left(y_{j}-y_{i}\\right) \\sqrt{2}\\right\\rfloor$ for $i\\sqrt{Y}>\\sqrt{n} \/ \\sqrt{3 \\sqrt{2}}$, this is the required set. Comment. Any solution to this problem will need to use the fact that $\\sqrt{2}$ cannot be approximated well by rationals, either directly or implicitly (for example, by using facts about solutions to Pelllike equations). If $\\sqrt{2}$ were replaced by a value of $\\theta$ with very good rational approximations (from below), then an argument along the lines of Solution 3 would give long arithmetic progressions in $\\{\\lfloor i \\theta\\rfloor: 0 \\leqslant i0$ and infinitely many positive integers $n$ with the following property: there are infinitely many positive integers that cannot be expressed as the sum of fewer than $c n \\log (n)$ pairwise coprime $n^{\\text {th }}$ powers. (Canada)","solution":"Suppose, for an integer $n$, that we can find another integer $N$ satisfying the following property: $n$ is divisible by $\\varphi\\left(p^{e}\\right)$ for every prime power $p^{e}$ exactly dividing $N$. This property ensures that all $n^{\\text {th }}$ powers are congruent to 0 or 1 modulo each such prime power $p^{e}$, and hence that any sum of $m$ pairwise coprime $n^{\\text {th }}$ powers is congruent to $m$ or $m-1$ modulo $p^{e}$, since at most one of the $n^{\\text {th }}$ powers is divisible by $p$. Thus, if $k$ denotes the number of distinct prime factors of $N$, we find by the Chinese Remainder Theorem at most $2^{k} m$ residue classes modulo $N$ which are sums of at most $m$ pairwise coprime $n^{\\text {th }}$ powers. In particular, if $N>2^{k} m$ then there are infinitely many positive integers not expressible as a sum of at most $m$ pairwise coprime $n^{\\text {th }}$ powers. It thus suffices to prove that there are arbitrarily large pairs $(n, N)$ of integers satisfying $(\\dagger)$ such that $$ N>c \\cdot 2^{k} n \\log (n) $$ for some positive constant $c$. We construct such pairs as follows. Fix a positive integer $t$ and choose (distinct) prime numbers $p \\mid 2^{2^{t-1}}+1$ and $q \\mid 2^{2^{t}}+1$; we set $N=p q$. It is well-known that $2^{t} \\mid p-1$ and $2^{t+1} \\mid q-1$, hence $$ n=\\frac{(p-1)(q-1)}{2^{t}} $$ is an integer and the pair $(n, N)$ satisfies $(\\dagger)$. Estimating the size of $N$ and $n$ is now straightforward. We have $$ \\log _{2}(n) \\leqslant 2^{t-1}+2^{t}-t<2^{t+1}<2 \\cdot \\frac{N}{n} $$ which rearranges to $$ N>\\frac{1}{8} \\cdot 2^{2} n \\log _{2}(n) $$ and so we are done if we choose $c<\\frac{1}{8 \\log (2)} \\approx 0.18$. Comment 1. The trick in the above solution was to find prime numbers $p$ and $q$ congruent to 1 modulo some $d=2^{t}$ and which are not too large. An alternative way to do this is via Linnik's Theorem, which says that there are absolute constants $b$ and $L>1$ such that for any coprime integers $a$ and $d$, there is a prime congruent to $a$ modulo $d$ and of size $\\leqslant b d^{L}$. If we choose some $d$ not divisible by 3 and choose two distinct primes $p, q \\leqslant b \\cdot(3 d)^{L}$ congruent to 1 modulo $d$ (and, say, distinct modulo 3 ), then we obtain a pair $(n, N)$ satisfying $(\\dagger)$ with $N=p q$ and $n=\\frac{(p-1)(q-1)}{d}$. A straightforward computation shows that $$ N>C n^{1+\\frac{1}{2 L-1}} $$ for some constant $C$, which is in particular larger than any $c \\cdot 2^{2} n \\log (n)$ for $p$ large. Thus, the statement of the problem is true for any constant $c$. More strongly, the statement of the problem is still true when $c n \\log (n)$ is replaced by $n^{1+\\delta}$ for a sufficiently small $\\delta>0$. Solution 2, obtaining better bounds. As in the preceding solution, we seek arbitrarily large pairs of integers $n$ and $N$ satisfying ( $\\dagger$ ) such that $N>c 2^{k} n \\log (n)$. This time, to construct such pairs, we fix an integer $x \\geqslant 4$, set $N$ to be the lowest common multiple of $1,2, \\ldots, 2 x$, and set $n$ to be twice the lowest common multiple of $1,2, \\ldots, x$. The pair $(n, N)$ does indeed satisfy the condition, since if $p^{e}$ is a prime power divisor of $N$ then $\\frac{\\varphi\\left(p^{e}\\right)}{2} \\leqslant x$ is a factor of $\\frac{n}{2}=\\operatorname{lcm}_{r \\leqslant x}(r)$. Now $2 N \/ n$ is the product of all primes having a power lying in the interval $(x, 2 x]$, and hence $2 N \/ n>x^{\\pi(2 x)-\\pi(x)}$. Thus for sufficiently large $x$ we have $$ \\log \\left(\\frac{2 N}{2^{\\pi(2 x)} n}\\right)>(\\pi(2 x)-\\pi(x)) \\log (x)-\\log (2) \\pi(2 x) \\sim x $$ using the Prime Number Theorem $\\pi(t) \\sim t \/ \\log (t)$. On the other hand, $n$ is a product of at most $\\pi(x)$ prime powers less than or equal to $x$, and so we have the upper bound $$ \\log (n) \\leqslant \\pi(x) \\log (x) \\sim x $$ again by the Prime Number Theorem. Combined with the above inequality, we find that for any $\\epsilon>0$, the inequality $$ \\log \\left(\\frac{N}{2^{\\pi(2 x)} n}\\right)>(1-\\epsilon) \\log (n) $$ holds for sufficiently large $x$. Rearranging this shows that $$ N>2^{\\pi(2 x)} n^{2-\\epsilon}>2^{\\pi(2 x)} n \\log (n) $$ for all sufficiently large $x$ and we are done. Comment 2. The stronger bound $N>2^{\\pi(2 x)} n^{2-\\epsilon}$ obtained in the above proof of course shows that infinitely many positive integers cannot be written as a sum of at most $n^{2-\\epsilon}$ pairwise coprime $n^{\\text {th }}$ powers. By refining the method in Solution 2, these bounds can be improved further to show that infinitely many positive integers cannot be written as a sum of at most $n^{\\alpha}$ pairwise coprime $n^{\\text {th }}$ powers for any positive $\\alpha>0$. To do this, one fixes a positive integer $d$, sets $N$ equal to the product of the primes at most $d x$ which are congruent to 1 modulo $d$, and $n=d \\mathrm{lcm}_{r \\leqslant x}(r)$. It follows as in Solution 2 that $(n, N)$ satisfies $(\\dagger)$. Now the Prime Number Theorem in arithmetic progressions provides the estimates $\\log (N) \\sim \\frac{d}{\\varphi(d)} x$, $\\log (n) \\sim x$ and $\\pi(d x) \\sim \\frac{d x}{\\log (x)}$ for any fixed $d$. Combining these provides a bound $$ N>2^{\\pi(d x)} n^{d \/ \\varphi(d)-\\epsilon} $$ for any positive $\\epsilon$, valid for $x$ sufficiently large. Since the ratio $\\frac{d}{\\varphi(d)}$ can be made arbitrarily large by a judicious choice of $d$, we obtain the $n^{\\alpha}$ bound claimed. Comment 3. While big results from analytic number theory such as the Prime Number Theorem or Linnik's Theorem certainly can be used in this problem, they do not seem to substantially simplify matters: all known solutions involve first reducing to condition ( $\\dagger$ ), and even then analytic results do not make it clear how to proceed. For this reason, we regard this problem as suitable for the IMO. Rather than simplifying the problem, what nonelementary results from analytic number theory allow one to achieve is a strengthening of the main bound, typically replacing the $n \\log (n)$ growth with a power $n^{1+\\delta}$. However, we believe that such stronger bounds are unlikely to be found by students in the exam. The strongest bound we know how to achieve using purely elementary methods is a bound of the form $N>2^{k} n \\log (n)^{M}$ for any positive integer $M$. This is achieved by a variant of the argument in Solution 1, choosing primes $p_{0}, \\ldots, p_{M}$ with $p_{i} \\mid 2^{2^{t+i-1}}+1$ and setting $N=\\prod_{i} p_{i}$ and $n=$ $2^{-t M} \\prod_{i}\\left(p_{i}-1\\right)$.","tier":0} +{"problem_type":"Number Theory","problem_label":"N8","problem":"Let $a$ and $b$ be two positive integers. Prove that the integer $$ a^{2}+\\left\\lceil\\frac{4 a^{2}}{b}\\right\\rceil $$ is not a square. (Here $\\lceil z\\rceil$ denotes the least integer greater than or equal to $z$.)","solution":"Arguing indirectly, assume that $$ a^{2}+\\left\\lceil\\frac{4 a^{2}}{b}\\right\\rceil=(a+k)^{2}, \\quad \\text { or } \\quad\\left\\lceil\\frac{(2 a)^{2}}{b}\\right\\rceil=(2 a+k) k $$ Clearly, $k \\geqslant 1$. In other words, the equation $$ \\left\\lceil\\frac{c^{2}}{b}\\right\\rceil=(c+k) k $$ has a positive integer solution $(c, k)$, with an even value of $c$. Choose a positive integer solution of (1) with minimal possible value of $k$, without regard to the parity of $c$. From $$ \\frac{c^{2}}{b}>\\left\\lceil\\frac{c^{2}}{b}\\right\\rceil-1=c k+k^{2}-1 \\geqslant c k $$ and $$ \\frac{(c-k)(c+k)}{b}<\\frac{c^{2}}{b} \\leqslant\\left\\lceil\\frac{c^{2}}{b}\\right\\rceil=(c+k) k $$ it can be seen that $c>b k>c-k$, so $$ c=k b+r \\quad \\text { with some } 00$ and $0a$, so $$ \\begin{aligned} & c^{2}-10$ and $a<0$. Indeed, since $\\sum_{i=1}^{2019} u_{i}^{2}=1$, the variables $u_{i}$ cannot be all zero, and, since $\\sum_{i=1}^{2019} u_{i}=0$, the nonzero elements cannot be all positive or all negative. Let $P=\\left\\{i: u_{i}>0\\right\\}$ and $N=\\left\\{i: u_{i} \\leqslant 0\\right\\}$ be the indices of positive and nonpositive elements in the sequence, and let $p=|P|$ and $n=|N|$ be the sizes of these sets; then $p+n=2019$. By the condition $\\sum_{i=1}^{2019} u_{i}=0$ we have $0=\\sum_{i=1}^{2019} u_{i}=\\sum_{i \\in P} u_{i}-\\sum_{i \\in N}\\left|u_{i}\\right|$, so $$ \\sum_{i \\in P} u_{i}=\\sum_{i \\in N}\\left|u_{i}\\right| . $$ After this preparation, estimate the sum of squares of the positive and nonpositive elements as follows: $$ \\begin{aligned} & \\sum_{i \\in P} u_{i}^{2} \\leqslant \\sum_{i \\in P} b u_{i}=b \\sum_{i \\in P} u_{i}=b \\sum_{i \\in N}\\left|u_{i}\\right| \\leqslant b \\sum_{i \\in N}|a|=-n a b ; \\\\ & \\sum_{i \\in N} u_{i}^{2} \\leqslant \\sum_{i \\in N}|a| \\cdot\\left|u_{i}\\right|=|a| \\sum_{i \\in N}\\left|u_{i}\\right|=|a| \\sum_{i \\in P} u_{i} \\leqslant|a| \\sum_{i \\in P} b=-p a b . \\end{aligned} $$ The sum of these estimates is $$ 1=\\sum_{i=1}^{2019} u_{i}^{2}=\\sum_{i \\in P} u_{i}^{2}+\\sum_{i \\in N} u_{i}^{2} \\leqslant-(p+n) a b=-2019 a b ; $$ that proves $a b \\leqslant \\frac{-1}{2019}$. Comment 1. After observing $\\sum_{i \\in P} u_{i}^{2} \\leqslant b \\sum_{i \\in P} u_{i}$ and $\\sum_{i \\in N} u_{i}^{2} \\leqslant|a| \\sum_{i \\in P}\\left|u_{i}\\right|$, instead of $(2,3)$ an alternative continuation is $$ |a b| \\geqslant \\frac{\\sum_{i \\in P} u_{i}^{2}}{\\sum_{i \\in P} u_{i}} \\cdot \\frac{\\sum_{i \\in N} u_{i}^{2}}{\\sum_{i \\in N}\\left|u_{i}\\right|}=\\frac{\\sum_{i \\in P} u_{i}^{2}}{\\left(\\sum_{i \\in P} u_{i}\\right)^{2}} \\sum_{i \\in N} u_{i}^{2} \\geqslant \\frac{1}{p} \\sum_{i \\in N} u_{i}^{2} $$ (by the AM-QM or the Cauchy-Schwarz inequality) and similarly $|a b| \\geqslant \\frac{1}{n} \\sum_{i \\in P} u_{i}^{2}$.","tier":0} +{"problem_type":"Algebra","problem_label":"A2","problem":"Let $u_{1}, u_{2}, \\ldots, u_{2019}$ be real numbers satisfying $$ u_{1}+u_{2}+\\cdots+u_{2019}=0 \\quad \\text { and } \\quad u_{1}^{2}+u_{2}^{2}+\\cdots+u_{2019}^{2}=1 $$ Let $a=\\min \\left(u_{1}, u_{2}, \\ldots, u_{2019}\\right)$ and $b=\\max \\left(u_{1}, u_{2}, \\ldots, u_{2019}\\right)$. Prove that $$ a b \\leqslant-\\frac{1}{2019} $$ (Germany)","solution":"As in the previous solution we conclude that $a<0$ and $b>0$. For every index $i$, the number $u_{i}$ is a convex combination of $a$ and $b$, so $$ u_{i}=x_{i} a+y_{i} b \\quad \\text { with some weights } 0 \\leqslant x_{i}, y_{i} \\leqslant 1, \\text { with } x_{i}+y_{i}=1 \\text {. } $$ Let $X=\\sum_{i=1}^{2019} x_{i}$ and $Y=\\sum_{i=1}^{2019} y_{i}$. From $0=\\sum_{i=1}^{2019} u_{i}=\\sum_{i=1}^{2019}\\left(x_{i} a+y_{i} b\\right)=-|a| X+b Y$, we get $$ |a| X=b Y $$ From $\\sum_{i=1}^{2019}\\left(x_{i}+y_{i}\\right)=2019$ we have $$ X+Y=2019 $$ The system of linear equations $(4,5)$ has a unique solution: $$ X=\\frac{2019 b}{|a|+b}, \\quad Y=\\frac{2019|a|}{|a|+b} $$ Now apply the following estimate to every $u_{i}^{2}$ in their sum: $$ u_{i}^{2}=x_{i}^{2} a^{2}+2 x_{i} y_{i} a b+y_{i}^{2} b^{2} \\leqslant x_{i} a^{2}+y_{i} b^{2} $$ we obtain that $$ 1=\\sum_{i=1}^{2019} u_{i}^{2} \\leqslant \\sum_{i=1}^{2019}\\left(x_{i} a^{2}+y_{i} b^{2}\\right)=X a^{2}+Y b^{2}=\\frac{2019 b}{|a|+b}|a|^{2}+\\frac{2019|a|}{|a|+b} b^{2}=2019|a| b=-2019 a b . $$ Hence, $a b \\leqslant \\frac{-1}{2019}$. Comment 2. The idea behind Solution 2 is the following thought. Suppose we fix $a<0$ and $b>0$, fix $\\sum u_{i}=0$ and vary the $u_{i}$ to achieve the maximum value of $\\sum u_{i}^{2}$. Considering varying any two of the $u_{i}$ while preserving their sum: the maximum value of $\\sum u_{i}^{2}$ is achieved when those two are as far apart as possible, so all but at most one of the $u_{i}$ are equal to $a$ or $b$. Considering a weighted version of the problem, we see the maximum (with fractional numbers of $u_{i}$ having each value) is achieved when $\\frac{2019 b}{|a|+b}$ of them are $a$ and $\\frac{2019|a|}{|a|+b}$ are $b$. In fact, this happens in the solution: the number $u_{i}$ is replaced by $x_{i}$ copies of $a$ and $y_{i}$ copies of $b$.","tier":0} +{"problem_type":"Algebra","problem_label":"A3","problem":"Let $n \\geqslant 3$ be a positive integer and let $\\left(a_{1}, a_{2}, \\ldots, a_{n}\\right)$ be a strictly increasing sequence of $n$ positive real numbers with sum equal to 2 . Let $X$ be a subset of $\\{1,2, \\ldots, n\\}$ such that the value of $$ \\left|1-\\sum_{i \\in X} a_{i}\\right| $$ is minimised. Prove that there exists a strictly increasing sequence of $n$ positive real numbers $\\left(b_{1}, b_{2}, \\ldots, b_{n}\\right)$ with sum equal to 2 such that $$ \\sum_{i \\in X} b_{i}=1 $$ (New Zealand) Common remarks. In all solutions, we say an index set $X$ is $\\left(a_{i}\\right)$-minimising if it has the property in the problem for the given sequence $\\left(a_{i}\\right)$. Write $X^{c}$ for the complement of $X$, and $[a, b]$ for the interval of integers $k$ such that $a \\leqslant k \\leqslant b$. Note that $$ \\left|1-\\sum_{i \\in X} a_{i}\\right|=\\left|1-\\sum_{i \\in X^{c}} a_{i}\\right|, $$ so we may exchange $X$ and $X^{c}$ where convenient. Let $$ \\Delta=\\sum_{i \\in X^{c}} a_{i}-\\sum_{i \\in X} a_{i} $$ and note that $X$ is $\\left(a_{i}\\right)$-minimising if and only if it minimises $|\\Delta|$, and that $\\sum_{i \\in X} a_{i}=1$ if and only if $\\Delta=0$. In some solutions, a scaling process is used. If we have a strictly increasing sequence of positive real numbers $c_{i}$ (typically obtained by perturbing the $a_{i}$ in some way) such that $$ \\sum_{i \\in X} c_{i}=\\sum_{i \\in X^{c}} c_{i} $$ then we may put $b_{i}=2 c_{i} \/ \\sum_{j=1}^{n} c_{j}$. So it suffices to construct such a sequence without needing its sum to be 2 . The solutions below show various possible approaches to the problem. Solutions 1 and 2 perturb a few of the $a_{i}$ to form the $b_{i}$ (with scaling in the case of Solution 1, without scaling in the case of Solution 2). Solutions 3 and 4 look at properties of the index set $X$. Solution 3 then perturbs many of the $a_{i}$ to form the $b_{i}$, together with scaling. Rather than using such perturbations, Solution 4 constructs a sequence $\\left(b_{i}\\right)$ directly from the set $X$ with the required properties. Solution 4 can be used to give a complete description of sets $X$ that are $\\left(a_{i}\\right)$-minimising for some $\\left(a_{i}\\right)$.","solution":"Without loss of generality, assume $\\sum_{i \\in X} a_{i} \\leqslant 1$, and we may assume strict inequality as otherwise $b_{i}=a_{i}$ works. Also, $X$ clearly cannot be empty. If $n \\in X$, add $\\Delta$ to $a_{n}$, producing a sequence of $c_{i}$ with $\\sum_{i \\in X} c_{i}=\\sum_{i \\in X^{c}} c_{i}$, and then scale as described above to make the sum equal to 2 . Otherwise, there is some $k$ with $k \\in X$ and $k+1 \\in X^{c}$. Let $\\delta=a_{k+1}-a_{k}$. - If $\\delta>\\Delta$, add $\\Delta$ to $a_{k}$ and then scale. - If $\\delta<\\Delta$, then considering $X \\cup\\{k+1\\} \\backslash\\{k\\}$ contradicts $X$ being $\\left(a_{i}\\right)$-minimising. - If $\\delta=\\Delta$, choose any $j \\neq k, k+1$ (possible since $n \\geqslant 3$ ), and any $\\epsilon$ less than the least of $a_{1}$ and all the differences $a_{i+1}-a_{i}$. If $j \\in X$ then add $\\Delta-\\epsilon$ to $a_{k}$ and $\\epsilon$ to $a_{j}$, then scale; otherwise, add $\\Delta$ to $a_{k}$ and $\\epsilon \/ 2$ to $a_{k+1}$, and subtract $\\epsilon \/ 2$ from $a_{j}$, then scale.","tier":0} +{"problem_type":"Algebra","problem_label":"A3","problem":"Let $n \\geqslant 3$ be a positive integer and let $\\left(a_{1}, a_{2}, \\ldots, a_{n}\\right)$ be a strictly increasing sequence of $n$ positive real numbers with sum equal to 2 . Let $X$ be a subset of $\\{1,2, \\ldots, n\\}$ such that the value of $$ \\left|1-\\sum_{i \\in X} a_{i}\\right| $$ is minimised. Prove that there exists a strictly increasing sequence of $n$ positive real numbers $\\left(b_{1}, b_{2}, \\ldots, b_{n}\\right)$ with sum equal to 2 such that $$ \\sum_{i \\in X} b_{i}=1 $$ (New Zealand) Common remarks. In all solutions, we say an index set $X$ is $\\left(a_{i}\\right)$-minimising if it has the property in the problem for the given sequence $\\left(a_{i}\\right)$. Write $X^{c}$ for the complement of $X$, and $[a, b]$ for the interval of integers $k$ such that $a \\leqslant k \\leqslant b$. Note that $$ \\left|1-\\sum_{i \\in X} a_{i}\\right|=\\left|1-\\sum_{i \\in X^{c}} a_{i}\\right|, $$ so we may exchange $X$ and $X^{c}$ where convenient. Let $$ \\Delta=\\sum_{i \\in X^{c}} a_{i}-\\sum_{i \\in X} a_{i} $$ and note that $X$ is $\\left(a_{i}\\right)$-minimising if and only if it minimises $|\\Delta|$, and that $\\sum_{i \\in X} a_{i}=1$ if and only if $\\Delta=0$. In some solutions, a scaling process is used. If we have a strictly increasing sequence of positive real numbers $c_{i}$ (typically obtained by perturbing the $a_{i}$ in some way) such that $$ \\sum_{i \\in X} c_{i}=\\sum_{i \\in X^{c}} c_{i} $$ then we may put $b_{i}=2 c_{i} \/ \\sum_{j=1}^{n} c_{j}$. So it suffices to construct such a sequence without needing its sum to be 2 . The solutions below show various possible approaches to the problem. Solutions 1 and 2 perturb a few of the $a_{i}$ to form the $b_{i}$ (with scaling in the case of Solution 1, without scaling in the case of Solution 2). Solutions 3 and 4 look at properties of the index set $X$. Solution 3 then perturbs many of the $a_{i}$ to form the $b_{i}$, together with scaling. Rather than using such perturbations, Solution 4 constructs a sequence $\\left(b_{i}\\right)$ directly from the set $X$ with the required properties. Solution 4 can be used to give a complete description of sets $X$ that are $\\left(a_{i}\\right)$-minimising for some $\\left(a_{i}\\right)$.","solution":"This is similar to Suppose there exists $1 \\leqslant j \\leqslant n-1$ such that $j \\in X$ but $j+1 \\in X^{c}$. Then $a_{j+1}-a_{j} \\geqslant \\Delta$, because otherwise considering $X \\cup\\{j+1\\} \\backslash\\{j\\}$ contradicts $X$ being $\\left(a_{i}\\right)$-minimising. If $a_{j+1}-a_{j}>\\Delta$, put $$ b_{i}= \\begin{cases}a_{j}+\\Delta \/ 2, & \\text { if } i=j \\\\ a_{j+1}-\\Delta \/ 2, & \\text { if } i=j+1 \\\\ a_{i}, & \\text { otherwise }\\end{cases} $$ If $a_{j+1}-a_{j}=\\Delta$, choose any $\\epsilon$ less than the least of $\\Delta \/ 2, a_{1}$ and all the differences $a_{i+1}-a_{i}$. If $|X| \\geqslant 2$, choose $k \\in X$ with $k \\neq j$, and put $$ b_{i}= \\begin{cases}a_{j}+\\Delta \/ 2-\\epsilon, & \\text { if } i=j \\\\ a_{j+1}-\\Delta \/ 2, & \\text { if } i=j+1 \\\\ a_{k}+\\epsilon, & \\text { if } i=k \\\\ a_{i}, & \\text { otherwise }\\end{cases} $$ Otherwise, $\\left|X^{c}\\right| \\geqslant 2$, so choose $k \\in X^{c}$ with $k \\neq j+1$, and put $$ b_{i}= \\begin{cases}a_{j}+\\Delta \/ 2, & \\text { if } i=j \\\\ a_{j+1}-\\Delta \/ 2+\\epsilon, & \\text { if } i=j+1 \\\\ a_{k}-\\epsilon, & \\text { if } i=k \\\\ a_{i}, & \\text { otherwise }\\end{cases} $$ If there is no $1 \\leqslant j \\leqslant n$ such that $j \\in X$ but $j+1 \\in X^{c}$, there must be some $1\\Delta$, as otherwise considering $X \\cup\\{1\\}$ contradicts $X$ being $\\left(a_{i}\\right)$-minimising. Now put $$ b_{i}= \\begin{cases}a_{1}-\\Delta \/ 2, & \\text { if } i=1 \\\\ a_{n}+\\Delta \/ 2, & \\text { if } i=n \\\\ a_{i}, & \\text { otherwise }\\end{cases} $$","tier":0} +{"problem_type":"Algebra","problem_label":"A3","problem":"Let $n \\geqslant 3$ be a positive integer and let $\\left(a_{1}, a_{2}, \\ldots, a_{n}\\right)$ be a strictly increasing sequence of $n$ positive real numbers with sum equal to 2 . Let $X$ be a subset of $\\{1,2, \\ldots, n\\}$ such that the value of $$ \\left|1-\\sum_{i \\in X} a_{i}\\right| $$ is minimised. Prove that there exists a strictly increasing sequence of $n$ positive real numbers $\\left(b_{1}, b_{2}, \\ldots, b_{n}\\right)$ with sum equal to 2 such that $$ \\sum_{i \\in X} b_{i}=1 $$ (New Zealand) Common remarks. In all solutions, we say an index set $X$ is $\\left(a_{i}\\right)$-minimising if it has the property in the problem for the given sequence $\\left(a_{i}\\right)$. Write $X^{c}$ for the complement of $X$, and $[a, b]$ for the interval of integers $k$ such that $a \\leqslant k \\leqslant b$. Note that $$ \\left|1-\\sum_{i \\in X} a_{i}\\right|=\\left|1-\\sum_{i \\in X^{c}} a_{i}\\right|, $$ so we may exchange $X$ and $X^{c}$ where convenient. Let $$ \\Delta=\\sum_{i \\in X^{c}} a_{i}-\\sum_{i \\in X} a_{i} $$ and note that $X$ is $\\left(a_{i}\\right)$-minimising if and only if it minimises $|\\Delta|$, and that $\\sum_{i \\in X} a_{i}=1$ if and only if $\\Delta=0$. In some solutions, a scaling process is used. If we have a strictly increasing sequence of positive real numbers $c_{i}$ (typically obtained by perturbing the $a_{i}$ in some way) such that $$ \\sum_{i \\in X} c_{i}=\\sum_{i \\in X^{c}} c_{i} $$ then we may put $b_{i}=2 c_{i} \/ \\sum_{j=1}^{n} c_{j}$. So it suffices to construct such a sequence without needing its sum to be 2 . The solutions below show various possible approaches to the problem. Solutions 1 and 2 perturb a few of the $a_{i}$ to form the $b_{i}$ (with scaling in the case of Solution 1, without scaling in the case of Solution 2). Solutions 3 and 4 look at properties of the index set $X$. Solution 3 then perturbs many of the $a_{i}$ to form the $b_{i}$, together with scaling. Rather than using such perturbations, Solution 4 constructs a sequence $\\left(b_{i}\\right)$ directly from the set $X$ with the required properties. Solution 4 can be used to give a complete description of sets $X$ that are $\\left(a_{i}\\right)$-minimising for some $\\left(a_{i}\\right)$.","solution":"Without loss of generality, assume $\\sum_{i \\in X} a_{i} \\leqslant 1$, so $\\Delta \\geqslant 0$. If $\\Delta=0$ we can take $b_{i}=a_{i}$, so now assume that $\\Delta>0$. Suppose that there is some $k \\leqslant n$ such that $|X \\cap[k, n]|>\\left|X^{c} \\cap[k, n]\\right|$. If we choose the largest such $k$ then $|X \\cap[k, n]|-\\left|X^{c} \\cap[k, n]\\right|=1$. We can now find the required sequence $\\left(b_{i}\\right)$ by starting with $c_{i}=a_{i}$ for $i\\frac{n-k+1}{2}$ and $|X \\cap[\\ell, n]|<\\frac{n-\\ell+1}{2}$. We now construct our sequence $\\left(b_{i}\\right)$ using this claim. Let $k$ and $\\ell$ be the greatest values satisfying the claim, and without loss of generality suppose $k=n$ and $\\ell\\sum_{i \\in Y} a_{i}>1 $$ contradicting $X$ being $\\left(a_{i}\\right)$-minimising. Otherwise, we always have equality, meaning that $X=Y$. But now consider $Z=Y \\cup\\{n-1\\} \\backslash\\{n\\}$. Since $n \\geqslant 3$, we have $$ \\sum_{i \\in Y} a_{i}>\\sum_{i \\in Z} a_{i}>\\sum_{i \\in Y^{c}} a_{i}=2-\\sum_{i \\in Y} a_{i} $$ and so $Z$ contradicts $X$ being $\\left(a_{i}\\right)$-minimising.","tier":0} +{"problem_type":"Algebra","problem_label":"A5","problem":"Let $x_{1}, x_{2}, \\ldots, x_{n}$ be different real numbers. Prove that $$ \\sum_{1 \\leqslant i \\leqslant n} \\prod_{j \\neq i} \\frac{1-x_{i} x_{j}}{x_{i}-x_{j}}= \\begin{cases}0, & \\text { if } n \\text { is even } \\\\ 1, & \\text { if } n \\text { is odd }\\end{cases} $$ (Kazakhstan) Common remarks. Let $G\\left(x_{1}, x_{2}, \\ldots, x_{n}\\right)$ be the function of the $n$ variables $x_{1}, x_{2}, \\ldots, x_{n}$ on the LHS of the required identity. Solution 1 (Lagrange interpolation). Since both sides of the identity are rational functions, it suffices to prove it when all $x_{i} \\notin\\{ \\pm 1\\}$. Define $$ f(t)=\\prod_{i=1}^{n}\\left(1-x_{i} t\\right) $$ and note that $$ f\\left(x_{i}\\right)=\\left(1-x_{i}^{2}\\right) \\prod_{j \\neq i} 1-x_{i} x_{j} $$ Using the nodes $+1,-1, x_{1}, \\ldots, x_{n}$, the Lagrange interpolation formula gives us the following expression for $f$ : $$ \\sum_{i=1}^{n} f\\left(x_{i}\\right) \\frac{(x-1)(x+1)}{\\left(x_{i}-1\\right)\\left(x_{i}+1\\right)} \\prod_{j \\neq i} \\frac{x-x_{j}}{x_{i}-x_{j}}+f(1) \\frac{x+1}{1+1} \\prod_{1 \\leqslant i \\leqslant n} \\frac{x-x_{i}}{1-x_{i}}+f(-1) \\frac{x-1}{-1-1} \\prod_{1 \\leqslant i \\leqslant n} \\frac{x-x_{i}}{1-x_{i}} $$ The coefficient of $t^{n+1}$ in $f(t)$ is 0 , since $f$ has degree $n$. The coefficient of $t^{n+1}$ in the above expression of $f$ is $$ \\begin{aligned} 0 & =\\sum_{1 \\leqslant i \\leqslant n} \\frac{f\\left(x_{i}\\right)}{\\prod_{j \\neq i}\\left(x_{i}-x_{j}\\right) \\cdot\\left(x_{i}-1\\right)\\left(x_{i}+1\\right)}+\\frac{f(1)}{\\prod_{1 \\leqslant j \\leqslant n}\\left(1-x_{j}\\right) \\cdot(1+1)}+\\frac{f(-1)}{\\prod_{1 \\leqslant j \\leqslant n}\\left(-1-x_{j}\\right) \\cdot(-1-1)} \\\\ & =-G\\left(x_{1}, \\ldots, x_{n}\\right)+\\frac{1}{2}+\\frac{(-1)^{n+1}}{2} \\end{aligned} $$ Comment. The main difficulty is to think of including the two extra nodes $\\pm 1$ and evaluating the coefficient $t^{n+1}$ in $f$ when $n+1$ is higher than the degree of $f$. It is possible to solve the problem using Lagrange interpolation on the nodes $x_{1}, \\ldots, x_{n}$, but the definition of the polynomial being interpolated should depend on the parity of $n$. For $n$ even, consider the polynomial $$ P(x)=\\prod_{i}\\left(1-x x_{i}\\right)-\\prod_{i}\\left(x-x_{i}\\right) $$ Lagrange interpolation shows that $G$ is the coefficient of $x^{n-1}$ in the polynomial $P(x) \/\\left(1-x^{2}\\right)$, i.e. 0 . For $n$ odd, consider the polynomial $$ P(x)=\\prod_{i}\\left(1-x x_{i}\\right)-x \\prod_{i}\\left(x-x_{i}\\right) $$ Now $G$ is the coefficient of $x^{n-1}$ in $P(x) \/\\left(1-x^{2}\\right)$, which is 1 . Solution 2 (using symmetries). Observe that $G$ is symmetric in the variables $x_{1}, \\ldots, x_{n}$. Define $V=\\prod_{i0$. By Step 2, each of its monomials $\\mu x^{a} y^{b} z^{c}$ of the highest total degree satisfies $a, b \\geqslant c$. Applying other weak symmetries, we obtain $a, c \\geqslant b$ and $b, c \\geqslant a$; therefore, $P$ has a unique leading monomial of the form $\\mu(x y z)^{c}$. The polynomial $P_{0}(x, y, z)=P(x, y, z)-\\mu\\left(x y z-x^{2}-y^{2}-z^{2}\\right)^{c}$ has smaller total degree. Since $P_{0}$ is symmetric, it is representable as a polynomial function of $x y z-x^{2}-y^{2}-z^{2}$. Then $P$ is also of this form, completing the inductive step. Comment. We could alternatively carry out Step 1 by an induction on $n=\\operatorname{deg}_{z} P$, in a manner similar to Step 3. If $n=0$, the statement holds. Assume that $n>0$ and check the leading component of $P$ with respect to $z$ : $$ P(x, y, z)=Q_{n}(x, y) z^{n}+R(x, y, z) $$ where $\\operatorname{deg}_{z} Rv(x)$ and $v(x) \\geqslant v(y)$. Hence this $f$ satisfies the functional equation and 0 is an $f$-rare integer. Comment 3. In fact, if $v$ is an $f$-rare integer for an $f$ satisfying the functional equation, then its fibre $X_{v}=\\{v\\}$ must be a singleton. We may assume without loss of generality that $v=0$. We've already seen in Solution 1 that 0 is either the greatest or least element of $X_{0}$; replacing $f$ with the function $x \\mapsto-f(-x)$ if necessary, we may assume that 0 is the least element of $X_{0}$. We write $b$ for the largest element of $X_{0}$, supposing for contradiction that $b>0$, and write $N=(2 b)$ !. It now follows from (*) that we have $$ f(f(N b)+b)=f(f(0)+b)=f(b)=0 $$ from which we see that $f(N b)+b \\in X_{0} \\subseteq[0, b]$. It follows that $f(N b) \\in[-b, 0)$, since by construction $N b \\notin X_{v}$. Now it follows that $(f(N b)-0) \\cdot(f(N b)-b)$ is a divisor of $N$, so from ( $\\dagger$ ) we see that $f(N b)=f(0)=0$. This yields the desired contradiction.","tier":0} +{"problem_type":"Combinatorics","problem_label":"C1","problem":"The infinite sequence $a_{0}, a_{1}, a_{2}, \\ldots$ of (not necessarily different) integers has the following properties: $0 \\leqslant a_{i} \\leqslant i$ for all integers $i \\geqslant 0$, and $$ \\binom{k}{a_{0}}+\\binom{k}{a_{1}}+\\cdots+\\binom{k}{a_{k}}=2^{k} $$ for all integers $k \\geqslant 0$. Prove that all integers $N \\geqslant 0$ occur in the sequence (that is, for all $N \\geqslant 0$, there exists $i \\geqslant 0$ with $\\left.a_{i}=N\\right)$. (Netherlands)","solution":"We prove by induction on $k$ that every initial segment of the sequence, $a_{0}, a_{1}, \\ldots, a_{k}$, consists of the following elements (counted with multiplicity, and not necessarily in order), for some $\\ell \\geqslant 0$ with $2 \\ell \\leqslant k+1$ : $$ 0,1, \\ldots, \\ell-1, \\quad 0,1, \\ldots, k-\\ell $$ For $k=0$ we have $a_{0}=0$, which is of this form. Now suppose that for $k=m$ the elements $a_{0}, a_{1}, \\ldots, a_{m}$ are $0,0,1,1,2,2, \\ldots, \\ell-1, \\ell-1, \\ell, \\ell+1, \\ldots, m-\\ell-1, m-\\ell$ for some $\\ell$ with $0 \\leqslant 2 \\ell \\leqslant m+1$. It is given that $$ \\binom{m+1}{a_{0}}+\\binom{m+1}{a_{1}}+\\cdots+\\binom{m+1}{a_{m}}+\\binom{m+1}{a_{m+1}}=2^{m+1} $$ which becomes $$ \\begin{aligned} \\left(\\binom{m+1}{0}+\\binom{m+1}{1}\\right. & \\left.+\\cdots+\\binom{m+1}{\\ell-1}\\right) \\\\ & +\\left(\\binom{m+1}{0}+\\binom{m+1}{1}+\\cdots+\\binom{m+1}{m-\\ell}\\right)+\\binom{m+1}{a_{m+1}}=2^{m+1} \\end{aligned} $$ or, using $\\binom{m+1}{i}=\\binom{m+1}{m+1-i}$, that $$ \\begin{aligned} \\left(\\binom{m+1}{0}+\\binom{m+1}{1}\\right. & \\left.+\\cdots+\\binom{m+1}{\\ell-1}\\right) \\\\ & +\\left(\\binom{m+1}{m+1}+\\binom{m+1}{m}+\\cdots+\\binom{m+1}{\\ell+1}\\right)+\\binom{m+1}{a_{m+1}}=2^{m+1} \\end{aligned} $$ On the other hand, it is well known that $$ \\binom{m+1}{0}+\\binom{m+1}{1}+\\cdots+\\binom{m+1}{m+1}=2^{m+1} $$ and so, by subtracting, we get $$ \\binom{m+1}{a_{m+1}}=\\binom{m+1}{\\ell} $$ From this, using the fact that the binomial coefficients $\\binom{m+1}{i}$ are increasing for $i \\leqslant \\frac{m+1}{2}$ and decreasing for $i \\geqslant \\frac{m+1}{2}$, we conclude that either $a_{m+1}=\\ell$ or $a_{m+1}=m+1-\\ell$. In either case, $a_{0}, a_{1}, \\ldots, a_{m+1}$ is again of the claimed form, which concludes the induction. As a result of this description, any integer $N \\geqslant 0$ appears as a term of the sequence $a_{i}$ for some $0 \\leqslant i \\leqslant 2 N$.","tier":0} +{"problem_type":"Combinatorics","problem_label":"C3","problem":"Let $n$ be a positive integer. Harry has $n$ coins lined up on his desk, each showing heads or tails. He repeatedly does the following operation: if there are $k$ coins showing heads and $k>0$, then he flips the $k^{\\text {th }}$ coin over; otherwise he stops the process. (For example, the process starting with THT would be THT $\\rightarrow H H T \\rightarrow H T T \\rightarrow T T T$, which takes three steps.) Letting $C$ denote the initial configuration (a sequence of $n H$ 's and $T$ 's), write $\\ell(C)$ for the number of steps needed before all coins show $T$. Show that this number $\\ell(C)$ is finite, and determine its average value over all $2^{n}$ possible initial configurations $C$. Answer: The average is $\\frac{1}{4} n(n+1)$. Common remarks. Throughout all these solutions, we let $E(n)$ denote the desired average value.","solution":"We represent the problem using a directed graph $G_{n}$ whose vertices are the length- $n$ strings of $H$ 's and $T$ 's. The graph features an edge from each string to its successor (except for $T T \\cdots T T$, which has no successor). We will also write $\\bar{H}=T$ and $\\bar{T}=H$. The graph $G_{0}$ consists of a single vertex: the empty string. The main claim is that $G_{n}$ can be described explicitly in terms of $G_{n-1}$ : - We take two copies, $X$ and $Y$, of $G_{n-1}$. - In $X$, we take each string of $n-1$ coins and just append a $T$ to it. In symbols, we replace $s_{1} \\cdots s_{n-1}$ with $s_{1} \\cdots s_{n-1} T$. - In $Y$, we take each string of $n-1$ coins, flip every coin, reverse the order, and append an $H$ to it. In symbols, we replace $s_{1} \\cdots s_{n-1}$ with $\\bar{s}_{n-1} \\bar{s}_{n-2} \\cdots \\bar{s}_{1} H$. - Finally, we add one new edge from $Y$ to $X$, namely $H H \\cdots H H H \\rightarrow H H \\cdots H H T$. We depict $G_{4}$ below, in a way which indicates this recursive construction: ![](https:\/\/cdn.mathpix.com\/cropped\/2024_11_18_9ee8c2ac2c2f5002ff14g-034.jpg?height=515&width=1135&top_left_y=1850&top_left_x=466) We prove the claim inductively. Firstly, $X$ is correct as a subgraph of $G_{n}$, as the operation on coins is unchanged by an extra $T$ at the end: if $s_{1} \\cdots s_{n-1}$ is sent to $t_{1} \\cdots t_{n-1}$, then $s_{1} \\cdots s_{n-1} T$ is sent to $t_{1} \\cdots t_{n-1} T$. Next, $Y$ is also correct as a subgraph of $G_{n}$, as if $s_{1} \\cdots s_{n-1}$ has $k$ occurrences of $H$, then $\\bar{s}_{n-1} \\cdots \\bar{s}_{1} H$ has $(n-1-k)+1=n-k$ occurrences of $H$, and thus (provided that $k>0$ ), if $s_{1} \\cdots s_{n-1}$ is sent to $t_{1} \\cdots t_{n-1}$, then $\\bar{s}_{n-1} \\cdots \\bar{s}_{1} H$ is sent to $\\bar{t}_{n-1} \\cdots \\bar{t}_{1} H$. Finally, the one edge from $Y$ to $X$ is correct, as the operation does send $H H \\cdots H H H$ to HH $\\cdots$ HHT. To finish, note that the sequences in $X$ take an average of $E(n-1)$ steps to terminate, whereas the sequences in $Y$ take an average of $E(n-1)$ steps to reach $H H \\cdots H$ and then an additional $n$ steps to terminate. Therefore, we have $$ E(n)=\\frac{1}{2}(E(n-1)+(E(n-1)+n))=E(n-1)+\\frac{n}{2} $$ We have $E(0)=0$ from our description of $G_{0}$. Thus, by induction, we have $E(n)=\\frac{1}{2}(1+\\cdots+$ $n)=\\frac{1}{4} n(n+1)$, which in particular is finite.","tier":0} +{"problem_type":"Combinatorics","problem_label":"C3","problem":"Let $n$ be a positive integer. Harry has $n$ coins lined up on his desk, each showing heads or tails. He repeatedly does the following operation: if there are $k$ coins showing heads and $k>0$, then he flips the $k^{\\text {th }}$ coin over; otherwise he stops the process. (For example, the process starting with THT would be THT $\\rightarrow H H T \\rightarrow H T T \\rightarrow T T T$, which takes three steps.) Letting $C$ denote the initial configuration (a sequence of $n H$ 's and $T$ 's), write $\\ell(C)$ for the number of steps needed before all coins show $T$. Show that this number $\\ell(C)$ is finite, and determine its average value over all $2^{n}$ possible initial configurations $C$. Answer: The average is $\\frac{1}{4} n(n+1)$. Common remarks. Throughout all these solutions, we let $E(n)$ denote the desired average value.","solution":"We consider what happens with configurations depending on the coins they start and end with. - If a configuration starts with $H$, the last $n-1$ coins follow the given rules, as if they were all the coins, until they are all $T$, then the first coin is turned over. - If a configuration ends with $T$, the last coin will never be turned over, and the first $n-1$ coins follow the given rules, as if they were all the coins. - If a configuration starts with $T$ and ends with $H$, the middle $n-2$ coins follow the given rules, as if they were all the coins, until they are all $T$. After that, there are $2 n-1$ more steps: first coins $1,2, \\ldots, n-1$ are turned over in that order, then coins $n, n-1, \\ldots, 1$ are turned over in that order. As this covers all configurations, and the number of steps is clearly finite for 0 or 1 coins, it follows by induction on $n$ that the number of steps is always finite. We define $E_{A B}(n)$, where $A$ and $B$ are each one of $H, T$ or $*$, to be the average number of steps over configurations of length $n$ restricted to those that start with $A$, if $A$ is not $*$, and that end with $B$, if $B$ is not * (so * represents \"either $H$ or $T$ \"). The above observations tell us that, for $n \\geqslant 2$ : - $E_{H *}(n)=E(n-1)+1$. - $E_{* T}(n)=E(n-1)$. - $E_{H T}(n)=E(n-2)+1$ (by using both the observations for $H *$ and for $\\left.* T\\right)$. - $E_{T H}(n)=E(n-2)+2 n-1$. Now $E_{H *}(n)=\\frac{1}{2}\\left(E_{H H}(n)+E_{H T}(n)\\right)$, so $E_{H H}(n)=2 E(n-1)-E(n-2)+1$. Similarly, $E_{T T}(n)=2 E(n-1)-E(n-2)-1$. So $$ E(n)=\\frac{1}{4}\\left(E_{H T}(n)+E_{H H}(n)+E_{T T}(n)+E_{T H}(n)\\right)=E(n-1)+\\frac{n}{2} $$ We have $E(0)=0$ and $E(1)=\\frac{1}{2}$, so by induction on $n$ we have $E(n)=\\frac{1}{4} n(n+1)$.","tier":0} +{"problem_type":"Combinatorics","problem_label":"C3","problem":"Let $n$ be a positive integer. Harry has $n$ coins lined up on his desk, each showing heads or tails. He repeatedly does the following operation: if there are $k$ coins showing heads and $k>0$, then he flips the $k^{\\text {th }}$ coin over; otherwise he stops the process. (For example, the process starting with THT would be THT $\\rightarrow H H T \\rightarrow H T T \\rightarrow T T T$, which takes three steps.) Letting $C$ denote the initial configuration (a sequence of $n H$ 's and $T$ 's), write $\\ell(C)$ for the number of steps needed before all coins show $T$. Show that this number $\\ell(C)$ is finite, and determine its average value over all $2^{n}$ possible initial configurations $C$. Answer: The average is $\\frac{1}{4} n(n+1)$. Common remarks. Throughout all these solutions, we let $E(n)$ denote the desired average value.","solution":"Let $H_{i}$ be the number of heads in positions 1 to $i$ inclusive (so $H_{n}$ is the total number of heads), and let $I_{i}$ be 1 if the $i^{\\text {th }}$ coin is a head, 0 otherwise. Consider the function $$ t(i)=I_{i}+2\\left(\\min \\left\\{i, H_{n}\\right\\}-H_{i}\\right) $$ We claim that $t(i)$ is the total number of times coin $i$ is turned over (which implies that the process terminates). Certainly $t(i)=0$ when all coins are tails, and $t(i)$ is always a nonnegative integer, so it suffices to show that when the $k^{\\text {th }}$ coin is turned over (where $k=H_{n}$ ), $t(k)$ goes down by 1 and all the other $t(i)$ are unchanged. We show this by splitting into cases: - If $ik, \\min \\left\\{i, H_{n}\\right\\}=H_{n}$ both before and after the coin flip, and both $H_{n}$ and $H_{i}$ change by the same amount, so $t(i)$ is unchanged. - If $i=k$ and the coin is heads, $I_{i}$ goes down by 1 , as do both $\\min \\left\\{i, H_{n}\\right\\}=H_{n}$ and $H_{i}$; so $t(i)$ goes down by 1 . - If $i=k$ and the coin is tails, $I_{i}$ goes up by $1, \\min \\left\\{i, H_{n}\\right\\}=i$ is unchanged and $H_{i}$ goes up by 1 ; so $t(i)$ goes down by 1 . We now need to compute the average value of $$ \\sum_{i=1}^{n} t(i)=\\sum_{i=1}^{n} I_{i}+2 \\sum_{i=1}^{n} \\min \\left\\{i, H_{n}\\right\\}-2 \\sum_{i=1}^{n} H_{i} . $$ The average value of the first term is $\\frac{1}{2} n$, and that of the third term is $-\\frac{1}{2} n(n+1)$. To compute the second term, we sum over choices for the total number of heads, and then over the possible values of $i$, getting $$ 2^{1-n} \\sum_{j=0}^{n}\\binom{n}{j} \\sum_{i=1}^{n} \\min \\{i, j\\}=2^{1-n} \\sum_{j=0}^{n}\\binom{n}{j}\\left(n j-\\binom{j}{2}\\right) . $$ Now, in terms of trinomial coefficients, $$ \\sum_{j=0}^{n} j\\binom{n}{j}=\\sum_{j=1}^{n}\\binom{n}{n-j, j-1,1}=n \\sum_{j=0}^{n-1}\\binom{n-1}{j}=2^{n-1} n $$ and $$ \\sum_{j=0}^{n}\\binom{j}{2}\\binom{n}{j}=\\sum_{j=2}^{n}\\binom{n}{n-j, j-2,2}=\\binom{n}{2} \\sum_{j=0}^{n-2}\\binom{n-2}{j}=2^{n-2}\\binom{n}{2} $$ So the second term above is $$ 2^{1-n}\\left(2^{n-1} n^{2}-2^{n-2}\\binom{n}{2}\\right)=n^{2}-\\frac{n(n-1)}{4} $$ and the required average is $$ E(n)=\\frac{1}{2} n+n^{2}-\\frac{n(n-1)}{4}-\\frac{1}{2} n(n+1)=\\frac{n(n+1)}{4} . $$","tier":0} +{"problem_type":"Combinatorics","problem_label":"C3","problem":"Let $n$ be a positive integer. Harry has $n$ coins lined up on his desk, each showing heads or tails. He repeatedly does the following operation: if there are $k$ coins showing heads and $k>0$, then he flips the $k^{\\text {th }}$ coin over; otherwise he stops the process. (For example, the process starting with THT would be THT $\\rightarrow H H T \\rightarrow H T T \\rightarrow T T T$, which takes three steps.) Letting $C$ denote the initial configuration (a sequence of $n H$ 's and $T$ 's), write $\\ell(C)$ for the number of steps needed before all coins show $T$. Show that this number $\\ell(C)$ is finite, and determine its average value over all $2^{n}$ possible initial configurations $C$. Answer: The average is $\\frac{1}{4} n(n+1)$. Common remarks. Throughout all these solutions, we let $E(n)$ denote the desired average value.","solution":"Harry has built a Turing machine to flip the coins for him. The machine is initially positioned at the $k^{\\text {th }}$ coin, where there are $k$ heads (and the position before the first coin is considered to be the $0^{\\text {th }}$ coin). The machine then moves according to the following rules, stopping when it reaches the position before the first coin: if the coin at its current position is $H$, it flips the coin and moves to the previous coin, while if the coin at its current position is $T$, it flips the coin and moves to the next position. Consider the maximal sequences of consecutive moves in the same direction. Suppose the machine has $a$ consecutive moves to the next coin, before a move to the previous coin. After those $a$ moves, the $a$ coins flipped in those moves are all heads, as is the coin the machine is now at, so at least the next $a+1$ moves will all be moves to the previous coin. Similarly, $a$ consecutive moves to the previous coin are followed by at least $a+1$ consecutive moves to the next coin. There cannot be more than $n$ consecutive moves in the same direction, so this proves that the process terminates (with a move from the first coin to the position before the first coin). Thus we have a (possibly empty) sequence $a_{1}<\\cdotsj$, meaning that, as we follow the moves backwards, each coin is always in the correct state when flipped to result in a move in the required direction. (Alternatively, since there are $2^{n}$ possible configurations of coins and $2^{n}$ possible such ascending sequences, the fact that the sequence of moves determines at most one configuration of coins, and thus that there is an injection from configurations of coins to such ascending sequences, is sufficient for it to be a bijection, without needing to show that coins are in the right state as we move backwards.)","tier":0} +{"problem_type":"Combinatorics","problem_label":"C3","problem":"Let $n$ be a positive integer. Harry has $n$ coins lined up on his desk, each showing heads or tails. He repeatedly does the following operation: if there are $k$ coins showing heads and $k>0$, then he flips the $k^{\\text {th }}$ coin over; otherwise he stops the process. (For example, the process starting with THT would be THT $\\rightarrow H H T \\rightarrow H T T \\rightarrow T T T$, which takes three steps.) Letting $C$ denote the initial configuration (a sequence of $n H$ 's and $T$ 's), write $\\ell(C)$ for the number of steps needed before all coins show $T$. Show that this number $\\ell(C)$ is finite, and determine its average value over all $2^{n}$ possible initial configurations $C$. Answer: The average is $\\frac{1}{4} n(n+1)$. Common remarks. Throughout all these solutions, we let $E(n)$ denote the desired average value.","solution":"We explicitly describe what happens with an arbitrary sequence $C$ of $n$ coins. Suppose that $C$ contain $k$ heads at positions $1 \\leqslant c_{1}1$ be an integer. Suppose we are given $2 n$ points in a plane such that no three of them are collinear. The points are to be labelled $A_{1}, A_{2}, \\ldots, A_{2 n}$ in some order. We then consider the $2 n$ angles $\\angle A_{1} A_{2} A_{3}, \\angle A_{2} A_{3} A_{4}, \\ldots, \\angle A_{2 n-2} A_{2 n-1} A_{2 n}, \\angle A_{2 n-1} A_{2 n} A_{1}$, $\\angle A_{2 n} A_{1} A_{2}$. We measure each angle in the way that gives the smallest positive value (i.e. between $0^{\\circ}$ and $180^{\\circ}$ ). Prove that there exists an ordering of the given points such that the resulting $2 n$ angles can be separated into two groups with the sum of one group of angles equal to the sum of the other group. Comment. The first three solutions all use the same construction involving a line separating the points into groups of $n$ points each, but give different proofs that this construction works. Although Solution 1 is very short, the Problem Selection Committee does not believe any of the solutions is easy to find and thus rates this as a problem of medium difficulty.","solution":"Let $\\ell$ be a line separating the points into two groups ( $L$ and $R$ ) with $n$ points in each. Label the points $A_{1}, A_{2}, \\ldots, A_{2 n}$ so that $L=\\left\\{A_{1}, A_{3}, \\ldots, A_{2 n-1}\\right\\}$. We claim that this labelling works. Take a line $s=A_{2 n} A_{1}$. (a) Rotate $s$ around $A_{1}$ until it passes through $A_{2}$; the rotation is performed in a direction such that $s$ is never parallel to $\\ell$. (b) Then rotate the new $s$ around $A_{2}$ until it passes through $A_{3}$ in a similar manner. (c) Perform $2 n-2$ more such steps, after which $s$ returns to its initial position. The total (directed) rotation angle $\\Theta$ of $s$ is clearly a multiple of $180^{\\circ}$. On the other hand, $s$ was never parallel to $\\ell$, which is possible only if $\\Theta=0$. Now it remains to partition all the $2 n$ angles into those where $s$ is rotated anticlockwise, and the others.","tier":0} +{"problem_type":"Combinatorics","problem_label":"C6","problem":"Let $n>1$ be an integer. Suppose we are given $2 n$ points in a plane such that no three of them are collinear. The points are to be labelled $A_{1}, A_{2}, \\ldots, A_{2 n}$ in some order. We then consider the $2 n$ angles $\\angle A_{1} A_{2} A_{3}, \\angle A_{2} A_{3} A_{4}, \\ldots, \\angle A_{2 n-2} A_{2 n-1} A_{2 n}, \\angle A_{2 n-1} A_{2 n} A_{1}$, $\\angle A_{2 n} A_{1} A_{2}$. We measure each angle in the way that gives the smallest positive value (i.e. between $0^{\\circ}$ and $180^{\\circ}$ ). Prove that there exists an ordering of the given points such that the resulting $2 n$ angles can be separated into two groups with the sum of one group of angles equal to the sum of the other group. Comment. The first three solutions all use the same construction involving a line separating the points into groups of $n$ points each, but give different proofs that this construction works. Although Solution 1 is very short, the Problem Selection Committee does not believe any of the solutions is easy to find and thus rates this as a problem of medium difficulty.","solution":"When tracing a cyclic path through the $A_{i}$ in order, with straight line segments between consecutive points, let $\\theta_{i}$ be the exterior angle at $A_{i}$, with a sign convention that it is positive if the path turns left and negative if the path turns right. Then $\\sum_{i=1}^{2 n} \\theta_{i}=360 k^{\\circ}$ for some integer $k$. Let $\\phi_{i}=\\angle A_{i-1} A_{i} A_{i+1}($ indices $\\bmod 2 n)$, defined as in the problem; thus $\\phi_{i}=180^{\\circ}-\\left|\\theta_{i}\\right|$. Let $L$ be the set of $i$ for which the path turns left at $A_{i}$ and let $R$ be the set for which it turns right. Then $S=\\sum_{i \\in L} \\phi_{i}-\\sum_{i \\in R} \\phi_{i}=(180(|L|-|R|)-360 k)^{\\circ}$, which is a multiple of $360^{\\circ}$ since the number of points is even. We will show that the points can be labelled such that $S=0$, in which case $L$ and $R$ satisfy the required condition of the problem. Note that the value of $S$ is defined for a slightly larger class of configurations: it is OK for two points to coincide, as long as they are not consecutive, and OK for three points to be collinear, as long as $A_{i}, A_{i+1}$ and $A_{i+2}$ do not appear on a line in that order. In what follows it will be convenient, although not strictly necessary, to consider such configurations. Consider how $S$ changes if a single one of the $A_{i}$ is moved along some straight-line path (not passing through any $A_{j}$ and not lying on any line $A_{j} A_{k}$, but possibly crossing such lines). Because $S$ is a multiple of $360^{\\circ}$, and the angles change continuously, $S$ can only change when a point moves between $R$ and $L$. Furthermore, if $\\phi_{j}=0$ when $A_{j}$ moves between $R$ and $L, S$ is unchanged; it only changes if $\\phi_{j}=180^{\\circ}$ when $A_{j}$ moves between those sets. For any starting choice of points, we will now construct a new configuration, with labels such that $S=0$, that can be perturbed into the original one without any $\\phi_{i}$ passing through $180^{\\circ}$, so that $S=0$ for the original configuration with those labels as well. Take some line such that there are $n$ points on each side of that line. The new configuration has $n$ copies of a single point on each side of the line, and a path that alternates between sides of the line; all angles are 0 , so this configuration has $S=0$. Perturbing the points into their original positions, while keeping each point on its side of the line, no angle $\\phi_{i}$ can pass through $180^{\\circ}$, because no straight line can go from one side of the line to the other and back. So the perturbation process leaves $S=0$. Comment. More complicated variants of this solution are also possible; for example, a path defined using four quadrants of the plane rather than just two half-planes.","tier":0} +{"problem_type":"Combinatorics","problem_label":"C6","problem":"Let $n>1$ be an integer. Suppose we are given $2 n$ points in a plane such that no three of them are collinear. The points are to be labelled $A_{1}, A_{2}, \\ldots, A_{2 n}$ in some order. We then consider the $2 n$ angles $\\angle A_{1} A_{2} A_{3}, \\angle A_{2} A_{3} A_{4}, \\ldots, \\angle A_{2 n-2} A_{2 n-1} A_{2 n}, \\angle A_{2 n-1} A_{2 n} A_{1}$, $\\angle A_{2 n} A_{1} A_{2}$. We measure each angle in the way that gives the smallest positive value (i.e. between $0^{\\circ}$ and $180^{\\circ}$ ). Prove that there exists an ordering of the given points such that the resulting $2 n$ angles can be separated into two groups with the sum of one group of angles equal to the sum of the other group. Comment. The first three solutions all use the same construction involving a line separating the points into groups of $n$ points each, but give different proofs that this construction works. Although Solution 1 is very short, the Problem Selection Committee does not believe any of the solutions is easy to find and thus rates this as a problem of medium difficulty.","solution":"First, let $\\ell$ be a line in the plane such that there are $n$ points on one side and the other $n$ points on the other side. For convenience, assume $\\ell$ is horizontal (otherwise, we can rotate the plane). Then we can use the terms \"above\", \"below\", \"left\" and \"right\" in the usual way. We denote the $n$ points above the line in an arbitrary order as $P_{1}, P_{2}, \\ldots, P_{n}$, and the $n$ points below the line as $Q_{1}, Q_{2}, \\ldots, Q_{n}$. If we connect $P_{i}$ and $Q_{j}$ with a line segment, the line segment will intersect with the line $\\ell$. Denote the intersection as $I_{i j}$. If $P_{i}$ is connected to $Q_{j}$ and $Q_{k}$, where $jk$, then the sign of $\\angle Q_{j} P_{i} Q_{k}$ is taken to be the same as for $\\angle Q_{k} P_{i} Q_{j}$. Similarly, we can define the sign of $\\angle P_{j} Q_{i} P_{k}$ with $j1$ be an integer. Suppose we are given $2 n$ points in a plane such that no three of them are collinear. The points are to be labelled $A_{1}, A_{2}, \\ldots, A_{2 n}$ in some order. We then consider the $2 n$ angles $\\angle A_{1} A_{2} A_{3}, \\angle A_{2} A_{3} A_{4}, \\ldots, \\angle A_{2 n-2} A_{2 n-1} A_{2 n}, \\angle A_{2 n-1} A_{2 n} A_{1}$, $\\angle A_{2 n} A_{1} A_{2}$. We measure each angle in the way that gives the smallest positive value (i.e. between $0^{\\circ}$ and $180^{\\circ}$ ). Prove that there exists an ordering of the given points such that the resulting $2 n$ angles can be separated into two groups with the sum of one group of angles equal to the sum of the other group. Comment. The first three solutions all use the same construction involving a line separating the points into groups of $n$ points each, but give different proofs that this construction works. Although Solution 1 is very short, the Problem Selection Committee does not believe any of the solutions is easy to find and thus rates this as a problem of medium difficulty.","solution":"We shall think instead of the problem as asking us to assign a weight $\\pm 1$ to each angle, such that the weighted sum of all the angles is zero. Given an ordering $A_{1}, \\ldots, A_{2 n}$ of the points, we shall assign weights according to the following recipe: walk in order from point to point, and assign the left turns +1 and the right turns -1 . This is the same weighting as in Solution 3, and as in that solution, the weighted sum is a multiple of $360^{\\circ}$. We now aim to show the following: Lemma. Transposing any two consecutive points in the ordering changes the weighted sum by $\\pm 360^{\\circ}$ or 0 . Knowing that, we can conclude quickly: if the ordering $A_{1}, \\ldots, A_{2 n}$ has weighted angle sum $360 k^{\\circ}$, then the ordering $A_{2 n}, \\ldots, A_{1}$ has weighted angle sum $-360 k^{\\circ}$ (since the angles are the same, but left turns and right turns are exchanged). We can reverse the ordering of $A_{1}$, $\\ldots, A_{2 n}$ by a sequence of transpositions of consecutive points, and in doing so the weighted angle sum must become zero somewhere along the way. We now prove that lemma: Proof. Transposing two points amounts to taking a section $A_{k} A_{k+1} A_{k+2} A_{k+3}$ as depicted, reversing the central line segment $A_{k+1} A_{k+2}$, and replacing its two neighbours with the dotted lines. ![](https:\/\/cdn.mathpix.com\/cropped\/2024_11_18_9ee8c2ac2c2f5002ff14g-048.jpg?height=424&width=1357&top_left_y=1156&top_left_x=354) Figure 1: Transposing two consecutive vertices: before (left) and afterwards (right) In each triangle, we alter the sum by $\\pm 180^{\\circ}$. Indeed, using (anticlockwise) directed angles modulo $360^{\\circ}$, we either add or subtract all three angles of each triangle. Hence both triangles together alter the sum by $\\pm 180 \\pm 180^{\\circ}$, which is $\\pm 360^{\\circ}$ or 0 .","tier":0} +{"problem_type":"Combinatorics","problem_label":"C7","problem":"There are 60 empty boxes $B_{1}, \\ldots, B_{60}$ in a row on a table and an unlimited supply of pebbles. Given a positive integer $n$, Alice and Bob play the following game. In the first round, Alice takes $n$ pebbles and distributes them into the 60 boxes as she wishes. Each subsequent round consists of two steps: (a) Bob chooses an integer $k$ with $1 \\leqslant k \\leqslant 59$ and splits the boxes into the two groups $B_{1}, \\ldots, B_{k}$ and $B_{k+1}, \\ldots, B_{60}$. (b) Alice picks one of these two groups, adds one pebble to each box in that group, and removes one pebble from each box in the other group. Bob wins if, at the end of any round, some box contains no pebbles. Find the smallest $n$ such that Alice can prevent Bob from winning. (Czech Republic) Answer: $n=960$. In general, if there are $N>1$ boxes, the answer is $n=\\left\\lfloor\\frac{N}{2}+1\\right\\rfloor\\left\\lceil\\frac{N}{2}+1\\right\\rceil-1$. Common remarks. We present solutions for the general case of $N>1$ boxes, and write $M=\\left\\lfloor\\frac{N}{2}+1\\right\\rfloor\\left\\lceil\\frac{N}{2}+1\\right\\rceil-1$ for the claimed answer. For $1 \\leqslant kk$. We may then assume, without loss of generality, that Alice again picks the left group. Proof. Suppose Alice picks the right group in the second round. Then the combined effect of the two rounds is that each of the boxes $B_{k+1}, \\ldots, B_{\\ell}$ lost two pebbles (and the other boxes are unchanged). Hence this configuration is strictly dominated by that before the first round, and it suffices to consider only Alice's other response. Solution 1 (Alice). Alice initially distributes pebbles according to $V_{\\left\\lceil\\frac{N}{2}\\right\\rceil}$. Suppose the current configuration of pebbles dominates $V_{i}$. If Bob makes a $k$-move with $k \\geqslant i$ then Alice picks the left group, which results in a configuration that dominates $V_{i+1}$. Likewise, if Bob makes a $k$-move with $k4$, in the style of IMO 2014 Problem 6. (Thailand)","solution":"We will show Alice needs to ask at most $4 n-7$ questions. Her strategy has the following phases. In what follows, $S$ is the set of towns that Alice, so far, does not know to have more than one outgoing road (so initially $|S|=n$ ). Phase 1. Alice chooses any two towns, say $A$ and $B$. Without loss of generality, suppose that the King of Hearts' answer is that the road goes from $A$ to $B$. At the end of this phase, Alice has asked 1 question. Phase 2. During this phase there is a single (variable) town $T$ that is known to have at least one incoming road but not yet known to have any outgoing roads. Initially, $T$ is $B$. Alice does the following $n-2$ times: she picks a town $X$ she has not asked about before, and asks the direction of the road between $T$ and $X$. If it is from $X$ to $T, T$ is unchanged; if it is from $T$ to $X, X$ becomes the new choice of town $T$, as the previous $T$ is now known to have an outgoing road. At the end of this phase, Alice has asked a total of $n-1$ questions. The final town $T$ is not yet known to have any outgoing roads, while every other town has exactly one outgoing road known. The undirected graph of roads whose directions are known is a tree. Phase 3. During this phase, Alice asks about the directions of all roads between $T$ and another town she has not previously asked about, stopping if she finds two outgoing roads from $T$. This phase involves at most $n-2$ questions. If she does not find two outgoing roads from $T$, she has answered her original question with at most $2 n-3 \\leqslant 4 n-7$ questions, so in what follows we suppose that she does find two outgoing roads, asking a total of $k$ questions in this phase, where $2 \\leqslant k \\leqslant n-2$ (and thus $n \\geqslant 4$ for what follows). For every question where the road goes towards $T$, the town at the other end is removed from $S$ (as it already had one outgoing road known), while the last question resulted in $T$ being removed from $S$. So at the end of this phase, $|S|=n-k+1$, while a total of $n+k-1$ questions have been asked. Furthermore, the undirected graph of roads within $S$ whose directions are known contains no cycles (as $T$ is no longer a member of $S$, all questions asked in this phase involved $T$ and the graph was a tree before this phase started). Every town in $S$ has exactly one outgoing road known (not necessarily to another town in $S$ ). Phase 4. During this phase, Alice repeatedly picks any pair of towns in $S$ for which she does not know the direction of the road between them. Because every town in $S$ has exactly one outgoing road known, this always results in the removal of one of those two towns from $S$. Because there are no cycles in the graph of roads of known direction within $S$, this can continue until there are at most 2 towns left in $S$. If it ends with $t$ towns left, $n-k+1-t$ questions were asked in this phase, so a total of $2 n-t$ questions have been asked. Phase 5. During this phase, Alice asks about all the roads from the remaining towns in $S$ that she has not previously asked about. She has definitely already asked about any road between those towns (if $t=2$ ). She must also have asked in one of the first two phases about at least one other road involving one of those towns (as those phases resulted in a tree with $n>2$ vertices). So she asks at most $t(n-t)-1$ questions in this phase. At the end of this phase, Alice knows whether any town has at most one outgoing road. If $t=1$, at most $3 n-3 \\leqslant 4 n-7$ questions were needed in total, while if $t=2$, at most $4 n-7$ questions were needed in total. Comment 1. The version of this problem originally submitted asked only for an upper bound of $5 n$, which is much simpler to prove. The Problem Selection Committee preferred a version with an asymptotically optimal constant. In the following comment, we will show that the constant is optimal. Comment 2. We will show that Alice cannot always find out by asking at most $4 n-3\\left(\\log _{2} n\\right)-$ 15 questions, if $n \\geqslant 8$. To show this, we suppose the King of Hearts is choosing the directions as he goes along, only picking the direction of a road when Alice asks about it for the first time. We provide a strategy for the King of Hearts that ensures that, after the given number of questions, the map is still consistent both with the existence of a town with at most one outgoing road, and with the nonexistence of such a town. His strategy has the following phases. When describing how the King of Hearts' answer to a question is determined below, we always assume he is being asked about a road for the first time (otherwise, he just repeats his previous answer for that road). This strategy is described throughout in graph-theoretic terms (vertices and edges rather than towns and roads). Phase 1. In this phase, we consider the undirected graph formed by edges whose directions are known. The phase terminates when there are exactly 8 connected components whose undirected graphs are trees. The following invariant is maintained: in a component with $k$ vertices whose undirected graph is a tree, every vertex has at most $\\left[\\log _{2} k\\right\\rfloor$ edges into it. - If the King of Hearts is asked about an edge between two vertices in the same component, or about an edge between two components at least one of which is not a tree, he chooses any direction for that edge arbitrarily. - If he is asked about an edge between a vertex in component $A$ that has $a$ vertices and is a tree and a vertex in component $B$ that has $b$ vertices and is a tree, suppose without loss of generality that $a \\geqslant b$. He then chooses the edge to go from $A$ to $B$. In this case, the new number of edges into any vertex is at most $\\max \\left\\{\\left\\lfloor\\log _{2} a\\right\\rfloor,\\left\\lfloor\\log _{2} b\\right\\rfloor+1\\right\\} \\leqslant\\left\\lfloor\\log _{2}(a+b)\\right\\rfloor$. In all cases, the invariant is preserved, and the number of tree components either remains unchanged or goes down by 1. Assuming Alice does not repeat questions, the process must eventually terminate with 8 tree components, and at least $n-8$ questions having been asked. Note that each tree component contains at least one vertex with no outgoing edges. Colour one such vertex in each tree component red. Phase 2. Let $V_{1}, V_{2}$ and $V_{3}$ be the three of the red vertices whose components are smallest (so their components together have at most $\\left\\lfloor\\frac{3}{8} n\\right\\rfloor$ vertices, with each component having at most $\\left\\lfloor\\frac{3}{8} n-2\\right\\rfloor$ vertices). Let sets $C_{1}, C_{2}, \\ldots$ be the connected components after removing the $V_{j}$. By construction, there are no edges with known direction between $C_{i}$ and $C_{j}$ for $i \\neq j$, and there are at least five such components. If at any point during this phase, the King of Hearts is asked about an edge within one of the $C_{i}$, he chooses an arbitrary direction. If he is asked about an edge between $C_{i}$ and $C_{j}$ for $i \\neq j$, he answers so that all edges go from $C_{i}$ to $C_{i+1}$ and $C_{i+2}$, with indices taken modulo the number of components, and chooses arbitrarily for other pairs. This ensures that all vertices other than the $V_{j}$ will have more than one outgoing edge. For edges involving one of the $V_{j}$ he answers as follows, so as to remain consistent for as long as possible with both possibilities for whether one of those vertices has at most one outgoing edge. Note that as they were red vertices, they have no outgoing edges at the start of this phase. For edges between two of the $V_{j}$, he answers that the edges go from $V_{1}$ to $V_{2}$, from $V_{2}$ to $V_{3}$ and from $V_{3}$ to $V_{1}$. For edges between $V_{j}$ and some other vertex, he always answers that the edge goes into $V_{j}$, except for the last such edge for which he is asked the question for any given $V_{j}$, for which he answers that the edge goes out of $V_{j}$. Thus, as long as at least one of the $V_{j}$ has not had the question answered for all the vertices that are not among the $V_{j}$, his answers are still compatible both with all vertices having more than one outgoing edge, and with that $V_{j}$ having only one outgoing edge. At the start of this phase, each of the $V_{j}$ has at most $\\left\\lfloor\\log _{2}\\left\\lfloor\\frac{3}{8} n-2\\right\\rfloor\\right\\rfloor<\\left(\\log _{2} n\\right)-1$ incoming edges. Thus, Alice cannot determine whether some vertex has only one outgoing edge within $3(n-$ $\\left.3-\\left(\\left(\\log _{2} n\\right)-1\\right)\\right)-1$ questions in this phase; that is, $4 n-3\\left(\\log _{2} n\\right)-15$ questions total. Comment 3. We can also improve the upper bound slightly, to $4 n-2\\left(\\log _{2} n\\right)+1$. (We do not know where the precise minimum number of questions lies between $4 n-3\\left(\\log _{2} n\\right)+O(1)$ and $4 n-2\\left(\\log _{2} n\\right)+$ $O(1)$.) Suppose $n \\geqslant 5$ (otherwise no questions are required at all). To do this, we replace Phases 1 and 2 of the given solution with a different strategy that also results in a spanning tree where one vertex $V$ is not known to have any outgoing edges, and all other vertices have exactly one outgoing edge known, but where there is more control over the numbers of incoming edges. In Phases 3 and 4 we then take more care about the order in which pairs of towns are chosen, to ensure that each of the remaining towns has already had a question asked about at least $\\log _{2} n+O(1)$ edges. Define trees $T_{m}$ with $2^{m}$ vertices, exactly one of which (the root) has no outgoing edges and the rest of which have exactly one outgoing edge, as follows: $T_{0}$ is a single vertex, while $T_{m}$ is constructed by joining the roots of two copies of $T_{m-1}$ with an edge in either direction. If $n=2^{m}$ we can readily ask $n-1$ questions, resulting in a tree $T_{m}$ for the edges with known direction: first ask about $2^{m-1}$ disjoint pairs of vertices, then about $2^{m-2}$ disjoint pairs of the roots of the resulting $T_{1}$ trees, and so on. For the general case, where $n$ is not a power of 2 , after $k$ stages of this process we have $\\left\\lfloor n \/ 2^{k}\\right\\rfloor$ trees, each of which is like $T_{k}$ but may have some extra vertices (but, however, a unique root). If there are an even number of trees, then ask about pairs of their roots. If there are an odd number (greater than 1) of trees, when a single $T_{k}$ is left over, ask about its root together with that of one of the $T_{k+1}$ trees. Say $m=\\left\\lfloor\\log _{2} n\\right\\rfloor$. The result of that process is a single $T_{m}$ tree, possibly with some extra vertices but still a unique root $V$. That root has at least $m$ incoming edges, and we may list vertices $V_{0}$, $\\ldots, V_{m-1}$ with edges to $V$, such that, for all $0 \\leqslant i1$ we have $D\\left(x_{i}, x_{j}\\right)>d$, since $$ \\left|x_{i}-x_{j}\\right|=\\left|x_{i+1}-x_{i}\\right|+\\left|x_{j}-x_{i+1}\\right| \\geqslant 2^{d}+2^{d}=2^{d+1} . $$ Now choose the minimal $i \\leqslant t$ and the maximal $j \\geqslant t+1$ such that $D\\left(x_{i}, x_{i+1}\\right)=$ $D\\left(x_{i+1}, x_{i+2}\\right)=\\cdots=D\\left(x_{j-1}, x_{j}\\right)=d$. Let $E$ be the set of all the $x_{s}$ with even indices $i \\leqslant s \\leqslant j, O$ be the set of those with odd indices $i \\leqslant s \\leqslant j$, and $R$ be the rest of the elements (so that $\\mathcal{S}$ is the disjoint union of $E, O$ and $R$ ). Set $\\mathcal{S}_{O}=R \\cup O$ and $\\mathcal{S}_{E}=R \\cup E$; we have $\\left|\\mathcal{S}_{O}\\right|<|\\mathcal{S}|$ and $\\left|\\mathcal{S}_{E}\\right|<|\\mathcal{S}|$, so $w\\left(\\mathcal{S}_{O}\\right), w\\left(\\mathcal{S}_{E}\\right) \\leqslant 1$ by the inductive hypothesis. Clearly, $r_{\\mathcal{S}_{O}}(x) \\leqslant r_{\\mathcal{S}}(x)$ and $r_{\\mathcal{S}_{E}}(x) \\leqslant r_{\\mathcal{S}}(x)$ for any $x \\in R$, and thus $$ \\begin{aligned} \\sum_{x \\in R} 2^{-r_{\\mathcal{S}}(x)} & =\\frac{1}{2} \\sum_{x \\in R}\\left(2^{-r_{\\mathcal{S}}(x)}+2^{-r_{\\mathcal{S}}(x)}\\right) \\\\ & \\leqslant \\frac{1}{2} \\sum_{x \\in R}\\left(2^{-r_{\\mathcal{S}_{O}}(x)}+2^{-r_{\\mathcal{S}_{E}}(x)}\\right) \\end{aligned} $$ On the other hand, for every $x \\in O$, there is no $y \\in \\mathcal{S}_{O}$ such that $D_{\\mathcal{S}_{O}}(x, y)=d$ (as all candidates from $\\mathcal{S}$ were in $E$ ). Hence, we have $r_{\\mathcal{S}_{O}}(x) \\leqslant r_{\\mathcal{S}}(x)-1$, and thus $$ \\sum_{x \\in O} 2^{-r \\mathcal{S}_{\\mathcal{S}}(x)} \\leqslant \\frac{1}{2} \\sum_{x \\in O} 2^{-r s_{O}(x)} $$ Similarly, for every $x \\in E$, we have $$ \\sum_{x \\in E} 2^{-r_{\\mathcal{S}}(x)} \\leqslant \\frac{1}{2} \\sum_{x \\in E} 2^{-r_{\\mathcal{S}_{E}}(x)} $$ We can then combine these to give $$ \\begin{aligned} w(S) & =\\sum_{x \\in R} 2^{-r_{\\mathcal{S}}(x)}+\\sum_{x \\in O} 2^{-r_{\\mathcal{S}}(x)}+\\sum_{x \\in E} 2^{-r_{\\mathcal{S}}(x)} \\\\ & \\leqslant \\frac{1}{2} \\sum_{x \\in R}\\left(2^{-r_{\\mathcal{S}_{O}}(x)}+2^{-r_{\\mathcal{S}_{E}}(x)}\\right)+\\frac{1}{2} \\sum_{x \\in O} 2^{-r_{\\mathcal{S}_{O}}(x)}+\\frac{1}{2} \\sum_{x \\in E} 2^{-r_{\\mathcal{S}_{E}}(x)} \\\\ & \\left.=\\frac{1}{2}\\left(\\sum_{x \\in \\mathcal{S}_{O}} 2^{-r_{S_{O}}(x)}+\\sum_{x \\in \\mathcal{S}_{E}} 2^{-r s_{S_{E}}(x)}\\right) \\quad \\text { (since } \\mathcal{S}_{O}=O \\cup R \\text { and } \\mathcal{S}_{E}=E \\cup R\\right) \\\\ & \\left.\\left.=\\frac{1}{2}\\left(w\\left(\\mathcal{S}_{O}\\right)+w\\left(\\mathcal{S}_{E}\\right)\\right)\\right) \\quad \\text { (by definition of } w(\\cdot)\\right) \\\\ & \\leqslant 1 \\quad \\text { (by the inductive hypothesis) } \\end{aligned} $$ which completes the induction. Comment 1. The sets $O$ and $E$ above are not the only ones we could have chosen. Indeed, we could instead have used the following definitions: Let $d$ be the maximal scale between two distinct elements of $\\mathcal{S}$; that is, $d=D\\left(x_{1}, x_{n}\\right)$. Let $O=\\left\\{x \\in \\mathcal{S}: D\\left(x, x_{n}\\right)=d\\right\\}$ (a 'left' part of the set) and let $E=\\left\\{x \\in \\mathcal{S}: D\\left(x_{1}, x\\right)=d\\right\\}$ (a 'right' part of the set). Note that these two sets are disjoint, and nonempty (since they contain $x_{1}$ and $x_{n}$ respectively). The rest of the proof is then the same as in Solution 1. Comment 2. Another possible set $\\mathcal{F}$ containing $2^{k}$ members could arise from considering a binary tree of height $k$, allocating a real number to each leaf, and trying to make the scale between the values of two leaves dependent only on the (graph) distance between them. The following construction makes this more precise. We build up sets $\\mathcal{F}_{k}$ recursively. Let $\\mathcal{F}_{0}=\\{0\\}$, and then let $\\mathcal{F}_{k+1}=\\mathcal{F}_{k} \\cup\\left\\{x+3 \\cdot 4^{k}: x \\in \\mathcal{F}_{k}\\right\\}$ (i.e. each half of $\\mathcal{F}_{k+1}$ is a copy of $F_{k}$ ). We have that $\\mathcal{F}_{k}$ is contained in the interval $\\left[0,4^{k+1}\\right.$ ), and so it follows by induction on $k$ that every member of $F_{k+1}$ has $k$ different scales in its own half of $F_{k+1}$ (by the inductive hypothesis), and only the single scale $2 k+1$ in the other half of $F_{k+1}$. Both of the constructions presented here have the property that every member of $\\mathcal{F}$ has exactly $k$ different scales in $\\mathcal{F}$. Indeed, it can be seen that this must hold (up to a slight perturbation) for any such maximal set. Suppose there were some element $x$ with only $k-1$ different scales in $\\mathcal{F}$ (and every other element had at most $k$ different scales). Then we take some positive real $\\epsilon$, and construct a new set $\\mathcal{F}^{\\prime}=\\{y: y \\in \\mathcal{F}, y \\leqslant x\\} \\cup\\{y+\\epsilon: y \\in \\mathcal{F}, y \\geqslant x\\}$. We have $\\left|\\mathcal{F}^{\\prime}\\right|=|\\mathcal{F}|+1$, and if $\\epsilon$ is sufficiently small then $\\mathcal{F}^{\\prime}$ will also satisfy the property that no member has more than $k$ different scales in $\\mathcal{F}^{\\prime}$. This observation might be used to motivate the idea of weighting members of an arbitrary set $\\mathcal{S}$ of reals according to how many different scales they have in $\\mathcal{S}$.","tier":0} +{"problem_type":"Geometry","problem_label":"G1","problem":"Let $A B C$ be a triangle. Circle $\\Gamma$ passes through $A$, meets segments $A B$ and $A C$ again at points $D$ and $E$ respectively, and intersects segment $B C$ at $F$ and $G$ such that $F$ lies between $B$ and $G$. The tangent to circle $B D F$ at $F$ and the tangent to circle $C E G$ at $G$ meet at point $T$. Suppose that points $A$ and $T$ are distinct. Prove that line $A T$ is parallel to $B C$. (Nigeria)","solution":"Notice that $\\angle T F B=\\angle F D A$ because $F T$ is tangent to circle $B D F$, and moreover $\\angle F D A=\\angle C G A$ because quadrilateral $A D F G$ is cyclic. Similarly, $\\angle T G B=\\angle G E C$ because $G T$ is tangent to circle $C E G$, and $\\angle G E C=\\angle C F A$. Hence, $$ \\angle T F B=\\angle C G A \\text { and } \\quad \\angle T G B=\\angle C F A $$ ![](https:\/\/cdn.mathpix.com\/cropped\/2024_11_18_9ee8c2ac2c2f5002ff14g-058.jpg?height=624&width=1326&top_left_y=930&top_left_x=365) Triangles $F G A$ and $G F T$ have a common side $F G$, and by (1) their angles at $F, G$ are the same. So, these triangles are congruent. So, their altitudes starting from $A$ and $T$, respectively, are equal and hence $A T$ is parallel to line $B F G C$. Comment. Alternatively, we can prove first that $T$ lies on $\\Gamma$. For example, this can be done by showing that $\\angle A F T=\\angle A G T$ using (1). Then the statement follows as $\\angle T A F=\\angle T G F=\\angle G F A$.","tier":0} +{"problem_type":"Geometry","problem_label":"G2","problem":"Let $A B C$ be an acute-angled triangle and let $D, E$, and $F$ be the feet of altitudes from $A, B$, and $C$ to sides $B C, C A$, and $A B$, respectively. Denote by $\\omega_{B}$ and $\\omega_{C}$ the incircles of triangles $B D F$ and $C D E$, and let these circles be tangent to segments $D F$ and $D E$ at $M$ and $N$, respectively. Let line $M N$ meet circles $\\omega_{B}$ and $\\omega_{C}$ again at $P \\neq M$ and $Q \\neq N$, respectively. Prove that $M P=N Q$. (Vietnam)","solution":"Denote the centres of $\\omega_{B}$ and $\\omega_{C}$ by $O_{B}$ and $O_{C}$, let their radii be $r_{B}$ and $r_{C}$, and let $B C$ be tangent to the two circles at $T$ and $U$, respectively. ![](https:\/\/cdn.mathpix.com\/cropped\/2024_11_18_9ee8c2ac2c2f5002ff14g-059.jpg?height=801&width=1014&top_left_y=659&top_left_x=521) From the cyclic quadrilaterals $A F D C$ and $A B D E$ we have $$ \\angle M D O_{B}=\\frac{1}{2} \\angle F D B=\\frac{1}{2} \\angle B A C=\\frac{1}{2} \\angle C D E=\\angle O_{C} D N, $$ so the right-angled triangles $D M O_{B}$ and $D N O_{C}$ are similar. The ratio of similarity between the two triangles is $$ \\frac{D N}{D M}=\\frac{O_{C} N}{O_{B} M}=\\frac{r_{C}}{r_{B}} . $$ Let $\\varphi=\\angle D M N$ and $\\psi=\\angle M N D$. The lines $F M$ and $E N$ are tangent to $\\omega_{B}$ and $\\omega_{C}$, respectively, so $$ \\angle M T P=\\angle F M P=\\angle D M N=\\varphi \\quad \\text { and } \\quad \\angle Q U N=\\angle Q N E=\\angle M N D=\\psi $$ (It is possible that $P$ or $Q$ coincides with $T$ or $U$, or lie inside triangles $D M T$ or $D U N$, respectively. To reduce case-sensitivity, we may use directed angles or simply ignore angles $M T P$ and $Q U N$.) In the circles $\\omega_{B}$ and $\\omega_{C}$ the lengths of chords $M P$ and $N Q$ are $$ M P=2 r_{B} \\cdot \\sin \\angle M T P=2 r_{B} \\cdot \\sin \\varphi \\quad \\text { and } \\quad N Q=2 r_{C} \\cdot \\sin \\angle Q U N=2 r_{C} \\cdot \\sin \\psi $$ By applying the sine rule to triangle $D N M$ we get $$ \\frac{D N}{D M}=\\frac{\\sin \\angle D M N}{\\sin \\angle M N D}=\\frac{\\sin \\varphi}{\\sin \\psi} $$ Finally, putting the above observations together, we get $$ \\frac{M P}{N Q}=\\frac{2 r_{B} \\sin \\varphi}{2 r_{C} \\sin \\psi}=\\frac{r_{B}}{r_{C}} \\cdot \\frac{\\sin \\varphi}{\\sin \\psi}=\\frac{D M}{D N} \\cdot \\frac{\\sin \\varphi}{\\sin \\psi}=\\frac{\\sin \\psi}{\\sin \\varphi} \\cdot \\frac{\\sin \\varphi}{\\sin \\psi}=1, $$ so $M P=N Q$ as required.","tier":0} +{"problem_type":"Geometry","problem_label":"G4","problem":"Let $P$ be a point inside triangle $A B C$. Let $A P$ meet $B C$ at $A_{1}$, let $B P$ meet $C A$ at $B_{1}$, and let $C P$ meet $A B$ at $C_{1}$. Let $A_{2}$ be the point such that $A_{1}$ is the midpoint of $P A_{2}$, let $B_{2}$ be the point such that $B_{1}$ is the midpoint of $P B_{2}$, and let $C_{2}$ be the point such that $C_{1}$ is the midpoint of $P C_{2}$. Prove that points $A_{2}, B_{2}$, and $C_{2}$ cannot all lie strictly inside the circumcircle of triangle $A B C$. (Australia) ![](https:\/\/cdn.mathpix.com\/cropped\/2024_11_18_9ee8c2ac2c2f5002ff14g-062.jpg?height=763&width=715&top_left_y=578&top_left_x=662)","solution":"Since $$ \\angle A P B+\\angle B P C+\\angle C P A=2 \\pi=(\\pi-\\angle A C B)+(\\pi-\\angle B A C)+(\\pi-\\angle C B A), $$ at least one of the following inequalities holds: $$ \\angle A P B \\geqslant \\pi-\\angle A C B, \\quad \\angle B P C \\geqslant \\pi-\\angle B A C, \\quad \\angle C P A \\geqslant \\pi-\\angle C B A . $$ Without loss of generality, we assume that $\\angle B P C \\geqslant \\pi-\\angle B A C$. We have $\\angle B P C>\\angle B A C$ because $P$ is inside $\\triangle A B C$. So $\\angle B P C \\geqslant \\max (\\angle B A C, \\pi-\\angle B A C)$ and hence $$ \\sin \\angle B P C \\leqslant \\sin \\angle B A C . $$ Let the rays $A P, B P$, and $C P$ cross the circumcircle $\\Omega$ again at $A_{3}, B_{3}$, and $C_{3}$, respectively. We will prove that at least one of the ratios $\\frac{P B_{1}}{B_{1} B_{3}}$ and $\\frac{P C_{1}}{C_{1} C_{3}}$ is at least 1 , which yields that one of the points $B_{2}$ and $C_{2}$ does not lie strictly inside $\\Omega$. Because $A, B, C, B_{3}$ lie on a circle, the triangles $C B_{1} B_{3}$ and $B B_{1} A$ are similar, so $$ \\frac{C B_{1}}{B_{1} B_{3}}=\\frac{B B_{1}}{B_{1} A} $$ Applying the sine rule we obtain $$ \\frac{P B_{1}}{B_{1} B_{3}}=\\frac{P B_{1}}{C B_{1}} \\cdot \\frac{C B_{1}}{B_{1} B_{3}}=\\frac{P B_{1}}{C B_{1}} \\cdot \\frac{B B_{1}}{B_{1} A}=\\frac{\\sin \\angle A C P}{\\sin \\angle B P C} \\cdot \\frac{\\sin \\angle B A C}{\\sin \\angle P B A} . $$ Similarly, $$ \\frac{P C_{1}}{C_{1} C_{3}}=\\frac{\\sin \\angle P B A}{\\sin \\angle B P C} \\cdot \\frac{\\sin \\angle B A C}{\\sin \\angle A C P} $$ Multiplying these two equations we get $$ \\frac{P B_{1}}{B_{1} B_{3}} \\cdot \\frac{P C_{1}}{C_{1} C_{3}}=\\frac{\\sin ^{2} \\angle B A C}{\\sin ^{2} \\angle B P C} \\geqslant 1 $$ using (*), which yields the desired conclusion. Comment. It also cannot happen that all three points $A_{2}, B_{2}$, and $C_{2}$ lie strictly outside $\\Omega$. The same proof works almost literally, starting by assuming without loss of generality that $\\angle B P C \\leqslant \\pi-\\angle B A C$ and using $\\angle B P C>\\angle B A C$ to deduce that $\\sin \\angle B P C \\geqslant \\sin \\angle B A C$. It is possible for $A_{2}, B_{2}$, and $C_{2}$ all to lie on the circumcircle; from the above solution we may derive that this happens if and only if $P$ is the orthocentre of the triangle $A B C$, (which lies strictly inside $A B C$ if and only if $A B C$ is acute).","tier":0} +{"problem_type":"Geometry","problem_label":"G4","problem":"Let $P$ be a point inside triangle $A B C$. Let $A P$ meet $B C$ at $A_{1}$, let $B P$ meet $C A$ at $B_{1}$, and let $C P$ meet $A B$ at $C_{1}$. Let $A_{2}$ be the point such that $A_{1}$ is the midpoint of $P A_{2}$, let $B_{2}$ be the point such that $B_{1}$ is the midpoint of $P B_{2}$, and let $C_{2}$ be the point such that $C_{1}$ is the midpoint of $P C_{2}$. Prove that points $A_{2}, B_{2}$, and $C_{2}$ cannot all lie strictly inside the circumcircle of triangle $A B C$. (Australia) ![](https:\/\/cdn.mathpix.com\/cropped\/2024_11_18_9ee8c2ac2c2f5002ff14g-062.jpg?height=763&width=715&top_left_y=578&top_left_x=662)","solution":"Define points $A_{3}, B_{3}$, and $C_{3}$ as in Observe that $\\triangle P B C_{3} \\sim \\triangle P C B_{3}$. Let $X$ be the point on side $P B_{3}$ that corresponds to point $C_{1}$ on side $P C_{3}$ under this similarity. In other words, $X$ lies on segment $P B_{3}$ and satisfies $P X: X B_{3}=P C_{1}: C_{1} C_{3}$. It follows that $$ \\angle X C P=\\angle P B C_{1}=\\angle B_{3} B A=\\angle B_{3} C B_{1} . $$ Hence lines $C X$ and $C B_{1}$ are isogonal conjugates in $\\triangle P C B_{3}$. ![](https:\/\/cdn.mathpix.com\/cropped\/2024_11_18_9ee8c2ac2c2f5002ff14g-063.jpg?height=812&width=807&top_left_y=1207&top_left_x=533) Let $Y$ be the foot of the bisector of $\\angle B_{3} C P$ in $\\triangle P C B_{3}$. Since $P C_{1}0$ and $\\alpha+\\beta+\\gamma=1$. Then $$ A_{1}=\\frac{\\beta B+\\gamma C}{\\beta+\\gamma}=\\frac{1}{1-\\alpha} P-\\frac{\\alpha}{1-\\alpha} A, $$ so $$ A_{2}=2 A_{1}-P=\\frac{1+\\alpha}{1-\\alpha} P-\\frac{2 \\alpha}{1-\\alpha} A $$ Hence $$ \\left|A_{2}\\right|^{2}=\\left(\\frac{1+\\alpha}{1-\\alpha}\\right)^{2}|P|^{2}+\\left(\\frac{2 \\alpha}{1-\\alpha}\\right)^{2}|A|^{2}-\\frac{4 \\alpha(1+\\alpha)}{(1-\\alpha)^{2}} A \\cdot P $$ Using $|A|^{2}=1$ we obtain $$ \\frac{(1-\\alpha)^{2}}{2(1+\\alpha)}\\left|A_{2}\\right|^{2}=\\frac{1+\\alpha}{2}|P|^{2}+\\frac{2 \\alpha^{2}}{1+\\alpha}-2 \\alpha A \\cdot P $$ Likewise $$ \\frac{(1-\\beta)^{2}}{2(1+\\beta)}\\left|B_{2}\\right|^{2}=\\frac{1+\\beta}{2}|P|^{2}+\\frac{2 \\beta^{2}}{1+\\beta}-2 \\beta B \\cdot P $$ and $$ \\frac{(1-\\gamma)^{2}}{2(1+\\gamma)}\\left|C_{2}\\right|^{2}=\\frac{1+\\gamma}{2}|P|^{2}+\\frac{2 \\gamma^{2}}{1+\\gamma}-2 \\gamma C \\cdot P $$ Summing (1), (2) and (3) we obtain on the LHS the positive linear combination $$ \\text { LHS }=\\frac{(1-\\alpha)^{2}}{2(1+\\alpha)}\\left|A_{2}\\right|^{2}+\\frac{(1-\\beta)^{2}}{2(1+\\beta)}\\left|B_{2}\\right|^{2}+\\frac{(1-\\gamma)^{2}}{2(1+\\gamma)}\\left|C_{2}\\right|^{2} $$ and on the RHS the quantity $$ \\left(\\frac{1+\\alpha}{2}+\\frac{1+\\beta}{2}+\\frac{1+\\gamma}{2}\\right)|P|^{2}+\\left(\\frac{2 \\alpha^{2}}{1+\\alpha}+\\frac{2 \\beta^{2}}{1+\\beta}+\\frac{2 \\gamma^{2}}{1+\\gamma}\\right)-2(\\alpha A \\cdot P+\\beta B \\cdot P+\\gamma C \\cdot P) . $$ The first term is $2|P|^{2}$ and the last term is $-2 P \\cdot P$, so $$ \\begin{aligned} \\text { RHS } & =\\left(\\frac{2 \\alpha^{2}}{1+\\alpha}+\\frac{2 \\beta^{2}}{1+\\beta}+\\frac{2 \\gamma^{2}}{1+\\gamma}\\right) \\\\ & =\\frac{3 \\alpha-1}{2}+\\frac{(1-\\alpha)^{2}}{2(1+\\alpha)}+\\frac{3 \\beta-1}{2}+\\frac{(1-\\beta)^{2}}{2(1+\\beta)}+\\frac{3 \\gamma-1}{2}+\\frac{(1-\\gamma)^{2}}{2(1+\\gamma)} \\\\ & =\\frac{(1-\\alpha)^{2}}{2(1+\\alpha)}+\\frac{(1-\\beta)^{2}}{2(1+\\beta)}+\\frac{(1-\\gamma)^{2}}{2(1+\\gamma)} \\end{aligned} $$ Here we used the fact that $$ \\frac{3 \\alpha-1}{2}+\\frac{3 \\beta-1}{2}+\\frac{3 \\gamma-1}{2}=0 . $$ We have shown that a linear combination of $\\left|A_{1}\\right|^{2},\\left|B_{1}\\right|^{2}$, and $\\left|C_{1}\\right|^{2}$ with positive coefficients is equal to the sum of the coefficients. Therefore at least one of $\\left|A_{1}\\right|^{2},\\left|B_{1}\\right|^{2}$, and $\\left|C_{1}\\right|^{2}$ must be at least 1 , as required. Comment. This proof also works when $P$ is any point for which $\\alpha, \\beta, \\gamma>-1, \\alpha+\\beta+\\gamma=1$, and $\\alpha, \\beta, \\gamma \\neq 1$. (In any cases where $\\alpha=1$ or $\\beta=1$ or $\\gamma=1$, some points in the construction are not defined.) This page is intentionally left blank","tier":0} +{"problem_type":"Geometry","problem_label":"G5","problem":"Let $A B C D E$ be a convex pentagon with $C D=D E$ and $\\angle E D C \\neq 2 \\cdot \\angle A D B$. Suppose that a point $P$ is located in the interior of the pentagon such that $A P=A E$ and $B P=B C$. Prove that $P$ lies on the diagonal $C E$ if and only if area $(B C D)+\\operatorname{area}(A D E)=$ $\\operatorname{area}(A B D)+\\operatorname{area}(A B P)$. (Hungary)","solution":"Let $P^{\\prime}$ be the reflection of $P$ across line $A B$, and let $M$ and $N$ be the midpoints of $P^{\\prime} E$ and $P^{\\prime} C$ respectively. Convexity ensures that $P^{\\prime}$ is distinct from both $E$ and $C$, and hence from both $M$ and $N$. We claim that both the area condition and the collinearity condition in the problem are equivalent to the condition that the (possibly degenerate) right-angled triangles $A P^{\\prime} M$ and $B P^{\\prime} N$ are directly similar (equivalently, $A P^{\\prime} E$ and $B P^{\\prime} C$ are directly similar). ![](https:\/\/cdn.mathpix.com\/cropped\/2024_11_18_9ee8c2ac2c2f5002ff14g-066.jpg?height=535&width=549&top_left_y=806&top_left_x=756) For the equivalence with the collinearity condition, let $F$ denote the foot of the perpendicular from $P^{\\prime}$ to $A B$, so that $F$ is the midpoint of $P P^{\\prime}$. We have that $P$ lies on $C E$ if and only if $F$ lies on $M N$, which occurs if and only if we have the equality $\\angle A F M=\\angle B F N$ of signed angles modulo $\\pi$. By concyclicity of $A P^{\\prime} F M$ and $B F P^{\\prime} N$, this is equivalent to $\\angle A P^{\\prime} M=\\angle B P^{\\prime} N$, which occurs if and only if $A P^{\\prime} M$ and $B P^{\\prime} N$ are directly similar. ![](https:\/\/cdn.mathpix.com\/cropped\/2024_11_18_9ee8c2ac2c2f5002ff14g-066.jpg?height=306&width=441&top_left_y=1663&top_left_x=813) For the other equivalence with the area condition, we have the equality of signed areas $\\operatorname{area}(A B D)+\\operatorname{area}(A B P)=\\operatorname{area}\\left(A P^{\\prime} B D\\right)=\\operatorname{area}\\left(A P^{\\prime} D\\right)+\\operatorname{area}\\left(B D P^{\\prime}\\right)$. Using the identity $\\operatorname{area}(A D E)-\\operatorname{area}\\left(A P^{\\prime} D\\right)=\\operatorname{area}(A D E)+\\operatorname{area}\\left(A D P^{\\prime}\\right)=2$ area $(A D M)$, and similarly for $B$, we find that the area condition is equivalent to the equality $$ \\operatorname{area}(D A M)=\\operatorname{area}(D B N) . $$ Now note that $A$ and $B$ lie on the perpendicular bisectors of $P^{\\prime} E$ and $P^{\\prime} C$, respectively. If we write $G$ and $H$ for the feet of the perpendiculars from $D$ to these perpendicular bisectors respectively, then this area condition can be rewritten as $$ M A \\cdot G D=N B \\cdot H D $$ (In this condition, we interpret all lengths as signed lengths according to suitable conventions: for instance, we orient $P^{\\prime} E$ from $P^{\\prime}$ to $E$, orient the parallel line $D H$ in the same direction, and orient the perpendicular bisector of $P^{\\prime} E$ at an angle $\\pi \/ 2$ clockwise from the oriented segment $P^{\\prime} E$ - we adopt the analogous conventions at $B$.) ![](https:\/\/cdn.mathpix.com\/cropped\/2024_11_18_9ee8c2ac2c2f5002ff14g-067.jpg?height=529&width=575&top_left_y=204&top_left_x=758) To relate the signed lengths $G D$ and $H D$ to the triangles $A P^{\\prime} M$ and $B P^{\\prime} N$, we use the following calculation. Claim. Let $\\Gamma$ denote the circle centred on $D$ with both $E$ and $C$ on the circumference, and $h$ the power of $P^{\\prime}$ with respect to $\\Gamma$. Then we have the equality $$ G D \\cdot P^{\\prime} M=H D \\cdot P^{\\prime} N=\\frac{1}{4} h \\neq 0 . $$ Proof. Firstly, we have $h \\neq 0$, since otherwise $P^{\\prime}$ would lie on $\\Gamma$, and hence the internal angle bisectors of $\\angle E D P^{\\prime}$ and $\\angle P^{\\prime} D C$ would pass through $A$ and $B$ respectively. This would violate the angle inequality $\\angle E D C \\neq 2 \\cdot \\angle A D B$ given in the question. Next, let $E^{\\prime}$ denote the second point of intersection of $P^{\\prime} E$ with $\\Gamma$, and let $E^{\\prime \\prime}$ denote the point on $\\Gamma$ diametrically opposite $E^{\\prime}$, so that $E^{\\prime \\prime} E$ is perpendicular to $P^{\\prime} E$. The point $G$ lies on the perpendicular bisectors of the sides $P^{\\prime} E$ and $E E^{\\prime \\prime}$ of the right-angled triangle $P^{\\prime} E E^{\\prime \\prime}$; it follows that $G$ is the midpoint of $P^{\\prime} E^{\\prime \\prime}$. Since $D$ is the midpoint of $E^{\\prime} E^{\\prime \\prime}$, we have that $G D=\\frac{1}{2} P^{\\prime} E^{\\prime}$. Since $P^{\\prime} M=\\frac{1}{2} P^{\\prime} E$, we have $G D \\cdot P^{\\prime} M=\\frac{1}{4} P^{\\prime} E^{\\prime} \\cdot P^{\\prime} E=\\frac{1}{4} h$. The other equality $H D \\cdot P^{\\prime} N$ follows by exactly the same argument. ![](https:\/\/cdn.mathpix.com\/cropped\/2024_11_18_9ee8c2ac2c2f5002ff14g-067.jpg?height=727&width=672&top_left_y=1664&top_left_x=726) From this claim, we see that the area condition is equivalent to the equality $$ \\left(M A: P^{\\prime} M\\right)=\\left(N B: P^{\\prime} N\\right) $$ of ratios of signed lengths, which is equivalent to direct similarity of $A P^{\\prime} M$ and $B P^{\\prime} N$, as desired.","tier":0} +{"problem_type":"Geometry","problem_label":"G5","problem":"Let $A B C D E$ be a convex pentagon with $C D=D E$ and $\\angle E D C \\neq 2 \\cdot \\angle A D B$. Suppose that a point $P$ is located in the interior of the pentagon such that $A P=A E$ and $B P=B C$. Prove that $P$ lies on the diagonal $C E$ if and only if area $(B C D)+\\operatorname{area}(A D E)=$ $\\operatorname{area}(A B D)+\\operatorname{area}(A B P)$. (Hungary)","solution":"Along the perpendicular bisector of $C E$, define the linear function $$ f(X)=\\operatorname{area}(B C X)+\\operatorname{area}(A X E)-\\operatorname{area}(A B X)-\\operatorname{area}(A B P), $$ where, from now on, we always use signed areas. Thus, we want to show that $C, P, E$ are collinear if and only if $f(D)=0$. ![](https:\/\/cdn.mathpix.com\/cropped\/2024_11_18_9ee8c2ac2c2f5002ff14g-068.jpg?height=433&width=512&top_left_y=503&top_left_x=772) Let $P^{\\prime}$ be the reflection of $P$ across line $A B$. The point $P^{\\prime}$ does not lie on the line $C E$. To see this, we let $A^{\\prime \\prime}$ and $B^{\\prime \\prime}$ be the points obtained from $A$ and $B$ by dilating with scale factor 2 about $P^{\\prime}$, so that $P$ is the orthogonal projection of $P^{\\prime}$ onto $A^{\\prime \\prime} B^{\\prime \\prime}$. Since $A$ lies on the perpendicular bisector of $P^{\\prime} E$, the triangle $A^{\\prime \\prime} E P^{\\prime}$ is right-angled at $E$ (and $B^{\\prime \\prime} C P^{\\prime}$ similarly). If $P^{\\prime}$ were to lie on $C E$, then the lines $A^{\\prime \\prime} E$ and $B^{\\prime \\prime} C$ would be perpendicular to $C E$ and $A^{\\prime \\prime}$ and $B^{\\prime \\prime}$ would lie on the opposite side of $C E$ to $D$. It follows that the line $A^{\\prime \\prime} B^{\\prime \\prime}$ does not meet triangle $C D E$, and hence point $P$ does not lie inside $C D E$. But then $P$ must lie inside $A B C E$, and it is clear that such a point cannot reflect to a point $P^{\\prime}$ on $C E$. We thus let $O$ be the centre of the circle $C E P^{\\prime}$. The lines $A O$ and $B O$ are the perpendicular bisectors of $E P^{\\prime}$ and $C P^{\\prime}$, respectively, so $$ \\begin{aligned} \\operatorname{area}(B C O)+\\operatorname{area}(A O E) & =\\operatorname{area}\\left(O P^{\\prime} B\\right)+\\operatorname{area}\\left(P^{\\prime} O A\\right)=\\operatorname{area}\\left(P^{\\prime} B O A\\right) \\\\ & =\\operatorname{area}(A B O)+\\operatorname{area}\\left(B A P^{\\prime}\\right)=\\operatorname{area}(A B O)+\\operatorname{area}(A B P), \\end{aligned} $$ and hence $f(O)=0$. Notice that if point $O$ coincides with $D$ then points $A, B$ lie in angle domain $C D E$ and $\\angle E O C=2 \\cdot \\angle A O B$, which is not allowed. So, $O$ and $D$ must be distinct. Since $f$ is linear and vanishes at $O$, it follows that $f(D)=0$ if and only if $f$ is constant zero - we want to show this occurs if and only if $C, P, E$ are collinear. ![](https:\/\/cdn.mathpix.com\/cropped\/2024_11_18_9ee8c2ac2c2f5002ff14g-068.jpg?height=556&width=1254&top_left_y=1906&top_left_x=408) In the one direction, suppose firstly that $C, P, E$ are not collinear, and let $T$ be the centre of the circle $C E P$. The same calculation as above provides $$ \\operatorname{area}(B C T)+\\operatorname{area}(A T E)=\\operatorname{area}(P B T A)=\\operatorname{area}(A B T)-\\operatorname{area}(A B P) $$ $$ f(T)=-2 \\operatorname{area}(A B P) \\neq 0 $$ Hence, the linear function $f$ is nonconstant with its zero is at $O$, so that $f(D) \\neq 0$. In the other direction, suppose that the points $C, P, E$ are collinear. We will show that $f$ is constant zero by finding a second point (other than $O$ ) at which it vanishes. ![](https:\/\/cdn.mathpix.com\/cropped\/2024_11_18_9ee8c2ac2c2f5002ff14g-069.jpg?height=410&width=1098&top_left_y=366&top_left_x=479) Let $Q$ be the reflection of $P$ across the midpoint of $A B$, so $P A Q B$ is a parallelogram. It is easy to see that $Q$ is on the perpendicular bisector of $C E$; for instance if $A^{\\prime}$ and $B^{\\prime}$ are the points produced from $A$ and $B$ by dilating about $P$ with scale factor 2, then the projection of $Q$ to $C E$ is the midpoint of the projections of $A^{\\prime}$ and $B^{\\prime}$, which are $E$ and $C$ respectively. The triangles $B C Q$ and $A Q E$ are indirectly congruent, so $$ f(Q)=(\\operatorname{area}(B C Q)+\\operatorname{area}(A Q E))-(\\operatorname{area}(A B Q)-\\operatorname{area}(B A P))=0-0=0 $$ The points $O$ and $Q$ are distinct. To see this, consider the circle $\\omega$ centred on $Q$ with $P^{\\prime}$ on the circumference; since triangle $P P^{\\prime} Q$ is right-angled at $P^{\\prime}$, it follows that $P$ lies outside $\\omega$. On the other hand, $P$ lies between $C$ and $E$ on the line $C P E$. It follows that $C$ and $E$ cannot both lie on $\\omega$, so that $\\omega$ is not the circle $C E P^{\\prime}$ and $Q \\neq O$. Since $O$ and $Q$ are distinct zeroes of the linear function $f$, we have $f(D)=0$ as desired. Comment 1. The condition $\\angle E D C \\neq 2 \\cdot \\angle A D B$ cannot be omitted. If $D$ is the centre of circle $C E P^{\\prime}$, then the condition on triangle areas is satisfied automatically, without having $P$ on line $C E$. Comment 2. The \"only if\" part of this problem is easier than the \"if\" part. For example, in the second part of Solution 2, the triangles $E A Q$ and $Q B C$ are indirectly congruent, so the sum of their areas is 0 , and $D C Q E$ is a kite. Now one can easily see that $\\angle(A Q, D E)=\\angle(C D, C B)$ and $\\angle(B Q, D C)=\\angle(E D, E A)$, whence area $(B C D)=\\operatorname{area}(A Q D)+\\operatorname{area}(E Q A)$ and $\\operatorname{area}(A D E)=$ $\\operatorname{area}(B D Q)+\\operatorname{area}(B Q C)$, which yields the result. Comment 3. The origin of the problem is the following observation. Let $A B D H$ be a tetrahedron and consider the sphere $\\mathcal{S}$ that is tangent to the four face planes, internally to planes $A D H$ and $B D H$ and externally to $A B D$ and $A B H$ (or vice versa). It is known that the sphere $\\mathcal{S}$ exists if and only if area $(A D H)+\\operatorname{area}(B D H) \\neq \\operatorname{area}(A B H)+\\operatorname{area}(A B D)$; this relation comes from the usual formula for the volume of the tetrahedron. Let $T, T_{a}, T_{b}, T_{d}$ be the points of tangency between the sphere and the four planes, as shown in the picture. Rotate the triangle $A B H$ inward, the triangles $B D H$ and $A D H$ outward, into the triangles $A B P, B D C$ and $A D E$, respectively, in the plane $A B D$. Notice that the points $T_{d}, T_{a}, T_{b}$ are rotated to $T$, so we have $H T_{a}=H T_{b}=H T_{d}=P T=C T=E T$. Therefore, the point $T$ is the centre of the circle $C E P$. Hence, if the sphere exists then $C, E, P$ cannot be collinear. If the condition $\\angle E D C \\neq 2 \\cdot \\angle A D B$ is replaced by the constraint that the angles $\\angle E D A, \\angle A D B$ and $\\angle B D C$ satisfy the triangle inequality, it enables reconstructing the argument with the tetrahedron and the tangent sphere. ![](https:\/\/cdn.mathpix.com\/cropped\/2024_11_18_9ee8c2ac2c2f5002ff14g-070.jpg?height=1069&width=1306&top_left_y=825&top_left_x=381)","tier":0} +{"problem_type":"Geometry","problem_label":"G6","problem":"Let $I$ be the incentre of acute-angled triangle $A B C$. Let the incircle meet $B C, C A$, and $A B$ at $D, E$, and $F$, respectively. Let line $E F$ intersect the circumcircle of the triangle at $P$ and $Q$, such that $F$ lies between $E$ and $P$. Prove that $\\angle D P A+\\angle A Q D=\\angle Q I P$. (Slovakia)","solution":"Let $N$ and $M$ be the midpoints of the arcs $\\widehat{B C}$ of the circumcircle, containing and opposite vertex $A$, respectively. By $\\angle F A E=\\angle B A C=\\angle B N C$, the right-angled kites $A F I E$ and $N B M C$ are similar. Consider the spiral similarity $\\varphi$ (dilation in case of $A B=A C$ ) that moves $A F I E$ to $N B M C$. The directed angle in which $\\varphi$ changes directions is $\\angle(A F, N B)$, same as $\\angle(A P, N P)$ and $\\angle(A Q, N Q)$; so lines $A P$ and $A Q$ are mapped to lines $N P$ and $N Q$, respectively. Line $E F$ is mapped to $B C$; we can see that the intersection points $P=E F \\cap A P$ and $Q=E F \\cap A Q$ are mapped to points $B C \\cap N P$ and $B C \\cap N Q$, respectively. Denote these points by $P^{\\prime}$ and $Q^{\\prime}$, respectively. ![](https:\/\/cdn.mathpix.com\/cropped\/2024_11_18_9ee8c2ac2c2f5002ff14g-071.jpg?height=689&width=1512&top_left_y=883&top_left_x=272) Let $L$ be the midpoint of $B C$. We claim that points $P, Q, D, L$ are concyclic (if $D=L$ then line $B C$ is tangent to circle $P Q D$ ). Let $P Q$ and $B C$ meet at $Z$. By applying Menelaus' theorem to triangle $A B C$ and line $E F Z$, we have $$ \\frac{B D}{D C}=\\frac{B F}{F A} \\cdot \\frac{A E}{E C}=-\\frac{B Z}{Z C}, $$ so the pairs $B, C$ and $D, Z$ are harmonic. It is well-known that this implies $Z B \\cdot Z C=Z D \\cdot Z L$. (The inversion with pole $Z$ that swaps $B$ and $C$ sends $Z$ to infinity and $D$ to the midpoint of $B C$, because the cross-ratio is preserved.) Hence, $Z D \\cdot Z L=Z B \\cdot Z C=Z P \\cdot Z Q$ by the power of $Z$ with respect to the circumcircle; this proves our claim. By $\\angle M P P^{\\prime}=\\angle M Q Q^{\\prime}=\\angle M L P^{\\prime}=\\angle M L Q^{\\prime}=90^{\\circ}$, the quadrilaterals $M L P P^{\\prime}$ and $M L Q Q^{\\prime}$ are cyclic. Then the problem statement follows by $$ \\begin{aligned} \\angle D P A+\\angle A Q D & =360^{\\circ}-\\angle P A Q-\\angle Q D P=360^{\\circ}-\\angle P N Q-\\angle Q L P \\\\ & =\\angle L P N+\\angle N Q L=\\angle P^{\\prime} M L+\\angle L M Q^{\\prime}=\\angle P^{\\prime} M Q^{\\prime}=\\angle P I Q . \\end{aligned} $$","tier":0} +{"problem_type":"Geometry","problem_label":"G6","problem":"Let $I$ be the incentre of acute-angled triangle $A B C$. Let the incircle meet $B C, C A$, and $A B$ at $D, E$, and $F$, respectively. Let line $E F$ intersect the circumcircle of the triangle at $P$ and $Q$, such that $F$ lies between $E$ and $P$. Prove that $\\angle D P A+\\angle A Q D=\\angle Q I P$. (Slovakia)","solution":"Define the point $M$ and the same spiral similarity $\\varphi$ as in the previous solution. (The point $N$ is not necessary.) It is well-known that the centre of the spiral similarity that maps $F, E$ to $B, C$ is the Miquel point of the lines $F E, B C, B F$ and $C E$; that is, the second intersection of circles $A B C$ and $A E F$. Denote that point by $S$. By $\\varphi(F)=B$ and $\\varphi(E)=C$ the triangles $S B F$ and $S C E$ are similar, so we have $$ \\frac{S B}{S C}=\\frac{B F}{C E}=\\frac{B D}{C D} $$ By the converse of the angle bisector theorem, that indicates that line $S D$ bisects $\\angle B S C$ and hence passes through $M$. Let $K$ be the intersection point of lines $E F$ and $S I$. Notice that $\\varphi$ sends points $S, F, E, I$ to $S, B, C, M$, so $\\varphi(K)=\\varphi(F E \\cap S I)=B C \\cap S M=D$. By $\\varphi(I)=M$, we have $K D \\| I M$. ![](https:\/\/cdn.mathpix.com\/cropped\/2024_11_18_9ee8c2ac2c2f5002ff14g-072.jpg?height=669&width=706&top_left_y=839&top_left_x=675) We claim that triangles $S P I$ and $S D Q$ are similar, and so are triangles $S P D$ and $S I Q$. Let ray $S I$ meet the circumcircle again at $L$. Note that the segment $E F$ is perpendicular to the angle bisector $A M$. Then by $\\angle A M L=\\angle A S L=\\angle A S I=90^{\\circ}$, we have $M L \\| P Q$. Hence, $\\widetilde{P L}=\\widetilde{M Q}$ and therefore $\\angle P S L=\\angle M S Q=\\angle D S Q$. By $\\angle Q P S=\\angle Q M S$, the triangles $S P K$ and $S M Q$ are similar. Finally, $$ \\frac{S P}{S I}=\\frac{S P}{S K} \\cdot \\frac{S K}{S I}=\\frac{S M}{S Q} \\cdot \\frac{S D}{S M}=\\frac{S D}{S Q} $$ shows that triangles $S P I$ and $S D Q$ are similar. The second part of the claim can be proved analogously. Now the problem statement can be proved by $$ \\angle D P A+\\angle A Q D=\\angle D P S+\\angle S Q D=\\angle Q I S+\\angle S I P=\\angle Q I P . $$","tier":0} +{"problem_type":"Geometry","problem_label":"G6","problem":"Let $I$ be the incentre of acute-angled triangle $A B C$. Let the incircle meet $B C, C A$, and $A B$ at $D, E$, and $F$, respectively. Let line $E F$ intersect the circumcircle of the triangle at $P$ and $Q$, such that $F$ lies between $E$ and $P$. Prove that $\\angle D P A+\\angle A Q D=\\angle Q I P$. (Slovakia)","solution":"Denote the circumcircle of triangle $A B C$ by $\\Gamma$, and let rays $P D$ and $Q D$ meet $\\Gamma$ again at $V$ and $U$, respectively. We will show that $A U \\perp I P$ and $A V \\perp I Q$. Then the problem statement will follow as $$ \\angle D P A+\\angle A Q D=\\angle V U A+\\angle A V U=180^{\\circ}-\\angle U A V=\\angle Q I P . $$ Let $M$ be the midpoint of arc $\\widehat{B U V C}$ and let $N$ be the midpoint of arc $\\widehat{C A B}$; the lines $A I M$ and $A N$ being the internal and external bisectors of angle $B A C$, respectively, are perpendicular. Let the tangents drawn to $\\Gamma$ at $B$ and $C$ meet at $R$; let line $P Q$ meet $A U, A I, A V$ and $B C$ at $X, T, Y$ and $Z$, respectively. As in Solution 1, we observe that the pairs $B, C$ and $D, Z$ are harmonic. Projecting these points from $Q$ onto the circumcircle, we can see that $B, C$ and $U, P$ are also harmonic. Analogously, the pair $V, Q$ is harmonic with $B, C$. Consider the inversion about the circle with centre $R$, passing through $B$ and $C$. Points $B$ and $C$ are fixed points, so this inversion exchanges every point of $\\Gamma$ by its harmonic pair with respect to $B, C$. In particular, the inversion maps points $B, C, N, U, V$ to points $B, C, M, P, Q$, respectively. Combine the inversion with projecting $\\Gamma$ from $A$ to line $P Q$; the points $B, C, M, P, Q$ are projected to $F, E, T, P, Q$, respectively. ![](https:\/\/cdn.mathpix.com\/cropped\/2024_11_18_9ee8c2ac2c2f5002ff14g-073.jpg?height=1006&width=1384&top_left_y=1082&top_left_x=339) The combination of these two transformations is projective map from the lines $A B, A C$, $A N, A U, A V$ to $I F, I E, I T, I P, I Q$, respectively. On the other hand, we have $A B \\perp I F$, $A C \\perp I E$ and $A N \\perp A T$, so the corresponding lines in these two pencils are perpendicular. This proves $A U \\perp I P$ and $A V \\perp I Q$, and hence completes the solution.","tier":0} +{"problem_type":"Geometry","problem_label":"G7","problem":"The incircle $\\omega$ of acute-angled scalene triangle $A B C$ has centre $I$ and meets sides $B C$, $C A$, and $A B$ at $D, E$, and $F$, respectively. The line through $D$ perpendicular to $E F$ meets $\\omega$ again at $R$. Line $A R$ meets $\\omega$ again at $P$. The circumcircles of triangles $P C E$ and $P B F$ meet again at $Q \\neq P$. Prove that lines $D I$ and $P Q$ meet on the external bisector of angle $B A C$. (India) Common remarks. Throughout the solution, $\\angle(a, b)$ denotes the directed angle between lines $a$ and $b$, measured modulo $\\pi$.","solution":"Step 1. The external bisector of $\\angle B A C$ is the line through $A$ perpendicular to $I A$. Let $D I$ meet this line at $L$ and let $D I$ meet $\\omega$ at $K$. Let $N$ be the midpoint of $E F$, which lies on $I A$ and is the pole of line $A L$ with respect to $\\omega$. Since $A N \\cdot A I=A E^{2}=A R \\cdot A P$, the points $R$, $N, I$, and $P$ are concyclic. As $I R=I P$, the line $N I$ is the external bisector of $\\angle P N R$, so $P N$ meets $\\omega$ again at the point symmetric to $R$ with respect to $A N-i . e$ at $K$. Let $D N$ cross $\\omega$ again at $S$. Opposite sides of any quadrilateral inscribed in the circle $\\omega$ meet on the polar line of the intersection of the diagonals with respect to $\\omega$. Since $L$ lies on the polar line $A L$ of $N$ with respect to $\\omega$, the line $P S$ must pass through $L$. Thus it suffices to prove that the points $S, Q$, and $P$ are collinear. ![](https:\/\/cdn.mathpix.com\/cropped\/2024_11_18_9ee8c2ac2c2f5002ff14g-074.jpg?height=875&width=1350&top_left_y=1176&top_left_x=361) Step 2. Let $\\Gamma$ be the circumcircle of $\\triangle B I C$. Notice that $$ \\begin{aligned} & \\angle(B Q, Q C)=\\angle(B Q, Q P)+\\angle(P Q, Q C)=\\angle(B F, F P)+\\angle(P E, E C) \\\\ &=\\angle(E F, E P)+\\angle(F P, F E)=\\angle(F P, E P)=\\angle(D F, D E)=\\angle(B I, I C) \\end{aligned} $$ so $Q$ lies on $\\Gamma$. Let $Q P$ meet $\\Gamma$ again at $T$. It will now suffice to prove that $S, P$, and $T$ are collinear. Notice that $\\angle(B I, I T)=\\angle(B Q, Q T)=\\angle(B F, F P)=\\angle(F K, K P)$. Note $F D \\perp F K$ and $F D \\perp B I$ so $F K \\| B I$ and hence $I T$ is parallel to the line $K N P$. Since $D I=I K$, the line $I T$ crosses $D N$ at its midpoint $M$. Step 3. Let $F^{\\prime}$ and $E^{\\prime}$ be the midpoints of $D E$ and $D F$, respectively. Since $D E^{\\prime} \\cdot E^{\\prime} F=D E^{\\prime 2}=$ $B E^{\\prime} \\cdot E^{\\prime} I$, the point $E^{\\prime}$ lies on the radical axis of $\\omega$ and $\\Gamma$; the same holds for $F^{\\prime}$. Therefore, this radical axis is $E^{\\prime} F^{\\prime}$, and it passes through $M$. Thus $I M \\cdot M T=D M \\cdot M S$, so $S, I, D$, and $T$ are concyclic. This shows $\\angle(D S, S T)=\\angle(D I, I T)=\\angle(D K, K P)=\\angle(D S, S P)$, whence the points $S, P$, and $T$ are collinear, as desired. ![](https:\/\/cdn.mathpix.com\/cropped\/2024_11_18_9ee8c2ac2c2f5002ff14g-075.jpg?height=1089&width=992&top_left_y=198&top_left_x=532) Comment. Here is a longer alternative proof in step 1 that $P, S$, and $L$ are collinear, using a circular inversion instead of the fact that opposite sides of a quadrilateral inscribed in a circle $\\omega$ meet on the polar line with respect to $\\omega$ of the intersection of the diagonals. Let $G$ be the foot of the altitude from $N$ to the line $D I K L$. Observe that $N, G, K, S$ are concyclic (opposite right angles) so $$ \\angle D I P=2 \\angle D K P=\\angle G K N+\\angle D S P=\\angle G S N+\\angle N S P=\\angle G S P, $$ hence $I, G, S, P$ are concyclic. We have $I G \\cdot I L=I N \\cdot I A=r^{2}$ since $\\triangle I G N \\sim \\triangle I A L$. Inverting the circle $I G S P$ in circle $\\omega$, points $P$ and $S$ are fixed and $G$ is taken to $L$ so we find that $P, S$, and $L$ are collinear.","tier":0} +{"problem_type":"Geometry","problem_label":"G7","problem":"The incircle $\\omega$ of acute-angled scalene triangle $A B C$ has centre $I$ and meets sides $B C$, $C A$, and $A B$ at $D, E$, and $F$, respectively. The line through $D$ perpendicular to $E F$ meets $\\omega$ again at $R$. Line $A R$ meets $\\omega$ again at $P$. The circumcircles of triangles $P C E$ and $P B F$ meet again at $Q \\neq P$. Prove that lines $D I$ and $P Q$ meet on the external bisector of angle $B A C$. (India) Common remarks. Throughout the solution, $\\angle(a, b)$ denotes the directed angle between lines $a$ and $b$, measured modulo $\\pi$.","solution":"We start as in Step 1. Let $A R$ meet the circumcircle $\\Omega$ of $A B C$ again at $X$. The lines $A R$ and $A K$ are isogonal in the angle $B A C$; it is well known that in this case $X$ is the tangency point of $\\Omega$ with the $A$-mixtilinear circle. It is also well known that for this point $X$, the line $X I$ crosses $\\Omega$ again at the midpoint $M^{\\prime}$ of arc $B A C$. Step 2. Denote the circles $B F P$ and $C E P$ by $\\Omega_{B}$ and $\\Omega_{C}$, respectively. Let $\\Omega_{B}$ cross $A R$ and $E F$ again at $U$ and $Y$, respectively. We have $$ \\angle(U B, B F)=\\angle(U P, P F)=\\angle(R P, P F)=\\angle(R F, F A) $$ so $U B \\| R F$. ![](https:\/\/cdn.mathpix.com\/cropped\/2024_11_18_9ee8c2ac2c2f5002ff14g-076.jpg?height=1009&width=1474&top_left_y=883&top_left_x=248) Next, we show that the points $B, I, U$, and $X$ are concyclic. Since $$ \\angle(U B, U X)=\\angle(R F, R X)=\\angle(A F, A R)+\\angle(F R, F A)=\\angle\\left(M^{\\prime} B, M^{\\prime} X\\right)+\\angle(D R, D F) $$ it suffices to prove $\\angle(I B, I X)=\\angle\\left(M^{\\prime} B, M^{\\prime} X\\right)+\\angle(D R, D F)$, or $\\angle\\left(I B, M^{\\prime} B\\right)=\\angle(D R, D F)$. But both angles equal $\\angle(C I, C B)$, as desired. (This is where we used the fact that $M^{\\prime}$ is the midpoint of $\\operatorname{arc} B A C$ of $\\Omega$.) It follows now from circles $B U I X$ and $B P U F Y$ that $$ \\begin{aligned} \\angle(I U, U B)=\\angle(I X, B X)=\\angle\\left(M^{\\prime} X, B X\\right)= & \\frac{\\pi-\\angle A}{2} \\\\ & =\\angle(E F, A F)=\\angle(Y F, B F)=\\angle(Y U, B U) \\end{aligned} $$ so the points $Y, U$, and $I$ are collinear. Let $E F$ meet $B C$ at $W$. We have $$ \\angle(I Y, Y W)=\\angle(U Y, F Y)=\\angle(U B, F B)=\\angle(R F, A F)=\\angle(C I, C W) $$ so the points $W, Y, I$, and $C$ are concyclic. Similarly, if $V$ and $Z$ are the second meeting points of $\\Omega_{C}$ with $A R$ and $E F$, we get that the 4-tuples $(C, V, I, X)$ and $(B, I, Z, W)$ are both concyclic. Step 3. Let $Q^{\\prime}=C Y \\cap B Z$. We will show that $Q^{\\prime}=Q$. First of all, we have $$ \\begin{aligned} & \\angle\\left(Q^{\\prime} Y, Q^{\\prime} B\\right)=\\angle(C Y, Z B)=\\angle(C Y, Z Y)+\\angle(Z Y, B Z) \\\\ & =\\angle(C I, I W)+\\angle(I W, I B)=\\angle(C I, I B)=\\frac{\\pi-\\angle A}{2}=\\angle(F Y, F B), \\end{aligned} $$ so $Q^{\\prime} \\in \\Omega_{B}$. Similarly, $Q^{\\prime} \\in \\Omega_{C}$. Thus $Q^{\\prime} \\in \\Omega_{B} \\cap \\Omega_{C}=\\{P, Q\\}$ and it remains to prove that $Q^{\\prime} \\neq P$. If we had $Q^{\\prime}=P$, we would have $\\angle(P Y, P Z)=\\angle\\left(Q^{\\prime} Y, Q^{\\prime} Z\\right)=\\angle(I C, I B)$. This would imply $$ \\angle(P Y, Y F)+\\angle(E Z, Z P)=\\angle(P Y, P Z)=\\angle(I C, I B)=\\angle(P E, P F), $$ so circles $\\Omega_{B}$ and $\\Omega_{C}$ would be tangent at $P$. That is excluded in the problem conditions, so $Q^{\\prime}=Q$. ![](https:\/\/cdn.mathpix.com\/cropped\/2024_11_18_9ee8c2ac2c2f5002ff14g-077.jpg?height=966&width=1468&top_left_y=1070&top_left_x=251) Step 4. Now we are ready to show that $P, Q$, and $S$ are collinear. Notice that $A$ and $D$ are the poles of $E W$ and $D W$ with respect to $\\omega$, so $W$ is the pole of $A D$. Hence, $W I \\perp A D$. Since $C I \\perp D E$, this yields $\\angle(I C, W I)=\\angle(D E, D A)$. On the other hand, $D A$ is a symmedian in $\\triangle D E F$, so $\\angle(D E, D A)=\\angle(D N, D F)=\\angle(D S, D F)$. Therefore, $$ \\begin{aligned} \\angle(P S, P F)=\\angle(D S, D F)=\\angle(D E, D A)= & \\angle(I C, I W) \\\\ & =\\angle(Y C, Y W)=\\angle(Y Q, Y F)=\\angle(P Q, P F), \\end{aligned} $$ which yields the desired collinearity.","tier":0} +{"problem_type":"Geometry","problem_label":"G8","problem":"Let $\\mathcal{L}$ be the set of all lines in the plane and let $f$ be a function that assigns to each line $\\ell \\in \\mathcal{L}$ a point $f(\\ell)$ on $\\ell$. Suppose that for any point $X$, and for any three lines $\\ell_{1}, \\ell_{2}, \\ell_{3}$ passing through $X$, the points $f\\left(\\ell_{1}\\right), f\\left(\\ell_{2}\\right), f\\left(\\ell_{3}\\right)$ and $X$ lie on a circle. Prove that there is a unique point $P$ such that $f(\\ell)=P$ for any line $\\ell$ passing through $P$. (Australia) Common remarks. The condition on $f$ is equivalent to the following: There is some function $g$ that assigns to each point $X$ a circle $g(X)$ passing through $X$ such that for any line $\\ell$ passing through $X$, the point $f(\\ell)$ lies on $g(X)$. (The function $g$ may not be uniquely defined for all points, if some points $X$ have at most one value of $f(\\ell)$ other than $X$; for such points, an arbitrary choice is made.) If there were two points $P$ and $Q$ with the given property, $f(P Q)$ would have to be both $P$ and $Q$, so there is at most one such point, and it will suffice to show that such a point exists.","solution":"We provide a complete characterisation of the functions satisfying the given condition. Write $\\angle\\left(\\ell_{1}, \\ell_{2}\\right)$ for the directed angle modulo $180^{\\circ}$ between the lines $\\ell_{1}$ and $\\ell_{2}$. Given a point $P$ and an angle $\\alpha \\in\\left(0,180^{\\circ}\\right)$, for each line $\\ell$, let $\\ell^{\\prime}$ be the line through $P$ satisfying $\\angle\\left(\\ell^{\\prime}, \\ell\\right)=\\alpha$, and let $h_{P, \\alpha}(\\ell)$ be the intersection point of $\\ell$ and $\\ell^{\\prime}$. We will prove that there is some pair $(P, \\alpha)$ such that $f$ and $h_{P, \\alpha}$ are the same function. Then $P$ is the unique point in the problem statement. Given an angle $\\alpha$ and a point $P$, let a line $\\ell$ be called $(P, \\alpha)$-good if $f(\\ell)=h_{P, \\alpha}(\\ell)$. Let a point $X \\neq P$ be called $(P, \\alpha)$-good if the circle $g(X)$ passes through $P$ and some point $Y \\neq P, X$ on $g(X)$ satisfies $\\angle(P Y, Y X)=\\alpha$. It follows from this definition that if $X$ is $(P, \\alpha)$ good then every point $Y \\neq P, X$ of $g(X)$ satisfies this angle condition, so $h_{P, \\alpha}(X Y)=Y$ for every $Y \\in g(X)$. Equivalently, $f(\\ell) \\in\\left\\{X, h_{P, \\alpha}(\\ell)\\right\\}$ for each line $\\ell$ passing through $X$. This shows the following lemma. Lemma 1. If $X$ is $(P, \\alpha)$-good and $\\ell$ is a line passing through $X$ then either $f(\\ell)=X$ or $\\ell$ is $(P, \\alpha)$-good. Lemma 2. If $X$ and $Y$ are different $(P, \\alpha)$-good points, then line $X Y$ is $(P, \\alpha)$-good. Proof. If $X Y$ is not $(P, \\alpha)$-good then by the previous Lemma, $f(X Y)=X$ and similarly $f(X Y)=Y$, but clearly this is impossible as $X \\neq Y$. Lemma 3. If $\\ell_{1}$ and $\\ell_{2}$ are different $(P, \\alpha)$-good lines which intersect at $X \\neq P$, then either $f\\left(\\ell_{1}\\right)=X$ or $f\\left(\\ell_{2}\\right)=X$ or $X$ is $(P, \\alpha)$-good. Proof. If $f\\left(\\ell_{1}\\right), f\\left(\\ell_{2}\\right) \\neq X$, then $g(X)$ is the circumcircle of $X, f\\left(\\ell_{1}\\right)$ and $f\\left(\\ell_{2}\\right)$. Since $\\ell_{1}$ and $\\ell_{2}$ are $(P, \\alpha)$-good lines, the angles $$ \\angle\\left(P f\\left(\\ell_{1}\\right), f\\left(\\ell_{1}\\right) X\\right)=\\angle\\left(P f\\left(\\ell_{2}\\right), f\\left(\\ell_{2}\\right) X\\right)=\\alpha $$ so $P$ lies on $g(X)$. Hence, $X$ is $(P, \\alpha)$-good. Lemma 4. If $\\ell_{1}, \\ell_{2}$ and $\\ell_{3}$ are different $(P, \\alpha)$ good lines which intersect at $X \\neq P$, then $X$ is $(P, \\alpha)$-good. Proof. This follows from the previous Lemma since at most one of the three lines $\\ell_{i}$ can satisfy $f\\left(\\ell_{i}\\right)=X$ as the three lines are all $(P, \\alpha)$-good. Lemma 5. If $A B C$ is a triangle such that $A, B, C, f(A B), f(A C)$ and $f(B C)$ are all different points, then there is some point $P$ and some angle $\\alpha$ such that $A, B$ and $C$ are $(P, \\alpha)$-good points and $A B, B C$ and $C A$ are $(P, \\alpha)-\\operatorname{good}$ lines. ![](https:\/\/cdn.mathpix.com\/cropped\/2024_11_18_9ee8c2ac2c2f5002ff14g-079.jpg?height=415&width=612&top_left_y=207&top_left_x=728) Proof. Let $D, E, F$ denote the points $f(B C), f(A C), f(A B)$, respectively. Then $g(A)$, $g(B)$ and $g(C)$ are the circumcircles of $A E F, B D F$ and $C D E$, respectively. Let $P \\neq F$ be the second intersection of circles $g(A)$ and $g(B)$ (or, if these circles are tangent at $F$, then $P=F$ ). By Miquel's theorem (or an easy angle chase), $g(C)$ also passes through $P$. Then by the cyclic quadrilaterals, the directed angles $$ \\angle(P D, D C)=\\angle(P F, F B)=\\angle(P E, E A)=\\alpha $$ for some angle $\\alpha$. Hence, lines $A B, B C$ and $C A$ are all $(P, \\alpha)$-good, so by Lemma $3, A, B$ and $C$ are $(P, \\alpha)$-good. (In the case where $P=D$, the line $P D$ in the equation above denotes the line which is tangent to $g(B)$ at $P=D$. Similar definitions are used for $P E$ and $P F$ in the cases where $P=E$ or $P=F$.) Consider the set $\\Omega$ of all points $(x, y)$ with integer coordinates $1 \\leqslant x, y \\leqslant 1000$, and consider the set $L_{\\Omega}$ of all horizontal, vertical and diagonal lines passing through at least one point in $\\Omega$. A simple counting argument shows that there are 5998 lines in $L_{\\Omega}$. For each line $\\ell$ in $L_{\\Omega}$ we colour the point $f(\\ell)$ red. Then there are at most 5998 red points. Now we partition the points in $\\Omega$ into 10000 ten by ten squares. Since there are at most 5998 red points, at least one of these squares $\\Omega_{10}$ contains no red points. Let $(m, n)$ be the bottom left point in $\\Omega_{10}$. Then the triangle with vertices $(m, n),(m+1, n)$ and $(m, n+1)$ satisfies the condition of Lemma 5 , so these three vertices are all $(P, \\alpha)$-good for some point $P$ and angle $\\alpha$, as are the lines joining them. From this point on, we will simply call a point or line good if it is $(P, \\alpha)$-good for this particular pair $(P, \\alpha)$. Now by Lemma 1, the line $x=m+1$ is good, as is the line $y=n+1$. Then Lemma 3 implies that $(m+1, n+1)$ is good. By applying these two lemmas repeatedly, we can prove that the line $x+y=m+n+2$ is good, then the points $(m, n+2)$ and $(m+2, n)$ then the lines $x=m+2$ and $y=n+2$, then the points $(m+2, n+1),(m+1, n+2)$ and $(m+2, n+2)$ and so on until we have prove that all points in $\\Omega_{10}$ are good. Now we will use this to prove that every point $S \\neq P$ is good. Since $g(S)$ is a circle, it passes through at most two points of $\\Omega_{10}$ on any vertical line, so at most 20 points in total. Moreover, any line $\\ell$ through $S$ intersects at most 10 points in $\\Omega_{10}$. Hence, there are at least eight lines $\\ell$ through $S$ which contain a point $Q$ in $\\Omega_{10}$ which is not on $g(S)$. Since $Q$ is not on $g(S)$, the point $f(\\ell) \\neq Q$. Hence, by Lemma 1 , the line $\\ell$ is good. Hence, at least eight good lines pass through $S$, so by Lemma 4, the point $S$ is good. Hence, every point $S \\neq P$ is good, so by Lemma 2 , every line is good. In particular, every line $\\ell$ passing through $P$ is good, and therefore satisfies $f(\\ell)=P$, as required.","tier":0} +{"problem_type":"Geometry","problem_label":"G8","problem":"Let $\\mathcal{L}$ be the set of all lines in the plane and let $f$ be a function that assigns to each line $\\ell \\in \\mathcal{L}$ a point $f(\\ell)$ on $\\ell$. Suppose that for any point $X$, and for any three lines $\\ell_{1}, \\ell_{2}, \\ell_{3}$ passing through $X$, the points $f\\left(\\ell_{1}\\right), f\\left(\\ell_{2}\\right), f\\left(\\ell_{3}\\right)$ and $X$ lie on a circle. Prove that there is a unique point $P$ such that $f(\\ell)=P$ for any line $\\ell$ passing through $P$. (Australia) Common remarks. The condition on $f$ is equivalent to the following: There is some function $g$ that assigns to each point $X$ a circle $g(X)$ passing through $X$ such that for any line $\\ell$ passing through $X$, the point $f(\\ell)$ lies on $g(X)$. (The function $g$ may not be uniquely defined for all points, if some points $X$ have at most one value of $f(\\ell)$ other than $X$; for such points, an arbitrary choice is made.) If there were two points $P$ and $Q$ with the given property, $f(P Q)$ would have to be both $P$ and $Q$, so there is at most one such point, and it will suffice to show that such a point exists.","solution":"Note that for any distinct points $X, Y$, the circles $g(X)$ and $g(Y)$ meet on $X Y$ at the point $f(X Y) \\in g(X) \\cap g(Y) \\cap(X Y)$. We write $s(X, Y)$ for the second intersection point of circles $g(X)$ and $g(Y)$. Lemma 1. Suppose that $X, Y$ and $Z$ are not collinear, and that $f(X Y) \\notin\\{X, Y\\}$ and similarly for $Y Z$ and $Z X$. Then $s(X, Y)=s(Y, Z)=s(Z, X)$. Proof. The circles $g(X), g(Y)$ and $g(Z)$ through the vertices of triangle $X Y Z$ meet pairwise on the corresponding edges (produced). By Miquel's theorem, the second points of intersection of any two of the circles coincide. (See the diagram for Lemma 5 of Solution 1.) Now pick any line $\\ell$ and any six different points $Y_{1}, \\ldots, Y_{6}$ on $\\ell \\backslash\\{f(\\ell)\\}$. Pick a point $X$ not on $\\ell$ or any of the circles $g\\left(Y_{i}\\right)$. Reordering the indices if necessary, we may suppose that $Y_{1}, \\ldots, Y_{4}$ do not lie on $g(X)$, so that $f\\left(X Y_{i}\\right) \\notin\\left\\{X, Y_{i}\\right\\}$ for $1 \\leqslant i \\leqslant 4$. By applying the above lemma to triangles $X Y_{i} Y_{j}$ for $1 \\leqslant i0$, there is a point $P_{\\epsilon}$ with $g\\left(P_{\\epsilon}\\right)$ of radius at most $\\epsilon$. Then there is a point $P$ with the given property. Proof. Consider a sequence $\\epsilon_{i}=2^{-i}$ and corresponding points $P_{\\epsilon_{i}}$. Because the two circles $g\\left(P_{\\epsilon_{i}}\\right)$ and $g\\left(P_{\\epsilon_{j}}\\right)$ meet, the distance between $P_{\\epsilon_{i}}$ and $P_{\\epsilon_{j}}$ is at most $2^{1-i}+2^{1-j}$. As $\\sum_{i} \\epsilon_{i}$ converges, these points converge to some point $P$. For all $\\epsilon>0$, the point $P$ has distance at most $2 \\epsilon$ from $P_{\\epsilon}$, and all circles $g(X)$ pass through a point with distance at most $2 \\epsilon$ from $P_{\\epsilon}$, so distance at most $4 \\epsilon$ from $P$. A circle that passes distance at most $4 \\epsilon$ from $P$ for all $\\epsilon>0$ must pass through $P$, so by Lemma 1 the point $P$ has the given property. Lemma 3. Suppose no two of the circles $g(X)$ cut. Then there is a point $P$ with the given property. Proof. Consider a circle $g(X)$ with centre $Y$. The circle $g(Y)$ must meet $g(X)$ without cutting it, so has half the radius of $g(X)$. Repeating this argument, there are circles with arbitrarily small radius and the result follows by Lemma 2. Lemma 4. Suppose there are six different points $A, B_{1}, B_{2}, B_{3}, B_{4}, B_{5}$ such that no three are collinear, no four are concyclic, and all the circles $g\\left(B_{i}\\right)$ cut pairwise at $A$. Then there is a point $P$ with the given property. Proof. Consider some line $\\ell$ through $A$ that does not pass through any of the $B_{i}$ and is not tangent to any of the $g\\left(B_{i}\\right)$. Fix some direction along that line, and let $X_{\\epsilon}$ be the point on $\\ell$ that has distance $\\epsilon$ from $A$ in that direction. In what follows we consider only those $\\epsilon$ for which $X_{\\epsilon}$ does not lie on any $g\\left(B_{i}\\right)$ (this restriction excludes only finitely many possible values of $\\epsilon$ ). Consider the circle $g\\left(X_{\\epsilon}\\right)$. Because no four of the $B_{i}$ are concyclic, at most three of them lie on this circle, so at least two of them do not. There must be some sequence of $\\epsilon \\rightarrow 0$ such that it is the same two of the $B_{i}$ for all $\\epsilon$ in that sequence, so now restrict attention to that sequence, and suppose without loss of generality that $B_{1}$ and $B_{2}$ do not lie on $g\\left(X_{\\epsilon}\\right)$ for any $\\epsilon$ in that sequence. Then $f\\left(X_{\\epsilon} B_{1}\\right)$ is not $B_{1}$, so must be the other point of intersection of $X_{\\epsilon} B_{1}$ with $g\\left(B_{1}\\right)$, and the same applies with $B_{2}$. Now consider the three points $X_{\\epsilon}, f\\left(X_{\\epsilon} B_{1}\\right)$ and $f\\left(X_{\\epsilon} B_{2}\\right)$. As $\\epsilon \\rightarrow 0$, the angle at $X_{\\epsilon}$ tends to $\\angle B_{1} A B_{2}$ or $180^{\\circ}-\\angle B_{1} A B_{2}$, which is not 0 or $180^{\\circ}$ because no three of the points were collinear. All three distances between those points are bounded above by constant multiples of $\\epsilon$ (in fact, if the triangle is scaled by a factor of $1 \/ \\epsilon$, it tends to a fixed triangle). Thus the circumradius of those three points, which is the radius of $g\\left(X_{\\epsilon}\\right)$, is also bounded above by a constant multiple of $\\epsilon$, and so the result follows by Lemma 2 . Lemma 5. Suppose there are two points $A$ and $B$ such that $g(A)$ and $g(B)$ cut. Then there is a point $P$ with the given property. Proof. Suppose that $g(A)$ and $g(B)$ cut at $C$ and $D$. One of those points, without loss of generality $C$, must be $f(A B)$, and so lie on the line $A B$. We now consider two cases, according to whether $D$ also lies on that line.","tier":0} +{"problem_type":"Geometry","problem_label":"G8","problem":"Let $\\mathcal{L}$ be the set of all lines in the plane and let $f$ be a function that assigns to each line $\\ell \\in \\mathcal{L}$ a point $f(\\ell)$ on $\\ell$. Suppose that for any point $X$, and for any three lines $\\ell_{1}, \\ell_{2}, \\ell_{3}$ passing through $X$, the points $f\\left(\\ell_{1}\\right), f\\left(\\ell_{2}\\right), f\\left(\\ell_{3}\\right)$ and $X$ lie on a circle. Prove that there is a unique point $P$ such that $f(\\ell)=P$ for any line $\\ell$ passing through $P$. (Australia) Common remarks. The condition on $f$ is equivalent to the following: There is some function $g$ that assigns to each point $X$ a circle $g(X)$ passing through $X$ such that for any line $\\ell$ passing through $X$, the point $f(\\ell)$ lies on $g(X)$. (The function $g$ may not be uniquely defined for all points, if some points $X$ have at most one value of $f(\\ell)$ other than $X$; for such points, an arbitrary choice is made.) If there were two points $P$ and $Q$ with the given property, $f(P Q)$ would have to be both $P$ and $Q$, so there is at most one such point, and it will suffice to show that such a point exists.","solution":"For any point $X$, denote by $t(X)$ the line tangent to $g(X)$ at $X$; notice that $f(t(X))=X$, so $f$ is surjective. Step 1: We find a point $P$ for which there are at least two different lines $p_{1}$ and $p_{2}$ such that $f\\left(p_{i}\\right)=P$. Choose any point $X$. If $X$ does not have this property, take any $Y \\in g(X) \\backslash\\{X\\}$; then $f(X Y)=Y$. If $Y$ does not have the property, $t(Y)=X Y$, and the circles $g(X)$ and $g(Y)$ meet again at some point $Z$. Then $f(X Z)=Z=f(Y Z)$, so $Z$ has the required property. We will show that $P$ is the desired point. From now on, we fix two different lines $p_{1}$ and $p_{2}$ with $f\\left(p_{1}\\right)=f\\left(p_{2}\\right)=P$. Assume for contradiction that $f(\\ell)=Q \\neq P$ for some line $\\ell$ through $P$. We fix $\\ell$, and note that $Q \\in g(P)$. Step 2: We prove that $P \\in g(Q)$. Take an arbitrary point $X \\in \\ell \\backslash\\{P, Q\\}$. Two cases are possible for the position of $t(X)$ in relation to the $p_{i}$; we will show that each case (and subcase) occurs for only finitely many positions of $X$, yielding a contradiction. Case 2.1: $t(X)$ is parallel to one of the $p_{i}$; say, to $p_{1}$. Let $t(X) \\operatorname{cross} p_{2}$ at $R$. Then $g(R)$ is the circle $(P R X)$, as $f(R P)=P$ and $f(R X)=X$. Let $R Q$ cross $g(R)$ again at $S$. Then $f(R Q) \\in\\{R, S\\} \\cap g(Q)$, so $g(Q)$ contains one of the points $R$ and $S$. If $R \\in g(Q)$, then $R$ is one of finitely many points in the intersection $g(Q) \\cap p_{2}$, and each of them corresponds to a unique position of $X$, since $R X$ is parallel to $p_{1}$. If $S \\in g(Q)$, then $\\angle(Q S, S P)=\\angle(R S, S P)=\\angle(R X, X P)=\\angle\\left(p_{1}, \\ell\\right)$, so $\\angle(Q S, S P)$ is constant for all such points $X$, and all points $S$ obtained in such a way lie on one circle $\\gamma$ passing through $P$ and $Q$. Since $g(Q)$ does not contain $P$, it is different from $\\gamma$, so there are only finitely many points $S$. Each of them uniquely determines $R$ and thus $X$. ![](https:\/\/cdn.mathpix.com\/cropped\/2024_11_18_9ee8c2ac2c2f5002ff14g-083.jpg?height=843&width=909&top_left_y=207&top_left_x=568) So, Case 2.1 can occur for only finitely many points $X$. Case 2.2: $t(X)$ crosses $p_{1}$ and $p_{2}$ at $R_{1}$ and $R_{2}$, respectively. Clearly, $R_{1} \\neq R_{2}$, as $t(X)$ is the tangent to $g(X)$ at $X$, and $g(X)$ meets $\\ell$ only at $X$ and $Q$. Notice that $g\\left(R_{i}\\right)$ is the circle $\\left(P X R_{i}\\right)$. Let $R_{i} Q$ meet $g\\left(R_{i}\\right)$ again at $S_{i}$; then $S_{i} \\neq Q$, as $g\\left(R_{i}\\right)$ meets $\\ell$ only at $P$ and $X$. Then $f\\left(R_{i} Q\\right) \\in\\left\\{R_{i}, S_{i}\\right\\}$, and we distinguish several subcases. ![](https:\/\/cdn.mathpix.com\/cropped\/2024_11_18_9ee8c2ac2c2f5002ff14g-083.jpg?height=824&width=866&top_left_y=1501&top_left_x=595) Subcase 2.2.1: $f\\left(R_{1} Q\\right)=S_{1}, f\\left(R_{2} Q\\right)=S_{2}$; so $S_{1}, S_{2} \\in g(Q)$. In this case we have $0=\\angle\\left(R_{1} X, X P\\right)+\\angle\\left(X P, R_{2} X\\right)=\\angle\\left(R_{1} S_{1}, S_{1} P\\right)+\\angle\\left(S_{2} P, S_{2} R_{2}\\right)=$ $\\angle\\left(Q S_{1}, S_{1} P\\right)+\\angle\\left(S_{2} P, S_{2} Q\\right)$, which shows $P \\in g(Q)$. Subcase 2.2.2: $f\\left(R_{1} Q\\right)=R_{1}, f\\left(R_{2} Q\\right)=R_{2}$; so $R_{1}, R_{2} \\in g(Q)$. This can happen for at most four positions of $X$ - namely, at the intersections of $\\ell$ with a line of the form $K_{1} K_{2}$, where $K_{i} \\in g(Q) \\cap p_{i}$. Subcase 2.2.3: $f\\left(R_{1} Q\\right)=S_{1}, f\\left(R_{2} Q\\right)=R_{2}$ (the case $f\\left(R_{1} Q\\right)=R_{1}, f\\left(R_{2} Q\\right)=S_{2}$ is similar). In this case, there are at most two possible positions for $R_{2}$ - namely, the meeting points of $g(Q)$ with $p_{2}$. Consider one of them. Let $X$ vary on $\\ell$. Then $R_{1}$ is the projection of $X$ to $p_{1}$ via $R_{2}, S_{1}$ is the projection of $R_{1}$ to $g(Q)$ via $Q$. Finally, $\\angle\\left(Q S_{1}, S_{1} X\\right)=\\angle\\left(R_{1} S_{1}, S_{1} X\\right)=$ $\\angle\\left(R_{1} P, P X\\right)=\\angle\\left(p_{1}, \\ell\\right) \\neq 0$, so $X$ is obtained by a fixed projective transform $g(Q) \\rightarrow \\ell$ from $S_{1}$. So, if there were three points $X$ satisfying the conditions of this subcase, the composition of the three projective transforms would be the identity. But, if we apply it to $X=Q$, we successively get some point $R_{1}^{\\prime}$, then $R_{2}$, and then some point different from $Q$, a contradiction. Thus Case 2.2 also occurs for only finitely many points $X$, as desired. Step 3: We show that $f(P Q)=P$, as desired. The argument is similar to that in Step 2, with the roles of $Q$ and $X$ swapped. Again, we show that there are only finitely many possible positions for a point $X \\in \\ell \\backslash\\{P, Q\\}$, which is absurd. Case 3.1: $t(Q)$ is parallel to one of the $p_{i}$; say, to $p_{1}$. Let $t(Q)$ cross $p_{2}$ at $R$; then $g(R)$ is the circle $(P R Q)$. Let $R X \\operatorname{cross} g(R)$ again at $S$. Then $f(R X) \\in\\{R, S\\} \\cap g(X)$, so $g(X)$ contains one of the points $R$ and $S$. ![](https:\/\/cdn.mathpix.com\/cropped\/2024_11_18_9ee8c2ac2c2f5002ff14g-084.jpg?height=729&width=909&top_left_y=1126&top_left_x=568) Subcase 3.1.1: $S=f(R X) \\in g(X)$. We have $\\angle(t(X), Q X)=\\angle(S X, S Q)=\\angle(S R, S Q)=\\angle(P R, P Q)=\\angle\\left(p_{2}, \\ell\\right)$. Hence $t(X) \\| p_{2}$. Now we recall Case 2.1: we let $t(X)$ cross $p_{1}$ at $R^{\\prime}$, so $g\\left(R^{\\prime}\\right)=\\left(P R^{\\prime} X\\right)$, and let $R^{\\prime} Q$ meet $g\\left(R^{\\prime}\\right)$ again at $S^{\\prime}$; notice that $S^{\\prime} \\neq Q$. Excluding one position of $X$, we may assume that $R^{\\prime} \\notin g(Q)$, so $R^{\\prime} \\neq f\\left(R^{\\prime} Q\\right)$. Therefore, $S^{\\prime}=f\\left(R^{\\prime} Q\\right) \\in g(Q)$. But then, as in Case 2.1, we get $\\angle(t(Q), P Q)=\\angle\\left(Q S^{\\prime}, S^{\\prime} P\\right)=\\angle\\left(R^{\\prime} X, X P\\right)=\\angle\\left(p_{2}, \\ell\\right)$. This means that $t(Q)$ is parallel to $p_{2}$, which is impossible. Subcase 3.1.2: $R=f(R X) \\in g(X)$. In this case, we have $\\angle(t(X), \\ell)=\\angle(R X, R Q)=\\angle\\left(R X, p_{1}\\right)$. Again, let $R^{\\prime}=t(X) \\cap p_{1}$; this point exists for all but at most one position of $X$. Then $g\\left(R^{\\prime}\\right)=\\left(R^{\\prime} X P\\right)$; let $R^{\\prime} Q$ meet $g\\left(R^{\\prime}\\right)$ again at $S^{\\prime}$. Due to $\\angle\\left(R^{\\prime} X, X R\\right)=\\angle(Q X, Q R)=\\angle\\left(\\ell, p_{1}\\right), R^{\\prime}$ determines $X$ in at most two ways, so for all but finitely many positions of $X$ we have $R^{\\prime} \\notin g(Q)$. Therefore, for those positions we have $S^{\\prime}=f\\left(R^{\\prime} Q\\right) \\in g(Q)$. But then $\\angle\\left(R X, p_{1}\\right)=\\angle\\left(R^{\\prime} X, X P\\right)=\\angle\\left(R^{\\prime} S^{\\prime}, S^{\\prime} P\\right)=$ $\\angle\\left(Q S^{\\prime}, S^{\\prime} P\\right)=\\angle(t(Q), Q P)$ is fixed, so this case can hold only for one specific position of $X$ as well. Thus, in Case 3.1, there are only finitely many possible positions of $X$, yielding a contradiction. Case 3.2: $t(Q)$ crosses $p_{1}$ and $p_{2}$ at $R_{1}$ and $R_{2}$, respectively. By Step $2, R_{1} \\neq R_{2}$. Notice that $g\\left(R_{i}\\right)$ is the circle $\\left(P Q R_{i}\\right)$. Let $R_{i} X$ meet $g\\left(R_{i}\\right)$ at $S_{i}$; then $S_{i} \\neq X$. Then $f\\left(R_{i} X\\right) \\in\\left\\{R_{i}, S_{i}\\right\\}$, and we distinguish several subcases. ![](https:\/\/cdn.mathpix.com\/cropped\/2024_11_18_9ee8c2ac2c2f5002ff14g-085.jpg?height=854&width=866&top_left_y=424&top_left_x=589) Subcase 3.2.1: $f\\left(R_{1} X\\right)=S_{1}$ and $f\\left(R_{2} X\\right)=S_{2}$, so $S_{1}, S_{2} \\in g(X)$. As in Subcase 2.2.1, we have $0=\\angle\\left(R_{1} Q, Q P\\right)+\\angle\\left(Q P, R_{2} Q\\right)=\\angle\\left(X S_{1}, S_{1} P\\right)+\\angle\\left(S_{2} P, S_{2} X\\right)$, which shows $P \\in g(X)$. But $X, Q \\in g(X)$ as well, so $g(X)$ meets $\\ell$ at three distinct points, which is absurd. Subcase 3.2.2: $f\\left(R_{1} X\\right)=R_{1}, f\\left(R_{2} X\\right)=R_{2}$, so $R_{1}, R_{2} \\in g(X)$. Now three distinct collinear points $R_{1}, R_{2}$, and $Q$ belong to $g(X)$, which is impossible. Subcase 3.2.3: $f\\left(R_{1} X\\right)=S_{1}, f\\left(R_{2} X\\right)=R_{2}$ (the case $f\\left(R_{1} X\\right)=R_{1}, f\\left(R_{2} X\\right)=S_{2}$ is similar). We have $\\angle\\left(X R_{2}, R_{2} Q\\right)=\\angle\\left(X S_{1}, S_{1} Q\\right)=\\angle\\left(R_{1} S_{1}, S_{1} Q\\right)=\\angle\\left(R_{1} P, P Q\\right)=\\angle\\left(p_{1}, \\ell\\right)$, so this case can occur for a unique position of $X$. Thus, in Case 3.2 , there is only a unique position of $X$, again yielding the required contradiction.","tier":0} +{"problem_type":"Geometry","problem_label":"G8","problem":"Let $\\mathcal{L}$ be the set of all lines in the plane and let $f$ be a function that assigns to each line $\\ell \\in \\mathcal{L}$ a point $f(\\ell)$ on $\\ell$. Suppose that for any point $X$, and for any three lines $\\ell_{1}, \\ell_{2}, \\ell_{3}$ passing through $X$, the points $f\\left(\\ell_{1}\\right), f\\left(\\ell_{2}\\right), f\\left(\\ell_{3}\\right)$ and $X$ lie on a circle. Prove that there is a unique point $P$ such that $f(\\ell)=P$ for any line $\\ell$ passing through $P$. (Australia) Common remarks. The condition on $f$ is equivalent to the following: There is some function $g$ that assigns to each point $X$ a circle $g(X)$ passing through $X$ such that for any line $\\ell$ passing through $X$, the point $f(\\ell)$ lies on $g(X)$. (The function $g$ may not be uniquely defined for all points, if some points $X$ have at most one value of $f(\\ell)$ other than $X$; for such points, an arbitrary choice is made.) If there were two points $P$ and $Q$ with the given property, $f(P Q)$ would have to be both $P$ and $Q$, so there is at most one such point, and it will suffice to show that such a point exists.","solution":"We will get an upper bound on $n$ from the speed at which $v_{2}\\left(L_{n}\\right)$ grows. From $$ L_{n}=\\left(2^{n}-1\\right)\\left(2^{n}-2\\right) \\cdots\\left(2^{n}-2^{n-1}\\right)=2^{1+2+\\cdots+(n-1)}\\left(2^{n}-1\\right)\\left(2^{n-1}-1\\right) \\cdots\\left(2^{1}-1\\right) $$ we read $$ v_{2}\\left(L_{n}\\right)=1+2+\\cdots+(n-1)=\\frac{n(n-1)}{2} . $$ On the other hand, $v_{2}(m!)$ is expressed by the Legendre formula as $$ v_{2}(m!)=\\sum_{i=1}^{\\infty}\\left\\lfloor\\frac{m}{2^{i}}\\right\\rfloor . $$ As usual, by omitting the floor functions, $$ v_{2}(m!)<\\sum_{i=1}^{\\infty} \\frac{m}{2^{i}}=m $$ Thus, $L_{n}=m$ ! implies the inequality $$ \\frac{n(n-1)}{2}1.3 \\cdot 10^{12}$. For $n \\geqslant 7$ we prove (3) by the following inequalities: $$ \\begin{aligned} \\left(\\frac{n(n-1)}{2}\\right)! & =15!\\cdot 16 \\cdot 17 \\cdots \\frac{n(n-1)}{2}>2^{36} \\cdot 16^{\\frac{n(n-1)}{2}-15} \\\\ & =2^{2 n(n-1)-24}=2^{n^{2}} \\cdot 2^{n(n-2)-24}>2^{n^{2}} . \\end{aligned} $$ Putting together (2) and (3), for $n \\geqslant 6$ we get a contradiction, since $$ L_{n}<2^{n^{2}}<\\left(\\frac{n(n-1)}{2}\\right)!a^{3} . $$ So $3 a^{3} \\geqslant(a b c)^{2}>a^{3}$ and hence $3 a \\geqslant b^{2} c^{2}>a$. Now $b^{3}+c^{3}=a^{2}\\left(b^{2} c^{2}-a\\right) \\geqslant a^{2}$, and so $$ 18 b^{3} \\geqslant 9\\left(b^{3}+c^{3}\\right) \\geqslant 9 a^{2} \\geqslant b^{4} c^{4} \\geqslant b^{3} c^{5} $$ so $18 \\geqslant c^{5}$ which yields $c=1$. Now, note that we must have $a>b$, as otherwise we would have $2 b^{3}+1=b^{4}$ which has no positive integer solutions. So $$ a^{3}-b^{3} \\geqslant(b+1)^{3}-b^{3}>1 $$ and $$ 2 a^{3}>1+a^{3}+b^{3}>a^{3} $$ which implies $2 a^{3}>a^{2} b^{2}>a^{3}$ and so $2 a>b^{2}>a$. Therefore $$ 4\\left(1+b^{3}\\right)=4 a^{2}\\left(b^{2}-a\\right) \\geqslant 4 a^{2}>b^{4} $$ so $4>b^{3}(b-4)$; that is, $b \\leqslant 4$. Now, for each possible value of $b$ with $2 \\leqslant b \\leqslant 4$ we obtain a cubic equation for $a$ with constant coefficients. These are as follows: $$ \\begin{array}{rlrl} b=2: & & & a^{3}-4 a^{2}+9=0 \\\\ b=3: & & a^{3}-9 a^{2}+28=0 \\\\ b=4: & & a^{3}-16 a^{2}+65=0 . \\end{array} $$ The only case with an integer solution for $a$ with $b \\leqslant a$ is $b=2$, leading to $(a, b, c)=(3,2,1)$. Comment 1.1. Instead of writing down each cubic equation explicitly, we could have just observed that $a^{2} \\mid b^{3}+1$, and for each choice of $b$ checked each square factor of $b^{3}+1$ for $a^{2}$. We could also have observed that, with $c=1$, the relation $18 b^{3} \\geqslant b^{4} c^{4}$ becomes $b \\leqslant 18$, and we can simply check all possibilities for $b$ (instead of working to prove that $b \\leqslant 4$ ). This check becomes easier after using the factorisation $b^{3}+1=(b+1)\\left(b^{2}-b+1\\right)$ and observing that no prime besides 3 can divide both of the factors. Comment 1.2. Another approach to finish the problem after establishing that $c \\leqslant 1$ is to set $k=b^{2} c^{2}-a$, which is clearly an integer and must be positive as it is equal to $\\left(b^{3}+c^{3}\\right) \/ a^{2}$. Then we divide into cases based on whether $k=1$ or $k \\geqslant 2$; in the first case, we have $b^{3}+1=a^{2}=\\left(b^{2}-1\\right)^{2}$ whose only positive root is $b=2$, and in the second case we have $b^{2} \\leqslant 3 a$, and so $$ b^{4} \\leqslant(3 a)^{2} \\leqslant \\frac{9}{2}\\left(k a^{2}\\right)=\\frac{9}{2}\\left(b^{3}+1\\right), $$ which implies that $b \\leqslant 4$.","tier":0} +{"problem_type":"Number Theory","problem_label":"N2","problem":"Find all triples $(a, b, c)$ of positive integers such that $a^{3}+b^{3}+c^{3}=(a b c)^{2}$. (Nigeria) Answer: The solutions are $(1,2,3)$ and its permutations. Common remarks. Note that the equation is symmetric. In all solutions, we will assume without loss of generality that $a \\geqslant b \\geqslant c$, and prove that the only solution is $(a, b, c)=(3,2,1)$. The first two solutions all start by proving that $c=1$.","solution":"Again, we will start by proving that $c=1$. Suppose otherwise that $c \\geqslant 2$. We have $a^{3}+b^{3}+c^{3} \\leqslant 3 a^{3}$, so $b^{2} c^{2} \\leqslant 3 a$. Since $c \\geqslant 2$, this tells us that $b \\leqslant \\sqrt{3 a \/ 4}$. As the right-hand side of the original equation is a multiple of $a^{2}$, we have $a^{2} \\leqslant 2 b^{3} \\leqslant 2(3 a \/ 4)^{3 \/ 2}$. In other words, $a \\leqslant \\frac{27}{16}<2$, which contradicts the assertion that $a \\geqslant c \\geqslant 2$. So there are no solutions in this case, and so we must have $c=1$. Now, the original equation becomes $a^{3}+b^{3}+1=a^{2} b^{2}$. Observe that $a \\geqslant 2$, since otherwise $a=b=1$ as $a \\geqslant b$. The right-hand side is a multiple of $a^{2}$, so the left-hand side must be as well. Thus, $b^{3}+1 \\geqslant$ $a^{2}$. Since $a \\geqslant b$, we also have $$ b^{2}=a+\\frac{b^{3}+1}{a^{2}} \\leqslant 2 a+\\frac{1}{a^{2}} $$ and so $b^{2} \\leqslant 2 a$ since $b^{2}$ is an integer. Thus $(2 a)^{3 \/ 2}+1 \\geqslant b^{3}+1 \\geqslant a^{2}$, from which we deduce $a \\leqslant 8$. Now, for each possible value of $a$ with $2 \\leqslant a \\leqslant 8$ we obtain a cubic equation for $b$ with constant coefficients. These are as follows: $$ \\begin{array}{ll} a=2: & \\\\ b^{3}-4 b^{2}+9=0 \\\\ a=3: & \\\\ b^{3}-9 b^{2}+28=0 \\\\ a=4: & \\\\ a=5: 16 b^{2}+65=0 \\\\ a=6: & \\\\ b^{3}-25 b^{2}+126=0 \\\\ a=7: & \\\\ b^{3}-36 b^{2}+217=0 \\\\ a=8: & \\\\ b^{3}-49 b^{2}+344=0 \\\\ b^{3}-64 b^{2}+513=0 . \\end{array} $$ The only case with an integer solution for $b$ with $a \\geqslant b$ is $a=3$, leading to $(a, b, c)=(3,2,1)$. Comment 2.1. As in Solution 1, instead of writing down each cubic equation explicitly, we could have just observed that $b^{2} \\mid a^{3}+1$, and for each choice of $a$ checked each square factor of $a^{3}+1$ for $b^{2}$. Comment 2.2. This solution does not require initially proving that $c=1$, in which case the bound would become $a \\leqslant 108$. The resulting cases could, in principle, be checked by a particularly industrious student.","tier":0} +{"problem_type":"Number Theory","problem_label":"N2","problem":"Find all triples $(a, b, c)$ of positive integers such that $a^{3}+b^{3}+c^{3}=(a b c)^{2}$. (Nigeria) Answer: The solutions are $(1,2,3)$ and its permutations. Common remarks. Note that the equation is symmetric. In all solutions, we will assume without loss of generality that $a \\geqslant b \\geqslant c$, and prove that the only solution is $(a, b, c)=(3,2,1)$. The first two solutions all start by proving that $c=1$.","solution":"Set $k=\\left(b^{3}+c^{3}\\right) \/ a^{2} \\leqslant 2 a$, and rewrite the original equation as $a+k=(b c)^{2}$. Since $b^{3}$ and $c^{3}$ are positive integers, we have $(b c)^{3} \\geqslant b^{3}+c^{3}-1=k a^{2}-1$, so $$ a+k \\geqslant\\left(k a^{2}-1\\right)^{2 \/ 3} $$ As in Comment 1.2, $k$ is a positive integer; for each value of $k \\geqslant 1$, this gives us a polynomial inequality satisfied by $a$ : $$ k^{2} a^{4}-a^{3}-5 k a^{2}-3 k^{2} a-\\left(k^{3}-1\\right) \\leqslant 0 . $$ We now prove that $a \\leqslant 3$. Indeed, $$ 0 \\geqslant \\frac{k^{2} a^{4}-a^{3}-5 k a^{2}-3 k^{2} a-\\left(k^{3}-1\\right)}{k^{2}} \\geqslant a^{4}-a^{3}-5 a^{2}-3 a-k \\geqslant a^{4}-a^{3}-5 a^{2}-5 a $$ which fails when $a \\geqslant 4$. This leaves ten triples with $3 \\geqslant a \\geqslant b \\geqslant c \\geqslant 1$, which may be checked manually to give $(a, b, c)=(3,2,1)$.","tier":0} +{"problem_type":"Number Theory","problem_label":"N2","problem":"Find all triples $(a, b, c)$ of positive integers such that $a^{3}+b^{3}+c^{3}=(a b c)^{2}$. (Nigeria) Answer: The solutions are $(1,2,3)$ and its permutations. Common remarks. Note that the equation is symmetric. In all solutions, we will assume without loss of generality that $a \\geqslant b \\geqslant c$, and prove that the only solution is $(a, b, c)=(3,2,1)$. The first two solutions all start by proving that $c=1$.","solution":"Again, observe that $b^{3}+c^{3}=a^{2}\\left(b^{2} c^{2}-a\\right)$, so $b \\leqslant a \\leqslant b^{2} c^{2}-1$. We consider the function $f(x)=x^{2}\\left(b^{2} c^{2}-x\\right)$. It can be seen that that on the interval $\\left[0, b^{2} c^{2}-1\\right]$ the function $f$ is increasing if $x<\\frac{2}{3} b^{2} c^{2}$ and decreasing if $x>\\frac{2}{3} b^{2} c^{2}$. Consequently, it must be the case that $$ b^{3}+c^{3}=f(a) \\geqslant \\min \\left(f(b), f\\left(b^{2} c^{2}-1\\right)\\right) $$ First, suppose that $b^{3}+c^{3} \\geqslant f\\left(b^{2} c^{2}-1\\right)$. This may be written $b^{3}+c^{3} \\geqslant\\left(b^{2} c^{2}-1\\right)^{2}$, and so $$ 2 b^{3} \\geqslant b^{3}+c^{3} \\geqslant\\left(b^{2} c^{2}-1\\right)^{2}>b^{4} c^{4}-2 b^{2} c^{2} \\geqslant b^{4} c^{4}-2 b^{3} c^{4} $$ Thus, $(b-2) c^{4}<2$, and the only solutions to this inequality have $(b, c)=(2,2)$ or $b \\leqslant 3$ and $c=1$. It is easy to verify that the only case giving a solution for $a \\geqslant b$ is $(a, b, c)=(3,2,1)$. Otherwise, suppose that $b^{3}+c^{3}=f(a) \\geqslant f(b)$. Then, we have $$ 2 b^{3} \\geqslant b^{3}+c^{3}=a^{2}\\left(b^{2} c^{2}-a\\right) \\geqslant b^{2}\\left(b^{2} c^{2}-b\\right) . $$ Consequently $b c^{2} \\leqslant 3$, with strict inequality in the case that $b \\neq c$. Hence $c=1$ and $b \\leqslant 2$. Both of these cases have been considered already, so we are done. Comment 4.1. Instead of considering which of $f(b)$ and $f\\left(b^{2} c^{2}-1\\right)$ is less than $f(a)$, we may also proceed by explicitly dividing into cases based on whether $a \\geqslant \\frac{2}{3} b^{2} c^{2}$ or $a<\\frac{2}{3} b^{2} c^{2}$. The first case may now be dealt with as follows. We have $b^{3} c^{3}+1 \\geqslant b^{3}+c^{3}$ as $b^{3}$ and $c^{3}$ are positive integers, so we have $$ b^{3} c^{3}+1 \\geqslant b^{3}+c^{3} \\geqslant a^{2} \\geqslant \\frac{4}{9} b^{4} c^{4} $$ This implies $b c \\leqslant 2$, and hence $c=1$ and $b \\leqslant 2$.","tier":0} +{"problem_type":"Number Theory","problem_label":"N3","problem":"We say that a set $S$ of integers is rootiful if, for any positive integer $n$ and any $a_{0}, a_{1}, \\ldots, a_{n} \\in S$, all integer roots of the polynomial $a_{0}+a_{1} x+\\cdots+a_{n} x^{n}$ are also in $S$. Find all rootiful sets of integers that contain all numbers of the form $2^{a}-2^{b}$ for positive integers $a$ and $b$. (Czech Republic) Answer: The set $\\mathbb{Z}$ of all integers is the only such rootiful set.","solution":"The set $\\mathbb{Z}$ of all integers is clearly rootiful. We shall prove that any rootiful set $S$ containing all the numbers of the form $2^{a}-2^{b}$ for $a, b \\in \\mathbb{Z}_{>0}$ must be all of $\\mathbb{Z}$. First, note that $0=2^{1}-2^{1} \\in S$ and $2=2^{2}-2^{1} \\in S$. Now, $-1 \\in S$, since it is a root of $2 x+2$, and $1 \\in S$, since it is a root of $2 x^{2}-x-1$. Also, if $n \\in S$ then $-n$ is a root of $x+n$, so it suffices to prove that all positive integers must be in $S$. Now, we claim that any positive integer $n$ has a multiple in $S$. Indeed, suppose that $n=2^{\\alpha} \\cdot t$ for $\\alpha \\in \\mathbb{Z}_{\\geqslant 0}$ and $t$ odd. Then $t \\mid 2^{\\phi(t)}-1$, so $n \\mid 2^{\\alpha+\\phi(t)+1}-2^{\\alpha+1}$. Moreover, $2^{\\alpha+\\phi(t)+1}-2^{\\alpha+1} \\in S$, and so $S$ contains a multiple of every positive integer $n$. We will now prove by induction that all positive integers are in $S$. Suppose that $0,1, \\ldots, n-$ $1 \\in S$; furthermore, let $N$ be a multiple of $n$ in $S$. Consider the base- $n$ expansion of $N$, say $N=a_{k} n^{k}+a_{k-1} n^{k-1}+\\cdots+a_{1} n+a_{0}$. Since $0 \\leqslant a_{i}0}$. We show that, in fact, every integer $k$ with $|k|>2$ can be expressed as a root of a polynomial whose coefficients are of the form $2^{a}-2^{b}$. Observe that it suffices to consider the case where $k$ is positive, as if $k$ is a root of $a_{n} x^{n}+\\cdots+a_{1} x+a_{0}=0$, then $-k$ is a root of $(-1)^{n} a_{n} x^{n}+\\cdots-$ $a_{1} x+a_{0}=0$. Note that $$ \\left(2^{a_{n}}-2^{b_{n}}\\right) k^{n}+\\cdots+\\left(2^{a_{0}}-2^{b_{0}}\\right)=0 $$ is equivalent to $$ 2^{a_{n}} k^{n}+\\cdots+2^{a_{0}}=2^{b_{n}} k^{n}+\\cdots+2^{b_{0}} $$ Hence our aim is to show that two numbers of the form $2^{a_{n}} k^{n}+\\cdots+2^{a_{0}}$ are equal, for a fixed value of $n$. We consider such polynomials where every term $2^{a_{i}} k^{i}$ is at most $2 k^{n}$; in other words, where $2 \\leqslant 2^{a_{i}} \\leqslant 2 k^{n-i}$, or, equivalently, $1 \\leqslant a_{i} \\leqslant 1+(n-i) \\log _{2} k$. Therefore, there must be $1+\\left\\lfloor(n-i) \\log _{2} k\\right\\rfloor$ possible choices for $a_{i}$ satisfying these constraints. The number of possible polynomials is then $$ \\prod_{i=0}^{n}\\left(1+\\left\\lfloor(n-i) \\log _{2} k\\right\\rfloor\\right) \\geqslant \\prod_{i=0}^{n-1}(n-i) \\log _{2} k=n!\\left(\\log _{2} k\\right)^{n} $$ where the inequality holds as $1+\\lfloor x\\rfloor \\geqslant x$. As there are $(n+1)$ such terms in the polynomial, each at most $2 k^{n}$, such a polynomial must have value at most $2 k^{n}(n+1)$. However, for large $n$, we have $n!\\left(\\log _{2} k\\right)^{n}>2 k^{n}(n+1)$. Therefore there are more polynomials than possible values, so some two must be equal, as required.","tier":0} +{"problem_type":"Number Theory","problem_label":"N4","problem":"Let $\\mathbb{Z}_{>0}$ be the set of positive integers. A positive integer constant $C$ is given. Find all functions $f: \\mathbb{Z}_{>0} \\rightarrow \\mathbb{Z}_{>0}$ such that, for all positive integers $a$ and $b$ satisfying $a+b>C$, $$ a+f(b) \\mid a^{2}+b f(a) $$ (Croatia) Answer: The functions satisfying (*) are exactly the functions $f(a)=k a$ for some constant $k \\in \\mathbb{Z}_{>0}$ (irrespective of the value of $C$ ). Common remarks. It is easy to verify that the functions $f(a)=k a$ satisfy (*). Thus, in the proofs below, we will only focus on the converse implication: that condition (*) implies that $f=k a$. A common minor part of these solutions is the derivation of some relatively easy bounds on the function $f$. An upper bound is easily obtained by setting $a=1$ in (*), giving the inequality $$ f(b) \\leqslant b \\cdot f(1) $$ for all sufficiently large $b$. The corresponding lower bound is only marginally more difficult to obtain: substituting $b=1$ in the original equation shows that $$ a+f(1) \\mid\\left(a^{2}+f(a)\\right)-(a-f(1)) \\cdot(a+f(1))=f(1)^{2}+f(a) $$ for all sufficiently large $a$. It follows from this that one has the lower bound $$ f(a) \\geqslant a+f(1) \\cdot(1-f(1)) $$ again for all sufficiently large $a$. Each of the following proofs makes use of at least one of these bounds.","solution":"First, we show that $b \\mid f(b)^{2}$ for all $b$. To do this, we choose a large positive integer $n$ so that $n b-f(b) \\geqslant C$. Setting $a=n b-f(b)$ in (*) then shows that $$ n b \\mid(n b-f(b))^{2}+b f(n b-f(b)) $$ so that $b \\mid f(b)^{2}$ as claimed. Now in particular we have that $p \\mid f(p)$ for every prime $p$. If we write $f(p)=k(p) \\cdot p$, then the bound $f(p) \\leqslant f(1) \\cdot p$ (valid for $p$ sufficiently large) shows that some value $k$ of $k(p)$ must be attained for infinitely many $p$. We will show that $f(a)=k a$ for all positive integers $a$. To do this, we substitute $b=p$ in (*), where $p$ is any sufficiently large prime for which $k(p)=k$, obtaining $$ a+k p \\mid\\left(a^{2}+p f(a)\\right)-a(a+k p)=p f(a)-p k a . $$ For suitably large $p$ we have $\\operatorname{gcd}(a+k p, p)=1$, and hence we have $$ a+k p \\mid f(a)-k a $$ But the only way this can hold for arbitrarily large $p$ is if $f(a)-k a=0$. This concludes the proof. Comment. There are other ways to obtain the divisibility $p \\mid f(p)$ for primes $p$, which is all that is needed in this proof. For instance, if $f(p)$ were not divisible by $p$ then the arithmetic progression $p^{2}+b f(p)$ would attain prime values for infinitely many $b$ by Dirichlet's Theorem: hence, for these pairs p , b, we would have $p+f(b)=p^{2}+b f(p)$. Substituting $a \\mapsto b$ and $b \\mapsto p$ in $(*)$ then shows that $\\left(f(p)^{2}-p^{2}\\right)(p-1)$ is divisible by $b+f(p)$ and hence vanishes, which is impossible since $p \\nmid f(p)$ by assumption.","tier":0} +{"problem_type":"Number Theory","problem_label":"N4","problem":"Let $\\mathbb{Z}_{>0}$ be the set of positive integers. A positive integer constant $C$ is given. Find all functions $f: \\mathbb{Z}_{>0} \\rightarrow \\mathbb{Z}_{>0}$ such that, for all positive integers $a$ and $b$ satisfying $a+b>C$, $$ a+f(b) \\mid a^{2}+b f(a) $$ (Croatia) Answer: The functions satisfying (*) are exactly the functions $f(a)=k a$ for some constant $k \\in \\mathbb{Z}_{>0}$ (irrespective of the value of $C$ ). Common remarks. It is easy to verify that the functions $f(a)=k a$ satisfy (*). Thus, in the proofs below, we will only focus on the converse implication: that condition (*) implies that $f=k a$. A common minor part of these solutions is the derivation of some relatively easy bounds on the function $f$. An upper bound is easily obtained by setting $a=1$ in (*), giving the inequality $$ f(b) \\leqslant b \\cdot f(1) $$ for all sufficiently large $b$. The corresponding lower bound is only marginally more difficult to obtain: substituting $b=1$ in the original equation shows that $$ a+f(1) \\mid\\left(a^{2}+f(a)\\right)-(a-f(1)) \\cdot(a+f(1))=f(1)^{2}+f(a) $$ for all sufficiently large $a$. It follows from this that one has the lower bound $$ f(a) \\geqslant a+f(1) \\cdot(1-f(1)) $$ again for all sufficiently large $a$. Each of the following proofs makes use of at least one of these bounds.","solution":"First, we substitute $b=1$ in (*) and rearrange to find that $$ \\frac{f(a)+f(1)^{2}}{a+f(1)}=f(1)-a+\\frac{a^{2}+f(a)}{a+f(1)} $$ is a positive integer for sufficiently large $a$. Since $f(a) \\leqslant a f(1)$, for all sufficiently large $a$, it follows that $\\frac{f(a)+f(1)^{2}}{a+f(1)} \\leqslant f(1)$ also and hence there is a positive integer $k$ such that $\\frac{f(a)+f(1)^{2}}{a+f(1)}=k$ for infinitely many values of $a$. In other words, $$ f(a)=k a+f(1) \\cdot(k-f(1)) $$ for infinitely many $a$. Fixing an arbitrary choice of $a$ in (*), we have that $$ \\frac{a^{2}+b f(a)}{a+k b+f(1) \\cdot(k-f(1))} $$ is an integer for infinitely many $b$ (the same $b$ as above, maybe with finitely many exceptions). On the other hand, for $b$ taken sufficiently large, this quantity becomes arbitrarily close to $\\frac{f(a)}{k}$; this is only possible if $\\frac{f(a)}{k}$ is an integer and $$ \\frac{a^{2}+b f(a)}{a+k b+f(1) \\cdot(k-f(1))}=\\frac{f(a)}{k} $$ for infinitely many $b$. This rearranges to $$ \\frac{f(a)}{k} \\cdot(a+f(1) \\cdot(k-f(1)))=a^{2} $$ Hence $a^{2}$ is divisible by $a+f(1) \\cdot(k-f(1))$, and hence so is $f(1)^{2}(k-f(1))^{2}$. The only way this can occur for all $a$ is if $k=f(1)$, in which case $(* *)$ provides that $f(a)=k a$ for all $a$, as desired.","tier":0} +{"problem_type":"Number Theory","problem_label":"N4","problem":"Let $\\mathbb{Z}_{>0}$ be the set of positive integers. A positive integer constant $C$ is given. Find all functions $f: \\mathbb{Z}_{>0} \\rightarrow \\mathbb{Z}_{>0}$ such that, for all positive integers $a$ and $b$ satisfying $a+b>C$, $$ a+f(b) \\mid a^{2}+b f(a) $$ (Croatia) Answer: The functions satisfying (*) are exactly the functions $f(a)=k a$ for some constant $k \\in \\mathbb{Z}_{>0}$ (irrespective of the value of $C$ ). Common remarks. It is easy to verify that the functions $f(a)=k a$ satisfy (*). Thus, in the proofs below, we will only focus on the converse implication: that condition (*) implies that $f=k a$. A common minor part of these solutions is the derivation of some relatively easy bounds on the function $f$. An upper bound is easily obtained by setting $a=1$ in (*), giving the inequality $$ f(b) \\leqslant b \\cdot f(1) $$ for all sufficiently large $b$. The corresponding lower bound is only marginally more difficult to obtain: substituting $b=1$ in the original equation shows that $$ a+f(1) \\mid\\left(a^{2}+f(a)\\right)-(a-f(1)) \\cdot(a+f(1))=f(1)^{2}+f(a) $$ for all sufficiently large $a$. It follows from this that one has the lower bound $$ f(a) \\geqslant a+f(1) \\cdot(1-f(1)) $$ again for all sufficiently large $a$. Each of the following proofs makes use of at least one of these bounds.","solution":"Fix any two distinct positive integers $a$ and $b$. From (*) it follows that the two integers $$ \\left(a^{2}+c f(a)\\right) \\cdot(b+f(c)) \\text { and }\\left(b^{2}+c f(b)\\right) \\cdot(a+f(c)) $$ are both multiples of $(a+f(c)) \\cdot(b+f(c))$ for all sufficiently large $c$. Taking an appropriate linear combination to eliminate the $c f(c)$ term, we find after expanding out that the integer $$ \\left[a^{2} f(b)-b^{2} f(a)\\right] \\cdot f(c)+[(b-a) f(a) f(b)] \\cdot c+[a b(a f(b)-b f(a))] $$ is also a multiple of $(a+f(c)) \\cdot(b+f(c))$. But as $c$ varies, $(\\dagger)$ is bounded above by a positive multiple of $c$ while $(a+f(c)) \\cdot(b+f(c))$ is bounded below by a positive multiple of $c^{2}$. The only way that such a divisibility can hold is if in fact $$ \\left[a^{2} f(b)-b^{2} f(a)\\right] \\cdot f(c)+[(b-a) f(a) f(b)] \\cdot c+[a b(a f(b)-b f(a))]=0 $$ for sufficiently large $c$. Since the coefficient of $c$ in this linear relation is nonzero, it follows that there are constants $k, \\ell$ such that $f(c)=k c+\\ell$ for all sufficiently large $c$; the constants $k$ and $\\ell$ are necessarily integers. The value of $\\ell$ satisfies $$ \\left[a^{2} f(b)-b^{2} f(a)\\right] \\cdot \\ell+[a b(a f(b)-b f(a))]=0 $$ and hence $b \\mid \\ell a^{2} f(b)$ for all $a$ and $b$. Taking $b$ sufficiently large so that $f(b)=k b+\\ell$, we thus have that $b \\mid \\ell^{2} a^{2}$ for all sufficiently large $b$; this implies that $\\ell=0$. From ( $\\dagger \\dagger \\dagger$ ) it then follows that $\\frac{f(a)}{a}=\\frac{f(b)}{b}$ for all $a \\neq b$, so that there is a constant $k$ such that $f(a)=k a$ for all $a$ ( $k$ is equal to the constant defined earlier).","tier":0} +{"problem_type":"Number Theory","problem_label":"N4","problem":"Let $\\mathbb{Z}_{>0}$ be the set of positive integers. A positive integer constant $C$ is given. Find all functions $f: \\mathbb{Z}_{>0} \\rightarrow \\mathbb{Z}_{>0}$ such that, for all positive integers $a$ and $b$ satisfying $a+b>C$, $$ a+f(b) \\mid a^{2}+b f(a) $$ (Croatia) Answer: The functions satisfying (*) are exactly the functions $f(a)=k a$ for some constant $k \\in \\mathbb{Z}_{>0}$ (irrespective of the value of $C$ ). Common remarks. It is easy to verify that the functions $f(a)=k a$ satisfy (*). Thus, in the proofs below, we will only focus on the converse implication: that condition (*) implies that $f=k a$. A common minor part of these solutions is the derivation of some relatively easy bounds on the function $f$. An upper bound is easily obtained by setting $a=1$ in (*), giving the inequality $$ f(b) \\leqslant b \\cdot f(1) $$ for all sufficiently large $b$. The corresponding lower bound is only marginally more difficult to obtain: substituting $b=1$ in the original equation shows that $$ a+f(1) \\mid\\left(a^{2}+f(a)\\right)-(a-f(1)) \\cdot(a+f(1))=f(1)^{2}+f(a) $$ for all sufficiently large $a$. It follows from this that one has the lower bound $$ f(a) \\geqslant a+f(1) \\cdot(1-f(1)) $$ again for all sufficiently large $a$. Each of the following proofs makes use of at least one of these bounds.","solution":"Let $\\Gamma$ denote the set of all points $(a, f(a))$, so that $\\Gamma$ is an infinite subset of the upper-right quadrant of the plane. For a point $A=(a, f(a))$ in $\\Gamma$, we define a point $A^{\\prime}=\\left(-f(a),-f(a)^{2} \/ a\\right)$ in the lower-left quadrant of the plane, and let $\\Gamma^{\\prime}$ denote the set of all such points $A^{\\prime}$. ![](https:\/\/cdn.mathpix.com\/cropped\/2024_11_18_9ee8c2ac2c2f5002ff14g-095.jpg?height=552&width=652&top_left_y=449&top_left_x=702) Claim. For any point $A \\in \\Gamma$, the set $\\Gamma$ is contained in finitely many lines through the point $A^{\\prime}$. Proof. Let $A=(a, f(a))$. The functional equation (with $a$ and $b$ interchanged) can be rewritten as $b+f(a) \\mid a f(b)-b f(a)$, so that all but finitely many points in $\\Gamma$ are contained in one of the lines with equation $$ a y-f(a) x=m(x+f(a)) $$ for $m$ an integer. Geometrically, these are the lines through $A^{\\prime}=\\left(-f(a),-f(a)^{2} \/ a\\right)$ with gradient $\\frac{f(a)+m}{a}$. Since $\\Gamma$ is contained, with finitely many exceptions, in the region $0 \\leqslant y \\leqslant$ $f(1) \\cdot x$ and the point $A^{\\prime}$ lies strictly in the lower-left quadrant of the plane, there are only finitely many values of $m$ for which this line meets $\\Gamma$. This concludes the proof of the claim. Now consider any distinct points $A, B \\in \\Gamma$. It is clear that $A^{\\prime}$ and $B^{\\prime}$ are distinct. A line through $A^{\\prime}$ and a line through $B^{\\prime}$ only meet in more than one point if these two lines are equal to the line $A^{\\prime} B^{\\prime}$. It then follows from the above claim that the line $A^{\\prime} B^{\\prime}$ must contain all but finitely many points of $\\Gamma$. If $C$ is another point of $\\Gamma$, then the line $A^{\\prime} C^{\\prime}$ also passes through all but finitely many points of $\\Gamma$, which is only possible if $A^{\\prime} C^{\\prime}=A^{\\prime} B^{\\prime}$. We have thus seen that there is a line $\\ell$ passing through all points of $\\Gamma^{\\prime}$ and through all but finitely many points of $\\Gamma$. We claim that this line passes through the origin $O$ and passes through every point of $\\Gamma$. To see this, note that by construction $A, O, A^{\\prime}$ are collinear for every point $A \\in \\Gamma$. Since $\\ell=A A^{\\prime}$ for all but finitely many points $A \\in \\Gamma$, it thus follows that $O \\in \\ell$. Thus any $A \\in \\Gamma$ lies on the line $\\ell=A^{\\prime} O$. Since $\\Gamma$ is contained in a line through $O$, it follows that there is a real constant $k$ (the gradient of $\\ell$ ) such that $f(a)=k a$ for all $a$. The number $k$ is, of course, a positive integer. Comment. Without the $a+b>C$ condition, this problem is approachable by much more naive methods. For instance, using the given divisibility for $a, b \\in\\{1,2,3\\}$ one can prove by a somewhat tedious case-check that $f(2)=2 f(1)$ and $f(3)=3 f(1)$; this then forms the basis of an induction establishing that $f(n)=n f(1)$ for all $n$.","tier":0} +{"problem_type":"Number Theory","problem_label":"N6","problem":"Let $H=\\left\\{\\lfloor i \\sqrt{2}\\rfloor: i \\in \\mathbb{Z}_{>0}\\right\\}=\\{1,2,4,5,7, \\ldots\\}$, and let $n$ be a positive integer. Prove that there exists a constant $C$ such that, if $A \\subset\\{1,2, \\ldots, n\\}$ satisfies $|A| \\geqslant C \\sqrt{n}$, then there exist $a, b \\in A$ such that $a-b \\in H$. (Brazil) Common remarks. In all solutions, we will assume that $A$ is a set such that $\\{a-b: a, b \\in A\\}$ is disjoint from $H$, and prove that $|A|1-\\frac{1}{\\sqrt{2}} . $$ To see why, observe that $n \\in H$ if and only if $00}$. In other words, $01$ since $a_{i}-a_{1} \\notin H$. Furthermore, we must have $\\left\\{a_{i} \/ \\sqrt{2}\\right\\}<\\left\\{a_{j} \/ \\sqrt{2}\\right\\}$ whenever $i1 \/ \\sqrt{2}>$ $1-1 \/ \\sqrt{2}$, contradicting (1). Now, we have a sequence $0=a_{1}\\frac{1}{2 d \\sqrt{2}} $$ To see why this is the case, let $h=\\lfloor d \/ \\sqrt{2}\\rfloor$, so $\\{d \/ \\sqrt{2}\\}=d \/ \\sqrt{2}-h$. Then $$ \\left\\{\\frac{d}{\\sqrt{2}}\\right\\}\\left(\\frac{d}{\\sqrt{2}}+h\\right)=\\frac{d^{2}-2 h^{2}}{2} \\geqslant \\frac{1}{2} $$ since the numerator is a positive integer. Because $d \/ \\sqrt{2}+h<2 d \/ \\sqrt{2}$, inequality (2) follows. Let $d_{i}=a_{i+1}-a_{i}$, for $1 \\leqslant i\\sum_{i}\\left\\{\\frac{d_{i}}{\\sqrt{2}}\\right\\}>\\frac{1}{2 \\sqrt{2}} \\sum_{i} \\frac{1}{d_{i}} \\geqslant \\frac{(k-1)^{2}}{2 \\sqrt{2}} \\frac{1}{\\sum_{i} d_{i}}>\\frac{(k-1)^{2}}{2 \\sqrt{2}} \\cdot \\frac{1}{n} . $$ Here, the first inequality holds because $\\left\\{a_{k} \/ \\sqrt{2}\\right\\}<1-1 \/ \\sqrt{2}$, the second follows from (2), the third follows from an easy application of the AM-HM inequality (or Cauchy-Schwarz), and the fourth follows from the fact that $\\sum_{i} d_{i}=a_{k}k-1 $$ which provides the required bound on $k$.","tier":0} +{"problem_type":"Number Theory","problem_label":"N6","problem":"Let $H=\\left\\{\\lfloor i \\sqrt{2}\\rfloor: i \\in \\mathbb{Z}_{>0}\\right\\}=\\{1,2,4,5,7, \\ldots\\}$, and let $n$ be a positive integer. Prove that there exists a constant $C$ such that, if $A \\subset\\{1,2, \\ldots, n\\}$ satisfies $|A| \\geqslant C \\sqrt{n}$, then there exist $a, b \\in A$ such that $a-b \\in H$. (Brazil) Common remarks. In all solutions, we will assume that $A$ is a set such that $\\{a-b: a, b \\in A\\}$ is disjoint from $H$, and prove that $|A|0}\\right\\}$ is the complementary Beatty sequence to $H$ (in other words, $H$ and $J$ are disjoint with $H \\cup J=\\mathbb{Z}_{>0}$ ). Write $A=\\left\\{a_{1}0}$. For any $j>i$, we have $a_{j}-a_{i}=\\left\\lfloor\\alpha b_{j}\\right\\rfloor-\\left\\lfloor\\alpha b_{i}\\right\\rfloor$. Because $a_{j}-a_{i} \\in J$, we also have $a_{j}-a_{i}=\\lfloor\\alpha t\\rfloor$ for some positive integer $t$. Thus, $\\lfloor\\alpha t\\rfloor=\\left\\lfloor\\alpha b_{j}\\right\\rfloor-\\left\\lfloor\\alpha b_{i}\\right\\rfloor$. The right hand side must equal either $\\left\\lfloor\\alpha\\left(b_{j}-b_{i}\\right)\\right\\rfloor$ or $\\left\\lfloor\\alpha\\left(b_{j}-b_{i}\\right)\\right\\rfloor-1$, the latter of which is not a member of $J$ as $\\alpha>2$. Therefore, $t=b_{j}-b_{i}$ and so we have $\\left\\lfloor\\alpha b_{j}\\right\\rfloor-\\left\\lfloor\\alpha b_{i}\\right\\rfloor=\\left\\lfloor\\alpha\\left(b_{j}-b_{i}\\right)\\right\\rfloor$. For $1 \\leqslant i1 \/(2 d \\sqrt{2})$ for positive integers $d)$ proves that $1>\\left((k-1)^{2} \/(2 \\sqrt{2})\\right)(\\alpha \/ n)$, which again rearranges to give $$ \\sqrt{2 \\sqrt{2}-2} \\cdot \\sqrt{n}>k-1 $$ Comment. The use of Beatty sequences in Solution 2 is essentially a way to bypass (1). Both Solutions 1 and 2 use the fact that $\\sqrt{2}<2$; the statement in the question would still be true if $\\sqrt{2}$ did not have this property (for instance, if it were replaced with $\\alpha$ ), but any argument along the lines of Solutions 1 or 2 would be more complicated.","tier":0} +{"problem_type":"Number Theory","problem_label":"N6","problem":"Let $H=\\left\\{\\lfloor i \\sqrt{2}\\rfloor: i \\in \\mathbb{Z}_{>0}\\right\\}=\\{1,2,4,5,7, \\ldots\\}$, and let $n$ be a positive integer. Prove that there exists a constant $C$ such that, if $A \\subset\\{1,2, \\ldots, n\\}$ satisfies $|A| \\geqslant C \\sqrt{n}$, then there exist $a, b \\in A$ such that $a-b \\in H$. (Brazil) Common remarks. In all solutions, we will assume that $A$ is a set such that $\\{a-b: a, b \\in A\\}$ is disjoint from $H$, and prove that $|A|0} \\backslash H$, so all differences between elements of $A$ are in $J$. We start by making the following observation. Suppose we have a set $B \\subseteq\\{1,2, \\ldots, n\\}$ such that all of the differences between elements of $B$ are in $H$. Then $|A| \\cdot|B| \\leqslant 2 n$. To see why, observe that any two sums of the form $a+b$ with $a \\in A, b \\in B$ are different; otherwise, we would have $a_{1}+b_{1}=a_{2}+b_{2}$, and so $\\left|a_{1}-a_{2}\\right|=\\left|b_{2}-b_{1}\\right|$. However, then the left hand side is in $J$ whereas the right hand side is in $H$. Thus, $\\{a+b: a \\in A, b \\in B\\}$ is a set of size $|A| \\cdot|B|$ all of whose elements are no greater than $2 n$, yielding the claimed inequality. With this in mind, it suffices to construct a set $B$, all of whose differences are in $H$ and whose size is at least $C^{\\prime} \\sqrt{n}$ for some constant $C^{\\prime}>0$. To do so, we will use well-known facts about the negative Pell equation $X^{2}-2 Y^{2}=-1$; in particular, that there are infinitely many solutions and the values of $X$ are given by the recurrence $X_{1}=1, X_{2}=7$ and $X_{m}=6 X_{m-1}-X_{m-2}$. Therefore, we may choose $X$ to be a solution with $\\sqrt{n} \/ 6\\left(\\frac{X}{\\sqrt{2}}-Y\\right) \\geqslant \\frac{-3}{\\sqrt{2 n}}, $$ from which it follows that $\\{X \/ \\sqrt{2}\\}>1-(3 \/ \\sqrt{2 n})$. Combined with (1), this shows that all differences between elements of $B$ are in $H$. Comment. Some of the ideas behind Solution 3 may be used to prove that the constant $C=\\sqrt{2 \\sqrt{2}-2}$ (from Solutions 1 and 2) is optimal, in the sense that there are arbitrarily large values of $n$ and sets $A_{n} \\subseteq\\{1,2, \\ldots, n\\}$ of size roughly $C \\sqrt{n}$, all of whose differences are contained in $J$. To see why, choose $X$ to come from a sufficiently large solution to the Pell equation $X^{2}-2 Y^{2}=1$, so $\\{X \/ \\sqrt{2}\\} \\approx 1 \/(2 X \\sqrt{2})$. In particular, $\\{X\\},\\{2 X\\}, \\ldots,\\{[2 X \\sqrt{2}(1-1 \/ \\sqrt{2})\\rfloor X\\}$ are all less than $1-1 \/ \\sqrt{2}$. Thus, by (1) any positive integer of the form $i X$ for $1 \\leqslant i \\leqslant\\lfloor 2 X \\sqrt{2}(1-1 \/ \\sqrt{2})\\rfloor$ lies in $J$. Set $n \\approx 2 X^{2} \\sqrt{2}(1-1 \/ \\sqrt{2})$. We now have a set $A=\\{i X: i \\leqslant\\lfloor 2 X \\sqrt{2}(1-1 \/ \\sqrt{2})\\rfloor\\}$ containing roughly $2 X \\sqrt{2}(1-1 \/ \\sqrt{2})$ elements less than or equal to $n$ such that all of the differences lie in $J$, and we can see that $|A| \\approx C \\sqrt{n}$ with $C=\\sqrt{2 \\sqrt{2}-2}$.","tier":0} +{"problem_type":"Number Theory","problem_label":"N6","problem":"Let $H=\\left\\{\\lfloor i \\sqrt{2}\\rfloor: i \\in \\mathbb{Z}_{>0}\\right\\}=\\{1,2,4,5,7, \\ldots\\}$, and let $n$ be a positive integer. Prove that there exists a constant $C$ such that, if $A \\subset\\{1,2, \\ldots, n\\}$ satisfies $|A| \\geqslant C \\sqrt{n}$, then there exist $a, b \\in A$ such that $a-b \\in H$. (Brazil) Common remarks. In all solutions, we will assume that $A$ is a set such that $\\{a-b: a, b \\in A\\}$ is disjoint from $H$, and prove that $|A|\\sqrt{Y}$ elements; if that subsequence is decreasing, we may reflect (using (4) or (5)) to ensure that it is increasing. Call the subsequence $\\left\\{y_{1} \\sqrt{2}\\right\\},\\left\\{y_{2} \\sqrt{2}\\right\\}, \\ldots,\\left\\{y_{t} \\sqrt{2}\\right\\}$ for $t>\\sqrt{Y}$. Now, set $B=\\left\\{\\left\\lfloor y_{i} \\sqrt{2}\\right\\rfloor: 1 \\leqslant i \\leqslant t\\right\\}$. We have $\\left\\lfloor y_{j} \\sqrt{2}\\right\\rfloor-\\left\\lfloor y_{i} \\sqrt{2}\\right\\rfloor=\\left\\lfloor\\left(y_{j}-y_{i}\\right) \\sqrt{2}\\right\\rfloor$ for $i\\sqrt{Y}>\\sqrt{n} \/ \\sqrt{3 \\sqrt{2}}$, this is the required set. Comment. Any solution to this problem will need to use the fact that $\\sqrt{2}$ cannot be approximated well by rationals, either directly or implicitly (for example, by using facts about solutions to Pelllike equations). If $\\sqrt{2}$ were replaced by a value of $\\theta$ with very good rational approximations (from below), then an argument along the lines of Solution 3 would give long arithmetic progressions in $\\{\\lfloor i \\theta\\rfloor: 0 \\leqslant i\\left\\lceil\\frac{c^{2}}{b}\\right\\rceil-1=c k+k^{2}-1 \\geqslant c k $$ and $$ \\frac{(c-k)(c+k)}{b}<\\frac{c^{2}}{b} \\leqslant\\left\\lceil\\frac{c^{2}}{b}\\right\\rceil=(c+k) k $$ it can be seen that $c>b k>c-k$, so $$ c=k b+r \\quad \\text { with some } 00$ and $0a$, so $$ \\begin{aligned} & c^{2}-10$, then the left hand sides of $\\mathcal{I}(N,-x)$ and $\\mathcal{I}(N, x)$ coincide, while the right hand side of $\\mathcal{I}(N,-x)$ is larger than that of $\\mathcal{I}(N,-x)$ (their difference equals $2(N-1) x \\geqslant 0)$. Therefore, $\\mathcal{I}(N,-x)$ follows from $\\mathcal{I}(N, x)$. So, hereafter we suppose that $x>0$. Divide $\\mathcal{I}(N, x)$ by $x$ and let $t=(x-1)^{2} \/ x=x-2+1 \/ x$; then $\\mathcal{I}(n, x)$ reads as $$ f_{N}:=\\frac{x^{N}+x^{-N}}{2} \\leqslant\\left(1+\\frac{N}{2} t\\right)^{N} $$ The key identity is the expansion of $f_{N}$ as a polynomial in $t$ :","tier":0} +{"problem_type":"Algebra","problem_label":"A1","problem":"Version 1. Let $n$ be a positive integer, and set $N=2^{n}$. Determine the smallest real number $a_{n}$ such that, for all real $x$, $$ \\sqrt[N]{\\frac{x^{2 N}+1}{2}} \\leqslant a_{n}(x-1)^{2}+x $$ Version 2. For every positive integer $N$, determine the smallest real number $b_{N}$ such that, for all real $x$, $$ \\sqrt[N]{\\frac{x^{2 N}+1}{2}} \\leqslant b_{N}(x-1)^{2}+x $$ (Ireland)","solution":"2 (for Version 2). Here we present another proof of the inequality (2) for $x>0$, or, equivalently, for $t=(x-1)^{2} \/ x \\geqslant 0$. Instead of finding the coefficients of the polynomial $f_{N}=f_{N}(t)$ we may find its roots, which is in a sense more straightforward. Note that the recurrence (4) and the initial conditions $f_{0}=1, f_{1}=1+t \/ 2$ imply that $f_{N}$ is a polynomial in $t$ of degree $N$. It also follows by induction that $f_{N}(0)=1, f_{N}^{\\prime}(0)=N^{2} \/ 2$ : the recurrence relations read as $f_{N+1}(0)+f_{N-1}(0)=2 f_{N}(0)$ and $f_{N+1}^{\\prime}(0)+f_{N-1}^{\\prime}(0)=2 f_{N}^{\\prime}(0)+f_{N}(0)$, respectively. Next, if $x_{k}=\\exp \\left(\\frac{i \\pi(2 k-1)}{2 N}\\right)$ for $k \\in\\{1,2, \\ldots, N\\}$, then $$ -t_{k}:=2-x_{k}-\\frac{1}{x_{k}}=2-2 \\cos \\frac{\\pi(2 k-1)}{2 N}=4 \\sin ^{2} \\frac{\\pi(2 k-1)}{4 N}>0 $$ and $$ f_{N}\\left(t_{k}\\right)=\\frac{x_{k}^{N}+x_{k}^{-N}}{2}=\\frac{\\exp \\left(\\frac{i \\pi(2 k-1)}{2}\\right)+\\exp \\left(-\\frac{i \\pi(2 k-1)}{2}\\right)}{2}=0 . $$ So the roots of $f_{N}$ are $t_{1}, \\ldots, t_{N}$ and by the AM-GM inequality we have $$ \\begin{aligned} f_{N}(t)=\\left(1-\\frac{t}{t_{1}}\\right)\\left(1-\\frac{t}{t_{2}}\\right) \\ldots\\left(1-\\frac{t}{t_{N}}\\right) & \\leqslant\\left(1-\\frac{t}{N}\\left(\\frac{1}{t_{1}}+\\ldots+\\frac{1}{t_{n}}\\right)\\right)^{N}= \\\\ \\left(1+\\frac{t f_{N}^{\\prime}(0)}{N}\\right)^{N} & =\\left(1+\\frac{N}{2} t\\right)^{N} \\end{aligned} $$ Comment. The polynomial $f_{N}(t)$ equals to $\\frac{1}{2} T_{N}(t+2)$, where $T_{n}$ is the $n^{\\text {th }}$ Chebyshev polynomial of the first kind: $T_{n}(2 \\cos s)=2 \\cos n s, T_{n}(x+1 \/ x)=x^{n}+1 \/ x^{n}$.","tier":0} +{"problem_type":"Algebra","problem_label":"A1","problem":"Version 1. Let $n$ be a positive integer, and set $N=2^{n}$. Determine the smallest real number $a_{n}$ such that, for all real $x$, $$ \\sqrt[N]{\\frac{x^{2 N}+1}{2}} \\leqslant a_{n}(x-1)^{2}+x $$ Version 2. For every positive integer $N$, determine the smallest real number $b_{N}$ such that, for all real $x$, $$ \\sqrt[N]{\\frac{x^{2 N}+1}{2}} \\leqslant b_{N}(x-1)^{2}+x $$ (Ireland)","solution":"3 (for Version 2). Here we solve the problem when $N \\geqslant 1$ is an arbitrary real number. For a real number $a$ let $$ f(x)=\\left(\\frac{x^{2 N}+1}{2}\\right)^{\\frac{1}{N}}-a(x-1)^{2}-x $$ Then $f(1)=0$, $$ f^{\\prime}(x)=\\left(\\frac{x^{2 N}+1}{2}\\right)^{\\frac{1}{N}-1} x^{2 N-1}-2 a(x-1)-1 \\quad \\text { and } \\quad f^{\\prime}(1)=0 $$ $f^{\\prime \\prime}(x)=(1-N)\\left(\\frac{x^{2 N}+1}{2}\\right)^{\\frac{1}{N}-2} x^{4 N-2}+(2 N-1)\\left(\\frac{x^{2 N}+1}{2}\\right)^{\\frac{1}{N}-1} x^{2 N-2}-2 a \\quad$ and $\\quad f^{\\prime \\prime}(1)=N-2 a$. So if $a<\\frac{N}{2}$, the function $f$ has a strict local minimum at point 1 , and the inequality $f(x) \\leqslant$ $0=f(1)$ does not hold. This proves $b_{N} \\geqslant N \/ 2$. For $a=\\frac{N}{2}$ we have $f^{\\prime \\prime}(1)=0$ and $$ f^{\\prime \\prime \\prime}(x)=\\frac{1}{2}(1-N)(1-2 N)\\left(\\frac{x^{2 N}+1}{2}\\right)^{\\frac{1}{N}-3} x^{2 N-3}\\left(1-x^{2 N}\\right) \\quad \\begin{cases}>0 & \\text { if } 01\\end{cases} $$ Hence, $f^{\\prime \\prime}(x)<0$ for $x \\neq 1 ; f^{\\prime}(x)>0$ for $x<1$ and $f^{\\prime}(x)<0$ for $x>1$, finally $f(x)<0$ for $x \\neq 1$. Comment. Version 2 is much more difficult, of rather A5 or A6 difficulty. The induction in Version 1 is rather straightforward, while all three above solutions of Version 2 require some creativity. This page is intentionally left blank","tier":0} +{"problem_type":"Algebra","problem_label":"A2","problem":"Let $\\mathcal{A}$ denote the set of all polynomials in three variables $x, y, z$ with integer coefficients. Let $\\mathcal{B}$ denote the subset of $\\mathcal{A}$ formed by all polynomials which can be expressed as $$ (x+y+z) P(x, y, z)+(x y+y z+z x) Q(x, y, z)+x y z R(x, y, z) $$ with $P, Q, R \\in \\mathcal{A}$. Find the smallest non-negative integer $n$ such that $x^{i} y^{j} z^{k} \\in \\mathcal{B}$ for all nonnegative integers $i, j, k$ satisfying $i+j+k \\geqslant n$. (Venezuela)","solution":"We start by showing that $n \\leqslant 4$, i.e., any monomial $f=x^{i} y^{j} z^{k}$ with $i+j+k \\geqslant 4$ belongs to $\\mathcal{B}$. Assume that $i \\geqslant j \\geqslant k$, the other cases are analogous. Let $x+y+z=p, x y+y z+z x=q$ and $x y z=r$. Then $$ 0=(x-x)(x-y)(x-z)=x^{3}-p x^{2}+q x-r $$ therefore $x^{3} \\in \\mathcal{B}$. Next, $x^{2} y^{2}=x y q-(x+y) r \\in \\mathcal{B}$. If $k \\geqslant 1$, then $r$ divides $f$, thus $f \\in \\mathcal{B}$. If $k=0$ and $j \\geqslant 2$, then $x^{2} y^{2}$ divides $f$, thus we have $f \\in \\mathcal{B}$ again. Finally, if $k=0, j \\leqslant 1$, then $x^{3}$ divides $f$ and $f \\in \\mathcal{B}$ in this case also. In order to prove that $n \\geqslant 4$, we show that the monomial $x^{2} y$ does not belong to $\\mathcal{B}$. Assume the contrary: $$ x^{2} y=p P+q Q+r R $$ for some polynomials $P, Q, R$. If polynomial $P$ contains the monomial $x^{2}$ (with nonzero coefficient), then $p P+q Q+r R$ contains the monomial $x^{3}$ with the same nonzero coefficient. So $P$ does not contain $x^{2}, y^{2}, z^{2}$ and we may write $$ x^{2} y=(x+y+z)(a x y+b y z+c z x)+(x y+y z+z x)(d x+e y+f z)+g x y z $$ where $a, b, c ; d, e, f ; g$ are the coefficients of $x y, y z, z x ; x, y, z ; x y z$ in the polynomials $P$; $Q ; R$, respectively (the remaining coefficients do not affect the monomials of degree 3 in $p P+q Q+r R)$. By considering the coefficients of $x y^{2}$ we get $e=-a$, analogously $e=-b$, $f=-b, f=-c, d=-c$, thus $a=b=c$ and $f=e=d=-a$, but then the coefficient of $x^{2} y$ in the right hand side equals $a+d=0 \\neq 1$. Comment 1. The general question is the following. Call a polynomial $f\\left(x_{1}, \\ldots, x_{n}\\right)$ with integer coefficients nice, if $f(0,0, \\ldots, 0)=0$ and $f\\left(x_{\\pi_{1}}, \\ldots, x_{\\pi_{n}}\\right)=f\\left(x_{1}, \\ldots, x_{n}\\right)$ for any permutation $\\pi$ of $1, \\ldots, n$ (in other words, $f$ is symmetric and its constant term is zero.) Denote by $\\mathcal{I}$ the set of polynomials of the form $$ p_{1} q_{1}+p_{2} q_{2}+\\ldots+p_{m} q_{m} $$ where $m$ is an integer, $q_{1}, \\ldots, q_{m}$ are polynomials with integer coefficients, and $p_{1}, \\ldots, p_{m}$ are nice polynomials. Find the least $N$ for which any monomial of degree at least $N$ belongs to $\\mathcal{I}$. The answer is $n(n-1) \/ 2+1$. The lower bound follows from the following claim: the polynomial $$ F\\left(x_{1}, \\ldots, x_{n}\\right)=x_{2} x_{3}^{2} x_{4}^{3} \\cdot \\ldots \\cdot x_{n}^{n-1} $$ does not belong to $\\mathcal{I}$. Assume that $F=\\sum p_{i} q_{i}$, according to (2). By taking only the monomials of degree $n(n-1) \/ 2$, we can additionally assume that every $p_{i}$ and every $q_{i}$ is homogeneous, $\\operatorname{deg} p_{i}>0$, and $\\operatorname{deg} p_{i}+\\operatorname{deg} q_{i}=$ $\\operatorname{deg} F=n(n-1) \/ 2$ for all $i$. Consider the alternating sum $$ \\sum_{\\pi} \\operatorname{sign}(\\pi) F\\left(x_{\\pi_{1}}, \\ldots, x_{\\pi_{n}}\\right)=\\sum_{i=1}^{m} p_{i} \\sum_{\\pi} \\operatorname{sign}(\\pi) q_{i}\\left(x_{\\pi_{1}}, \\ldots, x_{\\pi_{n}}\\right):=S $$ where the summation is done over all permutations $\\pi$ of $1, \\ldots n$, and $\\operatorname{sign}(\\pi)$ denotes the sign of the permutation $\\pi$. Since $\\operatorname{deg} q_{i}=n(n-1) \/ 2-\\operatorname{deg} p_{i}1$ and that the proposition is proved for smaller values of $n$. We proceed by an internal induction on $S:=\\left|\\left\\{i: c_{i}=0\\right\\}\\right|$. In the base case $S=0$ the monomial $h$ is divisible by the nice polynomial $x_{1} \\cdot \\ldots x_{n}$, therefore $h \\in \\mathcal{I}$. Now assume that $S>0$ and that the claim holds for smaller values of $S$. Let $T=n-S$. We may assume that $c_{T+1}=\\ldots=c_{n}=0$ and $h=x_{1} \\cdot \\ldots \\cdot x_{T} g\\left(x_{1}, \\ldots, x_{n-1}\\right)$, where $\\operatorname{deg} g=n(n-1) \/ 2-T+1 \\geqslant(n-1)(n-2) \/ 2+1$. Using the outer induction hypothesis we represent $g$ as $p_{1} q_{1}+\\ldots+p_{m} q_{m}$, where $p_{i}\\left(x_{1}, \\ldots, x_{n-1}\\right)$ are nice polynomials in $n-1$ variables. There exist nice homogeneous polynomials $P_{i}\\left(x_{1}, \\ldots, x_{n}\\right)$ such that $P_{i}\\left(x_{1}, \\ldots, x_{n-1}, 0\\right)=p_{i}\\left(x_{1}, \\ldots, x_{n-1}\\right)$. In other words, $\\Delta_{i}:=p_{i}\\left(x_{1}, \\ldots, x_{n-1}\\right)-P_{i}\\left(x_{1}, \\ldots, x_{n-1}, x_{n}\\right)$ is divisible by $x_{n}$, let $\\Delta_{i}=x_{n} g_{i}$. We get $$ h=x_{1} \\cdot \\ldots \\cdot x_{T} \\sum p_{i} q_{i}=x_{1} \\cdot \\ldots \\cdot x_{T} \\sum\\left(P_{i}+x_{n} g_{i}\\right) q_{i}=\\left(x_{1} \\cdot \\ldots \\cdot x_{T} x_{n}\\right) \\sum g_{i} q_{i}+\\sum P_{i} q_{i} \\in \\mathcal{I} $$ The first term belongs to $\\mathcal{I}$ by the inner induction hypothesis. This completes both inductions. Comment 2. The solutions above work smoothly for the versions of the original problem and its extensions to the case of $n$ variables, where all polynomials are assumed to have real coefficients. In the version with integer coefficients, the argument showing that $x^{2} y \\notin \\mathcal{B}$ can be simplified: it is not hard to show that in every polynomial $f \\in \\mathcal{B}$, the sum of the coefficients of $x^{2} y, x^{2} z, y^{2} x, y^{2} z, z^{2} x$ and $z^{2} y$ is even. A similar fact holds for any number of variables and also implies that $N \\geqslant n(n-1) \/ 2+1$ in terms of the previous comment.","tier":0} +{"problem_type":"Algebra","problem_label":"A3","problem":"Suppose that $a, b, c, d$ are positive real numbers satisfying $(a+c)(b+d)=a c+b d$. Find the smallest possible value of $$ \\frac{a}{b}+\\frac{b}{c}+\\frac{c}{d}+\\frac{d}{a} $$","solution":"To show that $S \\geqslant 8$, apply the AM-GM inequality twice as follows: $$ \\left(\\frac{a}{b}+\\frac{c}{d}\\right)+\\left(\\frac{b}{c}+\\frac{d}{a}\\right) \\geqslant 2 \\sqrt{\\frac{a c}{b d}}+2 \\sqrt{\\frac{b d}{a c}}=\\frac{2(a c+b d)}{\\sqrt{a b c d}}=\\frac{2(a+c)(b+d)}{\\sqrt{a b c d}} \\geqslant 2 \\cdot \\frac{2 \\sqrt{a c} \\cdot 2 \\sqrt{b d}}{\\sqrt{a b c d}}=8 . $$ The above inequalities turn into equalities when $a=c$ and $b=d$. Then the condition $(a+c)(b+d)=a c+b d$ can be rewritten as $4 a b=a^{2}+b^{2}$. So it is satisfied when $a \/ b=2 \\pm \\sqrt{3}$. Hence, $S$ attains value 8 , e.g., when $a=c=1$ and $b=d=2+\\sqrt{3}$.","tier":0} +{"problem_type":"Algebra","problem_label":"A3","problem":"Suppose that $a, b, c, d$ are positive real numbers satisfying $(a+c)(b+d)=a c+b d$. Find the smallest possible value of $$ \\frac{a}{b}+\\frac{b}{c}+\\frac{c}{d}+\\frac{d}{a} $$","solution":"By homogeneity we may suppose that $a b c d=1$. Let $a b=C, b c=A$ and $c a=B$. Then $a, b, c$ can be reconstructed from $A, B$ and $C$ as $a=\\sqrt{B C \/ A}, b=\\sqrt{A C \/ B}$ and $c=\\sqrt{A B \/ C}$. Moreover, the condition $(a+c)(b+d)=a c+b d$ can be written in terms of $A, B, C$ as $$ A+\\frac{1}{A}+C+\\frac{1}{C}=b c+a d+a b+c d=(a+c)(b+d)=a c+b d=B+\\frac{1}{B} . $$ We then need to minimize the expression $$ \\begin{aligned} S & :=\\frac{a d+b c}{b d}+\\frac{a b+c d}{a c}=\\left(A+\\frac{1}{A}\\right) B+\\left(C+\\frac{1}{C}\\right) \\frac{1}{B} \\\\ & =\\left(A+\\frac{1}{A}\\right)\\left(B-\\frac{1}{B}\\right)+\\left(A+\\frac{1}{A}+C+\\frac{1}{C}\\right) \\frac{1}{B} \\\\ & =\\left(A+\\frac{1}{A}\\right)\\left(B-\\frac{1}{B}\\right)+\\left(B+\\frac{1}{B}\\right) \\frac{1}{B} . \\end{aligned} $$ Without loss of generality assume that $B \\geqslant 1$ (otherwise, we may replace $B$ by $1 \/ B$ and swap $A$ and $C$, this changes neither the relation nor the function to be maximized). Therefore, we can write $$ S \\geqslant 2\\left(B-\\frac{1}{B}\\right)+\\left(B+\\frac{1}{B}\\right) \\frac{1}{B}=2 B+\\left(1-\\frac{1}{B}\\right)^{2}=: f(B) $$ Clearly, $f$ increases on $[1, \\infty)$. Since $$ B+\\frac{1}{B}=A+\\frac{1}{A}+C+\\frac{1}{C} \\geqslant 4, $$ we have $B \\geqslant B^{\\prime}$, where $B^{\\prime}=2+\\sqrt{3}$ is the unique root greater than 1 of the equation $B^{\\prime}+1 \/ B^{\\prime}=4$. Hence, $$ S \\geqslant f(B) \\geqslant f\\left(B^{\\prime}\\right)=2\\left(B^{\\prime}-\\frac{1}{B^{\\prime}}\\right)+\\left(B^{\\prime}+\\frac{1}{B^{\\prime}}\\right) \\frac{1}{B^{\\prime}}=2 B^{\\prime}-\\frac{2}{B^{\\prime}}+\\frac{4}{B^{\\prime}}=8 $$ It remains to note that when $A=C=1$ and $B=B^{\\prime}$ we have the equality $S=8$.","tier":0} +{"problem_type":"Algebra","problem_label":"A3","problem":"Suppose that $a, b, c, d$ are positive real numbers satisfying $(a+c)(b+d)=a c+b d$. Find the smallest possible value of $$ \\frac{a}{b}+\\frac{b}{c}+\\frac{c}{d}+\\frac{d}{a} $$","solution":"We present another proof of the inequality $S \\geqslant 8$. We start with the estimate $$ \\left(\\frac{a}{b}+\\frac{c}{d}\\right)+\\left(\\frac{b}{c}+\\frac{d}{a}\\right) \\geqslant 2 \\sqrt{\\frac{a c}{b d}}+2 \\sqrt{\\frac{b d}{a c}} $$ Let $y=\\sqrt{a c}$ and $z=\\sqrt{b d}$, and assume, without loss of generality, that $a c \\geqslant b d$. By the AM-GM inequality, we have $$ y^{2}+z^{2}=a c+b d=(a+c)(b+d) \\geqslant 2 \\sqrt{a c} \\cdot 2 \\sqrt{b d}=4 y z . $$ Substituting $x=y \/ z$, we get $4 x \\leqslant x^{2}+1$. For $x \\geqslant 1$, this holds if and only if $x \\geqslant 2+\\sqrt{3}$. Now we have $$ 2 \\sqrt{\\frac{a c}{b d}}+2 \\sqrt{\\frac{b d}{a c}}=2\\left(x+\\frac{1}{x}\\right) . $$ Clearly, this is minimized by setting $x(\\geqslant 1)$ as close to 1 as possible, i.e., by taking $x=2+\\sqrt{3}$. Then $2(x+1 \/ x)=2((2+\\sqrt{3})+(2-\\sqrt{3}))=8$, as required.","tier":0} +{"problem_type":"Algebra","problem_label":"A4","problem":"Let $a, b, c, d$ be four real numbers such that $a \\geqslant b \\geqslant c \\geqslant d>0$ and $a+b+c+d=1$. Prove that $$ (a+2 b+3 c+4 d) a^{a} b^{b} c^{c} d^{d}<1 $$ (Belgium)","solution":"The weighted AM-GM inequality with weights $a, b, c, d$ gives $$ a^{a} b^{b} c^{c} d^{d} \\leqslant a \\cdot a+b \\cdot b+c \\cdot c+d \\cdot d=a^{2}+b^{2}+c^{2}+d^{2} $$ so it suffices to prove that $(a+2 b+3 c+4 d)\\left(a^{2}+b^{2}+c^{2}+d^{2}\\right)<1=(a+b+c+d)^{3}$. This can be done in various ways, for example: $$ \\begin{aligned} (a+b+c+d)^{3}> & a^{2}(a+3 b+3 c+3 d)+b^{2}(3 a+b+3 c+3 d) \\\\ & \\quad+c^{2}(3 a+3 b+c+3 d)+d^{2}(3 a+3 b+3 c+d) \\\\ \\geqslant & \\left(a^{2}+b^{2}+c^{2}+d^{2}\\right) \\cdot(a+2 b+3 c+4 d) \\end{aligned} $$","tier":0} +{"problem_type":"Algebra","problem_label":"A4","problem":"Let $a, b, c, d$ be four real numbers such that $a \\geqslant b \\geqslant c \\geqslant d>0$ and $a+b+c+d=1$. Prove that $$ (a+2 b+3 c+4 d) a^{a} b^{b} c^{c} d^{d}<1 $$ (Belgium)","solution":"From $b \\geqslant d$ we get $$ a+2 b+3 c+4 d \\leqslant a+3 b+3 c+3 d=3-2 a $$ If $a<\\frac{1}{2}$, then the statement can be proved by $$ (a+2 b+3 c+4 d) a^{a} b^{b} c^{c} d^{d} \\leqslant(3-2 a) a^{a} a^{b} a^{c} a^{d}=(3-2 a) a=1-(1-a)(1-2 a)<1 $$ From now on we assume $\\frac{1}{2} \\leqslant a<1$. By $b, c, d<1-a$ we have $$ b^{b} c^{c} d^{d}<(1-a)^{b} \\cdot(1-a)^{c} \\cdot(1-a)^{d}=(1-a)^{1-a} . $$ Therefore, $$ (a+2 b+3 c+4 d) a^{a} b^{b} c^{c} d^{d}<(3-2 a) a^{a}(1-a)^{1-a} $$ For $00 $$ so $g$ is strictly convex on $(0,1)$. By $g\\left(\\frac{1}{2}\\right)=\\log 2+2 \\cdot \\frac{1}{2} \\log \\frac{1}{2}=0$ and $\\lim _{x \\rightarrow 1-} g(x)=0$, we have $g(x) \\leqslant 0$ (and hence $f(x) \\leqslant 1$ ) for all $x \\in\\left[\\frac{1}{2}, 1\\right)$, and therefore $$ (a+2 b+3 c+4 d) a^{a} b^{b} c^{c} d^{d}0$ with $\\sum_{i} a_{i}=1$, the inequality $$ \\left(\\sum_{i} i a_{i}\\right) \\prod_{i} a_{i}^{a_{i}} \\leqslant 1 $$ does not necessarily hold. Indeed, let $a_{2}=a_{3}=\\ldots=a_{n}=\\varepsilon$ and $a_{1}=1-(n-1) \\varepsilon$, where $n$ and $\\varepsilon \\in(0,1 \/ n)$ will be chosen later. Then $$ \\left(\\sum_{i} i a_{i}\\right) \\prod_{i} a_{i}^{a_{i}}=\\left(1+\\frac{n(n-1)}{2} \\varepsilon\\right) \\varepsilon^{(n-1) \\varepsilon}(1-(n-1) \\varepsilon)^{1-(n-1) \\varepsilon} $$ If $\\varepsilon=C \/ n^{2}$ with an arbitrary fixed $C>0$ and $n \\rightarrow \\infty$, then the factors $\\varepsilon^{(n-1) \\varepsilon}=\\exp ((n-1) \\varepsilon \\log \\varepsilon)$ and $(1-(n-1) \\varepsilon)^{1-(n-1) \\varepsilon}$ tend to 1 , so the limit of (1) in this set-up equals $1+C \/ 2$. This is not simply greater than 1 , but it can be arbitrarily large.","tier":0} +{"problem_type":"Algebra","problem_label":"A5","problem":"A magician intends to perform the following trick. She announces a positive integer $n$, along with $2 n$ real numbers $x_{1}<\\ldots\\max _{z \\in \\mathcal{O}(0)}|z|$, this yields $f(a)=f(-a)$ and $f^{2 a^{2}}(0)=0$. Therefore, the sequence $\\left(f^{k}(0): k=0,1, \\ldots\\right)$ is purely periodic with a minimal period $T$ which divides $2 a^{2}$. Analogously, $T$ divides $2(a+1)^{2}$, therefore, $T \\mid \\operatorname{gcd}\\left(2 a^{2}, 2(a+1)^{2}\\right)=2$, i.e., $f(f(0))=0$ and $a(f(a)-f(-a))=f^{2 a^{2}}(0)=0$ for all $a$. Thus, $$ \\begin{array}{ll} f(a)=f(-a) \\quad \\text { for all } a \\neq 0 \\\\ \\text { in particular, } & f(1)=f(-1)=0 \\end{array} $$ Next, for each $n \\in \\mathbb{Z}$, by $E(n, 1-n)$ we get $$ n f(n)+(1-n) f(1-n)=f^{n^{2}+(1-n)^{2}}(1)=f^{2 n^{2}-2 n}(0)=0 $$ Assume that there exists some $m \\neq 0$ such that $f(m) \\neq 0$. Choose such an $m$ for which $|m|$ is minimal possible. Then $|m|>1$ due to $(\\boldsymbol{\\phi}) ; f(|m|) \\neq 0$ due to ( $\\boldsymbol{\\phi})$; and $f(1-|m|) \\neq 0$ due to $(\\Omega)$ for $n=|m|$. This contradicts to the minimality assumption. So, $f(n)=0$ for $n \\neq 0$. Finally, $f(0)=f^{3}(0)=f^{4}(2)=2 f(2)=0$. Clearly, the function $f(x) \\equiv 0$ satisfies the problem condition, which provides the first of the two answers. Case 2: All orbits are infinite. Since the orbits $\\mathcal{O}(a)$ and $\\mathcal{O}(a-1)$ differ by finitely many terms for all $a \\in \\mathbb{Z}$, each two orbits $\\mathcal{O}(a)$ and $\\mathcal{O}(b)$ have infinitely many common terms for arbitrary $a, b \\in \\mathbb{Z}$. For a minute, fix any $a, b \\in \\mathbb{Z}$. We claim that all pairs $(n, m)$ of nonnegative integers such that $f^{n}(a)=f^{m}(b)$ have the same difference $n-m$. Arguing indirectly, we have $f^{n}(a)=f^{m}(b)$ and $f^{p}(a)=f^{q}(b)$ with, say, $n-m>p-q$, then $f^{p+m+k}(b)=f^{p+n+k}(a)=f^{q+n+k}(b)$, for all nonnegative integers $k$. This means that $f^{\\ell+(n-m)-(p-q)}(b)=f^{\\ell}(b)$ for all sufficiently large $\\ell$, i.e., that the sequence $\\left(f^{n}(b)\\right)$ is eventually periodic, so $\\mathcal{O}(b)$ is finite, which is impossible. Now, for every $a, b \\in \\mathbb{Z}$, denote the common difference $n-m$ defined above by $X(a, b)$. We have $X(a-1, a)=1$ by (1). Trivially, $X(a, b)+X(b, c)=X(a, c)$, as if $f^{n}(a)=f^{m}(b)$ and $f^{p}(b)=f^{q}(c)$, then $f^{p+n}(a)=f^{p+m}(b)=f^{q+m}(c)$. These two properties imply that $X(a, b)=b-a$ for all $a, b \\in \\mathbb{Z}$. But (1) yields $f^{a^{2}+1}(f(a-1))=f^{a^{2}}(f(a))$, so $$ 1=X(f(a-1), f(a))=f(a)-f(a-1) \\quad \\text { for all } a \\in \\mathbb{Z} $$ Recalling that $f(-1)=0$, we conclude by (two-sided) induction on $x$ that $f(x)=x+1$ for all $x \\in \\mathbb{Z}$. Finally, the obtained function also satisfies the assumption. Indeed, $f^{n}(x)=x+n$ for all $n \\geqslant 0$, so $$ f^{a^{2}+b^{2}}(a+b)=a+b+a^{2}+b^{2}=a f(a)+b f(b) $$ Comment. There are many possible variations of the solution above, but it seems that finiteness of orbits seems to be a crucial distinction in all solutions. However, the case distinction could be made in different ways; in particular, there exist some versions of Case 1 which work whenever there is at least one finite orbit. We believe that Case 2 is conceptually harder than Case 1.","tier":0} +{"problem_type":"Algebra","problem_label":"A7","problem":"Let $n$ and $k$ be positive integers. Prove that for $a_{1}, \\ldots, a_{n} \\in\\left[1,2^{k}\\right]$ one has $$ \\sum_{i=1}^{n} \\frac{a_{i}}{\\sqrt{a_{1}^{2}+\\ldots+a_{i}^{2}}} \\leqslant 4 \\sqrt{k n} $$","solution":"Partition the set of indices $\\{1,2, \\ldots, n\\}$ into disjoint subsets $M_{1}, M_{2}, \\ldots, M_{k}$ so that $a_{\\ell} \\in\\left[2^{j-1}, 2^{j}\\right]$ for $\\ell \\in M_{j}$. Then, if $\\left|M_{j}\\right|=: p_{j}$, we have $$ \\sum_{\\ell \\in M_{j}} \\frac{a_{\\ell}}{\\sqrt{a_{1}^{2}+\\ldots+a_{\\ell}^{2}}} \\leqslant \\sum_{i=1}^{p_{j}} \\frac{2^{j}}{2^{j-1} \\sqrt{i}}=2 \\sum_{i=1}^{p_{j}} \\frac{1}{\\sqrt{i}} $$ where we used that $a_{\\ell} \\leqslant 2^{j}$ and in the denominator every index from $M_{j}$ contributes at least $\\left(2^{j-1}\\right)^{2}$. Now, using $\\sqrt{i}-\\sqrt{i-1}=\\frac{1}{\\sqrt{i}+\\sqrt{i-1}} \\geqslant \\frac{1}{2 \\sqrt{i}}$, we deduce that $$ \\sum_{\\ell \\in M_{j}} \\frac{a_{\\ell}}{\\sqrt{a_{1}^{2}+\\ldots+a_{\\ell}^{2}}} \\leqslant 2 \\sum_{i=1}^{p_{j}} \\frac{1}{\\sqrt{i}} \\leqslant 2 \\sum_{i=1}^{p_{j}} 2(\\sqrt{i}-\\sqrt{i-1})=4 \\sqrt{p_{j}} $$ Therefore, summing over $j=1, \\ldots, k$ and using the QM-AM inequality, we obtain $$ \\sum_{\\ell=1}^{n} \\frac{a_{\\ell}}{\\sqrt{a_{1}^{2}+\\ldots+a_{\\ell}^{2}}} \\leqslant 4 \\sum_{j=1}^{k} \\sqrt{\\left|M_{j}\\right|} \\leqslant 4 \\sqrt{k \\sum_{j=1}^{k}\\left|M_{j}\\right|}=4 \\sqrt{k n} $$ Comment. Consider the function $f\\left(a_{1}, \\ldots, a_{n}\\right)=\\sum_{i=1}^{n} \\frac{a_{i}}{\\sqrt{a_{1}^{2}+\\ldots+a_{i}^{2}}}$. One can see that rearranging the variables in increasing order can only increase the value of $f\\left(a_{1}, \\ldots, a_{n}\\right)$. Indeed, if $a_{j}>a_{j+1}$ for some index $j$ then we have $$ f\\left(a_{1}, \\ldots, a_{j-1}, a_{j+1}, a_{j}, a_{j+2}, \\ldots, a_{n}\\right)-f\\left(a_{1}, \\ldots, a_{n}\\right)=\\frac{a}{S}+\\frac{b}{\\sqrt{S^{2}-a^{2}}}-\\frac{b}{S}-\\frac{a}{\\sqrt{S^{2}-b^{2}}} $$ where $a=a_{j}, b=a_{j+1}$, and $S=\\sqrt{a_{1}^{2}+\\ldots+a_{j+1}^{2}}$. The positivity of the last expression above follows from $$ \\frac{b}{\\sqrt{S^{2}-a^{2}}}-\\frac{b}{S}=\\frac{a^{2} b}{S \\sqrt{S^{2}-a^{2}} \\cdot\\left(S+\\sqrt{S^{2}-a^{2}}\\right)}>\\frac{a b^{2}}{S \\sqrt{S^{2}-b^{2}} \\cdot\\left(S+\\sqrt{S^{2}-b^{2}}\\right)}=\\frac{a}{\\sqrt{S^{2}-b^{2}}}-\\frac{a}{S} . $$ Comment. If $ky-1$, hence $$ f(x)>\\frac{y-1}{f(y)} \\quad \\text { for all } x \\in \\mathbb{R}^{+} $$ If $y>1$, this provides a desired positive lower bound for $f(x)$. Now, let $M=\\inf _{x \\in \\mathbb{R}^{+}} f(x)>0$. Then, for all $y \\in \\mathbb{R}^{+}$, $$ M \\geqslant \\frac{y-1}{f(y)}, \\quad \\text { or } \\quad f(y) \\geqslant \\frac{y-1}{M} $$ Lemma 1. The function $f(x)$ (and hence $\\phi(x)$ ) is bounded on any segment $[p, q]$, where $0\\max \\left\\{C, \\frac{C}{y}\\right\\}$. Then all three numbers $x, x y$, and $x+f(x y)$ are greater than $C$, so (*) reads as $$ (x+x y+1)+1+y=(x+1) f(y)+1, \\text { hence } f(y)=y+1 $$ Comment 1. It may be useful to rewrite (*) in the form $$ \\phi(x+f(x y))+\\phi(x y)=\\phi(x) \\phi(y)+x \\phi(y)+y \\phi(x)+\\phi(x)+\\phi(y) $$ This general identity easily implies both (1) and (5). Comment 2. There are other ways to prove that $f(x) \\geqslant x+1$. Once one has proved this, they can use this stronger estimate instead of (3) in the proof of Lemma 1. Nevertheless, this does not make this proof simpler. So proving that $f(x) \\geqslant x+1$ does not seem to be a serious progress towards the solution of the problem. In what follows, we outline one possible proof of this inequality. First of all, we improve inequality (3) by noticing that, in fact, $f(x) f(y) \\geqslant y-1+M$, and hence $$ f(y) \\geqslant \\frac{y-1}{M}+1 $$ Now we divide the argument into two steps. Step 1: We show that $M \\leqslant 1$. Suppose that $M>1$; recall the notation $a=f(1)$. Substituting $y=1 \/ x$ in (*), we get $$ f(x+a)=f(x) f\\left(\\frac{1}{x}\\right)+1-\\frac{1}{x} \\geqslant M f(x), $$ provided that $x \\geqslant 1$. By a straightforward induction on $\\lceil(x-1) \/ a\\rceil$, this yields $$ f(x) \\geqslant M^{(x-1) \/ a} $$ Now choose an arbitrary $x_{0} \\in \\mathbb{R}^{+}$and define a sequence $x_{0}, x_{1}, \\ldots$ by $x_{n+1}=x_{n}+f\\left(x_{n}\\right) \\geqslant x_{n}+M$ for all $n \\geqslant 0$; notice that the sequence is unbounded. On the other hand, by (4) we get $$ a x_{n+1}>a f\\left(x_{n}\\right)=f\\left(x_{n+1}\\right) \\geqslant M^{\\left(x_{n+1}-1\\right) \/ a}, $$ which cannot hold when $x_{n+1}$ is large enough. Step 2: We prove that $f(y) \\geqslant y+1$ for all $y \\in \\mathbb{R}^{+}$. Arguing indirectly, choose $y \\in \\mathbb{R}^{+}$such that $f(y)n k$, so $a_{n} \\geqslant k+1$. Now the $n-k+1$ numbers $a_{k}, a_{k+1}, \\ldots, a_{n}$ are all greater than $k$; but there are only $n-k$ such values; this is not possible. If $a_{n}=n$ then $a_{1}, a_{2}, \\ldots, a_{n-1}$ must be a permutation of the numbers $1, \\ldots, n-1$ satisfying $a_{1} \\leqslant 2 a_{2} \\leqslant \\ldots \\leqslant(n-1) a_{n-1}$; there are $P_{n-1}$ such permutations. The last inequality in (*), $(n-1) a_{n-1} \\leqslant n a_{n}=n^{2}$, holds true automatically. If $\\left(a_{n-1}, a_{n}\\right)=(n, n-1)$, then $a_{1}, \\ldots, a_{n-2}$ must be a permutation of $1, \\ldots, n-2$ satisfying $a_{1} \\leqslant \\ldots \\leqslant(n-2) a_{n-2}$; there are $P_{n-2}$ such permutations. The last two inequalities in (*) hold true automatically by $(n-2) a_{n-2} \\leqslant(n-2)^{2}k$. If $t=k$ then we are done, so assume $t>k$. Notice that one of the numbers among the $t-k$ numbers $a_{k}, a_{k+1}, \\ldots, a_{t-1}$ is at least $t$, because there are only $t-k-1$ values between $k$ and $t$. Let $i$ be an index with $k \\leqslant ik+1$. Then the chain of inequalities $k t=k a_{k} \\leqslant \\ldots \\leqslant t a_{t}=k t$ should also turn into a chain of equalities. From this point we can find contradictions in several ways; for example by pointing to $a_{t-1}=\\frac{k t}{t-1}=k+\\frac{k}{t-1}$ which cannot be an integer, or considering the product of the numbers $(k+1) a_{k+1}, \\ldots,(t-1) a_{t-1}$; the numbers $a_{k+1}, \\ldots, a_{t-1}$ are distinct and greater than $k$, so $$ (k t)^{t-k-1}=(k+1) a_{k+1} \\cdot(k+2) a_{k+2} \\cdot \\ldots \\cdot(t-1) a_{t-1} \\geqslant((k+1)(k+2) \\cdot \\ldots \\cdot(t-1))^{2} $$ Notice that $(k+i)(t-i)=k t+i(t-k-i)>k t$ for $1 \\leqslant i(k t)^{t-k-1} $$ Therefore, the case $t>k+1$ is not possible.","tier":0} +{"problem_type":"Combinatorics","problem_label":"C2","problem":"In a regular 100-gon, 41 vertices are colored black and the remaining 59 vertices are colored white. Prove that there exist 24 convex quadrilaterals $Q_{1}, \\ldots, Q_{24}$ whose corners are vertices of the 100 -gon, so that - the quadrilaterals $Q_{1}, \\ldots, Q_{24}$ are pairwise disjoint, and - every quadrilateral $Q_{i}$ has three corners of one color and one corner of the other color. (Austria)","solution":"Call a quadrilateral skew-colored, if it has three corners of one color and one corner of the other color. We will prove the following Claim. If the vertices of a convex $(4 k+1)$-gon $P$ are colored black and white such that each color is used at least $k$ times, then there exist $k$ pairwise disjoint skew-colored quadrilaterals whose vertices are vertices of $P$. (One vertex of $P$ remains unused.) The problem statement follows by removing 3 arbitrary vertices of the 100-gon and applying the Claim to the remaining 97 vertices with $k=24$. Proof of the Claim. We prove by induction. For $k=1$ we have a pentagon with at least one black and at least one white vertex. If the number of black vertices is even then remove a black vertex; otherwise remove a white vertex. In the remaining quadrilateral, there are an odd number of black and an odd number of white vertices, so the quadrilateral is skew-colored. For the induction step, assume $k \\geqslant 2$. Let $b$ and $w$ be the numbers of black and white vertices, respectively; then $b, w \\geqslant k$ and $b+w=4 k+1$. Without loss of generality we may assume $w \\geqslant b$, so $k \\leqslant b \\leqslant 2 k$ and $2 k+1 \\leqslant w \\leqslant 3 k+1$. We want to find four consecutive vertices such that three of them are white, the fourth one is black. Denote the vertices by $V_{1}, V_{2}, \\ldots, V_{4 k+1}$ in counterclockwise order, such that $V_{4 k+1}$ is black, and consider the following $k$ groups of vertices: $$ \\left(V_{1}, V_{2}, V_{3}, V_{4}\\right),\\left(V_{5}, V_{6}, V_{7}, V_{8}\\right), \\ldots,\\left(V_{4 k-3}, V_{4 k-2}, V_{4 k-1}, V_{4 k}\\right) $$ In these groups there are $w$ white and $b-1$ black vertices. Since $w>b-1$, there is a group, $\\left(V_{i}, V_{i+1}, V_{i+2}, V_{i+3}\\right)$ that contains more white than black vertices. If three are white and one is black in that group, we are done. Otherwise, if $V_{i}, V_{i+1}, V_{i+2}, V_{i+3}$ are all white then let $V_{j}$ be the first black vertex among $V_{i+4}, \\ldots, V_{4 k+1}$ (recall that $V_{4 k+1}$ is black); then $V_{j-3}, V_{j-2}$ and $V_{j-1}$ are white and $V_{j}$ is black. Now we have four consecutive vertices $V_{i}, V_{i+1}, V_{i+2}, V_{i+3}$ that form a skew-colored quadrilateral. The remaining vertices form a convex ( $4 k-3$ )-gon; $w-3$ of them are white and $b-1$ are black. Since $b-1 \\geqslant k-1$ and $w-3 \\geqslant(2 k+1)-3>k-1$, we can apply the Claim with $k-1$. Comment. It is not true that the vertices of the 100 -gon can be split into 25 skew-colored quadrilaterals. A possible counter-example is when the vertices $V_{1}, V_{3}, V_{5}, \\ldots, V_{81}$ are black and the other vertices, $V_{2}, V_{4}, \\ldots, V_{80}$ and $V_{82}, V_{83}, \\ldots, V_{100}$ are white. For having 25 skew-colored quadrilaterals, there should be 8 containing three black vertices. But such a quadrilateral splits the other 96 vertices into four sets in such a way that at least two sets contain odd numbers of vertices and therefore they cannot be grouped into disjoint quadrilaterals. ![](https:\/\/cdn.mathpix.com\/cropped\/2024_11_18_bcdd154f2030efbe2fa4g-34.jpg?height=318&width=552&top_left_y=2462&top_left_x=752)","tier":0} +{"problem_type":"Combinatorics","problem_label":"C3","problem":"Let $n$ be an integer with $n \\geqslant 2$. On a slope of a mountain, $n^{2}$ checkpoints are marked, numbered from 1 to $n^{2}$ from the bottom to the top. Each of two cable car companies, $A$ and $B$, operates $k$ cable cars numbered from 1 to $k$; each cable car provides a transfer from some checkpoint to a higher one. For each company, and for any $i$ and $j$ with $1 \\leqslant im_{p-1}$. Choose $k$ to be the smallest index satisfying $m_{k}>m_{p-1}$; by our assumptions, we have $1\\left|R_{2}\\right|$. We will find a saddle subpair ( $R^{\\prime}, C^{\\prime}$ ) of ( $R_{1}, C_{1}$ ) with $\\left|R^{\\prime}\\right| \\leqslant\\left|R_{2}\\right|$; clearly, this implies the desired statement. Step 1: We construct maps $\\rho: R_{1} \\rightarrow R_{1}$ and $\\sigma: C_{1} \\rightarrow C_{1}$ such that $\\left|\\rho\\left(R_{1}\\right)\\right| \\leqslant\\left|R_{2}\\right|$, and $a\\left(\\rho\\left(r_{1}\\right), c_{1}\\right) \\geqslant a\\left(r_{1}, \\sigma\\left(c_{1}\\right)\\right)$ for all $r_{1} \\in R_{1}$ and $c_{1} \\in C_{1}$. Since $\\left(R_{1}, C_{1}\\right)$ is a saddle pair, for each $r_{2} \\in R_{2}$ there is $r_{1} \\in R_{1}$ such that $a\\left(r_{1}, c_{1}\\right) \\geqslant a\\left(r_{2}, c_{1}\\right)$ for all $c_{1} \\in C_{1}$; denote one such an $r_{1}$ by $\\rho_{1}\\left(r_{2}\\right)$. Similarly, we define four functions $$ \\begin{array}{llllll} \\rho_{1}: R_{2} \\rightarrow R_{1} & \\text { such that } & a\\left(\\rho_{1}\\left(r_{2}\\right), c_{1}\\right) \\geqslant a\\left(r_{2}, c_{1}\\right) & \\text { for all } & r_{2} \\in R_{2}, & c_{1} \\in C_{1} ; \\\\ \\rho_{2}: R_{1} \\rightarrow R_{2} & \\text { such that } & a\\left(\\rho_{2}\\left(r_{1}\\right), c_{2}\\right) \\geqslant a\\left(r_{1}, c_{2}\\right) & \\text { for all } & r_{1} \\in R_{1}, & c_{2} \\in C_{2} ; \\\\ \\sigma_{1}: C_{2} \\rightarrow C_{1} & \\text { such that } & a\\left(r_{1}, \\sigma_{1}\\left(c_{2}\\right)\\right) \\leqslant a\\left(r_{1}, c_{2}\\right) & \\text { for all } & r_{1} \\in R_{1}, & c_{2} \\in C_{2} ; \\\\ \\sigma_{2}: C_{1} \\rightarrow C_{2} & \\text { such that } & a\\left(r_{2}, \\sigma_{2}\\left(c_{1}\\right)\\right) \\leqslant a\\left(r_{2}, c_{1}\\right) & \\text { for all } & r_{2} \\in R_{2}, & c_{1} \\in C_{1} . \\end{array} $$ Set now $\\rho=\\rho_{1} \\circ \\rho_{2}: R_{1} \\rightarrow R_{1}$ and $\\sigma=\\sigma_{1} \\circ \\sigma_{2}: C_{1} \\rightarrow C_{1}$. We have $$ \\left|\\rho\\left(R_{1}\\right)\\right|=\\left|\\rho_{1}\\left(\\rho_{2}\\left(R_{1}\\right)\\right)\\right| \\leqslant\\left|\\rho_{1}\\left(R_{2}\\right)\\right| \\leqslant\\left|R_{2}\\right| . $$ Moreover, for all $r_{1} \\in R_{1}$ and $c_{1} \\in C_{1}$, we get $$ \\begin{aligned} a\\left(\\rho\\left(r_{1}\\right), c_{1}\\right)=a\\left(\\rho_{1}\\left(\\rho_{2}\\left(r_{1}\\right)\\right), c_{1}\\right) \\geqslant a\\left(\\rho_{2}\\left(r_{1}\\right), c_{1}\\right) & \\geqslant a\\left(\\rho_{2}\\left(r_{1}\\right), \\sigma_{2}\\left(c_{1}\\right)\\right) \\\\ & \\geqslant a\\left(r_{1}, \\sigma_{2}\\left(c_{1}\\right)\\right) \\geqslant a\\left(r_{1}, \\sigma_{1}\\left(\\sigma_{2}\\left(c_{1}\\right)\\right)\\right)=a\\left(r_{1}, \\sigma\\left(c_{1}\\right)\\right) \\end{aligned} $$ as desired. Step 2: Given maps $\\rho$ and $\\sigma$, we construct a proper saddle subpair $\\left(R^{\\prime}, C^{\\prime}\\right)$ of $\\left(R_{1}, C_{1}\\right)$. The properties of $\\rho$ and $\\sigma$ yield that $$ a\\left(\\rho^{i}\\left(r_{1}\\right), c_{1}\\right) \\geqslant a\\left(\\rho^{i-1}\\left(r_{1}\\right), \\sigma\\left(c_{1}\\right)\\right) \\geqslant \\ldots \\geqslant a\\left(r_{1}, \\sigma^{i}\\left(c_{1}\\right)\\right) $$ for each positive integer $i$ and all $r_{1} \\in R_{1}, c_{1} \\in C_{1}$. Consider the images $R^{i}=\\rho^{i}\\left(R_{1}\\right)$ and $C^{i}=\\sigma^{i}\\left(C_{1}\\right)$. Clearly, $R_{1}=R^{0} \\supseteq R^{1} \\supseteq R^{2} \\supseteq \\ldots$ and $C_{1}=C^{0} \\supseteq C^{1} \\supseteq C^{2} \\supseteq \\ldots$. Since both chains consist of finite sets, there is an index $n$ such that $R^{n}=R^{n+1}=\\ldots$ and $C^{n}=C^{n+1}=\\ldots$. Then $\\rho^{n}\\left(R^{n}\\right)=R^{2 n}=R^{n}$, so $\\rho^{n}$ restricted to $R^{n}$ is a bijection. Similarly, $\\sigma^{n}$ restricted to $C^{n}$ is a bijection from $C^{n}$ to itself. Therefore, there exists a positive integer $k$ such that $\\rho^{n k}$ acts identically on $R^{n}$, and $\\sigma^{n k}$ acts identically on $C^{n}$. We claim now that $\\left(R^{n}, C^{n}\\right)$ is a saddle subpair of $\\left(R_{1}, C_{1}\\right)$, with $\\left|R^{n}\\right| \\leqslant\\left|R^{1}\\right|=\\left|\\rho\\left(R_{1}\\right)\\right| \\leqslant$ $\\left|R_{2}\\right|$, which is what we needed. To check that this is a saddle pair, take any row $r^{\\prime}$; since $\\left(R_{1}, C_{1}\\right)$ is a saddle pair, there exists $r_{1} \\in R_{1}$ such that $a\\left(r_{1}, c_{1}\\right) \\geqslant a\\left(r^{\\prime}, c_{1}\\right)$ for all $c_{1} \\in C_{1}$. Set now $r_{*}=\\rho^{n k}\\left(r_{1}\\right) \\in R^{n}$. Then, for each $c \\in C^{n}$ we have $c=\\sigma^{n k}(c)$ and hence $$ a\\left(r_{*}, c\\right)=a\\left(\\rho^{n k}\\left(r_{1}\\right), c\\right) \\geqslant a\\left(r_{1}, \\sigma^{n k}(c)\\right)=a\\left(r_{1}, c\\right) \\geqslant a\\left(r^{\\prime}, c\\right) $$ which establishes condition $(i)$. Condition (ii) is checked similarly.","tier":0} +{"problem_type":"Combinatorics","problem_label":"C7","problem":"Consider any rectangular table having finitely many rows and columns, with a real number $a(r, c)$ in the cell in row $r$ and column $c$. A pair $(R, C)$, where $R$ is a set of rows and $C$ a set of columns, is called a saddle pair if the following two conditions are satisfied: (i) For each row $r^{\\prime}$, there is $r \\in R$ such that $a(r, c) \\geqslant a\\left(r^{\\prime}, c\\right)$ for all $c \\in C$; (ii) For each column $c^{\\prime}$, there is $c \\in C$ such that $a(r, c) \\leqslant a\\left(r, c^{\\prime}\\right)$ for all $r \\in R$. A saddle pair $(R, C)$ is called a minimal pair if for each saddle pair ( $R^{\\prime}, C^{\\prime}$ ) with $R^{\\prime} \\subseteq R$ and $C^{\\prime} \\subseteq C$, we have $R^{\\prime}=R$ and $C^{\\prime}=C$. Prove that any two minimal pairs contain the same number of rows. (Thailand)","solution":"Denote by $\\mathcal{R}$ and $\\mathcal{C}$ the set of all rows and the set of all columns of the table, respectively. Let $\\mathcal{T}$ denote the given table; for a set $R$ of rows and a set $C$ of columns, let $\\mathcal{T}[R, C]$ denote the subtable obtained by intersecting rows from $R$ and columns from $C$. We say that row $r_{1}$ exceeds row $r_{2}$ in range of columns $C$ (where $C \\subseteq \\mathcal{C}$ ) and write $r_{1} \\succeq_{C} r_{2}$ or $r_{2} \\leq_{C} r_{1}$, if $a\\left(r_{1}, c\\right) \\geqslant a\\left(r_{2}, c\\right)$ for all $c \\in C$. We say that a row $r_{1}$ is equal to a row $r_{2}$ in range of columns $C$ and write $r_{1} \\equiv_{C} r_{2}$, if $a\\left(r_{1}, c\\right)=a\\left(r_{2}, c\\right)$ for all $c \\in C$. We introduce similar notions, and use the same notation, for columns. Then conditions ( $i$ ) and (ii) in the definition of a saddle pair can be written as $(i)$ for each $r^{\\prime} \\in \\mathcal{R}$ there exists $r \\in R$ such that $r \\geq_{C} r^{\\prime}$; and (ii) for each $c^{\\prime} \\in \\mathcal{C}$ there exists $c \\in C$ such that $c \\leq_{R} c^{\\prime}$. Lemma. Suppose that $(R, C)$ is a minimal pair. Remove from the table several rows outside of $R$ and\/or several columns outside of $C$. Then $(R, C)$ remains a minimal pair in the new table. Proof. Obviously, $(R, C)$ remains a saddle pair. Suppose $\\left(R^{\\prime}, C^{\\prime}\\right)$ is a proper subpair of $(R, C)$. Since $(R, C)$ is a saddle pair, for each row $r^{*}$ of the initial table, there is a row $r \\in R$ such that $r \\geq_{C} r^{*}$. If ( $R^{\\prime}, C^{\\prime}$ ) became saddle after deleting rows not in $R$ and\/or columns not in $C$, there would be a row $r^{\\prime} \\in R^{\\prime}$ satisfying $r^{\\prime} \\geq_{C^{\\prime}} r$. Therefore, we would obtain that $r^{\\prime} \\geq_{C^{\\prime \\prime}} r^{*}$, which is exactly condition $(i)$ for the pair ( $R^{\\prime}, C^{\\prime}$ ) in the initial table; condition (ii) is checked similarly. Thus, ( $\\left.R^{\\prime}, C^{\\prime}\\right)$ was saddle in the initial table, which contradicts the hypothesis that $(R, C)$ was minimal. Hence, $(R, C)$ remains minimal after deleting rows and\/or columns. By the Lemma, it suffices to prove the statement of the problem in the case $\\mathcal{R}=R_{1} \\cup R_{2}$ and $\\mathcal{C}=C_{1} \\cup C_{2}$. Further, suppose that there exist rows that belong both to $R_{1}$ and $R_{2}$. Duplicate every such row, and refer one copy of it to the set $R_{1}$, and the other copy to the set $R_{2}$. Then $\\left(R_{1}, C_{1}\\right)$ and $\\left(R_{2}, C_{2}\\right)$ will remain minimal pairs in the new table, with the same numbers of rows and columns, but the sets $R_{1}$ and $R_{2}$ will become disjoint. Similarly duplicating columns in $C_{1} \\cap C_{2}$, we make $C_{1}$ and $C_{2}$ disjoint. Thus it is sufficient to prove the required statement in the case $R_{1} \\cap R_{2}=\\varnothing$ and $C_{1} \\cap C_{2}=\\varnothing$. The rest of the solution is devoted to the proof of the following claim including the statement of the problem. Claim. Suppose that $\\left(R_{1}, C_{1}\\right)$ and $\\left(R_{2}, C_{2}\\right)$ are minimal pairs in table $\\mathcal{T}$ such that $R_{2}=\\mathcal{R} \\backslash R_{1}$ and $C_{2}=\\mathcal{C} \\backslash C_{1}$. Then $\\left|R_{1}\\right|=\\left|R_{2}\\right|,\\left|C_{1}\\right|=\\left|C_{2}\\right| ;$ moreover, there are four bijections $$ \\begin{array}{llllll} \\rho_{1}: R_{2} \\rightarrow R_{1} & \\text { such that } & \\rho_{1}\\left(r_{2}\\right) \\equiv_{C_{1}} r_{2} & \\text { for all } & r_{2} \\in R_{2} ; \\\\ \\rho_{2}: R_{1} \\rightarrow R_{2} & \\text { such that } & \\rho_{2}\\left(r_{1}\\right) \\equiv_{C_{2}} r_{1} & \\text { for all } & r_{1} \\in R_{1} ; \\\\ \\sigma_{1}: C_{2} \\rightarrow C_{1} & \\text { such that } & \\sigma_{1}\\left(c_{2}\\right) \\equiv_{R_{1}} c_{2} & \\text { for all } & c_{2} \\in C_{2} ; \\\\ \\sigma_{2}: C_{1} \\rightarrow C_{2} & \\text { such that } & \\sigma_{2}\\left(c_{1}\\right) \\equiv_{R_{2}} c_{1} & \\text { for all } & c_{1} \\in C_{1} . \\end{array} $$ We prove the Claim by induction on $|\\mathcal{R}|+|\\mathcal{C}|$. In the base case we have $\\left|R_{1}\\right|=\\left|R_{2}\\right|=$ $\\left|C_{1}\\right|=\\left|C_{2}\\right|=1$; let $R_{i}=\\left\\{r_{i}\\right\\}$ and $C_{i}=\\left\\{c_{i}\\right\\}$. Since $\\left(R_{1}, C_{1}\\right)$ and $\\left(R_{2}, C_{2}\\right)$ are saddle pairs, we have $a\\left(r_{1}, c_{1}\\right) \\geqslant a\\left(r_{2}, c_{1}\\right) \\geqslant a\\left(r_{2}, c_{2}\\right) \\geqslant a\\left(r_{1}, c_{2}\\right) \\geqslant a\\left(r_{1}, c_{1}\\right)$, hence, the table consists of four equal numbers, and the statement follows. To prove the inductive step, introduce the maps $\\rho_{1}, \\rho_{2}, \\sigma_{1}$, and $\\sigma_{2}$ as in Solution 1 , see (1). Suppose first that all four maps are surjective. Then, in fact, we have $\\left|R_{1}\\right|=\\left|R_{2}\\right|,\\left|C_{1}\\right|=\\left|C_{2}\\right|$, and all maps are bijective. Moreover, for all $r_{2} \\in R_{2}$ and $c_{2} \\in C_{2}$ we have $$ \\begin{aligned} a\\left(r_{2}, c_{2}\\right) \\leqslant a\\left(r_{2}, \\sigma_{2}^{-1}\\left(c_{2}\\right)\\right) \\leqslant a\\left(\\rho_{1}\\left(r_{2}\\right), \\sigma_{2}^{-1}\\left(c_{2}\\right)\\right) \\leqslant a\\left(\\rho_{1}\\left(r_{2}\\right)\\right. & \\left., \\sigma_{1}^{-1} \\circ \\sigma_{2}^{-1}\\left(c_{2}\\right)\\right) \\\\ & \\leqslant a\\left(\\rho_{2} \\circ \\rho_{1}\\left(r_{2}\\right), \\sigma_{1}^{-1} \\circ \\sigma_{2}^{-1}\\left(c_{2}\\right)\\right) \\end{aligned} $$ Summing up, we get $$ \\sum_{\\substack{r_{2} \\in R_{2} \\\\ c_{2} \\in C_{2}}} a\\left(r_{2}, c_{2}\\right) \\leqslant \\sum_{\\substack{r_{2} \\in R_{2} \\\\ c_{2} \\in C_{2}}} a\\left(\\rho_{2} \\circ \\rho_{1}\\left(r_{2}\\right), \\sigma_{1}^{-1} \\circ \\sigma_{2}^{-1}\\left(c_{2}\\right)\\right) . $$ Since $\\rho_{1} \\circ \\rho_{2}$ and $\\sigma_{1}^{-1} \\circ \\sigma_{2}^{-1}$ are permutations of $R_{2}$ and $C_{2}$, respectively, this inequality is in fact equality. Therefore, all inequalities in (4) turn into equalities, which establishes the inductive step in this case. It remains to show that all four maps are surjective. For the sake of contradiction, we assume that $\\rho_{1}$ is not surjective. Now let $R_{1}^{\\prime}=\\rho_{1}\\left(R_{2}\\right)$ and $C_{1}^{\\prime}=\\sigma_{1}\\left(C_{2}\\right)$, and set $R^{*}=R_{1} \\backslash R_{1}^{\\prime}$ and $C^{*}=C_{1} \\backslash C_{1}^{\\prime}$. By our assumption, $R^{*} \\neq \\varnothing$. Let $\\mathcal{Q}$ be the table obtained from $\\mathcal{T}$ by removing the rows in $R^{*}$ and the columns in $C^{*}$; in other words, $\\mathcal{Q}=\\mathcal{T}\\left[R_{1}^{\\prime} \\cup R_{2}, C_{1}^{\\prime} \\cup C_{2}\\right]$. By the definition of $\\rho_{1}$, for each $r_{2} \\in R_{2}$ we have $\\rho_{1}\\left(r_{2}\\right) \\geq_{C_{1}} r_{2}$, so a fortiori $\\rho_{1}\\left(r_{2}\\right) \\geq_{C_{1}^{\\prime}} r_{2}$; moreover, $\\rho_{1}\\left(r_{2}\\right) \\in R_{1}^{\\prime}$. Similarly, $C_{1}^{\\prime} \\ni \\sigma_{1}\\left(c_{2}\\right) \\leq_{R_{1}^{\\prime}} c_{2}$ for each $c_{2} \\in C_{2}$. This means that $\\left(R_{1}^{\\prime}, C_{1}^{\\prime}\\right)$ is a saddle pair in $\\mathcal{Q}$. Recall that $\\left(R_{2}, C_{2}\\right)$ remains a minimal pair in $\\mathcal{Q}$, due to the Lemma. Therefore, $\\mathcal{Q}$ admits a minimal pair $\\left(\\bar{R}_{1}, \\bar{C}_{1}\\right)$ such that $\\bar{R}_{1} \\subseteq R_{1}^{\\prime}$ and $\\bar{C}_{1} \\subseteq C_{1}^{\\prime}$. For a minute, confine ourselves to the subtable $\\overline{\\mathcal{Q}}=\\mathcal{Q}\\left[\\bar{R}_{1} \\cup R_{2}, \\bar{C}_{1} \\cup C_{2}\\right]$. By the Lemma, the pairs $\\left(\\bar{R}_{1}, \\bar{C}_{1}\\right)$ and $\\left(R_{2}, C_{2}\\right)$ are also minimal in $\\overline{\\mathcal{Q}}$. By the inductive hypothesis, we have $\\left|R_{2}\\right|=\\left|\\bar{R}_{1}\\right| \\leqslant\\left|R_{1}^{\\prime}\\right|=\\left|\\rho_{1}\\left(R_{2}\\right)\\right| \\leqslant\\left|R_{2}\\right|$, so all these inequalities are in fact equalities. This implies that $\\bar{R}_{2}=R_{2}^{\\prime}$ and that $\\rho_{1}$ is a bijection $R_{2} \\rightarrow R_{1}^{\\prime}$. Similarly, $\\bar{C}_{1}=C_{1}^{\\prime}$, and $\\sigma_{1}$ is a bijection $C_{2} \\rightarrow C_{1}^{\\prime}$. In particular, $\\left(R_{1}^{\\prime}, C_{1}^{\\prime}\\right)$ is a minimal pair in $\\mathcal{Q}$. Now, by inductive hypothesis again, we have $\\left|R_{1}^{\\prime}\\right|=\\left|R_{2}\\right|,\\left|C_{1}^{\\prime}\\right|=\\left|C_{2}\\right|$, and there exist four bijections $$ \\begin{array}{ccccc} \\rho_{1}^{\\prime}: R_{2} \\rightarrow R_{1}^{\\prime} & \\text { such that } & \\rho_{1}^{\\prime}\\left(r_{2}\\right) \\equiv_{C_{1}^{\\prime}} r_{2} & \\text { for all } & r_{2} \\in R_{2} ; \\\\ \\rho_{2}^{\\prime}: R_{1}^{\\prime} \\rightarrow R_{2} & \\text { such that } & \\rho_{2}^{\\prime}\\left(r_{1}\\right) \\equiv_{C_{2}} r_{1} & \\text { for all } & r_{1} \\in R_{1}^{\\prime} ; \\\\ \\sigma_{1}^{\\prime}: C_{2} \\rightarrow C_{1}^{\\prime} & \\text { such that } & \\sigma_{1}^{\\prime}\\left(c_{2}\\right) \\equiv_{R_{1}^{\\prime}} c_{2} & \\text { for all } & c_{2} \\in C_{2} ; \\\\ \\sigma_{2}^{\\prime}: C_{1}^{\\prime} \\rightarrow C_{2} & \\text { such that } & \\sigma_{2}^{\\prime}\\left(c_{1}\\right) \\equiv_{R_{2}} c_{1} & \\text { for all } & c_{1} \\in C_{1}^{\\prime} . \\end{array} $$ Notice here that $\\sigma_{1}$ and $\\sigma_{1}^{\\prime}$ are two bijections $C_{2} \\rightarrow C_{1}^{\\prime}$ satisfying $\\sigma_{1}^{\\prime}\\left(c_{2}\\right) \\equiv_{R_{1}^{\\prime}} c_{2} \\geq_{R_{1}} \\sigma_{1}\\left(c_{2}\\right)$ for all $c_{2} \\in C_{2}$. Now, if $\\sigma_{1}^{\\prime}\\left(c_{2}\\right) \\neq \\sigma_{1}\\left(c_{2}\\right)$ for some $c_{2} \\in C_{2}$, then we could remove column $\\sigma_{1}^{\\prime}\\left(c_{2}\\right)$ from $C_{1}^{\\prime}$ obtaining another saddle pair $\\left(R_{1}^{\\prime}, C_{1}^{\\prime} \\backslash\\left\\{\\sigma_{1}^{\\prime}\\left(c_{2}\\right)\\right\\}\\right)$ in $\\mathcal{Q}$. This is impossible for a minimal pair $\\left(R_{1}^{\\prime}, C_{1}^{\\prime}\\right)$; hence the maps $\\sigma_{1}$ and $\\sigma_{1}^{\\prime}$ coincide. Now we are prepared to show that $\\left(R_{1}^{\\prime}, C_{1}^{\\prime}\\right)$ is a saddle pair in $\\mathcal{T}$, which yields a desired contradiction (since ( $R_{1}, C_{1}$ ) is not minimal). By symmetry, it suffices to find, for each $r^{\\prime} \\in \\mathcal{R}$, a row $r_{1} \\in R_{1}^{\\prime}$ such that $r_{1} \\geq_{C_{1}^{\\prime}} r^{\\prime}$. If $r^{\\prime} \\in R_{2}$, then we may put $r_{1}=\\rho_{1}\\left(r^{\\prime}\\right)$; so, in the sequel we assume $r^{\\prime} \\in R_{1}$. There exists $r_{2} \\in R_{2}$ such that $r^{\\prime} \\leq_{C_{2}} r_{2}$; set $r_{1}=\\left(\\rho_{2}^{\\prime}\\right)^{-1}\\left(r_{2}\\right) \\in R_{1}^{\\prime}$ and recall that $r_{1} \\equiv_{C_{2}}$ $r_{2} \\geq_{C_{2}} r^{\\prime}$. Therefore, implementing the bijection $\\sigma_{1}=\\sigma_{1}^{\\prime}$, for each $c_{1} \\in C_{1}^{\\prime}$ we get $$ a\\left(r^{\\prime}, c_{1}\\right) \\leqslant a\\left(r^{\\prime}, \\sigma_{1}^{-1}\\left(c_{1}\\right)\\right) \\leqslant a\\left(r_{1}, \\sigma_{1}^{-1}\\left(c_{1}\\right)\\right)=a\\left(r_{1}, \\sigma_{1}^{\\prime} \\circ \\sigma_{1}^{-1}\\left(c_{1}\\right)\\right)=a\\left(r_{1}, c_{1}\\right) $$ which shows $r^{\\prime} \\leq_{C_{1}^{\\prime}} r_{1}$, as desired. The inductive step is completed. Comment 1. For two minimal pairs $\\left(R_{1}, C_{1}\\right)$ and $\\left(R_{2}, C_{2}\\right)$, Solution 2 not only proves the required equalities $\\left|R_{1}\\right|=\\left|R_{2}\\right|$ and $\\left|C_{1}\\right|=\\left|C_{2}\\right|$, but also shows the existence of bijections (3). In simple words, this means that the four subtables $\\mathcal{T}\\left[R_{1}, C_{1}\\right], \\mathcal{T}\\left[R_{1}, C_{2}\\right], \\mathcal{T}\\left[R_{2}, C_{1}\\right]$, and $\\mathcal{T}\\left[R_{2}, C_{2}\\right]$ differ only by permuting rows\/columns. Notice that the existence of such bijections immediately implies that $\\left(R_{1}, C_{2}\\right)$ and $\\left(R_{2}, C_{1}\\right)$ are also minimal pairs. This stronger claim may also be derived directly from the arguments in Solution 1, even without the assumptions $R_{1} \\cap R_{2}=\\varnothing$ and $C_{1} \\cap C_{2}=\\varnothing$. Indeed, if $\\left|R_{1}\\right|=\\left|R_{2}\\right|$ and $\\left|C_{1}\\right|=\\left|C_{2}\\right|$, then similar arguments show that $R^{n}=R_{1}, C^{n}=C_{1}$, and for any $r \\in R^{n}$ and $c \\in C^{n}$ we have $$ a(r, c)=a\\left(\\rho^{n k}(r), c\\right) \\geqslant a\\left(\\rho^{n k-1}(r), \\sigma(c)\\right) \\geqslant \\ldots \\geqslant a\\left(r, \\sigma^{n k}(c)\\right)=a(r, c) . $$ This yields that all above inequalities turn into equalities. Moreover, this yields that all inequalities in (2) turn into equalities. Hence $\\rho_{1}, \\rho_{2}, \\sigma_{1}$, and $\\sigma_{2}$ satisfy (3). It is perhaps worth mentioning that one cannot necessarily find the maps in (3) so as to satisfy $\\rho_{1}=\\rho_{2}^{-1}$ and $\\sigma_{1}=\\sigma_{2}^{-1}$, as shown by the table below. | 1 | 0 | 0 | 1 | | :--- | :--- | :--- | :--- | | 0 | 1 | 1 | 0 | | 1 | 0 | 1 | 0 | | 0 | 1 | 0 | 1 | Comment 2. One may use the following, a bit more entertaining formulation of the same problem. On a specialized market, a finite number of products are being sold, and there are finitely many retailers each selling all the products by some prices. Say that retailer $r_{1}$ dominates retailer $r_{2}$ with respect to a set of products $P$ if $r_{1}$ 's price of each $p \\in P$ does not exceed $r_{2}$ 's price of $p$. Similarly, product $p_{1}$ exceeds product $p_{2}$ with respect to a set of retailers $R$, if $r$ 's price of $p_{1}$ is not less than $r$ 's price of $p_{2}$, for each $r \\in R$. Say that a set $R$ of retailers and a set $P$ of products form a saddle pair if for each retailer $r^{\\prime}$ there is $r \\in R$ dominating $r^{\\prime}$ with respect to $P$, and for each product $p^{\\prime}$ there is $p \\in P$ exceeding $p^{\\prime}$ with respect to $R$. A saddle pair $(R, P)$ is called a minimal pair if for each saddle pair $\\left(R^{\\prime}, P^{\\prime}\\right)$ with $R^{\\prime} \\subseteq R$ and $P^{\\prime} \\subseteq P$, we have $R^{\\prime}=R$ and $P^{\\prime}=P$. Prove that any two minimal pairs contain the same number of retailers.","tier":0} +{"problem_type":"Combinatorics","problem_label":"C8","problem":"Players $A$ and $B$ play a game on a blackboard that initially contains 2020 copies of the number 1. In every round, player $A$ erases two numbers $x$ and $y$ from the blackboard, and then player $B$ writes one of the numbers $x+y$ and $|x-y|$ on the blackboard. The game terminates as soon as, at the end of some round, one of the following holds: (1) one of the numbers on the blackboard is larger than the sum of all other numbers; (2) there are only zeros on the blackboard. Player $B$ must then give as many cookies to player $A$ as there are numbers on the blackboard. Player $A$ wants to get as many cookies as possible, whereas player $B$ wants to give as few as possible. Determine the number of cookies that $A$ receives if both players play optimally.","solution":"For a positive integer $n$, we denote by $S_{2}(n)$ the sum of digits in its binary representation. We prove that, in fact, if a board initially contains an even number $n>1$ of ones, then A can guarantee to obtain $S_{2}(n)$, but not more, cookies. The binary representation of 2020 is $2020=\\overline{11111100100}_{2}$, so $S_{2}(2020)=7$, and the answer follows. A strategy for $A$. At any round, while possible, A chooses two equal nonzero numbers on the board. Clearly, while $A$ can make such choice, the game does not terminate. On the other hand, $A$ can follow this strategy unless the game has already terminated. Indeed, if $A$ always chooses two equal numbers, then each number appearing on the board is either 0 or a power of 2 with non-negative integer exponent, this can be easily proved using induction on the number of rounds. At the moment when $A$ is unable to follow the strategy all nonzero numbers on the board are distinct powers of 2 . If the board contains at least one such power, then the largest of those powers is greater than the sum of the others. Otherwise there are only zeros on the blackboard, in both cases the game terminates. For every number on the board, define its range to be the number of ones it is obtained from. We can prove by induction on the number of rounds that for any nonzero number $k$ written by $B$ its range is $k$, and for any zero written by $B$ its range is a power of 2 . Thus at the end of each round all the ranges are powers of two, and their sum is $n$. Since $S_{2}(a+b) \\leqslant S_{2}(a)+S_{2}(b)$ for any positive integers $a$ and $b$, the number $n$ cannot be represented as a sum of less than $S_{2}(n)$ powers of 2 . Thus at the end of each round the board contains at least $S_{2}(n)$ numbers, while $A$ follows the above strategy. So $A$ can guarantee at least $S_{2}(n)$ cookies for himself. Comment. There are different proofs of the fact that the presented strategy guarantees at least $S_{2}(n)$ cookies for $A$. For instance, one may denote by $\\Sigma$ the sum of numbers on the board, and by $z$ the number of zeros. Then the board contains at least $S_{2}(\\Sigma)+z$ numbers; on the other hand, during the game, the number $S_{2}(\\Sigma)+z$ does not decrease, and its initial value is $S_{2}(n)$. The claim follows. A strategy for $B$. Denote $s=S_{2}(n)$. Let $x_{1}, \\ldots, x_{k}$ be the numbers on the board at some moment of the game after $B$ 's turn or at the beginning of the game. Say that a collection of $k$ signs $\\varepsilon_{1}, \\ldots, \\varepsilon_{k} \\in\\{+1,-1\\}$ is balanced if $$ \\sum_{i=1}^{k} \\varepsilon_{i} x_{i}=0 $$ We say that a situation on the board is good if $2^{s+1}$ does not divide the number of balanced collections. An appropriate strategy for $B$ can be explained as follows: Perform a move so that the situation remains good, while it is possible. We intend to show that in this case $B$ will not lose more than $S_{2}(n)$ cookies. For this purpose, we prove several lemmas. For a positive integer $k$, denote by $\\nu_{2}(k)$ the exponent of the largest power of 2 that divides $k$. Recall that, by Legendre's formula, $\\nu_{2}(n!)=n-S_{2}(n)$ for every positive integer $n$. Lemma 1. The initial situation is good. Proof. In the initial configuration, the number of balanced collections is equal to $\\binom{n}{n \/ 2}$. We have $$ \\nu_{2}\\left(\\binom{n}{n \/ 2}\\right)=\\nu_{2}(n!)-2 \\nu_{2}((n \/ 2)!)=\\left(n-S_{2}(n)\\right)-2\\left(\\frac{n}{2}-S_{2}(n \/ 2)\\right)=S_{2}(n)=s $$ Hence $2^{\\text {s+1 }}$ does not divide the number of balanced collections, as desired. Lemma 2. B may play so that after each round the situation remains good. Proof. Assume that the situation $\\left(x_{1}, \\ldots, x_{k}\\right)$ before a round is good, and that $A$ erases two numbers, $x_{p}$ and $x_{q}$. Let $N$ be the number of all balanced collections, $N_{+}$be the number of those having $\\varepsilon_{p}=\\varepsilon_{q}$, and $N_{-}$be the number of other balanced collections. Then $N=N_{+}+N_{-}$. Now, if $B$ replaces $x_{p}$ and $x_{q}$ by $x_{p}+x_{q}$, then the number of balanced collections will become $N_{+}$. If $B$ replaces $x_{p}$ and $x_{q}$ by $\\left|x_{p}-x_{q}\\right|$, then this number will become $N_{-}$. Since $2^{s+1}$ does not divide $N$, it does not divide one of the summands $N_{+}$and $N_{-}$, hence $B$ can reach a good situation after the round. Lemma 3. Assume that the game terminates at a good situation. Then the board contains at most $s$ numbers. Proof. Suppose, one of the numbers is greater than the sum of the other numbers. Then the number of balanced collections is 0 and hence divisible by $2^{s+1}$. Therefore, the situation is not good. Then we have only zeros on the blackboard at the moment when the game terminates. If there are $k$ of them, then the number of balanced collections is $2^{k}$. Since the situation is good, we have $k \\leqslant s$. By Lemmas 1 and 2, $B$ may act in such way that they keep the situation good. By Lemma 3, when the game terminates, the board contains at most $s$ numbers. This is what we aimed to prove. Comment 1. If the initial situation had some odd number $n>1$ of ones on the blackboard, player $A$ would still get $S_{2}(n)$ cookies, provided that both players act optimally. The proof of this fact is similar to the solution above, after one makes some changes in the definitions. Such changes are listed below. Say that a collection of $k$ signs $\\varepsilon_{1}, \\ldots, \\varepsilon_{k} \\in\\{+1,-1\\}$ is positive if $$ \\sum_{i=1}^{k} \\varepsilon_{i} x_{i}>0 $$ For every index $i=1,2, \\ldots, k$, we denote by $N_{i}$ the number of positive collections such that $\\varepsilon_{i}=1$. Finally, say that a situation on the board is good if $2^{s-1}$ does not divide at least one of the numbers $N_{i}$. Now, a strategy for $B$ again consists in preserving the situation good, after each round. Comment 2. There is an easier strategy for $B$, allowing, in the game starting with an even number $n$ of ones, to lose no more than $d=\\left\\lfloor\\log _{2}(n+2)\\right\\rfloor-1$ cookies. If the binary representation of $n$ contains at most two zeros, then $d=S_{2}(n)$, and hence the strategy is optimal in that case. We outline this approach below. First of all, we can assume that $A$ never erases zeros from the blackboard. Indeed, $A$ may skip such moves harmlessly, ignoring the zeros in the further process; this way, $A$ 's win will just increase. We say that a situation on the blackboard is pretty if the numbers on the board can be partitioned into two groups with equal sums. Clearly, if the situation before some round is pretty, then $B$ may play so as to preserve this property after the round. The strategy for $B$ is as follows: - $B$ always chooses a move that leads to a pretty situation. - If both possible moves of $B$ lead to pretty situations, then $B$ writes the sum of the two numbers erased by $A$. Since the situation always remains pretty, the game terminates when all numbers on the board are zeros. Suppose that, at the end of the game, there are $m \\geqslant d+1=\\left\\lfloor\\log _{2}(n+2)\\right\\rfloor$ zeros on the board; then $2^{m}-1>n \/ 2$. Now we analyze the whole process of the play. Let us number the zeros on the board in order of appearance. During the play, each zero had appeared after subtracting two equal numbers. Let $n_{1}, \\ldots, n_{m}$ be positive integers such that the first zero appeared after subtracting $n_{1}$ from $n_{1}$, the second zero appeared after subtracting $n_{2}$ from $n_{2}$, and so on. Since the sum of the numbers on the blackboard never increases, we have $2 n_{1}+\\ldots+2 n_{m} \\leqslant n$, hence $$ n_{1}+\\ldots+n_{m} \\leqslant n \/ 2<2^{m}-1 $$ There are $2^{m}$ subsets of the set $\\{1,2, \\ldots, m\\}$. For $I \\subseteq\\{1,2, \\ldots, m\\}$, denote by $f(I)$ the sum $\\sum_{i \\in I} n_{i}$. There are less than $2^{m}$ possible values for $f(I)$, so there are two distinct subsets $I$ and $J$ with $f(I)=f(J)$. Replacing $I$ and $J$ with $I \\backslash J$ and $J \\backslash I$, we assume that $I$ and $J$ are disjoint. Let $i_{0}$ be the smallest number in $I \\cup J$; without loss of generality, $i_{0} \\in I$. Consider the round when $A$ had erased two numbers equal to $n_{i_{0}}$, and $B$ had put the $i_{0}{ }^{\\text {th }}$ zero instead, and the situation before that round. For each nonzero number $z$ which is on the blackboard at this moment, we can keep track of it during the further play until it enters one of the numbers $n_{i}, i \\geqslant i_{0}$, which then turn into zeros. For every $i=i_{0}, i_{0}+1, \\ldots, m$, we denote by $X_{i}$ the collection of all numbers on the blackboard that finally enter the first copy of $n_{i}$, and by $Y_{i}$ the collection of those finally entering the second copy of $n_{i}$. Thus, each of $X_{i_{0}}$ and $Y_{i_{0}}$ consists of a single number. Since $A$ never erases zeros, the 2(m-iol $i_{0}$ ) defined sets are pairwise disjoint. Clearly, for either of the collections $X_{i}$ and $Y_{i}$, a signed sum of its elements equals $n_{i}$, for a proper choice of the signs. Therefore, for every $i=i_{0}, i_{0}+1, \\ldots, m$ one can endow numbers in $X_{i} \\cup Y_{i}$ with signs so that their sum becomes any of the numbers $-2 n_{i}, 0$, or $2 n_{i}$. Perform such endowment so as to get $2 n_{i}$ from each collection $X_{i} \\cup Y_{i}$ with $i \\in I,-2 n_{j}$ from each collection $X_{j} \\cup Y_{j}$ with $j \\in J$, and 0 from each remaining collection. The obtained signed sum of all numbers on the blackboard equals $$ \\sum_{i \\in I} 2 n_{i}-\\sum_{i \\in J} 2 n_{i}=0 $$ and the numbers in $X_{i_{0}}$ and $Y_{i_{0}}$ have the same (positive) sign. This means that, at this round, $B$ could add up the two numbers $n_{i_{0}}$ to get a pretty situation. According to the strategy, $B$ should have performed that, instead of subtracting the numbers. This contradiction shows that $m \\leqslant d$, as desired. This page is intentionally left blank","tier":0} +{"problem_type":"Geometry","problem_label":"G1","problem":"Let $A B C$ be an isosceles triangle with $B C=C A$, and let $D$ be a point inside side $A B$ such that $A D90^{\\circ}, \\angle C D A>90^{\\circ}$, and $\\angle D A B=\\angle B C D$. Denote by $E$ and $F$ the reflections of $A$ in lines $B C$ and $C D$, respectively. Suppose that the segments $A E$ and $A F$ meet the line $B D$ at $K$ and $L$, respectively. Prove that the circumcircles of triangles $B E K$ and $D F L$ are tangent to each other. (Slovakia)","solution":"Denote by $A^{\\prime}$ the reflection of $A$ in $B D$. We will show that that the quadrilaterals $A^{\\prime} B K E$ and $A^{\\prime} D L F$ are cyclic, and their circumcircles are tangent to each other at point $A^{\\prime}$. From the symmetry about line $B C$ we have $\\angle B E K=\\angle B A K$, while from the symmetry in $B D$ we have $\\angle B A K=\\angle B A^{\\prime} K$. Hence $\\angle B E K=\\angle B A^{\\prime} K$, which implies that the quadrilateral $A^{\\prime} B K E$ is cyclic. Similarly, the quadrilateral $A^{\\prime} D L F$ is also cyclic. ![](https:\/\/cdn.mathpix.com\/cropped\/2024_11_18_bcdd154f2030efbe2fa4g-54.jpg?height=632&width=1229&top_left_y=818&top_left_x=419) For showing that circles $A^{\\prime} B K E$ and $A^{\\prime} D L F$ are tangent it suffices to prove that $$ \\angle A^{\\prime} K B+\\angle A^{\\prime} L D=\\angle B A^{\\prime} D . $$ Indeed, by $A K \\perp B C$, $A L \\perp C D$, and again the symmetry in $B D$ we have $$ \\angle A^{\\prime} K B+\\angle A^{\\prime} L D=180^{\\circ}-\\angle K A^{\\prime} L=180^{\\circ}-\\angle K A L=\\angle B C D=\\angle B A D=\\angle B A^{\\prime} D, $$ as required. Comment 1. The key to the solution above is introducing the point $A^{\\prime}$; then the angle calculations can be done in many different ways.","tier":0} +{"problem_type":"Geometry","problem_label":"G3","problem":"Let $A B C D$ be a convex quadrilateral with $\\angle A B C>90^{\\circ}, \\angle C D A>90^{\\circ}$, and $\\angle D A B=\\angle B C D$. Denote by $E$ and $F$ the reflections of $A$ in lines $B C$ and $C D$, respectively. Suppose that the segments $A E$ and $A F$ meet the line $B D$ at $K$ and $L$, respectively. Prove that the circumcircles of triangles $B E K$ and $D F L$ are tangent to each other. (Slovakia)","solution":"Note that $\\angle K A L=180^{\\circ}-\\angle B C D$, since $A K$ and $A L$ are perpendicular to $B C$ and $C D$, respectively. Reflect both circles $(B E K)$ and $(D F L)$ in $B D$. Since $\\angle K E B=\\angle K A B$ and $\\angle D F L=\\angle D A L$, the images are the circles $(K A B)$ and $(L A D)$, respectively; so they meet at $A$. We need to prove that those two reflections are tangent at $A$. For this purpose, we observe that $$ \\angle A K B+\\angle A L D=180^{\\circ}-\\angle K A L=\\angle B C D=\\angle B A D . $$ Thus, there exists a ray $A P$ inside angle $\\angle B A D$ such that $\\angle B A P=\\angle A K B$ and $\\angle D A P=$ $\\angle D L A$. Hence the line $A P$ is a common tangent to the circles $(K A B)$ and $(L A D)$, as desired. Comment 2. The statement of the problem remains true for a more general configuration, e.g., if line $B D$ intersect the extension of segment $A E$ instead of the segment itself, etc. The corresponding restrictions in the statement are given to reduce case sensitivity.","tier":0} +{"problem_type":"Geometry","problem_label":"G4","problem":"In the plane, there are $n \\geqslant 6$ pairwise disjoint disks $D_{1}, D_{2}, \\ldots, D_{n}$ with radii $R_{1} \\geqslant R_{2} \\geqslant \\ldots \\geqslant R_{n}$. For every $i=1,2, \\ldots, n$, a point $P_{i}$ is chosen in disk $D_{i}$. Let $O$ be an arbitrary point in the plane. Prove that $$ O P_{1}+O P_{2}+\\ldots+O P_{n} \\geqslant R_{6}+R_{7}+\\ldots+R_{n} $$ (A disk is assumed to contain its boundary.) (Iran)","solution":"We will make use of the following lemma. Lemma. Let $D_{1}, \\ldots, D_{6}$ be disjoint disks in the plane with radii $R_{1}, \\ldots, R_{6}$. Let $P_{i}$ be a point in $D_{i}$, and let $O$ be an arbitrary point. Then there exist indices $i$ and $j$ such that $O P_{i} \\geqslant R_{j}$. Proof. Let $O_{i}$ be the center of $D_{i}$. Consider six rays $O O_{1}, \\ldots, O O_{6}$ (if $O=O_{i}$, then the ray $O O_{i}$ may be assumed to have an arbitrary direction). These rays partition the plane into six angles (one of which may be non-convex) whose measures sum up to $360^{\\circ}$; hence one of the angles, say $\\angle O_{i} O O_{j}$, has measure at most $60^{\\circ}$. Then $O_{i} O_{j}$ cannot be the unique largest side in (possibly degenerate) triangle $O O_{i} O_{j}$, so, without loss of generality, $O O_{i} \\geqslant O_{i} O_{j} \\geqslant R_{i}+R_{j}$. Therefore, $O P_{i} \\geqslant O O_{i}-R_{i} \\geqslant\\left(R_{i}+R_{j}\\right)-R_{i}=R_{j}$, as desired. Now we prove the required inequality by induction on $n \\geqslant 5$. The base case $n=5$ is trivial. For the inductive step, apply the Lemma to the six largest disks, in order to find indices $i$ and $j$ such that $1 \\leqslant i, j \\leqslant 6$ and $O P_{i} \\geqslant R_{j} \\geqslant R_{6}$. Removing $D_{i}$ from the configuration and applying the inductive hypothesis, we get $$ \\sum_{k \\neq i} O P_{k} \\geqslant \\sum_{\\ell \\geqslant 7} R_{\\ell} . $$ Adding up this inequality with $O P_{i} \\geqslant R_{6}$ we establish the inductive step. Comment 1. It is irrelevant to the problem whether the disks contain their boundaries or not. This condition is included for clarity reasons only. The problem statement remains true, and the solution works verbatim, if the disks are assumed to have disjoint interiors. Comment 2. There are several variations of the above solution. In particular, while performing the inductive step, one may remove the disk with the largest value of $O P_{i}$ and apply the inductive hypothesis to the remaining disks (the Lemma should still be applied to the six largest disks). Comment 3. While proving the Lemma, one may reduce it to a particular case when the disks are congruent, as follows: Choose the smallest radius $r$ of the disks in the Lemma statement, and then replace, for each $i$, the $i^{\\text {th }}$ disk with its homothetic copy, using the homothety centered at $P_{i}$ with ratio $r \/ R_{i}$. This argument shows that the Lemma is tightly connected to a circle packing problem, see, e.g., https:\/\/en.wikipedia.org\/wiki\/Circle_packing_in_a_circle. The known results on that problem provide versions of the Lemma for different numbers of disks, which lead to different inequalities of the same kind. E.g., for 4 disks the best possible estimate in the Lemma is $O P_{i} \\geqslant(\\sqrt{2}-1) R_{j}$, while for 13 disks it has the form $O P_{i} \\geqslant \\sqrt{5} R_{j}$. Arguing as in the above solution, one obtains the inequalities $$ \\sum_{i=1}^{n} O P_{i} \\geqslant(\\sqrt{2}-1) \\sum_{j=4}^{n} R_{j} \\quad \\text { and } \\quad \\sum_{i=1}^{n} O P_{i} \\geqslant \\sqrt{5} \\sum_{j=13}^{n} R_{j} . $$ However, there are some harder arguments which allow to improve these inequalities, meaning that the $R_{j}$ with large indices may be taken with much greater factors.","tier":0} +{"problem_type":"Geometry","problem_label":"G5","problem":"Let $A B C D$ be a cyclic quadrilateral with no two sides parallel. Let $K, L, M$, and $N$ be points lying on sides $A B, B C, C D$, and $D A$, respectively, such that $K L M N$ is a rhombus with $K L \\| A C$ and $L M \\| B D$. Let $\\omega_{1}, \\omega_{2}, \\omega_{3}$, and $\\omega_{4}$ be the incircles of triangles $A N K$, $B K L, C L M$, and $D M N$, respectively. Prove that the internal common tangents to $\\omega_{1}$ and $\\omega_{3}$ and the internal common tangents to $\\omega_{2}$ and $\\omega_{4}$ are concurrent. (Poland)","solution":"Let $I_{i}$ be the center of $\\omega_{i}$, and let $r_{i}$ be its radius for $i=1,2,3,4$. Denote by $T_{1}$ and $T_{3}$ the points of tangency of $\\omega_{1}$ and $\\omega_{3}$ with $N K$ and $L M$, respectively. Suppose that the internal common tangents to $\\omega_{1}$ and $\\omega_{3}$ meet at point $S$, which is the center of homothety $h$ with negative ratio (namely, with ratio $-\\frac{r_{3}}{r_{1}}$ ) mapping $\\omega_{1}$ to $\\omega_{3}$. This homothety takes $T_{1}$ to $T_{3}$ (since the tangents to $\\omega_{1}$ and $\\omega_{3}$ at $T_{1}$ to $T_{3}$ are parallel), hence $S$ is a point on the segment $T_{1} T_{3}$ with $T_{1} S: S T_{3}=r_{1}: r_{3}$. Construct segments $S_{1} S_{3} \\| K L$ and $S_{2} S_{4} \\| L M$ through $S$ with $S_{1} \\in N K, S_{2} \\in K L$, $S_{3} \\in L M$, and $S_{4} \\in M N$. Note that $h$ takes $S_{1}$ to $S_{3}$, hence $I_{1} S_{1} \\| I_{3} S_{3}$, and $S_{1} S: S S_{3}=r_{1}: r_{3}$. We will prove that $S_{2} S: S S_{4}=r_{2}: r_{4}$ or, equivalently, $K S_{1}: S_{1} N=r_{2}: r_{4}$. This will yield the problem statement; indeed, applying similar arguments to the intersection point $S^{\\prime}$ of the internal common tangents to $\\omega_{2}$ and $\\omega_{4}$, we see that $S^{\\prime}$ satisfies similar relations, and there is a unique point inside $K L M N$ satisfying them. Therefore, $S^{\\prime}=S$. ![](https:\/\/cdn.mathpix.com\/cropped\/2024_11_18_bcdd154f2030efbe2fa4g-56.jpg?height=670&width=1409&top_left_y=1207&top_left_x=326) Further, denote by $I_{A}, I_{B}, I_{C}, I_{D}$ and $r_{A}, r_{B}, r_{C}, r_{D}$ the incenters and inradii of triangles $D A B, A B C, B C D$, and $C D A$, respectively. One can shift triangle $C L M$ by $\\overrightarrow{L K}$ to glue it with triangle $A K N$ into a quadrilateral $A K C^{\\prime} N$ similar to $A B C D$. In particular, this shows that $r_{1}: r_{3}=r_{A}: r_{C}$; similarly, $r_{2}: r_{4}=r_{B}: r_{D}$. Moreover, the same shift takes $S_{3}$ to $S_{1}$, and it also takes $I_{3}$ to the incenter $I_{3}^{\\prime}$ of triangle $K C^{\\prime} N$. Since $I_{1} S_{1} \\| I_{3} S_{3}$, the points $I_{1}, S_{1}, I_{3}^{\\prime}$ are collinear. Thus, in order to complete the solution, it suffices to apply the following Lemma to quadrilateral $A K C^{\\prime} N$. Lemma 1. Let $A B C D$ be a cyclic quadrilateral, and define $I_{A}, I_{C}, r_{B}$, and $r_{D}$ as above. Let $I_{A} I_{C}$ meet $B D$ at $X$; then $B X: X D=r_{B}: r_{D}$. Proof. Consider an inversion centered at $X$; the images under that inversion will be denoted by primes, e.g., $A^{\\prime}$ is the image of $A$. By properties of inversion, we have $$ \\angle I_{C}^{\\prime} I_{A}^{\\prime} D^{\\prime}=\\angle X I_{A}^{\\prime} D^{\\prime}=\\angle X D I_{A}=\\angle B D A \/ 2=\\angle B C A \/ 2=\\angle A C I_{B} $$ We obtain $\\angle I_{A}^{\\prime} I_{C}^{\\prime} D^{\\prime}=\\angle C A I_{B}$ likewise; therefore, $\\triangle I_{C}^{\\prime} I_{A}^{\\prime} D^{\\prime} \\sim \\triangle A C I_{B}$. In the same manner, we get $\\triangle I_{C}^{\\prime} I_{A}^{\\prime} B^{\\prime} \\sim \\triangle A C I_{D}$, hence the quadrilaterals $I_{C}^{\\prime} B^{\\prime} I_{A}^{\\prime} D^{\\prime}$ and $A I_{D} C I_{B}$ are also similar. But the diagonals $A C$ and $I_{B} I_{D}$ of quadrilateral $A I_{D} C I_{B}$ meet at a point $Y$ such that $I_{B} Y$ : $Y I_{D}=r_{B}: r_{D}$. By similarity, we get $D^{\\prime} X: B^{\\prime} X=r_{B}: r_{D}$ and hence $B X: X D=D^{\\prime} X:$ $B^{\\prime} X=r_{B}: r_{D}$. Comment 1. The solution above shows that the problem statement holds also for any parallelogram $K L M N$ whose sides are parallel to the diagonals of $A B C D$, as no property specific for a rhombus has been used. This solution works equally well when two sides of quadrilateral $A B C D$ are parallel. Comment 2. The problem may be reduced to Lemma 1 by using different tools, e.g., by using mass point geometry, linear motion of $K, L, M$, and $N$, etc. Lemma 1 itself also can be proved in different ways. We present below one alternative proof. Proof. In the circumcircle of $A B C D$, let $K^{\\prime}, L^{\\prime} . M^{\\prime}$, and $N^{\\prime}$ be the midpoints of arcs $A B, B C$, $C D$, and $D A$ containing no other vertices of $A B C D$, respectively. Thus, $K^{\\prime}=C I_{B} \\cap D I_{A}$, etc. In the computations below, we denote by $[P]$ the area of a polygon $P$. We use similarities $\\triangle I_{A} B K^{\\prime} \\sim$ $\\triangle I_{A} D N^{\\prime}, \\triangle I_{B} K^{\\prime} L^{\\prime} \\sim \\triangle I_{B} A C$, etc., as well as congruences $\\triangle I_{B} K^{\\prime} L^{\\prime}=\\triangle B K^{\\prime} L^{\\prime}$ and $\\triangle I_{D} M^{\\prime} N^{\\prime}=$ $\\triangle D M^{\\prime} N^{\\prime}$ (e.g., the first congruence holds because $K^{\\prime} L^{\\prime}$ is a common internal bisector of angles $B K^{\\prime} I_{B}$ and $B L^{\\prime} I_{B}$ ). We have $$ \\begin{aligned} & \\frac{B X}{D X}=\\frac{\\left[I_{A} B I_{C}\\right]}{\\left[I_{A} D I_{C}\\right]}=\\frac{B I_{A} \\cdot B I_{C} \\cdot \\sin I_{A} B I_{C}}{D I_{A} \\cdot D I_{C} \\cdot \\sin I_{A} D I_{C}}=\\frac{B I_{A}}{D I_{A}} \\cdot \\frac{B I_{C}}{D I_{C}} \\cdot \\frac{\\sin N^{\\prime} B M^{\\prime}}{\\sin K^{\\prime} D L^{\\prime}} \\\\ & =\\frac{B K^{\\prime}}{D N^{\\prime}} \\cdot \\frac{B L^{\\prime}}{D M^{\\prime}} \\cdot \\frac{\\sin N^{\\prime} D M^{\\prime}}{\\sin K^{\\prime} B L^{\\prime}}=\\frac{B K^{\\prime} \\cdot B L^{\\prime} \\cdot \\sin K^{\\prime} B L^{\\prime}}{D N^{\\prime} \\cdot D M^{\\prime} \\cdot \\sin N^{\\prime} D M^{\\prime}} \\cdot \\frac{\\sin ^{2} N^{\\prime} D M^{\\prime}}{\\sin ^{2} K^{\\prime} B L^{\\prime}} \\\\ & \\quad=\\frac{\\left[K^{\\prime} B L^{\\prime}\\right]}{\\left[M^{\\prime} D N^{\\prime}\\right]} \\cdot \\frac{N^{\\prime} M^{\\prime 2}}{K^{\\prime} L^{\\prime 2}}=\\frac{\\left[K^{\\prime} I_{B} L^{\\prime}\\right] \\cdot \\frac{A^{\\prime} C^{\\prime 2}}{K^{\\prime} L^{\\prime 2}}}{\\left[M^{\\prime} I_{D} N^{\\prime}\\right] \\cdot \\frac{A^{\\prime} C^{\\prime 2}}{N^{\\prime} M^{\\prime 2}}}=\\frac{\\left[A I_{B} C\\right]}{\\left[A I_{D} C\\right]}=\\frac{r_{B}}{r_{D}}, \\end{aligned} $$ as required.","tier":0} +{"problem_type":"Geometry","problem_label":"G5","problem":"Let $A B C D$ be a cyclic quadrilateral with no two sides parallel. Let $K, L, M$, and $N$ be points lying on sides $A B, B C, C D$, and $D A$, respectively, such that $K L M N$ is a rhombus with $K L \\| A C$ and $L M \\| B D$. Let $\\omega_{1}, \\omega_{2}, \\omega_{3}$, and $\\omega_{4}$ be the incircles of triangles $A N K$, $B K L, C L M$, and $D M N$, respectively. Prove that the internal common tangents to $\\omega_{1}$ and $\\omega_{3}$ and the internal common tangents to $\\omega_{2}$ and $\\omega_{4}$ are concurrent. (Poland)","solution":"This solution is based on the following general Lemma. Lemma 2. Let $E$ and $F$ be distinct points, and let $\\omega_{i}, i=1,2,3,4$, be circles lying in the same halfplane with respect to $E F$. For distinct indices $i, j \\in\\{1,2,3,4\\}$, denote by $O_{i j}^{+}$ (respectively, $O_{i j}^{-}$) the center of homothety with positive (respectively, negative) ratio taking $\\omega_{i}$ to $\\omega_{j}$. Suppose that $E=O_{12}^{+}=O_{34}^{+}$and $F=O_{23}^{+}=O_{41}^{+}$. Then $O_{13}^{-}=O_{24}^{-}$. Proof. Applying Monge's theorem to triples of circles $\\omega_{1}, \\omega_{2}, \\omega_{4}$ and $\\omega_{1}, \\omega_{3}, \\omega_{4}$, we get that both points $O_{24}^{-}$and $O_{13}^{-}$lie on line $E O_{14}^{-}$. Notice that this line is distinct from $E F$. Similarly we obtain that both points $O_{24}^{-}$and $O_{13}^{-}$lie on $F O_{34}^{-}$. Since the lines $E O_{14}^{-}$and $F O_{34}^{-}$are distinct, both points coincide with the meeting point of those lines. ![](https:\/\/cdn.mathpix.com\/cropped\/2024_11_18_bcdd154f2030efbe2fa4g-57.jpg?height=716&width=1837&top_left_y=2073&top_left_x=108) Turning back to the problem, let $A B$ intersect $C D$ at $E$ and let $B C$ intersect $D A$ at $F$. Assume, without loss of generality, that $B$ lies on segments $A E$ and $C F$. We will show that the points $E$ and $F$, and the circles $\\omega_{i}$ satisfy the conditions of Lemma 2 , so the problem statement follows. In the sequel, we use the notation of $O_{i j}^{ \\pm}$from the statement of Lemma 2, applied to circles $\\omega_{1}, \\omega_{2}, \\omega_{3}$, and $\\omega_{4}$. Using the relations $\\triangle E C A \\sim \\triangle E B D, K N \\| B D$, and $M N \\| A C$. we get $$ \\frac{A N}{N D}=\\frac{A N}{A D} \\cdot \\frac{A D}{N D}=\\frac{K N}{B D} \\cdot \\frac{A C}{N M}=\\frac{A C}{B D}=\\frac{A E}{E D} $$ Therefore, by the angle bisector theorem, point $N$ lies on the internal angle bisector of $\\angle A E D$. We prove similarly that $L$ also lies on that bisector, and that the points $K$ and $M$ lie on the internal angle bisector of $\\angle A F B$. Since $K L M N$ is a rhombus, points $K$ and $M$ are symmetric in line $E L N$. Hence, the convex quadrilateral determined by the lines $E K, E M, K L$, and $M L$ is a kite, and therefore it has an incircle $\\omega_{0}$. Applying Monge's theorem to $\\omega_{0}, \\omega_{2}$, and $\\omega_{3}$, we get that $O_{23}^{+}$lies on $K M$. On the other hand, $O_{23}^{+}$lies on $B C$, as $B C$ is an external common tangent to $\\omega_{2}$ and $\\omega_{3}$. It follows that $F=O_{23}^{+}$. Similarly, $E=O_{12}^{+}=O_{34}^{+}$, and $F=O_{41}^{+}$. Comment 3. The reduction to Lemma 2 and the proof of Lemma 2 can be performed with the use of different tools, e.g., by means of Menelaus theorem, by projecting harmonic quadruples, by applying Monge's theorem in some other ways, etc. This page is intentionally left blank","tier":0} +{"problem_type":"Geometry","problem_label":"G6","problem":"Let $I$ and $I_{A}$ be the incenter and the $A$-excenter of an acute-angled triangle $A B C$ with $A B(t-1) \\frac{\\sqrt{3}}{2}$, or $$ t<1+\\frac{4 \\sqrt{M}}{\\sqrt{3}}<4 \\sqrt{M} $$ as $M \\geqslant 1$. Combining the estimates (1), (2), and (3), we finally obtain $$ \\frac{1}{4 \\delta} \\leqslant t<4 \\sqrt{M}<4 \\sqrt{2 n \\delta}, \\quad \\text { or } \\quad 512 n \\delta^{3}>1 $$ which does not hold for the chosen value of $\\delta$. Comment 1. As the proposer mentions, the exponent $-1 \/ 3$ in the problem statement is optimal. In fact, for any $n \\geqslant 2$, there is a configuration $\\mathcal{S}$ of $n$ points in the plane such that any two points in $\\mathcal{S}$ are at least 1 apart, but every line $\\ell$ separating $\\mathcal{S}$ is at most $c^{\\prime} n^{-1 \/ 3} \\log n$ apart from some point in $\\mathcal{S}$; here $c^{\\prime}$ is some absolute constant. The original proposal suggested to prove the estimate of the form $\\mathrm{cn}^{-1 \/ 2}$. That version admits much easier solutions. E.g., setting $\\delta=\\frac{1}{16} n^{-1 \/ 2}$ and applying (1), we see that $\\mathcal{S}$ is contained in a disk $D$ of radius $\\frac{1}{8} n^{1 \/ 2}$. On the other hand, for each point $X$ of $\\mathcal{S}$, let $D_{X}$ be the disk of radius $\\frac{1}{2}$ centered at $X$; all these disks have disjoint interiors and lie within the disk concentric to $D$, of radius $\\frac{1}{16} n^{1 \/ 2}+\\frac{1}{2}<\\frac{1}{2} n^{1 \/ 2}$. Comparing the areas, we get $$ n \\cdot \\frac{\\pi}{4} \\leqslant \\pi\\left(\\frac{n^{1 \/ 2}}{16}+\\frac{1}{2}\\right)^{2}<\\frac{\\pi n}{4} $$ which is a contradiction. The Problem Selection Committee decided to choose a harder version for the Shortlist. Comment 2. In this comment, we discuss some versions of the solution above, which avoid concentrating on the diameter of $\\mathcal{S}$. We start with introducing some terminology suitable for those versions. Put $\\delta=c n^{-1 \/ 3}$ for a certain sufficiently small positive constant $c$. For the sake of contradiction, suppose that, for some set $\\mathcal{S}$ satisfying the conditions in the problem statement, there is no separating line which is at least $\\delta$ apart from each point of $\\mathcal{S}$. Let $C$ be the convex hull of $\\mathcal{S}$. A line is separating if and only if it meets $C$ (we assume that a line passing through a point of $\\mathcal{S}$ is always separating). Consider a strip between two parallel separating lines $a$ and $a^{\\prime}$ which are, say, $\\frac{1}{4}$ apart from each other. Define a slice determined by the strip as the intersection of $\\mathcal{S}$ with the strip. The length of the slice is the diameter of the projection of the slice to $a$. In this terminology, the arguments used in the proofs of (2) and (3) show that for any slice $\\mathcal{T}$ of length $L$, we have $$ \\frac{1}{8 \\delta} \\leqslant|\\mathcal{T}| \\leqslant 1+\\frac{4}{\\sqrt{15}} L $$ The key idea of the solution is to apply these estimates to a peel slice, where line $a$ does not cross the interior of $C$. In the above solution, this idea was applied to one carefully chosen peel slice. Here, we outline some different approach involving many of them. We always assume that $n$ is sufficiently large. Consider a peel slice determined by lines $a$ and $a^{\\prime}$, where $a$ contains no interior points of $C$. We orient $a$ so that $C$ lies to the left of $a$. Line $a$ is called a supporting line of the slice, and the obtained direction is the direction of the slice; notice that the direction determines uniquely the supporting line and hence the slice. Fix some direction $\\mathbf{v}_{0}$, and for each $\\alpha \\in[0,2 \\pi)$ denote by $\\mathcal{T}_{\\alpha}$ the peel slice whose direction is $\\mathbf{v}_{0}$ rotated by $\\alpha$ counterclockwise. When speaking about the slice, we always assume that the figure is rotated so that its direction is vertical from the bottom to the top; then the points in $\\mathcal{T}$ get a natural order from the bottom to the top. In particular, we may speak about the top half $\\mathrm{T}(\\mathcal{T})$ consisting of $\\lfloor|\\mathcal{T}| \/ 2\\rfloor$ topmost points in $\\mathcal{T}$, and similarly about its bottom half $\\mathrm{B}(\\mathcal{T})$. By (4), each half contains at least 10 points when $n$ is large. Claim. Consider two angles $\\alpha, \\beta \\in[0, \\pi \/ 2]$ with $\\beta-\\alpha \\geqslant 40 \\delta=: \\phi$. Then all common points of $\\mathcal{T}_{\\alpha}$ and $\\mathcal{T}_{\\beta}$ lie in $\\mathrm{T}\\left(\\mathcal{T}_{\\alpha}\\right) \\cap \\mathrm{B}\\left(\\mathcal{T}_{\\beta}\\right)$. ![](https:\/\/cdn.mathpix.com\/cropped\/2024_11_18_bcdd154f2030efbe2fa4g-71.jpg?height=738&width=1338&top_left_y=201&top_left_x=366) Proof. By symmetry, it suffices to show that all those points lie in $\\mathrm{T}\\left(\\mathcal{T}_{\\alpha}\\right)$. Let $a$ be the supporting line of $\\mathcal{T}_{\\alpha}$, and let $\\ell$ be a line perpendicular to the direction of $\\mathcal{T}_{\\beta}$. Let $P_{1}, \\ldots, P_{k}$ list all points in $\\mathcal{T}_{\\alpha}$, numbered from the bottom to the top; by (4), we have $k \\geqslant \\frac{1}{8} \\delta^{-1}$. Introduce the Cartesian coordinates so that the (oriented) line $a$ is the $y$-axis. Let $P_{i}$ be any point in $\\mathrm{B}\\left(\\mathcal{T}_{\\alpha}\\right)$. The difference of ordinates of $P_{k}$ and $P_{i}$ is at least $\\frac{\\sqrt{15}}{4}(k-i)>\\frac{1}{3} k$, while their abscissas differ by at most $\\frac{1}{4}$. This easily yields that the projections of those points to $\\ell$ are at least $$ \\frac{k}{3} \\sin \\phi-\\frac{1}{4} \\geqslant \\frac{1}{24 \\delta} \\cdot 20 \\delta-\\frac{1}{4}>\\frac{1}{4} $$ apart from each other, and $P_{k}$ is closer to the supporting line of $\\mathcal{T}_{\\beta}$ than $P_{i}$, so that $P_{i}$ does not belong to $\\mathcal{T}_{\\beta}$. Now, put $\\alpha_{i}=40 \\delta i$, for $i=0,1, \\ldots,\\left\\lfloor\\frac{1}{40} \\delta^{-1} \\cdot \\frac{\\pi}{2}\\right\\rfloor$, and consider the slices $\\mathcal{T}_{\\alpha_{i}}$. The Claim yields that each point in $\\mathcal{S}$ is contained in at most two such slices. Hence, the union $\\mathcal{U}$ of those slices contains at least $$ \\frac{1}{2} \\cdot \\frac{1}{8 \\delta} \\cdot \\frac{1}{40 \\delta} \\cdot \\frac{\\pi}{2}=\\frac{\\lambda}{\\delta^{2}} $$ points (for some constant $\\lambda$ ), and each point in $\\mathcal{U}$ is at most $\\frac{1}{4}$ apart from the boundary of $C$. It is not hard now to reach a contradiction with (1). E.g., for each point $X \\in \\mathcal{U}$, consider a closest point $f(X)$ on the boundary of $C$. Obviously, $f(X) f(Y) \\geqslant X Y-\\frac{1}{2} \\geqslant \\frac{1}{2} X Y$ for all $X, Y \\in \\mathcal{U}$. This yields that the perimeter of $C$ is at least $\\mu \\delta^{-2}$, for some constant $\\mu$, and hence the diameter of $\\mathcal{S}$ is of the same order. Alternatively, one may show that the projection of $\\mathcal{U}$ to the line at the angle of $\\pi \/ 4$ with $\\mathbf{v}_{0}$ has diameter at least $\\mu \\delta^{-2}$ for some constant $\\mu$.","tier":0} +{"problem_type":"Number Theory","problem_label":"N1","problem":"Given a positive integer $k$, show that there exists a prime $p$ such that one can choose distinct integers $a_{1}, a_{2}, \\ldots, a_{k+3} \\in\\{1,2, \\ldots, p-1\\}$ such that $p$ divides $a_{i} a_{i+1} a_{i+2} a_{i+3}-i$ for all $i=1,2, \\ldots, k$. (South Africa)","solution":"First we choose distinct positive rational numbers $r_{1}, \\ldots, r_{k+3}$ such that $$ r_{i} r_{i+1} r_{i+2} r_{i+3}=i \\quad \\text { for } 1 \\leqslant i \\leqslant k $$ Let $r_{1}=x, r_{2}=y, r_{3}=z$ be some distinct primes greater than $k$; the remaining terms satisfy $r_{4}=\\frac{1}{r_{1} r_{2} r_{3}}$ and $r_{i+4}=\\frac{i+1}{i} r_{i}$. It follows that if $r_{i}$ are represented as irreducible fractions, the numerators are divisible by $x$ for $i \\equiv 1(\\bmod 4)$, by $y$ for $i \\equiv 2(\\bmod 4)$, by $z$ for $i \\equiv 3(\\bmod 4)$ and by none for $i \\equiv 0(\\bmod 4)$. Notice that $r_{i}3$ dividing a number of the form $x^{2}-x+1$ with integer $x$ there are two unconnected islands in $p$-Landia. For brevity's sake, when a bridge connects the islands numbered $m$ and $n$, we shall speak simply that it connects $m$ and $n$. A bridge connects $m$ and $n$ if $n \\equiv m^{2}+1(\\bmod p)$ or $m \\equiv n^{2}+1(\\bmod p)$. If $m^{2}+1 \\equiv n$ $(\\bmod p)$, we draw an arrow starting at $m$ on the bridge connecting $m$ and $n$. Clearly only one arrow starts at $m$ if $m^{2}+1 \\not \\equiv m(\\bmod p)$, and no arrows otherwise. The total number of bridges does not exceed the total number of arrows. Suppose $x^{2}-x+1 \\equiv 0(\\bmod p)$. We may assume that $1 \\leqslant x \\leqslant p$; then there is no arrow starting at $x$. Since $(1-x)^{2}-(1-x)+1=x^{2}-x+1,(p+1-x)^{2}+1 \\equiv(p+1-x)(\\bmod p)$, and there is also no arrow starting at $p+1-x$. If $x=p+1-x$, that is, $x=\\frac{p+1}{2}$, then $4\\left(x^{2}-x+1\\right)=p^{2}+3$ and therefore $x^{2}-x+1$ is not divisible by $p$. Thus the islands $x$ and $p+1-x$ are different, and no arrows start at either of them. It follows that the total number of bridges in $p$-Landia does not exceed $p-2$. Let $1,2, \\ldots, p$ be the vertices of a graph $G_{p}$, where an edge connects $m$ and $n$ if and only if there is a bridge between $m$ and $n$. The number of vertices of $G_{p}$ is $p$ and the number of edges is less than $p-1$. This means that the graph is not connected, which means that there are two islands not connected by a chain of bridges. It remains to prove that there are infinitely many primes $p$ dividing $x^{2}-x+1$ for some integer $x$. Let $p_{1}, p_{2}, \\ldots, p_{k}$ be any finite set of such primes. The number $\\left(p_{1} p_{2} \\cdot \\ldots \\cdot p_{k}\\right)^{2}-p_{1} p_{2} \\cdot \\ldots \\cdot p_{k}+1$ is greater than 1 and not divisible by any $p_{i}$; therefore it has another prime divisor with the required property.","tier":0} +{"problem_type":"Number Theory","problem_label":"N2","problem":"For each prime $p$, there is a kingdom of $p$-Landia consisting of $p$ islands numbered $1,2, \\ldots, p$. Two distinct islands numbered $n$ and $m$ are connected by a bridge if and only if $p$ divides $\\left(n^{2}-m+1\\right)\\left(m^{2}-n+1\\right)$. The bridges may pass over each other, but cannot cross. Prove that for infinitely many $p$ there are two islands in $p$-Landia not connected by a chain of bridges. (Denmark)","solution":"One can show, by using only arithmetical methods, that for infinitely many $p$, the kingdom of $p$-Ladia contains two islands connected to no other island, except for each other. Let arrows between islands have the same meaning as in the previous solution. Suppose that positive $a3$. It follows that $a b \\equiv a(1-a) \\equiv 1$ $(\\bmod p)$. If an arrow goes from $t$ to $a$, then $t$ must satisfy the congruence $t^{2}+1 \\equiv a \\equiv a^{2}+1$ $(\\bmod p)$; the only such $t \\neq a$ is $p-a$. Similarly, the only arrow going to $b$ goes from $p-b$. If one of the numbers $p-a$ and $p-b$, say, $p-a$, is not at the end of any arrow, the pair $a, p-a$ is not connected with the rest of the islands. This is true if at least one of the congruences $x^{2}+1 \\equiv-a, x^{2}+1 \\equiv-b$ has no solutions, that is, either $-a-1$ or $-b-1$ is a quadratic non-residue modulo $p$. Note that $x^{2}-x+1 \\equiv x^{2}-(a+b) x+a b \\equiv(x-a)(x-b)(\\bmod p)$. Substituting $x=-1$ we get $(-1-a)(-1-b) \\equiv 3(\\bmod p)$. If 3 is a quadratic non-residue modulo $p$, so is one of the numbers $-1-a$ and $-1-b$. Thus it is enough to find infinitely many primes $p>3$ dividing $x^{2}-x+1$ for some integer $x$ and such that 3 is a quadratic non-residue modulo $p$. If $x^{2}-x+1 \\equiv 0(\\bmod p)$ then $(2 x-1)^{2} \\equiv-3(\\bmod p)$, that is, -3 is a quadratic residue modulo $p$, so 3 is a quadratic non-residue if and only if -1 is also a non-residue, in other words, $p \\equiv-1(\\bmod 4)$. Similarly to the first solution, let $p_{1}, \\ldots, p_{k}$ be primes congruent to -1 modulo 4 and dividing numbers of the form $x^{2}-x+1$. The number $\\left(2 p_{1} \\cdot \\ldots \\cdot p_{k}\\right)^{2}-2 p_{1} \\cdot \\ldots \\cdot p_{k}+1$ is not divisible by any $p_{i}$ and is congruent to -1 modulo 4 , therefore, it has some prime divisor $p \\equiv-1(\\bmod 4)$ which has the required properties.","tier":0} +{"problem_type":"Number Theory","problem_label":"N3","problem":"Let $n$ be an integer with $n \\geqslant 2$. Does there exist a sequence $\\left(a_{1}, \\ldots, a_{n}\\right)$ of positive integers with not all terms being equal such that the arithmetic mean of every two terms is equal to the geometric mean of some (one or more) terms in this sequence? (Estonia)","solution":"Suppose that $a_{1}, \\ldots, a_{n}$ satisfy the required properties. Let $d=\\operatorname{gcd}\\left(a_{1} \\ldots, a_{n}\\right)$. If $d>1$ then replace the numbers $a_{1}, \\ldots, a_{n}$ by $\\frac{a_{1}}{d}, \\ldots, \\frac{a_{n}}{d}$; all arithmetic and all geometric means will be divided by $d$, so we obtain another sequence satisfying the condition. Hence, without loss of generality, we can assume that $\\operatorname{gcd}\\left(a_{1} \\ldots, a_{n}\\right)=1$. We show two numbers, $a_{m}$ and $a_{k}$ such that their arithmetic mean, $\\frac{a_{m}+a_{k}}{2}$ is different from the geometric mean of any (nonempty) subsequence of $a_{1} \\ldots, a_{n}$. That proves that there cannot exist such a sequence. Choose the index $m \\in\\{1, \\ldots, n\\}$ such that $a_{m}=\\max \\left(a_{1}, \\ldots, a_{n}\\right)$. Note that $a_{m} \\geqslant 2$, because $a_{1}, \\ldots, a_{n}$ are not all equal. Let $p$ be a prime divisor of $a_{m}$. Let $k \\in\\{1, \\ldots, n\\}$ be an index such that $a_{k}=\\max \\left\\{a_{i}: p \\nmid a_{i}\\right\\}$. Due to $\\operatorname{gcd}\\left(a_{1} \\ldots, a_{n}\\right)=1$, not all $a_{i}$ are divisible by $p$, so such a $k$ exists. Note that $a_{m}>a_{k}$ because $a_{m} \\geqslant a_{k}, p \\mid a_{m}$ and $p \\nmid a_{k}$. Let $b=\\frac{a_{m}+a_{k}}{2}$; we will show that $b$ cannot be the geometric mean of any subsequence of $a_{1}, \\ldots, a_{n}$. Consider the geometric mean, $g=\\sqrt[t]{a_{i_{1}} \\cdot \\ldots \\cdot a_{i_{t}}}$ of an arbitrary subsequence of $a_{1}, \\ldots, a_{n}$. If none of $a_{i_{1}}, \\ldots, a_{i_{t}}$ is divisible by $p$, then they are not greater than $a_{k}$, so $$ g=\\sqrt[t]{a_{i_{1}} \\cdot \\ldots \\cdot a_{i_{t}}} \\leqslant a_{k}<\\frac{a_{m}+a_{k}}{2}=b $$ and therefore $g \\neq b$. Otherwise, if at least one of $a_{i_{1}}, \\ldots, a_{i_{t}}$ is divisible by $p$, then $2 g=2 \\sqrt{t} \\sqrt{a_{i_{1}} \\cdot \\ldots \\cdot a_{i_{t}}}$ is either not an integer or is divisible by $p$, while $2 b=a_{m}+a_{k}$ is an integer not divisible by $p$, so $g \\neq b$ again.","tier":0} +{"problem_type":"Number Theory","problem_label":"N3","problem":"Let $n$ be an integer with $n \\geqslant 2$. Does there exist a sequence $\\left(a_{1}, \\ldots, a_{n}\\right)$ of positive integers with not all terms being equal such that the arithmetic mean of every two terms is equal to the geometric mean of some (one or more) terms in this sequence? (Estonia)","solution":"Like in the previous solution, we assume that the numbers $a_{1}, \\ldots, a_{n}$ have no common divisor greater than 1 . The arithmetic mean of any two numbers in the sequence is half of an integer; on the other hand, it is a (some integer order) root of an integer. This means each pair's mean is an integer, so all terms in the sequence must be of the same parity; hence they all are odd. Let $d=\\min \\left\\{\\operatorname{gcd}\\left(a_{i}, a_{j}\\right): a_{i} \\neq a_{j}\\right\\}$. By reordering the sequence we can assume that $\\operatorname{gcd}\\left(a_{1}, a_{2}\\right)=d$, the sum $a_{1}+a_{2}$ is maximal among such pairs, and $a_{1}>a_{2}$. We will show that $\\frac{a_{1}+a_{2}}{2}$ cannot be the geometric mean of any subsequence of $a_{1} \\ldots, a_{n}$. Let $a_{1}=x d$ and $a_{2}=y d$ where $x, y$ are coprime, and suppose that there exist some $b_{1}, \\ldots, b_{t} \\in\\left\\{a_{1}, \\ldots, a_{n}\\right\\}$ whose geometric mean is $\\frac{a_{1}+a_{2}}{2}$. Let $d_{i}=\\operatorname{gcd}\\left(a_{1}, b_{i}\\right)$ for $i=1,2, \\ldots, t$ and let $D=d_{1} d_{2} \\cdot \\ldots \\cdot d_{t}$. Then $$ D=d_{1} d_{2} \\cdot \\ldots \\cdot d_{t} \\left\\lvert\\, b_{1} b_{2} \\cdot \\ldots \\cdot b_{t}=\\left(\\frac{a_{1}+a_{2}}{2}\\right)^{t}=\\left(\\frac{x+y}{2}\\right)^{t} d^{t}\\right. $$ We claim that $D \\mid d^{t}$. Consider an arbitrary prime divisor $p$ of $D$. Let $\\nu_{p}(x)$ denote the exponent of $p$ in the prime factorization of $x$. If $p \\left\\lvert\\, \\frac{x+y}{2}\\right.$, then $p \\nmid x, y$, so $p$ is coprime with $x$; hence, $\\nu_{p}\\left(d_{i}\\right) \\leqslant \\nu_{p}\\left(a_{1}\\right)=\\nu_{p}(x d)=\\nu_{p}(d)$ for every $1 \\leqslant i \\leqslant t$, therefore $\\nu_{p}(D)=\\sum_{i} \\nu_{p}\\left(d_{i}\\right) \\leqslant$ $t \\nu_{p}(d)=\\nu_{p}\\left(d^{t}\\right)$. Otherwise, if $p$ is coprime to $\\frac{x+y}{2}$, we have $\\nu_{p}(D) \\leqslant \\nu_{p}\\left(d^{t}\\right)$ trivially. The claim has been proved. Notice that $d_{i}=\\operatorname{gcd}\\left(b_{i}, a_{1}\\right) \\geqslant d$ for $1 \\leqslant i \\leqslant t$ : if $b_{i} \\neq a_{1}$ then this follows from the definition of $d$; otherwise we have $b_{i}=a_{1}$, so $d_{i}=a_{1} \\geqslant d$. Hence, $D=d_{1} \\cdot \\ldots \\cdot d_{t} \\geqslant d^{t}$, and the claim forces $d_{1}=\\ldots=d_{t}=d$. Finally, by $\\frac{a_{1}+a_{2}}{2}>a_{2}$ there must be some $b_{k}$ which is greater than $a_{2}$. From $a_{1}>a_{2} \\geqslant$ $d=\\operatorname{gcd}\\left(a_{1}, b_{k}\\right)$ it follows that $a_{1} \\neq b_{k}$. Now the have a pair $a_{1}, b_{k}$ such that $\\operatorname{gcd}\\left(a_{1}, b_{k}\\right)=d$ but $a_{1}+b_{k}>a_{1}+a_{2}$; that contradicts the choice of $a_{1}$ and $a_{2}$. Comment. The original problem proposal contained a second question asking if there exists a nonconstant sequence $\\left(a_{1}, \\ldots, a_{n}\\right)$ of positive integers such that the geometric mean of every two terms is equal the arithmetic mean of some terms. For $n \\geqslant 3$ such a sequence is $(4,1,1, \\ldots, 1)$. The case $n=2$ can be done by the trivial estimates $$ \\min \\left(a_{1}, a_{2}\\right)<\\sqrt{a_{1} a_{2}}<\\frac{a_{1}+a_{2}}{2}<\\max \\left(a_{1}, a_{2}\\right) $$ The Problem Selection Committee found this variant less interesting and suggests using only the first question.","tier":0} +{"problem_type":"Number Theory","problem_label":"N4","problem":"For any odd prime $p$ and any integer $n$, let $d_{p}(n) \\in\\{0,1, \\ldots, p-1\\}$ denote the remainder when $n$ is divided by $p$. We say that $\\left(a_{0}, a_{1}, a_{2}, \\ldots\\right)$ is a $p$-sequence, if $a_{0}$ is a positive integer coprime to $p$, and $a_{n+1}=a_{n}+d_{p}\\left(a_{n}\\right)$ for $n \\geqslant 0$. (a) Do there exist infinitely many primes $p$ for which there exist $p$-sequences $\\left(a_{0}, a_{1}, a_{2}, \\ldots\\right)$ and $\\left(b_{0}, b_{1}, b_{2}, \\ldots\\right)$ such that $a_{n}>b_{n}$ for infinitely many $n$, and $b_{n}>a_{n}$ for infinitely many $n$ ? (b) Do there exist infinitely many primes $p$ for which there exist $p$-sequences $\\left(a_{0}, a_{1}, a_{2}, \\ldots\\right)$ and $\\left(b_{0}, b_{1}, b_{2}, \\ldots\\right)$ such that $a_{0}b_{n}$ for all $n \\geqslant 1$ ? (United Kingdom)","solution":"Fix some odd prime $p$, and let $T$ be the smallest positive integer such that $p \\mid 2^{T}-1$; in other words, $T$ is the multiplicative order of 2 modulo $p$. Consider any $p$-sequence $\\left(x_{n}\\right)=\\left(x_{0}, x_{1}, x_{2}, \\ldots\\right)$. Obviously, $x_{n+1} \\equiv 2 x_{n}(\\bmod p)$ and therefore $x_{n} \\equiv 2^{n} x_{0}(\\bmod p)$. This yields $x_{n+T} \\equiv x_{n}(\\bmod p)$ and therefore $d\\left(x_{n+T}\\right)=d\\left(x_{n}\\right)$ for all $n \\geqslant 0$. It follows that the sum $d\\left(x_{n}\\right)+d\\left(x_{n+1}\\right)+\\ldots+d\\left(x_{n+T-1}\\right)$ does not depend on $n$ and is thus a function of $x_{0}$ (and $p$ ) only; we shall denote this sum by $S_{p}\\left(x_{0}\\right)$, and extend the function $S_{p}(\\cdot)$ to all (not necessarily positive) integers. Therefore, we have $x_{n+k T}=x_{n}+k S_{p}\\left(x_{0}\\right)$ for all positive integers $n$ and $k$. Clearly, $S_{p}\\left(x_{0}\\right)=S_{p}\\left(2^{t} x_{0}\\right)$ for every integer $t \\geqslant 0$. In both parts, we use the notation $$ S_{p}^{+}=S_{p}(1)=\\sum_{i=0}^{T-1} d_{p}\\left(2^{i}\\right) \\quad \\text { and } \\quad S_{p}^{-}=S_{p}(-1)=\\sum_{i=0}^{T-1} d_{p}\\left(p-2^{i}\\right) $$ (a) Let $q>3$ be a prime and $p$ a prime divisor of $2^{q}+1$ that is greater than 3 . We will show that $p$ is suitable for part (a). Notice that $9 \\nmid 2^{q}+1$, so that such a $p$ exists. Moreover, for any two odd primes $qb_{0}$ and $a_{1}=p+2b_{0}+k S_{p}^{+}=b_{k \\cdot 2 q} \\quad \\text { and } \\quad a_{k \\cdot 2 q+1}=a_{1}+k S_{p}^{+}S_{p}\\left(y_{0}\\right)$ but $x_{0}y_{n}$ for every $n \\geqslant q+q \\cdot \\max \\left\\{y_{r}-x_{r}: r=0,1, \\ldots, q-1\\right\\}$. Now, since $x_{0}S_{p}^{-}$, then we can put $x_{0}=1$ and $y_{0}=p-1$. Otherwise, if $S_{p}^{+}p$ must be divisible by $p$. Indeed, if $n=p k+r$ is a good number, $k>0,00$. Let $\\mathcal{B}$ be the set of big primes, and let $p_{1}p_{1} p_{2}$, and let $p_{1}^{\\alpha}$ be the largest power of $p_{1}$ smaller than $n$, so that $n \\leqslant p_{1}^{\\alpha+1}0$. If $f(k)=f(n-k)$ for all $k$, it implies that $\\binom{n-1}{k}$ is not divisible by $p$ for all $k=1,2, \\ldots, n-2$. It is well known that it implies $n=a \\cdot p^{s}, a0, f(q)>0$, there exist only finitely many $n$ which are equal both to $a \\cdot p^{s}, a0$ for at least two primes less than $n$. Let $p_{0}$ be the prime with maximal $g(f, p)$ among all primes $p1$, let $p_{1}, \\ldots, p_{k}$ be all primes between 1 and $N$ and $p_{k+1}, \\ldots, p_{k+s}$ be all primes between $N+1$ and $2 N$. Since for $j \\leqslant k+s$ all prime divisors of $p_{j}-1$ do not exceed $N$, we have $$ \\prod_{j=1}^{k+s}\\left(p_{j}-1\\right)=\\prod_{i=1}^{k} p_{i}^{c_{i}} $$ with some fixed exponents $c_{1}, \\ldots, c_{k}$. Choose a huge prime number $q$ and consider a number $$ n=\\left(p_{1} \\cdot \\ldots \\cdot p_{k}\\right)^{q-1} \\cdot\\left(p_{k+1} \\cdot \\ldots \\cdot p_{k+s}\\right) $$ Then $$ \\varphi(d(n))=\\varphi\\left(q^{k} \\cdot 2^{s}\\right)=q^{k-1}(q-1) 2^{s-1} $$ and $$ d(\\varphi(n))=d\\left(\\left(p_{1} \\cdot \\ldots \\cdot p_{k}\\right)^{q-2} \\prod_{i=1}^{k+s}\\left(p_{i}-1\\right)\\right)=d\\left(\\prod_{i=1}^{k} p_{i}^{q-2+c_{i}}\\right)=\\prod_{i=1}^{k}\\left(q-1+c_{i}\\right) $$ so $$ \\frac{\\varphi(d(n))}{d(\\varphi(n))}=\\frac{q^{k-1}(q-1) 2^{s-1}}{\\prod_{i=1}^{k}\\left(q-1+c_{i}\\right)}=2^{s-1} \\cdot \\frac{q-1}{q} \\cdot \\prod_{i=1}^{k} \\frac{q}{q-1+c_{i}} $$ which can be made arbitrarily close to $2^{s-1}$ by choosing $q$ large enough. It remains to show that $s$ can be arbitrarily large, i.e. that there can be arbitrarily many primes between $N$ and $2 N$. This follows, for instance, from the well-known fact that $\\sum \\frac{1}{p}=\\infty$, where the sum is taken over the set $\\mathbb{P}$ of prime numbers. Indeed, if, for some constant $\\stackrel{p}{C}$, there were always at most $C$ primes between $2^{\\ell}$ and $2^{\\ell+1}$, we would have $$ \\sum_{p \\in \\mathbb{P}} \\frac{1}{p}=\\sum_{\\ell=0}^{\\infty} \\sum_{\\substack{p \\in \\mathbb{P} \\\\ p \\in\\left[2^{\\ell}, 2^{\\ell+1}\\right)}} \\frac{1}{p} \\leqslant \\sum_{\\ell=0}^{\\infty} \\frac{C}{2^{\\ell}}<\\infty, $$ which is a contradiction. Comment 1. Here we sketch several alternative elementary self-contained ways to perform the last step of the solution above. In particular, they avoid using divergence of $\\sum \\frac{1}{p}$. Suppose that for some constant $C$ and for every $k=1,2, \\ldots$ there exist at most $C$ prime numbers between $2^{k}$ and $2^{k+1}$. Consider the prime factorization of the factorial $\\left(2^{n}\\right)!=\\prod p^{\\alpha_{p}}$. We have $\\alpha_{p}=\\left\\lfloor 2^{n} \/ p\\right\\rfloor+\\left\\lfloor 2^{n} \/ p^{2}\\right\\rfloor+\\ldots$. Thus, for $p \\in\\left[2^{k}, 2^{k+1}\\right)$, we get $\\alpha_{p} \\leqslant 2^{n} \/ 2^{k}+2^{n} \/ 2^{k+1}+\\ldots=2^{n-k+1}$, therefore $p^{\\alpha_{p}} \\leqslant 2^{(k+1) 2^{n-k+1}}$. Combining this with the bound $(2 m)!\\geqslant m(m+1) \\cdot \\ldots \\cdot(2 m-1) \\geqslant m^{m}$ for $m=2^{n-1}$ we get $$ 2^{(n-1) \\cdot 2^{n-1}} \\leqslant\\left(2^{n}\\right)!\\leqslant \\prod_{k=1}^{n-1} 2^{C(k+1) 2^{n-k+1}} $$ or $$ \\sum_{k=1}^{n-1} C(k+1) 2^{1-k} \\geqslant \\frac{n-1}{2} $$ that fails for large $n$ since $C(k+1) 2^{1-k}<1 \/ 3$ for all but finitely many $k$. In fact, a much stronger inequality can be obtained in an elementary way: Note that the formula for $\\nu_{p}(n!)$ implies that if $p^{\\alpha}$ is the largest power of $p$ dividing $\\binom{n}{n \/ 2}$, then $p^{\\alpha} \\leqslant n$. By looking at prime factorization of $\\binom{n}{n \/ 2}$ we instantaneously infer that $$ \\pi(n) \\geqslant \\log _{n}\\binom{n}{n \/ 2} \\geqslant \\frac{\\log \\left(2^{n} \/ n\\right)}{\\log n} \\geqslant \\frac{n}{2 \\log n} . $$ This, in particular, implies that for infinitely many $n$ there are at least $\\frac{n}{3 \\log n}$ primes between $n$ and $2 n$.","tier":0} +{"problem_type":"Number Theory","problem_label":"N6","problem":"For a positive integer $n$, let $d(n)$ be the number of positive divisors of $n$, and let $\\varphi(n)$ be the number of positive integers not exceeding $n$ which are coprime to $n$. Does there exist a constant $C$ such that $$ \\frac{\\varphi(d(n))}{d(\\varphi(n))} \\leqslant C $$ for all $n \\geqslant 1$ ? (Cyprus)","solution":"In this solution we will use the Prime Number Theorem which states that $$ \\pi(m)=\\frac{m}{\\log m} \\cdot(1+o(1)) $$ as $m$ tends to infinity. Here and below $\\pi(m)$ denotes the number of primes not exceeding $m$, and $\\log$ the natural logarithm. Let $m>5$ be a large positive integer and let $n:=p_{1} p_{2} \\cdot \\ldots \\cdot p_{\\pi(m)}$ be the product of all primes not exceeding $m$. Then $\\varphi(d(n))=\\varphi\\left(2^{\\pi(m)}\\right)=2^{\\pi(m)-1}$. Consider the number $$ \\varphi(n)=\\prod_{k=1}^{\\pi(m)}\\left(p_{k}-1\\right)=\\prod_{s=1}^{\\pi(m \/ 2)} q_{s}^{\\alpha_{s}} $$ where $q_{1}, \\ldots, q_{\\pi(m \/ 2)}$ are primes not exceeding $m \/ 2$. Note that every term $p_{k}-1$ contributes at most one prime $q_{s}>\\sqrt{m}$ into the product $\\prod_{s} q_{s}^{\\alpha_{s}}$, so we have $$ \\sum_{s: q_{s}>\\sqrt{m}} \\alpha_{s} \\leqslant \\pi(m) \\Longrightarrow \\sum_{s: q_{s}>\\sqrt{m}}\\left(1+\\alpha_{s}\\right) \\leqslant \\pi(m)+\\pi(m \/ 2) . $$ Hence, applying the AM-GM inequality and the inequality $(A \/ x)^{x} \\leqslant e^{A \/ e}$, we obtain $$ \\prod_{s: q_{s}>\\sqrt{m}}\\left(\\alpha_{s}+1\\right) \\leqslant\\left(\\frac{\\pi(m)+\\pi(m \/ 2)}{\\ell}\\right)^{\\ell} \\leqslant \\exp \\left(\\frac{\\pi(m)+\\pi(m \/ 2)}{e}\\right) $$ where $\\ell$ is the number of primes in the interval $(\\sqrt{m}, m]$. We then use a trivial bound $\\alpha_{i} \\leqslant \\log _{2}(\\varphi(n)) \\leqslant \\log _{2} n<\\log _{2}\\left(m^{m}\\right)a_{j+1}>\\ldots>$ $a_{n}$; and (ii) $f$-avoidance: If $a0$, we should have $$ \\frac{(1+t)^{2 N}+1}{2} \\leqslant\\left(1+t+a_{n} t^{2}\\right)^{N} . $$ Expanding the brackets we get $$ \\left(1+t+a_{n} t^{2}\\right)^{N}-\\frac{(1+t)^{2 N}+1}{2}=\\left(N a_{n}-\\frac{N^{2}}{2}\\right) t^{2}+c_{3} t^{3}+\\ldots+c_{2 N} t^{2 N} $$ with some coefficients $c_{3}, \\ldots, c_{2 N}$. Since $a_{n}0$, then the left hand sides of $\\mathcal{I}(N,-x)$ and $\\mathcal{I}(N, x)$ coincide, while the right hand side of $\\mathcal{I}(N,-x)$ is larger than that of $\\mathcal{I}(N,-x)$ (their difference equals $2(N-1) x \\geqslant 0)$. Therefore, $\\mathcal{I}(N,-x)$ follows from $\\mathcal{I}(N, x)$. So, hereafter we suppose that $x>0$. Divide $\\mathcal{I}(N, x)$ by $x$ and let $t=(x-1)^{2} \/ x=x-2+1 \/ x$; then $\\mathcal{I}(n, x)$ reads as $$ f_{N}:=\\frac{x^{N}+x^{-N}}{2} \\leqslant\\left(1+\\frac{N}{2} t\\right)^{N} $$ The key identity is the expansion of $f_{N}$ as a polynomial in $t$ :","tier":0} +{"problem_type":"Algebra","problem_label":"A1","problem":"Version 1. Let $n$ be a positive integer, and set $N=2^{n}$. Determine the smallest real number $a_{n}$ such that, for all real $x$, $$ \\sqrt[N]{\\frac{x^{2 N}+1}{2}} \\leqslant a_{n}(x-1)^{2}+x $$ Version 2. For every positive integer $N$, determine the smallest real number $b_{N}$ such that, for all real $x$, $$ \\sqrt[N]{\\frac{x^{2 N}+1}{2}} \\leqslant b_{N}(x-1)^{2}+x $$ (Ireland) Answer for both versions : $a_{n}=b_{N}=N \/ 2$. Solution 1 (for Version 1). First of all, assume that $a_{n}0$, we should have $$ \\frac{(1+t)^{2 N}+1}{2} \\leqslant\\left(1+t+a_{n} t^{2}\\right)^{N} . $$ Expanding the brackets we get $$ \\left(1+t+a_{n} t^{2}\\right)^{N}-\\frac{(1+t)^{2 N}+1}{2}=\\left(N a_{n}-\\frac{N^{2}}{2}\\right) t^{2}+c_{3} t^{3}+\\ldots+c_{2 N} t^{2 N} $$ with some coefficients $c_{3}, \\ldots, c_{2 N}$. Since $a_{n}0$, or, equivalently, for $t=(x-1)^{2} \/ x \\geqslant 0$. Instead of finding the coefficients of the polynomial $f_{N}=f_{N}(t)$ we may find its roots, which is in a sense more straightforward. Note that the recurrence (4) and the initial conditions $f_{0}=1, f_{1}=1+t \/ 2$ imply that $f_{N}$ is a polynomial in $t$ of degree $N$. It also follows by induction that $f_{N}(0)=1, f_{N}^{\\prime}(0)=N^{2} \/ 2$ : the recurrence relations read as $f_{N+1}(0)+f_{N-1}(0)=2 f_{N}(0)$ and $f_{N+1}^{\\prime}(0)+f_{N-1}^{\\prime}(0)=2 f_{N}^{\\prime}(0)+f_{N}(0)$, respectively. Next, if $x_{k}=\\exp \\left(\\frac{i \\pi(2 k-1)}{2 N}\\right)$ for $k \\in\\{1,2, \\ldots, N\\}$, then $$ -t_{k}:=2-x_{k}-\\frac{1}{x_{k}}=2-2 \\cos \\frac{\\pi(2 k-1)}{2 N}=4 \\sin ^{2} \\frac{\\pi(2 k-1)}{4 N}>0 $$ and $$ f_{N}\\left(t_{k}\\right)=\\frac{x_{k}^{N}+x_{k}^{-N}}{2}=\\frac{\\exp \\left(\\frac{i \\pi(2 k-1)}{2}\\right)+\\exp \\left(-\\frac{i \\pi(2 k-1)}{2}\\right)}{2}=0 . $$ So the roots of $f_{N}$ are $t_{1}, \\ldots, t_{N}$ and by the AM-GM inequality we have $$ \\begin{aligned} f_{N}(t)=\\left(1-\\frac{t}{t_{1}}\\right)\\left(1-\\frac{t}{t_{2}}\\right) \\ldots\\left(1-\\frac{t}{t_{N}}\\right) & \\leqslant\\left(1-\\frac{t}{N}\\left(\\frac{1}{t_{1}}+\\ldots+\\frac{1}{t_{n}}\\right)\\right)^{N}= \\\\ \\left(1+\\frac{t f_{N}^{\\prime}(0)}{N}\\right)^{N} & =\\left(1+\\frac{N}{2} t\\right)^{N} \\end{aligned} $$ Comment. The polynomial $f_{N}(t)$ equals to $\\frac{1}{2} T_{N}(t+2)$, where $T_{n}$ is the $n^{\\text {th }}$ Chebyshev polynomial of the first kind: $T_{n}(2 \\cos s)=2 \\cos n s, T_{n}(x+1 \/ x)=x^{n}+1 \/ x^{n}$.","tier":0} +{"problem_type":"Algebra","problem_label":"A1","problem":"Version 1. Let $n$ be a positive integer, and set $N=2^{n}$. Determine the smallest real number $a_{n}$ such that, for all real $x$, $$ \\sqrt[N]{\\frac{x^{2 N}+1}{2}} \\leqslant a_{n}(x-1)^{2}+x $$ Version 2. For every positive integer $N$, determine the smallest real number $b_{N}$ such that, for all real $x$, $$ \\sqrt[N]{\\frac{x^{2 N}+1}{2}} \\leqslant b_{N}(x-1)^{2}+x $$ (Ireland) Answer for both versions : $a_{n}=b_{N}=N \/ 2$. Solution 1 (for Version 1). First of all, assume that $a_{n}0$, we should have $$ \\frac{(1+t)^{2 N}+1}{2} \\leqslant\\left(1+t+a_{n} t^{2}\\right)^{N} . $$ Expanding the brackets we get $$ \\left(1+t+a_{n} t^{2}\\right)^{N}-\\frac{(1+t)^{2 N}+1}{2}=\\left(N a_{n}-\\frac{N^{2}}{2}\\right) t^{2}+c_{3} t^{3}+\\ldots+c_{2 N} t^{2 N} $$ with some coefficients $c_{3}, \\ldots, c_{2 N}$. Since $a_{n}0 & \\text { if } 01\\end{cases} $$ Hence, $f^{\\prime \\prime}(x)<0$ for $x \\neq 1 ; f^{\\prime}(x)>0$ for $x<1$ and $f^{\\prime}(x)<0$ for $x>1$, finally $f(x)<0$ for $x \\neq 1$. Comment. Version 2 is much more difficult, of rather A5 or A6 difficulty. The induction in Version 1 is rather straightforward, while all three above solutions of Version 2 require some creativity. This page is intentionally left blank","tier":0} +{"problem_type":"Lemma.","problem_label":"A2","problem":"Let $\\mathcal{A}$ denote the set of all polynomials in three variables $x, y, z$ with integer coefficients. Let $\\mathcal{B}$ denote the subset of $\\mathcal{A}$ formed by all polynomials which can be expressed as $$ (x+y+z) P(x, y, z)+(x y+y z+z x) Q(x, y, z)+x y z R(x, y, z) $$ with $P, Q, R \\in \\mathcal{A}$. Find the smallest non-negative integer $n$ such that $x^{i} y^{j} z^{k} \\in \\mathcal{B}$ for all nonnegative integers $i, j, k$ satisfying $i+j+k \\geqslant n$. (Venezuela) Answer: $n=4$.","solution":"We start by showing that $n \\leqslant 4$, i.e., any monomial $f=x^{i} y^{j} z^{k}$ with $i+j+k \\geqslant 4$ belongs to $\\mathcal{B}$. Assume that $i \\geqslant j \\geqslant k$, the other cases are analogous. Let $x+y+z=p, x y+y z+z x=q$ and $x y z=r$. Then $$ 0=(x-x)(x-y)(x-z)=x^{3}-p x^{2}+q x-r $$ therefore $x^{3} \\in \\mathcal{B}$. Next, $x^{2} y^{2}=x y q-(x+y) r \\in \\mathcal{B}$. If $k \\geqslant 1$, then $r$ divides $f$, thus $f \\in \\mathcal{B}$. If $k=0$ and $j \\geqslant 2$, then $x^{2} y^{2}$ divides $f$, thus we have $f \\in \\mathcal{B}$ again. Finally, if $k=0, j \\leqslant 1$, then $x^{3}$ divides $f$ and $f \\in \\mathcal{B}$ in this case also. In order to prove that $n \\geqslant 4$, we show that the monomial $x^{2} y$ does not belong to $\\mathcal{B}$. Assume the contrary: $$ x^{2} y=p P+q Q+r R $$ for some polynomials $P, Q, R$. If polynomial $P$ contains the monomial $x^{2}$ (with nonzero coefficient), then $p P+q Q+r R$ contains the monomial $x^{3}$ with the same nonzero coefficient. So $P$ does not contain $x^{2}, y^{2}, z^{2}$ and we may write $$ x^{2} y=(x+y+z)(a x y+b y z+c z x)+(x y+y z+z x)(d x+e y+f z)+g x y z $$ where $a, b, c ; d, e, f ; g$ are the coefficients of $x y, y z, z x ; x, y, z ; x y z$ in the polynomials $P$; $Q ; R$, respectively (the remaining coefficients do not affect the monomials of degree 3 in $p P+q Q+r R)$. By considering the coefficients of $x y^{2}$ we get $e=-a$, analogously $e=-b$, $f=-b, f=-c, d=-c$, thus $a=b=c$ and $f=e=d=-a$, but then the coefficient of $x^{2} y$ in the right hand side equals $a+d=0 \\neq 1$. Comment 1. The general question is the following. Call a polynomial $f\\left(x_{1}, \\ldots, x_{n}\\right)$ with integer coefficients nice, if $f(0,0, \\ldots, 0)=0$ and $f\\left(x_{\\pi_{1}}, \\ldots, x_{\\pi_{n}}\\right)=f\\left(x_{1}, \\ldots, x_{n}\\right)$ for any permutation $\\pi$ of $1, \\ldots, n$ (in other words, $f$ is symmetric and its constant term is zero.) Denote by $\\mathcal{I}$ the set of polynomials of the form $$ p_{1} q_{1}+p_{2} q_{2}+\\ldots+p_{m} q_{m} $$ where $m$ is an integer, $q_{1}, \\ldots, q_{m}$ are polynomials with integer coefficients, and $p_{1}, \\ldots, p_{m}$ are nice polynomials. Find the least $N$ for which any monomial of degree at least $N$ belongs to $\\mathcal{I}$. The answer is $n(n-1) \/ 2+1$. The lower bound follows from the following claim: the polynomial $$ F\\left(x_{1}, \\ldots, x_{n}\\right)=x_{2} x_{3}^{2} x_{4}^{3} \\cdot \\ldots \\cdot x_{n}^{n-1} $$ does not belong to $\\mathcal{I}$. Assume that $F=\\sum p_{i} q_{i}$, according to (2). By taking only the monomials of degree $n(n-1) \/ 2$, we can additionally assume that every $p_{i}$ and every $q_{i}$ is homogeneous, $\\operatorname{deg} p_{i}>0$, and $\\operatorname{deg} p_{i}+\\operatorname{deg} q_{i}=$ $\\operatorname{deg} F=n(n-1) \/ 2$ for all $i$. Consider the alternating sum $$ \\sum_{\\pi} \\operatorname{sign}(\\pi) F\\left(x_{\\pi_{1}}, \\ldots, x_{\\pi_{n}}\\right)=\\sum_{i=1}^{m} p_{i} \\sum_{\\pi} \\operatorname{sign}(\\pi) q_{i}\\left(x_{\\pi_{1}}, \\ldots, x_{\\pi_{n}}\\right):=S $$ where the summation is done over all permutations $\\pi$ of $1, \\ldots n$, and $\\operatorname{sign}(\\pi)$ denotes the sign of the permutation $\\pi$. Since $\\operatorname{deg} q_{i}=n(n-1) \/ 2-\\operatorname{deg} p_{i}1$ and that the proposition is proved for smaller values of $n$. We proceed by an internal induction on $S:=\\left|\\left\\{i: c_{i}=0\\right\\}\\right|$. In the base case $S=0$ the monomial $h$ is divisible by the nice polynomial $x_{1} \\cdot \\ldots x_{n}$, therefore $h \\in \\mathcal{I}$. Now assume that $S>0$ and that the claim holds for smaller values of $S$. Let $T=n-S$. We may assume that $c_{T+1}=\\ldots=c_{n}=0$ and $h=x_{1} \\cdot \\ldots \\cdot x_{T} g\\left(x_{1}, \\ldots, x_{n-1}\\right)$, where $\\operatorname{deg} g=n(n-1) \/ 2-T+1 \\geqslant(n-1)(n-2) \/ 2+1$. Using the outer induction hypothesis we represent $g$ as $p_{1} q_{1}+\\ldots+p_{m} q_{m}$, where $p_{i}\\left(x_{1}, \\ldots, x_{n-1}\\right)$ are nice polynomials in $n-1$ variables. There exist nice homogeneous polynomials $P_{i}\\left(x_{1}, \\ldots, x_{n}\\right)$ such that $P_{i}\\left(x_{1}, \\ldots, x_{n-1}, 0\\right)=p_{i}\\left(x_{1}, \\ldots, x_{n-1}\\right)$. In other words, $\\Delta_{i}:=p_{i}\\left(x_{1}, \\ldots, x_{n-1}\\right)-P_{i}\\left(x_{1}, \\ldots, x_{n-1}, x_{n}\\right)$ is divisible by $x_{n}$, let $\\Delta_{i}=x_{n} g_{i}$. We get $$ h=x_{1} \\cdot \\ldots \\cdot x_{T} \\sum p_{i} q_{i}=x_{1} \\cdot \\ldots \\cdot x_{T} \\sum\\left(P_{i}+x_{n} g_{i}\\right) q_{i}=\\left(x_{1} \\cdot \\ldots \\cdot x_{T} x_{n}\\right) \\sum g_{i} q_{i}+\\sum P_{i} q_{i} \\in \\mathcal{I} $$ The first term belongs to $\\mathcal{I}$ by the inner induction hypothesis. This completes both inductions. Comment 2. The solutions above work smoothly for the versions of the original problem and its extensions to the case of $n$ variables, where all polynomials are assumed to have real coefficients. In the version with integer coefficients, the argument showing that $x^{2} y \\notin \\mathcal{B}$ can be simplified: it is not hard to show that in every polynomial $f \\in \\mathcal{B}$, the sum of the coefficients of $x^{2} y, x^{2} z, y^{2} x, y^{2} z, z^{2} x$ and $z^{2} y$ is even. A similar fact holds for any number of variables and also implies that $N \\geqslant n(n-1) \/ 2+1$ in terms of the previous comment.","tier":0} +{"problem_type":"Lemma.","problem_label":"A3","problem":"Suppose that $a, b, c, d$ are positive real numbers satisfying $(a+c)(b+d)=a c+b d$. Find the smallest possible value of $$ S=\\frac{a}{b}+\\frac{b}{c}+\\frac{c}{d}+\\frac{d}{a} $$ (Israel) Answer: The smallest possible value is 8.","solution":"To show that $S \\geqslant 8$, apply the AM-GM inequality twice as follows: $$ \\left(\\frac{a}{b}+\\frac{c}{d}\\right)+\\left(\\frac{b}{c}+\\frac{d}{a}\\right) \\geqslant 2 \\sqrt{\\frac{a c}{b d}}+2 \\sqrt{\\frac{b d}{a c}}=\\frac{2(a c+b d)}{\\sqrt{a b c d}}=\\frac{2(a+c)(b+d)}{\\sqrt{a b c d}} \\geqslant 2 \\cdot \\frac{2 \\sqrt{a c} \\cdot 2 \\sqrt{b d}}{\\sqrt{a b c d}}=8 . $$ The above inequalities turn into equalities when $a=c$ and $b=d$. Then the condition $(a+c)(b+d)=a c+b d$ can be rewritten as $4 a b=a^{2}+b^{2}$. So it is satisfied when $a \/ b=2 \\pm \\sqrt{3}$. Hence, $S$ attains value 8 , e.g., when $a=c=1$ and $b=d=2+\\sqrt{3}$.","tier":0} +{"problem_type":"Lemma.","problem_label":"A3","problem":"Suppose that $a, b, c, d$ are positive real numbers satisfying $(a+c)(b+d)=a c+b d$. Find the smallest possible value of $$ S=\\frac{a}{b}+\\frac{b}{c}+\\frac{c}{d}+\\frac{d}{a} $$ (Israel) Answer: The smallest possible value is 8.","solution":"By homogeneity we may suppose that $a b c d=1$. Let $a b=C, b c=A$ and $c a=B$. Then $a, b, c$ can be reconstructed from $A, B$ and $C$ as $a=\\sqrt{B C \/ A}, b=\\sqrt{A C \/ B}$ and $c=\\sqrt{A B \/ C}$. Moreover, the condition $(a+c)(b+d)=a c+b d$ can be written in terms of $A, B, C$ as $$ A+\\frac{1}{A}+C+\\frac{1}{C}=b c+a d+a b+c d=(a+c)(b+d)=a c+b d=B+\\frac{1}{B} . $$ We then need to minimize the expression $$ \\begin{aligned} S & :=\\frac{a d+b c}{b d}+\\frac{a b+c d}{a c}=\\left(A+\\frac{1}{A}\\right) B+\\left(C+\\frac{1}{C}\\right) \\frac{1}{B} \\\\ & =\\left(A+\\frac{1}{A}\\right)\\left(B-\\frac{1}{B}\\right)+\\left(A+\\frac{1}{A}+C+\\frac{1}{C}\\right) \\frac{1}{B} \\\\ & =\\left(A+\\frac{1}{A}\\right)\\left(B-\\frac{1}{B}\\right)+\\left(B+\\frac{1}{B}\\right) \\frac{1}{B} . \\end{aligned} $$ Without loss of generality assume that $B \\geqslant 1$ (otherwise, we may replace $B$ by $1 \/ B$ and swap $A$ and $C$, this changes neither the relation nor the function to be maximized). Therefore, we can write $$ S \\geqslant 2\\left(B-\\frac{1}{B}\\right)+\\left(B+\\frac{1}{B}\\right) \\frac{1}{B}=2 B+\\left(1-\\frac{1}{B}\\right)^{2}=: f(B) $$ Clearly, $f$ increases on $[1, \\infty)$. Since $$ B+\\frac{1}{B}=A+\\frac{1}{A}+C+\\frac{1}{C} \\geqslant 4, $$ we have $B \\geqslant B^{\\prime}$, where $B^{\\prime}=2+\\sqrt{3}$ is the unique root greater than 1 of the equation $B^{\\prime}+1 \/ B^{\\prime}=4$. Hence, $$ S \\geqslant f(B) \\geqslant f\\left(B^{\\prime}\\right)=2\\left(B^{\\prime}-\\frac{1}{B^{\\prime}}\\right)+\\left(B^{\\prime}+\\frac{1}{B^{\\prime}}\\right) \\frac{1}{B^{\\prime}}=2 B^{\\prime}-\\frac{2}{B^{\\prime}}+\\frac{4}{B^{\\prime}}=8 $$ It remains to note that when $A=C=1$ and $B=B^{\\prime}$ we have the equality $S=8$.","tier":0} +{"problem_type":"Lemma.","problem_label":"A3","problem":"Suppose that $a, b, c, d$ are positive real numbers satisfying $(a+c)(b+d)=a c+b d$. Find the smallest possible value of $$ S=\\frac{a}{b}+\\frac{b}{c}+\\frac{c}{d}+\\frac{d}{a} $$ (Israel) Answer: The smallest possible value is 8.","solution":"We present another proof of the inequality $S \\geqslant 8$. We start with the estimate $$ \\left(\\frac{a}{b}+\\frac{c}{d}\\right)+\\left(\\frac{b}{c}+\\frac{d}{a}\\right) \\geqslant 2 \\sqrt{\\frac{a c}{b d}}+2 \\sqrt{\\frac{b d}{a c}} $$ Let $y=\\sqrt{a c}$ and $z=\\sqrt{b d}$, and assume, without loss of generality, that $a c \\geqslant b d$. By the AM-GM inequality, we have $$ y^{2}+z^{2}=a c+b d=(a+c)(b+d) \\geqslant 2 \\sqrt{a c} \\cdot 2 \\sqrt{b d}=4 y z . $$ Substituting $x=y \/ z$, we get $4 x \\leqslant x^{2}+1$. For $x \\geqslant 1$, this holds if and only if $x \\geqslant 2+\\sqrt{3}$. Now we have $$ 2 \\sqrt{\\frac{a c}{b d}}+2 \\sqrt{\\frac{b d}{a c}}=2\\left(x+\\frac{1}{x}\\right) . $$ Clearly, this is minimized by setting $x(\\geqslant 1)$ as close to 1 as possible, i.e., by taking $x=2+\\sqrt{3}$. Then $2(x+1 \/ x)=2((2+\\sqrt{3})+(2-\\sqrt{3}))=8$, as required.","tier":0} +{"problem_type":"Lemma.","problem_label":"A5","problem":"A magician intends to perform the following trick. She announces a positive integer $n$, along with $2 n$ real numbers $x_{1}<\\ldots\\max _{z \\in \\mathcal{O}(0)}|z|$, this yields $f(a)=f(-a)$ and $f^{2 a^{2}}(0)=0$. Therefore, the sequence $\\left(f^{k}(0): k=0,1, \\ldots\\right)$ is purely periodic with a minimal period $T$ which divides $2 a^{2}$. Analogously, $T$ divides $2(a+1)^{2}$, therefore, $T \\mid \\operatorname{gcd}\\left(2 a^{2}, 2(a+1)^{2}\\right)=2$, i.e., $f(f(0))=0$ and $a(f(a)-f(-a))=f^{2 a^{2}}(0)=0$ for all $a$. Thus, $$ \\begin{array}{ll} f(a)=f(-a) \\quad \\text { for all } a \\neq 0 \\\\ \\text { in particular, } & f(1)=f(-1)=0 \\end{array} $$ Next, for each $n \\in \\mathbb{Z}$, by $E(n, 1-n)$ we get $$ n f(n)+(1-n) f(1-n)=f^{n^{2}+(1-n)^{2}}(1)=f^{2 n^{2}-2 n}(0)=0 $$ Assume that there exists some $m \\neq 0$ such that $f(m) \\neq 0$. Choose such an $m$ for which $|m|$ is minimal possible. Then $|m|>1$ due to $(\\boldsymbol{\\phi}) ; f(|m|) \\neq 0$ due to ( $\\boldsymbol{\\phi})$; and $f(1-|m|) \\neq 0$ due to $(\\Omega)$ for $n=|m|$. This contradicts to the minimality assumption. So, $f(n)=0$ for $n \\neq 0$. Finally, $f(0)=f^{3}(0)=f^{4}(2)=2 f(2)=0$. Clearly, the function $f(x) \\equiv 0$ satisfies the problem condition, which provides the first of the two answers. Case 2: All orbits are infinite. Since the orbits $\\mathcal{O}(a)$ and $\\mathcal{O}(a-1)$ differ by finitely many terms for all $a \\in \\mathbb{Z}$, each two orbits $\\mathcal{O}(a)$ and $\\mathcal{O}(b)$ have infinitely many common terms for arbitrary $a, b \\in \\mathbb{Z}$. For a minute, fix any $a, b \\in \\mathbb{Z}$. We claim that all pairs $(n, m)$ of nonnegative integers such that $f^{n}(a)=f^{m}(b)$ have the same difference $n-m$. Arguing indirectly, we have $f^{n}(a)=f^{m}(b)$ and $f^{p}(a)=f^{q}(b)$ with, say, $n-m>p-q$, then $f^{p+m+k}(b)=f^{p+n+k}(a)=f^{q+n+k}(b)$, for all nonnegative integers $k$. This means that $f^{\\ell+(n-m)-(p-q)}(b)=f^{\\ell}(b)$ for all sufficiently large $\\ell$, i.e., that the sequence $\\left(f^{n}(b)\\right)$ is eventually periodic, so $\\mathcal{O}(b)$ is finite, which is impossible. Now, for every $a, b \\in \\mathbb{Z}$, denote the common difference $n-m$ defined above by $X(a, b)$. We have $X(a-1, a)=1$ by (1). Trivially, $X(a, b)+X(b, c)=X(a, c)$, as if $f^{n}(a)=f^{m}(b)$ and $f^{p}(b)=f^{q}(c)$, then $f^{p+n}(a)=f^{p+m}(b)=f^{q+m}(c)$. These two properties imply that $X(a, b)=b-a$ for all $a, b \\in \\mathbb{Z}$. But (1) yields $f^{a^{2}+1}(f(a-1))=f^{a^{2}}(f(a))$, so $$ 1=X(f(a-1), f(a))=f(a)-f(a-1) \\quad \\text { for all } a \\in \\mathbb{Z} $$ Recalling that $f(-1)=0$, we conclude by (two-sided) induction on $x$ that $f(x)=x+1$ for all $x \\in \\mathbb{Z}$. Finally, the obtained function also satisfies the assumption. Indeed, $f^{n}(x)=x+n$ for all $n \\geqslant 0$, so $$ f^{a^{2}+b^{2}}(a+b)=a+b+a^{2}+b^{2}=a f(a)+b f(b) $$ Comment. There are many possible variations of the solution above, but it seems that finiteness of orbits seems to be a crucial distinction in all solutions. However, the case distinction could be made in different ways; in particular, there exist some versions of Case 1 which work whenever there is at least one finite orbit. We believe that Case 2 is conceptually harder than Case 1.","tier":0} +{"problem_type":"Lemma.","problem_label":"A7","problem":"Let $n$ and $k$ be positive integers. Prove that for $a_{1}, \\ldots, a_{n} \\in\\left[1,2^{k}\\right]$ one has $$ \\sum_{i=1}^{n} \\frac{a_{i}}{\\sqrt{a_{1}^{2}+\\ldots+a_{i}^{2}}} \\leqslant 4 \\sqrt{k n} $$ (Iran)","solution":"Partition the set of indices $\\{1,2, \\ldots, n\\}$ into disjoint subsets $M_{1}, M_{2}, \\ldots, M_{k}$ so that $a_{\\ell} \\in\\left[2^{j-1}, 2^{j}\\right]$ for $\\ell \\in M_{j}$. Then, if $\\left|M_{j}\\right|=: p_{j}$, we have $$ \\sum_{\\ell \\in M_{j}} \\frac{a_{\\ell}}{\\sqrt{a_{1}^{2}+\\ldots+a_{\\ell}^{2}}} \\leqslant \\sum_{i=1}^{p_{j}} \\frac{2^{j}}{2^{j-1} \\sqrt{i}}=2 \\sum_{i=1}^{p_{j}} \\frac{1}{\\sqrt{i}} $$ where we used that $a_{\\ell} \\leqslant 2^{j}$ and in the denominator every index from $M_{j}$ contributes at least $\\left(2^{j-1}\\right)^{2}$. Now, using $\\sqrt{i}-\\sqrt{i-1}=\\frac{1}{\\sqrt{i}+\\sqrt{i-1}} \\geqslant \\frac{1}{2 \\sqrt{i}}$, we deduce that $$ \\sum_{\\ell \\in M_{j}} \\frac{a_{\\ell}}{\\sqrt{a_{1}^{2}+\\ldots+a_{\\ell}^{2}}} \\leqslant 2 \\sum_{i=1}^{p_{j}} \\frac{1}{\\sqrt{i}} \\leqslant 2 \\sum_{i=1}^{p_{j}} 2(\\sqrt{i}-\\sqrt{i-1})=4 \\sqrt{p_{j}} $$ Therefore, summing over $j=1, \\ldots, k$ and using the QM-AM inequality, we obtain $$ \\sum_{\\ell=1}^{n} \\frac{a_{\\ell}}{\\sqrt{a_{1}^{2}+\\ldots+a_{\\ell}^{2}}} \\leqslant 4 \\sum_{j=1}^{k} \\sqrt{\\left|M_{j}\\right|} \\leqslant 4 \\sqrt{k \\sum_{j=1}^{k}\\left|M_{j}\\right|}=4 \\sqrt{k n} $$ Comment. Consider the function $f\\left(a_{1}, \\ldots, a_{n}\\right)=\\sum_{i=1}^{n} \\frac{a_{i}}{\\sqrt{a_{1}^{2}+\\ldots+a_{i}^{2}}}$. One can see that rearranging the variables in increasing order can only increase the value of $f\\left(a_{1}, \\ldots, a_{n}\\right)$. Indeed, if $a_{j}>a_{j+1}$ for some index $j$ then we have $$ f\\left(a_{1}, \\ldots, a_{j-1}, a_{j+1}, a_{j}, a_{j+2}, \\ldots, a_{n}\\right)-f\\left(a_{1}, \\ldots, a_{n}\\right)=\\frac{a}{S}+\\frac{b}{\\sqrt{S^{2}-a^{2}}}-\\frac{b}{S}-\\frac{a}{\\sqrt{S^{2}-b^{2}}} $$ where $a=a_{j}, b=a_{j+1}$, and $S=\\sqrt{a_{1}^{2}+\\ldots+a_{j+1}^{2}}$. The positivity of the last expression above follows from $$ \\frac{b}{\\sqrt{S^{2}-a^{2}}}-\\frac{b}{S}=\\frac{a^{2} b}{S \\sqrt{S^{2}-a^{2}} \\cdot\\left(S+\\sqrt{S^{2}-a^{2}}\\right)}>\\frac{a b^{2}}{S \\sqrt{S^{2}-b^{2}} \\cdot\\left(S+\\sqrt{S^{2}-b^{2}}\\right)}=\\frac{a}{\\sqrt{S^{2}-b^{2}}}-\\frac{a}{S} . $$ Comment. If $ky-1$, hence $$ f(x)>\\frac{y-1}{f(y)} \\quad \\text { for all } x \\in \\mathbb{R}^{+} $$ If $y>1$, this provides a desired positive lower bound for $f(x)$. Now, let $M=\\inf _{x \\in \\mathbb{R}^{+}} f(x)>0$. Then, for all $y \\in \\mathbb{R}^{+}$, $$ M \\geqslant \\frac{y-1}{f(y)}, \\quad \\text { or } \\quad f(y) \\geqslant \\frac{y-1}{M} $$ Lemma 1. The function $f(x)$ (and hence $\\phi(x)$ ) is bounded on any segment $[p, q]$, where $0\\max \\left\\{C, \\frac{C}{y}\\right\\}$. Then all three numbers $x, x y$, and $x+f(x y)$ are greater than $C$, so (*) reads as $$ (x+x y+1)+1+y=(x+1) f(y)+1, \\text { hence } f(y)=y+1 $$ Comment 1. It may be useful to rewrite (*) in the form $$ \\phi(x+f(x y))+\\phi(x y)=\\phi(x) \\phi(y)+x \\phi(y)+y \\phi(x)+\\phi(x)+\\phi(y) $$ This general identity easily implies both (1) and (5). Comment 2. There are other ways to prove that $f(x) \\geqslant x+1$. Once one has proved this, they can use this stronger estimate instead of (3) in the proof of Lemma 1. Nevertheless, this does not make this proof simpler. So proving that $f(x) \\geqslant x+1$ does not seem to be a serious progress towards the solution of the problem. In what follows, we outline one possible proof of this inequality. First of all, we improve inequality (3) by noticing that, in fact, $f(x) f(y) \\geqslant y-1+M$, and hence $$ f(y) \\geqslant \\frac{y-1}{M}+1 $$ Now we divide the argument into two steps. Step 1: We show that $M \\leqslant 1$. Suppose that $M>1$; recall the notation $a=f(1)$. Substituting $y=1 \/ x$ in (*), we get $$ f(x+a)=f(x) f\\left(\\frac{1}{x}\\right)+1-\\frac{1}{x} \\geqslant M f(x), $$ provided that $x \\geqslant 1$. By a straightforward induction on $\\lceil(x-1) \/ a\\rceil$, this yields $$ f(x) \\geqslant M^{(x-1) \/ a} $$ Now choose an arbitrary $x_{0} \\in \\mathbb{R}^{+}$and define a sequence $x_{0}, x_{1}, \\ldots$ by $x_{n+1}=x_{n}+f\\left(x_{n}\\right) \\geqslant x_{n}+M$ for all $n \\geqslant 0$; notice that the sequence is unbounded. On the other hand, by (4) we get $$ a x_{n+1}>a f\\left(x_{n}\\right)=f\\left(x_{n+1}\\right) \\geqslant M^{\\left(x_{n+1}-1\\right) \/ a}, $$ which cannot hold when $x_{n+1}$ is large enough. Step 2: We prove that $f(y) \\geqslant y+1$ for all $y \\in \\mathbb{R}^{+}$. Arguing indirectly, choose $y \\in \\mathbb{R}^{+}$such that $f(y)n k$, so $a_{n} \\geqslant k+1$. Now the $n-k+1$ numbers $a_{k}, a_{k+1}, \\ldots, a_{n}$ are all greater than $k$; but there are only $n-k$ such values; this is not possible. If $a_{n}=n$ then $a_{1}, a_{2}, \\ldots, a_{n-1}$ must be a permutation of the numbers $1, \\ldots, n-1$ satisfying $a_{1} \\leqslant 2 a_{2} \\leqslant \\ldots \\leqslant(n-1) a_{n-1}$; there are $P_{n-1}$ such permutations. The last inequality in (*), $(n-1) a_{n-1} \\leqslant n a_{n}=n^{2}$, holds true automatically. If $\\left(a_{n-1}, a_{n}\\right)=(n, n-1)$, then $a_{1}, \\ldots, a_{n-2}$ must be a permutation of $1, \\ldots, n-2$ satisfying $a_{1} \\leqslant \\ldots \\leqslant(n-2) a_{n-2}$; there are $P_{n-2}$ such permutations. The last two inequalities in (*) hold true automatically by $(n-2) a_{n-2} \\leqslant(n-2)^{2}k$. If $t=k$ then we are done, so assume $t>k$. Notice that one of the numbers among the $t-k$ numbers $a_{k}, a_{k+1}, \\ldots, a_{t-1}$ is at least $t$, because there are only $t-k-1$ values between $k$ and $t$. Let $i$ be an index with $k \\leqslant ik+1$. Then the chain of inequalities $k t=k a_{k} \\leqslant \\ldots \\leqslant t a_{t}=k t$ should also turn into a chain of equalities. From this point we can find contradictions in several ways; for example by pointing to $a_{t-1}=\\frac{k t}{t-1}=k+\\frac{k}{t-1}$ which cannot be an integer, or considering the product of the numbers $(k+1) a_{k+1}, \\ldots,(t-1) a_{t-1}$; the numbers $a_{k+1}, \\ldots, a_{t-1}$ are distinct and greater than $k$, so $$ (k t)^{t-k-1}=(k+1) a_{k+1} \\cdot(k+2) a_{k+2} \\cdot \\ldots \\cdot(t-1) a_{t-1} \\geqslant((k+1)(k+2) \\cdot \\ldots \\cdot(t-1))^{2} $$ Notice that $(k+i)(t-i)=k t+i(t-k-i)>k t$ for $1 \\leqslant i(k t)^{t-k-1} $$ Therefore, the case $t>k+1$ is not possible.","tier":0} +{"problem_type":"Combinatorics","problem_label":"C3","problem":"Let $n$ be an integer with $n \\geqslant 2$. On a slope of a mountain, $n^{2}$ checkpoints are marked, numbered from 1 to $n^{2}$ from the bottom to the top. Each of two cable car companies, $A$ and $B$, operates $k$ cable cars numbered from 1 to $k$; each cable car provides a transfer from some checkpoint to a higher one. For each company, and for any $i$ and $j$ with $1 \\leqslant i1$ of ones, then A can guarantee to obtain $S_{2}(n)$, but not more, cookies. The binary representation of 2020 is $2020=\\overline{11111100100}_{2}$, so $S_{2}(2020)=7$, and the answer follows. A strategy for $A$. At any round, while possible, A chooses two equal nonzero numbers on the board. Clearly, while $A$ can make such choice, the game does not terminate. On the other hand, $A$ can follow this strategy unless the game has already terminated. Indeed, if $A$ always chooses two equal numbers, then each number appearing on the board is either 0 or a power of 2 with non-negative integer exponent, this can be easily proved using induction on the number of rounds. At the moment when $A$ is unable to follow the strategy all nonzero numbers on the board are distinct powers of 2 . If the board contains at least one such power, then the largest of those powers is greater than the sum of the others. Otherwise there are only zeros on the blackboard, in both cases the game terminates. For every number on the board, define its range to be the number of ones it is obtained from. We can prove by induction on the number of rounds that for any nonzero number $k$ written by $B$ its range is $k$, and for any zero written by $B$ its range is a power of 2 . Thus at the end of each round all the ranges are powers of two, and their sum is $n$. Since $S_{2}(a+b) \\leqslant S_{2}(a)+S_{2}(b)$ for any positive integers $a$ and $b$, the number $n$ cannot be represented as a sum of less than $S_{2}(n)$ powers of 2 . Thus at the end of each round the board contains at least $S_{2}(n)$ numbers, while $A$ follows the above strategy. So $A$ can guarantee at least $S_{2}(n)$ cookies for himself. Comment. There are different proofs of the fact that the presented strategy guarantees at least $S_{2}(n)$ cookies for $A$. For instance, one may denote by $\\Sigma$ the sum of numbers on the board, and by $z$ the number of zeros. Then the board contains at least $S_{2}(\\Sigma)+z$ numbers; on the other hand, during the game, the number $S_{2}(\\Sigma)+z$ does not decrease, and its initial value is $S_{2}(n)$. The claim follows. A strategy for $B$. Denote $s=S_{2}(n)$. Let $x_{1}, \\ldots, x_{k}$ be the numbers on the board at some moment of the game after $B$ 's turn or at the beginning of the game. Say that a collection of $k$ signs $\\varepsilon_{1}, \\ldots, \\varepsilon_{k} \\in\\{+1,-1\\}$ is balanced if $$ \\sum_{i=1}^{k} \\varepsilon_{i} x_{i}=0 $$ We say that a situation on the board is good if $2^{s+1}$ does not divide the number of balanced collections. An appropriate strategy for $B$ can be explained as follows: Perform a move so that the situation remains good, while it is possible. We intend to show that in this case $B$ will not lose more than $S_{2}(n)$ cookies. For this purpose, we prove several lemmas. For a positive integer $k$, denote by $\\nu_{2}(k)$ the exponent of the largest power of 2 that divides $k$. Recall that, by Legendre's formula, $\\nu_{2}(n!)=n-S_{2}(n)$ for every positive integer $n$. Lemma 1. The initial situation is good. Proof. In the initial configuration, the number of balanced collections is equal to $\\binom{n}{n \/ 2}$. We have $$ \\nu_{2}\\left(\\binom{n}{n \/ 2}\\right)=\\nu_{2}(n!)-2 \\nu_{2}((n \/ 2)!)=\\left(n-S_{2}(n)\\right)-2\\left(\\frac{n}{2}-S_{2}(n \/ 2)\\right)=S_{2}(n)=s $$ Hence $2^{\\text {s+1 }}$ does not divide the number of balanced collections, as desired. Lemma 2. B may play so that after each round the situation remains good. Proof. Assume that the situation $\\left(x_{1}, \\ldots, x_{k}\\right)$ before a round is good, and that $A$ erases two numbers, $x_{p}$ and $x_{q}$. Let $N$ be the number of all balanced collections, $N_{+}$be the number of those having $\\varepsilon_{p}=\\varepsilon_{q}$, and $N_{-}$be the number of other balanced collections. Then $N=N_{+}+N_{-}$. Now, if $B$ replaces $x_{p}$ and $x_{q}$ by $x_{p}+x_{q}$, then the number of balanced collections will become $N_{+}$. If $B$ replaces $x_{p}$ and $x_{q}$ by $\\left|x_{p}-x_{q}\\right|$, then this number will become $N_{-}$. Since $2^{s+1}$ does not divide $N$, it does not divide one of the summands $N_{+}$and $N_{-}$, hence $B$ can reach a good situation after the round. Lemma 3. Assume that the game terminates at a good situation. Then the board contains at most $s$ numbers. Proof. Suppose, one of the numbers is greater than the sum of the other numbers. Then the number of balanced collections is 0 and hence divisible by $2^{s+1}$. Therefore, the situation is not good. Then we have only zeros on the blackboard at the moment when the game terminates. If there are $k$ of them, then the number of balanced collections is $2^{k}$. Since the situation is good, we have $k \\leqslant s$. By Lemmas 1 and 2, $B$ may act in such way that they keep the situation good. By Lemma 3, when the game terminates, the board contains at most $s$ numbers. This is what we aimed to prove. Comment 1. If the initial situation had some odd number $n>1$ of ones on the blackboard, player $A$ would still get $S_{2}(n)$ cookies, provided that both players act optimally. The proof of this fact is similar to the solution above, after one makes some changes in the definitions. Such changes are listed below. Say that a collection of $k$ signs $\\varepsilon_{1}, \\ldots, \\varepsilon_{k} \\in\\{+1,-1\\}$ is positive if $$ \\sum_{i=1}^{k} \\varepsilon_{i} x_{i}>0 $$ For every index $i=1,2, \\ldots, k$, we denote by $N_{i}$ the number of positive collections such that $\\varepsilon_{i}=1$. Finally, say that a situation on the board is good if $2^{s-1}$ does not divide at least one of the numbers $N_{i}$. Now, a strategy for $B$ again consists in preserving the situation good, after each round. Comment 2. There is an easier strategy for $B$, allowing, in the game starting with an even number $n$ of ones, to lose no more than $d=\\left\\lfloor\\log _{2}(n+2)\\right\\rfloor-1$ cookies. If the binary representation of $n$ contains at most two zeros, then $d=S_{2}(n)$, and hence the strategy is optimal in that case. We outline this approach below. First of all, we can assume that $A$ never erases zeros from the blackboard. Indeed, $A$ may skip such moves harmlessly, ignoring the zeros in the further process; this way, $A$ 's win will just increase. We say that a situation on the blackboard is pretty if the numbers on the board can be partitioned into two groups with equal sums. Clearly, if the situation before some round is pretty, then $B$ may play so as to preserve this property after the round. The strategy for $B$ is as follows: - $B$ always chooses a move that leads to a pretty situation. - If both possible moves of $B$ lead to pretty situations, then $B$ writes the sum of the two numbers erased by $A$. Since the situation always remains pretty, the game terminates when all numbers on the board are zeros. Suppose that, at the end of the game, there are $m \\geqslant d+1=\\left\\lfloor\\log _{2}(n+2)\\right\\rfloor$ zeros on the board; then $2^{m}-1>n \/ 2$. Now we analyze the whole process of the play. Let us number the zeros on the board in order of appearance. During the play, each zero had appeared after subtracting two equal numbers. Let $n_{1}, \\ldots, n_{m}$ be positive integers such that the first zero appeared after subtracting $n_{1}$ from $n_{1}$, the second zero appeared after subtracting $n_{2}$ from $n_{2}$, and so on. Since the sum of the numbers on the blackboard never increases, we have $2 n_{1}+\\ldots+2 n_{m} \\leqslant n$, hence $$ n_{1}+\\ldots+n_{m} \\leqslant n \/ 2<2^{m}-1 $$ There are $2^{m}$ subsets of the set $\\{1,2, \\ldots, m\\}$. For $I \\subseteq\\{1,2, \\ldots, m\\}$, denote by $f(I)$ the sum $\\sum_{i \\in I} n_{i}$. There are less than $2^{m}$ possible values for $f(I)$, so there are two distinct subsets $I$ and $J$ with $f(I)=f(J)$. Replacing $I$ and $J$ with $I \\backslash J$ and $J \\backslash I$, we assume that $I$ and $J$ are disjoint. Let $i_{0}$ be the smallest number in $I \\cup J$; without loss of generality, $i_{0} \\in I$. Consider the round when $A$ had erased two numbers equal to $n_{i_{0}}$, and $B$ had put the $i_{0}{ }^{\\text {th }}$ zero instead, and the situation before that round. For each nonzero number $z$ which is on the blackboard at this moment, we can keep track of it during the further play until it enters one of the numbers $n_{i}, i \\geqslant i_{0}$, which then turn into zeros. For every $i=i_{0}, i_{0}+1, \\ldots, m$, we denote by $X_{i}$ the collection of all numbers on the blackboard that finally enter the first copy of $n_{i}$, and by $Y_{i}$ the collection of those finally entering the second copy of $n_{i}$. Thus, each of $X_{i_{0}}$ and $Y_{i_{0}}$ consists of a single number. Since $A$ never erases zeros, the 2(m-iol $i_{0}$ ) defined sets are pairwise disjoint. Clearly, for either of the collections $X_{i}$ and $Y_{i}$, a signed sum of its elements equals $n_{i}$, for a proper choice of the signs. Therefore, for every $i=i_{0}, i_{0}+1, \\ldots, m$ one can endow numbers in $X_{i} \\cup Y_{i}$ with signs so that their sum becomes any of the numbers $-2 n_{i}, 0$, or $2 n_{i}$. Perform such endowment so as to get $2 n_{i}$ from each collection $X_{i} \\cup Y_{i}$ with $i \\in I,-2 n_{j}$ from each collection $X_{j} \\cup Y_{j}$ with $j \\in J$, and 0 from each remaining collection. The obtained signed sum of all numbers on the blackboard equals $$ \\sum_{i \\in I} 2 n_{i}-\\sum_{i \\in J} 2 n_{i}=0 $$ and the numbers in $X_{i_{0}}$ and $Y_{i_{0}}$ have the same (positive) sign. This means that, at this round, $B$ could add up the two numbers $n_{i_{0}}$ to get a pretty situation. According to the strategy, $B$ should have performed that, instead of subtracting the numbers. This contradiction shows that $m \\leqslant d$, as desired. This page is intentionally left blank","tier":0} +{"problem_type":"Geometry","problem_label":"G3","problem":"Let $A B C D$ be a convex quadrilateral with $\\angle A B C>90^{\\circ}, \\angle C D A>90^{\\circ}$, and $\\angle D A B=\\angle B C D$. Denote by $E$ and $F$ the reflections of $A$ in lines $B C$ and $C D$, respectively. Suppose that the segments $A E$ and $A F$ meet the line $B D$ at $K$ and $L$, respectively. Prove that the circumcircles of triangles $B E K$ and $D F L$ are tangent to each other. (Slovakia)","solution":"Denote by $A^{\\prime}$ the reflection of $A$ in $B D$. We will show that that the quadrilaterals $A^{\\prime} B K E$ and $A^{\\prime} D L F$ are cyclic, and their circumcircles are tangent to each other at point $A^{\\prime}$. From the symmetry about line $B C$ we have $\\angle B E K=\\angle B A K$, while from the symmetry in $B D$ we have $\\angle B A K=\\angle B A^{\\prime} K$. Hence $\\angle B E K=\\angle B A^{\\prime} K$, which implies that the quadrilateral $A^{\\prime} B K E$ is cyclic. Similarly, the quadrilateral $A^{\\prime} D L F$ is also cyclic. ![](https:\/\/cdn.mathpix.com\/cropped\/2024_11_18_bcdd154f2030efbe2fa4g-54.jpg?height=632&width=1229&top_left_y=818&top_left_x=419) For showing that circles $A^{\\prime} B K E$ and $A^{\\prime} D L F$ are tangent it suffices to prove that $$ \\angle A^{\\prime} K B+\\angle A^{\\prime} L D=\\angle B A^{\\prime} D . $$ Indeed, by $A K \\perp B C$, $A L \\perp C D$, and again the symmetry in $B D$ we have $$ \\angle A^{\\prime} K B+\\angle A^{\\prime} L D=180^{\\circ}-\\angle K A^{\\prime} L=180^{\\circ}-\\angle K A L=\\angle B C D=\\angle B A D=\\angle B A^{\\prime} D, $$ as required. Comment 1. The key to the solution above is introducing the point $A^{\\prime}$; then the angle calculations can be done in many different ways.","tier":0} +{"problem_type":"Geometry","problem_label":"G3","problem":"Let $A B C D$ be a convex quadrilateral with $\\angle A B C>90^{\\circ}, \\angle C D A>90^{\\circ}$, and $\\angle D A B=\\angle B C D$. Denote by $E$ and $F$ the reflections of $A$ in lines $B C$ and $C D$, respectively. Suppose that the segments $A E$ and $A F$ meet the line $B D$ at $K$ and $L$, respectively. Prove that the circumcircles of triangles $B E K$ and $D F L$ are tangent to each other. (Slovakia)","solution":"Note that $\\angle K A L=180^{\\circ}-\\angle B C D$, since $A K$ and $A L$ are perpendicular to $B C$ and $C D$, respectively. Reflect both circles $(B E K)$ and $(D F L)$ in $B D$. Since $\\angle K E B=\\angle K A B$ and $\\angle D F L=\\angle D A L$, the images are the circles $(K A B)$ and $(L A D)$, respectively; so they meet at $A$. We need to prove that those two reflections are tangent at $A$. For this purpose, we observe that $$ \\angle A K B+\\angle A L D=180^{\\circ}-\\angle K A L=\\angle B C D=\\angle B A D . $$ Thus, there exists a ray $A P$ inside angle $\\angle B A D$ such that $\\angle B A P=\\angle A K B$ and $\\angle D A P=$ $\\angle D L A$. Hence the line $A P$ is a common tangent to the circles $(K A B)$ and $(L A D)$, as desired. Comment 2. The statement of the problem remains true for a more general configuration, e.g., if line $B D$ intersect the extension of segment $A E$ instead of the segment itself, etc. The corresponding restrictions in the statement are given to reduce case sensitivity.","tier":0} +{"problem_type":"Geometry","problem_label":"G5","problem":"Let $A B C D$ be a cyclic quadrilateral with no two sides parallel. Let $K, L, M$, and $N$ be points lying on sides $A B, B C, C D$, and $D A$, respectively, such that $K L M N$ is a rhombus with $K L \\| A C$ and $L M \\| B D$. Let $\\omega_{1}, \\omega_{2}, \\omega_{3}$, and $\\omega_{4}$ be the incircles of triangles $A N K$, $B K L, C L M$, and $D M N$, respectively. Prove that the internal common tangents to $\\omega_{1}$ and $\\omega_{3}$ and the internal common tangents to $\\omega_{2}$ and $\\omega_{4}$ are concurrent. (Poland)","solution":"Let $I_{i}$ be the center of $\\omega_{i}$, and let $r_{i}$ be its radius for $i=1,2,3,4$. Denote by $T_{1}$ and $T_{3}$ the points of tangency of $\\omega_{1}$ and $\\omega_{3}$ with $N K$ and $L M$, respectively. Suppose that the internal common tangents to $\\omega_{1}$ and $\\omega_{3}$ meet at point $S$, which is the center of homothety $h$ with negative ratio (namely, with ratio $-\\frac{r_{3}}{r_{1}}$ ) mapping $\\omega_{1}$ to $\\omega_{3}$. This homothety takes $T_{1}$ to $T_{3}$ (since the tangents to $\\omega_{1}$ and $\\omega_{3}$ at $T_{1}$ to $T_{3}$ are parallel), hence $S$ is a point on the segment $T_{1} T_{3}$ with $T_{1} S: S T_{3}=r_{1}: r_{3}$. Construct segments $S_{1} S_{3} \\| K L$ and $S_{2} S_{4} \\| L M$ through $S$ with $S_{1} \\in N K, S_{2} \\in K L$, $S_{3} \\in L M$, and $S_{4} \\in M N$. Note that $h$ takes $S_{1}$ to $S_{3}$, hence $I_{1} S_{1} \\| I_{3} S_{3}$, and $S_{1} S: S S_{3}=r_{1}: r_{3}$. We will prove that $S_{2} S: S S_{4}=r_{2}: r_{4}$ or, equivalently, $K S_{1}: S_{1} N=r_{2}: r_{4}$. This will yield the problem statement; indeed, applying similar arguments to the intersection point $S^{\\prime}$ of the internal common tangents to $\\omega_{2}$ and $\\omega_{4}$, we see that $S^{\\prime}$ satisfies similar relations, and there is a unique point inside $K L M N$ satisfying them. Therefore, $S^{\\prime}=S$. ![](https:\/\/cdn.mathpix.com\/cropped\/2024_11_18_bcdd154f2030efbe2fa4g-56.jpg?height=670&width=1409&top_left_y=1207&top_left_x=326) Further, denote by $I_{A}, I_{B}, I_{C}, I_{D}$ and $r_{A}, r_{B}, r_{C}, r_{D}$ the incenters and inradii of triangles $D A B, A B C, B C D$, and $C D A$, respectively. One can shift triangle $C L M$ by $\\overrightarrow{L K}$ to glue it with triangle $A K N$ into a quadrilateral $A K C^{\\prime} N$ similar to $A B C D$. In particular, this shows that $r_{1}: r_{3}=r_{A}: r_{C}$; similarly, $r_{2}: r_{4}=r_{B}: r_{D}$. Moreover, the same shift takes $S_{3}$ to $S_{1}$, and it also takes $I_{3}$ to the incenter $I_{3}^{\\prime}$ of triangle $K C^{\\prime} N$. Since $I_{1} S_{1} \\| I_{3} S_{3}$, the points $I_{1}, S_{1}, I_{3}^{\\prime}$ are collinear. Thus, in order to complete the solution, it suffices to apply the following Lemma to quadrilateral $A K C^{\\prime} N$. Lemma 1. Let $A B C D$ be a cyclic quadrilateral, and define $I_{A}, I_{C}, r_{B}$, and $r_{D}$ as above. Let $I_{A} I_{C}$ meet $B D$ at $X$; then $B X: X D=r_{B}: r_{D}$. Proof. Consider an inversion centered at $X$; the images under that inversion will be denoted by primes, e.g., $A^{\\prime}$ is the image of $A$. By properties of inversion, we have $$ \\angle I_{C}^{\\prime} I_{A}^{\\prime} D^{\\prime}=\\angle X I_{A}^{\\prime} D^{\\prime}=\\angle X D I_{A}=\\angle B D A \/ 2=\\angle B C A \/ 2=\\angle A C I_{B} $$ We obtain $\\angle I_{A}^{\\prime} I_{C}^{\\prime} D^{\\prime}=\\angle C A I_{B}$ likewise; therefore, $\\triangle I_{C}^{\\prime} I_{A}^{\\prime} D^{\\prime} \\sim \\triangle A C I_{B}$. In the same manner, we get $\\triangle I_{C}^{\\prime} I_{A}^{\\prime} B^{\\prime} \\sim \\triangle A C I_{D}$, hence the quadrilaterals $I_{C}^{\\prime} B^{\\prime} I_{A}^{\\prime} D^{\\prime}$ and $A I_{D} C I_{B}$ are also similar. But the diagonals $A C$ and $I_{B} I_{D}$ of quadrilateral $A I_{D} C I_{B}$ meet at a point $Y$ such that $I_{B} Y$ : $Y I_{D}=r_{B}: r_{D}$. By similarity, we get $D^{\\prime} X: B^{\\prime} X=r_{B}: r_{D}$ and hence $B X: X D=D^{\\prime} X:$ $B^{\\prime} X=r_{B}: r_{D}$. Comment 1. The solution above shows that the problem statement holds also for any parallelogram $K L M N$ whose sides are parallel to the diagonals of $A B C D$, as no property specific for a rhombus has been used. This solution works equally well when two sides of quadrilateral $A B C D$ are parallel. Comment 2. The problem may be reduced to Lemma 1 by using different tools, e.g., by using mass point geometry, linear motion of $K, L, M$, and $N$, etc. Lemma 1 itself also can be proved in different ways. We present below one alternative proof. Proof. In the circumcircle of $A B C D$, let $K^{\\prime}, L^{\\prime} . M^{\\prime}$, and $N^{\\prime}$ be the midpoints of arcs $A B, B C$, $C D$, and $D A$ containing no other vertices of $A B C D$, respectively. Thus, $K^{\\prime}=C I_{B} \\cap D I_{A}$, etc. In the computations below, we denote by $[P]$ the area of a polygon $P$. We use similarities $\\triangle I_{A} B K^{\\prime} \\sim$ $\\triangle I_{A} D N^{\\prime}, \\triangle I_{B} K^{\\prime} L^{\\prime} \\sim \\triangle I_{B} A C$, etc., as well as congruences $\\triangle I_{B} K^{\\prime} L^{\\prime}=\\triangle B K^{\\prime} L^{\\prime}$ and $\\triangle I_{D} M^{\\prime} N^{\\prime}=$ $\\triangle D M^{\\prime} N^{\\prime}$ (e.g., the first congruence holds because $K^{\\prime} L^{\\prime}$ is a common internal bisector of angles $B K^{\\prime} I_{B}$ and $B L^{\\prime} I_{B}$ ). We have $$ \\begin{aligned} & \\frac{B X}{D X}=\\frac{\\left[I_{A} B I_{C}\\right]}{\\left[I_{A} D I_{C}\\right]}=\\frac{B I_{A} \\cdot B I_{C} \\cdot \\sin I_{A} B I_{C}}{D I_{A} \\cdot D I_{C} \\cdot \\sin I_{A} D I_{C}}=\\frac{B I_{A}}{D I_{A}} \\cdot \\frac{B I_{C}}{D I_{C}} \\cdot \\frac{\\sin N^{\\prime} B M^{\\prime}}{\\sin K^{\\prime} D L^{\\prime}} \\\\ & =\\frac{B K^{\\prime}}{D N^{\\prime}} \\cdot \\frac{B L^{\\prime}}{D M^{\\prime}} \\cdot \\frac{\\sin N^{\\prime} D M^{\\prime}}{\\sin K^{\\prime} B L^{\\prime}}=\\frac{B K^{\\prime} \\cdot B L^{\\prime} \\cdot \\sin K^{\\prime} B L^{\\prime}}{D N^{\\prime} \\cdot D M^{\\prime} \\cdot \\sin N^{\\prime} D M^{\\prime}} \\cdot \\frac{\\sin ^{2} N^{\\prime} D M^{\\prime}}{\\sin ^{2} K^{\\prime} B L^{\\prime}} \\\\ & \\quad=\\frac{\\left[K^{\\prime} B L^{\\prime}\\right]}{\\left[M^{\\prime} D N^{\\prime}\\right]} \\cdot \\frac{N^{\\prime} M^{\\prime 2}}{K^{\\prime} L^{\\prime 2}}=\\frac{\\left[K^{\\prime} I_{B} L^{\\prime}\\right] \\cdot \\frac{A^{\\prime} C^{\\prime 2}}{K^{\\prime} L^{\\prime 2}}}{\\left[M^{\\prime} I_{D} N^{\\prime}\\right] \\cdot \\frac{A^{\\prime} C^{\\prime 2}}{N^{\\prime} M^{\\prime 2}}}=\\frac{\\left[A I_{B} C\\right]}{\\left[A I_{D} C\\right]}=\\frac{r_{B}}{r_{D}}, \\end{aligned} $$ as required.","tier":0} +{"problem_type":"Geometry","problem_label":"G5","problem":"Let $A B C D$ be a cyclic quadrilateral with no two sides parallel. Let $K, L, M$, and $N$ be points lying on sides $A B, B C, C D$, and $D A$, respectively, such that $K L M N$ is a rhombus with $K L \\| A C$ and $L M \\| B D$. Let $\\omega_{1}, \\omega_{2}, \\omega_{3}$, and $\\omega_{4}$ be the incircles of triangles $A N K$, $B K L, C L M$, and $D M N$, respectively. Prove that the internal common tangents to $\\omega_{1}$ and $\\omega_{3}$ and the internal common tangents to $\\omega_{2}$ and $\\omega_{4}$ are concurrent. (Poland)","solution":"This solution is based on the following general Lemma. Lemma 2. Let $E$ and $F$ be distinct points, and let $\\omega_{i}, i=1,2,3,4$, be circles lying in the same halfplane with respect to $E F$. For distinct indices $i, j \\in\\{1,2,3,4\\}$, denote by $O_{i j}^{+}$ (respectively, $O_{i j}^{-}$) the center of homothety with positive (respectively, negative) ratio taking $\\omega_{i}$ to $\\omega_{j}$. Suppose that $E=O_{12}^{+}=O_{34}^{+}$and $F=O_{23}^{+}=O_{41}^{+}$. Then $O_{13}^{-}=O_{24}^{-}$. Proof. Applying Monge's theorem to triples of circles $\\omega_{1}, \\omega_{2}, \\omega_{4}$ and $\\omega_{1}, \\omega_{3}, \\omega_{4}$, we get that both points $O_{24}^{-}$and $O_{13}^{-}$lie on line $E O_{14}^{-}$. Notice that this line is distinct from $E F$. Similarly we obtain that both points $O_{24}^{-}$and $O_{13}^{-}$lie on $F O_{34}^{-}$. Since the lines $E O_{14}^{-}$and $F O_{34}^{-}$are distinct, both points coincide with the meeting point of those lines. ![](https:\/\/cdn.mathpix.com\/cropped\/2024_11_18_bcdd154f2030efbe2fa4g-57.jpg?height=716&width=1837&top_left_y=2073&top_left_x=108) Turning back to the problem, let $A B$ intersect $C D$ at $E$ and let $B C$ intersect $D A$ at $F$. Assume, without loss of generality, that $B$ lies on segments $A E$ and $C F$. We will show that the points $E$ and $F$, and the circles $\\omega_{i}$ satisfy the conditions of Lemma 2 , so the problem statement follows. In the sequel, we use the notation of $O_{i j}^{ \\pm}$from the statement of Lemma 2, applied to circles $\\omega_{1}, \\omega_{2}, \\omega_{3}$, and $\\omega_{4}$. Using the relations $\\triangle E C A \\sim \\triangle E B D, K N \\| B D$, and $M N \\| A C$. we get $$ \\frac{A N}{N D}=\\frac{A N}{A D} \\cdot \\frac{A D}{N D}=\\frac{K N}{B D} \\cdot \\frac{A C}{N M}=\\frac{A C}{B D}=\\frac{A E}{E D} $$ Therefore, by the angle bisector theorem, point $N$ lies on the internal angle bisector of $\\angle A E D$. We prove similarly that $L$ also lies on that bisector, and that the points $K$ and $M$ lie on the internal angle bisector of $\\angle A F B$. Since $K L M N$ is a rhombus, points $K$ and $M$ are symmetric in line $E L N$. Hence, the convex quadrilateral determined by the lines $E K, E M, K L$, and $M L$ is a kite, and therefore it has an incircle $\\omega_{0}$. Applying Monge's theorem to $\\omega_{0}, \\omega_{2}$, and $\\omega_{3}$, we get that $O_{23}^{+}$lies on $K M$. On the other hand, $O_{23}^{+}$lies on $B C$, as $B C$ is an external common tangent to $\\omega_{2}$ and $\\omega_{3}$. It follows that $F=O_{23}^{+}$. Similarly, $E=O_{12}^{+}=O_{34}^{+}$, and $F=O_{41}^{+}$. Comment 3. The reduction to Lemma 2 and the proof of Lemma 2 can be performed with the use of different tools, e.g., by means of Menelaus theorem, by projecting harmonic quadruples, by applying Monge's theorem in some other ways, etc. This page is intentionally left blank","tier":0} +{"problem_type":"Geometry","problem_label":"G6","problem":"Let $I$ and $I_{A}$ be the incenter and the $A$-excenter of an acute-angled triangle $A B C$ with $A B(t-1) \\frac{\\sqrt{3}}{2}$, or $$ t<1+\\frac{4 \\sqrt{M}}{\\sqrt{3}}<4 \\sqrt{M} $$ as $M \\geqslant 1$. Combining the estimates (1), (2), and (3), we finally obtain $$ \\frac{1}{4 \\delta} \\leqslant t<4 \\sqrt{M}<4 \\sqrt{2 n \\delta}, \\quad \\text { or } \\quad 512 n \\delta^{3}>1 $$ which does not hold for the chosen value of $\\delta$. Comment 1. As the proposer mentions, the exponent $-1 \/ 3$ in the problem statement is optimal. In fact, for any $n \\geqslant 2$, there is a configuration $\\mathcal{S}$ of $n$ points in the plane such that any two points in $\\mathcal{S}$ are at least 1 apart, but every line $\\ell$ separating $\\mathcal{S}$ is at most $c^{\\prime} n^{-1 \/ 3} \\log n$ apart from some point in $\\mathcal{S}$; here $c^{\\prime}$ is some absolute constant. The original proposal suggested to prove the estimate of the form $\\mathrm{cn}^{-1 \/ 2}$. That version admits much easier solutions. E.g., setting $\\delta=\\frac{1}{16} n^{-1 \/ 2}$ and applying (1), we see that $\\mathcal{S}$ is contained in a disk $D$ of radius $\\frac{1}{8} n^{1 \/ 2}$. On the other hand, for each point $X$ of $\\mathcal{S}$, let $D_{X}$ be the disk of radius $\\frac{1}{2}$ centered at $X$; all these disks have disjoint interiors and lie within the disk concentric to $D$, of radius $\\frac{1}{16} n^{1 \/ 2}+\\frac{1}{2}<\\frac{1}{2} n^{1 \/ 2}$. Comparing the areas, we get $$ n \\cdot \\frac{\\pi}{4} \\leqslant \\pi\\left(\\frac{n^{1 \/ 2}}{16}+\\frac{1}{2}\\right)^{2}<\\frac{\\pi n}{4} $$ which is a contradiction. The Problem Selection Committee decided to choose a harder version for the Shortlist. Comment 2. In this comment, we discuss some versions of the solution above, which avoid concentrating on the diameter of $\\mathcal{S}$. We start with introducing some terminology suitable for those versions. Put $\\delta=c n^{-1 \/ 3}$ for a certain sufficiently small positive constant $c$. For the sake of contradiction, suppose that, for some set $\\mathcal{S}$ satisfying the conditions in the problem statement, there is no separating line which is at least $\\delta$ apart from each point of $\\mathcal{S}$. Let $C$ be the convex hull of $\\mathcal{S}$. A line is separating if and only if it meets $C$ (we assume that a line passing through a point of $\\mathcal{S}$ is always separating). Consider a strip between two parallel separating lines $a$ and $a^{\\prime}$ which are, say, $\\frac{1}{4}$ apart from each other. Define a slice determined by the strip as the intersection of $\\mathcal{S}$ with the strip. The length of the slice is the diameter of the projection of the slice to $a$. In this terminology, the arguments used in the proofs of (2) and (3) show that for any slice $\\mathcal{T}$ of length $L$, we have $$ \\frac{1}{8 \\delta} \\leqslant|\\mathcal{T}| \\leqslant 1+\\frac{4}{\\sqrt{15}} L $$ The key idea of the solution is to apply these estimates to a peel slice, where line $a$ does not cross the interior of $C$. In the above solution, this idea was applied to one carefully chosen peel slice. Here, we outline some different approach involving many of them. We always assume that $n$ is sufficiently large. Consider a peel slice determined by lines $a$ and $a^{\\prime}$, where $a$ contains no interior points of $C$. We orient $a$ so that $C$ lies to the left of $a$. Line $a$ is called a supporting line of the slice, and the obtained direction is the direction of the slice; notice that the direction determines uniquely the supporting line and hence the slice. Fix some direction $\\mathbf{v}_{0}$, and for each $\\alpha \\in[0,2 \\pi)$ denote by $\\mathcal{T}_{\\alpha}$ the peel slice whose direction is $\\mathbf{v}_{0}$ rotated by $\\alpha$ counterclockwise. When speaking about the slice, we always assume that the figure is rotated so that its direction is vertical from the bottom to the top; then the points in $\\mathcal{T}$ get a natural order from the bottom to the top. In particular, we may speak about the top half $\\mathrm{T}(\\mathcal{T})$ consisting of $\\lfloor|\\mathcal{T}| \/ 2\\rfloor$ topmost points in $\\mathcal{T}$, and similarly about its bottom half $\\mathrm{B}(\\mathcal{T})$. By (4), each half contains at least 10 points when $n$ is large. Claim. Consider two angles $\\alpha, \\beta \\in[0, \\pi \/ 2]$ with $\\beta-\\alpha \\geqslant 40 \\delta=: \\phi$. Then all common points of $\\mathcal{T}_{\\alpha}$ and $\\mathcal{T}_{\\beta}$ lie in $\\mathrm{T}\\left(\\mathcal{T}_{\\alpha}\\right) \\cap \\mathrm{B}\\left(\\mathcal{T}_{\\beta}\\right)$. ![](https:\/\/cdn.mathpix.com\/cropped\/2024_11_18_bcdd154f2030efbe2fa4g-71.jpg?height=738&width=1338&top_left_y=201&top_left_x=366) Proof. By symmetry, it suffices to show that all those points lie in $\\mathrm{T}\\left(\\mathcal{T}_{\\alpha}\\right)$. Let $a$ be the supporting line of $\\mathcal{T}_{\\alpha}$, and let $\\ell$ be a line perpendicular to the direction of $\\mathcal{T}_{\\beta}$. Let $P_{1}, \\ldots, P_{k}$ list all points in $\\mathcal{T}_{\\alpha}$, numbered from the bottom to the top; by (4), we have $k \\geqslant \\frac{1}{8} \\delta^{-1}$. Introduce the Cartesian coordinates so that the (oriented) line $a$ is the $y$-axis. Let $P_{i}$ be any point in $\\mathrm{B}\\left(\\mathcal{T}_{\\alpha}\\right)$. The difference of ordinates of $P_{k}$ and $P_{i}$ is at least $\\frac{\\sqrt{15}}{4}(k-i)>\\frac{1}{3} k$, while their abscissas differ by at most $\\frac{1}{4}$. This easily yields that the projections of those points to $\\ell$ are at least $$ \\frac{k}{3} \\sin \\phi-\\frac{1}{4} \\geqslant \\frac{1}{24 \\delta} \\cdot 20 \\delta-\\frac{1}{4}>\\frac{1}{4} $$ apart from each other, and $P_{k}$ is closer to the supporting line of $\\mathcal{T}_{\\beta}$ than $P_{i}$, so that $P_{i}$ does not belong to $\\mathcal{T}_{\\beta}$. Now, put $\\alpha_{i}=40 \\delta i$, for $i=0,1, \\ldots,\\left\\lfloor\\frac{1}{40} \\delta^{-1} \\cdot \\frac{\\pi}{2}\\right\\rfloor$, and consider the slices $\\mathcal{T}_{\\alpha_{i}}$. The Claim yields that each point in $\\mathcal{S}$ is contained in at most two such slices. Hence, the union $\\mathcal{U}$ of those slices contains at least $$ \\frac{1}{2} \\cdot \\frac{1}{8 \\delta} \\cdot \\frac{1}{40 \\delta} \\cdot \\frac{\\pi}{2}=\\frac{\\lambda}{\\delta^{2}} $$ points (for some constant $\\lambda$ ), and each point in $\\mathcal{U}$ is at most $\\frac{1}{4}$ apart from the boundary of $C$. It is not hard now to reach a contradiction with (1). E.g., for each point $X \\in \\mathcal{U}$, consider a closest point $f(X)$ on the boundary of $C$. Obviously, $f(X) f(Y) \\geqslant X Y-\\frac{1}{2} \\geqslant \\frac{1}{2} X Y$ for all $X, Y \\in \\mathcal{U}$. This yields that the perimeter of $C$ is at least $\\mu \\delta^{-2}$, for some constant $\\mu$, and hence the diameter of $\\mathcal{S}$ is of the same order. Alternatively, one may show that the projection of $\\mathcal{U}$ to the line at the angle of $\\pi \/ 4$ with $\\mathbf{v}_{0}$ has diameter at least $\\mu \\delta^{-2}$ for some constant $\\mu$.","tier":0} +{"problem_type":"Number Theory","problem_label":"N3","problem":"Let $n$ be an integer with $n \\geqslant 2$. Does there exist a sequence $\\left(a_{1}, \\ldots, a_{n}\\right)$ of positive integers with not all terms being equal such that the arithmetic mean of every two terms is equal to the geometric mean of some (one or more) terms in this sequence? (Estonia) Answer: No such sequence exists.","solution":"Suppose that $a_{1}, \\ldots, a_{n}$ satisfy the required properties. Let $d=\\operatorname{gcd}\\left(a_{1} \\ldots, a_{n}\\right)$. If $d>1$ then replace the numbers $a_{1}, \\ldots, a_{n}$ by $\\frac{a_{1}}{d}, \\ldots, \\frac{a_{n}}{d}$; all arithmetic and all geometric means will be divided by $d$, so we obtain another sequence satisfying the condition. Hence, without loss of generality, we can assume that $\\operatorname{gcd}\\left(a_{1} \\ldots, a_{n}\\right)=1$. We show two numbers, $a_{m}$ and $a_{k}$ such that their arithmetic mean, $\\frac{a_{m}+a_{k}}{2}$ is different from the geometric mean of any (nonempty) subsequence of $a_{1} \\ldots, a_{n}$. That proves that there cannot exist such a sequence. Choose the index $m \\in\\{1, \\ldots, n\\}$ such that $a_{m}=\\max \\left(a_{1}, \\ldots, a_{n}\\right)$. Note that $a_{m} \\geqslant 2$, because $a_{1}, \\ldots, a_{n}$ are not all equal. Let $p$ be a prime divisor of $a_{m}$. Let $k \\in\\{1, \\ldots, n\\}$ be an index such that $a_{k}=\\max \\left\\{a_{i}: p \\nmid a_{i}\\right\\}$. Due to $\\operatorname{gcd}\\left(a_{1} \\ldots, a_{n}\\right)=1$, not all $a_{i}$ are divisible by $p$, so such a $k$ exists. Note that $a_{m}>a_{k}$ because $a_{m} \\geqslant a_{k}, p \\mid a_{m}$ and $p \\nmid a_{k}$. Let $b=\\frac{a_{m}+a_{k}}{2}$; we will show that $b$ cannot be the geometric mean of any subsequence of $a_{1}, \\ldots, a_{n}$. Consider the geometric mean, $g=\\sqrt[t]{a_{i_{1}} \\cdot \\ldots \\cdot a_{i_{t}}}$ of an arbitrary subsequence of $a_{1}, \\ldots, a_{n}$. If none of $a_{i_{1}}, \\ldots, a_{i_{t}}$ is divisible by $p$, then they are not greater than $a_{k}$, so $$ g=\\sqrt[t]{a_{i_{1}} \\cdot \\ldots \\cdot a_{i_{t}}} \\leqslant a_{k}<\\frac{a_{m}+a_{k}}{2}=b $$ and therefore $g \\neq b$. Otherwise, if at least one of $a_{i_{1}}, \\ldots, a_{i_{t}}$ is divisible by $p$, then $2 g=2 \\sqrt{t} \\sqrt{a_{i_{1}} \\cdot \\ldots \\cdot a_{i_{t}}}$ is either not an integer or is divisible by $p$, while $2 b=a_{m}+a_{k}$ is an integer not divisible by $p$, so $g \\neq b$ again.","tier":0} +{"problem_type":"Number Theory","problem_label":"N3","problem":"Let $n$ be an integer with $n \\geqslant 2$. Does there exist a sequence $\\left(a_{1}, \\ldots, a_{n}\\right)$ of positive integers with not all terms being equal such that the arithmetic mean of every two terms is equal to the geometric mean of some (one or more) terms in this sequence? (Estonia) Answer: No such sequence exists.","solution":"Like in the previous solution, we assume that the numbers $a_{1}, \\ldots, a_{n}$ have no common divisor greater than 1 . The arithmetic mean of any two numbers in the sequence is half of an integer; on the other hand, it is a (some integer order) root of an integer. This means each pair's mean is an integer, so all terms in the sequence must be of the same parity; hence they all are odd. Let $d=\\min \\left\\{\\operatorname{gcd}\\left(a_{i}, a_{j}\\right): a_{i} \\neq a_{j}\\right\\}$. By reordering the sequence we can assume that $\\operatorname{gcd}\\left(a_{1}, a_{2}\\right)=d$, the sum $a_{1}+a_{2}$ is maximal among such pairs, and $a_{1}>a_{2}$. We will show that $\\frac{a_{1}+a_{2}}{2}$ cannot be the geometric mean of any subsequence of $a_{1} \\ldots, a_{n}$. Let $a_{1}=x d$ and $a_{2}=y d$ where $x, y$ are coprime, and suppose that there exist some $b_{1}, \\ldots, b_{t} \\in\\left\\{a_{1}, \\ldots, a_{n}\\right\\}$ whose geometric mean is $\\frac{a_{1}+a_{2}}{2}$. Let $d_{i}=\\operatorname{gcd}\\left(a_{1}, b_{i}\\right)$ for $i=1,2, \\ldots, t$ and let $D=d_{1} d_{2} \\cdot \\ldots \\cdot d_{t}$. Then $$ D=d_{1} d_{2} \\cdot \\ldots \\cdot d_{t} \\left\\lvert\\, b_{1} b_{2} \\cdot \\ldots \\cdot b_{t}=\\left(\\frac{a_{1}+a_{2}}{2}\\right)^{t}=\\left(\\frac{x+y}{2}\\right)^{t} d^{t}\\right. $$ We claim that $D \\mid d^{t}$. Consider an arbitrary prime divisor $p$ of $D$. Let $\\nu_{p}(x)$ denote the exponent of $p$ in the prime factorization of $x$. If $p \\left\\lvert\\, \\frac{x+y}{2}\\right.$, then $p \\nmid x, y$, so $p$ is coprime with $x$; hence, $\\nu_{p}\\left(d_{i}\\right) \\leqslant \\nu_{p}\\left(a_{1}\\right)=\\nu_{p}(x d)=\\nu_{p}(d)$ for every $1 \\leqslant i \\leqslant t$, therefore $\\nu_{p}(D)=\\sum_{i} \\nu_{p}\\left(d_{i}\\right) \\leqslant$ $t \\nu_{p}(d)=\\nu_{p}\\left(d^{t}\\right)$. Otherwise, if $p$ is coprime to $\\frac{x+y}{2}$, we have $\\nu_{p}(D) \\leqslant \\nu_{p}\\left(d^{t}\\right)$ trivially. The claim has been proved. Notice that $d_{i}=\\operatorname{gcd}\\left(b_{i}, a_{1}\\right) \\geqslant d$ for $1 \\leqslant i \\leqslant t$ : if $b_{i} \\neq a_{1}$ then this follows from the definition of $d$; otherwise we have $b_{i}=a_{1}$, so $d_{i}=a_{1} \\geqslant d$. Hence, $D=d_{1} \\cdot \\ldots \\cdot d_{t} \\geqslant d^{t}$, and the claim forces $d_{1}=\\ldots=d_{t}=d$. Finally, by $\\frac{a_{1}+a_{2}}{2}>a_{2}$ there must be some $b_{k}$ which is greater than $a_{2}$. From $a_{1}>a_{2} \\geqslant$ $d=\\operatorname{gcd}\\left(a_{1}, b_{k}\\right)$ it follows that $a_{1} \\neq b_{k}$. Now the have a pair $a_{1}, b_{k}$ such that $\\operatorname{gcd}\\left(a_{1}, b_{k}\\right)=d$ but $a_{1}+b_{k}>a_{1}+a_{2}$; that contradicts the choice of $a_{1}$ and $a_{2}$. Comment. The original problem proposal contained a second question asking if there exists a nonconstant sequence $\\left(a_{1}, \\ldots, a_{n}\\right)$ of positive integers such that the geometric mean of every two terms is equal the arithmetic mean of some terms. For $n \\geqslant 3$ such a sequence is $(4,1,1, \\ldots, 1)$. The case $n=2$ can be done by the trivial estimates $$ \\min \\left(a_{1}, a_{2}\\right)<\\sqrt{a_{1} a_{2}}<\\frac{a_{1}+a_{2}}{2}<\\max \\left(a_{1}, a_{2}\\right) $$ The Problem Selection Committee found this variant less interesting and suggests using only the first question.","tier":0} +{"problem_type":"Number Theory","problem_label":"N4","problem":"For any odd prime $p$ and any integer $n$, let $d_{p}(n) \\in\\{0,1, \\ldots, p-1\\}$ denote the remainder when $n$ is divided by $p$. We say that $\\left(a_{0}, a_{1}, a_{2}, \\ldots\\right)$ is a $p$-sequence, if $a_{0}$ is a positive integer coprime to $p$, and $a_{n+1}=a_{n}+d_{p}\\left(a_{n}\\right)$ for $n \\geqslant 0$. (a) Do there exist infinitely many primes $p$ for which there exist $p$-sequences $\\left(a_{0}, a_{1}, a_{2}, \\ldots\\right)$ and $\\left(b_{0}, b_{1}, b_{2}, \\ldots\\right)$ such that $a_{n}>b_{n}$ for infinitely many $n$, and $b_{n}>a_{n}$ for infinitely many $n$ ? (b) Do there exist infinitely many primes $p$ for which there exist $p$-sequences $\\left(a_{0}, a_{1}, a_{2}, \\ldots\\right)$ and $\\left(b_{0}, b_{1}, b_{2}, \\ldots\\right)$ such that $a_{0}b_{n}$ for all $n \\geqslant 1$ ? (United Kingdom) Answer: Yes, for both parts.","solution":"Fix some odd prime $p$, and let $T$ be the smallest positive integer such that $p \\mid 2^{T}-1$; in other words, $T$ is the multiplicative order of 2 modulo $p$. Consider any $p$-sequence $\\left(x_{n}\\right)=\\left(x_{0}, x_{1}, x_{2}, \\ldots\\right)$. Obviously, $x_{n+1} \\equiv 2 x_{n}(\\bmod p)$ and therefore $x_{n} \\equiv 2^{n} x_{0}(\\bmod p)$. This yields $x_{n+T} \\equiv x_{n}(\\bmod p)$ and therefore $d\\left(x_{n+T}\\right)=d\\left(x_{n}\\right)$ for all $n \\geqslant 0$. It follows that the sum $d\\left(x_{n}\\right)+d\\left(x_{n+1}\\right)+\\ldots+d\\left(x_{n+T-1}\\right)$ does not depend on $n$ and is thus a function of $x_{0}$ (and $p$ ) only; we shall denote this sum by $S_{p}\\left(x_{0}\\right)$, and extend the function $S_{p}(\\cdot)$ to all (not necessarily positive) integers. Therefore, we have $x_{n+k T}=x_{n}+k S_{p}\\left(x_{0}\\right)$ for all positive integers $n$ and $k$. Clearly, $S_{p}\\left(x_{0}\\right)=S_{p}\\left(2^{t} x_{0}\\right)$ for every integer $t \\geqslant 0$. In both parts, we use the notation $$ S_{p}^{+}=S_{p}(1)=\\sum_{i=0}^{T-1} d_{p}\\left(2^{i}\\right) \\quad \\text { and } \\quad S_{p}^{-}=S_{p}(-1)=\\sum_{i=0}^{T-1} d_{p}\\left(p-2^{i}\\right) $$ (a) Let $q>3$ be a prime and $p$ a prime divisor of $2^{q}+1$ that is greater than 3 . We will show that $p$ is suitable for part (a). Notice that $9 \\nmid 2^{q}+1$, so that such a $p$ exists. Moreover, for any two odd primes $qb_{0}$ and $a_{1}=p+2b_{0}+k S_{p}^{+}=b_{k \\cdot 2 q} \\quad \\text { and } \\quad a_{k \\cdot 2 q+1}=a_{1}+k S_{p}^{+}S_{p}\\left(y_{0}\\right)$ but $x_{0}y_{n}$ for every $n \\geqslant q+q \\cdot \\max \\left\\{y_{r}-x_{r}: r=0,1, \\ldots, q-1\\right\\}$. Now, since $x_{0}S_{p}^{-}$, then we can put $x_{0}=1$ and $y_{0}=p-1$. Otherwise, if $S_{p}^{+}p$ must be divisible by $p$. Indeed, if $n=p k+r$ is a good number, $k>0,00$. Let $\\mathcal{B}$ be the set of big primes, and let $p_{1}p_{1} p_{2}$, and let $p_{1}^{\\alpha}$ be the largest power of $p_{1}$ smaller than $n$, so that $n \\leqslant p_{1}^{\\alpha+1}0$. If $f(k)=f(n-k)$ for all $k$, it implies that $\\binom{n-1}{k}$ is not divisible by $p$ for all $k=1,2, \\ldots, n-2$. It is well known that it implies $n=a \\cdot p^{s}, a0, f(q)>0$, there exist only finitely many $n$ which are equal both to $a \\cdot p^{s}, a0$ for at least two primes less than $n$. Let $p_{0}$ be the prime with maximal $g(f, p)$ among all primes $p1$, let $p_{1}, \\ldots, p_{k}$ be all primes between 1 and $N$ and $p_{k+1}, \\ldots, p_{k+s}$ be all primes between $N+1$ and $2 N$. Since for $j \\leqslant k+s$ all prime divisors of $p_{j}-1$ do not exceed $N$, we have $$ \\prod_{j=1}^{k+s}\\left(p_{j}-1\\right)=\\prod_{i=1}^{k} p_{i}^{c_{i}} $$ with some fixed exponents $c_{1}, \\ldots, c_{k}$. Choose a huge prime number $q$ and consider a number $$ n=\\left(p_{1} \\cdot \\ldots \\cdot p_{k}\\right)^{q-1} \\cdot\\left(p_{k+1} \\cdot \\ldots \\cdot p_{k+s}\\right) $$ Then $$ \\varphi(d(n))=\\varphi\\left(q^{k} \\cdot 2^{s}\\right)=q^{k-1}(q-1) 2^{s-1} $$ and $$ d(\\varphi(n))=d\\left(\\left(p_{1} \\cdot \\ldots \\cdot p_{k}\\right)^{q-2} \\prod_{i=1}^{k+s}\\left(p_{i}-1\\right)\\right)=d\\left(\\prod_{i=1}^{k} p_{i}^{q-2+c_{i}}\\right)=\\prod_{i=1}^{k}\\left(q-1+c_{i}\\right) $$ so $$ \\frac{\\varphi(d(n))}{d(\\varphi(n))}=\\frac{q^{k-1}(q-1) 2^{s-1}}{\\prod_{i=1}^{k}\\left(q-1+c_{i}\\right)}=2^{s-1} \\cdot \\frac{q-1}{q} \\cdot \\prod_{i=1}^{k} \\frac{q}{q-1+c_{i}} $$ which can be made arbitrarily close to $2^{s-1}$ by choosing $q$ large enough. It remains to show that $s$ can be arbitrarily large, i.e. that there can be arbitrarily many primes between $N$ and $2 N$. This follows, for instance, from the well-known fact that $\\sum \\frac{1}{p}=\\infty$, where the sum is taken over the set $\\mathbb{P}$ of prime numbers. Indeed, if, for some constant $\\stackrel{p}{C}$, there were always at most $C$ primes between $2^{\\ell}$ and $2^{\\ell+1}$, we would have $$ \\sum_{p \\in \\mathbb{P}} \\frac{1}{p}=\\sum_{\\ell=0}^{\\infty} \\sum_{\\substack{p \\in \\mathbb{P} \\\\ p \\in\\left[2^{\\ell}, 2^{\\ell+1}\\right)}} \\frac{1}{p} \\leqslant \\sum_{\\ell=0}^{\\infty} \\frac{C}{2^{\\ell}}<\\infty, $$ which is a contradiction. Comment 1. Here we sketch several alternative elementary self-contained ways to perform the last step of the solution above. In particular, they avoid using divergence of $\\sum \\frac{1}{p}$. Suppose that for some constant $C$ and for every $k=1,2, \\ldots$ there exist at most $C$ prime numbers between $2^{k}$ and $2^{k+1}$. Consider the prime factorization of the factorial $\\left(2^{n}\\right)!=\\prod p^{\\alpha_{p}}$. We have $\\alpha_{p}=\\left\\lfloor 2^{n} \/ p\\right\\rfloor+\\left\\lfloor 2^{n} \/ p^{2}\\right\\rfloor+\\ldots$. Thus, for $p \\in\\left[2^{k}, 2^{k+1}\\right)$, we get $\\alpha_{p} \\leqslant 2^{n} \/ 2^{k}+2^{n} \/ 2^{k+1}+\\ldots=2^{n-k+1}$, therefore $p^{\\alpha_{p}} \\leqslant 2^{(k+1) 2^{n-k+1}}$. Combining this with the bound $(2 m)!\\geqslant m(m+1) \\cdot \\ldots \\cdot(2 m-1) \\geqslant m^{m}$ for $m=2^{n-1}$ we get $$ 2^{(n-1) \\cdot 2^{n-1}} \\leqslant\\left(2^{n}\\right)!\\leqslant \\prod_{k=1}^{n-1} 2^{C(k+1) 2^{n-k+1}} $$ or $$ \\sum_{k=1}^{n-1} C(k+1) 2^{1-k} \\geqslant \\frac{n-1}{2} $$ that fails for large $n$ since $C(k+1) 2^{1-k}<1 \/ 3$ for all but finitely many $k$. In fact, a much stronger inequality can be obtained in an elementary way: Note that the formula for $\\nu_{p}(n!)$ implies that if $p^{\\alpha}$ is the largest power of $p$ dividing $\\binom{n}{n \/ 2}$, then $p^{\\alpha} \\leqslant n$. By looking at prime factorization of $\\binom{n}{n \/ 2}$ we instantaneously infer that $$ \\pi(n) \\geqslant \\log _{n}\\binom{n}{n \/ 2} \\geqslant \\frac{\\log \\left(2^{n} \/ n\\right)}{\\log n} \\geqslant \\frac{n}{2 \\log n} . $$ This, in particular, implies that for infinitely many $n$ there are at least $\\frac{n}{3 \\log n}$ primes between $n$ and $2 n$.","tier":0} +{"problem_type":"Number Theory","problem_label":"N6","problem":"For a positive integer $n$, let $d(n)$ be the number of positive divisors of $n$, and let $\\varphi(n)$ be the number of positive integers not exceeding $n$ which are coprime to $n$. Does there exist a constant $C$ such that $$ \\frac{\\varphi(d(n))}{d(\\varphi(n))} \\leqslant C $$ for all $n \\geqslant 1$ ? (Cyprus) Answer: No, such constant does not exist.","solution":"In this solution we will use the Prime Number Theorem which states that $$ \\pi(m)=\\frac{m}{\\log m} \\cdot(1+o(1)) $$ as $m$ tends to infinity. Here and below $\\pi(m)$ denotes the number of primes not exceeding $m$, and $\\log$ the natural logarithm. Let $m>5$ be a large positive integer and let $n:=p_{1} p_{2} \\cdot \\ldots \\cdot p_{\\pi(m)}$ be the product of all primes not exceeding $m$. Then $\\varphi(d(n))=\\varphi\\left(2^{\\pi(m)}\\right)=2^{\\pi(m)-1}$. Consider the number $$ \\varphi(n)=\\prod_{k=1}^{\\pi(m)}\\left(p_{k}-1\\right)=\\prod_{s=1}^{\\pi(m \/ 2)} q_{s}^{\\alpha_{s}} $$ where $q_{1}, \\ldots, q_{\\pi(m \/ 2)}$ are primes not exceeding $m \/ 2$. Note that every term $p_{k}-1$ contributes at most one prime $q_{s}>\\sqrt{m}$ into the product $\\prod_{s} q_{s}^{\\alpha_{s}}$, so we have $$ \\sum_{s: q_{s}>\\sqrt{m}} \\alpha_{s} \\leqslant \\pi(m) \\Longrightarrow \\sum_{s: q_{s}>\\sqrt{m}}\\left(1+\\alpha_{s}\\right) \\leqslant \\pi(m)+\\pi(m \/ 2) . $$ Hence, applying the AM-GM inequality and the inequality $(A \/ x)^{x} \\leqslant e^{A \/ e}$, we obtain $$ \\prod_{s: q_{s}>\\sqrt{m}}\\left(\\alpha_{s}+1\\right) \\leqslant\\left(\\frac{\\pi(m)+\\pi(m \/ 2)}{\\ell}\\right)^{\\ell} \\leqslant \\exp \\left(\\frac{\\pi(m)+\\pi(m \/ 2)}{e}\\right) $$ where $\\ell$ is the number of primes in the interval $(\\sqrt{m}, m]$. We then use a trivial bound $\\alpha_{i} \\leqslant \\log _{2}(\\varphi(n)) \\leqslant \\log _{2} n<\\log _{2}\\left(m^{m}\\right)a, c$, then $b \\nmid a+c$. Otherwise, since $b \\neq a+c$, we would have $b \\leqslant(a+c) \/ 2<\\max \\{a, c\\}$.","solution":"We prove the following stronger statement. Claim. Let $\\mathcal{S}$ be a good set consisting of $n \\geqslant 2$ positive integers. Then the elements of $\\mathcal{S}$ may be ordered as $a_{1}, a_{2}, \\ldots, a_{n}$ so that $a_{i} \\nmid a_{i-1}+a_{i+1}$ and $a_{i} \\nmid a_{i-1}-a_{i+1}$, for all $i=2,3, \\ldots, n-1$. Proof. Say that the ordering $a_{1}, \\ldots, a_{n}$ of $\\mathcal{S}$ is nice if it satisfies the required property. We proceed by induction on $n$. The base case $n=2$ is trivial, as there are no restrictions on the ordering. To perform the step of induction, suppose that $n \\geqslant 3$. Let $a=\\max \\mathcal{S}$, and set $\\mathcal{T}=\\mathcal{S} \\backslash\\{a\\}$. Use the inductive hypothesis to find a nice ordering $b_{1}, \\ldots, b_{n-1}$ of $\\mathcal{T}$. We will show that $a$ may be inserted into this sequence so as to reach a nice ordering of $\\mathcal{S}$. In other words, we will show that there exists a $j \\in\\{1,2, \\ldots, n\\}$ such that the ordering $$ N_{j}=\\left(b_{1}, \\ldots, b_{j-1}, a, b_{j}, b_{j+1}, \\ldots, b_{n-1}\\right) $$ is nice. Assume that, for some $j$, the ordering $N_{j}$ is not nice, so that some element $x$ in it divides either the sum or the difference of two adjacent ones. This did not happen in the ordering of $\\mathcal{T}$, hence $x \\in\\left\\{b_{j-1}, a, b_{j}\\right\\}$ (if, say, $b_{j-1}$ does not exist, then $x \\in\\left\\{a, b_{j}\\right\\}$; a similar agreement is applied hereafter). But the case $x=a$ is impossible: $a$ cannot divide $b_{j-1}-b_{j}$, since $0<\\left|b_{j-1}-b_{j}\\right|a, c$, then $b \\nmid a+c$. Otherwise, since $b \\neq a+c$, we would have $b \\leqslant(a+c) \/ 2<\\max \\{a, c\\}$.","solution":"We again prove a stronger statement. Claim. Let $\\mathcal{S}$ be an arbitrary set of $n \\geqslant 3$ positive integers. Then its elements can be ordered as $a_{1}, \\ldots, a_{n}$ so that, if $a_{i} \\mid a_{i-1}+a_{i+1}$, then $a_{i}=\\max \\mathcal{S}$. The claim easily implies what we need to prove, due to Observation A. To prove the Claim, introduce the function $f$ which assigns to any two elements $a, b \\in \\mathcal{S}$ with $aa_{j+1}>\\ldots>$ $a_{n}$; and (ii) $f$-avoidance: If $a3 b$.","solution":"(By contradiction) Suppose that there exist $4 n+2$ non-negative integers $x_{0}<$ $x_{1}<\\cdots5^{n} \\cdot 1 $$","tier":0} +{"problem_type":"Algebra","problem_label":"A1","problem":"Let $n$ be an integer, and let $A$ be a subset of $\\left\\{0,1,2,3, \\ldots, 5^{n}\\right\\}$ consisting of $4 n+2$ numbers. Prove that there exist $a, b, c \\in A$ such that $a3 b$.","solution":"Denote the maximum element of $A$ by $c$. For $k=0, \\ldots, 4 n-1$ let $$ A_{k}=\\left\\{x \\in A:\\left(1-(2 \/ 3)^{k}\\right) c \\leqslant x<\\left(1-(2 \/ 3)^{k+1}\\right) c\\right\\} $$ Note that $$ \\left(1-(2 \/ 3)^{4 n}\\right) c=c-(16 \/ 81)^{n} c>c-(1 \/ 5)^{n} c \\geqslant c-1 $$ which shows that the sets $A_{0}, A_{1}, \\ldots, A_{4 n-1}$ form a partition of $A \\backslash\\{c\\}$. Since $A \\backslash\\{c\\}$ has $4 n+1$ elements, by the pigeonhole principle some set $A_{k}$ does contain at least two elements of $A \\backslash\\{c\\}$. Denote these two elements $a$ and $b$ and assume $a3 b $$ as desired.","tier":0} +{"problem_type":"Algebra","problem_label":"A2","problem":"For every integer $n \\geqslant 1$ consider the $n \\times n$ table with entry $\\left\\lfloor\\frac{i j}{n+1}\\right\\rfloor$ at the intersection of row $i$ and column $j$, for every $i=1, \\ldots, n$ and $j=1, \\ldots, n$. Determine all integers $n \\geqslant 1$ for which the sum of the $n^{2}$ entries in the table is equal to $\\frac{1}{4} n^{2}(n-1)$.","solution":"First, observe that every pair $x, y$ of real numbers for which the sum $x+y$ is integer satisfies $$ \\lfloor x\\rfloor+\\lfloor y\\rfloor \\geqslant x+y-1 $$ The inequality is strict if $x$ and $y$ are integers, and it holds with equality otherwise. We estimate the sum $S$ as follows. $$ \\begin{aligned} 2 S=\\sum_{1 \\leqslant i, j \\leqslant n}\\left(\\left\\lfloor\\frac{i j}{n+1}\\right\\rfloor+\\left\\lfloor\\frac{i j}{n+1}\\right\\rfloor\\right)= & \\sum_{1 \\leqslant i, j \\leqslant n}\\left(\\left\\lfloor\\frac{i j}{n+1}\\right\\rfloor+\\left\\lfloor\\frac{(n+1-i) j}{n+1}\\right\\rfloor\\right) \\\\ & \\geqslant \\sum_{1 \\leqslant i, j \\leqslant n}(j-1)=\\frac{(n-1) n^{2}}{2} . \\end{aligned} $$ The inequality in the last line follows from (1) by setting $x=i j \/(n+1)$ and $y=(n+1-$ i) $j \/(n+1)$, so that $x+y=j$ is integral. Now $S=\\frac{1}{4} n^{2}(n-1)$ if and only if the inequality in the last line holds with equality, which means that none of the values $i j \/(n+1)$ with $1 \\leqslant i, j \\leqslant n$ may be integral. Hence, if $n+1$ is composite with factorisation $n+1=a b$ for $2 \\leqslant a, b \\leqslant n$, one gets a strict inequality for $i=a$ and $j=b$. If $n+1$ is a prime, then $i j \/(n+1)$ is never integral and $S=\\frac{1}{4} n^{2}(n-1)$.","tier":0} +{"problem_type":"Algebra","problem_label":"A2","problem":"For every integer $n \\geqslant 1$ consider the $n \\times n$ table with entry $\\left\\lfloor\\frac{i j}{n+1}\\right\\rfloor$ at the intersection of row $i$ and column $j$, for every $i=1, \\ldots, n$ and $j=1, \\ldots, n$. Determine all integers $n \\geqslant 1$ for which the sum of the $n^{2}$ entries in the table is equal to $\\frac{1}{4} n^{2}(n-1)$.","solution":"To simplify the calculation with indices, extend the table by adding a phantom column of index 0 with zero entries (which will not change the sum of the table). Fix a row $i$ with $1 \\leqslant i \\leqslant n$, and let $d:=\\operatorname{gcd}(i, n+1)$ and $k:=(n+1) \/ d$. For columns $j=0, \\ldots, n$, define the remainder $r_{j}:=i j \\bmod (n+1)$. We first prove the following Claim. For every integer $g$ with $1 \\leqslant g \\leqslant d$, the remainders $r_{j}$ with indices $j$ in the range $$ (g-1) k \\leqslant j \\leqslant g k-1 $$ form a permutation of the $k$ numbers $0 \\cdot d, 1 \\cdot d, 2 \\cdot d, \\ldots,(k-1) \\cdot d$. Proof. If $r_{j^{\\prime}}=r_{j}$ holds for two indices $j^{\\prime}$ and $j$ in (2), then $i\\left(j^{\\prime}-j\\right) \\equiv 0 \\bmod (n+1)$, so that $j^{\\prime}-j$ is a multiple of $k$; since $\\left|j^{\\prime}-j\\right| \\leqslant k-1$, this implies $j^{\\prime}=j$. Hence, the $k$ remainders are pairwise distinct. Moreover, each remainder $r_{j}=i j \\bmod (n+1)$ is a multiple of $d=\\operatorname{gcd}(i, n+1)$. This proves the claim. We then have $$ \\sum_{j=0}^{n} r_{j}=\\sum_{g=1}^{d} \\sum_{\\ell=0}^{(n+1) \/ d-1} \\ell d=d^{2} \\cdot \\frac{1}{2}\\left(\\frac{n+1}{d}-1\\right) \\frac{n+1}{d}=\\frac{(n+1-d)(n+1)}{2} $$ By using (3), compute the sum $S_{i}$ of row $i$ as follows: $$ \\begin{aligned} S_{i}=\\sum_{j=0}^{n}\\left\\lfloor\\frac{i j}{n+1}\\right\\rfloor= & \\sum_{j=0}^{n} \\frac{i j-r_{j}}{n+1}=\\frac{i}{n+1} \\sum_{j=0}^{n} j-\\frac{1}{n+1} \\sum_{j=0}^{n} r_{j} \\\\ & =\\frac{i}{n+1} \\cdot \\frac{n(n+1)}{2}-\\frac{1}{n+1} \\cdot \\frac{(n+1-d)(n+1)}{2}=\\frac{(i n-n-1+d)}{2} . \\end{aligned} $$ Equation (4) yields the following lower bound on the row sum $S_{i}$, which holds with equality if and only if $d=\\operatorname{gcd}(i, n+1)=1$ : $$ S_{i} \\geqslant \\frac{(i n-n-1+1)}{2}=\\frac{n(i-1)}{2} $$ By summing up the bounds (5) for the rows $i=1, \\ldots, n$, we get the following lower bound for the sum of all entries in the table $$ \\sum_{i=1}^{n} S_{i} \\geqslant \\sum_{i=1}^{n} \\frac{n}{2}(i-1)=\\frac{n^{2}(n-1)}{4} $$ In (6) we have equality if and only if equality holds in (5) for each $i=1, \\ldots, n$, which happens if and only if $\\operatorname{gcd}(i, n+1)=1$ for each $i=1, \\ldots, n$, which is equivalent to the fact that $n+1$ is a prime. Thus the sum of the table entries is $\\frac{1}{4} n^{2}(n-1)$ if and only if $n+1$ is a prime. Comment. To simplify the answer, in the problem statement one can make a change of variables by introducing $m:=n+1$ and writing everything in terms of $m$. The drawback is that the expression for the sum will then be $\\frac{1}{4}(m-1)^{2}(m-2)$ which seems more artificial.","tier":0} +{"problem_type":"Algebra","problem_label":"A3","problem":"Given a positive integer $n$, find the smallest value of $\\left\\lfloor\\frac{a_{1}}{1}\\right\\rfloor+\\left\\lfloor\\frac{a_{2}}{2}\\right\\rfloor+\\cdots+\\left\\lfloor\\frac{a_{n}}{n}\\right\\rfloor$ over all permutations $\\left(a_{1}, a_{2}, \\ldots, a_{n}\\right)$ of $(1,2, \\ldots, n)$.","solution":"Suppose that $2^{k} \\leqslant n<2^{k+1}$ with some nonnegative integer $k$. First we show a permutation $\\left(a_{1}, a_{2}, \\ldots, a_{n}\\right)$ such that $\\left\\lfloor\\frac{a_{1}}{1}\\right\\rfloor+\\left\\lfloor\\frac{a_{2}}{2}\\right\\rfloor+\\cdots+\\left\\lfloor\\frac{a_{n}}{n}\\right\\rfloor=k+1$; then we will prove that $\\left\\lfloor\\frac{a_{1}}{1}\\right\\rfloor+\\left\\lfloor\\frac{a_{2}}{2}\\right\\rfloor+\\cdots+\\left\\lfloor\\frac{a_{n}}{n}\\right\\rfloor \\geqslant k+1$ for every permutation. Hence, the minimal possible value will be $k+1$. I. Consider the permutation $$ \\begin{gathered} \\left(a_{1}\\right)=(1), \\quad\\left(a_{2}, a_{3}\\right)=(3,2), \\quad\\left(a_{4}, a_{5}, a_{6}, a_{7}\\right)=(7,4,5,6) \\\\ \\left(a_{2^{k-1}}, \\ldots, a_{2^{k}-1}\\right)=\\left(2^{k}-1,2^{k-1}, 2^{k-1}+1, \\ldots, 2^{k}-2\\right) \\\\ \\left(a_{2^{k}}, \\ldots, a_{n}\\right)=\\left(n, 2^{k}, 2^{k}+1, \\ldots, n-1\\right) \\end{gathered} $$ This permutation consists of $k+1$ cycles. In every cycle $\\left(a_{p}, \\ldots, a_{q}\\right)=(q, p, p+1, \\ldots, q-1)$ we have $q<2 p$, so $$ \\sum_{i=p}^{q}\\left\\lfloor\\frac{a_{i}}{i}\\right\\rfloor=\\left\\lfloor\\frac{q}{p}\\right\\rfloor+\\sum_{i=p+1}^{q}\\left\\lfloor\\frac{i-1}{i}\\right\\rfloor=1 ; $$ The total sum over all cycles is precisely $k+1$. II. In order to establish the lower bound, we prove a more general statement. Claim. If $b_{1}, \\ldots, b_{2^{k}}$ are distinct positive integers then $$ \\sum_{i=1}^{2^{k}}\\left\\lfloor\\frac{b_{i}}{i}\\right\\rfloor \\geqslant k+1 $$ From the Claim it follows immediately that $\\sum_{i=1}^{n}\\left\\lfloor\\frac{a_{i}}{i}\\right\\rfloor \\geqslant \\sum_{i=1}^{2^{k}}\\left\\lfloor\\frac{a_{i}}{i}\\right\\rfloor \\geqslant k+1$. Proof of the Claim. Apply induction on $k$. For $k=1$ the claim is trivial, $\\left\\lfloor\\frac{b_{1}}{1}\\right\\rfloor \\geqslant 1$. Suppose the Claim holds true for some positive integer $k$, and consider $k+1$. If there exists an index $j$ such that $2^{k}L$, it suffices to prove the inequalities $H\\left(-\\left(x_{i}+x_{j}\\right) \/ 2\\right) \\geqslant L$, that is to prove the original inequality in the case when all numbers are shifted in such a way that two variables $x_{i}$ and $x_{j}$ add up to zero. In the following we denote the shifted variables still by $x_{i}$. If $i=j$, i.e. $x_{i}=0$ for some index $i$, then we can remove $x_{i}$ which will decrease both sides by $2 \\sum_{k} \\sqrt{\\left|x_{k}\\right|}$. Similarly, if $x_{i}+x_{j}=0$ for distinct $i$ and $j$ we can remove both $x_{i}$ and $x_{j}$ which decreases both sides by $$ 2 \\sqrt{2\\left|x_{i}\\right|}+2 \\cdot \\sum_{k \\neq i, j}\\left(\\sqrt{\\left|x_{k}+x_{i}\\right|}+\\sqrt{\\left|x_{k}+x_{j}\\right|}\\right) $$ In either case we reduced our inequality to the case of smaller $n$. It remains to note that for $n=0$ and $n=1$ the inequality is trivial.","tier":0} +{"problem_type":"Algebra","problem_label":"A4","problem":"Show that for all real numbers $x_{1}, \\ldots, x_{n}$ the following inequality holds: $$ \\sum_{i=1}^{n} \\sum_{j=1}^{n} \\sqrt{\\left|x_{i}-x_{j}\\right|} \\leqslant \\sum_{i=1}^{n} \\sum_{j=1}^{n} \\sqrt{\\left|x_{i}+x_{j}\\right|} $$","solution":"For real $p$ consider the integral $$ I(p)=\\int_{0}^{\\infty} \\frac{1-\\cos (p x)}{x \\sqrt{x}} d x $$ which clearly converges to a strictly positive number. By changing the variable $y=|p| x$ one notices that $I(p)=\\sqrt{|p|} I(1)$. Hence, by using the trigonometric formula $\\cos (\\alpha-\\beta)-\\cos (\\alpha+$ $\\beta)=2 \\sin \\alpha \\sin \\beta$ we obtain $\\sqrt{|a+b|}-\\sqrt{|a-b|}=\\frac{1}{I(1)} \\int_{0}^{\\infty} \\frac{\\cos ((a-b) x)-\\cos ((a+b) x)}{x \\sqrt{x}} d x=\\frac{1}{I(1)} \\int_{0}^{\\infty} \\frac{2 \\sin (a x) \\sin (b x)}{x \\sqrt{x}} d x$, from which our inequality immediately follows: $$ \\sum_{i=1}^{n} \\sum_{j=1}^{n} \\sqrt{\\left|x_{i}+x_{j}\\right|}-\\sum_{i=1}^{n} \\sum_{j=1}^{n} \\sqrt{\\left|x_{i}-x_{j}\\right|}=\\frac{2}{I(1)} \\int_{0}^{\\infty} \\frac{\\left(\\sum_{i=1}^{n} \\sin \\left(x_{i} x\\right)\\right)^{2}}{x \\sqrt{x}} d x \\geqslant 0 $$ Comment 1. A more general inequality $$ \\sum_{i=1}^{n} \\sum_{j=1}^{n}\\left|x_{i}-x_{j}\\right|^{r} \\leqslant \\sum_{i=1}^{n} \\sum_{j=1}^{n}\\left|x_{i}+x_{j}\\right|^{r} $$ holds for any $r \\in[0,2]$. The first solution can be repeated verbatim for any $r \\in[0,1]$ but not for $r>1$. In the second solution, by putting $x^{r+1}$ in the denominator in place of $x \\sqrt{x}$ we can prove the inequality for any $r \\in(0,2)$ and the cases $r=0,2$ are easy to check by hand. Comment 2. In fact, the integral from Solution 2 can be computed explicitly, we have $I(1)=\\sqrt{2 \\pi}$.","tier":0} +{"problem_type":"Algebra","problem_label":"A5","problem":"Let $n \\geqslant 2$ be an integer, and let $a_{1}, a_{2}, \\ldots, a_{n}$ be positive real numbers such that $a_{1}+a_{2}+\\cdots+a_{n}=1$. Prove that $$ \\sum_{k=1}^{n} \\frac{a_{k}}{1-a_{k}}\\left(a_{1}+a_{2}+\\cdots+a_{k-1}\\right)^{2}<\\frac{1}{3} $$","solution":"For all $k \\leqslant n$, let $$ s_{k}=a_{1}+a_{2}+\\cdots+a_{k} \\quad \\text { and } \\quad b_{k}=\\frac{a_{k} s_{k-1}^{2}}{1-a_{k}} $$ with the convention that $s_{0}=0$. Note that $b_{k}$ is exactly a summand in the sum we need to estimate. We shall prove the inequality $$ b_{k}<\\frac{s_{k}^{3}-s_{k-1}^{3}}{3} $$ Indeed, it suffices to check that $$ \\begin{aligned} (1) & \\Longleftrightarrow 0<\\left(1-a_{k}\\right)\\left(\\left(s_{k-1}+a_{k}\\right)^{3}-s_{k-1}^{3}\\right)-3 a_{k} s_{k-1}^{2} \\\\ & \\Longleftrightarrow 0<\\left(1-a_{k}\\right)\\left(3 s_{k-1}^{2}+3 s_{k-1} a_{k}+a_{k}^{2}\\right)-3 s_{k-1}^{2} \\\\ & \\Longleftrightarrow 0<-3 a_{k} s_{k-1}^{2}+3\\left(1-a_{k}\\right) s_{k-1} a_{k}+\\left(1-a_{k}\\right) a_{k}^{2} \\\\ & \\Longleftrightarrow 0<3\\left(1-a_{k}-s_{k-1}\\right) s_{k-1} a_{k}+\\left(1-a_{k}\\right) a_{k}^{2} \\end{aligned} $$ which holds since $a_{k}+s_{k-1}=s_{k} \\leqslant 1$ and $a_{k} \\in(0,1)$. Thus, adding inequalities (1) for $k=1, \\ldots, n$, we conclude that $$ b_{1}+b_{2}+\\cdots+b_{n}<\\frac{s_{n}^{3}-s_{0}^{3}}{3}=\\frac{1}{3} $$ as desired. Comment 1. There are many ways of proving (1) which can be written as $$ \\frac{a s^{2}}{1-a}-\\frac{(a+s)^{3}-s^{3}}{3}<0 $$ for non-negative $a$ and $s$ satisfying $a+s \\leqslant 1$ and $a>0$. E.g., note that for any fixed $a$ the expression in (2) is quadratic in $s$ with the leading coefficient $a \/(1-a)-a>0$. Hence, it is convex as a function in $s$, so it suffices to check the inequality at $s=0$ and $s=1-a$. The former case is trivial and in the latter case the inequality can be rewritten as $$ a s-\\frac{3 a s(a+s)+a^{3}}{3}<0, $$ which is trivial since $a+s=1$.","tier":0} +{"problem_type":"Algebra","problem_label":"A5","problem":"Let $n \\geqslant 2$ be an integer, and let $a_{1}, a_{2}, \\ldots, a_{n}$ be positive real numbers such that $a_{1}+a_{2}+\\cdots+a_{n}=1$. Prove that $$ \\sum_{k=1}^{n} \\frac{a_{k}}{1-a_{k}}\\left(a_{1}+a_{2}+\\cdots+a_{k-1}\\right)^{2}<\\frac{1}{3} $$","solution":"First, let us define $$ S\\left(a_{1}, \\ldots, a_{n}\\right):=\\sum_{k=1}^{n} \\frac{a_{k}}{1-a_{k}}\\left(a_{1}+a_{2}+\\cdots+a_{k-1}\\right)^{2} $$ For some index $i$, denote $a_{1}+\\cdots+a_{i-1}$ by $s$. If we replace $a_{i}$ with two numbers $a_{i} \/ 2$ and $a_{i} \/ 2$, i.e. replace the tuple $\\left(a_{1}, \\ldots, a_{n}\\right)$ with $\\left(a_{1}, \\ldots, a_{i-1}, a_{i} \/ 2, a_{i} \/ 2, a_{i+1}, \\ldots, a_{n}\\right)$, the sum will increase by $$ \\begin{aligned} S\\left(a_{1}, \\ldots, a_{i} \/ 2, a_{i} \/ 2, \\ldots, a_{n}\\right)-S\\left(a_{1}, \\ldots, a_{n}\\right) & =\\frac{a_{i} \/ 2}{1-a_{i} \/ 2}\\left(s^{2}+\\left(s+a_{i} \/ 2\\right)^{2}\\right)-\\frac{a_{i}}{1-a_{i}} s^{2} \\\\ & =a_{i} \\frac{\\left(1-a_{i}\\right)\\left(2 s^{2}+s a_{i}+a_{i}^{2} \/ 4\\right)-\\left(2-a_{i}\\right) s^{2}}{\\left(2-a_{i}\\right)\\left(1-a_{i}\\right)} \\\\ & =a_{i} \\frac{\\left(1-a_{i}-s\\right) s a_{i}+\\left(1-a_{i}\\right) a_{i}^{2} \/ 4}{\\left(2-a_{i}\\right)\\left(1-a_{i}\\right)} \\end{aligned} $$ which is strictly positive. So every such replacement strictly increases the sum. By repeating this process and making maximal number in the tuple tend to zero, we keep increasing the sum which will converge to $$ \\int_{0}^{1} x^{2} d x=\\frac{1}{3} . $$ This completes the proof.","tier":0} +{"problem_type":"Algebra","problem_label":"A5","problem":"Let $n \\geqslant 2$ be an integer, and let $a_{1}, a_{2}, \\ldots, a_{n}$ be positive real numbers such that $a_{1}+a_{2}+\\cdots+a_{n}=1$. Prove that $$ \\sum_{k=1}^{n} \\frac{a_{k}}{1-a_{k}}\\left(a_{1}+a_{2}+\\cdots+a_{k-1}\\right)^{2}<\\frac{1}{3} $$","solution":"We sketch a probabilistic version of the first solution. Let $x_{1}, x_{2}, x_{3}$ be drawn uniformly and independently at random from the segment [0,1]. Let $I_{1} \\cup I_{2} \\cup \\cdots \\cup I_{n}$ be a partition of $[0,1]$ into segments of length $a_{1}, a_{2}, \\ldots, a_{n}$ in this order. Let $J_{k}:=I_{1} \\cup \\cdots \\cup I_{k-1}$ for $k \\geqslant 2$ and $J_{1}:=\\varnothing$. Then $$ \\begin{aligned} \\frac{1}{3}= & \\sum_{k=1}^{n} \\mathbb{P}\\left\\{x_{1} \\geqslant x_{2}, x_{3} ; x_{1} \\in I_{k}\\right\\} \\\\ = & \\sum_{k=1}^{n}\\left(\\mathbb{P}\\left\\{x_{1} \\in I_{k} ; x_{2}, x_{3} \\in J_{k}\\right\\}\\right. \\\\ + & 2 \\cdot \\mathbb{P}\\left\\{x_{1} \\geqslant x_{2} ; x_{1}, x_{2} \\in I_{k} ; x_{3} \\in J_{k}\\right\\} \\\\ & \\left.+\\mathbb{P}\\left\\{x_{1} \\geqslant x_{2}, x_{3} ; x_{1}, x_{2}, x_{3} \\in I_{k}\\right\\}\\right) \\\\ = & \\sum_{k=1}^{n}\\left(a_{k}\\left(a_{1}+\\cdots+a_{k-1}\\right)^{2}+2 \\cdot \\frac{a_{k}^{2}}{2} \\cdot\\left(a_{1}+\\cdots+a_{k-1}\\right)+\\frac{a_{k}^{3}}{3}\\right) \\\\ > & \\sum_{k=1}^{n}\\left(a_{k}\\left(a_{1}+\\cdots+a_{k-1}\\right)^{2}+a_{k}^{2}\\left(a_{1}+\\cdots+a_{k-1}\\right) \\cdot \\frac{a_{1}+\\cdots+a_{k-1}}{1-a_{k}}\\right) \\end{aligned} $$ where for the last inequality we used that $1-a_{k} \\geqslant a_{1}+\\cdots+a_{k-1}$. This completes the proof since $$ a_{k}+\\frac{a_{k}^{2}}{1-a_{k}}=\\frac{a_{k}}{1-a_{k}} $$","tier":0} +{"problem_type":"Algebra","problem_label":"A6","problem":"Let $A$ be a finite set of (not necessarily positive) integers, and let $m \\geqslant 2$ be an integer. Assume that there exist non-empty subsets $B_{1}, B_{2}, B_{3}, \\ldots, B_{m}$ of $A$ whose elements add up to the sums $m^{1}, m^{2}, m^{3}, \\ldots, m^{m}$, respectively. Prove that $A$ contains at least $m \/ 2$ elements.","solution":"Let $A=\\left\\{a_{1}, \\ldots, a_{k}\\right\\}$. Assume that, on the contrary, $k=|A|0$ and large enough $m$ we get $k \\geqslant(1 \/ 2-\\varepsilon) m$. The proof uses the fact that the combinations $\\sum c_{i}$ ! with $c_{i} \\in\\{0,1, \\ldots, i\\}$ are all distinct. Comment 2. The problem statement holds also if $A$ is a set of real numbers (not necessarily integers), the above proofs work in the real case.","tier":0} +{"problem_type":"Algebra","problem_label":"A7","problem":"Let $n \\geqslant 1$ be an integer, and let $x_{0}, x_{1}, \\ldots, x_{n+1}$ be $n+2$ non-negative real numbers that satisfy $x_{i} x_{i+1}-x_{i-1}^{2} \\geqslant 1$ for all $i=1,2, \\ldots, n$. Show that $$ x_{0}+x_{1}+\\cdots+x_{n}+x_{n+1}>\\left(\\frac{2 n}{3}\\right)^{3 \/ 2} $$","solution":"Lemma 1.1. If $a, b, c$ are non-negative numbers such that $a b-c^{2} \\geqslant 1$, then $$ (a+2 b)^{2} \\geqslant(b+2 c)^{2}+6 $$ Proof. $(a+2 b)^{2}-(b+2 c)^{2}=(a-b)^{2}+2(b-c)^{2}+6\\left(a b-c^{2}\\right) \\geqslant 6$. Lemma 1.2. $\\sqrt{1}+\\cdots+\\sqrt{n}>\\frac{2}{3} n^{3 \/ 2}$. Proof. Bernoulli's inequality $(1+t)^{3 \/ 2}>1+\\frac{3}{2} t$ for $0>t \\geqslant-1$ (or, alternatively, a straightforward check) gives $$ (k-1)^{3 \/ 2}=k^{3 \/ 2}\\left(1-\\frac{1}{k}\\right)^{3 \/ 2}>k^{3 \/ 2}\\left(1-\\frac{3}{2 k}\\right)=k^{3 \/ 2}-\\frac{3}{2} \\sqrt{k} . $$ Summing up (*) over $k=1,2, \\ldots, n$ yields $$ 0>n^{3 \/ 2}-\\frac{3}{2}(\\sqrt{1}+\\cdots+\\sqrt{n}) . $$ Now put $y_{i}:=2 x_{i}+x_{i+1}$ for $i=0,1, \\ldots, n$. We get $y_{0} \\geqslant 0$ and $y_{i}^{2} \\geqslant y_{i-1}^{2}+6$ for $i=1,2, \\ldots, n$ by Lemma 1.1. Thus, an easy induction on $i$ gives $y_{i} \\geqslant \\sqrt{6 i}$. Using this estimate and Lemma 1.2 we get $$ 3\\left(x_{0}+\\ldots+x_{n+1}\\right) \\geqslant y_{1}+\\ldots+y_{n} \\geqslant \\sqrt{6}(\\sqrt{1}+\\sqrt{2}+\\ldots+\\sqrt{n})>\\sqrt{6} \\cdot \\frac{2}{3} n^{3 \/ 2}=3\\left(\\frac{2 n}{3}\\right)^{3 \/ 2} $$","tier":0} +{"problem_type":"Algebra","problem_label":"A7","problem":"Let $n \\geqslant 1$ be an integer, and let $x_{0}, x_{1}, \\ldots, x_{n+1}$ be $n+2$ non-negative real numbers that satisfy $x_{i} x_{i+1}-x_{i-1}^{2} \\geqslant 1$ for all $i=1,2, \\ldots, n$. Show that $$ x_{0}+x_{1}+\\cdots+x_{n}+x_{n+1}>\\left(\\frac{2 n}{3}\\right)^{3 \/ 2} $$","solution":"Say that an index $i \\in\\{0,1, \\ldots, n+1\\}$ is good, if $x_{i} \\geqslant \\sqrt{\\frac{2}{3}} i$, otherwise call the index $i$ bad. Lemma 2.1. There are no two consecutive bad indices. Proof. Assume the contrary and consider two bad indices $j, j+1$ with minimal possible $j$. Since 0 is good, we get $j>0$, thus by minimality $j-1$ is a good index and we have $$ \\frac{2}{3} \\sqrt{j(j+1)}>x_{j} x_{j+1} \\geqslant x_{j-1}^{2}+1 \\geqslant \\frac{2}{3}(j-1)+1=\\frac{2}{3} \\cdot \\frac{j+(j+1)}{2} $$ that contradicts the AM-GM inequality for numbers $j$ and $j+1$. Lemma 2.2. If an index $j \\leqslant n-1$ is good, then $$ x_{j+1}+x_{j+2} \\geqslant \\sqrt{\\frac{2}{3}}(\\sqrt{j+1}+\\sqrt{j+2}) . $$ Proof. We have $$ x_{j+1}+x_{j+2} \\geqslant 2 \\sqrt{x_{j+1} x_{j+2}} \\geqslant 2 \\sqrt{x_{j}^{2}+1} \\geqslant 2 \\sqrt{\\frac{2}{3} j+1} \\geqslant \\sqrt{\\frac{2}{3} j+\\frac{2}{3}}+\\sqrt{\\frac{2}{3} j+\\frac{4}{3}}, $$ the last inequality follows from concavity of the square root function, or, alternatively, from the AM-QM inequality for the numbers $\\sqrt{\\frac{2}{3} j+\\frac{2}{3}}$ and $\\sqrt{\\frac{2}{3} j+\\frac{4}{3}}$. Let $S_{i}=x_{1}+\\ldots+x_{i}$ and $T_{i}=\\sqrt{\\frac{2}{3}}(\\sqrt{1}+\\ldots+\\sqrt{i})$. Lemma 2.3. If an index $i$ is good, then $S_{i} \\geqslant T_{i}$. Proof. Induction on $i$. The base case $i=0$ is clear. Assume that the claim holds for good indices less than $i$ and prove it for a good index $i>0$. If $i-1$ is good, then by the inductive hypothesis we get $S_{i}=S_{i-1}+x_{i} \\geqslant T_{i-1}+\\sqrt{\\frac{2}{3} i}=T_{i}$. If $i-1$ is bad, then $i>1$, and $i-2$ is good by Lemma 2.1. Then using Lemma 2.2 and the inductive hypothesis we get $$ S_{i}=S_{i-2}+x_{i-1}+x_{i} \\geqslant T_{i-2}+\\sqrt{\\frac{2}{3}}(\\sqrt{i-1}+\\sqrt{i})=T_{i} $$ Since either $n$ or $n+1$ is good by Lemma 2.1, Lemma 2.3 yields in both cases $S_{n+1} \\geqslant T_{n}$, and it remains to apply Lemma 1.2 from Solution 1. Comment 1. Another way to get (*) is the integral bound $$ k^{3 \/ 2}-(k-1)^{3 \/ 2}=\\int_{k-1}^{k} \\frac{3}{2} \\sqrt{x} d x<\\frac{3}{2} \\sqrt{k} . $$ Comment 2. If $x_{i}=\\sqrt{2 \/ 3} \\cdot(\\sqrt{i}+1)$, the conditions of the problem hold. Indeed, the inequality to check is $$ (\\sqrt{i}+1)(\\sqrt{i+1}+1)-(\\sqrt{i-1}+1)^{2} \\geqslant 3 \/ 2 $$ that rewrites as $$ \\sqrt{i}+\\sqrt{i+1}-2 \\sqrt{i-1} \\geqslant(i+1 \/ 2)-\\sqrt{i(i+1)}=\\frac{1 \/ 4}{i+1 \/ 2+\\sqrt{i(i+1)}}, $$ which follows from $$ \\sqrt{i}-\\sqrt{i-1}=\\frac{1}{\\sqrt{i}+\\sqrt{i-1}}>\\frac{1}{2 i} $$ For these numbers we have $x_{0}+\\ldots+x_{n+1}=\\left(\\frac{2 n}{3}\\right)^{3 \/ 2}+O(n)$, thus the multiplicative constant $(2 \/ 3)^{3 \/ 2}$ in the problem statement is sharp.","tier":0} +{"problem_type":"Algebra","problem_label":"A8","problem":"Determine all functions $f: \\mathbb{R} \\rightarrow \\mathbb{R}$ that satisfy $$ (f(a)-f(b))(f(b)-f(c))(f(c)-f(a))=f\\left(a b^{2}+b c^{2}+c a^{2}\\right)-f\\left(a^{2} b+b^{2} c+c^{2} a\\right) $$ for all real numbers $a, b, c$.","solution":"It is straightforward to check that above functions satisfy the equation. Now let $f(x)$ satisfy the equation, which we denote $E(a, b, c)$. Then clearly $f(x)+C$ also does; therefore, we may suppose without loss of generality that $f(0)=0$.We start with proving Lemma. Either $f(x) \\equiv 0$ or $f$ is injective. Proof. Denote by $\\Theta \\subseteq \\mathbb{R}^{2}$ the set of points $(a, b)$ for which $f(a)=f(b)$. Let $\\Theta^{*}=\\{(x, y) \\in \\Theta$ : $x \\neq y\\}$. The idea is that if $(a, b) \\in \\Theta$, then by $E(a, b, x)$ we get $$ H_{a, b}(x):=\\left(a b^{2}+b x^{2}+x a^{2}, a^{2} b+b^{2} x+x^{2} a\\right) \\in \\Theta $$ for all real $x$. Reproducing this argument starting with $(a, b) \\in \\Theta^{*}$, we get more and more points in $\\Theta$. There are many ways to fill in the details, we give below only one of them. Assume that $(a, b) \\in \\Theta^{*}$. Note that $$ g_{-}(x):=\\left(a b^{2}+b x^{2}+x a^{2}\\right)-\\left(a^{2} b+b^{2} x+x^{2} a\\right)=(a-b)(b-x)(x-a) $$ and $$ g_{+}(x):=\\left(a b^{2}+b x^{2}+x a^{2}\\right)+\\left(a^{2} b+b^{2} x+x^{2} a\\right)=\\left(x^{2}+a b\\right)(a+b)+x\\left(a^{2}+b^{2}\\right) . $$ Hence, there exists $x$ for which both $g_{-}(x) \\neq 0$ and $g_{+}(x) \\neq 0$. This gives a point $(\\alpha, \\beta)=$ $H_{a, b}(x) \\in \\Theta^{*}$ for which $\\alpha \\neq-\\beta$. Now compare $E(\\alpha, 1,0)$ and $E(\\beta, 1,0)$. The left-hand side expressions coincide, on right-hand side we get $f(\\alpha)-f\\left(\\alpha^{2}\\right)=f(\\beta)-f\\left(\\beta^{2}\\right)$, respectively. Hence, $f\\left(\\alpha^{2}\\right)=f\\left(\\beta^{2}\\right)$ and we get a point $\\left(\\alpha_{1}, \\beta_{1}\\right):=\\left(\\alpha^{2}, \\beta^{2}\\right) \\in \\Theta^{*}$ with both coordinates $\\alpha_{1}, \\beta_{1}$ non-negative. Continuing squaring the coordinates, we get a point $(\\gamma, \\delta) \\in \\Theta^{*}$ for which $\\delta>5 \\gamma \\geqslant 0$. Our nearest goal is to get a point $(0, r) \\in \\Theta^{*}$. If $\\gamma=0$, this is already done. If $\\gamma>0$, denote by $x$ a real root of the quadratic equation $\\delta \\gamma^{2}+\\gamma x^{2}+x \\delta^{2}=0$, which exists since the discriminant $\\delta^{4}-4 \\delta \\gamma^{3}$ is positive. Also $x<0$ since this equation cannot have non-negative root. For the point $H_{\\delta, \\gamma}(x)=:(0, r) \\in \\Theta$ the first coordinate is 0 . The difference of coordinates equals $-r=(\\delta-\\gamma)(\\gamma-x)(x-\\delta)<0$, so $r \\neq 0$ as desired. Now, let $(0, r) \\in \\Theta^{*}$. We get $H_{0, r}(x)=\\left(r x^{2}, r^{2} x\\right) \\in \\Theta$. Thus $f\\left(r x^{2}\\right)=f\\left(r^{2} x\\right)$ for all $x \\in \\mathbb{R}$. Replacing $x$ to $-x$ we get $f\\left(r x^{2}\\right)=f\\left(r^{2} x\\right)=f\\left(-r^{2} x\\right)$, so $f$ is even: $(a,-a) \\in \\Theta$ for all $a$. Then $H_{a,-a}(x)=\\left(a^{3}-a x^{2}+x a^{2},-a^{3}+a^{2} x+x^{2} a\\right) \\in \\Theta$ for all real $a, x$. Putting $x=\\frac{1+\\sqrt{5}}{2} a$ we obtain $\\left(0,(1+\\sqrt{5}) a^{3}\\right) \\in \\Theta$ which means that $f(y)=f(0)=0$ for every real $y$. Hereafter we assume that $f$ is injective and $f(0)=0$. By $E(a, b, 0)$ we get $$ f(a) f(b)(f(a)-f(b))=f\\left(a^{2} b\\right)-f\\left(a b^{2}\\right) . $$ Let $\\kappa:=f(1)$ and note that $\\kappa=f(1) \\neq f(0)=0$ by injectivity. Putting $b=1$ in ( $\\Omega$ ) we get $$ \\kappa f(a)(f(a)-\\kappa)=f\\left(a^{2}\\right)-f(a) . $$ Subtracting the same equality for $-a$ we get $$ \\kappa(f(a)-f(-a))(f(a)+f(-a)-\\kappa)=f(-a)-f(a) . $$ Now, if $a \\neq 0$, by injectivity we get $f(a)-f(-a) \\neq 0$ and thus $$ f(a)+f(-a)=\\kappa-\\kappa^{-1}=: \\lambda . $$ It follows that $$ f(a)-f(b)=f(-b)-f(-a) $$ for all non-zero $a, b$. Replace non-zero numbers $a, b$ in ( $\\odot$ ) with $-a,-b$, respectively, and add the two equalities. Due to ( $\\boldsymbol{\\uparrow}$ ) we get $$ (f(a)-f(b))(f(a) f(b)-f(-a) f(-b))=0 $$ thus $f(a) f(b)=f(-a) f(-b)=(\\lambda-f(a))(\\lambda-f(b))$ for all non-zero $a \\neq b$. If $\\lambda \\neq 0$, this implies $f(a)+f(b)=\\lambda$ that contradicts injectivity when we vary $b$ with fixed $a$. Therefore, $\\lambda=0$ and $\\kappa= \\pm 1$. Thus $f$ is odd. Replacing $f$ with $-f$ if necessary (this preserves the original equation) we may suppose that $f(1)=1$. Now, (\\&) yields $f\\left(a^{2}\\right)=f^{2}(a)$. Summing relations $(\\bigcirc)$ for pairs $(a, b)$ and $(a,-b)$, we get $-2 f(a) f^{2}(b)=-2 f\\left(a b^{2}\\right)$, i.e. $f(a) f\\left(b^{2}\\right)=f\\left(a b^{2}\\right)$. Putting $b=\\sqrt{x}$ for each non-negative $x$ we get $f(a x)=f(a) f(x)$ for all real $a$ and non-negative $x$. Since $f$ is odd, this multiplicativity relation is true for all $a, x$. Also, from $f\\left(a^{2}\\right)=f^{2}(a)$ we see that $f(x) \\geqslant 0$ for $x \\geqslant 0$. Next, $f(x)>0$ for $x>0$ by injectivity. Assume that $f(x)$ for $x>0$ does not have the form $f(x)=x^{\\tau}$ for a constant $\\tau$. The known property of multiplicative functions yields that the graph of $f$ is dense on $(0, \\infty)^{2}$. In particular, we may find positive $b<1 \/ 10$ for which $f(b)>1$. Also, such $b$ can be found if $f(x)=x^{\\tau}$ for some $\\tau<0$. Then for all $x$ we have $x^{2}+x b^{2}+b \\geqslant 0$ and so $E(1, b, x)$ implies that $$ f\\left(b^{2}+b x^{2}+x\\right)=f\\left(x^{2}+x b^{2}+b\\right)+(f(b)-1)(f(x)-f(b))(f(x)-1) \\geqslant 0-\\left((f(b)-1)^{3} \/ 4\\right. $$ is bounded from below (the quadratic trinomial bound $(t-f(1))(t-f(b)) \\geqslant-(f(b)-1)^{2} \/ 4$ for $t=f(x)$ is used). Hence, $f$ is bounded from below on $\\left(b^{2}-\\frac{1}{4 b},+\\infty\\right)$, and since $f$ is odd it is bounded from above on $\\left(0, \\frac{1}{4 b}-b^{2}\\right)$. This is absurd if $f(x)=x^{\\tau}$ for $\\tau<0$, and contradicts to the above dense graph condition otherwise. Therefore, $f(x)=x^{\\tau}$ for $x>0$ and some constant $\\tau>0$. Dividing $E(a, b, c)$ by $(a-b)(b-$ $c)(c-a)=\\left(a b^{2}+b c^{2}+c a^{2}\\right)-\\left(a^{2} b+b^{2} c+c^{2} a\\right)$ and taking a limit when $a, b, c$ all go to 1 (the divided ratios tend to the corresponding derivatives, say, $\\frac{a^{\\tau}-b^{\\tau}}{a-b} \\rightarrow\\left(x^{\\tau}\\right)_{x=1}^{\\prime}=\\tau$ ), we get $\\tau^{3}=\\tau \\cdot 3^{\\tau-1}, \\tau^{2}=3^{\\tau-1}, F(\\tau):=3^{\\tau \/ 2-1 \/ 2}-\\tau=0$. Since function $F$ is strictly convex, it has at most two roots, and we get $\\tau \\in\\{1,3\\}$.","tier":0} +{"problem_type":"Combinatorics","problem_label":"C1","problem":"Let $S$ be an infinite set of positive integers, such that there exist four pairwise distinct $a, b, c, d \\in S$ with $\\operatorname{gcd}(a, b) \\neq \\operatorname{gcd}(c, d)$. Prove that there exist three pairwise distinct $x, y, z \\in S$ such that $\\operatorname{gcd}(x, y)=\\operatorname{gcd}(y, z) \\neq \\operatorname{gcd}(z, x)$.","solution":"There exists $\\alpha \\in S$ so that $\\{\\operatorname{gcd}(\\alpha, s) \\mid s \\in S, s \\neq \\alpha\\}$ contains at least two elements. Since $\\alpha$ has only finitely many divisors, there is a $d \\mid \\alpha$ such that the set $B=\\{\\beta \\in$ $S \\mid \\operatorname{gcd}(\\alpha, \\beta)=d\\}$ is infinite. Pick $\\gamma \\in S$ so that $\\operatorname{gcd}(\\alpha, \\gamma) \\neq d$. Pick $\\beta_{1}, \\beta_{2} \\in B$ so that $\\operatorname{gcd}\\left(\\beta_{1}, \\gamma\\right)=\\operatorname{gcd}\\left(\\beta_{2}, \\gamma\\right)=: d^{\\prime}$. If $d=d^{\\prime}$, then $\\operatorname{gcd}\\left(\\alpha, \\beta_{1}\\right)=\\operatorname{gcd}\\left(\\gamma, \\beta_{1}\\right) \\neq \\operatorname{gcd}(\\alpha, \\gamma)$. If $d \\neq d^{\\prime}$, then either $\\operatorname{gcd}\\left(\\alpha, \\beta_{1}\\right)=\\operatorname{gcd}\\left(\\alpha, \\beta_{2}\\right)=d$ and $\\operatorname{gcd}\\left(\\beta_{1}, \\beta_{2}\\right) \\neq d$ or $\\operatorname{gcd}\\left(\\gamma, \\beta_{1}\\right)=\\operatorname{gcd}\\left(\\gamma, \\beta_{2}\\right)=d^{\\prime}$ and $\\operatorname{gcd}\\left(\\beta_{1}, \\beta_{2}\\right) \\neq d^{\\prime}$. Comment. The situation can be modelled as a complete graph on the infinite vertex set $S$, where every edge $\\{s, t\\}$ is colored by $c(s, t):=\\operatorname{gcd}(s, t)$. For every vertex the incident edges carry only finitely many different colors, and by the problem statement at least two different colors show up on the edge set. The goal is to show that there exists a bi-colored triangle (a triangle, whose edges carry exactly two different colors). For the proof, consider a vertex $v$ whose incident edges carry at least two different colors. Let $X \\subset S$ be an infinite subset so that $c(v, x) \\equiv c_{1}$ for all $x \\in X$. Let $y \\in S$ be a vertex so that $c(v, y) \\neq c_{1}$. Let $x_{1}, x_{2} \\in X$ be two vertices with $c\\left(y, x_{1}\\right)=c\\left(y, x_{2}\\right)=c_{2}$. If $c_{1}=c_{2}$, then the triangle $v, y, x_{1}$ is bi-colored. If $c_{1} \\neq c_{2}$, then one of $v, x_{1}, x_{2}$ and $y, x_{1}, x_{2}$ is bi-colored.","tier":0} +{"problem_type":"Combinatorics","problem_label":"C2","problem":"Let $n \\geqslant 3$ be an integer. An integer $m \\geqslant n+1$ is called $n$-colourful if, given infinitely many marbles in each of $n$ colours $C_{1}, C_{2}, \\ldots, C_{n}$, it is possible to place $m$ of them around a circle so that in any group of $n+1$ consecutive marbles there is at least one marble of colour $C_{i}$ for each $i=1, \\ldots, n$. Prove that there are only finitely many positive integers which are not $n$-colourful. Find the largest among them.","solution":"First suppose that there are $n(n-1)-1$ marbles. Then for one of the colours, say blue, there are at most $n-2$ marbles, which partition the non-blue marbles into at most $n-2$ groups with at least $(n-1)^{2}>n(n-2)$ marbles in total. Thus one of these groups contains at least $n+1$ marbles and this group does not contain any blue marble. Now suppose that the total number of marbles is at least $n(n-1)$. Then we may write this total number as $n k+j$ with some $k \\geqslant n-1$ and with $0 \\leqslant j \\leqslant n-1$. We place around a circle $k-j$ copies of the colour sequence $[1,2,3, \\ldots, n]$ followed by $j$ copies of the colour sequence $[1,1,2,3, \\ldots, n]$.","tier":0} +{"problem_type":"Combinatorics","problem_label":"C3","problem":"A thimblerigger has 2021 thimbles numbered from 1 through 2021. The thimbles are arranged in a circle in arbitrary order. The thimblerigger performs a sequence of 2021 moves; in the $k^{\\text {th }}$ move, he swaps the positions of the two thimbles adjacent to thimble $k$. Prove that there exists a value of $k$ such that, in the $k^{\\text {th }}$ move, the thimblerigger swaps some thimbles $a$ and $b$ such that $ak \\geqslant 1$. There are $2 n+1$ students standing in a circle. Each student $S$ has $2 k$ neighbours - namely, the $k$ students closest to $S$ on the right, and the $k$ students closest to $S$ on the left. Suppose that $n+1$ of the students are girls, and the other $n$ are boys. Prove that there is a girl with at least $k$ girls among her neighbours.","solution":"We replace the girls by 1's, and the boys by 0 's, getting the numbers $a_{1}, a_{2}, \\ldots, a_{2 n+1}$ arranged in a circle. We extend this sequence periodically by letting $a_{2 n+1+k}=a_{k}$ for all $k \\in \\mathbb{Z}$. We get an infinite periodic sequence $$ \\ldots, a_{1}, a_{2}, \\ldots, a_{2 n+1}, a_{1}, a_{2}, \\ldots, a_{2 n+1}, \\ldots $$ Consider the numbers $b_{i}=a_{i}+a_{i-k-1}-1 \\in\\{-1,0,1\\}$ for all $i \\in \\mathbb{Z}$. We know that $$ b_{m+1}+b_{m+2}+\\cdots+b_{m+2 n+1}=1 \\quad(m \\in \\mathbb{Z}) $$ in particular, this yields that there exists some $i_{0}$ with $b_{i_{0}}=1$. Now we want to find an index $i$ such that $$ b_{i}=1 \\quad \\text { and } \\quad b_{i+1}+b_{i+2}+\\cdots+b_{i+k} \\geqslant 0 $$ This will imply that $a_{i}=1$ and $$ \\left(a_{i-k}+a_{i-k+1}+\\cdots+a_{i-1}\\right)+\\left(a_{i+1}+a_{i+2}+\\cdots+a_{i+k}\\right) \\geqslant k $$ as desired. Suppose, to the contrary, that for every index $i$ with $b_{i}=1$ the sum $b_{i+1}+b_{i+2}+\\cdots+b_{i+k}$ is negative. We start from some index $i_{0}$ with $b_{i_{0}}=1$ and construct a sequence $i_{0}, i_{1}, i_{2}, \\ldots$, where $i_{j}(j>0)$ is the smallest possible index such that $i_{j}>i_{j-1}+k$ and $b_{i_{j}}=1$. We can choose two numbers among $i_{0}, i_{1}, \\ldots, i_{2 n+1}$ which are congruent modulo $2 n+1$ (without loss of generality, we may assume that these numbers are $i_{0}$ and $i_{T}$ ). On the one hand, for every $j$ with $0 \\leqslant j \\leqslant T-1$ we have $$ S_{j}:=b_{i_{j}}+b_{i_{j}+1}+b_{i_{j}+2}+\\cdots+b_{i_{j+1}-1} \\leqslant b_{i_{j}}+b_{i_{j}+1}+b_{i_{j}+2}+\\cdots+b_{i_{j}+k} \\leqslant 0 $$ since $b_{i_{j}+k+1}, \\ldots, b_{i_{j+1}-1} \\leqslant 0$. On the other hand, since $\\left(i_{T}-i_{0}\\right) \\mid(2 n+1)$, from (1) we deduce $$ S_{0}+\\cdots+S_{T-1}=\\sum_{i=i_{0}}^{i_{T}-1} b_{i}=\\frac{i_{T}-i_{0}}{2 n+1}>0 $$ This contradiction finishes the solution. Comment 1. After the problem is reduced to finding an index $i$ satisfying (2), one can finish the solution by applying the (existence part of) following statement. Lemma (Raney). If $\\left\\langle x_{1}, x_{2}, \\ldots, x_{m}\\right\\rangle$ is any sequence of integers whose sum is +1 , exactly one of the cyclic shifts $\\left\\langle x_{1}, x_{2}, \\ldots, x_{m}\\right\\rangle,\\left\\langle x_{2}, \\ldots, x_{m}, x_{1}\\right\\rangle, \\ldots,\\left\\langle x_{m}, x_{1}, \\ldots, x_{m-1}\\right\\rangle$ has all of its partial sums positive. A (possibly wider known) version of this lemma, which also can be used in order to solve the problem, is the following Claim (Gas stations problem). Assume that there are several fuel stations located on a circular route which together contain just enough gas to make one trip around. Then one can make it all the way around, starting at the right station with an empty tank. Both Raney's theorem and the Gas stations problem admit many different (parallel) proofs. Their ideas can be disguised in direct solutions of the problem at hand (as it, in fact, happens in the above solution); such solutions may avoid the introduction of the $b_{i}$. Below, in Comment 2 we present a variant of such solution, while in Comment 3 we present an alternative proof of Raney's theorem. Comment 2. Here is a version of the solution which avoids the use of the $b_{i}$. Suppose the contrary. Introduce the numbers $a_{i}$ as above. Starting from any index $s_{0}$ with $a_{s_{0}}=1$, we construct a sequence $s_{0}, s_{1}, s_{2}, \\ldots$ by letting $s_{i}$ to be the smallest index larger than $s_{i-1}+k$ such that $a_{s_{i}}=1$, for $i=1,2, \\ldots$. Choose two indices among $s_{1}, \\ldots, s_{2 n+1}$ which are congruent modulo $2 n+1$; we assume those two are $s_{0}$ and $s_{T}$, with $s_{T}-s_{0}=t(2 n+1)$. Notice here that $s_{T+1}-s_{T}=s_{1}-s_{0}$. For every $i=0,1,2, \\ldots, T$, put $$ L_{i}=s_{i+1}-s_{i} \\quad \\text { and } \\quad S_{i}=a_{s_{i}}+a_{s_{i}+1}+\\cdots+a_{s_{i+1}-1} . $$ Now, by the indirect assumption, for every $i=1,2, \\ldots, T$, we have $$ a_{s_{i}-k}+a_{s_{i}-k+1}+\\cdots+a_{s_{i}+k} \\leqslant a_{s_{i}}+(k-1)=k . $$ Recall that $a_{j}=0$ for all $j$ with $s_{i}+k\\overparen{C B}$. In the right-angled triangle $B B_{1} B_{2}$, the point $L_{B}$ is a point on the hypothenuse such that $L_{B} B_{1}=L_{B} B$, so $L_{B}$ is the midpoint of $B_{1} B_{2}$. Since $D D_{1}$ is the internal angle bisector of $\\angle A D C$, we have $$ \\angle B D D_{1}=\\frac{\\angle B D A-\\angle C D B}{2}=\\frac{\\angle B C A-\\angle C A B}{2}=\\angle B B_{2} D_{1}, $$ so the points $B, B_{2}, D$, and $D_{1}$ lie on some circle $\\omega_{B}$. Similarly, $L_{D}$ is the midpoint of $D_{1} D_{2}$, and the points $D, D_{2}, B$, and $B_{1}$ lie on some circle $\\omega_{D}$. Now we have $$ \\angle B_{2} D B_{1}=\\angle B_{2} D B-\\angle B_{1} D B=\\angle B_{2} D_{1} B-\\angle B_{1} D_{2} B=\\angle D_{2} B D_{1} . $$ Therefore, the corresponding sides of the triangles $D B_{1} B_{2}$ and $B D_{1} D_{2}$ are parallel, and the triangles are homothetical (in $H$ ). So their corresponding medians $D L_{B}$ and $B L_{D}$ are also parallel. ![](https:\/\/cdn.mathpix.com\/cropped\/2024_11_18_7107063274cfa469597ag-53.jpg?height=535&width=1492&top_left_y=1914&top_left_x=285) Yet alternatively, after obtaining the circles $\\omega_{B}$ and $\\omega_{D}$, one may notice that $H$ lies on their radical axis $B D$, whence $H B_{1} \\cdot H D_{2}=H D_{1} \\cdot H B_{2}$, or $$ \\frac{H B_{1}}{H D_{1}}=\\frac{H B_{2}}{H D_{1}} . $$ Since $h\\left(D_{1}\\right)=B_{1}$, this yields $h\\left(D_{2}\\right)=B_{2}$ and hence $h\\left(L_{D}\\right)=L_{B}$. Comment 3. Since $h$ preserves the line $A C$ and maps $B \\mapsto D$ and $D_{1} \\mapsto B_{1}$, we have $h\\left(\\gamma_{B}\\right)=\\gamma_{D}$. Therefore, $h\\left(O_{B}\\right)=O_{D}$; in particular, $H$ also lies on $O_{B} O_{D}$.","tier":0} +{"problem_type":"Geometry","problem_label":"G5","problem":"Let $A B C D$ be a cyclic quadrilateral whose sides have pairwise different lengths. Let $O$ be the circumcentre of $A B C D$. The internal angle bisectors of $\\angle A B C$ and $\\angle A D C$ meet $A C$ at $B_{1}$ and $D_{1}$, respectively. Let $O_{B}$ be the centre of the circle which passes through $B$ and is tangent to $A C$ at $D_{1}$. Similarly, let $O_{D}$ be the centre of the circle which passes through $D$ and is tangent to $A C$ at $B_{1}$. Assume that $B D_{1} \\| D B_{1}$. Prove that $O$ lies on the line $O_{B} O_{D}$.","solution":"Let $B D_{1}$ and $T_{B} D_{1}$ meet $\\Omega$ again at $X_{B}$ and $Y_{B}$, respectively. Then $$ \\angle B D_{1} C=\\angle B T_{B} D_{1}=\\angle B T_{B} Y_{B}=\\angle B X_{B} Y_{B} $$ which shows that $X_{B} Y_{B} \\| A C$. Similarly, let $D B_{1}$ and $T_{D} B_{1}$ meet $\\Omega$ again at $X_{D}$ and $Y_{D}$, respectively; then $X_{D} Y_{D} \\| A C$. Let $M_{D}$ and $M_{B}$ be the midpoints of the arcs $A B C$ and $A D C$, respectively; then the points $D_{1}$ and $B_{1}$ lie on $D M_{D}$ and $B M_{B}$, respectively. Let $K$ be the midpoint of $A C$ (which lies on $M_{B} M_{D}$ ). Applying Pascal's theorem to $M_{D} D X_{D} X_{B} B M_{B}$, we obtain that the points $D_{1}=M_{D} D \\cap X_{B} B, B_{1}=D X_{D} \\cap B M_{B}$, and $X_{D} X_{B} \\cap M_{B} M_{D}$ are collinear, which means that $X_{B} X_{D}$ passes through $K$. Due to symmetry, the diagonals of an isosceles trapezoid $X_{B} Y_{B} X_{D} Y_{D}$ cross at $K$. ![](https:\/\/cdn.mathpix.com\/cropped\/2024_11_18_7107063274cfa469597ag-54.jpg?height=1143&width=1164&top_left_y=888&top_left_x=446) Let $b$ and $d$ denote the distances from the lines $X_{B} Y_{B}$ and $X_{D} Y_{D}$, respectively, to $A C$. Then we get $$ \\frac{X_{B} Y_{B}}{X_{D} Y_{D}}=\\frac{b}{d}=\\frac{D_{1} X_{B}}{B_{1} X_{D}} $$ where the second equation holds in view of $D_{1} X_{B} \\| B_{1} X_{D}$. Therefore, the triangles $D_{1} X_{B} Y_{B}$ and $B_{1} X_{D} Y_{D}$ are similar. The triangles $D_{1} T_{B} B$ and $B_{1} T_{D} D$ are similar to them and hence to each other. Since $B D_{1} \\| D B_{1}$, these triangles are also homothetical. This yields $B T_{B} \\| D T_{D}$, as desired. Comment 4. The original problem proposal asked to prove that the relations $B D_{1} \\| D B_{1}$ and $O \\in O_{1} O_{2}$ are equivalent. After obtaining $B D_{1} \\| D B_{1} \\Rightarrow O \\in O_{1} O_{2}$, the converse proof is either repeated backwards mutatis mutandis, or can be obtained by the usual procedure of varying some points in the construction. The Problem Selection Committee chose the current version, because it is less technical, yet keeps most of the ideas.","tier":0} +{"problem_type":"Geometry","problem_label":"G6","problem":"Determine all integers $n \\geqslant 3$ satisfying the following property: every convex $n$-gon whose sides all have length 1 contains an equilateral triangle of side length 1. (Every polygon is assumed to contain its boundary.)","solution":"First we show that for every even $n \\geqslant 4$ there exists a polygon violating the required statement. Consider a regular $k$-gon $A_{0} A_{1}, \\ldots A_{k-1}$ with side length 1 . Let $B_{1}, B_{2}, \\ldots, B_{n \/ 2-1}$ be the points symmetric to $A_{1}, A_{2}, \\ldots, A_{n \/ 2-1}$ with respect to the line $A_{0} A_{n \/ 2}$. Then $P=$ $A_{0} A_{1} A_{2} \\ldots A_{n \/ 2-1} A_{n \/ 2} B_{n \/ 2-1} B_{n \/ 2-2} \\ldots B_{2} B_{1}$ is a convex $n$-gon whose sides all have length 1 . If $k$ is big enough, $P$ is contained in a strip of width $1 \/ 2$, which clearly does not contain any equilateral triangle of side length 1. ![](https:\/\/cdn.mathpix.com\/cropped\/2024_11_18_7107063274cfa469597ag-56.jpg?height=175&width=1363&top_left_y=826&top_left_x=346) Assume now that $n=2 k+1$. As the case $k=1$ is trivially true, we assume $k \\geqslant 2$ henceforth. Consider a convex $(2 k+1)$-gon $P$ whose sides all have length 1 . Let $d$ be its longest diagonal. The endpoints of $d$ split the perimeter of $P$ into two polylines, one of which has length at least $k+1$. Hence we can label the vertices of $P$ so that $P=A_{0} A_{1} \\ldots A_{2 k}$ and $d=A_{0} A_{\\ell}$ with $\\ell \\geqslant k+1$. We will show that, in fact, the polygon $A_{0} A_{1} \\ldots A_{\\ell}$ contains an equilateral triangle of side length 1 . Suppose that $\\angle A_{\\ell} A_{0} A_{1} \\geqslant 60^{\\circ}$. Since $d$ is the longest diagonal, we have $A_{1} A_{\\ell} \\leqslant A_{0} A_{\\ell}$, so $\\angle A_{0} A_{1} A_{\\ell} \\geqslant \\angle A_{\\ell} A_{0} A_{1} \\geqslant 60^{\\circ}$. It follows that there exists a point $X$ inside the triangle $A_{0} A_{1} A_{\\ell}$ such that the triangle $A_{0} A_{1} X$ is equilateral, and this triangle is contained in $P$. Similar arguments apply if $\\angle A_{\\ell-1} A_{\\ell} A_{0} \\geqslant 60^{\\circ}$. ![](https:\/\/cdn.mathpix.com\/cropped\/2024_11_18_7107063274cfa469597ag-56.jpg?height=224&width=730&top_left_y=1524&top_left_x=663) From now on, assume $\\angle A_{\\ell} A_{0} A_{1}<60^{\\circ}$ and $A_{\\ell-1} A_{\\ell} A_{0}<60^{\\circ}$. Consider an isosceles trapezoid $A_{0} Y Z A_{\\ell}$ such that $A_{0} A_{\\ell} \\| Y Z, A_{0} Y=Z A_{\\ell}=1$, and $\\angle A_{\\ell} A_{0} Y=\\angle Z A_{\\ell} A_{0}=60^{\\circ}$. Suppose that $A_{0} A_{1} \\ldots A_{\\ell}$ is contained in $A_{0} Y Z A_{\\ell}$. Note that the perimeter of $A_{0} A_{1} \\ldots A_{\\ell}$ equals $\\ell+A_{0} A_{\\ell}$ and the perimeter of $A_{0} Y Z A_{\\ell}$ equals $2 A_{0} A_{\\ell}+1$. ![](https:\/\/cdn.mathpix.com\/cropped\/2024_11_18_7107063274cfa469597ag-56.jpg?height=209&width=1320&top_left_y=1957&top_left_x=371) Recall a well-known fact stating that if a convex polygon $P_{1}$ is contained in a convex polygon $P_{2}$, then the perimeter of $P_{1}$ is at most the perimeter of $P_{2}$. Hence we obtain $$ \\ell+A_{0} A_{\\ell} \\leqslant 2 A_{0} A_{\\ell}+1, \\quad \\text { i.e. } \\quad \\ell-1 \\leqslant A_{0} A_{\\ell} . $$ On the other hand, the triangle inequality yields $$ A_{0} A_{\\ell}1 \/ 2$ and $P A_{\\ell}>1 \/ 2$. Choose points $Q \\in A_{0} P, R \\in P A_{\\ell}$, and $S \\in P A_{m}$ such that $P Q=P R=1 \/ 2$ and $P S=\\sqrt{3} \/ 2$. Then $Q R S$ is an equilateral triangle of side length 1 contained in $A_{0} A_{1} \\ldots A_{\\ell}$. ![](https:\/\/cdn.mathpix.com\/cropped\/2024_11_18_7107063274cfa469597ag-57.jpg?height=335&width=1332&top_left_y=404&top_left_x=365) Comment. In fact, for every odd $n$ a stronger statement holds, which is formulated in terms defined in the solution above: there exists an equilateral triangle $A_{i} A_{i+1} B$ contained in $A_{0} A_{1} \\ldots A_{\\ell}$ for some $0 \\leqslant i<\\ell$. We sketch an indirect proof below. As above, we get $\\angle A_{\\ell} A_{0} A_{1}<60^{\\circ}$ and $A_{\\ell-1} A_{\\ell} A_{0}<60^{\\circ}$. Choose an index $m \\in[1, \\ell-1]$ maximising the distance between $A_{m}$ and $A_{0} A_{\\ell}$. Arguments from the above solution yield $1120^{\\circ}$ and $\\angle A_{m-1} A_{m} A_{\\ell}>\\angle A_{0} A_{m} A_{\\ell} \\geqslant 60^{\\circ}$. We construct an equilateral triangle $A_{m-1} A_{m} B$ as in the figure below. If $B$ lies in $A_{0} A_{m-1} A_{m} A_{\\ell}$, then we are done. Otherwise $B$ and $A_{m}$ lie on different sides of $A_{0} A_{\\ell}$. As before, let $P$ be the projection of $A_{m}$ to $A_{0} A_{\\ell}$. We will show that $$ A_{0} A_{1}+A_{1} A_{2}+\\ldots+A_{m-1} A_{m}A C$ so that $\\angle B A D=\\angle D A C$. A point $E$ is constructed on the segment $A C$ so that $\\angle A D E=\\angle D C B$. Similarly, a point $F$ is constructed on the segment $A B$ so that $\\angle A D F=\\angle D B C$. A point $X$ is chosen on the line $A C$ so that $C X=B X$. Let $O_{1}$ and $O_{2}$ be the circumcentres of the triangles $A D C$ and $D X E$. Prove that the lines $B C, E F$, and $O_{1} O_{2}$ are concurrent.","solution":"Let $T A$ intersect the circle $(A B C)$ again at $M$. Due to the circles ( $B C E F$ ) and $(A M C B)$, and using the above Claim, we get $T M \\cdot T A=T F \\cdot T E=T B \\cdot T C=T D^{2}$; in particular, the points $A, M, E$, and $F$ are concyclic. Under the inversion with centre $T$ and radius $T D$, the point $M$ maps to $A$, and $B$ maps to $C$, which implies that the circle $(M B D)$ maps to the circle $(A D C)$. Their common point $D$ lies on the circle of the inversion, so the second intersection point $K$ also lies on that circle, which means $T K=T D$. It follows that the point $T$ and the centres of the circles ( $K D E$ ) and $(A D C)$ lie on the perpendicular bisector of $K D$. Since the center of $(A D C)$ is $O_{1}$, it suffices to show now that the points $D, K, E$, and $X$ are concyclic (the center of the corresponding circle will be $O_{2}$ ). The lines $B M, D K$, and $A C$ are the pairwise radical axes of the circles $(A B C M),(A C D K)$ and $(B M D K)$, so they are concurrent at some point $P$. Also, $M$ lies on the circle $(A E F)$, thus $$ \\begin{aligned} \\Varangle(E X, X B) & =\\Varangle(C X, X B)=\\Varangle(X C, B C)+\\Varangle(B C, B X)=2 \\Varangle(A C, C B) \\\\ & =\\Varangle(A C, C B)+\\Varangle(E F, F A)=\\Varangle(A M, B M)+\\Varangle(E M, M A)=\\Varangle(E M, B M), \\end{aligned} $$ so the points $M, E, X$, and $B$ are concyclic. Therefore, $P E \\cdot P X=P M \\cdot P B=P K \\cdot P D$, so the points $E, K, D$, and $X$ are concyclic, as desired. ![](https:\/\/cdn.mathpix.com\/cropped\/2024_11_18_7107063274cfa469597ag-61.jpg?height=935&width=1466&top_left_y=195&top_left_x=295) Comment 1. We present here a different solution which uses similar ideas. Perform the inversion $\\iota$ with centre $T$ and radius $T D$. It swaps $B$ with $C$ and $E$ with $F$; the point $D$ maps to itself. Let $X^{\\prime}=\\iota(X)$. Observe that the points $E, F, X$, and $X^{\\prime}$ are concyclic, as well as the points $B, C, X$, and $X^{\\prime}$. Then $$ \\begin{aligned} & \\Varangle\\left(C X^{\\prime}, X^{\\prime} F\\right)=\\Varangle\\left(C X^{\\prime}, X^{\\prime} X\\right)+\\Varangle\\left(X^{\\prime} X, X^{\\prime} F\\right)=\\Varangle(C B, B X)+\\Varangle(E X, E F) \\\\ &=\\Varangle(X C, C B)+\\Varangle(E C, E F)=\\Varangle(C A, C B)+\\Varangle(B C, B F)=\\Varangle(C A, A F), \\end{aligned} $$ therefore the points $C, X^{\\prime}, A$, and $F$ are concyclic. Let $X^{\\prime} F$ intersect $A C$ at $P$, and let $K$ be the second common point of $D P$ and the circle $(A C D)$. Then $$ P K \\cdot P D=P A \\cdot P C=P X^{\\prime} \\cdot P F=P E \\cdot P X $$ hence, the points $K, X, D$, and $E$ lie on some circle $\\omega_{1}$, while the points $K, X^{\\prime}, D$, and $F$ lie on some circle $\\omega_{2}$. (These circles are distinct since $\\angle E X F+\\angle E D F<\\angle E A F+\\angle D C B+\\angle D B C<180^{\\circ}$ ). The inversion $\\iota$ swaps $\\omega_{1}$ with $\\omega_{2}$ and fixes their common point $D$, so it fixes their second common point $K$. Thus $T D=T K$ and the perpendicular bisector of $D K$ passes through $T$, as well as through the centres of the circles $(C D K A)$ and $(D E K X)$. ![](https:\/\/cdn.mathpix.com\/cropped\/2024_11_18_7107063274cfa469597ag-61.jpg?height=775&width=1154&top_left_y=1994&top_left_x=451)","tier":0} +{"problem_type":"Geometry","problem_label":"G7","problem":"A point $D$ is chosen inside an acute-angled triangle $A B C$ with $A B>A C$ so that $\\angle B A D=\\angle D A C$. A point $E$ is constructed on the segment $A C$ so that $\\angle A D E=\\angle D C B$. Similarly, a point $F$ is constructed on the segment $A B$ so that $\\angle A D F=\\angle D B C$. A point $X$ is chosen on the line $A C$ so that $C X=B X$. Let $O_{1}$ and $O_{2}$ be the circumcentres of the triangles $A D C$ and $D X E$. Prove that the lines $B C, E F$, and $O_{1} O_{2}$ are concurrent.","solution":"We use only the first part of the Common remarks, namely, the facts that the tuples $(C, D, Q, E)$ and $(B, C, E, F)$ are both concyclic. We also introduce the point $T=$ $B C \\cap E F$. Let the circle $(C D E)$ meet $B C$ again at $E_{1}$. Since $\\angle E_{1} C Q=\\angle D C E$, the $\\operatorname{arcs} D E$ and $Q E_{1}$ of the circle $(C D Q)$ are equal, so $D Q \\| E E_{1}$. Since $B F E C$ is cyclic, the line $A D$ forms equal angles with $B C$ and $E F$, hence so does $E E_{1}$. Therefore, the triangle $E E_{1} T$ is isosceles, $T E=T E_{1}$, and $T$ lies on the common perpendicular bisector of $E E_{1}$ and $D Q$. Let $U$ and $V$ be the centres of circles $(A D E)$ and $(C D Q E)$, respectively. Then $U O_{1}$ is the perpendicular bisector of $A D$. Moreover, the points $U, V$, and $O_{2}$ belong to the perpendicular bisector of $D E$. Since $U O_{1} \\| V T$, in order to show that $O_{1} O_{2}$ passes through $T$, it suffices to show that $$ \\frac{O_{2} U}{O_{2} V}=\\frac{O_{1} U}{T V} $$ Denote angles $A, B$, and $C$ of the triangle $A B C$ by $\\alpha, \\beta$, and $\\gamma$, respectively. Projecting onto $A C$ we obtain $$ \\frac{O_{2} U}{O_{2} V}=\\frac{(X E-A E) \/ 2}{(X E+E C) \/ 2}=\\frac{A X}{C X}=\\frac{A X}{B X}=\\frac{\\sin (\\gamma-\\beta)}{\\sin \\alpha} $$ The projection of $O_{1} U$ onto $A C$ is $(A C-A E) \/ 2=C E \/ 2$; the angle between $O_{1} U$ and $A C$ is $90^{\\circ}-\\alpha \/ 2$, so $$ \\frac{O_{1} U}{E C}=\\frac{1}{2 \\sin (\\alpha \/ 2)} $$ Next, we claim that $E, V, C$, and $T$ are concyclic. Indeed, the point $V$ lies on the perpendicular bisector of $C E$, as well as on the internal angle bisector of $\\angle C T F$. Therefore, $V$ coincides with the midpoint of the arc $C E$ of the circle (TCE). Now we have $\\angle E V C=2 \\angle E E_{1} C=180^{\\circ}-(\\gamma-\\beta)$ and $\\angle V E T=\\angle V E_{1} T=90^{\\circ}-\\angle E_{1} E C=$ $90^{\\circ}-\\alpha \/ 2$. Therefore, $$ \\frac{E C}{T V}=\\frac{\\sin \\angle E T C}{\\sin \\angle V E T}=\\frac{\\sin (\\gamma-\\beta)}{\\cos (\\alpha \/ 2)} . $$ ![](https:\/\/cdn.mathpix.com\/cropped\/2024_11_18_7107063274cfa469597ag-62.jpg?height=986&width=1512&top_left_y=1774&top_left_x=266) Recalling (2) and multiplying (3) and (4) we establish (1): $$ \\frac{O_{2} U}{O_{2} V}=\\frac{\\sin (\\gamma-\\beta)}{\\sin \\alpha}=\\frac{1}{2 \\sin (\\alpha \/ 2)} \\cdot \\frac{\\sin (\\gamma-\\beta)}{\\cos (\\alpha \/ 2)}=\\frac{O_{1} U}{E C} \\cdot \\frac{E C}{T V}=\\frac{O_{1} U}{T V} $$","tier":0} +{"problem_type":"Geometry","problem_label":"G7","problem":"A point $D$ is chosen inside an acute-angled triangle $A B C$ with $A B>A C$ so that $\\angle B A D=\\angle D A C$. A point $E$ is constructed on the segment $A C$ so that $\\angle A D E=\\angle D C B$. Similarly, a point $F$ is constructed on the segment $A B$ so that $\\angle A D F=\\angle D B C$. A point $X$ is chosen on the line $A C$ so that $C X=B X$. Let $O_{1}$ and $O_{2}$ be the circumcentres of the triangles $A D C$ and $D X E$. Prove that the lines $B C, E F$, and $O_{1} O_{2}$ are concurrent.","solution":"Notice that $\\angle A Q E=\\angle Q C B$ and $\\angle A Q F=\\angle Q B C$; so, if we replace the point $D$ with $Q$ in the problem set up, the points $E, F$, and $T$ remain the same. So, by the Claim, we have $T Q^{2}=T B \\cdot T C=T D^{2}$. Thus, there exists a circle $\\Gamma$ centred at $T$ and passing through $D$ and $Q$. We denote the second meeting point of the circles $\\Gamma$ and $(A D C)$ by $K$. Let the line $A C$ meet the circle ( $D E K$ ) again at $Y$; we intend to prove that $Y=X$. As in Solution 1, this will yield that the point $T$, as well as the centres $O_{1}$ and $O_{2}$, all lie on the perpendicular bisector of $D K$. Let $L=A D \\cap B C$. We perform an inversion centred at $C$; the images of the points will be denoted by primes, e.g., $A^{\\prime}$ is the image of $A$. We obtain the following configuration, constructed in a triangle $A^{\\prime} C L^{\\prime}$. The points $D^{\\prime}$ and $Q^{\\prime}$ are chosen on the circumcircle $\\Omega$ of $A^{\\prime} L^{\\prime} C$ such that $\\Varangle\\left(L^{\\prime} C, D^{\\prime} C\\right)=$ $\\Varangle\\left(Q^{\\prime} C, A^{\\prime} C\\right)$, which means that $A^{\\prime} L^{\\prime} \\| D^{\\prime} Q^{\\prime}$. The lines $D^{\\prime} Q^{\\prime}$ and $A^{\\prime} C$ meet at $E^{\\prime}$. A circle $\\Gamma^{\\prime}$ centred on $C L^{\\prime}$ passes through $D^{\\prime}$ and $Q^{\\prime}$. Notice here that $B^{\\prime}$ lies on the segment $C L^{\\prime}$, and that $\\angle A^{\\prime} B^{\\prime} C=\\angle B A C=2 \\angle L A C=2 \\angle A^{\\prime} L^{\\prime} C$, so that $B^{\\prime} L^{\\prime}=B^{\\prime} A^{\\prime}$, and $B^{\\prime}$ lies on the perpendicular bisector of $A^{\\prime} L^{\\prime}$ (which coincides with that of $D^{\\prime} Q^{\\prime}$ ). All this means that $B^{\\prime}$ is the centre of $\\Gamma^{\\prime}$. Finally, $K^{\\prime}$ is the second meeting point of $A^{\\prime} D^{\\prime}$ and $\\Gamma^{\\prime}$, and $Y^{\\prime}$ is the second meeting point of the circle $\\left(D^{\\prime} K^{\\prime} E^{\\prime}\\right)$ and the line $A^{\\prime} E^{\\prime}$, We have $\\Varangle\\left(Y^{\\prime} K^{\\prime}, K^{\\prime} A^{\\prime}\\right)=\\Varangle\\left(Y^{\\prime} E^{\\prime}, E^{\\prime} D^{\\prime}\\right)=$ $\\Varangle\\left(Y^{\\prime} A^{\\prime}, A^{\\prime} L^{\\prime}\\right)$, so $A^{\\prime} L^{\\prime}$ is tangent to the circumcircle $\\omega$ of the triangle $Y^{\\prime} A^{\\prime} K^{\\prime}$. Let $O$ and $O^{*}$ be the centres of $\\Omega$ and $\\omega$, respectively. Then $O^{*} A^{\\prime} \\perp A^{\\prime} L^{\\prime} \\perp B^{\\prime} O$. The projections of vectors $\\overrightarrow{O^{*} A^{\\prime}}$ and $\\overrightarrow{B^{\\prime} O}$ onto $K^{\\prime} D^{\\prime}$ are equal to $\\overrightarrow{K^{\\prime} A^{\\prime}} \/ 2=\\overrightarrow{K^{\\prime} D^{\\prime}} \/ 2-\\overrightarrow{A^{\\prime} D^{\\prime}} \/ 2$. So $\\overrightarrow{O^{*} A^{\\prime}}=\\overrightarrow{B^{\\prime} O}$, or equivalently $\\overrightarrow{A^{\\prime} O}=\\overrightarrow{O^{*} B^{\\prime}}$. Projecting this equality onto $A^{\\prime} C$, we see that the projection of $\\overrightarrow{O^{*} \\overrightarrow{B^{\\prime}}}$ equals $\\overrightarrow{A^{\\prime} C} \/ 2$. Since $O^{*}$ is projected to the midpoint of $A^{\\prime} Y^{\\prime}$, this yields that $B^{\\prime}$ is projected to the midpoint of $C Y^{\\prime}$, i.e., $B^{\\prime} Y^{\\prime}=B^{\\prime} C$ and $\\angle B^{\\prime} Y^{\\prime} C=\\angle B^{\\prime} C Y^{\\prime}$. In the original figure, this rewrites as $\\angle C B Y=\\angle B C Y$, so $Y$ lies on the perpendicular bisector of $B C$, as desired. ![](https:\/\/cdn.mathpix.com\/cropped\/2024_11_18_7107063274cfa469597ag-63.jpg?height=843&width=1306&top_left_y=1783&top_left_x=381) Comment 2. The point $K$ appears to be the same in Solutions 1 and 3 (and Comment 1 as well). One can also show that $K$ lies on the circle passing through $A, X$, and the midpoint of the arc $B A C$. Comment 3. There are different proofs of the facts from the Common remarks, namely, the cyclicity of $B, C, E$, and $F$, and the Claim. We present one such alternative proof here. We perform the composition $\\phi$ of a homothety with centre $A$ and the reflection in $A D$, which maps $E$ to $B$. Let $U=\\phi(D)$. Then $\\Varangle(B C, C D)=\\Varangle(A D, D E)=\\Varangle(B U, U D)$, so the points $B, U, C$, and $D$ are concyclic. Therefore, $\\Varangle(C U, U D)=\\Varangle(C B, B D)=\\Varangle(A D, D F)$, so $\\phi(F)=C$. Then the coefficient of the homothety is $A C \/ A F=A B \/ A E$, and thus points $C, E, F$, and $B$ are concyclic. Denote the centres of the circles $(E D F)$ and $(B U C D)$ by $O_{3}$ and $O_{4}$, respectively. Then $\\phi\\left(O_{3}\\right)=$ $O_{4}$, hence $\\Varangle\\left(O_{3} D, D A\\right)=-\\Varangle\\left(O_{4} U, U A\\right)=\\Varangle\\left(O_{4} D, D A\\right)$, whence the circle $(B D C)$ is tangent to the circle ( $E D F$ ). Now, the radical axes of circles $(D E F),(B D C)$ and $(B C E F)$ intersect at $T$, and the claim follows. ![](https:\/\/cdn.mathpix.com\/cropped\/2024_11_18_7107063274cfa469597ag-64.jpg?height=869&width=941&top_left_y=682&top_left_x=563) This suffices for Solution 1 to work. However, Solutions 2 and 3 need properties of point $Q$, established in Common remarks before Solution 1. Comment 4. In the original problem proposal, the point $X$ was hidden. Instead, a circle $\\gamma$ was constructed such that $D$ and $E$ lie on $\\gamma$, and its center is collinear with $O_{1}$ and $T$. The problem requested to prove that, in a fixed triangle $A B C$, independently from the choice of $D$ on the bisector of $\\angle B A C$, all circles $\\gamma$ pass through a fixed point.","tier":0} +{"problem_type":"Geometry","problem_label":"G8","problem":"Let $\\omega$ be the circumcircle of a triangle $A B C$, and let $\\Omega_{A}$ be its excircle which is tangent to the segment $B C$. Let $X$ and $Y$ be the intersection points of $\\omega$ and $\\Omega_{A}$. Let $P$ and $Q$ be the projections of $A$ onto the tangent lines to $\\Omega_{A}$ at $X$ and $Y$, respectively. The tangent line at $P$ to the circumcircle of the triangle $A P X$ intersects the tangent line at $Q$ to the circumcircle of the triangle $A Q Y$ at a point $R$. Prove that $A R \\perp B C$.","solution":"Let $D$ be the point of tangency of $B C$ and $\\Omega_{A}$. Let $D^{\\prime}$ be the point such that $D D^{\\prime}$ is a diameter of $\\Omega_{A}$. Let $R^{\\prime}$ be (the unique) point such that $A R^{\\prime} \\perp B C$ and $R^{\\prime} D^{\\prime} \\| B C$. We shall prove that $R^{\\prime}$ coincides with $R$. Let $P X$ intersect $A B$ and $D^{\\prime} R^{\\prime}$ at $S$ and $T$, respectively. Let $U$ be the ideal common point of the parallel lines $B C$ and $D^{\\prime} R^{\\prime}$. Note that the (degenerate) hexagon $A S X T U C$ is circumscribed around $\\Omega_{A}$, hence by the Brianchon theorem $A T, S U$, and $X C$ concur at a point which we denote by $V$. Then $V S \\| B C$. It follows that $\\Varangle(S V, V X)=\\Varangle(B C, C X)=$ $\\Varangle(B A, A X)$, hence $A X S V$ is cyclic. Therefore, $\\Varangle(P X, X A)=\\Varangle(S V, V A)=\\Varangle\\left(R^{\\prime} T, T A\\right)$. Since $\\angle A P T=\\angle A R^{\\prime} T=90^{\\circ}$, the quadrilateral $A P R^{\\prime} T$ is cyclic. Hence, $$ \\Varangle(X A, A P)=90^{\\circ}-\\Varangle(P X, X A)=90^{\\circ}-\\Varangle\\left(R^{\\prime} T, T A\\right)=\\Varangle\\left(T A, A R^{\\prime}\\right)=\\Varangle\\left(T P, P R^{\\prime}\\right) . $$ It follows that $P R^{\\prime}$ is tangent to the circle $(A P X)$. Analogous argument shows that $Q R^{\\prime}$ is tangent to the circle $(A Q Y)$. Therefore, $R=R^{\\prime}$ and $A R \\perp B C$. ![](https:\/\/cdn.mathpix.com\/cropped\/2024_11_18_7107063274cfa469597ag-65.jpg?height=995&width=1380&top_left_y=1333&top_left_x=349) Comment 1. After showing $\\Varangle(P X, X A)=\\Varangle\\left(R^{\\prime} T, T A\\right)$ one can finish the solution as follows. There exists a spiral similarity mapping the triangle $A T R^{\\prime}$ to the triangle $A X P$. So the triangles $A T X$ and $A R^{\\prime} P$ are similar and equioriented. Thus, $\\Varangle(T X, X A)=\\Varangle\\left(R^{\\prime} P, P A\\right)$, which implies that $P R^{\\prime}$ is tangent to the circle ( $A P X$ ).","tier":0} +{"problem_type":"Geometry","problem_label":"G8","problem":"Let $\\omega$ be the circumcircle of a triangle $A B C$, and let $\\Omega_{A}$ be its excircle which is tangent to the segment $B C$. Let $X$ and $Y$ be the intersection points of $\\omega$ and $\\Omega_{A}$. Let $P$ and $Q$ be the projections of $A$ onto the tangent lines to $\\Omega_{A}$ at $X$ and $Y$, respectively. The tangent line at $P$ to the circumcircle of the triangle $A P X$ intersects the tangent line at $Q$ to the circumcircle of the triangle $A Q Y$ at a point $R$. Prove that $A R \\perp B C$.","solution":"Let $J$ and $r$ be the center and the radius of $\\Omega_{A}$. Denote the diameter of $\\omega$ by $d$ and its center by $O$. By Euler's formula, $O J^{2}=(d \/ 2)^{2}+d r$, so the power of $J$ with respect to $\\omega$ equals $d r$. Let $J X$ intersect $\\omega$ again at $L$. Then $J L=d$. Let $L K$ be a diameter of $\\omega$ and let $M$ be the midpoint of $J K$. Since $J L=L K$, we have $\\angle L M K=90^{\\circ}$, so $M$ lies on $\\omega$. Let $R^{\\prime}$ be the point such that $R^{\\prime} P$ is tangent to the circle $(A P X)$ and $A R^{\\prime} \\perp B C$. Note that the line $A R^{\\prime}$ is symmetric to the line $A O$ with respect to $A J$. ![](https:\/\/cdn.mathpix.com\/cropped\/2024_11_18_7107063274cfa469597ag-66.jpg?height=690&width=1438&top_left_y=406&top_left_x=316) Lemma. Let $M$ be the midpoint of the side $J K$ in a triangle $A J K$. Let $X$ be a point on the circle $(A M K)$ such that $\\angle J X K=90^{\\circ}$. Then there exists a point $T$ on the line $K X$ such that the triangles $A K J$ and $A J T$ are similar and equioriented. Proof. Note that $M X=M K$. We construct a parallelogram $A J N K$. Let $T$ be a point on $K X$ such that $\\Varangle(N J, J A)=\\Varangle(K J, J T)$. Then $$ \\Varangle(J N, N A)=\\Varangle(K A, A M)=\\Varangle(K X, X M)=\\Varangle(M K, K X)=\\Varangle(J K, K T) . $$ So there exists a spiral similarity with center $J$ mapping the triangle $A J N$ to the triangle $T J K$. Therefore, the triangles $N J K$ and $A J T$ are similar and equioriented. It follows that the triangles $A K J$ and $A J T$ are similar and equioriented. ![](https:\/\/cdn.mathpix.com\/cropped\/2024_11_18_7107063274cfa469597ag-66.jpg?height=1155&width=1087&top_left_y=1607&top_left_x=493) Back to the problem, we construct a point $T$ as in the lemma. We perform the composition $\\phi$ of inversion with centre $A$ and radius $A J$ and reflection in $A J$. It is known that every triangle $A E F$ is similar and equioriented to $A \\phi(F) \\phi(E)$. So $\\phi(K)=T$ and $\\phi(T)=K$. Let $P^{*}=\\phi(P)$ and $R^{*}=\\phi\\left(R^{\\prime}\\right)$. Observe that $\\phi(T K)$ is a circle with diameter $A P^{*}$. Let $A A^{\\prime}$ be a diameter of $\\omega$. Then $P^{*} K \\perp A K \\perp A^{\\prime} K$, so $A^{\\prime}$ lies on $P^{*} K$. The triangles $A R^{\\prime} P$ and $A P^{*} R^{*}$ are similar and equioriented, hence $\\Varangle\\left(A A^{\\prime}, A^{\\prime} P^{*}\\right)=\\Varangle\\left(A A^{\\prime}, A^{\\prime} K\\right)=\\Varangle(A X, X P)=\\Varangle(A X, X P)=\\Varangle\\left(A P, P R^{\\prime}\\right)=\\Varangle\\left(A R^{*}, R^{*} P^{*}\\right)$, so $A, A^{\\prime}, R^{*}$, and $P^{*}$ are concyclic. Since $A^{\\prime}$ and $R^{*}$ lie on $A O$, we obtain $R^{*}=A^{\\prime}$. So $R^{\\prime}=\\phi\\left(A^{\\prime}\\right)$, and $\\phi\\left(A^{\\prime}\\right) P$ is tangent to the circle $(A P X)$. An identical argument shows that $\\phi\\left(A^{\\prime}\\right) Q$ is tangent to the circle $(A Q Y)$. Therefore, $R=$ $\\phi\\left(A^{\\prime}\\right)$ and $A R \\perp B C$. Comment 2. One of the main ideas of Solution 2 is to get rid of the excircle, along with points $B$ and $C$. After doing so we obtain the following fact, which is, essentially, proved in Solution 2. Let $\\omega$ be the circumcircle of a triangle $A K_{1} K_{2}$. Let $J$ be a point such that the midpoints of $J K_{1}$ and $J K_{2}$ lie on $\\omega$. Points $X$ and $Y$ are chosen on $\\omega$ so that $\\angle J X K_{1}=\\angle J Y K_{2}=90^{\\circ}$. Let $P$ and $Q$ be the projections of $A$ onto $X K_{1}$ and $Y K_{2}$, respectively. The tangent line at $P$ to the circumcircle of the triangle $A P X$ intersects the tangent line at $Q$ to the circumcircle of the triangle $A Q Y$ at a point $R$. Then the reflection of the line $A R$ in $A J$ passes through the centre $O$ of $\\omega$.","tier":0} +{"problem_type":"Number Theory","problem_label":"N1","problem":"Determine all integers $n \\geqslant 1$ for which there exists a pair of positive integers $(a, b)$ such that no cube of a prime divides $a^{2}+b+3$ and $$ \\frac{a b+3 b+8}{a^{2}+b+3}=n $$","solution":"As $b \\equiv-a^{2}-3\\left(\\bmod a^{2}+b+3\\right)$, the numerator of the given fraction satisfies $$ a b+3 b+8 \\equiv a\\left(-a^{2}-3\\right)+3\\left(-a^{2}-3\\right)+8 \\equiv-(a+1)^{3} \\quad\\left(\\bmod a^{2}+b+3\\right) $$ As $a^{2}+b+3$ is not divisible by $p^{3}$ for any prime $p$, if $a^{2}+b+3$ divides $(a+1)^{3}$ then it does also divide $(a+1)^{2}$. Since $$ 0<(a+1)^{2}<2\\left(a^{2}+b+3\\right) $$ we conclude $(a+1)^{2}=a^{2}+b+3$. This yields $b=2(a-1)$ and $n=2$. The choice $(a, b)=(2,2)$ with $a^{2}+b+3=9$ shows that $n=2$ indeed is a solution.","tier":0} +{"problem_type":"Number Theory","problem_label":"N2","problem":"Let $n \\geqslant 100$ be an integer. The numbers $n, n+1, \\ldots, 2 n$ are written on $n+1$ cards, one number per card. The cards are shuffled and divided into two piles. Prove that one of the piles contains two cards such that the sum of their numbers is a perfect square.","solution":"To solve the problem it suffices to find three squares and three cards with numbers $a, b, c$ on them such that pairwise sums $a+b, b+c, a+c$ are equal to the chosen squares. By choosing the three consecutive squares $(2 k-1)^{2},(2 k)^{2},(2 k+1)^{2}$ we arrive at the triple $$ (a, b, c)=\\left(2 k^{2}-4 k, \\quad 2 k^{2}+1, \\quad 2 k^{2}+4 k\\right) $$ We need a value for $k$ such that $$ n \\leqslant 2 k^{2}-4 k, \\quad \\text { and } \\quad 2 k^{2}+4 k \\leqslant 2 n $$ A concrete $k$ is suitable for all $n$ with $$ n \\in\\left[k^{2}+2 k, 2 k^{2}-4 k+1\\right]=: I_{k} $$ For $k \\geqslant 9$ the intervals $I_{k}$ and $I_{k+1}$ overlap because $$ (k+1)^{2}+2(k+1) \\leqslant 2 k^{2}-4 k+1 $$ Hence $I_{9} \\cup I_{10} \\cup \\ldots=[99, \\infty)$, which proves the statement for $n \\geqslant 99$. Comment 1. There exist approaches which only work for sufficiently large $n$. One possible approach is to consider three cards with numbers $70 k^{2}, 99 k^{2}, 126 k^{2}$ on them. Then their pairwise sums are perfect squares and so it suffices to find $k$ such that $70 k^{2} \\geqslant n$ and $126 k^{2} \\leqslant 2 n$ which exists for sufficiently large $n$. Another approach is to prove, arguing by contradiction, that $a$ and $a-2$ are in the same pile provided that $n$ is large enough and $a$ is sufficiently close to $n$. For that purpose, note that every pair of neighbouring numbers in the sequence $a, x^{2}-a, a+(2 x+1), x^{2}+2 x+3-a, a-2$ adds up to a perfect square for any $x$; so by choosing $x=\\lfloor\\sqrt{2 a}\\rfloor+1$ and assuming that $n$ is large enough we conclude that $a$ and $a-2$ are in the same pile for any $a \\in[n+2,3 n \/ 2]$. This gives a contradiction since it is easy to find two numbers from $[n+2,3 n \/ 2]$ of the same parity which sum to a square. It then remains to separately cover the cases of small $n$ which appears to be quite technical. Comment 2. An alternative formulation for this problem could ask for a proof of the statement for all $n>10^{6}$. An advantage of this formulation is that some solutions, e.g. those mentioned in Comment 1 need not contain a technical part which deals with the cases of small $n$. However, the original formulation seems to be better because the bound it gives for $n$ is almost sharp, see the next comment for details. Comment 3. The statement of the problem is false for $n=98$. As a counterexample, the first pile may contain the even numbers from 98 to 126 , the odd numbers from 129 to 161 , and the even numbers from 162 to 196.","tier":0} +{"problem_type":"Number Theory","problem_label":"N3","problem":"Find all positive integers $n$ with the following property: the $k$ positive divisors of $n$ have a permutation $\\left(d_{1}, d_{2}, \\ldots, d_{k}\\right)$ such that for every $i=1,2, \\ldots, k$, the number $d_{1}+\\cdots+d_{i}$ is a perfect square.","solution":"For $i=1,2, \\ldots, k$ let $d_{1}+\\ldots+d_{i}=s_{i}^{2}$, and define $s_{0}=0$ as well. Obviously $0=s_{0}2 i>d_{i}>d_{i-1}>\\ldots>d_{1}$, so $j \\leqslant i$ is not possible. The only possibility is $j=i+1$. Hence, $$ \\begin{gathered} s_{i+1}+i=d_{i+1}=s_{i+1}^{2}-s_{i}^{2}=s_{i+1}^{2}-i^{2} \\\\ s_{i+1}^{2}-s_{i+1}=i(i+1) \\end{gathered} $$ By solving this equation we get $s_{i+1}=i+1$ and $d_{i+1}=2 i+1$, that finishes the proof. Now we know that the positive divisors of the number $n$ are $1,3,5, \\ldots, n-2, n$. The greatest divisor is $d_{k}=2 k-1=n$ itself, so $n$ must be odd. The second greatest divisor is $d_{k-1}=n-2$; then $n-2$ divides $n=(n-2)+2$, so $n-2$ divides 2 . Therefore, $n$ must be 1 or 3 . The numbers $n=1$ and $n=3$ obviously satisfy the requirements: for $n=1$ we have $k=1$ and $d_{1}=1^{2}$; for $n=3$ we have $k=2, d_{1}=1^{2}$ and $d_{1}+d_{2}=1+3=2^{2}$. This page is intentionally left blank","tier":0} +{"problem_type":"Number Theory","problem_label":"N4","problem":"Alice is given a rational number $r>1$ and a line with two points $B \\neq R$, where point $R$ contains a red bead and point $B$ contains a blue bead. Alice plays a solitaire game by performing a sequence of moves. In every move, she chooses a (not necessarily positive) integer $k$, and a bead to move. If that bead is placed at point $X$, and the other bead is placed at $Y$, then Alice moves the chosen bead to point $X^{\\prime}$ with $\\overrightarrow{Y X^{\\prime}}=r^{k} \\overrightarrow{Y X}$. Alice's goal is to move the red bead to the point $B$. Find all rational numbers $r>1$ such that Alice can reach her goal in at most 2021 moves.","solution":"Denote the red and blue beads by $\\mathcal{R}$ and $\\mathcal{B}$, respectively. Introduce coordinates on the line and identify the points with their coordinates so that $R=0$ and $B=1$. Then, during the game, the coordinate of $\\mathcal{R}$ is always smaller than the coordinate of $\\mathcal{B}$. Moreover, the distance between the beads always has the form $r^{\\ell}$ with $\\ell \\in \\mathbb{Z}$, since it only multiplies by numbers of this form. Denote the value of the distance after the $m^{\\text {th }}$ move by $d_{m}=r^{\\alpha_{m}}$, $m=0,1,2, \\ldots$ (after the $0^{\\text {th }}$ move we have just the initial position, so $\\alpha_{0}=0$ ). If some bead is moved in two consecutive moves, then Alice could instead perform a single move (and change the distance from $d_{i}$ directly to $d_{i+2}$ ) which has the same effect as these two moves. So, if Alice can achieve her goal, then she may as well achieve it in fewer (or the same) number of moves by alternating the moves of $\\mathcal{B}$ and $\\mathcal{R}$. In the sequel, we assume that Alice alternates the moves, and that $\\mathcal{R}$ is shifted altogether $t$ times. If $\\mathcal{R}$ is shifted in the $m^{\\text {th }}$ move, then its coordinate increases by $d_{m}-d_{m+1}$. Therefore, the total increment of $\\mathcal{R}$ 's coordinate, which should be 1 , equals $$ \\begin{aligned} \\text { either } \\quad\\left(d_{0}-d_{1}\\right)+\\left(d_{2}-d_{3}\\right)+\\cdots+\\left(d_{2 t-2}-d_{2 t-1}\\right) & =1+\\sum_{i=1}^{t-1} r^{\\alpha_{2 i}}-\\sum_{i=1}^{t} r^{\\alpha_{2 i-1}}, \\\\ \\text { or } \\quad\\left(d_{1}-d_{2}\\right)+\\left(d_{3}-d_{4}\\right)+\\cdots+\\left(d_{2 t-1}-d_{2 t}\\right) & =\\sum_{i=1}^{t} r^{\\alpha_{2 i-1}}-\\sum_{i=1}^{t} r^{\\alpha_{2 i}}, \\end{aligned} $$ depending on whether $\\mathcal{R}$ or $\\mathcal{B}$ is shifted in the first move. Moreover, in the former case we should have $t \\leqslant 1011$, while in the latter one we need $t \\leqslant 1010$. So both cases reduce to an equation $$ \\sum_{i=1}^{n} r^{\\beta_{i}}=\\sum_{i=1}^{n-1} r^{\\gamma_{i}}, \\quad \\beta_{i}, \\gamma_{i} \\in \\mathbb{Z} $$ for some $n \\leqslant 1011$. Thus, if Alice can reach her goal, then this equation has a solution for $n=1011$ (we can add equal terms to both sums in order to increase $n$ ). Conversely, if (1) has a solution for $n=1011$, then Alice can compose a corresponding sequence of distances $d_{0}, d_{1}, d_{2}, \\ldots, d_{2021}$ and then realise it by a sequence of moves. So the problem reduces to the solvability of (1) for $n=1011$. Assume that, for some rational $r$, there is a solution of (1). Write $r$ in lowest terms as $r=a \/ b$. Substitute this into (1), multiply by the common denominator, and collect all terms on the left hand side to get $$ \\sum_{i=1}^{2 n-1}(-1)^{i} a^{\\mu_{i}} b^{N-\\mu_{i}}=0, \\quad \\mu_{i} \\in\\{0,1, \\ldots, N\\} $$ for some $N \\geqslant 0$. We assume that there exist indices $j_{-}$and $j_{+}$such that $\\mu_{j_{-}}=0$ and $\\mu_{j_{+}}=N$. Reducing (2) modulo $a-b$ (so that $a \\equiv b$ ), we get $$ 0=\\sum_{i=1}^{2 n-1}(-1)^{i} a^{\\mu_{i}} b^{N-\\mu_{i}} \\equiv \\sum_{i=1}^{2 n-1}(-1)^{i} b^{\\mu_{i}} b^{N-\\mu_{i}}=-b^{N} \\quad \\bmod (a-b) $$ Since $\\operatorname{gcd}(a-b, b)=1$, this is possible only if $a-b=1$. Reducing (2) modulo $a+b$ (so that $a \\equiv-b$ ), we get $$ 0=\\sum_{i=1}^{2 n-1}(-1)^{i} a^{\\mu_{i}} b^{N-\\mu_{i}} \\equiv \\sum_{i=1}^{2 n-1}(-1)^{i}(-1)^{\\mu_{i}} b^{\\mu_{i}} b^{N-\\mu_{i}}=S b^{N} \\quad \\bmod (a+b) $$ for some odd (thus nonzero) $S$ with $|S| \\leqslant 2 n-1$. Since $\\operatorname{gcd}(a+b, b)=1$, this is possible only if $a+b \\mid S$. So $a+b \\leqslant 2 n-1$, and hence $b=a-1 \\leqslant n-1=1010$. Thus we have shown that any sought $r$ has the form indicated in the answer. It remains to show that for any $b=1,2, \\ldots, 1010$ and $a=b+1$, Alice can reach the goal. For this purpose, in (1) we put $n=a, \\beta_{1}=\\beta_{2}=\\cdots=\\beta_{a}=0$, and $\\gamma_{1}=\\gamma_{2}=\\cdots=\\gamma_{b}=1$. Comment 1. Instead of reducing modulo $a+b$, one can reduce modulo $a$ and modulo $b$. The first reduction shows that the number of terms in (2) with $\\mu_{i}=0$ is divisible by $a$, while the second shows that the number of terms with $\\mu_{i}=N$ is divisible by $b$. Notice that, in fact, $N>0$, as otherwise (2) contains an alternating sum of an odd number of equal terms, which is nonzero. Therefore, all terms listed above have different indices, and there are at least $a+b$ of them. Comment 2. Another way to investigate the solutions of equation (1) is to consider the Laurent polynomial $$ L(x)=\\sum_{i=1}^{n} x^{\\beta_{i}}-\\sum_{i=1}^{n-1} x^{\\gamma_{i}} . $$ We can pick a sufficiently large integer $d$ so that $P(x)=x^{d} L(x)$ is a polynomial in $\\mathbb{Z}[x]$. Then $$ P(1)=1, $$ and $$ 1 \\leqslant|P(-1)| \\leqslant 2021 $$ If $r=p \/ q$ with integers $p>q \\geqslant 1$ is a rational number with the properties listed in the problem statement, then $P(p \/ q)=L(p \/ q)=0$. As $P(x)$ has integer coefficients, $$ (p-q x) \\mid P(x) $$ Plugging $x=1$ into (5) gives $(p-q) \\mid P(1)=1$, which implies $p=q+1$. Moreover, plugging $x=-1$ into (5) gives $(p+q) \\mid P(-1)$, which, along with (4), implies $p+q \\leqslant 2021$ and $q \\leqslant 1010$. Hence $x=(q+1) \/ q$ for some integer $q$ with $1 \\leqslant q \\leqslant 1010$.","tier":0} +{"problem_type":"Number Theory","problem_label":"N5","problem":"Prove that there are only finitely many quadruples $(a, b, c, n)$ of positive integers such that $$ n!=a^{n-1}+b^{n-1}+c^{n-1} . $$","solution":"For fixed $n$ there are clearly finitely many solutions; we will show that there is no solution with $n>100$. So, assume $n>100$. By the AM-GM inequality, $$ \\begin{aligned} n! & =2 n(n-1)(n-2)(n-3) \\cdot(3 \\cdot 4 \\cdots(n-4)) \\\\ & \\leqslant 2(n-1)^{4}\\left(\\frac{3+\\cdots+(n-4)}{n-6}\\right)^{n-6}=2(n-1)^{4}\\left(\\frac{n-1}{2}\\right)^{n-6}<\\left(\\frac{n-1}{2}\\right)^{n-1} \\end{aligned} $$ thus $a, b, c<(n-1) \/ 2$. For every prime $p$ and integer $m \\neq 0$, let $\\nu_{p}(m)$ denote the $p$-adic valuation of $m$; that is, the greatest non-negative integer $k$ for which $p^{k}$ divides $m$. Legendre's formula states that $$ \\nu_{p}(n!)=\\sum_{s=1}^{\\infty}\\left\\lfloor\\frac{n}{p^{s}}\\right\\rfloor $$ and a well-know corollary of this formula is that $$ \\nu_{p}(n!)<\\sum_{s=1}^{\\infty} \\frac{n}{p^{s}}=\\frac{n}{p-1} $$ If $n$ is odd then $a^{n-1}, b^{n-1}, c^{n-1}$ are squares, and by considering them modulo 4 we conclude that $a, b$ and $c$ must be even. Hence, $2^{n-1} \\mid n$ ! but that is impossible for odd $n$ because $\\nu_{2}(n!)=\\nu_{2}((n-1)!)a+b$. On the other hand, $p \\mid c$ implies that $p100$. Comment 1. The original version of the problem asked to find all solutions to the equation. The solution to that version is not much different but is more technical. Comment 2. To find all solutions we can replace the bound $a, b, c<(n-1) \/ 2$ for all $n$ with a weaker bound $a, b, c \\leqslant n \/ 2$ only for even $n$, which is a trivial application of AM-GM to the tuple $(2,3, \\ldots, n)$. Then we may use the same argument for odd $n$ (it works for $n \\geqslant 5$ and does not require any bound on $a, b, c)$, and for even $n$ the same solution works for $n \\geqslant 6$ unless we have $a+b=n-1$ and $2 \\nu_{p}(n-1)=\\nu_{p}(n!)$. This is only possible for $p=3$ and $n=10$ in which case we can consider the original equation modulo 7 to deduce that $7 \\mid a b c$ which contradicts the fact that $7^{9}>10$ !. Looking at $n \\leqslant 4$ we find four solutions, namely, $$ (a, b, c, n)=(1,1,2,3),(1,2,1,3),(2,1,1,3),(2,2,2,4) $$ Comment 3. For sufficiently large $n$, the inequality $a, b, c<(n-1) \/ 2$ also follows from Stirling's formula.","tier":0} +{"problem_type":"Number Theory","problem_label":"N6","problem":"Determine all integers $n \\geqslant 2$ with the following property: every $n$ pairwise distinct integers whose sum is not divisible by $n$ can be arranged in some order $a_{1}, a_{2}, \\ldots, a_{n}$ so that $n$ divides $1 \\cdot a_{1}+2 \\cdot a_{2}+\\cdots+n \\cdot a_{n}$.","solution":"If $n=2^{k} a$, where $a \\geqslant 3$ is odd and $k$ is a positive integer, we can consider a set containing the number $2^{k}+1$ and $n-1$ numbers congruent to 1 modulo $n$. The sum of these numbers is congruent to $2^{k}$ modulo $n$ and therefore is not divisible by $n$; for any permutation $\\left(a_{1}, a_{2}, \\ldots, a_{n}\\right)$ of these numbers $$ 1 \\cdot a_{1}+2 \\cdot a_{2}+\\cdots+n \\cdot a_{n} \\equiv 1+\\cdots+n \\equiv 2^{k-1} a\\left(2^{k} a+1\\right) \\not \\equiv 0 \\quad\\left(\\bmod 2^{k}\\right) $$ and $a$ fortiori $1 \\cdot a_{1}+2 \\cdot a_{2}+\\cdots+n \\cdot a_{n}$ is not divisible by $n$. From now on, we suppose that $n$ is either odd or a power of 2 . Let $S$ be the given set of integers, and $s$ be the sum of elements of $S$. Lemma 1. If there is a permutation $\\left(a_{i}\\right)$ of $S$ such that $(n, s)$ divides $\\sum_{i=1}^{n} i a_{i}$, then there is a permutation $\\left(b_{i}\\right)$ of $S$ such that $n$ divides $\\sum_{i=1}^{n} i b_{i}$. Proof. Let $r=\\sum_{i=1}^{n} i a_{i}$. Consider the permutation $\\left(b_{i}\\right)$ defined by $b_{i}=a_{i+x}$, where $a_{j+n}=a_{j}$. For this permutation, we have $$ \\sum_{i=1}^{n} i b_{i}=\\sum_{i=1}^{n} i a_{i+x} \\equiv \\sum_{i=1}^{n}(i-x) a_{i} \\equiv r-s x \\quad(\\bmod n) $$ Since $(n, s)$ divides $r$, the congruence $r-s x \\equiv 0(\\bmod n)$ admits a solution. Lemma 2. Every set $T$ of $k m$ integers, $m>1$, can be partitioned into $m$ sets of $k$ integers so that in every set either the sum of elements is not divisible by $k$ or all the elements leave the same remainder upon division by $k$. Proof. The base case, $m=2$. If $T$ contains $k$ elements leaving the same remainder upon division by $k$, we form one subset $A$ of these elements; the remaining elements form a subset $B$. If $k$ does not divide the sum of all elements of $B$, we are done. Otherwise it is enough to exchange any element of $A$ with any element of $B$ not congruent to it modulo $k$, thus making sums of both $A$ and $B$ not divisible by $k$. This cannot be done only when all the elements of $T$ are congruent modulo $k$; in this case any partition will do. If no $k$ elements of $T$ have the same residue modulo $k$, there are three elements $a, b, c \\in T$ leaving pairwise distinct remainders upon division by $k$. Let $t$ be the sum of elements of $T$. It suffices to find $A \\subset T$ such that $|A|=k$ and $\\sum_{x \\in A} x \\not \\equiv 0, t(\\bmod k)$ : then neither the sum of elements of $A$ nor the sum of elements of $B=T \\backslash A$ is divisible by $k$. Consider $U^{\\prime} \\subset T \\backslash\\{a, b, c\\}$ with $\\left|U^{\\prime}\\right|=k-1$. The sums of elements of three sets $U^{\\prime} \\cup\\{a\\}, U^{\\prime} \\cup\\{b\\}, U^{\\prime} \\cup\\{c\\}$ leave three different remainders upon division by $k$, and at least one of them is not congruent either to 0 or to $t$. Now let $m>2$. If $T$ contains $k$ elements leaving the same remainder upon division by $k$, we form one subset $A$ of these elements and apply the inductive hypothesis to the remaining $k(m-1)$ elements. Otherwise, we choose any $U \\subset T,|U|=k-1$. Since all the remaining elements cannot be congruent modulo $k$, there is $a \\in T \\backslash U$ such that $a \\not \\equiv-\\sum_{x \\in U} x(\\bmod k)$. Now we can take $A=U \\cup\\{a\\}$ and apply the inductive hypothesis to $T \\backslash A$. Now we are ready to prove the statement of the problem for all odd $n$ and $n=2^{k}$. The proof is by induction. If $n$ is prime, the statement follows immediately from Lemma 1 , since in this case $(n, s)=1$. Turning to the general case, we can find prime $p$ and an integer $t$ such that $p^{t} \\mid n$ and $p^{t} \\nmid s$. By Lemma 2, we can partition $S$ into $p$ sets of $\\frac{n}{p}=k$ elements so that in every set either the sum of numbers is not divisible by $k$ or all numbers have the same residue modulo $k$. For sets in the first category, by the inductive hypothesis there is a permutation $\\left(a_{i}\\right)$ such that $k \\mid \\sum_{i=1}^{k} i a_{i}$. If $n$ (and therefore $k$ ) is odd, then for each permutation $\\left(b_{i}\\right)$ of a set in the second category we have $$ \\sum_{i=1}^{k} i b_{i} \\equiv b_{1} \\frac{k(k+1)}{2} \\equiv 0 \\quad(\\bmod k) $$ By combining such permutation for all sets of the partition, we get a permutation ( $c_{i}$ ) of $S$ such that $k \\mid \\sum_{i=1}^{n} i c_{i}$. Since this sum is divisible by $k$, and $k$ is divisible by $(n, s)$, we are done by Lemma 1 . If $n=2^{s}$, we have $p=2$ and $k=2^{s-1}$. Then for each of the subsets there is a permutation $\\left(a_{1}, \\ldots, a_{k}\\right)$ such that $\\sum_{i=1}^{k} i a_{i}$ is divisible by $2^{s-2}=\\frac{k}{2}$ : if the subset belongs to the first category, the expression is divisible even by $k$, and if it belongs to the second one, $$ \\sum_{i=1}^{k} i a_{i} \\equiv a_{1} \\frac{k(k+1)}{2} \\equiv 0\\left(\\bmod \\frac{k}{2}\\right) $$ Now the numbers of each permutation should be multiplied by all the odd or all the even numbers not exceeding $n$ in increasing order so that the resulting sums are divisible by $k$ : $$ \\sum_{i=1}^{k}(2 i-1) a_{i} \\equiv \\sum_{i=1}^{k} 2 i a_{i} \\equiv 2 \\sum_{i=1}^{k} i a_{i} \\equiv 0 \\quad(\\bmod k) $$ Combining these two sums, we again get a permutation $\\left(c_{i}\\right)$ of $S$ such that $k \\mid \\sum_{i=1}^{n} i c_{i}$, and finish the case by applying Lemma 1. Comment. We cannot dispense with the condition that $n$ does not divide the sum of all elements. Indeed, for each $n>1$ and the set consisting of $1,-1$, and $n-2$ elements divisible by $n$ the required permutation does not exist.","tier":0} +{"problem_type":"Number Theory","problem_label":"N7","problem":"Let $a_{1}, a_{2}, a_{3}, \\ldots$ be an infinite sequence of positive integers such that $a_{n+2 m}$ divides $a_{n}+a_{n+m}$ for all positive integers $n$ and $m$. Prove that this sequence is eventually periodic, i.e. there exist positive integers $N$ and $d$ such that $a_{n}=a_{n+d}$ for all $n>N$.","solution":"We will make repeated use of the following simple observation: Lemma 1. If a positive integer $d$ divides $a_{n}$ and $a_{n-m}$ for some $m$ and $n>2 m$, it also divides $a_{n-2 m}$. If $d$ divides $a_{n}$ and $a_{n-2 m}$, it also divides $a_{n-m}$. Proof. Both parts are obvious since $a_{n}$ divides $a_{n-2 m}+a_{n-m}$. Claim. The sequence $\\left(a_{n}\\right)$ is bounded. Proof. Suppose the contrary. Then there exist infinitely many indices $n$ such that $a_{n}$ is greater than each of the previous terms $a_{1}, a_{2}, \\ldots, a_{n-1}$. Let $a_{n}=k$ be such a term, $n>10$. For each $s<\\frac{n}{2}$ the number $a_{n}=k$ divides $a_{n-s}+a_{n-2 s}<2 k$, therefore $$ a_{n-s}+a_{n-2 s}=k $$ In particular, $$ a_{n}=a_{n-1}+a_{n-2}=a_{n-2}+a_{n-4}=a_{n-4}+a_{n-8} $$ that is, $a_{n-1}=a_{n-4}$ and $a_{n-2}=a_{n-8}$. It follows from Lemma 1 that $a_{n-1}$ divides $a_{n-1-3 s}$ for $3 sN$, then $a_{j}=t$ for infinitely many $j$. Clearly the sequence $\\left(a_{n+N}\\right)_{n>0}$ satisfies the divisibility condition, and it is enough to prove that this sequence is eventually periodic. Thus truncating the sequence if necessary, we can assume that each number appears infinitely many times in the sequence. Let $k$ be the maximum number appearing in the sequence. Lemma 2. If a positive integer $d$ divides $a_{n}$ for some $n$, then the numbers $i$ such that $d$ divides $a_{i}$ form an arithmetical progression with an odd difference. Proof. Let $i_{1}\\frac{k}{2}$, it follows from Lemma 1 that $a_{n-1}$ divides $a_{n-1-3 s}$ when $3 s\\frac{k}{2}$, all the terms $a_{n-2-6 s}$ with $6 s1$. We can choose $s$ so that $a_{3 s+3}=k$. Therefore $T$, which we already know to be odd and divisible by 3 , is greater than 3 , that is, at least 9 . Then $a_{3 s-3} \\neq k$, and the only other possibility is $a_{3 s-3}=k \/ 3$. However, $a_{3 s+3}=k$ must divide $a_{3 s}+a_{3 s-3}=2 k \/ 3$, which is impossible. We have proved then that $a_{3 s}=k$ for all $s>1$, which is the case (ii).","tier":0} +{"problem_type":"Number Theory","problem_label":"N8","problem":"For a polynomial $P(x)$ with integer coefficients let $P^{1}(x)=P(x)$ and $P^{k+1}(x)=$ $P\\left(P^{k}(x)\\right)$ for $k \\geqslant 1$. Find all positive integers $n$ for which there exists a polynomial $P(x)$ with integer coefficients such that for every integer $m \\geqslant 1$, the numbers $P^{m}(1), \\ldots, P^{m}(n)$ leave exactly $\\left\\lceil n \/ 2^{m}\\right\\rceil$ distinct remainders when divided by $n$.","solution":"Denote the set of residues modulo $\\ell$ by $\\mathbb{Z}_{\\ell}$. Observe that $P$ can be regarded as a function $\\mathbb{Z}_{\\ell} \\rightarrow \\mathbb{Z}_{\\ell}$ for any positive integer $\\ell$. Denote the cardinality of the set $P^{m}\\left(\\mathbb{Z}_{\\ell}\\right)$ by $f_{m, \\ell}$. Note that $f_{m, n}=\\left\\lceil n \/ 2^{m}\\right\\rceil$ for all $m \\geqslant 1$ if and only if $f_{m+1, n}=\\left\\lceil f_{m, n} \/ 2\\right\\rceil$ for all $m \\geqslant 0$. Part 1. The required polynomial exists when $n$ is a power of 2 or a prime. If $n$ is a power of 2 , set $P(x)=2 x$. If $n=p$ is an odd prime, every function $f: \\mathbb{Z}_{p} \\rightarrow \\mathbb{Z}_{p}$ coincides with some polynomial with integer coefficients. So we can pick the function that sends $x \\in\\{0,1, \\ldots, p-1\\}$ to $\\lfloor x \/ 2\\rfloor$. Part 2. The required polynomial does not exist when $n$ is not a prime power. Let $n=a b$ where $a, b>1$ and $\\operatorname{gcd}(a, b)=1$. Note that, since $\\operatorname{gcd}(a, b)=1$, $$ f_{m, a b}=f_{m, a} f_{m, b} $$ by the Chinese remainder theorem. Also, note that, if $f_{m, \\ell}=f_{m+1, \\ell}$, then $P$ permutes the image of $P^{m}$ on $\\mathbb{Z}_{\\ell}$, and therefore $f_{s, \\ell}=f_{m, \\ell}$ for all $s>m$. So, as $f_{m, a b}=1$ for sufficiently large $m$, we have for each $m$ $$ f_{m, a}>f_{m+1, a} \\quad \\text { or } \\quad f_{m, a}=1, \\quad f_{m, b}>f_{m+1, b} \\quad \\text { or } \\quad f_{m, b}=1 . $$ Choose the smallest $m$ such that $f_{m+1, a}=1$ or $f_{m+1, b}=1$. Without loss of generality assume that $f_{m+1, a}=1$. Then $f_{m+1, a b}=f_{m+1, b}1$. Let us choose the smallest $k$ for which this is so. To each residue in $P\\left(S_{r}\\right)$ we assign its residue modulo $p^{k-1}$; denote the resulting set by $\\bar{P}(S, r)$. We have $|\\bar{P}(S, r)|=p^{k-2}$ by virtue of minimality of $k$. Then $\\left|P\\left(S_{r}\\right)\\right|5 \\gamma \\geqslant 0$. Our nearest goal is to get a point $(0, r) \\in \\Theta^{*}$. If $\\gamma=0$, this is already done. If $\\gamma>0$, denote by $x$ a real root of the quadratic equation $\\delta \\gamma^{2}+\\gamma x^{2}+x \\delta^{2}=0$, which exists since the discriminant $\\delta^{4}-4 \\delta \\gamma^{3}$ is positive. Also $x<0$ since this equation cannot have non-negative root. For the point $H_{\\delta, \\gamma}(x)=:(0, r) \\in \\Theta$ the first coordinate is 0 . The difference of coordinates equals $-r=(\\delta-\\gamma)(\\gamma-x)(x-\\delta)<0$, so $r \\neq 0$ as desired. Now, let $(0, r) \\in \\Theta^{*}$. We get $H_{0, r}(x)=\\left(r x^{2}, r^{2} x\\right) \\in \\Theta$. Thus $f\\left(r x^{2}\\right)=f\\left(r^{2} x\\right)$ for all $x \\in \\mathbb{R}$. Replacing $x$ to $-x$ we get $f\\left(r x^{2}\\right)=f\\left(r^{2} x\\right)=f\\left(-r^{2} x\\right)$, so $f$ is even: $(a,-a) \\in \\Theta$ for all $a$. Then $H_{a,-a}(x)=\\left(a^{3}-a x^{2}+x a^{2},-a^{3}+a^{2} x+x^{2} a\\right) \\in \\Theta$ for all real $a, x$. Putting $x=\\frac{1+\\sqrt{5}}{2} a$ we obtain $\\left(0,(1+\\sqrt{5}) a^{3}\\right) \\in \\Theta$ which means that $f(y)=f(0)=0$ for every real $y$. Hereafter we assume that $f$ is injective and $f(0)=0$. By $E(a, b, 0)$ we get $$ f(a) f(b)(f(a)-f(b))=f\\left(a^{2} b\\right)-f\\left(a b^{2}\\right) . $$ Let $\\kappa:=f(1)$ and note that $\\kappa=f(1) \\neq f(0)=0$ by injectivity. Putting $b=1$ in ( $\\Omega$ ) we get $$ \\kappa f(a)(f(a)-\\kappa)=f\\left(a^{2}\\right)-f(a) . $$ Subtracting the same equality for $-a$ we get $$ \\kappa(f(a)-f(-a))(f(a)+f(-a)-\\kappa)=f(-a)-f(a) . $$ Now, if $a \\neq 0$, by injectivity we get $f(a)-f(-a) \\neq 0$ and thus $$ f(a)+f(-a)=\\kappa-\\kappa^{-1}=: \\lambda . $$ It follows that $$ f(a)-f(b)=f(-b)-f(-a) $$ for all non-zero $a, b$. Replace non-zero numbers $a, b$ in ( $\\odot$ ) with $-a,-b$, respectively, and add the two equalities. Due to ( $\\boldsymbol{\\uparrow}$ ) we get $$ (f(a)-f(b))(f(a) f(b)-f(-a) f(-b))=0 $$ thus $f(a) f(b)=f(-a) f(-b)=(\\lambda-f(a))(\\lambda-f(b))$ for all non-zero $a \\neq b$. If $\\lambda \\neq 0$, this implies $f(a)+f(b)=\\lambda$ that contradicts injectivity when we vary $b$ with fixed $a$. Therefore, $\\lambda=0$ and $\\kappa= \\pm 1$. Thus $f$ is odd. Replacing $f$ with $-f$ if necessary (this preserves the original equation) we may suppose that $f(1)=1$. Now, (\\&) yields $f\\left(a^{2}\\right)=f^{2}(a)$. Summing relations $(\\bigcirc)$ for pairs $(a, b)$ and $(a,-b)$, we get $-2 f(a) f^{2}(b)=-2 f\\left(a b^{2}\\right)$, i.e. $f(a) f\\left(b^{2}\\right)=f\\left(a b^{2}\\right)$. Putting $b=\\sqrt{x}$ for each non-negative $x$ we get $f(a x)=f(a) f(x)$ for all real $a$ and non-negative $x$. Since $f$ is odd, this multiplicativity relation is true for all $a, x$. Also, from $f\\left(a^{2}\\right)=f^{2}(a)$ we see that $f(x) \\geqslant 0$ for $x \\geqslant 0$. Next, $f(x)>0$ for $x>0$ by injectivity. Assume that $f(x)$ for $x>0$ does not have the form $f(x)=x^{\\tau}$ for a constant $\\tau$. The known property of multiplicative functions yields that the graph of $f$ is dense on $(0, \\infty)^{2}$. In particular, we may find positive $b<1 \/ 10$ for which $f(b)>1$. Also, such $b$ can be found if $f(x)=x^{\\tau}$ for some $\\tau<0$. Then for all $x$ we have $x^{2}+x b^{2}+b \\geqslant 0$ and so $E(1, b, x)$ implies that $$ f\\left(b^{2}+b x^{2}+x\\right)=f\\left(x^{2}+x b^{2}+b\\right)+(f(b)-1)(f(x)-f(b))(f(x)-1) \\geqslant 0-\\left((f(b)-1)^{3} \/ 4\\right. $$ is bounded from below (the quadratic trinomial bound $(t-f(1))(t-f(b)) \\geqslant-(f(b)-1)^{2} \/ 4$ for $t=f(x)$ is used). Hence, $f$ is bounded from below on $\\left(b^{2}-\\frac{1}{4 b},+\\infty\\right)$, and since $f$ is odd it is bounded from above on $\\left(0, \\frac{1}{4 b}-b^{2}\\right)$. This is absurd if $f(x)=x^{\\tau}$ for $\\tau<0$, and contradicts to the above dense graph condition otherwise. Therefore, $f(x)=x^{\\tau}$ for $x>0$ and some constant $\\tau>0$. Dividing $E(a, b, c)$ by $(a-b)(b-$ $c)(c-a)=\\left(a b^{2}+b c^{2}+c a^{2}\\right)-\\left(a^{2} b+b^{2} c+c^{2} a\\right)$ and taking a limit when $a, b, c$ all go to 1 (the divided ratios tend to the corresponding derivatives, say, $\\frac{a^{\\tau}-b^{\\tau}}{a-b} \\rightarrow\\left(x^{\\tau}\\right)_{x=1}^{\\prime}=\\tau$ ), we get $\\tau^{3}=\\tau \\cdot 3^{\\tau-1}, \\tau^{2}=3^{\\tau-1}, F(\\tau):=3^{\\tau \/ 2-1 \/ 2}-\\tau=0$. Since function $F$ is strictly convex, it has at most two roots, and we get $\\tau \\in\\{1,3\\}$.","tier":0} +{"problem_type":"Combinatorics","problem_label":"C2","problem":"Let $n \\geqslant 3$ be an integer. An integer $m \\geqslant n+1$ is called $n$-colourful if, given infinitely many marbles in each of $n$ colours $C_{1}, C_{2}, \\ldots, C_{n}$, it is possible to place $m$ of them around a circle so that in any group of $n+1$ consecutive marbles there is at least one marble of colour $C_{i}$ for each $i=1, \\ldots, n$. Prove that there are only finitely many positive integers which are not $n$-colourful. Find the largest among them. Answer: $m_{\\max }=n^{2}-n-1$.","solution":"First suppose that there are $n(n-1)-1$ marbles. Then for one of the colours, say blue, there are at most $n-2$ marbles, which partition the non-blue marbles into at most $n-2$ groups with at least $(n-1)^{2}>n(n-2)$ marbles in total. Thus one of these groups contains at least $n+1$ marbles and this group does not contain any blue marble. Now suppose that the total number of marbles is at least $n(n-1)$. Then we may write this total number as $n k+j$ with some $k \\geqslant n-1$ and with $0 \\leqslant j \\leqslant n-1$. We place around a circle $k-j$ copies of the colour sequence $[1,2,3, \\ldots, n]$ followed by $j$ copies of the colour sequence $[1,1,2,3, \\ldots, n]$.","tier":0} +{"problem_type":"Step 2: Red and green colouring.","problem_label":"C6","problem":"A hunter and an invisible rabbit play a game on an infinite square grid. First the hunter fixes a colouring of the cells with finitely many colours. The rabbit then secretly chooses a cell to start in. Every minute, the rabbit reports the colour of its current cell to the hunter, and then secretly moves to an adjacent cell that it has not visited before (two cells are adjacent if they share a side). The hunter wins if after some finite time either - the rabbit cannot move; or - the hunter can determine the cell in which the rabbit started. Decide whether there exists a winning strategy for the hunter. Answer: Yes, there exists a colouring that yields a winning strategy for the hunter.","solution":"A central idea is that several colourings $C_{1}, C_{2}, \\ldots, C_{k}$ can be merged together into a single product colouring $C_{1} \\times C_{2} \\times \\cdots \\times C_{k}$ as follows: the colours in the product colouring are ordered tuples $\\left(c_{1}, \\ldots, c_{n}\\right)$ of colours, where $c_{i}$ is a colour used in $C_{i}$, so that each cell gets a tuple consisting of its colours in the individual colourings $C_{i}$. This way, any information which can be determined from one of the individual colourings can also be determined from the product colouring. Now let the hunter merge the following colourings: - The first two colourings $C_{1}$ and $C_{2}$ allow the tracking of the horizontal and vertical movements of the rabbit. The colouring $C_{1}$ colours the cells according to the residue of their $x$-coordinates modulo 3 , which allows to determine whether the rabbit moves left, moves right, or moves vertically. Similarly, the colouring $C_{2}$ uses the residues of the $y$-coordinates modulo 3, which allows to determine whether the rabbit moves up, moves down, or moves horizontally. - Under the condition that the rabbit's $x$-coordinate is unbounded, colouring $C_{3}$ allows to determine the exact value of the $x$-coordinate: In $C_{3}$, the columns are coloured white and black so that the gaps between neighboring black columns are pairwise distinct. As the rabbit's $x$-coordinate is unbounded, it will eventually visit two black cells in distinct columns. With the help of colouring $C_{1}$ the hunter can catch that moment, and determine the difference of $x$-coordinates of those two black cells, hence deducing the precise column. Symmetrically, under the condition that the rabbit's $y$-coordinate is unbounded, there is a colouring $C_{4}$ that allows the hunter to determine the exact value of the $y$-coordinate. - Finally, under the condition that the sum $x+y$ of the rabbit's coordinates is unbounded, colouring $C_{5}$ allows to determine the exact value of this sum: The diagonal lines $x+y=$ const are coloured black and white, so that the gaps between neighboring black diagonals are pairwise distinct. Unless the rabbit gets stuck, at least two of the three values $x, y$ and $x+y$ must be unbounded as the rabbit keeps moving. Hence the hunter can eventually determine two of these three values; thus he does know all three. Finally the hunter works backwards with help of the colourings $C_{1}$ and $C_{2}$ and computes the starting cell of the rabbit. Comment. There are some variations of the solution above: e.g., the colourings $C_{3}, C_{4}$ and $C_{5}$ can be replaced with different ones. However, such alternatives are more technically involved, and we do not present them here.","tier":0} +{"problem_type":"Step 2: Red and green colouring.","problem_label":"C7","problem":"Consider a checkered $3 m \\times 3 m$ square, where $m$ is an integer greater than 1. A frog sits on the lower left corner cell $S$ and wants to get to the upper right corner cell $F$. The frog can hop from any cell to either the next cell to the right or the next cell upwards. Some cells can be sticky, and the frog gets trapped once it hops on such a cell. A set $X$ of cells is called blocking if the frog cannot reach $F$ from $S$ when all the cells of $X$ are sticky. A blocking set is minimal if it does not contain a smaller blocking set. (a) Prove that there exists a minimal blocking set containing at least $3 m^{2}-3 m$ cells. (b) Prove that every minimal blocking set contains at most $3 m^{2}$ cells. Note. An example of a minimal blocking set for $m=2$ is shown below. Cells of the set $X$ are marked by letters $x$. | | | | | | $F$ | | :--- | :--- | :--- | :--- | :--- | :--- | | $x$ | $x$ | | | | | | | | $x$ | | | | | | | | $x$ | | | | | | | | $x$ | | | $S$ | | $x$ | | | | Solution for part (a). In the following example the square is divided into $m$ stripes of size $3 \\times 3 \\mathrm{~m}$. It is easy to see that $X$ is a minimal blocking set. The first and the last stripe each contains $3 m-1$ cells from the set $X$; every other stripe contains $3 m-2$ cells, see Figure 1 . The total number of cells in the set $X$ is $3 m^{2}-2 m+2$. ![](https:\/\/cdn.mathpix.com\/cropped\/2024_11_18_7107063274cfa469597ag-35.jpg?height=480&width=778&top_left_y=1459&top_left_x=642) Figure 1 Solution 1 for part (b). For a given blocking set $X$, say that a non-sticky cell is red if the frog can reach it from $S$ via some hops without entering set $X$. We call a non-sticky cell blue if the frog can reach $F$ from that cell via hops without entering set $X$. One can regard the blue cells as those reachable from $F$ by anti-hops, i.e. moves downwards and to the left. We also colour all cells in $X$ green. It follows from the definition of the blocking set that no cell will be coloured twice. In Figure 2 we show a sample of a blocking set and the corresponding colouring. Now assume that $X$ is a minimal blocking set. We denote by $R$ (resp., $B$ and $G$ ) be the total number of red (resp., blue and green) cells. We claim that $G \\leqslant R+1$ and $G \\leqslant B+1$. Indeed, there are at most $2 R$ possible frog hops from red cells. Every green or red cell (except for $S$ ) is accessible by such hops. Hence $2 R \\geqslant G+(R-1)$, or equivalently $G \\leqslant R+1$. In order to prove the inequality $G \\leqslant B+1$, we turn over the board and apply the similar arguments. Therefore we get $9 m^{2} \\geqslant B+R+G \\geqslant 3 G-2$, so $G \\leqslant 3 m^{2}$. | $x$ | | | | | | | | | :--- | :--- | :--- | :--- | :--- | :--- | :--- | :--- | | | $x$ | | $x$ | | | $x$ | | | | $x$ | | | $x$ | | | $x$ | | | $x$ | | | $x$ | | | $x$ | | | $x$ | | | $x$ | | | $x$ | | | $x$ | | | $x$ | | | $x$ | | | $x$ | | | $x$ | | | $x$ | | | | $x$ | | | $x$ | | $x$ | | $S$ | | | | | | | | Figure 2 (a) ![](https:\/\/cdn.mathpix.com\/cropped\/2024_11_18_7107063274cfa469597ag-36.jpg?height=392&width=392&top_left_y=204&top_left_x=1095) Figure 2 (b) Solution 2 for part (b). We shall use the same colouring as in the above solution. Again, assume that $X$ is a minimal blocking set. Note that any $2 \\times 2$ square cannot contain more than 2 green cells. Indeed, on Figure 3(a) the cell marked with \"?\" does not block any path, while on Figure 3(b) the cell marked with \"?\" should be coloured red and blue simultaneously. So we can split all green cells into chains consisting of three types of links shown on Figure 4 (diagonal link in the other direction is not allowed, corresponding green cells must belong to different chains). For example, there are 3 chains in Figure 2(b). ![](https:\/\/cdn.mathpix.com\/cropped\/2024_11_18_7107063274cfa469597ag-36.jpg?height=175&width=260&top_left_y=1123&top_left_x=344) Figure 3 ![](https:\/\/cdn.mathpix.com\/cropped\/2024_11_18_7107063274cfa469597ag-36.jpg?height=141&width=447&top_left_y=1157&top_left_x=716) Figure 4 ![](https:\/\/cdn.mathpix.com\/cropped\/2024_11_18_7107063274cfa469597ag-36.jpg?height=161&width=453&top_left_y=1139&top_left_x=1264) Figure 5 We will inscribe green chains in disjoint axis-aligned rectangles so that the number of green cells in each rectangle will not exceed $1 \/ 3$ of the area of the rectangle. This will give us the bound $G \\leqslant 3 \\mathrm{~m}^{2}$. Sometimes the rectangle will be the minimal bounding rectangle of the chain, sometimes minimal bounding rectangles will be expanded in one or two directions in order to have sufficiently large area. Note that for any two consecutive cells in the chain the colouring of some neighbouring cells is uniquely defined (see Figure 5). In particular, this observation gives a corresponding rectangle for the chains of height (or width) 1 (see Figure 6(a)). A separate green cell can be inscribed in $1 \\times 3$ or $3 \\times 1$ rectangle with one red and one blue cell, see Figure 6(b)-(c), otherwise we get one of impossible configurations shown in Figure 3. ![](https:\/\/cdn.mathpix.com\/cropped\/2024_11_18_7107063274cfa469597ag-36.jpg?height=227&width=1001&top_left_y=1874&top_left_x=533) Figure 6 Figure 7 Any diagonal chain of length 2 is always inscribed in a $2 \\times 3$ or $3 \\times 2$ rectangle without another green cells. Indeed, one of the squares marked with \"?\" in Figure 7(a) must be red. If it is the bottom question mark, then the remaining cell in the corresponding $2 \\times 3$ rectangle must have the same colour, see Figure 7(b). A longer chain of height (or width) 2 always has a horizontal (resp., vertical) link and can be inscribed into a $3 \\times a$ rectangle. In this case we expand the minimal bounding rectangle across the long side which touches the mentioned link. On Figure 8(a) the corresponding expansion of the minimal bounding rectangle is coloured in light blue. The upper right corner cell must be also blue. Indeed it cannot be red or green. If it is not coloured in blue, see Figure 8(b), then all anti-hop paths from $F$ to \"?\" are blocked with green cells. And these green cells are surrounded by blue ones, what is impossible. In this case the green chain contains $a$ cells, which is exactly $1 \/ 3$ of the area of the rectangle. ![](https:\/\/cdn.mathpix.com\/cropped\/2024_11_18_7107063274cfa469597ag-37.jpg?height=158&width=275&top_left_y=321&top_left_x=636) Figure 8 (a) ![](https:\/\/cdn.mathpix.com\/cropped\/2024_11_18_7107063274cfa469597ag-37.jpg?height=270&width=398&top_left_y=205&top_left_x=1032) Figure 8 (b) In the remaining case the minimal bounding rectangle of the chain is of size $a \\times b$ where $a, b \\geqslant 3$. Denote by $\\ell$ the length of the chain (i.e. the number of cells in the chain). If the chain has at least two diagonal links (see Figure 9), then $\\ell \\leqslant a+b-3 \\leqslant a b \/ 3$. If the chain has only one diagonal link then $\\ell=a+b-2$. In this case the chain has horizontal and vertical end-links, and we expand the minimal bounding rectangle in two directions to get an $(a+1) \\times(b+1)$ rectangle. On Figure 10 a corresponding expansion of the minimal bounding rectangle is coloured in light red. Again the length of the chain does not exceed $1 \/ 3$ of the rectangle's area: $\\ell \\leqslant a+b-2 \\leqslant(a+1)(b+1) \/ 3$. On the next step we will use the following statement: all cells in constructed rectangles are coloured red, green or blue (the cells upwards and to the right of green cells are blue; the cells downwards and to the left of green cells are red). The proof repeats the same arguments as before (see Figure 8(b).) ![](https:\/\/cdn.mathpix.com\/cropped\/2024_11_18_7107063274cfa469597ag-37.jpg?height=141&width=258&top_left_y=1363&top_left_x=294) Figure 9 ![](https:\/\/cdn.mathpix.com\/cropped\/2024_11_18_7107063274cfa469597ag-37.jpg?height=183&width=304&top_left_y=1322&top_left_x=679) Figure 10 ![](https:\/\/cdn.mathpix.com\/cropped\/2024_11_18_7107063274cfa469597ag-37.jpg?height=241&width=323&top_left_y=1213&top_left_x=1112) (a) ![](https:\/\/cdn.mathpix.com\/cropped\/2024_11_18_7107063274cfa469597ag-37.jpg?height=230&width=324&top_left_y=1213&top_left_x=1454) (b) Figure 11 Note that all constructed rectangles are disjoint. Indeed, assume that two rectangles have a common cell. Using the above statement, one can see that the only such cell can be a common corner cell, as shown in Figure 11. Moreover, in this case both rectangles should be expanded, otherwise they would share a green corner cell. If they were expanded along the same axis (see Figure 11(a)), then again the common corner cannot be coloured correctly. If they were expanded along different axes (see Figure 11(b)) then the two chains have a common point and must be connected in one chain. (These arguments work for $2 \\times 3$ and $1 \\times 3$ rectangles in a similar manner.) Comment 1. We do not a priori know whether all points are either red, or blue, or green. One might colour the remaining cells in black. The arguments from Solution 2 allow to prove that black cells do not exist. (One can start with a black cell which is nearest to $S$. Its left and downward neighbours must be coloured green or blue. In all cases one gets a configuration similar to Figure 8(b).) Comment 2. The maximal possible size of a minimal blocking set in $3 m \\times 3 m$ rectangle seems to be $3 m^{2}-2 m+2$. One can prove a more precise upper bound on the cardinality of the minimal blocking set: $G \\leqslant$ $3 m^{2}-m+2$. Denote by $D_{R}$ the number of red branching cells (i.e. such cells which have 2 red subsequent neighbours). And let $D_{B}$ be the number of similar blue cells. Then a double counting argument allows to prove that $G \\leqslant R-D_{R}+1$ and $G \\leqslant B-D_{B}+1$. Thus, we can bound $G$ in terms of $D_{B}$ and $D_{R}$ as $$ 9 m^{2} \\geqslant R+B+G \\geqslant 3 G+D_{R}+D_{B}-2 . $$ Now one can estimate the number of branching cells in order to obtain that $G \\leqslant 3 m^{2}-m+2$. Comment 3. An example with $3 m^{2}-2 m+2$ green cells may look differently; see, e.g., Figure 12. ![](https:\/\/cdn.mathpix.com\/cropped\/2024_11_18_7107063274cfa469597ag-38.jpg?height=529&width=532&top_left_y=295&top_left_x=768) Figure 12","solution":null,"tier":0} +{"problem_type":"Step 2: Red and green colouring.","problem_label":"C8","problem":"Determine the largest $N$ for which there exists a table $T$ of integers with $N$ rows and 100 columns that has the following properties: (i) Every row contains the numbers $1,2, \\ldots, 100$ in some order. (ii) For any two distinct rows $r$ and $s$, there is a column $c$ such that $|T(r, c)-T(s, c)| \\geqslant 2$. Here $T(r, c)$ means the number at the intersection of the row $r$ and the column $c$. Answer: The largest such integer is $N=100!\/ 2^{50}$.","solution":"Non-existence of a larger table. Let us consider some fixed row in the table, and let us replace (for $k=1,2, \\ldots, 50$ ) each of two numbers $2 k-1$ and $2 k$ respectively by the symbol $x_{k}$. The resulting pattern is an arrangement of 50 symbols $x_{1}, x_{2}, \\ldots, x_{50}$, where every symbol occurs exactly twice. Note that there are $N=100!\/ 2^{50}$ distinct patterns $P_{1}, \\ldots, P_{N}$. If two rows $r \\neq s$ in the table have the same pattern $P_{i}$, then $|T(r, c)-T(s, c)| \\leqslant 1$ holds for all columns $c$. As this violates property (ii) in the problem statement, different rows have different patterns. Hence there are at most $N=100!\/ 2^{50}$ rows. Existence of a table with $N$ rows. We construct the table by translating every pattern $P_{i}$ into a corresponding row with the numbers $1,2, \\ldots, 100$. We present a procedure that inductively replaces the symbols by numbers. The translation goes through steps $k=1,2, \\ldots, 50$ in increasing order and at step $k$ replaces the two occurrences of symbol $x_{k}$ by $2 k-1$ and $2 k$. - The left occurrence of $x_{1}$ is replaced by 1 , and its right occurrence is replaced by 2. - For $k \\geqslant 2$, we already have the number $2 k-2$ somewhere in the row, and now we are looking for the places for $2 k-1$ and $2 k$. We make the three numbers $2 k-2,2 k-1,2 k$ show up (ordered from left to right) either in the order $2 k-2,2 k-1,2 k$, or as $2 k, 2 k-2,2 k-1$, or as $2 k-1,2 k, 2 k-2$. This is possible, since the number $2 k-2$ has been placed in the preceding step, and shows up before \/ between \/ after the two occurrences of the symbol $x_{k}$. We claim that the $N$ rows that result from the $N$ patterns yield a table with the desired property (ii). Indeed, consider the $r$-th and the $s$-th row ( $r \\neq s$ ), which by construction result from patterns $P_{r}$ and $P_{s}$. Call a symbol $x_{i}$ aligned, if it occurs in the same two columns in $P_{r}$ and in $P_{s}$. Let $k$ be the largest index, for which symbol $x_{k}$ is not aligned. Note that $k \\geqslant 2$. Consider the column $c^{\\prime}$ with $T\\left(r, c^{\\prime}\\right)=2 k$ and the column $c^{\\prime \\prime}$ with $T\\left(s, c^{\\prime \\prime}\\right)=2 k$. Then $T\\left(r, c^{\\prime \\prime}\\right) \\leqslant 2 k$ and $T\\left(s, c^{\\prime}\\right) \\leqslant 2 k$, as all symbols $x_{i}$ with $i \\geqslant k+1$ are aligned. - If $T\\left(r, c^{\\prime \\prime}\\right) \\leqslant 2 k-2$, then $\\left|T\\left(r, c^{\\prime \\prime}\\right)-T\\left(s, c^{\\prime \\prime}\\right)\\right| \\geqslant 2$ as desired. - If $T\\left(s, c^{\\prime}\\right) \\leqslant 2 k-2$, then $\\left|T\\left(r, c^{\\prime}\\right)-T\\left(s, c^{\\prime}\\right)\\right| \\geqslant 2$ as desired. - If $T\\left(r, c^{\\prime \\prime}\\right)=2 k-1$ and $T\\left(s, c^{\\prime}\\right)=2 k-1$, then the symbol $x_{k}$ is aligned; contradiction. In the only remaining case we have $c^{\\prime}=c^{\\prime \\prime}$, so that $T\\left(r, c^{\\prime}\\right)=T\\left(s, c^{\\prime}\\right)=2 k$ holds. Now let us consider the columns $d^{\\prime}$ and $d^{\\prime \\prime}$ with $T\\left(r, d^{\\prime}\\right)=2 k-1$ and $T\\left(s, d^{\\prime \\prime}\\right)=2 k-1$. Then $d \\neq d^{\\prime \\prime}$ (as the symbol $x_{k}$ is not aligned), and $T\\left(r, d^{\\prime \\prime}\\right) \\leqslant 2 k-2$ and $T\\left(s, d^{\\prime}\\right) \\leqslant 2 k-2$ (as all symbols $x_{i}$ with $i \\geqslant k+1$ are aligned). - If $T\\left(r, d^{\\prime \\prime}\\right) \\leqslant 2 k-3$, then $\\left|T\\left(r, d^{\\prime \\prime}\\right)-T\\left(s, d^{\\prime \\prime}\\right)\\right| \\geqslant 2$ as desired. - If $T\\left(s, c^{\\prime}\\right) \\leqslant 2 k-3$, then $\\left|T\\left(r, d^{\\prime}\\right)-T\\left(s, d^{\\prime}\\right)\\right| \\geqslant 2$ as desired. In the only remaining case we have $T\\left(r, d^{\\prime \\prime}\\right)=2 k-2$ and $T\\left(s, d^{\\prime}\\right)=2 k-2$. Now the row $r$ has the numbers $2 k-2,2 k-1,2 k$ in the three columns $d^{\\prime}, d^{\\prime \\prime}, c^{\\prime}$. As one of these triples violates the ordering property of $2 k-2,2 k-1,2 k$, we have the final contradiction. Comment 1. We can identify rows of the table $T$ with permutations of $\\mathcal{M}:=\\{1, \\ldots, 100\\}$; also for every set $S \\subset \\mathcal{M}$ each row induces a subpermutation of $S$ obtained by ignoring all entries not from $S$. The example from Solution 1 consists of all permutations for which all subpermutations of the 50 sets $\\{1,2\\},\\{2,3,4\\},\\{4,5,6\\}, \\ldots,\\{98,99,100\\}$ are even.","tier":0} +{"problem_type":"Step 2: Red and green colouring.","problem_label":"C8","problem":"Determine the largest $N$ for which there exists a table $T$ of integers with $N$ rows and 100 columns that has the following properties: (i) Every row contains the numbers $1,2, \\ldots, 100$ in some order. (ii) For any two distinct rows $r$ and $s$, there is a column $c$ such that $|T(r, c)-T(s, c)| \\geqslant 2$. Here $T(r, c)$ means the number at the intersection of the row $r$ and the column $c$. Answer: The largest such integer is $N=100!\/ 2^{50}$.","solution":"We provide a bit different proof why the example from Lemma. Let $\\pi_{1}$ and $\\pi_{2}$ be two permutations of the set $\\{1,2, \\ldots, n\\}$ such that $\\left|\\pi_{1}(i)-\\pi_{2}(i)\\right| \\leqslant 1$ for every $i$. Then there exists a set of disjoint pairs $(i, i+1)$ such that $\\pi_{2}$ is obtained from $\\pi_{1}$ by swapping elements in each pair from the set. Proof. We may assume that $\\pi_{1}(i)=i$ for every $i$ and proceed by induction on $n$. The case $n=1$ is trivial. If $\\pi_{2}(n)=n$, we simply apply the induction hypothesis. If $\\pi_{2}(n)=n-1$, then $\\pi_{2}(i)=n$ for some $iB C$. Suppose that $A D>D C$, and let $H=A C \\cap B D$. Then the rays $B B_{1}$ and $D D_{1}$ lie on one side of $B D$, as they contain the midpoints of the arcs $A D C$ and $A B C$, respectively. However, if $B D_{1} \\| D B_{1}$, then $B_{1}$ and $D_{1}$ should be separated by $H$. This contradiction shows that $A D\\overparen{C B}$. In the right-angled triangle $B B_{1} B_{2}$, the point $L_{B}$ is a point on the hypothenuse such that $L_{B} B_{1}=L_{B} B$, so $L_{B}$ is the midpoint of $B_{1} B_{2}$. Since $D D_{1}$ is the internal angle bisector of $\\angle A D C$, we have $$ \\angle B D D_{1}=\\frac{\\angle B D A-\\angle C D B}{2}=\\frac{\\angle B C A-\\angle C A B}{2}=\\angle B B_{2} D_{1}, $$ so the points $B, B_{2}, D$, and $D_{1}$ lie on some circle $\\omega_{B}$. Similarly, $L_{D}$ is the midpoint of $D_{1} D_{2}$, and the points $D, D_{2}, B$, and $B_{1}$ lie on some circle $\\omega_{D}$. Now we have $$ \\angle B_{2} D B_{1}=\\angle B_{2} D B-\\angle B_{1} D B=\\angle B_{2} D_{1} B-\\angle B_{1} D_{2} B=\\angle D_{2} B D_{1} . $$ Therefore, the corresponding sides of the triangles $D B_{1} B_{2}$ and $B D_{1} D_{2}$ are parallel, and the triangles are homothetical (in $H$ ). So their corresponding medians $D L_{B}$ and $B L_{D}$ are also parallel. ![](https:\/\/cdn.mathpix.com\/cropped\/2024_11_18_7107063274cfa469597ag-53.jpg?height=535&width=1492&top_left_y=1914&top_left_x=285) Yet alternatively, after obtaining the circles $\\omega_{B}$ and $\\omega_{D}$, one may notice that $H$ lies on their radical axis $B D$, whence $H B_{1} \\cdot H D_{2}=H D_{1} \\cdot H B_{2}$, or $$ \\frac{H B_{1}}{H D_{1}}=\\frac{H B_{2}}{H D_{1}} . $$ Since $h\\left(D_{1}\\right)=B_{1}$, this yields $h\\left(D_{2}\\right)=B_{2}$ and hence $h\\left(L_{D}\\right)=L_{B}$. Comment 3. Since $h$ preserves the line $A C$ and maps $B \\mapsto D$ and $D_{1} \\mapsto B_{1}$, we have $h\\left(\\gamma_{B}\\right)=\\gamma_{D}$. Therefore, $h\\left(O_{B}\\right)=O_{D}$; in particular, $H$ also lies on $O_{B} O_{D}$.","tier":0} +{"problem_type":"Geometry","problem_label":"G5","problem":"Let $A B C D$ be a cyclic quadrilateral whose sides have pairwise different lengths. Let $O$ be the circumcentre of $A B C D$. The internal angle bisectors of $\\angle A B C$ and $\\angle A D C$ meet $A C$ at $B_{1}$ and $D_{1}$, respectively. Let $O_{B}$ be the centre of the circle which passes through $B$ and is tangent to $A C$ at $D_{1}$. Similarly, let $O_{D}$ be the centre of the circle which passes through $D$ and is tangent to $A C$ at $B_{1}$. Assume that $B D_{1} \\| D B_{1}$. Prove that $O$ lies on the line $O_{B} O_{D}$. Common remarks. We introduce some objects and establish some preliminary facts common for all solutions below. Let $\\Omega$ denote the circle $(A B C D)$, and let $\\gamma_{B}$ and $\\gamma_{D}$ denote the two circles from the problem statement (their centres are $O_{B}$ and $O_{D}$, respectively). Clearly, all three centres $O, O_{B}$, and $O_{D}$ are distinct. Assume, without loss of generality, that $A B>B C$. Suppose that $A D>D C$, and let $H=A C \\cap B D$. Then the rays $B B_{1}$ and $D D_{1}$ lie on one side of $B D$, as they contain the midpoints of the arcs $A D C$ and $A B C$, respectively. However, if $B D_{1} \\| D B_{1}$, then $B_{1}$ and $D_{1}$ should be separated by $H$. This contradiction shows that $A D1 \/ 2$ and $P A_{\\ell}>1 \/ 2$. Choose points $Q \\in A_{0} P, R \\in P A_{\\ell}$, and $S \\in P A_{m}$ such that $P Q=P R=1 \/ 2$ and $P S=\\sqrt{3} \/ 2$. Then $Q R S$ is an equilateral triangle of side length 1 contained in $A_{0} A_{1} \\ldots A_{\\ell}$. ![](https:\/\/cdn.mathpix.com\/cropped\/2024_11_18_7107063274cfa469597ag-57.jpg?height=335&width=1332&top_left_y=404&top_left_x=365) Comment. In fact, for every odd $n$ a stronger statement holds, which is formulated in terms defined in the solution above: there exists an equilateral triangle $A_{i} A_{i+1} B$ contained in $A_{0} A_{1} \\ldots A_{\\ell}$ for some $0 \\leqslant i<\\ell$. We sketch an indirect proof below. As above, we get $\\angle A_{\\ell} A_{0} A_{1}<60^{\\circ}$ and $A_{\\ell-1} A_{\\ell} A_{0}<60^{\\circ}$. Choose an index $m \\in[1, \\ell-1]$ maximising the distance between $A_{m}$ and $A_{0} A_{\\ell}$. Arguments from the above solution yield $1120^{\\circ}$ and $\\angle A_{m-1} A_{m} A_{\\ell}>\\angle A_{0} A_{m} A_{\\ell} \\geqslant 60^{\\circ}$. We construct an equilateral triangle $A_{m-1} A_{m} B$ as in the figure below. If $B$ lies in $A_{0} A_{m-1} A_{m} A_{\\ell}$, then we are done. Otherwise $B$ and $A_{m}$ lie on different sides of $A_{0} A_{\\ell}$. As before, let $P$ be the projection of $A_{m}$ to $A_{0} A_{\\ell}$. We will show that $$ A_{0} A_{1}+A_{1} A_{2}+\\ldots+A_{m-1} A_{m}A C$ so that $\\angle B A D=\\angle D A C$. A point $E$ is constructed on the segment $A C$ so that $\\angle A D E=\\angle D C B$. Similarly, a point $F$ is constructed on the segment $A B$ so that $\\angle A D F=\\angle D B C$. A point $X$ is chosen on the line $A C$ so that $C X=B X$. Let $O_{1}$ and $O_{2}$ be the circumcentres of the triangles $A D C$ and $D X E$. Prove that the lines $B C, E F$, and $O_{1} O_{2}$ are concurrent. Common remarks. Let $Q$ be the isogonal conjugate of $D$ with respect to the triangle $A B C$. Since $\\angle B A D=\\angle D A C$, the point $Q$ lies on $A D$. Then $\\angle Q B A=\\angle D B C=\\angle F D A$, so the points $Q, D, F$, and $B$ are concyclic. Analogously, the points $Q, D, E$, and $C$ are concyclic. Thus $A F \\cdot A B=A D \\cdot A Q=A E \\cdot A C$ and so the points $B, F, E$, and $C$ are also concyclic. ![](https:\/\/cdn.mathpix.com\/cropped\/2024_11_18_7107063274cfa469597ag-60.jpg?height=478&width=1204&top_left_y=732&top_left_x=426) Let $T$ be the intersection of $B C$ and $F E$. Claim. TD $D^{2}=T B \\cdot T C=T F \\cdot T E$. Proof. We will prove that the circles $(D E F)$ and $(B D C)$ are tangent to each other. Indeed, using the above arguments, we get $$ \\begin{aligned} & \\angle B D F=\\angle A F D-\\angle A B D=\\left(180^{\\circ}-\\angle F A D-\\angle F D A\\right)-(\\angle A B C-\\angle D B C) \\\\ = & 180^{\\circ}-\\angle F A D-\\angle A B C=180^{\\circ}-\\angle D A E-\\angle F E A=\\angle F E D+\\angle A D E=\\angle F E D+\\angle D C B, \\end{aligned} $$ which implies the desired tangency. Since the points $B, C, E$, and $F$ are concyclic, the powers of the point $T$ with respect to the circles $(B D C)$ and $(E D F)$ are equal. So their radical axis, which coincides with the common tangent at $D$, passes through $T$, and hence $T D^{2}=T E \\cdot T F=T B \\cdot T C$.","solution":"Let $T A$ intersect the circle $(A B C)$ again at $M$. Due to the circles ( $B C E F$ ) and $(A M C B)$, and using the above Claim, we get $T M \\cdot T A=T F \\cdot T E=T B \\cdot T C=T D^{2}$; in particular, the points $A, M, E$, and $F$ are concyclic. Under the inversion with centre $T$ and radius $T D$, the point $M$ maps to $A$, and $B$ maps to $C$, which implies that the circle $(M B D)$ maps to the circle $(A D C)$. Their common point $D$ lies on the circle of the inversion, so the second intersection point $K$ also lies on that circle, which means $T K=T D$. It follows that the point $T$ and the centres of the circles ( $K D E$ ) and $(A D C)$ lie on the perpendicular bisector of $K D$. Since the center of $(A D C)$ is $O_{1}$, it suffices to show now that the points $D, K, E$, and $X$ are concyclic (the center of the corresponding circle will be $O_{2}$ ). The lines $B M, D K$, and $A C$ are the pairwise radical axes of the circles $(A B C M),(A C D K)$ and $(B M D K)$, so they are concurrent at some point $P$. Also, $M$ lies on the circle $(A E F)$, thus $$ \\begin{aligned} \\Varangle(E X, X B) & =\\Varangle(C X, X B)=\\Varangle(X C, B C)+\\Varangle(B C, B X)=2 \\Varangle(A C, C B) \\\\ & =\\Varangle(A C, C B)+\\Varangle(E F, F A)=\\Varangle(A M, B M)+\\Varangle(E M, M A)=\\Varangle(E M, B M), \\end{aligned} $$ so the points $M, E, X$, and $B$ are concyclic. Therefore, $P E \\cdot P X=P M \\cdot P B=P K \\cdot P D$, so the points $E, K, D$, and $X$ are concyclic, as desired. ![](https:\/\/cdn.mathpix.com\/cropped\/2024_11_18_7107063274cfa469597ag-61.jpg?height=935&width=1466&top_left_y=195&top_left_x=295) Comment 1. We present here a different solution which uses similar ideas. Perform the inversion $\\iota$ with centre $T$ and radius $T D$. It swaps $B$ with $C$ and $E$ with $F$; the point $D$ maps to itself. Let $X^{\\prime}=\\iota(X)$. Observe that the points $E, F, X$, and $X^{\\prime}$ are concyclic, as well as the points $B, C, X$, and $X^{\\prime}$. Then $$ \\begin{aligned} & \\Varangle\\left(C X^{\\prime}, X^{\\prime} F\\right)=\\Varangle\\left(C X^{\\prime}, X^{\\prime} X\\right)+\\Varangle\\left(X^{\\prime} X, X^{\\prime} F\\right)=\\Varangle(C B, B X)+\\Varangle(E X, E F) \\\\ &=\\Varangle(X C, C B)+\\Varangle(E C, E F)=\\Varangle(C A, C B)+\\Varangle(B C, B F)=\\Varangle(C A, A F), \\end{aligned} $$ therefore the points $C, X^{\\prime}, A$, and $F$ are concyclic. Let $X^{\\prime} F$ intersect $A C$ at $P$, and let $K$ be the second common point of $D P$ and the circle $(A C D)$. Then $$ P K \\cdot P D=P A \\cdot P C=P X^{\\prime} \\cdot P F=P E \\cdot P X $$ hence, the points $K, X, D$, and $E$ lie on some circle $\\omega_{1}$, while the points $K, X^{\\prime}, D$, and $F$ lie on some circle $\\omega_{2}$. (These circles are distinct since $\\angle E X F+\\angle E D F<\\angle E A F+\\angle D C B+\\angle D B C<180^{\\circ}$ ). The inversion $\\iota$ swaps $\\omega_{1}$ with $\\omega_{2}$ and fixes their common point $D$, so it fixes their second common point $K$. Thus $T D=T K$ and the perpendicular bisector of $D K$ passes through $T$, as well as through the centres of the circles $(C D K A)$ and $(D E K X)$. ![](https:\/\/cdn.mathpix.com\/cropped\/2024_11_18_7107063274cfa469597ag-61.jpg?height=775&width=1154&top_left_y=1994&top_left_x=451)","tier":0} +{"problem_type":"Geometry","problem_label":"G7","problem":"A point $D$ is chosen inside an acute-angled triangle $A B C$ with $A B>A C$ so that $\\angle B A D=\\angle D A C$. A point $E$ is constructed on the segment $A C$ so that $\\angle A D E=\\angle D C B$. Similarly, a point $F$ is constructed on the segment $A B$ so that $\\angle A D F=\\angle D B C$. A point $X$ is chosen on the line $A C$ so that $C X=B X$. Let $O_{1}$ and $O_{2}$ be the circumcentres of the triangles $A D C$ and $D X E$. Prove that the lines $B C, E F$, and $O_{1} O_{2}$ are concurrent. Common remarks. Let $Q$ be the isogonal conjugate of $D$ with respect to the triangle $A B C$. Since $\\angle B A D=\\angle D A C$, the point $Q$ lies on $A D$. Then $\\angle Q B A=\\angle D B C=\\angle F D A$, so the points $Q, D, F$, and $B$ are concyclic. Analogously, the points $Q, D, E$, and $C$ are concyclic. Thus $A F \\cdot A B=A D \\cdot A Q=A E \\cdot A C$ and so the points $B, F, E$, and $C$ are also concyclic. ![](https:\/\/cdn.mathpix.com\/cropped\/2024_11_18_7107063274cfa469597ag-60.jpg?height=478&width=1204&top_left_y=732&top_left_x=426) Let $T$ be the intersection of $B C$ and $F E$. Claim. TD $D^{2}=T B \\cdot T C=T F \\cdot T E$. Proof. We will prove that the circles $(D E F)$ and $(B D C)$ are tangent to each other. Indeed, using the above arguments, we get $$ \\begin{aligned} & \\angle B D F=\\angle A F D-\\angle A B D=\\left(180^{\\circ}-\\angle F A D-\\angle F D A\\right)-(\\angle A B C-\\angle D B C) \\\\ = & 180^{\\circ}-\\angle F A D-\\angle A B C=180^{\\circ}-\\angle D A E-\\angle F E A=\\angle F E D+\\angle A D E=\\angle F E D+\\angle D C B, \\end{aligned} $$ which implies the desired tangency. Since the points $B, C, E$, and $F$ are concyclic, the powers of the point $T$ with respect to the circles $(B D C)$ and $(E D F)$ are equal. So their radical axis, which coincides with the common tangent at $D$, passes through $T$, and hence $T D^{2}=T E \\cdot T F=T B \\cdot T C$.","solution":"We use only the first part of the Common remarks, namely, the facts that the tuples $(C, D, Q, E)$ and $(B, C, E, F)$ are both concyclic. We also introduce the point $T=$ $B C \\cap E F$. Let the circle $(C D E)$ meet $B C$ again at $E_{1}$. Since $\\angle E_{1} C Q=\\angle D C E$, the $\\operatorname{arcs} D E$ and $Q E_{1}$ of the circle $(C D Q)$ are equal, so $D Q \\| E E_{1}$. Since $B F E C$ is cyclic, the line $A D$ forms equal angles with $B C$ and $E F$, hence so does $E E_{1}$. Therefore, the triangle $E E_{1} T$ is isosceles, $T E=T E_{1}$, and $T$ lies on the common perpendicular bisector of $E E_{1}$ and $D Q$. Let $U$ and $V$ be the centres of circles $(A D E)$ and $(C D Q E)$, respectively. Then $U O_{1}$ is the perpendicular bisector of $A D$. Moreover, the points $U, V$, and $O_{2}$ belong to the perpendicular bisector of $D E$. Since $U O_{1} \\| V T$, in order to show that $O_{1} O_{2}$ passes through $T$, it suffices to show that $$ \\frac{O_{2} U}{O_{2} V}=\\frac{O_{1} U}{T V} $$ Denote angles $A, B$, and $C$ of the triangle $A B C$ by $\\alpha, \\beta$, and $\\gamma$, respectively. Projecting onto $A C$ we obtain $$ \\frac{O_{2} U}{O_{2} V}=\\frac{(X E-A E) \/ 2}{(X E+E C) \/ 2}=\\frac{A X}{C X}=\\frac{A X}{B X}=\\frac{\\sin (\\gamma-\\beta)}{\\sin \\alpha} $$ The projection of $O_{1} U$ onto $A C$ is $(A C-A E) \/ 2=C E \/ 2$; the angle between $O_{1} U$ and $A C$ is $90^{\\circ}-\\alpha \/ 2$, so $$ \\frac{O_{1} U}{E C}=\\frac{1}{2 \\sin (\\alpha \/ 2)} $$ Next, we claim that $E, V, C$, and $T$ are concyclic. Indeed, the point $V$ lies on the perpendicular bisector of $C E$, as well as on the internal angle bisector of $\\angle C T F$. Therefore, $V$ coincides with the midpoint of the arc $C E$ of the circle (TCE). Now we have $\\angle E V C=2 \\angle E E_{1} C=180^{\\circ}-(\\gamma-\\beta)$ and $\\angle V E T=\\angle V E_{1} T=90^{\\circ}-\\angle E_{1} E C=$ $90^{\\circ}-\\alpha \/ 2$. Therefore, $$ \\frac{E C}{T V}=\\frac{\\sin \\angle E T C}{\\sin \\angle V E T}=\\frac{\\sin (\\gamma-\\beta)}{\\cos (\\alpha \/ 2)} . $$ ![](https:\/\/cdn.mathpix.com\/cropped\/2024_11_18_7107063274cfa469597ag-62.jpg?height=986&width=1512&top_left_y=1774&top_left_x=266) Recalling (2) and multiplying (3) and (4) we establish (1): $$ \\frac{O_{2} U}{O_{2} V}=\\frac{\\sin (\\gamma-\\beta)}{\\sin \\alpha}=\\frac{1}{2 \\sin (\\alpha \/ 2)} \\cdot \\frac{\\sin (\\gamma-\\beta)}{\\cos (\\alpha \/ 2)}=\\frac{O_{1} U}{E C} \\cdot \\frac{E C}{T V}=\\frac{O_{1} U}{T V} $$","tier":0} +{"problem_type":"Geometry","problem_label":"G7","problem":"A point $D$ is chosen inside an acute-angled triangle $A B C$ with $A B>A C$ so that $\\angle B A D=\\angle D A C$. A point $E$ is constructed on the segment $A C$ so that $\\angle A D E=\\angle D C B$. Similarly, a point $F$ is constructed on the segment $A B$ so that $\\angle A D F=\\angle D B C$. A point $X$ is chosen on the line $A C$ so that $C X=B X$. Let $O_{1}$ and $O_{2}$ be the circumcentres of the triangles $A D C$ and $D X E$. Prove that the lines $B C, E F$, and $O_{1} O_{2}$ are concurrent. Common remarks. Let $Q$ be the isogonal conjugate of $D$ with respect to the triangle $A B C$. Since $\\angle B A D=\\angle D A C$, the point $Q$ lies on $A D$. Then $\\angle Q B A=\\angle D B C=\\angle F D A$, so the points $Q, D, F$, and $B$ are concyclic. Analogously, the points $Q, D, E$, and $C$ are concyclic. Thus $A F \\cdot A B=A D \\cdot A Q=A E \\cdot A C$ and so the points $B, F, E$, and $C$ are also concyclic. ![](https:\/\/cdn.mathpix.com\/cropped\/2024_11_18_7107063274cfa469597ag-60.jpg?height=478&width=1204&top_left_y=732&top_left_x=426) Let $T$ be the intersection of $B C$ and $F E$. Claim. TD $D^{2}=T B \\cdot T C=T F \\cdot T E$. Proof. We will prove that the circles $(D E F)$ and $(B D C)$ are tangent to each other. Indeed, using the above arguments, we get $$ \\begin{aligned} & \\angle B D F=\\angle A F D-\\angle A B D=\\left(180^{\\circ}-\\angle F A D-\\angle F D A\\right)-(\\angle A B C-\\angle D B C) \\\\ = & 180^{\\circ}-\\angle F A D-\\angle A B C=180^{\\circ}-\\angle D A E-\\angle F E A=\\angle F E D+\\angle A D E=\\angle F E D+\\angle D C B, \\end{aligned} $$ which implies the desired tangency. Since the points $B, C, E$, and $F$ are concyclic, the powers of the point $T$ with respect to the circles $(B D C)$ and $(E D F)$ are equal. So their radical axis, which coincides with the common tangent at $D$, passes through $T$, and hence $T D^{2}=T E \\cdot T F=T B \\cdot T C$.","solution":"Notice that $\\angle A Q E=\\angle Q C B$ and $\\angle A Q F=\\angle Q B C$; so, if we replace the point $D$ with $Q$ in the problem set up, the points $E, F$, and $T$ remain the same. So, by the Claim, we have $T Q^{2}=T B \\cdot T C=T D^{2}$. Thus, there exists a circle $\\Gamma$ centred at $T$ and passing through $D$ and $Q$. We denote the second meeting point of the circles $\\Gamma$ and $(A D C)$ by $K$. Let the line $A C$ meet the circle ( $D E K$ ) again at $Y$; we intend to prove that $Y=X$. As in Solution 1, this will yield that the point $T$, as well as the centres $O_{1}$ and $O_{2}$, all lie on the perpendicular bisector of $D K$. Let $L=A D \\cap B C$. We perform an inversion centred at $C$; the images of the points will be denoted by primes, e.g., $A^{\\prime}$ is the image of $A$. We obtain the following configuration, constructed in a triangle $A^{\\prime} C L^{\\prime}$. The points $D^{\\prime}$ and $Q^{\\prime}$ are chosen on the circumcircle $\\Omega$ of $A^{\\prime} L^{\\prime} C$ such that $\\Varangle\\left(L^{\\prime} C, D^{\\prime} C\\right)=$ $\\Varangle\\left(Q^{\\prime} C, A^{\\prime} C\\right)$, which means that $A^{\\prime} L^{\\prime} \\| D^{\\prime} Q^{\\prime}$. The lines $D^{\\prime} Q^{\\prime}$ and $A^{\\prime} C$ meet at $E^{\\prime}$. A circle $\\Gamma^{\\prime}$ centred on $C L^{\\prime}$ passes through $D^{\\prime}$ and $Q^{\\prime}$. Notice here that $B^{\\prime}$ lies on the segment $C L^{\\prime}$, and that $\\angle A^{\\prime} B^{\\prime} C=\\angle B A C=2 \\angle L A C=2 \\angle A^{\\prime} L^{\\prime} C$, so that $B^{\\prime} L^{\\prime}=B^{\\prime} A^{\\prime}$, and $B^{\\prime}$ lies on the perpendicular bisector of $A^{\\prime} L^{\\prime}$ (which coincides with that of $D^{\\prime} Q^{\\prime}$ ). All this means that $B^{\\prime}$ is the centre of $\\Gamma^{\\prime}$. Finally, $K^{\\prime}$ is the second meeting point of $A^{\\prime} D^{\\prime}$ and $\\Gamma^{\\prime}$, and $Y^{\\prime}$ is the second meeting point of the circle $\\left(D^{\\prime} K^{\\prime} E^{\\prime}\\right)$ and the line $A^{\\prime} E^{\\prime}$, We have $\\Varangle\\left(Y^{\\prime} K^{\\prime}, K^{\\prime} A^{\\prime}\\right)=\\Varangle\\left(Y^{\\prime} E^{\\prime}, E^{\\prime} D^{\\prime}\\right)=$ $\\Varangle\\left(Y^{\\prime} A^{\\prime}, A^{\\prime} L^{\\prime}\\right)$, so $A^{\\prime} L^{\\prime}$ is tangent to the circumcircle $\\omega$ of the triangle $Y^{\\prime} A^{\\prime} K^{\\prime}$. Let $O$ and $O^{*}$ be the centres of $\\Omega$ and $\\omega$, respectively. Then $O^{*} A^{\\prime} \\perp A^{\\prime} L^{\\prime} \\perp B^{\\prime} O$. The projections of vectors $\\overrightarrow{O^{*} A^{\\prime}}$ and $\\overrightarrow{B^{\\prime} O}$ onto $K^{\\prime} D^{\\prime}$ are equal to $\\overrightarrow{K^{\\prime} A^{\\prime}} \/ 2=\\overrightarrow{K^{\\prime} D^{\\prime}} \/ 2-\\overrightarrow{A^{\\prime} D^{\\prime}} \/ 2$. So $\\overrightarrow{O^{*} A^{\\prime}}=\\overrightarrow{B^{\\prime} O}$, or equivalently $\\overrightarrow{A^{\\prime} O}=\\overrightarrow{O^{*} B^{\\prime}}$. Projecting this equality onto $A^{\\prime} C$, we see that the projection of $\\overrightarrow{O^{*} \\overrightarrow{B^{\\prime}}}$ equals $\\overrightarrow{A^{\\prime} C} \/ 2$. Since $O^{*}$ is projected to the midpoint of $A^{\\prime} Y^{\\prime}$, this yields that $B^{\\prime}$ is projected to the midpoint of $C Y^{\\prime}$, i.e., $B^{\\prime} Y^{\\prime}=B^{\\prime} C$ and $\\angle B^{\\prime} Y^{\\prime} C=\\angle B^{\\prime} C Y^{\\prime}$. In the original figure, this rewrites as $\\angle C B Y=\\angle B C Y$, so $Y$ lies on the perpendicular bisector of $B C$, as desired. ![](https:\/\/cdn.mathpix.com\/cropped\/2024_11_18_7107063274cfa469597ag-63.jpg?height=843&width=1306&top_left_y=1783&top_left_x=381) Comment 2. The point $K$ appears to be the same in Solutions 1 and 3 (and Comment 1 as well). One can also show that $K$ lies on the circle passing through $A, X$, and the midpoint of the arc $B A C$. Comment 3. There are different proofs of the facts from the Common remarks, namely, the cyclicity of $B, C, E$, and $F$, and the Claim. We present one such alternative proof here. We perform the composition $\\phi$ of a homothety with centre $A$ and the reflection in $A D$, which maps $E$ to $B$. Let $U=\\phi(D)$. Then $\\Varangle(B C, C D)=\\Varangle(A D, D E)=\\Varangle(B U, U D)$, so the points $B, U, C$, and $D$ are concyclic. Therefore, $\\Varangle(C U, U D)=\\Varangle(C B, B D)=\\Varangle(A D, D F)$, so $\\phi(F)=C$. Then the coefficient of the homothety is $A C \/ A F=A B \/ A E$, and thus points $C, E, F$, and $B$ are concyclic. Denote the centres of the circles $(E D F)$ and $(B U C D)$ by $O_{3}$ and $O_{4}$, respectively. Then $\\phi\\left(O_{3}\\right)=$ $O_{4}$, hence $\\Varangle\\left(O_{3} D, D A\\right)=-\\Varangle\\left(O_{4} U, U A\\right)=\\Varangle\\left(O_{4} D, D A\\right)$, whence the circle $(B D C)$ is tangent to the circle ( $E D F$ ). Now, the radical axes of circles $(D E F),(B D C)$ and $(B C E F)$ intersect at $T$, and the claim follows. ![](https:\/\/cdn.mathpix.com\/cropped\/2024_11_18_7107063274cfa469597ag-64.jpg?height=869&width=941&top_left_y=682&top_left_x=563) This suffices for Solution 1 to work. However, Solutions 2 and 3 need properties of point $Q$, established in Common remarks before Solution 1. Comment 4. In the original problem proposal, the point $X$ was hidden. Instead, a circle $\\gamma$ was constructed such that $D$ and $E$ lie on $\\gamma$, and its center is collinear with $O_{1}$ and $T$. The problem requested to prove that, in a fixed triangle $A B C$, independently from the choice of $D$ on the bisector of $\\angle B A C$, all circles $\\gamma$ pass through a fixed point.","tier":0} +{"problem_type":"Number Theory","problem_label":"N1","problem":"Determine all integers $n \\geqslant 1$ for which there exists a pair of positive integers $(a, b)$ such that no cube of a prime divides $a^{2}+b+3$ and $$ \\frac{a b+3 b+8}{a^{2}+b+3}=n $$ Answer: The only integer with that property is $n=2$.","solution":"As $b \\equiv-a^{2}-3\\left(\\bmod a^{2}+b+3\\right)$, the numerator of the given fraction satisfies $$ a b+3 b+8 \\equiv a\\left(-a^{2}-3\\right)+3\\left(-a^{2}-3\\right)+8 \\equiv-(a+1)^{3} \\quad\\left(\\bmod a^{2}+b+3\\right) $$ As $a^{2}+b+3$ is not divisible by $p^{3}$ for any prime $p$, if $a^{2}+b+3$ divides $(a+1)^{3}$ then it does also divide $(a+1)^{2}$. Since $$ 0<(a+1)^{2}<2\\left(a^{2}+b+3\\right) $$ we conclude $(a+1)^{2}=a^{2}+b+3$. This yields $b=2(a-1)$ and $n=2$. The choice $(a, b)=(2,2)$ with $a^{2}+b+3=9$ shows that $n=2$ indeed is a solution.","tier":0} +{"problem_type":"Number Theory","problem_label":"N3","problem":"Find all positive integers $n$ with the following property: the $k$ positive divisors of $n$ have a permutation $\\left(d_{1}, d_{2}, \\ldots, d_{k}\\right)$ such that for every $i=1,2, \\ldots, k$, the number $d_{1}+\\cdots+d_{i}$ is a perfect square. Answer: $n=1$ and $n=3$.","solution":"For $i=1,2, \\ldots, k$ let $d_{1}+\\ldots+d_{i}=s_{i}^{2}$, and define $s_{0}=0$ as well. Obviously $0=s_{0}2 i>d_{i}>d_{i-1}>\\ldots>d_{1}$, so $j \\leqslant i$ is not possible. The only possibility is $j=i+1$. Hence, $$ \\begin{gathered} s_{i+1}+i=d_{i+1}=s_{i+1}^{2}-s_{i}^{2}=s_{i+1}^{2}-i^{2} \\\\ s_{i+1}^{2}-s_{i+1}=i(i+1) \\end{gathered} $$ By solving this equation we get $s_{i+1}=i+1$ and $d_{i+1}=2 i+1$, that finishes the proof. Now we know that the positive divisors of the number $n$ are $1,3,5, \\ldots, n-2, n$. The greatest divisor is $d_{k}=2 k-1=n$ itself, so $n$ must be odd. The second greatest divisor is $d_{k-1}=n-2$; then $n-2$ divides $n=(n-2)+2$, so $n-2$ divides 2 . Therefore, $n$ must be 1 or 3 . The numbers $n=1$ and $n=3$ obviously satisfy the requirements: for $n=1$ we have $k=1$ and $d_{1}=1^{2}$; for $n=3$ we have $k=2, d_{1}=1^{2}$ and $d_{1}+d_{2}=1+3=2^{2}$. This page is intentionally left blank","tier":0} +{"problem_type":"Number Theory","problem_label":"N4","problem":"Alice is given a rational number $r>1$ and a line with two points $B \\neq R$, where point $R$ contains a red bead and point $B$ contains a blue bead. Alice plays a solitaire game by performing a sequence of moves. In every move, she chooses a (not necessarily positive) integer $k$, and a bead to move. If that bead is placed at point $X$, and the other bead is placed at $Y$, then Alice moves the chosen bead to point $X^{\\prime}$ with $\\overrightarrow{Y X^{\\prime}}=r^{k} \\overrightarrow{Y X}$. Alice's goal is to move the red bead to the point $B$. Find all rational numbers $r>1$ such that Alice can reach her goal in at most 2021 moves. Answer: All $r=(b+1) \/ b$ with $b=1, \\ldots, 1010$.","solution":"Denote the red and blue beads by $\\mathcal{R}$ and $\\mathcal{B}$, respectively. Introduce coordinates on the line and identify the points with their coordinates so that $R=0$ and $B=1$. Then, during the game, the coordinate of $\\mathcal{R}$ is always smaller than the coordinate of $\\mathcal{B}$. Moreover, the distance between the beads always has the form $r^{\\ell}$ with $\\ell \\in \\mathbb{Z}$, since it only multiplies by numbers of this form. Denote the value of the distance after the $m^{\\text {th }}$ move by $d_{m}=r^{\\alpha_{m}}$, $m=0,1,2, \\ldots$ (after the $0^{\\text {th }}$ move we have just the initial position, so $\\alpha_{0}=0$ ). If some bead is moved in two consecutive moves, then Alice could instead perform a single move (and change the distance from $d_{i}$ directly to $d_{i+2}$ ) which has the same effect as these two moves. So, if Alice can achieve her goal, then she may as well achieve it in fewer (or the same) number of moves by alternating the moves of $\\mathcal{B}$ and $\\mathcal{R}$. In the sequel, we assume that Alice alternates the moves, and that $\\mathcal{R}$ is shifted altogether $t$ times. If $\\mathcal{R}$ is shifted in the $m^{\\text {th }}$ move, then its coordinate increases by $d_{m}-d_{m+1}$. Therefore, the total increment of $\\mathcal{R}$ 's coordinate, which should be 1 , equals $$ \\begin{aligned} \\text { either } \\quad\\left(d_{0}-d_{1}\\right)+\\left(d_{2}-d_{3}\\right)+\\cdots+\\left(d_{2 t-2}-d_{2 t-1}\\right) & =1+\\sum_{i=1}^{t-1} r^{\\alpha_{2 i}}-\\sum_{i=1}^{t} r^{\\alpha_{2 i-1}}, \\\\ \\text { or } \\quad\\left(d_{1}-d_{2}\\right)+\\left(d_{3}-d_{4}\\right)+\\cdots+\\left(d_{2 t-1}-d_{2 t}\\right) & =\\sum_{i=1}^{t} r^{\\alpha_{2 i-1}}-\\sum_{i=1}^{t} r^{\\alpha_{2 i}}, \\end{aligned} $$ depending on whether $\\mathcal{R}$ or $\\mathcal{B}$ is shifted in the first move. Moreover, in the former case we should have $t \\leqslant 1011$, while in the latter one we need $t \\leqslant 1010$. So both cases reduce to an equation $$ \\sum_{i=1}^{n} r^{\\beta_{i}}=\\sum_{i=1}^{n-1} r^{\\gamma_{i}}, \\quad \\beta_{i}, \\gamma_{i} \\in \\mathbb{Z} $$ for some $n \\leqslant 1011$. Thus, if Alice can reach her goal, then this equation has a solution for $n=1011$ (we can add equal terms to both sums in order to increase $n$ ). Conversely, if (1) has a solution for $n=1011$, then Alice can compose a corresponding sequence of distances $d_{0}, d_{1}, d_{2}, \\ldots, d_{2021}$ and then realise it by a sequence of moves. So the problem reduces to the solvability of (1) for $n=1011$. Assume that, for some rational $r$, there is a solution of (1). Write $r$ in lowest terms as $r=a \/ b$. Substitute this into (1), multiply by the common denominator, and collect all terms on the left hand side to get $$ \\sum_{i=1}^{2 n-1}(-1)^{i} a^{\\mu_{i}} b^{N-\\mu_{i}}=0, \\quad \\mu_{i} \\in\\{0,1, \\ldots, N\\} $$ for some $N \\geqslant 0$. We assume that there exist indices $j_{-}$and $j_{+}$such that $\\mu_{j_{-}}=0$ and $\\mu_{j_{+}}=N$. Reducing (2) modulo $a-b$ (so that $a \\equiv b$ ), we get $$ 0=\\sum_{i=1}^{2 n-1}(-1)^{i} a^{\\mu_{i}} b^{N-\\mu_{i}} \\equiv \\sum_{i=1}^{2 n-1}(-1)^{i} b^{\\mu_{i}} b^{N-\\mu_{i}}=-b^{N} \\quad \\bmod (a-b) $$ Since $\\operatorname{gcd}(a-b, b)=1$, this is possible only if $a-b=1$. Reducing (2) modulo $a+b$ (so that $a \\equiv-b$ ), we get $$ 0=\\sum_{i=1}^{2 n-1}(-1)^{i} a^{\\mu_{i}} b^{N-\\mu_{i}} \\equiv \\sum_{i=1}^{2 n-1}(-1)^{i}(-1)^{\\mu_{i}} b^{\\mu_{i}} b^{N-\\mu_{i}}=S b^{N} \\quad \\bmod (a+b) $$ for some odd (thus nonzero) $S$ with $|S| \\leqslant 2 n-1$. Since $\\operatorname{gcd}(a+b, b)=1$, this is possible only if $a+b \\mid S$. So $a+b \\leqslant 2 n-1$, and hence $b=a-1 \\leqslant n-1=1010$. Thus we have shown that any sought $r$ has the form indicated in the answer. It remains to show that for any $b=1,2, \\ldots, 1010$ and $a=b+1$, Alice can reach the goal. For this purpose, in (1) we put $n=a, \\beta_{1}=\\beta_{2}=\\cdots=\\beta_{a}=0$, and $\\gamma_{1}=\\gamma_{2}=\\cdots=\\gamma_{b}=1$. Comment 1. Instead of reducing modulo $a+b$, one can reduce modulo $a$ and modulo $b$. The first reduction shows that the number of terms in (2) with $\\mu_{i}=0$ is divisible by $a$, while the second shows that the number of terms with $\\mu_{i}=N$ is divisible by $b$. Notice that, in fact, $N>0$, as otherwise (2) contains an alternating sum of an odd number of equal terms, which is nonzero. Therefore, all terms listed above have different indices, and there are at least $a+b$ of them. Comment 2. Another way to investigate the solutions of equation (1) is to consider the Laurent polynomial $$ L(x)=\\sum_{i=1}^{n} x^{\\beta_{i}}-\\sum_{i=1}^{n-1} x^{\\gamma_{i}} . $$ We can pick a sufficiently large integer $d$ so that $P(x)=x^{d} L(x)$ is a polynomial in $\\mathbb{Z}[x]$. Then $$ P(1)=1, $$ and $$ 1 \\leqslant|P(-1)| \\leqslant 2021 $$ If $r=p \/ q$ with integers $p>q \\geqslant 1$ is a rational number with the properties listed in the problem statement, then $P(p \/ q)=L(p \/ q)=0$. As $P(x)$ has integer coefficients, $$ (p-q x) \\mid P(x) $$ Plugging $x=1$ into (5) gives $(p-q) \\mid P(1)=1$, which implies $p=q+1$. Moreover, plugging $x=-1$ into (5) gives $(p+q) \\mid P(-1)$, which, along with (4), implies $p+q \\leqslant 2021$ and $q \\leqslant 1010$. Hence $x=(q+1) \/ q$ for some integer $q$ with $1 \\leqslant q \\leqslant 1010$.","tier":0} +{"problem_type":"Number Theory","problem_label":"N6","problem":"Determine all integers $n \\geqslant 2$ with the following property: every $n$ pairwise distinct integers whose sum is not divisible by $n$ can be arranged in some order $a_{1}, a_{2}, \\ldots, a_{n}$ so that $n$ divides $1 \\cdot a_{1}+2 \\cdot a_{2}+\\cdots+n \\cdot a_{n}$. Answer: All odd integers and all powers of 2.","solution":"If $n=2^{k} a$, where $a \\geqslant 3$ is odd and $k$ is a positive integer, we can consider a set containing the number $2^{k}+1$ and $n-1$ numbers congruent to 1 modulo $n$. The sum of these numbers is congruent to $2^{k}$ modulo $n$ and therefore is not divisible by $n$; for any permutation $\\left(a_{1}, a_{2}, \\ldots, a_{n}\\right)$ of these numbers $$ 1 \\cdot a_{1}+2 \\cdot a_{2}+\\cdots+n \\cdot a_{n} \\equiv 1+\\cdots+n \\equiv 2^{k-1} a\\left(2^{k} a+1\\right) \\not \\equiv 0 \\quad\\left(\\bmod 2^{k}\\right) $$ and $a$ fortiori $1 \\cdot a_{1}+2 \\cdot a_{2}+\\cdots+n \\cdot a_{n}$ is not divisible by $n$. From now on, we suppose that $n$ is either odd or a power of 2 . Let $S$ be the given set of integers, and $s$ be the sum of elements of $S$. Lemma 1. If there is a permutation $\\left(a_{i}\\right)$ of $S$ such that $(n, s)$ divides $\\sum_{i=1}^{n} i a_{i}$, then there is a permutation $\\left(b_{i}\\right)$ of $S$ such that $n$ divides $\\sum_{i=1}^{n} i b_{i}$. Proof. Let $r=\\sum_{i=1}^{n} i a_{i}$. Consider the permutation $\\left(b_{i}\\right)$ defined by $b_{i}=a_{i+x}$, where $a_{j+n}=a_{j}$. For this permutation, we have $$ \\sum_{i=1}^{n} i b_{i}=\\sum_{i=1}^{n} i a_{i+x} \\equiv \\sum_{i=1}^{n}(i-x) a_{i} \\equiv r-s x \\quad(\\bmod n) $$ Since $(n, s)$ divides $r$, the congruence $r-s x \\equiv 0(\\bmod n)$ admits a solution. Lemma 2. Every set $T$ of $k m$ integers, $m>1$, can be partitioned into $m$ sets of $k$ integers so that in every set either the sum of elements is not divisible by $k$ or all the elements leave the same remainder upon division by $k$. Proof. The base case, $m=2$. If $T$ contains $k$ elements leaving the same remainder upon division by $k$, we form one subset $A$ of these elements; the remaining elements form a subset $B$. If $k$ does not divide the sum of all elements of $B$, we are done. Otherwise it is enough to exchange any element of $A$ with any element of $B$ not congruent to it modulo $k$, thus making sums of both $A$ and $B$ not divisible by $k$. This cannot be done only when all the elements of $T$ are congruent modulo $k$; in this case any partition will do. If no $k$ elements of $T$ have the same residue modulo $k$, there are three elements $a, b, c \\in T$ leaving pairwise distinct remainders upon division by $k$. Let $t$ be the sum of elements of $T$. It suffices to find $A \\subset T$ such that $|A|=k$ and $\\sum_{x \\in A} x \\not \\equiv 0, t(\\bmod k)$ : then neither the sum of elements of $A$ nor the sum of elements of $B=T \\backslash A$ is divisible by $k$. Consider $U^{\\prime} \\subset T \\backslash\\{a, b, c\\}$ with $\\left|U^{\\prime}\\right|=k-1$. The sums of elements of three sets $U^{\\prime} \\cup\\{a\\}, U^{\\prime} \\cup\\{b\\}, U^{\\prime} \\cup\\{c\\}$ leave three different remainders upon division by $k$, and at least one of them is not congruent either to 0 or to $t$. Now let $m>2$. If $T$ contains $k$ elements leaving the same remainder upon division by $k$, we form one subset $A$ of these elements and apply the inductive hypothesis to the remaining $k(m-1)$ elements. Otherwise, we choose any $U \\subset T,|U|=k-1$. Since all the remaining elements cannot be congruent modulo $k$, there is $a \\in T \\backslash U$ such that $a \\not \\equiv-\\sum_{x \\in U} x(\\bmod k)$. Now we can take $A=U \\cup\\{a\\}$ and apply the inductive hypothesis to $T \\backslash A$. Now we are ready to prove the statement of the problem for all odd $n$ and $n=2^{k}$. The proof is by induction. If $n$ is prime, the statement follows immediately from Lemma 1 , since in this case $(n, s)=1$. Turning to the general case, we can find prime $p$ and an integer $t$ such that $p^{t} \\mid n$ and $p^{t} \\nmid s$. By Lemma 2, we can partition $S$ into $p$ sets of $\\frac{n}{p}=k$ elements so that in every set either the sum of numbers is not divisible by $k$ or all numbers have the same residue modulo $k$. For sets in the first category, by the inductive hypothesis there is a permutation $\\left(a_{i}\\right)$ such that $k \\mid \\sum_{i=1}^{k} i a_{i}$. If $n$ (and therefore $k$ ) is odd, then for each permutation $\\left(b_{i}\\right)$ of a set in the second category we have $$ \\sum_{i=1}^{k} i b_{i} \\equiv b_{1} \\frac{k(k+1)}{2} \\equiv 0 \\quad(\\bmod k) $$ By combining such permutation for all sets of the partition, we get a permutation ( $c_{i}$ ) of $S$ such that $k \\mid \\sum_{i=1}^{n} i c_{i}$. Since this sum is divisible by $k$, and $k$ is divisible by $(n, s)$, we are done by Lemma 1 . If $n=2^{s}$, we have $p=2$ and $k=2^{s-1}$. Then for each of the subsets there is a permutation $\\left(a_{1}, \\ldots, a_{k}\\right)$ such that $\\sum_{i=1}^{k} i a_{i}$ is divisible by $2^{s-2}=\\frac{k}{2}$ : if the subset belongs to the first category, the expression is divisible even by $k$, and if it belongs to the second one, $$ \\sum_{i=1}^{k} i a_{i} \\equiv a_{1} \\frac{k(k+1)}{2} \\equiv 0\\left(\\bmod \\frac{k}{2}\\right) $$ Now the numbers of each permutation should be multiplied by all the odd or all the even numbers not exceeding $n$ in increasing order so that the resulting sums are divisible by $k$ : $$ \\sum_{i=1}^{k}(2 i-1) a_{i} \\equiv \\sum_{i=1}^{k} 2 i a_{i} \\equiv 2 \\sum_{i=1}^{k} i a_{i} \\equiv 0 \\quad(\\bmod k) $$ Combining these two sums, we again get a permutation $\\left(c_{i}\\right)$ of $S$ such that $k \\mid \\sum_{i=1}^{n} i c_{i}$, and finish the case by applying Lemma 1. Comment. We cannot dispense with the condition that $n$ does not divide the sum of all elements. Indeed, for each $n>1$ and the set consisting of $1,-1$, and $n-2$ elements divisible by $n$ the required permutation does not exist.","tier":0} +{"problem_type":"Number Theory","problem_label":"N8","problem":"For a polynomial $P(x)$ with integer coefficients let $P^{1}(x)=P(x)$ and $P^{k+1}(x)=$ $P\\left(P^{k}(x)\\right)$ for $k \\geqslant 1$. Find all positive integers $n$ for which there exists a polynomial $P(x)$ with integer coefficients such that for every integer $m \\geqslant 1$, the numbers $P^{m}(1), \\ldots, P^{m}(n)$ leave exactly $\\left\\lceil n \/ 2^{m}\\right\\rceil$ distinct remainders when divided by $n$. Answer: All powers of 2 and all primes.","solution":"Denote the set of residues modulo $\\ell$ by $\\mathbb{Z}_{\\ell}$. Observe that $P$ can be regarded as a function $\\mathbb{Z}_{\\ell} \\rightarrow \\mathbb{Z}_{\\ell}$ for any positive integer $\\ell$. Denote the cardinality of the set $P^{m}\\left(\\mathbb{Z}_{\\ell}\\right)$ by $f_{m, \\ell}$. Note that $f_{m, n}=\\left\\lceil n \/ 2^{m}\\right\\rceil$ for all $m \\geqslant 1$ if and only if $f_{m+1, n}=\\left\\lceil f_{m, n} \/ 2\\right\\rceil$ for all $m \\geqslant 0$. Part 1. The required polynomial exists when $n$ is a power of 2 or a prime. If $n$ is a power of 2 , set $P(x)=2 x$. If $n=p$ is an odd prime, every function $f: \\mathbb{Z}_{p} \\rightarrow \\mathbb{Z}_{p}$ coincides with some polynomial with integer coefficients. So we can pick the function that sends $x \\in\\{0,1, \\ldots, p-1\\}$ to $\\lfloor x \/ 2\\rfloor$. Part 2. The required polynomial does not exist when $n$ is not a prime power. Let $n=a b$ where $a, b>1$ and $\\operatorname{gcd}(a, b)=1$. Note that, since $\\operatorname{gcd}(a, b)=1$, $$ f_{m, a b}=f_{m, a} f_{m, b} $$ by the Chinese remainder theorem. Also, note that, if $f_{m, \\ell}=f_{m+1, \\ell}$, then $P$ permutes the image of $P^{m}$ on $\\mathbb{Z}_{\\ell}$, and therefore $f_{s, \\ell}=f_{m, \\ell}$ for all $s>m$. So, as $f_{m, a b}=1$ for sufficiently large $m$, we have for each $m$ $$ f_{m, a}>f_{m+1, a} \\quad \\text { or } \\quad f_{m, a}=1, \\quad f_{m, b}>f_{m+1, b} \\quad \\text { or } \\quad f_{m, b}=1 . $$ Choose the smallest $m$ such that $f_{m+1, a}=1$ or $f_{m+1, b}=1$. Without loss of generality assume that $f_{m+1, a}=1$. Then $f_{m+1, a b}=f_{m+1, b}1$. Let us choose the smallest $k$ for which this is so. To each residue in $P\\left(S_{r}\\right)$ we assign its residue modulo $p^{k-1}$; denote the resulting set by $\\bar{P}(S, r)$. We have $|\\bar{P}(S, r)|=p^{k-2}$ by virtue of minimality of $k$. Then $\\left|P\\left(S_{r}\\right)\\right|1$ and $a_{n+2}>1$. Since $\\left(a_{n+1}\\right)^{2}-1 \\leqslant\\left(1-a_{n}\\right)\\left(a_{n+2}-1\\right)$, we deduce that $0<1-a_{n}<1<1+a_{n+2}$, thus $\\left(a_{n+1}\\right)^{2}-1<\\left(a_{n+2}+1\\right)\\left(a_{n+2}-1\\right)=\\left(a_{n+2}\\right)^{2}-1$. On the other hand, $\\left(a_{n+2}\\right)^{2}-1 \\leqslant\\left(1-a_{n+3}\\right)\\left(a_{n+1}-1\\right)<\\left(1+a_{n+1}\\right)\\left(a_{n+1}-1\\right)=\\left(a_{n+1}\\right)^{2}-1$, a contradiction. We have shown that we cannot have two consecutive terms, except maybe $a_{1}$ and $a_{2}$, strictly greater than 1 . Finally, suppose $a_{2022}>1$. This implies that $a_{2021} \\leqslant 1$ and $a_{2023} \\leqslant 1$. Therefore $0<$ $\\left(a_{2022}\\right)^{2}-1 \\leqslant\\left(1-a_{2021}\\right)\\left(a_{2023}-1\\right) \\leqslant 0$, a contradiction. We conclude that $a_{2022} \\leqslant 1$.","tier":0} +{"problem_type":"Algebra","problem_label":"A2","problem":"Let $k \\geqslant 2$ be an integer. Find the smallest integer $n \\geqslant k+1$ with the property that there exists a set of $n$ distinct real numbers such that each of its elements can be written as a sum of $k$ other distinct elements of the set. (Slovakia)","solution":"First we show that $n \\geqslant k+4$. Suppose that there exists such a set with $n$ numbers and denote them by $a_{1}a_{1}+\\cdots+a_{k} \\geqslant a_{k+1}$, which gives a contradiction. If $n=k+2$ then we have $a_{1} \\geqslant a_{2}+\\cdots+a_{k+1} \\geqslant a_{k+2}$, that again gives a contradiction. If $n=k+3$ then we have $a_{1} \\geqslant a_{2}+\\cdots+a_{k+1}$ and $a_{3}+\\cdots+a_{k+2} \\geqslant a_{k+3}$. Adding the two inequalities we get $a_{1}+a_{k+2} \\geqslant a_{2}+a_{k+3}$, again a contradiction. It remains to give an example of a set with $k+4$ elements satisfying the condition of the problem. We start with the case when $k=2 l$ and $l \\geqslant 1$. In that case, denote by $A_{i}=\\{-i, i\\}$ and take the set $A_{1} \\cup \\cdots \\cup A_{l+2}$, which has exactly $k+4=2 l+4$ elements. We are left to show that this set satisfies the required condition. Note that if a number $i$ can be expressed in the desired way, then so can $-i$ by negating the expression. Therefore, we consider only $1 \\leqslant i \\leqslant l+2$. If $i0}$ be the set of positive real numbers. Find all functions $f: \\mathbb{R}_{>0} \\rightarrow \\mathbb{R}_{>0}$ such that, for every $x \\in \\mathbb{R}_{>0}$, there exists a unique $y \\in \\mathbb{R}_{>0}$ satisfying $$ x f(y)+y f(x) \\leqslant 2 $$ (Netherlands)","solution":"First we prove that the function $f(x)=1 \/ x$ satisfies the condition of the problem statement. The AM-GM inequality gives $$ \\frac{x}{y}+\\frac{y}{x} \\geqslant 2 $$ for every $x, y>0$, with equality if and only if $x=y$. This means that, for every $x>0$, there exists a unique $y>0$ such that $$ \\frac{x}{y}+\\frac{y}{x} \\leqslant 2, $$ namely $y=x$. Let now $f: \\mathbb{R}_{>0} \\rightarrow \\mathbb{R}_{>0}$ be a function that satisfies the condition of the problem statement. We say that a pair of positive real numbers $(x, y)$ is $\\operatorname{good}$ if $x f(y)+y f(x) \\leqslant 2$. Observe that if $(x, y)$ is good, then so is $(y, x)$. Lemma 1.0. If $(x, y)$ is good, then $x=y$. Proof. Assume that there exist positive real numbers $x \\neq y$ such that $(x, y)$ is good. The uniqueness assumption says that $y$ is the unique positive real number such that $(x, y)$ is good. In particular, $(x, x)$ is not a good pair. This means that $$ x f(x)+x f(x)>2 $$ and thus $x f(x)>1$. Similarly, $(y, x)$ is a good pair, so $(y, y)$ is not a good pair, which implies $y f(y)>1$. We apply the AM-GM inequality to obtain $$ x f(y)+y f(x) \\geqslant 2 \\sqrt{x f(y) \\cdot y f(x)}=2 \\sqrt{x f(x) \\cdot y f(y)}>2 . $$ This is a contradiction, since $(x, y)$ is a good pair. By assumption, for any $x>0$, there always exists a good pair containing $x$, however Lemma 1 implies that the only good pair that can contain $x$ is $(x, x)$, so $$ x f(x) \\leqslant 1 \\quad \\Longleftrightarrow \\quad f(x) \\leqslant \\frac{1}{x} $$ for every $x>0$. In particular, with $x=1 \/ f(t)$ for $t>0$, we obtain $$ \\frac{1}{f(t)} \\cdot f\\left(\\frac{1}{f(t)}\\right) \\leqslant 1 $$ Hence $$ t \\cdot f\\left(\\frac{1}{f(t)}\\right) \\leqslant t f(t) \\leqslant 1 $$ We claim that $(t, 1 \/ f(t))$ is a good pair for every $t>0$. Indeed, $$ t \\cdot f\\left(\\frac{1}{f(t)}\\right)+\\frac{1}{f(t)} f(t)=t \\cdot f\\left(\\frac{1}{f(t)}\\right)+1 \\leqslant 2 $$ Lemma 1 implies that $t=1 \/ f(t) \\Longleftrightarrow f(t)=1 \/ t$ for every $t>0$.","tier":0} +{"problem_type":"Algebra","problem_label":"A3","problem":"Let $\\mathbb{R}_{>0}$ be the set of positive real numbers. Find all functions $f: \\mathbb{R}_{>0} \\rightarrow \\mathbb{R}_{>0}$ such that, for every $x \\in \\mathbb{R}_{>0}$, there exists a unique $y \\in \\mathbb{R}_{>0}$ satisfying $$ x f(y)+y f(x) \\leqslant 2 $$ (Netherlands)","solution":"1. We give an alternative way to prove that $f(x)=1 \/ x$ assuming $f(x) \\leqslant 1 \/ x$ for every $x>0$. Indeed, if $f(x)<1 \/ x$ then for every $a>0$ with $f(x)<1 \/ a<1 \/ x$ (and there are at least two of them), we have $$ a f(x)+x f(a)<1+\\frac{x}{a}<2 $$ Hence $(x, a)$ is a good pair for every such $a$, a contradiction. We conclude that $f(x)=1 \/ x$.","tier":0} +{"problem_type":"Algebra","problem_label":"A3","problem":"Let $\\mathbb{R}_{>0}$ be the set of positive real numbers. Find all functions $f: \\mathbb{R}_{>0} \\rightarrow \\mathbb{R}_{>0}$ such that, for every $x \\in \\mathbb{R}_{>0}$, there exists a unique $y \\in \\mathbb{R}_{>0}$ satisfying $$ x f(y)+y f(x) \\leqslant 2 $$ (Netherlands)","solution":"2. We can also conclude from Lemma 1 and $f(x) \\leqslant 1 \/ x$ as follows. Lemma 2. The function $f$ is decreasing. Proof. Let $y>x>0$. Lemma 1 says that $(x, y)$ is not a good pair, but $(y, y)$ is. Hence $$ x f(y)+y f(x)>2 \\geqslant 2 y f(y)>y f(y)+x f(y) $$ where we used $y>x$ (and $f(y)>0$ ) in the last inequality. This implies that $f(x)>f(y)$, showing that $f$ is decreasing. We now prove that $f(x)=1 \/ x$ for all $x$. Fix a value of $x$ and note that for $y>x$ we must have $x f(x)+y f(x)>x f(y)+y f(x)>2$ (using that $f$ is decreasing for the first step), hence $f(x)>\\frac{2}{x+y}$. The last inequality is true for every $y>x>0$. If we fix $x$ and look for the supremum of the expression $\\frac{2}{x+y}$ over all $y>x$, we get $$ f(x) \\geqslant \\frac{2}{x+x}=\\frac{1}{x} $$ Since we already know that $f(x) \\leqslant 1 \/ x$, we conclude that $f(x)=1 \/ x$.","tier":0} +{"problem_type":"Algebra","problem_label":"A3","problem":"Let $\\mathbb{R}_{>0}$ be the set of positive real numbers. Find all functions $f: \\mathbb{R}_{>0} \\rightarrow \\mathbb{R}_{>0}$ such that, for every $x \\in \\mathbb{R}_{>0}$, there exists a unique $y \\in \\mathbb{R}_{>0}$ satisfying $$ x f(y)+y f(x) \\leqslant 2 $$ (Netherlands)","solution":"0. As in the first solution, we note that $f(x)=1 \/ x$ is a solution, and we set out to prove that it is the only one. We write $g(x)$ for the unique positive real number such that $(x, g(x))$ is a good pair. In this solution, we prove Lemma 2 without assuming Lemma 1. Lemma 2. The function $f$ is decreasing. Proof. Consider $x$ 2. Combining these two inequalities yields $$ x f(g(y))+g(y) f(x)>2 \\geqslant y f(g(y))+g(y) f(y) $$ or $f(g(y))(x-y)>g(y)(f(y)-f(x))$. Because $g(y)$ and $f(g(y))$ are both positive while $x-y$ is negative, it follows that $f(y)2$ and $y f(y)+y f(y)>2$, which implies $x f(x)+y f(y)>2$. Now $$ x f(x)+y f(y)>2 \\geqslant x f(y)+y f(x) $$ implies $(x-y)(f(x)-f(y))>0$, which contradicts the fact that $f$ is decreasing. So $y=x$ is the unique $y$ such that $(x, y)$ is a good pair, and in particular we have $f(x) \\leqslant 1 \/ x$. We can now conclude the proof as in any of the Solutions 1.x.","tier":0} +{"problem_type":"Algebra","problem_label":"A3","problem":"Let $\\mathbb{R}_{>0}$ be the set of positive real numbers. Find all functions $f: \\mathbb{R}_{>0} \\rightarrow \\mathbb{R}_{>0}$ such that, for every $x \\in \\mathbb{R}_{>0}$, there exists a unique $y \\in \\mathbb{R}_{>0}$ satisfying $$ x f(y)+y f(x) \\leqslant 2 $$ (Netherlands)","solution":"0. As in the other solutions we verify that the function $f(x)=1 \/ x$ is a solution. We first want to prove the following lemma: Lemma 3. For all $x \\in \\mathbb{R}_{>0}$ we actually have $x f(g(x))+g(x) f(x)=2$ (that is: the inequality is actually an equality). Proof. We proceed by contradiction: Assume there exists some number $x>0$ such that for $y=g(x)$ we have $x f(y)+y f(x)<2$. Then for any $0<\\epsilon<\\frac{2-x f(y)-y f(x)}{2 f(x)}$ we have, by uniqueness of $y$, that $x f(y+\\epsilon)+(y+\\epsilon) f(x)>2$. Therefore $$ \\begin{aligned} f(y+\\epsilon) & >\\frac{2-(y+\\epsilon) f(x)}{x}=\\frac{2-y f(x)-\\epsilon f(x)}{x} \\\\ & >\\frac{2-y f(x)-\\frac{2-x f(y)-y f(x)}{2}}{x} \\\\ & =\\frac{2-x f(y)-y f(x)}{2 x}+f(y)>f(y) . \\end{aligned} $$ Furthermore, for every such $\\epsilon$ we have $g(y+\\epsilon) f(y+\\epsilon)+(y+\\epsilon) f(g(y+\\epsilon)) \\leqslant 2$ and $g(y+\\epsilon) f(y)+y f(g(y+\\epsilon))>2($ since $y \\neq y+\\epsilon=g(g(y+\\epsilon))$ ). This gives us the two inequalities $$ f(g(y+\\epsilon)) \\leqslant \\frac{2-g(y+\\epsilon) f(y+\\epsilon)}{y+\\epsilon} \\quad \\text { and } \\quad f(g(y+\\epsilon))>\\frac{2-g(y+\\epsilon) f(y)}{y} . $$ Combining these two inequalities and rearranging the terms leads to the inequality $$ 2 \\epsilon0}$ we have $$ x f(y)+y f(x) \\geqslant 2 $$ since for $y=g(x)$ we have equality and by uniqueness for $y \\neq g(x)$ the inequality is strict. In particular for every $x \\in \\mathbb{R}_{>0}$ and for $y=x$ we have $2 x f(x) \\geqslant 2$, or equivalently $f(x) \\geqslant 1 \/ x$ for all $x \\in \\mathbb{R}_{>0}$. With this inequality we obtain for all $x \\in \\mathbb{R}_{>0}$ $$ 2 \\geqslant x f(g(x))+g(x) f(x) \\geqslant \\frac{x}{g(x)}+\\frac{g(x)}{x} \\geqslant 2 $$ where the first inequality comes from the problem statement. Consequently each of these inequalities must actually be an equality, and in particular we obtain $f(x)=1 \/ x$ for all $x \\in \\mathbb{R}_{>0}$.","tier":0} +{"problem_type":"Algebra","problem_label":"A3","problem":"Let $\\mathbb{R}_{>0}$ be the set of positive real numbers. Find all functions $f: \\mathbb{R}_{>0} \\rightarrow \\mathbb{R}_{>0}$ such that, for every $x \\in \\mathbb{R}_{>0}$, there exists a unique $y \\in \\mathbb{R}_{>0}$ satisfying $$ x f(y)+y f(x) \\leqslant 2 $$ (Netherlands)","solution":"Again, let us prove that $f(x)=1 \/ x$ is the only solution. Let again $g(x)$ be the unique positive real number such that $(x, g(x))$ is a good pair. Lemma 4. The function $f$ is strictly convex. Proof. Consider the function $q_{s}(x)=f(x)+s x$ for some real number $s$. If $f$ is not strictly convex, then there exist $u2^{s-3} $$ (Trinidad and Tobago)","solution":"Let $1 \\leqslant ac 2^{s} . $$ The above shows that $c=1 \/ 8$ is the best possible. A somewhat simpler ending to the proof can be given for $c=1 \/ 32$. End of solution for $c=1 \/ 32$. As in the original solution, we arrive at $$ s<2^{u+1} x_{a}+2^{v+1} x_{b}+((b-v)-(a+u)-1)=b-a+\\left(2^{u+1-\\alpha}+2^{v+1-\\beta}-(u+v+1)\\right) . $$ Now $2^{b-a} x_{a} x_{b} \\geqslant 2^{b-a} 2^{-u-1} 2^{-v-1}$, so it is enough to show $s-50$ such that the $\\frac{1}{2} n(n-1)$ differences $a_{j}-a_{i}$ for $1 \\leqslant i1$ of $x^{2}-x-1=0$ (the golden ratio) and set $\\left(a_{1}, a_{2}, a_{3}\\right)=$ $\\left(0, r, r+r^{2}\\right)$. Then $$ \\left(a_{2}-a_{1}, a_{3}-a_{2}, a_{3}-a_{1}\\right)=\\left(r, r^{2}, r+r^{2}=r^{3}\\right) $$ For $n=4$, take the root $r \\in(1,2)$ of $x^{3}-x-1=0$ (such a root exists because $1^{3}-1-1<0$ and $\\left.2^{3}-2-1>0\\right)$ and set $\\left(a_{1}, a_{2}, a_{3}, a_{4}\\right)=\\left(0, r, r+r^{2}, r+r^{2}+r^{3}\\right)$. Then $$ \\left(a_{2}-a_{1}, a_{3}-a_{2}, a_{4}-a_{3}, a_{3}-a_{1}, a_{4}-a_{2}, a_{4}-a_{1}\\right)=\\left(r, r^{2}, r^{3}, r^{4}, r^{5}, r^{6}\\right) $$ For $n \\geqslant 5$, we will proceed by contradiction. Suppose there exist numbers $a_{1}<\\cdots1$ satisfying the conditions of the problem. We start with a lemma: Lemma. We have $r^{n-1}>2$. Proof. There are only $n-1$ differences $a_{j}-a_{i}$ with $j=i+1$, so there exists an exponent $e \\leqslant n$ and a difference $a_{j}-a_{i}$ with $j \\geqslant i+2$ such that $a_{j}-a_{i}=r^{e}$. This implies $$ r^{n} \\geqslant r^{e}=a_{j}-a_{i}=\\left(a_{j}-a_{j-1}\\right)+\\left(a_{j-1}-a_{i}\\right)>r+r=2 r, $$ thus $r^{n-1}>2$ as desired. To illustrate the general approach, we first briefly sketch the idea behind the argument in the special case $n=5$. In this case, we clearly have $a_{5}-a_{1}=r^{10}$. Note that there are 3 ways to rewrite $a_{5}-a_{1}$ as a sum of two differences, namely $$ \\left(a_{5}-a_{4}\\right)+\\left(a_{4}-a_{1}\\right),\\left(a_{5}-a_{3}\\right)+\\left(a_{3}-a_{1}\\right),\\left(a_{5}-a_{2}\\right)+\\left(a_{2}-a_{1}\\right) . $$ Using the lemma above and convexity of the function $f(n)=r^{n}$, we argue that those three ways must be $r^{10}=r^{9}+r^{1}=r^{8}+r^{4}=r^{7}+r^{6}$. That is, the \"large\" exponents keep dropping by 1 , while the \"small\" exponents keep increasing by $n-2, n-3, \\ldots, 2$. Comparing any two such equations, we then get a contradiction unless $n \\leqslant 4$. Now we go back to the full proof for any $n \\geqslant 5$. Denote $b=\\frac{1}{2} n(n-1)$. Clearly, we have $a_{n}-a_{1}=r^{b}$. Consider the $n-2$ equations of the form: $$ a_{n}-a_{1}=\\left(a_{n}-a_{i}\\right)+\\left(a_{i}-a_{1}\\right) \\text { for } i \\in\\{2, \\ldots, n-1\\} $$ In each equation, one of the two terms on the right-hand side must be at least $\\frac{1}{2}\\left(a_{n}-a_{1}\\right)$. But from the lemma we have $r^{b-(n-1)}=r^{b} \/ r^{n-1}<\\frac{1}{2}\\left(a_{n}-a_{1}\\right)$, so there are at most $n-2$ sufficiently large elements in $\\left\\{r^{k} \\mid 1 \\leqslant k1$ and $f(r)=r^{n}$ is convex, we have $$ r^{b-1}-r^{b-2}>r^{b-2}-r^{b-3}>\\ldots>r^{b-(n-3)}-r^{b-(n-2)} $$ implying $$ r^{\\alpha_{2}}-r^{\\alpha_{1}}>r^{\\alpha_{3}}-r^{\\alpha_{2}}>\\ldots>r^{\\alpha_{n-2}}-r^{\\alpha_{n-3}} . $$ Convexity of $f(r)=r^{n}$ further implies $$ \\alpha_{2}-\\alpha_{1}>\\alpha_{3}-\\alpha_{2}>\\ldots>\\alpha_{n-2}-\\alpha_{n-3} $$ Note that $\\alpha_{n-2}-\\alpha_{n-3} \\geqslant 2$ : Otherwise we would have $\\alpha_{n-2}-\\alpha_{n-3}=1$ and thus $$ r^{\\alpha_{n-3}} \\cdot(r-1)=r^{\\alpha_{n-2}}-r^{\\alpha_{n-3}}=r^{b-(n-3)}-r^{b-(n-2)}=r^{b-(n-2)} \\cdot(r-1), $$ implying that $\\alpha_{n-3}=b-(n-2)$, a contradiction. Therefore, we have $$ \\begin{aligned} \\alpha_{n-2}-\\alpha_{1} & =\\left(\\alpha_{n-2}-\\alpha_{n-3}\\right)+\\cdots+\\left(\\alpha_{2}-\\alpha_{1}\\right) \\\\ & \\geqslant 2+3+\\cdots+(n-2) \\\\ & =\\frac{1}{2}(n-2)(n-1)-1=\\frac{1}{2} n(n-3) \\end{aligned} $$ On the other hand, from $\\alpha_{n-2} \\leqslant b-(n-1)$ and $\\alpha_{1} \\geqslant 1$ we get $$ \\alpha_{n-2}-\\alpha_{1} \\leqslant b-n=\\frac{1}{2} n(n-1)-n=\\frac{1}{2} n(n-3), $$ implying that equalities must occur everywhere and the claim about the small terms follows. Now, assuming $n-2 \\geqslant 2$, we have the two different equations: $$ r^{b}=r^{b-(n-2)}+r^{b-(n-2)-1} \\text { and } r^{b}=r^{b-(n-3)}+r^{b-(n-2)-3}, $$ which can be rewritten as $$ r^{n-1}=r+1 \\quad \\text { and } \\quad r^{n+1}=r^{4}+1 $$ Simple algebra now gives $$ r^{4}+1=r^{n+1}=r^{n-1} \\cdot r^{2}=r^{3}+r^{2} \\Longrightarrow(r-1)\\left(r^{3}-r-1\\right)=0 . $$ Since $r \\neq 1$, using Equation (1) we conclude $r^{3}=r+1=r^{n-1}$, thus $n=4$, which gives a contradiction.","tier":0} +{"problem_type":"Algebra","problem_label":"A6","problem":"Let $\\mathbb{R}$ be the set of real numbers. We denote by $\\mathcal{F}$ the set of all functions $f: \\mathbb{R} \\rightarrow \\mathbb{R}$ such that $$ f(x+f(y))=f(x)+f(y) $$ for every $x, y \\in \\mathbb{R}$. Find all rational numbers $q$ such that for every function $f \\in \\mathcal{F}$, there exists some $z \\in \\mathbb{R}$ satisfying $f(z)=q z$. (Indonesia)","solution":"Let $Z$ be the set of all rational numbers $q$ such that for every function $f \\in \\mathcal{F}$, there exists some $z \\in \\mathbb{R}$ satisfying $f(z)=q z$. Let further $$ S=\\left\\{\\frac{n+1}{n}: n \\in \\mathbb{Z}, n \\neq 0\\right\\} $$ We prove that $Z=S$ by showing the two inclusions: $S \\subseteq Z$ and $Z \\subseteq S$. We first prove that $S \\subseteq Z$. Let $f \\in \\mathcal{F}$ and let $P(x, y)$ be the relation $f(x+f(y))=f(x)+$ $f(y)$. First note that $P(0,0)$ gives $f(f(0))=2 f(0)$. Then, $P(0, f(0))$ gives $f(2 f(0))=3 f(0)$. We claim that $$ f(k f(0))=(k+1) f(0) $$ for every integer $k \\geqslant 1$. The claim can be proved by induction. The cases $k=1$ and $k=2$ have already been established. Assume that $f(k f(0))=(k+1) f(0)$ and consider $P(0, k f(0))$ which gives $$ f((k+1) f(0))=f(0)+f(k f(0))=(k+2) f(0) $$ This proves the claim. We conclude that $\\frac{k+1}{k} \\in Z$ for every integer $k \\geqslant 1$. Note that $P(-f(0), 0)$ gives $f(-f(0))=0$. We now claim that $$ f(-k f(0))=(-k+1) f(0) $$ for every integer $k \\geqslant 1$. The proof by induction is similar to the one above. We conclude that $\\frac{-k+1}{-k} \\in Z$ for every integer $k \\geqslant 1$. This shows that $S \\subseteq Z$. We now prove that $Z \\subseteq S$. Let $p$ be a rational number outside the set $S$. We want to prove that $p$ does not belong to $Z$. To that end, we construct a function $f \\in \\mathcal{F}$ such that $f(z) \\neq p z$ for every $z \\in \\mathbb{R}$. The strategy is to first construct a function $$ g:[0,1) \\rightarrow \\mathbb{Z} $$ and then define $f$ as $f(x)=g(\\{x\\})+\\lfloor x\\rfloor$. This function $f$ belongs to $\\mathcal{F}$. Indeed, $$ \\begin{aligned} f(x+f(y)) & =g(\\{x+f(y)\\})+\\lfloor x+f(y)\\rfloor \\\\ & =g(\\{x+g(\\{y\\})+\\lfloor y\\rfloor\\})+\\lfloor x+g(\\{y\\})+\\lfloor y\\rfloor\\rfloor \\\\ & =g(\\{x\\})+\\lfloor x\\rfloor+g(\\{y\\})+\\lfloor y\\rfloor \\\\ & =f(x)+f(y) \\end{aligned} $$ where we used that $g$ only takes integer values. Lemma 1. For every $\\alpha \\in[0,1)$, there exists $m \\in \\mathbb{Z}$ such that $$ m+n \\neq p(\\alpha+n) $$ for every $n \\in \\mathbb{Z}$. Proof. Note that if $p=1$ the claim is trivial. If $p \\neq 1$, then the claim is equivalent to the existence of an integer $m$ such that $$ \\frac{m-p \\alpha}{p-1} $$ is never an integer. Assume the contrary. That would mean that both $$ \\frac{m-p \\alpha}{p-1} \\quad \\text { and } \\quad \\frac{(m+1)-p \\alpha}{p-1} $$ are integers, and so is their difference. The latter is equal to $$ \\frac{1}{p-1} $$ Since we assumed $p \\notin S, 1 \/(p-1)$ is never an integer. This is a contradiction. Define $g:[0,1) \\rightarrow \\mathbb{Z}$ by $g(\\alpha)=m$ for any integer $m$ that satisfies the conclusion of Lemma 1. Note that $f(z) \\neq p z$ if and and only if $$ g(\\{z\\})+\\lfloor z\\rfloor \\neq p(\\{z\\}+\\lfloor z\\rfloor) $$ The latter is guaranteed by the construction of the function $g$. We conclude that $p \\notin Z$ as desired. This shows that $Z \\subset S$.","tier":0} +{"problem_type":"Algebra","problem_label":"A7","problem":"For a positive integer $n$ we denote by $s(n)$ the sum of the digits of $n$. Let $P(x)=$ $x^{n}+a_{n-1} x^{n-1}+\\cdots+a_{1} x+a_{0}$ be a polynomial, where $n \\geqslant 2$ and $a_{i}$ is a positive integer for all $0 \\leqslant i \\leqslant n-1$. Could it be the case that, for all positive integers $k, s(k)$ and $s(P(k))$ have the same parity? (Belarus)","solution":"With the notation above, we begin by choosing a positive integer $t$ such that $$ 10^{t}>\\max \\left\\{\\frac{100^{n-1} a_{n-1}}{\\left(10^{\\frac{1}{n-1}}-9^{\\frac{1}{n-1}}\\right)^{n-1}}, \\frac{a_{n-1}}{9} 10^{n-1}, \\frac{a_{n-1}}{9}\\left(10 a_{n-1}\\right)^{n-1}, \\ldots, \\frac{a_{n-1}}{9}\\left(10 a_{0}\\right)^{n-1}\\right\\} $$ As a direct consequence of $10^{t}$ being bigger than the first quantity listed in the above set, we get that the interval $$ I=\\left[\\left(\\frac{9}{a_{n-1}} 10^{t}\\right)^{\\frac{1}{n-1}},\\left(\\frac{1}{a_{n-1}} 10^{t+1}\\right)^{\\frac{1}{n-1}}\\right) $$ contains at least 100 consecutive positive integers. Let $X$ be a positive integer in $I$ such that $X$ is congruent to $1 \\bmod 100$. Since $X \\in I$ we have $$ 9 \\cdot 10^{t} \\leqslant a_{n-1} X^{n-1}<10^{t+1} $$ thus the first digit (from the left) of $a_{n-1} X^{n-1}$ must be 9 . Next, we observe that $a_{n-1}\\left(10 a_{i}\\right)^{n-1}<9 \\cdot 10^{t} \\leqslant a_{n-1} X^{n-1}$, thus $10 a_{i}10^{\\alpha i} a_{n-1} X^{n-1}>$ $10^{\\alpha i} a_{i} X^{i}$, the terms $a_{i} 10^{\\alpha i} X^{i}$ do not interact when added; in particular, there is no carryover caused by addition. Thus we have $s\\left(P\\left(10^{\\alpha} X\\right)\\right)=s\\left(X^{n}\\right)+s\\left(a_{n-1} X^{n-1}\\right)+\\cdots+s\\left(a_{0}\\right)$. We now look at $P\\left(10^{\\alpha-1} X\\right)=10^{(\\alpha-1) n} X^{n}+a_{n-1} 10^{(\\alpha-1)(n-1)} X^{n-1}+\\cdots+a_{0}$. Firstly, if $i10^{(\\alpha-1) i} a_{n-1} X^{n-1} \\geqslant 10^{(\\alpha-1) i+1} a_{i} X^{i}$, thus all terms $10^{(\\alpha-1) i} a_{i} X^{i}$ for $0 \\leqslant i \\leqslant n-1$ come in 'blocks', exactly as in the previous case. Finally, $10^{(\\alpha-1) n+1}>10^{(\\alpha-1)(n-1)} a_{n-1} X^{n-1} \\geqslant 10^{(\\alpha-1) n}$, thus $10^{(\\alpha-1)(n-1)} a_{n-1} X^{n-1}$ has exactly $(\\alpha-1) n+1$ digits, and its first digit is 9 , as established above. On the other hand, $10^{(\\alpha-1) n} X^{n}$ has exactly $(\\alpha-1) n$ zeros, followed by $01($ as $X$ is $1 \\bmod 100)$. Therefore, when we add the terms, the 9 and 1 turn into 0 , the 0 turns into 1 , and nothing else is affected. Putting everything together, we obtain $$ s\\left(P\\left(10^{\\alpha-1} X\\right)\\right)=s\\left(X^{n}\\right)+s\\left(a_{n-1} X^{n-1}\\right)+\\cdots+s\\left(a_{0}\\right)-9=s\\left(P\\left(10^{\\alpha} X\\right)\\right)-9 $$ thus $s\\left(P\\left(10^{\\alpha} X\\right)\\right)$ and $s\\left(P\\left(10^{\\alpha-1} X\\right)\\right)$ have different parities, as claimed.","tier":0} +{"problem_type":"Algebra","problem_label":"A8","problem":"For a positive integer $n$, an $n$-sequence is a sequence $\\left(a_{0}, \\ldots, a_{n}\\right)$ of non-negative integers satisfying the following condition: if $i$ and $j$ are non-negative integers with $i+j \\leqslant n$, then $a_{i}+a_{j} \\leqslant n$ and $a_{a_{i}+a_{j}}=a_{i+j}$. Let $f(n)$ be the number of $n$-sequences. Prove that there exist positive real numbers $c_{1}, c_{2}$ and $\\lambda$ such that $$ c_{1} \\lambda^{n}k$ for some $i$, and small if no such $i$ exists. For now we will assume that $\\left(a_{i}\\right)$ is not the identity sequence (in other words, that $a_{i} \\neq i$ for some $i$ ). Lemma 1. If $a_{r}=a_{s}$ and $r, sr$, and let $d$ be the minimum positive integer such that $a_{r+d}=a_{r}$. Then 1. The subsequence $\\left(a_{r}, a_{r+1}, \\ldots, a_{n}\\right)$ is periodic with minimal period $d$. That is, for $uv_{0}$. Thus $u_{0}=v_{0}$, so $d \\mid u-v$. 2. If $r=0$ there is nothing to prove. Otherwise $a_{0}=a_{2 a_{0}}$ so $2 a_{0}=0$. Then we have $a_{a_{i}}=a_{i}$ for all $i$, so $a_{i}=i$ for $ik+1$. We show that $a_{i} \\leqslant k$ for all $i$ by induction. Note that Lemma 2 already establishes this for $i \\leqslant k$. We must have $d \\mid a_{d \/ 2}$ and $a_{d \/ 2} \\leqslant kk$, if $a_{j} \\leqslant k$ for $jk+1$ and $a_{i}=i$ for all $0 \\leqslant ik$. We already have $a_{i}=i$ for $id$, this means that $a_{i}=i$ for $i \\leqslant k$. Finally, one can show inductively that $a_{i}=i$ for $kc_{1} \\lambda^{n}$ for some $c_{1}$, we note that $$ f(n)>g\\left(k+1,\\left\\lfloor\\frac{k+1}{3}\\right\\rfloor\\right) \\geqslant 3^{\\lfloor(k+1) \/ 3\\rfloor}>3^{n \/ 6}-1 . $$ To show that $f(n)2 \\cdot 3^{-1 \/ 6} \\approx 1.66537$. With a careful analysis one can show that the best possible value of $c_{2}$ is $\\frac{236567}{4930} 3^{1 \/ 3} \\approx$ 69.20662 .","tier":0} +{"problem_type":"Combinatorics","problem_label":"C1","problem":"A $\\pm 1$-sequence is a sequence of 2022 numbers $a_{1}, \\ldots, a_{2022}$, each equal to either +1 or -1 . Determine the largest $C$ so that, for any $\\pm 1$-sequence, there exists an integer $k$ and indices $1 \\leqslant t_{1}<\\ldots\\frac{3 n+1}{2}$, let $a=k-n-1, b=2 n-k+1$. Then $k>2 a+b, k>2 b+a$, so the configuration $A^{a} C^{b} A^{b} C^{a}$ will always have four blocks: $$ A^{a} C^{b} A^{b} C^{a} \\rightarrow C^{a} A^{a} C^{b} A^{b} \\rightarrow A^{b} C^{a} A^{a} C^{b} \\rightarrow C^{b} A^{b} C^{a} A^{a} \\rightarrow A^{a} C^{b} A^{b} C^{a} \\rightarrow \\ldots $$ Therefore a pair $(n, k)$ can have the desired property only if $n \\leqslant k \\leqslant \\frac{3 n+1}{2}$. We claim that all such pairs in fact do have the desired property. Clearly, the number of blocks in a configuration cannot increase, so whenever the operation is applied, it either decreases or remains constant. We show that unless there are only two blocks, after a finite amount of steps the number of blocks will decrease. Consider an arbitrary configuration with $c \\geqslant 3$ blocks. We note that as $k \\geqslant n$, the leftmost block cannot be moved, because in this case all $n$ coins of one type are in the leftmost block, meaning there are only two blocks. If a block which is not the leftmost or rightmost block is moved, its neighbor blocks will be merged, causing the number of blocks to decrease. Hence the only case in which the number of blocks does not decrease in the next step is if the rightmost block is moved. If $c$ is odd, the leftmost and the rightmost blocks are made of the same metal, so this would merge two blocks. Hence $c \\geqslant 4$ must be even. Suppose there is a configuration of $c$ blocks with the $i$-th block having size $a_{i}$ so that the operation always moves the rightmost block: $$ A^{a_{1}} \\ldots A^{a_{c-1}} C^{a_{c}} \\rightarrow C^{a_{c}} A^{a_{1}} \\ldots A^{a_{c-1}} \\rightarrow A^{a_{c-1}} C^{a_{c}} A^{a_{1}} \\ldots C^{a_{c-2}} \\rightarrow \\ldots $$ Because the rightmost block is always moved, $k \\geqslant 2 n+1-a_{i}$ for all $i$. Because $\\sum a_{i}=2 n$, summing this over all $i$ we get $c k \\geqslant 2 c n+c-\\sum a_{i}=2 c n+c-2 n$, so $k \\geqslant 2 n+1-\\frac{2 n}{c} \\geqslant \\frac{3 n}{2}+1$. But this contradicts $k \\leqslant \\frac{3 n+1}{2}$. Hence at some point the operation will not move the rightmost block, meaning that the number of blocks will decrease, as desired.","tier":0} +{"problem_type":"Combinatorics","problem_label":"C3","problem":"In each square of a garden shaped like a $2022 \\times 2022$ board, there is initially a tree of height 0 . A gardener and a lumberjack alternate turns playing the following game, with the gardener taking the first turn: - The gardener chooses a square in the garden. Each tree on that square and all the surrounding squares (of which there are at most eight) then becomes one unit taller. - The lumberjack then chooses four different squares on the board. Each tree of positive height on those squares then becomes one unit shorter. We say that a tree is majestic if its height is at least $10^{6}$. Determine the largest number $K$ such that the gardener can ensure there are eventually $K$ majestic trees on the board, no matter how the lumberjack plays. (Colombia)","solution":"We solve the problem for a general $3 N \\times 3 N$ board. First, we prove that the lumberjack has a strategy to ensure there are never more than $5 N^{2}$ majestic trees. Giving the squares of the board coordinates in the natural manner, colour each square where at least one of its coordinates are divisible by 3 , shown below for a $9 \\times 9$ board: ![](https:\/\/cdn.mathpix.com\/cropped\/2024_11_18_86efb4389f14b52b08e1g-28.jpg?height=900&width=914&top_left_y=1343&top_left_x=571) Then, as each $3 \\times 3$ square on the board contains exactly 5 coloured squares, each move of the gardener will cause at most 4 trees on non-coloured squares to grow. The lumberjack may therefore cut those trees, ensuring no tree on a non-coloured square has positive height after his turn. Hence there cannot ever be more majestic trees than coloured squares, which is $5 N^{2}$. Next, we prove the gardener may ensure there are $5 N^{2}$ majestic trees. In fact, we prove this statement in a modified game which is more difficult for the gardener: on the lumberjack's turn in the modified game, he may decrement the height of all trees on the board except those the gardener did not just grow, in addition to four of the trees the gardener just grew. Clearly, a sequence of moves for the gardener which ensures that there are $K$ majestic trees in the modified game also ensures this in the original game. Let $M=\\binom{9}{5}$; we say that a map is one of the $M$ possible ways to mark 5 squares on a $3 \\times 3$ board. In the modified game, after the gardener chooses a $3 \\times 3$ subboard on the board, the lumberjack chooses a map in this subboard, and the total result of the two moves is that each tree marked on the map increases its height by 1 , each tree in the subboard which is not in the map remains unchanged, and each tree outside the subboard decreases its height by 1. Also note that if the gardener chooses a $3 \\times 3$ subboard $M l$ times, the lumberjack will have to choose some map at least $l$ times, so there will be at least 5 trees which each have height $\\geqslant l$. The strategy for the gardener will be to divide the board into $N^{2}$ disjoint $3 \\times 3$ subboards, number them $0, \\ldots, N^{2}-1$ in some order. Then, for $b=N^{2}-1, \\ldots, 0$ in order, he plays $10^{6} M(M+1)^{b}$ times on subboard number $b$. Hence, on subboard number $b$, the moves on that subboard will first ensure 5 of its trees grows by at least $10^{6}(M+1)^{b}$, and then each move after that will decrease their heights by 1 . (As the trees on subboard $b$ had height 0 before the gardener started playing there, no move made on subboards $\\geqslant b$ decreased their heights.) As the gardener makes $10^{6} M(M+1)^{b-1}+\\ldots=10^{6}\\left((M+1)^{b}-1\\right)$ moves after he finishes playing on subboard $b$, this means that on subboard $b$, there will be 5 trees of height at least $10^{6}(M+1)^{b}-10^{6}\\left((M+1)^{b}-1\\right)=10^{6}$, hence each of the subboard has 5 majestic trees, which was what we wanted.","tier":0} +{"problem_type":"Combinatorics","problem_label":"C4","problem":"Let $n>3$ be a positive integer. Suppose that $n$ children are arranged in a circle, and $n$ coins are distributed between them (some children may have no coins). At every step, a child with at least 2 coins may give 1 coin to each of their immediate neighbours on the right and left. Determine all initial distributions of coins from which it is possible that, after a finite number of steps, each child has exactly one coin.","solution":"Number the children $1, \\ldots, n$, and denote the number of coins the $i$-th child has by $c_{i}$. A step of this process consists of reducing some $c_{i}$ by 2 , and increasing $c_{i-1}, c_{i+1}$ by 1 . (Indices are considered $(\\bmod n)$.) Because $(i-1)-2 i+(i+1)=0$, the quantity $\\sum_{i=1}^{n} i c_{i}(\\bmod n)$ will be invariant under this process. Hence a necessary condition for the children to end up with an uniform distribution of coins is that $$ \\sum_{i=1}^{n} i c_{i}=\\frac{n(n+1)}{2} \\quad(\\bmod n) $$ We will show that this condition is also sufficient. Consider an arbitrary initial distribution of coins. First, whenever child $i$ has more than one coin and $i \\neq n$, have child $i$ pass coins to its neighbors. (Child $i$ does nothing.) Then, after some amount of such steps, it must eventually become impossible to do any more steps because no child except perhaps child $i$ has more than 1 coin. (To see this, consider e.g. the quantity $\\sum_{i=1}^{n-1} i^{2} c_{i}$, which (as $(i-1)^{2}+(i+1)^{2}>2 i^{2}$ ) increases at each step.) Hence we can reach a state of the form $\\left(z_{1}, \\ldots, z_{n-1}, M\\right)$, where $z_{i}=0$ or 1 . Call such states semi-uniform states of irregularity $M$. Lemma. If there is a string of children having coins $a, \\overbrace{1, \\ldots, 1}^{k \\text { ones }}, b, \\overbrace{1, \\ldots, 1}^{k \\text { ones }}, c$, with $b \\geqslant 2$, after some sequence of steps we may reach the state $a+1, \\overbrace{1, \\ldots, 1}^{k \\text { ones }}, b-2, \\overbrace{1, \\ldots, 1}^{k \\text { ones }}, c+1$. We call performing this sequence of steps long-passing coins. Proof. This is simply repeated application of the operation. We prove the lemma by induction on $k$. For $k=0$, this is just the operation of the problem. If $k=1$, have the child with $b$ coins pass coins, then both of their neighbors pass coins, then the child with $b$ coins pass coins again. For $k \\geqslant 2$, first, have the child with $b$ coins pass coins, then have both their neigbors send coins, giving the state $$ a, \\overbrace{1, \\ldots, 1}^{k-2 \\text { ones }}, 2,0, b, 0,2, \\overbrace{1, \\ldots, 1}^{k-2 \\text { ones }}, c . $$ Now set aside the children with $a, b$ and $c$ coins, and have each child with 2 coins give them to their neighbors until there are no such children remaining. This results in the state $$ a+1,0, \\overbrace{1, \\ldots, 1}^{k-2}, b, \\overbrace{1, \\ldots, 1}^{k-2 \\text { ones }}, 0, c+1 $$ By the induction hypothesis, we can have the child with $b$ coins may pass a coin to each of the children with 0 coins, proving the lemma. Claim. We can reach a semi-uniform state of irregularity $M \\leqslant 2$. Proof. If $M>3$, because there are only $n$ coins in total, there must be at least two children with 0 coins. Consider the arc of the circle spanned by the two such children closest to the child with $M$ coins. It has the form $$ 0, \\overbrace{1, \\ldots, 1}^{a \\text { ones }}, M, \\overbrace{1, \\ldots, 1}^{b \\text { ones }}, 0 $$ If $a=b$, applying the previous lemma we can have the child with $M$ coins long-pass a coin to each of the children with 0 coins, which yields a semi-uniform state with lower $M$. Otherwise, WLOG $a>b$, so we can have the child with $M$ coins long-pass a coin to each of the children at distance $b$ from it, reaching a state of the form $(\\alpha:=a-b-1, \\beta:=b)$ $$ 0, \\overbrace{1, \\ldots, 1}^{\\alpha \\text { ones }}, 2, \\overbrace{1, \\ldots, 1}^{\\beta \\text { ones }}, M-2, \\overbrace{1, \\ldots, 1}^{c \\text { ones }} $$ The children in the rightmost string of ones need make no further moves, so consider only the leftmost string. If $\\alpha<\\beta$, have the child with 2 coins long-pass coins to the child with 0 coins to its left and some child with 1 coin to its right, reaching a new state of the form $0, \\overbrace{1, \\ldots, 1}^{\\alpha \\text { ones }}, 2, \\overbrace{1, \\ldots, 1}^{\\beta \\text { ones }}, M-2$ with a smaller $\\beta$. As $\\beta$ cannot decrease indefinitely, eventually $\\alpha \\geqslant \\beta$. If $\\alpha=\\beta$, have the child with 2 coins long-pass to the child with $M$ coins and the child with 0 coins, reaching a semi-uniform state of irregularity $M-1$ as desired. Otherwise, $\\alpha<\\beta$, so have the child with 2 coins long-pass to the child with $M$ coins and a child with 1 coin, reaching a state of the form $$ 0, \\overbrace{1, \\ldots, 1}^{x \\text { ones }}, 2, \\overbrace{1, \\ldots, 1}^{y \\text { ones }}, 0, \\overbrace{1, \\ldots, 1}^{z \\text { ones }}, M-1 $$ Now, consider only the substring between the two children with 0 coins, which has the form $0, \\overbrace{1, \\ldots, 1}^{x \\text { ones }}, 2, \\overbrace{1, \\ldots, 1}^{y \\text { ones }}, 0$. Repeatedly have the child in this substring with 2 coins long-pass to the closest child with 0 coins and some other child. If the other child has 1 coin, we have a new strictly shorter substring of the form $0, \\overbrace{1, \\ldots, 1}^{x \\text { ones }}, 2, \\overbrace{1, \\ldots, 1}^{y \\text { ones }}, 0$. Hence eventually it must happen that the other child also has 0 coins, at which point we reach a semi-uniform state of irregularity $M-1$, proving the claim. We have now shown that we can reach a semi-regular state of irregularity $M \\leqslant 2$, If $M=1$, each child must have one coin, as desired. Otherwise, we must have $M=2$, so there is one child with 0 coins, one child with 2 coins, and the remaining children all have 1 coin. Recall that the state we started with satisfied the invariant $$ \\sum_{i=1}^{n} i c_{i}=\\frac{n(n+1)}{2} \\quad(\\bmod n) $$ Because each step preserves this invariant, this must also be true of the current state. Number the children so that the child with $M$ coins is child number $n$, and suppose the child with 0 coins is child $k$. Then $\\frac{n(n+1)}{2}=\\sum_{i=1}^{n} i c_{i}=\\left(\\sum_{i=1}^{n} 1 \\cdot c_{i}\\right)-k=\\frac{n(n+1)}{2}-k(\\bmod n)$, so $k=0$ $(\\bmod n)$. But this is impossible, as no child except the child with $M$ coins has an index divisible by $n$. Hence we cannot end up in a semi-regular state of irregularity 2 , so we are done.","tier":0} +{"problem_type":"Combinatorics","problem_label":"C4","problem":"Let $n>3$ be a positive integer. Suppose that $n$ children are arranged in a circle, and $n$ coins are distributed between them (some children may have no coins). At every step, a child with at least 2 coins may give 1 coin to each of their immediate neighbours on the right and left. Determine all initial distributions of coins from which it is possible that, after a finite number of steps, each child has exactly one coin.","solution":"Encode the sequence $c_{i}$ as a polynomial $p(x)=\\sum_{i} a_{i} x_{i}$. The cyclic nature of the problem makes it natural to work modulo $x^{n}-1$. Child $i$ performing a step is equivalent to adding $x^{i}(x-1)^{2}$ to the polynomial, and we want to reach the polynomial $q(x)=1+x+\\ldots+$ $x^{n-1}$. Since we only add multiples of $(x-1)^{2}$, this is only possible if $p(x)=q(x)$ modulo the ideal generated by $x^{n}-1$ and $(x-1)^{2}$, i.e. $$ \\left(x^{n}-1,(x-1)^{2}\\right)=(x-1)\\left(\\frac{x^{n}-1}{x-1}, x-1\\right)=(x-1) \\cdot(n,(x-1)) $$ This is equivalent to $p(1)=q(1)$ (which simply translates to the condition that there are $n$ coins) and $p^{\\prime}(1)=q^{\\prime}(1)(\\bmod n)$, which translates to the invariant described in solution 1.","tier":0} +{"problem_type":"Combinatorics","problem_label":"C5","problem":"Let $m, n \\geqslant 2$ be integers, let $X$ be a set with $n$ elements, and let $X_{1}, X_{2}, \\ldots, X_{m}$ be pairwise distinct non-empty, not necessary disjoint subsets of $X$. A function $f: X \\rightarrow$ $\\{1,2, \\ldots, n+1\\}$ is called nice if there exists an index $k$ such that $$ \\sum_{x \\in X_{k}} f(x)>\\sum_{x \\in X_{i}} f(x) \\text { for all } i \\neq k $$ Prove that the number of nice functions is at least $n^{n}$. (Germany)","solution":"For a subset $Y \\subseteq X$, we write $f(Y)$ for $\\sum_{y \\in Y} f(y)$. Note that a function $f: X \\rightarrow$ $\\{1, \\ldots, n+1\\}$ is nice, if and only if $f\\left(X_{i}\\right)$ is maximized by a unique index $i \\in\\{1, \\ldots, m\\}$. We will first investigate the set $\\mathcal{F}$ of functions $f: X \\rightarrow\\{1, \\ldots, n\\}$; note that $|\\mathcal{F}|=n^{n}$. For every function $f \\in \\mathcal{F}$, define a corresponding function $f^{+}: X \\rightarrow\\{1,2, \\ldots, n+1\\}$ in the following way: Pick some set $X_{l}$ that maximizes the value $f\\left(X_{l}\\right)$. - For all $x \\in X_{l}$, define $f^{+}(x)=f(x)+1$. - For all $x \\in X \\backslash X_{l}$, define $f^{+}(x)=f(x)$. Claim. The resulting function $f^{+}$is nice. Proof. Note that $f^{+}\\left(X_{i}\\right)=f\\left(X_{i}\\right)+\\left|X_{i} \\cap X_{l}\\right|$ holds for all $X_{i}$. We show that $f^{+}\\left(X_{i}\\right)$ is maximized at the unique index $i=l$. Hence consider some arbitrary index $j \\neq l$. Then $X_{l} \\subset X_{j}$ is impossible, as this would imply $f\\left(X_{j}\\right)>f\\left(X_{l}\\right)$ and thereby contradict the choice of set $X_{l}$; this in particular yields $\\left|X_{l}\\right|>\\left|X_{j} \\cap X_{l}\\right|$. $$ f^{+}\\left(X_{l}\\right)=f\\left(X_{l}\\right)+\\left|X_{l}\\right| \\geqslant f\\left(X_{j}\\right)+\\left|X_{l}\\right|>f\\left(X_{j}\\right)+\\left|X_{j} \\cap X_{l}\\right|=f^{+}\\left(X_{j}\\right) $$ The first inequality follows since $X_{l}$ was chosen to maximize the value $f\\left(X_{l}\\right)$. The second (strict) inequality follows from $\\left|X_{l}\\right|>\\left|X_{j} \\cap X_{l}\\right|$ as observed above. This completes the proof of the claim. Next observe that function $f$ can be uniquely reconstructed from $f^{+}$: the claim yields that $f^{+}$has a unique maximizer $X_{l}$, and by decreasing the value of $f^{+}$on $X_{l}$ by 1 , we get we can fully determine the values of $f$. As each of the $n^{n}$ functions $f \\in \\mathcal{F}$ yields a (unique) corresponding nice function $f^{+}: X \\rightarrow\\{1,2, \\ldots, n+1\\}$, the proof is complete.","tier":0} +{"problem_type":"Combinatorics","problem_label":"C6","problem":"Let $n$ be a positive integer. We start with $n$ piles of pebbles, each initially containing a single pebble. One can perform moves of the following form: choose two piles, take an equal number of pebbles from each pile and form a new pile out of these pebbles. For each positive integer $n$, find the smallest number of non-empty piles that one can obtain by performing a finite sequence of moves of this form. (Croatia)","solution":"The solution we describe is simple, but not the most effective one. We can combine two piles of $2^{k-1}$ pebbles to make one pile of $2^{k}$ pebbles. In particular, given $2^{k}$ piles of one pebble, we may combine them as follows: $$ \\begin{array}{lcc} 2^{k} \\text { piles of } 1 \\text { pebble } & \\rightarrow & 2^{k-1} \\text { piles of } 2 \\text { pebbles } \\\\ 2^{k-1} \\text { piles of } 2 \\text { pebbles } & \\rightarrow & 2^{k-2} \\text { piles of } 4 \\text { pebbles } \\\\ 2^{k-2} \\text { piles of } 4 \\text { pebbles } & \\rightarrow & 2^{k-3} \\text { piles of } 8 \\text { pebbles } \\\\ & \\vdots \\\\ 2 \\text { piles of } 2^{k-1} \\text { pebbles } & \\rightarrow 1 \\text { pile of } 2^{k} \\text { pebbles } \\end{array} $$ This proves the desired result in the case when $n$ is a power of 2 . If $n$ is not a power of 2 , choose $N$ such that $2^{N}1$. In order to make a single pile of $n$ pebbles, we would have to start with a distribution in which the number of pebbles in each pile is divisible by the integer $m$. This is impossible when we start with all piles containing a single pebble. Remarks on starting configurations From any starting configuration that is not a single pile, if there are at least two piles with at least two pebbles, we can remove one pebble from two such piles, and form a new pile with 2 pebbles. We can repeat this until we have one pile of 2 pebbles and the rest are single pebble piles, and then proceed as in the solution. Hence, if $n$ is a power of two, we can make a single pile from any starting configuration. If $n$ is of the form $n=2^{k} m$ where $m>1$ is odd, then we can make a single pile from any starting configuration in which the number of pebbles in each pile is divisible by the integer $m$, otherwise two piles is the best we can do. Half of this is proven already. For the other half, assume we start with a configuration in which the number of pebbles in each pile is divisible by the integer $m$. Replace each pile of $t m$ pebbles with a pile of $t$ boulders. We now have a total of $2^{k}$ boulders, hence we can make them into one pile of $2^{k}$ boulders. Replacing the boulders with pebbles again, we are done.","tier":0} +{"problem_type":"Combinatorics","problem_label":"C6","problem":"Let $n$ be a positive integer. We start with $n$ piles of pebbles, each initially containing a single pebble. One can perform moves of the following form: choose two piles, take an equal number of pebbles from each pile and form a new pile out of these pebbles. For each positive integer $n$, find the smallest number of non-empty piles that one can obtain by performing a finite sequence of moves of this form. (Croatia)","solution":"We show an alternative strategy if $n$ is not a power of 2 . Write $n$ in binary form: $n=2^{i_{1}}+2^{i_{2}}+\\cdots+2^{i_{k}}$, where $i_{1}>i_{2}>\\cdots>i_{k}$. Now we make piles of sizes $2^{i_{1}}, 2^{i_{2}}, \\ldots, 2^{i_{k}}$. We call the pile with $2^{i_{1}}$ the large pile, all the others are the small piles. The strategy is the following: take the two smallest small pile. If they have the same size of $2^{a}$, we make a pile of size $2^{a+1}$. If they have different sizes, we double the smaller pile using the large pile (we allow the large pile to have a negative number of pebbles: later we prove that it is not possible). We call all the new piles small. When we have only one small pile, we terminate the process: we have at most 2 piles. After each move we have one less number of piles, and all the piles have cardinality power of 2. The number of pebbles is decreasing, and at the end of the process, it has a size of $n-2^{i_{2}+1} \\geqslant n-2^{i_{1}}>0$, thus we can manage to have two piles.","tier":0} +{"problem_type":"Combinatorics","problem_label":"C6","problem":"Let $n$ be a positive integer. We start with $n$ piles of pebbles, each initially containing a single pebble. One can perform moves of the following form: choose two piles, take an equal number of pebbles from each pile and form a new pile out of these pebbles. For each positive integer $n$, find the smallest number of non-empty piles that one can obtain by performing a finite sequence of moves of this form. (Croatia)","solution":"Throughout the solution, we will consider the moves in reverse order. Namely, imagine we have some piles of pebbles, and we are allowed to perform moves as follows: take a pile with an even number of pebbles, split it into two equal halves and add the pebbles from each half to a different pile, possibly forming new piles (we may assume for convenience that there are infinitely many empty piles at any given moment). Given a configuration of piles $\\mathcal{C}$, we will use $|\\mathcal{C}|$ to denote the number of non-empty piles in $\\mathcal{C}$. Given two configurations $\\mathcal{C}_{1}, \\mathcal{C}_{2}$, we will say that $\\mathcal{C}_{2}$ is reachable from $\\mathcal{C}_{1}$ if $\\mathcal{C}_{2}$ can be obtained by performing a finite sequence of moves starting from $\\mathcal{C}_{1}$. Call a configuration of piles $\\mathcal{C}$ - simple if each (non-empty) pile in $\\mathcal{C}$ consists of a single pebble; - good if at least one (non-empty) pile in $\\mathcal{C}$ has an even number of pebbles and the numbers of pebbles on the piles in $\\mathcal{C}$ have no non-trivial odd common divisor ( $\\mathcal{C}$ has the odd divisor property); - solvable if there exists a simple configuration which is reachable from $\\mathcal{C}$. The problem asks to find the smallest number of non-empty piles in a solvable configuration consisting of $n$ pebbles. We begin the process of answering this question by making the following observation: Lemma 1. Let $\\mathcal{C}$ be a configuration of piles. Let $\\mathcal{C}^{\\prime}$ be a configuration obtained by applying a single move to $\\mathcal{C}$. Then (i) if $\\mathcal{C}^{\\prime}$ has the odd divisor property, then so does $\\mathcal{C}$; (ii) the converse to (i) holds if $\\left|\\mathcal{C}^{\\prime}\\right| \\geqslant|\\mathcal{C}|$. Proof. Suppose that the move consists of splitting a pile of size $2 a$ and adding $a$ pebbles to each of two piles of sizes $b$ and $c$. Here, $a$ is a positive integer and $b, c$ are non-negative integers. Thus, $\\mathcal{C}^{\\prime}$ can be obtained from $\\mathcal{C}$ by replacing the piles of sizes $2 a, b, c$ by two piles of sizes $a+b$ and $a+c$. Note that the extra assumption $\\left|\\mathcal{C}^{\\prime}\\right| \\geqslant|\\mathcal{C}|$ of part (ii) holds if and only if at least one of $b, c$ is zero. (i) Suppose $\\mathcal{C}$ doesn't have the odd divisor property, i.e. there exists an odd integer $d>1$ such that the size of each pile in $\\mathcal{C}$ is divisible by $d$. In particular, $2 a, b, c$ are multiples of $d$, so since $d$ is odd, it follows that $a, b, c$ are all divisible by $d$. Thus, $a+b$ and $a+c$ are also divisible by $d$, so $d$ divides the size of each pile in $\\mathcal{C}^{\\prime}$. In conclusion, $\\mathcal{C}^{\\prime}$ doesn't have the odd divisor property. (ii) If $\\mathcal{C}^{\\prime}$ doesn't have the odd divisor property and at least one of $b, c$ is zero, then there exists an odd integer $d>1$ such that the size of each pile in $\\mathcal{C}^{\\prime}$ is divisible by $d$. In particular, $d$ divides $a+b$ and $a+c$, so since at least one of these numbers is equal to $a$, it follows that $d$ divides $a$. But then $d$ must divide all three of $a, b$ and $c$, and hence it certainly divides $2 a, b$ and $c$. Thus, $\\mathcal{C}$ doesn't have the odd divisor property, as desired. Lemma 2. If $\\mathcal{C}_{2}$ is reachable from $\\mathcal{C}_{1}$ and $\\mathcal{C}_{2}$ has the odd divisor property, then so does $\\mathcal{C}_{1}$. In particular, any solvable configuration has the odd divisor property. Proof. The first statement follows by inductively applying part (i) of Lemma 1. The second statement follows from the first because every simple configuration has the odd divisor property. The main claim is the following: Lemma 3. Let $\\mathcal{C}$ be a good configuration. Then there exists a configuration $\\mathcal{C}^{\\prime}$ with the following properties: - $\\mathcal{C}^{\\prime}$ is reachable from $\\mathcal{C}$ and $\\left|\\mathcal{C}^{\\prime}\\right|>|\\mathcal{C}|$; - $\\mathcal{C}^{\\prime}$ is either simple or good. Proof. Call a configuration terminal if it is a counterexample to the claim. The following claim is at the heart of the proof: Claim. Let $a_{1}, \\ldots, a_{k}$ be the numbers of pebbles on the non-empty piles of a terminal configuration $\\mathcal{C}$. Then there exists a unique $i \\in[k]$ such that $a_{i}$ is even. Moreover, for all $t \\geqslant 1$ we have $a_{j} \\equiv \\frac{a_{i}}{2}\\left(\\bmod 2^{t}\\right)$ for all $j \\neq i$. Proof of Claim. Since the configuration is good, there must exist $i \\in[k]$ such that $a_{i}$ is even. Moreover, by assumption, if we split the pile with $a_{i}$ pebbles into two equal halves, the resulting configuration will not be good. By part (ii) of Lemma 2,the only way this can happen is that $\\frac{a_{i}}{2}$ and $a_{j}$ for all $j \\neq i$ are odd. To prove the second assertion, we proceed by induction on $t$, with the case $t=1$ already being established. If $t \\geqslant 2$, then split the pile with $a_{i}$ pebbles into two equal halves and move one half to the pile with $a_{j}$ pebbles. Since $\\frac{a_{i}}{2}$ and $a_{j}$ are both odd, $a_{j}+\\frac{a_{i}}{2}$ is even, so by part (ii) of Lemma 2 , the resulting configuration $\\mathcal{C}^{\\prime}$ is good. Thus, $\\mathcal{C}^{\\prime}$ is terminal, so by the induction hypothesis, we have $\\frac{a_{i}}{2} \\equiv \\frac{1}{2}\\left(a_{j}+\\frac{a_{i}}{2}\\right)\\left(\\bmod 2^{t-1}\\right)$, whence $a_{j} \\equiv \\frac{a_{i}}{2}($ $\\bmod 2^{t}$ ), as desired. Suppose for contradiction that there exists a configuration as in the Claim. It follows that there exists $i \\in[k]$ and an odd integer $x$ such that $a_{i}=2 x$ and $a_{j}=x$ for all $j \\neq i$. Thus, $x$ is an odd common divisor of $a_{1}, \\ldots, a_{k}$, so by the odd divisor property, we must have $x=1$. But then we can obtain a simple configuration by splitting the only pile with two pebbles into two piles each consisting of a single pebble, which is a contradiction. With the aid of Lemmas 2 and 3, it is not hard to characterise all solvable configurations: Lemma 4. A configuration of piles $\\mathcal{C}$ is solvable if and only if it is simple or good. Proof. $(\\Longrightarrow)$ Suppose $\\mathcal{C}$ is not simple. Then since we have to be able to perform at least one move, there must be at least one non-empty pile in $\\mathcal{C}$ with an even number of pebbles. Moreover, by Lemma $2, \\mathcal{C}$ has the odd divisor property, so it must be good. $(\\Longleftarrow)$ This follows by repeatedly applying Lemma 3 until we reach a simple configuration. Note that the process must stop eventually since the number of non-empty piles is bounded from above. Finally, the answer to the problem is implied by the following corollary of Lemma 4: Lemma 5. Let $n$ be a positive integer. Then (i) a configuration consisting of a single pile of $n$ pebbles is solvable if and only if $n$ is a power of two; (ii) if $n \\geqslant 2$, then the configuration consisting of piles of sizes 2 and $n-2$ is good. Proof. (i) By Lemma 4, this configuration is solvable if and only if either $n=1$ or $n$ is even and has no non-trivial odd divisor, so the conclusion follows. (ii) Since 2 is even and has no non-trivial odd divisor, this configuration is certainly good, so the conclusion follows by Lemma 4. Common remarks. Instead of choosing pebbles from two piles, one could allow choosing an equal number of pebbles from each of $k$ piles, where $k \\geqslant 2$ is a fixed (prime) integer. However, this seems to yield a much harder problem - if $k=3$, numerical evidence suggests the same answer as in the case $k=2$ (with powers of two replaced by powers of three), but the case $k=5$ is already unclear.","tier":0} +{"problem_type":"Combinatorics","problem_label":"C7","problem":"Lucy starts by writing $s$ integer-valued 2022-tuples on a blackboard. After doing that, she can take any two (not necessarily distinct) tuples $\\mathbf{v}=\\left(v_{1}, \\ldots, v_{2022}\\right)$ and $\\mathbf{w}=\\left(w_{1}, \\ldots, w_{2022}\\right)$ that she has already written, and apply one of the following operations to obtain a new tuple: $$ \\begin{aligned} & \\mathbf{v}+\\mathbf{w}=\\left(v_{1}+w_{1}, \\ldots, v_{2022}+w_{2022}\\right) \\\\ & \\mathbf{v} \\vee \\mathbf{w}=\\left(\\max \\left(v_{1}, w_{1}\\right), \\ldots, \\max \\left(v_{2022}, w_{2022}\\right)\\right) \\end{aligned} $$ and then write this tuple on the blackboard. It turns out that, in this way, Lucy can write any integer-valued 2022-tuple on the blackboard after finitely many steps. What is the smallest possible number $s$ of tuples that she initially wrote? (Czech Republic)","solution":"We solve the problem for $n$-tuples for any $n \\geqslant 3$ : we will show that the answer is $s=3$, regardless of the value of $n$. First, let us briefly introduce some notation. For an $n$-tuple $\\mathbf{v}$, we will write $\\mathbf{v}_{i}$ for its $i$-th coordinate (where $1 \\leqslant i \\leqslant n$ ). For a positive integer $n$ and a tuple $\\mathbf{v}$ we will denote by $n \\cdot \\mathbf{v}$ the tuple obtained by applying addition on $\\mathbf{v}$ with itself $n$ times. Furthermore, we denote by $\\mathbf{e}(i)$ the tuple which has $i$-th coordinate equal to one and all the other coordinates equal to zero. We say that a tuple is positive if all its coordinates are positive, and negative if all its coordinates are negative. We will show that three tuples suffice, and then that two tuples do not suffice. Three tuples suffice. Write $\\mathbf{c}$ for the constant-valued tuple $\\mathbf{c}=(-1, \\ldots,-1)$. We note that it is enough for Lucy to be able to make the tuples $\\mathbf{e}(1), \\ldots, \\mathbf{e}(n), \\mathbf{c}$; from those any other tuple $\\mathbf{v}$ can be made as follows. First we choose some positive integer $k$ such that $k+\\mathbf{v}_{i}>0$ for all $i$. Then, by adding a positive number of copies of $\\mathbf{c}, \\mathbf{e}(1), \\ldots, \\mathbf{e}(n)$, she can make $$ k \\mathbf{c}+\\left(k+\\mathbf{v}_{1}\\right) \\cdot \\mathbf{e}(1)+\\cdots+\\left(k+\\mathbf{v}_{n}\\right) \\cdot \\mathbf{e}(n) $$ which we claim is equal to $\\mathbf{v}$. Indeed, this can be checked by comparing coordinates: the $i$-th coordinate of the right-hand side is $-k+\\left(k+\\mathbf{v}_{i}\\right)=\\mathbf{v}_{i}$ as needed. Lucy can take her three starting tuples to be $\\mathbf{a}, \\mathbf{b}$ and $\\mathbf{c}$, such that $\\mathbf{a}_{i}=-i^{2}, \\mathbf{b}_{i}=i$ and $\\mathbf{c}=-1$ (as above). For any $1 \\leqslant j \\leqslant n$, write $\\mathbf{d}(j)$ for the tuple $2 \\cdot \\mathbf{a}+4 j \\cdot \\mathbf{b}+\\left(2 j^{2}-1\\right) \\cdot \\mathbf{c}$, which Lucy can make by adding together $\\mathbf{a}, \\mathbf{b}$ and $\\mathbf{c}$ repeatedly. This has $i$ th term $$ \\begin{aligned} \\mathbf{d}(j)_{i} & =2 \\mathbf{a}_{i}+4 j \\mathbf{b}_{i}+\\left(2 j^{2}-1\\right) \\mathbf{c}_{i} \\\\ & =-2 i^{2}+4 i j-\\left(2 j^{2}-1\\right) \\\\ & =1-2(i-j)^{2} \\end{aligned} $$ This is 1 if $j=i$, and at most -1 otherwise. Hence Lucy can produce the tuple $\\mathbf{1}=(1, \\ldots, 1)$ as $\\mathbf{d}(1) \\vee \\cdots \\vee \\mathbf{d}(n)$. She can then produce the constant tuple $\\mathbf{0}=(0, \\ldots, 0)$ as $\\mathbf{1}+\\mathbf{c}$, and for any $1 \\leqslant j \\leqslant n$ she can then produce the tuple $\\mathbf{e}(j)$ as $\\mathbf{d}(j) \\vee \\mathbf{0}$. Since she can now produce $\\mathbf{e}(1), \\ldots, \\mathbf{e}(n)$ and already had c, she can (as we argued earlier) produce any integer-valued tuple. Two tuples do not suffice. We start with an observation: Let $a$ be a non-negative real number and suppose that two tuples $\\mathbf{v}$ and $\\mathbf{w}$ satisfy $\\mathbf{v}_{j} \\geqslant a \\mathbf{v}_{k}$ and $\\mathbf{w}_{j} \\geqslant a \\mathbf{w}_{k}$ for some $1 \\leqslant j, k \\leqslant n$. Then we claim that the same inequality holds for $\\mathbf{v}+\\mathbf{w}$ and $\\mathbf{v} \\vee \\mathbf{w}$ : Indeed, the property for the sum is verified by an easy computation: $$ (\\mathbf{v}+\\mathbf{w})_{j}=\\mathbf{v}_{j}+\\mathbf{w}_{j} \\geqslant a \\mathbf{v}_{k}+a \\mathbf{w}_{k}=a(\\mathbf{v}+\\mathbf{w})_{k} $$ For the second operation, we denote by $\\mathbf{m}$ the tuple $\\mathbf{v} \\vee \\mathbf{w}$. Then $\\mathbf{m}_{j} \\geqslant \\mathbf{v}_{j} \\geqslant a \\mathbf{v}_{k}$ and $\\mathbf{m}_{j} \\geqslant \\mathbf{w}_{j} \\geqslant a \\mathbf{w}_{k}$. Since $\\mathbf{m}_{k}=\\mathbf{v}_{k}$ or $\\mathbf{m}_{k}=\\mathbf{w}_{k}$, the observation follows. As a consequence of this observation we have that if all starting tuples satisfy such an inequality, then all generated tuples will also satisfy it, and so we would not be able to obtain every integer-valued tuple. Let us now prove that Lucy needs at least three starting tuples. For contradiction, let us suppose that Lucy started with only two tuples $\\mathbf{v}$ and $\\mathbf{w}$. We are going to distinguish two cases. In the first case, suppose we can find a coordinate $i$ such that $\\mathbf{v}_{i}, \\mathbf{w}_{i} \\geqslant 0$. Both operations preserve the sign, thus we can not generate any tuple that has negative $i$-th coordinate. Similarly for $\\mathbf{v}_{i}, \\mathbf{w}_{i} \\leqslant 0$. Suppose the opposite, i.e., for every $i$ we have either $\\mathbf{v}_{i}>0>\\mathbf{w}_{i}$, or $\\mathbf{v}_{i}<0<\\mathbf{w}_{i}$. Since we assumed that our tuples have at least three coordinates, by pigeonhole principle there exist two coordinates $j \\neq k$ such that $\\mathbf{v}_{j}$ has the same sign as $\\mathbf{v}_{k}$ and $\\mathbf{w}_{j}$ has the same sign as $\\mathbf{w}_{k}$ (because there are only two possible combinations of signs). Without loss of generality assume that $\\mathbf{v}_{j}, \\mathbf{v}_{k}>0$ and $\\mathbf{w}_{j}, \\mathbf{w}_{k}<0$. Let us denote the positive real number $\\mathbf{v}_{j} \/ \\mathbf{v}_{k}$ by $a$. If $\\mathbf{w}_{j} \/ \\mathbf{w}_{k} \\leqslant a$, then both inequalities $\\mathbf{v}_{j} \\geqslant a \\mathbf{v}_{k}$ and $\\mathbf{w}_{j} \\geqslant a \\mathbf{w}_{k}$ are satisfied. On the other hand, if $\\mathbf{w}_{j} \/ \\mathbf{w}_{k} \\leqslant a$, then both $\\mathbf{v}_{k} \\geqslant(1 \/ a) \\mathbf{v}_{j}$ and $\\mathbf{w}_{k} \\geqslant(1 \/ a) \\mathbf{w}_{j}$ are satisfied. In either case, we have found the desired inequality satisfied by both starting tuples, a contradiction with the observation above.","tier":0} +{"problem_type":"Combinatorics","problem_label":"C8","problem":"Alice fills the fields of an $n \\times n$ board with numbers from 1 to $n^{2}$, each number being used exactly once. She then counts the total number of good paths on the board. A good path is a sequence of fields of arbitrary length (including 1) such that: (i) The first field in the sequence is one that is only adjacent to fields with larger numbers, (ii) Each subsequent field in the sequence is adjacent to the previous field, (iii) The numbers written on the fields in the sequence are in increasing order. Two fields are considered adjacent if they share a common side. Find the smallest possible number of good paths Alice can obtain, as a function of $n$. (Serbia)","solution":"We will call any field that is only adjacent to fields with larger numbers a well. Other fields will be called non-wells. Let us make a second $n \\times n$ board $B$ where in each field we will write the number of good sequences which end on the corresponding field in the original board $A$. We will thus look for the minimal possible value of the sum of all entries in $B$. We note that any well has just one good path ending in it, consisting of just the well, and that any other field has the number of good paths ending in it equal to the sum of this quantity for all the adjacent fields with smaller values, since a good path can only come into the field from a field of lower value. Therefore, if we fill in the fields in $B$ in increasing order with respect to their values in $A$, it follows that each field not adjacent to any already filled field will receive a 1, while each field adjacent to already filled fields will receive the sum of the numbers already written on these adjacent fields. We note that there is at least one well in $A$, that corresponding with the field with the entry 1 in $A$. Hence, the sum of values of fields in $B$ corresponding to wells in $A$ is at least 1 . We will now try to minimize the sum of the non-well entries, i.e. of the entries in $B$ corresponding to the non-wells in $A$. We note that we can ascribe to each pair of adjacent fields the value of the lower assigned number and that the sum of non-well entries will then equal to the sum of the ascribed numbers. Since the lower number is still at least 1, the sum of non-well entries will at least equal the number of pairs of adjacent fields, which is $2 n(n-1)$. Hence, the total minimum sum of entries in $B$ is at least $2 n(n-1)+1=2 n^{2}-2 n+1$. The necessary conditions for the minimum to be achieved is for there to be only one well and for no two entries in $B$ larger than 1 to be adjacent to each other. We will now prove that the lower limit of $2 n^{2}-2 n+1$ entries can be achieved. This amounts to finding a way of marking a certain set of squares, those that have a value of 1 in $B$, such that no two unmarked squares are adjacent and that the marked squares form a connected tree with respect to adjacency. For $n=1$ and $n=2$ the markings are respectively the lone field and the L-trimino. Now, for $n>2$, let $s=2$ for $n \\equiv 0,2 \\bmod 3$ and $s=1$ for $n \\equiv 1 \\bmod 3$. We will take indices $k$ and $l$ to be arbitrary non-negative integers. For $n \\geqslant 3$ we will construct a path of marked squares in the first two columns consisting of all squares of the form $(1, i)$ where $i$ is not of the form $6 k+s$ and $(2, j)$ where $j$ is of the form $6 k+s-1,6 k+s$ or $6+s+1$. Obviously, this path is connected. Now, let us consider the fields $(2,6 k+s)$ and $(1,6 k+s+3)$. For each considered field $(i, j)$ we will mark all squares of the form $(l, j)$ for $l>i$ and $(i+2 k, j \\pm 1)$. One can easily see that no set of marked fields will produce a cycle, that the only fields of the unmarked form $(1,6 k+s),(2+2 l+1,6 k+s \\pm 1)$ and $(2+2 l, 6 k+s+3 \\pm 1)$ and that no two are adjacent, since the consecutive considered fields are in columns of opposite parity. Examples of markings are given for $n=3,4,5,6,7$, and the corresponding constructions for $A$ and $B$ are given for $n=5$. ![](https:\/\/cdn.mathpix.com\/cropped\/2024_11_18_86efb4389f14b52b08e1g-41.jpg?height=352&width=1648&top_left_y=384&top_left_x=204)","tier":0} +{"problem_type":"Combinatorics","problem_label":"C9","problem":"Let $\\mathbb{Z}_{\\geqslant 0}$ be the set of non-negative integers, and let $f: \\mathbb{Z}_{\\geqslant 0} \\times \\mathbb{Z}_{\\geqslant 0} \\rightarrow \\mathbb{Z}_{\\geqslant 0}$ be a bijection such that whenever $f\\left(x_{1}, y_{1}\\right)>f\\left(x_{2}, y_{2}\\right)$, we have $f\\left(x_{1}+1, y_{1}\\right)>f\\left(x_{2}+1, y_{2}\\right)$ and $f\\left(x_{1}, y_{1}+1\\right)>f\\left(x_{2}, y_{2}+1\\right)$. Let $N$ be the number of pairs of integers $(x, y)$, with $0 \\leqslant x, y<100$, such that $f(x, y)$ is odd. Find the smallest and largest possible value of $N$.","solution":"We defer the constructions to the end of the solution. Instead, we begin by characterizing all such functions $f$, prove a formula and key property for such functions, and then solve the problem, providing constructions. Characterization Suppose $f$ satisfies the given relation. The condition can be written more strongly as $$ \\begin{aligned} f\\left(x_{1}, y_{1}\\right)>f\\left(x_{2}, y_{2}\\right) & \\Longleftrightarrow f\\left(x_{1}+1, y_{1}\\right)>f\\left(x_{2}+1, y_{2}\\right) \\\\ & \\Longleftrightarrow f\\left(x_{1}, y_{1}+1\\right)>f\\left(x_{2}, y_{2}+1\\right) . \\end{aligned} $$ In particular, this means for any $(k, l) \\in \\mathbb{Z}^{2}, f(x+k, y+l)-f(x, y)$ has the same sign for all $x$ and $y$. Call a non-zero vector $(k, l) \\in \\mathbb{Z}_{\\geqslant 0} \\times \\mathbb{Z}_{\\geqslant 0}$ a needle if $f(x+k, y)-f(x, y+l)>0$ for all $x$ and $y$. It is not hard to see that needles and non-needles are both closed under addition, and thus under scalar division (whenever the quotient lives in $\\mathbb{Z}^{2}$ ). In addition, call a positive rational number $\\frac{k}{l}$ a grade if the vector $(k, l)$ is a needle. (Since needles are closed under scalar multiples and quotients, this is well-defined.) Claim. Grades are closed upwards. Proof. Consider positive rationals $k_{1} \/ l_{1}0$ and $(1,0)$ is a needle). Thus $\\left(k_{2}, l_{2}\\right)$ is a needle, as wanted. Claim. A grade exists. Proof. If no positive integer $n$ is a grade, then $f(1,0)>f(0, n)$ for all $n$ which is impossible. Similarly, there is an $n$ such that $f(0,1)x_{2}+y_{2} \\alpha$ then $f\\left(x_{1}, y_{1}\\right)>f\\left(x_{2}, y_{2}\\right)$. Proof. If both $x_{1} \\geqslant x_{2}$ and $y_{1} \\geqslant y_{2}$ this is clear. Suppose $x_{1} \\geqslant x_{2}$ and $y_{1}\\alpha$ is a grade. This gives $f\\left(x_{1}, y_{1}\\right)>f\\left(x_{2}, y_{2}\\right)$. Suppose $x_{1}0} \\mid x+(y+1) \\alpha \\leqslant a+b \\alpha<(x+1)+(y+1) \\alpha\\right\\}+ \\\\ \\#\\left\\{(a, 0) \\in \\mathbb{Z}_{\\geqslant 0} \\times \\mathbb{Z}_{\\geqslant 0} \\mid(x+1)+y \\alpha \\leqslant a<(x+1)+(y+1) \\alpha\\right\\}= \\\\ \\#\\left\\{(a, b) \\in \\mathbb{Z}_{\\geqslant 0} \\times \\mathbb{Z}_{\\geqslant 0} \\mid x+y \\alpha \\leqslant a+b \\alpha<(x+1)+y \\alpha\\right\\}+1=f(x+1, y)-f(x, y) . \\end{gathered} $$ From this claim we immediately get that $2500 \\leqslant N \\leqslant 7500$; now we show that those bounds are indeed sharp. Remember that if $\\alpha$ is irrational then $$ f(a, b)=\\#\\left\\{(x, y) \\in \\mathbb{Z}_{\\geqslant 0} \\times \\mathbb{Z}_{\\geqslant 0} \\mid x+y \\alphaA B$, let $O$ be its circumcentre, and let $D$ be a point on the segment $B C$. The line through $D$ perpendicular to $B C$ intersects the lines $A O, A C$ and $A B$ at $W, X$ and $Y$, respectively. The circumcircles of triangles $A X Y$ and $A B C$ intersect again at $Z \\neq A$. Prove that if $O W=O D$, then $D Z$ is tangent to the circle $A X Y$. (United Kingdom)","solution":"Let $A O$ intersect $B C$ at $E$. As $E D W$ is a right-angled triangle and $O$ is on $W E$, the condition $O W=O D$ means $O$ is the circumcentre of this triangle. So $O D=O E$ which establishes that $D, E$ are reflections in the perpendicular bisector of $B C$. Now observe: $$ 180^{\\circ}-\\angle D X Z=\\angle Z X Y=\\angle Z A Y=\\angle Z C D $$ which shows $C D X Z$ is cyclic. ![](https:\/\/cdn.mathpix.com\/cropped\/2024_11_18_86efb4389f14b52b08e1g-51.jpg?height=955&width=706&top_left_y=1076&top_left_x=681) We next show that $A Z \\| B C$. To do this, introduce point $Z^{\\prime}$ on circle $A B C$ such that $A Z^{\\prime} \\| B C$. By the previous result, it suffices to prove that $C D X Z^{\\prime}$ is cyclic. Notice that triangles $B A E$ and $C Z^{\\prime} D$ are reflections in the perpendicular bisector of $B C$. Using this and that $A, O, E$ are collinear: $$ \\angle D Z^{\\prime} C=\\angle B A E=\\angle B A O=90^{\\circ}-\\frac{1}{2} \\angle A O B=90^{\\circ}-\\angle C=\\angle D X C, $$ so $D X Z^{\\prime} C$ is cyclic, giving $Z \\equiv Z^{\\prime}$ as desired. Using $A Z \\| B C$ and $C D X Z$ cyclic we get: $$ \\angle A Z D=\\angle C D Z=\\angle C X Z=\\angle A Y Z, $$ which by the converse of alternate segment theorem shows $D Z$ is tangent to circle $A X Y$.","tier":0} +{"problem_type":"Geometry","problem_label":"G4","problem":"Let $A B C$ be an acute-angled triangle with $A C>A B$, let $O$ be its circumcentre, and let $D$ be a point on the segment $B C$. The line through $D$ perpendicular to $B C$ intersects the lines $A O, A C$ and $A B$ at $W, X$ and $Y$, respectively. The circumcircles of triangles $A X Y$ and $A B C$ intersect again at $Z \\neq A$. Prove that if $O W=O D$, then $D Z$ is tangent to the circle $A X Y$. (United Kingdom)","solution":"Notice that point $Z$ is the Miquel-point of lines $A C, B C, B A$ and $D Y$; then $B, D, Z, Y$ and $C, D, X, Y$ are concyclic. Moreover, $Z$ is the centre of the spiral similarity that maps $B C$ to $Y X$. By $B C \\perp Y X$, the angle of that similarity is $90^{\\circ}$; hence the circles $A B C Z$ and $A X Y Z$ are perpendicular, therefore the radius $O Z$ in circle $A B C Z$ is tangent to circle $A X Y Z$. ![](https:\/\/cdn.mathpix.com\/cropped\/2024_11_18_86efb4389f14b52b08e1g-52.jpg?height=949&width=934&top_left_y=528&top_left_x=561) By $O W=O D$, the triangle $O W D$ is isosceles, and $$ \\angle Z O A=2 \\angle Z B A=2 \\angle Z B Y=2 \\angle Z D Y=\\angle O D W+\\angle D W O, $$ so $D$ lies on line $Z O$ that is tangent to circle $A X Y$.","tier":0} +{"problem_type":"Geometry","problem_label":"G5","problem":"Let $A B C$ be a triangle, and let $\\ell_{1}$ and $\\ell_{2}$ be two parallel lines. For $i=1,2$, let $\\ell_{i}$ meet the lines $B C, C A$, and $A B$ at $X_{i}, Y_{i}$, and $Z_{i}$, respectively. Suppose that the line through $X_{i}$ perpendicular to $B C$, the line through $Y_{i}$ perpendicular to $C A$, and finally the line through $Z_{i}$ perpendicular to $A B$, determine a non-degenerate triangle $\\Delta_{i}$. Show that the circumcircles of $\\Delta_{1}$ and $\\Delta_{2}$ are tangent to each other. (Vietnam)","solution":"Throughout the solutions, $\\Varangle(p, q)$ will denote the directed angle between lines $p$ and $q$, taken modulo $180^{\\circ}$. Let the vertices of $\\Delta_{i}$ be $D_{i}, E_{i}, F_{i}$, such that lines $E_{i} F_{i}, F_{i} D_{i}$ and $D_{i} E_{i}$ are the perpendiculars through $X, Y$ and $Z$, respectively, and denote the circumcircle of $\\Delta_{i}$ by $\\omega_{i}$. In triangles $D_{1} Y_{1} Z_{1}$ and $D_{2} Y_{2} Z_{2}$ we have $Y_{1} Z_{1} \\| Y_{2} Z_{2}$ because they are parts of $\\ell_{1}$ and $\\ell_{2}$. Moreover, $D_{1} Y_{1} \\| D_{2} Y_{2}$ are perpendicular to $A C$ and $D_{1} Z_{1} \\| D_{2} Z_{2}$ are perpendicular to $A B$, so the two triangles are homothetic and their homothetic centre is $Y_{1} Y_{2} \\cap Z_{1} Z_{2}=A$. Hence, line $D_{1} D_{2}$ passes through $A$. Analogously, line $E_{1} E_{2}$ passes through $B$ and $F_{1} F_{2}$ passes through $C$. ![](https:\/\/cdn.mathpix.com\/cropped\/2024_11_18_86efb4389f14b52b08e1g-53.jpg?height=872&width=1300&top_left_y=1049&top_left_x=384) The corresponding sides of $\\Delta_{1}$ and $\\Delta_{2}$ are parallel, because they are perpendicular to the respective sides of triangle $A B C$. Hence, $\\Delta_{1}$ and $\\Delta_{2}$ are either homothetic, or they can be translated to each other. Using that $B, X_{2}, Z_{2}$ and $E_{2}$ are concyclic, $C, X_{2}, Y_{2}$ and $F_{2}$ are concyclic, $Z_{2} E_{2} \\perp A B$ and $Y_{2}, F_{2} \\perp A C$ we can calculate $$ \\begin{aligned} \\Varangle\\left(E_{1} E_{2}, F_{1} F_{2}\\right) & =\\Varangle\\left(E_{1} E_{2}, X_{1} X_{2}\\right)+\\Varangle\\left(X_{1} X_{2}, F_{1} F_{2}\\right)=\\Varangle\\left(B E_{2}, B X_{2}\\right)+\\Varangle\\left(C X_{2}, C F_{2}\\right) \\\\ & =\\Varangle\\left(Z_{2} E_{2}, Z_{2} X_{2}\\right)+\\Varangle\\left(Y_{2} X_{2}, Y_{2} F_{2}\\right)=\\Varangle\\left(Z_{2} E_{2}, \\ell_{2}\\right)+\\Varangle\\left(\\ell_{2}, Y_{2} F_{2}\\right) \\\\ & =\\Varangle\\left(Z_{2} E_{2}, Y_{2} F_{2}\\right)=\\Varangle(A B, A C) \\not \\equiv 0, \\end{aligned} $$ and conclude that lines $E_{1} E_{2}$ and $F_{1} F_{2}$ are not parallel. Hence, $\\Delta_{1}$ and $\\Delta_{2}$ are homothetic; the lines $D_{1} D_{2}, E_{1} E_{2}$, and $F_{1} F_{2}$ are concurrent at the homothetic centre of the two triangles. Denote this homothetic centre by $H$. For $i=1,2$, using (1), and that $A, Y_{i}, Z_{i}$ and $D_{i}$ are concyclic, $$ \\begin{aligned} \\Varangle\\left(H E_{i}, H F_{i}\\right) & =\\Varangle\\left(E_{1} E_{2}, F_{1} F_{2}\\right)=\\Varangle(A B, A C) \\\\ & =\\Varangle\\left(A Z_{i}, A Y_{i}\\right)=\\Varangle\\left(D_{i} Z_{i}, D_{i} Y_{i}\\right)=\\Varangle\\left(D_{i} E_{i}, D_{i} F_{i}\\right), \\end{aligned} $$ so $H$ lies on circle $\\omega_{i}$. The same homothety that maps $\\Delta_{1}$ to $\\Delta_{2}$, sends $\\omega_{1}$ to $\\omega_{2}$ as well. Point $H$, that is the centre of the homothety, is a common point of the two circles, That finishes proving that $\\omega_{1}$ and $\\omega_{2}$ are tangent to each other.","tier":0} +{"problem_type":"Geometry","problem_label":"G5","problem":"Let $A B C$ be a triangle, and let $\\ell_{1}$ and $\\ell_{2}$ be two parallel lines. For $i=1,2$, let $\\ell_{i}$ meet the lines $B C, C A$, and $A B$ at $X_{i}, Y_{i}$, and $Z_{i}$, respectively. Suppose that the line through $X_{i}$ perpendicular to $B C$, the line through $Y_{i}$ perpendicular to $C A$, and finally the line through $Z_{i}$ perpendicular to $A B$, determine a non-degenerate triangle $\\Delta_{i}$. Show that the circumcircles of $\\Delta_{1}$ and $\\Delta_{2}$ are tangent to each other. (Vietnam)","solution":"As in the first solution, let the vertices of $\\Delta_{i}$ be $D_{i}, E_{i}, F_{i}$, such that $E_{i} F_{i}, F_{i} D_{i}$ and $D_{i} E_{i}$ are the perpendiculars through $X_{i}, Y_{i}$ and $Z_{i}$, respectively. In the same way we conclude that $\\left(A, D_{1}, D_{2}\\right),\\left(B, E_{1}, E_{2}\\right)$ and $\\left(C, F_{1}, F_{2}\\right)$ are collinear. The corresponding sides of triangles $A B C$ and $D_{i} E_{i} F_{i}$ are perpendicular to each other. Hence, there is a spiral similarity with rotation $\\pm 90^{\\circ}$ that maps $A B C$ to $D_{i} E_{i} F_{i}$; let $M_{i}$ be the centre of that similarity. Hence, $\\Varangle\\left(M_{i} A, M_{i} D_{i}\\right)=\\Varangle\\left(M_{i} B, M_{i} E_{i}\\right)=\\Varangle\\left(M_{i} C, M_{i} F_{i}\\right)=90^{\\circ}$. The circle with diameter $A D_{i}$ passes through $M_{i}, Y_{i}, Z_{i}$, so $M_{i}, A, Y_{i}, Z_{i}, D_{i}$ are concyclic; analogously $\\left(M_{i}, B, X_{i}, Z_{i}, E_{i}\\right.$ ) and ( $\\left.M_{i}, C, X_{i}, Y_{i}, F_{i}\\right)$ are concyclic. By applying Desargues' theorem to triangles $A B C$ and $D_{i} E_{i} F_{i}$ we conclude that the lines $A D_{i}, B E_{i}$ and $B F_{i}$ are concurrent; let their intersection be $H$. Since $\\left(A, D_{1} \\cdot D_{2}\\right),\\left(B, E_{1} \\cdot E_{2}\\right)$ and $\\left(C, F_{1}, F_{2}\\right)$ are collinear, we obtain the same point $H$ for $i=1$ and $i=2$. ![](https:\/\/cdn.mathpix.com\/cropped\/2024_11_18_86efb4389f14b52b08e1g-54.jpg?height=849&width=858&top_left_y=1100&top_left_x=602) By $\\Varangle(C B, C H)=\\Varangle\\left(C X_{i}, C F_{i}\\right)=\\Varangle\\left(Y_{i} X_{i}, Y_{i} F_{i}\\right)=\\Varangle\\left(Y_{i} Z_{i}, Y_{i} D_{i}\\right)=\\Varangle\\left(A Z_{i}, A D_{i}\\right)=$ $\\Varangle(A B, A H)$, point $H$ lies on circle $A B C$. Analogously, from $\\Varangle\\left(F_{i} D_{i}, F_{i} H\\right)=\\Varangle\\left(F_{i} Y_{i}, F_{i} C\\right)=\\Varangle\\left(X_{i} Y_{i}, X_{i} C\\right)=\\Varangle\\left(X_{i} Z_{i}, X_{i} B\\right)=$ $\\Varangle\\left(E_{i} Z_{i}, E_{i} B\\right)=\\Varangle\\left(E_{i} D_{i}, E_{i} H\\right)$, we can see that point $H$ lies on circle $D_{i} E_{i} F_{i}$ as well. Therefore, circles $A B C$ and $D_{i} E_{i} F_{i}$ intersect at point $H$. The spiral similarity moves the circle $A B C$ to circle $D_{i} E_{i} F_{i}$, so the two circles are perpendicular. Hence, both circles $D_{1} E_{1} F_{1}$ and $D_{2} E_{2} F_{2}$ are tangent to the radius of circle $A B C$ at $H$. Comment 1. As the last picture suggests, the circles $A B C$ and $D_{i} E_{i} F_{i}$ pass through $M_{i}$. In fact, point $M_{i}$, being the second intersection of circles $D_{i} E_{i} F_{i}$ and $D_{i} Y_{i} Z_{i}$, the Miquel point of the lines $A Y_{i}, A Z_{i}, C X_{i}$ and $X_{i} Y_{i}$, so it is concyclic with $A, B, C$. Similarly, $M_{i}$ the Miquel point of lines $D_{i} E_{i}$, $E_{i} F_{i}, F_{i} Y_{i}$ and $X_{i} Y_{i}$, so it is concyclic with $D_{i}, E_{i}, D_{i}$. Comment 2. Instead of two lines $\\ell_{1}$ and $\\ell_{2}$, it is possible to reformulate the problem with a single, varying line $\\ell$ : Let $A B C$ be a triangle, and let $\\ell$ be a varying line whose direction is fixed. Let $\\ell$ intersect lines $B C$, $C A$, and $A B$ at $X, Y$, and $Z$, respectively. Suppose that the line through $X$, perpendicular to $B C$, the line through $Y$, perpendicular to $C A$, and the line through $Z$, perpendicular to $A B$, determine a non-degenerate triangle $\\Delta$. Show that as $\\ell$ varies, the circumcircles of the obtained triangles $\\Delta$ pass through a fixed point, and these circles are tangent to each other. A reasonable approach is finding the position of line $\\ell$ when the triangle $D E F$ degenerates to a single point. That happens when the line $X Y Z$ is the Simson line respect to point $D=E=F$ on the circumcircle $A B C$. Based on this observation a possible variant of the solution is as follows. Let $H$ be the second intersection of circles $A B C$ and line $A D$. Like in the solutions above, we can find that the line $A D$ is fixed, so $H$ is independent of the position of line $\\ell$. ![](https:\/\/cdn.mathpix.com\/cropped\/2024_11_18_86efb4389f14b52b08e1g-55.jpg?height=846&width=795&top_left_y=819&top_left_x=633) From $\\Varangle(H F, H D)=\\Varangle(H C, H A)=\\Varangle(B C, B A)=\\Varangle(B X, B Z)=\\Varangle(E X, E Z)=\\Varangle(E F, E D)$ we can see that circle $\\Delta$ passes through $H$. Hence, all circles $D E F$ passes through a fixed point. The corresponding sides of triangles $A B C$ and $D E F$ are perpendicular, so their circumcircle are perpendicular; that proves that circle $D E F$ is tangent to the radius of circle $A B C$ at $H$.","tier":0} +{"problem_type":"Geometry","problem_label":"G6","problem":"In an acute-angled triangle $A B C$, point $H$ is the foot of the altitude from $A$. Let $P$ be a moving point such that the bisectors $k$ and $\\ell$ of angles $P B C$ and $P C B$, respectively, intersect each other on the line segment $A H$. Let $k$ and $A C$ meet at $E$, let $\\ell$ and $A B$ meet at $F$, and let $E F$ and $A H$ meet at $Q$. Prove that, as $P$ varies, the line $P Q$ passes through a fixed point. (Iran)","solution":"Let the reflections of the line $B C$ with respect to the lines $A B$ and $A C$ intersect at point $K$. We will prove that $P, Q$ and $K$ are collinear, so $K$ is the common point of the varying line $P Q$. Let lines $B E$ and $C F$ intersect at $I$. For every point $O$ and $d>0$, denote by $(O, d)$ the circle centred at $O$ with radius $d$, and define $\\omega_{I}=(I, I H)$ and $\\omega_{A}=(A, A H)$. Let $\\omega_{K}$ and $\\omega_{P}$ be the incircle of triangle $K B C$ and the $P$-excircle of triangle $P B C$, respectively. Since $I H \\perp B C$ and $A H \\perp B C$, the circles $\\omega_{A}$ and $\\omega_{I}$ are tangent to each other at $H$. So, $H$ is the external homothetic centre of $\\omega_{A}$ and $\\omega_{I}$. From the complete quadrangle $B C E F$ we have $(A, I ; Q, H)=-1$, therefore $Q$ is the internal homothetic centre of $\\omega_{A}$ and $\\omega_{I}$. Since $B A$ and $C A$ are the external bisectors of angles $\\angle K B C$ and $\\angle K C B$, circle $\\omega_{A}$ is the $K$-excircle in triangle $B K C$. Hence, $K$ is the external homothetic centre of $\\omega_{A}$ and $\\omega_{K}$. Also it is clear that $P$ is the external homothetic centre of $\\omega_{I}$ and $\\omega_{P}$. Let point $T$ be the tangency point of $\\omega_{P}$ and $B C$, and let $T^{\\prime}$ be the tangency point of $\\omega_{K}$ and $B C$. Since $\\omega_{I}$ is the incircle and $\\omega_{P}$ is the $P$-excircle of $P B C, T C=B H$ and since $\\omega_{K}$ is the incircle and $\\omega_{A}$ is the $K$-excircle of $K B C, T^{\\prime} C=B H$. Therefore $T C=T^{\\prime} C$ and $T \\equiv T^{\\prime}$. It yields that $\\omega_{K}$ and $\\omega_{P}$ are tangent to each other at $T$. ![](https:\/\/cdn.mathpix.com\/cropped\/2024_11_18_86efb4389f14b52b08e1g-56.jpg?height=930&width=1169&top_left_y=1491&top_left_x=449) Let point $S$ be the internal homothetic centre of $\\omega_{A}$ and $\\omega_{P}$, and let $S^{\\prime}$ be the internal homothetic centre of $\\omega_{I}$ and $\\omega_{K}$. It's obvious that $S$ and $S^{\\prime}$ lie on $B C$. We claim that $S \\equiv S^{\\prime}$. To prove our claim, let $r_{A}, r_{I}, r_{P}$, and $r_{K}$ be the radii of $\\omega_{A}, \\omega_{I}, \\omega_{P}$ and $\\omega_{k}$, respectively. It is well known that if the sides of a triangle are $a, b, c$, its semiperimeter is $s=(a+b+c) \/ 2$, and the radii of the incircle and the $a$-excircle are $r$ and $r_{a}$, respectively, then $r \\cdot r_{a}=(s-b)(s-c)$. Applying this fact to triangle $P B C$ we get $r_{I} \\cdot r_{P}=B H \\cdot C H$. The same fact in triangle $K C B$ yields $r_{K} \\cdot r_{A}=C T \\cdot B T$. Since $B H=C T$ and $B T=C H$, from these two we get $$ \\frac{H S}{S T}=\\frac{r_{A}}{r_{P}}=\\frac{r_{I}}{r_{K}}=\\frac{H S^{\\prime}}{S^{\\prime} T} $$ so $S=S^{\\prime}$ indeed. Finally, by applying the generalised Monge's theorem to the circles $\\omega_{A}, \\omega_{I}$, and $\\omega_{K}$ (with two pairs of internal and one pair of external common tangents), we can see that points $Q$, $S$, and $K$ are collinear. Similarly one can show that $Q, S$ and $P$ are collinear, and the result follows.","tier":0} +{"problem_type":"Geometry","problem_label":"G6","problem":"In an acute-angled triangle $A B C$, point $H$ is the foot of the altitude from $A$. Let $P$ be a moving point such that the bisectors $k$ and $\\ell$ of angles $P B C$ and $P C B$, respectively, intersect each other on the line segment $A H$. Let $k$ and $A C$ meet at $E$, let $\\ell$ and $A B$ meet at $F$, and let $E F$ and $A H$ meet at $Q$. Prove that, as $P$ varies, the line $P Q$ passes through a fixed point. (Iran)","solution":"Again, let $B E$ and $C F$ meet at $I$, that is the incentre in triangle $B C P$; then $P I$ is the third angle bisector. From the tangent segments of the incircle we have $B P-C P=$ $B H-C H$; hence, the possible points $P$ lie on a branch of a hyperbola $\\mathcal{H}$ with foci $B, C$, and $H$ is a vertex of $\\mathcal{H}$. Since $P I$ bisects the angle between the radii $B P$ and $C P$ of the hyperbola, line $P I$ is tangent to $\\mathcal{H}$. ![](https:\/\/cdn.mathpix.com\/cropped\/2024_11_18_86efb4389f14b52b08e1g-57.jpg?height=1066&width=1306&top_left_y=1049&top_left_x=378) Let $K$ be the second intersection of $P Q$ and $\\mathcal{H}$, we will show that $A K$ is tangent to $\\mathcal{H}$ at $K$; this property determines $K$. Let $G=K I \\cap A P$ and $M=P I \\cap A K$. From the complete quadrangle $B C E F$ we can see that $(H, Q ; I, A)$ is harmonic, so in the complete quadrangle $A P I K$, point $H$ lies on line $G M$. Consider triangle $A I M$. Its side $A I$ is tangent to $\\mathcal{H}$ at $H$, the side $I M$ is tangent to $\\mathcal{H}$ at $P$, and $K$ is a common point of the third side $A M$ and the hyperbola such that the lines $A P$, $I K$ and $M H$ are concurrent at the generalised Gergonne-point $G$. It follows that the third side, $A M$ is also tangent to $\\mathcal{H}$ at $K$. (Alternatively, in the last step we can apply the converse of Brianchon's theorem to the degenerate hexagon $A H I P M K$. By the theorem there is a conic section $\\mathcal{H}^{\\prime}$ such that lines $A I, I M$ and $M A$ are tangent to $\\mathcal{H}^{\\prime}$ at $H, P$ and $K$, respectively. But the three points $H, K$ and $P$, together with the tangents at $H$ and $P$ uniquely determine $\\mathcal{H}^{\\prime}$, so indeed $\\mathcal{H}^{\\prime}=\\mathcal{H}$.)","tier":0} +{"problem_type":"Geometry","problem_label":"G7","problem":"Let $A B C$ and $A^{\\prime} B^{\\prime} C^{\\prime}$ be two triangles having the same circumcircle $\\omega$, and the same orthocentre $H$. Let $\\Omega$ be the circumcircle of the triangle determined by the lines $A A^{\\prime}, B B^{\\prime}$ and $C C^{\\prime}$. Prove that $H$, the centre of $\\omega$, and the centre of $\\Omega$ are collinear. (Denmark)","solution":"In what follows, $\\Varangle(p, q)$ will denote the directed angle between lines $p$ and $q$, taken modulo $180^{\\circ}$. Denote by $O$ the centre of $\\omega$. In any triangle, the homothety with ratio $-\\frac{1}{2}$ centred at the centroid of the triangle takes the vertices to the midpoints of the opposite sides and it takes the orthocentre to the circumcentre. Therefore the triangles $A B C$ and $A^{\\prime} B^{\\prime} C^{\\prime}$ share the same centroid $G$ and the midpoints of their sides lie on a circle $\\rho$ with centre on $O H$. We will prove that $\\omega, \\Omega$, and $\\rho$ are coaxial, so in particular it follows that their centres are collinear on OH . Let $D=B B^{\\prime} \\cap C C^{\\prime}, E=C C^{\\prime} \\cap A A^{\\prime}, F=A A^{\\prime} \\cap B B^{\\prime}, S=B C^{\\prime} \\cap B^{\\prime} C$, and $T=B C \\cap B^{\\prime} C^{\\prime}$. Since $D, S$, and $T$ are the intersections of opposite sides and of the diagonals in the quadrilateral $B B^{\\prime} C C^{\\prime}$ inscribed in $\\omega$, by Brocard's theorem triangle $D S T$ is self-polar with respect to $\\omega$, i.e. each vertex is the pole of the opposite side. We apply this in two ways. ![](https:\/\/cdn.mathpix.com\/cropped\/2024_11_18_86efb4389f14b52b08e1g-58.jpg?height=835&width=1589&top_left_y=1053&top_left_x=239) First, from $D$ being the pole of $S T$ it follows that the inverse $D^{*}$ of $D$ with respect to $\\omega$ is the projection of $D$ onto $S T$. In particular, $D^{*}$ lies on the circle with diameter $S D$. If $N$ denotes the midpoint of $S D$ and $R$ the radius of $\\omega$, then the power of $O$ with respect to this circle is $O N^{2}-N D^{2}=O D \\cdot O D^{*}=R^{2}$. By rearranging, we see that $N D^{2}$ is the power of $N$ with respect to $\\omega$. Second, from $T$ being the pole of $S D$ it follows that $O T$ is perpendicular to $S D$. Let $M$ and $M^{\\prime}$ denote the midpoints of $B C$ and $B^{\\prime} C^{\\prime}$. Then since $O M \\perp B C$ and $O M^{\\prime} \\perp B^{\\prime} C^{\\prime}$ it follows that $O M M^{\\prime} T$ is cyclic and $$ \\Varangle(S D, B C)=\\Varangle(O T, O M)=\\Varangle\\left(B^{\\prime} C^{\\prime}, M M^{\\prime}\\right) . $$ From $B B^{\\prime} C C^{\\prime}$ being cyclic we also have $\\Varangle\\left(B C, B B^{\\prime}\\right)=\\Varangle\\left(C C^{\\prime}, B^{\\prime} C^{\\prime}\\right)$, hence we obtain $$ \\begin{aligned} \\Varangle\\left(S D, B B^{\\prime}\\right) & =\\Varangle(S D, B C)+\\Varangle\\left(B C, B B^{\\prime}\\right) \\\\ & =\\Varangle\\left(B^{\\prime} C^{\\prime}, M M^{\\prime}\\right)+\\Varangle\\left(C C^{\\prime}, B^{\\prime} C^{\\prime}\\right)=\\Varangle\\left(C C^{\\prime}, M M^{\\prime}\\right) . \\end{aligned} $$ Now from the homothety mentioned in the beginning, we know that $M M^{\\prime}$ is parallel to $A A^{\\prime}$, hence the above implies that $\\Varangle\\left(S D, B B^{\\prime}\\right)=\\Varangle\\left(C C^{\\prime}, A A^{\\prime}\\right)$, which shows that $\\Omega$ is tangent to $S D$ at $D$. In particular, $N D^{2}$ is also the power of $N$ with respect to $\\Omega$. Additionally, from $B B^{\\prime} C C^{\\prime}$ being cyclic it follows that triangles $D B C$ and $D C^{\\prime} B^{\\prime}$ are inversely similar, so $\\Varangle\\left(B B^{\\prime}, D M^{\\prime}\\right)=\\Varangle\\left(D M, C C^{\\prime}\\right)$. This yields $$ \\begin{aligned} \\Varangle\\left(S D, D M^{\\prime}\\right) & =\\Varangle\\left(S D, B B^{\\prime}\\right)+\\Varangle\\left(B B^{\\prime}, D M^{\\prime}\\right) \\\\ & =\\Varangle\\left(C C^{\\prime}, M M^{\\prime}\\right)+\\Varangle\\left(D M, C C^{\\prime}\\right)=\\Varangle\\left(D M, M M^{\\prime}\\right), \\end{aligned} $$ which shows that the circle $D M M^{\\prime}$ is also tangent to $S D$. Since $N, M$, and $M^{\\prime}$ are collinear on the Newton-Gauss line of the complete quadrilateral determined by the lines $B B^{\\prime}, C C^{\\prime}, B C^{\\prime}$, and $B^{\\prime} C$, it follows that $N D^{2}=N M \\cdot N M^{\\prime}$. Hence $N$ has the same power with respect to $\\omega$, $\\Omega$, and $\\rho$. By the same arguments there exist points on the tangents to $\\Omega$ at $E$ and $F$ which have the same power with respect to $\\omega, \\Omega$, and $\\rho$. The tangents to a given circle at three distinct points cannot be concurrent, hence we obtain at least two distinct points with the same power with respect to $\\omega, \\Omega$, and $\\rho$. Hence the three circles are coaxial, as desired. Comment 1. Instead of invoking the Newton-Gauss line, one can also use a nice symmetry argument: If from the beginning we swapped the labels of $B^{\\prime}$ and $C^{\\prime}$, then in the proof above the labels of $D$ and $S$ would be swapped while the labels of $M$ and $M^{\\prime}$ do not change. The consequence is that the circle $S M M^{\\prime}$ is also tangent to $S D$. Since $N$ is the midpoint of $S D$ it then has the same power with respect to circles $D M M^{\\prime}$ and $S M M^{\\prime}$, so it lies on their radical axis $M M^{\\prime}$. Comment 2. There exists another triple of points on the common radical axis of $\\omega, \\Omega$, and $\\rho$ which can be used to solve the problem. We outline one such solution. ![](https:\/\/cdn.mathpix.com\/cropped\/2024_11_18_86efb4389f14b52b08e1g-59.jpg?height=752&width=1026&top_left_y=1297&top_left_x=518) Let $L$ and $L^{\\prime}$ denote the feet of the altitudes from $A$ and $A^{\\prime}$ in triangle $A B C$ and $A^{\\prime} B^{\\prime} C^{\\prime}$, respectively. Since $\\rho$ is the nine-point circle of the two triangles it contains both $L$ and $L^{\\prime}$. Furthermore, $H A \\cdot H L$ and $H A^{\\prime} \\cdot H L^{\\prime}$ both equal twice the power of $H$ with respect to $\\rho$, so $A, A^{\\prime}, L, L^{\\prime}$ are concyclic as well. Now let $\\ell=A A^{\\prime}$ and denote $P=L L^{\\prime} \\cap \\ell, K=B C \\cap \\ell$, and $K^{\\prime}=B^{\\prime} C^{\\prime} \\cap \\ell$. As $M M^{\\prime} \\| \\ell$ (shown in the previous solution) and $L L^{\\prime} M M^{\\prime}$ is cyclic $$ \\Varangle(B C, \\ell)=\\Varangle\\left(B C, M M^{\\prime}\\right)=\\Varangle\\left(L L^{\\prime}, B^{\\prime} C^{\\prime}\\right) $$ so $K, K^{\\prime}, L$, and $L^{\\prime}$ are also concyclic. From the cyclic quadrilaterals $A A^{\\prime} L L^{\\prime}$ and $K K^{\\prime} L L^{\\prime}$ we get $P A \\cdot P A^{\\prime}=P L \\cdot P L^{\\prime}=P K \\cdot P K^{\\prime}$. This implies that $P$ is the centre of the (unique) involution $\\sigma$ on $\\ell$ that swaps $A, A^{\\prime}$ and $K, K^{\\prime}$. On the other hand, by Desargues' involution theorem applied to the line $\\ell$, the quadrilateral $B B^{\\prime} C C^{\\prime}$, and its circumcircle $\\omega$, the involution $\\sigma$ also swaps $E$ and $F$. Hence $$ P A \\cdot P A^{\\prime}=P L \\cdot P L^{\\prime}=P E \\cdot P F $$ However, this means that $P$ has the same power with respect to $\\omega, \\Omega$, and $\\rho$, and by the same arguments there exist points on $B B^{\\prime}$ and $C C^{\\prime}$ with this property.","tier":0} +{"problem_type":"Geometry","problem_label":"G8","problem":"Let $A A^{\\prime} B C C^{\\prime} B^{\\prime}$ be a convex cyclic hexagon such that $A C$ is tangent to the incircle of the triangle $A^{\\prime} B^{\\prime} C^{\\prime}$, and $A^{\\prime} C^{\\prime}$ is tangent to the incircle of the triangle $A B C$. Let the lines $A B$ and $A^{\\prime} B^{\\prime}$ meet at $X$ and let the lines $B C$ and $B^{\\prime} C^{\\prime}$ meet at $Y$. Prove that if $X B Y B^{\\prime}$ is a convex quadrilateral, then it has an incircle.","solution":"Denote by $\\omega$ and $\\omega^{\\prime}$ the incircles of $\\triangle A B C$ and $\\triangle A^{\\prime} B^{\\prime} C^{\\prime}$ and let $I$ and $I^{\\prime}$ be the centres of these circles. Let $N$ and $N^{\\prime}$ be the second intersections of $B I$ and $B^{\\prime} I^{\\prime}$ with $\\Omega$, the circumcircle of $A^{\\prime} B C C^{\\prime} B^{\\prime} A$, and let $O$ be the centre of $\\Omega$. Note that $O N \\perp A C, O N^{\\prime} \\perp A^{\\prime} C^{\\prime}$ and $O N=O N^{\\prime}$ so $N N^{\\prime}$ is parallel to the angle bisector $I I^{\\prime}$ of $A C$ and $A^{\\prime} C^{\\prime}$. Thus $I I^{\\prime} \\| N N^{\\prime}$ which is antiparallel to $B B^{\\prime}$ with respect to $B I$ and $B^{\\prime} I^{\\prime}$. Therefore $B, I, I^{\\prime}, B^{\\prime}$ are concyclic. ![](https:\/\/cdn.mathpix.com\/cropped\/2024_11_18_86efb4389f14b52b08e1g-60.jpg?height=958&width=1072&top_left_y=926&top_left_x=498) Further define $P$ as the intersection of $A C$ and $A^{\\prime} C^{\\prime}$ and $M$ as the antipode of $N^{\\prime}$ in $\\Omega$. Consider the circle $\\Gamma_{1}$ with centre $N$ and radius $N A=N C$ and the circle $\\Gamma_{2}$ with centre $M$ and radius $M A^{\\prime}=M C^{\\prime}$. Their radical axis passes through $P$ and is perpendicular to $M N \\perp N N^{\\prime} \\| I P$, so $I$ lies on their radical axis. Therefore, since $I$ lies on $\\Gamma_{1}$, it must also lie on $\\Gamma_{2}$. Thus, if we define $Z$ as the second intersection of $M I$ with $\\Omega$, we have that $I$ is the incentre of triangle $Z A^{\\prime} C^{\\prime}$. (Note that the point $Z$ can also be constructed directly via Poncelet's porism.) Consider the incircle $\\omega_{c}$ with centre $I_{c}$ of triangle $C^{\\prime} B^{\\prime} Z$. Note that $\\angle Z I C^{\\prime}=90^{\\circ}+$ $\\frac{1}{2} \\angle Z A^{\\prime} C^{\\prime}=90^{\\circ}+\\frac{1}{2} \\angle Z B^{\\prime} C^{\\prime}=\\angle Z I_{c} C^{\\prime}$, so $Z, I, I_{c}, C^{\\prime}$ are concyclic. Similarly $B^{\\prime}, I^{\\prime}, I_{c}, C^{\\prime}$ are concyclic. The external centre of dilation from $\\omega$ to $\\omega_{c}$ is the intersection of $I I_{c}$ and $C^{\\prime} Z$ ( $D$ in the picture), that is the radical centre of circles $\\Omega, C^{\\prime} I_{c} I Z$ and $I I^{\\prime} I_{c}$. Similarly, the external centre of dilation from $\\omega^{\\prime}$ to $\\omega_{c}$ is the intersection of $I^{\\prime} I_{c}$ and $B^{\\prime} C^{\\prime}$ ( $D^{\\prime}$ in the picture), that is the radical centre of circles $\\Omega, B^{\\prime} I^{\\prime} I_{c} C^{\\prime}$ and $I I^{\\prime} I_{c}$. Therefore the Monge line of $\\omega, \\omega^{\\prime}$ and $\\omega_{c}$ is line $D D^{\\prime}$, and the radical axis of $\\Omega$ and circle $I I^{\\prime} I_{c}$ coincide. Hence the external centre $T$ of dilation from $\\omega$ to $\\omega^{\\prime}$ is also on the radical axis of $\\Omega$ and circle $I I^{\\prime} I_{c}$. ![](https:\/\/cdn.mathpix.com\/cropped\/2024_11_18_86efb4389f14b52b08e1g-61.jpg?height=1029&width=964&top_left_y=228&top_left_x=546) Now since $B, I, I^{\\prime}, B^{\\prime}$ are concyclic, the intersection $T^{\\prime}$ of $B B^{\\prime}$ and $I I^{\\prime}$ is on the radical axis of $\\Omega$ and circle $I I^{\\prime} I_{c}$. Thus $T^{\\prime}=T$ and $T$ lies on line $B B^{\\prime}$. Finally, construct a circle $\\Omega_{0}$ tangent to $A^{\\prime} B^{\\prime}, B^{\\prime} C^{\\prime}, A B$ on the same side of these lines as $\\omega^{\\prime}$. The centre of dilation from $\\omega^{\\prime}$ to $\\Omega_{0}$ is $B^{\\prime}$, so by Monge's theorem the external centre of dilation from $\\Omega_{0}$ to $\\omega$ must be on the line $T B B^{\\prime}$. However, it is on line $A B$, so it must be $B$ and $B C$ must be tangent to $\\Omega_{0}$ as desired. ![](https:\/\/cdn.mathpix.com\/cropped\/2024_11_18_86efb4389f14b52b08e1g-61.jpg?height=918&width=1357&top_left_y=1637&top_left_x=361)","tier":0} +{"problem_type":"Number Theory","problem_label":"N1","problem":"A number is called Norwegian if it has three distinct positive divisors whose sum is equal to 2022. Determine the smallest Norwegian number. (Note: The total number of positive divisors of a Norwegian number is allowed to be larger than 3.) (Cyprus)","solution":"Observe that 1344 is a Norwegian number as 6, 672 and 1344 are three distinct divisors of 1344 and $6+672+1344=2022$. It remains to show that this is the smallest such number. Assume for contradiction that $N<1344$ is Norwegian and let $N \/ a, N \/ b$ and $N \/ c$ be the three distinct divisors of $N$, with $a\\frac{2022}{1344}=\\frac{337}{224}=\\frac{3}{2}+\\frac{1}{224} $$ If $a>1$ then $$ \\frac{1}{a}+\\frac{1}{b}+\\frac{1}{c} \\leqslant \\frac{1}{2}+\\frac{1}{3}+\\frac{1}{4}=\\frac{13}{12}<\\frac{3}{2} $$ so it must be the case that $a=1$. Similarly, it must hold that $b<4$ since otherwise $$ 1+\\frac{1}{b}+\\frac{1}{c} \\leqslant 1+\\frac{1}{4}+\\frac{1}{5}<\\frac{3}{2} . $$ This leaves two cases to check, $b=2$ and $b=3$. Case $b=3$. Then $$ \\frac{1}{c}>\\frac{3}{2}+\\frac{1}{224}-1-\\frac{1}{3}>\\frac{1}{6}, $$ so $c=4$ or $c=5$. If $c=4$ then $$ 2022=N\\left(1+\\frac{1}{3}+\\frac{1}{4}\\right)=\\frac{19}{12} N $$ but this is impossible as $19 \\nmid 2022$. If $c=5$ then $$ 2022=N\\left(1+\\frac{1}{3}+\\frac{1}{5}\\right)=\\frac{23}{15} N $$ which again is impossible, as $23 \\nmid 2022$. Case $b=2$. Note that $c<224$ since $$ \\frac{1}{c}>\\frac{3}{2}+\\frac{1}{224}-1-\\frac{1}{2}=\\frac{1}{224} $$ It holds that $$ 2022=N\\left(1+\\frac{1}{2}+\\frac{1}{c}\\right)=\\frac{3 c+2}{2 c} N \\Rightarrow(3 c+2) N=4044 c $$ Since $(c, 3 c-2)=(c, 2) \\in\\{1,2\\}$, then $3 c+2 \\mid 8088=2^{3} \\cdot 3 \\cdot 337$ which implies that $3 c+2 \\mid 2^{3} \\cdot 337$. But since $3 c+2 \\geqslant 3 \\cdot 3+2>8=2^{3}$ and $3 c+2 \\neq 337$, then it must hold that $3 c+2 \\geqslant 2 \\cdot 337$, contradicting $c<224$.","tier":0} +{"problem_type":"Number Theory","problem_label":"N2","problem":"Find all positive integers $n>2$ such that $$ n!\\mid \\prod_{\\substack{p2$ and 3 divides one of $p_{m-2}, p_{m-1}=p_{m-2}+2$ and $p_{m}=p_{m-2}+4$. This implies $p_{m-2}=3$ and thus $p_{m}=7$, giving $7 \\leqslant n<11$. Finally, a quick computation shows that $7!\\mid \\prod_{p1$ be a positive integer, and let $d>1$ be a positive integer coprime to $a$. Let $x_{1}=1$ and, for $k \\geqslant 1$, define $$ x_{k+1}= \\begin{cases}x_{k}+d & \\text { if } a \\text { doesn't divide } x_{k} \\\\ x_{k} \/ a & \\text { if } a \\text { divides } x_{k}\\end{cases} $$ Find the greatest positive integer $n$ for which there exists an index $k$ such that $x_{k}$ is divisible by $a^{n}$. (Croatia)","solution":"By trivial induction, $x_{k}$ is coprime to $d$. By induction and the fact that there can be at most $a-1$ consecutive increasing terms in the sequence, it also holds that $x_{k}1$ be a positive integer, and let $d>1$ be a positive integer coprime to $a$. Let $x_{1}=1$ and, for $k \\geqslant 1$, define $$ x_{k+1}= \\begin{cases}x_{k}+d & \\text { if } a \\text { doesn't divide } x_{k} \\\\ x_{k} \/ a & \\text { if } a \\text { divides } x_{k}\\end{cases} $$ Find the greatest positive integer $n$ for which there exists an index $k$ such that $x_{k}$ is divisible by $a^{n}$. (Croatia)","solution":"Like in the first solution, $x_{k}$ is relatively prime to $d$ and $x_{k}0}: 0d$ then $y-d \\in S$ but $a \\cdot y \\notin S$; otherwise, if $yd, \\\\ a \\cdot y & \\text { if } y1$ be an index such that $x_{k_{1}}=1$. Then $$ x_{k_{1}}=1, \\quad x_{k_{1}-1}=f^{-1}(1)=a, x_{k_{1}-2}=f^{-1}(a)=a^{2}, \\ldots, \\quad x_{k_{1}-n}=a^{n} $$","tier":0} +{"problem_type":"Number Theory","problem_label":"N3","problem":"Let $a>1$ be a positive integer, and let $d>1$ be a positive integer coprime to $a$. Let $x_{1}=1$ and, for $k \\geqslant 1$, define $$ x_{k+1}= \\begin{cases}x_{k}+d & \\text { if } a \\text { doesn't divide } x_{k} \\\\ x_{k} \/ a & \\text { if } a \\text { divides } x_{k}\\end{cases} $$ Find the greatest positive integer $n$ for which there exists an index $k$ such that $x_{k}$ is divisible by $a^{n}$. (Croatia)","solution":"Like in the first solution, $x_{k}$ is relatively prime to $d$ and $x_{k}a^{n} \/ a=a^{n-1}$ the LHS is strictly less than $a^{v-n}$. This implies that on the RHS, the coefficients of $a^{v-n}, a^{v-n+1}, \\ldots$ must all be zero, i.e. $z_{v-n}=z_{v-n+1}=\\cdots=z_{v-1}=0$. This implies that there are $n$ consecutive decreasing indices in the original sequence.","tier":0} +{"problem_type":"Number Theory","problem_label":"N4","problem":"Find all triples of positive integers $(a, b, p)$ with $p$ prime and $$ a^{p}=b!+p $$ (Belgium)","solution":"Clearly, $a>1$. We consider three cases. Case 1: We have $ab$ which is also impossible since in this case we have $b!\\leqslant a!a>1$. Case 2: We have $a>p$. In this case $b!=a^{p}-p>p^{p}-p \\geqslant p!$ so $b>p$ which means that $a^{p}=b!+p$ is divisible by $p$. Hence, $a$ is divisible by $p$ and $b!=a^{p}-p$ is not divisible by $p^{2}$. This means that $b<2 p$. If $a(2 p-1)!+p \\geqslant b!+p$. Comment. The inequality $p^{2 p}>(2 p-1)!+p$ can be shown e.g. by using $$ (2 p-1)!=[1 \\cdot(2 p-1)] \\cdot[2 \\cdot(2 p-2)] \\cdots \\cdots[(p-1)(p+1)] \\cdot p<\\left(\\left(\\frac{2 p}{2}\\right)^{2}\\right)^{p-1} \\cdot p=p^{2 p-1} $$ where the inequality comes from applying AM-GM to each of the terms in square brackets. Case 3: We have $a=p$. In this case $b!=p^{p}-p$. One can check that the values $p=2,3$ lead to the claimed solutions and $p=5$ does not lead to a solution. So we now assume that $p \\geqslant 7$. We have $b!=p^{p}-p>p$ ! and so $b \\geqslant p+1$ which implies that $v_{2}((p+1)!) \\leqslant v_{2}(b!)=v_{2}\\left(p^{p-1}-1\\right) \\stackrel{L T E}{=} 2 v_{2}(p-1)+v_{2}(p+1)-1=v_{2}\\left(\\frac{p-1}{2} \\cdot(p-1) \\cdot(p+1)\\right)$, where in the middle we used lifting-the-exponent lemma. On the RHS we have three factors of $(p+1)!$. But, due to $p+1 \\geqslant 8$, there are at least 4 even numbers among $1,2, \\ldots, p+1$, so this case is not possible.","tier":0} +{"problem_type":"Number Theory","problem_label":"N4","problem":"Find all triples of positive integers $(a, b, p)$ with $p$ prime and $$ a^{p}=b!+p $$ (Belgium)","solution":"The cases $a \\neq p$ are covered as in solution 1 , as are $p=2,3$. For $p \\geqslant 5$ we have $b!=p\\left(p^{p-1}-1\\right)$. By Zsigmondy's Theorem there exists some prime $q$ that divides $p^{p-1}-1$ but does not divide $p^{k}-1$ for $k(2 p-1)^{p-1} p>p^{p}>p^{p}-p$, a contradiction.","tier":0} +{"problem_type":"Number Theory","problem_label":"N4","problem":"Find all triples of positive integers $(a, b, p)$ with $p$ prime and $$ a^{p}=b!+p $$ (Belgium)","solution":"The cases $a \\neq p$ are covered as in solution 1 , as are $p=2,3$. Also $b>p$, as $p^{p}>p!+p$ for $p>2$. The cases $p=5,7,11$ are also checked manually, so assume $p \\geqslant 13$. Let $q \\mid p+1$ be an odd prime. By LTE $$ v_{q}\\left(p^{p}-p\\right)=v_{q}\\left(\\left(p^{2}\\right)^{\\frac{p-1}{2}}-1\\right)=v_{q}\\left(p^{2}-1\\right)+v_{q}\\left(\\frac{p-1}{2}\\right)=v_{q}(p+1) $$ But $b \\geqslant p+1$, so then $v_{q}(b!)>v_{q}(p+1)$, since $qq^{d-1} \\geqslant 2 d $$ provided $d \\geqslant 2$ and $q>3$, or $d \\geqslant 3$. If $q=3, d=2$ and $p \\geqslant 13$ then $v_{q}(b!) \\geqslant v_{q}(p!) \\geqslant v_{q}(13!)=5>2 d$. Either way, $d \\leqslant 1$. If $p>2 q+1$ (so $p>3 q$, as $q \\mid p-1$ ) then $$ v_{q}(b!) \\geqslant v_{q}((3 q)!)=3, $$ so we must have $q \\geqslant \\frac{p}{2}$, in other words, $p-1=2 q$. This implies that $p=2^{k}-1$ and $q=2^{k-1}-1$ are both prime, but it is not possible to have two consecutive Mersenne primes.","tier":0} +{"problem_type":"Number Theory","problem_label":"N4","problem":"Find all triples of positive integers $(a, b, p)$ with $p$ prime and $$ a^{p}=b!+p $$ (Belgium)","solution":"Let $a=p, b>p$ and $p \\geqslant 5$ (the remaining cases are dealt with as in solution 3). Modulo $(p+1)^{2}$ it holds that $p^{p}-p=(p+1-1)^{p}-p \\equiv\\binom{p}{1}(p+1)(-1)^{p-1}+(-1)^{p}-p=p(p+1)-1-p=p^{2}-1 \\not \\equiv 0 \\quad \\bmod \\left((p+1)^{2}\\right)$. Since $p \\geqslant 5$, the numbers 2 and $\\frac{p+1}{2}$ are distinct and less than or equal to $p$. Therefore, $p+1 \\mid p$, and so $(p+1)^{2} \\mid(p+1)$ !. But $b \\geqslant p+1$, so $b!\\equiv 0 \\not \\equiv p^{p}-p \\bmod (p+1)^{2}$, a contradiction.","tier":0} +{"problem_type":"Number Theory","problem_label":"N5","problem":"For each $1 \\leqslant i \\leqslant 9$ and $T \\in \\mathbb{N}$, define $d_{i}(T)$ to be the total number of times the digit $i$ appears when all the multiples of 1829 between 1 and $T$ inclusive are written out in base 10. Show that there are infinitely many $T \\in \\mathbb{N}$ such that there are precisely two distinct values among $d_{1}(T), d_{2}(T), \\ldots, d_{9}(T)$. (United Kingdom)","solution":"Let $n:=1829$. First, we choose some $k$ such that $n \\mid 10^{k}-1$. For instance, any multiple of $\\varphi(n)$ would work since $n$ is coprime to 10 . We will show that either $T=10^{k}-1$ or $T=10^{k}-2$ has the desired property, which completes the proof since $k$ can be taken to be arbitrary large. For this it suffices to show that $\\#\\left\\{d_{i}\\left(10^{k}-1\\right): 1 \\leqslant i \\leqslant 9\\right\\} \\leqslant 2$. Indeed, if $$ \\#\\left\\{d_{i}\\left(10^{k}-1\\right): 1 \\leqslant i \\leqslant 9\\right\\}=1 $$ then, since $10^{k}-1$ which consists of all nines is a multiple of $n$, we have $$ d_{i}\\left(10^{k}-2\\right)=d_{i}\\left(10^{k}-1\\right) \\text { for } i \\in\\{1, \\ldots, 8\\}, \\text { and } d_{9}\\left(10^{k}-2\\right)0$ independent of the set $Q$ such that for any positive integer $N>100$, the number of special integers in $[1, N]$ is at least $c N$. (For example, if $Q=\\{3,7\\}$, then $p(42)=3, q(42)=2, p(63)=3, q(63)=3, p(2022)=3$, $q(2022)=1$. (Costa Rica)","solution":"Let us call two positive integers $m, n$ friends if $p(m)+p(n)$ and $q(m)+q(n)$ are both even integers. We start by noting that the pairs $(p(k), q(k))$ modulo 2 can take at most 4 different values; thus, among any five different positive integers there are two which are friends. In addition, both functions $p$ and $q$ satisfy $f(a b)=f(a)+f(b)$ for any $a, b$. Therefore, if $m$ and $n$ are divisible by $d$, then both $p$ and $q$ satisfy the equality $f(m)+f(n)=f(m \/ d)+$ $f(n \/ d)+2 f(d)$. This implies that $m, n$ are friends if and only if $m \/ d, n \/ d$ are friends. Let us call a set of integers $\\left\\{n_{1}, n_{2}, \\ldots, n_{5}\\right\\}$ an interesting set if for any indexes $i, j$, the difference $d_{i j}=\\left|n_{i}-n_{j}\\right|$ divides both $n_{i}$ and $n_{j}$. We claim that if elements of an interesting set are all positive, then we can obtain a special integer. Indeed, if we were able to construct such a set, then there would be a pair of integers $\\left\\{n_{i}, n_{j}\\right\\}$ which are friends, according to the first observation. Additionally, the second observation yields that the quotients $n_{i} \/ d_{i j}, n_{j} \/ d_{i j}$ form a pair of friends, which happen to be consecutive integers, thus giving a special integer as desired. In order to construct a family of interesting sets, we can start by observing that the set $\\{0,6,8,9,12\\}$ is an interesting set. Using that $72=2^{3} \\cdot 3^{2}$ is the least common multiple of all pairwise differences in this set, we obtain a family of interesting sets by considering $$ \\{72 k, 72 k+6,72 k+8,72 k+9,72 k+12\\} $$ for any $k \\geqslant 1$. If we consider the quotients (of these numbers by the appropriate differences), then we obtain that the set $$ S_{k}=\\{6 k, 8 k, 9 k, 12 k, 12 k+1,18 k+2,24 k+2,24 k+3,36 k+3,72 k+8\\}, $$ has at least one special integer. In particular, the interval $[1,100 k]$ contains the sets $S_{1}, S_{2}, \\ldots, S_{k}$, each of which has a special number. Any special number can be contained in at most ten sets $S_{k}$, from where we conclude that the number of special integers in $[1,100 k]$ is at least $k \/ 10$. Finally, let $N=100 k+r$, with $k \\geqslant 1$ and $0 \\leqslant r<100$, so that we have $N<100(k+1) \\leqslant$ $200 k$. Then the number of special integers in $[1, N]$ is at least $k \/ 10>N \/ 2000$, as we wanted to prove. Comment 1. The statement is also true for $N \\geqslant 15$ as at least one of the numbers $7,14,15$ is special. Comment 2. Another approach would be to note that if $p(2 n), p(2 n+1), p(2 n+2)$ all have the same parity then one of the numbers $n, 2 n, 2 n+1$ is special. Indeed, if $q(n)+q(n+1)$ is even then $n$ is special since $p(n)+p(n+1) \\equiv p(2 n)+p(2 n+2) \\equiv 0(\\bmod 2)$. Otherwise, if $q(n)+q(n+1)$ is odd, so is $q(2 n)+q(2 n+2)$ which implies that exactly one of the numbers $2 n, 2 n+1$ is special. Unfortunately, it seems hard to show that the set of such $n$ has positive density: see a recent paper https:\/\/arxiv.org\/abs\/1509.01545 for the proof that all eight patterns of the parities of $p(n), p(n+1), p(n+2)$ appear for a positive proportion of positive integers. This page is intentionally left blank","tier":0} +{"problem_type":"Number Theory","problem_label":"N7","problem":"Let $k$ be a positive integer and let $S$ be a finite set of odd prime numbers. Prove that there is at most one way (modulo rotation and reflection) to place the elements of $S$ around a circle such that the product of any two neighbors is of the form $x^{2}+x+k$ for some positive integer $x$. (U.S.A.)","solution":"Let us allow the value $x=0$ as well; we prove the same statement under this more general constraint. Obviously that implies the statement with the original conditions. Call a pair $\\{p, q\\}$ of primes with $p \\neq q$ special if $p q=x^{2}+x+k$ for some nonnegative integer $x$. The following claim is the key mechanism of the problem: Claim. (a) For every prime $r$, there are at most two primes less than $r$ forming a special pair with $r$. (b) If such $p$ and $q$ exist, then $\\{p, q\\}$ is itself special. We present two proofs of the claim. Proof 1. We are interested in integers $1 \\leqslant x\\sqrt{m}$ choices for $x$ and $y$, so there are more than $m$ possible pairs $(x, y)$. Hence, two of these sums are congruent modulo $m$ : $5^{k+1} x_{1}+3^{k+1} y_{1} \\equiv 5^{k+1} x_{2}+3^{k+1} y_{2}(\\bmod m)$. Now choose $a=x_{1}-x_{2}$ and $b=y_{1}-y_{2}$; at least one of $a, b$ is nonzero, and $$ 5^{k+1} a+3^{k+1} b \\equiv 0 \\quad(\\bmod m), \\quad|a|,|b| \\leqslant \\sqrt{m} $$ From $$ 0 \\equiv\\left(5^{k+1} a\\right)^{2}-\\left(3^{k+1} b\\right)^{2}=5^{n+1} a^{2}-3^{n+1} b^{2} \\equiv 5 \\cdot 3^{n} a^{2}-3^{n+1} b^{2}=3^{n}\\left(5 a^{2}-3 b^{2}\\right) \\quad(\\bmod m) $$ we can see that $\\left|5 a^{2}-3 b^{2}\\right|$ is a multiple of $m$. Since at least one of $a$ and $b$ is nonzero, $5 a^{2} \\neq 3 b^{2}$. Hence, by the choice of $a, b$, we have $0<\\left|5 a^{2}-3 b^{2}\\right| \\leqslant \\max \\left(5 a^{2}, 3 b^{2}\\right) \\leqslant 5 m$. That shows that $m_{1} \\leqslant 5 m$. II. Next, we show that $m_{1}$ cannot be divisible by 2,3 and 5 . Since $m_{1}$ equals either $\\left|5 a^{2}-3 b^{2}\\right|$ or $\\left|a^{2}-15 b^{2}\\right|$ with some integers $a, b$, we have six cases to check. In all six cases, we will get a contradiction by presenting another multiple of $m$, smaller than $m_{1}$. - If $5 \\mid m_{1}$ and $m_{1}=\\left|5 a^{2}-3 b^{2}\\right|$, then $5 \\mid b$ and $\\left|a^{2}-15\\left(\\frac{b}{5}\\right)^{2}\\right|=\\frac{m_{1}}{5}1$ and $c$ are integers, and $X, Y$ are positive integers such that $X \\leqslant mm$, then $c x^{2} \\equiv d y^{2}(\\bmod m)$ has a solution such that at least one of $x, y$ is nonzero, $|x|5$ of $5 a^{2}-3 b^{2}$ or $a^{2}-15 b^{2}$, we have $$ 1=\\left(\\frac{15}{p}\\right)=\\left(\\frac{3}{p}\\right)\\left(\\frac{5}{p}\\right)=(-1)^{\\frac{p-1}{2}}\\left(\\frac{p}{3}\\right)\\left(\\frac{p}{5}\\right) $$ where $\\left(\\frac{a}{p}\\right)$ stands for the Legendre symbol. Considering the remainders of $p$ when divided by 4,3 and 5, (1) leads to $$ p \\equiv \\pm 1, \\pm 7, \\pm 11 \\text { or } \\pm 17 \\bmod 60 $$ These remainders form a subgroup of the reduced remainders modulo 60. Since 13 and 37 are not elements in this subgroup, the number $m=2^{n}+65$ cannot be a product of such primes. Instead of handling the prime divisors of $m$ separately, we can use Jacobi symbols for further simplification, as shown in the next solution.","tier":0} +{"problem_type":"Number Theory","problem_label":"N8","problem":"Prove that $5^{n}-3^{n}$ is not divisible by $2^{n}+65$ for any positive integer $n$.","solution":"Suppose again that $5^{n} \\equiv 3^{n}\\left(\\bmod m=2^{n}+65\\right)$. Like in the first solution, we conclude that $n$ must be odd, and $n \\geqslant 3$, so $8 \\mid 2^{n}$. Using Jacobi symbols, $$ -1=\\left(\\frac{2^{n}+65}{5}\\right)=\\left(\\frac{5}{2^{n}+65}\\right)=\\left(\\frac{5^{n}}{2^{n}+65}\\right)=\\left(\\frac{3^{n}}{2^{n}+65}\\right)=\\left(\\frac{3}{2^{n}+65}\\right)=\\left(\\frac{2^{n}+65}{3}\\right)=1 $$ contradiction. ## - ![](https:\/\/cdn.mathpix.com\/cropped\/2024_11_18_86efb4389f14b52b08e1g-75.jpg?height=98&width=726&top_left_y=279&top_left_x=114) + - \\$4=4 \\times 1.20\\$1 ![](https:\/\/cdn.mathpix.com\/cropped\/2024_11_18_86efb4389f14b52b08e1g-75.jpg?height=84&width=889&top_left_y=1990&top_left_x=379) Cly D - 1 vaiv ![](https:\/\/cdn.mathpix.com\/cropped\/2024_11_18_86efb4389f14b52b08e1g-75.jpg?height=58&width=116&top_left_y=2247&top_left_x=536) +1N ![](https:\/\/cdn.mathpix.com\/cropped\/2024_11_18_86efb4389f14b52b08e1g-75.jpg?height=135&width=646&top_left_y=2301&top_left_x=348)![](https:\/\/cdn.mathpix.com\/cropped\/2024_11_18_86efb4389f14b52b08e1g-75.jpg?height=1874&width=164&top_left_y=419&top_left_x=1607) ![](https:\/\/cdn.mathpix.com\/cropped\/2024_11_18_86efb4389f14b52b08e1g-75.jpg?height=130&width=432&top_left_y=1647&top_left_x=0) r \/ A M - 18 ![](https:\/\/cdn.mathpix.com\/cropped\/2024_11_18_86efb4389f14b52b08e1g-75.jpg?height=73&width=652&top_left_y=2395&top_left_x=139) ![](https:\/\/cdn.mathpix.com\/cropped\/2024_11_18_86efb4389f14b52b08e1g-75.jpg?height=58&width=637&top_left_y=2447&top_left_x=154) A ..... ![](https:\/\/cdn.mathpix.com\/cropped\/2024_11_18_86efb4389f14b52b08e1g-75.jpg?height=75&width=415&top_left_y=2501&top_left_x=148) ![](https:\/\/cdn.mathpix.com\/cropped\/2024_11_18_86efb4389f14b52b08e1g-75.jpg?height=61&width=452&top_left_y=2539&top_left_x=605) ![](https:\/\/cdn.mathpix.com\/cropped\/2024_11_18_86efb4389f14b52b08e1g-75.jpg?height=75&width=532&top_left_y=2587&top_left_x=599)","tier":0} +{"problem_type":"Algebra","problem_label":"A2","problem":"Let $k \\geqslant 2$ be an integer. Find the smallest integer $n \\geqslant k+1$ with the property that there exists a set of $n$ distinct real numbers such that each of its elements can be written as a sum of $k$ other distinct elements of the set. (Slovakia) Answer: $n=k+4$.","solution":"First we show that $n \\geqslant k+4$. Suppose that there exists such a set with $n$ numbers and denote them by $a_{1}a_{1}+\\cdots+a_{k} \\geqslant a_{k+1}$, which gives a contradiction. If $n=k+2$ then we have $a_{1} \\geqslant a_{2}+\\cdots+a_{k+1} \\geqslant a_{k+2}$, that again gives a contradiction. If $n=k+3$ then we have $a_{1} \\geqslant a_{2}+\\cdots+a_{k+1}$ and $a_{3}+\\cdots+a_{k+2} \\geqslant a_{k+3}$. Adding the two inequalities we get $a_{1}+a_{k+2} \\geqslant a_{2}+a_{k+3}$, again a contradiction. It remains to give an example of a set with $k+4$ elements satisfying the condition of the problem. We start with the case when $k=2 l$ and $l \\geqslant 1$. In that case, denote by $A_{i}=\\{-i, i\\}$ and take the set $A_{1} \\cup \\cdots \\cup A_{l+2}$, which has exactly $k+4=2 l+4$ elements. We are left to show that this set satisfies the required condition. Note that if a number $i$ can be expressed in the desired way, then so can $-i$ by negating the expression. Therefore, we consider only $1 \\leqslant i \\leqslant l+2$. If $i0}$ be the set of positive real numbers. Find all functions $f: \\mathbb{R}_{>0} \\rightarrow \\mathbb{R}_{>0}$ such that, for every $x \\in \\mathbb{R}_{>0}$, there exists a unique $y \\in \\mathbb{R}_{>0}$ satisfying $$ x f(y)+y f(x) \\leqslant 2 . $$ (Netherlands) Answer: The function $f(x)=1 \/ x$ is the only solution.","solution":"First we prove that the function $f(x)=1 \/ x$ satisfies the condition of the problem statement. The AM-GM inequality gives $$ \\frac{x}{y}+\\frac{y}{x} \\geqslant 2 $$ for every $x, y>0$, with equality if and only if $x=y$. This means that, for every $x>0$, there exists a unique $y>0$ such that $$ \\frac{x}{y}+\\frac{y}{x} \\leqslant 2, $$ namely $y=x$. Let now $f: \\mathbb{R}_{>0} \\rightarrow \\mathbb{R}_{>0}$ be a function that satisfies the condition of the problem statement. We say that a pair of positive real numbers $(x, y)$ is $\\operatorname{good}$ if $x f(y)+y f(x) \\leqslant 2$. Observe that if $(x, y)$ is good, then so is $(y, x)$. Lemma 1.0. If $(x, y)$ is good, then $x=y$. Proof. Assume that there exist positive real numbers $x \\neq y$ such that $(x, y)$ is good. The uniqueness assumption says that $y$ is the unique positive real number such that $(x, y)$ is good. In particular, $(x, x)$ is not a good pair. This means that $$ x f(x)+x f(x)>2 $$ and thus $x f(x)>1$. Similarly, $(y, x)$ is a good pair, so $(y, y)$ is not a good pair, which implies $y f(y)>1$. We apply the AM-GM inequality to obtain $$ x f(y)+y f(x) \\geqslant 2 \\sqrt{x f(y) \\cdot y f(x)}=2 \\sqrt{x f(x) \\cdot y f(y)}>2 . $$ This is a contradiction, since $(x, y)$ is a good pair. By assumption, for any $x>0$, there always exists a good pair containing $x$, however Lemma 1 implies that the only good pair that can contain $x$ is $(x, x)$, so $$ x f(x) \\leqslant 1 \\quad \\Longleftrightarrow \\quad f(x) \\leqslant \\frac{1}{x} $$ for every $x>0$. In particular, with $x=1 \/ f(t)$ for $t>0$, we obtain $$ \\frac{1}{f(t)} \\cdot f\\left(\\frac{1}{f(t)}\\right) \\leqslant 1 $$ Hence $$ t \\cdot f\\left(\\frac{1}{f(t)}\\right) \\leqslant t f(t) \\leqslant 1 $$ We claim that $(t, 1 \/ f(t))$ is a good pair for every $t>0$. Indeed, $$ t \\cdot f\\left(\\frac{1}{f(t)}\\right)+\\frac{1}{f(t)} f(t)=t \\cdot f\\left(\\frac{1}{f(t)}\\right)+1 \\leqslant 2 $$ Lemma 1 implies that $t=1 \/ f(t) \\Longleftrightarrow f(t)=1 \/ t$ for every $t>0$.","tier":0} +{"problem_type":"Algebra","problem_label":"A3","problem":"Let $\\mathbb{R}_{>0}$ be the set of positive real numbers. Find all functions $f: \\mathbb{R}_{>0} \\rightarrow \\mathbb{R}_{>0}$ such that, for every $x \\in \\mathbb{R}_{>0}$, there exists a unique $y \\in \\mathbb{R}_{>0}$ satisfying $$ x f(y)+y f(x) \\leqslant 2 . $$ (Netherlands) Answer: The function $f(x)=1 \/ x$ is the only solution.","solution":"1. We give an alternative way to prove that $f(x)=1 \/ x$ assuming $f(x) \\leqslant 1 \/ x$ for every $x>0$. Indeed, if $f(x)<1 \/ x$ then for every $a>0$ with $f(x)<1 \/ a<1 \/ x$ (and there are at least two of them), we have $$ a f(x)+x f(a)<1+\\frac{x}{a}<2 $$ Hence $(x, a)$ is a good pair for every such $a$, a contradiction. We conclude that $f(x)=1 \/ x$.","tier":0} +{"problem_type":"Algebra","problem_label":"A3","problem":"Let $\\mathbb{R}_{>0}$ be the set of positive real numbers. Find all functions $f: \\mathbb{R}_{>0} \\rightarrow \\mathbb{R}_{>0}$ such that, for every $x \\in \\mathbb{R}_{>0}$, there exists a unique $y \\in \\mathbb{R}_{>0}$ satisfying $$ x f(y)+y f(x) \\leqslant 2 . $$ (Netherlands) Answer: The function $f(x)=1 \/ x$ is the only solution.","solution":"2. We can also conclude from Lemma 1 and $f(x) \\leqslant 1 \/ x$ as follows. Lemma 2. The function $f$ is decreasing. Proof. Let $y>x>0$. Lemma 1 says that $(x, y)$ is not a good pair, but $(y, y)$ is. Hence $$ x f(y)+y f(x)>2 \\geqslant 2 y f(y)>y f(y)+x f(y) $$ where we used $y>x$ (and $f(y)>0$ ) in the last inequality. This implies that $f(x)>f(y)$, showing that $f$ is decreasing. We now prove that $f(x)=1 \/ x$ for all $x$. Fix a value of $x$ and note that for $y>x$ we must have $x f(x)+y f(x)>x f(y)+y f(x)>2$ (using that $f$ is decreasing for the first step), hence $f(x)>\\frac{2}{x+y}$. The last inequality is true for every $y>x>0$. If we fix $x$ and look for the supremum of the expression $\\frac{2}{x+y}$ over all $y>x$, we get $$ f(x) \\geqslant \\frac{2}{x+x}=\\frac{1}{x} $$ Since we already know that $f(x) \\leqslant 1 \/ x$, we conclude that $f(x)=1 \/ x$.","tier":0} +{"problem_type":"Algebra","problem_label":"A3","problem":"Let $\\mathbb{R}_{>0}$ be the set of positive real numbers. Find all functions $f: \\mathbb{R}_{>0} \\rightarrow \\mathbb{R}_{>0}$ such that, for every $x \\in \\mathbb{R}_{>0}$, there exists a unique $y \\in \\mathbb{R}_{>0}$ satisfying $$ x f(y)+y f(x) \\leqslant 2 . $$ (Netherlands) Answer: The function $f(x)=1 \/ x$ is the only solution.","solution":"0. As in the first solution, we note that $f(x)=1 \/ x$ is a solution, and we set out to prove that it is the only one. We write $g(x)$ for the unique positive real number such that $(x, g(x))$ is a good pair. In this solution, we prove Lemma 2 without assuming Lemma 1. Lemma 2. The function $f$ is decreasing. Proof. Consider $x$ 2. Combining these two inequalities yields $$ x f(g(y))+g(y) f(x)>2 \\geqslant y f(g(y))+g(y) f(y) $$ or $f(g(y))(x-y)>g(y)(f(y)-f(x))$. Because $g(y)$ and $f(g(y))$ are both positive while $x-y$ is negative, it follows that $f(y)2$ and $y f(y)+y f(y)>2$, which implies $x f(x)+y f(y)>2$. Now $$ x f(x)+y f(y)>2 \\geqslant x f(y)+y f(x) $$ implies $(x-y)(f(x)-f(y))>0$, which contradicts the fact that $f$ is decreasing. So $y=x$ is the unique $y$ such that $(x, y)$ is a good pair, and in particular we have $f(x) \\leqslant 1 \/ x$. We can now conclude the proof as in any of the Solutions 1.x.","tier":0} +{"problem_type":"Algebra","problem_label":"A3","problem":"Let $\\mathbb{R}_{>0}$ be the set of positive real numbers. Find all functions $f: \\mathbb{R}_{>0} \\rightarrow \\mathbb{R}_{>0}$ such that, for every $x \\in \\mathbb{R}_{>0}$, there exists a unique $y \\in \\mathbb{R}_{>0}$ satisfying $$ x f(y)+y f(x) \\leqslant 2 . $$ (Netherlands) Answer: The function $f(x)=1 \/ x$ is the only solution.","solution":"0. As in the other solutions we verify that the function $f(x)=1 \/ x$ is a solution. We first want to prove the following lemma: Lemma 3. For all $x \\in \\mathbb{R}_{>0}$ we actually have $x f(g(x))+g(x) f(x)=2$ (that is: the inequality is actually an equality). Proof. We proceed by contradiction: Assume there exists some number $x>0$ such that for $y=g(x)$ we have $x f(y)+y f(x)<2$. Then for any $0<\\epsilon<\\frac{2-x f(y)-y f(x)}{2 f(x)}$ we have, by uniqueness of $y$, that $x f(y+\\epsilon)+(y+\\epsilon) f(x)>2$. Therefore $$ \\begin{aligned} f(y+\\epsilon) & >\\frac{2-(y+\\epsilon) f(x)}{x}=\\frac{2-y f(x)-\\epsilon f(x)}{x} \\\\ & >\\frac{2-y f(x)-\\frac{2-x f(y)-y f(x)}{2}}{x} \\\\ & =\\frac{2-x f(y)-y f(x)}{2 x}+f(y)>f(y) . \\end{aligned} $$ Furthermore, for every such $\\epsilon$ we have $g(y+\\epsilon) f(y+\\epsilon)+(y+\\epsilon) f(g(y+\\epsilon)) \\leqslant 2$ and $g(y+\\epsilon) f(y)+y f(g(y+\\epsilon))>2($ since $y \\neq y+\\epsilon=g(g(y+\\epsilon))$ ). This gives us the two inequalities $$ f(g(y+\\epsilon)) \\leqslant \\frac{2-g(y+\\epsilon) f(y+\\epsilon)}{y+\\epsilon} \\quad \\text { and } \\quad f(g(y+\\epsilon))>\\frac{2-g(y+\\epsilon) f(y)}{y} . $$ Combining these two inequalities and rearranging the terms leads to the inequality $$ 2 \\epsilon0}$ we have $$ x f(y)+y f(x) \\geqslant 2 $$ since for $y=g(x)$ we have equality and by uniqueness for $y \\neq g(x)$ the inequality is strict. In particular for every $x \\in \\mathbb{R}_{>0}$ and for $y=x$ we have $2 x f(x) \\geqslant 2$, or equivalently $f(x) \\geqslant 1 \/ x$ for all $x \\in \\mathbb{R}_{>0}$. With this inequality we obtain for all $x \\in \\mathbb{R}_{>0}$ $$ 2 \\geqslant x f(g(x))+g(x) f(x) \\geqslant \\frac{x}{g(x)}+\\frac{g(x)}{x} \\geqslant 2 $$ where the first inequality comes from the problem statement. Consequently each of these inequalities must actually be an equality, and in particular we obtain $f(x)=1 \/ x$ for all $x \\in \\mathbb{R}_{>0}$.","tier":0} +{"problem_type":"Algebra","problem_label":"A3","problem":"Let $\\mathbb{R}_{>0}$ be the set of positive real numbers. Find all functions $f: \\mathbb{R}_{>0} \\rightarrow \\mathbb{R}_{>0}$ such that, for every $x \\in \\mathbb{R}_{>0}$, there exists a unique $y \\in \\mathbb{R}_{>0}$ satisfying $$ x f(y)+y f(x) \\leqslant 2 . $$ (Netherlands) Answer: The function $f(x)=1 \/ x$ is the only solution.","solution":"Again, let us prove that $f(x)=1 \/ x$ is the only solution. Let again $g(x)$ be the unique positive real number such that $(x, g(x))$ is a good pair. Lemma 4. The function $f$ is strictly convex. Proof. Consider the function $q_{s}(x)=f(x)+s x$ for some real number $s$. If $f$ is not strictly convex, then there exist $u0$ such that the $\\frac{1}{2} n(n-1)$ differences $a_{j}-a_{i}$ for $1 \\leqslant i1$ of $x^{2}-x-1=0$ (the golden ratio) and set $\\left(a_{1}, a_{2}, a_{3}\\right)=$ $\\left(0, r, r+r^{2}\\right)$. Then $$ \\left(a_{2}-a_{1}, a_{3}-a_{2}, a_{3}-a_{1}\\right)=\\left(r, r^{2}, r+r^{2}=r^{3}\\right) $$ For $n=4$, take the root $r \\in(1,2)$ of $x^{3}-x-1=0$ (such a root exists because $1^{3}-1-1<0$ and $\\left.2^{3}-2-1>0\\right)$ and set $\\left(a_{1}, a_{2}, a_{3}, a_{4}\\right)=\\left(0, r, r+r^{2}, r+r^{2}+r^{3}\\right)$. Then $$ \\left(a_{2}-a_{1}, a_{3}-a_{2}, a_{4}-a_{3}, a_{3}-a_{1}, a_{4}-a_{2}, a_{4}-a_{1}\\right)=\\left(r, r^{2}, r^{3}, r^{4}, r^{5}, r^{6}\\right) $$ For $n \\geqslant 5$, we will proceed by contradiction. Suppose there exist numbers $a_{1}<\\cdots1$ satisfying the conditions of the problem. We start with a lemma: Lemma. We have $r^{n-1}>2$. Proof. There are only $n-1$ differences $a_{j}-a_{i}$ with $j=i+1$, so there exists an exponent $e \\leqslant n$ and a difference $a_{j}-a_{i}$ with $j \\geqslant i+2$ such that $a_{j}-a_{i}=r^{e}$. This implies $$ r^{n} \\geqslant r^{e}=a_{j}-a_{i}=\\left(a_{j}-a_{j-1}\\right)+\\left(a_{j-1}-a_{i}\\right)>r+r=2 r, $$ thus $r^{n-1}>2$ as desired. To illustrate the general approach, we first briefly sketch the idea behind the argument in the special case $n=5$. In this case, we clearly have $a_{5}-a_{1}=r^{10}$. Note that there are 3 ways to rewrite $a_{5}-a_{1}$ as a sum of two differences, namely $$ \\left(a_{5}-a_{4}\\right)+\\left(a_{4}-a_{1}\\right),\\left(a_{5}-a_{3}\\right)+\\left(a_{3}-a_{1}\\right),\\left(a_{5}-a_{2}\\right)+\\left(a_{2}-a_{1}\\right) . $$ Using the lemma above and convexity of the function $f(n)=r^{n}$, we argue that those three ways must be $r^{10}=r^{9}+r^{1}=r^{8}+r^{4}=r^{7}+r^{6}$. That is, the \"large\" exponents keep dropping by 1 , while the \"small\" exponents keep increasing by $n-2, n-3, \\ldots, 2$. Comparing any two such equations, we then get a contradiction unless $n \\leqslant 4$. Now we go back to the full proof for any $n \\geqslant 5$. Denote $b=\\frac{1}{2} n(n-1)$. Clearly, we have $a_{n}-a_{1}=r^{b}$. Consider the $n-2$ equations of the form: $$ a_{n}-a_{1}=\\left(a_{n}-a_{i}\\right)+\\left(a_{i}-a_{1}\\right) \\text { for } i \\in\\{2, \\ldots, n-1\\} $$ In each equation, one of the two terms on the right-hand side must be at least $\\frac{1}{2}\\left(a_{n}-a_{1}\\right)$. But from the lemma we have $r^{b-(n-1)}=r^{b} \/ r^{n-1}<\\frac{1}{2}\\left(a_{n}-a_{1}\\right)$, so there are at most $n-2$ sufficiently large elements in $\\left\\{r^{k} \\mid 1 \\leqslant k1$ and $f(r)=r^{n}$ is convex, we have $$ r^{b-1}-r^{b-2}>r^{b-2}-r^{b-3}>\\ldots>r^{b-(n-3)}-r^{b-(n-2)} $$ implying $$ r^{\\alpha_{2}}-r^{\\alpha_{1}}>r^{\\alpha_{3}}-r^{\\alpha_{2}}>\\ldots>r^{\\alpha_{n-2}}-r^{\\alpha_{n-3}} . $$ Convexity of $f(r)=r^{n}$ further implies $$ \\alpha_{2}-\\alpha_{1}>\\alpha_{3}-\\alpha_{2}>\\ldots>\\alpha_{n-2}-\\alpha_{n-3} $$ Note that $\\alpha_{n-2}-\\alpha_{n-3} \\geqslant 2$ : Otherwise we would have $\\alpha_{n-2}-\\alpha_{n-3}=1$ and thus $$ r^{\\alpha_{n-3}} \\cdot(r-1)=r^{\\alpha_{n-2}}-r^{\\alpha_{n-3}}=r^{b-(n-3)}-r^{b-(n-2)}=r^{b-(n-2)} \\cdot(r-1), $$ implying that $\\alpha_{n-3}=b-(n-2)$, a contradiction. Therefore, we have $$ \\begin{aligned} \\alpha_{n-2}-\\alpha_{1} & =\\left(\\alpha_{n-2}-\\alpha_{n-3}\\right)+\\cdots+\\left(\\alpha_{2}-\\alpha_{1}\\right) \\\\ & \\geqslant 2+3+\\cdots+(n-2) \\\\ & =\\frac{1}{2}(n-2)(n-1)-1=\\frac{1}{2} n(n-3) \\end{aligned} $$ On the other hand, from $\\alpha_{n-2} \\leqslant b-(n-1)$ and $\\alpha_{1} \\geqslant 1$ we get $$ \\alpha_{n-2}-\\alpha_{1} \\leqslant b-n=\\frac{1}{2} n(n-1)-n=\\frac{1}{2} n(n-3), $$ implying that equalities must occur everywhere and the claim about the small terms follows. Now, assuming $n-2 \\geqslant 2$, we have the two different equations: $$ r^{b}=r^{b-(n-2)}+r^{b-(n-2)-1} \\text { and } r^{b}=r^{b-(n-3)}+r^{b-(n-2)-3}, $$ which can be rewritten as $$ r^{n-1}=r+1 \\quad \\text { and } \\quad r^{n+1}=r^{4}+1 $$ Simple algebra now gives $$ r^{4}+1=r^{n+1}=r^{n-1} \\cdot r^{2}=r^{3}+r^{2} \\Longrightarrow(r-1)\\left(r^{3}-r-1\\right)=0 . $$ Since $r \\neq 1$, using Equation (1) we conclude $r^{3}=r+1=r^{n-1}$, thus $n=4$, which gives a contradiction.","tier":0} +{"problem_type":"Algebra","problem_label":"A6","problem":"Let $\\mathbb{R}$ be the set of real numbers. We denote by $\\mathcal{F}$ the set of all functions $f: \\mathbb{R} \\rightarrow \\mathbb{R}$ such that $$ f(x+f(y))=f(x)+f(y) $$ for every $x, y \\in \\mathbb{R}$. Find all rational numbers $q$ such that for every function $f \\in \\mathcal{F}$, there exists some $z \\in \\mathbb{R}$ satisfying $f(z)=q z$. (Indonesia) Answer: The desired set of rational numbers is $\\left\\{\\frac{n+1}{n}: n \\in \\mathbb{Z}, n \\neq 0\\right\\}$.","solution":"Let $Z$ be the set of all rational numbers $q$ such that for every function $f \\in \\mathcal{F}$, there exists some $z \\in \\mathbb{R}$ satisfying $f(z)=q z$. Let further $$ S=\\left\\{\\frac{n+1}{n}: n \\in \\mathbb{Z}, n \\neq 0\\right\\} $$ We prove that $Z=S$ by showing the two inclusions: $S \\subseteq Z$ and $Z \\subseteq S$. We first prove that $S \\subseteq Z$. Let $f \\in \\mathcal{F}$ and let $P(x, y)$ be the relation $f(x+f(y))=f(x)+$ $f(y)$. First note that $P(0,0)$ gives $f(f(0))=2 f(0)$. Then, $P(0, f(0))$ gives $f(2 f(0))=3 f(0)$. We claim that $$ f(k f(0))=(k+1) f(0) $$ for every integer $k \\geqslant 1$. The claim can be proved by induction. The cases $k=1$ and $k=2$ have already been established. Assume that $f(k f(0))=(k+1) f(0)$ and consider $P(0, k f(0))$ which gives $$ f((k+1) f(0))=f(0)+f(k f(0))=(k+2) f(0) $$ This proves the claim. We conclude that $\\frac{k+1}{k} \\in Z$ for every integer $k \\geqslant 1$. Note that $P(-f(0), 0)$ gives $f(-f(0))=0$. We now claim that $$ f(-k f(0))=(-k+1) f(0) $$ for every integer $k \\geqslant 1$. The proof by induction is similar to the one above. We conclude that $\\frac{-k+1}{-k} \\in Z$ for every integer $k \\geqslant 1$. This shows that $S \\subseteq Z$. We now prove that $Z \\subseteq S$. Let $p$ be a rational number outside the set $S$. We want to prove that $p$ does not belong to $Z$. To that end, we construct a function $f \\in \\mathcal{F}$ such that $f(z) \\neq p z$ for every $z \\in \\mathbb{R}$. The strategy is to first construct a function $$ g:[0,1) \\rightarrow \\mathbb{Z} $$ and then define $f$ as $f(x)=g(\\{x\\})+\\lfloor x\\rfloor$. This function $f$ belongs to $\\mathcal{F}$. Indeed, $$ \\begin{aligned} f(x+f(y)) & =g(\\{x+f(y)\\})+\\lfloor x+f(y)\\rfloor \\\\ & =g(\\{x+g(\\{y\\})+\\lfloor y\\rfloor\\})+\\lfloor x+g(\\{y\\})+\\lfloor y\\rfloor\\rfloor \\\\ & =g(\\{x\\})+\\lfloor x\\rfloor+g(\\{y\\})+\\lfloor y\\rfloor \\\\ & =f(x)+f(y) \\end{aligned} $$ where we used that $g$ only takes integer values. Lemma 1. For every $\\alpha \\in[0,1)$, there exists $m \\in \\mathbb{Z}$ such that $$ m+n \\neq p(\\alpha+n) $$ for every $n \\in \\mathbb{Z}$. Proof. Note that if $p=1$ the claim is trivial. If $p \\neq 1$, then the claim is equivalent to the existence of an integer $m$ such that $$ \\frac{m-p \\alpha}{p-1} $$ is never an integer. Assume the contrary. That would mean that both $$ \\frac{m-p \\alpha}{p-1} \\quad \\text { and } \\quad \\frac{(m+1)-p \\alpha}{p-1} $$ are integers, and so is their difference. The latter is equal to $$ \\frac{1}{p-1} $$ Since we assumed $p \\notin S, 1 \/(p-1)$ is never an integer. This is a contradiction. Define $g:[0,1) \\rightarrow \\mathbb{Z}$ by $g(\\alpha)=m$ for any integer $m$ that satisfies the conclusion of Lemma 1. Note that $f(z) \\neq p z$ if and and only if $$ g(\\{z\\})+\\lfloor z\\rfloor \\neq p(\\{z\\}+\\lfloor z\\rfloor) $$ The latter is guaranteed by the construction of the function $g$. We conclude that $p \\notin Z$ as desired. This shows that $Z \\subset S$.","tier":0} +{"problem_type":"Algebra","problem_label":"A7","problem":"For a positive integer $n$ we denote by $s(n)$ the sum of the digits of $n$. Let $P(x)=$ $x^{n}+a_{n-1} x^{n-1}+\\cdots+a_{1} x+a_{0}$ be a polynomial, where $n \\geqslant 2$ and $a_{i}$ is a positive integer for all $0 \\leqslant i \\leqslant n-1$. Could it be the case that, for all positive integers $k, s(k)$ and $s(P(k))$ have the same parity? (Belarus) Answer: No. For any such polynomial there exists a positive integer $k$ such that $s(k)$ and $s(P(k))$ have different parities.","solution":"With the notation above, we begin by choosing a positive integer $t$ such that $$ 10^{t}>\\max \\left\\{\\frac{100^{n-1} a_{n-1}}{\\left(10^{\\frac{1}{n-1}}-9^{\\frac{1}{n-1}}\\right)^{n-1}}, \\frac{a_{n-1}}{9} 10^{n-1}, \\frac{a_{n-1}}{9}\\left(10 a_{n-1}\\right)^{n-1}, \\ldots, \\frac{a_{n-1}}{9}\\left(10 a_{0}\\right)^{n-1}\\right\\} $$ As a direct consequence of $10^{t}$ being bigger than the first quantity listed in the above set, we get that the interval $$ I=\\left[\\left(\\frac{9}{a_{n-1}} 10^{t}\\right)^{\\frac{1}{n-1}},\\left(\\frac{1}{a_{n-1}} 10^{t+1}\\right)^{\\frac{1}{n-1}}\\right) $$ contains at least 100 consecutive positive integers. Let $X$ be a positive integer in $I$ such that $X$ is congruent to $1 \\bmod 100$. Since $X \\in I$ we have $$ 9 \\cdot 10^{t} \\leqslant a_{n-1} X^{n-1}<10^{t+1} $$ thus the first digit (from the left) of $a_{n-1} X^{n-1}$ must be 9 . Next, we observe that $a_{n-1}\\left(10 a_{i}\\right)^{n-1}<9 \\cdot 10^{t} \\leqslant a_{n-1} X^{n-1}$, thus $10 a_{i}10^{\\alpha i} a_{n-1} X^{n-1}>$ $10^{\\alpha i} a_{i} X^{i}$, the terms $a_{i} 10^{\\alpha i} X^{i}$ do not interact when added; in particular, there is no carryover caused by addition. Thus we have $s\\left(P\\left(10^{\\alpha} X\\right)\\right)=s\\left(X^{n}\\right)+s\\left(a_{n-1} X^{n-1}\\right)+\\cdots+s\\left(a_{0}\\right)$. We now look at $P\\left(10^{\\alpha-1} X\\right)=10^{(\\alpha-1) n} X^{n}+a_{n-1} 10^{(\\alpha-1)(n-1)} X^{n-1}+\\cdots+a_{0}$. Firstly, if $i10^{(\\alpha-1) i} a_{n-1} X^{n-1} \\geqslant 10^{(\\alpha-1) i+1} a_{i} X^{i}$, thus all terms $10^{(\\alpha-1) i} a_{i} X^{i}$ for $0 \\leqslant i \\leqslant n-1$ come in 'blocks', exactly as in the previous case. Finally, $10^{(\\alpha-1) n+1}>10^{(\\alpha-1)(n-1)} a_{n-1} X^{n-1} \\geqslant 10^{(\\alpha-1) n}$, thus $10^{(\\alpha-1)(n-1)} a_{n-1} X^{n-1}$ has exactly $(\\alpha-1) n+1$ digits, and its first digit is 9 , as established above. On the other hand, $10^{(\\alpha-1) n} X^{n}$ has exactly $(\\alpha-1) n$ zeros, followed by $01($ as $X$ is $1 \\bmod 100)$. Therefore, when we add the terms, the 9 and 1 turn into 0 , the 0 turns into 1 , and nothing else is affected. Putting everything together, we obtain $$ s\\left(P\\left(10^{\\alpha-1} X\\right)\\right)=s\\left(X^{n}\\right)+s\\left(a_{n-1} X^{n-1}\\right)+\\cdots+s\\left(a_{0}\\right)-9=s\\left(P\\left(10^{\\alpha} X\\right)\\right)-9 $$ thus $s\\left(P\\left(10^{\\alpha} X\\right)\\right)$ and $s\\left(P\\left(10^{\\alpha-1} X\\right)\\right)$ have different parities, as claimed.","tier":0} +{"problem_type":"Algebra","problem_label":"A8","problem":"For a positive integer $n$, an $n$-sequence is a sequence $\\left(a_{0}, \\ldots, a_{n}\\right)$ of non-negative integers satisfying the following condition: if $i$ and $j$ are non-negative integers with $i+j \\leqslant n$, then $a_{i}+a_{j} \\leqslant n$ and $a_{a_{i}+a_{j}}=a_{i+j}$. Let $f(n)$ be the number of $n$-sequences. Prove that there exist positive real numbers $c_{1}, c_{2}$ and $\\lambda$ such that $$ c_{1} \\lambda^{n}k$ for some $i$, and small if no such $i$ exists. For now we will assume that $\\left(a_{i}\\right)$ is not the identity sequence (in other words, that $a_{i} \\neq i$ for some $i$ ). Lemma 1. If $a_{r}=a_{s}$ and $r, sr$, and let $d$ be the minimum positive integer such that $a_{r+d}=a_{r}$. Then 1. The subsequence $\\left(a_{r}, a_{r+1}, \\ldots, a_{n}\\right)$ is periodic with minimal period $d$. That is, for $uv_{0}$. Thus $u_{0}=v_{0}$, so $d \\mid u-v$. 2. If $r=0$ there is nothing to prove. Otherwise $a_{0}=a_{2 a_{0}}$ so $2 a_{0}=0$. Then we have $a_{a_{i}}=a_{i}$ for all $i$, so $a_{i}=i$ for $ik+1$. We show that $a_{i} \\leqslant k$ for all $i$ by induction. Note that Lemma 2 already establishes this for $i \\leqslant k$. We must have $d \\mid a_{d \/ 2}$ and $a_{d \/ 2} \\leqslant kk$, if $a_{j} \\leqslant k$ for $jk+1$ and $a_{i}=i$ for all $0 \\leqslant ik$. We already have $a_{i}=i$ for $id$, this means that $a_{i}=i$ for $i \\leqslant k$. Finally, one can show inductively that $a_{i}=i$ for $kc_{1} \\lambda^{n}$ for some $c_{1}$, we note that $$ f(n)>g\\left(k+1,\\left\\lfloor\\frac{k+1}{3}\\right\\rfloor\\right) \\geqslant 3^{\\lfloor(k+1) \/ 3\\rfloor}>3^{n \/ 6}-1 . $$ To show that $f(n)2 \\cdot 3^{-1 \/ 6} \\approx 1.66537$. With a careful analysis one can show that the best possible value of $c_{2}$ is $\\frac{236567}{4930} 3^{1 \/ 3} \\approx$ 69.20662 .","tier":0} +{"problem_type":"Combinatorics","problem_label":"C1","problem":"A $\\pm 1$-sequence is a sequence of 2022 numbers $a_{1}, \\ldots, a_{2022}$, each equal to either +1 or -1 . Determine the largest $C$ so that, for any $\\pm 1$-sequence, there exists an integer $k$ and indices $1 \\leqslant t_{1}<\\ldots\\frac{3 n+1}{2}$, let $a=k-n-1, b=2 n-k+1$. Then $k>2 a+b, k>2 b+a$, so the configuration $A^{a} C^{b} A^{b} C^{a}$ will always have four blocks: $$ A^{a} C^{b} A^{b} C^{a} \\rightarrow C^{a} A^{a} C^{b} A^{b} \\rightarrow A^{b} C^{a} A^{a} C^{b} \\rightarrow C^{b} A^{b} C^{a} A^{a} \\rightarrow A^{a} C^{b} A^{b} C^{a} \\rightarrow \\ldots $$ Therefore a pair $(n, k)$ can have the desired property only if $n \\leqslant k \\leqslant \\frac{3 n+1}{2}$. We claim that all such pairs in fact do have the desired property. Clearly, the number of blocks in a configuration cannot increase, so whenever the operation is applied, it either decreases or remains constant. We show that unless there are only two blocks, after a finite amount of steps the number of blocks will decrease. Consider an arbitrary configuration with $c \\geqslant 3$ blocks. We note that as $k \\geqslant n$, the leftmost block cannot be moved, because in this case all $n$ coins of one type are in the leftmost block, meaning there are only two blocks. If a block which is not the leftmost or rightmost block is moved, its neighbor blocks will be merged, causing the number of blocks to decrease. Hence the only case in which the number of blocks does not decrease in the next step is if the rightmost block is moved. If $c$ is odd, the leftmost and the rightmost blocks are made of the same metal, so this would merge two blocks. Hence $c \\geqslant 4$ must be even. Suppose there is a configuration of $c$ blocks with the $i$-th block having size $a_{i}$ so that the operation always moves the rightmost block: $$ A^{a_{1}} \\ldots A^{a_{c-1}} C^{a_{c}} \\rightarrow C^{a_{c}} A^{a_{1}} \\ldots A^{a_{c-1}} \\rightarrow A^{a_{c-1}} C^{a_{c}} A^{a_{1}} \\ldots C^{a_{c-2}} \\rightarrow \\ldots $$ Because the rightmost block is always moved, $k \\geqslant 2 n+1-a_{i}$ for all $i$. Because $\\sum a_{i}=2 n$, summing this over all $i$ we get $c k \\geqslant 2 c n+c-\\sum a_{i}=2 c n+c-2 n$, so $k \\geqslant 2 n+1-\\frac{2 n}{c} \\geqslant \\frac{3 n}{2}+1$. But this contradicts $k \\leqslant \\frac{3 n+1}{2}$. Hence at some point the operation will not move the rightmost block, meaning that the number of blocks will decrease, as desired.","tier":0} +{"problem_type":"Combinatorics","problem_label":"C3","problem":"In each square of a garden shaped like a $2022 \\times 2022$ board, there is initially a tree of height 0 . A gardener and a lumberjack alternate turns playing the following game, with the gardener taking the first turn: - The gardener chooses a square in the garden. Each tree on that square and all the surrounding squares (of which there are at most eight) then becomes one unit taller. - The lumberjack then chooses four different squares on the board. Each tree of positive height on those squares then becomes one unit shorter. We say that a tree is majestic if its height is at least $10^{6}$. Determine the largest number $K$ such that the gardener can ensure there are eventually $K$ majestic trees on the board, no matter how the lumberjack plays. (Colombia) Answer: $K=5 \\cdot \\frac{2022^{2}}{9}=2271380$. In general, for a $3 N \\times 3 N$ board, $K=5 N^{2}$.","solution":"We solve the problem for a general $3 N \\times 3 N$ board. First, we prove that the lumberjack has a strategy to ensure there are never more than $5 N^{2}$ majestic trees. Giving the squares of the board coordinates in the natural manner, colour each square where at least one of its coordinates are divisible by 3 , shown below for a $9 \\times 9$ board: ![](https:\/\/cdn.mathpix.com\/cropped\/2024_11_18_86efb4389f14b52b08e1g-28.jpg?height=900&width=914&top_left_y=1343&top_left_x=571) Then, as each $3 \\times 3$ square on the board contains exactly 5 coloured squares, each move of the gardener will cause at most 4 trees on non-coloured squares to grow. The lumberjack may therefore cut those trees, ensuring no tree on a non-coloured square has positive height after his turn. Hence there cannot ever be more majestic trees than coloured squares, which is $5 N^{2}$. Next, we prove the gardener may ensure there are $5 N^{2}$ majestic trees. In fact, we prove this statement in a modified game which is more difficult for the gardener: on the lumberjack's turn in the modified game, he may decrement the height of all trees on the board except those the gardener did not just grow, in addition to four of the trees the gardener just grew. Clearly, a sequence of moves for the gardener which ensures that there are $K$ majestic trees in the modified game also ensures this in the original game. Let $M=\\binom{9}{5}$; we say that a map is one of the $M$ possible ways to mark 5 squares on a $3 \\times 3$ board. In the modified game, after the gardener chooses a $3 \\times 3$ subboard on the board, the lumberjack chooses a map in this subboard, and the total result of the two moves is that each tree marked on the map increases its height by 1 , each tree in the subboard which is not in the map remains unchanged, and each tree outside the subboard decreases its height by 1. Also note that if the gardener chooses a $3 \\times 3$ subboard $M l$ times, the lumberjack will have to choose some map at least $l$ times, so there will be at least 5 trees which each have height $\\geqslant l$. The strategy for the gardener will be to divide the board into $N^{2}$ disjoint $3 \\times 3$ subboards, number them $0, \\ldots, N^{2}-1$ in some order. Then, for $b=N^{2}-1, \\ldots, 0$ in order, he plays $10^{6} M(M+1)^{b}$ times on subboard number $b$. Hence, on subboard number $b$, the moves on that subboard will first ensure 5 of its trees grows by at least $10^{6}(M+1)^{b}$, and then each move after that will decrease their heights by 1 . (As the trees on subboard $b$ had height 0 before the gardener started playing there, no move made on subboards $\\geqslant b$ decreased their heights.) As the gardener makes $10^{6} M(M+1)^{b-1}+\\ldots=10^{6}\\left((M+1)^{b}-1\\right)$ moves after he finishes playing on subboard $b$, this means that on subboard $b$, there will be 5 trees of height at least $10^{6}(M+1)^{b}-10^{6}\\left((M+1)^{b}-1\\right)=10^{6}$, hence each of the subboard has 5 majestic trees, which was what we wanted.","tier":0} +{"problem_type":"Combinatorics","problem_label":"C4","problem":"Let $n>3$ be a positive integer. Suppose that $n$ children are arranged in a circle, and $n$ coins are distributed between them (some children may have no coins). At every step, a child with at least 2 coins may give 1 coin to each of their immediate neighbours on the right and left. Determine all initial distributions of coins from which it is possible that, after a finite number of steps, each child has exactly one coin. (Israel) Answer: All distributions where $\\sum_{i=1}^{n} i c_{i}=\\frac{n(n+1)}{2}(\\bmod n)$, where $c_{i}$ denotes the number of coins the $i$-th child starts with.","solution":"Number the children $1, \\ldots, n$, and denote the number of coins the $i$-th child has by $c_{i}$. A step of this process consists of reducing some $c_{i}$ by 2 , and increasing $c_{i-1}, c_{i+1}$ by 1 . (Indices are considered $(\\bmod n)$.) Because $(i-1)-2 i+(i+1)=0$, the quantity $\\sum_{i=1}^{n} i c_{i}(\\bmod n)$ will be invariant under this process. Hence a necessary condition for the children to end up with an uniform distribution of coins is that $$ \\sum_{i=1}^{n} i c_{i}=\\frac{n(n+1)}{2} \\quad(\\bmod n) $$ We will show that this condition is also sufficient. Consider an arbitrary initial distribution of coins. First, whenever child $i$ has more than one coin and $i \\neq n$, have child $i$ pass coins to its neighbors. (Child $i$ does nothing.) Then, after some amount of such steps, it must eventually become impossible to do any more steps because no child except perhaps child $i$ has more than 1 coin. (To see this, consider e.g. the quantity $\\sum_{i=1}^{n-1} i^{2} c_{i}$, which (as $(i-1)^{2}+(i+1)^{2}>2 i^{2}$ ) increases at each step.) Hence we can reach a state of the form $\\left(z_{1}, \\ldots, z_{n-1}, M\\right)$, where $z_{i}=0$ or 1 . Call such states semi-uniform states of irregularity $M$. Lemma. If there is a string of children having coins $a, \\overbrace{1, \\ldots, 1}^{k \\text { ones }}, b, \\overbrace{1, \\ldots, 1}^{k \\text { ones }}, c$, with $b \\geqslant 2$, after some sequence of steps we may reach the state $a+1, \\overbrace{1, \\ldots, 1}^{k \\text { ones }}, b-2, \\overbrace{1, \\ldots, 1}^{k \\text { ones }}, c+1$. We call performing this sequence of steps long-passing coins. Proof. This is simply repeated application of the operation. We prove the lemma by induction on $k$. For $k=0$, this is just the operation of the problem. If $k=1$, have the child with $b$ coins pass coins, then both of their neighbors pass coins, then the child with $b$ coins pass coins again. For $k \\geqslant 2$, first, have the child with $b$ coins pass coins, then have both their neigbors send coins, giving the state $$ a, \\overbrace{1, \\ldots, 1}^{k-2 \\text { ones }}, 2,0, b, 0,2, \\overbrace{1, \\ldots, 1}^{k-2 \\text { ones }}, c . $$ Now set aside the children with $a, b$ and $c$ coins, and have each child with 2 coins give them to their neighbors until there are no such children remaining. This results in the state $$ a+1,0, \\overbrace{1, \\ldots, 1}^{k-2}, b, \\overbrace{1, \\ldots, 1}^{k-2 \\text { ones }}, 0, c+1 $$ By the induction hypothesis, we can have the child with $b$ coins may pass a coin to each of the children with 0 coins, proving the lemma. Claim. We can reach a semi-uniform state of irregularity $M \\leqslant 2$. Proof. If $M>3$, because there are only $n$ coins in total, there must be at least two children with 0 coins. Consider the arc of the circle spanned by the two such children closest to the child with $M$ coins. It has the form $$ 0, \\overbrace{1, \\ldots, 1}^{a \\text { ones }}, M, \\overbrace{1, \\ldots, 1}^{b \\text { ones }}, 0 $$ If $a=b$, applying the previous lemma we can have the child with $M$ coins long-pass a coin to each of the children with 0 coins, which yields a semi-uniform state with lower $M$. Otherwise, WLOG $a>b$, so we can have the child with $M$ coins long-pass a coin to each of the children at distance $b$ from it, reaching a state of the form $(\\alpha:=a-b-1, \\beta:=b)$ $$ 0, \\overbrace{1, \\ldots, 1}^{\\alpha \\text { ones }}, 2, \\overbrace{1, \\ldots, 1}^{\\beta \\text { ones }}, M-2, \\overbrace{1, \\ldots, 1}^{c \\text { ones }} $$ The children in the rightmost string of ones need make no further moves, so consider only the leftmost string. If $\\alpha<\\beta$, have the child with 2 coins long-pass coins to the child with 0 coins to its left and some child with 1 coin to its right, reaching a new state of the form $0, \\overbrace{1, \\ldots, 1}^{\\alpha \\text { ones }}, 2, \\overbrace{1, \\ldots, 1}^{\\beta \\text { ones }}, M-2$ with a smaller $\\beta$. As $\\beta$ cannot decrease indefinitely, eventually $\\alpha \\geqslant \\beta$. If $\\alpha=\\beta$, have the child with 2 coins long-pass to the child with $M$ coins and the child with 0 coins, reaching a semi-uniform state of irregularity $M-1$ as desired. Otherwise, $\\alpha<\\beta$, so have the child with 2 coins long-pass to the child with $M$ coins and a child with 1 coin, reaching a state of the form $$ 0, \\overbrace{1, \\ldots, 1}^{x \\text { ones }}, 2, \\overbrace{1, \\ldots, 1}^{y \\text { ones }}, 0, \\overbrace{1, \\ldots, 1}^{z \\text { ones }}, M-1 $$ Now, consider only the substring between the two children with 0 coins, which has the form $0, \\overbrace{1, \\ldots, 1}^{x \\text { ones }}, 2, \\overbrace{1, \\ldots, 1}^{y \\text { ones }}, 0$. Repeatedly have the child in this substring with 2 coins long-pass to the closest child with 0 coins and some other child. If the other child has 1 coin, we have a new strictly shorter substring of the form $0, \\overbrace{1, \\ldots, 1}^{x \\text { ones }}, 2, \\overbrace{1, \\ldots, 1}^{y \\text { ones }}, 0$. Hence eventually it must happen that the other child also has 0 coins, at which point we reach a semi-uniform state of irregularity $M-1$, proving the claim. We have now shown that we can reach a semi-regular state of irregularity $M \\leqslant 2$, If $M=1$, each child must have one coin, as desired. Otherwise, we must have $M=2$, so there is one child with 0 coins, one child with 2 coins, and the remaining children all have 1 coin. Recall that the state we started with satisfied the invariant $$ \\sum_{i=1}^{n} i c_{i}=\\frac{n(n+1)}{2} \\quad(\\bmod n) $$ Because each step preserves this invariant, this must also be true of the current state. Number the children so that the child with $M$ coins is child number $n$, and suppose the child with 0 coins is child $k$. Then $\\frac{n(n+1)}{2}=\\sum_{i=1}^{n} i c_{i}=\\left(\\sum_{i=1}^{n} 1 \\cdot c_{i}\\right)-k=\\frac{n(n+1)}{2}-k(\\bmod n)$, so $k=0$ $(\\bmod n)$. But this is impossible, as no child except the child with $M$ coins has an index divisible by $n$. Hence we cannot end up in a semi-regular state of irregularity 2 , so we are done.","tier":0} +{"problem_type":"Combinatorics","problem_label":"C4","problem":"Let $n>3$ be a positive integer. Suppose that $n$ children are arranged in a circle, and $n$ coins are distributed between them (some children may have no coins). At every step, a child with at least 2 coins may give 1 coin to each of their immediate neighbours on the right and left. Determine all initial distributions of coins from which it is possible that, after a finite number of steps, each child has exactly one coin. (Israel) Answer: All distributions where $\\sum_{i=1}^{n} i c_{i}=\\frac{n(n+1)}{2}(\\bmod n)$, where $c_{i}$ denotes the number of coins the $i$-th child starts with.","solution":"Encode the sequence $c_{i}$ as a polynomial $p(x)=\\sum_{i} a_{i} x_{i}$. The cyclic nature of the problem makes it natural to work modulo $x^{n}-1$. Child $i$ performing a step is equivalent to adding $x^{i}(x-1)^{2}$ to the polynomial, and we want to reach the polynomial $q(x)=1+x+\\ldots+$ $x^{n-1}$. Since we only add multiples of $(x-1)^{2}$, this is only possible if $p(x)=q(x)$ modulo the ideal generated by $x^{n}-1$ and $(x-1)^{2}$, i.e. $$ \\left(x^{n}-1,(x-1)^{2}\\right)=(x-1)\\left(\\frac{x^{n}-1}{x-1}, x-1\\right)=(x-1) \\cdot(n,(x-1)) $$ This is equivalent to $p(1)=q(1)$ (which simply translates to the condition that there are $n$ coins) and $p^{\\prime}(1)=q^{\\prime}(1)(\\bmod n)$, which translates to the invariant described in solution 1.","tier":0} +{"problem_type":"Combinatorics","problem_label":"C6","problem":"Let $n$ be a positive integer. We start with $n$ piles of pebbles, each initially containing a single pebble. One can perform moves of the following form: choose two piles, take an equal number of pebbles from each pile and form a new pile out of these pebbles. For each positive integer $n$, find the smallest number of non-empty piles that one can obtain by performing a finite sequence of moves of this form. (Croatia) Answer: 1 if $n$ is a power of two, and 2 otherwise.","solution":"The solution we describe is simple, but not the most effective one. We can combine two piles of $2^{k-1}$ pebbles to make one pile of $2^{k}$ pebbles. In particular, given $2^{k}$ piles of one pebble, we may combine them as follows: $$ \\begin{array}{lcc} 2^{k} \\text { piles of } 1 \\text { pebble } & \\rightarrow & 2^{k-1} \\text { piles of } 2 \\text { pebbles } \\\\ 2^{k-1} \\text { piles of } 2 \\text { pebbles } & \\rightarrow & 2^{k-2} \\text { piles of } 4 \\text { pebbles } \\\\ 2^{k-2} \\text { piles of } 4 \\text { pebbles } & \\rightarrow & 2^{k-3} \\text { piles of } 8 \\text { pebbles } \\\\ & \\vdots \\\\ 2 \\text { piles of } 2^{k-1} \\text { pebbles } & \\rightarrow 1 \\text { pile of } 2^{k} \\text { pebbles } \\end{array} $$ This proves the desired result in the case when $n$ is a power of 2 . If $n$ is not a power of 2 , choose $N$ such that $2^{N}1$. In order to make a single pile of $n$ pebbles, we would have to start with a distribution in which the number of pebbles in each pile is divisible by the integer $m$. This is impossible when we start with all piles containing a single pebble. Remarks on starting configurations From any starting configuration that is not a single pile, if there are at least two piles with at least two pebbles, we can remove one pebble from two such piles, and form a new pile with 2 pebbles. We can repeat this until we have one pile of 2 pebbles and the rest are single pebble piles, and then proceed as in the solution. Hence, if $n$ is a power of two, we can make a single pile from any starting configuration. If $n$ is of the form $n=2^{k} m$ where $m>1$ is odd, then we can make a single pile from any starting configuration in which the number of pebbles in each pile is divisible by the integer $m$, otherwise two piles is the best we can do. Half of this is proven already. For the other half, assume we start with a configuration in which the number of pebbles in each pile is divisible by the integer $m$. Replace each pile of $t m$ pebbles with a pile of $t$ boulders. We now have a total of $2^{k}$ boulders, hence we can make them into one pile of $2^{k}$ boulders. Replacing the boulders with pebbles again, we are done.","tier":0} +{"problem_type":"Combinatorics","problem_label":"C6","problem":"Let $n$ be a positive integer. We start with $n$ piles of pebbles, each initially containing a single pebble. One can perform moves of the following form: choose two piles, take an equal number of pebbles from each pile and form a new pile out of these pebbles. For each positive integer $n$, find the smallest number of non-empty piles that one can obtain by performing a finite sequence of moves of this form. (Croatia) Answer: 1 if $n$ is a power of two, and 2 otherwise.","solution":"We show an alternative strategy if $n$ is not a power of 2 . Write $n$ in binary form: $n=2^{i_{1}}+2^{i_{2}}+\\cdots+2^{i_{k}}$, where $i_{1}>i_{2}>\\cdots>i_{k}$. Now we make piles of sizes $2^{i_{1}}, 2^{i_{2}}, \\ldots, 2^{i_{k}}$. We call the pile with $2^{i_{1}}$ the large pile, all the others are the small piles. The strategy is the following: take the two smallest small pile. If they have the same size of $2^{a}$, we make a pile of size $2^{a+1}$. If they have different sizes, we double the smaller pile using the large pile (we allow the large pile to have a negative number of pebbles: later we prove that it is not possible). We call all the new piles small. When we have only one small pile, we terminate the process: we have at most 2 piles. After each move we have one less number of piles, and all the piles have cardinality power of 2. The number of pebbles is decreasing, and at the end of the process, it has a size of $n-2^{i_{2}+1} \\geqslant n-2^{i_{1}}>0$, thus we can manage to have two piles.","tier":0} +{"problem_type":"Combinatorics","problem_label":"C6","problem":"Let $n$ be a positive integer. We start with $n$ piles of pebbles, each initially containing a single pebble. One can perform moves of the following form: choose two piles, take an equal number of pebbles from each pile and form a new pile out of these pebbles. For each positive integer $n$, find the smallest number of non-empty piles that one can obtain by performing a finite sequence of moves of this form. (Croatia) Answer: 1 if $n$ is a power of two, and 2 otherwise.","solution":"Throughout the solution, we will consider the moves in reverse order. Namely, imagine we have some piles of pebbles, and we are allowed to perform moves as follows: take a pile with an even number of pebbles, split it into two equal halves and add the pebbles from each half to a different pile, possibly forming new piles (we may assume for convenience that there are infinitely many empty piles at any given moment). Given a configuration of piles $\\mathcal{C}$, we will use $|\\mathcal{C}|$ to denote the number of non-empty piles in $\\mathcal{C}$. Given two configurations $\\mathcal{C}_{1}, \\mathcal{C}_{2}$, we will say that $\\mathcal{C}_{2}$ is reachable from $\\mathcal{C}_{1}$ if $\\mathcal{C}_{2}$ can be obtained by performing a finite sequence of moves starting from $\\mathcal{C}_{1}$. Call a configuration of piles $\\mathcal{C}$ - simple if each (non-empty) pile in $\\mathcal{C}$ consists of a single pebble; - good if at least one (non-empty) pile in $\\mathcal{C}$ has an even number of pebbles and the numbers of pebbles on the piles in $\\mathcal{C}$ have no non-trivial odd common divisor ( $\\mathcal{C}$ has the odd divisor property); - solvable if there exists a simple configuration which is reachable from $\\mathcal{C}$. The problem asks to find the smallest number of non-empty piles in a solvable configuration consisting of $n$ pebbles. We begin the process of answering this question by making the following observation: Lemma 1. Let $\\mathcal{C}$ be a configuration of piles. Let $\\mathcal{C}^{\\prime}$ be a configuration obtained by applying a single move to $\\mathcal{C}$. Then (i) if $\\mathcal{C}^{\\prime}$ has the odd divisor property, then so does $\\mathcal{C}$; (ii) the converse to (i) holds if $\\left|\\mathcal{C}^{\\prime}\\right| \\geqslant|\\mathcal{C}|$. Proof. Suppose that the move consists of splitting a pile of size $2 a$ and adding $a$ pebbles to each of two piles of sizes $b$ and $c$. Here, $a$ is a positive integer and $b, c$ are non-negative integers. Thus, $\\mathcal{C}^{\\prime}$ can be obtained from $\\mathcal{C}$ by replacing the piles of sizes $2 a, b, c$ by two piles of sizes $a+b$ and $a+c$. Note that the extra assumption $\\left|\\mathcal{C}^{\\prime}\\right| \\geqslant|\\mathcal{C}|$ of part (ii) holds if and only if at least one of $b, c$ is zero. (i) Suppose $\\mathcal{C}$ doesn't have the odd divisor property, i.e. there exists an odd integer $d>1$ such that the size of each pile in $\\mathcal{C}$ is divisible by $d$. In particular, $2 a, b, c$ are multiples of $d$, so since $d$ is odd, it follows that $a, b, c$ are all divisible by $d$. Thus, $a+b$ and $a+c$ are also divisible by $d$, so $d$ divides the size of each pile in $\\mathcal{C}^{\\prime}$. In conclusion, $\\mathcal{C}^{\\prime}$ doesn't have the odd divisor property. (ii) If $\\mathcal{C}^{\\prime}$ doesn't have the odd divisor property and at least one of $b, c$ is zero, then there exists an odd integer $d>1$ such that the size of each pile in $\\mathcal{C}^{\\prime}$ is divisible by $d$. In particular, $d$ divides $a+b$ and $a+c$, so since at least one of these numbers is equal to $a$, it follows that $d$ divides $a$. But then $d$ must divide all three of $a, b$ and $c$, and hence it certainly divides $2 a, b$ and $c$. Thus, $\\mathcal{C}$ doesn't have the odd divisor property, as desired. Lemma 2. If $\\mathcal{C}_{2}$ is reachable from $\\mathcal{C}_{1}$ and $\\mathcal{C}_{2}$ has the odd divisor property, then so does $\\mathcal{C}_{1}$. In particular, any solvable configuration has the odd divisor property. Proof. The first statement follows by inductively applying part (i) of Lemma 1. The second statement follows from the first because every simple configuration has the odd divisor property. The main claim is the following: Lemma 3. Let $\\mathcal{C}$ be a good configuration. Then there exists a configuration $\\mathcal{C}^{\\prime}$ with the following properties: - $\\mathcal{C}^{\\prime}$ is reachable from $\\mathcal{C}$ and $\\left|\\mathcal{C}^{\\prime}\\right|>|\\mathcal{C}|$; - $\\mathcal{C}^{\\prime}$ is either simple or good. Proof. Call a configuration terminal if it is a counterexample to the claim. The following claim is at the heart of the proof: Claim. Let $a_{1}, \\ldots, a_{k}$ be the numbers of pebbles on the non-empty piles of a terminal configuration $\\mathcal{C}$. Then there exists a unique $i \\in[k]$ such that $a_{i}$ is even. Moreover, for all $t \\geqslant 1$ we have $a_{j} \\equiv \\frac{a_{i}}{2}\\left(\\bmod 2^{t}\\right)$ for all $j \\neq i$. Proof of Claim. Since the configuration is good, there must exist $i \\in[k]$ such that $a_{i}$ is even. Moreover, by assumption, if we split the pile with $a_{i}$ pebbles into two equal halves, the resulting configuration will not be good. By part (ii) of Lemma 2,the only way this can happen is that $\\frac{a_{i}}{2}$ and $a_{j}$ for all $j \\neq i$ are odd. To prove the second assertion, we proceed by induction on $t$, with the case $t=1$ already being established. If $t \\geqslant 2$, then split the pile with $a_{i}$ pebbles into two equal halves and move one half to the pile with $a_{j}$ pebbles. Since $\\frac{a_{i}}{2}$ and $a_{j}$ are both odd, $a_{j}+\\frac{a_{i}}{2}$ is even, so by part (ii) of Lemma 2 , the resulting configuration $\\mathcal{C}^{\\prime}$ is good. Thus, $\\mathcal{C}^{\\prime}$ is terminal, so by the induction hypothesis, we have $\\frac{a_{i}}{2} \\equiv \\frac{1}{2}\\left(a_{j}+\\frac{a_{i}}{2}\\right)\\left(\\bmod 2^{t-1}\\right)$, whence $a_{j} \\equiv \\frac{a_{i}}{2}($ $\\bmod 2^{t}$ ), as desired. Suppose for contradiction that there exists a configuration as in the Claim. It follows that there exists $i \\in[k]$ and an odd integer $x$ such that $a_{i}=2 x$ and $a_{j}=x$ for all $j \\neq i$. Thus, $x$ is an odd common divisor of $a_{1}, \\ldots, a_{k}$, so by the odd divisor property, we must have $x=1$. But then we can obtain a simple configuration by splitting the only pile with two pebbles into two piles each consisting of a single pebble, which is a contradiction. With the aid of Lemmas 2 and 3, it is not hard to characterise all solvable configurations: Lemma 4. A configuration of piles $\\mathcal{C}$ is solvable if and only if it is simple or good. Proof. $(\\Longrightarrow)$ Suppose $\\mathcal{C}$ is not simple. Then since we have to be able to perform at least one move, there must be at least one non-empty pile in $\\mathcal{C}$ with an even number of pebbles. Moreover, by Lemma $2, \\mathcal{C}$ has the odd divisor property, so it must be good. $(\\Longleftarrow)$ This follows by repeatedly applying Lemma 3 until we reach a simple configuration. Note that the process must stop eventually since the number of non-empty piles is bounded from above. Finally, the answer to the problem is implied by the following corollary of Lemma 4: Lemma 5. Let $n$ be a positive integer. Then (i) a configuration consisting of a single pile of $n$ pebbles is solvable if and only if $n$ is a power of two; (ii) if $n \\geqslant 2$, then the configuration consisting of piles of sizes 2 and $n-2$ is good. Proof. (i) By Lemma 4, this configuration is solvable if and only if either $n=1$ or $n$ is even and has no non-trivial odd divisor, so the conclusion follows. (ii) Since 2 is even and has no non-trivial odd divisor, this configuration is certainly good, so the conclusion follows by Lemma 4. Common remarks. Instead of choosing pebbles from two piles, one could allow choosing an equal number of pebbles from each of $k$ piles, where $k \\geqslant 2$ is a fixed (prime) integer. However, this seems to yield a much harder problem - if $k=3$, numerical evidence suggests the same answer as in the case $k=2$ (with powers of two replaced by powers of three), but the case $k=5$ is already unclear.","tier":0} +{"problem_type":"Combinatorics","problem_label":"C7","problem":"Lucy starts by writing $s$ integer-valued 2022-tuples on a blackboard. After doing that, she can take any two (not necessarily distinct) tuples $\\mathbf{v}=\\left(v_{1}, \\ldots, v_{2022}\\right)$ and $\\mathbf{w}=\\left(w_{1}, \\ldots, w_{2022}\\right)$ that she has already written, and apply one of the following operations to obtain a new tuple: $$ \\begin{aligned} & \\mathbf{v}+\\mathbf{w}=\\left(v_{1}+w_{1}, \\ldots, v_{2022}+w_{2022}\\right) \\\\ & \\mathbf{v} \\vee \\mathbf{w}=\\left(\\max \\left(v_{1}, w_{1}\\right), \\ldots, \\max \\left(v_{2022}, w_{2022}\\right)\\right) \\end{aligned} $$ and then write this tuple on the blackboard. It turns out that, in this way, Lucy can write any integer-valued 2022-tuple on the blackboard after finitely many steps. What is the smallest possible number $s$ of tuples that she initially wrote? (Czech Republic) Answer: The smallest possible number is $s=3$.","solution":"We solve the problem for $n$-tuples for any $n \\geqslant 3$ : we will show that the answer is $s=3$, regardless of the value of $n$. First, let us briefly introduce some notation. For an $n$-tuple $\\mathbf{v}$, we will write $\\mathbf{v}_{i}$ for its $i$-th coordinate (where $1 \\leqslant i \\leqslant n$ ). For a positive integer $n$ and a tuple $\\mathbf{v}$ we will denote by $n \\cdot \\mathbf{v}$ the tuple obtained by applying addition on $\\mathbf{v}$ with itself $n$ times. Furthermore, we denote by $\\mathbf{e}(i)$ the tuple which has $i$-th coordinate equal to one and all the other coordinates equal to zero. We say that a tuple is positive if all its coordinates are positive, and negative if all its coordinates are negative. We will show that three tuples suffice, and then that two tuples do not suffice. Three tuples suffice. Write $\\mathbf{c}$ for the constant-valued tuple $\\mathbf{c}=(-1, \\ldots,-1)$. We note that it is enough for Lucy to be able to make the tuples $\\mathbf{e}(1), \\ldots, \\mathbf{e}(n), \\mathbf{c}$; from those any other tuple $\\mathbf{v}$ can be made as follows. First we choose some positive integer $k$ such that $k+\\mathbf{v}_{i}>0$ for all $i$. Then, by adding a positive number of copies of $\\mathbf{c}, \\mathbf{e}(1), \\ldots, \\mathbf{e}(n)$, she can make $$ k \\mathbf{c}+\\left(k+\\mathbf{v}_{1}\\right) \\cdot \\mathbf{e}(1)+\\cdots+\\left(k+\\mathbf{v}_{n}\\right) \\cdot \\mathbf{e}(n) $$ which we claim is equal to $\\mathbf{v}$. Indeed, this can be checked by comparing coordinates: the $i$-th coordinate of the right-hand side is $-k+\\left(k+\\mathbf{v}_{i}\\right)=\\mathbf{v}_{i}$ as needed. Lucy can take her three starting tuples to be $\\mathbf{a}, \\mathbf{b}$ and $\\mathbf{c}$, such that $\\mathbf{a}_{i}=-i^{2}, \\mathbf{b}_{i}=i$ and $\\mathbf{c}=-1$ (as above). For any $1 \\leqslant j \\leqslant n$, write $\\mathbf{d}(j)$ for the tuple $2 \\cdot \\mathbf{a}+4 j \\cdot \\mathbf{b}+\\left(2 j^{2}-1\\right) \\cdot \\mathbf{c}$, which Lucy can make by adding together $\\mathbf{a}, \\mathbf{b}$ and $\\mathbf{c}$ repeatedly. This has $i$ th term $$ \\begin{aligned} \\mathbf{d}(j)_{i} & =2 \\mathbf{a}_{i}+4 j \\mathbf{b}_{i}+\\left(2 j^{2}-1\\right) \\mathbf{c}_{i} \\\\ & =-2 i^{2}+4 i j-\\left(2 j^{2}-1\\right) \\\\ & =1-2(i-j)^{2} \\end{aligned} $$ This is 1 if $j=i$, and at most -1 otherwise. Hence Lucy can produce the tuple $\\mathbf{1}=(1, \\ldots, 1)$ as $\\mathbf{d}(1) \\vee \\cdots \\vee \\mathbf{d}(n)$. She can then produce the constant tuple $\\mathbf{0}=(0, \\ldots, 0)$ as $\\mathbf{1}+\\mathbf{c}$, and for any $1 \\leqslant j \\leqslant n$ she can then produce the tuple $\\mathbf{e}(j)$ as $\\mathbf{d}(j) \\vee \\mathbf{0}$. Since she can now produce $\\mathbf{e}(1), \\ldots, \\mathbf{e}(n)$ and already had c, she can (as we argued earlier) produce any integer-valued tuple. Two tuples do not suffice. We start with an observation: Let $a$ be a non-negative real number and suppose that two tuples $\\mathbf{v}$ and $\\mathbf{w}$ satisfy $\\mathbf{v}_{j} \\geqslant a \\mathbf{v}_{k}$ and $\\mathbf{w}_{j} \\geqslant a \\mathbf{w}_{k}$ for some $1 \\leqslant j, k \\leqslant n$. Then we claim that the same inequality holds for $\\mathbf{v}+\\mathbf{w}$ and $\\mathbf{v} \\vee \\mathbf{w}$ : Indeed, the property for the sum is verified by an easy computation: $$ (\\mathbf{v}+\\mathbf{w})_{j}=\\mathbf{v}_{j}+\\mathbf{w}_{j} \\geqslant a \\mathbf{v}_{k}+a \\mathbf{w}_{k}=a(\\mathbf{v}+\\mathbf{w})_{k} $$ For the second operation, we denote by $\\mathbf{m}$ the tuple $\\mathbf{v} \\vee \\mathbf{w}$. Then $\\mathbf{m}_{j} \\geqslant \\mathbf{v}_{j} \\geqslant a \\mathbf{v}_{k}$ and $\\mathbf{m}_{j} \\geqslant \\mathbf{w}_{j} \\geqslant a \\mathbf{w}_{k}$. Since $\\mathbf{m}_{k}=\\mathbf{v}_{k}$ or $\\mathbf{m}_{k}=\\mathbf{w}_{k}$, the observation follows. As a consequence of this observation we have that if all starting tuples satisfy such an inequality, then all generated tuples will also satisfy it, and so we would not be able to obtain every integer-valued tuple. Let us now prove that Lucy needs at least three starting tuples. For contradiction, let us suppose that Lucy started with only two tuples $\\mathbf{v}$ and $\\mathbf{w}$. We are going to distinguish two cases. In the first case, suppose we can find a coordinate $i$ such that $\\mathbf{v}_{i}, \\mathbf{w}_{i} \\geqslant 0$. Both operations preserve the sign, thus we can not generate any tuple that has negative $i$-th coordinate. Similarly for $\\mathbf{v}_{i}, \\mathbf{w}_{i} \\leqslant 0$. Suppose the opposite, i.e., for every $i$ we have either $\\mathbf{v}_{i}>0>\\mathbf{w}_{i}$, or $\\mathbf{v}_{i}<0<\\mathbf{w}_{i}$. Since we assumed that our tuples have at least three coordinates, by pigeonhole principle there exist two coordinates $j \\neq k$ such that $\\mathbf{v}_{j}$ has the same sign as $\\mathbf{v}_{k}$ and $\\mathbf{w}_{j}$ has the same sign as $\\mathbf{w}_{k}$ (because there are only two possible combinations of signs). Without loss of generality assume that $\\mathbf{v}_{j}, \\mathbf{v}_{k}>0$ and $\\mathbf{w}_{j}, \\mathbf{w}_{k}<0$. Let us denote the positive real number $\\mathbf{v}_{j} \/ \\mathbf{v}_{k}$ by $a$. If $\\mathbf{w}_{j} \/ \\mathbf{w}_{k} \\leqslant a$, then both inequalities $\\mathbf{v}_{j} \\geqslant a \\mathbf{v}_{k}$ and $\\mathbf{w}_{j} \\geqslant a \\mathbf{w}_{k}$ are satisfied. On the other hand, if $\\mathbf{w}_{j} \/ \\mathbf{w}_{k} \\leqslant a$, then both $\\mathbf{v}_{k} \\geqslant(1 \/ a) \\mathbf{v}_{j}$ and $\\mathbf{w}_{k} \\geqslant(1 \/ a) \\mathbf{w}_{j}$ are satisfied. In either case, we have found the desired inequality satisfied by both starting tuples, a contradiction with the observation above.","tier":0} +{"problem_type":"Common remarks.","problem_label":"C8","problem":"Alice fills the fields of an $n \\times n$ board with numbers from 1 to $n^{2}$, each number being used exactly once. She then counts the total number of good paths on the board. A good path is a sequence of fields of arbitrary length (including 1) such that: (i) The first field in the sequence is one that is only adjacent to fields with larger numbers, (ii) Each subsequent field in the sequence is adjacent to the previous field, (iii) The numbers written on the fields in the sequence are in increasing order. Two fields are considered adjacent if they share a common side. Find the smallest possible number of good paths Alice can obtain, as a function of $n$. (Serbia) Answer: $2 n^{2}-2 n+1$.","solution":"We will call any field that is only adjacent to fields with larger numbers a well. Other fields will be called non-wells. Let us make a second $n \\times n$ board $B$ where in each field we will write the number of good sequences which end on the corresponding field in the original board $A$. We will thus look for the minimal possible value of the sum of all entries in $B$. We note that any well has just one good path ending in it, consisting of just the well, and that any other field has the number of good paths ending in it equal to the sum of this quantity for all the adjacent fields with smaller values, since a good path can only come into the field from a field of lower value. Therefore, if we fill in the fields in $B$ in increasing order with respect to their values in $A$, it follows that each field not adjacent to any already filled field will receive a 1, while each field adjacent to already filled fields will receive the sum of the numbers already written on these adjacent fields. We note that there is at least one well in $A$, that corresponding with the field with the entry 1 in $A$. Hence, the sum of values of fields in $B$ corresponding to wells in $A$ is at least 1 . We will now try to minimize the sum of the non-well entries, i.e. of the entries in $B$ corresponding to the non-wells in $A$. We note that we can ascribe to each pair of adjacent fields the value of the lower assigned number and that the sum of non-well entries will then equal to the sum of the ascribed numbers. Since the lower number is still at least 1, the sum of non-well entries will at least equal the number of pairs of adjacent fields, which is $2 n(n-1)$. Hence, the total minimum sum of entries in $B$ is at least $2 n(n-1)+1=2 n^{2}-2 n+1$. The necessary conditions for the minimum to be achieved is for there to be only one well and for no two entries in $B$ larger than 1 to be adjacent to each other. We will now prove that the lower limit of $2 n^{2}-2 n+1$ entries can be achieved. This amounts to finding a way of marking a certain set of squares, those that have a value of 1 in $B$, such that no two unmarked squares are adjacent and that the marked squares form a connected tree with respect to adjacency. For $n=1$ and $n=2$ the markings are respectively the lone field and the L-trimino. Now, for $n>2$, let $s=2$ for $n \\equiv 0,2 \\bmod 3$ and $s=1$ for $n \\equiv 1 \\bmod 3$. We will take indices $k$ and $l$ to be arbitrary non-negative integers. For $n \\geqslant 3$ we will construct a path of marked squares in the first two columns consisting of all squares of the form $(1, i)$ where $i$ is not of the form $6 k+s$ and $(2, j)$ where $j$ is of the form $6 k+s-1,6 k+s$ or $6+s+1$. Obviously, this path is connected. Now, let us consider the fields $(2,6 k+s)$ and $(1,6 k+s+3)$. For each considered field $(i, j)$ we will mark all squares of the form $(l, j)$ for $l>i$ and $(i+2 k, j \\pm 1)$. One can easily see that no set of marked fields will produce a cycle, that the only fields of the unmarked form $(1,6 k+s),(2+2 l+1,6 k+s \\pm 1)$ and $(2+2 l, 6 k+s+3 \\pm 1)$ and that no two are adjacent, since the consecutive considered fields are in columns of opposite parity. Examples of markings are given for $n=3,4,5,6,7$, and the corresponding constructions for $A$ and $B$ are given for $n=5$. ![](https:\/\/cdn.mathpix.com\/cropped\/2024_11_18_86efb4389f14b52b08e1g-41.jpg?height=352&width=1648&top_left_y=384&top_left_x=204)","tier":0} +{"problem_type":"Common remarks.","problem_label":"C9","problem":"Let $\\mathbb{Z}_{\\geqslant 0}$ be the set of non-negative integers, and let $f: \\mathbb{Z}_{\\geqslant 0} \\times \\mathbb{Z}_{\\geqslant 0} \\rightarrow \\mathbb{Z}_{\\geqslant 0}$ be a bijection such that whenever $f\\left(x_{1}, y_{1}\\right)>f\\left(x_{2}, y_{2}\\right)$, we have $f\\left(x_{1}+1, y_{1}\\right)>f\\left(x_{2}+1, y_{2}\\right)$ and $f\\left(x_{1}, y_{1}+1\\right)>f\\left(x_{2}, y_{2}+1\\right)$. Let $N$ be the number of pairs of integers $(x, y)$, with $0 \\leqslant x, y<100$, such that $f(x, y)$ is odd. Find the smallest and largest possible value of $N$. Answer: The optimal bounds are $2500 \\leqslant N \\leqslant 7500$.","solution":"We defer the constructions to the end of the solution. Instead, we begin by characterizing all such functions $f$, prove a formula and key property for such functions, and then solve the problem, providing constructions. Characterization Suppose $f$ satisfies the given relation. The condition can be written more strongly as $$ \\begin{aligned} f\\left(x_{1}, y_{1}\\right)>f\\left(x_{2}, y_{2}\\right) & \\Longleftrightarrow f\\left(x_{1}+1, y_{1}\\right)>f\\left(x_{2}+1, y_{2}\\right) \\\\ & \\Longleftrightarrow f\\left(x_{1}, y_{1}+1\\right)>f\\left(x_{2}, y_{2}+1\\right) . \\end{aligned} $$ In particular, this means for any $(k, l) \\in \\mathbb{Z}^{2}, f(x+k, y+l)-f(x, y)$ has the same sign for all $x$ and $y$. Call a non-zero vector $(k, l) \\in \\mathbb{Z}_{\\geqslant 0} \\times \\mathbb{Z}_{\\geqslant 0}$ a needle if $f(x+k, y)-f(x, y+l)>0$ for all $x$ and $y$. It is not hard to see that needles and non-needles are both closed under addition, and thus under scalar division (whenever the quotient lives in $\\mathbb{Z}^{2}$ ). In addition, call a positive rational number $\\frac{k}{l}$ a grade if the vector $(k, l)$ is a needle. (Since needles are closed under scalar multiples and quotients, this is well-defined.) Claim. Grades are closed upwards. Proof. Consider positive rationals $k_{1} \/ l_{1}0$ and $(1,0)$ is a needle). Thus $\\left(k_{2}, l_{2}\\right)$ is a needle, as wanted. Claim. A grade exists. Proof. If no positive integer $n$ is a grade, then $f(1,0)>f(0, n)$ for all $n$ which is impossible. Similarly, there is an $n$ such that $f(0,1)x_{2}+y_{2} \\alpha$ then $f\\left(x_{1}, y_{1}\\right)>f\\left(x_{2}, y_{2}\\right)$. Proof. If both $x_{1} \\geqslant x_{2}$ and $y_{1} \\geqslant y_{2}$ this is clear. Suppose $x_{1} \\geqslant x_{2}$ and $y_{1}\\alpha$ is a grade. This gives $f\\left(x_{1}, y_{1}\\right)>f\\left(x_{2}, y_{2}\\right)$. Suppose $x_{1}0} \\mid x+(y+1) \\alpha \\leqslant a+b \\alpha<(x+1)+(y+1) \\alpha\\right\\}+ \\\\ \\#\\left\\{(a, 0) \\in \\mathbb{Z}_{\\geqslant 0} \\times \\mathbb{Z}_{\\geqslant 0} \\mid(x+1)+y \\alpha \\leqslant a<(x+1)+(y+1) \\alpha\\right\\}= \\\\ \\#\\left\\{(a, b) \\in \\mathbb{Z}_{\\geqslant 0} \\times \\mathbb{Z}_{\\geqslant 0} \\mid x+y \\alpha \\leqslant a+b \\alpha<(x+1)+y \\alpha\\right\\}+1=f(x+1, y)-f(x, y) . \\end{gathered} $$ From this claim we immediately get that $2500 \\leqslant N \\leqslant 7500$; now we show that those bounds are indeed sharp. Remember that if $\\alpha$ is irrational then $$ f(a, b)=\\#\\left\\{(x, y) \\in \\mathbb{Z}_{\\geqslant 0} \\times \\mathbb{Z}_{\\geqslant 0} \\mid x+y \\alpha\\frac{2022}{1344}=\\frac{337}{224}=\\frac{3}{2}+\\frac{1}{224} $$ If $a>1$ then $$ \\frac{1}{a}+\\frac{1}{b}+\\frac{1}{c} \\leqslant \\frac{1}{2}+\\frac{1}{3}+\\frac{1}{4}=\\frac{13}{12}<\\frac{3}{2} $$ so it must be the case that $a=1$. Similarly, it must hold that $b<4$ since otherwise $$ 1+\\frac{1}{b}+\\frac{1}{c} \\leqslant 1+\\frac{1}{4}+\\frac{1}{5}<\\frac{3}{2} . $$ This leaves two cases to check, $b=2$ and $b=3$. Case $b=3$. Then $$ \\frac{1}{c}>\\frac{3}{2}+\\frac{1}{224}-1-\\frac{1}{3}>\\frac{1}{6}, $$ so $c=4$ or $c=5$. If $c=4$ then $$ 2022=N\\left(1+\\frac{1}{3}+\\frac{1}{4}\\right)=\\frac{19}{12} N $$ but this is impossible as $19 \\nmid 2022$. If $c=5$ then $$ 2022=N\\left(1+\\frac{1}{3}+\\frac{1}{5}\\right)=\\frac{23}{15} N $$ which again is impossible, as $23 \\nmid 2022$. Case $b=2$. Note that $c<224$ since $$ \\frac{1}{c}>\\frac{3}{2}+\\frac{1}{224}-1-\\frac{1}{2}=\\frac{1}{224} $$ It holds that $$ 2022=N\\left(1+\\frac{1}{2}+\\frac{1}{c}\\right)=\\frac{3 c+2}{2 c} N \\Rightarrow(3 c+2) N=4044 c $$ Since $(c, 3 c-2)=(c, 2) \\in\\{1,2\\}$, then $3 c+2 \\mid 8088=2^{3} \\cdot 3 \\cdot 337$ which implies that $3 c+2 \\mid 2^{3} \\cdot 337$. But since $3 c+2 \\geqslant 3 \\cdot 3+2>8=2^{3}$ and $3 c+2 \\neq 337$, then it must hold that $3 c+2 \\geqslant 2 \\cdot 337$, contradicting $c<224$.","tier":0} +{"problem_type":"Number Theory","problem_label":"N2","problem":"Find all positive integers $n>2$ such that $$ n!\\mid \\prod_{\\substack{p2$ and 3 divides one of $p_{m-2}, p_{m-1}=p_{m-2}+2$ and $p_{m}=p_{m-2}+4$. This implies $p_{m-2}=3$ and thus $p_{m}=7$, giving $7 \\leqslant n<11$. Finally, a quick computation shows that $7!\\mid \\prod_{p1$ be a positive integer, and let $d>1$ be a positive integer coprime to $a$. Let $x_{1}=1$ and, for $k \\geqslant 1$, define $$ x_{k+1}= \\begin{cases}x_{k}+d & \\text { if } a \\text { doesn't divide } x_{k} \\\\ x_{k} \/ a & \\text { if } a \\text { divides } x_{k}\\end{cases} $$ Find the greatest positive integer $n$ for which there exists an index $k$ such that $x_{k}$ is divisible by $a^{n}$. (Croatia) Answer: $n$ is the exponent with $d1$ be a positive integer, and let $d>1$ be a positive integer coprime to $a$. Let $x_{1}=1$ and, for $k \\geqslant 1$, define $$ x_{k+1}= \\begin{cases}x_{k}+d & \\text { if } a \\text { doesn't divide } x_{k} \\\\ x_{k} \/ a & \\text { if } a \\text { divides } x_{k}\\end{cases} $$ Find the greatest positive integer $n$ for which there exists an index $k$ such that $x_{k}$ is divisible by $a^{n}$. (Croatia) Answer: $n$ is the exponent with $d0}: 0d$ then $y-d \\in S$ but $a \\cdot y \\notin S$; otherwise, if $yd, \\\\ a \\cdot y & \\text { if } y1$ be an index such that $x_{k_{1}}=1$. Then $$ x_{k_{1}}=1, \\quad x_{k_{1}-1}=f^{-1}(1)=a, x_{k_{1}-2}=f^{-1}(a)=a^{2}, \\ldots, \\quad x_{k_{1}-n}=a^{n} $$","tier":0} +{"problem_type":"Number Theory","problem_label":"N3","problem":"Let $a>1$ be a positive integer, and let $d>1$ be a positive integer coprime to $a$. Let $x_{1}=1$ and, for $k \\geqslant 1$, define $$ x_{k+1}= \\begin{cases}x_{k}+d & \\text { if } a \\text { doesn't divide } x_{k} \\\\ x_{k} \/ a & \\text { if } a \\text { divides } x_{k}\\end{cases} $$ Find the greatest positive integer $n$ for which there exists an index $k$ such that $x_{k}$ is divisible by $a^{n}$. (Croatia) Answer: $n$ is the exponent with $da^{n} \/ a=a^{n-1}$ the LHS is strictly less than $a^{v-n}$. This implies that on the RHS, the coefficients of $a^{v-n}, a^{v-n+1}, \\ldots$ must all be zero, i.e. $z_{v-n}=z_{v-n+1}=\\cdots=z_{v-1}=0$. This implies that there are $n$ consecutive decreasing indices in the original sequence.","tier":0} +{"problem_type":"Number Theory","problem_label":"N4","problem":"Find all triples of positive integers $(a, b, p)$ with $p$ prime and $$ a^{p}=b!+p $$ (Belgium) Answer: $(2,2,2)$ and $(3,4,3)$.","solution":"Clearly, $a>1$. We consider three cases. Case 1: We have $ab$ which is also impossible since in this case we have $b!\\leqslant a!a>1$. Case 2: We have $a>p$. In this case $b!=a^{p}-p>p^{p}-p \\geqslant p!$ so $b>p$ which means that $a^{p}=b!+p$ is divisible by $p$. Hence, $a$ is divisible by $p$ and $b!=a^{p}-p$ is not divisible by $p^{2}$. This means that $b<2 p$. If $a(2 p-1)!+p \\geqslant b!+p$. Comment. The inequality $p^{2 p}>(2 p-1)!+p$ can be shown e.g. by using $$ (2 p-1)!=[1 \\cdot(2 p-1)] \\cdot[2 \\cdot(2 p-2)] \\cdots \\cdots[(p-1)(p+1)] \\cdot p<\\left(\\left(\\frac{2 p}{2}\\right)^{2}\\right)^{p-1} \\cdot p=p^{2 p-1} $$ where the inequality comes from applying AM-GM to each of the terms in square brackets. Case 3: We have $a=p$. In this case $b!=p^{p}-p$. One can check that the values $p=2,3$ lead to the claimed solutions and $p=5$ does not lead to a solution. So we now assume that $p \\geqslant 7$. We have $b!=p^{p}-p>p$ ! and so $b \\geqslant p+1$ which implies that $v_{2}((p+1)!) \\leqslant v_{2}(b!)=v_{2}\\left(p^{p-1}-1\\right) \\stackrel{L T E}{=} 2 v_{2}(p-1)+v_{2}(p+1)-1=v_{2}\\left(\\frac{p-1}{2} \\cdot(p-1) \\cdot(p+1)\\right)$, where in the middle we used lifting-the-exponent lemma. On the RHS we have three factors of $(p+1)!$. But, due to $p+1 \\geqslant 8$, there are at least 4 even numbers among $1,2, \\ldots, p+1$, so this case is not possible.","tier":0} +{"problem_type":"Number Theory","problem_label":"N4","problem":"Find all triples of positive integers $(a, b, p)$ with $p$ prime and $$ a^{p}=b!+p $$ (Belgium) Answer: $(2,2,2)$ and $(3,4,3)$.","solution":"The cases $a \\neq p$ are covered as in solution 1 , as are $p=2,3$. For $p \\geqslant 5$ we have $b!=p\\left(p^{p-1}-1\\right)$. By Zsigmondy's Theorem there exists some prime $q$ that divides $p^{p-1}-1$ but does not divide $p^{k}-1$ for $k(2 p-1)^{p-1} p>p^{p}>p^{p}-p$, a contradiction.","tier":0} +{"problem_type":"Number Theory","problem_label":"N4","problem":"Find all triples of positive integers $(a, b, p)$ with $p$ prime and $$ a^{p}=b!+p $$ (Belgium) Answer: $(2,2,2)$ and $(3,4,3)$.","solution":"The cases $a \\neq p$ are covered as in solution 1 , as are $p=2,3$. Also $b>p$, as $p^{p}>p!+p$ for $p>2$. The cases $p=5,7,11$ are also checked manually, so assume $p \\geqslant 13$. Let $q \\mid p+1$ be an odd prime. By LTE $$ v_{q}\\left(p^{p}-p\\right)=v_{q}\\left(\\left(p^{2}\\right)^{\\frac{p-1}{2}}-1\\right)=v_{q}\\left(p^{2}-1\\right)+v_{q}\\left(\\frac{p-1}{2}\\right)=v_{q}(p+1) $$ But $b \\geqslant p+1$, so then $v_{q}(b!)>v_{q}(p+1)$, since $qq^{d-1} \\geqslant 2 d $$ provided $d \\geqslant 2$ and $q>3$, or $d \\geqslant 3$. If $q=3, d=2$ and $p \\geqslant 13$ then $v_{q}(b!) \\geqslant v_{q}(p!) \\geqslant v_{q}(13!)=5>2 d$. Either way, $d \\leqslant 1$. If $p>2 q+1$ (so $p>3 q$, as $q \\mid p-1$ ) then $$ v_{q}(b!) \\geqslant v_{q}((3 q)!)=3, $$ so we must have $q \\geqslant \\frac{p}{2}$, in other words, $p-1=2 q$. This implies that $p=2^{k}-1$ and $q=2^{k-1}-1$ are both prime, but it is not possible to have two consecutive Mersenne primes.","tier":0} +{"problem_type":"Number Theory","problem_label":"N4","problem":"Find all triples of positive integers $(a, b, p)$ with $p$ prime and $$ a^{p}=b!+p $$ (Belgium) Answer: $(2,2,2)$ and $(3,4,3)$.","solution":"Let $a=p, b>p$ and $p \\geqslant 5$ (the remaining cases are dealt with as in solution 3). Modulo $(p+1)^{2}$ it holds that $p^{p}-p=(p+1-1)^{p}-p \\equiv\\binom{p}{1}(p+1)(-1)^{p-1}+(-1)^{p}-p=p(p+1)-1-p=p^{2}-1 \\not \\equiv 0 \\quad \\bmod \\left((p+1)^{2}\\right)$. Since $p \\geqslant 5$, the numbers 2 and $\\frac{p+1}{2}$ are distinct and less than or equal to $p$. Therefore, $p+1 \\mid p$, and so $(p+1)^{2} \\mid(p+1)$ !. But $b \\geqslant p+1$, so $b!\\equiv 0 \\not \\equiv p^{p}-p \\bmod (p+1)^{2}$, a contradiction.","tier":0} +{"problem_type":"Number Theory","problem_label":"N6","problem":"Let $Q$ be a set of prime numbers, not necessarily finite. For a positive integer $n$ consider its prime factorisation; define $p(n)$ to be the sum of all the exponents and $q(n)$ to be the sum of the exponents corresponding only to primes in $Q$. A positive integer $n$ is called special if $p(n)+p(n+1)$ and $q(n)+q(n+1)$ are both even integers. Prove that there is a constant $c>0$ independent of the set $Q$ such that for any positive integer $N>100$, the number of special integers in $[1, N]$ is at least $c N$. (For example, if $Q=\\{3,7\\}$, then $p(42)=3, q(42)=2, p(63)=3, q(63)=3, p(2022)=3$, $q(2022)=1$.) (Costa Rica)","solution":"Let us call two positive integers $m, n$ friends if $p(m)+p(n)$ and $q(m)+q(n)$ are both even integers. We start by noting that the pairs $(p(k), q(k))$ modulo 2 can take at most 4 different values; thus, among any five different positive integers there are two which are friends. In addition, both functions $p$ and $q$ satisfy $f(a b)=f(a)+f(b)$ for any $a, b$. Therefore, if $m$ and $n$ are divisible by $d$, then both $p$ and $q$ satisfy the equality $f(m)+f(n)=f(m \/ d)+$ $f(n \/ d)+2 f(d)$. This implies that $m, n$ are friends if and only if $m \/ d, n \/ d$ are friends. Let us call a set of integers $\\left\\{n_{1}, n_{2}, \\ldots, n_{5}\\right\\}$ an interesting set if for any indexes $i, j$, the difference $d_{i j}=\\left|n_{i}-n_{j}\\right|$ divides both $n_{i}$ and $n_{j}$. We claim that if elements of an interesting set are all positive, then we can obtain a special integer. Indeed, if we were able to construct such a set, then there would be a pair of integers $\\left\\{n_{i}, n_{j}\\right\\}$ which are friends, according to the first observation. Additionally, the second observation yields that the quotients $n_{i} \/ d_{i j}, n_{j} \/ d_{i j}$ form a pair of friends, which happen to be consecutive integers, thus giving a special integer as desired. In order to construct a family of interesting sets, we can start by observing that the set $\\{0,6,8,9,12\\}$ is an interesting set. Using that $72=2^{3} \\cdot 3^{2}$ is the least common multiple of all pairwise differences in this set, we obtain a family of interesting sets by considering $$ \\{72 k, 72 k+6,72 k+8,72 k+9,72 k+12\\} $$ for any $k \\geqslant 1$. If we consider the quotients (of these numbers by the appropriate differences), then we obtain that the set $$ S_{k}=\\{6 k, 8 k, 9 k, 12 k, 12 k+1,18 k+2,24 k+2,24 k+3,36 k+3,72 k+8\\}, $$ has at least one special integer. In particular, the interval $[1,100 k]$ contains the sets $S_{1}, S_{2}, \\ldots, S_{k}$, each of which has a special number. Any special number can be contained in at most ten sets $S_{k}$, from where we conclude that the number of special integers in $[1,100 k]$ is at least $k \/ 10$. Finally, let $N=100 k+r$, with $k \\geqslant 1$ and $0 \\leqslant r<100$, so that we have $N<100(k+1) \\leqslant$ $200 k$. Then the number of special integers in $[1, N]$ is at least $k \/ 10>N \/ 2000$, as we wanted to prove. Comment 1. The statement is also true for $N \\geqslant 15$ as at least one of the numbers $7,14,15$ is special. Comment 2. Another approach would be to note that if $p(2 n), p(2 n+1), p(2 n+2)$ all have the same parity then one of the numbers $n, 2 n, 2 n+1$ is special. Indeed, if $q(n)+q(n+1)$ is even then $n$ is special since $p(n)+p(n+1) \\equiv p(2 n)+p(2 n+2) \\equiv 0(\\bmod 2)$. Otherwise, if $q(n)+q(n+1)$ is odd, so is $q(2 n)+q(2 n+2)$ which implies that exactly one of the numbers $2 n, 2 n+1$ is special. Unfortunately, it seems hard to show that the set of such $n$ has positive density: see a recent paper https:\/\/arxiv.org\/abs\/1509.01545 for the proof that all eight patterns of the parities of $p(n), p(n+1), p(n+2)$ appear for a positive proportion of positive integers. This page is intentionally left blank","tier":0} +{"problem_type":"Number Theory","problem_label":"N7","problem":"Let $k$ be a positive integer and let $S$ be a finite set of odd prime numbers. Prove that there is at most one way (modulo rotation and reflection) to place the elements of $S$ around a circle such that the product of any two neighbors is of the form $x^{2}+x+k$ for some positive integer $x$.","solution":"Let us allow the value $x=0$ as well; we prove the same statement under this more general constraint. Obviously that implies the statement with the original conditions. Call a pair $\\{p, q\\}$ of primes with $p \\neq q$ special if $p q=x^{2}+x+k$ for some nonnegative integer $x$. The following claim is the key mechanism of the problem: Claim. (a) For every prime $r$, there are at most two primes less than $r$ forming a special pair with $r$. (b) If such $p$ and $q$ exist, then $\\{p, q\\}$ is itself special. We present two proofs of the claim. Proof 1. We are interested in integers $1 \\leqslant x\\sqrt{m}$ choices for $x$ and $y$, so there are more than $m$ possible pairs $(x, y)$. Hence, two of these sums are congruent modulo $m$ : $5^{k+1} x_{1}+3^{k+1} y_{1} \\equiv 5^{k+1} x_{2}+3^{k+1} y_{2}(\\bmod m)$. Now choose $a=x_{1}-x_{2}$ and $b=y_{1}-y_{2}$; at least one of $a, b$ is nonzero, and $$ 5^{k+1} a+3^{k+1} b \\equiv 0 \\quad(\\bmod m), \\quad|a|,|b| \\leqslant \\sqrt{m} $$ From $$ 0 \\equiv\\left(5^{k+1} a\\right)^{2}-\\left(3^{k+1} b\\right)^{2}=5^{n+1} a^{2}-3^{n+1} b^{2} \\equiv 5 \\cdot 3^{n} a^{2}-3^{n+1} b^{2}=3^{n}\\left(5 a^{2}-3 b^{2}\\right) \\quad(\\bmod m) $$ we can see that $\\left|5 a^{2}-3 b^{2}\\right|$ is a multiple of $m$. Since at least one of $a$ and $b$ is nonzero, $5 a^{2} \\neq 3 b^{2}$. Hence, by the choice of $a, b$, we have $0<\\left|5 a^{2}-3 b^{2}\\right| \\leqslant \\max \\left(5 a^{2}, 3 b^{2}\\right) \\leqslant 5 m$. That shows that $m_{1} \\leqslant 5 m$. II. Next, we show that $m_{1}$ cannot be divisible by 2,3 and 5 . Since $m_{1}$ equals either $\\left|5 a^{2}-3 b^{2}\\right|$ or $\\left|a^{2}-15 b^{2}\\right|$ with some integers $a, b$, we have six cases to check. In all six cases, we will get a contradiction by presenting another multiple of $m$, smaller than $m_{1}$. - If $5 \\mid m_{1}$ and $m_{1}=\\left|5 a^{2}-3 b^{2}\\right|$, then $5 \\mid b$ and $\\left|a^{2}-15\\left(\\frac{b}{5}\\right)^{2}\\right|=\\frac{m_{1}}{5}1$ and $c$ are integers, and $X, Y$ are positive integers such that $X \\leqslant mm$, then $c x^{2} \\equiv d y^{2}(\\bmod m)$ has a solution such that at least one of $x, y$ is nonzero, $|x|5$ of $5 a^{2}-3 b^{2}$ or $a^{2}-15 b^{2}$, we have $$ 1=\\left(\\frac{15}{p}\\right)=\\left(\\frac{3}{p}\\right)\\left(\\frac{5}{p}\\right)=(-1)^{\\frac{p-1}{2}}\\left(\\frac{p}{3}\\right)\\left(\\frac{p}{5}\\right) $$ where $\\left(\\frac{a}{p}\\right)$ stands for the Legendre symbol. Considering the remainders of $p$ when divided by 4,3 and 5, (1) leads to $$ p \\equiv \\pm 1, \\pm 7, \\pm 11 \\text { or } \\pm 17 \\bmod 60 $$ These remainders form a subgroup of the reduced remainders modulo 60. Since 13 and 37 are not elements in this subgroup, the number $m=2^{n}+65$ cannot be a product of such primes. Instead of handling the prime divisors of $m$ separately, we can use Jacobi symbols for further simplification, as shown in the next solution.","tier":0} +{"problem_type":"Number Theory","problem_label":"N8","problem":"Prove that $5^{n}-3^{n}$ is not divisible by $2^{n}+65$ for any positive integer $n$. (Belgium)","solution":"Suppose again that $5^{n} \\equiv 3^{n}\\left(\\bmod m=2^{n}+65\\right)$. Like in the first solution, we conclude that $n$ must be odd, and $n \\geqslant 3$, so $8 \\mid 2^{n}$. Using Jacobi symbols, $$ -1=\\left(\\frac{2^{n}+65}{5}\\right)=\\left(\\frac{5}{2^{n}+65}\\right)=\\left(\\frac{5^{n}}{2^{n}+65}\\right)=\\left(\\frac{3^{n}}{2^{n}+65}\\right)=\\left(\\frac{3}{2^{n}+65}\\right)=\\left(\\frac{2^{n}+65}{3}\\right)=1 $$ contradiction. ## - ![](https:\/\/cdn.mathpix.com\/cropped\/2024_11_18_86efb4389f14b52b08e1g-75.jpg?height=98&width=726&top_left_y=279&top_left_x=114) + - \\$4=4 \\times 1.20\\$1 ![](https:\/\/cdn.mathpix.com\/cropped\/2024_11_18_86efb4389f14b52b08e1g-75.jpg?height=84&width=889&top_left_y=1990&top_left_x=379) Cly D - 1 vaiv ![](https:\/\/cdn.mathpix.com\/cropped\/2024_11_18_86efb4389f14b52b08e1g-75.jpg?height=58&width=116&top_left_y=2247&top_left_x=536) +1N ![](https:\/\/cdn.mathpix.com\/cropped\/2024_11_18_86efb4389f14b52b08e1g-75.jpg?height=135&width=646&top_left_y=2301&top_left_x=348)![](https:\/\/cdn.mathpix.com\/cropped\/2024_11_18_86efb4389f14b52b08e1g-75.jpg?height=1874&width=164&top_left_y=419&top_left_x=1607) ![](https:\/\/cdn.mathpix.com\/cropped\/2024_11_18_86efb4389f14b52b08e1g-75.jpg?height=130&width=432&top_left_y=1647&top_left_x=0) r \/ A M - 18 ![](https:\/\/cdn.mathpix.com\/cropped\/2024_11_18_86efb4389f14b52b08e1g-75.jpg?height=73&width=652&top_left_y=2395&top_left_x=139) ![](https:\/\/cdn.mathpix.com\/cropped\/2024_11_18_86efb4389f14b52b08e1g-75.jpg?height=58&width=637&top_left_y=2447&top_left_x=154) A ..... ![](https:\/\/cdn.mathpix.com\/cropped\/2024_11_18_86efb4389f14b52b08e1g-75.jpg?height=75&width=415&top_left_y=2501&top_left_x=148) ![](https:\/\/cdn.mathpix.com\/cropped\/2024_11_18_86efb4389f14b52b08e1g-75.jpg?height=61&width=452&top_left_y=2539&top_left_x=605) ![](https:\/\/cdn.mathpix.com\/cropped\/2024_11_18_86efb4389f14b52b08e1g-75.jpg?height=75&width=532&top_left_y=2587&top_left_x=599)","tier":0} diff --git a/IMO/segmented/en-compendium.jsonl b/IMO/segmented/en-compendium.jsonl new file mode 100644 index 0000000000000000000000000000000000000000..53b915630658748a3854f636a07ca15455797e0c --- /dev/null +++ b/IMO/segmented/en-compendium.jsonl @@ -0,0 +1,960 @@ +{"year":"1959","problem_phase":"contest","problem":"1. (POL) For every integer $n$ prove that the fraction $\\frac{21 n+4}{14 n+3}$ cannot be reduced any further.","solution":"1. The desired result $(14 n+3,21 n+4)=1$ follows from $$ 3(14 n+3)-2(21 n+4)=1 $$","problem_type":null,"tier":0} +{"year":"1959","problem_phase":"contest","problem":"2. (ROM) For which real numbers $x$ do the following equations hold: (a) $\\sqrt{x+\\sqrt{2 x-1}}+\\sqrt{x+\\sqrt{2 x-1}}=\\sqrt{2}$, (b) $\\sqrt{x+\\sqrt{2 x-1}}+\\sqrt{x+\\sqrt{2 x-1}}=1$, (c) $\\sqrt{x+\\sqrt{2 x-1}}+\\sqrt{x+\\sqrt{2 x-1}}=2$ ?","solution":"2. For the square roots to be real we must have $2 x-1 \\geq 0 \\Rightarrow x \\geq 1 \/ 2$ and $x \\geq \\sqrt{2 x-1} \\Rightarrow x^{2} \\geq 2 x-1 \\Rightarrow(x-1)^{2} \\geq 0$, which always holds. Then we have $\\sqrt{x+\\sqrt{2 x-1}}+\\sqrt{x-\\sqrt{2 x-1}}=c \\Longleftrightarrow$ $$ c^{2}=2 x+2 \\sqrt{x^{2}-\\sqrt{2 x-1}^{2}}=2 x+2|x-1|= \\begin{cases}2, & 1 \/ 2 \\leq x \\leq 1 \\\\ 4 x-2, & x \\geq 1\\end{cases} $$ (a) $c^{2}=2$. The equation holds for $1 \/ 2 \\leq x \\leq 1$. (b) $c^{2}=1$. The equation has no solution. (c) $c^{2}=4$. The equation holds for $4 x-2=4 \\Rightarrow x=3 \/ 2$.","problem_type":null,"tier":0} +{"year":"1959","problem_phase":"contest","problem":"3. (HUN) Let $x$ be an angle and let the real numbers $a, b, c, \\cos x$ satisfy the following equation: $$ a \\cos ^{2} x+b \\cos x+c=0 . $$ Write the analogous quadratic equation for $a, b, c, \\cos 2 x$. Compare the given and the obtained equality for $a=4, b=2, c=-1$. Second Day","solution":"3. Multiplying the equality by $4\\left(a \\cos ^{2} x-b \\cos x+c\\right)$, we obtain $4 a^{2} \\cos ^{4} x+$ $2\\left(4 a c-2 b^{2}\\right) \\cos ^{2} x+4 c^{2}=0$. Plugging in $2 \\cos ^{2} x=1+\\cos 2 x$ we obtain (after quite a bit of manipulation): $$ a^{2} \\cos ^{2} 2 x+\\left(2 a^{2}+4 a c-2 b^{2}\\right) \\cos 2 x+\\left(a^{2}+4 a c-2 b^{2}+4 c^{2}\\right)=0 $$ For $a=4, b=2$, and $c=-1$ we get $4 \\cos ^{2} x+2 \\cos x-1=0$ and $16 \\cos ^{2} 2 x+8 \\cos 2 x-4=0 \\Rightarrow 4 \\cos ^{2} 2 x+2 \\cos 2 x-1=0$.","problem_type":null,"tier":0} +{"year":"1959","problem_phase":"contest","problem":"4. (HUN) Construct a right-angled triangle whose hypotenuse $c$ is given if it is known that the median from the right angle equals the geometric mean of the remaining two sides of the triangle.","solution":"4. Analysis. Let $a$ and $b$ be the other two sides of the triangle. From the conditions of the problem we have $c^{2}=a^{2}+b^{2}$ and $c \/ 2=\\sqrt{a b} \\Leftrightarrow 3 \/ 2 c^{2}=$ $a^{2}+b^{2}+2 a b=(a+b)^{2} \\Leftrightarrow \\sqrt{3 \/ 2} c=a+b$. Given a desired $\\triangle A B C$ let $D$ be a point on $(A C$ such that $C D=C B$. In that case, $A D=a+b=\\sqrt{3 \/ 2} c$, and also, since $B C=C D$, it follows that $\\angle A D B=45^{\\circ}$. Construction. From a segment of length $c$ we elementarily construct a segment $A D$ of length $\\sqrt{3 \/ 2} c$. We then construct a ray ( $D X$ such that $\\angle A D X=45^{\\circ}$ and a circle $k(A, c)$ that intersects the ray at point $B$. Finally, we construct the perpendicular from $B$ to $A D$; point $C$ is the foot of that perpendicular. Proof. It holds that $A B=c$, and, since $C B=C D$, it also holds that $A C+$ $C B=A C+C D=A D=\\sqrt{3 \/ 2} c$. From this it follows that $\\sqrt{A C \\cdot C B}=$ $c \/ 2$. Since $B C$ is perpendicular to $A D$, it follows that $\\measuredangle B C A=90^{\\circ}$. Thus $A B C$ is the desired triangle. Discussion. Since $A B \\sqrt{2}=\\sqrt{2} c>\\sqrt{3 \/ 2} c=A D>A B$, the circle $k$ intersects the ray $D X$ in exactly two points, which correspond to two symmetric solutions.","problem_type":null,"tier":0} +{"year":"1959","problem_phase":"contest","problem":"5. (ROM) A segment $A B$ is given and on it a point $M$. On the same side of $A B$ squares $A M C D$ and $B M F E$ are constructed. The circumcircles of the two squares, whose centers are $P$ and $Q$, intersect in $M$ and another point $N$. (a) Prove that lines $F A$ and $B C$ intersect at $N$. (b) Prove that all such constructed lines $M N$ pass through the same point $S$, regardless of the selection of $M$. (c) Find the locus of the midpoints of all segments $P Q$, as $M$ varies along the segment $A B$.","solution":"5. (a) It suffices to prove that $A F \\perp B C$, since then for the intersection point $X$ we have $\\angle A X C=\\angle B X F=90^{\\circ}$, implying that $X$ belongs to the circumcircles of both squares and thus that $X=N$. The relation $A F \\perp B C$ holds because from $M A=M C, M F=M B$, and $\\angle A M C=\\angle F M B$ it follows that $\\triangle A M F$ is obtained by rotating $\\triangle B M C$ by $90^{\\circ}$ around $M$. (b) Since $N$ is on the circumcircle of $B M F E$, it follows that $\\angle A N M=$ $\\angle M N B=45^{\\circ}$. Hence $M N$ is the bisector of $\\angle A N B$. It follows that $M N$ passes through the midpoint of the $\\operatorname{arc} \\widehat{A B}$ of the circle with diameter $A B$ (i.e., the circumcircle of $\\triangle A B N$ ) not containing $N$. (c) Let us introduce a coordinate system such that $A=(0,0), B=(b, 0)$, and $M=(m, 0)$. Setting in general $W=\\left(x_{W}, y_{W}\\right)$ for an arbitrary point $W$ and denoting by $R$ the midpoint of $P Q$, we have $y_{R}=\\left(y_{P}+\\right.$ $\\left.y_{Q}\\right) \/ 2=(m+b-m) \/ 4=b \/ 4$ and $x_{R}=\\left(x_{P}+x_{Q}\\right) \/ 2=(m+m+b) \/ 4=$ $(2 m+b) \/ 4$, the parameter $m$ varying from 0 to $b$. Thus the locus of all points $R$ is the closed segment $R_{1} R_{2}$ where $R_{1}=(b \/ 4, b \/ 4)$ and $R_{2}=(b \/ 4,3 b \/ 4)$.","problem_type":null,"tier":0} +{"year":"1959","problem_phase":"contest","problem":"6. (CZS) Let $\\alpha$ and $\\beta$ be two planes intersecting at a line $p$. In $\\alpha$ a point $A$ is given and in $\\beta$ a point $C$ is given, neither of which lies on $p$. Construct $B$ in $\\alpha$ and $D$ in $\\beta$ such that $A B C D$ is an equilateral trapezoid, $A B \\| C D$, in which a circle can be inscribed.","solution":"6. Analysis. For $A B \\| C D$ to hold evidently neither must intersect $p$ and hence constructing lines $r$ in $\\alpha$ through $A$ and $s$ in $\\beta$ through $C$, both being parallel to $p$, we get that $B \\in r$ and $D \\in s$. Hence the problem reduces to a planar problem in $\\gamma$, determined by $r$ and $s$. Denote by $A^{\\prime}$ the foot of the perpendicular from $A$ to $s$. Since $A B C D$ is isosceles and has an incircle, it follows $A D=B C=(A B+C D) \/ 2=A^{\\prime} C$. The remaining parts of the problem are now obvious.","problem_type":null,"tier":0} +{"year":"1960","problem_phase":"contest","problem":"1. (BUL) Find all the three-digit numbers for which one obtains, when dividing the number by 11 , the sum of the squares of the digits of the initial number.","solution":"1. Given the number $\\overline{a c b}$, since $11 \\mid \\overline{a c b}$, it follows that $c=a+b$ or $c=$ $a+b-11$. In the first case, $a^{2}+b^{2}+(a+b)^{2}=10 a+b$, and in the second case, $a^{2}+b^{2}+(a+b-11)^{2}=10(a-1)+b$. In the first case the LHS is even, and hence $b \\in\\{0,2,4,6,8\\}$, while in the second case it is odd, and hence $b \\in\\{1,3,5,7,9\\}$. Analyzing the 10 quadratic equations for $a$ we obtain that the only valid solutions are 550 and 803.","problem_type":null,"tier":0} +{"year":"1960","problem_phase":"contest","problem":"2. (HUN) For which real numbers $x$ does the following inequality hold: $$ \\frac{4 x^{2}}{(1-\\sqrt{1+2 x})^{2}}<2 x+9 ? $$","solution":"2. The LHS term is well-defined for $x \\geq-1 \/ 2$ and $x \\neq 0$. Furthermore, $4 x^{2} \/(1-\\sqrt{1+2 x})^{2}=(1+\\sqrt{1+2 x})^{2}$. Since $f(x)=(1+\\sqrt{1+2 x})^{2}-2 x-$ $9=2 \\sqrt{1+2 x}-7$ is increasing and since $f(45 \/ 8)=0$, it follows that the inequality holds precisely for $-1 \/ 2 \\leq x<45 \/ 8$ and $x \\neq 0$.","problem_type":null,"tier":0} +{"year":"1960","problem_phase":"contest","problem":"3. (ROM) A right-angled triangle $A B C$ is given for which the hypotenuse $B C$ has length $a$ and is divided into $n$ equal segments, where $n$ is odd. Let $\\alpha$ be the angle with which the point $A$ sees the segment containing the middle of the hypotenuse. Prove that $$ \\tan \\alpha=\\frac{4 n h}{\\left(n^{2}-1\\right) a}, $$ where $h$ is the height of the triangle. ## Second Day","solution":"3. Let $B^{\\prime} C^{\\prime}$ be the middle of the $n=2 k+1$ segments and let $D$ be the foot of the perpendicular from $A$ to the hypotenuse. Let us assume $\\mathcal{B}\\left(C, D, C^{\\prime}, B^{\\prime}, B\\right)$. Then from $C Da b$ there are two solutions, if $h^{2}=a b$ there is only one solution, and if $h^{2}a^{2}>b^{2}$ and $a>0$.","problem_type":null,"tier":0} +{"year":"1961","problem_phase":"contest","problem":"2. (POL) Let $a, b$, and $c$ be the lengths of a triangle whose area is $S$. Prove that $$ a^{2}+b^{2}+c^{2} \\geq 4 S \\sqrt{3} $$ In what case does equality hold?","solution":"2. Using $S=b c \\sin \\alpha \/ 2, a^{2}=b^{2}+c^{2}-2 b c \\cos \\alpha$ and $(\\sqrt{3} \\sin \\alpha+\\cos \\alpha) \/ 2=$ $\\cos \\left(\\alpha-60^{\\circ}\\right)$ we have $$ \\begin{gathered} a^{2}+b^{2}+c^{2} \\geq 4 S \\sqrt{3} \\Leftrightarrow b^{2}+c^{2} \\geq b c(\\sqrt{3} \\sin \\alpha+\\cos \\alpha) \\Leftrightarrow \\\\ \\Leftrightarrow(b-c)^{2}+2 b c\\left(1-\\cos \\left(\\alpha-60^{\\circ}\\right)\\right) \\geq 0, \\end{gathered} $$ where equality holds if and only if $b=c$ and $\\alpha=60^{\\circ}$, i.e., if the triangle is equilateral.","problem_type":null,"tier":0} +{"year":"1961","problem_phase":"contest","problem":"3. (BUL) Solve the equation $\\cos ^{n} x-\\sin ^{n} x=1$, where $n$ is a given positive integer. ## Second Day","solution":"3. For $n \\geq 2$ we have $$ \\begin{aligned} 1 & =\\cos ^{n} x-\\sin ^{n} x \\leq\\left|\\cos ^{n} x-\\sin ^{n} x\\right| \\\\ & \\leq\\left|\\cos ^{n} x\\right|+\\left|\\sin ^{n} x\\right| \\leq \\cos ^{2} x+\\sin ^{2} x=1 \\end{aligned} $$ Hence $\\sin ^{2} x=\\left|\\sin ^{n} x\\right|$ and $\\cos ^{2} x=\\left|\\cos ^{n} x\\right|$, from which it follows that $\\sin x, \\cos x \\in\\{1,0,-1\\} \\Rightarrow x \\in \\pi \\mathbb{Z} \/ 2$. By inspection one obtains the set of solutions $\\{m \\pi \\mid m \\in \\mathbb{Z}\\}$ for even $n$ and $\\{2 m \\pi, 2 m \\pi-\\pi \/ 2 \\mid m \\in \\mathbb{Z}\\}$ for odd $n$. For $n=1$ we have $1=\\cos x-\\sin x=-\\sqrt{2} \\sin (x-\\pi \/ 4)$, which yields the set of solutions $$ \\{2 m \\pi, 2 m \\pi-\\pi \/ 2 \\mid m \\in \\mathbb{Z}\\} $$","problem_type":null,"tier":0} +{"year":"1961","problem_phase":"contest","problem":"4. (GDR) In the interior of $\\triangle P_{1} P_{2} P_{3}$ a point $P$ is given. Let $Q_{1}, Q_{2}$, and $Q_{3}$ respectively be the intersections of $P P_{1}, P P_{2}$, and $P P_{3}$ with the opposing edges of $\\triangle P_{1} P_{2} P_{3}$. Prove that among the ratios $P P_{1} \/ P Q_{1}, P P_{2} \/ P Q_{2}$, and $P P_{3} \/ P Q_{3}$ there exists at least one not larger than 2 and at least one not smaller than 2.","solution":"4. Let $x_{i}=P P_{i} \/ P Q_{i}$ for $i=1,2,3$. For all $i$ we have $$ \\frac{1}{x_{i}+1}=\\frac{P Q_{i}}{P_{i} Q_{i}}=\\frac{S_{P P_{j} P_{k}}}{S_{P_{1} P_{2} P_{3}}} $$ where the indices $j$ and $k$ are distinct and different from $i$. Hence we have $$ \\begin{aligned} f\\left(x_{1}, x_{2}, x_{3}\\right) & =\\frac{1}{x_{1}+1}+\\frac{1}{x_{2}+1}+\\frac{1}{x_{3}+1} \\\\ & =\\frac{S\\left(P P_{2} P_{3}\\right)+S\\left(P P_{1} P_{3}\\right)+S\\left(P P_{2} P_{3}\\right)}{S\\left(P_{1} P_{2} P_{3}\\right)}=1 \\end{aligned} $$ It follows that $1 \/\\left(x_{i}+1\\right) \\geq 1 \/ 3$ for some $i$ and $1 \/\\left(x_{j}+1\\right) \\leq 1 \/ 3$ for some $j$. Consequently, $x_{i} \\leq 2$ and $x_{j} \\geq 2$.","problem_type":null,"tier":0} +{"year":"1961","problem_phase":"contest","problem":"5. (CZS) Construct a triangle $A B C$ if the following elements are given: $A C=b, A B=c$, and $\\measuredangle A M B=\\omega\\left(\\omega<90^{\\circ}\\right)$, where $M$ is the midpoint of $B C$. Prove that the construction has a solution if and only if $$ b \\tan \\frac{\\omega}{2} \\leq c\\frac{1}{2} . $$","solution":"2. We note that $f(x)=\\sqrt{3-x}-\\sqrt{x+1}$ is well-defined only for $-1 \\leq x \\leq 3$ and is decreasing (and obviously continuous) on this interval. We also note that $f(-1)=2>1 \/ 2$ and $f(1-\\sqrt{31} \/ 8)=\\sqrt{(1 \/ 4+\\sqrt{31} \/ 4)^{2}}-$ $\\sqrt{(1 \/ 4-\\sqrt{31} \/ 4)^{2}}=1 \/ 2$. Hence the inequality is satisfied for $-1 \\leq x<$ $1-\\sqrt{31} \/ 8$.","problem_type":null,"tier":0} +{"year":"1962","problem_phase":"contest","problem":"3. (CZS) A cube $A B C D A^{\\prime} B^{\\prime} C^{\\prime} D^{\\prime}$ is given. The point $X$ is moving at a constant speed along the square $A B C D$ in the direction from $A$ to $B$. The point $Y$ is moving with the same constant speed along the square $B C C^{\\prime} B^{\\prime}$ in the direction from $B^{\\prime}$ to $C^{\\prime}$. Initially, $X$ and $Y$ start out from $A$ and $B^{\\prime}$ respectively. Find the locus of all the midpoints of $X Y$. Second Day","solution":"3. By inspecting the four different stages of this periodic motion we easily obtain that the locus of the midpoints of $X Y$ is the edges of $M N C Q$, where $M, N$, and $Q$ are the centers of $A B B^{\\prime} A^{\\prime}, B C C^{\\prime} B^{\\prime}$, and $A B C D$, respectively.","problem_type":null,"tier":0} +{"year":"1962","problem_phase":"contest","problem":"4. (ROM) Solve the equation $$ \\cos ^{2} x+\\cos ^{2} 2 x+\\cos ^{2} 3 x=1 . $$","solution":"4. Since $\\cos 2 x=1+\\cos ^{2} x$ and $\\cos \\alpha+\\cos \\beta=2 \\cos \\left(\\frac{\\alpha+\\beta}{2}\\right) \\cos \\left(\\frac{\\alpha-\\beta}{2}\\right)$, we have $\\cos ^{2} x+\\cos ^{2} 2 x+\\cos ^{2} 3 x=1 \\Leftrightarrow \\cos 2 x+\\cos 4 x+2 \\cos ^{2} 3 x=$ $2 \\cos 3 x(\\cos x+\\cos 3 x)=0 \\Leftrightarrow 4 \\cos 3 x \\cos 2 x \\cos x=0$. Hence the solutions are $x \\in\\{\\pi \/ 2+m \\pi, \\pi \/ 4+m \\pi \/ 2, \\pi \/ 6+m \\pi \/ 3 \\mid m \\in \\mathbb{Z}\\}$.","problem_type":null,"tier":0} +{"year":"1962","problem_phase":"contest","problem":"5. (BUL) On the circle $k$ three points $A, B$, and $C$ are given. Construct the fourth point on the circle $D$ such that one can inscribe a circle in $A B C D$.","solution":"5. Analysis. Let $A B C D$ be the desired quadrilateral. Let us assume w.l.o.g. that $A B>B C$ (for $A B=B C$ the construction is trivial). For a tangent quadrilateral we have $A D-D C=A B-B C$. Let $X$ be a point on $A D$ such that $D X=D C$. We then have $A X=A B-B C$ and $\\measuredangle A X C=$ $\\measuredangle A D C+\\measuredangle C D X=180^{\\circ}-\\angle A B C \/ 2$. Constructing $X$ and hence $D$ is now obvious.","problem_type":null,"tier":0} +{"year":"1962","problem_phase":"contest","problem":"6. (GDR) Let $A B C$ be an isosceles triangle with circumradius $r$ and inradius $\\rho$. Prove that the distance $d$ between the circumcenter and incenter is given by $$ d=\\sqrt{r(r-2 \\rho)} . $$","solution":"6. This problem is a special case, when the triangle is isosceles, of Euler's formula, which holds for all triangles.","problem_type":null,"tier":0} +{"year":"1962","problem_phase":"contest","problem":"7. (USS) Prove that a tetrahedron $S A B C$ has five different spheres that touch all six lines determined by its edges if and only if it is regular.","solution":"7. The spheres are arranged in a similar manner as in the planar case where we have one incircle and three excircles. Here we have one \"insphere\" and four \"exspheres\" corresponding to each of the four sides. Each vertex of the tetrahedron effectively has three tangent lines drawn from it to each of the five spheres. Repeatedly using the equality of the three tangent segments from a vertex (in the same vein as for tangent planar quadrilaterals) we obtain $S A+B C=S B+C A=S C+A B$ from the insphere. From the exsphere opposite of $S$ we obtain $S A-B C=S B-C A=S C-A B$, hence $S A=S B=S C$ and $A B=B C=C A$. By symmetry, we also have $A B=A C=A S$. Hence indeed, all the edges of the tetrahedron are equal in length and thus we have shown that the tetrahedron is regular.","problem_type":null,"tier":0} +{"year":"1963","problem_phase":"contest","problem":"1. (CZS) Determine all real solutions of the equation $\\sqrt{x^{2}-p}+2 \\sqrt{x^{2}-1}=$ $x$, where $p$ is a real number.","solution":"1. Obviously, $x \\geq 0$; hence squaring the given equation yields an equivalent equation $5 x^{2}-p-4+4 \\sqrt{\\left(x^{2}-1\\right)\\left(x^{2}-p\\right)}=x^{2}$, i.e., $4 \\sqrt{\\left(x^{2}-1\\right)\\left(x^{2}-p\\right)}=$ $(p+4)-4 x^{2}$. If $4 x^{2} \\leq(p+4)$, we may square the equation once again to get $-16(p+1) x^{2}+16 p=-8(p+4) x^{2}+(p+4)^{2}$, which is equivalent to $x^{2}=(4-p)^{2} \/[4(4-2 p)]$, i.e., $x=(4-p) \/(2 \\sqrt{4-2 p})$. For this to be a solution we must have $p \\leq 2$ and $(4-p)^{2} \/(4-2 p)=4 x^{2} \\leq(p+4)$. Hence $4 \/ 3 \\leq p \\leq 2$. Otherwise there is no solution.","problem_type":null,"tier":0} +{"year":"1963","problem_phase":"contest","problem":"2. (USS) Find the locus of points in space that are vertices of right angles of which one ray passes through a given point and the other intersects a given segment.","solution":"2. Let $A$ be the given point, $B C$ the given segment, and $\\mathcal{B}_{1}, \\mathcal{B}_{2}$ the closed balls with the diameters $A B$ and $A C$ respectively. Consider one right angle $\\angle A O K$ with $K \\in[B C]$. If $B^{\\prime}, C^{\\prime}$ are the feet of the perpendiculars from $B, C$ to $A O$ respectively, then $O$ lies on the segment $B^{\\prime} C^{\\prime}$, which implies that it lies on exactly one of the segments $A B^{\\prime}, A C^{\\prime}$. Hence $O$ belongs to exactly one of the balls $\\mathcal{B}_{1}, \\mathcal{B}_{2}$; i.e., $O \\in \\mathcal{B}_{1} \\Delta \\mathcal{B}_{2}$. This is obviously the required locus.","problem_type":null,"tier":0} +{"year":"1963","problem_phase":"contest","problem":"3. (HUN) Prove that if all the angles of a convex $n$-gon are equal and the lengths of consecutive edges $a_{1}, \\ldots, a_{n}$ satisfy $a_{1} \\geq a_{2} \\geq \\cdots \\geq a_{n}$, then $a_{1}=a_{2}=\\cdots=a_{n}$. Second Day","solution":"3. Let $\\overrightarrow{O A_{1}}, \\overrightarrow{O A_{2}}, \\ldots, \\overrightarrow{O A_{n}}$ be the vectors corresponding respectively to the edges $a_{1}, a_{2}, \\ldots, a_{n}$ of the polygon. By the conditions of the problem, these vectors satisfy $\\overrightarrow{O A_{1}}+\\cdots+\\overrightarrow{O A_{n}}=\\overrightarrow{0}, \\angle A_{1} O A_{2}=\\angle A_{2} O A_{3}=\\cdots=$ $\\angle A_{n} O A_{1}=2 \\pi \/ n$ and $O A_{1} \\geq O A_{2} \\geq \\cdots \\geq O A_{n}$. Our task is to prove that $O A_{1}=\\cdots=O A_{n}$. Let $l$ be the line through $O$ perpendicular to $O A_{n}$, and $B_{1}, \\ldots, B_{n-1}$ the projections of $A_{1}, \\ldots, A_{n-1}$ onto $l$ respectively. By the assumptions, the sum of the $\\overrightarrow{O B_{i}}$ 's is $\\overrightarrow{0}$. On the other hand, since $O B_{i} \\leq O B_{n-i}$ for all $i \\leq n \/ 2$, all the sums $\\overrightarrow{O B_{i}}+\\overrightarrow{O B_{n-i}}$ lie on the same side of the point $O$. Hence all these sums must be equal to $\\overrightarrow{0}$. Consequently, $O A_{i}=O A_{n-i}$, from which the result immediately follows.","problem_type":null,"tier":0} +{"year":"1963","problem_phase":"contest","problem":"4. (USS) Find all solutions $x_{1}, \\ldots, x_{5}$ to the system of equations $$ \\left\\{\\begin{array}{l} x_{5}+x_{2}=y x_{1}, \\\\ x_{1}+x_{3}=y x_{2}, \\\\ x_{2}+x_{4}=y x_{3}, \\\\ x_{3}+x_{5}=y x_{4}, \\\\ x_{4}+x_{1}=y x_{5}, \\end{array}\\right. $$ where $y$ is a real parameter.","solution":"4. Summing up all the equations yields $2\\left(x_{1}+x_{2}+x_{3}+x_{4}+x_{5}\\right)=y\\left(x_{1}+\\right.$ $x_{2}+x_{3}+x_{4}+x_{5}$ ). If $y=2$, then the given equations imply $x_{1}-x_{2}=$ $x_{2}-x_{3}=\\cdots=x_{5}-x_{1}$; hence $x_{1}=x_{2}=\\cdots=x_{5}$, which is clearly a solution. If $y \\neq 2$, then $x_{1}+\\cdots+x_{5}=0$, and summing the first three equalities gives $x_{2}=y\\left(x_{1}+x_{2}+x_{3}\\right)$. Using that $x_{1}+x_{3}=y x_{2}$ we obtain $x_{2}=\\left(y^{2}+y\\right) x_{2}$, i.e., $\\left(y^{2}+y-1\\right) x_{2}=0$. If $y^{2}+y-1 \\neq 0$, then $x_{2}=0$, and similarly $x_{1}=\\cdots=x_{5}=0$. If $y^{2}+y-1=0$, it is easy to prove that the last two equations are the consequence of the first three. Thus choosing any values for $x_{1}$ and $x_{5}$ will give exactly one solution for $x_{2}, x_{3}, x_{4}$.","problem_type":null,"tier":0} +{"year":"1963","problem_phase":"contest","problem":"5. (GDR) Prove that $\\cos \\frac{\\pi}{7}-\\cos \\frac{2 \\pi}{7}+\\cos \\frac{3 \\pi}{7}=\\frac{1}{2}$.","solution":"5. The LHS of the desired identity equals $S=\\cos (\\pi \/ 7)+\\cos (3 \\pi \/ 7)+$ $\\cos (5 \\pi \/ 7)$. Now $$ S \\sin \\frac{\\pi}{7}=\\frac{\\sin \\frac{2 \\pi}{7}}{2}+\\frac{\\sin \\frac{4 \\pi}{7}-\\sin \\frac{2 \\pi}{7}}{2}+\\frac{\\sin \\frac{6 \\pi}{7}-\\sin \\frac{4 \\pi}{7}}{2}=\\frac{\\sin \\frac{6 \\pi}{7}}{2} \\Rightarrow S=\\frac{1}{2} . $$","problem_type":null,"tier":0} +{"year":"1963","problem_phase":"contest","problem":"6. (HUN) Five students $A, B, C, D$, and $E$ have taken part in a certain competition. Before the competition, two persons $X$ and $Y$ tried to guess the rankings. $X$ thought that the ranking would be $A, B, C, D, E$; and $Y$ thought that the ranking would be $D, A, E, C, B$. At the end, it was revealed that $X$ didn't guess correctly any rankings of the participants, and moreover, didn't guess any of the orderings of pairs of consecutive participants. On the other hand, $Y$ guessed the correct rankings of two participants and the correct ordering of two pairs of consecutive participants. Determine the rankings of the competition.","solution":"6. The result is $E D A C B$.","problem_type":null,"tier":0} +{"year":"1964","problem_phase":"contest","problem":"1. (CZS) (a) Find all natural numbers $n$ such that the number $2^{n}-1$ is divisible by 7 . (b) Prove that for all natural numbers $n$ the number $2^{n}+1$ is not divisible by 7 .","solution":"1. Let $n=3 k+r$, where $0 \\leq r<2$. Then $2^{n}=2^{3 k+r}=8^{k} \\cdot 2^{r} \\equiv 2^{r}(\\bmod 7)$. Thus the remainder of $2^{n}$ modulo 7 is $1,2,4$ if $n \\equiv 0,1,2(\\bmod 3)$. Hence $2^{n}-1$ is divisible by 7 if and only if $3 \\mid n$, while $2^{n}+1$ is never divisible by 7 .","problem_type":null,"tier":0} +{"year":"1964","problem_phase":"contest","problem":"2. (HUN) Denote by $a, b, c$ the lengths of the sides of a triangle. Prove that $$ a^{2}(b+c-a)+b^{2}(c+a-b)+c^{2}(a+b-c) \\leq 3 a b c $$","solution":"2. By substituting $a=x+y, b=y+z$, and $c=z+x(x, y, z>0)$ the given inequality becomes $$ 6 x y z \\leq x^{2} y+x y^{2}+y^{2} z+y z^{2}+z^{2} x+z x^{2}, $$ which follows immediately by the AM-GM inequality applied to $x^{2} y, x y^{2}$, $x^{2} z, x z^{2}, y^{2} z, y z^{2}$.","problem_type":null,"tier":0} +{"year":"1964","problem_phase":"contest","problem":"3. (YUG) The incircle is inscribed in a triangle $A B C$ with sides $a, b, c$. Three tangents to the incircle are drawn, each of which is parallel to one side of the triangle $A B C$. These tangents form three smaller triangles (internal to $\\triangle A B C$ ) with the sides of $\\triangle A B C$. In each of these triangles an incircle is inscribed. Determine the sum of areas of all four incircles. Second Day","solution":"3. Let $r$ be the radius of the incircle of $\\triangle A B C, r_{a}, r_{b}, r_{c}$ the radii of the smaller circles corresponding to $A, B, C$, and $h_{a}, h_{b}, h_{c}$ the altitudes from $A, B, C$ respectively. The coefficient of similarity between the smaller triangle at $A$ and the triangle $A B C$ is $1-2 r \/ h_{a}$, from which we easily obtain $r_{a}=\\left(h_{a}-2 r\\right) r \/ h_{a}=(s-a) r \/ s$. Similarly, $r_{b}=(s-b) r \/ s$ and $r_{c}=(s-c) r \/ s$. Now a straightforward computation gives that the sum of areas of the four circles is given by $$ \\Sigma=\\frac{(b+c-a)(c+a-b)(a+b-c)\\left(a^{2}+b^{2}+c^{2}\\right) \\pi}{(a+b+c)^{3}} $$","problem_type":null,"tier":0} +{"year":"1964","problem_phase":"contest","problem":"4. (HUN) Each of 17 students talked with every other student. They all talked about three different topics. Each pair of students talked about one topic. Prove that there are three students that talked about the same topic among themselves.","solution":"4. Let us call the topics $T_{1}, T_{2}, T_{3}$. Consider an arbitrary student $A$. By the pigeonhole principle there is a topic, say $T_{3}$, he discussed with at least 6 other students. If two of these 6 students discussed $T_{3}$, then we are done. Suppose now that the 6 students discussed only $T_{1}$ and $T_{2}$ and choose one of them, say $B$. By the pigeonhole principle he discussed one of the topics, say $T_{2}$, with three of these students. If two of these three students also discussed $T_{2}$, then we are done. Otherwise, all the three students discussed only $T_{1}$, which completes the task.","problem_type":null,"tier":0} +{"year":"1964","problem_phase":"contest","problem":"5. (ROM) Five points are given in the plane. Among the lines that connect these five points, no two coincide and no two are parallel or perpendicular. Through each point we construct an altitude to each of the other lines. What is the maximal number of intersection points of these altitudes (excluding the initial five points)?","solution":"5. Let us first compute the number of intersection points of the perpendiculars passing through two distinct points $B$ and $C$. The perpendiculars from $B$ to the lines through $C$ other than $B C$ meet all perpendiculars from $C$, which counts to $3 \\cdot 6=18$ intersection points. Each perpendicular from $B$ to the 3 lines not containing $C$ can intersect at most 5 of the perpendiculars passing through $C$, which counts to another $3 \\cdot 5=15$ intersection points. Thus there are $18+15=33$ intersection points corresponding to $B, C$. It follows that the required total number is at most $10 \\cdot 33=330$. But some of these points, namely the orthocenters of the triangles with vertices at the given points, are counted thrice. There are 10 such points. Hence the maximal number of intersection points is $330-2 \\cdot 10=310$. Remark. The jury considered only the combinatorial part of the problem and didn't require an example in which 310 points appear. However, it is \"easily\" verified that, for instance, the set of points $A(1,1), B(e, \\pi)$, $C\\left(e^{2}, \\pi^{2}\\right), D\\left(e^{3}, \\pi^{3}\\right), E\\left(e^{4}, \\pi^{4}\\right)$ works.","problem_type":null,"tier":0} +{"year":"1964","problem_phase":"contest","problem":"6. (POL) Given a tetrahedron $A B C D$, let $D_{1}$ be the centroid of the triangle $A B C$ and let $A_{1}, B_{1}, C_{1}$ be the intersection points of the lines parallel to $D D_{1}$ and passing through the points $A, B, C$ with the opposite faces of the tetrahedron. Prove that the volume of the tetrahedron $A B C D$ is onethird the volume of the tetrahedron $A_{1} B_{1} C_{1} D_{1}$. Does the result remain true if the point $D_{1}$ is replaced with any point inside the triangle $A B C$ ?","solution":"6. We shall prove that the statement is valid in the general case, for an arbitrary point $D_{1}$ inside $\\triangle A B C$. Since $D_{1}$ belongs to the plane $A B C$, there are real numbers $a, b, c$ such that $(a+b+c) \\overrightarrow{D D_{1}}=a \\overrightarrow{D A}+b \\overrightarrow{D B}+c \\overrightarrow{D C}$. Since $A A_{1} \\| D D_{1}$, it holds that $\\overrightarrow{A A_{1}}=k \\overrightarrow{D D_{1}}$ for some $k \\in \\mathbb{R}$. Now it is easy to get $\\overrightarrow{D A_{1}}=-(b \\overrightarrow{D B}+c \\overrightarrow{D C}) \/ a, \\overrightarrow{D B_{1}}=-(a \\overrightarrow{D A}+c \\overrightarrow{D C}) \/ b$, and $\\overrightarrow{D C_{1}}=-(a \\overrightarrow{D A}+b \\overrightarrow{D B}) \/ c$. This implies $$ \\begin{aligned} & \\overrightarrow{D_{1} A_{1}}=-\\frac{a^{2} \\overrightarrow{D A}+b(a+2 b+c) \\overrightarrow{D B}+c(a+b+2 c) \\overrightarrow{D C}}{a(a+b+c)} \\\\ & \\overrightarrow{D_{1} B_{1}}=-\\frac{a(2 a+b+c) \\overrightarrow{D A}+b^{2} \\overrightarrow{D B}+c(a+b+2 c) \\overrightarrow{D C}}{b(a+b+c)}, \\text { and } \\\\ & \\overrightarrow{D_{1} C_{1}}=-\\frac{a(2 a+b+c) \\overrightarrow{D A}+b(a+2 b+c) \\overrightarrow{D B}+c^{2} \\overrightarrow{D C}}{c(a+b+c)} \\end{aligned} $$ By using $$ 6 V_{D_{1} A_{1} B_{1} C_{1}}=\\left|\\left[\\overrightarrow{D_{1} A_{1}}, \\overrightarrow{D_{1} B_{1}}, \\overrightarrow{D_{1} C_{1}}\\right]\\right| \\text { and } 6 V_{D A B C}=|[\\overrightarrow{D A}, \\overrightarrow{D B}, \\overrightarrow{D C}]| $$ we get $$ V_{D_{1} A_{1} B_{1} C_{1}}=\\frac{\\left\\|\\begin{array}{ccc} a^{2} & b(a+2 b+c) & c(a+b+2 c) \\\\ a(2 a+b+c) & b^{2} & c(a+b+2 c) \\\\ a(2 a+b+c) b(a+2 b+c) & c^{2} \\end{array}\\right\\|}{6 a b c(a+b+c)^{3}}=3 V_{D A B C} $$","problem_type":null,"tier":0} +{"year":"1965","problem_phase":"contest","problem":"1. (YUG) Find all real numbers $x \\in[0,2 \\pi]$ such that $$ 2 \\cos x \\leq|\\sqrt{1+\\sin 2 x}-\\sqrt{1-\\sin 2 x}| \\leq \\sqrt{2} $$","solution":"1. Let us set $S=|\\sqrt{1+\\sin 2 x}-\\sqrt{1-\\sin 2 x}|$. Observe that $S^{2}=2-$ $2 \\sqrt{1-\\sin ^{2} 2 x}=2-2|\\cos 2 x| \\leq 2$, implying $S \\leq \\sqrt{2}$. Thus the righthand inequality holds for all $x$. It remains to investigate the left-hand inequality. If $\\pi \/ 2 \\leq x \\leq 3 \\pi \/ 2$, then $\\cos x \\leq 0$ and the inequality trivially holds. Assume now that $\\cos x>$ 0 . Then the inequality is equivalent to $2+2 \\cos 2 x=4 \\cos ^{2} x \\leq S^{2}=$ $2-2|\\cos 2 x|$, which is equivalent to $\\cos 2 x \\leq 0$, i.e., to $x \\in[\\pi \/ 4, \\pi \/ 2] \\cup$ $[3 \\pi \/ 2,7 \\pi \/ 4]$. Hence the solution set is $\\pi \/ 4 \\leq x \\leq 7 \\pi \/ 4$.","problem_type":null,"tier":0} +{"year":"1965","problem_phase":"contest","problem":"2. (POL) Consider the system of equations $$ \\left\\{\\begin{array}{l} a_{11} x_{1}+a_{12} x_{2}+a_{13} x_{3}=0 \\\\ a_{21} x_{1}+a_{22} x_{2}+a_{23} x_{3}=0 \\\\ a_{31} x_{1}+a_{32} x_{2}+a_{33} x_{3}=0 \\end{array}\\right. $$ whose coefficients satisfy the following conditions: (a) $a_{11}, a_{22}, a_{33}$ are positive real numbers; (b) all other coefficients are negative; (c) in each of the equations the sum of the coefficients is positive. Prove that $x_{1}=x_{2}=x_{3}=0$ is the only solution to the system.","solution":"2. Suppose that $\\left(x_{1}, x_{2}, x_{3}\\right)$ is a solution. We may assume w.l.o.g. that $\\left|x_{1}\\right| \\geq$ $\\left|x_{2}\\right| \\geq\\left|x_{3}\\right|$. Suppose that $\\left|x_{1}\\right|>0$. From the first equation we obtain that $$ 0=\\left|x_{1}\\right| \\cdot\\left|a_{11}+a_{12} \\frac{x_{2}}{x_{1}}+a_{13} \\frac{x_{3}}{x_{1}}\\right| \\geq\\left|x_{1}\\right| \\cdot\\left(a_{11}-\\left|a_{12}\\right|-\\left|a_{13}\\right|\\right)>0 $$ which is a contradiction. Hence $\\left|x_{1}\\right|=0$ and consequently $x_{1}=x_{2}=x_{3}=$ 0.","problem_type":null,"tier":0} +{"year":"1965","problem_phase":"contest","problem":"3. (CZS) A tetrahedron $A B C D$ is given. The lengths of the edges $A B$ and $C D$ are $a$ and $b$, respectively, the distance between the lines $A B$ and $C D$ is $d$, and the angle between them is equal to $\\omega$. The tetrahedron is divided into two parts by the plane $\\pi$ parallel to the lines $A B$ and $C D$. Calculate the ratio of the volumes of the parts if the ratio between the distances of the plane $\\pi$ from $A B$ and $C D$ is equal to $k$. Second Day","solution":"3. Let $d$ denote the distance between the lines $A B$ and $C D$. Being parallel to $A B$ and $C D$, the plane $\\pi$ intersects the faces of the tetrahedron in a parallelogram $E F G H$. Let $X \\in A B$ be a points such that $H X \\| D B$. Clearly $V_{A E H B F G}=V_{A X E H}+$ $V_{X E H B F G}$. Let $M N$ be the common perpendicular to lines $A B$ and $C D(M \\in A B, N \\in C D)$ and let $M N, B N$ meet the plane $\\pi$ at $Q$ and $R$ respectively. Then it holds that $B R \/ R N=M Q \/ Q N=k$ and consequently $A X \/ X B=A E \/ E C=$ $A H \/ H D=B F \/ F C=B G \/ G D=$ $k$. Now we have $V_{A X E H} \/ V_{A B C D}=$ ![](https:\/\/cdn.mathpix.com\/cropped\/2024_11_18_8e985d6b9c83aa3e9d0eg-352.jpg?height=417&width=535&top_left_y=1138&top_left_x=819) $k^{3} \/(k+1)^{3}$ \u3002 Furthermore, if $h=3 V_{A B C D} \/ S_{A B C}$ is the height of $A B C D$ from $D$, then $$ \\begin{aligned} V_{X E H B F G} & =\\frac{1}{2} S_{X B F E} \\frac{k}{k+1} h \\text { and } \\\\ S_{X B F E} & =S_{A B C}-S_{A X E}-S_{E F C}=\\frac{(k+1)^{2}-1-k^{2}}{(k+1)^{2}}=\\frac{2 k}{(1+k)^{2}} \\end{aligned} $$ These relations give us $V_{X E H B F G} \/ V_{A B C D}=3 k^{2} \/(1+k)^{3}$. Finally, $$ \\frac{V_{A E H B F G}}{V_{A B C D}}=\\frac{k^{3}+3 k^{2}}{(k+1)^{3}} $$ Similarly, $V_{C E F D H G} \/ V_{A B C D}=(3 k+1) \/(k+1)^{3}$, and hence the required ratio is $\\left(k^{3}+3 k^{2}\\right) \/(3 k+1)$.","problem_type":null,"tier":0} +{"year":"1965","problem_phase":"contest","problem":"4. (USS) Find four real numbers $x_{1}, x_{2}, x_{3}, x_{4}$ such that the sum of any of the numbers and the product of other three is equal to 2 .","solution":"4. It is easy to see that all $x_{i}$ are nonzero. Let $x_{1} x_{2} x_{3} x_{4}=p$. The given system of equations can be rewritten as $x_{i}+p \/ x_{i}=2, i=1,2,3,4$. The equation $x+p \/ x=2$ has at most two real solutions, say $y$ and $z$. Then each $x_{i}$ is equal either to $y$ or to $z$. There are three cases: (i) $x_{1}=x_{2}=x_{3}=x_{4}=y$. Then $y+y^{3}=2$ and hence $y=1$. (ii) $x_{1}=x_{2}=x_{3}=y, x_{4}=z$. Then $z+y^{3}=y+y^{2} z=2$. It is easy to obtain that the only possibilities for $(y, z)$ are $(-1,3)$ and $(1,1)$. (iii) $x_{1}=x_{2}=y, x_{3}=x_{4}$. In this case the only possibility is $y=z=1$. Hence the solutions for $\\left(x_{1}, x_{2}, x_{3}, x_{4}\\right)$ are $(1,1,1,1),(-1,-1,-1,3)$, and the cyclic permutations.","problem_type":null,"tier":0} +{"year":"1965","problem_phase":"contest","problem":"5. (ROM) Given a triangle $O A B$ such that $\\angle A O B=\\alpha<90^{\\circ}$, let $M$ be an arbitrary point of the triangle different from $O$. Denote by $P$ and $Q$ the feet of the perpendiculars from $M$ to $O A$ and $O B$, respectively. Let $H$ be the orthocenter of the triangle $O P Q$. Find the locus of points $H$ when: (a) $M$ belongs to the segment $A B$; (b) $M$ belongs to the interior of $\\triangle O A B$.","solution":"5. (a) Let $A^{\\prime}$ and $B^{\\prime}$ denote the feet of the perpendiculars from $A$ and $B$ to $O B$ and $O A$ respectively. We claim that $H \\in A^{\\prime} B^{\\prime}$. Indeed, since $M P H Q$ is a parallelogram, we have $B^{\\prime} P \/ B^{\\prime} A=B M \/ B A=$ $M Q \/ A A^{\\prime}=P H \/ A A^{\\prime}$, which implies by Thales's theorem that $H \\in$ $A^{\\prime} B^{\\prime}$. It is easy to see that the locus of $H$ is the whole segment $A^{\\prime} B^{\\prime}$. (b) In this case the locus of points $H$ is obviously the interior of the triangle $O A^{\\prime} B^{\\prime}$.","problem_type":null,"tier":0} +{"year":"1965","problem_phase":"contest","problem":"6. (POL) We are given $n \\geq 3$ points in the plane. Let $d$ be the maximal distance between two of the given points. Prove that the number of pairs of points whose distance is equal to $d$ is less than or equal to $n$.","solution":"6. We recall the simple statement that every two diameters of a set must have a common point. Consider any point $B$ that is an endpoint of $k \\geq 2$ diameters $B C_{1}, B C_{2}$, $\\ldots, B C_{k}$. We may assume w.l.o.g. that all the points $C_{1}, \\ldots, C_{k}$ lie on the $\\operatorname{arc} C_{1} C_{k}$, whose center is $B$ and measure does not exceed $60^{\\circ}$. We observe that for $1$ $S\\left(K L C_{1}\\right)>S\\left(K B_{1} C_{1}\\right)=S\\left(A_{1} B_{1} C_{1}\\right)=S \/ 4$. Hence, by the pigeonhole principle one of the remaining three triangles $\\triangle M A L, \\triangle K B M$, and $\\triangle L C K$ must have an area less than or equal to $S \/ 4$. This completes the proof.","problem_type":null,"tier":0} +{"year":"1967","problem_phase":"longlisted","problem":"1. (BUL 1) Prove that all numbers in the sequence $$ \\frac{107811}{3}, \\frac{110778111}{3}, \\frac{111077781111}{3}, \\ldots $$ are perfect cubes.","solution":"1. Let us denote the $n$th term of the given sequence by $a_{n}$. Then $$ \\begin{aligned} a_{n} & =\\frac{1}{3}\\left(\\frac{10^{3 n+3}-10^{2 n+3}}{9}+7 \\frac{10^{2 n+2}-10^{n+1}}{9}+\\frac{10^{n+2}-1}{9}\\right) \\\\ & =\\frac{1}{27}\\left(10^{3 n+3}-3 \\cdot 10^{2 n+2}+3 \\cdot 10^{n+1}-1\\right)=\\left(\\frac{10^{n+1}-1}{3}\\right)^{3} . \\end{aligned} $$","problem_type":null,"tier":0} +{"year":"1967","problem_phase":"longlisted","problem":"10. (CZS 4) The square $A B C D$ is to be decomposed into $n$ triangles (nonoverlapping) all of whose angles are acute. Find the smallest integer $n$ for which there exists a solution to this problem and construct at least one decomposition for this $n$. Answer whether it is possible to ask additionally that (at least) one of these triangles has a perimeter less than an arbitrarily given positive number.","solution":"10. Let $n$ be the number of triangles and let $b$ and $i$ be the numbers of vertices on the boundary and in the interior of the square, respectively. Since all the triangles are acute, each of the vertices of the square belongs to at least two triangles. Additionally, every vertex on the boundary belongs to at least three, and every vertex in the interior belongs to at least five triangles. Therefore $$ 3 n \\geq 8+3 b+5 i $$ Moreover, the sum of angles at any vertex that lies in the interior, on the boundary, or at a vertex of the square is equal to $2 \\pi, \\pi, \\pi \/ 2$ respectively. The sum of all angles of the triangles equals $n \\pi$, which gives us $n \\pi=4 \\cdot \\pi \/ 2+b \\pi+2 i \\pi$, i.e., $n=$ $2+b+2 i$. This relation together with (1) easily yields that $i \\geq 2$. Since each of the vertices inside the square belongs to at least five triangles, and at most two contain both, it follows that $n \\geq 8$. ![](https:\/\/cdn.mathpix.com\/cropped\/2024_11_18_8e985d6b9c83aa3e9d0eg-357.jpg?height=525&width=489&top_left_y=1547&top_left_x=813) It is shown in the figure that the square can be decomposed into eight acute triangles. Obviously one of them can have an arbitrarily small perimeter.","problem_type":null,"tier":0} +{"year":"1967","problem_phase":"longlisted","problem":"11. (CZS 5) Let $n$ be a positive integer. Find the maximal number of noncongruent triangles whose side lengths are integers less than or equal to $n$.","solution":"11. We have to find the number $p_{n}$ of triples of positive integers $(a, b, c)$ satisfying $a \\leq b \\leq c \\leq n$ and $a+b>c$. Let us denote by $p_{n}(k)$ the number of such triples with $c=k, k=1,2, \\ldots, n$. For $k$ even, $p_{n}(k)=k+(k-2)+(k-4)+\\cdots+2=\\left(k^{2}+2 k\\right) \/ 4$, and for $k$ odd, $p_{n}(k)=\\left(k^{2}+2 k+1\\right) \/ 4$. Hence $p_{n}=p_{n}(1)+p_{n}(2)+\\cdots+p_{n}(n)= \\begin{cases}n(n+2)(2 n+5) \/ 24, & \\text { for } 2 \\mid n, \\\\ (n+1)(n+3)(2 n+1) \/ 24, & \\text { for } 2 \\nmid n .\\end{cases}$","problem_type":null,"tier":0} +{"year":"1967","problem_phase":"longlisted","problem":"12. (CZS 6) Given a segment $A B$ of the length 1, define the set $M$ of points in the following way: it contains the two points $A, B$, and also all points obtained from $A, B$ by iterating the following rule: $(*)$ for every pair of points $X, Y$ in $M$, the set $M$ also contains the point $Z$ of the segment $X Y$ for which $Y Z=3 X Z$. (a) Prove that the set $M$ consists of points $X$ from the segment $A B$ for which the distance from the point $A$ is either $$ A X=\\frac{3 k}{4^{n}} \\quad \\text { or } \\quad A X=\\frac{3 k-2}{4^{n}} $$ where $n, k$ are nonnegative integers. (b) Prove that the point $X_{0}$ for which $A X_{0}=1 \/ 2=X_{0} B$ does not belong to the set $M$.","solution":"12. Let us denote by $M_{n}$ the set of points of the segment $A B$ obtained from $A$ and $B$ by not more than $n$ iterations of $(*)$. It can be proved by induction that $$ M_{n}=\\left\\{X \\in A B \\left\\lvert\\, A X=\\frac{3 k}{4^{n}}\\right. \\text { or } \\frac{3 k-2}{4^{n}} \\text { for some } k \\in \\mathbb{N}\\right\\} $$ Thus (a) immediately follows from $M=\\bigcup M_{n}$. It also follows that if $a, b \\in \\mathbb{N}$ and $a \/ b \\in M$, then $3 \\mid a(b-a)$. Therefore $1 \/ 2 \\notin M$.","problem_type":null,"tier":0} +{"year":"1967","problem_phase":"longlisted","problem":"13. (GDR 1) Find whether among all quadrilaterals whose interiors lie inside a semicircle of radius $r$ there exists one (or more) with maximal area. If so, determine their shape and area.","solution":"13. The maximum area is $3 \\sqrt{3} r^{2} \/ 4$ (where $r$ is the radius of the semicircle) and is attained in the case of a trapezoid with two vertices at the endpoints of the diameter of the semicircle and the other two vertices dividing the semicircle into three equal arcs.","problem_type":null,"tier":0} +{"year":"1967","problem_phase":"longlisted","problem":"14. (GDR 2) Which fraction $p \/ q$, where $p, q$ are positive integers less than 100 , is closest to $\\sqrt{2}$ ? Find all digits after the decimal point in the decimal representation of this fraction that coincide with digits in the decimal representation of $\\sqrt{2}$ (without using any tables).","solution":"14. We have that $$ \\left|\\frac{p}{q}-\\sqrt{2}\\right|=\\frac{|p-q \\sqrt{2}|}{q}=\\frac{\\left|p^{2}-2 q^{2}\\right|}{q(p+q \\sqrt{2})} \\geq \\frac{1}{q(p+q \\sqrt{2})} $$ because $\\left|p^{2}-2 q^{2}\\right| \\geq 1$. The greatest solution to the equation $\\left|p^{2}-2 q^{2}\\right|=1$ with $p, q \\leq 100$ is $(p, q)=(99,70)$. It is easy to verify using (1) that $\\frac{99}{70}$ best approximates $\\sqrt{2}$ among the fractions $p \/ q$ with $p, q \\leq 100$. Second solution. By using some basic facts about Farey sequences one can find that $\\frac{41}{29}<\\sqrt{2}<\\frac{99}{70}$ and that $\\frac{41}{29}<\\frac{p}{q}<\\frac{99}{70}$ implies $p \\geq 41+99>100$ because $99 \\cdot 29-41 \\cdot 70=1$. Of the two fractions $41 \/ 29$ and $99 \/ 70$, the latter is closer to $\\sqrt{2}$.","problem_type":null,"tier":0} +{"year":"1967","problem_phase":"longlisted","problem":"15. (GDR 3) Suppose $\\tan \\alpha=p \/ q$, where $p$ and $q$ are integers and $q \\neq 0$. Prove that the number $\\tan \\beta$ for which $\\tan 2 \\beta=\\tan 3 \\alpha$ is rational only when $p^{2}+q^{2}$ is the square of an integer.","solution":"15. Given that $\\tan \\alpha \\in \\mathbb{Q}$, we have that $\\tan \\beta$ is rational if and only if $\\tan \\gamma$ is rational, where $\\gamma=\\beta-\\alpha$ and $2 \\gamma=\\alpha$. Putting $t=\\tan \\gamma$ we obtain $\\frac{p}{q}=\\tan 2 \\gamma=\\frac{2 t}{1-t^{2}}$, which leads to the quadratic equation $p t^{2}+2 q t-p=0$. This equation has rational solutions if and only if its discriminant $4\\left(p^{2}+q^{2}\\right)$ is a perfect square, and the result follows.","problem_type":null,"tier":0} +{"year":"1967","problem_phase":"longlisted","problem":"16. (GDR 4) Prove the following statement: If $r_{1}$ and $r_{2}$ are real numbers whose quotient is irrational, then any real number $x$ can be approximated arbitrarily well by numbers of the form $z_{k_{1}, k_{2}}=k_{1} r_{1}+k_{2} r_{2}, k_{1}, k_{2}$ integers; i.e., for every real number $x$ and every positive real number $p$ two integers $k_{1}$ and $k_{2}$ can be found such that $\\left|x-\\left(k_{1} r_{1}+k_{2} r_{2}\\right)\\right|1$, and the series terminates. Show also that $x$ can be expressed as the sum of reciprocals of different integers, each of which is greater than $10^{6}$.","solution":"18. In the first part, it is sufficient to show that each rational number of the form $m \/ n!, m, n \\in \\mathbb{N}$, can be written uniquely in the required form. We prove this by induction on $n$. The statement is trivial for $n=1$. Let us assume it holds for $n-1$, and let there be given a rational number $m \/ n$ !. Let us take $a_{n} \\in\\{0, \\ldots, n-1\\}$ such that $m-a_{n}=n m_{1}$ for some $m_{1} \\in \\mathbb{N}$. By the inductive hypothesis, there are unique $a_{1} \\in \\mathbb{N}_{0}, a_{i} \\in\\{0, \\ldots, i-1\\}(i=1, \\ldots, n-1)$ such that $m_{1} \/(n-1)!=\\sum_{i=1}^{n-1} a_{i} \/ i$ !, and then $$ \\frac{m}{n!}=\\frac{m_{1}}{(n-1)!}+\\frac{a_{n}}{n!}=\\sum_{i=1}^{n} \\frac{a_{i}}{i!} $$ as desired. On the other hand, if $m \/ n!=\\sum_{i=1}^{n} a_{i} \/ i$ !, multiplying by $n$ ! we see that $m-a_{n}$ must be a multiple of $n$, so the choice of $a_{n}$ was unique and therefore the representation itself. This completes the induction. In particular, since $a_{i} \\mid i!$ and $i!\/ a_{i}>(i-1)!\\geq(i-1)!\/ a_{i-1}$, we conclude that each rational $q, 00$ be a rational number. For any integer $m>10^{6}$, let $n>m$ be the greatest integer such that $y=$ $x-\\frac{1}{m}-\\frac{1}{m+1}-\\cdots-\\frac{1}{n}>0$. Then $y$ can be written as the sum of reciprocals of different positive integers, which all must be greater than $n$. The result follows immediately.","problem_type":null,"tier":0} +{"year":"1967","problem_phase":"longlisted","problem":"19. (GBR 6) The $n$ points $P_{1}, P_{2}, \\ldots, P_{n}$ are placed inside or on the boundary of a disk of radius 1 in such a way that the minimum distance $d_{n}$ between any two of these points has its largest possible value $D_{n}$. Calculate $D_{n}$ for $n=2$ to 7 and justify your answer.","solution":"19. Suppose $n \\leq 6$. Let us decompose the disk by its radii into $n$ congruent regions, so that one of the points $P_{j}$ lies on the boundaries of two of these regions. Then one of these regions contains two of the $n$ given points. Since the diameter of each of these regions is $2 \\sin \\frac{\\pi}{n}$, we have $d_{n} \\leq 2 \\sin \\frac{\\pi}{n}$. This value is attained if $P_{i}$ are the vertices of a regular $n$-gon inscribed in the boundary circle. Hence $D_{n}=2 \\sin \\frac{\\pi}{n}$. For $n=7$ we have $D_{7} \\leq D_{6}=1$. This value is attained if six of the seven points form a regular hexagon inscribed in the boundary circle and the seventh is at the center. Hence $D_{7}=1$.","problem_type":null,"tier":0} +{"year":"1967","problem_phase":"longlisted","problem":"2. (BUL 2) Prove that $\\frac{1}{3} n^{2}+\\frac{1}{2} n+\\frac{1}{6} \\geq(n!)^{2 \/ n}$ ( $n$ is a positive integer) and that equality is possible only in the case $n=1$.","solution":"2. $(n!)^{2 \/ n}=\\left((1 \\cdot 2 \\cdots n)^{1 \/ n}\\right)^{2} \\leq\\left(\\frac{1+2+\\cdots+n}{n}\\right)^{2}=\\left(\\frac{n+1}{2}\\right)^{2} \\leq \\frac{1}{3} n^{2}+\\frac{1}{2} n+\\frac{1}{6}$.","problem_type":null,"tier":0} +{"year":"1967","problem_phase":"longlisted","problem":"20. (HUN 1) In space, $n$ points $(n \\geq 3)$ are given. Every pair of points determines some distance. Suppose all distances are different. Connect every point with the nearest point. Prove that it is impossible to obtain a polygonal line in such a way. ${ }^{1}$","solution":"20. The statement so formulated is false. It would be true under the additional assumption that the polygonal line is closed. However, from the offered solution, which is not clear, it does not seem that the proposer had this in mind.","problem_type":null,"tier":0} +{"year":"1967","problem_phase":"longlisted","problem":"21. (HUN 2) Without using any tables, find the exact value of the product $$ P=\\cos \\frac{\\pi}{15} \\cos \\frac{2 \\pi}{15} \\cos \\frac{3 \\pi}{15} \\cos \\frac{4 \\pi}{15} \\cos \\frac{5 \\pi}{15} \\cos \\frac{6 \\pi}{15} \\cos \\frac{7 \\pi}{15} $$","solution":"21. Using the formula $$ \\cos x \\cos 2 x \\cos 4 x \\cdots \\cos 2^{n-1} x=\\frac{\\sin 2^{n} x}{2^{n} \\sin x} $$ which is shown by simple induction, we obtain $$ \\begin{gathered} \\cos \\frac{\\pi}{15} \\cos \\frac{2 \\pi}{15} \\cos \\frac{4 \\pi}{15} \\cos \\frac{7 \\pi}{15}=-\\cos \\frac{\\pi}{15} \\cos \\frac{2 \\pi}{15} \\cos \\frac{4 \\pi}{15} \\cos \\frac{8 \\pi}{15}=\\frac{1}{16} \\\\ \\cos \\frac{3 \\pi}{15} \\cos \\frac{6 \\pi}{15} \\end{gathered}=\\frac{1}{4}, \\quad \\cos \\frac{5 \\pi}{15}=\\frac{1}{2} . $$ Multiplying these equalities, we get that the required product $P$ equals 1\/128.","problem_type":null,"tier":0} +{"year":"1967","problem_phase":"longlisted","problem":"22. (HUN 3) The distance between the centers of the circles $k_{1}$ and $k_{2}$ with radii $r$ is equal to $r$. Points $A$ and $B$ are on the circle $k_{1}$, symmetric with respect to the line connecting the centers of the circles. Point $P$ is an arbitrary point on $k_{2}$. Prove that $$ P A^{2}+P B^{2} \\geq 2 r^{2} $$ When does equality hold?","solution":"22. Let $O_{1}$ and $O_{2}$ be the centers of circles $k_{1}$ and $k_{2}$ and let $C$ be the midpoint of the segment $A B$. Using the well-known relation for elements of a triangle, we obtain $$ P A^{2}+P B^{2}=2 P C^{2}+2 C A^{2} \\geq 2 O_{1} C^{2}+2 C A^{2}=2 O_{1} A^{2}=2 r^{2} $$ Equality holds if $P$ coincides with $O_{1}$ or if $A$ and $B$ coincide with $O_{2}$.","problem_type":null,"tier":0} +{"year":"1967","problem_phase":"longlisted","problem":"23. (HUN 4) Prove that for an arbitrary pair of vectors $f$ and $g$ in the plane, the inequality $$ a f^{2}+b f g+c g^{2} \\geq 0 $$ holds if and only if the following conditions are fulfilled: $a \\geq 0, c \\geq 0$, $4 a c \\geq b^{2}$.","solution":"23. Suppose that $a \\geq 0, c \\geq 0,4 a c \\geq b^{2}$. If $a=0$, then $b=0$, and the inequality reduces to the obvious $c g^{2} \\geq 0$. Also, if $a>0$, then $$ a f^{2}+b f g+c g^{2}=a\\left(f+\\frac{b}{2 a} g\\right)^{2}+\\frac{4 a c-b^{2}}{4 a} g^{2} \\geq 0 $$ Suppose now that $a f^{2}+b f g+c g^{2} \\geq 0$ holds for an arbitrary pair of vectors $f, g$. Substituting $f$ by $t g(t \\in \\mathbb{R})$ we get that $\\left(a t^{2}+b t+c\\right) g^{2} \\geq 0$ holds for any real number $t$. Therefore $a \\geq 0, c \\geq 0,4 a c \\geq b^{2}$.","problem_type":null,"tier":0} +{"year":"1967","problem_phase":"longlisted","problem":"24. (HUN 5) ${ }^{\\text {IMO6 }}$ Father has left to his children several identical gold coins. According to his will, the oldest child receives one coin and one-seventh of the remaining coins, the next child receives two coins and one-seventh of the remaining coins, the third child receives three coins and one-seventh of the remaining coins, and so on through the youngest child. If every child inherits an integer number of coins, find the number of children and the number of coins.","solution":"24. Let the $k$ th child receive $x_{k}$ coins. By the condition of the problem, the number of coins that remain after him was $6\\left(x_{k}-k\\right)$. This gives us a recurrence relation $$ x_{k+1}=k+1+\\frac{6\\left(x_{k}-k\\right)-k-1}{7}=\\frac{6}{7} x_{k}+\\frac{6}{7}, $$ which, together with the condition $x_{1}=1+(m-1) \/ 7$, yields $$ x_{k}=\\frac{6^{k-1}}{7^{k}}(m-36)+6 \\text { for } 1 \\leq k \\leq n . $$ Since we are given $x_{n}=n$, we obtain $6^{n-1}(m-36)=7^{n}(n-6)$. It follows that $6^{n-1} \\mid n-6$, which is possible only for $n=6$. Hence, $n=6$ and $m=36$.","problem_type":null,"tier":0} +{"year":"1967","problem_phase":"longlisted","problem":"25. (HUN 6) Three disks of diameter $d$ are touching a sphere at their centers. Moreover, each disk touches the other two disks. How do we choose the radius $R$ of the sphere so that the axis of the whole figure makes an angle [^0]of $60^{\\circ}$ with the line connecting the center of the sphere with the point on the disks that is at the largest distance from the axis? (The axis of the figure is the line having the property that rotation of the figure through $120^{\\circ}$ about that line brings the figure to its initial position. The disks are all on one side of the plane, pass through the center of the sphere, and are orthogonal to the axes.)","solution":"25. The answer is $R=(4+\\sqrt{3}) d \/ 6$.","problem_type":null,"tier":0} +{"year":"1967","problem_phase":"longlisted","problem":"26. (ITA 1) Let $A B C D$ be a regular tetrahedron. To an arbitrary point $M$ on one edge, say $C D$, corresponds the point $P=P(M)$, which is the intersection of two lines $A H$ and $B K$, drawn from $A$ orthogonally to $B M$ and from $B$ orthogonally to $A M$. What is the locus of $P$ as $M$ varies?","solution":"26. Let $L$ be the midpoint of the edge $A B$. Since $P$ is the orthocenter of $\\triangle A B M$ and $M L$ is its altitude, $P$ lies on $M L$ and therefore belongs to the triangular area $L C D$. Moreover, from the similarity of triangles $A L P$ and $M L B$ we have $L P \\cdot L M=L A \\cdot L B=a^{2} \/ 4$, where $a$ is the side length of tetrahedron $A B C D$. It easily follows that the locus of $P$ is the image of the segment $C D$ under the inversion of the plane $L C D$ with center $L$ and radius $a \/ 2$. This locus is the arc of a circle with center $L$ and endpoints at the orthocenters of triangles $A B C$ and $A B D$.","problem_type":null,"tier":0} +{"year":"1967","problem_phase":"longlisted","problem":"27. (ITA 2) Which regular polygons can be obtained (and how) by cutting a cube with a plane?","solution":"27. Regular polygons with 3,4 , and 6 sides can be obtained by cutting a cube with a plane, as shown in the figure. A polygon with more than 6 sides cannot be obtained in such a way, for a cube has 6 faces. Also, if a pentagon is obtained by cutting a ![](https:\/\/cdn.mathpix.com\/cropped\/2024_11_18_8e985d6b9c83aa3e9d0eg-361.jpg?height=299&width=312&top_left_y=991&top_left_x=935) cube with a plane, then its sides lying on opposite faces are parallel; hence it cannot be regular.","problem_type":null,"tier":0} +{"year":"1967","problem_phase":"longlisted","problem":"28. (ITA 3) Find values of the parameter $u$ for which the expression $$ y=\\frac{\\tan (x-u)+\\tan x+\\tan (x+u)}{\\tan (x-u) \\tan x \\tan (x+u)} $$ does not depend on $x$.","solution":"28. The given expression can be transformed into $$ y=\\frac{4 \\cos 2 u+2}{\\cos 2 u-\\cos 2 x}-3 . $$ It does not depend on $x$ if and only if $\\cos 2 u=-1 \/ 2$, i.e., $u= \\pm \\pi \/ 3+k \\pi$ for some $k \\in \\mathbb{Z}$.","problem_type":null,"tier":0} +{"year":"1967","problem_phase":"longlisted","problem":"29. (ITA 4) ${ }^{\\mathrm{IMO} 4}$ The triangles $A_{0} B_{0} C_{0}$ and $A^{\\prime} B^{\\prime} C^{\\prime}$ have all their angles acute. Describe how to construct one of the triangles $A B C$, similar to $A^{\\prime} B^{\\prime} C^{\\prime}$ and circumscribing $A_{0} B_{0} C_{0}$ (so that $A, B, C$ correspond to $A^{\\prime}$, $B^{\\prime}, C^{\\prime}$, and $A B$ passes through $C_{0}, B C$ through $A_{0}$, and $C A$ through $B_{0}$ ). Among these triangles $A B C$, describe, and prove, how to construct the triangle with the maximum area.","solution":"29. Let arc $l_{a}$ be the locus of points $A$ lying on the opposite side from $A_{0}$ with respect to the line $B_{0} C_{0}$ such that $\\angle B_{0} A C_{0}=\\angle A^{\\prime}$. Let $k_{a}$ be the circle containing $l_{a}$, and let $S_{a}$ be the center of $k_{a}$. We similarly define $l_{b}, l_{c}, k_{b}, k_{c}, S_{b}, S_{c}$. It is easy to show that circles $k_{a}, k_{b}, k_{c}$ have a common point $S$ inside $\\triangle A B C$. Let $A_{1}, B_{1}, C_{1}$ be the points on the arcs $l_{a}, l_{b}, l_{c}$ diametrically opposite to $S$ with respect to $S_{a}, S_{b}, S_{c}$ respectively. Then $A_{0} \\in B_{1} C_{1}$ because $\\angle B_{1} A_{0} S=\\angle C_{1} A_{0} S=90^{\\circ}$; similarly, $B_{0} \\in A_{1} C_{1}$ and $C_{0} \\in A_{1} B_{1}$. Hence the triangle $A_{1} B_{1} C_{1}$ is circumscribed about $\\triangle A_{0} B_{0} C_{0}$ and similar to $\\triangle A^{\\prime} B^{\\prime} C^{\\prime}$. Moreover, we claim that $\\triangle A_{1} B_{1} C_{1}$ is the triangle $A B C$ with the desired properties having the maximum side $B C$ and hence the maximum area. Indeed, if $A B C$ is any other such triangle and $S_{b}^{\\prime}, S_{c}^{\\prime}$ are the projections of $S_{b}$ and $S_{c}$ onto the line $B C$, it holds that $B C=2 S_{b}^{\\prime} S_{c}^{\\prime} \\leq 2 S_{b} S_{c}=B_{1} C_{1}$, which proves the maximality of $B_{1} C_{1}$.","problem_type":null,"tier":0} +{"year":"1967","problem_phase":"longlisted","problem":"3. (BUL 3) Prove the trigonometric inequality $\\cos x<1-\\frac{x^{2}}{2}+\\frac{x^{4}}{16}$, where $x \\in(0, \\pi \/ 2)$.","solution":"3. Consider the function $f:[0, \\pi \/ 2] \\rightarrow \\mathbb{R}$ defined by $f(x)=1-x^{2} \/ 2+$ $x^{4} \/ 16-\\cos x$. It is easy to calculate that $f^{\\prime}(0)=f^{\\prime \\prime}(0)=f^{\\prime \\prime \\prime}(0)=0$ and $f^{\\prime \\prime \\prime \\prime}(x)=$ $3 \/ 2-\\cos x$. Since $f^{\\prime \\prime \\prime \\prime}(x)>0, f^{\\prime \\prime \\prime}(x)$ is increasing. Together with $f^{\\prime \\prime \\prime}(0)=0$, this gives $f^{\\prime \\prime \\prime}(x)>0$ for $x>0$; hence $f^{\\prime \\prime}(x)$ is increasing, etc. Continuing in the same way we easily conclude that $f(x)>0$.","problem_type":null,"tier":0} +{"year":"1967","problem_phase":"longlisted","problem":"30. (MON 1) Given $m+n$ numbers $a_{i}(i=1,2, \\ldots, m), b_{j}(j=1,2, \\ldots, n)$, determine the number of pairs $\\left(a_{i}, b_{j}\\right)$ for which $|i-j| \\geq k$, where $k$ is a nonnegative integer.","solution":"30. We assume w.l.o.g. that $m \\leq n$. Let $r$ and $s$ be the numbers of pairs for which $i-j \\geq k$ and of those for which $j-i \\geq k$. The desired number is $r+s$. We easily find that $$ \\begin{aligned} & r= \\begin{cases}(m-k)(m-k+1) \/ 2, & k0(i=1,2, \\ldots, k), k \\in N, n \\in N$.","solution":"47. Using the $\\mathrm{A}-\\mathrm{G}$ mean inequality we get $$ \\begin{gathered} (n+k-1) x_{1}^{n} x_{2} \\cdots x_{k} \\leq n x_{1}^{n+k-1}+x_{2}^{n+k-1}+\\cdots+x_{k}^{n+k-1} \\\\ (n+k-1) x_{1} x_{2}^{n} \\cdots x_{k} \\leq x_{1}^{n+k-1}+n x_{2}^{n+k-1}+\\cdots+x_{k}^{n+k-1} \\\\ \\cdots \\cdots \\cdots \\\\ \\cdots \\cdots \\cdots \\\\ (n+k-1) x_{1} x_{2} \\cdots x_{k}^{n} \\leq x_{1}^{n+k-1}+x_{2}^{n+k-1}+\\cdots+n x_{k}^{n+k-1} \\end{gathered} $$ By adding these inequalities and dividing by $n+k-1$ we obtain the desired one. Remark. This is also an immediate consequence of Muirhead's inequality.","problem_type":null,"tier":0} +{"year":"1967","problem_phase":"longlisted","problem":"48. (SWE 1) Determine all positive roots of the equation $x^{x}=1 \/ \\sqrt{2}$.","solution":"48. Put $f(x)=x \\ln x$. The given equation is equivalent to $f(x)=f(1 \/ 2)$, which has the solutions $x_{1}=1 \/ 2$ and $x_{2}=1 \/ 4$. Since the function $f$ is decreasing on $(0,1 \/ e)$, and increasing on $(1 \/ e,+\\infty)$, this equation has no other solutions.","problem_type":null,"tier":0} +{"year":"1967","problem_phase":"longlisted","problem":"49. (SWE 2) Let $n$ and $k$ be positive integers such that $1 \\leq n \\leq N+1$, $1 \\leq k \\leq N+1$. Show that $$ \\min _{n \\neq k}|\\sin n-\\sin k|<\\frac{2}{N} $$","solution":"49. Since $\\sin 1, \\sin 2, \\ldots, \\sin (N+1) \\in(-1,1)$, two of these $N+1$ numbers have distance less than $2 \/ N$. Therefore $|\\sin n-\\sin k|<2 \/ N$ for some integers $1 \\leq k, n \\leq N+1, n \\neq k$.","problem_type":null,"tier":0} +{"year":"1967","problem_phase":"longlisted","problem":"5. (BUL 5) Solve the system $$ \\begin{aligned} & x^{2}+x-1=y \\\\ & y^{2}+y-1=z \\\\ & z^{2}+z-1=x \\end{aligned} $$","solution":"5. If one of $x, y, z$ is equal to 1 or -1 , then we obtain solutions $(-1,-1,-1)$ and $(1,1,1)$. We claim that these are the only solutions to the system. Let $f(t)=t^{2}+t-1$. If among $x, y, z$ one is greater than 1 , say $x>1$, we have $xf(x)=y>f(y)=z>f(z)=x$, a contradiction. This proves our claim.","problem_type":null,"tier":0} +{"year":"1967","problem_phase":"longlisted","problem":"50. (SWE 3) The function $\\varphi(x, y, z)$, defined for all triples $(x, y, z)$ of real numbers, is such that there are two functions $f$ and $g$ defined for all pairs of real numbers such that $$ \\varphi(x, y, z)=f(x+y, z)=g(x, y+z) $$ for all real $x, y$, and $z$. Show that there is a function $h$ of one real variable such that $$ \\varphi(x, y, z)=h(x+y+z) $$ for all real $x, y$, and $z$.","solution":"50. Since $\\varphi(x, y, z)=f(x+y, z)=\\varphi(0, x+y, z)=g(0, x+y+z)$, it is enough to put $h(t)=g(0, t)$.","problem_type":null,"tier":0} +{"year":"1967","problem_phase":"longlisted","problem":"51. (SWE 4) A subset $S$ of the set of integers $0, \\ldots, 99$ is said to have property A if it is impossible to fill a crossword puzzle with 2 rows and 2 columns with numbers in $S$ ( 0 is written as 00,1 as 01 , and so on). Determine the maximal number of elements in sets $S$ with property A.","solution":"51. If there exist two numbers $\\overline{a b}, \\overline{b c} \\in S$, then one can fill a crossword puzzle as $\\left(\\begin{array}{ll}a & b \\\\ b & c\\end{array}\\right)$. The converse is obvious. Hence the set $S$ has property $A$ if and only if the set of first digits and the set of second digits of numbers in $S$ are disjoint. Thus the maximum size of $S$ is 25 .","problem_type":null,"tier":0} +{"year":"1967","problem_phase":"longlisted","problem":"52. (SWE 5) In the plane a point $O$ and a sequence of points $P_{1}, P_{2}, P_{3}, \\ldots$ are given. The distances $O P_{1}, O P_{2}, O P_{3}, \\ldots$ are $r_{1}, r_{2}, r_{3}, \\ldots$, where $r_{1} \\leq$ $r_{2} \\leq r_{3} \\leq \\cdots$. Let $\\alpha$ satisfy $0<\\alpha<1$. Suppose that for every $n$ the distance from the point $P_{n}$ to any other point of the sequence is greater than or equal to $r_{n}^{\\alpha}$. Determine the exponent $\\beta$, as large as possible, such that for some $C$ independent of $n,{ }^{2}$ $$ r_{n} \\geq C n^{\\beta}, \\quad n=1,2, \\ldots $$","solution":"52. This problem is not elementary. The solution offered by the proposer was not quite clear and complete (the existence was not proved).","problem_type":null,"tier":0} +{"year":"1967","problem_phase":"longlisted","problem":"53. (SWE 6) In making Euclidean constructions in geometry it is permitted to use a straightedge and compass. In the constructions considered in this question, no compasses are permitted, but the straightedge is assumed to have two parallel edges, which can be used for constructing two parallel lines through two given points whose distance is at least equal to the breadth of the ruler. Then the distance between the parallel lines is equal to the breadth of the straightedge. Carry through the following constructions with such a straightedge. Construct: (a) The bisector of a given angle. (b) The midpoint of a given rectilinear segment. (c) The center of a circle through three given noncollinear points. (d) A line through a given point parallel to a given line.","solution":"53. (a) We can construct two lines parallel to the rays of the angle, at equal distances from the rays. The intersection of these two lines lies on the bisector of the angle. (b) If the length of a segment $A B$ exceeds the breadth of the ruler, we can construct parallel lines through $A$ and $B$ in two different ways. The diagonal in the resulting rhombus is the perpendicular bisector of the segment $A B$. If the segment $A B$ is too short, we can construct a line $l$ parallel to $A B$ and centrally project $A B$ onto $l$ from a point $C$ chosen sufficiently close to the segment, thus obtaining an arbitrarily long segment $A^{\\prime} B^{\\prime} \\|$ $A B$. Then we construct the midpoint $D^{\\prime}$ of $A^{\\prime} B^{\\prime}$ as above. The line $D^{\\prime} C$ intersects the segment $A B$ at its midpoint $D$. By means of lines parallel to $D C$ the segment $A B$ can be prolonged symmetrically, and then the perpendicular bisector can be found as above. (c) follows immediately from part (b). (d) Let there be given a point $P$ and a line $l$. We draw an arbitrary line through $P$ that intersects $l$ at $A$, and two lines $l_{1}$ and $l_{2}$ parallel to $A P$, at equal distances from $A P$ and on either side of $A P$. Line $l_{1}$ intersects $l$ at $B$. We can construct the midpoint $C$ of $A P$. If $B C$ intersects $l_{2}$ at $D$, then $P D$ is parallel to $l$.","problem_type":null,"tier":0} +{"year":"1967","problem_phase":"longlisted","problem":"54. (USS 1) Is it possible to put 100 (or 200) points on a wooden cube such that by all rotations of the cube the points map into themselves? Justify your answer.","solution":"54. Let $S$ be the given set of points on the cube. Let $x, y, z$ denote the numbers of points from $S$ lying at a vertex, at the midpoint of an edge, at the midpoint of a face of the cube, respectively, and let $u$ be the number of all other points from $S$. Either there are no points from $S$ at the vertices of the cube, or there is a point from $S$ at each vertex. Hence $x$ is either 0 or 8 . Similarly, $y$ is either 0 or 12 , and $z$ is either 0 or 6 . Any other point of $S$ has 24 possible images under rotations of the cube. Hence $u$ is divisible by 24 . Since $n=x+y+z+u$ and $6 \\mid y, z, u$, it follows that either $6 \\mid n$ or $6 \\mid n-8$, i.e., $n \\equiv 0$ or $n \\equiv 2(\\bmod 6)$. Thus $n=200$ is possible, while $n=100$ is not, because $n \\equiv 4(\\bmod 6)$.","problem_type":null,"tier":0} +{"year":"1967","problem_phase":"longlisted","problem":"55. (USS 2) Find all $x$ for which for all $n$, $$ \\sin x+\\sin 2 x+\\sin 3 x+\\cdots+\\sin n x \\leq \\frac{\\sqrt{3}}{2} $$","solution":"55. It is enough to find all $x$ from $(0,2 \\pi]$ such that the given inequality holds for all $n$. Suppose $0\\sqrt{3} \/ 2$. Suppose now that $2 \\pi \/ 3 \\leq x<2 \\pi$. We have $$ \\sin x+\\cdots+\\sin n x=\\frac{\\cos \\frac{x}{2}-\\cos \\frac{2 n+1}{2} x}{2 \\sin \\frac{x}{2}} \\leq \\frac{\\cos \\frac{x}{2}+1}{2 \\sin \\frac{x}{2}}=\\frac{\\cot \\frac{x}{4}}{2} \\leq \\frac{\\sqrt{3}}{2} . $$ For $x=2 \\pi$ the given inequality clearly holds for all $n$. Hence, the inequality holds for all $n$ if and only if $2 \\pi \/ 3+2 k \\pi \\leq x \\leq 2 \\pi+2 k \\pi$ for some integer $k$.","problem_type":null,"tier":0} +{"year":"1967","problem_phase":"longlisted","problem":"56. (USS 3) In a group of interpreters each one speaks one or several foreign languages; 24 of them speak Japanese, 24 Malay, 24 Farsi. Prove that it is possible to select a subgroup in which exactly 12 interpreters speak Japanese, exactly 12 speak Malay, and exactly 12 speak Farsi.","solution":"56. We shall prove by induction on $n$ the following statement: If in some group of interpreters exactly $n$ persons, $n \\geq 2$, speak each of the three languages, then it is possible to select a subgroup in which each language is spoken by exactly two persons. The statement of the problem easily follows from this: it suffices to select six such groups. The case $n=2$ is trivial. Let us assume $n \\geq 2$, and let $N_{j}, N_{m}, N_{f}, N_{j m}$, $N_{j f}, N_{m f}, N_{j m f}$ be the sets of those interpreters who speak only Japanese, only Malay, only Farsi, only Japanese and Malay, only Japanese and Farsi, only Malay and Farsi, and all the three languages, respectively, and $n_{j}, n_{m}$, $n_{f}, n_{j m}, n_{j f}, n_{m f}, n_{j m f}$ the cardinalities of these sets, respectively. By the condition of the problem, $n_{j}+n_{j m}+n_{j f}+n_{j m f}=n_{m}+n_{j m}+n_{m f}+n_{j m f}=$ $n_{f}+n_{j f}+n_{m f}+n_{j m f}=24$, and consequently $$ n_{j}-n_{m f}=n_{m}-n_{j f}=n_{f}-n_{j m}=c $$ Now if $c<0$, then $n_{j m}, n_{j f}, n_{m f}>0$, and it is enough to select one interpreter from each of the sets $N_{j m}, N_{j f}, N_{m f}$. If $c>0$, then $n_{j}, n_{m}, n_{f}>0$, and it is enough to select one interpreter from each of the sets $N_{j}, N_{m}, N_{f}$ and then use the inductive assumption. Also, if $c=0$, then w.l.o.g. $n_{j}=n_{m f}>0$, and it is enough to select one interpreter from each of the sets $N_{j}, N_{m f}$ and then use the inductive hypothesis. This completes the induction.","problem_type":null,"tier":0} +{"year":"1967","problem_phase":"longlisted","problem":"57. (USS 4) ${ }^{\\mathrm{IMO} 5}$ Consider the sequence $\\left(c_{n}\\right)$ : $$ \\begin{gathered} c_{1}=a_{1}+a_{2}+\\cdots+a_{8}, \\\\ c_{2}=a_{1}^{2}+a_{2}^{2}+\\cdots+a_{8}^{2}, \\\\ \\cdots \\\\ \\cdots \\cdots \\cdots \\\\ c_{n}=a_{1}^{n}+a_{2}^{n}+\\cdots+a_{8}^{n}, \\end{gathered} $$ [^1]where $a_{1}, a_{2}, \\ldots, a_{8}$ are real numbers, not all equal to zero. Given that among the numbers of the sequence $\\left(c_{n}\\right)$ there are infinitely many equal to zero, determine all the values of $n$ for which $c_{n}=0$.","solution":"57. Obviously $c_{n}>0$ for all even $n$. Thus $c_{n}=0$ is possible only for an odd $n$. Let us assume $a_{1} \\leq a_{2} \\leq \\cdots \\leq a_{8}$ : in particular, $a_{1} \\leq 0 \\leq a_{8}$. If $\\left|a_{1}\\right|<\\left|a_{8}\\right|$, then there exists $n_{0}$ such that for every odd $n>n_{0}, 7\\left|a_{1}\\right|^{n}<$ $a_{8}^{n} \\Rightarrow a_{1}^{n}+\\cdots+a_{7}^{n}+a_{8}^{n}>7 a_{1}^{n}+a_{8}^{n}>0$, contradicting the condition that $c_{n}=0$ for infinitely many $n$. Similarly $\\left|a_{1}\\right|>\\left|a_{8}\\right|$ is impossible, and we conclude that $a_{1}=-a_{8}$. Continuing in the same manner we can show that $a_{2}=-a_{7}, a_{3}=-a_{6}$ and $a_{4}=-a_{5}$. Hence $c_{n}=0$ for every odd $n$.","problem_type":null,"tier":0} +{"year":"1967","problem_phase":"longlisted","problem":"58. (USS 5) A linear binomial $l(z)=A z+B$ with complex coefficients $A$ and $B$ is given. It is known that the maximal value of $|l(z)|$ on the segment $-1 \\leq x \\leq 1(y=0)$ of the real line in the complex plane $(z=x+i y)$ is equal to $M$. Prove that for every $z$ $$ |l(z)| \\leq M \\rho, $$ where $\\rho$ is the sum of distances from the point $P=z$ to the points $Q_{1}$ : $z=1$ and $Q_{3}: z=-1$.","solution":"58. The following sequence of equalities and inequalities gives an even stronger estimate than needed. $$ \\begin{aligned} |l(z)| & =|A z+B|=\\frac{1}{2}|(z+1)(A+B)+(z-1)(A-B)| \\\\ & =\\frac{1}{2}|(z+1) f(1)+(z-1) f(-1)| \\\\ & \\leq \\frac{1}{2}(|z+1| \\cdot|f(1)|+|z-1| \\cdot|f(-1)|) \\\\ & \\leq \\frac{1}{2}(|z+1|+|z-1|) M=\\frac{1}{2} \\rho M . \\end{aligned} $$","problem_type":null,"tier":0} +{"year":"1967","problem_phase":"longlisted","problem":"59. (USS 6) On the circle with center $O$ and radius 1 the point $A_{0}$ is fixed and points $A_{1}, A_{2}, \\ldots, A_{999}, A_{1000}$ are distributed in such a way that $\\angle A_{0} O A_{k}=k$ (in radians). Cut the circle at points $A_{0}, A_{1}, \\ldots, A_{1000}$. How many arcs with different lengths are obtained? ### 3.10 The Tenth IMO","solution":"59. By the $\\operatorname{arc} A B$ we shall always mean the positive $\\operatorname{arc} A B$. We denote by $|A B|$ the length of arc $A B$. Let a basic arc be one of the $n+1$ arcs into which the circle is partitioned by the points $A_{0}, A_{1}, \\ldots, A_{n}$, where $n \\in \\mathbb{N}$. Suppose that $A_{p} A_{0}$ and $A_{0} A_{q}$ are the basic arcs with an endpoint at $A_{0}$, and that $x_{n}, y_{n}$ are their lengths, respectively. We show by induction on $n$ that for each $n$ the length of a basic arc is equal to $x_{n}, y_{n}$ or $x_{n}+y_{n}$. The statement is trivial for $n=1$. Assume that it holds for $n$, and let $A_{i} A_{n+1}, A_{n+1} A_{j}$ be basic arcs. We shall prove that these two arcs have lengths $x_{n}, y_{n}$, or $x_{n}+y_{n}$. If $i, j$ are both strictly positive, then $\\left|A_{i} A_{n+1}\\right|=$ $\\left|A_{i-1} A_{n}\\right|$ and $\\left|A_{n+1} A_{j}\\right|=\\left|A_{n} A_{j-1}\\right|$ are equal to $x_{n}, y_{n}$, or $x_{n}+y_{n}$ by the inductive hypothesis. Let us assume now that $i=0$, i.e., that $A_{p} A_{n+1}$ and $A_{n+1} A_{0}$ are basic arcs. Then $\\left|A_{p} A_{n+1}\\right|=\\left|A_{0} A_{n+1-p}\\right| \\geq\\left|A_{0} A_{q}\\right|=y_{n}$ and similarly $\\left|A_{n+1} A_{q}\\right| \\geq x_{n}$, but $\\left|A_{p} A_{q}\\right|=x_{n}+y_{n}$, from which it follows that $\\left|A_{p} A_{n+1}\\right|=\\left|A_{0} A_{q}\\right|=y_{n}$ and consequently $n+1=p+q$. Also, $x_{n+1}=\\left|A_{n+1} A_{0}\\right|=y_{n}-x_{n}$ and $y_{n+1}=y_{n}$. Now, all basic arcs have lengths $y_{n}-x_{n}, x_{n}, y_{n}, x_{n}+y_{n}$. A presence of a basic arc of length $x_{n}+y_{n}$ would spoil our inductive step. However, if any basic arc $A_{k} A_{l}$ has length $x_{n}+y_{n}$, then we must have $l-q=k-p$ because $2 \\pi$ is irrational, and therefore the arc $A_{k} A_{l}$ contains either the point $A_{k-p}$ (if $k \\geq p$ ) or the point $A_{k+q}$ (if $k1 \\\\ \\left(p_{k}, p_{k}+q_{k}\\right), \\text { if }\\left\\{p_{k} \/(2 \\pi)\\right\\}+\\left\\{q_{k} \/(2 \\pi)\\right\\}<1 \\end{array}\\right. $$ It is now \"easy\" to calculate that $p_{19}=p_{20}=333, q_{19}=377, q_{20}=710$, and thus $n_{19}=709<1000<1042=n_{20}$. It follows that the lengths of the basic arcs for $n=1000$ take exactly three different values.","problem_type":null,"tier":0} +{"year":"1967","problem_phase":"longlisted","problem":"6. (BUL 6) Solve the system $$ \\begin{aligned} |x+y|+|1-x| & =6 \\\\ |x+y+1|+|1-y| & =4 \\end{aligned} $$","solution":"6. The given system has two solutions: $(-2,-1)$ and $(-14 \/ 3,13 \/ 3)$.","problem_type":null,"tier":0} +{"year":"1967","problem_phase":"longlisted","problem":"7. (CZS 1) Find all real solutions of the system of equations $$ \\begin{aligned} x_{1}+x_{2}+\\cdots+x_{n} & =a, \\\\ x_{1}^{2}+x_{2}^{2}+\\cdots+x_{n}^{2} & =a^{2}, \\\\ \\ldots \\cdots \\cdots \\cdots+x_{n}^{n} & =a^{n} . \\end{aligned} $$","solution":"7. Let $S_{k}=x_{1}^{k}+x_{2}^{k}+\\cdots+x_{n}^{k}$ and let $\\sigma_{k}, k=1,2, \\ldots, n$ denote the $k$ th elementary symmetric polynomial in $x_{1}, \\ldots, x_{n}$. The given system can be written as $S_{k}=a^{k}, k=1, \\ldots, n$. Using Newton's formulas $$ k \\sigma_{k}=S_{1} \\sigma_{k-1}-S_{2} \\sigma_{k-2}+\\cdots+(-1)^{k} S_{k-1} \\sigma_{1}+(-1)^{k-1} S_{k}, \\quad k=1,2, \\ldots, n $$ the system easily leads to $\\sigma_{1}=a$ and $\\sigma_{k}=0$ for $k=2, \\ldots, n$. By Vieta's formulas, $x_{1}, x_{2}, \\ldots, x_{n}$ are the roots of the polynomial $x^{n}-a x^{n-1}$, i.e., $a, 0,0, \\ldots, 0$ in some order. Remark. This solution does not use the assumption that the $x_{j}$ 's are real.","problem_type":null,"tier":0} +{"year":"1967","problem_phase":"longlisted","problem":"8. $(\\mathbf{C Z S} 2)^{\\mathrm{IMO1}} A B C D$ is a parallelogram; $A B=a, A D=1, \\alpha$ is the size of $\\angle D A B$, and the three angles of the triangle $A B D$ are acute. Prove that the four circles $K_{A}, K_{B}, K_{C}, K_{D}$, each of radius 1, whose centers are the vertices $A, B, C, D$, cover the parallelogram if and only if $a \\leq$ $\\cos \\alpha+\\sqrt{3} \\sin \\alpha$.","solution":"8. The circles $K_{A}, K_{B}, K_{C}, K_{D}$ cover the parallelogram if and only if for every point $X$ inside the parallelogram, the length of one of the segments $X A, X B, X C, X D$ does not exceed 1. Let $O$ and $r$ be the center and radius of the circumcircle of $\\triangle A B D$. For every point $X$ inside $\\triangle A B D$, it holds that $X A \\leq r$ or $X B \\leq r$ or $X D \\leq r$. Similarly, for $X$ inside $\\triangle B C D, X B \\leq r$ or $X C \\leq r$ or $X D \\leq r$. Hence $K_{A}, K_{B}, K_{C}, K_{D}$ cover the parallelogram if and only if $r \\leq 1$, which is equivalent to $\\angle A B D \\geq 30^{\\circ}$. However, this last is exactly equivalent to $a=A B=2 r \\sin \\angle A D B \\leq 2 \\sin \\left(\\alpha+30^{\\circ}\\right)=\\sqrt{3} \\sin \\alpha+\\cos \\alpha$.","problem_type":null,"tier":0} +{"year":"1967","problem_phase":"longlisted","problem":"9. (CZS 3) The circle $k$ and its diameter $A B$ are given. Find the locus of the centers of circles inscribed in the triangles having one vertex on $A B$ and two other vertices on $k$.","solution":"9. The incenter of any such triangle lies inside the circle $k$. We shall show that every point $S$ interior to the circle $S$ is the incenter of one such triangle. If $S$ lies on the segment $A B$, then it is obviously the incenter of an isosceles triangle inscribed in $k$ that has $A B$ as an axis of symmetry. Let us now suppose $S$ does not lie on $A B$. Let $X$ and $Y$ be the intersection points of lines $A S$ and $B S$ with $k$, and let $Z$ be the foot of the perpendicular from $S$ to $A B$. Since the quadrilateral $B Z S X$ is cyclic, we have $\\angle Z X S=$ $\\angle A B S=\\angle S X Y$ and analogously $\\angle Z Y S=\\angle S Y X$, which implies that $S$ is the incenter of $\\triangle X Y Z$.","problem_type":null,"tier":0} +{"year":"1968","problem_phase":"shortlisted","problem":"1. (SWE 2) Two ships sail on the sea with constant speeds and fixed directions. It is known that at 9:00 the distance between them was 20 miles; at 9:35, 15 miles; and at 9:55, 13 miles. At what moment were the ships the smallest distance from each other, and what was that distance?","solution":"1. Since the ships are sailing with constant speeds and directions, the second ship is sailing at a constant speed and direction in reference to the first ship. Let $A$ be the constant position of the first ship in this frame. Let $B_{1}$, $B_{2}, B_{3}$, and $B$ on line $b$ defining the trajectory of the ship be positions of the second ship with respect to the first ship at 9:00, 9:35, 9:55, and at the moment the two ships were closest. Then we have the following equations for distances (in miles): $$ \\begin{gathered} A B_{1}=20, \\quad A B_{2}=15, \\quad A B_{3}=13 \\\\ B_{1} B_{2}: B_{2} B_{3}=7: 4, \\quad A B_{i}^{2}=A B^{2}+B B_{i}^{2} \\end{gathered} $$ Since $B B_{1}>B B_{2}>B B_{3}$, it follows that $\\mathcal{B}\\left(B_{3}, B, B_{2}, B_{1}\\right)$ or $\\mathcal{B}\\left(B, B_{3}, B_{2}\\right.$, $B_{1}$ ). We get a system of three quadratic equations with three unknowns: $A B, B B_{3}$ and $B_{3} B_{2}$ ( $B B_{3}$ being negative if $\\mathcal{B}\\left(B_{3}, B, B_{1}, B_{2}\\right)$, positive otherwise). This can be solved by eliminating $A B$ and then $B B_{3}$. The unique solution ends up being $$ A B=12, \\quad B B_{3}=5, \\quad B_{3} B_{2}=4 $$ and consequently, the two ships are closest at 10:20 when they are at a distance of 12 miles.","problem_type":null,"tier":0} +{"year":"1968","problem_phase":"shortlisted","problem":"10. (ROM 4) Consider two segments of length $a, b(a>b)$ and a segment of length $c=\\sqrt{a b}$. (a) For what values of $a \/ b$ can these segments be sides of a triangle? (b) For what values of $a \/ b$ is this triangle right-angled, obtuse-angled, or acute-angled?","solution":"10. (a) Let us set $k=a \/ b>1$. Then $a=k b$ and $c=\\sqrt{k} b$, and $a>c>b$. The segments $a, b, c$ form a triangle if and only if $k<\\sqrt{k}+1$, which holds if and only if $1120^{\\circ}-60^{\\circ}=60^{\\circ}$ (because $\\angle A_{1} A_{j} A_{i}<60^{\\circ}$ ); hence $\\angle A_{i} A_{j} A_{k} \\geq 120^{\\circ}$. This proves that the denotation is correct. Remark. It is easy to show that the diameter is unique. Hence the denotation is also unique.","problem_type":null,"tier":0} +{"year":"1968","problem_phase":"shortlisted","problem":"21. (CZS 2) Let $a_{0}, a_{1}, \\ldots, a_{k}(k \\geq 1)$ be positive integers. Find all positive integers $y$ such that $$ a_{0}\\left|y ; \\quad\\left(a_{0}+a_{1}\\right)\\right|\\left(y+a_{1}\\right) ; \\ldots ; \\quad\\left(a_{0}+a_{n}\\right) \\mid\\left(y+a_{n}\\right) . $$","solution":"21. The given conditions are equivalent to $y-a_{0}$ being divisible by $a_{0}, a_{0}+$ $a_{1}, a_{0}+a_{2}, \\ldots, a_{0}+a_{n}$, i.e., to $y=k\\left[a_{0}, a_{0}+a_{1}, \\ldots, a_{0}+a_{n}\\right]+a_{0}, k \\in \\mathbb{N}_{0}$.","problem_type":null,"tier":0} +{"year":"1968","problem_phase":"shortlisted","problem":"22. (CZS 3) ${ }^{\\mathrm{IMO} 2}$ Find all positive integers $x$ for which $p(x)=x^{2}-10 x-22$, where $p(x)$ denotes the product of the digits of $x$.","solution":"22. It can be shown by induction on the number of digits of $x$ that $p(x) \\leq x$ for all $x \\in \\mathbb{N}$. It follows that $x^{2}-10 x-22 \\leq x$, which implies $x \\leq 12$. Since $0k-j$ and $9^{k-j-1} 9$ !\/ $(9-j)$ ! otherwise. If the $i$ th digit is not 0 , then the above results are multiplied by 8 .","problem_type":null,"tier":0} +{"year":"1968","problem_phase":"shortlisted","problem":"25. (MON 2) Given $k$ parallel lines and a few points on each of them, find the number of all possible triangles with vertices at these given points. ${ }^{4}$","solution":"25. The answer is $$ \\sum_{1 \\leq p0$ be a real number and $f(x)$ a real function defined on all of $\\mathbb{R}$, satisfying for all $x \\in \\mathbb{R}$, $$ f(x+a)=\\frac{1}{2}+\\sqrt{f(x)-f(x)^{2}} . $$ (a) Prove that the function $f$ is periodic; i.e., there exists $b>0$ such that for all $x, f(x+b)=f(x)$. (b) Give an example of such a nonconstant function for $a=1$. [^2]","solution":"26. (a) We shall show that the period of $f$ is $2 a$. From $(f(x+a)-1 \/ 2)^{2}=$ $f(x)-f(x)^{2}$ we obtain $$ \\left(f(x)-f(x)^{2}\\right)+\\left(f(x+a)-f(x+a)^{2}\\right)=\\frac{1}{4} $$ Subtracting the above relation for $x+a$ in place of $x$ we get $f(x)-$ $f(x)^{2}=f(x+2 a)-f(x+2 a)^{2}$, which implies $(f(x)-1 \/ 2)^{2}=$ $(f(x+2 a)-1 \/ 2)^{2}$. Since $f(x) \\geq 1 \/ 2$ holds for all $x$ by the condition of the problem, we conclude that $f(x+2 a)=f(x)$. (b) The following function, as is directly verified, satisfies the conditions: $$ f(x)=\\left\\{\\begin{array}{cl} 1 \/ 2 & \\text { if } 2 n \\leq x<2 n+1, \\\\ 1 & \\text { if } 2 n+1 \\leq x<2 n+2, \\end{array} \\text { for } n=0,1,2, \\ldots\\right. $$","problem_type":null,"tier":0} +{"year":"1968","problem_phase":"shortlisted","problem":"3. (POL 4) ${ }^{\\mathrm{IMO} 4}$ Prove that in any tetrahedron there is a vertex such that the lengths of its sides through that vertex are sides of a triangle.","solution":"3. A triangle cannot be formed out of three lengths if and only if one of them is larger than the sum of the other two. Let us assume this is the case for all triplets of edges out of each vertex in a tetrahedron $A B C D$. Let w.l.o.g. $A B$ be the largest edge of the tetrahedron. Then $A B \\geq A C+A D$ and $A B \\geq B C+B D$, from which it follows that $2 A B \\geq A C+A D+B C+B D$. This implies that either $A B \\geq A C+B C$ or $A B \\geq A D+B D$, contradicting the triangle inequality. Hence the three edges coming out of at least one of the vertices $A$ and $B$ form a triangle. Remark. The proof can be generalized to prove that in a polyhedron with only triangular surfaces there is a vertex such that the edges coming out of this vertex form a triangle.","problem_type":null,"tier":0} +{"year":"1968","problem_phase":"shortlisted","problem":"4. (BUL 2) ${ }^{\\mathrm{IMO} 3}$ Let $a, b, c$ be real numbers. Prove that the system of equations $$ \\left\\{\\begin{array}{r} a x_{1}^{2}+b x_{1}+c=x_{2} \\\\ a x_{2}^{2}+b x_{2}+c=x_{3} \\\\ \\cdots \\cdots \\cdots \\cdots \\\\ a x_{n-1}^{2}+b x_{n-1}+c=x_{n} \\\\ a x_{n}^{2}+b x_{n}+c=x_{1} \\end{array}\\right. $$ has a unique real solution if and only if $(b-1)^{2}-4 a c=0$. Remark. It is assumed that $a \\neq 0$.","solution":"4. We will prove the equivalence in the two directions separately: $(\\Rightarrow)$ Suppose $\\left\\{x_{1}, \\ldots, x_{n}\\right\\}$ is the unique solution of the equation. Since $\\left\\{x_{n}, x_{1}, x_{2} \\ldots, x_{n-1}\\right\\}$ is also a solution, it follows that $x_{1}=x_{2}=\\cdots=$ $x_{n}=x$ and the system of equations reduces to a single equation $a x^{2}+$ $(b-1) x+c=0$. For the solution for $x$ to be unique the discriminant $(b-1)^{2}-4 a c$ of this quadratic equation must be 0 . $(\\Leftarrow)$ Assume $(b-1)^{2}-4 a c=0$. Adding up the equations, we get $$ \\sum_{i=1}^{n} f\\left(x_{i}\\right)=0, \\quad \\text { where } \\quad f(x)=a x^{2}+(b-1) x+c $$ But by the assumed condition, $f(x)=a\\left(x+\\frac{b-1}{2 a}\\right)^{2}$. Hence we must have $f\\left(x_{i}\\right)=0$ for all $i$, and $x_{i}=-\\frac{b-1}{2 a}$, which is indeed a solution.","problem_type":null,"tier":0} +{"year":"1968","problem_phase":"shortlisted","problem":"5. (BUL 5) Let $h_{n}$ be the apothem (distance from the center to one of the sides) of a regular $n$-gon $(n \\geq 3)$ inscribed in a circle of radius $r$. Prove the inequality $$ (n+1) h_{n+1}-n h_{n}>r . $$ Also prove that if $r$ on the right side is replaced with a greater number, the inequality will not remain true for all $n \\geq 3$.","solution":"5. We have $h_{k}=r \\cos (\\pi \/ k)$ for all $k \\in \\mathbb{N}$. Using $\\cos x=1-2 \\sin ^{2}(x \/ 2)$ and $\\cos x=2 \/\\left(1+\\tan ^{2}(x \/ 2)\\right)-1$ and $\\tan x>x>\\sin x$ for all $01 \\\\ \\Leftrightarrow & 1+2 n\\left(1-\\frac{1}{1+\\pi^{2} \/\\left(4 n^{2}\\right)}\\right)-\\frac{\\pi^{2}}{2(n+1)}>1 \\\\ \\Leftrightarrow & 1+\\frac{\\pi^{2}}{2}\\left(\\frac{1}{n+\\pi^{2} \/(4 n)}-\\frac{1}{n+1}\\right)>1, \\end{aligned} $$ where the last inequality holds because $\\pi^{2}<4 n$. It is also apparent that as $n$ tends to infinity the term in parentheses tends to 0 , and hence it is not possible to strengthen the bound. This completes the proof.","problem_type":null,"tier":0} +{"year":"1968","problem_phase":"shortlisted","problem":"6. (HUN 1) If $a_{i}(i=1,2, \\ldots, n)$ are distinct non-zero real numbers, prove that the equation $$ \\frac{a_{1}}{a_{1}-x}+\\frac{a_{2}}{a_{2}-x}+\\cdots+\\frac{a_{n}}{a_{n}-x}=n $$ has at least $n-1$ real roots.","solution":"6. We define $f(x)=\\frac{a_{1}}{a_{1}-x}+\\frac{a_{2}}{a_{2}-x}+\\cdots+\\frac{a_{n}}{a_{n}-x}$. Let us assume w.l.o.g. $a_{1}1$. We then have $z=n^{4}+4 m^{4}=$ $\\left(n^{2}+2 m^{2}\\right)^{2}-(2 m n)^{2}=\\left(n^{2}+2 m^{2}+2 m n\\right)\\left(n^{2}+2 m^{2}-2 m n\\right)$. Since $n^{2}+2 m^{2}-2 m n=(n-m)^{2}+m^{2} \\geq m^{2}>1$, it follows that $z$ must be composite. Thus we have found infinitely many $a$ that satisfy the condition of the problem.","problem_type":null,"tier":0} +{"year":"1969","problem_phase":"contest","problem":"2. Let $a_{1}, a_{2}, \\ldots, a_{n}$ be real constants and $$ y(x)=\\cos \\left(a_{1}+x\\right)+\\frac{\\cos \\left(a_{2}+x\\right)}{2}+\\frac{\\cos \\left(a_{3}+x\\right)}{2^{2}}+\\cdots+\\frac{\\cos \\left(a_{n}+x\\right)}{2^{n-1}} . $$ If $x_{1}, x_{2}$ are real and $y\\left(x_{1}\\right)=y\\left(x_{2}\\right)=0$, prove that $x_{1}-x_{2}=m \\pi$ for some integer $m$.","solution":"2. Using $\\cos (a+x)=\\cos a \\cos x-\\sin a \\sin x$, we obtain $f(x)=A \\sin x+$ $B \\cos x$ where $A=-\\sin a_{1}-\\sin a_{2} \/ 2-\\cdots-\\sin a_{n} \/ 2^{n-1}$ and $B=\\cos a_{1}+$ $\\cos a_{2} \/ 2+\\cdots+\\cos a_{n} \/ 2^{n-1}$. Numbers $A$ and $B$ cannot both be equal to 0 , for otherwise $f$ would be identically equal to 0 , while on the other hand, we have $f\\left(-a_{1}\\right)=\\cos \\left(a_{1}-a_{1}\\right)+\\cos \\left(a_{2}-a_{1}\\right) \/ 2+\\cdots+\\cos \\left(a_{n}-a_{1}\\right) \/ 2^{n-1} \\geq$ $1-1 \/ 2-\\cdots-1 \/ 2^{n-1}=1 \/ 2^{n-1}>0$. Setting $A=C \\cos \\phi$ and $B=C \\sin \\phi$, where $C \\neq 0$ (such $C$ and $\\phi$ always exist), we get $f(x)=C \\sin (x+\\phi)$. It follows that the zeros of $f$ are of the form $x_{0} \\in-\\phi+\\pi \\mathbb{Z}$, from which $f\\left(x_{1}\\right)=f\\left(x_{2}\\right) \\Rightarrow x_{1}-x_{2}=m \\pi$ immediately follows.","problem_type":null,"tier":0} +{"year":"1969","problem_phase":"contest","problem":"3. Find conditions on the positive real number $a$ such that there exists a tetrahedron $k$ of whose edges $(k=1,2,3,4,5)$ have length $a$, and the other $6-k$ edges have length 1 . Second Day (July 11)","solution":"3. We have several cases: $1^{\\circ} k=1$. W.l.o.g. let $A B=a$ and the remaining segments have length 1. Let $M$ be the midpoint of $C D$. Then $A M=B M=\\sqrt{3} \/ 2(\\triangle C D A$ and $\\triangle C D B$ are equilateral) and $01$. Assume $A B=A C=A D=a$. Varying $A$ along the line perpendicular to the plane $B C D$ and through the center of $\\triangle B C D$ we achieve all values of $a>1 \/ \\sqrt{3}$. For $a<1 \/ \\sqrt{3}$ we can observe a similar tetrahedron with three edges of length $1 \/ a$ and three of length 1 and proceed as before. $4^{\\circ} k=4$. By observing the similar tetrahedron we reduce this case to $k=2$ with length $1 \/ a$ instead of $a$. Thus we get $a>\\sqrt{2-\\sqrt{3}}$. $5^{\\circ} k=5$. We reduce to $k=1$ and get $a>1 \/ \\sqrt{3}$.","problem_type":null,"tier":0} +{"year":"1969","problem_phase":"contest","problem":"4. Let $A B$ be a diameter of a circle $\\gamma$. A point $C$ different from $A$ and $B$ is on the circle $\\gamma$. Let $D$ be the projection of the point $C$ onto the line $A B$. Consider three other circles $\\gamma_{1}, \\gamma_{2}$, and $\\gamma_{3}$ with the common tangent $A B: \\gamma_{1}$ inscribed in the triangle $A B C$, and $\\gamma_{2}$ and $\\gamma_{3}$ tangent to both (the segment) $C D$ and $\\gamma$. Prove that $\\gamma_{1}, \\gamma_{2}$, and $\\gamma_{3}$ have two common tangents.","solution":"4. Let $O$ be the midpoint of $A B$, i.e., the center of $\\gamma$. Let $O_{1}, O_{2}$, and $O_{3}$ respectively be the centers of $\\gamma_{1}, \\gamma_{2}$, and $\\gamma_{3}$ and let $r_{1}, r_{2}, r_{3}$ respectively be the radii of $\\gamma_{1}, \\gamma_{2}$ and $\\gamma_{3}$. Let $C_{1}, C_{2}$, and $C_{3}$ respectively be the points of tangency of $\\gamma_{1}, \\gamma_{2}$ and $\\gamma_{3}$ with $A B$. Let $D_{2}$ and $D_{3}$ respectively be the points of tangency of $\\gamma_{2}$ and $\\gamma_{3}$ with $C D$. Finally, let $G_{2}$ and $G_{3}$ respectively be the points of tangency of $\\gamma_{2}$ and $\\gamma_{3}$ with $\\gamma$. We have $\\mathcal{B}\\left(G_{2}, O_{2}, O\\right)$, $G_{2} O_{2}=O_{2} D_{2}$, and $G_{2} O=O B$. Hence, $G_{2}, D_{2}, B$ are collinear. Similarly, $G_{3}, D_{3}, A$ are collinear. It follows that $A G_{2} D_{2} D$ and $B G_{3} D_{3} D$ are cyclic, since $\\angle A G_{2} D_{2}=\\angle D_{2} D A=\\angle D_{3} D B=\\angle B G_{3} D_{3}=90^{\\circ}$. Hence $B C_{2}^{2}=B D_{2} \\cdot B G_{2}=B D \\cdot B A=B C^{2} \\Rightarrow B C_{2}=B C$ and hence $A C_{2}=A B-B C$. Similarly, $A C_{3}=A C$. We thus have $A C_{1}=(A C+A B-B C) \/ 2=\\left(A C_{3}+A C_{2}\\right) \/ 2$. Hence, $C_{1}$ is the midpoint of $C_{2} C_{3}$. We also have $r_{2}+r_{3}=C_{2} C_{3}=A C+B C-A B=2 r_{1}$, from which it follows that $O_{1}, O_{2}, O_{3}$ are collinear. Second solution. We shall prove the statement for arbitrary points $A, B, C$ on $\\gamma$. Let us apply the inversion $\\psi$ with respect to the circle $\\gamma_{1}$. We denote by $\\widehat{X}$ the image of an object $X$ under $\\psi$. Also, $\\psi$ maps lines $B C, C A, A B$ onto circles $\\widehat{a}, \\widehat{b}, \\widehat{c}$, respectively. Circles $\\widehat{a}, \\widehat{b}, \\widehat{c}$ pass through the center $O_{1}$ of $\\gamma_{1}$ and have radii equal to the radius of $\\widehat{\\gamma}$. Let $P, Q, R$ be the centers of $\\widehat{a}, \\widehat{b}, \\widehat{c}$ respectively. The line $C D$ maps onto a circle $k$ through $\\widehat{C}$ and $O_{1}$ that is perpendicular to $\\widehat{c}$. Therefore its center $K$ lies in the intersection of the tangent $t$ to $\\widehat{c}$ and the line $P Q$ (which bisects $\\left.\\widehat{C} O_{1}\\right)$. Let $O$ be a point such that $R O_{1} K O$ is a parallelogram and $\\gamma_{2}^{\\prime}, \\gamma_{3}^{\\prime}$ the circles centered at $O$ tangent to $k$. It is easy to see that $\\gamma_{2}^{\\prime}$ and $\\gamma_{3}^{\\prime}$ are also tangent to $\\widehat{c}$, since $O R$ and $O K$ have lengths equal to the radii of $k$ and $\\widehat{c}$. Hence $\\gamma_{2}^{\\prime}$ and $\\gamma_{3}^{\\prime}$ are the images of $\\gamma_{2}$ and $\\gamma_{3}$ under $\\psi$. Moreover, since $Q \\widehat{A} O K$ and $P \\widehat{B} O K$ are parallelograms and $Q, P, K$ are collinear, it follows that $\\widehat{A}, \\widehat{B}, O$ are also collinear. Hence the centers of $\\gamma_{1}, \\gamma_{2}, \\gamma_{3}$ are collinear, lying on the line $O_{1} O$, and the statement follows. Third solution. Moreover, the statement holds for an arbitrary point $D \\in B C$. Let $E, F, G, H$ be the points of tangency of $\\gamma_{2}$ with $A B, C D$ and of $\\gamma_{3}$ with $A B, C D$, respectively. Let $O_{i}$ be the center of $\\gamma_{i}, i=1,2,3$. As is shown in the third solution of (SL93-3), $E F$ and $G H$ meet at $O_{1}$. Hence the problem of proving the collinearity of $O_{1}, O_{2}, O_{3}$ reduces to the following simple problem: Let $D, E, F, G, H$ be points such that $D \\in E G, F \\in D H$ and $D E=D F, D G=D H$. Let $O_{1}, O_{2}, O_{3}$ be points such that $\\angle O_{2} E D=$ $\\angle O_{2} F D=90^{\\circ}, \\angle O_{3} G D=\\angle O_{3} H D=90^{\\circ}$, and $O_{1}=E F \\cap G H$. Then $O_{1}, O_{2}, O_{3}$ are collinear. Let $K_{2}=D O_{2} \\cap E F$ and $K_{3}=D O_{3} \\cap G H$. Then $O_{2} K_{2} \/ O_{2} D=$ $D K_{3} \/ D O_{3}=K_{2} O_{1} \/ D O_{3}$ and hence by Thales' theorem $O_{1} \\in O_{2} O_{3}$.","problem_type":null,"tier":0} +{"year":"1969","problem_phase":"contest","problem":"5. Given $n$ points in the plane such that no three of them are collinear, prove that one can find at least $\\binom{n-3}{2}$ convex quadrilaterals with their vertices at these points.","solution":"5. We first prove the following lemma. Lemma. If of five points in a plane no three belong to a single line, then there exist four that are the vertices of a convex quadrilateral. Proof. If the convex hull of the five points $A, B, C, D, E$ is a pentagon or a quadrilateral, the statement automatically holds. If the convex hull is a triangle, then w.l.o.g. let $\\triangle A B C$ be that triangle and $D, E$ points in its interior. Let the line $D E$ w.l.o.g. intersect $[A B]$ and $[A C]$. Then $B, C, D, E$ form the desired quadrilateral. We now observe each quintuplet of points within the set. There are $\\binom{n}{5}$ such quintuplets, and for each of them there is at least one quadruplet of points forming a convex quadrilateral. Each quadruplet, however, will be counted up to $n-4$ times. Hence we have found at least $\\frac{1}{n-4}\\binom{n}{5}$ quadruplets. Since $\\frac{1}{n-4}\\binom{n}{5} \\geq\\binom{ n-3}{2} \\Leftrightarrow(n-5)(n-6)(n+8) \\geq 0$, which always holds, it follows that we have found at least $\\binom{n-3}{2}$ desired quadruplets of points.","problem_type":"Geometry","tier":0} +{"year":"1969","problem_phase":"contest","problem":"6. Under the conditions $x_{1}, x_{2}>0, x_{1} y_{1}>z_{1}^{2}$, and $x_{2} y_{2}>z_{2}^{2}$, prove the inequality $$ \\frac{8}{\\left(x_{1}+x_{2}\\right)\\left(y_{1}+y_{2}\\right)-\\left(z_{1}+z_{2}\\right)^{2}} \\leq \\frac{1}{x_{1} y_{1}-z_{1}^{2}}+\\frac{1}{x_{2} y_{2}-z_{2}^{2}} $$","solution":"6. Define $u_{1}=\\sqrt{x_{1} y_{1}}+z_{1}, u_{2}=\\sqrt{x_{2} y_{2}}+z_{2}, v_{1}=\\sqrt{x_{1} y_{1}}-z_{1}$, and $v_{2}=$ $\\sqrt{x_{2} y_{2}}-z_{2}$. By expanding both sides of the equation we can easily verify $\\left(x_{1}+x_{2}\\right)\\left(y_{1}+y_{2}\\right)-\\left(z_{1}+z_{2}\\right)^{2}=\\left(u_{1}+u_{2}\\right)\\left(v_{1}+v_{2}\\right)+\\left(\\sqrt{x_{1} y_{2}}-\\sqrt{x_{2} y_{1}}\\right)^{2} \\geq$ $\\left(u_{1}+u_{2}\\right)\\left(v_{1}+v_{2}\\right)$. Since $x_{i} y_{i}-z_{i}^{2}=u_{i} v_{i}$ for $i=1,2$, it suffices to prove $$ \\begin{aligned} & \\frac{8}{\\left(u_{1}+u_{2}\\right)\\left(v_{1}+v_{2}\\right)} \\leq \\frac{1}{u_{1} v_{1}}+\\frac{1}{u_{2} v_{2}} \\\\ \\Leftrightarrow & 8 u_{1} u_{2} v_{1} v_{2} \\leq\\left(u_{1}+u_{2}\\right)\\left(v_{1}+v_{2}\\right)\\left(u_{1} v_{1}+u_{2} v_{2}\\right) \\end{aligned} $$ which trivially follows from the AM-GM inequalities $2 \\sqrt{u_{1} u_{2}} \\leq u_{1}+u_{2}$, $2 \\sqrt{v_{1} v_{2}} \\leq v_{1}+v_{2}$ and $2 \\sqrt{u_{1} v_{1} u_{2} v_{2}} \\leq u_{1} v_{1}+u_{2} v_{2}$. Equality holds if and only if $x_{1} y_{2}=x_{2} y_{1}, u_{1}=u_{2}$ and $v_{1}=v_{2}$, i.e. if and only if $x_{1}=x_{2}, y_{1}=y_{2}$ and $z_{1}=z_{2}$. Second solution. Let us define $f(x, y, z)=1 \/\\left(x y-z^{2}\\right)$. The problem actually states that $$ 2 f\\left(\\frac{x_{1}+x_{2}}{2}, \\frac{y_{1}+y_{2}}{2}, \\frac{z_{1}+z_{2}}{2}\\right) \\leq f\\left(x_{1}, y_{1}, z_{1}\\right)+f\\left(x_{2}, y_{2}, z_{2}\\right) $$ i.e., that the function $f$ is convex on the set $D=\\left\\{(x, y, z) \\in \\mathbb{R}^{2} \\mid x y-\\right.$ $\\left.z^{2}>0\\right\\}$. It is known that a twice continuously differentiable function $f\\left(t_{1}, t_{2}, \\ldots, t_{n}\\right)$ is convex if and only if its Hessian $\\left[f_{i j}^{\\prime \\prime}\\right]_{i, j=1}^{n}$ is positive semidefinite, or equivalently, if its principal minors $D_{k}=\\operatorname{det}\\left[f_{i j}^{\\prime \\prime}\\right]_{i, j=1}^{k}, k=$ $1,2, \\ldots, n$, are nonnegative. In the case of our $f$ this is directly verified: $D_{1}=2 y^{2} \/\\left(x y-z^{2}\\right)^{3}, D_{2}=3 x y+z^{2} \/\\left(x y-z^{2}\\right)^{5}, D_{3}=6 \/\\left(x y-z^{2}\\right)^{6}$ are obviously positive.","problem_type":null,"tier":0} +{"year":"1970","problem_phase":"shortlisted","problem":"1. (BEL 3) Consider a regular $2 n$-gon and the $n$ diagonals of it that pass through its center. Let $P$ be a point of the inscribed circle and let $a_{1}, a_{2}, \\ldots, a_{n}$ be the angles in which the diagonals mentioned are visible from the point $P$. Prove that $$ \\sum_{i=1}^{n} \\tan ^{2} a_{i}=2 n \\frac{\\cos ^{2} \\frac{\\pi}{2 n}}{\\sin ^{4} \\frac{\\pi}{2 n}} $$","solution":"1. Denote respectively by $R$ and $r$ the radii of the circumcircle and incircle, by $A_{1}, \\ldots, A_{n}, B_{1}, \\ldots, B_{n}$, the vertices of the $2 n$-gon and by $O$ its center. Let $P^{\\prime}$ be the point symmetric to $P$ with respect to $O$. Then $A_{i} P^{\\prime} B_{i} P$ is a parallelogram, and applying cosine theorem on triangles $A_{i} B_{i} P$ and $P P^{\\prime} B_{i}$ yields $$ \\begin{aligned} 4 R^{2} & =P A_{i}^{2}+P B_{i}^{2}-2 P A_{i} \\cdot P B_{i} \\cos a_{i} \\\\ 4 r^{2} & =P B_{i}^{2}+P^{\\prime} B_{i}^{2}-2 P B_{i} \\cdot P^{\\prime} B_{i} \\cos \\angle P B_{i} P^{\\prime} \\end{aligned} $$ Since $A_{i} P^{\\prime} B_{i} P$ is a parallelogram, we have that $P^{\\prime} B_{i}=P A_{i}$ and $\\angle P B_{i} P^{\\prime}=\\pi-a_{i}$. Subtracting the expression for $4 r^{2}$ from the one for $4 R^{2}$ yields $4\\left(R^{2}-r^{2}\\right)=-4 P A_{i} \\cdot P B_{i} \\cos a_{i}=-8 S_{\\triangle A_{i} B_{i} P} \\cot a_{i}$, hence we conclude that $$ \\tan ^{2} a_{i}=\\frac{4 S_{\\triangle A_{i} B_{i} P}^{2}}{\\left(R^{2}-r^{2}\\right)^{2}} $$ Denote by $M_{i}$ the foot of the perpendicular from $P$ to $A_{i} B_{i}$ and let $m_{i}=$ $P M_{i}$. Then $S_{\\triangle A_{i} B_{i} P}=R m_{i}$. Substituting this into (1) and adding up these relations for $i=1,2, \\ldots, n$, we obtain $$ \\sum_{i=1}^{n} \\tan ^{2} a_{i}=\\frac{4 R^{2}}{\\left(R^{2}-r^{2}\\right)^{2}}\\left(\\sum_{i=1}^{n} m_{i}^{2}\\right) $$ Note that all the points $M_{i}$ lie on a circle with diameter $O P$ and form a regular $n$-gon. Denote its center by $F$. We have that $m_{i}^{2}=\\left\\|\\overrightarrow{P M_{i}}\\right\\|^{2}=$ $\\left\\|\\overrightarrow{F M_{i}}-\\overrightarrow{F P}\\right\\|^{2}=\\left\\|\\overrightarrow{F M}_{i}^{2}\\right\\|+\\left\\|\\overrightarrow{F P}^{2}\\right\\|-2\\left\\langle\\overrightarrow{F M_{i}}, \\overrightarrow{F P}\\right\\rangle=r^{2} \/ 2-2\\left\\langle\\overrightarrow{F M_{i}}, \\overrightarrow{F P}\\right\\rangle$. From this it follows that $\\sum_{i=1}^{n} m_{i}^{2}=2 n(r \/ 2)^{2}-2 \\sum_{i=1}^{n}\\left\\langle\\overrightarrow{F M_{i}}, \\overrightarrow{F P}\\right\\rangle=$ $2 n(r \/ 2)^{2}-2\\left\\langle\\sum_{i=1}^{n} \\overrightarrow{F M_{i}}, \\overrightarrow{F P}\\right\\rangle=2 n(r \/ 2)^{2}$, because $\\sum_{i=1}^{n} \\overrightarrow{F M_{i}}=\\overrightarrow{0}$. Thus $$ \\sum_{i=1}^{n} \\tan ^{2} a_{i}=\\frac{4 R^{2}}{\\left(R^{2}-r^{2}\\right)^{2}} 2 n\\left(\\frac{r}{2}\\right)^{2}=2 n \\frac{(r \/ R)^{2}}{\\left(1-(r \/ R)^{2}\\right)^{2}}=2 n \\frac{\\cos ^{2} \\frac{\\pi}{2 n}}{\\sin ^{4} \\frac{\\pi}{2 n}} $$ Remark. For $n=1$ there is no regular 2-gon. However, if we think of a 2-gon as a line segment, the statement will remain true.","problem_type":null,"tier":0} +{"year":"1970","problem_phase":"shortlisted","problem":"10. (SWE 4) ${ }^{\\mathrm{IMO} 3}$ Let $1=a_{0} \\leq a_{1} \\leq a_{2} \\leq \\cdots \\leq a_{n} \\leq \\cdots$ be a sequence of real numbers. Consider the sequence $b_{1}, b_{2}, \\ldots$ defined by: $$ b_{n}=\\sum_{k=1}^{n}\\left(1-\\frac{a_{k-1}}{a_{k}}\\right) \\frac{1}{\\sqrt{a_{k}}} $$ Prove that: (a) For all natural numbers $n, 0 \\leq b_{n}<2$. (b) Given an arbitrary $0 \\leq b<2$, there is a sequence $a_{0}, a_{1}, \\ldots, a_{n}, \\ldots$ of the above type such that $b_{n}>b$ is true for infinitely many natural numbers $n$.","solution":"10. (a) Since $a_{n-1}1$, and let $a_{k}=q^{k}, k=1,2, \\ldots$. Then $\\left(1-a_{k-1} \/ a_{k}\\right) \/ \\sqrt{a_{k}}=(1-1 \/ q) \/ q^{k \/ 2}$, and consequently $$ b_{n}=\\left(1-\\frac{1}{q}\\right) \\sum_{k=1}^{n} \\frac{1}{q^{k \/ 2}}=\\frac{\\sqrt{q}+1}{q}\\left(1-\\frac{1}{q^{n \/ 2}}\\right) . $$ Since $(\\sqrt{q}+1) \/ q$ can be arbitrarily close to 2 , one can set $q$ such that $(\\sqrt{q}+1) \/ q>b$. Then $b_{n} \\geq b$ for all sufficiently large $n$. Second solution. (a) Note that $$ b_{n}=\\sum_{k=1}^{n}\\left(1-\\frac{a_{k-1}}{a_{k}}\\right) \\frac{1}{\\sqrt{a_{k}}}=\\sum_{k=1}^{n}\\left(a_{k}-a_{k-1}\\right) \\cdot \\frac{1}{a_{k}^{3 \/ 2}} $$ hence $b_{n}$ represents exactly the lower Darboux sum for the function $f(x)=x^{-3 \/ 2}$ on the interval $\\left[a_{0}, a_{n}\\right]$. Then $b_{n} \\leq \\int_{a_{0}}^{a_{n}} x^{-3 \/ 2} d x<$ $\\int_{1}^{+\\infty} x^{-3 \/ 2} d x=2$. (b) For each $b<2$ there exists a number $\\alpha>1$ such that $\\int_{1}^{\\alpha} x^{-3 \/ 2} d x>$ $b+(2-b) \/ 2$. Now, by Darboux's theorem, there exists an array $1=$ $a_{0} \\leq a_{1} \\leq \\cdots \\leq a_{n}=\\alpha$ such that the corresponding Darboux sums are arbitrarily close to the value of the integral. In particular, there is an array $a_{0}, \\ldots, a_{n}$ with $b_{n}>b$.","problem_type":null,"tier":0} +{"year":"1970","problem_phase":"shortlisted","problem":"11. (SWE 6) Let $P, Q, R$ be polynomials and let $S(x)=P\\left(x^{3}\\right)+x Q\\left(x^{3}\\right)+$ $x^{2} R\\left(x^{3}\\right)$ be a polynomial of degree $n$ whose roots $x_{1}, \\ldots, x_{n}$ are distinct. Construct with the aid of the polynomials $P, Q, R$ a polynomial $T$ of degree $n$ that has the roots $x_{1}^{3}, x_{2}^{3}, \\ldots, x_{n}^{3}$.","solution":"11. Let $S(x)=\\left(x-x_{1}\\right)\\left(x-x_{2}\\right) \\cdots\\left(x-x_{n}\\right)$. We have $x^{3}-x_{i}^{3}=\\left(x-x_{i}\\right)(\\omega x-$ $\\left.x_{i}\\right)\\left(\\omega^{2} x-x_{i}\\right)$, where $\\omega$ is a primitive third root of 1 . Multiplying these equalities for $i=1, \\ldots, n$ we obtain $$ T\\left(x^{3}\\right)=\\left(x^{3}-x_{1}^{3}\\right)\\left(x^{3}-x_{2}^{3}\\right) \\cdots\\left(x^{3}-x_{n}^{3}\\right)=S(x) S(\\omega x) S\\left(\\omega^{2} x\\right) $$ Since $S(\\omega x)=P\\left(x^{3}\\right)+\\omega x Q\\left(x^{3}\\right)+\\omega^{2} x^{2} R\\left(x^{3}\\right)$ and $S\\left(\\omega^{2} x\\right)=P\\left(x^{3}\\right)+$ $\\omega^{2} x Q\\left(x^{3}\\right)+\\omega x^{2} R\\left(x^{3}\\right)$, the above expression reduces to $$ T\\left(x^{3}\\right)=P^{3}\\left(x^{3}\\right)+x^{3} Q^{3}\\left(x^{3}\\right)+x^{6} R^{3}\\left(x^{3}\\right)-3 P\\left(x^{3}\\right) Q\\left(x^{3}\\right) R\\left(x^{3}\\right) $$ Therefore the zeros of the polynomial $$ T(x)=P^{3}(x)+x Q^{3}(x)+x^{2} R^{3}(x)-3 P(x) Q(x) R(x) $$ are exactly $x_{1}^{3}, \\ldots, x_{n}^{3}$. It is easily verified that $\\operatorname{deg} T=\\operatorname{deg} S=n$, and hence $T$ is the desired polynomial.","problem_type":null,"tier":0} +{"year":"1970","problem_phase":"shortlisted","problem":"12. (USS 4) ${ }^{\\mathrm{IMO} 6}$ We are given 100 points in the plane, no three of which are on the same line. Consider all triangles that have all vertices chosen from the 100 given points. Prove that at most $70 \\%$ of these triangles are acute angled.","solution":"12. Lemma. Five points are given in the plane such that no three of them are collinear. Then there are at least three triangles with vertices at these points that are not acute-angled. Proof. We consider three cases, according to whether the convex hull of these points is a triangle, quadrilateral, or pentagon. (i) Let a triangle $A B C$ be the convex hull and two other points $D$ and $E$ lie inside the triangle. At least two of the triangles $A D B, B D C$ and $C D A$ have obtuse angles at the point $D$. Similarly, at least two of the triangles $A E B, B E C$ and $C E A$ are obtuse-angled. Thus there are at least four non-acute-angled triangles. (ii) Suppose that $A B C D$ is the convex hull and that $E$ is a point of its interior. At least one angle of the quadrilateral is not acute, determining one non-acute-angled triangle. Also, the point $E$ lies in the interior of either $\\triangle A B C$ or $\\triangle C D A$ hence, as in the previous case, it determines another two obtuse-angled triangles. (iii) It is easy to see that at least two of the angles of the pentagon are not acute. We may assume that these two angles are among the angles corresponding to vertices $A, B$, and $C$. Now consider the quadrilateral $A C D E$. At least one its angles is not acute. Hence, there are at least three triangles that are not acute-angled. Now we consider all combinations of 5 points chosen from the given 100. There are $\\binom{100}{5}$ such combinations, and for each of them there are at least three non-acute-angled triangles with vertices in it. On the other hand, vertices of each of the triangles are counted $\\binom{97}{2}$ times. Hence there are at least $3\\binom{100}{5} \/\\binom{97}{2}$ non-acute-angled triangles with vertices in the given 100 points. Since the number of all triangles with vertices in the given points is $\\binom{100}{3}$, the ratio between the number of acute-angled triangles and the number of all triangles cannot be greater than $$ 1-\\frac{3\\binom{100}{5}}{\\binom{97}{2}\\binom{100}{3}}=0.7 $$","problem_type":null,"tier":0} +{"year":"1970","problem_phase":"shortlisted","problem":"2. (ROM 1) ${ }^{\\mathrm{IMO} 2}$ Let $a$ and $b$ be the bases of two number systems and let $$ \\begin{array}{ll} A_{n}={\\overline{x_{1} x_{2} \\ldots x_{n}}}^{(a)}, & A_{n+1}={\\overline{x_{0} x_{1} x_{2} \\ldots x_{n}}}^{(a)}, \\\\ B_{n}={\\overline{x_{1} x_{2} \\ldots x_{n}}}^{(b)}, & B_{n+1}={\\overline{x_{0} x_{1} x_{2} \\ldots x_{n}}}^{(b)}, \\end{array} $$ be numbers in the number systems with respective bases $a$ and $b$, so that $x_{0}, x_{1}, x_{2}, \\ldots, x_{n}$ denote digits in the number system with base $a$ as well as in the number system with base $b$. Suppose that neither $x_{0}$ nor $x_{1}$ is zero. Prove that $a>b$ if and only if $$ \\frac{A_{n}}{A_{n+1}}<\\frac{B_{n}}{B_{n+1}} . $$","solution":"2. Suppose that $a>b$. Consider the polynomial $P(X)=x_{1} X^{n-1}+x_{2} X^{n-2}+$ $\\cdots+x_{n-1} X+x_{n}$. We have $A_{n}=P(a), B_{n}=P(b), A_{n+1}=x_{0} a^{n}+$ $P(a)$, and $B_{n+1}=x_{0} b^{n}+P(b)$. Now $A_{n} \/ A_{n+1}b$, we have that $a^{i}>b^{i}$ and hence $x_{i} a^{n} b^{n-i} \\geq x_{i} b^{n} a^{n-i}$ (also, for $i \\geq 1$ the inequality is strict). Summing up all these inequalities for $i=1, \\ldots, n$ we get $a^{n} P(b)>b^{n} P(a)$, which completes the proof for $a>b$. On the other hand, for $aB_{n} \/ B_{n+1}$, while for $a=b$ we have equality. Thus $A_{n} \/ A_{n+1}<$ $B_{n} \/ B_{n+1} \\Leftrightarrow a>b$.","problem_type":null,"tier":0} +{"year":"1970","problem_phase":"shortlisted","problem":"3. (BUL 6) ${ }^{\\mathrm{IMO}}$ In the tetrahedron $S A B C$ the angle $B S C$ is a right angle, and the projection of the vertex $S$ to the plane $A B C$ is the intersection of the altitudes of the triangle $A B C$. Let $z$ be the radius of the inscribed circle of the triangle $A B C$. Prove that $$ S A^{2}+S B^{2}+S C^{2} \\geq 18 z^{2} $$","solution":"3 . We shall use the following lemma Lemma. If an altitude of a tetrahedron passes through the orthocenter of the opposite side, then each of the other altitudes possesses the same property. Proof. Denote the tetrahedron by $S A B C$ and let $a=B C, b=C A$, $c=A B, m=S A, n=S B, p=S C$. It is enough to prove that an altitude passes through the orthocenter of the opposite side if and only if $a^{2}+m^{2}=b^{2}+n^{2}=c^{2}+p^{2}$. Suppose that the foot $S^{\\prime}$ of the altitude from $S$ is the orthocenter of $A B C$. Then $S S^{\\prime} \\perp A B C \\Rightarrow S B^{2}-S C^{2}=S^{\\prime} B^{2}-S^{\\prime} C^{2}$. But from $A S^{\\prime} \\perp B C$ it follows that $A B^{2}-A C^{2}=S^{\\prime} B^{2}-S^{\\prime} C^{2}$. From these two equalities it can be concluded that $n^{2}-p^{2}=c^{2}-b^{2}$, or equivalently, $n^{2}+b^{2}=c^{2}+p^{2}$. Analogously, $a^{2}+m^{2}=n^{2}+b^{2}$, so we have proved the first part of the equivalence. Now suppose that $a^{2}+m^{2}=b^{2}+n^{2}=c^{2}+p^{2}$. Defining $S^{\\prime}$ as before, we get $n^{2}-p^{2}=S^{\\prime} B^{2}-S^{\\prime} C^{2}$. From the condition $n^{2}-p^{2}=c^{2}-b^{2}$ $\\left(\\Leftrightarrow b^{2}+n^{2}=c^{2}+p^{2}\\right.$ ) we conclude that $A S^{\\prime} \\perp B C$. In the same way $C S^{\\prime} \\perp A B$, which proves that $S^{\\prime}$ is the orthocenter of $\\triangle A B C$. The lemma is thus proven. Now using the lemma it is easy to see that if one of the angles at $S$ is right, than so are the others. Indeed, suppose that $\\angle A S B=\\pi \/ 2$. From the lemma we have that the altitude from $C$ passes through the orthocenter of $\\triangle A S B$, which is $S$, so $C S \\perp A S B$ and $\\angle C S A=\\angle C S B=\\pi \/ 2$. Therefore $m^{2}+n^{2}=c^{2}, n^{2}+p^{2}=a^{2}$, and $p^{2}+m^{2}=b^{2}$, so it follows that $m^{2}+n^{2}+p^{2}=\\left(a^{2}+b^{2}+c^{2}\\right) \/ 2$. By the inequality between the arithmetic and quadric means, we have that $\\left(a^{2}+b^{2}+c^{2}\\right) \/ 2 \\geq 2 s^{2} \/ 3$, where $s$ denotes the semiperimeter of $\\triangle A B C$. It remains to be shown that $2 s^{2} \/ 3 \\geq 18 r^{2}$. Since $S_{\\triangle A B C}=s r$, this is equivalent to $2 s^{4} \/ 3 \\geq$ $18 S_{A B C}^{2}=18 s(s-a)(s-b)(s-c)$ by Heron's formula. This reduces to $s^{3} \\geq 27(s-a)(s-b)(s-c)$, which is an obvious consequence of the AM-GM mean inequality. Remark. In the place of the lemma one could prove that the opposite edges of the tetrahedron are mutually perpendicular and proceed in the same way.","problem_type":null,"tier":0} +{"year":"1970","problem_phase":"shortlisted","problem":"4. (CZS 1) $)^{\\mathrm{IMO} 4}$ For what natural numbers $n$ can the product of some of the numbers $n, n+1, n+2, n+3, n+4, n+5$ be equal to the product of the remaining ones?","solution":"4. Suppose that $n$ is such a natural number. If a prime number $p$ divides any of the numbers $n, n+1, \\ldots, n+5$, then it must divide another one of them, so the only possibilities are $p=2,3,5$. Moreover, $n+1, n+2, n+3, n+4$ have no prime divisors other than 2 and 3 (if some prime number greater than 3 divides one of them, then none of the remaining numbers can have that divisor). Since two of these numbers are odd, they must be powers of 3 (greater than 1). However, there are no two powers of 3 whose difference is 2 . Therefore there is no such natural number $n$. Second solution. Obviously, none of $n, n+1, \\ldots, n+5$ is divisible by 7; hence they form a reduced system of residues. We deduce that $n(n+$ 1) $\\cdots(n+5) \\equiv 1 \\cdot 2 \\cdots 6 \\equiv-1(\\bmod 7)$. If $\\{n, \\ldots, n+5\\}$ can be partitioned into two subsets with the same products, both congruent to, say, $p$ modulo 7 , then $p^{2} \\equiv-1(\\bmod 7)$, which is impossible. Remark. Erd\u0151s has proved that a set $n, n+1, \\ldots, n+m$ of consecutive natural numbers can never be partitioned into two subsets with equal products of elements.","problem_type":null,"tier":0} +{"year":"1970","problem_phase":"shortlisted","problem":"5. (CZS 3) Let $M$ be an interior point of the tetrahedron $A B C D$. Prove that $$ \\begin{aligned} & \\overrightarrow{M A} \\operatorname{vol}(M B C D)+\\overrightarrow{M B} \\operatorname{vol}(M A C D) \\\\ & \\quad+\\overrightarrow{M C} \\operatorname{vol}(M A B D)+\\overrightarrow{M D} \\operatorname{vol}(M A B C)=0 \\end{aligned} $$ ( $\\operatorname{vol}(P Q R S)$ denotes the volume of the tetrahedron $P Q R S)$.","solution":"5. Denote respectively by $A_{1}, B_{1}, C_{1}$ and $D_{1}$ the points of intersection of the lines $A M, B M, C M$, and $D M$ with the opposite sides of the tetrahedron. Since $\\operatorname{vol}(M B C D)=\\operatorname{vol}(A B C D) \\overrightarrow{M A_{1}} \/ \\overrightarrow{A A_{1}}$, the relation we have to prove is equivalent to $$ \\overrightarrow{M A} \\cdot \\frac{\\overrightarrow{M A_{1}}}{\\overrightarrow{A A_{1}}}+\\overrightarrow{M B} \\cdot \\frac{\\overrightarrow{M B_{1}}}{\\overrightarrow{B B_{1}}}+\\overrightarrow{M C} \\cdot \\frac{\\overrightarrow{M C_{1}}}{\\overrightarrow{C C_{1}}}+\\overrightarrow{M D} \\cdot \\frac{\\overrightarrow{M D_{1}}}{\\overrightarrow{D D_{1}}}=0 $$ There exist unique real numbers $\\alpha, \\beta, \\gamma$, and $\\delta$ such that $\\alpha+\\beta+\\gamma+\\delta=1$ and for every point $O$ in space $$ \\overrightarrow{O M}=\\alpha \\overrightarrow{O A}+\\beta \\overrightarrow{O B}+\\gamma \\overrightarrow{O C}+\\delta \\overrightarrow{O D} $$ (This follows easily from $\\overrightarrow{O M}=\\overrightarrow{O A}+\\overrightarrow{A M}=\\overrightarrow{O A}+k \\overrightarrow{A B}+l \\overrightarrow{A C}+m \\overrightarrow{A D}=$ $\\overrightarrow{A B}+k(\\overrightarrow{O B}-\\overrightarrow{O A})+l(\\overrightarrow{O C}-\\overrightarrow{O A})+m(\\overrightarrow{O D}-\\overrightarrow{O A})$ for some $k, l, m \\in \\mathbb{R}$.) Further, from the condition that $A_{1}$ belongs to the plane $B C D$ we obtain for every $O$ in space the following equality for some $\\beta^{\\prime}, \\gamma^{\\prime}, \\delta^{\\prime}$ : $$ \\overrightarrow{O A_{1}}=\\beta^{\\prime} \\overrightarrow{O B}+\\gamma^{\\prime} \\overrightarrow{O C}+\\delta^{\\prime} \\overrightarrow{O D} $$ However, for $\\lambda=\\overrightarrow{M A_{1}} \/ \\overrightarrow{A A_{1}}, \\overrightarrow{O M}=\\lambda \\overrightarrow{O A}+(1-\\lambda) \\overrightarrow{O A_{1}}$; hence substituting (2) and (3) in this expression and equating coefficients for $\\overrightarrow{O A}$ we obtain $\\lambda=\\overrightarrow{M A_{1}} \/ \\overrightarrow{A A_{1}}=\\alpha$. Analogously, $\\beta=\\overrightarrow{M B_{1}} \/ \\overrightarrow{B B_{1}}, \\gamma=\\overrightarrow{M C_{1}} \/ \\overrightarrow{C C_{1}}$, and $\\delta=\\overrightarrow{M D_{1}} \/ \\overrightarrow{D D_{1}}$; hence (1) follows immediately for $O=M$. Remark. The statement of the problem actually follows from the fact that $M$ is the center of mass of the system with masses $\\operatorname{vol}(M B C D)$, $\\operatorname{vol}(M A C D), \\operatorname{vol}(M A B D), \\operatorname{vol}(M A B C)$ at $A, B, C, D$ respectively. Our proof is actually a formal verification of this fact.","problem_type":null,"tier":0} +{"year":"1970","problem_phase":"shortlisted","problem":"6. (FRA 1) In the triangle $A B C$ let $B^{\\prime}$ and $C^{\\prime}$ be the midpoints of the sides $A C$ and $A B$ respectively and $H$ the foot of the altitude passing through the vertex $A$. Prove that the circumcircles of the triangles $A B^{\\prime} C^{\\prime}, B C^{\\prime} H$, and $B^{\\prime} C H$ have a common point $I$ and that the line $H I$ passes through the midpoint of the segment $B^{\\prime} C^{\\prime}$.","solution":"6. Let $F$ be the midpoint of $B^{\\prime} C^{\\prime}, A^{\\prime}$ the midpoint of $B C$, and $I$ the intersection point of the line $H F$ and the circle circumscribed about $\\triangle B H C^{\\prime}$. Denote by $M$ the intersection point of the line $A A^{\\prime}$ with the circumscribed circle about the triangle $A B C$. Triangles $H B^{\\prime} C^{\\prime}$ and $A B C$ are similar. Since $\\angle C^{\\prime} I F=\\angle A B C=\\angle A^{\\prime} M C, \\angle C^{\\prime} F I=\\angle A A^{\\prime} B=\\angle M A^{\\prime} C$, $2 C^{\\prime} F=C^{\\prime} B^{\\prime}$, and $2 A^{\\prime} C=C B$, it follows that $\\triangle C^{\\prime} I B^{\\prime} \\sim \\triangle C M B$, hence $\\angle F I B^{\\prime}=\\angle A^{\\prime} M B=\\angle A C B$. Now one concludes that $I$ belongs to the circumscribed circles of $\\triangle A B^{\\prime} C^{\\prime}$ (since $\\left.\\angle C^{\\prime} I B^{\\prime}=180^{\\circ}-\\angle C^{\\prime} A B^{\\prime}\\right)$ and $\\triangle H C B^{\\prime}$. Second Solution. We denote the angles of $\\triangle A B C$ by $\\alpha, \\beta, \\gamma$. Evidently $\\triangle A B C \\sim \\triangle H C^{\\prime} B^{\\prime}$. Within $\\triangle H C^{\\prime} B^{\\prime}$ there exists a unique point $I$ such that $\\angle H I B^{\\prime}=180^{\\circ}-\\gamma, \\angle H I C^{\\prime}=180^{\\circ}-\\beta$, and $\\angle C^{\\prime} I B^{\\prime}=180^{\\circ}-\\alpha$, and all three circles must contain this point. Let $H I$ and $B^{\\prime} C^{\\prime}$ intersect in $F$. It remains to show that $F B^{\\prime}=F C^{\\prime}$. From $\\angle H I B^{\\prime}+\\angle H B^{\\prime} F=180^{\\circ}$ we obtain $\\angle I H B^{\\prime}=\\angle I B^{\\prime} F$. Similarly, $\\angle I H C^{\\prime}=\\angle I C^{\\prime} F$. Thus circles around $\\triangle I H C^{\\prime}$ and $\\triangle I H B^{\\prime}$ are both tangent to $B^{\\prime} C^{\\prime}$, giving us $F B^{\\prime 2}=$ $F I \\cdot F H=F C^{\\prime 2}$.","problem_type":null,"tier":0} +{"year":"1970","problem_phase":"shortlisted","problem":"7. (USS 5) For which digits $a$ do exist integers $n \\geq 4$ such that each digit of $\\frac{n(n+1)}{2}$ equals $a$ ?","solution":"7. For $a=5$ one can take $n=10$, while for $a=6$ one takes $n=11$. Now assume $a \\notin\\{5,6\\}$. If there exists an integer $n$ such that each digit of $n(n+1) \/ 2$ is equal to $a$, then there is an integer $k$ such that $n(n+1) \/ 2=\\left(10^{k}-1\\right) a \/ 9$. After multiplying both sides of the equation by 72 , one obtains $36 n^{2}+36 n=$ $8 a \\cdot 10^{k}-8 a$, which is equivalent to $$ 9(2 n+1)^{2}=8 a \\cdot 10^{k}-8 a+9 $$ So $8 a \\cdot 10^{k}-8 a+9$ is the square of some odd integer. This means that its last digit is 1,5 , or 9 . Therefore $a \\in\\{1,3,5,6,8\\}$. If $a=3$ or $a=8$, the number on the RHS of (1) is divisible by 5 , but not by 25 (for $k \\geq 2$ ), and thus cannot be a square. It remains to check the case $a=1$. In that case, (1) becomes $9(2 n+1)^{2}=8 \\cdot 10^{k}+1$, or equivalently $[3(2 n+1)-1][3(2 n+1)+1]=8 \\cdot 10^{k} \\Rightarrow(3 n+1)(3 n+2)=2 \\cdot 10^{k}$. Since the factors $3 n+1,3 n+2$ are relatively prime, this implies that one of them is $2^{k+1}$ and the other one is $5^{k}$. It is directly checked that their difference really equals 1 only for $k=1$ and $n=1$, which is excluded. Hence, the desired $n$ exists only for $a \\in\\{5,6\\}$.","problem_type":null,"tier":0} +{"year":"1970","problem_phase":"shortlisted","problem":"8. (POL 2) ${ }^{\\mathrm{IMO} 1}$ Given a point $M$ on the side $A B$ of the triangle $A B C$, let $r_{1}$ and $r_{2}$ be the radii of the inscribed circles of the triangles $A C M$ and $B C M$ respectively and let $\\rho_{1}$ and $\\rho_{2}$ be the radii of the excircles of the triangles $A C M$ and $B C M$ at the sides $A M$ and $B M$ respectively. Let $r$ and $\\rho$ denote the radii of the inscribed circle and the excircle at the side $A B$ of the triangle $A B C$ respectively. Prove that $$ \\frac{r_{1}}{\\rho_{1}} \\frac{r_{2}}{\\rho_{2}}=\\frac{r}{\\rho} $$","solution":"8. Let $A C=b, B C=a, A M=x, B M=y, C M=l$. Denote by $I_{1}$ the incenter and by $S_{1}$ the center of the excircle of $\\triangle A M C$. Suppose that $P_{1}$ and $Q_{1}$ are feet of perpendiculars from $I_{1}$ and $S_{1}$, respectively, to the line $A C$. Then $\\triangle I_{1} C P_{1} \\sim \\triangle S_{1} C Q_{1}$, hence $r_{1} \/ \\rho_{1}=C P_{1} \/ C Q_{1}$. We have $C P_{1}=(A C+M C-A M) \/ 2=(b+l-x) \/ 2$ and $C Q_{1}=$ $(A C+M C+A M) \/ 2=(b+l+x) \/ 2$. Hence $$ \\frac{r_{1}}{\\rho_{1}}=\\frac{b+l-x}{b+l+x} $$ We similarly obtain $$ \\frac{r_{2}}{\\rho_{2}}=\\frac{b+l-y}{b+l+y} \\text { and } \\frac{r}{\\rho}=\\frac{a+b-x-y}{a+b+x+y} $$ What we have to prove is now equivalent to $$ \\frac{(b+l-x)(a+l-y)}{(b+l+x)(a+l+y)}=\\frac{a+b-x-y}{a+b+x+y} $$ Multiplying both sides of (1) by $(a+l+y)(b+l+x)(a+b+x+y)$ we obtain an expression that reduces to $l^{2} x+l^{2} y+x^{2} y+x y^{2}=b^{2} y+a^{2} x$. Dividing both sides by $c=x+y$, we get that (1) is equivalent to $l^{2}=$ $b^{2} y \/(x+y)+a^{2} x \/(x+y)-x y$, which is exactly Stewart's theorem for $l$. This finally proves the desired result.","problem_type":null,"tier":0} +{"year":"1970","problem_phase":"shortlisted","problem":"9. (GDR 3) Let $u_{1}, u_{2}, \\ldots, u_{n}, v_{1}, v_{2}, \\ldots, v_{n}$ be real numbers. Prove that $$ 1+\\sum_{i=1}^{n}\\left(u_{i}+v_{i}\\right)^{2} \\leq \\frac{4}{3}\\left(1+\\sum_{i=1}^{n} u_{i}^{2}\\right)\\left(1+\\sum_{i=1}^{n} v_{i}^{2}\\right) $$ In what case does equality hold?","solution":"9. Let us set $a=\\sqrt{\\sum_{i=1}^{n} u_{i}^{2}}$ and $b=\\sqrt{\\sum_{i=1}^{n} v_{i}^{2}}$. By Minkowski's inequality (for $p=2$ ) we have $\\sum_{i=1}^{n}\\left(u_{i}+v_{i}\\right)^{2} \\leq(a+b)^{2}$. Hence the LHS of the desired inequality is not greater than $1+(a+b)^{2}$, while the RHS is equal to $4\\left(1+a^{2}\\right)\\left(1+b^{2}\\right) \/ 3$. Now it is sufficient to prove that $$ 3+3(a+b)^{2} \\leq 4\\left(1+a^{2}\\right)\\left(1+b^{2}\\right) $$ The last inequality can be reduced to the trivial $0 \\leq(a-b)^{2}+(2 a b-1)^{2}$. The equality in the initial inequality holds if and only if $u_{i} \/ v_{i}=c$ for some $c \\in \\mathbb{R}$ and $a=b=1 \/ \\sqrt{2}$.","problem_type":null,"tier":0} +{"year":"1971","problem_phase":"shortlisted","problem":"1. (BUL 2) Consider a sequence of polynomials $P_{0}(x), P_{1}(x), P_{2}(x), \\ldots$, $P_{n}(x), \\ldots$, where $P_{0}(x)=2, P_{1}(x)=x$ and for every $n \\geq 1$ the following equality holds: $$ P_{n+1}(x)+P_{n-1}(x)=x P_{n}(x) $$ Prove that there exist three real numbers $a, b, c$ such that for all $n \\geq 1$, $$ \\left(x^{2}-4\\right)\\left[P_{n}^{2}(x)-4\\right]=\\left[a P_{n+1}(x)+b P_{n}(x)+c P_{n-1}(x)\\right]^{2} $$","solution":"1. Assuming that $a, b, c$ in (1) exist, let us find what their values should be. Since $P_{2}(x)=x^{2}-2$, equation (1) for $n=1$ becomes $\\left(x^{2}-4\\right)^{2}=$ $\\left[a\\left(x^{2}-2\\right)+b x+2 c\\right]^{2}$. Therefore, there are two possibilities for $(a, b, c)$ : $(1,0,-1)$ and $(-1,0,1)$. In both cases we must prove that $$ \\left(x^{2}-4\\right)\\left[P_{n}(x)^{2}-4\\right]=\\left[P_{n+1}(x)-P_{n-1}(x)\\right]^{2} $$ It suffices to prove (2) for all $x$ in the interval $[-2,2]$. In this interval we can set $x=2 \\cos t$ for some real $t$. We prove by induction that $$ P_{n}(x)=2 \\cos n t \\quad \\text { for all } n $$ This is trivial for $n=0,1$. Assume (3) holds for some $n-1$ and $n$. Then $P_{n+1}(x)=4 \\cos t \\cos n t-2 \\cos (n-1) t=2 \\cos (n+1) t$ by the additive formula for the cosine. This completes the induction. Now (2) reduces to the obviously correct equality $$ 16 \\sin ^{2} t \\sin ^{2} n t=(2 \\cos (n+1) t-2 \\cos (n-1) t)^{2} $$ Second solution. If $x$ is fixed, the linear recurrence relation $P_{n+1}(x)+$ $P_{n-1}(x)=x P_{n}(x)$ can be solved in the standard way. The characteristic polynomial $t^{2}-x t+1$ has zeros $t_{1,2}$ with $t_{1}+t_{2}=x$ and $t_{1} t_{2}=1$; hence, the general $P_{n}(x)$ has the form $a t_{1}^{n}+b t_{2}^{n}$ for some constants $a$, $b$. From $P_{0}=2$ and $P_{1}=x$ we obtain that $$ P_{n}(x)=t_{1}^{n}+t_{2}^{n} . $$ Plugging in these values and using $t_{1} t_{2}=1$ one easily verifies (2).","problem_type":null,"tier":0} +{"year":"1971","problem_phase":"shortlisted","problem":"10. (POL 2) ${ }^{\\mathrm{IMO} 3}$ Prove that the sequence $2^{n}-3(n>1)$ contains a subsequence of numbers relatively prime in pairs.","solution":"10. We use induction. Suppose that every two of the numbers $a_{1}=2^{n_{1}}-$ $3, a_{2}=2^{n_{2}}-3, \\ldots, a_{k}=2^{n_{k}}-3$, where $2=n_{1}n_{k}$.","problem_type":null,"tier":0} +{"year":"1971","problem_phase":"shortlisted","problem":"11. (POL 3) The matrix $$ \\left(\\begin{array}{ccc} a_{11} & \\ldots & a_{1 n} \\\\ \\vdots & \\ldots & \\vdots \\\\ a_{n 1} & \\ldots & a_{n n} \\end{array}\\right) $$ satisfies the inequality $\\sum_{j=1}^{n}\\left|a_{j 1} x_{1}+\\cdots+a_{j n} x_{n}\\right| \\leq M$ for each choice of numbers $x_{i}$ equal to $\\pm 1$. Show that $$ \\left|a_{11}+a_{22}+\\cdots+a_{n n}\\right| \\leq M $$","solution":"11. We use induction. The statement for $n=1$ is trivial. Suppose that it holds for $n=k$ and consider $n=k+1$. From the given condition, we have $$ \\begin{gathered} \\sum_{j=1}^{k}\\left|a_{j, 1} x_{1}+\\cdots+a_{j, k} x_{k}+a_{j, k+1}\\right| \\\\ +\\left|a_{k+1,1} x_{1}+\\cdots+a_{k+1, k} x_{k}+a_{k+1, k+1}\\right| \\leq M \\\\ \\sum_{j=1}^{k}\\left|a_{j, 1} x_{1}+\\cdots+a_{j, k} x_{k}-a_{j, k+1}\\right| \\\\ +\\left|a_{k+1,1} x_{1}+\\cdots+a_{k+1, k} x_{k}-a_{k+1, k+1}\\right| \\leq M \\end{gathered} $$ for each choice of $x_{i}= \\pm 1$. Since $|a+b|+|a-b| \\geq 2|a|$ for all $a, b$, we obtain $$ \\begin{aligned} 2 \\sum_{j=1}^{k}\\left|a_{j 1} x_{1}+\\cdots+a_{j k} x_{k}\\right|+2\\left|a_{k+1, k+1}\\right| & \\leq 2 M, \\text { that is, } \\\\ \\sum_{j=1}^{k}\\left|a_{j 1} x_{1}+\\cdots+a_{j k} x_{k}\\right| & \\leq M-\\left|a_{k+1, k+1}\\right| \\end{aligned} $$ Now by the inductive assumption $\\sum_{j=1}^{k}\\left|a_{j j}\\right| \\leq M-\\left|a_{k+1, k+1}\\right|$, which is equivalent to the desired inequality.","problem_type":null,"tier":0} +{"year":"1971","problem_phase":"shortlisted","problem":"12. (POL 6) Two congruent equilateral triangles $A B C$ and $A^{\\prime} B^{\\prime} C^{\\prime}$ in the plane are given. Show that the midpoints of the segments $A A^{\\prime}, B B^{\\prime}, C C^{\\prime}$ either are collinear or form an equilateral triangle.","solution":"12. Let us start with the case $A=A^{\\prime}$. If the triangles $A B C$ and $A^{\\prime} B^{\\prime} C^{\\prime}$ are oppositely oriented, then they are symmetric with respect to some axis, and the statement is true. Suppose that they are equally oriented. There is a rotation around $A$ by $60^{\\circ}$ that maps $A B B^{\\prime}$ onto $A C C^{\\prime}$. This rotation also maps the midpoint $B_{0}$ of $B B^{\\prime}$ onto the midpoint $C_{0}$ of $C C^{\\prime}$, hence the triangle $A B_{0} C_{0}$ is equilateral. In the general case, when $A \\neq A^{\\prime}$, let us denote by $T$ the translation that maps $A$ onto $A^{\\prime}$. Let $X^{\\prime}$ be the image of a point $X$ under the (unique) isometry mapping $A B C$ onto $A^{\\prime} B^{\\prime} C^{\\prime}$, and $X^{\\prime \\prime}$ the image of $X$ under $T$. Furthermore, let $X_{0}, X_{0}^{\\prime}$ be the midpoints of segments $X X^{\\prime}, X^{\\prime} X^{\\prime \\prime}$. Then $X_{0}$ is the image of $X_{0}^{\\prime}$ under the translation $-(1 \/ 2) T$. However, since it has already been proven that the triangle $A_{0}^{\\prime} B_{0}^{\\prime} C_{0}^{\\prime}$ is equilateral, its image $A_{0} B_{0} C_{0}$ under $(1 \/ 2) T$ is also equilateral. The statement of the problem is thus proven.","problem_type":null,"tier":0} +{"year":"1971","problem_phase":"shortlisted","problem":"13. (SWE 5) ${ }^{\\mathrm{IMO} 6}$ Consider the $n \\times n$ array of nonnegative integers $$ \\left(\\begin{array}{cccc} a_{11} & a_{12} & \\ldots & a_{1 n} \\\\ a_{21} & a_{22} & \\ldots & a_{2 n} \\\\ \\vdots & \\vdots & & \\vdots \\\\ a_{n 1} & a_{n 2} & \\ldots & a_{n n} \\end{array}\\right) $$ with the following property: If an element $a_{i j}$ is zero, then the sum of the elements of the $i$ th row and the $j$ th column is greater than or equal to $n$. Prove that the sum of all the elements is greater than or equal to $\\frac{1}{2} n^{2}$.","solution":"13. Let $p$ be the least of all the sums of elements in one row or column. If $p \\geq n \/ 2$, then the sum of all elements of the array is $s \\geq n p \\geq n^{2} \/ 2$. Now suppose that $p1248$. For each segment $l_{i}=A_{i} A_{i+1}$ of the broken line, consider the figure $V_{i}$ obtained by a circle of radius 1 whose center moves along it, and let $\\overline{V_{i}}$ be obtained by cutting off the circle of radius 1 with center at the starting point of $l_{i}$. The area of $\\overline{V_{i}}$ is equal to $2 A_{i} A_{i+1}$. It is clear that the union of all the figures $\\overline{V_{i}}$ together with a semicircle with center in $A_{1}$ and a semicircle with center in $A_{n}$ contains $V$ completely. Therefore $$ S(V) \\leq \\pi+2 A_{1} A_{2}+2 A_{2} A_{3}+\\cdots+2 A_{n-1} A_{n}=\\pi+2 L . $$ This completes the proof.","problem_type":null,"tier":0} +{"year":"1971","problem_phase":"shortlisted","problem":"15. (USS 2) Natural numbers from 1 to 99 (not necessarily distinct) are written on 99 cards. It is given that the sum of the numbers on any subset of cards (including the set of all cards) is not divisible by 100. Show that all the cards contain the same number.","solution":"15. Assume the opposite. Then one can numerate the cards 1 to 99, with a number $n_{i}$ written on the card $i$, so that $n_{98} \\neq n_{99}$. Denote by $x_{i}$ the remainder of $n_{1}+n_{2}+\\cdots+n_{i}$ upon division by 100 , for $i=1,2, \\ldots, 99$. All $x_{i}$ must be distinct: Indeed, if $x_{i}=x_{j}, i0, i=1,2,3,4$.","solution":"17. We use the following obvious consequences of $(a+b)^{2} \\geq 4 a b$ : $$ \\begin{aligned} & \\frac{1}{\\left(a_{1}+a_{2}\\right)\\left(a_{3}+a_{4}\\right)} \\geq \\frac{4}{\\left(a_{1}+a_{2}+a_{3}+a_{4}\\right)^{2}} \\\\ & \\frac{1}{\\left(a_{1}+a_{4}\\right)\\left(a_{2}+a_{3}\\right)} \\geq \\frac{4}{\\left(a_{1}+a_{2}+a_{3}+a_{4}\\right)^{2}} \\end{aligned} $$ Now we have $$ \\begin{aligned} & \\frac{a_{1}+a_{3}}{a_{1}+a_{2}}+\\frac{a_{2}+a_{4}}{a_{2}+a_{3}}+\\frac{a_{3}+a_{1}}{a_{3}+a_{4}}+\\frac{a_{4}+a_{2}}{a_{4}+a_{1}} \\\\ = & \\frac{\\left(a_{1}+a_{3}\\right)\\left(a_{1}+a_{2}+a_{3}+a_{4}\\right)}{\\left(a_{1}+a_{2}\\right)\\left(a_{3}+a_{4}\\right)}+\\frac{\\left(a_{2}+a_{4}\\right)\\left(a_{1}+a_{2}+a_{3}+a_{4}\\right)}{\\left(a_{1}+a_{4}\\right)\\left(a_{2}+a_{3}\\right)} \\\\ \\geq & \\frac{4\\left(a_{1}+a_{3}\\right)}{a_{1}+a_{2}+a_{3}+a_{4}}+\\frac{4\\left(a_{2}+a_{4}\\right)}{a_{1}+a_{2}+a_{3}+a_{4}}=4 . \\end{aligned} $$","problem_type":null,"tier":0} +{"year":"1971","problem_phase":"shortlisted","problem":"2. (BUL 5) ${ }^{\\mathrm{IMO} 5}$ Prove that for every natural number $m \\geq 1$ there exists a finite set $S_{m}$ of points in the plane satisfying the following condition: If $A$ is any point in $S_{m}$, then there are exactly $m$ points in $S_{m}$ whose distance to $A$ equals 1.","solution":"2 . We will construct such a set $S_{m}$ of $2^{m}$ points. Take vectors $u_{1}, \\ldots, u_{m}$ in a given plane, such that $\\left|u_{i}\\right|=1 \/ 2$ and $0 \\neq\\left|c_{1} u_{1}+c_{2} u_{2}+\\cdots+c_{n} u_{n}\\right| \\neq 1 \/ 2$ for any choice of numbers $c_{i}$ equal to 0 or $\\pm 1$. Such vectors are easily constructed by induction on $m$ : For $u_{1}, \\ldots, u_{m-1}$ fixed, there are only finitely many vector values $u_{m}$ that violate the upper condition, and we may set $u_{m}$ to be any other vector of length $1 \/ 2$. Let $S_{m}$ be the set of all points $M_{0}+\\varepsilon_{1} u_{1}+\\varepsilon_{2} u_{2}+\\cdots+\\varepsilon_{m} u_{m}$, where $M_{0}$ is any fixed point in the plane and $\\varepsilon_{i}= \\pm 1$ for $i=1, \\ldots, m$. Then $S_{m}$ obviously satisfies the condition of the problem.","problem_type":null,"tier":0} +{"year":"1971","problem_phase":"shortlisted","problem":"3. (GDR 1) Knowing that the system $$ \\begin{aligned} x+y+z & =3, \\\\ x^{3}+y^{3}+z^{3} & =15, \\\\ x^{4}+y^{4}+z^{4} & =35, \\end{aligned} $$ has a real solution $x, y, z$ for which $x^{2}+y^{2}+z^{2}<10$, find the value of $x^{5}+y^{5}+z^{5}$ for that solution.","solution":"3. Let $x, y, z$ be a solution of the given system with $x^{2}+y^{2}+z^{2}=\\alpha<10$. Then $$ x y+y z+z x=\\frac{(x+y+z)^{2}-\\left(x^{2}+y^{2}+z^{2}\\right)}{2}=\\frac{9-\\alpha}{2} . $$ Furthermore, $3 x y z=x^{3}+y^{3}+z^{3}-(x+y+z)\\left(x^{2}+y^{2}+z^{2}-x y-y z-z x\\right)$, which gives us $x y z=3(9-\\alpha) \/ 2-4$. We now have $$ \\begin{aligned} 35= & x^{4}+y^{4}+z^{4}=\\left(x^{3}+y^{3}+z^{3}\\right)(x+y+z) \\\\ & -\\left(x^{2}+y^{2}+z^{2}\\right)(x y+y z+z x)+x y z(x+y+z) \\\\ = & 45-\\frac{\\alpha(9-\\alpha)}{2}+\\frac{9(9-\\alpha)}{2}-12 . \\end{aligned} $$ The solutions in $\\alpha$ are $\\alpha=7$ and $\\alpha=11$. Therefore $\\alpha=7$, xyz $=-1$, $x y+x z+y z=1$, and $$ \\begin{aligned} x^{5}+y^{5}+z^{5}= & \\left(x^{4}+y^{4}+z^{4}\\right)(x+y+z) \\\\ & -\\left(x^{3}+y^{3}+z^{3}\\right)(x y+x z+y z)+x y z\\left(x^{2}+y^{2}+z^{2}\\right) \\\\ = & 35 \\cdot 3-15 \\cdot 1+7 \\cdot(-1)=83 . \\end{aligned} $$","problem_type":null,"tier":0} +{"year":"1971","problem_phase":"shortlisted","problem":"4. (GBR 3) We are given two mutually tangent circles in the plane, with radii $r_{1}, r_{2}$. A line intersects these circles in four points, determining three segments of equal length. Find this length as a function of $r_{1}$ and $r_{2}$ and the condition for the solvability of the problem.","solution":"4. In the coordinate system in which the $x$-axis passes through the centers of the circles and the $y$-axis is their common tangent, the circles have equations $$ x^{2}+y^{2}+2 r_{1} x=0, \\quad x^{2}+y^{2}-2 r_{2} x=0 . $$ Let $p$ be the desired line with equation $y=a x+b$. The abscissas of points of intersection of $p$ with both circles satisfy one of $$ \\left(1+a^{2}\\right) x^{2}+2\\left(a b+r_{1}\\right) x+b^{2}=0, \\quad\\left(1+a^{2}\\right) x^{2}+2\\left(a b-r_{2}\\right) x+b^{2}=0 $$ Let us denote the lengths of the chords and their projections onto the $x$-axis by $d$ and $d_{1}$, respectively. From these equations it follows that $$ d_{1}^{2}=\\frac{4\\left(a b+r_{1}\\right)^{2}}{\\left(1+a^{2}\\right)^{2}}-\\frac{4 b^{2}}{1+a^{2}}=\\frac{4\\left(a b-r_{2}\\right)^{2}}{\\left(1+a^{2}\\right)^{2}}-\\frac{4 b^{2}}{1+a^{2}} $$ Consider the point of intersection of $p$ with the $y$-axis. This point has equal powers with respect to both circles. Hence, if that point divides the segment determined on $p$ by the two circles on two segments of lengths $x$ and $y$, this power equals $x(x+d)=y(y+d)$, which implies $x=y=d \/ 2$. Thus each of the equations in (1) has two roots, one of which is thrice the other. This fact gives us $\\left(a b+r_{1}\\right)^{2}=4\\left(1+a^{2}\\right) b^{2} \/ 3$. From (1) and this we obtain $$ \\begin{gathered} a b=\\frac{r_{2}-r_{1}}{2}, \\quad 4 b^{2}+a^{2} b^{2}=3\\left[\\left(a b+r_{1}\\right)^{2}-a^{2} b^{2}\\right]=3 r_{1} r_{2} \\\\ a^{2}=\\frac{4\\left(r_{2}-r_{1}\\right)^{2}}{14 r_{1} r_{2}-r_{1}^{2}-r_{2}^{2}}, \\quad b^{2}=\\frac{14 r_{1} r_{2}-r_{1}^{2}-r_{2}^{2}}{16} \\\\ d_{1}^{2}=\\frac{\\left(14 r_{1} r_{2}-r_{1}^{2}-r_{2}^{2}\\right)^{2}}{36\\left(r_{1}+r_{2}\\right)^{2}} \\end{gathered} $$ Finally, since $d^{2}=d_{1}^{2}\\left(1+a^{2}\\right)$, we conclude that $$ d^{2}=\\frac{1}{12}\\left(14 r_{1} r_{2}-r_{1}^{2}-r_{2}^{2}\\right), $$ and that the problem is solvable if and only if $7-4 \\sqrt{3} \\leq \\frac{r_{1}}{r_{2}} \\leq 7+4 \\sqrt{3}$.","problem_type":null,"tier":0} +{"year":"1971","problem_phase":"shortlisted","problem":"5. (HUN 1) ${ }^{\\mathrm{IMO}}$ Let $a, b, c, d, e$ be real numbers. Prove that the expression $$ \\begin{gathered} (a-b)(a-c)(a-d)(a-e)+(b-a)(b-c)(b-d)(b-e)+(c-a)(c-b)(c-d)(c-e) \\\\ +(d-a)(d-b)(d-c)(d-e)+(e-a)(e-b)(e-c)(e-d) \\end{gathered} $$ is nonnegative.","solution":"5. Without loss of generality, we may assume that $a \\geq b \\geq c \\geq d \\geq e$. Then $a-b=-(b-a) \\geq 0, a-c \\geq b-c \\geq 0, a-d \\geq b-d \\geq 0$ and $a-e \\geq b-e \\geq 0$, and hence $$ (a-b)(a-c)(a-d)(a-e)+(b-a)(b-c)(b-d)(b-e) \\geq 0 $$ Analogously, $(d-a)(d-b)(d-c)(d-e)+(e-a)(e-b)(e-c)(e-d) \\geq 0$. Finally, $(c-a)(c-b)(c-d)(c-e) \\geq 0$ as a product of two nonnegative numbers, from which the inequality stated in the problem follows. Remark. The problem in an alternative formulation, accepted for the IMO, asked to prove that the analogous inequality $$ \\begin{gathered} \\left(a_{1}-a_{2}\\right)\\left(a_{1}-a_{2}\\right) \\cdots\\left(a_{1}-a_{n}\\right)+\\left(a_{2}-a_{1}\\right)\\left(a_{2}-a_{3}\\right) \\cdots\\left(a_{2}-a_{n}\\right)+\\cdots \\\\ +\\left(a_{n}-a_{1}\\right)\\left(a_{n}-a_{2}\\right) \\cdots\\left(a_{n}-a_{n-1}\\right) \\geq 0 \\end{gathered} $$ holds for arbitrary real numbers $a_{i}$ if and only if $n=3$ or $n=5$. The case $n=3$ is analogous to $n=5$. For $n=4$, a counterexample is $a_{1}=0, a_{2}=a_{3}=a_{4}=1$, while for $n>5$ one can take $a_{1}=a_{2}=\\cdots=$ $a_{n-4}=0, a_{n-3}=a_{n-2}=a_{n-1}=2, a_{n}=1$ as a counterexample.","problem_type":null,"tier":0} +{"year":"1971","problem_phase":"shortlisted","problem":"6. (HUN 7) Let $n \\geq 2$ be a natural number. Find a way to assign natural numbers to the vertices of a regular $2^{n}$-gon such that the following conditions are satisfied: (1) only digits 1 and 2 are used; (2) each number consists of exactly $n$ digits; (3) different numbers are assigned to different vertices; (4) the numbers assigned to two neighboring vertices differ at exactly one digit.","solution":"6. The proof goes by induction on $n$. For $n=2$, the following numeration satisfies the conditions (a)-(d): $C_{1}=11, C_{2}=12, C_{3}=22, C_{4}=21$. Suppose that $n>2$, and that the numeration $C_{1}, C_{2}, \\ldots, C_{2^{n-1}}$ of a regular $2^{n-1}$-gon, in cyclical order, satisfies (i)-(iv). Then one can assign to the vertices of a $2^{n}$-gon cyclically the following numbers: $$ \\overline{1 C_{1}}, \\overline{1 C_{2}}, \\ldots, \\overline{1 C_{2^{n-1}}}, \\overline{2 C_{2^{n-1}}}, \\ldots, \\overline{2 C_{2}}, \\overline{2 C_{1}} $$ The conditions (i), (ii) obviously hold, while (iii) and (iv) follow from the inductive assumption.","problem_type":null,"tier":0} +{"year":"1971","problem_phase":"shortlisted","problem":"7. (NET 1) ${ }^{\\mathrm{IMO} \/}$ Given a tetrahedron $A B C D$ whose all faces are acuteangled triangles, set $$ \\sigma=\\measuredangle D A B+\\measuredangle B C D-\\measuredangle A B C-\\measuredangle C D A $$ Consider all closed broken lines $X Y Z T X$ whose vertices $X, Y, Z, T$ lie in the interior of segments $A B, B C, C D, D A$ respectively. Prove that: (a) if $\\sigma \\neq 0$, then there is no broken line $X Y Z T$ of minimal length; (b) if $\\sigma=0$, then there are infinitely many such broken lines of minimal length. That length equals $2 A C \\sin (\\alpha \/ 2)$, where $$ \\alpha=\\measuredangle B A C+\\measuredangle C A D+\\measuredangle D A B $$","solution":"7. (a) Suppose that $X, Y, Z$ are fixed on segments $A B, B C, C D$. It is proven in a standard way that if $\\angle A T X \\neq \\angle Z T D$, then $Z T+T X$ can be reduced. It follows that if there exists a broken line $X Y Z T X$ of minimal length, then the following conditions hold: $$ \\begin{aligned} & \\angle D A B=\\pi-\\angle A T X-\\angle A X T \\\\ & \\angle A B C=\\pi-\\angle B X Y-\\angle B Y X=\\pi-\\angle A X T-\\angle C Y Z \\\\ & \\angle B C D=\\pi-\\angle C Y Z-\\angle C Z Y \\\\ & \\angle C D A=\\pi-\\angle D T Z-\\angle D Z T=\\pi-\\angle A T X-\\angle C Z Y . \\end{aligned} $$ Thus $\\sigma=0$. (b) Now let $\\sigma=0$. Let us cut the surface of the tetrahedron along the edges $A C, C D$, and $D B$ and set it down into a plane. Consider the plane figure $\\mathcal{S}=A C D^{\\prime} B D^{\\prime \\prime} C^{\\prime}$ thus obtained made up of triangles $B C D^{\\prime}, A B C, A B D^{\\prime \\prime}$, and $A C^{\\prime} D^{\\prime \\prime}$, with $Z^{\\prime}, T^{\\prime}, Z^{\\prime \\prime}$ respectively on $C D^{\\prime}, A D^{\\prime \\prime}, C^{\\prime} D^{\\prime \\prime}$ (here $C^{\\prime}$ corresponds to $C$, etc.). Since $\\angle C^{\\prime} D^{\\prime \\prime} A+\\angle D^{\\prime \\prime} A B+\\angle A B C+\\angle B C D^{\\prime}=0$ as an oriented angle (because $\\sigma=0$ ), the lines $C D^{\\prime}$ and $C^{\\prime} D^{\\prime \\prime}$ are parallel and equally oriented; i.e., $C D^{\\prime} D^{\\prime \\prime} C^{\\prime}$ is a parallelogram. The broken line $X Y Z T X$ has minimal length if and only if $Z^{\\prime \\prime}, T^{\\prime}, X$, $Y, Z^{\\prime}$ are collinear (where $Z^{\\prime} Z^{\\prime \\prime} \\|$ $C C^{\\prime}$ ), and then this length equals $Z^{\\prime} Z^{\\prime \\prime}=C C^{\\prime}=2 A C \\sin (\\alpha \/ 2)$. There is an infinity of such lines, one for every line $Z^{\\prime} Z^{\\prime \\prime}$ parallel to $C C^{\\prime}$ that meets the interiors of all the segments $C B, B A, A D^{\\prime \\prime}$. Such ![](https:\/\/cdn.mathpix.com\/cropped\/2024_11_18_8e985d6b9c83aa3e9d0eg-389.jpg?height=300&width=516&top_left_y=408&top_left_x=835) $Z^{\\prime} Z^{\\prime \\prime}$ exist. Indeed, the triangles $C A B$ and $D^{\\prime \\prime} A B$ are acute-angled, and thus the segment $A B$ has a common interior point with the parallelogram $C D^{\\prime} D^{\\prime \\prime} C^{\\prime}$. Therefore the desired result follows.","problem_type":null,"tier":0} +{"year":"1971","problem_phase":"shortlisted","problem":"8. (NET 4) Determine whether there exist distinct real numbers $a, b, c, t$ for which: (i) the equation $a x^{2}+b t x+c=0$ has two distinct real roots $x_{1}, x_{2}$, (ii) the equation $b x^{2}+c t x+a=0$ has two distinct real roots $x_{2}, x_{3}$, (iii) the equation $c x^{2}+a t x+b=0$ has two distinct real roots $x_{3}, x_{1}$.","solution":"8. Suppose that $a, b, c, t$ satisfy all the conditions. Then $a b c \\neq 0$ and $$ x_{1} x_{2}=\\frac{c}{a}, \\quad x_{2} x_{3}=\\frac{a}{b}, \\quad x_{3} x_{1}=\\frac{b}{c} . $$ Multiplying these equations, we obtain $x_{1}^{2} x_{2}^{2} x_{3}^{2}=1$, and hence $x_{1} x_{2} x_{3}=$ $\\varepsilon= \\pm 1$. From (1) we get $x_{1}=\\varepsilon b \/ a, x_{2}=\\varepsilon c \/ b, x_{3}=\\varepsilon a \/ c$. Substituting $x_{1}$ in the first equation, we get $a b^{2} \/ a^{2}+t \\varepsilon b^{2} \/ a+c=0$, which gives us $$ b^{2}(1+t \\varepsilon)=-a c . $$ Analogously, $c^{2}(1+t \\varepsilon)=-a b$ and $a^{2}(1+t \\varepsilon)=-b c$, and therefore $(1+$ $t \\varepsilon)^{3}=-1$; i.e., $1+t \\varepsilon=-1$, since it is real. This also implies together with (1) that $b^{2}=a c, c^{2}=a b$, and $a^{2}=b c$, and consequently $$ a=b=c \\text {. } $$ Thus the three equations in the problem are equal, which is impossible. Hence, such $a, b, c, t$ do not exist.","problem_type":null,"tier":0} +{"year":"1971","problem_phase":"shortlisted","problem":"9. (POL 1) Let $T_{k}=k-1$ for $k=1,2,3,4$ and $$ T_{2 k-1}=T_{2 k-2}+2^{k-2}, \\quad T_{2 k}=T_{2 k-5}+2^{k} \\quad(k \\geq 3) $$ Show that for all $k$, $$ 1+T_{2 n-1}=\\left[\\frac{12}{7} 2^{n-1}\\right] \\quad \\text { and } \\quad 1+T_{2 n}=\\left[\\frac{17}{7} 2^{n-1}\\right] $$ where $[x]$ denotes the greatest integer not exceeding $x$.","solution":"9. We use induction. Since $T_{1}=0, T_{2}=1, T_{3}=2, T_{4}=3, T_{5}=5, T_{6}=8$, the statement is true for $n=1,2,3$. Suppose that both formulas from the problem hold for some $n \\geq 3$. Then $$ \\begin{aligned} & T_{2 n+1}=1+T_{2 n}+2^{n-1}=\\left[\\frac{17}{7} 2^{n-1}+2^{n-1}\\right]=\\left[\\frac{12}{7} 2^{n}\\right] \\\\ & T_{2 n+2}=1+T_{2 n-3}+2^{n+1}=\\left[\\frac{12}{7} 2^{n-2}+2^{n+1}\\right]=\\left[\\frac{17}{7} 2^{n}\\right] . \\end{aligned} $$ Therefore the formulas hold for $n+1$, which completes the proof.","problem_type":null,"tier":0} +{"year":"1972","problem_phase":"shortlisted","problem":"1. (BUL 7) ${ }^{\\mathrm{IMO} 5}$ Let $f$ and $\\varphi$ be real functions defined on the set $\\mathbb{R}$ satisfying the functional equation $$ f(x+y)+f(x-y)=2 \\varphi(y) f(x) $$ for arbitrary real $x, y$ (give examples of such functions). Prove that if $f(x)$ is not identically 0 and $|f(x)| \\leq 1$ for all $x$, then $|\\varphi(x)| \\leq 1$ for all $x$.","solution":"1. Suppose that $f\\left(x_{0}\\right) \\neq 0$ and for a given $y$ define the sequence $x_{k}$ by the formula $$ x_{k+1}= \\begin{cases}x_{k}+y, & \\text { if }\\left|f\\left(x_{k}+y\\right)\\right| \\geq\\left|f\\left(x_{k}-y\\right)\\right| \\\\ x_{k}-y, & \\text { otherwise. }\\end{cases} $$ It follows from (1) that $\\left|f\\left(x_{k+1}\\right)\\right| \\geq|\\varphi(y)|\\left|f\\left(x_{k}\\right)\\right|$; hence by induction, $\\left|f\\left(x_{k}\\right)\\right| \\geq|\\varphi(y)|^{k}\\left|f\\left(x_{0}\\right)\\right|$. Since $\\left|f\\left(x_{k}\\right)\\right| \\leq 1$ for all $k$, we obtain $|\\varphi(y)| \\leq 1$. Second solution. Let $M=\\sup f(x) \\leq 1$, and $x_{k}$ any sequence, possibly constant, such that $f\\left(x_{k}\\right) \\rightarrow M, k \\rightarrow \\infty$. Then for all $k$, $$ |\\varphi(y)|=\\frac{\\left|f\\left(x_{k}+y\\right)+f\\left(x_{k}-y\\right)\\right|}{2\\left|f\\left(x_{k}\\right)\\right|} \\leq \\frac{2 M}{2\\left|f\\left(x_{k}\\right)\\right|} \\rightarrow 1, \\quad k \\rightarrow \\infty $$","problem_type":null,"tier":0} +{"year":"1972","problem_phase":"shortlisted","problem":"10. (NET 3) ${ }^{\\text {IMO2 }}$ Prove that for each $n \\geq 4$ every cyclic quadrilateral can be decomposed into $n$ cyclic quadrilaterals.","solution":"10. Consider first a triangle. It can be decomposed into $k=3$ cyclic quadrilaterals by perpendiculars from some interior point of it to the sides; also, it can be decomposed into a cyclic quadrilateral and a triangle, and it follows by induction that this decomposition is possible for every $k$. Since every triangle can be cut into two triangles, the required decomposition is possible for each $n \\geq 6$. It remains to treat the cases $n=4$ and $n=5$. $n=4$. If the center $O$ of the circumcircle is inside a cyclic quadrilateral $A B C D$, then the required decomposition is effected by perpendiculars from $O$ to the four sides. Otherwise, let $C$ and $D$ be the vertices of the obtuse angles of the quadrilateral. Draw the perpendiculars at $C$ and $D$ to the lines $B C$ and $A D$ respectively, and choose points $P$ and $Q$ on them such that $P Q \\| A B$. Then the required decomposition is effected by $C P, P Q, Q D$ and the perpendiculars from $P$ and $Q$ to $A B$. $n=5$. If $A B C D$ is an isosceles trapezoid with $A B \\| C D$ and $A D=B C$, then it is trivially decomposed by lines parallel to $A B$. Otherwise, $A B C D$ can be decomposed into a cyclic quadrilateral and a trapezoid; this trapezoid can be cut into an isosceles trapezoid and a triangle, which can further be cut into three cyclic quadrilaterals and an isosceles trapezoid. Remark. It can be shown that the assertion is not true for $n=2$ and $n=3$.","problem_type":null,"tier":0} +{"year":"1972","problem_phase":"shortlisted","problem":"11. (NET 6) Consider a sequence of circles $K_{1}, K_{2}, K_{3}, K_{4}, \\ldots$ of radii $r_{1}, r_{2}, r_{3}, r_{4}, \\ldots$, respectively, situated inside a triangle $A B C$. The circle $K_{1}$ is tangent to $A B$ and $A C ; K_{2}$ is tangent to $K_{1}, B A$, and $B C ; K_{3}$ is tangent to $K_{2}, C A$, and $C B ; K_{4}$ is tangent to $K_{3}, A B$, and $A C$; etc. (a) Prove the relation $$ r_{1} \\cot \\frac{1}{2} A+2 \\sqrt{r_{1} r_{2}}+r_{2} \\cot \\frac{1}{2} B=r\\left(\\cot \\frac{1}{2} A+\\cot \\frac{1}{2} B\\right), $$ where $r$ is the radius of the incircle of the triangle $A B C$. Deduce the existence of a $t_{1}$ such that $$ r_{1}=r \\cot \\frac{1}{2} B \\cot \\frac{1}{2} C \\sin ^{2} t_{1} . $$ (b) Prove that the sequence of circles $K_{1}, K_{2}, \\ldots$ is periodic.","solution":"11. Let $\\angle A=2 x, \\angle B=2 y, \\angle C=2 z$. (a) Denote by $M_{i}$ the center of $K_{i}, i=1,2, \\ldots$ If $N_{1}, N_{2}$ are the projections of $M_{1}, M_{2}$ onto $A B$, we have $A N_{1}=r_{1} \\cot x, N_{2} B=r_{2} \\cot y$, and $N_{1} N_{2}=\\sqrt{\\left(r_{1}+r_{2}\\right)^{2}-\\left(r_{1}-r_{2}\\right)^{2}}=2 \\sqrt{r_{1} r_{2}}$. The required relation between $r_{1}, r_{2}$ follows from $A B=A N_{1}+N_{1} N_{2}+N_{2} B$. If this relation is further considered as a quadratic equation in $\\sqrt{r_{2}}$, then its discriminant, which equals $$ \\Delta=4\\left(r(\\cot x+\\cot y) \\cot y-r_{1}(\\cot x \\cot y-1)\\right), $$ must be nonnegative, and therefore $r_{1} \\leq r \\cot y \\cot z$. Then $t_{1}, t_{2}, \\ldots$ exist, and we can assume that $t_{i} \\in[0, \\pi \/ 2]$. (b) Substituting $r_{1}=r \\cot y \\cot z \\sin ^{2} t_{1}, r_{2}=r \\cot z \\cot x \\sin ^{2} t_{2}$ in the relation of (a) we obtain that $\\sin ^{2} t_{1}+\\sin ^{2} t_{2}+k^{2}+2 k \\sin t_{1} \\sin t_{2}=1$, where we set $k=\\sqrt{\\tan x \\tan y}$. It follows that $\\left(k+\\sin t_{1} \\sin t_{2}\\right)^{2}=$ $\\left(1-\\sin ^{2} t_{1}\\right)\\left(1-\\sin ^{2} t_{2}\\right)=\\cos ^{2} t_{1} \\cos ^{2} t_{2}$, and hence $$ \\cos \\left(t_{1}+t_{2}\\right)=\\cos t_{1} \\cos t_{2}-\\sin t_{1} \\sin t_{2}=k=\\sqrt{\\tan x \\tan y} $$ which is constant. Writing the analogous relations for each $t_{i}, t_{i+1}$ we conclude that $t_{1}+t_{2}=t_{4}+t_{5}, t_{2}+t_{3}=t_{5}+t_{6}$, and $t_{3}+t_{4}=t_{6}+t_{7}$. It follows that $t_{1}=t_{7}$, i.e., $K_{1}=K_{7}$.","problem_type":null,"tier":0} +{"year":"1972","problem_phase":"shortlisted","problem":"12. (USS 2) ${ }^{\\mathrm{IMO} 1} \\mathrm{~A}$ set of 10 positive integers is given such that the decimal expansion of each of them has two digits. Prove that there are two disjoint subsets of the set with equal sums of their elements.","solution":"12. First we observe that it is not essential to require the subsets to be disjoint (if they aren't, one simply excludes their intersection). There are $2^{10}-1=$ 1023 different subsets and at most 990 different sums. By the pigeonhole principle there are two different subsets with equal sums.","problem_type":null,"tier":0} +{"year":"1972","problem_phase":"shortlisted","problem":"2. (CZS 1) We are given $3 n$ points $A_{1}, A_{2}, \\ldots, A_{3 n}$ in the plane, no three of them collinear. Prove that one can construct $n$ disjoint triangles with vertices at the points $A_{i}$.","solution":"2. We use induction. For $n=1$ the assertion is obvious. Assume that it is true for a positive integer $n$. Let $A_{1}, A_{2}, \\ldots, A_{3 n+3}$ be given $3 n+3$ points, and let w.l.o.g. $A_{1} A_{2} \\ldots A_{m}$ be their convex hull. Among all the points $A_{i}$ distinct from $A_{1}, A_{2}$, we choose the one, say $A_{k}$, for which the angle $\\angle A_{k} A_{1} A_{2}$ is minimal (this point is uniquely determined, since no three points are collinear). The line $A_{1} A_{k}$ separates the plane into two half-planes, one of which contains $A_{2}$ only, and the other one all the remaining $3 n$ points. By the inductive hypothesis, one can construct $n$ disjoint triangles with vertices in these $3 n$ points. Together with the triangle $A_{1} A_{2} A_{k}$, they form the required system of disjoint triangles.","problem_type":null,"tier":0} +{"year":"1972","problem_phase":"shortlisted","problem":"3. (CZS 4) Let $x_{1}, x_{2}, \\ldots, x_{n}$ be real numbers satisfying $x_{1}+x_{2}+\\cdots+x_{n}=$ 0 . Let $m$ be the least and $M$ the greatest among them. Prove that $$ x_{1}^{2}+x_{2}^{2}+\\cdots+x_{n}^{2} \\leq-n m M $$","solution":"3. We have for each $k=1,2, \\ldots, n$ that $m \\leq x_{k} \\leq M$, which gives $(M-$ $\\left.x_{k}\\right)\\left(m-x_{k}\\right) \\leq 0$. It follows directly that $$ 0 \\geq \\sum_{k=1}^{n}\\left(M-x_{k}\\right)\\left(m-x_{k}\\right)=n m M-(m+M) \\sum_{k=1}^{n} x_{k}+\\sum_{k=1}^{n} x_{k}^{2} $$ But $\\sum_{k=1}^{n} x_{k}=0$, implying the required inequality.","problem_type":null,"tier":0} +{"year":"1972","problem_phase":"shortlisted","problem":"4. (GDR 1) Let $n_{1}, n_{2}$ be positive integers. Consider in a plane $E$ two disjoint sets of points $M_{1}$ and $M_{2}$ consisting of $2 n_{1}$ and $2 n_{2}$ points, respectively, and such that no three points of the union $M_{1} \\cup M_{2}$ are collinear. Prove that there exists a straightline $g$ with the following property: Each of the two half-planes determined by $g$ on $E$ ( $g$ not being included in either) contains exactly half of the points of $M_{1}$ and exactly half of the points of $M_{2}$.","solution":"4. Choose in $E$ a half-line $s$ beginning at a point $O$. For every $\\alpha$ in the interval $\\left[0,180^{\\circ}\\right]$, denote by $s(\\alpha)$ the line obtained by rotation of $s$ about $O$ by $\\alpha$, and by $g(\\alpha)$ the oriented line containing $s(\\alpha)$ on which $s(\\alpha)$ defines the positive direction. For each $P$ in $M_{i}, i=1,2$, let $P(\\alpha)$ be the foot of the perpendicular from $P$ to $g(\\alpha)$, and $l_{P}(\\alpha)$ the oriented (positive, negative or zero) distance of $P(\\alpha)$ from $O$. Then for $i=1,2$ one can arrange the $l_{P}(\\alpha)\\left(P \\in M_{i}\\right)$ in ascending order, as $l_{1}(\\alpha), l_{2}(\\alpha), \\ldots, l_{2 n_{i}}(\\alpha)$. Call $J_{i}(\\alpha)$ the interval $\\left[l_{n_{i}}(\\alpha), l_{n_{i}+1}(\\alpha)\\right]$. It is easy to see that any line perpendicular to $g(\\alpha)$ and passing through the point with the distance $l$ in the interior of $J_{i}(\\alpha)$ from $O$, will divide the set $M_{i}$ into two subsets of equal cardinality. Therefore it remains to show that for some $\\alpha$, the interiors of intervals $J_{1}(\\alpha)$ and $J_{2}(\\alpha)$ have a common point. If this holds for $\\alpha=0$, then we have finished. Suppose w.l.o.g. that $J_{1}(0)$ lies on $g(0)$ to the left of $J_{2}(0)$; then $J_{1}\\left(180^{\\circ}\\right)$ lies to the right of $J_{2}\\left(180^{\\circ}\\right)$. Note that $J_{1}$ and $J_{2}$ cannot simultaneously degenerate to a point (otherwise, we would have four collinear points in $M_{1} \\cup M_{2}$ ); also, each of them degenerates to a point for only finitely many values of $\\alpha$. Since $J_{1}(\\alpha)$ and $J_{2}(\\alpha)$ move continuously, there exists a subinterval $I$ of $\\left[0,180^{\\circ}\\right]$ on which they are not disjoint. Thus, at some point of $I$, they are both nondegenerate and have a common interior point, as desired.","problem_type":null,"tier":0} +{"year":"1972","problem_phase":"shortlisted","problem":"5. (GDR 2) Prove the following assertion: The four altitudes of a tetrahedron $A B C D$ intersect in a point if and only if $$ A B^{2}+C D^{2}=B C^{2}+A D^{2}=C A^{2}+B D^{2} $$","solution":"5. Lemma. If $X, Y, Z, T$ are points in space, then the lines $X Z$ and $Y T$ are perpendicular if and only if $X Y^{2}+Z T^{2}=Y Z^{2}+T X^{2}$. Proof. Consider the plane $\\pi$ through $X Z$ parallel to $Y T$. If $Y^{\\prime}, T^{\\prime}$ are the feet of the perpendiculars to $\\pi$ from $Y, T$ respectively, then $$ \\begin{aligned} & \\\\ & X Y^{2}+Z T^{2}=X Y^{\\prime 2}+Z T^{\\prime 2}+2 Y Y^{\\prime 2} \\\\ & \\text { and } \\quad Y Z^{2}+T X^{2}=Y^{\\prime} Z^{2}+T^{\\prime} X^{2}+2 Y Y^{\\prime 2} \\end{aligned} $$ Since by the Pythagorean theorem $X Y^{\\prime 2}+Z T^{\\prime 2}=Y^{\\prime} Z^{2}+T^{\\prime} X^{2}$, i.e., $X Y^{\\prime 2}-Y^{\\prime} Z^{2}=X T^{\\prime 2}-T^{\\prime} Z^{2}$, if and only if $Y^{\\prime} T^{\\prime} \\perp X Z$, the statement follows. Assume that the four altitudes intersect in a point $P$. Then we have $D P \\perp$ $A B C \\Rightarrow D P \\perp A B$ and $C P \\perp A B D \\Rightarrow C P \\perp A B$, which implies that $C D P \\perp A B$, and $C D \\perp A B$. By the lemma, $A C^{2}+B D^{2}=A D^{2}+B C^{2}$. Using the same procedure we obtain the relation $A D^{2}+B C^{2}=A B^{2}+$ $C D^{2}$ \u3002 Conversely, assume that $A B^{2}+C D^{2}=A C^{2}+B D^{2}=A D^{2}+B C^{2}$. The lemma implies that $A B \\perp C D, A C \\perp B D, A D \\perp B C$. Let $\\pi$ be the plane containing $C D$ that is perpendicular to $A B$, and let $h_{D}$ be the altitude from $D$ to $A B C$. Since $\\pi \\perp A B$, we have $\\pi \\perp A B C \\Rightarrow h_{D} \\subset \\pi$ and $\\pi \\perp A B D \\Rightarrow h_{C} \\subset \\pi$. The altitudes $h_{D}$ and $h_{C}$ are not parallel; thus they have an intersection point $P_{C D}$. Analogously, $h_{B} \\cap h_{C}=\\left\\{P_{B C}\\right\\}$ and $h_{B} \\cap h_{D}=\\left\\{P_{B D}\\right\\}$, where both these points belong to $\\pi$. On the other hand, $h_{B}$ doesn't belong to $\\pi$; otherwise, it would be perpendicular to both $A C D$ and $A B \\subset \\pi$, i.e. $A B \\subset A C D$, which is impossible. Hence, $h_{B}$ can have at most one common point with $\\pi$, implying $P_{B D}=P_{C D}$. Analogously, $P_{A B}=P_{B D}=P_{C D}=P_{A B C D}$.","problem_type":null,"tier":0} +{"year":"1972","problem_phase":"shortlisted","problem":"6. (GDR 3) Show that for any $n \\not \\equiv 0(\\bmod 10)$ there exists a multiple of $n$ not containing the digit 0 in its decimal expansion.","solution":"6. Let $n=2^{\\alpha} 5^{\\beta} m$, where $\\alpha=0$ or $\\beta=0$. These two cases are analogous, and we treat only $\\alpha=0, n=5^{\\beta} m$. The case $m=1$ is settled by the following lemma. Lemma. For any integer $\\beta \\geq 1$ there exists a multiple $M_{\\beta}$ of $5^{\\beta}$ with $\\beta$ digits in decimal expansion, all different from 0. Proof. For $\\beta=1, M_{1}=5$ works. Assume that the lemma is true for $\\beta=k$. There is a positive integer $C_{k} \\leq 5$ such that $C_{k} 2^{k}+m_{k} \\equiv$ $0(\\bmod 5)$, where $5^{k} m_{k}=M_{k}$, i.e. $C_{k} 10^{k}+M_{k} \\equiv 0\\left(\\bmod 5^{k+1}\\right)$. Then $M_{k+1}=C_{k} 10^{k}+M_{k}$ satisfies the conditions, and proves the lemma. In the general case, consider, the sequence $1,10^{\\beta}, 10^{2 \\beta}, \\ldots$ It contains two numbers congruent modulo $\\left(10^{\\beta}-1\\right) m$, and therefore for some $k>0$, $10^{k \\beta} \\equiv 1\\left(\\bmod \\left(10^{\\beta}-1\\right) m\\right)$ (this is in fact a consequence of Fermat's theorem). The number $$ \\frac{10^{k \\beta}-1}{10^{\\beta}-1} M_{\\beta}=10^{(k-1) \\beta} M_{\\beta}+10^{(k-2) \\beta} M_{\\beta}+\\cdots+M_{\\beta} $$ is a multiple of $n=5^{\\beta} m$ with the required property.","problem_type":null,"tier":0} +{"year":"1972","problem_phase":"shortlisted","problem":"7. $(\\mathbf{G B R} 1)^{\\mathrm{IMO} 6}$ (a) A plane $\\pi$ passes through the vertex $O$ of the regular tetrahedron $O P Q R$. We define $p, q, r$ to be the signed distances of $P, Q, R$ from $\\pi$ measured along a directed normal to $\\pi$. Prove that $$ p^{2}+q^{2}+r^{2}+(q-r)^{2}+(r-p)^{2}+(p-q)^{2}=2 a^{2} $$ where $a$ is the length of an edge of a tetrahedron. (b) Given four parallel planes not all of which are coincident, show that a regular tetrahedron exists with a vertex on each plane.","solution":"7. (i) Consider the circumscribing cube $O Q_{1} P R_{1} O_{1} Q P_{1} R$ (that is, the cube in which the edges of the tetrahedron are small diagonals), of side $b=a \\sqrt{2} \/ 2$. The left-hand side is the sum of squares of the projections of the edges of the tetrahedron onto a perpendicular $l$ to $\\pi$. On the other hand, if $l$ ![](https:\/\/cdn.mathpix.com\/cropped\/2024_11_18_8e985d6b9c83aa3e9d0eg-395.jpg?height=363&width=333&top_left_y=609&top_left_x=936) forms angles $\\varphi_{1}, \\varphi_{2}, \\varphi_{3}$ with $O O_{1}, O Q_{1}, O R_{1}$ respectively, then the projections of $O P$ and $Q R$ onto $l$ have lengths $b\\left(\\cos \\varphi_{2}+\\cos \\varphi_{3}\\right)$ and $b\\left|\\cos \\varphi_{2}-\\cos \\varphi_{3}\\right|$. Summing up all these expressions, we obtain $$ 4 b^{2}\\left(\\cos ^{2} \\varphi_{1}+\\cos ^{2} \\varphi_{2}+\\cos ^{2} \\varphi_{3}\\right)=4 b^{2}=2 a^{2} $$ (ii) We construct a required tetrahedron of edge length $a$ given in (i). Take $O$ arbitrarily on $\\pi_{0}$, and let $p, q, r$ be the distances of $O$ from $\\pi_{1}, \\pi_{2}, \\pi_{3}$. Since $a>p, q, r,|p-q|$, we can choose $P$ on $\\pi_{1}$ anywhere at distance $a$ from $O$, and $Q$ at one of the two points on $\\pi_{2}$ at distance $a$ from both $O$ and $P$. Consider the fourth vertex of the tetrahedron: its distance from $\\pi_{0}$ will satisfy the equation from (i); i.e., there are two values for this distance; clearly, one of them is $r$, putting $R$ on $\\pi_{3}$.","problem_type":null,"tier":0} +{"year":"1972","problem_phase":"shortlisted","problem":"8. (GBR 2) ${ }^{\\mathrm{IMO} 3}$ Let $m$ and $n$ be nonnegative integers. Prove that $m!n!(m+$ $n)$ ! divides $(2 m)!(2 n)!$.","solution":"8. Let $f(m, n)=\\frac{(2 m)!(2 n)!}{m!n!(m+n)!}$. Then it is directly shown that $$ f(m, n)=4 f(m, n-1)-f(m+1, n-1), $$ and thus $n$ may be successively reduced until one obtains $f(m, n)=$ $\\sum_{r} c_{r} f(r, 0)$. Now $f(r, 0)$ is a simple binomial coefficient, and the $c_{r}$ 's are integers. Second solution. For each prime $p$, the greatest exponents of $p$ that divide the numerator $(2 m)!(2 n)$ ! and denominator $m!n!(m+n)$ ! are respectively $$ \\sum_{k>0}\\left(\\left[\\frac{2 m}{p^{k}}\\right]+\\left[\\frac{2 n}{p^{k}}\\right]\\right) \\quad \\text { and } \\quad \\sum_{k>0}\\left(\\left[\\frac{m}{p^{k}}\\right]+\\left[\\frac{n}{p^{k}}\\right]+\\left[\\frac{m+n}{p^{k}}\\right]\\right) $$ hence it suffices to show that the first exponent is not less than the second one for every $p$. This follows from the fact that for each real $x,[2 x]+[2 y] \\geq$ $[x]+[y]+[x+y]$, which is straightforward to prove (for example, using $[2 x]=[x]+[x+1 \/ 2])$.","problem_type":null,"tier":0} +{"year":"1972","problem_phase":"shortlisted","problem":"9. (NET 2) $)^{\\mathrm{IMO} 4}$ Find all solutions in positive real numbers $x_{i}(i=$ $1,2,3,4,5)$ of the following system of inequalities: $$ \\begin{aligned} & \\left(x_{1}^{2}-x_{3} x_{5}\\right)\\left(x_{2}^{2}-x_{3} x_{5}\\right) \\leq 0, \\\\ & \\left(x_{2}^{2}-x_{4} x_{1}\\right)\\left(x_{3}^{2}-x_{4} x_{1}\\right) \\leq 0, \\\\ & \\left(x_{3}^{2}-x_{5} x_{2}\\right)\\left(x_{4}^{2}-x_{5} x_{2}\\right) \\leq 0, \\\\ & \\left(x_{4}^{2}-x_{1} x_{3}\\right)\\left(x_{5}^{2}-x_{1} x_{3}\\right) \\leq 0, \\\\ & \\left(x_{5}^{2}-x_{2} x_{4}\\right)\\left(x_{1}^{2}-x_{2} x_{4}\\right) \\leq 0 . \\end{aligned} $$","solution":"9. Clearly $x_{1}=x_{2}=x_{3}=x_{4}=x_{5}$ is a solution. We shall show that this describes all solutions. Suppose that not all $x_{i}$ are equal. Then among $x_{3}, x_{5}, x_{2}, x_{4}, x_{1}$ two consecutive are distinct: Assume w.l.o.g. that $x_{3} \\neq x_{5}$. Moreover, since $\\left(1 \/ x_{1}, \\ldots, 1 \/ x_{5}\\right)$ is a solution whenever $\\left(x_{1}, \\ldots, x_{5}\\right)$ is, we may assume that $x_{3}x_{3}$. Then $x_{5}^{2}>x_{1} x_{3}$, which together with (iv) gives $x_{4}^{2} \\leq x_{1} x_{3}x_{5} x_{3}$, a contradiction. Consider next the case $x_{1}>x_{2}$. We infer from (i) that $x_{1} \\geq \\sqrt{x_{3} x_{5}}>x_{3}$ and $x_{2} \\leq \\sqrt{x_{3} x_{5}}x_{2}$. Second solution. $$ \\begin{aligned} 0 & \\geq L_{1}=\\left(x_{1}^{2}-x_{3} x_{5}\\right)\\left(x_{2}^{2}-x_{3} x_{5}\\right)=x_{1}^{2} x_{2}^{2}+x_{3}^{2} x_{5}^{2}-\\left(x_{1}^{2}+x_{2}^{2}\\right) x_{3} x_{5} \\\\ & \\geq x_{1}^{2} x_{2}^{2}+x_{3}^{2} x_{5}^{2}-\\frac{1}{2}\\left(x_{1}^{2} x_{3}^{2}+x_{1}^{2} x_{5}^{2}+x_{2}^{2} x_{3}^{2}+x_{2}^{2} x_{5}^{2}\\right) \\end{aligned} $$ and analogously for $L_{2}, \\ldots, L_{5}$. Therefore $L_{1}+L_{2}+L_{3}+L_{4}+L_{5} \\geq 0$, with the only case of equality $x_{1}=x_{2}=x_{3}=x_{4}=x_{5}$.","problem_type":null,"tier":0} +{"year":"1973","problem_phase":"shortlisted","problem":"1. (BUL 6) Let a tetrahedron $A B C D$ be inscribed in a sphere $S$. Find the locus of points $P$ inside the sphere $S$ for which the equality $$ \\frac{A P}{P A_{1}}+\\frac{B P}{P B_{1}}+\\frac{C P}{P C_{1}}+\\frac{D P}{P D_{1}}=4 $$ holds, where $A_{1}, B_{1}, C_{1}$, and $D_{1}$ are the intersection points of $S$ with the lines $A P, B P, C P$, and $D P$, respectively.","solution":"1. The condition of the point $P$ can be written in the form $\\frac{A P^{2}}{A P \\cdot P A_{1}}+\\frac{B P^{2}}{B P \\cdot P B_{1}}+$ $\\frac{C P^{2}}{C P \\cdot P C_{1}}+\\frac{D P^{2}}{D P \\cdot P D_{1}}=4$. All the four denominators are equal to $R^{2}-O P^{2}$, i.e., to the power of $P$ with respect to $S$. Thus the condition becomes $$ A P^{2}+B P^{2}+C P^{2}+D P^{2}=4\\left(R^{2}-O P^{2}\\right) $$ Let $M$ and $N$ be the midpoints of segments $A B$ and $C D$ respectively, and $G$ the midpoint of $M N$, or the centroid of $A B C D$. By Stewart's formula, an arbitrary point $P$ satisfies $$ \\begin{aligned} A P^{2}+B P^{2}+C P^{2}+D P^{2} & =2 M P^{2}+2 N P^{2}+\\frac{1}{2} A B^{2}+\\frac{1}{2} C D^{2} \\\\ & =4 G P^{2}+M N^{2}+\\frac{1}{2}\\left(A B^{2}+C D^{2}\\right) \\end{aligned} $$ Particularly, for $P \\equiv O$ we get $4 R^{2}=4 O G^{2}+M N^{2}+\\frac{1}{2}\\left(A B^{2}+C D^{2}\\right)$, and the above equality becomes $$ A P^{2}+B P^{2}+C P^{2}+D P^{2}=4 G P^{2}+4 R^{2}-4 O G^{2} $$ Therefore (1) is equivalent to $O G^{2}=O P^{2}+G P^{2} \\Leftrightarrow \\angle O P G=90^{\\circ}$. Hence the locus of points $P$ is the sphere with diameter $O G$. Now the converse is easy.","problem_type":null,"tier":0} +{"year":"1973","problem_phase":"shortlisted","problem":"10. (SWE 3) ${ }^{\\mathrm{IMO} 6}$ Let $a_{1}, a_{2}, \\ldots, a_{n}$ be positive numbers and $q$ a given real number, $00 $$ we analogously obtain $q b_{k+1}-b_{k}<0$. Finally, $$ \\begin{aligned} b_{1}+b_{2}+\\cdots+b_{n}= & a_{1}\\left(q^{n-1}+\\cdots+q+1\\right)+\\ldots \\\\ & +a_{k}\\left(q^{n-k}+\\cdots+q+1+q+\\cdots+q^{k-1}\\right)+\\ldots \\\\ \\leq & \\left(a_{1}+a_{2}+\\cdots+a_{n}\\right)\\left(1+2 q+2 q^{2}+\\cdots+2 q^{n-1}\\right) \\\\ < & \\frac{1+q}{1-q}\\left(a_{1}+\\cdots+a_{n}\\right) \\end{aligned} $$","problem_type":null,"tier":0} +{"year":"1973","problem_phase":"shortlisted","problem":"11. (SWE 4) ${ }^{\\mathrm{IMO} 3}$ Determine the minimum of $a^{2}+b^{2}$ if $a$ and $b$ are real numbers for which the equation $$ x^{4}+a x^{3}+b x^{2}+a x+1=0 $$ has at least one real solution.","solution":"11. Putting $x+\\frac{1}{x}=t$ we also get $x^{2}+\\frac{1}{x^{2}}=t^{2}-2$, and the given equation reduces to $t^{2}+a t+b-2=0$. Since $x=\\frac{t \\pm \\sqrt{t^{2}-4}}{2}, x$ will be real if and only if $|t| \\geq 2, t \\in \\mathbb{R}$. Thus we need the minimum value of $a^{2}+b^{2}$ under the condition $a t+b=-\\left(t^{2}-2\\right),|t| \\geq 2$. However, by the Cauchy-Schwarz inequality we have $$ \\left(a^{2}+b^{2}\\right)\\left(t^{2}+1\\right) \\geq(a t+b)^{2}=\\left(t^{2}-2\\right)^{2} $$ It follows that $a^{2}+b^{2} \\geq h(t)=\\frac{\\left(t^{2}-2\\right)^{2}}{t^{2}+1}$. Since $h(t)=\\left(t^{2}+1\\right)+\\frac{9}{t^{2}+1}-6$ is increasing for $t \\geq 2$, we conclude that $a^{2}+b^{2} \\geq h(2)=\\frac{4}{5}$. The cases of equality are easy to examine: These are $a= \\pm \\frac{4}{5}$ and $b=-\\frac{2}{5}$. Second solution. In fact, there was no need for considering $x=t+1 \/ t$. By the Cauchy-Schwarz inequality we have $\\left(a^{2}+2 b^{2}+a^{2}\\right)\\left(x^{6}+x^{4} \/ 2+x^{2}\\right) \\geq$ $\\left(a x^{3}+b x^{2}+a x\\right)^{2}=\\left(x^{4}+1\\right)^{2}$. Hence $$ a^{2}+b^{2} \\geq \\frac{\\left(x^{4}+1\\right)^{2}}{2 x^{6}+x^{4}+2 x^{2}} \\geq \\frac{4}{5} $$ with equality for $x=1$.","problem_type":null,"tier":0} +{"year":"1973","problem_phase":"shortlisted","problem":"12. (SWE 6) Consider the two square matrices $$ A=\\left[\\begin{array}{rrrrr} 1 & 1 & 1 & 1 & 1 \\\\ 1 & 1 & 1 & -1 & -1 \\\\ 1 & -1 & -1 & 1 & 1 \\\\ 1 & -1 & -1 & -1 & 1 \\\\ 1 & 1 & -1 & 1 & -1 \\end{array}\\right] \\quad \\text { and } \\quad B=\\left[\\begin{array}{rrrrr} 1 & 1 & 1 & 1 & 1 \\\\ 1 & 1 & 1 & -1 & -1 \\\\ 1 & 1 & -1 & 1 & -1 \\\\ 1 & -1 & -1 & 1 & 1 \\\\ 1 & -1 & 1 & -1 & 1 \\end{array}\\right] $$ with entries 1 and -1 . The following operations will be called elementary: (1) Changing signs of all numbers in one row; (2) Changing signs of all numbers in one column; (3) Interchanging two rows (two rows exchange their positions); (4) Interchanging two columns. Prove that the matrix $B$ cannot be obtained from the matrix $A$ using these operations.","solution":"12. Observe that the absolute values of the determinants of the given matrices are invariant under all the admitted operations. The statement follows from $\\operatorname{det} A=16 \\neq \\operatorname{det} B=0$.","problem_type":null,"tier":0} +{"year":"1973","problem_phase":"shortlisted","problem":"13. (YUG 4) Find the sphere of maximal radius that can be placed inside every tetrahedron that has all altitudes of length greater than or equal to 1.","solution":"13. Let $S_{1}, S_{2}, S_{3}, S_{4}$ denote the areas of the faces of the tetrahedron, $V$ its volume, $h_{1}, h_{2}, h_{3}, h_{4}$ its altitudes, and $r$ the radius of its inscribed sphere. Since $$ 3 V=S_{1} h_{1}=S_{2} h_{2}=S_{3} h_{3}=S_{4} h_{4}=\\left(S_{1}+S_{2}+S_{3}+S_{4}\\right) r, $$ it follows that $$ \\frac{1}{h_{1}}+\\frac{1}{h_{2}}+\\frac{1}{h_{3}}+\\frac{1}{h_{4}}=\\frac{1}{r} . $$ In our case, $h_{1}, h_{2}, h_{3}, h_{4} \\geq 1$, hence $r \\geq 1 \/ 4$. On the other hand, it is clear that a sphere of radius greater than $1 \/ 4$ cannot be inscribed in a tetrahedron all of whose altitudes have length equal to 1 . Thus the answer is $1 \/ 4$.","problem_type":null,"tier":0} +{"year":"1973","problem_phase":"shortlisted","problem":"14. (YUG 5) ${ }^{\\mathrm{IMO} 4} \\mathrm{~A}$ soldier has to investigate whether there are mines in an area that has the form of an equilateral triangle. The radius of his detector is equal to one-half of an altitude of the triangle. The soldier starts from one vertex of the triangle. Determine the shortest path that the soldier has to traverse in order to check the whole region.","solution":"14. Suppose that the soldier starts at the vertex $A$ of the equilateral triangle $A B C$ of side length $a$. Let $\\varphi, \\psi$ be the arcs of circles with centers $B$ and $C$ and radii $a \\sqrt{3} \/ 4$ respectively, that lie inside the triangle. In order to check the vertices $B, C$, he must visit some points $D \\in \\varphi$ and $E \\in \\psi$. ![](https:\/\/cdn.mathpix.com\/cropped\/2024_11_18_8e985d6b9c83aa3e9d0eg-401.jpg?height=324&width=394&top_left_y=1496&top_left_x=887) Thus his path cannot be shorter than the path $A D E$ (or $A E D$ ) itself. The length of the path $A D E$ is $A D+D E \\geq A D+D C-a \\sqrt{3} \/ 4$. Let $F$ be the reflection of $C$ across the line $M N$, where $M, N$ are the midpoints of $A B$ and $B C$. Then $D C \\geq D F$ and hence $A D+D C \\geq A D+D F \\geq A F$. Consequently $A D+D E \\geq A F-a \\frac{\\sqrt{3}}{4}=a\\left(\\frac{\\sqrt{7}}{2}-\\frac{\\sqrt{3}}{4}\\right)$, with equality if and only if $D$ is the midpoint of $\\operatorname{arc} \\varphi$ and $E=(C D) \\cap \\psi$. Moreover, it is easy to verify that, in following the path $A D E$, the soldier will check the whole region. Therefore this path (as well as the one symmetric to it) is shortest possible path that the soldier can take in order to check the entire field.","problem_type":null,"tier":0} +{"year":"1973","problem_phase":"shortlisted","problem":"15. (CUB 1) Prove that for all $n \\in \\mathbb{N}$ the following is true: $$ 2^{n} \\prod_{k=1}^{n} \\sin \\frac{k \\pi}{2 n+1}=\\sqrt{2 n+1} $$","solution":"15. If $z=\\cos \\theta+i \\sin \\theta$, then $z-z^{-1}=2 i \\sin \\theta$. Now put $z=\\cos \\frac{\\pi}{2 n+1}+$ $i \\sin \\frac{\\pi}{2 n+1}$. Using de Moivre's formula we transform the required equality into $$ A=\\prod_{k=1}^{n}\\left(z^{k}-z^{-k}\\right)=i^{n} \\sqrt{2 n+1} $$ On the other hand, the complex numbers $z^{2 k}(k=-n,-n+1, \\ldots, n)$ are the roots of $x^{2 n+1}-1$, and hence $$ \\prod_{k=1}^{n}\\left(x-z^{2 k}\\right)\\left(x-z^{-2 k}\\right)=\\frac{x^{2 n+1}-1}{x-1}=x^{2 n}+\\cdots+x+1 $$ Now we go back to proving (1). We have $$ (-1)^{n} z^{n(n+1) \/ 2} A=\\prod_{k=1}^{n}\\left(1-z^{2 k}\\right) \\quad \\text { and } \\quad z^{-n(n+1) \/ 2} A=\\prod_{k=1}^{n}\\left(1-z^{-2 k}\\right) $$ Multiplying these two equalities, we obtain $(-1)^{n} A^{2}=\\prod_{k=1}^{n}\\left(1-z^{2 k}\\right)(1-$ $\\left.z^{-2 k}\\right)=2 n+1$, by (2). Therefore $A= \\pm i^{-n} \\sqrt{2 n+1}$. This actually implies that the required product is $\\pm \\sqrt{2 n+1}$, but it must be positive, since all the sines are, and the result follows.","problem_type":null,"tier":0} +{"year":"1973","problem_phase":"shortlisted","problem":"16. (CUB 2) Given $a, \\theta \\in \\mathbb{R}, m \\in \\mathbb{N}$, and $P(x)=x^{2 m}-2|a|^{m} x^{m} \\cos \\theta+a^{2 m}$, factorize $P(x)$ as a product of $m$ real quadratic polynomials.","solution":"16. First, we have $P(x)=Q(x) R(x)$ for $Q(x)=x^{m}-|a|^{m} e^{i \\theta}$ and $R(x)=$ $x^{m}-|a|^{m} e^{-i \\theta}$, where $e^{i \\varphi}$ means of course $\\cos \\varphi+i \\sin \\varphi$. It remains to factor both $Q$ and $R$. Suppose that $Q(x)=\\left(x-q_{1}\\right) \\cdots\\left(x-q_{m}\\right)$ and $R(x)=\\left(x-r_{1}\\right) \\cdots\\left(x-r_{m}\\right)$. Considering $Q(x)$, we see that $\\left|q_{k}^{m}\\right|=|a|^{m}$ and also $\\left|q_{k}\\right|=|a|$ for $k=$ $1, \\ldots, m$. Thus we may put $q_{k}=|a| e^{i \\beta_{k}}$ and obtain by de Moivre's formula $q_{k}^{m}=|a|^{m} e^{i m \\beta_{k}}$. It follows that $m \\beta_{k}=\\theta+2 j \\pi$ for some $j \\in \\mathbb{Z}$, and we have exactly $m$ possibilities for $\\beta_{k}$ modulo $2 \\pi$ : $\\beta_{k}=\\frac{\\theta+2(k-1) \\pi}{m}$ for $k=1,2, \\ldots, m$. Thus $q_{k}=|a| e^{i \\beta_{k}}$; analogously we obtain for $R(x)$ that $r_{k}=|a| e^{-i \\beta_{k}}$. Consequently, $x^{m}-|a|^{m} e^{i \\theta}=\\prod_{k=1}^{m}\\left(x-|a| e^{i \\beta_{k}}\\right) \\quad$ and $\\quad x^{m}-|a|^{m} e^{-i \\theta}=\\prod_{k=1}^{m}\\left(x-|a| e^{-i \\beta_{k}}\\right)$. Finally, grouping the $k$ th factors of both polynomials, we get $$ \\begin{aligned} P(x) & =\\prod_{k=1}^{m}\\left(x-|a| e^{i \\beta_{k}}\\right)\\left(x-|a| e^{-i \\beta_{k}}\\right)=\\prod_{k=1}^{m}\\left(x^{2}-2|a| x \\cos \\beta_{k}+a^{2}\\right) \\\\ & =\\prod_{k=1}^{m}\\left(x^{2}-2|a| x \\cos \\frac{\\theta+2(k-1) \\pi}{m}+a^{2}\\right) \\end{aligned} $$","problem_type":null,"tier":0} +{"year":"1973","problem_phase":"shortlisted","problem":"17. (POL 1) ${ }^{\\mathrm{IMO} 5}$ Let $\\mathcal{F}$ be a nonempty set of functions $f: \\mathbb{R} \\rightarrow \\mathbb{R}$ of the form $f(x)=a x+b$, where $a$ and $b$ are real numbers and $a \\neq 0$. Suppose that $\\mathcal{F}$ satisfies the following conditions: (1) If $f, g \\in \\mathcal{F}$, then $g \\circ f \\in \\mathcal{F}$, where $(g \\circ f)(x)=g[f(x)]$. (2) If $f \\in \\mathcal{F}$ and $f(x)=a x+b$, then the inverse $f^{-1}$ of $f$ belongs to $\\mathcal{F}$ $\\left(f^{-1}(x)=(x-b) \/ a\\right)$. (3) None of the functions $f(x)=x+c$, for $c \\neq 0$, belong to $\\mathcal{F}$. Prove that there exists $x_{0} \\in \\mathbb{R}$ such that $f\\left(x_{0}\\right)=x_{0}$ for all $f \\in \\mathcal{F}$.","solution":"17. Let $f_{1}(x)=a x+b$ and $f_{2}(x)=c x+d$ be two functions from $\\mathcal{F}$. We define $g(x)=f_{1} \\circ f_{2}(x)=a c x+(a d+b)$ and $h(x)=f_{2} \\circ f_{1}(x)=a c x+(b c+d)$. By the condition for $\\mathcal{F}$, both $g(x)$ and $h(x)$ belong to $\\mathcal{F}$. Moreover, there exists $h^{-1}(x)=\\frac{x-(b c+d)}{a c}$, and $$ h^{-1} \\circ g(x)=\\frac{a c x+(a d+b)-(b c+d)}{a c}=x+\\frac{(a d+b)-(b c+d)}{a c} $$ belongs to $\\mathcal{F}$. Now it follows that we must have $a d+b=b c+d$ for every $f_{1}, f_{2} \\in \\mathcal{F}$, which is equivalent to $\\frac{b}{a-1}=\\frac{d}{c-1}=k$. But these formulas exactly describe the fixed points of $f_{1}$ and $f_{2}: f_{1}(x)=a x+b=x \\Rightarrow x=$ $\\frac{b}{a-1}$. Hence all the functions in $\\mathcal{F}$ fix the point $k$.","problem_type":null,"tier":0} +{"year":"1973","problem_phase":"shortlisted","problem":"2. (CZS 1) Given a circle $K$, find the locus of vertices $A$ of parallelograms $A B C D$ with diagonals $A C \\leq B D$, such that $B D$ is inside $K$.","solution":"2. Let $D^{\\prime}$ be the reflection of $D$ across $A$. Since $B C A D^{\\prime}$ is then a parallelogram, the condition $B D \\geq A C$ is equivalent to $B D \\geq B D^{\\prime}$, which is in turn equivalent to $\\angle B A D \\geq \\angle B A D^{\\prime}$, i.e. to $\\angle B A D \\geq 90^{\\circ}$. Thus the needed locus is actually the locus of points $A$ for which there exist points $B, D$ inside $K$ with $\\angle B A D=90^{\\circ}$. Such points $B, D$ exist if and only if the two tangents from $A$ to $K$, say $A P$ and $A Q$, determine an obtuse angle. Then if $P, Q \\in K$, we have $\\angle P A O=\\angle Q A O=\\varphi>45^{\\circ}$; hence $O A=\\frac{O P}{\\sin \\varphi}0$. Point $G$ belongs to a plane $A B C$ with $A \\in O x, B \\in O y, C \\in O z$ if and only if there exist positive real numbers $\\lambda, \\mu, \\nu$ with sum 1 such that $\\lambda \\overrightarrow{O A}+\\mu \\overrightarrow{O B}+\\nu \\overrightarrow{O C}=\\overrightarrow{O G}$, which is equivalent to $\\lambda \\alpha=\\mu \\beta=\\nu \\gamma=1$. Such $\\lambda, \\mu, \\nu$ exist if and only if $$ \\alpha, \\beta, \\gamma>0 \\quad \\text { and } \\quad \\frac{1}{\\alpha}+\\frac{1}{\\beta}+\\frac{1}{\\gamma}=1 $$ Since the volume of $O A B C$ is proportional to the product $\\alpha \\beta \\gamma$, it is minimized when $\\frac{1}{\\alpha} \\cdot \\frac{1}{\\beta} \\cdot \\frac{1}{\\gamma}$ is maximized, which occurs when $\\alpha=\\beta=\\gamma=3$ and $G$ is the centroid of $\\triangle A B C$.","problem_type":null,"tier":0} +{"year":"1974","problem_phase":"shortlisted","problem":"1. I 1 (USA 4) ${ }^{\\mathrm{IMO}}$ Alice, Betty, and Carol took the same series of examinations. There was one grade of $A$, one grade of $B$, and one grade of $C$ for each examination, where $A, B, C$ are different positive integers. The final test scores were | Alice | Betty | Carol | | :---: | :---: | :---: | | 20 | 10 | 9 | If Betty placed first in the arithmetic examination, who placed second in the spelling examination?","solution":"1. Denote by $n$ the number of exams. We have $n(A+B+C)=20+10+9=39$, and since $A, B, C$ are distinct, their sum is at least 6 ; therefore $n=3$ and $A+B+C=13$. Assume w.l.o.g. that $A>B>C$. Since Betty gained $A$ points in arithmetic, but fewer than 13 points in total, she had $C$ points in both remaining exams (in spelling as well). Furthermore, Carol also gained fewer than 13 points, but with at least $B$ points on two examinations (on which Betty scored $C$ ), including spelling. If she had $A$ in spelling, then she would have at least $A+B+C=13$ points in total, a contradiction. Hence, Carol scored $B$ and placed second in spelling. Remark. Moreover, it follows that Alice, Betty, and Carol scored $B+A+A$, $A+C+C$, and $C+B+B$ respectively, and that $A=8, B=4, C=1$.","problem_type":null,"tier":0} +{"year":"1974","problem_phase":"shortlisted","problem":"10. II 4 (FIN 3) ${ }^{\\mathrm{IMO} 2}$ Let $\\triangle A B C$ be a triangle. Prove that there exists a point $D$ on the side $A B$ such that $C D$ is the geometric mean of $A D$ and $B D$ if and only if $\\sqrt{\\sin A \\sin B} \\leq \\sin \\frac{C}{2}$.","solution":"10. If we set $\\angle A C D=\\gamma_{1}$ and $\\angle B C D=\\gamma_{2}$ for a point $D$ on the segment $A B$, then by the sine theorem, $$ f(D)=\\frac{C D^{2}}{A D \\cdot B D}=\\frac{C D}{A D} \\cdot \\frac{C D}{B D}=\\frac{\\sin \\alpha \\sin \\beta}{\\sin \\gamma_{1} \\sin \\gamma_{2}} . $$ The denominator of the last fraction is $$ \\begin{aligned} \\sin \\gamma_{1} \\sin \\gamma_{2} & =\\frac{1}{2}\\left(\\cos \\left(\\gamma_{1}-\\gamma_{2}\\right)-\\cos \\left(\\gamma_{1}+\\gamma_{2}\\right)\\right) \\\\ & =\\frac{1}{2}\\left(\\cos \\left(\\gamma_{1}-\\gamma_{2}\\right)-\\cos \\gamma\\right) \\leq \\frac{1-\\cos \\gamma}{2}=\\sin ^{2} \\frac{\\gamma}{2} \\end{aligned} $$ from which we deduce that the set of values of $f(D)$ is the interval $\\left[\\frac{\\sin \\alpha \\sin \\beta}{\\sin ^{2} \\frac{1}{2}},+\\infty\\right.$ ). Hence $f(D)=1$ (equivalently, $\\left.C D^{2}=A D \\cdot B D\\right)$ is possible if and only if $\\sin \\alpha \\sin \\beta \\leq \\sin ^{2} \\frac{\\gamma}{2}$, i.e., $$ \\sqrt{\\sin \\alpha \\sin \\beta} \\leq \\sin \\frac{\\gamma}{2} $$ Second solution. Let $E$ be the second point of intersection of the line $C D$ with the circumcircle $k$ of $A B C$. Since $A D \\cdot B D=C D \\cdot E D$ (power of $D$ with respect to $k$ ), $C D^{2}=A D \\cdot B D$ ie equivalent to $E D \\geq C D$. Clearly the ratio $\\frac{E D}{C D}(D \\in A B)$ takes a minimal value when $E$ is the midpoint of the arc $A B$ not containing $C$. (This follows from $E D: C D=E^{\\prime} D: C^{\\prime} D$ when $C^{\\prime}$ and $E^{\\prime}$ are respectively projections from $C$ and $E$ onto $A B$.) On the other hand, it is directly shown that in this case $$ \\frac{E D}{C D}=\\frac{\\sin ^{2} \\frac{\\gamma}{2}}{\\sin \\alpha \\sin \\beta} $$ and the assertion follows.","problem_type":null,"tier":0} +{"year":"1974","problem_phase":"shortlisted","problem":"11. II 5 (BUL 1) ${ }^{\\mathrm{IMO} 4}$ Consider a partition of an $8 \\times 8$ chessboard into $p$ rectangles whose interiors are disjoint such that each of them has an equal number of white and black cells. Assume that $a_{1}2 a_{n}$ for all $n$. For $n=1$ the claim is trivial. If it holds for $i \\leq n$, then $a_{i} \\leq 2^{i-n} a_{n}$; thus we obtain from (1) $$ a_{n+1}>a_{n}\\left(3-\\frac{1}{2}-\\frac{1}{2^{2}}-\\cdots-\\frac{1}{2^{n}}\\right)>2 a_{n} $$ Therefore $a_{n} \\geq 2^{n}$ for all $n$ (moreover, one can show from (1) that $a_{n} \\geq$ $(n+2) 2^{n-1}$ ); hence there exist good words of length $n$. Remark. If there are two nonallowed words (instead of one) of each length greater than 1, the statement of the problem need not remain true.","problem_type":null,"tier":0} +{"year":"1974","problem_phase":"shortlisted","problem":"2. I 2 (POL 1) Prove that the squares with sides $1 \/ 1,1 \/ 2,1 \/ 3, \\ldots$ may be put into the square with side $3 \/ 2$ in such a way that no two of them have any interior point in common.","solution":"2. We denote by $q_{i}$ the square with side $\\frac{1}{i}$. Let us divide the big square into rectangles $r_{i}$ by parallel lines, where the size of $r_{i}$ is $\\frac{3}{2} \\times \\frac{1}{2^{i}}$ for $i=2,3, \\ldots$ and $\\frac{3}{2} \\times 1$ for $i=1$ (this can be done because $1+\\sum_{i=2}^{\\infty} \\frac{1}{2^{i}}=\\frac{3}{2}$ ). In rectangle $r_{1}$, one can put the squares $q_{1}, q_{2}, q_{3}$, as is done on the figure. Also, since $\\frac{1}{2^{i}}+\\cdots+\\frac{1}{2^{i+1}-1}<2^{i} \\cdot \\frac{1}{2^{i}}=1<\\frac{3}{2}$, in each $r_{i}, i \\geq 2$, one can put $q_{2^{i}}, \\ldots, q_{2^{i+1}-1}$. This completes the proof. ![](https:\/\/cdn.mathpix.com\/cropped\/2024_11_18_8e985d6b9c83aa3e9d0eg-404.jpg?height=252&width=254&top_left_y=1103&top_left_x=664) Remark. It can be shown that the squares $q_{1}, q_{2}$ cannot fit in any square of side less than $\\frac{3}{2}$.","problem_type":null,"tier":0} +{"year":"1974","problem_phase":"shortlisted","problem":"3. I 3 (SWE 3) ${ }^{\\mathrm{IMO}}$ Let $P(x)$ be a polynomial with integer coefficients. If $n(P)$ is the number of (distinct) integers $k$ such that $P^{2}(k)=1$, prove that $$ n(P)-\\operatorname{deg}(P) \\leq 2 $$ where $\\operatorname{deg}(P)$ denotes the degree of the polynomial $P$.","solution":"3. For $\\operatorname{deg}(P) \\leq 2$ the statement is obvious, since $n(P) \\leq \\operatorname{deg}\\left(P^{2}\\right)=$ $2 \\operatorname{deg}(P) \\leq \\operatorname{deg}(P)+2$ \u3002 Suppose now that $\\operatorname{deg}(P) \\geq 3$ and $n(P)>\\operatorname{deg}(P)+2$. Then there is at least one integer $b$ for which $P(b)=-1$, and at least one $x$ with $P(x)=1$. We may assume w.l.o.g. that $b=0$ (if necessary, we consider the polynomial $P(x+b)$ instead). If $k_{1}, \\ldots, k_{m}$ are all integers for which $P\\left(k_{i}\\right)=1$, then $P(x)=Q(x)\\left(x-k_{1}\\right) \\cdots\\left(x-k_{m}\\right)+1$ for some polynomial $Q(x)$ with integer coefficients. Setting $x=0$ we obtain $(-1)^{m} Q(0) k_{1} \\cdots k_{m}=1-P(0)=2$. It follows that $k_{1} \\cdots k_{m} \\mid 2$, and hence $m$ is at most 3 . The same holds for the polynomial $-P(x)$, and thus $P(x)=-1$ also has at most 3 integer solutions. This counts for 6 solutions of $P^{2}(x)=1$ in total, implying the statement for $\\operatorname{deg}(P) \\geq 4$. It remains to verify the statement for $n=3$. If $\\operatorname{deg}(P)=3$ and $n(P)=6$, then it follows from the above consideration that $P(x)$ is either $-\\left(x^{2}-\\right.$ $1)(x-2)+1$ or $\\left(x^{2}-1\\right)(x+2)+1$. It is directly checked that $n(P)$ equals only 4 in both cases.","problem_type":null,"tier":0} +{"year":"1974","problem_phase":"shortlisted","problem":"4. I 4 (USS 4) The sum of the squares of five real numbers $a_{1}, a_{2}, a_{3}, a_{4}, a_{5}$ equals 1. Prove that the least of the numbers $\\left(a_{i}-a_{j}\\right)^{2}$, where $i, j=$ $1,2,3,4,5$ and $i \\neq j$, does not exceed $1 \/ 10$.","solution":"4. Assume w.l.o.g. that $a_{1} \\leq a_{2} \\leq a_{3} \\leq a_{4} \\leq a_{5}$. If $m$ is the least value of $\\left|a_{i}-a_{j}\\right|, i \\neq j$, then $a_{i+1}-a_{i} \\geq m$ for $i=1,2, \\ldots, 5$, and consequently $a_{i}-a_{j} \\geq(i-j) m$ for any $i, j \\in\\{1, \\ldots, 5\\}, i>j$. Then it follows that $$ \\sum_{i>j}\\left(a_{i}-a_{j}\\right)^{2} \\geq m^{2} \\sum_{i>j}(i-j)^{2}=50 m^{2} $$ On the other hand, by the condition of the problem, $$ \\sum_{i>j}\\left(a_{i}-a_{j}\\right)^{2}=5 \\sum_{i=1}^{5} a_{i}^{2}-\\left(a_{1}+\\cdots+a_{5}\\right)^{2} \\leq 5 $$ Therefore $50 m^{2} \\leq 5$; i.e., $m^{2} \\leq \\frac{1}{10}$.","problem_type":null,"tier":0} +{"year":"1974","problem_phase":"shortlisted","problem":"5. I 5 (GBR 3) Let $A_{r}, B_{r}, C_{r}$ be points on the circumference of a given circle $S$. From the triangle $A_{r} B_{r} C_{r}$, called $\\triangle_{r}$, the triangle $\\triangle_{r+1}$ is obtained by constructing the points $A_{r+1}, B_{r+1}, C_{r+1}$ on $S$ such that $A_{r+1} A_{r}$ is parallel to $B_{r} C_{r}, B_{r+1} B_{r}$ is parallel to $C_{r} A_{r}$, and $C_{r+1} C_{r}$ is parallel to $A_{r} B_{r}$. Each angle of $\\triangle_{1}$ is an integer number of degrees and those integers are not multiples of 45 . Prove that at least two of the triangles $\\triangle_{1}, \\triangle_{2}, \\ldots, \\triangle_{15}$ are congruent.","solution":"5. All the angles are assumed to be oriented and measured modulo $180^{\\circ}$. Denote by $\\alpha_{i}, \\beta_{i}, \\gamma_{i}$ the angles of triangle $\\triangle_{i}$, at $A_{i}, B_{i}, C_{i}$ respectively. Let us determine the angles of $\\triangle_{i+1}$. If $D_{i}$ is the intersection of lines $B_{i} B_{i+1}$ and $C_{i} C_{i+1}$, we have $\\angle B_{i+1} A_{i+1} C_{i+1}=\\angle D_{i} B_{i} C_{i+1}=\\angle B_{i} D_{i} C_{i+1}+$ $\\angle D_{i} C_{i+1} B_{i}=\\angle B_{i} D_{i} C_{i}-\\angle B_{i} C_{i+1} C_{i}=-2 \\angle B_{i} A_{i} C_{i}$. We conclude that $$ \\alpha_{i+1}=-2 \\alpha_{i}, \\quad \\text { and analogously } \\quad \\beta_{i+1}=-2 \\beta_{i}, \\quad \\gamma_{i+1}=-2 \\gamma_{i} $$ Therefore $\\alpha_{r+t}=(-2)^{t} \\alpha_{r}$. However, since $(-2)^{12} \\equiv 1(\\bmod 45)$ and consequently $(-2)^{14} \\equiv(-2)^{2}(\\bmod 180)$, it follows that $\\alpha_{15}=\\alpha_{3}$, since all values are modulo $180^{\\circ}$. Analogously, $\\beta_{15}=\\beta_{3}$ and $\\gamma_{15}=\\gamma_{3}$, and moreover, $\\triangle_{3}$ and $\\triangle_{15}$ are inscribed in the same circle; hence $\\triangle_{3} \\cong \\triangle_{15}$.","problem_type":null,"tier":0} +{"year":"1974","problem_phase":"shortlisted","problem":"6. I 6 (ROM 4) ${ }^{\\mathrm{IMO} 3}$ Does there exist a natural number $n$ for which the number $$ \\sum_{k=0}^{n}\\binom{2 n+1}{2 k+1} 2^{3 k} $$ is divisible by 5 ?","solution":"6. We set $$ \\begin{aligned} & x=\\sum_{k=0}^{n}\\binom{2 n+1}{2 k+1} 2^{3 k}=\\frac{1}{\\sqrt{8}} \\sum_{k=0}^{n}\\binom{2 n+1}{2 k+1} \\sqrt{8}^{2 k+1}, \\\\ & y=\\sum_{k=0}^{n}\\binom{2 n+1}{2 k} 2^{3 k}=\\sum_{k=0}^{n}\\binom{2 n+1}{2 k} \\sqrt{8}^{2 k} \\end{aligned} $$ Both $x$ and $y$ are positive integers. Also, from the binomial formula we obtain $$ y+x \\sqrt{8}=\\sum_{i=0}^{2 n+1}\\binom{2 n+1}{i} \\sqrt{8}^{i}=(1+\\sqrt{8})^{2 n+1} $$ and similarly $$ y-x \\sqrt{8}=(1-\\sqrt{8})^{2 n+1} . $$ Multiplying these equalities, we get $y^{2}-8 x^{2}=(1+\\sqrt{8})^{2 n+1}(1-\\sqrt{8})^{2 n+1}=$ $-7^{2 n+1}$. Reducing modulo 5 gives us $$ 3 x^{2}-y^{2} \\equiv 2^{2 n+1} \\equiv 2 \\cdot(-1)^{n} $$ Now we see that if $x$ is divisible by 5 , then $y^{2} \\equiv \\pm 2(\\bmod 5)$, which is impossible. Therefore $x$ is never divisible by 5 . Second solution. Another standard way is considering recurrent formulas. If we set $$ x_{m}=\\sum_{k}\\binom{m}{2 k+1} 8^{k}, \\quad y_{m}=\\sum_{k}\\binom{m}{2 k} 8^{k} $$ then since $\\binom{a}{b}=\\binom{a-1}{b}+\\binom{a-1}{b-1}$, it follows that $x_{m+1}=x_{m}+y_{m}$ and $y_{m+1}=8 x_{m}+y_{m}$; therefore $x_{m+1}=2 x_{m}+7 x_{m-1}$. We need to show that none of $x_{2 n+1}$ are divisible by 5 . Considering the sequence $\\left\\{x_{m}\\right\\}$ modulo 5 , we get that $x_{m}=0,1,2,1,1,4,0,3,1,3,3,2,0,4,3,4,4,1, \\ldots$ Zeros occur in the initial position of blocks of length 6 , where each subsequent block is obtained by multiplying the previous one by 3 (modulo 5 ). Consequently, $x_{m}$ is divisible by 5 if and only if $m$ is a multiple of 6 , which cannot happen if $m=2 n+1$.","problem_type":null,"tier":0} +{"year":"1974","problem_phase":"shortlisted","problem":"7. II 1 (POL 2) Let $a_{i}, b_{i}$ be coprime positive integers for $i=1,2, \\ldots, k$, and $m$ the least common multiple of $b_{1}, \\ldots, b_{k}$. Prove that the greatest common divisor of $a_{1} \\frac{m}{b_{1}}, \\ldots, a_{k} \\frac{m}{b_{k}}$ equals the greatest common divisor of $a_{1}, \\ldots, a_{k}$.","solution":"7. Consider an arbitrary prime number $p$. If $p \\mid m$, then there exists $b_{i}$ that is divisible by the same power of $p$ as $m$. Then $p$ divides neither $a_{i} \\frac{m}{b_{i}}$ nor $a_{i}$, because $\\left(a_{i}, b_{i}\\right)=1$. If otherwise $p \\nmid m$, then $\\frac{m}{b_{i}}$ is not divisible by $p$ for any $i$, hence $p$ divides $a_{i}$ and $a_{i} \\frac{m}{b_{i}}$ to the same power. Therefore $\\left(a_{1}, \\ldots, a_{k}\\right)$ and $\\left(a_{1} \\frac{m}{b_{1}}, \\ldots, a_{k} \\frac{m}{b_{k}}\\right)$ have the same factorization; hence they are equal. Second solution. For $k=2$ we easily verify the formula $\\left(m \\frac{a_{1}}{b_{1}}, m \\frac{a_{2}}{b_{2}}\\right)=$ $\\frac{m}{b_{1} b_{2}}\\left(a_{1} b_{2}, a_{2} b_{1}\\right)=\\frac{1}{b_{1} b_{2}}\\left[b_{1}, b_{2}\\right]\\left(a_{1}, a_{2}\\right)\\left(b_{1}, b_{2}\\right)=\\left(a_{1}, a_{2}\\right)$, since $\\left[b_{1}, b_{2}\\right]$. $\\left(b_{1}, b_{2}\\right)=b_{1} b_{2}$. We proceed by induction: $$ \\begin{aligned} \\left(a_{1} \\frac{m}{b_{1}}, \\ldots, a_{k} \\frac{m}{b_{k}}, a_{k+1} \\frac{m}{b_{k+1}}\\right) & =\\left(\\frac{m}{\\left[b_{1}, \\ldots, b_{k}\\right]}\\left(a_{1}, \\ldots, a_{k}\\right), a_{k+1} \\frac{m}{b_{k+1}}\\right) \\\\ & =\\left(a_{1}, \\ldots, a_{k}, a_{k+1}\\right) . \\end{aligned} $$","problem_type":null,"tier":0} +{"year":"1974","problem_phase":"shortlisted","problem":"8. II 2 (NET 3) ${ }^{\\mathrm{IMO} 5}$ If $a, b, c, d$ are arbitrary positive real numbers, find all possible values of $$ S=\\frac{a}{a+b+d}+\\frac{b}{a+b+c}+\\frac{c}{b+c+d}+\\frac{d}{a+c+d} . $$","solution":"8. It is clear that $$ \\begin{gathered} \\frac{a}{a+b+c+d}+\\frac{b}{a+b+c+d}+\\frac{c}{a+b+c+d}+\\frac{d}{a+b+c+d}0, a_{i+1}>a_{i}$.) Prove that there are infinitely many $m$ for which positive integers $x, y, h, k$ can be found such that $01, a_{k_{i}} \\equiv a_{k_{1}}\\left(\\bmod a_{1}\\right)$; hence $a_{k_{i}}=a_{k_{1}}+y a_{1}$ for some integer $y>0$. It follows that for every $r=0,1, \\ldots, a_{1}-1$ there is exactly one member of the corresponding $\\left(a_{k_{i}}\\right)_{i \\geq 1}$ that cannot be represented as $x a_{l}+y a_{m}$, and hence at most $a_{1}+1$ members of $\\left(a_{k}\\right)$ in total are not representable in the given form.","problem_type":null,"tier":0} +{"year":"1975","problem_phase":"shortlisted","problem":"12. (GRE) Consider on the first quadrant of the trigonometric circle the $\\operatorname{arcs} A M_{1}=x_{1}, A M_{2}=x_{2}, A M_{3}=x_{3}, \\ldots, A M_{\\nu}=x_{\\nu}$, such that $x_{1}<$ $x_{2}k+1$. Without loss of generality we may suppose that $k=0, m=n-1$ and that no two segments $A_{k} A_{k+1}$ and $A_{m} A_{m+1}$ intersect for $0 \\leq kA_{0} A_{1}$, hence $\\angle A_{0} A_{1} A_{2}>$ $\\angle A_{1} A_{2} A_{3}$, a contradiction. Let $n=4$. From $A_{3} A_{2}>A_{1} A_{2}$ we conclude that $\\angle A_{3} A_{1} A_{2}>\\angle A_{1} A_{3} A_{2}$. Using the inequality $\\angle A_{0} A_{3} A_{2}>\\angle A_{0} A_{1} A_{2}$ we obtain that $\\angle A_{0} A_{3} A_{1}>$ $\\angle A_{0} A_{1} A_{3}$ implying $A_{0} A_{1}>A_{0} A_{3}$. Now we have $A_{2} A_{3}\\cdots>\\alpha_{n-1}$; hence $\\alpha_{n-1}<\\frac{360^{\\circ}}{n-1} \\leq 90^{\\circ}$. Consequently $\\angle A_{n-2} A_{n-1} A_{0} \\geq 90^{\\circ}$ and $A_{0} A_{n-2}>A_{n-1} A_{n-2}$. On the other hand, $A_{0} A_{n-2}2$. Adding these up we obtain $x_{n}^{2} \\geq x_{0}^{2}+2 n$, which proves the first inequality. On the other hand, $x_{k+1}=x_{k}+\\frac{1}{x_{k}} \\leq x_{k}+0.2$ (for $x_{k} \\geq 5$ ), and one also deduces from (1) that $x_{k+1}^{2}-x_{k}^{2}-0.2\\left(x_{k+1}-x_{k}\\right)=\\left(x_{k+1}+x_{k}-\\right.$ $0.2)\\left(x_{k+1}-x_{k}\\right) \\leq 2$. Again, adding these inequalities up, $(k=0, \\ldots, n-1)$ yields $$ x_{n}^{2} \\leq 2 n+x_{0}^{2}+0.2\\left(x_{n}-x_{0}\\right)=2 n+24+0.2 x_{n} $$ Solving the corresponding quadratic equation, we obtain $x_{n}<0.1+$ $\\sqrt{2 n+24.01}<0.1++\\sqrt{2 n+25}$.","problem_type":null,"tier":0} +{"year":"1975","problem_phase":"shortlisted","problem":"15. (USS) ${ }^{\\mathrm{IMO} 5}$ Is it possible to plot 1975 points on a circle with radius 1 so that the distance between any two of them is a rational number (distances have to be measured by chords)?","solution":"15. Assume that the center of the circle is at the origin $O(0,0)$, and that the points $A_{1}, A_{2}, \\ldots, A_{1975}$ are arranged on the upper half-circle so that $\\angle A_{i} O A_{1}=\\alpha_{i}\\left(\\alpha_{1}=0\\right)$. The distance $A_{i} A_{j}$ equals $2 \\sin \\frac{\\alpha_{j}-\\alpha_{i}}{2}=$ $2 \\sin \\frac{\\alpha_{j}}{2} \\cos \\frac{\\alpha_{i}}{2}-\\cos \\frac{\\alpha_{j}}{2} \\sin \\frac{\\alpha_{i}}{2}$, and it will be rational if all $\\sin \\frac{\\alpha_{k}}{2}, \\cos \\frac{\\alpha_{k}}{2}$ are rational. Finally, observe that there exist infinitely many angles $\\alpha$ such that both $\\sin \\alpha, \\cos \\alpha$ are rational, and that such $\\alpha$ can be arbitrarily small. For example, take $\\alpha$ so that $\\sin \\alpha=\\frac{2 t}{t^{2}+1}$ and $\\cos \\alpha=\\frac{t^{2}-1}{t^{2}+1}$ for any $t \\in \\mathbb{Q}$.","problem_type":null,"tier":0} +{"year":"1975","problem_phase":"shortlisted","problem":"2. (CZS) ${ }^{\\mathrm{IMO} 1}$ Let $x_{1} \\geq x_{2} \\geq \\cdots \\geq x_{n}$ and $y_{1} \\geq y_{2} \\geq \\cdots \\geq y_{n}$ be two $n$-tuples of numbers. Prove that $$ \\sum_{i=1}^{n}\\left(x_{i}-y_{i}\\right)^{2} \\leq \\sum_{i=1}^{n}\\left(x_{i}-z_{i}\\right)^{2} $$ is true when $z_{1}, z_{2}, \\ldots, z_{n}$ denote $y_{1}, y_{2}, \\ldots, y_{n}$ taken in another order.","solution":"2. Since there are finitely many arrangements of the $z_{i}$ 's, assume that $z_{1}, \\ldots, z_{n}$ is the one for which $\\sum_{i=1}^{n}\\left(x_{i}-z_{i}\\right)^{2}$ is minimal. We claim that in this case $i\\Delta a_{n+1}$. Suppose that for some $n, \\Delta a_{n}<0$ : Then for each $k \\geq n, \\Delta a_{k}<\\Delta a_{n}$; hence $a_{n}-a_{n+m}=\\Delta a_{n}+\\cdots+\\Delta a_{n+m-1}0$, or equivalently, let the graph of $f_{a}$ lie below the graph of $f$. In this case also $f(2 a)>f(a)$, since otherwise, the graphs of $f$ and $f_{a}$ would intersect between $a$ and $2 a$. Continuing in this way we are led to $0=f(0)<$ $f(a)f(1-a)>f(1-2 a)>\\cdots>f(1-n a)$. Choosing values of $f$ at $i a, 1-i a, i=1, \\ldots, n$, so that they satisfy $f(1-n a)<\\cdotsa_{i}$. Since $4=2^{2}$, we may assume that all $a_{i}$ are either 2 or 3 , and $M=2^{k} 3^{l}$, where $2 k+3 l=1976$. (4) $k \\geq 3$ does not yield the maximal value, since $2 \\cdot 2 \\cdot 2<3 \\cdot 3$. Hence $k \\leq 2$ and $2 k \\equiv 1976(\\bmod 3)$ gives us $k=1, l=658$ and $M=2 \\cdot 3^{658}$.","problem_type":null,"tier":0} +{"year":"1976","problem_phase":"shortlisted","problem":"11. (VIE 1) Prove that there exist infinitely many positive integers $n$ such that the decimal representation of $5^{n}$ contains a block of 1976 consecutive zeros.","solution":"11. We shall show by induction that $5^{2^{k}}-1=2^{k+2} q_{k}$ for each $k=0,1, \\ldots$, where $q_{k} \\in \\mathbb{N}$. Indeed, the statement is true for $k=0$, and if it holds for some $k$ then $5^{2^{k+1}}-1=\\left(5^{2^{k}}+1\\right)\\left(5^{2^{k}}-1\\right)=2^{k+3} d_{k+1}$ where $d_{k+1}=$ $\\left(5^{2^{k}}+1\\right) d_{k} \/ 2$ is an integer by the inductive hypothesis. Let us now choose $n=2^{k}+k+2$. We have $5^{n}=10^{k+2} q_{k}+5^{k+2}$. It follows from $5^{4}<10^{3}$ that $5^{k+2}$ has at most $[3(k+2) \/ 4]+2$ nonzero digits, while $10^{k+2} q_{k}$ ends in $k+2$ zeros. Hence the decimal representation of $5^{n}$ contains at least $[(k+2) \/ 4]-2$ consecutive zeros. Now it suffices to take $k>4 \\cdot 1978$.","problem_type":null,"tier":0} +{"year":"1976","problem_phase":"shortlisted","problem":"12. (VIE 2) The polynomial $1976\\left(x+x^{2}+\\cdots+x^{n}\\right)$ is decomposed into a sum of polynomials of the form $a_{1} x+a_{2} x^{2}+\\ldots+a_{n} x^{n}$, where $a_{1}, a_{2}, \\cdots, a_{n}$ are distinct positive integers not greater than $n$. Find all values of $n$ for which such a decomposition is possible.","solution":"12. Suppose the decomposition into $k$ polynomials is possible. The sum of coefficients of each polynomial $a_{1} x+a_{2} x^{2}+\\cdots+a_{n} x^{n}$ equals $1+\\cdots+$ $n=n(n+1) \/ 2$ while the sum of coefficients of $1976\\left(x+x^{2}+\\cdots+x^{n}\\right)$ is $1976 n$. Hence we must have $1976 n=k n(n+1) \/ 2$, which reduces to $(n+1) \\mid 3952=2^{4} \\cdot 13 \\cdot 19$. In other words, $n$ is of the form $n=2^{\\alpha} 13^{\\beta} 19^{\\gamma}-1$, with $0 \\leq \\alpha \\leq 4,0 \\leq \\beta \\leq 1,0 \\leq \\gamma \\leq 1$. We can immediately eliminate the values $n=0$ and $n=3951$ that correspond to $\\alpha=\\beta=\\gamma=0$ and $\\alpha=4, \\beta=\\gamma=1$. We claim that all other values $n$ are permitted. There are two cases. $\\alpha \\leq 3$. In this case $k=3952 \/(n+1)$ is even. The simple choice of the polynomials $P=x+2 x^{2}+\\cdots+n x^{n}$ and $P^{\\prime}=n x+(n-1) x^{2}+\\cdots+x^{n}$ suffices, since $k\\left(P+P^{\\prime}\\right) \/ 2=1976\\left(x+x^{2}+\\cdots+x^{n}\\right)$. $\\alpha=4$. Then $k$ is odd. Consider $(k-3) \/ 2$ pairs $\\left(P, P^{\\prime}\\right)$ of the former case and $$ \\begin{aligned} P_{1}= & {\\left[n x+(n-1) x^{3}+\\cdots+\\frac{n+1}{2} x^{n}\\right] } \\\\ & +\\left[\\frac{n-1}{2} x^{2}+\\frac{n-3}{2} x^{4}+\\cdots+x^{n-1}\\right] \\\\ P_{2}= & {\\left[\\frac{n+1}{2} x+\\frac{n-1}{2} x^{3}+\\cdots+x^{n}\\right] } \\\\ & +\\left[n x^{2}+(n-1) x^{4}+\\cdots+\\frac{n+3}{2} x^{n-1}\\right] \\end{aligned} $$ Then $P+P_{1}+P_{2}=3(n+1)\\left(x+x^{2}+\\cdots+x^{n}\\right) \/ 2$ and therefore $(k-3)\\left(P+P^{\\prime}\\right) \/ 2+\\left(P+P_{1}+P_{2}\\right)=1976\\left(x+x^{2}+\\cdots+x^{n}\\right)$. It follows that the desired decomposition is possible if and only if $10$, and let $c_{k}$ be the maximal such $c_{i}$. Assuming w.l.o.g. that $c_{k-1}2$, then $a \/ b \\leq 5 \/ 3$, and if $a>5$, then $a \/ b \\leq 3 \/ 2$. If $a_{1}>2$, then $\\frac{a_{1}}{b_{1}} \\cdot \\frac{a_{2}}{b_{2}} \\cdot \\frac{a_{3}}{b_{3}}<(5 \/ 3)^{3}<5$, a contradiction. Hence $a_{1}=2$. If also $a_{2}=2$, then $a_{3} \/ b_{3}=5 \/ 4 \\leq \\sqrt[3]{2}$, which is impossible. Also, if $a_{2} \\geq 6$, then $\\frac{a_{2}}{b_{2}} \\cdot \\frac{a_{3}}{b_{3}} \\leq(1.5)^{2}<2.5$, again a contradiction. We thus have the following cases: (i) $a_{1}=2, a_{2}=3$, then $a_{3} \/ b_{3}=5 \/ 3$, which holds only if $a_{3}=5$; (ii) $a_{1}=2, a_{2}=4$, then $a_{3} \/ b_{3}=15 \/ 8$, which is impossible; (iii) $a_{1}=2, a_{2}=5$, then $a_{3} \/ b_{3}=3 \/ 2$, which holds only if $a_{3}=6$. The only possible sizes of the box are therefore $(2,3,5)$ and $(2,5,6)$.","problem_type":null,"tier":0} +{"year":"1976","problem_phase":"shortlisted","problem":"7. (POL 1b) Let $I=(0,1]$ be the unit interval of the real line. For a given number $a \\in(0,1)$ we define a map $T: I \\rightarrow I$ by the formula $$ T(x, y)= \\begin{cases}x+(1-a) & \\text { if } 00$ such that $T^{n}(J) \\cap J \\neq \\emptyset$.","solution":"7. The map $T$ transforms the interval $(0, a]$ onto $(1-a, 1]$ and the interval $(a, 1]$ onto $(0,1-a]$. Clearly $T$ preserves the measure. Since the measure of the interval $[0,1]$ is finite, there exist two positive integers $k, l>k$ such that $T^{k}(J)$ and $T^{l}(J)$ are not disjoint. But the map $T$ is bijective; hence $T^{l-k}(J)$ and $J$ are not disjoint.","problem_type":null,"tier":0} +{"year":"1976","problem_phase":"shortlisted","problem":"8. (SWE 3) Let $P$ be a polynomial with real coefficients such that $P(x)>0$ if $x>0$. Prove that there exist polynomials $Q$ and $R$ with nonnegative coefficients such that $P(x)=\\frac{Q(x)}{R(x)}$ if $x>0$.","solution":"8. Every polynomial with real coefficients can be factored as a product of linear and quadratic polynomials with real coefficients. Thus it suffices to prove the result only for a quadratic polynomial $P(x)=x^{2}-2 a x+b^{2}$, with $a>0$ and $b^{2}>a^{2}$. Using the identity $$ \\left(x^{2}+b^{2}\\right)^{2 n}-(2 a x)^{2 n}=\\left(x^{2}-2 a x+b^{2}\\right) \\sum_{k=0}^{2 n-1}\\left(x^{2}+b^{2}\\right)^{k}(2 a x)^{2 n-k-1} $$ we have solved the problem if we can choose $n$ such that $b^{2 n}\\binom{2 n}{n}>2^{2 n} a^{2 n}$. However, it is is easy to show that $2 n\\binom{2 n}{n}<2^{2 n}$; hence it is enough to take $n$ such that $(b \/ a)^{2 n}>2 n$. Since $\\lim _{n \\rightarrow \\infty}(2 n)^{1 \/(2 n)}=12$, then by simple induction $P_{n}(x)>x$ for all $n$. Similarly, if $x<-1$, then $P_{1}(x)>2$, which implies $P_{n}(x)>2$ for all $n$. It follows that all real roots of the equation $P_{n}(x)=x$ lie in the interval $[-2,2]$, and thus have the form $x=2 \\cos t$. Now we observe that $P_{1}(2 \\cos t)=4 \\cos ^{2} t-2=2 \\cos 2 t$, and in general $P_{n}(2 \\cos t)=2 \\cos 2^{n} t$. Our equation becomes $$ \\cos 2^{n} t=\\cos t $$ which indeed has $2^{n}$ different solutions $t=\\frac{2 \\pi m}{2^{n}-1}\\left(m=0,1, \\ldots, 2^{n-1}-1\\right)$ and $t=\\frac{2 \\pi m}{2^{n}+1}\\left(m=1,2, \\ldots, 2^{n-1}\\right)$.","problem_type":null,"tier":0} +{"year":"1977","problem_phase":"longlisted","problem":"1. (BUL 1) A pentagon $A B C D E$ inscribed in a circle for which $B CC S$.","solution":"1. Let $P$ be the projection of $S$ onto the plane $A B C D E$. Obviously $B S>C S$ is equivalent to $B P>C P$. The conditions of the problem imply that $P A>P B$ and $P A>P E$. The locus of such points $P$ is the region of the plane that is determined by the perpendicular bisectors of segments $A B$ and $A E$ and that contains the point diametrically opposite $A$. But since $A B1977-90=1887$ it follows that $q>43$; hence $q=44$ and $r=41$. It remains to find positive integers $a$ and $b$ satisfying $a^{2}+b^{2}=44(a+b)+41$, or equivalently $$ (a-22)^{2}+(b-22)^{2}=1009 $$ The only solutions to this Diophantine equation are $(|a-22|,|b-22|) \\in$ $\\{(15,28),(28,15)\\}$, which yield $(a, b) \\in\\{(7,50),(37,50),(50,7),(50,37)\\}$.","problem_type":null,"tier":0} +{"year":"1977","problem_phase":"longlisted","problem":"11. (FRG 2) Let $n$ and $z$ be integers greater than 1 and $(n, z)=1$. Prove: (a) At least one of the numbers $z_{i}=1+z+z^{2}+\\cdots+z^{i}, i=0,1, \\ldots, n-1$, is divisible by $n$. (b) If $(z-1, n)=1$, then at least one of the numbers $z_{i}, i=0,1, \\ldots, n-2$, is divisible by $n$.","solution":"11. (a) Suppose to the contrary that none of the numbers $z_{0}, z_{1}, \\ldots, z_{n-1}$ is divisible by $n$. Then two of these numbers, say $z_{k}$ and $z_{l}(0 \\leq k1$ and let $M$ be the set of all numbers of the form $z_{k}=1+z+\\cdots+z^{k}, k=0,1, \\ldots$. Determine the set $T$ of divisors of at least one of the numbers $z_{k}$ from $M$.","solution":"12. According to part (a) of the previous problem we can conclude that $T=$ $\\{n \\in \\mathbb{N} \\mid(n, z)=1\\}$.","problem_type":null,"tier":0} +{"year":"1977","problem_phase":"longlisted","problem":"13. (FRG 4) (SL77-4).","solution":"13. The figure $\\Phi$ contains two points $A$ and $B$ having maximum distance. Let $h$ be the semicircle with diameter $A B$ that lies in $\\Phi$, and let $k$ be the circle containing $h$. Consider any point $M$ inside $k$. The line passing through $M$ that is orthogonal to $A M$ meets $h$ in some point $P$ (because $\\angle A M B>90^{\\circ}$ ). Let $h^{\\prime}$ and $\\overline{h^{\\prime}}$ be the two semicircles with diameter $A P$, where $M \\in h^{\\prime}$. Since $\\overline{h^{\\prime}}$ contains a point $C$ such that $B C>A B$, it cannot be contained in $\\Phi$, implying that $h^{\\prime} \\subset \\Phi$. Hence $M$ belongs to $\\Phi$. Since $\\Phi$ contains no points outside the circle $k$, it must coincide with the disk determined by $k$. On the other hand, any disk has the required property.","problem_type":null,"tier":0} +{"year":"1977","problem_phase":"longlisted","problem":"14. (FRG 5) (SL77-5).","solution":"14. We prove by induction on $n$ that independently of the word $w_{0}$, the given algorithm generates all words of length $n$. This is clear for $n=1$. Suppose now the statement is true for $n-1$, and that we are given a word $w_{0}=$ $c_{1} c_{2} \\ldots c_{n}$ of length $n$. Obviously, the words $w_{0}, w_{1}, \\ldots, w_{2^{n-1}-1}$ all have the $n$th digit $c_{n}$, and by the inductive hypothesis these are all words whose $n$th digit is $c_{n}$. Similarly, by the inductive hypothesis $w_{2^{n-1}}, \\ldots, w_{2^{n-1}}$ are all words whose $n$th digit is $1-c_{n}$, and the induction is complete.","problem_type":null,"tier":0} +{"year":"1977","problem_phase":"longlisted","problem":"15. (GDR 1) Let $n$ be an integer greater than 1 . In the Cartesian coordinate system we consider all squares with integer vertices $(x, y)$ such that $1 \\leq$ $x, y \\leq n$. Denote by $p_{k}(k=0,1,2, \\ldots)$ the number of pairs of points that are vertices of exactly $k$ such squares. Prove that $\\sum_{k}(k-1) p_{k}=0$.","solution":"15. Each segment is an edge of at most two squares and a diagonal of at most one square. Therefore $p_{k}=0$ for $k>3$, and we have to prove that $$ p_{0}=p_{2}+2 p_{3} $$ Let us calculate the number $q(n)$ of considered squares. Each of these squares is inscribed in a square with integer vertices and sides parallel to the coordinate axes. There are $(n-s)^{2}$ squares of side $s$ with integer vertices and sides parallel to the coordinate axes, and each of them circumscribes exactly $s$ of the considered squares. It follows that $q(n)=\\sum_{s=1}^{n-1}(n-s)^{2} s=n^{2}\\left(n^{2}-1\\right) \/ 12$. Computing the number of edges and diagonals of the considered squares in two ways, we obtain that $$ p_{1}+2 p_{2}+3 p_{3}=6 q(n) $$ On the other hand, the total number of segments with endpoints in the considered integer points is given by $$ p_{0}+p_{1}+p_{2}+p_{3}=\\binom{n^{2}}{2}=\\frac{n^{2}\\left(n^{2}-1\\right)}{2}=6 q(n) . $$ Now (1) follows immediately from (2) and (3).","problem_type":null,"tier":0} +{"year":"1977","problem_phase":"longlisted","problem":"16. (GDR 2) (SL77-6).","solution":"16. For $i=k$ and $j=l$ the system is reduced to $1 \\leq i, j \\leq n$, and has exactly $n^{2}$ solutions. Let us assume that $i \\neq k$ or $j \\neq l$. The points $A(i, j), B(k, l)$, $C(-j+k+l, i-k+l), D(i-j+l, i+j-k)$ are vertices of a negatively oriented square with integer vertices lying inside the square $[1, n] \\times[1, n]$, and each of these squares corresponds to exactly 4 solutions to the system. By the previous problem there are exactly $q(n)=n^{2}\\left(n^{2}-1\\right) \/ 12$ such squares. Hence the number of solutions is equal to $n^{2}+4 q(n)=n^{2}\\left(n^{2}+2\\right) \/ 3$.","problem_type":null,"tier":0} +{"year":"1977","problem_phase":"longlisted","problem":"17. (GDR 3) A ball $K$ of radius $r$ is touched from the outside by mutually equal balls of radius $R$. Two of these balls are tangent to each other. Moreover, for two balls $K_{1}$ and $K_{2}$ tangent to $K$ and tangent to each other there exist two other balls tangent to $K_{1}, K_{2}$ and also to $K$. How many balls are tangent to $K$ ? For a given $r$ determine $R$.","solution":"17. Centers of the balls that are tangent to $K$ are vertices of a regular polyhedron with triangular faces, with edge length $2 R$ and radius of circumscribed sphere $r+R$. Therefore the number $n$ of these balls is 4,6 , or 20 . It is straightforward to obtain that: (i) If $n=4$, then $r+R=2 R(\\sqrt{6} \/ 4)$, whence $R=r(2+\\sqrt{6})$. (ii) If $n=6$, then $r+R=2 R(\\sqrt{2} \/ 2)$, whence $R=r(1+\\sqrt{2})$. (iii) If $n=20$, then $r+R=2 R \\sqrt{5+\\sqrt{5}} \/ 8$, whence $R=r[\\sqrt{5-2 \\sqrt{5}}+$ $$ (3-\\sqrt{5}) \/ 2] $$","problem_type":null,"tier":0} +{"year":"1977","problem_phase":"longlisted","problem":"18. (GDR 4) Given an isosceles triangle $A B C$ with a right angle at $C$, construct the center $M$ and radius $r$ of a circle cutting on segments $A B, B C, C A$ the segments $D E, F G$, and $H K$, respectively, such that $\\angle D M E+\\angle F M G+\\angle H M K=180^{\\circ}$ and $D E: F G: H K=A B: B C:$ $C A$.","solution":"18. Let $U$ be the midpoint of the segment $A B$. The point $M$ belongs to $C U$ and $C M=(\\sqrt{5}-1) C U \/ 2, r=C U \\sqrt{\\sqrt{5}-2}$.","problem_type":null,"tier":0} +{"year":"1977","problem_phase":"longlisted","problem":"19. (GBR 1) Given any integer $m>1$ prove that there exist infinitely many positive integers $n$ such that the last $m$ digits of $5^{n}$ are a sequence $a_{m}, a_{m-1}, \\ldots, a_{1}=5\\left(0 \\leq a_{j}<10\\right)$ in which each digit except the last is of opposite parity to its successor (i.e., if $a_{i}$ is even, then $a_{i-1}$ is odd, and if $a_{i}$ is odd, then $a_{i-1}$ is even).","solution":"19. We shall prove the statement by induction on $m$. For $m=2$ it is trivial, since each power of 5 greater than 5 ends in 25 . Suppose that the statement is true for some $m \\geq 2$, and that the last $m$ digits of $5^{n}$ alternate in parity. It can be shown by induction that the maximum power of 2 that divides $5^{2^{m-2}}-1$ is $2^{m}$, and consequently the difference $5^{n+2^{m-2}}-5^{n}$ is divisible by $10^{m}$ but not by $2 \\cdot 10^{m}$. It follows that the last $m$ digits of the numbers $5^{n+2^{m-2}}$ and $5^{n}$ coincide, but the digits at the position $m+1$ have opposite parity. Hence the last $m+1$ digits of one of these two powers of 5 alternate in parity. The inductive proof is completed.","problem_type":null,"tier":0} +{"year":"1977","problem_phase":"longlisted","problem":"2. (BUL 2) (SL77-1).","solution":"2. We shall prove by induction on $n$ that $f(x)>f(n)$ whenever $x>n$. The case $n=0$ is trivial. Suppose that $n \\geq 1$ and that $x>k$ implies $f(x)>f(k)$ for all $kn$, then $m-1 \\geq n$ and consequently $f(m-1) \\geq n$. But in this case the inequality $f(m)>$ $f(f(m-1))$ contradicts the minimality property of $m$. The inductive proof is thus completed. It follows that $f$ is strictly increasing, so $f(n+1)>f(f(n))$ implies that $n+1>f(n)$. But since $f(n) \\geq n$ we must have $f(n)=n$.","problem_type":null,"tier":0} +{"year":"1977","problem_phase":"longlisted","problem":"20. (GBR 2) (SL77-7).","solution":"20. There exist $u, v$ such that $a \\cos x+b \\sin x=r \\cos (x-u)$ and $A \\cos 2 x+$ $B \\sin 2 x=R \\cos 2(x-v)$, where $r=\\sqrt{a^{2}+b^{2}}$ and $R=\\sqrt{A^{2}+B^{2}}$. Then $1-f(x)=r \\cos (x-u)+R \\cos 2(x-v) \\leq 1$ holds for all $x \\in \\mathbb{R}$. There exists $x \\in \\mathbb{R}$ such that $\\cos (x-u) \\geq 0$ and $\\cos 2(x-v)=1$ (indeed, either $x=v$ or $x=v+\\pi$ works). It follows that $R \\leq 1$. Similarly, there exists $x \\in \\mathbb{R}$ such that $\\cos (x-u)=1 \/ \\sqrt{2}$ and $\\cos 2(x-v) \\geq 0$ (either $x=u-\\pi \/ 4$ or $x=u+\\pi \/ 4$ works). It follows that $r \\leq \\sqrt{2}$. Remark. The proposition of this problem contained as an addendum the following, more difficult, inequality: $$ \\sqrt{a^{2}+b^{2}}+\\sqrt{A^{2}+B^{2}} \\leq 2 . $$ The proof follows from the existence of $x \\in \\mathbb{R}$ such that $\\cos (x-u) \\geq 1 \/ 2$ and $\\cos 2(x-v) \\geq 1 \/ 2$.","problem_type":null,"tier":0} +{"year":"1977","problem_phase":"longlisted","problem":"21. (GBR 3) Given that $x_{1}+x_{2}+x_{3}=y_{1}+y_{2}+y_{3}=x_{1} y_{1}+x_{2} y_{2}+x_{3} y_{3}=0$, prove that $$ \\frac{x_{1}^{2}}{x_{1}^{2}+x_{2}^{2}+x_{3}^{2}}+\\frac{y_{1}^{2}}{y_{1}^{2}+y_{2}^{2}+y_{3}^{2}}=\\frac{2}{3} $$","solution":"21. Let us consider the vectors $v_{1}=\\left(x_{1}, x_{2}, x_{3}\\right), v_{2}=\\left(y_{1}, y_{2}, y_{3}\\right), v_{3}=(1,1,1)$ in space. The given equalities express the condition that these three vectors are mutually perpendicular. Also, $\\frac{x_{1}^{2}}{x_{1}^{2}+x_{2}^{2}+x_{3}^{2}}, \\frac{y_{1}^{2}}{y_{1}^{2}+y_{2}^{2}+y_{3}^{2}}$, and $1 \/ 3$ are the squares of the projections of the vector $(1,0,0)$ onto the directions of $v_{1}, v_{2}, v_{3}$, respectively. The result follows from the fact that the sum of squares of projections of a unit vector on three mutually perpendicular directions is 1.","problem_type":null,"tier":0} +{"year":"1977","problem_phase":"longlisted","problem":"22. (GBR 4) (SL77-8).","solution":"22. Since the quadrilateral $O A_{1} B B_{1}$ is cyclic, $\\angle O A_{1} B_{1}=\\angle O B C$. By using the analogous equalities we obtain $\\angle O A_{4} B_{4}=\\angle O B_{3} C_{3}=\\angle O C_{2} D_{2}=$ $\\angle O D_{1} A_{1}=\\angle O A B$, and similarly $\\angle O B_{4} A_{4}=\\angle O B A$. Hence $\\triangle O A_{4} B_{4} \\sim$ $\\triangle O A B$. Analogously, we have for the other three pairs of triangles $\\triangle O B_{4} C_{4} \\sim \\triangle O B C, \\triangle O C_{4} D_{4} \\sim \\triangle O C D, \\triangle O D_{4} A_{4} \\sim \\triangle O D A$, and consequently $A B C D \\sim A_{4} B_{4} C_{4} D_{4}$.","problem_type":null,"tier":0} +{"year":"1977","problem_phase":"longlisted","problem":"23. (HUN 1) (SL77-9).","solution":"23. Every polynomial $q\\left(x_{1}, \\ldots, x_{n}\\right)$ with integer coefficients can be expressed in the form $q=r_{1}+x_{1} r_{2}$, where $r_{1}, r_{2}$ are polynomials in $x_{1}, \\ldots, x_{n}$ with integer coefficients in which the variable $x_{1}$ occurs only with even exponents. Thus if $q_{1}=r_{1}-x_{1} r_{2}$, the polynomial $q q_{1}=r_{1}^{2}-x_{1}^{2} r_{2}^{2}$ contains $x_{1}$ only with even exponents. We can continue inductively constructing polynomials $q_{j}, j=2,3, \\ldots, n$, such that $q q_{1} q_{2} \\cdots q_{j}$ contains each of variables $x_{1}, x_{2}, \\ldots, x_{j}$ only with even exponents. Thus the polynomial $q q_{1} \\cdots q_{n}$ is a polynomial in $x_{1}^{2}, \\ldots, x_{n}^{2}$. The polynomials $f$ and $g$ exist for every $n \\in \\mathbb{N}$. In fact, it suffices to construct $q_{1}, \\ldots, q_{n}$ for the polynomial $q=x_{1}+\\cdots+x_{n}$ and take $f=$ $q_{1} q_{2} \\cdots q_{n}$.","problem_type":null,"tier":0} +{"year":"1977","problem_phase":"longlisted","problem":"24. (HUN 2) Determine all real functions $f(x)$ that are defined and continuous on the interval $(-1,1)$ and that satisfy the functional equation $$ f(x+y)=\\frac{f(x)+f(y)}{1-f(x) f(y)} \\quad(x, y, x+y \\in(-1,1)) . $$","solution":"24. Setting $x=y=0$ gives us $f(0)=0$. Let us put $g(x)=\\arctan f(x)$. The given functional equation becomes $\\tan g(x+y)=\\tan (g(x)+g(y))$; hence $$ g(x+y)=g(x)+g(y)+k(x, y) \\pi $$ where $k(x, y)$ is an integer function. But $k(x, y)$ is continuous and $k(0,0)=$ 0 , therefore $k(x, y)=0$. Thus we obtain the classical Cauchy's functional equation $g(x+y)=g(x)+g(y)$ on the interval $(-1,1)$, all of whose continuous solutions are of the form $g(x)=a x$ for some real $a$. Moreover, $g(x) \\in(-\\pi, \\pi)$ implies $|a| \\leq \\pi \/ 2$. Therefore $f(x)=\\tan a x$ for some $|a| \\leq \\pi \/ 2$, and this is indeed a solution to the given equation.","problem_type":null,"tier":0} +{"year":"1977","problem_phase":"longlisted","problem":"25. (HUN 3) Prove the identity $$ (z+a)^{n}=z^{n}+a \\sum_{k=1}^{n}\\binom{n}{k}(a-k b)^{k-1}(z+k b)^{n-k} . $$","solution":"25. Let $$ f_{n}(z)=z^{n}+a \\sum_{k=1}^{n}\\binom{n}{k}(a-k b)^{k-1}(z+k b)^{n-k} . $$ We shall prove by induction on $n$ that $f_{n}(z)=(z+a)^{n}$. This is trivial for $n=1$. Suppose that the statement is true for some positive integer $n-1$. Then $$ \\begin{aligned} f_{n}^{\\prime}(z) & =n z^{n-1}+a \\sum_{k=1}^{n-1}\\binom{n}{k}(n-k)(a-k b)^{k-1}(z+k b)^{n-k-1} \\\\ & =n z^{n-1}+n a \\sum_{k=1}^{n-1}\\binom{n-1}{k}(a-k b)^{k-1}(z+k b)^{n-k-1} \\\\ & =n f_{n-1}(z)=n(z+a)^{n-1} \\end{aligned} $$ It remains to prove that $f_{n}(-a)=0$. For $z=-a$ we have by the lemma of (SL81-13), $$ \\begin{aligned} f_{n}(-a) & =(-a)^{n}+a \\sum_{k=1}^{n}\\binom{n}{k}(-1)^{n-k}(a-k b)^{n-1} \\\\ & =a \\sum_{k=0}^{n}\\binom{n}{k}(-1)^{n-k}(a-k b)^{n-1}=0 . \\end{aligned} $$","problem_type":null,"tier":0} +{"year":"1977","problem_phase":"longlisted","problem":"26. (NET 1) Let $p$ be a prime number greater than 5 . Let $V$ be the collection of all positive integers $n$ that can be written in the form $n=k p+1$ or $n=k p-1(k=1,2, \\ldots)$. A number $n \\in V$ is called indecomposable in $V$ if it is impossible to find $k, l \\in V$ such that $n=k l$. Prove that there exists a number $N \\in V$ that can be factorized into indecomposable factors in $V$ in more than one way.","solution":"26. The result is an immediate consequence (for $G=\\{-1,1\\}$ ) of the following generalization. (1) Let $G$ be a proper subgroup of $\\mathbb{Z}_{n}^{*}$ (the multiplicative group of residue classes modulo $n$ coprime to $n$ ), and let $V$ be the union of elements of $G$. A number $m \\in V$ is called indecomposable in $V$ if there do not exist numbers $p, q \\in V, p, q \\notin\\{-1,1\\}$, such that $p q=m$. There exists a number $r \\in V$ that can be expressed as a product of elements indecomposable in $V$ in more than one way. First proof. We shall start by proving the following lemma. Lemma. There are infinitely many primes not in $V$ that do not divide $n$. Proof. There is at least one such prime: In fact, any number other than $\\pm 1$ not in $V$ must have a prime factor not in $V$, since $V$ is closed under multiplication. If there were a finite number of such primes, say $p_{1}, p_{2}, \\ldots, p_{k}$, then one of the numbers $p_{1} p_{2} \\cdots p_{k}+n, p_{1}^{2} p_{2} \\cdots p_{k}+n$ is not in $V$ and is coprime to $n$ and $p_{1}, \\ldots, p_{k}$, which is a contradiction. [This lemma is actually a direct consequence of Dirichlet's theorem.] Let us consider two such primes $p, q$ that are congruent modulo $n$. Let $p^{k}$ be the least power of $p$ that is in $V$. Then $p^{k}, q^{k}, p^{k-1} q, p q^{k-1}$ belong to $V$ and are indecomposable in $V$. It follows that $$ r=p^{k} \\cdot q^{k}=p^{k-1} q \\cdot p q^{k-1} $$ has the desired property. Second proof. Let $p$ be any prime not in $V$ that does not divide $n$, and let $p^{k}$ be the least power of $p$ that is in $V$. Obviously $p^{k}$ is indecomposable in $V$. Then the number $$ r=p^{k} \\cdot\\left(p^{k-1}+n\\right)(p+n)=p\\left(p^{k-1}+n\\right) \\cdot p^{k-1}(p+n) $$ has at least two different factorizations into indecomposable factors.","problem_type":null,"tier":0} +{"year":"1977","problem_phase":"longlisted","problem":"27. (NET 2) (SL77-10).","solution":"27. The result is a consequence of the generalization from the previous problem for $G=\\{1\\}$. Remark. There is an explicit example: $r=(n-1)^{2} \\cdot(2 n-1)^{2}=[(n-$ 1) $(2 n-1)]^{2}$.","problem_type":null,"tier":0} +{"year":"1977","problem_phase":"longlisted","problem":"28. (NET 3) (SL77-11).","solution":"28. The recurrent relations give us that $$ x_{i+1}=\\left[\\frac{x_{i}+\\left[n \/ x_{i}\\right]}{2}\\right]=\\left[\\frac{x_{i}+n \/ x_{i}}{2}\\right] \\geq[\\sqrt{n}] $$ On the other hand, if $x_{i}>[\\sqrt{n}]$ for some $i$, then we have $x_{i+1}\\left(x_{i}+n \/ x_{i}\\right) \/ 2$, i.e., to $x_{i}^{2}>n$. Therefore $x_{i}=[\\sqrt{n}]$ holds for at least one $i \\leq n-[\\sqrt{n}]+1$. Remark. If $n+1$ is a perfect square, then $x_{i}=[\\sqrt{n}]$ implies $x_{i+1}=$ $[\\sqrt{n}]+1$. Otherwise, $x_{i}=[\\sqrt{n}]$ implies $x_{i+1}=[\\sqrt{n}]$.","problem_type":null,"tier":0} +{"year":"1977","problem_phase":"longlisted","problem":"29. (NET 4) (SL77-12).","solution":"29. Let us denote the midpoints of segments $L M, A N, B L, M N, B K, C M$, $N K, C L, D N, K L, D M, A K$ by $P_{1}, P_{2}, P_{3}, P_{4}, P_{5}, P_{6}, P_{7}, P_{8}, P_{9}, P_{10}$, $P_{11}, P_{12}$, respectively. We shall prove that the dodecagon $P_{1} P_{2} P_{3} \\ldots P_{11} P_{12}$ is regular. From $B L=B A$ and $\\angle A B L=30^{\\circ}$ it follows that $\\angle B A L=75^{\\circ}$. Similarly $\\angle D A M=75^{\\circ}$, and therefore $\\angle L A M=60^{\\circ}$, which together with $A L=A M$ implies that the triangle $A L M$ is equilateral. Now, from the triangles $O L M$ and $A L N$, we get ![](https:\/\/cdn.mathpix.com\/cropped\/2024_11_18_8e985d6b9c83aa3e9d0eg-426.jpg?height=383&width=433&top_left_y=233&top_left_x=863) $O P_{1}=L M \/ 2, O P_{2}=A L \/ 2$ and $O P_{2} \\| A L$. Hence $O P_{1}=O P_{2}$, $\\angle P_{1} O P_{2}=\\angle P_{1} A L=30^{\\circ}$ and $\\angle P_{2} O M=\\angle L A D=15^{\\circ}$. The desired result follows from symmetry.","problem_type":null,"tier":0} +{"year":"1977","problem_phase":"longlisted","problem":"3. (BUL 3) In a company of $n$ persons, each person has no more than $d$ acquaintances, and in that company there exists a group of $k$ persons, $k \\geq d$, who are not acquainted with each other. Prove that the number of acquainted pairs is not greater than $\\left[n^{2} \/ 4\\right]$.","solution":"3. Let $v_{1}, v_{2}, \\ldots, v_{k}$ be $k$ persons who are not acquainted with each other. Let us denote by $m$ the number of acquainted couples and by $d_{j}$ the number of acquaintances of person $v_{j}$. Then $m \\leq d_{k+1}+d_{k+2}+\\cdots+d_{n} \\leq d(n-k) \\leq k(n-k) \\leq\\left(\\frac{k+(n-k)}{2}\\right)^{2}=\\frac{n^{2}}{4}$.","problem_type":null,"tier":0} +{"year":"1977","problem_phase":"longlisted","problem":"30. (NET 5) A triangle $A B C$ with $\\angle A=30^{\\circ}$ and $\\angle C=54^{\\circ}$ is given. On $B C$ a point $D$ is chosen such that $\\angle C A D=12^{\\circ}$. On $A B$ a point $E$ is chosen such that $\\angle A C E=6^{\\circ}$. Let $S$ be the point of intersection of $A D$ and $C E$. Prove that $B S=B C$.","solution":"30. Suppose $\\angle S B A=x$. By the trigonometric form of Ceva's theorem we have $$ \\frac{\\sin \\left(96^{\\circ}-x\\right)}{\\sin x} \\frac{\\sin 18^{\\circ}}{\\sin 12^{\\circ}} \\frac{\\sin 6^{\\circ}}{\\sin 48^{\\circ}}=1 $$ We claim that $x=12^{\\circ}$ is a solution of this equation. To prove this, it is enough to show that $\\sin 84^{\\circ} \\sin 6^{\\circ} \\sin 18^{\\circ}=\\sin 48^{\\circ} \\sin 12^{\\circ} \\sin 12^{\\circ}$, which is equivalent to $\\sin 18^{\\circ}=2 \\sin 48^{\\circ} \\sin 12^{\\circ}=\\cos 36^{\\circ}-\\cos 60^{\\circ}$. The last equality can be checked directly. Since the equation is equivalent to $\\left(\\sin 96^{\\circ} \\cot x-\\cos 96^{\\circ}\\right) \\sin 6^{\\circ} \\sin 18^{\\circ}=$ $\\sin 48^{\\circ} \\sin 12^{\\circ}$, the solution $x \\in[0, \\pi)$ is unique. Hence $x=12^{\\circ}$. Second solution. We know that if $a, b, c, a^{\\prime}, b^{\\prime}, c^{\\prime}$ are points on the unit circle in the complex plane, the lines $a a^{\\prime}, b b^{\\prime}, c c^{\\prime}$ are concurrent if and only if $$ \\left(a-b^{\\prime}\\right)\\left(b-c^{\\prime}\\right)\\left(c-a^{\\prime}\\right)=\\left(a-c^{\\prime}\\right)\\left(b-a^{\\prime}\\right)\\left(c-b^{\\prime}\\right) $$ We shall prove that $x=12^{\\circ}$. We may suppose that $A B C$ is the triangle in the complex plane with vertices $a=1, b=\\epsilon^{9}, c=\\epsilon^{14}$, where $\\epsilon=$ $\\cos \\frac{\\pi}{15}+i \\sin \\frac{\\pi}{15}$. If $a^{\\prime}=\\epsilon^{12}, b^{\\prime}=\\epsilon^{28}, c^{\\prime}=\\epsilon$, our task is the same as proving that lines $a a^{\\prime}, b b^{\\prime}, c c^{\\prime}$ are concurrent, or by (1) that $$ \\left(1-\\epsilon^{28}\\right)\\left(\\epsilon^{9}-\\epsilon\\right)\\left(\\epsilon^{14}-\\epsilon^{12}\\right)-(1-\\epsilon)\\left(\\epsilon^{9}-\\epsilon^{12}\\right)\\left(\\epsilon^{14}-\\epsilon^{28}\\right)=0 . $$ The last equality holds, since the left-hand side is divisible by the minimum polynomial of $\\epsilon: z^{8}+z^{7}-z^{5}-z^{4}-z^{3}+z+1$.","problem_type":null,"tier":0} +{"year":"1977","problem_phase":"longlisted","problem":"31. (POL 1) Let $f$ be a function defined on the set of pairs of nonzero rational numbers whose values are positive real numbers. Suppose that $f$ satisfies the following conditions: (1) $f(a b, c)=f(a, c) f(b, c), f(c, a b)=f(c, a) f(c, b)$; (2) $f(a, 1-a)=1$. Prove that $f(a, a)=f(a,-a)=1, f(a, b) f(b, a)=1$.","solution":"31. We obtain from (1) that $f(1, c)=f(1, c) f(1, c)$; hence $f(1, c)=1$ and consequently $f(-1, c) f(-1, c)=f(1, c)=1$, i.e. $f(-1, c)=1$. Analogously, $f(c, 1)=f(c,-1)=1$. Clearly $f(1,1)=f(-1,1)=f(1,-1)=1$. Now let us assume that $a \\neq 1$. Observe that $f\\left(x^{-1}, y\\right)=f\\left(x, y^{-1}\\right)=f(x, y)^{-1}$. Thus by (1) and (2) we get $$ \\begin{aligned} 1 & =f(a, 1-a) f(1 \/ a, 1-1 \/ a) \\\\ & =f(a, 1-a) f\\left(a, \\frac{1}{1-1 \/ a}\\right)=f\\left(a, \\frac{1-a}{1-1 \/ a}\\right)=f(a,-a) \\end{aligned} $$ We now have $f(a, a)=f(a,-1) f(a,-a)=1 \\cdot 1=1$ and $1=f(a b, a b)=$ $f(a, a b) f(b, a b)=f(a, a) f(a, b) f(b, a) f(b, b)=f(a, b) f(b, a)$.","problem_type":null,"tier":0} +{"year":"1977","problem_phase":"longlisted","problem":"32. (POL 2) In a room there are nine men. Among every three of them there are two mutually acquainted. Prove that some four of them are mutually acquainted.","solution":"32. It is a known result that among six persons there are 3 mutually acquainted or 3 mutually unacquainted. By the condition of the problem the last case is excluded. If there is a man in the room who is not acquainted with four of the others, then these four men are mutually acquainted. Otherwise, each man is acquainted with at least five others, and since the sum of numbers of acquaintances of all men in the room is even, one of the men is acquainted with at least six men. Among these six there are three mutually acquainted, and they together with the first one make a group of four mutually acquainted men.","problem_type":null,"tier":0} +{"year":"1977","problem_phase":"longlisted","problem":"33. (POL 3) A circle $K$ centered at $(0,0)$ is given. Prove that for every vector $\\left(a_{1}, a_{2}\\right)$ there is a positive integer $n$ such that the circle $K$ translated by the vector $n\\left(a_{1}, a_{2}\\right)$ contains a lattice point (i.e., a point both of whose coordinates are integers).","solution":"33. Let $r$ be the radius of $K$ and $s>\\sqrt{2} \/ r$ an integer. Consider the points $A_{k}\\left(k a_{1}-\\left[k a_{1}\\right], k a_{2}-\\left[k a_{2}\\right]\\right)$, where $k=0,1,2, \\ldots, s^{2}$. Since all these points are in the unit square, two of them, say $A_{p}, A_{q}, q>p$, are in a small square with side $1 \/ s$, and consequently $A_{p} A_{q} \\leq \\sqrt{2} \/ s0$ for $j=1,2, \\ldots, n$ and $a_{1} \\leq \\cdots \\leq a_{n}3\\left(\\sum_{j=1}^{n} m_{j}\\right)\\left[\\sum_{j=1}^{n} m_{j}\\left(a_{j} b_{j}+b_{j} c_{j}+c_{j} a_{j}\\right)\\right] . $$","solution":"38. The condition says that the quadratic equation $f(x)=0$ has distinct real solutions, where $$ f(x)=3 x^{2} \\sum_{j=1}^{n} m_{j}-2 x \\sum_{j=1}^{n} m_{j}\\left(a_{j}+b_{j}+c_{j}\\right)+\\sum_{j=1}^{n} m_{j}\\left(a_{j} b_{j}+b_{j} c_{j}+c_{j} a_{j}\\right) $$ It is easy to verify that the function $f$ is the derivative of $$ F(x)=\\sum_{j=1}^{n} m_{j}\\left(x-a_{j}\\right)\\left(x-b_{j}\\right)\\left(x-c_{j}\\right) $$ Since $F\\left(a_{1}\\right) \\leq 0 \\leq F\\left(a_{n}\\right), F\\left(b_{1}\\right) \\leq 0 \\leq F\\left(b_{n}\\right)$ and $F\\left(c_{1}\\right) \\leq 0 \\leq F\\left(c_{n}\\right)$, $F(x)$ has three distinct real roots, and hence by Rolle's theorem its derivative $f(x)$ has two distinct real roots.","problem_type":null,"tier":0} +{"year":"1977","problem_phase":"longlisted","problem":"39. (ROM 5) Consider 37 distinct points in space, all with integer coordinates. Prove that we may find among them three distinct points such that their barycenter has integers coordinates.","solution":"39. By the pigeonhole principle, we can find 5 distinct points among the given 37 such that their $x$-coordinates are congruent and their $y$-coordinates are congruent modulo 3 . Now among these 5 points either there exist three with $z$-coordinates congruent modulo 3 , or there exist three whose $z$ coordinates are congruent to $0,1,2$ modulo 3 . These three points are the desired ones. Remark. The minimum number $n$ such that among any $n$ integer points in space one can find three points whose barycenter is an integer point is $n=19$. Each proof of this result seems to consist in studying a great number of cases.","problem_type":null,"tier":0} +{"year":"1977","problem_phase":"longlisted","problem":"4. (BUL 4) We are given $n$ points in space. Some pairs of these points are connected by line segments so that the number of segments equals $\\left[n^{2} \/ 4\\right]$, and a connected triangle exists. Prove that any point from which the maximal number of segments starts is a vertex of a connected triangle.","solution":"4. Consider any vertex $v_{n}$ from which the maximal number $d$ of segments start, and suppose it is not a vertex of a triangle. Let $\\mathcal{A}=$ $\\left\\{v_{1}, v_{2}, \\ldots, v_{d}\\right\\}$ be the set of points that are connected to $v_{n}$, and let $\\mathcal{B}=\\left\\{v_{d+1}, v_{d+2}, \\ldots, v_{n}\\right\\}$ be the set of the other points. Since $v_{n}$ is not a vertex of a triangle, there is no segment both of whose vertices lie in $\\mathcal{A}$; i.e., each segment has an end in $\\mathcal{B}$. Thus, if $d_{j}$ denotes the number of segments at $v_{j}$ and $m$ denotes the total number of segments, we have $$ m \\leq d_{d+1}+d_{d+2}+\\cdots+d_{n} \\leq d(n-d) \\leq\\left[\\frac{n^{2}}{4}\\right]=m $$ This means that each inequality must be equality, implying that each point in $\\mathcal{B}$ is a vertex of $d$ segments, and each of these segments has the other end in $\\mathcal{A}$. Then there is no triangle at all, which is a contradiction.","problem_type":null,"tier":0} +{"year":"1977","problem_phase":"longlisted","problem":"40. (SWE 1) The numbers $1,2,3, \\ldots, 64$ are placed on a chessboard, one number in each square. Consider all squares on the chessboard of size $2 \\times 2$. Prove that there are at least three such squares for which the sum of the 4 numbers contained exceeds 100.","solution":"40. Let us divide the chessboard into 16 squares $Q_{1}, Q_{2}, \\ldots, Q_{16}$ of size $2 \\times 2$. Let $s_{k}$ be the sum of numbers in $Q_{k}$, and let us assume that $s_{1} \\geq s_{2} \\geq$ $\\cdots \\geq s_{16}$. Since $s_{4}+s_{5}+\\cdots+s_{16} \\geq 1+2+\\cdots+52=1378$, we must have $s_{4} \\geq 100$ and hence $s_{1}, s_{2}, s_{3} \\geq 100$ as well.","problem_type":null,"tier":0} +{"year":"1977","problem_phase":"longlisted","problem":"41. (SWE 2) A wheel consists of a fixed circular disk and a mobile circular ring. On the disk the numbers $1,2,3, \\ldots, N$ are marked, and on the ring $N$ integers $a_{1}, a_{2}, \\ldots, a_{N}$ of sum 1 are marked (see the figure). The ring can be turned into $N$ different positions in which the numbers on the disk and on the ring match each other. Multiply every number on the ring with the corresponding number on the disk and form the sum of $N$ products. In this way a ![](https:\/\/cdn.mathpix.com\/cropped\/2024_11_18_8e985d6b9c83aa3e9d0eg-127.jpg?height=349&width=362&top_left_y=824&top_left_x=919) sum is obtained for every position of the ring. Prove that the $N$ sums are different.","solution":"41. The considered sums are congruent modulo $n$ to $S_{k}=\\sum_{i=1}^{N}(i+k) a_{i}$, $k=0,1, \\ldots, N-1$. Since $S_{k}=S_{0}+k\\left(a_{1}+\\cdots+a_{n}\\right)=S_{0}+k$, all these sums give distinct residues modulo $n$ and therefore are distinct.","problem_type":null,"tier":0} +{"year":"1977","problem_phase":"longlisted","problem":"42. (SWE 3) The sequence $a_{n, k}, k=1,2,3, \\ldots, 2^{n}, n=0,1,2, \\ldots$, is defined by the following recurrence formula: $$ \\begin{aligned} a_{1} & =2, \\quad a_{n, k}=2 a_{n-1, k}^{3}, \\quad a_{n, k+2^{n-1}}=\\frac{1}{2} a_{n-1, k}^{3} \\\\ \\text { for } k & =1,2,3, \\ldots, 2^{n-1}, n=0,1,2, \\ldots . \\end{aligned} $$ Prove that the numbers $a_{n, k}$ are all different.","solution":"42. It can be proved by induction on $n$ that $\\left\\{a_{n, k} \\mid 1 \\leq k \\leq 2^{n}\\right\\}=\\left\\{2^{m} \\mid m=3^{n}+3^{n-1} s_{1}+\\cdots+3^{1} s_{n-1}+s_{n}\\left(s_{i}= \\pm 1\\right)\\right\\}$. Thus the result is an immediate consequence of the following lemma. Lemma. Each positive integer $s$ can be uniquely represented in the form $$ s=3^{n}+3^{n-1} s_{1}+\\cdots+3^{1} s_{n-1}+s_{n}, \\quad \\text { where } s_{i} \\in\\{-1,0,1\\} $$ Proof. Both the existence and the uniqueness can be shown by simple induction on $s$. The statement is trivial for $s=1$, while for $s>1$ there exist $q \\in \\mathbb{N}, r \\in\\{-1,0,1\\}$ such that $s=3 q+r$, and $q$ has a unique representation of the form (1).","problem_type":null,"tier":0} +{"year":"1977","problem_phase":"longlisted","problem":"43. (FIN 1) Evaluate $$ S=\\sum_{k=1}^{n} k(k+1) \\cdots(k+p), $$ where $n$ and $p$ are positive integers.","solution":"43. Since $\\left.k(k+1) \\cdots(k+p)=(p+1)!\\binom{k+p}{p+1}=(p+1)!\\left[\\begin{array}{c}k+p+1 \\\\ p+2\\end{array}\\right)-\\binom{k+p}{p+2}\\right]$, it follows that $\\sum_{k=1}^{n} k(k+1) \\cdots(k+p)=(p+1)!\\binom{n+p+1}{p+2}=\\frac{n(n+1) \\cdots(n+p+1)}{p+2}$.","problem_type":null,"tier":0} +{"year":"1977","problem_phase":"longlisted","problem":"44. (FIN 2) Let $E$ be a finite set of points in space such that $E$ is not contained in a plane and no three points of $E$ are collinear. Show that $E$ contains the vertices of a tetrahedron $T=A B C D$ such that $T \\cap E=$ $\\{A, B, C, D\\}$ (including interior points of $T$ ) and such that the projection of $A$ onto the plane $B C D$ is inside a triangle that is similar to the triangle $B C D$ and whose sides have midpoints $B, C, D$.","solution":"44. Let $d(X, \\sigma)$ denote the distance from a point $X$ to a plane $\\sigma$. Let us consider the pair $(A, \\pi)$ where $A \\in E$ and $\\pi$ is a plane containing some three points $B, C, D \\in E$ such that $d(A, \\pi)$ is the smallest possible. We may suppose that $B, C, D$ are selected such that $\\triangle B C D$ contains no other points of $E$. Let $A^{\\prime}$ be the projection of $A$ on $\\pi$, and let $l_{b}, l_{c}, l_{d}$ be lines through $B, C, D$ parallel to $C D, D B, B C$ respectively. If $A^{\\prime}$ is in the half-plane determined by $l_{d}$ not containing $B C$, then $d(D, A B C) \\leq d\\left(A^{\\prime}, A B C\\right)|x|$. There is no loss of generality in assuming $x>0$. To obtain the estimate from below, set $$ \\begin{aligned} a_{1} & =f\\left(-\\frac{x+t}{2}\\right)-f(-(x+t)), & a_{2}=f(0)-f\\left(-\\frac{x+t}{2}\\right), \\\\ a_{3} & =f\\left(\\frac{x+t}{2}\\right)-f(0), & a_{4}=f(x+t)-f\\left(\\frac{x+t}{2}\\right) . \\end{aligned} $$ Since $-(x+t)\\frac{a_{4}}{a_{1}+a_{2}+a_{3}}>\\frac{a_{3} \/ 2}{4 a_{3}+2 a_{3}+a_{3}}=14^{-1} . $$ To obtain the estimate from above, set $$ \\begin{aligned} b_{1} & =f(0)-f\\left(-\\frac{x+t}{3}\\right), & b_{2}=f\\left(\\frac{x+t}{3}\\right)-f(0), \\\\ b_{3} & =f\\left(\\frac{2(x+t)}{3}\\right)-f\\left(\\frac{x+t}{3}\\right), & b_{4}=f(x+t)-f\\left(\\frac{2(x+t)}{3}\\right) . \\end{aligned} $$ If $t<2 x$, then $x-t<-(x+t) \/ 3$ and therefore $f(x)-f(x-t) \\geq b_{1}$. If $t \\geq 2 x$, then $(x+t) \/ 3 \\leq x$ and therefore $f(x)-f(x-t) \\geq b_{2}$. Since $2^{-1}2 C D \/ 3$, then $N$ coincides with C .","problem_type":null,"tier":0} +{"year":"1977","problem_phase":"longlisted","problem":"48. (USS 2) The intersection of a plane with a regular tetrahedron with edge $a$ is a quadrilateral with perimeter $P$. Prove that $2 a \\leq P \\leq 3 a$.","solution":"48. Let a plane cut the edges $A B, B C, C D, D A$ at points $K, L, M, N$ respectively. Let $D^{\\prime}, A^{\\prime}, B^{\\prime}$ be distinct points in the plane $A B C$ such that the triangles $B C D^{\\prime}, C D^{\\prime} A^{\\prime}, D^{\\prime} A^{\\prime} B^{\\prime}$ are equilateral, and $M^{\\prime} \\in\\left[C D^{\\prime}\\right], N^{\\prime} \\in\\left[D^{\\prime} A^{\\prime}\\right]$, and $K^{\\prime} \\in\\left[A^{\\prime} B^{\\prime}\\right]$ such that $C M^{\\prime}=C M$, $A^{\\prime} N^{\\prime}=A N$, and $A^{\\prime} K^{\\prime}=A K$. The perimeter $P$ of the quadrilateral $K L M N$ is equal to the length of the polygonal line $K L M^{\\prime} N^{\\prime} K^{\\prime}$, which is not less than $K K^{\\prime}$. It follows that $P \\geq 2 a$. ![](https:\/\/cdn.mathpix.com\/cropped\/2024_11_18_8e985d6b9c83aa3e9d0eg-430.jpg?height=265&width=548&top_left_y=997&top_left_x=810) Let us consider all quadrilaterals $K L M N$ that are obtained by intersecting the tetrahedron by a plane parallel to a fixed plane $\\alpha$. The lengths of the segments $K L, L M, M N, N K$ are linear functions in $A K$, and so is $P$. Thus $P$ takes its maximum at an endpoint of the interval, i.e., when the plane $K L M N$ passes through one of the vertices $A, B, C, D$, and it is easy to see that in this case $P \\leq 3 a$.","problem_type":null,"tier":0} +{"year":"1977","problem_phase":"longlisted","problem":"49. (USS 3) Find all pairs of integers $(p, q)$ for which all roots of the trinomials $x^{2}+p x+q$ and $x^{2}+q x+p$ are integers.","solution":"49. If one of $p, q$, say $p$, is zero, then $-q$ is a perfect square. Conversely, $(p, q)=\\left(0,-t^{2}\\right)$ and $(p, q)=\\left(-t^{2}, 0\\right)$ satisfy the conditions for $t \\in \\mathbb{Z}$. We now assume that $p, q$ are nonzero. If the trinomial $x^{2}+p x+q$ has two integer roots $x_{1}, x_{2}$, then $|q|=\\left|x_{1} x_{2}\\right| \\geq\\left|x_{1}\\right|+\\left|x_{2}\\right|-1 \\geq|p|-1$. Similarly, if $x^{2}+q x+p$ has integer roots, then $|p| \\geq|q|-1$ and $q^{2}-4 p$ is a square. Thus we have two cases to investigate: (i) $|p|=|q|$. Then $p^{2}-4 q=p^{2} \\pm 4 p$ is a square, so $(p, q)=(4,4)$. (ii) $|p|=|q| \\pm 1$. The solutions for $(p, q)$ are $(t,-1-t)$ for $t \\in \\mathbb{Z}$ and $(5,6)$, $(6,5)$.","problem_type":null,"tier":0} +{"year":"1977","problem_phase":"longlisted","problem":"5. (CZS 1) (SL77-2).","solution":"5. Let us denote by $I$ and $E$ the sets of interior boundary points and exterior boundary points. Let $A B C D$ be the square inscribed in the circle $k$ with sides parallel to the coordinate axes. Lines $A B, B C, C D, D A$ divide the plane into 9 regions: $\\mathcal{R}, \\mathcal{R}_{A}, \\mathcal{R}_{B}$, $\\mathcal{R}_{C}, \\mathcal{R}_{D}, \\mathcal{R}_{A B}, \\mathcal{R}_{B C}, \\mathcal{R}_{C D}, \\mathcal{R}_{D A}$. There is a unique pair of lattice points $A_{I} \\in \\mathcal{R}, A_{E} \\in$ $\\mathcal{R}_{A}$ that are opposite vertices of a unit square. We similarly define $B_{I}, C_{I}, D_{I}, B_{E}, C_{E}, D_{E}$. Let us form a graph $G$ by connecting each point from $E$ lying in $\\mathcal{R}_{A B}$ (respectively $\\mathcal{R}_{B C}, \\mathcal{R}_{C D}, \\mathcal{R}_{D A}$ ) to its up- ![](https:\/\/cdn.mathpix.com\/cropped\/2024_11_18_8e985d6b9c83aa3e9d0eg-420.jpg?height=389&width=505&top_left_y=271&top_left_x=825) per (respectively left, lower, right) neighbor point (which clearly belongs to $I$ ). It is easy to see that: (i) All vertices from $I$ other than $A_{I}, B_{I}, C_{I}, D_{I}$ have degree 1. (ii) $A_{E}$ is not in $E$ if and only if $A_{I} \\in I$ and $\\operatorname{deg} A_{I}=2$. (iii) No other lattice points inside $\\mathcal{R}_{A}$ belong to $E$. Thus if $m$ is the number of edges of the graph $G$ and $s$ is the number of points among $A_{E}, B_{E}, C_{E}$, and $D_{E}$ that are in $E$, using (i)-(iii) we easily obtain $|E|=m+s$ and $|I|=m-(4-s)=|E|+4$.","problem_type":null,"tier":0} +{"year":"1977","problem_phase":"longlisted","problem":"50. (USS 4) Determine all positive integers $n$ for which there exists a polynomial $P_{n}(x)$ of degree $n$ with integer coefficients that is equal to $n$ at $n$ different integer points and that equals zero at zero.","solution":"50. Suppose that $P_{n}(x)=n$ for $x \\in\\left\\{x_{1}, x_{2}, \\ldots, x_{n}\\right\\}$. Then $$ P_{n}(x)=\\left(x-x_{1}\\right)\\left(x-x_{2}\\right) \\cdots\\left(x-x_{n}\\right)+n . $$ From $P_{n}(0)=0$ we obtain $n=\\left|x_{1} x_{2} \\cdots x_{n}\\right| \\geq 2^{n-2}$ (because at least $n-2$ factors are different from $\\pm 1$ ) and therefore $n \\geq 2^{n-2}$. It follows that $n \\leq 4$. For each positive integer $n \\leq 4$ there exists a polynomial $P_{n}$. Here is the list of such polynomials: $$ \\begin{array}{ll} n=1: \\pm x, & n=2: 2 x^{2}, x^{2} \\pm x,-x^{2} \\pm 3 x \\\\ n=3: \\pm\\left(x^{3}-x\\right)+3 x^{2}, & n=4:-x^{4}+5 x^{2} . \\end{array} $$","problem_type":null,"tier":0} +{"year":"1977","problem_phase":"longlisted","problem":"51. (USS 5) Several segments, which we shall call white, are given, and the sum of their lengths is 1 . Several other segments, which we shall call black, are given, and the sum of their lengths is 1 . Prove that every such system of segments can be distributed on the segment that is 1.51 long in the following way: Segments of the same color are disjoint, and segments of different colors are either disjoint or one is inside the other. Prove that there exists a system that cannot be distributed in that way on the segment that is 1.49 long.","solution":"51. We shall use the following algorithm: Choose a segment of maximum length (\"basic\" segment) and put on it unused segments of the opposite color without overlapping, each time of the maximum possible length, as long as it is possible. Repeat the procedure with remaining segments until all the segments are used. Let us suppose that the last basic segment is black. Then the length of the used part of any white basic segment is greater than the free part, and consequently at least one-half of the length of the white segments has been used more than once. Therefore all basic segments have total length at most 1.5 and can be distributed on a segment of length 1.51. On the other hand, if we are given two white segments of lengths 0.5 and two black segments of lengths 0.999 and 0.001 , we cannot distribute them on a segment of length less than 1.499.","problem_type":null,"tier":0} +{"year":"1977","problem_phase":"longlisted","problem":"52. (USA 1) Two perpendicular chords are drawn through a given interior point $P$ of a circle with radius $R$. Determine, with proof, the maximum and the minimum of the sum of the lengths of these two chords if the distance from $P$ to the center of the circle is $k R$.","solution":"52. The maximum and minimum are $2 R \\sqrt{4-2 k^{2}}$ and $2 R\\left(1+\\sqrt{1-k^{2}}\\right)$ respectively.","problem_type":null,"tier":0} +{"year":"1977","problem_phase":"longlisted","problem":"53. (USA 2) Find all pairs of integers $a$ and $b$ for which $$ 7 a+14 b=5 a^{2}+5 a b+5 b^{2} $$","solution":"53. The discriminant of the given equation considered as a quadratic equation in $b$ is $196-75 a^{2}$. Thus $75 a^{2} \\leq 196$ and hence $-1 \\leq a \\leq 1$. Now the integer solutions of the given equation are easily found: $(-1,3),(0,0),(1,2)$.","problem_type":null,"tier":0} +{"year":"1977","problem_phase":"longlisted","problem":"54. (USA 3) If $0 \\leq a \\leq b \\leq c \\leq d$, prove that $$ a^{b} b^{c} c^{d} d^{a} \\geq b^{a} c^{b} d^{c} a^{d} $$","solution":"54. We shall use the following lemma. Lemma. If a real function $f$ is convex on the interval $I$ and $x, y, z \\in I$, $x \\leq y \\leq z$, then $$ (y-z) f(x)+(z-x) f(y)+(x-y) f(z) \\leq 0 . $$ Proof. The inequality is obvious for $x=y=z$. If $xs_{k+m}$ for $0 \\leq k \\leq l-m$ and $s_{k}a_{k}$ in the ring, then $q n=r m$, which implies $m^{\\prime}\\left|q, n^{\\prime}\\right| p$ and thus $k=p+q \\geq m^{\\prime}+n^{\\prime}$. But since all $i_{1}, i_{2}, \\ldots, i_{k}$ are congruent modulo $d$, we have $k \\leq m^{\\prime}+n^{\\prime}-1$, a contradiction. Hence there exists a sequence of length $m+n-d-1$ with the required property.","problem_type":null,"tier":0} +{"year":"1977","problem_phase":"longlisted","problem":"58. (VIE 2) Prove that for every triangle the following inequality holds: $$ \\frac{a b+b c+c a}{4 S} \\geq \\cot \\frac{\\pi}{6} $$ where $a, b, c$ are lengths of the sides and $S$ is the area of the triangle.","solution":"58. The following inequality (Finsler and Hadwiger, 1938) is sharper than the one we have to prove: $$ 2 a b+2 b c+2 c a-a^{2}-b^{2}-c^{2} \\geq 4 S \\sqrt{3} $$ First proof. Let us set $2 x=b+c-a, 2 y=c+a-b, 2 z=a+b-c$. Then $x, y, z>0$ and the inequality (1) becomes $$ y^{2} z^{2}+z^{2} x^{2}+x^{2} y^{2} \\geq x y z(x+y+z) $$ which is equivalent to the obvious inequality $(x y-y z)^{2}+(y z-z x)^{2}+$ $(z x-x y)^{2} \\geq 0$. Second proof. Using the known relations for a triangle $$ \\begin{aligned} a^{2}+b^{2}+c^{2} & =2 s^{2}-2 r^{2}-8 r R, \\\\ a b+b c+c a & =s^{2}+r^{2}+4 r R, \\\\ S & =r s, \\end{aligned} $$ where $r$ and $R$ are the radii of the incircle and the circumcircle, $s$ the semiperimeter and $S$ the area, we can transform (1) into $$ s \\sqrt{3} \\leq 4 R+r . $$ The last inequality is a consequence of the inequalities $2 r \\leq R$ and $s^{2} \\leq$ $4 R^{2}+4 R r+3 r^{2}$, where the last one follows from the equality $H I^{2}=$ $4 R^{2}+4 R r+3 r^{2}-s^{2}$ ( $H$ and $I$ being the orthocenter and the incenter of the triangle).","problem_type":null,"tier":0} +{"year":"1977","problem_phase":"longlisted","problem":"59. (VIE 3) (SL77-16).","solution":"59. Let us consider the set $R$ of pairs of coordinates of the points from $E$ reduced modulo 3 . If some element of $R$ occurs thrice, then the corresponding points are vertices of a triangle with integer barycenter. Also, no three elements from $E$ can have distinct $x$-coordinates and distinct $y$ coordinates. By an easy discussion we can conclude that the set $R$ contains at most four elements. Hence $|E| \\leq 8$. An example of a set $E$ consisting of 8 points that satisfies the required condition is $$ E=\\{(0,0),(1,0),(0,1),(1,1),(3,6),(4,6),(3,7),(4,7)\\} $$","problem_type":null,"tier":0} +{"year":"1977","problem_phase":"longlisted","problem":"6. (CZS 2) Let $x_{1}, x_{2}, \\ldots, x_{n}(n \\geq 1)$ be real numbers such that $0 \\leq x_{j} \\leq \\pi$, $j=1,2, \\ldots, n$. Prove that if $\\sum_{j=1}^{n}\\left(\\cos x_{j}+1\\right)$ is an odd integer, then $\\sum_{j=1}^{n} \\sin x_{j} \\geq 1$.","solution":"6. Let $\\langle y\\rangle$ denote the distance from $y \\in \\mathbb{R}$ to the closest even integer. We claim that $$ \\langle 1+\\cos x\\rangle \\leq \\sin x \\quad \\text { for all } x \\in[0, \\pi] $$ Indeed, if $\\cos x \\geq 0$, then $\\langle 1+\\cos x\\rangle=1-\\cos x \\leq 1-\\cos ^{2} x=\\sin ^{2} x \\leq$ $\\sin x$; the proof is similar if $\\cos x<0$. We note that $\\langle x+y\\rangle \\leq\\langle x\\rangle+\\langle y\\rangle$ holds for all $x, y \\in \\mathbb{R}$. Therefore $$ \\sum_{j=1}^{n} \\sin x_{j} \\geq \\sum_{j=1}^{n}\\left\\langle 1+\\cos x_{j}\\right\\rangle \\geq\\left\\langle\\sum_{j=1}^{n}\\left(1+\\cos x_{j}\\right)\\right\\rangle=1 $$","problem_type":null,"tier":0} +{"year":"1977","problem_phase":"longlisted","problem":"60. (VIE 4) Suppose $x_{0}, x_{1}, \\ldots, x_{n}$ are integers and $x_{0}>x_{1}>\\cdots>x_{n}$. Prove that at least one of the numbers $\\left|F\\left(x_{0}\\right)\\right|,\\left|F\\left(x_{1}\\right)\\right|,\\left|F\\left(x_{2}\\right)\\right|, \\ldots$, $\\left|F\\left(x_{n}\\right)\\right|$, where $$ F(x)=x^{n}+a_{1} x^{n-1}+\\cdots+a_{n}, \\quad a_{i} \\in \\mathbb{R}, \\quad i=1, \\ldots, n $$ is greater than $\\frac{n!}{2^{n}}$.","solution":"60. By Lagrange's interpolation formula we have $$ F(x)=\\sum_{j=0}^{n} F\\left(x_{j}\\right) \\frac{\\prod_{i \\neq j}\\left(x-x_{j}\\right)}{\\prod_{i \\neq j}\\left(x_{i}-x_{j}\\right)} . $$ Since the leading coefficient in $F(x)$ is 1 , it follows that $$ 1=\\sum_{j=0}^{n} \\frac{F\\left(x_{j}\\right)}{\\prod_{i \\neq j}\\left(x_{i}-x_{j}\\right)} $$ Since $$ \\left|\\prod_{i \\neq j}\\left(x_{i}-x_{j}\\right)\\right|=\\prod_{i=0}^{j-1}\\left|x_{i}-x_{j}\\right| \\prod_{i=j+1}^{n}\\left|x_{i}-x_{j}\\right| \\geq j!(n-j)! $$ we have $$ 1 \\leq \\sum_{j=0}^{n} \\frac{\\left|F\\left(x_{j}\\right)\\right|}{\\left|\\prod_{i \\neq j}\\left(x_{i}-x_{j}\\right)\\right|} \\leq \\frac{1}{n!} \\sum_{j=0}^{n}\\binom{n}{j}\\left|F\\left(x_{j}\\right)\\right| \\leq \\frac{2^{n}}{n!} \\max \\left|F\\left(x_{j}\\right)\\right| . $$ Now the required inequality follows immediately.","problem_type":null,"tier":0} +{"year":"1977","problem_phase":"longlisted","problem":"7. (CZS 3) Prove the following assertion: If $c_{1}, c_{2}, \\ldots, c_{n}(n \\geq 2)$ are real numbers such that $$ (n-1)\\left(c_{1}^{2}+c_{2}^{2}+\\cdots+c_{n}^{2}\\right)=\\left(c_{1}+c_{2}+\\cdots+c_{n}\\right)^{2} $$ then either all these numbers are nonnegative or all these numbers are nonpositive.","solution":"7. Let us suppose that $c_{1} \\leq c_{2} \\leq \\cdots \\leq c_{n}$ and that $c_{1}<0k$, i.e., at least $k+1$, belong to the same subset, say $M_{t}$. Then we choose $s, t$. The latter case is similar. Second solution. For all $i, j \\in\\{1,2, \\ldots, k\\}$, consider the set $N_{i j}=\\{r \\mid$ $\\left.2 r \\in M_{i}, 2 r-1 \\in M_{j}\\right\\}$. Then $\\left\\{N_{i j} \\mid i, j\\right\\}$ is a partition of $\\{1,2, \\ldots, n\\}$ into $k^{2}$ subsets. For $n \\geq k^{3}+1$ one of these subsets contains at least $k+1$ elements, and the statement follows. Remark. The statement is not necessarily true when $n=k^{3}$.","problem_type":null,"tier":0} +{"year":"1978","problem_phase":"shortlisted","problem":"10. (NET 1) ${ }^{\\text {IMO6 }}$ An international society has its members in 6 different countries. The list of members contains 1978 names, numbered $1,2, \\ldots$, 1978. Prove that there is at least one member whose number is the sum of the numbers of two, not necessarily distinct, of his compatriots.","solution":"10. Assume the opposite. One of the countries, say $A$, contains at least 330 members $a_{1}, a_{2}, \\ldots, a_{330}$ of the society $(6 \\cdot 329=1974)$. Consider the differences $a_{330}-a_{i},=1,2, \\ldots, 329$ : the members with these numbers are not in $A$, so at least 66 of them, $a_{330}-a_{i_{1}}, \\ldots, a_{330}-a_{i_{66}}$, belong to the same country, say $B$. Then the differences $\\left(a_{i_{66}}-a_{330}\\right)-\\left(a_{i_{j}}-a_{330}\\right)=$ $a_{i_{66}}-a_{i_{j}}, j=1,2, \\ldots, 65$, are neither in $A$ nor in $B$. Continuing this procedure, we find that 17 of these differences are in the same country, say $C$, then 6 among 16 differences of themselves in a country $D$, and 3 among 5 differences of themselves in $E$; finally, one among two differences of these 3 differences belong to country $F$, so that the difference of themselves cannot be in any country. This is a contradiction. Remark. The following stronger $([6!e]=1957)$ statement can be proved in the same way. Schurr's lemma. If $n$ is a natural number and $e$ the logarithm base, then for every partition of the set $\\{1,2, \\ldots,[e n!]\\}$ into $n$ subsets one of these subsets contains some two elements and their difference.","problem_type":null,"tier":0} +{"year":"1978","problem_phase":"shortlisted","problem":"11. (SWE 2) A function $f: I \\rightarrow \\mathbb{R}$, defined on an interval $I$, is called concave if $f(\\theta x+(1-\\theta) y) \\geq \\theta f(x)+(1-\\theta) f(y)$ for all $x, y \\in I$ and $0 \\leq \\theta \\leq 1$. Assume that the functions $f_{1}, \\ldots, f_{n}$, having all nonnegative values, are concave. Prove that the function $\\left(f_{1} f_{2} \\ldots f_{n}\\right)^{1 \/ n}$ is concave.","solution":"11. Set $F(x)=f_{1}(x) f_{2}(x) \\cdots f_{n}(x)$ : we must prove concavity of $F^{1 \/ n}$. By the assumption, $$ \\begin{aligned} F(\\theta x+(1-\\theta) y) & \\geq \\prod_{i=1}^{n}\\left[\\theta f_{i}(x)+(1-\\theta) f(y)\\right] \\\\ & =\\sum_{k=0}^{n} \\theta^{k}(1-\\theta)^{n-k} \\sum f_{i_{1}}(x) \\ldots f_{i_{k}}(x) f_{i_{k+1}}(y) f_{i_{n}}(y) \\end{aligned} $$ where the second sum goes through all $\\binom{n}{k} k$-subsets $\\left\\{i_{1}, \\ldots, i_{k}\\right\\}$ of $\\{1, \\ldots, n\\}$. The inequality between the arithmetic and geometric means now gives us $$ \\sum f_{i_{1}}(x) f_{i_{2}}(x) \\cdots f_{i_{k}}(x) f_{i_{k+1}}(y) f_{i_{n}}(y) \\geq\\binom{ n}{k} F(x)^{k \/ n} F(y)^{(n-k) \/ n} $$ Inserting this in the above inequality and using the binomial formula, we finally obtain $$ \\begin{aligned} F(\\theta x+(1-\\theta) y) & \\geq \\sum_{k=0}^{n} \\theta^{k}(1-\\theta)^{n-k}\\binom{n}{k} F(x)^{k \/ n} F(y)^{(n-k) \/ n} \\\\ & =\\left(\\theta F(x)^{1 \/ n}+(1-\\theta) F(y)^{1 \/ n}\\right)^{n}, \\end{aligned} $$ which proves the assertion.","problem_type":null,"tier":0} +{"year":"1978","problem_phase":"shortlisted","problem":"12. (USA 1) ${ }^{\\mathrm{IMO} 4}$ In a triangle $A B C$ we have $A B=A C$. A circle is tangent internally to the circumcircle of $A B C$ and also to the sides $A B, A C$, at $P, Q$ respectively. Prove that the midpoint of $P Q$ is the center of the incircle of $A B C$.","solution":"12. Let $O$ be the center of the smaller circle, $T$ its contact point with the circumcircle of $A B C$, and $J$ the midpoint of segment $B C$. The figure is symmetric with respect to the line through $A, O, J, T$. A homothety centered at $A$ taking $T$ into $J$ will take the smaller circle into the incircle of $A B C$, hence will take $O$ into the incenter $I$. On the other hand, $\\angle A B T=\\angle A C T=90^{\\circ}$ implies that the quadrilaterals $A B T C$ and $A P O Q$ are similar. Hence the above homothety also maps $O$ to the midpoint of $P Q$. This finishes the proof. Remark. The assertion is true for a nonisosceles triangle $A B C$ as well, and this (more difficult) case is a matter of SL93-3.","problem_type":null,"tier":0} +{"year":"1978","problem_phase":"shortlisted","problem":"13. (USA 6) ${ }^{\\mathrm{IMO} 2}$ Given any point $P$ in the interior of a sphere with radius $R$, three mutually perpendicular segments $P A, P B, P C$ are drawn terminating on the sphere and having one common vertex in $P$. Consider the rectangular parallelepiped of which $P A, P B, P C$ are coterminal edges. Find the locus of the point $Q$ that is diagonally opposite $P$ in the parallelepiped when $P$ and the sphere are fixed.","solution":"13. Lemma. If $M N P Q$ is a rectangle and $O$ any point in space, then $O M^{2}+$ $O P^{2}=O N^{2}+O Q^{2}$. Proof. Let $O_{1}$ be the projection of $O$ onto $M N P Q$, and $m, n, p, q$ denote the distances of $O_{1}$ from $M N, N P, P Q, Q M$, respectively. Then $O M^{2}=O O_{1}^{2}+q^{2}+m^{2}, O N^{2}=O O_{1}^{2}+m^{2}+n^{2}, O P^{2}=O O_{1}^{2}+n^{2}+p^{2}$, $O Q^{2}=O O_{1}^{2}+p^{2}+q^{2}$, and the lemma follows immediately. Now we return to the problem. Let $O$ be the center of the given sphere $S$, and $X$ the point opposite $P$ in the face of the parallelepiped through $P, A, B$. By the lemma, we have $O P^{2}+O Q^{2}=O C^{2}+O X^{2}$ and $O P^{2}+$ $O X^{2}=O A^{2}+O B^{2}$. Hence $2 O P^{2}+O Q^{2}=O A^{2}+O B^{2}+O C^{2}=3 R^{2}$, i.e. $O Q=\\sqrt{3 R^{2}-O P^{2}}>R$. We claim that the locus of $Q$ is the whole sphere $\\left(O, \\sqrt{3 R^{2}-O P^{2}}\\right)$. Choose any point $Q$ on this sphere. Since $O Q>R>O P$, the sphere with diameter $P Q$ intersects $S$ on a circle. Let $C$ be an arbitrary point on this circle, and $X$ the point opposite $C$ in the rectangle $P C Q X$. By the lemma, $O P^{2}+O Q^{2}=O C^{2}+O X^{2}$, hence $O X^{2}=2 R^{2}-O P^{2}>R^{2}$. The plane passing through $P$ and perpendicular to $P C$ intersects $S$ in a circle $\\gamma$; both $P, X$ belong to this plane, $P$ being inside and $X$ outside the circle, so that the circle with diameter $P X$ intersects $\\gamma$ at some point $B$. Finally, we choose $A$ to be the point opposite $B$ in the rectangle $P B X A$ : we deduce that $O A^{2}+O B^{2}=O P^{2}+O X^{2}$, and consequently $A \\in S$. By the construction, there is a rectangular parallelepiped through $P, A, B, C, X, Q$.","problem_type":null,"tier":0} +{"year":"1978","problem_phase":"shortlisted","problem":"14. (VIE 2) Prove that it is possible to place $2 n(2 n+1)$ parallelepipedic (rectangular) pieces of soap of dimensions $1 \\times 2 \\times(n+1)$ in a cubic box with edge $2 n+1$ if and only if $n$ is even or $n=1$. Remark. It is assumed that the edges of the pieces of soap are parallel to the edges of the box.","solution":"14. We label the cells of the cube by $\\left(a_{1}, a_{2}, a_{3}\\right), a_{i} \\in\\{1,2, \\ldots, 2 n+1\\}$, in a natural way: for example, as Cartesian coordinates of centers of the cells $\\left((1,1,1)\\right.$ is one corner, etc.). Notice that there should be $(2 n+1)^{3}-$ $2 n(2 n+1) \\cdot 2(n+1)=2 n+1$ void cells, i.e., those not covered by any piece of soap. $n=1$. In this case, six pieces of soap $1 \\times 2 \\times 2$ can be placed on the following positions: $[(1,1,1),(2,2,1)],[(3,1,1),(3,2,2)],[(2,3,1),(3,3,2)]$ and the symmetric ones with respect to the center of the box. (Here $[A, B]$ denotes the rectangle with opposite corners at $A, B$.) $n$ is even. Each of the $2 n+1$ planes $P_{k}=\\left\\{\\left(a_{1}, a_{2}, k\\right) \\mid a_{i}=1, \\ldots, 2 n+1\\right\\}$ can receive $2 n$ pieces of soap: In fact, $P_{k}$ can be partitioned into four $n \\times(n+1)$ rectangles at the corners and the central cell, while an $n \\times(n+1)$ rectangle can receive $n \/ 2$ pieces of soap. $n$ is odd, $n>1$. Let us color a cell $\\left(a_{1}, a_{2}, a_{3}\\right)$ blue, red, or yellow if exactly three, two or one $a_{i}$ respectively is equal to $n+1$. Thus there are 1 blue, $6 n$ red, and $12 n^{2}$ yellow cells. We notice that each piece of soap must contain at least one colored cell (because $2(n+1)>2 n+1)$. Also, every piece of soap contains an even number (actually, $1 \\cdot 2,1(n+1)$, or $2(n+1)$ ) of cells in $P_{k}$. On the other hand, $2 n+1$ cells are void, i.e., one in each plane. There are several cases for a piece of soap $S$ : (i) $S$ consists of 1 blue, $n+1$ red and $n$ yellow cells; (ii) $S$ consists of 2 red and $2 n$ yellow cells (and no blue cells); (iii) $S$ contains 1 red cell, $n+1$ yellow cells, and the are rest uncolored; (iv) $S$ contains 2 yellow cells and no blue or red ones. From the descriptions of the last three cases, we can deduce that if $S$ contains $r$ red cells and no blue, then it contains exactly $2+(n-1) r$ red ones. $\\quad(*)$ Now, let $B_{1}, \\ldots, B_{k}$ be all boxes put in the cube, with a possible exception for the one covering the blue cell: thus $k=2 n(2 n+1)$ if the blue cell is void, or $k=2 n(2 n+1)-1$ otherwise. Let $r_{i}$ and $y_{i}$ respectively be the numbers of red and yellow cells inside $B_{i}$. By (*) we have $y_{1}+\\cdots+y_{k}=2 k+(n-1)\\left(r_{1}+\\cdots+r_{k}\\right)$. If the blue cell is void, then $r_{1}+\\cdots+r_{k}=6 n$ and consequently $y_{1}+\\cdots+y_{k}=$ $4 n(2 n+1)+6 n(n-1)=14 n^{2}-2 n$, which is impossible because there are only $12 n^{2}<14 n^{2}-2 n$ yellow cells. Otherwise, $r_{1}+\\cdots+r_{k} \\geq 5 n-2$ (because $n+1$ red cells are covered by the box containing the blue cell, and one can be void) and consequently $y_{1}+\\cdots+y_{k} \\geq 4 n(2 n+$ $1)-2+(n-1)(5 n-2)=13 n^{2}-3 n$; since there are $n$ more yellow cells in the box containing the blue one, this counts for $13 n^{2}-2 n>12 n^{2}$ ( $n \\geq 3$ ), again impossible. Remark. The following solution of the case $n$ odd is simpler, but does not work for $n=3$. For $k=1,2,3$, let $m_{k}$ be the number of pieces whose long sides are perpendicular to the plane $\\pi_{k}\\left(a_{k}=n+1\\right)$. Each of these $m_{k}$ pieces covers exactly 2 cells of $\\pi_{k}$, while any other piece covers $n+1$, $2(n+1)$, or none. It follows that $4 n^{2}+4 n-2 m_{k}$ is divisible by $n+1$, and so is $2 m_{k}$. This further implies that $2 m_{1}+2 m_{2}+2 m_{3}=4 n(2 n+1)$ is a multiple of $n+1$, which is impossible for each odd $n$ except $n=1$ and $n=3$.","problem_type":null,"tier":0} +{"year":"1978","problem_phase":"shortlisted","problem":"15. (YUG 1) Let $p$ be a prime and $A=\\left\\{a_{1}, \\ldots, a_{p-1}\\right\\}$ an arbitrary subset of the set of natural numbers such that none of its elements is divisible by $p$. Let us define a mapping $f$ from $\\mathcal{P}(A)$ (the set of all subsets of $A$ ) to the set $P=\\{0,1, \\ldots, p-1\\}$ in the following way: (i) if $B=\\left\\{a_{i_{1}}, \\ldots, a_{i_{k}}\\right\\} \\subset A$ and $\\sum_{j=1}^{k} a_{i_{j}} \\equiv n(\\bmod p)$, then $f(B)=n$, (ii) $f(\\emptyset)=0, \\emptyset$ being the empty set. Prove that for each $n \\in P$ there exists $B \\subset A$ such that $f(B)=n$.","solution":"15. Let $C_{n}=\\left\\{a_{1}, \\ldots, a_{n}\\right\\}\\left(C_{0}=\\emptyset\\right)$ and $P_{n}=\\left\\{f(B) \\mid B \\subseteq C_{n}\\right\\}$. We claim that $P_{n}$ contains at least $n+1$ distinct elements. First note that $P_{0}=\\{0\\}$ contains one element. Suppose that $P_{n+1}=P_{n}$ for some $n$. Since $P_{n+1}=$ $\\left\\{a_{n+1}+r \\mid r \\in P_{n}\\right\\}$, it follows that for each $r \\in P_{n}$, also $r+b_{n} \\in P_{n}$. Then obviously $0 \\in P_{n}$ implies $k b_{n} \\in P_{n}$ for all $k$; therefore $P_{n}=P$ has at least $p \\geq n+1$ elements. Otherwise, if $P_{n+1} \\supset P_{n}$ for all $n$, then $\\left|P_{n+1}\\right| \\geq\\left|P_{n}\\right|+1$ and hence $\\left|P_{n}\\right| \\geq n+1$, as claimed. Consequently, $\\left|P_{p-1}\\right| \\geq p$. (All the operations here are performed modulo $p$.)","problem_type":null,"tier":0} +{"year":"1978","problem_phase":"shortlisted","problem":"16. (YUG 2) Determine all the triples $(a, b, c)$ of positive real numbers such that the system $$ \\begin{array}{r} a x+b y-c z=0 \\\\ a \\sqrt{1-x^{2}}+b \\sqrt{1-y^{2}}-c \\sqrt{1-z^{2}}=0 \\end{array} $$ is compatible in the set of real numbers, and then find all its real solutions.","solution":"16. Clearly $|x| \\leq 1$. As $x$ runs over $[-1,1]$, the vector $u=\\left(a x, a \\sqrt{1-x^{2}}\\right)$ runs over all vectors of length $a$ in the plane having a nonnegative vertical component. Putting $v=\\left(b y, b \\sqrt{1-y^{2}}\\right), w=\\left(c z, c \\sqrt{1-z^{2}}\\right)$, the system becomes $u+v=w$, with vectors $u, v, w$ of lengths $a, b, c$ respectively in the upper half-plane. Then $a, b, c$ are sides of a (possibly degenerate) triangle; i.e, $|a-b| \\leq c \\leq a+b$ is a necessary condition. Conversely, if $a, b, c$ satisfy this condition, one constructs a triangle $O M N$ with $O M=a, O N=b, M N=c$. If the vectors $\\overrightarrow{O M}, \\overrightarrow{O N}$ have a positive nonnegative component, then so does their sum. For every such triangle, putting $u=\\overrightarrow{O M}, v=\\overrightarrow{O N}$, and $w=\\overrightarrow{O M}+\\overrightarrow{O N}$ gives a solution, and every solution is given by one such triangle. This triangle is uniquely determined up to congruence: $\\alpha=\\angle M O N=\\angle(u, v)$ and $\\beta=\\angle(u, w)$. Therefore, all solutions of the system are $$ \\begin{aligned} & x=\\cos t, \\quad y=\\cos (t+\\alpha), \\quad z=y=\\cos (t+\\beta), \\quad t \\in[0, \\pi-\\alpha] \\quad \\text { or } \\\\ & x=\\cos t, \\quad y=\\cos (t-\\alpha), \\quad z=y=\\cos (t-\\beta), \\quad t \\in[\\alpha, \\pi] . \\end{aligned} $$","problem_type":null,"tier":0} +{"year":"1978","problem_phase":"shortlisted","problem":"17. (FRA 3) Prove that for any positive integers $x, y, z$ with $x y-z^{2}=1$ one can find nonnegative integers $a, b, c, d$ such that $x=a^{2}+b^{2}, y=c^{2}+d^{2}$, $z=a c+b d$. Set $z=(2 q)$ ! to deduce that for any prime number $p=4 q+1, p$ can be represented as the sum of squares of two integers.","solution":"17. Let $z_{0} \\geq 1$ be a positive integer. Supposing that the statement is true for all triples $(x, y, z)$ with $z1$ and $x_{0}2\\left(z_{0}-z_{0}\\right)=0$. Moreover, $x y-z^{2}=x_{0}\\left(x_{0}+y_{0}-\\right.$ $\\left.2 z_{0}\\right)-\\left(z_{0}-x_{0}\\right)^{2}=x_{0} y_{0}-z_{0}^{2}=1$ and $z0$. Now it follows from the first part that there exist integers $a, b$ such that $x=p=a^{2}+b^{2}$. Second solution. Another possibility is using arithmetic of Gaussian integers. Lemma. Suppose $m, n, p, q$ are elements of $\\mathbb{Z}$ or any other unique factorization domain, with $m n=p q$. then there exist elements $a, b, c, d$ such that $m=a b, n=c d, p=a c, q=b d$. Proof is direct, for example using factorization of $a, b, c, d$ into primes. We now apply this lemma to the Gaussian integers in our case (because $\\mathbb{Z}[i]$ has the unique factorization property), having in mind that $x y=$ $z^{2}+1=(z+i)(z-i)$. We obtain (1) $x=a b$, (2) $y=c d$, (3) $z+i=a c$, (4) $z-i=b d$ for some $a, b, c, d \\in \\mathbb{Z}[i]$. Let $a=a_{1}+a_{2} i$, etc. By (3) and (4), $\\operatorname{gcd}\\left(a_{1}, a_{2}\\right)=$ $\\cdots=\\operatorname{gcd}\\left(d_{1}, d_{2}\\right)$. Then (1) and (2) give us $b=\\bar{a}, c=\\bar{d}$. The statement follows at once: $x=a b=a \\bar{a}=a_{1}^{2}+a_{2}^{2}, y=d \\bar{d}=d_{1}^{2}+d_{2}^{2}$ and $z+i=$ $\\left(a_{1} d_{1}+a_{2} d_{2}\\right)+\\imath\\left(a_{2} d_{1}-a_{1} d_{2}\\right) \\Rightarrow z=a_{1} d_{1}+a_{2} d_{2}$.","problem_type":null,"tier":0} +{"year":"1978","problem_phase":"shortlisted","problem":"2. (BUL 4) Two identically oriented equilateral triangles, $A B C$ with center $S$ and $A^{\\prime} B^{\\prime} C$, are given in the plane. We also have $A^{\\prime} \\neq S$ and $B^{\\prime} \\neq S$. If $M$ is the midpoint of $A^{\\prime} B$ and $N$ the midpoint of $A B^{\\prime}$, prove that the triangles $S B^{\\prime} M$ and $S A^{\\prime} N$ are similar.","solution":"2. Consider the transformation $\\phi$ of the plane defined as the homothety $\\mathcal{H}$ with center $B$ and coefficient 2 followed by the rotation $\\mathcal{R}$ about the center $O$ through an angle of $60^{\\circ}$. Being direct, this mapping must be a rotational homothety. We also see that $\\mathcal{H}$ maps $S$ into the point symmetric to $S$ with respect to $O A$, and $\\mathcal{R}$ takes it back to $S$. Hence $S$ is a fixed point, and is consequently also the center of $\\phi$. Therefore $\\phi$ is the rotational homothety about $S$ with the angle $60^{\\circ}$ ![](https:\/\/cdn.mathpix.com\/cropped\/2024_11_18_8e985d6b9c83aa3e9d0eg-435.jpg?height=319&width=462&top_left_y=920&top_left_x=858) and coefficient 2. (In fact, this could also be seen from the fact that $\\phi$ preserves angles of triangles and maps the segment $S R$ onto $S B$, where $R$ is the midpoint of $A B$.) Since $\\phi(M)=B^{\\prime}$, we conclude that $\\angle M S B^{\\prime}=60^{\\circ}$ and $S B^{\\prime} \/ S M=2$. Similarly, $\\angle N S A^{\\prime}=60^{\\circ}$ and $S A^{\\prime} \/ S N=2$, so triangles $M S B^{\\prime}$ and $N S A^{\\prime}$ are indeed similar. Second solution. Probably the simplest way here is using complex numbers. Put the origin at $O$ and complex numbers $a, a^{\\prime}$ at points $A, A^{\\prime}$, and denote the primitive sixth root of 1 by $\\omega$. Then the numbers at $B, B^{\\prime}$, $S$ and $N$ are $\\omega a, \\omega a^{\\prime},(a+\\omega a) \/ 3$, and $\\left(a+\\omega a^{\\prime}\\right) \/ 2$ respectively. Now it is easy to verify that $(n-s)=\\omega\\left(a^{\\prime}-s\\right) \/ 2$, i.e., that $\\angle N S A^{\\prime}=60^{\\circ}$ and $S A^{\\prime} \/ S N=2$.","problem_type":null,"tier":0} +{"year":"1978","problem_phase":"shortlisted","problem":"3. (CUB 3) ${ }^{\\mathrm{IMO} 1}$ Let $n>m \\geq 1$ be natural numbers such that the groups of the last three digits in the decimal representation of $1978^{m}, 1978^{n}$ coincide. Find the ordered pair $(m, n)$ of such $m, n$ for which $m+n$ is minimal.","solution":"3. What we need are $m, n$ for which $1978^{m}\\left(1978^{n-m}-1\\right)$ is divisible by $1000=8 \\cdot 125$. Since $1978^{n-m}-1$ is odd, it follows that $1978^{m}$ is divisible by 8 , so $m \\geq 3$. Also, $1978^{n-m}-1$ is divisible by 125 , i.e., $1978^{n-m} \\equiv 1(\\bmod 125)$. Note that $1978 \\equiv-2(\\bmod 5)$, and consequently also $-2^{n-m} \\equiv 1$. Hence $4 \\mid n-m=4 k, k \\geq 1$. It remains to find the least $k$ such that $1978^{4 k} \\equiv 1$ $(\\bmod 125)$. Since $1978^{4} \\equiv(-22)^{4}=484^{2} \\equiv(-16)^{2}=256 \\equiv 6$, we reduce it to $6^{k} \\equiv 1$. Now $6^{k}=(1+5)^{k} \\equiv 1+5 k+25\\binom{k}{2}(\\bmod 125)$, which reduces to $125 \\mid 5 k(5 k-3)$. But $5 k-3$ is not divisible by 5 , and so $25 \\mid k$. Therefore $100 \\mid n-m$, and the desired values are $m=3, n=103$.","problem_type":null,"tier":0} +{"year":"1978","problem_phase":"shortlisted","problem":"4. (CZS 2) Let $T_{1}$ be a triangle having $a, b, c$ as lengths of its sides and let $T_{2}$ be another triangle having $u, v, w$ as lengths of its sides. If $P, Q$ are the areas of the two triangles, prove that $$ 16 P Q \\leq a^{2}\\left(-u^{2}+v^{2}+w^{2}\\right)+b^{2}\\left(u^{2}-v^{2}+w^{2}\\right)+c^{2}\\left(u^{2}+v^{2}-w^{2}\\right) $$ When does equality hold?","solution":"4. Let $\\gamma, \\varphi$ be the angles of $T_{1}$ and $T_{2}$ opposite to $c$ and $w$ respectively. By the cosine theorem, the inequality is transformed into $$ \\begin{aligned} & a^{2}\\left(2 v^{2}-2 u v \\cos \\varphi\\right)+b^{2}\\left(2 u^{2}-2 u v \\cos \\varphi\\right) \\\\ & \\quad+2\\left(a^{2}+b^{2}-2 a b \\cos \\gamma\\right) u v \\cos \\varphi \\geq 4 a b u v \\sin \\gamma \\sin \\varphi \\end{aligned} $$ This is equivalent to $2\\left(a^{2} v^{2}+b^{2} u^{2}\\right)-4 a b u v(\\cos \\gamma \\cos \\varphi+\\sin \\gamma \\sin \\varphi) \\geq 0$, i.e., to $$ 2(a v-b u)^{2}+4 a b u v(1-\\cos (\\gamma-\\varphi)) \\geq 0 $$ which is clearly satisfied. Equality holds if and only if $\\gamma=\\varphi$ and $a \/ b=$ $u \/ v$, i.e., when the triangles are similar, $a$ corresponding to $u$ and $b$ to $v$.","problem_type":null,"tier":0} +{"year":"1978","problem_phase":"shortlisted","problem":"5. (GDR 2) For every integer $d \\geq 1$, let $M_{d}$ be the set of all positive integers that cannot be written as a sum of an arithmetic progression with difference $d$, having at least two terms and consisting of positive integers. Let $A=M_{1}, B=M_{2} \\backslash\\{2\\}, C=M_{3}$. Prove that every $c \\in C$ may be written in a unique way as $c=a b$ with $a \\in A, b \\in B$.","solution":"5. We first explicitly describe the elements of the sets $M_{1}, M_{2}$. $x \\notin M_{1}$ is equivalent to $x=a+(a+1)+\\cdots+(a+n-1)=n(2 a+n-1) \/ 2$ for some natural numbers $n, a, n \\geq 2$. Among $n$ and $2 a+n-1$, one is odd and the other even, and both are greater than 1 ; so $x$ has an odd factor $\\geq 3$. On the other hand, for every $x$ with an odd divisor $p>3$ it is easy to see that there exist corresponding $a, n$. Therefore $M_{1}=\\left\\{2^{k} \\mid k=0,1,2, \\ldots\\right\\}$. $x \\notin M_{2}$ is equivalent to $x=a+(a+2)+\\cdots+(a+2(n-1))=n(a+n-1)$, where $n \\geq 2$, i.e. to $x$ being composite. Therefore $M_{2}=\\{1\\} \\cup\\{p \\mid$ $p=$ prime $\\}$. $x \\notin M_{3}$ is equivalent to $x=a+(a+3)+\\cdots+(a+3(n-1))=$ $n(2 a+3(n-1)) \/ 2$. It remains to show that every $c \\in M_{3}$ can be written as $c=2^{k} p$ with $p$ prime. Suppose the opposite, that $c=2^{k} p q$, where $p, q$ are odd and $q \\geq p \\geq 3$. Then there exist positive integers $a, n(n \\geq 2)$ such that $c=n(2 a+3(n-1)) \/ 2$ and hence $c \\notin M_{3}$. Indeed, if $k=0$, then $n=2$ and $2 a+3=p q$ work; otherwise, setting $n=p$ one obtains $a=2^{k} q-$ $3(p-1) \/ 2 \\geq 2 q-3(p-1) \/ 2 \\geq(p+3) \/ 2>1$.","problem_type":null,"tier":0} +{"year":"1978","problem_phase":"shortlisted","problem":"6. (FRA 2) $)^{\\mathrm{IMO} 5}$ Let $\\varphi:\\{1,2,3, \\ldots\\} \\rightarrow\\{1,2,3, \\ldots\\}$ be injective. Prove that for all $n$, $$ \\sum_{k=1}^{n} \\frac{\\varphi(k)}{k^{2}} \\geq \\sum_{k=1}^{n} \\frac{1}{k} $$","solution":"6. For fixed $n$ and the set $\\{\\varphi(1), \\ldots, \\varphi(n)\\}$, there are finitely many possibilities for mapping $\\varphi$ to $\\{1, \\ldots, n\\}$. Suppose $\\varphi$ is the one among these for which $\\sum_{k=1}^{n} \\varphi(k) \/ k^{2}$ is minimal. If $i0 \\end{aligned} $$ which contradicts the assumption. This shows that $\\varphi(1)<\\cdots<\\varphi(n)$, and consequently $\\varphi(k) \\geq k$ for all $k$. Hence $$ \\sum_{k=1}^{n} \\frac{\\varphi(k)}{k^{2}} \\geq \\sum_{k=1}^{n} \\frac{k}{k^{2}}=\\sum_{k=1}^{n} \\frac{1}{k} $$","problem_type":null,"tier":0} +{"year":"1978","problem_phase":"shortlisted","problem":"7. (FRA 5) We consider three distinct half-lines $O x, O y, O z$ in a plane. Prove the existence and uniqueness of three points $A \\in O x, B \\in O y$, $C \\in O z$ such that the perimeters of the triangles $O A B, O B C, O C A$ are all equal to a given number $2 p>0$.","solution":"7. Let $x=O A, y=O B, z=O C, \\alpha=\\angle B O C, \\beta=\\angle C O A, \\gamma=\\angle A O B$. The conditions yield the equation $x+y+\\sqrt{x^{2}+y^{2}-2 x y \\cos \\gamma}=2 p$, which transforms to $(2 p-x-y)^{2}=x^{2}+y^{2}-2 x y \\cos \\gamma$, i.e. $(p-x)(p-y)=$ $x y(1-\\cos \\gamma)$. Thus $$ \\frac{p-x}{x} \\cdot \\frac{p-y}{y}=1-\\cos \\gamma $$ and analogously $\\frac{p-y}{y} \\cdot \\frac{p-z}{z}=1-\\cos \\alpha, \\frac{p-z}{z} \\cdot \\frac{p-x}{x}=1-\\cos \\beta$. Setting $u=\\frac{p-x}{x}, v=\\frac{p-y}{y}, w=\\frac{p-z}{z}$, the above system becomes $$ u v=1-\\cos \\gamma, \\quad v w=1-\\cos \\alpha, \\quad w u=1-\\cos \\beta $$ This system has a unique solution in positive real numbers $u, v, w$ : $u=\\sqrt{\\frac{(1-\\cos \\beta)(1-\\cos \\gamma)}{1-\\cos \\alpha}}$, etc. Finally, the values of $x, y, z$ are uniquely determined from $u, v, w$. Remark. It is not necessary that the three lines be in the same plane. Also, there could be any odd number of lines instead of three.","problem_type":null,"tier":0} +{"year":"1978","problem_phase":"shortlisted","problem":"8. (GBR 4) Let $S$ be the set of all the odd positive integers that are not multiples of 5 and that are less than $30 \\mathrm{~m}, \\mathrm{~m}$ being an arbitrary positive integer. What is the smallest integer $k$ such that in any subset of $k$ integers from $S$ there must be two different integers, one of which divides the other?","solution":"8. Take the subset $\\left\\{a_{i}\\right\\}=\\{1,7,11,13,17,19,23,29, \\ldots, 30 m-1\\}$ of $S$ containing all the elements of $S$ that are not multiples of 3 . There are 8 m such elements. Every element in $S$ can be uniquely expressed as $3^{t} a_{i}$ for some $i$ and $t \\geq 0$. In a subset of $S$ with $8 m+1$ elements, two of them will have the same $a_{i}$, hance one will divide the other. On the other hand, for each $i=1,2, \\ldots, 8 m$ choose $t \\geq 0$ such that $10 \\mathrm{~m}<$ $b_{i}=3^{t} a_{i}<30 \\mathrm{~m}$. Then there are $8 \\mathrm{~m} b_{i}$ 's in the interval $(10 \\mathrm{~m}, 30 \\mathrm{~m})$, and the quotient of any two of them is less than 3 , so none of them can divide any other. Thus the answer is 8 m .","problem_type":null,"tier":0} +{"year":"1978","problem_phase":"shortlisted","problem":"9. $\\mathbf{( G B R} \\mathbf{5})^{\\mathrm{IMO} 3}$ Let $\\{f(n)\\}$ be a strictly increasing sequence of positive integers: $02 \/ n^{2}$ is fixed, then $a b=\\left(s^{2}-(a-b)^{2}\\right) \/ 4$ is minimized when $|a-b|$ is maximized, i.e., when $b=1 \/ n^{2}$. Hence $y_{1} y_{2} \\cdots y_{n}$ is minimal when $y_{2}=y_{3}=\\cdots=y_{n}=1 \/ n^{2}$. Then $y_{1}=\\left(n^{2}-n+1\\right) \/ n^{2}$ and therefore $P_{\\min }=\\sqrt{n^{2}-n+1} \/ n^{n}$.","problem_type":null,"tier":0} +{"year":"1979","problem_phase":"shortlisted","problem":"12. (GDR 3) Let $R$ be a set of exactly 6 elements. A set $F$ of subsets of $R$ is called an $S$-family over $R$ if and only if it satisfies the following three conditions: (i) For no two sets $X, Y$ in $F$ is $X \\subseteq Y$; (ii) For any three sets $X, Y, Z$ in $F, X \\cup Y \\cup Z \\neq R$, (iii) $\\bigcup_{X \\in F} X=R$. We define $|F|$ to be the number of elements of $F$ (i.e., the number of subsets of $R$ belonging to $F$ ). Determine, if it exists, $h=\\max |F|$, the maximum being taken over all S-families over $R$.","solution":"12. The first criterion ensures that all sets in an $S$-family are distinct. Since the number of different families of subsets is finite, $h$ has to exist. In fact, we will show that $h=11$. First of all, if there exists $X \\in F$ such that $|X| \\geq 5$, then by (3) there exists $Y \\in F$ such that $X \\cup Y=R$. In this case $|F|$ is at most 2. Similarly, for $|X|=4$, for the remaining two elements either there exists a subset in $F$ that contains both, in which case we obtain the previous case, or there exist different $Y$ and $Z$ containing them, in which case $X \\cup Y \\cup Z=R$, which must not happen. Hence we can assume $|X| \\leq 4$ for all $X \\in F$. Assume $|X|=1$ for some $X$. In that case other sets must not contain that subset and hence must be contained in the remaining 5 -element subset. These elements must not be subsets of each other. From elementary combinatorics, the largest number of subsets of a 5 -element set of which none is subset of another is $\\binom{5}{2}=10$. This occurs when we take all 2-element subsets. These subsets also satisfy (2). Hence $|F|_{\\max }=11$ in this case. Otherwise, let us assume $|X|=3$ for some $X$. Let us define the following families of subsets: $G=\\{Z=Y \\backslash X \\mid Y \\in F\\}$ and $H=\\{Z=Y \\cap X \\mid Y \\in$ $F\\}$. Then no two sets in $G$ must complement each other in $R \\backslash X$, and $G$ must cover this set. Hence $G$ contains exactly the sets of each of the remaining 3 elements. For each element of $G$ no two sets in $H$ of which one is a subset of another may be paired with it. There can be only 3 such subsets selected within a 3 -element set $X$. Hence the number of remaining sets is smaller than $3 \\cdot 3=9$. Hence in this case $|F|_{\\max }=10$. In the remaining case all subsets have two elements. There are $\\binom{6}{2}=15$ of them. But for every three that complement each other one must be discarded; hence the maximal number for $F$ in this case is $2 \\cdot 15 \/ 3=10$. It follows that $h=11$.","problem_type":null,"tier":0} +{"year":"1979","problem_phase":"shortlisted","problem":"13. (GRE 1) Show that $\\frac{20}{60}<\\sin 20^{\\circ}<\\frac{21}{60}$.","solution":"13. From elementary trigonometry we have $\\sin 3 t=3 \\sin t-4 \\sin ^{3} t$. Hence, if we denote $y=\\sin 20^{\\circ}$, we have $\\sqrt{3} \/ 2=\\sin 60^{\\circ}=3 y-4 y^{3}$. Obviously $00$ for $0 \\leq x<1 \/ 2$. Now the desired inequality $\\frac{20}{60}=\\frac{1}{3}<\\sin 20^{\\circ}<\\frac{21}{60}=\\frac{7}{20}$ follows from $$ f\\left(\\frac{1}{3}\\right)<\\frac{\\sqrt{3}}{2}0$ for $x>1$, the function $f(x)$ takes its maximum at a point $x$ for which $f^{\\prime}(x)=(1-\\ln x) \/ x^{2}=0$. Hence $$ \\max f(x)=f(e)=e^{1 \/ e} . $$ It follows that the set of values of $f(x)$ for $x \\in \\mathbb{R}^{+}$is the interval $\\left(-\\infty, e^{1 \/ e}\\right)$, and consequently the desired set of bases $a$ of logarithms is $(0,1) \\cup\\left(1, e^{1 \/ e}\\right]$.","problem_type":null,"tier":0} +{"year":"1979","problem_phase":"shortlisted","problem":"15. (ISR 2) ${ }^{\\mathrm{IMO} 5}$ The nonnegative real numbers $x_{1}, x_{2}, x_{3}, x_{4}, x_{5}, a$ satisfy the following relations: $$ \\sum_{i=1}^{5} i x_{i}=a, \\quad \\sum_{i=1}^{5} i^{3} x_{i}=a^{2}, \\quad \\sum_{i=1}^{5} i^{5} x_{i}=a^{3} $$ What are the possible values of $a$ ?","solution":"15. We note that $\\sum_{i=1}^{5} i\\left(a-i^{2}\\right)^{2} x_{i}=a^{2} \\sum_{i=1}^{5} i x_{i}-2 a \\sum_{i=1}^{5} i^{3} x_{i}+\\sum_{i=1}^{5} i^{5} x_{i}=a^{2} \\cdot a-2 a \\cdot a^{2}+a^{3}=0$. Since the terms in the sum on the left are all nonnegative, it follows that all the terms have to be 0 . Thus, either $x_{i}=0$ for all $i$, in which case $a=0$, or $a=j^{2}$ for some $j$ and $x_{i}=0$ for $i \\neq j$. In this case, $x_{j}=a \/ j=j$. Hence, the only possible values of $a$ are $\\{0,1,4,9,16,25\\}$.","problem_type":null,"tier":0} +{"year":"1979","problem_phase":"shortlisted","problem":"16. (ISR 4) Let $K$ denote the set $\\{a, b, c, d, e\\} . F$ is a collection of 16 different subsets of $K$, and it is known that any three members of $F$ have at least one element in common. Show that all 16 members of $F$ have exactly one element in common.","solution":"16. Obviously, no two elements of $F$ can be complements of each other. If one of the sets has one element, then the conclusion is trivial. If there exist two different 2-element sets, then they must contain a common element, which in turn must then be contained in all other sets. Thus we can assume that there exists at most one 2 -element subset of $K$ in $F$. Since there can be at most 6 subsets of more than 3 elements of a 5 -element set, it follows that at least 9 out of 10 possible 3 -element subsets of $K$ belong to $F$. Let us assume, without loss of generality, that all sets but $\\{c, d, e\\}$ belong to $F$. Then sets $\\{a, b, c\\},\\{a, d, e\\}$, and $\\{b, c, d\\}$ have no common element, which is a contradiction. Hence it follows that all sets have a common element.","problem_type":null,"tier":0} +{"year":"1979","problem_phase":"shortlisted","problem":"17. (NET 1) Inside an equilateral triangle $A B C$ one constructs points $P$, $Q$ and $R$ such that $$ \\begin{aligned} & \\angle Q A B=\\angle P B A=15^{\\circ}, \\\\ & \\angle R B C=\\angle Q C B=20^{\\circ}, \\\\ & \\angle P C A=\\angle R A C=25^{\\circ} . \\end{aligned} $$ Determine the angles of triangle $P Q R$.","solution":"17. Let $K, L$, and $M$ be intersections of $C Q$ and $B R, A R$ and $C P$, and $A Q$ and $B P$, respectively. Let $\\angle X$ denote the angle of the hexagon $K Q M P L R$ at the vertex $X$, where $X$ is one of the six points. By an elementary calculation of angles we get $\\angle K=140^{\\circ}, \\angle L=130^{\\circ}, \\angle M=150^{\\circ}, \\angle P=100^{\\circ}, \\angle Q=95^{\\circ}, \\angle R=105^{\\circ}$. Since $\\angle K B C=\\angle K C B$, it follows that $K$ is on the symmetry line of $A B C$ through $A$. Analogous statements hold for $L$ and $M$. Let $K_{R}$ and $K_{Q}$ be points symmetric to $K$ with respect to $A R$ and $A Q$, respectively. Since $\\angle A K_{Q} Q=\\angle A K_{Q} K_{R}=70^{\\circ}$ and $\\angle A K_{R} R=\\angle A K_{R} K_{Q}=70^{\\circ}$, it follows that $K_{R}, R, Q$, and $K_{Q}$ are collinear. Hence $\\angle Q R K=$ $2 \\angle R-180^{\\circ}$ and $\\angle R Q K=2 \\angle Q-$ $180^{\\circ}$. We analogously get $\\angle P R L=$ $2 \\angle R-180^{\\circ}, \\angle R P L=2 \\angle P-$ $180^{\\circ}, \\angle Q P M=2 \\angle P-180^{\\circ}$ and $\\angle P Q M=2 \\angle Q-180^{\\circ}$. From these formulas we easily get $\\angle R P Q=$ $60^{\\circ}, \\angle R Q P=75^{\\circ}$, and $\\angle Q R P=$ $45^{\\circ}$.","problem_type":null,"tier":0} +{"year":"1979","problem_phase":"shortlisted","problem":"18. (POL 1) Let $m$ positive integers $a_{1}, \\ldots, a_{m}$ be given. Prove that there exist fewer than $2^{m}$ positive integers $b_{1}, \\ldots, b_{n}$ such that all sums of distinct $b_{k}$ 's are distinct and all $a_{i}(i \\leq m)$ occur among them.","solution":"18. Let us write all $a_{i}$ in binary representation. For $S \\subseteq\\{1,2, \\ldots, m\\}$ let us define $b(S)$ as the number in whose binary representation ones appear in exactly the slots where ones appear in all $a_{i}$ where $i \\subseteq S$ and don't appear in any other $a_{i}$. Some $b(S)$, including $b(\\emptyset)$, will equal 0 , and hence there are fewer than $2^{m}$ different positive $b(S)$. We note that no two positive $b\\left(S_{1}\\right)$ and $b\\left(S_{2}\\right)\\left(S_{1} \\neq S_{2}\\right)$ have ones in the same decimal places. Hence sums of distinct $b(S)$ 's are distinct. Moreover $$ a_{i}=\\sum_{i \\in S} b(S) $$ and hence the positive $b(S)$ are indeed the numbers $b_{1}, \\ldots, b_{n}$ whose existence we had to prove.","problem_type":null,"tier":0} +{"year":"1979","problem_phase":"shortlisted","problem":"19. (ROM 1) Consider the sequences $\\left(a_{n}\\right),\\left(b_{n}\\right)$ defined by $$ a_{1}=3, \\quad b_{1}=100, \\quad a_{n+1}=3^{a_{n}}, \\quad b_{n+1}=100^{b_{n}} . $$ Find the smallest integer $m$ for which $b_{m}>a_{100}$.","solution":"19. Let us define $i_{j}$ for two positive integers $i$ and $j$ in the following way: $i_{1}=i$ and $i_{j+1}=i^{i_{j}}$ for all positive integers $j$. Thus we must find the smallest $m$ such that $100_{m}>3_{100}$. Since $100_{1}=100>27=3_{2}$, we inductively have $100_{j}=10^{100_{j-1}}>3^{100_{j-1}}>3^{3_{j}}=3_{j+1}$ and hence $m \\leq 99$. We now prove that $m=99$ by proving $100_{98}<3_{100}$. We note that $\\left(100_{1}\\right)^{2}=10^{4}<27^{4}=3^{12}<3^{27}=3_{3}$. We also note for $d>12$ (which trivially holds for all $d=100_{i}$ ) that if $c>d^{2}$, then we have $$ 3^{c}>3^{d^{2}}>3^{12 d}=\\left(3^{12}\\right)^{d}>10000^{d}=\\left(100^{d}\\right)^{2} $$ Hence from $3_{3}>\\left(100_{1}\\right)^{2}$ it inductively follows that $3_{j}>\\left(100_{j-2}\\right)^{2}>$ $100_{j-2}$ and hence that $100_{99}>3_{100}>100_{98}$. Hence $m=99$.","problem_type":null,"tier":0} +{"year":"1979","problem_phase":"shortlisted","problem":"2. (BEL 4) From a bag containing 5 pairs of socks, each pair a different color, a random sample of 4 single socks is drawn. Any complete pairs in the sample are discarded and replaced by a new pair draw from the bag. The process continues until the bag is empty or there are 4 socks of different colors held outside the bag. What is the probability of the latter alternative?","solution":"2. The only way to arrive at the latter alternative is to draw four different socks in the first drawing or to draw only one pair in the first drawing and then draw two different socks in the last drawing. We will call these probabilities respectively $p_{1}, p_{2}, p_{3}$. We calculate them as follows: $$ p_{1}=\\frac{\\binom{5}{4} 2^{4}}{\\binom{10}{4}}=\\frac{8}{21}, \\quad p_{2}=\\frac{5\\binom{4}{2} 2^{2}}{\\binom{10}{4}}=\\frac{4}{7}, \\quad p_{3}=\\frac{4}{\\binom{6}{2}}=\\frac{4}{15} . $$ We finally calculate the desired probability: $P=p_{1}+p_{2} p_{3}=\\frac{8}{15}$.","problem_type":null,"tier":0} +{"year":"1979","problem_phase":"shortlisted","problem":"20. (SWE 2) Given the integer $n>1$ and the real number $a>0$ determine the maximum of $\\sum_{i=1}^{n-1} x_{i} x_{i+1}$ taken over all nonnegative numbers $x_{i}$ with sum $a$.","solution":"20. Let $x_{k}=\\max \\left\\{x_{1}, x_{2}, \\ldots, x_{n}\\right\\}$. Then $x_{i} x_{i+1} \\leq x_{i} x_{k}$ for $i=1,2, \\ldots, k-1$ and $x_{i} x_{i+1} \\leq x_{k} x_{i+1}$ for $i=k, \\ldots, n-1$. Summing up these inequalities for $i=1,2, \\ldots, n-1$ we obtain $$ \\sum_{i=1}^{n-1} \\leq x_{k}\\left(x_{1}+\\cdots+x_{k-1}+x_{k+1}+\\cdots+x_{n}\\right)=x_{k}\\left(a-x_{k}\\right) \\leq \\frac{a^{2}}{4} $$ We note that the value $a^{2} \/ 4$ is attained for $x_{1}=x_{2}=a \/ 2$ and $x_{3}=\\cdots=$ $x_{n}=0$. Hence $a^{2} \/ 4$ is the required maximum.","problem_type":null,"tier":0} +{"year":"1979","problem_phase":"shortlisted","problem":"21. (USS 1) Let $N$ be the number of integral solutions of the equation $$ x^{2}-y^{2}=z^{3}-t^{3} $$ satisfying the condition $0 \\leq x, y, z, t \\leq 10^{6}$, and let $M$ be the number of integral solutions of the equation $$ x^{2}-y^{2}=z^{3}-t^{3}+1 $$ satisfying the condition $0 \\leq x, y, z, t \\leq 10^{6}$. Prove that $N>M$.","solution":"21. Let $f(n)$ be the number of different ways $n \\in \\mathbb{N}$ can be expressed as $x^{2}+y^{3}$ where $x, y \\in\\left\\{0,1, \\ldots, 10^{6}\\right\\}$. Clearly $f(n)=0$ for $n<0$ or $n>10^{12}+10^{18}$. The first equation can be written as $x^{2}+t^{3}=y^{2}+z^{3}=n$, whereas the second equation can be written as $x^{2}+t^{3}=n+1, y^{2}+z^{3}=n$. Hence we obtain the following formulas for $M$ and $N$ : $$ M=\\sum_{i=0}^{m} f(i)^{2}, \\quad N=\\sum_{i=0}^{m-1} f(i) f(i+1) . $$ Using the AM-GM inequality we get $$ \\begin{aligned} N & =\\sum_{i=0}^{m-1} f(i) f(i+1) \\\\ & \\leq \\sum_{i=0}^{m-1} \\frac{f(i)^{2}+f(i+1)^{2}}{2}=\\frac{f(0)^{2}}{2}+\\sum_{i=1}^{m-1} f(i)^{2}+\\frac{f(m)^{2}}{2}0$. This completes our proof.","problem_type":null,"tier":0} +{"year":"1979","problem_phase":"shortlisted","problem":"22. (USS 3) ${ }^{\\mathrm{IMO} 3}$ There are two circles in the plane. Let a point $A$ be one of the points of intersection of these circles. Two points begin moving simultaneously with constant speeds from the point $A$, each point along its own circle. The two points return to the point $A$ at the same time. Prove that there is a point $P$ in the plane such that at every moment of time the distances from the point $P$ to the moving points are equal.","solution":"22. Let the centers of the two circles be denoted by $O$ and $O_{1}$ and their respective radii by $r$ and $r_{1}$, and let the positions of the points on the circles at time $t$ be denoted by $M(t)$ and $N(t)$. Let $Q$ be the point such that $O A O_{1} Q$ is a parallelogram. We will show that $Q$ is the point $P$ we are looking for, i.e., that $Q M(t)=$ $Q N(t)$ for all $t$. We note that $O Q=$ $O_{1} A=r_{1}, O_{1} Q=O A=r$ and ![](https:\/\/cdn.mathpix.com\/cropped\/2024_11_18_8e985d6b9c83aa3e9d0eg-449.jpg?height=353&width=469&top_left_y=1504&top_left_x=850) $\\angle Q O A=\\angle Q O_{1} A=\\phi$. Since the two points return to $A$ at the same time, it follows that $\\angle M(t) O A=\\angle N(t) O_{1} A=\\omega t$. Therefore $\\angle Q O M(t)=$ $\\angle Q O_{1} N(t)=\\phi+\\omega t$, from which it follows that $\\triangle Q O M(t) \\cong \\triangle Q O_{1} N(t)$. Hence $Q M(t)=Q N(t)$, as we claimed.","problem_type":null,"tier":0} +{"year":"1979","problem_phase":"shortlisted","problem":"23. (USA 4) Find all natural numbers $n$ for which $2^{8}+2^{11}+2^{n}$ is a perfect square.","solution":"23. It is easily verified that no solutions exist for $n \\leq 8$. Let us now assume that $n>8$. We note that $2^{8}+2^{11}+2^{n}=2^{8} \\cdot\\left(9+2^{n-8}\\right)$. Hence $9+2^{n-8}$ must also be a square, say $9+2^{n-8}=x^{2}, x \\in \\mathbb{N}$, i.e., $2^{n-8}=x^{2}-9=$ $(x-3)(x+3)$. Thus $x-3$ and $x+3$ are both powers of 2 , which is possible only for $x=5$ and $n=12$. Hence, $n=12$ is the only solution.","problem_type":null,"tier":0} +{"year":"1979","problem_phase":"shortlisted","problem":"24. (USA 5) A circle O with center $O$ on base $B C$ of an isosceles triangle $A B C$ is tangent to the equal sides $A B, A C$. If point $P$ on $A B$ and point $Q$ on $A C$ are selected such that $P B \\times C Q=(B C \/ 2)^{2}$, prove that line segment $P Q$ is tangent to circle O , and prove the converse.","solution":"24. Clearly $O$ is the midpoint of $B C$. Let $M$ and $N$ be the points of tangency of the circle with $A B$ and $A C$, respectively, and let $\\angle B A C=2 \\varphi$. Then $\\angle B O M=\\angle C O N=\\varphi$. Let us assume that $P Q$ touches the circle in $X$. If we set $\\angle P O M=$ $\\angle P O X=x$ and $\\angle Q O N=\\angle Q O X=y$, then $2 x+2 y=\\angle M O N=$ $180^{\\circ}-2 \\varphi$, i.e., $y=90^{\\circ}-\\varphi-x$. It follows that $\\angle O Q C=180^{\\circ}-\\angle Q O C-$ $\\angle O C Q=180^{\\circ}-(\\varphi+y)-\\left(90^{\\circ}-\\varphi\\right)=90^{\\circ}-y=x+\\varphi=\\angle B O P$. Hence the triangles $B O P$ and $C Q O$ are similar, and consequently $B P \\cdot C Q=$ $B O \\cdot C O=(B C \/ 2)^{2}$. Conversely, let $B P \\cdot C Q=(B C \/ 2)^{2}$ and let $Q^{\\prime}$ be the point on $(A C)$ such that $P Q^{\\prime}$ is tangent to the circle. Then $B P \\cdot C Q^{\\prime}=(B C \/ 2)^{2}$, which implies $Q \\equiv Q^{\\prime}$.","problem_type":null,"tier":0} +{"year":"1979","problem_phase":"shortlisted","problem":"25. (USA 6) ${ }^{\\mathrm{IMO} 4}$ Given a point $P$ in a given plane $\\pi$ and also a given point $Q$ not in $\\pi$, show how to determine a point $R$ in $\\pi$ such that $\\frac{Q P+P R}{Q R}$ is a maximum.","solution":"25. Let us first look for such a point $R$ on a line $l$ in $\\pi$ going through $P$. Let $\\angle Q P R=2 \\theta$. Consider a point $Q^{\\prime}$ on $l$ such that $Q^{\\prime} P=Q P$. Then we have $$ \\frac{Q P+P R}{Q R}=\\frac{R Q^{\\prime}}{Q R}=\\frac{\\sin Q^{\\prime} Q R}{\\sin Q Q^{\\prime} R} $$ Since $Q Q^{\\prime} P$ is fixed, the maximal value of the expression occurs when $\\angle Q Q^{\\prime} R=90^{\\circ}$. In this case $(Q P+P R) \/ Q R=1 \/ \\sin \\theta$. Looking at all possible lines $l$, we see that $\\theta$ is minimized when $l$ equals the projection of $P Q$ onto $\\pi$. Hence, the point $R$ is the intersection of the projection of $P Q$ onto $\\pi$ and the plane through $Q$ perpendicular to $P Q$.","problem_type":null,"tier":0} +{"year":"1979","problem_phase":"shortlisted","problem":"26. (YUG 4) Prove that the functional equations $$ \\begin{aligned} f(x+y) & =f(x)+f(y) \\\\ \\text { and } \\quad f(x+y+x y) & =f(x)+f(y)+f(x y) \\quad(x, y \\in \\mathbb{R}) \\end{aligned} $$ are equivalent.","solution":"26. Let us assume that $f(x+y)=f(x)+f(y)$ for all reals. In this case we trivially apply the equation to get $f(x+y+x y)=f(x+y)+f(x y)=$ $f(x)+f(y)+f(x y)$. Hence the equivalence is proved in the first direction. Now let us assume that $f(x+y+x y)=f(x)+f(y)+f(x y)$ for all reals. Plugging in $x=y=0$ we get $f(0)=0$. Plugging in $y=-1$ we get $f(x)=-f(-x)$. Plugging in $y=1$ we get $f(2 x+1)=2 f(x)+f(1)$ and hence $f(2(u+v+u v)+1)=2 f(u+v+u v)+f(1)=2 f(u v)+$ $2 f(u)+2 f(v)+f(1)$ for all real $u$ and $v$. On the other hand, plugging in $x=u$ and $y=2 v+1$ we get $f(2(u+v+u v)+1)=f(u+(2 v+$ 1) $+u(2 v+1))=f(u)+2 f(v)+f(1)+f(2 u v+u)$. Hence it follows that $2 f(u v)+2 f(u)+2 f(v)+f(1)=f(u)+2 f(v)+f(1)+f(2 u v+u)$, i.e., $$ f(2 u v+u)=2 f(u v)+f(u) $$ Plugging in $v=-1 \/ 2$ we get $0=2 f(-u \/ 2)+f(u)=-2 f(u \/ 2)+f(u)$. Hence, $f(u)=2 f(u \/ 2)$ and consequently $f(2 x)=2 f(x)$ for all reals. Now (1) reduces to $f(2 u v+u)=f(2 u v)+f(u)$. Plugging in $u=y$ and $x=2 u v$, we obtain $f(x)+f(y)=f(x+y)$ for all nonzero reals $x$ and $y$. Since $f(0)=0$, it trivially holds that $f(x+y)=f(x)+f(y)$ when one of $x$ and $y$ is 0 .","problem_type":null,"tier":0} +{"year":"1979","problem_phase":"shortlisted","problem":"3. (BUL 1) Find all polynomials $f(x)$ with real coefficients for which $$ f(x) f\\left(2 x^{2}\\right)=f\\left(2 x^{3}+x\\right) $$","solution":"3. An obvious solution is $f(x)=0$. We now look for nonzero solutions. We note that plugging in $x=0$ we get $f(0)^{2}=f(0)$; hence $f(0)=0$ or $f(0)=1$. If $f(0)=0$, then $f$ is of the form $f(x)=x^{k} g(x)$, where $g(0) \\neq 0$. Plugging this formula into $f(x) f\\left(2 x^{2}\\right)=f\\left(2 x^{3}+x\\right)$ we get $2^{k} x^{2 k} g(x) g\\left(2 x^{2}\\right)=\\left(2 x^{2}+1\\right)^{k} g\\left(2 x^{3}+x\\right)$. Plugging in $x=0$ gives us $g(0)=0$, which is a contradiction. Hence $f(0)=1$. For an arbitrary root $\\alpha$ of the polynomial $f, 2 \\alpha^{3}+\\alpha$ must also be a root. Let $\\alpha$ be a root of the largest modulus. If $|\\alpha|>1$ then $\\left|2 \\alpha^{3}+\\alpha\\right|>$ $2|\\alpha|^{3}-|\\alpha|>|\\alpha|$, which is impossible. It follows that $|\\alpha| \\leq 1$ and hence all roots of $f$ have modules less than or equal to 1 . But the product of all roots of $f$ is $|f(0)|=1$, which implies that all the roots have modulus 1. Consequently, for a root $\\alpha$ it holds that $|\\alpha|=\\left|2 \\alpha^{3}-\\alpha\\right|=1$. This is possible only if $\\alpha= \\pm \\imath$. Since the coefficients of $f$ are real it follows that $f$ must be of the form $f(x)=\\left(x^{2}+1\\right)^{k}$ where $k \\in \\mathbb{N}_{0}$. These polynomials satisfy the original formula. Hence, the solutions for $f$ are $f(x)=0$ and $f(x)=\\left(x^{2}+1\\right)^{k}, k \\in \\mathbb{N}_{0}$.","problem_type":null,"tier":0} +{"year":"1979","problem_phase":"shortlisted","problem":"4. (BUL 3) ${ }^{\\mathrm{IMO} 2} \\mathrm{~A}$ pentagonal prism $A_{1} A_{2} \\ldots A_{5} B_{1} B_{2} \\ldots B_{5}$ is given. The edges, the diagonals of the lateral walls and the internal diagonals of the prism are each colored either red or green in such a way that no triangle whose vertices are vertices of the prism has its three edges of the same color. Prove that all edges of the bases are of the same color.","solution":"4. Let us prove first that the edges $A_{1} A_{2}, A_{2} A_{3}, \\ldots, A_{5} A_{1}$ are of the same color. Assume the contrary, and let w.l.o.g. $A_{1} A_{2}$ be red and $A_{2} A_{3}$ be green. Three of the segments $A_{2} B_{l}(l=1,2,3,4,5)$, say $A_{2} B_{i}, A_{2} B_{j}, A_{2} B_{k}$, have to be of the same color, let it w.l.o.g. be red. Then $A_{1} B_{i}, A_{1} B_{j}, A_{1} B_{k}$ must be green. At least one of the sides of triangle $B_{i} B_{j} B_{k}$, say $B_{i} B_{j}$, must be an edge of the prism. Then looking at the triangles $A_{1} B_{i} B_{j}$ and $A_{2} B_{i} B_{j}$ we deduce that $B_{i} B_{j}$ can be neither green nor red, which is a contradiction. Hence all five edges of the pentagon $A_{1} A_{2} A_{3} A_{4} A_{5}$ have the same color. Similarly, all five edges of $B_{1} B_{2} B_{3} B_{4} B_{5}$ have the same color. We now show that the two colors are the same. Assume otherwise, i.e., that w.l.o.g. the $A$ edges are painted red and the $B$ edges green. Let us call segments of the form $A_{i} B_{j}$ diagonal ( $i$ and $j$ may be equal). We now count the diagonal segments by grouping the red segments based on their $A$ point, and the green segments based on their $B$ point. As above, the assumption that three of $A_{i} B_{j}$ for fixed $i$ are red leads to a contradiction. Hence at most two diagonal segments out of each $A_{i}$ may be red, which counts up to at most 10 red segments. Similarly, at most 10 diagonal segments can be green. But then we can paint at most 20 diagonal segments out of 25 , which is a contradiction. Hence all edges in the pentagons $A_{1} A_{2} A_{3} A_{4} A_{5}$ and $B_{1} B_{2} B_{3} B_{4} B_{5}$ have the same color.","problem_type":null,"tier":0} +{"year":"1979","problem_phase":"shortlisted","problem":"5. (CZS 2) Let $n \\geq 2$ be an integer. Find the maximal cardinality of a set $M$ of pairs $(j, k)$ of integers, $1 \\leq j2$ is there a set of $n$ consecutive positive integers such that the largest number in the set is a divisor of the least common multiple of the remaining $n-1$ numbers? (b) For which values of $n>2$ is there a unique set having the stated property?","solution":"1. Assume that the set $\\{a-n+1, a-n+2, \\ldots, a\\}$ of $n$ consecutive numbers satisfies the condition $a \\mid \\operatorname{lcm}[a-n+1, \\ldots, a-1]$. Let $a=p_{1}^{\\alpha_{1}} p_{2}^{\\alpha_{2}} \\ldots p_{r}^{\\alpha_{r}}$ be the canonic representation of $a$, where $p_{1}0$. Then for each $j=1,2, \\ldots, r$, there exists $m, m=$ $1,2, \\ldots, n-1$, such that $p_{j}^{\\alpha_{j}} \\mid a-m$, i.e., such that $p_{j}^{\\alpha_{j}} \\mid m$. Thus $p_{j}^{\\alpha_{j}} \\leq$ $n-1$. If $r=1$, then $a=p_{1}^{\\alpha_{1}} \\leq n-1$, which is impossible. Therefore $r \\geq 2$. But then there must exist two distinct prime numbers less than $n$; hence $n \\geq 4$. For $n=4$, we must have $p_{1}^{\\alpha_{1}}, p_{2}^{\\alpha_{2}} \\leq 3$, which leads to $p_{1}=2, p_{2}=3$, $\\alpha_{1}=\\alpha_{2}=1$. Therefore $a=6$, and $\\{3,4,5,6\\}$ is a unique set satisfying the condition of the problem. For every $n \\geq 5$ there exist at least two such sets. In fact, for $n=5$ we easily find two sets: $\\{2,3,4,5,6\\}$ and $\\{8,9,10,11,12\\}$. Suppose that $n \\geq 6$. Let $r, s, t$ be natural numbers such that $2^{r} \\leq n-1<2^{r+1}$, $3^{s} \\leq n-1<3^{s+1}, 5^{t} \\leq n-1<5^{t+1}$. Taking $a=2^{r} \\cdot 3^{s}$ and $a=2^{r} \\cdot 5^{t}$ we obtain two distinct sets with the required property. Thus the answers are (a) $n \\geq 4$ and (b) $n=4$.","problem_type":null,"tier":0} +{"year":"1981","problem_phase":"shortlisted","problem":"10. (FRA) Determine the smallest natural number $n$ having the following property: For every integer $p, p \\geq n$, it is possible to subdivide (partition) a given square into $p$ squares (not necessarily equal).","solution":"10. It is easy to see that partitioning into $p=2 k$ squares is possible for $k \\geq 2$ (Fig. 1). Furthermore, whenever it is possible to partition the square into $p$ squares, there is a partition of the square into $p+3$ squares: namely, in the partition into $p$ squares, divide one of them into four new squares. ![](https:\/\/cdn.mathpix.com\/cropped\/2024_11_18_8e985d6b9c83aa3e9d0eg-455.jpg?height=272&width=240&top_left_y=1678&top_left_x=393) Fig. 1 ![](https:\/\/cdn.mathpix.com\/cropped\/2024_11_18_8e985d6b9c83aa3e9d0eg-455.jpg?height=272&width=308&top_left_y=1678&top_left_x=919) Fig. 2 This implies that both $p=2 k$ and $p=2 k+3$ are possible if $k \\geq 2$, and therefore all $p \\geq 6$ are possible. On the other hand, partitioning the square into 5 squares is not possible. Assuming it is possible, one of its sides would be covered by exactly two squares, which cannot be of the same size (Fig. 2). The rest of the big square cannot be partitioned into three squares. Hence, the answer is $n=6$.","problem_type":null,"tier":0} +{"year":"1981","problem_phase":"shortlisted","problem":"11. (NET) On a semicircle with unit radius four consecutive chords $A B, B C$, $C D, D E$ with lengths $a, b, c, d$, respectively, are given. Prove that $$ a^{2}+b^{2}+c^{2}+d^{2}+a b c+b c d<4 $$","solution":"11. Let us denote the center of the semicircle by $O$, and $\\angle A O B=2 \\alpha$, $\\angle B O C=2 \\beta, A C=m, C E=n$. We claim that $a^{2}+b^{2}+n^{2}+a b n=4$. Indeed, since $a=2 \\sin \\alpha, b=2 \\sin \\beta$, $n=2 \\cos (\\alpha+\\beta)$, we have $$ \\begin{aligned} a^{2} & +b^{2}+n^{2}+a b n \\\\ & =4\\left(\\sin ^{2} \\alpha+\\sin ^{2} \\beta+\\cos ^{2}(\\alpha+\\beta)+2 \\sin \\alpha \\sin \\beta \\cos (\\alpha+\\beta)\\right) \\\\ & =4+4\\left(-\\frac{\\cos 2 \\alpha}{2}-\\frac{\\cos 2 \\beta}{2}+\\cos (\\alpha+\\beta) \\cos (\\alpha-\\beta)\\right) \\\\ & =4+4(\\cos (\\alpha+\\beta) \\cos (\\alpha-\\beta)-\\cos (\\alpha+\\beta) \\cos (\\alpha-\\beta))=4 \\end{aligned} $$ Analogously, $c^{2}+d^{2}+m^{2}+c d m=4$. By adding both equalities and subtracting $m^{2}+n^{2}=4$ we obtain $$ a^{2}+b^{2}+c^{2}+d^{2}+a b n+c d m=4 $$ Since $n>c$ and $m>b$, the desired inequality follows.","problem_type":null,"tier":0} +{"year":"1981","problem_phase":"shortlisted","problem":"12. (NET) ${ }^{\\mathrm{IMO} 3}$ Determine the maximum value of $m^{2}+n^{2}$ where $m$ and $n$ are integers satisfying $$ m, n \\in\\{1,2, \\ldots, 100\\} \\quad \\text { and } \\quad\\left(n^{2}-m n-m^{2}\\right)^{2}=1 $$","solution":"12. We will solve the contest problem (in which $m, n \\in\\{1,2, \\ldots, 1981\\}$ ). For $m=1, n$ can be either 1 or 2 . If $m>1$, then $n(n-m)=m^{2} \\pm 1>0$; hence $n-m>0$. Set $p=n-m$. Since $m^{2}-m p-p^{2}=m^{2}-p(m+p)=$ $-\\left(n^{2}-n m-m^{2}\\right)$, we see that $(m, n)$ is a solution of the equation if and only if $(p, m)$ is a solution too. Therefore, all the solutions of the equation are given as two consecutive members of the Fibonacci sequence $$ 1,1,2,3,5,8,13,21,34,55,89,144,233,377,610,987,1597,2584, \\ldots $$ So the required maximum is $987^{2}+1597^{2}$.","problem_type":null,"tier":0} +{"year":"1981","problem_phase":"shortlisted","problem":"13. (ROM) Let $P$ be a polynomial of degree $n$ satisfying $$ P(k)=\\binom{n+1}{k}^{-1} \\quad \\text { for } k=0,1, \\ldots, n $$ Determine $P(n+1)$.","solution":"13. Lemma. For any polynomial $P$ of degree at most $n$, $$ \\sum_{i=0}^{n+1}(-1)^{i}\\binom{n+1}{i} P(i)=0 $$ Proof. We shall use induction on $n$. For $n=0$ it is trivial. Assume that it is true for $n=k$ and suppose that $P(x)$ is a polynomial of degree $k+1$. Then $P(x)-P(x+1)$ clearly has degree at most $k$; hence (1) gives $$ \\begin{aligned} 0 & =\\sum_{i=0}^{k+1}(-1)^{i}\\binom{k+1}{i}(P(i)-P(i+1)) \\\\ & =\\sum_{i=0}^{k+1}(-1)^{i}\\binom{k+1}{i} P(i)+\\sum_{i=1}^{k+2}(-1)^{i}\\binom{k+1}{i-1} P(i) \\\\ & =\\sum_{i=0}^{k+2}(-1)^{i}\\binom{k+2}{i} P(i) \\end{aligned} $$ This completes the proof of the lemma. Now we apply the lemma to obtain the value of $P(n+1)$. Since $P(i)=$ $\\binom{n+1}{i}^{-1}$ for $i=0,1, \\ldots, n$, we have $$ 0=\\sum_{i=0}^{n+1}(-1)^{i}\\binom{n+1}{i} P(i)=(-1)^{n+1} P(n+1)+ \\begin{cases}1, & 2 \\mid n ; \\\\ 0, & 2 \\nmid n .\\end{cases} $$ It follows that $P(n+1)= \\begin{cases}1, & 2 \\mid n ; \\\\ 0, & 2 \\nmid n .\\end{cases}$","problem_type":null,"tier":0} +{"year":"1981","problem_phase":"shortlisted","problem":"14. (ROM) Prove that a convex pentagon (a five-sided polygon) $A B C D E$ with equal sides and for which the interior angles satisfy the condition $\\angle A \\geq \\angle B \\geq \\angle C \\geq \\angle D \\geq \\angle E$ is a regular pentagon.","solution":"14. We need the following lemma. Lemma. If a convex quadrilateral $P Q R S$ satisfies $P S=Q R$ and $\\angle S P Q \\geq$ $\\angle R Q P$, then $\\angle Q R S \\geq \\angle P S R$. Proof. If the lines $P S$ and $Q R$ are parallel, then this quadrilateral is a parallelogram, and the statement is trivial. Otherwise, let $X$ be the point of intersection of lines $P S$ and $Q R$. Assume that $\\angle S P Q+\\angle R Q P>180^{\\circ}$. Then $\\angle X P Q \\leq \\angle X Q P$ implies that $X P \\geq X Q$, and consequently $X S \\geq X R$. Hence, $\\angle Q R S=$ $\\angle X R S \\geq \\angle X S R=\\angle P S R$. Similarly, if $\\angle S P Q+\\angle R Q P<180^{\\circ}$, then $\\angle X P Q \\geq \\angle X Q P$, from which it follows that $X P \\leq X Q$, and thus $X S \\leq X R$; hence $\\angle Q R S=$ $180^{\\circ}-\\angle X R S \\geq 180^{\\circ}-\\angle X S R=\\angle P S R$. Now we apply the lemma to the quadrilateral $A B C D$. Since $\\angle B \\geq \\angle C$ and $A B=C D$, it follows that $\\angle C D A \\geq \\angle B A D$, which together with $\\angle E D A=\\angle E A D$ gives $\\angle D \\geq \\angle A$. Thus $\\angle A=\\angle B=\\angle C=\\angle D$. Analogously, by applying the lemma to $B C D E$ we obtain $\\angle E \\geq \\angle B$, and hence $\\angle B=\\angle C=\\angle D=\\angle E$.","problem_type":null,"tier":0} +{"year":"1981","problem_phase":"shortlisted","problem":"15. (GBR) ${ }^{\\mathrm{IMO} 1}$ Find the point $P$ inside the triangle $A B C$ for which $$ \\frac{B C}{P D}+\\frac{C A}{P E}+\\frac{A B}{P F} $$ is minimal, where $P D, P E, P F$ are the perpendiculars from $P$ to $B C, C A$, $A B$ respectively.","solution":"15. Set $B C=a, C A=b, A B=c$, and denote the area of $\\triangle A B C$ by $P$, and $a \/ P D+b \/ P E+c \/ P F$ by $S$. Since $a \\cdot P D+b \\cdot P E+c \\cdot P F=2 P$, by the Cauchy-Schwarz inequality we have $$ 2 P S=(a \\cdot P D+b \\cdot P E+c \\cdot P F)\\left(\\frac{a}{P D}+\\frac{b}{P E}+\\frac{c}{P F}\\right) \\geq(a+b+c)^{2} $$ with equality if and only if $P D=P E=P F$, i.e., $P$ is the incenter of $\\triangle A B C$. In that case, $S$ attains its minimum: $$ S_{\\min }=\\frac{(a+b+c)^{2}}{2 P} $$","problem_type":null,"tier":0} +{"year":"1981","problem_phase":"shortlisted","problem":"16. (GBR) A sequence of real numbers $u_{1}, u_{2}, u_{3}, \\ldots$ is determined by $u_{1}$ and the following recurrence relation for $n \\geq 1$ : $$ 4 u_{n+1}=\\sqrt[3]{64 u_{n}+15} $$ Describe, with proof, the behavior of $u_{n}$ as $n \\rightarrow \\infty$.","solution":"16. The sequence $\\left\\{u_{n}\\right\\}$ is bounded, whatever $u_{1}$ is. Indeed, assume the opposite, and let $u_{m}$ be the first member of the sequence such that $\\left|u_{m}\\right|>\\max \\left\\{2,\\left|u_{1}\\right|\\right\\}$. Then $\\left|u_{m-1}\\right|=\\left|u_{m}^{3}-15 \/ 64\\right|>\\left|u_{m}\\right|$, which is impossible. Next, let us see for what values of $u_{m}, u_{m+1}$ is greater, equal, or smaller, respectively. If $u_{m+1}=u_{m}$, then $u_{m}=u_{m+1}^{3}-15 \/ 64=u_{m}^{3}-15 \/ 64$; i.e., $u_{m}$ is a root of $x^{3}-x-15 \/ 64=0$. This equation factors as $(x+1 \/ 4)\\left(x^{2}-x \/ 4-\\right.$ $15 \/ 16)=0$, and hence $u_{m}$ is equal to $x_{1}=(1-\\sqrt{61}) \/ 8, x_{2}=-1 \/ 4$, or $x_{3}=(1+\\sqrt{61}) \/ 8$, and these are the only possible limits of the sequence. Each of $u_{m+1}>u_{m}, u_{m+1}0$ respectively. Thus the former is satisfied for $u_{m}$ in the interval $I_{1}=\\left(-\\infty, x_{1}\\right)$ or $I_{3}=\\left(x_{2}, x_{3}\\right)$, while the latter is satisfied for $u_{m}$ in $I_{2}=\\left(x_{1}, x_{2}\\right)$ or $I_{4}=\\left(x_{3}, \\infty\\right)$. Moreover, since the function $f(x)=\\sqrt[3]{x+15 \/ 64}$ is strictly increasing with fixed points $x_{1}, x_{2}, x_{3}$, it follows that $u_{m}$ will never escape from the interval $I_{1}, I_{2}, I_{3}$, or $I_{4}$ to which it belongs initially. Therefore: (1) if $u_{1}$ is one of $x_{1}, x_{2}, x_{3}$, the sequence $\\left\\{u_{m}\\right\\}$ is constant; (2) if $u_{1} \\in I_{1}$, then the sequence is strictly increasing and tends to $x_{1}$; (3) if $u_{1} \\in I_{2}$, then the sequence is strictly decreasing and tends to $x_{1}$; (4) if $u_{1} \\in I_{3}$, then the sequence is strictly increasing and tends to $x_{3}$; (5) if $u_{1} \\in I_{4}$, then the sequence is strictly decreasing and tends to $x_{3}$.","problem_type":null,"tier":0} +{"year":"1981","problem_phase":"shortlisted","problem":"17. (USS) ${ }^{\\text {IMO5 }}$ Three equal circles touch the sides of a triangle and have one common point $O$. Show that the center of the circle inscribed in and of the circle circumscribed about the triangle $A B C$ and the point $O$ are collinear.","solution":"17. Let us denote by $S_{A}, S_{B}, S_{C}$ the centers of the given circles, where $S_{A}$ lies on the bisector of $\\angle A$, etc. Then $S_{A} S_{B}\\left\\|A B, S_{B} S_{C}\\right\\| B C, S_{C} S_{A} \\| C A$, so that the inner bisectors of the angles of triangle $A B C$ are also inner bisectors of the angles of $\\triangle S_{A} S_{B} S_{C}$. These two triangles thus have a common incenter $S$, which is also the center of the homothety $\\chi$ mapping $\\triangle S_{A} S_{B} S_{C}$ onto $\\triangle A B C$. The point $O$ is the circumcenter of triangle $S_{A} S_{B} S_{C}$, and so is mapped by $\\chi$ onto the circumcenter $P$ of $A B C$. This means that $O, P$, and the center $S$ of $\\chi$ are collinear.","problem_type":null,"tier":0} +{"year":"1981","problem_phase":"shortlisted","problem":"18. (USS) Several equal spherical planets are given in outer space. On the surface of each planet there is a set of points that is invisible from any of the remaining planets. Prove that the sum of the areas of all these sets is equal to the area of the surface of one planet.","solution":"18. Let $C$ be the convex hull of the set of the planets: its border consists of parts of planes, parts of cylinders, and parts of the surfaces of some planets. These parts of planets consist exactly of all the invisible points; any point on a planet that is inside $C$ is visible. Thus it remains to show that the areas of all the parts of planets lying on the border of $C$ add up to the area of one planet. As we have seen, an invisible part of a planet is bordered by some main spherical arcs, parallel two by two. Now fix any planet $P$, and translate these arcs onto arcs on the surface of $P$. All these arcs partition the surface of $P$ into several parts, each of which corresponds to the invisible part of one of the planets. This correspondence is bijective, and therefore the statement follows.","problem_type":null,"tier":0} +{"year":"1981","problem_phase":"shortlisted","problem":"19. (YUG) A finite set of unit circles is given in a plane such that the area of their union $U$ is $S$. Prove that there exists a subset of mutually disjoint circles such that the area of their union is greater that $\\frac{2 S}{9}$.","solution":"19. Consider the partition of plane $\\pi$ into regular hexagons, each having inradius 2. Fix one of these hexagons, denoted by $\\gamma$. For any other hexagon $x$ in the partition, there exists a unique translation $\\tau_{x}$ taking it onto $\\gamma$. Define the mapping $\\varphi: \\pi \\rightarrow \\gamma$ as follows: If $A$ belongs to the interior of a hexagon $x$, then $\\varphi(A)=\\tau_{x}(A)$ (if $A$ is on the border of some hexagon, it does not actually matter where its image is). The total area of the images of the union of the given circles equals $S$, while the area of the hexagon $\\gamma$ is $8 \\sqrt{3}$. Thus there exists a point $B$ of $\\gamma$ that is covered at least $\\frac{S}{8 \\sqrt{3}}$ times, i.e., such that $\\varphi^{-1}(B)$ consists of at least $\\frac{S}{8 \\sqrt{3}}$ distinct points of the plane that belong to some of the circles. For any of these points, take a circle that contains it. All these circles are disjoint, with total area not less than $\\frac{\\pi}{8 \\sqrt{3}} S \\geq 2 S \/ 9$. Remark. The statement becomes false if the constant $2 \/ 9$ is replaced by any number greater than $1 \/ 4$. In that case a counterexample is, for example, a set of unit circles inside a circle of radius 2 covering a sufficiently large part of its area.","problem_type":null,"tier":0} +{"year":"1981","problem_phase":"shortlisted","problem":"2. (BUL) A sphere $S$ is tangent to the edges $A B, B C, C D, D A$ of a tetrahedron $A B C D$ at the points $E, F, G, H$ respectively. The points $E, F, G, H$ are the vertices of a square. Prove that if the sphere is tangent to the edge $A C$, then it is also tangent to the edge $B D$.","solution":"2. Lemma. Let $E, F, G, H, I$, and $K$ be points on edges $A B, B C, C D, D A$, $A C$, and $B D$ of a tetrahedron. Then there is a sphere that touches the edges at these points if and only if $$ \\begin{aligned} & A E=A H=A I, \\quad B E=B F=B K, \\\\ & C F=C G=C I, \\quad D G=D H=D K . \\end{aligned} $$ Proof. The \"only if\" side of the equivalence is obvious. We now assume (*). Denote by $\\epsilon, \\phi, \\gamma, \\eta, \\iota$, and $\\kappa$ planes through $E, F, G, H, I, K$ perpendicular to $A B, B C, C D, D A, A C$ and $B D$ respectively. Since the three planes $\\epsilon, \\eta$, and $\\iota$ are not mutually parallel, they intersect in a common point $O$. Clearly, $\\triangle A E O \\cong$ ![](https:\/\/cdn.mathpix.com\/cropped\/2024_11_18_8e985d6b9c83aa3e9d0eg-451.jpg?height=324&width=496&top_left_y=1374&top_left_x=845) $\\triangle A H O \\cong \\triangle A I O$; hence $O E=O H=O I=r$, and the sphere $\\sigma(O, r)$ is tangent to $A B, A D, A C$. To prove that $\\sigma$ is also tangent to $B C, C D, B D$ it suffices to show that planes $\\phi, \\gamma$, and $\\kappa$ also pass through $O$. Without loss of generality we can prove this for just $\\phi$. By the conditions for $E, F, I$, these are exactly the points of tangency of the incircle of $\\triangle A B C$ and its sides, and if $S$ is the incenter, then $S E \\perp A B, S F \\perp B C, S I \\perp A C$. Hence $\\epsilon, \\iota$, and $\\phi$ all pass through $S$ and are perpendicular to the plane $A B C$, and consequently all share the line $l$ through $S$ perpendicular to $A B C$. Since $l=\\epsilon \\cap \\iota$, the point $O$ will be situated on $l$, and hence $\\phi$ will also contain $O$. This completes our proof of the lemma. Let $A H=A E=x, B E=B F=y, C F=C G=z$, and $D G=D H=w$. If the sphere is also tangent to $A C$ at some point $I$, then $A I=x$ and $I C=z$. Using the stated lemma it suffices to prove that if $A C=x+z$, then $B D=y+w$. Let $E F=F G=G H=H I=t, \\angle B A D=\\alpha, \\angle A B C=\\beta, \\angle B C D=\\gamma$, and $\\angle A D C=\\delta$. We get $$ t^{2}=E H^{2}=A E^{2}+A H^{2}-2 \\cdot A E \\cdot A H \\cos \\alpha=2 x^{2}(1-\\cos \\alpha) $$ We similarly conclude that $t^{2}=2 y^{2}(1-\\cos \\beta)=2 z^{2}(1-\\cos \\gamma)=2 w^{2}(1-$ $\\cos \\delta$ ). Further, using that $A B=x+y, B C=y+z, \\cos \\beta=1-t^{2} \/ 2 y^{2}$, we obtain $$ A C^{2}=A B^{2}+B C^{2}-2 A B \\cdot B C \\cos \\beta=(x-z)^{2}+t^{2}\\left(\\frac{x}{y}+1\\right)\\left(\\frac{z}{y}+1\\right) $$ Analogously, from the triangle $A D C$ we get $A C^{2}=(x-z)^{2}+t^{2}(x \/ w+$ $1)(z \/ w+1)$, which gives $(x \/ y+1)(z \/ y+1)=(x \/ w+1)(z \/ w+1)$. Since $f(s)=(x \/ s+1)(z \/ s+1)$ is a decreasing function in $s$, it follows that $y=w$; similarly $x=z$. Hence $C F=C G=x$ and $D G=D H=y$. Hence $A C \\| E F$ and $A C: t=$ $A C: E F=A B: E B=(x+y): y$; i.e., $A C=t(x+y) \/ y$. Similarly, from the triangle $A B D$, we get that $B D=t(x+y) \/ x$. Hence if $A C=x+z=2 x$, it follows that $2 x=t(x+y) \/ y \\Rightarrow 2 x y=t(x+y) \\Rightarrow B D=t(x+y) \/ x=$ $2 y=y+w$. This completes the proof. Second solution. Without loss of generality, assume that $E F=2$. Consider the Cartesian system in which points $O, E, F, G, H$ respectively have coordinates $(0,0,0),(-1,-1, a),(1,-1, a),(1,1, a),(-1,1, a)$. Line $A H$ is perpendicular to $O H$ and $A E$ is perpendicular to $O E$; hence from Pythagoras's theorem $A O^{2}=A H^{2}+H O^{2}=A E^{2}+E O^{2}=A E^{2}+H O^{2}$, which implies $A H=A E$. Therefore the $y$-coordinate of $A$ is zero; analogously the $x$-coordinates of $B$ and $D$ and the $y$-coordinate of $C$ are 0 . Let $A$ have coordinates $\\left(x_{0}, 0, z_{1}\\right)$ : then $\\overrightarrow{E A}\\left(x_{0}+1,1, z_{1}-a\\right) \\perp \\overrightarrow{E O}(1,1,-a)$, i.e., $\\overrightarrow{E A} \\cdot \\overrightarrow{E O}=x_{0}+2+a\\left(a-z_{1}\\right)=0$. Similarly, for $B\\left(0, y_{0}, z_{2}\\right)$ we have $y_{0}+2+a\\left(a-z_{2}\\right)=0$. This gives us $$ z_{1}=\\frac{x_{0}+a^{2}+2}{a}, \\quad z_{2}=\\frac{y_{0}+a^{2}+2}{a} $$ We haven't used yet that $A\\left(x_{0}, 0, z_{1}\\right), E(-1,-1, a)$ and $B\\left(0, y_{0}, z_{2}\\right)$ are collinear, so let $A^{\\prime}, B^{\\prime}, E^{\\prime}$ be the feet of perpendiculars from $A, B, E$ to the plane $x y$. The line $A^{\\prime} B^{\\prime}$, given by $y_{0} x+x_{0} y=x_{0} y_{0}, z=0$, contains the point $E^{\\prime}(-1,-1,0)$, from which we obtain $$ \\left(x_{0}+1\\right)\\left(y_{0}+1\\right)=1 $$ In the same way, from the points $B$ and $C$ we get relations similar to (1) and (2) and conclude that $C$ has the coordinates $C\\left(-x_{0}, 0, z_{1}\\right)$. Similarly we get $D\\left(0,-y_{0}, z_{2}\\right)$. The condition that $A C$ is tangent to the sphere $\\sigma(O, O E)$ is equivalent to $z_{1}=\\sqrt{a^{2}+2}$, i.e., to $x_{0}=a \\sqrt{a^{2}+2}-\\left(a^{2}+2\\right)$. But then (2) implies that $y_{0}=-a \\sqrt{a^{2}+2}-\\left(a^{2}+2\\right)$ and $z_{2}=-\\sqrt{a^{2}+2}$, which means that the sphere $\\sigma$ is tangent to $B D$ as well. This finishes the proof.","problem_type":null,"tier":0} +{"year":"1981","problem_phase":"shortlisted","problem":"3. (CAN) Find the minimum value of $$ \\max (a+b+c, b+c+d, c+d+e, d+e+f, e+f+g) $$ subject to the constraints (i) $a, b, c, d, e, f, g \\geq 0$, (ii) $a+b+c+d+e+f+g=1$.","solution":"3. Denote $\\max (a+b+c, b+c+d, c+d+e, d+e+f, e+f+g)$ by $p$. We have $$ (a+b+c)+(c+d+e)+(e+f+g)=1+c+e \\leq 3 p $$ which implies that $p \\geq 1 \/ 3$. However, $p=1 \/ 3$ is achieved by taking $(a, b, c, d, e, f, g)=(1 \/ 3,0,0,1 \/ 3,0,0,1 \/ 3)$. Therefore the answer is $1 \/ 3$. Remark. In fact, one can prove a more general statement in the same way. Given positive integers $n, k, n \\geq k$, if $a_{1}, a_{2}, \\ldots, a_{n}$ are nonnegative real numbers with sum 1 , then the minimum value of $\\max _{i=1, \\ldots, n-k+1}\\left\\{a_{i}+\\right.$ $\\left.a_{i+1}+\\cdots+a_{i+k-1}\\right\\}$ is $1 \/ r$, where $r$ is the integer with $k(r-1)0$ is an integer depending on $n$. By (1), this is equivalent to $a\\left(\\alpha^{n}-(-1)^{n} \\alpha^{-n}\\right)+$ $b\\left(\\alpha^{n+1}+(-1)^{n} \\alpha^{-n-1}\\right)=\\alpha^{k_{n}}-(-1)^{k_{n}} \\alpha^{-k_{n}}$, i.e., $$ \\alpha^{k_{n}-n}=a+b \\alpha-\\alpha^{-2 n}(-1)^{n}\\left(a-b \\alpha^{-1}-(-\\alpha)^{n-k_{n}}\\right) \\rightarrow a+b \\alpha $$ as $n \\rightarrow \\infty$. Hence, since $k_{n}$ is an integer, $k_{n}-n$ must be constant from some point on: $k_{n}=n+k$ and $\\alpha^{k}=a+b \\alpha$. Then it follows from (2) that $\\alpha^{-k}=a-b \\alpha^{-1}$, and from (1) we conclude that $a f_{n}+b f_{n+1}=$ $f_{k+n}$ holds for every $n$. Putting $n=1$ and $n=2$ in the previous relation and solving the obtained system of equations we get $a=f_{k-1}$, $b=f_{k}$. It is easy to verify that such $a$ and $b$ satisfy the conditions. (b) As in (a), suppose that $u f_{n}^{2}+v f_{n+1}^{2}=f_{l_{n}}$ for all $n$. This leads to $$ \\begin{aligned} u+v \\alpha^{2}-\\sqrt{5} \\alpha^{l_{n}-2 n}= & 2(u-v)(-1)^{n} \\alpha^{-2 n} \\\\ & -\\left(u \\alpha^{-4 n}+v \\alpha^{-4 n-2}+(-1)^{l_{n}} \\sqrt{5} \\alpha^{-l_{n}-2 n}\\right) \\\\ \\rightarrow & 0 \\end{aligned} $$ as $n \\rightarrow \\infty$. Thus $u+v \\alpha^{2}=\\sqrt{5} \\alpha^{l_{n}-2 n}$, and $l_{n}-2 n=k$ is equal to a constant. Putting this into the above equation and multiplying by $\\alpha^{2 n}$ we get $u-v \\rightarrow 0$ as $n \\rightarrow \\infty$, i.e., $u=v$. Finally, substituting $n=1$ and $n=2$ in $u f_{n}^{2}+u f_{n+1}^{2}=f_{l_{n}}$ we easily get that the only possibility is $u=v=1$ and $k=1$. It is easy to verify that such $u$ and $v$ satisfy the conditions.","problem_type":null,"tier":0} +{"year":"1981","problem_phase":"shortlisted","problem":"5. (COL) A cube is assembled with 27 white cubes. The larger cube is then painted black on the outside and disassembled. A blind man reassembles it. What is the probability that the cube is now completely black on the outside? Give an approximation of the size of your answer.","solution":"5. There are four types of small cubes upon disassembling: (1) 8 cubes with three faces, painted black, at one corner; (2) 12 cubes with two black faces, both at one edge; (3) 6 cubes with one black face; (4) 1 completely white cube. All cubes of type (1) must go to corners, and be placed in a correct way (one of three): for this step we have $3^{8} \\cdot 8$ ! possibilities. Further, all cubes of type (2) must go in a correct way (one of two) to edges, admitting $2^{12} \\cdot 12$ ! possibilities; similarly, there are $4^{6} \\cdot 6$ ! ways for cubes of type (3), and 24 ways for the cube of type (4). Thus the total number of good reassemblings is $3^{8} 8!\\cdot 2^{12} 12!\\cdot 4^{6} 6!\\cdot 24$, while the number of all possible reassemblings is $24^{27} \\cdot 27!$. The desired probability is $\\frac{3^{8} 8!\\cdot 2^{12} 12!\\cdot 4^{6} 6!\\cdot 24}{24^{27} \\cdot 27!}$. It is not necessary to calculate these numbers to find out that the blind man practically has no chance to reassemble the cube in a right way: in fact, the probability is of order $1.8 \\cdot 10^{-37}$.","problem_type":null,"tier":0} +{"year":"1981","problem_phase":"shortlisted","problem":"6. (CUB) Let $P(z)$ and $Q(z)$ be complex-variable polynomials, with degree not less than 1. Let $$ P_{k}=\\{z \\in \\mathbb{C} \\mid P(z)=k\\}, \\quad Q_{k}=\\{z \\in \\mathbb{C} \\mid Q(z)=k\\} $$ Let also $P_{0}=Q_{0}$ and $P_{1}=Q_{1}$. Prove that $P(z) \\equiv Q(z)$.","solution":"6. Assume w.l.o.g. that $n=\\operatorname{deg} P \\geq \\operatorname{deg} Q$, and let $P_{0}=\\left\\{z_{1}, z_{2}, \\ldots, z_{k}\\right\\}$, $P_{1}=\\left\\{z_{k+1}, z_{k+2}, \\ldots z_{k+m}\\right\\}$. The polynomials $P$ and $Q$ match at $k+m$ points $z_{1}, z_{2}, \\ldots, z_{k+m}$; hence if we prove that $k+m>n$, the result will follow. By the assumption, $P(x)=\\left(x-z_{1}\\right)^{\\alpha_{1}} \\cdots\\left(x-z_{k}\\right)^{\\alpha_{k}}=\\left(x-z_{k+1}\\right)^{\\alpha_{k+1}} \\cdots\\left(x-z_{k+m}\\right)^{\\alpha_{k+m}}+1$ for some positive integers $\\alpha_{1}, \\ldots, \\alpha_{k+m}$. Let us consider $P^{\\prime}(x)$. As we know, it is divisible by $\\left(x-z_{i}\\right)^{\\alpha_{i}-1}$ for $i=1,2, \\ldots, k+m$; i.e., $$ \\prod_{i=1}^{k+m}\\left(x-z_{i}\\right)^{\\alpha_{i}-1} \\mid P^{\\prime}(x) $$ Therefore $2 n-k-m=\\operatorname{deg} \\prod_{i=1}^{k+m}\\left(x-z_{i}\\right)^{\\alpha_{i}-1} \\leq \\operatorname{deg} P^{\\prime}=n-1$, i.e., $k+m \\geq n+1$, as we claimed.","problem_type":null,"tier":0} +{"year":"1981","problem_phase":"shortlisted","problem":"7. (FIN) ${ }^{\\mathrm{IMO} 6}$ Assume that $f(x, y)$ is defined for all positive integers $x$ and $y$, and that the following equations are satisfied: $$ \\begin{aligned} f(0, y) & =y+1 \\\\ f(x+1,0) & =f(x, 1) \\\\ f(x+1, y+1) & =f(x, f(x+1, y)) \\end{aligned} $$ Determine $f(2,2), f(3,3)$ and $f(4,4)$. Alternative version: Determine $f(4,1981)$.","solution":"7. We immediately find that $f(1,0)=f(0,1)=2$. Then $f(1, y+1)=$ $f(0, f(1, y))=f(1, y)+1$; hence $f(1, y)=y+2$ for $y \\geq 0$. Next we find that $f(2,0)=f(1,1)=3$ and $f(2, y+1)=f(1, f(2, y))=f(2, y)+2$, from which $f(2, y)=2 y+3$. Particularly, $f(2,2)=7$. Further, $f(3,0)=$ $f(2,1)=5$ and $f(3, y+1)=f(2, f(3, y))=2 f(3, y)+3$. This gives by induction $f(3, y)=2^{y+3}-3$. For $y=3, f(3,3)=61$. Finally, from $f(4,0)=f(3,1)=13$ and $f(4, y+1)=f(3, f(4, y))=2^{f(4, y)+3}-3$, we conclude that $$ f(4, y)=2^{2 y^{2}}-3 \\quad(y+3 \\text { twos }) $$","problem_type":null,"tier":0} +{"year":"1981","problem_phase":"shortlisted","problem":"8. (FRG) ${ }^{\\mathrm{IMO} 2}$ Let $f(n, r)$ be the arithmetic mean of the minima of all $r$ subsets of the set $\\{1,2, \\ldots, n\\}$. Prove that $f(n, r)=\\frac{n+1}{r+1}$.","solution":"8. Since the number $k, k=1,2, \\ldots, n-r+1$, is the minimum in exactly $\\binom{n-k}{r-1} r$-element subsets of $\\{1,2, \\ldots, n\\}$, it follows that $$ f(n, r)=\\frac{1}{\\binom{n}{r}} \\sum_{k=1}^{n-r+1} k\\binom{n-k}{r-1} $$ To calculate the sum in the above expression, using the equality $\\binom{r+j}{j}=$ $\\sum_{i=0}^{j}\\binom{r+i-1}{r-1}$, we note that $$ \\begin{aligned} \\sum_{k=1}^{n-r+1} k\\binom{n-k}{r-1} & =\\sum_{j=0}^{n-r}\\left(\\sum_{i=0}^{j}\\binom{r+i-1}{r-1}\\right) \\\\ & =\\sum_{j=0}^{n-r}\\binom{r+j}{r}=\\binom{n+1}{r+1}=\\frac{n+1}{r+1}\\binom{n}{r} . \\end{aligned} $$ Therefore $f(n, r)=(n+1) \/(r+1)$.","problem_type":null,"tier":0} +{"year":"1981","problem_phase":"shortlisted","problem":"9. (FRG) A sequence $\\left(a_{n}\\right)$ is defined by means of the recursion $$ a_{1}=1, \\quad a_{n+1}=\\frac{1+4 a_{n}+\\sqrt{1+24 a_{n}}}{16} $$ Find an explicit formula for $a_{n}$.","solution":"9. If we put $1+24 a_{n}=b_{n}^{2}$, the given recurrent relation becomes $$ \\frac{2}{3} b_{n+1}^{2}=\\frac{3}{2}+\\frac{b_{n}^{2}}{6}+b_{n}=\\frac{2}{3}\\left(\\frac{3}{2}+\\frac{b_{n}}{2}\\right)^{2}, \\quad \\text { i.e., } \\quad b_{n+1}=\\frac{3+b_{n}}{2} $$ where $b_{1}=5$. To solve this recurrent equation, we set $c_{n}=2^{n-1} b_{n}$. From (1) we obtain $$ \\begin{aligned} c_{n+1} & =c_{n}+3 \\cdot 2^{n-1}=\\cdots=c_{1}+3\\left(1+2+2^{2}+\\cdots+2^{n-1}\\right) \\\\ & =5+3\\left(2^{n}-1\\right)=3 \\cdot 2^{n}+2 \\end{aligned} $$ Therefore $b_{n}=3+2^{-n+2}$ and consequently $$ a_{n}=\\frac{b_{n}^{2}-1}{24}=\\frac{1}{3}\\left(1+\\frac{3}{2^{n}}+\\frac{1}{2^{2 n-1}}\\right)=\\frac{1}{3}\\left(1+\\frac{1}{2^{n-1}}\\right)\\left(1+\\frac{1}{2^{n}}\\right) . $$","problem_type":null,"tier":0} +{"year":"1982","problem_phase":"shortlisted","problem":"1. A1 (GBR 3) ${ }^{\\mathrm{IMO}}$ The function $f(n)$ is defined for all positive integers $n$ and takes on nonnegative integer values. Also, for all $m, n$, $$ \\begin{gathered} f(m+n)-f(m)-f(n)=0 \\text { or } 1 \\\\ f(2)=0, \\quad f(3)>0, \\quad \\text { and } \\quad f(9999)=3333 \\end{gathered} $$ Determine $f(1982)$.","solution":"1. From $f(1)+f(1) \\leq f(2)=0$ we obtain $f(1)=0$. Since $0A_{1} C$. Similarly, $C_{1} C>C_{1} A$. Hence the perpendicular bisector $l_{A C}$ of $A C$ separates points $A_{1}$ and $C_{1}$. Since $B_{1}, D_{1}$ lie on $l_{A C}$, this means that $A_{1}$ and $C_{1}$ are on opposite sides $B_{1} D_{1}$. Similarly one can show that $B_{1}$ and $D_{1}$ are on opposite sides of $A_{1} C_{1}$. (b) Since $A_{2} B_{2} \\perp C_{1} D_{1}$ and $C_{1} D_{1} \\perp A B$, it follows that $A_{2} B_{2} \\| A B$. Similarly $A_{2} C_{2}\\left\\|A C, A_{2} D_{2}\\right\\| A D, B_{2} C_{2}\\left\\|B C, B_{2} D_{2}\\right\\| B D$, and $C_{2} D_{2} \\| C D$. Hence $\\triangle A_{2} B_{2} C_{2} \\sim \\triangle A B C$ and $\\triangle A_{2} D_{2} C_{2} \\sim \\triangle A D C$, and the result follows.","problem_type":"Combinatorics","tier":0} +{"year":"1982","problem_phase":"shortlisted","problem":"15. C3 (CAN 5) Show that $$ \\frac{1-s^{a}}{1-s} \\leq(1+s)^{a-1} $$ holds for every $1 \\neq s>0$ real and $04000$. (b) The sequence $x_{n}=1 \/ 2^{n}$ obviously satisfies the required condition. Second solution to part (a). For each $n \\in \\mathbb{N}$, let us find a constant $c_{n}$ such that the inequality $x_{0}^{2} \/ x_{1}+\\cdots+x_{n-1}^{2} \/ x_{n} \\geq c_{n} x_{0}$ holds for any sequence $x_{0} \\geq x_{1} \\geq \\cdots \\geq x_{n}>0$. For $n=1$ we can take $c_{1}=1$. Assuming that $c_{n}$ exists, we have $$ \\frac{x_{0}^{2}}{x_{1}}+\\left(\\frac{x_{1}^{2}}{x_{2}}+\\cdots+\\frac{x_{n}^{2}}{x_{n+1}}\\right) \\geq \\frac{x_{0}^{2}}{x_{1}}+c_{n} x_{1} \\geq 2 \\sqrt{x_{0}^{2} c_{n}}=x_{0} \\cdot 2 \\sqrt{c_{n}} $$ Thus we can take $c_{n+1}=2 \\sqrt{c_{n}}$. Then inductively $c_{n}=2^{2-1 \/ 2^{n-2}}$, and since $c_{n} \\rightarrow 4$ as $n \\rightarrow \\infty$, the result follows. Third solution. Since $\\left\\{x_{n}\\right\\}$ is decreasing, there exists $\\lim _{n \\rightarrow \\infty} x_{n}=x \\geq 0$. If $x>0$, then $x_{n-1}^{2} \/ x_{n} \\geq x_{n} \\geq x$ holds for each $n$, and the result is trivial. If otherwise $x=0$, then we note that $x_{n-1}^{2} \/ x_{n} \\geq 4\\left(x_{n-1}-x_{n}\\right)$ for each $n$, with equality if and only if $x_{n-1}=2 x_{n}$. Hence $$ \\lim _{n \\rightarrow \\infty} \\sum_{k=1}^{n} \\frac{x_{k-1}^{2}}{x_{k}} \\geq \\lim _{n \\rightarrow \\infty} \\sum_{k=1}^{n} 4\\left(x_{k-1}-x_{k}\\right)=4 x_{0}=4 $$ Equality holds if and only if $x_{n-1}=2 x_{n}$ for all $n$, and consequently $x_{n}=1 \/ 2^{n}$.","problem_type":"Algebra","tier":0} +{"year":"1982","problem_phase":"shortlisted","problem":"4. A4 (BUL 2) Determine all real values of the parameter $a$ for which the equation $$ 16 x^{4}-a x^{3}+(2 a+17) x^{2}-a x+16=0 $$ has exactly four distinct real roots that form a geometric progression.","solution":"4. Suppose that $a$ satisfies the requirements of the problem and that $x, q x$, $q^{2} x, q^{3} x$ are the roots of the given equation. Then $x \\neq 0$ and we may assume that $|q|>1$, so that $|x|<|q x|<\\left|q^{2} x\\right|<\\left|q^{3} x\\right|$. Since the equation is symmetric, $1 \/ x$ is also a root and therefore $1 \/ x=q^{3} x$, i.e., $q=x^{-2 \/ 3}$. It follows that the roots are $x, x^{1 \/ 3}, x^{-1 \/ 3}, x^{-1}$. Now by Vieta's formula we have $x+x^{1 \/ 3}+x^{-1 \/ 3}+x^{-1}=a \/ 16$ and $x^{4 \/ 3}+x^{2 \/ 3}+2+x^{-2 \/ 3}+x^{-4 \/ 3}=$ $(2 a+17) \/ 16$. On setting $z=x^{1 \/ 3}+x^{-1 \/ 3}$ these equations become $$ \\begin{aligned} z^{3}-2 z & =a \/ 16 \\\\ \\left(z^{2}-2\\right)^{2}+z^{2}-2 & =(2 a+17) \/ 16 \\end{aligned} $$ Substituting $a=16\\left(z^{3}-2 z\\right)$ in the second equation leads to $z^{4}-2 z^{3}-$ $3 z^{2}+4 z+15 \/ 16=0$. We observe that this polynomial factors as $(z+$ $3 \/ 2)(z-5 \/ 2)\\left(z^{2}-z-1 \/ 4\\right)$. Since $|z|=\\left|x^{1 \/ 3}+x^{-1 \/ 3}\\right| \\geq 2$, the only viable value is $z=5 \/ 2$. Consequently $a=170$ and the roots are $1 \/ 8,1 \/ 2,2,8$.","problem_type":"Algebra","tier":0} +{"year":"1982","problem_phase":"shortlisted","problem":"5. A5 (NET 2) ${ }^{\\mathrm{IMO}}$ Let $A_{1} A_{2} A_{3} A_{4} A_{5} A_{6}$ be a regular hexagon. Each of its diagonals $A_{i-1} A_{i+1}$ is divided into the same ratio $\\frac{\\lambda}{1-\\lambda}$, where $0<\\lambda<1$, by a point $B_{i}$ in such a way that $A_{i}, B_{i}$, and $B_{i+2}$ are collinear ( $i \\equiv$ $1, \\ldots, 6(\\bmod 6))$. Compute $\\lambda$.","solution":"5. We first observe that $\\triangle A_{5} B_{4} A_{4} \\cong$ $\\triangle A_{3} B_{2} A_{2}$. Since $\\angle A_{5} A_{3} A_{2}=90^{\\circ}$, we have $\\angle A_{2} B_{4} A_{4}=\\angle A_{2} B_{4} A_{3}+$ $\\angle A_{3} B_{4} A_{4}=\\left(90^{\\circ}-\\angle B_{2} A_{2} A_{3}\\right)+$ $\\left(\\angle B_{4} A_{5} A_{4}+\\angle A_{5} A_{4} B_{4}\\right)=90^{\\circ}+$ $\\angle B_{4} A_{5} A_{4}=120^{\\circ}$. Hence $B_{4}$ belongs to the circle with center $A_{3}$ and radius $A_{3} A_{4}$, so $A_{3} A_{4}=A_{3} B_{4}$. ![](https:\/\/cdn.mathpix.com\/cropped\/2024_11_18_8e985d6b9c83aa3e9d0eg-461.jpg?height=311&width=428&top_left_y=1245&top_left_x=866) Thus $\\lambda=A_{3} B_{4} \/ A_{3} A_{5}=A_{3} A_{4} \/ A_{3} A_{5}=1 \/ \\sqrt{3}$.","problem_type":"Algebra","tier":0} +{"year":"1982","problem_phase":"shortlisted","problem":"6. A6 (VIE 1) ${ }^{\\text {IMO6 }}$ Let $S$ be a square with sides of length 100 and let $L$ be a path within $S$ that does not meet itself and that is composed of linear segments $A_{0} A_{1}, A_{1} A_{2}, \\ldots, A_{n-1} A_{n}$ with $A_{0} \\neq A_{n}$. Suppose that for every point $P$ of the boundary of $S$ there is a point of $L$ at a distance from $P$ not greater than $\\frac{1}{2}$. Prove that there are two points $X$ and $Y$ in $L$ such that the distance between $X$ and $Y$ is not greater than 1 and the length of that part of $L$ that lies between $X$ and $Y$ is not smaller than 198.","solution":"6. Denote by $d(U, V)$ the distance between points or sets of points $U$ and $V$. For $P, Q \\in L$ we shall denote by $L_{P Q}$ the part of $L$ between points $P$ and $Q$ and by $l_{P Q}$ the length of this part. Let us denote by $S_{i}(i=1,2,3,4)$ the vertices of $S$ and by $T_{i}$ points of $L$ such that $S_{i} T_{i} \\leq 1 \/ 2$ in such a way that $l_{A_{0} T_{1}}$ is the least of the $l_{A_{0} T_{i}}$ 's, $S_{2}$ and $S_{4}$ are neighbors of $S_{1}$, and $l_{A_{0} T_{2}}2^{n}$ cities. We shall prove that there is a round trip by at least one $A_{i}$ containing an odd number of stops. For $n=1$ the statement is trivial, since one airline serves at least 3 cities and hence $P_{1} P_{2} P_{3} P_{1}$ is a round trip with 3 landings. We use induction on $n$, and assume that $n>1$. Suppose the contrary, that all round trips by $A_{n}$ consist of an even number of stops. Then we can separate the cities into two nonempty classes $Q=\\left\\{Q_{1}, \\ldots, Q_{r}\\right\\}$ and $R=\\left\\{R_{1}, \\ldots, R_{s}\\right\\}$ (where $r+s=N$ ), so that each flight by $A_{n}$ runs between a $Q$-city and an $R$-city. (Indeed, take any city $Q_{1}$ served by $A_{n}$; include each city linked to $Q_{1}$ by $A_{n}$ in $R$, then include in $Q$ each city linked by $A_{n}$ to any $R$-city, etc. Since all round trips are even, no contradiction can arise.) At least one of $r, s$ is larger than $2^{n-1}$, say $r>2^{n-1}$. But, only $A_{1}, \\ldots, A_{n-1}$ run between cities in $\\left\\{Q_{1}, \\ldots, Q_{r}\\right\\}$; hence by the induction hypothesis at least one of them flies a round trip with an odd number of landings, a contradiction. It only remains to notice that for $n=10,2^{n}=1024<1983$. Remark. If there are $N=2^{n}$ cities, there is a schedule with $n$ airlines that contain no odd round trip by any of the airlines. Let the cities be $P_{k}$, $k=0, \\ldots, 2^{n}-1$, and write $k$ in the binary system as an $n$-digit number $\\overline{a_{1} \\ldots a_{n}}$ (e.g., $1=(0 \\ldots 001)_{2}$ ). Link $P_{k}$ and $P_{l}$ by $A_{i}$ if the $i$ th digits $k$ and $l$ are distinct but the first $i-1$ digits are the same. All round trips under $A_{i}$ are even, since the $i$ th digit alternates.","problem_type":null,"tier":0} +{"year":"1983","problem_phase":"shortlisted","problem":"10. (FIN 1) Let $p$ and $q$ be integers. Show that there exists an interval $I$ of length $1 \/ q$ and a polynomial $P$ with integral coefficients such that $$ \\left|P(x)-\\frac{p}{q}\\right|<\\frac{1}{q^{2}} $$ for all $x \\in I$.","solution":"10. Choose $P(x)=\\frac{p}{q}\\left((q x-1)^{2 n+1}+1\\right), I=[1 \/ 2 q, 3 \/ 2 q]$. Then all the coefficients of $P$ are integers, and $$ \\left|P(x)-\\frac{p}{q}\\right|=\\left|\\frac{p}{q}(q x-1)^{2 n+1}\\right| \\leq\\left|\\frac{p}{q}\\right| \\frac{1}{2^{2 n+1}}, $$ for $x \\in I$. The desired inequality follows if $n$ is chosen large enough.","problem_type":null,"tier":0} +{"year":"1983","problem_phase":"shortlisted","problem":"11. (FIN $\\mathbf{2}^{\\prime}$ ) Let $f:[0,1] \\rightarrow \\mathbb{R}$ be continuous and satisfy: $$ \\begin{aligned} b f(2 x) & =f(x), & & 0 \\leq x \\leq 1 \/ 2 \\\\ f(x) & =b+(1-b) f(2 x-1), & & 1 \/ 2 \\leq x \\leq 1 \\end{aligned} $$ where $b=\\frac{1+c}{2+c}, c>0$. Show that $01 \/ 2$. Assume $x=\\sum_{j=0}^{n} a_{j} 2^{-j}$, and for $k \\geq 2, v=x+2^{-n-k+1}, u=x+2^{-n-k}=(v+x) \/ 2$. Then $f(v)=$ $f(x)+b_{0} \\ldots b_{n} b^{k-2}$ and $f(u)=f(x)+b_{0} \\ldots b_{n} b^{k-1}>(f(v)+f(x)) \/ 2$. This means that the point $(u, f(u))$ lies above the line joining $(x, f(x))$ and $(v, f(v))$. By induction, every $(x, f(x))$, where $x$ has a finite binary expansion, lies above the line joining $(0,0)$ and $(1 \/ 2, b)$ if $0x$. For the second inequality, observe that $$ \\begin{aligned} f(x)-x & =\\sum_{j=1}^{\\infty}\\left(b_{0} \\ldots b_{j-1}-2^{-j}\\right) a_{j} \\\\ & <\\sum_{j=1}^{\\infty}\\left(b^{j}-2^{-j}\\right) a_{j}<\\sum_{j=1}^{\\infty}\\left(b^{j}-2^{-j}\\right)=\\frac{b}{1-b}-1=c . \\end{aligned} $$ By continuity, these inequalities also hold for $x$ with infinite binary representations.","problem_type":null,"tier":0} +{"year":"1983","problem_phase":"shortlisted","problem":"12. (GBR 4) $)^{\\mathrm{IMO} 1}$ Find all functions $f$ defined on the positive real numbers and taking positive real values that satisfy the following conditions: (i) $f(x f(y))=y f(x)$ for all positive real $x, y$. (ii) $f(x) \\rightarrow 0$ as $x \\rightarrow+\\infty$.","solution":"12. Putting $y=x$ in (1) we see that there exist positive real numbers $z$ such that $f(z)=z$ (this is true for every $z=x f(x)$ ). Let $a$ be any of them. Then $f\\left(a^{2}\\right)=f(a f(a))=a f(a)=a^{2}$, and by induction, $f\\left(a^{n}\\right)=a^{n}$. If $a>1$, then $a^{n} \\rightarrow+\\infty$ as $n \\rightarrow \\infty$, and we have a contradiction with (2). Again, $a=f(a)=f(1 \\cdot a)=a f(1)$, so $f(1)=1$. Then, $a f\\left(a^{-1}\\right)=$ $f\\left(a^{-1} f(a)\\right)=f(1)=1$, and by induction, $f\\left(a^{-n}\\right)=a^{-n}$. This shows that $a \\nless 1$. Hence, $a=1$. It follows that $x f(x)=1$, i.e., $f(x)=1 \/ x$ for all $x$. This function satisfies (1) and (2), so $f(x)=1 \/ x$ is the unique solution.","problem_type":null,"tier":0} +{"year":"1983","problem_phase":"shortlisted","problem":"13. (LUX 2) Let $E$ be the set of $1983^{3}$ points of the space $\\mathbb{R}^{3}$ all three of whose coordinates are integers between 0 and 1982 (including 0 and 1982). A coloring of $E$ is a map from $E$ to the set $\\{$ red, blue $\\}$. How many colorings of $E$ are there satisfying the following property: The number of red vertices among the 8 vertices of any right-angled parallelepiped is a multiple of 4 ?","solution":"13. Given any coloring of the $3 \\times 1983-2$ points of the axes, we prove that there is a unique coloring of $E$ having the given property and extending this coloring. The first thing to notice is that given any rectangle $R_{1}$ parallel to a coordinate plane and whose edges are parallel to the axes, there is an even number $r_{1}$ of red vertices on $R_{1}$. Indeed, let $R_{2}$ and $R_{3}$ be two other rectangles that are translated from $R_{1}$ orthogonally to $R_{1}$ and let $r_{2}, r_{3}$ be the numbers of red vertices on $R_{2}$ and $R_{3}$ respectively. Then $r_{1}+r_{2}$, $r_{1}+r_{3}$, and $r_{2}+r_{3}$ are multiples of 4 , so $r_{1}=\\left(r_{1}+r_{2}+r_{1}+r_{3}-r_{2}-r_{3}\\right) \/ 2$ is even. Since any point of a coordinate plane is a vertex of a rectangle whose remaining three vertices lie on the corresponding axes, this determines uniquely the coloring of the coordinate planes. Similarly, the coloring of the inner points of the parallelepiped is completely determined. The solution is hence $2^{3 \\times 1983-2}=2^{5947}$.","problem_type":null,"tier":0} +{"year":"1983","problem_phase":"shortlisted","problem":"14. (POL 2) ${ }^{\\mathrm{IMO} 5}$ Prove or disprove: From the interval $[1, \\ldots, 30000]$ one can select a set of 1000 integers containing no arithmetic triple (three consecutive numbers of an arithmetic progression).","solution":"14. Let $T_{n}$ be the set of all nonnegative integers whose ternary representations consist of at most $n$ digits and do not contain a digit 2 . The cardinality of $T_{n}$ is $2^{n}$, and the greatest integer in $T_{n}$ is $11 \\ldots 1=3^{0}+3^{1}+\\cdots+3^{n-1}=$ $\\left(3^{n}-1\\right) \/ 2$. We claim that there is no arithmetic triple in $T_{n}$. To see this, suppose $x, y, z \\in T_{n}$ and $2 y=x+z$. Then $2 y$ has only 0 's and 2's in its ternary representation, and a number of this form can be the sum of two integers $x, z \\in T_{n}$ in only one way, namely $x=z=y$. But $\\left|T_{10}\\right|=2^{10}=1024$ and $\\max T_{10}=\\left(3^{10}-1\\right) \/ 2=29524<30000$. Thus the answer is yes.","problem_type":null,"tier":0} +{"year":"1983","problem_phase":"shortlisted","problem":"15. (POL 3) Decide whether there exists a set $M$ of natural numbers satisfying the following conditions: (i) For any natural number $m>1$ there are $a, b \\in M$ such that $a+b=m$. (ii) If $a, b, c, d \\in M, a, b, c, d>10$ and $a+b=c+d$, then $a=c$ or $a=d$.","solution":"15. There is no such set. Suppose $M$ satisfies $(a)$ and $(b)$ and let $q_{n}=$ $|\\{a \\in M: a \\leq n\\}|$. Consider the differences $b-a$, where $a, b \\in M$ and $10\\frac{\\sqrt{3}}{2}(\\sqrt{n}-1) \\min _{1 \\leq i\\sqrt{3} \/ 2 \\cdot(\\sqrt{n}-1) a $$ Proof of the lemma. If a nonobtuse triangle with sides $a \\geq b \\geq c$ has a circumscribed circle of radius $R$, we have $R=a \/(2 \\sin \\alpha) \\leq a \/ \\sqrt{3}$. Now we show that there exists a disk $D$ of radius $R$ containing $A=$ $\\left\\{P_{1}, \\ldots, P_{n}\\right\\}$ whose border $C$ is such that $C \\cap A$ is not included in an open semicircle, and hence contains either two diametrically opposite points and $R \\leq b \/ 2$, or an acute-angled triangle and $R \\leq b \/ \\sqrt{3}$. Among all disks whose borders pass through three points of $A$ and that contain all of $A$, let $D$ be the one of least radius. Suppose that $C \\cap A$ is contained in an arc of central angle less than $180^{\\circ}$, and that $P_{i}, P_{j}$ are its endpoints. Then there exists a circle through $P_{i}, P_{j}$ of smaller radius that contains $A$, a contradiction. Thus $D$ has the required property, and the assertion follows.","problem_type":null,"tier":0} +{"year":"1983","problem_phase":"shortlisted","problem":"18. (FRG 3) ${ }^{\\mathrm{IMO} 3}$ Let $a, b, c$ be positive integers satisfying $(a, b)=(b, c)=$ $(c, a)=1$. Show that $2 a b c-a b-b c-c a$ is the largest integer not representable as $$ x b c+y c a+z a b $$ with nonnegative integers $x, y, z$.","solution":"18. Let $\\left(x_{0}, y_{0}, z_{0}\\right)$ be one solution of $b c x+c a y+a b z=n$ (not necessarily nonnegative). By subtracting $b c x_{0}+c a y_{0}+a b z_{0}=n$ we get $$ b c\\left(x-x_{0}\\right)+c a\\left(y-y_{0}\\right)+a b\\left(z-z_{0}\\right)=0 . $$ Since $(a, b)=(a, c)=1$, we must have $a \\mid x-x_{0}$ or $x-x_{0}=a s$. Substituting this in the last equation gives $$ b c s+c\\left(y-y_{0}\\right)+b\\left(z-z_{0}\\right)=0 $$ Since $(b, c)=1$, we have $b \\mid y-y_{0}$ or $y-y_{0}=b t$. If we substitute this in the last equation we get $b c s+b c t+b\\left(z-z_{0}\\right)=0$, or $c s+c t+z-z_{0}=0$, or $z-z_{0}=-c(s+t)$. In $x=x_{0}+a s$ and $y=y_{0}+b t$, we can choose $s$ and $t$ such that $0 \\leq x \\leq a-1$ and $0 \\leq y \\leq b-1$. If $n>2 a b c-b c-c a-a b$, then $a b z=n-b c x-a c y>2 a b c-a b-b c-c a-b c(a-1)-c a(b-1)=-a b$ or $z>-1$, i.e., $z \\geq 0$. Hence, it is representable as $b c x+c a y+a b z$ with $x, y, z \\geq 0$. Now we prove that $2 a b c-b c-c a-a b$ is not representable as $b c x+c a y+a b z$ with $x, y, z \\geq 0$. Suppose that $b c x+c a y+a b z=2 a b c-a b-b c-c a$ with $x, y, z \\geq 0$. Then $$ b c(x+1)+c a(y+1)+a b(z+1)=2 a b c $$ with $x+1, y+1, z+1 \\geq 1$. Since $(a, b)=(a, c)=1$, we have $a \\mid x+1$ and thus $a \\leq x+1$. Similarly $b \\leq y+1$ and $c \\leq z+1$. Thus $b c a+c a b+a b c \\leq 2 a b c$, a contradiction.","problem_type":null,"tier":0} +{"year":"1983","problem_phase":"shortlisted","problem":"19. (ROM 1) Let $\\left(F_{n}\\right)_{n \\geq 1}$ be the Fibonacci sequence $F_{1}=F_{2}=1, F_{n+2}=$ $F_{n+1}+F_{n}(n \\geq 1)$, and $P(x)$ the polynomial of degree 990 satisfying $$ P(k)=F_{k}, \\quad \\text { for } k=992, \\ldots, 1982 $$ Prove that $P(1983)=F_{1983}-1$.","solution":"19. For all $n$, there exists a unique polynomial $P_{n}$ of degree $n$ such that $P_{n}(k)=F_{k}$ for $n+2 \\leq k \\leq 2 n+2$ and $P_{n}(2 n+3)=F_{2 n+3}-1$. For $n=0$, we have $F_{1}=F_{2}=1, F_{3}=2, P_{0}=1$. Now suppose that $P_{n-1}$ has been constructed and let $P_{n}$ be the polynomial of degree $n$ satisfying $P_{n}(X+2)-P_{n}(X+1)=P_{n-1}(X)$ and $P_{n}(n+2)=F_{n+2}$. (The mapping $\\mathbb{R}_{n}[X] \\rightarrow \\mathbb{R}_{n-1}[X] \\times \\mathbb{R}, P \\mapsto(Q, P(n+2)$ ), where $Q(X)=$ $P(X+2)-P(X+1)$, is bijective, since it is injective and those two spaces have the same dimension; clearly $\\operatorname{deg} Q=\\operatorname{deg} P-1$.) Thus for $n+2 \\leq k \\leq 2 n+2$ we have $P_{n}(k+1)=P_{n}(k)+F_{k-1}$ and $P_{n}(n+2)=F_{n+2}$; hence by induction on $k, P_{n}(k)=F_{k}$ for $n+2 \\leq k \\leq 2 n+2$ and $$ P_{n}(2 n+3)=F_{2 n+2}+P_{n-1}(2 n+1)=F_{2 n+3}-1 $$ Finally, $P_{990}$ is exactly the polynomial $P$ of the terms of the problem, for $P_{990}-P$ has degree less than or equal to 990 and vanishes at the 991 points $k=992, \\ldots, 1982$.","problem_type":null,"tier":0} +{"year":"1983","problem_phase":"shortlisted","problem":"2. (BEL 1) Let $n$ be a positive integer. Let $\\sigma(n)$ be the sum of the natural divisors $d$ of $n$ (including 1 and $n$ ). We say that an integer $m \\geq 1$ is superabundant (P.Erd\u00f6s, 1944) if $\\forall k \\in\\{1,2, \\ldots, m-1\\}, \\frac{\\sigma(m)}{m}>\\frac{\\sigma(k)}{k}$. Prove that there exists an infinity of superabundant numbers.","solution":"2. By definition, $\\sigma(n)=\\sum_{d \\mid n} d=\\sum_{d \\mid n} n \/ d=n \\sum_{d \\mid n} 1 \/ d$, hence $\\sigma(n) \/ n=$ $\\sum_{d \\mid n} 1 \/ d$. In particular, $\\sigma(n!) \/ n!=\\sum_{d \\mid n!} 1 \/ d \\geq \\sum_{k=1}^{n} 1 \/ k$. It follows that the sequence $\\sigma(n) \/ n$ is unbounded, and consequently there exist an infinite number of integers $n$ such that $\\sigma(n) \/ n$ is strictly greater than $\\sigma(k) \/ k$ for $k0$.","solution":"20. If $\\left(x_{1}, x_{2}, \\ldots, x_{n}\\right)$ satisfies the system with parameter $a$, then $\\left(-x_{1},-x_{2}\\right.$, $\\ldots,-x_{n}$ ) satisfies the system with parameter $-a$. Hence it is sufficient to consider only $a \\geq 0$. Let $\\left(x_{1}, \\ldots, x_{n}\\right)$ be a solution. Suppose $x_{1} \\leq a, x_{2} \\leq a, \\ldots, x_{n} \\leq a$. Summing the equations we get $$ \\left(x_{1}-a\\right)^{2}+\\cdots+\\left(x_{n}-a\\right)^{2}=0 $$ and see that $(a, a, \\ldots, a)$ is the only such solution. Now suppose that $x_{k} \\geq a$ for some $k$. According to the $k$ th equation, $$ x_{k+1}\\left|x_{k+1}\\right|=x_{k}^{2}-\\left(x_{k}-a\\right)^{2}=a\\left(2 x_{k}-a\\right) \\geq a^{2} $$ which implies that $x_{k+1} \\geq a$ as well (here $x_{n+1}=x_{1}$ ). Consequently, all $x_{1}, x_{2}, \\ldots, x_{n}$ are greater than or equal to $a$, and as above $(a, a, \\ldots, a)$ is the only solution.","problem_type":null,"tier":0} +{"year":"1983","problem_phase":"shortlisted","problem":"21. (SWE 1) Find the greatest integer less than or equal to $\\sum_{k=1}^{2^{1983}} k^{1 \/ 1983-1}$.","solution":"21. Using the identity $$ a^{n}-b^{n}=(a-b) \\sum_{m=0}^{n-1} a^{n-m-1} b^{m} $$ with $a=k^{1 \/ n}$ and $b=(k-1)^{1 \/ n}$ one obtains $$ 1<\\left(k^{1 \/ n}-(k-1)^{1 \/ n}\\right) n k^{1-1 \/ n} \\text { for all integers } n>1 \\text { and } k \\geq 1 $$ This gives us the inequality $k^{1 \/ n-1}1$ and $k \\geq 1$. In a similar way one proves that $n\\left((k+1)^{1 \/ n}-k^{1 \/ n}\\right)1$ and $k \\geq 1$. Hence for $n>1$ and $m>1$ it holds that $$ \\begin{aligned} n \\sum_{k=1}^{m}\\left((k+1)^{1 \/ n}-k^{1 \/ n}\\right) & <\\sum_{k=1}^{m} k^{1 \/ n-1} \\\\ & 1$ and $t>1$. For $1 \\leq k \\leq n$ put $k=v s+u$, where $0 \\leq v \\leq t-1$ and $1 \\leq u \\leq s$, and let $a_{k}=a_{v s+u}$ be the unique integer in the set $\\{1,2,3, \\ldots, n\\}$ such that $v s+u t-a_{v s+u}$ is a multiple of $n$. To prove that this construction gives a permutation, assume that $a_{k_{1}}=a_{k_{2}}$, where $k_{i}=v_{i} s+u_{i}, i=1,2$. Then $\\left(v_{1}-v_{2}\\right) s+\\left(u_{1}-u_{2}\\right) t$ is a multiple of $n=s t$. It follows that $t$ divides $\\left(v_{1}-v_{2}\\right)$, while $\\left|v_{1}-v_{2}\\right| \\leq t-1$, and that $s$ divides $\\left(u_{1}-u_{2}\\right)$, while $\\left|u_{1}-u_{2}\\right| \\leq s-1$. Hence, $v_{1}=v_{2}, u_{1}=u_{2}$, and $k_{1}=k_{2}$. We have proved that $a_{1}, \\ldots, a_{n}$ is a permutation of $\\{1,2, \\ldots, n\\}$ and hence $$ \\sum_{k=1}^{n} k \\cos \\frac{2 \\pi a_{k}}{n}=\\sum_{v=0}^{t-1}\\left(\\sum_{u=1}^{s}(v s+u) \\cos \\left(\\frac{2 \\pi v}{t}+\\frac{2 \\pi u}{s}\\right)\\right) $$ Using $\\sum_{u=1}^{s} \\cos (2 \\pi u \/ s)=\\sum_{u=1}^{s} \\sin (2 \\pi u \/ s)=0$ and the additive formulas for cosine, one finds that $$ \\begin{aligned} \\sum_{k=1}^{n} k \\cos \\frac{2 \\pi a_{k}}{n}= & \\sum_{v=0}^{t-1}\\left(\\cos \\frac{2 \\pi v}{t} \\sum_{u=1}^{s} u \\cos \\frac{2 \\pi u}{s}-\\sin \\frac{2 \\pi v}{t} \\sum_{u=1}^{s} u \\sin \\frac{2 \\pi u}{s}\\right) \\\\ = & \\left(\\sum_{u=1}^{s} u \\cos \\frac{2 \\pi u}{s}\\right)\\left(\\sum_{v=0}^{t-1} \\cos \\frac{2 \\pi v}{t}\\right) \\\\ & -\\left(\\sum_{u=1}^{s} u \\sin \\frac{2 \\pi u}{s}\\right)\\left(\\sum_{v=0}^{t-1} \\sin \\frac{2 \\pi v}{t}\\right)=0 \\end{aligned} $$","problem_type":null,"tier":0} +{"year":"1983","problem_phase":"shortlisted","problem":"23. (USS 1) ${ }^{\\mathrm{IMO} 2}$ Let $K$ be one of the two intersection points of the circles $W_{1}$ and $W_{2}$. Let $O_{1}$ and $O_{2}$ be the centers of $W_{1}$ and $W_{2}$. The two common tangents to the circles meet $W_{1}$ and $W_{2}$ respectively in $P_{1}$ and $P_{2}$, the first tangent, and $Q_{1}$ and $Q_{2}$, the second tangent. Let $M_{1}$ and $M_{2}$ be the midpoints of $P_{1} Q_{1}$ and $P_{2} Q_{2}$, respectively. Prove that $$ \\angle O_{1} K O_{2}=\\angle M_{1} K M_{2} $$","solution":"23. We note that $\\angle O_{1} K O_{2}=\\angle M_{1} K M_{2}$ is equivalent to $\\angle O_{1} K M_{1}=$ $\\angle O_{2} K M_{2}$. Let $S$ be the intersection point of the common tangents, and let $L$ be the second point of intersection of $S K$ and $W_{1}$. Since $\\triangle S O_{1} P_{1} \\sim \\triangle S P_{1} M_{1}$, we have $S K$. $S L=S P_{1}^{2}=S O_{1} \\cdot S M_{1}$ which implies that points $O_{1}, L, K, M_{1}$ lie on a circle. Hence $\\angle O_{1} K M_{1}=$ $\\angle O_{1} L M_{1}=\\angle O_{2} K M_{2}$. ![](https:\/\/cdn.mathpix.com\/cropped\/2024_11_18_8e985d6b9c83aa3e9d0eg-474.jpg?height=295&width=551&top_left_y=959&top_left_x=809)","problem_type":null,"tier":0} +{"year":"1983","problem_phase":"shortlisted","problem":"24. (USS 2) Let $d_{n}$ be the last nonzero digit of the decimal representation of $n$ !. Prove that $d_{n}$ is aperiodic; that is, there do not exist $T$ and $n_{0}$ such that for all $n \\geq n_{0}, d_{n+T}=d_{n}$.","solution":"24. See the solution of (SL91-15).","problem_type":null,"tier":0} +{"year":"1983","problem_phase":"shortlisted","problem":"25. (USS 3) Prove that every partition of 3-dimensional space into three disjoint subsets has the following property: One of these subsets contains all possible distances; i.e., for every $a \\in \\mathbb{R}_{+}$, there are points $M$ and $N$ inside that subset such that distance between $M$ and $N$ is exactly $a$.","solution":"25. Suppose the contrary, that $\\mathbb{R}^{3}=P_{1} \\cup P_{2} \\cup P_{3}$ is a partition such that $a_{1} \\in \\mathbb{R}^{+}$is not realized by $P_{1}, a_{2} \\in \\mathbb{R}^{+}$is not realized by $P_{2}$ and $a_{3} \\in \\mathbb{R}^{+}$ not realized by $P_{3}$, where w.l.o.g. $a_{1} \\geq a_{2} \\geq a_{3}$. If $P_{1}=\\emptyset=P_{2}$, then $P_{3}=\\mathbb{R}^{3}$, which is impossible. If $P_{1}=\\emptyset$, and $X \\in P_{2}$, the sphere centered at $X$ with radius $a_{2}$ is included in $P_{3}$ and $a_{3} \\leq a_{2}$ is realized, which is impossible. If $P_{1} \\neq \\emptyset$, let $X_{1} \\in P_{1}$. The sphere $S$ centered in $X_{1}$, of radius $a_{1}$ is included in $P_{2} \\cap P_{3}$. Since $a_{1} \\geq a_{3}, S \\not \\subset P_{3}$. Let $X_{2} \\in P_{2} \\cap S$. The circle $\\left\\{Y \\in S \\mid d\\left(X_{2}, Y\\right)=a_{2}\\right\\}$ is included in $P_{3}$, but $a_{2} \\leq a_{1}$; hence it has radius $r=a_{2} \\sqrt{1-a_{2}^{2} \/\\left(4 a_{1}^{2}\\right)} \\geq a_{2} \\sqrt{3} \/ 2$ and $a_{3} \\leq a_{2} \\leq a_{2} \\sqrt{3}<2 r$; hence $a_{3}$ is realized by $P_{3}$.","problem_type":null,"tier":0} +{"year":"1983","problem_phase":"shortlisted","problem":"3. (BEL 3) $)^{\\mathrm{IMO} 4}$ We say that a set $E$ of points of the Euclidian plane is \"Pythagorean\" if for any partition of $E$ into two sets $A$ and $B$, at least one of the sets contains the vertices of a right-angled triangle. Decide whether the following sets are Pythagorean: (a) a circle; (b) an equilateral triangle (that is, the set of three vertices and the points of the three edges).","solution":"3. (a) A circle is not Pythagorean. Indeed, consider the partition into two semicircles each closed at one and open at the other end. (b) An equilateral triangle, call it $P Q R$, is Pythagorean. Let $P^{\\prime}, Q^{\\prime}$, and $R^{\\prime}$ be the points on $Q R, R P$, and $P Q$ such that $P R^{\\prime}: R^{\\prime} Q=Q P^{\\prime}$ : $P^{\\prime} R=R Q^{\\prime}: Q^{\\prime} P=1: 2$. Then $Q^{\\prime} R^{\\prime} \\perp P Q$, etc. Suppose that $P Q R$ is not Pythagorean, and consider a partition into $A, B$, neither of which contains the vertices of a right-angled triangle. At least two of $P^{\\prime}, Q^{\\prime}$, and $R^{\\prime}$ belong to the same class, say $P^{\\prime}, Q^{\\prime} \\in A$. Then $[P R] \\backslash\\left\\{Q^{\\prime}\\right\\} \\subset B$ and hence $R^{\\prime} \\in A$ (otherwise, if $R^{\\prime \\prime}$ is the foot of the perpendicular from $R^{\\prime}$ to $P R, \\triangle R R^{\\prime} R^{\\prime \\prime}$ is right-angled with all vertices in $B$ ). But this implies again that $[P Q] \\backslash\\left\\{R^{\\prime}\\right\\} \\subset B$, and thus $B$ contains vertices of a rectangular triangle. This is a contradiction.","problem_type":null,"tier":0} +{"year":"1983","problem_phase":"shortlisted","problem":"4. (BEL 5) On the sides of the triangle $A B C$, three similar isosceles triangles $A B P(A P=P B), A Q C(A Q=Q C)$, and $B R C(B R=R C)$ are constructed. The first two are constructed externally to the triangle $A B C$, but the third is placed in the same half-plane determined by the line $B C$ as the triangle $A B C$. Prove that $A P R Q$ is a parallelogram.","solution":"4. The rotational homothety centered at $C$ that sends $B$ to $R$ also sends $A$ to $Q$; hence the triangles $A B C$ and $Q R C$ are similar. For the same reason, $\\triangle A B C$ and $\\triangle P B R$ are similar. Moreover, $B R=C R$; hence $\\triangle C R Q \\cong$ $\\triangle R B P$. Thus $P R=Q C=A Q$ and $Q R=P B=P A$, so $A P Q R$ is a parallelogram.","problem_type":null,"tier":0} +{"year":"1983","problem_phase":"shortlisted","problem":"5. (BRA 1) Consider the set of all strictly decreasing sequences of $n$ natural numbers having the property that in each sequence no term divides any other term of the sequence. Let $A=\\left(a_{j}\\right)$ and $B=\\left(b_{j}\\right)$ be any two such sequences. We say that $A$ precedes $B$ if for some $k, a_{k}2(n-k)+1$. So we cannot begin with $x_{k+1}$ for any $k>0$. Now assume that there is an equality for some $k$. There are two cases: (i) Suppose $x_{1}+x_{2}+\\cdots+x_{i} \\leq 2 i(i=1, \\ldots, k)$ and $x_{1}+\\cdots+x_{k}=2 k+1$, $x_{1}+\\cdots+x_{k+l} \\leq 2(k+l)+1(1 \\leq l \\leq n-1-k)$. For $i \\leq k-1$ we have $x_{i+1}+\\cdots+x_{n}=2(n+1)-\\left(x_{1}+\\cdots+x_{i}\\right)>2(n-i)+1$, so we cannot take $r=i$. If there is a $j \\geq 1$ such that $x_{1}+x_{2}+\\cdots+x_{k+j} \\leq 2(k+j)$, then also $x_{k+j+1}+\\cdots+x_{n}>2(n-k-j)+1$. If $(\\forall j \\geq 1) x_{1}+\\cdots+x_{k+j}=$ $2(k+j)+1$, then $x_{n}=3$ and $x_{k+1}=\\cdots=x_{n-1}=2$. In this case we directly verify that we cannot take $r=k+j$. However, we can also take $r=k$ : for $k+l \\leq n-1, x_{k+1}+\\cdots+x_{k+l} \\leq 2(k+l)+1-(2 k+1)=2 l$, also $x_{k+1}+\\cdots+x_{n}=2(n-k)+1$, and moreover $x_{1} \\leq 2, x_{1}+x_{2} \\leq 4, \\ldots$. (ii) Suppose $x_{1}+\\cdots+x_{i} \\leq 2 i(1 \\leq i \\leq n-2)$ and $x_{1}+\\cdots+x_{n-1}=2 n-1$. Then we can obviously take $r=n-1$. On the other hand, for any $1 \\leq i \\leq n-2, x_{i+1}+\\cdots+x_{n-1}+x_{n}=\\left(x_{1}+\\cdots+x_{n-1}\\right)-\\left(x_{1}+\\cdots+\\right.$ $\\left.x_{i}\\right)+3>2(n-i)+1$, so we cannot take another $r \\neq 0$.","problem_type":null,"tier":0} +{"year":"1983","problem_phase":"shortlisted","problem":"7. (CAN 5) Let $a$ be a positive integer and let $\\left\\{a_{n}\\right\\}$ be defined by $a_{0}=0$ and $$ a_{n+1}=\\left(a_{n}+1\\right) a+(a+1) a_{n}+2 \\sqrt{a(a+1) a_{n}\\left(a_{n}+1\\right)} \\quad(n=1,2 \\ldots) $$ Show that for each positive integer $n, a_{n}$ is a positive integer.","solution":"7. Clearly, each $a_{n}$ is positive and $\\sqrt{a_{n+1}}=\\sqrt{a_{n}} \\sqrt{a+1}+\\sqrt{a_{n}+1} \\sqrt{a}$. Notice that $\\sqrt{a_{n+1}+1}=\\sqrt{a+1} \\sqrt{a_{n}+1}+\\sqrt{a} \\sqrt{a_{n}}$. Therefore $$ \\begin{aligned} & (\\sqrt{a+1}-\\sqrt{a})\\left(\\sqrt{a_{n}+1}-\\sqrt{a_{n}}\\right) \\\\ & \\quad=\\left(\\sqrt{a+1} \\sqrt{a_{n}+1}+\\sqrt{a} \\sqrt{a_{n}}\\right)-\\left(\\sqrt{a_{n}} \\sqrt{a+1}+\\sqrt{a_{n}+1} \\sqrt{a}\\right) \\\\ & \\quad=\\sqrt{a_{n+1}+1}-\\sqrt{a_{n+1}} \\end{aligned} $$ By induction, $\\sqrt{a_{n+1}}-\\sqrt{a_{n}}=(\\sqrt{a+1}-\\sqrt{a})^{n}$. Similarly, $\\sqrt{a_{n+1}}+\\sqrt{a_{n}}=$ $(\\sqrt{a+1}+\\sqrt{a})^{n}$. Hence, $$ \\sqrt{a_{n}}=\\frac{1}{2}\\left[(\\sqrt{a+1}+\\sqrt{a})^{n}-(\\sqrt{a+1}-\\sqrt{a})^{n}\\right] $$ from which the result follows.","problem_type":null,"tier":0} +{"year":"1983","problem_phase":"shortlisted","problem":"8. (SPA 2) In a test, $3 n$ students participate, who are located in three rows of $n$ students in each. The students leave the test room one by one. If $N_{1}(t), N_{2}(t), N_{3}(t)$ denote the numbers of students in the first, second, and third row respectively at time $t$, find the probability that for each $t$ during the test, $$ \\left|N_{i}(t)-N_{j}(t)\\right|<2, \\quad i \\neq j, \\quad i, j=1,2, \\ldots $$","solution":"8. Situations in which the condition of the statement is fulfilled are the following: $S_{1}: N_{1}(t)=N_{2}(t)=N_{3}(t)$ $S_{2}: N_{i}(t)=N_{j}(t)=h, N_{k}(t)=h+1$, where $(i, j, k)$ is a permutation of the set $\\{1,2,3\\}$. In this case the first student to leave must be from row $k$. This leads to the situation $S_{1}$. $S_{3}: N_{i}(t)=h, N_{j}(t)=N_{k}(t)=h+1,((i, j, k)$ is a permutation of the set $\\{1,2,3\\})$. In this situation the first student leaving the room belongs to row $j$ (or $k$ ) and the second to row $k$ (or $j$ ). After this we arrive at the situation $S_{1}$. Hence, the initial situation is $S_{1}$ and after each triple of students leaving the room the situation $S_{1}$ must recur. We shall compute the probability $P_{h}$ that from a situation $S_{1}$ with $3 h$ students in the room $(h \\leq n)$ one arrives at a situation $S_{1}$ with $3(h-1)$ students in the room: $$ P_{h}=\\frac{(3 h) \\cdot(2 h) \\cdot h}{(3 h) \\cdot(3 h-1) \\cdot(3 h-2)}=\\frac{3!h^{3}}{3 h(3 h-1)(3 h-2)} $$ Since the room becomes empty after the repetition of $n$ such processes, which are independent, we obtain for the probability sought $$ P=\\prod_{h=1}^{n} P_{h}=\\frac{(3!)^{n}(n!)^{3}}{(3 n)!} $$","problem_type":null,"tier":0} +{"year":"1983","problem_phase":"shortlisted","problem":"9. (USA 3) ${ }^{\\mathrm{IMO} 6}$ If $a, b$, and $c$ are sides of a triangle, prove that $$ a^{2} b(a-b)+b^{2} c(b-c)+c^{2} a(c-a) \\geq 0 $$ Determine when there is equality.","solution":"9. For any triangle of sides $a, b, c$ there exist 3 nonnegative numbers $x, y, z$ such that $a=y+z, b=z+x, c=x+y$ (these numbers correspond to the division of the sides of a triangle by the point of contact of the incircle). The inequality becomes $$ (y+z)^{2}(z+x)(y-x)+(z+x)^{2}(x+y)(z-y)+(x+y)^{2}(y+z)(x-z) \\geq 0 $$ Expanding, we get $x y^{3}+y z^{3}+z x^{3} \\geq x y z(x+y+z)$. This follows from Cauchy's inequality $\\left(x y^{3}+y z^{3}+z x^{3}\\right)(z+x+y) \\geq(\\sqrt{x y z}(x+y+z))^{2}$ with equality if and only if $x y^{3} \/ z=y z^{3} \/ x=z x^{3} \/ y$, or equivalently $x=y=z$, i.e., $a=b=c$.","problem_type":null,"tier":0} +{"year":"1984","problem_phase":"shortlisted","problem":"1. (FRA 1) Find all solutions of the following system of $n$ equations in $n$ variables: $$ \\begin{aligned} & x_{1}\\left|x_{1}\\right|-\\left(x_{1}-a\\right)\\left|x_{1}-a\\right|=x_{2}\\left|x_{2}\\right| \\\\ & x_{2}\\left|x_{2}\\right|-\\left(x_{2}-a\\right)\\left|x_{2}-a\\right|=x_{3}\\left|x_{3}\\right| \\\\ & \\cdots \\\\ & x_{n}\\left|x_{n}\\right|-\\left(x_{n}-a\\right)\\left|x_{n}-a\\right|=x_{1}\\left|x_{1}\\right| \\end{aligned} $$ where $a$ is a given number.","solution":"1. This is the same problem as (SL83-20).","problem_type":null,"tier":0} +{"year":"1984","problem_phase":"shortlisted","problem":"10. (GBR 1) Prove that the product of five consecutive positive integers cannot be the square of an integer.","solution":"10. Suppose that the product of some five consecutive numbers is a square. It is easily seen that among them at least one, say $n$, is divisible neither by 2 nor 3 . Since $n$ is coprime to the remaining four numbers, it is itself a square of a number $m$ of the form $6 k \\pm 1$. Thus $n=(6 k \\pm 1)^{2}=24 r+1$, where $r=k(3 k \\pm 1) \/ 2$. Note that neither of the numbers $24 r-1,24 r+5$ is one of our five consecutive numbers because it is not a square. Hence the five numbers must be $24 r, 24 r+1, \\ldots, 24 r+4$. However, the number $24 r+4=(6 k \\pm 1)^{2}+3$ is divisible by $6 r+1$, which implies that it is a square as well. It follows that these two squares are 1 and 4 , which is impossible.","problem_type":null,"tier":0} +{"year":"1984","problem_phase":"shortlisted","problem":"11. (CAN 1) Let $n$ be a natural number and $a_{1}, a_{2}, \\ldots, a_{2 n}$ mutually distinct integers. Find all integers $x$ satisfying $$ \\left(x-a_{1}\\right) \\cdot\\left(x-a_{2}\\right) \\cdots\\left(x-a_{2 n}\\right)=(-1)^{n}(n!)^{2} $$","solution":"11. Suppose that an integer $x$ satisfies the equation. Then the numbers $x-$ $a_{1}, x-a_{2}, \\ldots, x-a_{2 n}$ are $2 n$ distinct integers whose product is $1 \\cdot(-1)$. $2 \\cdot(-2) \\cdots n \\cdot(-n)$. From here it is obvious that the numbers $x-a_{1}, x-a_{2}, \\ldots, x-a_{2 n}$ are some reordering of the numbers $-n,-n+1, \\ldots,-1,1, \\ldots, n-1, n$. It follows that their sum is 0 , and therefore $x=\\left(a_{1}+a_{2}+\\cdots+a_{2 n}\\right) \/ 2 n$. This is the only solution if $\\left\\{a_{1}, a_{2}, \\ldots, a_{2 n}\\right\\}=\\{x-n, \\ldots, x-1, x+1, \\ldots, x+n\\}$ for some $x \\in \\mathbb{N}$. Otherwise there is no solution.","problem_type":null,"tier":0} +{"year":"1984","problem_phase":"shortlisted","problem":"12. (NET 1) ${ }^{\\mathrm{IMO} 2}$ Find two positive integers $a, b$ such that none of the numbers $a, b, a+b$ is divisible by 7 and $(a+b)^{7}-a^{7}-b^{7}$ is divisible by $7^{7}$.","solution":"12. By the binomial formula we have $$ \\begin{aligned} (a+b)^{7}-a^{7}-b^{7} & =7 a b\\left[\\left(a^{5}+b^{5}\\right)+3 a b\\left(a^{3}+b^{3}\\right)+5 a^{2} b^{2}(a+b)\\right] \\\\ & =7 a b(a+b)\\left(a^{2}+a b+b^{2}\\right)^{2} \\end{aligned} $$ Thus it will be enough to find $a$ and $b$ such that $7 \\nmid a, b$ and $7^{3} \\mid a^{2}+a b+b^{2}$. Such numbers must satisfy $(a+b)^{2}>a^{2}+a b+b^{2} \\geq 7^{3}=343$, implying $a+b \\geq 19$. Trying $a=1$ we easily find the example $(a, b)=(1,18)$.","problem_type":null,"tier":0} +{"year":"1984","problem_phase":"shortlisted","problem":"13. (BUL 5) Prove that the volume of a tetrahedron inscribed in a right circular cylinder of volume 1 does not exceed $\\frac{2}{3 \\pi}$.","solution":"13. Let $Z$ be the given cylinder of radius $r$, altitude $h$, and volume $\\pi r^{2} h=1, k_{1}$ and $k_{2}$ the circles surrounding its bases, and $V$ the volume of an inscribed tetrahedron $A B C D$. We claim that there is no loss of generality in assuming that $A, B, C, D$ all lie on $k_{1} \\cup k_{2}$. Indeed, if the vertices $A, B, C$ are fixed and $D$ moves along a segment $E F$ parallel to the axis of the cylinder $\\left(E \\in k_{1}, F \\in k_{2}\\right)$, the maximum distance of $D$ from the plane $A B C$ (and consequently the maximum value of $V$ ) is achieved either at $E$ or at $F$. Hence we shall consider only the following two cases: (i) $A, B \\in k_{1}$ and $C, D \\in k_{2}$. Let $P, Q$ be the projections of $A, B$ on the plane of $k_{2}$, and $R, S$ the projections of $C, D$ on the plane of $k_{1}$, respectively. Then $V$ is one-third of the volume $V^{\\prime}$ of the prism $A R B S C P D Q$ with bases $A R B S$ and $C P D Q$. The area of the quadrilateral $A R B S$ inscribed in $k_{1}$ does not exceed the area of the square inscribed therein, which is $2 r^{2}$. Hence $3 V=V^{\\prime} \\leq 2 r^{2} h=2 \/ \\pi$. (ii) $A, B, C \\in k_{1}$ and $D \\in k_{2}$. The area of the triangle $A B C$ does not exceed the area of an equilateral triangle inscribed in $k_{1}$, which is $3 \\sqrt{3} r^{2} \/ 4$. Consequently, $V \\leq \\frac{\\sqrt{3}}{4} r^{2} h=\\frac{\\sqrt{3}}{4 \\pi}<\\frac{2}{3 \\pi}$.","problem_type":null,"tier":0} +{"year":"1984","problem_phase":"shortlisted","problem":"14. (ROM 5) ${ }^{\\mathrm{IMO} 4}$ Let $A B C D$ be a convex quadrilateral for which the circle with diameter $A B$ is tangent to the line $C D$. Show that the circle with diameter $C D$ is tangent to the line $A B$ if and only if the lines $B C$ and $A D$ are parallel.","solution":"14. Let $M$ and $N$ be the midpoints of $A B$ and $C D$, and let $M^{\\prime}, N^{\\prime}$ be their projections on $C D$ and $A B$, respectively. We know that $M M^{\\prime}=A B \/$, and hence $$ S_{A B C D}=S_{A M D}+S_{B M C}+S_{C M D}=\\frac{1}{2}\\left(S_{A B D}+S_{A B C}\\right)+\\frac{1}{4} A B \\cdot C D $$ The line $A B$ is tangent to the circle with diameter $C D$ if and only if $N N^{\\prime}=C D \/ 2$, or equivalently, $$ S_{A B C D}=S_{A N D}+S_{B N C}+S_{A N B}=\\frac{1}{2}\\left(S_{B C D}+S_{A C D}\\right)+\\frac{1}{4} A B \\cdot C D $$ By (1), this is further equivalent to $S_{A B C}+S_{A B D}=S_{B C D}+S_{A C D}$. But since $S_{A B C}+S_{A C D}=S_{A B D}+S_{B C D}=S_{A B C D}$, this reduces to $S_{A B C}=S_{B C D}$, i.e., to $B C \\| A D$.","problem_type":null,"tier":0} +{"year":"1984","problem_phase":"shortlisted","problem":"15. (LUX 2) Angles of a given triangle $A B C$ are all smaller than $120^{\\circ}$. Equilateral triangles $A F B, B D C$ and $C E A$ are constructed in the exterior of $\\triangle A B C$. (a) Prove that the lines $A D, B E$, and $C F$ pass through one point $S$. (b) Prove that $S D+S E+S F=2(S A+S B+S C)$.","solution":"15. (a) Since rotation by $60^{\\circ}$ around $A$ transforms the triangle $C A F$ into $\\triangle E A B$, it follows that $\\measuredangle(C F, E B)=60^{\\circ}$. We similarly deduce that $\\measuredangle(E B, A D)=\\measuredangle(A D, F C)=60^{\\circ}$. Let $S$ be the intersection point of $B E$ and $A D$. Since $\\measuredangle C S E=\\measuredangle C A E=60^{\\circ}$, it follows that $E A S C$ is cyclic. Therefore $\\measuredangle(A S, S C)=60^{\\circ}=\\measuredangle(A D, F C)$, which implies that $S$ lies on $C F$ as well. (b) A rotation of $E A S C$ around $E$ by $60^{\\circ}$ transforms $A$ into $C$ and $S$ into a point $T$ for which $S E=S T=S C+C T=S C+S A$. Summing the equality $S E=S C+S A$ and the analogous equalities $S D=S B+S C$ and $S F=S A+S B$ yields the result.","problem_type":null,"tier":0} +{"year":"1984","problem_phase":"shortlisted","problem":"16. (POL 1) ${ }^{\\mathrm{IMO} 6}$ Let $a, b, c, d$ be odd positive integers such that $ab+c$ (indeed, $(d+a)^{2}-(d-a)^{2}=(c+b)^{2}-(c-b)^{2}=4 a d=4 b c$ and $d-a>c-b>0)$. Thus $k>m$. From $d=2^{k}-a$ and $c=2^{m}-b$ we get $a\\left(2^{k}-a\\right)=b\\left(2^{m}-b\\right)$, or equivalently, $$ (b+a)(b-a)=2^{m}\\left(b-2^{k-m} a\\right) $$ Since $2^{k-m} a$ is even and $b$ is odd, the highest power of 2 that divides the right-hand side of $(1)$ is $m$. Hence $(b+a)(b-a)$ is divisible by $2^{m}$ but not by $2^{m+1}$, which implies $b+a=2^{m_{1}} p$ and $b-a=2^{m_{2}} q$, where $m_{1}, m_{2} \\geq 1$, $m_{1}+m_{2}=m$, and $p, q$ are odd. Furthermore, $b=\\left(2^{m_{1}} p+2^{m_{2}} q\\right) \/ 2$ and $a=\\left(2^{m_{1}} p-2^{m_{2}} q\\right) \/ 2$ are odd, so either $m_{1}=1$ or $m_{2}=1$. Note that $m_{1}=1$ is not possible, since it would imply that $b-a=2^{m-1} q \\geq 2^{m-1}$, although $b+c=2^{m}$ and $bm$, it follows that $a=1$. Remark. Now it is not difficult to prove that all fourtuplets $(a, b, c, d)$ that satisfy the given conditions are of the form $\\left(1,2^{m-1}-1,2^{m-1}+1,2^{2 m-2}-\\right.$ 1 ), where $m \\in \\mathbb{N}, m \\geq 3$.","problem_type":null,"tier":0} +{"year":"1984","problem_phase":"shortlisted","problem":"17. (FRG 3) In a permutation $\\left(x_{1}, x_{2}, \\ldots, x_{n}\\right)$ of the set $1,2, \\ldots, n$ we call a pair $\\left(x_{i}, x_{j}\\right)$ discordant if $ix_{j}$. Let $d(n, k)$ be the number of such permutations with exactly $k$ discordant pairs. Find $d(n, 2)$ and $d(n, 3)$.","solution":"17. For any $m=0,1, \\ldots, n-1$, we shall find the number of permutations $\\left(x_{1}, x_{2}, \\ldots, x_{n}\\right)$ with exactly $k$ discordant pairs such that $x_{n}=n-m$. This $x_{n}$ is a member of exactly $m$ discordant pairs, and hence the permutation $\\left(x_{1}, \\ldots, x_{n-1}\\right.$ of the set $\\{1,2, \\ldots, n\\} \\backslash\\{m\\}$ must have exactly $k-m$ discordant pairs: there are $d(n-1, k-m)$ such permutations. Therefore $$ \\begin{aligned} d(n, k) & =d(n-1, k)+d(n-1, k-1) \\cdots+d(n-1, k-n+1) \\\\ & =d(n-1, k)+d(n, k-1) \\end{aligned} $$ (note that $d(n, k)$ is 0 if $k<0$ or $k>\\binom{n}{2}$ ). We now proceed to calculate $d(n, 2)$ and $d(n, 3)$. Trivially, $d(n, 0)=1$. It follows that $d(n, 1)=d(n-1,1)+d(n, 0)=d(n-1,1)+1$, which yields $d(n, 1)=d(1,1)+n-1=n-1$. Further, $d(n, 2)=d(n-1,2)+d(n, 1)=d(n-1,2)+n-1=d(2,2)+$ $2+3+\\cdots+n-1=\\left(n^{2}-n-2\\right) \/ 2$. Finally, using the known formula $1^{2}+2^{2}+\\cdots+k^{2}=k(k+1)(2 k+1) \/ 6$, we have $d(n, 3)=d(n-1,3)+d(n, 2)=d(n-1,3)+\\left(n^{2}-n-2\\right) \/ 2=$ $d(2,3)+\\sum_{i=3}^{n}\\left(n^{2}-n-2\\right) \/ 2=\\left(n^{3}-7 n+6\\right) \/ 6$.","problem_type":null,"tier":0} +{"year":"1984","problem_phase":"shortlisted","problem":"18. (USA 5) Inside triangle $A B C$ there are three circles $k_{1}, k_{2}, k_{3}$ each of which is tangent to two sides of the triangle and to its incircle $k$. The radii of $k_{1}, k_{2}, k_{3}$ are 1, 4, and 9 . Determine the radius of $k$.","solution":"18. Suppose that circles $k_{1}\\left(O_{1}, r_{1}\\right), k_{2}\\left(O_{2}, r_{2}\\right)$, and $k_{3}\\left(O_{3}, r_{3}\\right)$ touch the edges of the angles $\\angle B A C, \\angle A B C$, and $\\angle A C B$, respectively. Denote also by $O$ and $r$ the center and radius of the incircle. Let $P$ be the point of tangency of the incircle with $A B$ and let $F$ be the foot of the perpendicular from $O_{1}$ to $O P$. From $\\triangle O_{1} F O$ we obtain $\\cot (\\alpha \/ 2)=2 \\sqrt{r r_{1}} \/\\left(r-r_{1}\\right)$ and analogously $\\cot (\\beta \/ 2)=2 \\sqrt{r r_{2}} \/\\left(r-r_{2}\\right), \\cot (\\gamma \/ 2)=2 \\sqrt{r r_{3}} \/\\left(r-r_{3}\\right)$. We will now use a well-known trigonometric identity for the angles of a triangle: $$ \\cot \\frac{\\alpha}{2}+\\cot \\frac{\\beta}{2}+\\cot \\frac{\\gamma}{2}=\\cot \\frac{\\alpha}{2} \\cdot \\cot \\frac{\\beta}{2} \\cdot \\cot \\frac{\\gamma}{2} . $$ (This identity follows from $\\tan (\\gamma \/ 2)=\\cot (\\alpha \/ 2+\\beta \/ 2)$ and the formula for the cotangent of a sum.) Plugging in the obtained cotangents, we get $$ \\begin{aligned} \\frac{2 \\sqrt{r r_{1}}}{r-r_{1}}+\\frac{2 \\sqrt{r r_{2}}}{r-r_{2}}+\\frac{2 \\sqrt{r r_{3}}}{r-r_{3}}= & \\frac{2 \\sqrt{r r_{1}}}{r-r_{1}} \\cdot \\frac{2 \\sqrt{r r_{2}}}{r-r_{2}} \\cdot \\frac{2 \\sqrt{r r_{3}}}{r-r_{3}} \\Rightarrow \\\\ & \\sqrt{r_{1}}\\left(r-r_{2}\\right)\\left(r-r_{3}\\right)+\\sqrt{r_{2}}\\left(r-r_{1}\\right)\\left(r-r_{3}\\right) \\\\ & +\\sqrt{r_{3}}\\left(r-r_{1}\\right)\\left(r-r_{2}\\right)=4 r \\sqrt{r_{1} r_{2} r_{3}} . \\end{aligned} $$ For $r_{1}=1, r_{2}=4$, and $r_{3}=9$ we get $(r-4)(r-9)+2(r-1)(r-9)+3(r-1)(r-4)=24 r \\Rightarrow 6(r-1)(r-11)=0$. Clearly, $r=11$ is the only viable value for $r$.","problem_type":null,"tier":0} +{"year":"1984","problem_phase":"shortlisted","problem":"19. (CAN 5) The triangular array $\\left(a_{n, k}\\right)$ of numbers is given by $a_{n, 1}=1 \/ n$, for $n=1,2, \\ldots, a_{n, k+1}=a_{n-1, k}-a_{n, k}$, for $1 \\leq k \\leq n-1$. Find the harmonic mean of the 1985th row.","solution":"19. First, we shall prove that the numbers in the $n$th row are exactly the numbers $$ \\frac{1}{n\\binom{n-1}{0}}, \\frac{1}{n\\binom{n-1}{1}}, \\frac{1}{n\\binom{n-1}{2}}, \\ldots, \\frac{1}{n\\binom{n-1}{n-1}} $$ The proof of this fact can be done by induction. For small $n$, the statement can be easily verified. Assuming that the statement is true for some $n$, we have that the $k$ th element in the $(n+1)$ st row is, as is directly verified, $$ \\frac{1}{n\\binom{n-1}{k-1}}-\\frac{1}{(n+1)\\binom{n}{k-1}}=\\frac{1}{(n+1)\\binom{n}{k}} $$ Thus (1) is proved. Now the geometric mean of the elements of the $n$th row becomes: $$ \\frac{1}{n \\sqrt[n]{\\binom{n-1}{0} \\cdot\\binom{n-1}{1} \\cdots\\binom{n-1}{n-1}}} \\geq \\frac{1}{n\\left(\\frac{\\binom{n-1}{0}+\\binom{n-1}{1}+\\cdots+\\binom{n-1}{n-1}}{n}\\right)}=\\frac{1}{2^{n-1}} $$ The desired result follows directly from substituting $n=1984$.","problem_type":null,"tier":0} +{"year":"1984","problem_phase":"shortlisted","problem":"2. (CAN 2) Prove: (a) There are infinitely many triples of positive integers $m, n, p$ such that $4 m n-m-n=p^{2}-1$. (b) There are no positive integers $m, n, p$ such that $4 m n-m-n=p^{2}$.","solution":"2. (a) For $m=t(t-1) \/ 2$ and $n=t(t+1) \/ 2$ we have $4 m n-m-n=$ $\\left(t^{2}-1\\right)^{2}-1$ (b) Suppose that $4 m n-m-n=p^{2}$, or equivalently, $(4 m-1)(4 n-1)=$ $4 p^{2}+1$. The number $4 m-1$ has at least one prime divisor, say $q$, that is of the form $4 k+3$. Then $4 p^{2} \\equiv-1(\\bmod q)$. However, by Fermat's theorem we have $$ 1 \\equiv(2 p)^{q-1}=\\left(4 p^{2}\\right)^{\\frac{q-1}{2}} \\equiv(-1)^{\\frac{q-1}{2}}(\\bmod q) $$ which is impossible since $(q-1) \/ 2=2 k+1$ is odd.","problem_type":null,"tier":0} +{"year":"1984","problem_phase":"shortlisted","problem":"20. (USA 2) Determine all pairs $(a, b)$ of positive real numbers with $a \\neq 1$ such that $$ \\log _{a} b<\\log _{a+1}(b+1) $$ ### 3.26 The Twenty-Sixth IMO","solution":"20. Define the set $S=\\mathbb{R}^{+} \\backslash\\{1\\}$. The given inequality is equivalent to $\\ln b \/ \\ln a<\\ln (b+1) \/ \\ln (a+1)$. If $b=1$, it is obvious that each $a \\in S$ satisfies this inequality. Suppose now that $b$ is also in $S$. Let us define on $S$ a function $f(x)=\\ln (x+1) \/ \\ln x$. Since $\\ln (x+1)>\\ln x$ and $1 \/ x>1 \/ x+1>0$, we have $$ f^{\\prime}(x)=\\frac{\\frac{\\ln x}{x+1}-\\frac{\\ln (x+1)}{x}}{\\ln ^{2} x}<0 \\quad \\text { for all } x $$ Hence $f$ is always decreasing. We also note that $f(x)<0$ for $x<1$ and that $f(x)>0$ for $x>1$ (at $x=1$ there is a discontinuity). Let us assume $b>1$. From $\\ln b \/ \\ln a<\\ln (b+1) \/ \\ln (a+1)$ we get $f(b)>$ $f(a)$. This holds for $b>a$ or for $a<1$. Now let us assume $b<1$. This time we get $f(b)1$. Hence all the solutions to $\\log _{a} b<\\log _{a+1}(b+1)$ are $\\{b=1, a \\in S\\}$, $\\{a>b>1\\},\\{b>1>a\\},\\{a3)$ vertices and let $p$ be its perimeter. Prove that $$ \\frac{n-3}{2}<\\frac{d}{p}<\\frac{1}{2}\\left(\\left[\\frac{n}{2}\\right]\\left[\\frac{n+1}{2}\\right]-2\\right) . $$","solution":"4. Consider the convex $n$-gon $A_{1} A_{2} \\ldots A_{n}$ (the indices are considered modulo $n)$. For any diagonal $A_{i} A_{j}$ we have $A_{i} A_{j}+A_{i+1} A_{j+1}>A_{i} A_{i+1}+A_{j} A_{j+1}$. Summing all such $n(n-3) \/ 2$ inequalities, we obtain $2 d>(n-3) p$, proving the first inequality. Let us now prove the second inequality. We notice that for each diagonal $A_{i} A_{i+j}$ (we may assume w.l.o.g. that $j \\leq[n \/ 2]$ ) the following relation holds: $$ A_{i} A_{i+j}0$ such that its color appears somewhere on the circle $C(X)$.","solution":"8. Suppose that the statement of the problem is false. Consider two arbitrary circles $R=(O, r)$ and $S=(O, s)$ with $0y^{\\prime}$ and $z>z^{\\prime}$, but then $\\sqrt{y-a}+\\sqrt{z-a}>\\sqrt{y^{\\prime}-a}+\\sqrt{z^{\\prime}-a}$, which is a contradiction. We shall now prove the existence of at least one solution. Let $P$ be an arbitrary point in the plane and $K, L, M$ points such that $P K=\\sqrt{a}$, $P L=\\sqrt{b}, P M=\\sqrt{c}$, and $\\angle K P L=\\angle L P M=\\angle M P K=120^{\\circ}$. The lines through $K, L, M$ perpendicular respectively to $P K, P L, P M$ form an equilateral triangle $A B C$, where $K \\in B C, L \\in A C$, and $M \\in A B$. Since its area equals $A B^{2} \\sqrt{3} \/ 4=S_{\\triangle B P C}+S_{\\triangle A P C}+$ $S_{\\triangle A P B}=A B(\\sqrt{a}+\\sqrt{b}+\\sqrt{c}) \/ 2$, it follows that $A B=1$. Therefore $x=P A^{2}, y=P B^{2}$, and $z=P C^{2}$ is a solution of the system (indeed, $\\sqrt{y-a}+\\sqrt{z-a}=\\sqrt{P B^{2}-P K^{2}}+\\sqrt{P C^{2}-P K^{2}}=B K+C K=1$, etc.).","problem_type":null,"tier":0} +{"year":"1985","problem_phase":"shortlisted","problem":"1. (MON 1) ${ }^{\\mathrm{IMO} 4}$ Given a set $M$ of 1985 positive integers, none of which has a prime divisor larger than 26 , prove that the set has four distinct elements whose geometric mean is an integer.","solution":"1. Since there are 9 primes ( $p_{1}=2513$ distinct two-element subsets of $M$ each having a square as the product of elements. Reasoning as above, we find at least one (in fact many) pair of such squares whose product is a fourth power.","problem_type":null,"tier":0} +{"year":"1985","problem_phase":"shortlisted","problem":"10. 2b.(VIE 1) Prove that for every point $M$ on the surface of a regular tetrahedron there exists a point $M^{\\prime}$ such that there are at least three different curves on the surface joining $M$ to $M^{\\prime}$ with the smallest possible length among all curves on the surface joining $M$ to $M^{\\prime}$.","solution":"10. If $M$ is at a vertex of the regular tetrahedron $A B C D(A B=1)$, then one can take $M^{\\prime}$ at the center of the opposite face of the tetrahedron. Let $M$ be on the face $(A B C)$ of the tetrahedron, excluding the vertices. Consider a continuous mapping $f$ of $\\mathbb{C}$ onto the surface $S$ of $A B C D$ that maps $m+n e^{2 \\pi \/ 3}$ for $m, n \\in$ $\\mathbb{Z}$ onto $A, B, C, D$ if $(m, n) \\equiv$ $(1,1),(1,0),(0,1),(0,0)(\\bmod 2)$ re- ![](https:\/\/cdn.mathpix.com\/cropped\/2024_11_18_8e985d6b9c83aa3e9d0eg-484.jpg?height=464&width=446&top_left_y=1356&top_left_x=857) spectively, and maps each unit equilateral triangle with vertices of the form $m+n e^{2 \\pi \/ 3}$ isometrically onto the corresponding face of $A B C D$. The point $M$ then has one preimage $M_{j}, j=1,2, \\ldots, 6$, in each of the six preimages of $\\triangle A B C$ having two vertices on the unit circle. The $M_{j}$ 's form a convex centrally symmetric (possibly degenerate) hexagon. Of the triangles formed by two adjacent sides of this hexagon consider the one, say $M_{1} M_{2} M_{3}$, with the smallest radius of circumcircle and denote by $\\widehat{M^{\\prime}}$ its circumcenter. Then we can choose $M^{\\prime}=f\\left(\\widehat{M^{\\prime}}\\right)$. Indeed, the images of the segments $M_{1} \\widehat{M^{\\prime}}, M_{2} \\widehat{M^{\\prime}}, M_{3} \\widehat{M^{\\prime}}$ are three different shortest paths on $S$ from $M$ to $M^{\\prime}$.","problem_type":null,"tier":0} +{"year":"1985","problem_phase":"shortlisted","problem":"11. 3a.(USS 3) Find a method by which one can compute the coefficients of $P(x)=x^{6}+a_{1} x^{5}+\\cdots+a_{6}$ from the roots of $P(x)=0$ by performing not more than 15 additions and 15 multiplications.","solution":"11. Let $-x_{1}, \\ldots,-x_{6}$ be the roots of the polynomial. Let $s_{k, i}(k \\leq i \\leq 6)$ denote the sum of all products of $k$ of the numbers $x_{1}, \\ldots, x_{i}$. By Vieta's formula we have $a_{k}=s_{k, 6}$ for $k=1, \\ldots, 6$. Since $s_{k, i}=s_{k-1, i-1} x_{i}+$ $s_{k, i-1}$, one can compute the $a_{k}$ by the following scheme (the horizontal and vertical arrows denote multiplications and additions respectively): ![](https:\/\/cdn.mathpix.com\/cropped\/2024_11_18_8e985d6b9c83aa3e9d0eg-485.jpg?height=457&width=609&top_left_y=646&top_left_x=486)","problem_type":null,"tier":0} +{"year":"1985","problem_phase":"shortlisted","problem":"12. 3b.(GBR 4) A sequence of polynomials $P_{m}(x, y, z), m=0,1,2, \\ldots$, in $x, y$, and $z$ is defined by $P_{0}(x, y, z)=1$ and by $$ P_{m}(x, y, z)=(x+z)(y+z) P_{m-1}(x, y, z+1)-z^{2} P_{m-1}(x, y, z) $$ for $m>0$. Prove that each $P_{m}(x, y, z)$ is symmetric, in other words, is unaltered by any permutation of $x, y, z$.","solution":"12. We shall prove by induction on $m$ that $P_{m}(x, y, z)$ is symmetric and that $$ (x+y) P_{m}(x, z, y+1)-(x+z) P_{m}(x, y, z+1)=(y-z) P_{m}(x, y, z) $$ holds for all $x, y, z$. This is trivial for $m=0$. Assume now that it holds for $m=n-1$. Since obviously $P_{n}(x, y, z)=P_{n}(y, x, z)$, the symmetry of $P_{n}$ will follow if we prove that $P_{n}(x, y, z)=P_{n}(x, z, y)$. Using (1) we have $P_{n}(x, z, y)-$ $P_{n}(x, y, z)=(y+z)\\left[(x+y) P_{n-1}(x, z, y+1)-(x+z) P_{n-1}(x, y, z+1)\\right]-\\left(y^{2}-\\right.$ $\\left.z^{2}\\right) P_{n-1}(x, y, z)=(y+z)(y-z) P_{n-1}(x, y, z)-\\left(y^{2}-z^{2}\\right) P_{n-1}(x, y, z)=0$. It remains to prove (1) for $m=n$. Using the already established symmetry we have $$ \\begin{aligned} & (x+y) P_{n}(x, z, y+1)-(x+z) P_{n}(x, y, z+1) \\\\ & =(x+y) P_{n}(y+1, z, x)-(x+z) P_{n}(z+1, y, x) \\\\ & =(x+y)\\left[(y+x+1)(z+x) P_{n-1}(y+1, z, x+1)-x^{2} P_{n-1}(y+1, z, x)\\right] \\\\ & \\quad-(x+z)\\left[(z+x+1)(y+x) P_{n-1}(z+1, y, x+1)-x^{2} P_{n-1}(z+1, y, x)\\right] \\\\ & =(x+y)(x+z)(y-z) P_{n-1}(x+1, y, z)-x^{2}(y-z) P_{n-1}(x, y, z) \\\\ & =(y-z) P_{n}(z, y, x)=(y-z) P_{n}(x, y, z), \\end{aligned} $$ as claimed.","problem_type":null,"tier":0} +{"year":"1985","problem_phase":"shortlisted","problem":"13. 4a.(BUL 1) Let $m$ boxes be given, with some balls in each box. Let $n1$. If initially there is only one ball in the boxes, then after $k$ operations the number of balls will be $1+k m$, which is never divisible by $n$. Hence the task cannot be done.","problem_type":null,"tier":0} +{"year":"1985","problem_phase":"shortlisted","problem":"14. 4b.(IRE 4) A set of 1985 points is distributed around the circumference of a circle and each of the points is marked with 1 or -1 . A point is called \"good\" if the partial sums that can be formed by starting at that point and proceeding around the circle for any distance in either direction are all strictly positive. Show that if the number of points marked with -1 is less than 662 , there must be at least one good point.","solution":"14. It suffices to prove the existence of a good point in the case of exactly 661 -1 's. We prove by induction on $k$ that in any arrangement with $3 k+2$ points $k$ of which are -1 's a good point exists. For $k=1$ this is clear by inspection. Assume that the assertion holds for all arrangements of $3 n+2$ points and consider an arrangement of $3(n+1)+2$ points. Now there exists a sequence of consecutive -1 's surrounded by two +1 's. There is a point $P$ which is good for the arrangement obtained by removing the two +1 's bordering the sequence of -1 's and one of these -1 's. Since $P$ is out of this sequence, clearly the removal either leaves a partial sum as it was or diminishes it by 1 , so $P$ is good for the original arrangement. Second solution. Denote the number on an arbitrary point by $a_{1}$, and the numbers on successive points going in the positive direction by $a_{2}, a_{3}, \\ldots$ (in particular, $a_{k+1985}=a_{k}$ ). We define the partial sums $s_{0}=0, s_{n}=$ $a_{1}+a_{2}+\\cdots+a_{n}$ for all positive integers $n$; then $s_{k+1985}=s_{k}+s_{1985}$ and $s_{1985} \\geq 663$. Since $s_{1985 m} \\geq 663 m$ and $3 \\cdot 663 m>1985(m+2)+1$ for large $m$, not all values $0,1,2, \\ldots 663 m$ can appear thrice among the $1985(m+2)+1$ sums $s_{-1985}, s_{-1984}, \\ldots, s_{1985(m+1)}$ (and none of them appears out of this set). Thus there is an integral value $s>0$ that appears at most twice as a partial sum, say $s_{k}=s_{l}=s, ks$ must hold for all $i>l$, and $s_{i}s$ and $s_{q}Y Z$, and $X^{\\prime} Y^{\\prime} \\cdot Y^{\\prime} Z^{\\prime}=X Y \\cdot Y Z$. Suppose that $2 X^{\\prime} Y^{\\prime}>X Y$ (otherwise, we may cut off congruent rectangles from both the original ones until we reduce them to the case of $2 X^{\\prime} Y^{\\prime}>X Y$ ). Let $U \\in X Y$ and $V \\in Z T$ be points such that $Y U=T V=X^{\\prime} Y^{\\prime}$ and $W \\in X V$ be a point such that $U W \\| X T$. Then translating $\\triangle X U W$ to a triangle $V Z R$ and $\\triangle X V T$ to a triangle $W R S$ results in a rectangle $U Y R S$ congruent to $X^{\\prime} Y^{\\prime} Z^{\\prime} T^{\\prime}$. Thus we have partitioned $K$ and $K^{\\prime}$ into translation-invariant parts. Although not all the parts are triangles, we may simply triangulate them.","problem_type":null,"tier":0} +{"year":"1985","problem_phase":"shortlisted","problem":"16. 5b.(BEL 2) If possible, construct an equilateral triangle whose three vertices are on three given circles.","solution":"16. Let the three circles be $\\alpha(A, a), \\beta(B, b)$, and $\\gamma(C, c)$, and assume $c \\leq a, b$. We denote by $\\mathcal{R}_{X, \\varphi}$ the rotation around $X$ through an angle $\\varphi$. Let $P Q R$ be an equilateral triangle, say of positive orientation (the case of negatively oriented $\\triangle P Q R$ is analogous), with $P \\in \\alpha, Q \\in \\beta$, and $R \\in \\gamma$. Then $Q=\\mathcal{R}_{P,-60^{\\circ}}(R) \\in \\mathcal{R}_{P,-60^{\\circ}}(\\gamma) \\cap \\beta$. Since the center of $\\mathcal{R}_{P,-60^{\\circ}}(\\gamma)$ is $\\mathcal{R}_{P,-60^{\\circ}}(C)=\\mathcal{R}_{C, 60^{\\circ}}(P)$ and it lies on $\\mathcal{R}_{C, 60^{\\circ}}(\\alpha)$, the union of circles $\\mathcal{R}_{P,-60^{\\circ}}(\\gamma)$ as $P$ varies on $\\alpha$ is the annulus $\\mathcal{U}$ with center $A^{\\prime}=\\mathcal{R}_{C, 60^{\\circ}}(A)$ and radii $a-c$ and $a+c$. Hence there is a solution if and only if $\\mathcal{U} \\cap \\beta$ is nonempty.","problem_type":null,"tier":0} +{"year":"1985","problem_phase":"shortlisted","problem":"17. 6a. (SWE 3) ${ }^{\\mathrm{IMO} O}$ The sequence $f_{1}, f_{2}, \\ldots, f_{n}, \\ldots$ of functions is defined for $x>0$ recursively by $$ f_{1}(x)=x, \\quad f_{n+1}(x)=f_{n}(x)\\left(f_{n}(x)+\\frac{1}{n}\\right) . $$ Prove that there exists one and only one positive number $a$ such that $0x$ for all $x>0$ implies that $y_{n}-x_{n}0$ with $\\prod_{i=1}^{n} y_{i}=1$. Then $\\frac{1}{1+y_{n-1}}+\\frac{1}{1+y_{n}}>\\frac{1}{1+y_{n-1} y_{n}}$, which is equivalent to $1+y_{n} y_{n-1}(1+$ $\\left.y_{n}+y_{n-1}\\right)>0$. Hence by the inductive hypothesis $$ \\sum_{i=1}^{n} \\frac{1}{1+y_{i}} \\geq \\sum_{i=1}^{n-2} \\frac{1}{1+y_{i}}+\\frac{1}{1+y_{n-1} y_{n}} \\geq 1 $$ Remark. The constant $n-1$ is best possible (take for example $x_{i}=a^{i}$ with $a$ arbitrarily large).","problem_type":null,"tier":0} +{"year":"1985","problem_phase":"shortlisted","problem":"19. (ISR 3) For which integers $n \\geq 3$ does there exist a regular $n$-gon in the plane such that all its vertices have integer coordinates in a rectangular coordinate system?","solution":"19. Suppose that for some $n>6$ there is a regular $n$-gon with vertices having integer coordinates, and that $A_{1} A_{2} \\ldots A_{n}$ is the smallest such $n$-gon, of side length $a$. If $O$ is the origin and $B_{i}$ the point such that $\\overrightarrow{O B_{i}}=\\overrightarrow{A_{i-1} A_{i}}$, $i=1,2, \\ldots, n$ (where $A_{0}=A_{n}$ ), then $B_{i}$ has integer coordinates and $B_{1} B_{2} \\ldots B_{n}$ is a regular polygon of side length $2 a \\sin (\\pi \/ n)j$, then by (ii), $\\langle k j\\rangle \\sim\\langle k j\\rangle-j=\\langle(k-1) j\\rangle$. If otherwise $\\langle k j\\rangle0$, then the midpoint $P$ of $M N$ lies inside the circle $C_{(m+n) \/ 2}$. This is trivial if $m=n$, so let $m \\neq n$. For fixed $M, P$ is in the image $C_{n}^{\\prime}$ of $C_{n}$ under the homothety with center $M$ and coefficient $1 \/ 2$. The center of the circle $C_{n}^{\\prime}$ is at the midpoint of $O_{n} M$. If we let both $M$ and $N$ vary, $P$ will be on the union of circles with radius $r_{n} \/ 2$ and centers in the image of $C_{m}$ under the homothety with center $O_{n}$ and coefficient $1 \/ 2$. Hence $P$ is not outside the circle centered at the midpoint $O_{m} O_{n}$ and with radius $\\left(r_{m}+r_{n}\\right) \/ 2$. It remains to show that $r_{(m+n) \/ 2}>\\left(r_{m}+r_{n}\\right) \/ 2$. But this inequality is easily reduced to $(m-n)^{2}>0$, which is true.","problem_type":null,"tier":0} +{"year":"1985","problem_phase":"shortlisted","problem":"6. (POL 1) Let $x_{n}=\\sqrt[2]{2+\\sqrt[3]{3+\\ldots+\\sqrt[n]{n}}}$. Prove that $$ x_{n+1}-x_{n}<\\frac{1}{n!}, \\quad n=2,3, \\ldots $$ ## Alternatives","solution":"6. Let us set $$ \\begin{aligned} & x_{n, i}=\\sqrt[i]{i+\\sqrt[i+1]{i+1+\\cdots+\\sqrt[n]{n}}} \\\\ & y_{n, i}=x_{n+1, i}^{i-1}+x_{n+1, i}^{i-2} x_{n, i}+\\cdots+x_{n, i}^{i-1} \\end{aligned} $$ In particular, $x_{n, 2}=x_{n}$ and $x_{n, i}=0$ for $i>n$. We observe that for $n>i>2$, $$ x_{n+1, i}-x_{n, i}=\\frac{x_{n+1, i}^{i}-x_{n, i}^{i}}{y_{n, i}}=\\frac{x_{n+1, i+1}-x_{n, i+1}}{y_{n, i}} . $$ Since $y_{n, i}>i x_{n, i}^{i-1} \\geq i^{1+(i-1) \/ i} \\geq i^{3 \/ 2}$ and $x_{n+1, n+1}-x_{n, n+1}=\\sqrt[n+1]{n+1}$, simple induction gives $$ x_{n+1}-x_{n} \\leq \\frac{\\sqrt[n+1]{n+1}}{(n!)^{3 \/ 2}}<\\frac{1}{n!} \\quad \\text { for } n>2 $$ The inequality for $n=2$ is directly verified.","problem_type":null,"tier":0} +{"year":"1985","problem_phase":"shortlisted","problem":"7. 1a.(CZS 3) The positive integers $x_{1}, \\ldots, x_{n}, n \\geq 3$, satisfy $x_{1}k_{j}$, then $x_{i} \\geq p x_{j} \\geq 2 x_{i} \\geq 2 x_{1}$, which is impossible. Thus $y_{1} y_{2} \\ldots y_{n}=P \/ p^{k} \\geq n!$. If equality holds, we must have $y_{i}=1, y_{j}=2$ and $y_{k}=3$ for some $i, j, k$. Thus $p \\geq 5$, which implies that either $y_{i} \/ y_{j} \\leq 1 \/ 2$ or $y_{i} \/ y_{j} \\geq 5 \/ 2$, which is impossible. Hence the inequality is strict.","problem_type":null,"tier":0} +{"year":"1985","problem_phase":"shortlisted","problem":"8. 1b.(TUR 5) Find the smallest positive integer $n$ such that (i) $n$ has exactly 144 distinct positive divisors, and (ii) there are ten consecutive integers among the positive divisors of $n$.","solution":"8. Among ten consecutive integers that divide $n$, there must exist numbers divisible by $2^{3}, 3^{2}, 5$, and 7 . Thus the desired number has the form $n=$ $2^{\\alpha_{1}} 3^{\\alpha_{2}} 5^{\\alpha_{3}} 7^{\\alpha_{4}} 11^{\\alpha_{5}} \\cdots$, where $\\alpha_{1} \\geq 3, \\alpha_{2} \\geq 2, \\alpha_{3} \\geq 1, \\alpha_{4} \\geq 1$. Since $n$ has $\\left(\\alpha_{1}+1\\right)\\left(\\alpha_{2}+1\\right)\\left(\\alpha_{3}+1\\right) \\cdots$ distinct factors, and $\\left(\\alpha_{1}+1\\right)\\left(\\alpha_{2}+1\\right)\\left(\\alpha_{3}+\\right.$ $1)\\left(\\alpha_{4}+1\\right) \\geq 48$, we must have $\\left(\\alpha_{5}+1\\right) \\cdots \\leq 3$. Hence at most one $\\alpha_{j}$, $j>4$, is positive, and in the minimal $n$ this must be $\\alpha_{5}$. Checking through the possible combinations satisfying $\\left(\\alpha_{1}+1\\right)\\left(\\alpha_{2}+1\\right) \\cdots\\left(\\alpha_{5}+1\\right)=144$ one finds that the minimal $n$ is $2^{5} \\cdot 3^{2} \\cdot 5 \\cdot 7 \\cdot 11=110880$.","problem_type":null,"tier":0} +{"year":"1985","problem_phase":"shortlisted","problem":"9. 2a.(USA 3) Determine the radius of a sphere $S$ that passes through the centroids of each face of a given tetrahedron $T$ inscribed in a unit sphere with center $O$. Also, determine the distance from $O$ to the center of $S$ as a function of the edges of $T$.","solution":"9. Let $\\vec{a}, \\vec{b}, \\vec{c}, \\vec{d}$ denote the vectors $\\overrightarrow{O A}, \\overrightarrow{O B}, \\overrightarrow{O C}, \\overrightarrow{O D}$ respectively. Then $|\\vec{a}|=|\\vec{b}|=|\\vec{c}|=|\\vec{d}|=1$. The centroids of the faces are $(\\vec{b}+\\vec{c}+\\vec{d}) \/ 3$, $(\\vec{a}+\\vec{c}+\\vec{d}) \/ 3$, etc., and each of these is at distance $1 \/ 3$ from $P=$ $(\\vec{a}+\\vec{b}+\\vec{c}+\\vec{d}) \/ 3$; hence the required radius is $1 \/ 3$. To compute $|P|$ as a function of the edges of $A B C D$, observe that $A B^{2}=(\\vec{b}-\\vec{a})^{2}=$ $2-2 \\vec{a} \\cdot \\vec{b}$ etc. Now $$ \\begin{aligned} P^{2} & =\\frac{|\\vec{a}+\\vec{b}+\\vec{c}+\\vec{d}|^{2}}{9} \\\\ & =\\frac{16-2\\left(A B^{2}+B C^{2}+A C^{2}+A D^{2}+B D^{2}+C D^{2}\\right)}{9} \\end{aligned} $$","problem_type":null,"tier":0} +{"year":"1986","problem_phase":"shortlisted","problem":"1. (GBR 3) ${ }^{\\text {IMO5 }}$ Find, with proof, all functions $f$ defined on the nonnegative real numbers and taking nonnegative real values such that (i) $f[x f(y)] f(y)=f(x+y)$, (ii) $f(2)=0$ but $f(x) \\neq 0$ for $0 \\leq x<2$.","solution":"1. If $w>2$, then setting in (i) $x=w-2, y=2$, we get $f(w)=f((w-$ 2) $f(w)) f(2)=0$. Thus $$ f(x)=0 \\quad \\text { if and only if } \\quad x \\geq 2 $$ Now let $0 \\leq y<2$ and $x \\geq 0$. The LHS in (i) is zero if and only if $x f(y) \\geq 2$, while the RHS is zero if and only if $x+y \\geq 2$. It follows that $x \\geq 2 \/ f(y)$ if and only if $x \\geq 2-y$. Therefore $$ f(y)=\\left\\{\\begin{array}{cl} \\frac{2}{2-y} & \\text { for } 0 \\leq y<2 \\\\ 0 & \\text { for } y \\geq 2 \\end{array}\\right. $$ The confirmation that $f$ satisfies the given conditions is straightforward.","problem_type":null,"tier":0} +{"year":"1986","problem_phase":"shortlisted","problem":"10. (HUN 2) Three persons $A, B, C$, are playing the following game: A $k$ element subset of the set $\\{1, \\ldots, 1986\\}$ is randomly chosen, with an equal probability of each choice, where $k$ is a fixed positive integer less than or equal to 1986. The winner is $A, B$ or $C$, respectively, if the sum of the chosen numbers leaves a remainder of 0,1 , or 2 when divided by 3 . For what values of $k$ is this game a fair one? (A game is fair if the three outcomes are equally probable.)","solution":"10. The set $X=\\{1, \\ldots, 1986\\}$ splits into triads $T_{1}, \\ldots, T_{662}$, where $T_{j}=$ $\\{3 j-2,3 j-1,3 j\\}$. Let $\\mathcal{F}$ be the family of all $k$-element subsets $P$ such that $\\left|P \\cap T_{j}\\right|=1$ or 2 for some index $j$. If $j_{0}$ is the smallest such $j_{0}$, we define $P^{\\prime}$ to be the $k$-element set obtained from $P$ by replacing the elements of $P \\cap T_{j_{0}}$ by the ones following cyclically inside $T_{j_{0}}$. Let $s(P)$ denote the remainder modulo 3 of the sum of elements of $P$. Then $s(P), s\\left(P^{\\prime}\\right), s\\left(P^{\\prime \\prime}\\right)$ are distinct, and $P^{\\prime \\prime \\prime}=P$. Thus the operator ${ }^{\\prime}$ gives us a bijective correspondence between the sets $X \\in \\mathcal{F}$ with $s(P)=0$, those with $s(P)=1$, and those with $s(P)=2$. If $3 \\nmid k$ is not divisible by 3 , then each $k$-element subset of $X$ belongs to $\\mathcal{F}$, and the game is fair. If $3 \\mid k$, then $k$-element subsets not belonging to $\\mathcal{F}$ are those that are unions of several triads. Since every such subset has the sum of elements divisible by 3 , it follows that player $A$ has the advantage.","problem_type":null,"tier":0} +{"year":"1986","problem_phase":"shortlisted","problem":"11. (BUL 1) Let $f(n)$ be the least number of distinct points in the plane such that for each $k=1,2, \\ldots, n$ there exists a straight line containing exactly $k$ of these points. Find an explicit expression for $f(n)$. Simplified version. Show that $f(n)=\\left[\\frac{n+1}{2}\\right]\\left[\\frac{n+2}{2}\\right]$ ( $[x]$ denoting the greatest integer not exceeding $x$ ).","solution":"11. Let $X$ be a finite set in the plane and $l_{k}$ a line containing exactly $k$ points of $X(k=1, \\ldots, n)$. Then $l_{n}$ contains $n$ points, $l_{n-1}$ contains at least $n-2$ points not lying on $l_{n}, l_{n-2}$ contains at least $n-4$ points not lying on $l_{n}$ or $l_{n-1}$, etc. It follows that $$ |X| \\geq g(n)=n+(n-2)+(n-4)+\\cdots+\\left(n-2\\left[\\frac{n}{2}\\right]\\right) . $$ Hence $f(n) \\geq g(n)=\\left[\\frac{n+1}{2}\\right]\\left[\\frac{n+2}{2}\\right]$, where the last equality is easily proved by induction. We claim that $f(n)=g(n)$. To prove this, we shall inductively construct a set $X_{n}$ of cardinality $g(n)$ with the required property. For $n \\leq 2$ a one-point and two-point set satisfy the requirements. Assume that $X_{n}$ is a set of $g(n)$ points and that $l_{k}$ is a line containing exactly $k$ points of $X_{n}, k=1, \\ldots, n$. Consider any line $l$ not parallel to any of the $l_{k}$ 's and not containing any point of $X_{n}$ or any intersection point of the $l_{k}$. Let $l$ intersect $l_{k}$ in a point $P_{k}, k=1, \\ldots, n$, and let $P_{n+1}, P_{n+2}$ be two points on $l$ other than $P_{1}, \\ldots, P_{n}$. We define $X_{n+2}=X_{n} \\cup\\left\\{P_{1}, \\ldots, P_{n+2}\\right\\}$. The set $X_{n+2}$ consists of $g(n)+(n+2)=g(n+2)$ points. Since the lines $l, l_{n}, \\ldots, l_{2}, l_{1}$ meet $X_{n}$ in $n+2, n+1, \\ldots, 3,2$ points respectively (and there clearly exists a line containing only one point of $X_{n+2}$ ), this set also meets the demands.","problem_type":null,"tier":0} +{"year":"1986","problem_phase":"shortlisted","problem":"12. (GDR 3) ${ }^{\\mathrm{IMO} 3}$ To each vertex $P_{i}(i=1, \\ldots, 5)$ of a pentagon an integer $x_{i}$ is assigned, the sum $s=\\sum x_{i}$ being positive. The following operation is allowed, provided at least one of the $x_{i}$ 's is negative: Choose a negative $x_{i}$, replace it by $-x_{i}$, and add the former value of $x_{i}$ to the integers assigned to the two neighboring vertices of $P_{i}$ (the remaining two integers are left unchanged). This operation is to be performed repeatedly until all negative integers disappear. Decide whether this procedure must eventually terminate.","solution":"12. We define $f\\left(x_{1}, \\ldots, x_{5}\\right)=\\sum_{i=1}^{5}\\left(x_{i+1}-x_{i-1}\\right)^{2}\\left(x_{0}=x_{5}, x_{6}=x_{1}\\right)$. Assuming that $x_{3}<0$, according to the rules the lattice vector $X=$ $\\left(x_{1}, x_{2}, x_{3}, x_{4}, x_{5}\\right)$ changes into $Y=\\left(x_{1}, x_{2}+x_{3},-x_{3}, x_{4}+x_{3}, x_{5}\\right)$. Then $$ \\begin{aligned} f(Y)-f(X)= & \\left(x_{2}+x_{3}-x_{5}\\right)^{2}+\\left(x_{1}+x_{3}\\right)^{2}+\\left(x_{2}-x_{4}\\right)^{2} \\\\ & +\\left(x_{3}+x_{5}\\right)^{2}+\\left(x_{1}-x_{3}-x_{4}\\right)^{2}-\\left(x_{2}-x_{5}\\right)^{2} \\\\ & -\\left(x_{3}-x_{1}\\right)^{2}-\\left(x_{4}-x_{2}\\right)^{2}-\\left(x_{5}-x_{3}\\right)^{2}-\\left(x_{1}-x_{4}\\right)^{2} \\\\ = & 2 x_{3}\\left(x_{1}+x_{2}+x_{3}+x_{4}+x_{5}\\right)=2 x_{3} S<0 \\end{aligned} $$ Thus $f$ strictly decreases after each step, and since it takes only positive integer values, the number of steps must be finite. Remark. One could inspect the behavior of $g(x)=\\sum_{i=1}^{5} \\sum_{j=1}^{5} \\mid x_{i}+x_{i+1}+$ $\\cdots+x_{j-1} \\mid$ instead. Then $g(Y)-g(X)=\\left|S+x_{3}\\right|-\\left|S-x_{3}\\right|>0$.","problem_type":null,"tier":0} +{"year":"1986","problem_phase":"shortlisted","problem":"13. (FRG 3) A particle moves from $(0,0)$ to $(n, n)$ directed by a fair coin. For each head it moves one step east and for each tail it moves one step north. At $(n, y), y90^{\\circ}$ or $\\angle D>90^{\\circ}$. We similarly obtain that $A^{\\prime \\prime} C^{\\prime \\prime}=$ $B^{\\prime} D^{\\prime}\\left|\\frac{\\sin \\left(\\angle A^{\\prime}+\\angle C^{\\prime}\\right)}{2 \\sin \\angle A^{\\prime} \\sin \\angle C^{\\prime}}\\right|$. Therefore $$ A^{\\prime \\prime} C^{\\prime \\prime}=A C \\frac{\\sin ^{2}(\\angle A+\\angle C)}{4 \\sin \\angle A \\sin \\angle B \\sin \\angle C \\sin \\angle D} $$","problem_type":null,"tier":0} +{"year":"1986","problem_phase":"shortlisted","problem":"16. (ISR 1) ${ }^{\\mathrm{IMO}}$ Let $A, B$ be adjacent vertices of a regular $n$-gon in the plane and let $O$ be its center. Now let the triangle $A B O$ glide around the polygon in such a way that the points $A$ and $B$ move along the whole circumference of the polygon. Describe the figure traced by the vertex $O$.","solution":"16. Let $Z$ be the center of the polygon. Suppose that at some moment we have $A \\in P_{i-1} P_{i}$ and $B \\in$ $P_{i} P_{i+1}$, where $P_{i-1}, P_{i}, P_{i+1}$ are adjacent vertices of the polygon. Since $\\angle A O B=180^{\\circ}-\\angle P_{i-1} P_{i} P_{i+1}$, the quadrilateral $A P_{i} B O$ is cyclic. Hence $\\angle A P_{i} O=\\angle A B O=\\angle A P_{i} Z$, which means that $O \\in P_{i} Z$. ![](https:\/\/cdn.mathpix.com\/cropped\/2024_11_18_8e985d6b9c83aa3e9d0eg-495.jpg?height=329&width=390&top_left_y=251&top_left_x=903) Moreover, from $O P_{i}=2 r \\sin \\angle P_{i} A O$, where $r$ is the radius of circle $A P_{i} B O$, we obtain that $Z P_{i} \\leq O P_{i} \\leq Z P_{i} \/ \\cos (\\pi \/ n)$. Thus $O$ traces a segment $Z Z_{i}$ as $A$ and $B$ move along $P_{i-1} P_{i}$ and $P_{i} P_{i+1}$ respectively, where $Z_{i}$ is a point on the ray $P_{i} Z$ with $P_{i} Z_{i} \\cos (\\pi \/ n)=P_{i} Z$. When $A, B$ move along the whole circumference of the polygon, $O$ traces an asterisk consisting of $n$ segments of equal length emanating from $Z$ and pointing away from the vertices.","problem_type":null,"tier":0} +{"year":"1986","problem_phase":"shortlisted","problem":"17. ( $\\mathbf{C H N} 3)^{\\mathrm{IMO} 2}$ Let $A, B, C$ be fixed points in the plane. A man starts from a certain point $P_{0}$ and walks directly to $A$. At $A$ he turns his direction by $60^{\\circ}$ to the left and walks to $P_{1}$ such that $P_{0} A=A P_{1}$. After he does the same action 1986 times successively around the points $A, B, C, A, B, C, \\ldots$, he returns to the starting point. Prove that $\\triangle A B C$ is equilateral and that the vertices $A, B, C$ are arranged counterclockwise.","solution":"17. We use complex numbers to represent the position of a point in the plane. For convenience, let $A_{1}, A_{2}, A_{3}, A_{4}, A_{5}, \\ldots$ be $A, B, C, A, B, \\ldots$ respectively, and let $P_{0}$ be the origin. After the $k$ th step, the position of $P_{k}$ will be $P_{k}=A_{k}+\\left(P_{k-1}-A_{k}\\right) u, k=1,2,3, \\ldots$, where $u=e^{4 \\pi \\tau \/ 3}$. We easily obtain $$ P_{k}=(1-u)\\left(A_{k}+u A_{k-1}+u^{2} A_{k-2}+\\cdots+u^{k-1} A_{1}\\right) $$ The condition $P_{0} \\equiv P_{1986}$ is equivalent to $A_{1986}+u A_{1985}+\\cdots+u^{1984} A_{2}+$ $u^{1985} A_{1}=0$, which, having in mind that $A_{1}=A_{4}=A_{7}=\\cdots, A_{2}=A_{5}=$ $A_{8}=\\cdots, A_{3}=A_{6}=A_{9}=\\cdots$, reduces to $$ 662\\left(A_{3}+u A_{2}+u^{2} A_{1}\\right)=\\left(1+u^{3}+\\cdots+u^{1983}\\right)\\left(A_{3}+u A_{2}+u^{2} A_{1}\\right)=0 $$ It follows that $A_{3}-A_{1}=u\\left(A_{1}-A_{2}\\right)$, and the assertion follows. Second solution. Let $f_{P}$ denote the rotation with center $P$ through $120^{\\circ}$ clockwise. Let $f_{1}=f_{A}$. Then $f_{1}\\left(P_{0}\\right)=P_{1}$. Let $B^{\\prime}=f_{1}(B), C^{\\prime}=f_{1}(C)$, and $f_{2}=f_{B^{\\prime}}$. Then $f_{2}\\left(P_{1}\\right)=P_{2}$ and $f_{2}\\left(A B^{\\prime} C^{\\prime}\\right)=A^{\\prime} B^{\\prime} C^{\\prime \\prime}$. Finally, let $f_{3}=f_{C^{\\prime \\prime}}$ and $f_{3}\\left(A^{\\prime} B^{\\prime} C^{\\prime \\prime}\\right)=A^{\\prime \\prime} B^{\\prime \\prime} C^{\\prime \\prime}$. Then $g=f_{3} f_{2} f_{1}$ is a translation sending $P_{0}$ to $P_{3}$ and $C$ to $C^{\\prime \\prime}$. Now $P_{1986}=P_{0}$ implies that $g^{662}$ is the identity, and thus $C=C^{\\prime \\prime}$. Let $K$ be such that $A B K$ is equilateral and positively oriented. We observe that $f_{2} f_{1}(K)=K$; therefore the rotation $f_{2} f_{1}$ satisfies $f_{2} f_{1}(P) \\neq P$ for $P \\neq K$. Hence $f_{2} f_{1}(C)=C^{\\prime \\prime}=C$ implies $K=C$.","problem_type":null,"tier":0} +{"year":"1986","problem_phase":"shortlisted","problem":"18. (TUR 1) Let $A X, B Y, C Z$ be three cevians concurrent at an interior point $D$ of a triangle $A B C$. Prove that if two of the quadrangles $D Y A Z, D Z B X, D X C Y$ are circumscribable, so is the third.","solution":"18. We shall use the following criterion for a quadrangle to be circumscribable. Lemma. The quadrangle $A Y D Z$ is circumscribable if and only if $D B-$ $D C=A B-A C$. Proof. Suppose that $A Y D Z$ is circumscribable and that the incircle is tangent to $A Z, Z D, D Y, Y A$ at $M, N, P, Q$ respectively. Then $D B-D C=P B-N C=M B-Q C=A B-A C$. Conversely, assume that $D B-D C=A B-A C$ and let a tangent from $D$ to the incircle of the triangle $A C Z$ meet $C Z$ and $C A$ at $D^{\\prime} \\neq Z$ and $Y^{\\prime} \\neq A$ respectively. According to the first part we have $D^{\\prime} B-D^{\\prime} C=A B-A C$. It follows that $\\left|D^{\\prime} B-D B\\right|=$ ![](https:\/\/cdn.mathpix.com\/cropped\/2024_11_18_8e985d6b9c83aa3e9d0eg-496.jpg?height=315&width=387&top_left_y=236&top_left_x=900) $\\left|D^{\\prime} C-D C\\right|=D D^{\\prime}$, implying that $D^{\\prime} \\equiv D$. Let us assume that $D Z B X$ and $D X C Y$ are circumscribable. Using the lemma we obtain $D C-D A=B C-B A$ and $D A-D B=C A-C B$. Adding these two inequalities yields $D C-D B=A C-A B$, and the statement follows from the lemma.","problem_type":null,"tier":0} +{"year":"1986","problem_phase":"shortlisted","problem":"19. (BUL 2) A tetrahedron $A B C D$ is given such that $A D=B C=a$; $A C=B D=b ; A B \\cdot C D=c^{2}$. Let $f(P)=A P+B P+C P+D P$, where $P$ is an arbitrary point in space. Compute the least value of $f(P)$.","solution":"19. Let $M$ and $N$ be the midpoints of segments $A B$ and $C D$, respectively. The given conditions imply that $\\triangle A B D \\cong \\triangle B A C$ and $\\triangle C D A \\cong \\triangle D C B$; hence $M C=M D$ and $N A=N B$. It follows that $M$ and $N$ both lie on the perpendicular bisectors of $A B$ and $C D$, and consequently $M N$ is the common perpendicular bisector of $A B$ and $C D$. Points $B$ and $C$ are symmetric to $A$ and $D$ with respect to $M N$. Now if $P$ is a point in space and $P^{\\prime}$ the point symmetric to $P$ with respect to $M N$, we have $B P=A P^{\\prime}, C P=D P^{\\prime}$, and thus $f(P)=A P+A P^{\\prime}+D P+D P^{\\prime}$. Let $P P^{\\prime}$ intersect $M N$ in $Q$. Then $A P+A P^{\\prime} \\geq 2 A Q$ and $D P+D P^{\\prime} \\geq 2 D Q$, from which it follows that $f(P) \\geq 2(A Q+D Q)=f(Q)$. It remains to minimize $f(Q)$ with $Q$ moving along the line $M N$. Let us rotate point $D$ around $M N$ to a point $D^{\\prime}$ that belongs to the plane $A M N$, on the side of $M N$ opposite to $A$. Then $f(Q)=2\\left(A Q+D^{\\prime} Q\\right) \\geq$ $A D^{\\prime}$, and equality occurs when $Q$ is the intersection of $A D^{\\prime}$ and $M N$. Thus $\\min f(Q)=A D^{\\prime}$. We note that $4 M D^{2}=2 A D^{2}+2 B D^{2}-A B^{2}=$ $2 a^{2}+2 b^{2}-A B^{2}$ and $4 M N^{2}=4 M D^{2}-C D^{2}=2 a^{2}+2 b^{2}-A B^{2}-C D^{2}$. Now, $A D^{\\prime 2}=\\left(A M+D^{\\prime} N\\right)^{2}+M N^{2}$, which together with $A M+D^{\\prime} N=$ $(a+b) \/ 2$ gives us $$ A D^{\\prime 2}=\\frac{a^{2}+b^{2}+A B \\cdot C D}{2}=\\frac{a^{2}+b^{2}+c^{2}}{2} $$ We conclude that $\\min f(Q)=\\sqrt{\\left(a^{2}+b^{2}+c^{2}\\right) \/ 2}$.","problem_type":null,"tier":0} +{"year":"1986","problem_phase":"shortlisted","problem":"2. (SWE 2) Let $f(x)=x^{n}$ where $n$ is a fixed positive integer and $x=$ $1,2, \\ldots$. Is the decimal expansion $a=0 . f(1) f(2) f(3) \\ldots$ rational for any value of $n$ ? The decimal expansion of $a$ is defined as follows: If $f(x)=d_{1}(x) d_{2}(x) \\ldots$ $\\ldots d_{r(x)}(x)$ is the decimal expansion of $f(x)$, then $a=0.1 d_{1}(2) d_{2}(2) \\ldots$ $\\ldots d_{r(2)}(2) d_{1}(3) \\ldots d_{r(3)}(3) d_{1}(4) \\ldots$.","solution":"2. No. If $a$ were rational, its decimal expansion would be periodic from some point. Let $p$ be the number of decimals in the period. Since $f\\left(10^{2 p}\\right)$ has $2 n p$ zeros, it contains a full periodic part; hence the period would consist only of zeros, which is impossible.","problem_type":null,"tier":0} +{"year":"1986","problem_phase":"shortlisted","problem":"20. (CAN 3) Prove that the sum of the face angles at each vertex of a tetrahedron is a straight angle if and only if the faces are congruent triangles.","solution":"20. If the faces of the tetrahedron $A B C D$ are congruent triangles, we must have $A B=C D, A C=B D$, and $A D=B C$. Then the sum of angles at $A$ is $\\angle B A C+\\angle C A D+\\angle D A B=\\angle B D C+\\angle C B D+\\angle D C B=180^{\\circ}$. We now assume that the sum of angles at each vertex is $180^{\\circ}$. Let us construct triangles $B C D^{\\prime}, C A D^{\\prime \\prime}, A B D^{\\prime \\prime \\prime}$ in the plane $A B C$, exterior to $\\triangle A B C$, such that $\\triangle B C D^{\\prime} \\cong \\triangle B C D, \\triangle C A D^{\\prime \\prime} \\cong \\triangle C A D$, and $\\triangle A B D^{\\prime \\prime \\prime} \\cong \\triangle A B D$. Then by the assumption, $A \\in D^{\\prime \\prime} D^{\\prime \\prime \\prime}, B \\in D^{\\prime \\prime \\prime} D^{\\prime}$, and $C \\in D^{\\prime} D^{\\prime \\prime}$. Since also $D^{\\prime \\prime} A=D^{\\prime \\prime \\prime} A=D A$, etc., $A, B, C$ are the mid- points of segments $D^{\\prime \\prime} D^{\\prime \\prime \\prime}, D^{\\prime \\prime \\prime} D^{\\prime}, D^{\\prime} D^{\\prime \\prime}$ respectively. Thus the triangles $A B C, B C D^{\\prime}, C A D^{\\prime \\prime}, A B D^{\\prime \\prime \\prime}$ are congruent, and the statement follows.","problem_type":null,"tier":0} +{"year":"1986","problem_phase":"shortlisted","problem":"21. (TUR 2) Let $A B C D$ be a tetrahedron having each sum of opposite sides equal to 1. Prove that $$ r_{A}+r_{B}+r_{C}+r_{D} \\leq \\frac{\\sqrt{3}}{3} $$ where $r_{A}, r_{B}, r_{C}, r_{D}$ are the inradii of the faces, equality holding only if $A B C D$ is regular.","solution":"21. Since the sum of all edges of $A B C D$ is 3 , the statement of the problem is an immediate consequence of the following statement: Lemma. Let $r$ be the inradius of a triangle with sides $a, b, c$. Then $a+$ $b+c \\geq 6 \\sqrt{3} \\cdot r$, with equality if and only if the triangle is equilateral. Proof. If $S$ and $p$ denotes the area and semiperimeter of the triangle, by Heron's formula and the AM-GM inequality we have $$ \\begin{aligned} p r & =S=\\sqrt{p(p-a)(p-b)(p-c)} \\\\ & \\leq \\sqrt{p\\left(\\frac{(p-a)+(p-b)+(p-c)}{3}\\right)^{3}}=\\sqrt{\\frac{p^{4}}{27}}=\\frac{p^{2}}{3 \\sqrt{3}}, \\end{aligned} $$ i.e., $p \\geq 3 \\sqrt{3} \\cdot r$, which is equivalent to the claim.","problem_type":null,"tier":0} +{"year":"1986","problem_phase":"shortlisted","problem":"3. (USA 3) Let $A, B$, and $C$ be three points on the edge of a circular chord such that $B$ is due west of $C$ and $A B C$ is an equilateral triangle whose side is 86 meters long. A boy swam from $A$ directly toward $B$. After covering a distance of $x$ meters, he turned and swam westward, reaching the shore after covering a distance of $y$ meters. If $x$ and $y$ are both positive integers, determine $y$.","solution":"3. Let $E$ be the point where the boy turned westward, reaching the shore at $D$. Let the ray $D E$ cut $A C$ at $F$ and the shore again at $G$. Then $E F=$ $A E=x$ (because $A E F$ is an equilateral triangle) and $F G=D E=y$. From $A E \\cdot E B=D E \\cdot E G$ we obtain $x(86-x)=y(x+y)$. If $x$ is odd, then $x(86-x)$ is odd, while $y(x+y)$ is even. Hence $x$ is even, and so $y$ must also be even. Let $y=2 y_{1}$. The above equation can be rewritten as $$ \\left(x+y_{1}-43\\right)^{2}+\\left(2 y_{1}\\right)^{2}=\\left(43-y_{1}\\right)^{2} \\text {. } $$ Since $y_{1}<43$, we have $\\left(2 y_{1}, 43-y_{1}\\right)=1$, and thus $\\left(\\left|x+y_{1}-43\\right|, 2 y_{1}, 43-\\right.$ $\\left.y_{1}\\right)$ is a primitive Pythagorean triple. Consequently there exist integers $a>b>0$ such that $y_{1}=a b$ and $43-y_{1}=a^{2}+b^{2}$. We obtain that $a^{2}+b^{2}+a b=43$, which has the unique solution $a=6, b=1$. Hence $y=12$ and $x=2$ or $x=72$. Remark. The Diophantine equation $x(86-x)=y(x+y)$ can be also solved directly. Namely, we have that $x(344-3 x)=(2 y+x)^{2}$ is a square, and since $x$ is even, we have $(x, 344-3 x)=2$ or 4 . Consequently $x, 344-3 x$ are either both squares or both two times squares. The rest is easy.","problem_type":null,"tier":0} +{"year":"1986","problem_phase":"shortlisted","problem":"4. (CZS 3) Let $n$ be a positive integer and let $p$ be a prime number, $p>3$. Find at least $3(n+1)$ [easier version: $2(n+1)]$ sequences of positive integers $x, y, z$ satisfying $$ x y z=p^{n}(x+y+z) $$ that do not differ only by permutation.","solution":"4. Let $x=p^{\\alpha} x^{\\prime}, y=p^{\\beta} y^{\\prime}, z=p^{\\gamma} z^{\\prime}$ with $p \\nmid x^{\\prime} y^{\\prime} z^{\\prime}$ and $\\alpha \\geq \\beta \\geq \\gamma$. From the given equation it follows that $p^{n}(x+y)=z\\left(x y-p^{n}\\right)$ and consequently $z^{\\prime} \\mid x+y$. Since also $p^{\\gamma} \\mid x+y$, we have $z \\mid x+y$, i.e., $x+y=q z$. The given equation together with the last condition gives us $$ x y=p^{n}(q+1) \\quad \\text { and } \\quad x+y=q z . $$ Conversely, every solution of (1) gives a solution of the given equation. For $q=1$ and $q=2$ we obtain the following classes of $n+1$ solutions each: $$ \\begin{array}{ll} q=1:(x, y, z)=\\left(2 p^{i}, p^{n-i}, 2 p^{i}+p^{n-i}\\right) & \\text { for } i=0,1,2, \\ldots, n \\\\ q=2:(x, y, z)=\\left(3 p^{j}, p^{n-j}, \\frac{3 p^{j}+p^{n-j}}{2}\\right) & \\text { for } j=0,1,2, \\ldots, n \\end{array} $$ For $n=2 k$ these two classes have a common solution $\\left(2 p^{k}, p^{k}, 3 p^{k}\\right)$; otherwise, all these solutions are distinct. One further solution is given by $(x, y, z)=\\left(1, p^{n}\\left(p^{n}+3\\right) \/ 2, p^{2}+2\\right)$, not included in the above classes for $p>3$. Thus we have found $2(n+1)$ solutions. Another type of solution is obtained if we put $q=p^{k}+p^{n-k}$. This yields the solutions $$ (x, y, z)=\\left(p^{k}, p^{n}+p^{n-k}+p^{2 n-2 k}, p^{n-k}+1\\right) \\quad \\text { for } k=0,1, \\ldots, n $$ For $ky$ and $f(y)-y \\geq v \\geq f(x)-x$, then $f(z)=v+z$, for some number $z$ between $x$ and $y$. (ii) The equation $f(x)=0$ has at least one solution, and among the solutions of this equation, there is one that is not smaller than all the other solutions; (iii) $f(0)=1$. (iv) $f(1987) \\leq 1988$. (v) $f(x) f(y)=f(x f(y)+y f(x)-x y)$. Find $f(1987)$.","solution":"1. By (ii), $f(x)=0$ has at least one solution, and there is the greatest among them, say $x_{0}$. Then by (v), for any $x$, $$ 0=f(x) f\\left(x_{0}\\right)=f\\left(x f\\left(x_{0}\\right)+x_{0} f(x)-x_{0} x\\right)=f\\left(x_{0}(f(x)-x)\\right) $$ It follows that $x_{0} \\geq x_{0}(f(x)-x)$. Suppose $x_{0}>0$. By (i) and (iii), since $f\\left(x_{0}\\right)-x_{0}<01$ there are integers $e_{i}$ not all 0 and with $\\left|e_{i}\\right|0$ will occur exactly $p_{n-1}(k-1)$ times, so that the sum of the $d_{i}$ 's is $\\sum_{k=1}^{n} k p_{n-1}(k-1)=\\sum_{k=0}^{n-1}(k+$ 1) $p_{n-1}(k)=2(n-1)$ !. Summation over $i$ yields $$ Z=\\sum_{k=0}^{n} k^{2} p_{n}(k)=2 n!. $$ From (0), (1), and (2), we conclude that $$ \\sum_{k=0}^{n}(k-1)^{2} p_{n}(k)=\\sum_{k=0}^{n} k^{2} p_{n}(k)-2 \\sum_{k=0}^{n} k p_{n}(k)+\\sum_{k=0}^{n} p_{n}(k)=n! $$ Remark. Only the first part of this problem was given on the IMO.","problem_type":null,"tier":0} +{"year":"1987","problem_phase":"shortlisted","problem":"17. (ROM 1) Prove that there exists a four-coloring of the set $M=$ $\\{1,2, \\ldots, 1987\\}$ such that any arithmetic progression with 10 terms in the set $M$ is not monochromatic. Alternative formulation. Let $M=\\{1,2, \\ldots, 1987\\}$. Prove that there is a function $f: M \\rightarrow\\{1,2,3,4\\}$ that is not constant on every set of 10 terms from $M$ that form an arithmetic progression.","solution":"17. The number of 4 -colorings of the set $M$ is equal to $4^{1987}$. Let $A$ be the number of arithmetic progressions in $M$ with 10 terms. The number of colorings containing a monochromatic arithmetic progression with 10 terms is less than $4 A \\cdot 4^{1977}$. So, if $A<4^{9}$, then there exist 4 -colorings with the required property. Now we estimate the value of $A$. If the first term of a 10-term progression is $k$ and the difference is $d$, then $1 \\leq k \\leq 1978$ and $d \\leq\\left[\\frac{1987-k}{9}\\right]$; hence $$ A=\\sum_{k=1}^{1978}\\left[\\frac{1987-k}{9}\\right]<\\frac{1986+1985+\\cdots+9}{9}=\\frac{1995 \\cdot 1978}{18}<4^{9} $$","problem_type":null,"tier":0} +{"year":"1987","problem_phase":"shortlisted","problem":"18. (ROM 4) For any integer $r \\geq 1$, determine the smallest integer $h(r) \\geq 1$ such that for any partition of the set $\\{1,2, \\ldots, h(r)\\}$ into $r$ classes, there are integers $a \\geq 0,1 \\leq x \\leq y$, such that $a+x, a+y, a+x+y$ belong to the same class.","solution":"18. Note first that the statement that some $a+x, a+y, a+x+y$ belong to a class $C$ is equivalent to the following statement: (1) There are positive integers $p, q \\in C$ such that $p2 k$.","problem_type":null,"tier":0} +{"year":"1987","problem_phase":"shortlisted","problem":"19. (USS 2) Let $\\alpha, \\beta, \\gamma$ be positive real numbers such that $\\alpha+\\beta+\\gamma<\\pi$, $\\alpha+\\beta>\\gamma, \\beta+\\gamma>\\alpha, \\gamma+\\alpha>\\beta$. Prove that with the segments of lengths $\\sin \\alpha, \\sin \\beta, \\sin \\gamma$ we can construct a triangle and that its area is not greater than $$ \\frac{1}{8}(\\sin 2 \\alpha+\\sin 2 \\beta+\\sin 2 \\gamma) $$","solution":"19. The facts given in the problem allow us to draw a triangular pyramid with angles $2 \\alpha, 2 \\beta, 2 \\gamma$ at the top and lateral edges of length $1 \/ 2$. At the base there is a triangle whose side lengths are exactly $\\sin \\alpha, \\sin \\beta, \\sin \\gamma$. The area of this triangle does not exceed the sum of areas of the lateral sides, which equals $(\\sin 2 \\alpha+\\sin 2 \\beta+\\sin 2 \\gamma) \/ 8$.","problem_type":null,"tier":0} +{"year":"1987","problem_phase":"shortlisted","problem":"2. (USA 3) At a party attended by $n$ married couples, each person talks to everyone else at the party except his or her spouse. The conversations involve sets of persons or cliques $C_{1}, C_{2}, \\ldots, C_{k}$ with the following property: no couple are members of the same clique, but for every other pair of persons there is exactly one clique to which both members belong. Prove that if $n \\geq 4$, then $k \\geq 2 n$.","solution":"2. Let $d_{i}$ denote the number of cliques of which person $i$ is a member. Clearly $d_{i} \\geq 2$. We now distinguish two cases: (i) For some $i, d_{i}=2$. Suppose that $i$ is a member of two cliques, $C_{p}$ and $C_{q}$. Then $\\left|C_{p}\\right|=\\left|C_{q}\\right|=n$, since for each couple other than $i$ and his\/her spouse, one member is in $C_{p}$ and one in $C_{q}$. There are thus $(n-1)(n-2)$ pairs $(r, s)$ of nonspouse persons distinct from $i$, where $r \\in C_{p}, s \\in C_{q}$. We observe that each such pair accounts for a different clique. Otherwise, we find two members of $C_{p}$ or $C_{q}$ who belong to one other clique. It follows that $k \\geq 2+(n-1)(n-2) \\geq 2 n$ for $n \\geq 4$. (ii) For every $i, d_{i} \\geq 3$. Suppose that $k<2 n$. For $i=1,2, \\ldots, 2 n$ assign to person $i$ an indeterminant $x_{i}$, and for $j=1,2, \\ldots, k$ set $y=\\sum_{i \\in C_{j}} x_{i}$. From linear algebra, we know that if $k<2 n$, then there exist $x_{1}, x_{2}, \\ldots, x_{2 n}$, not all zero, such that $y_{1}=y_{2}=\\cdots=y_{k}=0$. On the other hand, suppose that $y_{1}=y_{2}=\\cdots=y_{k}=0$. Let $M$ be the set of the couples and $M^{\\prime}$ the set of all other pairs of persons. Then $$ \\begin{aligned} 0 & =\\sum_{j=1}^{k} y_{j}^{2}=\\sum_{i=1}^{2 n} d_{i} x_{i}^{2}+2 \\sum_{(i, j) \\in M^{\\prime}} x_{i} x_{j} \\\\ & =\\sum_{i=1}^{2 n}\\left(d_{i}-2\\right) x_{i}^{2}+\\left(x_{1}+x_{2}+\\cdots+x_{2 n}\\right)^{2}+\\sum_{(i, j) \\in M}\\left(x_{i}-x_{j}\\right)^{2} \\\\ & \\geq \\sum_{i=1}^{2 n} x_{i}^{2}>0 \\end{aligned} $$ if not all $x_{1}, x_{2}, \\ldots, x_{2 n}$ are zero, which is a contradiction. Hence $k \\geq$ $2 n$. Remark. The condition $n \\geq 4$ is essential. For a party attended by 3 couples $\\{(1,4),(2,5),(3,6)\\}$, there is a collection of 4 cliques satisfying the conditions: $\\{(1,2,3),(3,4,5),(5,6,1),(2,4,6)\\}$.","problem_type":null,"tier":0} +{"year":"1987","problem_phase":"shortlisted","problem":"20. (USS 3) ${ }^{\\text {IMO6 }}$ Let $f(x)=x^{2}+x+p, p \\in \\mathbb{N}$. Prove that if the numbers $f(0), f(1), \\ldots, f([\\sqrt{p \/ 3}])$ are primes, then all the numbers $f(0), f(1), \\ldots$, $f(p-2)$ are primes.","solution":"20. Let $y$ be the smallest nonnegative integer with $y \\leq p-2$ for which $f(y)$ is a composite number. Denote by $q$ the smallest prime divisor of $f(y)$. We claim that $y2 y$. Suppose the contrary, that $q \\leq 2 y$. Since $$ f(y)-f(x)=(y-x)(y+x+1) $$ we observe that $f(y)-f(q-1-y)=(2 y-q+1) q$, from which it follows that $f(q-1-y)$ is divisible by $q$. But by the assumptions, $q-1-yq+p-y-1 \\geq q . $$ Therefore $q \\geq 2 y+1$. Now, since $f(y)$, being composite, cannot be equal to $q$, and $q$ is its smallest prime divisor, we obtain that $f(y) \\geq q^{2}$. Consequently, $$ y^{2}+y+p \\geq q^{2} \\geq(2 y+1)^{2}=4 y^{2}+4 y+1 \\Rightarrow 3\\left(y^{2}+y\\right) \\leq p-1 $$ and from this we easily conclude that $y<\\sqrt{p \/ 3}$, which contradicts the condition of the problem. In this way, all the numbers $$ f(0), f(1), \\ldots, f(p-2) $$ must be prime.","problem_type":null,"tier":0} +{"year":"1987","problem_phase":"shortlisted","problem":"21. (USS 4) ${ }^{\\mathrm{IMO} 2}$ The prolongation of the bisector $A L(L \\in B C)$ in the acuteangled triangle $A B C$ intersects the circumscribed circle at point $N$. From point $L$ to the sides $A B$ and $A C$ are drawn the perpendiculars $L K$ and $L M$ respectively. Prove that the area of the triangle $A B C$ is equal to the area of the quadrilateral $A K N M$.","solution":"21. Let $P$ be the second point of intersection of segment $B C$ and the circle circumscribed about quadrilateral $A K L M$. Denote by $E$ the intersection point of the lines $K N$ and $B C$ and by $F$ the intersection point of the lines $M N$ and $B C$. Then $\\angle B C N=\\angle B A N$ and $\\angle M A L=$ $\\angle M P L$, as angles on the same arc. Since $A L$ is a bisector, $\\angle B C N=$ $\\angle B A L=\\angle M A L=\\angle M P L$, and ![](https:\/\/cdn.mathpix.com\/cropped\/2024_11_18_8e985d6b9c83aa3e9d0eg-507.jpg?height=452&width=414&top_left_y=813&top_left_x=866) consequently $P M \\| N C$. Similarly we prove $K P \\| B N$. Then the quadrilaterals $B K P N$ and $N P M C$ are trapezoids; hence $$ S_{B K E}=S_{N P E} \\quad \\text { and } \\quad S_{N P F}=S_{C M F} $$ Therefore $S_{A B C}=S_{A K N M}$.","problem_type":null,"tier":0} +{"year":"1987","problem_phase":"shortlisted","problem":"22. (VIE 3) ${ }^{\\mathrm{IMO} 4}$ Does there exist a function $f: \\mathbb{N} \\rightarrow \\mathbb{N}$, such that $f(f(n))=$ $n+1987$ for every natural number $n$ ?","solution":"22. Suppose that there exists such function $f$. Then we obtain $$ f(n+1987)=f(f(f(n)))=f(n)+1987 \\quad \\text { for all } n \\in \\mathbb{N} $$ and from here, by induction, $f(n+1987 t)=f(n)+1987 t$ for all $n, t \\in \\mathbb{N}$. Further, for any $r \\in\\{0,1, \\ldots, 1986\\}$, let $f(r)=1987 k+l, k, l \\in \\mathbb{N}$, $l \\leq 1986$. We have $$ r+1987=f(f(r))=f(l+1987 k)=f(l)+1987 k, $$ and consequently there are two possibilities: (i) $k=1 \\Rightarrow f(r)=l+1987$ and $f(l)=r$; (ii) $k=0 \\Rightarrow f(r)=l$ and $f(l)=r+1987$; in both cases, $r \\neq l$. In this way, the set $\\{0,1, \\ldots, 1986\\}$ decomposes to pairs $\\{a, b\\}$ such that $$ f(a)=b \\text { and } f(b)=a+1987, \\quad \\text { or } \\quad f(b)=a \\text { and } f(a)=b+1987 . $$ But the set $\\{0,1, \\ldots, 1986\\}$ has an odd number of elements, and cannot be decomposed into pairs. Contradiction.","problem_type":null,"tier":0} +{"year":"1987","problem_phase":"shortlisted","problem":"23. (YUG 2) Prove that for every natural number $k(k \\geq 2)$ there exists an irrational number $r$ such that for every natural number $m$, $$ \\left[r^{m}\\right] \\equiv-1 \\quad(\\bmod k) $$ Remark. An easier variant: Find $r$ as a root of a polynomial of second degree with integer coefficients.","solution":"23. If we prove the existence of $p, q \\in \\mathbb{N}$ such that the roots $r, s$ of $$ f(x)=x^{2}-k p \\cdot x+k q=0 $$ are irrational real numbers with $01$ ), then we are done, because from $r+s, r s \\equiv 0(\\bmod k)$ we get $r^{m}+s^{m} \\equiv 0$ $(\\bmod k)$, and $00>f(1)$, i.e., $$ k q>0>k(q-p)+1 \\quad \\Rightarrow \\quad p>q>0 $$ The irrationality of $r$ can be obtained by taking $q=p-1$, because the discriminant $D=(k p)^{2}-4 k p+4 k$, for $(k p-2)^{2}1$. Prove that for any integers $a_{1}, a_{2}, \\ldots, a_{m}$ and $b_{1}, b_{2}$, $\\ldots, b_{k}$ there must be two products $a_{i} b_{j}$ and $a_{s} b_{t}((i, j) \\neq(s, t))$ that give the same residue when divided by $m k$.","solution":"8. (a) Consider $$ a_{i}=i k+1, \\quad i=1,2, \\ldots, m ; \\quad b_{j}=j m+1, \\quad j=1,2, \\ldots, k $$ Assume that $m k \\mid a_{i} b_{j}-a_{s} b_{t}=(i k+1)(j m+1)-(s k+1)(t m+1)=$ $k m(i j-s t)+m(j-t)+k(i-s)$. Since $m$ divides this sum, we get that $m \\mid k(i-s)$, or, together with $\\operatorname{gcd}(k, m)=1$, that $i=s$. Similarly $j=t$, which proves part (a). (b) Suppose the opposite, i.e., that all the residues are distinct. Then the residue 0 must also occur, say at $a_{1} b_{1}: m k \\mid a_{1} b_{1}$; so, for some $a^{\\prime}$ and $b^{\\prime}, a^{\\prime}\\left|a_{1}, b^{\\prime}\\right| b_{1}$, and $a^{\\prime} b^{\\prime}=m k$. Assuming that for some $i, s \\neq i$, $a^{\\prime} \\mid a_{i}-a_{s}$, we obtain $m k=a^{\\prime} b^{\\prime} \\mid a_{i} b_{1}-a_{s} b_{1}$, a contradiction. This shows that $a^{\\prime} \\geq m$ and similarly $b^{\\prime} \\geq k$, and thus from $a^{\\prime} b^{\\prime}=m k$ we have $a^{\\prime}=m, b^{\\prime}=k$. We also get (1): all $a_{i}$ 's give distinct residues modulo $m=a^{\\prime}$, and all $b_{j}$ 's give distinct residues modulo $k=b^{\\prime}$. Now let $p$ be a common prime divisor of $m$ and $k$. By $(*)$, exactly $\\frac{p-1}{p} m$ of $a_{i}$ 's and exactly $\\frac{p-1}{p} k$ of $b_{j}$ 's are not divisible by $p$. Therefore there are precisely $\\frac{(p-1)^{2}}{p^{2}} m k$ products $a_{i} b_{j}$ that are not divisible by $p$, although from the assumption that they all give distinct residues it follows that the number of such products is $\\frac{p-1}{p} m k \\neq \\frac{(p-1)^{2}}{p^{2}} m k$. We have arrived at a contradiction, thus proving (b).","problem_type":null,"tier":0} +{"year":"1987","problem_phase":"shortlisted","problem":"9. (HUN 2) Does there exist a set $M$ in usual Euclidean space such that for every plane $\\lambda$ the intersection $M \\cap \\lambda$ is finite and nonempty?","solution":"9. The answer is yes. Consider the curve $$ C=\\left\\{(x, y, z) \\mid x=t, y=t^{3}, z=t^{5}, \\quad t \\in \\mathbb{R}\\right\\} $$ Any plane defined by an equation of the form $a x+b y+c z+d=0$ intersects the curve $C$ at points $\\left(t, t^{3}, t^{5}\\right)$ with $t$ satisfying $c t^{5}+b t^{3}+a t+d=0$. This last equation has at least one but only finitely many solutions.","problem_type":null,"tier":0} +{"year":"1988","problem_phase":"shortlisted","problem":"1. (BUL 1) An integer sequence is defined by $$ a_{n}=2 a_{n-1}+a_{n-2} \\quad(n>1), \\quad a_{0}=0, \\quad a_{1}=1 $$ Prove that $2^{k}$ divides $a_{n}$ if and only if $2^{k}$ divides $n$.","solution":"1. Assume that $p$ and $q$ are real and $b_{0}, b_{1}, b_{2}, \\ldots$ is a sequence such that $b_{n}=p b_{n-1}+q b_{n-2}$ for all $n>1$. From the equalities $b_{n}=p b_{n-1}+q b_{n-2}$, $b_{n+1}=p b_{n}+q b_{n-1}, b_{n+2}=p b_{n+1}+q b_{n}$, eliminating $b_{n+1}$ and $b_{n-1}$ we obtain that $b_{n+2}=\\left(p^{2}+2 q\\right) b_{n}-q^{2} b_{n-2}$. So the sequence $b_{0}, b_{2}, b_{4}, \\ldots$ has the property $$ b_{2 n}=P b_{2 n-2}+Q b_{2 n-4}, \\quad P=p^{2}+2 q, \\quad Q=-q^{2} . $$ We shall solve the problem by induction. The sequence $a_{n}$ has $p=2$, $q=1$, and hence $P=6, Q=-1$. Let $k=1$. Then $a_{0}=0, a_{1}=1$, and $a_{n}$ is of the same parity as $a_{n-2}$; i.e., it is even if and only if $n$ is even. Let $k \\geq 1$. We assume that for $n=2^{k} m$, the numbers $a_{n}$ are divisible by $2^{k}$, but divisible by $2^{k+1}$ if and only if $m$ is even. We assume also that the sequence $c_{0}, c_{1}, \\ldots$, with $c_{m}=a_{m \\cdot 2^{k}}$, satisfies the condition $c_{n}=$ $p c_{n-1}-c_{n-2}$, where $p \\equiv 2(\\bmod 4)($ for $k=1$ it is true). We shall prove the same statement for $k+1$. According to (1), $c_{2 n}=P c_{2 n-2}-c_{2 n-4}$, where $P=p^{2}-2$. Obviously $P \\equiv 2(\\bmod 4)$. Since $P=4 s+2$ for some integer $s$, and $c_{2 n}=2^{k+1} d_{2 n}, c_{0}=0, c_{1} \\equiv 2^{k}\\left(\\bmod 2^{k+1}\\right)$, and $c_{2}=p c_{1} \\equiv 2^{k+1}$ $\\left(\\bmod 2^{k+2}\\right)$, we have $$ c_{2 n}=(4 s+2) 2^{k+1} d_{2 n-2}-c_{2 n-4} \\equiv c_{2 n-4}\\left(\\bmod 2^{k+2}\\right) $$ i.e., $0 \\equiv c_{0} \\equiv c_{4} \\equiv c_{8} \\equiv \\cdots$ and $2^{k+1} \\equiv c_{2} \\equiv c_{6} \\equiv \\cdots\\left(\\bmod 2^{k+2}\\right)$, which proves the statement. Second solution. The recursion is solved by $$ a_{n}=\\frac{1}{2 \\sqrt{2}}\\left((1+\\sqrt{2})^{n}-(1-\\sqrt{2})^{n}\\right)=\\binom{n}{1}+2\\binom{n}{3}+2^{2}\\binom{n}{5}+\\cdots . $$ Let $n=2^{k} m$ with $m$ odd; then for $p>0$ the summand $$ 2^{p}\\binom{n}{2 p+1}=2^{k+p} m \\frac{(n-1) \\ldots(n-2 p)}{(2 p+1)!}=2^{k+p} \\frac{m}{2 p+1}\\binom{n-1}{2 p} $$ is divisible by $2^{k+p}$, because the denominator $2 p+1$ is odd. Hence $$ a_{n}=n+\\sum_{p>0} 2^{p}\\binom{n}{2 p+1}=2^{k} m+2^{k+1} N $$ for some integer $N$, so that $a_{n}$ is exactly divisible by $2^{k}$. Third solution. It can be proven by induction that $a_{2 n}=2 a_{n}\\left(a_{n}+a_{n+1}\\right)$. The required result follows easily, again by induction on $k$.","problem_type":null,"tier":0} +{"year":"1988","problem_phase":"shortlisted","problem":"10. (GDR 1) Let $N=\\{1,2, \\ldots, n\\}, n \\geq 2$. A collection $F=\\left\\{A_{1}, \\ldots, A_{t}\\right\\}$ of subsets $A_{i} \\subseteq N, i=1, \\ldots, t$, is said to be separating if for every pair $\\{x, y\\} \\subseteq N$, there is a set $A_{i} \\in F$ such that $A_{i} \\cap\\{x, y\\}$ contains just one element. A collection $F$ is said to be covering if every element of $N$ is contained in at least one set $A_{i} \\in F$. What is the smallest value $f(n)$ of $t$ such that there is a set $F=\\left\\{A_{1}, \\ldots, A_{t}\\right\\}$ that is simultaneously separating and covering?","solution":"10. We claim that if the family $\\left\\{A_{1}, \\ldots, A_{t}\\right\\}$ separates the $n$-set $N$, then $2^{t} \\geq n$. The proof goes by induction. The case $t=1$ is clear, so suppose that the claim holds for $t-1$. Since $A_{t}$ does not separate elements of its own or its complement, it follows that $\\left\\{A_{1}, \\ldots, A_{t-1}\\right\\}$ is separating for both $A_{t}$ and $N \\backslash A_{t}$, so that $\\left|A_{t}\\right|,\\left|N \\backslash A_{t}\\right| \\leq 2^{t-1}$. Then $|N| \\leq 2 \\cdot 2^{t-1}=2^{t}$, as claimed. Also, if the set $N$ with $N=2^{t}$ is separated by $\\left\\{A_{1}, \\ldots, A_{t}\\right\\}$, then (precisely) one element of $N$ is not covered. To show this, we again use induction. This is trivial for $t=1$, so let $t \\geq 1$. Since $A_{1}, \\ldots, A_{t-1}$ separate both $A_{t}$ and $N \\backslash A_{t}, N \\backslash A_{t}$ must have exactly $2^{t-1}$ elements, and thus one of its elements is not covered by $A_{1}, \\ldots, A_{t-1}$, and neither is covered by $A_{t}$. We conclude that a separating and covering family of $t$ subsets can exist only if $n \\leq 2^{t}-1$. We now construct such subsets for the set $N$ if $2^{t-1} \\leq n \\leq 2^{t}-1, t \\geq 1$. For $t=1$, put $A_{1}=\\{1\\}$. In the step from $t$ to $t+1$, let $N=N^{\\prime} \\cup N^{\\prime \\prime} \\cup\\{y\\}$, where $\\left|N^{\\prime}\\right|,\\left|N^{\\prime \\prime}\\right| \\leq 2^{t-1}$; let $A_{1}^{\\prime}, \\ldots, A_{t}^{\\prime}$ be subsets covering and separating $N^{\\prime}$ and $A_{1}^{\\prime \\prime}, \\ldots, A_{t}^{\\prime \\prime}$ such subsets for $N^{\\prime \\prime}$. Then the subsets $A_{i}=A_{i}^{\\prime} \\cup A_{i}^{\\prime \\prime}$ $(i=1, \\ldots, t)$ and $A_{t+1}=N^{\\prime \\prime} \\cup\\{y\\}$ obviously separate and cover $N$. The answer: $t=\\left[\\log _{2} n\\right]+1$. Second solution. Suppose that the sets $A_{1}, \\ldots, A_{t}$ cover and separate $N$. Label each element $x \\in N$ with a string of $\\left(x_{1} x_{2} \\ldots x_{t}\\right)$ of 0 's and 1's, where $x_{i}$ is 1 when $x \\in A_{i}, 0$ otherwise. Since the $A_{i}$ 's separate, these strings are distinct; since they cover, the string ( $00 \\ldots 0$ ) does not occur. Hence $n \\leq 2^{t}-1$. Conversely, for $2^{t-1} \\leq n<2^{t}$, represent the elements of $N$ in base 2 as strings of 0's and 1's of length $t$. For $1 \\leq i \\leq t$, take $A_{i}$ to be the set of numbers in $N$ whose binary string has a 1 in the $i$ th place. These sets clearly cover and separate.","problem_type":null,"tier":0} +{"year":"1988","problem_phase":"shortlisted","problem":"11. (GDR 3) The lock on a safe consists of three wheels, each of which may be set in eight different positions. Due to a defect in the safe mechanism the door will open if any two of the three wheels are in the correct position. What is the smallest number of combinations that must be tried if one is to guarantee being able to open the safe (assuming that the \"right combination\" is not known)?","solution":"11. The answer is 32 . Write the combinations as triples $k=(x, y, z), 0 \\leq$ $x, y, z \\leq 7$. Define the sets $K_{1}=\\{(1,0,0),(0,1,0),(0,0,1),(1,1,1)\\}$, $K_{2}=\\{(2,0,0),(0,2,0),(0,0,2),(2,2,2)\\}, K_{3}=\\{(0,0,0),(4,4,4)\\}$, and $K=\\left\\{k=k_{1}+k_{2}+k_{3} \\mid k_{i} \\in K_{i}, i=1,2,3\\right\\}$. There are 32 combinations in $K$. We shall prove that these combinations will open the safe in every case. Let $t=(a, b, c)$ be the right combination. Set $k_{3}=(0,0,0)$ if at least two of $a, b, c$ are less than 4 , and $k_{3}=(4,4,4)$ otherwise. In either case, the difference $t-k_{3}$ contains two nonnegative elements not greater than 3 . Choosing a suitable $k_{2}$ we can achieve that $t-k_{3}-k_{2}$ contains two elements that are 0,1 . So, there exists $k_{1}$ such that $t-k_{3}-k_{2}-k_{1}=t-k$ contains two zeros, for $k \\in K$. This proves that 32 is sufficient. Suppose that $K$ is a set of at most 31 combinations. We say that $k \\in K$ covers the combination $k_{1}$ if $k$ and $k_{1}$ differ in at most one position. One of the eight sets $M_{i}=\\{(i, y, z) \\mid 0 \\leq y, z \\leq 7\\}, i=0,1, \\ldots, 7$, contains at most three elements of $K$. Suppose w.l.o.g. that this is $M_{0}$. Further, among the eight sets $N_{j}=\\{(0, j, z) \\mid 0 \\leq z \\leq 7\\}, j=0, \\ldots, 7$, there are at least five, say w.l.o.g. $N_{0}, \\ldots, N_{4}$, not containing any of the combinations from $K$. Of the 40 elements of the set $N=\\{(0, y, z) \\mid 0 \\leq y \\leq 4,0 \\leq z \\leq 7\\}$, at most $5 \\cdot 3=15$ are covered by $K \\cap M_{0}$, and at least 25 aren't. Consequently, the intersection of $K$ with $L=\\{(x, y, z) \\mid 1 \\leq x \\leq 7,0 \\leq y \\leq 4,0 \\leq z \\leq 7\\}$ contains at least 25 elements. So $K$ has at most $31-25=6$ elements in the set $P=\\{(x, y, z) \\mid 0 \\leq x \\leq 7,5 \\leq y \\leq 7,0 \\leq z \\leq 7\\}$. This implies that for some $j \\in\\{5,6,7\\}$, say w.l.o.g. $j=7, K$ contains at most two elements in $Q_{j}=\\{(x, y, z) \\mid 0 \\leq x, z \\leq 7, y=j\\}$; denote them by $l_{1}, l_{2}$. Of the 64 elements of $Q_{7}$, at most 30 are covered by $l_{1}$ and $l_{2}$. But then there remain 34 uncovered elements, which must be covered by different elements of $K \\backslash Q_{7}$, having itself less at most 29 elements. Contradiction.","problem_type":null,"tier":0} +{"year":"1988","problem_phase":"shortlisted","problem":"12. (GRE 2) In a triangle $A B C$, choose any points $K \\in B C, L \\in A C$, $M \\in A B, N \\in L M, R \\in M K$, and $F \\in K L$. If $E_{1}, E_{2}, E_{3}, E_{4}, E_{5}$, $E_{6}$, and $E$ denote the areas of the triangles $A M R, C K R, B K F, A L F$, $B N M, C L N$, and $A B C$ respectively, show that $$ E \\geq 8 \\sqrt[6]{E_{1} E_{2} E_{3} E_{4} E_{5} E_{6}} $$ Remark. Points $K, L, M, N, R, F$ lie on segments $B C, A C, A B, L M$, $M K, K L$ respectively.","solution":"12. Let $E(X Y Z)$ stand for the area of a triangle $X Y Z$. We have $$ \\begin{gathered} \\frac{E_{1}}{E}=\\frac{E(A M R)}{E(A M K)} \\cdot \\frac{E(A M K)}{E(A B K)} \\cdot \\frac{E(A B K)}{E(A B C)}=\\frac{M R}{M K} \\cdot \\frac{A M}{A B} \\cdot \\frac{B K}{B C} \\Rightarrow \\\\ \\left(\\frac{E_{1}}{E}\\right)^{1 \/ 3} \\leq \\frac{1}{3}\\left(\\frac{M R}{M K}+\\frac{A M}{A B}+\\frac{B K}{B C}\\right) \\end{gathered} $$ We similarly obtain $$ \\left(\\frac{E_{2}}{E}\\right)^{1 \/ 3} \\leq \\frac{1}{3}\\left(\\frac{K R}{M K}+\\frac{B M}{A B}+\\frac{C K}{B C}\\right) $$ Therefore $\\left(E_{1} \/ E\\right)^{1 \/ 3}+\\left(E_{2} \/ E\\right)^{1 \/ 3} \\leq 1$, i.e., $\\sqrt[3]{E_{1}}+\\sqrt[3]{E_{2}} \\leq \\sqrt[3]{E}$. Analogously, $\\sqrt[3]{E_{3}}+\\sqrt[3]{E_{4}} \\leq \\sqrt[3]{E}$ and $\\sqrt[3]{E_{5}}+\\sqrt[3]{E_{6}} \\leq \\sqrt[3]{E}$; hence $$ \\begin{aligned} & 8 \\sqrt[6]{E_{1} E_{2} E_{3} E_{4} E_{5} E_{6}} \\\\ & \\quad=2\\left(\\sqrt[3]{E_{1}} \\sqrt[3]{E_{2}}\\right)^{1 \/ 2} \\cdot 2\\left(\\sqrt[3]{E_{3}} \\sqrt[3]{E_{4}}\\right)^{1 \/ 2} \\cdot 2\\left(\\sqrt[3]{E_{5}} \\sqrt[3]{E_{6}}\\right)^{1 \/ 2} \\\\ & \\quad \\leq\\left(\\sqrt[3]{E_{1}}+\\sqrt[3]{E_{2}}\\right) \\cdot\\left(\\sqrt[3]{E_{3}}+\\sqrt[3]{E_{4}}\\right) \\cdot\\left(\\sqrt[3]{E_{5}}+\\sqrt[3]{E_{6}}\\right) \\leq E \\end{aligned} $$","problem_type":null,"tier":0} +{"year":"1988","problem_phase":"shortlisted","problem":"13. (GRE 3) ${ }^{\\mathrm{IMO5}}$ In a right-angled triangle $A B C$, let $A D$ be the altitude drawn to the hypotenuse and let the straight line joining the incenters of the triangles $A B D, A C D$ intersect the sides $A B, A C$ at the points $K, L$ respectively. If $E$ and $E_{1}$ denote the areas of the triangles $A B C$ and $A K L$ respectively, show that $\\frac{E}{E_{1}} \\geq 2$.","solution":"13. Let $A B=c, A C=b, \\angle C B A=\\beta, B C=a$, and $A D=h$. Let $r_{1}$ and $r_{2}$ be the inradii of $A B D$ and $A D C$ respectively and $O_{1}$ and $O_{2}$ the centers of the respective incircles. We obviously have $r_{1} \/ r_{2}=$ $c \/ b$. We also have $D O_{1}=\\sqrt{2} r_{1}$, $D O_{2}=\\sqrt{2} r_{2}$, and $\\angle O_{1} D A=$ $\\angle O_{2} D A=45^{\\circ}$. Hence $\\angle O_{1} D O_{2}=$ $90^{\\circ}$ and $D O_{1} \/ D O_{2}=c \/ b$ from which it follows that $\\triangle O_{1} D O_{2} \\sim$ $\\triangle B A C$. ![](https:\/\/cdn.mathpix.com\/cropped\/2024_11_18_8e985d6b9c83aa3e9d0eg-516.jpg?height=308&width=520&top_left_y=648&top_left_x=831) We now define $P$ as the intersection of the circumcircle of $\\triangle O_{1} D O_{2}$ with $D A$. From the above similarity we have $\\angle D P O_{2}=\\angle D O_{1} O_{2}=\\beta=$ $\\angle D A C$. It follows that $P O_{2} \\| A C$ and from $\\angle O_{1} P O_{2}=90^{\\circ}$ it also follows that $P O_{1} \\| A B$. We also have $\\angle P O_{1} O_{2}=\\angle P O_{2} O_{1}=45^{\\circ}$; hence $\\angle L K A=\\angle K L A=45^{\\circ}$, and thus $A K=A L$. From $\\angle O_{1} K A=\\angle O_{1} D A=$ $45^{\\circ}, O_{1} A=O_{1} A$, and $\\angle O_{1} K A=\\angle O_{1} D A$ we have $\\triangle O_{1} K A \\cong \\triangle O_{1} D A$ and hence $A L=A K=A D=h$. Thus $$ \\frac{E}{E_{1}}=\\frac{a h \/ 2}{h^{2} \/ 2}=\\frac{a}{h}=\\frac{a^{2}}{a h}=\\frac{b^{2}+c^{2}}{b c} \\geq 2 . $$ Remark. It holds that for an arbitrary triangle $A B C, A K=A L$ if and only if $A B=A C$ or $\\measuredangle B A C=90^{\\circ}$.","problem_type":null,"tier":0} +{"year":"1988","problem_phase":"shortlisted","problem":"14. (HUN 1) For what values of $n$ does there exist an $n \\times n$ array of entries $-1,0$, or 1 such that the $2 n$ sums obtained by summing the elements of the rows and the columns are all different?","solution":"14. Consider an array $\\left[a_{i j}\\right]$ of the given property and denote the sums of the rows and the columns by $r_{i}$ and $c_{j}$ respectively. Among the $r_{i}$ 's and $c_{j}$ 's, one element of $[-n, n]$ is missing, so that there are at least $n$ nonnegative and $n$ nonpositive sums. By permuting rows and columns we can obtain an array in which $r_{1}, \\ldots, r_{k}$ and $c_{1}, \\ldots, c_{n-k}$ are nonnegative. Clearly $$ \\sum_{i=1}^{n}\\left|r_{i}\\right|+\\sum_{j=1}^{n}\\left|c_{j}\\right| \\geq \\sum_{r=-n}^{n}|r|-n=n^{2} $$ But on the other hand, $$ \\begin{aligned} \\sum_{i=1}^{n}\\left|r_{i}\\right|+\\sum_{j=1}^{n}\\left|c_{j}\\right| & =\\sum_{i=1}^{k} r_{i}-\\sum_{i=k+1}^{n} r_{i}+\\sum_{j=1}^{n-k} c_{j}-\\sum_{j=n-k+1}^{n} c_{j}= \\\\ & =\\sum_{i \\leq k} a_{i j}-\\sum_{i>k} a_{i j}+\\sum_{j \\leq n-k} a_{i j}-\\sum_{j>n-k} a_{i j}= \\\\ & =2 \\sum_{i=1}^{k} \\sum_{j=1}^{n-k} a_{i j}-2 \\sum_{i=k+1}^{n} \\sum_{j=n-k+1}^{n} a_{i j} \\leq 4 k(n-k) . \\end{aligned} $$ This yields $n^{2} \\leq 4 k(n-k)$, i.e., $(n-2 k)^{2} \\leq 0$, and thus $n$ must be even. We proceed to show by induction that for all even $n$ an array of the given type exists. For $n=2$ the array in Fig. 1 is good. Let such an $n \\times n$ array be given for some even $n \\geq 2$, with $c_{1}=n, c_{2}=-n+1, c_{3}=$ $n-2, \\ldots, c_{n-1}=2, c_{n}=-1$ and $r_{1}=n-1, r_{2}=-n+2, \\ldots, r_{n-1}=1$, $r_{n}=0$. Upon enlarging this array as indicated in Fig. 2, the positive sums are increased by 2 , the nonpositive sums are decreased by 2 , and the missing sums $-1,0,1,2$ occur in the new rows and columns, so that the obtained array $(n+2) \\times(n+2)$ is of the same type. ![](https:\/\/cdn.mathpix.com\/cropped\/2024_11_18_8e985d6b9c83aa3e9d0eg-517.jpg?height=89&width=87&top_left_y=1080&top_left_x=487) Fig. 1 | $n \\times n$ | | | 1 | 1 | | :---: | :---: | :---: | :---: | :---: | | | | | | | | | | | 1 | 1 | | | | | - | | -1 | | 1)-1 | 1 | - | 1 | 0 | | 1)-1 | | 1. | 1 | -1 | Fig. 2","problem_type":null,"tier":0} +{"year":"1988","problem_phase":"shortlisted","problem":"15. (ICE 1) Let $A B C$ be an acute-angled triangle. Three lines $L_{A}, L_{B}$, and $L_{C}$ are constructed through the vertices $A, B$, and $C$ respectively according to the following prescription: Let $H$ be the foot of the altitude drawn from the vertex $A$ to the side $B C$; let $S_{A}$ be the circle with diameter $A H$; let $S_{A}$ meet the sides $A B$ and $A C$ at $M$ and $N$ respectively, where $M$ and $N$ are distinct from $A$; then $L_{A}$ is the line through $A$ perpendicular to $M N$. The lines $L_{B}$ and $L_{C}$ are constructed similarly. Prove that $L_{A}$, $L_{B}$, and $L_{C}$ are concurrent.","solution":"15. Referring to the description of $L_{A}$, we have $\\angle A M N=\\angle A H N=90^{\\circ}-$ $\\angle H A C=\\angle C$, and similarly $\\angle A N M=\\angle B$. Since the triangle $A B C$ is acute-angled, the line $L_{A}$ lies inside the angle $A$. Hence if $P=L_{A} \\cap B C$ and $Q=L_{B} \\cap A C$, we get $\\angle B A P=90^{\\circ}-\\angle C$; hence $A P$ passes through the circumcenter $O$ of $\\triangle A B C$. Similarly we prove that $L_{B}$ and $L_{C}$ contains the circumcenter $O$ also. It follows that $L_{A}, L_{B}$ and $L_{C}$ intersect at the point $O$. Remark. Without identifying the point of intersection, one can prove the concurrence of the three lines using Ceva's theorem, in usual or trigonometric form.","problem_type":null,"tier":0} +{"year":"1988","problem_phase":"shortlisted","problem":"16. (IRE 1) $)^{\\mathrm{IMO} 4}$ Show that the solution set of the inequality $$ \\sum_{k=1}^{70} \\frac{k}{x-k} \\geq \\frac{5}{4} $$ is a union of disjoint intervals the sum of whose lengths is 1988.","solution":"16. Let $f(x)=\\sum_{k=1}^{70} \\frac{k}{x-k}$. For all integers $i=1, \\ldots, 70$ we have that $f(x)$ tends to plus infinity as $x$ tends downward to $i$, and $f(x)$ tends to minus infinity as $x$ tends upward to $i$. As $x$ tends to infinity, $f(x)$ tends to 0 . Hence it follows that there exist $x_{1}, x_{2}, \\ldots, x_{70}$ such that $1r)$ with center $O$. Fix $P$ on the small circle and consider the variable chord $P A$ of the small circle. Points $B$ and $C$ lie on the large circle; $B, P, C$ are collinear and $B C$ is perpendicular to $A P$. (a) For what value(s) of $\\angle O P A$ is the sum $B C^{2}+C A^{2}+A B^{2}$ extremal? (b) What are the possible positions of the midpoints $U$ of $B A$ and $V$ of $A C$ as $\\angle O P A$ varies?","solution":"18. (i) Define $\\angle A P O=\\phi$ and $S=A B^{2}+A C^{2}+B C^{2}$. We calculate $P A=$ $2 r \\cos \\phi$ and $P B, P C=\\sqrt{R^{2}-r^{2} \\cos ^{2} \\phi} \\pm r \\sin \\phi$. We also have $A B^{2}=$ $P A^{2}+P B^{2}, A C^{2}=P A^{2}+P C^{2}$ and $B C=B P+P C$. Combining all these we obtain $$ \\begin{aligned} S & =A B^{2}+A C^{2}+B C^{2}=2\\left(P A^{2}+P B^{2}+P C^{2}+P B \\cdot P C\\right) \\\\ & =2\\left(4 r^{2} \\cos ^{2} \\phi+2\\left(R^{2}-r^{2} \\cos ^{2} \\phi+r^{2} \\sin ^{2} \\phi\\right)+R^{2}-r^{2}\\right) \\\\ & =6 R^{2}+2 r^{2} . \\end{aligned} $$ Hence it follows that $S$ is constant; i.e., it does not depend on $\\phi$. (ii) Let $B_{1}$ and $C_{1}$ respectively be points such that $A P B B_{1}$ and $A P C C_{1}$ are rectangles. It is evident that $B_{1}$ and $C_{1}$ lie on the larger circle and that $\\overrightarrow{P U}=\\frac{1}{2} \\overrightarrow{P B_{1}}$ and $\\overrightarrow{P V}=\\frac{1}{2} \\overrightarrow{P C_{1}}$. It is evident that we can arrange for an arbitrary point on the larger circle to be $B_{1}$ or $C_{1}$. Hence, the locus of $U$ and $V$ is equal to the circle obtained when the larger circle is shrunk by a factor of $1 \/ 2$ with respect to point $P$.","problem_type":null,"tier":0} +{"year":"1988","problem_phase":"shortlisted","problem":"19. (MEX 1) Let $f(n)$ be a function defined on the set of all positive integers and having its values in the same set. Suppose that $f(f(m)+f(n))=m+n$ for all positive integers $n, m$. Find all possible values for $f(1988)$.","solution":"19. We will show that $f(n)=n$ for every $n$ (thus also $f(1988)=1988$ ). Let $f(1)=r$ and $f(2)=s$. We obtain respectively the following equalities: $f(2 r)=f(r+r)=2 ; f(2 s)=f(s+s)=4 ; f(4)=f(2+2)=4 r ; f(8)=$ $f(4+4)=4 s ; f(5 r)=f(4 r+r)=5 ; f(r+s)=3 ; f(8)=f(5+3)=6 r+s$. Then $4 s=6 r+s$, which means that $s=2 r$. Now we prove by induction that $f(n r)=n$ and $f(n)=n r$ for every $n \\geq 4$. First we have that $f(5)=f(2+3)=3 r+s=5 r$, so that the statement is true for $n=4$ and $n=5$. Suppose that it holds for $n-1$ and $n$. Then $f(n+1)=f(n-1+2)=(n-1) r+2 r=(n+1) r$, and $f((n+1) r)=f((n-1) r+2 r)=(n-1)+2=n+1$. This completes the induction. Since $4 r \\geq 4$, we have that $f(4 r)=4 r^{2}$, and also $f(4 r)=4$. Then $r=1$, and consequently $f(n)=n$ for every natural number $n$. Second solution. $f(f(1)+n+m)=f(f(1)+f(f(n)+f(m)))=1+f(n)+$ $f(m)$, so $f(n)+f(m)$ is a function of $n+m$. Hence $f(n+1)+f(1)=$ $f(n)+f(2)$ and $f(n+1)-f(n)=f(2)-f(1)$, implying that $f(n)=A n+B$ for some constants $A, B$. It is easy to check that $A=1, B=0$ is the only possibility.","problem_type":null,"tier":0} +{"year":"1988","problem_phase":"shortlisted","problem":"2. (BUL 3) Let $n$ be a positive integer. Find the number of odd coefficients of the polynomial $$ u_{n}(x)=\\left(x^{2}+x+1\\right)^{n} $$","solution":"2. For polynomials $f(x), g(x)$ with integer coefficients, we use the notation $f(x) \\sim g(x)$ if all the coefficients of $f-g$ are even. Let $n=2^{s}$. It is immediately shown by induction that $\\left(x^{2}+x+1\\right)^{2^{s}} \\sim x^{2^{s+1}}+x^{2^{s}}+1$, and the required number for $n=2^{s}$ is 3 . Let $n=2^{s}-1$. If $s$ is odd, then $n \\equiv 1(\\bmod 3)$, while for $s$ even, $n \\equiv 0$ $(\\bmod 3)$. Consider the polynomial $$ R_{s}(x)= \\begin{cases}(x+1)\\left(x^{2 n-1}+x^{2 n-4}+\\cdots+x^{n+3}\\right)+x^{n+1} & \\\\ +x^{n}+x^{n-1}+(x+1)\\left(x^{n-4}+x^{n-7}+\\cdots+1\\right), & 2 \\nmid s \\\\ (x+1)\\left(x^{2 n-1}+x^{2 n-4}+\\cdots+x^{n+2}\\right)+x^{n} & \\\\ +(x+1)\\left(x^{n-3}+x^{n-6}+\\cdots+1\\right), & 2 \\mid s\\end{cases} $$ It is easily checked that $\\left(x^{2}+x+1\\right) R_{s}(x) \\sim x^{2^{s+1}}+x^{2^{s}}+1 \\sim\\left(x^{2}+x+1\\right)^{2^{s}}$, so that $R_{s}(x) \\sim\\left(x^{2}+x+1\\right)^{2^{s}-1}$. In this case, the number of odd coefficients is $\\left(2^{s+2}-(-1)^{s}\\right) \/ 3$. Now we pass to the general case. Let the number $n$ be represented in the binary system as $$ n=\\underbrace{11 \\ldots 1}_{a_{k}} \\underbrace{00 \\ldots 0}_{b_{k}} \\underbrace{11 \\ldots 1}_{a_{k-1}} \\underbrace{00 \\ldots 0}_{b_{k-1}} \\cdots \\underbrace{11 \\ldots 1}_{a_{1}} \\underbrace{00 \\ldots 0}_{b_{1}} $$ $b_{i}>0(i>1), b_{1} \\geq 0$, and $a_{i}>0$. Then $n=\\sum_{i=1}^{k} 2^{s_{i}}\\left(2^{a_{i}}-1\\right)$, where $s_{i}=b_{1}+a_{1}+b_{2}+a_{2}+\\cdots+b_{i}$, and hence $$ u_{n}(x)=\\left(x^{2}+x+1\\right)^{n}=\\prod_{i=1}^{k}\\left(x^{2}+x+1\\right)^{2^{s_{i}}\\left(2^{a_{i}}-1\\right)} \\sim \\prod_{i=1}^{k} R_{a_{i}}\\left(x^{2^{s_{i}}}\\right) $$ Let $R_{a_{i}}\\left(x^{2^{s_{i}}}\\right) \\sim x^{r_{i, 1}}+\\cdots+x^{r_{i, d_{i}}}$; clearly $r_{i, j}$ is divisible by $2^{s_{i}}$ and $r_{i, j} \\leq 2^{s_{i}+1}\\left(2^{a_{i}}-1\\right)<2^{s_{i+1}}$, so that for any $j, r_{i, j}$ can have nonzero binary digits only in some position $t, s_{i} \\leq t \\leq s_{i+1}-1$. Therefore, in $$ \\prod_{i=1}^{k} R_{a_{i}}\\left(x^{2^{s_{i}}}\\right) \\sim \\prod_{i=1}^{k}\\left(x^{r_{i, 1}}+\\cdots+x^{r_{i, d_{i}}}\\right)=\\sum_{i=1}^{k} \\sum_{p_{i}=1}^{d_{i}} x^{r_{1, p_{1}}+r_{2, p_{2}}+\\cdots+r_{k, p_{k}}} $$ all the exponents $r_{1, p_{1}}+r_{2, p_{2}}+\\cdots+r_{k, p_{k}}$ are different, so that the number of odd coefficients in $u_{n}(x)$ is $$ \\prod_{i=1}^{k} d_{i}=\\prod_{i=1}^{k} \\frac{2^{a_{i}+2}-(-1)^{a_{i}}}{3} $$","problem_type":null,"tier":0} +{"year":"1988","problem_phase":"shortlisted","problem":"20. (MON 4) Find the least natural number $n$ such that if the set $\\{1,2, \\ldots, n\\}$ is arbitrarily divided into two nonintersecting subsets, then one of the subsets contains three distinct numbers such that the product of two of them equals the third.","solution":"20. Suppose that $A_{n}=\\{1,2, \\ldots, n\\}$ is partitioned into $B_{n}$ and $C_{n}$, and that neither $B_{n}$ nor $C_{n}$ contains 3 distinct numbers one of which is equal to the product of the other two. If $n \\geq 96$, then the divisors of 96 must be split up. Let w.l.o.g. $2 \\in B_{n}$. There are four cases. (i) $3 \\in B_{n}, 4 \\in B_{n}$. Then $6,8,12 \\in C_{n} \\Rightarrow 48,96 \\in B_{n}$. A contradiction for $96=2 \\cdot 48$. (ii) $3 \\in B_{n}, 4 \\in C_{n}$. Then $6 \\in C_{n}, 24 \\in B_{n}, 8,12,48 \\in C_{n}$. A contradiction for $48=6 \\cdot 8$. (iii) $3 \\in C_{n}, 4 \\in B_{n}$. Then $8 \\in C_{n}, 24 \\in B_{n}, 6,48 \\in C_{n}$. A contradiction for $48=6 \\cdot 8$. (iv) $3 \\in C_{n}, 4 \\in C_{n}$. Then $12 \\in B_{n}, 6,24 \\in C_{n}$. A contradiction for $24=4 \\cdot 6$. If $n=95$, there is a very large number of ways of partitioning $A_{n}$. For example, $B_{n}=\\left\\{1, p, p^{2}, p^{3} q^{2}, p^{4} q, p^{2} q r \\mid p, q, r=\\operatorname{distinct}\\right.$ primes $\\}$, $C_{n}=\\left\\{p^{3}, p^{4}, p^{5}, p^{6}, p q, p^{2} q, p^{3} q, p^{2} q^{2}, p q r \\mid p, q, r=\\right.$ distinct primes $\\}$. Then $B_{95}=\\{1,2,3,4,5,7,9,11,13,17,19,23,25,29,31,37,41$, $43,47,48,49,53,59,60,61,67,71,72,73,79,80,83,84,89,90\\}$.","problem_type":null,"tier":0} +{"year":"1988","problem_phase":"shortlisted","problem":"21. (POL 4) Forty-nine students solve a set of three problems. The score for each problem is a whole number of points from 0 to 7 . Prove that there exist two students $A$ and $B$ such that for each problem, $A$ will score at least as many points as $B$.","solution":"21. Let $X$ be the set of all ordered triples $a=\\left(a_{1}, a_{2}, a_{3}\\right)$ for $a_{i} \\in\\{0,1, \\ldots, 7\\}$. Write $a \\prec b$ if $a_{i} \\leq b_{i}$ for $i=1,2,3$ and $a \\neq b$. Call a subset $Y \\subset X$ independent if there are no $a, b \\in Y$ with $a \\prec b$. We shall prove that an independent set contains at most 48 elements. For $j=0,1, \\ldots, 21$ let $X_{j}=\\left\\{\\left(a_{1}, a_{2}, a_{3}\\right) \\in X \\mid a_{1}+a_{2}+a_{3}=j\\right\\}$. If $x \\prec y$ and $x \\in X_{j}, y \\in X_{j+1}$ for some $j$, then we say that $y$ is a successor of $x$, and $x$ a predecessor of $y$. Lemma. If $A$ is an $m$-element subset of $X_{j}$ and $j \\leq 10$, then there are at least $m$ distinct successors of the elements of $A$. Proof. For $k=0,1,2,3$ let $X_{j, k}=\\left\\{\\left(a_{1}, a_{2}, a_{3}\\right) \\in X_{j} \\mid \\min \\left(a_{1}, a_{2}, a_{3}, 7-\\right.\\right.$ $\\left.\\left.a_{1}, 7-a_{2}, 7-a_{3}\\right)=k\\right\\}$. It is easy to verify that every element of $X_{j, k}$ has at least two successors in $X_{j+1, k}$ and every element of $X_{j+1, k}$ has at most two predecessors in $X_{j, k}$. Therefore the number of elements of $A \\cap X_{j, k}$ is not greater than the number of their successors. Since $X_{j}$ is a disjoint union of $X_{j, k}, k=0,1,2,3$, the lemma follows. Similarly, elements of an $m$-element subset of $X_{j}, j \\geq 11$, have at least $m$ predecessors. Let $Y$ be an independent set, and let $p, q$ be integers such that $p<10a_{k}+m \\delta$, and consequently $a_{k+m}>1$ for large enough $m$, a contradiction. Thus $a_{k}-a_{k+1} \\geq 0$ for all $k$. Suppose that $a_{k}-a_{k+1}>2 \/ k^{2}$. Then for all $i2 \/ k^{2}$, so that $a_{i}-a_{k+1}>2(k+1-i) \/ k^{2}$, i.e., $a_{i}>2(k+1-i) \/ k^{2}, i=1,2, \\ldots, k$. But this implies $a_{1}+a_{2}+\\cdots+a_{k}>2 \/ k^{2}+4 \/ k^{2}+\\cdots+2 k \/ k^{2}=k(k+1) \/ k^{2}$, which is impossible. Therefore $a_{k}-a_{k+1} \\leq 2 \/ k^{2}$ for all $k$.","problem_type":null,"tier":0} +{"year":"1988","problem_phase":"shortlisted","problem":"25. (GBR 1) A positive integer is called a double number if its decimal representation consists of a block of digits, not commencing with 0 , followed immediately by an identical block. For instance, 360360 is a double number, but 36036 is not. Show that there are infinitely many double numbers that are perfect squares.","solution":"25. Observe that $1001=7 \\cdot 143$, i.e., $10^{3}=-1+7 a, a=143$. Then by the binomial theorem, $10^{21}=(-1+7 a)^{7}=-1+7^{2} b$ for some integer $b$, so that we also have $10^{21 n} \\equiv-1(\\bmod 49)$ for any odd integer $n>0$. Hence $N=\\frac{9}{49}\\left(10^{21 n}+1\\right)$ is an integer of $21 n$ digits, and $N\\left(10^{21 n}+1\\right)=$ $\\left(\\frac{3}{7}\\left(10^{21 n}+1\\right)\\right)^{2}$ is a double number that is a perfect square.","problem_type":null,"tier":0} +{"year":"1988","problem_phase":"shortlisted","problem":"26. (GBR 2) ${ }^{\\mathrm{IMO} 3} \\mathrm{~A}$ function $f$ defined on the positive integers (and taking positive integer values) is given by $$ \\begin{aligned} f(1) & =1, \\quad f(3)=3 \\\\ f(2 n) & =f(n) \\\\ f(4 n+1) & =2 f(2 n+1)-f(n) \\\\ f(4 n+3) & =3 f(2 n+1)-2 f(n) \\end{aligned} $$ for all positive integers $n$. Determine with proof the number of positive integers less than or equal to 1988 for which $f(n)=n$.","solution":"26. The overline in this problem will exclusively denote binary representation. We will show by induction that if $n=\\overline{c_{k} c_{k-1} \\ldots c_{0}}=\\sum_{i=0}^{k} c_{i} 2^{i}$ is the binary representation of $n\\left(c_{i} \\in\\{0,1\\}\\right)$, then $f(n)=\\overline{c_{0} c_{1} \\ldots c_{k}}=$ $\\sum_{i=0}^{k} c_{i} 2^{k-i}$ is the number whose binary representation is the palindrome of the binary representation of $n$. This evidently holds for $n \\in\\{1,2,3\\}$. Let us assume that the claim holds for all numbers up to $n-1$ and show it holds for $n=\\overline{c_{k} c_{k-1} \\ldots c_{0}}$. We observe three cases: (i) $c_{0}=0 \\Rightarrow n=2 m \\Rightarrow f(n)=f(m)=\\overline{0 c_{1} \\ldots c_{k}}=\\overline{c_{0} c_{1} \\ldots c_{k}}$. (ii) $c_{0}=1, c_{1}=0 \\Rightarrow n=4 m+1 \\Rightarrow f(n)=2 f(2 m+1)-f(m)=$ $2 \\cdot \\overline{1 c_{2} \\ldots c_{k}}-\\overline{c_{2} \\ldots c_{k}}=2^{k}+2 \\cdot \\overline{c_{2} \\ldots c_{k}}-\\overline{c_{2} \\ldots c_{k}}=\\overline{10 c_{2} \\ldots c_{k}}=$ $\\overline{c_{0} c_{1} \\ldots c_{k}}$. (iii) $c_{0}=1, c_{1}=1 \\Rightarrow n=4 m+3 \\Rightarrow f(n)=3 f(2 m+1)-2 f(m)=$ $3 \\cdot \\overline{1 c_{2} \\ldots c_{k}}-2 \\cdot \\overline{c_{2} \\ldots c_{k}}=2^{k}+2^{k-1}+3 \\cdot \\overline{c_{2} \\ldots c_{k}}-2 \\cdot \\overline{c_{2} \\ldots c_{k}}=$ $\\overline{11 c_{2} \\ldots c_{k}}=\\overline{c_{0} c_{1} \\ldots c_{k}}$. We thus have to find the number of palindromes in binary representation smaller than $1998=\\overline{11111000100}$. We note that for all $m \\in \\mathbb{N}$ the numbers of $2 m$ - and $(2 m-1)$-digit binary palindromes are both equal to $2^{m-1}$. We also note that $\\overline{11111011111}$ and $\\overline{11111111111}$ are the only 11-digit palindromes larger than 1998. Hence we count all palindromes of up to 11 digits and exclude the largest two. The number of $n \\leq 1998$ such that $f(n)=n$ is thus equal to $1+1+2+2+4+4+8+8+16+16+32-2=92$.","problem_type":null,"tier":0} +{"year":"1988","problem_phase":"shortlisted","problem":"27. (GBR 4) The triangle $A B C$ is acute-angled. Let $L$ be any line in the plane of the triangle and let $u, v, w$ be the lengths of the perpendiculars from $A, B, C$ respectively to $L$. Prove that $$ u^{2} \\tan A+v^{2} \\tan B+w^{2} \\tan C \\geq 2 \\Delta $$ where $\\Delta$ is the area of the triangle, and determine the lines $L$ for which equality holds.","solution":"27. Consider a Cartesian system with the $x$-axis on the line $B C$ and origin at the foot of the perpendicular from $A$ to $B C$, so that $A$ lies on the $y$-axis. Let $A$ be $(0, \\alpha), B(-\\beta, 0), C(\\gamma, 0)$, where $\\alpha, \\beta, \\gamma>0$ (because $A B C$ is acute-angled). Then $\\tan B=\\frac{\\alpha}{\\beta}, \\quad \\tan C=\\frac{\\alpha}{\\gamma} \\quad$ and $\\quad \\tan A=-\\tan (B+C)=\\frac{\\alpha(\\beta+\\gamma)}{\\alpha^{2}-\\beta \\gamma} ;$ here $\\tan A>0$, so $\\alpha^{2}>\\beta \\gamma$. Let $L$ have equation $x \\cos \\theta+y \\sin \\theta+p=0$. Then $$ \\begin{aligned} & u^{2} \\tan A+v^{2} \\tan B+w^{2} \\tan C \\\\ & =\\frac{\\alpha(\\beta+\\gamma)}{\\alpha^{2}-\\beta \\gamma}(\\alpha \\sin \\theta+p)^{2}+\\frac{\\alpha}{\\beta}(-\\beta \\cos \\theta+p)^{2}+\\frac{\\alpha}{\\gamma}(\\gamma \\cos \\theta+p)^{2} \\\\ & =\\left(\\alpha^{2} \\sin ^{2} \\theta+2 \\alpha p \\sin \\theta+p^{2}\\right) \\frac{\\alpha(\\beta+\\gamma)}{\\alpha^{2}-\\beta \\gamma}+\\alpha(\\beta+\\gamma) \\cos ^{2} \\theta+\\frac{\\alpha(\\beta+\\gamma)}{\\beta \\gamma} p^{2} \\\\ & =\\frac{\\alpha(\\beta+\\gamma)}{\\beta \\gamma\\left(\\alpha^{2}-\\beta \\gamma\\right)}\\left(\\alpha^{2} p^{2}+2 \\alpha p \\beta \\gamma \\sin \\theta+\\alpha^{2} \\beta \\gamma \\sin ^{2} \\theta+\\beta \\gamma\\left(\\alpha^{2}-\\beta \\gamma\\right) \\cos ^{2} \\theta\\right) \\\\ & =\\frac{\\alpha(\\beta+\\gamma)}{\\beta \\gamma\\left(\\alpha^{2}-\\beta \\gamma\\right)}\\left[(\\alpha p+\\beta \\gamma \\sin \\theta)^{2}+\\beta \\gamma\\left(\\alpha^{2}-\\beta \\gamma\\right)\\right] \\geq \\alpha(\\beta+\\gamma)=2 \\Delta \\end{aligned} $$ with equality when $\\alpha p+\\beta \\gamma \\sin \\theta=0$, i.e., if and only if $L$ passes through $(0, \\beta \\gamma \/ \\alpha)$, which is the orthocenter of the triangle.","problem_type":null,"tier":0} +{"year":"1988","problem_phase":"shortlisted","problem":"28. (GBR 5) The sequence $\\left\\{a_{n}\\right\\}$ of integers is defined by $a_{1}=2, a_{2}=7$, and $$ -\\frac{1}{2}1$.","solution":"28. The sequence is uniquely determined by the conditions, and $a_{1}=2, a_{2}=$ $7, a_{3}=25, a_{4}=89, a_{5}=317, \\ldots$; it satisfies $a_{n}=3 a_{n-1}+2 a_{n-2}$ for $n=3,4,5$. We show that the sequence $b_{n}$ given by $b_{1}=2, b_{2}=7$, $b_{n}=3 b_{n-1}+2 b_{n-2}$ has the same inequality property, i.e., that $b_{n}=a_{n}$ : $b_{n+1} b_{n-1}-b_{n}^{2}=\\left(3 b_{n}+2 b_{n-1}\\right) b_{n-1}-b_{n}\\left(3 b_{n-1}+2 b_{n-2}\\right)=-2\\left(b_{n} b_{n-2}-b_{n-1}^{2}\\right)$ for $n>2$ gives that $b_{n+1} b_{n-1}-b_{n}^{2}=(-2)^{n-2}$ for all $n \\geq 2$. But then $$ \\left|b_{n+1}-\\frac{b_{n}^{2}}{b_{n-1}}\\right|=\\frac{2^{n-2}}{b_{n-1}}<\\frac{1}{2}, $$ since it is easily shown that $b_{n-1}>2^{n-1}$ for all $n$. It is obvious that $a_{n}=b_{n}$ are odd for $n>1$.","problem_type":null,"tier":0} +{"year":"1988","problem_phase":"shortlisted","problem":"29. (USA 3) A number of signal lights are equally spaced along a one-way railroad track, labeled in order $1,2, \\ldots, N(N \\geq 2)$. As a safety rule, a train is not allowed to pass a signal if any other train is in motion on the length of track between it and the following signal. However, there is no limit to the number of trains that can be parked motionless at a signal, one behind the other. (Assume that the trains have zero length.) A series of $K$ freight trains must be driven from Signal 1 to Signal $N$. Each train travels at a distinct but constant speed (i.e., the speed is fixed and different from that of each of the other trains) at all times when it is not blocked by the safety rule. Show that regardless of the order in which the trains are arranged, the same time will elapse between the first train's departure from Signal 1 and the last train's arrival at Signal $N$.","solution":"29. Let the first train start from Signal 1 at time 0, and let $t_{j}$ be the time it takes for the $j$ th train in the series to travel from one signal to the next. By induction on $k$, we show that Train $k$ arrives at signal $n$ at time $s_{k}+(n-2) m_{k}$, where $s_{k}=t_{1}+\\cdots+t_{k}$ and $m_{k}=\\max _{j=1, \\ldots, k} t_{j}$. For $k=1$ the statement is clear. We now suppose that it is true for $k$ trains and for every $n$, and add a $(k+1)$ th train behind the others at Signal 1. There are two cases to consider: (i) $t_{k+1} \\geq m_{k}$, i.e., $m_{k+1}=t_{k+1}$. Then Train $k+1$ leaves Signal 1 when all the others reach Signal 2, which by the induction happens at time $s_{k}$. Since by the induction hypothesis Train $k$ arrives at Signal $i+1$ at time $s_{k}+(i-1) m_{k} \\leq s_{k}+(i-1) t_{k+1}$, Train $k+1$ is never forced to stop. The journey finishes at time $s_{k}+(n-1) t_{k+1}=s_{k+1}+(n-2) m_{k+1}$. (ii) $t_{k+1}0$ for some $i$, then $\\left|s+v_{i}\\right|>|s|$ ), and similarly $x_{1}, \\ldots, x_{p} \\geq 0$. Finally, suppose by the one-dimensional case that $y_{1}, \\ldots, y_{p}$ and $y_{p+1}, \\ldots, y_{m}$ are permuted in such a way that all the sums $y_{1}+\\cdots+y_{i}$ and $y_{p+1}+\\cdots+y_{p+i}$ are $\\leq 1$ in absolute value. We apply the construction of the one-dimensional case to $x_{1}, \\ldots, x_{m}$ taking, as described above, positive $z_{i}$ 's from $x_{1}, x_{2}, \\ldots, x_{p}$ and negative ones from $x_{p+1}, \\ldots, x_{m}$, but so that the order is preserved; this way we get a permutation $x_{\\sigma_{1}}, x_{\\sigma_{2}}, \\ldots, x_{\\sigma_{m}}$. It is then clear that each sum $y_{\\sigma_{1}}+y_{\\sigma_{2}}+$ $\\cdots+y_{\\sigma_{k}}$ decomposes into the sum $\\left(y_{1}+y_{2}+\\cdots+y_{l}\\right)+\\left(y_{p+1}+\\cdots+y_{p+n}\\right)$ (because of the preservation of order), and that each of these sums is less than or equal to 1 in absolute value. Thus each sum $u_{\\sigma_{1}}+\\cdots+u_{\\sigma_{k}}$ is composed of a vector of length at most 2 and an orthogonal vector of length at most 1 , and so is itself of length at most $\\sqrt{5}$.","problem_type":null,"tier":0} +{"year":"1988","problem_phase":"shortlisted","problem":"9. (FRG 1) ${ }^{\\text {IMO6 }}$ Let $a$ and $b$ be two positive integers such that $a b+1$ divides $a^{2}+b^{2}$. Show that $\\frac{a^{2}+b^{2}}{a b+1}$ is a perfect square.","solution":"9. Let us assume $\\frac{a^{2}+b^{2}}{a b+1}=k \\in \\mathbb{N}$. We then have $a^{2}-k a b+b^{2}=k$. Let us assume that $k$ is not an integer square, which implies $k \\geq 2$. Now we observe the minimal pair $(a, b)$ such that $a^{2}-k a b+b^{2}=k$ holds. We may assume w.l.o.g. that $a \\geq b$. For $a=b$ we get $k=(2-k) a^{2} \\leq 0$; hence we must have $a>b$. Let us observe the quadratic equation $x^{2}-k b x+b^{2}-k=0$, which has solutions $a$ and $a_{1}$. Since $a+a_{1}=k b$, it follows that $a_{1} \\in \\mathbb{Z}$. Since $a>k b$ implies $k>a+b^{2}>k b$ and $a=k b$ implies $k=b^{2}$, it follows that $ak$. Since $a a_{1}=b^{2}-k>0$ and $a>0$, it follows that $a_{1} \\in \\mathbb{N}$ and $a_{1}=\\frac{b^{2}-k}{a}<\\frac{a^{2}-1}{a}1, S$ can be split into $d$ identical blocks. Let $x_{n}$ be the number of nonrepeating binary sequences of length $n$. The total number of binary sequences of length $n$ is obviously $2^{n}$. Any sequence of length $n$ can be produced by repeating its unique longest nonrepeating initial block according to need. Hence, we obtain the recursion relation $\\sum_{d \\mid n} x_{d}=2^{n}$. This, along with $x_{1}=2$, gives us $a_{n}=x_{n}$ for all $n$. We now have that the sequences counted by $x_{n}$ can be grouped into groups of $n$, the sequences in the same group being cyclic shifts of each other. Hence, $n \\mid x_{n}=a_{n}$.","problem_type":null,"tier":0} +{"year":"1989","problem_phase":"shortlisted","problem":"12. (HUN 3) At $n$ distinct points of a circular race course there are $n$ cars ready to start. Each car moves at a constant speed and covers the circle in an hour. On hearing the initial signal, each of them selects a direction and starts moving immediately. If two cars meet, both of them change directions and go on without loss of speed. Show that at a certain moment each car will be at its starting point.","solution":"12. Assume that each car starts with a unique ranking number. Suppose that while turning back at a meeting point two cars always exchanged their ranking numbers. We can observe that ranking numbers move at a constant speed and direction. One hour later, after several exchanges, each starting point will be occupied by a car of the same ranking number and proceeding in the same direction as the one that started from there one hour ago. We now give the cars back their original ranking numbers. Since the sequence of the cars along the track cannot be changed, the only possibility is that the original situation has been rotated, maybe onto itself. Hence for some $d \\mid n$, after $d$ hours each car will be at its starting position and orientation.","problem_type":null,"tier":0} +{"year":"1989","problem_phase":"shortlisted","problem":"13. (ICE 3) ${ }^{\\mathrm{IMO}}$ The quadrilateral $A B C D$ has the following properties: (i) $A B=A D+B C$; (ii) there is a point $P$ inside it at a distance $x$ from the side $C D$ such that $A P=x+A D$ and $B P=x+B C$. Show that $$ \\frac{1}{\\sqrt{x}} \\geq \\frac{1}{\\sqrt{A D}}+\\frac{1}{\\sqrt{B C}} $$","solution":"13. Let us construct the circles $\\sigma_{1}$ with center $A$ and radius $R_{1}=A D, \\sigma_{2}$ with center $B$ and radius $R_{2}=B C$, and $\\sigma_{3}$ with center $P$ and radius $x$. The points $C$ and $D$ lie on $\\sigma_{2}$ and $\\sigma_{1}$ respectively, and $C D$ is tangent to $\\sigma_{3}$. From this it is plain that the greatest value of $x$ occurs when $C D$ is also tangent to $\\sigma_{1}$ and $\\sigma_{2}$. We shall show that in this case the required inequality is really an equality, i.e., that $\\frac{1}{\\sqrt{x}}=\\frac{1}{\\sqrt{A D}}+\\frac{1}{\\sqrt{B C}}$. Then the inequality will immediately follow. Denote the point of tangency of $C D$ with $\\sigma_{3}$ by $M$. By the Pythagorean theorem we have $C D=\\sqrt{\\left(R_{1}+R_{2}\\right)^{2}-\\left(R_{1}-R_{2}\\right)^{2}}=2 \\sqrt{R_{1} R_{2}}$. On the other hand, $C D=C M+M D=2 \\sqrt{R_{2} x}+2 \\sqrt{R_{1} x}$. Hence, we obtain $\\frac{1}{\\sqrt{x}}=\\frac{1}{\\sqrt{R_{1}}}+\\frac{1}{\\sqrt{R_{2}}}$.","problem_type":null,"tier":0} +{"year":"1989","problem_phase":"shortlisted","problem":"14. (IND 2) A bicentric quadrilateral is one that is both inscribable in and circumscribable about a circle. Show that for such a quadrilateral, the centers of the two associated circles are collinear with the point of intersection of the diagonals.","solution":"14. Lemma 1. In a quadrilateral $A B C D$ circumscribed about a circle, with points of tangency $P, Q, R, S$ on $D A, A B, B C, C D$ respectively, the lines $A C, B D, P R, Q S$ concur. Proof. Follows immediately, for example, from Brianchon's theorem. Lemma 2. Let a variable chord $X Y$ of a circle $C(I, r)$ subtend a right angle at a fixed point $Z$ within the circle. Then the locus of the midpoint $P$ of $X Y$ is a circle whose center is at the midpoint $M$ of $I Z$ and whose radius is $\\sqrt{r^{2} \/ 2-I Z^{2} \/ 4}$. Proof. From $\\angle X Z Y=90^{\\circ}$ follows $\\overrightarrow{Z X} \\cdot \\overrightarrow{Z Y}=(\\overrightarrow{I X}-\\overrightarrow{I Z}) \\cdot(\\overrightarrow{I Y}-\\overrightarrow{I Z})=0$. Therefore, $$ \\begin{aligned} \\overrightarrow{M P}^{2} & =(\\overrightarrow{M I}+\\overrightarrow{I P})^{2}=\\frac{1}{4}(-\\overrightarrow{I Z}+\\overrightarrow{I X}+\\overrightarrow{I Y})^{2} \\\\ & =\\frac{1}{4}\\left(I X^{2}+I Y^{2}-I Z^{2}+2(\\overrightarrow{I X}-\\overrightarrow{I Z}) \\cdot(\\overrightarrow{I Y}-\\overrightarrow{I Z})\\right) \\\\ & =\\frac{1}{2} r^{2}-\\frac{1}{4} I Z^{2} \\end{aligned} $$ Lemma 3. Using notation as in Lemma 1, if $A B C D$ is cyclic, $P R$ is perpendicular to $Q S$. Proof. Consider the inversion in $C(I, r)$, mapping $A$ to $A^{\\prime}$ etc. $(P, Q, R, S$ are fixed). As is easily seen, $A^{\\prime}, B^{\\prime}, C^{\\prime}, D^{\\prime}$ will lie at the midpoints of $P Q, Q R, R S, S P$, respectively. $A^{\\prime} B^{\\prime} C^{\\prime} D^{\\prime}$ is a parallelogram, but also cyclic, since inversion preserves circles; thus it must be a rectangle, and so $P R \\perp Q S$. Now we return to the main result. Let $I$ and $O$ be the incenter and circumcenter, $Z$ the intersection of the diagonals, and $P, Q, R, S, A^{\\prime}, B^{\\prime}, C^{\\prime}, D^{\\prime}$ points as defined in Lemmas 1 and 3. From Lemma 3, the chords $P Q, Q R, R S, S P$ subtend $90^{\\circ}$ at $Z$. Therefore by Lemma 2 the points $A^{\\prime}, B^{\\prime}, C^{\\prime}, D^{\\prime}$ lie on a circle whose center is the midpoint $Y$ of $I Z$. Since this circle is the image of the circle $A B C D$ under the considered inversion (centered at $I$ ), it follows that $I, O, Y$ are collinear, and hence so are $I, O, Z$. Remark. This is the famous Newton's theorem for bicentric quadrilaterals.","problem_type":null,"tier":0} +{"year":"1989","problem_phase":"shortlisted","problem":"15. (IRE 1) Let $a, b, c, d, m, n$ be positive integers such that $a^{2}+b^{2}+c^{2}+d^{2}=$ 1989, $a+b+c+d=m^{2}$, and the largest of $a, b, c, d$ is $n^{2}$. Determine, with proof, the values of $m$ and $n$.","solution":"15. By Cauchy's inequality, $44<\\sqrt{1989}a^{2}+b^{2}+c^{2}=$ $1989-d^{2}$, and so $d^{2}-49 d+206>0$. This inequality does not hold for $5 \\leq d \\leq 44$. Since $d \\geq \\sqrt{1989 \/ 4}>22$, $d$ must be at least 45 , which is impossible because $45^{2}>1989$. Thus we must have $m^{2}=81$ and $m=9$. Now, $4 d>81$ implies $d \\geq 21$. On the other hand, $d<\\sqrt{1989}$, and hence $d=25$ or $d=36$. Suppose that $d=25$ and put $a=25-p, b=25-q$, $c=25-r$ with $p, q, r \\geq 0$. From $a+b+c=56$ it follows that $p+q+r=19$, which, together with $(25-p)^{2}+(25-q)^{2}+(25-r)^{2}=1364$, gives us $p^{2}+q^{2}+r^{2}=439>361=(p+q+r)^{2}$, a contradiction. Therefore $d=36$ and $n=6$. Remark. A little more calculation yields the unique solution $a=12$, $b=15, c=18, d=36$.","problem_type":null,"tier":0} +{"year":"1989","problem_phase":"shortlisted","problem":"16. (ISR 1) The set $\\left\\{a_{0}, a_{1}, \\ldots, a_{n}\\right\\}$ of real numbers satisfies the following conditions: (i) $a_{0}=a_{n}=0$; (ii) for $1 \\leq k \\leq n-1$, $$ a_{k}=c+\\sum_{i=k}^{n-1} a_{i-k}\\left(a_{i}+a_{i+1}\\right) . $$ Prove that $c \\leq \\frac{1}{4 n}$.","solution":"16. Define $S_{k}=\\sum_{i=0}^{k} a_{i}(k=0,1, \\ldots, n)$ and $S_{-1}=0$. We note that $S_{n-1}=$ $S_{n}$. Hence $$ \\begin{aligned} S_{n} & =\\sum_{k=0}^{n-1} a_{k}=n c+\\sum_{k=0}^{n-1} \\sum_{i=k}^{n-1} a_{i-k}\\left(a_{i}+a_{i+1}\\right) \\\\ & =n c+\\sum_{i=0}^{n-1} \\sum_{k=0}^{i} a_{i-k}\\left(a_{i}+a_{i+1}\\right)=n c+\\sum_{i=0}^{n-1}\\left(a_{i}+a_{i+1}\\right) \\sum_{k=0}^{i} a_{i-k} \\\\ & =n c+\\sum_{i=0}^{n-1}\\left(S_{i+1}-S_{i-1}\\right) S_{i}=n c+S_{n}^{2} \\end{aligned} $$ i.e., $S_{n}^{2}-S_{n}+n c=0$. Since $S_{n}$ is real, the discriminant of the quadratic equation must be positive, and hence $c \\leq \\frac{1}{4 n}$.","problem_type":null,"tier":0} +{"year":"1989","problem_phase":"shortlisted","problem":"17. (MON 1) Given seven points in the plane, some of them are connected by segments so that: (i) among any three of the given points, two are connected by a segment; (i) the number of segments is minimal. How many segments does a figure satisfying (i) and (ii) contain? Give an example of such a figure.","solution":"17. A figure consisting of 9 lines is shown below. ![](https:\/\/cdn.mathpix.com\/cropped\/2024_11_18_8e985d6b9c83aa3e9d0eg-531.jpg?height=237&width=318&top_left_y=1210&top_left_x=627) Now we show that 8 lines are not sufficient. Assume the opposite. By the pigeonhole principle, there is a vertex, say $A$, that is joined to at most 2 other vertices. Let $B, C, D, E$ denote the vertices to which $A$ is not joined, and $F, G$ the other two vertices. Then any two vertices of $B, C, D, E$ must be mutually joined for an edge to exist within the triangle these two points form with A. This accounts for 6 segments. Since only two segments remain, among $A, F$, and $G$ at least two are not joined. Taking these two and one of $B, C, D, E$ that is not joined to any of them (it obviously exists), we get a triple of points, no two of which are joined; a contradiction. Second solution. Since (a) is equivalent to the fact that no three points make a \"blank triangle,\" by Turan's theorem the number of \"blank edges\" cannot exceed $\\left[7^{2} \/ 4\\right]=12$, leaving at least $7 \\cdot 6 \/ 2-12=9$ segments. For general $n$, the answer is $[(n-1) \/ 2]^{2}$.","problem_type":null,"tier":0} +{"year":"1989","problem_phase":"shortlisted","problem":"18. (MON 4) Given a convex polygon $A_{1} A_{2} \\ldots A_{n}$ with area $S$, and a point $M$ in the same plane, determine the area of polygon $M_{1} M_{2} \\ldots M_{n}$, where $M_{i}$ is the image of $M$ under rotation $\\mathcal{R}_{A_{i}}^{\\alpha}$ around $A_{i}$ by $\\alpha, i=1,2, \\ldots, n$.","solution":"18. Consider the triangle $M A_{i} M_{i}$. Obviously, the point $M_{i}$ is the image of $A_{i}$ under the composition $C$ of rotation $R_{M}^{\\alpha \/ 2-90^{\\circ}}$ and homothety $H_{M}^{2 \\sin (\\alpha \/ 2)}$. Therefore, the polygon $M_{1} M_{2} \\ldots M_{n}$ is obtained as the image of $A_{1} A_{2} \\ldots A_{n}$ under the rotational homothety $C$ with coefficient $2 \\sin (\\alpha \/ 2)$. Therefore $S_{M_{1} M_{2} \\ldots M_{n}}=4 \\sin ^{2}(\\alpha \/ 2) \\cdot S$.","problem_type":null,"tier":0} +{"year":"1989","problem_phase":"shortlisted","problem":"19. (MON 6) A positive integer is written in each square of an $m \\times n$ board. The allowed move is to add an integer $k$ to each of two adjacent numbers in such a way that no negative numbers are obtained. (Two squares are adjacent if they have a common side.) Find a necessary and sufficient condition for it to be possible for all the numbers to be zero by a finite sequence of moves.","solution":"19. Let us color the board in a chessboard fashion. Denote by $S_{b}$ and $S_{w}$ respectively the sum of numbers in the black and in the white squares. It is clear that every allowed move leaves the difference $S_{b}-S_{w}$ unchanged. Therefore a necessary condition for annulling all the numbers is $S_{b}=S_{w}$. We now show it is sufficient. Assuming $S_{b}=S_{w}$ let us observe a triple of (different) cells $a, b, c$ with respective values $x_{a}, x_{b}, x_{c}$ where $a$ and $c$ are both adjacent to $b$. We first prove that we can reduce $x_{a}$ to be 0 if $x_{a}>0$. If $x_{a} \\leq x_{b}$, we subtract $x_{a}$ from both $a$ and $b$. If $x_{a}>x_{b}$, we add $x_{a}-x_{b}$ to $b$ and $c$ and proceed as in the previous case. Applying the reduction in sequence, along the entire board, we reduce all cells except two neighboring cells to be 0 . Since $S_{b}=S_{w}$ is invariant, the two cells must have equal values and we can thus reduce them both to 0 .","problem_type":null,"tier":0} +{"year":"1989","problem_phase":"shortlisted","problem":"2. (AUS 3) Ali Barber, the carpet merchant, has a rectangular piece of carpet whose dimensions are unknown. Unfortunately, his tape measure is broken and he has no other measuring instruments. However, he finds that if he lays it flat on the floor of either of his storerooms, then each corner of the carpet touches a different wall of that room. If the two rooms have dimensions of 38 feet by 55 feet and 50 feet by 55 feet, what are the carpet dimensions?","solution":"2. Let the carpet have width $x$, length $y$. Suppose that the carpet $E F G H$ lies in a room $A B C D, E$ being on $A B, F$ on $B C, G$ on $C D$, and $H$ on $D A$. Then $\\triangle A E H \\equiv \\triangle C G F \\sim \\triangle B F E \\equiv \\triangle D H G$. Let $\\frac{y}{x}=k, A E=a$ and $A H=b$. In that case $B E=k b$ and $D H=k a$. Thus $a+k b=50, k a+b=55$, whence $a=\\frac{55 k-50}{k^{2}-1}$ and $b=\\frac{50 k-55}{k^{2}-1}$. Hence $x^{2}=a^{2}+b^{2}=\\frac{5525 k^{2}-11000 k+5525}{\\left(k^{2}-1\\right)^{2}}$, i.e., $$ x^{2}\\left(k^{2}-1\\right)^{2}=5525 k^{2}-11000 k+5525 $$ Similarly, from the equations for the second storeroom, we get $$ x^{2}\\left(k^{2}-1\\right)^{2}=4469 k^{2}-8360 k+4469 . $$ Combining the two equations, we get $5525 k^{2}-11000 k+5525=4469 k^{2}-$ $8360 k+4469$, which implies $k=2$ or $1 \/ 2$. Without loss of generality we have $y=2 x$ and $a+2 b=50,2 a+b=55$; hence $a=20, b=15$, $x=\\sqrt{15^{2}+20^{2}}=25$, and $y=50$. We have thus shown that the carpet is 25 feet by 50 feet.","problem_type":null,"tier":0} +{"year":"1989","problem_phase":"shortlisted","problem":"20. (NET 1) ${ }^{\\text {IMO3 }}$ Given a set $S$ in the plane containing $n$ points and satisfying the conditions: (i) no three points of $S$ are collinear, (ii) for every point $P$ of $S$ there exist at least $k$ points in $S$ that have the same distance to $P$, prove that the following inequality holds: $$ k<\\frac{1}{2}+\\sqrt{2 n} $$","solution":"20. Suppose $k \\geq 1 \/ 2+\\sqrt{2 n}$. Consider a point $P$ in $S$. There are at least $k$ points in $S$ having all the same distance to $P$, so there are at least $\\binom{k}{2}$ pairs of points $A, B$ with $A P=B P$. Since this is true for every point $P \\in S$, there are at least $n\\binom{k}{2}$ triples of points $(A, B, P)$ for which $A P=B P$ holds. However, $$ \\begin{aligned} n\\binom{k}{2} & =n \\frac{k(k-1)}{2} \\geq \\frac{n}{2}\\left(\\sqrt{2 n}+\\frac{1}{2}\\right)\\left(\\sqrt{2 n}-\\frac{1}{2}\\right) \\\\ & =\\frac{n}{2}\\left(2 n-\\frac{1}{4}\\right)>n(n-1)=2\\binom{n}{2} \\end{aligned} $$ Since $\\binom{n}{2}$ is the number of all possible pairs $(A, B)$ with $A, B \\in S$, there must exist a pair of points $A, B$ with more than two points $P_{i}$ such that $A P_{i}=B P_{i}$. These points $P_{i}$ are collinear (they lie on the perpendicular bisector of $A B$ ), contradicting condition (1).","problem_type":null,"tier":0} +{"year":"1989","problem_phase":"shortlisted","problem":"21. (NET 2) Prove that the intersection of a plane and a regular tetrahedron can be an obtuse-angled triangle and that the obtuse angle in any such triangle is always smaller than $120^{\\circ}$.","solution":"21. In order to obtain a triangle as the intersection we must have three points $P, Q, R$ on three sides of the tetrahedron passing through one vertex, say $T$. It is clear that we may suppose w.l.o.g. that $P$ is a vertex, and $Q$ and $R$ lie on the edges $T P_{1}$ and $T P_{2}\\left(P_{1}, P_{2}\\right.$ are vertices) or on their extensions respectively. Suppose that $\\overrightarrow{T Q}=\\lambda \\overrightarrow{T P_{1}}$ and $\\overrightarrow{T R}=\\mu \\overrightarrow{T P_{2}}$, where $\\lambda, \\mu>0$. Then $$ \\cos \\angle Q P R=\\frac{\\overrightarrow{P Q} \\cdot \\overrightarrow{P R}}{\\overline{P Q} \\cdot \\overline{P R}}=\\frac{(\\lambda-1)(\\mu-1)+1}{2 \\sqrt{\\lambda^{2}-\\lambda+1} \\sqrt{\\mu^{2}-\\mu+1}} $$ In order to obtain an obtuse angle (with $\\cos <0$ ) we must choose $\\mu<1$ and $\\lambda>\\frac{2-\\mu}{1-\\mu}>1$. Since $\\sqrt{\\lambda^{2}-\\lambda+1}>\\lambda-1$ and $\\sqrt{\\mu^{2}-\\mu+1}>1-\\mu$, we get that for $(\\lambda-1)(\\mu-1)+1<0$, $$ \\cos \\angle Q P R>\\frac{1-(1-\\mu)(\\lambda-1)}{2(1-\\mu)(\\lambda-1)}>-\\frac{1}{2} ; \\quad \\text { hence } \\angle Q P R<120^{\\circ} $$ Remark. After obtaining the formula for $\\cos \\angle Q P R$, the official solution was as follows: For fixed $\\mu_{0}<1$ and $\\lambda>1, \\cos \\angle Q P R$ is a decreasing function of $\\lambda$ : indeed, $$ \\frac{\\partial \\cos \\angle Q P R}{\\partial \\lambda}=\\frac{\\mu-(3-\\mu) \\lambda}{4\\left(\\lambda^{2}-\\lambda+1\\right)^{3 \/ 2}\\left(\\mu^{2}-\\mu+1\\right)^{1 \/ 2}}<0 $$ Similarly, for a fixed, sufficiently large $\\lambda_{0}, \\cos \\angle Q P R$ is decreasing for $\\mu$ decreasing to 0 . Since $\\lim _{\\lambda \\rightarrow 0, \\mu \\rightarrow 0+} \\cos \\angle Q P R=-1 \/ 2$, we conclude that $\\angle Q P R<120^{\\circ}$.","problem_type":null,"tier":0} +{"year":"1989","problem_phase":"shortlisted","problem":"22. (PHI 1) ${ }^{\\text {IMO1 }}$ Prove that the set $\\{1,2, \\ldots, 1989\\}$ can be expressed as the disjoint union of 17 subsets $A_{1}, A_{2}, \\ldots, A_{17}$ such that: (i) each $A_{i}$ contains the same number of elements; (ii) the sum of all elements of each $A_{i}$ is the same for $i=1,2, \\ldots, 17$.","solution":"22. The statement remains valid if 17 is replaced by any divisor $k$ of $1989=3^{2}$. $13 \\cdot 17,12$, is the twin of $x_{1}$, then $$ f\\left(x_{1}, x_{2}, \\ldots, x_{2 n}\\right)=\\left(x_{2}, \\ldots, x_{k-1}, x_{1}, x_{k}, \\ldots, x_{2 n}\\right) $$ The mapping $f$ is injective, but not surjective. Thus $F_{0}(n)0$ (because $a, b<0$ is impossible, and $a, b \\neq 0$ from the condition of the problem). Let $\\left(x_{0}, y_{0}, z_{0}, w_{0}\\right) \\neq$ $(0,0,0,0)$ be a solution of $x^{2}-a y^{2}-b z^{2}+a b w^{2}$. Then $$ x_{0}^{2}-a y_{0}^{2}=b\\left(z_{0}^{2}-a w_{0}^{2}\\right) $$ Multiplying both sides by $\\left(z_{0}^{2}-a w_{0}^{2}\\right)$, we get $$ \\begin{gathered} \\left(x_{0}^{2}-a y_{0}^{2}\\right)\\left(z_{0}^{2}-a w_{0}^{2}\\right)-b\\left(z_{0}^{2}-a w_{0}^{2}\\right)^{2}=0 \\\\ \\Leftrightarrow\\left(x_{0} z_{0}-a y_{0} w_{0}\\right)^{2}-a\\left(y_{0} z_{0}-x_{0} w_{0}\\right)^{2}-b\\left(z_{0}^{2}-a w_{0}^{2}\\right)^{2}=0 . \\end{gathered} $$ Hence, for $x_{1}=x_{0} z_{0}-a y_{0} w_{0}, \\quad y_{1}=y_{0} z_{0}-x_{0} w_{0}, \\quad z_{1}=z_{0}^{2}-a w_{0}^{2}$, we have $$ x_{1}^{2}-a y_{1}^{2}-b z_{1}^{2}=0 . $$ If $\\left(x_{1}, y_{1}, z_{1}\\right)$ is the trivial solution, then $z_{1}=0$ implies $z_{0}=w_{0}=0$ and similarly $x_{0}=y_{0}=0$ because $a$ is not a perfect square. This contradicts the initial assumption.","problem_type":null,"tier":0} +{"year":"1989","problem_phase":"shortlisted","problem":"26. (KOR 4) Let $n$ be a positive integer and let $a, b$ be given real numbers. Determine the range of $x_{0}$ for which $$ \\sum_{i=0}^{n} x_{i}=a \\quad \\text { and } \\quad \\sum_{i=0}^{n} x_{i}^{2}=b $$ where $x_{0}, x_{1}, \\ldots, x_{n}$ are real variables.","solution":"26. By the Cauchy-Schwarz inequality, $$ \\left(\\sum_{i=1}^{n} x_{i}\\right)^{2} \\leq n \\sum_{i=1}^{n} x_{i}^{2} $$ Since $\\sum_{i=1}^{n} x_{i}=a-x_{0}$ and $\\sum_{i=1}^{n} x_{i}^{2}=b-x_{0}^{2}$, we have $\\left(a-x_{0}\\right)^{2} \\leq$ $n\\left(b-x_{0}^{2}\\right)$, i.e., $$ (n+1) x_{0}^{2}-2 a x_{0}+\\left(a^{2}-n b\\right) \\leq 0 . $$ The discriminant of this quadratic is $D=4 n(n+1)\\left[b-a^{2} \/(n+1)\\right]$, so we conclude that (i) if $a^{2}>(n+1) b$, then such an $x_{0}$ does not exist; (ii) if $a^{2}=(n+1) b$, then $x_{0}=a \/ n+1 ; \\quad$ and (iii) if $a^{2}<(n+1) b$, then $\\frac{a-\\sqrt{D} \/ 2}{n+1} \\leq x_{0} \\leq \\frac{a+\\sqrt{D} \/ 2}{n+1}$. It is easy to see that these conditions for $x_{0}$ are also sufficient.","problem_type":null,"tier":0} +{"year":"1989","problem_phase":"shortlisted","problem":"27. (ROM 1) Let $m$ be a positive odd integer, $m \\geq 2$. Find the smallest positive integer $n$ such that $2^{1989}$ divides $m^{n}-1$.","solution":"27. Let $n$ be the required exponent, and suppose $n=2^{k} q$, where $q$ is an odd integer. Then we have $$ m^{n}-1=\\left(m^{2^{k}}-1\\right)\\left[\\left(m^{2^{k}(q-1)}+\\cdots+m^{2^{k}}+1\\right]=\\left(m^{2^{k}}-1\\right) A\\right. $$ where $A$ is odd. Therefore $m^{n}-1$ and $m^{2^{k}}-1$ are divisible by the same power of 2 , and so $n=2^{k}$. Next, we observe that $$ \\begin{aligned} m^{2^{k}}-1 & =\\left(m^{2^{k-1}}-1\\right)\\left(m^{2^{k-1}}+1\\right)=\\ldots \\\\ & =\\left(m^{2}-1\\right)\\left(m^{2}+1\\right)\\left(m^{4}+1\\right) \\cdots\\left(m^{2^{k-1}}+1\\right) \\end{aligned} $$ Let $s$ be the maximal positive integer for which $m \\equiv \\pm 1\\left(\\bmod 2^{s}\\right)$. Then $m^{2}-1$ is divisible by $2^{s+1}$ and not divisible by $2^{s+2}$. All the numbers $m^{2}+1, m^{4}+1, \\ldots, m^{2^{k-1}}+1$ are divisible by 2 and not by 4 . Hence $m^{2^{k}}-1$ is divisible by $2^{s+k}$ and not by $2^{s+k+1}$. It follows from the above consideration that the smallest exponent $n$ equals $2^{1989-s}$ if $s \\leq 1989$, and $n=1$ if $s>1989$.","problem_type":null,"tier":0} +{"year":"1989","problem_phase":"shortlisted","problem":"28. (ROM 2) Consider in a plane $\\Pi$ the points $O, A_{1}, A_{2}, A_{3}, A_{4}$ such that $\\sigma\\left(O A_{i} A_{j}\\right) \\geq 1$ for all $i, j=1,2,3,4, i \\neq j$. Prove that there is at least one pair $i_{0}, j_{0} \\in\\{1,2,3,4\\}$ such that $\\sigma\\left(O A_{i_{0}} A_{j_{0}}\\right) \\geq \\sqrt{2}$. (We have denoted by $\\sigma\\left(O A_{i} A_{j}\\right)$ the area of triangle $O A_{i} A_{j}$.)","solution":"28. Assume w.l.o.g. that the rays $O A_{1}, O A_{2}, O A_{3}, O A_{4}$ are arranged clockwise. Setting $O A_{1}=a, O A_{2}=b, O A_{3}=c, O A_{4}=d$, and $\\angle A_{1} O A_{2}=x$, $\\angle A_{2} O A_{3}=y, \\angle A_{3} O A_{4}=z$, we have $$ \\begin{aligned} & S_{1}=\\sigma\\left(O A_{1} A_{2}\\right)=\\frac{1}{2} a b|\\sin x|, S_{2}=\\sigma\\left(O A_{1} A_{3}\\right)=\\frac{1}{2} a c|\\sin (x+y)|, \\\\ & S_{3}=\\sigma\\left(O A_{1} A_{4}\\right)=\\frac{1}{2} a d|\\sin (x+y+z)|, S_{4}=\\sigma\\left(O A_{2} A_{3}\\right)=\\frac{1}{2} b c|\\sin y|, \\\\ & S_{5}=\\sigma\\left(O A_{2} A_{4}\\right)=\\frac{1}{2} b d|\\sin (y+z)|, S_{6}=\\sigma\\left(O A_{3} A_{4}\\right)=\\frac{1}{2} c d|\\sin z| . \\end{aligned} $$ Since $\\sin (x+y+z) \\sin y+\\sin x \\sin z=\\sin (x+y) \\sin (y+z)$, it follows that there exists a choice of $k, l \\in\\{0,1\\}$ such that $$ S_{1} S_{6}+(-1)^{k} S_{2} S_{5}+(-1)^{l} S_{3} S_{4}=0 $$ For example (w.l.o.g.), if $S_{3} S_{4}=S_{1} S_{6}+S_{2} S_{5}$, we have $$ \\left(\\max _{1 \\leq i \\leq 6} S_{i}\\right)^{2} \\geq S_{3} S_{4}=S_{1} S_{6}+S_{2} S_{5} \\geq 1+1=2 $$ i.e., $\\max _{1 \\leq i \\leq 6} S_{i} \\geq \\sqrt{2}$ as claimed.","problem_type":null,"tier":0} +{"year":"1989","problem_phase":"shortlisted","problem":"29. (ROM 4) A flock of 155 birds sit down on a circle $C$. Two birds $P_{i}, P_{j}$ are mutually visible if $m\\left(P_{i} P_{j}\\right) \\leq 10^{\\circ}$. Find the smallest number of mutually visible pairs of birds. (One assumes that a position (point) on $C$ can be occupied simultaneously by several birds.)","solution":"29. Let $P_{i}$, sitting at the place $A$, and $P_{j}$ sitting at $B$, be two birds that can see each other. Let $k$ and $l$ respectively be the number of birds visible from $B$ but not from $A$, and the number of those visible from $A$ but not from $B$. Assume that $k \\geq l$. Then if all birds from $B$ fly to $A$, each of them will see $l$ new birds, but won't see $k$ birds anymore. Hence the total number of mutually visible pairs does not increase, while the number of distinct positions occupied by at least one bird decreases by one. Repeating this operation as many times as possible one can arrive at a situation in which two birds see each other if and only if they are in the same position. The number of such distinct positions is at most 35 , while the total number of mutually visible pairs is not greater than at the beginning. Thus the problem is equivalent to the following one: (1) If $x_{i} \\geq 0$ are integers with $\\sum_{j=1}^{35} x_{j}=155$, find the least possible value of $\\sum_{j=1}^{35}\\left(x_{j}^{2}-x_{j}\\right) \/ 2$. If $x_{j} \\geq x_{i}+2$ for some $i, j$, then the sum of $\\left(x_{j}^{2}-x_{j}\\right) \/ 2$ decreases (for $x_{j}-x_{i}-2$ ) if $x_{i}, x_{j}$ are replaced with $x_{i}+1, x_{j}-1$. Consequently, our sum attains its minimum when the $x_{i}$ 's differ from each other by at most 1 . In this case, all the $x_{i}$ 's are equal to either $[155 \/ 35]=4$ or $[155 \/ 35]+1=5$, where $155=20 \\cdot 4+15 \\cdot 5$. It follows that the (minimum possible) number of mutually visible pairs is $20 \\cdot \\frac{4 \\cdot 3}{2}+15 \\cdot \\frac{5 \\cdot 4}{2}=270$. Second solution for (1). Considering the graph consisting of birds as vertices and pairs of mutually nonvisible birds as edges, we see that there is no complete 36 -subgraph. Turan's theorem gives the answer immediately. (See problem (SL89-17).)","problem_type":null,"tier":0} +{"year":"1989","problem_phase":"shortlisted","problem":"3. (AUS 4) Ali Barber, the carpet merchant, has a rectangular piece of carpet whose dimensions are unknown. Unfortunately, his tape measure is broken and he has no other measuring instruments. However, he finds that if he lays it flat on the floor of either of his storerooms, then each corner of the carpet touches a different wall of that room. He knows that the sides of the carpet are integral numbers of feet and that his two storerooms have the same (unknown) length, but widths of 38 feet and 50 feet respectively. What are the carpet dimensions?","solution":"3. Let the carpet have width $x$, length $y$. Let the length of the storerooms be $q$. Let $y \/ x=k$. Then, as in the previous problem, $(k q-50)^{2}+(50 k-q)^{2}=$ $(k q-38)^{2}+(38 k-q)^{2}$, i.e., $$ k q=22\\left(k^{2}+1\\right) $$ Also, as before, $x^{2}=\\left(\\frac{k q-50}{k^{2}-1}\\right)^{2}+\\left(\\frac{50 k-q}{k^{2}-1}\\right)^{2}$, i.e., $$ x^{2}\\left(q^{2}-1\\right)^{2}=\\left(k^{2}+1\\right)\\left(q^{2}-1900\\right) $$ which, together with (1), yields $$ x^{2} k^{2}\\left(k^{2}-1\\right)^{2}=\\left(k^{2}+1\\right)\\left(484 k^{4}-932 k^{2}+484\\right) $$ Since $k$ is rational, let $k=c \/ d$, where $c$ and $d$ are integers with $\\operatorname{gcd}(c, d)=$ 1. Then we obtain $$ x^{2} c^{2}\\left(c^{2}-d^{2}\\right)^{2}=c^{2}\\left(484 c^{4}-448 c^{2} d^{2}-448 d^{4}\\right)+484 d^{6} $$ We thus have $c^{2} \\mid 484 d^{6}$, but since $(c, d)=1$, we have $c^{2}|484 \\Rightarrow c| 22$. Analogously, $d \\mid 22$; thus $k=1,2,11,22, \\frac{1}{2}, \\frac{1}{11}, \\frac{1}{22}, \\frac{2}{11}, \\frac{11}{2}$. Since reciprocals lead to the same solution, we need only consider $k \\in\\left\\{1,2,11,22, \\frac{11}{2}\\right\\}$, yielding $q=44,55,244,485,125$, respectively. We can test these values by substituting them into (2). Only $k=2$ gives us an integer solution, namely $x=25, y=50$.","problem_type":null,"tier":0} +{"year":"1989","problem_phase":"shortlisted","problem":"30. (SWE 2) ${ }^{\\mathrm{IMO} 5}$ For which positive integers $n$ does there exist a positive integer $N$ such that none of the integers $1+N, 2+N, \\ldots, n+N$ is the power of a prime number?","solution":"30. For all $n$ such $N$ exists. For a given $n$ choose $N=(n+1)!^{2}+1$. Then $1+j$ is a proper factor of $N+j$ for $1 \\leq j \\leq n$. So if $N+j=p^{m}$ is a power of a prime $p$, then $1+j=p^{r}$ for some integer $r, 1 \\leq r1$ the equation $$ \\frac{x^{n}}{n!}+\\frac{x^{n-1}}{(n-1)!}+\\cdots+\\frac{x^{2}}{2!}+\\frac{x}{1!}+1=0 $$ has no rational roots.","solution":"4. First we note that for every integer $k>0$ and prime number $p, p^{k}$ doesn't divide $k!$. This follows from the fact that the highest exponent $r$ of $p$ for which $p^{r} \\mid k$ ! is $$ r=\\left[\\frac{k}{p}\\right]+\\left[\\frac{k}{p^{2}}\\right]+\\cdots<\\frac{k}{p}+\\frac{k}{p^{2}}+\\cdots=\\frac{k}{p-1}0$, we can write $k=2^{l} k^{\\prime}, k^{\\prime}$ being odd and $l$ a nonnegative integer. Let us set $v(k)=l$, and define $\\beta_{n}=v\\left(b_{n}\\right), \\gamma_{n}=v\\left(c_{n}\\right)$. We prove the following lemmas: Lemma 1. For every integer $p \\geq 0, b_{2^{p}}$ and $c_{2^{p}}$ are nonzero, and $\\beta_{2^{p}}=$ $\\gamma_{2^{p}}=p+2$. Proof. By induction on $p$. For $p=0, b_{1}=4$ and $c_{1}=-4$, so the assertion is true. Suppose that it holds for $p$. Then $$ (1+4 \\sqrt[3]{2}-4 \\sqrt[3]{4})^{2^{p+1}}=\\left(a+2^{p+2}\\left(b^{\\prime} \\sqrt[3]{2}+c^{\\prime} \\sqrt[3]{4}\\right)\\right)^{2} \\text { with } a, b^{\\prime}, \\text { and } c^{\\prime} \\text { odd. } $$ Then we easily obtain that $(1+4 \\sqrt[3]{2}-4 \\sqrt[3]{4})^{2^{p+1}}=A+2^{p+3}(B \\sqrt[3]{2}+$ $C \\sqrt[3]{4}$ ), where $A, B=a b^{\\prime}+2^{p+1} E, C=a c^{\\prime}+2^{p+1} F$ are odd. Therefore Lemma 1 holds for $p+1$. Lemma 2. Suppose that for integers $n, m \\geq 0, \\beta_{n}=\\gamma_{n}=\\lambda>\\beta_{m}=$ $\\gamma_{m}=\\mu$. Then $b_{n+m}, c_{n+m}$ are nonzero and $\\beta_{n+m}=\\gamma_{n+m}=\\mu$. Proof. Calculating $\\left(a^{\\prime}+2^{\\lambda}\\left(b^{\\prime} \\sqrt[3]{2}+c^{\\prime} \\sqrt[3]{4}\\right)\\right)\\left(a^{\\prime \\prime}+2^{\\mu}\\left(b^{\\prime \\prime} \\sqrt[3]{2}+c^{\\prime \\prime} \\sqrt[3]{4}\\right)\\right)$, with $a^{\\prime}, b^{\\prime}, c^{\\prime}, a^{\\prime \\prime}, b^{\\prime \\prime}, c^{\\prime \\prime}$ odd, we easily obtain the product $A+2^{\\mu}(B \\sqrt[3]{2}+$ $C \\sqrt[3]{4})$, where $A, B=a^{\\prime} b^{\\prime \\prime}+2^{\\lambda-\\mu} E$, and $C=a^{\\prime} c^{\\prime \\prime}+2^{\\lambda-\\mu} F$ are odd, which proves Lemma 2. Since every integer $n>0$ can be written as $n=2^{p_{r}}+\\cdots+2^{p_{1}}$, with $0 \\leq p_{1}<\\cdotsb$. Since the function $f(x)=\\frac{x}{x+c}$ is strictly increasing, we deduce $d(F, L)>d(D, L)$. Furthermore, $\\sin (\\alpha \/ 2)>\\sin (\\beta \/ 2)$, so we get from (1) that $F G>D E$. Similarly, $a1$, the scientist can climb only onto the rungs divisible by $k$ and we can just observe these rungs to obtain the situation equivalent to $a^{\\prime}=a \/ k, b^{\\prime}=b \/ k$, and $n^{\\prime}=a^{\\prime}+b^{\\prime}-1$. Thus let us assume that $(a, b)=1$ and show that $n=a+b-1$. We obviously have $n>a$. Consider $n=a+b-k, k \\geq 1$, and let us assume without loss of generality that $a>b$ (otherwise, we can reverse the problem starting from the top rung in our round trip). Then we can uniquely define the numbers $r_{i}, 0 \\leq r_{i}b-1$ we can move only $b$ rungs downward. If we end up at $b-k

0$ that $f^{k}(0)=0$. Since $f(m)=m+a$ or $f(m)=m-b$, it follows that $k$ can be written as $k=r+s$, where $r a-s b=0$. Since $a$ and $b$ are relatively prime, it follows that $k \\geq a+b$. Let us now prove that $f^{a+b}(0)=0$. In this case $a+b=r+s$ and hence $f^{a+b}(0)=(a+b-s) a-s b=(a+b)(a-s)$. Since $a+b \\mid f^{a+b}(0)$ and $f^{a+b}(0) \\in S$, it follows that $f^{a+b}(0)=0$. Thus for $(a, b)=1$ it follows that $k=a+b$. For other $a$ and $b$ we have $k=\\frac{a+b}{(a, b)}$.","problem_type":null,"tier":0} +{"year":"1990","problem_phase":"shortlisted","problem":"19. (POL 1) Let $P$ be a point inside a regular tetrahedron $T$ of unit volume. The four planes passing through $P$ and parallel to the faces of $T$ partition $T$ into 14 pieces. Let $f(P)$ be the joint volume of those pieces that are neither a tetrahedron nor a parallelepiped (i.e., pieces adjacent to an edge but not to a vertex). Find the exact bounds for $f(P)$ as $P$ varies over $T$.","solution":"19. Let $d_{1}, d_{2}, d_{3}, d_{4}$ be the distances of the point $P$ to the tetrahedron. Let $d$ be the height of the regular tetrahedron. Let $x_{i}=d_{i} \/ d$. Clearly, $x_{1}+$ $x_{2}+x_{3}+x_{4}=1$, and given this condition, the parameters vary freely as we vary $P$ within the tetrahedron. The four tetrahedra have volumes $x_{1}^{3}, x_{2}^{3}, x_{3}^{3}$, and $x_{4}^{3}$, and the four parallelepipeds have volumes of $6 x_{2} x_{3} x_{4}$, $6 x_{1} x_{3} x_{4}, 6 x_{1} x_{2} x_{4}$, and $6 x_{1} x_{2} x_{3}$. Hence, using $x_{1}+x_{2}+x_{3}+x_{4}=1$ and setting $g(x)=x^{2}(1-x)$, we directly verify that $$ \\begin{aligned} f(P) & =f\\left(x_{1}, x_{2}, x_{3}, x_{4}\\right)=1-\\sum_{i=1}^{4} x_{i}^{3}-6 \\sum_{1 \\leq i0$ we have $g\\left(x_{i}+x_{j}\\right)+g(0) \\geq$ $g\\left(x_{i}\\right)+g\\left(x_{j}\\right)$. Equality holds only when $x_{i}+x_{j}=2 \/ 3$. Assuming without loss of generality $x_{1} \\geq x_{2} \\geq x_{3} \\geq x_{4}$, we have $g\\left(x_{1}\\right)+$ $g\\left(x_{2}\\right)+g\\left(x_{3}\\right)+g\\left(x_{4}\\right)k$ and hence there exist two numbers $x, y \\in S(n)$ such that $k \\mid x-y$. Let us show that $w=|x-y|$ is the desired number. By definition $k \\mid w$. We also have $$ w<1.2 \\cdot 10^{n-1} \\leq 1.2 \\cdot\\left(2^{3} \\sqrt{2}\\right)^{n-1} \\leq 1.2 \\cdot k^{3} \\sqrt{k} \\leq k^{4} $$ Finally, since $x, y \\in S(n)$, it follows that $w=|x-y|$ can be written using only the digits $\\{0,1,8,9\\}$. This completes the proof.","problem_type":null,"tier":0} +{"year":"1990","problem_phase":"shortlisted","problem":"21. (ROM $\\mathbf{1}^{\\prime}$ ) Let $n$ be a composite natural number and $p$ a proper divisor of $n$. Find the binary representation of the smallest natural number $N$ such that $\\frac{\\left(1+2^{p}+2^{n-p}\\right) N-1}{2^{n}}$ is an integer.","solution":"21. We must solve the congruence $\\left(1+2^{p}+2^{n-p}\\right) N \\equiv 1\\left(\\bmod 2^{n}\\right)$. Since $(1+$ $2^{p}+2^{n-p}$ ) and $2^{n}$ are coprime, there clearly exists a unique $N$ satisfying this equation and $0b$ we have in binary representation $$ 2^{a}-2^{b}=\\underbrace{11 \\ldots 11}_{a-b \\text { times }} \\underbrace{00 \\ldots 00}_{b \\text { times }} $$ the binary representation of $N$ is calculated as follows: $$ N=\\left\\{\\begin{array}{cc} \\underbrace{11 \\ldots 11}_{p \\text { times }} \\underbrace{11 \\ldots 11}_{p \\text { times }} \\underbrace{00 \\ldots 00}_{p \\text { times }} \\ldots \\underbrace{11 \\ldots 11}_{p \\text { times }} \\underbrace{00 \\ldots 00}_{p-1 \\text { times }} 1, & 2 \\nmid \\frac{n}{p} \\\\ \\underbrace{11 \\ldots 11}_{p-1 \\text { times }} \\underbrace{00 \\ldots 00}_{p+1} \\underbrace{11 \\ldots 11}_{p \\text { times }} \\underbrace{00 \\ldots 00}_{p \\text { times }} \\ldots \\underbrace{11 \\ldots 11}_{p \\text { times }} \\underbrace{00 \\ldots 00}_{p-1 \\text { times }} 1, & 2 \\left\\lvert\\, \\frac{n}{p}\\right. \\end{array}\\right. $$","problem_type":null,"tier":0} +{"year":"1990","problem_phase":"shortlisted","problem":"22. (ROM 4) Ten localities are served by two international airlines such that there exists a direct service (without stops) between any two of these localities and all airline schedules offer round-trip service between the cities they serve. Prove that at least one of the airlines can offer two disjoint round trips each containing an odd number of landings.","solution":"22. We can assume without loss of generality that each connection is serviced by only one airline and the problem reduces to finding two disjoint monochromatic cycles of the same color and of odd length on a complete graph of 10 points colored by two colors. We use the following two standard lemmas: Lemma 1. Given a complete graph on six points whose edges are colored with two colors there exists a monochromatic triangle. $\\operatorname{Proof}$. Let us denote the vertices by $c_{1}, c_{2}, c_{3}, c_{4}, c_{5}, c_{6}$. By the pigeonhole principle at least three vertices out of $c_{1}$, say $c_{2}, c_{3}, c_{4}$, are of the same color, let us call it red. Assuming that at least one of the edges connecting points $c_{2}, c_{3}, c_{4}$ is red, the connected points along with $c_{1}$ form a red triangle. Otherwise, edges connecting $c_{2}, c_{3}, c_{4}$ are all of the opposite color, let us call it blue, and hence in all cases we have a monochromatic triangle. Lemma 2. Given a complete graph on five points whose edges are colored two colors there exists a monochromatic triangle or a monochromatic cycle of length five. Proof. Let us denote the vertices by $c_{1}, c_{2}, c_{3}, c_{4}, c_{5}$. Assume that out of a point $c_{i}$ three vertices are of the same color. We can then proceed as in Lemma 1 to obtain a monochromatic triangle. Otherwise, each point is connected to other points with exactly two red and two blue vertices. Hence, we obtain monochromatic cycles starting from a single point and moving along the edges of the same color. Since each cycle must be of length at least three (i.e., we cannot have more than one cycle of one color), it follows that for both red and blue we must have one cycle of length five of that color. We now apply the lemmas. Let us denote the vertices by $c_{1}, c_{2}, \\ldots, c_{10}$. We apply Lemma 1 to vertices $c_{1}, \\ldots, c_{6}$ to obtain a monochromatic triangle. Out of the seven remaining vertices we select 6 and again apply Lemma 1 to obtain another monochromatic triangle. If they are of the same color, we are done. Otherwise, out of the nine edges connecting the two triangles of opposite color at least 5 are of the same color, we can assume blue w.l.o.g., and hence a vertex of a red triangle must contain at least two blue edges whose endpoints are connected with a blue edge. Hence there exist two triangles of different colors joined at a vertex. These take up five points. Applying Lemma 2 on the five remaining points, we obtain a monochromatic cycle of odd length that is of the same color as one of the two joined triangles and disjoint from both of them.","problem_type":null,"tier":0} +{"year":"1990","problem_phase":"shortlisted","problem":"23. (ROM 5) ${ }^{\\mathrm{IMO} 3}$ Find all positive integers $n$ having the property that $\\frac{2^{n}+1}{n^{2}}$ is an integer.","solution":"23. Let us assume $n>1$. Obviously $n$ is odd. Let $p \\geq 3$ be the smallest prime divisor of $n$. In this case $(p-1, n)=1$. Since $2^{n}+1 \\mid 2^{2 n}-1$, we have that $p \\mid 2^{2 n}-1$. Thus it follows from Fermat's little theorem and elementary number theory that $p \\mid\\left(2^{2 n}-1,2^{p-1}-1\\right)=2^{(2 n, p-1)}-1$. Since $(2 n, p-1) \\leq 2$, it follows that $p \\mid 3$ and hence $p=3$. Let us assume now that $n$ is of the form $n=3^{k} d$, where $2,3 \\nmid d$. We first prove that $k=1$. Lemma. If $2^{m}-1$ is divisible by $3^{r}$, then $m$ is divisible by $3^{r-1}$. Proof. This is the lemma from (SL97-14) with $p=3, a=2^{2}, k=m$, $\\alpha=1$, and $\\beta=r$. Since $3^{2 k}$ divides $n^{2} \\mid 2^{2 n}-1$, we can apply the lemma to $m=2 n$ and $r=2 k$ to conclude that $3^{2 k-1} \\mid n=3^{k} d$. Hence $k=1$. Finally, let us assume $d>1$ and let $q$ be the smallest prime factor of $d$. Obviously $q$ is odd, $q \\geq 5$, and $(n, q-1) \\in\\{1,3\\}$. We then have $q \\mid 2^{2 n}-1$ and $q \\mid 2^{q-1}-1$. Consequently, $q \\mid 2^{(2 n, q-1)}-1=2^{2(n, q-1)}-1$, which divides $2^{6}-1=63=3^{2} \\cdot 7$, so we must have $q=7$. However, in that case we obtain $7|n| 2^{n}+1$, which is a contradiction, since powers of two can only be congruent to 1,2 and 4 modulo 7 . It thus follows that $d=1$ and $n=3$. Hence $n>1 \\Rightarrow n=3$. It is easily verified that $n=1$ and $n=3$ are indeed solutions. Hence these are the only solutions.","problem_type":null,"tier":0} +{"year":"1990","problem_phase":"shortlisted","problem":"24. (THA 2) Let $a, b, c, d$ be nonnegative real numbers such that $a b+b c+$ $c d+d a=1$. Show that $$ \\frac{a^{3}}{b+c+d}+\\frac{b^{3}}{a+c+d}+\\frac{c^{3}}{a+b+d}+\\frac{d^{3}}{a+b+c} \\geq \\frac{1}{3} $$ ![](https:\/\/cdn.mathpix.com\/cropped\/2024_11_18_8e985d6b9c83aa3e9d0eg-262.jpg?height=62&width=1190&top_left_y=791&top_left_x=173) a function $f: \\mathbb{Q}^{+} \\rightarrow \\mathbb{Q}^{+}$such that $$ f(x f(y))=\\frac{f(x)}{y}, \\quad \\text { for all } x, y \\text { in } \\mathbb{Q}^{+} $$","solution":"24. Let us denote $A=b+c+d, B=a+c+d, C=a+b+d, D=a+b+c$. Since $a b+b c+c d+d a=1$ the numbers $A, B, C, D$ are all positive. By trivially applying the AM-GM inequality we have: $$ a^{2}+b^{2}+c^{2}+d^{2} \\geq a b+b c+c d+d a=1 . $$ We will prove the inequality assuming only that $A, B, C, D$ are positive and $a^{2}+b^{2}+c^{2}+d^{2} \\geq 1$. In this case we may assume without loss of generality that $a \\geq b \\geq c \\geq d \\geq 0$. Hence $a^{3} \\geq b^{3} \\geq c^{3} \\geq d^{3} \\geq 0$ and $\\frac{1}{A} \\geq \\frac{1}{B} \\geq \\frac{1}{C} \\geq \\frac{1}{D}>0$. Using the Chebyshev and Cauchy inequalities we obtain: $$ \\begin{aligned} & \\frac{a^{3}}{A}+\\frac{b^{3}}{B}+\\frac{c^{3}}{C}+\\frac{d^{3}}{D} \\\\ & \\geq \\frac{1}{4}\\left(a^{3}+b^{3}+c^{3}+d^{3}\\right)\\left(\\frac{1}{A}+\\frac{1}{B}+\\frac{1}{C}+\\frac{1}{D}\\right) \\\\ & \\geq \\frac{1}{16}\\left(a^{2}+b^{2}+c^{2}+d^{2}\\right)(a+b+c+d)\\left(\\frac{1}{A}+\\frac{1}{B}+\\frac{1}{C}+\\frac{1}{D}\\right) \\\\ & =\\frac{1}{48}\\left(a^{2}+b^{2}+c^{2}+d^{2}\\right)(A+B+C+D)\\left(\\frac{1}{A}+\\frac{1}{B}+\\frac{1}{C}+\\frac{1}{D}\\right) \\geq \\frac{1}{3} \\end{aligned} $$ This completes the proof.","problem_type":null,"tier":0} +{"year":"1990","problem_phase":"shortlisted","problem":"26. (USA 2) Let $P$ be a cubic polynomial with rational coefficients, and let $q_{1}, q_{2}, q_{3}, \\ldots$ be a sequence of rational numbers such that $q_{n}=P\\left(q_{n+1}\\right)$ for all $n \\geq 1$. Prove that there exists $k \\geq 1$ such that for all $n \\geq 1, q_{n+k}=q_{n}$.","solution":"26. We note that $|P(x) \/ x| \\rightarrow \\infty$. Hence, there exists an integer number $M$ such that $M>\\left|q_{1}\\right|$ and $|P(x)| \\leq|x| \\Rightarrow|x|\\left|q_{i}\\right| \\geq M$ and this ultimately contradicts $\\left|q_{1}\\right|0\\right)$ both belonging to the set $\\{j T+1, j T+2, \\ldots, j T+T\\}$ such that $q_{m_{j}}=q_{m_{j}+k_{j}}$. Since $k_{j}90^{\\circ}$.","solution":"5. Let $O$ be the circumcenter of $A B C, E$ the midpoint of $O H$, and $R$ and $r$ the radii of the circumcircle and incircle respectively. We use the following facts from elementary geometry: $\\overrightarrow{O H}=3 \\overrightarrow{O G}, O K^{2}=R^{2}-2 R r$, and $K E=\\frac{R}{2}-r$. Hence $\\overrightarrow{K H}=2 \\overrightarrow{K E}-\\overrightarrow{K O}$ and $\\overrightarrow{K G}=\\frac{2 \\overrightarrow{K E}+\\overrightarrow{K O}}{3}$. We then obtain $$ \\overrightarrow{K H} \\cdot \\overrightarrow{K G}=\\frac{1}{3}\\left(4 K E^{2}-K O^{2}\\right)=-\\frac{2}{3} r(R-2 r)<0 . $$ Hence $\\cos \\angle G K H<0 \\Rightarrow \\angle G K H>90^{\\circ}$.","problem_type":null,"tier":0} +{"year":"1990","problem_phase":"shortlisted","problem":"6. (FRG 2) ${ }^{\\mathrm{IMO} 5}$ Two players $A$ and $B$ play a game in which they choose numbers alternately according to the following rule: At the beginning, an initial natural number $n_{0}>1$ is given. Knowing $n_{2 k}$, player $A$ may choose any $n_{2 k+1} \\in \\mathbb{N}$ such that $$ n_{2 k} \\leq n_{2 k+1} \\leq n_{2 k}^{2} $$ Then player $B$ chooses a number $n_{2 k+2} \\in \\mathbb{N}$ such that $$ \\frac{n_{2 k+1}}{n_{2 k+2}}=p^{r} $$ where $p$ is a prime number and $r \\in \\mathbb{N}$. It is stipulated that player $A$ wins the game if he (she) succeeds in choosing the number 1990, and player $B$ wins if he (she) succeeds in choosing 1. For which natural numbers $n_{0}$ can player $A$ manage to win the game, for which $n_{0}$ can player $B$ manage to win, and for which $n_{0}$ can players $A$ and $B$ each force a tie?","solution":"6. Let $W$ denote the set of all $n_{0}$ for which player $A$ has a winning strategy, $L$ the set of all $n_{0}$ for which player $B$ has a winning strategy, and $T$ the set of all $n_{0}$ for which a tie is ensured. Lemma. Assume $\\{m, m+1, \\ldots 1990\\} \\subseteq W$ and that there exists $s \\leq 1990$ such that $s \/ p^{r} \\geq m$, where $p^{r}$ is the largest degree of a prime that divides $s$. Then all integers $x$ such that $\\sqrt{s} \\leq x1990$, it follows that for $n_{0} \\in\\{45, \\ldots, 1990\\}$ player $A$ can choose 1990 in the first move. Hence $\\{45, \\ldots, 1990\\} \\subseteq W$. Using $m=45$ and selecting $s=420=2^{2} \\cdot 3 \\cdot 5 \\cdot 7$ we apply the lemma to get that all integers $x$ such that $\\sqrt{420}<21 \\leq x \\leq 1990$ are in $W$. Again, using $m=21$ and selecting $s=168=2^{3} \\cdot 3 \\cdot 7$ we apply the lemma to get that all integers $x$ such that $\\sqrt{168}<13 \\leq x \\leq 1990$ are in $W$. Selecting $s=105$ we obtain the new value for $m$ at $m=11$. Selecting $s=60$ we obtain $m=8$. Thus $\\{8, \\ldots, 1990\\} \\subseteq W$. For $n_{0}>1990$ there exists $r \\in N$ such that $2^{r} \\cdot 3^{2}1$, the final solution is $L=\\{2,3,4,5\\}, T=\\{6,7\\}$, and $W=\\{x \\in \\mathbb{N} \\mid x \\geq 8\\}$.","problem_type":null,"tier":0} +{"year":"1990","problem_phase":"shortlisted","problem":"7. (GRE 2) Let $f(0)=f(1)=0$ and $$ f(n+2)=4^{n+2} f(n+1)-16^{n+1} f(n)+n \\cdot 2^{n^{2}}, \\quad n=0,1,2,3, \\ldots $$ Show that the numbers $f(1989), f(1990), f(1991)$ are divisible by 13.","solution":"7. Let $f(n)=g(n) 2^{n^{2}}$ for all $n$. The recursion then transforms into $g(n+$ 2) $-2 g(n+1)+g(n)=n \\cdot 16^{-n-1}$ for $n \\in \\mathbb{N}_{0}$. By summing this equation from 0 to $n-1$, we get $$ g(n+1)-g(n)=\\frac{1}{15^{2}} \\cdot\\left(1-(15 n+1) 16^{-n}\\right) $$ By summing up again from 0 to $n-1$ we get $g(n)=\\frac{1}{15^{3}} \\cdot(15 n-32+$ $\\left.(15 n+2) 16^{-n+1}\\right)$. Hence $$ f(n)=\\frac{1}{15^{3}} \\cdot\\left(15 n+2+(15 n-32) 16^{n-1}\\right) \\cdot 2^{(n-2)^{2}} $$ Now let us look at the values of $f(n)$ modulo 13: $$ f(n) \\equiv 15 n+2+(15 n-32) 16^{n-1} \\equiv 2 n+2+(2 n-6) 3^{n-1} $$ We have $3^{3} \\equiv 1(\\bmod 13)$. Plugging in $n \\equiv 1(\\bmod 13)$ and $n \\equiv 1(\\bmod$ 3 ) for $n=1990$ gives us $f(1990) \\equiv 0(\\bmod 13)$. We similarly calculate $f(1989) \\equiv 0$ and $f(1991) \\equiv 0(\\bmod 13)$.","problem_type":null,"tier":0} +{"year":"1990","problem_phase":"shortlisted","problem":"8. (HUN 1) For a given positive integer $k$ denote the square of the sum of its digits by $f_{1}(k)$ and let $f_{n+1}(k)=f_{1}\\left(f_{n}(k)\\right)$. Determine the value of $f_{1991}\\left(2^{1990}\\right)$.","solution":"8. Since $2^{1990}<8^{700}<10^{700}$, we have $f_{1}\\left(2^{1990}\\right)<(9 \\cdot 700)^{2}<4 \\cdot 10^{7}$. We then have $f_{2}\\left(2^{1990}\\right)<(3+9 \\cdot 7)^{2}<4900$ and finally $f_{3}\\left(2^{1990}\\right)<(3+9 \\cdot 3)^{2}=30^{2}$. It is easily shown that $f_{k}(n) \\equiv f_{k-1}(n)^{2}(\\bmod 9)$. Since $2^{6} \\equiv 1(\\bmod 9)$, we have $2^{1990} \\equiv 2^{4} \\equiv 7$ (all congruences in this problem will be $\\bmod 9$ ). It follows that $f_{1}\\left(2^{1990}\\right) \\equiv 7^{2} \\equiv 4$ and $f_{2}\\left(2^{1990}\\right) \\equiv 4^{2} \\equiv 7$. Indeed, it follows that $f_{2 k}\\left(2^{1990}\\right) \\equiv 7$ and $f_{2 k+1}\\left(2^{1990}\\right) \\equiv 4$ for all integer $k>0$. Thus $f_{3}\\left(2^{1990}\\right)=r^{2}$ where $r<30$ is an integer and $r \\equiv f_{2}\\left(2^{1990}\\right) \\equiv 7$. It follows that $r \\in\\{7,16,25\\}$ and hence $f_{3}\\left(2^{1990}\\right) \\in\\{49,256,625\\}$. It follows that $f_{4}\\left(2^{1990}\\right)=169, f_{5}\\left(2^{1990}\\right)=256$, and inductively $f_{2 k}\\left(2^{1990}\\right)=169$ and $f_{2 k+1}\\left(2^{1990}\\right)=256$ for all integer $k>1$. Hence $f_{1991}\\left(2^{1990}\\right)=256$.","problem_type":null,"tier":0} +{"year":"1990","problem_phase":"shortlisted","problem":"9. (HUN 3) The incenter of the triangle $A B C$ is $K$. The midpoint of $A B$ is $C_{1}$ and that of $A C$ is $B_{1}$. The lines $C_{1} K$ and $A C$ meet at $B_{2}$, the lines $B_{1} K$ and $A B$ at $C_{2}$. If the areas of the triangles $A B_{2} C_{2}$ and $A B C$ are equal, what is the measure of angle $\\angle C A B$ ?","solution":"9. Let $a, b, c$ be the lengths of the sides of $\\triangle A B C, s=\\frac{a+b+c}{2}, r$ the inradius of the triangle, and $c_{1}$ and $b_{1}$ the lengths of $A B_{2}$ and $A C_{2}$ respectively. As usual we will denote by $S(X Y Z)$ the area of $\\triangle X Y Z$. We have $$ \\begin{gathered} S\\left(A C_{1} B_{2}\\right)=\\frac{A C_{1} \\cdot A B_{2}}{A C \\cdot A B} S(A B C)=\\frac{c_{1} r s}{2 b} \\\\ S\\left(A K B_{2}\\right)=\\frac{c_{1} r}{2}, \\quad S\\left(A C_{1} K\\right)=\\frac{c r}{4} \\end{gathered} $$ From $S\\left(A C_{1} B_{2}\\right)=S\\left(A K B_{2}\\right)+S\\left(A C_{1} K\\right)$ we get $\\frac{c_{1} r s}{2 b}=\\frac{c_{1} r}{2}+\\frac{c r}{4}$; therefore $(a-b+c) c_{1}=b c$. By looking at the area of $\\triangle A B_{1} C_{2}$ we similarly obtain $(a+b-c) b_{1}=b c$. From these two equations and from $S(A B C)=S\\left(A B_{2} C_{2}\\right)$, from which we have $b_{1} c_{1}=b c$, we obtain $$ a^{2}-(b-c)^{2}=b c \\Rightarrow \\frac{b^{2}+c^{2}-a^{2}}{2 b c}=\\cos (\\angle B A C)=\\frac{1}{2} \\Rightarrow \\angle B A C=60^{\\circ} $$","problem_type":null,"tier":0} +{"year":"1991","problem_phase":"shortlisted","problem":"1. (PHI 3) Let $A B C$ be any triangle and $P$ any point in its interior. Let $P_{1}, P_{2}$ be the feet of the perpendiculars from $P$ to the two sides $A C$ and $B C$. Draw $A P$ and $B P$, and from $C$ drop perpendiculars to $A P$ and $B P$. Let $Q_{1}$ and $Q_{2}$ be the feet of these perpendiculars. Prove that the lines $Q_{1} P_{2}, Q_{2} P_{1}$, and $A B$ are concurrent.","solution":"1. All the angles $\\angle P P_{1} C, \\angle P P_{2} C, \\angle P Q_{1} C, \\angle P Q_{2} C$ are right, hence $P_{1}, P_{2}$, $Q_{1}, Q_{2}$ lie on the circle with diameter $P C$. The result now follows immediately from Pascal's theorem applied to the hexagon $P_{1} P P_{2} Q_{1} C Q_{2}$. It tells us that the points of intersection of the three pairs of lines $P_{1} C, P Q_{1}$ (intersection $A$ ), $P_{1} Q_{2}, P_{2} Q_{1}$ (intersection ![](https:\/\/cdn.mathpix.com\/cropped\/2024_11_18_8e985d6b9c83aa3e9d0eg-553.jpg?height=333&width=335&top_left_y=376&top_left_x=919) $X)$ and $P Q_{2}, P_{2} C$ (intersection $B$ ) are collinear.","problem_type":null,"tier":0} +{"year":"1991","problem_phase":"shortlisted","problem":"10. (USA 5) ${ }^{\\mathrm{IMO} 4}$ Suppose $G$ is a connected graph with $n$ edges. Prove that it is possible to label the edges of $G$ from 1 to $n$ in such a way that in every vertex $v$ of $G$ with two or more incident edges, the set of numbers labeling those edges has no common divisor greater than 1.","solution":"10. We start at some vertex $v_{0}$ and walk along distinct edges of the graph, numbering them $1,2, \\ldots$ in the order of appearance, until this is no longer possible without reusing an edge. If there are still edges which are not numbered, one of them has a vertex which has already been visited (else $G$ would not be connected). Starting from this vertex, we continue to walk along unused edges resuming the numbering, until we eventually get stuck. Repeating this procedure as long as possible, we shall number all the edges. Let $v$ be a vertex which is incident with $e \\geq 2$ edges. If $v=v_{0}$, then it is on the edge 1 , so the gcd at $v$ is 1 . If $v \\neq v_{0}$, suppose that it was reached for the first time by the edge $r$. At that time there was at least one unused edge incident with $v$ (as $e \\geq 2$ ), hence one of them was labelled by $r+1$. The gcd at $v$ again equals $\\operatorname{gcd}(r, r+1)=1$.","problem_type":null,"tier":0} +{"year":"1991","problem_phase":"shortlisted","problem":"11. (AUS 4) Prove that $$ \\sum_{m=0}^{995} \\frac{(-1)^{m}}{1991-m}\\binom{1991-m}{m}=\\frac{1}{1991} $$","solution":"11. To start with, observe that $\\frac{1}{n-m}\\binom{n-m}{m}=\\frac{1}{n}\\left[\\binom{n-m}{m}+\\binom{n-m-1}{m-1}\\right]$. For $n=1,2, \\ldots$ set $S_{n}=\\sum_{m=0}^{[n \/ 2]}(-1)^{m}\\binom{n-m}{m}$. Using the identity $\\binom{m}{k}=$ $\\binom{m-1}{k}+\\binom{m-1}{k-1}$ we obtain the following relation for $S_{n}$ : $$ \\begin{aligned} S_{n+1} & =\\sum_{m}(-1)^{m}\\binom{n-m+1}{m} \\\\ & =\\sum_{m}(-1)^{m}\\binom{n-m}{m}+\\sum_{m}(-1)^{m}\\binom{n-m}{m-1}=S_{n}-S_{n-1} . \\end{aligned} $$ Since the initial members of the sequence $S_{n}$ are $1,1,0,-1,-1,0,1,1, \\ldots$, we thus find that $S_{n}$ is periodic with period 6 . Now the sum from the problem reduces to $$ \\begin{gathered} \\frac{1}{1991}\\binom{1991}{0}-\\frac{1}{1991}\\left[\\binom{1990}{1}+\\binom{1989}{0}\\right]+\\cdots-\\frac{1}{1991}\\left[\\binom{996}{995}+\\binom{995}{994}\\right] \\\\ =\\frac{1}{1991}\\left(S_{1991}-S_{1989}\\right)=\\frac{1}{1991}(0-(-1))=\\frac{1}{1991} . \\end{gathered} $$","problem_type":null,"tier":0} +{"year":"1991","problem_phase":"shortlisted","problem":"12. $(\\mathbf{C H N} 3)^{\\mathrm{IMO} 3}$ Let $S=\\{1,2,3, \\ldots, 280\\}$. Find the minimal natural number $n$ such that in any $n$-element subset of $S$ there are five numbers that are pairwise relatively prime.","solution":"12. Let $A_{m}$ be the set of those elements of $S$ which are divisible by $m$. By the inclusion-exclusion principle, the number of elements divisible by $2,3,5$ or 7 equals $$ \\begin{aligned} & \\left|A_{2} \\cup A_{3} \\cup A_{5} \\cup A_{7}\\right| \\\\ & =\\left|A_{2}\\right|+\\left|A_{3}\\right|+\\left|A_{5}\\right|+\\left|A_{7}\\right|-\\left|A_{6}\\right|-\\left|A_{10}\\right|-\\left|A_{14}\\right|-\\left|A_{15}\\right| \\\\ & \\quad-\\left|A_{21}\\right|-\\left|A_{35}\\right|+\\left|A_{30}\\right|+\\left|A_{42}\\right|+\\left|A_{70}\\right|+\\left|A_{105}\\right|-\\left|A_{210}\\right| \\\\ & =140+93+56+40-46-28-20-18 \\\\ & \\quad-13-8+9+6+4+2-1=216 . \\end{aligned} $$ Among any five elements of the set $A_{2} \\cup A_{3} \\cup A_{5} \\cup A_{7}$, one of the sets $A_{2}, A_{3}, A_{5}, A_{7}$ contains at least two, and those two are not relatively prime. Therefore $n>216$. We claim that the answer is $n=217$. First notice that the set $A_{2} \\cup A_{3} \\cup$ $A_{5} \\cup A_{7}$ consists of four prime $(2,3,5,7)$ and 212 composite numbers. The set $S \\backslash A$ contains exactly 8 composite numbers: namely, $11^{2}, 11 \\cdot 13,11$. $17,11 \\cdot 19,11 \\cdot 23,13^{2}, 13 \\cdot 17,13 \\cdot 19$. Thus $S$ consists of the unity, 220 composite numbers and 59 primes. Let $A$ be a 217 -element subset of $S$, and suppose that there are no five pairwise relatively prime numbers in $A$. Then $A$ can contain at most 4 primes (or unity and three primes) and at least 213 composite numbers. Hence the set $S \\backslash A$ contains at most 7 composite numbers. Consequently, at least one of the following 8 five-element sets is disjoint with $S \\backslash A$, and is thus entirely contained in $A$ : $$ \\begin{array}{ll} \\{2 \\cdot 23,3 \\cdot 19,5 \\cdot 17,7 \\cdot 13,11 \\cdot 11\\}, & \\{2 \\cdot 29,3 \\cdot 23,5 \\cdot 19,7 \\cdot 17,11 \\cdot 13\\}, \\\\ \\{2 \\cdot 31,3 \\cdot 29,5 \\cdot 23,7 \\cdot 19,11 \\cdot 17\\}, & \\{2 \\cdot 37,3 \\cdot 31,5 \\cdot 29,7 \\cdot 23,11 \\cdot 19\\}, \\\\ \\{2 \\cdot 41,3 \\cdot 37,5 \\cdot 31,7 \\cdot 29,11 \\cdot 23\\}, & \\{2 \\cdot 43,3 \\cdot 41,5 \\cdot 37,7 \\cdot 31,13 \\cdot 17\\}, \\\\ \\{2 \\cdot 47,3 \\cdot 43,5 \\cdot 41,7 \\cdot 37,13 \\cdot 19\\}, & \\{2 \\cdot 2,3 \\cdot 3,5 \\cdot 5,7 \\cdot 7,13 \\cdot 13\\} . \\end{array} $$ As each of these sets consists of five numbers relatively prime in pairs, the claim is proved.","problem_type":null,"tier":0} +{"year":"1991","problem_phase":"shortlisted","problem":"13. (POL 4) Given any integer $n \\geq 2$, assume that the integers $a_{1}, a_{2}, \\ldots, a_{n}$ are not divisible by $n$ and, moreover, that $n$ does not divide $a_{1}+a_{2}+$ $\\cdots+a_{n}$. Prove that there exist at least $n$ different sequences $\\left(e_{1}, e_{2}, \\cdots, e_{n}\\right)$ consisting of zeros or ones such that $e_{1} a_{1}+e_{2} a_{2}+\\cdots+e_{n} a_{n}$ is divisible by $n$.","solution":"13. Call a sequence $e_{1}, \\ldots, e_{n}$ good if $e_{1} a_{1}+\\cdots+e_{n} a_{n}$ is divisible by $n$. Among the sums $s_{0}=0, s_{1}=a_{1}, s_{2}=a_{1}+a_{2}, \\ldots, s_{n}=a_{1}+\\cdots+a_{n}$, two give the same remainder modulo $n$, and their difference corresponds to a good sequence. To show that, permuting the $a_{i}$ 's, we can find $n-1$ different sequences, we use the following Lemma. Let $A$ be a $k \\times n(k \\leq n-2)$ matrix of zeros and ones, whose every row contains at least one 0 and at least two 1 's. Then it is possible to permute columns of $A$ is such a way that in any row 1 's do not form a block. Proof. We will use the induction on $k$. The case $k=1$ and arbitrary $n \\geq 3$ is trivial. Suppose that $k \\geq 2$ and that for $k-1$ and any $n \\geq k+1$ the lemma is true. Consider a $k \\times n$ matrix $A, n \\geq k+2$. We mark an element $a_{i j}$ if either it is the only zero in the $i$-th row, or one of the 1 's in the row if it contains exactly two 1 's. Since $n \\geq 4$, every row contains at most two marked elements, which adds up to at most $2 k<2 n$ marked elements in total. It follows that there is a column with at most one marked element. Assume w.l.o.g. that it is the first column and that $a_{1 j}$ isn't marked for $j>1$. The matrix $B$, obtained by omitting the first row and first column from $A$, satisfies the conditions of the lemma. Therefore, we can permute columns of $B$ and get the required form. Considered as a permutation of column of $A$, this permutation may leave a block of 1's only in the first row of $A$. In the case that it is so, if $a_{11}=1$ we put the first column in the last place, otherwise we put it between any two columns having 1's in the first row. The obtained matrix has the required property. Suppose now that we have got $k$ different nontrivial good sequences $e_{1}^{i}, \\ldots, e_{n}^{i}, i=1, \\ldots, k$, and that $k \\leq n-2$. The matrix $A=\\left(e_{j}^{i}\\right)$ fulfils the conditions of Lemma, hence there is a permutation $\\sigma$ from Lemma. Now among the sums $s_{0}=0, s_{1}=a_{\\sigma(1)}, s_{2}=a_{\\sigma(1)}+a_{\\sigma(2)}$, $\\ldots, s_{n}=a_{\\sigma(1)}+\\cdots+a_{\\sigma(n)}$, two give the same remainder modulo $n$. Let $s_{p} \\equiv s_{q}(\\bmod n), pm>$ 0 , as large as we like, such that $10^{k} \\equiv 10^{m}(\\bmod T)$, using for example Euler's theorem. It is obvious that $a_{10^{k}-1}=a_{10^{k}}$ and hence, taking $k$ sufficiently large and using the periodicity, we see that $$ a_{2 \\cdot 10^{k}-10^{m}-1}=a_{10^{k}-1}=a_{10^{k}}=a_{2 \\cdot 10^{k}-10^{m}} $$ Since $\\left(2 \\cdot 10^{k}-10^{m}\\right)!=\\left(2 \\cdot 10^{k}-10^{m}\\right)\\left(2 \\cdot 10^{k}-10^{m}-1\\right)!$ and the last nonzero digit of $2 \\cdot 10^{k}-10^{m}$ is nine, we must have $a_{2 \\cdot 10^{k}-10^{m}-1}=5$ (if $s$ is a digit, the last digit of $9 s$ is $s$ only if $s=5$ ). But this means that 5 divides $n$ ! with a greater power than 2 does, which is impossible. Indeed, if the exponents of these powers are $\\alpha_{2}, \\alpha_{5}$ respectively, then $\\alpha_{5}=$ $[n \/ 5]+\\left[n \/ 5^{2}\\right]+\\cdots \\leq \\alpha_{2}=[n \/ 2]+\\left[n \/ 2^{2}\\right]+\\cdots$.","problem_type":null,"tier":0} +{"year":"1991","problem_phase":"shortlisted","problem":"16. (ROM 1) ${ }^{\\mathrm{IMO} 2}$ Let $n>6$ and $a_{1}3$. Then $r=p-1$ and $a_{k+1}=a_{1}+k(p-1)=1+k(p-1)$. Since $n-1$ also must belong to the progression, we have $p-1 \\mid n-2$. Let $q$ be any prime divisor of $p-1$. Then also $q \\mid n-2$. On the other hand, since $q0)$ modulo 3 , we get that $5^{z} \\equiv 1(\\bmod 3)$, hence $z$ is even, say $z=2 z_{1}$. The equation then becomes $3^{x}=5^{2 z_{1}}-4^{y}=\\left(5^{z_{1}}-2^{y}\\right)\\left(5^{z_{1}}+2^{y}\\right)$. Each factor $5^{z_{1}}-2^{y}$ and $5^{z_{1}}+2^{y}$ is a power of 3 , for which the only possibility is $5^{z_{1}}+2^{y}=3^{x}$ and $5^{z_{1}}-2^{y}=$ 1. Again modulo 3 these equations reduce to $(-1)^{z_{1}}+(-1)^{y}=0$ and $(-1)^{z_{1}}-(-1)^{y}=1$, implying that $z_{1}$ is odd and $y$ is even. Particularly, $y \\geq 2$. Reducing the equation $5^{z_{1}}+2^{y}=3^{x}$ modulo 4 we get that $3^{x} \\equiv 1$, hence $x$ is even. Now if $y>2$, modulo 8 this equation yields $5 \\equiv 5^{z_{1}} \\equiv$ $3^{x} \\equiv 1$, a contradiction. Hence $y=2, z_{1}=1$. The only solution of the original equation is $x=y=z=2$.","problem_type":null,"tier":0} +{"year":"1991","problem_phase":"shortlisted","problem":"18. (BUL 1) Find the highest degree $k$ of 1991 for which $1991^{k}$ divides the number $$ 1990^{1991^{1992}}+1992^{1991^{1990}} $$","solution":"18. For integers $a>0, n>0$ and $\\alpha \\geq 0$, we shall write $a^{\\alpha} \\| n$ when $a^{\\alpha} \\mid n$ and $a^{\\alpha+1} \\nmid n$. Lemma. For every odd number $a \\geq 3$ and an integer $n \\geq 0$ it holds that $$ a^{n+1} \\|(a+1)^{a^{n}}-1 \\quad \\text { and } \\quad a^{n+1} \\|(a-1)^{a^{n}}+1 $$ Proof. We shall prove the first relation by induction (the second is analogous). For $n=0$ the statement is obvious. Suppose that it holds for some $n$, i.e. that $(1+a)^{a^{n}}=1+N a^{n+1}, a \\nmid N$. Then $$ (1+a)^{a^{n+1}}=\\left(1+N a^{n+1}\\right)^{a}=1+a \\cdot N a^{n+1}+\\binom{a}{2} N^{2} a^{2 n+2}+M a^{3 n+3} $$ for some integer $M$. Since $\\binom{a}{2}$ is divisible by $a$ for $a$ odd, we deduce that the part of the above sum behind $1+a \\cdot N a^{n+1}$ is divisible by $a^{n+3}$. Hence $(1+a)^{a^{n+1}}=1+N^{\\prime} a^{n+2}$, where $a \\nmid N^{\\prime}$. It follows immediately from Lemma that $$ 1991^{1993} \\| 1990^{1991^{1992}}+1 \\quad \\text { and } \\quad 1991^{1991} \\| 1992^{1991^{1990}}-1 $$ Adding these two relations we obtain immediately that $k=1991$ is the desired value.","problem_type":null,"tier":0} +{"year":"1991","problem_phase":"shortlisted","problem":"19. (IRE 5) Let $a$ be a rational number with $0a_{n}$, the sequence $\\left\\{a_{n}\\right\\}$ is strictly increasing. Hence the set of values of $\\cos \\left(2^{n} \\pi a\\right), n=0,1,2, \\ldots$, is infinite (because $\\sqrt{17}$ is irrational). However, if $a$ were rational, then the set of values of $\\cos m \\pi a, m=1,2, \\ldots$, would be finite, a contradiction. Therefore the only possible value for $a$ is $2 \/ 3$.","problem_type":null,"tier":0} +{"year":"1991","problem_phase":"shortlisted","problem":"2. (JAP 5) For an acute triangle $A B C, M$ is the midpoint of the segment $B C, P$ is a point on the segment $A M$ such that $P M=B M, H$ is the foot of the perpendicular line from $P$ to $B C, Q$ is the point of intersection of segment $A B$ and the line passing through $H$ that is perpendicular to $P B$, and finally, $R$ is the point of intersection of the segment $A C$ and the line passing through $H$ that is perpendicular to $P C$. Show that the circumcircle of $\\triangle Q H R$ is tangent to the side $B C$ at point $H$.","solution":"2. Let $H Q$ meet $P B$ at $Q^{\\prime}$ and $H R$ meet $P C$ at $R^{\\prime}$. From $M P=M B=M C$ we have $\\angle B P C=90^{\\circ}$. So $P R^{\\prime} H Q^{\\prime}$ is a rectangle. Since $P H$ is perpendicular to $B C$, it follows that the circle with diameter $P H$, through $P, R^{\\prime}, H, Q^{\\prime}$, is tangent to $B C$. It is now sufficient to show that $Q R$ is parallel to $Q^{\\prime} R^{\\prime}$. Let $C P$ meet $A B$ at $X$, and $B P$ meet $A C$ at $Y$. Since $P$ is on the median, it follows (for ![](https:\/\/cdn.mathpix.com\/cropped\/2024_11_18_8e985d6b9c83aa3e9d0eg-553.jpg?height=403&width=432&top_left_y=806&top_left_x=877) example, by Ceva's theorem) that $A X \/ X B=A Y \/ Y C$, i.e. that $X Y$ is parallel to $B C$. Consequently, $P Y \/ B P=P X \/ C P$. Since $H Q$ is parallel to $C X$, we have $Q Q^{\\prime} \/ H Q^{\\prime}=$ $P X \/ C P$ and similarly $R R^{\\prime} \/ H R^{\\prime}=P Y \/ B P$. It follows that $Q Q^{\\prime} \/ H Q^{\\prime}=$ $R R^{\\prime} \/ H R^{\\prime}$, hence $Q R$ is parallel to $Q^{\\prime} R^{\\prime}$ as required. Second solution. It suffices to show that $\\angle R H C=\\angle R Q H$, or equivalently $R H: Q H=P C: P B$. We assume $P C: P B=1: x$. Let $X \\in A B$ and $Y \\in A C$ be points such that $M X \\perp P B$ and $M Y \\perp P C$. Since $M X$ bisects $\\angle A M B$ and $M Y$ bisects $A M C$, we deduce $$ \\begin{aligned} & A X: X B=A M: M B=A Y: Y C \\Rightarrow X Y \\| B C \\Rightarrow \\\\ & \\quad \\Rightarrow \\triangle X Y M \\sim \\triangle C B P \\Rightarrow X M: M Y=1: x . \\end{aligned} $$ Now from $C H: H B=1: x^{2}$ we obtain $R H: M Y=C H: C M=1: \\frac{1+x^{2}}{2}$ and $Q H: M X=B H: B M=x^{2}: \\frac{1+x^{2}}{2}$. Therefore $$ R H: Q H=\\frac{2}{1+x^{2}} M Y: \\frac{2 x^{2}}{1+x^{2}} M X=1: x $$","problem_type":null,"tier":0} +{"year":"1991","problem_phase":"shortlisted","problem":"20. (IRE 3) Let $\\alpha$ be the positive root of the equation $x^{2}=1991 x+1$. For natural numbers $m, n$ define $$ m * n=m n+[\\alpha m][\\alpha n] $$ where $[x]$ is the greatest integer not exceeding $x$. Prove that for all natural numbers $p, q, r$, $$ (p * q) * r=p *(q * r) $$","solution":"20. We prove the result with 1991 replaced by any positive integer $k$. For natural numbers $p, q$, let $\\epsilon=(\\alpha p-[\\alpha p])(\\alpha q-[\\alpha q])$. Then $0<\\epsilon<1$ and $$ \\epsilon=\\alpha^{2} p q-\\alpha(p[\\alpha q]+q[\\alpha p])+[\\alpha p][\\alpha q] . $$ Multiplying this equality by $\\alpha-k$ and using $\\alpha^{2}=k \\alpha+1$, i.e. $\\alpha(\\alpha-k)=1$, we get $$ (\\alpha-k) \\epsilon=\\alpha(p q+[\\alpha p][\\alpha q])-(p[\\alpha q]+q[\\alpha p]+k[\\alpha p][\\alpha q]) $$ Since $0<(\\alpha-k) \\epsilon<1$, we have $[\\alpha(p * q)]=p[\\alpha q]+q[\\alpha p]+k[\\alpha p][\\alpha q]$. Now $$ \\begin{aligned} (p * q) * r & =(p * q) r+[\\alpha(p * q)][\\alpha r]= \\\\ & =p q r+[\\alpha p][\\alpha q] r+[\\alpha q][\\alpha r] p+[\\alpha r][\\alpha p] q+k[\\alpha p][\\alpha q][\\alpha r] . \\end{aligned} $$ Since the last expression is symmetric, the same formula is obtained for $p *(q * r)$.","problem_type":null,"tier":0} +{"year":"1991","problem_phase":"shortlisted","problem":"21. (HKG 6) Let $f(x)$ be a monic polynomial of degree 1991 with integer coefficients. Define $g(x)=f^{2}(x)-9$. Show that the number of distinct integer solutions of $g(x)=0$ cannot exceed 1995.","solution":"21. The polynomial $g(x)$ factorizes as $g(x)=f(x)^{2}-9=(f(x)-3)(f(x)+3)$. If one of the equations $f(x)+3=0$ and $f(x)-3=0$ has no integer solutions, then the number of integer solutions of $g(x)=0$ clearly does not exceed 1991. Suppose now that both $f(x)+3=0$ and $f(x)-3=0$ have integer solutions. Let $x_{1}, \\ldots, x_{k}$ be distinct integer solutions of the former, and $x_{k+1}, \\ldots, x_{k+l}$ be distinct integer solutions of the latter equation. There exist monic polynomials $p(x), q(x)$ with integer coefficients such that $f(x)+3=\\left(x-x_{1}\\right)\\left(x-x_{2}\\right) \\ldots\\left(x-x_{k}\\right) p(x)$ and $f(x)-3=$ $\\left(x-x_{k+1}\\right)\\left(x-x_{k+2}\\right) \\ldots\\left(x-x_{k+l}\\right) q(x)$. Thus we obtain $\\left(x-x_{1}\\right)\\left(x-x_{2}\\right) \\ldots\\left(x-x_{k}\\right) p(x)-\\left(x-x_{k+1}\\right)\\left(x-x_{k+2}\\right) \\ldots\\left(x-x_{k+l}\\right) q(x)=6$. Putting $x=x_{k+1}$ we get $\\left(x_{k+1}-x_{1}\\right)\\left(x_{k+1}-x_{2}\\right) \\cdots\\left(x_{k+1}-x_{k}\\right) \\mid 6$, and since the product of more than four distinct integers cannot divide 6 , this implies $k \\leq 4$. Similarly $l \\leq 4$; hence $g(x)=0$ has at most 8 distinct integer solutions. Remark. The proposer provided a solution for the upper bound of 1995 roots which was essentially the same as that of (IMO74-6).","problem_type":null,"tier":0} +{"year":"1991","problem_phase":"shortlisted","problem":"22. (USA 4) Real constants $a, b, c$ are such that there is exactly one square all of whose vertices lie on the cubic curve $y=x^{3}+a x^{2}+b x+c$. Prove that the square has sides of length $\\sqrt[4]{72}$.","solution":"22. Suppose w.l.o.g. that the center of the square is at the origin $O(0,0)$. We denote the curve $y=f(x)=x^{3}+a x^{2}+b x+c$ by $\\gamma$ and the vertices of the square by $A, B, C, D$ in this order. At first, the symmetry with respect to the point $O$ maps $\\gamma$ into the curve $\\bar{\\gamma}\\left(y=f(-x)=x^{3}-a x^{2}+b x-c\\right)$. Obviously $\\bar{\\gamma}$ also passes through $A, B, C, D$, and thus has four different intersection points with $\\gamma$. Then $2 a x^{2}+2 c$ has at least four distinct solution, which implies $a=c=0$. Particularly, $\\gamma$ passes through $O$ and intersects all quadrants, and hence $b<0$. Further, the curve $\\gamma^{\\prime}$, obtained by rotation of $\\gamma$ around $O$ for $90^{\\circ}$, has an equation $-x=f(y)$ and also contains the points $A, B, C, D$ and $O$. The intersection points $(x, y)$ of $\\gamma \\cap \\gamma^{\\prime}$ are determined by $-x=f(f(x))$, and hence they are roots of a polynomial $p(x)=f(f(x))+x$ of 9-th degree. But the number of times that one cubic actually crosses the other in each quadrant is in the general case even (draw the picture!), and since $A B C D$ is the only square lying on $\\gamma \\cap \\gamma^{\\prime}$, the intersection points $A, B, C, D$ must be double. It follows that $$ p(x)=x[(x-r)(x+r)(x-s)(x+s)]^{2}, $$ where $r, s$ are the $x$-coordinates of $A$ and $B$. On the other hand, $p(x)$ is defined by $\\left(x^{3}+b x\\right)^{3}+b\\left(x^{3}+b x\\right)+x$, and therefore equating of coefficients with (1) yields $$ \\begin{array}{cc} 3 b=-2\\left(r^{2}+s^{2}\\right), & 3 b^{2}=\\left(r^{2}+s^{2}\\right)^{2}+2 r^{2} s^{2}, \\\\ b\\left(b^{2}+1\\right)=-2 r^{2} s^{2}\\left(r^{2}+s^{2}\\right), & b^{2}+1=r^{4} s^{4} . \\end{array} $$ Straightforward solving this system of equations gives $b=-\\sqrt{8}$ and $r^{2}+$ $s^{2}=\\sqrt{18}$. The line segment from $O$ to $(r, s)$ is half a diagonal of the square, and thus a side of the square has length $a=\\sqrt{2\\left(r^{2}+s^{2}\\right)}=\\sqrt[4]{72}$.","problem_type":null,"tier":0} +{"year":"1991","problem_phase":"shortlisted","problem":"23. (IND 2) Let $f$ and $g$ be two integer-valued functions defined on the set of all integers such that (a) $f(m+f(f(n)))=-f(f(m+1)-n$ for all integers $m$ and $n$; (b) $g$ is a polynomial function with integer coefficients and $g(n)=g(f(n))$ for all integers $n$. Determine $f(1991)$ and the most general form of $g$.","solution":"23. From (i), replacing $m$ by $f(f(m))$, we get $$ f(f(f(m))+f(f(n)))=-f(f(f(f(m))+1))-n $$ analogously $\\quad f(f(f(n))+f(f(m)))=-f(f(f(f(n))+1))-m$. From these relations we get $f(f(f(f(m))+1))-f(f(f(f(n))+1))=m-n$. Again from (i), $$ \\begin{aligned} & f(f(f(f(m))+1))=f(-m-f(f(2))) \\\\ & \\text { and } \\quad f(f(f(f(n))+1))=f(-n-f(f(2))) . \\end{aligned} $$ Setting $f(f(2))=k$ we obtain $f(-m-k)-f(-n-k)=m-n$ for all integers $m, n$. This implies $f(m)=f(0)-m$. Then also $f(f(m))=m$, and using this in (i) we finally get $$ f(n)=-n-1 \\quad \\text { for all integers } n $$ Particularly $f(1991)=-1992$. From (ii) we obtain $g(n)=g(-n-1)$ for all integers $n$. Since $g$ is a polynomial, it must also satisfy $g(x)=g(-x-1)$ for all real $x$. Let us now express $g$ as a polynomial on $x+1 \/ 2: g(x)=h(x+1 \/ 2)$. Then $h$ satisfies $h(x+1 \/ 2)=h(-x-1 \/ 2)$, i.e. $h(y)=h(-y)$, hence it is a polynomial in $y^{2}$; thus $g$ is a polynomial in $(x+1 \/ 2)^{2}=x^{2}+x+1 \/ 4$. Hence $g(n)=p\\left(n^{2}+n\\right)$ (for some polynomial $p$ ) is the most general form of $g$.","problem_type":null,"tier":0} +{"year":"1991","problem_phase":"shortlisted","problem":"24. (IND 1) An odd integer $n \\geq 3$ is said to be \"nice\" if there is at least one permutation $a_{1}, a_{2}, \\ldots, a_{n}$ of $1,2, \\ldots, n$ such that the $n$ sums $a_{1}-a_{2}+$ $a_{3}-\\cdots-a_{n-1}+a_{n}, a_{2}-a_{3}+a_{4}-\\cdots-a_{n}+a_{1}, a_{3}-a_{4}+a_{5}-\\cdots-a_{1}+$ $a_{2}, \\ldots, a_{n}-a_{1}+a_{2}-\\cdots-a_{n-2}+a_{n-1}$ are all positive. Determine the set of all \"nice\" integers.","solution":"24. Let $y_{k}=a_{k}-a_{k+1}+a_{k+2}-\\cdots+a_{k+n-1}$ for $k=1,2, \\ldots, n$, where we define $x_{i+n}=x_{i}$ for $1 \\leq i \\leq n$. We then have $y_{1}+y_{2}=2 a_{1}, y_{2}+y_{3}=$ $2 a_{2}, \\ldots, y_{n}+y_{1}=2 a_{n}$. (i) Let $n=4 k-1$ for some integer $k>0$. Then for each $i=1,2, \\ldots, n$ we have that $y_{i}=\\left(a_{i}+a_{i+1}+\\cdots+a_{i-1}\\right)-2\\left(a_{i+1}+a_{i+3}+\\cdots+a_{i-2}\\right)=1+$ $2+\\cdots+(4 k-1)-2\\left(a_{i+1}+a_{i+3}+\\cdots+a_{i-2}\\right)$ is even. Suppose now that $a_{1}, \\ldots, a_{n}$ is a good permutation. Then each $y_{i}$ is positive and even, so $y_{i} \\geq 2$. But for some $t \\in\\{1, \\ldots, n\\}$ we must have $a_{t}=1$, and thus $y_{t}+y_{t+1}=2 a_{t}=2$ which is impossible. Hence the numbers $n=4 k-1$ are not good. (ii) Let $n=4 k+1$ for some integer $k>0$. Then $2,4, \\ldots, 4 k, 4 k+1,4 k-$ $1, \\ldots, 3,1$ is a permutation with the desired property. Indeed, in this case $y_{1}=y_{4 k+1}=1, y_{2}=y_{4 k}=3, \\ldots, y_{2 k}=y_{2 k+2}=4 k-1$, $y_{2 k+1}=4 k+1$. Therefore all nice numbers are given by $4 k+1, k \\in \\mathbb{N}$.","problem_type":null,"tier":0} +{"year":"1991","problem_phase":"shortlisted","problem":"25. (USA 1) Suppose that $n \\geq 2$ and $x_{1}, x_{2}, \\ldots, x_{n}$ are real numbers between 0 and 1 (inclusive). Prove that for some index $i$ between 1 and $n-1$ the inequality $$ x_{i}\\left(1-x_{i+1}\\right) \\geq \\frac{1}{4} x_{1}\\left(1-x_{n}\\right) $$ holds.","solution":"25. Since replacing $x_{1}$ by 1 can only reduce the set of indices $i$ for which the desired inequality holds, we may assume $x_{1}=1$. Similarly we may assume $x_{n}=0$. Now we can let $i$ be the largest index such that $x_{i}>1 \/ 2$. Then $x_{i+1} \\leq 1 \/ 2$, hence $$ x_{i}\\left(1-x_{i+1}\\right) \\geq \\frac{1}{4}=\\frac{1}{4} x_{1}\\left(1-x_{n}\\right) . $$","problem_type":null,"tier":0} +{"year":"1991","problem_phase":"shortlisted","problem":"26. (CZS 1) Let $n \\geq 2$ be a natural number and let the real numbers $p, a_{1}, a_{2}, \\ldots, a_{n}, b_{1}, b_{2}, \\ldots, b_{n}$ satisfy $1 \/ 2 \\leq p \\leq 1,0 \\leq a_{i}, 0 \\leq b_{i} \\leq p$, $i=1, \\ldots, n$, and $\\sum_{i=1}^{n} a_{i}=\\sum_{i=1}^{n} b_{i}=1$. Prove the inequality $$ \\sum_{i=1}^{n} b_{i} \\prod_{\\substack{j=1 \\\\ j \\neq i}}^{n} a_{j} \\leq \\frac{p}{(n-1)^{n-1}} $$","solution":"26. Without loss of generality we can assume $b_{1} \\geq b_{2} \\geq \\cdots \\geq b_{n}$. We denote by $A_{i}$ the product $a_{1} a_{2} \\ldots a_{i-1} a_{i+1} \\ldots a_{n}$. If for some $i0 $$ because $x_{k-1}+x_{k} \\leq \\frac{2}{3}\\left(x_{1}+x_{k-1}+x_{k-2}\\right) \\leq \\frac{2}{3}$. Repeating this procedure we can reduce the number of nonzero $x_{i}$ 's to two, increasing the value of $F$ in each step. It remains to maximize $F$ over $n$-tuples $\\left(x_{1}, x_{2}, 0, \\ldots, 0\\right)$ with $x_{1}, x_{2} \\geq 0, x_{1}+x_{2}=1$ : in this case $F$ equals $x_{1} x_{2}$ and attains its maximum value $\\frac{1}{4}$ when $x_{1}=x_{2}=\\frac{1}{2}, x_{3}=\\ldots, x_{n}=0$.","problem_type":null,"tier":0} +{"year":"1991","problem_phase":"shortlisted","problem":"28. (NET 1) ${ }^{\\mathrm{IMO}}$ Given a real number $a>1$, construct an infinite and bounded sequence $x_{0}, x_{1}, x_{2}, \\ldots$ such that for all natural numbers $i$ and $j, i \\neq j$, the following inequality holds: $$ \\left|x_{i}-x_{j}\\right||i-j|^{a} \\geq 1 $$","solution":"28. Let $x_{n}=c(n \\sqrt{2}-[n \\sqrt{2}])$ for some constant $c>0$. For $i>j$, putting $p=[i \\sqrt{2}]-[j \\sqrt{2}]$, we have $\\left|x_{i}-x_{j}\\right|=c|(i-j) \\sqrt{2}-p|=\\frac{\\left|2(i-j)^{2}-p^{2}\\right| c}{(i-j) \\sqrt{2}+p} \\geq \\frac{c}{(i-j) \\sqrt{2}+p} \\geq \\frac{c}{4(i-j)}$, because $p<(i-j) \\sqrt{2}+1$. Taking $c=4$, we obtain that for any $i>j$, $(i-j)\\left|x_{i}-x_{j}\\right| \\geq 1$. Of course, this implies $(i-j)^{a}\\left|x_{i}-x_{j}\\right| \\geq 1$ for any $a>1$ \u3002 Remark. The constant 4 can be replaced with $3 \/ 2+\\sqrt{2}$. Second solution. Another example of a sequence $\\left\\{x_{n}\\right\\}$ is constructed in the following way: $x_{1}=0, x_{2}=1, x_{3}=2$ and $x_{3^{k} i+m}=x_{m}+\\frac{i}{3^{k}}$ for $i=1,2$ and $1 \\leq m \\leq 3^{k}$. It is easily shown that $|i-j| \\cdot\\left|x_{i}-x_{j}\\right| \\geq 1 \/ 3$ for any $i \\neq j$. Third solution. If $n=b_{0}+2 b_{1}+\\cdots+2^{k} b_{k}, b_{i} \\in\\{0,1\\}$, then one can set $x_{n}$ to be $=b_{0}+2^{-a} b_{1}+\\cdots+2^{-k a} b_{k}$. In this case it holds that $|i-j|^{a}\\left|x_{i}-x_{j}\\right| \\geq$ $\\frac{2^{a}-2}{2^{a}-1}$.","problem_type":null,"tier":0} +{"year":"1991","problem_phase":"shortlisted","problem":"29. (FIN 2) We call a set $S$ on the real line $\\mathbb{R}$ superinvariant if for any stretching $A$ of the set by the transformation taking $x$ to $A(x)=x_{0}+$ $a\\left(x-x_{0}\\right)$ there exists a translation $B, B(x)=x+b$, such that the images of $S$ under $A$ and $B$ agree; i.e., for any $x \\in S$ there is a $y \\in S$ such that $A(x)=B(y)$ and for any $t \\in S$ there is a $u \\in S$ such that $B(t)=A(u)$. Determine all superinvariant sets. Remark. It is assumed that $a>0$.","solution":"29. One easily observes that the following sets are super-invariant: one-point set, its complement, closed and open half-lines or their complements, and the whole real line. To show that these are the only possibilities, we first observe that $S$ is super-invariant if and only if for each $a>0$ there is a $b$ such that $x \\in S \\Leftrightarrow a x+b \\in S$. (i) Suppose that for some $a$ there are two such $b$ 's: $b_{1}$ and $b_{2}$. Then $x \\in$ $S \\Leftrightarrow a x+b_{1} \\in S$ and $x \\in S \\Leftrightarrow a x+b_{2} \\in S$, which implies that $S$ is periodic: $y \\in S \\Leftrightarrow y+\\frac{b_{1}-b_{2}}{a} \\in S$. Since $S$ is identical to a translate of any stretching of $S$, all positive numbers are periods of $S$. Therefore $S \\equiv \\mathbb{R}$. (ii) Assume that, for each $a, b=f(a)$ is unique. Then for any $a_{1}$ and $a_{2}$, $$ \\begin{aligned} x \\in S & \\Leftrightarrow a_{1} x+f\\left(a_{1}\\right) \\in S \\Leftrightarrow a_{1} a_{2} x+a_{2} f\\left(a_{1}\\right)+f\\left(a_{2}\\right) \\in S \\\\ & \\Leftrightarrow a_{2} x+f\\left(a_{2}\\right) \\in S \\Leftrightarrow a_{1} a_{2} x+a_{1} f\\left(a_{2}\\right)+f\\left(a_{1}\\right) \\in S . \\end{aligned} $$ As above it follows that $a_{1} f\\left(a_{2}\\right)+f\\left(a_{1}\\right)=a_{2} f\\left(a_{1}\\right)+f\\left(a_{2}\\right)$, or equivalently $f\\left(a_{1}\\right)\\left(a_{2}-1\\right)=f\\left(a_{2}\\right)\\left(a_{1}-1\\right)$. Hence (for some $\\left.c\\right), f(a)=c(a-1)$ for all $a$. Now $x \\in S \\Leftrightarrow a x+c(a-1) \\in S$ actually means that $y-c \\in S \\Leftrightarrow a y-c \\in S$ for all $a$. Then it is easy to conclude that $\\{y-c \\mid y \\in S\\}$ is either a half-line or the whole line, and so is $S$.","problem_type":null,"tier":0} +{"year":"1991","problem_phase":"shortlisted","problem":"3. (PRK 1) Let $S$ be any point on the circumscribed circle of $\\triangle P Q R$. Then the feet of the perpendiculars from $S$ to the three sides of the triangle lie on the same straight line. Denote this line by $l(S, P Q R)$. Suppose that the hexagon $A B C D E F$ is inscribed in a circle. Show that the four lines $l(A, B D F), l(B, A C E), l(D, A B F)$, and $l(E, A B C)$ intersect at one point if and only if $C D E F$ is a rectangle.","solution":"3. Consider the problem with the unit circle on the complex plane. For convenience, we use the same letter for a point in the plane and its corresponding complex number. Lemma 1. Line $l(S, P Q R)$ contains the point $Z=\\frac{P+Q+R+S}{2}$. Proof. Suppose $P^{\\prime}, Q^{\\prime}, R^{\\prime}$ are the feet of perpendiculars from $S$ to $Q R$, $R P, P Q$ respectively. It suffices to show that $P^{\\prime}, Q^{\\prime}, R^{\\prime}, Z$ are on the same line. Let us first represent $P^{\\prime}$ by $Q, R, S$. Since $P^{\\prime} \\in Q R$, we have $\\frac{P^{\\prime}-Q}{R-Q}=\\overline{\\left(\\frac{P^{\\prime}-Q}{R-Q}\\right)}$, that is, $$ \\left(P^{\\prime}-Q\\right)(\\bar{R}-\\bar{Q})=\\left(\\overline{P^{\\prime}}-\\bar{Q}\\right)(R-Q) $$ On the other hand, since $S P^{\\prime} \\perp Q R$, the ratio $\\frac{P^{\\prime}-S}{R-Q}$ is purely imaginary. Thus $$ \\left(P^{\\prime}-S\\right)(\\bar{R}-\\bar{Q})=-\\left(\\overline{P^{\\prime}}-\\bar{S}\\right)(R-Q) $$ Eliminating $\\overline{P^{\\prime}}$ from (1) and (2) and using the fact that $\\bar{X}=X^{-1}$ for $X$ on the unit circle, we obtain $P^{\\prime}=(Q+R+S-Q R \/ S) \/ 2$ and analogously $Q^{\\prime}=(P+R+S-P R \/ S) \/ 2$ and $R^{\\prime}=(P+Q+S-$ $P Q \/ S) \/ 2$. Hence $Z-P^{\\prime}=(P+Q R \/ S) \/ 2, Z-Q^{\\prime}=(Q+P R \/ S) \/ 2$ and $Z-R^{\\prime}=(R+P Q \/ S) \/ 2$. Setting $P=p^{2}, Q=q^{2}, R=r^{2}$, $S=s^{2}$ we obtain $Z-P^{\\prime}=\\frac{p q r}{2 s}\\left(\\frac{p s}{q r}+\\frac{q r}{p s}\\right), Z-Q^{\\prime}=\\frac{p q r}{2 s}\\left(\\frac{q s}{p r}+\\frac{p r}{q s}\\right)$ and $Z-P^{\\prime}=\\frac{p q r}{2 s}\\left(\\frac{r s}{p q}+\\frac{p q}{r s}\\right)$. Since $x+x^{-1}=2 \\operatorname{Re} x$ is real for all $x$ on the unit circle, it follows that the ratio of every pair of these differences is real, which means that $Z, P^{\\prime}, Q^{\\prime}, R^{\\prime}$ belong to the same line. Lemma 2. If $P, Q, R, S$ are four different points on a circle, then the lines $l(P, Q R S), l(Q, R S P), l(R, S P Q), l(S, P Q R)$ intersect at one point. Proof. By Lemma 1, they all pass through $\\frac{P+Q+R+S}{2}$. Now we can find the needed conditions for $A, B, \\ldots, F$. In fact, the lines $l(A, B D F), l(D, A B F)$ meet at $Z_{1}=\\frac{A+B+D+F}{2}$, and $l(B, A C E)$, $l(E, A B C)$ meet at $Z_{2}=\\frac{A+B+C+E}{2}$. Hence, $Z_{1} \\equiv Z_{2}$ if and only if $D-C=E-F \\Leftrightarrow C D E F$ is a rectangle. Remark. The line $l(S, P Q R)$ is widely known as Simson line; the proof that the feet of perpendiculars are collinear is straightforward. The key claim, Lemma 1, is a known property of Simson lines, and can be shown elementarily: * $l(S, P Q R)$ passes through the midpoint $X$ of $H S$, where $H$ is the orthocenter of $P Q R$.","problem_type":null,"tier":0} +{"year":"1991","problem_phase":"shortlisted","problem":"30. (BUL 3) Two students $A$ and $B$ are playing the following game: Each of them writes down on a sheet of paper a positive integer and gives the sheet to the referee. The referee writes down on a blackboard two integers, one of which is the sum of the integers written by the players. After that, the referee asks student $A$ : \"Can you tell the integer written by the other student?\" If $A$ answers \"no,\" the referee puts the same question to student $B$. If $B$ answers \"no,\" the referee puts the question back to $A$, and so on. Assume that both students are intelligent and truthful. Prove that after a finite number of questions, one of the students will answer \"yes.\"","solution":"30. Let $a$ and $b$ be the integers written by $A$ and $B$ respectively, and let $xx-r_{n-1}, B$ would know that $a+b>x$ and hence $a+b=y$, while if $b0$, there exists an $m$ for which $s_{m}-r_{m}<0$, a contradiction.","problem_type":null,"tier":0} +{"year":"1991","problem_phase":"shortlisted","problem":"4. (FRA 2) ${ }^{\\mathrm{IMO5}}$ Let $A B C$ be a triangle and $M$ an interior point in $A B C$. Show that at least one of the angles $\\measuredangle M A B, \\measuredangle M B C$, and $\\measuredangle M C A$ is less than or equal to $30^{\\circ}$.","solution":"4. Assume the contrary, that $\\angle M A B, \\angle M B C, \\angle M C A$ are all greater than $30^{\\circ}$. By the sine Ceva theorem, it holds that $$ \\begin{aligned} & \\sin \\angle M A C \\sin \\angle M B A \\sin \\angle M C B \\\\ = & \\sin \\angle M A B \\sin \\angle M B C \\sin \\angle M C A>\\sin ^{3} 30^{\\circ}=\\frac{1}{8} . \\end{aligned} $$ On the other hand, since $\\angle M A C+\\angle M B A+\\angle M C B<180^{\\circ}-3 \\cdot 30^{\\circ}=90^{\\circ}$, Jensen's inequality applied on the concave function $\\ln \\sin x(x \\in[0, \\pi])$ gives us $\\sin \\angle M A C \\sin \\angle M B A \\sin \\angle M C B<\\sin ^{3} 30^{\\circ}$, contradicting (*). Second solution. Denote the intersections of $P A, P B, P C$ with $B C, C A$, $A B$ by $A_{1}, B_{1}, C_{1}$, respectively. Suppose that each of the angles $\\angle P A B$, $\\angle P B C, \\angle P C A$ is greater than $30^{\\circ}$ and denote $P A=2 x, P B=2 y, P C=$ $2 z$. Then $P C_{1}>x, P A_{1}>y, P B_{1}>z$. On the other hand, we know that $$ \\frac{P C_{1}}{P C+P C_{1}}+\\frac{P A_{1}}{P A+P A_{1}}+\\frac{P B_{1}}{P B+P B_{1}}=\\frac{S_{A B P}}{S_{A B C}}+\\frac{S_{P B C}}{S_{A B C}}+\\frac{S_{A P C}}{S_{A B C}}=1 . $$ Since the function $\\frac{t}{p+t}$ is increasing, we obtain $\\frac{x}{2 z+x}+\\frac{y}{2 x+y}+\\frac{z}{2 y+z}<1$. But on the contrary, Cauchy-Schwartz inequality (or alternatively Jensen's inequality) yields $$ \\frac{x}{2 z+x}+\\frac{y}{2 x+y}+\\frac{z}{2 y+z} \\geq \\frac{(x+y+z)^{2}}{x(2 z+x)+y(2 x+y)+z(2 y+z)}=1 . $$","problem_type":null,"tier":0} +{"year":"1991","problem_phase":"shortlisted","problem":"5. (SPA 4) In the triangle $A B C$, with $\\measuredangle A=60^{\\circ}$, a parallel $I F$ to $A C$ is drawn through the incenter $I$ of the triangle, where $F$ lies on the side $A B$. The point $P$ on the side $B C$ is such that $3 B P=B C$. Show that $\\measuredangle B F P=\\measuredangle B \/ 2$.","solution":"5. Let $P_{1}$ be the point on the side $B C$ such that $\\angle B F P_{1}=\\beta \/ 2$. Then $\\angle B P_{1} F=180^{\\circ}-3 \\beta \/ 2$, and the sine law gives us $\\frac{B F}{B P_{1}}=\\frac{\\sin (3 \\beta \/ 2)}{\\sin (\\beta \/ 2)}=$ $3-4 \\sin ^{2}(\\beta \/ 2)=1+2 \\cos \\beta$. Now we calculate $\\frac{B F}{B P}$. We have $\\angle B I F=120^{\\circ}-\\beta \/ 2, \\angle B F I=60^{\\circ}$ and $\\angle B I C=120^{\\circ}, \\angle B C I=\\gamma \/ 2=60^{\\circ}-\\beta \/ 2$. By the sine law, $$ B F=B I \\frac{\\sin \\left(120^{\\circ}-\\beta \/ 2\\right)}{\\sin 60^{\\circ}}, \\quad B P=\\frac{1}{3} B C=B I \\frac{\\sin 120^{\\circ}}{3 \\sin \\left(60^{\\circ}-\\beta \/ 2\\right)} . $$ It follows that $\\frac{B F}{B P}=\\frac{3 \\sin \\left(60^{\\circ}-\\beta \/ 2\\right) \\sin \\left(60^{\\circ}+\\beta \/ 2\\right)}{\\sin ^{2} 60^{\\circ}}=4 \\sin \\left(60^{\\circ}-\\beta \/ 2\\right) \\sin \\left(60^{\\circ}+\\right.$ $\\beta \/ 2)=2\\left(\\cos \\beta-\\cos 120^{\\circ}\\right)=2 \\cos \\beta+1=\\frac{B F}{B P_{1}}$. Therefore $P \\equiv P_{1}$.","problem_type":null,"tier":0} +{"year":"1991","problem_phase":"shortlisted","problem":"6. (USS 4) ${ }^{\\mathrm{IMO}}$ Prove for each triangle $A B C$ the inequality $$ \\frac{1}{4}<\\frac{I A \\cdot I B \\cdot I C}{l_{A} l_{B} l_{C}} \\leq \\frac{8}{27} $$ where $I$ is the incenter and $l_{A}, l_{B}, l_{C}$ are the lengths of the angle bisectors of $A B C$.","solution":"6. Let $a, b, c$ be sides of the triangle. Let $A_{1}$ be the intersection of line $A I$ with $B C$. By the known fact, $B A_{1}: A_{1} C=c: b$ and $A I: I A_{1}=A B: B A_{1}$, hence $B A_{1}=\\frac{a c}{b+c}$ and $\\frac{A I}{I A_{1}}=\\frac{A B}{B A_{1}}=\\frac{b+c}{a}$. Consequently $\\frac{A I}{l_{A}}=\\frac{b+c}{a+b+c}$. Put $a=n+p, b=p+m, c=m+n$ : it is obvious that $m, n, p$ are positive. Our inequality becomes $$ 2<\\frac{(2 m+n+p)(m+2 n+p)(m+n+2 p)}{(m+n+p)^{3}} \\leq \\frac{64}{27} $$ The right side inequality immediately follows from the inequality between arithmetic and geometric means applied on $2 m+n+p, m+2 n+p$ and $m+n+2 p$. For the left side inequality, denote by $T=m+n+p$. Then we can write $(2 m+n+p)(m+2 n+p)(m+n+2 p)=(T+m)(T+n)(T+p)$ and $(T+m)(T+n)(T+p)=T^{3}+(m+n+p) T^{2}+(m n+n p+p n) T+m n p>2 T^{3}$. Remark. The inequalities cannot be improved. In fact, $\\frac{A I \\cdot B I \\cdot C I}{l_{A} l_{B} l_{C}}$ is equal to $8 \/ 27$ for $a=b=c$, while it can be arbitrarily close to $1 \/ 4$ if $a=b$ and $c$ is sufficiently small.","problem_type":null,"tier":0} +{"year":"1991","problem_phase":"shortlisted","problem":"7. (CHN 2) Let $O$ be the center of the circumsphere of a tetrahedron $A B C D$. Let $L, M, N$ be the midpoints of $B C, C A, A B$ respectively, and assume that $A B+B C=A D+C D, B C+C A=B D+A D$, and $C A+A B=$ $C D+B D$. Prove that $\\angle L O M=\\angle M O N=\\angle N O L$.","solution":"7. The given equations imply $A B=C D, A C=B D, A D=B C$. Let $L_{1}$, $M_{1}, N_{1}$ be the midpoints of $A D, B D, C D$ respectively. Then the above equalities yield $$ \\begin{gathered} L_{1} M_{1}=A B \/ 2=L M \\\\ L_{1} M_{1}\\|A B\\| L M \\\\ L_{1} M=C D \/ 2=L M_{1} \\\\ L_{1} M\\|C D\\| L M_{1} \\end{gathered} $$ Thus $L, M, L_{1}, M_{1}$ are coplanar and $L M L_{1} M_{1}$ is a rhombus as well as ![](https:\/\/cdn.mathpix.com\/cropped\/2024_11_18_8e985d6b9c83aa3e9d0eg-556.jpg?height=317&width=410&top_left_y=248&top_left_x=888) $M N M_{1} N_{1}$ and $L N L_{1} N_{1}$. Then the segments $L L_{1}, M M_{1}, N N_{1}$ have the common midpoint $Q$ and $Q L \\perp Q M$, $Q L \\perp Q N, Q M \\perp Q N$. We also infer that the line $N N_{1}$ is perpendicular to the plane $L M L_{1} M_{1}$ and hence to the line $A B$. Thus $Q A=Q B$, and similarly, $Q B=Q C=Q D$, hence $Q$ is just the center $O$, and $\\angle L O M=$ $\\angle M O N=\\angle N O L=90^{\\circ}$.","problem_type":null,"tier":0} +{"year":"1991","problem_phase":"shortlisted","problem":"8. (NET 1) Let $S$ be a set of $n$ points in the plane. No three points of $S$ are collinear. Prove that there exists a set $P$ containing $2 n-5$ points satisfying the following condition: In the interior of every triangle whose three vertices are elements of $S$ lies a point that is an element of $P$.","solution":"8. Let $P_{1}\\left(x_{1}, y_{1}\\right), P_{2}\\left(x_{2}, y_{2}\\right), \\ldots, P_{n}\\left(x_{n}, y_{n}\\right)$ be the $n$ points of $S$ in the coordinate plane. We may assume $x_{1}y$ is a positive integer). This will imply the desired result, since starting from the pair $(1,1)$ we can obtain arbitrarily many solutions. First, we show that $\\operatorname{gcd}\\left(x_{1}, y\\right)=1$. Suppose to the contrary that $\\operatorname{gcd}\\left(x_{1}, y\\right)$ $=d>1$. Then $d\\left|x_{1}\\right| y^{2}+m \\Rightarrow d \\mid m$, which implies $d|y| x^{2}+m \\Rightarrow d \\mid x$. But this last is impossible, since $\\operatorname{gcd}(x, y)=1$. Thus it remains to show that $x_{1} \\mid y^{2}+m$ and $y \\mid x_{1}^{2}+m$. The former relation is obvious. Since $\\operatorname{gcd}(x, y)=1$, the latter is equivalent to $y \\mid\\left(x x_{1}\\right)^{2}+m x^{2}=y^{4}+2 m y^{2}+$ $m^{2}+m x^{2}$, which is true because $y \\mid m\\left(m+x^{2}\\right)$ by the assumption. Hence $\\left(y, x_{1}\\right)$ indeed satisfies all the required conditions. Remark. The original problem asked to prove the existence of a pair $(x, y)$ of positive integers satisfying the given conditions such that $x+y \\leq m+1$. The problem in this formulation is trivial, since the pair $x=y=1$ satisfies the conditions. Moreover, this is sometimes the only solution with $x+y \\leq m+1$. For example, for $m=3$ the least nontrivial solution is $\\left(x_{0}, y_{0}\\right)=(1,4)$.","problem_type":null,"tier":0} +{"year":"1992","problem_phase":"shortlisted","problem":"10. (ITA 1) ${ }^{\\mathrm{IMO5}}$ Let $V$ be a finite subset of Euclidean space consisting of points $(x, y, z)$ with integer coordinates. Let $S_{1}, S_{2}, S_{3}$ be the projections of $V$ onto the $y z, x z, x y$ planes, respectively. Prove that $$ |V|^{2} \\leq\\left|S_{1}\\right|\\left|S_{2}\\right|\\left|S_{3}\\right| $$ ( $|X|$ denotes the number of elements of $X$ ).","solution":"10. Let us set $S(x)=\\{(y, z) \\mid(x, y, z) \\in V\\}, S_{y}(x)=\\left\\{z \\mid(x, z) \\in S_{y}\\right\\}$ and $S_{z}(x)=\\left\\{y \\mid(x, y) \\in S_{z}\\right\\}$. Clearly $S(x) \\subset S_{x}$ and $S(x) \\subset S_{y}(x) \\times S_{z}(x)$. It follows that $$ \\begin{aligned} |V| & =\\sum_{x}|S(x)| \\leq \\sum_{x} \\sqrt{\\left|S_{x}\\right|\\left|S_{y}(x)\\right|\\left|S_{z}(x)\\right|} \\\\ & =\\sqrt{\\left|S_{x}\\right|} \\sum_{x} \\sqrt{\\left|S_{y}(x)\\right|\\left|S_{z}(x)\\right|} \\end{aligned} $$ Using the Cauchy-Schwarz inequality we also get $$ \\sum_{x} \\sqrt{\\left|S_{y}(x)\\right|\\left|S_{z}(x)\\right|} \\leq \\sqrt{\\sum_{x}\\left|S_{y}(x)\\right|} \\sqrt{\\sum_{x}\\left|S_{z}(x)\\right|}=\\sqrt{\\left|S_{y}\\right|\\left|S_{z}\\right|} $$ Now (1) and (2) together yield $|V| \\leq \\sqrt{\\left|S_{x}\\right|\\left|S_{y}\\right|\\left|S_{z}\\right|}$.","problem_type":null,"tier":0} +{"year":"1992","problem_phase":"shortlisted","problem":"11. (JAP 2) In a triangle $A B C$, let $D$ and $E$ be the intersections of the bisectors of $\\angle A B C$ and $\\angle A C B$ with the sides $A C, A B$, respectively. Determine the angles $\\angle A, \\angle B, \\angle C$ if $$ \\measuredangle B D E=24^{\\circ}, \\quad \\measuredangle C E D=18^{\\circ} . $$","solution":"11. Let $I$ be the incenter of $\\triangle A B C$. Since $90^{\\circ}+\\alpha \/ 2=\\angle B I C=\\angle D I E=$ $138^{\\circ}$, we obtain that $\\angle A=96^{\\circ}$. ![](https:\/\/cdn.mathpix.com\/cropped\/2024_11_18_8e985d6b9c83aa3e9d0eg-571.jpg?height=295&width=1043&top_left_y=1086&top_left_x=260) Let $D^{\\prime}$ and $E^{\\prime}$ be the points symmetric to $D$ and $E$ with respect to $C E$ and $B D$ respectively, and let $S$ be the intersection point of $E D^{\\prime}$ and $B D$. Then $\\angle B D E^{\\prime}=24^{\\circ}$ and $\\angle D^{\\prime} D E^{\\prime}=\\angle D^{\\prime} D E-\\angle E^{\\prime} D E=24^{\\circ}$, which means that $D E^{\\prime}$ bisects the angle $S D D^{\\prime}$. Moreover, $\\angle E^{\\prime} S B=\\angle E S B=$ $\\angle E D S+\\angle D E S=60^{\\circ}$ and hence $S E^{\\prime}$ bisects the angle $D^{\\prime} S B$. It follows that $E^{\\prime}$ is the excenter of $\\triangle D^{\\prime} D S$ and consequently $\\angle D^{\\prime} D C=\\angle D D^{\\prime} C=$ $\\angle S D^{\\prime} E^{\\prime}=\\left(180^{\\circ}-72^{\\circ}\\right) \/ 2=54^{\\circ}$. Finally, $\\angle C=180^{\\circ}-2 \\cdot 54^{\\circ}=72^{\\circ}$ and $\\angle B=12^{\\circ}$.","problem_type":null,"tier":0} +{"year":"1992","problem_phase":"shortlisted","problem":"12. (NET 1) Let $f, g$, and $a$ be polynomials with real coefficients, $f$ and $g$ in one variable and $a$ in two variables. Suppose $$ f(x)-f(y)=a(x, y)(g(x)-g(y)) \\quad \\text { for all } x, y \\in \\mathbb{R} $$ Prove that there exists a polynomial $h$ with $f(x)=h(g(x))$ for all $x \\in \\mathbb{R}$.","solution":"12. Let us set $\\operatorname{deg} f=n$ and $\\operatorname{deg} g=m$. We shall prove the result by induction on $n$. If $n1$. It is easy to see that $\\alpha\\left(n_{m}\\right)=2^{m}-m$. On the other hand, squaring and simplifying yields $n_{m}^{2}=1+\\sum_{i1$ be a constant to be chosen later, and let $N_{i}=2^{m_{i}} n_{i}-1$ where $m_{i}>\\alpha\\left(n_{i}\\right)$ is such that $m_{i} \/ \\alpha\\left(n_{i}\\right) \\rightarrow \\theta$ as $i \\rightarrow \\infty$. Then $\\alpha\\left(N_{i}\\right)=\\alpha\\left(n_{i}\\right)+m_{i}-1$, whereas $N_{i}^{2}=2^{2 m_{i}} n_{i}^{2}-2^{m_{i}+1} n_{i}+1$ and $\\alpha\\left(N_{i}^{2}\\right)=\\alpha\\left(n_{i}^{2}\\right)-\\alpha\\left(n_{i}\\right)+m_{i}$. It follows that $$ \\lim _{i \\rightarrow \\infty} \\frac{\\alpha\\left(N_{i}^{2}\\right)}{\\alpha\\left(N_{i}\\right)}=\\lim _{i \\rightarrow \\infty} \\frac{\\alpha\\left(n_{i}^{2}\\right)+(\\theta-1) \\alpha\\left(n_{i}\\right)}{(1+\\theta) \\alpha\\left(n_{i}\\right)}=\\frac{\\theta-1}{\\theta+1} $$ which is equal to $\\gamma \\in[0,1]$ for $\\theta=\\frac{1+\\gamma}{1-\\gamma}$ (for $\\gamma=1$ we set $m_{i} \/ \\alpha\\left(n_{i}\\right) \\rightarrow$ $\\infty)$. (3) Let be given a sequence $\\left(n_{i}\\right)_{i=1}^{\\infty}$ with $\\alpha\\left(n_{i}^{2}\\right) \/ \\alpha\\left(n_{i}\\right) \\rightarrow \\gamma$. Taking $m_{i}>$ $\\alpha\\left(n_{i}\\right)$ and $N_{i}=2^{m_{i}} n_{i}+1$ we easily find that $\\alpha\\left(N_{i}\\right)=\\alpha\\left(n_{i}\\right)+1$ and $\\alpha\\left(N_{i}^{2}\\right)=\\alpha\\left(n_{i}^{2}\\right)+\\alpha\\left(n_{i}\\right)+1$. Hence $\\alpha\\left(N_{i}^{2}\\right) \/ \\alpha\\left(N_{i}\\right)=\\gamma+1$. Continuing this procedure we can construct a sequence $t_{i}$ such that $\\alpha\\left(t_{i}^{2}\\right) \/ \\alpha\\left(t_{i}\\right)=$ $\\gamma+k$ for an arbitrary $k \\in \\mathbb{N}$.","problem_type":null,"tier":0} +{"year":"1992","problem_phase":"shortlisted","problem":"18. (USA 2) Let $[x]$ denote the greatest integer less than or equal to $x$. Pick any $x_{1}$ in $[0,1)$ and define the sequence $x_{1}, x_{2}, x_{3}, \\ldots$ by $x_{n+1}=0$ if $x_{n}=0$ and $x_{n+1}=1 \/ x_{n}-\\left[1 \/ x_{n}\\right]$ otherwise. Prove that $$ x_{1}+x_{2}+\\cdots+x_{n}<\\frac{F_{1}}{F_{2}}+\\frac{F_{2}}{F_{3}}+\\cdots+\\frac{F_{n}}{F_{n+1}} $$ where $F_{1}=F_{2}=1$ and $F_{n+2}=F_{n+1}+F_{n}$ for $n \\geq 1$.","solution":"18. Let us define inductively $f^{1}(x)=f(x)=\\frac{1}{x+1}$ and $f^{n}(x)=f\\left(f^{n-1}(x)\\right)$, and let $g_{n}(x)=x+f(x)+f^{2}(x)+\\cdots+f^{n}(x)$. We shall prove first the following statement. Lemma. The function $g_{n}(x)$ is strictly increasing on $[0,1]$, and $g_{n-1}(1)=$ $F_{1} \/ F_{2}+F_{2} \/ F_{3}+\\cdots+F_{n} \/ F_{n+1}$. Proof. Since $f(x)-f(y)=\\frac{y-x}{(1+x)(1+y)}$ is smaller in absolute value than $x-y$, it follows that $x>y$ implies $f^{2 k}(x)>f^{2 k}(y)$ and $f^{2 k+1}(x)<$ $f^{2 k+1}(y)$, and moreover that for every integer $k \\geq 0$, $$ \\left[f^{2 k}(x)-f^{2 k}(y)\\right]+\\left[f^{2 k+1}(x)-f^{2 k+1}(y)\\right]>0 $$ Hence if $x>y$, we have $g_{n}(x)-g_{n}(y)=(x-y)+[f(x)-f(y)]+\\cdots+$ $\\left[f^{n}(x)-f^{n}(y)\\right]>0$, which yields the first part of the lemma. The second part follows by simple induction, since $f^{k}(1)=F_{k+1} \/ F_{k+2}$. If some $x_{i}=0$ and consequently $x_{j}=0$ for all $j \\geq i$, then the problem reduces to the problem with $i-1$ instead of $n$. Thus we may assume that all $x_{1}, \\ldots, x_{n}$ are different from 0 . If we write $a_{i}=\\left[1 \/ x_{i}\\right]$, then $x_{i}=\\frac{1}{a_{i}+x_{i+1}}$. Thus we can regard $x_{i}$ as functions of $x_{n}$ depending on $a_{1}, \\ldots, a_{n-1}$. Suppose that $x_{n}, a_{n-1}, \\ldots, a_{3}, a_{2}$ are fixed. Then $x_{2}, x_{3}, \\ldots, x_{n}$ are all fixed, and $x_{1}=\\frac{1}{a_{1}+x_{2}}$ is maximal when $a_{1}=1$. Hence the sum $S=$ $x_{1}+x_{2}+\\cdots+x_{n}$ is maximized for $a_{1}=1$. We shall show by induction on $i$ that $S$ is maximized for $a_{1}=a_{2}=\\cdots=$ $a_{i}=1$. In fact, assuming that the statement holds for $i-1$ and thus $a_{1}=$ $\\cdots=a_{i-1}=1$, having $x_{n}, a_{n-1}, \\ldots, a_{i+1}$ fixed we have that $x_{n}, \\ldots, x_{i+1}$ are also fixed, and that $x_{i-1}=f\\left(x_{i}\\right), \\ldots, x_{1}=f^{i-1}\\left(x_{i}\\right)$. Hence by the lemma, $S=g_{i-1}\\left(x_{i}\\right)+x_{i+1}+\\cdots+x_{n}$ is maximal when $x_{i}=\\frac{1}{a_{i}+x_{i+1}}$ is maximal, that is, for $a_{i}=1$. Thus the induction is complete. It follows that $x_{1}+\\cdots+x_{n}$ is maximal when $a_{1}=\\cdots=a_{n-1}=1$, so that $x_{1}+\\cdots+x_{n}=g_{n-1}\\left(x_{1}\\right)$. By the lemma, the latter does not exceed $g_{n-1}(1)$. This completes the proof. Remark. The upper bound is the best possible, because it is approached by taking $x_{n}$ close to 1 and inductively (in reverse) defining $x_{i-1}=\\frac{1}{1+x_{i}}=$ $\\frac{1}{a_{i}+x_{i}}$.","problem_type":null,"tier":0} +{"year":"1992","problem_phase":"shortlisted","problem":"19. (IRE 1) Let $f(x)=x^{8}+4 x^{6}+2 x^{4}+28 x^{2}+1$. Let $p>3$ be a prime and suppose there exists an integer $z$ such that $p$ divides $f(z)$. Prove that there exist integers $z_{1}, z_{2}, \\ldots, z_{8}$ such that if $$ g(x)=\\left(x-z_{1}\\right)\\left(x-z_{2}\\right) \\cdots\\left(x-z_{8}\\right) $$ then all coefficients of $f(x)-g(x)$ are divisible by $p$.","solution":"19. Observe that $f(x)=\\left(x^{4}+2 x^{2}+3\\right)^{2}-8\\left(x^{2}-1\\right)^{2}=\\left[x^{4}+2(1-\\sqrt{2}) x^{2}+\\right.$ $3+2 \\sqrt{2}]\\left[x^{4}+2(1+\\sqrt{2}) x^{2}+3-2 \\sqrt{2}\\right]$. Now it is easy to find that the roots of $f$ are $$ x_{1,2,3,4}= \\pm i(i \\sqrt[4]{2} \\pm 1) \\quad \\text { and } \\quad x_{5,6,7,8}= \\pm i(\\sqrt[4]{2} \\pm 1) $$ In other words, $x_{k}=\\alpha_{i}+\\beta_{j}$, where $\\alpha_{i}^{2}=-1$ and $\\beta_{j}^{4}=2$. We claim that any root of $f$ can be obtained from any other using rational functions. In fact, we have $$ \\begin{aligned} & x^{3}=-\\alpha_{i}-3 \\beta_{j}+3 \\alpha_{i} \\beta_{j}^{2}+\\beta_{j}^{3} \\\\ & x^{5}=11 \\alpha_{i}+7 \\beta_{j}-10 \\alpha_{i} \\beta_{j}^{2}-10 \\beta_{j}^{3} \\\\ & x^{7}=-71 \\alpha_{i}-49 \\beta_{j}+35 \\alpha_{i} \\beta_{j}^{2}+37 \\beta_{j}^{3} \\end{aligned} $$ from which we easily obtain that $\\alpha_{i}=24^{-1}\\left(127 x+5 x^{3}+19 x^{5}+5 x^{7}\\right), \\quad \\beta_{j}=24^{-1}\\left(151 x+5 x^{3}+19 x^{5}+5 x^{7}\\right)$. Since all other values of $\\alpha$ and $\\beta$ can be obtained as rational functions of $\\alpha_{i}$ and $\\beta_{j}$, it follows that all the roots $x_{l}$ are rational functions of a particular root $x_{k}$. We now note that if $x_{1}$ is an integer such that $f\\left(x_{1}\\right)$ is divisible by $p$, then $p>3$ and $x_{1} \\in \\mathbb{Z}_{p}$ is a root of the polynomial $f$. By the previous consideration, all remaining roots $x_{2}, \\ldots, x_{8}$ of $f$ over the field $\\mathbb{Z}_{p}$ are rational functions of $x_{1}$, since 24 is invertible in $Z_{p}$. Then $f(x)$ factors as $$ f(x)=\\left(x-x_{1}\\right)\\left(x-x_{2}\\right) \\cdots\\left(x-x_{8}\\right), $$ and the result follows.","problem_type":null,"tier":0} +{"year":"1992","problem_phase":"shortlisted","problem":"2. (CHN 1) Let $\\mathbb{R}^{+}$be the set of all nonnegative real numbers. Given two positive real numbers $a$ and $b$, suppose that a mapping $f: \\mathbb{R}^{+} \\rightarrow \\mathbb{R}^{+}$ satisfies the functional equation $$ f(f(x))+a f(x)=b(a+b) x $$ Prove that there exists a unique solution of this equation.","solution":"2. Let us define $x_{n}$ inductively as $x_{n}=f\\left(x_{n-1}\\right)$, where $x_{0} \\geq 0$ is a fixed real number. It follows from the given equation in $f$ that $x_{n+2}=-a x_{n+1}+$ $b(a+b) x_{n}$. The general solution to this equation is of the form $$ x_{n}=\\lambda_{1} b^{n}+\\lambda_{2}(-a-b)^{n}, $$ where $\\lambda_{1}, \\lambda_{2} \\in \\mathbb{R}$ satisfy $x_{0}=\\lambda_{1}+\\lambda_{2}$ and $x_{1}=\\lambda_{1} b-\\lambda_{2}(a+b)$. In order to have $x_{n} \\geq 0$ for all $n$ we must have $\\lambda_{2}=0$. Hence $x_{0}=\\lambda_{1}$ and $f\\left(x_{0}\\right)=x_{1}=\\lambda_{1} b=b x_{0}$. Since $x_{0}$ was arbitrary, we conclude that $f(x)=b x$ is the only possible solution of the functional equation. It is easily verified that this is indeed a solution.","problem_type":null,"tier":0} +{"year":"1992","problem_phase":"shortlisted","problem":"20. (FRA 1) ${ }^{\\mathrm{IMO} 4}$ In the plane, let there be given a circle $C$, a line $l$ tangent to $C$, and a point $M$ on $l$. Find the locus of points $P$ that have the following property: There exist two points $Q$ and $R$ on $l$ such that $M$ is the midpoint of $Q R$ and $C$ is the incircle of $P Q R$.","solution":"20. Denote by $U$ the point of tangency of the circle $C$ and the line $l$. Let $X$ and $U^{\\prime}$ be the points symmetric to $U$ with respect to $S$ and $M$ respectively; these points do not depend on the choice of $P$. Also, let $C^{\\prime}$ be the excircle of $\\triangle P Q R$ corresponding to $P, S^{\\prime}$ the center of $C^{\\prime}$, and $W, W^{\\prime}$ the points of tangency of $C$ and $C^{\\prime}$ with the line $P Q$ respectively. Obviously, $\\triangle W S P \\sim \\triangle W^{\\prime} S^{\\prime} P$. Since $S X \\| S^{\\prime} U^{\\prime}$ and $S X: S^{\\prime} U^{\\prime}=$ $S W: S^{\\prime} W^{\\prime}=S P: S^{\\prime} P$, we deduce that $\\Delta S X P \\sim \\Delta S^{\\prime} U^{\\prime} P$, and consequently that $P$ lies on the line $X U^{\\prime}$. On the other hand, it is easy to show that each point $P$ of the ray $U^{\\prime} X$ over $X$ satisfies the required condition. Thus the desired locus is ![](https:\/\/cdn.mathpix.com\/cropped\/2024_11_18_8e985d6b9c83aa3e9d0eg-575.jpg?height=543&width=539&top_left_y=908&top_left_x=799) the extension of $U^{\\prime} X$ over $X$.","problem_type":null,"tier":0} +{"year":"1992","problem_phase":"shortlisted","problem":"21. (GBR 2) ${ }^{\\mathrm{IMO} 6}$ For each positive integer $n$, denote by $s(n)$ the greatest integer such that for all positive integers $k \\leq s(n), n^{2}$ can be expressed as a sum of squares of $k$ positive integers. (a) Prove that $s(n) \\leq n^{2}-14$ for all $n \\geq 4$. (b) Find a number $n$ such that $s(n)=\\overline{n^{2}}-14$. (c) Prove that there exist infinitely many positive integers $n$ such that $s(n)=n^{2}-14$.","solution":"21. (a) Representing $n^{2}$ as a sum of $n^{2}-13$ squares is equivalent to representing 13 as a sum of numbers of the form $x^{2}-1, x \\in \\mathbb{N}$, such as $0,3,8,15, \\ldots$ But it is easy to check that this is impossible, and hence $s(n) \\leq n^{2}-14$. (b) Let us prove that $s(13)=13^{2}-14=155$. Observe that $$ \\begin{aligned} 13^{2} & =8^{2}+8^{2}+4^{2}+4^{2}+3^{2} \\\\ & =8^{2}+8^{2}+4^{2}+4^{2}+2^{2}+2^{2}+1^{2} \\\\ & =8^{2}+8^{2}+4^{2}+3^{2}+3^{2}+2^{2}+1^{2}+1^{2}+1^{2} \\end{aligned} $$ Given any representation of $n^{2}$ as a sum of $m$ squares one of which is even, we can construct a representation as a sum of $m+3$ squares by dividing the odd square into four equal squares. Thus the first equality enables us to construct representations with $5,8,11, \\ldots, 155$ squares, the second to construct ones with $7,10,13, \\ldots, 154$ squares, and the third with $9,12, \\ldots, 153$ squares. It remains only to represent $13^{2}$ as a sum of $k=2,3,4,6$ squares. This can be done as follows: $$ \\begin{aligned} 13^{2} & =12^{2}+5^{2}=12^{2}+4^{2}+3^{2} \\\\ & =11^{2}+4^{2}+4^{2}+4^{2}=12^{2}+3^{2}+2^{2}+2^{2}+2^{2}+2^{2} \\end{aligned} $$ (c) We shall prove that whenever $s(n)=n^{2}-14$ for some $n \\geq 13$, it also holds that $s(2 n)=(2 n)^{2}-14$. This will imply that $s(n)=n^{2}-14$ for any $n=2^{t} \\cdot 13$. If $n^{2}=x_{1}^{2}+\\cdots+x_{r}^{2}$, then we have $(2 n)^{2}=\\left(2 x_{1}\\right)^{2}+\\cdots+\\left(2 x_{r}\\right)^{2}$. Replacing $\\left(2 x_{i}\\right)^{2}$ with $x_{i}^{2}+x_{i}^{2}+x_{i}^{2}+x_{i}^{2}$ as long as it is possible we can obtain representations of $(2 n)^{2}$ consisting of $r, r+3, \\ldots, 4 r$ squares. This gives representations of $(2 n)^{2}$ into $k$ squares for any $k \\leq 4 n^{2}-62$. Further, we observe that each number $m \\geq 14$ can be written as a sum of $k \\geq m$ numbers of the form $x^{2}-1, x \\in \\mathbb{N}$, which is easy to verify. Therefore if $k \\leq 4 n^{2}-14$, it follows that $4 n^{2}-k$ is a sum of $k$ numbers of the form $x^{2}-1$ (since $k \\geq 4 n^{2}-k \\geq 14$ ), and consequently $4 n^{2}$ is a sum of $k$ squares. Remark. One can find exactly the value of $f(n)$ for each $n$ : $$ f(n)= \\begin{cases}1, & \\text { if } n \\text { has a prime divisor congruent to } 3 \\bmod 4 \\\\ 2, & \\text { if } n \\text { is of the form } 5 \\cdot 2^{k}, k \\text { a positive integer; } \\\\ n^{2}-14, & \\text { otherwise. }\\end{cases} $$","problem_type":null,"tier":0} +{"year":"1992","problem_phase":"shortlisted","problem":"3. (CHN 2) The diagonals of a quadrilateral $A B C D$ are perpendicular: $A C \\perp B D$. Four squares, $A B E F, B C G H, C D I J, D A K L$, are erected externally on its sides. The intersection points of the pairs of straight lines $C L, D F ; D F, A H ; A H, B J ; B J, C L$ are denoted by $P_{1}, Q_{1}, R_{1}, S_{1}$, respectively, and the intersection points of the pairs of straight lines $A I, B K$; $B K, C E ; C E, D G ; D G, A I$ are denoted by $P_{2}, Q_{2}, R_{2}, S_{2}$, respectively. Prove that $P_{1} Q_{1} R_{1} S_{1} \\cong P_{2} Q_{2} R_{2} S_{2}$.","solution":"3. Consider two squares $A B^{\\prime} C D^{\\prime}$ and $A^{\\prime} B C^{\\prime} D$. Since $A C \\perp B D$, these two squares are homothetic, which implies that the lines $A A^{\\prime}, B B^{\\prime}, C C^{\\prime}, D D^{\\prime}$ are concurrent at a certain point $O$. Since the rotation about $A$ by $90^{\\circ}$ takes $\\triangle A B K$ into $\\triangle A F D$, it follows that $B K \\perp D F$. Denote by $T$ the intersection of $B K$ and $D F$. The rotation about some point $X$ by $90^{\\circ}$ maps $B K$ into $D F$ if and only if $T X$ bisects an angle between $B K$ and $D F$. Therefore $\\angle F T A=$ ![](https:\/\/cdn.mathpix.com\/cropped\/2024_11_18_8e985d6b9c83aa3e9d0eg-567.jpg?height=342&width=470&top_left_y=1659&top_left_x=847) $\\angle A T K=45^{\\circ}$. Moreover, the quadrilateral $B A^{\\prime} D T$ is cyclic, which implies that $\\angle B T A^{\\prime}=B D A^{\\prime}=45^{\\circ}$ and consequently that the points $A, T, A^{\\prime}$ are collinear. It follows that the point $O$ lies on a bisector of $\\angle B T D$ and therefore the rotation $\\mathcal{R}$ about $O$ by $90^{\\circ}$ takes $B K$ into $D F$. Analogously, $\\mathcal{R}$ maps the lines $C E, D G, A I$ into $A H, B J, C L$. Hence the quadrilateral $P_{1} Q_{1} R_{1} S_{1}$ is the image of the quadrilateral $P_{2} Q_{2} R_{2} S_{2}$, and the result follows.","problem_type":null,"tier":0} +{"year":"1992","problem_phase":"shortlisted","problem":"4. $(\\mathbf{C H N} 3)^{\\mathrm{IMO}}$ Given nine points in space, no four of which are coplanar, find the minimal natural number $n$ such that for any coloring with red or blue of $n$ edges drawn between these nine points there always exists a triangle having all edges of the same color.","solution":"4. There are 36 possible edges in total. If not more than 3 edges are left undrawn, then we can choose 6 of the given 9 points no two of which are connected by an undrawn edge. These 6 points together with the edges between them form a two-colored complete graph, and thus by a wellknown result there exists at least one monochromatic triangle. It follows that $n \\leq 33$. In order to show that $n=33$, we shall give an example of a graph with 32 edges that does not contain a monochromatic triangle. Let us start with a complete graph $C_{5}$ with 5 vertices. Its edges can be colored in two colors so that there is no monochromatic triangle (Fig. 1). Furthermore, given a graph $\\mathcal{H}$ with $k$ vertices without monochromatic triangles, we can add to it a new vertex, join it to all vertices of $\\mathcal{H}$ except $A$, and color each edge $B X$ in the same way as $A X$. The obtained graph obviously contains no monochromatic triangles. Applying this construction four times to the graph $C_{5}$ we get an example like that of Fig. 2. ![](https:\/\/cdn.mathpix.com\/cropped\/2024_11_18_8e985d6b9c83aa3e9d0eg-568.jpg?height=338&width=396&top_left_y=1069&top_left_x=326) Fig. 1 ![](https:\/\/cdn.mathpix.com\/cropped\/2024_11_18_8e985d6b9c83aa3e9d0eg-568.jpg?height=326&width=396&top_left_y=1079&top_left_x=868) Fig. 2 Second solution. For simplicity, we call the colors red and blue. Let $r(k, l)$ be the least positive integer $r$ such that each complete $r$-graph whose edges are colored in red and blue contains either a complete red $k$-graph or a complete blue $l$-graph. Also, let $t(n, k)$ be the greatest possible number of edges in a graph with $n$ vertices that does not contain a complete $k$-graph. These numbers exist by the theorems of Ramsey and Tur\u00e1n. Let us assume that $r(k, l)0$ for all $x>0$. It also follows that $f(x)<0$ for $x<0$. In other words, $f$ preserves sign. Now setting $x>0$ and $y=-f(x)$ in the given functional equation we obtain $$ f(x-f(x))=f\\left(\\sqrt{x}^{2}+f(-x)\\right)=-x+f(\\sqrt{x})^{2}=-(x-f(x)) . $$ But since $f$ preserves sign, this implies that $f(x)=x$ for $x>0$. Moreover, since $f(-x)=-f(x)$, it follows that $f(x)=x$ for all $x$. It is easily verified that this is indeed a solution.","problem_type":null,"tier":0} +{"year":"1992","problem_phase":"shortlisted","problem":"7. (IND 4) Circles $G, G_{1}, G_{2}$ are three circles related to each other as follows: Circles $G_{1}$ and $G_{2}$ are externally tangent to one another at a point $W$ and both these circles are internally tangent to the circle $G$. Points $A, B, C$ are located on the circle $G$ as follows: Line $B C$ is a direct common tangent to the pair of circles $G_{1}$ and $G_{2}$, and line $W A$ is the transverse common tangent at $W$ to $G_{1}$ and $G_{2}$, with $W$ and $A$ lying on the same side of the line $B C$. Prove that $W$ is the incenter of the triangle $A B C$.","solution":"7. Let $G_{1}, G_{2}$ touch the chord $B C$ at $P, Q$ and touch the circle $G$ at $R, S$ respectively. Let $D$ be the midpoint of the complementary $\\operatorname{arc} B C$ of $G$. The homothety centered at $R$ mapping $G_{1}$ onto $G$ also maps the line $B C$ onto a tangent of $G$ parallel to $B C$. It follows that this line touches $G$ at point $D$, which is therefore the image of $P$ under the homothety. Hence $R, P$, and $D$ are collinear. Since $\\angle D B P=\\angle D C B=\\angle D R B$, it follows that $\\triangle D B P \\sim \\triangle D R B$ and consequently that $D P \\cdot D R=D B^{2}$. Similarly, points $S, Q, D$ are collinear and satisfy $D Q \\cdot D S=D B^{2}=D P \\cdot D R$. Hence $D$ lies on the radical axis of the circles $G_{1}$ and $G_{2}$, i.e., on their common tangent $A W$, which also implies that $A W$ bisects the angle $B A D$. Furthermore, since $D B=D C=D W=\\sqrt{D P \\cdot D R}$, it follows from the lemma of (SL99-14) that $W$ is the incenter of $\\triangle A B C$. Remark. According to the third solution of (SL93-3), both $P W$ and $Q W$ contain the incenter of $\\triangle A B C$, and the result is immediate. The problem can also be solved by inversion centered at $W$.","problem_type":null,"tier":0} +{"year":"1992","problem_phase":"shortlisted","problem":"8. (IND 5) Show that in the plane there exists a convex polygon of 1992 sides satisfying the following conditions: (i) its side lengths are $1,2,3, \\ldots, 1992$ in some order; (ii) the polygon is circumscribable about a circle. Alternative formulation. Does there exist a 1992-gon with side lengths $1,2,3, \\ldots, 1992$ circumscribed about a circle? Answer the same question for a 1990-gon.","solution":"8. For simplicity, we shall write $n$ instead of 1992. Lemma. There exists a tangent $n$-gon $A_{1} A_{2} \\ldots A_{n}$ with sides $A_{1} A_{2}=a_{1}$, $A_{2} A_{3}=a_{2}, \\ldots, A_{n} A_{1}=a_{n}$ if and only if the system $$ x_{1}+x_{2}=a_{1}, x_{2}+x_{3}=a_{2}, \\ldots, x_{n}+x_{1}=a_{n} $$ has a solution $\\left(x_{1}, \\ldots, x_{n}\\right)$ in positive reals. Proof. Suppose that such an $n$-gon $A_{1} A_{2} \\ldots A_{n}$ exists. Let the side $A_{i} A_{i+1}$ touch the inscribed circle at point $P_{i}$ (where $\\left.A_{n+1}=A_{1}\\right)$. Then $x_{1}=$ $A_{1} P_{n}=A_{1} P_{1}, x_{2}=A_{2} P_{1}=A_{2} P_{2}, \\ldots, x_{n}=A_{n} P_{n-1}=A_{n} P_{n}$ is clearly a positive solution of (1). Now suppose that the system (1) has a positive real solution $\\left(x_{1}, \\ldots\\right.$, $x_{n}$ ). Let us draw a polygonal line $A_{1} A_{2} \\ldots A_{n+1}$ touching a circle of radius $r$ at points $P_{1}, P_{2}, \\ldots, P_{n}$ respectively such that $A_{1} P_{1}=$ $A_{n+1} P_{n}=x_{1}$ and $A_{i} P_{i}=A_{i} P_{i-1}=x_{i}$ for $i=2, \\ldots, n$. Observe that $O A_{1}=O A_{n+1}=\\sqrt{x_{1}^{2}+r^{2}}$ and the function $f(r)=\\angle A_{1} O A_{2}+$ $\\angle A_{2} O A_{3}+\\cdots+\\angle A_{n} O A_{n+1}=$ $2\\left(\\arctan \\frac{x_{1}}{r}+\\cdots+\\arctan \\frac{x_{n}}{r}\\right)$ is continuous. Thus $A_{1} A_{2} \\ldots A_{n+1}$ is a closed simple polygonal line if and only if $f(r)=360^{\\circ}$. But such an $r$ exists, since $f(r) \\rightarrow 0$ ![](https:\/\/cdn.mathpix.com\/cropped\/2024_11_18_8e985d6b9c83aa3e9d0eg-570.jpg?height=331&width=444&top_left_y=1156&top_left_x=887) when $r \\rightarrow \\infty$ and $f(r) \\rightarrow \\infty$ when $r \\rightarrow 0$. This proves the second direction of the lemma. For $n=4 k$, the system (1) is solvable in positive reals if $a_{i}=i$ for $i \\equiv 1,2$ $(\\bmod 4), a_{i}=i+1$ for $i \\equiv 3$ and $a_{i}=i-1$ for $i \\equiv 0(\\bmod 4)$. Indeed, one solution is given by $x_{i}=1 \/ 2$ for $i \\equiv 1, x_{i}=3 \/ 2$ for $i \\equiv 3$ and $x_{i}=i-3 \/ 2$ for $i \\equiv 0,2(\\bmod 4)$. Remark. For $n=4 k+2$ there is no such $n$-gon. In fact, solvability of the system (1) implies $a_{1}+a_{3}+\\cdots=a_{2}+a_{4}+\\cdots$, while in the case $n=4 k+2$ the sum $a_{1}+a_{2}+\\cdots+a_{n}$ is odd.","problem_type":null,"tier":0} +{"year":"1992","problem_phase":"shortlisted","problem":"9. (IRN 1) Let $f(x)$ be a polynomial with rational coefficients and $\\alpha$ be a real number such that $\\alpha^{3}-\\alpha=f(\\alpha)^{3}-f(\\alpha)=33^{1992}$. Prove that for each $n \\geq 1$, $$ \\left(f^{(n)}(\\alpha)\\right)^{3}-f^{(n)}(\\alpha)=33^{1992} $$ where $f^{(n)}(x)=f(f(\\ldots f(x)))$, and $n$ is a positive integer.","solution":"9. Since the equation $x^{3}-x-c=0$ has only one real root for every $c>$ $2 \/(3 \\sqrt{3}), \\alpha$ is the unique real root of $x^{3}-x-33^{1992}=0$. Hence $f^{n}(\\alpha)=$ $f(\\alpha)=\\alpha$. Remark. Consider any irreducible polynomial $g(x)$ in the place of $x^{3}-$ $x-33^{1992}$. The problem amounts to proving that if $\\alpha$ and $f(\\alpha)$ are roots of $g$, then any $f^{(n)}(\\alpha)$ is also a root of $g$. In fact, since $g(f(x))$ vanishes at $x=\\alpha$, it must be divisible by the minimal polynomial of $\\alpha$, that is, $g(x)$. It follows by induction that $g\\left(f^{(n)}(x)\\right)$ is divisible by $g(x)$ for all $n \\in \\mathbb{N}$, and hence $g\\left(f^{(n)}(\\alpha)\\right)=0$.","problem_type":null,"tier":0} +{"year":"1993","problem_phase":"shortlisted","problem":"1. (BRA 1) Show that there exists a finite set $A \\subset \\mathbb{R}^{2}$ such that for every $X \\in A$ there are points $Y_{1}, Y_{2}, \\ldots, Y_{1993}$ in $A$ such that the distance between $X$ and $Y_{i}$ is equal to 1 , for every $i$.","solution":"1. First we notice that for a rational point $O$ (i.e., with rational coordinates), there exist 1993 rational points in each quadrant of the unit circle centered at $O$. In fact, it suffices to take $$ X=\\left\\{\\left.O+\\left( \\pm \\frac{t^{2}-1}{t^{2}+1}, \\pm \\frac{2 t}{t^{2}+1}\\right) \\right\\rvert\\, t=1,2, \\ldots, 1993\\right\\} $$ Now consider the set $A=\\{(i \/ q, j \/ q) \\mid i, j=0,1, \\ldots, 2 q\\}$, where $q=$ $\\prod_{i=1}^{1993}\\left(t^{2}+1\\right)$. We claim that $A$ gives a solution for the problem. Indeed, for any $P \\in A$ there is a quarter of the unit circle centered at $P$ that is contained in the square $[0,2] \\times[0,2]$. As explained above, there are 1993 rational points on this quarter circle, and by definition of $q$ they all belong to $A$. Remark. Substantially the same problem was proposed by Bulgaria for IMO 71: see (SL71-2), where we give another possible construction of a set $A$.","problem_type":null,"tier":0} +{"year":"1993","problem_phase":"shortlisted","problem":"10. (IND 5) A natural number $n$ is said to have the property $P$ if whenever $n$ divides $a^{n}-1$ for some integer $a, n^{2}$ also necessarily divides $a^{n}-1$. (a) Show that every prime number has property $P$. (b) Show that there are infinitely many composite numbers $n$ that possess property $P$.","solution":"10. (a) Let $n=p$ be a prime and let $p \\mid a^{p}-1$. By Fermat's theorem $p \\mid$ $a^{p-1}-1$, so that $p \\mid a^{\\operatorname{gcd}(p, p-1)}-1=a-1$, i.e., $a \\equiv 1(\\bmod p)$. Since then $a^{i} \\equiv 1(\\bmod p)$, we obtain $p \\mid a^{p-1}+\\cdots+a+1$ and hence $p^{2} \\mid a^{p}-1=(a-1)\\left(a^{p-1}+\\cdots+a+1\\right)$. (b) Let $n=p_{1} \\cdots p_{k}$ be a product of distinct primes and let $n \\mid a^{n}-1$. Then from $p_{i} \\mid a^{n}-1=\\left(a^{\\left(n \/ p_{i}\\right)}\\right)^{p_{i}}-1$ and part (a) we conclude that $p_{i}^{2} \\mid a^{n}-1$. Since this is true for all indices $i$, we also have $n^{2} \\mid a^{n}-1$; hence $n$ has the property $P$.","problem_type":null,"tier":0} +{"year":"1993","problem_phase":"shortlisted","problem":"11. (IRE 1) ${ }^{\\mathrm{IMO1}}$ Let $n>1$ be an integer and let $f(x)=x^{n}+5 x^{n-1}+3$. Prove that there do not exist polynomials $g(x), h(x)$, each having integer coefficients and degree at least one, such that $f(x)=g(x) h(x)$.","solution":"11. Due to the extended Eisenstein criterion, $f$ must have an irreducible factor of degree not less than $n-1$. Since $f$ has no integral zeros, it must be irreducible. Second solution. The proposer's solution was as follows. Suppose that $f(x)=g(x) h(x)$, where $g, h$ are nonconstant polynomials with integer coefficients. Since $|f(0)|=3$, either $|g(0)|=1$ or $|h(0)|=1$. We may assume $|g(0)|=1$ and that $g(x)=\\left(x-\\alpha_{1}\\right) \\cdots\\left(x-\\alpha_{k}\\right)$. Then $\\left|\\alpha_{1} \\cdots \\alpha_{k}\\right|=$ 1. Since $\\alpha_{i}^{n-1}\\left(\\alpha_{i}+5\\right)=-3$, taking the product over $i=1,2, \\ldots, k$ yields $\\left|\\left(\\alpha_{1}+5\\right) \\cdots\\left(\\alpha_{k}+5\\right)\\right|=|g(-5)|=3^{k}$. But $f(-5)=g(-5) h(-5)=3$, so the only possibility is $\\operatorname{deg} g=k=1$. This is impossible, because $f$ has no integral zeros. Remark. Generalizing this solution, it can be shown that if $a, m, n$ are positive integers and $p1$, let $S_{N}=\\{(m, n) \\in S \\mid m+n=N\\}$. If $f(m, n)=\\left(m_{1}, n_{1}\\right)$, then $m_{1}+n_{1}=m+n$ with $m_{1}$ odd and $m_{1} \\leq \\frac{n}{2}<$ $\\frac{N}{2}1$ and $b^{n}-1 \\mid a$. Show that the representation of the number $a$ in the base $b$ contains at least $n$ digits different from zero.","solution":"19. Let $s$ be the minimum number of nonzero digits that can appear in the $b$ adic representation of any number divisible by $b^{n}-1$. Among all numbers divisible by $b^{n}-1$ and having $s$ nonzero digits in base $b$, we choose the number $A$ with the minimum sum of digits. Let $A=a_{1} b^{n_{1}}+\\cdots+a_{s} b^{n_{s}}$, where $0n_{2}>\\cdots>n_{s}$. First, suppose that $n_{i} \\equiv n_{j}(\\bmod n), i \\neq j$. Consider the number $$ B=A-a_{i} b^{n_{i}}-a_{j} b^{n_{j}}+\\left(a_{i}+a_{j}\\right) b^{n_{j}+k n} $$ with $k$ chosen large enough so that $n_{j}+k n>n_{1}$ : this number is divisible by $b^{n}-1$ as well. But if $a_{i}+a_{j}-\\frac{c_{i}}{n}$ implies $$ \\sum_{i=1}^{n} q_{i}>-\\sum_{i=1}^{n} \\frac{c_{i}}{n} \\geq-1 $$ which leads to $\\sum_{i=1}^{n} q_{i} \\geq 0$. The proof is complete.","problem_type":null,"tier":0} +{"year":"1993","problem_phase":"shortlisted","problem":"21. (GBR 1) A circle $S$ is said to cut a circle $\\Sigma$ diametrally if their common chord is a diameter of $\\Sigma$. Let $S_{A}, S_{B}, S_{C}$ be three circles with distinct centers $A, B, C$ respectively. Prove that $A, B, C$ are collinear if and only if there is no unique circle $S$ that cuts each of $S_{A}, S_{B}, S_{C}$ diametrally. Prove further that if there exists more than one circle $S$ that cuts each of $S_{A}, S_{B}, S_{C}$ diametrally, then all such circles pass through two fixed points. Locate these points in relation to the circles $S_{A}, S_{B}, S_{C}$.","solution":"21. Assume that $S$ is a circle with center $O$ that cuts $S_{i}$ diametrically in points $P_{i}, Q_{i}, i \\in\\{A, B, C\\}$, and denote by $r_{i}, r$ the radii of $S_{i}$ and $S$ respectively. Since $O A$ is perpendicular to $P_{A} Q_{A}$, it follows by Pythagoras's theorem that $O A^{2}+A P_{A}^{2}=O P_{A}^{2}$, i.e., $r_{A}^{2}+O A^{2}=r^{2}$. Analogously $r_{B}^{2}+O B^{2}=r^{2}$ and $r_{C}^{2}+O C^{2}=r^{2}$. Thus if $O_{A}, O_{B}, O_{C}$ are the feet of perpendiculars from $O$ to $B C, C A, A B$ respectively, then $O_{C} A^{2}-O_{C} B^{2}=r_{B}^{2}-r_{A}^{2}$. Since the left-hand side is a monotonic function of $O_{C} \\in A B$, the point $O_{C}$ is uniquely determined by the imposed conditions. The same holds for $O_{A}$ and $O_{B}$. If $A, B, C$ are not collinear, then the positions of $O_{A}, O_{B}, O_{C}$ uniquely determine the point $O$, and therefore the circle $S$ also. On the other hand, if $A, B, C$ are collinear, all one can deduce is that $O$ lies on the lines $l_{A}, l_{B}, l_{C}$ through $O_{A}, O_{B}, O_{C}$, perpendicular to $B C, C A, A B$ respectively. By this, $l_{A}, l_{B}, l_{C}$ are parallel, so $O$ can be either anywhere on the line if these lines coincide, or ![](https:\/\/cdn.mathpix.com\/cropped\/2024_11_18_8e985d6b9c83aa3e9d0eg-586.jpg?height=480&width=544&top_left_y=1409&top_left_x=810) nowhere if they don't coincide. So if there exists more than one circle $S$, $A, B, C$ lie on a line and the foot $O^{\\prime}$ of the perpendicular from $O$ to the line $A B C$ is fixed. If $X, Y$ are the intersection points of $S$ and the line $A B C$, then $r^{2}=O X^{2}=O A^{2}+r_{A}^{2}$ and consequently $O^{\\prime} X^{2}=O^{\\prime} A^{2}+r_{A}^{2}$, which implies that $X, Y$ are fixed.","problem_type":null,"tier":0} +{"year":"1993","problem_phase":"shortlisted","problem":"22. (GBR 2) ${ }^{\\mathrm{IMO} 2} A, B, C, D$ are four points in the plane, with $C, D$ on the same side of the line $A B$, such that $A C \\cdot B D=A D \\cdot B C$ and $\\measuredangle A D B=$ $90^{\\circ}+\\measuredangle A C B$. Find the ratio $$ \\frac{A B \\cdot C D}{A C \\cdot B D} $$ and prove that circles $A C D, B C D$ are orthogonal. (Intersecting circles are said to be orthogonal if at either common point their tangents are perpendicular.)","solution":"22. Let $M$ be the point inside $\\angle A D B$ that satisfies $D M=D B$ and $D M \\perp$ $D B$. Then $\\angle A D M=\\angle A C B$ and $A D \/ D M=A C \/ C B$. It follows that the triangles $A D M, A C B$ are similar; hence $\\angle C A D=\\angle B A M$ (because $\\angle C A B=\\angle D A M$ ) and $A B \/ A M=A C \/ A D$. Consequently the triangles $C A D, B A M$ are similar and therefore $\\frac{A C}{A B}=\\frac{C D}{B M}=$ $\\frac{C D}{\\sqrt{2} B D}$. Hence $\\frac{A B \\cdot C D}{A C \\cdot B D}=\\sqrt{2}$. ![](https:\/\/cdn.mathpix.com\/cropped\/2024_11_18_8e985d6b9c83aa3e9d0eg-587.jpg?height=380&width=391&top_left_y=239&top_left_x=882) Let $C T, C U$ be the tangents at $C$ to the circles $A C D, B C D$ respectively. Then (in oriented angles) $\\angle T C U=\\angle T C D+\\angle D C U=\\angle C A D+\\angle C B D=$ $90^{\\circ}$, as required. Second solution to the first part. Denote by $E, F, G$ the feet of the perpendiculars from $D$ to $B C, C A, A B$. Consider the pedal triangle $E F G$. Since $F G=A D \\sin \\angle A$, from the sine theorem we have $F G: G E: E F=$ $(C D \\cdot A B):(B D \\cdot A C):(A D \\cdot B C)$. Thus $E G=F G$. On the other hand, $\\angle E G F=\\angle E G D+\\angle D G F=\\angle C B D+\\angle C A D=90^{\\circ}$ implies that $E F: E G=\\sqrt{2}: 1$; hence the required ratio is $\\sqrt{2}$. Third solution to the first part. Under inversion centered at $C$ and with power $r^{2}=C A \\cdot C B$, the triangle $D A B$ maps into a right-angled isosceles triangle $D^{*} A^{*} B^{*}$, where $$ D^{*} A^{*}=\\frac{A D \\cdot B C}{C D}, D^{*} B^{*}=\\frac{A C \\cdot B D}{C D}, A^{*} B^{*}=\\frac{A B \\cdot C D}{C D} . $$ Thus $D^{*} B^{*}: A^{*} B^{*}=\\sqrt{2}$, and this is the required ratio.","problem_type":null,"tier":0} +{"year":"1993","problem_phase":"shortlisted","problem":"23. (GBR 3) A finite set of (distinct) positive integers is called a \" $D S$-set\" if each of the integers divides the sum of them all. Prove that every finite set of positive integers is a subset of some $D S$-set.","solution":"23. Let the given numbers be $a_{1}, \\ldots, a_{n}$. Put $s=a_{1}+\\cdots+a_{n}$ and $m=$ $\\operatorname{lcm}\\left(a_{1}, \\ldots, a_{n}\\right)$ and write $m=2^{k} r$ with $k \\geq 0$ and $r$ odd. Let the binary expansion of $r$ be $r=2^{k_{0}}+2^{k_{1}}+\\cdots+2^{k_{t}}$, with $0=k_{0}<\\cdots1$ : $$ \\begin{aligned} x_{1}^{2} & =a x_{2}+1, \\\\ x_{2}^{2} & =a x_{3}+1 \\\\ \\cdots & \\cdots \\\\ x_{999}^{2} & =a x_{1000}+1 \\\\ x_{1000}^{2} & =a x_{1}+1 \\end{aligned} $$","solution":"25. We need only consider the case $a>1$ (since the case $a<-1$ is reduced to $a>1$ by taking $a^{\\prime}=-a, x_{i}^{\\prime}=-x_{i}$ ). Since the left sides of the equations are nonnegative, we have $x_{i} \\geq-\\frac{1}{a}>-1, i=1, \\ldots, 1000$. Suppose w.l.o.g. that $x_{1}=\\max \\left\\{x_{i}\\right\\}$. In particular, $x_{1} \\geq x_{2}, x_{3}$. If $x_{1} \\geq 0$, then we deduce that $x_{1000}^{2} \\geq 1 \\Rightarrow x_{1000} \\geq 1$; further, from this we deduce that $x_{999}>1$ etc., so either $x_{i}>1$ for all $i$ or $x_{i}<0$ for all $i$. (i) $x_{i}>1$ for every $i$. Then $x_{1} \\geq x_{2}$ implies $x_{1}^{2} \\geq x_{2}^{2}$, so $x_{2} \\geq x_{3}$. Thus $x_{1} \\geq x_{2} \\geq \\cdots \\geq x_{1000} \\geq x_{1}$, and consequently $x_{1}=\\cdots=x_{1000}$. In this case the only solution is $x_{i}=\\frac{1}{2}\\left(a+\\sqrt{a^{2}+4}\\right)$ for all $i$. (ii) $x_{i}<0$ for every $i$. Then $x_{1} \\geq x_{3}$ implies $x_{1}^{2} \\leq x_{3}^{2} \\Rightarrow x_{2} \\leq x_{4}$. Similarly, this leads to $x_{3} \\geq x_{5}$, etc. Hence $x_{1} \\geq x_{3} \\geq x_{5} \\geq \\cdots \\geq x_{999} \\geq x_{1}$ and $x_{2} \\leq x_{4} \\leq \\cdots \\leq x_{2}$, so we deduce that $x_{1}=x_{3}=\\cdots$ and $x_{2}=x_{4}=$ $\\cdots$. Therefore the system is reduced to $x_{1}^{2}=a x_{2}+1, x_{2}^{2}=a x_{1}+1$. Subtracting these equations, one obtains $\\left(x_{1}-x_{2}\\right)\\left(x_{1}+x_{2}+a\\right)=0$. There are two possibilities: (1) If $x_{1}=x_{2}$, then $x_{1}=x_{2}=\\cdots=\\frac{1}{2}\\left(a-\\sqrt{a^{2}+4}\\right)$. (2) $x_{1}+x_{2}+a=0$ is equivalent to $x_{1}^{2}+a x_{1}+\\left(a^{2}-1\\right)=0$. The discriminant of the last equation is $4-3 a^{2}$. Therefore if $a>\\frac{2}{\\sqrt{3}}$, this case yields no solutions, while if $a \\leq \\frac{2}{\\sqrt{3}}$, we obtain $x_{1}=$ $\\frac{1}{2}\\left(-a-\\sqrt{4-3 a^{2}}\\right), x_{2}=\\frac{1}{2}\\left(-a+\\sqrt{4-3 a^{2}}\\right)$, or vice versa.","problem_type":null,"tier":0} +{"year":"1993","problem_phase":"shortlisted","problem":"26. (VIE 2) Let $a, b, c, d$ be four nonnegative numbers satisfying $a+b+c+d=$ 1. Prove the inequality $$ a b c+b c d+c d a+d a b \\leq \\frac{1}{27}+\\frac{176}{27} a b c d $$","solution":"26. Set $$ \\begin{aligned} f(a, b, c, d) & =a b c+b c d+c d a+d a b-\\frac{176}{27} a b c d \\\\ & =a b(c+d)+c d\\left(a+b-\\frac{176}{27} a b\\right) . \\end{aligned} $$ If $a+b-\\frac{176}{a} b \\leq 0$, by the arithmetic-geometric inequality we have $f(a, b, c, d) \\leq a b(c+d) \\leq \\frac{1}{27}$. On the other hand, if $a+b-\\frac{176}{a} b>0$, the value of $f$ increases if $c, d$ are replaced by $\\frac{c+d}{2}, \\frac{c+d}{2}$. Consider now the following fourtuplets: $$ \\begin{gathered} P_{0}(a, b, c, d), P_{1}\\left(a, b, \\frac{c+d}{2}, \\frac{c+d}{2}\\right), P_{2}\\left(\\frac{a+b}{2}, \\frac{a+b}{2}, \\frac{c+d}{2}, \\frac{c+d}{2}\\right), \\\\ P_{3}\\left(\\frac{1}{4}, \\frac{a+b}{2}, \\frac{c+d}{2}, \\frac{1}{4}\\right), P_{4}\\left(\\frac{1}{4}, \\frac{1}{4}, \\frac{1}{4}, \\frac{1}{4}\\right) \\end{gathered} $$ From the above considerations we deduce that for $i=0,1,2,3$ either $f\\left(P_{i}\\right) \\leq f\\left(P_{i+1}\\right)$, or directly $f\\left(P_{i}\\right) \\leq 1 \/ 27$. Since $f\\left(P_{4}\\right)=1 \/ 27$, in every case we are led to $$ f(a, b, c, d)=f\\left(P_{0}\\right) \\leq \\frac{1}{27} $$ Equality occurs only in the cases $(0,1 \/ 3,1 \/ 3,1 \/ 3)$ (with permutations) and ( $1 \/ 4,1 \/ 4,1 \/ 4,1 \/ 4)$. Remark. Lagrange multipliers also work. On the boundary of the set one of the numbers $a, b, c, d$ is 0 , and the inequality immediately follows, while for an extremum point in the interior, among $a, b, c, d$ there are at most two distinct values, in which case one easily verifies the inequality.","problem_type":null,"tier":0} +{"year":"1993","problem_phase":"shortlisted","problem":"3. (SPA 1) Consider the triangle $A B C$, its circumcircle $k$ with center $O$ and radius $R$, and its incircle with center $I$ and radius $r$. Another circle $k_{c}$ is tangent to the sides $C A, C B$ at $D, E$, respectively, and it is internally tangent to $k$. Show that the incenter $I$ is the midpoint of $D E$.","solution":"3. Let $O_{1}$ and $\\rho$ be the center and radius of $k_{c}$. It is clear that $C, I, O_{1}$ are collinear and $C I \/ C O_{1}=r \/ \\rho$. By Stewart's theorem applied to $\\triangle O C O_{1}$, $$ O I^{2}=\\frac{r}{\\rho} O O_{1}^{2}+\\left(1-\\frac{r}{\\rho}\\right) O C^{2}-C I \\cdot I O_{1} . $$ Since $O O_{1}=R-\\rho, O C=R$ and by Euler's formula $O I^{2}=R^{2}-2 R r$, substituting these values in (1) gives $C I \\cdot I O_{1}=r \\rho$, or equivalently $C O_{1}$. $I O_{1}=\\rho^{2}=D O_{1}^{2}$. Hence the triangles $C O_{1} D$ and $D O_{1} I$ are similar, implying $\\angle D I O_{1}=90^{\\circ}$. Since $C D=C E$ and the line $C O_{1}$ bisects the segment $D E$, it follows that $I$ is the midpoint of $D E$. Second solution. Under the inversion with center $C$ and power $a b, k_{c}$ is transformed into the excircle of $\\widehat{A} \\widehat{B} C$ corresponding to $C$. Thus $C D=$ $\\frac{a b}{s}$, where $s$ is the common semiperimeter of $\\triangle A B C$ and $\\triangle \\widehat{A} \\widehat{B} C$, and consequently the distance from $D$ to $B C$ is $\\frac{a b}{s} \\sin C=\\frac{2 S_{A B C}}{s}=2 r$. The statement follows immediately. Third solution. We shall prove a stronger statement: Let $A B C D$ be a convex quadrilateral inscribed in a circle $k$, and $k^{\\prime}$ the circle that is tangent to segments $B O, A O$ at $K, L$ respectively (where $O=B D \\cap A C$ ), and internally to $k$ at $M$. Then $K L$ contains the incenters $I, J$ of $\\triangle A B C$ and $\\triangle A B D$. Let $K^{\\prime}, K^{\\prime \\prime}, L^{\\prime}, L^{\\prime \\prime}, N$ denote the midpoints of arcs $B C, B D, A C, A D, A B$ that don't contain $M ; X^{\\prime}, X^{\\prime \\prime}$ the points on $k$ defined by $X^{\\prime} N=N X^{\\prime \\prime}=$ $K^{\\prime} K^{\\prime \\prime}=L^{\\prime} L^{\\prime \\prime}$ (as oriented arcs); and set $S=A K^{\\prime} \\cap B L^{\\prime \\prime}, \\bar{M}=N S \\cap k$, $\\bar{K}=K^{\\prime \\prime} M \\cap B O, \\bar{L}=L^{\\prime} M \\cap A O$. It is clear that $I=A K^{\\prime} \\cap B L^{\\prime}, J=A K^{\\prime \\prime} \\cap B L^{\\prime \\prime}$. Furthermore, $X^{\\prime} \\bar{M}$ contains $I$ (to see this, use the fact that for $A, B, C, D, E, F$ on $k$, lines $A D, B E, C F$ are concurrent if and only if $A B \\cdot C D \\cdot E F=B C \\cdot D E \\cdot F A$, and then express $A \\bar{M} \/ \\bar{M} B$ by applying this rule to $A M B K^{\\prime} N L^{\\prime \\prime}$ and show that $A K^{\\prime}, \\bar{M} X^{\\prime}, B L^{\\prime}$ are concurrent). Analogously, $X^{\\prime \\prime} \\bar{M}$ contains $J$. Now the points $B, \\bar{K}, I, S, \\bar{M}$ lie on a circle $(\\angle B \\overline{K M}=\\angle B I \\bar{M}=\\angle B S \\bar{M})$, and points $A, \\bar{L}, J, S, \\bar{M}$ do so as well. Lines $I \\bar{K}, J \\bar{L}$ are parallel to $K^{\\prime \\prime} L^{\\prime}$ (because $\\angle \\overline{M K} I=\\angle \\bar{M} B I=$ $\\left.\\angle \\bar{M} K^{\\prime \\prime} L^{\\prime}\\right)$. On the other hand, the quadrilateral $A B I J$ is cyclic, and simple calculation with angles shows that $I J$ is also parallel to $K^{\\prime \\prime} L^{\\prime}$. Hence $\\bar{K}, I, J, \\bar{L}$ are collinear. ![](https:\/\/cdn.mathpix.com\/cropped\/2024_11_18_8e985d6b9c83aa3e9d0eg-578.jpg?height=496&width=523&top_left_y=967&top_left_x=827) Finally, $\\bar{K} \\equiv K, \\bar{L} \\equiv L$, and $\\bar{M} \\equiv M$ because the homothety centered at $M$ that maps $k^{\\prime}$ to $k$ sends $K$ to $K^{\\prime \\prime}$ and $L$ to $L^{\\prime}$ (thus $M, K, K^{\\prime \\prime}$, as well as $M, L, L^{\\prime}$, must be collinear). As is seen now, the deciphered picture yields many other interesting properties. Thus, for example, $N, S, M$ are collinear, i.e., $\\angle A M S=\\angle B M S$. Fourth solution. We give an alternative proof of the more general statement in the third solution. Let $W$ be the foot of the perpendicular from $B$ to $A C$. We define $q=C W, h=B W, t=O L=O K, x=A L$, $\\theta=\\measuredangle W B O(\\theta$ is negative if $\\mathcal{B}(O, W, A), \\theta=0$ if $W=O)$, and as usual, $a=B C, b=A C, c=A B$. Let $\\alpha=\\measuredangle K L C$ and $\\beta=\\measuredangle I L C$ (both angles must be acute). Our goal is to prove $\\alpha=\\beta$. We note that $90^{\\circ}-\\theta=2 \\alpha$. One easily gets $$ \\tan \\alpha=\\frac{\\cos \\theta}{1+\\sin \\theta}, \\quad \\tan \\beta=\\frac{\\frac{2 S_{A B C}}{a+b+c}}{\\frac{b+c-a}{2}-x} $$ Applying Casey's theorem to $A, B, C, k^{\\prime}$, we get $A C \\cdot B K+A L \\cdot B C=$ $A B \\cdot C L$, i.e., $b\\left(\\frac{h}{\\cos \\theta}-t\\right)+x a=c(b-x)$. Using that $t=b-x-q-h \\tan \\theta$ we get $$ x=\\frac{b(b+c-q)-b h\\left(\\frac{1}{\\cos \\theta}+\\tan \\theta\\right)}{a+b+c} . $$ Plugging (2) into the second equation of (1) and using $b h=2 S_{A B C}$ and $c^{2}=b^{2}+a^{2}-2 b q$, we obtain $\\tan \\alpha=\\tan \\beta$, i.e., $\\alpha=\\beta$, which completes our proof.","problem_type":null,"tier":0} +{"year":"1993","problem_phase":"shortlisted","problem":"4. (SPA 2) In the triangle $A B C$, let $D, E$ be points on the side $B C$ such that $\\angle B A D=\\angle C A E$. If $M, N$ are, respectively, the points of tangency with $B C$ of the incircles of the triangles $A B D$ and $A C E$, show that $$ \\frac{1}{M B}+\\frac{1}{M D}=\\frac{1}{N C}+\\frac{1}{N E} $$","solution":"4. Let $h$ be the altitude from $A$ and $\\varphi=\\angle B A D$. We have $B M=\\frac{1}{2}(B D+$ $A B-A D)$ and $M D=\\frac{1}{2}(B D-A B+A D)$, so $$ \\begin{aligned} \\frac{1}{M B}+\\frac{1}{M D} & =\\frac{B D}{M B \\cdot M D}=\\frac{4 B D}{B D^{2}-A B^{2}-A D^{2}+2 A B \\cdot A D} \\\\ & =\\frac{4 B D}{2 A B \\cdot A D(1-\\cos \\varphi)}=\\frac{2 B D \\sin \\varphi}{2 S_{A B D}(1-\\cos \\varphi)} \\\\ & =\\frac{2 B D \\sin \\varphi}{B D \\cdot h(1-\\cos \\varphi)}=\\frac{2}{h \\tan \\frac{\\varphi}{2}} . \\end{aligned} $$ It follows that $\\frac{1}{M B}+\\frac{1}{M D}$ depends only on $h$ and $\\varphi$. Specially, $\\frac{1}{N C}+\\frac{1}{N E}=$ $\\frac{2}{h \\tan (\\varphi \/ 2)}$ as well.","problem_type":null,"tier":0} +{"year":"1993","problem_phase":"shortlisted","problem":"5. (FIN 3) ${ }^{\\mathrm{IMO} 3}$ On an infinite chessboard, a solitaire game is played as follows: At the start, we have $n^{2}$ pieces occupying $n^{2}$ squares that form a square of side $n$. The only allowed move is a jump horizontally or vertically over an occupied square to an unoccupied one, and the piece that has been jumped over is removed. For what positive integers $n$ can the game end with only one piece remaining on the board?","solution":"5. For $n=1$ the game is trivially over. If $n=2$, it can end, for example, in the following way: ![](https:\/\/cdn.mathpix.com\/cropped\/2024_11_18_8e985d6b9c83aa3e9d0eg-579.jpg?height=144&width=534&top_left_y=1229&top_left_x=510) Fig. 1 The sequence of moves shown in Fig. 2 enables us to remove three pieces placed in a $1 \\times 3$ rectangle, using one more piece and one more free cell. In that way, for any $n \\geq 4$ we can reduce an $(n+3) \\times(n+3)$ square to an $n \\times n$ square (Fig. 3). Therefore the game can end for every $n$ that is not divisible by 3 . ![](https:\/\/cdn.mathpix.com\/cropped\/2024_11_18_8e985d6b9c83aa3e9d0eg-579.jpg?height=123&width=595&top_left_y=1716&top_left_x=265) Fig. 2 ![](https:\/\/cdn.mathpix.com\/cropped\/2024_11_18_8e985d6b9c83aa3e9d0eg-579.jpg?height=272&width=272&top_left_y=1642&top_left_x=1050) Fig. 3 Suppose now that one can play the game on a $3 k \\times 3 k$ square so that at the end only one piece remains. Denote the cells by $(i, j), i, j \\in\\{1, \\ldots, 3 k\\}$, and let $S_{0}, S_{1}, S_{2}$ denote the numbers of pieces on those squares $(i, j)$ for which $i+j$ gives remainder $0,1,2$ respectively upon division by 3 . Initially $S_{0}=S_{1}=S_{2}=3 k^{2}$. After each move, two of $S_{0}, S_{1}, S_{2}$ diminish and one increases by one. Thus each move reverses the parity of the $S_{i}$ 's, so that $S_{0}, S_{1}, S_{2}$ are always of the same parity. But in the final position one of the $S_{i}$ 's must be equal to 1 and the other two must be 0 , which is impossible.","problem_type":null,"tier":0} +{"year":"1993","problem_phase":"shortlisted","problem":"6. (GER 1) ${ }^{\\mathrm{IMO} 5}$ Let $\\mathbb{N}=\\{1,2,3, \\ldots\\}$. Determine whether there exists a strictly increasing function $f: \\mathbb{N} \\rightarrow \\mathbb{N}$ with the following properties: $$ \\begin{aligned} f(1) & =2 \\\\ f(f(n)) & =f(n)+n \\quad(n \\in \\mathbb{N}) \\end{aligned} $$","solution":"6. Notice that for $\\alpha=\\frac{1+\\sqrt{5}}{2}, \\alpha^{2} n=\\alpha n+n$ for all $n \\in \\mathbb{N}$. We shall show that $f(n)=\\left[\\alpha n+\\frac{1}{2}\\right]$ (the closest integer to $\\alpha n$ ) satisfies the requirements. Observe that $f$ is strictly increasing and $f(1)=2$. By the definition of $f$, $|f(n)-\\alpha n| \\leq \\frac{1}{2}$ and $f(f(n))-f(n)-n$ is an integer. On the other hand, $$ \\begin{aligned} |f(f(n))-f(n)-n| & =\\left|f(f(n))-f(n)-\\alpha^{2} n+\\alpha n\\right| \\\\ & =\\left|f(f(n))-\\alpha f(n)+\\alpha f(n)-\\alpha^{2} n-f(n)+\\alpha n\\right| \\\\ & =|(\\alpha-1)(f(n)-\\alpha n)+(f(f(n))-\\alpha f(n))| \\\\ & \\leq(\\alpha-1)|f(n)-\\alpha n|+|f(f(n))-\\alpha f(n)| \\\\ & \\leq \\frac{1}{2}(\\alpha-1)+\\frac{1}{2}=\\frac{1}{2} \\alpha<1, \\end{aligned} $$ which implies that $f(f(n))-f(n)-n=0$.","problem_type":null,"tier":0} +{"year":"1993","problem_phase":"shortlisted","problem":"7. (GEO 3) Let $a, b, c$ be given integers $a>0, a c-b^{2}=P=P_{1} \\cdots P_{m}$ where $P_{1}, \\ldots, P_{m}$ are (distinct) prime numbers. Let $M(n)$ denote the number of pairs of integers $(x, y)$ for which $$ a x^{2}+2 b x y+c y^{2}=n $$ Prove that $M(n)$ is finite and $M(n)=M\\left(P^{k} \\cdot n\\right)$ for every integer $k \\geq 0$.","solution":"7. Multiplying by $a$ and $c$ the equation $$ a x^{2}+2 b x y+c y^{2}=P^{k} n $$ gives $(a x+b y)^{2}+P y^{2}=a P^{k} n$ and $(b x+c y)^{2}+P x^{2}=c P^{k} n$. It follows immediately that $M(n)$ is finite; moreover, $(a x+b y)^{2}$ and $(b x+$ $c y)^{2}$ are divisible by $P$, and consequently $a x+b y, b x+c y$ are divisible by $P$ because $P$ is not divisible by a square greater than 1 . Thus there exist integers $X, Y$ such that $b x+c y=P X, a x+b y=-P Y$. Then $x=-b X-c Y$ and $y=a X+b Y$. Introducing these values into (1) and simplifying the expression obtained we get $$ a X^{2}+2 b X Y+c Y^{2}=P^{k-1} n $$ Hence $(x, y) \\mapsto(X, Y)$ is a bijective correspondence between integral solutions of (1) and (2), so that $M\\left(P^{k} n\\right)=M\\left(P^{k-1} n\\right)=\\cdots=M(n)$.","problem_type":null,"tier":0} +{"year":"1993","problem_phase":"shortlisted","problem":"8. (IND 1) Define a sequence $\\langle f(n)\\rangle_{n=1}^{\\infty}$ of positive integers by $f(1)=1$ and $$ f(n)= \\begin{cases}f(n-1)-n, & \\text { if } f(n-1)>n ; \\\\ f(n-1)+n, & \\text { if } f(n-1) \\leq n,\\end{cases} $$ for $n \\geq 2$. Let $S=\\{n \\in \\mathbb{N} \\mid f(n)=1993\\}$. (a) Prove that $S$ is an infinite set. (b) Find the least positive integer in $S$. (c) If all the elements of $S$ are written in ascending order as $n_{1}0$. Then $f(n+1)=n+2, f(n+$ $2)=2 n+4, f(n+3)=n+1, f(n+4)=2 n+5, f(n+5)=n$, and so by induction $f(n+2 k)=2 n+3+k, f(n+2 k-1)=n+3-k$ for $k=1,2, \\ldots, n+2$. Particularly, $n^{\\prime}=3 n+3$ is the smallest value greater than $n$ for which $f\\left(n^{\\prime}\\right)=1$. It follows that all numbers $n$ with $f(n)=1$ are given by $n=b_{i}$, where $b_{0}=1, b_{n}=3 b_{n-1}+3$. Furthermore, $b_{n}=3+3 b_{n-1}=3+3^{2}+3^{2} b_{n-2}=\\cdots=3+3^{2}+\\cdots+3^{n}+3^{n}=$ $=\\frac{1}{2}\\left(5 \\cdot 3^{n}-3\\right)$. It is seen from above that if $n \\leq b_{i}$, then $f(n) \\leq f\\left(b_{i}-1\\right)=b_{i}+1$. Hence if $f(n)=1993$, then $n \\geq b_{i} \\geq 1992$ for some $i$. The smallest such $b_{i}$ is $b_{7}=5466$, and $f\\left(b_{i}+2 k-1\\right)=b_{i}+3-k=1993$ implies $k=3476$. Thus the least integer in $S$ is $n_{1}=5466+2 \\cdot 3476-1=12417$. All the elements of $S$ are given by $n_{i}=b_{i+6}+2 k-1$, where $b_{i+6}+3-k=$ 1993, i.e., $k=b_{i+6}-1990$. Therefore $n_{i}=3 b_{i+6}-3981=\\frac{1}{2}\\left(5 \\cdot 3^{i+7}-7971\\right)$. Clearly $S$ is infinite and $\\lim _{i \\rightarrow \\infty} \\frac{n_{i+1}}{n_{i}}=3$.","problem_type":null,"tier":0} +{"year":"1993","problem_phase":"shortlisted","problem":"9. (IND 4) (a) Show that the set $\\mathbb{Q}^{+}$of all positive rational numbers can be partitioned into three disjoint subsets $A, B, C$ satisfying the following conditions: $$ B A=B, \\quad B^{2}=C, \\quad B C=A, $$ where $H K$ stands for the set $\\{h k \\mid h \\in H, k \\in K\\}$ for any two subsets $H, K$ of $\\mathbb{Q}^{+}$and $H^{2}$ stands for $H H$. (b) Show that all positive rational cubes are in $A$ for such a partition of $\\mathbb{Q}^{+}$. (c) Find such a partition $\\mathbb{Q}^{+}=A \\cup B \\cup C$ with the property that for no positive integer $n \\leq 34$ are both $n$ and $n+1$ in $A$; that is, $$ \\min \\{n \\in \\mathbb{N} \\mid n \\in A, n+1 \\in A\\}>34 $$","solution":"9. We shall first complete the \"multiplication table\" for the sets $A, B, C$. It is clear that this multiplication is commutative and associative, so that we have the following relations: $$ \\begin{aligned} & A C=(A B) B=B B=C \\\\ & A^{2}=A A=(A B) C=B C=A \\\\ & C^{2}=C C=B(B C)=B A=B \\end{aligned} $$ (a) Now put 1 in $A$ and distribute the primes arbitrarily in $A, B, C$. This distribution uniquely determines the partition of $\\mathbb{Q}^{+}$with the stated property. Indeed, if an arbitrary rational number $$ x=p_{1}^{\\alpha_{1}} \\cdots p_{k}^{\\alpha_{k}} q_{1}^{\\beta_{1}} \\cdots q_{l}^{\\beta_{l}} r_{1}^{\\gamma_{1}} \\cdots r_{m}^{\\gamma_{m}} $$ is given, where $p_{i} \\in A, q_{i} \\in B, r_{i} \\in C$ are primes, it is easy to see that $x$ belongs to $A, B$, or $C$ according as $\\beta_{1}+\\cdots+\\beta_{l}+2 \\gamma_{1}+\\cdots+2 \\gamma_{m}$ is congruent to 0,1 , or $2(\\bmod 3)$. (b) In every such partition, cubes all belong to $A$. In fact, $A^{3}=A^{2} A=$ $A A=A, B^{3}=B^{2} B=C B=A, C^{3}=C^{2} C=B C=A$. (c) By (b) we have $1,8,27 \\in A$. Then $2 \\notin A$, and since the problem is symmetric with respect to $B, C$, we can assume $2 \\in B$ and consequently $4 \\in C$. Also $7 \\notin A$, and also $7 \\notin B$ (otherwise, $28=4 \\cdot 7 \\in A$ and $27 \\in A$ ), so $7 \\in C, 14 \\in A, 28 \\in B$. Further, we see that $3 \\notin A$ (since otherwise $9 \\in A$ and $8 \\in A$ ). Put 3 in $C$. Then $5 \\notin B$ (otherwise $15 \\in A$ and $14 \\in A$ ), so let $5 \\in C$ too. Consequently $6,10 \\in A$. Also $13 \\notin A$, and $13 \\notin C$ because $26 \\notin A$, so $13 \\in B$. Now it is easy to distribute the remaining primes $11,17,19,23,29,31$ : one possibility is $$ \\begin{aligned} & A=\\{1,6,8,10,14,19,23,27,29,31,33, \\ldots\\}, \\\\ & C=\\{3,4,5,7,18,22,24,26,30,32,34, \\ldots\\} \\\\ & B=\\{2,9,11,12,13,15,16,17,20,21,25,28,35, \\ldots\\} \\end{aligned} $$ Remark. It can be proved that $\\min \\{n \\in \\mathbb{N} \\mid n \\in A, n+1 \\in A\\} \\leq 77$.","problem_type":null,"tier":0} +{"year":"1994","problem_phase":"shortlisted","problem":"1. A1 (USA) Let $a_{0}=1994$ and $a_{n+1}=\\frac{a_{n}^{2}}{a_{n}+1}$ for each nonnegative integer $n$. Prove that $1994-n$ is the greatest integer less than or equal to $a_{n}$, $0 \\leq n \\leq 998$.","solution":"1. Obviously $a_{0}>a_{1}>a_{2}>\\cdots$. Since $a_{k}-a_{k+1}=1-\\frac{1}{a_{k}+1}$, we have $a_{n}=a_{0}+\\left(a_{1}-a_{0}\\right)+\\cdots+\\left(a_{n}-a_{n-1}\\right)=1994-n+\\frac{1}{a_{0}+1}+\\cdots+\\frac{1}{a_{n-1}+1}>$ $1994-n$. Also, for $1 \\leq n \\leq 998$, $$ \\frac{1}{a_{0}+1}+\\cdots+\\frac{1}{a_{n-1}+1}<\\frac{n}{a_{n-1}+1}<\\frac{998}{a_{997}+1}<1 $$ because as above, $a_{997}>997$. Hence $\\left\\lfloor a_{n}\\right\\rfloor=1994-n$.","problem_type":"Algebra","tier":0} +{"year":"1994","problem_phase":"shortlisted","problem":"10. C5 (SWE) At a round table are 1994 girls, playing a game with a deck of $n$ cards. Initially, one girl holds all the cards. In each turn, if at least one girl holds at least two cards, one of these girls must pass a card to each of her two neighbors. The game ends when and only when each girl is holding at most one card. (a) Prove that if $n \\geq 1994$, then the game cannot end. (b) Prove that if $n<1994$, then the game must end.","solution":"10. (a) The case $n>1994$ is trivial. Suppose that $n=1994$. Label the girls $G_{1}$ to $G_{1994}$, and let $G_{1}$ initially hold all the cards. At any moment give to each card the value $i, i=1, \\ldots, 1994$, if $G_{i}$ holds it. Define the characteristic $C$ of a position as the sum of all these values. Initially $C=1994$. In each move, if $G_{i}$ passes cards to $G_{i-1}$ and $G_{i+1}$ (where $G_{0}=G_{1994}$ and $G_{1995}=G_{1}$ ), $C$ changes for $\\pm 1994$ or does not change, so that it remains divisible by 1994. But if the game ends, the characteristic of the final position will be $C=1+2+\\cdots+1994=$ $997 \\cdot 1995$, which is not divisible by 1994. (b) Whenever a card is passed from one girl to another for the first time, let the girls sign their names on it. Thereafter, if one of them passes a card to her neighbor, we shall assume that the passed card is exactly the one signed by both of them. Thus each signed card is stuck between two neighboring girls, so if $n<1994$, there are two neighbors who never exchange cards. Consequently, there is a girl $G$ who played only a finite number of times. If her neighbor plays infinitely often, then after her last move, $G$ will continue to accumulate cards indefinitely, which is impossible. Hence every girl plays finitely many times.","problem_type":"Combinatorics","tier":0} +{"year":"1994","problem_phase":"shortlisted","problem":"11. C6 (FIN) On an infinite square grid, two players alternately mark symbols on empty cells. The first player always marks $X$ 's, the second $O$ 's. One symbol is marked per turn. The first player wins if there are 11 consecutive $X$ 's in a row, column, or diagonal. Prove that the second player can prevent the first from winning.","solution":"11. Tile the table with dominoes and numbers as shown in the picture. The second player will not lose if whenever the first player plays in a cell of a domino, he plays in the other cell of the domino, and whenever the first player plays on a number, he plays on the same number that is diagonally adjacent. ![](https:\/\/cdn.mathpix.com\/cropped\/2024_11_18_8e985d6b9c83aa3e9d0eg-593.jpg?height=385&width=399&top_left_y=1025&top_left_x=851)","problem_type":"Combinatorics","tier":0} +{"year":"1994","problem_phase":"shortlisted","problem":"12. C7 (BRA) Prove that for any integer $n \\geq 2$, there exists a set of $2^{n-1}$ points in the plane such that no 3 lie on a line and no $2 n$ are the vertices of a convex $2 n$-gon.","solution":"12. Define $S_{n}$ recursively as follows: Let $S_{2}=\\{(0,0),(1,1)\\}$ and $S_{n+1}=$ $S_{n} \\cup T_{n}$, where $T_{n}=\\left\\{\\left(x+2^{n-1}, y+M_{n}\\right) \\mid(x, y) \\in S_{n}\\right\\}$, with $M_{n}$ chosen large enough so that the entire set $T_{n}$ lies above every line passing through two points of $S_{n}$. By definition, $S_{n}$ has exactly $2^{n-1}$ points and contains no three collinear points. We claim that no $2 n$ points of this set are the vertices of a convex $2 n$-gon. Consider an arbitrary convex polygon $\\mathcal{P}$ with vertices in $S_{n}$. Join by a diagonal $d$ the two vertices of $\\mathcal{P}$ having the smallest and greatest $x$ coordinates. This diagonal divides $\\mathcal{P}$ into two convex polygons $\\mathcal{P}_{1}, \\mathcal{P}_{2}$, the former lying above $d$. We shall show by induction that both $\\mathcal{P}_{1}, \\mathcal{P}_{2}$ have at most $n$ vertices. Assume to the contrary that $\\mathcal{P}_{1}$ has at least $n+1$ vertices $A_{1}\\left(x_{1}, y_{1}\\right), \\ldots, A_{n+1}\\left(x_{n+1}, y_{n+1}\\right)$ in $S_{n}$, with $x_{1}<\\cdots\\cdots>\\frac{y_{n+1}-y_{n}}{x_{n+1}-x_{n}}$. By the induction hypothesis, not more than $n-1$ of these vertices belong to $S_{n-1}$ or $T_{n-1}$, so let $A_{k-1}, A_{k} \\in S_{n-1}$, $A_{k+1} \\in T_{n-1}$. But by the construction of $T_{n-1}, \\frac{y_{k+1}-y_{k}}{x_{k+1}-x_{k}}>\\frac{y_{k}-y_{k-1}}{x_{k}-x_{k-1}}$, which gives a contradiction. Similarly, $\\mathcal{P}_{2}$ has no more than $n$ vertices, and therefore $\\mathcal{P}$ itself has at most $2 n-2$ vertices.","problem_type":"Combinatorics","tier":0} +{"year":"1994","problem_phase":"shortlisted","problem":"13. G1 (FRA) A semicircle $\\Gamma$ is drawn on one side of a straight line $l$. $C$ and $D$ are points on $\\Gamma$. The tangents to $\\Gamma$ at $C$ and $D$ meet $l$ at $B$ and $A$ respectively, with the center of the semicircle between them. Let $E$ be the point of intersection of $A C$ and $B D$, and $F$ the point on $l$ such that $E F$ is perpendicular to $l$. Prove that $E F$ bisects $\\angle C F D$.","solution":"13. Extend $A D$ and $B C$ to meet at $P$, and let $Q$ be the foot of the perpendicular from $P$ to $A B$. Denote by $O$ the center of $\\Gamma$. Since $\\triangle P A Q \\sim \\triangle O A D$ and $\\triangle P B Q \\sim \\triangle O B C$, we obtain $\\frac{A Q}{A D}=\\frac{P Q}{O D}=\\frac{P Q}{O C}=\\frac{B Q}{B C}$. Therefore $\\frac{A Q}{Q B} \\cdot \\frac{B C}{C P} \\cdot \\frac{P D}{D A}=1$, so by the converse Ceva theorem, $A C, B D$, and $P Q$ are concurrent. It follows that $Q \\equiv F$. Finally, since the points $O, C, P, D, F$ are concyclic, we have $\\angle D F P=\\angle D O P=\\angle P O C=\\angle P F C$.","problem_type":"Geometry","tier":0} +{"year":"1994","problem_phase":"shortlisted","problem":"14. G2 (UKR) $A B C D$ is a quadrilateral with $B C$ parallel to $A D . M$ is the midpoint of $C D, P$ that of $M A$ and $Q$ that of $M B$. The lines $D P$ and $C Q$ meet at $N$. Prove that $N$ is not outside triangle $A B M .{ }^{8}$","solution":"14. Although it does not seem to have been noticed at the jury, the statement of the problem is false. For $A(0,0), B(0,4), C(1,4), D(7,0)$, we have $M(4,2), P(2,1), Q(2,3)$ and $N(9 \/ 2,1 \/ 2) \\notin \\triangle A B M$. The official solution, if it can be called so, actually shows that $N$ lies inside $A B C D$ and goes as follows: The case $A D=B C$ is trivial, so let $A D>B C$. Let $L$ be the midpoint of $A B$. Complete the parallelograms $A D M X$ and $B C M Y$. Now $N=D X \\cap C Y$, so let $C Y$ and $D X$ intersect $A B$ at $K$ and $H$ respectively. From $L X=L Y$ and $$ \\frac{H L}{L X}=\\frac{H A}{A D}<\\frac{L A}{A D}<\\frac{K B}{A D}<\\frac{K B}{B C}=\\frac{K L}{L Y} $$ we get $H Ln \\geq 2$. Since $m^{3}+1 \\equiv 1$ and $m n-1 \\equiv-1(\\bmod n)$, we deduce $\\frac{n^{3}+1}{m n-1}=k n-1$ for some integer $k>0$. On the other hand, $k n-1<\\frac{n^{3}+1}{n^{2}-1}=n+\\frac{1}{n-1} \\leq 2 n-1$ gives that $k=1$, and therefore $n^{3}+1=(m n-1)(n-1)$. This yields $m=\\frac{n^{2}+1}{n-1}=n+1+\\frac{2}{n-1} \\in \\mathbb{N}$, so $n \\in\\{2,3\\}$ and $m=5$. The solutions with $ma_{2}>\\cdots>a_{m}$. We claim that for $i=1, \\ldots, m$, $a_{i}+a_{m+1-i} \\geq n+1$. Indeed, otherwise $a_{i}+a_{m+1-i}, \\ldots, a_{i}+a_{m-1}, a_{i}+a_{m}$ are $i$ different elements of $A$ greater than $a_{i}$, which is impossible. Now by adding for $i=1, \\ldots, m$ we obtain $2\\left(a_{1}+\\cdots+a_{m}\\right) \\geq m(n+1)$, and the result follows.","problem_type":"Algebra","tier":0} +{"year":"1994","problem_phase":"shortlisted","problem":"20. N3 (FIN) ${ }^{\\text {IMO6 }}$ Find a set $A$ of positive integers such that for any infinite set $P$ of prime numbers, there exist positive integers $m \\in A$ and $n \\notin A$, both the product of the same number of distinct elements of $P$.","solution":"20. Let $A$ be the set of all numbers of the form $p_{1} p_{2} \\ldots p_{p_{1}}$, where $p_{1}z_{n}$ when $x_{n-1}$ is even and $2 y_{n}>z_{n}>y_{n}$ when $x_{n-1}$ is odd. In fact, $n=1$ is the trivial case, while if it holds for $n \\geq 1$, then $y_{n+1}=2 y_{n}>z_{n}=z_{n+1}$ if $x_{n}$ is even, and $2 y_{n+1}=2 y_{n}>y_{n}+z_{n}=z_{n+1}$ if $x_{n}$ is odd (since then $x_{n-1}$ is even). If $x_{1}=0$, then $x_{0}=3$ is good. Suppose $x_{n}=0$ for some $n \\geq 2$. Then $x_{n-1}$ is odd and $x_{n-2}$ is even, so that $y_{n-1}>z_{n-1}$. We claim that a pair $\\left(y_{n-1}, z_{n-1}\\right)$, where $2^{k}=y_{n-1}>z_{n-1}>0$ and $z_{n-1} \\equiv 1$ $(\\bmod 4)$, uniquely determines $x_{0}=f\\left(y_{n-1}, z_{n-1}\\right)$. We see that $x_{n-1}=$ $\\frac{1}{2} y_{n-1}+z_{n-1}$, and define $\\left(x_{k}, y_{k}, z_{k}\\right)$ backwards as follows, until we get $\\left(y_{k}, z_{k}\\right)=(4,1)$. If $y_{k}>z_{k}$, then $x_{k-1}$ must have been even, so we define $\\left(x_{k-1}, y_{k-1}, z_{k-1}\\right)=\\left(2 x_{k}, y_{k} \/ 2, z_{k}\\right)$; otherwise $x_{k-1}$ must have been odd, so we put $\\left(x_{k-1}, y_{k-1}, z_{k-1}\\right)=\\left(x_{k}-y_{k} \/ 2+z_{k}, y_{k}, z_{k}-y_{k}\\right)$. We eventually arrive at $\\left(y_{0}, z_{0}\\right)=(4,1)$ and a good integer $x_{0}=f\\left(y_{n-1}, z_{n-1}\\right)$, as claimed. Thus for example $\\left(y_{n-1}, z_{n-1}\\right)=(64,61)$ implies $x_{n-1}=93$, $\\left(x_{n-2}, y_{n-2}, z_{n-2}\\right)=(186,32,61)$ etc., and $x_{0}=1953$, while in the case of $\\left(y_{n-1}, z_{n-1}\\right)=(128,1)$ we get $x_{0}=2080$. Note that $y^{\\prime}>y \\Rightarrow f\\left(y^{\\prime}, z^{\\prime}\\right)>f(y, z)$ and $z^{\\prime}>z \\Rightarrow f\\left(y, z^{\\prime}\\right)>f(y, z)$. Therefore there are no $y, z$ for which $19531$, there exists $j>i$ such that $x_{i}^{i}$ divides $x_{j}^{j}$. (b) Is it true that $x_{1}$ must divide $x_{j}^{j}$ for some $j>1$ ?","solution":"23. (a) Let $p$ be a prime divisor of $x_{i}, i>1$, and let $x_{j} \\equiv u_{j}(\\bmod p)$ where $0 \\leq u_{j} \\leq p-1$ (particularly $u_{i} \\equiv 0$ ). Then $u_{j+1} \\equiv u_{j} u_{j-1}+$ $1(\\bmod p)$. The number of possible pairs $\\left(u_{j}, u_{j+1}\\right)$ is finite, so $u_{j}$ is eventually periodic. We claim that for some $d_{p}>0, u_{i+d_{p}}=0$. Indeed, suppose the contrary and let $\\left(u_{m}, u_{m+1}, \\ldots, u_{m+d-1}\\right)$ be the first period for $m \\geq i$. Then $m \\neq i$. By the assumption $u_{m-1} \\not \\equiv$ $u_{m+d-1}$, but $u_{m-1} u_{m} \\equiv u_{m+1}-1 \\equiv u_{m+d+1}-1 \\equiv u_{m+d-1} u_{m+d} \\equiv$ $u_{m+d-1} u_{m}(\\bmod p)$, which is impossible if $p \\nmid u_{m}$. Hence there is a $d_{p}$ with $u_{i}=u_{i+d_{p}}=0$ and moreover $u_{i+1}=u_{i+d_{p}+1}=1$, so the sequence $u_{j}$ is periodic with period $d_{p}$ starting from $u_{i}$. Let $m$ be the least common multiple of all $d_{p}$ 's, where $p$ goes through all prime divisors of $x_{i}$. Then the same primes divide every $x_{i+k m}, k=1,2, \\ldots$, so for large enough $k$ and $j=i+k m, x_{i}^{i} \\mid x_{j}^{j}$. (b) If $i=1$, we cannot deduce that $x_{i+1} \\equiv 1(\\bmod p)$. The following example shows that the statement from (a) need not be true in this case. Take $x_{1}=22$ and $x_{2}=9$. Then $x_{n}$ is even if and only if $n \\equiv 1(\\bmod$ 3 ), but modulo 11 the sequence $\\left\\{x_{n}\\right\\}$ is $0,9,1,10,0,1,1,2,3,7,0, \\ldots$, so $11 \\mid x_{n}(n>1)$ if and only if $n \\equiv 5(\\bmod 6)$. Thus for no $n>1$ can we have $22 \\mid x_{n}$.","problem_type":"Number Theory","tier":0} +{"year":"1994","problem_phase":"shortlisted","problem":"24. N7 (GBR) A wobbly number is a positive integer whose digits in base 10 are alternately nonzero and zero, the units digit being nonzero. Determine all positive integers that do not divide any wobbly number.","solution":"24. A multiple of 10 does not divide any wobbly number. Also, if $25 \\mid n$, then every multiple of $n$ ends with $25,50,75$, or 00 ; hence it is not wobbly. We now show that every other number $n$ divides some wobbly number. (i) Let $n$ be odd and not divisible by 5 . For any $k \\geq 1$ there exists $l$ such that $\\left(10^{k}-1\\right) n$ divides $10^{l}-1$, and thus also divides $10^{k l}-1$. Consequently, $v_{k}=\\frac{10^{k l}-1}{10^{k}-1}$ is divisible by $n$, and it is wobbly when $k=2$ (indeed, $v_{2}=101 \\ldots 01$ ). If $n$ is divisible by 5 , one can simply take $5 v_{2}$ instead. (ii) Let $n$ be a power of 2 . We prove by induction on $m$ that $2^{2 m+1}$ has a wobbly multiple $w_{m}$ with exactly $m$ nonzero digits. For $m=1$, take $w_{1}=8$. Suppose that for some $m \\geq 1$ there is a wobbly $w_{m}=$ $2^{2 m+1} d_{m}$. Then the numbers $a \\cdot 10^{2 m}+w_{m}$ are wobbly and divisible by $2^{2 m+1}$ when $a \\in\\{2,4,6,8\\}$. Moreover, one of these numbers is divisible by $2^{2 m+3}$. Indeed, it suffices to choose $a$ such that $\\frac{a}{2}+d_{m}$ is divisible by 4 . This proves the induction step. (iii) Let $n=2^{m} r$, where $m \\geq 1$ and $r$ is odd, $5 \\nmid r$. Then $v_{2 m} w_{m}$ is wobbly and divisible by both $2^{m}$ and $r$ (using notation from (i), $r \\mid v_{2 m}$ ).","problem_type":"Number Theory","tier":0} +{"year":"1994","problem_phase":"shortlisted","problem":"3. A3 (GBR) ${ }^{\\mathrm{IMO} 5}$ Let $S$ be the set of real numbers greater than -1 . Find all functions $f: S \\rightarrow S$ such that $$ f(x+f(y)+x f(y))=y+f(x)+y f(x) \\quad \\text { for all } x \\text { and } y \\text { in } S, $$ and $f(x) \/ x$ is strictly increasing for $-10$. Let Peter make transfers of money into the first account as follows. Write $b=a q+r$ with $0 \\leq rk$. Remove this card, slide all cards from the $(k+1)$ st to the $l$ th position one place to the right, and replace the card $l$ in the $l$ th position. (a) Prove that the game lasts at most $2^{n}-1$ moves. (b) Prove that there exists a unique initial configuration for which the game lasts exactly $2^{n}-1$ moves.","solution":"9. (a) For $i=1, \\ldots, n$, let $d_{i}$ be 0 if the card $i$ is in the $i$ th position, and 1 otherwise. Define $b=d_{1}+2 d_{2}+2^{2} d_{3}+\\cdots+2^{n-1} d_{n}$, so that $0 \\leq b \\leq$ $2^{n}-1$, and $b=0$ if and only if the game is over. After each move some digit $d_{l}$ changes from 1 to 0 while $d_{l+1}, d_{l+2}, \\ldots$ remain unchanged. Hence $b$ decreases after each move, and consequently the game ends after at most $2^{n}-1$ moves. (b) Suppose the game lasts exactly $2^{n}-1$ moves. Then each move decreases $b$ for exactly one, so playing the game in reverse (starting from the final configuration), every move is uniquely determined. It follows that if the configuration that allows a game lasting $2^{n}-1$ moves exists, it must be unique. Consider the initial configuration $0, n, n-1, \\ldots, 2,1$. We prove by induction that the game will last exactly $2^{n}-1$ moves, and that the card 0 will get to the 0 th position only in the last move. This is trivial for $n=1$, so suppose that the claim is true for some $n=m-1 \\geq 1$ and consider the case $n=m$. Obviously the card 0 does not move until the card $m$ gets to the 0 -th position. But if we ignore the card 0 and consider the card $m$ to be the card 0 , the induction hypothesis gives that the card $m$ will move to the 0 th position only after $2^{m-1}-1$ moves. After these $2^{m-1}-1$ moves, we come to the configuration $0, m-1, \\ldots, 2,1, m$. The next move yields $m, 0, m-1, \\ldots, 2,1$, so by the induction hypothesis again we need $2^{m-1}-1$ moves more to finish the game.","problem_type":"Combinatorics","tier":0} +{"year":"1995","problem_phase":"shortlisted","problem":"1. A1 (RUS) ${ }^{\\mathrm{IMO} 2}$ Let $a, b$, and $c$ be positive real numbers such that $a b c=1$. Prove that $$ \\frac{1}{a^{3}(b+c)}+\\frac{1}{b^{3}(a+c)}+\\frac{1}{c^{3}(a+b)} \\geq \\frac{3}{2} $$","solution":"1. Let $x=\\frac{1}{a}, y=\\frac{1}{b}, z=\\frac{1}{c}$. Then $x y z=1$ and $$ S=\\frac{1}{a^{3}(b+c)}+\\frac{1}{b^{3}(c+a)}+\\frac{1}{c^{3}(a+b)}=\\frac{x^{2}}{y+z}+\\frac{y^{2}}{z+x}+\\frac{z^{2}}{x+y} $$ We must prove that $S \\geq \\frac{3}{2}$. From the Cauchy-Schwarz inequality, $$ [(y+z)+(z+x)+(x+y)] \\cdot S \\geq(x+y+z)^{2} \\Rightarrow S \\geq \\frac{x+y+z}{2} $$ It follows from the A-G mean inequality that $\\frac{x+y+z}{2} \\geq \\frac{3}{2} \\sqrt[3]{x y z}=\\frac{3}{2}$; hence the proof is complete. Equality holds if and only if $x=y=z=1$, i.e., $a=b=c=1$. Remark. After reducing the problem to $\\frac{x^{2}}{y+z}+\\frac{y^{2}}{z+x}+\\frac{z^{2}}{x+y} \\geq \\frac{3}{2}$, we can solve the problem using Jensen's inequality applied to the function $g(u, v)=$ $u^{2} \/ v$. The problem can also be solved using Muirhead's inequality.","problem_type":"Algebra","tier":0} +{"year":"1995","problem_phase":"shortlisted","problem":"10. G4 (UKR) An acute triangle $A B C$ is given. Points $A_{1}$ and $A_{2}$ are taken on the side $B C$ (with $A_{2}$ between $A_{1}$ and $C$ ), $B_{1}$ and $B_{2}$ on the side $A C$ (with $B_{2}$ between $B_{1}$ and $A$ ), and $C_{1}$ and $C_{2}$ on the side $A B$ (with $C_{2}$ between $C_{1}$ and $B$ ) such that $$ \\angle A A_{1} A_{2}=\\angle A A_{2} A_{1}=\\angle B B_{1} B_{2}=\\angle B B_{2} B_{1}=\\angle C C_{1} C_{2}=\\angle C C_{2} C_{1} . $$ The lines $A A_{1}, B B_{1}$, and $C C_{1}$ form a triangle, and the lines $A A_{2}, B B_{2}$, and $C C_{2}$ form a second triangle. Prove that all six vertices of these two triangles lie on a single circle.","solution":"10. Let the two triangles be $X_{1} Y_{1} Z_{1}, X_{2} Y_{2} Z_{2}$, with $X_{1}=B B_{1} \\cap C C_{1}, Y_{1}=$ $C C_{1} \\cap A A_{1}, Z_{1}=A A_{1} \\cap B B_{1}$, $X_{2}=B B_{2} \\cap C C_{2}, Y_{2}=C C_{2} \\cap$ $A A_{2}, Z_{2}=A A_{2} \\cap B B_{2}$. First, we observe that $\\angle A B B_{2}=\\angle A C C_{1}$ and $\\angle A B B_{1}=\\angle A C C_{2}$. Consequently $\\angle B Z_{1} A_{1}=\\angle B A A_{1}+$ $\\angle A B B_{1}=\\angle B C C_{2}+\\angle C_{2} C A=$ $\\angle C$ and similarly $\\angle A Z_{2} B_{2}=\\angle C$, $\\angle A Y_{1} C_{1}=\\angle C Y_{2} A_{2}=\\angle B$. Also, $\\triangle A B B_{2} \\sim \\triangle A C C_{1}$; hence ![](https:\/\/cdn.mathpix.com\/cropped\/2024_11_18_8e985d6b9c83aa3e9d0eg-601.jpg?height=399&width=475&top_left_y=1280&top_left_x=849) $A C_{1} \/ A C=A B_{2} \/ A B$. From the sine formula, we obtain $$ \\begin{aligned} \\frac{A Z_{1}}{\\sin \\angle A B Z_{1}} & =\\frac{A B}{\\sin \\angle A Z_{1} B}=\\frac{A B}{\\sin \\angle C}=\\frac{A C}{\\sin \\angle B}=\\frac{A C}{\\sin \\angle A Y_{2} C} \\\\ & =\\frac{A Y_{2}}{\\sin \\angle A C Y_{2}} \\Longrightarrow A Z_{1}=A Y_{2} . \\end{aligned} $$ Analogously, $B X_{1}=B Z_{2}$ and $C Y_{1}=C X_{2}$. Furthermore, again from the sine formula, $$ \\begin{aligned} \\frac{A Y_{1}}{\\sin \\angle A C_{1} Y_{1}} & =\\frac{A C_{1}}{\\sin \\angle A Y_{1} C_{1}}=\\frac{A C_{1}}{A C} \\frac{A C}{\\sin \\angle B} \\\\ & =\\frac{A B_{2}}{A B} \\frac{A B}{\\sin \\angle C}=\\frac{A B_{2}}{\\sin \\angle A Z_{2} B_{2}}=\\frac{A Z_{2}}{\\sin \\angle A B_{2} Z_{2}} . \\end{aligned} $$ Hence, $A Y_{1}=A Z_{2}$ and, analogously, $B Z_{1}=B X_{2}$ and $C X_{1}=C Y_{2}$. We deduce that $Y_{1} Z_{2} \\| B C$ and $Z_{2} X_{1} \\| A C$, which gives us $\\angle Y_{1} Z_{2} X_{1}=$ $180^{\\circ}-\\angle C=180^{\\circ}-\\angle Y_{1} Z_{1} X_{1}$. It follows that $Z_{2}$ lies on the circle circumscribed about $\\triangle X_{1} Y_{1} Z_{1}$. Similarly, so do $X_{2}$ and $Y_{2}$. Second solution. Let $H$ be the orthocenter of $\\triangle A B C$. Triangles $A H B$, $B H C, C H A, A B C$ have the same circumradius $R$. Additionally, $$ \\angle H A A_{i}=\\angle H B B_{i}=\\angle H C C_{i}=\\theta(i=1,2) . $$ Since $\\angle H B X_{1}=\\angle H C X_{1}=\\theta, B C X_{1} H$ is concyclic and therefore $H X_{1}=$ $2 R \\sin \\theta$. The same holds for $H Y_{1}, H Z_{1}, H X_{2}, H Y_{2}, H Z_{2}$. Hence $X_{i}, Y_{i}, Z_{i}$ $(i=1,2)$ lie on a circle centered at $H$.","problem_type":"Geometry","tier":0} +{"year":"1995","problem_phase":"shortlisted","problem":"11. G5 (NZL) ${ }^{\\mathrm{IMO} 5}$ Let $A B C D E F$ be a convex hexagon with $A B=B C=$ $C D, D E=E F=F A$, and $\\measuredangle B C D=\\measuredangle E F A=\\pi \/ 3$ (that is, $60^{\\circ}$ ). Let $G$ and $H$ be two points interior to the hexagon such that angles $A G B$ and $D H E$ are both $2 \\pi \/ 3$ (that is, $120^{\\circ}$ ). Prove that $A G+G B+G H+D H+$ $H E \\geq C F$.","solution":"11. Triangles $B C D$ and $E F A$ are equilateral, and hence $B E$ is an axis of symmetry of $A B D E$. Let $C^{\\prime}, F^{\\prime}$ respectively be the points symmetric to $C, F$ with respect to $B E$. The points $G$ and $H$ lie on the circumcircles of $A B C^{\\prime}$ and $D E F^{\\prime}$ respectively (because, for instance, $\\angle A G B=120^{\\circ}=$ $\\left.180^{\\circ}-\\angle A C^{\\prime} B\\right)$; hence from Ptolemy's theorem we have $A G+G B=C^{\\prime} G$ and $D H+H E=H F^{\\prime}$. Therefore $$ A G+G B+G H+D H+H E=C^{\\prime} G+G H+H F^{\\prime} \\geq C^{\\prime} F^{\\prime}=C F $$ with equality if and only if $G$ and $H$ both lie on $C^{\\prime} F^{\\prime}$. Remark. Since by Ptolemy's inequality $A G+G B \\geq C^{\\prime} G$ and $D H+H E \\geq$ $H F^{\\prime}$, the result holds without the condition $\\angle A G B=\\angle D H E=120^{\\circ}$.","problem_type":"Geometry","tier":0} +{"year":"1995","problem_phase":"shortlisted","problem":"12. G6 (USA) Let $A_{1} A_{2} A_{3} A_{4}$ be a tetrahedron, $G$ its centroid, and $A_{1}^{\\prime}, A_{2}^{\\prime}, A_{3}^{\\prime}$, and $A_{4}^{\\prime}$ the points where the circumsphere of $A_{1} A_{2} A_{3} A_{4}$ intersects $G A_{1}, G A_{2}, G A_{3}$, and $G A_{4}$, respectively. Prove that $$ G A_{1} \\cdot G A_{2} \\cdot G A_{3} \\cdot G A_{4} \\leq G A_{1}^{\\prime} \\cdot G A_{2}^{\\prime} \\cdot G A_{3}^{\\prime} \\cdot G A_{4}^{\\prime} $$ and $$ \\frac{1}{G A_{1}^{\\prime}}+\\frac{1}{G A_{2}^{\\prime}}+\\frac{1}{G A_{3}^{\\prime}}+\\frac{1}{G A_{4}^{\\prime}} \\leq \\frac{1}{G A_{1}}+\\frac{1}{G A_{2}}+\\frac{1}{G A_{3}}+\\frac{1}{G A_{4}} $$","solution":"12. Let $O$ be the circumcenter and $R$ the circumradius of $\\underset{\\longrightarrow}{A_{1} A_{2} A_{3} A_{4}}$. We have $O A_{i}^{2}=\\left(\\overrightarrow{O G}+\\left(\\overrightarrow{O A_{i}}-\\overrightarrow{O G}\\right)\\right)^{2}=O G^{2}+G A_{i}^{2}+2 \\overrightarrow{O G} \\cdot \\overrightarrow{G A_{i}}$. Summing up these equalities for $i=1,2,3,4$ and using that $\\sum_{i=1}^{4} \\overrightarrow{G A_{i}}=\\overrightarrow{0}$, we obtain $$ \\sum_{i=1}^{4} O A_{i}^{2}=4 O G^{2}+\\sum_{i=1}^{4} G A_{i}^{2} \\Longleftrightarrow \\sum_{i=1}^{4} G A_{i}^{2}=4\\left(R^{2}-O G^{2}\\right) $$ Now we have that the potential of $G$ with respect to the sphere equals $G A_{i} \\cdot G A_{i}^{\\prime}=R^{2}-O G^{2}$. Plugging in these expressions for $G A_{i}^{\\prime}$, we reduce the inequalities we must prove to $$ \\begin{aligned} G A_{1} \\cdot G A_{2} \\cdot G A_{3} \\cdot G A_{4} & \\leq\\left(R^{2}-O G^{2}\\right)^{2} \\\\ \\text { and } \\quad\\left(R^{2}-O G^{2}\\right) \\sum_{i=1}^{4} \\frac{1}{G A_{i}} & \\geq \\sum_{i=1}^{4} G A_{i} \\end{aligned} $$ Inequality (2) immediately follows from (1) and the quadratic-geometric mean inequality for $G A_{i}$. Since from the Cauchy-Schwarz inequality we have $\\sum_{i=1}^{4} G A_{i}^{4} \\geq \\frac{1}{4}\\left(\\sum_{i=1}^{4} G A_{i}\\right)^{2}$ and $\\left(\\sum_{i=1}^{4} G A_{i}\\right)\\left(\\sum_{i=1}^{4} \\frac{1}{G A_{i}}\\right) \\geq 16$, inequality (3) follows from (1) and from $$ \\left(\\sum_{i=1}^{4} G A_{i}^{2}\\right)\\left(\\sum_{i=1}^{4} \\frac{1}{G A_{i}}\\right) \\geq \\frac{1}{4}\\left(\\sum_{i=1}^{4} G A_{i}\\right)^{2}\\left(\\sum_{i=1}^{4} \\frac{1}{G A_{i}}\\right) \\geq 4 \\sum_{i=1}^{4} G A_{i} $$","problem_type":"Geometry","tier":0} +{"year":"1995","problem_phase":"shortlisted","problem":"13. G7 (LAT) $O$ is a point inside a convex quadrilateral $A B C D$ of area $S . K, L, M$, and $N$ are interior points of the sides $A B, B C, C D$, and $D A$ respectively. If $O K B L$ and $O M D N$ are parallelograms, prove that $\\sqrt{S} \\geq \\sqrt{S_{1}}+\\sqrt{S_{2}}$, where $S_{1}$ and $S_{2}$ are the areas of $O N A K$ and $O L C M$ respectively.","solution":"13. If $O$ lies on $A C$, then $A B C D, A K O N$, and $O L C M$ are similar; hence $A C=A O+O C$ implies $\\sqrt{S}=\\sqrt{S_{1}}+\\sqrt{S_{2}}$. Assume that $O$ does not lie on $A C$ and that w.l.o.g. it lies inside triangle $A D C$. Let us denote by $T_{1}, T_{2}$ the areas of parallelograms $K B L O, N O M D$ respectively. Consider a line through $O$ that intersects $A D, D C, C B, B A$ respectively at $X, Y, Z, W$ so that $O W \/ O X=O Z \/ O Y$ (such a line exists by a continuity argument: the left side is smaller when $W=X=A$, but greater when $Y=Z=C$ ). The desired inequality is equivalent to $T_{1}+T_{2} \\geq 2 \\sqrt{S_{1} S_{2}}$. Since triangles $W K O, O L Z, W B Z$ are similar and $W O+O Z=W Z$, we have $\\sqrt{S_{W K O}}+\\sqrt{S_{O L Z}}=\\sqrt{S_{W B Z}}=$ $\\sqrt{S_{W K O}+S_{O L Z}+T_{1}}$, which implies $T_{1}=2 \\sqrt{S_{W K O} S_{O L Z}}$. Similarly, $T_{2}=2 \\sqrt{S_{X N O} S_{O M Y}}$. Since $O W \/ O Z=O X \/ O Y$, we have $S_{W K O} \/ S_{X N O}=S_{O L Z} \/ S_{O M Y}$. ![](https:\/\/cdn.mathpix.com\/cropped\/2024_11_18_8e985d6b9c83aa3e9d0eg-603.jpg?height=394&width=489&top_left_y=903&top_left_x=835) Therefore we obtain $$ \\begin{aligned} T_{1}+T_{2} & =2 \\sqrt{S_{W K O} S_{O L Z}}+2 \\sqrt{S_{X N O} S_{O M Y}} \\\\ & =2 \\sqrt{\\left(S_{W K O}+S_{X N O}\\right)\\left(S_{O L Z}+S_{O M Y}\\right)} \\geq 2 \\sqrt{S_{1} S_{2}} \\end{aligned} $$ Second solution. By an affine transformation of the plane one can transform any nondegenerate quadrilateral into a cyclic one, thereby preserving parallelness and ratios of areas. Thus we may assume w.l.o.g. that $A B C D$ is cyclic. By a well-known formula, the area of a cyclic quadrilateral with sides $a, b, c, d$ and semiperimeter $p$ is given by $$ S=\\sqrt{(p-a)(p-b)(p-c)(p-d)} $$ Let us set $A K=a_{1}, K B=b_{1}, B L=a_{2}, L C=b_{2}, C M=a_{3}, M D=b_{3}$, $D N=a_{4}, N A=b_{4}$. Then the sides of quadrilateral $A K O N$ are $a_{i}$, the sides of $C L O M$ are $b_{i}$, and the sides of $A B C D$ are $a_{i}+b_{i}(i=1,2,3,4)$. If $p$ and $q$ are the semiperimeters of $A K O N$ and $C L O M$, and $x_{i}=p-a_{i}$, $y_{i}=q-b_{i}$, then we have $S_{1}=\\sqrt{x_{1} x_{2} x_{3} x_{4}}, S_{2}=\\sqrt{y_{1} y_{2} y_{3} y_{4}}$, and $S=$ $\\sqrt{\\left(x_{1}+y_{1}\\right)\\left(x_{2}+y_{2}\\right)\\left(x_{3}+y_{3}\\right)\\left(x_{4}+y_{4}\\right)}$. Thus we need to show that $$ \\sqrt[4]{x_{1} x_{2} x_{3} x_{4}}+\\sqrt[4]{y_{1} y_{2} y_{3} y_{4}} \\leq \\sqrt[4]{\\left(x_{1}+y_{1}\\right)\\left(x_{2}+y_{2}\\right)\\left(x_{3}+y_{3}\\right)\\left(x_{4}+y_{4}\\right)} $$ By setting $y_{i}=t_{i} x_{i}$ we reduce this inequality to $1+\\sqrt[4]{t_{1} t_{2} t_{3} t_{4}} \\leq$ $\\sqrt[4]{\\left(1+t_{1}\\right)\\left(1+t_{2}\\right)\\left(1+t_{3}\\right)\\left(1+t_{4}\\right)}$. One way to prove the last inequality is to apply the simple inequality $$ 1+\\sqrt{u v} \\leq \\sqrt{(1+u)(1+v)} $$ to $\\sqrt{t_{1} t_{2}}, \\sqrt{t_{3} t_{4}}$ and then to $t_{1}, t_{2}$ and $t_{3}, t_{4}$.","problem_type":"Geometry","tier":0} +{"year":"1995","problem_phase":"shortlisted","problem":"14. G8 (COL) Let $A B C$ be a triangle. A circle passing through $B$ and $C$ intersects the sides $A B$ and $A C$ again at $C^{\\prime}$ and $B^{\\prime}$, respectively. Prove that $B B^{\\prime}, C C^{\\prime}$, and $H H^{\\prime}$ are concurrent, where $H$ and $H^{\\prime}$ are the orthocenters of triangles $A B C$ and $A B^{\\prime} C^{\\prime}$ respectively.","solution":"14. Let $B B^{\\prime}$ cut $C C^{\\prime}$ at $P$. Since $\\angle B^{\\prime} B C^{\\prime}=\\angle B^{\\prime} C C^{\\prime}$, it follows that $\\angle P B H=\\angle P C H$. Let $D$ and $E$ be points such that $B P C D$ and $H P C E$ are parallelograms (consequently, so is $B H E D$ ). Triangles $B A C$ and $C^{\\prime} A B^{\\prime}$ are similar, from which we deduce that $\\triangle B^{\\prime} H^{\\prime} C^{\\prime}$ and $\\triangle B H C$ are similar, as well as $\\triangle B^{\\prime} P C^{\\prime}$ and $\\triangle B D C$. Hence $B^{\\prime} P C^{\\prime} H^{\\prime}$ and $B D C H$ are similar, from which we obtain $\\angle H^{\\prime} P B^{\\prime}=\\angle H D B$. Now $\\angle C D E=\\angle P B H=\\angle P C H=$ $\\angle C H E$ implies that $H C E D$ is a cyclic quadrilateral. Therefore $\\angle B P H=\\angle D C E=\\angle D H E=$ $\\angle H D B=\\angle H^{\\prime} P B^{\\prime}$; hence $H H^{\\prime}$ also passes through $P$. ![](https:\/\/cdn.mathpix.com\/cropped\/2024_11_18_8e985d6b9c83aa3e9d0eg-604.jpg?height=415&width=356&top_left_y=775&top_left_x=906) Second solution. Observe that $\\triangle H B C \\sim \\triangle H^{\\prime} B^{\\prime} C^{\\prime}, \\angle P B H=\\angle P C H$ and $\\angle P B^{\\prime} H^{\\prime}=\\angle P C^{\\prime} H^{\\prime}$. By Ceva's theorem in trigonometric form applied to $\\triangle B P C$ and the point $H$, we have $\\frac{\\sin \\angle B P H}{\\sin \\angle H P C}=\\frac{\\sin \\angle H B P}{\\sin \\angle H B C} \\cdot \\frac{\\sin \\angle H C B}{\\sin \\angle H C P}=\\frac{\\sin \\angle H C B}{\\sin \\angle H B C}$. Similarly, Ceva's theorem for $\\triangle B^{\\prime} P C^{\\prime}$ and point $H^{\\prime}$ yields $\\frac{\\sin \\angle B^{\\prime} P H^{\\prime}}{\\sin \\angle H^{\\prime} P C^{\\prime}}=\\frac{\\sin \\angle H^{\\prime} C^{\\prime} B^{\\prime}}{\\sin \\angle H^{\\prime} B^{\\prime} C^{\\prime}}$. Thus it follows that $$ \\frac{\\sin \\angle B^{\\prime} P H^{\\prime}}{\\sin \\angle H^{\\prime} P C^{\\prime}}=\\frac{\\sin \\angle B P H}{\\sin \\angle H P C} $$ which finally implies that $\\angle B P H=\\angle B^{\\prime} P H^{\\prime}$.","problem_type":"Geometry","tier":0} +{"year":"1995","problem_phase":"shortlisted","problem":"15. N1 (ROM) Let $k$ be a positive integer. Prove that there are infinitely many perfect squares of the form $n 2^{k}-7$, where $n$ is a positive integer.","solution":"15. We show by induction on $k$ that there exists a positive integer $a_{k}$ for which $a_{k}^{2} \\equiv-7\\left(\\bmod 2^{k}\\right)$. The statement of the problem follows, since every $a_{k}+r 2^{k}(r=0,1, \\ldots)$ also satisfies this condition. Note that for $k=1,2,3$ one can take $a_{k}=1$. Now suppose that $a_{k}^{2} \\equiv-7$ $\\left(\\bmod 2^{k}\\right)$ for some $k>3$. Then either $a_{k}^{2} \\equiv-7\\left(\\bmod 2^{k+1}\\right)$ or $a_{k}^{2} \\equiv 2^{k}-7$ $\\left(\\bmod 2^{k+1}\\right)$. In the former case, take $a_{k+1}=a_{k}$. In the latter case, set $a_{k+1}=a_{k}+2^{k-1}$. Then $a_{k+1}^{2}=a_{k}^{2}+2^{k} a_{k}+2^{2 k-2} \\equiv a_{k}^{2}+2^{k} \\equiv-7(\\bmod$ $2^{k+1}$ ) because $a_{k}$ is odd.","problem_type":"Number Theory","tier":0} +{"year":"1995","problem_phase":"shortlisted","problem":"16. N2 (RUS) Let $\\mathbb{Z}$ denote the set of all integers. Prove that for any integers $A$ and $B$, one can find an integer $C$ for which $M_{1}=\\left\\{x^{2}+A x+B: x \\in \\mathbb{Z}\\right\\}$ and $M_{2}=\\left\\{2 x^{2}+2 x+C: x \\in \\mathbb{Z}\\right\\}$ do not intersect.","solution":"16. If $A$ is odd, then every number in $M_{1}$ is of the form $x(x+A)+B \\equiv B$ $(\\bmod 2)$, while numbers in $M_{2}$ are congruent to $C$ modulo 2 . Thus it is enough to take $C \\equiv B+1(\\bmod 2)$. If $A$ is even, then all numbers in $M_{1}$ have the form $\\left(X+\\frac{A}{2}\\right)^{2}+B-\\frac{A^{2}}{4}$ and are congruent to $B-\\frac{A^{2}}{4}$ or $B-\\frac{A^{2}}{4}+1$ modulo 4 , while numbers in $M_{2}$ are congruent to $C$ modulo 4 . So one can choose any $C \\equiv B-\\frac{A^{2}}{4}+2$ $(\\bmod 4)$.","problem_type":"Number Theory","tier":0} +{"year":"1995","problem_phase":"shortlisted","problem":"17. N3 $(\\mathbf{C Z E})^{\\mathrm{IMO} 3}$ Determine all integers $n>3$ such that there are $n$ points $A_{1}, A_{2}, \\ldots, A_{n}$ in the plane that satisfy the following two conditions simultaneously: (a) No three lie on the same line. (b) There exist real numbers $p_{1}, p_{2}, \\ldots, p_{n}$ such that the area of $\\triangle A_{i} A_{j} A_{k}$ is equal to $p_{i}+p_{j}+p_{k}$, for $1 \\leq i2$. Therefore $c=\\frac{b^{2}+z}{z^{2} b-1}<1$, a contradiction. The only solutions for $(x, y)$ are $(4,2),(4,6),(5,2),(5,3)$.","problem_type":"Number Theory","tier":0} +{"year":"1995","problem_phase":"shortlisted","problem":"19. N5 (IRE) At a meeting of $12 k$ people, each person exchanges greetings with exactly $3 k+6$ others. For any two people, the number who exchange greetings with both is the same. How many people are at the meeting?","solution":"19. For each two people let $n$ be the number of people exchanging greetings with both of them. To determine $n$ in terms of $k$, we shall count in two ways the number of triples $(A, B, C)$ of people such that $A$ exchanged greetings with both $B$ and $C$, but $B$ and $C$ mutually did not. There are $12 k$ possibilities for $A$, and for each $A$ there are $(3 k+6)$ possibilities for $B$. Since there are $n$ people who exchanged greetings with both $A$ and $B$, there are $3 k+5-n$ who did so with $A$ but not with $B$. Thus the number of triples $(A, B, C)$ is $12 k(3 k+6)(3 k+5-n)$. On the other hand, there are $12 k$ possible choices of $B$, and $12 k-1-(3 k+6)=9 k-7$ possible choices of $C$; for every $B, C, A$ can be chosen in $n$ ways, so the number of considered triples equals $12 k n(9 k-7)$. Hence $(3 k+6)(3 k+5-n)=n(9 k-7)$, i.e., $n=\\frac{3(k+2)(3 k+5)}{12 k-1}$. This gives us that $\\frac{4 n}{3}=\\frac{12 k^{2}+44 k+40}{12 k-1}=k+4-\\frac{3 k-44}{12 k-1}$ is an integer too. It is directly verified that only $k=3$ gives an integer value for $n$, namely $n=6$. Remark. The solution is complete under the assumption that such a $k$ exists. We give an example of such a party with 36 persons, $k=3$. Let the people sit in a $6 \\times 6$ array $\\left[P_{i j}\\right]_{i, j=1}^{6}$, and suppose that two persons $P_{i j}, P_{k l}$ exchanged greetings if and only if $i=k$ or $j=l$ or $i-j \\equiv k-l$ (mod 6). Thus each person exchanged greetings with exactly 15 others, and it is easily verified that this party satisfies the conditions.","problem_type":"Number Theory","tier":0} +{"year":"1995","problem_phase":"shortlisted","problem":"2. A2 (SWE) Let $a$ and $b$ be nonnegative integers such that $a b \\geq c^{2}$, where $c$ is an integer. Prove that there is a number $n$ and integers $x_{1}, x_{2}, \\ldots, x_{n}, y_{1}, y_{2}, \\ldots, y_{n}$ such that $$ \\sum_{i=1}^{n} x_{i}^{2}=a, \\quad \\sum_{i=1}^{n} y_{i}^{2}=b, \\quad \\text { and } \\quad \\sum_{i=1}^{n} x_{i} y_{i}=c $$","solution":"2. We may assume $c \\geq 0$ (otherwise, we may simply put $-y_{i}$ in the place of $\\left.y_{i}\\right)$. Also, we may assume $a \\geq b$. If $b \\geq c$, it is enough to take $n=a+b-c$, $x_{1}=\\cdots=x_{a}=1, y_{1}=\\cdots=y_{c}=y_{a+1}=\\cdots=y_{a+b-c}=1$, and the other $x_{i}$ 's and $y_{i}$ 's equal to 0 , so we need only consider the case $a>c>b$. We proceed to prove the statement of the problem by induction on $a+b$. The case $a+b=1$ is trivial. Assume that the statement is true when $a+b \\leq$ $N$, and let $a+b=N+1$. The triple $(a+b-2 c, b, c-b)$ satisfies the condition (since $(a+b-2 c) b-(c-b)^{2}=a b-c^{2}$ ), so by the induction hypothesis there are $n$-tuples $\\left(x_{i}\\right)_{i=1}^{n}$ and $\\left(y_{i}\\right)_{i=1}^{n}$ with the wanted property. It is easy to verify that $\\left(x_{i}+y_{i}\\right)_{i=1}^{n}$ and $\\left(y_{i}\\right)_{i=1}^{n}$ give a solution for $(a, b, c)$.","problem_type":"Algebra","tier":0} +{"year":"1995","problem_phase":"shortlisted","problem":"20. N6 (POL) ${ }^{\\mathrm{IMO} 06}$ Let $p$ be an odd prime. Find the number of $p$-element subsets $A$ of $\\{1,2, \\ldots, 2 p\\}$ such that the sum of all elements of $A$ is divisible by $p$.","solution":"20. We shall consider the set $M=\\{0,1, \\ldots, 2 p-1\\}$ instead. Let $M_{1}=$ $\\{0,1, \\ldots, p-1\\}$ and $M_{2}=\\{p, p+1, \\ldots, 2 p-1\\}$. We shall denote by $|A|$ and $\\sigma(A)$ the number of elements and the sum of elements of the set $A$; also, let $C_{p}$ be the family of all $p$-element subsets of $M$. Define the mapping $T: C_{p} \\rightarrow C_{p}$ as $T(A)=\\left\\{x+1 \\mid x \\in A \\cap M_{1}\\right\\} \\cup\\left\\{A \\cap M_{2}\\right\\}$, the addition being modulo $p$. There are exactly two fixed points of $T$ : these are $M_{1}$ and $M_{2}$. Now if $A$ is any subset from $C_{p}$ distinct from $M_{1}, M_{2}$, and $k=\\left|A \\cap M_{1}\\right|$ with $1 \\leq k \\leq p-1$, then for $i=0,1, \\ldots, p-1$, $\\sigma\\left(T^{i}(A)\\right)=\\sigma(A)+i k(\\bmod p)$. Hence subsets $A, T(A), \\ldots, T^{p-1}(A)$ are distinct, and exactly one of them has sum of elements divisible by $p$. Since $\\sigma\\left(M_{1}\\right), \\sigma\\left(M_{2}\\right)$ are divisible by $p$ and $C_{p} \\backslash\\left\\{M_{1}, M_{2}\\right\\}$ decomposes into families of the form $\\left\\{A, T(A), \\ldots, T^{p-1}(A)\\right\\}$, we conclude that the required number is $\\frac{1}{p}\\left(\\left|C_{p}\\right|-2\\right)+2=\\frac{1}{p}\\left(\\binom{2 p}{p}-2\\right)+2$. Second solution. Let $C_{k}$ be the family of all $k$-element subsets of $\\{1,2, \\ldots, 2 p\\}$. Denote by $M_{k}(k=1,2, \\ldots, p)$ the family of $p$-element multisets with $k$ distinct elements from $\\{1,2, \\ldots, 2 p\\}$, exactly one of which appears more than once, that have sum of elements divisible by $p$. It is clear that every subset from $C_{k}, k1$ that satisfies the following condition? The set of positive integers can be partitioned into $n$ nonempty subsets such that an arbitrary sum of $n-1$ integers, one taken from each of any $n-1$ of the subsets, lies in the remaining subset.","solution":"21. We shall show that there is no such $n$. Certainly, $n=2$ does not work, so suppose $n \\geq 3$. Let $a, b$ be distinct elements of $A_{1}$, and $c$ any integer greater than $-a$ and $-b$. We claim that $a+c, b+c$ belong to the same subsets. Suppose to the contrary that $a+c \\in A_{1}$ and $b+c \\in A_{2}$, and take arbitrary elements $x_{i} \\in A_{i}, i=3, \\ldots, n$. The number $b+x_{3}+\\cdots+x_{n}$ is in $A_{2}$, so that $s=(a+c)+\\left(b+x_{3}+\\cdots+x_{n}\\right)+x_{4}+\\cdots+x_{n}$ must be in $A_{3}$. On the other hand, $a+x_{3}+\\cdots+x_{n} \\in A_{2}$, so $s=\\left(a+x_{3}+\\cdots+x_{n}\\right)+$ $(b+c)+x_{4}+\\cdots+x_{n}$ is in $A_{1}$, a contradiction. Similarly, if $a+c \\in A_{2}$ and $b+c \\in A_{3}$, then $s=a+(b+c)+x_{4}+\\cdots+x_{n}$ belongs to $A_{2}$, but also $s=b+(a+c)+x_{4}+\\cdots+x_{n} \\in A_{3}$, which is impossible. For $i=1, \\ldots, n$ choose $x_{i} \\in A_{i}$; set $s=x_{1}+\\cdots+x_{n}$ and $y_{i}=s-x_{i}$. Then $y_{i} \\in A_{i}$. By what has been proved above, $2 x_{i}=x_{i}+x_{i}$ belongs to the same subset as $x_{i}+y_{i}=s$ does. It follows that all numbers $2 x_{i}, i=1, \\ldots, n$, are in the same subset. Since we can arbitrarily take $x_{i}$ from each set $A_{i}$, it follows that all even numbers belong to the same set, say $A_{1}$. Similarly, $2 x_{i}+1=\\left(x_{i}+1\\right)+x_{i}$ is in the subset to which $\\left(x_{i}+1\\right)+y_{i}=s+1$ belongs for all $i=1, \\ldots, n$; hence all odd numbers greater than 1 are in the same subset, say $A_{2}$. By the above considerations, $3-2=1 \\in A_{2}$ also. But then nothing remains in $A_{3}, \\ldots, A_{n}$, a contradiction.","problem_type":"Number Theory","tier":0} +{"year":"1995","problem_phase":"shortlisted","problem":"22. N8 (GER) Let $p$ be an odd prime. Determine positive integers $x$ and $y$ for which $x \\leq y$ and $\\sqrt{2 p}-\\sqrt{x}-\\sqrt{y}$ is nonnegative and as small as possible.","solution":"22. Let $u=\\sqrt{2 p}-\\sqrt{x}-\\sqrt{y}$ and $v=u(2 \\sqrt{2 p}-u)=2 p-(\\sqrt{2 p}-u)^{2}=$ $2 p-x-y-\\sqrt{4 x y}$ for $x, y \\in \\mathbb{N}, x \\leq y$. Obviously $u \\geq 0$ if and only if $v \\geq 0$, and $u, v$ attain minimum positive values simultaneously. Note that $v \\neq 0$. Otherwise $u=0$ too, so $y=(\\sqrt{2 p}-\\sqrt{x})^{2}=2 p-x-2 \\sqrt{2 p x}$, which implies that $2 p x$ is a square, and consequently $x$ is divisible by $2 p$, which is impossible. Now let $z$ be the smallest integer greater than $\\sqrt{4 x y}$. We have $z^{2}-1 \\geq 4 x y$, $z \\leq 2 p-x-y$, and $z \\leq p$ because $\\sqrt{4 x y} \\leq(\\sqrt{x}+\\sqrt{y})^{2}<2 p$. It follows that $$ v=2 p-x-y-\\sqrt{4 x y} \\geq z-\\sqrt{z^{2}-1}=\\frac{1}{z+\\sqrt{z^{2}-1}} \\geq \\frac{1}{p+\\sqrt{p^{2}-1}} $$ Equality holds if and only if $z=x+y=p$ and $4 x y=p^{2}-1$, which is satisfied only when $x=\\frac{p-1}{2}$ and $y=\\frac{p+1}{2}$. Hence for these values of $x, y$, both $u$ and $v$ attain positive minima.","problem_type":"Number Theory","tier":0} +{"year":"1995","problem_phase":"shortlisted","problem":"23. S1 (UKR) Does there exist a sequence $F(1), F(2), F(3), \\ldots$ of nonnegative integers that simultaneously satisfies the following three conditions? (a) Each of the integers $0,1,2, \\ldots$ occurs in the sequence. (b) Each positive integer occurs in the sequence infinitely often. (c) For any $n \\geq 2$, $$ F\\left(F\\left(n^{163}\\right)\\right)=F(F(n))+F(F(361)) . $$","solution":"23. By putting $F(1)=0$ and $F(361)=1$, condition (c) becomes $F\\left(F\\left(n^{163}\\right)\\right)=$ $F(F(n))$ for $n \\geq 2$. For $n=2,3, \\ldots, 360$ let $F(n)=n$, and inductively define $F(n)$ for $n \\geq 362$ as follows: $$ F(n)= \\begin{cases}F(m), & \\text { if } n=m^{163}, m \\in \\mathbb{N} ; \\\\ \\text { the least number not in }\\{F(k) \\mid k0$, so $f(x)=$ $x p(x) \/ q(x)$ is a positive integer too. Let $\\left\\{p_{0}, p_{1}, p_{2}, \\ldots\\right\\}$ be all prime numbers in increasing order. Since it easily follows by induction that all $x_{n}$ 's are square-free, we can assign to each of them a unique code according to which primes divide it: if $p_{m}$ is the largest prime dividing $x_{n}$, the code corresponding to $x_{n}$ will be $\\ldots 0 s_{m} s_{m-1} \\ldots s_{0}$, with $s_{i}=1$ if $p_{i} \\mid x_{n}$ and $s_{i}=0$ otherwise. Let us investigate how $f$ acts on these codes. If the code of $x_{n}$ ends with 0 , then $x_{n}$ is odd, so the code of $f\\left(x_{n}\\right)=x_{n+1}$ is obtained from that of $x_{n}$ by replacing $s_{0}=0$ by $s_{0}=1$. Furthermore, if the code of $x_{n}$ ends with $011 \\ldots 1$, then the code of $x_{n+1}$ ends with $100 \\ldots 0$ instead. Thus if we consider the codes as binary numbers, $f$ acts on them as an addition of 1 . Hence the code of $x_{n}$ is the binary representation of $n$ and thus $x_{n}$ uniquely determines $n$. Specifically, if $x_{n}=1995=3 \\cdot 5 \\cdot 7 \\cdot 19$, then its code is 10001110 and corresponds to $n=142$.","problem_type":null,"tier":0} +{"year":"1995","problem_phase":"shortlisted","problem":"26. S4 (NZL) Suppose that $x_{1}, x_{2}, x_{3}, \\ldots$ are positive real numbers for which $$ x_{n}^{n}=\\sum_{j=0}^{n-1} x_{n}^{j} $$ for $n=1,2,3, \\ldots$ Prove that for all $n$, $$ 2-\\frac{1}{2^{n-1}} \\leq x_{n}<2-\\frac{1}{2^{n}} $$","solution":"26. For $n=1$ the result is trivial, since $x_{1}=1$. Suppose now that $n \\geq 2$ and let $f_{n}(x)=x^{n}-\\sum_{i=0}^{n-1} x^{i}$. Note that $x_{n}$ is the unique positive real root of $f_{n}$, because $\\frac{f_{n}(x)}{x^{n-1}}=x-1-\\frac{1}{x}-\\cdots-\\frac{1}{x^{n-1}}$ is strictly increasing on $\\mathbb{R}^{+}$. Consider $g_{n}(x)=(x-1) f_{n}(x)=(x-2) x^{n}+1$. Obviously $g_{n}(x)$ has no positive roots other than 1 and $x_{n}>1$. Observe that $\\left(1-\\frac{1}{2^{n}}\\right)^{n}>$ $1-\\frac{n}{2^{n}} \\geq \\frac{1}{2}$ for $n \\geq 2$ (by Bernoulli's inequality). Since then $$ g_{n}\\left(2-\\frac{1}{2^{n}}\\right)=-\\frac{1}{2^{n}}\\left(2-\\frac{1}{2^{n}}\\right)^{n}+1=1-\\left(1-\\frac{1}{2^{n+1}}\\right)^{n}>0 $$ and $$ g_{n}\\left(2-\\frac{1}{2^{n-1}}\\right)=-\\frac{1}{2^{n-1}}\\left(2-\\frac{1}{2^{n-1}}\\right)^{n}+1=1-2\\left(1-\\frac{1}{2^{n}}\\right)^{n}<0 $$ we conclude that $x_{n}$ is between $2-\\frac{1}{2^{n-1}}$ and $2-\\frac{1}{2^{n}}$, as required. Remark. Moreover, $\\lim _{n \\rightarrow \\infty} 2^{n}\\left(2-x_{n}\\right)=1$.","problem_type":null,"tier":0} +{"year":"1995","problem_phase":"shortlisted","problem":"27. S5 (FIN) For positive integers $n$, the numbers $f(n)$ are defined inductively as follows: $f(1)=1$, and for every positive integer $n, f(n+1)$ is the greatest integer $m$ such that there is an arithmetic progression of positive integers $a_{1}95$ for all $n$. If to the contrary $f(n) \\leq 95$, we have $f(m)=n+f(m+95-f(n))$, so by induction $f(m)=k n+f(m+k(95-f(n))) \\geq k n$ for all $k$, which is impossible. Now for $m>95$ we have $f(m+f(n)-95)=n+f(m)$, and again by induction $f(m+k(f(n)-95))=k n+f(m)$ for all $m, n, k$. It follows that with $n$ fixed, $$ (\\forall m) \\lim _{k \\rightarrow \\infty} \\frac{f(m+k(f(n)-95))}{m+k(f(n)-95)}=\\frac{n}{f(n)-95} $$ hence $$ \\lim _{s \\rightarrow \\infty} \\frac{f(s)}{s}=\\frac{n}{f(n)-95} $$ Hence $\\frac{n}{f(n)-95}$ does not depend on $n$, i.e., $f(n) \\equiv c n+95$ for some constant $c$. It is easily checked that only $c=1$ is possible.","problem_type":null,"tier":0} +{"year":"1995","problem_phase":"shortlisted","problem":"3. A3 (UKR) Let $n$ be an integer, $n \\geq 3$. Let $a_{1}, a_{2}, \\ldots, a_{n}$ be real numbers such that $2 \\leq a_{i} \\leq 3$ for $i=1,2, \\ldots, n$. If $s=a_{1}+a_{2}+\\cdots+a_{n}$, prove that $$ \\frac{a_{1}^{2}+a_{2}^{2}-a_{3}^{2}}{a_{1}+a_{2}-a_{3}}+\\frac{a_{2}^{2}+a_{3}^{2}-a_{4}^{2}}{a_{2}+a_{3}-a_{4}}+\\cdots+\\frac{a_{n}^{2}+a_{1}^{2}-a_{2}^{2}}{a_{n}+a_{1}-a_{2}} \\leq 2 s-2 n $$","solution":"3. Write $A_{i}=\\frac{a_{i}^{2}+a_{i+1}^{2}-a_{i+2}^{2}}{a_{i}+a_{i+1}-a_{i+2}}=a_{i}+a_{i+1}+a_{i+2}-\\frac{2 a_{i} a_{i+1}}{a_{i}+a_{i+1}-a_{i+2}}$. Since $2 a_{i} a_{i+1} \\geq$ $4\\left(a_{i}+a_{i+1}-2\\right)$ (which is equivalent to $\\left.\\left(a_{i}-2\\right)\\left(a_{i+1}-2\\right) \\geq 0\\right)$, it follows that $A_{i} \\leq a_{i}+a_{i+1}+a_{i+2}-4\\left(1+\\frac{a_{i+2}-2}{a_{i}+a_{i+1}-a_{i+2}}\\right) \\leq a_{i}+a_{i+1}+a_{i+2}-$ $4\\left(1+\\frac{a_{i+2}-2}{4}\\right)$, because $1 \\leq a_{i}+a_{i+1}-a_{i+2} \\leq 4$. Therefore $A_{i} \\leq a_{i}+$ $a_{i+1}-2$, so $\\sum_{i=1}^{n} A_{i} \\leq 2 s-2 n$ as required.","problem_type":"Algebra","tier":0} +{"year":"1995","problem_phase":"shortlisted","problem":"4. A4 (USA) Let $a, b$, and $c$ be given positive real numbers. Determine all positive real numbers $x, y$, and $z$ such that $$ x+y+z=a+b+c $$ and $$ 4 x y z-\\left(a^{2} x+b^{2} y+c^{2} z\\right)=a b c $$","solution":"4. The second equation is equivalent to $\\frac{a^{2}}{y z}+\\frac{b^{2}}{z x}+\\frac{c^{2}}{x y}+\\frac{a b c}{x y z}=4$. Let $x_{1}=$ $\\frac{a}{\\sqrt{y z}}, y_{1}=\\frac{b}{\\sqrt{z x}}, z_{1}=\\frac{c}{\\sqrt{x y}}$. Then $x_{1}^{2}+y_{1}^{2}+z_{1}^{2}+x_{1} y_{1} z_{1}=4$, where $00$ is clearly impossible. On the other hand, if $u v w \\leq 0$, then two of $u, v, w$ are nonnegative, say $u, v \\geq 0$. Taking into account $w=-u-v$, the above equality reduces to $2\\left[(a+c-2 v) u^{2}+(b+c-2 u) v^{2}+2 c u v\\right]=0$, so $u=v=0$. Third solution. The fact that we are given two equations and three variables suggests that this is essentially a problem on inequalities. Setting $f(x, y, z)=4 x y z-a^{2} x-b^{2} y-c^{2} z$, we should show that $\\max f(x, y, z)=$ $a b c$, for $0\\frac{n-1}{4}$. Then $f(1 \/ x)=f\\left(x+1 \/ x^{2}\\right)-f(x)<1 \/ 4$, so $f(1 \/ x)>-1 \/ 2$. On the other hand, this implies $\\left(\\frac{n-1}{4}\\right)^{2}\\left(\\sum_{i=1}^{n-1}(n-i) x_{i}\\right)\\left(\\sum_{j=2}^{n}(j-1) x_{j}\\right) . $$","solution":"6. Let $y_{i}=x_{i+1}+\\cdots+x_{n}, Y=\\sum_{j=2}^{n}(j-1) x_{j}$, and $z_{i}=\\frac{n(n-1)}{2} y_{i}-(n-$ $i) Y$. Then $\\frac{n(n-1)}{2} \\sum_{i0$. Since $\\sum_{i=1}^{n-1} y_{i}=Y$ and $\\sum_{i=1}^{n-1}(n-i)=\\frac{n(n-1)}{2}$, we have $\\sum z_{i}=0$. Note that $Y<\\sum_{j=2}^{n}(j-1) x_{n}=\\frac{n(n-1)}{2} x_{n}$, and consequently $z_{n-1}=$ $\\frac{n(n-1)}{2} x_{n}-Y>0$. Furthermore, we have $$ \\frac{z_{i+1}}{n-i-1}-\\frac{z_{i}}{n-i}=\\frac{n(n-1)}{2}\\left(\\frac{y_{i+1}}{n-i-1}-\\frac{y_{i}}{n-i}\\right)>0 $$ which means that $\\frac{z_{1}}{n-1}<\\frac{z_{2}}{n-2}<\\cdots<\\frac{z_{n-1}}{1}$. Therefore there is a $k$ for which $z_{1}, \\ldots, z_{k} \\leq 0$ and $z_{k+1}, \\ldots, z_{n-1}>0$. But then $z_{i}\\left(x_{i}-x_{k}\\right) \\geq 0$, i.e., $x_{i} z_{i} \\geq x_{k} z_{i}$ for all $i$, so $\\sum_{i=1}^{n-1} x_{i} z_{i}>\\sum_{i=1}^{n-1} x_{k} z_{i}=0$ as required. Second solution. Set $X=\\sum_{j=1}^{n-1}(n-j) x_{j}$ and $Y=\\sum_{j=2}^{n}(j-1) x_{j}$. Since $4 X Y=(X+Y)^{2}-(X-Y)^{2}$, the RHS of the inequality becomes $$ X Y=\\frac{1}{4}\\left[(n-1)^{2}\\left(\\sum_{i=1}^{n} x_{i}\\right)^{2}-\\left(\\sum_{i=1}^{n}(2 i-1-n) x_{i}\\right)^{2}\\right] $$ The LHS equals $\\frac{1}{4}\\left((n-1)^{2}\\left(\\sum_{i=1}^{n} x_{i}\\right)^{2}-(n-1) \\sum_{i(n-1) \\sum_{i0$ (so, $x_{j}-x_{i}=d_{i}+d_{i+1}+\\cdots+d_{j-1}$ ) and expanding the obtained expressions, we reduce this inequality to $\\sum_{k} k^{2}(n-k)^{2} d_{k}^{2}+2 \\sum_{k\\sum_{k}(n-1) k(n-k) d_{k}^{2}+$ $2 \\sum_{kC A$. Let $O$ be the circumcenter, $H$ its orthocenter, and $F$ the foot of its altitude $C H$. Let the perpendicular to $O F$ at $F$ meet the side $C A$ at $P$. Prove that $\\angle F H P=\\angle B A C$. Possible second part: What happens if $|B C| \\leq|C A|$ (the triangle still being acute-angled)?","solution":"12. It is easy to see that $P$ lies on the segment $A C$. Let $E$ be the foot of the altitude $B H$ and $Y, Z$ the midpoints of $A C, A B$ respectively. Draw the perpendicular $H R$ to $F P(R \\in F P)$. Since $Y$ is the circumcenter of $\\triangle F C A$, we have $\\angle F Y A=180^{\\circ}-2 \\angle A$. Also, $O F P Y$ is cyclic; hence $\\angle O P F=\\angle O Y F=2 \\angle A-90^{\\circ}$. Next, $\\triangle O Z F$ and $\\triangle H R F$ are similar, so $O Z \/ O F=H R \/ H F$. This leads to $H R \\cdot O F=H F \\cdot O Z=\\frac{1}{2} H F$. $H C=\\frac{1}{2} H E \\cdot H B=H E \\cdot O Y \\Longrightarrow$ $H R \/ H E=O Y \/ O F$. Moreover, $\\angle E H R=\\angle F O Y$; hence the triangles $E H R$ and $F O Y$ are similar. Consequently $\\angle H P C=\\angle H R E=$ $\\angle O Y F=2 \\angle A-90^{\\circ}$, and finally, $\\angle F H P=\\angle H P C+\\angle H C P=\\angle A$. ![](https:\/\/cdn.mathpix.com\/cropped\/2024_11_18_8e985d6b9c83aa3e9d0eg-616.jpg?height=331&width=387&top_left_y=501&top_left_x=895) Second solution. As before, $\\angle H F Y=90^{\\circ}-\\angle A$, so it suffices to show that $H P \\perp F Y$. The points $O, F, P, Y$ lie on a circle, say $\\Omega_{1}$ with center at the midpoint $Q$ of $O P$. Furthermore, the points $F, Y$ lie on the nine-point circle $\\Omega$ of $\\triangle A B C$ with center at the midpoint $N$ of $O H$. The segment $F Y$ is the common chord of $\\Omega_{1}$ and $\\Omega$, from which we deduce that $N Q \\perp F Y$. However, $N Q \\| H P$, and the result follows. Third solution. Let $H^{\\prime}$ be the point symmetric to $H$ with respect to $A B$. Then $H^{\\prime}$ lies on the circumcircle of $A B C$. Let the line $F P$ meet the circumcircle at $U, V$ and meet $H^{\\prime} B$ at $P^{\\prime}$. Since $O F \\perp U V, F$ is the midpoint of $U V$. By the butterfly theorem, $F$ is also the midpoint of $P P^{\\prime}$. Therefore $\\triangle H^{\\prime} F P^{\\prime} \\cong F H P$; hence $\\angle F H P=\\angle F H^{\\prime} B=\\angle A$. Remark. It is possible to solve the problem using trigonometry. For example, $\\frac{F Z}{Z O}=\\frac{F K}{K P}=\\frac{\\sin (A-B)}{\\cos C}$, where $K$ is on $C F$ with $P K \\perp C F$. Then $\\frac{C F}{K P}=\\frac{\\sin (A-B)}{\\cos C}+\\tan A$, from which one obtains formulas for $K P$ and $K H$. Finally, we can calculate $\\tan \\angle F H P=\\frac{K P}{K H}=\\cdots=\\tan A$. Second remark. Here is what happens when $B C \\leq C A$. If $\\angle A>45^{\\circ}$, then $\\angle F H P=\\angle A$. If $\\angle A=45^{\\circ}$, the point $P$ escapes to infinity. If $\\angle A<45^{\\circ}$, the point $P$ appears on the extension of $A C$ over $C$, and $\\angle F H P=180^{\\circ}-\\angle A$.","problem_type":"Geometry","tier":0} +{"year":"1996","problem_phase":"shortlisted","problem":"13. G4 (USA) Let $\\triangle A B C$ be an equilateral triangle and let $P$ be a point in its interior. Let the lines $A P, B P, C P$ meet the sides $B C, C A, A B$ in the points $A_{1}, B_{1}, C_{1}$ respectively. Prove that $$ A_{1} B_{1} \\cdot B_{1} C_{1} \\cdot C_{1} A_{1} \\geq A_{1} B \\cdot B_{1} C \\cdot C_{1} A $$","solution":"13. By the law of cosines applied to $\\triangle C A_{1} B_{1}$, we obtain $$ A_{1} B_{1}^{2}=A_{1} C^{2}+B_{1} C^{2}-A_{1} C \\cdot B_{1} C \\geq A_{1} C \\cdot B_{1} C $$ Analogously, $B_{1} C_{1}^{2} \\geq B_{1} A \\cdot C_{1} A$ and $C_{1} A_{1}^{2} \\geq C_{1} B \\cdot A_{1} B$, so that multiplying these inequalities yields $$ A_{1} B_{1}^{2} \\cdot B_{1} C_{1}^{2} \\cdot C_{1} A_{1}^{2} \\geq A_{1} B \\cdot A_{1} C \\cdot B_{1} A \\cdot B_{1} C \\cdot C_{1} A \\cdot C_{1} B $$ Now, the lines $A A_{1}, B B_{1}, C C_{1}$ concur, so by Ceva's theorem, $A_{1} B \\cdot B_{1} C$. $C_{1} A=A B_{1} \\cdot B C_{1} \\cdot C A_{1}$, which together with (1) gives the desired inequality. Equality holds if and only if $C A_{1}=C B_{1}$, etc.","problem_type":"Geometry","tier":0} +{"year":"1996","problem_phase":"shortlisted","problem":"14. G5 (ARM) ${ }^{\\mathrm{IMO5}}$ Let $A B C D E F$ be a convex hexagon such that $A B$ is parallel to $D E, B C$ is parallel to $E F$, and $C D$ is parallel to $A F$. Let $R_{A}, R_{C}, R_{E}$ be the circumradii of triangles $F A B, B C D, D E F$ respectively, and let $P$ denote the perimeter of the hexagon. Prove that $$ R_{A}+R_{C}+R_{E} \\geq \\frac{P}{2} $$","solution":"14. Let $a, b, c, d, e$, and $f$ denote the lengths of the sides $A B, B C, C D, D E$, $E F$, and $F A$ respectively. Note that $\\angle A=\\angle D, \\angle B=\\angle E$, and $\\angle C=\\angle F$. Draw the lines $P Q$ and $R S$ through $A$ and $D$ perpendicular to $B C$ and $E F$ respectively $(P, R \\in B C, Q, S \\in E F)$. Then $B F \\geq P Q=R S$. Therefore $2 B F \\geq$ $P Q+R S$, or ![](https:\/\/cdn.mathpix.com\/cropped\/2024_11_18_8e985d6b9c83aa3e9d0eg-617.jpg?height=322&width=389&top_left_y=327&top_left_x=885) $$ \\begin{array}{ll} & 2 B F \\geq(a \\sin B+f \\sin C)+(c \\sin C+d \\sin B), \\\\ \\text { and similarly, } & 2 B D \\geq(c \\sin A+b \\sin B)+(e \\sin B+f \\sin A), \\\\ & 2 D F \\geq(e \\sin C+d \\sin A)+(a \\sin A+b \\sin C) \\end{array} $$ Next, we have the following formulas for the considered circumradii: $$ R_{A}=\\frac{B F}{2 \\sin A}, \\quad R_{C}=\\frac{B D}{2 \\sin C}, \\quad R_{E}=\\frac{D F}{2 \\sin E} $$ It follows from (1) that $$ \\begin{aligned} R_{A}+R_{C}+R_{E} & \\geq \\frac{1}{4} a\\left(\\frac{\\sin B}{\\sin A}+\\frac{\\sin A}{\\sin B}\\right)+\\frac{1}{4} b\\left(\\frac{\\sin C}{\\sin B}+\\frac{\\sin B}{\\sin C}\\right)+\\cdots \\\\ & \\geq \\frac{1}{2}(a+b+\\cdots)=\\frac{P}{2} \\end{aligned} $$ with equality if and only if $\\angle A=\\angle B=\\angle C=120^{\\circ}$ and $F B \\perp B C$ etc., i.e., if and only if the hexagon is regular. Second solution. Let us construct points $A^{\\prime \\prime}, C^{\\prime \\prime}, E^{\\prime \\prime}$ such that $A B A^{\\prime \\prime} F$, $C D C^{\\prime \\prime} B$, and $E F E^{\\prime \\prime} D$ are parallelograms. It follows that $A^{\\prime \\prime}, C^{\\prime \\prime}, B$ are collinear and also $C^{\\prime \\prime}, E^{\\prime \\prime}, B$ and $E^{\\prime \\prime}, A^{\\prime \\prime}, F$. Furthermore, let $A^{\\prime}$ be the intersection of the perpendiculars through $F$ and $B$ to $F A^{\\prime \\prime}$ and $B A^{\\prime \\prime}$, respectively, and let $C^{\\prime}$ and $E^{\\prime}$ be analogously defined. Since $A^{\\prime} F A^{\\prime \\prime} B$ is cyclic with the diameter being $A^{\\prime} A^{\\prime \\prime}$ and since $\\triangle F A^{\\prime \\prime} B \\cong$ $\\triangle B A F$, it follows that $2 R_{A}=$ ![](https:\/\/cdn.mathpix.com\/cropped\/2024_11_18_8e985d6b9c83aa3e9d0eg-617.jpg?height=372&width=504&top_left_y=1470&top_left_x=830) $A^{\\prime} A^{\\prime \\prime}=x$. Similarly, $2 R_{C}=C^{\\prime} C^{\\prime \\prime}=y$ and $2 R_{E}=E^{\\prime} E^{\\prime \\prime}=z$. We also have $A B=$ $F A^{\\prime \\prime}=y_{a}, A F=A^{\\prime \\prime} B=z_{a}, C D=C^{\\prime \\prime} B=z_{c}, C B=C^{\\prime \\prime} D=x_{c}$, $E F=E^{\\prime \\prime} D=x_{e}$, and $E D=E^{\\prime \\prime} F=y_{e}$. The original inequality we must prove now becomes $$ x+y+z \\geq y_{a}+z_{a}+z_{c}+x_{c}+x_{e}+y_{e} . $$ We now follow and generalize the standard proof of the Erd\u0151s-Mordell inequality (for the triangle $A^{\\prime} C^{\\prime} E^{\\prime}$ ), which is what (1) is equivalent to when $A^{\\prime \\prime}=C^{\\prime \\prime}=E^{\\prime \\prime}$. We set $C^{\\prime} E^{\\prime}=a, A^{\\prime} E^{\\prime}=c$ and $A^{\\prime} C^{\\prime}=e$. Let $A_{1}$ be the point symmetric to $A^{\\prime \\prime}$ with respect to the bisector of $\\angle E^{\\prime} A^{\\prime} C^{\\prime}$. Let $F_{1}$ and $B_{1}$ be the feet of the perpendiculars from $A_{1}$ to $A^{\\prime} C^{\\prime}$ and $A^{\\prime} E^{\\prime}$, respectively. In that case, $A_{1} F_{1}=A^{\\prime \\prime} F=y_{a}$ and $A_{1} B_{1}=A^{\\prime \\prime} B=z_{a}$. We have $$ \\begin{aligned} a x=A^{\\prime} A_{1} \\cdot E^{\\prime} C^{\\prime} \\geq 2 S_{A^{\\prime} E^{\\prime} A_{1} C^{\\prime}} & =2 S_{A^{\\prime} E^{\\prime} A_{1}}+2 S_{A^{\\prime} C^{\\prime} A_{1}} \\\\ & =c z_{a}+e y_{a} . \\end{aligned} $$ Similarly, $c y \\geq e x_{c}+a z_{c}$ and $e z \\geq a y_{e}+c x_{e}$. Thus $$ \\begin{aligned} x+y+z & \\geq \\frac{c}{a} z_{a}+\\frac{a}{c} z_{c}+\\frac{e}{c} x_{c}+\\frac{c}{e} x_{e}+\\frac{a}{e} y_{e}+\\frac{e}{a} y_{a} \\\\ & =\\left(\\frac{c}{a}+\\frac{a}{c}\\right)\\left(\\frac{z_{a}+z_{c}}{2}\\right)+\\left(\\frac{c}{a}-\\frac{a}{c}\\right)\\left(\\frac{z_{a}-z_{c}}{2}\\right)+\\cdots . \\end{aligned} $$ Let us set $a_{1}=\\frac{x_{c}-x_{e}}{2}, c_{1}=\\frac{y_{e}-y_{a}}{2}, e_{1}=\\frac{z_{a}-z_{c}}{2}$. We note that $\\triangle A^{\\prime \\prime} C^{\\prime \\prime} E^{\\prime \\prime} \\sim$ $\\triangle A^{\\prime} C^{\\prime} E^{\\prime}$ and hence $a_{1} \/ a=c_{1} \/ c=e_{1} \/ e=k$. Thus $\\left(\\frac{c}{a}-\\frac{a}{c}\\right) e_{1}+$ $\\left(\\frac{e}{c}-\\frac{c}{e}\\right) a_{1}+\\left(\\frac{a}{e}-\\frac{e}{a}\\right) c_{1}=k\\left(\\frac{c e}{a}-\\frac{a e}{c}+\\frac{e a}{c}-\\frac{c a}{e}+\\frac{a c}{e}-\\frac{e c}{a}\\right)=0$. Equation (2) reduces to $$ \\begin{aligned} x+y+z \\geq & \\left(\\frac{c}{a}+\\frac{a}{c}\\right)\\left(\\frac{z_{a}+z_{c}}{2}\\right)+\\left(\\frac{e}{c}+\\frac{c}{e}\\right)\\left(\\frac{x_{e}+x_{c}}{2}\\right) \\\\ & +\\left(\\frac{a}{e}+\\frac{e}{a}\\right)\\left(\\frac{y_{a}+y_{e}}{2}\\right) . \\end{aligned} $$ Using $c \/ a+a \/ c, e \/ c+c \/ e, a \/ e+e \/ a \\geq 2$ we finally get $x+y+z \\geq$ $y_{a}+z_{a}+z_{c}+x_{c}+x_{e}+y_{e}$. Equality holds if and only if $a=c=e$ and $A^{\\prime \\prime}=C^{\\prime \\prime}=E^{\\prime \\prime}=$ center of $\\triangle A^{\\prime} C^{\\prime} E^{\\prime}$, i.e., if and only if $A B C D E F$ is regular. Remark. From the second proof it is evident that the Erd\u0151s-Mordell inequality is a special case of the problem. if $P_{a}, P_{b}, P_{c}$ are the feet of the perpendiculars from a point $P$ inside $\\triangle A B C$ to the sides $B C, C A, A B$, and $P_{a} P P_{b} P_{c}^{\\prime}, P_{b} P P_{c} P_{a}^{\\prime}, P_{c} P P_{a} P_{b}^{\\prime}$ parallelograms, we can apply the problem to the hexagon $P_{a} P_{c}^{\\prime} P_{b} P_{a}^{\\prime} P_{c} P_{b}^{\\prime}$ to prove the Erd\u0151s-Mordell inequality for $\\triangle A B C$ and point $P$.","problem_type":"Geometry","tier":0} +{"year":"1996","problem_phase":"shortlisted","problem":"15. G6 (ARM) Let the sides of two rectangles be $\\{a, b\\}$ and $\\{c, d\\}$ with $a\\frac{1}{8}$. Hence $\\lambda \\mu \\nu \\leq \\frac{1}{8}$, with equality if and only if $\\lambda=\\mu=\\nu=\\frac{1}{2}$. This implies that $O$ is the centroid of $A B C$, and consequently, that the triangle is equilateral. Second solution. In the official solution, the inequality to be proved is transformed into $$ \\cos (A-B) \\cos (B-C) \\cos (C-A) \\geq 8 \\cos A \\cos B \\cos C $$ Since $\\frac{\\cos (B-C)}{\\cos A}=-\\frac{\\cos (B-C)}{\\cos (B+C)}=\\frac{\\tan B \\tan C+1}{\\tan B \\tan C-1}$, the last inequality becomes $(x y+1)(y z+1)(z x+1) \\geq 8(x y-1)(y z-1)(z x-1)$, where we write $x, y, z$ for $\\tan A, \\tan B, \\tan C$. Using the relation $x+y+z=x y z$, we can reduce this inequality to $$ (2 x+y+z)(x+2 y+z)(x+y+2 z) \\geq 8(x+y)(y+z)(z+x) $$ This follows from the AM-GM inequality: $2 x+y+z=(x+y)+(x+z) \\geq$ $2 \\sqrt{(x+y)(x+z)}$, etc.","problem_type":"Geometry","tier":0} +{"year":"1996","problem_phase":"shortlisted","problem":"17. G8 (RUS) Let $A B C D$ be a convex quadrilateral, and let $R_{A}, R_{B}, R_{C}$, and $R_{D}$ denote the circumradii of the triangles $D A B, A B C, B C D$, and $C D A$ respectively. Prove that $R_{A}+R_{C}>R_{B}+R_{D}$ if and only if $$ \\angle A+\\angle C>\\angle B+\\angle D . $$","solution":"17. Let the diagonals $A C$ and $B D$ meet in $X$. Either $\\angle A X B$ or $\\angle A X D$ is geater than or equal to $90^{\\circ}$, so we assume w.l.o.g. that $\\angle A X B \\geq 90^{\\circ}$. Let $\\alpha, \\beta, \\alpha^{\\prime}, \\beta^{\\prime}$ denote $\\angle C A B, \\angle A B D, \\angle B D C, \\angle D C A$. These angles are all acute and satisfy $\\alpha+\\beta=\\alpha^{\\prime}+\\beta^{\\prime}$. Furthermore, $$ R_{A}=\\frac{A D}{2 \\sin \\beta}, \\quad R_{B}=\\frac{B C}{2 \\sin \\alpha}, \\quad R_{C}=\\frac{B C}{2 \\sin \\alpha^{\\prime}}, \\quad R_{D}=\\frac{A D}{2 \\sin \\beta^{\\prime}} $$ Let $\\angle B+\\angle D=180^{\\circ}$. Then $A, B, C, D$ are concyclic and trivially $R_{A}+$ $R_{C}=R_{B}+R_{D}$. Let $\\angle B+\\angle D>180^{\\circ}$. Then $D$ lies within the circumcircle of $A B C$, which implies that $\\beta>\\beta^{\\prime}$. Similarly $\\alpha<\\alpha^{\\prime}$, so we obtain $R_{A}R_{D}$ and $R_{C}>R_{B}$, so $R_{A}+R_{C}>R_{B}+R_{D}$.","problem_type":"Geometry","tier":0} +{"year":"1996","problem_phase":"shortlisted","problem":"18. G9 (UKR) In the plane are given a point $O$ and a polygon $\\mathcal{F}$ (not necessarily convex). Let $P$ denote the perimeter of $\\mathcal{F}, D$ the sum of the distances from $O$ to the vertices of $\\mathcal{F}$, and $H$ the sum of the distances from $O$ to the lines containing the sides of $\\mathcal{F}$. Prove that $$ D^{2}-H^{2} \\geq \\frac{P^{2}}{4} $$","solution":"18. We first prove the result in the simplest case. Given a 2 -gon $A B A$ and a point $O$, let $a, b, c, h$ denote $O A, O B, A B$, and the distance of $O$ from $A B$. Then $D=a+b, P=2 c$, and $H=2 h$, so we should show that $$ (a+b)^{2} \\geq 4 h^{2}+c^{2} $$ Indeed, let $l$ be the line through $O$ parallel to $A B$, and $D$ the point symmetric to $B$ with respect to $l$. Then $(a+b)^{2}=(O A+O B)^{2}=(O A+$ $O D)^{2} \\geq A D^{2}=c^{2}+4 h^{2}$. Now we pass to the general case. Let $A_{1} A_{2} \\ldots A_{n}$ be the polygon $\\mathcal{F}$ and denote by $d_{i}, p_{i}$, and $h_{i}$ respectively $O A_{i}, A_{i} A_{i+1}$, and the distance of $O$ from $A_{i} A_{i+1}$ (where $A_{n+1}=A_{1}$ ). By the case proved above, we have for each $i, d_{i}+d_{i+1} \\geq \\sqrt{4 h_{i}^{2}+p_{i}^{2}}$. Summing these inequalities for $i=1, \\ldots, n$ and squaring, we obtain $$ 4 D^{2} \\geq\\left(\\sum_{i=1}^{n} \\sqrt{4 h_{i}^{2}+p_{i}^{2}}\\right)^{2} $$ It remains only to prove that $\\sum_{i=1}^{n} \\sqrt{4 h_{i}^{2}+p_{i}^{2}} \\geq \\sqrt{\\sum_{i=1}^{n}\\left(4 h_{i}^{2}+p_{i}^{2}\\right)}=$ $\\sqrt{4 H^{2}+D^{2}}$. But this follows immediately from the Minkowski inequality. Equality holds if and only if it holds in (1) and in the Minkowski inequality, i.e., if and only if $d_{1}=\\cdots=d_{n}$ and $h_{1} \/ p_{1}=\\cdots=h_{n} \/ p_{n}$. This means that $\\mathcal{F}$ is inscribed in a circle with center at $O$ and $p_{1}=\\cdots=p_{n}$, so $\\mathcal{F}$ is a regular polygon and $O$ its center.","problem_type":"Geometry","tier":0} +{"year":"1996","problem_phase":"shortlisted","problem":"19. N1 (UKR) Four integers are marked on a circle. At each step we simultaneously replace each number by the difference between this number and the next number on the circle, in a given direction (that is, the numbers $a, b, c, d$ are replaced by $a-b, b-c, c-d, d-a)$. Is it possible after 1996 such steps to have numbers $a, b, c, d$ such that the numbers $|b c-a d|,|a c-b d|,|a b-c d|$ are primes?","solution":"19. It is easy to check that after 4 steps we will have all $a, b, c, d$ even. Thus $|a b-c d|,|a c-b d|,|a d-b c|$ remain divisible by 4 , and clearly are not prime. The answer is no. Second solution. After one step we have $a+b+c+d=0$. Then $a c-b d=$ $a c+b(a+b+c)=(a+b)(b+c)$ etc., so $$ |a b-c d| \\cdot|a c-b d| \\cdot|a d-b c|=(a+b)^{2}(a+c)^{2}(b+c)^{2} $$ However, the product of three primes cannot be a square, hence the answer is $n o$.","problem_type":"Number Theory","tier":0} +{"year":"1996","problem_phase":"shortlisted","problem":"2. A2 (IRE) Let $a_{1} \\geq a_{2} \\geq \\cdots \\geq a_{n}$ be real numbers such that $$ a_{1}^{k}+a_{2}^{k}+\\cdots+a_{n}^{k} \\geq 0 $$ for all integers $k>0$. Let $p=\\max \\left\\{\\left|a_{1}\\right|, \\ldots,\\left|a_{n}\\right|\\right\\}$. Prove that $p=a_{1}$ and that $$ \\left(x-a_{1}\\right)\\left(x-a_{2}\\right) \\cdots\\left(x-a_{n}\\right) \\leq x^{n}-a_{1}^{n} $$ for all $x>a_{1}$.","solution":"2. Clearly $a_{1}>0$, and if $p \\neq a_{1}$, we must have $a_{n}<0,\\left|a_{n}\\right|>\\left|a_{1}\\right|$, and $p=-a_{n}$. But then for sufficiently large odd $k,-a_{n}^{k}=\\left|a_{n}\\right|^{k}>(n-1)\\left|a_{1}\\right|^{k}$, so that $a_{1}^{k}+\\cdots+a_{n}^{k} \\leq(n-1)\\left|a_{1}\\right|^{k}-\\left|a_{n}\\right|^{k}<0$, a contradiction. Hence $p=a_{1}$. Now let $x>a_{1}$. From $a_{1}+\\cdots+a_{n} \\geq 0$ we deduce $\\sum_{j=2}^{n}\\left(x-a_{j}\\right) \\leq$ $(n-1)\\left(x+\\frac{a_{1}}{n-1}\\right)$, so by the AM-GM inequality, $$ \\left(x-a_{2}\\right) \\cdots\\left(x-a_{n}\\right) \\leq\\left(x+\\frac{a_{1}}{n-1}\\right)^{n-1} \\leq x^{n-1}+x^{n-2} a_{1}+\\cdots+a_{1}^{n-1} $$ The last inequality holds because $\\binom{n-1}{r} \\leq(n-1)^{r}$ for all $r \\geq 0$. Multiplying (1) by $\\left(x-a_{1}\\right)$ yields the desired inequality.","problem_type":"Algebra","tier":0} +{"year":"1996","problem_phase":"shortlisted","problem":"20. N2 (RUS) ${ }^{\\mathrm{IMO} 4}$ The positive integers $a$ and $b$ are such that the numbers $15 a+16 b$ and $16 a-15 b$ are both squares of positive integers. What is the least possible value that can be taken on by the smaller of these two squares?","solution":"20. Let $15 a+16 b=x^{2}$ and $16 a-15 b=y^{2}$, where $x, y \\in \\mathbb{N}$. Then we obtain $x^{4}+y^{4}=(15 a+16 b)^{2}+(16 a-15 b)^{2}=\\left(15^{2}+16^{2}\\right)\\left(a^{2}+b^{2}\\right)=481\\left(a^{2}+b^{2}\\right)$. In particular, $481=13 \\cdot 37 \\mid x^{4}+y^{4}$. We have the following lemma. Lemma. Suppose that $p \\mid x^{4}+y^{4}$, where $x, y \\in \\mathbb{Z}$ and $p$ is an odd prime, where $p \\not \\equiv 1(\\bmod 8)$. Then $p \\mid x$ and $p \\mid y$. Proof. Since $p \\mid x^{8}-y^{8}$ and by Fermat's theorem $p \\mid x^{p-1}-y^{p-1}$, we deduce that $p \\mid x^{d}-y^{d}$, where $d=(p-1,8)$. But $d \\neq 8$, so $d \\mid 4$. Thus $p \\mid x^{4}-y^{4}$, which implies that $p \\mid 2 y^{4}$, i.e., $p \\mid y$ and $p \\mid x$. In particular, we can conclude that $13 \\mid x, y$ and $37 \\mid x, y$. Hence $x$ and $y$ are divisible by 481 . Thus each of them is at least 481. On the other hand, $x=y=481$ is possible. It is sufficient to take $a=$ $31 \\cdot 481$ and $b=481$. Second solution. Note that $15 x^{2}+16 y^{2}=481 a^{2}$. It can be directly verified that the divisibility of $15 x^{2}+16 y^{2}$ by 13 and by 37 implies that both $x$ and $y$ are divisible by both primes. Thus $481 \\mid x, y$.","problem_type":"Number Theory","tier":0} +{"year":"1996","problem_phase":"shortlisted","problem":"21. N3 (BUL) A finite sequence of integers $a_{0}, a_{1}, \\ldots, a_{n}$ is called quadratic if for each $i \\in\\{1,2, \\ldots, n\\}$ we have the equality $\\left|a_{i}-a_{i-1}\\right|=i^{2}$. (a) Prove that for any two integers $b$ and $c$, there exist a natural number $n$ and a quadratic sequence with $a_{0}=b$ and $a_{n}=c$. (b) Find the smallest natural number $n$ for which there exists a quadratic sequence with $a_{0}=0$ and $a_{n}=1996$.","solution":"21. (a) It clearly suffices to show that for every integer $c$ there exists a quadratic sequence with $a_{0}=0$ and $a_{n}=c$, i.e., that $c$ can be expressed as $\\pm 1^{2} \\pm 2^{2} \\pm \\cdots \\pm n^{2}$. Since $$ (n+1)^{2}-(n+2)^{2}-(n+3)^{2}+(n+4)^{2}=4 $$ we observe that if our claim is true for $c$, then it is also true for $c \\pm 4$. Thus it remains only to prove the claim for $c=0,1,2,3$. But one immediately finds $1=1^{2}, 2=-1^{2}-2^{2}-3^{2}+4^{2}$, and $3=-1^{2}+2^{2}$, while the case $c=0$ is trivial. (b) We have $a_{0}=0$ and $a_{n}=1996$. Since $a_{n} \\leq 1^{2}+2^{2}+\\cdots+n^{2}=$ $\\frac{1}{6} n(n+1)(2 n+1)$, we get $a_{17} \\leq 1785$, so $n \\geq 18$. On the other hand, $a_{18}$ is of the same parity as $1^{2}+2^{2}+\\cdots+18^{2}=2109$, so it cannot be equal to 1996. Therefore we must have $n \\geq 19$. To construct a required sequence with $n=19$, we note that $1^{2}+2^{2}+\\cdots+19^{2}=$ $2470=1996+2 \\cdot 237$; hence it is enough to write 237 as a sum of distinct squares. Since $237=14^{2}+5^{2}+4^{2}$, we finally obtain $$ 1996=1^{2}+2^{2}+3^{2}-4^{2}-5^{2}+6^{2}+\\cdots+13^{2}-14^{2}+15^{2}+\\cdots+19^{2} $$","problem_type":"Number Theory","tier":0} +{"year":"1996","problem_phase":"shortlisted","problem":"22. N4 (BUL) Find all positive integers $a$ and $b$ for which $$ \\left[\\frac{a^{2}}{b}\\right]+\\left[\\frac{b^{2}}{a}\\right]=\\left[\\frac{a^{2}+b^{2}}{a b}\\right]+a b $$ where as usual, $[t]$ refers to greatest integer that is less than or equal to $t$.","solution":"22. Let $a, b \\in \\mathbb{N}$ satisfy the given equation. It is not possible that $a=b$ (since it leads to $a^{2}+2=2 a$ ), so we assume w.l.o.g. that $a>b$. Next, for $a>b=1$ the equation becomes $a^{2}=2 a$, and one obtains a solution $(a, b)=(2,1)$. Let $b>1$. If $\\left[\\frac{a^{2}}{b}\\right]=\\alpha$ and $\\left[\\frac{b^{2}}{a}\\right]=\\beta$, then we trivially have $a b \\geq$ $\\alpha \\beta$. Since also $\\frac{a^{2}+b^{2}}{a b} \\geq 2$, we obtain $\\alpha+\\beta \\geq \\alpha \\beta+2$, or equivalently $(\\alpha-1)(\\beta-1) \\leq-1$. But $\\alpha \\geq 1$, and therefore $\\beta=0$. It follows that $a>b^{2}$, i.e., $a=b^{2}+c$ for some $c>0$. Now the given equation becomes $b^{3}+2 b c+\\left[\\frac{c^{2}}{b}\\right]=\\left[\\frac{b^{4}+2 b^{2} c+b^{2}+c^{2}}{b^{3}+b c}\\right]+b^{3}+b c$, which reduces to $$ (c-1) b+\\left[\\frac{c^{2}}{b}\\right]=\\left[\\frac{b^{2}(c+1)+c^{2}}{b^{3}+b c}\\right] $$ If $c=1$, then (1) always holds, since both sides are 0 . We obtain a family of solutions $(a, b)=\\left(n, n^{2}+1\\right)$ or $(a, b)=\\left(n^{2}+1, n\\right)$. Note that the solution $(1,2)$ found earlier is obtained for $n=1$. If $c>1$, then $(1)$ implies that $\\frac{b^{2}(c+1)+c^{2}}{b^{3}+b c} \\geq(c-1) b$. This simplifies to $$ c^{2}\\left(b^{2}-1\\right)+b^{2}\\left(c\\left(b^{2}-2\\right)-\\left(b^{2}+1\\right)\\right) \\leq 0 $$ Since $c \\geq 2$ and $b^{2}-2 \\geq 0$, the only possibility is $b=2$. But then (2) becomes $3 c^{2}+8 c-20 \\leq 0$, which does not hold for $c \\geq 2$. Hence the only solutions are $\\left(n, n^{2}+1\\right)$ and $\\left(n^{2}+1, n\\right), n \\in \\mathbb{N}$.","problem_type":"Number Theory","tier":0} +{"year":"1996","problem_phase":"shortlisted","problem":"23. N5 (ROM) Let $\\mathbb{N}_{0}$ denote the set of nonnegative integers. Find a bijective function $f$ from $\\mathbb{N}_{0}$ into $\\mathbb{N}_{0}$ such that for all $m, n \\in \\mathbb{N}_{0}$, $$ f(3 m n+m+n)=4 f(m) f(n)+f(m)+f(n) . $$","solution":"23. We first observe that the given functional equation is equivalent to $$ 4 f\\left(\\frac{(3 m+1)(3 n+1)-1}{3}\\right)+1=(4 f(m)+1)(4 f(n)+1) . $$ This gives us the idea of introducing a function $g: 3 \\mathbb{N}_{0}+1 \\rightarrow 4 \\mathbb{N}_{0}+1$ defined as $g(x)=4 f\\left(\\frac{x-1}{3}\\right)+1$. By the above equality, $g$ will be multiplicative, i.e., $$ g(x y)=g(x) g(y) \\quad \\text { for all } x, y \\in 3 \\mathbb{N}_{0}+1 $$ Conversely, any multiplicative bijection $g$ from $3 \\mathbb{N}_{0}+1$ onto $4 \\mathbb{N}_{0}+1$ gives us a function $f$ with the required property: $f(x)=\\frac{g(3 x+1)-1}{4}$. It remains to give an example of such a function $g$. Let $P_{1}, P_{2}, Q_{1}, Q_{2}$ be the sets of primes of the forms $3 k+1,3 k+2,4 k+1$, and $4 k+3$, respectively. It is well known that these sets are infinite. Take any bijection $h$ from $P_{1} \\cup P_{2}$ onto $Q_{1} \\cup Q_{2}$ that maps $P_{1}$ bijectively onto $Q_{1}$ and $P_{2}$ bijectively onto $Q_{2}$. Now define $g$ as follows: $g(1)=1$, and for $n=p_{1} p_{2} \\cdots p_{m}\\left(p_{i}\\right.$ 's need not be different) define $g(n)=h\\left(p_{1}\\right) h\\left(p_{2}\\right) \\cdots h\\left(p_{m}\\right)$. Note that $g$ is well-defined. Indeed, among the $p_{i}$ 's an even number are of the form $3 k+2$, and consequently an even number of $h\\left(p_{i}\\right) \\mathrm{s}$ are of the form $4 k+3$. Hence the product of the $h\\left(p_{i}\\right)$ 's is of the form $4 k+1$. Also, it is obvious that $g$ is multiplicative. Thus, the defined $g$ satisfies all the required properties.","problem_type":"Number Theory","tier":0} +{"year":"1996","problem_phase":"shortlisted","problem":"24. C1 (FIN) ${ }^{\\mathrm{IMO} 1}$ We are given a positive integer $r$ and a rectangular board $A B C D$ with dimensions $|A B|=20,|B C|=12$. The rectangle is divided into a grid of $20 \\times 12$ unit squares. The following moves are permitted on the board: One can move from one square to another only if the distance between the centers of the two squares is $\\sqrt{r}$. The task is to find a sequence of moves leading from the square corresponding to vertex $A$ to the square corresponding to vertex $B$. (a) Show that the task cannot be done if $r$ is divisible by 2 or 3 . (b) Prove that the task is possible when $r=73$. (c) Is there a solution when $r=97$ ?","solution":"24. We shall work on the array of lattice points defined by $\\mathcal{A}=\\left\\{(x, y) \\in \\mathbb{Z}^{2} \\mid\\right.$ $0 \\leq x \\leq 19,0 \\leq y \\leq 11\\}$. Our task is to move from $(0,0)$ to $(19,0)$ via the points of $\\mathcal{A}$ so that each move has the form $(x, y) \\rightarrow(x+a, y+b)$, where $a, b \\in \\mathbb{Z}$ and $a^{2}+b^{2}=r$. (a) If $r$ is even, then $a+b$ is even whenever $a^{2}+b^{2}=r(a, b \\in \\mathbb{Z})$. Thus the parity of $x+y$ does not change after each move, so we cannot reach $(19,0)$ from $(0,0)$. If $3 \\mid r$, then both $a$ and $b$ are divisible by 3 , so if a point $(x, y)$ can be reached from $(0,0)$, we must have $3 \\mid x$. Since $3 \\nmid 19$, we cannot get to $(19,0)$. (b) We have $r=73=8^{2}+3^{2}$, so each move is either $(x, y) \\rightarrow(x \\pm 8, y \\pm 3)$ or $(x, y) \\rightarrow(x \\pm 3, y \\pm 8)$. One possible solution is shown in Fig. 1. (c) We have $97=9^{2}+4^{2}$. Let us partition $\\mathcal{A}$ as $\\mathcal{B} \\cup \\mathcal{C}$, where $\\mathcal{B}=$ $\\{(x, y) \\in \\mathcal{A} \\mid 4 \\leq y \\leq 7\\}$. It is easily seen that moves of the type $(x, y) \\rightarrow(x \\pm 9, y \\pm 4)$ always take us from the set $\\mathcal{B}$ to $\\mathcal{C}$ and vice versa, while the moves $(x, y) \\rightarrow(x \\pm 4, y \\pm 9)$ always take us from $\\mathcal{C}$ to $\\mathcal{C}$. Furthermore, each move of the type $(x, y) \\rightarrow(x \\pm 9, y \\pm 4)$ changes the parity of $x$, so to get from $(0,0)$ to $(19,0)$ we must have an odd number of such moves. On the other hand, with an odd number of such moves, starting from $\\mathcal{C}$ we can end up only in $\\mathcal{B}$, although the point $(19,0)$ is not in $\\mathcal{B}$. Hence, the answer is no. Remark. Part (c) can also be solved by examining all cells that can be reached from $(0,0)$. All these cells are marked in Fig. 2. ![](https:\/\/cdn.mathpix.com\/cropped\/2024_11_18_8e985d6b9c83aa3e9d0eg-623.jpg?height=259&width=420&top_left_y=1081&top_left_x=289) Fig. 1 ![](https:\/\/cdn.mathpix.com\/cropped\/2024_11_18_8e985d6b9c83aa3e9d0eg-623.jpg?height=256&width=414&top_left_y=1083&top_left_x=886) Fig. 2","problem_type":"Combinatorics","tier":0} +{"year":"1996","problem_phase":"shortlisted","problem":"25. C2 (UKR) An $(n-1) \\times(n-1)$ square is divided into $(n-1)^{2}$ unit squares in the usual manner. Each of the $n^{2}$ vertices of these squares is to be colored red or blue. Find the number of different colorings such that each unit square has exactly two red vertices. (Two coloring schemes are regarded as different if at least one vertex is colored differently in the two schemes.)","solution":"25. Let the vertices in the bottom row be assigned an arbitrary coloring, and suppose that some two adjacent vertices receive the same color. The number of such colorings equals $2^{n}-2$. It is easy to see that then the colors of the remaining vertices get fixed uniquely in order to satisfy the requirement. So in this case there are $2^{n}-2$ possible colorings. Next, suppose that the vertices in the bottom row are colored alternately red and blue. There are two such colorings. In this case, the same must hold for every row, and thus we get $2^{n}$ possible colorings. It follows that the total number of considered colorings is $\\left(2^{n}-2\\right)+2^{n}=$ $2^{n+1}-2$.","problem_type":"Combinatorics","tier":0} +{"year":"1996","problem_phase":"shortlisted","problem":"26. C3 (USA) Let $k, m, n$ be integers such that $12$ and that the result holds for $n-1$. Suppose that $S \\subseteq\\{1,2, \\ldots, k\\}$ does not contain $n$ distinct elements with the sum $m$, and let $x$ be the smallest element of $S$. We may assume that $x \\leq r_{k}(m, n)$, because otherwise the statement is clear. Consider the set $S^{\\prime}=\\{y-x \\mid$ $y \\in S, y \\neq x\\}$. Then $S^{\\prime}$ is a subset of $\\{1,2, \\ldots, k-x\\}$ no $n-1$ elements of which have the sum $m-n x$. Also, it is easily checked that $n-1 \\leq$ $m-n x-1 \\leq k-x$, so we may apply the induction hypothesis, which yields that $$ |S| \\leq 1+k-x-r_{k}(m-n x, n-1)=k-\\left[\\frac{m-x}{n-1}-\\frac{n}{2}\\right] . $$ On the other hand, $\\left(\\frac{m-x}{n-1}-\\frac{n}{2}\\right)-r_{k}(m, n)=\\frac{m-n x-\\frac{n(n-1)}{2}}{n(n-1)} \\geq 0$ because $x \\leq r_{k}(m, n)$; hence (2) implies $|S| \\leq k-r_{k}(m, n)$ as claimed.","problem_type":"Combinatorics","tier":0} +{"year":"1996","problem_phase":"shortlisted","problem":"27. C4 (FIN) Determine whether or not there exist two disjoint infinite sets $\\mathcal{A}$ and $\\mathcal{B}$ of points in the plane satisfying the following conditions: (i) No three points in $\\mathcal{A} \\cup \\mathcal{B}$ are collinear, and the distance between any two points in $\\mathcal{A} \\cup \\mathcal{B}$ is at least 1. (ii) There is a point of $\\mathcal{A}$ in any triangle whose vertices are in $\\mathcal{B}$, and there is a point of $\\mathcal{B}$ in any triangle whose vertices are in $\\mathcal{A}$.","solution":"27. Suppose that such sets of points $\\mathcal{A}, \\mathcal{B}$ exist. First, we observe that there exist five points $A, B, C, D, E$ in $\\mathcal{A}$ such that their convex hull does not contain any other point of $\\mathcal{A}$. Indeed, take any point $A \\in \\mathcal{A}$. Since any two points of $\\mathcal{A}$ are at distance at least 1 , the number of points $X \\in \\mathcal{A}$ with $X A \\leq r$ is finite for every $r>0$. Thus it is enough to choose four points $B, C, D, E$ of $\\mathcal{A}$ that are closest to $A$. Now consider the convex hull $\\mathcal{C}$ of $A, B, C, D, E$. Suppose that $\\mathcal{C}$ is a pentagon, say $A B C D E$. Then each of the disjoint triangles $A B C, A C D, A D E$ contains a point of $\\mathcal{B}$. Denote these points by $P, Q, R$. Then $\\triangle P Q R$ contains some point $F \\in \\mathcal{A}$, so $F$ is inside $A B C D E$, a contradiction. Suppose that $\\mathcal{C}$ is a quadrilateral, say $A B C D$, with $E$ lying within $A B C D$. Then the triangles $A B E, B C E, C D E, D A E$ contain some points $P, Q, R, S$ of $\\mathcal{B}$ that form two disjoint triangles. It follows that there are two points of $\\mathcal{A}$ inside $A B C D$, which is a contradiction. Finally, suppose that $\\mathcal{C}$ is a triangle with two points of $\\mathcal{A}$ inside. Then $\\mathcal{C}$ is the union of five disjoint triangles with vertices in $\\mathcal{A}$, so there are at least five points of $\\mathcal{B}$ inside $\\mathcal{C}$. These five points make at least three disjoint triangles containing three points of $\\mathcal{A}$. This is again a contradiction. It follows that no such sets $\\mathcal{A}, \\mathcal{B}$ exist.","problem_type":"Combinatorics","tier":0} +{"year":"1996","problem_phase":"shortlisted","problem":"28. C5 (FRA) ${ }^{\\mathrm{IMO} 6}$ Let $p, q, n$ be three positive integers with $p+q1, k=q t, l=p t$, and $n=(p+q) t$. Consider the sequence $y_{i}=x_{i+p+q}-x_{i}, i=0, \\ldots, n-p-q$. We claim that at least one of the $y_{i}$ 's equals zero. We begin by noting that each $y_{i}$ is of the form $u p-v q$, where $u+v=p+q$; therefore $y_{i}=(u+v) p-$ $v(p+q)=(p-v)(p+q)$ is always divisible by $p+q$. Moreover, $y_{i+1}-y_{i}=$ $\\left(x_{i+p+q+1}-x_{i+p+q}\\right)-\\left(x_{i+1}-x_{i}\\right)$ is 0 or $\\pm(p+q)$. We conclude that if no $y_{i}$ is 0 then all $y_{i}$ 's are of the same sign. But this is in contradiction with the relation $y_{0}+y_{p+q}+\\cdots+y_{n-p-q}=x_{n}-x_{0}=0$. Consequently some $y_{i}$ is zero, as claimed. Second solution. As before we assume $(p, q)=1$. Let us define a sequence of points $A_{i}\\left(y_{i}, z_{i}\\right)(i=0,1, \\ldots, n)$ in $\\mathbb{N}_{0}^{2}$ inductively as follows. Set $A_{0}=$ $(0,0)$ and define $\\left(y_{i+1}, z_{i+1}\\right)$ as $\\left(y_{i}, z_{i}+1\\right)$ if $x_{i+1}=x_{i}+p$ and $\\left(y_{i}+1, z_{i}\\right)$ otherwise. The points $A_{i}$ form a trajectory $L$ in $\\mathbb{N}_{0}^{2}$ continuously moving upwards and rightwards by steps of length 1 . Clearly, $x_{i}=p z_{i}-q y_{i}$ for all $i$. Since $x_{n}=0$, it follows that $\\left(z_{n}, y_{n}\\right)=(k q, k p), k \\in \\mathbb{N}$. Since $y_{n}+z_{n}=n>p+q$, it follows that $k>1$. We observe that $x_{i}=x_{j}$ if and only if $A_{i} A_{j} \\| A_{0} A_{n}$. We shall show that such $i, j$ with $i1$. Such an index $j$ exists, since otherwise the game is over. Then one must make at least one move in the $j$ th cell, which implies that $x_{j}, x_{j}^{\\prime} \\geq 1$. However, then the sequences $\\left\\{x_{i}\\right\\}$ and $\\left\\{x_{i}^{\\prime}\\right\\}$ with $x_{j}$ and $x_{j}^{\\prime}$ decreased by 1 also satisfy (1) for a sequence $\\left\\{b_{i}\\right\\}$ where $b_{j-1}, b_{j}, b_{j+1}$ is replaced with $b_{j-1}+1, b_{j}-2, b_{j+1}+1$. This contradicts the assumption of minimal $\\min \\left\\{\\sum_{i \\in \\mathbb{Z}} x_{i}, \\sum_{i \\in \\mathbb{Z}} x_{i}^{\\prime}\\right\\}$ for the initial $\\left\\{b_{i}\\right\\}$.","problem_type":"Combinatorics","tier":0} +{"year":"1996","problem_phase":"shortlisted","problem":"3. A3 (GRE) Let $a>2$ be given, and define recursively $$ a_{0}=1, \\quad a_{1}=a, \\quad a_{n+1}=\\left(\\frac{a_{n}^{2}}{a_{n-1}^{2}}-2\\right) a_{n} $$ Show that for all $k \\in \\mathbb{N}$, we have $$ \\frac{1}{a_{0}}+\\frac{1}{a_{1}}+\\frac{1}{a_{2}}+\\cdots+\\frac{1}{a_{k}}<\\frac{1}{2}\\left(2+a-\\sqrt{a^{2}-4}\\right) . $$","solution":"3. Since $a_{1}>2$, it can be written as $a_{1}=b+b^{-1}$ for some $b>0$. Furthermore, $a_{1}^{2}-2=b^{2}+b^{-2}$ and hence $a_{2}=\\left(b^{2}+b^{-2}\\right)\\left(b+b^{-1}\\right)$. We prove that $$ a_{n}=\\left(b+b^{-1}\\right)\\left(b^{2}+b^{-2}\\right)\\left(b^{4}+b^{-4}\\right) \\cdots\\left(b^{2^{n-1}}+b^{-2^{n-1}}\\right) $$ by induction. Indeed, $\\frac{a_{n+1}}{a_{n}}=\\left(\\frac{a_{n}}{a_{n-1}}\\right)^{2}-2=\\left(b^{2^{n-1}}+b^{-2^{n-1}}\\right)^{2}-2=$ $b^{2^{n}}+b^{-2^{n}}$. Now we have $$ \\begin{aligned} \\sum_{i=1}^{n} \\frac{1}{a_{i}}= & 1+\\frac{b}{b^{2}+1}+\\frac{b^{3}}{\\left(b^{2}+1\\right)\\left(b^{4}+1\\right)}+\\cdots \\\\ & \\cdots+\\frac{b^{2^{n}-1}}{\\left(b^{2}+1\\right)\\left(b^{4}+1\\right) \\ldots\\left(b^{2}+1\\right)} \\end{aligned} $$ Note that $\\frac{1}{2}\\left(a+2-\\sqrt{a^{2}-4}\\right)=1+\\frac{1}{b}$; hence we must prove that the right side in (1) is less than $\\frac{1}{b}$. This follows from the fact that $$ \\begin{aligned} & \\frac{b^{2^{k}}}{\\left(b^{2}+1\\right)\\left(b^{4}+1\\right) \\cdots\\left(b^{2^{k}}+1\\right)} \\\\ & \\quad=\\frac{1}{\\left(b^{2}+1\\right)\\left(b^{4}+1\\right) \\cdots\\left(b^{2^{k-1}}+1\\right)}-\\frac{1}{\\left(b^{2}+1\\right)\\left(b^{4}+1\\right) \\cdots\\left(b^{2^{k}}+1\\right)} \\end{aligned} $$ hence the right side in (1) equals $\\frac{1}{b}\\left(1-\\frac{1}{\\left(b^{2}+1\\right)\\left(b^{4}+1\\right) \\ldots\\left(b^{2^{n}}+1\\right)}\\right)$, and this is clearly less than $1 \/ b$.","problem_type":"Algebra","tier":0} +{"year":"1996","problem_phase":"shortlisted","problem":"30. C7 (IRE) Let $U$ be a finite set and let $f, g$ be bijective functions from $U$ onto itself. Let $S=\\{w \\in U: f(f(w))=g(g(w))\\}, \\quad T=\\{w \\in U: f(g(w))=g(f(w))\\}$, and suppose that $U=S \\cup T$. Prove that for $w \\in U, f(w) \\in S$ if and only if $g(w) \\in S$.","solution":"30. For convenience, we shall write $f^{2}, f g, \\ldots$ for the functions $f \\circ f, f \\circ g, \\ldots$ We need two lemmas. Lemma 1. If $f(x) \\in S$ and $g(x) \\in T$, then $x \\in S \\cap T$. Proof. The given condition means that $f^{3}(x)=g^{2} f(x)$ and $g f g(x)=$ $f g^{2}(x)$. Since $x \\in S \\cup T=U$, we have two cases: $x \\in S$. Then $f^{2}(x)=g^{2}(x)$, which also implies $f^{3}(x)=f g^{2}(x)$. Therefore $g f g(x)=f g^{2}(x)=f^{3}(x)=g^{2} f(x)$, and since $g$ is a bijection, we obtain $f g(x)=g f(x)$, i.e., $x \\in T$. $x \\in T$. Then $f g(x)=g f(x)$, so $g^{2} f(x)=g f g(x)$. It follows that $f^{3}(x)=g^{2} f(x)=g f g(x)=f g^{2}(x)$, and since $f$ is a bijection, we obtain $x \\in S$. Hence $x \\in S \\cap T$ in both cases. Similarly, $f(x) \\in T$ and $g(x) \\in S$ again imply $x \\in S \\cap T$. Lemma 2. $f(S \\cap T)=g(S \\cap T)=S \\cap T$. Proof. By symmetry, it is enough to prove $f(S \\cap T)=S \\cap T$, or in other words that $f^{-1}(S \\cap T)=S \\cap T$. Since $S \\cap T$ is finite, this is equivalent to $f(S \\cap T) \\subseteq S \\cap T$. Let $f(x) \\in S \\cap T$. Then if $g(x) \\in S$ (since $f(x) \\in T$ ), Lemma 1 gives $x \\in S \\cap T$; similarly, if $g(x) \\in T$, then by Lemma $1, x \\in S \\cap T$. Now we return to the problem. Assume that $f(x) \\in S$. If $g(x) \\notin S$, then $g(x) \\in T$, so from Lemma 1 we deduce that $x \\in S \\cap T$. Then Lemma 2 claims that $g(x) \\in S \\cap T$ too, a contradiction. Analogously, from $g(x) \\in S$ we are led to $f(x) \\in S$. This finishes the proof.","problem_type":"Combinatorics","tier":0} +{"year":"1996","problem_phase":"shortlisted","problem":"4. A4 (KOR) Let $a_{1}, a_{2}, \\ldots, a_{n}$ be nonnegative real numbers, not all zero. (a) Prove that $x^{n}-a_{1} x^{n-1}-\\cdots-a_{n-1} x-a_{n}=0$ has precisely one positive real root. (b) Let $A=\\sum_{j=1}^{n} a_{j}, B=\\sum_{j=1}^{n} j a_{j}$, and let $R$ be the positive real root of the equation in part (a). Prove that $$ A^{A} \\leq R^{B} $$","solution":"4. Consider the function $$ f(x)=\\frac{a_{1}}{x}+\\frac{a_{2}}{x^{2}}+\\cdots+\\frac{a_{n}}{x^{n}} . $$ Since $f$ is strictly decreasing from $+\\infty$ to 0 on the interval $(0,+\\infty)$, there exists exactly one $R>0$ for which $f(R)=1$. This $R$ is also the only positive real root of the given polynomial. Since $\\ln x$ is a concave function on $(0,+\\infty)$, Jensen's inequality gives us $$ \\sum_{j=1}^{n} \\frac{a_{j}}{A}\\left(\\ln \\frac{A}{R^{j}}\\right) \\leq \\ln \\left(\\sum_{j=1}^{n} \\frac{a_{j}}{A} \\cdot \\frac{A}{R^{j}}\\right)=\\ln f(R)=0 . $$ Therefore $\\sum_{j=1}^{n} a_{j}(\\ln A-j \\ln R) \\leq 0$, which is equivalent to $A \\ln A \\leq$ $B \\ln R$, i.e., $A^{A} \\leq R^{B}$.","problem_type":"Algebra","tier":0} +{"year":"1996","problem_phase":"shortlisted","problem":"5. A5 (ROM) Let $P(x)$ be the real polynomial function $P(x)=a x^{3}+$ $b x^{2}+c x+d$. Prove that if $|P(x)| \\leq 1$ for all $x$ such that $|x| \\leq 1$, then $$ |a|+|b|+|c|+|d| \\leq 7 $$","solution":"5. Considering the polynomials $\\pm P( \\pm x)$ we may assume w.l.o.g. that $a, b \\geq$ 0 . We have four cases: (1) $c \\geq 0, d \\geq 0$. Then $|a|+|b|+|c|+|d|=a+b+c+d=P(1) \\leq 1$. (2) $c \\geq 0, d<0$. Then $|a|+|b|+|c|+|d|=a+b+c-d=P(1)-2 P(0) \\leq 3$. (3) $c<0, d \\geq 0$. Then $$ \\begin{aligned} |a|+|b|+|c|+|d| & =a+b-c+d \\\\ & =\\frac{4}{3} P(1)-\\frac{1}{3} P(-1)-\\frac{8}{3} P(1 \/ 2)+\\frac{8}{3} P(-1 \/ 2) \\leq 7 . \\end{aligned} $$ (4) $c<0, d<0$. Then $$ \\begin{aligned} |a|+|b|+|c|+|d| & =a+b-c-d \\\\ & =\\frac{5}{3} P(1)-4 P(1 \/ 2)+\\frac{4}{3} P(-1 \/ 2) \\leq 7 \\end{aligned} $$ Remark. It can be shown that the maximum of 7 is attained only for $P(x)= \\pm\\left(4 x^{3}-3 x\\right)$.","problem_type":"Algebra","tier":0} +{"year":"1996","problem_phase":"shortlisted","problem":"6. A6 (IRE) Let $n$ be an even positive integer. Prove that there exists a positive integer $k$ such that $$ k=f(x)(x+1)^{n}+g(x)\\left(x^{n}+1\\right) $$ for some polynomials $f(x), g(x)$ having integer coefficients. If $k_{0}$ denotes the least such $k$, determine $k_{0}$ as a function of $n$. A6 ${ }^{\\prime}$ Let $n$ be an even positive integer. Prove that there exists a positive integer $k$ such that $$ k=f(x)(x+1)^{n}+g(x)\\left(x^{n}+1\\right) $$ for some polynomials $f(x), g(x)$ having integer coefficients. If $k_{0}$ denotes the least such $k$, show that $k_{0}=2^{q}$, where $q$ is the odd integer determined by $n=q 2^{r}, r \\in \\mathbb{N}$. A6\" Prove that for each positive integer $n$, there exist polynomials $f(x), g(x)$ having integer coefficients such that $$ f(x)(x+1)^{2^{n}}+g(x)\\left(x^{2^{n}}+1\\right)=2 . $$","solution":"6. Let $f(x), g(x)$ be polynomials with integer coefficients such that $$ f(x)(x+1)^{n}+g(x)\\left(x^{n}+1\\right)=k_{0} $$ Write $n=2^{r} m$ for $m$ odd and note that $x^{n}+1=\\left(x^{2^{r}}+1\\right) B(x)$, where $B(x)=x^{2^{r}(m-1)}-x^{2^{r}(m-2)}+\\cdots-x^{2^{r}}+1$. Moreover, $B(-1)=1$; hence $B(x)-1=(x+1) c(x)$ and thus $$ R(x) B(x)+1=(B(x)-1)^{n}=(x+1)^{n} c(x)^{n} $$ for some polynomials $c(x)$ and $R(x)$. The zeros of the polynomial $x^{2^{r}}+1$ are $\\omega_{j}$, with $\\omega_{1}=\\cos \\frac{\\pi}{2^{r}}+i \\sin \\frac{\\pi}{2^{r}}$, and $\\omega_{j}=\\omega^{2 j-1}$ for $1 \\leq j \\leq 2^{r}$. We have $$ \\left(\\omega_{1}+1\\right)\\left(\\omega_{2}+1\\right) \\cdots\\left(\\omega_{2^{r+1}}+1\\right)=2 $$ From $(*)$ we also get $f\\left(\\omega_{j}\\right)\\left(\\omega_{j}+1\\right)^{n}=k_{0}$ for $j=1,2, \\ldots, 2^{r}$. Since $A=f\\left(\\omega_{1}\\right) f\\left(\\omega_{2}\\right) \\cdots f\\left(\\omega_{2^{r}}\\right)$ is a symmetric polynomial in $\\omega_{1}, \\ldots, \\omega_{2^{r}}$ with integer coefficients, $A$ is an integer. Consequently, taking the product over $j=1,2, \\ldots, 2^{r}$ and using (2) we deduce that $2^{n} A=k_{0}^{2^{r}}$ is divisible by $2^{n}=2^{2^{r} m}$. Hence $2^{m} \\mid k_{0}$. Furthermore, since $\\omega_{j}+1=\\left(\\omega_{1}+1\\right) p_{j}\\left(\\omega_{1}\\right)$ for some polynomial $p_{j}$ with integer coefficients, (2) gives $\\left(\\omega_{1}+1\\right)^{2^{r}} p\\left(\\omega_{1}\\right)=2$, where $p(x)=$ $p_{2}(x) \\cdots p_{2^{r}}(x)$ has integer coefficients. But then the polynomial $(x+$ $1)^{2^{2}} p(x)-2$ has a zero $x=\\omega_{1}$, so it is divisible by its minimal polynomial $x^{2^{r}}+1$. Therefore $$ (x+1)^{2^{r}} p(x)=2+\\left(x^{2^{r}}+1\\right) q(x) $$ for some polynomial $q(x)$. Raising (3) to the $m$ th power we get $(x+$ $1)^{n} p(x)^{n}=2^{m}+\\left(x^{2^{r}}+1\\right) Q(x)$ for some polynomial $Q(x)$ with integer coefficients. Now using (1) we obtain $$ \\begin{aligned} (x+1)^{n} c(x)^{n}\\left(x^{2^{r}}+1\\right) Q(x) & =\\left(x^{2^{r}}+1\\right) Q(x)+\\left(x^{2^{r}}+1\\right) Q(x) B(x) R(x) \\\\ & =(x+1)^{n} p(x)^{n}-2^{m}+\\left(x^{n}+1\\right) Q(X) R(x) . \\end{aligned} $$ Therefore $(x+1)^{n} f(x)+\\left(x^{n}+1\\right) g(x)=2^{m}$ for some polynomials $f(x), g(x)$ with integer coefficients, and $k_{0}=2^{m}$.","problem_type":"Algebra","tier":0} +{"year":"1996","problem_phase":"shortlisted","problem":"7. A7 (ARM) Let $f$ be a function from the set of real numbers $\\mathbb{R}$ into itself such that for all $x \\in \\mathbb{R}$, we have $|f(x)| \\leq 1$ and $$ f\\left(x+\\frac{13}{42}\\right)+f(x)=f\\left(x+\\frac{1}{6}\\right)+f\\left(x+\\frac{1}{7}\\right) . $$ Prove that $f$ is a periodic function (that is, there exists a nonzero real number $c$ such that $f(x+c)=f(x)$ for all $x \\in \\mathbb{R})$.","solution":"7. We are given that $f(x+a+b)-f(x+a)=f(x+b)-f(x)$, where $a=1 \/ 6$ and $b=1 \/ 7$. Summing up these equations for $x, x+b, \\ldots, x+6 b$ we obtain $f(x+a+1)-f(x+a)=f(x+1)-f(x)$. Summing up the new equations for $x, x+a, \\ldots, x+5 a$ we obtain that $$ f(x+2)-f(x+1)=f(x+1)-f(x) . $$ It follows by induction that $f(x+n)-f(x)=n[f(x+1)-f(x)]$. If $f(x+1) \\neq f(x)$, then $f(x+n)-f(x)$ will exceed in absolute value an arbitrarily large number for a sufficiently large $n$, contradicting the assumption that $f$ is bounded. Hence $f(x+1)=f(x)$ for all $x$.","problem_type":"Algebra","tier":0} +{"year":"1996","problem_phase":"shortlisted","problem":"8. A8 (ROM) ${ }^{\\mathrm{IMO} 3}$ Let $\\mathbb{N}_{0}$ denote the set of nonnegative integers. Find all functions $f$ from $\\mathbb{N}_{0}$ into itself such that $$ f(m+f(n))=f(f(m))+f(n), \\quad \\forall m, n \\in \\mathbb{N}_{0} $$","solution":"8. Putting $m=n=0$ we obtain $f(0)=0$ and consequently $f(f(n))=f(n)$ for all $n$. Thus the given functional equation is equivalent to $$ f(m+f(n))=f(m)+f(n), \\quad f(0)=0 $$ Clearly one solution is $(\\forall x) f(x)=0$. Suppose $f$ is not the zero function. We observe that $f$ has nonzero fixed points (for example, any $f(n)$ is a fixed point). Let $a$ be the smallest nonzero fixed point of $f$. By induction, each $k a(k \\in \\mathbb{N})$ is a fixed point too. We claim that all fixed points of $f$ are of this form. Indeed, suppose that $b=k a+i$ is a fixed point, where $i1$, $$ a(n)=a([n \/ 2])+(-1)^{\\frac{n(n+1)}{2}} . \\quad(\\text { Here }[t]=\\text { the greatest integer } \\leq t .) $$ (a) Determine the maximum and minimum value of $a(n)$ over $n \\leq 1996$ and find all $n \\leq 1996$ for which these extreme values are attained. (b) How many terms $a(n), n \\leq 1996$, are equal to 0 ?","solution":"9. From the definition of $a(n)$ we obtain $$ a(n)-a([n \/ 2])=\\left\\{\\begin{array}{r} 1 \\text { if } n \\equiv 0 \\text { or } n \\equiv 3(\\bmod 4) \\\\ -1 \\text { if } n \\equiv 1 \\text { or } n \\equiv 2(\\bmod 4) . \\end{array}\\right. $$ Let $n=\\overline{b_{k} b_{k-1} \\ldots b_{1} b_{0}}$ be the binary representation of $n$, where we assume $b_{k}=1$. If we define $p(n)$ and $q(n)$ to be the number of indices $i=0,1, \\ldots, k-1$ with $b_{i}=b_{i+1}$ and the number of $i=0,1, \\ldots, k-1$ with $b_{i} \\neq b_{i+1}$ respectively, we get $$ a(n)=p(n)-q(n) $$ (a) The maximum value of $a(n)$ for $n \\leq 1996$ is 9 when $p(n)=9$ and $q(n)=0$, i.e., in the case $n=\\overline{1111111111}_{2}=1023$. The minimum value is -10 and is attained when $p(n)=0$ and $q(n)=$ 10, i.e., only for $n=\\overline{10101010101}_{2}=1365$. (b) From (1) we have that $a(n)=0$ is equivalent to $p(n)=q(n)=k \/ 2$. Hence $k$ must be even, and the $k \/ 2$ indices $i$ for which $b_{i}=b_{i+1}$ can be chosen in exactly $\\binom{k}{k \/ 2}$ ways. Thus the number of positive integers $n<2^{11}=2048$ with $a(n)=0$ is equal to $$ \\binom{0}{0}+\\binom{2}{1}+\\binom{4}{2}+\\binom{6}{3}+\\binom{8}{4}+\\binom{10}{5}=351 $$ But five of these numbers exceed 1996: these are $2002=\\overline{11111010010}_{2}$, $2004=\\overline{11111010100}_{2}, 2006=\\overline{11111010110}_{2}, 2010=\\overline{11111011010}_{2}$, $2026=\\overline{11111101010}_{2}$. Therefore there are 346 numbers $n \\leq 1996$ for which $a(n)=0$.","problem_type":"Algebra","tier":0} +{"year":"1997","problem_phase":"shortlisted","problem":"1. (BLR) $)^{\\mathrm{IMO}} \\mathrm{An}$ infinite square grid is colored in the chessboard pattern. For any pair of positive integers $m, n$ consider a right-angled triangle whose vertices are grid points and whose legs, of lengths $m$ and $n$, run along the lines of the grid. Let $S_{b}$ be the total area of the black part of the triangle and $S_{w}$ the total area of its white part. Define the function $f(m, n)=\\left|S_{b}-S_{w}\\right|$. (a) Calculate $f(m, n)$ for all $m, n$ that have the same parity. (b) Prove that $f(m, n) \\leq \\frac{1}{2} \\max (m, n)$. (c) Show that $f(m, n)$ is not bounded from above.","solution":"1. Let $A B C$ be the given triangle, with $\\angle B=90^{\\circ}$ and $A B=m, B C=n$. For an arbitrary polygon $\\mathcal{P}$ we denote by $w(\\mathcal{P})$ and $b(\\mathcal{P})$ respectively the total areas of the white and black parts of $\\mathcal{P}$. (a) Let $D$ be the fourth vertex of the rectangle $A B C D$. When $m$ and $n$ are of the same parity, the coloring of the rectangle $A B C D$ is centrally symmetric with respect to the midpoint of $A C$. It follows that $w(A B C)=\\frac{1}{2} w(A B C D)$ and $b(A B C)=\\frac{1}{2} b(A B C D)$; thus $f(m, n)=\\frac{1}{2}|w(A B C D)-b(A B C D)|$. Hence $f(m, n)$ equals $\\frac{1}{2}$ if $m$ and $n$ are both odd, and 0 otherwise. (b) The result when $m, n$ are of the same parity follows from (a). Suppose that $m>n$, where $m$ and $n$ are of different parity. Choose a point $E$ on $A B$ such that $A E=1$. Since by (a) $|w(E B C)-b(E B C)|=$ $f(m-1, n) \\leq \\frac{1}{2}$, we have $f(m, n) \\leq \\frac{1}{2}+|w(E A C)-b(E A C)| \\leq$ $\\frac{1}{2}+S(E A C)=\\frac{1}{2}+\\frac{n-1}{2}=\\frac{n}{2}$. Therefore $f(m, n) \\leq \\frac{1}{2} \\min (m, n)$. (c) Let us calculate $f(m, n)$ for $m=2 k+1, n=2 k, k \\in \\mathbb{N}$. With $E$ defined as in (b), we have $B E=B C=2 k$. If the square at $B$ is w.l.o.g. white, $C E$ passes only through black squares. The white part of $\\triangle E A C$ then consists of $2 k$ similar triangles with areas $\\frac{1}{2} \\frac{i}{2 k} \\frac{i}{2 k+1}=\\frac{i^{2}}{4 k(2 k+1)}$, where $i=1,2, \\ldots, 2 k$. The total white area of $E A C$ is $$ \\frac{1}{4 k(2 k+1)}\\left(1^{2}+2^{2}+\\cdots+(2 k)^{2}\\right)=\\frac{4 k+1}{12} . $$ Therefore the black area is $(8 k-1) \/ 12$, and $f(2 k+1,2 k)=(2 k-1) \/ 6$, which is not bounded.","problem_type":null,"tier":0} +{"year":"1997","problem_phase":"shortlisted","problem":"10. ( $\\mathbf{C Z E}$ ) Find all positive integers $k$ for which the following statement is true: If $F(x)$ is a polynomial with integer coefficients satisfying the condition $$ 0 \\leq F(c) \\leq k \\quad \\text { for each } c \\in\\{0,1, \\ldots, k+1\\} $$ then $F(0)=F(1)=\\cdots=F(k+1)$.","solution":"10. Suppose that $k \\geq 4$. Consider any polynomial $F(x)$ with integer coefficients such that $0 \\leq F(x) \\leq k$ for $x=0,1, \\ldots, k+1$. Since $F(k+1)-F(0)$ is divisible by $k+1$, we must have $F(k+1)=F(0)$. Hence $$ F(x)-F(0)=x(x-k-1) Q(x) $$ for some polynomial $Q(x)$ with integer coefficients. In particular, $F(x)-$ $F(0)$ is divisible by $x(k+1-x)>k+1$ for every $x=2,3, \\ldots, k-1$, so $F(x)=F(0)$ must hold for any $x=2,3, \\ldots, k-1$. It follows that $$ F(x)-F(0)=x(x-2)(x-3) \\cdots(x-k+1)(x-k-1) R(x) $$ for some polynomial $R(x)$ with integer coefficients. Thus $k \\geq \\mid F(1)-$ $F(0)|=k(k-2)!| R(1) \\mid$, although $k(k-2)!>k$ for $k \\geq 4$. In this case we have $F(1)=F(0)$ and similarly $F(k)=F(0)$. Hence, the statement is true for $k \\geq 4$. It is easy to find counterexamples for $k \\leq 3$. These are, for example, $$ F(x)= \\begin{cases}x(2-x) & \\text { for } k=1 \\\\ x(3-x) & \\text { for } k=2 \\\\ x(2-x)^{2}(4-x) & \\text { for } k=3\\end{cases} $$","problem_type":null,"tier":0} +{"year":"1997","problem_phase":"shortlisted","problem":"11. (NET) Let $P(x)$ be a polynomial with real coefficients such that $P(x)>$ 0 for all $x \\geq 0$. Prove that there exists a positive integer $n$ such that $(1+x)^{n} P(x)$ is a polynomial with nonnegative coefficients.","solution":"11. All real roots of $P(x)$ (if any) are negative: say $-a_{1},-a_{2}, \\ldots,-a_{k}$. Then $P(x)$ can be factored as $$ P(x)=C\\left(x+a_{1}\\right) \\cdots\\left(x+a_{k}\\right)\\left(x^{2}-b_{1} x+c_{1}\\right) \\cdots\\left(x^{2}-b_{m} x+c_{m}\\right) $$ where $x^{2}-b_{i} x+c_{i}$ are quadratic polynomials without real roots. Since the product of polynomials with positive coefficients is again a polynomial with positive coefficients, it will be sufficient to prove the result for each of the factors in (1). The case of $x+a_{j}$ is trivial. It remains only to prove the claim for every polynomial $x^{2}-b x+c$ with $b^{2}<4 c$. From the binomial formula, we have for any $n \\in \\mathbb{N}$, $$ (1+x)^{n}\\left(x^{2}-b x+c\\right)=\\sum_{i=0}^{n+2}\\left[\\binom{n}{i-2}-b\\binom{n}{i-1}+c\\binom{n}{i}\\right] x^{i}=\\sum_{i=0}^{n+2} C_{i} x^{i} $$ where $$ C_{i}=\\frac{n!\\left((b+c+1) i^{2}-((b+2 c) n+(2 b+3 c+1)) i+c\\left(n^{2}+3 n+2\\right)\\right) x^{i}}{i!(n-i+2)!} $$ The coefficients $C_{i}$ of $x^{i}$ appear in the form of a quadratic polynomial in $i$ depending on $n$. We claim that for large enough $n$ this polynomial has negative discriminant, and is thus positive for every $i$. Indeed, this discriminant equals $D=((b+2 c) n+(2 b+3 c+1))^{2}-4(b+c+1) c\\left(n^{2}+\\right.$ $3 n+2)=\\left(b^{2}-4 c\\right) n^{2}-2 U n+V$, where $U=2 b^{2}+b c+b-4 c$ and $V=(2 b+c+1)^{2}-4 c$, and since $b^{2}-4 c<0$, for large $n$ it clearly holds that $D<0$.","problem_type":null,"tier":0} +{"year":"1997","problem_phase":"shortlisted","problem":"12. (ITA) Let $p$ be a prime number and let $f(x)$ be a polynomial of degree $d$ with integer coefficients such that: (i) $f(0)=0, f(1)=1$; (ii) for every positive integer $n$, the remainder of the division of $f(n)$ by $p$ is either 0 or 1. Prove that $d \\geq p-1$.","solution":"12. Lemma. For any polynomial $P$ of degree at most $n$, the following equality holds: $$ \\sum_{i=0}^{n+1}(-1)^{i}\\binom{n+1}{i} P(i)=0 $$ Proof. See (SL81-13). Suppose to the contrary that the degree of $f$ is at most $p-2$. Then it follows from the lemma that $$ 0=\\sum_{i=0}^{p-1}(-1)^{i}\\binom{p-1}{i} f(i) \\equiv \\sum_{i=0}^{p-1} f(i)(\\bmod p) $$ since $\\binom{p-1}{i}=\\frac{(p-1)(p-2) \\cdots(p-i)}{i!} \\equiv(-1)^{i}(\\bmod p)$. But this is clearly impossible if $f(i)$ equals 0 or 1 modulo $p$ and $f(0)=0, f(1)=1$. Remark. In proving the essential relation $\\sum_{i=0}^{p-1} f(i) \\equiv 0(\\bmod p)$, it is clearly enough to show that $S_{k}=1^{k}+2^{k}+\\cdots+(p-1)^{k}$ is divisible by $p$ for every $k \\leq p-2$. This can be shown in two other ways. (1) By induction. Assume that $S_{0} \\equiv \\cdots \\equiv S_{k-1}(\\bmod p)$. By the binomial formula we have $$ 0 \\equiv \\sum_{n=0}^{p-1}\\left[(n+1)^{k+1}-n^{k+1}\\right] \\equiv(k+1) S_{k}+\\sum_{i=0}^{k-1}\\binom{k+1}{i} S_{i}(\\bmod p), $$ and the inductive step follows. (2) Using the primitive root $g$ modulo $p$. Then $$ S_{k} \\equiv 1+g^{k}+\\cdots+g^{k(p-2)}=\\frac{g^{k(p-1)}-1}{g^{k}-1} \\equiv 0(\\bmod p) . $$","problem_type":null,"tier":0} +{"year":"1997","problem_phase":"shortlisted","problem":"13. (IND) In town $A$, there are $n$ girls and $n$ boys, and each girl knows each boy. In town $B$, there are $n$ girls $g_{1}, g_{2}, \\ldots, g_{n}$ and $2 n-1$ boys $b_{1}, b_{2}, \\ldots$, $b_{2 n-1}$. The girl $g_{i}, i=1,2, \\ldots, n$, knows the boys $b_{1}, b_{2}, \\ldots, b_{2 i-1}$, and no others. For all $r=1,2, \\ldots, n$, denote by $A(r), B(r)$ the number of different ways in which $r$ girls from town $A$, respectively town $B$, can dance with $r$ boys from their own town, forming $r$ pairs, each girl with a boy she knows. Prove that $A(r)=B(r)$ for each $r=1,2, \\ldots, n$.","solution":"13. Denote $A(r)$ and $B(r)$ by $A(n, r)$ and $B(n, r)$ respectively. The numbers $A(n, r)$ can be found directly: one can choose $r$ girls and $r$ boys in $\\binom{n}{r}^{2}$ ways, and pair them in $r$ ! ways. Hence $$ A(n, r)=\\binom{n}{r}^{2} \\cdot r!=\\frac{n!^{2}}{(n-r)!^{2} r!} $$ Now we establish a recurrence relation between the $B(n, r)$ 's. Let $n \\geq 2$ and $2 \\leq r \\leq n$. There are two cases for a desired selection of $r$ pairs of girls and boys: (i) One of the girls dancing is $g_{n}$. Then the other $r-1$ girls can choose their partners in $B(n-1, r-1)$ ways and $g_{n}$ can choose any of the remaining $2 n-r$ boys. Thus, the total number of choices in this case is $(2 n-r) B(n-1, r-1)$. (ii) $g_{n}$ is not dancing. Then there are exactly $B(n-1, r)$ possible choices. Therefore, for every $n \\geq 2$ it holds that $$ B(n, r)=(2 n-r) B(n-1, r-1)+B(n-1, r) \\quad \\text { for } r=2, \\ldots, n $$ Here we assume that $B(n, r)=0$ for $r>n$, while $B(n, 1)=1+3+\\cdots+$ $(2 n-1)=n^{2}$ \u3002 It is directly verified that the numbers $A(n, r)$ satisfy the same initial conditions and recurrence relations, from which it follows that $A(n, r)=$ $B(n, r)$ for all $n$ and $r \\leq n$.","problem_type":null,"tier":0} +{"year":"1997","problem_phase":"shortlisted","problem":"14. (IND) Let $b, m, n$ be positive integers such that $b>1$ and $m \\neq n$. Prove that if $b^{m}-1$ and $b^{n}-1$ have the same prime divisors, then $b+1$ is a power of 2 .","solution":"14. We use the following nonstandard notation: ( $1^{\\circ}$ ) for $x, y \\in \\mathbb{N}, x \\sim y$ means that $x$ and $y$ have the same prime divisors; $\\left(2^{\\circ}\\right)$ for a prime $p$ and integers $r \\geq 0$ and $x>0, p^{r} \\| x$ means that $x$ is divisible by $p^{r}$, but not by $p^{r+1}$. First, $b^{m}-1 \\sim b^{n}-1$ is obviously equivalent to $b^{m}-1 \\sim \\operatorname{gcd}\\left(b^{m}-1, b^{n}-\\right.$ $1)=b^{d}-1$, where $d=\\operatorname{gcd}(m, n)$. Setting $b^{d}=a$ and $m=k d$, we reduce the condition of the problem to $a^{k}-1 \\sim a-1$. We are going to show that this implies that $a+1$ is a power of 2 . This will imply that $d$ is odd (for even $d$, $a+1=b^{d}+1$ cannot be divisible by 4 , and consequently $b+1$, as a divisor of $a+1$, is also a power of 2 . But before that, we need the following important lemma (Theorem 2.126). Lemma. Let $a, k$ be positive integers and $p$ an odd prime. If $\\alpha \\geq 1$ and $\\beta \\geq 0$ are such that $p^{\\alpha} \\| a-1$ and $p^{\\beta} \\| k$, then $p^{\\alpha+\\beta} \\| a^{k}-1$. Proof. We use induction on $\\beta$. If $\\beta=0$, then $\\frac{a^{k}-1}{a-1}=a^{k-1}+\\cdots+a+1 \\equiv k$ $(\\bmod p)($ because $a \\equiv 1$ ), and it is not divisible by $p$. Suppose that the lemma is true for some $\\beta \\geq 0$, and let $k=p^{\\beta+1} t$ where $p \\nmid t$. By the induction hypothesis, $a^{k \/ p}=a^{p^{\\beta} t}=m p^{\\alpha+\\beta}+1$ for some $m$ not divisible by $p$. Furthermore, $$ a^{k}-1=\\left(m p^{\\alpha+\\beta}+1\\right)^{p}-1=\\left(m p^{\\alpha+\\beta}\\right)^{p}+\\cdots+\\binom{p}{2}\\left(m p^{\\alpha+\\beta}\\right)^{2}+m p^{\\alpha+\\beta+1} $$ Since $p \\left\\lvert\\,\\binom{ p}{2}=\\frac{p(p-1)}{2}\\right.$, all summands except for the last one are divisible by $p^{\\alpha+\\beta+2}$. Hence $p^{\\alpha+\\beta+1} \\| a^{k}-1$, completing the induction. Now let $a^{k}-1 \\sim a-1$ for some $a, k>1$. Suppose that $p$ is an odd prime divisor of $k$, with $p^{\\beta} \\| k$. Then putting $X=a^{p^{\\beta}-1}+\\cdots+a+1$ we also have $(a-1) X=a^{p^{\\beta}}-1 \\sim a-1$; hence each prime divisor $q$ of $X$ must also divide $a-1$. But then $a^{i} \\equiv 1(\\bmod q)$ for each $i \\in \\mathbb{N}_{0}$, which gives us $X \\equiv p^{\\beta}(\\bmod q)$. Therefore $q \\mid p^{\\beta}$, i.e., $q=p$; hence $X$ is a power of $p$. On the other hand, since $p \\mid a-1$, we put $p^{\\alpha} \\| a-1$. From the lemma we obtain $p^{\\alpha+\\beta} \\| a^{p^{\\beta}}-1$, and deduce that $p^{\\beta} \\| X$. But $X$ has no prime divisors other than $p$, so we must have $X=p^{\\beta}$. This is clearly impossible, because $X>p^{\\beta}$ for $a>1$. Thus our assumption that $k$ has an odd prime divisor leads to a contradiction: in other words, $k$ must be a power of 2 . Now $a^{k}-1 \\sim a-1$ implies $a-1 \\sim a^{2}-1=(a-1)(a+1)$, and thus every prime divisor $q$ of $a+1$ must also divide $a-1$. Consequently $q=2$, so it follows that $a+1$ is a power of 2 . As we explained above, this gives that $b+1$ is also a power of 2 . Remark. In fact, one can continue and show that $k$ must be equal to 2 . It is not possible for $a^{4}-1 \\sim a^{2}-1$ to hold. Similarly, we must have $d=1$. Therefore all possible triples $(b, m, n)$ with $m>n$ are $\\left(2^{s}-1,2,1\\right)$.","problem_type":null,"tier":0} +{"year":"1997","problem_phase":"shortlisted","problem":"15. (RUS) An infinite arithmetic progression whose terms are positive integers contains the square of an integer and the cube of an integer. Show that it contains the sixth power of an integer.","solution":"15. Let $a+b t, t=0,1,2, \\ldots$, be a given arithmetic progression that contains a square and a cube $(a, b>0)$. We use induction on the progression step $b$ to prove that the progression contains a sixth power. (i) $b=1$ : this case is trivial. (ii) $b=p^{m}$ for some prime $p$ and $m>0$. The case $p^{m} \\mid a$ trivially reduces to the previous case, so let us have $p^{m} \\nmid a$. Suppose that $\\operatorname{gcd}(a, p)=1$. If $x, y$ are integers such that $x^{2} \\equiv y^{3} \\equiv a$ (here all the congruences will be $\\bmod p^{m}$ ), then $x^{6} \\equiv a^{3}$ and $y^{6} \\equiv a^{2}$. Consider an integer $y_{1}$ such that $y y_{1} \\equiv 1$. It satisfies $a^{2}\\left(x y_{1}\\right)^{6} \\equiv$ $x^{6} y^{6} y_{1}^{6} \\equiv x^{6} \\equiv a^{3}$, and consequently $\\left(x y_{1}\\right)^{6} \\equiv a$. Hence a sixth power exists in the progression. If $\\operatorname{gcd}(a, p)>1$, we can write $a=p^{k} c$, where $k1$ and $\\operatorname{gcd}\\left(b_{1}, b_{2}\\right)=1$. It is given that progressions $a+b_{1} t$ and $a+b_{2} t$ both contain a square and a cube, and therefore by the inductive hypothesis they both contain sixth powers: say $z_{1}^{6}$ and $z_{2}^{6}$, respectively. By the Chinese remainder theorem, there exists $z \\in \\mathbb{N}$ such that $z \\equiv z_{1}\\left(\\bmod b_{1}\\right)$ and $z \\equiv z_{2}\\left(\\bmod b_{2}\\right)$. But then $z^{6}$ belongs to both of the progressions $a+b_{1} t$ and $a+b_{2} t$. Hence $z^{6}$ is a member of the progression $a+b t$.","problem_type":null,"tier":0} +{"year":"1997","problem_phase":"shortlisted","problem":"16. (BLR) In an acute-angled triangle $A B C$, let $A D, B E$ be altitudes and $A P, B Q$ internal bisectors. Denote by $I$ and $O$ the incenter and the circumcenter of the triangle, respectively. Prove that the points $D, E$, and $I$ are collinear if and only if the points $P, Q$, and $O$ are collinear.","solution":"16. Let $d_{a}(X), d_{b}(X), d_{c}(X)$ denote the distances of a point $X$ interior to $\\triangle A B C$ from the lines $B C, C A, A B$ respectively. We claim that $X \\in P Q$ if and only if $d_{a}(X)+d_{b}(X)=d_{c}(X)$. Indeed, if $X \\in P Q$ and $P X=$ $k P Q$ then $d_{a}(X)=k d_{a}(Q), d_{b}(X)=(1-k) d_{b}(P)$, and $d_{c}(X)=(1-$ $k) d_{c}(P)+k d_{c}(Q)$, and simple substitution yields $d_{a}(X)+d_{b}(X)=d_{c}(X)$. The converse follows easily. In particular, $O \\in P Q$ if and only if $d_{a}(O)+$ $d_{b}(O)=d_{c}(O)$, i.e., $\\cos \\alpha+\\cos \\beta=\\cos \\gamma$. We shall now show that $I \\in D E$ if and only if $A E+B D=D E$. Let $K$ be the point on the segment $D E$ such that $A E=E K$. Then $\\angle E K A=$ $\\frac{1}{2} \\angle D E C=\\frac{1}{2} \\angle C B A=\\angle I B A$; hence the points $A, B, I, K$ are concyclic. The point $I$ lies on $D E$ if and only if $\\angle B K D=\\angle B A I=\\frac{1}{2} \\angle B A C=$ $\\frac{1}{2} \\angle C D E=\\angle D B K$, which is equivalent to $K D=B D$, i.e., to $A E+B D=$ $D E$. But since $A E=A B \\cos \\alpha, B D=A B \\cos \\beta$, and $D E=A B \\cos \\gamma$, we have that $I \\in D E \\Leftrightarrow \\cos \\alpha+\\cos \\beta=\\cos \\gamma$. The conditions for $O \\in P Q$ and $I \\in D E$ are thus equivalent. Second solution. We know that three points $X, Y, Z$ are collinear if and only if for some $\\lambda, \\mu \\in \\mathbb{R}$ with sum 1 , we have $\\lambda \\overrightarrow{C X}+\\mu \\overrightarrow{C Y}=\\overrightarrow{C Z}$. Specially, if $\\overrightarrow{C X}=p \\overrightarrow{C A}$ and $\\overrightarrow{C Y}=q \\overrightarrow{C B}$ for some $p, q$, and $\\overrightarrow{C Z}=k \\overrightarrow{C A}+$ $l \\overrightarrow{C B}$, then $Z$ lies on $X Y$ if and only if $k q+l p=p q$. Using known relations in a triangle we directly obtain $$ \\begin{array}{rlrl} \\overrightarrow{C P} & =\\frac{\\sin \\beta}{\\sin \\beta+\\sin \\gamma} \\overrightarrow{C B}, & \\overrightarrow{C Q}=\\frac{\\sin \\alpha}{\\sin \\alpha+\\sin \\gamma} \\overrightarrow{C A} \\\\ \\overrightarrow{C O} & =\\frac{\\sin 2 \\alpha \\cdot \\overrightarrow{C A}+\\sin 2 \\beta \\cdot \\overrightarrow{C B}}{\\sin 2 \\alpha+\\sin 2 \\beta+\\sin 2 \\gamma} ; & \\overrightarrow{C D}=\\frac{\\tan \\beta}{\\tan \\beta+\\tan \\gamma} \\overrightarrow{C B} \\\\ \\overrightarrow{C E} & =\\frac{\\tan \\beta}{\\tan \\beta+\\tan \\gamma} \\overrightarrow{C A}, & \\overrightarrow{C I} & =\\frac{\\sin \\alpha \\cdot \\overrightarrow{C A}+\\sin \\beta \\cdot \\overrightarrow{C B}}{\\sin \\alpha+\\sin \\beta+\\sin \\gamma} \\end{array} $$ Now by the above considerations we get that the conditions (1) $P, Q, O$ are collinear and (2) $D, E, I$ are collinear are both equivalent to $\\cos \\alpha+\\cos \\beta=$ $\\cos \\gamma$.","problem_type":null,"tier":0} +{"year":"1997","problem_phase":"shortlisted","problem":"17. $(\\mathbf{C Z E})^{\\mathrm{IMO} 5}$ Find all pairs of integers $x, y \\geq 1$ satisfying the equation $x^{y^{2}}=y^{x}$.","solution":"17. We note first that $x$ and $y$ must be powers of the same positive integer. Indeed, if $x=p_{1}^{\\alpha_{1}} \\cdots p_{k}^{\\alpha_{k}}$ and $y=p_{1}^{\\beta_{1}} \\cdots p_{k}^{\\beta_{k}}$ (some of $\\alpha_{i}$ and $\\beta_{i}$ may be 0 , but not both for the same index $i$ ), then $x^{y^{2}}=y^{x}$ implies $\\frac{\\alpha_{i}}{\\beta_{i}}=\\frac{x}{y^{2}}=\\frac{p}{q}$ for some $p, q>0$ with $\\operatorname{gcd}(p, q)=1$, so for $a=p_{1}^{\\alpha_{1} \/ p} \\cdots p_{k}^{\\alpha_{k} \/ p}$ we can take $x=a^{p}$ and $y=a^{q}$. If $a=1$, then $(x, y)=(1,1)$ is the trivial solution. Let $a>1$. The given equation becomes $a^{p a^{2 q}}=a^{q a^{p}}$, which reduces to $p a^{2 q}=q a^{p}$. Hence $p \\neq q$, so we distinguish two cases: (i) $p>q$. Then from $a^{2 q}2 q$. We can rewrite the equation as $p=a^{p-2 q} q$, and putting $p=2 q+d, d>0$, we obtain $d=q\\left(a^{d}-2\\right)$. By induction, $2^{d}-2>d$ for each $d>2$, so we must have $d \\leq 2$. For $d=1$ we get $q=1$ and $a=p=3$, and therefore $(x, y)=(27,3)$, which is indeed a solution. For $d=2$ we get $q=1$, $a=2$, and $p=4$, so $(x, y)=(16,2)$, which is another solution. (ii) $p0$, this is transformed to $a^{d}=a^{\\left(2 a^{d}-1\\right) p}$, or equivalently to $d=\\left(2 a^{d}-1\\right) p$. However, this equality cannot hold, because $2 a^{d}-1>d$ for each $a \\geq 2$, $d \\geq 1$. The only solutions are thus $(1,1),(16,2)$, and $(27,3)$.","problem_type":null,"tier":0} +{"year":"1997","problem_phase":"shortlisted","problem":"18. (GBR) The altitudes through the vertices $A, B, C$ of an acute-angled triangle $A B C$ meet the opposite sides at $D, E, F$, respectively. The line through $D$ parallel to $E F$ meets the lines $A C$ and $A B$ at $Q$ and $R$, respectively. The line $E F$ meets $B C$ at $P$. Prove that the circumcircle of the triangle $P Q R$ passes through the midpoint of $B C$.","solution":"18. By symmetry, assume that $A B>A C$. The point $D$ lies between $M$ and $P$ as well as between $Q$ and $R$, and if we show that $D M \\cdot D P=D Q \\cdot D R$, it will imply that $M, P, Q, R$ lie on a circle. Since the triangles $A B C, A E F, A Q R$ are similar, the points $B, C, Q, R$ lie on a circle. Hence $D B \\cdot D C=D Q \\cdot D R$, and it remains to prove that $$ D B \\cdot D C=D M \\cdot D P $$ However, the points $B, C, E, F$ are concyclic, but so are the points $E, F, D, M$ (they lie on the nine-point circle), and we obtain $P B \\cdot P C=$ $P E \\cdot P F=P D \\cdot P M$. Set $P B=x$ and $P C=y$. We have $P M=\\frac{x+y}{2}$ and hence $P D=\\frac{2 x y}{x+y}$. It follows that $D B=P B-P D=\\frac{x(x-y)}{x+y}$, $D C=\\frac{y(x-y)}{x+y}$, and $D M=\\frac{(x-y)^{2}}{2(x+y)}$, from which we immediately obtain $D B \\cdot D C=D M \\cdot D P=\\frac{x y(x-y)^{2}}{(x+y)^{2}}$, as needed.","problem_type":null,"tier":0} +{"year":"1997","problem_phase":"shortlisted","problem":"19. (IRE) Let $a_{1} \\geq \\cdots \\geq a_{n} \\geq a_{n+1}=0$ be a sequence of real numbers. Prove that $$ \\sqrt{\\sum_{k=1}^{n} a_{k}} \\leq \\sum_{k=1}^{n} \\sqrt{k}\\left(\\sqrt{a_{k}}-\\sqrt{a_{k+1}}\\right) $$","solution":"19. Using that $a_{n+1}=0$ we can transform the desired inequality into $$ \\begin{aligned} & \\sqrt{a_{1}+}+a_{2}+\\cdots+a_{n+1} \\\\ & \\quad \\leq \\sqrt{1} \\sqrt{a_{1}}+(\\sqrt{2}-\\sqrt{1}) \\sqrt{a_{2}}+\\cdots+(\\sqrt{n+1}-\\sqrt{n}) \\sqrt{a_{n+1}} \\end{aligned} $$ We shall prove by induction on $n$ that (1) holds for any $a_{1} \\geq a_{2} \\geq \\cdots \\geq$ $a_{n+1} \\geq 0$, i.e., not only when $a_{n+1}=0$. For $n=0$ the inequality is obvious. For the inductive step from $n-1$ to $n$, where $n \\geq 1$, we need to prove the inequality $$ \\sqrt{a_{1}+\\cdots+a_{n+1}}-\\sqrt{a_{1}+\\cdots+a_{n}} \\leq(\\sqrt{n+1}-\\sqrt{n}) \\sqrt{a_{n+1}} $$ Putting $S=a_{1}+a_{2}+\\cdots+a_{n}$, this simplifies to $\\sqrt{S+a_{n+1}}-\\sqrt{S} \\leq$ $\\sqrt{n a_{n+1}+a_{n+1}}-\\sqrt{n a_{n+1}}$. For $a_{n+1}=0$ the inequality is obvious. For $a_{n+1}>0$ we have that the function $f(x)=\\sqrt{x+a_{n+1}}-\\sqrt{x}=$ $\\frac{a_{n+1}}{\\sqrt{x+a_{n+1}}+\\sqrt{x}}$ is strictly decreasing on $\\mathbb{R}^{+}$; hence (2) will follow if we show that $S \\geq n a_{n+1}$. However, this last is true because $a_{1}, \\ldots, a_{n} \\geq a_{n+1}$. Equality holds if and only if $a_{1}=a_{2}=\\cdots=a_{k}$ and $a_{k+1}=\\cdots=a_{n+1}=$ 0 for some $k$. Second solution. Setting $b_{k}=\\sqrt{a_{k}}-\\sqrt{a_{k+1}}$ for $k=1, \\ldots, n$ we have $a_{i}=\\left(b_{i}+\\cdots+b_{n}\\right)^{2}$, so the desired inequality after squaring becomes $$ \\sum_{k=1}^{n} k b_{k}^{2}+2 \\sum_{1 \\leq k1$, then the $k$ th term from the left in $R_{n}$ is equal to 1 if and only if the $k$ th term from the right in $R_{n}$ is different from 1.","solution":"2. For any sequence $X=\\left(x_{1}, x_{2}, \\ldots, x_{n}\\right)$ let us define $$ \\bar{X}=\\left(1,2, \\ldots, x_{1}, 1,2, \\ldots, x_{2}, \\ldots, 1,2, \\ldots, x_{n}\\right) $$ Also, for any two sequences $A, B$ we denote their concatenation by $A B$. It clearly holds that $\\overline{A B}=\\bar{A} \\bar{B}$. The sequences $R_{1}, R_{2}, \\ldots$ are given by $R_{1}=(1)$ and $R_{n}=\\overline{R_{n-1}}(n)$ for $n>1$. We consider the family of sequences $Q_{n i}$ for $n, i \\in \\mathbb{N}, i \\leq n$, defined as follows: $Q_{n 1}=(1), \\quad Q_{n n}=(n), \\quad$ and $\\quad Q_{n i}=Q_{n-1, i-1} Q_{n-1, i} \\quad$ if $1\\frac{n+1}{2}$ for any $\\pi$. Further, we note that if $\\pi^{\\prime}$ is obtained from $\\pi$ by interchanging two neighboring elements, say $y_{k}$ and $y_{k+1}$, then $S(\\pi)$ and $S\\left(\\pi^{\\prime}\\right)$ differ by $\\left|y_{k}+y_{k+1}\\right| \\leq n+1$, and consequently they must be of the same sign. Now consider the identity permutation $\\pi_{0}=\\left(x_{1}, \\ldots, x_{n}\\right)$ and the reverse permutation $\\overline{\\pi_{0}}=\\left(x_{n}, \\ldots, x_{1}\\right)$. There is a sequence of permutations $\\pi_{0}, \\pi_{1}, \\ldots, \\pi_{m}=\\overline{\\pi_{0}}$ such that for each $i, \\pi_{i+1}$ is obtained from $\\pi_{i}$ by interchanging two neighboring elements. Indeed, by successive interchanges we can put $x_{n}$ in the first place, then $x_{n-1}$ in the second place, etc. Hence all $S\\left(\\pi_{0}\\right), \\ldots, S\\left(\\pi_{m}\\right)$ are of the same sign. However, since $\\left|S\\left(\\pi_{0}\\right)+S\\left(\\pi_{m}\\right)\\right|=(n+1)\\left|x_{1}+\\cdots+x_{n}\\right|=n+1$, this implies that one of $S\\left(\\pi_{0}\\right)$ and $S\\left(\\overline{\\pi_{0}}\\right)$ is smaller than $\\frac{n+1}{2}$ in absolute value, contradicting the initial assumption.","problem_type":null,"tier":0} +{"year":"1997","problem_phase":"shortlisted","problem":"22. (UKR) (a) Do there exist functions $f: \\mathbb{R} \\rightarrow \\mathbb{R}$ and $g: \\mathbb{R} \\rightarrow \\mathbb{R}$ such that $$ f(g(x))=x^{2} \\quad \\text { and } \\quad g(f(x))=x^{3} \\quad \\text { for all } x \\in \\mathbb{R} ? $$ (b) Do there exist functions $f: \\mathbb{R} \\rightarrow \\mathbb{R}$ and $g: \\mathbb{R} \\rightarrow \\mathbb{R}$ such that $$ f(g(x))=x^{2} \\quad \\text { and } \\quad g(f(x))=x^{4} \\quad \\text { for all } x \\in \\mathbb{R} ? $$","solution":"22. (a) Suppose that $f$ and $g$ are such functions. From $g(f(x))=x^{3}$ we have $f\\left(x_{1}\\right) \\neq f\\left(x_{2}\\right)$ whenever $x_{1} \\neq x_{2}$. In particular, $f(-1), f(0)$, and $f(1)$ are three distinct numbers. However, since $f(x)^{2}=f(g(f(x)))=$ $f\\left(x^{3}\\right)$, each of the numbers $f(-1), f(0), f(1)$ is equal to its square, and so must be either 0 or 1 . This contradiction shows that no such $f, g$ exist. (b) The answer is yes. We begin with constructing functions $F, G:(1, \\infty)$ $\\rightarrow(1, \\infty)$ with the property $F(G(x))=x^{2}$ and $G(F(x))=x^{4}$ for $x>$ 1. Define the functions $\\varphi, \\psi$ by $F\\left(2^{2^{t}}\\right)=2^{2^{\\varphi(t)}}$ and $G\\left(2^{2^{t}}\\right)=2^{2^{\\psi(t)}}$. These functions determine $F$ and $G$ on the entire interval $(1, \\infty)$, and satisfy $\\varphi(\\psi(t))=t+1$ and $\\psi(\\varphi(t))=t+2$. It is easy to find examples of $\\varphi$ and $\\psi$ : for example, $\\varphi(t)=\\frac{1}{2} t+1, \\psi(t)=2 t$. Thus we also arrive at an example for $F, G$ : $$ F(x)=2^{2^{\\frac{1}{2} \\log _{2} \\log _{2} x+1}}=2^{2 \\sqrt{\\log _{2} x}}, \\quad G(x)=2^{2^{2 \\log _{2} \\log _{2} x}}=2^{\\log _{2}^{2} x} $$ It remains only to extend these functions to the whole of $\\mathbb{R}$. This can be done as follows: $$ \\widetilde{f}(x)= \\begin{cases}F(x) & \\text { for } x>1 \\\\ 1 \/ F(1 \/ x) & \\text { for } 0 1 } \\\\ { x } & { \\text { for } x \\in \\{ 0 , 1 \\} ; } \\end{array} \\left\\{\\begin{array}{cl} 1 \/ G(1 \/ x) & \\text { for } 0\\beta_{4}$. Then point $A$ lies inside the circle $B C D$, which is further equivalent to $\\beta_{1}>\\alpha_{2}$. On the other hand, from $\\alpha_{1}+\\beta_{2}=\\alpha_{3}+\\beta_{4}$ we deduce $\\alpha_{3}>\\beta_{2}$, and similarly $\\beta_{3}>\\alpha_{4}$. Therefore, since the cosine is strictly decreasing on $(0, \\pi)$, the left side of (1) is strictly negative, yielding a contradiction.","problem_type":null,"tier":0} +{"year":"1997","problem_phase":"shortlisted","problem":"24. (LIT) ${ }^{\\mathrm{IMO} 6}$ For a positive integer $n$, let $f(n)$ denote the number of ways to represent $n$ as the sum of powers of 2 with nonnegative integer exponents. Representations that differ only in the ordering in their summands are not considered to be distinct. (For instance, $f(4)=4$ because the number 4 can be represented in the following four ways: $4 ; 2+2 ; 2+1+1 ; 1+1+1+1$.) Prove that the inequality $$ 2^{n^{2} \/ 4}(2 m-1) f(2 m)$, which together with the above inequality gives $$ f(8 m)=f(0)+f(1)+\\cdots+f(4 m)>4 m f(2 m) . $$ Finally, we have that the inequality $f\\left(2^{n}\\right)>2^{n^{2} \/ 4}$ holds for $n=2$ and $n=3$, while for larger $n$ we have by induction $f\\left(2^{n}\\right)>2^{n-1} f\\left(2^{n-2}\\right)>$ $2^{n-1+(n-2)^{2} \/ 4}=2^{n^{2} \/ 4}$. This completes the proof. Remark. Despite the fact that the lower estimate is more difficult, it is much weaker than the upper estimate. It can be shown that $f\\left(2^{n}\\right)$ eventually (for large $n$ ) exceeds $2^{c n^{2}}$ for any $c<\\frac{1}{2}$.","problem_type":null,"tier":0} +{"year":"1997","problem_phase":"shortlisted","problem":"25. (POL) The bisectors of angles $A, B, C$ of a triangle $A B C$ meet its circumcircle again at the points $K, L, M$, respectively. Let $R$ be an internal point on the side $A B$. The points $P$ and $Q$ are defined by the following conditions: $R P$ is parallel to $A K$, and $B P$ is perpendicular to $B L ; R Q$ is parallel to $B L$, and $A Q$ is perpendicular to $A K$. Show that the lines $K P, L Q, M R$ have a point in common.","solution":"25. Let $M R$ meet the circumcircle of triangle $A B C$ again at a point $X$. We claim that $X$ is the common point of the lines $K P, L Q, M R$. By symmetry, it will be enough to show that $X$ lies on $K P$. It is easy to see that $X$ and $P$ lie on the same side of $A B$ as $K$. Let $I_{a}=A K \\cap B P$ be the excenter of $\\triangle A B C$ corresponding to $A$. It is easy to calculate that $\\angle A I_{a} B=\\gamma \/ 2$, from which we get $\\angle R P B=\\angle A I_{a} B=\\angle M C B=\\angle R X B$. Therefore $R, B, P, X$ are concyclic. Now if $P$ and $K$ are on distinct sides of $B X$ (the other case is similar), we have $\\angle R X P=180^{\\circ}-\\angle R B P=90^{\\circ}-$ $\\beta \/ 2=\\angle M A K=180^{\\circ}-\\angle R X K$, from which it follows that $K, X, P$ are collinear, as claimed. Remark. It is not essential for the statement of the problem that $R$ be an internal point of $A B$. Work with cases can be avoided using oriented ![](https:\/\/cdn.mathpix.com\/cropped\/2024_11_18_8e985d6b9c83aa3e9d0eg-640.jpg?height=399&width=439&top_left_y=248&top_left_x=858) angles.","problem_type":null,"tier":0} +{"year":"1997","problem_phase":"shortlisted","problem":"26. (ITA) For every integer $n \\geq 2$ determine the minimum value that the sum $a_{0}+a_{1}+\\cdots+a_{n}$ can take for nonnegative numbers $a_{0}, a_{1}, \\ldots, a_{n}$ satisfying the condition $$ a_{0}=1, \\quad a_{i} \\leq a_{i+1}+a_{i+2} \\quad \\text { for } i=0, \\ldots, n-2 $$ ### 3.39 The Thirty-Ninth IMO","solution":"26. Let us first examine the case that all the inequalities in the problem are actually equalities. Then $a_{n-2}=a_{n-1}+a_{n}, a_{n-3}=2 a_{n-1}+a_{n}, \\ldots, a_{0}=$ $F_{n} a_{n-1}+F_{n-1} a_{n}=1$, where $F_{n}$ is the $n$th Fibonacci number. Then it is easy to see (from $F_{1}+F_{2}+\\cdots+F_{k}=F_{k+2}$ ) that $a_{0}+\\cdots+a_{n}=$ $\\left(F_{n+2}-1\\right) a_{n-1}+F_{n+1} a_{n}=\\frac{F_{n+2}-1}{F_{n}}+\\left(F_{n+1}-\\frac{F_{n-1}\\left(F_{n+2}-1\\right)}{F_{n}}\\right) a_{n}$. Since $\\frac{F_{n-1}\\left(F_{n+2}-1\\right)}{F_{n}} \\leq F_{n+1}$, it follows that $a_{0}+a_{1}+\\cdots+a_{n} \\geq \\frac{F_{n+2}-1}{F_{n}}$, with equality holding if and only if $a_{n}=0$ and $a_{n-1}=\\frac{1}{F_{n}}$. We denote by $M_{n}$ the required minimum in the general case. We shall prove by induction that $M_{n}=\\frac{F_{n+2}-1}{F_{n}}$. For $M_{1}=1$ and $M_{2}=2$ it is easy to show that the formula holds; hence the inductive basis is true. Suppose that $n>2$. The sequences $1, \\frac{a_{2}}{a_{1}}, \\ldots, \\frac{a_{n}}{a_{1}}$ and $1, \\frac{a_{3}}{a_{2}}, \\ldots, \\frac{a_{n}}{a_{2}}$ also satisfy the conditions of the problem. Hence we have $$ a_{0}+\\cdots+a_{n}=a_{0}+a_{1}\\left(1+\\frac{a_{2}}{a_{1}}+\\cdots+\\frac{a_{n}}{a_{1}}\\right) \\geq 1+a_{1} M_{n-1} $$ and $$ a_{0}+\\cdots+a_{n}=a_{0}+a_{1}+a_{2}\\left(1+\\frac{a_{3}}{a_{2}}+\\cdots+\\frac{a_{n}}{a_{2}}\\right) \\geq 1+a_{1}+a_{2} M_{n-2} $$ Multiplying the first inequality by $M_{n-2}-1$ and the second one by $M_{n-1}$, adding the inequalities and using that $a_{1}+a_{2} \\geq 1$, we obtain ( $M_{n-1}+$ $\\left.M_{n-2}+1\\right)\\left(a_{0}+\\cdots+a_{n}\\right) \\geq M_{n-1} M_{n-2}+M_{n-1}+M_{n-2}+1$, so $$ M_{n} \\geq \\frac{M_{n-1} M_{n-2}+M_{n-1}+M_{n-2}+1}{M_{n-1}+M_{n-2}+1} $$ Since $M_{n-1}=\\frac{F_{n+1}-1}{F_{n-1}}$ and $M_{n-2}=\\frac{F_{n}-1}{F_{n-2}}$, the above inequality easily yields $M_{n} \\geq \\frac{F_{n+2}-1}{F_{n}}$. However, we have shown above that equality can occur; hence $\\frac{F_{n+2}-1}{F_{n}}$ is indeed the required minimum.","problem_type":null,"tier":0} +{"year":"1997","problem_phase":"shortlisted","problem":"3. (GER) For each finite set $U$ of nonzero vectors in the plane we define $l(U)$ to be the length of the vector that is the sum of all vectors in $U$. Given a finite set $V$ of nonzero vectors in the plane, a subset $B$ of $V$ is said to be maximal if $l(B)$ is greater than or equal to $l(A)$ for each nonempty subset $A$ of $V$. (a) Construct sets of 4 and 5 vectors that have 8 and 10 maximal subsets respectively. (b) Show that for any set $V$ consisting of $n \\geq 1$ vectors, the number of maximal subsets is less than or equal to $2 n$.","solution":"3. (a) For $n=4$, consider a convex quadrilateral $A B C D$ in which $A B=$ $B C=A C=B D$ and $A D=D C$, and take the vectors $\\overrightarrow{A B}, \\overrightarrow{B C}$, $\\overrightarrow{C D}, \\overrightarrow{D A}$. For $n=5$, take the vectors $\\overrightarrow{A B}, \\overrightarrow{B C}, \\overrightarrow{C D}, \\overrightarrow{D E}, \\overrightarrow{E A}$ for any regular pentagon $A B C D E$. (b) Let us draw the vectors of $V$ as originated from the same point $O$. Consider any maximal subset $B \\subset V$, and denote by $u$ the sum of all vectors from $B$. If $l$ is the line through $O$ perpendicular to $u$, then $B$ contains exactly those vectors from $V$ that lie on the same side of $l$ as $u$ does, and no others. Indeed, if any $v \\notin B$ lies on the same side of $l$, then $|u+v| \\geq|u|$; similarly, if some $v \\in B$ lies on the other side of $l$, then $|u-v| \\geq|u|$. Therefore every maximal subset is determined by some line $l$ as the set of vectors lying on the same side of $l$. It is obvious that in this way we get at most $2 n$ sets.","problem_type":null,"tier":0} +{"year":"1997","problem_phase":"shortlisted","problem":"4. (IRN) ${ }^{\\mathrm{IMO} 4} \\mathrm{An} n \\times n$ matrix with entries from $\\{1,2, \\ldots, 2 n-1\\}$ is called a coveralls matrix if for each $i$ the union of the $i$ th row and the $i$ th column contains $2 n-1$ distinct entries. Show that: (a) There exist no coveralls matrices for $n=1997$. (b) Coveralls matrices exist for infinitely many values of $n$.","solution":"4. (a) Suppose that an $n \\times n$ coveralls matrix $A$ exists for some $n>1$. Let $x \\in\\{1,2, \\ldots, 2 n-1\\}$ be a fixed number that does not appear on the fixed diagonal of $A$. Such an element must exist, since the diagonal can contain at most $n$ different numbers. Let us call the union of the $i$ th row and the $i$ th column the $i$ th cross. There are $n$ crosses, and each of them contains exactly one $x$. On the other hand, each entry $x$ of $A$ is contained in exactly two crosses. Hence $n$ must be even. However, 1997 is an odd number; hence no coveralls matrix exists for $n=1997$. (b) For $n=2, A_{2}=\\left[\\begin{array}{ll}1 & 2 \\\\ 3 & 1\\end{array}\\right]$ is a coveralls matrix. For $n=4$, one such matrix is, for example, $$ A_{4}=\\left[\\begin{array}{llll} 1 & 2 & 5 & 6 \\\\ 3 & 1 & 7 & 5 \\\\ 4 & 6 & 1 & 2 \\\\ 7 & 4 & 3 & 1 \\end{array}\\right] $$ This construction can be generalized. Suppose that we are given an $n \\times n$ coveralls matrix $A_{n}$. Let $B_{n}$ be the matrix obtained from $A_{n}$ by adding $2 n$ to each entry, and $C_{n}$ the matrix obtained from $B_{n}$ by replacing each diagonal entry (equal to $2 n+1$ by induction) with $2 n$. Then the matrix $$ A_{2 n}=\\left[\\begin{array}{ll} A_{n} & B_{n} \\\\ C_{n} & A_{n} \\end{array}\\right] $$ is coveralls. To show this, suppose that $i \\leq n$ (the case $i>n$ is similar). The $i$ th cross is composed of the $i$ th cross of $A_{n}$, the $i$ th row of $B_{n}$, and the $i$ th column of $C_{n}$. The $i$ th cross of $A_{i}$ covers $1,2, \\ldots, 2 n-1$. The $i$ th row of $B_{n}$ covers all numbers of the form $2 n+j$, where $j$ is covered by the $i$ th row of $A_{n}$ (including $j=1$ ). Similarly, the $i$ th column of $C_{n}$ covers $2 n$ and all numbers of the form $2 n+k$, where $k>1$ is covered by the $i$ th column of $A_{n}$. Thus we see that all numbers are accounted for in the $i$ th cross of $A_{2 n}$, and hence $A_{2 n}$ is a desired coveralls matrix. It follows that we can find a coveralls matrix whenever $n$ is a power of 2 . Second solution for part $b$. We construct a coveralls matrix explicitly for $n=2^{k}$. We consider the coordinates\/cells of the matrix elements modulo $n$ throughout the solution. We define the $i$-diagonal $(0 \\leq i<$ $n$ ) to be the set of cells of the form $(j, j+i)$, for all $j$. We note that each cross contains exactly one cell from the 0 -diagonal (the main diagonal) and two cells from each $i$-diagonal. For two cells within an $i$ diagonal, $x$ and $y$, we define $x$ and $y$ to be related if there exists a cross containing both $x$ and $y$. Evidently, for every cell $x$ not on the 0 -diagonal there are exactly two other cells related to it. The relation thus breaks up each $i$-diagonal $(i>0)$ into cycles of length larger than 1 . Due to the diagonal translational symmetry (modulo $n$ ), all the cycles within a given $i$-diagonal must be of equal length and thus of an even length, since $n=2^{k}$. The construction of a coveralls matrix is now obvious. We select a number, say 1, to place on all the cells of the 0-diagonal. We pair up the remaining numbers and assign each pair to an $i$-diagonal, say $(2 i, 2 i+1)$. Going along each cycle within the $i$-diagonal we alternately assign values of $2 i$ and $2 i+1$. Since the cycle has an even length, a cell will be related only to a cell of a different number, and hence each cross will contain both $2 i$ and $2 i+1$.","problem_type":null,"tier":0} +{"year":"1997","problem_phase":"shortlisted","problem":"5. (ROM) Let $A B C D$ be a regular tetrahedron and $M, N$ distinct points in the planes $A B C$ and $A D C$ respectively. Show that the segments $M N, B N, M D$ are the sides of a triangle.","solution":"5. We shall prove first the 2-dimensional analogue: Lemma. Given an equilateral triangle $A B C$ and two points $M, N$ on the sides $A B$ and $A C$ respectively, there exists a triangle with sides $C M, B N, M N$. Proof. Consider a regular tetrahedron $A B C D$. Since $C M=D M$ and $B N=D N$, one such triangle is $D M N$. Now, to solve the problem for a regular tetrahedron $A B C D$, we consider a 4-dimensional polytope $A B C D E$ whose faces $A B C D, A B C E, A B D E$, $A C D E, B C D E$ are regular tetrahedra. We don't know what it looks like, but it yields a desired triangle: for $M \\in A B C$ and $N \\in A D C$, we have $D M=E M$ and $B N=E N$; hence the desired triangle is $E M N$. Remark. A solution that avoids embedding in $\\mathbb{R}^{4}$ is possible, but no longer so short.","problem_type":null,"tier":0} +{"year":"1997","problem_phase":"shortlisted","problem":"6. (IRE) (a) Let $n$ be a positive integer. Prove that there exist distinct positive integers $x, y, z$ such that $$ x^{n-1}+y^{n}=z^{n+1} . $$ (b) Let $a, b, c$ be positive integers such that $a$ and $b$ are relatively prime and $c$ is relatively prime either to $a$ or to $b$. Prove that there exist infinitely many triples $(x, y, z)$ of distinct positive integers $x, y, z$ such that $$ x^{a}+y^{b}=z^{c} . $$ Original formulation: Let $a, b, c, n$ be positive integers such that $n$ is odd and $a c$ is relatively prime to $2 b$. Prove that there exist distinct positive integers $x, y, z$ such that (i) $x^{a}+y^{b}=z^{c}$, and (ii) $x y z$ is relatively prime to $n$.","solution":"6. (a) One solution is $$ x=2^{n^{2}} 3^{n+1}, \\quad y=2^{n^{2}-n} 3^{n}, \\quad z=2^{n^{2}-2 n+2} 3^{n-1} . $$ (b) Suppose w.l.o.g. that $\\operatorname{gcd}(c, a)=1$. We look for a solution of the form $$ x=p^{m}, \\quad y=p^{n}, \\quad z=q p^{r}, \\quad p, q, m, n, r \\in \\mathbb{N} . $$ Then $x^{a}+y^{b}=p^{m a}+p^{n b}$ and $z^{c}=q^{c} p^{r c}$, and we see that it is enough to assume $m a-1=n b=r c$ (there are infinitely many such triples $(m, n, r))$ and $q^{c}=p+1$.","problem_type":null,"tier":0} +{"year":"1997","problem_phase":"shortlisted","problem":"7. (RUS) Let $A B C D E F$ be a convex hexagon such that $A B=B C, C D=$ $D E, E F=F A$. Prove that $$ \\frac{B C}{B E}+\\frac{D E}{D A}+\\frac{F A}{F C} \\geq \\frac{3}{2} $$ When does equality occur?","solution":"7. Let us set $A C=a, C E=b, E A=c$. Applying Ptolemy's inequality for the quadrilateral $A C E F$ we get $$ A C \\cdot E F+C E \\cdot A F \\geq A E \\cdot C F $$ Since $E F=A F$, this implies $\\frac{F A}{F C} \\geq \\frac{c}{a+b}$. Similarly $\\frac{B C}{B E} \\geq \\frac{a}{b+c}$ and $\\frac{D E}{D A} \\geq$ $\\frac{b}{c+a}$. Now, $$ \\frac{B C}{B E}+\\frac{D E}{D A}+\\frac{F A}{F C} \\geq \\frac{a}{b+c}+\\frac{b}{c+a}+\\frac{c}{a+b} $$ Hence it is enough to prove that $$ \\frac{a}{b+c}+\\frac{b}{c+a}+\\frac{c}{a+b} \\geq \\frac{3}{2} $$ If we now substitute $x=b+c, y=c+a, z=a+b$ and $S=a+b+c$ the inequality (1) becomes equivalent to $S(1 \/ x+1 \/ y+1 \/ y)-3 \\geq 3 \/ 2$ which follows immediately form $1 \/ x+1 \/ y+1 \/ z \\geq 9 \/(x+y+z)=9 \/(2 S)$. Equality occurs if it holds in Ptolemy's inequalities and also $a=b=c$. The former happens if and only if the hexagon is cyclic. Hence the only case of equality is when $A B C D E F$ is regular.","problem_type":null,"tier":0} +{"year":"1997","problem_phase":"shortlisted","problem":"8. (GBR) ${ }^{\\mathrm{IMO} 2}$ Four different points $A, B, C, D$ are chosen on a circle $\\Gamma$ such that the triangle $B C D$ is not right-angled. Prove that: (a) The perpendicular bisectors of $A B$ and $A C$ meet the line $A D$ at certain points $W$ and $V$, respectively, and that the lines $C V$ and $B W$ meet at a certain point $T$. (b) The length of one of the line segments $A D, B T$, and $C T$ is the sum of the lengths of the other two. Original formulation. In triangle $A B C$ the angle at $A$ is the smallest. A line through $A$ meets the circumcircle again at the point $U$ lying on the $\\operatorname{arc} B C$ opposite to $A$. The perpendicular bisectors of $C A$ and $A B$ meet $A U$ at $V$ and $W$, respectively, and the lines $C V, B W$ meet at $T$. Show that $A U=T B+T C$.","solution":"8. (a) Denote by $b$ and $c$ the perpendicular bisectors of $A B$ and $A C$ respectively. If w.l.o.g. $b$ and $A D$ do not intersect (are parallel), then $\\angle B C D=\\angle B A D=90^{\\circ}$, a contradiction. Hence $V, W$ are well-defined. Now, $\\angle D W B=2 \\angle D A B$ and $\\angle D V C=2 \\angle D A C$ as oriented angles, and therefore $\\angle(W B, V C)=2(\\angle D V C-\\angle D W B)=2 \\angle B A C=$ $2 \\angle B C D$ is not equal to 0 . Consequently $C V$ and $B W$ meet at some $T$ with $\\angle B T C=2 \\angle B A C$. (b) Let $B^{\\prime}$ be the second point of intersection of $B W$ with $\\Gamma$. Clearly $A D=B B^{\\prime}$. But we also have $\\angle B T C=2 \\angle B A C=2 \\angle B B^{\\prime} C$, which implies that $C T=T B^{\\prime}$. It follows that $A D=B B^{\\prime}=\\left|B T \\pm T B^{\\prime}\\right|=$ $|B T \\pm C T|$. Remark. This problem is also solved easily using trigonometry.","problem_type":null,"tier":0} +{"year":"1997","problem_phase":"shortlisted","problem":"9. (USA) Let $A_{1} A_{2} A_{3}$ be a nonisosceles triangle with incenter $I$. Let $C_{i}$, $i=1,2,3$, be the smaller circle through $I$ tangent to $A_{i} A_{i+1}$ and $A_{i} A_{i+2}$ (the addition of indices being mod 3 ). Let $B_{i}, i=1,2,3$, be the second point of intersection of $C_{i+1}$ and $C_{i+2}$. Prove that the circumcenters of the triangles $A_{1} B_{1} I, A_{2} B_{2} I, A_{3} B_{3} I$ are collinear.","solution":"9. For $i=1,2,3$ (all indices in this problem will be modulo 3 ) we denote by $O_{i}$ the center of $C_{i}$ and by $M_{i}$ the midpoint of the $\\operatorname{arc} A_{i+1} A_{i+2}$ that does not contain $A_{i}$. First we have that $O_{i+1} O_{i+2}$ is the perpendicular bisector of $I B_{i}$, and thus it contains the circumcenter $R_{i}$ of $A_{i} B_{i} I$. Additionally, it is easy to show that $T_{i+1} A_{i}=T_{i+1} I$ and $T_{i+2} A_{i}=$ $T_{i+2} I$, which implies that $R_{i}$ lies on the line $T_{i+1} T_{i+2}$. Therefore $R_{i}=$ $O_{i+1} O_{i+2} \\cap T_{i+1} T_{i+2}$. ![](https:\/\/cdn.mathpix.com\/cropped\/2024_11_18_8e985d6b9c83aa3e9d0eg-631.jpg?height=360&width=548&top_left_y=579&top_left_x=806) Now, the lines $T_{1} O_{1}, T_{2} O_{2}, T_{3} O_{3}$ are concurrent at $I$. By Desargues's theorem, the points of intersection of $O_{i+1} O_{i+2}$ and $T_{i+1} T_{i+2}$, i.e., the $R_{i}$ 's, lie on a line for $i=1,2,3$. Second solution. The centers of three circles passing through the same point $I$ and not touching each other are collinear if and only if they have another common point. Hence it is enough to show that the circles $A_{i} B_{i} I$ have a common point other than $I$. Now apply inversion at center $I$ and with an arbitrary power. We shall denote by $X^{\\prime}$ the image of $X$ under this inversion. In our case, the image of the circle $C_{i}$ is the line $B_{i+1}^{\\prime} B_{i+2}^{\\prime}$ while the image of the line $A_{i+1} A_{i+2}$ is the circle $I A_{i+1}^{\\prime} A_{i+2}^{\\prime}$ that is tangent to $B_{i}^{\\prime} B_{i+2}^{\\prime}$, and $B_{i}^{\\prime} B_{i+2}^{\\prime}$. These three circles have equal radii, so their centers $P_{1}, P_{2}, P_{3}$ form a triangle also homothetic to $\\triangle B_{1}^{\\prime} B_{2}^{\\prime} B_{3}^{\\prime}$. Consequently, points $A_{1}^{\\prime}, A_{2}^{\\prime}, A_{3}^{\\prime}$, that are the reflections of $I$ across the sides of $P_{1} P_{2} P_{3}$, are vertices of a triangle also homothetic to $B_{1}^{\\prime} B_{2}^{\\prime} B_{3}^{\\prime}$. It follows that $A_{1}^{\\prime} B_{1}^{\\prime}, A_{2}^{\\prime} B_{2}^{\\prime}, A_{3}^{\\prime} B_{3}^{\\prime}$ are concurrent at some point $J^{\\prime}$, i.e., that the circles $A_{i} B_{i} I$ all pass through $J$.","problem_type":null,"tier":0} +{"year":"1998","problem_phase":"shortlisted","problem":"1. (LUX) ${ }^{\\mathrm{IMO}} \\mathrm{A}$ convex quadrilateral $A B C D$ has perpendicular diagonals. The perpendicular bisectors of $A B$ and $C D$ meet at a unique point $P$ inside $A B C D$. Prove that $A B C D$ is cyclic if and only if triangles $A B P$ and $C D P$ have equal areas.","solution":"1. We begin with the following observation: Suppose that $P$ lies in $\\triangle A E B$, where $E$ is the intersection of $A C$ and $B D$ (the other cases are similar). Let $M, N$ be the feet of the perpendiculars from $P$ to $A C$ and $B D$ respectively. We have $S_{A B P}=S_{A B E}-S_{A E P}-S_{B E P}=\\frac{1}{2}(A E \\cdot B E-A E \\cdot E N-B E$. $E M)=\\frac{1}{2}(A M \\cdot B N-E M \\cdot E N)$. Similarly, $S_{C D P}=\\frac{1}{2}(C M \\cdot D N-E M$. $E N)$. Therefore, we obtain $$ S_{A B P}-S_{C D P}=\\frac{A M \\cdot B N-C M \\cdot D N}{2} $$ Now suppose that $A B C D$ is cyclic. Then $P$ is the circumcenter of $A B C D$; hence $M$ and $N$ are the midpoints of $A C$ and $B D$. Hence $A M=C M$ and $B N=D N$; thus (1) gives us $S_{A B P}=S_{C D P}$. On the other hand, suppose that ![](https:\/\/cdn.mathpix.com\/cropped\/2024_11_18_8e985d6b9c83aa3e9d0eg-641.jpg?height=311&width=410&top_left_y=703&top_left_x=875) $A B C D$ is not cyclic and let w.l.o.g. $P A=P B>P C=P D$. Then we must have $A M>C M$ and $B N>$ $D N$, and consequently by (1), $S_{A B P}>S_{C D P}$. This proves the other implication. Second solution. Let $F$ and $G$ denote the midpoints of $A B$ and $C D$, and assume that $P$ is on the same side of $F G$ as $B$ and $C$. Since $P F \\perp A B$, $P G \\perp C D$, and $\\angle F E B=\\angle A B E, \\angle G E C=\\angle D C E$, a direct computation yields $\\angle F P G=\\angle F E G=90^{\\circ}+\\angle A B E+\\angle D C E$. Taking into account that $S_{A B P}=\\frac{1}{2} A B \\cdot F P=F E \\cdot F P$, we note that $S_{A B P}=S_{C D P}$ is equivalent to $F E \\cdot F P=G E \\cdot G P$, i.e., to $F E \/ E G=$ $G P \/ P F$. But this last is equivalent to triangles $E F G$ and $P G F$ being similar, which holds if and only if $E F P G$ is a parallelogram. This last is equivalent to $\\angle E F P=\\angle E G P$, or $2 \\angle A B E=2 \\angle D C E$. Thus $S_{A B P}=$ $S_{C D P}$ is equivalent to $A B C D$ being cyclic. Remark. The problems also allows an analytic solution, for example putting the $x$ and $y$ axes along the diagonals $A C$ and $B D$.","problem_type":null,"tier":0} +{"year":"1998","problem_phase":"shortlisted","problem":"10. (AUS) Let $r_{1}, r_{2}, \\ldots, r_{n}$ be real numbers greater than or equal to 1 . Prove that $$ \\frac{1}{r_{1}+1}+\\frac{1}{r_{2}+1}+\\cdots+\\frac{1}{r_{n}+1} \\geq \\frac{n}{\\sqrt[n]{r_{1} r_{2} \\cdots r_{n}}+1} $$","solution":"10. We shall first prove the inequality for $n$ of the form $2^{k}, k=0,1,2, \\ldots$ The case $k=0$ is clear. For $k=1$, we have $$ \\frac{1}{r_{1}+1}+\\frac{1}{r_{2}+1}-\\frac{2}{\\sqrt{r_{1} r_{2}}+1}=\\frac{\\left(\\sqrt{r_{1} r_{2}}-1\\right)\\left(\\sqrt{r_{1}}-\\sqrt{r_{2}}\\right)^{2}}{\\left(r_{1}+1\\right)\\left(r_{2}+1\\right)\\left(\\sqrt{r_{1} r_{2}}+1\\right)} \\geq 0 $$ For the inductive step it suffices to show that the claim for $k$ and 2 implies that for $k+1$. Indeed, $$ \\begin{aligned} \\sum_{i=1}^{2^{k+1}} \\frac{1}{r_{i}+1} & \\geq \\frac{2^{k}}{\\sqrt[2^{k}]{r_{1} r_{2} \\cdots r_{2^{k}}}+1}+\\frac{2^{k}}{\\sqrt[2^{k}]{r_{2^{k}+1^{\\prime} r_{2^{k}}+2^{\\cdots r_{2^{k+1}}}+1}}} \\\\ & \\geq \\frac{2^{k+1}}{\\sqrt[2^{k+1}]{r_{1} r_{2} \\cdots r_{2^{k+1}}}+1} \\end{aligned} $$ and the induction is complete. We now show that if the statement holds for $2^{k}$, then it holds for every $n<2^{k}$ as well. Put $r_{n+1}=r_{n+2}=\\cdots=r_{2^{k}}=\\sqrt[n]{r_{1} r_{2} \\ldots r_{n}}$. Then (1) becomes $$ \\frac{1}{r_{1}+1}+\\cdots+\\frac{1}{r_{n}+1}+\\frac{2^{k}-n}{\\sqrt[n]{r_{1} \\cdots r_{n}}+1} \\geq \\frac{2^{k}}{\\sqrt[n]{r_{1} \\cdots r_{n}}+1} $$ This proves the claim. Second solution. Define $r_{i}=e^{x_{i}}$, where $x_{i}>0$. The function $f(x)=\\frac{1}{1+e^{x}}$ is convex for $x>0$ : indeed, $f^{\\prime \\prime}(x)=\\frac{e^{x}\\left(e^{x}-1\\right)}{\\left(e^{x}+1\\right)^{3}}>0$. Thus by Jensen's inequality applied to $f\\left(x_{1}\\right), \\ldots, f\\left(x_{n}\\right)$, we get $\\frac{1}{r_{1}+1}+\\cdots+\\frac{1}{r_{n}+1} \\geq \\frac{n}{\\sqrt[n]{r_{1} \\cdots r_{n}}+1}$.","problem_type":null,"tier":0} +{"year":"1998","problem_phase":"shortlisted","problem":"11. (RUS) Let $x, y$, and $z$ be positive real numbers such that $x y z=1$. Prove that $$ \\frac{x^{3}}{(1+y)(1+z)}+\\frac{y^{3}}{(1+z)(1+x)}+\\frac{z^{3}}{(1+x)(1+y)} \\geq \\frac{3}{4} $$","solution":"11. The given inequality is equivalent to $x^{3}(x+1)+y^{3}(y+1)+z^{3}(z+1) \\geq$ $\\frac{3}{4}(x+1)(y+1)(z+1)$. By the A-G mean inequality, it will be enough to prove a stronger inequality: $$ x^{4}+x^{3}+y^{4}+y^{3}+z^{4}+z^{3} \\geq \\frac{1}{4}\\left[(x+1)^{3}+(y+1)^{3}+(z+1)^{3}\\right] . $$ If we set $S_{k}=x^{k}+y^{k}+z^{k}$, (1) takes the form $S_{4}+S_{3} \\geq \\frac{1}{4} S_{3}+\\frac{3}{4} S_{2}+\\frac{3}{4} S_{1}+\\frac{3}{4}$. Note that by the A-G mean inequality, $S_{1}=x+y+z \\geq 3$. Thus it suffices to prove the following: $$ \\text { If } S_{1} \\geq 3 \\text { and } m>n \\text { are positive integers, then } S_{m} \\geq S_{n} \\text {. } $$ This can be shown in many ways. For example, by H\u00f6lder's inequality, $$ \\left(x^{m}+y^{m}+z^{m}\\right)^{n \/ m}(1+1+1)^{(m-n) \/ m} \\geq x^{n}+y^{n}+z^{n} . $$ (Another way is using the Chebyshev inequality: if $x \\geq y \\geq z$ then $x^{k-1} \\geq$ $y^{k-1} \\geq z^{k-1}$; hence $S_{k}=x \\cdot x^{k-1}+y \\cdot y^{k-1}+z \\cdot z^{k-1} \\geq \\frac{1}{3} S_{1} S_{k-1}$, and the claim follows by induction.) Second solution. Assume that $x \\geq y \\geq z$. Then also $\\frac{1}{(y+1)(z+1)} \\geq$ $\\frac{1}{(x+1)(z+1)} \\geq \\frac{1}{(x+1)(y+1)}$. Hence Chebyshev's inequality gives that $$ \\begin{aligned} & \\frac{x^{3}}{(1+y)(1+z)}+\\frac{y^{3}}{(1+x)(1+z)}+\\frac{z^{3}}{(1+x)(1+y)} \\\\ \\geq & \\frac{1}{3} \\frac{\\left(x^{3}+y^{3}+z^{3}\\right) \\cdot(3+x+y+z)}{(1+x)(1+y)(1+z)} \\end{aligned} $$ Now if we put $x+y+z=3 S$, we have $x^{3}+y^{3}+z^{3} \\geq 3 S$ and $(1+$ $x)(1+y)(1+z) \\leq(1+a)^{3}$ by the A-G mean inequality. Thus the needed inequality reduces to $\\frac{6 S^{3}}{(1+S)^{3}} \\geq \\frac{3}{4}$, which is obviously true because $S \\geq 1$. Remark. Both these solutions use only that $x+y+z \\geq 3$.","problem_type":null,"tier":0} +{"year":"1998","problem_phase":"shortlisted","problem":"12. (POL) Let $n \\geq k \\geq 0$ be integers. The numbers $c(n, k)$ are defined as follows: $$ \\begin{aligned} c(n, 0) & =c(n, n)=1 & & \\text { for all } n \\geq 0 \\\\ c(n+1, k) & =2^{k} c(n, k)+c(n, k-1) & & \\text { for } n \\geq k \\geq 1 \\end{aligned} $$ Prove that $c(n, k)=c(n, n-k)$ for all $n \\geq k \\geq 0$.","solution":"12. The assertion is clear for $n=0$. We shall prove the general case by induction on $n$. Suppose that $c(m, i)=c(m, m-i)$ for all $i$ and $m \\leq n$. Then by the induction hypothesis and the recurrence formula we have $c(n+1, k)=2^{k} c(n, k)+c(n, k-1)$ and $c(n+1, n+1-k)=$ $2^{n+1-k} c(n, n+1-k)+c(n, n-k)=2^{n+1-k} c(n, k-1)+c(n, k)$. Thus it remains only to show that $$ \\left(2^{k}-1\\right) c(n, k)=\\left(2^{n+1-k}-1\\right) c(n, k-1) $$ We prove this also by induction on $n$. By the induction hypothesis, $$ c(n-1, k)=\\frac{2^{n-k}-1}{2^{k}-1} c(n-1, k-1) $$ and $$ c(n-1, k-2)=\\frac{2^{k-1}-1}{2^{n+1-k}-1} c(n-1, k-1) $$ Using these formulas and the recurrence formula we obtain $\\left(2^{k}-1\\right) c(n, k)-$ $\\left(2^{n+1-k}-1\\right) c(n, k-1)=\\left(2^{2 k}-2^{k}\\right) c(n-1, k)-\\left(2^{n}-3 \\cdot 2^{k-1}+1\\right) c(n-$ $1, k-1)-\\left(2^{n+1-k}-1\\right) c(n-1, k-2)=\\left(2^{n}-2^{k}\\right) c(n-1, k-1)-\\left(2^{n}-\\right.$ $\\left.3 \\cdot 2^{k-1}+1\\right) c(n-1, k-1)-\\left(2^{k-1}-1\\right) c(n-1, k-1)=0$. This completes the proof. Second solution. The given recurrence formula resembles that of binomial coefficients, so it is natural to search for an explicit formula of the form $c(n, k)=\\frac{F(n)}{F(k) F(n-k)}$, where $F(m)=f(1) f(2) \\cdots f(m)($ with $F(0)=1$ ) and $f$ is a certain function from the natural numbers to the real numbers. If there is such an $f$, then $c(n, k)=c(n, n-k)$ follows immediately. After substitution of the above relation, the recurrence equivalently reduces to $f(n+1)=2^{k} f(n-k+1)+f(k)$. It is easy to see that $f(m)=2^{m}-1$ satisfies this relation. Remark. If we introduce the polynomial $P_{n}(x)=\\sum_{k=0}^{n} c(n, k) x^{k}$, the recurrence relation gives $P_{0}(x)=1$ and $P_{n+1}(x)=x P_{n}(x)+P_{n}(2 x)$. As a consequence of the problem, all polynomials in this sequence are symmetric, i.e., $P_{n}(x)=x^{n} P_{n}\\left(x^{-1}\\right)$.","problem_type":null,"tier":0} +{"year":"1998","problem_phase":"shortlisted","problem":"13. (BUL) ${ }^{\\mathrm{IMO} 6}$ Determine the least possible value of $f(1998)$, where $f$ is a function from the set $\\mathbb{N}$ of positive integers into itself such that for all $m, n \\in \\mathbb{N}$, $$ f\\left(n^{2} f(m)\\right)=m[f(n)]^{2} $$","solution":"13. Denote by $\\mathcal{F}$ the set of functions considered. Let $f \\in \\mathcal{F}$, and let $f(1)=a$. Putting $n=1$ and $m=1$ we obtain $f(f(z))=a^{2} z$ and $f\\left(a z^{2}\\right)=f(z)^{2}$ for all $z \\in \\mathbb{N}$. These equations, together with the original one, imply $f(x)^{2} f(y)^{2}=f(x)^{2} f\\left(a y^{2}\\right)=f\\left(x^{2} f\\left(f\\left(a y^{2}\\right)\\right)\\right)=f\\left(x^{2} a^{3} y^{2}\\right)=$ $f\\left(a(a x y)^{2}\\right)=f(a x y)^{2}$, or $f(a x y)=f(x) f(y)$ for all $x, y \\in \\mathbb{N}$. Thus $f(a x)=a f(x)$, and we conclude that $$ a f(x y)=f(x) f(y) \\quad \\text { for all } x, y \\in \\mathbb{N} $$ We now prove that $f(x)$ is divisible by $a$ for each $x \\in \\mathbb{N}$. In fact, we inductively get that $f(x)^{k}=a^{k-1} f\\left(x^{k}\\right)$ is divisible by $a^{k-1}$ for every $k$. If $p^{\\alpha}$ and $p^{\\beta}$ are the exact powers of a prime $p$ that divide $f(x)$ and $a$ respectively, we deduce that $k \\alpha \\geq(k-1) \\beta$ for all $k$, so we must have $\\alpha \\geq \\beta$ for any $p$. Therefore $a \\mid f(x)$. Now we consider the function on natural numbers $g(x)=f(x) \/ a$. The above relations imply $$ g(1)=1, \\quad g(x y)=g(x) g(y), \\quad g(g(x))=x \\quad \\text { for all } x, y \\in \\mathbb{N} $$ Since $g \\in \\mathcal{F}$ and $g(x) \\leq f(x)$ for all $x$, we may restrict attention to the functions $g$ only. Clearly $g$ is bijective. We observe that $g$ maps a prime to a prime. Assume to the contrary that $g(p)=u v, u, v>1$. Then $g(u v)=p$, so either $g(u)=1$ and $g(v)=1$. Thus either $g(1)=u$ or $g(1)=v$, which is impossible. We return to the problem of determining the least possible value of $g(1998)$. Since $g(1998)=g\\left(2 \\cdot 3^{3} \\cdot 37\\right)=g(2) \\cdot g(3)^{3} \\cdot g(37)$, and $g(2)$, $g(3), g(37)$ are distinct primes, $g(1998)$ is not smaller than $2^{3} \\cdot 3 \\cdot 5=120$. On the other hand, the value of 120 is attained for any function $g$ satisfying (2) and $g(2)=3, g(3)=2, g(5)=37, g(37)=5$. Hence the answer is 120 .","problem_type":null,"tier":0} +{"year":"1998","problem_phase":"shortlisted","problem":"14. $(\\mathbf{G B R})^{\\mathrm{IMO} 4}$ Determine all pairs $(x, y)$ of positive integers such that $x^{2} y+$ $x+y$ is divisible by $x y^{2}+y+7$.","solution":"14. If $x^{2} y+x+y$ is divisible by $x y^{2}+y+7$, then so is the number $y\\left(x^{2} y+\\right.$ $x+y)-x\\left(x y^{2}+y+7\\right)=y^{2}-7 x$. If $y^{2}-7 x \\geq 0$, then since $y^{2}-7 x0$ is divisible by $x y^{2}+y+7$. But then $x y^{2}+y+7 \\leq 7 x-y^{2}<7 x$, from which we obtain $y \\leq 2$. For $y=1$, we are led to $x+8 \\mid 7 x-1$, and hence $x+8 \\mid 7(x+8)-(7 x-1)=57$. Thus the only possibilities are $x=11$ and $x=49$, and the obtained pairs $(11,1),(49,1)$ are indeed solutions. For $y=2$, we have $4 x+9 \\mid 7 x-4$, so that $7(4 x+9)-4(7 x-4)=79$ is divisible by $4 x+9$. We do not get any new solutions in this case. Therefore all required pairs $(x, y)$ are $\\left(7 t^{2}, 7 t\\right)(t \\in \\mathbb{N}),(11,1)$, and $(49,1)$.","problem_type":null,"tier":0} +{"year":"1998","problem_phase":"shortlisted","problem":"15. (AUS) Determine all pairs $(a, b)$ of real numbers such that $a\\lfloor b n\\rfloor=b\\lfloor a n\\rfloor$ for all positive integers $n$. (Note that $\\lfloor x\\rfloor$ denotes the greatest integer less than or equal to $x$.)","solution":"15. The condition is obviously satisfied if $a=0$ or $b=0$ or $a=b$ or $a, b$ are both integers. We claim that these are the only solutions. Suppose that $a, b$ belong to none of the above categories. The quotient $a \/ b=\\lfloor a\\rfloor \/\\lfloor b\\rfloor$ is a nonzero rational number: let $a \/ b=p \/ q$, where $p$ and $q$ are coprime nonzero integers. Suppose that $p \\notin\\{-1,1\\}$. Then $p$ divides $\\lfloor a n\\rfloor$ for all $n$, so in particular $p$ divides $\\lfloor a\\rfloor$ and thus $a=k p+\\varepsilon$ for some $k \\in \\mathbb{N}$ and $0 \\leq \\varepsilon<1$. Note that $\\varepsilon \\neq 0$, since otherwise $b=k q$ would also be an integer. It follows that there exists an $n \\in \\mathbb{N}$ such that $1 \\leq n \\varepsilon<2$. But then $\\lfloor n a\\rfloor=\\lfloor k n p+n \\varepsilon\\rfloor=k n p+1$ is not divisible by $p$, a contradiction. Similarly, $q \\notin\\{-1,1\\}$ is not possible. Therefore we must have $p, q= \\pm 1$, and since $a \\neq b$, the only possibility is $b=-a$. However, this leads to $\\lfloor-a\\rfloor=-\\lfloor a\\rfloor$, which is not valid if $a$ is not an integer.","problem_type":null,"tier":0} +{"year":"1998","problem_phase":"shortlisted","problem":"16. (UKR) Determine the smallest integer $n \\geq 4$ for which one can choose four different numbers $a, b, c$, and $d$ from any $n$ distinct integers such that $a+b-c-d$ is divisible by 20 .","solution":"16. Let $S$ be a set of integers such that for no four distinct elements $a, b, c, d \\in$ $S$, it holds that $20 \\mid a+b-c-d$. It is easily seen that there cannot exist distinct elements $a, b, c, d$ with $a \\equiv b$ and $c \\equiv d(\\bmod 20)$. Consequently, if the elements of $S$ give $k$ different residues modulo 20, then $S$ itself has at most $k+2$ elements. Next, consider these $k$ elements of $S$ with different residues modulo 20. They give $\\frac{k(k-1)}{2}$ different sums of two elements. For $k \\geq 7$ there are at least 21 such sums, and two of them, say $a+b$ and $c+d$, are equal modulo 20 ; it is easy to see that $a, b, c, d$ are discinct. It follows that $k$ cannot exceed 6 , and consequently $S$ has at most 8 elements. An example of a set $S$ with 8 elements is $\\{0,20,40,1,2,4,7,12\\}$. Hence the answer is $n=9$.","problem_type":null,"tier":0} +{"year":"1998","problem_phase":"shortlisted","problem":"17. (GBR) A sequence of integers $a_{1}, a_{2}, a_{3}, \\ldots$ is defined as follows: $a_{1}=1$, and for $n \\geq 1, a_{n+1}$ is the smallest integer greater than $a_{n}$ such that $a_{i}+a_{j} \\neq 3 a_{k}$ for any $i, j, k$ in $\\{1,2, \\ldots, n+1\\}$, not necessarily distinct. Determine $a_{1998}$.","solution":"17. Initially, we determine that the first few values for $a_{n}$ are $1,3,4,7,10$, $12,13,16,19,21,22,25$. Since these are exactly the numbers of the forms $3 k+1$ and $9 k+3$, we conjecture that this is the general pattern. In fact, it is easy to see that the equation $x+y=3 z$ has no solution in the set $K=\\{3 k+1,9 k+3 \\mid k \\in \\mathbb{N}\\}$. We shall prove that the sequence $\\left\\{a_{n}\\right\\}$ is actually this set ordered increasingly. Suppose $a_{n}>25$ is the first member of the sequence not belonging to $K$. We have several cases: (i) $a_{n}=3 r+2, r \\in \\mathbb{N}$. By the assumption, one of $r+1, r+2, r+3$ is of the form $3 k+1$ (and smaller than $a_{n}$ ), and therefore is a member $a_{i}$ of the sequence. Then $3 a_{i}$ equals $a_{n}+1$, $a_{n}+4$, or $a_{n}+7$, which is a contradiction because $1,4,7$ are in the sequence. (ii) $a_{n}=9 r, r \\in \\mathbb{N}$. Then $a_{n}+a_{2}=3(3 r+1)$, although $3 r+1$ is in the sequence, a contradiction. (iii) $a_{n}=9 r+6, r \\in \\mathbb{N}$. Then one of the numbers $3 r+3,3 r+6,3 r+9$ is a member $a_{j}$ of the sequence, and thus $3 a_{j}$ is equal to $a_{n}+3$, $a_{n}+12$, or $a_{n}+21$, where $3,12,21$ are members of the sequence, again a contradiction. Once we have revealed the structure of the sequence, it is easy to compute $a_{1998}$. We have $1998=4 \\cdot 499+2$, which implies $a_{1998}=9 \\cdot 499+a_{2}=4494$.","problem_type":null,"tier":0} +{"year":"1998","problem_phase":"shortlisted","problem":"18. (BUL) Determine all positive integers $n$ for which there exists an integer $m$ such that $2^{n}-1$ is a divisor of $m^{2}+9$.","solution":"18. We claim that, if $2^{n}-1$ divides $m^{2}+9$ for some $m \\in \\mathbb{N}$, then $n$ must be a power of 2 . Suppose otherwise that $n$ has an odd divisor $d>1$. Then $2^{d}-1 \\mid 2^{n}-1$ is also a divisor of $m^{2}+9=m^{2}+3^{2}$. However, $2^{d}-1$ has some prime divisor $p$ of the form $4 k-1$, and by a well-known fact, $p$ divides both $m$ and 3 . Hence $p=3$ divides $2^{d}-1$, which is impossible, because for $d$ odd, $2^{d} \\equiv 2(\\bmod 3)$. Hence $n=2^{r}$ for some $r \\in \\mathbb{N}$. Now let $n=2^{r}$. We prove the existence of $m$ by induction on $r$. The case $r=1$ is trivial. Now for any $r>1$ note that $2^{2^{r}}-1=\\left(2^{2^{r-1}}-1\\right)\\left(2^{2^{r-1}}+\\right.$ 1). The induction hypothesis claims that there exists an $m_{1}$ such that $2^{2^{r-1}}-1 \\mid m_{1}^{2}+9$. We also observe that $2^{2^{r-1}}+1 \\mid m_{2}^{2}+9$ for simple $m_{2}=3 \\cdot 2^{2^{r-2}}$. By the Chinese remainder theorem, there is an $m \\in \\mathbb{N}$ that satisfies $m \\equiv m_{1}\\left(\\bmod 2^{2^{r-1}}-1\\right)$ and $m \\equiv m_{2}\\left(\\bmod 2^{2^{r-1}}+1\\right)$. It is easy to see that this $m^{2}+9$ will be divisible by both $2^{2^{r-1}}-1$ and $2^{2^{r-1}}+1$, i.e., that $2^{2^{r}}-1 \\mid m^{2}+9$. This completes the induction.","problem_type":null,"tier":0} +{"year":"1998","problem_phase":"shortlisted","problem":"19. (BLR) ${ }^{\\mathrm{IMO} 3}$ For any positive integer $n$, let $\\tau(n)$ denote the number of its positive divisors (including 1 and itself). Determine all positive integers $m$ for which there exists a positive integer $n$ such that $\\frac{\\tau\\left(n^{2}\\right)}{\\tau(n)}=m$.","solution":"19. For $n=p_{1}^{\\alpha_{1}} p_{2}^{\\alpha_{2}} \\cdots p_{r}^{\\alpha_{r}}$, where $p_{i}$ are distinct primes and $\\alpha_{i}$ natural numbers, we have $\\tau(n)=\\left(\\alpha_{1}+1\\right) \\cdots\\left(\\alpha_{r}+1\\right)$ and $\\tau\\left(n^{2}\\right)=\\left(2 \\alpha_{1}+1\\right) \\ldots\\left(2 \\alpha_{r}+1\\right)$. Putting $k_{i}=\\alpha_{i}+1$, the problem reduces to determining all natural values of $m$ that can be represented as $$ m=\\frac{2 k_{1}-1}{k_{1}} \\cdot \\frac{2 k_{2}-1}{k_{2}} \\cdots \\frac{2 k_{r}-1}{k_{r}} $$ Since the numerator $\\tau\\left(n^{2}\\right)$ is odd, $m$ must be odd too. We claim that every odd $m$ has a representation of the form (1). The proof will be done by induction. This is clear for $m=1$. Now for every $m=2 k-1$ with $k$ odd the result follows easily, since $m=\\frac{2 k-1}{k} \\cdot k$, and $k$ can be written as (1). We cannot do the same if $k$ is even; however, in the case $m=4 k-1$ with $k$ odd, we can write it as $m=\\frac{12 k-3}{6 k-1} \\cdot \\frac{6 k-1}{3 k} \\cdot k$, and this works. In general, suppose that $m=2^{t} k-1$, with $k$ odd. Following the same pattern, we can write $m$ as $$ m=\\frac{2^{t}\\left(2^{t}-1\\right) k-\\left(2^{t}-1\\right)}{2^{t-1}\\left(2^{t}-1\\right) k-\\left(2^{t-1}-1\\right)} \\cdots \\frac{4\\left(2^{t}-1\\right) k-3}{2\\left(2^{t}-1\\right) k-1} \\cdot \\frac{2\\left(2^{t}-1\\right) k-1}{\\left(2^{t}-1\\right) k} \\cdot k $$ The induction is finished. Hence $m$ can be represented as $\\frac{\\tau\\left(n^{2}\\right)}{\\tau(n)}$ if and only if it is odd.","problem_type":null,"tier":0} +{"year":"1998","problem_phase":"shortlisted","problem":"2. (POL) Let $A B C D$ be a cyclic quadrilateral. Let $E$ and $F$ be variable points on the sides $A B$ and $C D$, respectively, such that $A E: E B=C F$ : $F D$. Let $P$ be the point on the segment $E F$ such that $P E: P F=A B$ : $C D$. Prove that the ratio between the areas of triangles $A P D$ and $B P C$ does not depend on the choice of $E$ and $F$.","solution":"2. If $A D$ and $B C$ are parallel, then $A B C D$ is an isosceles trapezoid with $A B=C D$, so $P$ is the midpoint of $E F$. Let $M$ and $N$ be the midpoints of $A B$ and $C D$. Then $M N \\| B C$, and the distance $d(E, M N)$ equals the distance $d(F, M N)$ because $B$ and $D$ are the same distance from $M N$ and $E M \/ B M=F N \/ D N$. It follows that the midpoint $P$ of $E F$ lies on $M N$, and consequently $S_{A P D}: S_{B P C}=A D: B C$. If $A D$ and $B C$ are not parallel, then they meet at some point $Q$. It is plain that $\\triangle Q A B \\sim \\triangle Q C D$, and since $A E \/ A B=C F \/ C D$, we also deduce that $\\triangle Q A E \\sim \\triangle Q C F$. Therefore $\\angle A Q E=\\angle C Q F$. Further, from these similarities we obtain $Q E \/ Q F=Q A \/ Q C=A B \/ C D=P E \/ P F$, which in turn means that $Q P$ is the internal bisector of $\\angle E Q F$. But since $\\angle A Q E=\\angle C Q F$, this is also the internal bisector of $\\angle A Q B$. Hence $P$ is at equal distances from $A D$ and $B C$, so again $S_{A P D}: S_{B P C}=A D: B C$. Remark. The part $A B \\| C D$ could also be regarded as a limiting case of the other part. Second solution. Denote $\\lambda=\\frac{A E}{A B}, A B=a, B C=b, C D=c, D A=d$, $\\angle D A B=\\alpha, \\angle A B C=\\beta$. Since $d(P, A D)=\\frac{c \\cdot d(E, A D)+a \\cdot d(F, A D)}{a+c}$, we have $S_{A P D}=\\frac{c S_{E A D}+a S_{F A D}}{a+c}=\\frac{\\lambda c S_{A B D}+(1-\\lambda) a S_{A C D}}{a+c}$. Since $S_{A B D}=\\frac{1}{2} a d \\sin \\alpha$ and $S_{A C D}=\\frac{1}{2} c d \\sin \\beta$, we are led to $S_{A P D}=\\frac{a c d}{a+c}[\\lambda \\sin \\alpha+(1-\\lambda) \\sin \\beta]$, and analogously $S_{B P C}=\\frac{a b c}{a+c}[\\lambda \\sin \\alpha+(1-\\lambda) \\sin \\beta]$. Thus we obtain $S_{A P D}: S_{B P C}=d: b$.","problem_type":null,"tier":0} +{"year":"1998","problem_phase":"shortlisted","problem":"20. (ARG) Prove that for each positive integer $n$, there exists a positive integer with the following properties: (i) It has exactly $n$ digits. (ii) None of the digits is 0 . (iii) It is divisible by the sum of its digits.","solution":"20. We first consider the special case $n=3^{r}$. Then the simplest choice $\\frac{10^{n}-1}{9}=$ $11 \\ldots 1$ ( $n$ digits) works. This can be shown by induction: it is true for $r=$ 1, while the inductive step follows from $10^{3^{r}}-1=\\left(10^{3^{r-1}}-1\\right)\\left(10^{2 \\cdot 3^{r-1}}+\\right.$ $10^{3^{r-1}}+1$ ), because the second factor is divisible by 3 . In the general case, let $k \\geq n \/ 2$ be a positive integer and $a_{1}, \\ldots, a_{n-k}$ be nonzero digits. We have $$ \\begin{aligned} A & =\\left(10^{k}-1\\right) \\overline{a_{1} a_{2} \\ldots a_{n-k}} \\\\ & =\\overline{a_{1} a_{2} \\ldots a_{n-k-1} a_{n-k}^{\\prime} \\underbrace{99 \\ldots 99}_{2 k-n}} b_{1} b_{2} \\ldots b_{n-k-1} b_{n-k}^{\\prime} \\end{aligned} $$ where $a_{n-k}^{\\prime}=a_{n-k}-1, b_{i}=9-a_{i}$, and $b_{n-k}^{\\prime}=9-a_{n-k}^{\\prime}$. The sum of digits of $A$ equals $9 k$ independently of the choice of digits $a_{1}, \\ldots, a_{n-k}$. Thus we need only choose $k \\geq \\frac{n}{2}$ and digits $a_{1}, \\ldots, a_{n-k-1} \\notin\\{0,9\\}$ and $a_{n-k} \\in\\{0,1\\}$ in order for the conditions to be fulfilled. Let us choose $$ k=\\left\\{\\begin{array}{l} 3^{r}, \\quad \\text { if } 3^{r}U W \\cdot V W$. Proof. Let $X W^{2}>U W \\cdot V W$, and let $X_{0}$ be a point on the segment $X W$ such that $X_{0} W^{2} \\geq U W \\cdot V W$. Then $X_{0} W \/ U W=V W \/ X_{0} W$, so that triangles $X_{0} W U$ and $V W X_{0}$ are similar. Thus $\\angle U X_{0} V=\\angle U X_{0} W+$ $\\angle W U X_{0}=90^{\\circ}$, which immediately implies that $\\angle U X V<90^{\\circ}$. Similarly, if $X W^{2} \\leq U W \\cdot V W$, then $\\angle U X V \\geq 90^{\\circ}$. Since $B I \\perp R S$, it will be enough by the lemma to show that $B I^{2}>$ $B R \\cdot B S$. Note that $\\triangle B K R \\sim \\triangle B S L$ : in fact, we have $\\angle K B R=\\angle S B L=$ $90^{\\circ}-\\beta \/ 2$ and $\\angle B K R=\\angle A K M=\\angle K L M=\\angle B S L=90^{\\circ}-\\alpha \/ 2$. In particular, we obtain $B R \/ B K=B L \/ B S=B K \/ B S$, so that $B R \\cdot B S=$ $B K^{2}B K$ and $B M>B L$. We conclude that $\\angle M B E<\\frac{1}{2} \\angle M B K$ and $\\angle M B F<\\frac{1}{2} \\angle M B L$. Adding these two inequalities gives $\\angle E B F<$ $\\beta \/ 2$. Therefore $\\angle R I S<90^{\\circ}$. Remark. It can be shown (using vectors) that the statement remains true for an arbitrary line $t$ passing through $B$.","problem_type":null,"tier":0} +{"year":"1998","problem_phase":"shortlisted","problem":"4. (ARM) Let $M$ and $N$ be points inside triangle $A B C$ such that $$ \\angle M A B=\\angle N A C \\quad \\text { and } \\quad \\angle M B A=\\angle N B C . $$ Prove that $$ \\frac{A M \\cdot A N}{A B \\cdot A C}+\\frac{B M \\cdot B N}{B A \\cdot B C}+\\frac{C M \\cdot C N}{C A \\cdot C B}=1 $$","solution":"4. Let $K$ be the point on the ray $B N$ with $\\angle B C K=\\angle B M A$. Since $\\angle K B C=\\angle A B M$, we get $\\triangle B C K \\sim \\triangle B M A$. It follows that $B C \/ B M=$ $B K \/ B A$, which implies that also $\\triangle B A K \\sim \\triangle B M C$. The quadrilateral $A N C K$ is cyclic, because $\\angle B K C=\\angle B A M=\\angle N A C$. Then by Ptolemy's theorem we obtain $$ A C \\cdot B K=A C \\cdot B N+A N \\cdot C K+C N \\cdot A K $$ On the other hand, from the similarities noted above we get $$ C K=\\frac{B C \\cdot A M}{B M}, A K=\\frac{A B \\cdot C M}{B M} \\text { and } B K=\\frac{A B \\cdot B C}{B M} . $$ After substitution of these values, the equality (1) becomes $$ \\frac{A B \\cdot B C \\cdot A C}{B M}=A C \\cdot B N+\\frac{B C \\cdot A M \\cdot A N}{B M}+\\frac{A B \\cdot C M \\cdot C N}{B M}, $$ which is exactly the equality we must prove multiplied by $\\frac{A B \\cdot B C \\cdot C A}{B M}$.","problem_type":null,"tier":0} +{"year":"1998","problem_phase":"shortlisted","problem":"5. (FRA) Let $A B C$ be a triangle, $H$ its orthocenter, $O$ its circumcenter, and $R$ its circumradius. Let $D$ be the reflection of $A$ across $B C, E$ that of $B$ across $C A$, and $F$ that of $C$ across $A B$. Prove that $D, E$, and $F$ are collinear if and only if $O H=2 R$.","solution":"5. Let $G$ be the centroid of $\\triangle A B C$ and $\\mathcal{H}$ the homothety with center $G$ and ratio $-\\frac{1}{2}$. It is well-known that $\\mathcal{H}$ maps $H$ into $O$. For every other point $X$, let us denote by $X^{\\prime}$ its image under $\\mathcal{H}$. Also, let $A_{2} B_{2} C_{2}$ be the triangle in which $A, B, C$ are the midpoints of $B_{2} C_{2}, C_{2} A_{2}$, and $A_{2} B_{2}$, respectively. It is clear that $A^{\\prime}, B^{\\prime}, C^{\\prime}$ are the midpoints of sides $B C, C A, A B$ respectively. Furthermore, $D^{\\prime}$ is the reflection of $A^{\\prime}$ across $B^{\\prime} C^{\\prime}$. Thus $D^{\\prime}$ must lie on $B_{2} C_{2}$ and $A^{\\prime} D^{\\prime} \\perp$ ![](https:\/\/cdn.mathpix.com\/cropped\/2024_11_18_8e985d6b9c83aa3e9d0eg-643.jpg?height=484&width=552&top_left_y=612&top_left_x=806) $B_{2} C_{2}$. However, it also holds that $O A^{\\prime} \\perp B_{2} C_{2}$, so we conclude that $O, D^{\\prime}, A^{\\prime}$ are collinear and $D^{\\prime}$ is the projection of $O$ on $B_{2} C_{2}$. Analogously, $E^{\\prime}, F^{\\prime}$ are the projections of $O$ on $C_{2} A_{2}$ and $A_{2} B_{2}$. Now we apply Simson's theorem. It claims that $D^{\\prime}, E^{\\prime}, F^{\\prime}$ are collinear (which is equivalent to $D, E, F$ being collinear) if and only if $O$ lies on the circumcircle of $A_{2} B_{2} C_{2}$. However, this circumcircle is centered at $H$ with radius $2 R$, so the last condition is equivalent to $H O=2 R$.","problem_type":null,"tier":0} +{"year":"1998","problem_phase":"shortlisted","problem":"6. (POL) Let $A B C D E F$ be a convex hexagon such that $\\angle B+\\angle D+\\angle F=$ $360^{\\circ}$ and $$ \\frac{A B}{B C} \\cdot \\frac{C D}{D E} \\cdot \\frac{E F}{F A}=1 $$ Prove that $$ \\frac{B C}{C A} \\cdot \\frac{A E}{E F} \\cdot \\frac{F D}{D B}=1 $$","solution":"6. Let $P$ be the point such that $\\triangle C D P$ and $\\triangle C B A$ are similar and equally oriented. Since then $\\angle D C P=\\angle B C A$ and $\\frac{B C}{C A}=\\frac{D C}{C P}$, it follows that $\\angle A C P=\\angle B C D$ and $\\frac{A C}{C P}=\\frac{B C}{C D}$, so $\\triangle A C P \\sim \\triangle B C D$. In particular, $\\frac{B C}{C A}=\\frac{D B}{P A}$. Furthermore, by the conditions of the problem we have $\\angle E D P=360^{\\circ}-$ $\\angle B-\\angle D=\\angle F$ and $\\frac{P D}{D E}=\\frac{P D}{C D} \\cdot \\frac{C D}{D E}=\\frac{A B}{B C} \\cdot \\frac{C D}{D E}=\\frac{A F}{F E}$. Therefore $\\triangle E D P \\sim \\triangle E F A$ as well, so that similarly as above we conclude that $\\triangle A E P \\sim \\triangle F E D$ and consequently $\\frac{A E}{E F}=\\frac{P A}{F D}$. Finally, $\\frac{B C}{C A} \\cdot \\frac{A E}{E F} \\cdot \\frac{F D}{D B}=\\frac{D B}{P A} \\cdot \\frac{P A}{F D} \\cdot \\frac{F D}{D B}=1$. Second solution. Let $a, b, c, d, e, f$ be the complex coordinates of $A, B$, $C, D, E, F$, respectively. The condition of the problem implies that $\\frac{a-b}{b-c}$. $\\frac{c-d}{d-e} \\cdot \\frac{e-f}{f-a}=-1$. On the other hand, since $(a-b)(c-d)(e-f)+(b-c)(d-e)(f-a)=$ $(b-c)(a-e)(f-d)+(c-a)(e-f)(d-b)$ holds identically, we immediately deduce that $\\frac{b-c}{c-a} \\cdot \\frac{a-e}{e-f} \\cdot \\frac{f-d}{d-b}=-1$. Taking absolute values gives $\\frac{B C}{C A} \\cdot \\frac{A E}{E F}$. $\\frac{F D}{D B}=1$.","problem_type":null,"tier":0} +{"year":"1998","problem_phase":"shortlisted","problem":"7. (GBR) Let $A B C$ be a triangle such that $\\angle A C B=2 \\angle A B C$. Let $D$ be the point on the side $B C$ such that $C D=2 B D$. The segment $A D$ is extended to $E$ so that $A D=D E$. Prove that $$ \\angle E C B+180^{\\circ}=2 \\angle E B C . $$","solution":"7. We shall use the following result. Lemma. In a triangle $A B C$ with $B C=a, C A=b$, and $A B=c$, i. $\\angle C=2 \\angle B$ if and only if $c^{2}=b^{2}+a b$; ii. $\\angle C+180^{\\circ}=2 \\angle B$ if and only if $c^{2}=b^{2}-a b$. Proof. i. Take a point $D$ on the extension of $B C$ over $C$ such that $C D=b$. The condition $\\angle C=2 \\angle B$ is equivalent to $\\angle A D C=\\frac{1}{2} \\angle C=\\angle B$, and thus to $A D=A B=c$. This is further equivalent to triangles $C A D$ and $A B D$ being similar, so $C A \/ A D=A B \/ B D$, i.e., $c^{2}=$ $b(a+b)$. ii. Take a point $E$ on the ray $C B$ such that $C E=b$. As above, $\\angle C+180^{\\circ}=2 \\angle B$ if and only if $\\triangle C A E \\sim \\triangle A B E$, which is equivalent to $E B \/ B A=E A \/ A C$, or $c^{2}=b(b-a)$. Let $F, G$ be points on the ray $C B$ such that $C F=\\frac{1}{3} a$ and $C G=\\frac{4}{3} a$. Set $B C=a, C A=b, A B=c, E C=b_{1}$, and $E B=c_{1}$. By the lemma it follows that $c^{2}=b^{2}+a b$. Also $b_{1}=A G$ and $c_{1}=A F$, so Stewart's theorem gives us $c_{1}^{2}=\\frac{2}{3} b^{2}+\\frac{1}{3} c^{2}-\\frac{2}{9} a^{2}=b^{2}+\\frac{1}{3} a b-\\frac{2}{9} a^{2}$ and $b_{1}^{2}=$ $-\\frac{1}{3} b^{2}+\\frac{4}{3} c^{2}+\\frac{4}{9} a^{2}=b^{2}+\\frac{4}{3} a b+\\frac{4}{9} a^{2}$. It follows that $b_{1}=\\frac{2}{3} a+b$ and $c_{1}^{2}=b_{1}^{2}-\\left(a b+\\frac{2}{3} a^{2}\\right)=b_{1}^{2}-a b_{1}$. The statement of the problem follows immediately by the lemma.","problem_type":null,"tier":0} +{"year":"1998","problem_phase":"shortlisted","problem":"8. (IND) Let $A B C$ be a triangle such that $\\angle A=90^{\\circ}$ and $\\angle B<\\angle C$. The tangent at $A$ to its circumcircle $\\omega$ meets the line $B C$ at $D$. Let $E$ be the reflection of $A$ across $B C, X$ the foot of the perpendicular from $A$ to $B E$, and $Y$ the midpoint of $A X$. Let the line $B Y$ meet $\\omega$ again at $Z$. Prove that the line $B D$ is tangent to the circumcircle of triangle $A D Z$.","solution":"8. Let $M$ be the point of intersection of $A E$ and $B C$, and let $N$ be the point on $\\omega$ diametrically opposite $A$. Since $\\angle B<\\angle C$, points $N$ and $B$ are on the same side of $A E$. Furthermore, $\\angle N A E=\\angle B A X=$ $90^{\\circ}-\\angle A B E$; hence the triangles $N A E$ and $B A X$ are similar. Consequently, $\\triangle B A Y$ and $\\triangle N A M$ are also similar, since $M$ is the midpoint ![](https:\/\/cdn.mathpix.com\/cropped\/2024_11_18_8e985d6b9c83aa3e9d0eg-644.jpg?height=279&width=555&top_left_y=1171&top_left_x=802) of $A E$. Thus $\\angle A N Z=\\angle A B Z=\\angle A B Y=\\angle A N M$, implying that $N, M, Z$ are collinear. Now we have $\\angle Z M D=90^{\\circ}-\\angle Z M A=\\angle E A Z=$ $\\angle Z E D$ (the last equality because $E D$ is tangent to $\\omega$ ); hence $Z M E D$ is a cyclic quadrilateral. It follows that $\\angle Z D M=\\angle Z E A=\\angle Z A D$, which is enough to conclude that $M D$ is tangent to the circumcircle of $A Z D$. Remark. The statement remains valid if $\\angle B \\geq \\angle C$.","problem_type":null,"tier":0} +{"year":"1998","problem_phase":"shortlisted","problem":"9. (MON) Let $a_{1}, a_{2}, \\ldots, a_{n}$ be positive real numbers such that $a_{1}+a_{2}+$ $\\cdots+a_{n}<1$. Prove that $$ \\frac{a_{1} a_{2} \\cdots a_{n}\\left[1-\\left(a_{1}+a_{2}+\\cdots+a_{n}\\right)\\right]}{\\left(a_{1}+a_{2}+\\cdots+a_{n}\\right)\\left(1-a_{1}\\right)\\left(1-a_{2}\\right) \\cdots\\left(1-a_{n}\\right)} \\leq \\frac{1}{n^{n+1}} . $$","solution":"9. Set $a_{n+1}=1-\\left(a_{1}+\\cdots+a_{n}\\right)$. Then $a_{n+1}>0$, and the desired inequality becomes $$ \\frac{a_{1} a_{2} \\cdots a_{n+1}}{\\left(1-a_{1}\\right)\\left(1-a_{2}\\right) \\cdots\\left(1-a_{n+1}\\right)} \\leq \\frac{1}{n^{n+1}} $$ To prove it, we observe that $1-a_{i}=a_{1}+\\cdots+a_{i-1}+a_{i+1}+\\cdots+a_{n+1} \\geq n \\sqrt[n]{a_{1} \\cdots a_{i-1} a_{i+1} \\cdots a_{n+1}}$. Multiplying these inequalities for $i=1,2, \\ldots, n+1$, we get exactly the inequality we need.","problem_type":null,"tier":0} +{"year":"1999","problem_phase":"shortlisted","problem":"1. $\\mathbf{N} \\mathbf{1}(\\mathbf{T W N}){ }^{\\mathrm{IMO}+4}$ Find all pairs of positive integers $(x, p)$ such that $p$ is a prime, $x \\leq 2 p$, and $x^{p-1}$ is a divisor of $(p-1)^{x}+1$.","solution":"1. Obviously $(1, p)$ (where $p$ is an arbitrary prime) and $(2,2)$ are solutions and the only solutions to the problem for $x<3$ or $p<3$. Let us now assume $x, p \\geq 3$. Since $p$ is odd, $(p-1)^{x}+1$ is odd, and hence $x$ is odd. Let $q$ be the largest prime divisor of $x$, which also must be odd. We have $q|x| x^{p-1} \\mid(p-1)^{x}+1 \\Rightarrow(p-1)^{x} \\equiv-1(\\bmod q)$. Also from Fermat's little theorem $(p-1)^{q-1} \\equiv 1(\\bmod q)$. Since $q-1$ and $x$ are coprime, there exist integers $\\alpha, \\beta$ such that $x \\alpha=(q-1) \\beta+1$. We also note that $\\alpha$ must be odd. We now have $p-1 \\equiv(p-1)^{(q-1) \\beta+1} \\equiv(p-1)^{x \\alpha} \\equiv-1(\\bmod q)$ and hence $q \\mid p \\Rightarrow q=p$. Since $x$ is odd, $p \\mid x$, and $x \\leq 2 p$, it follows $x=p$ for all $x, p \\geq 3$. Thus $$ p^{p-1} \\left\\lvert\\,(p-1)^{x}+1=p^{2} \\cdot\\left(p^{p-2}-\\binom{p}{1} p^{p-1}+\\cdots-\\binom{p}{p-2}+1\\right) .\\right. $$ Since the expression in parenthesis is not divisible by $p$, it follows that $p^{p-1} \\mid p^{2}$ and hence $p \\leq 3$. One can easily verify that $(3,3)$ is a valid solution. We have shown that the only solutions are $(1, p),(2,2)$, and $(3,3)$, where $p$ is an arbitrary prime.","problem_type":null,"tier":0} +{"year":"1999","problem_phase":"shortlisted","problem":"10. G4 (GBR) For a triangle $T=A B C$ we take the point $X$ on the side $(A B)$ such that $A X \/ X B=4 \/ 5$, the point $Y$ on the segment $(C X)$ such that $C Y=2 Y X$, and, if possible, the point $Z$ on the ray ( $C A$ such that $\\measuredangle C X Z=180^{\\circ}-\\measuredangle A B C$. We denote by $\\Sigma$ the set of all triangles $T$ for which $\\measuredangle X Y Z=45^{\\circ}$. Prove that all the triangles from $\\Sigma$ are similar and find the measure of their smallest angle.","solution":"10. We use the following lemma. Lemma. Let $A B C$ be a triangle and $X \\in A B$ such that $\\overrightarrow{A X}: \\overrightarrow{X B}=m: n$. Then $(m+n) \\cot \\angle C X B=n \\cot A-m \\cot B$ and $m \\cot \\angle A C X=$ $(n+m) \\cot C+n \\cot A$. Proof. Let $C D$ be the altitude from $C$ and $h$ its length. Then using oriented segments we have $A X=A D+D X=h \\cot A-h \\cot \\angle C X B$ and $B X=B D+D X=h \\cot B+h \\cot \\angle C X B$. The first formula in the lemma now follows from $n \\cdot A X=m \\cdot B X$. The second formula immediately follows from the first part applied to the triangle $A C X$ and the point $X^{\\prime} \\in A C$ such that $X X^{\\prime} \\| B C$. Let us set $\\cot A=x, \\cot B=y$, and $\\cot C=z$. Applying the second formula in the lemma to $\\triangle A B C$ and the point $X$, we obtain $4 \\cot \\angle A C X=$ $9 z+5 x$. Applying the first formula in the lemma to $\\triangle C X Z$ and the point $Y$ and using $\\angle X Y Z=45^{\\circ}$ and $\\cot \\angle C X Z=-y$, we obtain $3 \\cot \\angle X Y Z=$ $\\cot \\angle A C X-2 \\cot \\angle C X Z=\\frac{9 z+5 x}{4}+2 y \\Rightarrow 5 x+8 y+9 z=12$. We now use the well-known relation for cotangents of a triangle $x y+y z+$ $x z=1$ to get $9=9(x+y) z+9 x y=(x+y)(12-5 x-8 z)+9 x y=9 \\Rightarrow$ $(4 y+x-3)^{2}+9(x-1)^{2}=0 \\Rightarrow x=1, y=\\frac{1}{2}, z=\\frac{1}{3}$. It follows that $x, y$, and $z$ have fixed values, and hence all triangles $T$ in $\\Sigma$ are similar, with their smallest angle $A$ having cotangent 1 and thus being equal to $\\angle A=45^{\\circ}$.","problem_type":"Geometry","tier":0} +{"year":"1999","problem_phase":"shortlisted","problem":"11. G5 (FRA) Let $A B C$ be a triangle, $\\Omega$ its incircle and $\\Omega_{a}, \\Omega_{b}, \\Omega_{c}$ three circles three circles orthogonal to $\\Omega$ passing through $B$ and $C, A$ and $C$, and $A$ and $B$ respectively. The circles $\\Omega_{a}, \\Omega_{b}$ meet again in $C^{\\prime}$; in the same way we obtain the points $B^{\\prime}$ and $A^{\\prime}$. Prove that the radius of the circumcircle of $A^{\\prime} B^{\\prime} C^{\\prime}$ is half the radius of $\\Omega$.","solution":"11. Let $\\Omega(I, r)$ be the incircle of $\\triangle A B C$. Let $D, E$, and $F$ denote the points where $\\Omega$ touches $B C, A C$, and $A B$, respectively. Let $P, Q$, and $R$ denote the midpoints of $E F, D F$, and $D E$ respectively. We prove that $\\Omega_{a}$ passes through $Q$ and $R$. Since $\\triangle I Q D \\sim \\triangle I D B$ and $\\triangle I R D \\sim \\triangle I D C$, we obtain $I Q \\cdot I B=I R \\cdot I C=r^{2}$. We conclude that $B, C, Q$, and $R$ lie on a single circle $\\Gamma_{a}$. Moreover, since the power of $I$ with respect to $\\Gamma_{a}$ is $r^{2}$, it follows for a tangent $I X$ from $I$ to $\\Gamma_{a}$ that $X$ lies on $\\Omega$ and hence $\\Omega$ is perpendicular to $\\Gamma_{a}$. From the uniqueness of $\\Omega_{a}$ it follows that $\\Omega_{a}=\\Gamma_{a}$. Thus $\\Omega_{a}$ contains $Q$ and $R$. Similarly $\\Omega_{b}$ contains $P$ and $R$ and $\\Omega_{c}$ contains $P$ and $Q$. Hence, $A^{\\prime}=P, B^{\\prime}=Q$ and $C^{\\prime}=R$. Therefore the radius of the circumcircle of $\\triangle A^{\\prime} B^{\\prime} C^{\\prime}$ is half the radius of $\\Omega$.","problem_type":"Geometry","tier":0} +{"year":"1999","problem_phase":"shortlisted","problem":"12. G6 (RUS) ${ }^{\\mathrm{IMO} 5}$ Two circles $\\Omega_{1}$ and $\\Omega_{2}$ touch internally the circle $\\Omega$ in $M$ and $N$, and the center of $\\Omega_{2}$ is on $\\Omega_{1}$. The common chord of the circles $\\Omega_{1}$ and $\\Omega_{2}$ intersects $\\Omega$ in $A$ and $B . M A$ and $M B$ intersect $\\Omega_{1}$ in $C$ and $D$. Prove that $\\Omega_{2}$ is tangent to $C D$.","solution":"12. We first introduce the following lemmas. Lemma 1. Let $A B C$ be a triangle, $I$ its inenter and $I_{a}$ the center of the excircle touching $B C$. Let $A^{\\prime}$ be the center of the arc $\\widehat{B C}$ of the circumcircle not containing $A$. Then $A^{\\prime} B=A^{\\prime} C=A^{\\prime} I=A^{\\prime} I_{a}$. Proof. The result follows from a straightforward calculation of the relevant angles. Lemma 2. Let two circles $k_{1}$ and $k_{2}$ meet each other at points $X$ and $Y$ and touch a circle $k$ internally in points $M$ and $N$, respectively. Let $A$ be one of the intersections of the line $X Y$ with $k$. Let $A M$ and $A N$ intersect $k_{1}$ and $k_{2}$ respectively at $C$ and $E$. Then $C E$ is a common tangent of $k_{1}$ and $k_{2}$. Proof. Since $A C \\cdot A M=A X \\cdot A Y=A E \\cdot A N$, the points $M, N, E, C$ lie on a circle. Let $M N$ meet $k_{1}$ again at $Z$. If $M^{\\prime}$ is any point on the common tangent at $M$, then $\\angle M C Z=\\angle M^{\\prime} M Z=\\angle M^{\\prime} M N=\\angle M A N$ (as oriented angles), implying that $C Z \\| A N$. It follows that $\\angle A C E=$ $\\angle A N M=\\angle C Z M$. Hence $C E$ is tangent to $k_{1}$ and analogously to $k_{2}$. In the main problem, let us define $E$ and $F$ respectively as intersections of $N A$ and $N B$ with $\\Omega_{2}$. Then applying Lemma 2 we get that $C E$ and $D F$ are the common tangents of $\\Omega_{1}$ and $\\Omega_{2}$. If the circles have the same radii, the result trivially holds. Otherwise, let $G$ be the intersection of $C E$ and $D F$. Let $O_{1}$ and $O_{2}$ be the centers of $\\Omega_{1}$ and $\\Omega_{2}$. Since $O_{1} D=O_{1} C$ and $\\angle O_{1} D G=\\angle O_{1} C G=90^{\\circ}$, it follows that $O_{1}$ is the midpoint of the shorter arc of the circumcircle of $\\triangle C D G$. The center $O_{2}$ is located on the bisector of $\\angle C G D$, since $\\Omega_{2}$ ![](https:\/\/cdn.mathpix.com\/cropped\/2024_11_18_8e985d6b9c83aa3e9d0eg-660.jpg?height=412&width=529&top_left_y=1000&top_left_x=813) touches both $G C$ and $G D$. However, it also sits on $\\Omega_{1}$, and using Lemma 1 we obtain that $O_{2}$ is either at the incenter or at the excenter of $\\triangle C D G$ opposite $G$. Hence, $\\Omega_{2}$ is either the incircle or the excircle of $C D G$ and thus in both cases touches $C D$. Second solution. Let $O$ be the center of $\\Gamma$, and $r, r_{1}, r_{2}$ the radii of $\\Gamma, \\Gamma_{1}, \\Gamma_{2}$. It suffices to show that the distance $d\\left(O_{2}, C D\\right)$ is equal to $r_{2}$. The homothety with center $M$ and ratio $r \/ r_{1}$ takes $\\Gamma_{1}, C, D$ into $\\Gamma, A, B$, respectively; hence $C D \\| A B$ and $d(C, A B)=\\frac{r-r_{1}}{r} d(M, A B)$. Let $O_{1} O_{2}$ meet $X Y$ at $R$. Then $d\\left(O_{2}, C D\\right)=O_{2} R+\\frac{r-r_{1}}{r} d(M, A B)$, i.e., $$ d\\left(O_{2}, C D\\right)=O_{2} R+\\frac{r-r_{1}}{r}\\left[O_{1} O_{2}-O_{2} R+r_{1} \\cos \\angle O O_{1} O_{2}\\right] $$ since $O, O_{1}$, and $M$ are collinear. We have $O_{1} X=O_{1} O_{2}=r_{1}, O O_{1}=$ $r-r_{1}, O O_{2}=r-r_{2}$, and $O_{2} X=r_{2}$. Using the cosine law in the triangles $O O_{1} O_{2}$ and $X O_{1} O_{2}$, we obtain that $\\cos \\angle O O_{1} O_{2}=\\frac{2 r_{1}^{2}-2 r r_{1}+2 r r_{2}-r_{2}^{2}}{2 r_{1}\\left(r-r_{1}\\right)}$ and $O_{2} R=\\frac{r_{2}^{2}}{2 r_{1}}$. Substituting these values in (1) we get $d\\left(O_{2}, C D\\right)=r_{2}$.","problem_type":"Geometry","tier":0} +{"year":"1999","problem_phase":"shortlisted","problem":"13. G7 (ARM) The point $M$ inside the convex quadrilateral $A B C D$ is such that $M A=M C, \\angle A M B=\\angle M A D+\\angle M C D, \\angle C M D=\\angle M C B+\\angle M A B$. Prove that $A B \\cdot C M=B C \\cdot M D$ and $B M \\cdot A D=M A \\cdot C D$.","solution":"13. Let us construct a convex quadrilateral $P Q R S$ and an interior point $T$ such that $\\triangle P T Q \\cong \\triangle A M B, \\triangle Q T R \\sim \\triangle A M D$, and $\\triangle P T S \\sim \\triangle C M D$. We then have $T S=\\frac{M D \\cdot P T}{M C}=M D$ and $\\frac{T R}{T S}=\\frac{T R \\cdot T Q \\cdot T P}{T Q \\cdot T P \\cdot T S}=\\frac{M D \\cdot M B \\cdot M C}{M A \\cdot M A \\cdot M D}=$ $\\frac{M B}{M C}$ (using $M A=M C$ ). We also have $\\angle S T R=\\angle B M C$ and therefore $\\triangle R T S \\sim \\triangle B M C$. Now the relations between angles become $$ \\angle T P S+\\angle T Q R=\\angle P T Q \\quad \\text { and } \\quad \\angle T P Q+\\angle T S R=\\angle P T S $$ implying that $P Q \\| R S$ and $Q R \\| P S$. Hence $P Q R S$ is a parallelogram and hence $A B=P Q=R S$ and $Q R=P S$. It follows that $\\frac{B C}{M C}=\\frac{R S}{T S}=$ $\\frac{A B}{M D} \\Rightarrow A B \\cdot C M=B C \\cdot M D$ and $\\frac{A D \\cdot B M}{A M}=\\frac{A D \\cdot Q T}{A M}=Q R=P S=$ $\\frac{C D \\cdot T S}{M D}=C D \\Rightarrow B M \\cdot A D=M A \\cdot C D$.","problem_type":"Geometry","tier":0} +{"year":"1999","problem_phase":"shortlisted","problem":"14. G8 (RUS) Points $A, B, C$ divide the circumcircle $\\Omega$ of the triangle $A B C$ into three arcs. Let $X$ be a variable point on the $\\operatorname{arc} A B$, and let $O_{1}, O_{2}$ be the incenters of the triangles $C A X$ and $C B X$. Prove that the circumcircle of the triangle $X O_{1} O_{2}$ intersects $\\Omega$ in a fixed point.","solution":"14. We first introduce the same lemma as in problem 12 and state it here without proof. Lemma. Let $A B C$ be a triangle and $I$ the center of its incircle. Let $M$ be the center of the $\\operatorname{arc} \\widehat{B C}$ of the circumcircle not containing $A$. Then $M B=M C=M I$. Let the circle $X O_{1} O_{2}$ intersect the circle $\\Omega$ again at point $T$. Let $M$ and $N$ be respectively the midpoints of $\\operatorname{arcs} \\widehat{B C}$ and $\\widehat{A C}$, and let $P$ be the intersection of $\\Omega$ and the line through $C$ parallel to $M N$. Then the lemma gives $M P=N C=N I=N O_{1}$ and $N P=M C=$ $M I=M O_{2}$. Since $O_{1}$ and $O_{2}$ lie on $X N$ and $X M$ respectively, we have $\\angle N T M=\\angle N X M=\\angle O_{1} X O_{2}=\\angle O_{1} T O_{2}$ and hence $\\angle N T O_{1}=$ $\\angle M T O_{2}$. Moreover, $\\angle T N O_{1}=\\angle T N X=\\angle T M O_{2}$, from which it follows that $\\triangle O_{1} N T \\sim \\triangle O_{2} M T$. Thus $\\frac{N T}{M P}=\\frac{N T}{N O_{1}}=\\frac{M T}{M O_{2}}=\\frac{M T}{N P} \\Rightarrow$ $M P \\cdot M T=N P \\cdot N T \\Rightarrow S_{M P T}=S_{N P T}$. It follows that $T P$ bisects the segment $M N$, and hence it passes through $I$. We conclude that $T$ belongs to the line $P I$ and does not depend on $X$. Remark. An alternative approach is to apply an inversion at point $C$. Points $O_{1}$ and $O_{2}$ become excenters of $\\triangle A X C$ and $\\triangle B X C$, and $T$ becomes the projection of $I_{c}$ onto $A B$.","problem_type":"Geometry","tier":0} +{"year":"1999","problem_phase":"shortlisted","problem":"15. A1 (POL) ${ }^{\\mathrm{IMO} 2}$ Let $n \\geq 2$ be a fixed integer. Find the least constant $C$ such that the inequality $$ \\sum_{ib)$ are respectively their smallest elements. Lemma 1. Numbers $x, y$, and $x+y$ cannot belong to three different sets. Proof. The number $f(x, x+y)=f(y, x+y)$ must belong to both the set containing $y$ and the set containing $x$, a contradiction. Lemma 2. The subset $C$ contains a multiple of $b$. Moreover, if $k b$ is the smallest such multiple, then $(k-1) b \\in B$ and $(k-1) b+1, k b+1 \\in A$. Proof. Let $r$ be the residue of $c$ modulo $b$. If $r=0$, the first statement automatically holds. Let $02$ that for all $s$ we have $f(n, s)=\\frac{1}{s}\\binom{n-1}{s-1}\\binom{n}{s-1}$. We shall prove that the given formula holds also for all $f(n+1, s)$, where $s \\geq 2$. We say that an $(n+1, s)$ - or $(n+1, s+1)$-path is related to a given $(n, s)$ path if it is obtained from the given path by inserting a step $E N$ between two moves or at the beginning or the end of the path. We note that by inserting the step between two moves that form a step one obtains an $(n+1, s)$-path; in all other cases one obtains an $(n+1, s+1)$-path. For each $(n, s)$-path there are exactly $2 n+1-s$ related $(n+1, s+1)$-paths, and for each $(n, s+1)$-path there are $s+1$ related $(n+1, s+1)$-paths. Also, each $(n+1, s+1)$-path is related to exactly $s+1$ different $(n, s)$ - or $(n, s+1)$-paths. Thus: $$ \\begin{aligned} (s+1) f(n+1, s+1) & =(2 n+1-s) f(n, s)+(s+1) f(n, s+1) \\\\ & =\\frac{2 n+1-s}{s}\\binom{n-1}{s-1}\\binom{n}{s-1}+\\binom{n-1}{s}\\binom{n}{s} \\\\ & =\\binom{n}{s}\\binom{n+1}{s}, \\end{aligned} $$ i.e., $f(n+1, s+1)=\\frac{1}{s+1}\\binom{n}{s}\\binom{n+1}{s}$. This completes the proof.","problem_type":"Combinatorics","tier":0} +{"year":"1999","problem_phase":"shortlisted","problem":"22. C2 (CAN) (a) If a $5 \\times n$ rectangle can be tiled using $n$ pieces like those shown in the diagram, prove that $n$ is even. ![](https:\/\/cdn.mathpix.com\/cropped\/2024_11_18_8e985d6b9c83aa3e9d0eg-314.jpg?height=111&width=365&top_left_y=1607&top_left_x=644) (b) Show that there are more than $2 \\cdot 3^{k-1}$ ways to tile a fixed $5 \\times 2 k$ rectangle ( $k \\geq 3$ ) with $2 k$ pieces. (Symmetric constructions are considered to be different.)","solution":"22. (a) Color the first, third, and fifth row red, and the remaining squares white. There in total $n$ pieces and $3 n$ red squares. Since each piece can cover at most three red squares, it follows that each piece colors exactly three red squares. Then it follows that the two white squares it covers must be on the same row; otherwise, the piece has to cover at least three. Hence, each white row can be partitioned into pairs of squares belonging to the same piece. Thus it follows that the number of white squares in a row, which is $n$, must be even. (b) Let $a_{k}$ denote the number of different tilings of a $5 \\times 2 k$ rectangle. Let $b_{k}$ be the number of tilings that cannot be partitioned into two smaller tilings along a vertical line (without cutting any pieces). It is easy to see that $a_{1}=b_{1}=2, b_{2}=2, a_{2}=6=2 \\cdot 3, b_{3}=4$, and subsequently, by induction, $b_{3 k} \\geq 4, b_{3 k+1} \\geq 2$, and $b_{3 k+2} \\geq 2$. We also have $a_{k}=b_{k}+\\sum_{i=1}^{k-1} b_{i} a_{k-i}$. For $k \\geq 3$ we now have inductively $$ a_{k}>2+\\sum_{i=1}^{k-1} 2 a_{k-i} \\geq 2 \\cdot 3^{k-1}+2 a_{k-1} \\geq 2 \\cdot 3^{k} $$","problem_type":"Combinatorics","tier":0} +{"year":"1999","problem_phase":"shortlisted","problem":"23. C3 (GBR) A biologist watches a chameleon. The chameleon catches flies and rests after each catch. The biologist notices that: (i) the first fly is caught after a resting period of one minute; (ii) the resting period before catching the $2 m$ th fly is the same as the resting period before catching the $m$ th fly and one minute shorter than the resting period before catching the $(2 m+1)$ th fly; (iii) when the chameleon stops resting, he catches a fly instantly. (a) How many flies were caught by the chameleon before his first resting period of 9 minutes? (b) After how many minutes will the chameleon catch his 98th fly? (c) How many flies were caught by the chameleon after 1999 minutes passed?","solution":"23. Let $r(m)$ denote the rest period before the $m$ th catch, $t(m)$ the number of minutes before the $m$ th catch, and $f(n)$ as the number of flies caught in $n$ minutes. We have $r(1)=1, r(2 m)=r(m)$, and $r(2 m+1)=f(m)+1$. We then have by induction that $r(m)$ is the number of ones in the binary representation of $m$. We also have $t(m)=\\sum_{i=1}^{m} r(i)$ and $f(t(m))=m$. From the recursive relations for $r$ we easily derive $t(2 m+1)=2 t(m)+m+1$ and consequently $t(2 m)=2 t(m)+m-r(m)$. We then have, by induction on $p, t\\left(2^{p} m\\right)=2^{p} t(m)+p \\cdot m \\cdot 2^{p-1}-\\left(2^{p}-1\\right) r(m)$. (a) We must find the smallest number $m$ such that $r(m+1)=9$. The smallest number with nine binary digits is $\\overline{111111111}_{2}=511$; hence the required $m$ is 510 . (b) We must calculate $t(98)$. Using the recursive formulas we have $t(98)=$ $2 t(49)+49-r(49), t(49)=2 t(24)+25$, and $t(24)=8 t(3)+36-7 r(3)$. Since we have $t(3)=4, r(3)=2$ and $r(49)=r\\left(\\overline{110001}_{2}\\right)=3$, it follows $t(24)=54 \\Rightarrow t(49)=133 \\Rightarrow t(98)=312$. (c) We must find $m_{c}$ such that $t\\left(m_{c}\\right) \\leq 1999e>2$, it follows that $\\left|\\bigcup_{j=1}^{N} A_{i_{j}}\\right| \\geq \\frac{1}{2}|S|$; hence the chosen $i_{1}<\\cdots3$ be a prime number. For each nonempty subset $T$ of $\\{0,1,2,3, \\ldots, p-1\\}$ let $E(T)$ be the set of all $(p-1)$-tuples $\\left(x_{1}, \\ldots, x_{p-1}\\right)$, where each $x_{i} \\in T$ and $x_{1}+2 x_{2}+\\cdots+(p-1) x_{p-1}$ is divisible by $p$ and let $|E(T)|$ denote the number of elements in $E(T)$. Prove that $$ |E(\\{0,1,3\\})| \\geq|E(\\{0,1,2\\})|, $$ with equality if and only if $p=5$.","solution":"27. Denote $A=\\{0,1,2\\}$ and $B=\\{0,1,3\\}$. Let $f_{T}(x)=\\sum_{a \\in T} x^{a}$. Then define $F_{T}(x)=f_{T}(x) f_{T}\\left(x^{2}\\right) \\cdots f_{T}\\left(x^{p-1}\\right)$. We can write $F_{T}(x)=\\sum_{i=0}^{p(p-1)} a_{i} x^{i}$, where $a_{i}$ is the number of ways to select an array $\\left\\{x_{1}, \\ldots, x_{p-1}\\right\\}$ where $x_{i} \\in T$ for all $i$ and $x_{1}+2 x_{2}+\\cdots+(p-1) x_{p-1}=i$. Let $w=\\cos (2 \\pi \/ p)+$ $i \\sin (2 \\pi \/ p)$, a $p$ th root of unity. Noting that $$ 1+w^{j}+w^{2 j}+\\cdots+w^{(p-1) j}=\\left\\{\\begin{array}{c} p, p \\mid j, \\\\ 0, p \\nmid j, \\end{array}\\right. $$ it follows that $F_{T}(1)+F_{T}(w)+\\cdots+F_{T}\\left(w^{p-1}\\right)=p E(T)$. Since $|A|=|B|=3$, it follows that $F_{A}(1)=F_{B}(1)=3^{p-1}$. We also have for $p \\nmid i, j$ that $F_{T}\\left(w^{i}\\right)=F_{T}(w)$. Finally, we have $$ F_{A}(w)=\\prod_{i=1}^{p-1}\\left(1+w^{i}+w^{2 i}\\right)=\\prod_{i=1}^{p-1} \\frac{1-w^{3 i}}{1-w^{i}}=1 $$ Hence, combining these results, we obtain $$ E(A)=\\frac{3^{p-1}+p-1}{p} \\text { and } E(B)=\\frac{3^{p-1}+(p-1) F_{B}(w)}{p} $$ It remains to demonstrate that $F_{B}(w) \\geq 1$ for all $p$ and that equality holds only for $p=5$. Since $E(B)$ is an integer, it follows that $F_{B}(w)$ is an integer and $F_{B}(w) \\equiv 1(\\bmod p)$. Since $f_{B}\\left(w^{p-i}\\right)=\\overline{f_{B}\\left(w^{i}\\right)}$, it follows that $F_{B}(w)=\\left|f_{B}(w)\\right|^{2}\\left|f_{B}\\left(w^{2}\\right)\\right|^{2} \\cdots\\left|f_{B}\\left(w^{(p-1) \/ 2}\\right)\\right|^{2}>0$. Hence $F_{B}(w) \\geq 1$. It remains to show that $F_{B}(w)=1$ if and only if $p=5$. We have the formula $(x-w)\\left(x-w^{2}\\right) \\cdots\\left(x-w^{p-1}\\right)=x^{p-1}+x^{p-2}+\\cdots+x+1=\\frac{x^{p}-1}{x-1}$. Let $f_{B}(x)=x^{3}+x+1=(x-\\lambda)(x-\\mu)(x-\\nu)$, where $\\lambda$, $\\mu$, and $\\nu$ are the three zeros of the polynomial $f_{B}(x)$. It follows that $F_{B}(w)=\\left(\\frac{\\lambda^{p}-1}{\\lambda-1}\\right)\\left(\\frac{\\mu^{p}-1}{\\mu-1}\\right)\\left(\\frac{\\nu^{p}-1}{\\nu-1}\\right)=-\\frac{1}{3}\\left(\\lambda^{p}-1\\right)\\left(\\mu^{p}-1\\right)\\left(\\nu^{p}-1\\right)$, since $(\\lambda-1)(\\mu-1)(\\nu-1)=-f_{B}(1)=-3$. We also have $\\lambda+\\mu+\\nu=0$, $\\lambda \\mu \\nu=-1, \\lambda \\mu+\\lambda \\nu+\\mu \\nu=1$, and $\\lambda^{2}+\\mu^{2}+\\nu^{2}=(\\lambda+\\mu+\\nu)^{2}-2(\\lambda \\mu+$ $\\lambda \\nu+\\mu \\nu)=-2$. By induction (using that $\\left(\\lambda^{r}+\\mu^{r}+\\nu^{r}\\right)+\\left(\\lambda^{r-2}+\\mu^{r-2}+\\right.$ $\\left.\\nu^{r-2}\\right)+\\left(\\lambda^{r-3}+\\mu^{r-3}+\\nu^{r-3}\\right)=0$ ), it follows that $\\lambda^{r}+\\mu^{r}+\\nu^{r}$ is an integer for all $r \\in \\mathbb{N}$. Let us assume $F_{B}(x)=1$. It follows that $\\left(\\lambda^{p}-1\\right)\\left(\\mu^{p}-1\\right)\\left(\\nu^{p}-1\\right)=-3$. Hence $\\lambda^{p}, \\mu^{p}, \\nu^{p}$ are roots of the polynomial $p(x)=x^{3}-q x^{2}+(1+q) x+1$, where $q=\\lambda^{p}+\\mu^{p}+\\nu^{p}$. Since $f_{B}(x)$ is an increasing function in real numbers, it follows that it has only one real root (w.l.o.g.) $\\lambda$, the other two roots being complex conjugates. From $f_{B}(-1)<00$; otherwise, $q\\left(x^{2}-x\\right)$ intersects $x^{3}+x+1$ at a value smaller than $\\lambda$. Additionally, as $p$ increases, $\\lambda^{p}$ approaches 0 , and hence $q$ must increase. For $p=5$ we have $1+w+w^{3}=-w^{2}\\left(1+w^{2}\\right)$ and hence $G(w)=\\prod_{i=1}^{p-1}(1+$ $\\left.w^{2 j}\\right)=1$. For a zero of $f_{B}(x)$ we have $x^{5}=-x^{3}-x^{2}=-x^{2}+x+1$ and hence $q=\\lambda^{5}+\\mu^{5}+\\nu^{5}=-\\left(\\lambda^{2}+\\mu^{2}+\\nu^{2}\\right)+(\\lambda+\\mu+\\nu)+3=5$. For $p>5$ we also have $q \\geq 6$. Assuming again $F_{B}(x)=1$ and defining $p(x)$ as before, we have $p(-1)<0, p(0)>0, p(2)<0$, and $p(x)>0$ for a sufficiently large $x>2$. It follows that $p(x)$ must have three distinct real roots. However, since $\\mu^{p}, \\nu^{p} \\in \\mathbb{R} \\Rightarrow \\nu^{p}=\\overline{\\mu^{p}}=\\mu^{p}$, it follows that $p(x)$ has at most two real roots, which is a contradiction. Hence, it follows that $F_{B}(x)>1$ for $p>5$ and thus $E(A) \\leq E(B)$, where equality holds only for $p=5$.","problem_type":"Combinatorics","tier":0} +{"year":"1999","problem_phase":"shortlisted","problem":"3. N3 (RUS) Prove that there exist two strictly increasing sequences $\\left(a_{n}\\right)$ and $\\left(b_{n}\\right)$ such that $a_{n}\\left(a_{n}+1\\right)$ divides $b_{n}^{2}+1$ for every natural number $n$.","solution":"3. We first prove the following lemma. Lemma. For $d, c \\in \\mathbb{N}$ and $d^{2} \\mid c^{2}+1$ there exists $b \\in \\mathbb{N}$ such that $d^{2}\\left(d^{2}+1\\right) \\mid b^{2}+1$. Proof. It is enough to set $b=c+d^{2} c-d^{3}=c+d^{2}(c-d)$. Using the lemma it suffices to find increasing sequences $d_{n}$ and $c_{n}$ such that $c_{n}-d_{n}$ is an increasing sequence and $d_{n}^{2} \\mid c_{n}^{2}+1$. We then obtain the desired sequences $a_{n}$ and $b_{n}$ from $a_{n}=d_{n}^{2}$ and $b_{n}=c_{n}+d_{n}^{2}\\left(c_{n}-d_{n}\\right)$. It is easy to check that $d_{n}=2^{2 n}+1$ and $c_{n}=2^{n d_{n}}$ satisfy the required conditions. Hence we have demonstrated the existence of increasing sequences $a_{n}$ and $b_{n}$ such that $a_{n}\\left(a_{n}+1\\right) \\mid b_{n}^{2}+1$. Remark. There are many solutions to this problem. For example, it is sufficient to prove that the Pell-type equation $5 a_{n}\\left(a_{n}+1\\right)=b_{n}^{2}+1$ has an infinity of solutions in positive integers. Alternatively, one can show that $a_{n}\\left(a_{n}+1\\right)$ can be represented as a sum of two coprime squares for infinitely many $a_{n}$, which implies the existence of $b_{n}$.","problem_type":"Number Theory","tier":0} +{"year":"1999","problem_phase":"shortlisted","problem":"4. N4 (FRA) Denote by $S$ the set of all primes $p$ such that the decimal representation of $1 \/ p$ has its fundamental period divisible by 3 . For every $p \\in S$ such that $1 \/ p$ has its fundamental period $3 r$ one may write $1 \/ p=$ $0 . a_{1} a_{2} \\ldots a_{3 r} a_{1} a_{2} \\ldots a_{3 r} \\ldots$, where $r=r(p)$; for every $p \\in S$ and every integer $k \\geq 1$ define $f(k, p)$ by $$ f(k, p)=a_{k}+a_{k+r(p)}+a_{k+2 r(p)} $$ (a) Prove that $S$ is infinite. (b) Find the highest value of $f(k, p)$ for $k \\geq 1$ and $p \\in S$.","solution":"4. (a) The fundamental period of $p$ is the smallest integer $d(p)$ such that $p \\mid 10^{d(p)}-1$. Let $s$ be an arbitrary prime and set $N_{s}=10^{2 s}+10^{s}+1$. In that case $N_{s} \\equiv 3(\\bmod 9)$. Let $p_{s} \\neq 37$ be a prime dividing $N_{s} \/ 3$. Clearly $p_{s} \\neq 3$. We claim that such a prime exists and that $3 \\mid d\\left(p_{s}\\right)$. The prime $p_{s}$ exists, since otherwise $N_{s}$ could be written in the form $N_{s}=3 \\cdot 37^{k} \\equiv$ $3(\\bmod 4)$, while on the other hand for $s>1$ we have $N_{s} \\equiv 1(\\bmod 4)$. Now we prove $3 \\mid d\\left(p_{s}\\right)$. We have $p_{s}\\left|N_{s}\\right| 10^{3 s}-1$ and hence $d\\left(p_{s}\\right) \\mid 3 s$. We cannot have $d\\left(p_{s}\\right) \\mid s$, for otherwise $p_{s}\\left|10^{s}-1 \\Rightarrow p_{s}\\right|\\left(10^{2 s}+\\right.$ $\\left.10^{s}+1,10^{s}-1\\right)=3$; and we cannot have $d\\left(p_{s}\\right) \\mid 3$, for otherwise $p_{s} \\mid 10^{3}-1=999=3^{3} \\cdot 37$, both of which contradict $p_{s} \\neq 3,37$. It follows that $d\\left(p_{s}\\right)=3 s$. Hence for every prime $s$ there exists a prime $p_{s}$ such that $d\\left(p_{s}\\right)=3 s$. It follows that the cardinality of $S$ is infinite. (b) Let $r=r(s)$ be the fundamental period of $p \\in S$. Then $p \\mid 10^{3 r}-1$, $p \\nmid 10^{r}-1 \\Rightarrow p \\mid 10^{2 r}+10^{r}+1$. Let $x_{j}=\\frac{10^{j-1}}{p}$ and $y_{j}=\\left\\{x_{j}\\right\\}=$ $0 . a_{j} a_{j+1} a_{j+2} \\ldots$. Then $a_{j}<10 y_{j}$, and hence $$ f(k, p)=a_{k}+a_{k+r}+a_{k+2 r}<10\\left(y_{k}+y_{k+r}+y_{k+2 r}\\right) . $$ We note that $x_{k}+x_{k+s(p)}+x_{k+2 s(p)}=\\frac{10^{k-1} N_{p}}{p}$ is an integer, from which it follows that $y_{k}+y_{k+s(p)}+y_{k+2 s(p)} \\in \\mathbb{N}$. Hence $y_{k}+y_{k+s(p)}+$ $y_{k+2 s(p)} \\leq 2$. It follows that $f(k, p)<20$. We note that $f(2,7)=$ $4+8+7=19$. Hence 19 is the greatest possible value of $f(k, p)$.","problem_type":"Number Theory","tier":0} +{"year":"1999","problem_phase":"shortlisted","problem":"5. N5 (ARM) Let $n, k$ be positive integers such that $n$ is not divisible by 3 and $k \\geq n$. Prove that there exists a positive integer $m$ that is divisible by $n$ and the sum of whose digits in decimal representation is $k$.","solution":"5. Since one can arbitrarily add zeros at the end of $m$, which increases divisibility by 2 and 5 to an arbitrary exponent, it suffices to assume $2,5 \\nmid n$. If $(n, 10)=1$, there exists an integer $w \\geq 2$ such that $10^{w} \\equiv 1(\\bmod n)$. We also note that $10^{i w} \\equiv 1(\\bmod n)$ and $10^{j w+1} \\equiv 10(\\bmod n)$ for all integers $i$ and $j$. Let us assume that $m$ is of the form $m=\\sum_{i=1}^{u} 10^{i w}+\\sum_{j=1}^{v} 10^{j w+1}$ for integers $u, v \\geq 0$ (where if $u$ or $v$ is 0 , the corresponding sum is 0 ). Obviously, the sum of the digits of $m$ is equal to $u+v$, and also $m \\equiv u+10 v(\\bmod n)$. Hence our problem reduces to finding integers $u, v \\geq 0$ such that $u+v=k$ and $n \\mid u+10 v=k+9 v$. Since $(n, 9)=1$, it follows that there exists some $v_{0}$ such that $0 \\leq v_{0}$ $w+1$. Now suppose that $r<100$. Since $w+r=(w-1)+(r+1), r+1$ must be in the blue box. But then $(r+1)+w=r+(w+1)$ implies that $w+1$ must be red, which is a contradiction. Hence the red box contains only the card 100 . Since $99+w=100+(w-1)$, we deduce that the card 99 is in the white box. Moreover, if any of the cards $k$, $2 \\leq k \\leq 99$, were in the blue box, then since $k+99=(k-1)+100$, the card $k-1$ should be in the red box, which is impossible. Hence the blue box contains only the card 1 , whereas the cards $2,3, \\ldots, 99$ are all in the white box. In general, one box contains 1 , another box only 100 , while the remaining contains all the other cards. There are exactly 6 such arrangements, and the trick works in each of them. Therefore the answer is 12.","problem_type":null,"tier":0} +{"year":"2000","problem_phase":"shortlisted","problem":"10. A4 (GBR) The function $F$ is defined on the set of nonnegative integers and takes nonnegative integer values satisfying the following conditions: For every $n \\geq 0$, (i) $F(4 n)=F(2 n)+F(n)$; (ii) $F(4 n+2)=F(4 n)+1$; (iii) $F(2 n+1)=F(2 n)+1$. Prove that for each positive integer $m$, the number of integers $n$ with $0 \\leq n<2^{m}$ and $F(4 n)=F(3 n)$ is $F\\left(2^{m+1}\\right)$.","solution":"10. Clearly $F(0)=0$ by (i). Moreover, it follows by induction from (i) that $F\\left(2^{n}\\right)=f_{n+1}$ where $f_{n}$ denotes the $n$th Fibonacci's number. In general, if $n=\\epsilon_{k} 2^{k}+\\epsilon_{k-1} 2^{k-1}+\\cdots+\\epsilon_{1} \\cdot 2+\\epsilon_{0}$ (where $\\epsilon_{i} \\in\\{0,1\\}$ ), it is straightforward to verify that $$ F(n)=\\epsilon_{k} f_{k+1}+\\epsilon_{k-1} f_{k}+\\cdots+\\epsilon_{1} f_{2}+\\epsilon_{0} f_{1} $$ We observe that if the binary representation of $n$ contains no two adjacent ones, then $F(3 n)=F(4 n)$. Indeed, if $n=\\epsilon_{k_{r}} 2^{k_{r}}+\\cdots+\\epsilon_{k_{0}} 2^{k_{0}}$, where $k_{i+1}-k_{i} \\geq 2$ for all $i$, then $3 n=\\epsilon_{k_{r}}\\left(2^{k_{r}+1}+2^{k_{r}}\\right)+\\cdots+\\epsilon_{k_{0}}\\left(2^{k_{0}+1}+2^{k_{0}}\\right)$. According to this, in computing $F(3 n)$ each $f_{i+1}$ in (1) is replaced by $f_{i+1}+f_{i+2}=f_{i+3}$, leading to the value of $F(4 n)$. We shall prove the converse: $F(3 n) \\leq F(4 n)$ holds for all $n \\geq 0$, with equality if and only if the binary representation of $n$ contains no two adjacent ones. We prove by induction on $m \\geq 1$ that this holds for all $n$ satisfying $0 \\leq n<$ $2^{m}$. The verification for the early values of $m$ is direct. Assume it is true for a certain $m$ and let $2^{m} \\leq n \\leq 2^{n+1}$. If $n=2^{m}+p, 0 \\leq p<2^{m}$, then (1) implies $F(4 n)=F\\left(2^{m+2}+4 p\\right)=f_{m+3}+F(4 p)$. Now we distinguish three cases: (i) If $3 p<2^{m}$, then the binary representation of $3 p$ does not carry into that of $3 \\cdot 2^{m}$. Then it follows from (1) and the induction hypothesis that $F(3 n)=F\\left(3 \\cdot 2^{m}\\right)+F(3 p)=f_{m+3}+F(3 p) \\leq f_{m+3}+F(4 p)=F(4 n)$. Equality holds if and only if $F(3 p)=F(4 p)$, i.e. $p$ has no two adjacent binary ones. (ii) If $2^{m} \\leq 3 p<2^{m+1}$, then the binary representation of $3 p$ carries 1 into that of $3 \\cdot 2^{m}$. Thus $F(3 n)=f_{m+3}+\\left(F(3 p)-f_{m+1}\\right)=f_{m+2}+F(3 p)<$ $f_{m+3}+F(4 p)=F(4 n)$. (iii) If $2^{m+1} \\leq p<3 \\cdot 2^{m}$, then the binary representation of $3 p$ caries 10 into that of $3 \\cdot 2^{m}$, which implies $$ F(3 n)=f_{m+3}+f_{m+1}+\\left(F(3 p)-f_{m+2}\\right)=2 f_{m+1}+F(3 p)0$, this proves the result.","problem_type":"Algebra","tier":0} +{"year":"2000","problem_phase":"shortlisted","problem":"12. A6 (IRE) A nonempty set $A$ of real numbers is called a $B_{3}$-set if the conditions $a_{1}, a_{2}, a_{3}, a_{4}, a_{5}, a_{6} \\in A$ and $a_{1}+a_{2}+a_{3}=a_{4}+a_{5}+a_{6}$ imply that the sequences $\\left(a_{1}, a_{2}, a_{3}\\right)$ and $\\left(a_{4}, a_{5}, a_{6}\\right)$ are identical up to a permutation. Let $A=\\left\\{a_{0}=0g(i) \\geq 0$ that satisfy $b_{i}-b_{i-1}=$ $a_{f(i)}-a_{g(i)}$ for all $i$. The number $b_{i+1}-b_{i-1} \\in D(B)=D(A)$ can be written in the form $a_{u}-a_{v}, u>v \\geq 0$. Then $b_{i+1}-b_{i-1}=b_{i+1}-b_{i}+b_{i}-b_{i-1}$ implies $a_{f(i+1)}+a_{f(i)}+a_{v}=a_{g(i+1)}+a_{g(i)}+a_{u}$, so the $B_{3}$ property of $A$ implies that $(f(i+1), f(i), v)$ and $(g(i+1), g(i), u)$ coincide up to a permutation. It follows that either $f(i+1)=g(i)$ or $f(i)=g(i+1)$. Hence if we define $R=\\left\\{i \\in \\mathbb{N}_{0} \\mid f(i+1)=g(i)\\right\\}$ and $S=\\left\\{i \\in \\mathbb{N}_{0} \\mid f(i)=g(i+1)\\right\\}$ it holds that $R \\cup S=\\mathbb{N}_{0}$. Lemma. If $i \\in R$, then also $i+1 \\in R$. Proof. Suppose to the contrary that $i \\in R$ and $i+1 \\in S$, i.e., $g(i)=$ $f(i+1)=g(i+2)$. There are integers $x$ and $y$ such that $b_{i+2}-b_{i-1}=$ $a_{x}-a_{y}$. Then $a_{x}-a_{y}=a_{f(i+2)}-a_{g(i+2)}+a_{f(i+1)}-a_{g(i+1)}+a_{f(i)}-$ $a_{g(i)}=a_{f(i+2)}+a_{f(i)}-a_{g(i+1)}-a_{g(i)}$, so by the $B_{3}$ property $(x, g(i+$ $1), g(i))$ and $(y, f(i+2), f(i))$ coincide up to a permutation. But this is impossible, since $f(i+2), f(i)>g(i+2)=g(i)=f(i+1)>g(i+1)$. This proves the lemma. Therefore if $i \\in R \\neq \\emptyset$, then it follows that every $j>i$ belongs to $R$. Consequently $g(i)=f(i+1)>g(i+1)=f(i+2)>g(i+2)=f(i+3)>$ $\\cdots$ is an infinite decreasing sequence of nonnegative integers, which is impossible. Hence $S=\\mathbb{N}_{0}$, i.e., $$ b_{i+1}-b_{i}=a_{f(i+1)}-a_{f(i)} \\quad \\text { for all } i \\in \\mathbb{N}_{0} $$ Thus $f(0)=g(1)0$. Furthermore, by the induction hypothesis the polynomials of the form $Q_{k}(x)$ take at least $2^{k-2}$ values at $x=n$. Hence the total number of values of $Q(n)$ for $Q \\in M(P)$ is at least $1+1+2+2^{2}+\\cdots+2^{m-1}=2^{m}$. Now we return to the main result. Suppose that $P(x)=a_{2000} x^{2000}$ $+a_{1999} x^{1999}+a_{0}$ is an $n$-independent polynomial. Since $P_{2}(x)=a_{2000} x^{2000}$ $+a_{1998} x^{1998}+\\cdots+a_{2} x^{2}+a_{0}$ is a polynomial in $t=x^{2}$ of degree 1000 , by the lemma it takes at least $2^{1000}$ distinct values at $x=n$. Hence $\\{Q(n) \\mid Q \\in$ $M(P)\\}$ contains at least $2^{1000}$ elements. On the other hand, interchanging the coefficients $b_{i}$ and $b_{j}$ in a polynomial $Q(x)=b_{2000} x^{2000}+\\cdots+b_{0}$ modifies the value of $Q$ at $x=n$ by $\\left(b_{i}-b_{j}\\right)\\left(n^{i}-n^{j}\\right)=\\left(a_{k}-a_{l}\\right)\\left(n^{i}-n^{j}\\right)$ for some $k, l$. Hence there are fewer than $2001^{4}$ possible modifications of the value at $n$. Since $2001^{4}<2^{1000}$, we have arrived at a contradiction.","problem_type":"Algebra","tier":0} +{"year":"2000","problem_phase":"shortlisted","problem":"14. N1 (JAP) Determine all positive integers $n \\geq 2$ that satisfy the following condition: For all integers $a, b$ relatively prime to $n$, $$ a \\equiv b(\\bmod n) \\quad \\text { if and only if } \\quad a b \\equiv 1(\\bmod n) $$","solution":"14. The given condition is obviously equivalent to $a^{2} \\equiv 1(\\bmod n)$ for all integers $a$ coprime to $n$. Let $n=p_{1}^{\\alpha_{1}} p_{2}^{\\alpha_{2}} \\cdots p_{k}^{\\alpha_{k}}$ be the factorization of $n$ onto primes. Since by the Chinese remainder theorem the numbers coprime to $n$ can give any remainder modulo $p_{i}^{\\alpha_{i}}$ except 0 , our condition is equivalent to $a^{2} \\equiv 1\\left(\\bmod p_{i}^{\\alpha_{i}}\\right)$ for all $i$ and integers $a$ coprime to $p_{i}$. Now if $p_{i} \\geq 3$, we have $2^{2} \\equiv 1\\left(\\bmod p_{i}^{\\alpha_{i}}\\right)$, so $p_{i}=3$ and $\\alpha_{i}=2$. If $p_{j}=2$, then $3^{2} \\equiv 1\\left(\\bmod 2^{\\alpha_{j}}\\right)$ implies $\\alpha_{j} \\leq 3$. Hence $n$ is a divisor of $2^{3} \\cdot 3=24$. Conversely, each $n \\mid 24$ has the desired property.","problem_type":"Number Theory","tier":0} +{"year":"2000","problem_phase":"shortlisted","problem":"15. N2 (FRA) For a positive integer $n$, let $d(n)$ be the number of all positive divisors of $n$. Find all positive integers $n$ such that $d(n)^{3}=4 n$.","solution":"15. Let $n=p_{1}^{\\alpha_{1}} p_{2}^{\\alpha_{2}} \\cdots p_{k}^{\\alpha_{k}}$ be the factorization of $n$ onto primes $\\left(p_{1}5$ or $\\beta_{i}>1$, then $\\frac{p_{i}^{\\beta_{i}}}{3 \\beta_{i}+1}>\\frac{5}{4}$, which is impossible. We conclude that $p_{2}=5$ and $k=2$, so $n \\stackrel{2000}{=}$. Hence the solutions for $n$ are 2, 128, and 2000.","problem_type":"Number Theory","tier":0} +{"year":"2000","problem_phase":"shortlisted","problem":"16. N3 (RUS) ${ }^{\\mathrm{IMO} 5}$ Does there exist a positive integer $n$ such that $n$ has exactly 2000 prime divisors and $2^{n}+1$ is divisible by $n$ ?","solution":"16. More generally, we will prove by induction on $k$ that for each $k \\in \\mathbb{N}$ there exists $n_{k} \\in \\mathbb{N}$ that has exactly $k$ distinct prime divisors such that $n_{k} \\mid 2^{n_{k}}+1$ and $3 \\mid n_{k}$. For $k=1, n_{1}=3$ satisfies the given conditions. Now assume that $k \\geq 1$ and $n_{k}=3^{\\alpha} m$ where $3 \\nmid m$, so that $m$ has exactly $k-1$ prime divisors. Then the number $3 n_{k}=3^{\\alpha+1} m$ has exactly $k$ prime divisors and $2^{3 n_{k}}+1=$ $\\left(2^{n_{k}}+1\\right)\\left(2^{2 n_{k}}-2^{n_{k}}+1\\right)$ is divisible by $3 n_{k}$, since $3 \\mid 2^{2 n_{k}}-2^{n_{k}}+1$. We shall find a prime $p$ not dividing $n_{k}$ such that $n_{k+1}=3 p n_{k}$. It is enough to find $p$ such that $p \\mid 2^{3 n_{k}}+1$ and $p \\nmid 2^{n_{k}}+1$. Moreover, we shall show that for every integer $a>2$ there exists a prime number $p$ that divides $a^{3}+1=(a+1)\\left(a^{2}-a+1\\right)$ but not $a+1$. To prove this we observe that $\\operatorname{gcd}\\left(a^{2}-a+1, a+1\\right)=\\operatorname{gcd}(3, a+1)$. Now if $3 \\nmid a+1$, we can simply take $p=3$; otherwise, if $a=3 b-1$, then $a^{2}-a+1=9 b^{2}-9 b+3$ is not divisible by $3^{2}$; hence we can take for $p$ any prime divisor of $\\frac{a^{2}-a+1}{3}$.","problem_type":"Number Theory","tier":0} +{"year":"2000","problem_phase":"shortlisted","problem":"17. N4\uff08BRA\uff09Determine all triples of positive integers $(a, m, n)$ such that $a^{m}+1$ divides $(a+1)^{n}$.","solution":"17. Trivially all triples $(a, 1, n)$ and $(1, m, n)$ are solutions. Assume now that $a>1$ and $m>1$. If $m$ is even, then $a^{m}+1 \\equiv(-1)^{m}+1 \\equiv 2(\\bmod a+1)$, which implies that $a^{m}+1=2^{t}$. In particular, $a$ is odd. But this is impossible, since $2p$ for $p>3$, so we must have $P=p=3$ and $b=2$. Since $b=a^{m_{1}}$, we obtain $a=2$ and $m=3$. The triple $(2,3, n)$ is indeed a solution if $n \\geq 2$. Hence the set of solutions is $\\{(a, 1, n),(1, m, n) \\mid a, m, n \\in \\mathbb{N}\\} \\cup\\{(2,3, n) \\mid$ $n \\geq 2\\}$. Remark. This problem is very similar to (SL97-14).","problem_type":"Number Theory","tier":0} +{"year":"2000","problem_phase":"shortlisted","problem":"18. N5 (BUL) Prove that there exist infinitely many positive integers $n$ such that $p=n r$, where $p$ and $r$ are respectively the semiperimeter and the inradius of a triangle with integer side lengths.","solution":"18. It is known that the area of the triangle is $S=p r=p^{2} \/ n$ and $S=$ $\\sqrt{p(p-a)(p-b)(p-c)}$. It follows that $p^{3}=n^{2}(p-a)(p-b)(p-c)$, which by putting $x=p-a, y=p-b$, and $z=p-c$ transforms into $$ (x+y+z)^{3}=n^{2} x y z . $$ We will be done if we show that (1) has a solution in positive integers for infinitely many natural numbers $n$. Let us assume that $z=k(x+y)$ for an integer $k>0$. Then (1) becomes $(k+1)^{3}(x+y)^{2}=k n^{2} x y$. Further, by setting $n=3(k+1)$ this equation reduces to $$ (k+1)(x+y)^{2}=9 k x y $$ Set $t=x \/ y$. Then (2) has solutions in positive integers if and only if $(k+$ 1) $(t+1)^{2}=9 k t$ has a rational solution, i.e., if and only if its discriminant $D=k(5 k-4)$ is a perfect square. Setting $k=u^{2}$, we are led to show that $5 u^{2}-4=v^{2}$ has infinitely many integer solutions. But this is a classic Pell-type equation, whose solution is every Fibonacci number $u=F_{2 i+1}$. This completes the proof.","problem_type":"Number Theory","tier":0} +{"year":"2000","problem_phase":"shortlisted","problem":"19. N6 (ROM) Show that the set of positive integers that cannot be represented as a sum of distinct perfect squares is finite.","solution":"19. Suppose that a natural number $N$ satisfies $N=a_{1}^{2}+\\cdots+a_{k}^{2}, 2 N=$ $b_{1}^{2}+\\cdots+b_{l}^{2}$, where $a_{i}, b_{j}$ are natural numbers such that none of the ratios $a_{i} \/ a_{j}, b_{i} \/ b_{j}, a_{i} \/ b_{j}, b_{j} \/ a_{i}$ is a power of 2. We claim that every natural number $n>\\sum_{i=0}^{4 N-2}(2 i N+1)^{2}$ can be represented as a sum of distinct squares. Suppose $n=4 q N+r, 0 \\leq r<4 N$. Then $$ n=4 N s+\\sum_{i=0}^{r-1}(2 i N+1)^{2} $$ for some positive integer $s$, so it is enough to show that $4 N s$ is a sum of distinct even squares. Let $s=\\sum_{c=1}^{C} 2^{2 u_{c}}+\\sum_{d=1}^{D} 2^{2 v_{d}+1}$ be the binary expansion of $s$. Then $$ 4 N s=\\sum_{c=1}^{C} \\sum_{i=1}^{k}\\left(2^{u_{c}+1} a_{i}\\right)^{2}+\\sum_{d=1}^{D} \\sum_{j=1}^{l}\\left(2^{u_{d}+1} b_{j}\\right)^{2} $$ where all the summands are distinct by the condition on $a_{i}, b_{j}$. It remains to choose an appropriate $N$ : for example $N=29$, because $29=5^{2}+2^{2}$ and $58=7^{2}+3^{2}$. Second solution. It can be directly checked that every odd integer $67<$ $n \\leq 211$ can be represented as a sum of distinct squares. For any $n>211$ we can choose an integer $m$ such that $m^{2}>\\frac{n}{2}$ and $n-m^{2}$ is odd and greater than 67 , and therefore by the induction hypothesis can be written as a sum of distinct squares. Hence $n$ is also a sum of distinct squares.","problem_type":"Number Theory","tier":0} +{"year":"2000","problem_phase":"shortlisted","problem":"2. C2 (ITA) A brick staircase with three steps of width 2 is made of twelve unit cubes. Determine all integers $n$ for which it is possible to build a cube of side $n$ using such bricks. ![](https:\/\/cdn.mathpix.com\/cropped\/2024_11_18_8e985d6b9c83aa3e9d0eg-317.jpg?height=175&width=141&top_left_y=739&top_left_x=725)","solution":"2. Since the volume of each brick is 12 , the side of any such cube must be divisible by 6 . Suppose that a cube of side $n=6 k$ can be built using $\\frac{n^{3}}{12}=18 k^{3}$ bricks. Set a coordinate system in which the cube is given as $[0, n] \\times[0, n] \\times[0, n]$ and color in black each unit cube $[2 p, 2 p+1] \\times[2 q, 2 q+1] \\times[2 r, 2 r+1]$. There are exactly $\\frac{n^{3}}{9}=27 k^{3}$ black cubes. Each brick covers either one or three black cubes, which is in any case an odd number. It follows that the total number of black cubes must be even, which implies that $k$ is even. Hence $12 \\mid n$. On the other hand, two bricks can be fitted together to give a $2 \\times 3 \\times 4$ box. Using such boxes one can easily build a cube of side 12 , and consequently any cube of side divisible by 12 .","problem_type":"Combinatorics","tier":0} +{"year":"2000","problem_phase":"shortlisted","problem":"20. G1 (NET) In the plane we are given two circles intersecting at $X$ and $Y$. Prove that there exist four points $A, B, C, D$ with the following property: For every circle touching the two given circles at $A$ and $B$, and meeting the line $X Y$ at $C$ and $D$, each of the lines $A C, A D, B C, B D$ passes through one of these points.","solution":"20. Denote by $k_{1}, k_{2}$ the given circles and by $k_{3}$ the circle through $A, B, C, D$. We shall consider the case that $k_{3}$ is inside $k_{1}$ and $k_{2}$, since the other case is analogous. Let $A C$ and $A D$ meet $k_{1}$ at points $P$ and $R$, and $B C$ and $B D$ meet $k_{2}$ at $Q$ and $S$ respectively. We claim that $P Q$ and $R S$ are the common tangents to $k_{1}$ and $k_{2}$, and therefore $P, Q, R, S$ are the desired points. The circles $k_{1}$ and $k_{3}$ are tangent to each other, so we have $D C \\| R P$. Since ![](https:\/\/cdn.mathpix.com\/cropped\/2024_11_18_8e985d6b9c83aa3e9d0eg-680.jpg?height=374&width=468&top_left_y=669&top_left_x=848) $$ A C \\cdot C P=X C \\cdot C Y=B C \\cdot C Q $$ the quadrilateral $A B Q P$ is cyclic, implying that $\\angle A P Q=\\angle A B Q=$ $\\angle A D C=\\angle A R P$. It follows that $P Q$ is tangent to $k_{1}$. Similarly, $P Q$ is tangent to $k_{2}$.","problem_type":"Geometry","tier":0} +{"year":"2000","problem_phase":"shortlisted","problem":"21. G2 (RUS) ${ }^{\\mathrm{IMO} 1}$ Two circles $G_{1}$ and $G_{2}$ intersect at $M$ and $N$. Let $A B$ be the line tangent to these circles at $A$ and $B$, respectively, such that $M$ lies closer to $A B$ than $N$. Let $C D$ be the line parallel to $A B$ and passing through $M$, with $C$ on $G_{1}$ and $D$ on $G_{2}$. Lines $A C$ and $B D$ meet at $E$; lines $A N$ and $C D$ meet at $P$; lines $B N$ and $C D$ meet at $Q$. Show that $E P=E Q$.","solution":"21. Let $K$ be the intersection point of the lines $M N$ and $A B$. Since $K A^{2}=K M \\cdot K N=K B^{2}$, it follows that $K$ is the midpoint of the segment $A B$, and consequently $M$ is the midpoint of $A B$. Thus it will be enough to show that $E M \\perp$ $P Q$, or equivalently that $E M \\perp$ $A B$. However, since $A B$ is tangent to the circle $G_{1}$ we have $\\angle B A M=$ ![](https:\/\/cdn.mathpix.com\/cropped\/2024_11_18_8e985d6b9c83aa3e9d0eg-680.jpg?height=356&width=509&top_left_y=1295&top_left_x=812) $\\angle A C M=\\angle E A B$, and similarly $\\angle A B M=\\angle E B A$. This implies that the triangles $E A B$ and $M A B$ are congruent. Hence $E$ and $M$ are symmetric with respect to $A B$; hence $E M \\perp A B$. Remark. The proposer has suggested an alternative version of the problem: to prove that $E N$ bisects the angle $C N D$. This can be proved by noting that $E A N B$ is cyclic.","problem_type":"Geometry","tier":0} +{"year":"2000","problem_phase":"shortlisted","problem":"22. G3 (IND) Let $O$ be the circumcenter and $H$ the orthocenter of an acute triangle $A B C$. Show that there exist points $D, E$, and $F$ on sides $B C$, $C A$, and $A B$ respectively such that $O D+D H=O E+E H=O F+F H$ and the lines $A D, B E$, and $C F$ are concurrent.","solution":"22. Let $L$ be the point symmetric to $H$ with respect to $B C$. It is well known that $L$ lies on the circumcircle $k$ of $\\triangle A B C$. Let $D$ be the intersection point of $O L$ and $B C$. We similarly define $E$ and $F$. Then $$ O D+D H=O D+D L=O L=O E+E H=O F+F H $$ We shall prove that $A D, B E$, and $C F$ are concurrent. Let line $A O$ meet $B C$ at $D^{\\prime}$. It is easy to see that $\\angle O D^{\\prime} D=\\angle O D D^{\\prime}$; hence the perpendicular bisector of $B C$ bisects $D D^{\\prime}$ as well. Hence $B D=C D^{\\prime}$. If we define $E^{\\prime}$ and $F^{\\prime}$ analogously, we have $C E=A E^{\\prime}$ and $A F=B F^{\\prime}$. Since the lines $A D^{\\prime}, B E^{\\prime}, C F^{\\prime}$ meet at $O$, it follows that $\\frac{B D}{D C} \\cdot \\frac{C E}{E A} \\cdot \\frac{A F}{F B}=$ ![](https:\/\/cdn.mathpix.com\/cropped\/2024_11_18_8e985d6b9c83aa3e9d0eg-681.jpg?height=399&width=430&top_left_y=252&top_left_x=867) $\\frac{B D^{\\prime}}{D^{\\prime} C} \\cdot \\frac{C E^{\\prime}}{E^{\\prime} A} \\cdot \\frac{A F^{\\prime}}{F^{\\prime} B}=1$. This proves our claim by Ceva's theorem.","problem_type":"Geometry","tier":0} +{"year":"2000","problem_phase":"shortlisted","problem":"23. G4 (RUS) Let $A_{1} A_{2} \\ldots A_{n}$ be a convex polygon, $n \\geq 4$. Prove that $A_{1} A_{2} \\ldots A_{n}$ is cyclic if and only if to each vertex $A_{j}$ one can assign a pair $\\left(b_{j}, c_{j}\\right)$ of real numbers, $j=1,2, \\ldots n$, such that $$ A_{i} A_{j}=b_{j} c_{i}-b_{i} c_{j} \\quad \\text { for all } i, j \\text { with } 1 \\leq i \\leq j \\leq n $$","solution":"23. First, suppose that there are numbers $\\left(b_{i}, c_{i}\\right)$ assigned to the vertices of the polygon such that $$ A_{i} A_{j}=b_{j} c_{i}-b_{i} c_{j} \\quad \\text { for all } i, j \\text { with } 1 \\leq i \\leq j \\leq n $$ In order to show that the polygon is cyclic, it is enough to prove that $A_{1}, A_{2}, A_{3}, A_{i}$ lie on a circle for each $i, 4 \\leq i \\leq n$, or equivalently, by Ptolemy's theorem, that $A_{1} A_{2} \\cdot A_{3} A_{i}+A_{2} A_{3} \\cdot A_{i} A_{1}=A_{1} A_{3} \\cdot A_{2} A_{i}$. But this is straightforward with regard to (1). Now suppose that $A_{1} A_{2} \\ldots A_{n}$ is a cyclic quadrilateral. By Ptolemy's theorem we have $A_{i} A_{j}=A_{2} A_{j} \\cdot \\frac{A_{1} A_{i}}{A_{1} A_{2}}-A_{2} A_{i} \\cdot \\frac{A_{1} A_{j}}{A_{1} A_{2}}$ for all $i, j$. This suggests taking $b_{1}=-A_{1} A_{2}, b_{i}=A_{2} A_{i}$ for $i \\geq 2$ and $c_{i}=\\frac{A_{1} A_{i}}{A_{1} A_{2}}$ for all $i$. Indeed, using Ptolemy's theorem, one easily verifies (1).","problem_type":"Geometry","tier":0} +{"year":"2000","problem_phase":"shortlisted","problem":"24. G5 (GBR) The tangents at $B$ and $A$ to the circumcircle of an acuteangled triangle $A B C$ meet the tangent at $C$ at $T$ and $U$ respectively. $A T$ meets $B C$ at $P$, and $Q$ is the midpoint of $A P ; B U$ meets $C A$ at $R$, and $S$ is the midpoint of $B R$. Prove that $\\angle A B Q=\\angle B A S$. Determine, in terms of ratios of side lengths, the triangles for which this angle is a maximum.","solution":"24. Since $\\angle A B T=180^{\\circ}-\\gamma$ and $\\angle A C T=180^{\\circ}-\\beta$, the law of sines gives $\\frac{B P}{P C}=\\frac{S_{A B T}}{S_{A C T}}=\\frac{A B \\cdot B T \\cdot \\sin \\gamma}{A B \\cdot B T \\cdot \\sin \\beta}=\\frac{A B \\sin \\gamma}{A C \\sin \\beta}=\\frac{c^{2}}{b^{2}}$, which implies $B P=\\frac{c^{2} a}{b^{2}+c^{2}}$. Denote by $M$ and $N$ the feet of perpendiculars from $P$ and $Q$ on $A B$. We have $\\cot \\angle A B Q=\\frac{B N}{N Q}=\\frac{2 B N}{P M}=\\frac{B A+B M}{B P \\sin \\beta}=\\frac{c+B P \\cos \\beta}{B P \\sin \\beta}=\\frac{b^{2}+c^{2}+a c \\cos \\beta}{c a \\sin \\beta}=$ $\\frac{2\\left(b^{2}+c^{2}\\right)+a^{2}+c^{2}-b^{2}}{2 c a \\sin \\beta}=\\frac{a^{2}+b^{2}+3 c^{2}}{4 S_{A B C}}=2 \\cot \\alpha+2 \\cot \\beta+\\cot \\gamma$. Similarly, $\\cot \\angle B A S=2 \\cot \\alpha+2 \\cot \\beta+\\cot \\gamma$; hence $\\angle A B Q=\\angle B A S$. Now put $p=\\cot \\alpha$ and $q=\\cot \\beta$. Since $p+q \\geq 0$, the A-G mean inequality gives us $\\cot \\angle A B Q=2 p+2 q+\\frac{1-p q}{p+q} \\geq 2 p+2 q+\\frac{1-(p+q)^{2} \/ 4}{p+q}=\\frac{7}{4}(p+q)+$ $\\frac{1}{p+q} \\geq 2 \\sqrt{\\frac{7}{4}}=\\sqrt{7}$. Hence $\\angle A B Q \\leq \\arctan \\frac{1}{\\sqrt{7}}$. Equality holds if and only if $\\cot \\alpha=\\cot \\beta=\\frac{1}{\\sqrt{7}}$, i.e., when $a: b: c=1: 1: \\frac{1}{\\sqrt{2}}$.","problem_type":"Geometry","tier":0} +{"year":"2000","problem_phase":"shortlisted","problem":"25. G6 (ARG) Let $A B C D$ be a convex quadrilateral with $A B$ not parallel to $C D$, let $X$ be a point inside $A B C D$ such that $\\measuredangle A D X=\\measuredangle B C X<90^{\\circ}$ and $\\measuredangle D A X=\\measuredangle C B X<90^{\\circ}$. If $Y$ is the point of intersection of the perpendicular bisectors of $A B$ and $C D$, prove that $\\measuredangle A Y B=2 \\measuredangle A D X$.","solution":"25. By the condition of the problem, $\\triangle A D X$ and $\\triangle B C X$ are similar. Then there exist points $Y^{\\prime}$ and $Z^{\\prime}$ on the perpendicular bisector of $A B$ such that $\\triangle A Y^{\\prime} Z^{\\prime}$ is similar and oriented the same as $\\triangle A D X$, and $\\triangle B Y^{\\prime} Z^{\\prime}$ is (being congruent to $\\triangle A Y^{\\prime} Z^{\\prime}$ ) similar and oriented the same as $\\triangle B C X$. Since then $A D \/ A Y^{\\prime}=A X \/ A Z^{\\prime}$ and $\\angle D A Y^{\\prime}=\\angle X A Z^{\\prime}, \\triangle A D Y^{\\prime}$ and $\\triangle A X Z^{\\prime}$ are also similar, implying $\\frac{A D}{A X}=\\frac{D Y^{\\prime}}{X Z^{\\prime}}$. Analogously, $\\frac{B C}{B X}=\\frac{C Y^{\\prime}}{X Z^{\\prime}}$. It follows from $\\frac{A D}{A X}=\\frac{B C}{B X}$ that $C Y^{\\prime}=D Y^{\\prime}$, which means that $Y^{\\prime}$ lies on the perpendicular bisector of $C D$. Hence $Y^{\\prime} \\equiv Y$. Now $\\angle A Y B=2 \\angle A Y Z^{\\prime}=2 \\angle A D X$, as desired.","problem_type":"Geometry","tier":0} +{"year":"2000","problem_phase":"shortlisted","problem":"26. G7 (IRN) Ten gangsters are standing on a flat surface, and the distances between them are all distinct. At twelve o'clock, when the church bells start chiming, each of them fatally shoots the one among the other nine gangsters who is the nearest. At least how many gangsters will be killed?","solution":"26. The problem can be reformulated in the following way: Given a set $S$ of ten points in the plane such that the distances between them are all distinct, for each point $P \\in S$ we mark the point $Q \\in S \\backslash\\{P\\}$ nearest to $P$. Find the least possible number of marked points. Observe that each point $A \\in S$ is the nearest to at most five other points. Indeed, for any six points $P_{1}, \\ldots, P_{6}$ one of the angles $P_{i} A P_{j}$ is at most $60^{\\circ}$, in which case $P_{i} P_{j}$ is smaller than one of the distances $A P_{i}, A P_{j}$. It follows that at least two points are marked. Now suppose that exactly two points, say $A$ and $B$, are marked. Then $A B$ is the minimal distance of the points from $S$, so by the previous observation the rest of the set $S$ splits into two subsets of four points according to whether the nearest point is $A$ or $B$. Let these subsets be $\\left\\{A_{1}, A_{2}, A_{3}, A_{4}\\right\\}$ and $\\left\\{B_{1}, B_{2}, B_{3}, B_{4}\\right\\}$ respectively. Assume that the points are labelled so that the angles $A_{i} A A_{i+1}$ are successively adjacent as well as the angles $B_{i} B B_{i+1}$, and that $A_{1}, B_{1}$ lie on one side of $A B$, and $A_{4}, B_{4}$ lie on the other side. Since all the angles $A_{i} A A_{i+1}$ and $B_{i} B B_{i+1}$ are greater than $60^{\\circ}$, it follows that $$ \\angle A_{1} A B+\\angle B A A_{4}+\\angle B_{1} B A+\\angle A B B_{4}<360^{\\circ} . $$ Therefore $\\angle A_{1} A B+\\angle B_{1} B A<180^{\\circ}$ or $\\angle A_{4} A B+\\angle B_{4} B A<180^{\\circ}$. Without loss of generality, let us assume the first inequality. On the other hand, note that the quadrilateral $A B B_{1} A_{1}$ is convex because $A_{1}$ and $B_{1}$ are on different sides of the perpendicular bisector of $A B$. From $A_{1} B_{1}>A_{1} A$ and $B B_{1}>A B$ we obtain $\\angle A_{1} A B_{1}>\\angle A_{1} B_{1} A$ and $\\angle B A B_{1}>\\angle A B_{1} B$. Adding these relations yields $\\angle A_{1} A B>\\angle A_{1} B_{1} B$. Similarly, $\\angle B_{1} B A>\\angle B_{1} A_{1} A$. Adding these two inequalities, we get $$ 180^{\\circ}>\\angle A_{1} A B+\\angle B_{1} B A>\\angle A_{1} B_{1} B+\\angle B_{1} A_{1} A $$ hence the sum of the angles of the quadrilateral $A B B_{1} A_{1}$ is less than $360^{\\circ}$, which is a contradiction. Thus at least 3 points are marked. An example of a configuration in which exactly 3 gangsters are killed is shown below. ![](https:\/\/cdn.mathpix.com\/cropped\/2024_11_18_8e985d6b9c83aa3e9d0eg-682.jpg?height=250&width=441&top_left_y=1700&top_left_x=579)","problem_type":"Geometry","tier":0} +{"year":"2000","problem_phase":"shortlisted","problem":"27. G8 (RUS) ${ }^{\\mathrm{IMO}} A_{1} A_{2} A_{3}$ is an acute-angled triangle. The foot of the altitude from $A_{i}$ is $K_{i}$, and the incircle touches the side opposite $A_{i}$ at $L_{i}$. The line $K_{1} K_{2}$ is reflected in the line $L_{1} L_{2}$. Similarly, the line $K_{2} K_{3}$ is reflected in $L_{2} L_{3}$, and $K_{3} K_{1}$ is reflected in $L_{3} L_{1}$. Show that the three new lines form a triangle with vertices on the incircle.","solution":"27. Denote by $\\alpha_{1}, \\alpha_{2}, \\alpha_{3}$ the angles of $\\triangle A_{1} A_{2} A_{3}$ at vertices $A_{1}, A_{2}, A_{3}$ respectively. Let $T_{1}, T_{2}, T_{3}$ be the points symmetric to $L_{1}, L_{2}, L_{3}$ with respect to $A_{1} I, A_{2} I$, and $A_{3} I$ respectively. We claim that $T_{1} T_{2} T_{3}$ is the desired triangle. Denote by $S_{1}$ and $R_{1}$ the points symmetric to $K_{1}$ and $K_{3}$ with respect to $L_{1} L_{3}$. It is enough to show that $T_{1}$ and $T_{3}$ lie on the line $R_{1} S_{1}$. To prove this, we shall prove that $\\angle K_{1} S_{1} T_{1}=\\angle K^{\\prime} K_{1} S_{1}$ for a point $K^{\\prime}$ on the line $K_{1} K_{3}$ such that $K_{3}$ and $K^{\\prime}$ lie on different sides of $K_{1}$. We show first that $S_{1} \\in A_{1} I$. Let $X$ be the point of intersection of lines $A_{1} I$ and $L_{1} L_{3}$. We see from the triangle $A_{1} L_{3} X$ that $\\angle L_{1} X I=$ $\\alpha_{3} \/ 2=\\angle L_{1} A_{3} I$, which implies that ![](https:\/\/cdn.mathpix.com\/cropped\/2024_11_18_8e985d6b9c83aa3e9d0eg-683.jpg?height=489&width=559&top_left_y=289&top_left_x=805) $L_{1} X A_{3} I$ is cyclic. We now have $\\angle A_{1} X A_{3}=90^{\\circ}=\\angle A_{1} K_{1} A_{3}$; hence $A_{1} K_{1} X A_{3}$ is also cyclic. It follows that $\\angle K_{1} X I=\\angle K_{1} A_{3} A_{1}=\\alpha_{3}=2 \\angle L_{1} X I$; hence $X_{1} L_{1}$ bisects the angle $K_{1} X_{1} I$. Hence $S_{1} \\in X I$ as claimed. Now we have $\\angle K_{1} S_{1} T_{1}=\\angle K_{1} S_{1} L_{1}+2 \\angle L_{1} S_{1} X=\\angle S_{1} K_{1} L_{1}+2 \\angle L_{1} K_{1} X$. It remains to prove that $K_{1} X$ bisects $\\angle A_{3} K_{1} K^{\\prime}$. From the cyclic quadrilateral $A_{1} K_{1} X A_{3}$ we see that $\\angle X K_{1} A_{3}=\\alpha_{1} \/ 2$. Since $A_{1} K_{3} K_{1} A_{3}$ is cyclic, we also have $\\angle K^{\\prime} K_{1} A_{3}=\\alpha_{1}=2 \\angle X K_{1} A_{3}$, which proves the claim.","problem_type":"Geometry","tier":0} +{"year":"2000","problem_phase":"shortlisted","problem":"3. C3 (COL) Let $n \\geq 4$ be a fixed positive integer. Given a set $S=$ $\\left\\{P_{1}, P_{2}, \\ldots, P_{n}\\right\\}$ of points in the plane such that no three are collinear and no four concyclic, let $a_{t}, 1 \\leq t \\leq n$, be the number of circles $P_{i} P_{j} P_{k}$ that contain $P_{t}$ in their interior, and let $$ m(S)=a_{1}+a_{2}+\\cdots+a_{n} $$ Prove that there exists a positive integer $f(n)$, depending only on $n$, such that the points of $S$ are the vertices of a convex polygon if and only if $m(S)=f(n)$.","solution":"3. Clearly $m(S)$ is the number of pairs of point and triangle $\\left(P_{t}, P_{i} P_{j} P_{k}\\right)$ such that $P_{t}$ lies inside the circle $P_{i} P_{j} P_{k}$. Consider any four-element set $S_{i j k l}=\\left\\{P_{i}, P_{j}, P_{k}, P_{l}\\right\\}$. If the convex hull of $S_{i j k l}$ is the triangle $P_{i} P_{j} P_{k}$, then we have $a_{i}=a_{j}=a_{k}=0, a_{l}=1$. Suppose that the convex hull is the quadrilateral $P_{i} P_{j} P_{k} P_{l}$. Since this quadrilateral is not cyclic, we may suppose that $\\angle P_{i}+\\angle P_{k}<180^{\\circ}<\\angle P_{j}+\\angle P_{l}$. In this case $a_{i}=a_{k}=0$ and $a_{j}=a_{l}=1$. Therefore $m\\left(S_{i j k l}\\right)$ is 2 if $P_{i}, P_{j}, P_{k}, P_{l}$ are vertices of a convex quadrilateral, and 1 otherwise. There are $\\binom{n}{4}$ four-element subsets $S_{i j k l}$. If $a(S)$ is the number of such subsets whose points determine a convex quadrilateral, we have $m(S)=$ $2 a(S)+\\left(\\binom{n}{4}-a(S)\\right)=\\binom{n}{4}+a(S) \\leq 2\\binom{n}{4}$. Equality holds if and only if every four distinct points of $S$ determine a convex quadrilateral, i.e. if and only if the points of $S$ determine a convex polygon. Hence $f(n)=2\\binom{n}{4}$ has the desired property.","problem_type":"Combinatorics","tier":0} +{"year":"2000","problem_phase":"shortlisted","problem":"4. C4 (CZE) Let $n$ and $k$ be positive integers such that $n \/ 22 a$ and $c>2 b$. Show that there exists a real number $t$ with the property that all the three numbers $t a, t b, t c$ have their fractional parts lying in the interval (1\/3,2\/3].","solution":"8. We note that $\\{t a\\}$ lies in $\\left(\\frac{1}{3}, \\frac{2}{3}\\right]$ if and only if there is an integer $k$ such that $k+\\frac{1}{3}0$. Let $m$ be the number of positive terms among $x_{1}, \\ldots, x_{n}$. Since $x_{i}$ counts the terms equal to $i$, the sum $x_{1}+\\cdots+x_{n}$ counts the total number of positive terms in the sequence, which is known to be $m+1$. Therefore among $x_{1}, \\ldots, x_{n}$ exactly $m-1$ terms are equal to 1 , one is equal to 2 , and the others are 0 . Only $x_{0}$ can exceed 2 , and consequently at most one of $x_{3}, x_{4}, \\ldots$ can be positive. It follows that $m \\leq 3$. (i) $m=1$ : Then $x_{2}=2$ (since $x_{1}=2$ is impossible), so $x_{0}=2$. The resulting sequence is $(2,0,2,0)$. (ii) $m=2$ : Either $x_{1}=2$ or $x_{2}=2$. These cases yield $(1,2,1,0)$ and $(2,1,2,0,0)$ respectively. (iii) $m=3$ : This means that $x_{k}>0$ for some $k>2$. Hence $x_{0}=k$ and $x_{k}=1$. Further, $x_{1}=1$ is impossible, so $x_{1}=2$ and $x_{2}=1$; there are no more positive terms in the sequence. The resulting sequence is $(p, 2,1, \\underbrace{0, \\ldots, 0}_{p-3}, 1,0,0,0)$.","problem_type":"Combinatorics","tier":0} +{"year":"2001","problem_phase":"shortlisted","problem":"12. C6 (CAN) For a positive integer $n$ define a sequence of zeros and ones to be balanced if it contains $n$ zeros and $n$ ones. Two balanced sequences $a$ and $b$ are neighbors if you can move one of the $2 n$ symbols of $a$ to another position to form $b$. For instance, when $n=4$, the balanced sequences 01101001 and 00110101 are neighbors because the third (or fourth) zero in the first sequence can be moved to the first or second position to form the second sequence. Prove that there is a set $S$ of at most $\\frac{1}{n+1}\\binom{2 n}{n}$ balanced sequences such that every balanced sequence is equal to or is a neighbor of at least one sequence in $S$.","solution":"12. For each balanced sequence $a=\\left(a_{1}, a_{2}, \\ldots, a_{2 n}\\right)$ denote by $f(a)$ the sum of $j$ 's for which $a_{j}=1$ (for example, $f(100101)=1+4+6=11$ ). Partition the $\\binom{2 n}{n}$ balanced sequences into $n+1$ classes according to the residue of $f$ modulo $n+1$. Now take $S$ to be a class of minimum size: obviously $|S| \\leq \\frac{1}{n+1}\\binom{2 n}{n}$. We claim that every balanced sequence $a$ is either a member of $S$ or a neighbor of a member of $S$. We consider two cases. (i) Let $a_{1}$ be 1 . It is easy to see that moving this 1 just to the right of the $k$ th 0 , we obtain a neighboring balanced sequence $b$ with $f(b)=$ $f(a)+k$. Thus if $a \\notin S$, taking a suitable $k \\in\\{1,2, \\ldots, n\\}$ we can achieve that $b \\in S$. (ii) Let $a_{1}$ be 0 . Taking this 0 just to the right of the $k$ th 1 gives a neighbor $b$ with $f(b)=f(a)-k$, and the conclusion is similar to that of (i). This justifies our claim.","problem_type":"Combinatorics","tier":0} +{"year":"2001","problem_phase":"shortlisted","problem":"13. C7 (FRA) A pile of $n$ pebbles is placed in a vertical column. This configuration is modified according to the following rules. A pebble can be moved if it is at the top of a column that contains at least two more pebbles than the column immediately to its right. (If there are no pebbles to the right, think of this as a column with 0 pebbles.) At each stage, choose a pebble from among those that can be moved (if there are any) and place it at the top of the column to its right. If no pebbles can be moved, the configuration is called a final configuration. For each $n$, show that no matter what choices are made at each stage, the final configuration is unique. Describe that configuration in terms of $n$.","solution":"13. At any moment, let $p_{i}$ be the number of pebbles in the $i$ th column, $i=$ $1,2, \\ldots$ The final configuration has obvious properties $p_{1} \\geq p_{2} \\geq \\cdots$ and $p_{i+1} \\in\\left\\{p_{i}, p_{i}-1\\right\\}$. We claim that $p_{i+1}=p_{i}>0$ is possible for at most one $i$. Assume the opposite. Then the final configuration has the property that for some $r$ and $s>r$ we have $p_{r+1}=p_{r}, p_{s+1}=p_{s}>0$ and $p_{r+k}=$ $p_{r+1}-k+1$ for all $k=1, \\ldots, s-r$. Consider the earliest configuration, say $C$, with this property. What was the last move before $C$ ? The only possibilities are moving a pebble either from the $r$ th or from the $s$ th column; however, in both cases the configuration preceding this last move had the same property, contradicting the assumption that $C$ is the earliest. Therefore the final configuration looks as follows: $p_{1}=a \\in \\mathbb{N}$, and for some $r, p_{i}$ equals $a-(i-1)$ if $i \\leq r$, and $a-(i-2)$ otherwise. It is easy to determine $a, r$ : since $n=p_{1}+p_{2}+\\cdots=\\frac{(a+1)(a+2)}{2}-r$, we get $\\frac{a(a+1)}{2} \\leq n<\\frac{(a+1)(a+2)}{2}$, from which we uniquely find $a$ and then $r$ as well. The final configuration for $n=13$ : ![](https:\/\/cdn.mathpix.com\/cropped\/2024_11_18_8e985d6b9c83aa3e9d0eg-690.jpg?height=162&width=155&top_left_y=922&top_left_x=1027)","problem_type":"Combinatorics","tier":0} +{"year":"2001","problem_phase":"shortlisted","problem":"14. C8 (GER) ${ }^{\\mathrm{IMO} 3}$ Twenty-one girls and twenty-one boys took part in a mathematical competition. It turned out that (i) each contestant solved at most six problems, and (ii) for each pair of a girl and a boy, there was at least one problem that was solved by both the girl and the boy. Show that there is a problem that was solved by at least three girls and at least three boys.","solution":"14. We say that a problem is difficult for boys if at most two boys solved it, and difficult for girls if at most two girls solved it. Let us estimate the number of pairs boy-girl both of whom solved some problem difficult for boys. Consider any girl. By the condition (ii), among the six problems she solved, at least one was solved by at least 3 boys, and hence at most 5 were difficult for boys. Since each of these problems was solved by at most 2 boys and there are 21 girls, the considered number of pairs does not exceed $5 \\cdot 2 \\cdot 21=210$. Similarly, there are at most 210 pairs boy-girl both of whom solved some problem difficult for girls. On the other hand, there are $21^{2}>2 \\cdot 210$ pairs boy-girl, and each of them solved one problem in common. Thus some problems were difficult neither for girls nor for boys, as claimed. Remark. The statement can be generalized: if $2(m-1)(n-1)+1$ boys and as many girls participated, and nobody solved more than $m$ problems, then some problem was solved by at least $n$ boys and $n$ girls.","problem_type":"Combinatorics","tier":0} +{"year":"2001","problem_phase":"shortlisted","problem":"15. G1 (UKR) Let $A_{1}$ be the center of the square inscribed in acute triangle $A B C$ with two vertices of the square on side $B C$. Thus one of the two remaining vertices of the square is on side $A B$ and the other is on $A C$. Points $B_{1}, C_{1}$ are defined in a similar way for inscribed squares with two vertices on sides $A C$ and $A B$, respectively. Prove that lines $A A_{1}, B B_{1}, C C_{1}$ are concurrent.","solution":"15. Let $M N P Q$ be the square inscribed in $\\triangle A B C$ with $M \\in A B, N \\in A C$, $P, Q \\in B C$, and let $A A_{1}$ meet $M N, P Q$ at $K, X$ respectively. Put $M K=$ $P X=m, N K=Q X=n$, and $M N=d$. Then $$ \\frac{B X}{X C}=\\frac{m}{n}=\\frac{B X+m}{X C+n}=\\frac{B P}{C Q}=\\frac{d \\cot \\beta+d}{d \\cot \\gamma+d}=\\frac{\\cot \\beta+1}{\\cot \\gamma+1} . $$ Similarly, if $B B_{1}$ and $C C_{1}$ meet $A C$ and $B C$ at $Y, Z$ respectively then $\\frac{C Y}{Y A}=\\frac{\\cot \\gamma+1}{\\cot \\alpha+1}$ and $\\frac{A Z}{Z B}=\\frac{\\cot \\alpha+1}{\\cot \\beta+1}$. Therefore $\\frac{B X}{X C} \\frac{C Y}{Y A} \\frac{A Z}{Z B}=1$, so by Ceva's theorem, $A X, B Y, C Z$ have a common point. Second solution. Let $A_{2}$ be the center of the square constructed over $B C$ outside $\\triangle A B C$. Since this square and the inscribed square corresponding to the side $B C$ are homothetic, $A, A_{1}$, and $A_{2}$ are collinear. Points $B_{2}, C_{2}$ are analogously defined. Denote the angles $B A A_{2}, A_{2} A C, C B B_{2}$, $B_{2} B A, A C C_{2}, C_{2} C B$ by $\\alpha_{1}, \\alpha_{2}, \\beta_{1}, \\beta_{2}, \\gamma_{1}, \\gamma_{2}$. By the law of sines we have $$ \\frac{\\sin \\alpha_{1}}{\\sin \\alpha_{2}}=\\frac{\\sin \\left(\\beta+45^{\\circ}\\right)}{\\sin \\left(\\gamma+45^{\\circ}\\right)}, \\frac{\\sin \\beta_{1}}{\\sin \\beta_{2}}=\\frac{\\sin \\left(\\gamma+45^{\\circ}\\right)}{\\sin \\left(\\alpha+45^{\\circ}\\right)}, \\frac{\\sin \\gamma_{1}}{\\sin \\gamma_{2}}=\\frac{\\sin \\left(\\alpha+45^{\\circ}\\right)}{\\sin \\left(\\beta+45^{\\circ}\\right)} $$ Since the product of these ratios is 1 , by the trigonometric Ceva's theorem $A A_{2}, B B_{2}, C C_{2}$ are concurrent.","problem_type":"Geometry","tier":0} +{"year":"2001","problem_phase":"shortlisted","problem":"16. G2 (KOR) ${ }^{\\mathrm{IMO} 1}$ In acute triangle $A B C$ with circumcenter $O$ and altitude $A P, \\measuredangle C \\geq \\measuredangle B+30^{\\circ}$. Prove that $\\measuredangle A+\\measuredangle C O P<90^{\\circ}$.","solution":"16. Since $\\angle O C P=90^{\\circ}-\\angle A$, we are led to showing that $\\angle O C P>\\angle C O P$, i.e., $O P>C P$. By the triangle inequality it suffices to prove $C P<\\frac{1}{2} C O$. Let $C O=R$. The law of sines yields $C P=A C \\cos \\gamma=2 R \\sin \\beta \\cos \\gamma<$ $2 R \\sin \\beta \\cos \\left(\\beta+30^{\\circ}\\right)$. Finally, we have $$ 2 \\sin \\beta \\cos \\left(\\beta+30^{\\circ}\\right)=\\sin \\left(2 \\beta+30^{\\circ}\\right)-\\sin 30^{\\circ} \\leq \\frac{1}{2} $$ which completes the proof.","problem_type":"Geometry","tier":0} +{"year":"2001","problem_phase":"shortlisted","problem":"17. G3 (GBR) Let $A B C$ be a triangle with centroid $G$. Determine, with proof, the position of the point $P$ in the plane of $A B C$ such that $$ A P \\cdot A G+B P \\cdot B G+C P \\cdot C G $$ is a minimum, and express this minimum value in terms of the side lengths of $A B C$.","solution":"17. Let us investigate a more general problem, in which $G$ is any point of the plane such that $A G, B G, C G$ are sides of a triangle. Let $F$ be the point in the plane such that $B C: C F: F B=A G: B G: C G$ and $F, A$ lie on different sides of $B C$. Then by Ptolemy's inequality, on $B P C F$ we have $A G \\cdot A P+B G \\cdot B P+C G \\cdot C P=A G \\cdot A P+\\frac{A G}{B C}(C F$. $B P+B F \\cdot C P) \\geq A G \\cdot A P+\\frac{A G}{B C} B C \\cdot P F$. Hence $$ A G \\cdot A P+B G \\cdot B P+C G \\cdot C P \\geq A G \\cdot A F $$ where equality holds if and only if $P$ lies on the segment $A F$ and on the circle $B C F$. Now we return to the case of $G$ the centroid of $\\triangle A B C$. We claim that $F$ is then the point $\\widehat{G}$ in which the line $A G$ meets again the circumcircle of $\\triangle B G C$. Indeed, if $M$ is the midpoint of $A B$, by the law of sines we have $\\frac{B C}{C \\widehat{G}}=$ $\\frac{\\sin \\angle B \\widehat{G} C}{\\sin \\angle C B \\widehat{G}}=\\frac{\\sin \\angle B G M}{\\sin \\angle A G M}=\\frac{A G}{B G}$, and similarly $\\frac{B C}{B \\widehat{G}}=\\frac{A G}{C G}$. Thus (1) implies ![](https:\/\/cdn.mathpix.com\/cropped\/2024_11_18_8e985d6b9c83aa3e9d0eg-691.jpg?height=342&width=507&top_left_y=1514&top_left_x=831) $$ A G \\cdot A P+B G \\cdot B P+C G \\cdot C P \\geq A G \\cdot A \\widehat{G} $$ It is easily seen from the above considerations that equality holds if and only if $P \\equiv G$, and then the (minimum) value of $A G \\cdot A P+B G \\cdot B P+$ $C G \\cdot C P$ equals $$ A G^{2}+B G^{2}+C G^{2}=\\frac{a^{2}+b^{2}+c^{2}}{3} $$ Second solution. Notice that $A G \\cdot A P \\geq \\overrightarrow{A G} \\cdot \\overrightarrow{A P}=\\overrightarrow{A G} \\cdot(\\overrightarrow{A G}+\\overrightarrow{P G})$. Summing this inequality with analogous inequalities for $B G \\cdot B P$ and $C G \\cdot C P$ gives us $A G \\cdot A P+B G \\cdot B P+C G \\cdot C P \\geq A G^{2}+B G^{2}+C G^{2}+$ $(\\overrightarrow{A G}+\\overrightarrow{B G}+\\overrightarrow{C G}) \\cdot \\overrightarrow{P G}=A G^{2}+B G^{2}+C G^{2}=\\frac{a^{2}+b^{2}+c^{2}}{3}$. Equality holds if and only if $P \\equiv Q$.","problem_type":"Geometry","tier":0} +{"year":"2001","problem_phase":"shortlisted","problem":"18. G4 (FRA) Let $M$ be a point in the interior of triangle $A B C$. Let $A^{\\prime}$ lie on $B C$ with $M A^{\\prime}$ perpendicular to $B C$. Define $B^{\\prime}$ on $C A$ and $C^{\\prime}$ on $A B$ similarly. Define $$ p(M)=\\frac{M A^{\\prime} \\cdot M B^{\\prime} \\cdot M C^{\\prime}}{M A \\cdot M B \\cdot M C} $$ Determine, with proof, the location of $M$ such that $p(M)$ is maximal. Let $\\mu(A B C)$ denote the maximum value. For which triangles $A B C$ is the value of $\\mu(A B C)$ maximal?","solution":"18. Let $\\alpha_{1}, \\beta_{1}, \\gamma_{1}, \\alpha_{2}, \\beta_{2}, \\gamma_{2}$ denote the angles $\\angle M A B, \\angle M B C, \\angle M C A$, $\\angle M A C, \\angle M B A, \\angle M C B$ respectively. Then $\\frac{M B^{\\prime} \\cdot M C^{\\prime}}{M A^{2}}=\\sin \\alpha_{1} \\sin \\alpha_{2}$, $\\frac{M C^{\\prime} \\cdot M A^{\\prime}}{M B^{2}}=\\sin \\beta_{1} \\sin \\beta_{2}, \\frac{M A^{\\prime} \\cdot M B^{\\prime}}{M C^{2}}=\\sin \\gamma_{1} \\sin \\gamma_{2}$; hence $$ p(M)^{2}=\\sin \\alpha_{1} \\sin \\alpha_{2} \\sin \\beta_{1} \\sin \\beta_{2} \\sin \\gamma_{1} \\sin \\gamma_{2} $$ Since $$ \\sin \\alpha_{1} \\sin \\alpha_{2}=\\frac{1}{2}\\left(\\cos \\left(\\alpha_{1}-\\alpha_{2}\\right)-\\cos \\left(\\alpha_{1}+\\alpha_{2}\\right) \\leq \\frac{1}{2}(1-\\cos \\alpha)=\\sin ^{2} \\frac{\\alpha}{2}\\right. $$ we conclude that $$ p(M) \\leq \\sin \\frac{\\alpha}{2} \\sin \\frac{\\beta}{2} \\sin \\frac{\\gamma}{2} $$ Equality occurs when $\\alpha_{1}=\\alpha_{2}, \\beta_{1}=\\beta_{2}$, and $\\gamma_{1}=\\gamma_{2}$, that is, when $M$ is the incenter of $\\triangle A B C$. It is well known that $\\mu(A B C)=\\sin \\frac{\\alpha}{2} \\sin \\frac{\\beta}{2} \\sin \\frac{\\gamma}{2}$ is maximal when $\\triangle A B C$ is equilateral (it follows, for example, from Jensen's inequality applied to $\\ln \\sin x)$. Hence $\\max \\mu(A B C)=\\frac{1}{8}$.","problem_type":"Geometry","tier":0} +{"year":"2001","problem_phase":"shortlisted","problem":"19. G5 (GRE) Let $A B C$ be an acute triangle. Let $D A C, E A B$, and $F B C$ be isosceles triangles exterior to $A B C$, with $D A=D C, E A=E B$, and $F B=F C$ such that $$ \\angle A D C=2 \\angle B A C, \\quad \\angle B E A=2 \\angle A B C, \\quad \\angle C F B=2 \\angle A C B . $$ Let $D^{\\prime}$ be the intersection of lines $D B$ and $E F$, let $E^{\\prime}$ be the intersection of $E C$ and $D F$, and let $F^{\\prime}$ be the intersection of $F A$ and $D E$. Find, with proof, the value of the sum $$ \\frac{D B}{D D^{\\prime}}+\\frac{E C}{E E^{\\prime}}+\\frac{F A}{F F^{\\prime}} $$","solution":"19. It is easy to see that the hexagon $A E B F C D$ is convex and $\\angle A E B+$ $\\angle B F C+\\angle C D A=360^{\\circ}$. Using this relation we obtain that the circles $\\omega_{1}, \\omega_{2}, \\omega_{3}$ with centers at $D, E, F$ and radii $D A, E B, F C$ respectively all pass through a common point $O$. Indeed, if $\\omega_{1} \\cap \\omega_{2}=\\{O\\}$, then $\\angle A O B=180^{\\circ}-\\angle A E B \/ 2$ and $\\angle B O C=180^{\\circ}-\\angle B F C \/ 2$; hence $\\angle C O A=180^{\\circ}-\\angle C D A \/ 2$ as well, i.e., $O \\in \\omega_{3}$. The point $O$ is the re- ![](https:\/\/cdn.mathpix.com\/cropped\/2024_11_18_8e985d6b9c83aa3e9d0eg-692.jpg?height=491&width=419&top_left_y=1318&top_left_x=870) flection of $A$ with respect to $D E$. Similarly, it is also the reflection of $B$ with respect to $E F$, and that of $C$ with respect to $F D$. Hence $$ \\frac{D B}{D D^{\\prime}}=1+\\frac{D^{\\prime} B}{D D^{\\prime}}=1+\\frac{S_{E B F}}{S_{E D F}}=1+\\frac{S_{O E F}}{S_{D E F}} $$ Analogously $\\frac{E C}{E E^{\\prime}}=1+\\frac{S_{O D F}}{S_{D E F}}$ and $\\frac{F A}{F F^{\\prime}}=1+\\frac{S_{O D E}}{S_{D E F}}$. Adding these relations gives us $$ \\frac{D B}{D D^{\\prime}}+\\frac{E C}{E E^{\\prime}}+\\frac{F A}{F F^{\\prime}}=3+\\frac{S_{O E F}+S_{O D F}+S_{O D E}}{S_{D E F}}=4 . $$","problem_type":"Geometry","tier":0} +{"year":"2001","problem_phase":"shortlisted","problem":"2. A2 (POL) Let $a_{0}, a_{1}, a_{2}, \\ldots$ be an arbitrary infinite sequence of positive numbers. Show that the inequality $1+a_{n}>a_{n-1} \\sqrt[n]{2}$ holds for infinitely many positive integers $n$.","solution":"2. It follows from Bernoulli's inequality that for each $n \\in \\mathbb{N},\\left(1+\\frac{1}{n}\\right)^{n} \\geq 2$, or $\\sqrt[n]{2} \\leq 1+\\frac{1}{n}$. Consequently, it will be enough to show that $1+a_{n}>$ $\\left(1+\\frac{1}{n}\\right) a_{n-1}$. Assume the opposite. Then there exists $N$ such that for each $n \\geq N$, $$ 1+a_{n} \\leq\\left(1+\\frac{1}{n}\\right) a_{n-1}, \\quad \\text { i.e., } \\quad \\frac{1}{n+1}+\\frac{a_{n}}{n+1} \\leq \\frac{a_{n-1}}{n} $$ Summing for $n=N, \\ldots, m$ yields $\\frac{a_{m}}{m+1} \\leq \\frac{a_{N-1}}{N}-\\left(\\frac{1}{N+1}+\\cdots+\\frac{1}{m+1}\\right)$. However, it is well known that the sum $\\frac{1}{N+1}+\\cdots+\\frac{1}{m+1}$ can be arbitrarily large for $m$ large enough, so that $\\frac{a_{m}^{N+1}}{m+1}$ is eventually negative. This contradiction yields the result. Second solution. Suppose that $1+a_{n} \\leq \\sqrt[n]{2} a_{n-1}$ for all $n \\geq N$. Set $b_{n}=2^{-(1+1 \/ 2+\\cdots+1 \/ n)}$ and multiply both sides of the above inequality to obtain $b_{n}+b_{n} a_{n} \\leq b_{n-1} a_{n-1}$. Thus $$ b_{N} a_{N}>b_{N} a_{N}-b_{n} a_{n} \\geq b_{N}+b_{N+1}+\\cdots+b_{n} $$ However, it can be shown that $\\sum_{n>N} b_{N}$ diverges: in fact, since $1+\\frac{1}{2}+$ $\\cdots+\\frac{1}{n}<1+\\ln n$, we have $b_{n}>2^{-1-\\ln n}=\\frac{1}{2} n^{-\\ln 2}>\\frac{1}{2 n}$, and we already know that $\\sum_{n>N} \\frac{1}{2 n}$ diverges. Remark. As can be seen from both solutions, the value 2 in the problem can be increased to $e$.","problem_type":"Algebra","tier":0} +{"year":"2001","problem_phase":"shortlisted","problem":"20. G6 (IND) Let $A B C$ be a triangle and $P$ an exterior point in the plane of the triangle. Suppose $A P, B P, C P$ meet the sides $B C, C A, A B$ (or extensions thereof) in $D, E, F$, respectively. Suppose further that the areas of triangles $P B D, P C E, P A F$ are all equal. Prove that each of these areas is equal to the area of triangle $A B C$ itself.","solution":"20. By Ceva's theorem, we can choose real numbers $x, y, z$ such that $$ \\frac{\\overrightarrow{B D}}{\\overrightarrow{D C}}=\\frac{z}{y}, \\frac{\\overrightarrow{C E}}{\\overrightarrow{E A}}=\\frac{x}{z}, \\text { and } \\frac{\\overrightarrow{A F}}{\\overrightarrow{F B}}=\\frac{y}{x} $$ The point $P$ lies outside the triangle $A B C$ if and only if $x, y, z$ are not all of the same sign. In what follows, $S_{X}$ will denote the signed area of a figure $X$. Let us assume that the area $S_{A B C}$ of $\\triangle A B C$ is 1 . Since $S_{P B C}: S_{P C A}$ : $S_{P A B}=x: y: z$ and $S_{P B D}: S_{P D C}=z: y$, it follows that $S_{P B D}=\\frac{z}{y+z} \\frac{x}{x+y+z}$. Hence $S_{P B D}=\\frac{1}{y(y+z)} \\frac{x y z}{x+y+z}, S_{P C E}=\\frac{1}{z(z+x)} \\frac{x y z}{x+y+z}$, $S_{P A F}=\\frac{1}{x(x+y)} \\frac{x y z}{x+y+z}$. By the condition of the problem we have $\\left|S_{P B D}\\right|=$ $\\left|S_{P C E}\\right|=\\left|S_{P A F}\\right|$, or $$ |x(x+y)|=|y(y+z)|=|z(z+x)| . $$ Obviously $x, y, z$ are nonzero, so that we can put w.l.o.g. $z=1$. At least two of the numbers $x(x+y), y(y+1), 1(1+x)$ are equal, so we can assume that $x(x+y)=y(y+1)$. We distinguish two cases: (i) $x(x+y)=y(y+1)=1+x$. Then $x=y^{2}+y-1$, from which we obtain $\\left(y^{2}+y-1\\right)\\left(y^{2}+2 y-1\\right)=y(y+1)$. Simplification gives $y^{4}+3 y^{3}-y^{2}-4 y+1=0$, or $$ (y-1)\\left(y^{3}+4 y^{2}+3 y-1\\right)=0 $$ If $y=1$, then also $z=x=1$, so $P$ is the centroid of $\\triangle A B C$, which is not an exterior point. Hence $y^{3}+4 y^{2}+3 y-1=0$. Now the signed area of each of the triangles $P B D, P C E, P A F$ equals $$ \\begin{aligned} S_{P A F} & =\\frac{y z}{(x+y)(x+y+z)} \\\\ & =\\frac{y}{\\left(y^{2}+2 y-1\\right)\\left(y^{2}+2 y\\right)}=\\frac{1}{y^{3}+4 y^{2}+3 y-2}=-1 . \\end{aligned} $$ It is easy to check that not both of $x, y$ are positive, implying that $P$ is indeed outside $\\triangle A B C$. This is the desired result. (ii) $x(x+y)=y(y+1)=-1-x$. In this case we are led to $$ f(y)=y^{4}+3 y^{3}+y^{2}-2 y+1=0 . $$ We claim that this equation has no real solutions. In fact, assume that $y_{0}$ is a real root of $f(y)$. We must have $y_{0}<0$, and hence $u=-y_{0}>0$ satisfies $3 u^{3}-u^{4}=(u+1)^{2}$. On the other hand, $$ \\begin{aligned} 3 u^{3}-u^{4} & =u^{3}(3-u)=4 u\\left(\\frac{u}{2}\\right)\\left(\\frac{u}{2}\\right)(3-u) \\\\ & \\leq 4 u\\left(\\frac{u \/ 2+u \/ 2+3-u}{3}\\right)^{3}=4 u \\\\ & \\leq(u+1)^{2} \\end{aligned} $$ where at least one of the inequalities is strict, a contradiction. Remark. The official solution was incomplete, missing the case (ii).","problem_type":"Geometry","tier":0} +{"year":"2001","problem_phase":"shortlisted","problem":"21. G7 (BUL) Let $O$ be an interior point of acute triangle $A B C$. Let $A_{1}$ lie on $B C$ with $O A_{1}$ perpendicular to $B C$. Define $B_{1}$ on $C A$ and $C_{1}$ on $A B$ similarly. Prove that $O$ is the circumcenter of $A B C$ if and only if the perimeter of $A_{1} B_{1} C_{1}$ is not less than any one of the perimeters of $A B_{1} C_{1}, B C_{1} A_{1}$, and $C A_{1} B_{1}$.","solution":"21. We denote by $p(X Y Z)$ the perimeter of a triangle $X Y Z$. If $O$ is the circumcenter of $\\triangle A B C$, then $A_{1}, B_{1}, C_{1}$ are the midpoints of the corresponding sides of the triangle, and hence $p\\left(A_{1} B_{1} C_{1}\\right)=$ $p\\left(A B_{1} C_{1}\\right)=p\\left(A_{1} B C_{1}\\right)=p\\left(A_{1} B_{1} C\\right)$. Conversely, suppose that $p\\left(A_{1} B_{1} C_{1}\\right) \\geq p\\left(A B_{1} C_{1}\\right), p\\left(A_{1} B C_{1}\\right), p\\left(A_{1} B_{1} C\\right)$. Let $\\alpha_{1}, \\alpha_{2}, \\beta_{1}, \\beta_{2}, \\gamma_{1}, \\gamma_{2}$ denote $\\angle B_{1} A_{1} C, \\angle C_{1} A_{1} B, \\angle C_{1} B_{1} A, \\angle A_{1} B_{1} C$, $\\angle A_{1} C_{1} B, \\angle B_{1} C_{1} A$. Suppose that $\\gamma_{1}, \\beta_{2} \\geq \\alpha$. If $A_{2}$ is the fourth vertex of the parallelogram $B_{1} A C_{1} A_{2}$, then these conditions imply that $A_{1}$ is in the interior or on the border of $\\triangle B_{1} C_{1} A_{2}$, and therefore $p\\left(A_{1} B_{1} C_{1}\\right) \\leq p\\left(A_{2} B_{1} C_{1}\\right)=$ $p\\left(A B_{1} C_{1}\\right)$. Moreover, if one of the inequalities $\\gamma_{1} \\geq \\alpha, \\beta_{2} \\geq \\alpha$ is strict, ![](https:\/\/cdn.mathpix.com\/cropped\/2024_11_18_8e985d6b9c83aa3e9d0eg-694.jpg?height=346&width=543&top_left_y=902&top_left_x=804) then $p\\left(A_{1} B_{1} C_{1}\\right)$ is strictly less than $p\\left(A B_{1} C_{1}\\right)$, contrary to the assumption. Hence $$ \\begin{aligned} & \\beta_{2} \\geq \\alpha \\Longrightarrow \\gamma_{1} \\leq \\alpha \\\\ & \\gamma_{2} \\geq \\beta \\Longrightarrow \\alpha_{1} \\leq \\beta \\\\ & \\alpha_{2} \\geq \\gamma \\Longrightarrow \\beta_{1} \\leq \\gamma \\end{aligned} $$ the last two inequalities being obtained analogously to the first one. Because of the symmetry, there is no loss of generality in assuming that $\\gamma_{1} \\leq \\alpha$. Then since $\\gamma_{1}+\\alpha_{2}=180^{\\circ}-\\beta=\\alpha+\\gamma$, it follows that $\\alpha_{2} \\geq \\gamma$. From (1) we deduce $\\beta_{1} \\leq \\gamma$, which further implies $\\gamma_{2} \\geq \\beta$. Similarly, this leads to $\\alpha_{1} \\leq \\beta$ and $\\beta_{2} \\geq \\alpha$. To sum up, $$ \\gamma_{1} \\leq \\alpha \\leq \\beta_{2}, \\quad \\alpha_{1} \\leq \\beta \\leq \\gamma_{2}, \\quad \\beta_{1} \\leq \\gamma \\leq \\alpha_{2} $$ Since $O A_{1} B C_{1}$ and $O B_{1} C A_{1}$ are cyclic, we have $\\angle A_{1} O B=\\gamma_{1}$ and $\\angle A_{1} O C=\\beta_{2}$. Hence $B O: C O=\\cos \\beta_{2}: \\cos \\gamma_{1}$, hence $B O \\leq C O$. Analogously, $C O \\leq A O$ and $A O \\leq B O$. Therefore $A O=B O=C O$, i.e., $O$ is the circumcenter of $A B C$.","problem_type":"Geometry","tier":0} +{"year":"2001","problem_phase":"shortlisted","problem":"22. G8 (ISR) ${ }^{\\mathrm{IMO}}$ Let $A B C$ be a triangle with $\\measuredangle B A C=60^{\\circ}$. Let $A P$ bisect $\\angle B A C$ and let $B Q$ bisect $\\angle A B C$, with $P$ on $B C$ and $Q$ on $A C$. If $A B+$ $B P=A Q+Q B$, what are the angles of the triangle?","solution":"22. Let $S$ and $T$ respectively be the points on the extensions of $A B$ and $A Q$ over $B$ and $Q$ such that $B S=B P$ and $Q T=Q B$. It is given that $A S=$ $A B+B P=A Q+Q B=A T$. Since $\\angle P A S=\\angle P A T$, the triangles $A P S$ and $A P T$ are congruent, from which we deduce that $\\angle A T P=\\angle A S P=$ $\\beta \/ 2=\\angle Q B P$. Hence $\\angle Q T P=\\angle Q B P$. If $P$ does not lie on $B T$, then the last equality implies that $\\triangle Q B P$ and $\\triangle Q T P$ are congruent, so $P$ lies on the internal bisector of $\\angle B Q T$. But $P$ also lies on the internal bisector of $\\angle Q A B$; consequently, $P$ is an excenter of $\\triangle Q A B$, thus lying on the internal bisector of $\\angle Q B S$ as well. It follows that $\\angle P B Q=\\beta \/ 2=\\angle P B S=180^{\\circ}-\\beta$, so $\\beta=120^{\\circ}$, which is impossible. Therefore $P \\in B T$, which means that $T \\equiv C$. Now from $Q C=Q B$ we conclude that $120^{\\circ}-\\beta=\\gamma=\\beta \/ 2$, i.e., $\\beta=80^{\\circ}$ and $\\gamma=40^{\\circ}$.","problem_type":"Geometry","tier":0} +{"year":"2001","problem_phase":"shortlisted","problem":"23. N1 (AUS) Prove that there is no positive integer $n$ such that for $k=$ $1,2, \\ldots, 9$, the leftmost digit (in decimal notation) of $(n+k)$ ! equals $k$.","solution":"23. For each positive integer $x$, define $\\alpha(x)=x \/ 10^{r}$ if $r$ is the positive integer satisfying $10^{r} \\leq x<10^{r+1}$. Observe that if $\\alpha(x) \\alpha(y)<10$ for some $x, y \\in \\mathbb{N}$, then $\\alpha(x y)=\\alpha(x) \\alpha(y)$. If, as usual, $[t]$ means the integer part of $t$, then $[\\alpha(x)]$ is actually the leftmost digit of $x$. Now suppose that $n$ is a positive integer such that $k \\leq \\alpha((n+k)!)\\alpha(n+k)$ (the opposite can hold only if $\\alpha(n+k) \\geq 9)$. Therefore $$ 1<\\alpha(n+2)<\\cdots<\\alpha(n+9) \\leq \\frac{5}{4} . $$ On the other hand, this implies that $\\alpha((n+4)!)=\\alpha((n+1)!) \\alpha(n+2) \\alpha(n+$ 3) $\\alpha(n+4)<(5 \/ 4)^{3} \\alpha((n+1)$ ! $)<4$, contradicting the assumption that the leftmost digit of $(n+4)$ ! is 4 .","problem_type":"Number Theory","tier":0} +{"year":"2001","problem_phase":"shortlisted","problem":"24. N2 (COL) Consider the system $$ \\begin{aligned} x+y & =z+u \\\\ 2 x y & =z u . \\end{aligned} $$ Find the greatest value of the real constant $m$ such that $m \\leq x \/ y$ for every positive integer solution $x, y, z, u$ of the system with $x \\geq y$.","solution":"24. We shall find the general solution to the system. Squaring both sides of the first equation and subtracting twice the second equation we obtain $(x-y)^{2}=z^{2}+u^{2}$. Thus $(z, u, x-y)$ is a Pythagorean triple. Then it is well known that there are positive integers $t, a, b$ such that $z=t\\left(a^{2}-b^{2}\\right)$, $u=2 t a b$ (or vice versa), and $x-y=t\\left(a^{2}+b^{2}\\right)$. Using that $x+y=z+u$ we come to the general solution: $$ x=t\\left(a^{2}+a b\\right), \\quad y=t\\left(a b-b^{2}\\right) ; \\quad z=t\\left(a^{2}-b^{2}\\right), \\quad u=2 t a b . $$ Putting $a \/ b=k$ we obtain $$ \\frac{x}{y}=\\frac{k^{2}+k}{k-1}=3+(k-1)+\\frac{2}{k-1} \\geq 3+2 \\sqrt{2} $$ with equality for $k-1=\\sqrt{2}$. On the other hand, $k$ can be arbitrarily close to $1+\\sqrt{2}$, and so $x \/ y$ can be arbitrarily close to $3+2 \\sqrt{2}$. Hence $m=3+2 \\sqrt{2}$. Remark. There are several other techniques for solving the given system. The exact lower bound of $m$ itself can be obtained as follows: by the $\\operatorname{system}\\left(\\frac{x}{y}\\right)^{2}-6 \\frac{x}{y}+1=\\left(\\frac{z-u}{y}\\right)^{2} \\geq 0$, so $x \/ y \\geq 3+2 \\sqrt{2}$.","problem_type":"Number Theory","tier":0} +{"year":"2001","problem_phase":"shortlisted","problem":"25. N3 (GBR) Let $a_{1}=11^{11}, a_{2}=12^{12}, a_{3}=13^{13}$, and $$ a_{n}=\\left|a_{n-1}-a_{n-2}\\right|+\\left|a_{n-2}-a_{n-3}\\right|, \\quad n \\geq 4 $$ Determine $a_{14^{14}}$.","solution":"25. Define $b_{n}=\\left|a_{n+1}-a_{n}\\right|$ for $n \\geq 1$. From the equalities $a_{n+1}=b_{n-1}+b_{n-2}$, from $a_{n}=b_{n-2}+b_{n-3}$ we obtain $b_{n}=\\left|b_{n-1}-b_{n-3}\\right|$. From this relation we deduce that $b_{m} \\leq \\max \\left(b_{n}, b_{n+1}, b_{n+2}\\right)$ for all $m \\geq n$, and consequently $b_{n}$ is bounded. Lemma. If $\\max \\left(b_{n}, b_{n+1}, b_{n+2}\\right)=M \\geq 2$, then $\\max \\left(b_{n+6}, b_{n+7}, b_{n+8}\\right) \\leq$ $M-1$. Proof. Assume the opposite. Suppose that $b_{j}=M, j \\in\\{n, n+1, n+2\\}$, and let $b_{j+1}=x$ and $b_{j+2}=y$. Thus $b_{j+3}=M-y$. If $x, y, M-y$ are all less than $M$, then the contradiction is immediate. The remaining cases are these: (i) $x=M$. Then the sequence has the form $M, M, y, M-y, y, \\ldots$, and since $\\max (y, M-y, y)=M$, we must have $y=0$ or $y=M$. (ii) $y=M$. Then the sequence has the form $M, x, M, 0, x, M-x, \\ldots$, and since $\\max (0, x, M-x)=M$, we must have $x=0$ or $x=M$. (iii) $y=0$. Then the sequence is $M, x, 0, M, M-x, M-x, x, \\ldots$, and since $\\max (M-x, x, x)=M$, we have $x=0$ or $x=M$. In every case $M$ divides both $x$ and $y$. From the recurrence formula $M$ also divides $b_{i}$ for every $ib>c>d$ be positive integers and suppose $$ a c+b d=(b+d+a-c)(b+d-a+c) . $$ Prove that $a b+c d$ is not prime.","solution":"27. The given equality is equivalent to $a^{2}-a c+c^{2}=b^{2}+b d+d^{2}$. Hence $(a b+c d)(a d+b c)=a c\\left(b^{2}+b d+d^{2}\\right)+b d\\left(a^{2}-a c+c^{2}\\right)$, or equivalently, $$ (a b+c d)(a d+b c)=(a c+b d)\\left(a^{2}-a c+c^{2}\\right) $$ Now suppose that $a b+c d$ is prime. It follows from $a>b>c>d$ that $$ a b+c d>a c+b d>a d+b c $$ hence $a c+b d$ is relatively prime with $a b+c d$. But then (1) implies that $a c+b d$ divides $a d+b c$, which is impossible by (2). Remark. Alternatively, (1) could be obtained by applying the law of cosines and Ptolemy's theorem on a quadrilateral $X Y Z T$ with $X Y=a$, $Y Z=c, Z T=b, T X=d$ and $\\angle Y=60^{\\circ}, \\angle T=120^{\\circ}$.","problem_type":"Number Theory","tier":0} +{"year":"2001","problem_phase":"shortlisted","problem":"28. N6 (RUS) Is it possible to find 100 positive integers not exceeding 25,000 such that all pairwise sums of them are different?","solution":"28. Yes. The desired result is an immediate consequence of the following fact applied on $p=101$. Lemma. For any odd prime number $p$, there exist $p$ nonnegative integers less than $2 p^{2}$ with all pairwise sums mutually distinct. Proof. We claim that the numbers $a_{n}=2 n p+\\left(n^{2}\\right)$ have the desired property, where $(x)$ denotes the remainder of $x$ upon division by $p$. Suppose that $a_{k}+a_{l}=a_{m}+a_{n}$. By the construction of $a_{i}$, we have $2 p(k+l) \\leq a_{k}+a_{l}<2 p(k+l+1)$. Hence we must have $k+l=m+n$, and therefore also $\\left(k^{2}\\right)+\\left(l^{2}\\right)=\\left(m^{2}\\right)+\\left(n^{2}\\right)$. Thus $$ k+l \\equiv m+n \\quad \\text { and } \\quad k^{2}+l^{2} \\equiv m^{2}+n^{2} \\quad(\\bmod p) . $$ But then it holds that $(k-l)^{2}=2\\left(k^{2}+l^{2}\\right)-(k+l)^{2} \\equiv(m-n)^{2}(\\bmod$ $p)$, so $k-l \\equiv \\pm(m-n)$, which leads to $(k, l)=(m, n)$. This proves the lemma.","problem_type":"Number Theory","tier":0} +{"year":"2001","problem_phase":"shortlisted","problem":"3. A3 (ROM) Let $x_{1}, x_{2}, \\ldots, x_{n}$ be arbitrary real numbers. Prove the inequality $$ \\frac{x_{1}}{1+x_{1}^{2}}+\\frac{x_{2}}{1+x_{1}^{2}+x_{2}^{2}}+\\cdots+\\frac{x_{n}}{1+x_{1}^{2}+\\cdots+x_{n}^{2}}<\\sqrt{n} $$","solution":"3. By the arithmetic-quadratic mean inequality, it suffices to prove that $$ \\frac{x_{1}^{2}}{\\left(1+x_{1}^{2}\\right)^{2}}+\\frac{x_{2}^{2}}{\\left(1+x_{1}^{2}+x_{2}^{2}\\right)^{2}}+\\cdots+\\frac{x_{n}^{2}}{\\left(1+x_{1}^{2}+\\cdots+x_{n}^{2}\\right)^{2}}<1 . $$ Observe that for $k \\geq 2$ the following holds: $$ \\begin{aligned} \\frac{x_{k}^{2}}{\\left(1+x_{1}^{2}+\\cdots+x_{k}^{2}\\right)^{2}} & \\leq \\frac{x_{k}^{2}}{\\left(1+\\cdots+x_{k-1}^{2}\\right)\\left(1+\\cdots+x_{k}^{2}\\right)} \\\\ & =\\frac{1}{1+x_{1}^{2}+\\cdots+x_{k-1}^{2}}-\\frac{1}{1+x_{1}^{2}+\\cdots+x_{k}^{2}} \\end{aligned} $$ For $k=1$ we have $\\frac{x_{1}^{2}}{\\left(1+x_{1}\\right)^{2}} \\leq 1-\\frac{1}{1+x_{1}^{2}}$. Summing these inequalities, we obtain $$ \\frac{x_{1}^{2}}{\\left(1+x_{1}^{2}\\right)^{2}}+\\cdots+\\frac{x_{n}^{2}}{\\left(1+x_{1}^{2}+\\cdots+x_{n}^{2}\\right)^{2}} \\leq 1-\\frac{1}{1+x_{1}^{2}+\\cdots+x_{n}^{2}}<1 $$ Second solution. Let $a_{n}(k)=\\sup \\left(\\frac{x_{1}}{k^{2}+x_{1}^{2}}+\\cdots+\\frac{x_{n}}{k^{2}+x_{1}^{2}+\\cdots+x_{n}^{2}}\\right)$ and $a_{n}=$ $a_{n}(1)$. We must show that $a_{n}<\\sqrt{n}$. Replacing $x_{i}$ by $k x_{i}$ shows that $a_{n}(k)=a_{n} \/ k$. Hence $$ a_{n}=\\sup _{x_{1}}\\left(\\frac{x_{1}}{1+x_{1}^{2}}+\\frac{a_{n-1}}{\\sqrt{1+x_{1}^{2}}}\\right)=\\sup _{\\theta}\\left(\\sin \\theta \\cos \\theta+a_{n-1} \\cos \\theta\\right), $$ where $\\tan \\theta=x_{1}$. The above supremum can be computed explicitly: $$ a_{n}=\\frac{1}{8 \\sqrt{2}}\\left(3 a_{n-1}+\\sqrt{a_{n-1}^{2}+8}\\right) \\sqrt{4-a_{n-1}^{2}+a_{n-1} \\sqrt{a_{n-1}^{2}+8}} . $$ However, the required inequality is weaker and can be proved more easily: if $a_{n-1}<\\sqrt{n-1}$, then by (1) $a_{n}<\\sin \\theta+\\sqrt{n-1} \\cos \\theta=\\sqrt{n} \\sin (\\theta+\\alpha) \\leq$ $\\sqrt{n}$, for $\\alpha \\in(0, \\pi \/ 2)$ with $\\tan \\alpha=\\sqrt{n}$.","problem_type":"Algebra","tier":0} +{"year":"2001","problem_phase":"shortlisted","problem":"4. A4 (LIT) Find all functions $f: \\mathbb{R} \\rightarrow \\mathbb{R}$ satisfying $$ f(x y)(f(x)-f(y))=(x-y) f(x) f(y) $$ for all $x, y$.","solution":"4. Let $(*)$ denote the given functional equation. Substituting $y=1$ we get $f(x)^{2}=x f(x) f(1)$. If $f(1)=0$, then $f(x)=0$ for all $x$, which is the trivial solution. Suppose $f(1)=C \\neq 0$. Let $G=\\{y \\in \\mathbb{R} \\mid f(y) \\neq 0\\}$. Then $$ f(x)=\\left\\{\\begin{array}{cl} C x & \\text { if } x \\in G \\\\ 0 & \\text { otherwise } \\end{array}\\right. $$ We must determine the structure of $G$ so that the function defined by (1) satisfies (*). (1) Clearly $1 \\in G$, because $f(1) \\neq 0$. (2) If $x \\in G, y \\notin G$, then by $(*)$ it holds $f(x y) f(x)=0$, so $x y \\notin G$. (3) If $x, y \\in G$, then $x \/ y \\in G$ (otherwise by $2^{\\circ}, y(x \/ y)=x \\notin G$ ). (4) If $x, y \\in G$, then by $2^{\\circ}$ we have $x^{-1} \\in G$, so $x y=y \/ x^{-1} \\in G$. Hence $G$ is a set that contains 1 , does not contain 0 , and is closed under multiplication and division. Conversely, it is easy to verify that every such $G$ in (1) gives a function satisfying (*).","problem_type":"Algebra","tier":0} +{"year":"2001","problem_phase":"shortlisted","problem":"5. A5 (BUL) Find all positive integers $a_{1}, a_{2}, \\ldots, a_{n}$ such that $$ \\frac{99}{100}=\\frac{a_{0}}{a_{1}}+\\frac{a_{1}}{a_{2}}+\\cdots+\\frac{a_{n-1}}{a_{n}} $$ where $a_{0}=1$ and $\\left(a_{k+1}-1\\right) a_{k-1} \\geq a_{k}^{2}\\left(a_{k}-1\\right)$ for $k=1,2, \\ldots, n-1$.","solution":"5. Let $a_{1}, a_{2}, \\ldots, a_{n}$ satisfy the conditions of the problem. Then $a_{k}>a_{k-1}$, and hence $a_{k} \\geq 2$ for $k=1, \\ldots, n$. The inequality $\\left(a_{k+1}-1\\right) a_{k-1} \\geq$ $a_{k}^{2}\\left(a_{k}-1\\right)$ can be rewritten as $$ \\frac{a_{k-1}}{a_{k}}+\\frac{a_{k}}{a_{k+1}-1} \\leq \\frac{a_{k-1}}{a_{k}-1} . $$ Summing these inequalities for $k=i+1, \\ldots, n-1$ and using the obvious inequality $\\frac{a_{n-1}}{a_{n}}<\\frac{a_{n-1}}{a_{n}-1}$, we obtain $\\frac{a_{i}}{a_{i+1}}+\\cdots+\\frac{a_{n-1}}{a_{n}}<\\frac{a_{i}}{a_{i+1}-1}$. Therefore $$ \\frac{a_{i}}{a_{i+1}} \\leq \\frac{99}{100}-\\frac{a_{0}}{a_{1}}-\\cdots-\\frac{a_{i-1}}{a_{i}}<\\frac{a_{i}}{a_{i+1}-1} \\quad \\text { for } i=1,2, \\ldots, n-1 $$ Consequently, given $a_{0}, a_{1}, \\ldots, a_{i}$, there is at most one possibility for $a_{i+1}$. In our case, (1) yields $a_{1}=2, a_{2}=5, a_{3}=56, a_{4}=280^{2}=78400$. These values satisfy the conditions of the problem, so that this is a unique solution.","problem_type":"Algebra","tier":0} +{"year":"2001","problem_phase":"shortlisted","problem":"6. A6 (KOR) ${ }^{\\mathrm{IMO} 2}$ Prove that for all positive real numbers $a, b, c$, $$ \\frac{a}{\\sqrt{a^{2}+8 b c}}+\\frac{a}{\\sqrt{b^{2}+8 c a}}+\\frac{c}{\\sqrt{c^{2}+8 a b}} \\geq 1 $$","solution":"6. We shall determine a constant $k>0$ such that $$ \\frac{a}{\\sqrt{a^{2}+8 b c}} \\geq \\frac{a^{k}}{a^{k}+b^{k}+c^{k}} \\quad \\text { for all } a, b, c>0 $$ This inequality is equivalent to $\\left(a^{k}+b^{k}+c^{k}\\right)^{2} \\geq a^{2 k-2}\\left(a^{2}+8 b c\\right)$, which further reduces to $$ \\left(a^{k}+b^{k}+c^{k}\\right)^{2}-a^{2 k} \\geq 8 a^{2 k-2} b c $$ On the other hand, the AM-GM inequality yields $$ \\left(a^{k}+b^{k}+c^{k}\\right)^{2}-a^{2 k}=\\left(b^{k}+c^{k}\\right)\\left(2 a^{k}+b^{k}+c^{k}\\right) \\geq 8 a^{k \/ 2} b^{3 k \/ 4} c^{3 k \/ 4} $$ and therefore $k=4 \/ 3$ is a good choice. Now we have $$ \\begin{aligned} & \\frac{a}{\\sqrt{a^{2}+8 b c}}+\\frac{b}{\\sqrt{b^{2}+8 c a}}+\\frac{c}{\\sqrt{c^{2}+8 a b}} \\\\ & \\geq \\frac{a^{4 \/ 3}}{a^{4 \/ 3}+b^{4 \/ 3}+c^{4 \/ 3}}+\\frac{b^{4 \/ 3}}{a^{4 \/ 3}+b^{4 \/ 3}+c^{4 \/ 3}}+\\frac{c^{4 \/ 3}}{a^{4 \/ 3}+b^{4 \/ 3}+c^{4 \/ 3}}=1 \\end{aligned} $$ Second solution. The numbers $x=\\frac{a}{\\sqrt{a^{2}+8 b c}}, y=\\frac{b}{\\sqrt{b^{2}+8 c a}}$ and $z=\\frac{c}{\\sqrt{c^{2}+8 a b}}$ satisfy $$ f(x, y, z)=\\left(\\frac{1}{x^{2}}-1\\right)\\left(\\frac{1}{y^{2}}-1\\right)\\left(\\frac{1}{z^{2}}-1\\right)=8^{3} . $$ Our task is to prove $x+y+z \\geq 1$. Since $f$ is decreasing on each of the variables $x, y, z$, this is the same as proving that $x, y, z>0, x+y+z=1$ implies $f(x, y, z) \\geq 8^{3}$. However, since $\\frac{1}{x^{2}}-1=\\frac{(x+y+z)^{2}-x^{2}}{x^{2}}=\\frac{(2 x+y+z)(y+z)}{x^{2}}$, the inequality $f(x, y, z) \\geq 8^{3}$ becomes $$ \\frac{(2 x+y+z)(x+2 y+z)(x+y+2 z)(y+z)(z+x)(x+y)}{x^{2} y^{2} z^{2}} \\geq 8^{3} $$ which follows immediately by the AM-GM inequality. Third solution. We shall prove a more general fact: the inequality $\\frac{a}{\\sqrt{a^{2}+k b c}}+\\frac{b}{\\sqrt{b^{2}+k c a}}+\\frac{c}{\\sqrt{c^{2}+k a b}} \\geq \\frac{3}{\\sqrt{1+k}}$ is true for all $a, b, c>0$ if and only if $k \\geq 8$. Firstly suppose that $k \\geq 8$. Setting $x=b c \/ a^{2}, y=c a \/ b^{2}, z=a b \/ c^{2}$, we reduce the desired inequality to $$ F(x, y, z)=f(x)+f(y)+f(z) \\geq \\frac{3}{\\sqrt{1+k}}, \\quad \\text { where } f(t)=\\frac{1}{\\sqrt{1+k t}} $$ for $x, y, z>0$ such that $x y z=1$. We shall prove (2) using the method of Lagrange multipliers. The boundary of the set $D=\\left\\{(x, y, z) \\in \\mathbb{R}_{+}^{3} \\mid x y z=1\\right\\}$ consists of points $(x, y, z)$ with one of $x, y, z$ being 0 and another one being $+\\infty$. If w.l.o.g. $x=0$, then $F(x, y, z) \\geq f(x)=1 \\geq 3 \/ \\sqrt{1+k}$. Suppose now that $(x, y, z)$ is a point of local minimum of $F$ on $D$. There exists $\\lambda \\in \\mathbb{R}$ such that $(x, y, z)$ is stationary point of the function $F(x, y, z)+\\lambda x y z$. Then $(x, y, z, \\lambda)$ is a solution to the system $f^{\\prime}(x)+\\lambda y z=$ $f^{\\prime}(y)+\\lambda x z=f^{\\prime}(z)+\\lambda x y=0, x y z=1$. Eliminating $\\lambda$ gives us $$ x f^{\\prime}(x)=y f^{\\prime}(y)=z f^{\\prime}(z), \\quad x y z=1 $$ The function $t f^{\\prime}(t)=\\frac{-k t}{2(1+k t)^{3 \/ 2}}$ decreases on the interval $(0,2 \/ k]$ and increases on $[2 \/ k,+\\infty)$ because $\\left(t f^{\\prime}(t)\\right)^{\\prime}=\\frac{k(k t-2)}{4(1+k t)^{5 \/ 2}}$. It follows that two of the numbers $x, y, z$ are equal. If $x=y=z$, then $(1,1,1)$ is the only solution to (3). Suppose that $x=y \\neq z$. Since $\\left(y f^{\\prime}(y)\\right)^{2}-\\left(z f^{\\prime}(z)\\right)^{2}=$ $\\frac{k^{2}(z-y)\\left(k^{3} y^{2} z^{2}-3 k y z-y-z\\right)}{4(1+k y)^{3}(1+k z)^{3}},(3)$ gives us $y^{2} z=1$ and $k^{3} y^{2} z^{2}-3 k y z-y-z=$ 0 . Eliminating $z$ we obtain an equation in $y, k^{3} \/ y^{2}-3 k \/ y-y-1 \/ y^{2}=0$, whose only real solution is $y=k-1$. Thus $\\left(k-1, k-1,1 \/(k-1)^{2}\\right)$ and the cyclic permutations are the only solutions to (3) with $x, y, z$ being not all equal. Since $F\\left(k-1, k-1,1 \/(k-1)^{2}\\right)=(k+1) \/ \\sqrt{k^{2}-k+1}>$ $F(1,1,1)=1$, the inequality (2) follows. For $0\\frac{a}{\\sqrt{a^{2}+8 b c}}+$ $\\frac{b}{\\sqrt{b^{2}+8 c a}}+\\frac{c}{\\sqrt{c^{2}+8 a b}} \\geq 1$. If we fix $c$ and let $a, b$ tend to 0 , the first two summands will tend to 0 while the third will tend to 1 . Hence the inequality cannot be improved.","problem_type":"Algebra","tier":0} +{"year":"2001","problem_phase":"shortlisted","problem":"7. $\\mathbf{C 1}$ (COL) Let $A=\\left(a_{1}, a_{2}, \\ldots, a_{2001}\\right)$ be a sequence of positive integers. Let $m$ be the number of 3 -element subsequences $\\left(a_{i}, a_{j}, a_{k}\\right)$ with $1 \\leq i<$ $j$ $\\sin \\alpha=\\frac{2}{O_{i} O_{j}}$. By the lemma we have $l(P Q)+l(R S)=4 \\alpha r \\geq \\frac{8 r}{O_{i} O_{j}}$, and hence $$ \\frac{1}{O_{i} O_{j}} \\leq \\frac{l(P Q)+l(R S)}{8 r} $$ Now sum all these inequalities for $iA B$. As usual, $R, r, \\alpha, \\beta, \\gamma$ denote the circumradius and the inradius and the angles of $\\triangle A B C$, respectively. We have $\\tan \\angle B K M=D M \/ D K$. Straightforward calculation gives $D M=\\frac{1}{2} A D=R \\sin \\beta \\sin \\gamma$ and $D K=\\frac{D C-D B}{2}-\\frac{K C-K B}{2}=R \\sin (\\beta-$ $\\gamma)-R(\\sin \\beta-\\sin \\gamma)=4 R \\sin \\frac{\\beta-\\gamma}{2} \\sin \\frac{\\beta}{2} \\sin \\frac{\\gamma}{2}$, so we obtain $$ \\tan \\angle B K M=\\frac{\\sin \\beta \\sin \\gamma}{4 \\sin \\frac{\\beta-\\gamma}{2} \\sin \\frac{\\beta}{2} \\sin \\frac{\\gamma}{2}}=\\frac{\\cos \\frac{\\beta}{2} \\cos \\frac{\\gamma}{2}}{\\sin \\frac{\\beta-\\gamma}{2}} $$ To calculate the angle $B K N^{\\prime}$, we apply the inversion $\\psi$ with center at $K$ and power $B K \\cdot C K$. For each object $X$, we denote by $\\widehat{X}$ its image under $\\psi$. The incircle $\\Omega$ maps to a ![](https:\/\/cdn.mathpix.com\/cropped\/2024_11_18_8e985d6b9c83aa3e9d0eg-702.jpg?height=215&width=545&top_left_y=1844&top_left_x=807) line $\\widehat{\\Omega}$ parallel to $\\widehat{B} \\widehat{C}$, at distance $\\frac{B K \\cdot C K}{2 r}$ from $\\widehat{B} \\widehat{C}$. Thus the point $\\widehat{N^{\\prime}}$ is the projection of the midpoint $\\widehat{U}$ of $\\widehat{B} \\widehat{C}$ onto $\\widehat{\\Omega}$. Hence $$ \\tan \\angle B K N^{\\prime}=\\tan \\angle \\widehat{B} K \\widehat{N^{\\prime}}=\\frac{\\widehat{U} \\widehat{N^{\\prime}}}{\\widehat{U} K}=\\frac{B K \\cdot C K}{r(C K-B K)} . $$ Again, one easily checks that $K B \\cdot K C=b c \\sin ^{2} \\frac{\\alpha}{2}$ and $r=4 R \\sin \\frac{\\alpha}{2}$. $\\sin \\frac{\\beta}{2} \\cdot \\sin \\frac{\\gamma}{2}$, which implies $$ \\begin{aligned} \\tan \\angle B K N^{\\prime} & =\\frac{b c \\sin ^{2} \\frac{\\alpha}{2}}{r(b-c)} \\\\ & =\\frac{4 R^{2} \\sin \\beta \\sin \\gamma \\sin ^{2} \\frac{\\alpha}{2}}{4 R \\sin \\frac{\\alpha}{2} \\sin \\frac{\\beta}{2} \\sin \\frac{\\gamma}{2} \\cdot 2 R(\\sin \\beta-\\sin \\gamma)}=\\frac{\\cos \\frac{\\beta}{2} \\cos \\frac{\\gamma}{2}}{\\sin \\frac{\\beta-\\gamma}{2}} . \\end{aligned} $$ Hence $\\angle B K M=\\angle B K N^{\\prime}$, which implies that $K, M, N^{\\prime}$ are indeed collinear; thus $N^{\\prime} \\equiv N$.","problem_type":"Geometry","tier":0} +{"year":"2002","problem_phase":"shortlisted","problem":"14. G8 (ARM) Let $S_{1}$ and $S_{2}$ be circles meeting at the points $A$ and $B$. A line through $A$ meets $S_{1}$ at $C$ and $S_{2}$ at $D$. Points $M, N, K$ lie on the line segments $C D, B C, B D$ respectively, with $M N$ parallel to $B D$ and $M K$ parallel to $B C$. Let $E$ and $F$ be points on those $\\operatorname{arcs} B C$ of $S_{1}$ and $B D$ of $S_{2}$ respectively that do not contain $A$. Given that $E N$ is perpendicular to $B C$ and $F K$ is perpendicular to $B D$, prove that $\\measuredangle E M F=90^{\\circ}$.","solution":"14. Let $G$ be the other point of intersection of the line $F K$ with the arc $B A D$. Since $B N \/ N C=D K \/ K B$ and $\\angle C E B=\\angle B G D$ the triangles $C E B$ and $B G D$ are similar. Thus $B N \/ N E=D K \/ K G=F K \/ K B$. From $B N=M K$ and $B K=$ $M N$ it follows that $M N \/ N E=$ $F K \/ K M$. But we also have that $\\angle M N E=90^{\\circ}+\\angle M N B=90^{\\circ}+$ ![](https:\/\/cdn.mathpix.com\/cropped\/2024_11_18_8e985d6b9c83aa3e9d0eg-703.jpg?height=327&width=539&top_left_y=1011&top_left_x=817) $\\angle M K B=\\angle F K M$, and hence $\\triangle M N E \\sim \\triangle F K M$. Now $\\angle E M F=\\angle N M K-\\angle N M E-\\angle K M F=\\angle N M K-\\angle N M E-$ $\\angle N E M=\\angle N M K-90^{\\circ}+\\angle B N M=90^{\\circ}$ as claimed.","problem_type":"Geometry","tier":0} +{"year":"2002","problem_phase":"shortlisted","problem":"15. A1 (CZE) Find all functions $f$ from the reals to the reals such that $$ f(f(x)+y)=2 x+f(f(y)-x) $$ for all real $x, y$.","solution":"15. We observe first that $f$ is surjective. Indeed, setting $y=-f(x)$ gives $f(f(-f(x))-x)=f(0)-2 x$, where the right-hand expression can take any real value. In particular, there exists $x_{0}$ for which $f\\left(x_{0}\\right)=0$. Now setting $x=x_{0}$ in the functional equation yields $f(y)=2 x_{0}+f\\left(f(y)-x_{0}\\right)$, so we obtain $$ f(z)=z-x_{0} \\quad \\text { for } z=f(y)-x_{0} $$ Since $f$ is surjective, $z$ takes all real values. Hence for all $z, f(z)=z+c$ for some constant $c$, and this is indeed a solution.","problem_type":"Algebra","tier":0} +{"year":"2002","problem_phase":"shortlisted","problem":"16. A2 (YUG) Let $a_{1}, a_{2}, \\ldots$ be an infinite sequence of real numbers for which there exists a real number $c$ with $0 \\leq a_{i} \\leq c$ for all $i$ such that $$ \\left|a_{i}-a_{j}\\right| \\geq \\frac{1}{i+j} \\quad \\text { for all } i, j \\text { with } i \\neq j $$ Prove that $c \\geq 1$.","solution":"16. For $n \\geq 2$, let $\\left(k_{1}, k_{2}, \\ldots, k_{n}\\right)$ be the permutation of $\\{1,2, \\ldots, n\\}$ with $a_{k_{1}} \\leq a_{k_{2}} \\leq \\cdots \\leq a_{k_{n}}$. Then from the condition of the problem, using the Cauchy-Schwarz inequality, we obtain $$ \\begin{aligned} c & \\geq a_{k_{n}}-a_{k_{1}}=\\left|a_{k_{n}}-a_{k_{n-1}}\\right|+\\cdots+\\left|a_{k_{3}}-a_{k_{2}}\\right|+\\left|a_{k_{2}}-a_{k_{1}}\\right| \\\\ & \\geq \\frac{1}{k_{1}+k_{2}}+\\frac{1}{k_{2}+k_{3}}+\\cdots+\\frac{1}{k_{n-1}+k_{n}} \\\\ & \\geq \\frac{(n-1)^{2}}{\\left(k_{1}+k_{2}\\right)+\\left(k_{2}+k_{3}\\right)+\\cdots+\\left(k_{n-1}+k_{n}\\right)} \\\\ & =\\frac{(n-1)^{2}}{2\\left(k_{1}+k_{2}+\\cdots+k_{n}\\right)-k_{1}-k_{n}} \\geq \\frac{(n-1)^{2}}{n^{2}+n-3} \\geq \\frac{n-1}{n+2} . \\end{aligned} $$ Therefore $c \\geq 1-\\frac{3}{n+2}$ for every positive integer $n$. But if $c<1$, this inequality is obviously false for all $n>\\frac{3}{1-c}-2$. We conclude that $c \\geq 1$. Remark. The least value of $c$ is not greater than $2 \\ln 2$. An example of a sequence $\\left\\{a_{n}\\right\\}$ with $0 \\leq a_{n} \\leq 2 \\ln 2$ can be constructed inductively as follows: Given $a_{1}, a_{2}, \\ldots, a_{n-1}$, then $a_{n}$ can be any number from $[0,2 \\ln 2]$ that does not belong to any of the intervals $\\left(a_{i}-\\frac{1}{i+n}, a_{i}+\\frac{1}{i+n}\\right)(i=$ $1,2, \\ldots, n-1)$, and the total length of these intervals is always less than or equal to $$ \\frac{2}{n+1}+\\frac{2}{n+2}+\\cdots+\\frac{2}{2 n-1}<2 \\ln 2 $$","problem_type":"Algebra","tier":0} +{"year":"2002","problem_phase":"shortlisted","problem":"17. A3 (POL) Let $P$ be a cubic polynomial given by $P(x)=a x^{3}+b x^{2}+c x+$ $d$, where $a, b, c, d$ are integers and $a \\neq 0$. Suppose that $x P(x)=y P(y)$ for infinitely many pairs $x, y$ of integers with $x \\neq y$. Prove that the equation $P(x)=0$ has an integer root.","solution":"17. Let $x, y$ be distinct integers satisfying $x P(x)=y P(y)$; this is equivalent to $a\\left(x^{4}-y^{4}\\right)+b\\left(x^{3}-y^{3}\\right)+c\\left(x^{2}-y^{2}\\right)+d(x-y)=0$. Dividing by $x-y$ we obtain $$ a\\left(x^{3}+x^{2} y+x y^{2}+y^{3}\\right)+b\\left(x^{2}+x y+y^{2}\\right)+c(x+y)+d=0 . $$ Putting $x+y=p, x^{2}+y^{2}=q$ leads to $x^{2}+x y+y^{2}=\\frac{p^{2}+q}{2}$, so the above equality becomes $$ a p q+\\frac{b}{2}\\left(p^{2}+q\\right)+c p+d=0, \\quad \\text { i.e. } \\quad(2 a p+b) q=-\\left(b p^{2}+2 c p+2 d\\right) $$ Since $q \\geq p^{2} \/ 2$, it follows that $p^{2}|2 a p+b| \\leq 2\\left|b p^{2}+2 c p+2 d\\right|$, which is possible only for finitely many values of $p$, although there are infinitely many pairs $(x, y)$ with $x P(x)=y P(y)$. Hence there exists $p$ such that $x P(x)=(p-x) P(p-x)$ for infinitely many $x$, and therefore for all $x$. If $p \\neq 0$, then $p$ is a root of $P(x)$. If $p=0$, the above relation gives $P(x)=-P(-x)$. This forces $b=d=0$, so $P(x)=x\\left(a x^{2}+c\\right)$. Thus 0 is a root of $P(x)$.","problem_type":"Algebra","tier":0} +{"year":"2002","problem_phase":"shortlisted","problem":"18. A4 (IND) ${ }^{\\text {IMO5 }}$ Find all functions $f$ from the reals to the reals such that $$ (f(x)+f(z))(f(y)+f(t))=f(x y-z t)+f(x t+y z) $$ for all real $x, y, z, t$.","solution":"18. Putting $x=z=0$ and $t=y$ into the given equation gives $4 f(0) f(y)=$ $2 f(0)$ for all $y$. If $f(0) \\neq 0$, then we deduce $f(y)=\\frac{1}{2}$, i.e., $f$ is identically equal to $\\frac{1}{2}$. Now we suppose that $f(0)=0$. Setting $z=t=0$ we obtain $$ f(x y)=f(x) f(y) \\quad \\text { for all } x, y \\in \\mathbb{R} $$ Thus if $f(y)=0$ for some $y \\neq 0$, then $f$ is identically zero. So, assume $f(y) \\neq 0$ whenever $y \\neq 0$. Next, we observe that $f$ is strictly increasing on the set of positive reals. Actually, it follows from (1) that $f(x)=f(\\sqrt{x})^{2} \\geq 0$ for all $x \\geq 0$, so that the given equation for $t=x$ and $z=y$ yields $f\\left(x^{2}+y^{2}\\right)=(f(x)+f(y))^{2} \\geq$ $f\\left(x^{2}\\right)$ for all $x, y \\geq 0$. Using (1) it is easy to get $f(1)=1$. Now plugging $t=y$ into the given equation, we are led to $$ 2[f(x)+f(z)]=f(x-z)+f(x+z) \\quad \\text { for all } x, z $$ In particular, $f(z)=f(-z)$. Further, it is easy to get by induction from (2) that $f(n x)=n^{2} f(x)$ for all integers $n$ (and consequently for all rational numbers as well). Therefore $f(q)=f(-q)=q^{2}$ for all $q \\in \\mathbb{Q}$. But $f$ is increasing for $x>0$, so we must have $f(x)=x^{2}$ for all $x$. It is easy to verify that $f(x)=0, f(x)=\\frac{1}{2}$ and $f(x)=x^{2}$ are indeed solutions.","problem_type":"Algebra","tier":0} +{"year":"2002","problem_phase":"shortlisted","problem":"19. A5 (IND) Let $n$ be a positive integer that is not a perfect cube. Define real numbers $a, b, c$ by $$ a=\\sqrt[3]{n}, \\quad b=\\frac{1}{a-[a]}, \\quad c=\\frac{1}{b-[b]}, $$ where $[x]$ denotes the integer part of $x$. Prove that there are infinitely many such integers $n$ with the property that there exist integers $r, s, t$, not all zero, such that $r a+s b+t c=0$.","solution":"19. Write $m=[\\sqrt[3]{n}]$. To simplify the calculation, we shall assume that $[b]=1$. Then $a=\\sqrt[3]{n}, b=\\frac{1}{\\sqrt[3]{n}-m}=\\frac{1}{n-m^{3}}\\left(m^{2}+m \\sqrt[3]{n}+\\sqrt[3]{n^{2}}\\right), c=\\frac{1}{b-1}=$ $u+v \\sqrt[3]{n}+w \\sqrt[3]{n^{2}}$ for certain rational numbers $u, v, w$. Obviously, integers $r, s, t$ with $r a+s b+t c=0$ exist if (and only if) $u=m^{2} w$, i.e., if ( $b-$ 1) $\\left(m^{2} w+v \\sqrt[3]{n}+w \\sqrt[3]{n^{2}}\\right)=1$ for some rational $v, w$. When the last equality is expanded and simplified, comparing the coefficients at $1, \\sqrt[3]{n}, \\sqrt[3]{n^{2}}$ one obtains $$ \\begin{array}{rlrl} 1: & v+\\left(\\left(m^{2}+m^{3}-n\\right) m^{2}+m\\right) w & =n-m^{3}, \\\\ \\sqrt[3]{n}: & \\left(m^{2}+m^{3}-n\\right) v+ & \\left(m^{3}+n\\right) w & =0, \\\\ \\sqrt[3]{n^{2}}: & m v+ & \\left(2 m^{2}+m^{3}-n\\right) w & =0 . \\end{array} $$ In order for the system (1) to have a solution $v, w$, we must have $\\left(2 m^{2}+\\right.$ $\\left.m^{3}-n\\right)\\left(m^{2}+m^{3}-n\\right)=m\\left(m^{3}+n\\right)$. This quadratic equation has solutions $n=m^{3}$ and $n=m^{3}+3 m^{2}+m$. The former is not possible, but the latter gives $a-[a]>\\frac{1}{2}$, so $[b]=1$, and the system (1) in $v, w$ is solvable. Hence every number $n=m^{3}+3 m^{2}+m, m \\in \\mathbb{N}$, satisfies the condition of the problem.","problem_type":"Algebra","tier":0} +{"year":"2002","problem_phase":"shortlisted","problem":"2. N2 (ROM) ${ }^{\\mathrm{IMO} 4}$ Let $n \\geq 2$ be a positive integer, with divisors $1=d_{1}<$ $d_{2}<\\cdots1$. Certainly $n \\geq 2$ and $A$ is infinite. Define $f_{i}: A \\rightarrow A$ as $f_{i}(x)=b_{i} x+c_{i}$ for each $i$. By condition (ii), $f_{i}(x)=f_{j}(y)$ implies $i=j$ and $x=y$; iterating this argument, we deduce that $f_{i_{1}}\\left(\\ldots f_{i_{m}}(x) \\ldots\\right)=f_{j_{1}}\\left(\\ldots f_{j_{m}}(x) \\ldots\\right)$ implies $i_{1}=j_{1}, \\ldots, i_{m}=j_{m}$ and $x=y$. As an illustration, we shall consider the case $b_{1}=b_{2}=b_{3}=2$ first. If $a$ is large enough, then for any $i_{1}, \\ldots, i_{m} \\in\\{1,2,3\\}$ we have $f_{i_{1}} \\circ \\cdots \\circ f_{i_{m}}(a) \\leq$ $2.1^{m} a$. However, we obtain $3^{m}$ values in this way, so they cannot be all distinct if $m$ is sufficiently large, a contradiction. In the general case, let real numbers $d_{i}>b_{i}, i=1,2 \\ldots, n$, be chosen such that $\\frac{1}{d_{1}}+\\cdots+\\frac{1}{d_{n}}>1$ : for $a$ large enough, $f_{i}(x)0$ be arbitrary rational numbers with sum 1 ; denote by $N_{0}$ the least common multiple of their denominators. Let $N$ be a fixed multiple of $N_{0}$, so that each $k_{j} N$ is an integer. Consider all combinations $f_{i_{1}} \\circ \\cdots \\circ f_{i_{N}}$ of $N$ functions, among which each $f_{i}$ appears exactly $k_{i} N$ times. There are $F_{N}=\\frac{N!}{\\left(k_{1} N\\right)!\\cdots\\left(k_{n} N\\right)!}$ such combinations, so they give $F_{N}$ distinct values when applied to $a$. On the other hand, $f_{i_{1}} \\circ \\cdots \\circ f_{i_{N}}(a) \\leq\\left(d_{1}^{k_{1}} \\cdots d_{n}^{k_{n}}\\right)^{N} a$. Therefore $$ \\left(d_{1}^{k_{1}} \\cdots d_{n}^{k_{n}}\\right)^{N} a \\geq F_{N} \\quad \\text { for all } N, N_{0} \\mid N $$ It remains to find a lower estimate for $F_{N}$. In fact, it is straightforward to verify that $F_{N+N_{0}} \/ F_{N}$ tends to $Q^{N_{0}}$, where $Q=1 \/\\left(k_{1}^{k_{1}} \\cdots k_{n}^{k_{n}}\\right)$. Consequently, for every $q0$ such that $F_{N}>p q^{N}$. Then (1) implies that $$ \\left(\\frac{d_{1}^{k_{1}} \\cdots d_{n}^{k_{n}}}{q}\\right)^{N}>\\frac{p}{a} \\text { for every multiple } N \\text { of } N_{0} $$ and hence $d_{1}^{k_{1}} \\cdots d_{n}^{k_{n}} \/ q \\geq 1$. This must hold for every $q7$ this is possible as well: it follows by induction from Figure 2. ![](https:\/\/cdn.mathpix.com\/cropped\/2024_11_18_8e985d6b9c83aa3e9d0eg-707.jpg?height=186&width=195&top_left_y=302&top_left_x=402) Fig. 1 ![](https:\/\/cdn.mathpix.com\/cropped\/2024_11_18_8e985d6b9c83aa3e9d0eg-707.jpg?height=239&width=249&top_left_y=251&top_left_x=903) Fig. 2","problem_type":"Combinatorics","tier":0} +{"year":"2002","problem_phase":"shortlisted","problem":"23. C3 (COL) Let $n$ be a positive integer. A sequence of $n$ positive integers (not necessarily distinct) is called full if it satisfies the following condition: For each positive integer $k \\geq 2$, if the number $k$ appears in the sequence, then so does the number $k-1$, and moreover, the first occurrence of $k-1$ comes before the last occurrence of $k$. For each $n$, how many full sequences are there?","solution":"23. We claim that there are $n$ ! full sequences. To show this, we construct a bijection with the set of permutations of $\\{1,2, \\ldots, n\\}$. Consider a full sequence $\\left(a_{1}, a_{2}, \\ldots, a_{n}\\right)$, and let $m$ be the greatest of the numbers $a_{1}, \\ldots, a_{n}$. Let $S_{k}, 1 \\leq k \\leq m$, be the set of those indices $i$ for which $a_{i}=k$. Then $S_{1}, \\ldots S_{m}$ are nonempty and form a partition of the set $\\{1,2, \\ldots, n\\}$. Now we write down the elements of $S_{1}$ in descending order, then the elements of $S_{2}$ in descending order and so on. This maps the full sequence to a permutation of $\\{1,2, \\ldots, n\\}$. Moreover, this map is reversible, since each permutation uniquely breaks apart into decreasing sequences $S_{1}^{\\prime}, S_{2}^{\\prime}, \\ldots, S_{m}^{\\prime}$, so that $\\max S_{i}^{\\prime}>\\min S_{i-1}^{\\prime}$. Therefore the full sequences are in bijection with the permutations of $\\{1,2, \\ldots, n\\}$. Second solution. Let there be given a full sequence of length $n$. Removing from it the first occurrence of the highest number, we obtain a full sequence of length $n-1$. On the other hand, each full sequence of length $n-1$ can be obtained from exactly $n$ full sequences of length $n$. Therefore, if $x_{n}$ is the number of full sequences of length $n$, we deduce $x_{n}=n x_{n-1}$.","problem_type":"Combinatorics","tier":0} +{"year":"2002","problem_phase":"shortlisted","problem":"24. C4 (BUL) Let $T$ be the set of ordered triples $(x, y, z)$, where $x, y, z$ are integers with $0 \\leq x, y, z \\leq 9$. Players $A$ and $B$ play the following guessing game: Player $A$ chooses a triple $(x, y, z)$ in $T$, and Player $B$ has to discover A's triple in as few moves as possible. A move consists of the following: $B$ gives $A$ a triple $(a, b, c)$ in $T$, and $A$ replies by giving $B$ the number $|x+y-a-b|+|y+z-b-c|+|z+x-c-a|$. Find the minimum number of moves that $B$ needs to be sure of determining $A$ 's triple.","solution":"24. Two moves are not sufficient. Indeed, the answer to each move is an even number between 0 and 54 , so the answer takes at most 28 distinct values. Consequently, two moves give at most $28^{2}=784$ distinct outcomes, which is less than $10^{3}=1000$. We now show that three moves are sufficient. With the first move $(0,0,0)$, we get the reply $2(x+y+z)$, so we now know the value of $s=x+y+z$. Now there are several cases: (i) $s \\leq 9$. Then we ask $(9,0,0)$ as the second move and get $(9-x-y)+$ $(9-x-z)+(y+z)=18-2 x$, so we come to know $x$. Asking $(0,9,0)$ we obtain $y$, which is enough, since $z=s-x-y$. (ii) $10 \\leq s \\leq 17$. In this case the second move is $(9, s-9,0)$. The answer is $z+(9-x)+|x+z-9|=2 k$, where $k=z$ if $x+z \\geq 9$, or $k=9-x$ if $x+z<9$. In both cases we have $z \\leq k \\leq y+z \\leq s$. Let $s-k \\leq 9$. Then in the third move we ask $(s-k, 0, k)$ and obtain $|z-k|+|k-y-z|+y$, which is actually $(k-z)+(y+z-k)+y=2 y$. Thus we also find out $x+z$, and thus deduce whether $k$ is $z$ or $9-x$. Consequently we determine both $x$ and $z$. Let $s-k>9$. In this case, the third move is $(9, s-k-9, k)$. The answer is $|s-k-x-y|+|s-9-y-z|+|k+9-z-x|=$ $(k-z)+(9-x)+(9-x+k-z)=18+2 k-2(x+z)$, from which we find out again whether $k$ is $z$ or $9-x$. Now we are easily done. (iii) $18 \\leq s \\leq 27$. Then as in the first case, asking $(0,9,9)$ and $(9,0,9)$ we obtain $x$ and $y$.","problem_type":"Combinatorics","tier":0} +{"year":"2002","problem_phase":"shortlisted","problem":"25. C5 (BRA) Let $r \\geq 2$ be a fixed positive integer, and let $\\mathcal{F}$ be an infinite family of sets, each of size $r$, no two of which are disjoint. Prove that there exists a set of size $r-1$ that meets each set in $\\mathcal{F}$.","solution":"25. Assume to the contrary that no set of size less than $r$ meets all sets in $\\mathcal{F}$. Consider any set $A$ of size less than $r$ that is contained in infinitely many sets of $\\mathcal{F}$. By the assumption, $A$ is disjoint from some set $B \\in \\mathcal{F}$. Then of the infinitely many sets that contain $A$, each must meet $B$, so some element $b$ of $B$ belongs to infinitely many of them. But then the set $A \\cup\\{b\\}$ is contained in infinitely many sets of $\\mathcal{F}$ as well. Such a set $A$ exists: for example, the empty set. Now taking for $A$ the largest such set we come to a contradiction.","problem_type":"Combinatorics","tier":0} +{"year":"2002","problem_phase":"shortlisted","problem":"26. C6 (POL) Let $n$ be an even positive integer. Show that there is a permutation $x_{1}, x_{2}, \\ldots, x_{n}$ of $1,2, \\ldots, n$ such that for every $1 \\leq i \\leq n$ the number $x_{i+1}$ is one of $2 x_{i}, 2 x_{i}-1,2 x_{i}-n, 2 x_{i}-n-1$ (where we take $x_{n+1}=x_{1}$ ).","solution":"26. Write $n=2 m$. We shall define a directed graph $G$ with vertices $1, \\ldots, m$ and edges labelled $1,2, \\ldots, 2 m$ in such a way that the edges issuing from $i$ are labelled $2 i-1$ and $2 i$, and those entering it are labelled $i$ and $i+m$. What we need is an Euler circuit in $G$, namely a closed path that passes each edge exactly once. Indeed, if $x_{i}$ is the $i$ th edge in such a circuit, then $x_{i}$ enters some vertex $j$ and $x_{i+1}$ leaves it, so $x_{i} \\equiv j(\\bmod m)$ and $x_{i+1}=2 j-1$ or $2 j$. Hence $2 x_{i} \\equiv 2 j$ and $x_{i+1} \\equiv 2 x_{i}$ or $2 x_{i}-1(\\bmod n)$, as required. The graph $G$ is connected: by induction on $k$ there is a path from 1 to $k$, since 1 is connected to $j$ with $2 j=k$ or $2 j-1=k$, and there is an edge from $j$ to $k$. Also, the in-degree and out-degree of each vertex of $G$ are equal (to 2), and thus by a known result, $G$ contains an Euler circuit.","problem_type":"Combinatorics","tier":0} +{"year":"2002","problem_phase":"shortlisted","problem":"27. C7 (NZL) Among a group of 120 people, some pairs are friends. A weak quartet is a set of four people containing exactly one pair of friends. What is the maximum possible number of weak quartets?","solution":"27. For a graph $G$ on 120 vertices (i.e., people at the party), write $q(G)$ for the number of weak quartets in $G$. Our solution will consist of three parts. First, we prove that some graph $G$ with maximal $q(G)$ breaks up as a disjoint union of complete graphs. This will follow if we show that any two adjacent vertices $x, y$ have the same neighbors (apart from themselves). Let $G_{x}$ be the graph obtained from $G$ by \"copying\" $x$ to $y$ (i.e., for each $z \\neq x, y$, we add the edge $z y$ if $z x$ is an edge, and delete $z y$ if $z x$ is not an edge). Similarly $G_{y}$ is the graph obtained from $G$ by copying $y$ to $x$. We claim that $2 q(G) \\leq q\\left(G_{x}\\right)+q\\left(G_{y}\\right)$. Indeed, the number of weak quartets containing neither $x$ nor $y$ is the same in $G, G_{x}$, and $G_{y}$, while the number of those containing both $x$ and $y$ is not less in $G_{x}$ and $G_{y}$ than in $G$. Also, the number containing exactly one of $x$ and $y$ in $G_{x}$ is at least twice the number in $G$ containing $x$ but not $y$, while the number containing exactly one of $x$ and $y$ in $G_{y}$ is at least twice the number in $G$ containing $y$ but not $x$. This justifies our claim by adding. It follows that for an extremal graph $G$ we must have $q(G)=q\\left(G_{x}\\right)=q\\left(G_{y}\\right)$. Repeating the copying operation pair by pair ( $y$ to $x$, then their common neighbor $z$ to both $x, y$, etc.) we eventually obtain an extremal graph consisting of disjoint complete graphs. Second, suppose the complete graphs in $G$ have sizes $a_{1}, a_{2}, \\ldots, a_{n}$. Then $$ q(G)=\\sum_{i=1}^{n}\\binom{a_{i}}{2} \\sum_{\\substack{j2^{a b}>2^{2 a} 2^{2 b}>Q^{2}$ if $a, b \\geq 5$ ). Then $N$ has at least twice as many divisors as $Q$ (because for every $d \\mid Q$ both $d$ and $N \/ d$ are divisors of $N$ ), which counts up to $4^{n}$ divisors, as required. Remark. With some knowledge of cyclotomic polynomials, one can show that $2^{p_{1} \\cdots p_{n}}+1$ has at least $2^{2^{n-1}}$ divisors, far exceeding $4^{n}$.","problem_type":"Number Theory","tier":0} +{"year":"2002","problem_phase":"shortlisted","problem":"4. N4 (GER) Is there a positive integer $m$ such that the equation $$ \\frac{1}{a}+\\frac{1}{b}+\\frac{1}{c}+\\frac{1}{a b c}=\\frac{m}{a+b+c} $$ has infinitely many solutions in positive integers $a, b, c$ ?","solution":"4. For $a=b=c=1$ we obtain $m=12$. We claim that the given equation has infinitely many solutions in positive integers $a, b, c$ for this value of $m$. After multiplication by $a b c(a+b+c)$ the equation $\\frac{1}{a}+\\frac{1}{b}+\\frac{1}{c}+\\frac{1}{a b c}-\\frac{12}{a+b+c}=0$ becomes $$ a^{2}(b+c)+b^{2}(c+a)+c^{2}(a+b)+a+b+c-9 a b c=0 $$ We must show that this equation has infinitely many solutions in positive integers. Suppose that $(a, b, c)$ is one such solution with $a1$. We show that all $a_{n}$ 's are integers. This procedure is fairly standard. The above relation for $n$ and $n-1$ gives $a_{n+1} a_{n-2}=a_{n} a_{n-1}+1$ and $a_{n-1} a_{n-2}+1=a_{n} a_{n-3}$, so that adding yields $a_{n-2}\\left(a_{n-1}+a_{n+1}\\right)=$ $a_{n}\\left(a_{n-1}+a_{n-3}\\right)$. Therefore $\\frac{a_{n+1}+a_{n-1}}{a_{n}}=\\frac{a_{n-1}+a_{n-3}}{a_{n-2}}=\\cdots$, from which it follows that $$ \\frac{a_{n+1}+a_{n-1}}{a_{n}}=\\left\\{\\begin{array}{l} \\frac{a_{2}+a_{0}}{a_{1}}=2 \\text { for } n \\text { odd; } \\\\ \\frac{a_{3}+a_{1}}{a_{2}}=3 \\text { for } n \\text { even. } \\end{array}\\right. $$ It is now an immediate consequence that every $a_{n}$ is integral. Also, the above consideration implies that $\\left(a_{n-1}, a_{n}, a_{n+1}\\right)$ is a solution of (1) for each $n \\geq 1$. Since $a_{n}$ is strictly increasing, this gives an infinity of solutions in integers. Remark. There are infinitely many values of $m \\in \\mathbb{N}$ for which the given equation has at least one solution in integers, and each of those values admits an infinity of solutions.","problem_type":"Number Theory","tier":0} +{"year":"2002","problem_phase":"shortlisted","problem":"5. N5 (IRN) Let $m, n \\geq 2$ be positive integers, and let $a_{1}, a_{2}, \\ldots, a_{n}$ be integers, none of which is a multiple of $m^{n-1}$. Show that there exist integers $e_{1}, e_{2}, \\ldots, e_{n}$, not all zero, with $\\left|e_{i}\\right|$ $\\alpha^{m}+\\alpha-1=0$; hence $m<2 n$. Now we have $F(x) \/ G(x)=x^{m-n}-\\left(x^{m-n+2}-x^{m-n}-x+1\\right) \/ G(x)$, so $H(x)=x^{m-n+2}-x^{m-n}-x+1$ is also divisible by $G(x)$; but $\\operatorname{deg} H(x)=$ $m-n+2 \\leq n+1=\\operatorname{deg} G(x)+1$, from which we deduce that either $H(x)=G(x)$ or $H(x)=(x-a) G(x)$ for some $a \\in \\mathbb{Z}$. The former case is impossible. In the latter case we must have $m=2 n-1$, and thus $H(x)=x^{n+1}-x^{n-1}-x+1$; on the other hand, putting $x=1$ gives $a=1$, so $H(x)=(x-1)\\left(x^{n}+x^{2}-1\\right)=x^{n+1}-x^{n}+x^{3}-x^{2}-x+1$. This is possible only if $n=3$ and $m=5$. Remark. It is an old (though difficult) result that the polynomial $x^{n} \\pm$ $x^{k} \\pm 1$ is either irreducible or equals $x^{2} \\pm x+1$ times an irreducible factor.","problem_type":"Number Theory","tier":0} +{"year":"2002","problem_phase":"shortlisted","problem":"7. G1 (FRA) Let $B$ be a point on a circle $S_{1}$, and let $A$ be a point distinct from $B$ on the tangent at $B$ to $S_{1}$. Let $C$ be a point not on $S_{1}$ such that the line segment $A C$ meets $S_{1}$ at two distinct points. Let $S_{2}$ be the circle touching $A C$ at $C$ and touching $S_{1}$ at a point $D$ on the opposite side of $A C$ from $B$. Prove that the circumcenter of triangle $B C D$ lies on the circumcircle of triangle $A B C$.","solution":"7. To avoid working with cases, we use oriented angles modulo $180^{\\circ}$. Let $K$ be the circumcenter of $\\triangle B C D$, and $X$ any point on the common tangent to the circles at $D$. Since the tangents at the ends of a chord are equally inclined to the chord, we have $\\angle B A C=\\angle A B D+\\angle B D C+\\angle D C A=$ $\\angle B D X+\\angle B D C+\\angle X D C=2 \\angle B D C=\\angle B K C$. It follows that $B, C, A, K$ are concyclic, as required.","problem_type":"Geometry","tier":0} +{"year":"2002","problem_phase":"shortlisted","problem":"8. G2 (KOR) Let $A B C$ be a triangle for which there exists an interior point $F$ such that $\\angle A F B=\\angle B F C=\\angle C F A$. Let the lines $B F$ and $C F$ meet the sides $A C$ and $A B$ at $D$ and $E$ respectively. Prove that $$ A B+A C \\geq 4 D E $$","solution":"8. Construct equilateral triangles $A C P$ and $A B Q$ outside the triangle $A B C$. Since $\\angle A P C+\\angle A F C=60^{\\circ}+120^{\\circ}=180^{\\circ}$, the points $A, C, F, P$ lie on a circle; hence $\\angle A F P=\\angle A C P=60^{\\circ}=\\angle A F D$, so $D$ lies on the segment $F P$; similarly, $E$ lies on $F Q$. Further, note that $$ \\frac{F P}{F D}=1+\\frac{D P}{F D}=1+\\frac{S_{A P C}}{S_{A F C}} \\geq 4 $$ with equality if $F$ is the midpoint of the smaller $\\operatorname{arc} A C$ : hence $F D \\leq \\frac{1}{4} F P$ and $F E \\leq \\frac{1}{4} F Q$. Then by the law of cosines, $$ \\begin{aligned} D E & =\\sqrt{F D^{2}+F E^{2}+F D \\cdot F E} \\\\ & \\leq \\frac{1}{4} \\sqrt{F P^{2}+F Q^{2}+F P \\cdot F Q}=\\frac{1}{4} P Q \\leq A P+A Q=A B+A C . \\end{aligned} $$ Equality holds if and only if $\\triangle A B C$ is equilateral.","problem_type":"Geometry","tier":0} +{"year":"2002","problem_phase":"shortlisted","problem":"9. G3 (KOR) ${ }^{\\mathrm{IMO} 2}$ The circle $S$ has center $O$, and $B C$ is a diameter of $S$. Let $A$ be a point of $S$ such that $\\measuredangle A O B<120^{\\circ}$. Let $D$ be the midpoint of the arc $A B$ that does not contain $C$. The line through $O$ parallel to $D A$ meets the line $A C$ at $I$. The perpendicular bisector of $O A$ meets $S$ at $E$ and at $F$. Prove that $I$ is the incenter of the triangle $C E F$.","solution":"9. Since $\\angle B C A=\\frac{1}{2} \\angle B O A=\\angle B O D$, the lines $C A$ and $O D$ are parallel, so that $O D A I$ is a parallelogram. It follows that $A I=O D=O E=A E=$ $A F$. Hence $\\angle I F E=\\angle I F A-\\angle E F A=\\angle A I F-\\angle E C A=\\angle A I F-\\angle A C F=\\angle C F I$. Also, from $A E=A F$ we get that $C I$ bisects $\\angle E C F$. Therefore $I$ is the incenter of $\\triangle C E F$.","problem_type":"Geometry","tier":0} +{"year":"2003","problem_phase":"shortlisted","problem":"1. A1 (USA) Let $a_{i j}, i=1,2,3, j=1,2,3$, be real numbers such that $a_{i j}$ is positive for $i=j$ and negative for $i \\neq j$. Prove that there exist positive real numbers $c_{1}, c_{2}, c_{3}$ such that the numbers $$ a_{11} c_{1}+a_{12} c_{2}+a_{13} c_{3}, \\quad a_{21} c_{1}+a_{22} c_{2}+a_{23} c_{3}, \\quad a_{31} c_{1}+a_{32} c_{2}+a_{33} c_{3} $$ are all negative, all positive, or all zero.","solution":"1. Consider the points $O(0,0,0), P\\left(a_{11}, a_{21}, a_{31}\\right), Q\\left(a_{12}, a_{22}, a_{32}\\right), R\\left(a_{13}, a_{23}\\right.$, $\\left.a_{33}\\right)$ in three-dimensional Euclidean space. It is enough to find a point $U\\left(u_{1}, u_{2}, u_{3}\\right)$ in the interior of the triangle $P Q R$ whose coordinates are all positive, all negative, or all zero (indeed, then we have $\\overrightarrow{O U}=c_{1} \\overrightarrow{O P}+$ $c_{2} \\overrightarrow{O Q}+c_{3} \\overrightarrow{O R}$ for some $c_{1}, c_{2}, c_{3}>0$ with $\\left.c_{1}+c_{2}+c_{3}=1\\right)$. Let $P^{\\prime}\\left(a_{11}, a_{21}, 0\\right), Q^{\\prime}\\left(a_{12}, a_{22}, 0\\right)$, and $R^{\\prime}\\left(a_{13}, a_{23}, 0\\right)$ be the projections of $P, Q$, and $R$ onto the $O x y$ plane. We see that $P^{\\prime}, Q^{\\prime}, R^{\\prime}$ lie in the fourth, second, and third quadrants, respectively. We have the following two cases: (i) $O$ is in the exterior of $\\triangle P^{\\prime} Q^{\\prime} R^{\\prime}$. Set $S^{\\prime}=O R^{\\prime} \\cap P^{\\prime} Q^{\\prime}$ and let $S$ be the point of the segment $P Q$ that projects to $S^{\\prime}$. The point $S$ has its $z$ coordinate negative (because the $z$ coordinates of $P$ and $Q$ are negative). Thus any point ![](https:\/\/cdn.mathpix.com\/cropped\/2024_11_18_8e985d6b9c83aa3e9d0eg-710.jpg?height=290&width=446&top_left_y=702&top_left_x=875) of the segment $S R$ sufficiently close to $S$ has all coordinates negative. (ii) $O$ is in the interior or on the boundary of $\\triangle P^{\\prime} Q^{\\prime} R^{\\prime}$. Let $T$ be the point in the plane $P Q R$ whose projection is $O$. If $T=O$, then all coordinates of $T$ are zero, and we are done. Otherwise $O$ is interior to $\\triangle P^{\\prime} Q^{\\prime} R^{\\prime}$. Suppose that the $z$ coordinate of $T$ is positive (negative). Since $x$ and $y$ coordinates of $T$ are equal to 0 , there is a point $U$ inside $P Q R$ close to $T$ with both $x$ and $y$ coordinates positive (respectively negative), and this point $U$ has all coordinates of the same sign.","problem_type":"Algebra","tier":0} +{"year":"2003","problem_phase":"shortlisted","problem":"10. C4 (IRN) Let $x_{1}, \\ldots, x_{n}$ and $y_{1}, \\ldots, y_{n}$ be real numbers. Let $A=$ $\\left(a_{i j}\\right)_{1 \\leq i, j \\leq n}$ be the matrix with entries $$ a_{i j}= \\begin{cases}1, & \\text { if } x_{i}+y_{j} \\geq 0 \\\\ 0, & \\text { if } x_{i}+y_{j}<0\\end{cases} $$ Suppose that $B$ is an $n \\times n$ matrix whose entries are 0,1 such that the sum of the elements in each row and each column of $B$ is equal to the corresponding sum for the matrix $A$. Prove that $A=B$.","solution":"10. Denote by $b_{i j}$ the entries of the matrix $B$. Suppose the contrary, i.e., that there is a pair $\\left(i_{0}, j_{0}\\right)$ such that $a_{i_{0}, j_{0}} \\neq b_{i_{0}, j_{0}}$. We may assume without loss of generality that $a_{i_{0}, j_{0}}=0$ and $b_{i_{0}, j_{0}}=1$. Since the sums of elements in the $i_{0}$ th rows of the matrices $A$ and $B$ are equal, there is some $j_{1}$ for which $a_{i_{0}, j_{1}}=1$ and $b_{i_{0}, j_{1}}=0$. Similarly, from the fact that the sums in the $j_{1}$ th columns of the matrices $A$ and $B$ are equal, we conclude that there exists $i_{1}$ such that $a_{i_{1}, j_{1}}=0$ and $b_{i_{1}, j_{1}}=1$. Continuing this procedure, we construct two sequences $i_{k}, j_{k}$ such that $a_{i_{k}, j_{k}}=0, b_{i_{k}, j_{k}}=1, a_{i_{k}, j_{k+1}}=1, b_{i_{k}, j_{k+1}}=0$. Since the set of the pairs $\\left(i_{k}, j_{k}\\right)$ is finite, there are two different numbers $t, s$ such that $\\left(i_{t}, j_{t}\\right)=\\left(i_{s}, j_{s}\\right)$. From the given condition we have that $x_{i_{k}}+y_{i_{k}}<0$ and $x_{i_{k+1}}+y_{i_{k+1}} \\geq 0$. But $j_{t}=j_{s}$, and hence $0 \\leq \\sum_{k=s}^{t-1}\\left(x_{i_{k}}+y_{j_{k+1}}\\right)=$ $\\sum_{k=s}^{t-1}\\left(x_{i_{k}}+y_{j_{k}}\\right)<0$, a contradiction.","problem_type":"Combinatorics","tier":0} +{"year":"2003","problem_phase":"shortlisted","problem":"11. C5 (ROM) Every point with integer coordinates in the plane is the center of a disk with radius $1 \/ 1000$. (a) Prove that there exists an equilateral triangle whose vertices lie in different disks. (b) Prove that every equilateral triangle with vertices in different disks has side length greater than 96.","solution":"11. (a) By the pigeonhole principle there are two different integers $x_{1}, x_{2}$, $x_{1}>x_{2}$, such that $\\left|\\left\\{x_{1} \\sqrt{3}\\right\\}-\\left\\{x_{2} \\sqrt{3}\\right\\}\\right|<0.001$. Set $a=x_{1}-x_{2}$. Consider the equilateral triangle with vertices $(0,0),(2 a, 0),(a, a \\sqrt{3})$. The points $(0,0)$ and $(2 a, 0)$ are lattice points, and we claim that the point $(a, a \\sqrt{3})$ is at distance less than 0.001 from a lattice point. Indeed, since $0.001>\\left|\\left\\{x_{1} \\sqrt{3}\\right\\}-\\left\\{x_{2} \\sqrt{3}\\right\\}\\right|=\\left|a \\sqrt{3}-\\left(\\left[x_{1} \\sqrt{3}\\right]-\\left[x_{2} \\sqrt{3}\\right]\\right)\\right|$, we see that the distance between the points $(a, a \\sqrt{3})$ and $\\left(a,\\left[x_{1} \\sqrt{3}\\right]-\\right.$ $\\left.\\left[x_{2} \\sqrt{3}\\right]\\right)$ is less than 0.001 , and the point $\\left(a,\\left[x_{1} \\sqrt{3}\\right]-\\left[x_{2} \\sqrt{3}\\right]\\right)$ is with integer coefficients. (b) Suppose that $P^{\\prime} Q^{\\prime} R^{\\prime}$ is an equilateral triangle with side length $l \\leq 96$ such that each of its vertices $P^{\\prime}, Q^{\\prime}, R^{\\prime}$ lies in a disk of radius 0.001 centered at a lattice point. Denote by $P, Q, R$ the centers of these disks. Then we have $l-0.002 \\leq P Q, Q R, R P \\leq l+0.002$. Since $P Q R$ is not an equilateral triangle, two of its sides are different, say $P Q \\neq Q R$. On the other hand, $P Q^{2}, Q R^{2}$ are integers, so we have $1 \\leq\\left|P Q^{2}-Q R^{2}\\right|=(P Q+Q R)|P Q-Q R| \\leq 0.004(P Q+Q R) \\leq$ $(2 l+0.004) \\cdot 0.004 \\leq 2 \\cdot 96.002 \\cdot 0.004<1$, which is a contradiction.","problem_type":"Combinatorics","tier":0} +{"year":"2003","problem_phase":"shortlisted","problem":"12. C6 (SAF) Let $f(k)$ be the number of integers $n$ that satisfy the following conditions: (i) $0 \\leq n<10^{k}$, so $n$ has exactly $k$ digits (in decimal notation), with leading zeros allowed; (ii) the digits of $n$ can be permuted in such a way that they yield an integer divisible by 11 . Prove that $f(2 m)=10 f(2 m-1)$ for every positive integer $m$.","solution":"12. Denote by $\\overline{a_{k-1} a_{k-2} \\ldots a_{0}}$ the decimal representation of a number whose digits are $a_{k-1}, \\ldots, a_{0}$. We will use the following well-known fact: $$ \\overline{a_{k-1} a_{k-2} \\ldots a_{0}} \\equiv i(\\bmod 11) \\Longleftrightarrow \\sum_{l=0}^{k-1}(-1)^{l} a_{l} \\equiv i(\\bmod 11) . $$ Let $m$ be a positive integer. Define $A$ as the set of integers $n(0 \\leq n<$ $10^{2 m}$ ) whose right $2 m-1$ digits can be so permuted to yield an integer divisible by 11 , and $B$ as the set of integers $n\\left(0 \\leq n<10^{2 m-1}\\right)$ whose digits can be permuted resulting in an integer divisible by 11. Suppose that $a=\\overline{a_{2 m-1} \\ldots a_{0}} \\in A$. Then there that satisfies $$ \\sum_{l=0}^{2 m-1}(-1)^{l} b_{l} \\equiv 0(\\bmod 11) $$ The $2 m$-tuple $\\left(b_{2 m-1}, \\ldots, b_{0}\\right)$ satisfies (1) if and only if the $2 m$-tuple $\\left(k b_{2 m-1}+l, \\ldots, k b_{0}+l\\right)$ satisfies ( 1 ), where $k, l \\in \\mathbb{Z}, 11 \\nmid k$. Since $a_{0}+1 \\not \\equiv 0(\\bmod 11)$, we can choose $k$ from the set $\\{1, \\ldots, 10\\}$ such that $\\left(a_{0}+1\\right) k \\equiv 1(\\bmod 11)$. Thus there is a permutation of the $2 m$-tuple $\\left(\\left(a_{2 m-1}+1\\right) k-1, \\ldots,\\left(a_{1}+1\\right) k-1,0\\right)$ satisfying (1). Interchanging odd and even positions if necessary, we may assume that this permutation keeps the 0 at the last position. Since $\\left(a_{i}+1\\right) k$ is not divisible by 11 for any $i$, there is a unique $b_{i} \\in\\{0,1, \\ldots, 9\\}$ such that $b_{i} \\equiv\\left(a_{i}+1\\right) k-1(\\bmod 11)$. Hence the number $\\overline{b_{2 m-1} \\ldots b_{1}}$ belongs to $B$. Thus for fixed $a_{0} \\in\\{0,1,2, \\ldots, 9\\}$, to each $a \\in A$ such that the last digit of $a$ is $a_{0}$ we associate a unique $b \\in B$. Conversely, having $a_{0} \\in$ $\\{0,1,2, \\ldots, 9\\}$ fixed, from any number $\\overline{b_{2 m-1} \\ldots b_{1}} \\in B$ we can reconstruct $\\overline{a_{2 m-1} \\ldots a_{1} a_{0}} \\in A$. Hence $|A|=10|B|$, i.e., $f(2 m)=10 f(2 m-1)$.","problem_type":"Combinatorics","tier":0} +{"year":"2003","problem_phase":"shortlisted","problem":"13. G1 (FIN) ${ }^{\\mathrm{IMO} 4}$ Let $A B C D$ be a cyclic quadrilateral. Let $P, Q, R$ be the feet of the perpendiculars from $D$ to the lines $B C, C A, A B$, respectively. Show that $P Q=Q R$ if and only if the bisectors of $\\angle A B C$ and $\\angle A D C$ are concurrent with $A C$.","solution":"13. Denote by $K$ and $L$ the intersections of the bisectors of $\\angle A B C$ and $\\angle A D C$ with the line $A C$, respectively. Since $A B: B C=A K: K C$ and $A D: D C=A L: L C$, we have to prove that $$ P Q=Q R \\Leftrightarrow \\frac{A B}{B C}=\\frac{A D}{D C} . $$ Since the quadrilaterals $A Q D R$ and $Q P C D$ are cyclic, we see that ![](https:\/\/cdn.mathpix.com\/cropped\/2024_11_18_8e985d6b9c83aa3e9d0eg-715.jpg?height=403&width=399&top_left_y=1567&top_left_x=862) $\\angle R D Q=\\angle B A C$ and $\\angle Q D P=\\angle A C B$. By the law of sines it follows that $\\frac{A B}{B C}=\\frac{\\sin (\\angle A C B)}{\\sin (\\angle B A C)}$ and that $Q R=A D \\sin (\\angle R D Q), Q P=$ $C D \\sin (\\angle Q D P)$. Now we have $$ \\frac{A B}{B C}=\\frac{\\sin (\\angle A C B)}{\\sin (\\angle B A C)}=\\frac{\\sin (\\angle Q D P)}{\\sin (\\angle R D Q)}=\\frac{A D \\cdot Q P}{Q R \\cdot C D} $$ The statement (1) follows directly.","problem_type":"Geometry","tier":0} +{"year":"2003","problem_phase":"shortlisted","problem":"14. G2 (GRE) Three distinct points $A, B, C$ are fixed on a line in this order. Let $\\Gamma$ be a circle passing through $A$ and $C$ whose center does not lie on the line $A C$. Denote by $P$ the intersection of the tangents to $\\Gamma$ at $A$ and $C$. Suppose $\\Gamma$ meets the segment $P B$ at $Q$. Prove that the intersection of the bisector of $\\angle A Q C$ and the line $A C$ does not depend on the choice of $\\Gamma$.","solution":"14. Denote by $R$ the intersection point of the bisector of $\\angle A Q C$ and the line $A C$. From $\\triangle A C Q$ we get $$ \\frac{A R}{R C}=\\frac{A Q}{Q C}=\\frac{\\sin \\angle Q C A}{\\sin \\angle Q A C} $$ By the sine version of Ceva's theorem we have $\\frac{\\sin \\angle A P B}{\\sin \\angle B P C} \\cdot \\frac{\\sin \\angle Q A C}{\\sin \\angle P A Q}$. $\\frac{\\sin \\angle Q C P}{\\sin \\angle Q C A}=1$, which is equivalent to $$ \\frac{\\sin \\angle A P B}{\\sin \\angle B P C}=\\left(\\frac{\\sin \\angle Q C A}{\\sin \\angle Q A C}\\right)^{2} $$ because $\\angle Q C A=\\angle P A Q$ and $\\angle Q A C=\\angle Q C P$. Denote by $S(X Y Z)$ the area of a triangle $X Y Z$. Then $$ \\frac{\\sin \\angle A P B}{\\sin \\angle B P C}=\\frac{A P \\cdot B P \\cdot \\sin \\angle A P B}{B P \\cdot C P \\cdot \\sin \\angle B P C}=\\frac{S(\\Delta A B P)}{S(\\Delta B C P)}=\\frac{A B}{B C}, $$ which implies that $\\left(\\frac{A R}{R C}\\right)^{2}=\\frac{A B}{B C}$. Hence $R$ does not depend on $\\Gamma$.","problem_type":"Geometry","tier":0} +{"year":"2003","problem_phase":"shortlisted","problem":"15. G3 (IND) Let $A B C$ be a triangle and let $P$ be a point in its interior. Denote by $D, E, F$ the feet of the perpendiculars from $P$ to the lines $B C$, $C A$, and $A B$, respectively. Suppose that $$ A P^{2}+P D^{2}=B P^{2}+P E^{2}=C P^{2}+P F^{2} $$ Denote by $I_{A}, I_{B}, I_{C}$ the excenters of the triangle $A B C$. Prove that $P$ is the circumcenter of the triangle $I_{A} I_{B} I_{C}$.","solution":"15. From the given equality we see that $0=\\left(B P^{2}+P E^{2}\\right)-\\left(C P^{2}+P F^{2}\\right)=$ $B F^{2}-C E^{2}$, so $B F=C E=x$ for some $x$. Similarly, there are $y$ and $z$ such that $C D=A F=y$ and $B D=A E=z$. It is easy to verify that $D$, $E$, and $F$ must lie on the segments $B C, C A, A B$. Denote by $a, b, c$ the length of the segments $B C, C A, A B$. It follows that $a=z+y, b=z+x, c=x+y$, so $D, E, F$ are the points where the excircles touch the sides of $\\triangle A B C$. Hence $P, D$, and $I_{A}$ are collinear and $$ \\angle P I_{A} C=\\angle D I_{A} C=90^{\\circ}-\\frac{180^{\\circ}-\\angle A C B}{2}=\\frac{\\angle A C B}{2} $$ In the same way we obtain that $\\angle P I_{B} C=\\frac{\\angle A C B}{2}$ and $P I_{B}=P I_{A}$. Analogously, we get $P I_{C}=P I_{B}$, which implies that $P$ is the circumcenter of the triangle $I_{A} I_{B} I_{C}$.","problem_type":"Geometry","tier":0} +{"year":"2003","problem_phase":"shortlisted","problem":"16. G4 (ARM) Let $\\Gamma_{1}, \\Gamma_{2}, \\Gamma_{3}, \\Gamma_{4}$ be distinct circles such that $\\Gamma_{1}, \\Gamma_{3}$ are externally tangent at $P$, and $\\Gamma_{2}, \\Gamma_{4}$ are externally tangent at the same point $P$. Suppose that $\\Gamma_{1}$ and $\\Gamma_{2} ; \\Gamma_{2}$ and $\\Gamma_{3} ; \\Gamma_{3}$ and $\\Gamma_{4} ; \\Gamma_{4}$ and $\\Gamma_{1}$ meet at $A, B, C, D$, respectively, and that all these points are different from $P$. Prove that $$ \\frac{A B \\cdot B C}{A D \\cdot D C}=\\frac{P B^{2}}{P D^{2}} $$","solution":"16. Apply an inversion with center at $P$ and radius $r$; let $\\widehat{X}$ denote the image of $X$. The circles $\\Gamma_{1}, \\Gamma_{2}, \\Gamma_{3}, \\Gamma_{4}$ are transformed into lines $\\widehat{\\Gamma_{1}}, \\widehat{\\Gamma_{2}}, \\widehat{\\Gamma_{3}}, \\widehat{\\Gamma}_{4}$, where $\\widehat{\\Gamma_{1}} \\| \\widehat{\\Gamma_{3}}$ and $\\widehat{\\Gamma_{2}} \\| \\widehat{\\Gamma_{4}}$, and therefore $\\widehat{A} \\widehat{B} \\widehat{C} \\widehat{D}$ is a parallelogram. Further, we have $A B=\\frac{r^{2}}{P \\widehat{A} \\cdot P \\widehat{B}} \\widehat{A} \\widehat{B}, B C=\\frac{r^{2}}{P \\widehat{B} \\cdot P \\widehat{C}} \\widehat{B} \\widehat{C}, C D=\\frac{r^{2}}{P \\widehat{C} \\cdot P \\widehat{D}} \\widehat{C} \\widehat{D}$, $D A=\\frac{r^{2}}{P \\widehat{D} \\cdot P \\widehat{A}} \\widehat{D} \\widehat{A}$ and $P B=\\frac{r^{2}}{P \\widehat{B}}, P D=\\frac{r^{2}}{P \\widehat{D}}$. The equality to be proven becomes $$ \\frac{P \\widehat{D}^{2}}{P \\widehat{B}^{2}} \\cdot \\frac{\\widehat{A} \\widehat{B} \\cdot \\widehat{B} \\widehat{C}}{\\widehat{A} \\cdot \\widehat{D} \\widehat{C}}=\\frac{P \\widehat{D}^{2}}{P \\widehat{B}^{2}} $$ which holds because $\\widehat{A} \\widehat{B}=\\widehat{C} \\widehat{D}$ and $\\widehat{B} \\widehat{C}=\\widehat{D} \\widehat{A}$.","problem_type":"Geometry","tier":0} +{"year":"2003","problem_phase":"shortlisted","problem":"17. G5 (KOR) Let $A B C$ be an isosceles triangle with $A C=B C$, whose incenter is $I$. Let $P$ be a point on the circumcircle of the triangle $A I B$ lying inside the triangle $A B C$. The lines through $P$ parallel to $C A$ and $C B$ meet $A B$ at $D$ and $E$, respectively. The line through $P$ parallel to $A B$ meets $C A$ and $C B$ at $F$ and $G$, respectively. Prove that the lines $D F$ and $E G$ intersect on the circumcircle of the triangle $A B C$.","solution":"17. The triangles $P D E$ and $C F G$ are homothetic; hence lines $F D, G E$, and $C P$ intersect at one point. Let $Q$ be the intersection point of the line $C P$ and the circumcircle of $\\triangle A B C$. The required statement will follow if we show that $Q$ lies on the lines $G E$ and $F D$. Since $\\angle C F G=\\angle C B A=\\angle C Q A$, the quadrilateral $A Q P F$ is cyclic. Analogously, $B Q P G$ is cyclic. However, the isosceles trapezoid $B D P G$ is also cyclic; it follows that $B, Q, D, P, G$ lie on a circle. Therefore we get $$ \\angle P Q F=\\angle P A C, \\angle P Q D=\\angle P B A . $$ Since $I$ is the incenter of $\\triangle A B C$, we have $\\angle C A I=\\frac{1}{2} \\angle C A B=$ $\\frac{1}{2} \\angle C B A=\\angle I B A$; hence $C A$ is the tangent at $A$ to the circumcircle of $\\triangle A B I$. This implies that $\\angle P A C=$ $\\angle P B A$, and it follows from (1) that $\\angle P Q F=\\angle P Q D$, i.e., that $F, D, Q$ are also collinear. Similarly, $G, E, Q$ are collinear and the claim is thus proved. ![](https:\/\/cdn.mathpix.com\/cropped\/2024_11_18_8e985d6b9c83aa3e9d0eg-717.jpg?height=412&width=358&top_left_y=632&top_left_x=903)","problem_type":"Geometry","tier":0} +{"year":"2003","problem_phase":"shortlisted","problem":"18. G6 (POL) ${ }^{\\mathrm{IMO} 3}$ Each pair of opposite sides of a convex hexagon has the following property: The distance between their midpoints is equal to $\\sqrt{3} \/ 2$ times the sum of their lengths. Prove that all the angles of the hexagon are equal.","solution":"18. Let $A B C D E F$ be the given hexagon. We shall use the following lemma. Lemma. If $\\angle X Z Y \\geq 60^{\\circ}$ and if $M$ is the midpoint of $X Y$, then $M Z \\leq$ $\\frac{\\sqrt{3}}{2} X Y$, with equality if and only if $\\triangle X Y Z$ is equilateral. Proof. Let $Z^{\\prime}$ be the point such that $\\triangle X Y Z^{\\prime}$ is equilateral. Then $Z$ is inside the circle circumscribed about $\\triangle X Y Z^{\\prime}$. Consequently $M Z \\leq$ $M Z^{\\prime}=\\frac{\\sqrt{3}}{2} X Y$, with equality if and only if $Z=Z^{\\prime}$. Set $A D \\cap B E=P, B E \\cap C F=Q$, and $C F \\cap A D=R$. Suppose $\\angle A P B=$ $\\angle D P E>60^{\\circ}$, and let $K, L$ be the midpoints of the segments $A B$ and $D E$ respectively. Then by the lemma, $$ \\frac{\\sqrt{3}}{2}(A B+D E)=K L \\leq P K+P L<\\frac{\\sqrt{3}}{2}(A B+D E), $$ which is impossible. Therefore $\\angle A P B \\leq 60^{\\circ}$ and similarly $\\angle B Q C \\leq 60^{\\circ}$, $\\angle C R D \\leq 60^{\\circ}$. But the sum of the angles $A P B, B Q C, C R D$ is $180^{\\circ}$, from which we conclude that these angles are all equal to $60^{\\circ}$, and moreover that the triangles $A P B, B Q C, C R D$ are equilateral. Thus $\\angle A B C=\\angle A B P+$ $\\angle Q B C=120^{\\circ}$, and in the same way all angles of the hexagon are equal to $120^{\\circ}$.","problem_type":"Geometry","tier":0} +{"year":"2003","problem_phase":"shortlisted","problem":"19. G7 (SAF) Let $A B C$ be a triangle with semiperimeter $s$ and inradius $r$. The semicircles with diameters $B C, C A, A B$ are drawn outside of the triangle $A B C$. The circle tangent to all three semicircles has radius $t$. Prove that $$ \\frac{s}{2}\\frac{s}{2}$. Now set the coordinate system such that we have the points $D_{1}(0,0), E(-e, 0), F(f, 0)$ and such that the $y$ coordinate of $D$ is positive. ![](https:\/\/cdn.mathpix.com\/cropped\/2024_11_18_8e985d6b9c83aa3e9d0eg-718.jpg?height=528&width=529&top_left_y=233&top_left_x=831) Apply the inversion with center $D_{1}$ and unit radius. This inversion maps the circles $\\Gamma_{e}$ and $\\Gamma_{f}$ to the lines $\\widehat{\\Gamma_{e}}\\left[x=-\\frac{1}{2 e}\\right]$ and $\\widehat{\\Gamma_{e}}\\left[x=\\frac{1}{2 f}\\right]$ respectively, and the circle $\\gamma$ goes to the line $\\widehat{\\gamma}\\left[y=\\frac{1}{r}\\right]$. The images $\\widehat{\\Gamma_{d}}$ and $\\widehat{\\Gamma_{g}}$ of $\\Gamma_{d}, \\Gamma_{g}$ are the circles that touch the lines $\\widehat{\\Gamma_{e}}$ and $\\widehat{\\Gamma_{f}}$. Since $\\widehat{\\Gamma_{d}}, \\widehat{\\Gamma_{g}}$ are perpendicular to $\\gamma$, they have radii equal to $R=\\frac{1}{4 e}+\\frac{1}{4 f}$ and centers at $\\left(-\\frac{1}{4 e}+\\frac{1}{4 f}, \\frac{1}{r}\\right)$ and $\\left(-\\frac{1}{4 e}+\\frac{1}{4 f}, \\frac{1}{r}+2 R\\right)$ respectively. Let $p$ and $P$ be the distances from $D_{1}(0,0)$ to the centers of $\\Gamma_{g}$ and $\\widehat{\\Gamma_{g}}$ respectively. We have that $P^{2}=\\left(\\frac{1}{4 e}-\\frac{1}{4 f}\\right)^{2}+\\left(\\frac{1}{r}+2 R\\right)^{2}$, and that the circles $\\Gamma_{g}$ and $\\widehat{\\Gamma_{g}}$ are homothetic with center of homothety $D_{1}$; hence $p \/ P=g \/ R$. On the other hand, $\\widehat{\\Gamma_{g}}$ is the image of $\\Gamma_{g}$ under inversion; hence the product of the tangents from $D_{1}$ to these two circles is equal to 1 . In other words, we obtain $\\sqrt{p^{2}-g^{2}} \\cdot \\sqrt{P^{2}-R^{2}}=1$. Using the relation $p \/ P=g \/ R$ we get $g=\\frac{R}{P^{2}-R^{2}}$. The inequality we have to prove is equivalent to $(4+2 \\sqrt{3}) g \\leq r$. This can be proved as follows: $$ \\begin{aligned} r-(4+2 \\sqrt{3}) g & =\\frac{r\\left(P^{2}-R^{2}-(4+2 \\sqrt{3}) R \/ r\\right)}{P^{2}-R^{2}} \\\\ & =\\frac{r\\left(\\left(\\frac{1}{r}+2 R\\right)^{2}+\\left(\\frac{1}{4 e}-\\frac{1}{4 f}\\right)^{2}-R^{2}-(4+2 \\sqrt{3}) \\frac{R}{r}\\right)}{P^{2}-R^{2}} \\\\ & =\\frac{r}{P^{2}-R^{2}}\\left(\\left(R \\sqrt{3}-\\frac{1}{r}\\right)^{2}+\\left(\\frac{1}{4 e}-\\frac{1}{4 f}\\right)^{2}\\right) \\geq 0 \\end{aligned} $$ Remark. One can obtain a symmetric formula for $g$ : $$ \\frac{1}{2 g}=\\frac{1}{s-a}+\\frac{1}{s-b}+\\frac{1}{s-c}+\\frac{2}{r} $$","problem_type":"Geometry","tier":0} +{"year":"2003","problem_phase":"shortlisted","problem":"2. A2 (AUS) Find all nondecreasing functions $f: \\mathbb{R} \\rightarrow \\mathbb{R}$ such that (i) $f(0)=0, f(1)=1$; (ii) $f(a)+f(b)=f(a) f(b)+f(a+b-a b)$ for all real numbers $a, b$ such that $a<10$, and therefore (1) reduces to $g(1) g(y z)=g(y) g(z)$ for all $y, z>0$. We have two cases: (i) $g(1)=0$. By (1) we have $g(z)=0$ for all $z>0$. Then any nondecreasing function $g: \\mathbb{R} \\rightarrow \\mathbb{R}$ with $g(-1)=-1$ and $g(z)=0$ for $z \\geq 0$ satisfies (1) and gives a solution: $f$ is nondecreasing, $f(0)=0$ and $f(x)=1$ for every $x \\geq 1$ (ii) $g(1) \\neq 0$. Then the function $h(x)=\\frac{g(x)}{g(1)}$ is nondecreasing and satisfies $h(0)=0, h(1)=1$, and $h(x y)=h(x) h(y)$. Fix $a>0$, and let $h(a)=$ $b=a^{k}$ for some $k \\in \\mathbb{R}$. It follows by induction that $h\\left(a^{q}\\right)=h(a)^{q}=$ $\\left(a^{q}\\right)^{k}$ for every rational number $q$. But $h$ is nondecreasing, so $k \\geq 0$, and since the set $\\left\\{a^{q} \\mid q \\in \\mathbb{Q}\\right\\}$ is dense in $\\mathbb{R}^{+}$, we conclude that $h(x)=x^{k}$ for every $x>0$. Finally, putting $g(1)=c$, we obtain $g(x)=c x^{k}$ for all $x>0$. Then $g(-x)=-x^{k}$ for all $x>0$. This $g$ obviously satisfies (1). Hence $$ f(x)=\\left\\{\\begin{array}{ll} c(x-1)^{k}, & \\text { if } x>1 ; \\\\ 1, & \\text { if } x=1 ; \\\\ 1-(1-x)^{k}, & \\text { if } x<1 \\end{array} \\quad \\text { where } c>0 \\text { and } k \\geq 0\\right. $$","problem_type":"Algebra","tier":0} +{"year":"2003","problem_phase":"shortlisted","problem":"20. N1 (POL) Let $m$ be a fixed integer greater than 1 . The sequence $x_{0}, x_{1}, x_{2}, \\ldots$ is defined as follows: $$ x_{i}= \\begin{cases}2^{i}, & \\text { if } 0 \\leq i \\leq m-1 \\\\ \\sum_{j=1}^{m} x_{i-j}, & \\text { if } i \\geq m\\end{cases} $$ Find the greatest $k$ for which the sequence contains $k$ consecutive terms divisible by $m$.","solution":"20. Let $r_{i}$ be the remainder when $x_{i}$ is divided by $m$. Since there are at most $m^{m}$ types of $m$-consecutive blocks in the sequence $\\left(r_{i}\\right)$, some type will repeat at least twice. Then since the entire sequence is determined by one $m$-consecutive block, the entire sequence will be periodic. The formula works both forward and backward; hence using the rule $x_{i}=$ $x_{i+m}-\\sum_{j=1}^{m-1} x_{i+j}$ we can define $x_{-1}, x_{-2}, \\ldots$. Thus we obtain that $$ \\left(r_{-m}, \\ldots, r_{-1}\\right)=(0,0, \\ldots, 0,1) $$ Hence there are $m-1$ consecutive terms in the sequence $\\left(x_{i}\\right)$ that are divisible by $m$. If there were $m$ consecutive terms in the sequence $\\left(x_{i}\\right)$ divisible by $m$, then by the recurrence relation all the terms of $\\left(x_{i}\\right)$ would be divisible by $m$, which is impossible.","problem_type":"Number Theory","tier":0} +{"year":"2003","problem_phase":"shortlisted","problem":"21. N2 (USA) Each positive integer $a$ undergoes the following procedure in order to obtain the number $d=d(a)$ : (1) move the last digit of $a$ to the first position to obtain the number $b$; (2) square $b$ to obtain the number $c$; (3) move the first digit of $c$ to the end to obtain the number $d$. (All the numbers in the problem are considered to be represented in base 10.) For example, for $a=2003$, we have $b=3200, c=10240000$, and $d=02400001=2400001=d(2003)$. Find all numbers $a$ for which $d(a)=a^{2}$.","solution":"21. Let $a$ be a positive integer for which $d(a)=a^{2}$. Suppose that $a$ has $n+1$ digits, $n \\geq 0$. Denote by $s$ the last digit of $a$ and by $f$ the first digit of $c$. Then $a=\\overline{* \\ldots * s}$, where $*$ stands for a digit that is not important to us at the moment. We have $\\overline{\\ldots * s^{2}}=a^{2}=d=\\overline{* \\ldots * f}$ and $b^{2}=\\overline{s * \\ldots *}^{2}=$ $c=\\overline{f * \\ldots *}$. We cannot have $s=0$, since otherwise $c$ would have at most $2 n$ digits, while $a^{2}$ has either $2 n+1$ or $2 n+2$ digits. The following table gives all possibilities for $s$ and $f$ : | $s$ | 1 | 2 | 3 | 4 | 5 | 6 | 7 | 8 | 9 | | :---: | :---: | :---: | :---: | :---: | :---: | :---: | :---: | :---: | :---: | | $f=$ last digit of $\\overline{* \\ldots * s}^{2}$ | 1 | 4 | 9 | 6 | 5 | 6 | 9 | 4 | 1 | | $f=$ first digit of $\\overline{s * \\ldots *}^{2}$ | $1,2,3$ | $4-8$ | 9,1 | 1,2 | 2,3 | 3,4 | $4,5,6$ | $6,7,8$ | 8,9 | We obtain from the table that $s \\in\\{1,2,3\\}$ and $f=s^{2}$, and consequently $c=b^{2}$ and $d$ have exactly $2 n+1$ digits each. Put $a=10 x+s$, where $x<10^{n}$. Then $b=10^{n} s+x, c=10^{2 n} s^{2}+2 \\cdot 10^{n} s x+x^{2}$, and $d=$ $2 \\cdot 10^{n+1} s x+10 x^{2}+s^{2}$, so from $d=a^{2}$ it follows that $x=2 s \\cdot \\frac{10^{n}-1}{9}$. Thus $a=\\underbrace{6 \\ldots 6}_{n} 3, a=\\underbrace{4 \\ldots 4}_{n} 2$ or $a=\\underbrace{2 \\ldots 2}_{n} 1$. For $n \\geq 1$ we see that $a$ cannot be $a=6 \\ldots 63$ or $a=4 \\ldots 42$ (otherwise $a^{2}$ would have $2 n+2$ digits). Therefore $a$ equals $1,2,3$ or $\\underbrace{2 \\ldots 2}_{n} 1$ for $n \\geq 0$. It is easy to verify that these numbers have the required property.","problem_type":"Number Theory","tier":0} +{"year":"2003","problem_phase":"shortlisted","problem":"22. N3 (BUL) ${ }^{\\mathrm{IMO} 2}$ Determine all pairs $(a, b)$ of positive integers such that $$ \\frac{a^{2}}{2 a b^{2}-b^{3}+1} $$ is a positive integer.","solution":"22. Let $a$ and $b$ be positive integers for which $\\frac{a^{2}}{2 a b^{2}-b^{3}+1}=k$ is a positive integer. Since $k>0$, it follows that $2 a b^{2} \\geq b^{3}$, so $2 a \\geq b$. If $2 a>b$, then from $2 a b^{2}-b^{3}+1>0$ we see that $a^{2}>b^{2}(2 a-b)+1>b^{2}$, i.e. $a>b$. Therefore, if $a \\leq b$, then $a=b \/ 2$. We can rewrite the given equation as a quadratic equation in $a, a^{2}-$ $2 k b^{2} a+k\\left(b^{3}-1\\right)=0$, which has two solutions, say $a_{1}$ and $a_{2}$, one of which is in $\\mathbb{N}_{0}$. From $a_{1}+a_{2}=2 k b^{2}$ and $a_{1} a_{2}=k\\left(b^{3}-1\\right)$ it follows that the other solution is also in $\\mathbb{N}_{0}$. Suppose w.l.o.g. that $a_{1} \\geq a_{2}$. Then $a_{1} \\geq k b^{2}$ and $$ 0 \\leq a_{2}=\\frac{k\\left(b^{3}-1\\right)}{a_{1}} \\leq \\frac{k\\left(b^{3}-1\\right)}{k b^{2}}M$. Also, straightforward calculation implies $$ \\left(b^{2 n+1}+\\frac{b^{n+2}+b^{n+1}}{2}-b^{3}\\right)^{2}M$ there is an integer $a_{n}$ such that $\\left|a_{n}\\right|3$ we get $a_{n}= \\pm(3 b-5)$. If $a_{n}=3 b-5$, then substituting in (1) yields $\\frac{1}{4} b^{2 n}\\left(b^{4}-14 b^{3}+45 b^{2}-\\right.$ $52 b+20)=0$, with the unique positive integer solution $b=10$. Also, if $a_{n}=-3 b+5$, we similarly obtain $\\frac{1}{4} b^{2 n}\\left(b^{4}-14 b^{3}-3 b^{2}+28 b+20\\right)-$ $2 b^{n+1}\\left(3 b^{2}-2 b-5\\right)=0$ for each $n$, which is impossible. For $b=10$ it is easy to show that $x_{n}=\\left(\\frac{10^{n}+5}{3}\\right)^{2}$ for all $n$. This proves the statement. Second solution. In problems of this type, computing $z_{n}=\\sqrt{x_{n}}$ asymptotically usually works. From $\\lim _{n \\rightarrow \\infty} \\frac{b^{2 n}}{(b-1) x_{n}}=1$ we infer that $\\lim _{n \\rightarrow \\infty} \\frac{b^{n}}{z_{n}}=\\sqrt{b-1}$. Furthermore, from $\\left(b z_{n}+z_{n+1}\\right)\\left(b z_{n}-z_{n+1}\\right)=b^{2} x_{n}-x_{n+1}=b^{n+2}+3 b^{2}-2 b-5$ we obtain $$ \\lim _{n \\rightarrow \\infty}\\left(b z_{n}-z_{n+1}\\right)=\\frac{b \\sqrt{b-1}}{2} $$ Since the $z_{n}$ 's are integers for all $n \\geq M$, we conclude that $b z_{n}-z_{n+1}=$ $\\frac{b \\sqrt{b-1}}{2}$ for all $n$ sufficiently large. Hence $b-1$ is a perfect square, and moreover $b$ divides $2 z_{n+1}$ for all large $n$. It follows that $b \\mid 10$; hence the only possibility is $b=10$.","problem_type":"Number Theory","tier":0} +{"year":"2003","problem_phase":"shortlisted","problem":"24. N5 (KOR) An integer $n$ is said to be good if $|n|$ is not the square of an integer. Determine all integers $m$ with the following property: $m$ can be represented in infinitely many ways as a sum of three distinct good integers whose product is the square of an odd integer.","solution":"24. Suppose that $m=u+v+w$ where $u, v, w$ are good integers whose product is a perfect square of an odd integer. Since $u v w$ is an odd perfect square, we have that $u v w \\equiv 1(\\bmod 4)$. Thus either two or none of the numbers $u, v, w$ are congruent to 3 modulo 4 . In both cases $u+v+w \\equiv 3(\\bmod 4)$. Hence $m \\equiv 3(\\bmod 4)$. Now we shall prove the converse: every $m \\equiv 3(\\bmod 4)$ has infinitely many representations of the desired type. Let $m=4 k+3$. We shall represent $m$ in the form $$ 4 k+3=x y+y z+z x, \\quad \\text { for } x, y, z \\text { odd. } $$ The product of the summands is an odd square. Set $x=1+2 l$ and $y=1-2 l$. In order to satisfy (1), $z$ must satisfy $z=2 l^{2}+2 k+1$. The summands $x y, y z, z x$ are distinct except for finitely many $l$, so it remains only to prove that for infinitely many integers $l,|x y|,|y z|$, and $|z x|$ are not perfect squares. First, observe that $|x y|=4 l^{2}-1$ is not a perfect square for any $l \\neq 0$. Let $p, q>m$ be fixed different prime numbers. The system of congruences $1+2 l \\equiv p\\left(\\bmod p^{2}\\right)$ and $1-2 l \\equiv q\\left(\\bmod q^{2}\\right)$ has infinitely many solutions $l$ by the Chinese remainder theorem. For any such $l$, the number $z=$ $2 l^{2}+2 k+1$ is divisible by neither $p$ nor $q$, and hence $|x z|$ (respectively $|y z|)$ is divisible by $p$, but not by $p^{2}$ (respectively by $q$, but not by $q^{2}$ ). Thus $x z$ and $y z$ are also good numbers.","problem_type":"Number Theory","tier":0} +{"year":"2003","problem_phase":"shortlisted","problem":"25. N6 (FRA) ${ }^{\\text {IMO6 }}$ Let $p$ be a prime number. Prove that there exists a prime number $q$ such that for every integer $n$, the number $n^{p}-p$ is not divisible by $q$.","solution":"25. Suppose that for every prime $q$, there exists an $n$ for which $n^{p} \\equiv p(\\bmod$ $q$ ). Assume that $q=k p+1$. By Fermat's theorem we deduce that $p^{k} \\equiv$ $n^{k p}=n^{q-1} \\equiv 1(\\bmod q)$, so $q \\mid p^{k}-1$. It is known that any prime $q$ such that $q \\left\\lvert\\, \\frac{p^{p}-1}{p-1}\\right.$ must satisfy $q \\equiv 1(\\bmod$ $p$ ). Indeed, from $q \\mid p^{q-1}-1$ it follows that $q \\mid p^{\\operatorname{gcd}(p, q-1)}-1$; but $q \\nmid p-1$ because $\\frac{p^{p}-1}{p-1} \\equiv 1(\\bmod p-1)$, so $\\operatorname{gcd}(p, q-1) \\neq 1$. Hence $\\operatorname{gcd}(p, q-1)=p$. Now suppose $q$ is any prime divisor of $\\frac{p^{p}-1}{p-1}$. Then $q \\mid \\operatorname{gcd}\\left(p^{k}-1, p^{p}-1\\right)=$ $p^{\\operatorname{gcd}(p, k)}-1$, which implies that $\\operatorname{gcd}(p, k)>1$, so $p \\mid k$. Consequently $q \\equiv 1$ $\\left(\\bmod p^{2}\\right)$. However, the number $\\frac{p^{p}-1}{p-1}=p^{p-1}+\\cdots+p+1$ must have at least one prime divisor that is not congruent to 1 modulo $p^{2}$. Thus we arrived at a contradiction. Remark. Taking $q \\equiv 1(\\bmod p)$ is natural, because for every other $q, n^{p}$ takes all possible residues modulo $q$ (including $p$ too). Indeed, if $p \\nmid q-1$, then there is an $r \\in \\mathbb{N}$ satisfying $p r \\equiv 1(\\bmod q-1)$; hence for any $a$ the congruence $n^{p} \\equiv a(\\bmod q)$ has the solution $n \\equiv a^{r}(\\bmod q)$. The statement of the problem itself is a special case of the Chebotarev's theorem.","problem_type":"Number Theory","tier":0} +{"year":"2003","problem_phase":"shortlisted","problem":"26. N7 (BRA) The sequence $a_{0}, a_{1}, a_{2}, \\ldots$ is defined as follows: $$ a_{0}=2, \\quad a_{k+1}=2 a_{k}^{2}-1 \\quad \\text { for } k \\geq 0 $$ Prove that if an odd prime $p$ divides $a_{n}$, then $2^{n+3}$ divides $p^{2}-1$.","solution":"26. Define the sequence $x_{k}$ of positive reals by $a_{k}=\\cosh x_{k}$ ( $\\cosh$ is the hyperbolic cosine defined by $\\left.\\cosh t=\\frac{e^{t}+e^{-t}}{2}\\right)$. Since $\\cosh \\left(2 x_{k}\\right)=2 a_{k}^{2}-1=$ $\\cosh x_{k+1}$, it follows that $x_{k+1}=2 x_{k}$ and thus $x_{k}=\\lambda \\cdot 2^{k}$ for some $\\lambda>0$. From the condition $a_{0}=2$ we obtain $\\lambda=\\log (2+\\sqrt{3})$. Therefore $$ a_{n}=\\frac{(2+\\sqrt{3})^{2^{n}}+(2-\\sqrt{3})^{2^{n}}}{2} $$ Let $p$ be a prime number such that $p \\mid a_{n}$. We distinguish the following two cases: (i) There exists an $m \\in \\mathbb{Z}$ such that $m^{2} \\equiv 3(\\bmod p)$. Then we have $$ (2+m)^{2^{n}}+(2-m)^{2^{n}} \\equiv 0(\\bmod p) $$ Since $(2+m)(2-m)=4-m^{2} \\equiv 1(\\bmod p)$, multiplying both sides of $(1)$ by $(2+m)^{2^{n}}$ gives $(2+m)^{2^{n+1}} \\equiv-1(\\bmod p)$. It follows that the multiplicative order of $(2+m)$ modulo $p$ is $2^{n+2}$, or $2^{n+2} \\mid p-1$, which implies that $2^{n+3} \\mid(p-1)(p+1)=p^{2}-1$. (ii) $m^{2} \\equiv 3(\\bmod p)$ has no integer solutions. We will work in the algebraic extension $\\mathbb{Z}_{p}(\\sqrt{3})$ of the field $\\mathbb{Z}_{p}$. In this field $\\sqrt{3}$ plays the role of $m$, so as in the previous case we obtain $(2+\\sqrt{3})^{2^{n+1}}=-1$; i.e., the order of $2+\\sqrt{3}$ in the multiplicative group $\\mathbb{Z}_{p}(\\sqrt{3})^{*}$ is $2^{n+2}$. We cannot finish the proof as in the previous case: in fact, we would conclude only that $2^{n+2}$ divides the order $p^{2}-1$ of the group. However, it will be enough to find a $u \\in \\mathbb{Z}_{p}(\\sqrt{3})$ such that $u^{2}=2+\\sqrt{3}$, since then the order of $u$ is equal to $2^{n+3}$. Note that $(1+\\sqrt{3})^{2}=2(2+\\sqrt{3})$. Thus it is sufficient to prove that $\\frac{1}{2}$ is a perfect square in $\\mathbb{Z}_{p}(\\sqrt{3})$. But we know that in this field $a_{n}=$ $0=2 a_{n-1}^{2}-1$, and hence $2 a_{n-1}^{2}=1$ which implies $\\frac{1}{2}=a_{n-1}^{2}$. This completes the proof.","problem_type":"Number Theory","tier":0} +{"year":"2003","problem_phase":"shortlisted","problem":"27. N8 (IRN) Let $p$ be a prime number and let $A$ be a set of positive integers that satisfies the following conditions: (i) the set of prime divisors of the elements in $A$ consists of $p-1$ elements; (ii) for any nonempty subset of $A$, the product of its elements is not a perfect $p$ th power. What is the largest possible number of elements in $A$ ?","solution":"27. Let $p_{1}, p_{2}, \\ldots, p_{r}$ be distinct primes, where $r=p-1$. Consider the sets $B_{i}=\\left\\{p_{i}, p_{i}^{p+1}, \\ldots, p_{i}^{(r-1) p+1}\\right\\}$ and $B=\\bigcup_{i=1}^{r} B_{i}$. Then $B$ has $(p-1)^{2}$ elements and satisfies (i) and (ii). Now suppose that $|A| \\geq r^{2}+1$ and that $A$ satisfies (i) and (ii), and let $\\left\\{t_{1}, \\ldots, t_{r^{2}+1}\\right\\}$ be distinct elements of $A$, where $t_{j}=p_{1}^{\\alpha_{j_{1}}} \\cdot p_{2}^{\\alpha_{j_{2}}} \\cdots p_{r}^{\\alpha_{j_{r}}}$. We shall show that the product of some elements of $A$ is a perfect $p$ th power, i.e., that there exist $\\tau_{j} \\in\\{0,1\\}\\left(1 \\leq j \\leq r^{2}+1\\right)$, not all equal to 0 , such that $T=t_{1}^{\\tau_{1}} \\cdot t_{2}^{\\tau_{2}} \\cdots t_{r^{2}+1}^{\\tau_{r^{2}+1}}$ is a $p$ th power. This is equivalent to the condition that $$ \\sum_{j=1}^{r^{2}+1} \\alpha_{i j} \\tau_{j} \\equiv 0(\\bmod p) $$ holds for all $i=1, \\ldots, r$. By Fermat's theorem it is sufficient to find integers $x_{1}, \\ldots, x_{r^{2}+1}$, not all zero, such that the relation $$ \\sum_{j=1}^{r^{2}+1} \\alpha_{i j} x_{j}^{r} \\equiv 0(\\bmod p) $$ is satisfied for all $i \\in\\{1, \\ldots, r\\}$. Set $F_{i}=\\sum_{j=1}^{r^{2}+1} \\alpha_{i j} x_{j}^{r}$. We want to find $x_{1}, \\ldots, x_{r}$ such that $F_{1} \\equiv F_{2} \\equiv \\cdots \\equiv F_{r} \\equiv 0(\\bmod p)$, which is by Fermat's theorem equivalent to $$ F\\left(x_{1}, \\ldots, x_{r}\\right)=F_{1}^{r}+F_{2}^{r}+\\cdots+F_{r}^{r} \\equiv 0(\\bmod p) . $$ Of course, one solution of (1) is $(0, \\ldots, 0)$ : we are not satisfied with it because it generates the empty subset of $A$, but it tells us that (1) has at least one solution. We shall prove that the number of solutions of (1) is divisible by $p$, which will imply the existence of a nontrivial solution and thus complete the proof. To do this, consider the sum $\\sum F\\left(x_{1}, \\ldots, x_{r^{2}+1}\\right)^{r}$ taken over all vectors $\\left(x_{1}, \\ldots, x_{r^{2}+1}\\right)$ in the vector space $\\mathbb{Z}_{p}^{r^{2}+1}$. Our statement is equivalent to $$ \\sum F\\left(x_{1}, \\ldots, x_{r^{2}+1}\\right)^{r} \\equiv 0(\\bmod p) $$ Since the degree of $F^{r}$ is $r^{2}$, in each monomial in $F^{r}$ at least one of the variables is missing. Consider any of these monomials, say $b x_{i_{1}}^{a_{1}} x_{i_{2}}^{a_{2}} \\cdots x_{i_{k}}^{a_{k}}$. Then the sum $\\sum b x_{i_{1}}^{a_{1}} x_{i_{2}}^{a_{2}} \\cdots x_{i_{k}}^{a_{k}}$, taken over the set of all vectors $\\left(x_{1}, \\ldots, x_{r^{2}+1}\\right) \\in \\mathbb{Z}_{p}^{r^{2}+1}$, is equal to $$ p^{r^{2}+1-u} \\cdot \\sum_{\\left(x_{i_{1}}, \\ldots, x_{i_{k}}\\right) \\in \\mathbb{Z}_{p}^{k}} b x_{i_{1}}^{a_{1}} x_{i_{2}}^{a_{2}} \\cdots x_{i_{k}}^{a_{k}}, $$ which is divisible by $p$, so that (2) is proved. Thus the answer is $(p-1)^{2}$.","problem_type":"Number Theory","tier":0} +{"year":"2003","problem_phase":"shortlisted","problem":"3. A3 (GEO) Consider pairs of sequences of positive real numbers $a_{1} \\geq$ $a_{2} \\geq a_{3} \\geq \\cdots, b_{1} \\geq b_{2} \\geq b_{3} \\geq \\cdots$ and the sums $A_{n}=a_{1}+\\cdots+a_{n}$, $B_{n}=b_{1}+\\cdots+b_{n}, n=1,2, \\ldots$. For any pair define $c_{i}=\\min \\left\\{a_{i}, b_{i}\\right\\}$ and $C_{n}=c_{1}+\\cdots+c_{n}, n=1,2, \\ldots$ (a) Does there exist a pair $\\left(a_{i}\\right)_{i \\geq 1},\\left(b_{i}\\right)_{i \\geq 1}$ such that the sequences $\\left(A_{n}\\right)_{n \\geq 1}$ and $\\left(B_{n}\\right)_{n \\geq 1}$ are unbounded while the sequence $\\left(C_{n}\\right)_{n \\geq 1}$ is bounded? (b) Does the answer to question (1) change by assuming additionally that $b_{i}=1 \/ i, i=1,2, \\ldots ?$ Justify your answer.","solution":"3. (a) Given any sequence $c_{n}$ (in particular, such that $C_{n}$ converges), we shall construct $a_{n}$ and $b_{n}$ such that $A_{n}$ and $B_{n}$ diverge. First, choose $n_{1}$ such that $n_{1} c_{1}>1$ and set $a_{1}=a_{2}=\\cdots=a_{n_{1}}=$ $c_{1}$ : this uniquely determines $b_{2}=c_{2}, \\ldots, b_{n_{1}}=c_{n_{1}}$. Next, choose $n_{2}$ such that $\\left(n_{2}-n_{1}\\right) c_{n_{1}+1}>1$ and set $b_{n_{1}+1}=\\cdots=b_{n_{2}}=c_{n_{1}+1}$; again $a_{n_{1}+1}, \\ldots, a_{n_{2}}$ is hereby determined. Then choose $n_{3}$ with $\\left(n_{3}-\\right.$ $\\left.n_{2}\\right) c_{n_{2}+1}>1$ and set $a_{n_{2}+1}=\\cdots=a_{n_{3}}=c_{n_{2}+1}$, and so on. It is plain that in this way we construct decreasing sequences $a_{n}, b_{n}$ such that $\\sum a_{n}$ and $\\sum b_{n}$ diverge, since they contain an infinity of subsums that exceed 1 ; on the other hand, $c_{n}=\\min \\left(a_{n}, b_{n}\\right)$ and $C_{n}$ is convergent. (b) The answer changes in this situation. Suppose to the contrary that there is such a pair of sequences $\\left(a_{n}\\right)$ and $\\left(b_{n}\\right)$. There are infinitely many indices $i$ such that $c_{i}=b_{i}$ (otherwise all but finitely many terms of the sequence $\\left(c_{n}\\right)$ would be equal to the terms of the sequence $\\left(a_{n}\\right)$, which has an unbounded sum). Thus for any $n_{0} \\in \\mathbb{N}$ there is $j \\geq 2 n_{0}$ such that $c_{j}=b_{j}$. Then we have $$ \\sum_{k=n_{0}}^{j} c_{k} \\geq \\sum_{k=n_{0}}^{j} c_{j}=\\left(j-n_{0}\\right) \\frac{1}{j} \\geq \\frac{1}{2} $$ Hence the sequence ( $C_{n}$ ) is unbounded, a contradiction.","problem_type":"Algebra","tier":0} +{"year":"2003","problem_phase":"shortlisted","problem":"4. A4 (IRE) ${ }^{\\mathrm{IMO} 5}$ Let $n$ be a positive integer and let $x_{1} \\leq x_{2} \\leq \\cdots \\leq x_{n}$ be real numbers. (a) Prove that $$ \\left(\\sum_{i, j=1}^{n}\\left|x_{i}-x_{j}\\right|\\right)^{2} \\leq \\frac{2\\left(n^{2}-1\\right)}{3} \\sum_{i, j=1}^{n}\\left(x_{i}-x_{j}\\right)^{2} $$ (b) Show that equality holds if and only if $x_{1}, \\ldots, x_{n}$ is an arithmetic progession.","solution":"4. By the Cauchy-Schwarz inequality we have $$ \\left(\\sum_{i, j=1}^{n}(i-j)^{2}\\right)\\left(\\sum_{i, j=1}^{n}\\left(x_{i}-x_{j}\\right)^{2}\\right) \\geq\\left(\\sum_{i, j=1}^{n}|i-j| \\cdot\\left|x_{i}-x_{j}\\right|\\right)^{2} . $$ On the other hand, it is easy to prove (for example by induction) that $$ \\sum_{i, j=1}^{n}(i-j)^{2}=(2 n-2) \\cdot 1^{2}+(2 n-4) \\cdot 2^{2}+\\cdots+2 \\cdot(n-1)^{2}=\\frac{n^{2}\\left(n^{2}-1\\right)}{6} $$ and that $$ \\sum_{i, j=1}^{n}|i-j| \\cdot\\left|x_{i}-x_{j}\\right|=\\frac{n}{2} \\sum_{i, j=1}^{n}\\left|x_{i}-x_{j}\\right| $$ Thus the inequality (1) becomes $$ \\frac{n^{2}\\left(n^{2}-1\\right)}{6}\\left(\\sum_{i, j=1}^{n}\\left(x_{i}-x_{j}\\right)^{2}\\right) \\geq \\frac{n^{2}}{4}\\left(\\sum_{i, j=1}^{n}\\left|x_{i}-x_{j}\\right|\\right)^{2} $$ which is equivalent to the required one.","problem_type":"Algebra","tier":0} +{"year":"2003","problem_phase":"shortlisted","problem":"5. A5 (KOR) Let $\\mathbb{R}^{+}$be the set of all positive real numbers. Find all functions $f: \\mathbb{R}^{+} \\rightarrow \\mathbb{R}^{+}$that satisfy the following conditions: (i) $f(x y z)+f(x)+f(y)+f(z)=f(\\sqrt{x y}) f(\\sqrt{y z}) f(\\sqrt{z x})$ for all $x, y, z \\in$ $\\mathbb{R}^{+}$. (ii) $f(x)0$ we get $f(1)=2$. Also putting $x=t s, y=\\frac{t}{s}, z=\\frac{s}{t}$ in (i) gives $$ f(t) f(s)=f(t s)+f(t \/ s) $$ In particular, for $s=1$ the last equality yields $f(t)=f(1 \/ t)$; hence $f(t) \\geq f(1)=2$ for each $t$. It follows that there exists $g(t) \\geq 1$ such that $\\bar{f}(t)=g(t)+\\frac{1}{g(t)}$. Now it follows by induction from (1) that $g\\left(t^{n}\\right)=$ $g(t)^{n}$ for every integer $n$, and therefore $g\\left(t^{q}\\right)=g(t)^{q}$ for every rational $q$. Consequently, if $t>1$ is fixed, we have $f\\left(t^{q}\\right)=a^{q}+a^{-q}$, where $a=g(t)$. But since the set of $a^{q}(q \\in \\mathbb{Q})$ is dense in $\\mathbb{R}^{+}$and $f$ is monotone on $(0,1]$ and $[1, \\infty)$, it follows that $f\\left(t^{r}\\right)=a^{r}+a^{-r}$ for every real $r$. Therefore, if $k$ is such that $t^{k}=a$, we have $$ f(x)=x^{k}+x^{-k} \\quad \\text { for every } x \\in \\mathbb{R} $$","problem_type":"Algebra","tier":0} +{"year":"2003","problem_phase":"shortlisted","problem":"6. A6 (USA) Let $n$ be a positive integer and let $\\left(x_{1}, \\ldots, x_{n}\\right),\\left(y_{1}, \\ldots, y_{n}\\right)$ be two sequences of positive real numbers. Suppose $\\left(z_{2}, z_{3}, \\ldots, z_{2 n}\\right)$ is a sequence of positive real numbers such that $$ z_{i+j}^{2} \\geq x_{i} y_{j} \\quad \\text { for all } 1 \\leq i, j \\leq n $$ Let $M=\\max \\left\\{z_{2}, \\ldots, z_{2 n}\\right\\}$. Prove that $$ \\left(\\frac{M+z_{2}+\\cdots+z_{2 n}}{2 n}\\right)^{2} \\geq\\left(\\frac{x_{1}+\\cdots+x_{n}}{n}\\right)\\left(\\frac{y_{1}+\\cdots+y_{n}}{n}\\right) . $$","solution":"6. Set $X=\\max \\left\\{x_{1}, \\ldots, x_{n}\\right\\}$ and $Y=\\max \\left\\{y_{1}, \\ldots, y_{n}\\right\\}$. By replacing $x_{i}$ by $x_{i}^{\\prime}=\\frac{x_{i}}{X}, y_{i}$ by $y_{i}^{\\prime}=\\frac{y_{i}}{Y}$ and $z_{i}$ by $z_{i}^{\\prime}=\\frac{z_{i}}{\\sqrt{X Y}}$, we may assume that $X=Y=1$. It is sufficient to prove that $$ M+z_{2}+\\cdots+z_{2 n} \\geq x_{1}+\\cdots+x_{n}+y_{1}+\\cdots+y_{n} $$ because this implies the result by the A-G mean inequality. To prove (1) it is enough to prove that for any $r$, the number of terms greater than $r$ on the left-hand side of (1) is at least that number on the right-hand side of (1). If $r \\geq 1$, then there are no terms on the right-hand side greater than $r$. Suppose that $r<1$ and consider the sets $A=\\left\\{i \\mid 1 \\leq i \\leq n, x_{i}>r\\right\\}$ and $B=\\left\\{i \\mid 1 \\leq i \\leq n, y_{i}>r\\right\\}$. Set $a=|A|$ and $b=|B|$. If $x_{i}>r$ and $y_{j}>r$, then $z_{i+j} \\geq \\sqrt{x_{i} y_{j}}>r$; hence $$ C=\\left\\{k \\mid 2 \\leq k \\leq 2 n, z_{k}>r\\right\\} \\supseteq A+B=\\{\\alpha+\\beta \\mid \\alpha \\in A, \\beta \\in B\\} $$ It is easy to verify that $|A+B| \\geq|A|+|B|-1$. It follows that the number of $z_{k}$ 's greater than $r$ is $\\geq a+b-1$. But in that case $M>r$, implying that at least $a+b$ elements of the left-hand side of (1) is greater than $r$, which completes the proof.","problem_type":"Algebra","tier":0} +{"year":"2003","problem_phase":"shortlisted","problem":"7. C1 (BRA) ${ }^{\\mathrm{IMO} 1}$ Let $A$ be a 101-element subset of the set $S=\\{1,2, \\ldots$, $1000000\\}$. Prove that there exist numbers $t_{1}, t_{2}, \\ldots, t_{100}$ in $S$ such that the sets $$ A_{j}=\\left\\{x+t_{j} \\mid x \\in A\\right\\}, \\quad j=1,2, \\ldots, 100 $$ are pairwise disjoint.","solution":"7. Consider the set $D=\\{x-y \\mid x, y \\in A\\}$. Obviously, the number of elements of the set $D$ is less than or equal to $101 \\cdot 100+1$. The sets $A+t_{i}$ and $A+t_{j}$ are disjoint if and only if $t_{i}-t_{j} \\notin D$. Now we shall choose inductively 100 elements $t_{1}, \\ldots, t_{100}$. Let $t_{1}$ be any element of the set $S \\backslash D$ (such an element exists, since the number of elements of $S$ is greater than the number of elements of $D$ ). Suppose now that we have chosen $k(k \\leq 99)$ elements $t_{1}, \\ldots, t_{k}$ from $D$ such that the difference of any two of the chosen elements does not belong to $D$. We can select $t_{k+1}$ to be an element of $S$ that does not belong to any of the sets $t_{1}+D, t_{2}+D, \\ldots, t_{k}+D$ (this is possible to do, since each of the previous sets has at most $101 \\cdot 100+1$ elements; hence their union has at most $99(101 \\cdot 100+1)=999999<1000000$ elements $)$.","problem_type":"Combinatorics","tier":0} +{"year":"2003","problem_phase":"shortlisted","problem":"8. C2 (GEO) Let $D_{1}, \\ldots, D_{n}$ be closed disks in the plane. (A closed disk is a region bounded by a circle, taken jointly with this circle.) Suppose that every point in the plane is contained in at most 2003 disks $D_{i}$. Prove that there exists disk $D_{k}$ that intersects at most $7 \\cdot 2003-1$ other disks $D_{i}$.","solution":"8. Let $S$ be the disk with the smallest radius, say $s$, and $O$ the center of that disk. Divide the plane into 7 regions: one bounded by disk $s$ and 6 regions $T_{1}, \\ldots, T_{6}$ shown in the figure. Any of the disks different from $S$, say $D_{k}$, has its center in one of the seven regions. If its center is inside $S$ then $D_{k}$ contains point $O$. Hence the number of disks different from $S$ having their centers in $S$ is at most 2002. Consider a disk $D_{k}$ that intersects $S$ and whose center is in the region $T_{i}$. Let $P_{i}$ be the point such that $O P_{i}$ bisects the region $T_{i}$ and ![](https:\/\/cdn.mathpix.com\/cropped\/2024_11_18_8e985d6b9c83aa3e9d0eg-713.jpg?height=453&width=503&top_left_y=799&top_left_x=828) $O P_{i}=s \\sqrt{3}$. We claim that $D_{k}$ contains $P_{i}$. Divide the region $T_{i}$ by a line $l_{i}$ through $P_{i}$ perpendicular to $O P_{i}$ into two regions $U_{i}$ and $V_{i}$, where $O$ and $U_{i}$ are on the same side of $l_{i}$. Let $K$ be the center of $D_{k}$. Consider two cases: (i) $K \\in U_{i}$. Since the disk with the center $P_{i}$ and radius $s$ contains $U_{i}$, we see that $K P_{i} \\leq s$. Hence $D_{k}$ contains $P_{i}$. (ii) $K \\in V_{i}$. Denote by $L$ the intersection point of the segment $K O$ with the circle $s$. We want to prove that $K L>K P_{i}$. It is enough to prove that $\\angle K P_{i} L>\\angle K L P_{i}$. However, it is obvious that $\\angle L P_{i} O \\leq 30^{\\circ}$ and $\\angle L O P_{i} \\leq 30^{\\circ}$, hence $\\angle K L P_{i} \\leq 60^{\\circ}$, while $\\angle N P_{i} L=90^{\\circ}-\\angle L P_{i} O \\geq$ $60^{\\circ}$. This implies that $\\angle K P_{i} L \\geq \\angle N P_{i} L \\geq 60^{\\circ} \\geq \\angle K L P_{i}$ ( $N$ is the point on the edge of $T_{i}$ as shown in the figure). Our claim is thus proved. Now we see that the number of disks with centers in $T_{i}$ that intersect $S$ is less than or equal to 2003, and the total number of disks that intersect $S$ is not greater than $2002+6 \\cdot 2003=7 \\cdot 2003-1$.","problem_type":"Combinatorics","tier":0} +{"year":"2003","problem_phase":"shortlisted","problem":"9. C3 (LIT) Let $n \\geq 5$ be a given integer. Determine the largest integer $k$ for which there exists a polygon with $n$ vertices (convex or not, with non-self-intersecting boundary) having $k$ internal right angles.","solution":"9. Suppose that $k$ of the angles of an $n$-gon are right. Since the other $n-k$ angles are less than $360^{\\circ}$ and the sum of the angles is $(n-2) 180^{\\circ}$, we have the inequality $k \\cdot 90^{\\circ}+(n-k) 360^{\\circ}>(n-2) 180^{\\circ}$, which is equivalent to $k<\\frac{2 n+4}{3}$. Since $n$ and $k$ are integers, it follows that $k \\leq\\left[\\frac{2 n}{3}\\right]+1$. If $n=5$, then $\\left[\\frac{2 n}{3}\\right]+1=4$, but if a pentagon has four right angles, the other angle is equal to $180^{\\circ}$, which is impossible. Hence for $n=5$, $k \\leq 3$. It is easy to construct a pentagon with 3 right angles, e.g., as in the picture below. Now we shall show by induction that for $n \\geq 6$ there is an $n$-gon with $\\left[\\frac{2 n}{3}\\right]+1$ internal right angles. For $n=6,7,8$ examples are presented in the picture. Assume that there is a $(n-3)$ gon with $\\left[\\frac{2(n-3)}{3}\\right]+1=\\left[\\frac{2 n}{3}\\right]-1$ internal right angles. Then one of the internal angles, say $\\angle B A C$, is not convex. Interchange the vertex $A$ with four new vertices $A_{1}, A_{2}, A_{3}, A_{4}$ as shown in the picture such that $\\angle B A_{1} A_{2}=\\angle A_{3} A_{4} C=90^{\\circ}$. ![](https:\/\/cdn.mathpix.com\/cropped\/2024_11_18_8e985d6b9c83aa3e9d0eg-714.jpg?height=195&width=1024&top_left_y=781&top_left_x=292)","problem_type":"Combinatorics","tier":0} +{"year":"2004","problem_phase":"shortlisted","problem":"1. A1 (KOR) ${ }^{\\mathrm{IMO} 4}$ Let $n \\geq 3$ be an integer and $t_{1}, t_{2}, \\ldots, t_{n}$ positive real numbers such that $$ n^{2}+1>\\left(t_{1}+t_{2}+\\cdots+t_{n}\\right)\\left(\\frac{1}{t_{1}}+\\frac{1}{t_{2}}+\\cdots+\\frac{1}{t_{n}}\\right) . $$ Show that $t_{i}, t_{j}, t_{k}$ are the side lengths of a triangle for all $i, j, k$ with $1 \\leq it_{3}$. We have $$ \\left(\\sum_{i=1}^{n} t_{i}\\right)\\left(\\sum_{i=1}^{n} \\frac{1}{t}_{i}\\right)=n^{2}+\\sum_{in \/ 2$, and 0 otherwise. This matrix satisfies the conditions from the problem and all row sums and column sums are equal to $\\pm n \/ 2$. Hence $C \\geq n \/ 2$. Let us show that $C=n \/ 2$. Assume to the contrary that there is a matrix $B=\\left(b_{i j}\\right)_{i, j=1}^{n}$ all of whose row sums and column sums are either greater than $n \/ 2$ or smaller than $-n \/ 2$. We may assume w.l.o.g. that at least $n \/ 2$ row sums are positive and, permuting rows if necessary, that the first $n \/ 2$ rows have positive sums. The sum of entries in the $n \/ 2 \\times n$ submatrix $B^{\\prime}$ consisting of first $n \/ 2$ rows is greater than $n^{2} \/ 4$, and since each column of $B^{\\prime}$ has sum at most $n \/ 2$, it follows that more than $n \/ 2$ column sums of $B^{\\prime}$, and therefore also of $B$, are positive. Again, suppose w.l.o.g. that the first $n \/ 2$ column sums are positive. Thus the sums $R^{+}$and $C^{+}$of entries in the first $n \/ 2$ rows and in the first $n \/ 2$ columns respectively are greater than $n^{2} \/ 4$. Now the sum of all entries of $B$ can be written as $$ \\sum a_{i j}=R^{+}+C^{+}+\\sum_{\\substack{i>n \/ 2 \\\\ j>n \/ 2}} a_{i j}-\\sum_{\\substack{i \\leq n \/ 2 \\\\ j \\leq n \/ 2}} a_{i j}>\\frac{n^{2}}{2}-\\frac{n^{2}}{4}-\\frac{n^{2}}{4}=0 $$ a contradiction. Hence $C=n \/ 2$, as claimed.","problem_type":"Combinatorics","tier":0} +{"year":"2004","problem_phase":"shortlisted","problem":"12. C5 (NZL) Let $N$ be a positive integer. Two players $A$ and $B$, taking turns, write numbers from the set $\\{1, \\ldots, N\\}$ on a blackboard. $A$ begins the game by writing 1 on his first move. Then, if a player has written $n$ on a certain move, his adversary is allowed to write $n+1$ or $2 n$ (provided the number he writes does not exceed $N$ ). The player who writes $N$ wins. We say that $N$ is of type $A$ or of type $B$ according as $A$ or $B$ has a winning strategy. (a) Determine whether $N=2004$ is of type $A$ or of type $B$. (b) Find the least $N>2004$ whose type is different from that of 2004.","solution":"12. We say that a number $n \\in\\{1,2, \\ldots, N\\}$ is winning if the player who is on turn has a winning strategy, and losing otherwise. The game is of type $A$ if and only if 1 is a losing number. Let us define $n_{0}=N, n_{i+1}=\\left[n_{i} \/ 2\\right]$ for $i=0,1, \\ldots$ and let $k$ be such that $n_{k}=1$. Consider the sets $A_{i}=\\left\\{n_{i+1}+1, \\ldots, n_{i}\\right\\}$. We call a set $A_{i}$ all-winning if all numbers from $A_{i}$ are winning, even-winning if even numbers are winning and odd are losing, and odd-winning if odd numbers are winning and even are losing. (i) Suppose $A_{i}$ is even-winning and consider $A_{i+1}$. Multiplying any number from $A_{i+1}$ by 2 yields an even number from $A_{i}$, which is a losing number. Thus $x \\in A_{i+1}$ is winning if and only if $x+1$ is losing, i.e., if and only if it is even. Hence $A_{i+1}$ is also even-winning. (ii) Suppose $A_{i}$ is odd-winning. Then each $k \\in A_{i+1}$ is winning, since $2 k$ is losing. Hence $A_{i+1}$ is all-winning. (iii) Suppose $A_{i}$ is all-winning. Multiplying $x \\in A_{i+1}$ by two is then a losing move, so $x$ is winning if and only if $x+1$ is losing. Since $n_{i+1}$ is losing, $A_{i+1}$ is odd-winning if $n_{i+1}$ is even and even-winning otherwise. We observe that $A_{0}$ is even-winning if $N$ is odd and odd-winning otherwise. Also, if some $A_{i}$ is even-winning, then all $A_{i+1}, A_{i+2}, \\ldots$ are evenwinning and thus 1 is losing; i.e., the game is of type $A$. The game is of type $B$ if and only if the sets $A_{0}, A_{1}, \\ldots$ are alternately odd-winning and allwinning with $A_{0}$ odd-winning, which is equivalent to $N=n_{0}, n_{2}, n_{4}, \\ldots$ all being even. Thus $N$ is of type $B$ if and only if all digits at the odd positions in the binary representation of $N$ are zeros. Since $2004=\\overline{11111010100}$ in the binary system, 2004 is of type $A$. The least $N>2004$ that is of type $B$ is $\\overline{100000000000}=2^{11}=2048$. Thus the answer to part (b) is 2048.","problem_type":"Combinatorics","tier":0} +{"year":"2004","problem_phase":"shortlisted","problem":"13. C6 (IRN) For an $n \\times n$ matrix $A$, let $X_{i}$ be the set of entries in row $i$, and $Y_{j}$ the set of entries in column $j, 1 \\leq i, j \\leq n$. We say that $A$ is golden if $X_{1}, \\ldots, X_{n}, Y_{1}, \\ldots, Y_{n}$ are distinct sets. Find the least integer $n$ such that there exists a $2004 \\times 2004$ golden matrix with entries in the set $\\{1,2, \\ldots, n\\}$.","solution":"13. Since $X_{i}, Y_{i}, i=1, \\ldots, 2004$, are 4008 distinct subsets of the set $S_{n}=$ $\\{1,2, \\ldots, n\\}$, it follows that $2^{n} \\geq 4008$, i.e. $n \\geq 12$. Suppose $n=12$. Let $\\mathcal{X}=\\left\\{X_{1}, \\ldots, X_{2004}\\right\\}, \\mathcal{Y}=\\left\\{Y_{1}, \\ldots, Y_{2004}\\right\\}, \\mathcal{A}=$ $\\mathcal{X} \\cup \\mathcal{Y}$. Exactly $2^{12}-4008=88$ subsets of $S_{n}$ do not occur in $\\mathcal{A}$. Since each row intersects each column, we have $X_{i} \\cap Y_{j} \\neq \\emptyset$ for all $i, j$. Suppose $\\left|X_{i}\\right|,\\left|Y_{j}\\right| \\leq 3$ for some indices $i, j$. Since then $\\left|X_{i} \\cup Y_{j}\\right| \\leq 5$, any of at least $2^{7}>88$ subsets of $S_{n} \\backslash\\left(X_{i} \\cap Y_{j}\\right)$ can occur in neither $\\mathcal{X}$ nor $\\mathcal{Y}$, which is impossible. Hence either in $\\mathcal{X}$ or in $\\mathcal{Y}$ all subsets are of size at least 4. Suppose w.l.o.g. that $k=\\left|X_{l}\\right|=\\min _{i}\\left|X_{i}\\right| \\geq 4$. There are $$ n_{k}=\\binom{12-k}{0}+\\binom{12-k}{1}+\\cdots+\\binom{12-k}{k-1} $$ subsets of $S \\backslash X_{l}$ with fewer than $k$ elements, and none of them can be either in $\\mathcal{X}$ (because $\\left|X_{l}\\right|$ is minimal in $\\mathcal{X}$ ) or in $\\mathcal{Y}$. Hence we must have $n_{k} \\leq 88$. Since $n_{4}=93$ and $n_{5}=99$, it follows that $k \\geq 6$. But then none of the $\\binom{12}{0}+\\cdots+\\binom{12}{5}=1586$ subsets of $S_{n}$ is in $\\mathcal{X}$, hence at least $1586-88=1498$ of them are in $\\mathcal{Y}$. The 1498 complements of these subsets also do not occur in $\\mathcal{X}$, which adds to 3084 subsets of $S_{n}$ not occurring in $\\mathcal{X}$. This is clearly a contradiction. Now we construct a golden matrix for $n=13$. Let $$ A_{1}=\\left[\\begin{array}{ll} 1 & 1 \\\\ 2 & 3 \\end{array}\\right] \\quad \\text { and } \\quad A_{m}=\\left[\\begin{array}{ll} A_{m-1} & A_{m-1} \\\\ A_{m-1} & B_{m-1} \\end{array}\\right] \\text { for } m=2,3, \\ldots $$ where $B_{m-1}$ is the $2^{m-1} \\times 2^{m-1}$ matrix with all entries equal to $m+2$. It can be easily proved by induction that each of the matrices $A_{m}$ is golden. Moreover, every upper-left square submatrix of $A_{m}$ of size greater than $2^{m-1}$ is also golden. Since $2^{10}<2004<2^{11}$, we thus obtain a golden matrix of size 2004 with entries in $S_{13}$.","problem_type":"Combinatorics","tier":0} +{"year":"2004","problem_phase":"shortlisted","problem":"14. C7 (EST) ${ }^{\\mathrm{IMO} 3}$ Determine all $m \\times n$ rectangles that can be covered with hooks made up of 6 unit squares, as in the figure: ![](https:\/\/cdn.mathpix.com\/cropped\/2024_11_18_8e985d6b9c83aa3e9d0eg-338.jpg?height=153&width=150&top_left_y=1988&top_left_x=722) Rotations and reflections of hooks are allowed. The rectangle must be covered without gaps and overlaps. No part of a hook may cover area outside the rectangle.","solution":"14. Suppose that an $m \\times n$ rectangle can be covered by \"hooks\". For any hook $H$ there is a unique hook $K$ that covers its \" inside\" square. Then also $H$ covers the inside square of $K$, so the set of hooks can be partitioned into pairs of type $\\{H, K\\}$, each of which forms one of the following two figures consisting of 12 squares: ![](https:\/\/cdn.mathpix.com\/cropped\/2024_11_18_8e985d6b9c83aa3e9d0eg-730.jpg?height=252&width=979&top_left_y=972&top_left_x=306) Thus the $m \\times n$ rectangle is covered by these tiles. It immediately follows that $12 \\mid \\mathrm{mn}$. Suppose one of $m, n$ is divisible by 4 . Let w.l.o.g. $4 \\mid m$. If $3 \\mid n$, one can easily cover the rectangle by $3 \\times 4$ rectangles and therefore by hooks. Also, if $12 \\mid m$ and $n \\notin\\{1,2,5\\}$, then there exist $k, l \\in \\mathbb{N}_{0}$ such that $n=3 k+4 l$, and thus the rectangle $m \\times n$ can be partitioned into $3 \\times 12$ and $4 \\times 12$ rectangles all of which can be covered by hooks. If $12 \\mid m$ and $n=1,2$, or 5 , then it is easy to see that covering by hooks is not possible. Now suppose that $4 \\nmid m$ and $4 \\nmid n$. Then $m, n$ are even and the number of tiles is odd. Assume that the total number of tiles of types $A_{1}$ and $B_{1}$ is odd (otherwise the total number of tiles of types $A_{2}$ and $B_{2}$ is odd, which is analogous). If we color in black all columns whose indices are divisible by 4 , we see that each tile of type $A_{1}$ or $B_{1}$ covers three black squares, which yields an odd number in total. Hence the total number of black squares covered by the tiles of types $A_{2}$ and $B_{2}$ must be odd. This is impossible, since each such tile covers two or four black squares.","problem_type":"Combinatorics","tier":0} +{"year":"2004","problem_phase":"shortlisted","problem":"15. C8 (POL) For a finite graph $G$, let $f(G)$ be the number of triangles and $g(G)$ the number of tetrahedra formed by edges of $G$. Find the least constant $c$ such that $$ g(G)^{3} \\leq c \\cdot f(G)^{4} \\text { for every graph } G $$","solution":"15. Denote by $V_{1}, \\ldots, V_{n}$ the vertices of a graph $G$ and by $E$ the set of its edges. For each $i=1, \\ldots, n$, let $A_{i}$ be the set of vertices connected to $V_{i}$ by an edge, $G_{i}$ the subgraph of $G$ whose set of vertices is $A_{i}$, and $E_{i}$ the set of edges of $G_{i}$. Also, let $v_{i}, e_{i}$, and $t_{i}=f\\left(G_{i}\\right)$ be the numbers of vertices, edges, and triangles in $G_{i}$ respectively. The numbers of tetrahedra and triangles one of whose vertices is $V_{i}$ are respectively equal to $t_{i}$ and $e_{i}$. Hence $$ \\sum_{i=1}^{n} v_{i}=2|E|, \\quad \\sum_{i=1}^{n} e_{i}=3 f(G) \\quad \\text { and } \\quad \\sum_{i=1}^{n} t_{i}=4 g(G) . $$ Since $e_{i} \\leq v_{i}\\left(v_{i}-1\\right) \/ 2 \\leq v_{i}^{2} \/ 2$ and $e_{i} \\leq|E|$, we obtain $e_{i}^{2} \\leq v_{i}^{2}|E| \/ 2$, i.e., $e_{i} \\leq v_{i} \\sqrt{|E| \/ 2}$. Summing over all $i$ yields $3 f(G) \\leq 2|E| \\sqrt{|E| \/ 2}$, or equivalently $f(G)^{2} \\leq 2|E|^{3} \/ 9$. Since this relation holds for each graph $G_{i}$, it follows that $$ t_{i}=f\\left(G_{i}\\right)=f\\left(G_{i}\\right)^{1 \/ 3} f\\left(G_{i}\\right)^{2 \/ 3} \\leq\\left(\\frac{2}{9}\\right)^{1 \/ 3} f(G)^{1 \/ 3} e_{i} $$ Summing the last inequality for $i=1, \\ldots, n$ gives us $$ 4 g(G) \\leq 3\\left(\\frac{2}{9}\\right)^{1 \/ 3} f(G)^{1 \/ 3} \\cdot f(G), \\quad \\text { i.e. } \\quad g(G)^{3} \\leq \\frac{3}{32} f(G)^{4} $$ The constant $c=3 \/ 32$ is the best possible. Indeed, in a complete graph $C_{n}$ it holds that $g\\left(K_{n}\\right)^{3} \/ f\\left(K_{n}\\right)^{4}=\\binom{n}{4}^{3}\\binom{n}{3}^{-4} \\rightarrow \\frac{3}{32}$ as $n \\rightarrow \\infty$. Remark. Let $N_{k}$ be the number of complete $k$-subgraphs in a finite graph $G$. Continuing inductively, one can prove that $N_{k+1}^{k} \\leq \\frac{k!}{(k+1)^{k}} N_{k}^{k+1}$.","problem_type":"Combinatorics","tier":0} +{"year":"2004","problem_phase":"shortlisted","problem":"16. G1 (ROM) ${ }^{\\mathrm{IMO} 1}$ Let $A B C$ be an acute-angled triangle with $A B \\neq A C$. The circle with diameter $B C$ intersects the sides $A B$ and $A C$ at $M$ and $N$, respectively. Denote by $O$ the midpoint of $B C$. The bisectors of the angles $B A C$ and $M O N$ intersect at $R$. Prove that the circumcircles of the triangles $B M R$ and $C N R$ have a common point lying on the line segment $B C$.","solution":"16. Note that $\\triangle A N M \\sim \\triangle A B C$ and consequently $A M \\neq A N$. Since $O M=$ $O N$, it follows that $O R$ is a perpendicular bisector of $M N$. Thus, $R$ is the common point of the median of $M N$ and the bisector of $\\angle M A N$. Then it follows from a well-known fact that $R$ lies on the circumcircle of $\\triangle A M N$. Let $K$ be the intersection of $A R$ and $B C$. We then have $\\angle M R A=$ $\\angle M N A=\\angle A B K$ and $\\angle N R A=\\angle N M A=\\angle A C K$, from which we conclude that $R M B K$ and $R N C K$ are cyclic. Thus $K$ is the desired intersection of the circumcircles of $\\triangle B M R$ and $\\triangle C N R$ and it indeed lies on $B C$.","problem_type":"Geometry","tier":0} +{"year":"2004","problem_phase":"shortlisted","problem":"17. G2 (KAZ) The circle $\\Gamma$ and the line $\\ell$ do not intersect. Let $A B$ be the diameter of $\\Gamma$ perpendicular to $\\ell$, with $B$ closer to $\\ell$ than $A$. An arbitrary point $C \\neq A, B$ is chosen on $\\Gamma$. The line $A C$ intersects $\\ell$ at $D$. The line $D E$ is tangent to $\\Gamma$ at $E$, with $B$ and $E$ on the same side of $A C$. Let $B E$ intersect $\\ell$ at $F$, and let $A F$ intersect $\\Gamma$ at $G \\neq A$. Prove that the reflection of $G$ in $A B$ lies on the line $C F$.","solution":"17. Let $H$ be the reflection of $G$ about $A B(G H \\| \\ell)$. Let $M$ be the intersection of $A B$ and $\\ell$. Since $\\angle F E A=\\angle F M A=90^{\\circ}$, it follows that $A E M F$ is cyclic and hence $\\angle D F E=\\angle B A E=\\angle D E F$. The last equality holds because $D E$ is tangent to $\\Gamma$. It follows that $D E=$ $D F$ and hence $D F^{2}=D E^{2}=$ ![](https:\/\/cdn.mathpix.com\/cropped\/2024_11_18_8e985d6b9c83aa3e9d0eg-731.jpg?height=396&width=459&top_left_y=1591&top_left_x=841) $D C \\cdot D A$ (the power of $D$ with respect to $\\Gamma$ ). It then follows that $\\angle D C F=\\angle D F A=\\angle H G A=\\angle H C A$. Thus it follows that $H$ lies on $C F$ as desired.","problem_type":"Geometry","tier":0} +{"year":"2004","problem_phase":"shortlisted","problem":"18. G3 (KOR) Let $O$ be the circumcenter of an acute-angled triangle $A B C$ with $\\angle B<\\angle C$. The line $A O$ meets the side $B C$ at $D$. The circumcenters of the triangles $A B D$ and $A C D$ are $E$ and $F$, respectively. Extend the sides $B A$ and $C A$ beyond $A$, and choose on the respective extension points $G$ and $H$ such that $A G=A C$ and $A H=A B$. Prove that the quadrilateral $E F G H$ is a rectangle if and only if $\\angle A C B-\\angle A B C=60^{\\circ}$.","solution":"18. It is important to note that since $\\beta<\\gamma, \\angle A D C=90^{\\circ}-\\gamma+\\beta$ is acute. It is elementary that $\\angle C A O=90^{\\circ}-\\beta$. Let $X$ and $Y$ respectively be the intersections of $F E$ and $G H$ with $A D$. We trivially get $X \\in E F \\perp A D$ and $\\triangle A G H \\cong \\triangle A C B$. Consequently, $\\angle G A Y=\\angle O A B=90^{\\circ}-\\gamma=$ $90^{\\circ}-\\angle A G Y$. Hence, $G H \\perp A D$ and thus $G H \\| F E$. That $E F G H$ is a rectangle is now equivalent to $F X=G Y$ and $E X=H Y$. We have that $G Y=A G \\sin \\gamma=A C \\sin \\gamma$ and $F X=A F \\sin \\gamma$ (since $\\angle A F X=\\gamma$ ). Thus, $$ F X=G Y \\Leftrightarrow C F=A F=A C \\Leftrightarrow \\angle A F C=60^{\\circ} \\Leftrightarrow \\angle A D C=30^{\\circ} . $$ Since $\\angle A D C=180^{\\circ}-\\angle D C A-\\angle D A C=180^{\\circ}-\\gamma-\\left(90^{\\circ}-\\beta\\right)$, it immediately follows that $F X=G Y \\Leftrightarrow \\gamma-\\beta=60^{\\circ}$. We similarly obtain $E X=H Y \\Leftrightarrow \\gamma-\\beta=60^{\\circ}$, proving the statement of the problem.","problem_type":"Geometry","tier":0} +{"year":"2004","problem_phase":"shortlisted","problem":"19. $\\mathbf{G} 4 \\mathbf{( P O L})^{\\mathrm{IMO}}$ In a convex quadrilateral $A B C D$ the diagonal $B D$ does not bisect the angles $A B C$ and $C D A$. The point $P$ lies inside $A B C D$ and satisfies $$ \\angle P B C=\\angle D B A \\quad \\text { and } \\quad \\angle P D C=\\angle B D A . $$ Prove that $A B C D$ is a cyclic quadrilateral if and only if $A P=C P$.","solution":"19. Assume first that the points $A, B, C, D$ are concyclic. Let the lines $B P$ and $D P$ meet the circumcircle of $A B C D$ again at $E$ and $F$, respectively. Then it follows from the given conditions that $\\widehat{A B}=\\widehat{C F}$ and $\\widehat{A D}=\\widehat{C E}$; hence $B F \\| A C$ and $D E \\| A C$. Therefore $B F E D$ and $B F A C$ are isosceles trapezoids and thus $P=B E \\cap D F$ lies on the common bisector of segments $B F, E D, A C$. Hence $A P=C P$. Assume in turn that $A P=C P$. Let $P$ w.l.o.g. lie in the triangles $A C D$ and $B C D$. Let $B P$ and $D P$ meet $A C$ at $K$ and $L$, respectively. The points $A$ and $C$ are isogonal conjugates with respect to $\\triangle B D P$, which implies that $\\angle A P K=\\angle C P L$. Since $A P=C P$, we infer that $K$ and $L$ are symmetric with respect to the perpendicular bisector $p$ of $A C$. Let $E$ be the reflection of $D$ in $p$. Then $E$ lies on the line $B P$, and the triangles $A P D$ and $C P E$ are congruent. Thus $\\angle B D C=\\angle A D P=\\angle B E C$, which means that the points $B, C, E, D$ are concyclic. Moreover, $A, C, E, D$ are also concyclic. Hence, $A B C D$ is a cyclic quadrilateral.","problem_type":null,"tier":0} +{"year":"2004","problem_phase":"shortlisted","problem":"2. A2 (ROM) An infinite sequence $a_{0}, a_{1}, a_{2}, \\ldots$ of real numbers satisfies the condition $$ a_{n}=\\left|a_{n+1}-a_{n+2}\\right| \\text { for every } n \\geq 0 $$ with $a_{0}$ and $a_{1}$ positive and distinct. Can this sequence be bounded?","solution":"2. We claim that the sequence $\\left\\{a_{n}\\right\\}$ must be unbounded. The condition of the sequence is equivalent to $a_{n}>0$ and $a_{n+1}=a_{n}+a_{n-1}$ or $a_{n}-a_{n-1}$. In particular, if $a_{n}\\max \\left\\{a_{n}, a_{n-1}\\right\\}$. Let us remove all $a_{n}$ such that $a_{n}a_{n+1}$. We distinguish two cases: (i) If $a_{n+1}>a_{n}$, we have $b_{m}=a_{n+1}$ and $b_{m-1} \\geq a_{n-1}$ (since $b_{m-1}$ is either $a_{n-1}$ or $a_{n}$ ). Then $b_{m+1}-b_{m}=a_{n+2}-a_{n+1}=a_{n}=a_{n+1}-$ $a_{n-1}=b_{m}-a_{n-1} \\geq b_{m}-b_{m-1}$. (ii) If $a_{n+1}3$ is a prime. Assume to the contrary that $m \\in \\mathbb{N}$ is such that $m=p^{p-1} \\tau(m)$. Then $p^{p-1} \\mid m$, so we may set $m=p^{\\alpha} k$, where $\\alpha, k \\in \\mathbb{N}$, $\\alpha \\geq p-1$, and $p \\nmid k$. Let $k=p_{1}^{\\alpha_{1}} \\cdots p_{r}^{\\alpha_{r}}$ be the decomposition of $k$ into primes. Then $\\tau(k)=\\left(\\alpha_{1}+1\\right) \\cdots\\left(\\alpha_{r}+1\\right)$ and $\\tau(m)=(\\alpha+1) \\tau(k)$. Our equation becomes $$ p^{\\alpha-p+1} k=(\\alpha+1) \\tau(k) . $$ We observe that $\\alpha \\neq p-1$ : otherwise the RHS would be divisible by $p$ and the LHS would not be so. It follows that $\\alpha \\geq p$, which also easily implies that $p^{\\alpha-p+1} \\geq \\frac{p}{p+1}(\\alpha+1)$. Furthermore, since $\\alpha+1$ cannot be divisible by $p^{\\alpha-p+1}$ for any $\\alpha \\geq p$, it follows that $p \\mid \\tau(k)$. Thus if $p \\mid \\tau(k)$, then at least one $\\alpha_{i}+1$ is divisible by $p$ and consequently $\\alpha_{i} \\geq p-1$ for some $i$. Hence $k \\geq \\frac{p_{i}^{\\alpha_{i}}}{\\alpha_{i}+1} \\tau(k) \\geq \\frac{2^{p-1}}{p} \\tau(k)$. But then we have $$ p^{\\alpha-p+1} k \\geq \\frac{p}{p+1}(\\alpha+1) \\cdot \\frac{2^{p-1}}{p} \\tau(k)>(\\alpha+1) \\tau(k), $$ contradicting (1). Therefore (1) has no solutions in $\\mathbb{N}$. Remark. There are many other values of $a$ for which the considered equation has no solutions in $\\mathbb{N}$ : for example, $a=6 p$ for a prime $p \\geq 5$.","problem_type":"Number Theory","tier":0} +{"year":"2004","problem_phase":"shortlisted","problem":"25. N2 (RUS) The function $\\psi$ from the set $\\mathbb{N}$ of positive integers into itself is defined by the equality $$ \\psi(n)=\\sum_{k=1}^{n}(k, n), \\quad n \\in \\mathbb{N} $$ where $(k, n)$ denotes the greatest common divisor of $k$ and $n$. (a) Prove that $\\psi(m n)=\\psi(m) \\psi(n)$ for every two relatively prime $m, n \\in$ $\\mathbb{N}$. (b) Prove that for each $a \\in \\mathbb{N}$ the equation $\\psi(x)=a x$ has a solution. (c) Find all $a \\in \\mathbb{N}$ such that the equation $\\psi(x)=a x$ has a unique solution.","solution":"25. Let $n$ be a natural number. For each $k=1,2, \\ldots, n$, the number $(k, n)$ is a divisor of $n$. Consider any divisor $d$ of $n$. If $(k, n)=n \/ d$, then $k=n l \/ d$ for some $l \\in \\mathbb{N}$, and $(k, n)=(l, d) n \/ d$, which implies that $l$ is coprime to $d$ and $l \\leq d$. It follows that $(k, n)$ is equal to $n \/ d$ for exactly $\\varphi(d)$ natural numbers $k \\leq n$. Therefore $$ \\psi(n)=\\sum_{k=1}^{n}(k, n)=\\sum_{d \\mid n} \\varphi(d) \\frac{n}{d}=n \\sum_{d \\mid n} \\frac{\\varphi(d)}{d} $$ (a) Let $n, m$ be coprime. Then each divisor $f$ of $m n$ can be uniquely expressed as $f=d e$, where $d \\mid n$ and $e \\mid m$. We now have by (1) $$ \\begin{aligned} \\psi(m n) & =m n \\sum_{f \\mid m n} \\frac{\\varphi(f)}{f}=m n \\sum_{d|n, e| m} \\frac{\\varphi(d e)}{d e} \\\\ & =m n \\sum_{d|n, e| m} \\frac{\\varphi(d)}{d} \\frac{\\varphi(e)}{e}=\\left(n \\sum_{d \\mid n} \\frac{\\varphi(d)}{d}\\right)\\left(m \\sum_{e \\mid m} \\frac{\\varphi(e)}{e}\\right) \\\\ & =\\psi(m) \\psi(n) . \\end{aligned} $$ (b) Let $n=p^{k}$, where $p$ is a prime and $k$ a positive integer. According to (1), $$ \\frac{\\psi(n)}{n}=\\sum_{i=0}^{k} \\frac{\\varphi\\left(p^{i}\\right)}{p^{i}}=1+\\frac{k(p-1)}{p} $$ Setting $p=2$ and $k=2(a-1)$ we obtain $\\psi(n)=a n$ for $n=2^{2(a-1)}$. (c) We note that $\\psi\\left(p^{p}\\right)=p^{p+1}$ if $p$ is a prime. Hence, if $a$ has an odd prime factor $p$ and $a_{1}=a \/ p$, then $x=p^{p} 2^{2 a_{1}-2}$ is a solution of $\\psi(x)=a x$ different from $x=2^{2 a-2}$. Now assume that $a=2^{k}$ for some $k \\in \\mathbb{N}$. Suppose $x=2^{\\alpha} y$ is a positive integer such that $\\psi(x)=2^{k} x$. Then $2^{\\alpha+k} y=\\psi(x)=\\psi\\left(2^{\\alpha}\\right) \\psi(y)=$ $(\\alpha+2) 2^{\\alpha-1} \\psi(y)$, i.e., $2^{k+1} y=(\\alpha+2) \\psi(y)$. We notice that for each odd $y, \\psi(y)$ is (by definition) the sum of an odd number of odd summands and therefore odd. It follows that $\\psi(y) \\mid y$. On the other hand, $\\psi(y)>$ $y$ for $y>1$, so we must have $y=1$. Consequently $\\alpha=2^{k+1}-2=2 a-2$, giving us the unique solution $x=2^{2 a-2}$. Thus $\\psi(x)=a x$ has a unique solution if and only if $a$ is a power of 2 .","problem_type":"Number Theory","tier":0} +{"year":"2004","problem_phase":"shortlisted","problem":"26. N3 (IRN) A function $f$ from the set of positive integers $\\mathbb{N}$ into itself is such that for all $m, n \\in \\mathbb{N}$ the number $\\left(m^{2}+n\\right)^{2}$ is divisible by $f^{2}(m)+$ $f(n)$. Prove that $f(n)=n$ for each $n \\in \\mathbb{N}$.","solution":"26. For $m=n=1$ we obtain that $f(1)^{2}+f(1)$ divides $\\left(1^{2}+1\\right)^{2}=4$, from which we find that $f(1)=1$. Next, we show that $f(p-1)=p-1$ for each prime $p$. By the hypothesis for $m=1$ and $n=p-1, f(p-1)+1$ divides $p^{2}$, so $f(p-1)$ equals either $p-1$ or $p^{2}-1$. If $f(p-1)=p^{2}-1$, then $f(1)+f(p-1)^{2}=p^{4}-2 p^{2}+2$ divides $\\left(1+(p-1)^{2}\\right)^{2}1$, denote by $P_{n}$ the product of all positive integers $x$ less than $n$ and such that $n$ divides $x^{2}-1$. For each $n>1$, find the remainder of $P_{n}$ on division by $n$.","solution":"29. Let $S_{n}=\\left\\{x \\in \\mathbb{N}|x \\leq n, n| x^{2}-1\\right\\}$. It is easy to check that $P_{n} \\equiv 1$ $(\\bmod n)$ for $n=2$ and $P_{n} \\equiv-1(\\bmod n)$ for $n \\in\\{3,4\\}$, so from now on we assume $n>4$. We note that if $x \\in S_{n}$, then also $n-x \\in S_{n}$ and $(x, n)=1$. Thus $S_{n}$ splits into pairs $\\{x, n-x\\}$, where $x \\in S_{n}$ and $x \\leq n \/ 2$. In each of these pairs the product of elements gives remainder -1 upon division by $n$. Therefore $P_{n} \\equiv(-1)^{m}$, where $S_{n}$ has $2 m$ elements. It remains to find the parity of $m$. Suppose first that $n>4$ is divisible by 4 . Whenever $x \\in S_{n}$, the numbers $|n \/ 2-x|, n-x, n-|n \/ 2-x|$ also belong to $S_{n}$ (indeed, $n \\mid(n \/ 2-x)^{2}-1=$ $n^{2} \/ 4-n x+x^{2}-1$ because $n \\mid n^{2} \/ 4$, etc.). In this way the set $S_{n}$ splits into four-element subsets $\\{x, n \/ 2-x, n \/ 2+x, n-x\\}$, where $x \\in S_{n}$ and $x4\\right)$. Therefore $m=\\left|S_{n}\\right| \/ 2$ is even and $P_{n} \\equiv 1$ $(\\bmod m)$. Now let $n$ be odd. If $n \\mid x^{2}-1=(x-1)(x+1)$, then there exist natural numbers $a, b$ such that $a b=n, a|x-1, b| x+1$. Obviously $a$ and $b$ are coprime. Conversely, given any odd $a, b \\in \\mathbb{N}$ such that $(a, b)=1$ and $a b=n$, by the Chinese remainder theorem there exists $x \\in\\{1,2, \\ldots, n-1\\}$ such that $a \\mid x-1$ and $b \\mid x+1$. This gives a bijection between all ordered pairs $(a, b)$ with $a b=n$ and $(a, b)=1$ and the elements of $S_{n}$. Now if $n=p_{1}^{\\alpha_{1}} \\cdots p_{k}^{\\alpha_{k}}$ is the decomposition of $n$ into primes, the number of pairs $(a, b)$ is equal to $2^{k}$ (since for every $i$, either $p_{i}^{\\alpha_{i}} \\mid a$ or $p_{i}^{\\alpha_{i}} \\mid b$ ), and hence $m=2^{k-1}$. Thus $P_{n} \\equiv-1(\\bmod n)$ if $n$ is a power of an odd prime, and $P_{n} \\equiv 1$ otherwise. Finally, let $n$ be even but not divisible by 4 . Then $x \\in S_{n}$ if and only if $x$ or $n-x$ belongs to $S_{n \/ 2}$ and $x$ is odd. Since $n \/ 2$ is odd, for each $x \\in S_{n \/ 2}$ either $x$ or $x+n \/ 2$ belongs to $S_{n}$, and by the case of $n$ odd we have $S_{n} \\equiv \\pm 1(\\bmod n \/ 2)$, depending on whether or not $n \/ 2$ is a power of a prime. Since $S_{n}$ is odd, it follows that $P_{n} \\equiv-1(\\bmod n)$ if $n \/ 2$ is a power of a prime, and $P_{n} \\equiv 1$ otherwise. Second solution. Obviously $S_{n}$ is closed under multiplication modulo $n$. This implies that $S_{n}$ with multiplication modulo $n$ is a subgroup of $\\mathbb{Z}_{n}$, and therefore there exist elements $a_{1}=-1, a_{2}, \\ldots, a_{k} \\in S_{n}$ that generate $S_{n}$. In other words, since the $a_{i}$ are of order two, $S_{n}$ consists of products $\\prod_{i \\in A} a_{i}$, where $A$ runs over all subsets of $\\{1,2, \\ldots, k\\}$. Thus $S_{n}$ has $2^{k}$ elements, and the product of these elements equals $P_{n} \\equiv\\left(a_{1} a_{2} \\cdots a_{k}\\right)^{2^{k-1}}$ $(\\bmod n)$. Since $a_{i}^{2} \\equiv 1(\\bmod n)$, it follows that $P_{n} \\equiv 1$ if $k \\geq 2$, i.e., if $\\left|S_{n}\\right|>2$. Otherwise $P_{n} \\equiv-1(\\bmod n)$. We note that $\\left|S_{n}\\right|>2$ is equivalent to the existence of $a \\in S_{n}$ with $10$ can be uniquely expressed as a continued fraction of the form $a_{0}+1 \/\\left(a_{1}+1 \/\\left(a_{2}+1 \/\\left(\\cdots+1 \/ a_{n}\\right)\\right)\\right)$ (where $a_{0} \\in \\mathbb{N}_{0}, a_{1}, \\ldots, a_{n} \\in \\mathbb{N}$ ). Then we write $x=\\left[a_{0} ; a_{1}, a_{2}, \\ldots, a_{n}\\right]$. Since $n$ depends only on $x$, the function $s(x)=(-1)^{n}$ is well-defined. For $x<0$ we define $s(x)=-s(-x)$, and set $s(0)=1$. We claim that this $s(x)$ satisfies the requirements of the problem. The equality $s(x) s(y)=-1$ trivially holds if $x+y=0$. Suppose that $x y=1$. We may assume w.l.o.g. that $x>y>0$. Then $x>1$, so if $x=\\left[a_{0} ; a_{1}, a_{2}, \\ldots, a_{n}\\right]$, then $a_{0} \\geq 1$ and $y=0+1 \/ x=$ $\\left[0 ; a_{0}, a_{1}, a_{2}, \\ldots, a_{n}\\right]$. It follows that $s(x)=(-1)^{n}, s(y)=(-1)^{n+1}$, and hence $s(x) s(y)=-1$. Finally, suppose that $x+y=1$. We consider two cases: (i) Let $x, y>0$. We may assume w.l.o.g. that $x>1 \/ 2$. Then there exist natural numbers $a_{2}, \\ldots, a_{n}$ such that $x=\\left[0 ; 1, a_{2}, \\ldots, a_{n}\\right]=$ $1 \/(1+1 \/ t)$, where $t=\\left[a_{2}, \\ldots, a_{n}\\right]$. Since $y=1-x=1 \/(1+t)=$ $\\left[0 ; 1+a_{2}, a_{3}, \\ldots, a_{n}\\right]$, we have $s(x)=(-1)^{n}$ and $s(y)=(-1)^{n-1}$, giving us $s(x) s(y)=-1$. (ii) Let $x>0>y$. If $a_{0}, \\ldots, a_{n} \\in \\mathbb{N}$ are such that $-y=\\left[a_{0} ; a_{1}, \\ldots, a_{n}\\right]$, then $x=\\left[1+a_{0} ; a_{1}, \\ldots, a_{n}\\right]$. Thus $s(y)=-s(-y)=-(-1)^{n}$ and $s(x)=(-1)^{n}$, so again $s(x) s(y)=-1$.","problem_type":"Algebra","tier":0} +{"year":"2004","problem_phase":"shortlisted","problem":"30. N7 (BUL) Let $p$ be an odd prime and $n$ a positive integer. In the coordinate plane, eight distinct points with integer coordinates lie on a circle with diameter of length $p^{n}$. Prove that there exists a triangle with vertices at three of the given points such that the squares of its side lengths are integers divisible by $p^{n+1}$.","solution":"30. We shall denote by $k$ the given circle with diameter $p^{n}$. Let $A, B$ be lattice points (i.e., points with integer coordinates). We shall denote by $\\mu(A B)$ the exponent of the highest power of $p$ that divides the integer $A B^{2}$. We observe that if $S$ is the area of a triangle $A B C$ where $A, B, C$ are lattice points, then $2 S$ is an integer. According to Heron's formula and the formula for the circumradius, a triangle $A B C$ whose circumcenter has diameter $p^{n}$ satisfies $$ 2 A B^{2} B C^{2}+2 B C^{2} C A^{2}+2 C A^{2} A B^{2}-A B^{4}-B C^{4}-C A^{4}=16 S^{2} $$ and $$ A B^{2} \\cdot B C^{2} \\cdot C A^{2}=(2 S)^{2} p^{2 n} $$ Lemma 1. Let $A, B$, and $C$ be lattice points on $k$. If none of $A B^{2}, B C^{2}$, $C A^{2}$ is divisible by $p^{n+1}$, then $\\mu(A B), \\mu(B C), \\mu(C A)$ are $0, n, n$ in some order. Proof. Let $k=\\min \\{\\mu(A B), \\mu(B C), \\mu(C A)\\}$. By (1), $(2 S)^{2}$ is divisible by $p^{2 k}$. Together with (2), this gives us $\\mu(A B)+\\mu(B C)+\\mu(C A)=$ $2 k+2 n$. On the other hand, if none of $A B^{2}, B C^{2}, C A^{2}$ is divisible by $p^{n+1}$, then $\\mu(A B)+\\mu(B C)+\\mu(C A) \\leq k+2 n$. Therefore $k=0$ and the remaining two of $\\mu(A B), \\mu(B C), \\mu(C A)$ are equal to $n$. Lemma 2. Among every four lattice points on $k$, there exist two, say $M, N$, such that $\\mu(M N) \\geq n+1$. Proof. Assume that this doesn't hold for some points $A, B, C, D$ on $k$. By Lemma $1, \\mu$ for some of the segments $A B, A C, \\ldots, C D$ is 0 , say $\\mu(A C)=0$. It easily follows by Lemma 1 that then $\\mu(B D)=0$ and $\\mu(A B)=\\mu(B C)=\\mu(C D)=\\mu(D A)=n$. Let $a, b, c, d, e, f \\in \\mathbb{N}$ be such that $A B^{2}=p^{n} a, B C^{2}=p^{n} b, C D^{2}=p^{n} c, D A^{2}=p^{n} d, A C^{2}=e$, $B D^{2}=f$. By Ptolemy's theorem we have $\\sqrt{e f}=p^{n}(\\sqrt{a c}+\\sqrt{b d})$. Taking squares, we get that $\\frac{e f}{p^{2 n}}=(\\sqrt{a c}+\\sqrt{b d})^{2}=a c+b d+2 \\sqrt{a b c d}$ is rational and hence an integer. It follows that ef is divisible by $p^{2 n}$, a contradiction. Now we consider eight lattice points $A_{1}, A_{2}, \\ldots, A_{8}$ on $k$. We color each segment $A_{i} A_{j}$ red if $\\mu\\left(A_{i} A_{j}\\right)>n$ and black otherwise, and thus obtain a graph $G$. The degree of a point $X$ will be the number of red segments with an endpoint in $X$. We distinguish three cases: (i) There is a point, say $A_{8}$, whose degree is at most 1 . We may suppose w.l.o.g. that $A_{8} A_{7}$ is red and $A_{8} A_{1}, \\ldots, A_{8} A_{6}$ black. By a well-known fact, the segments joining vertices $A_{1}, A_{2}, \\ldots, A_{6}$ determine either a red triangle, in which case there is nothing to prove, or a black triangle, say $A_{1} A_{2} A_{3}$. But in the latter case the four points $A_{1}, A_{2}, A_{3}, A_{8}$ do not determine any red segment, a contradiction to Lemma 2 . (ii) All points have degree 2. Then the set of red segments partitions into cycles. If one of these cycles has length 3 , then the proof is complete. If all the cycles have length at least 4 , then we have two possibilities: two 4 -cycles, say $A_{1} A_{2} A_{3} A_{4}$ and $A_{5} A_{6} A_{7} A_{8}$, or one 8-cycle, $A_{1} A_{2} \\ldots A_{8}$. In both cases, the four points $A_{1}, A_{3}, A_{5}, A_{7}$ do not determine any red segment, a contradiction. (iii) There is a point of degree at least 3 , say $A_{1}$. Suppose that $A_{1} A_{2}$, $A_{1} A_{3}$, and $A_{1} A_{4}$ are red. We claim that $A_{2}, A_{3}, A_{4}$ determine at least one red segment, which will complete the solution. If not, by Lemma $1, \\mu\\left(A_{2} A_{3}\\right), \\mu\\left(A_{3} A_{4}\\right), \\mu\\left(A_{4} A_{2}\\right)$ are $n, n, 0$ in some order. Assuming w.l.o.g. that $\\mu\\left(A_{2} A_{3}\\right)=0$, denote by $S$ the area of triangle $A_{1} A_{2} A_{3}$. Now by formula (1), $2 S$ is not divisible by $p$. On the other hand, since $\\mu\\left(A_{1} A_{2}\\right) \\geq n+1$ and $\\mu\\left(A_{1} A_{3}\\right) \\geq n+1$, it follows from (2) that $2 S$ is divisible by $p$, a contradiction.","problem_type":"Number Theory","tier":0} +{"year":"2004","problem_phase":"shortlisted","problem":"4. A4 (KOR) ${ }^{\\mathrm{IMO} 2}$ Find all polynomials $P(x)$ with real coefficients that satisfy the equality $$ P(a-b)+P(b-c)+P(c-a)=2 P(a+b+c) $$ for all triples $a, b, c$ of real numbers such that $a b+b c+c a=0$.","solution":"4. Let $P(x)=a_{0}+a_{1} x+\\cdots+a_{n} x^{n}$. For every $x \\in \\mathbb{R}$ the triple $(a, b, c)=$ $(6 x, 3 x,-2 x)$ satisfies the condition $a b+b c+c a=0$. Then the condition on $P$ gives us $P(3 x)+P(5 x)+P(-8 x)=2 P(7 x)$ for all $x$, implying that for all $i=0,1,2, \\ldots, n$ the following equality holds: $$ \\left(3^{i}+5^{i}+(-8)^{i}-2 \\cdot 7^{i}\\right) a_{i}=0 $$ Suppose that $a_{i} \\neq 0$. Then $K(i)=3^{i}+5^{i}+(-8)^{i}-2 \\cdot 7^{i}=0$. But $K(i)$ is negative for $i$ odd and positive for $i=0$ or $i \\geq 6$ even. Only for $i=2$ and $i=4$ do we have $K(i)=0$. It follows that $P(x)=a_{2} x^{2}+a_{4} x^{4}$ for some real numbers $a_{2}, a_{4}$. It is easily verified that all such $P(x)$ satisfy the required condition.","problem_type":"Algebra","tier":0} +{"year":"2004","problem_phase":"shortlisted","problem":"5. A5 (THA) Let $a, b, c>0$ and $a b+b c+c a=1$. Prove the inequality $$ \\sqrt[3]{\\frac{1}{a}+6 b}+\\sqrt[3]{\\frac{1}{b}+6 c}+\\sqrt[3]{\\frac{1}{c}+6 a} \\leq \\frac{1}{a b c} $$","solution":"5. By the general mean inequality $\\left(M_{1} \\leq M_{3}\\right)$, the LHS of the inequality to be proved does not exceed $$ E=\\frac{3}{\\sqrt[3]{3}} \\sqrt[3]{\\frac{1}{a}+\\frac{1}{b}+\\frac{1}{c}+6(a+b+c)} $$ From $a b+b c+c a=1$ we obtain that $3 a b c(a+b+c)=3(a b \\cdot a c+$ $a b \\cdot b c+a c \\cdot b c) \\leq(a b+a c+b c)^{2}=1$; hence $6(a+b+c) \\leq \\frac{2}{a b c}$. Since $\\frac{1}{a}+\\frac{1}{b}+\\frac{1}{c}=\\frac{a b+b c+c a}{a b c}=\\frac{1}{a b c}$, it follows that $$ E \\leq \\frac{3}{\\sqrt[3]{3}} \\sqrt[3]{\\frac{3}{a b c}} \\leq \\frac{1}{a b c} $$ where the last inequality follows from the AM-GM inequality $1=a b+b c+$ $c a \\geq 3 \\sqrt[3]{(a b c)^{2}}$, i.e., $a b c \\leq 1 \/(3 \\sqrt{3})$. The desired inequality now follows. Equality holds if and only if $a=b=c=1 \/ \\sqrt{3}$.","problem_type":"Algebra","tier":0} +{"year":"2004","problem_phase":"shortlisted","problem":"6. A6 (RUS) Find all functions $f: \\mathbb{R} \\rightarrow \\mathbb{R}$ satisfying the equation $$ f\\left(x^{2}+y^{2}+2 f(x y)\\right)=(f(x+y))^{2} \\quad \\text { for all } x, y \\in \\mathbb{R} $$","solution":"6. Let us make the substitution $z=x+y, t=x y$. Given $z, t \\in \\mathbb{R}, x, y$ are real if and only if $4 t \\leq z^{2}$. Define $g(x)=2(f(x)-x)$. Now the given functional equation transforms into $$ f\\left(z^{2}+g(t)\\right)=(f(z))^{2} \\text { for all } t, z \\in \\mathbb{R} \\text { with } z^{2} \\geq 4 t $$ Let us set $c=g(0)=2 f(0)$. Substituting $t=0$ into (1) gives us $$ f\\left(z^{2}+c\\right)=(f(z))^{2} \\text { for all } z \\in \\mathbb{R} $$ If $c<0$, then taking $z$ such that $z^{2}+c=0$, we obtain from (2) that $f(z)^{2}=c \/ 2$, which is impossible; hence $c \\geq 0$. We also observe that $$ x>c \\quad \\text { implies } \\quad f(x) \\geq 0 $$ If $g$ is a constant function, we easily find that $c=0$ and therefore $f(x)=x$, which is indeed a solution. Suppose $g$ is nonconstant, and let $a, b \\in \\mathbb{R}$ be such that $g(a)-g(b)=d>0$. For some sufficiently large $K$ and each $u, v \\geq K$ with $v^{2}-u^{2}=d$ the equality $u^{2}+g(a)=v^{2}+g(b)$ by (1) and (3) implies $f(u)=f(v)$. This further leads to $g(u)-g(v)=2(v-u)=\\frac{d}{u+\\sqrt{u^{2}+d}}$. Therefore every value from some suitably chosen segment $[\\delta, 2 \\delta]$ can be expressed as $g(u)-g(v)$, with $u$ and $v$ bounded from above by some $M$. Consider any $x, y$ with $y>x \\geq 2 \\sqrt{M}$ and $\\deltaN$ in (2) we obtain $k^{2}=k$, so $k=0$ or $k=1$ \u3002 By (2) we have $f(-z)= \\pm f(z)$, and thus $|f(z)| \\leq 1$ for all $z \\leq-N$. Hence $g(u)=2 f(u)-2 u \\geq-2-2 u$ for $u \\leq-N$, which implies that $g$ is unbounded. Hence for each $z$ there exists $t$ such that $z^{2}+g(t)>N$, and consequently $f(z)^{2}=f\\left(z^{2}+g(t)\\right)=k=k^{2}$. Therefore $f(z)= \\pm k$ for each $z$. If $k=0$, then $f(x) \\equiv 0$, which is clearly a solution. Assume $k=1$. Then $c=2 f(0)=2$ (because $c \\geq 0$ ), which together with (3) implies $f(x)=1$ for all $x \\geq 2$. Suppose that $f(t)=-1$ for some $t<2$. Then $t-g(t)=3 t+2>4 t$. If also $t-g(t) \\geq 0$, then for some $z \\in \\mathbb{R}$ we have $z^{2}=t-g(t)>4 t$, which by (1) leads to $f(z)^{2}=f\\left(z^{2}+g(t)\\right)=f(t)=-1$, which is impossible. Hence $t-g(t)<0$, giving us $t<-2 \/ 3$. On the other hand, if $X$ is any subset of $(-\\infty,-2 \/ 3)$, the function $f$ defined by $f(x)=-1$ for $x \\in X$ and $f(x)=1$ satisfies the requirements of the problem. To sum up, the solutions are $f(x)=x, f(x)=0$ and all functions of the form $$ f(x)= \\begin{cases}1, & x \\notin X \\\\ -1, & x \\in X\\end{cases} $$ where $X \\subset(-\\infty,-2 \/ 3)$.","problem_type":"Algebra","tier":0} +{"year":"2004","problem_phase":"shortlisted","problem":"7. A7 (IRE) Let $a_{1}, a_{2}, \\ldots, a_{n}$ be positive real numbers, $n>1$. Denote by $g_{n}$ their geometric mean, and by $A_{1}, A_{2}, \\ldots, A_{n}$ the sequence of arithmetic means defined by $A_{k}=\\frac{a_{1}+a_{2}+\\cdots+a_{k}}{k}, k=1,2, \\ldots, n$. Let $G_{n}$ be the geometric mean of $A_{1}, A_{2}, \\ldots, A_{n}$. Prove the inequality $$ n \\sqrt[n]{\\frac{G_{n}}{A_{n}}}+\\frac{g_{n}}{G_{n}} \\leq n+1 $$ and establish the cases of equality.","solution":"7. Let us set $c_{k}=A_{k-1} \/ A_{k}$ for $k=1,2, \\ldots, n$, where we define $A_{0}=0$. We observe that $a_{k} \/ A_{k}=\\left(k A_{k}-(k-1) A_{k-1}\\right) \/ A_{k}=k-(k-1) c_{k}$. Now we can write the LHS of the inequality to be proved in terms of $c_{k}$, as follows: $$ \\sqrt[n]{\\frac{G_{n}}{A_{n}}}=\\sqrt[n^{2}]{c_{2} c_{3}^{2} \\cdots c_{n}^{n-1}} \\text { and } \\frac{g_{n}}{G_{n}}=\\sqrt[n]{\\prod_{k=1}^{n}\\left(k-(k-1) c_{k}\\right)} $$ By the $A M-G M$ inequality we have $$ \\begin{aligned} n \\sqrt[n^{2}]{1^{n(n+1) \/ 2} c_{2} c_{3}^{2} \\ldots c_{n}^{n-1}} & \\leq \\frac{1}{n}\\left(\\frac{n(n+1)}{2}+\\sum_{k=2}^{n}(k-1) c_{k}\\right) \\\\ & =\\frac{n+1}{2}+\\frac{1}{n} \\sum_{k=1}^{n}(k-1) c_{k} . \\end{aligned} $$ Also by the AM-GM inequality, we have $$ \\sqrt[n]{\\prod_{k=1}^{n}\\left(k-(k-1) c_{k}\\right)} \\leq \\frac{n+1}{2}-\\frac{1}{n} \\sum_{k=1}^{n}(k-1) c_{k} $$ Adding (1) and (2), we obtain the desired inequality. Equality holds if and only if $a_{1}=a_{2}=\\cdots=a_{n}$.","problem_type":"Algebra","tier":0} +{"year":"2004","problem_phase":"shortlisted","problem":"8. C1 (PUR) There are 10001 students at a university. Some students join together to form several clubs (a student may belong to different clubs). Some clubs join together to form several societies (a club may belong to different societies). There are a total of $k$ societies. Suppose that the following conditions hold: (i) Each pair of students are in exactly one club. (ii) For each student and each society, the student is in exactly one club of the society. (iii) Each club has an odd number of students. In addition, a club with $2 m+1$ students ( $m$ is a positive integer) is in exactly $m$ societies. Find all possible values of $k$.","solution":"8. Let us write $n=10001$. Denote by $\\mathcal{T}$ the set of ordered triples $(a, C, \\mathcal{S})$, where $a$ is a student, $C$ a club, and $\\mathcal{S}$ a society such that $a \\in C$ and $C \\in \\mathcal{S}$. We shall count $|\\mathcal{T}|$ in two different ways. Fix a student $a$ and a society $\\mathcal{S}$. By (ii), there is a unique club $C$ such that $(a, C, \\mathcal{S}) \\in \\mathcal{T}$. Since the ordered pair $(a, \\mathcal{S})$ can be chosen in $n k$ ways, we have that $|\\mathcal{T}|=n k$. Now fix a club $C$. By (iii), $C$ is in exactly $(|C|-1) \/ 2$ societies, so there are $|C|(|C|-1) \/ 2$ triples from $\\mathcal{T}$ with second coordinate $C$. If $\\mathcal{C}$ is the set of all clubs, we obtain $|\\mathcal{T}|=\\sum_{C \\in \\mathcal{C}} \\frac{|C|(|C|-1)}{2}$. But we also conclude from (i) that $$ \\sum_{C \\in \\mathcal{C}} \\frac{|C|(|C|-1)}{2}=\\frac{n(n-1)}{2} $$ Therefore $n(n-1) \/ 2=n k$, i.e., $k=(n-1) \/ 2=5000$. On the other hand, for $k=(n-1) \/ 2$ there is a desired configuration with only one club $C$ that contains all students and $k$ identical societies with only one element (the club $C$ ). It is easy to verify that (i)-(iii) hold.","problem_type":"Combinatorics","tier":0} +{"year":"2004","problem_phase":"shortlisted","problem":"9. C2 (GER) Let $n$ and $k$ be positive integers. There are given $n$ circles in the plane. Every two of them intersect at two distinct points, and all points of intersection they determine are distinct. Each intersection point must be colored with one of $n$ distinct colors so that each color is used at least once, and exactly $k$ distinct colors occur on each circle. Find all values of $n \\geq 2$ and $k$ for which such a coloring is possible.","solution":"9. Obviously we must have $2 \\leq k \\leq n$. We shall prove that the possible values for $k$ and $n$ are $2 \\leq k \\leq n \\leq 3$ and $3 \\leq k \\leq n$. Denote all colors and circles by $1, \\ldots, n$. Let $F(i, j)$ be the set of colors of the common points of circles $i$ and $j$. Suppose that $k=2i$. We now prove by induction on $k$ that a desired coloring exists for each $n \\geq k \\geq 3$. Let there be given $n$ circles. By the inductive hypothesis, circles $1,2, \\ldots, n-1$ can be colored in $n-1$ colors, $k$ of which appear on each circle, such that color $i$ appears on circle $i$. Then we set $F(i, n)=\\{i, n\\}$ for $i=1, \\ldots, k$ and $F(i, n)=\\{n\\}$ for $i>n$. We thus obtain a coloring of the $n$ circles in $n$ colors, such that $k+1$ colors (including color $i$ ) appear on each circle $i$.","problem_type":"Combinatorics","tier":0} diff --git a/USAJMO/md/en-JMO-2010-notes.md b/USAJMO/md/en-JMO-2010-notes.md new file mode 100644 index 0000000000000000000000000000000000000000..da9a1981ffee2c7bb4c71a97e05a01f84d7a95ad --- /dev/null +++ b/USAJMO/md/en-JMO-2010-notes.md @@ -0,0 +1,255 @@ +# JMO 2010 Solution Notes + +Evan Chen《陳誼廷》 + +15 April 2024 + + +#### Abstract + +This is a compilation of solutions for the 2010 JMO. The ideas of the solution are a mix of my own work, the solutions provided by the competition organizers, and solutions found by the community. However, all the writing is maintained by me. + +These notes will tend to be a bit more advanced and terse than the "official" solutions from the organizers. In particular, if a theorem or technique is not known to beginners but is still considered "standard", then I often prefer to use this theory anyways, rather than try to work around or conceal it. For example, in geometry problems I typically use directed angles without further comment, rather than awkwardly work around configuration issues. Similarly, sentences like "let $\mathbb{R}$ denote the set of real numbers" are typically omitted entirely. + +Corrections and comments are welcome! + + +## Contents + +0 Problems +1 Solutions to Day 1 +1.1 JMO 2010/1, proposed by Andy Niedermier ..... 3 +1.2 JMO 2010/2, proposed by Răzvan Gelca ..... 4 +1.3 JMO 2010/3, proposed by Titu Andreescu +2 Solutions to Day 2 ..... 7 +2.1 JMO 2010/4, proposed by Zuming Feng ..... 7 +2.2 JMO 2010/5, proposed by Gregory Galperin ..... 8 +2.3 JMO 2010/6, proposed by Zuming Feng ..... 9 + +## §0 Problems + +1. Let $P(n)$ be the number of permutations $\left(a_{1}, \ldots, a_{n}\right)$ of the numbers $(1,2, \ldots, n)$ for which $k a_{k}$ is a perfect square for all $1 \leq k \leq n$. Find with proof the smallest $n$ such that $P(n)$ is a multiple of 2010 . +2. Let $n>1$ be an integer. Find, with proof, all sequences $x_{1}, x_{2}, \ldots, x_{n-1}$ of positive integers with the following three properties: +(a) $x_{1}1$ be an integer. Find, with proof, all sequences $x_{1}, x_{2}, \ldots, x_{n-1}$ of positive integers with the following three properties: +(a) $x_{1}0$, consider a triangle with vertices at $\left(a, a^{2}\right),\left(-a, a^{2}\right)$ and $\left(b, b^{2}\right)$. Then the area of this triangle was equal to + +$$ +\frac{1}{2}(2 a)\left(b^{2}-a^{2}\right)=a\left(b^{2}-a^{2}\right) . +$$ + +To make this equal $2^{2 n} m^{2}$, simply pick $a=2^{2 n}$, and then pick $b$ such that $b^{2}-m^{2}=2^{4 n}$, for example $m=2^{4 n-2}-1$ and $b=2^{4 n-2}+1$. + +## §2.2 JMO 2010/5, proposed by Gregory Galperin + +Available online at https://aops.com/community/p1860912. + +## Problem statement + +Two permutations $a_{1}, a_{2}, \ldots, a_{2010}$ and $b_{1}, b_{2}, \ldots, b_{2010}$ of the numbers $1,2, \ldots, 2010$ are said to intersect if $a_{k}=b_{k}$ for some value of $k$ in the range $1 \leq k \leq 2010$. Show that there exist 1006 permutations of the numbers $1,2, \ldots, 2010$ such that any other such permutation is guaranteed to intersect at least one of these 1006 permutations. + +A valid choice is the following 1006 permutations: + +| 1 | 2 | 3 | $\cdots$ | 1004 | 1005 | 1006 | 1007 | 1008 | $\cdots$ | 2009 | 2010 | +| :---: | :---: | :---: | :---: | :---: | :---: | :---: | :---: | :---: | :---: | :---: | :---: | +| 2 | 3 | 4 | $\cdots$ | 1005 | 1006 | 1 | 1007 | 1008 | $\cdots$ | 2009 | 2010 | +| 3 | 4 | 5 | $\cdots$ | 1006 | 1 | 2 | 1007 | 1008 | $\cdots$ | 2009 | 2010 | +| $\vdots$ | $\vdots$ | $\vdots$ | $\ddots$ | $\vdots$ | $\vdots$ | $\vdots$ | $\vdots$ | $\vdots$ | $\vdots$ | $\vdots$ | $\vdots$ | +| 1004 | 1005 | 1006 | $\cdots$ | 1001 | 1002 | 1003 | 1007 | 1008 | $\cdots$ | 2009 | 2010 | +| 1005 | 1006 | 1 | $\cdots$ | 1002 | 1003 | 1004 | 1007 | 1008 | $\cdots$ | 2009 | 2010 | +| 1006 | 1 | 2 | $\cdots$ | 1003 | 1004 | 1005 | 1007 | 1008 | $\cdots$ | 2009 | 2010 | + +This works. Indeed, any permutation should have one of $\{1,2, \ldots, 1006\}$ somewhere in the first 1006 positions, so one will get an intersection. + +Remark. In fact, the last 1004 entries do not matter with this construction, and we chose to leave them as $1007,1008, \ldots, 2010$ only for concreteness. + +Remark. Using Hall's marriage lemma one may prove that the result becomes false with 1006 replaced by 1005 . + +## §2.3 JMO 2010/6, proposed by Zuming Feng + +Available online at https://aops.com/community/p1860753. + +## Problem statement + +Let $A B C$ be a triangle with $\angle A=90^{\circ}$. Points $D$ and $E$ lie on sides $A C$ and $A B$, respectively, such that $\angle A B D=\angle D B C$ and $\angle A C E=\angle E C B$. Segments $B D$ and $C E$ meet at $I$. Determine whether or not it is possible for segments $A B, A C, B I$, $I D, C I, I E$ to all have integer lengths. + +The answer is no. We prove that it is not even possible that $A B, A C, C I, I B$ are all integers. +![](https://cdn.mathpix.com/cropped/2024_11_19_a4f5a16d632e1d6ed723g-9.jpg?height=427&width=504&top_left_y=923&top_left_x=776) + +First, we claim that $\angle B I C=135^{\circ}$. To see why, note that + +$$ +\angle I B C+\angle I C B=\frac{\angle B}{2}+\frac{\angle C}{2}=\frac{90^{\circ}}{2}=45^{\circ} . +$$ + +So, $\angle B I C=180^{\circ}-(\angle I B C+\angle I C B)=135^{\circ}$, as desired. +We now proceed by contradiction. The Pythagorean theorem implies + +$$ +B C^{2}=A B^{2}+A C^{2} +$$ + +and so $B C^{2}$ is an integer. However, the law of cosines gives + +$$ +\begin{aligned} +B C^{2} & =B I^{2}+C I^{2}-2 B I \cdot C I \cos \angle B I C \\ +& =B I^{2}+C I^{2}+B I \cdot C I \cdot \sqrt{2} . +\end{aligned} +$$ + +which is irrational, and this produces the desired contradiction. + diff --git a/USAJMO/md/en-JMO-2011-notes.md b/USAJMO/md/en-JMO-2011-notes.md new file mode 100644 index 0000000000000000000000000000000000000000..ba968b385f4aad81a0a9e2a76b96b9104a47c491 --- /dev/null +++ b/USAJMO/md/en-JMO-2011-notes.md @@ -0,0 +1,245 @@ +# JMO 2011 Solution Notes + +Evan Chen《陳誼廷》 + +15 April 2024 + + +#### Abstract + +This is a compilation of solutions for the 2011 JMO. The ideas of the solution are a mix of my own work, the solutions provided by the competition organizers, and solutions found by the community. However, all the writing is maintained by me. + +These notes will tend to be a bit more advanced and terse than the "official" solutions from the organizers. In particular, if a theorem or technique is not known to beginners but is still considered "standard", then I often prefer to use this theory anyways, rather than try to work around or conceal it. For example, in geometry problems I typically use directed angles without further comment, rather than awkwardly work around configuration issues. Similarly, sentences like "let $\mathbb{R}$ denote the set of real numbers" are typically omitted entirely. + +Corrections and comments are welcome! + + +## Contents + +0 Problems +1 Solutions to Day 1 +1.1 JMO 2011/1, proposed by Titu Andreescu ..... 3 +1.2 JMO 2011/2, proposed by Titu Andreescu ..... 4 +1.3 JMO 2011/3, proposed by Zuming Feng +2 Solutions to Day 2 ..... 7 +2.1 JMO 2011/4, proposed by Gabriel Carroll ..... 7 +2.2 JMO 2011/5, proposed by Zuming Feng ..... 8 +2.3 JMO 2011/6, proposed by Sam Vandervelde ..... 9 + +## §0 Problems + +1. Find all positive integers $n$ such that $2^{n}+12^{n}+2011^{n}$ is a perfect square. +2. Let $a, b, c$ be positive real numbers such that $a^{2}+b^{2}+c^{2}+(a+b+c)^{2} \leq 4$. Prove that + +$$ +\frac{a b+1}{(a+b)^{2}}+\frac{b c+1}{(b+c)^{2}}+\frac{c a+1}{(c+a)^{2}} \geq 3 +$$ + +3. For a point $P=\left(a, a^{2}\right)$ in the coordinate plane, let $\ell(P)$ denote the line passing through $P$ with slope $2 a$. Consider the set of triangles with vertices of the form $P_{1}=\left(a_{1}, a_{1}^{2}\right), P_{2}=\left(a_{2}, a_{2}^{2}\right), P_{3}=\left(a_{3}, a_{3}^{2}\right)$, such that the intersection of the lines $\ell\left(P_{1}\right), \ell\left(P_{2}\right), \ell\left(P_{3}\right)$ form an equilateral triangle $\Delta$. Find the locus of the center of $\Delta$ as $P_{1} P_{2} P_{3}$ ranges over all such triangles. +4. A word is defined as any finite string of letters. A word is a palindrome if it reads the same backwards and forwards. Let a sequence of words $W_{0}, W_{1}, W_{2}, \ldots$ be defined as follows: $W_{0}=a, W_{1}=b$, and for $n \geq 2, W_{n}$ is the word formed by writing $W_{n-2}$ followed by $W_{n-1}$. Prove that for any $n \geq 1$, the word formed by writing $W_{1}, W_{2}, W_{3}, \ldots, W_{n}$ in succession is a palindrome. +5. Points $A, B, C, D, E$ lie on a circle $\omega$ and point $P$ lies outside the circle. The given points are such that (i) lines $P B$ and $P D$ are tangent to $\omega$, (ii) $P, A, C$ are collinear, and (iii) $\overline{D E} \| \overline{A C}$. Prove that $\overline{B E}$ bisects $\overline{A C}$. +6. Consider the assertion that for each positive integer $n \geq 2$, the remainder upon dividing $2^{2^{n}}$ by $2^{n}-1$ is a power of 4 . Either prove the assertion or find (with proof) a counterexample. + +## §1 Solutions to Day 1 + +## §1.1 JMO 2011/1, proposed by Titu Andreescu + +Available online at https://aops.com/community/p2254778. + +## Problem statement + +Find all positive integers $n$ such that $2^{n}+12^{n}+2011^{n}$ is a perfect square. + +The answer $n=1$ works, because $2^{1}+12^{1}+2011^{1}=45^{2}$. We prove it's the only one. + +- If $n \geq 2$ is even, then modulo 3 we have $2^{n}+12^{n}+2011^{n} \equiv 1+0+1 \equiv 2(\bmod 3)$ so it is not a square. +- If $n \geq 3$ is odd, then modulo 4 we have $2^{n}+12^{n}+2011^{n} \equiv 0+0+3 \equiv 3(\bmod 4)$ so it is not a square. + +This completes the proof. + +## §1.2 JMO 2011/2, proposed by Titu Andreescu + +Available online at https://aops.com/community/p2254758. + +## Problem statement + +Let $a, b, c$ be positive real numbers such that $a^{2}+b^{2}+c^{2}+(a+b+c)^{2} \leq 4$. Prove that + +$$ +\frac{a b+1}{(a+b)^{2}}+\frac{b c+1}{(b+c)^{2}}+\frac{c a+1}{(c+a)^{2}} \geq 3 +$$ + +The condition becomes $2 \geq a^{2}+b^{2}+c^{2}+a b+b c+c a$. Therefore, + +$$ +\begin{aligned} +\sum_{\text {cyc }} \frac{2 a b+2}{(a+b)^{2}} & \geq \sum_{\text {cyc }} \frac{2 a b+\left(a^{2}+b^{2}+c^{2}+a b+b c+c a\right)}{(a+b)^{2}} \\ +& =\sum_{\text {cyc }} \frac{(a+b)^{2}+(c+a)(c+b)}{(a+b)^{2}} \\ +& =3+\sum_{\text {cyc }} \frac{(c+a)(c+b)}{(a+b)^{2}} \\ +& \geq 3+3 \sqrt[3]{\prod_{\text {cyc }} \frac{(c+a)(c+b)}{(a+b)^{2}}}=3+3=6 +\end{aligned} +$$ + +with the last line by AM-GM. This completes the proof. + +## §1.3 JMO 2011/3, proposed by Zuming Feng + +Available online at https://aops.com/community/p2254823. + +## Problem statement + +For a point $P=\left(a, a^{2}\right)$ in the coordinate plane, let $\ell(P)$ denote the line passing through $P$ with slope $2 a$. Consider the set of triangles with vertices of the form $P_{1}=\left(a_{1}, a_{1}^{2}\right), P_{2}=\left(a_{2}, a_{2}^{2}\right), P_{3}=\left(a_{3}, a_{3}^{2}\right)$, such that the intersection of the lines $\ell\left(P_{1}\right), \ell\left(P_{2}\right), \ell\left(P_{3}\right)$ form an equilateral triangle $\Delta$. Find the locus of the center of $\Delta$ as $P_{1} P_{2} P_{3}$ ranges over all such triangles. + +The answer is the line $y=-1 / 4$. I did not find this problem inspiring, so I will not write out most of the boring calculations since most solutions are just going to be "use Cartesian coordinates and grind all the way through". + +The "nice" form of the main claim is as follows (which is certainly overkill for the present task, but is too good to resist including): + +Claim (Naoki Sato) - In general, the orthocenter of $\Delta$ lies on the directrix $y=-1 / 4$ of the parabola (even if the triangle $\Delta$ is not equilateral). + +Proof. By writing out the equation $y=2 a_{i} x-a_{i}^{2}$ for $\ell\left(P_{i}\right)$, we find the vertices of the triangle are located at + +$$ +\left(\frac{a_{1}+a_{2}}{2}, a_{1} a_{2}\right) ; \quad\left(\frac{a_{2}+a_{3}}{2}, a_{2} a_{3}\right) ; \quad\left(\frac{a_{3}+a_{1}}{2}, a_{3} a_{1}\right) . +$$ + +The coordinates of the orthocenter can be checked explicitly to be + +$$ +H=\left(\frac{a_{1}+a_{2}+a_{3}+4 a_{1} a_{2} a_{3}}{2},-\frac{1}{4}\right) . +$$ + +An advanced synthetic proof of this fact is given at https://aops.com/community/ p2255814. +This claim already shows that every point lies on $y=-1 / 4$. We now turn to showing that, even when restricted to equilateral triangles, we can achieve every point on $y=-1 / 4$. In what follows $a=a_{1}, b=a_{2}, c=a_{3}$ for legibility. + +Claim - Lines $\ell(a), \ell(b), \ell(c)$ form an equilateral triangle if and only if + +$$ +\begin{aligned} +a+b+c & =-12 a b c \\ +a b+b c+c a & =-\frac{3}{4} . +\end{aligned} +$$ + +Moreover, the $x$-coordinate of the equilateral triangle is $\frac{1}{3}(a+b+c)$. +Proof. The triangle is equilateral if and only if the centroid and orthocenter coincide, i.e. + +$$ +\left(\frac{a+b+c}{3}, \frac{a b+b c+c a}{3}\right)=G=H=\left(\frac{a+b+c+4 a b c}{2},-\frac{1}{4}\right) . +$$ + +Setting the $x$ and $y$ coordinates equal, we derive the claimed equations. + +Let $\lambda$ be any real number. We are tasked to show that + +$$ +P(X)=X^{3}-3 \lambda \cdot X^{2}-\frac{3}{4} X+\frac{\lambda}{4} +$$ + +has three real roots (with multiplicity); then taking those roots as $(a, b, c)$ yields a valid equilateral-triangle triple whose $x$-coordinate is exactly $\lambda$, be the previous claim. + +To prove that, pick the values + +$$ +\begin{aligned} +P(-\sqrt{3} / 2) & =-2 \lambda \\ +P(0) & =\frac{1}{4} \lambda \\ +P(\sqrt{3} / 2) & =-2 \lambda . +\end{aligned} +$$ + +The intermediate value theorem (at least for $\lambda \neq 0$ ) implies that $P$ should have at least two real roots now, and since $P$ has degree 3, it has all real roots. That's all. + +## §2 Solutions to Day 2 + +## §2.1 JMO 2011/4, proposed by Gabriel Carroll + +Available online at https://aops.com/community/p2254808. + +## Problem statement + +A word is defined as any finite string of letters. A word is a palindrome if it reads the same backwards and forwards. Let a sequence of words $W_{0}, W_{1}, W_{2}, \ldots$ be defined as follows: $W_{0}=a, W_{1}=b$, and for $n \geq 2, W_{n}$ is the word formed by writing $W_{n-2}$ followed by $W_{n-1}$. Prove that for any $n \geq 1$, the word formed by writing $W_{1}, W_{2}$, $W_{3}, \ldots, W_{n}$ in succession is a palindrome. + +To aid in following the solution, here are the first several words: + +$$ +\begin{aligned} +& W_{0}=a \\ +& W_{1}=b \\ +& W_{2}=a b \\ +& W_{3}=b a b \\ +& W_{4}=a b b a b \\ +& W_{5}=b a b a b b a b \\ +& W_{6}=a b b a b b a b a b b a b \\ +& W_{7}=b a b a b b a b a b b a b b a b a b b a b +\end{aligned} +$$ + +We prove that $W_{1} W_{2} \ldots W_{n}$ is a palindrome by induction on $n$. The base cases $n=$ $1,2,3,4$ can be verified by hand. + +For the inductive step, we let $\bar{X}$ denote the word $X$ written backwards. Then + +$$ +\begin{aligned} +W_{1} W_{2} \ldots W_{n-3} W_{n-2} W_{n-1} W_{n} & \stackrel{\mathrm{IH}}{=}\left(\overline{W_{n-1} W_{n-2} W_{n-3}} \ldots \overline{W_{2} W_{1}}\right) W_{n} \\ +& =\left(\overline{W_{n-1} W_{n-2} W_{n-3}} \ldots \overline{W_{2} W_{1}}\right) W_{n-2} W_{n-1} \\ +& =\overline{W_{n-1} W_{n-2}}\left(\overline{W_{n-3}} \ldots \overline{W_{2} W_{1}}\right) W_{n-2} W_{n-1} +\end{aligned} +$$ + +with the first equality being by the induction hypothesis. By induction hypothesis again the inner parenthesized term is also a palindrome, and so this completes the proof. + +## §2.2 JMO 2011/5, proposed by Zuming Feng + +Available online at https://aops.com/community/p2254813. + +## Problem statement + +Points $A, B, C, D, E$ lie on a circle $\omega$ and point $P$ lies outside the circle. The given points are such that (i) lines $P B$ and $P D$ are tangent to $\omega$, (ii) $P, A, C$ are collinear, and (iii) $\overline{D E} \| \overline{A C}$. Prove that $\overline{B E}$ bisects $\overline{A C}$. + +We present two solutions. +【 First solution using harmonic bundles. Let $M=\overline{B E} \cap \overline{A C}$ and let $\infty$ be the point at infinity along $\overline{D E} \| \overline{A C}$. +![](https://cdn.mathpix.com/cropped/2024_11_19_3c39ee86a3666408bff8g-8.jpg?height=646&width=817&top_left_y=962&top_left_x=628) + +Note that $A B C D$ is harmonic, so + +$$ +-1=(A C ; B D) \stackrel{E}{=}(A C ; M \infty) +$$ + +implying $M$ is the midpoint of $\overline{A C}$. +I Second solution using complex numbers (Cynthia Du). Suppose we let $b, d, e$ be free on unit circle, so $p=\frac{2 b d}{b+d}$. Then $d / c=a / e$, and $a+c=p+a c \bar{p}$. Consequently, + +$$ +\begin{aligned} +a c & =d e \\ +\frac{1}{2}(a+c) & =\frac{b d}{b+d}+d e \cdot \frac{1}{b+d}=\frac{d(b+e)}{b+d} \\ +\frac{a+c}{2 a c} & =\frac{(b+e)}{e(b+d)} +\end{aligned} +$$ + +From here it's easy to see + +$$ +\frac{a+c}{2}+\frac{a+c}{2 a c} \cdot b e=b+e +$$ + +which is what we wanted to prove. + +## §2.3 JMO 2011/6, proposed by Sam Vandervelde + +Available online at https://aops.com/community/p2254810. + +## Problem statement + +Consider the assertion that for each positive integer $n \geq 2$, the remainder upon dividing $2^{2^{n}}$ by $2^{n}-1$ is a power of 4 . Either prove the assertion or find (with proof) a counterexample. + +We claim $n=25$ is a counterexample. Since $2^{25} \equiv 2^{0}\left(\bmod 2^{25}-1\right)$, we have + +$$ +2^{2^{25}} \equiv 2^{2^{25} \bmod 25} \equiv 2^{7} \bmod 2^{25}-1 +$$ + +and the right-hand side is actually the remainder, since $0<2^{7}<2^{25}$. But $2^{7}$ is not a power of 4. + +Remark. Really, the problem is just equivalent for asking $2^{n}$ to have odd remainder when divided by $n$. + diff --git a/USAJMO/md/en-JMO-2012-notes.md b/USAJMO/md/en-JMO-2012-notes.md new file mode 100644 index 0000000000000000000000000000000000000000..91b90f5af10234962a85c2e63ada6420f0c86bf7 --- /dev/null +++ b/USAJMO/md/en-JMO-2012-notes.md @@ -0,0 +1,301 @@ +# JMO 2012 Solution Notes + +Evan Chen《陳誼廷》 + +15 April 2024 + + +#### Abstract + +This is a compilation of solutions for the 2012 JMO. The ideas of the solution are a mix of my own work, the solutions provided by the competition organizers, and solutions found by the community. However, all the writing is maintained by me. + +These notes will tend to be a bit more advanced and terse than the "official" solutions from the organizers. In particular, if a theorem or technique is not known to beginners but is still considered "standard", then I often prefer to use this theory anyways, rather than try to work around or conceal it. For example, in geometry problems I typically use directed angles without further comment, rather than awkwardly work around configuration issues. Similarly, sentences like "let $\mathbb{R}$ denote the set of real numbers" are typically omitted entirely. + +Corrections and comments are welcome! + + +## Contents + +0 Problems ..... 2 +1 Solutions to Day 1 +1.1 JMO 2012/1, proposed by Sungyoon Kim, Inseok Seo ..... 3 +1.2 JMO 2012/2, proposed by Titu Andreescu ..... 4 +1.3 JMO 2012/3, proposed by Titu Andreescu ..... 6 +2 Solutions to Day 2 ..... 8 +2.1 JMO 2012/4, proposed by Sam Vandervelde ..... 8 +2.2 JMO 2012/5, proposed by Warut Suksompong ..... 9 +2.3 JMO 2012/6, proposed by Titu Andreescu, Cosmin Pohoata ..... 10 + +## §0 Problems + +1. Given a triangle $A B C$, let $P$ and $Q$ be points on segments $\overline{A B}$ and $\overline{A C}$, respectively, such that $A P=A Q$. Let $S$ and $R$ be distinct points on segment $\overline{B C}$ such that $S$ lies between $B$ and $R, \angle B P S=\angle P R S$, and $\angle C Q R=\angle Q S R$. Prove that $P, Q$, $R, S$ are concyclic. +2. Find all integers $n \geq 3$ such that among any $n$ positive real numbers $a_{1}, a_{2}, \ldots$, $a_{n}$ with + +$$ +\max \left(a_{1}, a_{2}, \ldots, a_{n}\right) \leq n \cdot \min \left(a_{1}, a_{2}, \ldots, a_{n}\right), +$$ + +there exist three that are the side lengths of an acute triangle. +3. For $a, b, c>0$ prove that + +$$ +\frac{a^{3}+3 b^{3}}{5 a+b}+\frac{b^{3}+3 c^{3}}{5 b+c}+\frac{c^{3}+3 a^{3}}{5 c+a} \geq \frac{2}{3}\left(a^{2}+b^{2}+c^{2}\right) +$$ + +4. Let $\alpha$ be an irrational number with $0<\alpha<1$, and draw a circle in the plane whose circumference has length 1 . Given any integer $n \geq 3$, define a sequence of points $P_{1}, P_{2}, \ldots, P_{n}$ as follows. First select any point $P_{1}$ on the circle, and for $2 \leq k \leq n$ define $P_{k}$ as the point on the circle for which the length of $\operatorname{arc} P_{k-1} P_{k}$ is $\alpha$, when travelling counterclockwise around the circle from $P_{k-1}$ to $P_{k}$. Suppose that $P_{a}$ and $P_{b}$ are the nearest adjacent points on either side of $P_{n}$. Prove that $a+b \leq n$. +5. For distinct positive integers $a, b<2012$, define $f(a, b)$ to be the number of integers $k$ with $1 \leq k<2012$ such that the remainder when $a k$ divided by 2012 is greater than that of $b k$ divided by 2012 . Let $S$ be the minimum value of $f(a, b)$, where $a$ and $b$ range over all pairs of distinct positive integers less than 2012. Determine $S$. +6. Let $P$ be a point in the plane of $\triangle A B C$, and $\gamma$ a line through $P$. Let $A^{\prime}, B^{\prime}, C^{\prime}$ be the points where the reflections of lines $P A, P B, P C$ with respect to $\gamma$ intersect lines $B C, C A, A B$ respectively. Prove that $A^{\prime}, B^{\prime}, C^{\prime}$ are collinear. + +## §1 Solutions to Day 1 + +## §1.1 JMO 2012/1, proposed by Sungyoon Kim, Inseok Seo + +Available online at https://aops.com/community/p2669111. + +## Problem statement + +Given a triangle $A B C$, let $P$ and $Q$ be points on segments $\overline{A B}$ and $\overline{A C}$, respectively, such that $A P=A Q$. Let $S$ and $R$ be distinct points on segment $\overline{B C}$ such that $S$ lies between $B$ and $R, \angle B P S=\angle P R S$, and $\angle C Q R=\angle Q S R$. Prove that $P, Q, R$, $S$ are concyclic. + +Assume for contradiction that $(P R S)$ and $(Q R S)$ are distinct. Then $\overline{R S}$ is the radical axis of these two circles. However, $\overline{A P}$ is tangent to $(P R S)$ and $\overline{A Q}$ is tangent to $(Q R S)$, so point $A$ has equal power to both circles, which is impossible since $A$ does not lie on line $B C$. + +## §1.2 JMO 2012/2, proposed by Titu Andreescu + +Available online at https://aops.com/community/p2669112. + +## Problem statement + +Find all integers $n \geq 3$ such that among any $n$ positive real numbers $a_{1}, a_{2}, \ldots, a_{n}$ with + +$$ +\max \left(a_{1}, a_{2}, \ldots, a_{n}\right) \leq n \cdot \min \left(a_{1}, a_{2}, \ldots, a_{n}\right) +$$ + +there exist three that are the side lengths of an acute triangle. + +The answer is all $n \geq 13$. +Define $\left(F_{n}\right)$ as the sequence of Fibonacci numbers, by $F_{1}=F_{2}=1$ and $F_{n+1}=$ $F_{n}+F_{n-1}$. We will find that Fibonacci numbers show up naturally when we work through the main proof, so we will isolate the following calculation now to make the subsequent solution easier to read. + +Claim - For positive integers $m$, we have $F_{m} \leq m^{2}$ if and only if $m \leq 12$. +Proof. A table of the first 14 Fibonacci numbers is given below. + +$$ +\begin{array}{rrrrrrrrrrrrrr} +F_{1} & F_{2} & F_{3} & F_{4} & F_{5} & F_{6} & F_{7} & F_{8} & F_{9} & F_{10} & F_{11} & F_{12} & F_{13} & F_{14} \\ +\hline 1 & 1 & 2 & 3 & 5 & 8 & 13 & 21 & 34 & 55 & 89 & 144 & 233 & 377 +\end{array} +$$ + +By examining the table, we see that $F_{m} \leq m^{2}$ is true for $m=1,2, \ldots 12$, and in fact $F_{12}=12^{2}=144$. However, $F_{m}>m^{2}$ for $m=13$ and $m=14$. + +Now it remains to prove that $F_{m}>m^{2}$ for $m \geq 15$. The proof is by induction with base cases $m=13$ and $m=14$ being checked already. For the inductive step, if $m \geq 15$ then we have + +$$ +\begin{aligned} +F_{m} & =F_{m-1}+F_{m-2}>(m-1)^{2}+(m-2)^{2} \\ +& =2 m^{2}-6 m+5=m^{2}+(m-1)(m-5)>m^{2} +\end{aligned} +$$ + +as desired. +We now proceed to the main problem. The hypothesis $\max \left(a_{1}, a_{2}, \ldots, a_{n}\right) \leq n$. $\min \left(a_{1}, a_{2}, \ldots, a_{n}\right)$ will be denoted by $(\dagger)$. + +Proof that all $n \geq 13$ have the property. We first show now that every $n \geq 13$ has the desired property. Suppose for contradiction that no three numbers are the sides of an acute triangle. Assume without loss of generality (by sorting the numbers) that $a_{1} \leq a_{2} \leq \cdots \leq a_{n}$. Then since $a_{i-1}, a_{i}, a_{i+1}$ are not the sides of an acute triangle for each $i \geq 2$, we have that $a_{i+1}^{2} \geq a_{i}^{2}+a_{i-1}^{2}$; writing this out gives + +$$ +\begin{aligned} +a_{3}^{2} & \geq a_{2}^{2}+a_{1}^{2} \geq 2 a_{1}^{2} \\ +a_{4}^{2} & \geq a_{3}^{2}+a_{2}^{2} \geq 2 a_{1}^{2}+a_{1}^{2}=3 a_{1}^{2} \\ +a_{5}^{2} & \geq a_{4}^{2}+a_{3}^{2} \geq 3 a_{1}^{2}+2 a_{1}^{2}=5 a_{1}^{2} \\ +a_{6}^{2} & \geq a_{5}^{2}+a_{4}^{2} \geq 5 a_{1}^{2}+3 a_{1}^{2}=8 a_{1}^{2} +\end{aligned} +$$ + +and so on. The Fibonacci numbers appear naturally and by induction, we conclude that $a_{i}^{2} \geq F_{i} a_{1}^{2}$. In particular, $a_{n}^{2} \geq F_{n} a_{1}^{2}$. + +However, we know $\max \left(a_{1}, \ldots, a_{n}\right)=a_{n}$ and $\min \left(a_{1}, \ldots, a_{n}\right)=a_{1}$, so $(\dagger)$ reads $a_{n} \leq n \cdot a_{1}$. Therefore we have $F_{n} \leq n^{2}$, and so $n \leq 12$, contradiction! + +Proof that no $n \leq 12$ have the property. Assume that $n \leq 12$. The above calculation also suggests a way to pick the counterexample: we choose $a_{i}=\sqrt{F_{i}}$ for every $i$. Then $\min \left(a_{1}, \ldots, a_{n}\right)=a_{1}=1$ and $\max \left(a_{1}, \ldots, a_{n}\right)=\sqrt{F_{n}}$, so $(\dagger)$ is true as long as $n \leq 12$. And indeed no three numbers form the sides of an acute triangle: if $i0$ prove that + +$$ +\frac{a^{3}+3 b^{3}}{5 a+b}+\frac{b^{3}+3 c^{3}}{5 b+c}+\frac{c^{3}+3 a^{3}}{5 c+a} \geq \frac{2}{3}\left(a^{2}+b^{2}+c^{2}\right) +$$ + +Here are two possible approaches. +Cauchy-Schwarz approach. Apply Titu lemma to get + +$$ +\sum_{\mathrm{cyc}} \frac{a^{3}}{5 a+b}=\sum_{\mathrm{cyc}} \frac{a^{4}}{5 a^{2}+a b} \geq \frac{\left(a^{2}+b^{2}+c^{2}\right)^{2}}{\sum_{\mathrm{cyc}}\left(5 a^{2}+a b\right)} \geq \frac{a^{2}+b^{2}+c^{2}}{6} +$$ + +where the last step follows from the identity $\sum_{\text {cyc }}\left(5 a^{2}+a b\right) \leq 6\left(a^{2}+b^{2}+c^{2}\right)$. +Similarly, + +$$ +\sum_{\text {cyc }} \frac{b^{3}}{5 a+b}=\sum_{\mathrm{cyc}} \frac{b^{4}}{5 a b+b^{2}} \geq \frac{\left(a^{2}+b^{2}+c^{2}\right)^{2}}{\sum_{\mathrm{cyc}}\left(5 a b+b^{2}\right)} \geq \frac{a^{2}+b^{2}+c^{2}}{6} +$$ + +using the fact that $\sum_{\text {cyc }} 5 a b+b^{2} \leq 6\left(a^{2}+b^{2}+c^{2}\right)$. +Therefore, adding the first display to three times the second display implies the result. +đ Cauchy-Schwarz approach. The main magical claim is: +Claim - We have + +$$ +\frac{a^{3}+3 b^{3}}{5 a+b} \geq \frac{25}{36} b^{2}-\frac{1}{36} a^{2} +$$ + +Proof. Let $x=a / b>0$. The desired inequality is equivalent to + +$$ +\frac{x^{3}+3}{5 x+1} \geq \frac{25-x^{2}}{36} +$$ + +However, + +$$ +\begin{aligned} +36\left(x^{3}+3\right)-(5 x+1)\left(25-x^{2}\right) & =41 x^{3}+x^{2}-125 x+83 \\ +& =(x-1)^{2}(41 x+83) \geq 0 +\end{aligned} +$$ + +Sum the claim cyclically to finish. +Remark (Derivation of the main claim). The overall strategy is to hope for a constant $k$ such that + +$$ +\frac{a^{3}+3 b^{3}}{5 a+b} \geq k a^{2}+\left(\frac{2}{3}-k\right) b^{2} +$$ + +is true. Letting $x=a / b$ as above and expanding, we need a value $k$ such that the cubic polynomial +$P(x):=\left(x^{3}+3\right)-(5 x+1)\left(k x^{2}+\left(\frac{2}{3}-k\right)\right)=(1-5 k) x^{3}-k x^{2}+\left(5 k-\frac{10}{3}\right) x+\left(k+\frac{7}{3}\right)$ +is nonnegative everywhere. Since $P(1)=0$ necessarily, in order for $P(1-\varepsilon)$ and $P(1+\varepsilon)$ to both be nonnegative (for small $\varepsilon$ ), the polynomial $P$ must have a double root at 1 , meaning the first derivative $P^{\prime}(1)=0$ needs to vanish. In other words, we need + +$$ +3(1-5 k)-2 k+\left(5 k-\frac{10}{3}\right)=0 +$$ + +Solving gives $k=-1 / 36$. One then factors out the repeated root $(x-1)^{2}$ from the resulting $P$. + +## §2 Solutions to Day 2 + +## §2.1 JMO 2012/4, proposed by Sam Vandervelde + +Available online at https://aops.com/community/p2669956. + +## Problem statement + +Let $\alpha$ be an irrational number with $0<\alpha<1$, and draw a circle in the plane whose circumference has length 1 . Given any integer $n \geq 3$, define a sequence of points $P_{1}, P_{2}, \ldots, P_{n}$ as follows. First select any point $P_{1}$ on the circle, and for $2 \leq k \leq n$ define $P_{k}$ as the point on the circle for which the length of $\operatorname{arc} P_{k-1} P_{k}$ is $\alpha$, when travelling counterclockwise around the circle from $P_{k-1}$ to $P_{k}$. Suppose that $P_{a}$ and $P_{b}$ are the nearest adjacent points on either side of $P_{n}$. Prove that $a+b \leq n$. + +No points coincide since $\alpha$ is irrational. +Assume for contradiction that $nr_{b}$ or $2012-r_{a}>2012-r_{b}$. + +This implies $S \geq \frac{1}{2} \varphi(2012)=502$. +But this can actually be achieved by taking $a=4$ and $b=1010$, since + +- If $k$ is even, then $a k \equiv b k(\bmod 2012)$ so no even $k$ counts towards $S$; and +- If $k \equiv 0(\bmod 503)$, then $a k \equiv 0(\bmod 2012)$ so no such $k$ counts towards $S$. + +This gives the final answer $S \geq 502$. +Remark. A similar proof works with 2012 replaced by any $n$ and will give an answer of $\frac{1}{2} \varphi(n)$. For composite $n$, one uses the Chinese remainder theorem to pick distinct $a$ and $b$ not divisible by $n$ such that $\operatorname{lcm}(a-b, a)=n$. + +## §2.3 JMO 2012/6, proposed by Titu Andreescu, Cosmin Pohoata + +Available online at https://aops.com/community/p2669960. + +## Problem statement + +Let $P$ be a point in the plane of $\triangle A B C$, and $\gamma$ a line through $P$. Let $A^{\prime}, B^{\prime}, C^{\prime}$ be the points where the reflections of lines $P A, P B, P C$ with respect to $\gamma$ intersect lines $B C, C A, A B$ respectively. Prove that $A^{\prime}, B^{\prime}, C^{\prime}$ are collinear. + +We present three solutions. +【 First solution (complex numbers). Let $p=0$ and set $\gamma$ as the real line. Then $A^{\prime}$ is the intersection of $b c$ and $p \bar{a}$. So, we get + +$$ +a^{\prime}=\frac{\bar{a}(\bar{b} c-b \bar{c})}{(\bar{b}-\bar{c}) \bar{a}-(b-c) a} . +$$ + +![](https://cdn.mathpix.com/cropped/2024_11_19_4c570ff524ef777a3bedg-10.jpg?height=364&width=427&top_left_y=1103&top_left_x=820) + +Note that + +$$ +\bar{a}^{\prime}=\frac{a(b \bar{c}-\bar{b} c)}{(b-c) a-(\bar{b}-\bar{c}) \bar{a}} . +$$ + +Thus it suffices to prove + +This is equivalent to + +$$ +0=\operatorname{det}\left[\begin{array}{lll} +\bar{a}(\bar{b} c-b \bar{c}) & a(\bar{b} c-b \bar{c}) & (\bar{b}-\bar{c}) \bar{a}-(b-c) a \\ +\bar{b}(\bar{c} a-c \bar{a}) & b(\bar{c} a-c \bar{a}) & (\bar{c}-\bar{a}) \bar{b}-(c-a) b \\ +\bar{c}(\bar{a} b-a \bar{b}) & c(\bar{a} b-a \bar{b}) & (\bar{a}-\bar{b}) \bar{c}-(a-b) c +\end{array}\right] . +$$ + +This determinant has the property that the rows sum to zero, and we're done. +Remark. Alternatively, if you don't notice that you could just blindly expand: + +$$ +\begin{aligned} +& \sum_{\mathrm{cyc}}((\bar{b}-\bar{c}) \bar{a}-(b-c) a) \cdot-\operatorname{det}\left[\begin{array}{ll} +b & \bar{b} \\ +c & \bar{c} +\end{array}\right](\bar{c} a-c \bar{a})(\bar{a} b-a \bar{b}) \\ += & (\bar{b} c-c \bar{b})(\bar{c} a-c \bar{a})(\bar{a} b-a \bar{b}) \sum_{\mathrm{cyc}}(a b-a c+\overline{c a}-\bar{b} \bar{a})=0 +\end{aligned} +$$ + +\I Second solution (Desargues involution). We let $C^{\prime \prime}=\overline{A^{\prime} B^{\prime}} \cap \overline{A B}$. Consider complete quadrilateral $A B C A^{\prime} B^{\prime} C^{\prime \prime} C$. We see that there is an involutive pairing $\tau$ at $P$ swapping $\left(P A, P A^{\prime}\right),\left(P B, P B^{\prime}\right),\left(P C, P C^{\prime \prime}\right)$. From the first two, we see $\tau$ coincides with reflection about $\ell$, hence conclude $C^{\prime \prime}=C$. + +ब Third solution (barycentric), by Catherine $\mathbf{X u}$. We will perform barycentric coordinates on the triangle $P C C^{\prime}$, with $P=(1,0,0), C^{\prime}=(0,1,0)$, and $C=(0,0,1)$. Set $a=C C^{\prime}, b=C P, c=C^{\prime} P$ as usual. Since $A, B, C^{\prime}$ are collinear, we will define $A=(p: k: q)$ and $B=(p: \ell: q)$. + +Claim - Line $\gamma$ is the angle bisector of $\angle A P A^{\prime}, \angle B P B^{\prime}$, and $\angle C P C^{\prime}$. +Proof. Since $A^{\prime} P$ is the reflection of $A P$ across $\gamma$, etc. +Thus $B^{\prime}$ is the intersection of the isogonal of $B$ with respect to $\angle P$ with the line $C A$; that is, + +$$ +B^{\prime}=\left(\frac{p}{k} \frac{b^{2}}{\ell}: \frac{b^{2}}{\ell}: \frac{c^{2}}{q}\right) +$$ + +Analogously, $A^{\prime}$ is the intersection of the isogonal of $A$ with respect to $\angle P$ with the line $C B$; that is, + +$$ +A^{\prime}=\left(\frac{p}{\ell} \frac{b^{2}}{k}: \frac{b^{2}}{k}: \frac{c^{2}}{q}\right) +$$ + +The ratio of the first to third coordinate in these two points is both $b^{2} p q: c^{2} k \ell$, so it follows $A^{\prime}, B^{\prime}$, and $C^{\prime}$ are collinear. + +Remark (Problem reference). The converse of this problem appears as problem 1052 attributed S. V. Markelov in the book Geometriya: 9-11 Klassy: Ot Uchebnoy Zadachi k Tvorcheskoy, 1996, by I. F. Sharygin. + diff --git a/USAJMO/md/en-JMO-2013-notes.md b/USAJMO/md/en-JMO-2013-notes.md new file mode 100644 index 0000000000000000000000000000000000000000..e03a864179f35c8f4ba674064011a981cdbad3df --- /dev/null +++ b/USAJMO/md/en-JMO-2013-notes.md @@ -0,0 +1,256 @@ +# JMO 2013 Solution Notes + +Evan Chen《陳誼廷》 + +15 April 2024 + + +#### Abstract + +This is a compilation of solutions for the 2013 JMO. The ideas of the solution are a mix of my own work, the solutions provided by the competition organizers, and solutions found by the community. However, all the writing is maintained by me. + +These notes will tend to be a bit more advanced and terse than the "official" solutions from the organizers. In particular, if a theorem or technique is not known to beginners but is still considered "standard", then I often prefer to use this theory anyways, rather than try to work around or conceal it. For example, in geometry problems I typically use directed angles without further comment, rather than awkwardly work around configuration issues. Similarly, sentences like "let $\mathbb{R}$ denote the set of real numbers" are typically omitted entirely. + +Corrections and comments are welcome! + + +## Contents + +0 Problems +1 Solutions to Day 1 +1.1 JMO 2013/1, proposed by Titu Andreescu ..... 3 +1.2 JMO 2013/2, proposed by Sungyoon Kim ..... 4 +1.3 JMO 2013/3, proposed by Zuming Feng +2 Solutions to Day 2 ..... 6 +2.1 JMO 2013/4, proposed by Kiran Kedlaya ..... 6 +2.2 JMO 2013/5, proposed by Zuming Feng ..... 7 +2.3 JMO 2013/6, proposed by Titu Andreescu ..... 8 + +## §0 Problems + +1. Are there integers $a$ and $b$ such that $a^{5} b+3$ and $a b^{5}+3$ are both perfect cubes of integers? +2. Each cell of an $m \times n$ board is filled with some nonnegative integer. Two numbers in the filling are said to be adjacent if their cells share a common side. The filling is called a garden if it satisfies the following two conditions: +(i) The difference between any two adjacent numbers is either 0 or 1 . +(ii) If a number is less than or equal to all of its adjacent numbers, then it is equal to 0. + +Determine the number of distinct gardens in terms of $m$ and $n$. +3. In triangle $A B C$, points $P, Q, R$ lie on sides $B C, C A, A B$, respectively. Let $\omega_{A}$, $\omega_{B}, \omega_{C}$ denote the circumcircles of triangles $A Q R, B R P, C P Q$, respectively. Given the fact that segment $A P$ intersects $\omega_{A}, \omega_{B}, \omega_{C}$ again at $X, Y, Z$ respectively, prove that $Y X / X Z=B P / P C$. +4. Let $f(n)$ be the number of ways to write $n$ as a sum of powers of 2 , where we keep track of the order of the summation. For example, $f(4)=6$ because 4 can be written as $4,2+2,2+1+1,1+2+1,1+1+2$, and $1+1+1+1$. Find the smallest $n$ greater than 2013 for which $f(n)$ is odd. +5. Quadrilateral $X A B Y$ is inscribed in the semicircle $\omega$ with diameter $\overline{X Y}$. Segments $A Y$ and $B X$ meet at $P$. Point $Z$ is the foot of the perpendicular from $P$ to line $\overline{X Y}$. Point $C$ lies on $\omega$ such that line $X C$ is perpendicular to line $A Z$. Let $Q$ be the intersection of segments $A Y$ and $X C$. Prove that + +$$ +\frac{B Y}{X P}+\frac{C Y}{X Q}=\frac{A Y}{A X} +$$ + +6. Find all real numbers $x, y, z \geq 1$ satisfying + +$$ +\min (\sqrt{x+x y z}, \sqrt{y+x y z}, \sqrt{z+x y z})=\sqrt{x-1}+\sqrt{y-1}+\sqrt{z-1} . +$$ + +## §1 Solutions to Day 1 + +## §1.1 JMO 2013/1, proposed by Titu Andreescu + +Available online at https://aops.com/community/p3041819. + +## Problem statement + +Are there integers $a$ and $b$ such that $a^{5} b+3$ and $a b^{5}+3$ are both perfect cubes of integers? + +No, there do not exist such $a$ and $b$. +We prove this in two cases. + +- Assume $3 \mid a b$. WLOG we have $3 \mid a$, but then $a^{5} b+3 \equiv 3(\bmod 9)$, contradiction. +- Assume $3 \nmid a b$. Then $a^{5} b+3$ is a cube not divisible by 3 , so it is $\pm 1 \bmod 9$, and we conclude + +$$ +a^{5} b \in\{5,7\} \quad(\bmod 9) +$$ + +Analogously + +$$ +a b^{5} \in\{5,7\} \quad(\bmod 9) +$$ + +We claim however these two equations cannot hold simultaneously. Indeed $(a b)^{6} \equiv 1$ $(\bmod 9)$ by Euler's theorem, despite $5 \cdot 5 \equiv 7,5 \cdot 7 \equiv 8,7 \cdot 7 \equiv 4 \bmod 9$. + +## §1.2 JMO 2013/2, proposed by Sungyoon Kim + +Available online at https://aops.com/community/p3041818. + +## Problem statement + +Each cell of an $m \times n$ board is filled with some nonnegative integer. Two numbers in the filling are said to be adjacent if their cells share a common side. The filling is called a garden if it satisfies the following two conditions: +(i) The difference between any two adjacent numbers is either 0 or 1 . +(ii) If a number is less than or equal to all of its adjacent numbers, then it is equal to 0 . + +Determine the number of distinct gardens in terms of $m$ and $n$. + +The numerical answer is $2^{m n}-1$. But we claim much more, by giving an explicit description of all gardens: + +Let $S$ be any nonempty subset of the $m n$ cells. Suppose we fill each cell $\theta$ with the minimum (taxicab) distance from $\theta$ to some cell in $S$ (in particular, we write 0 if $\theta \in S$ ). Then + +- This gives a garden, and +- All gardens are of this form. + +Since there are $2^{m n}-1$ such nonempty subsets $S$, this would finish the problem. An example of a garden with $|S|=3$ is shown below. + +$$ +\left[\begin{array}{llllll} +2 & 1 & 2 & 1 & 0 & 1 \\ +1 & 0 & 1 & 2 & 1 & 2 \\ +1 & 1 & 2 & 3 & 2 & 3 \\ +0 & 1 & 2 & 3 & 3 & 4 +\end{array}\right] +$$ + +It is actually fairly easy to see that this procedure always gives a garden; so we focus our attention on showing that every garden is of this form. + +Given a garden, note first that it has at least one cell with a zero in it - by considering the minimum number across the entire garden. Now let $S$ be the (thus nonempty) set of cells with a zero written in them. We contend that this works, i.e. the following sentence holds: + +Claim - If a cell $\theta$ is labeled $d$, then the minimum distance from that cell to a cell in $S$ is $d$. + +Proof. The proof is by induction on $d$, with $d=0$ being by definition. Now, consider any cell $\theta$ labeled $d \geq 1$. Every neighbor of $\theta$ has label at least $d-1$, so any path will necessarily take $d-1$ steps after leaving $\theta$. Conversely, there is some $d-1$ adjacent to $\theta$ by (ii). Stepping on this cell and using the minimal path (by induction hypothesis) gives us a path to a cell in $S$ with length exactly $d$. So the shortest path does indeed have distance $d$, as desired. + +## §1.3 JMO 2013/3, proposed by Zuming Feng + +Available online at https://aops.com/community/p3041822. + +## Problem statement + +In triangle $A B C$, points $P, Q, R$ lie on sides $B C, C A, A B$, respectively. Let $\omega_{A}$, $\omega_{B}, \omega_{C}$ denote the circumcircles of triangles $A Q R, B R P, C P Q$, respectively. Given the fact that segment $A P$ intersects $\omega_{A}, \omega_{B}, \omega_{C}$ again at $X, Y, Z$ respectively, prove that $Y X / X Z=B P / P C$. + +Let $M$ be the concurrence point of $\omega_{A}, \omega_{B}, \omega_{C}$ (by Miquel's theorem). +![](https://cdn.mathpix.com/cropped/2024_11_19_0a837349e2b78b0909e9g-5.jpg?height=704&width=812&top_left_y=873&top_left_x=622) + +Then $M$ is the center of a spiral similarity sending $\overline{Y Z}$ to $\overline{B C}$. So it suffices to show that this spiral similarity also sends $X$ to $P$, but + +$$ +\measuredangle M X Y=\measuredangle M X A=\measuredangle M R A=\measuredangle M R B=\measuredangle M P B +$$ + +so this follows. + +## §2 Solutions to Day 2 + +## §2.1 JMO 2013/4, proposed by Kiran Kedlaya + +Available online at https://aops.com/community/p3043748. + +## Problem statement + +Let $f(n)$ be the number of ways to write $n$ as a sum of powers of 2 , where we keep track of the order of the summation. For example, $f(4)=6$ because 4 can be written as $4,2+2,2+1+1,1+2+1,1+1+2$, and $1+1+1+1$. Find the smallest $n$ greater than 2013 for which $f(n)$ is odd. + +The answer is 2047. +For convenience, we agree that $f(0)=1$. Then by considering cases on the first number in the representation, we derive the recurrence + +$$ +f(n)=\sum_{k=0}^{\left\lfloor\log _{2} n\right\rfloor} f\left(n-2^{k}\right) +$$ + +We wish to understand the parity of $f$. The first few values are + +$$ +\begin{aligned} +& f(0)=1 \\ +& f(1)=1 \\ +& f(2)=2 \\ +& f(3)=3 \\ +& f(4)=6 \\ +& f(5)=10 \\ +& f(6)=18 \\ +& f(7)=31 . +\end{aligned} +$$ + +Inspired by the data we make the key claim that +Claim - $f(n)$ is odd if and only if $n+1$ is a power of 2. + +Proof. We call a number repetitive if it is zero or its binary representation consists entirely of 1 's. So we want to prove that $f(n)$ is odd if and only if $n$ is repetitive. + +This only takes a few cases: + +- If $n=2^{k}$, then $(\odot)$ has exactly two repetitive terms on the right-hand side, namely 0 and $2^{k}-1$. +- If $n=2^{k}+2^{\ell}-1$, then $(\odot)$ has exactly two repetitive terms on the right-hand side, namely $2^{\ell+1}-1$ and $2^{\ell}-1$. +- If $n=2^{k}-1$, then $(\bigcirc)$ has exactly one repetitive terms on the right-hand side, namely $2^{k-1}-1$. +- For other $n$, there are no repetitive terms at all on the right-hand side of $(\Omega)$. + +Thus the induction checks out. +So the final answer to the problem is 2047. + +## §2.2 JMO 2013/5, proposed by Zuming Feng + +Available online at https://aops.com/community/p3043750. + +## Problem statement + +Quadrilateral $X A B Y$ is inscribed in the semicircle $\omega$ with diameter $\overline{X Y}$. Segments $A Y$ and $B X$ meet at $P$. Point $Z$ is the foot of the perpendicular from $P$ to line $\overline{X Y}$. Point $C$ lies on $\omega$ such that line $X C$ is perpendicular to line $A Z$. Let $Q$ be the intersection of segments $A Y$ and $X C$. Prove that + +$$ +\frac{B Y}{X P}+\frac{C Y}{X Q}=\frac{A Y}{A X} +$$ + +Let $\beta=\angle Y X P$ and $\alpha=\angle P Y X$ and set $X Y=1$. We do not direct angles in the following solution. +![](https://cdn.mathpix.com/cropped/2024_11_19_0a837349e2b78b0909e9g-7.jpg?height=552&width=1012&top_left_y=1046&top_left_x=525) + +Observe that + +$$ +\angle A Z X=\angle A P X=\alpha+\beta +$$ + +since $A P Z X$ is cyclic. In particular, $\angle C X Y=90^{\circ}-(\alpha+\beta)$. It is immediate that + +$$ +B Y=\sin \beta, \quad C Y=\cos (\alpha+\beta), \quad A Y=\cos \alpha, \quad A X=\sin \alpha +$$ + +The Law of Sines on $\triangle X P Y$ gives $X P=X Y \frac{\sin \alpha}{\sin (\alpha+\beta)}$, and on $\triangle X Q Y$ gives $X Q=$ $X Y \frac{\sin \alpha}{\sin (90+\beta)}=\frac{\sin \alpha}{\cos \beta}$. So, the given is equivalent to + +$$ +\frac{\sin \beta}{\frac{\sin \alpha}{\sin (\alpha+\beta)}}+\frac{\cos (\alpha+\beta)}{\frac{\sin \alpha}{\cos \beta}}=\frac{\cos \alpha}{\sin \alpha} +$$ + +which is equivalent to $\cos \alpha=\cos \beta \cos (\alpha+\beta)+\sin \beta \sin (\alpha+\beta)$. This is obvious, because the right-hand side is just $\cos ((\alpha+\beta)-\beta)$. + +## §2.3 JMO 2013/6, proposed by Titu Andreescu + +Available online at https://aops.com/community/p3043752. + +## Problem statement + +Find all real numbers $x, y, z \geq 1$ satisfying + +$$ +\min (\sqrt{x+x y z}, \sqrt{y+x y z}, \sqrt{z+x y z})=\sqrt{x-1}+\sqrt{y-1}+\sqrt{z-1} +$$ + +Set $x=1+a, y=1+b, z=1+c$ which eliminates the $x, y, z \geq 1$ condition. Assume without loss of generality that $a \leq b \leq c$. Then the given equation rewrites as + +$$ +\sqrt{(1+a)(1+(1+b)(1+c))}=\sqrt{a}+\sqrt{b}+\sqrt{c} +$$ + +In fact, we are going to prove the left-hand side always exceeds the right-hand side, and then determine the equality cases. We have: + +$$ +\begin{aligned} +(1+a)(1+(1+b)(1+c)) & =(a+1)(1+(b+1)(1+c)) \\ +& \leq(a+1)\left(1+(\sqrt{b}+\sqrt{c})^{2}\right) \\ +& \leq(\sqrt{a}+(\sqrt{b}+\sqrt{c}))^{2} +\end{aligned} +$$ + +by two applications of Cauchy-Schwarz. +Equality holds if $b c=1$ and $1 / a=\sqrt{b}+\sqrt{c}$. Letting $c=t^{2}$ for $t \geq 1$, we recover $b=t^{-2} \leq t^{2}$ and $a=\frac{1}{t+1 / t} \leq t^{2}$. + +Hence the solution set is + +$$ +(x, y, z)=\left(1+\left(\frac{t}{t^{2}+1}\right)^{2}, 1+\frac{1}{t^{2}}, 1+t^{2}\right) +$$ + +and permutations, for any $t>0$. + diff --git a/USAJMO/md/en-JMO-2014-notes.md b/USAJMO/md/en-JMO-2014-notes.md new file mode 100644 index 0000000000000000000000000000000000000000..72e8a204596b476eb71eb97d1106044347c4bd99 --- /dev/null +++ b/USAJMO/md/en-JMO-2014-notes.md @@ -0,0 +1,268 @@ +# JMO 2014 Solution Notes + +Evan Chen《陳誼廷》 + +15 April 2024 + + +#### Abstract + +This is a compilation of solutions for the 2014 JMO. The ideas of the solution are a mix of my own work, the solutions provided by the competition organizers, and solutions found by the community. However, all the writing is maintained by me. + +These notes will tend to be a bit more advanced and terse than the "official" solutions from the organizers. In particular, if a theorem or technique is not known to beginners but is still considered "standard", then I often prefer to use this theory anyways, rather than try to work around or conceal it. For example, in geometry problems I typically use directed angles without further comment, rather than awkwardly work around configuration issues. Similarly, sentences like "let $\mathbb{R}$ denote the set of real numbers" are typically omitted entirely. + +Corrections and comments are welcome! + + +## Contents + +0 Problems +1 Solutions to Day 1 +1.1 JMO 2014/1, proposed by Titu Andreescu ..... 3 +1.2 JMO 2014/2, proposed by Zuming Feng ..... 4 +1.3 JMO 2014/3, proposed by Titu Andreescu +2 Solutions to Day 2 +2.1 JMO 2014/4, proposed by Palmer Mebane ..... 8 +2.2 JMO 2014/5, proposed by Palmer Mebane +2.3 JMO 2014/6, proposed by Titu Andreescu, Cosmin Pohoata ..... 11 + +## §0 Problems + +1. Let $a, b, c$ be real numbers greater than or equal to 1 . Prove that + +$$ +\min \left(\frac{10 a^{2}-5 a+1}{b^{2}-5 b+10}, \frac{10 b^{2}-5 b+1}{c^{2}-5 c+10}, \frac{10 c^{2}-5 c+1}{a^{2}-5 a+10}\right) \leq a b c +$$ + +2. Let $\triangle A B C$ be a non-equilateral, acute triangle with $\angle A=60^{\circ}$, and let $O$ and $H$ denote the circumcenter and orthocenter of $\triangle A B C$, respectively. +(a) Prove that line $O H$ intersects both segments $A B$ and $A C$ at two points $P$ and $Q$, respectively. +(b) Denote by $s$ and $t$ the respective areas of triangle $A P Q$ and quadrilateral $B P Q C$. Determine the range of possible values for $s / t$. +3. Find all $f: \mathbb{Z} \rightarrow \mathbb{Z}$ such that + +$$ +x f(2 f(y)-x)+y^{2} f(2 x-f(y))=\frac{f(x)^{2}}{x}+f(y f(y)) +$$ + +for all $x, y \in \mathbb{Z}$ such that $x \neq 0$. +4. Let $b \geq 2$ be a fixed integer, and let $s_{b}(n)$ denote the sum of the base- $b$ digits of $n$. Show that there are infinitely many positive integers that cannot be represented in the from $n+s_{b}(n)$ where $n$ is a positive integer. +5. Let $k$ be a positive integer. Two players $A$ and $B$ play a game on an infinite grid of regular hexagons. Initially all the grid cells are empty. Then the players alternately take turns with $A$ moving first. In her move, $A$ may choose two adjacent hexagons in the grid which are empty and place a counter in both of them. In his move, $B$ may choose any counter on the board and remove it. If at any time there are $k$ consecutive grid cells in a line all of which contain a counter, $A$ wins. Find the minimum value of $k$ for which $A$ cannot win in a finite number of moves, or prove that no such minimum value exists. +6. Let $A B C$ be a triangle with incenter $I$, incircle $\gamma$ and circumcircle $\Gamma$. Let $M, N, P$ be the midpoints of $\overline{B C}, \overline{C A}, \overline{A B}$ and let $E, F$ be the tangency points of $\gamma$ with $\overline{C A}$ and $\overline{A B}$, respectively. Let $U, V$ be the intersections of line $E F$ with line $M N$ and line $M P$, respectively, and let $X$ be the midpoint of $\operatorname{arc} B A C$ of $\Gamma$. +(a) Prove that $I$ lies on ray $C V$. +(b) Prove that line $X I$ bisects $\overline{U V}$. + +## §1 Solutions to Day 1 + +## §1.1 JMO 2014/1, proposed by Titu Andreescu + +Available online at https://aops.com/community/p3477681. + +## Problem statement + +Let $a, b, c$ be real numbers greater than or equal to 1 . Prove that + +$$ +\min \left(\frac{10 a^{2}-5 a+1}{b^{2}-5 b+10}, \frac{10 b^{2}-5 b+1}{c^{2}-5 c+10}, \frac{10 c^{2}-5 c+1}{a^{2}-5 a+10}\right) \leq a b c +$$ + +Notice that + +$$ +\frac{10 a^{2}-5 a+1}{a^{2}-5 a+10} \leq a^{3} +$$ + +since it rearranges to $(a-1)^{5} \geq 0$. Cyclically multiply to get + +$$ +\prod_{\mathrm{cyc}}\left(\frac{10 a^{2}-5 a+1}{b^{2}-5 b+10}\right) \leq(a b c)^{3} +$$ + +and the minimum is at most the geometric mean. + +## §1.2 JMO 2014/2, proposed by Zuming Feng + +Available online at https://aops.com/community/p3477702. + +## Problem statement + +Let $\triangle A B C$ be a non-equilateral, acute triangle with $\angle A=60^{\circ}$, and let $O$ and $H$ denote the circumcenter and orthocenter of $\triangle A B C$, respectively. +(a) Prove that line $O H$ intersects both segments $A B$ and $A C$ at two points $P$ and $Q$, respectively. +(b) Denote by $s$ and $t$ the respective areas of triangle $A P Q$ and quadrilateral $B P Q C$. Determine the range of possible values for $s / t$. + +We begin with some synthetic work. Let $I$ denote the incenter, and recall ("fact 5") that the arc midpoint $M$ is the center of $(B I C)$, which we denote by $\gamma$. + +Now we have that + +$$ +\angle B O C=\angle B I C=\angle B H C=120^{\circ} . +$$ + +Since all three centers lie inside $A B C$ (as it was acute), and hence on the opposite side of $\overline{B C}$ as $M$, it follows that $O, I, H$ lie on minor arc $B C$ of $\gamma$. + +We note this implies (a) already, as line $O H$ meets line $B C$ outside of segment $B C$. +![](https://cdn.mathpix.com/cropped/2024_11_19_3739490e48f1a6399d35g-04.jpg?height=806&width=552&top_left_y=1339&top_left_x=752) + +Claim - Triangle $A P Q$ is equilateral with side length $\frac{b+c}{3}$. + +Proof. Let $R$ be the circumradius. We have $R=O M=O A=M H$, and even $A H=$ $2 R \cos A=R$, so $A O M H$ is a rhombus. Thus $\overline{O H} \perp \overline{A M}$ and in this way we derive that $\triangle A P Q$ is isosceles, hence equilateral. + +Finally, since $\angle P B H=30^{\circ}$, and $\angle B P H=120^{\circ}$, it follows that $\triangle B P H$ is isosceles and $B P=P H$. Similarly, $C Q=Q H$. So $b+c=A P+B P+A Q+Q C=A P+A Q+P Q$ as needed. + +Finally, we turn to the boring task of extracting the numerical answer. We have + +$$ +\frac{s}{s+t}=\frac{[A P Q]}{[A B C]}=\frac{\frac{\sqrt{3}}{4}\left(\frac{b+c}{3}\right)^{2}}{\frac{\sqrt{3}}{4} b c}=\frac{b^{2}+2 b c+c^{2}}{9 b c}=\frac{1}{9}\left(2+\frac{b}{c}+\frac{c}{b}\right) . +$$ + +So the problem is reduced to analyzing the behavior of $b / c$. For this, we imagine fixing $\Gamma$ the circumcircle of $A B C$, as well as the points $B$ and $C$. Then as we vary $A$ along the "topmost" arc of measure $120^{\circ}$, we find $b / c$ is monotonic with values $1 / 2$ and 2 at endpoints, and by continuity all values $b / c \in(1 / 2,2)$ can be achieved. + +So + +$$ +\frac{1}{2}<\frac{b}{c}<2 \Longrightarrow 4 / 9<\frac{s}{s+t}<1 / 2 \Longrightarrow 4 / 5<\frac{s}{t}<1 +$$ + +as needed. + +## §1.3 JMO 2014/3, proposed by Titu Andreescu + +Available online at https://aops.com/community/p3477690. + +## Problem statement + +Find all $f: \mathbb{Z} \rightarrow \mathbb{Z}$ such that + +$$ +x f(2 f(y)-x)+y^{2} f(2 x-f(y))=\frac{f(x)^{2}}{x}+f(y f(y)) +$$ + +for all $x, y \in \mathbb{Z}$ such that $x \neq 0$. + +The answer is $f(x) \equiv 0$ and $f(x) \equiv x^{2}$. Check that these work. +Now let's prove these are the only solutions. Put $y=0$ to obtain + +$$ +x f(2 f(0)-x)=\frac{f(x)^{2}}{x}+f(0) . +$$ + +The nicest part of the problem is the following step: + +$$ +\text { Claim - We have } f(0)=0 +$$ + +Proof. If not, select a prime $p \nmid f(0)$ and put $x=p \neq 0$. In the above, we find that $p \mid f(p)^{2}$, so $p \mid f(p)$ and hence $p \left\lvert\, \frac{f(p)^{2}}{p}\right.$. From here we derive $p \mid f(0)$, contradiction. Hence + +$$ +f(0)=0 +$$ + +Claim - We have $f(x) \in\left\{0, x^{2}\right\}$ for each individual $x$. +Proof. The above then implies that + +$$ +x^{2} f(-x)=f(x)^{2} +$$ + +holds for all nonzero $x$, but also for $x=0$. Let us now check that $f$ is an even function. In the above, we may also derive $f(-x)^{2}=x^{2} f(x)$. If $f(x) \neq f(-x)$ (and hence $x \neq 0$ ), then subtracting the above and factoring implies that $f(x)+f(-x)=-x^{2}$; we can then obtain by substituting the relation + +$$ +\left[f(x)+\frac{1}{2} x^{2}\right]^{2}=-\frac{3}{4} x^{4}<0 +$$ + +which is impossible. This means $f(x)^{2}=x^{2} f(x)$, thus + +$$ +f(x) \in\left\{0, x^{2}\right\} \quad \forall x . +$$ + +Now suppose there exists a nonzero integer $t$ with $f(t)=0$. We will prove that $f(x) \equiv 0$. Put $y=t$ in the given to obtain that + +$$ +t^{2} f(2 x)=0 +$$ + +for any integer $x \neq 0$, and hence conclude that $f(2 \mathbb{Z}) \equiv 0$. Then selecting $x=2 k \neq 0$ in the given implies that + +$$ +y^{2} f(4 k-f(y))=f(y f(y)) +$$ + +Assume for contradiction that $f(m)=m^{2}$ now for some odd $m \neq 0$. Evidently + +$$ +m^{2} f\left(4 k-m^{2}\right)=f\left(m^{3}\right) +$$ + +If $f\left(m^{3}\right) \neq 0$ this forces $f\left(4 k-m^{2}\right) \neq 0$, and hence $m^{2}\left(4 k-m^{2}\right)^{2}=m^{6}$ for arbitrary $k \neq 0$, which is clearly absurd. That means + +$$ +f\left(4 k-m^{2}\right)=f\left(m^{2}-4 k\right)=f\left(m^{3}\right)=0 +$$ + +for each $k \neq 0$. Since $m$ is odd, $m^{2} \equiv 1(\bmod 4)$, and so $f(n)=0$ for all $n$ other than $\pm m^{2}$ (since we cannot select $\left.k=0\right)$. + +Now $f(m)=m^{2}$ means that $m= \pm 1$. Hence either $f(x) \equiv 0$ or + +$$ +f(x)= \begin{cases}1 & x= \pm 1 \\ 0 & \text { otherwise }\end{cases} +$$ + +To show that the latter fails, we simply take $x=5$ and $y=1$ in the given. +Hence, the only solutions are $f(x) \equiv 0$ and $f(x) \equiv x^{2}$. + +## §2 Solutions to Day 2 + +## §2.1 JMO 2014/4, proposed by Palmer Mebane + +Available online at https://aops.com/community/p3478579. + +## Problem statement + +Let $b \geq 2$ be a fixed integer, and let $s_{b}(n)$ denote the sum of the base- $b$ digits of $n$. Show that there are infinitely many positive integers that cannot be represented in the from $n+s_{b}(n)$ where $n$ is a positive integer. + +For brevity let $f(n)=n+s_{b}(n)$. Select any integer $M$. Observe that $f(x) \geq b^{2 M}$ for any $x \geq b^{2 M}$, but also $f\left(b^{2 M}-k\right) \geq b^{2 M}$ for $k=1,2, \ldots, M$, since the base- $b$ expansion of $b^{2 M}-k$ will start out with at least $M$ digits $b-1$. + +Thus $f$ omits at least $M$ values in $\left[1, b^{2 M}\right]$ for any $M$. + +## §2.2 JMO 2014/5, proposed by Palmer Mebane + +Available online at https://aops.com/community/p3478584. + +## Problem statement + +Let $k$ be a positive integer. Two players $A$ and $B$ play a game on an infinite grid of regular hexagons. Initially all the grid cells are empty. Then the players alternately take turns with $A$ moving first. In her move, $A$ may choose two adjacent hexagons in the grid which are empty and place a counter in both of them. In his move, $B$ may choose any counter on the board and remove it. If at any time there are $k$ consecutive grid cells in a line all of which contain a counter, $A$ wins. Find the minimum value of $k$ for which $A$ cannot win in a finite number of moves, or prove that no such minimum value exists. + +The answer is $k=6$. +【 Proof that $A$ cannot win if $k=6$. We give a strategy for $B$ to prevent $A$ 's victory. Shade in every third cell, as shown in the figure below. Then $A$ can never cover two shaded cells simultaneously on her turn. Now suppose $B$ always removes a counter on a shaded cell (and otherwise does whatever he wants). Then he can prevent $A$ from ever getting six consecutive counters, because any six consecutive cells contain two shaded cells. +![](https://cdn.mathpix.com/cropped/2024_11_19_3739490e48f1a6399d35g-09.jpg?height=500&width=561&top_left_y=1372&top_left_x=753) + +【 Example of a strategy for $A$ when $k=5$. We describe a winning strategy for $A$ explicitly. Note that after $B$ 's first turn there is one counter, so then $A$ may create an equilateral triangle, and hence after $B$ 's second turn there are two consecutive counters. Then, on her third turn, $A$ places a pair of counters two spaces away on the same line. Label the two inner cells $x$ and $y$ as shown below. +![](https://cdn.mathpix.com/cropped/2024_11_19_3739490e48f1a6399d35g-09.jpg?height=295&width=500&top_left_y=2168&top_left_x=778) + +Now it is $B$ 's turn to move; in order to avoid losing immediately, he must remove either $x$ or $y$. Then on any subsequent turn, $A$ can replace $x$ or $y$ (whichever was removed) and add one more adjacent counter. This continues until either $x$ or $y$ has all its neighbors +filled (we ask $A$ to do so in such a way that she avoids filling in the two central cells between $x$ and $y$ as long as possible). + +So, let's say without loss of generality (by symmetry) that $x$ is completely surrounded by tokens. Again, $B$ must choose to remove $x$ (or $A$ wins on her next turn). After $x$ is removed by $B$, consider the following figure. +![](https://cdn.mathpix.com/cropped/2024_11_19_3739490e48f1a6399d35g-10.jpg?height=398&width=612&top_left_y=503&top_left_x=728) + +We let $A$ play in the two marked green cells. Then, regardless of what move $B$ plays, one of the two choices of moves marked in red lets $A$ win. Thus, we have described a winning strategy when $k=5$ for $A$. + +## §2.3 JMO 2014/6, proposed by Titu Andreescu, Cosmin Pohoata + +Available online at https://aops.com/community/p3478583. + +## Problem statement + +Let $A B C$ be a triangle with incenter $I$, incircle $\gamma$ and circumcircle $\Gamma$. Let $M, N, P$ be the midpoints of $\overline{B C}, \overline{C A}, \overline{A B}$ and let $E, F$ be the tangency points of $\gamma$ with $\overline{C A}$ and $\overline{A B}$, respectively. Let $U, V$ be the intersections of line $E F$ with line $M N$ and line $M P$, respectively, and let $X$ be the midpoint of $\operatorname{arc} B A C$ of $\Gamma$. +(a) Prove that $I$ lies on ray $C V$. +(b) Prove that line $X I$ bisects $\overline{U V}$. + +The fact that $I=\overline{B U} \cap \overline{C V}$ and is Lemma 1.45 from EGMO. +As for (b), we note: +Claim - Line $I X$ is a symmedian of $\triangle I B C$. +Proof. Recall that $(B I C)$ has circumcenter coinciding with the antipode of $X$ (by "Fact 5 "). So this follows from the fact that $\overline{X B}$ and $\overline{X C}$ are tangent. + +Since $B V U C$ is cyclic with diagonals intersecting at $I$, and $I X$ is symmedian of $\triangle I B C$, it is median of $\triangle I U V$, as needed. + +Remark (Alternate solution to (b) by Gunmay Handa). It's well known that $X$ is the midpoint of $\overline{I_{b} I_{c}}$ (by considering the nine-point circle of the excentral triangle). However, $\overline{U V} \| \overline{I_{b} I_{c}}$ and $I=\overline{I_{b} U} \cap \overline{I_{c} V}$, implying the result. + diff --git a/USAJMO/md/en-JMO-2015-notes.md b/USAJMO/md/en-JMO-2015-notes.md new file mode 100644 index 0000000000000000000000000000000000000000..b05f0e8716dca80a9d7dc277d7e81d3accefaa50 --- /dev/null +++ b/USAJMO/md/en-JMO-2015-notes.md @@ -0,0 +1,371 @@ +# JMO 2015 Solution Notes + +Evan Chen《陳誼廷》 + +15 April 2024 + + +#### Abstract + +This is a compilation of solutions for the 2015 JMO. The ideas of the solution are a mix of my own work, the solutions provided by the competition organizers, and solutions found by the community. However, all the writing is maintained by me. + +These notes will tend to be a bit more advanced and terse than the "official" solutions from the organizers. In particular, if a theorem or technique is not known to beginners but is still considered "standard", then I often prefer to use this theory anyways, rather than try to work around or conceal it. For example, in geometry problems I typically use directed angles without further comment, rather than awkwardly work around configuration issues. Similarly, sentences like "let $\mathbb{R}$ denote the set of real numbers" are typically omitted entirely. + +Corrections and comments are welcome! + + +## Contents + +0 Problems ..... 2 +1 Solutions to Day 1 +1.1 JMO 2015/1, proposed by Razvan Gelca ..... 3 +1.2 JMO 2015/2, proposed by Titu Andreescu +1.3 JMO 2015/3, proposed by Zuming Feng, Jacek Fabrykowski ..... 6 +2 Solutions to Day 2 +2.1 JMO 2015/4, proposed by Iurie Boreico ..... 9 +2.2 JMO 2015/5, proposed by Sungyoon Kim ..... 10 +2.3 JMO 2015/6, proposed by Maria Monks Gillespie ..... 11 + +## §0 Problems + +1. Given a sequence of real numbers, a move consists of choosing two terms and replacing each with their arithmetic mean. Show that there exists a sequence of 2015 distinct real numbers such that after one initial move is applied to the sequence - no matter what move - there is always a way to continue with a finite sequence of moves so as to obtain in the end a constant sequence. +2. Solve in integers the equation + +$$ +x^{2}+x y+y^{2}=\left(\frac{x+y}{3}+1\right)^{3} . +$$ + +3. Quadrilateral $A P B Q$ is inscribed in circle $\omega$ with $\angle P=\angle Q=90^{\circ}$ and $A P=$ $A Q0$ + +$$ +\begin{aligned} +f(a)+f(a+3 d) & =f(a+d)+f(a+2 d) \\ +f(a-d)+f(a+2 d) & =f(a)+f(a+d) +\end{aligned} +$$ + +which imply + +$$ +f(a-d)+f(a+3 d)=2 f(a+d) +$$ + +Thus we conclude that for arbitrary $x$ and $y$ we have + +$$ +f(x)+f(y)=2 f\left(\frac{x+y}{2}\right) +$$ + +thus $f$ satisfies Jensen functional equation over $\mathbb{Q}$, so linear. +The solution can be made to avoid appealing to Jensen's functional equation; here is a presentation of such a solution based on the official ones. Let $d>0$ be a positive integer, and let $n$ be an integer. Consider the two equations + +$$ +\begin{aligned} +& f\left(\frac{2 n-1}{2 d}\right)+f\left(\frac{2 n+2}{2 d}\right)=f\left(\frac{2 n}{2 d}\right)+f\left(\frac{2 n+1}{2 d}\right) \\ +& f\left(\frac{2 n-2}{2 d}\right)+f\left(\frac{2 n+1}{2 d}\right)=f\left(\frac{2 n-1}{2 d}\right)+f\left(\frac{2 n}{2 d}\right) +\end{aligned} +$$ + +Summing them and simplifying implies that + +$$ +f\left(\frac{n-1}{d}\right)+f\left(\frac{n+1}{d}\right)=2 f\left(\frac{n}{d}\right) +$$ + +or equivalently + +$$ +f\left(\frac{n}{d}\right)-f\left(\frac{n-1}{d}\right)=f\left(\frac{n+1}{d}\right)-f\left(\frac{n}{d}\right) . +$$ + +This implies that on the set of rational numbers with denominator dividing $d$, the function $f$ is linear. + +In particular, we should have $f\left(\frac{n}{d}\right)=f(0)+\frac{n}{d} f(1)$ since $\frac{n}{d}, 0,1$ have denominators dividing $d$. This is the same as saying $f(q)=f(0)+q(f(1)-f(0))$ for any $q \in \mathbb{Q}$, which is what we wanted to prove. + +## §2.2 JMO 2015/5, proposed by Sungyoon Kim + +Available online at https://aops.com/community/p4774099. + +## Problem statement + +Let $A B C D$ be a cyclic quadrilateral. Prove that there exists a point $X$ on segment $\overline{B D}$ such that $\angle B A C=\angle X A D$ and $\angle B C A=\angle X C D$ if and only if there exists a point $Y$ on segment $\overline{A C}$ such that $\angle C B D=\angle Y B A$ and $\angle C D B=\angle Y D A$. + +Both conditions are equivalent to $A B C D$ being harmonic. +Here is a complex solution. Extend $U$ and $V$ and shown. Thus $u=b d / a$ and $v=b d / c$. +![](https://cdn.mathpix.com/cropped/2024_11_19_cdde4c7c994bd47d202eg-10.jpg?height=441&width=455&top_left_y=873&top_left_x=800) + +Note $\overline{A V} \cap \overline{C U}$ lies on the perpendicular bisector of $\overline{B D}$ unconditionally. Then $X$ exists as described if and only if the midpoint of $\overline{B D}$ lies on $\overline{A V}$. In complex numbers this is $a+v=m+a v \bar{m}$, or + +$$ +a+\frac{b d}{c}=\frac{b+d}{2}+\frac{a b d}{c} \cdot \frac{b+d}{2 b d} \Longleftrightarrow 2(a c+b d)=(b+d)(a+c) +$$ + +which is symmetric. + +## §2.3 JMO 2015/6, proposed by Maria Monks Gillespie + +Available online at https://aops.com/community/p4774079. + +## Problem statement + +Steve is piling $m \geq 1$ indistinguishable stones on the squares of an $n \times n$ grid. Each square can have an arbitrarily high pile of stones. After he finished piling his stones in some manner, he can then perform stone moves, defined as follows. Consider any four grid squares, which are corners of a rectangle, i.e. in positions $(i, k),(i, l),(j, k)$, $(j, l)$ for some $1 \leq i, j, k, l \leq n$, such that $i0$. + +Claim - The circumcenter has coordinates $\left(\frac{c-b}{2}, \frac{a}{2}-\frac{b c}{2 a}\right)$. + +Proof. This is a known lemma but but we reproduce its proof for completeness. It uses the following steps: + +- By power of a point, the second intersection of line $A H$ with the circumcircle is $\left(0,-\frac{b c}{a}\right)$. +- Since the orthocenter is the reflection of this point across line $B C$, the orthocenter is given exactly by $\left(0, \frac{b c}{a}\right)$. +- The centroid is is $\frac{\vec{A}+\vec{B}+\vec{C}}{3}=\left(\frac{c-b}{3}, \frac{a}{3}\right)$. +- Since $\vec{H}-\vec{O}=3(\vec{G}-\vec{O})$ according to the Euler line, we have $\vec{O}=\frac{3}{2} \vec{G}-\frac{1}{2} \vec{H}$. This gives the desired formula. + +Note that $H Q=\frac{H A \cdot H C}{A C}=\frac{a c}{\sqrt{a^{2}+c^{2}}}$. If we let $T$ be the foot from $Q$ to $B C$, then $\triangle H Q T \tilde{+} \triangle A H C$ and so the $x$-coordinate of $Q$ is given by $H Q \cdot \frac{A H}{A C}=\frac{a^{2} c}{a^{2}+c^{2}}$. Repeating the analogous calculation for $Q$ and $P$ gives + +$$ +\begin{aligned} +Q & =\left(\frac{a^{2} c}{a^{2}+c^{2}}, \frac{a c^{2}}{a^{2}+c^{2}}\right) \\ +P & =\left(-\frac{a^{2} b}{a^{2}+b^{2}}, \frac{a b^{2}}{a^{2}+b^{2}}\right) . +\end{aligned} +$$ + +Then, $O, P, Q$ are collinear if and only if the following shoelace determinant vanishes (with denominators cleared out): + +$$ +\begin{aligned} +0 & =\operatorname{det}\left[\begin{array}{ccc} +-a^{2} b & a b^{2} & a^{2}+b^{2} \\ +a^{2} c & a c^{2} & a^{2}+c^{2} \\ +a(c-b) & a^{2}-b c & 2 a +\end{array}\right]=a \operatorname{det}\left[\begin{array}{ccc} +-a b & a b^{2} & a^{2}+b^{2} \\ +a c & a c^{2} & a^{2}+c^{2} \\ +c-b & a^{2}-b c & 2 a +\end{array}\right] \\ +& =a \operatorname{det}\left[\begin{array}{ccc} +-a(b+c) & a\left(b^{2}-c^{2}\right) & b^{2}-c^{2} \\ +a c & a c^{2} & a^{2}+c^{2} \\ +c-b & a^{2}-b c & 2 a +\end{array}\right]=a(b+c) \operatorname{det}\left[\begin{array}{ccc} +-a & a(b-c) & b-c \\ +a c & a c^{2} & a^{2}+c^{2} \\ +c-b & a^{2}-b c & 2 a +\end{array}\right] \\ +& =a(b+c) \cdot\left[-a\left(a^{2} c^{2}-a^{4}+b c\left(a^{2}+c^{2}\right)\right)+a c(b-c)\left(-a^{2}-b c\right)-(b-c)^{2} \cdot a^{3}\right] \\ +& =a^{2}(b+c)\left(a^{4}-a^{2} b^{2}-b^{2} c^{2}-c^{2} a^{2}\right) . +\end{aligned} +$$ + +On the other hand, + +$$ +\begin{aligned} +A H^{2} & =a^{2} \\ +2 A O^{2} & =2\left[\left(\frac{c-b}{2}\right)^{2}+\left(-\frac{a}{2}-\frac{b c}{2 a}\right)^{2}\right]=\frac{a^{2}+b^{2}+c^{2}+\frac{b^{2} c^{2}}{a^{2}}}{2} \\ +\Longrightarrow A H^{2}-2 A O^{2} & =\frac{1}{2}\left(a^{2}-b^{2}-c^{2}-\frac{b^{2} c^{2}}{a^{2}}\right) . +\end{aligned} +$$ + +So the conditions are equivalent. + +## §2.3 JMO 2016/6, proposed by Titu Andreescu + +Available online at https://aops.com/community/p6220308. + +## Problem statement + +Find all functions $f: \mathbb{R} \rightarrow \mathbb{R}$ such that for all real numbers $x$ and $y$, + +$$ +(f(x)+x y) \cdot f(x-3 y)+(f(y)+x y) \cdot f(3 x-y)=(f(x+y))^{2} . +$$ + +We claim that the only two functions satisfying the requirements are $f(x) \equiv 0$ and $f(x) \equiv x^{2}$. These work. + +First, taking $x=y=0$ in the given yields $f(0)=0$, and then taking $x=0$ gives $f(y) f(-y)=f(y)^{2}$. So also $f(-y)^{2}=f(y) f(-y)$, from which we conclude $f$ is even. Then taking $x=-y$ gives + +$$ +\forall x \in \mathbb{R}: \quad f(x)=x^{2} \quad \text { or } \quad f(4 x)=0 +$$ + +for all $x$. +Remark. Note that an example of a function satisfying $(\boldsymbol{\star})$ is + +$$ +f(x)= \begin{cases}x^{2} & \text { if }|x|<1 \\ 1-\cos \left(\frac{\pi}{2} \cdot x^{1337}\right) & \text { if } 1 \leq|x|<4 \\ 0 & \text { if }|x| \geq 4\end{cases} +$$ + +So, yes, we are currently in a world of trouble, still. (This function is even continuous; I bring this up to emphasize that "continuity" is completely unrelated to the issue at hand.) + +Now we claim + +$$ +\text { Claim }-f(z)=0 \Longleftrightarrow f(2 z)=0 +$$ + +Proof. Let $(x, y)=(3 t, t)$ in the given to get + +$$ +\left(f(t)+3 t^{2}\right) f(8 t)=f(4 t)^{2} +$$ + +Now if $f(4 t) \neq 0$ (in particular, $t \neq 0$ ), then $f(8 t) \neq 0$. Thus we have $(\boldsymbol{\phi})$ in the reverse direction. + +Then $f(4 t) \neq 0 \xlongequal{(\star)} f(t)=t^{2} \neq 0 \xlongequal{(\bullet)} f(2 t) \neq 0$ implies the forwards direction, the last step being the reverse direction + +By putting together $(\boldsymbol{\star})$ and $(\boldsymbol{\leftrightarrow})$ we finally get + +$$ +\forall x \in \mathbb{R}: \quad f(x)=x^{2} \quad \text { or } \quad f(x)=0 +$$ + +We are now ready to approach the main problem. Assume there's an $a \neq 0$ for which $f(a)=0$; we show that $f \equiv 0$. + +Let $b \in \mathbb{R}$ be given. Since $f$ is even, we can assume without loss of generality that $a, b>0$. Also, note that $f(x) \geq 0$ for all $x$ by ( $($ ). By using ( $\boldsymbol{\sim}$ ) we can generate $c>b$ such that $f(c)=0$ by taking $c=2^{n} a$ for a large enough integer $n$. Now, select $x, y>0$ such that $x-3 y=b$ and $x+y=c$. That is, + +$$ +(x, y)=\left(\frac{3 c+b}{4}, \frac{c-b}{4}\right) . +$$ + +Substitution into the original equation gives + +$$ +0=(f(x)+x y) f(b)+(f(y)+x y) f(3 x-y) \geq(f(x)+x y) f(b) +$$ + +But since $f(b) \geq 0$, it follows $f(b)=0$, as desired. + diff --git a/USAJMO/md/en-JMO-2017-notes.md b/USAJMO/md/en-JMO-2017-notes.md new file mode 100644 index 0000000000000000000000000000000000000000..effdcca7c361d4887fa57a5361f7d57a9767ba61 --- /dev/null +++ b/USAJMO/md/en-JMO-2017-notes.md @@ -0,0 +1,305 @@ +# JMO 2017 Solution Notes + +Evan Chen《陳誼廷》 + +15 April 2024 + + +#### Abstract + +This is a compilation of solutions for the 2017 JMO. The ideas of the solution are a mix of my own work, the solutions provided by the competition organizers, and solutions found by the community. However, all the writing is maintained by me. + +These notes will tend to be a bit more advanced and terse than the "official" solutions from the organizers. In particular, if a theorem or technique is not known to beginners but is still considered "standard", then I often prefer to use this theory anyways, rather than try to work around or conceal it. For example, in geometry problems I typically use directed angles without further comment, rather than awkwardly work around configuration issues. Similarly, sentences like "let $\mathbb{R}$ denote the set of real numbers" are typically omitted entirely. + +Corrections and comments are welcome! + + +## Contents + +0 Problems +1 Solutions to Day 1 +1.1 JMO 2017/1, proposed by Gregory Galperin ..... 3 +1.2 JMO 2017/2, proposed by Titu Andreescu ..... 4 +1.3 JMO 2017/3, proposed by Titu Andreescu, Luis Gonzalez, Cosmin Pohoata +2 Solutions to Day 2 ..... 6 +2.1 JMO 2017/4, proposed by Titu Andreescu +2.2 JMO 2017/5, proposed by Ivan Borsenco +2.3 JMO 2017/6, proposed by Maria Monks + +## §0 Problems + +1. Prove that there exist infinitely many pairs of relatively prime positive integers $a, b>1$ for which $a+b$ divides $a^{b}+b^{a}$. +2. Show that the Diophantine equation + +$$ +\left(3 x^{3}+x y^{2}\right)\left(x^{2} y+3 y^{3}\right)=(x-y)^{7} +$$ + +has infinitely many solutions in positive integers, and characterize all the solutions. +3. Let $A B C$ be an equilateral triangle and $P$ a point on its circumcircle. Set $D=$ $\overline{P A} \cap \overline{B C}, E=\overline{P B} \cap \overline{C A}, F=\overline{P C} \cap \overline{A B}$. Prove that the area of triangle $D E F$ is twice the area of triangle $A B C$. +4. Are there any triples $(a, b, c)$ of positive integers such that $(a-2)(b-2)(c-2)+12$ is a prime number that properly divides the positive number $a^{2}+b^{2}+c^{2}+a b c-2017$ ? +5. Let $O$ and $H$ be the circumcenter and the orthocenter of an acute triangle $A B C$. Points $M$ and $D$ lie on side $B C$ such that $B M=C M$ and $\angle B A D=\angle C A D$. Ray $M O$ intersects the circumcircle of triangle $B H C$ in point $N$. Prove that $\angle A D O=\angle H A N$. +6. Let $P_{1}, P_{2}, \ldots, P_{2 n}$ be $2 n$ distinct points on the unit circle $x^{2}+y^{2}=1$, other than $(1,0)$. Each point is colored either red or blue, with exactly $n$ red points and $n$ blue points. Let $R_{1}, R_{2}, \ldots, R_{n}$ be any ordering of the red points. Let $B_{1}$ be the nearest blue point to $R_{1}$ traveling counterclockwise around the circle starting from $R_{1}$. Then let $B_{2}$ be the nearest of the remaining blue points to $R_{2}$ travelling counterclockwise around the circle from $R_{2}$, and so on, until we have labeled all of the blue points $B_{1}, \ldots, B_{n}$. Show that the number of counterclockwise arcs of the form $R_{i} \rightarrow B_{i}$ that contain the point $(1,0)$ is independent of the way we chose the ordering $R_{1}, \ldots, R_{n}$ of the red points. + +## §1 Solutions to Day 1 + +## §1.1 JMO 2017/1, proposed by Gregory Galperin + +Available online at https://aops.com/community/p8108366. + +## Problem statement + +Prove that there exist infinitely many pairs of relatively prime positive integers $a, b>1$ for which $a+b$ divides $a^{b}+b^{a}$. + +One construction: let $d \equiv 1(\bmod 4), d>1$. Let $x=\frac{d^{d}+2^{d}}{d+2}$. Then set + +$$ +a=\frac{x+d}{2}, \quad b=\frac{x-d}{2} +$$ + +To see this works, first check that $b$ is odd and $a$ is even. Let $d=a-b$ be odd. Then: + +$$ +\begin{aligned} +a+b \mid a^{b}+b^{a} & \Longleftrightarrow(-b)^{b}+b^{a} \equiv 0 \quad(\bmod a+b) \\ +& \Longleftrightarrow b^{a-b} \equiv 1 \quad(\bmod a+b) \\ +& \Longleftrightarrow b^{d} \equiv 1 \quad(\bmod d+2 b) \\ +& \Longleftrightarrow(-2)^{d} \equiv d^{d} \quad(\bmod d+2 b) \\ +& \Longleftrightarrow d+2 b \mid d^{d}+2^{d} . +\end{aligned} +$$ + +So it would be enough that + +$$ +d+2 b=\frac{d^{d}+2^{d}}{d+2} \Longrightarrow b=\frac{1}{2}\left(\frac{d^{d}+2^{d}}{d+2}-d\right) +$$ + +which is what we constructed. Also, since $\operatorname{gcd}(x, d)=1$ it follows $\operatorname{gcd}(a, b)=\operatorname{gcd}(d, b)=$ 1. + +Remark. Ryan Kim points out that in fact, $(a, b)=(2 n-1,2 n+1)$ is always a solution. + +## §1.2 JMO 2017/2, proposed by Titu Andreescu + +Available online at https://aops.com/community/p8108503. + +## Problem statement + +Show that the Diophantine equation + +$$ +\left(3 x^{3}+x y^{2}\right)\left(x^{2} y+3 y^{3}\right)=(x-y)^{7} +$$ + +has infinitely many solutions in positive integers, and characterize all the solutions. + +Let $x=d a, y=d b$, where $\operatorname{gcd}(a, b)=1$ and $a>b$. The equation is equivalent to + +$$ +(a-b)^{7} \mid a b\left(a^{2}+3 b^{2}\right)\left(3 a^{2}+b^{2}\right) +$$ + +with the ratio of the two becoming $d$. +Claim - The equation ( $\star$ ) holds if and only if $a-b=1$. +Proof. Obviously if $a-b=1$ then $(\star)$ is true. Conversely, suppose $(\star)$ holds. + +- If $a$ and $b$ are both odd, then $a^{2}+3 b^{2} \equiv 4(\bmod 8)$. Similarly $3 a^{2}+b^{2} \equiv 4(\bmod 8)$. Hence $2^{4}$ exactly divides right-hand side, contradiction. +- Now suppose $a-b$ is odd. We have $\operatorname{gcd}(a-b, a)=\operatorname{gcd}(a-b, b)=1$ by Euclid, but also + +$$ +\operatorname{gcd}\left(a-b, a^{2}+3 b^{2}\right)=\operatorname{gcd}\left(a-b, 4 b^{2}\right)=1 +$$ + +and similarly $\operatorname{gcd}\left(a-b, 3 a^{2}+b^{2}\right)=1$. Thus $a-b$ is coprime to each of $a, b, a^{2}+3 b^{2}$, $3 a^{2}+b^{2}$ and this forces $a-b=1$. + +This therefore describes all solutions: namely, for any $b \geq 1$, if we set $a=b+1$ and $d=a b\left(a^{2}+3 b^{2}\right)\left(3 a^{2}+b\right)$ then $(x, y)=(d a, d b)$ works and any solution is of this form. + +Remark. One can give different cosmetic representations of the same solution set. For example, we could write $b=\frac{1}{2}(n-1)$ and $a=\frac{1}{2}(n+1)$ with $n>1$ any odd integer. Then $d=a b\left(a^{2}+3 b^{2}\right)\left(3 a^{2}+b^{2}\right)=\frac{(n-1)(n+1)\left(n^{2}+n+1\right)\left(n^{2}-n+1\right)}{4}=\frac{n^{6}-1}{4}$, and hence the solution is + +$$ +(x, y)=(d a, d b)=\left(\frac{(n+1)\left(n^{6}-1\right)}{8}, \frac{(n-1)\left(n^{6}-1\right)}{8}\right) +$$ + +which is a little simpler to write. The smallest solutions are $(364,182),(11718,7812), \ldots$ + +## §1.3 JMO 2017/3, proposed by Titu Andreescu, Luis Gonzalez, Cosmin Pohoata + +Available online at https://aops.com/community/p8108450. + +## Problem statement + +Let $A B C$ be an equilateral triangle and $P$ a point on its circumcircle. Set $D=$ $\overline{P A} \cap \overline{B C}, E=\overline{P B} \cap \overline{C A}, F=\overline{P C} \cap \overline{A B}$. Prove that the area of triangle $D E F$ is twice the area of triangle $A B C$. +\ First solution (barycentric). We invoke barycentric coordinates on $A B C$. Let $P=(u: v: w)$, with $u v+v w+w u=0$ (circumcircle equation with $a=b=c$ ). Then $D=(0: v: w), E=(u: 0: w), F=(u: v: 0)$. Hence + +$$ +\begin{aligned} +\frac{[D E F]}{[A B C]} & =\frac{1}{(u+v)(v+w)(w+u)} \operatorname{det}\left[\begin{array}{lll} +0 & v & w \\ +u & 0 & w \\ +u & v & 0 +\end{array}\right] \\ +& =\frac{2 u v w}{(u+v)(v+w)(w+u)} \\ +& =\frac{2 u v w}{(u+v+w)(u v+v w+w u)-u v w} \\ +& =\frac{2 u v w}{-u v w}=-2 +\end{aligned} +$$ + +as desired (areas signed). +『 Second solution ("nice" lengths). WLOG $A B P C$ is convex. Let $x=A B=B C=$ $C A$. By Ptolemy's theorem and strong Ptolemy, + +$$ +\begin{aligned} +P A & =P B+P C \\ +P A^{2} & =P B \cdot P C+A B \cdot A C=P B \cdot P C+x^{2} \\ +\Longrightarrow x^{2} & +P B^{2}+P B \cdot P C+P C^{2} . +\end{aligned} +$$ + +Also, $P D \cdot P A=P B \cdot P C$ and similarly since $\overline{P A}$ bisects $\angle B P C$ (causing $\triangle B P D \sim$ $\triangle A P C)$. + +Now $P$ is the Fermat point of $\triangle D E F$, since $\angle D P F=\angle F P E=\angle E P D=120^{\circ}$. Thus + +$$ +\begin{aligned} +{[D E F] } & =\frac{\sqrt{3}}{4} \sum_{\mathrm{cyc}} P E \cdot P F \\ +& =\frac{\sqrt{3}}{4} \sum_{\mathrm{cyc}}\left(\frac{P A \cdot P C}{P B}\right)\left(\frac{P A \cdot P B}{P C}\right) \\ +& =\frac{\sqrt{3}}{4} \sum_{\mathrm{cyc}} P A^{2} \\ +& =\frac{\sqrt{3}}{4}\left((P B+P C)^{2}+P B^{2}+P C^{2}\right) \\ +& =\frac{\sqrt{3}}{4} \cdot 2 x^{2}=2[A B C] . +\end{aligned} +$$ + +## §2 Solutions to Day 2 + +## §2.1 JMO 2017/4, proposed by Titu Andreescu + +Available online at https://aops.com/community/p8117256. + +## Problem statement + +Are there any triples $(a, b, c)$ of positive integers such that $(a-2)(b-2)(c-2)+12$ is a prime number that properly divides the positive number $a^{2}+b^{2}+c^{2}+a b c-2017 ?$ + +No such $(a, b, c)$. +Assume not. Let $x=a-2, y=b-2, z=c-2$, hence $x, y, z \geq-1$. + +$$ +\begin{aligned} +a^{2}+b^{2}+c^{2}+a b c-2017 & =(x+2)^{2}+(y+2)^{2}+(z+2)^{2} \\ +& +(x+2)(y+2)(z+2)-2017 \\ +& =(x+y+z+4)^{2}+(x y z+12)-45^{2} +\end{aligned} +$$ + +Thus the divisibility relation becomes + +$$ +p=x y z+12 \mid(x+y+z+4)^{2}-45^{2}>0 +$$ + +so either + +$$ +\begin{aligned} +& p=x y z+12 \mid x+y+z-41 \\ +& p=x y z+12 \mid x+y+z+49 +\end{aligned} +$$ + +Assume $x \geq y \geq z$, hence $x \geq 14$ (since $x+y+z \geq 41$ ). We now eliminate several edge cases to get $x, y, z \neq-1$ and a little more: + +Claim - We have $x \geq 17, y \geq 5, z \geq 1$, and $\operatorname{gcd}(x y z, 6)=1$. + +Proof. First, we check that neither $y$ nor $z$ is negative. + +- If $x>0$ and $y=z=-1$, then we want $p=x+12$ to divide either $x-43$ or $x+47$. We would have $0 \equiv x-43 \equiv-55(\bmod p)$ or $0 \equiv x+47 \equiv 35(\bmod p)$, but $p>11$ contradiction. +- If $x, y>0$, and $z=-1$, then $p=12-x y>0$. However, this is clearly incompatible with $x \geq 14$. + +Finally, obviously $x y z \neq 0$ (else $p=12$ ). So $p=x y z+12 \geq 14 \cdot 1^{2}+12=26$ or $p \geq 29$. Thus $\operatorname{gcd}(6, p)=1$ hence $\operatorname{gcd}(6, x y z)=1$. + +We finally check that $y=1$ is impossible, which forces $y \geq 5$. If $y=1$ and hence $z=1$ then $p=x+12$ should divide either $x+51$ or $x-39$. These give $39 \equiv 0(\bmod p)$ or $25 \equiv 0(\bmod p)$, but we are supposed to have $p \geq 29$. + +In that situation $x+y+z-41$ and $x+y+z+49$ are both even, so whichever one is divisible by $p$ is actually divisible by $2 p$. Now we deduce that: + +$$ +x+y+z+49 \geq 2 p=2 x y z+24 \Longrightarrow 25 \geq 2 x y z-x-y-z +$$ + +But $x \geq 17$ and $y \geq 5$ thus + +$$ +\begin{aligned} +2 x y z-x-y-z & =z(2 x y-1)-x-y \\ +& \geq 2 x y-1-x-y \\ +& >(x-1)(y-1)>60 +\end{aligned} +$$ + +which is a contradiction. Having exhausted all the cases we conclude no solutions exist. +Remark. The condition that $x+y+z-41>0$ (which comes from "properly divides") cannot be dropped. Examples of solutions in which $x+y+z-41=0$ include $(x, y, z)=(31,5,5)$ and $(x, y, z)=(29,11,1)$. + +## §2.2 JMO 2017/5, proposed by Ivan Borsenco + +Available online at https://aops.com/community/p8117237. + +## Problem statement + +Let $O$ and $H$ be the circumcenter and the orthocenter of an acute triangle $A B C$. Points $M$ and $D$ lie on side $B C$ such that $B M=C M$ and $\angle B A D=\angle C A D$. Ray $M O$ intersects the circumcircle of triangle $B H C$ in point $N$. Prove that $\angle A D O=\angle H A N$. + +Let $P$ and $Q$ be the arc midpoints of $\widehat{B C}$, so that $A D M Q$ is cyclic (as $\measuredangle Q A D=$ $\measuredangle Q M D=90^{\circ}$ ). Since it's known that $(B H C)$ and $(A B C)$ are reflections across line $B C$, it follows $N$ is the reflection of the arc midpoint $P$ across $M$. + +Claim - Quadrilateral $A D N O$ is cyclic. + +Proof. Since $P N \cdot P O=\frac{1}{2} P N \cdot 2 P O=P M \cdot P Q=P D \cdot P A$. +![](https://cdn.mathpix.com/cropped/2024_11_19_ecd01a6fb7aaf697c013g-08.jpg?height=806&width=738&top_left_y=1156&top_left_x=659) + +To finish, note that $\measuredangle H A N=\measuredangle O N A=\measuredangle O D A$. +Remark. The orthocenter $H$ is superficial and can be deleted basically immediately. One can reverse-engineer the fact that $A D N O$ is cyclic from the truth of the problem statement. + +Remark. One can also show $A D N O$ concyclic by just computing $\measuredangle D A O=\measuredangle P A O$ and $\measuredangle D N O=\measuredangle D P N=\measuredangle A P Q$ in terms of the angles of the triangle, or even more directly just because + +$$ +\measuredangle D N O=\measuredangle D N P=\measuredangle N P D=\measuredangle O P D=\measuredangle O N A=\measuredangle H A N +$$ + +## §2.3 JMO 2017/6, proposed by Maria Monks + +Available online at https://aops.com/community/p8117190. + +## Problem statement + +Let $P_{1}, P_{2}, \ldots, P_{2 n}$ be $2 n$ distinct points on the unit circle $x^{2}+y^{2}=1$, other than $(1,0)$. Each point is colored either red or blue, with exactly $n$ red points and $n$ blue points. Let $R_{1}, R_{2}, \ldots, R_{n}$ be any ordering of the red points. Let $B_{1}$ be the nearest blue point to $R_{1}$ traveling counterclockwise around the circle starting from $R_{1}$. Then let $B_{2}$ be the nearest of the remaining blue points to $R_{2}$ travelling counterclockwise around the circle from $R_{2}$, and so on, until we have labeled all of the blue points $B_{1}$, $\ldots, B_{n}$. Show that the number of counterclockwise arcs of the form $R_{i} \rightarrow B_{i}$ that contain the point $(1,0)$ is independent of the way we chose the ordering $R_{1}, \ldots, R_{n}$ of the red points. + +We present two solutions, one based on swapping and one based on an invariant. +\ First "local" solution by swapping two points. Let $1 \leq ib$ and the card labeled $a$ is to the left of the card labeled $b$. +Karl picks up the card labeled 1 and inserts it back into the sequence in the opposite position: if the card labeled 1 had $i$ cards to its left, then it now has $i$ cards to its right. He then picks up the card labeled 2 and reinserts it in the same manner, and so on, until he has picked up and put back each of the cards $1, \ldots, n$ exactly once in that order. +For example, if $n=4$, then one example of a process is + +$$ +3142 \longrightarrow 3412 \longrightarrow 2341 \longrightarrow 2431 \longrightarrow 2341 +$$ + +which has three swapped pairs both before and after. +Show that, no matter what lineup of cards Karl started with, his final lineup has the same number of swapped pairs as the starting lineup. + +## §1 Solutions to Day 1 + +## §1.1 JMO 2018/1, proposed by Zachary Franco, Zuming Feng + +Available online at https://aops.com/community/p10226138. + +## Problem statement + +For each positive integer $n$, find the number of $n$-digit positive integers for which no two consecutive digits are equal, and the last digit is a prime. + +Almost trivial. Let $a_{n}$ be the desired answer. We have + +$$ +a_{n}+a_{n-1}=4 \cdot 9^{n-1} +$$ + +for all $n$, by padding the $(n-1)$ digit numbers with a leading zero. +Since $a_{0}=0, a_{1}=4$, solving the recursion gives + +$$ +a_{n}=\frac{2}{5}\left(9^{n}-(-1)^{n}\right) +$$ + +The end. +Remark. For concreteness, the first few terms are $0,4,32,292, \ldots$. + +## §1.2 JMO 2018/2, proposed by Titu Andreescu + +Available online at https://aops.com/community/p10226140. + +## Problem statement + +Let $a, b, c$ be positive real numbers such that $a+b+c=4 \sqrt[3]{a b c}$. Prove that + +$$ +2(a b+b c+c a)+4 \min \left(a^{2}, b^{2}, c^{2}\right) \geq a^{2}+b^{2}+c^{2} +$$ + +WLOG let $c=\min (a, b, c)=1$ by scaling. The given inequality becomes equivalent to + +$$ +4 a b+2 a+2 b+3 \geq(a+b)^{2} \quad \forall a+b=4(a b)^{1 / 3}-1 +$$ + +Now, let $t=(a b)^{1 / 3}$ and eliminate $a+b$ using the condition, to get + +$$ +4 t^{3}+2(4 t-1)+3 \geq(4 t-1)^{2} \Longleftrightarrow 0 \leq 4 t^{3}-16 t^{2}+16 t=4 t(t-2)^{2} +$$ + +which solves the problem. +Equality occurs only if $t=2$, meaning $a b=8$ and $a+b=7$, which gives + +$$ +\{a, b\}=\left\{\frac{7 \pm \sqrt{17}}{2}\right\} +$$ + +with the assumption $c=1$. Scaling gives the curve of equality cases. + +## §1.3 JMO 2018/3, proposed by Ray Li + +Available online at https://aops.com/community/p10226149. + +## Problem statement + +Let $A B C D$ be a quadrilateral inscribed in circle $\omega$ with $\overline{A C} \perp \overline{B D}$. Let $E$ and $F$ be the reflections of $D$ over $\overline{B A}$ and $\overline{B C}$, respectively, and let $P$ be the intersection of $\overline{B D}$ and $\overline{E F}$. Suppose that the circumcircles of $E P D$ and $F P D$ meet $\omega$ at $Q$ and $R$ different from $D$. Show that $E Q=F R$. + +Most of this problem is about realizing where the points $P, Q, R$ are. +【 First solution (Evan Chen). Let $X, Y$, be the feet from $D$ to $\overline{B A}, \overline{B C}$, and let $Z=\overline{B D} \cap \overline{A C}$. By Simson theorem, the points $X, Y, Z$ are collinear. Consequently, the point $P$ is the reflection of $D$ over $Z$, and so we conclude $P$ is the orthocenter of $\triangle A B C$. +![](https://cdn.mathpix.com/cropped/2024_11_19_c3196aba41526ceabc46g-5.jpg?height=738&width=1007&top_left_y=1070&top_left_x=533) + +Suppose now we extend ray $C P$ to meet $\omega$ again at $Q^{\prime}$. Then $\overline{B A}$ is the perpendicular bisector of both $\overline{P Q^{\prime}}$ and $\overline{D E}$; consequently, $P Q^{\prime} E D$ is an isosceles trapezoid. In particular, it is cyclic, and so $Q^{\prime}=Q$. In the same way $R$ is the second intersection of ray $\overline{A P}$ with $\omega$. + +Now, because of the two isosceles trapezoids we have found, we conclude + +$$ +E Q=P D=F R +$$ + +as desired. +Remark. Alternatively, after identifying $P$, one can note $\overline{B Q E}$ and $\overline{B R F}$ are collinear. Since $B E=B D=B F$, upon noticing $B Q=B P=B R$ we are also done. + +【I Second solution (Danielle Wang). Here is a solution which does not identify the point $P$ at all. We know that $B E=B D=B F$, by construction. +![](https://cdn.mathpix.com/cropped/2024_11_19_c3196aba41526ceabc46g-6.jpg?height=669&width=1172&top_left_y=242&top_left_x=419) + +Claim - The points $B, Q, E$ are collinear. Similarly the points $B, R, F$ are collinear. + +Proof. Work directed modulo $180^{\circ}$. Let $Q^{\prime}$ be the intersection of $\overline{B E}$ with $(A B C D)$. Let $\alpha=\measuredangle D E B=\measuredangle B D E$ and $\beta=\measuredangle B F D=\measuredangle F D B$. + +Observe that $B E=B D=B F$, so $B$ is the circumcenter of $\triangle D E F$. Thus, $\measuredangle D E P=$ $\measuredangle D E F=90^{\circ}-\beta$. Then + +$$ +\begin{aligned} +\measuredangle D P E & =\measuredangle D E P+\measuredangle P D E=\left(90^{\circ}-\beta\right)+\alpha \\ +& =\alpha-\beta+90^{\circ} \\ +\measuredangle D Q^{\prime} B & =\measuredangle D C B=\measuredangle D C A+\measuredangle A C B \\ +& =\measuredangle D B A-\left(90^{\circ}-\measuredangle D B C\right)=-\left(90^{\circ}-\alpha\right)-\left(90^{\circ}-\left(90^{\circ}-\beta\right)\right) \\ +& =\alpha-\beta+90^{\circ} . +\end{aligned} +$$ + +Thus $Q^{\prime}$ lies on the desired circle, so $Q^{\prime}=Q$. +Now, by power of a point we have $B Q \cdot B E=B P \cdot B D=B R \cdot B F$, so $B Q=B P=B R$. Hence $E Q=P D=F R$. + +## §2 Solutions to Day 2 + +## §2.1 JMO 2018/4, proposed by Titu Andreescu + +Available online at https://aops.com/community/p10232384. + +## Problem statement + +Find all real numbers $x$ for which there exists a triangle $A B C$ with circumradius 2 , such that $\angle A B C \geq 90^{\circ}$, and + +$$ +x^{4}+a x^{3}+b x^{2}+c x+1=0 +$$ + +where $a=B C, b=C A, c=A B$. + +The answer is $x=-\frac{1}{2}(\sqrt{6} \pm \sqrt{2})$. +We prove this the only possible answer. Evidently $x<0$. Now, note that + +$$ +a^{2}+c^{2} \leq b^{2} \leq 4 b +$$ + +since $b \leq 4$ (the diameter of its circumcircle). Then, + +$$ +\begin{aligned} +0 & =x^{4}+a x^{3}+b x^{2}+c x+1 \\ +& =x^{2}\left[\left(x+\frac{1}{2} a\right)^{2}+\left(\frac{1}{x}+\frac{1}{2} c\right)^{2}+\left(b-\frac{a^{2}+c^{2}}{4}\right)\right] \\ +& \geq 0+0+0=0 +\end{aligned} +$$ + +In order for equality to hold, we must have $x=-\frac{1}{2} a, 1 / x=-\frac{1}{2} c$, and $a^{2}+c^{2}=b^{2}=4 b$. This gives us $b=4, a c=4, a^{2}+c^{2}=16$. Solving for $a, c>0$ implies + +$$ +\{a, c\}=\{\sqrt{6} \pm \sqrt{2}\} +$$ + +This gives the $x$ values claimed above; by taking $a, b, c$ as deduced here, we find they work too. + +Remark. Note that by perturbing $\triangle A B C$ slightly, we see $a$ priori that the set of possible $x$ should consist of unions of intervals (possibly trivial). So it makes sense to try inequalities no matter what. + +## §2.2 JMO 2018/5, proposed by Ankan Bhattacharya + +Available online at https://aops.com/community/p10232389. + +## Problem statement + +Let $p$ be a prime, and let $a_{1}, \ldots, a_{p}$ be integers. Show that there exists an integer $k$ such that the numbers + +$$ +a_{1}+k, a_{2}+2 k, \ldots, a_{p}+p k +$$ + +produce at least $\frac{1}{2} p$ distinct remainders upon division by $p$. + +For each $k=0, \ldots, p-1$ let $G_{k}$ be the graph on $\{1, \ldots, p\}$ where we join $\{i, j\}$ if and only if + +$$ +a_{i}+i k \equiv a_{j}+j k \quad(\bmod p) \Longleftrightarrow k \equiv-\frac{a_{i}-a_{j}}{i-j} \quad(\bmod p) +$$ + +So we want a graph $G_{k}$ with at least $\frac{1}{2} p$ connected components. +However, each $\{i, j\}$ appears in exactly one graph $G_{k}$, so some graph has at most $\frac{1}{p}\binom{p}{2}=\frac{1}{2}(p-1)$ edges (by "pigeonhole"). This graph has at least $\frac{1}{2}(p+1)$ connected components, as desired. + +Remark. Here is an example for $p=5$ showing equality can occur: + +$$ +\left[\begin{array}{lllll} +0 & 0 & 3 & 4 & 3 \\ +0 & 1 & 0 & 2 & 2 \\ +0 & 2 & 2 & 0 & 1 \\ +0 & 3 & 4 & 3 & 0 \\ +0 & 4 & 1 & 1 & 4 +\end{array}\right] . +$$ + +Ankan Bhattacharya points out more generally that $a_{i}=i^{2}$ is sharp in general. + +## §2.3 JMO 2018/6, proposed by Maria Monks Gillespie + +Available online at https://aops.com/community/p10232393. + +## Problem statement + +Karl starts with $n$ cards labeled $1,2, \ldots, n$ lined up in random order on his desk. He calls a pair $(a, b)$ of cards swapped if $a>b$ and the card labeled $a$ is to the left of the card labeled $b$. + +Karl picks up the card labeled 1 and inserts it back into the sequence in the opposite position: if the card labeled 1 had $i$ cards to its left, then it now has $i$ cards to its right. He then picks up the card labeled 2 and reinserts it in the same manner, and so on, until he has picked up and put back each of the cards $1, \ldots, n$ exactly once in that order. + +For example, if $n=4$, then one example of a process is + +$$ +3142 \longrightarrow 3412 \longrightarrow 2341 \longrightarrow 2431 \longrightarrow 2341 +$$ + +which has three swapped pairs both before and after. +Show that, no matter what lineup of cards Karl started with, his final lineup has the same number of swapped pairs as the starting lineup. + +The official solution is really tricky. Call the process $P$. +We define a new process $P^{\prime}$ where, when re-inserting card $i$, we additionally change its label from $i$ to $n+i$. An example of $P^{\prime}$ also starting with 3142 is: + +$$ +3142 \longrightarrow 3452 \longrightarrow 6345 \longrightarrow 6475 \longrightarrow 6785 . +$$ + +Note that now, each step of $P^{\prime}$ preserves the number of inversions. Moreover, the final configuration of $P^{\prime}$ is the same as the final configuration of $P$ with all cards incremented by $n$, and of course thus has the same number of inversions. Boom. + diff --git a/USAJMO/md/en-JMO-2019-notes.md b/USAJMO/md/en-JMO-2019-notes.md new file mode 100644 index 0000000000000000000000000000000000000000..2579ec42492f0a277163019f57a9c8f0aa8de877 --- /dev/null +++ b/USAJMO/md/en-JMO-2019-notes.md @@ -0,0 +1,438 @@ +# JMO 2019 Solution Notes + +Evan Chen《陳誼廷》 + +15 April 2024 + + +#### Abstract + +This is a compilation of solutions for the 2019 JMO. The ideas of the solution are a mix of my own work, the solutions provided by the competition organizers, and solutions found by the community. However, all the writing is maintained by me. + +These notes will tend to be a bit more advanced and terse than the "official" solutions from the organizers. In particular, if a theorem or technique is not known to beginners but is still considered "standard", then I often prefer to use this theory anyways, rather than try to work around or conceal it. For example, in geometry problems I typically use directed angles without further comment, rather than awkwardly work around configuration issues. Similarly, sentences like "let $\mathbb{R}$ denote the set of real numbers" are typically omitted entirely. + +Corrections and comments are welcome! + + +## Contents + +0 Problems +1 Solutions to Day 1 +1.1 JMO 2019/1, proposed by Jim Propp ..... 3 +1.2 JMO 2019/2, proposed by Ankan Bhattacharya ..... 5 +1.3 JMO 2019/3, proposed by Ankan Bhattacharya ..... 7 +2 Solutions to Day 2 ..... 10 +2.1 JMO 2019/4, proposed by Ankan Bhattacharya, Zack Chroman, Anant Mudgal ..... 10 +2.2 JMO 2019/5, proposed by Ricky Liu ..... 14 +2.3 JMO 2019/6, proposed by Yannick Yao ..... 15 + +## §0 Problems + +1. There are $a+b$ bowls arranged in a row, numbered 1 through $a+b$, where $a$ and $b$ are given positive integers. Initially, each of the first $a$ bowls contains an apple, and each of the last $b$ bowls contains a pear. A legal move consists of moving an apple from bowl $i$ to bowl $i+1$ and a pear from bowl $j$ to bowl $j-1$, provided that the difference $i-j$ is even. We permit multiple fruits in the same bowl at the same time. The goal is to end up with the first $b$ bowls each containing a pear and the last $a$ bowls each containing an apple. Show that this is possible if and only if the product $a b$ is even. +2. For which pairs of integers $(a, b)$ do there exist functions $f: \mathbb{Z} \rightarrow \mathbb{Z}$ and $g: \mathbb{Z} \rightarrow \mathbb{Z}$ obeying + +$$ +f(g(x))=x+a \quad \text { and } \quad g(f(x))=x+b +$$ + +for all integers $x$ ? +3. Let $A B C D$ be a cyclic quadrilateral satisfying $A D^{2}+B C^{2}=A B^{2}$. The diagonals of $A B C D$ intersect at $E$. Let $P$ be a point on side $\overline{A B}$ satisfying $\angle A P D=\angle B P C$. Show that line $P E$ bisects $\overline{C D}$. +4. Let $A B C$ be a triangle with $\angle B>90^{\circ}$ and let $E$ and $F$ be the feet of the altitudes from $B$ and $C$. Can line $E F$ be tangent to the $A$-excircle? +5. Let $n$ be a nonnegative integer. Determine the number of ways to choose sets $S_{i j} \subseteq\{1,2, \ldots, 2 n\}$, for all $0 \leq i \leq n$ and $0 \leq j \leq n$ (not necessarily distinct), such that + +- $\left|S_{i j}\right|=i+j$, and +- $S_{i j} \subseteq S_{k l}$ if $0 \leq i \leq k \leq n$ and $0 \leq j \leq l \leq n$. + +6. Let $m$ and $n$ be relatively prime positive integers. The numbers $\frac{m}{n}$ and $\frac{n}{m}$ are written on a blackboard. At any point, Evan may pick two of the numbers $x$ and $y$ written on the board and write either their arithmetic mean $\frac{1}{2}(x+y)$ or their harmonic mean $\frac{2 x y}{x+y}$. For which $(m, n)$ can Evan write 1 on the board in finitely many steps? + +## §1 Solutions to Day 1 + +## §1.1 JMO 2019/1, proposed by Jim Propp + +Available online at https://aops.com/community/p12189456. + +## Problem statement + +There are $a+b$ bowls arranged in a row, numbered 1 through $a+b$, where $a$ and $b$ are given positive integers. Initially, each of the first $a$ bowls contains an apple, and each of the last $b$ bowls contains a pear. A legal move consists of moving an apple from bowl $i$ to bowl $i+1$ and a pear from bowl $j$ to bowl $j-1$, provided that the difference $i-j$ is even. We permit multiple fruits in the same bowl at the same time. The goal is to end up with the first $b$ bowls each containing a pear and the last $a$ bowls each containing an apple. Show that this is possible if and only if the product $a b$ is even. + +First we show that if $a b$ is even then the goal is possible. We prove the result by induction on $a+b$. + +- If $\min (a, b)=0$ there is nothing to check. +- If $\min (a, b)=1$, say $a=1$, then $b$ is even, and we can swap the (only) leftmost apple with the rightmost pear by working only with those fruits. +- Now assume $\min (a, b) \geq 2$ and $a+b$ is odd. Then we can swap the leftmost apple with rightmost pear by working only with those fruits, reducing to the situation of $(a-1, b-1)$ which is possible by induction (at least one of them is even). +- Finally assume $\min (a, b) \geq 2$ and $a+b$ is even (i.e. $a$ and $b$ are both even). Then we can swap the apple in position 1 with the pear in position $a+b-1$, and the apple in position 2 with the pear in position $a+b$. This reduces to the situation of $(a-2, b-2)$ which is also possible by induction. + +Now we show that the result is impossible if $a b$ is odd. Define + +$$ +\begin{aligned} +& X=\text { number apples in odd-numbered bowls } \\ +& Y=\text { number pears in odd-numbered bowls. } +\end{aligned} +$$ + +Note that $X-Y$ does not change under this operation. However, if $a$ and $b$ are odd, then we initially have $X=\frac{1}{2}(a+1)$ and $Y=\frac{1}{2}(b-1)$, while the target position has $X=\frac{1}{2}(a-1)$ and $Y=\frac{1}{2}(b+1)$. So when $a b$ is odd this is not possible. + +Remark. Another proof that $a b$ must be even is as follows. +First, note that apples only move right and pears only move left, a successful operation must take exactly $a b$ moves. So it is enough to prove that the number of moves made must be even. + +However, the number of fruits in odd-numbered bowls either increases by +2 or -2 in each move (according to whether $i$ and $j$ are both even or both odd), and since it ends up being the same at the end, the number of moves must be even. + +Alternatively, as pointed out in the official solutions, one can consider the sums of squares of positions of fruits. The quantity changes by + +$$ +\left[(i+1)^{2}+(j-1)^{2}\right]-\left(i^{2}+j^{2}\right)=2(i-j)+2 \equiv 2(\bmod 4) +$$ + +at each step, and eventually the sums of squares returns to zero, as needed. + +## §1.2 JMO 2019/2, proposed by Ankan Bhattacharya + +Available online at https://aops.com/community/p12189493. + +## Problem statement + +For which pairs of integers $(a, b)$ do there exist functions $f: \mathbb{Z} \rightarrow \mathbb{Z}$ and $g: \mathbb{Z} \rightarrow \mathbb{Z}$ obeying + +$$ +f(g(x))=x+a \quad \text { and } \quad g(f(x))=x+b +$$ + +for all integers $x$ ? + +The answer is if $a=b$ or $a=-b$. In the former case, one can take $f(x) \equiv x+a$ and $g(x) \equiv x$. In the latter case, one can take $f(x) \equiv-x+a$ and $g(x)=-x$. + +Now we prove these are the only possibilities. First: +Claim - The functions $f$ and $g$ are bijections. +Proof. Surjectivity is obvious. To see injective, note that if $f(u)=f(v)$ then $g(f(u))=$ $g(f(v)) \Longrightarrow u+b=v+b \Longrightarrow u=v$, and similarly for $g$. + +Note also that for any $x$, we have + +$$ +\begin{aligned} +& f(x+b)=f(g(f(x)))=f(x)+a \\ +& g(x+a)=g(f(g(x)))=g(x)+b . +\end{aligned} +$$ + +If either $a$ is zero or $b$ is zero, we immediately get the other is zero, and hence done. So assume $a b \neq 0$. + +If $|b|>|a|$, then two of + +$$ +\{f(0), f(1), \ldots, f(b-1)\} \quad(\bmod |a|) +$$ + +coincide, which together with repeatedly applying the first equation above will then give a contradiction to injectivity of $f$. Similarly, if $|a|>|b|$ swapping the roles of $f$ and $g$ (and $a$ and $b$ ) will give a contradiction to injectivity of $g$. This completes the proof. + +Remark. Here is a way to visualize the argument, so one can see pictorially what is going on. We draw two parallel number lines indexed by $\mathbb{Z}$. Starting from 0 , we draw red arrow from 0 to $f(0)$, and then a blue arrow from $f(0)$ to $g(f(0))=b$, and then a red arrow from $b$ to $g(b)=f(0)+a$, and so on. These arrows can be extended both directions, leading to an infinite "squaretooth" wave. The following is a picture of an example with $a, b>0$. +![](https://cdn.mathpix.com/cropped/2024_11_19_10fe2e18e797a050a222g-05.jpg?height=358&width=1112&top_left_y=2134&top_left_x=478) + +The problem is essentially trying to decompose our two copies of $\mathbb{Z}$ into multiple squaretooth +waves. For this to be possible, we expect the "width" of the waves on the top and bottom must be the same - i.e., that $|a|=|b|$. + +Remark. This also suggests how to classify all functions $f$ and $g$ satisfying the condition. If $a=b=0$ then any pair of functions $f$ and $g$ which are inverses to each other is okay. There are thus uncountably many pairs of functions $(f, g)$ here. + +If $a=b>0$, then one sets $f(0), f(1), \ldots, f(a-1)$ as any values which are distinct modulo $b$, at which point $f$ and $g$ are uniquely determined. An example for $a=b=3$ is + +$$ +f(x)=\left\{\begin{array}{lll} +x+42 & x \equiv 0 & (\bmod 3) \\ +x+13 & x \equiv 1 & (\bmod 3) \\ +x-37 & x \equiv 2 & (\bmod 3), +\end{array} \quad g(x)=\left\{\begin{array}{lll} +x-39 & x \equiv 0 & (\bmod 3) \\ +x+40 & x \equiv 1 & (\bmod 3) \\ +x-10 & x \equiv 2 & (\bmod 3) +\end{array}\right.\right. +$$ + +The analysis for $a=b<0$ and $a=-b$ are similar, but we don't include the details here. + +## §1.3 JMO 2019/3, proposed by Ankan Bhattacharya + +Available online at https://aops.com/community/p12189455. + +## Problem statement + +Let $A B C D$ be a cyclic quadrilateral satisfying $A D^{2}+B C^{2}=A B^{2}$. The diagonals of $A B C D$ intersect at $E$. Let $P$ be a point on side $\overline{A B}$ satisfying $\angle A P D=\angle B P C$. Show that line $P E$ bisects $\overline{C D}$. + +Here are three solutions. The first two are similar although the first one makes use of symmedians. The last solution by inversion is more advanced. + +【 First solution using symmedians. We define point $P$ to obey + +$$ +\frac{A P}{B P}=\frac{A D^{2}}{B C^{2}}=\frac{A E^{2}}{B E^{2}} +$$ + +so that $\overline{P E}$ is the $E$-symmedian of $\triangle E A B$, therefore the $E$-median of $\triangle E C D$. +Now, note that + +$$ +A D^{2}=A P \cdot A B \quad \text { and } \quad B C^{2}=B P \cdot B A +$$ + +This implies $\triangle A P D \sim \triangle A D B$ and $\triangle B P C \sim \triangle B C A$. Thus + +$$ +\measuredangle D P A=\measuredangle A D B=\measuredangle A C B=\measuredangle B C P +$$ + +and so $P$ satisfies the condition as in the statement (and is the unique point to do so), as needed. + +【 Second solution using only angle chasing (by proposer). We again re-define $P$ to obey $A D^{2}=A P \cdot A B$ and $B C^{2}=B P \cdot B A$. As before, this gives $\triangle A P D \sim \triangle A B D$ and $\triangle B P C \sim \triangle B D P$ and so we let + +$$ +\theta:=\measuredangle D P A=\measuredangle A D B=\measuredangle A C B=\measuredangle B C P . +$$ + +Our goal is to now show $\overline{P E}$ bisects $\overline{C D}$. +Let $K=\overline{A C} \cap \overline{P D}$ and $L=\overline{A D} \cap \overline{P C}$. Since $\measuredangle K P A=\theta=\measuredangle A C B$, quadrilateral $B P K C$ is cyclic. Similarly, so is $A P L D$. +![](https://cdn.mathpix.com/cropped/2024_11_19_10fe2e18e797a050a222g-08.jpg?height=804&width=795&top_left_y=246&top_left_x=633) + +Finally $A K L B$ is cyclic since + +$$ +\measuredangle B K A=\measuredangle B K C=\measuredangle B P C=\theta=\measuredangle D P A=\measuredangle D L A=\measuredangle B L A . +$$ + +This implies $\measuredangle C K L=\measuredangle L B A=\measuredangle D C K$, so $\overline{K L} \| \overline{B C}$. Then $P E$ bisects $\overline{B C}$ by Ceva's theorem on $\triangle P C D$. + +【 Third solution (using inversion). By hypothesis, the circle $\omega_{a}$ centered at $A$ with radius $A D$ is orthogonal to the circle $\omega_{b}$ centered at $B$ with radius $B C$. For brevity, we let $\mathbf{I}_{a}$ and $\mathbf{I}_{b}$ denote inversion with respect to $\omega_{a}$ and $\omega_{b}$. +We let $P$ denote the intersection of $\overline{A B}$ with the radical axis of $\omega_{a}$ and $\omega_{b}$; hence $P=\mathbf{I}_{a}(B)=\mathbf{I}_{b}(A)$. This already implies that + +$$ +\measuredangle D P A \stackrel{\mathbf{I}_{a}}{=} \measuredangle A D B=\measuredangle A C B \stackrel{\mathbf{I}_{b}}{=} \measuredangle B P C +$$ + +so $P$ satisfies the angle condition. +![](https://cdn.mathpix.com/cropped/2024_11_19_10fe2e18e797a050a222g-08.jpg?height=732&width=1012&top_left_y=1844&top_left_x=522) + +Claim - The point $K=\mathbf{I}_{a}(C)$ lies on $\omega_{b}$ and $\overline{D P}$. Similarly $L=\mathbf{I}_{b}(D)$ lies on $\omega_{a}$ and $\overline{C P}$. + +Proof. The first assertion follows from the fact that $\omega_{b}$ is orthogonal to $\omega_{a}$. For the other, since $(B C D)$ passes through $A$, it follows $P=\mathbf{I}_{a}(B), K=\mathbf{I}_{a}(C)$, and $D=\mathbf{I}_{a}(D)$ are collinear. + +Finally, since $C, L, P$ are collinear, we get $A$ is concyclic with $K=\mathbf{I}_{a}(C), L=\mathbf{I}_{a}(L)$, $B=\mathbf{I}_{a}(P)$, i.e. that $A K L B$ is cyclic. So $\overline{K L} \| \overline{C D}$ by Reim's theorem, and hence $\overline{P E}$ bisects $\overline{C D}$ by Ceva's theorem. + +## §2 Solutions to Day 2 + +## §2.1 JMO 2019/4, proposed by Ankan Bhattacharya, Zack Chroman, Anant Mudgal + +Available online at https://aops.com/community/p12195848. + +## Problem statement + +Let $A B C$ be a triangle with $\angle B>90^{\circ}$ and let $E$ and $F$ be the feet of the altitudes from $B$ and $C$. Can line $E F$ be tangent to the $A$-excircle? + +We show it is not possible, by contradiction (assuming $E F$ is indeed tangent). Thus $B E C F$ is a convex cyclic quadrilateral inscribed in a circle with diameter $\overline{B C}$. Note also that the $A$-excircle lies on the opposite side from $A$ as line $E F$, since $A, E, C$ are collinear in that order. + +【 First solution by similarity. Note that $\triangle A E F$ is similar to $\triangle A B C$ (and oppositely oriented). However, since they have the same $A$-exradius, it follows they are congruent. +![](https://cdn.mathpix.com/cropped/2024_11_19_10fe2e18e797a050a222g-10.jpg?height=272&width=621&top_left_y=1195&top_left_x=729) + +Consequently we get $E F=B C$. But this implies $B F C E$ is a rectangle, contradiction. +【 Second length solution by tangent lengths. By $t(\bullet)$ we mean the length of the tangent from $P$ to the $A$-excircle. It is a classical fact for example that $t(A)=s$. The main idea is to use the fact that + +$$ +a \cos A=E F=t(E)+t(F) . +$$ + +Here $E F=a \cos A$ follows from the extended law of sines applied to the circle with diameter $\overline{B C}$, since there we have $E F=B C \sin \angle E C F=a \sin \angle A C F=a \cos A$. We may now compute + +$$ +\begin{aligned} +& t(E)=t(A)-A E=s-c \cos A \\ +& t(F)=t(A)-A F=s-b \cos A +\end{aligned} +$$ + +Therefore, + +$$ +\begin{aligned} +a \cos A=2 s-(b+c) \cos A \Longrightarrow \quad(a+b+c) \cos A & =2 s \\ +\Longrightarrow \cos A & =1 . +\end{aligned} +$$ + +This is an obvious contradiction. +Remark. On the other hand, there really is an equality case with $A$ being some point at infinity (meaning $\cos A=1$ ). So, this problem is "sharper" than one might expect; the answer is not "obviously no". + +I Third solution by Pitot and trigonometry. In fact, the $t(\bullet)$ notation from the previous solution gives us a classical theorem once we note the $A$-excircle is tangent to all four lines $E F, B C, B F$ and $C E$ : + +Claim (Pitot theorem) - We have $B F+E F=B C+C E$. + +Proof. Here is a proof for completeness. By $t(B)$ we mean the length of the tangent from $B$ to the $A$-excircle, and define $t(C), t(E), t(F)$ similarly. Then + +$$ +\begin{array}{ll} +B F=t(B)-t(F) & E F=t(E)+t(F) \\ +B C=t(B)+t(C) & C E=t(E)-t(C) +\end{array} +$$ + +and summing gives the result. +![](https://cdn.mathpix.com/cropped/2024_11_19_10fe2e18e797a050a222g-11.jpg?height=809&width=686&top_left_y=995&top_left_x=685) + +We now calculate all the lengths using trigonometry: + +$$ +\begin{aligned} +& B C=a \\ +& B F=a \cos \left(180^{\circ}-B\right)=a \cos (A+C) \\ +& C E=a \cos C \\ +& E F=B C \sin \angle E C F=a \sin \angle A C F=a \cos A +\end{aligned} +$$ + +Thus, we apparently have + +$$ +\cos (A+C)+\cos A=1+\cos C +$$ + +but this is impossible since $\cos (A+C)<\cos C$ (since $A+C=180-B<90^{\circ}$ ) and $\cos A<1$. + +【 Fourth solution by Pitot and Ptolemy (Evan Chen). We give a trig-free way to finish from Pitot's theorem + +$$ +B F+E F=B C+C E . +$$ + +Assume that $x=B F, y=C E$, and $B C=1$; then the above relation becomes + +$$ +1+y-x=B C+C E-B F=E F=E F \cdot 1=x y+\sqrt{\left(1-x^{2}\right)\left(1-y^{2}\right)} +$$ + +with the last step by Ptolemy's theorem. This rearranges to give + +$$ +(1+y)(1-x)=\sqrt{\left(1-x^{2}\right)\left(1-y^{2}\right)} \Longrightarrow \frac{1+y}{1-y}=\frac{1+x}{1-x} \Longrightarrow x=y +$$ + +but that means $B E C F$ is a rectangle: contradicting the fact that lines $B E$ and $C F$ meet at a point $A$. + +『 Fifth solution, by angle chasing only! Let $J$ denote the $A$-excenter. Then $J$ should be the intersection of the internal bisectors of $\angle F E C$ and $\angle F B C$, so it is the midpoint of arc $\widehat{F C}$ on the circle with diameter $\overline{B C}$. +![](https://cdn.mathpix.com/cropped/2024_11_19_10fe2e18e797a050a222g-12.jpg?height=824&width=690&top_left_y=1093&top_left_x=683) + +But now we get $\angle B J C=90^{\circ}$ from $J$ lying on this circle. Yet $\angle B J C=90^{\circ}-\frac{1}{2} \angle A$ in general, so $\angle A=0^{\circ}$ which is impossible. + +【 Sixth solution (Zuming Feng). This is similar to the preceding solution, but phrased using contradiction and inequalities. We let $X$ and $Y$ denote the tangency points of the $A$-excircle on lines $A B$ and $A C$. Moreover, let $J$ denote the $A$-excenter. +![](https://cdn.mathpix.com/cropped/2024_11_19_10fe2e18e797a050a222g-12.jpg?height=350&width=809&top_left_y=2275&top_left_x=635) + +Note that $A B>A E$ and $A X=A Y$, therefore $B X$ $\tan \angle Y E J$, thus + +$$ +\angle X B J>\angle Y E J +$$ + +However, if line $E F$ was actually tangent to the $A$-excircle, we would have + +$$ +2 \angle X B J=\angle X B C=\angle F B C=\angle F E C=\angle F E Y=2 \angle J E Y +$$ + +which is a contradiction. + +【 Seventh solution, by complex numbers, for comedic effect (Evan Chen). Let us denote the tangency points of the $A$-excircle with sides $B C, C A, A B$ as $x, y, z$. Assume moreover that line $E F$ is tangent to the $A$-excircle at a point $P$. + +Also, for brevity let $s=x y+y z+z x$. Then, we have + +$$ +\begin{aligned} +& E=\frac{2 p y}{p+y}=\frac{1}{2}\left(b+y+y-y^{2} \bar{b}\right)=\frac{z x}{z+x}+y-\frac{y^{2}}{z+x} \\ +& \Longrightarrow \frac{2}{\frac{1}{p}+\frac{1}{y}}=\frac{x y+x z+z x-y^{2}}{z+x} \Longrightarrow \frac{\frac{1}{p}+\frac{1}{y}}{2}=\frac{x+z}{s-y^{2}} +\end{aligned} +$$ + +Similarly by considering the point $F$, + +$$ +\frac{\frac{1}{p}+\frac{1}{z}}{2}=\frac{x+y}{s-z^{2}} +$$ + +Thus we can eliminate $P$ and obtain + +$$ +\begin{aligned} +\Longrightarrow \frac{\frac{1}{y}-\frac{1}{z}}{2} & =\frac{x+z}{s-y^{2}}-\frac{x+y}{s-z^{2}}=\frac{-s(y-z)+x\left(y^{2}-z^{2}\right)+\left(y^{3}-z^{3}\right)}{\left(s-y^{2}\right)\left(s-z^{2}\right)} \\ +\Longleftrightarrow \frac{1}{2 y z} & =\frac{s-x(y+z)-\left(y^{2}+y z+z^{2}\right)}{\left(s-y^{2}\right)\left(s-z^{2}\right)}=\frac{-\left(y^{2}+z^{2}\right)}{\left(s-y^{2}\right)\left(s-z^{2}\right)} \\ +\Longleftrightarrow 0 & =\left(s-y^{2}\right)\left(s-z^{2}\right)+2 y z\left(y^{2}+z^{2}\right) \\ +& =[x(y+z)+y(z-y)][x(y+z)+z(y-z)]+2 y z\left(y^{2}+z^{2}\right) \\ +& =x^{2}(y+z)^{2}-(y-z)^{2} \cdot x(y+z)+y z\left(2 y^{2}+2 z^{2}-(y-z)^{2}\right) \\ +& =x^{2}(y+z)^{2}-(y-z)^{2} \cdot x(y+z)+y z(y+z)^{2} \\ +& =x y z(y+z)\left[\frac{x}{y}+\frac{x}{z}-\frac{y}{z}-\frac{z}{y}+2+\frac{y}{x}+\frac{z}{x}\right] +\end{aligned} +$$ + +However, $\triangle X Y Z$ is obtuse with $\angle X>90^{\circ}$, we have $y+z \neq 0$. Note that + +$$ +\begin{aligned} +& \frac{x}{y}+\frac{y}{x}=2 \operatorname{Re} \frac{x}{y}=2 \cos (2 \angle X Z Y) \\ +& \frac{x}{z}+\frac{z}{x}=2 \operatorname{Re} \frac{x}{z}=2 \cos (2 \angle X Y Z) \\ +& \frac{y}{z}+\frac{z}{y}=2 \operatorname{Re} \frac{y}{z}<2 +\end{aligned} +$$ + +and since $\cos (2 \angle X Z Y)+\cos (2 \angle X Y Z)>0$ (say by sum-to-product), we are done. + +## §2.2 JMO 2019/5, proposed by Ricky Liu + +Available online at https://aops.com/community/p12195861. + +## Problem statement + +Let $n$ be a nonnegative integer. Determine the number of ways to choose sets $S_{i j} \subseteq\{1,2, \ldots, 2 n\}$, for all $0 \leq i \leq n$ and $0 \leq j \leq n$ (not necessarily distinct), such that + +- $\left|S_{i j}\right|=i+j$, and +- $S_{i j} \subseteq S_{k l}$ if $0 \leq i \leq k \leq n$ and $0 \leq j \leq l \leq n$. + +The answer is $(2 n)!\cdot 2^{n^{2}}$. First, we note that $\varnothing=S_{00} \subsetneq S_{01} \subsetneq \cdots \subsetneq S_{n n}=\{1, \ldots, 2 n\}$ and thus multiplying by (2n)! we may as well assume $S_{0 i}=\{1, \ldots, i\}$ and $S_{i n}=\{1, \ldots, n+i\}$. We illustrate this situation by placing the sets in a grid, as below for $n=4$; our goal is to fill in the rest of the grid. +$\left[\begin{array}{ccccc}1234 & 12345 & 123456 & 1234567 & 12345678 \\ 123 & & & & \\ 12 & & & & \\ 1 & & & & \\ \varnothing & & & & \end{array}\right]$ + +We claim the number of ways to do so is $2^{n^{2}}$. In fact, more strongly even the partial fillings are given exactly by powers of 2 . + +Claim - Fix a choice $T$ of cells we wish to fill in, such that whenever a cell is in $T$, so are all the cells above and left of it. (In other words, $T$ is a Young tableau.) The number of ways to fill in these cells with sets satisfying the inclusion conditions is $2^{|T|}$. + +An example is shown below, with an indeterminate set marked in red (and the rest of $T$ marked in blue). +$\left[\begin{array}{ccccc}1234 & 12345 & 123456 & 1234567 & 12345678 \\ 123 & 1234 & 12346 & 123467 & \\ 12 & 124 & 1234 \text { or } 1246 & & \\ 1 & 12 & & & \\ \varnothing & 2 & & & \end{array}\right]$ + +Proof. The proof is by induction on $|T|$, with $|T|=0$ being vacuous. +Now suppose we have a corner $\left[\begin{array}{cc}B & C \\ A & S\end{array}\right]$ where $A, B, C$ are fixed and $S$ is to be chosen. Then we may write $B=A \cup\{x\}$ and $C=A \cup\{x, y\}$ for $x, y \notin A$. Then the two choices of $S$ are $A \cup\{x\}$ (i.e. $B$ ) and $A \cup\{y\}$, and both of them are seen to be valid. + +In this way, we gain a factor of 2 any time we add one cell as above to $T$. Since we can achieve any Young tableau in this way, the induction is complete. + +## §2.3 JMO 2019/6, proposed by Yannick Yao + +Available online at https://aops.com/community/p12195834. + +## Problem statement + +Let $m$ and $n$ be relatively prime positive integers. The numbers $\frac{m}{n}$ and $\frac{n}{m}$ are written on a blackboard. At any point, Evan may pick two of the numbers $x$ and $y$ written on the board and write either their arithmetic mean $\frac{1}{2}(x+y)$ or their harmonic mean $\frac{2 x y}{x+y}$. For which $(m, n)$ can Evan write 1 on the board in finitely many steps? + +We claim this is possible if and only $m+n$ is a power of 2 . Let $q=m / n$, so the numbers on the board are $q$ and $1 / q$. +\I Impossibility. The main idea is the following. +Claim - Suppose $p$ is an odd prime. Then if the initial numbers on the board are $-1(\bmod p)$, then all numbers on the board are $-1(\bmod p)$. + +Proof. Let $a \equiv b \equiv-1(\bmod p)$. Note that $2 \not \equiv 0(\bmod p)$ and $a+b \equiv-2 \not \equiv 0(\bmod p)$. Thus $\frac{a+b}{2}$ and $\frac{2 a b}{a+b}$ both make sense modulo $p$ and are equal to $-1(\bmod p)$. + +Thus if there exists any odd prime divisor $p$ of $m+n$ (implying $p \nmid m n$ ), then + +$$ +q \equiv \frac{1}{q} \equiv-1 \quad(\bmod p) . +$$ + +and hence all numbers will be $-1(\bmod p)$ forever. This implies that it's impossible to write 1 , whenever $m+n$ is divisible by some odd prime. + +【 Construction. Conversely, suppose $m+n$ is a power of 2 . We will actually construct 1 without even using the harmonic mean. +![](https://cdn.mathpix.com/cropped/2024_11_19_10fe2e18e797a050a222g-15.jpg?height=152&width=1004&top_left_y=1786&top_left_x=526) + +Note that + +$$ +\frac{n}{m+n} \cdot q+\frac{m}{m+n} \cdot \frac{1}{q}=1 +$$ + +and obviously by taking appropriate midpoints (in a binary fashion) we can achieve this using arithmetic mean alone. + diff --git a/USAJMO/md/en-JMO-2020-notes.md b/USAJMO/md/en-JMO-2020-notes.md new file mode 100644 index 0000000000000000000000000000000000000000..0c949849221f1f6c8782eaaf2c5b3037675e2fd8 --- /dev/null +++ b/USAJMO/md/en-JMO-2020-notes.md @@ -0,0 +1,279 @@ +# JMO 2020 Solution Notes + +Evan Chen《陳誼廷》 + +15 April 2024 + + +#### Abstract + +This is a compilation of solutions for the 2020 JMO. The ideas of the solution are a mix of my own work, the solutions provided by the competition organizers, and solutions found by the community. However, all the writing is maintained by me. + +These notes will tend to be a bit more advanced and terse than the "official" solutions from the organizers. In particular, if a theorem or technique is not known to beginners but is still considered "standard", then I often prefer to use this theory anyways, rather than try to work around or conceal it. For example, in geometry problems I typically use directed angles without further comment, rather than awkwardly work around configuration issues. Similarly, sentences like "let $\mathbb{R}$ denote the set of real numbers" are typically omitted entirely. + +Corrections and comments are welcome! + + +## Contents + +0 Problems +1 Solutions to Day 1 +1.1 JMO 2020/1, proposed by Milan Haiman ..... 3 +1.2 JMO 2020/2, proposed by Titu Andreescu, Waldemar Pompe ..... 4 +1.3 JMO 2020/3, proposed by Alex Zhai +2 Solutions to Day 2 +2.1 JMO 2020/4, proposed by Milan Haiman ..... 8 +2.2 JMO 2020/5, proposed by Ankan Bhattacharya ..... 10 +2.3 JMO 2020/6, proposed by Ankan Bhattacharya ..... 12 + +## §0 Problems + +1. Let $n \geq 2$ be an integer. Carl has $n$ books arranged on a bookshelf. Each book has a height and a width. No two books have the same height, and no two books have the same width. + +Initially, the books are arranged in increasing order of height from left to right. In a move, Carl picks any two adjacent books where the left book is wider and shorter than the right book, and swaps their locations. Carl does this repeatedly until no further moves are possible. + +Prove that regardless of how Carl makes his moves, he must stop after a finite number of moves, and when he does stop, the books are sorted in increasing order of width from left to right. +2. Let $\omega$ be the incircle of a fixed equilateral triangle $A B C$. Let $\ell$ be a variable line that is tangent to $\omega$ and meets the interior of segments $B C$ and $C A$ at points $P$ and $Q$, respectively. A point $R$ is chosen such that $P R=P A$ and $Q R=Q B$. Find all possible locations of the point $R$, over all choices of $\ell$. +3. An empty $2020 \times 2020 \times 2020$ cube is given, and a $2020 \times 2020$ grid of square unit cells is drawn on each of its six faces. A beam is a $1 \times 1 \times 2020$ rectangular prism. Several beams are placed inside the cube subject to the following conditions: + +- The two $1 \times 1$ faces of each beam coincide with unit cells lying on opposite faces of the cube. (Hence, there are $3 \cdot 2020^{2}$ possible positions for a beam.) +- No two beams have intersecting interiors. +- The interiors of each of the four $1 \times 2020$ faces of each beam touch either a face of the cube or the interior of the face of another beam. +What is the smallest positive number of beams that can be placed to satisfy these conditions? + +4. Let $A B C D$ be a convex quadrilateral inscribed in a circle and satisfying + +$$ +D A0$. +Claim - If $N_{z}>0$, then we have $N_{x}+N_{y} \geq n$. + +Proof. Again orient the cube so the $z$-plane touches the ground. We see that for each of the $n$ layers of the cube (from top to bottom), there is at least one $x$-beam or $y$-beam. (Pictorially, some of the $x$ and $y$ beams form a "staircase".) This completes the proof. + +Proceeding in a similar fashion, we arrive at the three relations + +$$ +\begin{aligned} +& N_{x}+N_{y} \geq n \\ +& N_{y}+N_{z} \geq n \\ +& N_{z}+N_{x} \geq n +\end{aligned} +$$ + +Summing gives $N_{x}+N_{y}+N_{z} \geq 3 n / 2$ too. +Remark. The problem condition has the following "physics" interpretation. Imagine the cube is a metal box which is sturdy enough that all beams must remain orthogonal to the faces of the box (i.e. the beams cannot spin). Then the condition of the problem is exactly what is needed so that, if the box is shaken or rotated, the beams will not move. + +Remark. Walter Stromquist points out that the number of constructions with 3030 beams is actually enormous: not dividing out by isometries, the number is $(2 \cdot 1010!)^{3}$. + +## §2 Solutions to Day 2 + +## §2.1 JMO 2020/4, proposed by Milan Haiman + +Available online at https://aops.com/community/p15952890. + +## Problem statement + +Let $A B C D$ be a convex quadrilateral inscribed in a circle and satisfying + +$$ +D A0$ is given. Suppose that $n$ equilateral triangles with side length 1 and with non-overlapping interiors are drawn inside $\Delta$, such that each unit equilateral triangle has sides parallel to $\Delta$, but with opposite orientation. Prove that + +$$ +n \leq \frac{2}{3} L^{2} . +$$ + +4. Carina has three pins, labeled $A, B$, and $C$, respectively, located at the origin of the coordinate plane. In a move, Carina may move a pin to an adjacent lattice point at distance 1 away. What is the least number of moves that Carina can make in order for triangle $A B C$ to have area 2021? +5. A finite set $S$ of positive integers has the property that, for each $s \in S$, and each positive integer divisor $d$ of $s$, there exists a unique element $t \in S$ satisfying $\operatorname{gcd}(s, t)=d$. (The elements $s$ and $t$ could be equal.) +Given this information, find all possible values for the number of elements of $S$. +6. Let $n \geq 4$ be an integer. Find all positive real solutions to the following system of $2 n$ equations: + +$$ +\begin{array}{rlrl} +a_{1} & =\frac{1}{a_{2 n}}+\frac{1}{a_{2}}, & a_{2} & =a_{1}+a_{3}, \\ +a_{3} & =\frac{1}{a_{2}}+\frac{1}{a_{4}}, & a_{4} & =a_{3}+a_{5}, \\ +a_{5} & =\frac{1}{a_{4}}+\frac{1}{a_{6}}, & a_{6} & =a_{5}+a_{7}, \\ +& \vdots & \vdots \\ +a_{2 n-1} & =\frac{1}{a_{2 n-2}}+\frac{1}{a_{2 n}}, & a_{2 n} & =a_{2 n-1}+a_{1} . +\end{array} +$$ + +## §1 Solutions to Day 1 + +## §1.1 JMO 2021/1, proposed by Vincent Huang + +Available online at https://aops.com/community/p21498724. + +## Problem statement + +Find all functions $f: \mathbb{N} \rightarrow \mathbb{N}$ which satisfy $f\left(a^{2}+b^{2}\right)=f(a) f(b)$ and $f\left(a^{2}\right)=f(a)^{2}$ for all positive integers $a$ and $b$. + +The answer is $f \equiv 1$ only, which works. We prove it's the only one. +The bulk of the problem is: +Claim - If $f(a)=f(b)=1$ and $a>b$, then $f\left(a^{2}-b^{2}\right)=f(2 a b)=1$. + +Proof. Write + +$$ +\begin{aligned} +1=f(a) f(b) & =f\left(a^{2}+b^{2}\right)=\sqrt{f\left(\left(a^{2}+b^{2}\right)^{2}\right)} \\ +& =\sqrt{f\left(\left(a^{2}-b^{2}\right)^{2}+(2 a b)^{2}\right)} \\ +& =\sqrt{f\left(a^{2}-b^{2}\right) f(2 a b)} . +\end{aligned} +$$ + +By setting $a=b=1$ in the given statement we get $f(1)=f(2)=1$. Now a simple induction on $n$ shows $f(n)=1$ : + +- If $n=2 k$ take $(u, v)=(k, 1)$ hence $2 u v=n$. +- If $n=2 k+1$ take $(u, v)=(k+1, k)$ hence $u^{2}-v^{2}=n$. + + +## §1.2 JMO 2021/2, proposed by Ankan Bhattacharya + +Available online at https://aops.com/community/p21498558. + +## Problem statement + +Rectangles $B C C_{1} B_{2}, C A A_{1} C_{2}$, and $A B B_{1} A_{2}$ are erected outside an acute triangle $A B C$. Suppose that + +$$ +\angle B C_{1} C+\angle C A_{1} A+\angle A B_{1} B=180^{\circ} . +$$ + +Prove that lines $B_{1} C_{2}, C_{1} A_{2}$, and $A_{1} B_{2}$ are concurrent. + +The angle condition implies the circumcircles of the three rectangles concur at a single point $P$. +![](https://cdn.mathpix.com/cropped/2024_11_19_160003dbb585e37c9fb6g-04.jpg?height=812&width=786&top_left_y=982&top_left_x=641) + +Then $\measuredangle C P B_{2}=\measuredangle C P A_{1}=90^{\circ}$, hence $P$ lies on $A_{1} B_{2}$ etc., so we're done. +Remark. As one might guess from the two-sentence solution, the entire difficulty of the problem is getting the characterization of the concurrence point. + +## §1.3 JMO 2021/3, proposed by Alex Zhai + +Available online at https://aops.com/community/p21499596. + +## Problem statement + +An equilateral triangle $\Delta$ of side length $L>0$ is given. Suppose that $n$ equilateral triangles with side length 1 and with non-overlapping interiors are drawn inside $\Delta$, such that each unit equilateral triangle has sides parallel to $\Delta$, but with opposite orientation. Prove that + +$$ +n \leq \frac{2}{3} L^{2} +$$ + +We present the approach of Andrew Gu. For each triangle, we draw a green regular hexagon of side length $1 / 2$ as shown below. +![](https://cdn.mathpix.com/cropped/2024_11_19_160003dbb585e37c9fb6g-05.jpg?height=227&width=181&top_left_y=986&top_left_x=940) + +Claim - All the hexagons are disjoint and lie inside $\Delta$. +Proof. Annoying casework. +Since each hexagon has area $\frac{3 \sqrt{3}}{8}$ and lies inside $\Delta$, we conclude + +$$ +\frac{3 \sqrt{3}}{8} \cdot n \leq \frac{\sqrt{3}}{4} L^{2} \Longrightarrow n \leq \frac{2}{3} L^{2} +$$ + +Remark. The constant $\frac{2}{3}$ is sharp and cannot be improved. The following tessellation shows how to achieve the $\frac{2}{3}$ density. In the figure on the left, one of the green hexagons is drawn in for illustration. The version on the right has all the hexagons. +![](https://cdn.mathpix.com/cropped/2024_11_19_160003dbb585e37c9fb6g-05.jpg?height=591&width=1133&top_left_y=1874&top_left_x=458) + +## §2 Solutions to Day 2 + +## §2.1 JMO 2021/4, proposed by Brandon Wang + +Available online at https://aops.com/community/p21498566. + +## Problem statement + +Carina has three pins, labeled $A, B$, and $C$, respectively, located at the origin of the coordinate plane. In a move, Carina may move a pin to an adjacent lattice point at distance 1 away. What is the least number of moves that Carina can make in order for triangle $A B C$ to have area 2021? + +The answer is 128 . +Define the bounding box of triangle $A B C$ to be the smallest axis-parallel rectangle which contains all three of the vertices $A, B, C$. +![](https://cdn.mathpix.com/cropped/2024_11_19_160003dbb585e37c9fb6g-06.jpg?height=401&width=804&top_left_y=1050&top_left_x=629) + +## Lemma + +The area of a triangle $A B C$ is at most half the area of the bounding box. + +Proof. This can be proven by explicit calculation in coordinates. Nonetheless, we outline a geometric approach. By considering the smallest/largest $x$ coordinate and the smallest/largest $y$ coordinate, one can check that some vertex of the triangle must coincide with a corner of the bounding box (there are four "extreme" coordinates across the $3 \cdot 2=6$ coordinates of our three points). + +So, suppose the bounding box is $A X Y Z$. Imagine fixing $C$ and varying $B$ along the perimeter entire rectangle. The area is a linear function of $B$, so the maximal area should be achieved when $B$ coincides with one of the vertices $\{A, X, Y, Z\}$. But obviously the area of $\triangle A B C$ is + +- exactly 0 if $B=A$, +- at most half the bounding box if $B \in\{X, Z\}$ by one-half-base-height, +- at most half the bounding box if $B=Y$, since $\triangle A B C$ is contained inside either $\triangle A Y Z$ or $\triangle A X Z$. + +We now proceed to the main part of the proof. + +Claim - If $n$ moves are made, the bounding box has area at most $(n / 2)^{2}$. (In other words, a bounding box of area $A$ requires at least $\lceil 2 \sqrt{A}\rceil$ moves.) + +Proof. The sum of the width and height of the bounding box increases by at most 1 each move, hence the width and height have sum at most $n$. So, by AM-GM, their product is at most $(n / 2)^{2}$. + +This immediately implies $n \geq 128$, since the bounding box needs to have area at least $4042>63.5^{2}$. + +On the other hand, if we start all the pins at the point $(3,18)$ then we can reach the following three points in 128 moves: + +$$ +\begin{aligned} +& A=(0,0) \\ +& B=(64,18) \\ +& C=(3,64) +\end{aligned} +$$ + +and indeed triangle $A B C$ has area exactly 2021. +Remark. In fact, it can be shown that to obtain an area of $n / 2$, the bounding-box bound of $\lceil 2 \sqrt{n}\rceil$ moves is best possible, i.e. there will in fact exist a triangle with area $n / 2$. However, since this was supposed to be a JMO4 problem, the committee made a choice to choose $n=4042$ so that contestants only needed to give a single concrete triangle rather than a general construction for all integers $n$. + +## §2.2 JMO 2021/5, proposed by Carl Schildkraut + +Available online at https://aops.com/community/p21498580. + +## Problem statement + +A finite set $S$ of positive integers has the property that, for each $s \in S$, and each positive integer divisor $d$ of $s$, there exists a unique element $t \in S$ satisfying $\operatorname{gcd}(s, t)=d$. (The elements $s$ and $t$ could be equal.) +Given this information, find all possible values for the number of elements of $S$. + +The answer is that $|S|$ must be a power of 2 (including 1 ), or $|S|=0$ (a trivial case we do not discuss further). + +Construction. For any nonnegative integer $k$, a construction for $|S|=2^{k}$ is given by + +$$ +S=\left\{\left(p_{1} \text { or } q_{1}\right) \times\left(p_{2} \text { or } q_{2}\right) \times \cdots \times\left(p_{k} \text { or } q_{k}\right)\right\} +$$ + +for $2 k$ distinct primes $p_{1}, \ldots, p_{k}, q_{1}, \ldots, q_{k}$. +【 Converse. The main claim is as follows. +Claim - In any valid set $S$, for any prime $p$ and $x \in S, \nu_{p}(x) \leq 1$. +Proof. Assume for contradiction $e=\nu_{p}(x) \geq 2$. + +- On the one hand, by taking $x$ in the statement, we see $\frac{e}{e+1}$ of the elements of $S$ are divisible by $p$. +- On the other hand, consider a $y \in S$ such that $\nu_{p}(y)=1$ which must exist (say if $\operatorname{gcd}(x, y)=p)$. Taking $y$ in the statement, we see $\frac{1}{2}$ of the elements of $S$ are divisible by $p$. + +So $e=1$, contradiction. +Now since $|S|$ equals the number of divisors of any element of $S$, we are done. + +## §2.3 JMO 2021/6, proposed by Mohsen Jamaali + +Available online at https://aops.com/community/p21498967. + +## Problem statement + +Let $n \geq 4$ be an integer. Find all positive real solutions to the following system of $2 n$ equations: + +$$ +\begin{aligned} +a_{1} & =\frac{1}{a_{2 n}}+\frac{1}{a_{2}}, & a_{2} & =a_{1}+a_{3}, \\ +a_{3} & =\frac{1}{a_{2}}+\frac{1}{a_{4}}, & a_{4} & =a_{3}+a_{5} \\ +a_{5} & =\frac{1}{a_{4}}+\frac{1}{a_{6}}, & a_{6} & =a_{5}+a_{7}, \\ +& \vdots & & \vdots \\ +a_{2 n-1} & =\frac{1}{a_{2 n-2}}+\frac{1}{a_{2 n}}, & a_{2 n} & =a_{2 n-1}+a_{1} +\end{aligned} +$$ + +The answer is that the only solution is $(1,2,1,2, \ldots, 1,2)$ which works. +We will prove $a_{2 k}$ is a constant sequence, at which point the result is obvious. +\ा First approach (Andrew Gu). Apparently, with indices modulo 2n, we should have + +$$ +a_{2 k}=\frac{1}{a_{2 k-2}}+\frac{2}{a_{2 k}}+\frac{1}{a_{2 k+2}} +$$ + +for every index $k$ (this eliminates all $a_{\text {odd }}$ 's). Define + +$$ +m=\min _{k} a_{2 k} \quad \text { and } \quad M=\max _{k} a_{2 k} +$$ + +Look at the indices $i$ and $j$ achieving $m$ and $M$ to respectively get + +$$ +\begin{aligned} +& m=\frac{2}{m}+\frac{1}{a_{2 i-2}}+\frac{1}{a_{2 i+2}} \geq \frac{2}{m}+\frac{1}{M}+\frac{1}{M}=\frac{2}{m}+\frac{2}{M} \\ +& M=\frac{2}{M}+\frac{1}{a_{2 j-2}}+\frac{1}{a_{2 j+2}} \leq \frac{2}{M}+\frac{1}{m}+\frac{1}{m}=\frac{2}{m}+\frac{2}{M} +\end{aligned} +$$ + +Together this gives $m \geq M$, so $m=M$. That means $a_{2 i}$ is constant as $i$ varies, solving the problem. + +ब Second approach (author's solution). As before, we have + +$$ +a_{2 k}=\frac{1}{a_{2 k-2}}+\frac{2}{a_{2 k}}+\frac{1}{a_{2 k+2}} +$$ + +The proof proceeds in three steps. + +- Define + +$$ +S=\sum_{k} a_{2 k}, \quad \text { and } \quad T=\sum_{k} \frac{1}{a_{2 k}} +$$ + +Summing gives $S=4 T$. On the other hand, Cauchy-Schwarz says $S \cdot T \geq n^{2}$, so $T \geq \frac{1}{2} n$. + +- On the other hand, + +$$ +1=\frac{1}{a_{2 k-2} a_{2 k}}+\frac{2}{a_{2 k}^{2}}+\frac{1}{a_{2 k} a_{2 k+2}} +$$ + +Sum this modified statement to obtain + +$$ +n=\sum_{k}\left(\frac{1}{a_{2 k}}+\frac{1}{a_{2 k+2}}\right)^{2} \stackrel{\text { QM-AM }}{\geq} \frac{1}{n}\left(\sum_{k} \frac{1}{a_{2 k}}+\frac{1}{a_{2 k+2}}\right)^{2}=\frac{1}{n}(2 T)^{2} +$$ + +So $T \leq \frac{1}{2} n$. + +- Since $T \leq \frac{1}{2} n$ and $T \geq \frac{1}{2} n$, we must have equality everywhere above. This means $a_{2 k}$ is a constant sequence. + +Remark. The problem is likely intractable over $\mathbb{C}$, in the sense that one gets a high-degree polynomial which almost certainly has many complex roots. So it seems likely that most solutions must involve some sort of inequality, using the fact we are over $\mathbb{R}_{>0}$ instead. + diff --git a/USAJMO/md/en-JMO-2022-notes.md b/USAJMO/md/en-JMO-2022-notes.md new file mode 100644 index 0000000000000000000000000000000000000000..02701cd524caf5c6ba3863b9cad0fd0bd21d525d --- /dev/null +++ b/USAJMO/md/en-JMO-2022-notes.md @@ -0,0 +1,315 @@ +# JMO 2022 Solution Notes + +Evan Chen《陳誼廷》 + +15 April 2024 + + +#### Abstract + +This is a compilation of solutions for the 2022 JMO. The ideas of the solution are a mix of my own work, the solutions provided by the competition organizers, and solutions found by the community. However, all the writing is maintained by me. + +These notes will tend to be a bit more advanced and terse than the "official" solutions from the organizers. In particular, if a theorem or technique is not known to beginners but is still considered "standard", then I often prefer to use this theory anyways, rather than try to work around or conceal it. For example, in geometry problems I typically use directed angles without further comment, rather than awkwardly work around configuration issues. Similarly, sentences like "let $\mathbb{R}$ denote the set of real numbers" are typically omitted entirely. + +Corrections and comments are welcome! + + +## Contents + +0 Problems +1 Solutions to Day 1 +1.1 JMO 2022/1, proposed by Holden Mui ..... 3 +1.2 JMO 2022/2, proposed by Ankan Bhattacharya ..... 4 +1.3 JMO 2022/3, proposed by Ankan Bhattacharya +2 Solutions to Day 2 ..... 7 +2.1 JMO 2022/4, proposed by Ankan Bhattacharya ..... 7 +2.2 JMO 2022/5, proposed by Holden Mui ..... 9 +2.3 JMO 2022/6, proposed by Ankan Bhattacharya ..... 10 + +## §0 Problems + +1. For which positive integers $m$ does there exist an infinite sequence in $\mathbb{Z} / m \mathbb{Z}$ which is both an arithmetic progression and a geometric progression, but is nonconstant? +2. Let $a$ and $b$ be positive integers. Every cell of an $(a+b+1) \times(a+b+1)$ grid is colored either amber or bronze such that there are at least $a^{2}+a b-b$ amber cells and at least $b^{2}+a b-a$ bronze cells. Prove that it is possible to choose $a$ amber cells and $b$ bronze cells such that no two of the $a+b$ chosen cells lie in the same row or column. +3. Let $b \geq 2$ and $w \geq 2$ be fixed integers, and $n=b+w$. Given are $2 b$ identical black rods and $2 w$ identical white rods, each of side length 1. + +We assemble a regular $2 n$-gon using these rods so that parallel sides are the same color. Then, a convex $2 b$-gon $B$ is formed by translating the black rods, and a convex $2 w$-gon $W$ is formed by translating the white rods. An example of one way of doing the assembly when $b=3$ and $w=2$ is shown below, as well as the resulting polygons $B$ and $W$. +![](https://cdn.mathpix.com/cropped/2024_11_19_b7a51d82629ab4dffabag-02.jpg?height=609&width=1017&top_left_y=1083&top_left_x=571) + +Prove that the difference of the areas of $B$ and $W$ depends only on the numbers $b$ and $w$, and not on how the $2 n$-gon was assembled. +4. Let $A B C D$ be a rhombus, and let $K$ and $L$ be points such that $K$ lies inside the rhombus, $L$ lies outside the rhombus, and $K A=K B=L C=L D$. Prove that there exist points $X$ and $Y$ on lines $A C$ and $B D$ such that $K X L Y$ is also a rhombus. +5. Find all pairs of primes $(p, q)$ for which $p-q$ and $p q-q$ are both perfect squares. +6. Let $a_{0}, b_{0}, c_{0}$ be complex numbers, and define + +$$ +\begin{aligned} +a_{n+1} & =a_{n}^{2}+2 b_{n} c_{n} \\ +b_{n+1} & =b_{n}^{2}+2 c_{n} a_{n} \\ +c_{n+1} & =c_{n}^{2}+2 a_{n} b_{n} +\end{aligned} +$$ + +for all nonnegative integers $n$. Suppose that $\max \left\{\left|a_{n}\right|,\left|b_{n}\right|,\left|c_{n}\right|\right\} \leq 2022$ for all $n \geq 0$. Prove that + +$$ +\left|a_{0}\right|^{2}+\left|b_{0}\right|^{2}+\left|c_{0}\right|^{2} \leq 1 +$$ + +## §1 Solutions to Day 1 + +## §1.1 JMO 2022/1, proposed by Holden Mui + +Available online at https://aops.com/community/p24774800. + +## Problem statement + +For which positive integers $m$ does there exist an infinite sequence in $\mathbb{Z} / m \mathbb{Z}$ which is both an arithmetic progression and a geometric progression, but is nonconstant? + +Answer: $m$ must not be squarefree. +The problem is essentially asking when there exists a nonconstant arithmetic progression in $\mathbb{Z} / m \mathbb{Z}$ which is also a geometric progression. Now, + +- If $m$ is squarefree, then consider three $(s-d, d, s+d)$ in arithmetic progression. It's geometric if and only if $d^{2}=(s-d)(s+d)(\bmod m)$, meaning $d^{2} \equiv 0(\bmod m)$. Then $d \equiv 0(\bmod m)$. So any arithmetic progression which is also geometric is constant in this case. +- Conversely if $p^{2} \mid m$ for some prime $p$, then any arithmetic progression with common difference $m / p$ is geometric by the same calculation. + + +## §1.2 JMO 2022/2, proposed by Ankan Bhattacharya + +Available online at https://aops.com/community/p24774812. + +## Problem statement + +Let $a$ and $b$ be positive integers. Every cell of an $(a+b+1) \times(a+b+1)$ grid is colored either amber or bronze such that there are at least $a^{2}+a b-b$ amber cells and at least $b^{2}+a b-a$ bronze cells. Prove that it is possible to choose $a$ amber cells and $b$ bronze cells such that no two of the $a+b$ chosen cells lie in the same row or column. + +Claim - There exists a transversal $T_{a}$ with at least $a$ amber cells. Analogously, there exists a transversal $T_{b}$ with at least $b$ bronze cells. + +Proof. If one picks a random transversal, the expected value of the number of amber cells is at least + +$$ +\frac{a^{2}+a b-b}{a+b+1}=(a-1)+\frac{1}{a+b+1}>a-1 +$$ + +Now imagine we transform $T_{a}$ to $T_{b}$ in some number of steps, by repeatedly choosing cells $c$ and $c^{\prime}$ and swapping them with the two other corners of the rectangle formed by their row/column, as shown in the figure. +![](https://cdn.mathpix.com/cropped/2024_11_19_b7a51d82629ab4dffabag-04.jpg?height=241&width=818&top_left_y=1379&top_left_x=619) + +By "discrete intermediate value theorem", the number of amber cells will be either $a$ or $a+1$ at some point during this transformation. This completes the proof. + +## §1.3 JMO 2022/3, proposed by Ankan Bhattacharya + +Available online at https://aops.com/community/p24775345. + +## Problem statement + +Let $b \geq 2$ and $w \geq 2$ be fixed integers, and $n=b+w$. Given are $2 b$ identical black rods and $2 w$ identical white rods, each of side length 1. + +We assemble a regular $2 n$-gon using these rods so that parallel sides are the same color. Then, a convex $2 b$-gon $B$ is formed by translating the black rods, and a convex $2 w$-gon $W$ is formed by translating the white rods. An example of one way of doing the assembly when $b=3$ and $w=2$ is shown below, as well as the resulting polygons $B$ and $W$. +![](https://cdn.mathpix.com/cropped/2024_11_19_b7a51d82629ab4dffabag-05.jpg?height=621&width=1021&top_left_y=860&top_left_x=521) + +Prove that the difference of the areas of $B$ and $W$ depends only on the numbers $b$ and $w$, and not on how the $2 n$-gon was assembled. + +We are going to prove that one may swap a black rod with an adjacent white rod (as well as the rods parallel to them) without affecting the difference in the areas of $B-W$. Let $\vec{u}$ and $\vec{v}$ denote the originally black and white vectors that were adjacent on the $2 n$-gon and are now going to be swapped. Let $\vec{x}$ denote the sum of all the other black vectors between $\vec{u}$ and $-\vec{u}$, and define $\vec{y}$ similarly. See the diagram below, where $B_{0}$ and $W_{0}$ are the polygons before the swap, and $B_{1}$ and $W_{1}$ are the resulting changed polygons. +![](https://cdn.mathpix.com/cropped/2024_11_19_b7a51d82629ab4dffabag-06.jpg?height=738&width=1234&top_left_y=222&top_left_x=414) + +Observe that the only change in $B$ and $W$ is in the parallelograms shown above in each diagram. Letting $\wedge$ denote the wedge product, we need to show that + +$$ +\vec{u} \wedge \vec{x}-\vec{v} \wedge \vec{y}=\vec{v} \wedge \vec{x}-\vec{u} \wedge \vec{y} +$$ + +which can be rewritten as + +$$ +(\vec{u}-\vec{v}) \wedge(\vec{x}+\vec{y})=0 +$$ + +In other words, it would suffice to show $\vec{u}-\vec{v}$ and $\vec{x}+\vec{y}$ are parallel. (Students not familiar with wedge products can replace every $\wedge$ with the cross product $\times$ instead.) + +Claim - Both $\vec{u}-\vec{v}$ and $\vec{x}+\vec{y}$ are perpendicular to vector $\vec{u}+\vec{v}$. + +Proof. We have $(\vec{u}-\vec{v}) \perp(\vec{u}+\vec{v})$ because $\vec{u}$ and $\vec{v}$ are the same length. +For the other perpendicularity, note that $\vec{u}+\vec{v}+\vec{x}+\vec{y}$ traces out a diameter of the circumcircle of the original $2 n$-gon; call this diameter $A B$, so + +$$ +A+\vec{u}+\vec{v}+\vec{x}+\vec{y}=B +$$ + +Now point $A+\vec{u}+\vec{v}$ is a point on this semicircle, which means (by the inscribed angle theorem) the angle between $\vec{u}+\vec{v}$ and $\vec{x}+\vec{y}$ is $90^{\circ}$. + +## §2 Solutions to Day 2 + +## §2.1 JMO 2022/4, proposed by Ankan Bhattacharya + +Available online at https://aops.com/community/p24774800. + +## Problem statement + +Let $A B C D$ be a rhombus, and let $K$ and $L$ be points such that $K$ lies inside the rhombus, $L$ lies outside the rhombus, and $K A=K B=L C=L D$. Prove that there exist points $X$ and $Y$ on lines $A C$ and $B D$ such that $K X L Y$ is also a rhombus. + +To start, notice that $\triangle A K B \cong \triangle D L C$ by SSS. Then by the condition $K$ lies inside the rhombus while $L$ lies outside it, we find that the two congruent triangles are just translations of each other (i.e. they have the same orientation). + +『 First solution. Let $M$ be the midpoint of $\overline{K L}$ and is $O$ the center of the rhombus. + +$$ +\text { Claim }-\overline{M O} \perp \overline{A B} . +$$ + +Proof. Let $U$ and $V$ denote the midpoint of $\overline{A B}$ and $\overline{C D}$ respectively. Then $\overline{K U}$ and $\overline{L V}$ are obviously translates, and perpendicular to $\overline{A B} \| \overline{C D}$. Since $M$ is the midpoint of $\overline{K L}$ and $O$ is the midpoint of $\overline{U V}$, the result follows. + +We choose $X$ and $Y$ to be the intersections of the perpendicular bisector of $\overline{K L}$ with $\overline{A C}$ and $\overline{B D}$. +![](https://cdn.mathpix.com/cropped/2024_11_19_b7a51d82629ab4dffabag-07.jpg?height=598&width=803&top_left_y=1551&top_left_x=632) + +Claim - The midpoint of $\overline{X Y}$ coincides with the midpoint of $\overline{K L}$. +Proof. Because + +$$ +\begin{aligned} +& \overline{X Y} \perp \overline{K L} \| \overline{B C} \\ +& \overline{M O} \perp \overline{A B} \\ +& \overline{B D} \perp \overline{A C} +\end{aligned} +$$ + +it follows that $\triangle M O Y$, which was determined by the three lines $\overline{X Y}, \overline{M O}, \overline{B D}$, is similar to $\triangle A B C$. In particular, it is isosceles with $M Y=M O$. Analogously, $M X=M O$. + +Remark. It is also possible to simply use coordinates to prove both claims. + +【 Second solution (author's solution). In this solution, we instead define $X$ and $Y$ as the intersections of the circles centered at $K$ and $L$ of equal radii $K A$, which will be denoted $\omega_{K}$ and $\omega_{L}$. It is clear that $K X L Y$ is a rhombus under this construction, so it suffices to show that $X$ and $Y$ lie on $A C$ and $B D$ (in some order). +![](https://cdn.mathpix.com/cropped/2024_11_19_b7a51d82629ab4dffabag-08.jpg?height=698&width=803&top_left_y=790&top_left_x=632) + +To see this, let $\overline{A C}$ meet $\omega_{K}$ again at $X^{\prime}$. We have + +$$ +\measuredangle C X^{\prime} D=\measuredangle B X^{\prime} C=\measuredangle B X^{\prime} A=\frac{1}{2} \mathrm{~m} \overparen{A B}=\mathrm{m} \overparen{C D} +$$ + +where the arcs are directed modulo $360^{\circ}$; here $\overparen{A B}$ is the arc of $\omega_{K}$ cut out by $\measuredangle A X B$, and $\overparen{D C}$ is the analogous arc of $\omega_{L}$. This implies $X^{\prime}$ lies on $\omega_{L}$ by the inscribed angle theorem. Hence $X=X^{\prime}$, and it follows $X$ lies on $\overline{A C}$. + +Analogously $Y$ lies on $B D$. +Remark. The angle calculation above can also be replaced with a length calculation, as follows. + +Let $M$ and $N$ be the projections of $K$ and $L$ onto $\overline{A C}$, respectively. Then $X^{\prime}$ is the reflection of $A$ across $M$; analogously, the second intersection $X^{\prime \prime}$ with $\overline{A C}$ should be the reflection of $C$ across $N$. So to get $X=X^{\prime}=X^{\prime \prime}$, we would need to show $A C=2 M N$. + +However, note that $A K L D$ is a parallelogram. As $M N$ was the projection of $\overline{K L}$ onto $\overline{A C}$, its length should be the same as the projection of $\overline{A D}$ onto $\overline{A C}$, which is obviously $\frac{1}{2} A C$ because the projection of $D$ onto $\overline{A C}$ is exactly the midpoint of $\overline{A C}$ (i.e. the center of the rhombus). + +## §2.2 JMO 2022/5, proposed by Holden Mui + +Available online at https://aops.com/community/p24774670. + +## Problem statement + +Find all pairs of primes $(p, q)$ for which $p-q$ and $p q-q$ are both perfect squares. + +The answer is $(3,2)$ only. This obviously works so we focus on showing it is the only one. +【 Approach using difference of squares (from author). Set + +$$ +\begin{aligned} +a^{2} & =p-q \\ +b^{2} & =p q-q . +\end{aligned} +$$ + +Note that $01$, it follows $s_{n}$ is unbounded, contradicting $\max \left\{\left|a_{n}\right|,\left|b_{n}\right|,\left|c_{n}\right|\right\} \leq 2022$. + +Remark. The originally intended solution was to capture all three recursions in the following way. First, change the recursion to + +$$ +\begin{aligned} +a_{n+1} & =a_{n}^{2}+2 b_{n} c_{n} \\ +c_{n+1} & =b_{n}^{2}+2 c_{n} a_{n} \\ +b_{n+1} & =c_{n}^{2}+2 a_{n} b_{n} +\end{aligned} +$$ + +which is OK because we are just rearranging the terms in each triple. Then if $\omega$ is any complex number with $\omega^{3}=1$, and we define + +$$ +z_{n}:=a_{n}+b_{n} \omega+c_{n} \omega^{2}, +$$ + +the recursion amounts to saying that $z_{n+1}=z_{n}^{2}$. This allows us to analyze $\left|z_{n}\right|$ in a similar way as above, as now $\left|z_{n}\right|=\left|z_{0}\right|^{2^{n}}$. + diff --git a/USAJMO/md/en-JMO-2023-notes.md b/USAJMO/md/en-JMO-2023-notes.md new file mode 100644 index 0000000000000000000000000000000000000000..b6fe777d60999394abd6835c1e0de5b2e2a3bd8f --- /dev/null +++ b/USAJMO/md/en-JMO-2023-notes.md @@ -0,0 +1,444 @@ +# JMO 2023 Solution Notes + +Evan Chen《陳誼廷》 + +15 April 2024 + + +#### Abstract + +This is a compilation of solutions for the 2023 JMO. The ideas of the solution are a mix of my own work, the solutions provided by the competition organizers, and solutions found by the community. However, all the writing is maintained by me. + +These notes will tend to be a bit more advanced and terse than the "official" solutions from the organizers. In particular, if a theorem or technique is not known to beginners but is still considered "standard", then I often prefer to use this theory anyways, rather than try to work around or conceal it. For example, in geometry problems I typically use directed angles without further comment, rather than awkwardly work around configuration issues. Similarly, sentences like "let $\mathbb{R}$ denote the set of real numbers" are typically omitted entirely. + +Corrections and comments are welcome! + + +## Contents + +0 Problems +1 Solutions to Day 1 +1.1 JMO 2023/1, proposed by Titu Andreescu ..... 3 +1.2 JMO 2023/2, proposed by Holden Mui ..... 4 +1.3 JMO 2023/3, proposed by Holden Mui +2 Solutions to Day 2 ..... 10 +2.1 JMO 2023/4, proposed by David Torres ..... 10 +2.2 JMO 2023/5, proposed by Carl Schildkraut ..... 13 +2.3 JMO 2023/6, proposed by Anton Trygub ..... 14 + +## §0 Problems + +1. Find all triples of positive integers $(x, y, z)$ satisfying + +$$ +2(x+y+z+2 x y z)^{2}=(2 x y+2 y z+2 z x+1)^{2}+2023 +$$ + +2. In an acute triangle $A B C$, let $M$ be the midpoint of $\overline{B C}$. Let $P$ be the foot of the perpendicular from $C$ to $A M$. Suppose that the circumcircle of triangle $A B P$ intersects line $B C$ at two distinct points $B$ and $Q$. Let $N$ be the midpoint of $\overline{A Q}$. Prove that $N B=N C$. +3. Consider an $n$-by- $n$ board of unit squares for some odd positive integer $n$. We say that a collection $C$ of identical dominoes is a maximal grid-aligned configuration on the board if $C$ consists of $\left(n^{2}-1\right) / 2$ dominoes where each domino covers exactly two neighboring squares and the dominoes don't overlap: $C$ then covers all but one square on the board. We are allowed to slide (but not rotate) a domino on the board to cover the uncovered square, resulting in a new maximal gridaligned configuration with another square uncovered. Let $k(C)$ be the number of distinct maximal grid-aligned configurations obtainable from $C$ by repeatedly sliding dominoes. + +Find the maximum possible value of $k(C)$ as a function of $n$. +4. Two players, Blake and Ruby, play the following game on an infinite grid of unit squares, all initially colored white. The players take turns starting with Blake. On Blake's turn, Blake selects one white unit square and colors it blue. On Ruby's turn, Ruby selects two white unit squares and colors them red. The players alternate until Blake decides to end the game. At this point, Blake gets a score, given by the number of unit squares in the largest (in terms of area) simple polygon containing only blue unit squares. + +What is the largest score Blake can guarantee? +5. Positive integers $a$ and $N$ are fixed, and $N$ positive integers are written on a blackboard. Alice and Bob play the following game. On Alice's turn, she must replace some integer $n$ on the board with $n+a$, and on Bob's turn he must replace some even integer $n$ on the board with $n / 2$. Alice goes first and they alternate turns. If on his turn Bob has no valid moves, the game ends. + +After analyzing the $N$ integers on the board, Bob realizes that, regardless of what moves Alice makes, he will be able to force the game to end eventually. Show that, in fact, for this value of $a$ and these $N$ integers on the board, the game is guaranteed to end regardless of Alice's or Bob's moves. +6. Isosceles triangle $A B C$, with $A B=A C$, is inscribed in circle $\omega$. Let $D$ be an arbitrary point inside $B C$ such that $B D \neq D C$. Ray $A D$ intersects $\omega$ again at $E$ (other than $A$ ). Point $F$ (other than $E$ ) is chosen on $\omega$ such that $\angle D F E=90^{\circ}$. Line $F E$ intersects rays $A B$ and $A C$ at points $X$ and $Y$, respectively. Prove that $\angle X D E=\angle E D Y$. + +## §1 Solutions to Day 1 + +## §1.1 JMO 2023/1, proposed by Titu Andreescu + +Available online at https://aops.com/community/p27349258. + +## Problem statement + +Find all triples of positive integers $(x, y, z)$ satisfying + +$$ +2(x+y+z+2 x y z)^{2}=(2 x y+2 y z+2 z x+1)^{2}+2023 +$$ + +Answer: $(3,3,2)$ and permutations. +The solution hinges upon the following claim: +Claim - The identity + +$$ +2(x+y+z+2 x y z)^{2}-(2 x y+2 y z+2 z x+1)^{2}=\left(2 x^{2}-1\right)\left(2 y^{2}-1\right)\left(2 z^{2}-1\right) +$$ + +is true. + +Proof. This can be proved by manually expanding; we show where it "came from". In algebraic number theory, there is a norm function Norm: $\mathbb{Q}(\sqrt{2}) \rightarrow \mathbb{Q}$ defined by + +$$ +\operatorname{Norm}(a+b \sqrt{2})=a^{2}-2 b^{2} +$$ + +which is multiplicative, meaning + +$$ +\operatorname{Norm}(u \cdot v)=\operatorname{Norm}(u) \cdot \operatorname{Norm}(v) +$$ + +This means that for any rational numbers $x, y, z$, we should have + +$$ +\begin{aligned} +& \operatorname{Norm}((1+\sqrt{2} x)(1+\sqrt{2} y)(1+\sqrt{2} z)) \\ += & \operatorname{Norm}(1+\sqrt{2} x) \cdot \operatorname{Norm}(1+\sqrt{2} y) \cdot \operatorname{Norm}(1+\sqrt{2} z) +\end{aligned} +$$ + +But $(1+\sqrt{2} x)(1+\sqrt{2} y)(1+\sqrt{2} z)=(2 x y+2 y z+2 z x+1)+(x+y+z+2 x y z) \sqrt{2}$ so the above equation is the negative of the desired identity. + +We are thus reduced to find positive integers $x, y, z$ satisfying + +$$ +\left(2 x^{2}-1\right)\left(2 y^{2}-1\right)\left(2 z^{2}-1\right)=2023=7 \cdot 17^{2} +$$ + +Each of the factors is a positive integer greater than 1 . The only divisors of 2023 of the form $2 t^{2}-1$ are $1,7,17$. This gives the answers claimed. + +## §1.2 JMO 2023/2, proposed by Holden Mui + +Available online at https://aops.com/community/p27349297. + +## Problem statement + +In an acute triangle $A B C$, let $M$ be the midpoint of $\overline{B C}$. Let $P$ be the foot of the perpendicular from $C$ to $A M$. Suppose that the circumcircle of triangle $A B P$ intersects line $B C$ at two distinct points $B$ and $Q$. Let $N$ be the midpoint of $\overline{A Q}$. Prove that $N B=N C$. + +We show several different approaches. In all solutions, let $D$ denote the foot of the altitude from $A$. +![](https://cdn.mathpix.com/cropped/2024_11_19_9c5e556ae0970c152728g-04.jpg?height=892&width=1003&top_left_y=913&top_left_x=532) + +【 Most common synthetic approach. The solution hinges on the following claim: +Claim - $Q$ coincides with the reflection of $D$ across $M$. +Proof. Note that $\measuredangle A D C=\measuredangle A P C=90^{\circ}$, so $A D P C$ is cyclic. Then by power of a point (with the lengths directed), + +$$ +M B \cdot M Q=M A \cdot M P=M C \cdot M D . +$$ + +Since $M B=M C$, the claim follows. +It follows that $\overline{M N} \| \overline{A D}$, as $M$ and $N$ are respectively the midpoints of $\overline{A Q}$ and $\overline{D Q}$. Thus $\overline{M N} \perp \overline{B C}$, and so $N$ lies on the perpendicular bisector of $\overline{B C}$, as needed. + +Remark (David Lin). One can prove the main claim without power of a point as well, as follows: Let $R$ be the foot from $B$ to $\overline{A M}$, so $B R C P$ is a parallelogram. Note that $A B D R$ +is cyclic, and hence + +$$ +\measuredangle D R M=\measuredangle D B A=Q B A=\measuredangle Q P A=\measuredangle Q P M +$$ + +Thus, $\overline{D R} \| \overline{P Q}$, so $D R Q P$ is also a parallelogram. + +## \ Synthetic approach with no additional points at all. + +Claim - $\triangle B P C \sim \triangle A N M$ (oppositely oriented). + +Proof. We have $\triangle B M P \sim \triangle A M Q$ from the given concyclicity of $A B P Q$. Then + +$$ +\frac{B M}{B P}=\frac{A M}{A Q} \Longrightarrow \frac{2 B M}{B P}=\frac{A M}{A Q / 2} \Longrightarrow \frac{B C}{B P}=\frac{A M}{A N} +$$ + +implying the similarity (since $\measuredangle M A Q=\measuredangle B P M$ ). +This similarity gives us the equality of directed angles + +$$ +\measuredangle(B C, M N)=-\measuredangle(P C, A M)=90^{\circ} +$$ + +as desired. +\ Synthetic approach using only the point $R$. Again let $R$ be the foot from $B$ to $\overline{A M}$, so $B R C P$ is a parallelogram. + +Claim - $A R Q C$ is cyclic; equivalently, $\triangle M A Q \sim \triangle M C R$. + +Proof. $M R \cdot M A=M P \cdot M A=M B \cdot M Q=M C \cdot M Q$. +Note that in $\triangle M C R$, the $M$-median is parallel to $\overline{C P}$ and hence perpendicular to $\overline{R M}$. The same should be true in $\triangle M A Q$ by the similarity, so $\overline{M N} \perp \overline{M Q}$ as needed. + +【 Cartesian coordinates approach with power of a point. Suppose we set $B=(-1,0)$, $M=(0,0), C=(1,0)$, and $A=(a, b)$. One may compute: + +$$ +\begin{aligned} +\overleftrightarrow{A M}: 0 & =b x-a y \Longleftrightarrow y=\frac{b}{a} x \\ +\overleftrightarrow{C P}: 0 & =a(x-1)+b y \Longleftrightarrow y=-\frac{a}{b}(x-1)=-\frac{a}{b} x+\frac{a}{b} \\ +P & =\left(\frac{a^{2}}{a^{2}+b^{2}}, \frac{a b}{a^{2}+b^{2}}\right) +\end{aligned} +$$ + +Now note that + +$$ +A M=\sqrt{a^{2}+b^{2}}, \quad P M=\frac{a}{\sqrt{a^{2}+b^{2}}} +$$ + +together with power of a point + +$$ +A M \cdot P M=B M \cdot Q M +$$ + +to immediately deduce that $Q=(a, 0)$. Hence $N=(0, b / 2)$ and we're done. + +【 Cartesian coordinates approach without power of a point (outline). After computing $A$ and $P$ as above, one could also directly calculate + +$$ +\begin{aligned} +& \text { Perpendicular bisector of } \overline{A B}: y=-\frac{a+1}{b} x+\frac{a^{2}+b^{2}-1}{2 b} \\ +& \text { Perpendicular bisector of } \overline{P B}: y=-\left(\frac{2 a}{b}+\frac{b}{a}\right) x-\frac{b}{2 a} \\ +& \text { Perpendicular bisector of } \overline{P A}: y=-\frac{a}{b} x+\frac{a+a^{2}+b^{2}}{2 b} \\ +& \text { Circumcenter of } \triangle P A B=\left(-\frac{a+1}{2}, \frac{2 a^{2}+2 a+b^{2}}{2 b}\right) +\end{aligned} +$$ + +This is enough to extract the coordinates of $Q=(\bullet, 0)$, because $B=(-1,0)$ is given, and the $x$-coordinate of the circumcenter should be the average of the $x$-coordinates of $B$ and $Q$. In other words, $Q=(-a, 0)$. Hence, $N=\left(0, \frac{b}{2}\right)$, as needed. +\I III-advised barycentric approach (outline). Use reference triangle $A B C$. The $A$ median is parametrized by $(t: 1: 1)$ for $t \in \mathbb{R}$. So because of $\overline{C P} \perp \overline{A M}$, we are looking for $t$ such that + +$$ +\left(\frac{t \vec{A}+\vec{B}+\vec{C}}{t+2}-\vec{C}\right) \perp\left(A-\frac{\vec{B}+\vec{C}}{2}\right) +$$ + +This is equivalent to + +$$ +(t \vec{A}+\vec{B}-(t+1) \vec{C}) \perp(2 \vec{A}-\vec{B}-\vec{C}) +$$ + +By the perpendicularity formula for barycentric coordinates (EGMO 7.16), this is equivalent to + +$$ +\begin{aligned} +0 & =a^{2} t-b^{2} \cdot(3 t+2)+c^{2} \cdot(2-t) \\ +& =\left(a^{2}-3 b^{2}-c^{2}\right) t-2\left(b^{2}-c^{2}\right) \\ +\Longrightarrow t & =\frac{2\left(b^{2}-c^{2}\right)}{a^{2}-3 b^{2}-c^{2}} +\end{aligned} +$$ + +In other words, + +$$ +P=\left(2\left(b^{2}-c^{2}\right): a^{2}-3 b^{2}-c^{2}: a^{2}-3 b^{2}-c^{2}\right) . +$$ + +A long calculation gives $a^{2} y_{P} z_{P}+b^{2} z_{P} x_{P}+c^{2} x_{P} y_{P}=\left(a^{2}-3 b^{2}-c^{2}\right)\left(a^{2}-b^{2}+c^{2}\right)\left(a^{2}-\right.$ $\left.2 b^{2}-2 c^{2}\right)$. Together with $x_{P}+y_{P}+z_{P}=2 a^{2}-4 b^{2}-4 c^{2}$, this makes the equation of $(A B P)$ as + +$$ +0=-a^{2} y z-b^{2} z x-c^{2} x y+\frac{a^{2}-b^{2}+c^{2}}{2} z(x+y+z) +$$ + +To solve for $Q$, set $x=0$ to get to get + +$$ +a^{2} y z=\frac{a^{2}-b^{2}+c^{2}}{2} z(y+z) \Longrightarrow \frac{y}{z}=\frac{a^{2}-b^{2}+c^{2}}{a^{2}+b^{2}-c^{2}} +$$ + +In other words, + +$$ +Q=\left(0: a^{2}-b^{2}+c^{2}: a^{2}+b^{2}-c^{2}\right) +$$ + +Taking the average with $A=(1,0,0)$ then gives + +$$ +N=\left(2 a^{2}: a^{2}-b^{2}+c^{2}: a^{2}+b^{2}-c^{2}\right) . +$$ + +The equation for the perpendicular bisector of $\overline{B C}$ is given by (see EGMO 7.19) + +$$ +0=a^{2}(z-y)+x\left(c^{2}-b^{2}\right) +$$ + +which contains $N$, as needed. + +『 Extremely ill-advised complex numbers approaches (outline). Suppose we pick $a$, $b, c$ as the unit circle, and let $m=(b+c) / 2$. Using the fully general "foot" formula, one can get + +$$ +p=\frac{(a-m) \bar{c}+(\bar{a}-\bar{m}) c+\bar{a} m-a \bar{m}}{2(\bar{a}-\bar{m})}=\frac{a^{2} b-a^{2} c-a b^{2}-2 a b c-a c^{2}+b^{2} c+3 b c^{2}}{4 b c-2 a(b+c)} +$$ + +Meanwhile, an extremely ugly calculation will eventually yield + +$$ +q=\frac{\frac{b c}{a}+b+c-a}{2} +$$ + +SO + +$$ +n=\frac{a+q}{2}=\frac{a+b+c+\frac{b c}{a}}{4}=\frac{(a+b)(a+c)}{2 a} +$$ + +There are a few ways to then verify $N B=N C$. The simplest seems to be to verify that + +$$ +\frac{n-\frac{b+c}{2}}{b-c}=\frac{a-b-c+\frac{b c}{a}}{4(b-c)}=\frac{(a-b)(a-c)}{2 a(b-c)} +$$ + +is pure imaginary, which is clear. + +## §1.3 JMO 2023/3, proposed by Holden Mui + +Available online at https://aops.com/community/p27349423. + +## Problem statement + +Consider an $n$-by- $n$ board of unit squares for some odd positive integer $n$. We say that a collection $C$ of identical dominoes is a maximal grid-aligned configuration on the board if $C$ consists of $\left(n^{2}-1\right) / 2$ dominoes where each domino covers exactly two neighboring squares and the dominoes don't overlap: $C$ then covers all but one square on the board. We are allowed to slide (but not rotate) a domino on the board to cover the uncovered square, resulting in a new maximal grid-aligned configuration with another square uncovered. Let $k(C)$ be the number of distinct maximal grid-aligned configurations obtainable from $C$ by repeatedly sliding dominoes. +Find the maximum possible value of $k(C)$ as a function of $n$. + +The answer is that + +$$ +k(C) \leq\left(\frac{n+1}{2}\right)^{2} +$$ + +Remark (Comparison with USAMO version). In the USAMO version of the problem, students instead are asked to find all possible values of $k(C)$. The answer is $k(C) \in$ $\left\{1,2, \ldots,\left(\frac{n-1}{2}\right)^{2}\right\} \cup\left\{\left(\frac{n+1}{2}\right)^{2}\right\}$. + +Index the squares by coordinates $(x, y) \in\{1,2, \ldots, n\}^{2}$. We say a square is special if it is empty or it has the same parity in both coordinates as the empty square. + +Construct a directed graph $G=G(C)$ whose vertices are special squares as follows: for each domino on a special square $s$, we draw a directed edge from $s$ to the special square that domino points to, if any. (If the special square has both odd coordinates, all special squares have an outgoing edge except the empty cell. In the even-even case, some arrows may point "off the board" and not be drawn.) +![](https://cdn.mathpix.com/cropped/2024_11_19_9c5e556ae0970c152728g-08.jpg?height=612&width=615&top_left_y=1816&top_left_x=726) + +Now focus specifically on the weakly connected component $T$ of $G$ (i.e. the connected component of the undirected version of $G$ ) containing the empty square. + +Claim - The graph $T$ has no cycles, even undirected. Hence, the undirected version of $T$ is tree. + +Proof. Assume for contradiction $T$ had an undirected cycle. Then if we look at the direction of arrows along the cycle, because every vertex of $T$ had outdegree at most 1, the arrows must all point in the same direction (i.e. we actually have a directed cycle). But then $T$ must consist solely of this cycle. Yet the empty square has outdegree 0 , contradiction. + +Notice that all the arrows along $T$ point towards the empty cell, and moving a domino corresponds to flipping an arrow. Therefore: + +Claim - $k(C)$ is exactly the number of vertices of $T$. + +Proof. Starting with the underlying tree, the set of possible graphs is described by picking one vertex to be the sink (the empty cell) and then directing all arrows towards it. + +This implies that $k(C) \leq\left(\frac{n+1}{2}\right)^{2}$, the total number of vertices of $G$ (this could only occur if the special squares are odd-odd, not even-even). Equality is achieved as long as $T$ is a spanning tree; one example of a way to achieve this is using the snake configuration below. +![](https://cdn.mathpix.com/cropped/2024_11_19_9c5e556ae0970c152728g-09.jpg?height=464&width=466&top_left_y=1270&top_left_x=795) + +## §2 Solutions to Day 2 + +## §2.1 JMO 2023/4, proposed by David Torres + +Available online at https://aops.com/community/p27349414. + +## Problem statement + +Two players, Blake and Ruby, play the following game on an infinite grid of unit squares, all initially colored white. The players take turns starting with Blake. On Blake's turn, Blake selects one white unit square and colors it blue. On Ruby's turn, Ruby selects two white unit squares and colors them red. The players alternate until Blake decides to end the game. At this point, Blake gets a score, given by the number of unit squares in the largest (in terms of area) simple polygon containing only blue unit squares. +What is the largest score Blake can guarantee? + +The answer is 4 squares. + +ब Algorithm for Blake to obtain at least 4 squares. We simply let Blake start with any cell blue, then always draw adjacent to a previously drawn blue cell until this is no longer possible. + +Note that for $n \leq 3$, any connected region of $n$ blue cells has more than $2 n$ liberties (non-blue cells adjacent to a blue cell); up to translation, rotation, and reflection, all the cases are shown in the figure below with liberties being denoted by circles. +![](https://cdn.mathpix.com/cropped/2024_11_19_9c5e556ae0970c152728g-10.jpg?height=655&width=818&top_left_y=1481&top_left_x=622) + +So as long as $n \leq 3$, it's impossible that Ruby has blocked every liberty, since Ruby has colored exactly $2 n$ cells red. Therefore, this algorithm could only terminate once $n \geq 4$. +\ Algorithm for Ruby to prevent more than 4 squares. Divide the entire grid into $2 \times 2$ squares, which we call windows. Any time Blake makes a move in a cell $c$, let Ruby mark any orthogonal neighbors of $c$ in its window; then place any leftover red cells arbitrarily. + +Claim - It's impossible for any window to contain two orthogonally adjacent blue cells. + +Proof. By construction: if there were somehow two adjacent blue cells in the same window, whichever one was played first should have caused red cells to be added. + +We show this gives the upper bound of 4 squares. Consider a blue cell $w$, and assume WLOG it is in the southeast corner of a window. Label squares $x, y, z$ as shown below. +![](https://cdn.mathpix.com/cropped/2024_11_19_9c5e556ae0970c152728g-11.jpg?height=506&width=510&top_left_y=661&top_left_x=773) + +Note that by construction, the blue polygon cannot leave the square $\{w, x, y, z\}$, since whenever one of these four cells is blue, its neighbours outside that square are guaranteed to be red. This implies the bound. + +Remark (For Tetris fans). Here is a comedic alternative finish after proving the claim. Consider the possible tetrominoes (using the notation of https://en.wikipedia.org/wiki/ Tetromino\#One-sided_tetrominoes). We claim that only the square (0) is obtainable; as + +- T, J/L, and I all have three cells in a row, so they can't occur; +- S and Z can't occur either; if the bottom row of an S crossed a window boundary, then the top row doesn't for example. + +Moreover, the only way a blue 0 could be obtained is if each of it cells is in a different window. In that case, no additional blue cells can be added: it's fully surrounded by red. + +Finally, for any $k$-omino with $k>4$, one can find a tetromino as a subset. (Proof: take the orthogonal adjacency graph of the $k$-omino, choose a spanning tree, and delete leaves from the tree until there are only four vertices left.) + +Remark (Common wrong approach). Suppose Ruby employs the following algorithm whenever Blake places a square $x$. If either the north and west neighbors of $x$ are unoccupied, place red squares on both of them. With any leftover red squares, place them at other neighbors of $x$ if possible. Finally, place any other red squares arbitrarily. (Another variant, the one Evan originally came up with, is to place east if possible when west is occupied, place south if possible when north is occupied, and then place any remaining red squares arbitrarily.) + +As written, this strategy does not work. The reason is that one can end up in the following situation (imagine the blue square in the center is played first; moves for Ruby are drawn as red X's): +![](https://cdn.mathpix.com/cropped/2024_11_19_9c5e556ae0970c152728g-12.jpg?height=410&width=429&top_left_y=246&top_left_x=822) + +In order to prevent Blake from winning, Ruby would need to begin playing moves not adjacent to Blake's most recent move. + +Thus in order for this solution to be made correct, one needs a careful algorithm for how Ruby should play when the north and west neighbors are not available. As far as I am aware, there are some specifications that work (and some that don't), but every working algorithm I have seen seems to involve some amount of casework. + +It is even more difficult to come up with a solution involving playing on just "some" two neighbors of recently added blue squares without the "prefer north and west" idea. + +## §2.2 JMO 2023/5, proposed by Carl Schildkraut + +Available online at https://aops.com/community/p27349336. + +## Problem statement + +Positive integers $a$ and $N$ are fixed, and $N$ positive integers are written on a blackboard. Alice and Bob play the following game. On Alice's turn, she must replace some integer $n$ on the board with $n+a$, and on Bob's turn he must replace some even integer $n$ on the board with $n / 2$. Alice goes first and they alternate turns. If on his turn Bob has no valid moves, the game ends. + +After analyzing the $N$ integers on the board, Bob realizes that, regardless of what moves Alice makes, he will be able to force the game to end eventually. Show that, in fact, for this value of $a$ and these $N$ integers on the board, the game is guaranteed to end regardless of Alice's or Bob's moves. + +For $N=1$, there is nothing to prove. We address $N \geq 2$ only henceforth. Let $S$ denote the numbers on the board. + +Claim - When $N \geq 2$, if $\nu_{2}(x)<\nu_{2}(a)$ for all $x \in S$, the game must terminate no matter what either player does. + +Proof. The $\nu_{2}$ of a number is unchanged by Alice's move and decreases by one on Bob's move. The game ends when every $\nu_{2}$ is zero. + +Hence, in fact the game will always terminate in exactly $\sum_{x \in S} \nu_{2}(x)$ moves in this case, regardless of what either player does. + +Claim - When $N \geq 2$, if there exists a number $x$ on the board such that $\nu_{2}(x) \geq$ $\nu_{2}(a)$, then Alice can cause the game to go on forever. + +Proof. Denote by $x$ the first entry of the board (its value changes over time). Then Alice's strategy is to: + +- Operate on the first entry if $\nu_{2}(x)=\nu_{2}(a)$ (the new entry thus has $\nu_{2}(x+a)>\nu_{2}(a)$ ); +- Operate on any other entry besides the first one, otherwise. + +A double induction then shows that + +- Just before each of Bob's turns, $\nu_{2}(x)>\nu_{2}(a)$ always holds; and +- After each of Bob's turns, $\nu_{2}(x) \geq \nu_{2}(a)$ always holds. + +In particular Bob will never run out of legal moves, since halving $x$ is always legal. + +## §2.3 JMO 2023/6, proposed by Anton Trygub + +Available online at https://aops.com/community/p27349508. + +## Problem statement + +Isosceles triangle $A B C$, with $A B=A C$, is inscribed in circle $\omega$. Let $D$ be an arbitrary point inside $B C$ such that $B D \neq D C$. Ray $A D$ intersects $\omega$ again at $E$ (other than $A$ ). Point $F$ (other than $E$ ) is chosen on $\omega$ such that $\angle D F E=90^{\circ}$. Line $F E$ intersects rays $A B$ and $A C$ at points $X$ and $Y$, respectively. Prove that $\angle X D E=\angle E D Y$. + +We present three solutions. +【 Angle chasing solution. Note that $(B D A)$ and $(C D A)$ are congruent, since $B A=C A$ and $\angle B D A+\angle C D A=180^{\circ}$. So these two circles are reflections around line $E D$. Moreover, $(D E F)$ is obviously also symmetric around line $E D$. +![](https://cdn.mathpix.com/cropped/2024_11_19_9c5e556ae0970c152728g-14.jpg?height=812&width=1009&top_left_y=1182&top_left_x=518) + +Hence, the radical axis of $(B D A)$ and $(D E F)$, and the radical axis of $(C D A)$ and $(D E F)$, should be symmetric about line $D E$. But these radical axii are exactly lines $X D$ and $Y D$, so we're done. + +Remark (Motivation). The main idea is that you can replace $D X$ and $D Y$ with the radical axii, letting $X^{\prime}$ and $Y^{\prime}$ be the second intersections of the blue circles. Then for the problem to be true, you'd need $X^{\prime}$ and $Y^{\prime}$ to be reflections. That's equivalent to $(B D A)$ and $(C D A)$ being congruent; you check it and it's indeed true. + +【 Harmonic solution (mine). Let $T$ be the point on line $\overline{X F E Y}$ such that $\angle E D T=90^{\circ}$, and let $\overline{A T}$ meet $\omega$ again at $K$. Then + +$$ +T D^{2}=T F \cdot T E=T K \cdot T A \Longrightarrow \angle D K T=90^{\circ} +$$ + +so line $D K$ passes through the antipode $M$ of $A$. +![](https://cdn.mathpix.com/cropped/2024_11_19_9c5e556ae0970c152728g-15.jpg?height=940&width=1394&top_left_y=315&top_left_x=334) + +Thus, + +$$ +-1=(A M ; C B)_{\omega} \stackrel{D}{=}(E K ; B C)_{\omega} \stackrel{A}{=}(T E ; X Y) +$$ + +and since $\angle E D T=90^{\circ}$ we're done. +Remark (Motivation). The idea is to kill the points $X$ and $Y$ by reinterpreting the desired condition as $(T D ; X Y)=-1$ and then projecting through $A$ onto $\omega$. This eliminates points $X$ and $Y$ altogether and reduces the problem to showing that $\overline{T A}$ passes through the harmonic conjugate of $E$ with respect to $B C$ on $\omega$. + +The labels on the diagram are slightly misleading in that $\triangle E B C$ should probably be thought of as the "reference" triangle. + +【 Pascal solution (Zuming Feng). Extend ray $F D$ to the antipode $T$ of $E$ on $\omega$. Then, + +- By Pascal's theorem on $E F T A B C$, the points $X, D$, and $P:=\overline{E C} \cap \overline{A T}$ are collinear. +- Similarly by Pascal's theorem on $E F T A C B$, the points the points $Y, D$, and $Q:=\overline{E B} \cap \overline{A T}$ are collinear. +![](https://cdn.mathpix.com/cropped/2024_11_19_9c5e556ae0970c152728g-16.jpg?height=740&width=1406&top_left_y=244&top_left_x=328) + +Now it suffices to prove $\overline{E D}$ bisects $\angle Q D P$. However, $\overline{E D}$ is the angle bisector of $\angle Q E P=\angle B E C$, but also $\overline{E A} \perp \overline{Q P}$. Thus triangle $Q E P$ is isosceles with $Q E=P E$, and $\overline{E A}$ cuts it in half. Since $D$ is on $\overline{E A}$, the result follows now. + diff --git a/USAJMO/md/en-JMO-2024-notes.md b/USAJMO/md/en-JMO-2024-notes.md new file mode 100644 index 0000000000000000000000000000000000000000..a4adb3790a1f0190065ca42e603348a35c0ba6ad --- /dev/null +++ b/USAJMO/md/en-JMO-2024-notes.md @@ -0,0 +1,429 @@ +# JMO 2024 Solution Notes + +Evan Chen《陳誼廷》 + +23 April 2024 + + +#### Abstract + +This is a compilation of solutions for the 2024 JMO. The ideas of the solution are a mix of my own work, the solutions provided by the competition organizers, and solutions found by the community. However, all the writing is maintained by me. + +These notes will tend to be a bit more advanced and terse than the "official" solutions from the organizers. In particular, if a theorem or technique is not known to beginners but is still considered "standard", then I often prefer to use this theory anyways, rather than try to work around or conceal it. For example, in geometry problems I typically use directed angles without further comment, rather than awkwardly work around configuration issues. Similarly, sentences like "let $\mathbb{R}$ denote the set of real numbers" are typically omitted entirely. + +Corrections and comments are welcome! + + +## Contents + +0 Problems ..... 2 +1 Solutions to Day 1 ..... 3 +1.1 JMO 2024/1, proposed by Evan o'Dorney ..... 3 +1.2 JMO 2024/2, proposed by Serena An and Claire Zhang ..... 5 +1.3 JMO 2024/3, proposed by John Berman ..... 8 +2 Solutions to Day 2 ..... 10 +2.1 JMO 2024/4, proposed by Alec Sun ..... 10 +2.2 JMO 2024/5, proposed by Carl Schildkraut ..... 12 +2.3 JMO 2024/6, proposed by Anton Trygub ..... 14 + +## §0 Problems + +1. Let $A B C D$ be a cyclic quadrilateral with $A B=7$ and $C D=8$. Points $P$ and $Q$ are selected on line segment $A B$ so that $A P=B Q=3$. Points $R$ and $S$ are selected on line segment $C D$ so that $C R=D S=2$. Prove that $P Q R S$ is a cyclic quadrilateral. +2. Let $m$ and $n$ be positive integers. Let $S$ be the set of lattice points $(x, y)$ with $1 \leq x \leq 2 m$ and $1 \leq y \leq 2 n$. A configuration of $m n$ rectangles is called happy if each point of $S$ is the vertex of exactly one rectangle. Prove that the number of happy configurations is odd. +3. A sequence $a_{1}, a_{2}, \ldots$ of positive integers is defined recursively by $a_{1}=2$ and + +$$ +a_{n+1}=a_{n}^{n+1}-1 \quad \text { for } n \geq 1 +$$ + +Prove that for every odd prime $p$ and integer $k$, some term of the sequence is divisible by $p^{k}$. +4. Let $n \geq 3$ be an integer. Rowan and Colin play a game on an $n \times n$ grid of squares, where each square is colored either red or blue. Rowan is allowed to permute the rows of the grid and Colin is allowed to permute the columns. A grid coloring is orderly if: + +- no matter how Rowan permutes the rows of the coloring, Colin can then permute the columns to restore the original grid coloring; and +- no matter how Colin permutes the columns of the coloring, Rowan can then permute the rows to restore the original grid coloring. +In terms of $n$, how many orderly colorings are there? + +5. Solve over $\mathbb{R}$ the functional equation $f\left(x^{2}-y\right)+2 y f(x)=f(f(x))+f(y)$. +6. Point $D$ is selected inside acute triangle $A B C$ so that $\angle D A C=\angle A C B$ and $\angle B D C=90^{\circ}+\angle B A C$. Point $E$ is chosen on ray $B D$ so that $A E=E C$. Let $M$ be the midpoint of $B C$. Show that line $A B$ is tangent to the circumcircle of triangle $B E M$. + +## §1 Solutions to Day 1 + +## §1.1 JMO 2024/1, proposed by Evan o'Dorney + +Available online at https://aops.com/community/p30216434. + +## Problem statement + +Let $A B C D$ be a cyclic quadrilateral with $A B=7$ and $C D=8$. Points $P$ and $Q$ are selected on line segment $A B$ so that $A P=B Q=3$. Points $R$ and $S$ are selected on line segment $C D$ so that $C R=D S=2$. Prove that $P Q R S$ is a cyclic quadrilateral. + +Here are two possible approaches. +\l The one-liner. The four points $P, Q, R, S$ have equal power -12 with respect to $(A B C D)$. So in fact they're on a circle concentric with $(A B C D)$. +\ The external power solution. We distinguish between two cases. + +Case where $A B$ and $C D$ are not parallel. We let lines $A B$ and $C D$ meet at $T$. Without loss of generality, $A$ lies between $B$ and $T$ and $D$ lies between $C$ and $T$. Let $x=T A$ and $y=T D$, as shown below. +![](https://cdn.mathpix.com/cropped/2024_11_19_805564b9ec1a761a333dg-03.jpg?height=541&width=812&top_left_y=1346&top_left_x=633) + +By power of a point, + +$$ +\begin{aligned} +A B C D \text { cyclic } \Longleftrightarrow x(x+7) & =y(y+8) \\ +P Q R S \text { cyclic } \Longleftrightarrow(x+3)(x+4) & =(y+2)(y+6) . +\end{aligned} +$$ + +However, the latter equation is just the former with 12 added to both sides. (That is, $(x+3)(x+4)=x(x+7)+12$ while $(y+2)(y+6)=y(y+8)+12$.$) So the conclusion is$ immediate. + +Case where $A B$ and $C D$ are parallel. In that case $A B C D$ is an isosceles trapezoid. Then the entire picture is symmetric around the common perpendicular bisector of the lines $A B$ and $C D$. Now $P Q R S$ is also an isosceles trapezoid, so it's cyclic too. +![](https://cdn.mathpix.com/cropped/2024_11_19_805564b9ec1a761a333dg-04.jpg?height=420&width=420&top_left_y=244&top_left_x=818) + +## §1.2 JMO 2024/2, proposed by Serena An and Claire Zhang + +Available online at https://aops.com/community/p30216444. + +## Problem statement + +Let $m$ and $n$ be positive integers. Let $S$ be the set of lattice points $(x, y)$ with $1 \leq x \leq 2 m$ and $1 \leq y \leq 2 n$. A configuration of $m n$ rectangles is called happy if each point of $S$ is the vertex of exactly one rectangle. Prove that the number of happy configurations is odd. + +There are several possible approaches to the problem; most of them involve pairing some of the happy configurations in various ways, leaving only a few configurations which remain fixed. We present the original proposer's solution and Evan's more complicated one. + +I Original proposer's solution. To this end, let's denote by $f(2 m, 2 n)$ the number of happy configurations for a $2 m \times 2 n$ grid of lattice points (not necessarily equally spaced - this doesn't change the count). We already have the following easy case. + +Claim - We have $f(2,2 n)=(2 n-1)!!=(2 n-1) \cdot(2 n-3) \cdots \cdots 3 \cdot 1$. +Proof. The top row is the top edge of some rectangle and there are $2 n-1$ choices for the bottom edge of that rectangle. It then follows $f(2,2 n)=(2 n-1) \cdot f(2,2 n-2)$ and the conclusion follows by induction on $n$, with $f(2,2)=1$. + +We will prove that: +Claim - Assume $m, n \geq 1$. When $f(2 m, 2 n) \equiv f(2 m-2,2 n)(\bmod 2)$. +Proof. Given a happy configuration $\mathcal{C}$, let $\tau(\mathcal{C})$ be the happy configuration obtained by swapping the last two columns. Obviously $\tau(\tau(\mathcal{C}))=\mathcal{C}$ for every happy $\mathcal{C}$. So in general, we can consider two different kinds of configurations $\mathcal{C}$, those for which $\tau(\mathcal{C}) \neq \mathcal{C}$, so we get pairs $\{\mathcal{C}, \tau(\mathcal{C})\}$, and those with $\tau(\mathcal{C})=\mathcal{C}$. + +Now configurations fixed by $\tau$ can be described readily: this occurs if and only if the last two columns are self-contained, meaning every rectangle with a vertex in these columns is completely contained in these two columns. +![](https://cdn.mathpix.com/cropped/2024_11_19_805564b9ec1a761a333dg-06.jpg?height=809&width=847&top_left_y=241&top_left_x=596) + +Hence it follows that + +$$ +f(2 m, 2 n)=2(\text { number of pairs })+f(2 m-2,2 n) \cdot f(2,2 n) +$$ + +Taking modulo 2 gives the result. +By the same token $f(2 m, 2 n) \equiv f(2 m, 2 n-2)(\bmod 2)$. So all $f$-values have the same parity, and from $f(2,2)=1$ we're done. + +Remark. There are many variations of the solution using different kinds of $\tau$. The solution with $\tau$ swapping two rows seems to be the simplest. +\l Evan's permutation-based solution. Retain the notation $f(2 m, 2 n)$ from before. Given a happy configuration, consider all the rectangles whose left edge is in the first column. Highlight every column containing the right edge of such a rectangle. For example, in the figure below, there are two highlighted columns. (The rectangles are drawn crooked so one can tell them apart.) +![](https://cdn.mathpix.com/cropped/2024_11_19_805564b9ec1a761a333dg-06.jpg?height=515&width=832&top_left_y=1861&top_left_x=612) + +We organize happy configurations based on the set of highlighted columns. Specifically, define the relation $\sim$ on configurations by saying that $\mathcal{C} \sim \mathcal{C}^{\prime}$ if they differ by any permutation of the highlighted columns. This is an equivalence relation. And in general, if there are $k$ highlighted columns, its equivalence class under $\sim$ has $k$ ! elements. + +Then + +Claim - $f(2 m, 2 n)$ has the same parity as the number of happy configurations with exactly one highlighted column. + +Proof. Since $k$ ! is even for all $k \geq 2$, but odd when $k=1$. +There are $2 m-1$ ways to pick a single highlighted column, and then $f(2,2 n)=(2 n-1)!!$ ways to use the left column and highlighted column. So the count in the claim is exactly given by + +$$ +(2 m-1) \cdot(2 n-1)!!f(2 m-2,2 n) +$$ + +This implies $f(2 m, 2 n) \equiv f(2 m-2,2 n)(\bmod 2)$ and proceeding by induction as before solves the problem. + +## §1.3 JMO 2024/3, proposed by John Berman + +Available online at https://aops.com/community/p30216450. + +## Problem statement + +A sequence $a_{1}, a_{2}, \ldots$ of positive integers is defined recursively by $a_{1}=2$ and + +$$ +a_{n+1}=a_{n}^{n+1}-1 \quad \text { for } n \geq 1 +$$ + +Prove that for every odd prime $p$ and integer $k$, some term of the sequence is divisible by $p^{k}$. + +We start with the following. +Claim - Assume $n$ is a positive integer divisible by $p-1$. Then either $a_{n-1} \equiv 0$ $(\bmod p)$ or $a_{n} \equiv 0(\bmod p)$. + +Proof. Suppose that $a_{n-1} \not \equiv 0(\bmod p)$. Then by Fermat's little theorem, + +$$ +a_{n}=a_{n-1}^{n}-1 \equiv x^{p-1}-1 \equiv 0 \quad(\bmod p) +$$ + +where $x:=a_{n-1}^{\frac{n}{p-1}}$ is an integer not divisible by $p$. +Claim - If $n \geq 2$ is even, then + +$$ +a_{n}^{n+1} \mid a_{n+2} +$$ + +Proof. Note $a_{n+2}=a_{n+1}^{n+2}-1$. The right-hand side is a difference of $(n+2)^{\text {nd }}$ powers, which for even $n$ is divisible by $a_{n+1}+1=a_{n}^{n+1}$. + +By considering multiples $n$ of $p-1$ which are larger than $k$, we see that if $a_{n} \equiv 0$ $(\bmod p)$ ever happens, we are done by combining the two previous claims. So the difficult case of the problem is the bad situation where $a_{n-1} \equiv 0(\bmod p)$ occurs for almost all $n \equiv 0(\bmod p-1)$. + +To resolve the difficult case and finish the problem, we zoom in on specific $n$ that will let us use lifting the exponent on $a_{n-1}$. + +Claim - Suppose $n$ is an (even) integer satisfying + +$$ +\begin{aligned} +& n \equiv 0 \quad(\bmod p-1) \\ +& n \equiv 1 \quad\left(\bmod p^{k-1}\right) +\end{aligned} +$$ + +If $a_{n-1} \equiv 0(\bmod p)$, then in fact $p^{k} \mid a_{n-1}$. +Proof. Write $a_{n-1}=a_{n-2}^{n-1}-1$. We know $a_{n-2} \not \equiv 0(\bmod p)$, and so $a_{n-2}^{n} \equiv 1(\bmod p)$. Taking modulo $p$ gives + +$$ +0 \equiv \frac{1}{a_{n-2}}-1 \quad(\bmod p) \Longrightarrow a_{n-2} \equiv 1 \quad(\bmod p) +$$ + +Hence lifting the exponent applies and we get + +$$ +\nu_{p}\left(a_{n-1}\right) \equiv \nu_{p}\left(a_{n-2}^{n-1}-1\right)=\nu_{p}\left(a_{n-2}-1\right)+\nu_{p}(n-1) \geq 1+(k-1)=k +$$ + +as desired. + +Remark. The first few terms are $a_{1}=2, a_{2}=3, a_{3}=26, a_{4}=456975$, and $a_{5}=$ 19927930852449199486318359374, ... No element of the sequence is divisible by 4: the residues modulo 4 alternate between 2 and 3 . + +Remark. The second claim is important for the solution to work once $k \geq 2$. One could imagine a variation of the first claim that states if $n$ is divisible by $\varphi\left(p^{k}\right)=p^{k-1}(p-1)$, then either $a_{n-1} \equiv 0(\bmod p)$ or $a_{n} \equiv 0\left(\bmod p^{k}\right)$. However this gives an obstruction (for $k \geq 2)$ where we are guaranteed to have $n-1 \not \equiv 0(\bmod p)$ now, so lifting the exponent will never give additional factors of $p$ we want. + +## §2 Solutions to Day 2 + +## §2.1 JMO 2024/4, proposed by Alec Sun + +Available online at https://aops.com/community/p30227193. + +## Problem statement + +Let $n \geq 3$ be an integer. Rowan and Colin play a game on an $n \times n$ grid of squares, where each square is colored either red or blue. Rowan is allowed to permute the rows of the grid and Colin is allowed to permute the columns. A grid coloring is orderly if: + +- no matter how Rowan permutes the rows of the coloring, Colin can then permute the columns to restore the original grid coloring; and +- no matter how Colin permutes the columns of the coloring, Rowan can then permute the rows to restore the original grid coloring. +In terms of $n$, how many orderly colorings are there? + +The answer is $2 n!+2$. In fact, we can describe all the orderly colorings as follows: + +- The all-blue coloring. +- The all-red coloring. +- Each of the $n$ ! colorings where every row/column has exactly one red cell. +- Each of the $n$ ! colorings where every row/column has exactly one blue cell. + +These obviously work; we turn our attention to proving these are the only ones. +For the other direction, fix a orderly coloring $\mathcal{A}$. +Consider any particular column $C$ in $\mathcal{A}$ and let $m$ denote the number of red cells that $C$ has. Any row permutation (say $\sigma$ ) that Rowan chooses will transform $C$ into some column $\sigma(C)$, and our assumption requires $\sigma(C)$ has to appear somewhere in the original assignment $\mathcal{A}$. An example for $n=7, m=2$, and a random $\sigma$ is shown below. +![](https://cdn.mathpix.com/cropped/2024_11_19_805564b9ec1a761a333dg-10.jpg?height=692&width=815&top_left_y=1910&top_left_x=618) + +On the other hand, the number of possible patterns of $\sigma(C)$ is easily seen to be exactly ( $\binom{n}{m}$ - and they must all appear. In particular, if $m \notin\{0,1, n-1, n\}$, then we immediately get a contradiction because $\mathcal{A}$ would need too many columns (there are only $n$ columns in $\mathcal{A}$, which is fewer than $\binom{n}{m}$ ). Moreover, if either $m=1$ or $m=n-1$, these columns are all the columns of $\mathcal{A}$; these lead to the $2 n$ ! main cases we found before. + +The only remaining case is when $m \in\{0, n\}$ for every column, i.e. every column is monochromatic. It's easy to see in that case the columns must all be the same color. + +## §2.2 JMO 2024/5, proposed by Carl Schildkraut + +Available online at https://aops.com/community/p30227204. + +## Problem statement + +Solve over $\mathbb{R}$ the functional equation $f\left(x^{2}-y\right)+2 y f(x)=f(f(x))+f(y)$. + +The answer is $f(x) \equiv x^{2}, f(x) \equiv 0, f(x) \equiv-x^{2}$, which obviously work. +Let $P(x, y)$ be the usual assertion. +Claim - We have $f(0)=0$ and $f$ even. + +Proof. Combine $P(1,1 / 2)$ with $P(1,0)$ to get $f(0)=0$. Use $P(0, y)$ to deduce $f$ is even. + +Claim $-f(x) \in\left\{-x^{2}, 0, x^{2}\right\}$ for every $x \in \mathbb{R}$. +Proof. Note that $P\left(x, x^{2} / 2\right)$ and $P(x, 0)$ respectively give + +$$ +x^{2} f(x)=f\left(x^{2}\right)=f(f(x)) +$$ + +Repeating this key identity several times gives + +$$ +\begin{aligned} +f(f(f(x))) & =f\left(f\left(x^{2}\right)\right)=f\left(x^{4}\right)=x^{4} f\left(x^{2}\right) \\ +& =f(x)^{2} \cdot f(f(x))=f(x)^{2} f\left(x^{2}\right)=f(x)^{3} x^{2} +\end{aligned} +$$ + +Suppose $t \neq 0$ is such that $f\left(t^{2}\right) \neq 0$. Then the above equalities imply + +$$ +t^{4} f\left(t^{2}\right)=f(t)^{2} f\left(t^{2}\right) \Longrightarrow f(t)= \pm t^{2} +$$ + +and then + +$$ +f(t)^{2} f\left(t^{2}\right)=f(t)^{3} t^{2} \Longrightarrow f\left(t^{2}\right)= \pm t^{2} +$$ + +Together with $f$ even, we get the desired result. + +Remark. Another proof is possible here that doesn't use as iterations of $f$ : the idea is to "show $f$ is injective up to sign outside its kernel". Specifically, if $f(a)=f(b) \neq 0$, then $a^{2} f(a)=f(f(a))=f(f(b))=b^{2} f(b) \Longrightarrow a^{2}=b^{2}$. But we also have $f(f(x))=f\left(x^{2}\right)$, so we are done except in the case $f(f(x))=f\left(x^{2}\right)=0$. That would imply $x^{2} f(x)=0$, so the claim follows. + +Now, note that $P(1, y)$ gives + +$$ +f(1-y)+2 y \cdot f(1)=f(1)+f(y) +$$ + +We consider cases on $f(1)$ and show that $f$ matches the desired form. + +- If $f(1)=1$, then $f(1-y)+(2 y-1)=f(y)$. Consider the nine possibilities that arise: + +$$ +\begin{array}{lll} +(1-y)^{2}+(2 y-1)=y^{2} & 0+(2 y-1)=y^{2} & -(1-y)^{2}+(2 y-1)=y^{2} \\ +(1-y)^{2}+(2 y-1)=0 & 0+(2 y-1)=0 & -(1-y)^{2}+(2 y-1)=0 \\ +(1-y)^{2}+(2 y-1)=-y^{2} & 0+(2 y-1)=-y^{2} & -(1-y)^{2}+(2 y-1)=-y^{2} +\end{array} +$$ + +Each of the last eight equations is a nontrivial polynomial equation. Hence, there is some constant $C>100$ such that the latter eight equations are all false for any real number $y>C$. Consequently, $f(y)=y^{2}$ for $y>C$. +Finally, for any real number $z>0$, take $x, y>C$ such that $x^{2}-y=z$; then $P(x, y)$ proves $f(z)=z^{2}$ too. + +- Note that (as $f$ is even), $f$ works iff $-f$ works, so the case $f(1)=-1$ is analogous. +- If $f(1)=0$, then $f(1-y)=f(y)$. Hence for any $y$ such that $|1-y| \neq|y|$, we conclude $f(y)=0$. Then take $P(2,7 / 2) \Longrightarrow f(1 / 2)=0$. + +Remark. There is another clever symmetry approach possible after the main claim. The idea is to write + +$$ +P\left(x, y^{2}\right) \Longrightarrow f\left(x^{2}-y^{2}\right)+2 y^{2} f(x)=f(f(x))+f(f(y)) +$$ + +Since $f$ is even gives $f\left(x^{2}-y^{2}\right)=f\left(y^{2}-x^{2}\right)$, one can swap the roles of $x$ and $y$ to get $2 y^{2} f(x)=2 x^{2} f(y)$. Set $y=1$ to finish. + +## §2.3 JMO 2024/6, proposed by Anton Trygub + +Available online at https://aops.com/community/p30227196. + +## Problem statement + +Point $D$ is selected inside acute triangle $A B C$ so that $\angle D A C=\angle A C B$ and $\angle B D C=$ $90^{\circ}+\angle B A C$. Point $E$ is chosen on ray $B D$ so that $A E=E C$. Let $M$ be the midpoint of $B C$. Show that line $A B$ is tangent to the circumcircle of triangle $B E M$. + +This problem has several approaches and we showcase a collection of them. +ब The author's original solution. Complete isosceles trapezoid $A B Q C$ (so $D \in \overline{A Q}$ ). Reflect $B$ across $E$ to point $F$. +![](https://cdn.mathpix.com/cropped/2024_11_19_805564b9ec1a761a333dg-14.jpg?height=663&width=718&top_left_y=959&top_left_x=675) + +Claim - We have $D Q C F$ is cyclic. +Proof. Since $E A=E C$, we have $\overline{Q F} \perp \overline{A C}$ as line $Q F$ is the image of the perpendicular bisector of $\overline{A C}$ under a homothety from $B$ with scale factor 2 . Then + +$$ +\begin{aligned} +\measuredangle F D C & =-\measuredangle C D B=180^{\circ}-\left(90^{\circ}+\measuredangle C A B\right)=90^{\circ}-\measuredangle C A B \\ +& =90^{\circ}-\measuredangle Q C A=\measuredangle F Q C . +\end{aligned} +$$ + +To conclude, note that + +$$ +\measuredangle B E M=\measuredangle B F C=\measuredangle D F C=\measuredangle D Q C=\measuredangle A Q C=\measuredangle A B C=\measuredangle A B M +$$ + +Remark (Motivation). Here is one possible way to come up with the construction of point $F$ (at least this is what led Evan to find it). If one directs all the angles in the obvious way, there are really two points $D$ and $D^{\prime}$ that are possible, although one is outside the triangle; they give corresponding points $E$ and $E^{\prime}$. The circles $B E M$ and $B E^{\prime} M$ must then actually coincide since they are both alleged to be tangent to line $A B$. See the figure below. +![](https://cdn.mathpix.com/cropped/2024_11_19_805564b9ec1a761a333dg-15.jpg?height=1112&width=1047&top_left_y=238&top_left_x=513) + +One can already prove using angle chasing that $\overline{A B}$ is tangent to $\left(B E E^{\prime}\right)$. So the point of the problem is to show that $M$ lies on this circle too. However, from looking at the diagram, one may realize that in fact it seems + +$$ +\triangle M E E^{\prime} \stackrel{ }{\sim} \triangle C D D^{\prime} +$$ + +is going to be true from just those marked in the figure (and this would certainly imply the desired concyclic conclusion). Since $M$ is a midpoint, it makes sense to dilate $\triangle E M E^{\prime}$ from $B$ by a factor of 2 to get $\triangle F C F^{\prime}$ so that the desired similarity is actually a spiral similarity at $C$. Then the spiral similarity lemma says that the desired similarity is equivalent to requiring $\overline{D D^{\prime}} \cap \overline{F F^{\prime}}=Q$ to lie on both $(C D F)$ and $\left(C D^{\prime} F^{\prime}\right)$. Hence the key construction and claim from the solution are both discovered naturally, and we find the solution above. (The points $D^{\prime}, E^{\prime}, F^{\prime}$ can then be deleted to hide the motivation.) + +Another short solution. Let $Z$ be on line $B D E$ such that $\angle B A Z=90^{\circ}$. This lets us interpret the angle condition as follows: + +$$ +\text { Claim - Points } A, D, Z, C \text { are cyclic. } +$$ + +Proof. Because $\measuredangle Z A C=90^{\circ}-A=180^{\circ}-\measuredangle C D B=\measuredangle Z D C$. +![](https://cdn.mathpix.com/cropped/2024_11_19_805564b9ec1a761a333dg-16.jpg?height=804&width=701&top_left_y=249&top_left_x=683) + +Define $W$ as the midpoint of $\overline{B Z}$, so $\overline{M W} \| \overline{C Z}$. And let $O$ denote the center of $(A B C)$. + +Claim - Points $M, E, O, W$ are cyclic. + +Proof. Note that + +$$ +\begin{aligned} +\measuredangle M O E & =\measuredangle(\overline{O M}, \overline{B C})+\measuredangle(\overline{B C}, \overline{A C})+\measuredangle(\overline{A C}, \overline{O E}) \\ +& =90^{\circ}+\measuredangle B C A+90^{\circ} \\ +& =\measuredangle B C A=\measuredangle C A D=\measuredangle C Z D=\measuredangle M W D=\measuredangle M W E . +\end{aligned} +$$ + +To finish, note + +$$ +\begin{aligned} +\measuredangle M E B & =\measuredangle M E W=\measuredangle M O W \\ +& =\measuredangle(\overline{M O}, \overline{B C})+\measuredangle(\overline{B C}, \overline{A B})+\measuredangle(\overline{A B}, \overline{O W}) \\ +& =90^{\circ}+\measuredangle C B A+90^{\circ}=\measuredangle C B A=\measuredangle M B A . +\end{aligned} +$$ + +This implies the desired tangency. + +I A Menelaus-based approach (Kevin Ren). Let $P$ be on $\overline{B C}$ with $A P=P C$. Let $Y$ be the point on line $A B$ such that $\angle A C Y=90^{\circ}$; as $\angle A Y C=90^{\circ}-A$ it follows $B D Y C$ is cyclic. Let $K=\overline{A P} \cap \overline{C Y}$, so $\triangle A C K$ is a right triangle with $P$ the midpoint of its hypotenuse. +![](https://cdn.mathpix.com/cropped/2024_11_19_805564b9ec1a761a333dg-17.jpg?height=1192&width=1075&top_left_y=249&top_left_x=496) + +Claim - Triangles BPE and DYK are similar. +Proof. We have $\measuredangle M P E=\measuredangle C P E=\measuredangle K C P=\measuredangle P K C$ and $\measuredangle E B P=\measuredangle D B C=$ $\measuredangle D Y C=\measuredangle D Y K$. + +Claim - Triangles $B E M$ and $Y D C$ are similar. + +Proof. By Menelaus $\triangle P C K$ with respect to collinear points $A, B, Y$ that + +$$ +\frac{B P}{B C} \frac{Y C}{Y K} \frac{A K}{A P}=1 +$$ + +Since $A K / A P=2$ (note that $P$ is the midpoint of the hypotenuse of right triangle $A C K)$ and $B C=2 B M$, this simplifies to + +$$ +\frac{B P}{B M}=\frac{Y K}{Y C} +$$ + +To finish, note that + +$$ +\measuredangle D B A=\measuredangle D B Y=\measuredangle D C Y=\measuredangle B M E +$$ + +implying the desired tangency. + +『 A spiral similarity approach (Hans $\mathbf{Y u}$ ). As in the previous solution, let $Y$ be the point on line $A B$ such that $\angle A C Y=90^{\circ}$; so $B D Y C$ is cyclic. Let $\Gamma$ be the circle through $B$ and $M$ tangent to $\overline{A B}$, and let $\Omega:=(B C Y D)$. We need to show $E \in \Gamma$. +![](https://cdn.mathpix.com/cropped/2024_11_19_805564b9ec1a761a333dg-18.jpg?height=1101&width=1029&top_left_y=429&top_left_x=519) + +Denote by $S$ the second intersection of $\Gamma$ and $\Omega$. The main idea behind is to consider the spiral similarity + +$$ +\Psi: \Omega \rightarrow \Gamma \quad C \mapsto M \text { and } Y \mapsto B +$$ + +centered at $S$ (due to the spiral similarity lemma), and show that $\Psi(D)=E$. The spiral similarity lemma already promises $\Psi(D)$ lies on line $B D$. + +Claim - We have $\Psi(A)=O$, the circumcenter of $A B C$. + +Proof. Note $\triangle O B M \AA \triangle A Y C$; both are right triangles with $\measuredangle B A C=\measuredangle B O M$. + +Claim - $\Psi$ maps line $A D$ to line $O P$. + +Proof. If we let $P$ be on $\overline{B C}$ with $A P=P C$ as before, + +$$ +\measuredangle(\overline{A D}, \overline{O P})=\measuredangle A P O=\measuredangle O P C=\measuredangle Y C P=\measuredangle(\overline{Y C}, \overline{B M}) +$$ + +As $\Psi$ maps line $Y C$ to line $B M$ and $\Psi(A)=O$, we're done. +Hence $\Psi(D)$ should not only lie on $B D$ but also line $O P$. This proves $\Psi(D)=E$, so $E \in \Gamma$ as needed. + diff --git a/USAJMO/raw/en-JMO-2010-notes.pdf b/USAJMO/raw/en-JMO-2010-notes.pdf new file mode 100644 index 0000000000000000000000000000000000000000..762573f0f6b20176f0af95de5e6d4840480622e0 --- /dev/null +++ b/USAJMO/raw/en-JMO-2010-notes.pdf @@ -0,0 +1,3 @@ +version https://git-lfs.github.com/spec/v1 +oid sha256:ac1fd689e5980b21e9e4844b788b18b69f06f54d3a4716fe4e14f7b1be98d569 +size 201220 diff --git a/USAJMO/raw/en-JMO-2011-notes.pdf b/USAJMO/raw/en-JMO-2011-notes.pdf new file mode 100644 index 0000000000000000000000000000000000000000..80db28c18e6db19f31714571a92f0822b8da42f4 --- /dev/null +++ b/USAJMO/raw/en-JMO-2011-notes.pdf @@ -0,0 +1,3 @@ +version https://git-lfs.github.com/spec/v1 +oid sha256:c3f28d1babb3a2d96fb9fd2da331f2e7e1b4229c4c6eaf0b74d88889690e05c8 +size 174383 diff --git a/USAJMO/raw/en-JMO-2012-notes.pdf b/USAJMO/raw/en-JMO-2012-notes.pdf new file mode 100644 index 0000000000000000000000000000000000000000..d025f1c0fb27e3488338142a4b4001d21fa06744 --- /dev/null +++ b/USAJMO/raw/en-JMO-2012-notes.pdf @@ -0,0 +1,3 @@ +version https://git-lfs.github.com/spec/v1 +oid sha256:2cfd0a7d751932737fd9c8e1b87d93be681becd16abe000d4f9afc3065ec830a +size 238137 diff --git a/USAJMO/raw/en-JMO-2013-notes.pdf b/USAJMO/raw/en-JMO-2013-notes.pdf new file mode 100644 index 0000000000000000000000000000000000000000..95bba459a9ad0e16a66a96d434046e42b2c64021 --- /dev/null +++ b/USAJMO/raw/en-JMO-2013-notes.pdf @@ -0,0 +1,3 @@ +version https://git-lfs.github.com/spec/v1 +oid sha256:98ef8e6e094d2a68b54fbc9321d17e6b6756498898b2e121c007e9985b0340eb +size 182635 diff --git a/USAJMO/raw/en-JMO-2014-notes.pdf b/USAJMO/raw/en-JMO-2014-notes.pdf new file mode 100644 index 0000000000000000000000000000000000000000..465c7b7430fd19e4452f5584ec36f59bdb1396e7 --- /dev/null +++ b/USAJMO/raw/en-JMO-2014-notes.pdf @@ -0,0 +1,3 @@ +version https://git-lfs.github.com/spec/v1 +oid sha256:260b3ea5bf594587d31c1073b4a7d0825cc3b10eab21a4bde91397396a9a5ad1 +size 205736 diff --git a/USAJMO/raw/en-JMO-2015-notes.pdf b/USAJMO/raw/en-JMO-2015-notes.pdf new file mode 100644 index 0000000000000000000000000000000000000000..5a0c7c03bd367fdd742c45051f8a5f9e6a1d6dc9 --- /dev/null +++ b/USAJMO/raw/en-JMO-2015-notes.pdf @@ -0,0 +1,3 @@ +version https://git-lfs.github.com/spec/v1 +oid sha256:71ad4eb8c117fd3601982f52c0df1dcec617b36d96a0742f5f262c0c627066b6 +size 245186 diff --git a/USAJMO/raw/en-JMO-2016-notes.pdf b/USAJMO/raw/en-JMO-2016-notes.pdf new file mode 100644 index 0000000000000000000000000000000000000000..2549be8a982b46db6796775135ce039a2724a14e --- /dev/null +++ b/USAJMO/raw/en-JMO-2016-notes.pdf @@ -0,0 +1,3 @@ +version https://git-lfs.github.com/spec/v1 +oid sha256:51d72fe1be8318d501b54bdd540947b969bd1dca89ecc601c94de693e4bb6f15 +size 273742 diff --git a/USAJMO/raw/en-JMO-2017-notes.pdf b/USAJMO/raw/en-JMO-2017-notes.pdf new file mode 100644 index 0000000000000000000000000000000000000000..f81ab8b5399f030724a3c8c438adf30655bb2b45 --- /dev/null +++ b/USAJMO/raw/en-JMO-2017-notes.pdf @@ -0,0 +1,3 @@ +version https://git-lfs.github.com/spec/v1 +oid sha256:717e0a880a5ab14af135caf0e220e9ed39a7ea49764df6406719f16781e8501b +size 275553 diff --git a/USAJMO/raw/en-JMO-2018-notes.pdf b/USAJMO/raw/en-JMO-2018-notes.pdf new file mode 100644 index 0000000000000000000000000000000000000000..afc6b06b01f8bb22b32b9589bab94679ead5fe4b --- /dev/null +++ b/USAJMO/raw/en-JMO-2018-notes.pdf @@ -0,0 +1,3 @@ +version https://git-lfs.github.com/spec/v1 +oid sha256:ae62cbf19c544a9ac936f5933d9bfc5491e29eb0e464514b42526b7fb601f82b +size 209348 diff --git a/USAJMO/raw/en-JMO-2019-notes.pdf b/USAJMO/raw/en-JMO-2019-notes.pdf new file mode 100644 index 0000000000000000000000000000000000000000..af2a8cf3bbb21bf20b6e98a6a43594cc271a6d45 --- /dev/null +++ b/USAJMO/raw/en-JMO-2019-notes.pdf @@ -0,0 +1,3 @@ +version https://git-lfs.github.com/spec/v1 +oid sha256:d06c29dd8faea20eba408404c195294b2b990fce447afd4954e779daa3fc6bf3 +size 354952 diff --git a/USAJMO/raw/en-JMO-2020-notes.pdf b/USAJMO/raw/en-JMO-2020-notes.pdf new file mode 100644 index 0000000000000000000000000000000000000000..fd267f9c0104db107203504a86de4e100a5df808 --- /dev/null +++ b/USAJMO/raw/en-JMO-2020-notes.pdf @@ -0,0 +1,3 @@ +version https://git-lfs.github.com/spec/v1 +oid sha256:ca621a761f1b052de83d1fb0677afc9d3da78349b8c20cc99a7fcc211fe1a7bc +size 347432 diff --git a/USAJMO/raw/en-JMO-2021-notes.pdf b/USAJMO/raw/en-JMO-2021-notes.pdf new file mode 100644 index 0000000000000000000000000000000000000000..8537d7f8152349358520bb13a20d4e7e36f73f8a --- /dev/null +++ b/USAJMO/raw/en-JMO-2021-notes.pdf @@ -0,0 +1,3 @@ +version https://git-lfs.github.com/spec/v1 +oid sha256:2d193e9ca0dab15ae0959888329818412c409536317aa9b292153ad1eebed842 +size 208350 diff --git a/USAJMO/raw/en-JMO-2022-notes.pdf b/USAJMO/raw/en-JMO-2022-notes.pdf new file mode 100644 index 0000000000000000000000000000000000000000..a6c023cd09651a6fa92495714c1155e34345a8db --- /dev/null +++ b/USAJMO/raw/en-JMO-2022-notes.pdf @@ -0,0 +1,3 @@ +version https://git-lfs.github.com/spec/v1 +oid sha256:47bed9738bc2928cfdf41872a90da5ae5b60ce88f91e3558b2b9220c1a6cf73d +size 318400 diff --git a/USAJMO/raw/en-JMO-2023-notes.pdf b/USAJMO/raw/en-JMO-2023-notes.pdf new file mode 100644 index 0000000000000000000000000000000000000000..6c65742335b595c744b2f140c2dfa447b2cf0224 --- /dev/null +++ b/USAJMO/raw/en-JMO-2023-notes.pdf @@ -0,0 +1,3 @@ +version https://git-lfs.github.com/spec/v1 +oid sha256:d756e3972b26068fa6d0797a0ff903721169f7e9711908f2b8d2ebe9dd3ad230 +size 290700 diff --git a/USAJMO/raw/en-JMO-2024-notes.pdf b/USAJMO/raw/en-JMO-2024-notes.pdf new file mode 100644 index 0000000000000000000000000000000000000000..0abcfc2e74f71c7c3e3bbedbdf7699a95a874b23 --- /dev/null +++ b/USAJMO/raw/en-JMO-2024-notes.pdf @@ -0,0 +1,3 @@ +version https://git-lfs.github.com/spec/v1 +oid sha256:42716cc3b1e58dd988337920a215dcd68aaab111f51b7d44222a7653bee0b32b +size 397163 diff --git a/USAJMO/segment_script/segment_usamo.py b/USAJMO/segment_script/segment_usamo.py new file mode 100644 index 0000000000000000000000000000000000000000..0fb17da47cecbb1c7443d21332018f569c13d2ff --- /dev/null +++ b/USAJMO/segment_script/segment_usamo.py @@ -0,0 +1,306 @@ +# ----------------------------------------------------------------------------- +# Author: Marina +# Date: 2024-11-15 +# ----------------------------------------------------------------------------- +''' Script to segment md files in en-usamo, en-tstst, en-tst, en-jmo folder using regex. +To run: +`python segment_script/segment_usamo.py` +''' + +import warnings +warnings.filterwarnings("ignore", category=DeprecationWarning) + +import os +import re +import pandas as pd +from rapidfuzz import fuzz + + +# "## §0 Problems" -> match +section_re = re.compile(r"^#{1,2}\s(?:Contents|Problem|§[\d.]+.*)") +# section_re = re.compile(r"^#{1,2}\s(?:§[\d.]+.*)") + +# "## §1.3 USAMO 2024/3, proposed by Krit Boonsiriseth" ->3 +# "## §2.3 TSTST 2011/6" ->6 +# "## §1.1 TSTST 2021/1, proposed by Holden Mui" -> 1 +# "## §1.2 USA TST 2020/2, proposed by Merlijn Staps" -> 2 +# "# §1.3 USA TST 2019/3, proposed by Nikolai Beluhov" -> 3 +solution_label_re = re.compile( + r"^#{1,2}\s§[\d.]+\s[A-Za-z0-9 ]+\s\d{4}/(\d+)(?:,\s.*)?$" +) + +# "1. Prove that the average of the numbers $n" -> match +# "2 . For any nonempty set $S$ of real numbers,"" -> match +problem_re = re.compile(r"^(\d+)\s?\.\s(.*(?:\n\s+.*)*)") + +# "## Problem statement extra text" -> match +# "Problem statement" -> match +# "##Problem statement (missing space)" -> match +solution_re = re.compile(r"^#{0,2}\s?Problem statement\b.*$") + +# not actually used, only for debugging +pattern_debug = re.compile( + r"^[【『\\]*.*?\b(First|Second|Third|Fourth|Fifth|Sixth|Seventh|Eighth|Ninth|Tenth|Complex|Inversion|Synthetic|One|Another|Solution)\b.*\b(solution|approach|proof)\b.*", + re.IGNORECASE +) + +# "Solution 1" -> match +# "【 Solution 1" -> match +solution_split_re1 = re.compile(r"\bSolution\s[1-9]\b") + +# "First solution" -> match +# "【 Third approach" -> match +# "【 Second approach" -> match +solution_split_re2 = re.compile(r"\b(First|Second|Third|Fourth|Fifth|Sixth|Seventh|Eighth|Ninth|Synthetic)\b\s+(solution|approach|proof)\b") + +# catch special cases (ugly but it works). To generate them run script with `DEBUG = True`` +# and identify in console which special_cases should be catched as first line of a solution +DEBUG = False +special_cases = [ +"【 First short solution, by Jeffrey Kwan. Let $p_{0", +"II Second longer solution using an invariant. Visu", +"【 Complex solution (Evan Chen). Toss on the comple", +"Second (longer) solution. If one does not notice t", +"『 Second calculation approach (along the lines of ", +"T Outline of second approach (by convexity, due t", +"I Inversion solution submitted by Ankan Bhattacha", +"【 Complex numbers approach with Apollonian circles", +" A second solution. Both lemmas above admit varia", +"【 A third remixed solution. We use Lemma I and Lem", +"【I A fourth remixed solution. We also can combine ", +"I First grid-based solution. The following solutio", +"Another short solution. Let $Z$ be on line $B D E$", +"【 Most common synthetic approach. The solution hin", +"\\ First \"local\" solution by swapping two points. L", +"Second general solution by angle chasing. By Rei", +"Third general solution by Pascal. Extend rays $A", +"【 Second length solution by tangent lengths. By $t", +"【 Angle chasing solution. Note that $(B D A)$ and", +"【 Harmonic solution (mine). Let $T$ be the point o", +"【 Pascal solution (Zuming Feng). Extend ray $F D$", +"『 A spiral similarity approach (Hans $\mathbf{Y u}", +"ब The author's original solution. Complete isoscel", +"l Evan's permutation-based solution. Retain the n", +"I Original proposer's solution. To this end, let's", +"【 Cartesian coordinates approach with power of a p", +"【 Cartesian coordinates approach without power of", +"I III-advised barycentric approach (outline). Use", +"【 Approach using difference of squares (from autho", +"【 Divisibility approach (Aharshi Roy). Since $p q-", +"Solution with Danielle Wang: the answer is that $|", +"【 Homothety solution (Alex Whatley). Let $G, N, O$", +"【 Power of a point solution (Zuming Feng, official", +"【 Solution by Luke Robitaille. Let $Q$ be the seco", +"ๆ Solution with coaxial circles (Pitchayut Saengru", +"【 Solution to generalization (Nikolai Beluhov). We", +"【 Approach by deleting teams (Gopal Goel). Initial", +"【 Approach by adding colors. For a constructive al", +"【 Solution using spiral similarity. We will ignore", +"『 Barycentric solution (by Carl, Krit, Milan). We", +"I A Menelaus-based approach (Kevin Ren). Let $P$ b", +"【 Barycentric solution. First, we find the coordin", +"【 Angle chasing solution (Mason Fang). Obviously $", +"【 Inversive solution (Kelin Zhu). Invert about $A$", +"l The one-liner. ", +" The external power solution. We distinguish betw", +"Cauchy-Schwarz approach. Apply Titu lemma to get", +"đ Cauchy-Schwarz approach. The main magical claim ", +"『 Alternate solution (by proposer). Let $L$ be dia" +] + + +def add_content(current_dict): + if not current_dict["lines"] or not current_dict["label"] : + return + text_str = " ".join(current_dict["lines"]).strip() + entry = {"label": current_dict["label"]} + if current_dict["class"] == "problem": + entry["problem"] = text_str + current_dict["problems"].append(entry) + elif current_dict["class"] == "solution": + entry["solution"] = text_str + entry["solution_lines"] = current_dict["lines"] + current_dict["solutions"].append(entry) + + +def parse(file): + with open(file, 'r') as file: + content = file.read() + current = { + "label": None, + "class": None, + "lines": [], + "problems": [], + "solutions": [] + } + for line in content.splitlines(): + if match := section_re.match(line): # match a section + add_content(current) + if "problems" in line.lower(): # match problem section + current["class"] = "problem" + elif sub_match:= solution_label_re.match(line): # match solutions section, extract label for join + current["class"] = "other" + current["label"] = sub_match.group(1) + elif match := solution_re.match(line): # match solutions subsection + current["class"] = "solution" + else: + current["class"] = "other" + current["lines"] = [] + elif match := problem_re.match(line): # match a problem + if current["class"] == "solution": # handle wrong problem match + current["lines"].append(line) + else: + add_content(current) + label, text = match.groups() + current["label"] = label + current["lines"] = [text] + else: + if current["class"]=="solution" or current["class"]=="problem": + current["lines"].append(line) + add_content(current) + problems_df = pd.DataFrame(current["problems"]) + solutions_df = pd.DataFrame(current["solutions"]) + return problems_df, solutions_df + + +def parse_solution(lines): + """parses lines of a solution, finds multiple solutions and splits them""" + solutions = [] + current = [] + for line in lines: + if match := solution_split_re1.search(line): # match a solution (case1) + solutions.append(" ".join(current).strip()) + current = [line] + elif match := solution_split_re2.search(line): # match a solution (case1) + solutions.append(" ".join(current).strip()) + current = [line] + elif any(case.lower() in line.lower() for case in special_cases): # match a solution (handle special_case) + solutions.append(" ".join(current).strip()) + current = [line] + elif any(case.lower() in line[:50].lower() for case in ["solution", "approach", "proof"]): + if DEBUG: + if not any(case.lower() in line[:50].lower() for case in ["remark", "proof.", "proof", "approaches", "solutions"]): + print(line[:50]) + else: + current.append(line) + solutions.append(" ".join(current).strip()) + return solutions + +def find_mult_solutions(solutions_df): + """apply parse_solution to all df""" + solutions_df["solution"] = solutions_df["solution_lines"].apply(lambda v: parse_solution(v)) + solutions_df = solutions_df.drop(columns=["solution_lines"]) + solutions_df = solutions_df.explode('solution', ignore_index=True) + return solutions_df + + +def join(problems_df, solutions_df): + pairs_df = problems_df.merge(solutions_df, on=["label"], how="outer") + return pairs_df + + +def clean(pairs_df): + '''removes the problem statement from the solution in an approximate way''' + def find_closest_char(s, i, char): + left = s.rfind(char, 0, i) # Find the last '.' before index i + right = s.find(char, i) # Find the first '.' after or at index i + if left == -1 and right == -1: + return None # No '.' found + elif left == -1: # No '.' on the left + return right + elif right == -1: # No '.' on the right + return left + else: # Closest '.' on either side + return left if abs(i - left) <= abs(i - right) else right + def remove_approx_match(row, threshold=90): + problem = row["problem"] + solution = row["solution"] + similarity = fuzz.partial_ratio(problem, solution) + if similarity >= threshold: + i = find_closest_char(solution, len(problem), problem[-1]) + if i is not None: + solution = solution[i+1:] + return solution + pairs_df["solution"] = pairs_df.apply(remove_approx_match, axis=1) + return pairs_df + + +def process_mult_solutions(pairs_df): + '''in case of multiple solutions, prepend common text to all solutions''' + def prepend_to_solution(group): + if len(group) == 1: + return group + first_row = group.iloc[0] + comment = f"{first_row['solution']}" + group = group.iloc[1:].copy() + group["solution"] = group["solution"].apply(lambda x: f"{comment} {x}") + return group + pairs_df = pairs_df.groupby("label", group_keys=False).apply(prepend_to_solution).reset_index(drop=True) + return pairs_df + + +def add_metadata(pairs_df, yar, tier): + pairs_df['year'] = year + pairs_df['tier'] = tier # according to omnimath + return pairs_df[['year', 'label', 'problem', 'solution']] + + +def write_pairs(filename, pairs_df): + pairs_df.to_json(filename, orient="records", lines=True) + + +configs = [ + ('en-usamo', lambda year: f'en-USAMO-{year}-notes', range(1996, 2025), 1), + ('en-tstst', lambda year: f'en-sols-TSTST-{year}', range(2011, 2024), 0), + ('en-tst', lambda year: f'en-sols-TST-IMO-{year}', range(2014, 2024), 0), + # ('en-elmo', lambda year: f'en-ELMO-{year}-sols', range(2010, 2017), None), #TODO: needs another parser + # ('en-elmo', lambda year: f'en-ELMO-{year}-SL', range(2017, 2019), None), #TODO: needs another parser + ('en-jmo', lambda year: f'en-JMO-{year}-notes', range(2010, 2025), 3), + # ('en-usemo', lambda year: f'en-report-usemo-{year}', range(2019, 2023), None), #TODO: needs another parser +] + + +total_problem_count = 0 +total_solution_count = 0 +for base, basename_, years, tier in configs: + print(base) + problem_count = 0 + solution_count = 0 + seg_base = f"{base}-seg" + os.makedirs(seg_base, exist_ok=True) + for year in years: + basename = basename_(year) + file_path = f"{base}/{basename}.md" + if os.path.exists(file_path): + # print(basename) + problems, solutions = parse(file_path) + solutions = find_mult_solutions(solutions) + pairs_df = join(problems, solutions) + pairs_df = clean(pairs_df) + pairs_df = process_mult_solutions(pairs_df) + pairs_df = add_metadata(pairs_df, year, tier) + problem_count += len(problems) + solution_count += len(pairs_df) + # print(pairs_df) + write_pairs(f"{seg_base}/{basename}.jsonl", pairs_df) + print(f"problem count: {problem_count}") + print(f"solution count: {solution_count}") + total_problem_count += problem_count + total_solution_count += solution_count +print(f"total problem count: {total_problem_count}") +print(f"total solution count: {total_solution_count}") + +# en-usamo +# problem count: 174 +# solution count: 203 +# en-tstst +# problem count: 105 +# solution count: 168 +# en-tst +# problem count: 51 +# solution count: 82 +# en-jmo +# problem count: 90 +# solution count: 127 +# total problem count: 420 +# total solution count: 580 \ No newline at end of file diff --git a/USAJMO/segmented/en-JMO-2010-notes.jsonl b/USAJMO/segmented/en-JMO-2010-notes.jsonl new file mode 100644 index 0000000000000000000000000000000000000000..02fdf38c95f3da79d343a6e60fd8c6067a59f075 --- /dev/null +++ b/USAJMO/segmented/en-JMO-2010-notes.jsonl @@ -0,0 +1,7 @@ +{"year":2010,"label":"1","problem":"Let $P(n)$ be the number of permutations $\\left(a_{1}, \\ldots, a_{n}\\right)$ of the numbers $(1,2, \\ldots, n)$ for which $k a_{k}$ is a perfect square for all $1 \\leq k \\leq n$. Find with proof the smallest $n$ such that $P(n)$ is a multiple of 2010 .","solution":" The answer is $n=4489$. We begin by giving a complete description of $P(n)$ : Claim - We have $$ P(n)=\\prod_{c \\text { squarefree }}\\left\\lfloor\\sqrt{\\frac{n}{c}}\\right\\rfloor! $$ $$ S_{c}=\\left\\{c \\cdot 1^{2}, c \\cdot 2^{2}, c \\cdot 3^{2}, \\ldots\\right\\} \\cap\\{1,2, \\ldots, n\\} $$ and each integer from 1 through $n$ will be in exactly one $S_{c}$. Note also that $$ \\left|S_{c}\\right|=\\left\\lfloor\\sqrt{\\frac{n}{c}}\\right\\rfloor . $$ Then, the permutations in the problem are exactly those which send elements of $S_{c}$ to elements of $S_{c}$. In other words, $$ P(n)=\\prod_{c \\text { squarefree }}\\left|S_{c}\\right|!=\\prod_{c \\text { squarefree }}\\left\\lfloor\\sqrt{\\frac{n}{c}}\\right\\rfloor! $$ We want the smallest $n$ such that 2010 divides $P(n)$. - Note that $P\\left(67^{2}\\right)$ contains 67 ! as a term, which is divisible by 2010 , so $67^{2}$ is a candidate. - On the other hand, if $n<67^{2}$, then no term in the product for $P(n)$ is divisible by the prime 67 . So $n=67^{2}=4489$ is indeed the minimum."} +{"year":2010,"label":"2","problem":"Let $n>1$ be an integer. Find, with proof, all sequences $x_{1}, x_{2}, \\ldots, x_{n-1}$ of positive integers with the following three properties: (a) $x_{1}0$, consider a triangle with vertices at $\\left(a, a^{2}\\right),\\left(-a, a^{2}\\right)$ and $\\left(b, b^{2}\\right)$. Then the area of this triangle was equal to $$ \\frac{1}{2}(2 a)\\left(b^{2}-a^{2}\\right)=a\\left(b^{2}-a^{2}\\right) . $$ To make this equal $2^{2 n} m^{2}$, simply pick $a=2^{2 n}$, and then pick $b$ such that $b^{2}-m^{2}=2^{4 n}$, for example $m=2^{4 n-2}-1$ and $b=2^{4 n-2}+1$."} +{"year":2010,"label":"5","problem":"Two permutations $a_{1}, a_{2}, \\ldots, a_{2010}$ and $b_{1}, b_{2}, \\ldots, b_{2010}$ of the numbers $1,2, \\ldots, 2010$ are said to intersect if $a_{k}=b_{k}$ for some value of $k$ in the range $1 \\leq k \\leq 2010$. Show that there exist 1006 permutations of the numbers $1,2, \\ldots, 2010$ such that any other such permutation is guaranteed to intersect at least one of these 1006 permutations.","solution":" A valid choice is the following 1006 permutations: | 1 | 2 | 3 | $\\cdots$ | 1004 | 1005 | 1006 | 1007 | 1008 | $\\cdots$ | 2009 | 2010 | | :---: | :---: | :---: | :---: | :---: | :---: | :---: | :---: | :---: | :---: | :---: | :---: | | 2 | 3 | 4 | $\\cdots$ | 1005 | 1006 | 1 | 1007 | 1008 | $\\cdots$ | 2009 | 2010 | | 3 | 4 | 5 | $\\cdots$ | 1006 | 1 | 2 | 1007 | 1008 | $\\cdots$ | 2009 | 2010 | | $\\vdots$ | $\\vdots$ | $\\vdots$ | $\\ddots$ | $\\vdots$ | $\\vdots$ | $\\vdots$ | $\\vdots$ | $\\vdots$ | $\\vdots$ | $\\vdots$ | $\\vdots$ | | 1004 | 1005 | 1006 | $\\cdots$ | 1001 | 1002 | 1003 | 1007 | 1008 | $\\cdots$ | 2009 | 2010 | | 1005 | 1006 | 1 | $\\cdots$ | 1002 | 1003 | 1004 | 1007 | 1008 | $\\cdots$ | 2009 | 2010 | | 1006 | 1 | 2 | $\\cdots$ | 1003 | 1004 | 1005 | 1007 | 1008 | $\\cdots$ | 2009 | 2010 | This works. Indeed, any permutation should have one of $\\{1,2, \\ldots, 1006\\}$ somewhere in the first 1006 positions, so one will get an intersection. Remark. In fact, the last 1004 entries do not matter with this construction, and we chose to leave them as $1007,1008, \\ldots, 2010$ only for concreteness. Remark. Using Hall's marriage lemma one may prove that the result becomes false with 1006 replaced by 1005 ."} +{"year":2010,"label":"6","problem":"Let $A B C$ be a triangle with $\\angle A=90^{\\circ}$. Points $D$ and $E$ lie on sides $A C$ and $A B$, respectively, such that $\\angle A B D=\\angle D B C$ and $\\angle A C E=\\angle E C B$. Segments $B D$ and $C E$ meet at $I$. Determine whether or not it is possible for segments $A B, A C, B I$, $I D, C I, I E$ to all have integer lengths.","solution":" The answer is no. We prove that it is not even possible that $A B, A C, C I, I B$ are all integers. ![](https:\/\/cdn.mathpix.com\/cropped\/2024_11_19_a4f5a16d632e1d6ed723g-9.jpg?height=427&width=504&top_left_y=923&top_left_x=776) First, we claim that $\\angle B I C=135^{\\circ}$. To see why, note that $$ \\angle I B C+\\angle I C B=\\frac{\\angle B}{2}+\\frac{\\angle C}{2}=\\frac{90^{\\circ}}{2}=45^{\\circ} . $$ So, $\\angle B I C=180^{\\circ}-(\\angle I B C+\\angle I C B)=135^{\\circ}$, as desired. We now proceed by contradiction. The Pythagorean theorem implies $$ B C^{2}=A B^{2}+A C^{2} $$ and so $B C^{2}$ is an integer. However, the law of cosines gives $$ \\begin{aligned} B C^{2} & =B I^{2}+C I^{2}-2 B I \\cdot C I \\cos \\angle B I C \\\\ & =B I^{2}+C I^{2}+B I \\cdot C I \\cdot \\sqrt{2} . \\end{aligned} $$ which is irrational, and this produces the desired contradiction."} diff --git a/USAJMO/segmented/en-JMO-2011-notes.jsonl b/USAJMO/segmented/en-JMO-2011-notes.jsonl new file mode 100644 index 0000000000000000000000000000000000000000..7e528d1cef368e8439b237c732bf455939565d4a --- /dev/null +++ b/USAJMO/segmented/en-JMO-2011-notes.jsonl @@ -0,0 +1,7 @@ +{"year":2011,"label":"1","problem":"Find all positive integers $n$ such that $2^{n}+12^{n}+2011^{n}$ is a perfect square.","solution":" The answer $n=1$ works, because $2^{1}+12^{1}+2011^{1}=45^{2}$. We prove it's the only one. - If $n \\geq 2$ is even, then modulo 3 we have $2^{n}+12^{n}+2011^{n} \\equiv 1+0+1 \\equiv 2(\\bmod 3)$ so it is not a square. - If $n \\geq 3$ is odd, then modulo 4 we have $2^{n}+12^{n}+2011^{n} \\equiv 0+0+3 \\equiv 3(\\bmod 4)$ so it is not a square."} +{"year":2011,"label":"2","problem":"Let $a, b, c$ be positive real numbers such that $a^{2}+b^{2}+c^{2}+(a+b+c)^{2} \\leq 4$. Prove that $$ \\frac{a b+1}{(a+b)^{2}}+\\frac{b c+1}{(b+c)^{2}}+\\frac{c a+1}{(c+a)^{2}} \\geq 3 $$","solution":" The condition becomes $2 \\geq a^{2}+b^{2}+c^{2}+a b+b c+c a$. Therefore, $$ \\begin{aligned} \\sum_{\\text {cyc }} \\frac{2 a b+2}{(a+b)^{2}} & \\geq \\sum_{\\text {cyc }} \\frac{2 a b+\\left(a^{2}+b^{2}+c^{2}+a b+b c+c a\\right)}{(a+b)^{2}} \\\\ & =\\sum_{\\text {cyc }} \\frac{(a+b)^{2}+(c+a)(c+b)}{(a+b)^{2}} \\\\ & =3+\\sum_{\\text {cyc }} \\frac{(c+a)(c+b)}{(a+b)^{2}} \\\\ & \\geq 3+3 \\sqrt[3]{\\prod_{\\text {cyc }} \\frac{(c+a)(c+b)}{(a+b)^{2}}}=3+3=6 \\end{aligned} $$ with the last line by AM-GM. This completes the proof."} +{"year":2011,"label":"3","problem":"For a point $P=\\left(a, a^{2}\\right)$ in the coordinate plane, let $\\ell(P)$ denote the line passing through $P$ with slope $2 a$. Consider the set of triangles with vertices of the form $P_{1}=\\left(a_{1}, a_{1}^{2}\\right), P_{2}=\\left(a_{2}, a_{2}^{2}\\right), P_{3}=\\left(a_{3}, a_{3}^{2}\\right)$, such that the intersection of the lines $\\ell\\left(P_{1}\\right), \\ell\\left(P_{2}\\right), \\ell\\left(P_{3}\\right)$ form an equilateral triangle $\\Delta$. Find the locus of the center of $\\Delta$ as $P_{1} P_{2} P_{3}$ ranges over all such triangles.","solution":" The answer is the line $y=-1 \/ 4$. I did not find this problem inspiring, so I will not write out most of the boring calculations since most solutions are just going to be \"use Cartesian coordinates and grind all the way through\". The \"nice\" form of the main claim is as follows (which is certainly overkill for the present task, but is too good to resist including): Claim (Naoki Sato) - In general, the orthocenter of $\\Delta$ lies on the directrix $y=-1 \/ 4$ of the parabola (even if the triangle $\\Delta$ is not equilateral). $$ \\left(\\frac{a_{1}+a_{2}}{2}, a_{1} a_{2}\\right) ; \\quad\\left(\\frac{a_{2}+a_{3}}{2}, a_{2} a_{3}\\right) ; \\quad\\left(\\frac{a_{3}+a_{1}}{2}, a_{3} a_{1}\\right) . $$ The coordinates of the orthocenter can be checked explicitly to be $$ H=\\left(\\frac{a_{1}+a_{2}+a_{3}+4 a_{1} a_{2} a_{3}}{2},-\\frac{1}{4}\\right) . $$ This claim already shows that every point lies on $y=-1 \/ 4$. We now turn to showing that, even when restricted to equilateral triangles, we can achieve every point on $y=-1 \/ 4$. In what follows $a=a_{1}, b=a_{2}, c=a_{3}$ for legibility. Claim - Lines $\\ell(a), \\ell(b), \\ell(c)$ form an equilateral triangle if and only if $$ \\begin{aligned} a+b+c & =-12 a b c \\\\ a b+b c+c a & =-\\frac{3}{4} . \\end{aligned} $$ Moreover, the $x$-coordinate of the equilateral triangle is $\\frac{1}{3}(a+b+c)$. $$ \\left(\\frac{a+b+c}{3}, \\frac{a b+b c+c a}{3}\\right)=G=H=\\left(\\frac{a+b+c+4 a b c}{2},-\\frac{1}{4}\\right) . $$ Setting the $x$ and $y$ coordinates equal, we derive the claimed equations. Let $\\lambda$ be any real number. We are tasked to show that $$ P(X)=X^{3}-3 \\lambda \\cdot X^{2}-\\frac{3}{4} X+\\frac{\\lambda}{4} $$ has three real roots (with multiplicity); then taking those roots as $(a, b, c)$ yields a valid equilateral-triangle triple whose $x$-coordinate is exactly $\\lambda$, be the previous claim. To prove that, pick the values $$ \\begin{aligned} P(-\\sqrt{3} \/ 2) & =-2 \\lambda \\\\ P(0) & =\\frac{1}{4} \\lambda \\\\ P(\\sqrt{3} \/ 2) & =-2 \\lambda . \\end{aligned} $$ The intermediate value theorem (at least for $\\lambda \\neq 0$ ) implies that $P$ should have at least two real roots now, and since $P$ has degree 3, it has all real roots. That's all."} +{"year":2011,"label":"4","problem":"A word is defined as any finite string of letters. A word is a palindrome if it reads the same backwards and forwards. Let a sequence of words $W_{0}, W_{1}, W_{2}, \\ldots$ be defined as follows: $W_{0}=a, W_{1}=b$, and for $n \\geq 2, W_{n}$ is the word formed by writing $W_{n-2}$ followed by $W_{n-1}$. Prove that for any $n \\geq 1$, the word formed by writing $W_{1}, W_{2}, W_{3}, \\ldots, W_{n}$ in succession is a palindrome.","solution":" $$ \\begin{aligned} & W_{0}=a \\\\ & W_{1}=b \\\\ & W_{2}=a b \\\\ & W_{3}=b a b \\\\ & W_{4}=a b b a b \\\\ & W_{5}=b a b a b b a b \\\\ & W_{6}=a b b a b b a b a b b a b \\\\ & W_{7}=b a b a b b a b a b b a b b a b a b b a b \\end{aligned} $$ We prove that $W_{1} W_{2} \\ldots W_{n}$ is a palindrome by induction on $n$. The base cases $n=$ $1,2,3,4$ can be verified by hand. For the inductive step, we let $\\bar{X}$ denote the word $X$ written backwards. Then $$ \\begin{aligned} W_{1} W_{2} \\ldots W_{n-3} W_{n-2} W_{n-1} W_{n} & \\stackrel{\\mathrm{IH}}{=}\\left(\\overline{W_{n-1} W_{n-2} W_{n-3}} \\ldots \\overline{W_{2} W_{1}}\\right) W_{n} \\\\ & =\\left(\\overline{W_{n-1} W_{n-2} W_{n-3}} \\ldots \\overline{W_{2} W_{1}}\\right) W_{n-2} W_{n-1} \\\\ & =\\overline{W_{n-1} W_{n-2}}\\left(\\overline{W_{n-3}} \\ldots \\overline{W_{2} W_{1}}\\right) W_{n-2} W_{n-1} \\end{aligned} $$ with the first equality being by the induction hypothesis. By induction hypothesis again the inner parenthesized term is also a palindrome, and so this completes the proof."} +{"year":2011,"label":"5","problem":"Points $A, B, C, D, E$ lie on a circle $\\omega$ and point $P$ lies outside the circle. The given points are such that (i) lines $P B$ and $P D$ are tangent to $\\omega$, (ii) $P, A, C$ are collinear, and (iii) $\\overline{D E} \\| \\overline{A C}$. Prove that $\\overline{B E}$ bisects $\\overline{A C}$.","solution":" \u3010 First solution using harmonic bundles. Let $M=\\overline{B E} \\cap \\overline{A C}$ and let $\\infty$ be the point at infinity along $\\overline{D E} \\| \\overline{A C}$. ![](https:\/\/cdn.mathpix.com\/cropped\/2024_11_19_3c39ee86a3666408bff8g-8.jpg?height=646&width=817&top_left_y=962&top_left_x=628) Note that $A B C D$ is harmonic, so $$ -1=(A C ; B D) \\stackrel{E}{=}(A C ; M \\infty) $$ implying $M$ is the midpoint of $\\overline{A C}$."} +{"year":2011,"label":"5","problem":"Points $A, B, C, D, E$ lie on a circle $\\omega$ and point $P$ lies outside the circle. The given points are such that (i) lines $P B$ and $P D$ are tangent to $\\omega$, (ii) $P, A, C$ are collinear, and (iii) $\\overline{D E} \\| \\overline{A C}$. Prove that $\\overline{B E}$ bisects $\\overline{A C}$.","solution":" I Second solution using complex numbers (Cynthia Du). Suppose we let $b, d, e$ be free on unit circle, so $p=\\frac{2 b d}{b+d}$. Then $d \/ c=a \/ e$, and $a+c=p+a c \\bar{p}$. Consequently, $$ \\begin{aligned} a c & =d e \\\\ \\frac{1}{2}(a+c) & =\\frac{b d}{b+d}+d e \\cdot \\frac{1}{b+d}=\\frac{d(b+e)}{b+d} \\\\ \\frac{a+c}{2 a c} & =\\frac{(b+e)}{e(b+d)} \\end{aligned} $$ From here it's easy to see $$ \\frac{a+c}{2}+\\frac{a+c}{2 a c} \\cdot b e=b+e $$ which is what we wanted to prove."} +{"year":2011,"label":"6","problem":"Consider the assertion that for each positive integer $n \\geq 2$, the remainder upon dividing $2^{2^{n}}$ by $2^{n}-1$ is a power of 4 . Either prove the assertion or find (with proof) a counterexample.","solution":" We claim $n=25$ is a counterexample. Since $2^{25} \\equiv 2^{0}\\left(\\bmod 2^{25}-1\\right)$, we have $$ 2^{2^{25}} \\equiv 2^{2^{25} \\bmod 25} \\equiv 2^{7} \\bmod 2^{25}-1 $$ and the right-hand side is actually the remainder, since $0<2^{7}<2^{25}$. But $2^{7}$ is not a power of 4. Remark. Really, the problem is just equivalent for asking $2^{n}$ to have odd remainder when divided by $n$."} diff --git a/USAJMO/segmented/en-JMO-2012-notes.jsonl b/USAJMO/segmented/en-JMO-2012-notes.jsonl new file mode 100644 index 0000000000000000000000000000000000000000..797c5504a2ee0e199c11952c0bb94c6868306cd8 --- /dev/null +++ b/USAJMO/segmented/en-JMO-2012-notes.jsonl @@ -0,0 +1,9 @@ +{"year":2012,"label":"1","problem":"Given a triangle $A B C$, let $P$ and $Q$ be points on segments $\\overline{A B}$ and $\\overline{A C}$, respectively, such that $A P=A Q$. Let $S$ and $R$ be distinct points on segment $\\overline{B C}$ such that $S$ lies between $B$ and $R, \\angle B P S=\\angle P R S$, and $\\angle C Q R=\\angle Q S R$. Prove that $P, Q$, $R, S$ are concyclic.","solution":" Assume for contradiction that $(P R S)$ and $(Q R S)$ are distinct. Then $\\overline{R S}$ is the radical axis of these two circles. However, $\\overline{A P}$ is tangent to $(P R S)$ and $\\overline{A Q}$ is tangent to $(Q R S)$, so point $A$ has equal power to both circles, which is impossible since $A$ does not lie on line $B C$."} +{"year":2012,"label":"2","problem":"Find all integers $n \\geq 3$ such that among any $n$ positive real numbers $a_{1}, a_{2}, \\ldots$, $a_{n}$ with $$ \\max \\left(a_{1}, a_{2}, \\ldots, a_{n}\\right) \\leq n \\cdot \\min \\left(a_{1}, a_{2}, \\ldots, a_{n}\\right), $$ there exist three that are the side lengths of an acute triangle.","solution":" The answer is all $n \\geq 13$. Define $\\left(F_{n}\\right)$ as the sequence of Fibonacci numbers, by $F_{1}=F_{2}=1$ and $F_{n+1}=$ $F_{n}+F_{n-1}$. We will find that Fibonacci numbers show up naturally when we work through the main proof, so we will isolate the following calculation now to make the subsequent solution easier to read. Claim - For positive integers $m$, we have $F_{m} \\leq m^{2}$ if and only if $m \\leq 12$. $$ \\begin{array}{rrrrrrrrrrrrrr} F_{1} & F_{2} & F_{3} & F_{4} & F_{5} & F_{6} & F_{7} & F_{8} & F_{9} & F_{10} & F_{11} & F_{12} & F_{13} & F_{14} \\\\ \\hline 1 & 1 & 2 & 3 & 5 & 8 & 13 & 21 & 34 & 55 & 89 & 144 & 233 & 377 \\end{array} $$ By examining the table, we see that $F_{m} \\leq m^{2}$ is true for $m=1,2, \\ldots 12$, and in fact $F_{12}=12^{2}=144$. However, $F_{m}>m^{2}$ for $m=13$ and $m=14$. Now it remains to prove that $F_{m}>m^{2}$ for $m \\geq 15$. The proof is by induction with base cases $m=13$ and $m=14$ being checked already. For the inductive step, if $m \\geq 15$ then we have $$ \\begin{aligned} F_{m} & =F_{m-1}+F_{m-2}>(m-1)^{2}+(m-2)^{2} \\\\ & =2 m^{2}-6 m+5=m^{2}+(m-1)(m-5)>m^{2} \\end{aligned} $$ as desired. We now proceed to the main problem. The hypothesis $\\max \\left(a_{1}, a_{2}, \\ldots, a_{n}\\right) \\leq n$. $\\min \\left(a_{1}, a_{2}, \\ldots, a_{n}\\right)$ will be denoted by $(\\dagger)$. $$ \\begin{aligned} a_{3}^{2} & \\geq a_{2}^{2}+a_{1}^{2} \\geq 2 a_{1}^{2} \\\\ a_{4}^{2} & \\geq a_{3}^{2}+a_{2}^{2} \\geq 2 a_{1}^{2}+a_{1}^{2}=3 a_{1}^{2} \\\\ a_{5}^{2} & \\geq a_{4}^{2}+a_{3}^{2} \\geq 3 a_{1}^{2}+2 a_{1}^{2}=5 a_{1}^{2} \\\\ a_{6}^{2} & \\geq a_{5}^{2}+a_{4}^{2} \\geq 5 a_{1}^{2}+3 a_{1}^{2}=8 a_{1}^{2} \\end{aligned} $$ and so on. The Fibonacci numbers appear naturally and by induction, we conclude that $a_{i}^{2} \\geq F_{i} a_{1}^{2}$. In particular, $a_{n}^{2} \\geq F_{n} a_{1}^{2}$. However, we know $\\max \\left(a_{1}, \\ldots, a_{n}\\right)=a_{n}$ and $\\min \\left(a_{1}, \\ldots, a_{n}\\right)=a_{1}$, so $(\\dagger)$ reads $a_{n} \\leq n \\cdot a_{1}$. Therefore we have $F_{n} \\leq n^{2}$, and so $n \\leq 12$, contradiction!"} +{"year":2012,"label":"3","problem":"For $a, b, c>0$ prove that $$ \\frac{a^{3}+3 b^{3}}{5 a+b}+\\frac{b^{3}+3 c^{3}}{5 b+c}+\\frac{c^{3}+3 a^{3}}{5 c+a} \\geq \\frac{2}{3}\\left(a^{2}+b^{2}+c^{2}\\right) $$","solution":" Cauchy-Schwarz approach. Apply Titu lemma to get $$ \\sum_{\\mathrm{cyc}} \\frac{a^{3}}{5 a+b}=\\sum_{\\mathrm{cyc}} \\frac{a^{4}}{5 a^{2}+a b} \\geq \\frac{\\left(a^{2}+b^{2}+c^{2}\\right)^{2}}{\\sum_{\\mathrm{cyc}}\\left(5 a^{2}+a b\\right)} \\geq \\frac{a^{2}+b^{2}+c^{2}}{6} $$ where the last step follows from the identity $\\sum_{\\text {cyc }}\\left(5 a^{2}+a b\\right) \\leq 6\\left(a^{2}+b^{2}+c^{2}\\right)$. Similarly, $$ \\sum_{\\text {cyc }} \\frac{b^{3}}{5 a+b}=\\sum_{\\mathrm{cyc}} \\frac{b^{4}}{5 a b+b^{2}} \\geq \\frac{\\left(a^{2}+b^{2}+c^{2}\\right)^{2}}{\\sum_{\\mathrm{cyc}}\\left(5 a b+b^{2}\\right)} \\geq \\frac{a^{2}+b^{2}+c^{2}}{6} $$ using the fact that $\\sum_{\\text {cyc }} 5 a b+b^{2} \\leq 6\\left(a^{2}+b^{2}+c^{2}\\right)$. Therefore, adding the first display to three times the second display implies the result."} +{"year":2012,"label":"3","problem":"For $a, b, c>0$ prove that $$ \\frac{a^{3}+3 b^{3}}{5 a+b}+\\frac{b^{3}+3 c^{3}}{5 b+c}+\\frac{c^{3}+3 a^{3}}{5 c+a} \\geq \\frac{2}{3}\\left(a^{2}+b^{2}+c^{2}\\right) $$","solution":" \u0111 Cauchy-Schwarz approach. The main magical claim is: Claim - We have $$ \\frac{a^{3}+3 b^{3}}{5 a+b} \\geq \\frac{25}{36} b^{2}-\\frac{1}{36} a^{2} $$ $$ \\frac{x^{3}+3}{5 x+1} \\geq \\frac{25-x^{2}}{36} $$ However, $$ \\begin{aligned} 36\\left(x^{3}+3\\right)-(5 x+1)\\left(25-x^{2}\\right) & =41 x^{3}+x^{2}-125 x+83 \\\\ & =(x-1)^{2}(41 x+83) \\geq 0 \\end{aligned} $$ Sum the claim cyclically to finish. Remark (Derivation of the main claim). The overall strategy is to hope for a constant $k$ such that $$ \\frac{a^{3}+3 b^{3}}{5 a+b} \\geq k a^{2}+\\left(\\frac{2}{3}-k\\right) b^{2} $$ is true. Letting $x=a \/ b$ as above and expanding, we need a value $k$ such that the cubic polynomial $P(x):=\\left(x^{3}+3\\right)-(5 x+1)\\left(k x^{2}+\\left(\\frac{2}{3}-k\\right)\\right)=(1-5 k) x^{3}-k x^{2}+\\left(5 k-\\frac{10}{3}\\right) x+\\left(k+\\frac{7}{3}\\right)$ is nonnegative everywhere. Since $P(1)=0$ necessarily, in order for $P(1-\\varepsilon)$ and $P(1+\\varepsilon)$ to both be nonnegative (for small $\\varepsilon$ ), the polynomial $P$ must have a double root at 1 , meaning the first derivative $P^{\\prime}(1)=0$ needs to vanish. In other words, we need $$ 3(1-5 k)-2 k+\\left(5 k-\\frac{10}{3}\\right)=0 $$ Solving gives $k=-1 \/ 36$. One then factors out the repeated root $(x-1)^{2}$ from the resulting $P$."} +{"year":2012,"label":"4","problem":"Let $\\alpha$ be an irrational number with $0<\\alpha<1$, and draw a circle in the plane whose circumference has length 1 . Given any integer $n \\geq 3$, define a sequence of points $P_{1}, P_{2}, \\ldots, P_{n}$ as follows. First select any point $P_{1}$ on the circle, and for $2 \\leq k \\leq n$ define $P_{k}$ as the point on the circle for which the length of $\\operatorname{arc} P_{k-1} P_{k}$ is $\\alpha$, when travelling counterclockwise around the circle from $P_{k-1}$ to $P_{k}$. Suppose that $P_{a}$ and $P_{b}$ are the nearest adjacent points on either side of $P_{n}$. Prove that $a+b \\leq n$.","solution":" No points coincide since $\\alpha$ is irrational. Assume for contradiction that $nr_{b}$ or $2012-r_{a}>2012-r_{b}$. This implies $S \\geq \\frac{1}{2} \\varphi(2012)=502$. But this can actually be achieved by taking $a=4$ and $b=1010$, since - If $k$ is even, then $a k \\equiv b k(\\bmod 2012)$ so no even $k$ counts towards $S$; and - If $k \\equiv 0(\\bmod 503)$, then $a k \\equiv 0(\\bmod 2012)$ so no such $k$ counts towards $S$. This gives the final answer $S \\geq 502$."} +{"year":2012,"label":"6","problem":"Let $P$ be a point in the plane of $\\triangle A B C$, and $\\gamma$ a line through $P$. Let $A^{\\prime}, B^{\\prime}, C^{\\prime}$ be the points where the reflections of lines $P A, P B, P C$ with respect to $\\gamma$ intersect lines $B C, C A, A B$ respectively. Prove that $A^{\\prime}, B^{\\prime}, C^{\\prime}$ are collinear.","solution":" \u3010 First solution (complex numbers). Let $p=0$ and set $\\gamma$ as the real line. Then $A^{\\prime}$ is the intersection of $b c$ and $p \\bar{a}$. So, we get $$ a^{\\prime}=\\frac{\\bar{a}(\\bar{b} c-b \\bar{c})}{(\\bar{b}-\\bar{c}) \\bar{a}-(b-c) a} . $$ ![](https:\/\/cdn.mathpix.com\/cropped\/2024_11_19_4c570ff524ef777a3bedg-10.jpg?height=364&width=427&top_left_y=1103&top_left_x=820) Note that $$ \\bar{a}^{\\prime}=\\frac{a(b \\bar{c}-\\bar{b} c)}{(b-c) a-(\\bar{b}-\\bar{c}) \\bar{a}} . $$ Thus it suffices to prove This is equivalent to $$ 0=\\operatorname{det}\\left[\\begin{array}{lll} \\bar{a}(\\bar{b} c-b \\bar{c}) & a(\\bar{b} c-b \\bar{c}) & (\\bar{b}-\\bar{c}) \\bar{a}-(b-c) a \\\\ \\bar{b}(\\bar{c} a-c \\bar{a}) & b(\\bar{c} a-c \\bar{a}) & (\\bar{c}-\\bar{a}) \\bar{b}-(c-a) b \\\\ \\bar{c}(\\bar{a} b-a \\bar{b}) & c(\\bar{a} b-a \\bar{b}) & (\\bar{a}-\\bar{b}) \\bar{c}-(a-b) c \\end{array}\\right] . $$ This determinant has the property that the rows sum to zero, and we're done. Remark. Alternatively, if you don't notice that you could just blindly expand: $$ \\begin{aligned} & \\sum_{\\mathrm{cyc}}((\\bar{b}-\\bar{c}) \\bar{a}-(b-c) a) \\cdot-\\operatorname{det}\\left[\\begin{array}{ll} b & \\bar{b} \\\\ c & \\bar{c} \\end{array}\\right](\\bar{c} a-c \\bar{a})(\\bar{a} b-a \\bar{b}) \\\\ = & (\\bar{b} c-c \\bar{b})(\\bar{c} a-c \\bar{a})(\\bar{a} b-a \\bar{b}) \\sum_{\\mathrm{cyc}}(a b-a c+\\overline{c a}-\\bar{b} \\bar{a})=0 \\end{aligned} $$"} +{"year":2012,"label":"6","problem":"Let $P$ be a point in the plane of $\\triangle A B C$, and $\\gamma$ a line through $P$. Let $A^{\\prime}, B^{\\prime}, C^{\\prime}$ be the points where the reflections of lines $P A, P B, P C$ with respect to $\\gamma$ intersect lines $B C, C A, A B$ respectively. Prove that $A^{\\prime}, B^{\\prime}, C^{\\prime}$ are collinear.","solution":" \\I Second solution (Desargues involution). We let $C^{\\prime \\prime}=\\overline{A^{\\prime} B^{\\prime}} \\cap \\overline{A B}$. Consider complete quadrilateral $A B C A^{\\prime} B^{\\prime} C^{\\prime \\prime} C$. We see that there is an involutive pairing $\\tau$ at $P$ swapping $\\left(P A, P A^{\\prime}\\right),\\left(P B, P B^{\\prime}\\right),\\left(P C, P C^{\\prime \\prime}\\right)$. From the first two, we see $\\tau$ coincides with reflection about $\\ell$, hence conclude $C^{\\prime \\prime}=C$."} +{"year":2012,"label":"6","problem":"Let $P$ be a point in the plane of $\\triangle A B C$, and $\\gamma$ a line through $P$. Let $A^{\\prime}, B^{\\prime}, C^{\\prime}$ be the points where the reflections of lines $P A, P B, P C$ with respect to $\\gamma$ intersect lines $B C, C A, A B$ respectively. Prove that $A^{\\prime}, B^{\\prime}, C^{\\prime}$ are collinear.","solution":" \u092c Third solution (barycentric), by Catherine $\\mathbf{X u}$. We will perform barycentric coordinates on the triangle $P C C^{\\prime}$, with $P=(1,0,0), C^{\\prime}=(0,1,0)$, and $C=(0,0,1)$. Set $a=C C^{\\prime}, b=C P, c=C^{\\prime} P$ as usual. Since $A, B, C^{\\prime}$ are collinear, we will define $A=(p: k: q)$ and $B=(p: \\ell: q)$. Claim - Line $\\gamma$ is the angle bisector of $\\angle A P A^{\\prime}, \\angle B P B^{\\prime}$, and $\\angle C P C^{\\prime}$. Thus $B^{\\prime}$ is the intersection of the isogonal of $B$ with respect to $\\angle P$ with the line $C A$; that is, $$ B^{\\prime}=\\left(\\frac{p}{k} \\frac{b^{2}}{\\ell}: \\frac{b^{2}}{\\ell}: \\frac{c^{2}}{q}\\right) $$ Analogously, $A^{\\prime}$ is the intersection of the isogonal of $A$ with respect to $\\angle P$ with the line $C B$; that is, $$ A^{\\prime}=\\left(\\frac{p}{\\ell} \\frac{b^{2}}{k}: \\frac{b^{2}}{k}: \\frac{c^{2}}{q}\\right) $$ The ratio of the first to third coordinate in these two points is both $b^{2} p q: c^{2} k \\ell$, so it follows $A^{\\prime}, B^{\\prime}$, and $C^{\\prime}$ are collinear. Remark (Problem reference). The converse of this problem appears as problem 1052 attributed S. V. Markelov in the book Geometriya: 9-11 Klassy: Ot Uchebnoy Zadachi k Tvorcheskoy, 1996, by I. F. Sharygin."} diff --git a/USAJMO/segmented/en-JMO-2013-notes.jsonl b/USAJMO/segmented/en-JMO-2013-notes.jsonl new file mode 100644 index 0000000000000000000000000000000000000000..c794e884e1f752d85517441440d400a5746a90d1 --- /dev/null +++ b/USAJMO/segmented/en-JMO-2013-notes.jsonl @@ -0,0 +1,6 @@ +{"year":2013,"label":"1","problem":"Are there integers $a$ and $b$ such that $a^{5} b+3$ and $a b^{5}+3$ are both perfect cubes of integers?","solution":" No, there do not exist such $a$ and $b$. We prove this in two cases. - Assume $3 \\mid a b$. WLOG we have $3 \\mid a$, but then $a^{5} b+3 \\equiv 3(\\bmod 9)$, contradiction. - Assume $3 \\nmid a b$. Then $a^{5} b+3$ is a cube not divisible by 3 , so it is $\\pm 1 \\bmod 9$, and we conclude $$ a^{5} b \\in\\{5,7\\} \\quad(\\bmod 9) $$ Analogously $$ a b^{5} \\in\\{5,7\\} \\quad(\\bmod 9) $$ We claim however these two equations cannot hold simultaneously. Indeed $(a b)^{6} \\equiv 1$ $(\\bmod 9)$ by Euler's theorem, despite $5 \\cdot 5 \\equiv 7,5 \\cdot 7 \\equiv 8,7 \\cdot 7 \\equiv 4 \\bmod 9$."} +{"year":2013,"label":"2","problem":"Each cell of an $m \\times n$ board is filled with some nonnegative integer. Two numbers in the filling are said to be adjacent if their cells share a common side. The filling is called a garden if it satisfies the following two conditions: (i) The difference between any two adjacent numbers is either 0 or 1 . (ii) If a number is less than or equal to all of its adjacent numbers, then it is equal to 0. Determine the number of distinct gardens in terms of $m$ and $n$.","solution":" The numerical answer is $2^{m n}-1$. But we claim much more, by giving an explicit description of all gardens: Let $S$ be any nonempty subset of the $m n$ cells. Suppose we fill each cell $\\theta$ with the minimum (taxicab) distance from $\\theta$ to some cell in $S$ (in particular, we write 0 if $\\theta \\in S$ ). Then - This gives a garden, and - All gardens are of this form. Since there are $2^{m n}-1$ such nonempty subsets $S$, this would finish the problem. An example of a garden with $|S|=3$ is shown below. $$ \\left[\\begin{array}{llllll} 2 & 1 & 2 & 1 & 0 & 1 \\\\ 1 & 0 & 1 & 2 & 1 & 2 \\\\ 1 & 1 & 2 & 3 & 2 & 3 \\\\ 0 & 1 & 2 & 3 & 3 & 4 \\end{array}\\right] $$ It is actually fairly easy to see that this procedure always gives a garden; so we focus our attention on showing that every garden is of this form. Given a garden, note first that it has at least one cell with a zero in it - by considering the minimum number across the entire garden. Now let $S$ be the (thus nonempty) set of cells with a zero written in them. We contend that this works, i.e. the following sentence holds: Claim - If a cell $\\theta$ is labeled $d$, then the minimum distance from that cell to a cell in $S$ is $d$."} +{"year":2013,"label":"3","problem":"In triangle $A B C$, points $P, Q, R$ lie on sides $B C, C A, A B$, respectively. Let $\\omega_{A}$, $\\omega_{B}, \\omega_{C}$ denote the circumcircles of triangles $A Q R, B R P, C P Q$, respectively. Given the fact that segment $A P$ intersects $\\omega_{A}, \\omega_{B}, \\omega_{C}$ again at $X, Y, Z$ respectively, prove that $Y X \/ X Z=B P \/ P C$.","solution":" Let $M$ be the concurrence point of $\\omega_{A}, \\omega_{B}, \\omega_{C}$ (by Miquel's theorem). ![](https:\/\/cdn.mathpix.com\/cropped\/2024_11_19_0a837349e2b78b0909e9g-5.jpg?height=704&width=812&top_left_y=873&top_left_x=622) Then $M$ is the center of a spiral similarity sending $\\overline{Y Z}$ to $\\overline{B C}$. So it suffices to show that this spiral similarity also sends $X$ to $P$, but $$ \\measuredangle M X Y=\\measuredangle M X A=\\measuredangle M R A=\\measuredangle M R B=\\measuredangle M P B $$ so this follows."} +{"year":2013,"label":"4","problem":"Let $f(n)$ be the number of ways to write $n$ as a sum of powers of 2 , where we keep track of the order of the summation. For example, $f(4)=6$ because 4 can be written as $4,2+2,2+1+1,1+2+1,1+1+2$, and $1+1+1+1$. Find the smallest $n$ greater than 2013 for which $f(n)$ is odd.","solution":" The answer is 2047. For convenience, we agree that $f(0)=1$. Then by considering cases on the first number in the representation, we derive the recurrence $$ f(n)=\\sum_{k=0}^{\\left\\lfloor\\log _{2} n\\right\\rfloor} f\\left(n-2^{k}\\right) $$ We wish to understand the parity of $f$. The first few values are $$ \\begin{aligned} & f(0)=1 \\\\ & f(1)=1 \\\\ & f(2)=2 \\\\ & f(3)=3 \\\\ & f(4)=6 \\\\ & f(5)=10 \\\\ & f(6)=18 \\\\ & f(7)=31 . \\end{aligned} $$ Inspired by the data we make the key claim that Claim - $f(n)$ is odd if and only if $n+1$ is a power of 2. This only takes a few cases: - If $n=2^{k}$, then $(\\odot)$ has exactly two repetitive terms on the right-hand side, namely 0 and $2^{k}-1$. - If $n=2^{k}+2^{\\ell}-1$, then $(\\odot)$ has exactly two repetitive terms on the right-hand side, namely $2^{\\ell+1}-1$ and $2^{\\ell}-1$. - If $n=2^{k}-1$, then $(\\bigcirc)$ has exactly one repetitive terms on the right-hand side, namely $2^{k-1}-1$. - For other $n$, there are no repetitive terms at all on the right-hand side of $(\\Omega)$. Thus the induction checks out. So the final answer to the problem is 2047."} +{"year":2013,"label":"5","problem":"Quadrilateral $X A B Y$ is inscribed in the semicircle $\\omega$ with diameter $\\overline{X Y}$. Segments $A Y$ and $B X$ meet at $P$. Point $Z$ is the foot of the perpendicular from $P$ to line $\\overline{X Y}$. Point $C$ lies on $\\omega$ such that line $X C$ is perpendicular to line $A Z$. Let $Q$ be the intersection of segments $A Y$ and $X C$. Prove that $$ \\frac{B Y}{X P}+\\frac{C Y}{X Q}=\\frac{A Y}{A X} $$","solution":" Let $\\beta=\\angle Y X P$ and $\\alpha=\\angle P Y X$ and set $X Y=1$. We do not direct angles in the following solution. ![](https:\/\/cdn.mathpix.com\/cropped\/2024_11_19_0a837349e2b78b0909e9g-7.jpg?height=552&width=1012&top_left_y=1046&top_left_x=525) Observe that $$ \\angle A Z X=\\angle A P X=\\alpha+\\beta $$ since $A P Z X$ is cyclic. In particular, $\\angle C X Y=90^{\\circ}-(\\alpha+\\beta)$. It is immediate that $$ B Y=\\sin \\beta, \\quad C Y=\\cos (\\alpha+\\beta), \\quad A Y=\\cos \\alpha, \\quad A X=\\sin \\alpha $$ The Law of Sines on $\\triangle X P Y$ gives $X P=X Y \\frac{\\sin \\alpha}{\\sin (\\alpha+\\beta)}$, and on $\\triangle X Q Y$ gives $X Q=$ $X Y \\frac{\\sin \\alpha}{\\sin (90+\\beta)}=\\frac{\\sin \\alpha}{\\cos \\beta}$. So, the given is equivalent to $$ \\frac{\\sin \\beta}{\\frac{\\sin \\alpha}{\\sin (\\alpha+\\beta)}}+\\frac{\\cos (\\alpha+\\beta)}{\\frac{\\sin \\alpha}{\\cos \\beta}}=\\frac{\\cos \\alpha}{\\sin \\alpha} $$ which is equivalent to $\\cos \\alpha=\\cos \\beta \\cos (\\alpha+\\beta)+\\sin \\beta \\sin (\\alpha+\\beta)$. This is obvious, because the right-hand side is just $\\cos ((\\alpha+\\beta)-\\beta)$."} +{"year":2013,"label":"6","problem":"Find all real numbers $x, y, z \\geq 1$ satisfying $$ \\min (\\sqrt{x+x y z}, \\sqrt{y+x y z}, \\sqrt{z+x y z})=\\sqrt{x-1}+\\sqrt{y-1}+\\sqrt{z-1} . $$","solution":" Set $x=1+a, y=1+b, z=1+c$ which eliminates the $x, y, z \\geq 1$ condition. Assume without loss of generality that $a \\leq b \\leq c$. Then the given equation rewrites as $$ \\sqrt{(1+a)(1+(1+b)(1+c))}=\\sqrt{a}+\\sqrt{b}+\\sqrt{c} $$ In fact, we are going to prove the left-hand side always exceeds the right-hand side, and then determine the equality cases. We have: $$ \\begin{aligned} (1+a)(1+(1+b)(1+c)) & =(a+1)(1+(b+1)(1+c)) \\\\ & \\leq(a+1)\\left(1+(\\sqrt{b}+\\sqrt{c})^{2}\\right) \\\\ & \\leq(\\sqrt{a}+(\\sqrt{b}+\\sqrt{c}))^{2} \\end{aligned} $$ by two applications of Cauchy-Schwarz. Equality holds if $b c=1$ and $1 \/ a=\\sqrt{b}+\\sqrt{c}$. Letting $c=t^{2}$ for $t \\geq 1$, we recover $b=t^{-2} \\leq t^{2}$ and $a=\\frac{1}{t+1 \/ t} \\leq t^{2}$. $$ (x, y, z)=\\left(1+\\left(\\frac{t}{t^{2}+1}\\right)^{2}, 1+\\frac{1}{t^{2}}, 1+t^{2}\\right) $$ and permutations, for any $t>0$."} diff --git a/USAJMO/segmented/en-JMO-2014-notes.jsonl b/USAJMO/segmented/en-JMO-2014-notes.jsonl new file mode 100644 index 0000000000000000000000000000000000000000..fcc566debea5a3458a127000ebd4470f6f64c379 --- /dev/null +++ b/USAJMO/segmented/en-JMO-2014-notes.jsonl @@ -0,0 +1,6 @@ +{"year":2014,"label":"1","problem":"Let $a, b, c$ be real numbers greater than or equal to 1 . Prove that $$ \\min \\left(\\frac{10 a^{2}-5 a+1}{b^{2}-5 b+10}, \\frac{10 b^{2}-5 b+1}{c^{2}-5 c+10}, \\frac{10 c^{2}-5 c+1}{a^{2}-5 a+10}\\right) \\leq a b c $$","solution":" Notice that $$ \\frac{10 a^{2}-5 a+1}{a^{2}-5 a+10} \\leq a^{3} $$ since it rearranges to $(a-1)^{5} \\geq 0$. Cyclically multiply to get $$ \\prod_{\\mathrm{cyc}}\\left(\\frac{10 a^{2}-5 a+1}{b^{2}-5 b+10}\\right) \\leq(a b c)^{3} $$ and the minimum is at most the geometric mean."} +{"year":2014,"label":"2","problem":"Let $\\triangle A B C$ be a non-equilateral, acute triangle with $\\angle A=60^{\\circ}$, and let $O$ and $H$ denote the circumcenter and orthocenter of $\\triangle A B C$, respectively. (a) Prove that line $O H$ intersects both segments $A B$ and $A C$ at two points $P$ and $Q$, respectively. (b) Denote by $s$ and $t$ the respective areas of triangle $A P Q$ and quadrilateral $B P Q C$. Determine the range of possible values for $s \/ t$.","solution":" We begin with some synthetic work. Let $I$ denote the incenter, and recall (\"fact 5\") that the arc midpoint $M$ is the center of $(B I C)$, which we denote by $\\gamma$. Now we have that $$ \\angle B O C=\\angle B I C=\\angle B H C=120^{\\circ} . $$ Since all three centers lie inside $A B C$ (as it was acute), and hence on the opposite side of $\\overline{B C}$ as $M$, it follows that $O, I, H$ lie on minor arc $B C$ of $\\gamma$. We note this implies (a) already, as line $O H$ meets line $B C$ outside of segment $B C$. ![](https:\/\/cdn.mathpix.com\/cropped\/2024_11_19_3739490e48f1a6399d35g-04.jpg?height=806&width=552&top_left_y=1339&top_left_x=752) Claim - Triangle $A P Q$ is equilateral with side length $\\frac{b+c}{3}$. Finally, since $\\angle P B H=30^{\\circ}$, and $\\angle B P H=120^{\\circ}$, it follows that $\\triangle B P H$ is isosceles and $B P=P H$. Similarly, $C Q=Q H$. So $b+c=A P+B P+A Q+Q C=A P+A Q+P Q$ as needed. Finally, we turn to the boring task of extracting the numerical answer. We have $$ \\frac{s}{s+t}=\\frac{[A P Q]}{[A B C]}=\\frac{\\frac{\\sqrt{3}}{4}\\left(\\frac{b+c}{3}\\right)^{2}}{\\frac{\\sqrt{3}}{4} b c}=\\frac{b^{2}+2 b c+c^{2}}{9 b c}=\\frac{1}{9}\\left(2+\\frac{b}{c}+\\frac{c}{b}\\right) . $$ So the problem is reduced to analyzing the behavior of $b \/ c$. For this, we imagine fixing $\\Gamma$ the circumcircle of $A B C$, as well as the points $B$ and $C$. Then as we vary $A$ along the \"topmost\" arc of measure $120^{\\circ}$, we find $b \/ c$ is monotonic with values $1 \/ 2$ and 2 at endpoints, and by continuity all values $b \/ c \\in(1 \/ 2,2)$ can be achieved. So $$ \\frac{1}{2}<\\frac{b}{c}<2 \\Longrightarrow 4 \/ 9<\\frac{s}{s+t}<1 \/ 2 \\Longrightarrow 4 \/ 5<\\frac{s}{t}<1 $$ as needed."} +{"year":2014,"label":"3","problem":"Find all $f: \\mathbb{Z} \\rightarrow \\mathbb{Z}$ such that $$ x f(2 f(y)-x)+y^{2} f(2 x-f(y))=\\frac{f(x)^{2}}{x}+f(y f(y)) $$ for all $x, y \\in \\mathbb{Z}$ such that $x \\neq 0$.","solution":" The answer is $f(x) \\equiv 0$ and $f(x) \\equiv x^{2}$. Check that these work. $$ x f(2 f(0)-x)=\\frac{f(x)^{2}}{x}+f(0) . $$ The nicest part of the problem is the following step: $$ \\text { Claim - We have } f(0)=0 $$ $$ f(0)=0 $$ Claim - We have $f(x) \\in\\left\\{0, x^{2}\\right\\}$ for each individual $x$. $$ x^{2} f(-x)=f(x)^{2} $$ holds for all nonzero $x$, but also for $x=0$. Let us now check that $f$ is an even function. In the above, we may also derive $f(-x)^{2}=x^{2} f(x)$. If $f(x) \\neq f(-x)$ (and hence $x \\neq 0$ ), then subtracting the above and factoring implies that $f(x)+f(-x)=-x^{2}$; we can then obtain by substituting the relation $$ \\left[f(x)+\\frac{1}{2} x^{2}\\right]^{2}=-\\frac{3}{4} x^{4}<0 $$ which is impossible. This means $f(x)^{2}=x^{2} f(x)$, thus $$ f(x) \\in\\left\\{0, x^{2}\\right\\} \\quad \\forall x . $$ Now suppose there exists a nonzero integer $t$ with $f(t)=0$. We will prove that $f(x) \\equiv 0$. Put $y=t$ in the given to obtain that $$ t^{2} f(2 x)=0 $$ for any integer $x \\neq 0$, and hence conclude that $f(2 \\mathbb{Z}) \\equiv 0$. Then selecting $x=2 k \\neq 0$ in the given implies that $$ y^{2} f(4 k-f(y))=f(y f(y)) $$ Assume for contradiction that $f(m)=m^{2}$ now for some odd $m \\neq 0$. Evidently $$ m^{2} f\\left(4 k-m^{2}\\right)=f\\left(m^{3}\\right) $$ If $f\\left(m^{3}\\right) \\neq 0$ this forces $f\\left(4 k-m^{2}\\right) \\neq 0$, and hence $m^{2}\\left(4 k-m^{2}\\right)^{2}=m^{6}$ for arbitrary $k \\neq 0$, which is clearly absurd. That means $$ f\\left(4 k-m^{2}\\right)=f\\left(m^{2}-4 k\\right)=f\\left(m^{3}\\right)=0 $$ for each $k \\neq 0$. Since $m$ is odd, $m^{2} \\equiv 1(\\bmod 4)$, and so $f(n)=0$ for all $n$ other than $\\pm m^{2}$ (since we cannot select $\\left.k=0\\right)$. Now $f(m)=m^{2}$ means that $m= \\pm 1$. Hence either $f(x) \\equiv 0$ or $$ f(x)= \\begin{cases}1 & x= \\pm 1 \\\\ 0 & \\text { otherwise }\\end{cases} $$ To show that the latter fails, we simply take $x=5$ and $y=1$ in the given."} +{"year":2014,"label":"4","problem":"Let $b \\geq 2$ be a fixed integer, and let $s_{b}(n)$ denote the sum of the base- $b$ digits of $n$. Show that there are infinitely many positive integers that cannot be represented in the from $n+s_{b}(n)$ where $n$ is a positive integer.","solution":" For brevity let $f(n)=n+s_{b}(n)$. Select any integer $M$. Observe that $f(x) \\geq b^{2 M}$ for any $x \\geq b^{2 M}$, but also $f\\left(b^{2 M}-k\\right) \\geq b^{2 M}$ for $k=1,2, \\ldots, M$, since the base- $b$ expansion of $b^{2 M}-k$ will start out with at least $M$ digits $b-1$. Thus $f$ omits at least $M$ values in $\\left[1, b^{2 M}\\right]$ for any $M$."} +{"year":2014,"label":"5","problem":"Let $k$ be a positive integer. Two players $A$ and $B$ play a game on an infinite grid of regular hexagons. Initially all the grid cells are empty. Then the players alternately take turns with $A$ moving first. In her move, $A$ may choose two adjacent hexagons in the grid which are empty and place a counter in both of them. In his move, $B$ may choose any counter on the board and remove it. If at any time there are $k$ consecutive grid cells in a line all of which contain a counter, $A$ wins. Find the minimum value of $k$ for which $A$ cannot win in a finite number of moves, or prove that no such minimum value exists.","solution":" The answer is $k=6$. ![](https:\/\/cdn.mathpix.com\/cropped\/2024_11_19_3739490e48f1a6399d35g-09.jpg?height=500&width=561&top_left_y=1372&top_left_x=753) \u3010 Example of a strategy for $A$ when $k=5$. We describe a winning strategy for $A$ explicitly. Note that after $B$ 's first turn there is one counter, so then $A$ may create an equilateral triangle, and hence after $B$ 's second turn there are two consecutive counters. Then, on her third turn, $A$ places a pair of counters two spaces away on the same line. Label the two inner cells $x$ and $y$ as shown below. ![](https:\/\/cdn.mathpix.com\/cropped\/2024_11_19_3739490e48f1a6399d35g-09.jpg?height=295&width=500&top_left_y=2168&top_left_x=778) Now it is $B$ 's turn to move; in order to avoid losing immediately, he must remove either $x$ or $y$. Then on any subsequent turn, $A$ can replace $x$ or $y$ (whichever was removed) and add one more adjacent counter. This continues until either $x$ or $y$ has all its neighbors filled (we ask $A$ to do so in such a way that she avoids filling in the two central cells between $x$ and $y$ as long as possible). So, let's say without loss of generality (by symmetry) that $x$ is completely surrounded by tokens. Again, $B$ must choose to remove $x$ (or $A$ wins on her next turn). After $x$ is removed by $B$, consider the following figure. ![](https:\/\/cdn.mathpix.com\/cropped\/2024_11_19_3739490e48f1a6399d35g-10.jpg?height=398&width=612&top_left_y=503&top_left_x=728) We let $A$ play in the two marked green cells. Then, regardless of what move $B$ plays, one of the two choices of moves marked in red lets $A$ win. Thus, we have described a winning strategy when $k=5$ for $A$."} +{"year":2014,"label":"6","problem":"Let $A B C$ be a triangle with incenter $I$, incircle $\\gamma$ and circumcircle $\\Gamma$. Let $M, N, P$ be the midpoints of $\\overline{B C}, \\overline{C A}, \\overline{A B}$ and let $E, F$ be the tangency points of $\\gamma$ with $\\overline{C A}$ and $\\overline{A B}$, respectively. Let $U, V$ be the intersections of line $E F$ with line $M N$ and line $M P$, respectively, and let $X$ be the midpoint of $\\operatorname{arc} B A C$ of $\\Gamma$. (a) Prove that $I$ lies on ray $C V$. (b) Prove that line $X I$ bisects $\\overline{U V}$.","solution":" The fact that $I=\\overline{B U} \\cap \\overline{C V}$ and is Lemma 1.45 from EGMO. As for (b), we note: Claim - Line $I X$ is a symmedian of $\\triangle I B C$. Since $B V U C$ is cyclic with diagonals intersecting at $I$, and $I X$ is symmedian of $\\triangle I B C$, it is median of $\\triangle I U V$, as needed."} diff --git a/USAJMO/segmented/en-JMO-2015-notes.jsonl b/USAJMO/segmented/en-JMO-2015-notes.jsonl new file mode 100644 index 0000000000000000000000000000000000000000..7f52153e75e14aec88583bc2b9b1f3a118135516 --- /dev/null +++ b/USAJMO/segmented/en-JMO-2015-notes.jsonl @@ -0,0 +1,8 @@ +{"year":2015,"label":"1","problem":"Given a sequence of real numbers, a move consists of choosing two terms and replacing each with their arithmetic mean. Show that there exists a sequence of 2015 distinct real numbers such that after one initial move is applied to the sequence - no matter what move - there is always a way to continue with a finite sequence of moves so as to obtain in the end a constant sequence.","solution":" One valid example of a sequence is $0,1, \\ldots, 2014$. We will show how to achieve the all-1007 sequence based on the first move. Say two numbers are opposites if their average is 1007 . We consider 1007 as its own opposite. We consider two cases: - First, suppose the first initial move did not involve the number 1007. Suppose the two numbers changed were $a$ and $b$, replaced by $c=\\frac{1}{2}(a+b)$ twice. - If $a$ and $b$ are opposites, we simply operate on all the other pairs of opposites. - Otherwise let $a^{\\prime}$ and $b^{\\prime}$ be the opposites of $a$ and $b$, so all four of $a, b, a^{\\prime}, b^{\\prime}$ are distinct. Then operate on $a^{\\prime}$ and $b^{\\prime}$ to get $c^{\\prime}=2014-c$. We work with only these four numbers ande replace them as follows: $$ \\begin{array}{cccc} \\frac{1}{2}(a+b) & \\frac{1}{2}(a+b) & a^{\\prime} & b^{\\prime} \\\\ \\frac{1}{2}(a+b) & \\frac{1}{2}(a+b) & \\frac{1}{2}\\left(a^{\\prime}+b^{\\prime}\\right) & \\frac{1}{2}\\left(a^{\\prime}+b^{\\prime}\\right) \\\\ 1007 & \\frac{1}{2}(a+b) & 1007 & \\frac{1}{2}\\left(a^{\\prime}+b^{\\prime}\\right) \\\\ 1007 & 1007 & 1007 & 1007 \\end{array} $$ Finally, we operate on the remaining 1005 pairs of opposites. - Now suppose the first initial move involved the number 1007 and some $a$. Let $k$ be any number other than $a$ or its opposite, and let $a^{\\prime}, k^{\\prime}$ be the opposites of $a$ and $k$. We work with only these five numbers: and replace them in the following way: $$ \\begin{array}{ccccc} \\frac{1}{2}(a+1007) & \\frac{1}{2}(a+1007) & a^{\\prime} & k & k^{\\prime} \\\\ \\frac{1}{2}(a+1007) & \\frac{1}{2}(a+1007) & a^{\\prime} & 1007 & 1007 \\\\ \\frac{1}{2}(a+1007) & \\frac{1}{2}(a+1007) & \\frac{1}{2}\\left(a^{\\prime}+1007\\right) & \\frac{1}{2}\\left(a^{\\prime}+1007\\right) & 1007 \\\\ 1007 & \\frac{1}{2}(a+1007) & 1007 & \\frac{1}{2}\\left(a^{\\prime}+1007\\right) & 1007 \\\\ 1007 & 1007 & 1007 & 1007 & 1007 \\end{array} $$ Finally, we operate on the remaining 1005 pairs of opposites. However for \"most\" sequences one expects the result to not be possible. As a simple example, the goal is impossible for $(0,1, \\ldots, 2013,2015)$ since the average of the terms is $1007+\\frac{1}{2015}$, but in the process the only denominators ever generated are powers of 2 . This narrows the search somewhat."} +{"year":2015,"label":"2","problem":"Solve in integers the equation $$ x^{2}+x y+y^{2}=\\left(\\frac{x+y}{3}+1\\right)^{3} . $$","solution":" We do the trick of setting $a=x+y$ and $b=x-y$. This rewrites the equation as $$ \\frac{1}{4}\\left((a+b)^{2}+(a+b)(a-b)+(a-b)^{2}\\right)=\\left(\\frac{a}{3}+1\\right)^{3} $$ where $a, b \\in \\mathbb{Z}$ have the same parity. This becomes $$ 3 a^{2}+b^{2}=4\\left(\\frac{a}{3}+1\\right)^{3} $$ which is enough to imply $3 \\mid a$, so let $a=3 c$. Miraculously, this becomes $$ b^{2}=(c-2)^{2}(4 c+1) $$ $$ x=\\frac{1}{8}\\left(3\\left(m^{2}-1\\right) \\pm\\left(m^{3}-9 m\\right)\\right) \\quad \\text { and } \\quad y=\\frac{1}{8}\\left(3\\left(m^{2}-1\\right) \\mp\\left(m^{3}-9 m\\right)\\right) . $$ For mod 8 reasons, this always generates a valid integer solution, so this is the complete curve of solutions. Actually, putting $m=2 n+1$ gives the much nicer curve $$ x=n^{3}+3 n^{2}-1 \\quad \\text { and } \\quad y=-n^{3}+3 n+1 $$ and permutations."} +{"year":2015,"label":"3","problem":"Quadrilateral $A P B Q$ is inscribed in circle $\\omega$ with $\\angle P=\\angle Q=90^{\\circ}$ and $A P=$ $A Q0$. Claim - The circumcenter has coordinates $\\left(\\frac{c-b}{2}, \\frac{a}{2}-\\frac{b c}{2 a}\\right)$. - By power of a point, the second intersection of line $A H$ with the circumcircle is $\\left(0,-\\frac{b c}{a}\\right)$. - Since the orthocenter is the reflection of this point across line $B C$, the orthocenter is given exactly by $\\left(0, \\frac{b c}{a}\\right)$. - The centroid is is $\\frac{\\vec{A}+\\vec{B}+\\vec{C}}{3}=\\left(\\frac{c-b}{3}, \\frac{a}{3}\\right)$. - Since $\\vec{H}-\\vec{O}=3(\\vec{G}-\\vec{O})$ according to the Euler line, we have $\\vec{O}=\\frac{3}{2} \\vec{G}-\\frac{1}{2} \\vec{H}$. This gives the desired formula. Note that $H Q=\\frac{H A \\cdot H C}{A C}=\\frac{a c}{\\sqrt{a^{2}+c^{2}}}$. If we let $T$ be the foot from $Q$ to $B C$, then $\\triangle H Q T \\tilde{+} \\triangle A H C$ and so the $x$-coordinate of $Q$ is given by $H Q \\cdot \\frac{A H}{A C}=\\frac{a^{2} c}{a^{2}+c^{2}}$. Repeating the analogous calculation for $Q$ and $P$ gives $$ \\begin{aligned} Q & =\\left(\\frac{a^{2} c}{a^{2}+c^{2}}, \\frac{a c^{2}}{a^{2}+c^{2}}\\right) \\\\ P & =\\left(-\\frac{a^{2} b}{a^{2}+b^{2}}, \\frac{a b^{2}}{a^{2}+b^{2}}\\right) . \\end{aligned} $$ Then, $O, P, Q$ are collinear if and only if the following shoelace determinant vanishes (with denominators cleared out): $$ \\begin{aligned} 0 & =\\operatorname{det}\\left[\\begin{array}{ccc} -a^{2} b & a b^{2} & a^{2}+b^{2} \\\\ a^{2} c & a c^{2} & a^{2}+c^{2} \\\\ a(c-b) & a^{2}-b c & 2 a \\end{array}\\right]=a \\operatorname{det}\\left[\\begin{array}{ccc} -a b & a b^{2} & a^{2}+b^{2} \\\\ a c & a c^{2} & a^{2}+c^{2} \\\\ c-b & a^{2}-b c & 2 a \\end{array}\\right] \\\\ & =a \\operatorname{det}\\left[\\begin{array}{ccc} -a(b+c) & a\\left(b^{2}-c^{2}\\right) & b^{2}-c^{2} \\\\ a c & a c^{2} & a^{2}+c^{2} \\\\ c-b & a^{2}-b c & 2 a \\end{array}\\right]=a(b+c) \\operatorname{det}\\left[\\begin{array}{ccc} -a & a(b-c) & b-c \\\\ a c & a c^{2} & a^{2}+c^{2} \\\\ c-b & a^{2}-b c & 2 a \\end{array}\\right] \\\\ & =a(b+c) \\cdot\\left[-a\\left(a^{2} c^{2}-a^{4}+b c\\left(a^{2}+c^{2}\\right)\\right)+a c(b-c)\\left(-a^{2}-b c\\right)-(b-c)^{2} \\cdot a^{3}\\right] \\\\ & =a^{2}(b+c)\\left(a^{4}-a^{2} b^{2}-b^{2} c^{2}-c^{2} a^{2}\\right) . \\end{aligned} $$ On the other hand, $$ \\begin{aligned} A H^{2} & =a^{2} \\\\ 2 A O^{2} & =2\\left[\\left(\\frac{c-b}{2}\\right)^{2}+\\left(-\\frac{a}{2}-\\frac{b c}{2 a}\\right)^{2}\\right]=\\frac{a^{2}+b^{2}+c^{2}+\\frac{b^{2} c^{2}}{a^{2}}}{2} \\\\ \\Longrightarrow A H^{2}-2 A O^{2} & =\\frac{1}{2}\\left(a^{2}-b^{2}-c^{2}-\\frac{b^{2} c^{2}}{a^{2}}\\right) . \\end{aligned} $$ So the conditions are equivalent."} +{"year":2016,"label":"6","problem":"Find all functions $f: \\mathbb{R} \\rightarrow \\mathbb{R}$ such that for all real numbers $x$ and $y$, $$ (f(x)+x y) \\cdot f(x-3 y)+(f(y)+x y) \\cdot f(3 x-y)=(f(x+y))^{2} . $$","solution":" We claim that the only two functions satisfying the requirements are $f(x) \\equiv 0$ and $f(x) \\equiv x^{2}$. These work. First, taking $x=y=0$ in the given yields $f(0)=0$, and then taking $x=0$ gives $f(y) f(-y)=f(y)^{2}$. So also $f(-y)^{2}=f(y) f(-y)$, from which we conclude $f$ is even. Then taking $x=-y$ gives $$ \\forall x \\in \\mathbb{R}: \\quad f(x)=x^{2} \\quad \\text { or } \\quad f(4 x)=0 $$ for all $x$. Remark. Note that an example of a function satisfying $(\\boldsymbol{\\star})$ is $$ f(x)= \\begin{cases}x^{2} & \\text { if }|x|<1 \\\\ 1-\\cos \\left(\\frac{\\pi}{2} \\cdot x^{1337}\\right) & \\text { if } 1 \\leq|x|<4 \\\\ 0 & \\text { if }|x| \\geq 4\\end{cases} $$ So, yes, we are currently in a world of trouble, still. (This function is even continuous; I bring this up to emphasize that \"continuity\" is completely unrelated to the issue at hand.) Now we claim $$ \\text { Claim }-f(z)=0 \\Longleftrightarrow f(2 z)=0 $$ $$ \\left(f(t)+3 t^{2}\\right) f(8 t)=f(4 t)^{2} $$ Now if $f(4 t) \\neq 0$ (in particular, $t \\neq 0$ ), then $f(8 t) \\neq 0$. Thus we have $(\\boldsymbol{\\phi})$ in the reverse direction. Then $f(4 t) \\neq 0 \\xlongequal{(\\star)} f(t)=t^{2} \\neq 0 \\xlongequal{(\\bullet)} f(2 t) \\neq 0$ implies the forwards direction, the last step being the reverse direction By putting together $(\\boldsymbol{\\star})$ and $(\\boldsymbol{\\leftrightarrow})$ we finally get $$ \\forall x \\in \\mathbb{R}: \\quad f(x)=x^{2} \\quad \\text { or } \\quad f(x)=0 $$ Let $b \\in \\mathbb{R}$ be given. Since $f$ is even, we can assume without loss of generality that $a, b>0$. Also, note that $f(x) \\geq 0$ for all $x$ by ( $($ ). By using ( $\\boldsymbol{\\sim}$ ) we can generate $c>b$ such that $f(c)=0$ by taking $c=2^{n} a$ for a large enough integer $n$. Now, select $x, y>0$ such that $x-3 y=b$ and $x+y=c$. That is, $$ (x, y)=\\left(\\frac{3 c+b}{4}, \\frac{c-b}{4}\\right) . $$ Substitution into the original equation gives $$ 0=(f(x)+x y) f(b)+(f(y)+x y) f(3 x-y) \\geq(f(x)+x y) f(b) $$ But since $f(b) \\geq 0$, it follows $f(b)=0$, as desired."} diff --git a/USAJMO/segmented/en-JMO-2017-notes.jsonl b/USAJMO/segmented/en-JMO-2017-notes.jsonl new file mode 100644 index 0000000000000000000000000000000000000000..efb099d045091062ddd565bad01c32d37c4e4b68 --- /dev/null +++ b/USAJMO/segmented/en-JMO-2017-notes.jsonl @@ -0,0 +1,8 @@ +{"year":2017,"label":"1","problem":"Prove that there exist infinitely many pairs of relatively prime positive integers $a, b>1$ for which $a+b$ divides $a^{b}+b^{a}$.","solution":" One construction: let $d \\equiv 1(\\bmod 4), d>1$. Let $x=\\frac{d^{d}+2^{d}}{d+2}$. Then set $$ a=\\frac{x+d}{2}, \\quad b=\\frac{x-d}{2} $$ To see this works, first check that $b$ is odd and $a$ is even. Let $d=a-b$ be odd. Then: $$ \\begin{aligned} a+b \\mid a^{b}+b^{a} & \\Longleftrightarrow(-b)^{b}+b^{a} \\equiv 0 \\quad(\\bmod a+b) \\\\ & \\Longleftrightarrow b^{a-b} \\equiv 1 \\quad(\\bmod a+b) \\\\ & \\Longleftrightarrow b^{d} \\equiv 1 \\quad(\\bmod d+2 b) \\\\ & \\Longleftrightarrow(-2)^{d} \\equiv d^{d} \\quad(\\bmod d+2 b) \\\\ & \\Longleftrightarrow d+2 b \\mid d^{d}+2^{d} . \\end{aligned} $$ So it would be enough that $$ d+2 b=\\frac{d^{d}+2^{d}}{d+2} \\Longrightarrow b=\\frac{1}{2}\\left(\\frac{d^{d}+2^{d}}{d+2}-d\\right) $$ which is what we constructed. Also, since $\\operatorname{gcd}(x, d)=1$ it follows $\\operatorname{gcd}(a, b)=\\operatorname{gcd}(d, b)=$ 1. Remark. Ryan Kim points out that in fact, $(a, b)=(2 n-1,2 n+1)$ is always a solution."} +{"year":2017,"label":"2","problem":"Show that the Diophantine equation $$ \\left(3 x^{3}+x y^{2}\\right)\\left(x^{2} y+3 y^{3}\\right)=(x-y)^{7} $$ has infinitely many solutions in positive integers, and characterize all the solutions.","solution":"Show that the Diophantine equation $$ \\left(3 x^{3}+x y^{2}\\right)\\left(x^{2} y+3 y^{3}\\right)=(x-y)^{7} $$ Let $x=d a, y=d b$, where $\\operatorname{gcd}(a, b)=1$ and $a>b$. The equation is equivalent to $$ (a-b)^{7} \\mid a b\\left(a^{2}+3 b^{2}\\right)\\left(3 a^{2}+b^{2}\\right) $$ with the ratio of the two becoming $d$. Claim - The equation ( $\\star$ ) holds if and only if $a-b=1$. - If $a$ and $b$ are both odd, then $a^{2}+3 b^{2} \\equiv 4(\\bmod 8)$. Similarly $3 a^{2}+b^{2} \\equiv 4(\\bmod 8)$. Hence $2^{4}$ exactly divides right-hand side, contradiction. - Now suppose $a-b$ is odd. We have $\\operatorname{gcd}(a-b, a)=\\operatorname{gcd}(a-b, b)=1$ by Euclid, but also $$ \\operatorname{gcd}\\left(a-b, a^{2}+3 b^{2}\\right)=\\operatorname{gcd}\\left(a-b, 4 b^{2}\\right)=1 $$ and similarly $\\operatorname{gcd}\\left(a-b, 3 a^{2}+b^{2}\\right)=1$. Thus $a-b$ is coprime to each of $a, b, a^{2}+3 b^{2}$, $3 a^{2}+b^{2}$ and this forces $a-b=1$. Remark. One can give different cosmetic representations of the same solution set. For example, we could write $b=\\frac{1}{2}(n-1)$ and $a=\\frac{1}{2}(n+1)$ with $n>1$ any odd integer. Then $d=a b\\left(a^{2}+3 b^{2}\\right)\\left(3 a^{2}+b^{2}\\right)=\\frac{(n-1)(n+1)\\left(n^{2}+n+1\\right)\\left(n^{2}-n+1\\right)}{4}=\\frac{n^{6}-1}{4}$, and hence the solution is $$ (x, y)=(d a, d b)=\\left(\\frac{(n+1)\\left(n^{6}-1\\right)}{8}, \\frac{(n-1)\\left(n^{6}-1\\right)}{8}\\right) $$ which is a little simpler to write. The smallest solutions are $(364,182),(11718,7812), \\ldots$"} +{"year":2017,"label":"3","problem":"Let $A B C$ be an equilateral triangle and $P$ a point on its circumcircle. Set $D=$ $\\overline{P A} \\cap \\overline{B C}, E=\\overline{P B} \\cap \\overline{C A}, F=\\overline{P C} \\cap \\overline{A B}$. Prove that the area of triangle $D E F$ is twice the area of triangle $A B C$.","solution":" \\ First solution (barycentric). We invoke barycentric coordinates on $A B C$. Let $P=(u: v: w)$, with $u v+v w+w u=0$ (circumcircle equation with $a=b=c$ ). Then $D=(0: v: w), E=(u: 0: w), F=(u: v: 0)$. Hence $$ \\begin{aligned} \\frac{[D E F]}{[A B C]} & =\\frac{1}{(u+v)(v+w)(w+u)} \\operatorname{det}\\left[\\begin{array}{lll} 0 & v & w \\\\ u & 0 & w \\\\ u & v & 0 \\end{array}\\right] \\\\ & =\\frac{2 u v w}{(u+v)(v+w)(w+u)} \\\\ & =\\frac{2 u v w}{(u+v+w)(u v+v w+w u)-u v w} \\\\ & =\\frac{2 u v w}{-u v w}=-2 \\end{aligned} $$ as desired (areas signed)."} +{"year":2017,"label":"3","problem":"Let $A B C$ be an equilateral triangle and $P$ a point on its circumcircle. Set $D=$ $\\overline{P A} \\cap \\overline{B C}, E=\\overline{P B} \\cap \\overline{C A}, F=\\overline{P C} \\cap \\overline{A B}$. Prove that the area of triangle $D E F$ is twice the area of triangle $A B C$.","solution":" \u300e Second solution (\"nice\" lengths). WLOG $A B P C$ is convex. Let $x=A B=B C=$ $C A$. By Ptolemy's theorem and strong Ptolemy, $$ \\begin{aligned} P A & =P B+P C \\\\ P A^{2} & =P B \\cdot P C+A B \\cdot A C=P B \\cdot P C+x^{2} \\\\ \\Longrightarrow x^{2} & +P B^{2}+P B \\cdot P C+P C^{2} . \\end{aligned} $$ Also, $P D \\cdot P A=P B \\cdot P C$ and similarly since $\\overline{P A}$ bisects $\\angle B P C$ (causing $\\triangle B P D \\sim$ $\\triangle A P C)$. Now $P$ is the Fermat point of $\\triangle D E F$, since $\\angle D P F=\\angle F P E=\\angle E P D=120^{\\circ}$. Thus $$ \\begin{aligned} {[D E F] } & =\\frac{\\sqrt{3}}{4} \\sum_{\\mathrm{cyc}} P E \\cdot P F \\\\ & =\\frac{\\sqrt{3}}{4} \\sum_{\\mathrm{cyc}}\\left(\\frac{P A \\cdot P C}{P B}\\right)\\left(\\frac{P A \\cdot P B}{P C}\\right) \\\\ & =\\frac{\\sqrt{3}}{4} \\sum_{\\mathrm{cyc}} P A^{2} \\\\ & =\\frac{\\sqrt{3}}{4}\\left((P B+P C)^{2}+P B^{2}+P C^{2}\\right) \\\\ & =\\frac{\\sqrt{3}}{4} \\cdot 2 x^{2}=2[A B C] . \\end{aligned} $$"} +{"year":2017,"label":"4","problem":"Are there any triples $(a, b, c)$ of positive integers such that $(a-2)(b-2)(c-2)+12$ is a prime number that properly divides the positive number $a^{2}+b^{2}+c^{2}+a b c-2017$ ?","solution":"$ No such $(a, b, c)$. Assume not. Let $x=a-2, y=b-2, z=c-2$, hence $x, y, z \\geq-1$. $$ \\begin{aligned} a^{2}+b^{2}+c^{2}+a b c-2017 & =(x+2)^{2}+(y+2)^{2}+(z+2)^{2} \\\\ & +(x+2)(y+2)(z+2)-2017 \\\\ & =(x+y+z+4)^{2}+(x y z+12)-45^{2} \\end{aligned} $$ Thus the divisibility relation becomes $$ p=x y z+12 \\mid(x+y+z+4)^{2}-45^{2}>0 $$ so either $$ \\begin{aligned} & p=x y z+12 \\mid x+y+z-41 \\\\ & p=x y z+12 \\mid x+y+z+49 \\end{aligned} $$ Assume $x \\geq y \\geq z$, hence $x \\geq 14$ (since $x+y+z \\geq 41$ ). We now eliminate several edge cases to get $x, y, z \\neq-1$ and a little more: Claim - We have $x \\geq 17, y \\geq 5, z \\geq 1$, and $\\operatorname{gcd}(x y z, 6)=1$. - If $x>0$ and $y=z=-1$, then we want $p=x+12$ to divide either $x-43$ or $x+47$. We would have $0 \\equiv x-43 \\equiv-55(\\bmod p)$ or $0 \\equiv x+47 \\equiv 35(\\bmod p)$, but $p>11$ contradiction. - If $x, y>0$, and $z=-1$, then $p=12-x y>0$. However, this is clearly incompatible with $x \\geq 14$. Finally, obviously $x y z \\neq 0$ (else $p=12$ ). So $p=x y z+12 \\geq 14 \\cdot 1^{2}+12=26$ or $p \\geq 29$. Thus $\\operatorname{gcd}(6, p)=1$ hence $\\operatorname{gcd}(6, x y z)=1$. We finally check that $y=1$ is impossible, which forces $y \\geq 5$. If $y=1$ and hence $z=1$ then $p=x+12$ should divide either $x+51$ or $x-39$. These give $39 \\equiv 0(\\bmod p)$ or $25 \\equiv 0(\\bmod p)$, but we are supposed to have $p \\geq 29$. In that situation $x+y+z-41$ and $x+y+z+49$ are both even, so whichever one is divisible by $p$ is actually divisible by $2 p$. Now we deduce that: $$ x+y+z+49 \\geq 2 p=2 x y z+24 \\Longrightarrow 25 \\geq 2 x y z-x-y-z $$ But $x \\geq 17$ and $y \\geq 5$ thus $$ \\begin{aligned} 2 x y z-x-y-z & =z(2 x y-1)-x-y \\\\ & \\geq 2 x y-1-x-y \\\\ & >(x-1)(y-1)>60 \\end{aligned} $$ which is a contradiction. Having exhausted all the cases we conclude no solutions exist. Remark. The condition that $x+y+z-41>0$ (which comes from \"properly divides\") cannot be dropped. Examples of solutions in which $x+y+z-41=0$ include $(x, y, z)=(31,5,5)$ and $(x, y, z)=(29,11,1)$."} +{"year":2017,"label":"5","problem":"Let $O$ and $H$ be the circumcenter and the orthocenter of an acute triangle $A B C$. Points $M$ and $D$ lie on side $B C$ such that $B M=C M$ and $\\angle B A D=\\angle C A D$. Ray $M O$ intersects the circumcircle of triangle $B H C$ in point $N$. Prove that $\\angle A D O=\\angle H A N$.","solution":" Let $P$ and $Q$ be the arc midpoints of $\\widehat{B C}$, so that $A D M Q$ is cyclic (as $\\measuredangle Q A D=$ $\\measuredangle Q M D=90^{\\circ}$ ). Since it's known that $(B H C)$ and $(A B C)$ are reflections across line $B C$, it follows $N$ is the reflection of the arc midpoint $P$ across $M$. Claim - Quadrilateral $A D N O$ is cyclic. ![](https:\/\/cdn.mathpix.com\/cropped\/2024_11_19_ecd01a6fb7aaf697c013g-08.jpg?height=806&width=738&top_left_y=1156&top_left_x=659) To finish, note that $\\measuredangle H A N=\\measuredangle O N A=\\measuredangle O D A$. Remark. The orthocenter $H$ is superficial and can be deleted basically immediately. One can reverse-engineer the fact that $A D N O$ is cyclic from the truth of the problem statement. Remark. One can also show $A D N O$ concyclic by just computing $\\measuredangle D A O=\\measuredangle P A O$ and $\\measuredangle D N O=\\measuredangle D P N=\\measuredangle A P Q$ in terms of the angles of the triangle, or even more directly just because $$ \\measuredangle D N O=\\measuredangle D N P=\\measuredangle N P D=\\measuredangle O P D=\\measuredangle O N A=\\measuredangle H A N $$"} +{"year":2017,"label":"6","problem":"Let $P_{1}, P_{2}, \\ldots, P_{2 n}$ be $2 n$ distinct points on the unit circle $x^{2}+y^{2}=1$, other than $(1,0)$. Each point is colored either red or blue, with exactly $n$ red points and $n$ blue points. Let $R_{1}, R_{2}, \\ldots, R_{n}$ be any ordering of the red points. Let $B_{1}$ be the nearest blue point to $R_{1}$ traveling counterclockwise around the circle starting from $R_{1}$. Then let $B_{2}$ be the nearest of the remaining blue points to $R_{2}$ travelling counterclockwise around the circle from $R_{2}$, and so on, until we have labeled all of the blue points $B_{1}, \\ldots, B_{n}$. Show that the number of counterclockwise arcs of the form $R_{i} \\rightarrow B_{i}$ that contain the point $(1,0)$ is independent of the way we chose the ordering $R_{1}, \\ldots, R_{n}$ of the red points.","solution":" \\ First \"local\" solution by swapping two points. Let $1 \\leq i0$ implies $$ \\{a, c\\}=\\{\\sqrt{6} \\pm \\sqrt{2}\\} $$ This gives the $x$ values claimed above; by taking $a, b, c$ as deduced here, we find they work too. Remark. Note that by perturbing $\\triangle A B C$ slightly, we see $a$ priori that the set of possible $x$ should consist of unions of intervals (possibly trivial). So it makes sense to try inequalities no matter what."} +{"year":2018,"label":"5","problem":"Let $p$ be a prime, and let $a_{1}, \\ldots, a_{p}$ be integers. Show that there exists an integer $k$ such that the numbers $$ a_{1}+k, a_{2}+2 k, \\ldots, a_{p}+p k $$ produce at least $\\frac{1}{2} p$ distinct remainders upon division by $p$.","solution":" For each $k=0, \\ldots, p-1$ let $G_{k}$ be the graph on $\\{1, \\ldots, p\\}$ where we join $\\{i, j\\}$ if and only if $$ a_{i}+i k \\equiv a_{j}+j k \\quad(\\bmod p) \\Longleftrightarrow k \\equiv-\\frac{a_{i}-a_{j}}{i-j} \\quad(\\bmod p) $$ So we want a graph $G_{k}$ with at least $\\frac{1}{2} p$ connected components. However, each $\\{i, j\\}$ appears in exactly one graph $G_{k}$, so some graph has at most $\\frac{1}{p}\\binom{p}{2}=\\frac{1}{2}(p-1)$ edges (by \"pigeonhole\"). This graph has at least $\\frac{1}{2}(p+1)$ connected components, as desired. Remark. Here is an example for $p=5$ showing equality can occur: $$ \\left[\\begin{array}{lllll} 0 & 0 & 3 & 4 & 3 \\\\ 0 & 1 & 0 & 2 & 2 \\\\ 0 & 2 & 2 & 0 & 1 \\\\ 0 & 3 & 4 & 3 & 0 \\\\ 0 & 4 & 1 & 1 & 4 \\end{array}\\right] . $$ Ankan Bhattacharya points out more generally that $a_{i}=i^{2}$ is sharp in general."} +{"year":2018,"label":"6","problem":"Karl starts with $n$ cards labeled $1,2, \\ldots, n$ lined up in random order on his desk. He calls a pair $(a, b)$ of cards swapped if $a>b$ and the card labeled $a$ is to the left of the card labeled $b$. Karl picks up the card labeled 1 and inserts it back into the sequence in the opposite position: if the card labeled 1 had $i$ cards to its left, then it now has $i$ cards to its right. He then picks up the card labeled 2 and reinserts it in the same manner, and so on, until he has picked up and put back each of the cards $1, \\ldots, n$ exactly once in that order. For example, if $n=4$, then one example of a process is $$ 3142 \\longrightarrow 3412 \\longrightarrow 2341 \\longrightarrow 2431 \\longrightarrow 2341 $$ which has three swapped pairs both before and after. Show that, no matter what lineup of cards Karl started with, his final lineup has the same number of swapped pairs as the starting lineup.","solution":" We define a new process $P^{\\prime}$ where, when re-inserting card $i$, we additionally change its label from $i$ to $n+i$. An example of $P^{\\prime}$ also starting with 3142 is: $$ 3142 \\longrightarrow 3452 \\longrightarrow 6345 \\longrightarrow 6475 \\longrightarrow 6785 . $$ Note that now, each step of $P^{\\prime}$ preserves the number of inversions. Moreover, the final configuration of $P^{\\prime}$ is the same as the final configuration of $P$ with all cards incremented by $n$, and of course thus has the same number of inversions. Boom."} diff --git a/USAJMO/segmented/en-JMO-2019-notes.jsonl b/USAJMO/segmented/en-JMO-2019-notes.jsonl new file mode 100644 index 0000000000000000000000000000000000000000..67ff4d453042af4ae2a7e1d2ca2f4fa3c5b0b5e1 --- /dev/null +++ b/USAJMO/segmented/en-JMO-2019-notes.jsonl @@ -0,0 +1,14 @@ +{"year":2019,"label":"1","problem":"There are $a+b$ bowls arranged in a row, numbered 1 through $a+b$, where $a$ and $b$ are given positive integers. Initially, each of the first $a$ bowls contains an apple, and each of the last $b$ bowls contains a pear. A legal move consists of moving an apple from bowl $i$ to bowl $i+1$ and a pear from bowl $j$ to bowl $j-1$, provided that the difference $i-j$ is even. We permit multiple fruits in the same bowl at the same time. The goal is to end up with the first $b$ bowls each containing a pear and the last $a$ bowls each containing an apple. Show that this is possible if and only if the product $a b$ is even.","solution":" First we show that if $a b$ is even then the goal is possible. We prove the result by induction on $a+b$. - If $\\min (a, b)=0$ there is nothing to check. - If $\\min (a, b)=1$, say $a=1$, then $b$ is even, and we can swap the (only) leftmost apple with the rightmost pear by working only with those fruits. - Now assume $\\min (a, b) \\geq 2$ and $a+b$ is odd. Then we can swap the leftmost apple with rightmost pear by working only with those fruits, reducing to the situation of $(a-1, b-1)$ which is possible by induction (at least one of them is even). - Finally assume $\\min (a, b) \\geq 2$ and $a+b$ is even (i.e. $a$ and $b$ are both even). Then we can swap the apple in position 1 with the pear in position $a+b-1$, and the apple in position 2 with the pear in position $a+b$. This reduces to the situation of $(a-2, b-2)$ which is also possible by induction. Now we show that the result is impossible if $a b$ is odd. Define $$ \\begin{aligned} & X=\\text { number apples in odd-numbered bowls } \\\\ & Y=\\text { number pears in odd-numbered bowls. } \\end{aligned} $$ Note that $X-Y$ does not change under this operation. However, if $a$ and $b$ are odd, then we initially have $X=\\frac{1}{2}(a+1)$ and $Y=\\frac{1}{2}(b-1)$, while the target position has $X=\\frac{1}{2}(a-1)$ and $Y=\\frac{1}{2}(b+1)$. So when $a b$ is odd this is not possible. First, note that apples only move right and pears only move left, a successful operation must take exactly $a b$ moves. So it is enough to prove that the number of moves made must be even. However, the number of fruits in odd-numbered bowls either increases by +2 or -2 in each move (according to whether $i$ and $j$ are both even or both odd), and since it ends up being the same at the end, the number of moves must be even. Alternatively, as pointed out in the official solutions, one can consider the sums of squares of positions of fruits. The quantity changes by $$ \\left[(i+1)^{2}+(j-1)^{2}\\right]-\\left(i^{2}+j^{2}\\right)=2(i-j)+2 \\equiv 2(\\bmod 4) $$ at each step, and eventually the sums of squares returns to zero, as needed."} +{"year":2019,"label":"2","problem":"For which pairs of integers $(a, b)$ do there exist functions $f: \\mathbb{Z} \\rightarrow \\mathbb{Z}$ and $g: \\mathbb{Z} \\rightarrow \\mathbb{Z}$ obeying $$ f(g(x))=x+a \\quad \\text { and } \\quad g(f(x))=x+b $$ for all integers $x$ ?","solution":" The answer is if $a=b$ or $a=-b$. In the former case, one can take $f(x) \\equiv x+a$ and $g(x) \\equiv x$. In the latter case, one can take $f(x) \\equiv-x+a$ and $g(x)=-x$. Now we prove these are the only possibilities. First: Claim - The functions $f$ and $g$ are bijections. Note also that for any $x$, we have $$ \\begin{aligned} & f(x+b)=f(g(f(x)))=f(x)+a \\\\ & g(x+a)=g(f(g(x)))=g(x)+b . \\end{aligned} $$ If either $a$ is zero or $b$ is zero, we immediately get the other is zero, and hence done. So assume $a b \\neq 0$. If $|b|>|a|$, then two of $$ \\{f(0), f(1), \\ldots, f(b-1)\\} \\quad(\\bmod |a|) $$ coincide, which together with repeatedly applying the first equation above will then give a contradiction to injectivity of $f$. Similarly, if $|a|>|b|$ swapping the roles of $f$ and $g$ (and $a$ and $b$ ) will give a contradiction to injectivity of $g$. This completes the proof. Remark. Here is a way to visualize the argument, so one can see pictorially what is going on. We draw two parallel number lines indexed by $\\mathbb{Z}$. Starting from 0 , we draw red arrow from 0 to $f(0)$, and then a blue arrow from $f(0)$ to $g(f(0))=b$, and then a red arrow from $b$ to $g(b)=f(0)+a$, and so on. These arrows can be extended both directions, leading to an infinite \"squaretooth\" wave. The following is a picture of an example with $a, b>0$. ![](https:\/\/cdn.mathpix.com\/cropped\/2024_11_19_10fe2e18e797a050a222g-05.jpg?height=358&width=1112&top_left_y=2134&top_left_x=478) The problem is essentially trying to decompose our two copies of $\\mathbb{Z}$ into multiple squaretooth waves. For this to be possible, we expect the \"width\" of the waves on the top and bottom must be the same - i.e., that $|a|=|b|$. Remark. This also suggests how to classify all functions $f$ and $g$ satisfying the condition. If $a=b=0$ then any pair of functions $f$ and $g$ which are inverses to each other is okay. There are thus uncountably many pairs of functions $(f, g)$ here. If $a=b>0$, then one sets $f(0), f(1), \\ldots, f(a-1)$ as any values which are distinct modulo $b$, at which point $f$ and $g$ are uniquely determined. An example for $a=b=3$ is $$ f(x)=\\left\\{\\begin{array}{lll} x+42 & x \\equiv 0 & (\\bmod 3) \\\\ x+13 & x \\equiv 1 & (\\bmod 3) \\\\ x-37 & x \\equiv 2 & (\\bmod 3), \\end{array} \\quad g(x)=\\left\\{\\begin{array}{lll} x-39 & x \\equiv 0 & (\\bmod 3) \\\\ x+40 & x \\equiv 1 & (\\bmod 3) \\\\ x-10 & x \\equiv 2 & (\\bmod 3) \\end{array}\\right.\\right. $$ The analysis for $a=b<0$ and $a=-b$ are similar, but we don't include the details here."} +{"year":2019,"label":"3","problem":"Let $A B C D$ be a cyclic quadrilateral satisfying $A D^{2}+B C^{2}=A B^{2}$. The diagonals of $A B C D$ intersect at $E$. Let $P$ be a point on side $\\overline{A B}$ satisfying $\\angle A P D=\\angle B P C$. Show that line $P E$ bisects $\\overline{C D}$.","solution":" \u3010 First solution using symmedians. We define point $P$ to obey $$ \\frac{A P}{B P}=\\frac{A D^{2}}{B C^{2}}=\\frac{A E^{2}}{B E^{2}} $$ so that $\\overline{P E}$ is the $E$-symmedian of $\\triangle E A B$, therefore the $E$-median of $\\triangle E C D$. Now, note that $$ A D^{2}=A P \\cdot A B \\quad \\text { and } \\quad B C^{2}=B P \\cdot B A $$ This implies $\\triangle A P D \\sim \\triangle A D B$ and $\\triangle B P C \\sim \\triangle B C A$. Thus $$ \\measuredangle D P A=\\measuredangle A D B=\\measuredangle A C B=\\measuredangle B C P $$ and so $P$ satisfies the condition as in the statement (and is the unique point to do so), as needed."} +{"year":2019,"label":"3","problem":"Let $A B C D$ be a cyclic quadrilateral satisfying $A D^{2}+B C^{2}=A B^{2}$. The diagonals of $A B C D$ intersect at $E$. Let $P$ be a point on side $\\overline{A B}$ satisfying $\\angle A P D=\\angle B P C$. Show that line $P E$ bisects $\\overline{C D}$.","solution":" \u3010 Second solution using only angle chasing (by proposer). We again re-define $P$ to obey $A D^{2}=A P \\cdot A B$ and $B C^{2}=B P \\cdot B A$. As before, this gives $\\triangle A P D \\sim \\triangle A B D$ and $\\triangle B P C \\sim \\triangle B D P$ and so we let $$ \\theta:=\\measuredangle D P A=\\measuredangle A D B=\\measuredangle A C B=\\measuredangle B C P . $$ Our goal is to now show $\\overline{P E}$ bisects $\\overline{C D}$. Let $K=\\overline{A C} \\cap \\overline{P D}$ and $L=\\overline{A D} \\cap \\overline{P C}$. Since $\\measuredangle K P A=\\theta=\\measuredangle A C B$, quadrilateral $B P K C$ is cyclic. Similarly, so is $A P L D$. ![](https:\/\/cdn.mathpix.com\/cropped\/2024_11_19_10fe2e18e797a050a222g-08.jpg?height=804&width=795&top_left_y=246&top_left_x=633) Finally $A K L B$ is cyclic since $$ \\measuredangle B K A=\\measuredangle B K C=\\measuredangle B P C=\\theta=\\measuredangle D P A=\\measuredangle D L A=\\measuredangle B L A . $$ This implies $\\measuredangle C K L=\\measuredangle L B A=\\measuredangle D C K$, so $\\overline{K L} \\| \\overline{B C}$. Then $P E$ bisects $\\overline{B C}$ by Ceva's theorem on $\\triangle P C D$."} +{"year":2019,"label":"3","problem":"Let $A B C D$ be a cyclic quadrilateral satisfying $A D^{2}+B C^{2}=A B^{2}$. The diagonals of $A B C D$ intersect at $E$. Let $P$ be a point on side $\\overline{A B}$ satisfying $\\angle A P D=\\angle B P C$. Show that line $P E$ bisects $\\overline{C D}$.","solution":" \u3010 Third solution (using inversion). By hypothesis, the circle $\\omega_{a}$ centered at $A$ with radius $A D$ is orthogonal to the circle $\\omega_{b}$ centered at $B$ with radius $B C$. For brevity, we let $\\mathbf{I}_{a}$ and $\\mathbf{I}_{b}$ denote inversion with respect to $\\omega_{a}$ and $\\omega_{b}$. We let $P$ denote the intersection of $\\overline{A B}$ with the radical axis of $\\omega_{a}$ and $\\omega_{b}$; hence $P=\\mathbf{I}_{a}(B)=\\mathbf{I}_{b}(A)$. This already implies that $$ \\measuredangle D P A \\stackrel{\\mathbf{I}_{a}}{=} \\measuredangle A D B=\\measuredangle A C B \\stackrel{\\mathbf{I}_{b}}{=} \\measuredangle B P C $$ so $P$ satisfies the angle condition. ![](https:\/\/cdn.mathpix.com\/cropped\/2024_11_19_10fe2e18e797a050a222g-08.jpg?height=732&width=1012&top_left_y=1844&top_left_x=522) Claim - The point $K=\\mathbf{I}_{a}(C)$ lies on $\\omega_{b}$ and $\\overline{D P}$. Similarly $L=\\mathbf{I}_{b}(D)$ lies on $\\omega_{a}$ and $\\overline{C P}$. Finally, since $C, L, P$ are collinear, we get $A$ is concyclic with $K=\\mathbf{I}_{a}(C), L=\\mathbf{I}_{a}(L)$, $B=\\mathbf{I}_{a}(P)$, i.e. that $A K L B$ is cyclic. So $\\overline{K L} \\| \\overline{C D}$ by Reim's theorem, and hence $\\overline{P E}$ bisects $\\overline{C D}$ by Ceva's theorem."} +{"year":2019,"label":"4","problem":"Let $A B C$ be a triangle with $\\angle B>90^{\\circ}$ and let $E$ and $F$ be the feet of the altitudes from $B$ and $C$. Can line $E F$ be tangent to the $A$-excircle?","solution":" We show it is not possible, by contradiction (assuming $E F$ is indeed tangent). Thus $B E C F$ is a convex cyclic quadrilateral inscribed in a circle with diameter $\\overline{B C}$. Note also that the $A$-excircle lies on the opposite side from $A$ as line $E F$, since $A, E, C$ are collinear in that order. \u3010 First solution by similarity. Note that $\\triangle A E F$ is similar to $\\triangle A B C$ (and oppositely oriented). However, since they have the same $A$-exradius, it follows they are congruent. ![](https:\/\/cdn.mathpix.com\/cropped\/2024_11_19_10fe2e18e797a050a222g-10.jpg?height=272&width=621&top_left_y=1195&top_left_x=729) Consequently we get $E F=B C$. But this implies $B F C E$ is a rectangle, contradiction."} +{"year":2019,"label":"4","problem":"Let $A B C$ be a triangle with $\\angle B>90^{\\circ}$ and let $E$ and $F$ be the feet of the altitudes from $B$ and $C$. Can line $E F$ be tangent to the $A$-excircle?","solution":" We show it is not possible, by contradiction (assuming $E F$ is indeed tangent). Thus $B E C F$ is a convex cyclic quadrilateral inscribed in a circle with diameter $\\overline{B C}$. Note also that the $A$-excircle lies on the opposite side from $A$ as line $E F$, since $A, E, C$ are collinear in that order. \u3010 Second length solution by tangent lengths. By $t(\\bullet)$ we mean the length of the tangent from $P$ to the $A$-excircle. It is a classical fact for example that $t(A)=s$. The main idea is to use the fact that $$ a \\cos A=E F=t(E)+t(F) . $$ Here $E F=a \\cos A$ follows from the extended law of sines applied to the circle with diameter $\\overline{B C}$, since there we have $E F=B C \\sin \\angle E C F=a \\sin \\angle A C F=a \\cos A$. We may now compute $$ \\begin{aligned} & t(E)=t(A)-A E=s-c \\cos A \\\\ & t(F)=t(A)-A F=s-b \\cos A \\end{aligned} $$ Therefore, $$ \\begin{aligned} a \\cos A=2 s-(b+c) \\cos A \\Longrightarrow \\quad(a+b+c) \\cos A & =2 s \\\\ \\Longrightarrow \\cos A & =1 . \\end{aligned} $$ This is an obvious contradiction. Remark. On the other hand, there really is an equality case with $A$ being some point at infinity (meaning $\\cos A=1$ ). So, this problem is \"sharper\" than one might expect; the answer is not \"obviously no\"."} +{"year":2019,"label":"4","problem":"Let $A B C$ be a triangle with $\\angle B>90^{\\circ}$ and let $E$ and $F$ be the feet of the altitudes from $B$ and $C$. Can line $E F$ be tangent to the $A$-excircle?","solution":" We show it is not possible, by contradiction (assuming $E F$ is indeed tangent). Thus $B E C F$ is a convex cyclic quadrilateral inscribed in a circle with diameter $\\overline{B C}$. Note also that the $A$-excircle lies on the opposite side from $A$ as line $E F$, since $A, E, C$ are collinear in that order. I Third solution by Pitot and trigonometry. In fact, the $t(\\bullet)$ notation from the previous solution gives us a classical theorem once we note the $A$-excircle is tangent to all four lines $E F, B C, B F$ and $C E$ : Claim (Pitot theorem) - We have $B F+E F=B C+C E$. $$ \\begin{array}{ll} B F=t(B)-t(F) & E F=t(E)+t(F) \\\\ B C=t(B)+t(C) & C E=t(E)-t(C) \\end{array} $$ and summing gives the result. ![](https:\/\/cdn.mathpix.com\/cropped\/2024_11_19_10fe2e18e797a050a222g-11.jpg?height=809&width=686&top_left_y=995&top_left_x=685) We now calculate all the lengths using trigonometry: $$ \\begin{aligned} & B C=a \\\\ & B F=a \\cos \\left(180^{\\circ}-B\\right)=a \\cos (A+C) \\\\ & C E=a \\cos C \\\\ & E F=B C \\sin \\angle E C F=a \\sin \\angle A C F=a \\cos A \\end{aligned} $$ Thus, we apparently have $$ \\cos (A+C)+\\cos A=1+\\cos C $$ but this is impossible since $\\cos (A+C)<\\cos C$ (since $A+C=180-B<90^{\\circ}$ ) and $\\cos A<1$."} +{"year":2019,"label":"4","problem":"Let $A B C$ be a triangle with $\\angle B>90^{\\circ}$ and let $E$ and $F$ be the feet of the altitudes from $B$ and $C$. Can line $E F$ be tangent to the $A$-excircle?","solution":" We show it is not possible, by contradiction (assuming $E F$ is indeed tangent). Thus $B E C F$ is a convex cyclic quadrilateral inscribed in a circle with diameter $\\overline{B C}$. Note also that the $A$-excircle lies on the opposite side from $A$ as line $E F$, since $A, E, C$ are collinear in that order. \u3010 Fourth solution by Pitot and Ptolemy (Evan Chen). We give a trig-free way to finish from Pitot's theorem $$ B F+E F=B C+C E . $$ Assume that $x=B F, y=C E$, and $B C=1$; then the above relation becomes $$ 1+y-x=B C+C E-B F=E F=E F \\cdot 1=x y+\\sqrt{\\left(1-x^{2}\\right)\\left(1-y^{2}\\right)} $$ with the last step by Ptolemy's theorem. This rearranges to give $$ (1+y)(1-x)=\\sqrt{\\left(1-x^{2}\\right)\\left(1-y^{2}\\right)} \\Longrightarrow \\frac{1+y}{1-y}=\\frac{1+x}{1-x} \\Longrightarrow x=y $$ but that means $B E C F$ is a rectangle: contradicting the fact that lines $B E$ and $C F$ meet at a point $A$."} +{"year":2019,"label":"4","problem":"Let $A B C$ be a triangle with $\\angle B>90^{\\circ}$ and let $E$ and $F$ be the feet of the altitudes from $B$ and $C$. Can line $E F$ be tangent to the $A$-excircle?","solution":" We show it is not possible, by contradiction (assuming $E F$ is indeed tangent). Thus $B E C F$ is a convex cyclic quadrilateral inscribed in a circle with diameter $\\overline{B C}$. Note also that the $A$-excircle lies on the opposite side from $A$ as line $E F$, since $A, E, C$ are collinear in that order. \u300e Fifth solution, by angle chasing only! Let $J$ denote the $A$-excenter. Then $J$ should be the intersection of the internal bisectors of $\\angle F E C$ and $\\angle F B C$, so it is the midpoint of arc $\\widehat{F C}$ on the circle with diameter $\\overline{B C}$. ![](https:\/\/cdn.mathpix.com\/cropped\/2024_11_19_10fe2e18e797a050a222g-12.jpg?height=824&width=690&top_left_y=1093&top_left_x=683) But now we get $\\angle B J C=90^{\\circ}$ from $J$ lying on this circle. Yet $\\angle B J C=90^{\\circ}-\\frac{1}{2} \\angle A$ in general, so $\\angle A=0^{\\circ}$ which is impossible."} +{"year":2019,"label":"4","problem":"Let $A B C$ be a triangle with $\\angle B>90^{\\circ}$ and let $E$ and $F$ be the feet of the altitudes from $B$ and $C$. Can line $E F$ be tangent to the $A$-excircle?","solution":" We show it is not possible, by contradiction (assuming $E F$ is indeed tangent). Thus $B E C F$ is a convex cyclic quadrilateral inscribed in a circle with diameter $\\overline{B C}$. Note also that the $A$-excircle lies on the opposite side from $A$ as line $E F$, since $A, E, C$ are collinear in that order. \u3010 Sixth solution (Zuming Feng). This is similar to the preceding solution, but phrased using contradiction and inequalities. We let $X$ and $Y$ denote the tangency points of the $A$-excircle on lines $A B$ and $A C$. Moreover, let $J$ denote the $A$-excenter. ![](https:\/\/cdn.mathpix.com\/cropped\/2024_11_19_10fe2e18e797a050a222g-12.jpg?height=350&width=809&top_left_y=2275&top_left_x=635) Note that $A B>A E$ and $A X=A Y$, therefore $B X$ $\\tan \\angle Y E J$, thus $$ \\angle X B J>\\angle Y E J $$ However, if line $E F$ was actually tangent to the $A$-excircle, we would have $$ 2 \\angle X B J=\\angle X B C=\\angle F B C=\\angle F E C=\\angle F E Y=2 \\angle J E Y $$ which is a contradiction."} +{"year":2019,"label":"4","problem":"Let $A B C$ be a triangle with $\\angle B>90^{\\circ}$ and let $E$ and $F$ be the feet of the altitudes from $B$ and $C$. Can line $E F$ be tangent to the $A$-excircle?","solution":" We show it is not possible, by contradiction (assuming $E F$ is indeed tangent). Thus $B E C F$ is a convex cyclic quadrilateral inscribed in a circle with diameter $\\overline{B C}$. Note also that the $A$-excircle lies on the opposite side from $A$ as line $E F$, since $A, E, C$ are collinear in that order. \u3010 Seventh solution, by complex numbers, for comedic effect (Evan Chen). Let us denote the tangency points of the $A$-excircle with sides $B C, C A, A B$ as $x, y, z$. Assume moreover that line $E F$ is tangent to the $A$-excircle at a point $P$. Also, for brevity let $s=x y+y z+z x$. Then, we have $$ \\begin{aligned} & E=\\frac{2 p y}{p+y}=\\frac{1}{2}\\left(b+y+y-y^{2} \\bar{b}\\right)=\\frac{z x}{z+x}+y-\\frac{y^{2}}{z+x} \\\\ & \\Longrightarrow \\frac{2}{\\frac{1}{p}+\\frac{1}{y}}=\\frac{x y+x z+z x-y^{2}}{z+x} \\Longrightarrow \\frac{\\frac{1}{p}+\\frac{1}{y}}{2}=\\frac{x+z}{s-y^{2}} \\end{aligned} $$ Similarly by considering the point $F$, $$ \\frac{\\frac{1}{p}+\\frac{1}{z}}{2}=\\frac{x+y}{s-z^{2}} $$ Thus we can eliminate $P$ and obtain $$ \\begin{aligned} \\Longrightarrow \\frac{\\frac{1}{y}-\\frac{1}{z}}{2} & =\\frac{x+z}{s-y^{2}}-\\frac{x+y}{s-z^{2}}=\\frac{-s(y-z)+x\\left(y^{2}-z^{2}\\right)+\\left(y^{3}-z^{3}\\right)}{\\left(s-y^{2}\\right)\\left(s-z^{2}\\right)} \\\\ \\Longleftrightarrow \\frac{1}{2 y z} & =\\frac{s-x(y+z)-\\left(y^{2}+y z+z^{2}\\right)}{\\left(s-y^{2}\\right)\\left(s-z^{2}\\right)}=\\frac{-\\left(y^{2}+z^{2}\\right)}{\\left(s-y^{2}\\right)\\left(s-z^{2}\\right)} \\\\ \\Longleftrightarrow 0 & =\\left(s-y^{2}\\right)\\left(s-z^{2}\\right)+2 y z\\left(y^{2}+z^{2}\\right) \\\\ & =[x(y+z)+y(z-y)][x(y+z)+z(y-z)]+2 y z\\left(y^{2}+z^{2}\\right) \\\\ & =x^{2}(y+z)^{2}-(y-z)^{2} \\cdot x(y+z)+y z\\left(2 y^{2}+2 z^{2}-(y-z)^{2}\\right) \\\\ & =x^{2}(y+z)^{2}-(y-z)^{2} \\cdot x(y+z)+y z(y+z)^{2} \\\\ & =x y z(y+z)\\left[\\frac{x}{y}+\\frac{x}{z}-\\frac{y}{z}-\\frac{z}{y}+2+\\frac{y}{x}+\\frac{z}{x}\\right] \\end{aligned} $$ However, $\\triangle X Y Z$ is obtuse with $\\angle X>90^{\\circ}$, we have $y+z \\neq 0$. Note that $$ \\begin{aligned} & \\frac{x}{y}+\\frac{y}{x}=2 \\operatorname{Re} \\frac{x}{y}=2 \\cos (2 \\angle X Z Y) \\\\ & \\frac{x}{z}+\\frac{z}{x}=2 \\operatorname{Re} \\frac{x}{z}=2 \\cos (2 \\angle X Y Z) \\\\ & \\frac{y}{z}+\\frac{z}{y}=2 \\operatorname{Re} \\frac{y}{z}<2 \\end{aligned} $$ and since $\\cos (2 \\angle X Z Y)+\\cos (2 \\angle X Y Z)>0$ (say by sum-to-product), we are done."} +{"year":2019,"label":"5","problem":"Let $n$ be a nonnegative integer. Determine the number of ways to choose sets $S_{i j} \\subseteq\\{1,2, \\ldots, 2 n\\}$, for all $0 \\leq i \\leq n$ and $0 \\leq j \\leq n$ (not necessarily distinct), such that - $\\left|S_{i j}\\right|=i+j$, and - $S_{i j} \\subseteq S_{k l}$ if $0 \\leq i \\leq k \\leq n$ and $0 \\leq j \\leq l \\leq n$.","solution":" The answer is $(2 n)!\\cdot 2^{n^{2}}$. First, we note that $\\varnothing=S_{00} \\subsetneq S_{01} \\subsetneq \\cdots \\subsetneq S_{n n}=\\{1, \\ldots, 2 n\\}$ and thus multiplying by (2n)! we may as well assume $S_{0 i}=\\{1, \\ldots, i\\}$ and $S_{i n}=\\{1, \\ldots, n+i\\}$. We illustrate this situation by placing the sets in a grid, as below for $n=4$; our goal is to fill in the rest of the grid. $\\left[\\begin{array}{ccccc}1234 & 12345 & 123456 & 1234567 & 12345678 \\\\ 123 & & & & \\\\ 12 & & & & \\\\ 1 & & & & \\\\ \\varnothing & & & & \\end{array}\\right]$ We claim the number of ways to do so is $2^{n^{2}}$. In fact, more strongly even the partial fillings are given exactly by powers of 2 . Claim - Fix a choice $T$ of cells we wish to fill in, such that whenever a cell is in $T$, so are all the cells above and left of it. (In other words, $T$ is a Young tableau.) The number of ways to fill in these cells with sets satisfying the inclusion conditions is $2^{|T|}$. An example is shown below, with an indeterminate set marked in red (and the rest of $T$ marked in blue). $\\left[\\begin{array}{ccccc}1234 & 12345 & 123456 & 1234567 & 12345678 \\\\ 123 & 1234 & 12346 & 123467 & \\\\ 12 & 124 & 1234 \\text { or } 1246 & & \\\\ 1 & 12 & & & \\\\ \\varnothing & 2 & & & \\end{array}\\right]$ Now suppose we have a corner $\\left[\\begin{array}{cc}B & C \\\\ A & S\\end{array}\\right]$ where $A, B, C$ are fixed and $S$ is to be chosen. Then we may write $B=A \\cup\\{x\\}$ and $C=A \\cup\\{x, y\\}$ for $x, y \\notin A$. Then the two choices of $S$ are $A \\cup\\{x\\}$ (i.e. $B$ ) and $A \\cup\\{y\\}$, and both of them are seen to be valid. In this way, we gain a factor of 2 any time we add one cell as above to $T$. Since we can achieve any Young tableau in this way, the induction is complete."} +{"year":2019,"label":"6","problem":"Let $m$ and $n$ be relatively prime positive integers. The numbers $\\frac{m}{n}$ and $\\frac{n}{m}$ are written on a blackboard. At any point, Evan may pick two of the numbers $x$ and $y$ written on the board and write either their arithmetic mean $\\frac{1}{2}(x+y)$ or their harmonic mean $\\frac{2 x y}{x+y}$. For which $(m, n)$ can Evan write 1 on the board in finitely many steps?","solution":" We claim this is possible if and only $m+n$ is a power of 2 . Let $q=m \/ n$, so the numbers on the board are $q$ and $1 \/ q$. \\I Impossibility. The main idea is the following. Claim - Suppose $p$ is an odd prime. Then if the initial numbers on the board are $-1(\\bmod p)$, then all numbers on the board are $-1(\\bmod p)$. Thus if there exists any odd prime divisor $p$ of $m+n$ (implying $p \\nmid m n$ ), then $$ q \\equiv \\frac{1}{q} \\equiv-1 \\quad(\\bmod p) . $$ and hence all numbers will be $-1(\\bmod p)$ forever. This implies that it's impossible to write 1 , whenever $m+n$ is divisible by some odd prime. \u3010 Construction. Conversely, suppose $m+n$ is a power of 2 . We will actually construct 1 without even using the harmonic mean. ![](https:\/\/cdn.mathpix.com\/cropped\/2024_11_19_10fe2e18e797a050a222g-15.jpg?height=152&width=1004&top_left_y=1786&top_left_x=526) Note that $$ \\frac{n}{m+n} \\cdot q+\\frac{m}{m+n} \\cdot \\frac{1}{q}=1 $$ and obviously by taking appropriate midpoints (in a binary fashion) we can achieve this using arithmetic mean alone."} diff --git a/USAJMO/segmented/en-JMO-2020-notes.jsonl b/USAJMO/segmented/en-JMO-2020-notes.jsonl new file mode 100644 index 0000000000000000000000000000000000000000..7d01f8a9b87bf544be09e080d0b549f10c7f3503 --- /dev/null +++ b/USAJMO/segmented/en-JMO-2020-notes.jsonl @@ -0,0 +1,9 @@ +{"year":2020,"label":"1","problem":"Let $n \\geq 2$ be an integer. Carl has $n$ books arranged on a bookshelf. Each book has a height and a width. No two books have the same height, and no two books have the same width. Initially, the books are arranged in increasing order of height from left to right. In a move, Carl picks any two adjacent books where the left book is wider and shorter than the right book, and swaps their locations. Carl does this repeatedly until no further moves are possible. Prove that regardless of how Carl makes his moves, he must stop after a finite number of moves, and when he does stop, the books are sorted in increasing order of width from left to right.","solution":" We say that a pair of books $(A, B)$ is height-inverted if $A$ is to the left of $B$ and taller than $A$. Similarly define width-inverted pairs. Note that every operation decreases the number of width-inverted pairs. This proves the procedure terminates, since the number of width-inverted pairs starts at $\\binom{n}{2}$ and cannot increase indefinitely. Now consider a situation where no more moves are possible. Assume for contradiction two consecutive books $(A, B)$ are still width-inverted. Since the operation isn't possible anymore, they are also height-inverted. In particular, the operation could never have swapped $A$ and $B$. But this contradicts the assumption there were no height-inverted pairs initially."} +{"year":2020,"label":"2","problem":"Let $\\omega$ be the incircle of a fixed equilateral triangle $A B C$. Let $\\ell$ be a variable line that is tangent to $\\omega$ and meets the interior of segments $B C$ and $C A$ at points $P$ and $Q$, respectively. A point $R$ is chosen such that $P R=P A$ and $Q R=Q B$. Find all possible locations of the point $R$, over all choices of $\\ell$.","solution":" Let $r$ be the inradius. Let $T$ be the tangency point of $\\overline{P Q}$ on arc $\\widehat{D E}$ of the incircle, which we consider varying. We define $R_{1}$ and $R_{2}$ to be the two intersections of the circle centered at $P$ with radius $P A$, and the circle centered at $Q$ with radius $Q B$. We choose $R_{1}$ to lie on the opposite side of $C$ as line $P Q$. ![](https:\/\/cdn.mathpix.com\/cropped\/2024_11_19_7a2c3e8ae10363caaf59g-04.jpg?height=906&width=1098&top_left_y=1003&top_left_x=482) Claim - The point $R_{1}$ is the unique point on ray $T I$ with $R_{1} I=2 r$. Note that since TASD is an isosceles trapezoid, it follows $P A=P S$. Similarly, $Q B=Q S$. So it follows that $S=R_{1}$. Since $T$ can be any point on the open arc $\\widehat{D E}$, it follows that the locus of $R_{1}$ is exactly the open $120^{\\circ}$ arc of $\\widehat{A B}$ of the circle centered at $I$ with radius $2 r$ (i.e. the circumcircle of $A B C)$. It remains to characterize $R_{2}$. Since $T I=r, I R_{1}=2 r$, it follows $T R_{2}=3 r$ and $I R_{2}=4 r$. Define $A^{\\prime}$ on ray $D I$ such that $A^{\\prime} I=4 r$, and $B^{\\prime}$ on ray $I E$ such that $B^{\\prime} I=4 r$. Then it follows, again by homothety, that the locus of $R_{2}$ is the $120^{\\circ}$ arc $\\widehat{A^{\\prime} B^{\\prime}}$ of the circle centered at $I$ with radius $4 r$. In conclusion, the locus of $R$ is the two open $120^{\\circ}$ arcs we identified."} +{"year":2020,"label":"3","problem":"An empty $2020 \\times 2020 \\times 2020$ cube is given, and a $2020 \\times 2020$ grid of square unit cells is drawn on each of its six faces. A beam is a $1 \\times 1 \\times 2020$ rectangular prism. Several beams are placed inside the cube subject to the following conditions: - The two $1 \\times 1$ faces of each beam coincide with unit cells lying on opposite faces of the cube. (Hence, there are $3 \\cdot 2020^{2}$ possible positions for a beam.) - No two beams have intersecting interiors. - The interiors of each of the four $1 \\times 2020$ faces of each beam touch either a face of the cube or the interior of the face of another beam. What is the smallest positive number of beams that can be placed to satisfy these conditions?","solution":" \\I Answer. 3030 beams. \u3010 Construction. We first give a construction with $3 n \/ 2$ beams for any $n \\times n \\times n$ box, where $n$ is an even integer. Shown below is the construction for $n=6$, which generalizes. (The left figure shows the cube in 3 d ; the right figure shows a direct view of the three visible faces.) ![](https:\/\/cdn.mathpix.com\/cropped\/2024_11_19_7a2c3e8ae10363caaf59g-06.jpg?height=815&width=1303&top_left_y=1551&top_left_x=315) ![](https:\/\/cdn.mathpix.com\/cropped\/2024_11_19_7a2c3e8ae10363caaf59g-06.jpg?height=295&width=281&top_left_y=2031&top_left_x=1113) Left face ![](https:\/\/cdn.mathpix.com\/cropped\/2024_11_19_7a2c3e8ae10363caaf59g-06.jpg?height=295&width=278&top_left_y=2034&top_left_x=1460) Right face To be explicit, impose coordinate axes such that one corner of the cube is the origin. We specify a beam by two opposite corners. The $3 n \/ 2$ beams come in three directions, $n \/ 2$ in each direction: - $(0,0,0) \\rightarrow(1,1, n),(2,2,0) \\rightarrow(3,3, n),(4,4,0) \\rightarrow(5,5, n)$, and so on; - $(1,0,0) \\rightarrow(2, n, 1),(3,0,2) \\rightarrow(4, n, 3),(5,0,4) \\rightarrow(6, n, 5)$, and so on; - $(0,1,1) \\rightarrow(n, 2,2),(0,3,3) \\rightarrow(n, 4,4),(0,5,5) \\rightarrow(n, 6,6)$, and so on. This gives the figure we drew earlier and shows 3030 beams is possible. Necessity. We now show at least $3 n \/ 2$ beams are necessary. Maintain coordinates, and call the beams $x$-beams, $y$-beams, $z$-beams according to which plane their long edges are perpendicular too. Let $N_{x}, N_{y}, N_{z}$ be the number of these. Claim - If $\\min \\left(N_{x}, N_{y}, N_{z}\\right)=0$, then at least $n^{2}$ beams are needed. We henceforth assume $\\min \\left(N_{x}, N_{y}, N_{z}\\right)>0$. Claim - If $N_{z}>0$, then we have $N_{x}+N_{y} \\geq n$. Proceeding in a similar fashion, we arrive at the three relations $$ \\begin{aligned} & N_{x}+N_{y} \\geq n \\\\ & N_{y}+N_{z} \\geq n \\\\ & N_{z}+N_{x} \\geq n \\end{aligned} $$ Summing gives $N_{x}+N_{y}+N_{z} \\geq 3 n \/ 2$ too. Remark. The problem condition has the following \"physics\" interpretation. Imagine the cube is a metal box which is sturdy enough that all beams must remain orthogonal to the faces of the box (i.e. the beams cannot spin). Then the condition of the problem is exactly what is needed so that, if the box is shaken or rotated, the beams will not move. Remark. Walter Stromquist points out that the number of constructions with 3030 beams is actually enormous: not dividing out by isometries, the number is $(2 \\cdot 1010!)^{3}$."} +{"year":2020,"label":"4","problem":"Let $A B C D$ be a convex quadrilateral inscribed in a circle and satisfying $$ D Ab$, then $f\\left(a^{2}-b^{2}\\right)=f(2 a b)=1$. $$ \\begin{aligned} 1=f(a) f(b) & =f\\left(a^{2}+b^{2}\\right)=\\sqrt{f\\left(\\left(a^{2}+b^{2}\\right)^{2}\\right)} \\\\ & =\\sqrt{f\\left(\\left(a^{2}-b^{2}\\right)^{2}+(2 a b)^{2}\\right)} \\\\ & =\\sqrt{f\\left(a^{2}-b^{2}\\right) f(2 a b)} . \\end{aligned} $$ By setting $a=b=1$ in the given statement we get $f(1)=f(2)=1$. Now a simple induction on $n$ shows $f(n)=1$ : - If $n=2 k$ take $(u, v)=(k, 1)$ hence $2 u v=n$. - If $n=2 k+1$ take $(u, v)=(k+1, k)$ hence $u^{2}-v^{2}=n$."} +{"year":2021,"label":"2","problem":"Rectangles $B C C_{1} B_{2}, C A A_{1} C_{2}$, and $A B B_{1} A_{2}$ are erected outside an acute triangle $A B C$. Suppose that $$ \\angle B C_{1} C+\\angle C A_{1} A+\\angle A B_{1} B=180^{\\circ} . $$ Prove that lines $B_{1} C_{2}, C_{1} A_{2}$, and $A_{1} B_{2}$ are concurrent.","solution":" The angle condition implies the circumcircles of the three rectangles concur at a single point $P$. ![](https:\/\/cdn.mathpix.com\/cropped\/2024_11_19_160003dbb585e37c9fb6g-04.jpg?height=812&width=786&top_left_y=982&top_left_x=641) Then $\\measuredangle C P B_{2}=\\measuredangle C P A_{1}=90^{\\circ}$, hence $P$ lies on $A_{1} B_{2}$ etc., so we're done. Remark. As one might guess from the two-sentence solution, the entire difficulty of the problem is getting the characterization of the concurrence point."} +{"year":2021,"label":"3","problem":"An equilateral triangle $\\Delta$ of side length $L>0$ is given. Suppose that $n$ equilateral triangles with side length 1 and with non-overlapping interiors are drawn inside $\\Delta$, such that each unit equilateral triangle has sides parallel to $\\Delta$, but with opposite orientation. Prove that $$ n \\leq \\frac{2}{3} L^{2} . $$","solution":" ![](https:\/\/cdn.mathpix.com\/cropped\/2024_11_19_160003dbb585e37c9fb6g-05.jpg?height=227&width=181&top_left_y=986&top_left_x=940) Claim - All the hexagons are disjoint and lie inside $\\Delta$. Since each hexagon has area $\\frac{3 \\sqrt{3}}{8}$ and lies inside $\\Delta$, we conclude $$ \\frac{3 \\sqrt{3}}{8} \\cdot n \\leq \\frac{\\sqrt{3}}{4} L^{2} \\Longrightarrow n \\leq \\frac{2}{3} L^{2} $$ Remark. The constant $\\frac{2}{3}$ is sharp and cannot be improved. The following tessellation shows how to achieve the $\\frac{2}{3}$ density. In the figure on the left, one of the green hexagons is drawn in for illustration. The version on the right has all the hexagons. ![](https:\/\/cdn.mathpix.com\/cropped\/2024_11_19_160003dbb585e37c9fb6g-05.jpg?height=591&width=1133&top_left_y=1874&top_left_x=458)"} +{"year":2021,"label":"4","problem":"Carina has three pins, labeled $A, B$, and $C$, respectively, located at the origin of the coordinate plane. In a move, Carina may move a pin to an adjacent lattice point at distance 1 away. What is the least number of moves that Carina can make in order for triangle $A B C$ to have area 2021?","solution":" The answer is 128 . Define the bounding box of triangle $A B C$ to be the smallest axis-parallel rectangle which contains all three of the vertices $A, B, C$. ![](https:\/\/cdn.mathpix.com\/cropped\/2024_11_19_160003dbb585e37c9fb6g-06.jpg?height=401&width=804&top_left_y=1050&top_left_x=629) ## Lemma The area of a triangle $A B C$ is at most half the area of the bounding box. So, suppose the bounding box is $A X Y Z$. Imagine fixing $C$ and varying $B$ along the perimeter entire rectangle. The area is a linear function of $B$, so the maximal area should be achieved when $B$ coincides with one of the vertices $\\{A, X, Y, Z\\}$. But obviously the area of $\\triangle A B C$ is - exactly 0 if $B=A$, - at most half the bounding box if $B \\in\\{X, Z\\}$ by one-half-base-height, - at most half the bounding box if $B=Y$, since $\\triangle A B C$ is contained inside either $\\triangle A Y Z$ or $\\triangle A X Z$. Claim - If $n$ moves are made, the bounding box has area at most $(n \/ 2)^{2}$. (In other words, a bounding box of area $A$ requires at least $\\lceil 2 \\sqrt{A}\\rceil$ moves.) This immediately implies $n \\geq 128$, since the bounding box needs to have area at least $4042>63.5^{2}$. On the other hand, if we start all the pins at the point $(3,18)$ then we can reach the following three points in 128 moves: $$ \\begin{aligned} & A=(0,0) \\\\ & B=(64,18) \\\\ & C=(3,64) \\end{aligned} $$ and indeed triangle $A B C$ has area exactly 2021. Remark. In fact, it can be shown that to obtain an area of $n \/ 2$, the bounding-box bound of $\\lceil 2 \\sqrt{n}\\rceil$ moves is best possible, i.e. there will in fact exist a triangle with area $n \/ 2$. However, since this was supposed to be a JMO4 problem, the committee made a choice to choose $n=4042$ so that contestants only needed to give a single concrete triangle rather than a general construction for all integers $n$."} +{"year":2021,"label":"5","problem":"A finite set $S$ of positive integers has the property that, for each $s \\in S$, and each positive integer divisor $d$ of $s$, there exists a unique element $t \\in S$ satisfying $\\operatorname{gcd}(s, t)=d$. (The elements $s$ and $t$ could be equal.) Given this information, find all possible values for the number of elements of $S$.","solution":" The answer is that $|S|$ must be a power of 2 (including 1 ), or $|S|=0$ (a trivial case we do not discuss further). Construction. For any nonnegative integer $k$, a construction for $|S|=2^{k}$ is given by $$ S=\\left\\{\\left(p_{1} \\text { or } q_{1}\\right) \\times\\left(p_{2} \\text { or } q_{2}\\right) \\times \\cdots \\times\\left(p_{k} \\text { or } q_{k}\\right)\\right\\} $$ for $2 k$ distinct primes $p_{1}, \\ldots, p_{k}, q_{1}, \\ldots, q_{k}$. \u3010 Converse. The main claim is as follows. Claim - In any valid set $S$, for any prime $p$ and $x \\in S, \\nu_{p}(x) \\leq 1$. - On the one hand, by taking $x$ in the statement, we see $\\frac{e}{e+1}$ of the elements of $S$ are divisible by $p$. - On the other hand, consider a $y \\in S$ such that $\\nu_{p}(y)=1$ which must exist (say if $\\operatorname{gcd}(x, y)=p)$. Taking $y$ in the statement, we see $\\frac{1}{2}$ of the elements of $S$ are divisible by $p$. So $e=1$, contradiction. Now since $|S|$ equals the number of divisors of any element of $S$, we are done."} +{"year":2021,"label":"6","problem":"Let $n \\geq 4$ be an integer. Find all positive real solutions to the following system of $2 n$ equations: $$ \\begin{array}{rlrl} a_{1} & =\\frac{1}{a_{2 n}}+\\frac{1}{a_{2}}, & a_{2} & =a_{1}+a_{3}, \\\\ a_{3} & =\\frac{1}{a_{2}}+\\frac{1}{a_{4}}, & a_{4} & =a_{3}+a_{5}, \\\\ a_{5} & =\\frac{1}{a_{4}}+\\frac{1}{a_{6}}, & a_{6} & =a_{5}+a_{7}, \\\\ & \\vdots & \\vdots \\\\ a_{2 n-1} & =\\frac{1}{a_{2 n-2}}+\\frac{1}{a_{2 n}}, & a_{2 n} & =a_{2 n-1}+a_{1} . \\end{array} $$","solution":" We will prove $a_{2 k}$ is a constant sequence, at which point the result is obvious. \\\u093e First approach (Andrew Gu). Apparently, with indices modulo 2n, we should have $$ a_{2 k}=\\frac{1}{a_{2 k-2}}+\\frac{2}{a_{2 k}}+\\frac{1}{a_{2 k+2}} $$ for every index $k$ (this eliminates all $a_{\\text {odd }}$ 's). Define $$ m=\\min _{k} a_{2 k} \\quad \\text { and } \\quad M=\\max _{k} a_{2 k} $$ Look at the indices $i$ and $j$ achieving $m$ and $M$ to respectively get $$ \\begin{aligned} & m=\\frac{2}{m}+\\frac{1}{a_{2 i-2}}+\\frac{1}{a_{2 i+2}} \\geq \\frac{2}{m}+\\frac{1}{M}+\\frac{1}{M}=\\frac{2}{m}+\\frac{2}{M} \\\\ & M=\\frac{2}{M}+\\frac{1}{a_{2 j-2}}+\\frac{1}{a_{2 j+2}} \\leq \\frac{2}{M}+\\frac{1}{m}+\\frac{1}{m}=\\frac{2}{m}+\\frac{2}{M} \\end{aligned} $$ Together this gives $m \\geq M$, so $m=M$. That means $a_{2 i}$ is constant as $i$ varies, solving the problem."} +{"year":2021,"label":"6","problem":"Let $n \\geq 4$ be an integer. Find all positive real solutions to the following system of $2 n$ equations: $$ \\begin{array}{rlrl} a_{1} & =\\frac{1}{a_{2 n}}+\\frac{1}{a_{2}}, & a_{2} & =a_{1}+a_{3}, \\\\ a_{3} & =\\frac{1}{a_{2}}+\\frac{1}{a_{4}}, & a_{4} & =a_{3}+a_{5}, \\\\ a_{5} & =\\frac{1}{a_{4}}+\\frac{1}{a_{6}}, & a_{6} & =a_{5}+a_{7}, \\\\ & \\vdots & \\vdots \\\\ a_{2 n-1} & =\\frac{1}{a_{2 n-2}}+\\frac{1}{a_{2 n}}, & a_{2 n} & =a_{2 n-1}+a_{1} . \\end{array} $$","solution":" We will prove $a_{2 k}$ is a constant sequence, at which point the result is obvious. \u092c Second approach (author's solution). As before, we have $$ a_{2 k}=\\frac{1}{a_{2 k-2}}+\\frac{2}{a_{2 k}}+\\frac{1}{a_{2 k+2}} $$ - Define $$ S=\\sum_{k} a_{2 k}, \\quad \\text { and } \\quad T=\\sum_{k} \\frac{1}{a_{2 k}} $$ Summing gives $S=4 T$. On the other hand, Cauchy-Schwarz says $S \\cdot T \\geq n^{2}$, so $T \\geq \\frac{1}{2} n$. - On the other hand, $$ 1=\\frac{1}{a_{2 k-2} a_{2 k}}+\\frac{2}{a_{2 k}^{2}}+\\frac{1}{a_{2 k} a_{2 k+2}} $$ Sum this modified statement to obtain $$ n=\\sum_{k}\\left(\\frac{1}{a_{2 k}}+\\frac{1}{a_{2 k+2}}\\right)^{2} \\stackrel{\\text { QM-AM }}{\\geq} \\frac{1}{n}\\left(\\sum_{k} \\frac{1}{a_{2 k}}+\\frac{1}{a_{2 k+2}}\\right)^{2}=\\frac{1}{n}(2 T)^{2} $$ So $T \\leq \\frac{1}{2} n$. - Since $T \\leq \\frac{1}{2} n$ and $T \\geq \\frac{1}{2} n$, we must have equality everywhere above. This means $a_{2 k}$ is a constant sequence. Remark. The problem is likely intractable over $\\mathbb{C}$, in the sense that one gets a high-degree polynomial which almost certainly has many complex roots. So it seems likely that most solutions must involve some sort of inequality, using the fact we are over $\\mathbb{R}_{>0}$ instead."} diff --git a/USAJMO/segmented/en-JMO-2022-notes.jsonl b/USAJMO/segmented/en-JMO-2022-notes.jsonl new file mode 100644 index 0000000000000000000000000000000000000000..ad75db293befc4c2bdb159b5c217bc345b68e799 --- /dev/null +++ b/USAJMO/segmented/en-JMO-2022-notes.jsonl @@ -0,0 +1,8 @@ +{"year":2022,"label":"1","problem":"For which positive integers $m$ does there exist an infinite sequence in $\\mathbb{Z} \/ m \\mathbb{Z}$ which is both an arithmetic progression and a geometric progression, but is nonconstant?","solution":" Answer: $m$ must not be squarefree. The problem is essentially asking when there exists a nonconstant arithmetic progression in $\\mathbb{Z} \/ m \\mathbb{Z}$ which is also a geometric progression. Now, - If $m$ is squarefree, then consider three $(s-d, d, s+d)$ in arithmetic progression. It's geometric if and only if $d^{2}=(s-d)(s+d)(\\bmod m)$, meaning $d^{2} \\equiv 0(\\bmod m)$. Then $d \\equiv 0(\\bmod m)$. So any arithmetic progression which is also geometric is constant in this case. - Conversely if $p^{2} \\mid m$ for some prime $p$, then any arithmetic progression with common difference $m \/ p$ is geometric by the same calculation."} +{"year":2022,"label":"2","problem":"Let $a$ and $b$ be positive integers. Every cell of an $(a+b+1) \\times(a+b+1)$ grid is colored either amber or bronze such that there are at least $a^{2}+a b-b$ amber cells and at least $b^{2}+a b-a$ bronze cells. Prove that it is possible to choose $a$ amber cells and $b$ bronze cells such that no two of the $a+b$ chosen cells lie in the same row or column.","solution":" Claim - There exists a transversal $T_{a}$ with at least $a$ amber cells. Analogously, there exists a transversal $T_{b}$ with at least $b$ bronze cells. $$ \\frac{a^{2}+a b-b}{a+b+1}=(a-1)+\\frac{1}{a+b+1}>a-1 $$ Now imagine we transform $T_{a}$ to $T_{b}$ in some number of steps, by repeatedly choosing cells $c$ and $c^{\\prime}$ and swapping them with the two other corners of the rectangle formed by their row\/column, as shown in the figure. ![](https:\/\/cdn.mathpix.com\/cropped\/2024_11_19_b7a51d82629ab4dffabag-04.jpg?height=241&width=818&top_left_y=1379&top_left_x=619) By \"discrete intermediate value theorem\", the number of amber cells will be either $a$ or $a+1$ at some point during this transformation. This completes the proof."} +{"year":2022,"label":"3","problem":"Let $b \\geq 2$ and $w \\geq 2$ be fixed integers, and $n=b+w$. Given are $2 b$ identical black rods and $2 w$ identical white rods, each of side length 1. We assemble a regular $2 n$-gon using these rods so that parallel sides are the same color. Then, a convex $2 b$-gon $B$ is formed by translating the black rods, and a convex $2 w$-gon $W$ is formed by translating the white rods. An example of one way of doing the assembly when $b=3$ and $w=2$ is shown below, as well as the resulting polygons $B$ and $W$. ![](https:\/\/cdn.mathpix.com\/cropped\/2024_11_19_b7a51d82629ab4dffabag-02.jpg?height=609&width=1017&top_left_y=1083&top_left_x=571) Prove that the difference of the areas of $B$ and $W$ depends only on the numbers $b$ and $w$, and not on how the $2 n$-gon was assembled.","solution":" We are going to prove that one may swap a black rod with an adjacent white rod (as well as the rods parallel to them) without affecting the difference in the areas of $B-W$. Let $\\vec{u}$ and $\\vec{v}$ denote the originally black and white vectors that were adjacent on the $2 n$-gon and are now going to be swapped. Let $\\vec{x}$ denote the sum of all the other black vectors between $\\vec{u}$ and $-\\vec{u}$, and define $\\vec{y}$ similarly. See the diagram below, where $B_{0}$ and $W_{0}$ are the polygons before the swap, and $B_{1}$ and $W_{1}$ are the resulting changed polygons. ![](https:\/\/cdn.mathpix.com\/cropped\/2024_11_19_b7a51d82629ab4dffabag-06.jpg?height=738&width=1234&top_left_y=222&top_left_x=414) Observe that the only change in $B$ and $W$ is in the parallelograms shown above in each diagram. Letting $\\wedge$ denote the wedge product, we need to show that $$ \\vec{u} \\wedge \\vec{x}-\\vec{v} \\wedge \\vec{y}=\\vec{v} \\wedge \\vec{x}-\\vec{u} \\wedge \\vec{y} $$ which can be rewritten as $$ (\\vec{u}-\\vec{v}) \\wedge(\\vec{x}+\\vec{y})=0 $$ In other words, it would suffice to show $\\vec{u}-\\vec{v}$ and $\\vec{x}+\\vec{y}$ are parallel. (Students not familiar with wedge products can replace every $\\wedge$ with the cross product $\\times$ instead.) Claim - Both $\\vec{u}-\\vec{v}$ and $\\vec{x}+\\vec{y}$ are perpendicular to vector $\\vec{u}+\\vec{v}$. For the other perpendicularity, note that $\\vec{u}+\\vec{v}+\\vec{x}+\\vec{y}$ traces out a diameter of the circumcircle of the original $2 n$-gon; call this diameter $A B$, so $$ A+\\vec{u}+\\vec{v}+\\vec{x}+\\vec{y}=B $$ Now point $A+\\vec{u}+\\vec{v}$ is a point on this semicircle, which means (by the inscribed angle theorem) the angle between $\\vec{u}+\\vec{v}$ and $\\vec{x}+\\vec{y}$ is $90^{\\circ}$."} +{"year":2022,"label":"4","problem":"Let $A B C D$ be a rhombus, and let $K$ and $L$ be points such that $K$ lies inside the rhombus, $L$ lies outside the rhombus, and $K A=K B=L C=L D$. Prove that there exist points $X$ and $Y$ on lines $A C$ and $B D$ such that $K X L Y$ is also a rhombus.","solution":" To start, notice that $\\triangle A K B \\cong \\triangle D L C$ by SSS. Then by the condition $K$ lies inside the rhombus while $L$ lies outside it, we find that the two congruent triangles are just translations of each other (i.e. they have the same orientation). \u300e First solution. Let $M$ be the midpoint of $\\overline{K L}$ and is $O$ the center of the rhombus. $$ \\text { Claim }-\\overline{M O} \\perp \\overline{A B} . $$ We choose $X$ and $Y$ to be the intersections of the perpendicular bisector of $\\overline{K L}$ with $\\overline{A C}$ and $\\overline{B D}$. ![](https:\/\/cdn.mathpix.com\/cropped\/2024_11_19_b7a51d82629ab4dffabag-07.jpg?height=598&width=803&top_left_y=1551&top_left_x=632) Claim - The midpoint of $\\overline{X Y}$ coincides with the midpoint of $\\overline{K L}$. $$ \\begin{aligned} & \\overline{X Y} \\perp \\overline{K L} \\| \\overline{B C} \\\\ & \\overline{M O} \\perp \\overline{A B} \\\\ & \\overline{B D} \\perp \\overline{A C} \\end{aligned} $$ it follows that $\\triangle M O Y$, which was determined by the three lines $\\overline{X Y}, \\overline{M O}, \\overline{B D}$, is similar to $\\triangle A B C$. In particular, it is isosceles with $M Y=M O$. Analogously, $M X=M O$. Remark. It is also possible to simply use coordinates to prove both claims."} +{"year":2022,"label":"4","problem":"Let $A B C D$ be a rhombus, and let $K$ and $L$ be points such that $K$ lies inside the rhombus, $L$ lies outside the rhombus, and $K A=K B=L C=L D$. Prove that there exist points $X$ and $Y$ on lines $A C$ and $B D$ such that $K X L Y$ is also a rhombus.","solution":" To start, notice that $\\triangle A K B \\cong \\triangle D L C$ by SSS. Then by the condition $K$ lies inside the rhombus while $L$ lies outside it, we find that the two congruent triangles are just translations of each other (i.e. they have the same orientation). \u3010 Second solution (author's solution). In this solution, we instead define $X$ and $Y$ as the intersections of the circles centered at $K$ and $L$ of equal radii $K A$, which will be denoted $\\omega_{K}$ and $\\omega_{L}$. It is clear that $K X L Y$ is a rhombus under this construction, so it suffices to show that $X$ and $Y$ lie on $A C$ and $B D$ (in some order). ![](https:\/\/cdn.mathpix.com\/cropped\/2024_11_19_b7a51d82629ab4dffabag-08.jpg?height=698&width=803&top_left_y=790&top_left_x=632) To see this, let $\\overline{A C}$ meet $\\omega_{K}$ again at $X^{\\prime}$. We have $$ \\measuredangle C X^{\\prime} D=\\measuredangle B X^{\\prime} C=\\measuredangle B X^{\\prime} A=\\frac{1}{2} \\mathrm{~m} \\overparen{A B}=\\mathrm{m} \\overparen{C D} $$ where the arcs are directed modulo $360^{\\circ}$; here $\\overparen{A B}$ is the arc of $\\omega_{K}$ cut out by $\\measuredangle A X B$, and $\\overparen{D C}$ is the analogous arc of $\\omega_{L}$. This implies $X^{\\prime}$ lies on $\\omega_{L}$ by the inscribed angle theorem. Hence $X=X^{\\prime}$, and it follows $X$ lies on $\\overline{A C}$. Analogously $Y$ lies on $B D$. Remark. The angle calculation above can also be replaced with a length calculation, as follows. Let $M$ and $N$ be the projections of $K$ and $L$ onto $\\overline{A C}$, respectively. Then $X^{\\prime}$ is the reflection of $A$ across $M$; analogously, the second intersection $X^{\\prime \\prime}$ with $\\overline{A C}$ should be the reflection of $C$ across $N$. So to get $X=X^{\\prime}=X^{\\prime \\prime}$, we would need to show $A C=2 M N$. However, note that $A K L D$ is a parallelogram. As $M N$ was the projection of $\\overline{K L}$ onto $\\overline{A C}$, its length should be the same as the projection of $\\overline{A D}$ onto $\\overline{A C}$, which is obviously $\\frac{1}{2} A C$ because the projection of $D$ onto $\\overline{A C}$ is exactly the midpoint of $\\overline{A C}$ (i.e. the center of the rhombus)."} +{"year":2022,"label":"5","problem":"Find all pairs of primes $(p, q)$ for which $p-q$ and $p q-q$ are both perfect squares.","solution":" The answer is $(3,2)$ only. This obviously works so we focus on showing it is the only one. \u3010 Approach using difference of squares (from author). Set $$ \\begin{aligned} a^{2} & =p-q \\\\ b^{2} & =p q-q . \\end{aligned} $$ Note that $01$, it follows $s_{n}$ is unbounded, contradicting $\\max \\left\\{\\left|a_{n}\\right|,\\left|b_{n}\\right|,\\left|c_{n}\\right|\\right\\} \\leq 2022$. $$ \\begin{aligned} a_{n+1} & =a_{n}^{2}+2 b_{n} c_{n} \\\\ c_{n+1} & =b_{n}^{2}+2 c_{n} a_{n} \\\\ b_{n+1} & =c_{n}^{2}+2 a_{n} b_{n} \\end{aligned} $$ which is OK because we are just rearranging the terms in each triple. Then if $\\omega$ is any complex number with $\\omega^{3}=1$, and we define $$ z_{n}:=a_{n}+b_{n} \\omega+c_{n} \\omega^{2}, $$ the recursion amounts to saying that $z_{n+1}=z_{n}^{2}$. This allows us to analyze $\\left|z_{n}\\right|$ in a similar way as above, as now $\\left|z_{n}\\right|=\\left|z_{0}\\right|^{2^{n}}$."} diff --git a/USAJMO/segmented/en-JMO-2023-notes.jsonl b/USAJMO/segmented/en-JMO-2023-notes.jsonl new file mode 100644 index 0000000000000000000000000000000000000000..ce9c8bd67ed7d4cd6910c2485863657ff998dddb --- /dev/null +++ b/USAJMO/segmented/en-JMO-2023-notes.jsonl @@ -0,0 +1,13 @@ +{"year":2023,"label":"1","problem":"Find all triples of positive integers $(x, y, z)$ satisfying $$ 2(x+y+z+2 x y z)^{2}=(2 x y+2 y z+2 z x+1)^{2}+2023 $$","solution":" Answer: $(3,3,2)$ and permutations. Claim - The identity $$ 2(x+y+z+2 x y z)^{2}-(2 x y+2 y z+2 z x+1)^{2}=\\left(2 x^{2}-1\\right)\\left(2 y^{2}-1\\right)\\left(2 z^{2}-1\\right) $$ is true. $$ \\operatorname{Norm}(a+b \\sqrt{2})=a^{2}-2 b^{2} $$ which is multiplicative, meaning $$ \\operatorname{Norm}(u \\cdot v)=\\operatorname{Norm}(u) \\cdot \\operatorname{Norm}(v) $$ This means that for any rational numbers $x, y, z$, we should have $$ \\begin{aligned} & \\operatorname{Norm}((1+\\sqrt{2} x)(1+\\sqrt{2} y)(1+\\sqrt{2} z)) \\\\ = & \\operatorname{Norm}(1+\\sqrt{2} x) \\cdot \\operatorname{Norm}(1+\\sqrt{2} y) \\cdot \\operatorname{Norm}(1+\\sqrt{2} z) \\end{aligned} $$ But $(1+\\sqrt{2} x)(1+\\sqrt{2} y)(1+\\sqrt{2} z)=(2 x y+2 y z+2 z x+1)+(x+y+z+2 x y z) \\sqrt{2}$ so the above equation is the negative of the desired identity. We are thus reduced to find positive integers $x, y, z$ satisfying $$ \\left(2 x^{2}-1\\right)\\left(2 y^{2}-1\\right)\\left(2 z^{2}-1\\right)=2023=7 \\cdot 17^{2} $$ Each of the factors is a positive integer greater than 1 . The only divisors of 2023 of the form $2 t^{2}-1$ are $1,7,17$. This gives the answers claimed."} +{"year":2023,"label":"2","problem":"In an acute triangle $A B C$, let $M$ be the midpoint of $\\overline{B C}$. Let $P$ be the foot of the perpendicular from $C$ to $A M$. Suppose that the circumcircle of triangle $A B P$ intersects line $B C$ at two distinct points $B$ and $Q$. Let $N$ be the midpoint of $\\overline{A Q}$. Prove that $N B=N C$.","solution":" ![](https:\/\/cdn.mathpix.com\/cropped\/2024_11_19_9c5e556ae0970c152728g-04.jpg?height=892&width=1003&top_left_y=913&top_left_x=532) \u3010 Most common synthetic approach. The solution hinges on the following claim: Claim - $Q$ coincides with the reflection of $D$ across $M$. $$ M B \\cdot M Q=M A \\cdot M P=M C \\cdot M D . $$ Since $M B=M C$, the claim follows. It follows that $\\overline{M N} \\| \\overline{A D}$, as $M$ and $N$ are respectively the midpoints of $\\overline{A Q}$ and $\\overline{D Q}$. Thus $\\overline{M N} \\perp \\overline{B C}$, and so $N$ lies on the perpendicular bisector of $\\overline{B C}$, as needed. Remark (David Lin). One can prove the main claim without power of a point as well, as follows: Let $R$ be the foot from $B$ to $\\overline{A M}$, so $B R C P$ is a parallelogram. Note that $A B D R$ is cyclic, and hence $$ \\measuredangle D R M=\\measuredangle D B A=Q B A=\\measuredangle Q P A=\\measuredangle Q P M $$ Thus, $\\overline{D R} \\| \\overline{P Q}$, so $D R Q P$ is also a parallelogram."} +{"year":2023,"label":"2","problem":"In an acute triangle $A B C$, let $M$ be the midpoint of $\\overline{B C}$. Let $P$ be the foot of the perpendicular from $C$ to $A M$. Suppose that the circumcircle of triangle $A B P$ intersects line $B C$ at two distinct points $B$ and $Q$. Let $N$ be the midpoint of $\\overline{A Q}$. Prove that $N B=N C$.","solution":" ![](https:\/\/cdn.mathpix.com\/cropped\/2024_11_19_9c5e556ae0970c152728g-04.jpg?height=892&width=1003&top_left_y=913&top_left_x=532) ## \\ Synthetic approach with no additional points at all. Claim - $\\triangle B P C \\sim \\triangle A N M$ (oppositely oriented). $$ \\frac{B M}{B P}=\\frac{A M}{A Q} \\Longrightarrow \\frac{2 B M}{B P}=\\frac{A M}{A Q \/ 2} \\Longrightarrow \\frac{B C}{B P}=\\frac{A M}{A N} $$ implying the similarity (since $\\measuredangle M A Q=\\measuredangle B P M$ ). This similarity gives us the equality of directed angles $$ \\measuredangle(B C, M N)=-\\measuredangle(P C, A M)=90^{\\circ} $$ as desired."} +{"year":2023,"label":"2","problem":"In an acute triangle $A B C$, let $M$ be the midpoint of $\\overline{B C}$. Let $P$ be the foot of the perpendicular from $C$ to $A M$. Suppose that the circumcircle of triangle $A B P$ intersects line $B C$ at two distinct points $B$ and $Q$. Let $N$ be the midpoint of $\\overline{A Q}$. Prove that $N B=N C$.","solution":" ![](https:\/\/cdn.mathpix.com\/cropped\/2024_11_19_9c5e556ae0970c152728g-04.jpg?height=892&width=1003&top_left_y=913&top_left_x=532) \\ Synthetic approach using only the point $R$. Again let $R$ be the foot from $B$ to $\\overline{A M}$, so $B R C P$ is a parallelogram. Claim - $A R Q C$ is cyclic; equivalently, $\\triangle M A Q \\sim \\triangle M C R$. Note that in $\\triangle M C R$, the $M$-median is parallel to $\\overline{C P}$ and hence perpendicular to $\\overline{R M}$. The same should be true in $\\triangle M A Q$ by the similarity, so $\\overline{M N} \\perp \\overline{M Q}$ as needed."} +{"year":2023,"label":"2","problem":"In an acute triangle $A B C$, let $M$ be the midpoint of $\\overline{B C}$. Let $P$ be the foot of the perpendicular from $C$ to $A M$. Suppose that the circumcircle of triangle $A B P$ intersects line $B C$ at two distinct points $B$ and $Q$. Let $N$ be the midpoint of $\\overline{A Q}$. Prove that $N B=N C$.","solution":" ![](https:\/\/cdn.mathpix.com\/cropped\/2024_11_19_9c5e556ae0970c152728g-04.jpg?height=892&width=1003&top_left_y=913&top_left_x=532) \u3010 Cartesian coordinates approach with power of a point. Suppose we set $B=(-1,0)$, $M=(0,0), C=(1,0)$, and $A=(a, b)$. One may compute: $$ \\begin{aligned} \\overleftrightarrow{A M}: 0 & =b x-a y \\Longleftrightarrow y=\\frac{b}{a} x \\\\ \\overleftrightarrow{C P}: 0 & =a(x-1)+b y \\Longleftrightarrow y=-\\frac{a}{b}(x-1)=-\\frac{a}{b} x+\\frac{a}{b} \\\\ P & =\\left(\\frac{a^{2}}{a^{2}+b^{2}}, \\frac{a b}{a^{2}+b^{2}}\\right) \\end{aligned} $$ Now note that $$ A M=\\sqrt{a^{2}+b^{2}}, \\quad P M=\\frac{a}{\\sqrt{a^{2}+b^{2}}} $$ together with power of a point $$ A M \\cdot P M=B M \\cdot Q M $$ to immediately deduce that $Q=(a, 0)$. Hence $N=(0, b \/ 2)$ and we're done."} +{"year":2023,"label":"2","problem":"In an acute triangle $A B C$, let $M$ be the midpoint of $\\overline{B C}$. Let $P$ be the foot of the perpendicular from $C$ to $A M$. Suppose that the circumcircle of triangle $A B P$ intersects line $B C$ at two distinct points $B$ and $Q$. Let $N$ be the midpoint of $\\overline{A Q}$. Prove that $N B=N C$.","solution":" ![](https:\/\/cdn.mathpix.com\/cropped\/2024_11_19_9c5e556ae0970c152728g-04.jpg?height=892&width=1003&top_left_y=913&top_left_x=532) \u3010 Cartesian coordinates approach without power of a point (outline). After computing $A$ and $P$ as above, one could also directly calculate $$ \\begin{aligned} & \\text { Perpendicular bisector of } \\overline{A B}: y=-\\frac{a+1}{b} x+\\frac{a^{2}+b^{2}-1}{2 b} \\\\ & \\text { Perpendicular bisector of } \\overline{P B}: y=-\\left(\\frac{2 a}{b}+\\frac{b}{a}\\right) x-\\frac{b}{2 a} \\\\ & \\text { Perpendicular bisector of } \\overline{P A}: y=-\\frac{a}{b} x+\\frac{a+a^{2}+b^{2}}{2 b} \\\\ & \\text { Circumcenter of } \\triangle P A B=\\left(-\\frac{a+1}{2}, \\frac{2 a^{2}+2 a+b^{2}}{2 b}\\right) \\end{aligned} $$ This is enough to extract the coordinates of $Q=(\\bullet, 0)$, because $B=(-1,0)$ is given, and the $x$-coordinate of the circumcenter should be the average of the $x$-coordinates of $B$ and $Q$. In other words, $Q=(-a, 0)$. Hence, $N=\\left(0, \\frac{b}{2}\\right)$, as needed."} +{"year":2023,"label":"2","problem":"In an acute triangle $A B C$, let $M$ be the midpoint of $\\overline{B C}$. Let $P$ be the foot of the perpendicular from $C$ to $A M$. Suppose that the circumcircle of triangle $A B P$ intersects line $B C$ at two distinct points $B$ and $Q$. Let $N$ be the midpoint of $\\overline{A Q}$. Prove that $N B=N C$.","solution":" ![](https:\/\/cdn.mathpix.com\/cropped\/2024_11_19_9c5e556ae0970c152728g-04.jpg?height=892&width=1003&top_left_y=913&top_left_x=532) \\I III-advised barycentric approach (outline). Use reference triangle $A B C$. The $A$ median is parametrized by $(t: 1: 1)$ for $t \\in \\mathbb{R}$. So because of $\\overline{C P} \\perp \\overline{A M}$, we are looking for $t$ such that $$ \\left(\\frac{t \\vec{A}+\\vec{B}+\\vec{C}}{t+2}-\\vec{C}\\right) \\perp\\left(A-\\frac{\\vec{B}+\\vec{C}}{2}\\right) $$ This is equivalent to $$ (t \\vec{A}+\\vec{B}-(t+1) \\vec{C}) \\perp(2 \\vec{A}-\\vec{B}-\\vec{C}) $$ By the perpendicularity formula for barycentric coordinates (EGMO 7.16), this is equivalent to $$ \\begin{aligned} 0 & =a^{2} t-b^{2} \\cdot(3 t+2)+c^{2} \\cdot(2-t) \\\\ & =\\left(a^{2}-3 b^{2}-c^{2}\\right) t-2\\left(b^{2}-c^{2}\\right) \\\\ \\Longrightarrow t & =\\frac{2\\left(b^{2}-c^{2}\\right)}{a^{2}-3 b^{2}-c^{2}} \\end{aligned} $$ In other words, $$ P=\\left(2\\left(b^{2}-c^{2}\\right): a^{2}-3 b^{2}-c^{2}: a^{2}-3 b^{2}-c^{2}\\right) . $$ A long calculation gives $a^{2} y_{P} z_{P}+b^{2} z_{P} x_{P}+c^{2} x_{P} y_{P}=\\left(a^{2}-3 b^{2}-c^{2}\\right)\\left(a^{2}-b^{2}+c^{2}\\right)\\left(a^{2}-\\right.$ $\\left.2 b^{2}-2 c^{2}\\right)$. Together with $x_{P}+y_{P}+z_{P}=2 a^{2}-4 b^{2}-4 c^{2}$, this makes the equation of $(A B P)$ as $$ 0=-a^{2} y z-b^{2} z x-c^{2} x y+\\frac{a^{2}-b^{2}+c^{2}}{2} z(x+y+z) $$ To solve for $Q$, set $x=0$ to get to get $$ a^{2} y z=\\frac{a^{2}-b^{2}+c^{2}}{2} z(y+z) \\Longrightarrow \\frac{y}{z}=\\frac{a^{2}-b^{2}+c^{2}}{a^{2}+b^{2}-c^{2}} $$ In other words, $$ Q=\\left(0: a^{2}-b^{2}+c^{2}: a^{2}+b^{2}-c^{2}\\right) $$ Taking the average with $A=(1,0,0)$ then gives $$ N=\\left(2 a^{2}: a^{2}-b^{2}+c^{2}: a^{2}+b^{2}-c^{2}\\right) . $$ The equation for the perpendicular bisector of $\\overline{B C}$ is given by (see EGMO 7.19) $$ 0=a^{2}(z-y)+x\\left(c^{2}-b^{2}\\right) $$ which contains $N$, as needed. $$ p=\\frac{(a-m) \\bar{c}+(\\bar{a}-\\bar{m}) c+\\bar{a} m-a \\bar{m}}{2(\\bar{a}-\\bar{m})}=\\frac{a^{2} b-a^{2} c-a b^{2}-2 a b c-a c^{2}+b^{2} c+3 b c^{2}}{4 b c-2 a(b+c)} $$ Meanwhile, an extremely ugly calculation will eventually yield $$ q=\\frac{\\frac{b c}{a}+b+c-a}{2} $$ SO $$ n=\\frac{a+q}{2}=\\frac{a+b+c+\\frac{b c}{a}}{4}=\\frac{(a+b)(a+c)}{2 a} $$ There are a few ways to then verify $N B=N C$. The simplest seems to be to verify that $$ \\frac{n-\\frac{b+c}{2}}{b-c}=\\frac{a-b-c+\\frac{b c}{a}}{4(b-c)}=\\frac{(a-b)(a-c)}{2 a(b-c)} $$ is pure imaginary, which is clear."} +{"year":2023,"label":"3","problem":"Consider an $n$-by- $n$ board of unit squares for some odd positive integer $n$. We say that a collection $C$ of identical dominoes is a maximal grid-aligned configuration on the board if $C$ consists of $\\left(n^{2}-1\\right) \/ 2$ dominoes where each domino covers exactly two neighboring squares and the dominoes don't overlap: $C$ then covers all but one square on the board. We are allowed to slide (but not rotate) a domino on the board to cover the uncovered square, resulting in a new maximal gridaligned configuration with another square uncovered. Let $k(C)$ be the number of distinct maximal grid-aligned configurations obtainable from $C$ by repeatedly sliding dominoes. Find the maximum possible value of $k(C)$ as a function of $n$.","solution":" The answer is that $$ k(C) \\leq\\left(\\frac{n+1}{2}\\right)^{2} $$ Remark (Comparison with USAMO version). In the USAMO version of the problem, students instead are asked to find all possible values of $k(C)$. The answer is $k(C) \\in$ $\\left\\{1,2, \\ldots,\\left(\\frac{n-1}{2}\\right)^{2}\\right\\} \\cup\\left\\{\\left(\\frac{n+1}{2}\\right)^{2}\\right\\}$. Index the squares by coordinates $(x, y) \\in\\{1,2, \\ldots, n\\}^{2}$. We say a square is special if it is empty or it has the same parity in both coordinates as the empty square. Construct a directed graph $G=G(C)$ whose vertices are special squares as follows: for each domino on a special square $s$, we draw a directed edge from $s$ to the special square that domino points to, if any. (If the special square has both odd coordinates, all special squares have an outgoing edge except the empty cell. In the even-even case, some arrows may point \"off the board\" and not be drawn.) ![](https:\/\/cdn.mathpix.com\/cropped\/2024_11_19_9c5e556ae0970c152728g-08.jpg?height=612&width=615&top_left_y=1816&top_left_x=726) Now focus specifically on the weakly connected component $T$ of $G$ (i.e. the connected component of the undirected version of $G$ ) containing the empty square. Claim - The graph $T$ has no cycles, even undirected. Hence, the undirected version of $T$ is tree. Notice that all the arrows along $T$ point towards the empty cell, and moving a domino corresponds to flipping an arrow. Therefore: Claim - $k(C)$ is exactly the number of vertices of $T$. This implies that $k(C) \\leq\\left(\\frac{n+1}{2}\\right)^{2}$, the total number of vertices of $G$ (this could only occur if the special squares are odd-odd, not even-even). Equality is achieved as long as $T$ is a spanning tree; one example of a way to achieve this is using the snake configuration below. ![](https:\/\/cdn.mathpix.com\/cropped\/2024_11_19_9c5e556ae0970c152728g-09.jpg?height=464&width=466&top_left_y=1270&top_left_x=795)"} +{"year":2023,"label":"4","problem":"Two players, Blake and Ruby, play the following game on an infinite grid of unit squares, all initially colored white. The players take turns starting with Blake. On Blake's turn, Blake selects one white unit square and colors it blue. On Ruby's turn, Ruby selects two white unit squares and colors them red. The players alternate until Blake decides to end the game. At this point, Blake gets a score, given by the number of unit squares in the largest (in terms of area) simple polygon containing only blue unit squares. What is the largest score Blake can guarantee?","solution":" The answer is 4 squares. \u092c Algorithm for Blake to obtain at least 4 squares. We simply let Blake start with any cell blue, then always draw adjacent to a previously drawn blue cell until this is no longer possible. Note that for $n \\leq 3$, any connected region of $n$ blue cells has more than $2 n$ liberties (non-blue cells adjacent to a blue cell); up to translation, rotation, and reflection, all the cases are shown in the figure below with liberties being denoted by circles. ![](https:\/\/cdn.mathpix.com\/cropped\/2024_11_19_9c5e556ae0970c152728g-10.jpg?height=655&width=818&top_left_y=1481&top_left_x=622) So as long as $n \\leq 3$, it's impossible that Ruby has blocked every liberty, since Ruby has colored exactly $2 n$ cells red. Therefore, this algorithm could only terminate once $n \\geq 4$. \\ Algorithm for Ruby to prevent more than 4 squares. Divide the entire grid into $2 \\times 2$ squares, which we call windows. Any time Blake makes a move in a cell $c$, let Ruby mark any orthogonal neighbors of $c$ in its window; then place any leftover red cells arbitrarily. Claim - It's impossible for any window to contain two orthogonally adjacent blue cells. We show this gives the upper bound of 4 squares. Consider a blue cell $w$, and assume WLOG it is in the southeast corner of a window. Label squares $x, y, z$ as shown below. ![](https:\/\/cdn.mathpix.com\/cropped\/2024_11_19_9c5e556ae0970c152728g-11.jpg?height=506&width=510&top_left_y=661&top_left_x=773) Note that by construction, the blue polygon cannot leave the square $\\{w, x, y, z\\}$, since whenever one of these four cells is blue, its neighbours outside that square are guaranteed to be red. This implies the bound. Remark (For Tetris fans). Here is a comedic alternative finish after proving the claim. Consider the possible tetrominoes (using the notation of https:\/\/en.wikipedia.org\/wiki\/ Tetromino\\#One-sided_tetrominoes). We claim that only the square (0) is obtainable; as - T, J\/L, and I all have three cells in a row, so they can't occur; - S and Z can't occur either; if the bottom row of an S crossed a window boundary, then the top row doesn't for example. Moreover, the only way a blue 0 could be obtained is if each of it cells is in a different window. In that case, no additional blue cells can be added: it's fully surrounded by red. Finally, for any $k$-omino with $k>4$, one can find a tetromino as a subset. (Proof: take the orthogonal adjacency graph of the $k$-omino, choose a spanning tree, and delete leaves from the tree until there are only four vertices left.) As written, this strategy does not work. The reason is that one can end up in the following situation (imagine the blue square in the center is played first; moves for Ruby are drawn as red X's): ![](https:\/\/cdn.mathpix.com\/cropped\/2024_11_19_9c5e556ae0970c152728g-12.jpg?height=410&width=429&top_left_y=246&top_left_x=822) In order to prevent Blake from winning, Ruby would need to begin playing moves not adjacent to Blake's most recent move. It is even more difficult to come up with a solution involving playing on just \"some\" two neighbors of recently added blue squares without the \"prefer north and west\" idea."} +{"year":2023,"label":"5","problem":"Positive integers $a$ and $N$ are fixed, and $N$ positive integers are written on a blackboard. Alice and Bob play the following game. On Alice's turn, she must replace some integer $n$ on the board with $n+a$, and on Bob's turn he must replace some even integer $n$ on the board with $n \/ 2$. Alice goes first and they alternate turns. If on his turn Bob has no valid moves, the game ends. After analyzing the $N$ integers on the board, Bob realizes that, regardless of what moves Alice makes, he will be able to force the game to end eventually. Show that, in fact, for this value of $a$ and these $N$ integers on the board, the game is guaranteed to end regardless of Alice's or Bob's moves.","solution":" For $N=1$, there is nothing to prove. We address $N \\geq 2$ only henceforth. Let $S$ denote the numbers on the board. Claim - When $N \\geq 2$, if $\\nu_{2}(x)<\\nu_{2}(a)$ for all $x \\in S$, the game must terminate no matter what either player does. Hence, in fact the game will always terminate in exactly $\\sum_{x \\in S} \\nu_{2}(x)$ moves in this case, regardless of what either player does. Claim - When $N \\geq 2$, if there exists a number $x$ on the board such that $\\nu_{2}(x) \\geq$ $\\nu_{2}(a)$, then Alice can cause the game to go on forever. - Operate on the first entry if $\\nu_{2}(x)=\\nu_{2}(a)$ (the new entry thus has $\\nu_{2}(x+a)>\\nu_{2}(a)$ ); - Operate on any other entry besides the first one, otherwise. A double induction then shows that - Just before each of Bob's turns, $\\nu_{2}(x)>\\nu_{2}(a)$ always holds; and - After each of Bob's turns, $\\nu_{2}(x) \\geq \\nu_{2}(a)$ always holds. In particular Bob will never run out of legal moves, since halving $x$ is always legal."} +{"year":2023,"label":"6","problem":"Isosceles triangle $A B C$, with $A B=A C$, is inscribed in circle $\\omega$. Let $D$ be an arbitrary point inside $B C$ such that $B D \\neq D C$. Ray $A D$ intersects $\\omega$ again at $E$ (other than $A$ ). Point $F$ (other than $E$ ) is chosen on $\\omega$ such that $\\angle D F E=90^{\\circ}$. Line $F E$ intersects rays $A B$ and $A C$ at points $X$ and $Y$, respectively. Prove that $\\angle X D E=\\angle E D Y$.","solution":" \u3010 Angle chasing solution. Note that $(B D A)$ and $(C D A)$ are congruent, since $B A=C A$ and $\\angle B D A+\\angle C D A=180^{\\circ}$. So these two circles are reflections around line $E D$. Moreover, $(D E F)$ is obviously also symmetric around line $E D$. ![](https:\/\/cdn.mathpix.com\/cropped\/2024_11_19_9c5e556ae0970c152728g-14.jpg?height=812&width=1009&top_left_y=1182&top_left_x=518) Hence, the radical axis of $(B D A)$ and $(D E F)$, and the radical axis of $(C D A)$ and $(D E F)$, should be symmetric about line $D E$. But these radical axii are exactly lines $X D$ and $Y D$, so we're done. Remark (Motivation). The main idea is that you can replace $D X$ and $D Y$ with the radical axii, letting $X^{\\prime}$ and $Y^{\\prime}$ be the second intersections of the blue circles. Then for the problem to be true, you'd need $X^{\\prime}$ and $Y^{\\prime}$ to be reflections. That's equivalent to $(B D A)$ and $(C D A)$ being congruent; you check it and it's indeed true."} +{"year":2023,"label":"6","problem":"Isosceles triangle $A B C$, with $A B=A C$, is inscribed in circle $\\omega$. Let $D$ be an arbitrary point inside $B C$ such that $B D \\neq D C$. Ray $A D$ intersects $\\omega$ again at $E$ (other than $A$ ). Point $F$ (other than $E$ ) is chosen on $\\omega$ such that $\\angle D F E=90^{\\circ}$. Line $F E$ intersects rays $A B$ and $A C$ at points $X$ and $Y$, respectively. Prove that $\\angle X D E=\\angle E D Y$.","solution":" \u3010 Harmonic solution (mine). Let $T$ be the point on line $\\overline{X F E Y}$ such that $\\angle E D T=90^{\\circ}$, and let $\\overline{A T}$ meet $\\omega$ again at $K$. Then $$ T D^{2}=T F \\cdot T E=T K \\cdot T A \\Longrightarrow \\angle D K T=90^{\\circ} $$ so line $D K$ passes through the antipode $M$ of $A$. ![](https:\/\/cdn.mathpix.com\/cropped\/2024_11_19_9c5e556ae0970c152728g-15.jpg?height=940&width=1394&top_left_y=315&top_left_x=334) Thus, $$ -1=(A M ; C B)_{\\omega} \\stackrel{D}{=}(E K ; B C)_{\\omega} \\stackrel{A}{=}(T E ; X Y) $$ and since $\\angle E D T=90^{\\circ}$ we're done. Remark (Motivation). The idea is to kill the points $X$ and $Y$ by reinterpreting the desired condition as $(T D ; X Y)=-1$ and then projecting through $A$ onto $\\omega$. This eliminates points $X$ and $Y$ altogether and reduces the problem to showing that $\\overline{T A}$ passes through the harmonic conjugate of $E$ with respect to $B C$ on $\\omega$. The labels on the diagram are slightly misleading in that $\\triangle E B C$ should probably be thought of as the \"reference\" triangle."} +{"year":2023,"label":"6","problem":"Isosceles triangle $A B C$, with $A B=A C$, is inscribed in circle $\\omega$. Let $D$ be an arbitrary point inside $B C$ such that $B D \\neq D C$. Ray $A D$ intersects $\\omega$ again at $E$ (other than $A$ ). Point $F$ (other than $E$ ) is chosen on $\\omega$ such that $\\angle D F E=90^{\\circ}$. Line $F E$ intersects rays $A B$ and $A C$ at points $X$ and $Y$, respectively. Prove that $\\angle X D E=\\angle E D Y$.","solution":" \u3010 Pascal solution (Zuming Feng). Extend ray $F D$ to the antipode $T$ of $E$ on $\\omega$. Then, - By Pascal's theorem on $E F T A B C$, the points $X, D$, and $P:=\\overline{E C} \\cap \\overline{A T}$ are collinear. - Similarly by Pascal's theorem on $E F T A C B$, the points the points $Y, D$, and $Q:=\\overline{E B} \\cap \\overline{A T}$ are collinear. ![](https:\/\/cdn.mathpix.com\/cropped\/2024_11_19_9c5e556ae0970c152728g-16.jpg?height=740&width=1406&top_left_y=244&top_left_x=328) Now it suffices to prove $\\overline{E D}$ bisects $\\angle Q D P$. However, $\\overline{E D}$ is the angle bisector of $\\angle Q E P=\\angle B E C$, but also $\\overline{E A} \\perp \\overline{Q P}$. Thus triangle $Q E P$ is isosceles with $Q E=P E$, and $\\overline{E A}$ cuts it in half. Since $D$ is on $\\overline{E A}$, the result follows now."} diff --git a/USAJMO/segmented/en-JMO-2024-notes.jsonl b/USAJMO/segmented/en-JMO-2024-notes.jsonl new file mode 100644 index 0000000000000000000000000000000000000000..f5d6f3d224c2d961f7b2abb0e43c0682bdfbbedc --- /dev/null +++ b/USAJMO/segmented/en-JMO-2024-notes.jsonl @@ -0,0 +1,11 @@ +{"year":2024,"label":"1","problem":"Let $A B C D$ be a cyclic quadrilateral with $A B=7$ and $C D=8$. Points $P$ and $Q$ are selected on line segment $A B$ so that $A P=B Q=3$. Points $R$ and $S$ are selected on line segment $C D$ so that $C R=D S=2$. Prove that $P Q R S$ is a cyclic quadrilateral.","solution":" \\l The one-liner. The four points $P, Q, R, S$ have equal power -12 with respect to $(A B C D)$. So in fact they're on a circle concentric with $(A B C D)$."} +{"year":2024,"label":"1","problem":"Let $A B C D$ be a cyclic quadrilateral with $A B=7$ and $C D=8$. Points $P$ and $Q$ are selected on line segment $A B$ so that $A P=B Q=3$. Points $R$ and $S$ are selected on line segment $C D$ so that $C R=D S=2$. Prove that $P Q R S$ is a cyclic quadrilateral.","solution":" \\ The external power solution. We distinguish between two cases. Case where $A B$ and $C D$ are not parallel. We let lines $A B$ and $C D$ meet at $T$. Without loss of generality, $A$ lies between $B$ and $T$ and $D$ lies between $C$ and $T$. Let $x=T A$ and $y=T D$, as shown below. ![](https:\/\/cdn.mathpix.com\/cropped\/2024_11_19_805564b9ec1a761a333dg-03.jpg?height=541&width=812&top_left_y=1346&top_left_x=633) By power of a point, $$ \\begin{aligned} A B C D \\text { cyclic } \\Longleftrightarrow x(x+7) & =y(y+8) \\\\ P Q R S \\text { cyclic } \\Longleftrightarrow(x+3)(x+4) & =(y+2)(y+6) . \\end{aligned} $$ However, the latter equation is just the former with 12 added to both sides. (That is, $(x+3)(x+4)=x(x+7)+12$ while $(y+2)(y+6)=y(y+8)+12$.$) So the conclusion is$ immediate. Case where $A B$ and $C D$ are parallel. In that case $A B C D$ is an isosceles trapezoid. Then the entire picture is symmetric around the common perpendicular bisector of the lines $A B$ and $C D$. Now $P Q R S$ is also an isosceles trapezoid, so it's cyclic too. ![](https:\/\/cdn.mathpix.com\/cropped\/2024_11_19_805564b9ec1a761a333dg-04.jpg?height=420&width=420&top_left_y=244&top_left_x=818)"} +{"year":2024,"label":"2","problem":"Let $m$ and $n$ be positive integers. Let $S$ be the set of lattice points $(x, y)$ with $1 \\leq x \\leq 2 m$ and $1 \\leq y \\leq 2 n$. A configuration of $m n$ rectangles is called happy if each point of $S$ is the vertex of exactly one rectangle. Prove that the number of happy configurations is odd.","solution":" I Original proposer's solution. To this end, let's denote by $f(2 m, 2 n)$ the number of happy configurations for a $2 m \\times 2 n$ grid of lattice points (not necessarily equally spaced - this doesn't change the count). We already have the following easy case. Claim - We have $f(2,2 n)=(2 n-1)!!=(2 n-1) \\cdot(2 n-3) \\cdots \\cdots 3 \\cdot 1$. We will prove that: Claim - Assume $m, n \\geq 1$. When $f(2 m, 2 n) \\equiv f(2 m-2,2 n)(\\bmod 2)$. Now configurations fixed by $\\tau$ can be described readily: this occurs if and only if the last two columns are self-contained, meaning every rectangle with a vertex in these columns is completely contained in these two columns. ![](https:\/\/cdn.mathpix.com\/cropped\/2024_11_19_805564b9ec1a761a333dg-06.jpg?height=809&width=847&top_left_y=241&top_left_x=596) Hence it follows that $$ f(2 m, 2 n)=2(\\text { number of pairs })+f(2 m-2,2 n) \\cdot f(2,2 n) $$ Taking modulo 2 gives the result. By the same token $f(2 m, 2 n) \\equiv f(2 m, 2 n-2)(\\bmod 2)$. So all $f$-values have the same parity, and from $f(2,2)=1$ we're done."} +{"year":2024,"label":"2","problem":"Let $m$ and $n$ be positive integers. Let $S$ be the set of lattice points $(x, y)$ with $1 \\leq x \\leq 2 m$ and $1 \\leq y \\leq 2 n$. A configuration of $m n$ rectangles is called happy if each point of $S$ is the vertex of exactly one rectangle. Prove that the number of happy configurations is odd.","solution":" \\l Evan's permutation-based solution. Retain the notation $f(2 m, 2 n)$ from before. Given a happy configuration, consider all the rectangles whose left edge is in the first column. Highlight every column containing the right edge of such a rectangle. For example, in the figure below, there are two highlighted columns. (The rectangles are drawn crooked so one can tell them apart.) ![](https:\/\/cdn.mathpix.com\/cropped\/2024_11_19_805564b9ec1a761a333dg-06.jpg?height=515&width=832&top_left_y=1861&top_left_x=612) We organize happy configurations based on the set of highlighted columns. Specifically, define the relation $\\sim$ on configurations by saying that $\\mathcal{C} \\sim \\mathcal{C}^{\\prime}$ if they differ by any permutation of the highlighted columns. This is an equivalence relation. And in general, if there are $k$ highlighted columns, its equivalence class under $\\sim$ has $k$ ! elements. Then Claim - $f(2 m, 2 n)$ has the same parity as the number of happy configurations with exactly one highlighted column. There are $2 m-1$ ways to pick a single highlighted column, and then $f(2,2 n)=(2 n-1)!!$ ways to use the left column and highlighted column. So the count in the claim is exactly given by $$ (2 m-1) \\cdot(2 n-1)!!f(2 m-2,2 n) $$ This implies $f(2 m, 2 n) \\equiv f(2 m-2,2 n)(\\bmod 2)$ and proceeding by induction as before solves the problem."} +{"year":2024,"label":"3","problem":"A sequence $a_{1}, a_{2}, \\ldots$ of positive integers is defined recursively by $a_{1}=2$ and $$ a_{n+1}=a_{n}^{n+1}-1 \\quad \\text { for } n \\geq 1 $$ Prove that for every odd prime $p$ and integer $k$, some term of the sequence is divisible by $p^{k}$.","solution":" We start with the following. Claim - Assume $n$ is a positive integer divisible by $p-1$. Then either $a_{n-1} \\equiv 0$ $(\\bmod p)$ or $a_{n} \\equiv 0(\\bmod p)$. $$ a_{n}=a_{n-1}^{n}-1 \\equiv x^{p-1}-1 \\equiv 0 \\quad(\\bmod p) $$ where $x:=a_{n-1}^{\\frac{n}{p-1}}$ is an integer not divisible by $p$. Claim - If $n \\geq 2$ is even, then $$ a_{n}^{n+1} \\mid a_{n+2} $$ By considering multiples $n$ of $p-1$ which are larger than $k$, we see that if $a_{n} \\equiv 0$ $(\\bmod p)$ ever happens, we are done by combining the two previous claims. So the difficult case of the problem is the bad situation where $a_{n-1} \\equiv 0(\\bmod p)$ occurs for almost all $n \\equiv 0(\\bmod p-1)$. To resolve the difficult case and finish the problem, we zoom in on specific $n$ that will let us use lifting the exponent on $a_{n-1}$. Claim - Suppose $n$ is an (even) integer satisfying $$ \\begin{aligned} & n \\equiv 0 \\quad(\\bmod p-1) \\\\ & n \\equiv 1 \\quad\\left(\\bmod p^{k-1}\\right) \\end{aligned} $$ If $a_{n-1} \\equiv 0(\\bmod p)$, then in fact $p^{k} \\mid a_{n-1}$. $$ 0 \\equiv \\frac{1}{a_{n-2}}-1 \\quad(\\bmod p) \\Longrightarrow a_{n-2} \\equiv 1 \\quad(\\bmod p) $$ Hence lifting the exponent applies and we get $$ \\nu_{p}\\left(a_{n-1}\\right) \\equiv \\nu_{p}\\left(a_{n-2}^{n-1}-1\\right)=\\nu_{p}\\left(a_{n-2}-1\\right)+\\nu_{p}(n-1) \\geq 1+(k-1)=k $$ as desired. Remark. The first few terms are $a_{1}=2, a_{2}=3, a_{3}=26, a_{4}=456975$, and $a_{5}=$ 19927930852449199486318359374, ... No element of the sequence is divisible by 4: the residues modulo 4 alternate between 2 and 3 . Remark. The second claim is important for the solution to work once $k \\geq 2$. One could imagine a variation of the first claim that states if $n$ is divisible by $\\varphi\\left(p^{k}\\right)=p^{k-1}(p-1)$, then either $a_{n-1} \\equiv 0(\\bmod p)$ or $a_{n} \\equiv 0\\left(\\bmod p^{k}\\right)$. However this gives an obstruction (for $k \\geq 2)$ where we are guaranteed to have $n-1 \\not \\equiv 0(\\bmod p)$ now, so lifting the exponent will never give additional factors of $p$ we want."} +{"year":2024,"label":"4","problem":"Let $n \\geq 3$ be an integer. Rowan and Colin play a game on an $n \\times n$ grid of squares, where each square is colored either red or blue. Rowan is allowed to permute the rows of the grid and Colin is allowed to permute the columns. A grid coloring is orderly if: - no matter how Rowan permutes the rows of the coloring, Colin can then permute the columns to restore the original grid coloring; and - no matter how Colin permutes the columns of the coloring, Rowan can then permute the rows to restore the original grid coloring. In terms of $n$, how many orderly colorings are there?","solution":" The answer is $2 n!+2$. In fact, we can describe all the orderly colorings as follows: - The all-blue coloring. - The all-red coloring. - Each of the $n$ ! colorings where every row\/column has exactly one red cell. - Each of the $n$ ! colorings where every row\/column has exactly one blue cell. These obviously work; we turn our attention to proving these are the only ones. For the other direction, fix a orderly coloring $\\mathcal{A}$. Consider any particular column $C$ in $\\mathcal{A}$ and let $m$ denote the number of red cells that $C$ has. Any row permutation (say $\\sigma$ ) that Rowan chooses will transform $C$ into some column $\\sigma(C)$, and our assumption requires $\\sigma(C)$ has to appear somewhere in the original assignment $\\mathcal{A}$. An example for $n=7, m=2$, and a random $\\sigma$ is shown below. ![](https:\/\/cdn.mathpix.com\/cropped\/2024_11_19_805564b9ec1a761a333dg-10.jpg?height=692&width=815&top_left_y=1910&top_left_x=618) On the other hand, the number of possible patterns of $\\sigma(C)$ is easily seen to be exactly ( $\\binom{n}{m}$ - and they must all appear. In particular, if $m \\notin\\{0,1, n-1, n\\}$, then we immediately get a contradiction because $\\mathcal{A}$ would need too many columns (there are only $n$ columns in $\\mathcal{A}$, which is fewer than $\\binom{n}{m}$ ). Moreover, if either $m=1$ or $m=n-1$, these columns are all the columns of $\\mathcal{A}$; these lead to the $2 n$ ! main cases we found before. The only remaining case is when $m \\in\\{0, n\\}$ for every column, i.e. every column is monochromatic. It's easy to see in that case the columns must all be the same color."} +{"year":2024,"label":"5","problem":"Solve over $\\mathbb{R}$ the functional equation $f\\left(x^{2}-y\\right)+2 y f(x)=f(f(x))+f(y)$.","solution":" The answer is $f(x) \\equiv x^{2}, f(x) \\equiv 0, f(x) \\equiv-x^{2}$, which obviously work. Let $P(x, y)$ be the usual assertion. Claim - We have $f(0)=0$ and $f$ even. Claim $-f(x) \\in\\left\\{-x^{2}, 0, x^{2}\\right\\}$ for every $x \\in \\mathbb{R}$. $$ x^{2} f(x)=f\\left(x^{2}\\right)=f(f(x)) $$ Repeating this key identity several times gives $$ \\begin{aligned} f(f(f(x))) & =f\\left(f\\left(x^{2}\\right)\\right)=f\\left(x^{4}\\right)=x^{4} f\\left(x^{2}\\right) \\\\ & =f(x)^{2} \\cdot f(f(x))=f(x)^{2} f\\left(x^{2}\\right)=f(x)^{3} x^{2} \\end{aligned} $$ Suppose $t \\neq 0$ is such that $f\\left(t^{2}\\right) \\neq 0$. Then the above equalities imply $$ t^{4} f\\left(t^{2}\\right)=f(t)^{2} f\\left(t^{2}\\right) \\Longrightarrow f(t)= \\pm t^{2} $$ and then $$ f(t)^{2} f\\left(t^{2}\\right)=f(t)^{3} t^{2} \\Longrightarrow f\\left(t^{2}\\right)= \\pm t^{2} $$ Together with $f$ even, we get the desired result. Now, note that $P(1, y)$ gives $$ f(1-y)+2 y \\cdot f(1)=f(1)+f(y) $$ We consider cases on $f(1)$ and show that $f$ matches the desired form. - If $f(1)=1$, then $f(1-y)+(2 y-1)=f(y)$. Consider the nine possibilities that arise: $$ \\begin{array}{lll} (1-y)^{2}+(2 y-1)=y^{2} & 0+(2 y-1)=y^{2} & -(1-y)^{2}+(2 y-1)=y^{2} \\\\ (1-y)^{2}+(2 y-1)=0 & 0+(2 y-1)=0 & -(1-y)^{2}+(2 y-1)=0 \\\\ (1-y)^{2}+(2 y-1)=-y^{2} & 0+(2 y-1)=-y^{2} & -(1-y)^{2}+(2 y-1)=-y^{2} \\end{array} $$ Each of the last eight equations is a nontrivial polynomial equation. Hence, there is some constant $C>100$ such that the latter eight equations are all false for any real number $y>C$. Consequently, $f(y)=y^{2}$ for $y>C$. Finally, for any real number $z>0$, take $x, y>C$ such that $x^{2}-y=z$; then $P(x, y)$ proves $f(z)=z^{2}$ too. - Note that (as $f$ is even), $f$ works iff $-f$ works, so the case $f(1)=-1$ is analogous. - If $f(1)=0$, then $f(1-y)=f(y)$. Hence for any $y$ such that $|1-y| \\neq|y|$, we conclude $f(y)=0$. Then take $P(2,7 \/ 2) \\Longrightarrow f(1 \/ 2)=0$. $$ P\\left(x, y^{2}\\right) \\Longrightarrow f\\left(x^{2}-y^{2}\\right)+2 y^{2} f(x)=f(f(x))+f(f(y)) $$ Since $f$ is even gives $f\\left(x^{2}-y^{2}\\right)=f\\left(y^{2}-x^{2}\\right)$, one can swap the roles of $x$ and $y$ to get $2 y^{2} f(x)=2 x^{2} f(y)$. Set $y=1$ to finish."} +{"year":2024,"label":"6","problem":"Point $D$ is selected inside acute triangle $A B C$ so that $\\angle D A C=\\angle A C B$ and $\\angle B D C=90^{\\circ}+\\angle B A C$. Point $E$ is chosen on ray $B D$ so that $A E=E C$. Let $M$ be the midpoint of $B C$. Show that line $A B$ is tangent to the circumcircle of triangle $B E M$.","solution":" \u092c The author's original solution. Complete isosceles trapezoid $A B Q C$ (so $D \\in \\overline{A Q}$ ). Reflect $B$ across $E$ to point $F$. ![](https:\/\/cdn.mathpix.com\/cropped\/2024_11_19_805564b9ec1a761a333dg-14.jpg?height=663&width=718&top_left_y=959&top_left_x=675) Claim - We have $D Q C F$ is cyclic. $$ \\begin{aligned} \\measuredangle F D C & =-\\measuredangle C D B=180^{\\circ}-\\left(90^{\\circ}+\\measuredangle C A B\\right)=90^{\\circ}-\\measuredangle C A B \\\\ & =90^{\\circ}-\\measuredangle Q C A=\\measuredangle F Q C . \\end{aligned} $$ To conclude, note that $$ \\measuredangle B E M=\\measuredangle B F C=\\measuredangle D F C=\\measuredangle D Q C=\\measuredangle A Q C=\\measuredangle A B C=\\measuredangle A B M $$ Remark (Motivation). Here is one possible way to come up with the construction of point $F$ (at least this is what led Evan to find it). If one directs all the angles in the obvious way, there are really two points $D$ and $D^{\\prime}$ that are possible, although one is outside the triangle; they give corresponding points $E$ and $E^{\\prime}$. The circles $B E M$ and $B E^{\\prime} M$ must then actually coincide since they are both alleged to be tangent to line $A B$. See the figure below. ![](https:\/\/cdn.mathpix.com\/cropped\/2024_11_19_805564b9ec1a761a333dg-15.jpg?height=1112&width=1047&top_left_y=238&top_left_x=513) One can already prove using angle chasing that $\\overline{A B}$ is tangent to $\\left(B E E^{\\prime}\\right)$. So the point of the problem is to show that $M$ lies on this circle too. However, from looking at the diagram, one may realize that in fact it seems $$ \\triangle M E E^{\\prime} \\stackrel{ }{\\sim} \\triangle C D D^{\\prime} $$ is going to be true from just those marked in the figure (and this would certainly imply the desired concyclic conclusion). Since $M$ is a midpoint, it makes sense to dilate $\\triangle E M E^{\\prime}$ from $B$ by a factor of 2 to get $\\triangle F C F^{\\prime}$ so that the desired similarity is actually a spiral similarity at $C$. Then the spiral similarity lemma says that the desired similarity is equivalent to requiring $\\overline{D D^{\\prime}} \\cap \\overline{F F^{\\prime}}=Q$ to lie on both $(C D F)$ and $\\left(C D^{\\prime} F^{\\prime}\\right)$. Hence the key construction and claim from the solution are both discovered naturally, and we find the solution above. (The points $D^{\\prime}, E^{\\prime}, F^{\\prime}$ can then be deleted to hide the motivation.)"} +{"year":2024,"label":"6","problem":"Point $D$ is selected inside acute triangle $A B C$ so that $\\angle D A C=\\angle A C B$ and $\\angle B D C=90^{\\circ}+\\angle B A C$. Point $E$ is chosen on ray $B D$ so that $A E=E C$. Let $M$ be the midpoint of $B C$. Show that line $A B$ is tangent to the circumcircle of triangle $B E M$.","solution":" Another short solution. Let $Z$ be on line $B D E$ such that $\\angle B A Z=90^{\\circ}$. This lets us interpret the angle condition as follows: $$ \\text { Claim - Points } A, D, Z, C \\text { are cyclic. } $$ ![](https:\/\/cdn.mathpix.com\/cropped\/2024_11_19_805564b9ec1a761a333dg-16.jpg?height=804&width=701&top_left_y=249&top_left_x=683) Define $W$ as the midpoint of $\\overline{B Z}$, so $\\overline{M W} \\| \\overline{C Z}$. And let $O$ denote the center of $(A B C)$. Claim - Points $M, E, O, W$ are cyclic. $$ \\begin{aligned} \\measuredangle M O E & =\\measuredangle(\\overline{O M}, \\overline{B C})+\\measuredangle(\\overline{B C}, \\overline{A C})+\\measuredangle(\\overline{A C}, \\overline{O E}) \\\\ & =90^{\\circ}+\\measuredangle B C A+90^{\\circ} \\\\ & =\\measuredangle B C A=\\measuredangle C A D=\\measuredangle C Z D=\\measuredangle M W D=\\measuredangle M W E . \\end{aligned} $$ To finish, note $$ \\begin{aligned} \\measuredangle M E B & =\\measuredangle M E W=\\measuredangle M O W \\\\ & =\\measuredangle(\\overline{M O}, \\overline{B C})+\\measuredangle(\\overline{B C}, \\overline{A B})+\\measuredangle(\\overline{A B}, \\overline{O W}) \\\\ & =90^{\\circ}+\\measuredangle C B A+90^{\\circ}=\\measuredangle C B A=\\measuredangle M B A . \\end{aligned} $$ This implies the desired tangency."} +{"year":2024,"label":"6","problem":"Point $D$ is selected inside acute triangle $A B C$ so that $\\angle D A C=\\angle A C B$ and $\\angle B D C=90^{\\circ}+\\angle B A C$. Point $E$ is chosen on ray $B D$ so that $A E=E C$. Let $M$ be the midpoint of $B C$. Show that line $A B$ is tangent to the circumcircle of triangle $B E M$.","solution":" I A Menelaus-based approach (Kevin Ren). Let $P$ be on $\\overline{B C}$ with $A P=P C$. Let $Y$ be the point on line $A B$ such that $\\angle A C Y=90^{\\circ}$; as $\\angle A Y C=90^{\\circ}-A$ it follows $B D Y C$ is cyclic. Let $K=\\overline{A P} \\cap \\overline{C Y}$, so $\\triangle A C K$ is a right triangle with $P$ the midpoint of its hypotenuse. ![](https:\/\/cdn.mathpix.com\/cropped\/2024_11_19_805564b9ec1a761a333dg-17.jpg?height=1192&width=1075&top_left_y=249&top_left_x=496) Claim - Triangles BPE and DYK are similar. Claim - Triangles $B E M$ and $Y D C$ are similar. $$ \\frac{B P}{B C} \\frac{Y C}{Y K} \\frac{A K}{A P}=1 $$ Since $A K \/ A P=2$ (note that $P$ is the midpoint of the hypotenuse of right triangle $A C K)$ and $B C=2 B M$, this simplifies to $$ \\frac{B P}{B M}=\\frac{Y K}{Y C} $$ To finish, note that $$ \\measuredangle D B A=\\measuredangle D B Y=\\measuredangle D C Y=\\measuredangle B M E $$ implying the desired tangency."} +{"year":2024,"label":"6","problem":"Point $D$ is selected inside acute triangle $A B C$ so that $\\angle D A C=\\angle A C B$ and $\\angle B D C=90^{\\circ}+\\angle B A C$. Point $E$ is chosen on ray $B D$ so that $A E=E C$. Let $M$ be the midpoint of $B C$. Show that line $A B$ is tangent to the circumcircle of triangle $B E M$.","solution":" \u300e A spiral similarity approach (Hans $\\mathbf{Y u}$ ). As in the previous solution, let $Y$ be the point on line $A B$ such that $\\angle A C Y=90^{\\circ}$; so $B D Y C$ is cyclic. Let $\\Gamma$ be the circle through $B$ and $M$ tangent to $\\overline{A B}$, and let $\\Omega:=(B C Y D)$. We need to show $E \\in \\Gamma$. ![](https:\/\/cdn.mathpix.com\/cropped\/2024_11_19_805564b9ec1a761a333dg-18.jpg?height=1101&width=1029&top_left_y=429&top_left_x=519) Denote by $S$ the second intersection of $\\Gamma$ and $\\Omega$. The main idea behind is to consider the spiral similarity $$ \\Psi: \\Omega \\rightarrow \\Gamma \\quad C \\mapsto M \\text { and } Y \\mapsto B $$ centered at $S$ (due to the spiral similarity lemma), and show that $\\Psi(D)=E$. The spiral similarity lemma already promises $\\Psi(D)$ lies on line $B D$. Claim - We have $\\Psi(A)=O$, the circumcenter of $A B C$. Claim - $\\Psi$ maps line $A D$ to line $O P$. $$ \\measuredangle(\\overline{A D}, \\overline{O P})=\\measuredangle A P O=\\measuredangle O P C=\\measuredangle Y C P=\\measuredangle(\\overline{Y C}, \\overline{B M}) $$ As $\\Psi$ maps line $Y C$ to line $B M$ and $\\Psi(A)=O$, we're done. Hence $\\Psi(D)$ should not only lie on $B D$ but also line $O P$. This proves $\\Psi(D)=E$, so $E \\in \\Gamma$ as needed."} diff --git a/USAMO/md/en-USAMO-1996-notes.md b/USAMO/md/en-USAMO-1996-notes.md new file mode 100644 index 0000000000000000000000000000000000000000..2da26ba890a8c9f32015d04514f443f32393c2e6 --- /dev/null +++ b/USAMO/md/en-USAMO-1996-notes.md @@ -0,0 +1,209 @@ +# USAMO 1996 Solution Notes + +Evan Chen《陳誼廷》 + +15 April 2024 + +This is a compilation of solutions for the 1996 USAMO. The ideas of the solution are a mix of my own work, the solutions provided by the competition organizers, and solutions found by the community. However, all the writing is maintained by me. + +These notes will tend to be a bit more advanced and terse than the "official" solutions from the organizers. In particular, if a theorem or technique is not known to beginners but is still considered "standard", then I often prefer to use this theory anyways, rather than try to work around or conceal it. For example, in geometry problems I typically use directed angles without further comment, rather than awkwardly work around configuration issues. Similarly, sentences like "let $\mathbb{R}$ denote the set of real numbers" are typically omitted entirely. + +Corrections and comments are welcome! + +## Contents + +0 Problems ..... 2 +1 Solutions to Day 1 ..... 3 +1.1 USAMO 1996/1 ..... 3 +1.2 USAMO 1996/2 ..... 4 +1.3 USAMO 1996/3 +2 Solutions to Day 2 ..... 6 +2.1 USAMO 1996/4 ..... 6 +2.2 USAMO 1996/5 ..... 7 +2.3 USAMO 1996/6 ..... 8 + +## §0 Problems + +1. Prove that the average of the numbers $n \sin n^{\circ}$ for $n=2,4,6, \ldots, 180$ is $\cot 1^{\circ}$. +2. For any nonempty set $S$ of real numbers, let $\sigma(S)$ denote the sum of the elements of $S$. Given a set $A$ of $n$ positive integers, consider the collection of all distinct sums $\sigma(S)$ as $S$ ranges over the nonempty subsets of $A$. Prove that this collection of sums can be partitioned into $n$ classes so that in each class, the ratio of the largest sum to the smallest sum does not exceed 2 . +3. Let $A B C$ be a triangle. Prove that there is a line $\ell$ (in the plane of triangle $A B C$ ) such that the intersection of the interior of triangle $A B C$ and the interior of its reflection $A^{\prime} B^{\prime} C^{\prime}$ in $\ell$ has area more than $\frac{2}{3}$ the area of triangle $A B C$. +4. An $n$-term sequence $\left(x_{1}, x_{2}, \ldots, x_{n}\right)$ in which each term is either 0 or 1 is called a binary sequence of length $n$. Let $a_{n}$ be the number of binary sequences of length $n$ containing no three consecutive terms equal to $0,1,0$ in that order. Let $b_{n}$ be the number of binary sequences of length $n$ that contain no four consecutive terms equal to $0,0,1,1$ or $1,1,0,0$ in that order. Prove that $b_{n+1}=2 a_{n}$ for all positive integers $n$. +5. Let $A B C$ be a triangle, and $M$ an interior point such that $\angle M A B=10^{\circ}, \angle M B A=$ $20^{\circ}, \angle M A C=40^{\circ}$ and $\angle M C A=30^{\circ}$. Prove that the triangle is isosceles. +6. Determine with proof whether there is a subset $X \subseteq \mathbb{Z}$ with the following property: for any $n \in \mathbb{Z}$, there is exactly one solution to $a+2 b=n$, with $a, b \in X$. + +## §1 Solutions to Day 1 + +## §1.1 USAMO 1996/1 + +Available online at https://aops.com/community/p353049. + +## Problem statement + +Prove that the average of the numbers $n \sin n^{\circ}$ for $n=2,4,6, \ldots, 180$ is $\cot 1^{\circ}$. + +Because + +$$ +n \sin n^{\circ}+(180-n) \sin \left(180^{\circ}-n^{\circ}\right)=180 \sin n^{\circ} +$$ + +So enough to show that + +$$ +\sum_{n=0}^{89} \sin (2 n)^{\circ}=\cot 1^{\circ} +$$ + +Let $\zeta=\cos 2^{\circ}+i \sin 2^{\circ}$ be a primitive root. Then + +$$ +\begin{aligned} +\sum_{n=0}^{89} \frac{\zeta^{n}-\zeta^{-n}}{2 i} & =\frac{1}{2 i}\left[\frac{\zeta^{90}-1}{\zeta-1}-\frac{\zeta^{-90}-1}{\zeta^{-1}-1}\right] \\ +& =\frac{1}{2 i}\left[\frac{-2}{\zeta-1}-\frac{-2}{\zeta^{-1}-1}\right] \\ +& =\frac{1}{-i} \frac{\zeta^{-1}-\zeta}{(\zeta-1)\left(\zeta^{-1}-1\right)}=i \cdot \frac{\zeta+1}{\zeta-1} +\end{aligned} +$$ + +Also, + +$$ +\begin{aligned} +\cot 1^{\circ} & =\frac{\cos 1^{\circ}}{\sin 1^{\circ}}=\frac{\left(\cos 1^{\circ}\right)^{2}}{\cos 1^{\circ} \sin 1^{\circ}} \\ +& =\frac{\frac{\cos 2^{\circ}+1}{2}}{\frac{\sin 2^{\circ}}{2}}=\frac{\frac{1}{2}\left(\zeta+\zeta^{-1}\right)+1}{\frac{1}{2 i}\left(\zeta-\zeta^{-1}\right)} \\ +& =i \cdot \frac{(\zeta+1)^{2}}{\zeta^{2}-1}=i \cdot \frac{\zeta+1}{\zeta-1} +\end{aligned} +$$ + +So we're done. + +## §1.2 USAMO 1996/2 + +Available online at https://aops.com/community/p353051. + +## Problem statement + +For any nonempty set $S$ of real numbers, let $\sigma(S)$ denote the sum of the elements of $S$. Given a set $A$ of $n$ positive integers, consider the collection of all distinct sums $\sigma(S)$ as $S$ ranges over the nonempty subsets of $A$. Prove that this collection of sums can be partitioned into $n$ classes so that in each class, the ratio of the largest sum to the smallest sum does not exceed 2. + +By induction on $n$ with $n=1$ being easy. +For the inductive step, assume + +$$ +A=\left\{a_{1}>a_{2}>\cdots>a_{n}\right\} +$$ + +Fix any index $k$ with the property that + +$$ +a_{k}>\frac{\sigma(A)}{2^{k}} +$$ + +(which must exist since $\frac{1}{2}+\frac{1}{4}+\cdots+\frac{1}{2^{k}}<1$ ). Then + +- We make $k$ classes for the sums between $\frac{\sigma(A)}{2^{k}}$ and $\sigma(A)$; this handles every set which has any element in $\left\{a_{1}, \ldots, a_{k}\right\}$. +- We make $n-k$ classes via induction hypothesis on $\left\{a_{k+1}, \ldots, a_{n}\right\}$. + +This solves the problem. + +## §1.3 USAMO 1996/3 + +Available online at https://aops.com/community/p353052. + +## Problem statement + +Let $A B C$ be a triangle. Prove that there is a line $\ell$ (in the plane of triangle $A B C$ ) such that the intersection of the interior of triangle $A B C$ and the interior of its reflection $A^{\prime} B^{\prime} C^{\prime}$ in $\ell$ has area more than $\frac{2}{3}$ the area of triangle $A B C$. + +All that's needed is: +Claim - If $A B C$ is a triangle where $\frac{1}{2}<\frac{A B}{A C}<1$, then the $\angle A$ bisector works. + +Proof. Let the $\angle A$-bisector meet $B C$ at $D$. The overlapped area is $2[A B D]$ and + +$$ +\frac{[A B D]}{[A B C]}=\frac{B D}{B C}=\frac{A B}{A B+A C} +$$ + +by angle bisector theorem. +In general, suppose $x0$ prove that + +$$ +\frac{1}{a^{3}+b^{3}+a b c}+\frac{1}{b^{3}+c^{3}+a b c}+\frac{1}{c^{3}+a^{3}+a b c} \leq \frac{1}{a b c} . +$$ + +6. Suppose the sequence of nonnegative integers $a_{1}, a_{2}, \ldots, a_{1997}$ satisfies + +$$ +a_{i}+a_{j} \leq a_{i+j} \leq a_{i}+a_{j}+1 +$$ + +for all $i, j \geq 1$ with $i+j \leq 1997$. Show that there exists a real number $x$ such that $a_{n}=\lfloor n x\rfloor$ for all $1 \leq n \leq 1997$. + +## §1 Solutions to Day 1 + +## §1.1 USAMO 1997/1 + +Available online at https://aops.com/community/p343871. + +## Problem statement + +Let $p_{1}, p_{2}, p_{3}, \ldots$ be the prime numbers listed in increasing order, and let $02$, we have $\left|\frac{c-a}{-5}\right| \leq \frac{|c|+9}{5}<|c|$. Thus, we eventually reach a pair with $|c| \leq 2$. +- Similarly, if $|b|>9$, we have $\left|\frac{b-a}{-2}\right| \leq \frac{|b|+9}{2}<|b|$, so we eventually reach a pair with $|b| \leq 9$. +this leaves us with $5 \cdot 19=95$ ordered pairs to check (though only about one third have $b \equiv c(\bmod 3))$. This can be done by the following code: + +``` +import functools +@functools.lru_cache() +def f(x0, yO): + if x0 == 0 and y0 == 0: + return 0 + if x0 % 2 == (y0 % 5) % 2: + d = y0 % 5 + else: + d = (y0 % 5) + 5 + x1 = (x0 - d) // (-2) + y1 = (y0 - d) // (-5) + return 1 + f(x1, y1) +for x in range(-9, 10): +for y in range(-2, 3): + if (x % 3 == y % 3): + print(f"({x:2d}, {y:2d}) finished in {f(x,y)} moves") +``` + +As this gives the output +![](https://cdn.mathpix.com/cropped/2024_11_19_35408d86cf32c99e435fg-06.jpg?height=1317&width=664&top_left_y=1169&top_left_x=296) +we are done. + +## §2 Solutions to Day 2 + +## §2.1 USAMO 1997/4 + +Available online at https://aops.com/community/p343875. + +## Problem statement + +To clip a convex $n$-gon means to choose a pair of consecutive sides $A B, B C$ and to replace them by the three segments $A M, M N$, and $N C$, where $M$ is the midpoint of $A B$ and $N$ is the midpoint of $B C$. In other words, one cuts off the triangle $M B N$ to obtain a convex $(n+1)$-gon. A regular hexagon $\mathcal{P}_{6}$ of area 1 is clipped to obtain a heptagon $\mathcal{P}_{7}$. Then $\mathcal{P}_{7}$ is clipped (in one of the seven possible ways) to obtain an octagon $\mathcal{P}_{8}$, and so on. Prove that no matter how the clippings are done, the area of $\mathcal{P}_{n}$ is greater than $\frac{1}{3}$, for all $n \geq 6$. + +Call the original hexagon $A B C D E F$. We show the area common to triangles $A C E$ and $B D F$ is in every $\mathcal{P}_{n}$; this solves the problem since the area is $1 / 3$. + +For every side of a clipped polygon, we define its foundation recursively as follows: + +- $A B, B C, C D, D E, E F, F A$ are each their own foundation (we also call these original sides). +- When a new clipped edge is added, its foundation is the union of the foundations of the two edges it touches. + +Hence, any foundations are nonempty subsets of original sides. +Claim - All foundations are in fact at most two-element sets of adjacent original sides. + +Proof. It's immediate by induction that any two adjacent sides have at most two elements in the union of their foundations, and if there are two, they are two adjacent original sides. + +Now, if a side has foundation contained in $\{A B, B C\}$, say, then the side should be contained within triangle $A B C$. Hence the side does not touch $A C$. This proves the problem. + +## §2.2 USAMO 1997/5 + +Available online at https://aops.com/community/p2971. + +## Problem statement + +If $a, b, c>0$ prove that + +$$ +\frac{1}{a^{3}+b^{3}+a b c}+\frac{1}{b^{3}+c^{3}+a b c}+\frac{1}{c^{3}+a^{3}+a b c} \leq \frac{1}{a b c} +$$ + +From $a^{3}+b^{3} \geq a b(a+b)$, the left-hand side becomes + +$$ +\sum_{\mathrm{cyc}} \frac{1}{a^{3}+b^{3}+a b c} \leq \sum_{\mathrm{cyc}} \frac{1}{a b(a+b+c)}=\frac{1}{a b c} \sum_{\mathrm{cyc}} \frac{c}{a+b+c}=\frac{1}{a b c} +$$ + +## §2.3 USAMO 1997/6 + +Available online at https://aops.com/community/p343876. + +## Problem statement + +Suppose the sequence of nonnegative integers $a_{1}, a_{2}, \ldots, a_{1997}$ satisfies + +$$ +a_{i}+a_{j} \leq a_{i+j} \leq a_{i}+a_{j}+1 +$$ + +for all $i, j \geq 1$ with $i+j \leq 1997$. Show that there exists a real number $x$ such that $a_{n}=\lfloor n x\rfloor$ for all $1 \leq n \leq 1997$. + +We are trying to show there exists an $x \in \mathbb{R}$ such that + +$$ +\frac{a_{n}}{n} \leq x<\frac{a_{n}+1}{n} \quad \forall n +$$ + +This means we need to show + +$$ +\max _{i} \frac{a_{i}}{i}<\min _{j} \frac{a_{j}+1}{j} . +$$ + +Replace 1997 by $N$. We will prove this by induction, but we will need some extra hypotheses on the indices $i, j$ which are used above. + +## Claim - Suppose that + +- Integers $a_{1}, a_{2}, \ldots, a_{N}$ satisfy the given conditions. +- Let $i=\operatorname{argmax}_{n} \frac{a_{n}}{n}$; if there are ties, pick the smallest $i$. +- Let $j=\operatorname{argmin}_{n} \frac{a_{n}+1}{n}$; if there are ties, pick the smallest $j$. + +Then + +$$ +\frac{a_{i}}{i}<\frac{a_{j}+1}{j} +$$ + +Moreover, these two fractions are in lowest terms, and are adjacent in the Farey sequence of order $\max (i, j)$. + +Proof. By induction on $N \geq 1$ with the base case clear. So suppose we have the induction hypothesis with numbers $a_{1}, \ldots, a_{N-1}$, with $i$ and $j$ as promised. + +Now, consider the new number $a_{N}$. We have two cases: + +- Suppose $i+j>N$. Then, no fraction with denominator $N$ can lie strictly inside the interval; so we may write for some integer $b$ + +$$ +\frac{b}{N} \leq \frac{a_{i}}{i}<\frac{a_{j}+1}{j} \leq \frac{b+1}{N} +$$ + +For purely algebraic reasons we have + +$$ +\frac{b-a_{i}}{N-i} \leq \frac{b}{N} \leq \frac{a_{i}}{i}<\frac{a_{j}+1}{j} \leq \frac{b+1}{N} \leq \frac{b-a_{j}}{N-j} . +$$ + +Now, + +$$ +\begin{aligned} +a_{N} & \geq a_{i}+a_{N-i} \geq a_{i}+(N-i) \cdot \frac{a_{i}}{i} \\ +& \geq a_{i}+\left(b-a_{i}\right)=b \\ +a_{N} & \leq a_{j}+a_{N-j}+1 \leq\left(a_{j}+1\right)+(N-j) \cdot \frac{a_{j}+1}{j} \\ +& =\left(a_{j}+1\right)+\left(b-a_{j}\right)=b+1 +\end{aligned} +$$ + +Thus $a_{N} \in\{b, b+1\}$. This proves that $\frac{a_{N}}{N} \leq \frac{a_{i}}{i}$ while $\frac{a_{N}+1}{N} \geq \frac{a_{j}+1}{j}$. Moreover, the pair $(i, j)$ does not change, so all inductive hypotheses carry over. + +- On the other hand, suppose $i+j=N$. Then we have + +$$ +\frac{a_{i}}{i}<\frac{a_{i}+a_{j}+1}{N}<\frac{a_{j}+1}{j} . +$$ + +Now, we know $a_{N}$ could be either $a_{i}+a_{j}$ or $a_{i}+a_{j}+1$. If it's the former, then $(i, j)$ becomes $(i, N)$. If it's the latter, then $(i, j)$ becomes $(N, j)$. The properties of Farey sequences ensure that the $\frac{a_{i}+a_{j}+1}{N}$ is reduced, either way. + diff --git a/USAMO/md/en-USAMO-1998-notes.md b/USAMO/md/en-USAMO-1998-notes.md new file mode 100644 index 0000000000000000000000000000000000000000..54eb3c2809475b9b8f524b719029d25415d4006e --- /dev/null +++ b/USAMO/md/en-USAMO-1998-notes.md @@ -0,0 +1,207 @@ +# USAMO 1998 Solution Notes + +Evan ChEn《陳誼廷》 + +15 April 2024 + +This is a compilation of solutions for the 1998 USAMO. The ideas of the solution are a mix of my own work, the solutions provided by the competition organizers, and solutions found by the community. However, all the writing is maintained by me. + +These notes will tend to be a bit more advanced and terse than the "official" solutions from the organizers. In particular, if a theorem or technique is not known to beginners but is still considered "standard", then I often prefer to use this theory anyways, rather than try to work around or conceal it. For example, in geometry problems I typically use directed angles without further comment, rather than awkwardly work around configuration issues. Similarly, sentences like "let $\mathbb{R}$ denote the set of real numbers" are typically omitted entirely. + +Corrections and comments are welcome! + +## Contents + +0 Problems ..... 2 +1 Solutions to Day 1 ..... 3 +1.1 USAMO 1998/1 ..... 3 +1.2 USAMO 1998/2 ..... 4 +1.3 USAMO 1998/3 +2 Solutions to Day 2 ..... 6 +2.1 USAMO 1998/4 ..... 6 +2.2 USAMO 1998/5 +2.3 USAMO 1998/6 ..... 8 + +## §0 Problems + +1. Suppose that the set $\{1,2, \ldots, 1998\}$ has been partitioned into disjoint pairs $\left\{a_{i}, b_{i}\right\}$ $(1 \leq i \leq 999)$ so that for all $i,\left|a_{i}-b_{i}\right|$ equals 1 or 6 . Prove that the sum + +$$ +\left|a_{1}-b_{1}\right|+\left|a_{2}-b_{2}\right|+\cdots+\left|a_{999}-b_{999}\right| +$$ + +ends in the digit 9. +2. Let $\mathcal{C}_{1}$ and $\mathcal{C}_{2}$ be concentric circles, with $\mathcal{C}_{2}$ in the interior of $\mathcal{C}_{1}$. From a point $A$ on $\mathcal{C}_{1}$ one draws the tangent $A B$ to $\mathcal{C}_{2}\left(B \in \mathcal{C}_{2}\right)$. Let $C$ be the second point of intersection of ray $A B$ and $\mathcal{C}_{1}$, and let $D$ be the midpoint of $\overline{A B}$. A line passing through $A$ intersects $\mathcal{C}_{2}$ at $E$ and $F$ in such a way that the perpendicular bisectors of $\overline{D E}$ and $\overline{C F}$ intersect at a point $M$ on line $A B$. Find, with proof, the ratio $A M / M C$. +3. Let $a_{0}, a_{1}, \ldots, a_{n}$ be numbers from the interval $(0, \pi / 2)$ such that $\tan \left(a_{0}-\frac{\pi}{4}\right)+$ $\tan \left(a_{1}-\frac{\pi}{4}\right)+\cdots+\tan \left(a_{n}-\frac{\pi}{4}\right) \geq n-1$. Prove that + +$$ +\tan a_{0} \tan a_{1} \cdots \tan a_{n} \geq n^{n+1} +$$ + +4. A computer screen shows a $98 \times 98$ chessboard, colored in the usual way. One can select with a mouse any rectangle with sides on the lines of the chessboard and click the mouse button: as a result, the colors in the selected rectangle switch (black becomes white, white becomes black). Find, with proof, the minimum number of mouse clicks needed to make the chessboard all one color. +5. Prove that for each $n \geq 2$, there is a set $S$ of $n$ integers such that $(a-b)^{2}$ divides $a b$ for every distinct $a, b \in S$. +6. Let $n \geq 5$ be an integer. Find the largest integer $k$ (as a function of $n$ ) such that there exists a convex $n$-gon $A_{1} A_{2} \ldots A_{n}$ for which exactly $k$ of the quadrilaterals $A_{i} A_{i+1} A_{i+2} A_{i+3}$ have an inscribed circle, where indices are taken modulo $n$. + +## §1 Solutions to Day 1 + +## §1.1 USAMO 1998/1 + +Available online at https://aops.com/community/p343865. + +## Problem statement + +Suppose that the set $\{1,2, \ldots, 1998\}$ has been partitioned into disjoint pairs $\left\{a_{i}, b_{i}\right\}$ $(1 \leq i \leq 999)$ so that for all $i,\left|a_{i}-b_{i}\right|$ equals 1 or 6 . Prove that the sum + +$$ +\left|a_{1}-b_{1}\right|+\left|a_{2}-b_{2}\right|+\cdots+\left|a_{999}-b_{999}\right| +$$ + +ends in the digit 9. + +Let $S$ be the sum. Modulo 2, + +$$ +S=\sum\left|a_{i}-b_{i}\right| \equiv \sum\left(a_{i}+b_{i}\right)=1+2+\cdots+1998 \equiv 1 \quad(\bmod 2) +$$ + +Modulo 5, + +$$ +S=\sum\left|a_{i}-b_{i}\right|=1 \cdot 999 \equiv 4 \quad(\bmod 5) +$$ + +So $S \equiv 9(\bmod 10)$. + +## §1.2 USAMO 1998/2 + +Available online at https://aops.com/community/p343866. + +## Problem statement + +Let $\mathcal{C}_{1}$ and $\mathcal{C}_{2}$ be concentric circles, with $\mathcal{C}_{2}$ in the interior of $\mathcal{C}_{1}$. From a point $A$ on $\mathcal{C}_{1}$ one draws the tangent $A B$ to $\mathcal{C}_{2}\left(B \in \mathcal{C}_{2}\right)$. Let $C$ be the second point of intersection of ray $A B$ and $\mathcal{C}_{1}$, and let $D$ be the midpoint of $\overline{A B}$. A line passing through $A$ intersects $\mathcal{C}_{2}$ at $E$ and $F$ in such a way that the perpendicular bisectors of $\overline{D E}$ and $\overline{C F}$ intersect at a point $M$ on line $A B$. Find, with proof, the ratio $A M / M C$. + +By power of a point we have + +$$ +A E \cdot A F=A B^{2}=\left(\frac{1}{2} A B\right) \cdot(2 A B)=A D \cdot A C +$$ + +and hence $C D E F$ is cyclic. Then $M$ is the circumcenter of quadrilateral $C D E F$. +![](https://cdn.mathpix.com/cropped/2024_11_19_4d9e3ecba99734fe13c2g-4.jpg?height=806&width=792&top_left_y=1179&top_left_x=638) + +Thus $M$ is the midpoint of $\overline{C D}$ (and we are given already that $B$ is the midpoint of $\overline{A C}$, $D$ is the midpoint of $\overline{A B}$ ). Thus a quick computation along $\overline{A C}$ gives $A M / M C=5 / 3$. + +## §1.3 USAMO 1998/3 + +Available online at https://aops.com/community/p343867. + +## Problem statement + +Let $a_{0}, a_{1}, \ldots, a_{n}$ be numbers from the interval $(0, \pi / 2)$ such that $\tan \left(a_{0}-\frac{\pi}{4}\right)+$ $\tan \left(a_{1}-\frac{\pi}{4}\right)+\cdots+\tan \left(a_{n}-\frac{\pi}{4}\right) \geq n-1$. Prove that + +$$ +\tan a_{0} \tan a_{1} \cdots \tan a_{n} \geq n^{n+1} . +$$ + +Let $x_{i}=\tan \left(a_{i}-\frac{\pi}{4}\right)$. Then we have that + +$$ +\tan a_{i}=\tan \left(a_{i}-45^{\circ}+45^{\circ}\right)=\frac{x_{i}+1}{1-x_{i}} +$$ + +If we further substitute $y_{i}=\frac{1-x_{i}}{2} \in(0,1)$, then we have to prove that the following statement: + +Claim - If $\sum_{0}^{n} y_{i} \leq 1$ and $y_{i} \geq 0$, we have + +$$ +\prod_{i=1}^{n}\left(\frac{1}{y_{i}}-1\right) \geq n^{n+1} +$$ + +Proof. Homogenizing, we have to prove that + +$$ +\prod_{i=1}^{n}\left(\frac{y_{0}+y_{1}+y_{2}+\cdots+y_{n}}{y_{i}}-1\right) \geq n^{n+1} +$$ + +By AM-GM, we have + +$$ +\frac{y_{1}+y_{2}+y_{3}+\cdots+y_{n}}{y_{0}} \geq n \sqrt[n]{\frac{y_{1} y_{2} y_{3} \ldots y_{n}}{y_{1}}} +$$ + +Cyclic product works. + +Remark. Alternatively, the function $x \mapsto \log (1 / x-1)$ is a convex function on $(0,1)$ so Jensen inequality should also work. + +## §2 Solutions to Day 2 + +## §2.1 USAMO 1998/4 + +Available online at https://aops.com/community/p343869. + +## Problem statement + +A computer screen shows a $98 \times 98$ chessboard, colored in the usual way. One can select with a mouse any rectangle with sides on the lines of the chessboard and click the mouse button: as a result, the colors in the selected rectangle switch (black becomes white, white becomes black). Find, with proof, the minimum number of mouse clicks needed to make the chessboard all one color. + +The answer is 98 . One of several possible constructions is to toggle all columns and rows with even indices. + +In the other direction, let $n=98$ and suppose that $k$ rectangles are used, none of which are $n \times n$ (else we may delete it). Then, for any two orthogonally adjacent cells, the edge between them must be contained in the edge of one of the $k$ rectangles. + +We define a gridline to be a line segment that runs in the interior of the board from one side of the board to the other. Hence there are $2 n-2$ gridlines exactly. Moreover, we can classify these rectangles into two types: + +- Full length rectangles: these span from one edge of the board to the other. The two long sides completely cover two gridlines, but the other two sides of the rectangle do not. +- Partial length rectangles: each of four sides can partially cover "half a" gridline. + +See illustration below for $n=6$. +![](https://cdn.mathpix.com/cropped/2024_11_19_4d9e3ecba99734fe13c2g-6.jpg?height=321&width=327&top_left_y=1601&top_left_x=522) +![](https://cdn.mathpix.com/cropped/2024_11_19_4d9e3ecba99734fe13c2g-6.jpg?height=315&width=326&top_left_y=1604&top_left_x=865) + +Full length +![](https://cdn.mathpix.com/cropped/2024_11_19_4d9e3ecba99734fe13c2g-6.jpg?height=350&width=318&top_left_y=1601&top_left_x=1212) + +Partial length + +Since there are $2 n-2$ gridlines; and each rectangle can cover at most two gridlines in total (where partial-length rectangles are "worth $\frac{1}{2}$ " on each of the four sides), we immediately get the bound $2 k \geq 2 n-2$, or $k \geq n-1$. + +To finish, we prove that: +Claim - If equality holds and $k=n-1$, then $n$ is odd. + +Proof. If equality holds, then look at the horizontal gridlines and say two gridlines are related if some rectangle has horizontal edges along both gridlines. (Hence, the graph has degree either 1 or 2 at each vertex, for equality to hold.) The reader may verify the resulting graph consists only of even length cycles and single edges, which would mean $n-1$ is even. + +Hence for $n=98$ the answer is indeed 98 as claimed. + +## §2.2 USAMO 1998/5 + +Available online at https://aops.com/community/p343870. + +## Problem statement + +Prove that for each $n \geq 2$, there is a set $S$ of $n$ integers such that $(a-b)^{2}$ divides $a b$ for every distinct $a, b \in S$. + +This is a direct corollary of the more difficult USA TST 2015/2, reproduced below. +Prove that for every positive integer $n$, there exists a set $S$ of $n$ positive integers such that for any two distinct $a, b \in S, a-b$ divides $a$ and $b$ but none of the other elements of $S$. + +## §2.3 USAMO 1998/6 + +Available online at https://aops.com/community/p150148. + +## Problem statement + +Let $n \geq 5$ be an integer. Find the largest integer $k$ (as a function of $n$ ) such that there exists a convex $n$-gon $A_{1} A_{2} \ldots A_{n}$ for which exactly $k$ of the quadrilaterals $A_{i} A_{i+1} A_{i+2} A_{i+3}$ have an inscribed circle, where indices are taken modulo $n$. + +The main claim is the following: +Claim - We can't have both $A_{1} A_{2} A_{3} A_{4}$ and $A_{2} A_{3} A_{4} A_{5}$ be circumscribed. + +Proof. If not, then we have the following diagram, where $a=A_{1} A_{2}, b=A-2 A_{3}$, $c=A_{3} A_{4}, d=A_{4} A_{5}$. +![](https://cdn.mathpix.com/cropped/2024_11_19_4d9e3ecba99734fe13c2g-8.jpg?height=621&width=892&top_left_y=1049&top_left_x=585) + +Then $A_{1} A_{4}=c+a-b$ and $A_{5} A_{2}=b+d-c$. But now + +$$ +A_{1} A_{4}+A_{2} A_{5}=(c+a-b)+(b+d-c)=a+d=A_{1} A_{2}+A_{4} A_{5} +$$ + +but in the picture we have an obvious violation of the triangle inequality. +This immediately gives an upper bound of $\lfloor n / 2\rfloor$. +For the construction, one can construct a suitable cyclic $n$-gon by using a continuity argument (details to be added). + diff --git a/USAMO/md/en-USAMO-1999-notes.md b/USAMO/md/en-USAMO-1999-notes.md new file mode 100644 index 0000000000000000000000000000000000000000..d74f1b9c278e8603d8870982bbd452ea37ce0045 --- /dev/null +++ b/USAMO/md/en-USAMO-1999-notes.md @@ -0,0 +1,275 @@ +# USAMO 1999 Solution Notes + +Evan ChEn《陳誼廷》 + +15 April 2024 + +This is a compilation of solutions for the 1999 USAMO. The ideas of the solution are a mix of my own work, the solutions provided by the competition organizers, and solutions found by the community. However, all the writing is maintained by me. + +These notes will tend to be a bit more advanced and terse than the "official" solutions from the organizers. In particular, if a theorem or technique is not known to beginners but is still considered "standard", then I often prefer to use this theory anyways, rather than try to work around or conceal it. For example, in geometry problems I typically use directed angles without further comment, rather than awkwardly work around configuration issues. Similarly, sentences like "let $\mathbb{R}$ denote the set of real numbers" are typically omitted entirely. + +Corrections and comments are welcome! + +## Contents + +0 Problems ..... 2 +1 Solutions to Day 1 ..... 3 +1.1 USAMO 1999/1 ..... 3 +1.2 USAMO 1999/2 ..... 4 +1.3 USAMO 1999/3 +2 Solutions to Day 2 ..... 8 +2.1 USAMO 1999/4 ..... 8 +2.2 USAMO 1999/5 ..... 9 +2.3 USAMO 1999/6 ..... 10 + +## §0 Problems + +1. Some checkers placed on an $n \times n$ checkerboard satisfy the following conditions: +(a) every square that does not contain a checker shares a side with one that does; +(b) given any pair of squares that contain checkers, there is a sequence of squares containing checkers, starting and ending with the given squares, such that every two consecutive squares of the sequence share a side. +Prove that at least $\left(n^{2}-2\right) / 3$ checkers have been placed on the board. +2. Let $A B C D$ be a convex cyclic quadrilateral. Prove that + +$$ +|A B-C D|+|A D-B C| \geq 2|A C-B D| +$$ + +3. Let $p>2$ be a prime and let $a, b, c, d$ be integers not divisible by $p$, such that + +$$ +\left\{\frac{r a}{p}\right\}+\left\{\frac{r b}{p}\right\}+\left\{\frac{r c}{p}\right\}+\left\{\frac{r d}{p}\right\}=2 +$$ + +for any integer $r$ not divisible by $p$. (Here, $\{t\}=t-\lfloor t\rfloor$ is the fractional part.) Prove that at least two of the numbers $a+b, a+c, a+d, b+c, b+d, c+d$ are divisible by $p$. +4. Let $a_{1}, a_{2}, \ldots, a_{n}$ be a sequence of $n>3$ real numbers such that + +$$ +a_{1}+\cdots+a_{n} \geq n \quad \text { and } \quad a_{1}^{2}+\cdots+a_{n}^{2} \geq n^{2} +$$ + +Prove that $\max \left(a_{1}, \ldots, a_{n}\right) \geq 2$. +5. The Y2K Game is played on a $1 \times 2000$ grid as follows. Two players in turn write either an S or an O in an empty square. The first player who produces three consecutive boxes that spell SOS wins. If all boxes are filled without producing SOS then the game is a draw. Prove that the second player has a winning strategy. +6. Let $A B C D$ be an isosceles trapezoid with $A B \| C D$. The inscribed circle $\omega$ of triangle $B C D$ meets $C D$ at $E$. Let $F$ be a point on the (internal) angle bisector of $\angle D A C$ such that $E F \perp C D$. Let the circumscribed circle of triangle $A C F$ meet line $C D$ at $C$ and $G$. Prove that the triangle $A F G$ is isosceles. + +## §1 Solutions to Day 1 + +## §1.1 USAMO 1999/1 + +Available online at https://aops.com/community/p340035. + +## Problem statement + +Some checkers placed on an $n \times n$ checkerboard satisfy the following conditions: +(a) every square that does not contain a checker shares a side with one that does; +(b) given any pair of squares that contain checkers, there is a sequence of squares containing checkers, starting and ending with the given squares, such that every two consecutive squares of the sequence share a side. + +Prove that at least $\left(n^{2}-2\right) / 3$ checkers have been placed on the board. + +Take a spanning tree on the set $V$ of checkers where the $|V|-1$ edges of the tree are given by orthogonal adjacency. By condition (a) we have + +$$ +\sum_{v \in V}(4-\operatorname{deg} v) \geq n^{2}-|V| +$$ + +and since $\sum_{v \in V} \operatorname{deg} v=2(|V|-1)$ we get + +$$ +4|V|-(2|V|-2) \geq n^{2}-|V| +$$ + +which implies $|V| \geq \frac{n^{2}-2}{3}$. + +## §1.2 USAMO 1999/2 + +Available online at https://aops.com/community/p340036. + +## Problem statement + +Let $A B C D$ be a convex cyclic quadrilateral. Prove that + +$$ +|A B-C D|+|A D-B C| \geq 2|A C-B D| . +$$ + +Let the diagonals meet at $P$, and let $A P=p q, D P=p r, B P=q s, C P=r s$. Then set $A B=q x, C D=r x, A D=p y, B C=s y$. + +In this way we compute + +$$ +|A C-B D|=|(p-s)(q-r)| +$$ + +and + +$$ +|A B-C D|=|q-r| x +$$ + +By triangle inequality on $\triangle A X B$, we have $x \geq|p-s|$. So $|A B-C D| \geq|A C-B D|$. + +## §1.3 USAMO 1999/3 + +Available online at https://aops.com/community/p340038. + +## Problem statement + +Let $p>2$ be a prime and let $a, b, c, d$ be integers not divisible by $p$, such that + +$$ +\left\{\frac{r a}{p}\right\}+\left\{\frac{r b}{p}\right\}+\left\{\frac{r c}{p}\right\}+\left\{\frac{r d}{p}\right\}=2 +$$ + +for any integer $r$ not divisible by $p$. (Here, $\{t\}=t-\lfloor t\rfloor$ is the fractional part.) Prove that at least two of the numbers $a+b, a+c, a+d, b+c, b+d, c+d$ are divisible by $p$. + +First of all, we apparently have $r(a+b+c+d) \equiv 0(\bmod p)$ for every prime $p$, so it automatically follows that $a+b+c+d \equiv 0(\bmod p)$. By scaling appropriately, and also replacing each number with its remainder modulo $p$, we are going to assume that + +$$ +1=a \leq b \leq c \leq d

1$. Then evidently + +$$ +r-1>\left\lfloor\frac{r d}{p}\right\rfloor \geq\left\lfloor\frac{(r-1) d}{p}\right\rfloor=r-2 +$$ + +Now, we have that + +$$ +2(r-1)=\left\lfloor\frac{r b}{p}\right\rfloor+\left\lfloor\frac{r c}{p}\right\rfloor+\underbrace{\left\lfloor\frac{r d}{p}\right\rfloor}_{=r-2} . +$$ + +Thus $\left\lfloor\frac{r b}{p}\right\rfloor>\left\lfloor\frac{(r-1) b}{p}\right\rfloor$, and $\left\lfloor\frac{r c}{p}\right\rfloor>\left\lfloor\frac{(r-1) b}{p}\right\rfloor$. An example of this situation is illustrated below with $r=7$ (not to scale). +![](https://cdn.mathpix.com/cropped/2024_11_19_4e30714afd39bc4610deg-06.jpg?height=330&width=1129&top_left_y=1000&top_left_x=469) + +Right now, $\frac{b}{p}$ and $\frac{c}{p}$ are just to the right of $\frac{u}{r}$ and $\frac{v}{r}$ for some $u$ and $v$ with $u+v=r$. The issue is that the there is some fraction just to the right of $\frac{b}{p}$ and $\frac{c}{p}$ from an earlier value of $r$, and by hypothesis its denominator is going to be strictly greater than 1 . + +It is at this point we are going to use the properties of Farey sequences. When we consider the fractions with denominator $r+1$, they are going to lie outside of the interval they we have constrained $\frac{b}{p}$ and $\frac{c}{p}$ to lie in. + +Indeed, our minimality assumption on $r$ guarantees that there is no fraction with denominator less than $r$ between $\frac{u}{r}$ and $\frac{b}{p}$. So if $\frac{u}{r}<\frac{b}{p}<\frac{s}{t}$ (where $\frac{u}{r}$ and $\frac{s}{t}$ are the closest fractions with denominator at most $r$ to $\frac{b}{p}$ ) then Farey theory says the next fraction inside the interval $\left[\frac{u}{r}, \frac{s}{t}\right]$ is $\frac{u+s}{r+t}$, and since $t>1$, we have $r+t>r+1$. In other words, we get an inequality of the form + +$$ +\frac{u}{r}<\frac{b}{p}<\underbrace{\text { something }}_{=s / t} \leq \frac{u+1}{r+1} . +$$ + +The same holds for $\frac{c}{p}$ as + +$$ +\frac{v}{r}<\frac{c}{p}<\text { something } \leq \frac{v+1}{r+1} +$$ + +Finally, + +$$ +\frac{d}{p}<\frac{r-1}{r}<\frac{r}{r+1} +$$ + +So now we have that + +$$ +\left\lfloor\frac{(r+1) b}{p}\right\rfloor+\left\lfloor\frac{(r+1) c}{p}\right\rfloor+\left\lfloor\frac{(r+1) d}{p}\right\rfloor \leq u+v+(r-1)=2 r-1 +$$ + +which is a contradiction. + +Now, since + +$$ +\frac{p-3}{p-2}<\frac{d}{p} \Longrightarrow d>\frac{p(p-3)}{p-2}=p-1-\frac{2}{p-2} +$$ + +which for $p>2$ gives $d=p-1$. + +## §2 Solutions to Day 2 + +## §2.1 USAMO 1999/4 + +Available online at https://aops.com/community/p63591. + +## Problem statement + +Let $a_{1}, a_{2}, \ldots, a_{n}$ be a sequence of $n>3$ real numbers such that + +$$ +a_{1}+\cdots+a_{n} \geq n \quad \text { and } \quad a_{1}^{2}+\cdots+a_{n}^{2} \geq n^{2} +$$ + +Prove that $\max \left(a_{1}, \ldots, a_{n}\right) \geq 2$. + +Proceed by contradiction, assuming $a_{i}<2$ for all $i$. +If all $a_{i} \geq 0$, then $n^{2} \leq \sum_{i} a_{i}^{2}0)$, then we can replace them with $-(x+y)$ and 0 . So we may assume that there is exactly one negative term, say $a_{n}=-M$. + +Now, smooth all the nonnegative $a_{i}$ to be 2 , making all inequalities strict. Now, we have that + +$$ +\begin{aligned} +2(n-1)-M & >n \\ +4(n-1)+M^{2} & >n^{2} +\end{aligned} +$$ + +This gives $n-20$. + +## §1.2 USAMO 2000/2 + +Available online at https://aops.com/community/p338078. + +## Problem statement + +Let $S$ be the set of all triangles $A B C$ for which + +$$ +5\left(\frac{1}{A P}+\frac{1}{B Q}+\frac{1}{C R}\right)-\frac{3}{\min \{A P, B Q, C R\}}=\frac{6}{r} +$$ + +where $r$ is the inradius and $P, Q, R$ are the points of tangency of the incircle with sides $A B, B C, C A$ respectively. Prove that all triangles in $S$ are isosceles and similar to one another. + +We will prove the inequality + +$$ +\frac{2}{A P}+\frac{5}{B Q}+\frac{5}{C R} \geq \frac{6}{r} +$$ + +with equality when $A P: B Q: C R=1: 4: 4$. This implies the problem statement. +Letting $x=A P, y=B Q, z=C R$, the inequality becomes + +$$ +\frac{2}{x}+\frac{5}{y}+\frac{5}{z} \geq 6 \sqrt{\frac{x+y+z}{x y z}} . +$$ + +Squaring both sides and collecting terms gives + +$$ +\frac{4}{x^{2}}+\frac{25}{y^{2}}+\frac{25}{z^{2}}+\frac{14}{y z} \geq \frac{16}{x y}+\frac{16}{x z} +$$ + +If we replace $x=1 / a, y=4 / b, z=4 / c$, then it remains to prove the inequality + +$$ +64 a^{2}+25(b+c)^{2} \geq 64 a(b+c)+36 b c +$$ + +where equality holds when $a=b=c$. This follows by two applications of AM-GM: + +$$ +\begin{aligned} +16\left(4 a^{2}+(b+c)^{2}\right) & \geq 64 a(b+c) \\ +9(b+c)^{2} & \geq 36 b c . +\end{aligned} +$$ + +Again one can tell this is an inequality by counting degrees of freedom. + +## §1.3 USAMO 2000/3 + +Available online at https://aops.com/community/p338081. + +## Problem statement + +A game of solitaire is played with $R$ red cards, $W$ white cards, and $B$ blue cards. A player plays all the cards one at a time. With each play he accumulates a penalty. If he plays a blue card, then he is charged a penalty which is the number of white cards still in his hand. If he plays a white card, then he is charged a penalty which is twice the number of red cards still in his hand. If he plays a red card, then he is charged a penalty which is three times the number of blue cards still in his hand. + +Find, as a function of $R, W$, and $B$, the minimal total penalty a player can amass and the number of ways in which this minimum can be achieved. + +The minimum penalty is + +$$ +f(B, W, R)=\min (B W, 2 W R, 3 R B) +$$ + +or equivalently, the natural guess of "discard all cards of one color first" is actually optimal (though not necessarily unique). + +This can be proven directly by induction. Indeed the base case $B W R=0$ (in which case zero penalty is clearly achievable). The inductive step follows from + +$$ +f(B, W, R)=\min \left\{\begin{array}{l} +f(B-1, W, R)+W \\ +f(B, W-1, R)+2 R \\ +f(B, W, R-1)+3 B +\end{array}\right. +$$ + +It remains to characterize the strategies. This is an annoying calculation, so we just state the result. + +- If any of the three quantities $B W, 2 W R, 3 R B$ is strictly smaller than the other three, there is one optimal strategy. +- If $B W=2 W R<3 R B$, there are $W+1$ optimal strategies, namely discarding from 0 to $W$ white cards, then discarding all blue cards. (Each white card discarded still preserves $B W=2 W R$.) +- If $2 W R=3 R B1000$. Then obviously some column has more than two tokens, so at most 999 tokens don't emit a death ray (namely, any token in its own column). Thus there are at least $n-999$ death rays. On the other hand, we can have at most 999 death rays total (since it would not be okay for the whole board to have death rays, as some row should have more than two tokens). Therefore, $n \leq 999+999=1998$ as desired. + +## §2.2 USAMO 2000/5 + +Available online at https://aops.com/community/p338089. + +## Problem statement + +Let $A_{1} A_{2} A_{3}$ be a triangle, and let $\omega_{1}$ be a circle in its plane passing through $A_{1}$ and $A_{2}$. Suppose there exists circles $\omega_{2}, \omega_{3}, \ldots, \omega_{7}$ such that for $k=2,3, \ldots, 7$, circle $\omega_{k}$ is externally tangent to $\omega_{k-1}$ and passes through $A_{k}$ and $A_{k+1}$ (indices mod 3). Prove that $\omega_{7}=\omega_{1}$. + +The idea is to keep track of the subtended arc $\widehat{A_{i} A_{i+1}}$ of $\omega_{i}$ for each $i$. To this end, let $\beta=\measuredangle A_{1} A_{2} A_{3}, \gamma=\measuredangle A_{2} A_{3} A_{1}$ and $\alpha=\measuredangle A_{1} A_{2} A_{3}$. +![](https://cdn.mathpix.com/cropped/2024_11_19_c2e5e56d46560bd13f74g-7.jpg?height=795&width=812&top_left_y=910&top_left_x=625) + +Initially, we set $\theta=\measuredangle O_{1} A_{2} A_{1}$. Then we compute + +$$ +\begin{aligned} +& \measuredangle O_{1} A_{2} A_{1}=\theta \\ +& \measuredangle O_{2} A_{3} A_{2}=-\beta-\theta \\ +& \measuredangle O_{3} A_{1} A_{3}=\beta-\gamma+\theta \\ +& \measuredangle O_{4} A_{2} A_{1}=(\gamma-\beta-\alpha)-\theta +\end{aligned} +$$ + +and repeating the same calculation another round gives + +$$ +\measuredangle O_{7} A_{2} A_{1}=k-(k-\theta)=\theta +$$ + +with $k=\gamma-\beta-\alpha$. This implies $O_{7}=O_{1}$, so $\omega_{7}=\omega_{1}$. + +## §2.3 USAMO 2000/6, proposed by Gheorghita Zbaganu + +Available online at https://aops.com/community/p108437. + +## Problem statement + +Let $a_{1}, b_{1}, a_{2}, b_{2}, \ldots, a_{n}, b_{n}$ be nonnegative real numbers. Prove that + +$$ +\sum_{i, j=1}^{n} \min \left\{a_{i} a_{j}, b_{i} b_{j}\right\} \leq \sum_{i, j=1}^{n} \min \left\{a_{i} b_{j}, a_{j} b_{i}\right\} +$$ + +We present two solutions. +【 First solution by creating a single min (Vincent Huang and Ravi Boppana). Let $b_{i}=r_{i} a_{i}$ for each $i$, and rewrite the inequality as + +$$ +\sum_{i, j} a_{i} a_{j}\left[\min \left(r_{i}, r_{j}\right)-\min \left(1, r_{i} r_{j}\right)\right] \geq 0 +$$ + +We now do the key manipulation to convert the double min into a separate single min. Let $\varepsilon_{i}=+1$ if $r_{i} \geq 1$, and $\varepsilon_{i}=-1$ otherwise, and let $s_{i}=\left|r_{i}-1\right|$. Then we pass to absolute values: + +$$ +\begin{aligned} +2 \min \left(r_{i}, r_{j}\right)-2 \min \left(1, r_{i} r_{j}\right) & =\left|r_{i} r_{j}-1\right|-\left|r_{i}-r_{j}\right|-\left(r_{i}-1\right)\left(r_{j}-1\right) \\ +& =\left|r_{i} r_{j}-1\right|-\left|r_{i}-r_{j}\right|-\varepsilon_{i} \varepsilon_{j} s_{i} s_{j} \\ +& =\varepsilon_{i} \varepsilon_{j} \min \left(\left|1-r_{i} r_{j} \pm\left(r_{i}-r_{j}\right)\right|\right)-\varepsilon_{i} \varepsilon_{j} s_{i} s_{j} \\ +& =\varepsilon_{i} \varepsilon_{j} \min \left(s_{i}\left(r_{j}+1\right), s_{j}\left(r_{i}+1\right)\right)-\varepsilon_{i} \varepsilon_{j} s_{i} s_{j} \\ +& =\left(\varepsilon_{i} s_{i}\right)\left(\varepsilon_{j} s_{j}\right) \min \left(\frac{r_{j}+1}{s_{j}}-1, \frac{r_{i}+1}{s_{i}}-1\right) . +\end{aligned} +$$ + +So let us denote $x_{i}=a_{i} \varepsilon_{i} s_{i} \in \mathbb{R}$, and $t_{i}=\frac{r_{i}+1}{s_{i}}-1 \in \mathbb{R}_{\geq 0}$. Thus it suffices to prove that: + +Claim - We have + +$$ +\sum_{i, j} x_{i} x_{j} \min \left(t_{i}, t_{j}\right) \geq 0 +$$ + +for arbitrary $x_{i} \in \mathbb{R}, t_{i} \in \mathbb{R} \geq 0$. +Proof. One can just check this "by hand" by assuming $t_{1} \leq t_{2} \leq \cdots \leq t_{n}$; then the left-hand side becomes + +$$ +\sum_{i} t_{i} x_{i}^{2}+2 \sum_{i0 \text { and } a^{2}+b^{2}+c^{2}<1000\right\} . +$$ + +This is an intersection of open sets, so it is open. Its closure is + +$$ +\bar{U}=\left\{(a, b, c) \mid a, b, c \geq 0 \text { and } a^{2}+b^{2}+c^{2} \leq 1000\right\} . +$$ + +Hence the constraint set + +$$ +\bar{S}=\{\mathbf{x} \in \bar{U}: g(\bar{x})=4\} +$$ + +is compact, where $g(a, b, c)=a^{2}+b^{2}+c^{2}+a b c$. +Define + +$$ +f(a, b, c)=a^{2}+b^{2}+c^{2}+a b+b c+c a +$$ + +It's equivalent to show that $f \leq 6$ subject to $g$. Over $\bar{S}$, it must achieve a global maximum. Now we consider two cases. + +If $\mathbf{x}$ lies on the boundary, that means one of the components is zero (since $a^{2}+b^{2}+c^{2}=$ 1000 is clearly impossible). WLOG $c=0$, then we wish to show $a^{2}+b^{2}+a b \leq 6$ for $a^{2}+b^{2}=4$, which is trivial. + +Now for the interior $U$, we may use the method of Lagrange Multipliers. Consider a local maximum $\mathbf{x} \in U$. Compute + +$$ +\nabla f=\langle 2 a+b+c, 2 b+c+a, 2 c+a+b\rangle +$$ + +and + +$$ +\nabla g=\langle 2 a+b c, 2 b+c a, 2 c+a b\rangle +$$ + +Of course, $\nabla g \neq \mathbf{0}$ everywhere, so introducing our multiplier yields + +$$ +\langle 2 a+b+c, a+2 b+c, a+b+2 c\rangle=\lambda\langle 2 a+b c, 2 b+c a, 2 c+a b\rangle . +$$ + +Note that $\lambda \neq 0$ since $a, b, c>0$. Subtracting $2 a+b+c=\lambda(2 a+b c)$ from $a+2 b+c=$ $\lambda(2 b+c a)$ implies that + +$$ +(a-b)([2 \lambda-1]-\lambda c)=0 . +$$ + +We can derive similar equations for the others. Hence, we have three cases. + +1. If $a=b=c$, then $a=b=c=1$, and this satisfies $f(1,1,1) \leq 6$. +2. If $a, b, c$ are pairwise distinct, then we derive $a=b=c=2-\lambda^{-1}$, contradiction. +3. Now suppose that $a=b \neq c$. + +Meanwhile, the constraint (with $a=b$ ) reads + +$$ +\begin{aligned} +a^{2}+b^{2}+c^{2}+a b c=4 & \Longleftrightarrow c^{2}+a^{2} c+\left(2 a^{2}-4\right)=0 \\ +& \Longleftrightarrow(c+2)\left(c-\left(2-a^{2}\right)\right)=0 +\end{aligned} +$$ + +which since $c>0$ gives $c=2-a^{2}$. +Noah Walsh points out that at this point, we don't need to calculate the critical point; we just directly substitute $a=b$ and $c=2-a^{2}$ into the desired inequality: + +$$ +a^{2}+2 a(2-a)^{2}-a^{2}(2-a)^{2}=2-(a-1)^{2}\left(a^{2}-4 a+2\right) \leq 0 +$$ + +So any point here satisfies the inequality anyways. + +Remark. It can actually be shown that the critical point in the third case we skipped is pretty close: it is given by + +$$ +a=b=\frac{1+\sqrt{17}}{4} \quad c=\frac{1}{8}(7-\sqrt{17}) . +$$ + +This satisfies + +$$ +f(a, b, c)=3 a^{2}+2 a c+c^{2}=\frac{1}{32}(121+17 \sqrt{17}) \approx 5.97165 +$$ + +which is just a bit less than 6 . + +Remark. Equality holds for the upper bound if $(a, b, c)=(1,1,1)$ or $(a, b, c)=(\sqrt{2}, \sqrt{2}, 0)$ and permutations. The lower bound is achieved if $(a, b, c)=(2,0,0)$ and permutations. + +## §2 Solutions to Day 2 + +## §2.1 USAMO 2001/4 + +Available online at https://aops.com/community/p337872. + +## Problem statement + +Let $A B C$ be a triangle and $P$ any point such that $P A, P B, P C$ are the sides of an obtuse triangle, with $P A$ the longest side. Prove that $\angle B A C$ is acute. + +Using Ptolemy's inequality and Cauchy-Schwarz, + +$$ +\begin{aligned} +P A \cdot B C & \leq P B \cdot A C+P C \cdot A B \\ +& \leq \sqrt{\left(P B^{2}+P C^{2}\right)\left(A B^{2}+A C^{2}\right)} \\ +& <\sqrt{P A^{2} \cdot\left(A B^{2}+A C\right)^{2}}=P A \cdot \sqrt{A B^{2}+A C^{2}} +\end{aligned} +$$ + +meaning $B C^{2}B P^{2}+P C^{2}$, so $A B^{2}+A C^{2}>B C^{2}$, and we get $\angle B A C$ is acute. + +## §2.2 USAMO 2001/5 + +Available online at https://aops.com/community/p337875. + +## Problem statement + +Let $S \subseteq \mathbb{Z}$ be such that: +(a) there exist $a, b \in S$ with $\operatorname{gcd}(a, b)=\operatorname{gcd}(a-2, b-2)=1$; +(b) if $x$ and $y$ are elements of $S$ (possibly equal), then $x^{2}-y$ also belongs to $S$. + +Prove that $S=\mathbb{Z}$. + +Call an integer $d>0$ shifty if $S=S+d$ (meaning $S$ is invariant under shifting by $d$ ). +First, note that if $u, v \in S$, then for any $x \in S$, + +$$ +v^{2}-\left(u^{2}-x\right)=\left(v^{2}-u^{2}\right)+x \in S +$$ + +Since we can easily check that $|S|>1$ and $S \neq\{n,-n\}$ we conclude there exists a shifty integer. + +We claim 1 is shifty, which implies the problem. Assume for contradiction that 1 is not shifty. Then for GCD reasons the set of shifty integers must be $d \mathbb{Z}$ for some $d \geq 2$. + +Claim - We have $S \subseteq\left\{x: x^{2} \equiv m(\bmod d)\right\}$ for some fixed $m$. +Proof. Otherwise if we take any $p, q \in S$ with distinct squares modulo $d$, then $q^{2}-p^{2} \not \equiv 0$ $(\bmod d)$ is shifty, which is impossible. + +Now take $a, b \in S$ as in (a). In that case we need to have + +$$ +a^{2} \equiv b^{2} \equiv\left(a^{2}-a\right)^{2} \equiv\left(b^{2}-b\right)^{2} \quad(\bmod d) +$$ + +Passing to a prime $p \mid d$, we have the following: + +- Since $a^{2} \equiv\left(a^{2}-a\right)^{2}(\bmod p)$ or equivalently $a^{3}(a-2) \equiv 0(\bmod p)$, either $a \equiv 0$ $(\bmod p)$ or $a \equiv 2(\bmod p)$. +- Similarly, either $b \equiv 0(\bmod p)$ or $b \equiv 2(\bmod p)$. +- Since $a^{2} \equiv b^{2}(\bmod p)$, or $a \equiv \pm b(\bmod p)$, we find either $a \equiv b \equiv 0(\bmod p)$ or $a \equiv b \equiv 2(\bmod p)($ even if $p=2)$. + +This is a contradiction. +Remark. The condition (a) cannot be dropped, since otherwise we may take $S=\{2(\bmod p)\}$ or $S=\{0(\bmod p)\}$, say. + +## §2.3 USAMO 2001/6, proposed by Bjorn Poonen + +Available online at https://aops. com/community/p337877. + +## Problem statement + +Each point in the plane is assigned a real number. Suppose that for any nondegenerate triangle, the number at its incenter is the arithmetic mean of the three numbers at its vertices. Prove that all points in the plane are equal to each other. + +First, we claim that in an isosceles trapezoid $A B C D$ we have $a+c=b+d$. Indeed, suppose WLOG that rays $B A$ and $C D$ meet at $X$. Then triangles $X A C$ and $X B D$ share an incircle, proving the claim. + +Now, given any two points $A$ and $B$, construct regular pentagon $A B C D E$. We have $a+c=b+d=c+e=d+a=e+b$, so $a=b=c=d=e$. + diff --git a/USAMO/md/en-USAMO-2002-notes.md b/USAMO/md/en-USAMO-2002-notes.md new file mode 100644 index 0000000000000000000000000000000000000000..05f4626a1e7b73fcf12ba07e6af66868539c61d6 --- /dev/null +++ b/USAMO/md/en-USAMO-2002-notes.md @@ -0,0 +1,243 @@ +# USAMO 2002 Solution Notes + +Evan Chen《陳誼廷》 + +15 April 2024 + +This is a compilation of solutions for the 2002 USAMO. The ideas of the solution are a mix of my own work, the solutions provided by the competition organizers, and solutions found by the community. However, all the writing is maintained by me. + +These notes will tend to be a bit more advanced and terse than the "official" solutions from the organizers. In particular, if a theorem or technique is not known to beginners but is still considered "standard", then I often prefer to use this theory anyways, rather than try to work around or conceal it. For example, in geometry problems I typically use directed angles without further comment, rather than awkwardly work around configuration issues. Similarly, sentences like "let $\mathbb{R}$ denote the set of real numbers" are typically omitted entirely. + +Corrections and comments are welcome! + +## Contents + +0 Problems ..... 2 +1 Solutions to Day 1 ..... 3 +1.1 USAMO 2002/1, proposed by Gabriel Carroll ..... 3 +1.2 USAMO 2002/2 ..... 4 +1.3 USAMO 2002/3 +2 Solutions to Day 2 ..... 6 +2.1 USAMO 2002/4 ..... 6 +2.2 USAMO 2002/5, proposed by Gabriel Carroll ..... 7 +2.3 USAMO 2002/6 ..... 8 + +## §0 Problems + +1. Let $S$ be a set with 2002 elements, and let $N$ be an integer with $0 \leq N \leq 2^{2002}$. Prove that it is possible to color every subset of $S$ either black or white so that the following conditions hold: +(a) the union of any two white subsets is white; +(b) the union of any two black subsets is black; +(c) there are exactly $N$ white subsets. +2. Let $A B C$ be a triangle such that + +$$ +\left(\cot \frac{A}{2}\right)^{2}+\left(2 \cot \frac{B}{2}\right)^{2}+\left(3 \cot \frac{C}{2}\right)^{2}=\left(\frac{6 s}{7 r}\right)^{2} +$$ + +where $s$ and $r$ denote its semiperimeter and its inradius, respectively. Prove that triangle $A B C$ is similar to a triangle $T$ whose side lengths are all positive integers with no common divisors and determine these integers. +3. Prove that any monic polynomial (a polynomial with leading coefficient 1) of degree $n$ with real coefficients is the average of two monic polynomials of degree $n$ with $n$ real roots. +4. Determine all functions $f: \mathbb{R} \rightarrow \mathbb{R}$ such that + +$$ +f\left(x^{2}-y^{2}\right)=x f(x)-y f(y) +$$ + +for all pairs of real numbers $x$ and $y$. +5. Let $a, b$ be integers greater than 2. Prove that there exists a positive integer $k$ and a finite sequence $n_{1}, n_{2}, \ldots, n_{k}$ of positive integers such that $n_{1}=a, n_{k}=b$, and $n_{i} n_{i+1}$ is divisible by $n_{i}+n_{i+1}$ for each $i(1 \leq i0$. + +## §1 Solutions to Day 1 + +## §1.1 USAMO 2002/1, proposed by Gabriel Carroll + +Available online at https://aops.com/community/p337845. + +## Problem statement + +Let $S$ be a set with 2002 elements, and let $N$ be an integer with $0 \leq N \leq 2^{2002}$. Prove that it is possible to color every subset of $S$ either black or white so that the following conditions hold: +(a) the union of any two white subsets is white; +(b) the union of any two black subsets is black; +(c) there are exactly $N$ white subsets. + +We will solve the problem with 2002 replaced by an arbitrary integer $n \geq 0$. In other words, we prove: + +Claim - For any nonnegative integers $n$ and $N$ with $0 \leq N \leq 2^{n}$, it is possible to color the $2^{n}$ subsets of $\{1, \ldots, n\}$ black and white satisfying the conditions of the problem. + +The proof is by induction on $n$. When $n=1$ the problem is easy to do by hand, so this gives us a base case. + +For the inductive step, we divide into two cases: + +- If $N \leq 2^{n-1}$, then we take a coloring of subsets of $\{1, \ldots, n-1\}$ with $N$ white sets; then we color the remaining $2^{n-1}$ sets (which contain $n$ ) black. +- If $N>2^{n-1}$, then we take a coloring of subsets of $\{1, \ldots, n-1\}$ with $N-2^{n-1}$ white sets; then we color the remaining $2^{n-1}$ sets (which contain $n$ ) white. + + +## §1.2 USAMO 2002/2 + +Available online at https://aops.com/community/p337847. + +## Problem statement + +Let $A B C$ be a triangle such that + +$$ +\left(\cot \frac{A}{2}\right)^{2}+\left(2 \cot \frac{B}{2}\right)^{2}+\left(3 \cot \frac{C}{2}\right)^{2}=\left(\frac{6 s}{7 r}\right)^{2} +$$ + +where $s$ and $r$ denote its semiperimeter and its inradius, respectively. Prove that triangle $A B C$ is similar to a triangle $T$ whose side lengths are all positive integers with no common divisors and determine these integers. + +Let $x=s-a, y=s-b, z=s-c$ in the usual fashion, then the equation reads + +$$ +x^{2}+4 y^{2}+9 z^{2}=\left(\frac{6}{7}(x+y+z)\right)^{2} +$$ + +However, by Cauchy-Schwarz, we have + +$$ +\left(1+\frac{1}{4}+\frac{1}{9}\right)\left(x^{2}+4 y^{2}+9 z^{2}\right) \geq(x+y+z)^{2} +$$ + +with equality if and only if $1: \frac{1}{2}: \frac{1}{3}=x: 2 y: 3 z$, id est $x: y: z=1: \frac{1}{4}: \frac{1}{9}=36: 9: 4$. This is equivalent to $y+z: z+x: x+y=13: 40: 45$. + +Remark. You can tell this is not a geometry problem because you eliminate the cotangents right away to get an algebra problem...and then you realize the problem claims that one equation can determine three variables up to scaling, at which point you realize it has to be an inequality (otherwise degrees of freedom don't work). So of course, Cauchy-Schwarz... + +## §1.3 USAMO 2002/3 + +Available online at https://aops.com/community/p337849. + +## Problem statement + +Prove that any monic polynomial (a polynomial with leading coefficient 1) of degree $n$ with real coefficients is the average of two monic polynomials of degree $n$ with $n$ real roots. + +First, + +## Lemma + +If $p$ is a monic polynomial of degree $n$, and $p(1) p(2)<0, p(2) p(3)<0, \ldots$, $p(n-1) p(n)<0$ then $p$ has $n$ real roots. + +Proof. The intermediate value theorem already guarantees the existence of $n-1$ real roots. + +The last root is obtained by considering cases on $n(\bmod 2)$. + +- If $n$ is even, then $p(1)$ and $p(n)$ have opposite sign, while we must have either + +$$ +\lim _{x \rightarrow-\infty} p(x)=\lim _{x \rightarrow \infty} p(x)= \pm \infty +$$ + +so we get one more root. + +- The $n$ odd case is similar, with $p(1)$ and $p(n)$ now having the same sign, but $\lim _{x \rightarrow-\infty} p(x)=-\lim _{x \rightarrow \infty} p(x)$ instead. + +Let $f(n)$ be the monic polynomial and let $M>1000 \max _{t=1, \ldots, n}|f(t)|+1000$. Then we may select reals $a_{1}, \ldots, a_{n}$ and $b_{1}, \ldots, b_{n}$ such that for each $k=1, \ldots, n$, we have + +$$ +\begin{aligned} +a_{k}+b_{k} & =2 f(k) \\ +(-1)^{k} a_{k} & >M \\ +(-1)^{k+1} b_{k} & >M . +\end{aligned} +$$ + +We may interpolate monic polynomials $g$ and $h$ through the $a_{k}$ and $b_{k}$ (if the $a_{k}, b_{k}$ are selected "generically" from each other). Then one can easily check $f=\frac{1}{2}(g+h)$ works. + +## §2 Solutions to Day 2 + +## §2.1 USAMO 2002/4 + +Available online at https://aops.com/community/p337857. + +## Problem statement + +Determine all functions $f: \mathbb{R} \rightarrow \mathbb{R}$ such that + +$$ +f\left(x^{2}-y^{2}\right)=x f(x)-y f(y) +$$ + +for all pairs of real numbers $x$ and $y$. + +The answer is $f(x)=c x, c \in \mathbb{R}$ (these obviously work). +First, by putting $x=0$ and $y=0$ respectively we have + +$$ +f\left(x^{2}\right)=x f(x) \quad \text { and } \quad f\left(-y^{2}\right)=-y f(y) . +$$ + +From this we deduce that $f$ is odd, in particular $f(0)=0$. Then, we can rewrite the given as $f\left(x^{2}-y^{2}\right)+f\left(y^{2}\right)=f\left(x^{2}\right)$. Combined with the fact that $f$ is odd, we deduce that $f$ is additive (i.e. $f(a+b)=f(a)+f(b)$ ). + +Remark (Philosophy). At this point we have $f\left(x^{2}\right) \equiv x f(x)$ and $f$ additive, and everything we have including the given equation is a direct corollary of these two. So it makes sense to only focus on these two conditions. + +Then + +$$ +\begin{aligned} +f\left((x+1)^{2}\right) & =(x+1) f(x+1) \\ +\Longrightarrow f\left(x^{2}\right)+2 f(x)+f(1) & =(x+1) f(x)+(x+1) f(1) +\end{aligned} +$$ + +which readily gives $f(x)=f(1) x$. + +## §2.2 USAMO 2002/5, proposed by Gabriel Carroll + +Available online at https://aops.com/community/p337862. + +## Problem statement + +Let $a, b$ be integers greater than 2 . Prove that there exists a positive integer $k$ and a finite sequence $n_{1}, n_{2}, \ldots, n_{k}$ of positive integers such that $n_{1}=a, n_{k}=b$, and $n_{i} n_{i+1}$ is divisible by $n_{i}+n_{i+1}$ for each $i(1 \leq i2, n$ ! is connected to $(n+1)$ ! too: + +- $n!\rightarrow(n+1)$ ! if $n$ is even +- $n!\rightarrow 2 n!\rightarrow(n+1)!$ if $n$ is odd. + +This concludes the problem. + +## §2.3 USAMO 2002/6 + +Available online at https://aops.com/community/p337852. + +## Problem statement + +I have an $n \times n$ sheet of stamps, from which I've been asked to tear out blocks of three adjacent stamps in a single row or column. (I can only tear along the perforations separating adjacent stamps, and each block must come out of the sheet in one piece.) Let $b(n)$ be the smallest number of blocks I can tear out and make it impossible to tear out any more blocks. Prove that there are real constants $c$ and $d$ such that + +$$ +\frac{1}{7} n^{2}-c n \leq b(n) \leq \frac{1}{5} n^{2}+d n +$$ + +for all $n>0$. + +For the lower bound: there are $2 n(n-2)$ places one could put a block. Note that each block eliminates at most 14 such places. + +For the upper bound, the construction of $\frac{1}{5}$ is easy to build. Here is an illustration of one possible construction for $n=9$ which generalizes readily, using only vertical blocks. + +$$ +\left[\begin{array}{lllllllll} +A & & E & & I & L & & P & \\ +A & & E & G & & L & & P & R \\ +A & C & & G & & L & N & & R \\ +& C & & G & J & & N & & R \\ +& C & F & & J & & N & Q & \\ +B & & F & & J & M & & Q & \\ +B & & F & H & & M & & Q & S \\ +B & D & & H & & M & O & & S \\ +& D & & H & K & & O & & S +\end{array}\right] +$$ + +Actually, for the lower bound, one may improve $1 / 7$ to $1 / 6$. Count the number $A$ of pairs of adjacent squares one of which is torn out and the other which is not: + +- For every deleted block, there are eight neighboring squares, at least two on each long edge which have been deleted too. Hence $N \leq 6 b(n)$. +- For every block still alive and not on the border, there are four neighboring squares, and clearly at least two are deleted. Hence $N \geq 2\left((n-2)^{2}-3 b(n)\right)$. + +Collating these solves the problem. + diff --git a/USAMO/md/en-USAMO-2003-notes.md b/USAMO/md/en-USAMO-2003-notes.md new file mode 100644 index 0000000000000000000000000000000000000000..eaeaf246e7bdfa09abe268df1e42dbc8b2332082 --- /dev/null +++ b/USAMO/md/en-USAMO-2003-notes.md @@ -0,0 +1,276 @@ +# USAMO 2003 Solution Notes + +Evan Chen《陳誼廷》 + +15 April 2024 + +This is a compilation of solutions for the 2003 USAMO. The ideas of the solution are a mix of my own work, the solutions provided by the competition organizers, and solutions found by the community. However, all the writing is maintained by me. + +These notes will tend to be a bit more advanced and terse than the "official" solutions from the organizers. In particular, if a theorem or technique is not known to beginners but is still considered "standard", then I often prefer to use this theory anyways, rather than try to work around or conceal it. For example, in geometry problems I typically use directed angles without further comment, rather than awkwardly work around configuration issues. Similarly, sentences like "let $\mathbb{R}$ denote the set of real numbers" are typically omitted entirely. + +Corrections and comments are welcome! + +## Contents + +0 Problems ..... 2 +1 Solutions to Day 1 ..... 3 +1.1 USAMO 2003/1, proposed by Titu Andreescu ..... 3 +1.2 USAMO 2003/2 ..... 4 +1.3 USAMO 2003/3 ..... 6 +2 Solutions to Day 2 ..... 7 +2.1 USAMO 2003/4, proposed by Titu Andreescu, Zuming Feng ..... 7 +2.2 USAMO 2003/5, proposed by Zuming Feng, Titu Andreescu ..... 8 +2.3 USAMO 2003/6 ..... 9 + +## §0 Problems + +1. Prove that for every positive integer $n$ there exists an $n$-digit number divisible by $5^{n}$ all of whose digits are odd. +2. A convex polygon $\mathcal{P}$ in the plane is dissected into smaller convex polygons by drawing all of its diagonals. The lengths of all sides and all diagonals of the polygon $\mathcal{P}$ are rational numbers. Prove that the lengths of all sides of all polygons in the dissection are also rational numbers. +3. Let $n$ be a positive integer. For every sequence of integers + +$$ +A=\left(a_{0}, a_{1}, a_{2}, \ldots, a_{n}\right) +$$ + +satisfying $0 \leq a_{i} \leq i$, for $i=0, \ldots, n$, we define another sequence + +$$ +t(A)=\left(t\left(a_{0}\right), t\left(a_{1}\right), t\left(a_{2}\right), \ldots, t\left(a_{n}\right)\right) +$$ + +by setting $t\left(a_{i}\right)$ to be the number of terms in the sequence $A$ that precede the term $a_{i}$ and are different from $a_{i}$. Show that, starting from any sequence $A$ as above, fewer than $n$ applications of the transformation $t$ lead to a sequence $B$ such that $t(B)=B$. +4. Let $A B C$ be a triangle. A circle passing through $A$ and $B$ intersects segments $A C$ and $B C$ at $D$ and $E$, respectively. Lines $A B$ and $D E$ intersect at $F$, while lines $B D$ and $C F$ intersect at $M$. Prove that $M F=M C$ if and only if $M B \cdot M D=M C^{2}$. +5. Let $a, b, c$ be positive real numbers. Prove that + +$$ +\frac{(2 a+b+c)^{2}}{2 a^{2}+(b+c)^{2}}+\frac{(2 b+c+a)^{2}}{2 b^{2}+(c+a)^{2}}+\frac{(2 c+a+b)^{2}}{2 c^{2}+(a+b)^{2}} \leq 8 +$$ + +6. At the vertices of a regular hexagon are written six nonnegative integers whose sum is $2003^{2003}$. Bert is allowed to make moves of the following form: he may pick a vertex and replace the number written there by the absolute value of the difference between the numbers written at the two neighboring vertices. Prove that Bert can make a sequence of moves, after which the number 0 appears at all six vertices. + +## §1 Solutions to Day 1 + +## §1.1 USAMO 2003/1, proposed by Titu Andreescu + +Available online at https://aops.com/community/p336189. + +## Problem statement + +Prove that for every positive integer $n$ there exists an $n$-digit number divisible by $5^{n}$ all of whose digits are odd. + +This is immediate by induction on $n$. For $n=1$ we take 5 ; moving forward if $M$ is a working $n$-digit number then exactly one of + +$$ +\begin{aligned} +& N_{1}=10^{n}+M \\ +& N_{3}=3 \cdot 10^{n}+M \\ +& N_{5}=5 \cdot 10^{n}+M \\ +& N_{7}=7 \cdot 10^{n}+M \\ +& N_{9}=9 \cdot 10^{n}+M +\end{aligned} +$$ + +is divisible by $5^{n+1}$; as they are all divisible by $5^{n}$ and $N_{k} / 5^{n}$ are all distinct. + +## §1.2 USAMO 2003/2 + +Available online at https://aops.com/community/p336193. + +## Problem statement + +A convex polygon $\mathcal{P}$ in the plane is dissected into smaller convex polygons by drawing all of its diagonals. The lengths of all sides and all diagonals of the polygon $\mathcal{P}$ are rational numbers. Prove that the lengths of all sides of all polygons in the dissection are also rational numbers. + +Suppose $A B$ is a side of a polygon in the dissection, lying on diagonal $X Y$, with $X, A$, $B, Y$ in that order. Then + +$$ +A B=X Y-X A-Y B +$$ + +In this way, we see that it actually just suffices to prove the result for a quadrilateral. +We present two approaches to this end. +【 First approach (trig). Consider quadrilateral $A B C D$. There are twelve angles one can obtain using three of its four vertices, three at each vertex; denote this set of 12 angles by $S$ Note that: + +- The law of cosines implies $\cos \theta \in \mathbb{Q}$ for each $\theta \in S$. +- Hence, $(\sin \theta)^{2} \in \mathbb{Q}$ for $\theta \in S$. (This is because $\sin \theta^{2}+\cos ^{2} \theta$.) + +We say two angles $\theta_{1}$ and $\theta_{2}$ are equivalent if $\frac{\sin \theta_{1}}{\sin \theta_{2}}$ This is the same as saying, when $\sin \theta_{1}$ and $\sin \theta_{2}$ are written in simplest radical form, the part under the square root is the same. + +Now we contend: +Claim - The angles $\angle B A C, \angle C A D, \angle B A D$ are equivalent. +Proof. Note that + +$$ +\mathbb{Q} \ni \cos (\angle B A D)=\cos \angle B A C \cos \angle C A D-\sin \angle B A C \sin \angle C A D +$$ + +so $\angle B A C$ and $\angle C A D$ are equivalent. Then + +$$ +\sin (\angle B A D)=\sin \angle B A C \cos \angle C A D+\cos \angle B A C \sin \angle C A D +$$ + +implies $\angle B A D$ is equivalent to those two. + +Claim - The angles $\angle B A D, \angle D B A, \angle A D B$ are equivalent. +Proof. Law of sines on $\triangle B A D$. +Iterating the argument implies that all angles are equivalent. +Now, if $A B$ and $C D$ meet at $E$, the law of sines on $\triangle A E B$, etc. implies the result. + +I Second approach (barycentric coordinates). To do this, we apply barycentric coordinates. Consider quadrilateral $A B D C$ (note the changed order of vertices), with $A=(1,0,0), B=(0,1,0), C=(0,0,1)$. Let $D=(x, y, z)$, with $x+y+z=1$. By hypothesis, each of the numbers + +$$ +\begin{aligned} +-a^{2} y z+b^{2}(1-x) z+c^{2}(1-x) y & =A D^{2} \\ +a^{2}(1-y) z+b^{2} z x+c^{2}(1-y) x & =B D^{2} \\ +-a^{2}(1-z) y-b^{2}(1-z) x+c^{2} x y & =C D^{2} +\end{aligned} +$$ + +is rational. Let $W=a^{2} y z+b^{2} z x+c^{2} x y$. Then, + +$$ +\begin{aligned} +b^{2} z+c^{2} y & =A D^{2}+W \\ +a^{2} z+c^{2} x & =B D^{2}+W \\ +a^{2} y+b^{2} x & =C D^{2}+W +\end{aligned} +$$ + +This implies that $A D^{2}+B D^{2}+2 W-c^{2}=2 S_{C} z$ and cyclically (as usual $2 S_{C}=a^{2}+b^{2}-c^{2}$ ). If any of $S_{A}, S_{B}, S_{C}$ are zero, then we deduce $W$ is rational. Otherwise, we have that + +$$ +1=x+y+z=\sum_{\mathrm{cyc}} \frac{A D^{2}+B D^{2}+2 W-c^{2}}{2 S_{C}} +$$ + +which implies that $W$ is rational, because it appears with coefficient $\frac{1}{S_{A}}+\frac{1}{S_{B}}+\frac{1}{S_{C}} \neq 0$ (since $S_{B C}+S_{C A}+S_{A B}$ is actually the area of $A B C$ ). + +Hence from the rationality of $W$, we deduce that $x$ is rational as long as $S_{A} \neq 0$, and similarly for the others. So at most one of $x, y, z$ is irrational, but since $x+y+z=1$ this implies they are all rational. + +Finally, if $P=\overline{A D} \cap \overline{B C}$ then $A P=\frac{1}{y+z} A D$, so $A P$ is rational too, completing the proof. + +Remark. After the reduction to quadrilateral, a third alternate approach goes by quoting Putnam 2018 A6, reproduced below: + +Four points are given in the plane, with no three collinear, such that the squares of the $\binom{4}{2}=6$ pairwise distances are all rational. Show that the ratio of the areas between any two of the $\binom{4}{3}=4$ triangles determined by these points is also rational. +If $A B C D$ is the quadrilateral, the heights from $C$ and $D$ to $A B$ have rational ratio. Letting $P=A C \cap B D$, we see $A P / A B$ can be shown as rational via coordinates, as needed. + +## §1.3 USAMO 2003/3 + +Available online at https://aops.com/community/p336202. + +## Problem statement + +Let $n$ be a positive integer. For every sequence of integers + +$$ +A=\left(a_{0}, a_{1}, a_{2}, \ldots, a_{n}\right) +$$ + +satisfying $0 \leq a_{i} \leq i$, for $i=0, \ldots, n$, we define another sequence + +$$ +t(A)=\left(t\left(a_{0}\right), t\left(a_{1}\right), t\left(a_{2}\right), \ldots, t\left(a_{n}\right)\right) +$$ + +by setting $t\left(a_{i}\right)$ to be the number of terms in the sequence $A$ that precede the term $a_{i}$ and are different from $a_{i}$. Show that, starting from any sequence $A$ as above, fewer than $n$ applications of the transformation $t$ lead to a sequence $B$ such that $t(B)=B$. + +We go by strong induction on $n$ with the base cases $n=1$ and $n=2$ done by hand. Consider two cases: + +- If $a_{0}=0$ and $a_{1}=1$, then $1 \leq t\left(a_{i}\right) \leq i$ for $i \geq 1$; now apply induction to + +$$ +\left(t\left(a_{1}\right)-1, t\left(a_{2}\right)-1, \ldots, t\left(a_{n}\right)-1\right) +$$ + +- Otherwise, assume that $a_{0}=a_{1}=\cdots=a_{k-1}=0$ but $a_{k} \neq 0$, where $k \geq 2$. Assume $k1$ for which $a+b$ divides $a^{b}+b^{a}$. +2. Let $m_{1}, m_{2}, \ldots, m_{n}$ be a collection of $n$ positive integers, not necessarily distinct. For any sequence of integers $A=\left(a_{1}, \ldots, a_{n}\right)$ and any permutation $w=w_{1}, \ldots, w_{n}$ of $m_{1}, \ldots, m_{n}$, define an $A$-inversion of $w$ to be a pair of entries $w_{i}, w_{j}$ with $iw_{j}$, +- $w_{j}>a_{i} \geq w_{i}$, or +- $w_{i}>w_{j}>a_{i}$. + +Show that, for any two sequences of integers $A=\left(a_{1}, \ldots, a_{n}\right)$ and $B=\left(b_{1}, \ldots, b_{n}\right)$, and for any positive integer $k$, the number of permutations of $m_{1}, \ldots, m_{n}$ having exactly $k A$-inversions is equal to the number of permutations of $m_{1}, \ldots, m_{n}$ having exactly $k B$-inversions. +3. Let $A B C$ be a scalene triangle with circumcircle $\Omega$ and incenter $I$. Ray $A I$ meets $\overline{B C}$ at $D$ and $\Omega$ again at $M$; the circle with diameter $\overline{D M}$ cuts $\Omega$ again at $K$. Lines $M K$ and $B C$ meet at $S$, and $N$ is the midpoint of $\overline{I S}$. The circumcircles of $\triangle K I D$ and $\triangle M A N$ intersect at points $L_{1}$ and $L_{2}$. Prove that $\Omega$ passes through the midpoint of either $\overline{I L_{1}}$ or $\overline{I L_{2}}$. +4. Let $P_{1}, P_{2}, \ldots, P_{2 n}$ be $2 n$ distinct points on the unit circle $x^{2}+y^{2}=1$, other than $(1,0)$. Each point is colored either red or blue, with exactly $n$ red points and $n$ blue points. Let $R_{1}, R_{2}, \ldots, R_{n}$ be any ordering of the red points. Let $B_{1}$ be the nearest blue point to $R_{1}$ traveling counterclockwise around the circle starting from $R_{1}$. Then let $B_{2}$ be the nearest of the remaining blue points to $R_{2}$ travelling counterclockwise around the circle from $R_{2}$, and so on, until we have labeled all of the blue points $B_{1}, \ldots, B_{n}$. Show that the number of counterclockwise arcs of the form $R_{i} \rightarrow B_{i}$ that contain the point $(1,0)$ is independent of the way we chose the ordering $R_{1}, \ldots, R_{n}$ of the red points. +5. Find all real numbers $c>0$ such that there exists a labeling of the lattice points in $\mathbb{Z}^{2}$ with positive integers for which: + +- only finitely many distinct labels occur, and +- for each label $i$, the distance between any two points labeled $i$ is at least $c^{i}$. + +6. Find the minimum possible value of + +$$ +\frac{a}{b^{3}+4}+\frac{b}{c^{3}+4}+\frac{c}{d^{3}+4}+\frac{d}{a^{3}+4} +$$ + +given that $a, b, c, d$ are nonnegative real numbers such that $a+b+c+d=4$. + +## §1 Solutions to Day 1 + +## §1.1 USAMO 2017/1, proposed by Gregory Galperin + +Available online at https://aops.com/community/p8108366. + +## Problem statement + +Prove that there exist infinitely many pairs of relatively prime positive integers $a, b>1$ for which $a+b$ divides $a^{b}+b^{a}$. + +One construction: let $d \equiv 1(\bmod 4), d>1$. Let $x=\frac{d^{d}+2^{d}}{d+2}$. Then set + +$$ +a=\frac{x+d}{2}, \quad b=\frac{x-d}{2} . +$$ + +To see this works, first check that $b$ is odd and $a$ is even. Let $d=a-b$ be odd. Then: + +$$ +\begin{aligned} +a+b \mid a^{b}+b^{a} & \Longleftrightarrow(-b)^{b}+b^{a} \equiv 0 \quad(\bmod a+b) \\ +& \Longleftrightarrow b^{a-b} \equiv 1 \quad(\bmod a+b) \\ +& \Longleftrightarrow b^{d} \equiv 1 \quad(\bmod d+2 b) \\ +& \Longleftrightarrow(-2)^{d} \equiv d^{d}(\bmod d+2 b) \\ +& \Longleftrightarrow d+2 b \mid d^{d}+2^{d} . +\end{aligned} +$$ + +So it would be enough that + +$$ +d+2 b=\frac{d^{d}+2^{d}}{d+2} \Longrightarrow b=\frac{1}{2}\left(\frac{d^{d}+2^{d}}{d+2}-d\right) +$$ + +which is what we constructed. Also, since $\operatorname{gcd}(x, d)=1$ it follows $\operatorname{gcd}(a, b)=\operatorname{gcd}(d, b)=$ 1. + +Remark. Ryan Kim points out that in fact, $(a, b)=(2 n-1,2 n+1)$ is always a solution. + +## §1.2 USAMO 2017/2, proposed by Maria Monks + +Available online at https://aops.com/community/p8108658. + +## Problem statement + +Let $m_{1}, m_{2}, \ldots, m_{n}$ be a collection of $n$ positive integers, not necessarily distinct. For any sequence of integers $A=\left(a_{1}, \ldots, a_{n}\right)$ and any permutation $w=w_{1}, \ldots, w_{n}$ of $m_{1}, \ldots, m_{n}$, define an $A$-inversion of $w$ to be a pair of entries $w_{i}, w_{j}$ with $iw_{j}$, +- $w_{j}>a_{i} \geq w_{i}$, or +- $w_{i}>w_{j}>a_{i}$. + +Show that, for any two sequences of integers $A=\left(a_{1}, \ldots, a_{n}\right)$ and $B=\left(b_{1}, \ldots, b_{n}\right)$, and for any positive integer $k$, the number of permutations of $m_{1}, \ldots, m_{n}$ having exactly $k A$-inversions is equal to the number of permutations of $m_{1}, \ldots, m_{n}$ having exactly $k B$-inversions. + +The following solution was posted by Michael Ren, and I think it is the most natural one (since it captures all the combinatorial ideas using a $q$-generating function that is easier to think about, and thus makes the problem essentially a long computation). + +Denote by $M$ our multiset of $n$ positive integers. Define an inversion of a permutation to be pair $i0$ such that there exists a labeling of the lattice points in $\mathbb{Z}^{2}$ with positive integers for which: + +- only finitely many distinct labels occur, and +- for each label $i$, the distance between any two points labeled $i$ is at least $c^{i}$. + +The answer is $c<\sqrt{2}$. Here is a solution with Calvin Deng. +The construction for any $c<\sqrt{2}$ can be done as follows. Checkerboard color the lattice points and label the black ones with 1 . The white points then form a copy of $\mathbb{Z}^{2}$ again scaled up by $\sqrt{2}$ so we can repeat the procedure with 2 on half the resulting points. Continue this dyadic construction until a large $N$ for which $c^{N}<2^{\frac{1}{2}(N-1)}$, at which point we can just label all the points with $N$. + +I'll now prove that $c=\sqrt{2}$ (and hence $c \geq \sqrt{2}$ ) can't be done. +Claim - It is impossible to fill a $2^{n} \times 2^{n}$ square with labels not exceeding $2 n$. +The case $n=1$ is clear. So now assume it's true up to $n-1$; and assume for contradiction a $2^{n} \times 2^{n}$ square $S$ only contains labels up to $2 n$. (Of course every $2^{n-1} \times 2^{n-1}$ square contains an instance of a label at least $2 n-1$.) +![](https://cdn.mathpix.com/cropped/2024_11_19_31dda431671476b0de3cg-12.jpg?height=647&width=632&top_left_y=1484&top_left_x=712) + +Now, we contend there are fewer than four copies of $2 n$ : + +## Lemma + +In a unit square, among any four points, two of these points have distance $\leq 1$ apart. + +Proof. Look at the four rays emanating from the origin and note that two of them have included angle $\leq 90^{\circ}$. + +So WLOG the northwest quadrant has no $2 n$ 's. Take a $2 n-1$ in the northwest and draw a square of size $2^{n-1} \times 2^{n-1}$ directly right of it (with its top edge coinciding with the top of $S$ ). Then $A$ can't contain $2 n-1$, so it must contain a $2 n$ label; that $2 n$ label must be in the northeast quadrant. + +Then we define a square $B$ of size $2^{n-1} \times 2^{n-1}$ as follows. If $2 n-1$ is at least as high $2 n$, let $B$ be a $2^{n-1} \times 2^{n-1}$ square which touches $2 n-1$ north and is bounded east by $2 n$. Otherwise let $B$ be the square that touches $2 n-1$ west and is bounded north by $2 n$. We then observe $B$ can neither have $2 n-1$ nor $2 n$ in it, contradiction. + +Remark. To my knowledge, essentially all density arguments fail because of hexagonal lattice packing. + +## §2.3 USAMO 2017/6, proposed by Titu Andreescu + +Available online at https://aops.com/community/p8117097. + +## Problem statement + +Find the minimum possible value of + +$$ +\frac{a}{b^{3}+4}+\frac{b}{c^{3}+4}+\frac{c}{d^{3}+4}+\frac{d}{a^{3}+4} +$$ + +given that $a, b, c, d$ are nonnegative real numbers such that $a+b+c+d=4$. + +The minimum $\frac{2}{3}$ is achieved at $(a, b, c, d)=(2,2,0,0)$ and cyclic permutations. +The problem is an application of the tangent line trick: we observe the miraculous identity + +$$ +\frac{1}{b^{3}+4} \geq \frac{1}{4}-\frac{b}{12} +$$ + +since $12-(3-b)\left(b^{3}+4\right)=b(b+1)(b-2)^{2} \geq 0$. Moreover, + +$$ +a b+b c+c d+d a=(a+c)(b+d) \leq\left(\frac{(a+c)+(b+d)}{2}\right)^{2}=4 +$$ + +Thus + +$$ +\sum_{\mathrm{cyc}} \frac{a}{b^{3}+4} \geq \frac{a+b+c+d}{4}-\frac{a b+b c+c d+d a}{12} \geq 1-\frac{1}{3}=\frac{2}{3} +$$ + +Remark. The main interesting bit is the equality at $(a, b, c, d)=(2,2,0,0)$. This is the main motivation for trying tangent line trick, since a lower bound of the form $\sum a(1-\lambda b)$ preserves the unusual equality case above. Thus one takes the tangent at $b=2$ which miraculously passes through the point $(0,1 / 4)$ as well. + diff --git a/USAMO/md/en-USAMO-2018-notes.md b/USAMO/md/en-USAMO-2018-notes.md new file mode 100644 index 0000000000000000000000000000000000000000..e26cc85c914889df390272c821ae024c8ba969d3 --- /dev/null +++ b/USAMO/md/en-USAMO-2018-notes.md @@ -0,0 +1,392 @@ +# USAMO 2018 Solution Notes + +Evan ChEn《陳誼廷》 + +23 April 2024 + +This is a compilation of solutions for the 2018 USAMO. The ideas of the solution are a mix of my own work, the solutions provided by the competition organizers, and solutions found by the community. However, all the writing is maintained by me. + +These notes will tend to be a bit more advanced and terse than the "official" solutions from the organizers. In particular, if a theorem or technique is not known to beginners but is still considered "standard", then I often prefer to use this theory anyways, rather than try to work around or conceal it. For example, in geometry problems I typically use directed angles without further comment, rather than awkwardly work around configuration issues. Similarly, sentences like "let $\mathbb{R}$ denote the set of real numbers" are typically omitted entirely. + +Corrections and comments are welcome! + +## Contents + +0 Problems ..... 2 +1 Solutions to Day 1 ..... 3 +1.1 USAMO 2018/1, proposed by Titu Andreescu ..... 3 +1.2 USAMO 2018/2, proposed by Titu Andreescu, Nikolai Nikolov ..... 4 +1.3 USAMO 2018/3, proposed by Ivan Borsenco ..... 6 +2 Solutions to Day 2 ..... 9 +2.1 USAMO 2018/4, proposed by Ankan Bhattacharya ..... 9 +2.2 USAMO 2018/5, proposed by Kada Williams ..... 10 +2.3 USAMO 2018/6, proposed by Richard Stong ..... 13 + +## §0 Problems + +1. Let $a, b, c$ be positive real numbers such that $a+b+c=4 \sqrt[3]{a b c}$. Prove that + +$$ +2(a b+b c+c a)+4 \min \left(a^{2}, b^{2}, c^{2}\right) \geq a^{2}+b^{2}+c^{2} . +$$ + +2. Find all functions $f:(0, \infty) \rightarrow(0, \infty)$ such that + +$$ +f\left(x+\frac{1}{y}\right)+f\left(y+\frac{1}{z}\right)+f\left(z+\frac{1}{x}\right)=1 +$$ + +for all $x, y, z>0$ with $x y z=1$. +3. Let $n \geq 2$ be an integer, and let $\left\{a_{1}, \ldots, a_{m}\right\}$ denote the $m=\varphi(n)$ integers less than $n$ and relatively prime to $n$. Assume that every prime divisor of $m$ also divides $n$. Prove that $m$ divides $a_{1}^{k}+\cdots+a_{m}^{k}$ for every positive integer $k$. +4. Let $p$ be a prime, and let $a_{1}, \ldots, a_{p}$ be integers. Show that there exists an integer $k$ such that the numbers + +$$ +a_{1}+k, a_{2}+2 k, \ldots, a_{p}+p k +$$ + +produce at least $\frac{1}{2} p$ distinct remainders upon division by $p$. +5. Let $A B C D$ be a convex cyclic quadrilateral with $E=\overline{A C} \cap \overline{B D}, F=\overline{A B} \cap \overline{C D}$, $G=\overline{D A} \cap \overline{B C}$. The circumcircle of $\triangle A B E$ intersects line $C B$ at $B$ and $P$, and the circumcircle of $\triangle A D E$ intersects line $C D$ at $D$ and $Q$. Assume $C, B, P, G$ and $C$, $Q, D, F$ are collinear in that order. Let $M=\overline{F P} \cap \overline{G Q}$. Prove that $\angle M A C=90^{\circ}$. +6. Let $a_{n}$ be the number of permutations $\left(x_{1}, \ldots, x_{n}\right)$ of $(1, \ldots, n)$ such that the ratios $x_{k} / k$ are all distinct. Prove that $a_{n}$ is odd for all $n \geq 1$. + +## §1 Solutions to Day 1 + +## §1.1 USAMO 2018/1, proposed by Titu Andreescu + +Available online at https://aops.com/community/p10226140. + +## Problem statement + +Let $a, b, c$ be positive real numbers such that $a+b+c=4 \sqrt[3]{a b c}$. Prove that + +$$ +2(a b+b c+c a)+4 \min \left(a^{2}, b^{2}, c^{2}\right) \geq a^{2}+b^{2}+c^{2} +$$ + +WLOG let $c=\min (a, b, c)=1$ by scaling. The given inequality becomes equivalent to + +$$ +4 a b+2 a+2 b+3 \geq(a+b)^{2} \quad \forall a+b=4(a b)^{1 / 3}-1 +$$ + +Now, let $t=(a b)^{1 / 3}$ and eliminate $a+b$ using the condition, to get + +$$ +4 t^{3}+2(4 t-1)+3 \geq(4 t-1)^{2} \Longleftrightarrow 0 \leq 4 t^{3}-16 t^{2}+16 t=4 t(t-2)^{2} +$$ + +which solves the problem. +Equality occurs only if $t=2$, meaning $a b=8$ and $a+b=7$, which gives + +$$ +\{a, b\}=\left\{\frac{7 \pm \sqrt{17}}{2}\right\} +$$ + +with the assumption $c=1$. Scaling gives the curve of equality cases. + +## §1.2 USAMO 2018/2, proposed by Titu Andreescu, Nikolai Nikolov + +Available online at https://aops.com/community/p10226145. + +## Problem statement + +Find all functions $f:(0, \infty) \rightarrow(0, \infty)$ such that + +$$ +f\left(x+\frac{1}{y}\right)+f\left(y+\frac{1}{z}\right)+f\left(z+\frac{1}{x}\right)=1 +$$ + +for all $x, y, z>0$ with $x y z=1$. + +The main part of the problem is to show all solutions are linear. As always, let $x=b / c$, $y=c / a, z=a / b$ (classical inequality trick). Then the problem becomes + +$$ +\sum_{\mathrm{cyc}} f\left(\frac{b+c}{a}\right)=1 +$$ + +Let $f(t)=g\left(\frac{1}{t+1}\right)$, equivalently $g(s)=f(1 / s-1)$. Thus $g:(0,1) \rightarrow(0,1)$ which satisfies $\sum_{\mathrm{cyc}} g\left(\frac{a}{a+b+c}\right)=1$, or equivalently + +$$ +g(a)+g(b)+g(c)=1 \quad \forall a+b+c=1 . +$$ + +The rest of the solution is dedicated to solving this equivalent functional equation in $g$. It is a lot of technical details and I will only outline them (with apologies to the contestants who didn't have that luxury). + +Claim - The function $g$ is linear. +Proof. This takes several steps, all of which are technical. We begin by proving $g$ is linear over $[1 / 8,3 / 8]$. + +- First, whenever $a+b \leq 1$ we have + +$$ +1-g(1-(a+b))=g(a)+g(b)=2 g\left(\frac{a+b}{2}\right) +$$ + +Hence $g$ obeys Jensen's functional equation over $(0,1 / 2)$. + +- Define $h:[0,1] \rightarrow \mathbb{R}$ by $h(t)=g\left(\frac{2 t+1}{8}\right)-(1-t) \cdot g(1 / 8)-t \cdot g(3 / 8)$, then $h$ satisfies Jensen's functional equation too over $[0,1]$. We have also arranged that $h(0)=h(1)=0$, hence $h(1 / 2)=0$ as well. +- Since + +$$ +h(t)=h(t)+h(1 / 2)=2 h(t / 2+1 / 4)=h(t+1 / 2)+h(0)=h(t+1 / 2) +$$ + +for any $t<1 / 2$, we find $h$ is periodic modulo $1 / 2$. It follows one can extend $\widetilde{h}$ by + +$$ +\widetilde{h}: \mathbb{R} \rightarrow \mathbb{R} \quad \text { by } \quad \widetilde{h}(t)=h(t-\lfloor t\rfloor) +$$ + +and still satisfy Jensen's functional equation. Because $\widetilde{h}(0)=0$, it's well-known this implies $\widetilde{h}$ is additive (because $\widetilde{h}(x+y)=2 \widetilde{h}((x+y) / 2)=\widetilde{h}(x)+\widetilde{h}(y)$ for any real numbers $x$ to $y$ ). + +But $\widetilde{h}$ is bounded below on $[0,1]$ since $g \geq 0$, and since $\widetilde{h}$ is also additive, it follows (well-known) that $\widetilde{h}$ is linear. Thus $h$ is the zero function. So, the function $g$ is linear over $[1 / 8,3 / 8]$; thus we may write $g(x)=k x+\ell$, valid for $1 / 8 \leq x \leq 3 / 8$. + +Since $3 g(1 / 3)=1$, it follows $k+3 \ell=1$. +For $02$ case. + +## Corollary + +We have $\nu_{p}\left(1^{k}+\cdots+t^{k}\right) \geq \nu_{p}(t)-1$ for any $k, t, p$. + +Proof. Assume $p \mid t$. Handle the terms in that sum divisible by $p$ (by induction) and apply the lemma a bunch of times. + +Now the idea is to add primes $q$ one at a time to $n$, starting from the base case $n=p^{e}$. So, formally we proceed by induction on the number of prime divisors of $n$. We'll also assume $k \geq 1$ in what follows since the base case $k=0$ is easy. + +- First, suppose we want to go from $n$ to $n q$ where $q \nmid n$. In that case $\varphi(n q)$ gained $\nu_{p}(q-1)$ factors of $p$ and then we need to show $\nu_{p}(S(n q, k)) \geq \nu_{p}(\varphi(n))+\nu_{p}(q-1)$. The trick is to write + +$$ +A(n q)=\{a+n h \mid a \in A(n) \text { and } h=0, \ldots, q-1\} \backslash q A(n) +$$ + +and then expand using binomial theorem: + +$$ +\begin{aligned} +S(n q, k) & =\sum_{a \in A(n)} \sum_{h=0}^{q-1}(a+n h)^{k}-\sum_{a \in A(n)}(q a)^{k} \\ +& =-q^{k} S(n, k)+\sum_{a \in A(n)} \sum_{h=0}^{q-1} \sum_{j=0}^{k}\left[\binom{k}{j} a^{k-j} n^{j} h^{j}\right] \\ +& =-q^{k} S(n, k)+\sum_{j=0}^{k}\left[\binom{k}{j} n^{j}\left(\sum_{a \in A(n)} a^{k-j}\right)\left(\sum_{h=0}^{q-1} h^{j}\right)\right] \\ +& =-q^{k} S(n, k)+\sum_{j=0}^{k}\left[\binom{k}{j} n^{j} S(n, k-j)\left(\sum_{h=1}^{q-1} h^{j}\right)\right] \\ +& =\left(q-q^{k}\right) S(n, k)+\sum_{j=1}^{k}\left[\binom{k}{j} n^{j} S(n, k-j)\left(\sum_{h=1}^{q-1} h^{j}\right)\right] +\end{aligned} +$$ + +We claim every term here has enough powers of $p$. For the first term, $S(n, k)$ has at least $\nu_{p}(\varphi(n))$ factors of $p$; and we have the $q-q^{k}$ multiplier out there. For the other terms, we apply induction to $S(n, k-j)$; moreover $\sum_{h=1}^{q-1} h^{j}$ has at least $\nu_{p}(q-1)-1$ factors of $p$ by corollary, and we get one more factor of $p$ (at least) from $n^{j}$. + +- On the other hand, if $q$ already divides $n$, then this time + +$$ +A(n q)=\{a+n h \mid a \in A(n) \text { and } h=0, \ldots, q-1\} +$$ + +and we have no additional burden of $p$ to deal with; the same calculation gives + +$$ +S(n q, k)=q S(n, k)+\sum_{j=1}^{k}\left[\binom{k}{j} n^{j} S(n, k-j)\left(\sum_{h=1}^{q-1} h^{j}\right)\right] +$$ + +which certainly has enough factors of $p$ already. + +Remark. A curious bit about the problem is that $\nu_{p}(\varphi(n))$ can exceed $\nu_{p}(n)$, and so it is not true that the residues of $A(n)$ are well-behaved modulo $\varphi(n)$. + +As an example, let $n=2 \cdot 3 \cdot 7 \cdot 13=546$, so $m=\varphi(n)=1 \cdot 2 \cdot 6 \cdot 12=144$. Then $A(n)$ contains 26 elements which are $1 \bmod 9$ and 23 elements which are $4 \bmod 9$. + +Remark. The converse of the problem is true too (but asking both parts would make this too long for exam). + +## §2 Solutions to Day 2 + +## §2.1 USAMO 2018/4, proposed by Ankan Bhattacharya + +Available online at https://aops.com/community/p10232389. + +## Problem statement + +Let $p$ be a prime, and let $a_{1}, \ldots, a_{p}$ be integers. Show that there exists an integer $k$ such that the numbers + +$$ +a_{1}+k, a_{2}+2 k, \ldots, a_{p}+p k +$$ + +produce at least $\frac{1}{2} p$ distinct remainders upon division by $p$. + +For each $k=0, \ldots, p-1$ let $G_{k}$ be the graph on $\{1, \ldots, p\}$ where we join $\{i, j\}$ if and only if + +$$ +a_{i}+i k \equiv a_{j}+j k \quad(\bmod p) \Longleftrightarrow k \equiv-\frac{a_{i}-a_{j}}{i-j} \quad(\bmod p) . +$$ + +So we want a graph $G_{k}$ with at least $\frac{1}{2} p$ connected components. +However, each $\{i, j\}$ appears in exactly one graph $G_{k}$, so some graph has at most $\frac{1}{p}\binom{p}{2}=\frac{1}{2}(p-1)$ edges (by "pigeonhole"). This graph has at least $\frac{1}{2}(p+1)$ connected components, as desired. + +Remark. Here is an example for $p=5$ showing equality can occur: + +$$ +\left[\begin{array}{lllll} +0 & 0 & 3 & 4 & 3 \\ +0 & 1 & 0 & 2 & 2 \\ +0 & 2 & 2 & 0 & 1 \\ +0 & 3 & 4 & 3 & 0 \\ +0 & 4 & 1 & 1 & 4 +\end{array}\right] . +$$ + +Ankan Bhattacharya points out more generally that $a_{i}=i^{2}$ is sharp in general. + +## §2.2 USAMO 2018/5, proposed by Kada Williams + +Available online at https://aops.com/community/p10232392. + +## Problem statement + +Let $A B C D$ be a convex cyclic quadrilateral with $E=\overline{A C} \cap \overline{B D}, F=\overline{A B} \cap \overline{C D}$, $G=\overline{D A} \cap \overline{B C}$. The circumcircle of $\triangle A B E$ intersects line $C B$ at $B$ and $P$, and the circumcircle of $\triangle A D E$ intersects line $C D$ at $D$ and $Q$. Assume $C, B, P, G$ and $C$, $Q, D, F$ are collinear in that order. Let $M=\overline{F P} \cap \overline{G Q}$. Prove that $\angle M A C=90^{\circ}$. + +We present three general routes. (The second route, using the fact that $\overline{A C}$ is an angle bisector, has many possible variations.) +\l First solution (Miquel points). This is indeed a Miquel point problem, but the main idea is to focus on the self-intersecting cyclic quadrilateral $P B Q D$ as the key player, rather than on the given $A B C D$. + +Indeed, we will prove that $A$ is its Miquel point; this follows from the following two claims. + +Claim - The self-intersecting quadrilateral $P Q D B$ is cyclic. + +Proof. By power of a point from $C: C Q \cdot C D=C A \cdot C E=C B \cdot C P$. + +Claim - Point $E$ lies on line $P Q$. + +Proof. $\measuredangle A E P=\measuredangle A B P=\measuredangle A B C=\measuredangle A D C=\measuredangle A D Q=\measuredangle A E Q$. +![](https://cdn.mathpix.com/cropped/2024_11_19_3f8866632dd9408ace0ag-10.jpg?height=804&width=724&top_left_y=1617&top_left_x=649) + +To finish, let $H=\overline{P D} \cap \overline{B Q}$. By properties of the Miquel point, we have $A$ is the foot from $H$ to $\overline{C E}$. But also, points $M, A, H$ are collinear by Pappus theorem on $\overline{B P G}$ and $\overline{D Q F}$, as desired. + +【 Second solution (projective). We start with a synthetic observation. +Claim - The line $\overline{A C}$ bisects $\angle P A D$ and $\angle B A Q$. + +Proof. Angle chase: $\measuredangle P A C=\measuredangle P A E=\measuredangle P B E=\measuredangle C B D=\measuredangle C A D$. +There are three ways to finish from here: + +- (Michael Kural) Suppose the external bisector of $\angle P A D$ and $\angle B A Q$ meet lines $B C$ and $D C$ at $X$ and $Y$. Then + +$$ +-1=(G P ; X C)=(F D ; Y C) +$$ + +which is enough to imply that $\overline{X Y}, \overline{G Q}, \overline{P F}$ are concurrent (by so-called prism lemma). + +- (Daniel Liu) Alternatively, apply the dual Desargues involution theorem to complete quadrilateral GQFPCM, through the point $A$. This gives that an involutive pairing of + +$$ +(A C, A M)(A P, A Q)(A G, A F) +$$ + +This is easier to see if we project it onto the line $\ell$ through $C$ perpendicular to $\overline{A C}$; if we let $P^{\prime}, Q^{\prime}, G^{\prime}, F^{\prime}$ be the images of the last four lines, we find the involution coincides with negative inversion through $C$ with power $\sqrt{C P^{\prime} \cdot C Q^{\prime}}$ which implies that $\overline{A M} \cap \ell$ is an infinity point, as desired. + +- (Kada Williams) The official solution instead shows the external angle bisector by a long trig calculation. + +【 Third solution (inversion, Andrew Wu). Noting that $C E \cdot C A=C P \cdot C B=C Q \cdot C D$, we perform an inversion at $C$ swapping these pairs of points. The point $G$ is mapped to a point $G^{*}$ ray $C B$ for which $Q E G^{*} C$ is cyclic, but then + +$$ +\measuredangle C G^{*} E=\measuredangle C Q E=\measuredangle C Q P=\measuredangle D B C=\measuredangle C B E +$$ + +and so we conclude $E B=E G^{*}$. Similarly, $E D=E F^{*}$. +Finally, $M^{*}=\left(C G^{*} D\right) \cap\left(C F^{*} B\right) \neq C$, and we wish to show that $\angle E M^{*} C=90^{\circ}$. +![](https://cdn.mathpix.com/cropped/2024_11_19_3f8866632dd9408ace0ag-11.jpg?height=800&width=575&top_left_y=1830&top_left_x=746) + +Note that $M^{*}$ is the center of the spiral similarity sending $\overline{B G^{*}}$ to $\overline{F^{*} D}$. Hence it also maps the midpoint $K$ of $B G^{*}$ to the midpoint $L$ of $\overline{F^{*} E}$. Consequently, $M^{*}$ lies on the circumcircle $K L C$ as well. In other words, $E L C K M^{*}$ is a cyclic pentagon with circumdiameter $\overline{C E}$, as desired. + +## §2.3 USAMO 2018/6, proposed by Richard Stong + +Available online at https://aops.com/community/p10232388. + +## Problem statement + +Let $a_{n}$ be the number of permutations $\left(x_{1}, \ldots, x_{n}\right)$ of $(1, \ldots, n)$ such that the ratios $x_{k} / k$ are all distinct. Prove that $a_{n}$ is odd for all $n \geq 1$. + +This is the official solution; the proof has two main insights. +The first idea: + +## Lemma + +If a permutation $x$ works, so does the inverse permutation. +Thus it suffices to consider permutations $x$ in which all cycles have length at most 2 . Of course, there can be at most one fixed point (since that gives the ratio 1 ), and hence exactly one if $n$ is odd, none if $n$ is even. + +We consider the graph $K_{n}$ such that the edge $\{i, j\}$ is labeled with $i / j$ (for $i1$. +Finally, note that the number of neighbors of $\mathcal{M}$ is the product across all $\ell$ of the above. So it is odd if and only if each factor is odd, if and only if $n_{\ell}=1$ for every $\ell$. + +To finish, consider a huge simple graph $\Gamma$ on all the maximal matchings, with edge relations given by neighbor relation (we don't consider vertices to be connected to themselves). Observe that: + +- Fantastic matchings correspond to isolated vertices (of degree zero, with no other neighbors) of $\Gamma$. +- The rest of the vertices of $\Gamma$ have odd degrees (one less than the neighbor count) +- The graph $\Gamma$ has an even number of vertices of odd degree (this is true for any simple graph, see "handshake lemma"). +- The number of vertices of $\Gamma$ is odd, namely $(2\lceil n / 2\rceil-1)!!$ + +This concludes the proof. + diff --git a/USAMO/md/en-USAMO-2019-notes.md b/USAMO/md/en-USAMO-2019-notes.md new file mode 100644 index 0000000000000000000000000000000000000000..8c24ef0d1c980bf67c2f61b450b6be882d731891 --- /dev/null +++ b/USAMO/md/en-USAMO-2019-notes.md @@ -0,0 +1,386 @@ +# USAMO 2019 Solution Notes + +Evan Chen《陳誼廷》 + +15 April 2024 + +This is a compilation of solutions for the 2019 USAMO. The ideas of the solution are a mix of my own work, the solutions provided by the competition organizers, and solutions found by the community. However, all the writing is maintained by me. + +These notes will tend to be a bit more advanced and terse than the "official" solutions from the organizers. In particular, if a theorem or technique is not known to beginners but is still considered "standard", then I often prefer to use this theory anyways, rather than try to work around or conceal it. For example, in geometry problems I typically use directed angles without further comment, rather than awkwardly work around configuration issues. Similarly, sentences like "let $\mathbb{R}$ denote the set of real numbers" are typically omitted entirely. + +Corrections and comments are welcome! + +## Contents + +0 Problems ..... 2 +1 Solutions to Day 1 ..... 3 +1.1 USAMO 2019/1, proposed by Evan Chen ..... 3 +1.2 USAMO 2019/2, proposed by Ankan Bhattacharya ..... 5 +1.3 USAMO 2019/3, proposed by Titu Andreescu, Vlad Matei, Cosmin Pohoata ..... 8 +2 Solutions to Day 2 ..... 10 +2.1 USAMO 2019/4, proposed by Ricky Liu ..... 10 +2.2 USAMO 2019/5, proposed by Yannick Yao ..... 11 +2.3 USAMO 2019/6, proposed by Titu Andreescu, Gabriel Dospinescu ..... 12 + +## §0 Problems + +1. A function $f: \mathbb{N} \rightarrow \mathbb{N}$ satisfies + +$$ +\underbrace{f(f(\ldots f}_{f(n) \text { times }}(n) \ldots))=\frac{n^{2}}{f(f(n))} +$$ + +for all positive integers $n$. What are all possible values of $f(1000)$ ? +2. Let $A B C D$ be a cyclic quadrilateral satisfying $A D^{2}+B C^{2}=A B^{2}$. The diagonals of $A B C D$ intersect at $E$. Let $P$ be a point on side $\overline{A B}$ satisfying $\angle A P D=\angle B P C$. Show that line $P E$ bisects $\overline{C D}$. +3. Let $K$ be the set of positive integers not containing the decimal digit 7. Determine all polynomials $f(x)$ with nonnegative coefficients such that $f(x) \in K$ for all $x \in K$. +4. Let $n$ be a nonnegative integer. Determine the number of ways to choose sets $S_{i j} \subseteq\{1,2, \ldots, 2 n\}$, for all $0 \leq i \leq n$ and $0 \leq j \leq n$ (not necessarily distinct), such that + +- $\left|S_{i j}\right|=i+j$, and +- $S_{i j} \subseteq S_{k l}$ if $0 \leq i \leq k \leq n$ and $0 \leq j \leq l \leq n$. + +5. Let $m$ and $n$ be relatively prime positive integers. The numbers $\frac{m}{n}$ and $\frac{n}{m}$ are written on a blackboard. At any point, Evan may pick two of the numbers $x$ and $y$ written on the board and write either their arithmetic mean $\frac{1}{2}(x+y)$ or their harmonic mean $\frac{2 x y}{x+y}$. For which $(m, n)$ can Evan write 1 on the board in finitely many steps? +6. Find all polynomials $P$ with real coefficients such that + +$$ +\frac{P(x)}{y z}+\frac{P(y)}{z x}+\frac{P(z)}{x y}=P(x-y)+P(y-z)+P(z-x) +$$ + +for all nonzero real numbers $x, y, z$ obeying $2 x y z=x+y+z$. + +## §1 Solutions to Day 1 + +## §1.1 USAMO 2019/1, proposed by Evan Chen + +Available online at https://aops.com/community/p12189527. + +## Problem statement + +A function $f: \mathbb{N} \rightarrow \mathbb{N}$ satisfies + +$$ +\underbrace{f(f(\ldots f}_{f(n) \text { times }}(n) \ldots))=\frac{n^{2}}{f(f(n))} +$$ + +for all positive integers $n$. What are all possible values of $f(1000)$ ? + +Actually, we classify all such functions: $f$ can be any function which fixes odd integers and acts as an involution on the even integers. In particular, $f(1000)$ may be any even integer. + +It's easy to check that these all work, so now we check they are the only solutions. +Claim - $f$ is injective. + +Proof. If $f(a)=f(b)$, then $a^{2}=f^{f(a)}(a) f(f(a))=f^{f(b)}(b) f(f(b))=b^{2}$, so $a=b$. + +Claim - $f$ fixes the odd integers. + +Proof. We prove this by induction on odd $n \geq 1$. +Assume $f$ fixes $S=\{1,3, \ldots, n-2\}$ now (allowing $S=\varnothing$ for $n=1$ ). Now we have that + +$$ +f^{f(n)}(n) \cdot f^{2}(n)=n^{2} . +$$ + +However, neither of the two factors on the left-hand side can be in $S$ since $f$ was injective. Therefore they must both be $n$, and we have $f^{2}(n)=n$. + +Now let $y=f(n)$, so $f(y)=n$. Substituting $y$ into the given yields + +$$ +y^{2}=f^{n}(y) \cdot y=f^{n+1}(n) \cdot y=n y +$$ + +since $n+1$ is even. We conclude $n=y$, as desired. + +Remark (Motivation). After obtaining $f(1)=1$ and $f$ injective, here is one way to motivate where the above proof comes from. From the equation + +$$ +f^{f(n)}(n) \cdot f^{2}(n)=n^{2} +$$ + +it would be natural to consider the case where $n$ is prime, because $p^{2}$ only has a few possible factorizations. In fact, actually because of injectivity and $f(1)=1$, we would need to have + +$$ +f^{f(p)}(p)=f^{2}(p)=p +$$ + +in order for the equation to be true. Continuing on as in the proof above, one then gets $f(p)=p$ for odd primes $p$ (but no control over $f(2)$ ). + +The special case of prime $n$ then serves as a foothold by which one can continue the induction towards all numbers $n$, finding the induction works out exactly when the prime 2 never appears. + +As a general point, in mathematical problem-solving, one often needs to be willing to try out a proof idea or strategy and then retroactively determine what hypothesis is needed, rather than hoping one will always happen to guess exactly the right claim first. In other words, it may happen that one begins working out a proof of a claim before knowing exactly what the claim will turn out to say, and this is the case here (despite the fact the proof strategy uses induction). + +Thus, $f$ maps even integers to even integers. In light of this, we may let $g:=f(f(n))$ (which is also injective), so we conclude that + +$$ +g^{f(n) / 2}(n) g(n)=n^{2} \quad \text { for } n=2,4, \ldots +$$ + +Claim - The function $g$ is the identity function. + +Proof. The proof is similar to the earlier proof of the claim. Note that $g$ fixes the odd integers already. We proceed by induction to show $g$ fixes the even integers; so assume $g$ fixes the set $S=\{1,2, \ldots, n-1\}$, for some even integer $n \geq 2$. In the equation + +$$ +g^{f(n) / 2}(n) \cdot g(n)=n^{2} +$$ + +neither of the two factors may be less than $n$. So they must both be $n$. +These three claims imply that the solutions we claimed earlier are the only ones. +Remark. The last claim is not necessary to solve the problem; after realizing $f$ and injective fixes the odd integers, this answers the question about the values of $f(1000)$. However, we chose to present the "full" solution anyways. + +Remark. After noting $f$ is injective, another approach is outlined below. Starting from any $n$, consider the sequence + +$$ +n, f(n), f(f(n)) +$$ + +and so on. We may let $m$ be the smallest term of the sequence; then $m^{2}=f(f(m)) \cdot f^{f(m)}(m)$ which forces $f(f(m))=f^{f(m)}(m)=m$ by minimality. Thus the sequence is 2 -periodic. Therefore, $f(f(n))=n$ always holds, which is enough to finish. +\ Authorship comments. I will tell you a great story about this problem. Two days before the start of grading of USAMO 2017, I had a dream that I was grading a functional equation. When I woke up, I wrote it down, and it was + +$$ +f^{f(n)}(n)=\frac{n^{2}}{f(f(n))} +$$ + +You can guess the rest of the story (and imagine how surprised I was the solution set was interesting). I guess some dreams do come true, huh? + +## §1.2 USAMO 2019/2, proposed by Ankan Bhattacharya + +Available online at https://aops.com/community/p12189455. + +## Problem statement + +Let $A B C D$ be a cyclic quadrilateral satisfying $A D^{2}+B C^{2}=A B^{2}$. The diagonals of $A B C D$ intersect at $E$. Let $P$ be a point on side $\overline{A B}$ satisfying $\angle A P D=\angle B P C$. Show that line $P E$ bisects $\overline{C D}$. + +Here are three solutions. The first two are similar although the first one makes use of symmedians. The last solution by inversion is more advanced. + +【 First solution using symmedians. We define point $P$ to obey + +$$ +\frac{A P}{B P}=\frac{A D^{2}}{B C^{2}}=\frac{A E^{2}}{B E^{2}} +$$ + +so that $\overline{P E}$ is the $E$-symmedian of $\triangle E A B$, therefore the $E$-median of $\triangle E C D$. +Now, note that + +$$ +A D^{2}=A P \cdot A B \quad \text { and } \quad B C^{2}=B P \cdot B A +$$ + +This implies $\triangle A P D \sim \triangle A D B$ and $\triangle B P C \sim \triangle B C A$. Thus + +$$ +\measuredangle D P A=\measuredangle A D B=\measuredangle A C B=\measuredangle B C P +$$ + +and so $P$ satisfies the condition as in the statement (and is the unique point to do so), as needed. + +【 Second solution using only angle chasing (by proposer). We again re-define $P$ to obey $A D^{2}=A P \cdot A B$ and $B C^{2}=B P \cdot B A$. As before, this gives $\triangle A P D \sim \triangle A B D$ and $\triangle B P C \sim \triangle B D P$ and so we let + +$$ +\theta:=\measuredangle D P A=\measuredangle A D B=\measuredangle A C B=\measuredangle B C P . +$$ + +Our goal is to now show $\overline{P E}$ bisects $\overline{C D}$. +Let $K=\overline{A C} \cap \overline{P D}$ and $L=\overline{A D} \cap \overline{P C}$. Since $\measuredangle K P A=\theta=\measuredangle A C B$, quadrilateral $B P K C$ is cyclic. Similarly, so is $A P L D$. +![](https://cdn.mathpix.com/cropped/2024_11_19_dab316fe5a4e03dd61cdg-06.jpg?height=804&width=798&top_left_y=249&top_left_x=635) + +Finally $A K L B$ is cyclic since + +$$ +\measuredangle B K A=\measuredangle B K C=\measuredangle B P C=\theta=\measuredangle D P A=\measuredangle D L A=\measuredangle B L A . +$$ + +This implies $\measuredangle C K L=\measuredangle L B A=\measuredangle D C K$, so $\overline{K L} \| \overline{B C}$. Then $P E$ bisects $\overline{B C}$ by Ceva's theorem on $\triangle P C D$. + +【 Third solution (using inversion). By hypothesis, the circle $\omega_{a}$ centered at $A$ with radius $A D$ is orthogonal to the circle $\omega_{b}$ centered at $B$ with radius $B C$. For brevity, we let $\mathbf{I}_{a}$ and $\mathbf{I}_{b}$ denote inversion with respect to $\omega_{a}$ and $\omega_{b}$. + +We let $P$ denote the intersection of $\overline{A B}$ with the radical axis of $\omega_{a}$ and $\omega_{b}$; hence $P=\mathbf{I}_{a}(B)=\mathbf{I}_{b}(A)$. This already implies that + +$$ +\measuredangle D P A \stackrel{\mathbf{I}_{a}}{=} \measuredangle A D B=\measuredangle A C B \stackrel{\mathbf{I}_{b}}{=} \measuredangle B P C +$$ + +so $P$ satisfies the angle condition. +![](https://cdn.mathpix.com/cropped/2024_11_19_dab316fe5a4e03dd61cdg-06.jpg?height=732&width=1014&top_left_y=1844&top_left_x=521) + +Claim - The point $K=\mathbf{I}_{a}(C)$ lies on $\omega_{b}$ and $\overline{D P}$. Similarly $L=\mathbf{I}_{b}(D)$ lies on $\omega_{a}$ and $\overline{C P}$. + +Proof. The first assertion follows from the fact that $\omega_{b}$ is orthogonal to $\omega_{a}$. For the other, since $(B C D)$ passes through $A$, it follows $P=\mathbf{I}_{a}(B), K=\mathbf{I}_{a}(C)$, and $D=\mathbf{I}_{a}(D)$ are collinear. + +Finally, since $C, L, P$ are collinear, we get $A$ is concyclic with $K=\mathbf{I}_{a}(C), L=\mathbf{I}_{a}(L)$, $B=\mathbf{I}_{a}(P)$, i.e. that $A K L B$ is cyclic. So $\overline{K L} \| \overline{C D}$ by Reim's theorem, and hence $\overline{P E}$ bisects $\overline{C D}$ by Ceva's theorem. + +## §1.3 USAMO 2019/3, proposed by Titu Andreescu, Vlad Matei, Cosmin Pohoata + +Available online at https://aops.com/community/p12189457. + +## Problem statement + +Let $K$ be the set of positive integers not containing the decimal digit 7. Determine all polynomials $f(x)$ with nonnegative coefficients such that $f(x) \in K$ for all $x \in K$. + +The answer is only the obvious ones: $f(x)=10^{e} x, f(x)=k$, and $f(x)=10^{e} x+k$, for any choice of $k \in K$ and $e>\log _{10} k$ (with $e \geq 0$ ). + +Now assume $f$ satisfies $f(K) \subseteq K$; such polynomials will be called stable. We first prove the following claim which reduces the problem to the study of monomials. + +Lemma (Reduction to monomials) +If $f(x)=a_{0}+a_{1} x+a_{2} x^{2}+\ldots$ is stable, then each monomial $a_{0}, a_{1} x, a_{2} x^{2}, \ldots$ is stable. + +Proof. For any $x \in K$, plug in $f\left(10^{e} x\right)$ for large enough $e$ : the decimal representation of $f$ will contain $a_{0}, a_{1} x, a_{2} x^{2}$ with some zeros padded in between. + +Let's tackle the linear case next. Here is an ugly but economical proof. +Claim (Linear classification) - If $f(x)=c x$ is stable, then $c=10^{e}$ for some nonnegative integer $e$. + +Proof. We will show when $c \neq 10^{e}$ then we can find $x \in K$ such that $c x$ starts with the digit 7. This can actually be done with the following explicit cases in terms of how $c$ starts in decimal notation: + +- For $9 \cdot 10^{e} \leq c<10 \cdot 10^{e}$, pick $x=8$. +- For $8 \cdot 10^{e} \leq c<9 \cdot 10^{e}$, pick $x=88$. +- For $7 \cdot 10^{e} \leq c<8 \cdot 10^{e}$, pick $x=1$. +- For $4.4 \cdot 10^{e} \leq c<7 \cdot 10^{e}$, pick $11 \leq x \leq 16$. +- For $2.7 \cdot 10^{e} \leq c<4.4 \cdot 10^{e}$, pick $18 \leq x \leq 26$. +- For $2 \cdot 10^{e} \leq c<2.7 \cdot 10^{e}$, pick $28 \leq x \leq 36$. +- For $1.6 \cdot 10^{e} \leq c<2 \cdot 10^{e}$, pick $38 \leq x \leq 46$. +- For $1.3 \cdot 10^{e} \leq c<1.6 \cdot 10^{e}$, pick $48 \leq x \leq 56$. +- For $1.1 \cdot 10^{e} \leq c<1.3 \cdot 10^{e}$, pick $58 \leq x \leq 66$. +- For $1 \cdot 10^{e} \leq c<1.1 \cdot 10^{e}$, pick $x=699 \ldots 9$ for suitably many 9 's. + +The hardest part of the problem is the case where $\operatorname{deg} f>1$. We claim that no solutions exist then: + +Claim (Higher-degree classification) - No monomial of the form $f(x)=c x^{d}$ is stable for any $d>1$. + +Proof. Note that $f(10 x+3)$ is stable too. Thus + +$$ +f(10 x+3)=3^{d}+10 d \cdot 3^{d-1} x+100\binom{d}{2} \cdot 3^{d-1} x^{2}+\ldots +$$ + +is stable. By applying the lemma the linear monomial $10 d \cdot 3^{d-1} x$ is stable, so $10 d \cdot 3^{d-1}$ is a power of 10 , which can only happen if $d=1$. + +Thus the only nonconstant stable polynomials with nonnegative coefficients must be of the form $f(x)=10^{e} x+k$ for $e \geq 0$. It is straightforward to show we then need $k<10^{e}$ and this finishes the proof. + +Remark. The official solution replaces the proof for $f(x)=c x$ with Kronecker density. From $f(1)=c \in K$, we get $f(c)=c^{2} \in K$, et cetera and hence $c^{n} \in K$. But it is known that when $c$ is not a power of 10 , some power of $c$ starts with any specified prefix. + +## §2 Solutions to Day 2 + +## §2.1 USAMO 2019/4, proposed by Ricky Liu + +Available online at https://aops.com/community/p12195861. + +## Problem statement + +Let $n$ be a nonnegative integer. Determine the number of ways to choose sets $S_{i j} \subseteq\{1,2, \ldots, 2 n\}$, for all $0 \leq i \leq n$ and $0 \leq j \leq n$ (not necessarily distinct), such that + +- $\left|S_{i j}\right|=i+j$, and +- $S_{i j} \subseteq S_{k l}$ if $0 \leq i \leq k \leq n$ and $0 \leq j \leq l \leq n$. + +The answer is $(2 n)!\cdot 2^{n^{2}}$. First, we note that $\varnothing=S_{00} \subsetneq S_{01} \subsetneq \cdots \subsetneq S_{n n}=\{1, \ldots, 2 n\}$ and thus multiplying by (2n)! we may as well assume $S_{0 i}=\{1, \ldots, i\}$ and $S_{i n}=\{1, \ldots, n+i\}$. We illustrate this situation by placing the sets in a grid, as below for $n=4$; our goal is to fill in the rest of the grid. +$\left[\begin{array}{ccccc}1234 & 12345 & 123456 & 1234567 & 12345678 \\ 123 & & & & \\ 12 & & & & \\ 1 & & & & \\ \varnothing & & & & \end{array}\right]$ + +We claim the number of ways to do so is $2^{n^{2}}$. In fact, more strongly even the partial fillings are given exactly by powers of 2 . + +Claim - Fix a choice $T$ of cells we wish to fill in, such that whenever a cell is in $T$, so are all the cells above and left of it. (In other words, $T$ is a Young tableau.) The number of ways to fill in these cells with sets satisfying the inclusion conditions is $2^{|T|}$ 。 + +An example is shown below, with an indeterminate set marked in red (and the rest of $T$ marked in blue). +$\left[\begin{array}{ccccc}1234 & 12345 & 123456 & 1234567 & 12345678 \\ 123 & 1234 & 12346 & 123467 & \\ 12 & 124 & 1234 \text { or } 1246 & & \\ 1 & 12 & & & \\ \varnothing & 2 & & & \end{array}\right]$ + +Proof. The proof is by induction on $|T|$, with $|T|=0$ being vacuous. +Now suppose we have a corner $\left[\begin{array}{ll}B & C \\ A & S\end{array}\right]$ where $A, B, C$ are fixed and $S$ is to be chosen. Then we may write $B=A \cup\{x\}$ and $C=A \cup\{x, y\}$ for $x, y \notin A$. Then the two choices of $S$ are $A \cup\{x\}$ (i.e. $B$ ) and $A \cup\{y\}$, and both of them are seen to be valid. + +In this way, we gain a factor of 2 any time we add one cell as above to $T$. Since we can achieve any Young tableau in this way, the induction is complete. + +## §2.2 USAMO 2019/5, proposed by Yannick Yao + +Available online at https://aops.com/community/p12195834. + +## Problem statement + +Let $m$ and $n$ be relatively prime positive integers. The numbers $\frac{m}{n}$ and $\frac{n}{m}$ are written on a blackboard. At any point, Evan may pick two of the numbers $x$ and $y$ written on the board and write either their arithmetic mean $\frac{1}{2}(x+y)$ or their harmonic mean $\frac{2 x y}{x+y}$. For which $(m, n)$ can Evan write 1 on the board in finitely many steps? + +We claim this is possible if and only $m+n$ is a power of 2 . Let $q=m / n$, so the numbers on the board are $q$ and $1 / q$. +\I Impossibility. The main idea is the following. +Claim - Suppose $p$ is an odd prime. Then if the initial numbers on the board are $-1(\bmod p)$, then all numbers on the board are $-1(\bmod p)$. + +Proof. Let $a \equiv b \equiv-1(\bmod p)$. Note that $2 \not \equiv 0(\bmod p)$ and $a+b \equiv-2 \not \equiv 0(\bmod p)$. Thus $\frac{a+b}{2}$ and $\frac{2 a b}{a+b}$ both make sense modulo $p$ and are equal to $-1(\bmod p)$. + +Thus if there exists any odd prime divisor $p$ of $m+n$ (implying $p \nmid m n$ ), then + +$$ +q \equiv \frac{1}{q} \equiv-1 \quad(\bmod p) . +$$ + +and hence all numbers will be $-1(\bmod p)$ forever. This implies that it's impossible to write 1 , whenever $m+n$ is divisible by some odd prime. + +【 Construction. Conversely, suppose $m+n$ is a power of 2 . We will actually construct 1 without even using the harmonic mean. +![](https://cdn.mathpix.com/cropped/2024_11_19_dab316fe5a4e03dd61cdg-11.jpg?height=152&width=1004&top_left_y=1786&top_left_x=526) + +Note that + +$$ +\frac{n}{m+n} \cdot q+\frac{m}{m+n} \cdot \frac{1}{q}=1 +$$ + +and obviously by taking appropriate midpoints (in a binary fashion) we can achieve this using arithmetic mean alone. + +## §2.3 USAMO 2019/6, proposed by Titu Andreescu, Gabriel Dospinescu + +Available online at https://aops.com/community/p12195858. + +## Problem statement + +Find all polynomials $P$ with real coefficients such that + +$$ +\frac{P(x)}{y z}+\frac{P(y)}{z x}+\frac{P(z)}{x y}=P(x-y)+P(y-z)+P(z-x) +$$ + +for all nonzero real numbers $x, y, z$ obeying $2 x y z=x+y+z$. + +The given can be rewritten as saying that + +$$ +\begin{aligned} +Q(x, y, z) & :=x P(x)+y P(y)+z P(z) \\ +& -x y z(P(x-y)+P(y-z)+P(z-x)) +\end{aligned} +$$ + +is a polynomial vanishing whenever $x y z \neq 0$ and $2 x y z=x+y+z$, for real numbers $x, y$, $z$. + +Claim - This means $Q(x, y, z)$ vanishes also for any complex numbers $x, y, z$ obeying $2 x y z=x+y+z$. + +Proof. Indeed, this means that the rational function + +$$ +R(x, y):=Q\left(x, y, \frac{x+y}{2 x y-1}\right) +$$ + +vanishes for any real numbers $x$ and $y$ such that $x y \neq \frac{1}{2}, x \neq 0, y \neq 0, x+y \neq 0$. This can only occur if $R$ is identically zero as a rational function with real coefficients. If we then regard $R$ as having complex coefficients, the conclusion then follows. + +Remark (Algebraic geometry digression on real dimension). Note here we use in an essential way that $z$ can be solved for in terms of $x$ and $y$. If $s(x, y, z)=2 x y z-(x+y+z)$ is replaced with some general condition, the result may become false; e.g. we would certainly not expect the result to hold when $s(x, y, z)=x^{2}+y^{2}+z^{2}-(x y+y z+z x)$ since for real numbers $s=0$ only when $x=y=z$ ! + +The general condition we need here is that $s(x, y, z)=0$ should have "real dimension two". Here is a proof using this language, in our situation. + +Let $M \subset \mathbb{R}^{3}$ be the surface $s=0$. We first contend $M$ is two-dimensional manifold. Indeed, the gradient $\nabla s=\langle 2 y z-1,2 z x-1,2 x y-1\rangle$ vanishes only at the points $( \pm 1 / \sqrt{2}, \pm 1 / \sqrt{2}, \pm 1 / \sqrt{2}$ ) where the $\pm$ signs are all taken to be the same. These points do not lie on $M$, so the result follows by the regular value theorem. In particular the topological closure of points on $M$ with $x y z \neq 0$ is all of $M$ itself; so $Q$ vanishes on all of $M$. + +If we now identify $M$ with the semi-algebraic set consisting of maximal ideals ( $x-a, y-$ $b, z-c)$ in Spec $\mathbb{R}[x, y, z]$ satisfying $2 a b c=a+b+c$, then we have real dimension two, and thus the Zariski closure of $M$ is a two-dimensional closed subset of $\operatorname{Spec} \mathbb{R}[x, y, z]$. Thus it must be $Z=\mathcal{V}(2 x y z-(x+y+z))$, since this $Z$ is an irreducible two-dimensional closed subset (say, by Krull's principal ideal theorem) containing $M$. Now $Q$ is a global section vanishing on all of $Z$, therefore $Q$ is contained in the (radical, principal) ideal $(2 x y z-(x+y+z))$ as needed. So it is actually divisible by $2 x y z-(x+y+z)$ as desired. + +Now we regard $P$ and $Q$ as complex polynomials instead. First, note that substituting $(x, y, z)=(t,-t, 0)$ implies $P$ is even. We then substitute + +$$ +(x, y, z)=\left(x, \frac{i}{\sqrt{2}}, \frac{-i}{\sqrt{2}}\right) +$$ + +to get + +$$ +\begin{aligned} +& x P(x)+\frac{i}{\sqrt{2}}\left(P\left(\frac{i}{\sqrt{2}}\right)-P\left(\frac{-i}{\sqrt{2}}\right)\right) \\ += & \frac{1}{2} x(P(x-i / \sqrt{2})+P(x+i / \sqrt{2})+P(\sqrt{2} i)) +\end{aligned} +$$ + +which in particular implies that + +$$ +P\left(x+\frac{i}{\sqrt{2}}\right)+P\left(x-\frac{i}{\sqrt{2}}\right)-2 P(x) \equiv P(\sqrt{2} i) +$$ + +identically in $x$. The left-hand side is a second-order finite difference in $x$ (up to scaling the argument), and the right-hand side is constant, so this implies $\operatorname{deg} P \leq 2$. + +Since $P$ is even and $\operatorname{deg} P \leq 2$, we must have $P(x)=c x^{2}+d$ for some real numbers $c$ and $d$. A quick check now gives the answer $P(x)=c\left(x^{2}+3\right)$ which all work. + diff --git a/USAMO/md/en-USAMO-2020-notes.md b/USAMO/md/en-USAMO-2020-notes.md new file mode 100644 index 0000000000000000000000000000000000000000..417bebcba60a93a67c380c8f4e65692a2c6bb3a3 --- /dev/null +++ b/USAMO/md/en-USAMO-2020-notes.md @@ -0,0 +1,420 @@ +# USAMO 2020 Solution Notes + +Evan Chen《陳誼廷》 + +15 April 2024 + +This is a compilation of solutions for the 2020 USAMO. The ideas of the solution are a mix of my own work, the solutions provided by the competition organizers, and solutions found by the community. However, all the writing is maintained by me. + +These notes will tend to be a bit more advanced and terse than the "official" solutions from the organizers. In particular, if a theorem or technique is not known to beginners but is still considered "standard", then I often prefer to use this theory anyways, rather than try to work around or conceal it. For example, in geometry problems I typically use directed angles without further comment, rather than awkwardly work around configuration issues. Similarly, sentences like "let $\mathbb{R}$ denote the set of real numbers" are typically omitted entirely. + +Corrections and comments are welcome! + +## Contents + +0 Problems ..... 2 +1 Solutions to Day 1 ..... 3 +1.1 USAMO 2020/1, proposed by Zuming Feng ..... 3 +1.2 USAMO 2020/2, proposed by Alex Zhai ..... 5 +1.3 USAMO 2020/3, proposed by Richard Stong, Toni Bluher ..... 7 +2 Solutions to Day 2 ..... 9 +2.1 USAMO 2020/4, proposed by Ankan Bhattacharya ..... 9 +2.2 USAMO 2020/5, proposed by Carl Schildkraut ..... 11 +2.3 USAMO 2020/6, proposed by David Speyer, Kiran Kedlaya ..... 13 + +## §0 Problems + +1. Let $A B C$ be a fixed acute triangle inscribed in a circle $\omega$ with center $O$. A variable point $X$ is chosen on minor arc $A B$ of $\omega$, and segments $C X$ and $A B$ meet at $D$. Denote by $O_{1}$ and $O_{2}$ the circumcenters of triangles $A D X$ and $B D X$, respectively. Determine all points $X$ for which the area of triangle $O O_{1} O_{2}$ is minimized. +2. An empty $2020 \times 2020 \times 2020$ cube is given, and a $2020 \times 2020$ grid of square unit cells is drawn on each of its six faces. A beam is a $1 \times 1 \times 2020$ rectangular prism. Several beams are placed inside the cube subject to the following conditions: + +- The two $1 \times 1$ faces of each beam coincide with unit cells lying on opposite faces of the cube. (Hence, there are $3 \cdot 2020^{2}$ possible positions for a beam.) +- No two beams have intersecting interiors. +- The interiors of each of the four $1 \times 2020$ faces of each beam touch either a face of the cube or the interior of the face of another beam. + +What is the smallest positive number of beams that can be placed to satisfy these conditions? +3. Let $p$ be an odd prime. An integer $x$ is called a quadratic non-residue if $p$ does not divide $x-t^{2}$ for any integer $t$. +Denote by $A$ the set of all integers $a$ such that $1 \leq a0$. +Claim - If $N_{z}>0$, then we have $N_{x}+N_{y} \geq n$. + +Proof. Again orient the cube so the z-plane touches the ground. We see that for each of the $n$ layers of the cube (from top to bottom), there is at least one $x$-beam or $y$-beam. (Pictorially, some of the $x$ and $y$ beams form a "staircase".) This completes the proof. + +Proceeding in a similar fashion, we arrive at the three relations + +$$ +\begin{aligned} +& N_{x}+N_{y} \geq n \\ +& N_{y}+N_{z} \geq n \\ +& N_{z}+N_{x} \geq n +\end{aligned} +$$ + +Summing gives $N_{x}+N_{y}+N_{z} \geq 3 n / 2$ too. +Remark. The problem condition has the following "physics" interpretation. Imagine the cube is a metal box which is sturdy enough that all beams must remain orthogonal to the faces of the box (i.e. the beams cannot spin). Then the condition of the problem is exactly what is needed so that, if the box is shaken or rotated, the beams will not move. + +Remark. Walter Stromquist points out that the number of constructions with 3030 beams is actually enormous: not dividing out by isometries, the number is $(2 \cdot 1010!)^{3}$. + +## §1.3 USAMO 2020/3, proposed by Richard Stong, Toni Bluher + +Available online at https://aops.com/community/p15952782. + +## Problem statement + +Let $p$ be an odd prime. An integer $x$ is called a quadratic non-residue if $p$ does not divide $x-t^{2}$ for any integer $t$. + +Denote by $A$ the set of all integers $a$ such that $1 \leq a0}$ instead. + +## §2.3 USAMO 2021/6, proposed by Ankan Bhattacharya + +Available online at https://aops.com/community/p21498548. + +## Problem statement + +Let $A B C D E F$ be a convex hexagon satisfying $\overline{A B}\|\overline{D E}, \overline{B C}\| \overline{E F}, \overline{C D} \| \overline{F A}$, and + +$$ +A B \cdot D E=B C \cdot E F=C D \cdot F A +$$ + +Let $X, Y$, and $Z$ be the midpoints of $\overline{A D}, \overline{B E}$, and $\overline{C F}$. Prove that the circumcenter of $\triangle A C E$, the circumcenter of $\triangle B D F$, and the orthocenter of $\triangle X Y Z$ are collinear. + +We present two solutions. + +Parallelogram solution found by contestants. Note that the following figure is intentionally not drawn to scale, to aid legibility. We construct parallelograms $A B C E^{\prime}$, etc as shown. Note that this gives two congruent triangles $A^{\prime} C^{\prime} E^{\prime}$ and $B^{\prime} D^{\prime} F^{\prime}$. (Assuming that triangle $X Y Z$ is non-degenerate, the triangles $A^{\prime} C^{\prime} E^{\prime}$ and $B^{\prime} D^{\prime} F^{\prime}$ will also be nondegenerate.) +![](https://cdn.mathpix.com/cropped/2024_11_19_128c4918e9847b51d7d1g-13.jpg?height=777&width=809&top_left_y=1279&top_left_x=629) + +Claim - If $A B \cdot D E=B C \cdot E F=C D \cdot F A=k$, then the circumcenters of $A C E$ and $A^{\prime} C^{\prime} E^{\prime}$ coincide. + +Proof. The power of $A$ to $\left(A^{\prime} C^{\prime} E^{\prime}\right)$ is $A E^{\prime} \cdot A C^{\prime}=B C \cdot E F=k$; same for $C$ and $E$. +![](https://cdn.mathpix.com/cropped/2024_11_19_128c4918e9847b51d7d1g-14.jpg?height=992&width=495&top_left_y=246&top_left_x=780) + +Claim - Triangle $X Y Z$ is the vector average of the (congruent) medial triangles of triangles $A^{\prime} C^{\prime} E^{\prime}$ and $B^{\prime} D^{\prime} F^{\prime}$. + +Proof. If $M$ and $N$ are the midpoints of $\overline{C^{\prime} E^{\prime}}$ and $\overline{B^{\prime} F^{\prime}}$, then $X$ is the midpoint of $\overline{M N}$ by vector calculation: + +$$ +\begin{aligned} +\frac{\vec{M}+\vec{N}}{2} & =\frac{\frac{\vec{C}^{\prime}+\vec{E}^{\prime}}{2}+\frac{\vec{B}^{\prime}+\vec{F}^{\prime}}{2}}{2} \\ +& =\frac{\overrightarrow{C^{\prime}}+\vec{E}^{\prime}+\vec{B}^{\prime}+\vec{F}^{\prime}}{4} \\ +& =\frac{(\vec{A}+\vec{E}-\vec{F})+(\vec{C}+\vec{A}-\vec{B})+(\vec{D}+\vec{F}-\vec{E})+(\vec{B}+\vec{D}-\vec{C})}{4} \\ +& =\frac{\vec{A}+\vec{D}}{2}=\vec{X} . +\end{aligned} +$$ + +Hence the orthocenter of $X Y Z$ is the midpoint of the orthocenters of the medial triangles of $A^{\prime} C^{\prime} E^{\prime}$ and $B^{\prime} D^{\prime} F^{\prime}$ - that is, their circumcenters. + +【 Author's solution. Call $M N P$ and $U V W$ the medial triangles of $A C E$ and $B D F$. +![](https://cdn.mathpix.com/cropped/2024_11_19_128c4918e9847b51d7d1g-15.jpg?height=966&width=1100&top_left_y=248&top_left_x=478) + +Claim - In trapezoid $A B D E$, the perpendicular bisector of $\overline{X Y}$ is the same as the perpendicular bisector of the midline $\overline{W N}$. + +Proof. This is true for any trapezoid: because $W X=\frac{1}{2} A B=Y N$. + +Claim - The points $V, W, M, N$ are cyclic. + +Proof. By power of a point from $Y$, since + +$$ +W Y \cdot Y N=\frac{1}{2} D E \cdot \frac{1}{2} A B=\frac{1}{2} E F \cdot \frac{1}{2} B C=V Y \cdot Y M +$$ + +Applying all the cyclic variations of the above two claims, it follows that all six points $U, V, W, M, N, P$ are concyclic, and the center of that circle coincides with the circumcenter of $\triangle X Y Z$. + +Remark. It is also possible to implement ideas from the first solution here, by showing all six midpoints have equal power to $(X Y Z)$. + +Claim - The orthocenter of $X Y Z$ is the midpoint of the circumcenters of $\triangle A C E$ and $\triangle B D F$. + +Proof. Apply complex numbers with the unit circle coinciding with the circumcircle of $N V P W M U$. Then + +$$ +\begin{aligned} +\operatorname{orthocenter}(X Y Z) & =x+y+z=\frac{a+b+c+d+e+f}{2} \\ +\operatorname{circumcenter}(A C E) & =\operatorname{orthocenter}(M N P) +\end{aligned} +$$ + +$$ +=m+n+p=\frac{c+e}{2}+\frac{e+a}{2}+\frac{a+c}{2}=a+c+e +$$ + +circumcenter $(B D F)=$ orthocenter $(U V W)$ + +$$ +=u+v+w=\frac{d+f}{2}+\frac{f+b}{2}+\frac{b+d}{2}=b+d+f +$$ + diff --git a/USAMO/md/en-USAMO-2022-notes.md b/USAMO/md/en-USAMO-2022-notes.md new file mode 100644 index 0000000000000000000000000000000000000000..6d94808d3d84312355c418a380d43abfbac9415d --- /dev/null +++ b/USAMO/md/en-USAMO-2022-notes.md @@ -0,0 +1,372 @@ +# USAMO 2022 Solution Notes + +Evan Chen《陳誼廷》 + +15 April 2024 + +This is a compilation of solutions for the 2022 USAMO. The ideas of the solution are a mix of my own work, the solutions provided by the competition organizers, and solutions found by the community. However, all the writing is maintained by me. + +These notes will tend to be a bit more advanced and terse than the "official" solutions from the organizers. In particular, if a theorem or technique is not known to beginners but is still considered "standard", then I often prefer to use this theory anyways, rather than try to work around or conceal it. For example, in geometry problems I typically use directed angles without further comment, rather than awkwardly work around configuration issues. Similarly, sentences like "let $\mathbb{R}$ denote the set of real numbers" are typically omitted entirely. + +Corrections and comments are welcome! + +## Contents + +0 Problems ..... 2 +1 Solutions to Day 1 ..... 3 +1.1 USAMO 2022/1, proposed by Ankan Bhattacharya ..... 3 +1.2 USAMO 2022/2, proposed by Ankan Bhattacharya ..... 4 +1.3 USAMO 2022/3, proposed by Hung-Hsun Hans Yu ..... 6 +2 Solutions to Day 2 ..... 8 +2.1 USAMO 2022/4, proposed by Holden Mui ..... 8 +2.2 USAMO 2022/5, proposed by Gabriel Carroll ..... 9 +2.3 USAMO 2022/6, proposed by Yannick Yao ..... 10 + +## §0 Problems + +1. Let $a$ and $b$ be positive integers. Every cell of an $(a+b+1) \times(a+b+1)$ grid is colored either amber or bronze such that there are at least $a^{2}+a b-b$ amber cells and at least $b^{2}+a b-a$ bronze cells. Prove that it is possible to choose $a$ amber cells and $b$ bronze cells such that no two of the $a+b$ chosen cells lie in the same row or column. +2. Let $b \geq 2$ and $w \geq 2$ be fixed integers, and $n=b+w$. Given are $2 b$ identical black rods and $2 w$ identical white rods, each of side length 1. + +We assemble a regular $2 n$-gon using these rods so that parallel sides are the same color. Then, a convex $2 b$-gon $B$ is formed by translating the black rods, and a convex $2 w$-gon $W$ is formed by translating the white rods. An example of one way of doing the assembly when $b=3$ and $w=2$ is shown below, as well as the resulting polygons $B$ and $W$. +![](https://cdn.mathpix.com/cropped/2024_11_19_0824a9a7bcda8c860a08g-02.jpg?height=609&width=1013&top_left_y=952&top_left_x=571) + +Prove that the difference of the areas of $B$ and $W$ depends only on the numbers $b$ and $w$, and not on how the $2 n$-gon was assembled. +3. Solve over positive real numbers the functional equation + +$$ +f(x)=f(f(f(x))+y)+f(x f(y)) f(x+y) +$$ + +4. Find all pairs of primes $(p, q)$ for which $p-q$ and $p q-q$ are both perfect squares. +5. A function $f: \mathbb{R} \rightarrow \mathbb{R}$ is essentially increasing if $f(s) \leq f(t)$ holds whenever $s \leq t$ are real numbers such that $f(s) \neq 0$ and $f(t) \neq 0$. + +Find the smallest integer $k$ such that for any 2022 real numbers $x_{1}, x_{2}, \ldots, x_{2022}$, there exist $k$ essentially increasing functions $f_{1}, \ldots, f_{k}$ such that + +$$ +f_{1}(n)+f_{2}(n)+\cdots+f_{k}(n)=x_{n} \quad \text { for every } n=1,2, \ldots, 2022 +$$ + +6. There are 2022 users on a social network called Mathbook, and some of them are Mathbook-friends. (On Mathbook, friendship is always mutual and permanent.) +Starting now, Mathbook will only allow a new friendship to be formed between two users if they have at least two friends in common. What is the minimum number of friendships that must already exist so that every user could eventually become friends with every other user? + +## §1 Solutions to Day 1 + +## §1.1 USAMO 2022/1, proposed by Ankan Bhattacharya + +Available online at https://aops.com/community/p24774812. + +## Problem statement + +Let $a$ and $b$ be positive integers. Every cell of an $(a+b+1) \times(a+b+1)$ grid is colored either amber or bronze such that there are at least $a^{2}+a b-b$ amber cells and at least $b^{2}+a b-a$ bronze cells. Prove that it is possible to choose $a$ amber cells and $b$ bronze cells such that no two of the $a+b$ chosen cells lie in the same row or column. + +Claim - There exists a transversal $T_{a}$ with at least $a$ amber cells. Analogously, there exists a transversal $T_{b}$ with at least $b$ bronze cells. + +Proof. If one picks a random transversal, the expected value of the number of amber cells is at least + +$$ +\frac{a^{2}+a b-b}{a+b+1}=(a-1)+\frac{1}{a+b+1}>a-1 +$$ + +Now imagine we transform $T_{a}$ to $T_{b}$ in some number of steps, by repeatedly choosing cells $c$ and $c^{\prime}$ and swapping them with the two other corners of the rectangle formed by their row/column, as shown in the figure. +![](https://cdn.mathpix.com/cropped/2024_11_19_0824a9a7bcda8c860a08g-03.jpg?height=243&width=809&top_left_y=1466&top_left_x=629) + +By "discrete intermediate value theorem", the number of amber cells will be either $a$ or $a+1$ at some point during this transformation. This completes the proof. + +## §1.2 USAMO 2022/2, proposed by Ankan Bhattacharya + +Available online at https://aops.com/community/p24775345. + +## Problem statement + +Let $b \geq 2$ and $w \geq 2$ be fixed integers, and $n=b+w$. Given are $2 b$ identical black rods and $2 w$ identical white rods, each of side length 1. + +We assemble a regular $2 n$-gon using these rods so that parallel sides are the same color. Then, a convex $2 b$-gon $B$ is formed by translating the black rods, and a convex $2 w$-gon $W$ is formed by translating the white rods. An example of one way of doing the assembly when $b=3$ and $w=2$ is shown below, as well as the resulting polygons $B$ and $W$. +![](https://cdn.mathpix.com/cropped/2024_11_19_0824a9a7bcda8c860a08g-04.jpg?height=623&width=1021&top_left_y=859&top_left_x=521) + +Prove that the difference of the areas of $B$ and $W$ depends only on the numbers $b$ and $w$, and not on how the $2 n$-gon was assembled. + +We are going to prove that one may swap a black rod with an adjacent white rod (as well as the rods parallel to them) without affecting the difference in the areas of $B-W$. Let $\vec{u}$ and $\vec{v}$ denote the originally black and white vectors that were adjacent on the $2 n$-gon and are now going to be swapped. Let $\vec{x}$ denote the sum of all the other black vectors between $\vec{u}$ and $-\vec{u}$, and define $\vec{y}$ similarly. See the diagram below, where $B_{0}$ and $W_{0}$ are the polygons before the swap, and $B_{1}$ and $W_{1}$ are the resulting changed polygons. +![](https://cdn.mathpix.com/cropped/2024_11_19_0824a9a7bcda8c860a08g-05.jpg?height=734&width=1220&top_left_y=227&top_left_x=424) + +Observe that the only change in $B$ and $W$ is in the parallelograms shown above in each diagram. Letting $\wedge$ denote the wedge product, we need to show that + +$$ +\vec{u} \wedge \vec{x}-\vec{v} \wedge \vec{y}=\vec{v} \wedge \vec{x}-\vec{u} \wedge \vec{y} +$$ + +which can be rewritten as + +$$ +(\vec{u}-\vec{v}) \wedge(\vec{x}+\vec{y})=0 +$$ + +In other words, it would suffice to show $\vec{u}-\vec{v}$ and $\vec{x}+\vec{y}$ are parallel. (Students not familiar with wedge products can replace every $\wedge$ with the cross product $\times$ instead.) + +Claim - Both $\vec{u}-\vec{v}$ and $\vec{x}+\vec{y}$ are perpendicular to vector $\vec{u}+\vec{v}$. + +Proof. We have $(\vec{u}-\vec{v}) \perp(\vec{u}+\vec{v})$ because $\vec{u}$ and $\vec{v}$ are the same length. +For the other perpendicularity, note that $\vec{u}+\vec{v}+\vec{x}+\vec{y}$ traces out a diameter of the circumcircle of the original $2 n$-gon; call this diameter $A B$, so + +$$ +A+\vec{u}+\vec{v}+\vec{x}+\vec{y}=B +$$ + +Now point $A+\vec{u}+\vec{v}$ is a point on this semicircle, which means (by the inscribed angle theorem) the angle between $\vec{u}+\vec{v}$ and $\vec{x}+\vec{y}$ is $90^{\circ}$. + +## §1.3 USAMO 2022/3, proposed by Hung-Hsun Hans Yu + +Available online at https://aops.com/community/p24774907. + +## Problem statement + +Solve over positive real numbers the functional equation + +$$ +f(x)=f(f(f(x))+y)+f(x f(y)) f(x+y) . +$$ + +The answer is $f(x) \equiv c / x$ for any $c>0$. This works, so we'll prove this is the only solution. The following is based on the solution posted by pad on AoPS. + +In what follows, $f^{n}$ as usual denotes $f$ iterated $n$ times, and $P(x, y)$ is the given statement. Also, we introduce the notation $Q$ for the statement + +$$ +Q(a, b): \quad f(a) \geq f(b) \Longrightarrow f(f(b)) \geq a . +$$ + +To see why this statement $Q$ is true, assume for contradiction that $a>f(f(b))$; then consider $P(b, a-f(f(b)))$ to get a contradiction. + +The main idea of the problem is the following: +Claim - Any function $f: \mathbb{R}_{>0} \rightarrow \mathbb{R}_{>0}$ obeying statement $Q$ satisfies $f^{2}(x)=f^{4}(x)$. +Proof. From $Q(t, t)$ we get + +$$ +f^{2}(t) \geq t \quad \text { for all } t>0 +$$ + +So this already implies $f^{4}(x) \geq f^{2}(x)$ by choosing $t=f^{2}(x)$. It also gives $f(x) \leq f^{3}(x) \leq$ $f^{5}(x)$ by choosing $t=f(x), t=f^{3}(x)$. + +Then $Q\left(f^{4}(x), x\right)$ is valid and gives $f^{2}(x) \geq f^{4}(x)$, as needed. + +Claim - The function $f$ is injective. +Proof. Suppose $f(u)=f(v)$ for some $u>v$. From $Q(u, v)$ and $Q(v, u)$ we have $f^{2}(v) \geq u$ and $f^{2}(u) \geq v$. Note that for all $x>0$ we have statements + +$$ +\begin{aligned} +& P\left(f^{2}(x), u\right) \Longrightarrow f^{3}(x)=f(x+u)+f(x f(u)) f(x+u)=(1+f(x f(u))) f(x+u) \\ +& P\left(f^{2}(x), v\right) \Longrightarrow f^{3}(x)=f(x+v)+f(x f(v)) f(x+v)=(1+f(x f(v))) f(x+v) . +\end{aligned} +$$ + +It follows that $f(x+u)=f(x+v)$ for all $x>0$. +This means that $f$ is periodic with period $T=u-v>0$. However, this is incompatible with $Q$, because we would have $Q(1+n T, 1)$ for all positive integers $n$, which is obviously absurd. + +Since $f$ is injective, we obtain that $f^{2}(x)=x$. Thus $P(x, y)$ now becomes the statement + +$$ +P(x, y): \quad f(x)=f(x+y) \cdot[1+f(x f(y))] . +$$ + +In particular + +$$ +P(1, y) \Longrightarrow f(1+y)=\frac{f(1)}{1+y} +$$ + +so $f$ is determined on inputs greater than 1 . Finally, if $a, b>1$ we get + +$$ +P(a, b) \Longrightarrow \frac{1}{a}=\frac{1}{a+b} \cdot\left[1+f\left(\frac{a}{b} f(1)\right)\right] +$$ + +which is enough to determine $f$ on all inputs, by varying $(a, b)$. + +## §2 Solutions to Day 2 + +## §2.1 USAMO 2022/4, proposed by Holden Mui + +Available online at https://aops.com/community/p24774670. + +## Problem statement + +Find all pairs of primes $(p, q)$ for which $p-q$ and $p q-q$ are both perfect squares. + +The answer is $(3,2)$ only. This obviously works so we focus on showing it is the only one. + +『 Approach using difference of squares (from author). Set + +$$ +\begin{aligned} +a^{2} & =p-q \\ +b^{2} & =p q-q . +\end{aligned} +$$ + +Note that $0N$ and choose $x_{n}=-n$ for each $n=1, \ldots, N$. Now for each index $1 \leq n \leq N$, define + +$$ +S(n)=\left\{\text { indices } i \text { for which } f_{i}(n) \neq 0\right\} \subseteq\{1, \ldots, k\} +$$ + +As each $S(n t)$ is nonempty, by pigeonhole, two $S(n)$ 's coincide, say $S(n)=S\left(n^{\prime}\right)$ for $nx_{n^{\prime}}$ in that case due to the essentially increasing condition. + +【 Construction. It suffices to do $N=2^{k}-1$. Rather than drown the reader in notation, we'll just illustrate an example of the (inductive) construction for $k=4$. Empty cells are zero. + +| | $f_{1}$ | $f_{2}$ | $f_{3}$ | $f_{4}$ | +| ---: | ---: | ---: | ---: | ---: | +| $x_{1}=3$ | 3 | | | | +| $x_{2}=1$ | 10 | -9 | | | +| $x_{3}=4$ | | 4 | | | +| $x_{4}=1$ | 100 | 200 | $-\mathbf{2 9 9}$ | | +| $x_{5}=5$ | | 200 | -195 | | +| $x_{6}=9$ | 100 | | -91 | | +| $x_{7}=2$ | | | 2 | | +| $x_{8}=6$ | 1000 | 2000 | 4000 | $-\mathbf{6 9 9 4}$ | +| $x_{9}=5$ | | 2000 | 4000 | -5995 | +| $x_{10}=3$ | 1000 | | 4000 | -4997 | +| $x_{11}=5$ | | | 4000 | -3995 | +| $x_{12}=8$ | 1000 | 2000 | | -2992 | +| $x_{13}=9$ | | 2000 | | -1991 | +| $x_{14}=7$ | 1000 | | | -993 | +| $x_{15}=9$ | | | | 9 | + +The general case is handled in the same way with powers of 10 replaced by powers of $B$, for a sufficiently large number $B$. + +## §2.3 USAMO 2022/6, proposed by Yannick Yao + +Available online at https://aops.com/community/p24774626. + +## Problem statement + +There are 2022 users on a social network called Mathbook, and some of them are Mathbook-friends. (On Mathbook, friendship is always mutual and permanent.) + +Starting now, Mathbook will only allow a new friendship to be formed between two users if they have at least two friends in common. What is the minimum number of friendships that must already exist so that every user could eventually become friends with every other user? + +With 2022 replaced by $n$, the answer is $\left\lceil\frac{3}{2} n\right\rceil-2$. +【 Terminology. Standard graph theory terms: starting from a graph $G$ on $n$ vertices, we're allowed to take any $C_{4}$ in the graph and complete it to a $K_{4}$. The problem asks the minimum number of edges needed so that this operation lets us transform $G$ to $K_{n}$. + +【T Construction. For even $n$, start with an edge $a b$, and then create $n / 2-1$ copies of $C_{4}$ that use $a b$ as an edge, as shown below for $n=14$ (six copies of $C_{4}$ ). +![](https://cdn.mathpix.com/cropped/2024_11_19_0824a9a7bcda8c860a08g-10.jpg?height=563&width=818&top_left_y=1306&top_left_x=625) + +This can be completed into $K_{n}$ by first completing the $n / 2-1 C_{4}$ 's into $K_{4}$, then connecting red vertices to every grey vertex, and then finishing up. + +The construction for odd $n$ is the same except with one extra vertex $c$ which is connected to both $a$ and $b$. + +【 Bound. Notice that additional operations or connections can never hurt. So we will describe a specific algorithm that performs operations on the graph until no more operations are possible. This means that if this algorithm terminates with anything other $G=K_{n}$, the graph was never completable to $K_{n}$ to begin with. + +The algorithm uses the following data: it keeps a list $\mathcal{C}$ of cliques of $G$, and a labeling $\mathcal{L}: E(G) \rightarrow \mathcal{C}$ which assigns to every edge one of the cliques that contains it. + +- Initially, $\mathcal{C}$ consists of one $K_{2}$ for every edge of $G$, and each edge is labeled in the obvious way. +- At each step, the algorithm arbitrarily takes any $C_{4}=a b c d$ whose four edges $a b$, $b c, c d, d a$ do not all have the same label. Consider these labels that appear (at least two, and up to four), and let $V$ be the union of all vertices in any of these 2-4 cliques. +- Do the following graph operations: connect $a c$ and $b d$, then connect every vertex in $V-\{a, b, c, d\}$ to each of $\{a, b, c, d\}$. Finally, complete this to a clique on $V$. +- Update $\mathcal{C}$ by merging these 2-4 cliques into a single clique $K_{V}$. +- Update $\mathcal{L}$ by replacing every edge that was labeled with one of these 2-4 cliques with the label $K_{V}$. Also, update every newly created edge to have label $K_{V}$. However, if there were existing edges not labeled with one of the 2-4 cliques, then we do not update these! +- Stop once every $C_{4}$ has only one label appearing among its edges. When this occurs, no operations are possible at all on the graph. + +A few steps of the process are illustrated below for a graph on six vertices with nine initial edges. There are initially nine $K_{2}$ 's labeled A, B, ..., I. Original edges are always bolder than added edges. The relabeled edges in each step are highlighted in color. Notice how we need an entirely separate operation to get $G$ to become L, even though no new edges are drawn in the graph. +![](https://cdn.mathpix.com/cropped/2024_11_19_0824a9a7bcda8c860a08g-11.jpg?height=912&width=1223&top_left_y=1257&top_left_x=428) + +Step 2: Operate on 1235. +Merges CIJ into K. +$\theta(\mathrm{K})=6$ +Step 3: Operate on 2356. +Merges GK into L. +$\theta(\mathrm{L})=7$ + +As we remarked, if the graph is going to be completable to $K_{n}$ at all, then this algorithm must terminate with $\mathcal{C}=\left\{K_{n}\right\}$. We will use this to prove our bound. + +We proceed by induction in the following way. For a clique $K$, let $\theta(K)$ denote the number of edges of the original graph $G$ which are labeled by $K$ (this does not include new edges added by the algorithm); hence the problem amounts to estimating how small $\theta\left(K_{n}\right)$ can be. We are trying to prove: + +Claim - At any point in the operation, if $K$ is a clique in the cover $\mathcal{C}$, then + +$$ +\theta(K) \geq \frac{3|K|}{2}-2 . +$$ + +where $|K|$ is the number of vertices in $K$. +Proof. By induction on the time step of the algorithm. The base case is clear, because then $K$ is just a single edge of $G$, so $\theta(K)=1$ and $|K|=2$. + +The inductive step is annoying casework based on the how the merge occurred. Let $C_{4}=a b c d$ be the 4 -cycle operated on. In general, the $\theta$ value of a newly created $K$ is exactly the sum of the $\theta$ values of the merged cliques, by definition. Meanwhile, $|K|$ is the number of vertices in the union of the merged cliques; so it's the sum of the sizes of these cliques minus some error due to overcounting of vertices appearing more than once. To be explicit: + +- Suppose we merged four cliques $W, X, Y, Z$. By definition, + +$$ +\begin{aligned} +\theta(K) & =\theta(W)+\theta(X)+\theta(Y)+\theta(Z) \\ +& \geq \frac{3}{2}(|W|+|X|+|Y|+|Z|)-8=\frac{3}{2}(|W|+|X|+|Y|+|Z|-4)-2 . +\end{aligned} +$$ + +On the other hand $|K| \leq|W|+|X|+|Y|+|Z|-4$; the -4 term comes from each of $\{a, b, c, d\}$ being in two (or more) of $\{W, X, Y, Z\}$. So this case is OK. + +- Suppose we merged three cliques $X, Y, Z$. By definition, + +$$ +\begin{aligned} +\theta(K) & =\theta(X)+\theta(Y)+\theta(Z) \\ +& \geq \frac{3}{2}(|X|+|Y|+|Z|)-6=\frac{3}{2}\left(|X|+|Y|+|Z|-\frac{8}{3}\right)-2 . +\end{aligned} +$$ + +On the other hand, $|K| \leq|X|+|Y|+|Z|-3$, since at least 3 of $\{a, b, c, d\}$ are repeated among $X, Y, Z$. Note in this case the desired inequality is actually strict. + +- Suppose we merged two cliques $Y, Z$. By definition, + +$$ +\begin{aligned} +\theta(K) & =\theta(Y)+\theta(Z) \\ +& \geq \frac{3}{2}(|Y|+|Z|)-4=\frac{3}{2}\left(|Y|+|Z|-\frac{4}{3}\right)-2 . +\end{aligned} +$$ + +On the other hand, $|K| \leq|Y|+|Z|-2$, since at least 2 of $\{a, b, c, d\}$ are repeated among $Y, Z$. Note in this case the desired inequality is actually strict. + +Remark. Several subtle variations of this method do not seem to work. + +- It does not seem possible to require the cliques in $\mathcal{C}$ to be disjoint, which is why it's necessary to introduce a label function $\mathcal{L}$ as well. +- It seems you do have to label the newly created edges, even though they do not count towards any $\theta$ value. Otherwise the termination of the algorithm doesn't tell you enough. +- Despite this, relabeling existing edges, like G in step 1 of the example, 1 seems to cause a lot of issues. The induction becomes convoluted if $\theta(K)$ is not exactly the sum of $\theta$-values of the subparts, while the disappearance of an edge from a clique will + +】 also break induction. + diff --git a/USAMO/md/en-USAMO-2023-notes.md b/USAMO/md/en-USAMO-2023-notes.md new file mode 100644 index 0000000000000000000000000000000000000000..810886b2699284b254c0d958a20284e433f69f8b --- /dev/null +++ b/USAMO/md/en-USAMO-2023-notes.md @@ -0,0 +1,445 @@ +# USAMO 2023 Solution Notes + +Evan ChEn《陳誼廷》 + +15 April 2024 + +This is a compilation of solutions for the 2023 USAMO. The ideas of the solution are a mix of my own work, the solutions provided by the competition organizers, and solutions found by the community. However, all the writing is maintained by me. + +These notes will tend to be a bit more advanced and terse than the "official" solutions from the organizers. In particular, if a theorem or technique is not known to beginners but is still considered "standard", then I often prefer to use this theory anyways, rather than try to work around or conceal it. For example, in geometry problems I typically use directed angles without further comment, rather than awkwardly work around configuration issues. Similarly, sentences like "let $\mathbb{R}$ denote the set of real numbers" are typically omitted entirely. + +Corrections and comments are welcome! + +## Contents + +0 Problems ..... 2 +1 Solutions to Day 1 ..... 3 +1.1 USAMO 2023/1, proposed by Holden Mui ..... 3 +1.2 USAMO 2023/2, proposed by Carl Schildkraut ..... 7 +1.3 USAMO 2023/3, proposed by Holden Mui ..... 8 +2 Solutions to Day 2 ..... 12 +2.1 USAMO 2023/4, proposed by Carl Schildkraut ..... 12 +2.2 USAMO 2023/5, proposed by Ankan Bhattacharya ..... 13 +2.3 USAMO 2023/6, proposed by Zack Chroman ..... 15 + +## §0 Problems + +1. In an acute triangle $A B C$, let $M$ be the midpoint of $\overline{B C}$. Let $P$ be the foot of the perpendicular from $C$ to $A M$. Suppose that the circumcircle of triangle $A B P$ intersects line $B C$ at two distinct points $B$ and $Q$. Let $N$ be the midpoint of $\overline{A Q}$. Prove that $N B=N C$. +2. Solve over the positive real numbers the functional equation + +$$ +f(x y+f(x))=x f(y)+2 +$$ + +3. Consider an $n$-by- $n$ board of unit squares for some odd positive integer $n$. We say that a collection $C$ of identical dominoes is a maximal grid-aligned configuration on the board if $C$ consists of $\left(n^{2}-1\right) / 2$ dominoes where each domino covers exactly two neighboring squares and the dominoes don't overlap: $C$ then covers all but one square on the board. We are allowed to slide (but not rotate) a domino on the board to cover the uncovered square, resulting in a new maximal gridaligned configuration with another square uncovered. Let $k(C)$ be the number of distinct maximal grid-aligned configurations obtainable from $C$ by repeatedly sliding dominoes. +Find all possible values of $k(C)$ as a function of $n$. +4. Positive integers $a$ and $N$ are fixed, and $N$ positive integers are written on a blackboard. Alice and Bob play the following game. On Alice's turn, she must replace some integer $n$ on the board with $n+a$, and on Bob's turn he must replace some even integer $n$ on the board with $n / 2$. Alice goes first and they alternate turns. If on his turn Bob has no valid moves, the game ends. +After analyzing the $N$ integers on the board, Bob realizes that, regardless of what moves Alice makes, he will be able to force the game to end eventually. Show that, in fact, for this value of $a$ and these $N$ integers on the board, the game is guaranteed to end regardless of Alice's or Bob's moves. +5. Let $n \geq 3$ be an integer. We say that an arrangement of the numbers $1,2, \ldots, n^{2}$ in an $n \times n$ table is row-valid if the numbers in each row can be permuted to form an arithmetic progression, and column-valid if the numbers in each column can be permuted to form an arithmetic progression. +For what values of $n$ is it possible to transform any row-valid arrangement into a column-valid arrangement by permuting the numbers in each row? +6. Let $A B C$ be a triangle with incenter $I$ and excenters $I_{a}, I_{b}, I_{c}$ opposite $A, B$, and $C$, respectively. Given an arbitrary point $D$ on the circumcircle of $\triangle A B C$ that does not lie on any of the lines $I I_{a}, I_{b} I_{c}$, or $B C$, suppose the circumcircles of $\triangle D I I_{a}$ and $\triangle D I_{b} I_{c}$ intersect at two distinct points $D$ and $F$. If $E$ is the intersection of lines $D F$ and $B C$, prove that $\angle B A D=\angle E A C$. + +## §1 Solutions to Day 1 + +## §1.1 USAMO 2023/1, proposed by Holden Mui + +Available online at https://aops.com/community/p27349297. + +## Problem statement + +In an acute triangle $A B C$, let $M$ be the midpoint of $\overline{B C}$. Let $P$ be the foot of the perpendicular from $C$ to $A M$. Suppose that the circumcircle of triangle $A B P$ intersects line $B C$ at two distinct points $B$ and $Q$. Let $N$ be the midpoint of $\overline{A Q}$. Prove that $N B=N C$. + +We show several different approaches. In all solutions, let $D$ denote the foot of the altitude from $A$. +![](https://cdn.mathpix.com/cropped/2024_11_19_5f0e796e43a11586ccbfg-03.jpg?height=892&width=995&top_left_y=999&top_left_x=536) + +【 Most common synthetic approach. The solution hinges on the following claim: +Claim - $Q$ coincides with the reflection of $D$ across $M$. + +Proof. Note that $\measuredangle A D C=\measuredangle A P C=90^{\circ}$, so $A D P C$ is cyclic. Then by power of a point (with the lengths directed), + +$$ +M B \cdot M Q=M A \cdot M P=M C \cdot M D +$$ + +Since $M B=M C$, the claim follows. +It follows that $\overline{M N} \| \overline{A D}$, as $M$ and $N$ are respectively the midpoints of $\overline{A Q}$ and $\overline{D Q}$. Thus $\overline{M N} \perp \overline{B C}$, and so $N$ lies on the perpendicular bisector of $\overline{B C}$, as needed. + +Remark (David Lin). One can prove the main claim without power of a point as well, as follows: Let $R$ be the foot from $B$ to $\overline{A M}$, so $B R C P$ is a parallelogram. Note that $A B D R$ is cyclic, and hence + +$$ +\measuredangle D R M=\measuredangle D B A=Q B A=\measuredangle Q P A=\measuredangle Q P M +$$ + +Thus, $\overline{D R} \| \overline{P Q}$, so $D R Q P$ is also a parallelogram. + +## ब Synthetic approach with no additional points at all. + +## Claim - $\triangle B P C \sim \triangle A N M$ (oppositely oriented). + +Proof. We have $\triangle B M P \sim \triangle A M Q$ from the given concyclicity of $A B P Q$. Then + +$$ +\frac{B M}{B P}=\frac{A M}{A Q} \Longrightarrow \frac{2 B M}{B P}=\frac{A M}{A Q / 2} \Longrightarrow \frac{B C}{B P}=\frac{A M}{A N} +$$ + +implying the similarity (since $\measuredangle M A Q=\measuredangle B P M$ ). +This similarity gives us the equality of directed angles + +$$ +\measuredangle(B C, M N)=-\measuredangle(P C, A M)=90^{\circ} +$$ + +as desired. + +ब Synthetic approach using only the point $R$. Again let $R$ be the foot from $B$ to $\overline{A M}$, so $B R C P$ is a parallelogram. + +Claim - $A R Q C$ is cyclic; equivalently, $\triangle M A Q \sim \triangle M C R$. + +Proof. $M R \cdot M A=M P \cdot M A=M B \cdot M Q=M C \cdot M Q$. +Note that in $\triangle M C R$, the $M$-median is parallel to $\overline{C P}$ and hence perpendicular to $\overline{R M}$. The same should be true in $\triangle M A Q$ by the similarity, so $\overline{M N} \perp \overline{M Q}$ as needed. +\ Cartesian coordinates approach with power of a point. Suppose we set $B=(-1,0)$, $M=(0,0), C=(1,0)$, and $A=(a, b)$. One may compute: + +$$ +\begin{aligned} +\overleftrightarrow{A M}: 0 & =b x-a y \Longleftrightarrow y=\frac{b}{a} x \\ +\overleftrightarrow{C P}: 0 & =a(x-1)+b y \Longleftrightarrow y=-\frac{a}{b}(x-1)=-\frac{a}{b} x+\frac{a}{b} \\ +P & =\left(\frac{a^{2}}{a^{2}+b^{2}}, \frac{a b}{a^{2}+b^{2}}\right) +\end{aligned} +$$ + +Now note that + +$$ +A M=\sqrt{a^{2}+b^{2}}, \quad P M=\frac{a}{\sqrt{a^{2}+b^{2}}} +$$ + +together with power of a point + +$$ +A M \cdot P M=B M \cdot Q M +$$ + +to immediately deduce that $Q=(a, 0)$. Hence $N=(0, b / 2)$ and we're done. + +【 Cartesian coordinates approach without power of a point (outline). After computing $A$ and $P$ as above, one could also directly calculate + +$$ +\begin{aligned} +& \text { Perpendicular bisector of } \overline{A B}: y=-\frac{a+1}{b} x+\frac{a^{2}+b^{2}-1}{2 b} \\ +& \text { Perpendicular bisector of } \overline{P B}: y=-\left(\frac{2 a}{b}+\frac{b}{a}\right) x-\frac{b}{2 a} \\ +& \text { Perpendicular bisector of } \overline{P A}: y=-\frac{a}{b} x+\frac{a+a^{2}+b^{2}}{2 b} \\ +& \text { Circumcenter of } \triangle P A B=\left(-\frac{a+1}{2}, \frac{2 a^{2}+2 a+b^{2}}{2 b}\right) +\end{aligned} +$$ + +This is enough to extract the coordinates of $Q=(\bullet, 0)$, because $B=(-1,0)$ is given, and the $x$-coordinate of the circumcenter should be the average of the $x$-coordinates of $B$ and $Q$. In other words, $Q=(-a, 0)$. Hence, $N=\left(0, \frac{b}{2}\right)$, as needed. +\I III-advised barycentric approach (outline). Use reference triangle $A B C$. The $A$ median is parametrized by $(t: 1: 1)$ for $t \in \mathbb{R}$. So because of $\overline{C P} \perp \overline{A M}$, we are looking for $t$ such that + +$$ +\left(\frac{t \vec{A}+\vec{B}+\vec{C}}{t+2}-\vec{C}\right) \perp\left(A-\frac{\vec{B}+\vec{C}}{2}\right) +$$ + +This is equivalent to + +$$ +(t \vec{A}+\vec{B}-(t+1) \vec{C}) \perp(2 \vec{A}-\vec{B}-\vec{C}) +$$ + +By the perpendicularity formula for barycentric coordinates (EGMO 7.16), this is equivalent to + +$$ +\begin{aligned} +0 & =a^{2} t-b^{2} \cdot(3 t+2)+c^{2} \cdot(2-t) \\ +& =\left(a^{2}-3 b^{2}-c^{2}\right) t-2\left(b^{2}-c^{2}\right) \\ +\Longrightarrow t & =\frac{2\left(b^{2}-c^{2}\right)}{a^{2}-3 b^{2}-c^{2}} +\end{aligned} +$$ + +In other words, + +$$ +P=\left(2\left(b^{2}-c^{2}\right): a^{2}-3 b^{2}-c^{2}: a^{2}-3 b^{2}-c^{2}\right) . +$$ + +A long calculation gives $a^{2} y_{P} z_{P}+b^{2} z_{P} x_{P}+c^{2} x_{P} y_{P}=\left(a^{2}-3 b^{2}-c^{2}\right)\left(a^{2}-b^{2}+c^{2}\right)\left(a^{2}-\right.$ $\left.2 b^{2}-2 c^{2}\right)$. Together with $x_{P}+y_{P}+z_{P}=2 a^{2}-4 b^{2}-4 c^{2}$, this makes the equation of $(A B P)$ as + +$$ +0=-a^{2} y z-b^{2} z x-c^{2} x y+\frac{a^{2}-b^{2}+c^{2}}{2} z(x+y+z) +$$ + +To solve for $Q$, set $x=0$ to get to get + +$$ +a^{2} y z=\frac{a^{2}-b^{2}+c^{2}}{2} z(y+z) \Longrightarrow \frac{y}{z}=\frac{a^{2}-b^{2}+c^{2}}{a^{2}+b^{2}-c^{2}} +$$ + +In other words, + +$$ +Q=\left(0: a^{2}-b^{2}+c^{2}: a^{2}+b^{2}-c^{2}\right) +$$ + +Taking the average with $A=(1,0,0)$ then gives + +$$ +N=\left(2 a^{2}: a^{2}-b^{2}+c^{2}: a^{2}+b^{2}-c^{2}\right) . +$$ + +The equation for the perpendicular bisector of $\overline{B C}$ is given by (see EGMO 7.19) + +$$ +0=a^{2}(z-y)+x\left(c^{2}-b^{2}\right) +$$ + +which contains $N$, as needed. + +『 Extremely ill-advised complex numbers approaches (outline). Suppose we pick $a$, $b, c$ as the unit circle, and let $m=(b+c) / 2$. Using the fully general "foot" formula, one can get + +$$ +p=\frac{(a-m) \bar{c}+(\bar{a}-\bar{m}) c+\bar{a} m-a \bar{m}}{2(\bar{a}-\bar{m})}=\frac{a^{2} b-a^{2} c-a b^{2}-2 a b c-a c^{2}+b^{2} c+3 b c^{2}}{4 b c-2 a(b+c)} +$$ + +Meanwhile, an extremely ugly calculation will eventually yield + +$$ +q=\frac{\frac{b c}{a}+b+c-a}{2} +$$ + +SO + +$$ +n=\frac{a+q}{2}=\frac{a+b+c+\frac{b c}{a}}{4}=\frac{(a+b)(a+c)}{2 a} +$$ + +There are a few ways to then verify $N B=N C$. The simplest seems to be to verify that + +$$ +\frac{n-\frac{b+c}{2}}{b-c}=\frac{a-b-c+\frac{b c}{a}}{4(b-c)}=\frac{(a-b)(a-c)}{2 a(b-c)} +$$ + +is pure imaginary, which is clear. + +## §1.2 USAMO 2023/2, proposed by Carl Schildkraut + +Available online at https://aops.com/community/p27349314. + +## Problem statement + +Solve over the positive real numbers the functional equation + +$$ +f(x y+f(x))=x f(y)+2 . +$$ + +The answer is $f(x) \equiv x+1$, which is easily verified to be the only linear solution. +We show conversely that $f$ is linear. Let $P(x, y)$ be the assertion. +Claim $-f$ is weakly increasing. +Proof. Assume for contradiction $a>b$ but $f(a)\nu_{2}(a)$ ); +- Operate on any other entry besides the first one, otherwise. + +A double induction then shows that + +- Just before each of Bob's turns, $\nu_{2}(x)>\nu_{2}(a)$ always holds; and +- After each of Bob's turns, $\nu_{2}(x) \geq \nu_{2}(a)$ always holds. + +In particular Bob will never run out of legal moves, since halving $x$ is always legal. + +## §2.2 USAMO 2023/5, proposed by Ankan Bhattacharya + +Available online at https://aops.com/community/p27349487. + +## Problem statement + +Let $n \geq 3$ be an integer. We say that an arrangement of the numbers $1,2, \ldots, n^{2}$ in an $n \times n$ table is row-valid if the numbers in each row can be permuted to form an arithmetic progression, and column-valid if the numbers in each column can be permuted to form an arithmetic progression. + +For what values of $n$ is it possible to transform any row-valid arrangement into a column-valid arrangement by permuting the numbers in each row? + +Answer: $n$ prime only. +【 Proof for $n$ prime. Suppose $n=p$. In an arithmetic progression with $p$ terms, it's easy to see that either every term has a different residue modulo $p$ (if the common difference is not a multiple of $p$ ), or all of the residues coincide (when the common difference is a multiple of $p$ ). + +So, look at the multiples of $p$ in a row-valid table; there is either 1 or $p$ per row. As there are $p$ such numbers total, there are two cases: + +- If all the multiples of $p$ are in the same row, then the common difference in each row is a multiple of $p$. In fact, it must be exactly $p$ for size reasons. In other words, up to permutation the rows are just the $k(\bmod p)$ numbers in some order, and this is obviously column-valid because we can now permute such that the $k$ th column contains exactly $\{(k-1) p+1,(k-1) p+2, \ldots, k p\}$. +- If all the multiples of $p$ are in different rows, then it follows each row contains every residue modulo $p$ exactly once. So we can permute to a column-valid arrangement by ensuring the $k$ th column contains all the $k(\bmod p)$ numbers. + +【 Counterexample for $n$ composite (due to Anton Trygub). Let $p$ be any prime divisor of $n$. Construct the table as follows: + +- Row 1 contains 1 through $n$. +- Rows 2 through $p+1$ contain the numbers from $p+1$ to $n p+n$ partitioned into arithmetic progressions with common difference $p$. +- The rest of the rows contain the remaining numbers in reading order. + +For example, when $p=2$ and $n=10$, we get the following table: +$\left[\begin{array}{cccccccccc}1 & 2 & 3 & 4 & 5 & 6 & 7 & 8 & 9 & 10 \\ \mathbf{1 1} & 13 & 15 & 17 & 19 & 21 & 23 & 25 & 27 & 29 \\ 12 & 14 & 16 & 18 & 20 & 22 & 24 & 26 & 28 & 30 \\ 31 & 32 & 33 & 34 & 35 & 36 & 37 & 38 & 39 & 40 \\ 41 & 42 & 43 & 44 & 45 & 46 & 47 & 48 & 49 & 50 \\ 51 & 52 & 53 & 54 & 55 & 56 & 57 & 58 & 59 & 60 \\ 61 & 62 & 63 & 64 & 65 & 66 & 67 & 68 & 69 & 70 \\ 71 & 72 & 73 & 74 & 75 & 76 & 77 & 78 & 79 & 80 \\ 81 & 82 & 83 & 84 & 85 & 86 & 87 & 88 & 89 & 90 \\ 91 & 92 & 93 & 94 & 95 & 96 & 97 & 98 & 99 & 100\end{array}\right]$ + +We claim this works fine. Assume for contradiction the rows may be permuted to obtain a column-valid arrangement. Then the $n$ columns should be arithmetic progressions whose smallest element is in $[1, n]$ and whose largest element is in $\left[n^{2}-n+1, n^{2}\right]$. These two elements must be congruent modulo $n-1$, so in particular the column containing 2 must end with $n^{2}-n+2$. + +Hence in that column, the common difference must in fact be exactly $n$. And yet $n+2$ and $2 n+2$ are in the same row, contradiction. + +## §2.3 USAMO 2023/6, proposed by Zack Chroman + +Available online at https://aops.com/community/p27349354. + +## Problem statement + +Let $A B C$ be a triangle with incenter $I$ and excenters $I_{a}, I_{b}, I_{c}$ opposite $A, B$, and $C$, respectively. Given an arbitrary point $D$ on the circumcircle of $\triangle A B C$ that does not lie on any of the lines $I I_{a}, I_{b} I_{c}$, or $B C$, suppose the circumcircles of $\triangle D I I_{a}$ and $\triangle D I_{b} I_{c}$ intersect at two distinct points $D$ and $F$. If $E$ is the intersection of lines $D F$ and $B C$, prove that $\angle B A D=\angle E A C$. + +Here are two approaches. +![](https://cdn.mathpix.com/cropped/2024_11_19_5f0e796e43a11586ccbfg-15.jpg?height=920&width=763&top_left_y=896&top_left_x=658) +\I Barycentric coordinates (Carl Schildkraut). With reference triangle $\triangle A B C$, set $D=(r: s: t)$. + +Claim - The equations of $\left(D I I_{a}\right)$ and $\left(D I_{b} I_{c}\right)$ are, respectively, + +$$ +\begin{aligned} +& 0=-a^{2} y z-b^{2} z x-c^{2} x y+(x+y+z) \cdot\left(b c x-\frac{b c r}{c s-b t}(c y-b z)\right) \\ +& 0=-a^{2} y z-b^{2} z x-c^{2} x y+(x+y+z) \cdot\left(-b c x+\frac{b c r}{c s+b t}(c y+b z)\right) . +\end{aligned} +$$ + +Proof. Since $D \in(A B C)$, we have $a^{2} s t+b^{2} t r+c^{2} r s=0$. Now each equation can be verified by direct substitution of three points. + +By EGMO Lemma 7.24, the radical axis is then given by + +$$ +\overline{D F}: b c x-\frac{b c r}{c s-b t}(c y-b z)=-b c x+\frac{b c r}{c s+b t}(c y+b z) . +$$ + +Now the point + +$$ +\left(0: \frac{b^{2}}{s}: \frac{c^{2}}{t}\right)=\left(0: b^{2} t: c^{2} s\right) +$$ + +lies on line $D F$ by inspection, and is obviously on line $B C$, hence it coincides with $E$. This lies on the isogonal of $\overline{A D}$ (by EGMO Lemma 7.6), as needed. + +『 Synthetic approach (Anant Mudgal). Focus on just $\left(D I I_{a}\right)$. Let $P$ be the second intersection of $\left(D I I_{a}\right)$ with $(A B C)$, and let $M$ be the midpoint of minor arc $\widehat{B C}$. Then by radical axis, lines $A M, D P$, and $B C$ are concurrent at a point $K$. + +Let $E^{\prime}=\overline{P M} \cap \overline{B C}$. +![](https://cdn.mathpix.com/cropped/2024_11_19_5f0e796e43a11586ccbfg-16.jpg?height=864&width=901&top_left_y=776&top_left_x=583) + +Claim - We have $\measuredangle B A D=\measuredangle E^{\prime} A C$. + +Proof. By shooting lemma, $A K E^{\prime} P$ is cyclic, so + +$$ +\measuredangle K A E^{\prime}=\measuredangle K P E^{\prime}=\measuredangle D P M=\measuredangle D A M +$$ + +Claim - The power of point $E^{\prime}$ with respect to $\left(D I I_{a}\right)$ is $2 E^{\prime} B \cdot E^{\prime} C$. + +Proof. Construct parallelogram $I E^{\prime} I_{a} X$. Since $M I^{2}=M E^{\prime} \cdot M P$, we can get + +$$ +\measuredangle X I_{a} I=\measuredangle I_{a} I E^{\prime}=\measuredangle M I E^{\prime}=\measuredangle M P I=\measuredangle X P I +$$ + +Hence $X$ lies on $\left(D I I_{a}\right)$, and $E^{\prime} X \cdot E^{\prime} P=2 E^{\prime} M \cdot E^{\prime} P=2 E^{\prime} B \cdot E^{\prime} C$. +Repeat the argument on $\left(D I_{b} I_{c}\right)$; the same point $E^{\prime}$ (because of the first claim) then has power $2 E^{\prime} B \cdot E^{\prime} C$ with respect to $\left(D I_{b} I_{c}\right)$. Hence $E^{\prime}$ lies on the radical axis of $\left(D I I_{a}\right)$ and $\left(D I_{b} I_{c}\right)$, ergo $E^{\prime}=E$. The first claim then solves the problem. + diff --git a/USAMO/md/en-USAMO-2024-notes.md b/USAMO/md/en-USAMO-2024-notes.md new file mode 100644 index 0000000000000000000000000000000000000000..425896feedc316155f630616d3e4d89c65de3a28 --- /dev/null +++ b/USAMO/md/en-USAMO-2024-notes.md @@ -0,0 +1,574 @@ +# USAMO 2024 Solution Notes + +Evan ChEn《陳誼廷》 + +25 April 2024 + +This is a compilation of solutions for the 2024 USAMO. The ideas of the solution are a mix of my own work, the solutions provided by the competition organizers, and solutions found by the community. However, all the writing is maintained by me. + +These notes will tend to be a bit more advanced and terse than the "official" solutions from the organizers. In particular, if a theorem or technique is not known to beginners but is still considered "standard", then I often prefer to use this theory anyways, rather than try to work around or conceal it. For example, in geometry problems I typically use directed angles without further comment, rather than awkwardly work around configuration issues. Similarly, sentences like "let $\mathbb{R}$ denote the set of real numbers" are typically omitted entirely. + +Corrections and comments are welcome! + +## Contents + +0 Problems ..... 2 +1 Solutions to Day 1 ..... 3 +1.1 USAMO 2024/1, proposed by Luke Robitaille ..... 3 +1.2 USAMO 2024/2, proposed by Rishabh Das ..... 4 +1.3 USAMO 2024/3, proposed by Krit Boonsiriseth ..... 7 +2 Solutions to Day 2 ..... 9 +2.1 USAMO 2024/4, proposed by Rishabh Das ..... 9 +2.2 USAMO 2024/5, proposed by Anton Trygub ..... 11 +2.3 USAMO 2024/6, proposed by Titu Andreescu and Gabriel Dospinescu ..... 16 + +## §0 Problems + +1. Find all integers $n \geq 3$ such that the following property holds: if we list the divisors of $n$ ! in increasing order as $1=d_{1}2$ be an integer and let $\ell \in\{1,2, \ldots, n\}$. A collection $A_{1}, \ldots, A_{k}$ of (not necessarily distinct) subsets of $\{1,2, \ldots, n\}$ is called $\ell$-large if $\left|A_{i}\right| \geq \ell$ for all $1 \leq i \leq k$. Find, in terms of $n$ and $\ell$, the largest real number $c$ such that the inequality + +$$ +\sum_{i=1}^{k} \sum_{j=1}^{k} x_{i} x_{j} \frac{\left|A_{i} \cap A_{j}\right|^{2}}{\left|A_{i}\right| \cdot\left|A_{j}\right|} \geq c\left(\sum_{i=1}^{k} x_{i}\right)^{2} +$$ + +holds for all positive integer $k$, all nonnegative real numbers $x_{1}, x_{2}, \ldots, x_{k}$, and all $\ell$-large collections $A_{1}, A_{2}, \ldots, A_{k}$ of subsets of $\{1,2, \ldots, n\}$. + +## §1 Solutions to Day 1 + +## §1.1 USAMO 2024/1, proposed by Luke Robitaille + +Available online at https://aops.com/community/p30216459. + +## Problem statement + +Find all integers $n \geq 3$ such that the following property holds: if we list the divisors of $n$ ! in increasing order as $1=d_{1}24-20$. +- For $n=6$ we have $18-15>20-18$. +- For $7 \leq n \leq 12$ we have because $14-12>15-14$ (and $13 \nmid n!$ ). + +Now assume $n \geq 13$. In that case, we have + +$$ +\left\lfloor\frac{n}{2}\right\rfloor^{2}-1 \geq 2 n +$$ + +So by Bertrand postulate, we can find a prime $p$ such that + +$$ +n0$. And the objective function is to minimize the quantity + +$$ +A:=\sum_{|v| \geq 50} f(v) . +$$ + +So the problem is transformed into an system of equations over $\mathbb{Z}_{\geq 0}$ (it's clear any assignment of values of $f(v)$ can be translated to a sequence ( $S_{1}, \ldots, S_{100}$ ) in the original notation). + +Note already that: +Claim - It suffices to assign $f(v)$ for $|v| \geq 50$. +Proof. If we have found a valid assignment of values to $f(v)$ for $|v| \geq 50$, then we can always arbitrarily assign values of $f(v)$ for $|v|<50$ by downwards induction on $|v|$ to satisfy the divisibility condition (without changing $M$ ). + +Thus, for the rest of the solution, we altogether ignore $f(v)$ for $|v|<50$ and only consider $P(u)$ for $|u| \geq 50$. + +I Construction. Consider the construction + +$$ +f_{0}(v)=2|v|-100 +$$ + +This construction is valid since if $|u|=100-k$ for $k \leq 50$ then + +$$ +\begin{aligned} +\sum_{v \supseteq u} f_{0}(v) & =\binom{k}{0} \cdot 100+\binom{k}{1} \cdot 98+\binom{k}{2} \cdot 96+\cdots+\binom{k}{k} \cdot(100-2 k) \\ +& =(100-k) \cdot 2^{k}=|u| \cdot 2^{k} +\end{aligned} +$$ + +is indeed a multiple of $|u|$, hence $P(u)$ is true. In that case, the objective function is + +$$ +A=\sum_{i=50}^{100}\binom{100}{i}(2 i-100)=50\binom{100}{50} +$$ + +as needed. +Remark. This construction is the "easy" half of the problem because it coincides with what you get from a greedy algorithm by downwards induction on $|u|$ (equivalently, induction on $k=100-|u| \geq 0)$. To spell out the first three steps, + +- We know $f(1 \ldots 1)$ is a nonzero multiple of 100 , so it makes sense to guess $f(1 \ldots 1)=$ 100. +- Then we have $f(1 \ldots 10)+100 \equiv 0(\bmod 99)$, and the smallest multiple of 99 which is at least 100 is 198 . So it makes sense to guess $f(1 \ldots 10)=98$, and similarly guess $f(v)=98$ whenever $|v|=99$. +- Now when we consider, say $v=1 \ldots 100$ with $|v|=98$, we get + +$$ +f(1 \ldots 100)+\underbrace{f(1 \ldots 101)}_{=98}+\underbrace{f(1 \ldots 110)}_{=98}+\underbrace{f(1 \ldots 111)}_{=100} \equiv 0 \quad(\bmod 98) +$$ + +we obtain $f(1 \ldots 100) \equiv 96(\bmod 98)$. That makes $f(1 \ldots 100)=96$ a reasonable guess. +Continuing in this way gives the construction above. + +I Proof of bound. We are going to use a smoothing argument: if we have a general working assignment $f$, we will mold it into $f_{0}$. + +We define a push-down on $v$ as the following operation: + +- Pick any $v$ such that $|v| \geq 50$ and $f(v) \geq|v|$. +- Decrease $f(v)$ by $|v|$. +- For every $w$ such that $w \subseteq v$ and $|w|=|v|-1$, increase $f(w)$ by 1 . + +Claim - Apply a push-down preserves the main divisibility condition. Moreover, it doesn't change $A$ unless $|v|=50$, where it decreases $A$ by 50 instead. + +Proof. The statement $P(u)$ is only affected when $u \subseteq v$ : to be precise, one term on the right-hand side of $P(u)$ increases by $|v|$, while $|v|-|u|$ terms decrease by 1 , for a net change of $+|u|$. So $P(u)$ still holds. + +To see $A$ doesn't change for $|v|>50$, note $|v|$ terms increase by 1 while one term decreases by $-|v|$. When $|v|=50$, only $f(v)$ decreases by 50 . + +Now, given a valid assignment, we can modify it as follows: + +- First apply pushdowns on $1 \ldots 1$ until $f(1 \ldots 1)=100$; +- Then we may apply pushdowns on each $v$ with $|v|=99$ until $f(v)<99$; +- Then we may apply pushdowns on each $v$ with $|v|=98$ until $f(v)<98$; +- . . .and so on, until we have $f(v)<50$ for $|v|=50$. + +Hence we get $f(1 \ldots 1)=100$ and $0 \leq f(v)<|v|$ for all $50 \leq|v| \leq 100$. However, by downwards induction on $|v|=99,98, \ldots, 50$, we also have + +$$ +f(v) \equiv f_{0}(v) \quad(\bmod |v|) \Longrightarrow f(v)=f_{0}(v) +$$ + +since $f_{0}(v)$ and $f(v)$ are both strictly less than $|v|$. So in fact $f=f_{0}$, and we're done. +Remark. The fact that push-downs actually don't change $A$ shows that the equality case we described is far from unique: in fact, we could have made nearly arbitrary sub-optimal decisions during the greedy algorithm and still ended up with an equality case. For a concrete example, the construction + +$$ +f(v)= \begin{cases}500 & |v|=100 \\ 94 & |v|=99 \\ 100-2|v| & 50 \leq|v| \leq 98\end{cases} +$$ + +works fine as well (where we arbitrarily chose 500 at the start, then used the greedy algorithm thereafter). + +## §1.3 USAMO 2024/3, proposed by Krit Boonsiriseth + +Available online at https://aops.com/community/p30216513. + +## Problem statement + +Let $(m, n)$ be positive integers with $n \geq 3$ and draw a regular $n$-gon. We wish to triangulate this $n$-gon into $n-2$ triangles, each colored one of $m$ colors, so that each color has an equal sum of areas. For which $(m, n)$ is such a triangulation and coloring possible? + +The answer is if and only if $m$ is a proper divisor of $n$. +Throughout this solution, we let $\omega=\exp (2 \pi i / n)$ and let the regular $n$-gon have vertices $1, \omega, \ldots, \omega^{n-1}$. We cache the following frequent calculation: + +## Lemma + +The triangle with vertices $\omega^{k}, \omega^{k+a}, \omega^{k+b}$ has signed area + +$$ +T(a, b):=\frac{\left(\omega^{a}-1\right)\left(\omega^{b}-1\right)\left(\omega^{-a}-\omega^{-b}\right)}{2 i} . +$$ + +Proof. Rotate by $\omega^{-k}$ to assume WLOG that $k=0$. Apply complex shoelace to the triangles with vertices $1, \omega^{a}, \omega^{b}$ to get + +$$ +\frac{1}{2 i} \operatorname{det}\left[\begin{array}{ccc} +1 & 1 & 1 \\ +\omega^{a} & \omega^{-a} & 1 \\ +\omega^{b} & \omega^{-b} & 1 +\end{array}\right]=\frac{1}{2 i} \operatorname{det}\left[\begin{array}{ccc} +0 & 0 & 1 \\ +\omega^{a}-1 & \omega^{-a}-1 & 1 \\ +\omega^{b}-1 & \omega^{-b}-1 & 1 +\end{array}\right] +$$ + +which equals the above. +【 Construction. It suffices to actually just take all the diagonals from the vertex 1 , and then color the triangles with the $m$ colors in cyclic order. For example, when $n=9$ and $m=3$, a coloring with red, green, blue would be: +![](https://cdn.mathpix.com/cropped/2024_11_19_bff0e8c9a0bc19d4709dg-07.jpg?height=617&width=615&top_left_y=1873&top_left_x=726) + +To see this works one can just do the shoelace calculation: for a given residue $r \bmod m$, we get an area + +$$ +\begin{aligned} +\sum_{j \equiv r \bmod m} \operatorname{Area}\left(\omega^{j}, \omega^{0}, \omega^{j+1}\right) & =\sum_{j \equiv r \bmod m} T(-j, 1) \\ +& =\sum_{j \equiv r \bmod m} \frac{\left(\omega^{-j}-1\right)\left(\omega^{1}-1\right)\left(\omega^{j}-\omega^{-1}\right)}{2 i} \\ +& =\frac{\omega-1}{2 i} \sum_{j \equiv r \bmod m}\left(\omega^{-j}-1\right)\left(\omega^{j}-\omega^{-1}\right) \\ +& =\frac{\omega-1}{2 i}\left(\frac{n}{m}\left(1+\omega^{-1}\right)+\sum_{j \equiv r \bmod m}\left(\omega^{-j}-\omega^{j}\right)\right) . +\end{aligned} +$$ + +(We allow degenerate triangles where $j \in\{-1,0\}$ with area zero.) However, if $m$ is a proper divisor of $m$, then $\sum_{j \equiv r \bmod m} \omega^{j}=\omega^{r}\left(1+\omega^{m}+\omega^{2 m}+\cdots+\omega^{n-m}\right)=0$. Similarly, $\sum_{j \equiv r \bmod m} \omega^{-j}=0$. So the inner sum vanishes, and the total area of the $m$ th color equals + +$$ +\frac{n}{m} \frac{(\omega-1)\left(\omega^{-1}+1\right)}{2 i} +$$ + +which does not depend on the residue $r$, proving the coloring works. +【 Proof of necessity. It's obvious that $mi \\ k-1 & i \geq k>0 \\ i & k=0\end{cases} +$$ + +And one can see for each $i$, the counts are all distinct (they are ( $i, 0,1, \ldots, k-1, k+1, \ldots, k)$ from bottom to top). This completes the construction. + +Construction when $m2$ be an integer and let $\ell \in\{1,2, \ldots, n\}$. A collection $A_{1}, \ldots, A_{k}$ of (not necessarily distinct) subsets of $\{1,2, \ldots, n\}$ is called $\ell$-large if $\left|A_{i}\right| \geq \ell$ for all $1 \leq i \leq k$. Find, in terms of $n$ and $\ell$, the largest real number $c$ such that the inequality + +$$ +\sum_{i=1}^{k} \sum_{j=1}^{k} x_{i} x_{j} \frac{\left|A_{i} \cap A_{j}\right|^{2}}{\left|A_{i}\right| \cdot\left|A_{j}\right|} \geq c\left(\sum_{i=1}^{k} x_{i}\right)^{2} +$$ + +holds for all positive integer $k$, all nonnegative real numbers $x_{1}, x_{2}, \ldots, x_{k}$, and all $\ell$-large collections $A_{1}, A_{2}, \ldots, A_{k}$ of subsets of $\{1,2, \ldots, n\}$. + +The answer turns out to be + +$$ +c=\frac{n+\ell^{2}-2 \ell}{n(n-1)} +$$ + +Throughout this solution, we work with vectors in $\mathbb{R}^{n^{2}}$. The entries will be indexed by ordered pairs $(p, q) \in\{1, \ldots, n\}^{2}$; the notation $\langle\bullet, \bullet\rangle$ denotes dot product, and $\|\bullet\|$ the vector norm. + +【 Rewriting as a dot product. For $i=1, \ldots, n$ define $\mathbf{v}_{i}$ by + +$$ +\mathbf{v}_{i}[p, q]:=\left\{\begin{array}{ll} +\frac{1}{\left|A_{i}\right|} & p \in A_{i} \text { and } q \in A_{i} \\ +0 & \text { otherwise; } +\end{array} \quad \mathbf{v}:=\sum_{i} x_{i} \mathbf{v}_{i}\right. +$$ + +Then + +$$ +\begin{aligned} +\sum_{i} \sum_{j} x_{i} x_{j} \frac{\left|A_{i} \cap A_{j}\right|^{2}}{\left|A_{i}\right|\left|A_{j}\right|} & =\sum_{i} \sum_{j} x_{i} x_{j}\left\langle\mathbf{v}_{i}, \mathbf{v}_{j}\right\rangle \\ +& =\left\langle\sum_{i} x_{i} \mathbf{v}_{i}, \sum_{j} x_{i} \mathbf{v}_{i}\right\rangle=\left\|\sum_{i} x_{i} \mathbf{v}_{i}\right\|^{2}=\|\mathbf{v}\|^{2} . +\end{aligned} +$$ + +【 Proof of the inequality for the claimed value of $c$. We define two more vectors $\mathbf{e}$ and $\mathbf{1}$; the vector $\mathbf{e}$ has 1 in the $(p, q)^{\text {th }}$ component if $p=q$, and 0 otherwise, while $\mathbf{1}$ has all-ones. In that case, note that + +$$ +\begin{aligned} +& \langle\mathbf{e}, \mathbf{v}\rangle=\sum_{i} x_{i}\left\langle\mathbf{e}, \mathbf{v}_{i}\right\rangle=\sum_{i} x_{i} \\ +& \langle\mathbf{1}, \mathbf{v}\rangle=\sum_{i} x_{i}\left\langle\mathbf{1}, \mathbf{v}_{i}\right\rangle=\sum_{i} x_{i}\left|A_{i}\right| . +\end{aligned} +$$ + +That means for any positive real constants $\alpha$ and $\beta$, by Cauchy-Schwarz for vectors, we should have + +$$ +\begin{aligned} +\|\alpha \mathbf{e}+\beta \mathbf{1}\|\|\mathbf{v}\| & \geq\langle\alpha \mathbf{e}+\beta \mathbf{1}, \mathbf{v}\rangle=\alpha\langle\mathbf{e}, \mathbf{v}\rangle+\beta\langle\mathbf{1}, \mathbf{v}\rangle \\ +& =\alpha \cdot \sum x_{i}+\beta \cdot \sum x_{i}\left|A_{i}\right| \\ +& \geq(\alpha+\ell \beta) \sum x_{i} . +\end{aligned} +$$ + +Set $\mathbf{w}:=\alpha \mathbf{e}+\beta \mathbf{1}$ for brevity. Then + +$$ +\mathbf{w}[p, q]= \begin{cases}\alpha+\beta & \text { if } p=q \\ \beta & \text { if } p \neq q\end{cases} +$$ + +SO + +$$ +\|\mathbf{w}\|=\sqrt{n \cdot(\alpha+\beta)^{2}+\left(n^{2}-n\right) \cdot \beta^{2}} +$$ + +Therefore, we get an lower bound + +$$ +\frac{\|\mathbf{v}\|}{\sum x_{i}} \geq \frac{\alpha+\ell \beta}{\sqrt{n \cdot(\alpha+\beta)^{2}+\left(n^{2}-n\right) \cdot \beta^{2}}} +$$ + +Letting $\alpha=n-\ell$ and $\beta=\ell-1$ gives a proof that the constant + +$$ +c=\frac{((n-\ell)+\ell(\ell-1))^{2}}{n \cdot(n-1)^{2}+\left(n^{2}-n\right) \cdot(\ell-1)^{2}}=\frac{\left(n+\ell^{2}-2 \ell\right)^{2}}{n(n-1)\left(n+\ell^{2}-2 \ell\right)}=\frac{n+\ell^{2}-2 \ell}{n(n-1)} +$$ + +makes the original inequality always true. (The choice of $\alpha: \beta$ is suggested by the example below.) + +【 Example showing this $c$ is best possible. Let $k=\binom{n}{\ell}$, let $A_{i}$ run over all $\binom{n}{\ell}$ subsets of $\{1, \ldots, n\}$ of size $\ell$, and let $x_{i}=1$ for all $i$. We claim this construction works. + +To verify this, it would be sufficient to show that $\mathbf{w}$ and $\mathbf{v}$ are scalar multiples, so that the above Cauchy-Schwarz is equality. However, we can compute + +$$ +\mathbf{w}[p, q]=\left\{\begin{array}{ll} +n-1 & \text { if } p=q \\ +\ell-1 & \text { if } p \neq q +\end{array}, \quad \mathbf{v}[p, q]= \begin{cases}\binom{n-1}{\ell-1} \cdot \frac{1}{\ell} & \text { if } p=q \\ +\binom{n-2}{\ell-2} \cdot \frac{1}{\ell} & \text { if } p \neq q\end{cases}\right. +$$ + +which are indeed scalar multiples, finishing the proof. + diff --git a/USAMO/raw/en-USAMO-1996-notes.pdf b/USAMO/raw/en-USAMO-1996-notes.pdf new file mode 100644 index 0000000000000000000000000000000000000000..8a7f82e0368957187eb3541af15d3f9f216b10a7 --- /dev/null +++ b/USAMO/raw/en-USAMO-1996-notes.pdf @@ -0,0 +1,3 @@ +version https://git-lfs.github.com/spec/v1 +oid sha256:ee256630d10a5ce7fc397141449cf74d6a229a4ce35a094a708bf2e8da6fa6c5 +size 157776 diff --git a/USAMO/raw/en-USAMO-1997-notes.pdf b/USAMO/raw/en-USAMO-1997-notes.pdf new file mode 100644 index 0000000000000000000000000000000000000000..7e566b48cb311044fd7f5d12974a25dcb436d48f --- /dev/null +++ b/USAMO/raw/en-USAMO-1997-notes.pdf @@ -0,0 +1,3 @@ +version https://git-lfs.github.com/spec/v1 +oid sha256:11852224f7929e56a6b3e7877e33c22c94edb9bb3c189b653b26a10b89611cb2 +size 168377 diff --git a/USAMO/raw/en-USAMO-1998-notes.pdf b/USAMO/raw/en-USAMO-1998-notes.pdf new file mode 100644 index 0000000000000000000000000000000000000000..cb76b74f3bb59bdc8d08167bb376b91824037724 --- /dev/null +++ b/USAMO/raw/en-USAMO-1998-notes.pdf @@ -0,0 +1,3 @@ +version https://git-lfs.github.com/spec/v1 +oid sha256:2912ee1778e5c1b46da6294954f04354c09b1df0ffc3fb1cf822fcd7a01a3715 +size 198163 diff --git a/USAMO/raw/en-USAMO-1999-notes.pdf b/USAMO/raw/en-USAMO-1999-notes.pdf new file mode 100644 index 0000000000000000000000000000000000000000..4ccefe164877df264e7a074e4a0c79438d75cac3 --- /dev/null +++ b/USAMO/raw/en-USAMO-1999-notes.pdf @@ -0,0 +1,3 @@ +version https://git-lfs.github.com/spec/v1 +oid sha256:e2f8a3a5bf494a2305686f95004a7ccff8567c252ec6d130984f81be5fe76c72 +size 211560 diff --git a/USAMO/raw/en-USAMO-2000-notes.pdf b/USAMO/raw/en-USAMO-2000-notes.pdf new file mode 100644 index 0000000000000000000000000000000000000000..220cefec8334b0faab0fb76c786801c1954dc7f9 --- /dev/null +++ b/USAMO/raw/en-USAMO-2000-notes.pdf @@ -0,0 +1,3 @@ +version https://git-lfs.github.com/spec/v1 +oid sha256:f2a9985bc02e1742e42af52c33a9ca26982709fe631e03b3ba28e2c4d8b47b40 +size 200447 diff --git a/USAMO/raw/en-USAMO-2001-notes.pdf b/USAMO/raw/en-USAMO-2001-notes.pdf new file mode 100644 index 0000000000000000000000000000000000000000..df6b36d70585c2705fcfb768087c3ac67e7ca4ad --- /dev/null +++ b/USAMO/raw/en-USAMO-2001-notes.pdf @@ -0,0 +1,3 @@ +version https://git-lfs.github.com/spec/v1 +oid sha256:c18552181c4eb427a2adf97c187c28430c4bb58e68a86b2793004c3bca64ed64 +size 170411 diff --git a/USAMO/raw/en-USAMO-2002-notes.pdf b/USAMO/raw/en-USAMO-2002-notes.pdf new file mode 100644 index 0000000000000000000000000000000000000000..e1620f619894b767b11184ed812c9d3c6bcfe602 --- /dev/null +++ b/USAMO/raw/en-USAMO-2002-notes.pdf @@ -0,0 +1,3 @@ +version https://git-lfs.github.com/spec/v1 +oid sha256:fc02443a455e0cfc3f88fdc8fac812db57fc43edadf6f5110d049c7f42a5e230 +size 144598 diff --git a/USAMO/raw/en-USAMO-2003-notes.pdf b/USAMO/raw/en-USAMO-2003-notes.pdf new file mode 100644 index 0000000000000000000000000000000000000000..dc5706511df540832fdd9aafe0857743573a5a9a --- /dev/null +++ b/USAMO/raw/en-USAMO-2003-notes.pdf @@ -0,0 +1,3 @@ +version https://git-lfs.github.com/spec/v1 +oid sha256:15603d0fcbc77cc5caa14832a986b163e3f3babbbe8c879ed4617c5fe1d9470e +size 254025 diff --git a/USAMO/raw/en-USAMO-2004-notes.pdf b/USAMO/raw/en-USAMO-2004-notes.pdf new file mode 100644 index 0000000000000000000000000000000000000000..e6af67e384654429778ce00be9f6ca5443df70d2 --- /dev/null +++ b/USAMO/raw/en-USAMO-2004-notes.pdf @@ -0,0 +1,3 @@ +version https://git-lfs.github.com/spec/v1 +oid sha256:25bc90d15305709b446ae007e9bd64d12fcfe60516d3622919f9ee0c307e37fa +size 212861 diff --git a/USAMO/raw/en-USAMO-2005-notes.pdf b/USAMO/raw/en-USAMO-2005-notes.pdf new file mode 100644 index 0000000000000000000000000000000000000000..5b76a254fcb7f98714f2aa94ed6a0d7046048ef5 --- /dev/null +++ b/USAMO/raw/en-USAMO-2005-notes.pdf @@ -0,0 +1,3 @@ +version https://git-lfs.github.com/spec/v1 +oid sha256:6b9e8a8c2737513881df728118f4ee68cdbb01dabe3fe0926cb96aa5de47df69 +size 226089 diff --git a/USAMO/raw/en-USAMO-2006-notes.pdf b/USAMO/raw/en-USAMO-2006-notes.pdf new file mode 100644 index 0000000000000000000000000000000000000000..c88db097a1d1df89e8b861387625c017c3bbe020 --- /dev/null +++ b/USAMO/raw/en-USAMO-2006-notes.pdf @@ -0,0 +1,3 @@ +version https://git-lfs.github.com/spec/v1 +oid sha256:81a1bc35cd77870bde5bfd0fcd26bcf00b48432c75d3d2c08867eef370d13f24 +size 229216 diff --git a/USAMO/raw/en-USAMO-2007-notes.pdf b/USAMO/raw/en-USAMO-2007-notes.pdf new file mode 100644 index 0000000000000000000000000000000000000000..5b1fbe47534fef46861c91dd7796c6da5265d5c5 --- /dev/null +++ b/USAMO/raw/en-USAMO-2007-notes.pdf @@ -0,0 +1,3 @@ +version https://git-lfs.github.com/spec/v1 +oid sha256:45d37028e2cd8ef54c94d57dac7d359f59d6717870d3ee1866f0e97d05f0b28d +size 206418 diff --git a/USAMO/raw/en-USAMO-2008-notes.pdf b/USAMO/raw/en-USAMO-2008-notes.pdf new file mode 100644 index 0000000000000000000000000000000000000000..f27e9e2af33416971df82a363e27d745eed3f1bb --- /dev/null +++ b/USAMO/raw/en-USAMO-2008-notes.pdf @@ -0,0 +1,3 @@ +version https://git-lfs.github.com/spec/v1 +oid sha256:a60339b6676f67dab30ec0c633a6a32b289515ab24c1d2b19379af101d500ab1 +size 223501 diff --git a/USAMO/raw/en-USAMO-2009-notes.pdf b/USAMO/raw/en-USAMO-2009-notes.pdf new file mode 100644 index 0000000000000000000000000000000000000000..5066c7b6ef23bdf94c5437c7e467d0dd019ef140 --- /dev/null +++ b/USAMO/raw/en-USAMO-2009-notes.pdf @@ -0,0 +1,3 @@ +version https://git-lfs.github.com/spec/v1 +oid sha256:966ff3224c31bb520ff85d496d9864ff2cc0c113d60afac04a7dd926cbca98f8 +size 261568 diff --git a/USAMO/raw/en-USAMO-2010-notes.pdf b/USAMO/raw/en-USAMO-2010-notes.pdf new file mode 100644 index 0000000000000000000000000000000000000000..b4fc1e79a00058f16437416d3f15256583f31f3e --- /dev/null +++ b/USAMO/raw/en-USAMO-2010-notes.pdf @@ -0,0 +1,3 @@ +version https://git-lfs.github.com/spec/v1 +oid sha256:8c3be3bb4ec563f8592835786e7046b3b4b0908908ed35237aa0b6e477f88c34 +size 255287 diff --git a/USAMO/raw/en-USAMO-2011-notes.pdf b/USAMO/raw/en-USAMO-2011-notes.pdf new file mode 100644 index 0000000000000000000000000000000000000000..0de65349bedbd5881fc5d016e0e8927e6fb0cded --- /dev/null +++ b/USAMO/raw/en-USAMO-2011-notes.pdf @@ -0,0 +1,3 @@ +version https://git-lfs.github.com/spec/v1 +oid sha256:a8f617b915fce90da74af50d16289c1ff447c22d9c7cd9a159e7530d5c2dcdd5 +size 229119 diff --git a/USAMO/raw/en-USAMO-2012-notes.pdf b/USAMO/raw/en-USAMO-2012-notes.pdf new file mode 100644 index 0000000000000000000000000000000000000000..5f285a43d0437dce6f0fa68d424dd183f83182e4 --- /dev/null +++ b/USAMO/raw/en-USAMO-2012-notes.pdf @@ -0,0 +1,3 @@ +version https://git-lfs.github.com/spec/v1 +oid sha256:231bd97940c09b27007c0b918034e1f1c672cd2cf4a03fc7e0e7208fa33965ac +size 210521 diff --git a/USAMO/raw/en-USAMO-2013-notes.pdf b/USAMO/raw/en-USAMO-2013-notes.pdf new file mode 100644 index 0000000000000000000000000000000000000000..46aa8d0c6caa45ee1bb2bd9afc111b626572b8b5 --- /dev/null +++ b/USAMO/raw/en-USAMO-2013-notes.pdf @@ -0,0 +1,3 @@ +version https://git-lfs.github.com/spec/v1 +oid sha256:32d6f791efc2f2756bc82755c7377e876b0db5cf8204400df90a161031da97a0 +size 284498 diff --git a/USAMO/raw/en-USAMO-2014-notes.pdf b/USAMO/raw/en-USAMO-2014-notes.pdf new file mode 100644 index 0000000000000000000000000000000000000000..3080b490e1e24fc9ad19a7b51edfba3c3beab20e --- /dev/null +++ b/USAMO/raw/en-USAMO-2014-notes.pdf @@ -0,0 +1,3 @@ +version https://git-lfs.github.com/spec/v1 +oid sha256:06860b10a30fd7432590eb2001bba6e050adf9393f40440595d3b29dd6d79cf0 +size 210605 diff --git a/USAMO/raw/en-USAMO-2015-notes.pdf b/USAMO/raw/en-USAMO-2015-notes.pdf new file mode 100644 index 0000000000000000000000000000000000000000..b16cdcd25ca84b8c287387dce5abcede08fcfd6d --- /dev/null +++ b/USAMO/raw/en-USAMO-2015-notes.pdf @@ -0,0 +1,3 @@ +version https://git-lfs.github.com/spec/v1 +oid sha256:e4378dedc7967df4d3555201d3c561c45ebd8e82501dcb3b0a4b7616645fc585 +size 282585 diff --git a/USAMO/raw/en-USAMO-2016-notes.pdf b/USAMO/raw/en-USAMO-2016-notes.pdf new file mode 100644 index 0000000000000000000000000000000000000000..31444a9c8c024c7ccfb5e47eb35b46eab6341a8c --- /dev/null +++ b/USAMO/raw/en-USAMO-2016-notes.pdf @@ -0,0 +1,3 @@ +version https://git-lfs.github.com/spec/v1 +oid sha256:40893afdae1c2995f39d411445cef40a0b7824ba41258e125344ab27ae8b40e1 +size 269616 diff --git a/USAMO/raw/en-USAMO-2017-notes.pdf b/USAMO/raw/en-USAMO-2017-notes.pdf new file mode 100644 index 0000000000000000000000000000000000000000..1f0be574e864a847385b0525627ead14e6e4f846 --- /dev/null +++ b/USAMO/raw/en-USAMO-2017-notes.pdf @@ -0,0 +1,3 @@ +version https://git-lfs.github.com/spec/v1 +oid sha256:b3e27a6f4019dda1874903587f174a42bb324a6d2fe79e1e89d4f822ae2ac08b +size 358717 diff --git a/USAMO/raw/en-USAMO-2018-notes.pdf b/USAMO/raw/en-USAMO-2018-notes.pdf new file mode 100644 index 0000000000000000000000000000000000000000..d474b33ce2745a36a6907603e2c3d428b891552d --- /dev/null +++ b/USAMO/raw/en-USAMO-2018-notes.pdf @@ -0,0 +1,3 @@ +version https://git-lfs.github.com/spec/v1 +oid sha256:5ee8c5e47e670922c3252411a8a73dbbe086a28ae87f3268ff2f4c7a451c456d +size 255174 diff --git a/USAMO/raw/en-USAMO-2019-notes.pdf b/USAMO/raw/en-USAMO-2019-notes.pdf new file mode 100644 index 0000000000000000000000000000000000000000..dacc7f924f61e6744bb51f619adf63b51e98a0c1 --- /dev/null +++ b/USAMO/raw/en-USAMO-2019-notes.pdf @@ -0,0 +1,3 @@ +version https://git-lfs.github.com/spec/v1 +oid sha256:4e7a9f6dbbdbda296a8da1085fa1f0f810c189a93a8e894b8b6f531d6d724c89 +size 286844 diff --git a/USAMO/raw/en-USAMO-2020-notes.pdf b/USAMO/raw/en-USAMO-2020-notes.pdf new file mode 100644 index 0000000000000000000000000000000000000000..0d5c17122f9af9e6579fe46c9be0c63b8ede9eed --- /dev/null +++ b/USAMO/raw/en-USAMO-2020-notes.pdf @@ -0,0 +1,3 @@ +version https://git-lfs.github.com/spec/v1 +oid sha256:096647928a7a3d1dc57c53daafa3928e470a3f1fd553eea0974a11169a3780d8 +size 310956 diff --git a/USAMO/raw/en-USAMO-2021-notes.pdf b/USAMO/raw/en-USAMO-2021-notes.pdf new file mode 100644 index 0000000000000000000000000000000000000000..8196ba2e90e61b158a5c827a13f2bbb67484a56f --- /dev/null +++ b/USAMO/raw/en-USAMO-2021-notes.pdf @@ -0,0 +1,3 @@ +version https://git-lfs.github.com/spec/v1 +oid sha256:122c189c1371cd5e5fb57775ba02463f6913ff077a9eb7cbf093f943eafb1201 +size 335295 diff --git a/USAMO/raw/en-USAMO-2022-notes.pdf b/USAMO/raw/en-USAMO-2022-notes.pdf new file mode 100644 index 0000000000000000000000000000000000000000..8fe5562c1787d4ae2473f5c6c275c7b0bea79cd6 --- /dev/null +++ b/USAMO/raw/en-USAMO-2022-notes.pdf @@ -0,0 +1,3 @@ +version https://git-lfs.github.com/spec/v1 +oid sha256:0dd2f5b55045d847dc0af1344583e822471a200c6703a81ae2be2a97428d8fb1 +size 292467 diff --git a/USAMO/raw/en-USAMO-2023-notes.pdf b/USAMO/raw/en-USAMO-2023-notes.pdf new file mode 100644 index 0000000000000000000000000000000000000000..50a7bbb2c145bd8dfae762ab82f8004952b2809f --- /dev/null +++ b/USAMO/raw/en-USAMO-2023-notes.pdf @@ -0,0 +1,3 @@ +version https://git-lfs.github.com/spec/v1 +oid sha256:77bedc262babdaf53b90f73e6db5e0e680e5060eda8c0cb6f9342e242e897ae0 +size 275480 diff --git a/USAMO/raw/en-USAMO-2024-notes.pdf b/USAMO/raw/en-USAMO-2024-notes.pdf new file mode 100644 index 0000000000000000000000000000000000000000..556ea4016d336b3081baca6497773bd4378ac3b4 --- /dev/null +++ b/USAMO/raw/en-USAMO-2024-notes.pdf @@ -0,0 +1,3 @@ +version https://git-lfs.github.com/spec/v1 +oid sha256:b046df6fb8b8760b86eaf5b26bc95e2de597acfa8d8bca175ff9ee3c72c79691 +size 354907 diff --git a/USAMO/segment_script/segment_usamo.py b/USAMO/segment_script/segment_usamo.py new file mode 100644 index 0000000000000000000000000000000000000000..ab2876749f34489c2e8b5b97d82ea93eb808b990 --- /dev/null +++ b/USAMO/segment_script/segment_usamo.py @@ -0,0 +1,306 @@ +# ----------------------------------------------------------------------------- +# Author: Marina +# Date: 2024-11-15 +# ----------------------------------------------------------------------------- +''' Script to segment md files in en-usamo, en-tstst, en-tst, en-jmo folder using regex. +To run: +`python segment_usamo.py` +''' + +import warnings +warnings.filterwarnings("ignore", category=DeprecationWarning) + +import os +import re +import pandas as pd +from rapidfuzz import fuzz + + +# "## §0 Problems" -> match +section_re = re.compile(r"^#{1,2}\s(?:Contents|Problem|§[\d.]+.*)") +# section_re = re.compile(r"^#{1,2}\s(?:§[\d.]+.*)") + +# "## §1.3 USAMO 2024/3, proposed by Krit Boonsiriseth" ->3 +# "## §2.3 TSTST 2011/6" ->6 +# "## §1.1 TSTST 2021/1, proposed by Holden Mui" -> 1 +# "## §1.2 USA TST 2020/2, proposed by Merlijn Staps" -> 2 +# "# §1.3 USA TST 2019/3, proposed by Nikolai Beluhov" -> 3 +solution_label_re = re.compile( + r"^#{1,2}\s§[\d.]+\s[A-Za-z0-9 ]+\s\d{4}/(\d+)(?:,\s.*)?$" +) + +# "1. Prove that the average of the numbers $n" -> match +# "2 . For any nonempty set $S$ of real numbers,"" -> match +problem_re = re.compile(r"^(\d+)\s?\.\s(.*(?:\n\s+.*)*)") + +# "## Problem statement extra text" -> match +# "Problem statement" -> match +# "##Problem statement (missing space)" -> match +solution_re = re.compile(r"^#{0,2}\s?Problem statement\b.*$") + +# not actually used, only for debugging +pattern_debug = re.compile( + r"^[【『\\]*.*?\b(First|Second|Third|Fourth|Fifth|Sixth|Seventh|Eighth|Ninth|Tenth|Complex|Inversion|Synthetic|One|Another|Solution)\b.*\b(solution|approach|proof)\b.*", + re.IGNORECASE +) + +# "Solution 1" -> match +# "【 Solution 1" -> match +solution_split_re1 = re.compile(r"\bSolution\s[1-9]\b") + +# "First solution" -> match +# "【 Third approach" -> match +# "【 Second approach" -> match +solution_split_re2 = re.compile(r"\b(First|Second|Third|Fourth|Fifth|Sixth|Seventh|Eighth|Ninth|Synthetic)\b\s+(solution|approach|proof)\b") + +# catch special cases (ugly but it works). To generate them run script with `DEBUG = True`` +# and identify in console which special_cases should be catched as first line of a solution +DEBUG = False +special_cases = [ +"【 First short solution, by Jeffrey Kwan. Let $p_{0", +"II Second longer solution using an invariant. Visu", +"【 Complex solution (Evan Chen). Toss on the comple", +"Second (longer) solution. If one does not notice t", +"『 Second calculation approach (along the lines of ", +"T Outline of second approach (by convexity, due t", +"I Inversion solution submitted by Ankan Bhattacha", +"【 Complex numbers approach with Apollonian circles", +" A second solution. Both lemmas above admit varia", +"【 A third remixed solution. We use Lemma I and Lem", +"【I A fourth remixed solution. We also can combine ", +"I First grid-based solution. The following solutio", +"Another short solution. Let $Z$ be on line $B D E$", +"【 Most common synthetic approach. The solution hin", +"\\ First \"local\" solution by swapping two points. L", +"Second general solution by angle chasing. By Rei", +"Third general solution by Pascal. Extend rays $A", +"【 Second length solution by tangent lengths. By $t", +"【 Angle chasing solution. Note that $(B D A)$ and", +"【 Harmonic solution (mine). Let $T$ be the point o", +"【 Pascal solution (Zuming Feng). Extend ray $F D$", +"『 A spiral similarity approach (Hans $\mathbf{Y u}", +"ब The author's original solution. Complete isoscel", +"l Evan's permutation-based solution. Retain the n", +"I Original proposer's solution. To this end, let's", +"【 Cartesian coordinates approach with power of a p", +"【 Cartesian coordinates approach without power of", +"I III-advised barycentric approach (outline). Use", +"【 Approach using difference of squares (from autho", +"【 Divisibility approach (Aharshi Roy). Since $p q-", +"Solution with Danielle Wang: the answer is that $|", +"【 Homothety solution (Alex Whatley). Let $G, N, O$", +"【 Power of a point solution (Zuming Feng, official", +"【 Solution by Luke Robitaille. Let $Q$ be the seco", +"ๆ Solution with coaxial circles (Pitchayut Saengru", +"【 Solution to generalization (Nikolai Beluhov). We", +"【 Approach by deleting teams (Gopal Goel). Initial", +"【 Approach by adding colors. For a constructive al", +"【 Solution using spiral similarity. We will ignore", +"『 Barycentric solution (by Carl, Krit, Milan). We", +"I A Menelaus-based approach (Kevin Ren). Let $P$ b", +"【 Barycentric solution. First, we find the coordin", +"【 Angle chasing solution (Mason Fang). Obviously $", +"【 Inversive solution (Kelin Zhu). Invert about $A$", +"l The one-liner. ", +" The external power solution. We distinguish betw", +"Cauchy-Schwarz approach. Apply Titu lemma to get", +"đ Cauchy-Schwarz approach. The main magical claim ", +"『 Alternate solution (by proposer). Let $L$ be dia" +] + + +def add_content(current_dict): + if not current_dict["lines"] or not current_dict["label"] : + return + text_str = " ".join(current_dict["lines"]).strip() + entry = {"label": current_dict["label"]} + if current_dict["class"] == "problem": + entry["problem"] = text_str + current_dict["problems"].append(entry) + elif current_dict["class"] == "solution": + entry["solution"] = text_str + entry["solution_lines"] = current_dict["lines"] + current_dict["solutions"].append(entry) + + +def parse(file): + with open(file, 'r') as file: + content = file.read() + current = { + "label": None, + "class": None, + "lines": [], + "problems": [], + "solutions": [] + } + for line in content.splitlines(): + if match := section_re.match(line): # match a section + add_content(current) + if "problems" in line.lower(): # match problem section + current["class"] = "problem" + elif sub_match:= solution_label_re.match(line): # match solutions section, extract label for join + current["class"] = "other" + current["label"] = sub_match.group(1) + elif match := solution_re.match(line): # match solutions subsection + current["class"] = "solution" + else: + current["class"] = "other" + current["lines"] = [] + elif match := problem_re.match(line): # match a problem + if current["class"] == "solution": # handle wrong problem match + current["lines"].append(line) + else: + add_content(current) + label, text = match.groups() + current["label"] = label + current["lines"] = [text] + else: + if current["class"]=="solution" or current["class"]=="problem": + current["lines"].append(line) + add_content(current) + problems_df = pd.DataFrame(current["problems"]) + solutions_df = pd.DataFrame(current["solutions"]) + return problems_df, solutions_df + + +def parse_solution(lines): + """parses lines of a solution, finds multiple solutions and splits them""" + solutions = [] + current = [] + for line in lines: + if match := solution_split_re1.search(line): # match a solution (case1) + solutions.append(" ".join(current).strip()) + current = [line] + elif match := solution_split_re2.search(line): # match a solution (case1) + solutions.append(" ".join(current).strip()) + current = [line] + elif any(case.lower() in line.lower() for case in special_cases): # match a solution (handle special_case) + solutions.append(" ".join(current).strip()) + current = [line] + elif any(case.lower() in line[:50].lower() for case in ["solution", "approach", "proof"]): + if DEBUG: + if not any(case.lower() in line[:50].lower() for case in ["remark", "proof.", "proof", "approaches", "solutions"]): + print(line[:50]) + else: + current.append(line) + solutions.append(" ".join(current).strip()) + return solutions + +def find_mult_solutions(solutions_df): + """apply parse_solution to all df""" + solutions_df["solution"] = solutions_df["solution_lines"].apply(lambda v: parse_solution(v)) + solutions_df = solutions_df.drop(columns=["solution_lines"]) + solutions_df = solutions_df.explode('solution', ignore_index=True) + return solutions_df + + +def join(problems_df, solutions_df): + pairs_df = problems_df.merge(solutions_df, on=["label"], how="outer") + return pairs_df + + +def clean(pairs_df): + '''removes the problem statement from the solution in an approximate way''' + def find_closest_char(s, i, char): + left = s.rfind(char, 0, i) # Find the last '.' before index i + right = s.find(char, i) # Find the first '.' after or at index i + if left == -1 and right == -1: + return None # No '.' found + elif left == -1: # No '.' on the left + return right + elif right == -1: # No '.' on the right + return left + else: # Closest '.' on either side + return left if abs(i - left) <= abs(i - right) else right + def remove_approx_match(row, threshold=90): + problem = row["problem"] + solution = row["solution"] + similarity = fuzz.partial_ratio(problem, solution) + if similarity >= threshold: + i = find_closest_char(solution, len(problem), problem[-1]) + if i is not None: + solution = solution[i+1:] + return solution + pairs_df["solution"] = pairs_df.apply(remove_approx_match, axis=1) + return pairs_df + + +def process_mult_solutions(pairs_df): + '''in case of multiple solutions, prepend common text to all solutions''' + def prepend_to_solution(group): + if len(group) == 1: + return group + first_row = group.iloc[0] + comment = f"{first_row['solution']}" + group = group.iloc[1:].copy() + group["solution"] = group["solution"].apply(lambda x: f"{comment} {x}") + return group + pairs_df = pairs_df.groupby("label", group_keys=False).apply(prepend_to_solution).reset_index(drop=True) + return pairs_df + + +def add_metadata(pairs_df, yar, tier): + pairs_df['year'] = year + pairs_df['tier'] = tier # according to omnimath + return pairs_df[['year', 'label', 'problem', 'solution']] + + +def write_pairs(filename, pairs_df): + pairs_df.to_json(filename, orient="records", lines=True) + + +configs = [ + ('en-usamo', lambda year: f'en-USAMO-{year}-notes', range(1996, 2025), 1), + ('en-tstst', lambda year: f'en-sols-TSTST-{year}', range(2011, 2024), 0), + ('en-tst', lambda year: f'en-sols-TST-IMO-{year}', range(2014, 2024), 0), + # ('en-elmo', lambda year: f'en-ELMO-{year}-sols', range(2010, 2017), None), #TODO: needs another parser + # ('en-elmo', lambda year: f'en-ELMO-{year}-SL', range(2017, 2019), None), #TODO: needs another parser + ('en-jmo', lambda year: f'en-JMO-{year}-notes', range(2010, 2025), 3), + # ('en-usemo', lambda year: f'en-report-usemo-{year}', range(2019, 2023), None), #TODO: needs another parser +] + + +total_problem_count = 0 +total_solution_count = 0 +for base, basename_, years, tier in configs: + print(base) + problem_count = 0 + solution_count = 0 + seg_base = f"{base}-seg" + os.makedirs(seg_base, exist_ok=True) + for year in years: + basename = basename_(year) + file_path = f"{base}/{basename}.md" + if os.path.exists(file_path): + # print(basename) + problems, solutions = parse(file_path) + solutions = find_mult_solutions(solutions) + pairs_df = join(problems, solutions) + pairs_df = clean(pairs_df) + pairs_df = process_mult_solutions(pairs_df) + pairs_df = add_metadata(pairs_df, year, tier) + problem_count += len(problems) + solution_count += len(pairs_df) + # print(pairs_df) + write_pairs(f"{seg_base}/{basename}.jsonl", pairs_df) + print(f"problem count: {problem_count}") + print(f"solution count: {solution_count}") + total_problem_count += problem_count + total_solution_count += solution_count +print(f"total problem count: {total_problem_count}") +print(f"total solution count: {total_solution_count}") + +# en-usamo +# problem count: 174 +# solution count: 203 +# en-tstst +# problem count: 105 +# solution count: 168 +# en-tst +# problem count: 51 +# solution count: 82 +# en-jmo +# problem count: 90 +# solution count: 127 +# total problem count: 420 +# total solution count: 580 \ No newline at end of file diff --git a/USAMO/segmented/en-USAMO-1996-notes.jsonl b/USAMO/segmented/en-USAMO-1996-notes.jsonl new file mode 100644 index 0000000000000000000000000000000000000000..f509ed5984c63cfde7b9ae3b914a5b691a599453 --- /dev/null +++ b/USAMO/segmented/en-USAMO-1996-notes.jsonl @@ -0,0 +1,6 @@ +{"year":1996,"label":"1","problem":"Prove that the average of the numbers $n \\sin n^{\\circ}$ for $n=2,4,6, \\ldots, 180$ is $\\cot 1^{\\circ}$.","solution":" Because $$ n \\sin n^{\\circ}+(180-n) \\sin \\left(180^{\\circ}-n^{\\circ}\\right)=180 \\sin n^{\\circ} $$ So enough to show that $$ \\sum_{n=0}^{89} \\sin (2 n)^{\\circ}=\\cot 1^{\\circ} $$ Let $\\zeta=\\cos 2^{\\circ}+i \\sin 2^{\\circ}$ be a primitive root. Then $$ \\begin{aligned} \\sum_{n=0}^{89} \\frac{\\zeta^{n}-\\zeta^{-n}}{2 i} & =\\frac{1}{2 i}\\left[\\frac{\\zeta^{90}-1}{\\zeta-1}-\\frac{\\zeta^{-90}-1}{\\zeta^{-1}-1}\\right] \\\\ & =\\frac{1}{2 i}\\left[\\frac{-2}{\\zeta-1}-\\frac{-2}{\\zeta^{-1}-1}\\right] \\\\ & =\\frac{1}{-i} \\frac{\\zeta^{-1}-\\zeta}{(\\zeta-1)\\left(\\zeta^{-1}-1\\right)}=i \\cdot \\frac{\\zeta+1}{\\zeta-1} \\end{aligned} $$ Also, $$ \\begin{aligned} \\cot 1^{\\circ} & =\\frac{\\cos 1^{\\circ}}{\\sin 1^{\\circ}}=\\frac{\\left(\\cos 1^{\\circ}\\right)^{2}}{\\cos 1^{\\circ} \\sin 1^{\\circ}} \\\\ & =\\frac{\\frac{\\cos 2^{\\circ}+1}{2}}{\\frac{\\sin 2^{\\circ}}{2}}=\\frac{\\frac{1}{2}\\left(\\zeta+\\zeta^{-1}\\right)+1}{\\frac{1}{2 i}\\left(\\zeta-\\zeta^{-1}\\right)} \\\\ & =i \\cdot \\frac{(\\zeta+1)^{2}}{\\zeta^{2}-1}=i \\cdot \\frac{\\zeta+1}{\\zeta-1} \\end{aligned} $$ So we're done."} +{"year":1996,"label":"2","problem":"For any nonempty set $S$ of real numbers, let $\\sigma(S)$ denote the sum of the elements of $S$. Given a set $A$ of $n$ positive integers, consider the collection of all distinct sums $\\sigma(S)$ as $S$ ranges over the nonempty subsets of $A$. Prove that this collection of sums can be partitioned into $n$ classes so that in each class, the ratio of the largest sum to the smallest sum does not exceed 2 .","solution":" By induction on $n$ with $n=1$ being easy. For the inductive step, assume $$ A=\\left\\{a_{1}>a_{2}>\\cdots>a_{n}\\right\\} $$ Fix any index $k$ with the property that $$ a_{k}>\\frac{\\sigma(A)}{2^{k}} $$ (which must exist since $\\frac{1}{2}+\\frac{1}{4}+\\cdots+\\frac{1}{2^{k}}<1$ ). Then - We make $k$ classes for the sums between $\\frac{\\sigma(A)}{2^{k}}$ and $\\sigma(A)$; this handles every set which has any element in $\\left\\{a_{1}, \\ldots, a_{k}\\right\\}$. - We make $n-k$ classes via induction hypothesis on $\\left\\{a_{k+1}, \\ldots, a_{n}\\right\\}$. This solves the problem."} +{"year":1996,"label":"3","problem":"Let $A B C$ be a triangle. Prove that there is a line $\\ell$ (in the plane of triangle $A B C$ ) such that the intersection of the interior of triangle $A B C$ and the interior of its reflection $A^{\\prime} B^{\\prime} C^{\\prime}$ in $\\ell$ has area more than $\\frac{2}{3}$ the area of triangle $A B C$.","solution":" All that's needed is: Claim - If $A B C$ is a triangle where $\\frac{1}{2}<\\frac{A B}{A C}<1$, then the $\\angle A$ bisector works. $$ \\frac{[A B D]}{[A B C]}=\\frac{B D}{B C}=\\frac{A B}{A B+A C} $$ by angle bisector theorem. In general, suppose $x2$, we have $\\left|\\frac{c-a}{-5}\\right| \\leq \\frac{|c|+9}{5}<|c|$. Thus, we eventually reach a pair with $|c| \\leq 2$. - Similarly, if $|b|>9$, we have $\\left|\\frac{b-a}{-2}\\right| \\leq \\frac{|b|+9}{2}<|b|$, so we eventually reach a pair with $|b| \\leq 9$. this leaves us with $5 \\cdot 19=95$ ordered pairs to check (though only about one third have $b \\equiv c(\\bmod 3))$. This can be done by the following code: ``` import functools @functools.lru_cache() def f(x0, yO): if x0 == 0 and y0 == 0: return 0 if x0 % 2 == (y0 % 5) % 2: d = y0 % 5 else: d = (y0 % 5) + 5 x1 = (x0 - d) \/\/ (-2) y1 = (y0 - d) \/\/ (-5) return 1 + f(x1, y1) for x in range(-9, 10): for y in range(-2, 3): if (x % 3 == y % 3): print(f\"({x:2d}, {y:2d}) finished in {f(x,y)} moves\") ``` As this gives the output ![](https:\/\/cdn.mathpix.com\/cropped\/2024_11_19_35408d86cf32c99e435fg-06.jpg?height=1317&width=664&top_left_y=1169&top_left_x=296) we are done."} +{"year":1997,"label":"4","problem":"To clip a convex $n$-gon means to choose a pair of consecutive sides $A B, B C$ and to replace them by the three segments $A M, M N$, and $N C$, where $M$ is the midpoint of $A B$ and $N$ is the midpoint of $B C$. In other words, one cuts off the triangle $M B N$ to obtain a convex $(n+1)$-gon. A regular hexagon $\\mathcal{P}_{6}$ of area 1 is clipped to obtain a heptagon $\\mathcal{P}_{7}$. Then $\\mathcal{P}_{7}$ is clipped (in one of the seven possible ways) to obtain an octagon $\\mathcal{P}_{8}$, and so on. Prove that no matter how the clippings are done, the area of $\\mathcal{P}_{n}$ is greater than $\\frac{1}{3}$, for all $n \\geq 6$.","solution":" Call the original hexagon $A B C D E F$. We show the area common to triangles $A C E$ and $B D F$ is in every $\\mathcal{P}_{n}$; this solves the problem since the area is $1 \/ 3$. For every side of a clipped polygon, we define its foundation recursively as follows: - $A B, B C, C D, D E, E F, F A$ are each their own foundation (we also call these original sides). - When a new clipped edge is added, its foundation is the union of the foundations of the two edges it touches. Hence, any foundations are nonempty subsets of original sides. Claim - All foundations are in fact at most two-element sets of adjacent original sides. Now, if a side has foundation contained in $\\{A B, B C\\}$, say, then the side should be contained within triangle $A B C$. Hence the side does not touch $A C$. This proves the problem."} +{"year":1997,"label":"5","problem":"If $a, b, c>0$ prove that $$ \\frac{1}{a^{3}+b^{3}+a b c}+\\frac{1}{b^{3}+c^{3}+a b c}+\\frac{1}{c^{3}+a^{3}+a b c} \\leq \\frac{1}{a b c} . $$","solution":" From $a^{3}+b^{3} \\geq a b(a+b)$, the left-hand side becomes $$ \\sum_{\\mathrm{cyc}} \\frac{1}{a^{3}+b^{3}+a b c} \\leq \\sum_{\\mathrm{cyc}} \\frac{1}{a b(a+b+c)}=\\frac{1}{a b c} \\sum_{\\mathrm{cyc}} \\frac{c}{a+b+c}=\\frac{1}{a b c} $$"} +{"year":1997,"label":"6","problem":"Suppose the sequence of nonnegative integers $a_{1}, a_{2}, \\ldots, a_{1997}$ satisfies $$ a_{i}+a_{j} \\leq a_{i+j} \\leq a_{i}+a_{j}+1 $$ for all $i, j \\geq 1$ with $i+j \\leq 1997$. Show that there exists a real number $x$ such that $a_{n}=\\lfloor n x\\rfloor$ for all $1 \\leq n \\leq 1997$.","solution":" We are trying to show there exists an $x \\in \\mathbb{R}$ such that $$ \\frac{a_{n}}{n} \\leq x<\\frac{a_{n}+1}{n} \\quad \\forall n $$ This means we need to show $$ \\max _{i} \\frac{a_{i}}{i}<\\min _{j} \\frac{a_{j}+1}{j} . $$ Replace 1997 by $N$. We will prove this by induction, but we will need some extra hypotheses on the indices $i, j$ which are used above. ## Claim - Suppose that - Integers $a_{1}, a_{2}, \\ldots, a_{N}$ satisfy the given conditions. - Let $i=\\operatorname{argmax}_{n} \\frac{a_{n}}{n}$; if there are ties, pick the smallest $i$. - Let $j=\\operatorname{argmin}_{n} \\frac{a_{n}+1}{n}$; if there are ties, pick the smallest $j$. Then $$ \\frac{a_{i}}{i}<\\frac{a_{j}+1}{j} $$ Moreover, these two fractions are in lowest terms, and are adjacent in the Farey sequence of order $\\max (i, j)$. Now, consider the new number $a_{N}$. We have two cases: - Suppose $i+j>N$. Then, no fraction with denominator $N$ can lie strictly inside the interval; so we may write for some integer $b$ $$ \\frac{b}{N} \\leq \\frac{a_{i}}{i}<\\frac{a_{j}+1}{j} \\leq \\frac{b+1}{N} $$ For purely algebraic reasons we have $$ \\frac{b-a_{i}}{N-i} \\leq \\frac{b}{N} \\leq \\frac{a_{i}}{i}<\\frac{a_{j}+1}{j} \\leq \\frac{b+1}{N} \\leq \\frac{b-a_{j}}{N-j} . $$ Now, $$ \\begin{aligned} a_{N} & \\geq a_{i}+a_{N-i} \\geq a_{i}+(N-i) \\cdot \\frac{a_{i}}{i} \\\\ & \\geq a_{i}+\\left(b-a_{i}\\right)=b \\\\ a_{N} & \\leq a_{j}+a_{N-j}+1 \\leq\\left(a_{j}+1\\right)+(N-j) \\cdot \\frac{a_{j}+1}{j} \\\\ & =\\left(a_{j}+1\\right)+\\left(b-a_{j}\\right)=b+1 \\end{aligned} $$ Thus $a_{N} \\in\\{b, b+1\\}$. This proves that $\\frac{a_{N}}{N} \\leq \\frac{a_{i}}{i}$ while $\\frac{a_{N}+1}{N} \\geq \\frac{a_{j}+1}{j}$. Moreover, the pair $(i, j)$ does not change, so all inductive hypotheses carry over. - On the other hand, suppose $i+j=N$. Then we have $$ \\frac{a_{i}}{i}<\\frac{a_{i}+a_{j}+1}{N}<\\frac{a_{j}+1}{j} . $$ Now, we know $a_{N}$ could be either $a_{i}+a_{j}$ or $a_{i}+a_{j}+1$. If it's the former, then $(i, j)$ becomes $(i, N)$. If it's the latter, then $(i, j)$ becomes $(N, j)$. The properties of Farey sequences ensure that the $\\frac{a_{i}+a_{j}+1}{N}$ is reduced, either way."} diff --git a/USAMO/segmented/en-USAMO-1998-notes.jsonl b/USAMO/segmented/en-USAMO-1998-notes.jsonl new file mode 100644 index 0000000000000000000000000000000000000000..ee04783025c4ed60129c5c22427469016db979aa --- /dev/null +++ b/USAMO/segmented/en-USAMO-1998-notes.jsonl @@ -0,0 +1,6 @@ +{"year":1998,"label":"1","problem":"Suppose that the set $\\{1,2, \\ldots, 1998\\}$ has been partitioned into disjoint pairs $\\left\\{a_{i}, b_{i}\\right\\}$ $(1 \\leq i \\leq 999)$ so that for all $i,\\left|a_{i}-b_{i}\\right|$ equals 1 or 6 . Prove that the sum $$ \\left|a_{1}-b_{1}\\right|+\\left|a_{2}-b_{2}\\right|+\\cdots+\\left|a_{999}-b_{999}\\right| $$ ends in the digit 9.","solution":" Let $S$ be the sum. Modulo 2, $$ S=\\sum\\left|a_{i}-b_{i}\\right| \\equiv \\sum\\left(a_{i}+b_{i}\\right)=1+2+\\cdots+1998 \\equiv 1 \\quad(\\bmod 2) $$ Modulo 5, $$ S=\\sum\\left|a_{i}-b_{i}\\right|=1 \\cdot 999 \\equiv 4 \\quad(\\bmod 5) $$ So $S \\equiv 9(\\bmod 10)$."} +{"year":1998,"label":"2","problem":"Let $\\mathcal{C}_{1}$ and $\\mathcal{C}_{2}$ be concentric circles, with $\\mathcal{C}_{2}$ in the interior of $\\mathcal{C}_{1}$. From a point $A$ on $\\mathcal{C}_{1}$ one draws the tangent $A B$ to $\\mathcal{C}_{2}\\left(B \\in \\mathcal{C}_{2}\\right)$. Let $C$ be the second point of intersection of ray $A B$ and $\\mathcal{C}_{1}$, and let $D$ be the midpoint of $\\overline{A B}$. A line passing through $A$ intersects $\\mathcal{C}_{2}$ at $E$ and $F$ in such a way that the perpendicular bisectors of $\\overline{D E}$ and $\\overline{C F}$ intersect at a point $M$ on line $A B$. Find, with proof, the ratio $A M \/ M C$.","solution":" By power of a point we have $$ A E \\cdot A F=A B^{2}=\\left(\\frac{1}{2} A B\\right) \\cdot(2 A B)=A D \\cdot A C $$ and hence $C D E F$ is cyclic. Then $M$ is the circumcenter of quadrilateral $C D E F$. ![](https:\/\/cdn.mathpix.com\/cropped\/2024_11_19_4d9e3ecba99734fe13c2g-4.jpg?height=806&width=792&top_left_y=1179&top_left_x=638) Thus $M$ is the midpoint of $\\overline{C D}$ (and we are given already that $B$ is the midpoint of $\\overline{A C}$, $D$ is the midpoint of $\\overline{A B}$ ). Thus a quick computation along $\\overline{A C}$ gives $A M \/ M C=5 \/ 3$."} +{"year":1998,"label":"3","problem":"Let $a_{0}, a_{1}, \\ldots, a_{n}$ be numbers from the interval $(0, \\pi \/ 2)$ such that $\\tan \\left(a_{0}-\\frac{\\pi}{4}\\right)+$ $\\tan \\left(a_{1}-\\frac{\\pi}{4}\\right)+\\cdots+\\tan \\left(a_{n}-\\frac{\\pi}{4}\\right) \\geq n-1$. Prove that $$ \\tan a_{0} \\tan a_{1} \\cdots \\tan a_{n} \\geq n^{n+1} $$","solution":"$ Let $x_{i}=\\tan \\left(a_{i}-\\frac{\\pi}{4}\\right)$. Then we have that $$ \\tan a_{i}=\\tan \\left(a_{i}-45^{\\circ}+45^{\\circ}\\right)=\\frac{x_{i}+1}{1-x_{i}} $$ If we further substitute $y_{i}=\\frac{1-x_{i}}{2} \\in(0,1)$, then we have to prove that the following statement: Claim - If $\\sum_{0}^{n} y_{i} \\leq 1$ and $y_{i} \\geq 0$, we have $$ \\prod_{i=1}^{n}\\left(\\frac{1}{y_{i}}-1\\right) \\geq n^{n+1} $$ $$ \\prod_{i=1}^{n}\\left(\\frac{y_{0}+y_{1}+y_{2}+\\cdots+y_{n}}{y_{i}}-1\\right) \\geq n^{n+1} $$ By AM-GM, we have $$ \\frac{y_{1}+y_{2}+y_{3}+\\cdots+y_{n}}{y_{0}} \\geq n \\sqrt[n]{\\frac{y_{1} y_{2} y_{3} \\ldots y_{n}}{y_{1}}} $$ Cyclic product works. Remark. Alternatively, the function $x \\mapsto \\log (1 \/ x-1)$ is a convex function on $(0,1)$ so Jensen inequality should also work."} +{"year":1998,"label":"4","problem":"A computer screen shows a $98 \\times 98$ chessboard, colored in the usual way. One can select with a mouse any rectangle with sides on the lines of the chessboard and click the mouse button: as a result, the colors in the selected rectangle switch (black becomes white, white becomes black). Find, with proof, the minimum number of mouse clicks needed to make the chessboard all one color.","solution":" The answer is 98 . One of several possible constructions is to toggle all columns and rows with even indices. In the other direction, let $n=98$ and suppose that $k$ rectangles are used, none of which are $n \\times n$ (else we may delete it). Then, for any two orthogonally adjacent cells, the edge between them must be contained in the edge of one of the $k$ rectangles. We define a gridline to be a line segment that runs in the interior of the board from one side of the board to the other. Hence there are $2 n-2$ gridlines exactly. Moreover, we can classify these rectangles into two types: - Full length rectangles: these span from one edge of the board to the other. The two long sides completely cover two gridlines, but the other two sides of the rectangle do not. - Partial length rectangles: each of four sides can partially cover \"half a\" gridline. See illustration below for $n=6$. ![](https:\/\/cdn.mathpix.com\/cropped\/2024_11_19_4d9e3ecba99734fe13c2g-6.jpg?height=321&width=327&top_left_y=1601&top_left_x=522) ![](https:\/\/cdn.mathpix.com\/cropped\/2024_11_19_4d9e3ecba99734fe13c2g-6.jpg?height=315&width=326&top_left_y=1604&top_left_x=865) Full length ![](https:\/\/cdn.mathpix.com\/cropped\/2024_11_19_4d9e3ecba99734fe13c2g-6.jpg?height=350&width=318&top_left_y=1601&top_left_x=1212) Partial length Since there are $2 n-2$ gridlines; and each rectangle can cover at most two gridlines in total (where partial-length rectangles are \"worth $\\frac{1}{2}$ \" on each of the four sides), we immediately get the bound $2 k \\geq 2 n-2$, or $k \\geq n-1$. To finish, we prove that: Claim - If equality holds and $k=n-1$, then $n$ is odd. Hence for $n=98$ the answer is indeed 98 as claimed."} +{"year":1998,"label":"5","problem":"Prove that for each $n \\geq 2$, there is a set $S$ of $n$ integers such that $(a-b)^{2}$ divides $a b$ for every distinct $a, b \\in S$.","solution":" This is a direct corollary of the more difficult USA TST 2015\/2, reproduced below. Prove that for every positive integer $n$, there exists a set $S$ of $n$ positive integers such that for any two distinct $a, b \\in S, a-b$ divides $a$ and $b$ but none of the other elements of $S$."} +{"year":1998,"label":"6","problem":"Let $n \\geq 5$ be an integer. Find the largest integer $k$ (as a function of $n$ ) such that there exists a convex $n$-gon $A_{1} A_{2} \\ldots A_{n}$ for which exactly $k$ of the quadrilaterals $A_{i} A_{i+1} A_{i+2} A_{i+3}$ have an inscribed circle, where indices are taken modulo $n$.","solution":" The main claim is the following: Claim - We can't have both $A_{1} A_{2} A_{3} A_{4}$ and $A_{2} A_{3} A_{4} A_{5}$ be circumscribed. ![](https:\/\/cdn.mathpix.com\/cropped\/2024_11_19_4d9e3ecba99734fe13c2g-8.jpg?height=621&width=892&top_left_y=1049&top_left_x=585) Then $A_{1} A_{4}=c+a-b$ and $A_{5} A_{2}=b+d-c$. But now $$ A_{1} A_{4}+A_{2} A_{5}=(c+a-b)+(b+d-c)=a+d=A_{1} A_{2}+A_{4} A_{5} $$ but in the picture we have an obvious violation of the triangle inequality. This immediately gives an upper bound of $\\lfloor n \/ 2\\rfloor$. For the construction, one can construct a suitable cyclic $n$-gon by using a continuity argument (details to be added)."} diff --git a/USAMO/segmented/en-USAMO-1999-notes.jsonl b/USAMO/segmented/en-USAMO-1999-notes.jsonl new file mode 100644 index 0000000000000000000000000000000000000000..1eebb8bfe58ac16561f0da64a0ce191f6960a4e4 --- /dev/null +++ b/USAMO/segmented/en-USAMO-1999-notes.jsonl @@ -0,0 +1,6 @@ +{"year":1999,"label":"1","problem":"Some checkers placed on an $n \\times n$ checkerboard satisfy the following conditions: (a) every square that does not contain a checker shares a side with one that does; (b) given any pair of squares that contain checkers, there is a sequence of squares containing checkers, starting and ending with the given squares, such that every two consecutive squares of the sequence share a side. Prove that at least $\\left(n^{2}-2\\right) \/ 3$ checkers have been placed on the board.","solution":" Take a spanning tree on the set $V$ of checkers where the $|V|-1$ edges of the tree are given by orthogonal adjacency. By condition (a) we have $$ \\sum_{v \\in V}(4-\\operatorname{deg} v) \\geq n^{2}-|V| $$ and since $\\sum_{v \\in V} \\operatorname{deg} v=2(|V|-1)$ we get $$ 4|V|-(2|V|-2) \\geq n^{2}-|V| $$ which implies $|V| \\geq \\frac{n^{2}-2}{3}$."} +{"year":1999,"label":"2","problem":"Let $A B C D$ be a convex cyclic quadrilateral. Prove that $$ |A B-C D|+|A D-B C| \\geq 2|A C-B D| $$","solution":"$ Let the diagonals meet at $P$, and let $A P=p q, D P=p r, B P=q s, C P=r s$. Then set $A B=q x, C D=r x, A D=p y, B C=s y$. In this way we compute $$ |A C-B D|=|(p-s)(q-r)| $$ and $$ |A B-C D|=|q-r| x $$ By triangle inequality on $\\triangle A X B$, we have $x \\geq|p-s|$. So $|A B-C D| \\geq|A C-B D|$."} +{"year":1999,"label":"3","problem":"Let $p>2$ be a prime and let $a, b, c, d$ be integers not divisible by $p$, such that $$ \\left\\{\\frac{r a}{p}\\right\\}+\\left\\{\\frac{r b}{p}\\right\\}+\\left\\{\\frac{r c}{p}\\right\\}+\\left\\{\\frac{r d}{p}\\right\\}=2 $$ for any integer $r$ not divisible by $p$. (Here, $\\{t\\}=t-\\lfloor t\\rfloor$ is the fractional part.) Prove that at least two of the numbers $a+b, a+c, a+d, b+c, b+d, c+d$ are divisible by $p$.","solution":" First of all, we apparently have $r(a+b+c+d) \\equiv 0(\\bmod p)$ for every prime $p$, so it automatically follows that $a+b+c+d \\equiv 0(\\bmod p)$. By scaling appropriately, and also replacing each number with its remainder modulo $p$, we are going to assume that $$ 1=a \\leq b \\leq c \\leq d

\\left\\lfloor\\frac{r d}{p}\\right\\rfloor \\geq\\left\\lfloor\\frac{(r-1) d}{p}\\right\\rfloor=r-2 $$ Now, we have that $$ 2(r-1)=\\left\\lfloor\\frac{r b}{p}\\right\\rfloor+\\left\\lfloor\\frac{r c}{p}\\right\\rfloor+\\underbrace{\\left\\lfloor\\frac{r d}{p}\\right\\rfloor}_{=r-2} . $$ Thus $\\left\\lfloor\\frac{r b}{p}\\right\\rfloor>\\left\\lfloor\\frac{(r-1) b}{p}\\right\\rfloor$, and $\\left\\lfloor\\frac{r c}{p}\\right\\rfloor>\\left\\lfloor\\frac{(r-1) b}{p}\\right\\rfloor$. An example of this situation is illustrated below with $r=7$ (not to scale). ![](https:\/\/cdn.mathpix.com\/cropped\/2024_11_19_4e30714afd39bc4610deg-06.jpg?height=330&width=1129&top_left_y=1000&top_left_x=469) Right now, $\\frac{b}{p}$ and $\\frac{c}{p}$ are just to the right of $\\frac{u}{r}$ and $\\frac{v}{r}$ for some $u$ and $v$ with $u+v=r$. The issue is that the there is some fraction just to the right of $\\frac{b}{p}$ and $\\frac{c}{p}$ from an earlier value of $r$, and by hypothesis its denominator is going to be strictly greater than 1 . It is at this point we are going to use the properties of Farey sequences. When we consider the fractions with denominator $r+1$, they are going to lie outside of the interval they we have constrained $\\frac{b}{p}$ and $\\frac{c}{p}$ to lie in. Indeed, our minimality assumption on $r$ guarantees that there is no fraction with denominator less than $r$ between $\\frac{u}{r}$ and $\\frac{b}{p}$. So if $\\frac{u}{r}<\\frac{b}{p}<\\frac{s}{t}$ (where $\\frac{u}{r}$ and $\\frac{s}{t}$ are the closest fractions with denominator at most $r$ to $\\frac{b}{p}$ ) then Farey theory says the next fraction inside the interval $\\left[\\frac{u}{r}, \\frac{s}{t}\\right]$ is $\\frac{u+s}{r+t}$, and since $t>1$, we have $r+t>r+1$. In other words, we get an inequality of the form $$ \\frac{u}{r}<\\frac{b}{p}<\\underbrace{\\text { something }}_{=s \/ t} \\leq \\frac{u+1}{r+1} . $$ The same holds for $\\frac{c}{p}$ as $$ \\frac{v}{r}<\\frac{c}{p}<\\text { something } \\leq \\frac{v+1}{r+1} $$ Finally, $$ \\frac{d}{p}<\\frac{r-1}{r}<\\frac{r}{r+1} $$ So now we have that $$ \\left\\lfloor\\frac{(r+1) b}{p}\\right\\rfloor+\\left\\lfloor\\frac{(r+1) c}{p}\\right\\rfloor+\\left\\lfloor\\frac{(r+1) d}{p}\\right\\rfloor \\leq u+v+(r-1)=2 r-1 $$ which is a contradiction. Now, since $$ \\frac{p-3}{p-2}<\\frac{d}{p} \\Longrightarrow d>\\frac{p(p-3)}{p-2}=p-1-\\frac{2}{p-2} $$ which for $p>2$ gives $d=p-1$."} +{"year":1999,"label":"4","problem":"Let $a_{1}, a_{2}, \\ldots, a_{n}$ be a sequence of $n>3$ real numbers such that $$ a_{1}+\\cdots+a_{n} \\geq n \\quad \\text { and } \\quad a_{1}^{2}+\\cdots+a_{n}^{2} \\geq n^{2} $$ Prove that $\\max \\left(a_{1}, \\ldots, a_{n}\\right) \\geq 2$.","solution":" Proceed by contradiction, assuming $a_{i}<2$ for all $i$. If all $a_{i} \\geq 0$, then $n^{2} \\leq \\sum_{i} a_{i}^{2}0)$, then we can replace them with $-(x+y)$ and 0 . So we may assume that there is exactly one negative term, say $a_{n}=-M$. Now, smooth all the nonnegative $a_{i}$ to be 2 , making all inequalities strict. Now, we have that $$ \\begin{aligned} 2(n-1)-M & >n \\\\ 4(n-1)+M^{2} & >n^{2} \\end{aligned} $$ This gives $n-20$."} +{"year":2000,"label":"2","problem":"Let $S$ be the set of all triangles $A B C$ for which $$ 5\\left(\\frac{1}{A P}+\\frac{1}{B Q}+\\frac{1}{C R}\\right)-\\frac{3}{\\min \\{A P, B Q, C R\\}}=\\frac{6}{r} $$ where $r$ is the inradius and $P, Q, R$ are the points of tangency of the incircle with sides $A B, B C, C A$ respectively. Prove that all triangles in $S$ are isosceles and similar to one another.","solution":" We will prove the inequality $$ \\frac{2}{A P}+\\frac{5}{B Q}+\\frac{5}{C R} \\geq \\frac{6}{r} $$ with equality when $A P: B Q: C R=1: 4: 4$. This implies the problem statement. Letting $x=A P, y=B Q, z=C R$, the inequality becomes $$ \\frac{2}{x}+\\frac{5}{y}+\\frac{5}{z} \\geq 6 \\sqrt{\\frac{x+y+z}{x y z}} . $$ Squaring both sides and collecting terms gives $$ \\frac{4}{x^{2}}+\\frac{25}{y^{2}}+\\frac{25}{z^{2}}+\\frac{14}{y z} \\geq \\frac{16}{x y}+\\frac{16}{x z} $$ If we replace $x=1 \/ a, y=4 \/ b, z=4 \/ c$, then it remains to prove the inequality $$ 64 a^{2}+25(b+c)^{2} \\geq 64 a(b+c)+36 b c $$ where equality holds when $a=b=c$. This follows by two applications of AM-GM: $$ \\begin{aligned} 16\\left(4 a^{2}+(b+c)^{2}\\right) & \\geq 64 a(b+c) \\\\ 9(b+c)^{2} & \\geq 36 b c . \\end{aligned} $$ Again one can tell this is an inequality by counting degrees of freedom."} +{"year":2000,"label":"3","problem":"A game of solitaire is played with $R$ red cards, $W$ white cards, and $B$ blue cards. A player plays all the cards one at a time. With each play he accumulates a penalty. If he plays a blue card, then he is charged a penalty which is the number of white cards still in his hand. If he plays a white card, then he is charged a penalty which is twice the number of red cards still in his hand. If he plays a red card, then he is charged a penalty which is three times the number of blue cards still in his hand. Find, as a function of $R, W$, and $B$, the minimal total penalty a player can amass and the number of ways in which this minimum can be achieved.","solution":" The minimum penalty is $$ f(B, W, R)=\\min (B W, 2 W R, 3 R B) $$ or equivalently, the natural guess of \"discard all cards of one color first\" is actually optimal (though not necessarily unique). This can be proven directly by induction. Indeed the base case $B W R=0$ (in which case zero penalty is clearly achievable). The inductive step follows from $$ f(B, W, R)=\\min \\left\\{\\begin{array}{l} f(B-1, W, R)+W \\\\ f(B, W-1, R)+2 R \\\\ f(B, W, R-1)+3 B \\end{array}\\right. $$ It remains to characterize the strategies. This is an annoying calculation, so we just state the result. - If any of the three quantities $B W, 2 W R, 3 R B$ is strictly smaller than the other three, there is one optimal strategy. - If $B W=2 W R<3 R B$, there are $W+1$ optimal strategies, namely discarding from 0 to $W$ white cards, then discarding all blue cards. (Each white card discarded still preserves $B W=2 W R$.) - If $2 W R=3 R B1000$. Then obviously some column has more than two tokens, so at most 999 tokens don't emit a death ray (namely, any token in its own column). Thus there are at least $n-999$ death rays. On the other hand, we can have at most 999 death rays total (since it would not be okay for the whole board to have death rays, as some row should have more than two tokens). Therefore, $n \\leq 999+999=1998$ as desired."} +{"year":2000,"label":"5","problem":"Let $A_{1} A_{2} A_{3}$ be a triangle, and let $\\omega_{1}$ be a circle in its plane passing through $A_{1}$ and $A_{2}$. Suppose there exists circles $\\omega_{2}, \\omega_{3}, \\ldots, \\omega_{7}$ such that for $k=2,3, \\ldots, 7$, circle $\\omega_{k}$ is externally tangent to $\\omega_{k-1}$ and passes through $A_{k}$ and $A_{k+1}$ (indices $\\bmod 3)$. Prove that $\\omega_{7}=\\omega_{1}$.","solution":" The idea is to keep track of the subtended arc $\\widehat{A_{i} A_{i+1}}$ of $\\omega_{i}$ for each $i$. To this end, let $\\beta=\\measuredangle A_{1} A_{2} A_{3}, \\gamma=\\measuredangle A_{2} A_{3} A_{1}$ and $\\alpha=\\measuredangle A_{1} A_{2} A_{3}$. ![](https:\/\/cdn.mathpix.com\/cropped\/2024_11_19_c2e5e56d46560bd13f74g-7.jpg?height=795&width=812&top_left_y=910&top_left_x=625) Initially, we set $\\theta=\\measuredangle O_{1} A_{2} A_{1}$. Then we compute $$ \\begin{aligned} & \\measuredangle O_{1} A_{2} A_{1}=\\theta \\\\ & \\measuredangle O_{2} A_{3} A_{2}=-\\beta-\\theta \\\\ & \\measuredangle O_{3} A_{1} A_{3}=\\beta-\\gamma+\\theta \\\\ & \\measuredangle O_{4} A_{2} A_{1}=(\\gamma-\\beta-\\alpha)-\\theta \\end{aligned} $$ and repeating the same calculation another round gives $$ \\measuredangle O_{7} A_{2} A_{1}=k-(k-\\theta)=\\theta $$ with $k=\\gamma-\\beta-\\alpha$. This implies $O_{7}=O_{1}$, so $\\omega_{7}=\\omega_{1}$."} +{"year":2000,"label":"6","problem":"Let $a_{1}, b_{1}, a_{2}, b_{2}, \\ldots, a_{n}, b_{n}$ be nonnegative real numbers. Prove that $$ \\sum_{i, j=1}^{n} \\min \\left\\{a_{i} a_{j}, b_{i} b_{j}\\right\\} \\leq \\sum_{i, j=1}^{n} \\min \\left\\{a_{i} b_{j}, a_{j} b_{i}\\right\\} $$","solution":" \u3010 First solution by creating a single min (Vincent Huang and Ravi Boppana). Let $b_{i}=r_{i} a_{i}$ for each $i$, and rewrite the inequality as $$ \\sum_{i, j} a_{i} a_{j}\\left[\\min \\left(r_{i}, r_{j}\\right)-\\min \\left(1, r_{i} r_{j}\\right)\\right] \\geq 0 $$ We now do the key manipulation to convert the double min into a separate single min. Let $\\varepsilon_{i}=+1$ if $r_{i} \\geq 1$, and $\\varepsilon_{i}=-1$ otherwise, and let $s_{i}=\\left|r_{i}-1\\right|$. Then we pass to absolute values: $$ \\begin{aligned} 2 \\min \\left(r_{i}, r_{j}\\right)-2 \\min \\left(1, r_{i} r_{j}\\right) & =\\left|r_{i} r_{j}-1\\right|-\\left|r_{i}-r_{j}\\right|-\\left(r_{i}-1\\right)\\left(r_{j}-1\\right) \\\\ & =\\left|r_{i} r_{j}-1\\right|-\\left|r_{i}-r_{j}\\right|-\\varepsilon_{i} \\varepsilon_{j} s_{i} s_{j} \\\\ & =\\varepsilon_{i} \\varepsilon_{j} \\min \\left(\\left|1-r_{i} r_{j} \\pm\\left(r_{i}-r_{j}\\right)\\right|\\right)-\\varepsilon_{i} \\varepsilon_{j} s_{i} s_{j} \\\\ & =\\varepsilon_{i} \\varepsilon_{j} \\min \\left(s_{i}\\left(r_{j}+1\\right), s_{j}\\left(r_{i}+1\\right)\\right)-\\varepsilon_{i} \\varepsilon_{j} s_{i} s_{j} \\\\ & =\\left(\\varepsilon_{i} s_{i}\\right)\\left(\\varepsilon_{j} s_{j}\\right) \\min \\left(\\frac{r_{j}+1}{s_{j}}-1, \\frac{r_{i}+1}{s_{i}}-1\\right) . \\end{aligned} $$ So let us denote $x_{i}=a_{i} \\varepsilon_{i} s_{i} \\in \\mathbb{R}$, and $t_{i}=\\frac{r_{i}+1}{s_{i}}-1 \\in \\mathbb{R}_{\\geq 0}$. Thus it suffices to prove that: Claim - We have $$ \\sum_{i, j} x_{i} x_{j} \\min \\left(t_{i}, t_{j}\\right) \\geq 0 $$ for arbitrary $x_{i} \\in \\mathbb{R}, t_{i} \\in \\mathbb{R} \\geq 0$. $$ \\sum_{i} t_{i} x_{i}^{2}+2 \\sum_{i0 \\text { and } a^{2}+b^{2}+c^{2}<1000\\right\\} . $$ This is an intersection of open sets, so it is open. Its closure is $$ \\bar{U}=\\left\\{(a, b, c) \\mid a, b, c \\geq 0 \\text { and } a^{2}+b^{2}+c^{2} \\leq 1000\\right\\} . $$ Hence the constraint set $$ \\bar{S}=\\{\\mathbf{x} \\in \\bar{U}: g(\\bar{x})=4\\} $$ is compact, where $g(a, b, c)=a^{2}+b^{2}+c^{2}+a b c$. Define $$ f(a, b, c)=a^{2}+b^{2}+c^{2}+a b+b c+c a $$ It's equivalent to show that $f \\leq 6$ subject to $g$. Over $\\bar{S}$, it must achieve a global maximum. Now we consider two cases. If $\\mathbf{x}$ lies on the boundary, that means one of the components is zero (since $a^{2}+b^{2}+c^{2}=$ 1000 is clearly impossible). WLOG $c=0$, then we wish to show $a^{2}+b^{2}+a b \\leq 6$ for $a^{2}+b^{2}=4$, which is trivial. Now for the interior $U$, we may use the method of Lagrange Multipliers. Consider a local maximum $\\mathbf{x} \\in U$. Compute $$ \\nabla f=\\langle 2 a+b+c, 2 b+c+a, 2 c+a+b\\rangle $$ and $$ \\nabla g=\\langle 2 a+b c, 2 b+c a, 2 c+a b\\rangle $$ Of course, $\\nabla g \\neq \\mathbf{0}$ everywhere, so introducing our multiplier yields $$ \\langle 2 a+b+c, a+2 b+c, a+b+2 c\\rangle=\\lambda\\langle 2 a+b c, 2 b+c a, 2 c+a b\\rangle . $$ Note that $\\lambda \\neq 0$ since $a, b, c>0$. Subtracting $2 a+b+c=\\lambda(2 a+b c)$ from $a+2 b+c=$ $\\lambda(2 b+c a)$ implies that $$ (a-b)([2 \\lambda-1]-\\lambda c)=0 . $$ We can derive similar equations for the others. Hence, we have three cases. 1. If $a=b=c$, then $a=b=c=1$, and this satisfies $f(1,1,1) \\leq 6$. 2. If $a, b, c$ are pairwise distinct, then we derive $a=b=c=2-\\lambda^{-1}$, contradiction. 3. Now suppose that $a=b \\neq c$. Meanwhile, the constraint (with $a=b$ ) reads $$ \\begin{aligned} a^{2}+b^{2}+c^{2}+a b c=4 & \\Longleftrightarrow c^{2}+a^{2} c+\\left(2 a^{2}-4\\right)=0 \\\\ & \\Longleftrightarrow(c+2)\\left(c-\\left(2-a^{2}\\right)\\right)=0 \\end{aligned} $$ which since $c>0$ gives $c=2-a^{2}$. Noah Walsh points out that at this point, we don't need to calculate the critical point; we just directly substitute $a=b$ and $c=2-a^{2}$ into the desired inequality: $$ a^{2}+2 a(2-a)^{2}-a^{2}(2-a)^{2}=2-(a-1)^{2}\\left(a^{2}-4 a+2\\right) \\leq 0 $$ So any point here satisfies the inequality anyways. Remark. It can actually be shown that the critical point in the third case we skipped is pretty close: it is given by $$ a=b=\\frac{1+\\sqrt{17}}{4} \\quad c=\\frac{1}{8}(7-\\sqrt{17}) . $$ This satisfies $$ f(a, b, c)=3 a^{2}+2 a c+c^{2}=\\frac{1}{32}(121+17 \\sqrt{17}) \\approx 5.97165 $$ which is just a bit less than 6 . Remark. Equality holds for the upper bound if $(a, b, c)=(1,1,1)$ or $(a, b, c)=(\\sqrt{2}, \\sqrt{2}, 0)$ and permutations. The lower bound is achieved if $(a, b, c)=(2,0,0)$ and permutations."} +{"year":2001,"label":"4","problem":"Let $A B C$ be a triangle and $P$ any point such that $P A, P B, P C$ are the sides of an obtuse triangle, with $P A$ the longest side. Prove that $\\angle B A C$ is acute.","solution":" Using Ptolemy's inequality and Cauchy-Schwarz, $$ \\begin{aligned} P A \\cdot B C & \\leq P B \\cdot A C+P C \\cdot A B \\\\ & \\leq \\sqrt{\\left(P B^{2}+P C^{2}\\right)\\left(A B^{2}+A C^{2}\\right)} \\\\ & <\\sqrt{P A^{2} \\cdot\\left(A B^{2}+A C\\right)^{2}}=P A \\cdot \\sqrt{A B^{2}+A C^{2}} \\end{aligned} $$ meaning $B C^{2}B P^{2}+P C^{2}$, so $A B^{2}+A C^{2}>B C^{2}$, and we get $\\angle B A C$ is acute."} +{"year":2001,"label":"5","problem":"Let $S \\subseteq \\mathbb{Z}$ be such that: (a) there exist $a, b \\in S$ with $\\operatorname{gcd}(a, b)=\\operatorname{gcd}(a-2, b-2)=1$; (b) if $x$ and $y$ are elements of $S$ (possibly equal), then $x^{2}-y$ also belongs to $S$. Prove that $S=\\mathbb{Z}$.","solution":" Call an integer $d>0$ shifty if $S=S+d$ (meaning $S$ is invariant under shifting by $d$ ). First, note that if $u, v \\in S$, then for any $x \\in S$, $$ v^{2}-\\left(u^{2}-x\\right)=\\left(v^{2}-u^{2}\\right)+x \\in S $$ Since we can easily check that $|S|>1$ and $S \\neq\\{n,-n\\}$ we conclude there exists a shifty integer. We claim 1 is shifty, which implies the problem. Assume for contradiction that 1 is not shifty. Then for GCD reasons the set of shifty integers must be $d \\mathbb{Z}$ for some $d \\geq 2$. Claim - We have $S \\subseteq\\left\\{x: x^{2} \\equiv m(\\bmod d)\\right\\}$ for some fixed $m$. Now take $a, b \\in S$ as in (a). In that case we need to have $$ a^{2} \\equiv b^{2} \\equiv\\left(a^{2}-a\\right)^{2} \\equiv\\left(b^{2}-b\\right)^{2} \\quad(\\bmod d) $$ Passing to a prime $p \\mid d$, we have the following: - Since $a^{2} \\equiv\\left(a^{2}-a\\right)^{2}(\\bmod p)$ or equivalently $a^{3}(a-2) \\equiv 0(\\bmod p)$, either $a \\equiv 0$ $(\\bmod p)$ or $a \\equiv 2(\\bmod p)$. - Similarly, either $b \\equiv 0(\\bmod p)$ or $b \\equiv 2(\\bmod p)$. - Since $a^{2} \\equiv b^{2}(\\bmod p)$, or $a \\equiv \\pm b(\\bmod p)$, we find either $a \\equiv b \\equiv 0(\\bmod p)$ or $a \\equiv b \\equiv 2(\\bmod p)($ even if $p=2)$. This is a contradiction. Remark. The condition (a) cannot be dropped, since otherwise we may take $S=\\{2(\\bmod p)\\}$ or $S=\\{0(\\bmod p)\\}$, say."} +{"year":2001,"label":"6","problem":"Each point in the plane is assigned a real number. Suppose that for any nondegenerate triangle, the number at its incenter is the arithmetic mean of the three numbers at its vertices. Prove that all points in the plane are equal to each other.","solution":" First, we claim that in an isosceles trapezoid $A B C D$ we have $a+c=b+d$. Indeed, suppose WLOG that rays $B A$ and $C D$ meet at $X$. Then triangles $X A C$ and $X B D$ share an incircle, proving the claim. Now, given any two points $A$ and $B$, construct regular pentagon $A B C D E$. We have $a+c=b+d=c+e=d+a=e+b$, so $a=b=c=d=e$."} diff --git a/USAMO/segmented/en-USAMO-2002-notes.jsonl b/USAMO/segmented/en-USAMO-2002-notes.jsonl new file mode 100644 index 0000000000000000000000000000000000000000..f2e1f9b7593db08a3904619a38e8356bc8062f65 --- /dev/null +++ b/USAMO/segmented/en-USAMO-2002-notes.jsonl @@ -0,0 +1,6 @@ +{"year":2002,"label":"1","problem":"Let $S$ be a set with 2002 elements, and let $N$ be an integer with $0 \\leq N \\leq 2^{2002}$. Prove that it is possible to color every subset of $S$ either black or white so that the following conditions hold: (a) the union of any two white subsets is white; (b) the union of any two black subsets is black; (c) there are exactly $N$ white subsets.","solution":" We will solve the problem with 2002 replaced by an arbitrary integer $n \\geq 0$. In other words, we prove: Claim - For any nonnegative integers $n$ and $N$ with $0 \\leq N \\leq 2^{n}$, it is possible to color the $2^{n}$ subsets of $\\{1, \\ldots, n\\}$ black and white satisfying the conditions of the problem. For the inductive step, we divide into two cases: - If $N \\leq 2^{n-1}$, then we take a coloring of subsets of $\\{1, \\ldots, n-1\\}$ with $N$ white sets; then we color the remaining $2^{n-1}$ sets (which contain $n$ ) black. - If $N>2^{n-1}$, then we take a coloring of subsets of $\\{1, \\ldots, n-1\\}$ with $N-2^{n-1}$ white sets; then we color the remaining $2^{n-1}$ sets (which contain $n$ ) white."} +{"year":2002,"label":"2","problem":"Let $A B C$ be a triangle such that $$ \\left(\\cot \\frac{A}{2}\\right)^{2}+\\left(2 \\cot \\frac{B}{2}\\right)^{2}+\\left(3 \\cot \\frac{C}{2}\\right)^{2}=\\left(\\frac{6 s}{7 r}\\right)^{2} $$ where $s$ and $r$ denote its semiperimeter and its inradius, respectively. Prove that triangle $A B C$ is similar to a triangle $T$ whose side lengths are all positive integers with no common divisors and determine these integers.","solution":" Let $x=s-a, y=s-b, z=s-c$ in the usual fashion, then the equation reads $$ x^{2}+4 y^{2}+9 z^{2}=\\left(\\frac{6}{7}(x+y+z)\\right)^{2} $$ However, by Cauchy-Schwarz, we have $$ \\left(1+\\frac{1}{4}+\\frac{1}{9}\\right)\\left(x^{2}+4 y^{2}+9 z^{2}\\right) \\geq(x+y+z)^{2} $$ with equality if and only if $1: \\frac{1}{2}: \\frac{1}{3}=x: 2 y: 3 z$, id est $x: y: z=1: \\frac{1}{4}: \\frac{1}{9}=36: 9: 4$. This is equivalent to $y+z: z+x: x+y=13: 40: 45$. Remark. You can tell this is not a geometry problem because you eliminate the cotangents right away to get an algebra problem...and then you realize the problem claims that one equation can determine three variables up to scaling, at which point you realize it has to be an inequality (otherwise degrees of freedom don't work). So of course, Cauchy-Schwarz..."} +{"year":2002,"label":"3","problem":"Prove that any monic polynomial (a polynomial with leading coefficient 1) of degree $n$ with real coefficients is the average of two monic polynomials of degree $n$ with $n$ real roots.","solution":" First, ## Lemma If $p$ is a monic polynomial of degree $n$, and $p(1) p(2)<0, p(2) p(3)<0, \\ldots$, $p(n-1) p(n)<0$ then $p$ has $n$ real roots. The last root is obtained by considering cases on $n(\\bmod 2)$. - If $n$ is even, then $p(1)$ and $p(n)$ have opposite sign, while we must have either $$ \\lim _{x \\rightarrow-\\infty} p(x)=\\lim _{x \\rightarrow \\infty} p(x)= \\pm \\infty $$ so we get one more root. - The $n$ odd case is similar, with $p(1)$ and $p(n)$ now having the same sign, but $\\lim _{x \\rightarrow-\\infty} p(x)=-\\lim _{x \\rightarrow \\infty} p(x)$ instead. Let $f(n)$ be the monic polynomial and let $M>1000 \\max _{t=1, \\ldots, n}|f(t)|+1000$. Then we may select reals $a_{1}, \\ldots, a_{n}$ and $b_{1}, \\ldots, b_{n}$ such that for each $k=1, \\ldots, n$, we have $$ \\begin{aligned} a_{k}+b_{k} & =2 f(k) \\\\ (-1)^{k} a_{k} & >M \\\\ (-1)^{k+1} b_{k} & >M . \\end{aligned} $$ We may interpolate monic polynomials $g$ and $h$ through the $a_{k}$ and $b_{k}$ (if the $a_{k}, b_{k}$ are selected \"generically\" from each other). Then one can easily check $f=\\frac{1}{2}(g+h)$ works."} +{"year":2002,"label":"4","problem":"Determine all functions $f: \\mathbb{R} \\rightarrow \\mathbb{R}$ such that $$ f\\left(x^{2}-y^{2}\\right)=x f(x)-y f(y) $$ for all pairs of real numbers $x$ and $y$.","solution":" The answer is $f(x)=c x, c \\in \\mathbb{R}$ (these obviously work). First, by putting $x=0$ and $y=0$ respectively we have $$ f\\left(x^{2}\\right)=x f(x) \\quad \\text { and } \\quad f\\left(-y^{2}\\right)=-y f(y) . $$ From this we deduce that $f$ is odd, in particular $f(0)=0$. Then, we can rewrite the given as $f\\left(x^{2}-y^{2}\\right)+f\\left(y^{2}\\right)=f\\left(x^{2}\\right)$. Combined with the fact that $f$ is odd, we deduce that $f$ is additive (i.e. $f(a+b)=f(a)+f(b)$ ). Remark (Philosophy). At this point we have $f\\left(x^{2}\\right) \\equiv x f(x)$ and $f$ additive, and everything we have including the given equation is a direct corollary of these two. So it makes sense to only focus on these two conditions. Then $$ \\begin{aligned} f\\left((x+1)^{2}\\right) & =(x+1) f(x+1) \\\\ \\Longrightarrow f\\left(x^{2}\\right)+2 f(x)+f(1) & =(x+1) f(x)+(x+1) f(1) \\end{aligned} $$ which readily gives $f(x)=f(1) x$."} +{"year":2002,"label":"5","problem":"Let $a, b$ be integers greater than 2. Prove that there exists a positive integer $k$ and a finite sequence $n_{1}, n_{2}, \\ldots, n_{k}$ of positive integers such that $n_{1}=a, n_{k}=b$, and $n_{i} n_{i+1}$ is divisible by $n_{i}+n_{i+1}$ for each $i(1 \\leq i2, n$ ! is connected to $(n+1)$ ! too: - $n!\\rightarrow(n+1)$ ! if $n$ is even - $n!\\rightarrow 2 n!\\rightarrow(n+1)!$ if $n$ is odd. This concludes the problem."} +{"year":2002,"label":"6","problem":"I have an $n \\times n$ sheet of stamps, from which I've been asked to tear out blocks of three adjacent stamps in a single row or column. (I can only tear along the perforations separating adjacent stamps, and each block must come out of the sheet in one piece.) Let $b(n)$ be the smallest number of blocks I can tear out and make it impossible to tear out any more blocks. Prove that there are real constants $c$ and $d$ such that $$ \\frac{1}{7} n^{2}-c n \\leq b(n) \\leq \\frac{1}{5} n^{2}+d n $$ for all $n>0$.","solution":" For the lower bound: there are $2 n(n-2)$ places one could put a block. Note that each block eliminates at most 14 such places. For the upper bound, the construction of $\\frac{1}{5}$ is easy to build. Here is an illustration of one possible construction for $n=9$ which generalizes readily, using only vertical blocks. $$ \\left[\\begin{array}{lllllllll} A & & E & & I & L & & P & \\\\ A & & E & G & & L & & P & R \\\\ A & C & & G & & L & N & & R \\\\ & C & & G & J & & N & & R \\\\ & C & F & & J & & N & Q & \\\\ B & & F & & J & M & & Q & \\\\ B & & F & H & & M & & Q & S \\\\ B & D & & H & & M & O & & S \\\\ & D & & H & K & & O & & S \\end{array}\\right] $$ Actually, for the lower bound, one may improve $1 \/ 7$ to $1 \/ 6$. Count the number $A$ of pairs of adjacent squares one of which is torn out and the other which is not: - For every deleted block, there are eight neighboring squares, at least two on each long edge which have been deleted too. Hence $N \\leq 6 b(n)$. - For every block still alive and not on the border, there are four neighboring squares, and clearly at least two are deleted. Hence $N \\geq 2\\left((n-2)^{2}-3 b(n)\\right)$. Collating these solves the problem."} diff --git a/USAMO/segmented/en-USAMO-2003-notes.jsonl b/USAMO/segmented/en-USAMO-2003-notes.jsonl new file mode 100644 index 0000000000000000000000000000000000000000..6e3cad3dc2390a5ff641d93f375ff1eb79b16dee --- /dev/null +++ b/USAMO/segmented/en-USAMO-2003-notes.jsonl @@ -0,0 +1,7 @@ +{"year":2003,"label":"1","problem":"Prove that for every positive integer $n$ there exists an $n$-digit number divisible by $5^{n}$ all of whose digits are odd.","solution":" This is immediate by induction on $n$. For $n=1$ we take 5 ; moving forward if $M$ is a working $n$-digit number then exactly one of $$ \\begin{aligned} & N_{1}=10^{n}+M \\\\ & N_{3}=3 \\cdot 10^{n}+M \\\\ & N_{5}=5 \\cdot 10^{n}+M \\\\ & N_{7}=7 \\cdot 10^{n}+M \\\\ & N_{9}=9 \\cdot 10^{n}+M \\end{aligned} $$ is divisible by $5^{n+1}$; as they are all divisible by $5^{n}$ and $N_{k} \/ 5^{n}$ are all distinct."} +{"year":2003,"label":"2","problem":"A convex polygon $\\mathcal{P}$ in the plane is dissected into smaller convex polygons by drawing all of its diagonals. The lengths of all sides and all diagonals of the polygon $\\mathcal{P}$ are rational numbers. Prove that the lengths of all sides of all polygons in the dissection are also rational numbers.","solution":" Suppose $A B$ is a side of a polygon in the dissection, lying on diagonal $X Y$, with $X, A$, $B, Y$ in that order. Then $$ A B=X Y-X A-Y B $$ In this way, we see that it actually just suffices to prove the result for a quadrilateral. \u3010 First approach (trig). Consider quadrilateral $A B C D$. There are twelve angles one can obtain using three of its four vertices, three at each vertex; denote this set of 12 angles by $S$ Note that: - The law of cosines implies $\\cos \\theta \\in \\mathbb{Q}$ for each $\\theta \\in S$. - Hence, $(\\sin \\theta)^{2} \\in \\mathbb{Q}$ for $\\theta \\in S$. (This is because $\\sin \\theta^{2}+\\cos ^{2} \\theta$.) We say two angles $\\theta_{1}$ and $\\theta_{2}$ are equivalent if $\\frac{\\sin \\theta_{1}}{\\sin \\theta_{2}}$ This is the same as saying, when $\\sin \\theta_{1}$ and $\\sin \\theta_{2}$ are written in simplest radical form, the part under the square root is the same. Now we contend: Claim - The angles $\\angle B A C, \\angle C A D, \\angle B A D$ are equivalent. $$ \\mathbb{Q} \\ni \\cos (\\angle B A D)=\\cos \\angle B A C \\cos \\angle C A D-\\sin \\angle B A C \\sin \\angle C A D $$ so $\\angle B A C$ and $\\angle C A D$ are equivalent. Then $$ \\sin (\\angle B A D)=\\sin \\angle B A C \\cos \\angle C A D+\\cos \\angle B A C \\sin \\angle C A D $$ implies $\\angle B A D$ is equivalent to those two. Claim - The angles $\\angle B A D, \\angle D B A, \\angle A D B$ are equivalent. Iterating the argument implies that all angles are equivalent. Now, if $A B$ and $C D$ meet at $E$, the law of sines on $\\triangle A E B$, etc. implies the result."} +{"year":2003,"label":"2","problem":"A convex polygon $\\mathcal{P}$ in the plane is dissected into smaller convex polygons by drawing all of its diagonals. The lengths of all sides and all diagonals of the polygon $\\mathcal{P}$ are rational numbers. Prove that the lengths of all sides of all polygons in the dissection are also rational numbers.","solution":" Suppose $A B$ is a side of a polygon in the dissection, lying on diagonal $X Y$, with $X, A$, $B, Y$ in that order. Then $$ A B=X Y-X A-Y B $$ In this way, we see that it actually just suffices to prove the result for a quadrilateral. I Second approach (barycentric coordinates). To do this, we apply barycentric coordinates. Consider quadrilateral $A B D C$ (note the changed order of vertices), with $A=(1,0,0), B=(0,1,0), C=(0,0,1)$. Let $D=(x, y, z)$, with $x+y+z=1$. By hypothesis, each of the numbers $$ \\begin{aligned} -a^{2} y z+b^{2}(1-x) z+c^{2}(1-x) y & =A D^{2} \\\\ a^{2}(1-y) z+b^{2} z x+c^{2}(1-y) x & =B D^{2} \\\\ -a^{2}(1-z) y-b^{2}(1-z) x+c^{2} x y & =C D^{2} \\end{aligned} $$ is rational. Let $W=a^{2} y z+b^{2} z x+c^{2} x y$. Then, $$ \\begin{aligned} b^{2} z+c^{2} y & =A D^{2}+W \\\\ a^{2} z+c^{2} x & =B D^{2}+W \\\\ a^{2} y+b^{2} x & =C D^{2}+W \\end{aligned} $$ This implies that $A D^{2}+B D^{2}+2 W-c^{2}=2 S_{C} z$ and cyclically (as usual $2 S_{C}=a^{2}+b^{2}-c^{2}$ ). If any of $S_{A}, S_{B}, S_{C}$ are zero, then we deduce $W$ is rational. Otherwise, we have that $$ 1=x+y+z=\\sum_{\\mathrm{cyc}} \\frac{A D^{2}+B D^{2}+2 W-c^{2}}{2 S_{C}} $$ which implies that $W$ is rational, because it appears with coefficient $\\frac{1}{S_{A}}+\\frac{1}{S_{B}}+\\frac{1}{S_{C}} \\neq 0$ (since $S_{B C}+S_{C A}+S_{A B}$ is actually the area of $A B C$ ). Hence from the rationality of $W$, we deduce that $x$ is rational as long as $S_{A} \\neq 0$, and similarly for the others. So at most one of $x, y, z$ is irrational, but since $x+y+z=1$ this implies they are all rational. Finally, if $P=\\overline{A D} \\cap \\overline{B C}$ then $A P=\\frac{1}{y+z} A D$, so $A P$ is rational too, completing the proof. Remark. After the reduction to quadrilateral, a third alternate approach goes by quoting Putnam 2018 A6, reproduced below: Four points are given in the plane, with no three collinear, such that the squares of the $\\binom{4}{2}=6$ pairwise distances are all rational. Show that the ratio of the areas between any two of the $\\binom{4}{3}=4$ triangles determined by these points is also rational. If $A B C D$ is the quadrilateral, the heights from $C$ and $D$ to $A B$ have rational ratio. Letting $P=A C \\cap B D$, we see $A P \/ A B$ can be shown as rational via coordinates, as needed."} +{"year":2003,"label":"3","problem":"Let $n$ be a positive integer. For every sequence of integers $$ A=\\left(a_{0}, a_{1}, a_{2}, \\ldots, a_{n}\\right) $$ satisfying $0 \\leq a_{i} \\leq i$, for $i=0, \\ldots, n$, we define another sequence $$ t(A)=\\left(t\\left(a_{0}\\right), t\\left(a_{1}\\right), t\\left(a_{2}\\right), \\ldots, t\\left(a_{n}\\right)\\right) $$ by setting $t\\left(a_{i}\\right)$ to be the number of terms in the sequence $A$ that precede the term $a_{i}$ and are different from $a_{i}$. Show that, starting from any sequence $A$ as above, fewer than $n$ applications of the transformation $t$ lead to a sequence $B$ such that $t(B)=B$.","solution":" We go by strong induction on $n$ with the base cases $n=1$ and $n=2$ done by hand. Consider two cases: - If $a_{0}=0$ and $a_{1}=1$, then $1 \\leq t\\left(a_{i}\\right) \\leq i$ for $i \\geq 1$; now apply induction to $$ \\left(t\\left(a_{1}\\right)-1, t\\left(a_{2}\\right)-1, \\ldots, t\\left(a_{n}\\right)-1\\right) $$ - Otherwise, assume that $a_{0}=a_{1}=\\cdots=a_{k-1}=0$ but $a_{k} \\neq 0$, where $k \\geq 2$. Assume $k1$ for which $a+b$ divides $a^{b}+b^{a}$.","solution":" One construction: let $d \\equiv 1(\\bmod 4), d>1$. Let $x=\\frac{d^{d}+2^{d}}{d+2}$. Then set $$ a=\\frac{x+d}{2}, \\quad b=\\frac{x-d}{2} . $$ To see this works, first check that $b$ is odd and $a$ is even. Let $d=a-b$ be odd. Then: $$ \\begin{aligned} a+b \\mid a^{b}+b^{a} & \\Longleftrightarrow(-b)^{b}+b^{a} \\equiv 0 \\quad(\\bmod a+b) \\\\ & \\Longleftrightarrow b^{a-b} \\equiv 1 \\quad(\\bmod a+b) \\\\ & \\Longleftrightarrow b^{d} \\equiv 1 \\quad(\\bmod d+2 b) \\\\ & \\Longleftrightarrow(-2)^{d} \\equiv d^{d}(\\bmod d+2 b) \\\\ & \\Longleftrightarrow d+2 b \\mid d^{d}+2^{d} . \\end{aligned} $$ So it would be enough that $$ d+2 b=\\frac{d^{d}+2^{d}}{d+2} \\Longrightarrow b=\\frac{1}{2}\\left(\\frac{d^{d}+2^{d}}{d+2}-d\\right) $$ which is what we constructed. Also, since $\\operatorname{gcd}(x, d)=1$ it follows $\\operatorname{gcd}(a, b)=\\operatorname{gcd}(d, b)=$ 1. Remark. Ryan Kim points out that in fact, $(a, b)=(2 n-1,2 n+1)$ is always a solution."} +{"year":2017,"label":"2","problem":"Let $m_{1}, m_{2}, \\ldots, m_{n}$ be a collection of $n$ positive integers, not necessarily distinct. For any sequence of integers $A=\\left(a_{1}, \\ldots, a_{n}\\right)$ and any permutation $w=w_{1}, \\ldots, w_{n}$ of $m_{1}, \\ldots, m_{n}$, define an $A$-inversion of $w$ to be a pair of entries $w_{i}, w_{j}$ with $iw_{j}$, - $w_{j}>a_{i} \\geq w_{i}$, or - $w_{i}>w_{j}>a_{i}$. Show that, for any two sequences of integers $A=\\left(a_{1}, \\ldots, a_{n}\\right)$ and $B=\\left(b_{1}, \\ldots, b_{n}\\right)$, and for any positive integer $k$, the number of permutations of $m_{1}, \\ldots, m_{n}$ having exactly $k A$-inversions is equal to the number of permutations of $m_{1}, \\ldots, m_{n}$ having exactly $k B$-inversions.","solution":" Denote by $M$ our multiset of $n$ positive integers. Define an inversion of a permutation to be pair $i0$ such that there exists a labeling of the lattice points in $\\mathbb{Z}^{2}$ with positive integers for which: - only finitely many distinct labels occur, and - for each label $i$, the distance between any two points labeled $i$ is at least $c^{i}$.","solution":" The construction for any $c<\\sqrt{2}$ can be done as follows. Checkerboard color the lattice points and label the black ones with 1 . The white points then form a copy of $\\mathbb{Z}^{2}$ again scaled up by $\\sqrt{2}$ so we can repeat the procedure with 2 on half the resulting points. Continue this dyadic construction until a large $N$ for which $c^{N}<2^{\\frac{1}{2}(N-1)}$, at which point we can just label all the points with $N$. I'll now prove that $c=\\sqrt{2}$ (and hence $c \\geq \\sqrt{2}$ ) can't be done. Claim - It is impossible to fill a $2^{n} \\times 2^{n}$ square with labels not exceeding $2 n$. The case $n=1$ is clear. So now assume it's true up to $n-1$; and assume for contradiction a $2^{n} \\times 2^{n}$ square $S$ only contains labels up to $2 n$. (Of course every $2^{n-1} \\times 2^{n-1}$ square contains an instance of a label at least $2 n-1$.) ![](https:\/\/cdn.mathpix.com\/cropped\/2024_11_19_31dda431671476b0de3cg-12.jpg?height=647&width=632&top_left_y=1484&top_left_x=712) Now, we contend there are fewer than four copies of $2 n$ : ## Lemma In a unit square, among any four points, two of these points have distance $\\leq 1$ apart. So WLOG the northwest quadrant has no $2 n$ 's. Take a $2 n-1$ in the northwest and draw a square of size $2^{n-1} \\times 2^{n-1}$ directly right of it (with its top edge coinciding with the top of $S$ ). Then $A$ can't contain $2 n-1$, so it must contain a $2 n$ label; that $2 n$ label must be in the northeast quadrant. Then we define a square $B$ of size $2^{n-1} \\times 2^{n-1}$ as follows. If $2 n-1$ is at least as high $2 n$, let $B$ be a $2^{n-1} \\times 2^{n-1}$ square which touches $2 n-1$ north and is bounded east by $2 n$. Otherwise let $B$ be the square that touches $2 n-1$ west and is bounded north by $2 n$. We then observe $B$ can neither have $2 n-1$ nor $2 n$ in it, contradiction. Remark. To my knowledge, essentially all density arguments fail because of hexagonal lattice packing."} +{"year":2017,"label":"6","problem":"Find the minimum possible value of $$ \\frac{a}{b^{3}+4}+\\frac{b}{c^{3}+4}+\\frac{c}{d^{3}+4}+\\frac{d}{a^{3}+4} $$ given that $a, b, c, d$ are nonnegative real numbers such that $a+b+c+d=4$.","solution":" The minimum $\\frac{2}{3}$ is achieved at $(a, b, c, d)=(2,2,0,0)$ and cyclic permutations. The problem is an application of the tangent line trick: we observe the miraculous identity $$ \\frac{1}{b^{3}+4} \\geq \\frac{1}{4}-\\frac{b}{12} $$ since $12-(3-b)\\left(b^{3}+4\\right)=b(b+1)(b-2)^{2} \\geq 0$. Moreover, $$ a b+b c+c d+d a=(a+c)(b+d) \\leq\\left(\\frac{(a+c)+(b+d)}{2}\\right)^{2}=4 $$ Thus $$ \\sum_{\\mathrm{cyc}} \\frac{a}{b^{3}+4} \\geq \\frac{a+b+c+d}{4}-\\frac{a b+b c+c d+d a}{12} \\geq 1-\\frac{1}{3}=\\frac{2}{3} $$ Remark. The main interesting bit is the equality at $(a, b, c, d)=(2,2,0,0)$. This is the main motivation for trying tangent line trick, since a lower bound of the form $\\sum a(1-\\lambda b)$ preserves the unusual equality case above. Thus one takes the tangent at $b=2$ which miraculously passes through the point $(0,1 \/ 4)$ as well."} diff --git a/USAMO/segmented/en-USAMO-2018-notes.jsonl b/USAMO/segmented/en-USAMO-2018-notes.jsonl new file mode 100644 index 0000000000000000000000000000000000000000..58c78aedb6a4976af5cafdb352fccb0d6cea1c88 --- /dev/null +++ b/USAMO/segmented/en-USAMO-2018-notes.jsonl @@ -0,0 +1,8 @@ +{"year":2018,"label":"1","problem":"Let $a, b, c$ be positive real numbers such that $a+b+c=4 \\sqrt[3]{a b c}$. Prove that $$ 2(a b+b c+c a)+4 \\min \\left(a^{2}, b^{2}, c^{2}\\right) \\geq a^{2}+b^{2}+c^{2} . $$","solution":" WLOG let $c=\\min (a, b, c)=1$ by scaling. The given inequality becomes equivalent to $$ 4 a b+2 a+2 b+3 \\geq(a+b)^{2} \\quad \\forall a+b=4(a b)^{1 \/ 3}-1 $$ Now, let $t=(a b)^{1 \/ 3}$ and eliminate $a+b$ using the condition, to get $$ 4 t^{3}+2(4 t-1)+3 \\geq(4 t-1)^{2} \\Longleftrightarrow 0 \\leq 4 t^{3}-16 t^{2}+16 t=4 t(t-2)^{2} $$ which solves the problem. Equality occurs only if $t=2$, meaning $a b=8$ and $a+b=7$, which gives $$ \\{a, b\\}=\\left\\{\\frac{7 \\pm \\sqrt{17}}{2}\\right\\} $$ with the assumption $c=1$. Scaling gives the curve of equality cases."} +{"year":2018,"label":"2","problem":"Find all functions $f:(0, \\infty) \\rightarrow(0, \\infty)$ such that $$ f\\left(x+\\frac{1}{y}\\right)+f\\left(y+\\frac{1}{z}\\right)+f\\left(z+\\frac{1}{x}\\right)=1 $$ for all $x, y, z>0$ with $x y z=1$.","solution":" The main part of the problem is to show all solutions are linear. As always, let $x=b \/ c$, $y=c \/ a, z=a \/ b$ (classical inequality trick). Then the problem becomes $$ \\sum_{\\mathrm{cyc}} f\\left(\\frac{b+c}{a}\\right)=1 $$ Let $f(t)=g\\left(\\frac{1}{t+1}\\right)$, equivalently $g(s)=f(1 \/ s-1)$. Thus $g:(0,1) \\rightarrow(0,1)$ which satisfies $\\sum_{\\mathrm{cyc}} g\\left(\\frac{a}{a+b+c}\\right)=1$, or equivalently $$ g(a)+g(b)+g(c)=1 \\quad \\forall a+b+c=1 . $$ Claim - The function $g$ is linear. - First, whenever $a+b \\leq 1$ we have $$ 1-g(1-(a+b))=g(a)+g(b)=2 g\\left(\\frac{a+b}{2}\\right) $$ Hence $g$ obeys Jensen's functional equation over $(0,1 \/ 2)$. - Define $h:[0,1] \\rightarrow \\mathbb{R}$ by $h(t)=g\\left(\\frac{2 t+1}{8}\\right)-(1-t) \\cdot g(1 \/ 8)-t \\cdot g(3 \/ 8)$, then $h$ satisfies Jensen's functional equation too over $[0,1]$. We have also arranged that $h(0)=h(1)=0$, hence $h(1 \/ 2)=0$ as well. - Since $$ h(t)=h(t)+h(1 \/ 2)=2 h(t \/ 2+1 \/ 4)=h(t+1 \/ 2)+h(0)=h(t+1 \/ 2) $$ for any $t<1 \/ 2$, we find $h$ is periodic modulo $1 \/ 2$. It follows one can extend $\\widetilde{h}$ by $$ \\widetilde{h}: \\mathbb{R} \\rightarrow \\mathbb{R} \\quad \\text { by } \\quad \\widetilde{h}(t)=h(t-\\lfloor t\\rfloor) $$ and still satisfy Jensen's functional equation. Because $\\widetilde{h}(0)=0$, it's well-known this implies $\\widetilde{h}$ is additive (because $\\widetilde{h}(x+y)=2 \\widetilde{h}((x+y) \/ 2)=\\widetilde{h}(x)+\\widetilde{h}(y)$ for any real numbers $x$ to $y$ ). But $\\widetilde{h}$ is bounded below on $[0,1]$ since $g \\geq 0$, and since $\\widetilde{h}$ is also additive, it follows (well-known) that $\\widetilde{h}$ is linear. Thus $h$ is the zero function. So, the function $g$ is linear over $[1 \/ 8,3 \/ 8]$; thus we may write $g(x)=k x+\\ell$, valid for $1 \/ 8 \\leq x \\leq 3 \/ 8$. Since $3 g(1 \/ 3)=1$, it follows $k+3 \\ell=1$. For $01$. Finally, note that the number of neighbors of $\\mathcal{M}$ is the product across all $\\ell$ of the above. So it is odd if and only if each factor is odd, if and only if $n_{\\ell}=1$ for every $\\ell$. To finish, consider a huge simple graph $\\Gamma$ on all the maximal matchings, with edge relations given by neighbor relation (we don't consider vertices to be connected to themselves). Observe that: - Fantastic matchings correspond to isolated vertices (of degree zero, with no other neighbors) of $\\Gamma$. - The rest of the vertices of $\\Gamma$ have odd degrees (one less than the neighbor count) - The graph $\\Gamma$ has an even number of vertices of odd degree (this is true for any simple graph, see \"handshake lemma\"). - The number of vertices of $\\Gamma$ is odd, namely $(2\\lceil n \/ 2\\rceil-1)!!$"} diff --git a/USAMO/segmented/en-USAMO-2019-notes.jsonl b/USAMO/segmented/en-USAMO-2019-notes.jsonl new file mode 100644 index 0000000000000000000000000000000000000000..cd2f3a77f6e743cc772571b7933f182dc5f92230 --- /dev/null +++ b/USAMO/segmented/en-USAMO-2019-notes.jsonl @@ -0,0 +1,8 @@ +{"year":2019,"label":"1","problem":"A function $f: \\mathbb{N} \\rightarrow \\mathbb{N}$ satisfies $$ \\underbrace{f(f(\\ldots f}_{f(n) \\text { times }}(n) \\ldots))=\\frac{n^{2}}{f(f(n))} $$ for all positive integers $n$. What are all possible values of $f(1000)$ ?","solution":" Actually, we classify all such functions: $f$ can be any function which fixes odd integers and acts as an involution on the even integers. In particular, $f(1000)$ may be any even integer. It's easy to check that these all work, so now we check they are the only solutions. Claim - $f$ is injective. Claim - $f$ fixes the odd integers. Assume $f$ fixes $S=\\{1,3, \\ldots, n-2\\}$ now (allowing $S=\\varnothing$ for $n=1$ ). Now we have that $$ f^{f(n)}(n) \\cdot f^{2}(n)=n^{2} . $$ However, neither of the two factors on the left-hand side can be in $S$ since $f$ was injective. Therefore they must both be $n$, and we have $f^{2}(n)=n$. Now let $y=f(n)$, so $f(y)=n$. Substituting $y$ into the given yields $$ y^{2}=f^{n}(y) \\cdot y=f^{n+1}(n) \\cdot y=n y $$ since $n+1$ is even. We conclude $n=y$, as desired. Remark (Motivation). After obtaining $f(1)=1$ and $f$ injective, here is one way to motivate where the above proof comes from. From the equation $$ f^{f(n)}(n) \\cdot f^{2}(n)=n^{2} $$ it would be natural to consider the case where $n$ is prime, because $p^{2}$ only has a few possible factorizations. In fact, actually because of injectivity and $f(1)=1$, we would need to have $$ f^{f(p)}(p)=f^{2}(p)=p $$ in order for the equation to be true. Continuing on as in the proof above, one then gets $f(p)=p$ for odd primes $p$ (but no control over $f(2)$ ). The special case of prime $n$ then serves as a foothold by which one can continue the induction towards all numbers $n$, finding the induction works out exactly when the prime 2 never appears. As a general point, in mathematical problem-solving, one often needs to be willing to try out a proof idea or strategy and then retroactively determine what hypothesis is needed, rather than hoping one will always happen to guess exactly the right claim first. In other words, it may happen that one begins working out a proof of a claim before knowing exactly what the claim will turn out to say, and this is the case here (despite the fact the proof strategy uses induction). Thus, $f$ maps even integers to even integers. In light of this, we may let $g:=f(f(n))$ (which is also injective), so we conclude that $$ g^{f(n) \/ 2}(n) g(n)=n^{2} \\quad \\text { for } n=2,4, \\ldots $$ Claim - The function $g$ is the identity function. $$ g^{f(n) \/ 2}(n) \\cdot g(n)=n^{2} $$ neither of the two factors may be less than $n$. So they must both be $n$. Remark. The last claim is not necessary to solve the problem; after realizing $f$ and injective fixes the odd integers, this answers the question about the values of $f(1000)$. However, we chose to present the \"full\" solution anyways. Remark. After noting $f$ is injective, another approach is outlined below. Starting from any $n$, consider the sequence $$ n, f(n), f(f(n)) $$ and so on. We may let $m$ be the smallest term of the sequence; then $m^{2}=f(f(m)) \\cdot f^{f(m)}(m)$ which forces $f(f(m))=f^{f(m)}(m)=m$ by minimality. Thus the sequence is 2 -periodic. Therefore, $f(f(n))=n$ always holds, which is enough to finish. \\ Authorship comments. I will tell you a great story about this problem. Two days before the start of grading of USAMO 2017, I had a dream that I was grading a functional equation. When I woke up, I wrote it down, and it was $$ f^{f(n)}(n)=\\frac{n^{2}}{f(f(n))} $$ You can guess the rest of the story (and imagine how surprised I was the solution set was interesting). I guess some dreams do come true, huh?"} +{"year":2019,"label":"2","problem":"Let $A B C D$ be a cyclic quadrilateral satisfying $A D^{2}+B C^{2}=A B^{2}$. The diagonals of $A B C D$ intersect at $E$. Let $P$ be a point on side $\\overline{A B}$ satisfying $\\angle A P D=\\angle B P C$. Show that line $P E$ bisects $\\overline{C D}$.","solution":" \u3010 First solution using symmedians. We define point $P$ to obey $$ \\frac{A P}{B P}=\\frac{A D^{2}}{B C^{2}}=\\frac{A E^{2}}{B E^{2}} $$ so that $\\overline{P E}$ is the $E$-symmedian of $\\triangle E A B$, therefore the $E$-median of $\\triangle E C D$. Now, note that $$ A D^{2}=A P \\cdot A B \\quad \\text { and } \\quad B C^{2}=B P \\cdot B A $$ This implies $\\triangle A P D \\sim \\triangle A D B$ and $\\triangle B P C \\sim \\triangle B C A$. Thus $$ \\measuredangle D P A=\\measuredangle A D B=\\measuredangle A C B=\\measuredangle B C P $$ and so $P$ satisfies the condition as in the statement (and is the unique point to do so), as needed."} +{"year":2019,"label":"2","problem":"Let $A B C D$ be a cyclic quadrilateral satisfying $A D^{2}+B C^{2}=A B^{2}$. The diagonals of $A B C D$ intersect at $E$. Let $P$ be a point on side $\\overline{A B}$ satisfying $\\angle A P D=\\angle B P C$. Show that line $P E$ bisects $\\overline{C D}$.","solution":" \u3010 Second solution using only angle chasing (by proposer). We again re-define $P$ to obey $A D^{2}=A P \\cdot A B$ and $B C^{2}=B P \\cdot B A$. As before, this gives $\\triangle A P D \\sim \\triangle A B D$ and $\\triangle B P C \\sim \\triangle B D P$ and so we let $$ \\theta:=\\measuredangle D P A=\\measuredangle A D B=\\measuredangle A C B=\\measuredangle B C P . $$ Our goal is to now show $\\overline{P E}$ bisects $\\overline{C D}$. Let $K=\\overline{A C} \\cap \\overline{P D}$ and $L=\\overline{A D} \\cap \\overline{P C}$. Since $\\measuredangle K P A=\\theta=\\measuredangle A C B$, quadrilateral $B P K C$ is cyclic. Similarly, so is $A P L D$. ![](https:\/\/cdn.mathpix.com\/cropped\/2024_11_19_dab316fe5a4e03dd61cdg-06.jpg?height=804&width=798&top_left_y=249&top_left_x=635) Finally $A K L B$ is cyclic since $$ \\measuredangle B K A=\\measuredangle B K C=\\measuredangle B P C=\\theta=\\measuredangle D P A=\\measuredangle D L A=\\measuredangle B L A . $$ This implies $\\measuredangle C K L=\\measuredangle L B A=\\measuredangle D C K$, so $\\overline{K L} \\| \\overline{B C}$. Then $P E$ bisects $\\overline{B C}$ by Ceva's theorem on $\\triangle P C D$."} +{"year":2019,"label":"2","problem":"Let $A B C D$ be a cyclic quadrilateral satisfying $A D^{2}+B C^{2}=A B^{2}$. The diagonals of $A B C D$ intersect at $E$. Let $P$ be a point on side $\\overline{A B}$ satisfying $\\angle A P D=\\angle B P C$. Show that line $P E$ bisects $\\overline{C D}$.","solution":" \u3010 Third solution (using inversion). By hypothesis, the circle $\\omega_{a}$ centered at $A$ with radius $A D$ is orthogonal to the circle $\\omega_{b}$ centered at $B$ with radius $B C$. For brevity, we let $\\mathbf{I}_{a}$ and $\\mathbf{I}_{b}$ denote inversion with respect to $\\omega_{a}$ and $\\omega_{b}$. We let $P$ denote the intersection of $\\overline{A B}$ with the radical axis of $\\omega_{a}$ and $\\omega_{b}$; hence $P=\\mathbf{I}_{a}(B)=\\mathbf{I}_{b}(A)$. This already implies that $$ \\measuredangle D P A \\stackrel{\\mathbf{I}_{a}}{=} \\measuredangle A D B=\\measuredangle A C B \\stackrel{\\mathbf{I}_{b}}{=} \\measuredangle B P C $$ so $P$ satisfies the angle condition. ![](https:\/\/cdn.mathpix.com\/cropped\/2024_11_19_dab316fe5a4e03dd61cdg-06.jpg?height=732&width=1014&top_left_y=1844&top_left_x=521) Claim - The point $K=\\mathbf{I}_{a}(C)$ lies on $\\omega_{b}$ and $\\overline{D P}$. Similarly $L=\\mathbf{I}_{b}(D)$ lies on $\\omega_{a}$ and $\\overline{C P}$. Finally, since $C, L, P$ are collinear, we get $A$ is concyclic with $K=\\mathbf{I}_{a}(C), L=\\mathbf{I}_{a}(L)$, $B=\\mathbf{I}_{a}(P)$, i.e. that $A K L B$ is cyclic. So $\\overline{K L} \\| \\overline{C D}$ by Reim's theorem, and hence $\\overline{P E}$ bisects $\\overline{C D}$ by Ceva's theorem."} +{"year":2019,"label":"3","problem":"Let $K$ be the set of positive integers not containing the decimal digit 7. Determine all polynomials $f(x)$ with nonnegative coefficients such that $f(x) \\in K$ for all $x \\in K$.","solution":" The answer is only the obvious ones: $f(x)=10^{e} x, f(x)=k$, and $f(x)=10^{e} x+k$, for any choice of $k \\in K$ and $e>\\log _{10} k$ (with $e \\geq 0$ ). Now assume $f$ satisfies $f(K) \\subseteq K$; such polynomials will be called stable. We first prove the following claim which reduces the problem to the study of monomials. Lemma (Reduction to monomials) If $f(x)=a_{0}+a_{1} x+a_{2} x^{2}+\\ldots$ is stable, then each monomial $a_{0}, a_{1} x, a_{2} x^{2}, \\ldots$ is stable. Let's tackle the linear case next. Here is an ugly but economical proof. Claim (Linear classification) - If $f(x)=c x$ is stable, then $c=10^{e}$ for some nonnegative integer $e$. - For $9 \\cdot 10^{e} \\leq c<10 \\cdot 10^{e}$, pick $x=8$. - For $8 \\cdot 10^{e} \\leq c<9 \\cdot 10^{e}$, pick $x=88$. - For $7 \\cdot 10^{e} \\leq c<8 \\cdot 10^{e}$, pick $x=1$. - For $4.4 \\cdot 10^{e} \\leq c<7 \\cdot 10^{e}$, pick $11 \\leq x \\leq 16$. - For $2.7 \\cdot 10^{e} \\leq c<4.4 \\cdot 10^{e}$, pick $18 \\leq x \\leq 26$. - For $2 \\cdot 10^{e} \\leq c<2.7 \\cdot 10^{e}$, pick $28 \\leq x \\leq 36$. - For $1.6 \\cdot 10^{e} \\leq c<2 \\cdot 10^{e}$, pick $38 \\leq x \\leq 46$. - For $1.3 \\cdot 10^{e} \\leq c<1.6 \\cdot 10^{e}$, pick $48 \\leq x \\leq 56$. - For $1.1 \\cdot 10^{e} \\leq c<1.3 \\cdot 10^{e}$, pick $58 \\leq x \\leq 66$. - For $1 \\cdot 10^{e} \\leq c<1.1 \\cdot 10^{e}$, pick $x=699 \\ldots 9$ for suitably many 9 's. The hardest part of the problem is the case where $\\operatorname{deg} f>1$. We claim that no solutions exist then: Claim (Higher-degree classification) - No monomial of the form $f(x)=c x^{d}$ is stable for any $d>1$. $$ f(10 x+3)=3^{d}+10 d \\cdot 3^{d-1} x+100\\binom{d}{2} \\cdot 3^{d-1} x^{2}+\\ldots $$ is stable. By applying the lemma the linear monomial $10 d \\cdot 3^{d-1} x$ is stable, so $10 d \\cdot 3^{d-1}$ is a power of 10 , which can only happen if $d=1$. Thus the only nonconstant stable polynomials with nonnegative coefficients must be of the form $f(x)=10^{e} x+k$ for $e \\geq 0$. It is straightforward to show we then need $k<10^{e}$ and this finishes the proof."} +{"year":2019,"label":"4","problem":"Let $n$ be a nonnegative integer. Determine the number of ways to choose sets $S_{i j} \\subseteq\\{1,2, \\ldots, 2 n\\}$, for all $0 \\leq i \\leq n$ and $0 \\leq j \\leq n$ (not necessarily distinct), such that - $\\left|S_{i j}\\right|=i+j$, and - $S_{i j} \\subseteq S_{k l}$ if $0 \\leq i \\leq k \\leq n$ and $0 \\leq j \\leq l \\leq n$.","solution":" The answer is $(2 n)!\\cdot 2^{n^{2}}$. First, we note that $\\varnothing=S_{00} \\subsetneq S_{01} \\subsetneq \\cdots \\subsetneq S_{n n}=\\{1, \\ldots, 2 n\\}$ and thus multiplying by (2n)! we may as well assume $S_{0 i}=\\{1, \\ldots, i\\}$ and $S_{i n}=\\{1, \\ldots, n+i\\}$. We illustrate this situation by placing the sets in a grid, as below for $n=4$; our goal is to fill in the rest of the grid. $\\left[\\begin{array}{ccccc}1234 & 12345 & 123456 & 1234567 & 12345678 \\\\ 123 & & & & \\\\ 12 & & & & \\\\ 1 & & & & \\\\ \\varnothing & & & & \\end{array}\\right]$ We claim the number of ways to do so is $2^{n^{2}}$. In fact, more strongly even the partial fillings are given exactly by powers of 2 . Claim - Fix a choice $T$ of cells we wish to fill in, such that whenever a cell is in $T$, so are all the cells above and left of it. (In other words, $T$ is a Young tableau.) The number of ways to fill in these cells with sets satisfying the inclusion conditions is $2^{|T|}$ \u3002 An example is shown below, with an indeterminate set marked in red (and the rest of $T$ marked in blue). $\\left[\\begin{array}{ccccc}1234 & 12345 & 123456 & 1234567 & 12345678 \\\\ 123 & 1234 & 12346 & 123467 & \\\\ 12 & 124 & 1234 \\text { or } 1246 & & \\\\ 1 & 12 & & & \\\\ \\varnothing & 2 & & & \\end{array}\\right]$ Now suppose we have a corner $\\left[\\begin{array}{ll}B & C \\\\ A & S\\end{array}\\right]$ where $A, B, C$ are fixed and $S$ is to be chosen. Then we may write $B=A \\cup\\{x\\}$ and $C=A \\cup\\{x, y\\}$ for $x, y \\notin A$. Then the two choices of $S$ are $A \\cup\\{x\\}$ (i.e. $B$ ) and $A \\cup\\{y\\}$, and both of them are seen to be valid. In this way, we gain a factor of 2 any time we add one cell as above to $T$. Since we can achieve any Young tableau in this way, the induction is complete."} +{"year":2019,"label":"5","problem":"Let $m$ and $n$ be relatively prime positive integers. The numbers $\\frac{m}{n}$ and $\\frac{n}{m}$ are written on a blackboard. At any point, Evan may pick two of the numbers $x$ and $y$ written on the board and write either their arithmetic mean $\\frac{1}{2}(x+y)$ or their harmonic mean $\\frac{2 x y}{x+y}$. For which $(m, n)$ can Evan write 1 on the board in finitely many steps?","solution":" We claim this is possible if and only $m+n$ is a power of 2 . Let $q=m \/ n$, so the numbers on the board are $q$ and $1 \/ q$. \\I Impossibility. The main idea is the following. Claim - Suppose $p$ is an odd prime. Then if the initial numbers on the board are $-1(\\bmod p)$, then all numbers on the board are $-1(\\bmod p)$. Thus if there exists any odd prime divisor $p$ of $m+n$ (implying $p \\nmid m n$ ), then $$ q \\equiv \\frac{1}{q} \\equiv-1 \\quad(\\bmod p) . $$ and hence all numbers will be $-1(\\bmod p)$ forever. This implies that it's impossible to write 1 , whenever $m+n$ is divisible by some odd prime. \u3010 Construction. Conversely, suppose $m+n$ is a power of 2 . We will actually construct 1 without even using the harmonic mean. ![](https:\/\/cdn.mathpix.com\/cropped\/2024_11_19_dab316fe5a4e03dd61cdg-11.jpg?height=152&width=1004&top_left_y=1786&top_left_x=526) Note that $$ \\frac{n}{m+n} \\cdot q+\\frac{m}{m+n} \\cdot \\frac{1}{q}=1 $$ and obviously by taking appropriate midpoints (in a binary fashion) we can achieve this using arithmetic mean alone."} +{"year":2019,"label":"6","problem":"Find all polynomials $P$ with real coefficients such that $$ \\frac{P(x)}{y z}+\\frac{P(y)}{z x}+\\frac{P(z)}{x y}=P(x-y)+P(y-z)+P(z-x) $$ for all nonzero real numbers $x, y, z$ obeying $2 x y z=x+y+z$.","solution":" The given can be rewritten as saying that $$ \\begin{aligned} Q(x, y, z) & :=x P(x)+y P(y)+z P(z) \\\\ & -x y z(P(x-y)+P(y-z)+P(z-x)) \\end{aligned} $$ is a polynomial vanishing whenever $x y z \\neq 0$ and $2 x y z=x+y+z$, for real numbers $x, y$, $z$. Claim - This means $Q(x, y, z)$ vanishes also for any complex numbers $x, y, z$ obeying $2 x y z=x+y+z$. $$ R(x, y):=Q\\left(x, y, \\frac{x+y}{2 x y-1}\\right) $$ vanishes for any real numbers $x$ and $y$ such that $x y \\neq \\frac{1}{2}, x \\neq 0, y \\neq 0, x+y \\neq 0$. This can only occur if $R$ is identically zero as a rational function with real coefficients. If we then regard $R$ as having complex coefficients, the conclusion then follows. Remark (Algebraic geometry digression on real dimension). Note here we use in an essential way that $z$ can be solved for in terms of $x$ and $y$. If $s(x, y, z)=2 x y z-(x+y+z)$ is replaced with some general condition, the result may become false; e.g. we would certainly not expect the result to hold when $s(x, y, z)=x^{2}+y^{2}+z^{2}-(x y+y z+z x)$ since for real numbers $s=0$ only when $x=y=z$ ! The general condition we need here is that $s(x, y, z)=0$ should have \"real dimension two\". Here is a proof using this language, in our situation. Let $M \\subset \\mathbb{R}^{3}$ be the surface $s=0$. We first contend $M$ is two-dimensional manifold. Indeed, the gradient $\\nabla s=\\langle 2 y z-1,2 z x-1,2 x y-1\\rangle$ vanishes only at the points $( \\pm 1 \/ \\sqrt{2}, \\pm 1 \/ \\sqrt{2}, \\pm 1 \/ \\sqrt{2}$ ) where the $\\pm$ signs are all taken to be the same. These points do not lie on $M$, so the result follows by the regular value theorem. In particular the topological closure of points on $M$ with $x y z \\neq 0$ is all of $M$ itself; so $Q$ vanishes on all of $M$. If we now identify $M$ with the semi-algebraic set consisting of maximal ideals ( $x-a, y-$ $b, z-c)$ in Spec $\\mathbb{R}[x, y, z]$ satisfying $2 a b c=a+b+c$, then we have real dimension two, and thus the Zariski closure of $M$ is a two-dimensional closed subset of $\\operatorname{Spec} \\mathbb{R}[x, y, z]$. Thus it must be $Z=\\mathcal{V}(2 x y z-(x+y+z))$, since this $Z$ is an irreducible two-dimensional closed subset (say, by Krull's principal ideal theorem) containing $M$. Now $Q$ is a global section vanishing on all of $Z$, therefore $Q$ is contained in the (radical, principal) ideal $(2 x y z-(x+y+z))$ as needed. So it is actually divisible by $2 x y z-(x+y+z)$ as desired. Now we regard $P$ and $Q$ as complex polynomials instead. First, note that substituting $(x, y, z)=(t,-t, 0)$ implies $P$ is even. We then substitute $$ (x, y, z)=\\left(x, \\frac{i}{\\sqrt{2}}, \\frac{-i}{\\sqrt{2}}\\right) $$ to get $$ \\begin{aligned} & x P(x)+\\frac{i}{\\sqrt{2}}\\left(P\\left(\\frac{i}{\\sqrt{2}}\\right)-P\\left(\\frac{-i}{\\sqrt{2}}\\right)\\right) \\\\ = & \\frac{1}{2} x(P(x-i \/ \\sqrt{2})+P(x+i \/ \\sqrt{2})+P(\\sqrt{2} i)) \\end{aligned} $$ which in particular implies that $$ P\\left(x+\\frac{i}{\\sqrt{2}}\\right)+P\\left(x-\\frac{i}{\\sqrt{2}}\\right)-2 P(x) \\equiv P(\\sqrt{2} i) $$ identically in $x$. The left-hand side is a second-order finite difference in $x$ (up to scaling the argument), and the right-hand side is constant, so this implies $\\operatorname{deg} P \\leq 2$. Since $P$ is even and $\\operatorname{deg} P \\leq 2$, we must have $P(x)=c x^{2}+d$ for some real numbers $c$ and $d$. A quick check now gives the answer $P(x)=c\\left(x^{2}+3\\right)$ which all work."} diff --git a/USAMO/segmented/en-USAMO-2020-notes.jsonl b/USAMO/segmented/en-USAMO-2020-notes.jsonl new file mode 100644 index 0000000000000000000000000000000000000000..7725a89a48d4d96e0ac6b8dff2f9ba91b9a1c897 --- /dev/null +++ b/USAMO/segmented/en-USAMO-2020-notes.jsonl @@ -0,0 +1,9 @@ +{"year":2020,"label":"1","problem":"Let $A B C$ be a fixed acute triangle inscribed in a circle $\\omega$ with center $O$. A variable point $X$ is chosen on minor arc $A B$ of $\\omega$, and segments $C X$ and $A B$ meet at $D$. Denote by $O_{1}$ and $O_{2}$ the circumcenters of triangles $A D X$ and $B D X$, respectively. Determine all points $X$ for which the area of triangle $O O_{1} O_{2}$ is minimized.","solution":" We prove $\\left[O O_{1} O_{2}\\right] \\geq \\frac{1}{4}[A B C]$, with equality if and only if $\\overline{C X} \\perp \\overline{A B}$. \u0111 First approach (Bobby Shen). We use two simultaneous inequalities: - Let $M$ and $N$ be the midpoints of $C X$ and $D X$. Then $M N$ equals the length of the $O$-altitude of $\\triangle O O_{1} O_{2}$, since $\\overline{O_{1} O_{2}}$ and $\\overline{D X}$ meet at $N$ at a right angle. Moreover, we have $$ M N=\\frac{1}{2} C D \\geq \\frac{1}{2} h_{a} $$ where $h_{a}$ denotes the $A$-altitude. - The projection of $O_{1} O_{2}$ onto line $A B$ has length exactly $A B \/ 2$. Thus $$ O_{1} O_{2} \\geq \\frac{1}{2} A B $$ So, we find $$ \\left[O O_{1} O_{2}\\right]=\\frac{1}{2} \\cdot M N \\cdot O_{1} O_{2} \\geq \\frac{1}{8} h_{a} \\cdot A B=\\frac{1}{4}[A B C] . $$ Note that equality occurs in both cases if and only if $\\overline{C X} \\perp \\overline{A B}$. So the area is minimized exactly when this occurs."} +{"year":2020,"label":"1","problem":"Let $A B C$ be a fixed acute triangle inscribed in a circle $\\omega$ with center $O$. A variable point $X$ is chosen on minor arc $A B$ of $\\omega$, and segments $C X$ and $A B$ meet at $D$. Denote by $O_{1}$ and $O_{2}$ the circumcenters of triangles $A D X$ and $B D X$, respectively. Determine all points $X$ for which the area of triangle $O O_{1} O_{2}$ is minimized.","solution":" We prove $\\left[O O_{1} O_{2}\\right] \\geq \\frac{1}{4}[A B C]$, with equality if and only if $\\overline{C X} \\perp \\overline{A B}$. \u300e Second approach (Evan's solution). We need two claims. Claim - We have $\\triangle O O_{1} O_{2} \\sim \\triangle C B A$, with opposite orientation. Therefore, the problem is equivalent to minimizing $O_{1} O_{2}$. ![](https:\/\/cdn.mathpix.com\/cropped\/2024_11_19_2585129426fba9be1ad0g-04.jpg?height=798&width=795&top_left_y=249&top_left_x=633) Claim (Salmon theorem) - We have $\\triangle X O_{1} O_{2} \\sim \\triangle X A B$. Let $\\theta=\\angle A D X$. The ratio of similarity in the previous claim is equal to $\\frac{X O_{1}}{X A}=\\frac{1}{2 \\sin \\theta}$. In other words, $$ O_{1} O_{2}=\\frac{A B}{2 \\sin \\theta} . $$ This is minimized when $\\theta=90^{\\circ}$, in which case $O_{1} O_{2}=A B \/ 2$ and $\\left[O O_{1} O_{2}\\right]=\\frac{1}{4}[A B C]$. This completes the solution."} +{"year":2020,"label":"2","problem":"An empty $2020 \\times 2020 \\times 2020$ cube is given, and a $2020 \\times 2020$ grid of square unit cells is drawn on each of its six faces. A beam is a $1 \\times 1 \\times 2020$ rectangular prism. Several beams are placed inside the cube subject to the following conditions: - The two $1 \\times 1$ faces of each beam coincide with unit cells lying on opposite faces of the cube. (Hence, there are $3 \\cdot 2020^{2}$ possible positions for a beam.) - No two beams have intersecting interiors. - The interiors of each of the four $1 \\times 2020$ faces of each beam touch either a face of the cube or the interior of the face of another beam. What is the smallest positive number of beams that can be placed to satisfy these conditions?","solution":" \u3010 A Answer. 3030 beams. \u3010 Construction. We first give a construction with $3 n \/ 2$ beams for any $n \\times n \\times n$ box, where $n$ is an even integer. Shown below is the construction for $n=6$, which generalizes. (The left figure shows the cube in 3 d ; the right figure shows a direct view of the three visible faces.) ![](https:\/\/cdn.mathpix.com\/cropped\/2024_11_19_2585129426fba9be1ad0g-05.jpg?height=815&width=1308&top_left_y=1551&top_left_x=314) ![](https:\/\/cdn.mathpix.com\/cropped\/2024_11_19_2585129426fba9be1ad0g-05.jpg?height=295&width=281&top_left_y=2031&top_left_x=1113) Left face ![](https:\/\/cdn.mathpix.com\/cropped\/2024_11_19_2585129426fba9be1ad0g-05.jpg?height=295&width=278&top_left_y=2034&top_left_x=1460) Right face To be explicit, impose coordinate axes such that one corner of the cube is the origin. We specify a beam by two opposite corners. The $3 n \/ 2$ beams come in three directions, $n \/ 2$ in each direction: - $(0,0,0) \\rightarrow(1,1, n),(2,2,0) \\rightarrow(3,3, n),(4,4,0) \\rightarrow(5,5, n)$, and so on; - $(1,0,0) \\rightarrow(2, n, 1),(3,0,2) \\rightarrow(4, n, 3),(5,0,4) \\rightarrow(6, n, 5)$, and so on; - $(0,1,1) \\rightarrow(n, 2,2),(0,3,3) \\rightarrow(n, 4,4),(0,5,5) \\rightarrow(n, 6,6)$, and so on. This gives the figure we drew earlier and shows 3030 beams is possible. Necessity. We now show at least $3 n \/ 2$ beams are necessary. Maintain coordinates, and call the beams $x$-beams, $y$-beams, $z$-beams according to which plane their long edges are perpendicular too. Let $N_{x}, N_{y}, N_{z}$ be the number of these. Claim - If $\\min \\left(N_{x}, N_{y}, N_{z}\\right)=0$, then at least $n^{2}$ beams are needed. We henceforth assume $\\min \\left(N_{x}, N_{y}, N_{z}\\right)>0$. Claim - If $N_{z}>0$, then we have $N_{x}+N_{y} \\geq n$. Proceeding in a similar fashion, we arrive at the three relations $$ \\begin{aligned} & N_{x}+N_{y} \\geq n \\\\ & N_{y}+N_{z} \\geq n \\\\ & N_{z}+N_{x} \\geq n \\end{aligned} $$ Summing gives $N_{x}+N_{y}+N_{z} \\geq 3 n \/ 2$ too. Remark. The problem condition has the following \"physics\" interpretation. Imagine the cube is a metal box which is sturdy enough that all beams must remain orthogonal to the faces of the box (i.e. the beams cannot spin). Then the condition of the problem is exactly what is needed so that, if the box is shaken or rotated, the beams will not move. Remark. Walter Stromquist points out that the number of constructions with 3030 beams is actually enormous: not dividing out by isometries, the number is $(2 \\cdot 1010!)^{3}$."} +{"year":2020,"label":"3","problem":"Let $p$ be an odd prime. An integer $x$ is called a quadratic non-residue if $p$ does not divide $x-t^{2}$ for any integer $t$. Denote by $A$ the set of all integers $a$ such that $1 \\leq a0}$ instead."} +{"year":2021,"label":"6","problem":"Let $A B C D E F$ be a convex hexagon satisfying $\\overline{A B}\\|\\overline{D E}, \\overline{B C}\\| \\overline{E F}, \\overline{C D} \\| \\overline{F A}$, and $$ A B \\cdot D E=B C \\cdot E F=C D \\cdot F A $$ Let $X, Y$, and $Z$ be the midpoints of $\\overline{A D}, \\overline{B E}$, and $\\overline{C F}$. Prove that the circumcenter of $\\triangle A C E$, the circumcenter of $\\triangle B D F$, and the orthocenter of $\\triangle X Y Z$ are collinear.","solution":" ![](https:\/\/cdn.mathpix.com\/cropped\/2024_11_19_128c4918e9847b51d7d1g-13.jpg?height=777&width=809&top_left_y=1279&top_left_x=629) Claim - If $A B \\cdot D E=B C \\cdot E F=C D \\cdot F A=k$, then the circumcenters of $A C E$ and $A^{\\prime} C^{\\prime} E^{\\prime}$ coincide. ![](https:\/\/cdn.mathpix.com\/cropped\/2024_11_19_128c4918e9847b51d7d1g-14.jpg?height=992&width=495&top_left_y=246&top_left_x=780) Claim - Triangle $X Y Z$ is the vector average of the (congruent) medial triangles of triangles $A^{\\prime} C^{\\prime} E^{\\prime}$ and $B^{\\prime} D^{\\prime} F^{\\prime}$. $$ \\begin{aligned} \\frac{\\vec{M}+\\vec{N}}{2} & =\\frac{\\frac{\\vec{C}^{\\prime}+\\vec{E}^{\\prime}}{2}+\\frac{\\vec{B}^{\\prime}+\\vec{F}^{\\prime}}{2}}{2} \\\\ & =\\frac{\\overrightarrow{C^{\\prime}}+\\vec{E}^{\\prime}+\\vec{B}^{\\prime}+\\vec{F}^{\\prime}}{4} \\\\ & =\\frac{(\\vec{A}+\\vec{E}-\\vec{F})+(\\vec{C}+\\vec{A}-\\vec{B})+(\\vec{D}+\\vec{F}-\\vec{E})+(\\vec{B}+\\vec{D}-\\vec{C})}{4} \\\\ & =\\frac{\\vec{A}+\\vec{D}}{2}=\\vec{X} . \\end{aligned} $$ Hence the orthocenter of $X Y Z$ is the midpoint of the orthocenters of the medial triangles of $A^{\\prime} C^{\\prime} E^{\\prime}$ and $B^{\\prime} D^{\\prime} F^{\\prime}$ - that is, their circumcenters. ![](https:\/\/cdn.mathpix.com\/cropped\/2024_11_19_128c4918e9847b51d7d1g-15.jpg?height=966&width=1100&top_left_y=248&top_left_x=478) Claim - In trapezoid $A B D E$, the perpendicular bisector of $\\overline{X Y}$ is the same as the perpendicular bisector of the midline $\\overline{W N}$. Claim - The points $V, W, M, N$ are cyclic. $$ W Y \\cdot Y N=\\frac{1}{2} D E \\cdot \\frac{1}{2} A B=\\frac{1}{2} E F \\cdot \\frac{1}{2} B C=V Y \\cdot Y M $$ Applying all the cyclic variations of the above two claims, it follows that all six points $U, V, W, M, N, P$ are concyclic, and the center of that circle coincides with the circumcenter of $\\triangle X Y Z$. Remark. It is also possible to implement ideas from the first solution here, by showing all six midpoints have equal power to $(X Y Z)$. Claim - The orthocenter of $X Y Z$ is the midpoint of the circumcenters of $\\triangle A C E$ and $\\triangle B D F$. $$ \\begin{aligned} \\operatorname{orthocenter}(X Y Z) & =x+y+z=\\frac{a+b+c+d+e+f}{2} \\\\ \\operatorname{circumcenter}(A C E) & =\\operatorname{orthocenter}(M N P) \\end{aligned} $$ $$ =m+n+p=\\frac{c+e}{2}+\\frac{e+a}{2}+\\frac{a+c}{2}=a+c+e $$ circumcenter $(B D F)=$ orthocenter $(U V W)$ $$ =u+v+w=\\frac{d+f}{2}+\\frac{f+b}{2}+\\frac{b+d}{2}=b+d+f $$"} diff --git a/USAMO/segmented/en-USAMO-2022-notes.jsonl b/USAMO/segmented/en-USAMO-2022-notes.jsonl new file mode 100644 index 0000000000000000000000000000000000000000..5bee7675a1eefc9bf81f2e76697733de31c3d6cd --- /dev/null +++ b/USAMO/segmented/en-USAMO-2022-notes.jsonl @@ -0,0 +1,6 @@ +{"year":2022,"label":"1","problem":"Let $a$ and $b$ be positive integers. Every cell of an $(a+b+1) \\times(a+b+1)$ grid is colored either amber or bronze such that there are at least $a^{2}+a b-b$ amber cells and at least $b^{2}+a b-a$ bronze cells. Prove that it is possible to choose $a$ amber cells and $b$ bronze cells such that no two of the $a+b$ chosen cells lie in the same row or column.","solution":" Claim - There exists a transversal $T_{a}$ with at least $a$ amber cells. Analogously, there exists a transversal $T_{b}$ with at least $b$ bronze cells. $$ \\frac{a^{2}+a b-b}{a+b+1}=(a-1)+\\frac{1}{a+b+1}>a-1 $$ Now imagine we transform $T_{a}$ to $T_{b}$ in some number of steps, by repeatedly choosing cells $c$ and $c^{\\prime}$ and swapping them with the two other corners of the rectangle formed by their row\/column, as shown in the figure. ![](https:\/\/cdn.mathpix.com\/cropped\/2024_11_19_0824a9a7bcda8c860a08g-03.jpg?height=243&width=809&top_left_y=1466&top_left_x=629) By \"discrete intermediate value theorem\", the number of amber cells will be either $a$ or $a+1$ at some point during this transformation. This completes the proof."} +{"year":2022,"label":"2","problem":"Let $b \\geq 2$ and $w \\geq 2$ be fixed integers, and $n=b+w$. Given are $2 b$ identical black rods and $2 w$ identical white rods, each of side length 1. We assemble a regular $2 n$-gon using these rods so that parallel sides are the same color. Then, a convex $2 b$-gon $B$ is formed by translating the black rods, and a convex $2 w$-gon $W$ is formed by translating the white rods. An example of one way of doing the assembly when $b=3$ and $w=2$ is shown below, as well as the resulting polygons $B$ and $W$. ![](https:\/\/cdn.mathpix.com\/cropped\/2024_11_19_0824a9a7bcda8c860a08g-02.jpg?height=609&width=1013&top_left_y=952&top_left_x=571) Prove that the difference of the areas of $B$ and $W$ depends only on the numbers $b$ and $w$, and not on how the $2 n$-gon was assembled.","solution":" We are going to prove that one may swap a black rod with an adjacent white rod (as well as the rods parallel to them) without affecting the difference in the areas of $B-W$. Let $\\vec{u}$ and $\\vec{v}$ denote the originally black and white vectors that were adjacent on the $2 n$-gon and are now going to be swapped. Let $\\vec{x}$ denote the sum of all the other black vectors between $\\vec{u}$ and $-\\vec{u}$, and define $\\vec{y}$ similarly. See the diagram below, where $B_{0}$ and $W_{0}$ are the polygons before the swap, and $B_{1}$ and $W_{1}$ are the resulting changed polygons. ![](https:\/\/cdn.mathpix.com\/cropped\/2024_11_19_0824a9a7bcda8c860a08g-05.jpg?height=734&width=1220&top_left_y=227&top_left_x=424) Observe that the only change in $B$ and $W$ is in the parallelograms shown above in each diagram. Letting $\\wedge$ denote the wedge product, we need to show that $$ \\vec{u} \\wedge \\vec{x}-\\vec{v} \\wedge \\vec{y}=\\vec{v} \\wedge \\vec{x}-\\vec{u} \\wedge \\vec{y} $$ which can be rewritten as $$ (\\vec{u}-\\vec{v}) \\wedge(\\vec{x}+\\vec{y})=0 $$ In other words, it would suffice to show $\\vec{u}-\\vec{v}$ and $\\vec{x}+\\vec{y}$ are parallel. (Students not familiar with wedge products can replace every $\\wedge$ with the cross product $\\times$ instead.) Claim - Both $\\vec{u}-\\vec{v}$ and $\\vec{x}+\\vec{y}$ are perpendicular to vector $\\vec{u}+\\vec{v}$. For the other perpendicularity, note that $\\vec{u}+\\vec{v}+\\vec{x}+\\vec{y}$ traces out a diameter of the circumcircle of the original $2 n$-gon; call this diameter $A B$, so $$ A+\\vec{u}+\\vec{v}+\\vec{x}+\\vec{y}=B $$ Now point $A+\\vec{u}+\\vec{v}$ is a point on this semicircle, which means (by the inscribed angle theorem) the angle between $\\vec{u}+\\vec{v}$ and $\\vec{x}+\\vec{y}$ is $90^{\\circ}$."} +{"year":2022,"label":"3","problem":"Solve over positive real numbers the functional equation $$ f(x)=f(f(f(x))+y)+f(x f(y)) f(x+y) $$","solution":"$ The answer is $f(x) \\equiv c \/ x$ for any $c>0$. This works, so we'll prove this is the only solution. The following is based on the solution posted by pad on AoPS. In what follows, $f^{n}$ as usual denotes $f$ iterated $n$ times, and $P(x, y)$ is the given statement. Also, we introduce the notation $Q$ for the statement $$ Q(a, b): \\quad f(a) \\geq f(b) \\Longrightarrow f(f(b)) \\geq a . $$ To see why this statement $Q$ is true, assume for contradiction that $a>f(f(b))$; then consider $P(b, a-f(f(b)))$ to get a contradiction. The main idea of the problem is the following: Claim - Any function $f: \\mathbb{R}_{>0} \\rightarrow \\mathbb{R}_{>0}$ obeying statement $Q$ satisfies $f^{2}(x)=f^{4}(x)$. $$ f^{2}(t) \\geq t \\quad \\text { for all } t>0 $$ So this already implies $f^{4}(x) \\geq f^{2}(x)$ by choosing $t=f^{2}(x)$. It also gives $f(x) \\leq f^{3}(x) \\leq$ $f^{5}(x)$ by choosing $t=f(x), t=f^{3}(x)$. Then $Q\\left(f^{4}(x), x\\right)$ is valid and gives $f^{2}(x) \\geq f^{4}(x)$, as needed. Claim - The function $f$ is injective. $$ \\begin{aligned} & P\\left(f^{2}(x), u\\right) \\Longrightarrow f^{3}(x)=f(x+u)+f(x f(u)) f(x+u)=(1+f(x f(u))) f(x+u) \\\\ & P\\left(f^{2}(x), v\\right) \\Longrightarrow f^{3}(x)=f(x+v)+f(x f(v)) f(x+v)=(1+f(x f(v))) f(x+v) . \\end{aligned} $$ It follows that $f(x+u)=f(x+v)$ for all $x>0$. This means that $f$ is periodic with period $T=u-v>0$. However, this is incompatible with $Q$, because we would have $Q(1+n T, 1)$ for all positive integers $n$, which is obviously absurd. Since $f$ is injective, we obtain that $f^{2}(x)=x$. Thus $P(x, y)$ now becomes the statement $$ P(x, y): \\quad f(x)=f(x+y) \\cdot[1+f(x f(y))] . $$ In particular $$ P(1, y) \\Longrightarrow f(1+y)=\\frac{f(1)}{1+y} $$ so $f$ is determined on inputs greater than 1 . Finally, if $a, b>1$ we get $$ P(a, b) \\Longrightarrow \\frac{1}{a}=\\frac{1}{a+b} \\cdot\\left[1+f\\left(\\frac{a}{b} f(1)\\right)\\right] $$ which is enough to determine $f$ on all inputs, by varying $(a, b)$."} +{"year":2022,"label":"4","problem":"Find all pairs of primes $(p, q)$ for which $p-q$ and $p q-q$ are both perfect squares.","solution":" The answer is $(3,2)$ only. This obviously works so we focus on showing it is the only one. $$ \\begin{aligned} a^{2} & =p-q \\\\ b^{2} & =p q-q . \\end{aligned} $$ Note that $0N$ and choose $x_{n}=-n$ for each $n=1, \\ldots, N$. Now for each index $1 \\leq n \\leq N$, define $$ S(n)=\\left\\{\\text { indices } i \\text { for which } f_{i}(n) \\neq 0\\right\\} \\subseteq\\{1, \\ldots, k\\} $$ As each $S(n t)$ is nonempty, by pigeonhole, two $S(n)$ 's coincide, say $S(n)=S\\left(n^{\\prime}\\right)$ for $nx_{n^{\\prime}}$ in that case due to the essentially increasing condition. \u3010 Construction. It suffices to do $N=2^{k}-1$. Rather than drown the reader in notation, we'll just illustrate an example of the (inductive) construction for $k=4$. Empty cells are zero. | | $f_{1}$ | $f_{2}$ | $f_{3}$ | $f_{4}$ | | ---: | ---: | ---: | ---: | ---: | | $x_{1}=3$ | 3 | | | | | $x_{2}=1$ | 10 | -9 | | | | $x_{3}=4$ | | 4 | | | | $x_{4}=1$ | 100 | 200 | $-\\mathbf{2 9 9}$ | | | $x_{5}=5$ | | 200 | -195 | | | $x_{6}=9$ | 100 | | -91 | | | $x_{7}=2$ | | | 2 | | | $x_{8}=6$ | 1000 | 2000 | 4000 | $-\\mathbf{6 9 9 4}$ | | $x_{9}=5$ | | 2000 | 4000 | -5995 | | $x_{10}=3$ | 1000 | | 4000 | -4997 | | $x_{11}=5$ | | | 4000 | -3995 | | $x_{12}=8$ | 1000 | 2000 | | -2992 | | $x_{13}=9$ | | 2000 | | -1991 | | $x_{14}=7$ | 1000 | | | -993 | | $x_{15}=9$ | | | | 9 | The general case is handled in the same way with powers of 10 replaced by powers of $B$, for a sufficiently large number $B$."} +{"year":2022,"label":"6","problem":"There are 2022 users on a social network called Mathbook, and some of them are Mathbook-friends. (On Mathbook, friendship is always mutual and permanent.) Starting now, Mathbook will only allow a new friendship to be formed between two users if they have at least two friends in common. What is the minimum number of friendships that must already exist so that every user could eventually become friends with every other user?","solution":" With 2022 replaced by $n$, the answer is $\\left\\lceil\\frac{3}{2} n\\right\\rceil-2$. \u3010 Terminology. Standard graph theory terms: starting from a graph $G$ on $n$ vertices, we're allowed to take any $C_{4}$ in the graph and complete it to a $K_{4}$. The problem asks the minimum number of edges needed so that this operation lets us transform $G$ to $K_{n}$. \u3010T Construction. For even $n$, start with an edge $a b$, and then create $n \/ 2-1$ copies of $C_{4}$ that use $a b$ as an edge, as shown below for $n=14$ (six copies of $C_{4}$ ). ![](https:\/\/cdn.mathpix.com\/cropped\/2024_11_19_0824a9a7bcda8c860a08g-10.jpg?height=563&width=818&top_left_y=1306&top_left_x=625) This can be completed into $K_{n}$ by first completing the $n \/ 2-1 C_{4}$ 's into $K_{4}$, then connecting red vertices to every grey vertex, and then finishing up. The construction for odd $n$ is the same except with one extra vertex $c$ which is connected to both $a$ and $b$. \u3010 Bound. Notice that additional operations or connections can never hurt. So we will describe a specific algorithm that performs operations on the graph until no more operations are possible. This means that if this algorithm terminates with anything other $G=K_{n}$, the graph was never completable to $K_{n}$ to begin with. The algorithm uses the following data: it keeps a list $\\mathcal{C}$ of cliques of $G$, and a labeling $\\mathcal{L}: E(G) \\rightarrow \\mathcal{C}$ which assigns to every edge one of the cliques that contains it. - Initially, $\\mathcal{C}$ consists of one $K_{2}$ for every edge of $G$, and each edge is labeled in the obvious way. - At each step, the algorithm arbitrarily takes any $C_{4}=a b c d$ whose four edges $a b$, $b c, c d, d a$ do not all have the same label. Consider these labels that appear (at least two, and up to four), and let $V$ be the union of all vertices in any of these 2-4 cliques. - Do the following graph operations: connect $a c$ and $b d$, then connect every vertex in $V-\\{a, b, c, d\\}$ to each of $\\{a, b, c, d\\}$. Finally, complete this to a clique on $V$. - Update $\\mathcal{C}$ by merging these 2-4 cliques into a single clique $K_{V}$. - Update $\\mathcal{L}$ by replacing every edge that was labeled with one of these 2-4 cliques with the label $K_{V}$. Also, update every newly created edge to have label $K_{V}$. However, if there were existing edges not labeled with one of the 2-4 cliques, then we do not update these! - Stop once every $C_{4}$ has only one label appearing among its edges. When this occurs, no operations are possible at all on the graph. A few steps of the process are illustrated below for a graph on six vertices with nine initial edges. There are initially nine $K_{2}$ 's labeled A, B, ..., I. Original edges are always bolder than added edges. The relabeled edges in each step are highlighted in color. Notice how we need an entirely separate operation to get $G$ to become L, even though no new edges are drawn in the graph. ![](https:\/\/cdn.mathpix.com\/cropped\/2024_11_19_0824a9a7bcda8c860a08g-11.jpg?height=912&width=1223&top_left_y=1257&top_left_x=428) Step 2: Operate on 1235. Merges CIJ into K. $\\theta(\\mathrm{K})=6$ Step 3: Operate on 2356. Merges GK into L. $\\theta(\\mathrm{L})=7$ As we remarked, if the graph is going to be completable to $K_{n}$ at all, then this algorithm must terminate with $\\mathcal{C}=\\left\\{K_{n}\\right\\}$. We will use this to prove our bound. We proceed by induction in the following way. For a clique $K$, let $\\theta(K)$ denote the number of edges of the original graph $G$ which are labeled by $K$ (this does not include new edges added by the algorithm); hence the problem amounts to estimating how small $\\theta\\left(K_{n}\\right)$ can be. We are trying to prove: Claim - At any point in the operation, if $K$ is a clique in the cover $\\mathcal{C}$, then $$ \\theta(K) \\geq \\frac{3|K|}{2}-2 . $$ where $|K|$ is the number of vertices in $K$. The inductive step is annoying casework based on the how the merge occurred. Let $C_{4}=a b c d$ be the 4 -cycle operated on. In general, the $\\theta$ value of a newly created $K$ is exactly the sum of the $\\theta$ values of the merged cliques, by definition. Meanwhile, $|K|$ is the number of vertices in the union of the merged cliques; so it's the sum of the sizes of these cliques minus some error due to overcounting of vertices appearing more than once. To be explicit: - Suppose we merged four cliques $W, X, Y, Z$. By definition, $$ \\begin{aligned} \\theta(K) & =\\theta(W)+\\theta(X)+\\theta(Y)+\\theta(Z) \\\\ & \\geq \\frac{3}{2}(|W|+|X|+|Y|+|Z|)-8=\\frac{3}{2}(|W|+|X|+|Y|+|Z|-4)-2 . \\end{aligned} $$ On the other hand $|K| \\leq|W|+|X|+|Y|+|Z|-4$; the -4 term comes from each of $\\{a, b, c, d\\}$ being in two (or more) of $\\{W, X, Y, Z\\}$. So this case is OK. - Suppose we merged three cliques $X, Y, Z$. By definition, $$ \\begin{aligned} \\theta(K) & =\\theta(X)+\\theta(Y)+\\theta(Z) \\\\ & \\geq \\frac{3}{2}(|X|+|Y|+|Z|)-6=\\frac{3}{2}\\left(|X|+|Y|+|Z|-\\frac{8}{3}\\right)-2 . \\end{aligned} $$ On the other hand, $|K| \\leq|X|+|Y|+|Z|-3$, since at least 3 of $\\{a, b, c, d\\}$ are repeated among $X, Y, Z$. Note in this case the desired inequality is actually strict. - Suppose we merged two cliques $Y, Z$. By definition, $$ \\begin{aligned} \\theta(K) & =\\theta(Y)+\\theta(Z) \\\\ & \\geq \\frac{3}{2}(|Y|+|Z|)-4=\\frac{3}{2}\\left(|Y|+|Z|-\\frac{4}{3}\\right)-2 . \\end{aligned} $$ On the other hand, $|K| \\leq|Y|+|Z|-2$, since at least 2 of $\\{a, b, c, d\\}$ are repeated among $Y, Z$. Note in this case the desired inequality is actually strict. Remark. Several subtle variations of this method do not seem to work. - It does not seem possible to require the cliques in $\\mathcal{C}$ to be disjoint, which is why it's necessary to introduce a label function $\\mathcal{L}$ as well. - It seems you do have to label the newly created edges, even though they do not count towards any $\\theta$ value. Otherwise the termination of the algorithm doesn't tell you enough. - Despite this, relabeling existing edges, like G in step 1 of the example, 1 seems to cause a lot of issues. The induction becomes convoluted if $\\theta(K)$ is not exactly the sum of $\\theta$-values of the subparts, while the disappearance of an edge from a clique will \u3011 also break induction."} diff --git a/USAMO/segmented/en-USAMO-2023-notes.jsonl b/USAMO/segmented/en-USAMO-2023-notes.jsonl new file mode 100644 index 0000000000000000000000000000000000000000..45620ef03f49c7335283a6e4ab7a4df000acec14 --- /dev/null +++ b/USAMO/segmented/en-USAMO-2023-notes.jsonl @@ -0,0 +1,10 @@ +{"year":2023,"label":"1","problem":"In an acute triangle $A B C$, let $M$ be the midpoint of $\\overline{B C}$. Let $P$ be the foot of the perpendicular from $C$ to $A M$. Suppose that the circumcircle of triangle $A B P$ intersects line $B C$ at two distinct points $B$ and $Q$. Let $N$ be the midpoint of $\\overline{A Q}$. Prove that $N B=N C$.","solution":" ![](https:\/\/cdn.mathpix.com\/cropped\/2024_11_19_5f0e796e43a11586ccbfg-03.jpg?height=892&width=995&top_left_y=999&top_left_x=536) \u3010 Most common synthetic approach. The solution hinges on the following claim: Claim - $Q$ coincides with the reflection of $D$ across $M$. $$ M B \\cdot M Q=M A \\cdot M P=M C \\cdot M D $$ Since $M B=M C$, the claim follows. It follows that $\\overline{M N} \\| \\overline{A D}$, as $M$ and $N$ are respectively the midpoints of $\\overline{A Q}$ and $\\overline{D Q}$. Thus $\\overline{M N} \\perp \\overline{B C}$, and so $N$ lies on the perpendicular bisector of $\\overline{B C}$, as needed. Remark (David Lin). One can prove the main claim without power of a point as well, as follows: Let $R$ be the foot from $B$ to $\\overline{A M}$, so $B R C P$ is a parallelogram. Note that $A B D R$ is cyclic, and hence $$ \\measuredangle D R M=\\measuredangle D B A=Q B A=\\measuredangle Q P A=\\measuredangle Q P M $$ Thus, $\\overline{D R} \\| \\overline{P Q}$, so $D R Q P$ is also a parallelogram."} +{"year":2023,"label":"1","problem":"In an acute triangle $A B C$, let $M$ be the midpoint of $\\overline{B C}$. Let $P$ be the foot of the perpendicular from $C$ to $A M$. Suppose that the circumcircle of triangle $A B P$ intersects line $B C$ at two distinct points $B$ and $Q$. Let $N$ be the midpoint of $\\overline{A Q}$. Prove that $N B=N C$.","solution":" ![](https:\/\/cdn.mathpix.com\/cropped\/2024_11_19_5f0e796e43a11586ccbfg-03.jpg?height=892&width=995&top_left_y=999&top_left_x=536) ## \u092c Synthetic approach with no additional points at all. ## Claim - $\\triangle B P C \\sim \\triangle A N M$ (oppositely oriented). $$ \\frac{B M}{B P}=\\frac{A M}{A Q} \\Longrightarrow \\frac{2 B M}{B P}=\\frac{A M}{A Q \/ 2} \\Longrightarrow \\frac{B C}{B P}=\\frac{A M}{A N} $$ implying the similarity (since $\\measuredangle M A Q=\\measuredangle B P M$ ). This similarity gives us the equality of directed angles $$ \\measuredangle(B C, M N)=-\\measuredangle(P C, A M)=90^{\\circ} $$ as desired."} +{"year":2023,"label":"1","problem":"In an acute triangle $A B C$, let $M$ be the midpoint of $\\overline{B C}$. Let $P$ be the foot of the perpendicular from $C$ to $A M$. Suppose that the circumcircle of triangle $A B P$ intersects line $B C$ at two distinct points $B$ and $Q$. Let $N$ be the midpoint of $\\overline{A Q}$. Prove that $N B=N C$.","solution":" ![](https:\/\/cdn.mathpix.com\/cropped\/2024_11_19_5f0e796e43a11586ccbfg-03.jpg?height=892&width=995&top_left_y=999&top_left_x=536) \u092c Synthetic approach using only the point $R$. Again let $R$ be the foot from $B$ to $\\overline{A M}$, so $B R C P$ is a parallelogram. Claim - $A R Q C$ is cyclic; equivalently, $\\triangle M A Q \\sim \\triangle M C R$. Note that in $\\triangle M C R$, the $M$-median is parallel to $\\overline{C P}$ and hence perpendicular to $\\overline{R M}$. The same should be true in $\\triangle M A Q$ by the similarity, so $\\overline{M N} \\perp \\overline{M Q}$ as needed. $$ \\begin{aligned} \\overleftrightarrow{A M}: 0 & =b x-a y \\Longleftrightarrow y=\\frac{b}{a} x \\\\ \\overleftrightarrow{C P}: 0 & =a(x-1)+b y \\Longleftrightarrow y=-\\frac{a}{b}(x-1)=-\\frac{a}{b} x+\\frac{a}{b} \\\\ P & =\\left(\\frac{a^{2}}{a^{2}+b^{2}}, \\frac{a b}{a^{2}+b^{2}}\\right) \\end{aligned} $$ Now note that $$ A M=\\sqrt{a^{2}+b^{2}}, \\quad P M=\\frac{a}{\\sqrt{a^{2}+b^{2}}} $$ together with power of a point $$ A M \\cdot P M=B M \\cdot Q M $$ to immediately deduce that $Q=(a, 0)$. Hence $N=(0, b \/ 2)$ and we're done."} +{"year":2023,"label":"1","problem":"In an acute triangle $A B C$, let $M$ be the midpoint of $\\overline{B C}$. Let $P$ be the foot of the perpendicular from $C$ to $A M$. Suppose that the circumcircle of triangle $A B P$ intersects line $B C$ at two distinct points $B$ and $Q$. Let $N$ be the midpoint of $\\overline{A Q}$. Prove that $N B=N C$.","solution":" ![](https:\/\/cdn.mathpix.com\/cropped\/2024_11_19_5f0e796e43a11586ccbfg-03.jpg?height=892&width=995&top_left_y=999&top_left_x=536) \u3010 Cartesian coordinates approach without power of a point (outline). After computing $A$ and $P$ as above, one could also directly calculate $$ \\begin{aligned} & \\text { Perpendicular bisector of } \\overline{A B}: y=-\\frac{a+1}{b} x+\\frac{a^{2}+b^{2}-1}{2 b} \\\\ & \\text { Perpendicular bisector of } \\overline{P B}: y=-\\left(\\frac{2 a}{b}+\\frac{b}{a}\\right) x-\\frac{b}{2 a} \\\\ & \\text { Perpendicular bisector of } \\overline{P A}: y=-\\frac{a}{b} x+\\frac{a+a^{2}+b^{2}}{2 b} \\\\ & \\text { Circumcenter of } \\triangle P A B=\\left(-\\frac{a+1}{2}, \\frac{2 a^{2}+2 a+b^{2}}{2 b}\\right) \\end{aligned} $$ This is enough to extract the coordinates of $Q=(\\bullet, 0)$, because $B=(-1,0)$ is given, and the $x$-coordinate of the circumcenter should be the average of the $x$-coordinates of $B$ and $Q$. In other words, $Q=(-a, 0)$. Hence, $N=\\left(0, \\frac{b}{2}\\right)$, as needed."} +{"year":2023,"label":"1","problem":"In an acute triangle $A B C$, let $M$ be the midpoint of $\\overline{B C}$. Let $P$ be the foot of the perpendicular from $C$ to $A M$. Suppose that the circumcircle of triangle $A B P$ intersects line $B C$ at two distinct points $B$ and $Q$. Let $N$ be the midpoint of $\\overline{A Q}$. Prove that $N B=N C$.","solution":" ![](https:\/\/cdn.mathpix.com\/cropped\/2024_11_19_5f0e796e43a11586ccbfg-03.jpg?height=892&width=995&top_left_y=999&top_left_x=536) \\I III-advised barycentric approach (outline). Use reference triangle $A B C$. The $A$ median is parametrized by $(t: 1: 1)$ for $t \\in \\mathbb{R}$. So because of $\\overline{C P} \\perp \\overline{A M}$, we are looking for $t$ such that $$ \\left(\\frac{t \\vec{A}+\\vec{B}+\\vec{C}}{t+2}-\\vec{C}\\right) \\perp\\left(A-\\frac{\\vec{B}+\\vec{C}}{2}\\right) $$ This is equivalent to $$ (t \\vec{A}+\\vec{B}-(t+1) \\vec{C}) \\perp(2 \\vec{A}-\\vec{B}-\\vec{C}) $$ By the perpendicularity formula for barycentric coordinates (EGMO 7.16), this is equivalent to $$ \\begin{aligned} 0 & =a^{2} t-b^{2} \\cdot(3 t+2)+c^{2} \\cdot(2-t) \\\\ & =\\left(a^{2}-3 b^{2}-c^{2}\\right) t-2\\left(b^{2}-c^{2}\\right) \\\\ \\Longrightarrow t & =\\frac{2\\left(b^{2}-c^{2}\\right)}{a^{2}-3 b^{2}-c^{2}} \\end{aligned} $$ In other words, $$ P=\\left(2\\left(b^{2}-c^{2}\\right): a^{2}-3 b^{2}-c^{2}: a^{2}-3 b^{2}-c^{2}\\right) . $$ A long calculation gives $a^{2} y_{P} z_{P}+b^{2} z_{P} x_{P}+c^{2} x_{P} y_{P}=\\left(a^{2}-3 b^{2}-c^{2}\\right)\\left(a^{2}-b^{2}+c^{2}\\right)\\left(a^{2}-\\right.$ $\\left.2 b^{2}-2 c^{2}\\right)$. Together with $x_{P}+y_{P}+z_{P}=2 a^{2}-4 b^{2}-4 c^{2}$, this makes the equation of $(A B P)$ as $$ 0=-a^{2} y z-b^{2} z x-c^{2} x y+\\frac{a^{2}-b^{2}+c^{2}}{2} z(x+y+z) $$ To solve for $Q$, set $x=0$ to get to get $$ a^{2} y z=\\frac{a^{2}-b^{2}+c^{2}}{2} z(y+z) \\Longrightarrow \\frac{y}{z}=\\frac{a^{2}-b^{2}+c^{2}}{a^{2}+b^{2}-c^{2}} $$ In other words, $$ Q=\\left(0: a^{2}-b^{2}+c^{2}: a^{2}+b^{2}-c^{2}\\right) $$ Taking the average with $A=(1,0,0)$ then gives $$ N=\\left(2 a^{2}: a^{2}-b^{2}+c^{2}: a^{2}+b^{2}-c^{2}\\right) . $$ The equation for the perpendicular bisector of $\\overline{B C}$ is given by (see EGMO 7.19) $$ 0=a^{2}(z-y)+x\\left(c^{2}-b^{2}\\right) $$ which contains $N$, as needed. $$ p=\\frac{(a-m) \\bar{c}+(\\bar{a}-\\bar{m}) c+\\bar{a} m-a \\bar{m}}{2(\\bar{a}-\\bar{m})}=\\frac{a^{2} b-a^{2} c-a b^{2}-2 a b c-a c^{2}+b^{2} c+3 b c^{2}}{4 b c-2 a(b+c)} $$ Meanwhile, an extremely ugly calculation will eventually yield $$ q=\\frac{\\frac{b c}{a}+b+c-a}{2} $$ SO $$ n=\\frac{a+q}{2}=\\frac{a+b+c+\\frac{b c}{a}}{4}=\\frac{(a+b)(a+c)}{2 a} $$ There are a few ways to then verify $N B=N C$. The simplest seems to be to verify that $$ \\frac{n-\\frac{b+c}{2}}{b-c}=\\frac{a-b-c+\\frac{b c}{a}}{4(b-c)}=\\frac{(a-b)(a-c)}{2 a(b-c)} $$ is pure imaginary, which is clear."} +{"year":2023,"label":"2","problem":"Solve over the positive real numbers the functional equation $$ f(x y+f(x))=x f(y)+2 $$","solution":"$ The answer is $f(x) \\equiv x+1$, which is easily verified to be the only linear solution. We show conversely that $f$ is linear. Let $P(x, y)$ be the assertion. Claim $-f$ is weakly increasing. Claim (Up to an error of $2, f$ is linear) - We have $$ |f(x)-(K x+C)| \\leq 2 $$ where $K:=\\frac{2}{f(1)}$ and $C:=f(f(1))-2$ are constants. $$ 2\\left\\lfloor\\frac{x}{f(1)}\\right\\rfloor+C \\leq f(x) \\leq 2\\left\\lceil\\frac{x}{f(1)}\\right\\rceil+C $$ which implies the result. Rewrite the previous claim to the simpler $f(x)=K x+O(1)$. Then for any $x$ and $y$, the above claim gives $$ K(x y+K x+O(1))+O(1)=x f(y)+2 $$ which means that $$ x \\cdot\\left(K y+K^{2}-f(y)\\right)=O(1) . $$ If we fix $y$ and consider large $x$, we see this can only happen if $K y+K^{2}-f(y)=0$, i.e. $f$ is linear."} +{"year":2023,"label":"3","problem":"Consider an $n$-by- $n$ board of unit squares for some odd positive integer $n$. We say that a collection $C$ of identical dominoes is a maximal grid-aligned configuration on the board if $C$ consists of $\\left(n^{2}-1\\right) \/ 2$ dominoes where each domino covers exactly two neighboring squares and the dominoes don't overlap: $C$ then covers all but one square on the board. We are allowed to slide (but not rotate) a domino on the board to cover the uncovered square, resulting in a new maximal gridaligned configuration with another square uncovered. Let $k(C)$ be the number of distinct maximal grid-aligned configurations obtainable from $C$ by repeatedly sliding dominoes. Find all possible values of $k(C)$ as a function of $n$.","solution":" The answer is that $$ k(C) \\in\\left\\{1,2, \\ldots,\\left(\\frac{n-1}{2}\\right)^{2}\\right\\} \\cup\\left\\{\\left(\\frac{n+1}{2}\\right)^{2}\\right\\} $$ Index the squares by coordinates $(x, y) \\in\\{1,2, \\ldots, n\\}^{2}$. We say a square is special if it is empty or it has the same parity in both coordinates as the empty square. We now proceed in two cases: \u092c The special squares have both odd coordinates. We construct a directed graph $G=G(C)$ whose vertices are special squares as follows: for each domino on a special square $s$, we draw a directed edge from $s$ to the special square that domino points to. Thus all special squares have an outgoing edge except the empty cell. ![](https:\/\/cdn.mathpix.com\/cropped\/2024_11_19_5f0e796e43a11586ccbfg-08.jpg?height=612&width=609&top_left_y=1664&top_left_x=729) Claim - Any undirected connected component of $G$ is acyclic unless the cycle contains the empty square inside it. This can be proven directly by induction, but for theatrical effect, we use Pick's theorem. Mark the center of every chessboard cell on or inside the cycle to get a lattice. The dominoes of the cycle then enclose a polyominoe which actually consists of $2 \\times 2$ squares, meaning its area is a multiple of 4. ![](https:\/\/cdn.mathpix.com\/cropped\/2024_11_19_5f0e796e43a11586ccbfg-09.jpg?height=466&width=466&top_left_y=458&top_left_x=795) Hence $B \/ 2+I-1$ is a multiple of 4 , in the notation of Pick's theorem. As $B$ is twice the number of dominoes, and a parity argument on the special squares shows that number is even, it follows that $B$ is also a multiple of 4 (these correspond to blue and black in the figure above). This means $I$ is odd (the red dots in the figure above), as desired. Let $T$ be the connected component containing the empty cell. By the claim, $T$ is acyclic, so it's a tree. Now, notice that all the arrows point along $T$ towards the empty cell, and moving a domino corresponds to flipping an arrow. Therefore: Claim $-k(C)$ is exactly the number of vertices of $T$. This implies that $k(C) \\leq\\left(\\frac{n+1}{2}\\right)^{2}$ in this case. Equality is achieved if $T$ is a spanning tree of $G$. One example of a way to achieve this is using the snake configuration below. ![](https:\/\/cdn.mathpix.com\/cropped\/2024_11_19_5f0e796e43a11586ccbfg-09.jpg?height=461&width=466&top_left_y=1711&top_left_x=795) Remark. In Russia 1997\/11.8 it's shown that as long as the missing square is a corner, we have $G=T$. The proof is given implicitly from our work here: when the empty cell is in a corner, it cannot be surrounded, ergo the resulting graph has no cycles at all. And since the overall graph has one fewer edge than vertex, it's a tree. Conversely, suppose $T$ was not a spanning tree, i.e. $T \\neq G$. Since in this odd-odd case, $G$ has one fewer edge than vertex, if $G$ is not a tree, then it must contain at least one cycle. That cycle encloses every special square of $T$. In particular, this means that $T$ can't contain any special squares from the outermost row or column of the $n \\times n$ grid. In this situation, we therefore have $k(C) \\leq\\left(\\frac{n-3}{2}\\right)^{2}$. \u3010 The special squares have both even coordinates. We construct the analogous graph $G$ on the same special squares. However, in this case, some of the points may not have outgoing edges, because their domino may \"point\" outside the grid. ![](https:\/\/cdn.mathpix.com\/cropped\/2024_11_19_5f0e796e43a11586ccbfg-10.jpg?height=806&width=803&top_left_y=582&top_left_x=632) As before, the connected component $T$ containing the empty square is a tree, and $k(C)$ is exactly the number of vertices of $T$. Thus to finish the problem we need to give, for each $k \\in\\left\\{1,2, \\ldots,\\left(\\frac{n-1}{2}\\right)^{2}\\right\\}$, an example of a configuration where $G$ has exactly $k$ vertices. The construction starts with a \"snake\" picture for $k=\\left(\\frac{n-1}{2}\\right)^{2}$, then decreases $k$ by one by perturbing a suitable set of dominoes. Rather than write out the procedure in words, we show the sequence of nine pictures for $n=7$ (where $k=9,8, \\ldots, 1$ ); the generalization to larger $n$ is straightforward. ![](https:\/\/cdn.mathpix.com\/cropped\/2024_11_19_5f0e796e43a11586ccbfg-11.jpg?height=1383&width=1380&top_left_y=241&top_left_x=339)"} +{"year":2023,"label":"4","problem":"Positive integers $a$ and $N$ are fixed, and $N$ positive integers are written on a blackboard. Alice and Bob play the following game. On Alice's turn, she must replace some integer $n$ on the board with $n+a$, and on Bob's turn he must replace some even integer $n$ on the board with $n \/ 2$. Alice goes first and they alternate turns. If on his turn Bob has no valid moves, the game ends. After analyzing the $N$ integers on the board, Bob realizes that, regardless of what moves Alice makes, he will be able to force the game to end eventually. Show that, in fact, for this value of $a$ and these $N$ integers on the board, the game is guaranteed to end regardless of Alice's or Bob's moves.","solution":" For $N=1$, there is nothing to prove. We address $N \\geq 2$ only henceforth. Let $S$ denote the numbers on the board. Claim - When $N \\geq 2$, if $\\nu_{2}(x)<\\nu_{2}(a)$ for all $x \\in S$, the game must terminate no matter what either player does. Hence, in fact the game will always terminate in exactly $\\sum_{x \\in S} \\nu_{2}(x)$ moves in this case, regardless of what either player does. Claim - When $N \\geq 2$, if there exists a number $x$ on the board such that $\\nu_{2}(x) \\geq$ $\\nu_{2}(a)$, then Alice can cause the game to go on forever. - Operate on the first entry if $\\nu_{2}(x)=\\nu_{2}(a)$ (the new entry thus has $\\nu_{2}(x+a)>\\nu_{2}(a)$ ); - Operate on any other entry besides the first one, otherwise. A double induction then shows that - Just before each of Bob's turns, $\\nu_{2}(x)>\\nu_{2}(a)$ always holds; and - After each of Bob's turns, $\\nu_{2}(x) \\geq \\nu_{2}(a)$ always holds. In particular Bob will never run out of legal moves, since halving $x$ is always legal."} +{"year":2023,"label":"5","problem":"Let $n \\geq 3$ be an integer. We say that an arrangement of the numbers $1,2, \\ldots, n^{2}$ in an $n \\times n$ table is row-valid if the numbers in each row can be permuted to form an arithmetic progression, and column-valid if the numbers in each column can be permuted to form an arithmetic progression. For what values of $n$ is it possible to transform any row-valid arrangement into a column-valid arrangement by permuting the numbers in each row?","solution":" Answer: $n$ prime only. So, look at the multiples of $p$ in a row-valid table; there is either 1 or $p$ per row. As there are $p$ such numbers total, there are two cases: - If all the multiples of $p$ are in the same row, then the common difference in each row is a multiple of $p$. In fact, it must be exactly $p$ for size reasons. In other words, up to permutation the rows are just the $k(\\bmod p)$ numbers in some order, and this is obviously column-valid because we can now permute such that the $k$ th column contains exactly $\\{(k-1) p+1,(k-1) p+2, \\ldots, k p\\}$. - If all the multiples of $p$ are in different rows, then it follows each row contains every residue modulo $p$ exactly once. So we can permute to a column-valid arrangement by ensuring the $k$ th column contains all the $k(\\bmod p)$ numbers. \u3010 Counterexample for $n$ composite (due to Anton Trygub). Let $p$ be any prime divisor of $n$. Construct the table as follows: - Row 1 contains 1 through $n$. - Rows 2 through $p+1$ contain the numbers from $p+1$ to $n p+n$ partitioned into arithmetic progressions with common difference $p$. - The rest of the rows contain the remaining numbers in reading order. For example, when $p=2$ and $n=10$, we get the following table: $\\left[\\begin{array}{cccccccccc}1 & 2 & 3 & 4 & 5 & 6 & 7 & 8 & 9 & 10 \\\\ \\mathbf{1 1} & 13 & 15 & 17 & 19 & 21 & 23 & 25 & 27 & 29 \\\\ 12 & 14 & 16 & 18 & 20 & 22 & 24 & 26 & 28 & 30 \\\\ 31 & 32 & 33 & 34 & 35 & 36 & 37 & 38 & 39 & 40 \\\\ 41 & 42 & 43 & 44 & 45 & 46 & 47 & 48 & 49 & 50 \\\\ 51 & 52 & 53 & 54 & 55 & 56 & 57 & 58 & 59 & 60 \\\\ 61 & 62 & 63 & 64 & 65 & 66 & 67 & 68 & 69 & 70 \\\\ 71 & 72 & 73 & 74 & 75 & 76 & 77 & 78 & 79 & 80 \\\\ 81 & 82 & 83 & 84 & 85 & 86 & 87 & 88 & 89 & 90 \\\\ 91 & 92 & 93 & 94 & 95 & 96 & 97 & 98 & 99 & 100\\end{array}\\right]$ We claim this works fine. Assume for contradiction the rows may be permuted to obtain a column-valid arrangement. Then the $n$ columns should be arithmetic progressions whose smallest element is in $[1, n]$ and whose largest element is in $\\left[n^{2}-n+1, n^{2}\\right]$. These two elements must be congruent modulo $n-1$, so in particular the column containing 2 must end with $n^{2}-n+2$. Hence in that column, the common difference must in fact be exactly $n$. And yet $n+2$ and $2 n+2$ are in the same row, contradiction."} +{"year":2023,"label":"6","problem":"Let $A B C$ be a triangle with incenter $I$ and excenters $I_{a}, I_{b}, I_{c}$ opposite $A, B$, and $C$, respectively. Given an arbitrary point $D$ on the circumcircle of $\\triangle A B C$ that does not lie on any of the lines $I I_{a}, I_{b} I_{c}$, or $B C$, suppose the circumcircles of $\\triangle D I I_{a}$ and $\\triangle D I_{b} I_{c}$ intersect at two distinct points $D$ and $F$. If $E$ is the intersection of lines $D F$ and $B C$, prove that $\\angle B A D=\\angle E A C$.","solution":" ![](https:\/\/cdn.mathpix.com\/cropped\/2024_11_19_5f0e796e43a11586ccbfg-15.jpg?height=920&width=763&top_left_y=896&top_left_x=658) \\I Barycentric coordinates (Carl Schildkraut). With reference triangle $\\triangle A B C$, set $D=(r: s: t)$. Claim - The equations of $\\left(D I I_{a}\\right)$ and $\\left(D I_{b} I_{c}\\right)$ are, respectively, $$ \\begin{aligned} & 0=-a^{2} y z-b^{2} z x-c^{2} x y+(x+y+z) \\cdot\\left(b c x-\\frac{b c r}{c s-b t}(c y-b z)\\right) \\\\ & 0=-a^{2} y z-b^{2} z x-c^{2} x y+(x+y+z) \\cdot\\left(-b c x+\\frac{b c r}{c s+b t}(c y+b z)\\right) . \\end{aligned} $$ By EGMO Lemma 7.24, the radical axis is then given by $$ \\overline{D F}: b c x-\\frac{b c r}{c s-b t}(c y-b z)=-b c x+\\frac{b c r}{c s+b t}(c y+b z) . $$ Now the point $$ \\left(0: \\frac{b^{2}}{s}: \\frac{c^{2}}{t}\\right)=\\left(0: b^{2} t: c^{2} s\\right) $$ lies on line $D F$ by inspection, and is obviously on line $B C$, hence it coincides with $E$. This lies on the isogonal of $\\overline{A D}$ (by EGMO Lemma 7.6), as needed. \u300e Synthetic approach (Anant Mudgal). Focus on just $\\left(D I I_{a}\\right)$. Let $P$ be the second intersection of $\\left(D I I_{a}\\right)$ with $(A B C)$, and let $M$ be the midpoint of minor arc $\\widehat{B C}$. Then by radical axis, lines $A M, D P$, and $B C$ are concurrent at a point $K$. Let $E^{\\prime}=\\overline{P M} \\cap \\overline{B C}$. ![](https:\/\/cdn.mathpix.com\/cropped\/2024_11_19_5f0e796e43a11586ccbfg-16.jpg?height=864&width=901&top_left_y=776&top_left_x=583) Claim - We have $\\measuredangle B A D=\\measuredangle E^{\\prime} A C$. $$ \\measuredangle K A E^{\\prime}=\\measuredangle K P E^{\\prime}=\\measuredangle D P M=\\measuredangle D A M $$ Claim - The power of point $E^{\\prime}$ with respect to $\\left(D I I_{a}\\right)$ is $2 E^{\\prime} B \\cdot E^{\\prime} C$. $$ \\measuredangle X I_{a} I=\\measuredangle I_{a} I E^{\\prime}=\\measuredangle M I E^{\\prime}=\\measuredangle M P I=\\measuredangle X P I $$ Hence $X$ lies on $\\left(D I I_{a}\\right)$, and $E^{\\prime} X \\cdot E^{\\prime} P=2 E^{\\prime} M \\cdot E^{\\prime} P=2 E^{\\prime} B \\cdot E^{\\prime} C$. Repeat the argument on $\\left(D I_{b} I_{c}\\right)$; the same point $E^{\\prime}$ (because of the first claim) then has power $2 E^{\\prime} B \\cdot E^{\\prime} C$ with respect to $\\left(D I_{b} I_{c}\\right)$. Hence $E^{\\prime}$ lies on the radical axis of $\\left(D I I_{a}\\right)$ and $\\left(D I_{b} I_{c}\\right)$, ergo $E^{\\prime}=E$. The first claim then solves the problem."} diff --git a/USAMO/segmented/en-USAMO-2024-notes.jsonl b/USAMO/segmented/en-USAMO-2024-notes.jsonl new file mode 100644 index 0000000000000000000000000000000000000000..2e98b5e8858b02c1d06014a558bf0fcd9eb94d3e --- /dev/null +++ b/USAMO/segmented/en-USAMO-2024-notes.jsonl @@ -0,0 +1,7 @@ +{"year":2024,"label":"1","problem":"Find all integers $n \\geq 3$ such that the following property holds: if we list the divisors of $n$ ! in increasing order as $1=d_{1}24-20$. - For $n=6$ we have $18-15>20-18$. - For $7 \\leq n \\leq 12$ we have because $14-12>15-14$ (and $13 \\nmid n!$ ). Now assume $n \\geq 13$. In that case, we have $$ \\left\\lfloor\\frac{n}{2}\\right\\rfloor^{2}-1 \\geq 2 n $$ So by Bertrand postulate, we can find a prime $p$ such that $$ n0$. And the objective function is to minimize the quantity $$ A:=\\sum_{|v| \\geq 50} f(v) . $$ So the problem is transformed into an system of equations over $\\mathbb{Z}_{\\geq 0}$ (it's clear any assignment of values of $f(v)$ can be translated to a sequence ( $S_{1}, \\ldots, S_{100}$ ) in the original notation). Note already that: Claim - It suffices to assign $f(v)$ for $|v| \\geq 50$. I Construction. Consider the construction $$ f_{0}(v)=2|v|-100 $$ This construction is valid since if $|u|=100-k$ for $k \\leq 50$ then $$ \\begin{aligned} \\sum_{v \\supseteq u} f_{0}(v) & =\\binom{k}{0} \\cdot 100+\\binom{k}{1} \\cdot 98+\\binom{k}{2} \\cdot 96+\\cdots+\\binom{k}{k} \\cdot(100-2 k) \\\\ & =(100-k) \\cdot 2^{k}=|u| \\cdot 2^{k} \\end{aligned} $$ is indeed a multiple of $|u|$, hence $P(u)$ is true. In that case, the objective function is $$ A=\\sum_{i=50}^{100}\\binom{100}{i}(2 i-100)=50\\binom{100}{50} $$ as needed. Remark. This construction is the \"easy\" half of the problem because it coincides with what you get from a greedy algorithm by downwards induction on $|u|$ (equivalently, induction on $k=100-|u| \\geq 0)$. To spell out the first three steps, - We know $f(1 \\ldots 1)$ is a nonzero multiple of 100 , so it makes sense to guess $f(1 \\ldots 1)=$ 100. - Then we have $f(1 \\ldots 10)+100 \\equiv 0(\\bmod 99)$, and the smallest multiple of 99 which is at least 100 is 198 . So it makes sense to guess $f(1 \\ldots 10)=98$, and similarly guess $f(v)=98$ whenever $|v|=99$. - Now when we consider, say $v=1 \\ldots 100$ with $|v|=98$, we get $$ f(1 \\ldots 100)+\\underbrace{f(1 \\ldots 101)}_{=98}+\\underbrace{f(1 \\ldots 110)}_{=98}+\\underbrace{f(1 \\ldots 111)}_{=100} \\equiv 0 \\quad(\\bmod 98) $$ we obtain $f(1 \\ldots 100) \\equiv 96(\\bmod 98)$. That makes $f(1 \\ldots 100)=96$ a reasonable guess. Continuing in this way gives the construction above. We define a push-down on $v$ as the following operation: - Pick any $v$ such that $|v| \\geq 50$ and $f(v) \\geq|v|$. - Decrease $f(v)$ by $|v|$. - For every $w$ such that $w \\subseteq v$ and $|w|=|v|-1$, increase $f(w)$ by 1 . Claim - Apply a push-down preserves the main divisibility condition. Moreover, it doesn't change $A$ unless $|v|=50$, where it decreases $A$ by 50 instead. To see $A$ doesn't change for $|v|>50$, note $|v|$ terms increase by 1 while one term decreases by $-|v|$. When $|v|=50$, only $f(v)$ decreases by 50 . Now, given a valid assignment, we can modify it as follows: - First apply pushdowns on $1 \\ldots 1$ until $f(1 \\ldots 1)=100$; - Then we may apply pushdowns on each $v$ with $|v|=99$ until $f(v)<99$; - Then we may apply pushdowns on each $v$ with $|v|=98$ until $f(v)<98$; - . . .and so on, until we have $f(v)<50$ for $|v|=50$. Hence we get $f(1 \\ldots 1)=100$ and $0 \\leq f(v)<|v|$ for all $50 \\leq|v| \\leq 100$. However, by downwards induction on $|v|=99,98, \\ldots, 50$, we also have $$ f(v) \\equiv f_{0}(v) \\quad(\\bmod |v|) \\Longrightarrow f(v)=f_{0}(v) $$ since $f_{0}(v)$ and $f(v)$ are both strictly less than $|v|$. So in fact $f=f_{0}$, and we're done. Remark. The fact that push-downs actually don't change $A$ shows that the equality case we described is far from unique: in fact, we could have made nearly arbitrary sub-optimal decisions during the greedy algorithm and still ended up with an equality case. For a concrete example, the construction $$ f(v)= \\begin{cases}500 & |v|=100 \\\\ 94 & |v|=99 \\\\ 100-2|v| & 50 \\leq|v| \\leq 98\\end{cases} $$ works fine as well (where we arbitrarily chose 500 at the start, then used the greedy algorithm thereafter)."} +{"year":2024,"label":"3","problem":"Let $(m, n)$ be positive integers with $n \\geq 3$ and draw a regular $n$-gon. We wish to triangulate this $n$-gon into $n-2$ triangles, each colored one of $m$ colors, so that each color has an equal sum of areas. For which $(m, n)$ is such a triangulation and coloring possible?","solution":" The answer is if and only if $m$ is a proper divisor of $n$. ## Lemma The triangle with vertices $\\omega^{k}, \\omega^{k+a}, \\omega^{k+b}$ has signed area $$ T(a, b):=\\frac{\\left(\\omega^{a}-1\\right)\\left(\\omega^{b}-1\\right)\\left(\\omega^{-a}-\\omega^{-b}\\right)}{2 i} . $$ $$ \\frac{1}{2 i} \\operatorname{det}\\left[\\begin{array}{ccc} 1 & 1 & 1 \\\\ \\omega^{a} & \\omega^{-a} & 1 \\\\ \\omega^{b} & \\omega^{-b} & 1 \\end{array}\\right]=\\frac{1}{2 i} \\operatorname{det}\\left[\\begin{array}{ccc} 0 & 0 & 1 \\\\ \\omega^{a}-1 & \\omega^{-a}-1 & 1 \\\\ \\omega^{b}-1 & \\omega^{-b}-1 & 1 \\end{array}\\right] $$ which equals the above. \u3010 Construction. It suffices to actually just take all the diagonals from the vertex 1 , and then color the triangles with the $m$ colors in cyclic order. For example, when $n=9$ and $m=3$, a coloring with red, green, blue would be: ![](https:\/\/cdn.mathpix.com\/cropped\/2024_11_19_bff0e8c9a0bc19d4709dg-07.jpg?height=617&width=615&top_left_y=1873&top_left_x=726) To see this works one can just do the shoelace calculation: for a given residue $r \\bmod m$, we get an area $$ \\begin{aligned} \\sum_{j \\equiv r \\bmod m} \\operatorname{Area}\\left(\\omega^{j}, \\omega^{0}, \\omega^{j+1}\\right) & =\\sum_{j \\equiv r \\bmod m} T(-j, 1) \\\\ & =\\sum_{j \\equiv r \\bmod m} \\frac{\\left(\\omega^{-j}-1\\right)\\left(\\omega^{1}-1\\right)\\left(\\omega^{j}-\\omega^{-1}\\right)}{2 i} \\\\ & =\\frac{\\omega-1}{2 i} \\sum_{j \\equiv r \\bmod m}\\left(\\omega^{-j}-1\\right)\\left(\\omega^{j}-\\omega^{-1}\\right) \\\\ & =\\frac{\\omega-1}{2 i}\\left(\\frac{n}{m}\\left(1+\\omega^{-1}\\right)+\\sum_{j \\equiv r \\bmod m}\\left(\\omega^{-j}-\\omega^{j}\\right)\\right) . \\end{aligned} $$ (We allow degenerate triangles where $j \\in\\{-1,0\\}$ with area zero.) However, if $m$ is a proper divisor of $m$, then $\\sum_{j \\equiv r \\bmod m} \\omega^{j}=\\omega^{r}\\left(1+\\omega^{m}+\\omega^{2 m}+\\cdots+\\omega^{n-m}\\right)=0$. Similarly, $\\sum_{j \\equiv r \\bmod m} \\omega^{-j}=0$. So the inner sum vanishes, and the total area of the $m$ th color equals $$ \\frac{n}{m} \\frac{(\\omega-1)\\left(\\omega^{-1}+1\\right)}{2 i} $$ which does not depend on the residue $r$, proving the coloring works. Repeating the same calculation as above, we find that if there was a valid triangulation and coloring, the total area of each color would equal $$ S:=\\frac{n}{m} \\frac{(\\omega-1)\\left(\\omega^{-1}+1\\right)}{2 i} . $$ ## However: Claim \u2014 The number $2 i \\cdot S$ is not an algebraic integer when $m \\nmid n$. However, each of the quantities $T(a, b)$ is $\\frac{1}{2 i}$ times an algebraic integer. Since a finite sum of algebraic integers is also an algebraic integer, such areas can never sum to $S$. Remark. If one wants to avoid citing the fact that $\\mathcal{O}_{K}=\\mathbb{Z}[\\omega]$, then one can instead note that $T(a, b)$ is actually always divisible by $(\\omega-1)\\left(\\omega^{-1}+1\\right)$ over the algebraic integers (at least one of $\\left\\{\\omega^{a}-1, \\omega^{b}-1, \\omega^{-a}-\\omega^{-b}\\right\\}$ is a multiple of $\\omega+1$, by casework on $a, b \\bmod 2$ ). Then one using $\\frac{2 i}{(\\omega-1)\\left(\\omega^{-1}+1\\right)}$ as the scaling factor instead of $2 i$, one sees that we actually need $\\frac{n}{m}$ to be an algebraic integer, which happens only when $m$ divides $n$."} +{"year":2024,"label":"4","problem":"Let $m$ and $n$ be positive integers. A circular necklace contains $m n$ beads, each either red or blue. It turned out that no matter how the necklace was cut into $m$ blocks of $n$ consecutive beads, each block had a distinct number of red beads. Determine, with proof, all possible values of the ordered pair $(m, n)$.","solution":" The answer is $m \\leq n+1$ only. Construction when $m=n+1$. For concreteness, here is the construction for $n=4$, which obviously generalizes. The beads are listed in reading order as an array with $n+1$ rows and $n$ columns. Four of the blue beads have been labeled $B_{1}, \\ldots, B_{n}$ to make them easier to track as they move. $$ T_{0}=\\left[\\begin{array}{llll} R & R & R & R \\\\ R & R & R & B_{1} \\\\ R & R & B & B_{2} \\\\ R & B & B & B_{3} \\\\ B & B & B & B_{4} \\end{array}\\right] $$ To prove this construction works, it suffices to consider the $n$ cuts $T_{0}, T_{1}, T_{2}, \\ldots, T_{n-1}$ made where $T_{i}$ differs from $T_{i-1}$ by having the cuts one bead later also have the property each row has a distinct red count: $$ T_{1}=\\left[\\begin{array}{llll} R & R & R & R \\\\ R & R & B_{1} & R \\\\ R & B & B_{2} & R \\\\ B & B & B_{3} & B \\\\ B & B & B_{4} & R \\end{array}\\right] \\quad T_{2}=\\left[\\begin{array}{cccc} R & R & R & R \\\\ R & B_{1} & R & R \\\\ B & B_{2} & R & B \\\\ B & B_{3} & B & B \\\\ B & B_{4} & R & R \\end{array}\\right] \\quad T_{3}=\\left[\\begin{array}{cccc} R & R & R & R \\\\ B_{1} & R & R & B \\\\ B_{2} & R & B & B \\\\ B_{3} & B & B & B \\\\ B_{4} & R & R & R \\end{array}\\right] $$ We can construct a table showing for each $1 \\leq k \\leq n+1$ the number of red beads which are in the $(k+1)$ st row of $T_{i}$ from the bottom: | $k$ | $T_{0}$ | $T_{1}$ | $T_{2}$ | $T_{3}$ | | :---: | :---: | :---: | :---: | :---: | | $k=4$ | 4 | 4 | 4 | 4 | | $k=3$ | 3 | 3 | 3 | 2 | | $k=2$ | 2 | 2 | 1 | 1 | | $k=1$ | 1 | 0 | 0 | 0 | | $k=0$ | 0 | 1 | 2 | 3 |. This suggests following explicit formula for the entry of the $(i, k)$ th cell which can then be checked straightforwardly: $$ \\#\\left(\\text { red cells in } k \\text { th row of } T_{i}\\right)= \\begin{cases}k & k>i \\\\ k-1 & i \\geq k>0 \\\\ i & k=0\\end{cases} $$ And one can see for each $i$, the counts are all distinct (they are ( $i, 0,1, \\ldots, k-1, k+1, \\ldots, k)$ from bottom to top). This completes the construction. Construction when $m2$ be an integer and let $\\ell \\in\\{1,2, \\ldots, n\\}$. A collection $A_{1}, \\ldots, A_{k}$ of (not necessarily distinct) subsets of $\\{1,2, \\ldots, n\\}$ is called $\\ell$-large if $\\left|A_{i}\\right| \\geq \\ell$ for all $1 \\leq i \\leq k$. Find, in terms of $n$ and $\\ell$, the largest real number $c$ such that the inequality $$ \\sum_{i=1}^{k} \\sum_{j=1}^{k} x_{i} x_{j} \\frac{\\left|A_{i} \\cap A_{j}\\right|^{2}}{\\left|A_{i}\\right| \\cdot\\left|A_{j}\\right|} \\geq c\\left(\\sum_{i=1}^{k} x_{i}\\right)^{2} $$ holds for all positive integer $k$, all nonnegative real numbers $x_{1}, x_{2}, \\ldots, x_{k}$, and all $\\ell$-large collections $A_{1}, A_{2}, \\ldots, A_{k}$ of subsets of $\\{1,2, \\ldots, n\\}$.","solution":" The answer turns out to be $$ c=\\frac{n+\\ell^{2}-2 \\ell}{n(n-1)} $$ \u3010 Rewriting as a dot product. For $i=1, \\ldots, n$ define $\\mathbf{v}_{i}$ by $$ \\mathbf{v}_{i}[p, q]:=\\left\\{\\begin{array}{ll} \\frac{1}{\\left|A_{i}\\right|} & p \\in A_{i} \\text { and } q \\in A_{i} \\\\ 0 & \\text { otherwise; } \\end{array} \\quad \\mathbf{v}:=\\sum_{i} x_{i} \\mathbf{v}_{i}\\right. $$ Then $$ \\begin{aligned} \\sum_{i} \\sum_{j} x_{i} x_{j} \\frac{\\left|A_{i} \\cap A_{j}\\right|^{2}}{\\left|A_{i}\\right|\\left|A_{j}\\right|} & =\\sum_{i} \\sum_{j} x_{i} x_{j}\\left\\langle\\mathbf{v}_{i}, \\mathbf{v}_{j}\\right\\rangle \\\\ & =\\left\\langle\\sum_{i} x_{i} \\mathbf{v}_{i}, \\sum_{j} x_{i} \\mathbf{v}_{i}\\right\\rangle=\\left\\|\\sum_{i} x_{i} \\mathbf{v}_{i}\\right\\|^{2}=\\|\\mathbf{v}\\|^{2} . \\end{aligned} $$ $$ \\begin{aligned} & \\langle\\mathbf{e}, \\mathbf{v}\\rangle=\\sum_{i} x_{i}\\left\\langle\\mathbf{e}, \\mathbf{v}_{i}\\right\\rangle=\\sum_{i} x_{i} \\\\ & \\langle\\mathbf{1}, \\mathbf{v}\\rangle=\\sum_{i} x_{i}\\left\\langle\\mathbf{1}, \\mathbf{v}_{i}\\right\\rangle=\\sum_{i} x_{i}\\left|A_{i}\\right| . \\end{aligned} $$ That means for any positive real constants $\\alpha$ and $\\beta$, by Cauchy-Schwarz for vectors, we should have $$ \\begin{aligned} \\|\\alpha \\mathbf{e}+\\beta \\mathbf{1}\\|\\|\\mathbf{v}\\| & \\geq\\langle\\alpha \\mathbf{e}+\\beta \\mathbf{1}, \\mathbf{v}\\rangle=\\alpha\\langle\\mathbf{e}, \\mathbf{v}\\rangle+\\beta\\langle\\mathbf{1}, \\mathbf{v}\\rangle \\\\ & =\\alpha \\cdot \\sum x_{i}+\\beta \\cdot \\sum x_{i}\\left|A_{i}\\right| \\\\ & \\geq(\\alpha+\\ell \\beta) \\sum x_{i} . \\end{aligned} $$ Set $\\mathbf{w}:=\\alpha \\mathbf{e}+\\beta \\mathbf{1}$ for brevity. Then $$ \\mathbf{w}[p, q]= \\begin{cases}\\alpha+\\beta & \\text { if } p=q \\\\ \\beta & \\text { if } p \\neq q\\end{cases} $$ SO $$ \\|\\mathbf{w}\\|=\\sqrt{n \\cdot(\\alpha+\\beta)^{2}+\\left(n^{2}-n\\right) \\cdot \\beta^{2}} $$ Therefore, we get an lower bound $$ \\frac{\\|\\mathbf{v}\\|}{\\sum x_{i}} \\geq \\frac{\\alpha+\\ell \\beta}{\\sqrt{n \\cdot(\\alpha+\\beta)^{2}+\\left(n^{2}-n\\right) \\cdot \\beta^{2}}} $$ Letting $\\alpha=n-\\ell$ and $\\beta=\\ell-1$ gives a proof that the constant $$ c=\\frac{((n-\\ell)+\\ell(\\ell-1))^{2}}{n \\cdot(n-1)^{2}+\\left(n^{2}-n\\right) \\cdot(\\ell-1)^{2}}=\\frac{\\left(n+\\ell^{2}-2 \\ell\\right)^{2}}{n(n-1)\\left(n+\\ell^{2}-2 \\ell\\right)}=\\frac{n+\\ell^{2}-2 \\ell}{n(n-1)} $$ makes the original inequality always true. (The choice of $\\alpha: \\beta$ is suggested by the example below.) \u3010 Example showing this $c$ is best possible. Let $k=\\binom{n}{\\ell}$, let $A_{i}$ run over all $\\binom{n}{\\ell}$ subsets of $\\{1, \\ldots, n\\}$ of size $\\ell$, and let $x_{i}=1$ for all $i$. We claim this construction works. To verify this, it would be sufficient to show that $\\mathbf{w}$ and $\\mathbf{v}$ are scalar multiples, so that the above Cauchy-Schwarz is equality. However, we can compute $$ \\mathbf{w}[p, q]=\\left\\{\\begin{array}{ll} n-1 & \\text { if } p=q \\\\ \\ell-1 & \\text { if } p \\neq q \\end{array}, \\quad \\mathbf{v}[p, q]= \\begin{cases}\\binom{n-1}{\\ell-1} \\cdot \\frac{1}{\\ell} & \\text { if } p=q \\\\ \\binom{n-2}{\\ell-2} \\cdot \\frac{1}{\\ell} & \\text { if } p \\neq q\\end{cases}\\right. $$ which are indeed scalar multiples, finishing the proof."} diff --git a/USA_TST/md/en-sols-TST-IMO-2014.md b/USA_TST/md/en-sols-TST-IMO-2014.md new file mode 100644 index 0000000000000000000000000000000000000000..882df83545a1376a8f459bf0e10e98ceef3831f7 --- /dev/null +++ b/USA_TST/md/en-sols-TST-IMO-2014.md @@ -0,0 +1,271 @@ +# USA TST 2014 Solution Notes + +Evan ChEn《陳誼廷》 + +15 April 2024 + +This is a compilation of solutions for the 2014 USA TST. The ideas of the solution are a mix of my own work, the solutions provided by the competition organizers, and solutions found by the community. However, all the writing is maintained by me. + +These notes will tend to be a bit more advanced and terse than the "official" solutions from the organizers. In particular, if a theorem or technique is not known to beginners but is still considered "standard", then I often prefer to use this theory anyways, rather than try to work around or conceal it. For example, in geometry problems I typically use directed angles without further comment, rather than awkwardly work around configuration issues. Similarly, sentences like "let $\mathbb{R}$ denote the set of real numbers" are typically omitted entirely. + +Corrections and comments are welcome! + +## Contents + +0 Problems ..... 2 +1 Solutions to Day 1 ..... 3 +1.1 USA TST 2014/1 ..... 3 +1.2 USA TST 2014/2, proposed by Victor Wang ..... 4 +1.3 USA TST 2014/3 ..... 6 +2 Solutions to Day 2 ..... 8 +2.1 USA TST 2014/4 ..... 8 +2.2 USA TST 2014/5, proposed by Po-Shen Loh ..... 9 +2.3 USA TST 2014/6 ..... 10 + +## §0 Problems + +1. Let $A B C$ be an acute triangle, and let $X$ be a variable interior point on the minor $\operatorname{arc} B C$ of its circumcircle. Let $P$ and $Q$ be the feet of the perpendiculars from $X$ to lines $C A$ and $C B$, respectively. Let $R$ be the intersection of line $P Q$ and the perpendicular from $B$ to $A C$. Let $\ell$ be the line through $P$ parallel to $X R$. Prove that as $X$ varies along minor arc $B C$, the line $\ell$ always passes through a fixed point. +2. Let $a_{1}, a_{2}, a_{3}, \ldots$ be a sequence of integers, with the property that every consecutive group of $a_{i}$ 's averages to a perfect square. More precisely, for all positive integers $n$ and $k$, the quantity + +$$ +\frac{a_{n}+a_{n+1}+\cdots+a_{n+k-1}}{k} +$$ + +is always the square of an integer. Prove that the sequence must be constant (all $a_{i}$ are equal to the same perfect square). +3. Let $n$ be an even positive integer, and let $G$ be an $n$-vertex (simple) graph with exactly $\frac{n^{2}}{4}$ edges. An unordered pair of distinct vertices $\{x, y\}$ is said to be amicable if they have a common neighbor (there is a vertex $z$ such that $x z$ and $y z$ are both edges). Prove that $G$ has at least $2\binom{n / 2}{2}$ pairs of vertices which are amicable. +4. Let $n$ be a positive even integer, and let $c_{1}, c_{2}, \ldots, c_{n-1}$ be real numbers satisfying + +$$ +\sum_{i=1}^{n-1}\left|c_{i}-1\right|<1 +$$ + +Prove that + +$$ +2 x^{n}-c_{n-1} x^{n-1}+c_{n-2} x^{n-2}-\cdots-c_{1} x^{1}+2 +$$ + +has no real roots. +5. Let $A B C D$ be a cyclic quadrilateral, and let $E, F, G$, and $H$ be the midpoints of $A B, B C, C D$, and $D A$ respectively. Let $W, X, Y$ and $Z$ be the orthocenters of triangles $A H E, B E F, C F G$ and $D G H$, respectively. Prove that the quadrilaterals $A B C D$ and $W X Y Z$ have the same area. +6. For a prime $p$, a subset $S$ of residues modulo $p$ is called a sum-free multiplicative subgroup of $\mathbb{F}_{p}$ if + +- there is a nonzero residue $\alpha$ modulo $p$ such that $S=\left\{1, \alpha^{1}, \alpha^{2}, \ldots\right\}$ (all considered $\bmod p$ ), and +- there are no $a, b, c \in S$ (not necessarily distinct) such that $a+b \equiv c(\bmod p)$. Prove that for every integer $N$, there is a prime $p$ and a sum-free multiplicative subgroup $S$ of $\mathbb{F}_{p}$ such that $|S| \geq N$. + + +## §1 Solutions to Day 1 + +## §1.1 USA TST 2014/1 + +Available online at https://aops.com/community/p3332310. + +## Problem statement + +Let $A B C$ be an acute triangle, and let $X$ be a variable interior point on the minor arc $B C$ of its circumcircle. Let $P$ and $Q$ be the feet of the perpendiculars from $X$ to lines $C A$ and $C B$, respectively. Let $R$ be the intersection of line $P Q$ and the perpendicular from $B$ to $A C$. Let $\ell$ be the line through $P$ parallel to $X R$. Prove that as $X$ varies along minor arc $B C$, the line $\ell$ always passes through a fixed point. + +The fixed point is the orthocenter, since $\ell$ is a Simson line. See Lemma 4.4 of Euclidean Geometry in Math Olympiads. + +## §1.2 USA TST 2014/2, proposed by Victor Wang + +Available online at https://aops.com/community/p3332299. + +## Problem statement + +Let $a_{1}, a_{2}, a_{3}, \ldots$ be a sequence of integers, with the property that every consecutive group of $a_{i}$ 's averages to a perfect square. More precisely, for all positive integers $n$ and $k$, the quantity + +$$ +\frac{a_{n}+a_{n+1}+\cdots+a_{n+k-1}}{k} +$$ + +is always the square of an integer. Prove that the sequence must be constant (all $a_{i}$ are equal to the same perfect square). + +Let $\nu_{p}(n)$ denote the largest exponent of $p$ dividing $n$. The problem follows from the following proposition. + +## Proposition + +Let $\left(a_{n}\right)$ be a sequence of integers and let $p$ be a prime. Suppose that every consecutive group of $a_{i}$ 's with length at most $p$ averages to a perfect square. Then $\nu_{p}\left(a_{i}\right)$ is independent of $i$. + +We proceed by induction on the smallest value of $\nu_{p}\left(a_{i}\right)$ as $i$ ranges (which must be even, as each of the $a_{i}$ are themselves a square). First we prove two claims. + +Claim - If $j \equiv k(\bmod p)$ then $a_{j} \equiv a_{k}(\bmod p)$. +Proof. Taking groups of length $p$ in our given, we find that $p \mid a_{j}+\cdots+a_{j+p-1}$ and $p \mid a_{j+1}+\cdots+a_{j+p}$ for any $j$. So $a_{j} \equiv a_{j+p}(\bmod p)$ and the conclusion follows. + +Claim - If some $a_{i}$ is divisible by $p$ then all of them are. + +Proof. The case $p=2$ is trivial so assume $p \geq 3$. Without loss of generality (via shifting indices) assume that $a_{1} \equiv 0(\bmod p)$, and define + +$$ +S_{n}=a_{1}+a_{2}+\cdots+a_{n} \equiv a_{2}+\cdots+a_{n} \quad(\bmod p) +$$ + +Call an integer $k$ with $2 \leq k0$, the result is vacuous for $x \leq 0$, so we restrict attention to $x>0$. + +Then letting $c_{i}=1-d_{i}$ for each $i$, the inequality we want to prove becomes + +$$ +x^{n}+1+\frac{x^{n+1}+1}{x+1}>\sum_{1}^{n-1} d_{i} x^{i} \quad \text { given } \sum\left|d_{i}\right|<1 +$$ + +But obviously $x^{n}+1>x^{i}$ for any $1 \leq i \leq n-1$ and $x>0$. So in fact $x^{n}+1>\sum_{1}^{n-1}\left|d_{i}\right| x^{i}$ holds for $x>0$, as needed. + +## §2.2 USA TST 2014/5, proposed by Po-Shen Loh + +Available online at https://aops.com/community/p3476291. + +## Problem statement + +Let $A B C D$ be a cyclic quadrilateral, and let $E, F, G$, and $H$ be the midpoints of $A B, B C, C D$, and $D A$ respectively. Let $W, X, Y$ and $Z$ be the orthocenters of triangles $A H E, B E F, C F G$ and $D G H$, respectively. Prove that the quadrilaterals $A B C D$ and $W X Y Z$ have the same area. + +The following solution is due to Grace Wang. +We begin with: +Claim - Point $W$ has coordinates $\frac{1}{2}(2 a+b+d)$. +Proof. The orthocenter of $\triangle D A B$ is $d+a+b$, and $\triangle A H E$ is homothetic to $\triangle D A B$ through $A$ with ratio $1 / 2$. Hence $w=\frac{1}{2}(a+(d+a+b))$ as needed. + +By symmetry, we have + +$$ +\begin{aligned} +w & =\frac{1}{2}(2 a+b+d) \\ +x & =\frac{1}{2}(2 b+c+a) \\ +y & =\frac{1}{2}(2 c+d+b) \\ +z & =\frac{1}{2}(2 d+a+c) . +\end{aligned} +$$ + +We see that $w-y=a-c, x-z=b-d$. So the diagonals of $W X Y Z$ have the same length as those of $A B C D$ as well as the same directed angle between them. This implies the areas are equal, too. + +## §2.3 USA TST 2014/6 + +Available online at https://aops.com/community/p3476292. + +## Problem statement + +For a prime $p$, a subset $S$ of residues modulo $p$ is called a sum-free multiplicative subgroup of $\mathbb{F}_{p}$ if + +- there is a nonzero residue $\alpha$ modulo $p$ such that $S=\left\{1, \alpha^{1}, \alpha^{2}, \ldots\right\}$ (all considered $\bmod p$ ), and +- there are no $a, b, c \in S($ not necessarily distinct $)$ such that $a+b \equiv c(\bmod p)$. Prove that for every integer $N$, there is a prime $p$ and a sum-free multiplicative subgroup $S$ of $\mathbb{F}_{p}$ such that $|S| \geq N$. + +We first prove the following general lemma. + +## Lemma + +If $f, g \in \mathbb{Z}[X]$ are relatively prime nonconstant polynomials, then for sufficiently large primes $p$, they have no common root modulo $p$. + +Proof. By Bézout Lemma, there exist polynomials $a(X)$ and $b(X)$ in $\mathbb{Z}[X]$ and a nonzero constant $c \in \mathbb{Z}$ satisfying the identity + +$$ +a(X) f(X)+b(X) g(X) \equiv c +$$ + +So, plugging in $X=r$ we get $p \mid c$, so the set of permissible primes $p$ is finite. +With this we can give the construction. + +## Claim - Suppose that + +- $n$ is a positive integer with $n \not \equiv 0(\bmod 3)$; +- $p$ is a prime which is $1 \bmod n$; and +- $\alpha$ is a primitive $n^{\prime}$ th root of unity modulo $p$. + +Then $|S|=n$ and, if $p$ is sufficiently large in $n$, is also sum-free. +Proof. The assertion $|S|=n$ is immediate from the choice of $\alpha$. As for sum-free, assume for contradiction that + +$$ +1+\alpha^{k} \equiv \alpha^{m} \quad(\bmod p) +$$ + +for some integers $k, m \in \mathbb{Z}$. This means $(X+1)^{n}-1$ and $X^{n}-1$ have common root $X=\alpha^{k}$. + +But + +$$ +\underset{\mathbb{Z}[x]}{\operatorname{gcd}}\left((X+1)^{n}-1, X^{n}-1\right)=1 \quad \forall n \not \equiv 0 \quad(\bmod 3) +$$ + +because when $3 \nmid n$ the two polynomials have no common complex roots. (Indeed, if $|\omega|=|1+\omega|=1$ then $\omega=-\frac{1}{2} \pm \frac{\sqrt{3}}{2} i$.) + +Thus $p$ is bounded by the lemma, as desired. + diff --git a/USA_TST/md/en-sols-TST-IMO-2015.md b/USA_TST/md/en-sols-TST-IMO-2015.md new file mode 100644 index 0000000000000000000000000000000000000000..5f83757a5138c93380d7893e7a9306e6adc3fa98 --- /dev/null +++ b/USA_TST/md/en-sols-TST-IMO-2015.md @@ -0,0 +1,359 @@ +# USA TST 2015 Solution Notes + +Evan ChEn《陳誼廷》 + +15 April 2024 + +This is a compilation of solutions for the 2015 USA TST. The ideas of the solution are a mix of my own work, the solutions provided by the competition organizers, and solutions found by the community. However, all the writing is maintained by me. + +These notes will tend to be a bit more advanced and terse than the "official" solutions from the organizers. In particular, if a theorem or technique is not known to beginners but is still considered "standard", then I often prefer to use this theory anyways, rather than try to work around or conceal it. For example, in geometry problems I typically use directed angles without further comment, rather than awkwardly work around configuration issues. Similarly, sentences like "let $\mathbb{R}$ denote the set of real numbers" are typically omitted entirely. + +Corrections and comments are welcome! + +## Contents + +0 Problems ..... 2 +1 Solutions to Day 1 ..... 3 +1.1 USA TST 2015/1, proposed by Evan Chen ..... 3 +1.2 USA TST 2015/2, proposed by Iurie Boreico ..... 5 +1.3 USA TST 2015/3, proposed by Linus Hamilton ..... 6 +2 Solutions to Day 2 ..... 7 +2.1 USA TST 2015/4, proposed by Victor Wang ..... 7 +2.2 USA TST 2015/5, proposed by Po-Shen Loh ..... 8 +2.3 USA TST 2015/6 ..... 9 + +## §0 Problems + +1. Let $A B C$ be a scalene triangle with incenter $I$ whose incircle is tangent to $\overline{B C}$, $\overline{C A}, \overline{A B}$ at $D, E, F$, respectively. Denote by $M$ the midpoint of $\overline{B C}$ and let $P$ be a point in the interior of $\triangle A B C$ so that $M D=M P$ and $\angle P A B=\angle P A C$. Let $Q$ be a point on the incircle such that $\angle A Q D=90^{\circ}$. Prove that either $\angle P Q E=90^{\circ}$ or $\angle P Q F=90^{\circ}$. +2. Prove that for every positive integer $n$, there exists a set $S$ of $n$ positive integers such that for any two distinct $a, b \in S, a-b$ divides $a$ and $b$ but none of the other elements of $S$. +3. A physicist encounters 2015 atoms called usamons. Each usamon either has one electron or zero electrons, and the physicist can't tell the difference. The physicist's only tool is a diode. The physicist may connect the diode from any usamon $A$ to any other usamon $B$. (This connection is directed.) When she does so, if usamon $A$ has an electron and usamon $B$ does not, then the electron jumps from $A$ to $B$. In any other case, nothing happens. In addition, the physicist cannot tell whether an electron jumps during any given step. The physicist's goal is to isolate two usamons that she is $100 \%$ sure are currently in the same state. Is there any series of diode usage that makes this possible? +4. Let $f: \mathbb{Q} \rightarrow \mathbb{Q}$ be a function such that for any $x, y \in \mathbb{Q}$, the number $f(x+y)-$ $f(x)-f(y)$ is an integer. Decide whether there must exist a constant $c$ such that $f(x)-c x$ is an integer for every rational number $x$. +5. Fix a positive integer $n$. A tournament on $n$ vertices has all its edges colored by $\chi$ colors, so that any two directed edges $u \rightarrow v$ and $v \rightarrow w$ have different colors. Over all possible tournaments on $n$ vertices, determine the minimum possible value of $\chi$. +6. Let $A B C$ be a non-equilateral triangle and let $M_{a}, M_{b}, M_{c}$ be the midpoints of the sides $B C, C A, A B$, respectively. Let $S$ be a point lying on the Euler line. Denote by $X, Y, Z$ the second intersections of $M_{a} S, M_{b} S, M_{c} S$ with the nine-point circle. Prove that $A X, B Y, C Z$ are concurrent. + +## §1 Solutions to Day 1 + +## §1.1 USA TST 2015/1, proposed by Evan Chen + +Available online at https://aops.com/community/p3683109. + +## Problem statement + +Let $A B C$ be a scalene triangle with incenter $I$ whose incircle is tangent to $\overline{B C}, \overline{C A}$, $\overline{A B}$ at $D, E, F$, respectively. Denote by $M$ the midpoint of $\overline{B C}$ and let $P$ be a point in the interior of $\triangle A B C$ so that $M D=M P$ and $\angle P A B=\angle P A C$. Let $Q$ be a point on the incircle such that $\angle A Q D=90^{\circ}$. Prove that either $\angle P Q E=90^{\circ}$ or $\angle P Q F=90^{\circ}$. + +We present two solutions. +\ Official solution. Assume without loss of generality that $A Bj$ the operation never does anything. The conclusion follows from this. + +Remark. This problem is not a "standard" olympiad problem, so I can't say it's trivial. But the idea is pretty natural I think. + +You can motivate it as follows: there's a sequence of diode operations you can do which forces the situation to be one of the $M_{k}$ above: first, use the diode into $U_{1}$ for all other $U_{i}$ 's, so that either no electrons exist at all or $U_{1}$ has an electron. Repeat with the other $U_{i}$. This leaves us at the situation described at the start of the problem. Then you could guess the answer was "no" just based on the fact that it's impossible for $n=2,3$ and that there doesn't seem to be a reasonable strategy. + +In this way it's possible to give a pretty good description of what it's possible to do. One possible phrasing: "the physicist can arrange the usamons in a line such that all the charged usamons are to the left of the un-charged usamons, but can't determine the number of charged usamons". + +## §2 Solutions to Day 2 + +## §2.1 USA TST 2015/4, proposed by Victor Wang + +Available online at https://aops.com/community/p4628083. + +## Problem statement + +Let $f: \mathbb{Q} \rightarrow \mathbb{Q}$ be a function such that for any $x, y \in \mathbb{Q}$, the number $f(x+y)-$ $f(x)-f(y)$ is an integer. Decide whether there must exist a constant $c$ such that $f(x)-c x$ is an integer for every rational number $x$. + +No, such a constant need not exist. +One possible solution is as follows: define a sequence by $x_{0}=1$ and + +$$ +\begin{aligned} +& 2 x_{1}=x_{0} \\ +& 2 x_{2}=x_{1}+1 \\ +& 2 x_{3}=x_{2} \\ +& 2 x_{4}=x_{3}+1 \\ +& 2 x_{5}=x_{4} \\ +& 2 x_{6}=x_{5}+1 +\end{aligned} +$$ + +Set $f\left(2^{-k}\right)=x_{k}$ and $f\left(2^{k}\right)=2^{k}$ for $k=0,1, \ldots$ Then, let + +$$ +f\left(a \cdot 2^{k}+\frac{b}{c}\right)=a f\left(2^{k}\right)+\frac{b}{c} +$$ + +for odd integers $a, b, c$. One can verify this works. +A second shorter solution (given by the proposer) is to set, whenever $\operatorname{gcd}(p, q)=1$ and $q>0$, + +$$ +f\left(\frac{p}{q}\right)=\frac{p}{q}(1!+2!+\cdots+q!) . +$$ + +Remark. Silly note: despite appearances, $f(x)=\lfloor x\rfloor$ is not a counterexample since one can take $c=0$. + +## §2.2 USA TST 2015/5, proposed by Po-Shen Loh + +Available online at https://aops.com/community/p4628085. + +## Problem statement + +Fix a positive integer $n$. A tournament on $n$ vertices has all its edges colored by $\chi$ colors, so that any two directed edges $u \rightarrow v$ and $v \rightarrow w$ have different colors. Over all possible tournaments on $n$ vertices, determine the minimum possible value of $\chi$. + +The answer is + +$$ +\chi=\left\lceil\log _{2} n\right\rceil +$$ + +First, we prove by induction on $n$ that $\chi \geq \log _{2} n$ for any coloring and any tournament. The base case $n=1$ is obvious. Now given any tournament, consider any used color $c$. Then it should be possible to divide the tournament into two subsets $A$ and $B$ such that all $c$-colored edges point from $A$ to $B$ (for example by letting $A$ be all vertices which are the starting point of a $c$-edge). +![](https://cdn.mathpix.com/cropped/2024_11_19_2c252c8bc1550e62c4c8g-08.jpg?height=486&width=512&top_left_y=1116&top_left_x=772) + +One of $A$ and $B$ has size at least $n / 2$, say $A$. Since $A$ has no $c$ edges, and uses at least $\log _{2}|A|$ colors other than $c$, we get + +$$ +\chi \geq 1+\log _{2}(n / 2)=\log _{2} n +$$ + +completing the induction. +One can read the construction off from the argument above, but here is a concrete description. For each integer $n$, consider the tournament whose vertices are the binary representations of $S=\{0, \ldots, n-1\}$. Instantiate colors $c_{1}, c_{2}, \ldots$. Then for $v, w \in S$, we look at the smallest order bit for which they differ; say the $k$ th one. If $v$ has a zero in the $k$ th bit, and $w$ has a one in the $k$ th bit, we draw $v \rightarrow w$. Moreover we color the edge with color $c_{k}$. This works and uses at most $\left\lceil\log _{2} n\right\rceil$ colors. + +Remark (Motivation). The philosophy "combinatorial optimization" applies here. The idea is given any color $c$, we can find sets $A$ and $B$ such that all $c$-edges point $A$ to $B$. Once you realize this, the next insight is to realize that you may as well color all the edges from $A$ to $B$ by $c$; after all, this doesn't hurt the condition and makes your life easier. Hence, if $f$ is the answer, we have already a proof that $f(n)=1+\max (f(|A|), f(|B|))$ and we choose $|A| \approx|B|$. This optimization also gives the inductive construction. + +## §2.3 USA TST 2015/6 + +Available online at https://aops.com/community/p4628087. + +## Problem statement + +Let $A B C$ be a non-equilateral triangle and let $M_{a}, M_{b}, M_{c}$ be the midpoints of the sides $B C, C A, A B$, respectively. Let $S$ be a point lying on the Euler line. Denote by $X, Y, Z$ the second intersections of $M_{a} S, M_{b} S, M_{c} S$ with the nine-point circle. Prove that $A X, B Y, C Z$ are concurrent. + +We assume now and forever that $A B C$ is scalene since the problem follows by symmetry in the isosceles case. We present four solutions. + +【 First solution by barycentric coordinates (Evan Chen). Let $A X$ meet $M_{b} M_{c}$ at $D$, and let $X$ reflected over $M_{b} M_{c}^{\prime}$ 's midpoint be $X^{\prime}$. Let $Y^{\prime}, Z^{\prime}, E, F$ be similarly defined. +![](https://cdn.mathpix.com/cropped/2024_11_19_2c252c8bc1550e62c4c8g-09.jpg?height=687&width=807&top_left_y=1070&top_left_x=633) + +By Cevian Nest Theorem it suffices to prove that $M_{a} D, M_{b} E, M_{c} F$ are concurrent. Taking the isotomic conjugate and recalling that $M_{a} M_{b} A M_{c}$ is a parallelogram, we see that it suffices to prove $M_{a} X^{\prime}, M_{b} Y^{\prime}, M_{c} Z^{\prime}$ are concurrent. + +We now use barycentric coordinates on $\triangle M_{a} M_{b} M_{c}$. Let + +$$ +S=\left(a^{2} S_{A}+t: b^{2} S_{B}+t: c^{2} S_{C}+t\right) +$$ + +(possibly $t=\infty$ if $S$ is the centroid). Let $v=b^{2} S_{B}+t, w=c^{2} S_{C}+t$. Hence + +$$ +X=\left(-a^{2} v w:\left(b^{2} w+c^{2} v\right) v:\left(b^{2} w+c^{2} v\right) w\right) +$$ + +Consequently, + +$$ +X^{\prime}=\left(a^{2} v w:-a^{2} v w+\left(b^{2} w+c^{2} v\right) w:-a^{2} v w+\left(b^{2} w+c^{2} v\right) v\right) +$$ + +We can compute + +$$ +b^{2} w+c^{2} v=(b c)^{2}\left(S_{B}+S_{C}\right)+\left(b^{2}+c^{2}\right) t=(a b c)^{2}+\left(b^{2}+c^{2}\right) t +$$ + +Thus + +$$ +-a^{2} v+b^{2} w+c^{2} v=\left(b^{2}+c^{2}\right) t+(a b c)^{2}-(a b)^{2} S_{B}-a^{2} t=S_{A}\left((a b)^{2}+t\right) +$$ + +Finally + +$$ +X^{\prime}=\left(a^{2} v w: S_{A}\left(c^{2} S_{C}+t\right)\left((a b)^{2}+2 t\right): S_{A}\left(b^{2} S_{B}+t\right)\left((a c)^{2}+2 t\right)\right) +$$ + +and from this it's evident that $A X^{\prime}, B Y^{\prime}, C Z^{\prime}$ are concurrent. +\ Second solution by moving points (Anant Mudgal). Let $H_{a}, H_{b}, H_{c}$ be feet of altitudes, and let $\gamma$ denote the nine-point circle. The main claim is that: + +Claim - Lines $X H_{a}, Y H_{b}, Z H_{c}$ are concurrent, + +Proof. In fact, we claim that the concurrence point lies on the Euler line $\ell$. This gives us a way to apply the moving points method: fix triangle $A B C$ and animate $S \in \ell$; then the map + +$$ +\begin{aligned} +& \ell \rightarrow \gamma \rightarrow \ell \\ +& S \mapsto X \mapsto S_{a}:=\ell \cap \overline{H_{a} X} +\end{aligned} +$$ + +is projective, because it consists of two perspectivities. So we want the analogous maps $S \mapsto S_{b}, S \mapsto S_{c}$ to coincide. For this it suffices to check three positions of $S$; since you're such a good customer here are four. + +- If $S$ is the orthocenter of $\triangle M_{a} M_{b} M_{c}$ (equivalently the circumcenter of $\triangle A B C$ ) then $S_{a}$ coincides with the circumcenter of $M_{a} M_{b} M_{c}$ (equivalently the nine-point center of $\triangle A B C)$. By symmetry $S_{b}$ and $S_{c}$ are too. +- If $S$ is the circumcenter of $\triangle M_{a} M_{b} M_{c}$ (equivalently the nine-point center of $\triangle A B C$ ) then $S_{a}$ coincides with the de Longchamps point of $\triangle M_{a} M_{b} M_{c}$ (equivalently orthocenter of $\triangle A B C)$. By symmetry $S_{b}$ and $S_{c}$ are too. +- If $S$ is either of the intersections of the Euler line with $\gamma$, then $S=S_{a}=S_{b}=S_{c}$ (as $S=X=Y=Z$ ). + +This concludes the proof. +![](https://cdn.mathpix.com/cropped/2024_11_19_2c252c8bc1550e62c4c8g-10.jpg?height=681&width=797&top_left_y=1947&top_left_x=638) + +We now use Trig Ceva to carry over the concurrence. By sine law, + +$$ +\frac{\sin \angle M_{c} A X}{\sin \angle A M_{c} X}=\frac{M_{c} X}{A X} +$$ + +and a similar relation for $M_{b}$ gives that + +$$ +\frac{\sin \angle M_{c} A X}{\sin \angle M_{b} A X}=\frac{\sin \angle A M_{c} X}{\sin \angle A M_{b} X} \cdot \frac{M_{c} X}{M_{b} X}=\frac{\sin \angle A M_{c} X}{\sin \angle A M_{b} X} \cdot \frac{\sin \angle X M_{a} M_{c}}{\sin \angle X M_{a} M_{b}} . +$$ + +Thus multiplying cyclically gives + +$$ +\prod_{\text {cyc }} \frac{\sin \angle M_{c} A X}{\sin \angle M_{b} A X}=\prod_{\text {cyc }} \frac{\sin \angle A M_{c} X}{\sin \angle A M_{b} X} \prod_{\text {cyc }} \frac{\sin \angle X M_{a} M_{c}}{\sin \angle X M_{a} M_{b}} . +$$ + +The latter product on the right-hand side equals 1 by Trig Ceva on $\triangle M_{a} M_{b} M_{c}$ with cevians $\overline{M_{a} X}, \overline{M_{b} Y}, \overline{M_{c} Z}$. The former product also equals 1 by Trig Ceva for the concurrence in the previous claim (and the fact that $\angle A M_{c} X=\angle H_{c} H_{a} X$ ). Hence the left-hand side equals 1 , implying the result. + +『 Third solution by moving points (Gopal Goel). In this solution, we will instead use barycentric coordinates with resect to $\triangle A B C$ to bound the degrees suitably, and then verify for seven distinct choices of $S$. + +We let $R$ denote the radius of $\triangle A B C$, and $N$ the nine-point center. +First, imagine solving for $X$ in the following way. Suppose $\vec{X}=\left(1-t_{a}\right) \vec{M}_{a}+t_{a} \vec{S}$. Then, using the dot product (with $|\vec{v}|^{2}=\vec{v} \cdot \vec{v}$ in general) + +$$ +\begin{aligned} +\frac{1}{4} R^{2} & =|\vec{X}-\vec{N}|^{2} \\ +& =\left|t_{a}\left(\vec{S}-\vec{M}_{a}\right)+\vec{M}_{a}-\vec{N}\right|^{2} \\ +& =\left|t_{a}\left(\vec{S}-\vec{M}_{a}\right)\right|^{2}+2 t_{a}\left(\vec{S}-\vec{M}_{a}\right) \cdot\left(\vec{M}_{a}-\vec{N}\right)+\left|\vec{M}_{a}-\vec{N}\right|^{2} \\ +& =t_{a}^{2}\left|\left(\vec{S}-\vec{M}_{a}\right)\right|^{2}+2 t_{a}\left(\vec{S}-\vec{M}_{a}\right) \cdot\left(\vec{M}_{a}-\vec{N}\right)+\frac{1}{4} R^{2} +\end{aligned} +$$ + +Since $t_{a} \neq 0$ we may solve to obtain + +$$ +t_{a}=-\frac{2\left(\vec{M}_{a}-\vec{N}\right) \cdot\left(\vec{S}-\vec{M}_{a}\right)}{\left|\vec{S}-\vec{M}_{a}\right|^{2}} +$$ + +Now imagine $S$ varies along the Euler line, meaning there should exist linear functions $\alpha, \beta, \gamma: \mathbb{R} \rightarrow \mathbb{R}$ such that + +$$ +S=(\alpha(s), \beta(s), \gamma(s)) \quad s \in \mathbb{R} +$$ + +with $\alpha(s)+\beta(s)+\gamma(s)=1$. Thus $t_{a}=\frac{f_{a}}{g_{a}}=\frac{f_{a}(s)}{g_{a}(s)}$ is the quotient of a linear function $f_{a}(s)$ and a quadratic function $g_{a}(s)$. + +So we may write: + +$$ +\begin{aligned} +X & =\left(1-t_{a}\right)\left(0, \frac{1}{2}, \frac{1}{2}\right)+t_{a}(\alpha, \beta, \gamma) \\ +& =\left(t_{a} \alpha, \frac{1}{2}\left(1-t_{a}\right)+t_{a} \beta, \frac{1}{2}\left(1-t_{a}\right)+t_{a} \gamma\right) +\end{aligned} +$$ + +$$ +=\left(2 f_{a} \alpha: g_{a}-f_{a}+2 f_{a} \beta: g_{a}-f_{a}+2 f_{a} \gamma\right) . +$$ + +Thus the coordinates of $X$ are quadratic polynomials in $s$ when written in this way. +In a similar way, the coordinates of $Y$ and $Z$ should be quadratic polynomials in $s$. The Ceva concurrence condition + +$$ +\prod_{\text {cyc }} \frac{g_{a}-f_{a}+2 f_{a} \beta}{g_{a}-f_{a}+2 f_{a} \gamma}=1 +$$ + +is thus a polynomial in $s$ of degree at most six. Our goal is to verify it is identically zero, thus it suffices to check seven positions of $S$. + +- If $S$ is the circumcenter of $\triangle M_{a} M_{b} M_{c}$ (equivalently the nine-point center of $\triangle A B C$ ) then $\overline{A X}, \overline{B Y}, \overline{C Z}$ are altitudes of $\triangle A B C$. +- If $S$ is the centroid of $\triangle M_{a} M_{b} M_{c}$ (equivalently the centroid of $\triangle A B C$ ), then $\overline{A X}$, $\overline{B Y}, \overline{C Z}$ are medians of $\triangle A B C$. +- If $S$ is either of the intersections of the Euler line with $\gamma$, then $S=X=Y=Z$ and all cevians concur at $S$. +- If $S$ lies on the $\overline{M_{a} M_{b}}$, then $Y=M_{a}, X=M_{c}$, and thus $\overline{A X} \cap \overline{B Y}=C$, which is of course concurrent with $\overline{C Z}$ (regardless of $Z$ ). Similarly if $S$ lies on the other sides of $\triangle M_{a} M_{b} M_{c}$. + +Thus we are also done. +【 Fourth solution using Pascal (official one). We give a different proof of the claim that $\overline{X H_{a}}, \overline{Y H_{b}}, \overline{Z H_{c}}$ are concurrent (and then proceed as in the end of the second solution). + +Let $H$ denote the orthocenter, $N$ the nine-point center, and moreover let $N_{a}, N_{b}, N_{c}$ denote the midpoints of $\overline{A H}, \overline{B H}, \overline{C H}$, which also lie on the nine-point circle (and are the antipodes of $M_{a}, M_{b}, M_{c}$ ). + +- By Pascal's theorem on $M_{b} N_{b} H_{b} M_{c} N_{c} H_{c}$, the point $P=\overline{M_{c} H_{b}} \cap \overline{M_{b} H_{c}}$ is collinear with $N=\overline{M_{b} N_{b}} \cap \overline{M_{c} N_{c}}$, and $H=\overline{N_{b} H_{b}} \cap \overline{N_{c} H_{c}}$. So $P$ lies on the Euler line. +- By Pascal's theorem on $M_{b} Y H_{b} M_{c} Z H_{c}$, the point $\overline{Y H_{b}} \cap \overline{Z H_{c}}$ is collinear with $S=\overline{M_{b} Y} \cap \overline{M_{c} Z}$ and $P=\overline{M_{b} H_{c}} \cap \overline{M_{c} H_{b}}$. Hence $Y H_{b}$ and $Z H_{c}$ meet on the Euler line, as needed. + diff --git a/USA_TST/md/en-sols-TST-IMO-2016.md b/USA_TST/md/en-sols-TST-IMO-2016.md new file mode 100644 index 0000000000000000000000000000000000000000..373054f320b45925353648ffa2df91ecbfe5da66 --- /dev/null +++ b/USA_TST/md/en-sols-TST-IMO-2016.md @@ -0,0 +1,260 @@ +# USA IMO TST 2016 Solutions
United States of America - IMO Team Selection Tests
Evan Chen《陳誼廷》
60 ${ }^{\text {th }}$ IMO 2016 Hong Kong + +## Contents + +0 Problems ..... 2 +1 Solutions to Day 1 ..... 3 +1.1 USA TST 2016/1, proposed by Maria Monks ..... 3 +1.2 USA TST 2016/2, proposed by Evan Chen ..... 4 +1.3 USA TST 2016/3, proposed by Mark Sellke ..... 6 +2 Solutions to Day 2 ..... 8 +2.1 USA TST 2016/4, proposed by Iurie Boreico ..... 8 +2.2 USA TST 2016/5, proposed by Zilin Jiang ..... 9 +2.3 USA TST 2016/6, proposed by Ivan Borsenco ..... 10 + +## §0 Problems + +1. Let $S=\{1, \ldots, n\}$. Given a bijection $f: S \rightarrow S$ an orbit of $f$ is a set of the form $\{x, f(x), f(f(x)), \ldots\}$ for some $x \in S$. We denote by $c(f)$ the number of distinct orbits of $f$. For example, if $n=3$ and $f(1)=2, f(2)=1, f(3)=3$, the two orbits are $\{1,2\}$ and $\{3\}$, hence $c(f)=2$. +Given $k$ bijections $f_{1}, \ldots, f_{k}$ from $S$ to itself, prove that + +$$ +c\left(f_{1}\right)+\cdots+c\left(f_{k}\right) \leq n(k-1)+c(f) +$$ + +where $f: S \rightarrow S$ is the composed function $f_{1} \circ \cdots \circ f_{k}$. +2. Let $A B C$ be a scalene triangle with circumcircle $\Omega$, and suppose the incircle of $A B C$ touches $B C$ at $D$. The angle bisector of $\angle A$ meets $B C$ and $\Omega$ at $K$ and $M$. The circumcircle of $\triangle D K M$ intersects the $A$-excircle at $S_{1}, S_{2}$, and $\Omega$ at $T \neq M$. Prove that line $A T$ passes through either $S_{1}$ or $S_{2}$. +3. Let $p$ be a prime number. Let $\mathbb{F}_{p}$ denote the integers modulo $p$, and let $\mathbb{F}_{p}[x]$ be the set of polynomials with coefficients in $\mathbb{F}_{p}$. Define $\Psi: \mathbb{F}_{p}[x] \rightarrow \mathbb{F}_{p}[x]$ by + +$$ +\Psi\left(\sum_{i=0}^{n} a_{i} x^{i}\right)=\sum_{i=0}^{n} a_{i} x^{x^{i}} . +$$ + +Prove that for nonzero polynomials $F, G \in \mathbb{F}_{p}[x]$, + +$$ +\Psi(\operatorname{gcd}(F, G))=\operatorname{gcd}(\Psi(F), \Psi(G)) +$$ + +4. Let $\sqrt{3}=1 . b_{1} b_{2} b_{3} \cdots(2)$ be the binary representation of $\sqrt{3}$. Prove that for any positive integer $n$, at least one of the digits $b_{n}, b_{n+1}, \ldots, b_{2 n}$ equals 1 . +5. Let $n \geq 4$ be an integer. Find all functions $W:\{1, \ldots, n\}^{2} \rightarrow \mathbb{R}$ such that for every partition $[n]=A \cup B \cup C$ into disjoint sets, + +$$ +\sum_{a \in A} \sum_{b \in B} \sum_{c \in C} W(a, b) W(b, c)=|A||B||C| . +$$ + +6. Let $A B C$ be an acute scalene triangle and let $P$ be a point in its interior. Let $A_{1}$, $B_{1}, C_{1}$ be projections of $P$ onto triangle sides $B C, C A, A B$, respectively. Find the locus of points $P$ such that $A A_{1}, B B_{1}, C C_{1}$ are concurrent and $\angle P A B+\angle P B C+$ $\angle P C A=90^{\circ}$. + +## §1 Solutions to Day 1 + +## §1.1 USA TST 2016/1, proposed by Maria Monks + +Available online at https://aops.com/community/p5679356. + +## Problem statement + +Let $S=\{1, \ldots, n\}$. Given a bijection $f: S \rightarrow S$ an orbit of $f$ is a set of the form $\{x, f(x), f(f(x)), \ldots\}$ for some $x \in S$. We denote by $c(f)$ the number of distinct orbits of $f$. For example, if $n=3$ and $f(1)=2, f(2)=1, f(3)=3$, the two orbits are $\{1,2\}$ and $\{3\}$, hence $c(f)=2$. + +Given $k$ bijections $f_{1}, \ldots, f_{k}$ from $S$ to itself, prove that + +$$ +c\left(f_{1}\right)+\cdots+c\left(f_{k}\right) \leq n(k-1)+c(f) +$$ + +where $f: S \rightarrow S$ is the composed function $f_{1} \circ \cdots \circ f_{k}$. + +Most motivated solution is to consider $n-c(f)$ and show this is the transposition distance. Dumb graph theory works as well. + +## §1.2 USA TST 2016/2, proposed by Evan Chen + +Available online at https://aops.com/community/p5679361. + +## Problem statement + +Let $A B C$ be a scalene triangle with circumcircle $\Omega$, and suppose the incircle of $A B C$ touches $B C$ at $D$. The angle bisector of $\angle A$ meets $B C$ and $\Omega$ at $K$ and $M$. The circumcircle of $\triangle D K M$ intersects the $A$-excircle at $S_{1}, S_{2}$, and $\Omega$ at $T \neq M$. Prove that line $A T$ passes through either $S_{1}$ or $S_{2}$. + +We present an angle-chasing solution, and then a more advanced alternative finish. +【 First solution (angle chasing). Assume for simplicity $A B
\operatorname{deg} B$, then the remainder when $A$ is divided by $B$ is in $\beth$. Suppose $\operatorname{deg} A=p^{k}$ and $B=x^{p^{k-1}}-$ $c_{2} x^{p^{k-2}}-\cdots-c_{k}$. Then + +$$ +\begin{aligned} +x^{p^{k}} & \equiv\left(c_{2} x^{p^{k-2}}+c_{3} x^{p^{k-3}}+\cdots+c_{k}\right)^{p} \quad(\bmod B) \\ +& \equiv c_{2} x^{p^{k-1}}+c_{3} x^{p^{k-2}} \cdots+c_{k} \quad(\bmod B) +\end{aligned} +$$ + +since exponentiation by $p$ commutes with addition in $\mathbb{F}_{p}$. This is enough to imply the conclusion. The proof if $\operatorname{deg} B$ is smaller less than $p^{k-1}$ is similar. + +Thus, if we view $\mathbb{F}_{p}[x]$ and $\beth$ as partially ordered sets under polynomial division, then gcd is the "greatest lower bound" or "meet" in both partially ordered sets. We will now prove that $\Psi$ is an isomorphism of the posets. We have already seen that $P|Q \Longrightarrow \Psi(P)| \Psi(Q)$ from the first solution. For the converse: + +Claim - If $\Psi(P) \mid \Psi(Q)$ then $P \mid Q$. + +Proof. Suppose $\Psi(P) \mid \Psi(Q)$, but $Q=P A+B$ where $\operatorname{deg} B<\operatorname{deg} P$. Thus $\Psi(P) \mid$ $\Psi(P A)+\Psi(B)$, hence $\Psi(P) \mid \Psi(B)$, but $\operatorname{deg} \Psi(P)>\operatorname{deg} \Psi(B)$ hence $\Psi(B)=0 \Longrightarrow$ $B=0$ 。 + +This completes the proof. +Remark. In fact $\Psi: \mathbb{F}_{p}[x] \rightarrow \beth$ is a ring isomorphism if we equip $\beth$ with function composition as the ring multiplication. Indeed in the proof of the first claim (that $P|Q \Longrightarrow \Psi(P)|$ $\Psi(Q)$ ) we saw that + +$$ +\Psi(R P)=\sum_{i=0}^{k} r_{i} \Psi(P)^{p^{i}}=\Psi(R) \circ \Psi(P) +$$ + +## §2 Solutions to Day 2 + +## §2.1 USA TST 2016/4, proposed by lurie Boreico + +Available online at https://aops.com/community/p6368201. + +## Problem statement + +Let $\sqrt{3}=1 . b_{1} b_{2} b_{3} \cdots(2)$ be the binary representation of $\sqrt{3}$. Prove that for any positive integer $n$, at least one of the digits $b_{n}, b_{n+1}, \ldots, b_{2 n}$ equals 1 . + +Assume the contrary, so that for some integer $k$ we have + +$$ +k<2^{n-1} \sqrt{3} United States of America - IMO Team Selection Tests
Evan Chen《陳誼廷》 + +$58^{\text {th }}$ IMO 2017 Brazil + +## Contents + +0 Problems +1 Solutions to Day 1 ..... 4 +1.1 USA TST 2017/1, proposed by Po-Shen Loh ..... 4 +1.2 USA TST 2017/2, proposed by Evan Chen ..... 6 +1.3 USA TST 2017/3, proposed by Alison Miller ..... 8 +2 Solutions to Day 2 ..... 10 +2.1 USA TST 2017/4, proposed by Linus Hamilton ..... 10 +2.2 USA TST 2017/5, proposed by Danielle Wang, Evan Chen ..... 12 +2.3 USA TST 2017/6, proposed by Noam Elkies ..... 13 + +## §0 Problems + +1. In a sports league, each team uses a set of at most $t$ signature colors. A set $S$ of teams is color-identifiable if one can assign each team in $S$ one of their signature colors, such that no team in $S$ is assigned any signature color of a different team in $S$. For all positive integers $n$ and $t$, determine the maximum integer $g(n, t)$ such that: In any sports league with exactly $n$ distinct colors present over all teams, one can always find a color-identifiable set of size at least $g(n, t)$. +2. Let $A B C$ be an acute scalene triangle with circumcenter $O$, and let $T$ be on line $B C$ such that $\angle T A O=90^{\circ}$. The circle with diameter $\overline{A T}$ intersects the circumcircle of $\triangle B O C$ at two points $A_{1}$ and $A_{2}$, where $O A_{1}1$ other contestant's guesses before writing your own. For each question, after all guesses are submitted, the emcee announces the correct answer. A correct guess is worth 0 points. An incorrect guess is worth -2 points for other contestants, but only -1 point for you, because you hacked the scoring system. After announcing the correct answer, the emcee proceeds to read out the next question. Show that if you are leading by $2^{n-1}$ points at any time, then you can surely win first place. +5. Let $A B C$ be a triangle with altitude $\overline{A E}$. The $A$-excircle touches $\overline{B C}$ at $D$, and intersects the circumcircle at two points $F$ and $G$. Prove that one can select points $V$ and $N$ on lines $D G$ and $D F$ such that quadrilateral $E V A N$ is a rhombus. +6. Prove that there are infinitely many triples $(a, b, p)$ of integers, with $p$ prime and $0
\operatorname{deg}\left(Q^{\prime} P-Q P^{\prime}\right) +\end{aligned} +$$ + +This can only occur if $Q^{\prime} P-Q P^{\prime}=0$ or $(P / Q)^{\prime}=0$ by the quotient rule! But $P / Q$ can't be constant, the end. + +Remark. The result is previously known; see e.g. Lemma 1.6 of http://math.mit.edu/ ebelmont/ec-notes.pdf or Exercise 6.5.L(a) of Vakil's notes. + +## §2 Solutions to Day 2 + +## §2.1 USA TST 2017/4, proposed by Linus Hamilton + +Available online at https://aops.com/community/p7732191. + +## Problem statement + +You are cheating at a trivia contest. For each question, you can peek at each of the $n>1$ other contestant's guesses before writing your own. For each question, after all guesses are submitted, the emcee announces the correct answer. A correct guess is worth 0 points. An incorrect guess is worth -2 points for other contestants, but only -1 point for you, because you hacked the scoring system. After announcing the correct answer, the emcee proceeds to read out the next question. Show that if you are leading by $2^{n-1}$ points at any time, then you can surely win first place. + +We will prove the result with $2^{n-1}$ replaced even by $2^{n-2}+1$. +We first make the following reductions. First, change the weights to be $+1,-1,0$ respectively (rather than $0,-2,-1$ ); this clearly has no effect. Also, WLOG that all contestants except you initially have score zero (and that your score exceeds $2^{n-2}$ ). WLOG ignore rounds in which all answers are the same. Finally, ignore rounds in which you get the correct answer, since that leaves you at least as well off as before - in other words, we'll assume your score is always fixed, but you can pick any group of people with the same answers and ensure they lose 1 point, while some other group gains 1 point. + +The key observation is the following. Consider two rounds $R_{1}$ and $R_{2}$ such that: + +- In round $R_{1}$, some set $S$ of contestants gains a point. +- In round $R_{2}$, the set $S$ of contestants all have the same answer. + +Then, if we copy the answers of contestants in $S$ during $R_{2}$, then the sum of the scorings in $R_{1}$ and $R_{2}$ cancel each other out. In other words we can then ignore $R_{1}$ and $R_{2}$ forever. + +We thus consider the following strategy. We keep a list $\mathcal{L}$ of subsets of $\{1, \ldots, n\}$, initially empty. Now do the following strategy: + +- On a round, suppose there exists a set $S$ of people with the same answer such that $S \in \mathcal{L}$. (If multiple exist, choose one arbitrarily.) Then, copy the answer of $S$, causing them to lose a point. Delete $S$ from $\mathcal{L}$. (Importantly, we do not add any new sets to $\mathcal{L}$.) +- Otherwise, copy any set $T$ of contestants, selecting $|T| \geq n / 2$ if possible. Let $S$ be the set of contestants who answer correctly (if any), and add $S$ to the list $\mathcal{L}$. Note that $|S| \leq n / 2$, since $S$ is disjoint from $T$. + +By construction, $\mathcal{L}$ has no duplicate sets. So the score of any contestant $c$ is bounded above by the number of times that $c$ appears among sets in $\mathcal{L}$. The number of such sets is clearly at most $\frac{1}{2} \cdot 2^{n-1}$. So, if you lead by $2^{n-2}+1$ then you ensure victory. This completes the proof! + +Remark. Several remarks are in order. First, we comment on the bound $2^{n-2}+1$ itself. The most natural solution using only the list idea gives an upper bound of $\left(2^{n}-2\right)+1$, which is the number of nonempty proper subsets of $\{1, \ldots, n\}$. Then, there are two optimizations one can observe: + +- In fact we can improve to the number of times any particular contestant $c$ appears in some set, rather than the total number of sets. +- When adding new sets $S$ to $\mathcal{L}$, one can ensure $|S| \leq n / 2$. + +Either observation alone improves the bound from $2^{n}-1$ to $2^{n-1}$, but both together give the bound $2^{n-2}+1$. Additionally, when $n$ is odd the calculation of subsets actually gives $2^{n-2}-\frac{1}{2}\binom{n-1}{\frac{n-1}{2}}+1$. This gives the best possible value at both $n=2$ and $n=3$. It seems likely some further improvements are possible, and the true bound is suspected to be polynomial in $n$. + +Secondly, the solution is highly motivated by considering a true/false contest in which only two distinct answers are given per question. However, a natural mistake (which graders assessed as a two-point deduction) is to try and prove that in fact one can "WLOG" we are in the two-question case. The proof of this requires substantially more care than expected. For instance, set $n=3$. If $\mathcal{L}=\{\{1\},\{2\},\{3\}\}$ then it becomes impossible to prevent a duplicate set from appearing in $\mathcal{L}$ if all contestants give distinct answers. One might attempt to fix this by instead adding to $\mathcal{L}$ the complement of the set $T$ described above. The example $\mathcal{L}=\{\{1,2\},\{2,3\},\{3,1\}\}$ (followed again by a round with all distinct answers) shows that this proposed fix does not work either. This issue affects all variations of the above approach. + +Because the USA TST did not have any solution-writing process at this time, this issue was not noticed until January 15 (less than a week before the exam). When it was brought up by email by Evan, every organizer who had testsolved the problem had apparently made the same error. + +Remark. Here are some motivations for the solution: + +1. The exponential bound $2^{n}$ suggests looking at subsets. +2. The $n=2$ case suggests the idea of "repeated rounds". (I think this $n=2$ case is actually really good.) +3. The "two distinct answers" case suggests looking at rounds as partitions (even though the WLOG does not work, at least not without further thought). +4. There's something weird about this problem: it's a finite bound over unbounded time. This is a hint to not worry excessively about the actual scores, which turn out to be almost irrelevant. + +## §2.2 USA TST 2017/5, proposed by Danielle Wang, Evan Chen + +Available online at https://aops.com/community/p7732197. + +## Problem statement + +Let $A B C$ be a triangle with altitude $\overline{A E}$. The $A$-excircle touches $\overline{B C}$ at $D$, and intersects the circumcircle at two points $F$ and $G$. Prove that one can select points $V$ and $N$ on lines $D G$ and $D F$ such that quadrilateral $E V A N$ is a rhombus. + +Let $I$ denote the incenter, $J$ the $A$-excenter, and $L$ the midpoint of $\overline{A E}$. Denote by $\overline{I Y}$, $\overline{I Z}$ the tangents from $I$ to the $A$-excircle. Note that lines $\overline{B C}, \overline{G F}, \overline{Y Z}$ then concur at $H$ (unless $A B=A C$, but this case is obvious), as it's the radical center of cyclic hexagon $B I C Y J Z$, the circumcircle and the $A$-excircle. +![](https://cdn.mathpix.com/cropped/2024_11_19_5d386b123511deaa59b4g-12.jpg?height=766&width=1200&top_left_y=968&top_left_x=431) + +Now let $\overline{H D}$ and $\overline{H T}$ be the tangents from $H$ to the $A$-excircle. It follows that $\overline{D T}$ is the symmedian of $\triangle D Z Y$, hence passes through $I=\overline{Y Y} \cap \overline{Z Z}$. Moreover, it's well known that $\overline{D I}$ passes through $L$, the midpoint of the $A$-altitude (for example by homothety). Finally, $(D T ; F G)=-1$, hence project through $D$ onto the line through $L$ parallel to $\overline{B C}$ to obtain $(\infty L ; V N)=-1$ as desired. + +【 Authorship comments. This is a joint proposal with Danielle Wang (mostly by her). The formulation given was that the tangents to the $A$-excircle at $F$ and $G$ was on line $\overline{D I}$; I solved this formulation using the radical axis argument above. I then got the idea to involve the point $L$, already knowing it was on $\overline{D I}$. Observing the harmonic quadrilateral, I took perspectivity through $M$ onto the line through $L$ parallel to $\overline{B C}$ (before this I had tried to use the $A$-altitude with little luck). This yields the rhombus in the problem. + +## §2.3 USA TST 2017/6, proposed by Noam Elkies + +Available online at https://aops.com/community/p7732203. + +## Problem statement + +Prove that there are infinitely many triples $(a, b, p)$ of integers, with $p$ prime and $0 United States of America - IMO Team Selection Tests
Evan Chen《陳誼廷》 + +$59^{\text {th }}$ IMO 2018 Romania + +## Contents + +0 Problems ..... 2 +1 Solutions to Day 1 ..... 3 +1.1 USA TST 2018/1, proposed by Ashwin Sah ..... 3 +1.2 USA TST 2018/2, proposed by Michael Kural, Yang Liu ..... 5 +1.3 USA TST 2018/3, proposed by Evan Chen ..... 7 +2 Solutions to Day 2 ..... 12 +2.1 USA TST 2018/4, proposed by Josh Brakensiek ..... 12 +2.2 USA TST 2018/5, proposed by Evan Chen ..... 14 +2.3 USA TST 2018/6, proposed by Mark Sellke ..... 16 + +## §0 Problems + +1. Let $n \geq 2$ be a positive integer, and let $\sigma(n)$ denote the sum of the positive divisors of $n$. Prove that the $n^{\text {th }}$ smallest positive integer relatively prime to $n$ is at least $\sigma(n)$, and determine for which $n$ equality holds. +2. Find all functions $f: \mathbb{Z}^{2} \rightarrow[0,1]$ such that for any integers $x$ and $y$, + +$$ +f(x, y)=\frac{f(x-1, y)+f(x, y-1)}{2} +$$ + +3. At a university dinner, there are 2017 mathematicians who each order two distinct entrées, with no two mathematicians ordering the same pair of entrées. The cost of each entrée is equal to the number of mathematicians who ordered it, and the university pays for each mathematician's less expensive entrée (ties broken arbitrarily). Over all possible sets of orders, what is the maximum total amount the university could have paid? +4. Let $n$ be a positive integer and let $S \subseteq\{0,1\}^{n}$ be a set of binary strings of length $n$. Given an odd number $x_{1}, \ldots, x_{2 k+1} \in S$ of binary strings (not necessarily distinct), their majority is defined as the binary string $y \in\{0,1\}^{n}$ for which the $i^{\text {th }}$ bit of $y$ is the most common bit among the $i^{\text {th }}$ bits of $x_{1}, \ldots, x_{2 k+1}$. (For example, if $n=4$ the majority of $0000,0000,1101,1100,0101$ is 0100 .) +Suppose that for some positive integer $k, S$ has the property $P_{k}$ that the majority of any $2 k+1$ binary strings in $S$ (possibly with repetition) is also in $S$. Prove that $S$ has the same property $P_{k}$ for all positive integers $k$. +5. Let $A B C D$ be a convex cyclic quadrilateral which is not a kite, but whose diagonals are perpendicular and meet at $H$. Denote by $M$ and $N$ the midpoints of $\overline{B C}$ and $\overline{C D}$. Rays $M H$ and $N H$ meet $\overline{A D}$ and $\overline{A B}$ at $S$ and $T$, respectively. Prove there exists a point $E$, lying outside quadrilateral $A B C D$, such that + +- ray $E H$ bisects both angles $\angle B E S, \angle T E D$, and +- $\angle B E N=\angle M E D$. + +6. Alice and Bob play a game. First, Alice secretly picks a finite set $S$ of lattice points in the Cartesian plane. Then, for every line $\ell$ in the plane which is horizontal, vertical, or has slope +1 or -1 , she tells Bob the number of points of $S$ that lie on $\ell$. Bob wins if he can then determine the set $S$. +Prove that if Alice picks $S$ to be of the form + +$$ +S=\left\{(x, y) \in \mathbb{Z}^{2} \mid m \leq x^{2}+y^{2} \leq n\right\} +$$ + +for some positive integers $m$ and $n$, then Bob can win. (Bob does not know in advance that $S$ is of this form.) + +## §1 Solutions to Day 1 + +## §1.1 USA TST 2018/1, proposed by Ashwin Sah + +Available online at https://aops.com/community/p9513094. + +## Problem statement + +Let $n \geq 2$ be a positive integer, and let $\sigma(n)$ denote the sum of the positive divisors of $n$. Prove that the $n^{\text {th }}$ smallest positive integer relatively prime to $n$ is at least $\sigma(n)$, and determine for which $n$ equality holds. + +The equality case is $n=p^{e}$ for $p$ prime and a positive integer $e$. It is easy to check that this works. + +【 First solution. In what follows, by $[a, b]$ we mean $\{a, a+1, \ldots, b\}$. First, we make the following easy observation. + +Claim - If $a$ and $d$ are positive integers, then precisely $\varphi(d)$ elements of $[a, a+d-1]$ are relatively prime to $d$. + +Let $d_{1}, d_{2}, \ldots, d_{k}$ denote denote the divisors of $n$ in some order. Consider the intervals + +$$ +\begin{aligned} +I_{1} & =\left[1, d_{1}\right] \\ +I_{2} & =\left[d_{1}+1, d_{1}+d_{2}\right] \\ +& \vdots \\ +I_{k} & =\left[d_{1}+\cdots+d_{k-1}+1, d_{1}+\cdots+d_{k}\right] . +\end{aligned} +$$ + +of length $d_{1}, \ldots, d_{k}$ respectively. The $j$ th interval will have exactly $\varphi\left(d_{j}\right)$ elements which are relatively prime $d_{j}$, hence at most $\varphi\left(d_{j}\right)$ which are relatively prime to $n$. Consequently, in $I=\bigcup_{j=1}^{k} I_{k}$ there are at most + +$$ +\sum_{j=1}^{k} \varphi\left(d_{j}\right)=\sum_{d \mid n} \varphi(d)=n +$$ + +integers relatively prime to $n$. On the other hand $I=[1, \sigma(n)]$ so this implies the inequality. + +We see that the equality holds for $n=p^{e}$. Assume now $p2$, then one can again show by induction $p_{3} \ldots p_{k} \geq 2^{k}-1$ (since $p_{3} \geq 7$ ), which also implies the result. + +## §1.2 USA TST 2018/2, proposed by Michael Kural, Yang Liu + +Available online at https://aops.com/community/p9513099. + +## Problem statement + +Find all functions $f: \mathbb{Z}^{2} \rightarrow[0,1]$ such that for any integers $x$ and $y$, + +$$ +f(x, y)=\frac{f(x-1, y)+f(x, y-1)}{2} +$$ + +We claim that the only functions $f$ are constant functions. (It is easy to see that they work.) + +TI First solution (hands-on). First, iterating the functional equation relation to the $n$th level shows that + +$$ +f(x, y)=\frac{1}{2^{n}} \sum_{i=0}^{n}\binom{n}{i} f(x-i, y-(n-i)) +$$ + +In particular, + +$$ +\begin{aligned} +|f(x, y)-f(x-1, y+1)| & =\frac{1}{2^{n}}\left|\sum_{i=0}^{n+1} f(x-i, y-(n-i)) \cdot\left(\binom{n}{i}-\binom{n}{i-1}\right)\right| \\ +& \leq \frac{1}{2^{n}} \sum_{i=0}^{n+1}\left|\binom{n}{i}-\binom{n}{i-1}\right| \\ +& =\frac{1}{2^{n}} \cdot 2\binom{n}{\lfloor n / 2\rfloor} +\end{aligned} +$$ + +where we define $\binom{n}{n+1}=\binom{n}{-1}=0$ for convenience. Since + +$$ +\binom{n}{\lfloor n / 2\rfloor}=o\left(2^{n}\right) +$$ + +it follows that $f$ must be constant. +Remark. A very similar proof extends to $d$ dimensions. + +I Second solution (random walks, Mark Sellke). We show that if $x+y=x^{\prime}+y^{\prime}$ then $f(x, y)=f\left(x^{\prime}, y^{\prime}\right)$. Let $Z_{n}, Z_{n}^{\prime}$ be random walks starting at $(x, y)$ and $\left(x^{\prime}, y^{\prime}\right)$ and moving down/left. Then $f\left(Z_{n}\right)$ is a martingale so we have + +$$ +\mathbb{E}\left[f\left(Z_{n}\right)\right]=f(x, y), \quad \mathbb{E}\left[f\left(Z_{n}^{\prime}\right)\right]=f\left(x^{\prime}, y^{\prime}\right) +$$ + +We'll take $Z_{n}, Z_{n}^{\prime}$ to be independent until they hit each other, after which they will stay together. Then + +$$ +\left|\mathbb{E}\left[f\left(Z_{n}\right)-f\left(Z_{n}^{\prime}\right)\right]\right| \leq \mathbb{E}\left[\left|f\left(Z_{n}\right)-f\left(Z_{n}^{\prime}\right)\right|\right] \leq p_{n} +$$ + +where $p_{n}$ is the probability that $Z_{n}, Z_{n}^{\prime}$ never collide. But the distance between $Z_{n}, Z_{n}^{\prime}$ is essentially a 1 -dimensional random walk, so they will collide with probability 1 , meaning $\lim _{n \rightarrow \infty} p_{n}=0$. Hence + +$$ +\left|f(x, y)-f\left(x^{\prime}, y^{\prime}\right)\right|=\left|\mathbb{E}\left[f\left(Z_{n}\right)-f\left(Z_{n}^{\prime}\right)\right]\right|=o(1) +$$ + +as desired. + +Remark. If the problem were in $\mathbb{Z}^{d}$ for large $d$, this solution wouldn't work as written because the independent random walks wouldn't hit each other. However, this isn't a serious problem because $Z_{n}, Z_{n}^{\prime}$ don't have to be independent before hitting each other. Indeed, if every time $Z_{n}, Z_{n}^{\prime}$ agree on a new coordinate we force them to agree on that coordinate forever, we can make the two walks individually have the distribution of a coordinate-decreasing random walk but make them intersect eventually with probability 1. The difference in each coordinate will be a 1-dimensional random walk which gets stuck at 0 . + +I Third solution (martingales). Imagine starting at $(x, y)$ and taking a random walk down and to the left. This is a martingale. As $f$ is bounded, this martingale converges with probability 1 . Let $X_{1}, X_{2}, \ldots$ each be random variables that represent either down moves or left moves with equal probability. Note that by the Hewitt-Savage 0-1 law, we have that for any real numbers $ax_{i+1} \geq x_{j-1}>x_{j}$. Then replacing $\left(x_{i}, x_{j}\right)$ by $\left(x_{i}-1, x_{j}+1\right)$ strictly increases $A$ preserving all conditions. Thus we may assume all numbers in $\left\{x_{1}, \ldots, x_{64}\right\}$ differ by at most 1 . +- Suppose $x_{65} \geq 4$. Then we can replace $\left(x_{1}, x_{2}, x_{3}, x_{4}, x_{65}\right)$ by $\left(x_{1}+1, x_{2}+1, x_{3}+\right.$ $\left.1, x_{4}+1, x_{65}-4\right)$ and strictly increase $A$. Hence we may assume $x_{65} \leq 3$. + +We will also tacitly assume $\sum x_{i}=4034$, since otherwise we can increase $x_{1}$. These two properties leave only four sequences to examine: + +- $x_{1}=x_{2}=x_{3}=\cdots=x_{63}=63, x_{64}=62$, and $x_{65}=3$, which gives $A=126948$. +- $x_{1}=x_{2}=x_{3}=\cdots=x_{63}=x_{64}=63$ and $x_{65}=2$, which gives $A=127010$. +- $x_{1}=64, x_{2}=x_{3}=\cdots=x_{63}=x_{64}=63$ and $x_{65}=1$, which gives $A=127009$. +- $x_{1}=x_{2}=64, x_{3}=\cdots=x_{63}=x_{64}=63$ and $x_{65}=0$, which gives $A=127009$. + +This proves that $A \leq 127010$. To see that equality occurs only in the second case above, note that all the smoothing operations other than incrementing $x_{1}$ were strict, and that $x_{1}$ could not have been incremented in this way as $x_{1}=x_{2}=63$. + +This shows that $S(G) \leq 127010$ for all graphs $G$, so it remains to show equality never occurs. Retain the notation $d_{i}$ and $a_{i}$ of the combinatorial bound now; we would need to have $d_{1}=\cdots=d_{64}=63$ and $d_{65}=2$ (in particular, deleting isolated vertices from $G$, we may assume $n=65$ ). In that case, we have $a_{i} \leq i-1$ but also $a_{65}=2$ by definition (the last vertex gets all edges associated to it). Finally, + +$$ +\begin{aligned} +S(G) & =\sum_{i=1}^{n} a_{i} d_{i}=63\left(a_{1}+\cdots+a_{64}\right)+a_{65} \\ +& =63\left(2017-a_{65}\right)+a_{65} \leq 63 \cdot 2015+2=126947 +\end{aligned} +$$ + +completing the proof. +Remark. Another way to finish once $S(G) \leq 127010$ is note there is a unique graph (up to isomorphism and deletion of universal vertices) with degree sequence $\left(d_{1}, \ldots, d_{65}\right)=$ $(63, \ldots, 63,2)$. Indeed, the complement of the graph has degree sequence $(1, \ldots, 1,63)$, and so it must be a 63 -star plus a single edge. One can then compute $S(G)$ explicitly for this graph. + +## 【 Some further remarks. + +Remark. Interestingly, the graph $C_{4}$ has $\binom{3}{2}+1=4$ edges and $S\left(C_{4}\right)=8$, while $S\left(L_{3}\right)=7$. This boundary case is visible in the combinatorial solution in the base case of the first claim. It also explains why we end up with the bound $S(G) \leq 127010$ in the second algebraic solution, and why it is necessary to analyze the equality cases so carefully; observe in $k=3$ the situation $d_{1}=d_{2}=d_{3}=d_{4}=2$. + +Remark. Some comments about further context for this problem: + +- The obvious generalization of 2017 to any constant was resolved in September 2018 by Mehtaab Sawhney and Ashwin Sah. The relevant paper is On the discrepancy between two Zagreb indices, published in Discrete Mathematics, Volume 341, Issue 9, pages 2575-2589. The arXiv link is https://arxiv.org/pdf/1801.02532.pdf. +- The quantity + +$$ +S(G)=\sum_{e=v w} \min (\operatorname{deg} v, \operatorname{deg} w) +$$ + +in the problem has an interpretation: it can be used to provide a bound on the number of triangles in a graph $G$. To be precise, $\# E(G) \leq \frac{1}{3} S(G)$, since an edge $e=v w$ is part of at most $\min (\operatorname{deg} v, \operatorname{deg} w)$ triangles. + +- For planar graphs it is known $S(G) \leq 18 n-36$ and it is conjectured that for $n$ large +enough, $S(G) \leq 18 n-72$. See https://mathoverflow.net/a/273694/70654. + +I Authorship comments. I came up with the quantity $S(G)$ in a failed attempt to provide a bound on the number of triangles in a graph, since this is natural to consider when you do a standard double-counting via the edges of the triangle. I think the problem was actually APMO 1989, and I ended up not solving the problem (the solution is much simpler), but the quantity $S(G)$ stuck in my head for a while after that. + +Later on that month I was keeping Danielle company while she was working on art project (flower necklace), and with not much to do except doodle on tables I began thinking about $S(G)$ again. I did have the sense that $S(G)$ should be maximized at a graph close to a complete graph. But to my frustration I could not prove it for a long time. Finally after many hours of trying various approaches I was able to at least show that $S(G)$ was maximized for complete graphs if the number of edges was a triangular number. + +I had come up with this in March 2016, which would have been perfect since 2016 is a triangular number, but it was too late to submit it to any contest (the USAMO and IMO deadlines were long past). So on December 31, 2016 I finally sat down and solved it for the case 2017, which took another few hours of thought, then submitted it to that year's IMO. To my dismay it was rejected, but I passed it along to the USA TST after that, thus making it just in time for the close of the calendar year. + +## §2 Solutions to Day 2 + +## §2.1 USA TST 2018/4, proposed by Josh Brakensiek + +Available online at https://aops.com/community/p9735607. + +## Problem statement + +Let $n$ be a positive integer and let $S \subseteq\{0,1\}^{n}$ be a set of binary strings of length $n$. Given an odd number $x_{1}, \ldots, x_{2 k+1} \in S$ of binary strings (not necessarily distinct), their majority is defined as the binary string $y \in\{0,1\}^{n}$ for which the $i^{\text {th }}$ bit of $y$ is the most common bit among the $i^{\text {th }}$ bits of $x_{1}, \ldots, x_{2 k+1}$. (For example, if $n=4$ the majority of $0000,0000,1101,1100,0101$ is 0100 .) + +Suppose that for some positive integer $k, S$ has the property $P_{k}$ that the majority of any $2 k+1$ binary strings in $S$ (possibly with repetition) is also in $S$. Prove that $S$ has the same property $P_{k}$ for all positive integers $k$. + +Let $M$ denote the majority function (of any length). + +【 First solution (induction). We prove all $P_{k}$ are equivalent by induction on $n \geq 2$, with the base case $n=2$ being easy to check by hand. (The case $n=1$ is also vacuous; however, the inductive step is not able to go from $n=1$ to $n=2$.) + +For the inductive step, we proceed by contradiction; assume $S$ satisfies $P_{\ell}$, but not $P_{k}$, so there exist $x_{1}, \ldots, x_{2 k+1} \in S$ whose majority $y=M\left(x_{1}, \ldots, x_{k}\right)$ is not in $S$. We contend that: + +Claim - Let $y_{i}$ be the string which differs from $y$ only in the $i^{\text {th }}$ bit. Then $y_{i} \in S$. + +Proof. For a string $s \in S$ we let $\hat{s}$ denote the string $s$ with the $i^{\text {th }}$ bit deleted (hence with $n-1$ bits). Now let + +$$ +T=\{\hat{s} \mid s \in S\} +$$ + +Since $S$ satisfies $P_{\ell}$, so does $T$; thus by the induction hypothesis on $n, T$ satisfies $P_{k}$. +Consequently, $T \ni M\left(\hat{x}_{1}, \ldots, \hat{x}_{2 k+1}\right)=\hat{y}$. Thus there exists $s \in S$ such that $\hat{s}=\hat{y}$. This implies $s=y$ or $s=y_{i}$. But since we assumed $y \notin S$ it follows $y_{i} \in S$ instead. + +Now take any $2 \ell+1$ copies of the $y_{i}$, about equally often (i.e. the number of times any two $y_{i}$ are taken differs by at most 1 ). We see the majority of these is $y$ itself, contradiction. + +【 Second solution (circuit construction). Note that $P_{k} \Longrightarrow P_{1}$ for any $k$, since + +$$ +M(\underbrace{a, \ldots, a}_{k}, \underbrace{b, \ldots, b}_{k}, c)=M(a, b, c) +$$ + +for any $a, b, c$. +We will now prove $P_{1}+P_{k} \Longrightarrow P_{k+1}$ for any $k$, which will prove the result. Actually, we will show that the majority of any $2 k+3$ strings $x_{1}, \ldots, x_{2 k+3}$ can be expressed by 3 and $(2 k+1)$-majorities. WLOG assume that $M\left(x_{1}, \ldots, x_{2 k+3}\right)=0 \ldots 0$, and let $\odot$ denote binary AND. + +Claim - We have $M\left(x_{1}, x_{2}, M\left(x_{3}, \ldots, x_{2 k+3}\right)\right)=x_{1} \odot x_{2}$. +Proof. Consider any particular bit. The result is clear if the bits are equal. Otherwise, if they differ, the result follows from the original hypothesis that $M\left(x_{1}, \ldots, x_{2 k+3}\right)=0 \ldots 0$ (removing two differing bits does not change the majority). + +By analogy we can construct any $x_{i} \odot x_{j}$. Finally, note that + +$$ +M\left(x_{1} \odot x_{2}, x_{2} \odot x_{3}, \ldots, x_{2 k+1} \odot x_{2 k+2}\right)=0 \ldots 0 +$$ + +as desired. (Indeed, if we look at any index, there were at most $k+1$ 's in the $x_{i}$ strings, and hence there will be at most $k$ 's among $x_{i} \odot x_{i+1}$ for $i=1, \ldots, 2 k+1$.) + +Remark. The second solution can be interpreted in circuit language as showing that all " $2 k+1$-majority gates" are equivalent. See also https://cstheory.stackexchange.com/ a/21399/48303, in which Valiant gives a probabilistic construction to prove that one can construct $(2 k+1)$-majority gates from a polynomial number of 3-majority gates. No explicit construction is known for this. + +## §2.2 USA TST 2018/5, proposed by Evan Chen + +Available online at https://aops.com/community/p9735608. + +## Problem statement + +Let $A B C D$ be a convex cyclic quadrilateral which is not a kite, but whose diagonals are perpendicular and meet at $H$. Denote by $M$ and $N$ the midpoints of $\overline{B C}$ and $\overline{C D}$. Rays $M H$ and $N H$ meet $\overline{A D}$ and $\overline{A B}$ at $S$ and $T$, respectively. Prove there exists a point $E$, lying outside quadrilateral $A B C D$, such that + +- ray $E H$ bisects both angles $\angle B E S, \angle T E D$, and +- $\angle B E N=\angle M E D$. + +The main claim is that $E$ is the intersection of $(A B C D)$ with the circle with diameter $\overline{A H}$. +![](https://cdn.mathpix.com/cropped/2024_11_19_420dc3a54c986f92a77dg-14.jpg?height=1012&width=786&top_left_y=1050&top_left_x=635) + +The following observation can be quickly made without reference to $E$. + +## Lemma + +We have $\angle H S A=\angle H T A=90^{\circ}$. Consequently, quadrilateral $B T S D$ is cyclic. + +Proof. This is direct angle chasing. In fact, $\overline{H M}$ passes through the circumcenter of $\triangle B H C$ and $\triangle H A D \sim \triangle H C B$, so $\overline{H S}$ ought to be the altitude of $\triangle H A D$. + +From here it follows that $E$ is the Miquel point of cyclic quadrilateral $B T S D$. Define $F$ to be the point diametrically opposite $A$, so that $E, H, F$ are collinear, $\overline{C F} \| \overline{B D}$. By +now we already have + +$$ +\measuredangle B E H=\measuredangle B E F=\measuredangle B A F=\measuredangle C A D=\measuredangle H A S=\measuredangle H E S +$$ + +so $\overline{E H}$ bisects $\angle B E S$, and $\angle T E D$. Hence it only remains to show $\angle B E M=\angle N E D$; we present several proofs below. + +I First proof (original solution). Let $P$ be the circumcenter of $B T S D$. The properties of the Miquel point imply $P$ lies on the common bisector $\overline{E H}$ already, and it also lies on the perpendicular bisector of $\overline{B D}$, hence it must be the midpoint of $\overline{H F}$. + +We now contend quadrilaterals $B M P S$ and $D N P T$ are cyclic. Obviously $\overline{M P}$ is the external angle bisector of $\angle B M S$, and $P B=P S$, so $P$ is the arc midpoint of $(B M S)$. The proof for $D N P T$ is analogous. + +It remains to show $\angle B E N=\angle M E D$, or equivalently $\angle B E M=\angle N E D$. By properties of Miquel point we have $E \in(B M P S) \cap(T P N D)$, so + +$$ +\measuredangle B E M=\measuredangle B P M=\measuredangle P B D=\measuredangle B D P=\measuredangle N P D=\measuredangle N E D +$$ + +as desired. + +【 Second proof (2011 G4). By 2011 G 4 , the circumcircle of $\triangle E M N$ is tangent to the circumcircle of $A B C D$. Hence if we extend $\overline{E M}$ and $\overline{E N}$ to meet $(A B C D)$ again at $X$ and $Y$, we get $\overline{X Y}\|\overline{M N}\| \overline{B D}$. Thus $\measuredangle B E M=\measuredangle B E X=\measuredangle Y E D=\measuredangle N E D$. + +I Third proof (involutions, submitted by Daniel Liu). Let $G=\overline{B N} \cap \overline{M D}$ denote the centroid of $\triangle B C D$, and note that it lies on $\overline{E H F}$. + +Now consider the dual of Desargues involution theorem on complete quadrilateral $B M D N C G$ at point $E$. We get + +$$ +(E B, E D), \quad(E M, E N), \quad(E C, E G) +$$ + +form an involutive pairing. +However, the bisector of $\angle B E D$, say $\ell$, is also the angle bisector of $\angle C E F$ (since $\overline{C F} \| \overline{B D})$. So the involution we found must coincide with reflection across $\ell$. This means $\angle M E N$ is bisected by $\ell$ as well, as desired. + +【 Authorship comments. This diagram actually comes from the inverted picture in IMO 2014/3 (which I attended). I had heard for many years that one could solve this problem quickly by inversion at $H$ afterwards. But when I actually tried to do it during an OTIS class years later, I ended up with the picture in the TST problem, and couldn't see why it was true! In the process of trying to reconstruct this rumored solution, I ended up finding most of the properties that ended up in the January TST problem (but were overkill for the original IMO problem). + +Let us make the equivalence explicit by deducing the IMO problem from our work. +Let rays $E M$ and $E N$ meet the circumcircles of $\triangle B H C$ and $\triangle B N C$ again at $X$ and $Y$, with $E M United States of America - IMO Team Selection Tests
Evan Chen《陳誼廷》
60 ${ }^{\text {th }}$ IMO 2019 United Kingdom + +## Contents + +0 Problems ..... 2 +1 Solutions to Day 1 ..... 3 +1.1 USA TST 2019/1, proposed by Merlijn Staps ..... 3 +1.2 USA TST 2019/2, proposed by Ashwin Sah, Yang Liu ..... 7 +1.3 USA TST 2019/3, proposed by Nikolai Beluhov ..... 11 +2 Solutions to Day 2 ..... 14 +2.1 USA TST 2019/4, proposed by Ankan Bhattacharya ..... 14 +2.2 USA TST 2019/5, proposed by Yannick Yao ..... 17 +2.3 USA TST 2019/6, proposed by Ankan Bhattachrya ..... 20 + +## §0 Problems + +1. Let $A B C$ be a triangle and let $M$ and $N$ denote the midpoints of $\overline{A B}$ and $\overline{A C}$, respectively. Let $X$ be a point such that $\overline{A X}$ is tangent to the circumcircle of triangle $A B C$. Denote by $\omega_{B}$ the circle through $M$ and $B$ tangent to $\overline{M X}$, and by $\omega_{C}$ the circle through $N$ and $C$ tangent to $\overline{N X}$. Show that $\omega_{B}$ and $\omega_{C}$ intersect on line $B C$. +2. Let $\mathbb{Z} / n \mathbb{Z}$ denote the set of integers considered modulo $n$ (hence $\mathbb{Z} / n \mathbb{Z}$ has $n$ elements). Find all positive integers $n$ for which there exists a bijective function $g: \mathbb{Z} / n \mathbb{Z} \rightarrow \mathbb{Z} / n \mathbb{Z}$, such that the 101 functions + +$$ +g(x), \quad g(x)+x, \quad g(x)+2 x, \quad \ldots, \quad g(x)+100 x +$$ + +are all bijections on $\mathbb{Z} / n \mathbb{Z}$. +3. A snake of length $k$ is an animal which occupies an ordered $k$-tuple $\left(s_{1}, \ldots, s_{k}\right)$ of cells in an $n \times n$ grid of square unit cells. These cells must be pairwise distinct, and $s_{i}$ and $s_{i+1}$ must share a side for $i=1, \ldots, k-1$. If the snake is currently occupying $\left(s_{1}, \ldots, s_{k}\right)$ and $s$ is an unoccupied cell sharing a side with $s_{1}$, the snake can move to occupy ( $s, s_{1}, \ldots, s_{k-1}$ ) instead. The snake has turned around if it occupied $\left(s_{1}, s_{2}, \ldots, s_{k}\right)$ at the beginning, but after a finite number of moves occupies $\left(s_{k}, s_{k-1}, \ldots, s_{1}\right)$ instead. +Determine whether there exists an integer $n>1$ such that one can place some snake of length at least $0.9 n^{2}$ in an $n \times n$ grid which can turn around. +4. We say a function $f: \mathbb{Z}_{\geq 0} \times \mathbb{Z}_{\geq 0} \rightarrow \mathbb{Z}$ is great if for any nonnegative integers $m$ and $n$, + +$$ +f(m+1, n+1) f(m, n)-f(m+1, n) f(m, n+1)=1 +$$ + +If $A=\left(a_{0}, a_{1}, \ldots\right)$ and $B=\left(b_{0}, b_{1}, \ldots\right)$ are two sequences of integers, we write $A \sim B$ if there exists a great function $f$ satisfying $f(n, 0)=a_{n}$ and $f(0, n)=b_{n}$ for every nonnegative integer $n$ (in particular, $a_{0}=b_{0}$ ). +Prove that if $A, B, C$, and $D$ are four sequences of integers satisfying $A \sim B$, $B \sim C$, and $C \sim D$, then $D \sim A$. +5. Let $n$ be a positive integer. Tasty and Stacy are given a circular necklace with $3 n$ sapphire beads and $3 n$ turquoise beads, such that no three consecutive beads have the same color. They play a cooperative game where they alternate turns removing three consecutive beads, subject to the following conditions: + +- Tasty must remove three consecutive beads which are turquoise, sapphire, and turquoise, in that order, on each of his turns. +- Stacy must remove three consecutive beads which are sapphire, turquoise, and sapphire, in that order, on each of her turns. +They win if all the beads are removed in $2 n$ turns. Prove that if they can win with Tasty going first, they can also win with Stacy going first. + +6. Let $A B C$ be a triangle with incenter $I$, and let $D$ be a point on line $B C$ satisfying $\angle A I D=90^{\circ}$. Let the excircle of triangle $A B C$ opposite the vertex $A$ be tangent to $\overline{B C}$ at point $A_{1}$. Define points $B_{1}$ on $\overline{C A}$ and $C_{1}$ on $\overline{A B}$ analogously, using the excircles opposite $B$ and $C$, respectively. +Prove that if quadrilateral $A B_{1} A_{1} C_{1}$ is cyclic, then $\overline{A D}$ is tangent to the circumcircle of $\triangle D B_{1} C_{1}$. + +## §1 Solutions to Day 1 + +## §1.1 USA TST 2019/1, proposed by Merlijn Staps + +Available online at https://aops.com/community/p11419585. + +## Problem statement + +Let $A B C$ be a triangle and let $M$ and $N$ denote the midpoints of $\overline{A B}$ and $\overline{A C}$, respectively. Let $X$ be a point such that $\overline{A X}$ is tangent to the circumcircle of triangle $A B C$. Denote by $\omega_{B}$ the circle through $M$ and $B$ tangent to $\overline{M X}$, and by $\omega_{C}$ the circle through $N$ and $C$ tangent to $\overline{N X}$. Show that $\omega_{B}$ and $\omega_{C}$ intersect on line $B C$. + +We present four solutions, the second of which shows that $M$ and $N$ can be replaced by any two points on $A B$ and $A C$ satisfying $A M / A B+A N / A C=1$. + +『 First solution using symmedians (Merlijn Staps). Let $\overline{X Y}$ be the other tangent from $X$ to $(A M N)$. + +Claim - Line $\overline{X M}$ is tangent to $(B M Y)$; hence $Y$ lies on $\omega_{B}$. +![](https://cdn.mathpix.com/cropped/2024_11_19_a0a7a76bd95576fab5b9g-03.jpg?height=823&width=1088&top_left_y=1276&top_left_x=484) + +Proof. Let $Z$ be the midpoint of $\overline{A Y}$. Then $\overline{M X}$ is the $M$-symmedian in triangle $A M Y$. Since $\overline{M Z} \| \overline{B Y}$, it follows that $\measuredangle A M X=\measuredangle Z M Y=\measuredangle B Y M$. We conclude that $\overline{X M}$ is tangent to the circumcircle of triangle $B M Y$. + +Similarly, $\omega_{C}$ is the circumcircle of triangle $C N Y$. As $A M Y N$ is cyclic too, it follows that $\omega_{B}$ and $\omega_{C}$ intersect on $\overline{B C}$, by Miquel's theorem. + +Remark. The converse of Miquel's theorem is true, which means the problem is equivalent to showing that the second intersection of the $\omega_{B}$ and $\omega_{C}$ moves along $(A M N)$. Thus the construction of $Y$ above is not so unnatural. + +【 Second solution (Jetze Zoethout). Let $\omega_{B}$ intersect $\overline{B C}$ again at $S$ and let $\overline{M S}$ intersect $\overline{A C}$ again at $Y$. Angle chasing gives $\measuredangle X M Y=\measuredangle X M S=\measuredangle M B S=\measuredangle A B C=$ $\measuredangle X A C=\measuredangle X A Y$, so $Y$ is on the circumcircle of triangle $A M X$. Furthermore, from $\measuredangle X M Y=\measuredangle A B C$ and $\measuredangle A C B=\measuredangle X A B=\measuredangle X Y M$ it follows that $\triangle A B C \sim \triangle X M Y$ and from $\measuredangle X A Y=\measuredangle M B S$ and $\measuredangle Y X A=\measuredangle Y M A=\measuredangle B M S$ it follows that $\triangle A X Y \sim$ $\triangle B M S$. +![](https://cdn.mathpix.com/cropped/2024_11_19_a0a7a76bd95576fab5b9g-04.jpg?height=749&width=1012&top_left_y=568&top_left_x=525) + +We now find + +$$ +\frac{A N}{A X}=\frac{A N / B M}{A X / B M}=\frac{A C / A B}{M S / X Y}=\frac{A B / A B}{M S / X M}=\frac{X M}{M S} +$$ + +which together with $\angle X M S=\angle X A N$ yields $\triangle X M S \sim \triangle X A N$. From $\measuredangle X S Y=$ $\measuredangle X S M=\measuredangle X N A=\measuredangle X N Y$ we now have that $S$ is on the circumcircle of triangle $X N Y$. Finally, we have $\measuredangle X N S=\measuredangle X Y S=\measuredangle X Y M=\measuredangle A C B=\measuredangle N C S$ so $\overline{X N}$ is tangent to the circle through $C, N$, and $S$, as desired. + +【 Third solution by moving points method. Fix triangle $A B C$ and animate $X$ along the tangent at $A$. We let $D$ denote the second intersection point of $\omega_{C}$ with line $\overline{B C}$. + +Claim - The composed map $X \mapsto D$ is a fractional linear transformation (i.e. a projective map) in terms of a real coordinate on line $\overline{A A}, \overline{B C}$. + +Proof. Let $\ell$ denote the perpendicular bisector of $\overline{C N}$, also equipped with a real coordinate. We let $P$ denote the intersection of $\overline{X M}$ with $\ell, S$ the circumcenter of $\triangle C M D$. Let $T$ denote the midpoint of $\overline{B D}$. + +We claim that the composed map + +$$ +\begin{aligned} +& \overline{A A} \rightarrow \ell \rightarrow \ell \rightarrow \overline{B C} \rightarrow \overline{B C} \\ +& \text { by } \quad X \mapsto P \mapsto S \mapsto T \mapsto D +\end{aligned} +$$ + +is projective, by showing each individual map is projective. +![](https://cdn.mathpix.com/cropped/2024_11_19_a0a7a76bd95576fab5b9g-05.jpg?height=438&width=801&top_left_y=249&top_left_x=633) + +- The map $X \mapsto P$ is projective since it is a perspectivity through $N$ from $\overline{A A}$ to $\ell$. +- The map $P \mapsto S$ is projective since it is equivalent to a negative inversion on $\ell$ at the midpoint of $\overline{N C}$ with radius $\frac{1}{2} N C$. (Note $\angle P N S=90^{\circ}$ is fixed.) +- The map $S \mapsto T$ is projective since it is a perspectivity $\ell \rightarrow \overline{B C}$ through the point at infinity perpendicular to $\overline{B C}$ (in fact, it is linear). +- The map $T \mapsto D$ is projective (in fact, linear) since it is a homothety through $C$ with fixed ratio 2 . + +Thus the composed map is projective as well. +Similarly, if we define $D^{\prime}$ so that $\overline{X M}$ is tangent to $\left(B M D^{\prime}\right)$, the map $X \mapsto D^{\prime}$ is projective as well. We aim to show $D=D^{\prime}$, and since the maps correspond to fractional linear transformations in projective coordinates, it suffices to verify it for three distinct choices of $X$. We do so: + +- If $X=\overline{A A} \cap \overline{M N}$, then $D$ and $D^{\prime}$ satisfy $M B=M D^{\prime}, N C=N D$. This means they are the feet of the $A$-altitude on $\overline{B C}$. +- As $X$ approaches $A$ the points $D$ and $D^{\prime}$ approach the infinity point along $\overline{B C}$. +- If $X$ is a point at infinity along $\overline{A A}$, then $D$ and $D^{\prime}$ coincide with the midpoint of $\overline{B C}$. + +This completes the solution. +Remark (Anant Mudgal). An alternative (shorter) way to show $X \mapsto D$ is projective is to notice $\measuredangle X N D$ is a constant angle. I left the longer "original" proof for instructional reasons. + +ब Fourth solution by isogonal conjugates (Anant Mudgal). Let $Y$ be the isogonal conjugate of $X$ in $\triangle A M N$ and $Z$ be the reflection of $Y$ in $\overline{M N}$. As $\overline{A X}$ is tangent to the circumcircle of $\triangle A M N$, it follows that $\overline{A Y} \| \overline{M N}$. Thus $Z$ lies on $\overline{B C}$ since $\overline{M N}$ bisects the strip made by $\overline{A Y}$ and $\overline{B C}$. +![](https://cdn.mathpix.com/cropped/2024_11_19_a0a7a76bd95576fab5b9g-06.jpg?height=838&width=1112&top_left_y=238&top_left_x=335) + +Finally, + +$$ +\measuredangle Z M X=\measuredangle Z M N+\measuredangle N M X=\measuredangle N M Y+\measuredangle Y M A=\measuredangle N M A=\measuredangle Z B M +$$ + +so $\overline{X M}$ is tangent to the circumcircle of $\triangle Z M B$, hence $Z$ lies on $\omega_{B}$. Similarly, $Z$ lies on $\omega_{C}$ and we're done. + +## §1.2 USA TST 2019/2, proposed by Ashwin Sah, Yang Liu + +Available online at https://aops.com/community/p11419598. + +## Problem statement + +Let $\mathbb{Z} / n \mathbb{Z}$ denote the set of integers considered modulo $n$ (hence $\mathbb{Z} / n \mathbb{Z}$ has $n$ elements). Find all positive integers $n$ for which there exists a bijective function $g: \mathbb{Z} / n \mathbb{Z} \rightarrow \mathbb{Z} / n \mathbb{Z}$, such that the 101 functions + +$$ +g(x), \quad g(x)+x, \quad g(x)+2 x, \quad \ldots, \quad g(x)+100 x +$$ + +are all bijections on $\mathbb{Z} / n \mathbb{Z}$. + +Call a function $g$ valiant if it obeys this condition. We claim the answer is all numbers relatively prime to 101 !. + +The construction is to just let $g$ be the identity function. +Before proceeding to the converse solution, we make a long motivational remark. +Remark (Motivation for both parts). The following solution is dense, and it is easier to think about some small cases first, to motivate the ideas. We consider the result where 101 is replaced by 2 or 3 . + +- If we replaced 101 with 2 , you can show $2 \nmid n$ easily: write + +$$ +\sum_{x} x \equiv \sum_{x} g(x) \equiv \sum_{x}(g(x)+x) \quad(\bmod n) +$$ + +which implies + +$$ +0 \equiv \sum_{x} x=\frac{1}{2} n(n+1) \quad(\bmod n) +$$ + +which means $\frac{1}{2} n(n+1) \equiv 0(\bmod n)$, hence $n$ odd. + +- If we replaced 101 with 3 , then you can try a similar approach using squares, since + +$$ +\begin{aligned} +0 & \equiv \sum_{x}\left[(g(x)+2 x)^{2}-2(g(x)+x)^{2}+g(x)^{2}\right] \quad(\bmod n) \\ +& =\sum_{x} 2 x^{2}=2 \cdot \frac{n(n+1)(2 n+1)}{6} +\end{aligned} +$$ + +which is enough to force $3 \nmid n$. + +We now present several different proofs of the converse, all of which generalize the ideas contained here. In everything that follows we assume $n>1$ for convenience. + +TI First solution (original one). The proof is split into two essentially orthogonal claims, which we state as lemmas. + +Lemma (Lemma I: elimination of $g$ ) +Assume valiant $g: \mathbb{Z} / n \mathbb{Z} \rightarrow \mathbb{Z} / n \mathbb{Z}$ exists. Then + +$$ +k!\sum_{x \in \mathbb{Z} / n \mathbb{Z}} x^{k} \equiv 0 \quad(\bmod n) +$$ + +for $k=0,1, \ldots, 100$. + +Proof. Define $g_{x}(T)=g(x)+T x$ for any integer $T$. If we view $g_{x}(T)^{k}$ as a polynomial in $\mathbb{Z}[T]$ of degree $k$ with leading coefficient $x^{k}$, then taking the $k$ th finite difference implies that, for any $x$, + +$$ +k!x^{k}=\binom{k}{0} g_{x}(k)^{k}-\binom{k}{1} g_{x}(k-1)^{k}+\binom{k}{2} g_{x}(k-2)^{k}-\cdots+(-1)^{k}\binom{k}{k} g_{x}(0)^{k} +$$ + +On the other hand, for any $1 \leq k \leq 100$ we should have + +$$ +\begin{aligned} +\sum_{x} g_{x}(0)^{k} \equiv \sum_{x} g_{x}(1)^{k} & \equiv \cdots \equiv \sum_{x} g_{x}(k)^{k} \\ +& \equiv S_{k}:=0^{k}+\cdots+(n-1)^{k} \quad(\bmod n) +\end{aligned} +$$ + +by the hypothesis. Thus we find + +$$ +k!\sum_{x} x^{k} \equiv\left[\binom{k}{0}-\binom{k}{1}+\binom{k}{2}-\cdots\right] S_{k} \equiv 0 \quad(\bmod n) +$$ + +for any $1 \leq k \leq 100$, but also obviously for $k=0$. +We now prove the following self-contained lemma. + +## Lemma (Lemma II: power sum calculation) + +Let $p$ be a prime, and let $n, M$ be positive integers such that + +$$ +M \quad \text { divides } \quad 1^{k}+2^{k}+\cdots+n^{k} +$$ + +for $k=0,1, \ldots, p-1$. If $p \mid n$ then $\nu_{p}(M)<\nu_{p}(n)$. + +Proof. The hypothesis means that that any polynomial $f(T) \in \mathbb{Z}[T]$ with $\operatorname{deg} f \leq p-1$ will have $\sum_{x=1}^{n} f(x) \equiv 0(\bmod M)$. In particular, we have + +$$ +\begin{aligned} +0 & \equiv \sum_{x=1}^{n}(x-1)(x-2) \cdots(x-(p-1)) \\ +& =(p-1)!\sum_{x=1}^{n}\binom{x-1}{p-1}=(p-1)!\binom{n}{p} \quad(\bmod M) . +\end{aligned} +$$ + +But now $\nu_{p}(M) \leq \nu_{p}\left(\binom{n}{p}\right)=\nu_{p}(n)-1$. +Now assume for contradiction that valiant $g: \mathbb{Z} / n \mathbb{Z} \rightarrow \mathbb{Z} / n \mathbb{Z}$ exists, and $p \leq 101$ is the smallest prime dividing $n$. Lemma I implies that $k!\sum_{x} x^{k} \equiv 0(\bmod n)$ for $k=1, \ldots, p-1$ and hence $\sum_{x} x^{k} \equiv 0(\bmod n)$ too. Thus $M=n$ holds in the previous lemma, impossible. +\ A second solution. Both lemmas above admit variations where we focus on working modulo $p^{e}$ rather than working modulo $n$. + +## Lemma (Lemma I') + +Assume valiant $g: \mathbb{Z} / n \mathbb{Z} \rightarrow \mathbb{Z} / n \mathbb{Z}$ exists. Let $p \leq 101$ be a prime, and $e=\nu_{p}(n)$. Then + +$$ +\sum_{x \in \mathbb{Z} / n \mathbb{Z}} x^{k} \equiv 0 \quad\left(\bmod p^{e}\right) +$$ + +for $k=0,1, \ldots, p-1$. + +Proof. This is weaker than Lemma I, but we give an independent specialized proof. Begin by writing + +$$ +\sum_{x}(g(x)+T x)^{k} \equiv \sum_{x} x^{k} \quad\left(\bmod p^{e}\right) . +$$ + +Both sides are integer polynomials in $T$, which vanish at $T=0,1, \ldots, p-1$ by hypothesis (since $p-1 \leq 100$ ). + +We now prove the following more general fact: if $f(T) \in \mathbb{Z}[T]$ is an integer polynomial with $\operatorname{deg} f \leq p-1$, such that $f(0) \equiv \cdots \equiv f(p-1) \equiv 0\left(\bmod p^{e}\right)$, then all coefficients of $f$ are divisible by $p^{e}$. The proof is by induction on $e \geq 1$. When $e=1$, this is just the assertion that the polynomial has at most $\operatorname{deg} f$ roots modulo $p$. When $e \geq 2$, we note that the previous result implies all coefficients are divisible by $p$, and then we divide all coefficients by $p$. + +Applied here, we have that all coefficients of + +$$ +f(T):=\sum_{x}(g(x)+T x)^{k}-\sum_{x} x^{k} +$$ + +are divisible by $p^{e}$. The leading $T^{k}$ coefficient is $\sum_{k} x^{k}$ as desired. + +## Lemma (Lemma II') + +If $e \geq 1$ is an integer, and $p$ is a prime, then + +$$ +\nu_{p}\left(1^{p-1}+2^{p-1}+\cdots+\left(p^{e}-1\right)^{p-1}\right)=e-1 +$$ + +Proof. First, note that the cases where $p=2$ or $e=1$ are easy; since if $p=2$ we have $\sum_{x=0}^{2^{e}-1} x \equiv 2^{e-1}\left(2^{e}-1\right) \equiv-2^{e-1}\left(\bmod 2^{e}\right)$, while if $e=1$ we have $1^{p-1}+\cdots+(p-1)^{p-1} \equiv$ $-1(\bmod p)$. Henceforth assume that $p>2, e>1$. + +Let $g$ be an integer which is a primitive root modulo $p^{e}$. Then, we can sum the terms which are relatively prime to $p$ as + +$$ +S_{0}:=\sum_{\operatorname{gcd}(x, p)=1} x^{p-1} \equiv \sum_{i=1}^{\varphi\left(p^{e}\right)} g^{(p-1) \cdot i} \equiv \frac{g^{p^{e-1}(p-1)^{2}}-1}{g^{p-1}-1} \quad\left(\bmod p^{e}\right) +$$ + +which implies $\nu_{p}\left(S_{0}\right)=e-1$, by lifting the exponent. More generally, for $r \geq 1$ we may set + +$$ +S_{r}:=\sum_{\nu_{p}(x)=r} x^{p-1} \equiv\left(p^{r}\right)^{p-1} \sum_{i=1}^{\varphi\left(p^{e-r}\right)} g_{r}^{(p-1) \cdot i} \quad\left(\bmod p^{e}\right) +$$ + +where $g_{r}$ is a primitive root modulo $p^{e-r}$. Repeating the exponent-lifting calculation shows that $\nu_{p}\left(S_{r}\right)=r(p-1)+((e-r)-1)>e$, as needed. + +Assume to the contrary that $p \leq 101$ is a prime dividing $n$, and a valiant $g: \mathbb{Z} / n \mathbb{Z} \rightarrow$ $\mathbb{Z} / n \mathbb{Z}$ exists. Take $k=p-1$ in Lemma I' to contradict Lemma II' + +【 A third remixed solution. We use Lemma I and Lemma II' from before. As before, assume $g: \mathbb{Z} / n \mathbb{Z} \rightarrow \mathbb{Z} / n \mathbb{Z}$ is valiant, and $n$ has a prime divisor $p \leq 101$. Also, let $e=\nu_{p}(n)$. + +Then $(p-1)!\sum_{x} x^{p-1} \equiv 0(\bmod n)$ by Lemma I, and now + +$$ +\begin{aligned} +0 & \equiv \sum_{x} x^{p-1} \quad\left(\bmod p^{e}\right) \\ +& \equiv \frac{n}{p^{e}} \sum_{x=1}^{p^{e}-1} x^{p-1} \not \equiv 0 \quad\left(\bmod p^{e}\right) +\end{aligned} +$$ + +by Lemma II', contradiction. + +【I A fourth remixed solution. We also can combine Lemma I' and Lemma II. As before, assume $g: \mathbb{Z} / n \mathbb{Z} \rightarrow \mathbb{Z} / n \mathbb{Z}$ is valiant, and let $p$ be the smallest prime divisor of $n$. + +Assume for contradiction $p \leq 101$. By Lemma I' we have + +$$ +\sum_{x} x^{k} \equiv 0 \quad\left(\bmod p^{e}\right) +$$ + +for $k=0, \ldots, p-1$. This directly contradicts Lemma II with $M=p^{e}$. + +# §1.3 USA TST 2019/3, proposed by Nikolai Beluhov + +Available online at https://aops.com/community/p11419601. + + +#### Abstract + +Problem statement A snake of length $k$ is an animal which occupies an ordered $k$-tuple $\left(s_{1}, \ldots, s_{k}\right)$ of cells in an $n \times n$ grid of square unit cells. These cells must be pairwise distinct, and $s_{i}$ and $s_{i+1}$ must share a side for $i=1, \ldots, k-1$. If the snake is currently occupying $\left(s_{1}, \ldots, s_{k}\right)$ and $s$ is an unoccupied cell sharing a side with $s_{1}$, the snake can move to occupy ( $s, s_{1}, \ldots, s_{k-1}$ ) instead. The snake has turned around if it occupied $\left(s_{1}, s_{2}, \ldots, s_{k}\right)$ at the beginning, but after a finite number of moves occupies $\left(s_{k}, s_{k-1}, \ldots, s_{1}\right)$ instead. + +Determine whether there exists an integer $n>1$ such that one can place some snake of length at least $0.9 n^{2}$ in an $n \times n$ grid which can turn around. + + +The answer is yes (and 0.9 is arbitrary). +I First grid-based solution. The following solution is due to Brian Lawrence. For illustration reasons, we give below a figure of a snake of length 89 turning around in an $11 \times 11$ square (which generalizes readily to odd $n$ ). We will see that a snake of length $(n-1)(n-2)-1$ can turn around in an $n \times n$ square, so this certainly implies the problem. +![](https://cdn.mathpix.com/cropped/2024_11_19_a0a7a76bd95576fab5b9g-11.jpg?height=806&width=1040&top_left_y=1370&top_left_x=511) + +Use the obvious coordinate system with $(1,1)$ in the bottom left. Start with the snake as shown in Figure 1, then have it move to $(2,1),(2, n),(n, n-1)$ as in Figure 2. Then, have the snake shift to the position in Figure 3; this is possible since the snake can just walk to $(n, n)$, then start walking to the left and then follow the route; by the time it reaches the $i$ th row from the top its tail will have vacated by then. Once it achieves Figure 3, move the head of the snake to $(3, n)$ to achieve Figure 4. +In Figure 5 and 6, the snake begins to "deform" its loop continuously. In general, this deformation by two squares is possible in the following way. The snake walks first to $(1, n)$ then retraces the steps left by its tail, except when it reaches $(n-1,3)$ it makes a +brief detour to $(n-2,3),(n-2,4),(n-1,4)$ and continues along its way; this gives the position in Figure 5. Then it retraces the entire loop again, except that when it reaches $(n-4,4)$ it turns directly down, and continues retracing its path; thus at the end of this second revolution, we arrive at Figure 6. + +By repeatedly doing perturbations of two cells, we can move move all the "bumps" in the path gradually to protrude from the right; Figure 7 shows a partial application of the procedure, with the final state as shown in Figure 8. + +In Figure 9, we stretch the bottom-most bump by two more cells; this shortens the "tail" by two units, which is fine. Doing this for all $(n-3) / 2$ bumps arrives at the situation in Figure 10, with the snake's head at $(3, n)$. We then begin deforming the turns on the bottom-right by two steps each as in Figure 11, which visually will increase the length of the head. Doing this arrives finally at the situation in Figure 12. Thus the snake has turned around. + +II Second solution phrased using graph theory (Nikolai Beluhov). Let $G$ be any undirected graph. Consider a snake of length $k$ lying within $G$, with each segment of the snake occupying one vertex, consecutive segments occupying adjacent vertices, and no two segments occupying the same vertex. One move of the snake consists of the snake's head advancing to an adjacent empty vertex and segment $i$ advancing to the vertex of segment $i-1$ for $i=2,3, \ldots, k$. + +The solution proceeds in two stages. First we construct a planar graph $G$ such that it is possible for a snake that occupies nearly all of $G$ to turn around inside $G$. Then we construct a subgraph $H$ of a grid adjacency graph such that $H$ is isomorphic to $G$ and $H$ occupies nearly all of the grid. + +For the first stage of the solution, we construct $G$ as follows. +Let $r$ and $\ell$ be positive integers. Start with $r$ disjoint main paths $p_{1}, p_{2}, \ldots, p_{r}$, each of length at least $\ell$, with $p_{i}$ leading from $A_{i}$ to $B_{i}$ for $i=1,2, \ldots, r$. Add to those $r$ linking paths, one leading from $B_{i}$ to $A_{i+1}$ for each $i=1,2, \ldots, r-1$, and one leading from $B_{r}$ to $A_{1}$. Finally, add to those two families of transit paths, with one family containing one transit path joining $A_{1}$ to each of $A_{2}, A_{3}, \ldots, A_{r}$ and the other containing one path joining $B_{r}$ to each of $B_{1}, B_{2}, \ldots, B_{r-1}$. We require that all paths specified in the construction have no interior vertices in common, with the exception of transit paths in the same family. + +We claim that a snake of length $(r-1) \ell$ can turn around inside $G$. +To this end, let the concatenation $A_{1} B_{1} A_{2} B_{2} \ldots A_{r} B_{r}$ of all main and linking paths be the great cycle. We refer to $A_{1} B_{1} A_{2} B_{2} \ldots A_{r} B_{r}$ as the counterclockwise orientation of the great cycle, and to $B_{r} A_{r} B_{r-1} A_{r-1} \ldots B_{1} A_{1}$ as its clockwise orientation. + +Place the snake so that its tail is at $A_{1}$ and its body extends counterclockwise along the great cycle. Then let the snake manoeuvre as follows. (We track only the snake's head, as its movement uniquely determines the movement of the complete body of the snake.) + +At phase 1, advance counterclockwise along the great cycle to $B_{r-1}$, take a detour along a transit path to $B_{r}$, and advance clockwise along the great cycle to $A_{r}$. + +For $i=2,3, \ldots, r-1$, at phase $i$, take a detour along a transit path to $A_{1}$, advance counterclockwise along the great cycle to $B_{r-i}$, take a detour along a transit path to $B_{r}$, and advance clockwise along the great cycle to $A_{r-i+1}$. + +At phase $r$, simply advance clockwise along the great cycle to $A_{1}$. +For the second stage of the solution, let $n$ be a sufficiently large positive integer. Consider an $n \times n$ grid $S$. Number the columns of $S$ from 1 to $n$ from left to right, and its rows from 1 to $n$ from bottom to top. + +Let $a_{1}, a_{2}, \ldots, a_{r+1}$ be cells of $S$ such that all of $a_{1}, a_{2}, \ldots, a_{r+1}$ lie in column 2 , $a_{1}$ lies in row $2, a_{r+1}$ lies in row $n-1$, and $a_{1}, a_{2}, \ldots, a_{r+1}$ are approximately equally spaced. Let $b_{1}, b_{2}, \ldots, b_{r}$ be cells of $S$ such that all of $b_{1}, b_{2}, \ldots, b_{r}$ lie in column $n-2$ and $b_{i}$ lies in the row of $a_{i+1}$ for $i=1,2, \ldots, r$. + +Construct $H$ as follows. For $i=1,2, \ldots, r$, let the main path from $a_{i}$ to $b_{i}$ fill up the rectangle bounded by the rows and columns of $a_{i}$ and $b_{i}$ nearly completely. Then every main path is of length approximately $\frac{1}{r} n^{2}$. + +For $i=1,2, \ldots, r-1$, let the linking path that leads from $b_{i}$ to $a_{i+1}$ lie inside the row of $b_{i}$ and $a_{i+1}$ and let the linking path that leads from $b_{r}$ to $a_{1}$ lie inside row $n$, column $n$, and row 1. + +Lastly, let the union of the first family of transit paths be column 1 and let the union of the second family of transit paths be column $n-1$, with the exception of their bottommost and topmost squares. + +As in the first stage of the solution, by this construction a snake of length $k$ approximately equal to $\frac{r-1}{r} n^{2}$ can turn around inside an $n \times n \operatorname{grid} S$. When $r$ is fixed and $n$ tends to infinity, $\frac{k}{n^{2}}$ tends to $\frac{r-1}{r}$. Furthermore, when $r$ tends to infinity, $\frac{r-1}{r}$ tends to 1. This gives the answer. + +## §2 Solutions to Day 2 + +## §2.1 USA TST 2019/4, proposed by Ankan Bhattacharya + +Available online at https://aops.com/community/p11625808. + +## Problem statement + +We say a function $f: \mathbb{Z}_{\geq 0} \times \mathbb{Z}_{\geq 0} \rightarrow \mathbb{Z}$ is great if for any nonnegative integers $m$ and n, + +$$ +f(m+1, n+1) f(m, n)-f(m+1, n) f(m, n+1)=1 +$$ + +If $A=\left(a_{0}, a_{1}, \ldots\right)$ and $B=\left(b_{0}, b_{1}, \ldots\right)$ are two sequences of integers, we write $A \sim B$ if there exists a great function $f$ satisfying $f(n, 0)=a_{n}$ and $f(0, n)=b_{n}$ for every nonnegative integer $n$ (in particular, $a_{0}=b_{0}$ ). + +Prove that if $A, B, C$, and $D$ are four sequences of integers satisfying $A \sim B$, $B \sim C$, and $C \sim D$, then $D \sim A$. + +We present two solutions. In what follows, we say $(A, B)$ form a great pair if $A \sim B$. +【 First solution (Nikolai Beluhov). Let $k=a_{0}=b_{0}=c_{0}=d_{0}$. We let $f, g, h$ be great functions for $(A, B),(B, C),(C, D)$ and write the following infinite array: + +$$ +\left[\begin{array}{ccccccc} +& \vdots & \vdots & b_{3} & \vdots & \vdots & \\ +\cdots & g(2,2) & g(2,1) & b_{2} & f(1,2) & f(2,2) & \cdots \\ +\cdots & g(1,2) & g(1,1) & b_{1} & f(1,1) & f(2,1) & \cdots \\ +c_{3} & c_{2} & c_{1} & k & a_{1} & a_{2} & a_{3} \\ +\cdots & h(2,1) & h(1,1) & d_{1} & & & \\ +\cdots & h(2,2) & h(1,2) & d_{2} & & & \\ +& \vdots & \vdots & d_{3} & & & \ddots +\end{array}\right] +$$ + +The greatness condition is then equivalent to saying that any $2 \times 2$ sub-grid has determinant $\pm 1$ (the sign is +1 in two quadrants and -1 in the other two), and we wish to fill in the lower-right quadrant. To this end, it suffices to prove the following. + +## Lemma + +Suppose we have a $3 \times 3$ sub-grid + +$$ +\left[\begin{array}{lll} +a & b & c \\ +x & y & z \\ +p & q & +\end{array}\right] +$$ + +satisfying the determinant conditions. Then we can fill in the ninth entry in the lower right with an integer while retaining greatness. + +Proof. We consider only the case where the $3 \times 3$ is completely contained inside the bottom-right quadrant, since the other cases are exactly the same (or even by flipping the signs of the top row or left column appropriately). + +If $y=0$ we have $-1=b z=b x=x q$, hence $q z=-1$, and we can fill in the entry arbitrarily. + +Otherwise, we have $b x \equiv x q \equiv b z \equiv-1(\bmod y)$. This is enough to imply $q z \equiv-1$ $(\bmod y)$, and so we can fill in the integer $\frac{q z+1}{y}$. + +Remark. In this case (of all +1 determinants), I think it turns out the bottom entry is exactly equal to $q z a-c y p-c-p$, which is obviously an integer. + +Second solution (Ankan Bhattacharya). We will give an explicit classification of great sequences: + +## Lemma + +The pair $(A, B)$ is great if and only if $a_{0}=b_{0}, a_{0} \mid a_{1} b_{1}+1$, and $a_{n} \mid a_{n-1}+a_{n+1}$ and $b_{n} \mid b_{n-1}+b_{n+1}$ for all $n$. + +Proof of necessity. It is clear that $a_{0}=b_{0}$. Then $a_{0} f(1,1)-a_{1} b_{1}=1$, i.e. $a_{0} \mid a_{1} b_{1}+1$. +Now, focus on six entries $f(x, y)$ with $x \in\{n-1, n, n+1\}$ and $y \in\{0,1\}$. Let $f(n-1,1)=u, f(n, 1)=v$, and $f(n+1,1)=w$, so + +$$ +\begin{aligned} +v a_{n-1}-u a_{n} & =1 \\ +w a_{n}-v a_{n+1} & =1 +\end{aligned} +$$ + +Then + +$$ +u+w=\frac{v\left(a_{n-1}+a_{n+1}\right)}{a_{n}} +$$ + +and from above $\operatorname{gcd}\left(v, a_{n}\right)=1$, so $a_{n} \mid a_{n-1}+a_{n+1}$; similarly for $b_{n}$. (If $a_{n}=0$, we have $v a_{n-1}=1$ and $v a_{n+1}=-1$, so this is OK.) + +Proof of sufficiency. Now consider two sequences $a_{0}, a_{1}, \ldots$ and $b_{0}, b_{1}, \ldots$ satisfying our criteria. We build a great function $f$ by induction on $(x, y)$. More strongly, we will assume as part of the inductive hypothesis that any two adjacent entries of $f$ are relatively prime and that for any three consecutive entries horizontally or vertically, the middle one divides the sum of the other two. + +First we set $f(1,1)$ so that $a_{0} f(1,1)=a_{1} b_{1}+1$, which is possible. +Consider an uninitialized $f(s, t)$; without loss of generality suppose $s \geq 2$. Then we know five values of $f$ and wish to set a sixth one $z$, as in the matrix below: + +$$ +\begin{array}{cc} +u & x \\ +v & y \\ +w & z +\end{array} +$$ + +(We imagine $a$-indices to increase southwards and $b$-indices to increase eastwards.) If $v \neq 0$, then the choice $y \cdot \frac{u+w}{v}-x$ works as $u y-v x=1$. If $v=0$, it easily follows that $\{u, w\}=\{1,-1\}$ and $y=w$ as $y w=1$. Then we set the uninitialized entry to anything. + +Now we verify that this is compatible with the inductive hypothesis. From the determinant 1 condition, it easily follows that $\operatorname{gcd}(w, z)=\operatorname{gcd}(v, z)=1$. The proof that $y \mid x+z$ is almost identical to a step performed in the "necessary" part of the lemma and we do not repeat it here. By induction, a desired great function $f$ exists. + +We complete the solution. Let $A, B, C$, and $D$ be integer sequences for which $(A, B)$, $(B, C)$, and $(C, D)$ are great. Then $a_{0}=b_{0}=c_{0}=d_{0}$, and each term in each sequence (after the zeroth term) divides the sum of its neighbors. Since $a_{0}$ divides all three of $a_{1} b_{1}+1, b_{1} c_{1}+1$, and $c_{1} d_{1}+1$, it follows $a_{0}$ divides $d_{1} a_{1}+1$, and thus $(D, A)$ is great as desired. + +Remark. To simplify the problem, we may restrict the codomain of great functions and elements in great pairs of sequences to $\mathbb{Z}_{>0}$. This allows the parts of the solution dealing with zero entries to be ignored. + +Remark. Of course, this solution also shows that any odd path (in the graph induced by the great relation on sequences) completes to an even cycle. If we stipulate that great functions must have $f(0,0)= \pm 1$, then even paths complete to cycles as well. Alternatively, we could change the great functional equation to + +$$ +f(x+1, y+1) f(x, y)-f(x+1, y) f(x, y+1)=-1 +$$ + +A quick counterexample to transitivity of $\sim$ as is without the condition $f(0,0)=1$, for concreteness: let $a_{n}=c_{n}=3+n$ and $b_{n}=3+2 n$ for $n \geq 0$. + +## §2.2 USA TST 2019/5, proposed by Yannick Yao + +Available online at https://aops.com/community/p11625809. + +## Problem statement + +Let $n$ be a positive integer. Tasty and Stacy are given a circular necklace with $3 n$ sapphire beads and $3 n$ turquoise beads, such that no three consecutive beads have the same color. They play a cooperative game where they alternate turns removing three consecutive beads, subject to the following conditions: + +- Tasty must remove three consecutive beads which are turquoise, sapphire, and turquoise, in that order, on each of his turns. +- Stacy must remove three consecutive beads which are sapphire, turquoise, and sapphire, in that order, on each of her turns. +They win if all the beads are removed in $2 n$ turns. Prove that if they can win with Tasty going first, they can also win with Stacy going first. + +In the necklace, we draw a divider between any two beads of the same color. Unless there are no dividers, this divides the necklace into several zigzags in which the beads in each zigzag alternate. Each zigzag has two endpoints (adjacent to dividers). + +Observe that the condition about not having three consecutive matching beads is equivalent to saying there are no zigzags of lengths 1. +![](https://cdn.mathpix.com/cropped/2024_11_19_a0a7a76bd95576fab5b9g-17.jpg?height=712&width=715&top_left_y=1426&top_left_x=676) + +The main claim is that the game is winnable (for either player going first) if and only if there are at most $2 n$ dividers. We prove this in two parts, the first part not using the hypothesis about three consecutive letters. + +Claim - The game cannot be won with Tasty going first if there are more than $2 n$ dividers. + +Proof. We claim each move removes at most one divider, which proves the result. +Consider removing a TST in some zigzag (necessarily of length at least 3). We illustrate the three possibilities in the following table, with Tasty's move shown in red. + +| Before | After | Change | +| :---: | :---: | :---: | +| . ST \| TST | TS . | ST \| TS . | One less divider; two zigzags merge | +| . . ST \| TSTST . | STST | One less divider; two zigzags merge | +| .STSTS . . | ..S\|S... | One more divider; a zigzag splits in t | + +The analysis for Stacy's move is identical. + +Claim - If there are at most $2 n$ dividers and there are no zigzags of length 1 then the game can be won (with either player going first). + +Proof. By symmetry it is enough to prove Tasty wins going first. +At any point if there are no dividers at all, then the necklace alternates $T S T S T \ldots$ and the game can be won. So we will prove that on each of Tasty's turns, if there exists at least one divider, then Tasty and Stacy can each make a move at an endpoint of some zigzag (i.e. the first two cases above). As we saw in the previous proof, such moves will (a) decrease the number of dividers by exactly one, (b) not introduce any singleton zigzags (because the old zigzags merge, rather than split). Since there are fewer than $2 n$ dividers, our duo can eliminate all dividers and then win. + +Note that as the number of $S$ and $T$ 's are equal, there must be an equal number of + +- zigzags of odd length ( $\geq 3$ ) with $T$ at the endpoints (i.e. one more $T$ than $S$ ), and +- zigzags of odd length $(\geq 3)$ with $S$ at the endpoints (i.e. one more $S$ than $T$ ). + +Now iff there is at least one of each, then Tasty removes a TST from the end of such a zigzag while Stacy removes an $S T S$ from the end of such a zigzag. + +Otherwise suppose all zigzags have even size. Then Tasty finds any zigzag of length $\geq 4$ (which must exist since the average zigzag length is 3 ) and removes TST from the end containing $T$. The resulting merged zigzag is odd and hence $S$ endpoints, hence Stacy can move as well. + +Remark. There are many equivalent ways to phrase the solution. For example, the number of dividers is equal to the number of pairs of two consecutive letters (rather than singleton letters). So the win condition can also be phrased in terms of the number of adjacent pairs of letters being at least $2 n$, or equivalently the number of differing pairs being at least $4 n$. + +If one thinks about the game as a process, this is a natural "monovariant" to consider anyways, so the solution is not so unmotivated. + +Remark. The constraint of no three consecutive identical beads is actually needed: a counterexample without this constraint is TTSTSTSTTSSS. (They win if Tasty goes first and lose if Stacy goes first.) + +Remark (Why induction is unlikely to work). Many contestants attempted induction. However, in doing so they often implicitly proved a different problem: "prove that if they can win with Tasty going first without ever creating a triplet, they can also win in such a way with Stacy going first". This essentially means nearly all induction attempts fail. + +Amusingly, even the modified problem (which is much more amenable to induction) sill seems difficult without some sort of global argument. Consider a position in which Tasty wins going first, with the sequence of winning moves being Tasty's first move in red below +and Stacy's second move in blue below: +![](https://cdn.mathpix.com/cropped/2024_11_19_a0a7a76bd95576fab5b9g-19.jpg?height=144&width=449&top_left_y=319&top_left_x=826) + +There is no "nearby" STS that Stacy can remove instead on her first turn, without introducing a triple- $T$ and also preventing Tasty from taking a TST. So it does not seem possible to easily change a Tasty-first winning sequence to a Stacy-first one, even in the modified version. + +## §2.3 USA TST 2019/6, proposed by Ankan Bhattachrya + +Available online at https://aops.com/community/p11625814. + +## Problem statement + +Let $A B C$ be a triangle with incenter $I$, and let $D$ be a point on line $B C$ satisfying $\angle A I D=90^{\circ}$. Let the excircle of triangle $A B C$ opposite the vertex $A$ be tangent to $\overline{B C}$ at point $A_{1}$. Define points $B_{1}$ on $\overline{C A}$ and $C_{1}$ on $\overline{A B}$ analogously, using the excircles opposite $B$ and $C$, respectively. +Prove that if quadrilateral $A B_{1} A_{1} C_{1}$ is cyclic, then $\overline{A D}$ is tangent to the circumcircle of $\triangle D B_{1} C_{1}$. + +We present two solutions. +【 First solution using spiral similarity (Ankan Bhattacharya). First, we prove the part of the problem which does not depend on the condition $A B_{1} A_{1} C_{1}$ is cyclic. + +## Lemma + +Let $A B C$ be a triangle and define $I, D, B_{1}, C_{1}$ as in the problem. Moreover, let $M$ denote the midpoint of $\overline{A D}$. Then $\overline{A D}$ is tangent to $\left(A B_{1} C_{1}\right)$, and moreover $\overline{B_{1} C_{1}} \| \overline{I M}$. + +Proof. Let $E$ and $F$ be the tangency points of the incircle. Denote by $Z$ the Miquel point of $B F E C$, i.e. the second intersection of the circle with diameter $\overline{A I}$ and the circumcircle. + +Note that $A, Z, D$ are collinear, by radical axis on $(A B C),(A F I E),(B I C)$. +![](https://cdn.mathpix.com/cropped/2024_11_19_a0a7a76bd95576fab5b9g-20.jpg?height=849&width=1109&top_left_y=1583&top_left_x=476) + +Then the spiral similarity gives us + +$$ +\frac{Z F}{Z E}=\frac{B F}{C E}=\frac{A C_{1}}{A B_{1}} +$$ + +which together with $\measuredangle F Z E=\measuredangle F A E=\measuredangle B A C$ implies that $\triangle Z F E$ and $\triangle A C_{1} B_{1}$ are (directly) similar. (See IMO Shortlist 2006 G9 for a similar application of spiral similarity.) + +Now the remainder of the proof is just angle chasing. First, since + +$$ +\measuredangle D A C_{1}=\measuredangle Z A F=\measuredangle Z E F=\measuredangle A B_{1} C_{1} +$$ + +we have $\overline{A D}$ is tangent to $\left(A B_{1} C_{1}\right)$. Moreover, to see that $\overline{I M} \| \overline{B_{1} C_{1}}$, write + +$$ +\begin{aligned} +\measuredangle\left(\overline{A I}, \overline{B_{1} C_{1}}\right) & =\measuredangle I A C+\measuredangle A B_{1} C_{1}=\measuredangle B A I+\measuredangle Z E F=\measuredangle F A I+\measuredangle Z A F \\ +& =\measuredangle Z A I=\measuredangle M A I=\measuredangle A I M +\end{aligned} +$$ + +the last step since $\triangle A I D$ is right with hypotenuse $\overline{A D}$, and median $\overline{I M}$. +Now we return to the present problem with the additional condition. +![](https://cdn.mathpix.com/cropped/2024_11_19_a0a7a76bd95576fab5b9g-21.jpg?height=523&width=1366&top_left_y=955&top_left_x=331) + +Claim - Given the condition, we actually have $\angle A B_{1} A_{1}=\angle A C_{1} A_{1}=90^{\circ}$. + +Proof. Let $I_{A}, I_{B}$ and $I_{C}$ be the excenters of $\triangle A B C$. Then the perpendiculars to $\overline{B C}$, $\overline{C A}, \overline{A B}$ from $A_{1}, B_{1}, C_{1}$ respectively meet at the so-called Bevan point $V$ (which is the circumcenter of $\left.\triangle I_{A} I_{B} I_{C}\right)$. + +Now $\triangle A B_{1} C_{1}$ has circumdiameter $\overline{A V}$. We are given $A_{1}$ lies on this circle, so if $V \neq A_{1}$ then $\overline{A A_{1}} \perp \overline{A_{1} V}$. But $\overline{A_{1} V} \perp \overline{B C}$ by definition, which would imply $\overline{A A_{1}} \| \overline{B C}$, which is absurd. + +Claim - Given the condition the points $B_{1}, I, C_{1}$ are collinear (hence with $M$ ). +Proof. By Pappus theorem on $\overline{I_{B} A I_{C}}$ and $\overline{B A_{1} C}$ after the previous claim. +To finish, since $\overline{D M A}$ was tangent to the circumcircle of $\triangle A B_{1} C_{1}$, we have $M D^{2}=$ $M A^{2}=M C_{1} \cdot M B_{1}$, implying the required tangency. + +Remark. The triangles satisfying the problem hypothesis are exactly the ones satisfying $r_{A}=2 R$, where $R$ and $r_{A}$ denote the circumradius and $A$-exradius. + +Remark. If $P$ is the foot of the $A$-altitude then this should also imply $A B_{1} P C_{1}$ is harmonic. + +【 Second solution by inversion and mixtilinears (Anant Mudgal). As in the end of the preceding solution, we have $\angle A B_{1} A_{1}=\angle A C_{1} A_{1}=90^{\circ}$ and $I \in \overline{B_{1} C_{1}}$. Let $M$ be the midpoint of minor arc $B C$ and $N$ be the midpoint of $\operatorname{arc} \widehat{B A C}$. Let $L$ be the intouch point on $\overline{B C}$. Let $O$ be the circumcenter of $\triangle A B C$. Let $K=\overline{A I} \cap \overline{B C}$. +![](https://cdn.mathpix.com/cropped/2024_11_19_a0a7a76bd95576fab5b9g-22.jpg?height=803&width=1232&top_left_y=475&top_left_x=409) + +Claim — We have $\angle\left(\overline{A I}, \overline{B_{1} C_{1}}\right)=\angle I A D$. + +Proof. Let $Z$ lie on $(A B C)$ with $\angle A Z I=90^{\circ}$. By radical axis theorem on $(A I Z),(B I C)$, and $(A B C)$, we conclude that $D$ lies on $\overline{A Z}$. Let $\overline{N I}$ meet $(A B C)$ again at $T \neq N$. + +Inversion in $(B I C)$ maps $\overline{A I}$ to $\overline{K I}$ and $(A B C)$ to $\overline{B C}$. Thus, $Z$ maps to $L$, so $Z, L, M$ are collinear. Since $B L=C V$ and $O I=O V$, we see that MLIN is a trapezoid with $\overline{I L} \| \overline{M N}$. Thus, $\overline{Z T} \| \overline{M N}$. + +It is known that $\overline{A T}$ and $\overline{A A_{1}}$ are isogonal in angle $B A C$. Since $\overline{A V}$ is a circumdiameter in $\left(A B_{1} C_{1}\right)$, so $\overline{A T} \perp \overline{B_{1} C_{1}}$. So $\measuredangle Z A I=\measuredangle N M T=90^{\circ}-\measuredangle T A I=\measuredangle\left(\overline{A I}, \overline{B_{1} C_{1}}\right)$. + +Let $X$ be the midpoint of $\overline{A D}$ and $G$ be the reflection of $I$ in $X$. Since $A I D G$ is a rectangle, we have $\measuredangle A I G=\measuredangle Z A I=\measuredangle\left(\overline{A I}, \overline{B_{1} C_{1}}\right)$, by the previous claim. So $\overline{I G}$ coincides with $\overline{B_{1} C_{1}}$. Now $\overline{A I}$ bisects $\angle B_{1} A C_{1}$ and $\angle I A G=90^{\circ}$, so $\left(\overline{I G} ; \overline{B_{1} C_{1}}\right)=-1$. + +Since $\angle I D G=90^{\circ}$, we see that $\overline{D I}$ and $\overline{D G}$ are bisectors of angle $B_{1} D C_{1}$. Now $\angle X D I=\angle X I D \Longrightarrow \angle X D C_{1}=\angle X I D-\angle I D B_{1}=\angle D B_{1} C_{1}$, so $\overline{X D}$ is tangent to $\left(D B_{1} C_{1}\right)$. + diff --git a/USA_TST/md/en-sols-TST-IMO-2020.md b/USA_TST/md/en-sols-TST-IMO-2020.md new file mode 100644 index 0000000000000000000000000000000000000000..be7094d0ee133c64215acdd99c469d265186b46a --- /dev/null +++ b/USA_TST/md/en-sols-TST-IMO-2020.md @@ -0,0 +1,483 @@ +# USA IMO TST 2020 Solutions
United States of America - IMO Team Selection Tests
Ankan Bhattacharya and Evan Chen
$61^{\text {th }}$ IMO 2020 Russia + +## Contents + +0 Problems +1 Solutions to Day 1 ..... 3 +1.1 USA TST 2020/1, proposed by Carl Schildkraut, Milan Haiman ..... 3 +1.2 USA TST 2020/2, proposed by Merlijn Staps ..... 5 +1.3 USA TST 2020/3, proposed by Nikolai Beluhov ..... 8 +2 Solutions to Day 2 ..... 12 +2.1 USA TST 2020/4, proposed by Zack Chroman, Mehtaab Sawhney ..... 12 +2.2 USA TST 2020/5, proposed by Carl Schildkraut ..... 14 +2.3 USA TST 2020/6, proposed by Michael Ren ..... 16 + +## §0 Problems + +1. Choose positive integers $b_{1}, b_{2}, \ldots$ satisfying + +$$ +1=\frac{b_{1}}{1^{2}}>\frac{b_{2}}{2^{2}}>\frac{b_{3}}{3^{2}}>\frac{b_{4}}{4^{2}}>\cdots +$$ + +and let $r$ denote the largest real number satisfying $\frac{b_{n}}{n^{2}} \geq r$ for all positive integers $n$. What are the possible values of $r$ across all possible choices of the sequence $\left(b_{n}\right)$ ? +2. Two circles $\Gamma_{1}$ and $\Gamma_{2}$ have common external tangents $\ell_{1}$ and $\ell_{2}$ meeting at $T$. Suppose $\ell_{1}$ touches $\Gamma_{1}$ at $A$ and $\ell_{2}$ touches $\Gamma_{2}$ at $B$. A circle $\Omega$ through $A$ and $B$ intersects $\Gamma_{1}$ again at $C$ and $\Gamma_{2}$ again at $D$, such that quadrilateral $A B C D$ is convex. + +Suppose lines $A C$ and $B D$ meet at point $X$, while lines $A D$ and $B C$ meet at point $Y$. Show that $T, X, Y$ are collinear. +3. Let $\alpha \geq 1$ be a real number. Hephaestus and Poseidon play a turn-based game on an infinite grid of unit squares. Before the game starts, Poseidon chooses a finite number of cells to be flooded. Hephaestus is building a levee, which is a subset of unit edges of the grid, called walls, forming a connected, non-self-intersecting path or loop. + +The game then begins with Hephaestus moving first. On each of Hephaestus's turns, he adds one or more walls to the levee, as long as the total length of the levee is at most $\alpha n$ after his $n$th turn. On each of Poseidon's turns, every cell which is adjacent to an already flooded cell and with no wall between them becomes flooded as well. + +Hephaestus wins if the levee forms a closed loop such that all flooded cells are contained in the interior of the loop - hence stopping the flood and saving the world. For which $\alpha$ can Hephaestus guarantee victory in a finite number of turns no matter how Poseidon chooses the initial cells to flood? +4. For a finite simple graph $G$, we define $G^{\prime}$ to be the graph on the same vertex set as $G$, where for any two vertices $u \neq v$, the pair $\{u, v\}$ is an edge of $G^{\prime}$ if and only if $u$ and $v$ have a common neighbor in $G$. Prove that if $G$ is a finite simple graph which is isomorphic to $\left(G^{\prime}\right)^{\prime}$, then $G$ is also isomorphic to $G^{\prime}$. +5. Find all integers $n \geq 2$ for which there exists an integer $m$ and a polynomial $P(x)$ with integer coefficients satisfying the following three conditions: + +- $m>1$ and $\operatorname{gcd}(m, n)=1$; +- the numbers $P(0), P^{2}(0), \ldots, P^{m-1}(0)$ are not divisible by $n$; and +- $P^{m}(0)$ is divisible by $n$. + +Here $P^{k}$ means $P$ applied $k$ times, so $P^{1}(0)=P(0), P^{2}(0)=P(P(0))$, etc. +6. Let $P_{1} P_{2} \ldots P_{100}$ be a cyclic 100-gon, and let $P_{i}=P_{i+100}$ for all $i$. Define $Q_{i}$ as the intersection of diagonals $\overline{P_{i-2} P_{i+1}}$ and $\overline{P_{i-1} P_{i+2}}$ for all integers $i$. +Suppose there exists a point $P$ satisfying $\overline{P P_{i}} \perp \overline{P_{i-1} P_{i+1}}$ for all integers $i$. Prove that the points $Q_{1}, Q_{2}, \ldots, Q_{100}$ are concyclic. + +## §1 Solutions to Day 1 + +## §1.1 USA TST 2020/1, proposed by Carl Schildkraut, Milan Haiman + +Available online at https://aops.com/community/p13654466. + +## Problem statement + +Choose positive integers $b_{1}, b_{2}, \ldots$ satisfying + +$$ +1=\frac{b_{1}}{1^{2}}>\frac{b_{2}}{2^{2}}>\frac{b_{3}}{3^{2}}>\frac{b_{4}}{4^{2}}>\cdots +$$ + +and let $r$ denote the largest real number satisfying $\frac{b_{n}}{n^{2}} \geq r$ for all positive integers $n$. What are the possible values of $r$ across all possible choices of the sequence $\left(b_{n}\right)$ ? + +The answer is $0 \leq r \leq 1 / 2$. Obviously $r \geq 0$. +In one direction, we show that +Claim (Greedy bound) - For all integers $n$, we have + +$$ +\frac{b_{n}}{n^{2}} \leq \frac{1}{2}+\frac{1}{2 n} +$$ + +Proof. This is by induction on $n$. For $n=1$ it is given. For the inductive step we have + +$$ +\begin{aligned} +b_{n} & \frac{\left[r(n+1)^{2}+(n+1)\right]+1}{(n+1)^{2}}>\frac{b_{n+1}}{(n+1)^{2}} +$$ + +where the middle inequality is true since it rearranges to $\frac{1}{n}>\frac{n+2}{(n+1)^{2}}$. + +## §1.2 USA TST 2020/2, proposed by Merlijn Staps + +Available online at https://aops.com/community/p13654481. + +## Problem statement + +Two circles $\Gamma_{1}$ and $\Gamma_{2}$ have common external tangents $\ell_{1}$ and $\ell_{2}$ meeting at $T$. Suppose $\ell_{1}$ touches $\Gamma_{1}$ at $A$ and $\ell_{2}$ touches $\Gamma_{2}$ at $B$. A circle $\Omega$ through $A$ and $B$ intersects $\Gamma_{1}$ again at $C$ and $\Gamma_{2}$ again at $D$, such that quadrilateral $A B C D$ is convex. +Suppose lines $A C$ and $B D$ meet at point $X$, while lines $A D$ and $B C$ meet at point $Y$. Show that $T, X, Y$ are collinear. + +We present four solutions. +ब First solution, elementary (original). We have $\triangle Y A C \sim \triangle Y B D$, from which it follows + +$$ +\frac{d(Y, A C)}{d(Y, B D)}=\frac{A C}{B D} +$$ + +Moreover, if we denote by $r_{1}$ and $r_{2}$ the radii of $\Gamma_{1}$ and $\Gamma_{2}$, then + +$$ +\frac{d(T, A C)}{d(T, B D)}=\frac{T A \sin \angle\left(A C, \ell_{1}\right)}{T B \sin \angle\left(B D, \ell_{2}\right)}=\frac{2 r_{1} \sin \angle\left(A C, \ell_{1}\right)}{2 r_{2} \sin \angle\left(B D, \ell_{2}\right)}=\frac{A C}{B D} +$$ + +the last step by the law of sines. +![](https://cdn.mathpix.com/cropped/2024_11_19_1d87ca6c42af65f32daag-05.jpg?height=721&width=1269&top_left_y=1447&top_left_x=396) + +This solves the problem up to configuration issues: we claim that $Y$ and $T$ both lie inside $\angle A X B \equiv \angle C X D$. WLOG $T A2$ then Hephaestus wins, but when $\alpha=2$ (and hence $\alpha \leq 2$ ) Hephaestus cannot contain even a single-cell flood initially. + +Strategy for $\alpha>2$ : Impose $\mathbb{Z}^{2}$ coordinates on the cells. Adding more flooded cells does not make our task easier, so let us assume that initially the cells $(x, y)$ with $|x|+|y| \leq d$ are flooded for some $d \geq 2$; thus on Hephaestus's $k$ th turn, the water is contained in $|x|+|y| \leq d+k-1$. Our goal is to contain the flood with a large rectangle. + +We pick large integers $N_{1}$ and $N_{2}$ such that + +$$ +\begin{gathered} +\alpha N_{1}>2 N_{1}+(2 d+3) \\ +\alpha\left(N_{1}+N_{2}\right)>2 N_{2}+\left(6 N_{1}+8 d+4\right) +\end{gathered} +$$ + +Mark the points $X_{i}, Y_{i}$ as shown in the figure for $1 \leq i \leq 6$. The red figures indicate the distance between the marked points on the rectangle. +![](https://cdn.mathpix.com/cropped/2024_11_19_1d87ca6c42af65f32daag-09.jpg?height=1004&width=983&top_left_y=246&top_left_x=542) + +We follow the following plan. + +- Turn 1: place wall $X_{1} Y_{1}$. This cuts off the flood to the north. +- Turns 2 through $N_{1}+1$ : extend the levee to segment $X_{2} Y_{2}$. This prevents further flooding to the north. +- Turn $N_{1}+2$ : add in broken lines $X_{4} X_{3} X_{2}$ and $Y_{4} Y_{3} Y_{2}$ all at once. This cuts off the flood west and east. +- Turns $N_{1}+2$ to $N_{1}+N_{2}+1$ : extend the levee along segments $X_{4} X_{5}$ and $Y_{4} Y_{5}$. This prevents further flooding west and east. +- Turn $N_{1}+N_{2}+2$ : add in the broken line $X_{5} X_{6} Y_{6} Y_{5}$ all at once and win. + +Proof for $\alpha=2$ : Suppose Hephaestus contains the flood on his $(n+1)$ st turn. We prove that $\alpha>2$ by showing that in fact at least $2 n+4$ walls have been constructed. + +Let $c_{0}, c_{1}, \ldots, c_{n}$ be a path of cells such that $c_{0}$ is the initial cell flooded, and in general $c_{i}$ is flooded on Poseidon's $i$ th turn from $c_{i-1}$. The levee now forms a closed loop enclosing all $c_{i}$. + +Claim - If $c_{i}$ and $c_{j}$ are adjacent then $|i-j|=1$. + +Proof. Assume $c_{i}$ and $c_{j}$ are adjacent but $|i-j|>1$. Then the two cells must be separated by a wall. But the levee forms a closed loop, and now $c_{i}$ and $c_{j}$ are on opposite sides. + +Thus the $c_{i}$ actually form a path. We color green any edge of the unit grid (wall or not) which is an edge of exactly one $c_{i}$ (i.e. the boundary of the polyomino). It is easy to see there are exactly $2 n+4$ green edges. + +Now, from the center of each cell $c_{i}$, shine a laser towards each green edge of $c_{i}$ (hence a total of $2 n+4$ lasers are emitted). An example below is shown for $n=6$, with the levee marked in brown. +![](https://cdn.mathpix.com/cropped/2024_11_19_1d87ca6c42af65f32daag-10.jpg?height=803&width=806&top_left_y=415&top_left_x=631) + +Claim - No wall is hit by more than one laser. +Proof. Assume for contradiction that a wall $w$ is hit by lasers from $c_{i}$ and $c_{j}$. WLOG that laser is vertical, so $c_{i}$ and $c_{j}$ are in the same column (e.g. $(i, j)=(0,5)$ in figure). + +We consider two cases on the position of $w$. + +- If $w$ is between $c_{i}$ and $c_{j}$, then we have found a segment intersecting the levee exactly once. But the endpoints of the segment lie inside the levee. This contradicts the assumption that the levee is a closed loop. +- Suppose $w$ lies above both $c_{i}$ and $c_{j}$ and assume WLOG $i2$. + +Remark (Author comments). The author provides the following remarks. + +- Even though the flood can be stopped when $\alpha=2+\varepsilon$, it takes a very long time to do that. Starting from a single flooded cell, the strategy I have outlined requires $\Theta\left(1 / \varepsilon^{2}\right)$ days. Starting from several flooded cells contained within an area of diameter $D$, it takes $\Theta\left(D / \varepsilon^{2}\right)$ days. I do not know any strategies that require fewer days than that. +- There is a gaping chasm between $\alpha \leq 2$ and $\alpha>2$. Since $\alpha \leq 2$ does not suffice even when only one cell is flooded in the beginning, there are in fact no initial +configurations at all for which it is sufficient. On the other hand, $\alpha>2$ works for all initial configurations. +- The second half of the solution essentially estimates the perimeter of a polyomino in terms of its diameter (where diameter is measured entirely within the polyomino). +It appears that this has not been done before, or at least I was unable to find any reference for it. I did find tons of references where the perimeter of a polyomino is estimated in terms of its area, but nothing concerning the diameter. +My argument is a formalisation of the intuition that if $P$ is any shortest path within some weirdly-shaped polyomino, then the boundary of that polyomino must hug $P$ rather closely so that $P$ cannot be shortened. + + +## §2 Solutions to Day 2 + +## §2.1 USA TST 2020/4, proposed by Zack Chroman, Mehtaab Sawhney + +Available online at https://aops.com/community/p13913804. + +## Problem statement + +For a finite simple graph $G$, we define $G^{\prime}$ to be the graph on the same vertex set as $G$, where for any two vertices $u \neq v$, the pair $\{u, v\}$ is an edge of $G^{\prime}$ if and only if $u$ and $v$ have a common neighbor in $G$. Prove that if $G$ is a finite simple graph which is isomorphic to $\left(G^{\prime}\right)^{\prime}$, then $G$ is also isomorphic to $G^{\prime}$. + +We say a vertex of a graph is fatal if it has degree at least 3, and some two of its neighbors are not adjacent. + +Claim - The graph $G^{\prime}$ has at least as many triangles as $G$, and has strictly more if $G$ has any fatal vertices. + +Proof. Obviously any triangle in $G$ persists in $G^{\prime}$. Moreover, suppose $v$ is a fatal vertex of $G$. Then the neighbors of $G$ will form a clique in $G^{\prime}$ which was not there already, so there are more triangles. + +Thus we only need to consider graphs $G$ with no fatal vertices. Looking at the connected components, the only possibilities are cliques (including single vertices), cycles, and paths. So in what follows we restrict our attention to graphs $G$ only consisting of such components. + +Remark (Warning). Beware: assuming $G$ is connected loses generality. For example, it could be that $G=G_{1} \sqcup G_{2}$, where $G_{1}^{\prime} \cong G_{2}$ and $G_{2}^{\prime} \cong G_{1}$. + +First, note that the following are stable under the operation: + +- an isolated vertex, +- a cycle of odd length, or +- a clique with at least three vertices. + +In particular, $G \cong G^{\prime \prime}$ holds for such graphs. +On the other hand, cycles of even length or paths of nonzero length will break into more connected components. For this reason, a graph $G$ with any of these components will not satisfy $G \cong G^{\prime \prime}$ because $G^{\prime}$ will have strictly more connected components than $G$, and $G^{\prime \prime}$ will have at least as many as $G^{\prime}$. + +Therefore $G \cong G^{\prime \prime}$ if and only if $G$ is a disjoint union of the three types of connected components named earlier. Since $G \cong G^{\prime}$ holds for such graphs as well, the problem statement follows right away. + +Remark. Note that the same proof works equally well for an arbitrary number of iterations $G^{\prime \prime} \ldots{ }^{\prime \prime} \cong G$, rather than just $G^{\prime \prime} \cong G$. + +Remark. The proposers included a variant of the problem where given any graph $G$, the operation stabilized after at most $O(\log n)$ operations, where $n$ was the number of vertices of $G$. + +## §2.2 USA TST 2020/5, proposed by Carl Schildkraut + +Available online at https://aops.com/community/p13913769. + +## Problem statement + +Find all integers $n \geq 2$ for which there exists an integer $m$ and a polynomial $P(x)$ with integer coefficients satisfying the following three conditions: + +- $m>1$ and $\operatorname{gcd}(m, n)=1$; +- the numbers $P(0), P^{2}(0), \ldots, P^{m-1}(0)$ are not divisible by $n$; and +- $P^{m}(0)$ is divisible by $n$. + +Here $P^{k}$ means $P$ applied $k$ times, so $P^{1}(0)=P(0), P^{2}(0)=P(P(0))$, etc. + +The answer is that this is possible if and only if there exists primes $p^{\prime}0 \mid P^{e}(0) \equiv 0 \bmod N\right\} +$$ + +where we set $\min \varnothing=0$ by convention. Note that in general we have + +$$ +\operatorname{zord}(P \bmod N)=\operatorname{lcm}_{q \mid N}(\operatorname{zord}(P \bmod q)) +$$ + +where the index runs over all prime powers $q \mid N$ (by Chinese remainder theorem). This will be used in both directions. + +Construction: First, we begin by giving a construction. The idea is to first use the following prime power case. + +Claim - Let $p^{e}$ be a prime power, and $1 \leq k United States of America - IMO Team Selection Test
Andrew Gu, Ankan Bhattacharya and Evan Chen
$62^{\text {th }}$ IMO 2021 Russia + +## Contents + +0 Problems ..... 2 +1 Solutions to Day 1 ..... 3 +1.1 USA TST 2021/1, proposed by Ankan Bhattacharya, Michael Ren ..... 3 +1.2 USA TST 2021/2, proposed by Andrew Gu, Frank Han ..... 4 +1.3 USA TST 2021/3, proposed by Gabriel Carroll ..... 7 + +## §0 Problems + +1. Determine all integers $s \geq 4$ for which there exist positive integers $a, b, c, d$ such that $s=a+b+c+d$ and $s$ divides $a b c+a b d+a c d+b c d$. +2. Points $A, V_{1}, V_{2}, B, U_{2}, U_{1}$ lie fixed on a circle $\Gamma$, in that order, and such that $B U_{2}>A U_{1}>B V_{2}>A V_{1}$. +Let $X$ be a variable point on the $\operatorname{arc} V_{1} V_{2}$ of $\Gamma$ not containing $A$ or $B$. Line $X A$ meets line $U_{1} V_{1}$ at $C$, while line $X B$ meets line $U_{2} V_{2}$ at $D$. +Prove there exists a fixed point $K$, independent of $X$, such that the power of $K$ to the circumcircle of $\triangle X C D$ is constant. +3. Find all functions $f: \mathbb{R} \rightarrow \mathbb{R}$ that satisfy the inequality + +$$ +f(y)-\left(\frac{z-y}{z-x} f(x)+\frac{y-x}{z-x} f(z)\right) \leq f\left(\frac{x+z}{2}\right)-\frac{f(x)+f(z)}{2} +$$ + +for all real numbers $xs$, contradiction. + +## §1.2 USA TST 2021/2, proposed by Andrew Gu, Frank Han + +Available online at https://aops.com/community/p20672623. + +## Problem statement + +Points $A, V_{1}, V_{2}, B, U_{2}, U_{1}$ lie fixed on a circle $\Gamma$, in that order, and such that $B U_{2}>A U_{1}>B V_{2}>A V_{1}$. + +Let $X$ be a variable point on the $\operatorname{arc} V_{1} V_{2}$ of $\Gamma$ not containing $A$ or $B$. Line $X A$ meets line $U_{1} V_{1}$ at $C$, while line $X B$ meets line $U_{2} V_{2}$ at $D$. + +Prove there exists a fixed point $K$, independent of $X$, such that the power of $K$ to the circumcircle of $\triangle X C D$ is constant. + +For brevity, we let $\ell_{i}$ denote line $U_{i} V_{i}$ for $i=1,2$. +We first give an explicit description of the fixed point $K$. Let $E$ and $F$ be points on $\Gamma$ such that $\overline{A E} \| \ell_{1}$ and $\overline{B F} \| \ell_{2}$. The problem conditions imply that $E$ lies between $U_{1}$ and $A$ while $F$ lies between $U_{2}$ and $B$. Then we let + +$$ +K=\overline{A F} \cap \overline{B E} +$$ + +This point exists because $A E F B$ are the vertices of a convex quadrilateral. +Remark (How to identify the fixed point). If we drop the condition that $X$ lies on the arc, then the choice above is motivated by choosing $X \in\{E, F\}$. Essentially, when one chooses $X \rightarrow E$, the point $C$ approaches an infinity point. So in this degenerate case, the only points whose power is finite to $(X C D)$ are bounded are those on line $B E$. The same logic shows that $K$ must lie on line $A F$. Therefore, if the problem is going to work, the fixed point must be exactly $\overline{A F} \cap \overline{B E}$. + +We give two possible approaches for proving the power of $K$ with respect to $(X C D)$ is fixed. + +【 First approach by Vincent Huang. We need the following claim: +Claim - Suppose distinct lines $A C$ and $B D$ meet at $X$. Then for any point $K$ + +$$ +\operatorname{pow}(K, X A B)+\operatorname{pow}(K, X C D)=\operatorname{pow}(K, X A D)+\operatorname{pow}(K, X B C) +$$ + +Proof. The difference between the left-hand side and right-hand side is a linear function in $K$, which vanishes at all of $A, B, C, D$. + +Construct the points $P=\ell_{1} \cap \overline{B E}$ and $Q=\ell_{2} \cap \overline{A F}$, which do not depend on $X$. +Claim - Quadrilaterals $B P C X$ and $A Q D X$ are cyclic. +Proof. By Reim's theorem: $\measuredangle C P B=\measuredangle A E B=\measuredangle A X B=\measuredangle C X B$, etc. +![](https://cdn.mathpix.com/cropped/2024_11_19_9733e686be42a6dd888ag-5.jpg?height=803&width=792&top_left_y=244&top_left_x=638) + +Now, for the particular $K$ we choose, we have + +$$ +\begin{aligned} +\operatorname{pow}(K, X C D) & =\operatorname{pow}(K, X A D)+\operatorname{pow}(K, X B C)-\operatorname{pow}(K, X A B) \\ +& =K A \cdot K Q+K B \cdot K P-\operatorname{pow}(K, \Gamma) . +\end{aligned} +$$ + +This is fixed, so the proof is completed. +【 Second approach by authors. Let $Y$ be the second intersection of $(X C D)$ with $\Gamma$. Let $S=\overline{E Y} \cap \ell_{1}$ and $T=\overline{F Y} \cap \ell_{2}$. + +Claim - Points $S$ and $T$ lies on ( $X C D)$ as well. + +Proof. By Reim's theorem: $\measuredangle C S Y=\measuredangle A E Y=\measuredangle A X Y=\measuredangle C X Y$, etc. +Now let $X^{\prime}$ be any other choice of $X$, and define $C^{\prime}$ and $D^{\prime}$ in the obvious way. We are going to show that $K$ lies on the radical axis of $(X C D)$ and $\left(X^{\prime} C^{\prime} D^{\prime}\right)$. +![](https://cdn.mathpix.com/cropped/2024_11_19_9733e686be42a6dd888ag-5.jpg?height=798&width=798&top_left_y=1828&top_left_x=629) + +The main idea is as follows: +Claim - The point $L=\overline{E Y} \cap \overline{A X^{\prime}}$ lies on the radical axis. By symmetry, so does the point $M=\overline{F Y} \cap \overline{B X^{\prime}}$ (not pictured). + +Proof. Again by Reim's theorem, $S C^{\prime} Y X^{\prime}$ is cyclic. Hence we have + +$$ +\operatorname{pow}\left(L, X^{\prime} C^{\prime} D^{\prime}\right)=L C^{\prime} \cdot L X^{\prime}=L S \cdot L Y=\operatorname{pow}(L, X C D) +$$ + +To conclude, note that by Pascal theorem on + +$$ +E Y F A X^{\prime} B +$$ + +it follows $K, L, M$ are collinear, as needed. +Remark. All the conditions about $U_{1}, V_{1}, U_{2}, V_{2}$ at the beginning are there to eliminate configuration issues, making the problem less obnoxious to the contestant. + +In particular, without the various assumptions, there exist configurations in which the point $K$ is at infinity. In these cases, the center of $X C D$ moves along a fixed line. + +## §1.3 USA TST 2021/3, proposed by Gabriel Carroll + +Available online at https://aops.com/community/p20672681. + +## Problem statement + +Find all functions $f: \mathbb{R} \rightarrow \mathbb{R}$ that satisfy the inequality + +$$ +f(y)-\left(\frac{z-y}{z-x} f(x)+\frac{y-x}{z-x} f(z)\right) \leq f\left(\frac{x+z}{2}\right)-\frac{f(x)+f(z)}{2} +$$ + +for all real numbers $x\max \{z-y, y-x\}$. Since $f$ has a supergradient $\alpha$ at $y$ over the interval $(y-\Delta, y+\Delta)$, and this interval includes $x$ and $z$, we have + +$$ +\begin{aligned} +\frac{z-y}{z-x} f(x)+\frac{y-x}{z-x} f(z) & \leq \frac{z-y}{z-x}(f(y)+\alpha(x-y))+\frac{y-x}{z-x}(f(y)+\alpha(z-y)) \\ +& =f(y) +\end{aligned} +$$ + +That is, $f$ is a concave function. Continuity follows from the fact that any concave function on $\mathbb{R}$ is automatically continuous. + +Lemma (see e.g. https://math.stackexchange.com/a/615161 for picture) +Any concave function $f$ on $\mathbb{R}$ is continuous. + +Proof. Suppose we wish to prove continuity at $p \in \mathbb{R}$. Choose any real numbers $a$ and $b$ with $a0$, let's define the function + +$$ +g(t)=\frac{f(y)-f(y-t)}{t}-\frac{f(y+t)-f(y)}{t} . +$$ + +We contend that $g(\varepsilon) \leq \frac{3}{5} g(3 \varepsilon)$ for any $\varepsilon>0$. Indeed by the problem condition, + +$$ +\begin{aligned} +& f(y) \leq f(y-\varepsilon)+\frac{f(y+\varepsilon)-f(y-3 \varepsilon)}{4} \\ +& f(y) \leq f(y+\varepsilon)-\frac{f(y+3 \varepsilon)-f(y-\varepsilon)}{4} . +\end{aligned} +$$ + +![](https://cdn.mathpix.com/cropped/2024_11_19_9733e686be42a6dd888ag-8.jpg?height=223&width=609&top_left_y=1699&top_left_x=1029) + +Summing gives the desired conclusion. +Now suppose that $f$ has two supergradients $\alpha<\alpha^{\prime}$ at point $y$. For small enough $\varepsilon$, we should have we have $f(y-\varepsilon) \leq f(y)-\alpha^{\prime} \varepsilon$ and $f(y+\varepsilon) \leq f(y)+\alpha \varepsilon$, hence + +$$ +g(\varepsilon)=\frac{f(y)-f(y-\varepsilon)}{\varepsilon}-\frac{f(y+\varepsilon)-f(y)}{\varepsilon} \geq \alpha^{\prime}-\alpha . +$$ + +This is impossible since $g(\varepsilon)$ may be arbitrarily small. + +Claim - The function $f$ is quadratic on the rational numbers. +Proof. Consider any four-term arithmetic progression $x, x+d, x+2 d, x+3 d$. Because $(f(x+2 d)-f(x+d)) / d$ and $(f(x+3 d)-f(x)) / 3 d$ are both supergradients of $f$ at the point $x+3 d / 2$, they must be equal, hence + +$$ +f(x+3 d)-3 f(x+2 d)+3 f(x+d)-f(x)=0 . +$$ + +If we fix $d=1 / n$, it follows inductively that $f$ agrees with a quadratic function $\widetilde{f}_{n}$ on the set $\frac{1}{n} \mathbb{Z}$. On the other hand, for any $m \neq n$, we apparently have $\widetilde{f}_{n}=\widetilde{f}_{m n}=\widetilde{f}_{m}$, so the quadratic functions on each "layer" are all equal. + +Since $f$ is continuous, it follows $f$ is quadratic, as needed. +Remark (Alternate finish using differentiability due to Michael Ren). In the proof of the main claim (about uniqueness of supergradients), we can actually notice the two terms $\frac{f(y)-f(y-t)}{t}$ and $\frac{f(y+t)-f(y)}{t}$ in the definition of $g(t)$ are both monotonic (by concavity). Since we supplied a proof that $\lim _{t \rightarrow 0} g(t)=0$, we find $f$ is differentiable. + +Now, if the derivative at some point exists, it must coincide with all the supergradients; (informally, this is why "tangent line trick" always has the slope as the derivative, and formally, we use the mean value theorem). In other words, we must have + +$$ +f(x+y)-f(x-y)=2 f^{\prime}(x) \cdot y +$$ + +holds for all real numbers $x$ and $y$. By choosing $y=1$ we obtain that $f^{\prime}(x)=f(x+1)-f(x-1)$ which means $f^{\prime}$ is also continuous. + +Finally differentiating both sides with respect to $y$ gives + +$$ +f^{\prime}(x+y)-f^{\prime}(x-y)=2 f^{\prime}(x) +$$ + +which means $f^{\prime}$ obeys Jensen's functional equation. Since $f^{\prime}$ is continuous, this means $f^{\prime}$ is linear. Thus $f$ is quadratic, as needed. + diff --git a/USA_TST/md/en-sols-TST-IMO-2023.md b/USA_TST/md/en-sols-TST-IMO-2023.md new file mode 100644 index 0000000000000000000000000000000000000000..cbda8bccf94294e77a5167d468fe58f1b6be24d4 --- /dev/null +++ b/USA_TST/md/en-sols-TST-IMO-2023.md @@ -0,0 +1,705 @@ +USA TST 2023 Solutions
United States of America - Team Selection Test
Andrew Gu, Evan Chen, and Gopal Goel
$64^{\text {rd }}$ IMO 2022 Japan and $12^{\text {th }}$ EGMO 2023 Slovenia + +# Contents + +0 Problems ..... 2 +1 Solutions to Day 1 ..... 3 +1.1 USA TST 2023/1, proposed by Kevin Cong ..... 3 +1.2 USA TST 2023/2, proposed by Kevin Cong ..... 5 +1.3 USA TST 2023/3, proposed by Sean Li ..... 12 +2 Solutions to Day 2 ..... 15 +2.1 USA TST 2023/4, proposed by Carl Schildkraut ..... 15 +2.2 USA TST 2023/5, proposed by Nikolai Beluhov ..... 16 +2.3 USA TST 2023/6, proposed by Maxim Li ..... 20 + +## §0 Problems + +1. There are 2022 equally spaced points on a circular track $\gamma$ of circumference 2022. The points are labeled $A_{1}, A_{2}, \ldots, A_{2022}$ in some order, each label used once. Initially, Bunbun the Bunny begins at $A_{1}$. She hops along $\gamma$ from $A_{1}$ to $A_{2}$, then from $A_{2}$ to $A_{3}$, until she reaches $A_{2022}$, after which she hops back to $A_{1}$. When hopping from $P$ to $Q$, she always hops along the shorter of the two $\operatorname{arcs} \overparen{P Q}$ of $\gamma$; if $\overline{P Q}$ is a diameter of $\gamma$, she moves along either semicircle. +Determine the maximal possible sum of the lengths of the 2022 arcs which Bunbun traveled, over all possible labellings of the 2022 points. +2. Let $A B C$ be an acute triangle. Let $M$ be the midpoint of side $B C$, and let $E$ and $F$ be the feet of the altitudes from $B$ and $C$, respectively. Suppose that the common external tangents to the circumcircles of triangles $B M E$ and $C M F$ intersect at a point $K$, and that $K$ lies on the circumcircle of $A B C$. Prove that line $A K$ is perpendicular to line $B C$. +3. Consider pairs $(f, g)$ of functions from the set of nonnegative integers to itself such that + +- $f(0) \geq f(1) \geq f(2) \geq \cdots \geq f(300) \geq 0 ;$ +- $f(0)+f(1)+f(2)+\cdots+f(300) \leq 300$; +- for any 20 nonnegative integers $n_{1}, n_{2}, \ldots, n_{20}$, not necessarily distinct, we have + +$$ +g\left(n_{1}+n_{2}+\cdots+n_{20}\right) \leq f\left(n_{1}\right)+f\left(n_{2}\right)+\cdots+f\left(n_{20}\right) . +$$ + +Determine the maximum possible value of $g(0)+g(1)+\cdots+g(6000)$ over all such pairs of functions. +4. For nonnegative integers $a$ and $b$, denote their bitwise xor by $a \oplus b$. (For example, $9 \oplus 10=1001_{2} \oplus 1010_{2}=0011_{2}=3$.) +Find all positive integers $a$ such that for any integers $x>y \geq 0$, we have + +$$ +x \oplus a x \neq y \oplus a y . +$$ + +5. Let $m$ and $n$ be fixed positive integers. Tsvety and Freyja play a game on an infinite grid of unit square cells. Tsvety has secretly written a real number inside of each cell so that the sum of the numbers within every rectangle of size either $m \times n$ or $n \times m$ is zero. Freyja wants to learn all of these numbers. +One by one, Freyja asks Tsvety about some cell in the grid, and Tsvety truthfully reveals what number is written in it. Freyja wins if, at any point, Freyja can simultaneously deduce the number written in every cell of the entire infinite grid. (If this never occurs, Freyja has lost the game and Tsvety wins.) +In terms of $m$ and $n$, find the smallest number of questions that Freyja must ask to win, or show that no finite number of questions can suffice. +6. Fix a function $f: \mathbb{N} \rightarrow \mathbb{N}$ and for any $m, n \in \mathbb{N}$ define + +$$ +\Delta(m, n)=\underbrace{f(f(\ldots f}_{f(n) \text { times }}(m) \ldots))-\underbrace{f(f(\ldots f}_{f(m) \text { times }}(n) \ldots)) . +$$ + +Suppose $\Delta(m, n) \neq 0$ for any distinct $m, n \in \mathbb{N}$. Show that $\Delta$ is unbounded, meaning that for any constant $C$ there exist $m, n \in \mathbb{N}$ with $|\Delta(m, n)|>C$. + +## §1 Solutions to Day 1 + +## §1.1 USA TST 2023/1, proposed by Kevin Cong + +Available online at https://aops.com/community/p26685816. + +## Problem statement + +There are 2022 equally spaced points on a circular track $\gamma$ of circumference 2022. The points are labeled $A_{1}, A_{2}, \ldots, A_{2022}$ in some order, each label used once. Initially, Bunbun the Bunny begins at $A_{1}$. She hops along $\gamma$ from $A_{1}$ to $A_{2}$, then from $A_{2}$ to $A_{3}$, until she reaches $A_{2022}$, after which she hops back to $A_{1}$. When hopping from $P$ to $Q$, she always hops along the shorter of the two $\operatorname{arcs} \overparen{P Q}$ of $\gamma$; if $\overline{P Q}$ is a diameter of $\gamma$, she moves along either semicircle. + +Determine the maximal possible sum of the lengths of the 2022 arcs which Bunbun traveled, over all possible labellings of the 2022 points. + +Replacing 2022 with $2 n$, the answer is $2 n^{2}-2 n+2$. (When $n=1011$, the number is 2042222.) +![](https://cdn.mathpix.com/cropped/2024_11_19_5c8ecd58885c530bab97g-03.jpg?height=402&width=899&top_left_y=1202&top_left_x=586) + +【 Construction. The construction for $n=5$ shown on the left half of the figure easily generalizes for all $n$. + +Remark. The validity of this construction can also be seen from the below proof. + +ब First proof of bound. Let $d_{i}$ be the shorter distance from $A_{2 i-1}$ to $A_{2 i+1}$. +Claim - The distance of the leg of the journey $A_{2 i-1} \rightarrow A_{2 i} \rightarrow A_{2 i+1}$ is at most $2 n-d_{i}$. + +Proof. Of the two arcs from $A_{2 i-1}$ to $A_{2 i+1}$, Bunbun will travel either $d_{i}$ or $2 n-d_{i}$. One of those arcs contains $A_{2 i}$ along the way. So we get a bound of $\max \left(d_{i}, 2 n-d_{i}\right)=2 n-d_{i}$. + +That means the total distance is at most + +$$ +\sum_{i=1}^{n}\left(2 n-d_{i}\right)=2 n^{2}-\left(d_{1}+d_{2}+\cdots+d_{n}\right) +$$ + +Claim - We have + +$$ +d_{1}+d_{2}+\cdots+d_{n} \geq 2 n-2 +$$ + +Proof. The left-hand side is the sum of the walk $A_{1} \rightarrow A_{3} \rightarrow \cdots \rightarrow A_{2 n-1} \rightarrow A_{1}$. Among the $n$ points here, two of them must have distance at least $n-1$ apart; the other $d_{i}$ 's contribute at least 1 each. So the bound is $(n-1)+(n-1) \cdot 1=2 n-2$. + +Second proof of bound. Draw the $n$ diameters through the $2 n$ arc midpoints, as shown on the right half of the figure for $n=5$ in red. + +Claim (Interpretation of distances) - The distance between any two points equals the number of diameters crossed to travel between the points. + +Proof. Clear. +With this in mind, call a diameter critical if it is crossed by all $2 n$ arcs. +Claim - At most one diameter is critical. +Proof. Suppose there were two critical diameters; these divide the circle into four arcs. Then all $2 n$ arcs cross both diameters, and so travel between opposite arcs. But this means that points in two of the four arcs are never accessed - contradiction. + +Claim - Every diameter is crossed an even number of times. + +Proof. Clear: the diameter needs to be crossed an even number of times for the loop to return to its origin. + +This immediately implies that the maximum possible total distance is achieved when one diameter is crossed all $2 n$ times, and every other diameter is crossed $2 n-2$ times, for a total distance of at most + +$$ +n \cdot(2 n-2)+2=2 n^{2}-2 n+2 +$$ + +## §1.2 USA TST 2023/2, proposed by Kevin Cong + +Available online at https://aops.com/community/p26685484. + +## Problem statement + +Let $A B C$ be an acute triangle. Let $M$ be the midpoint of side $B C$, and let $E$ and $F$ be the feet of the altitudes from $B$ and $C$, respectively. Suppose that the common external tangents to the circumcircles of triangles $B M E$ and $C M F$ intersect at a point $K$, and that $K$ lies on the circumcircle of $A B C$. Prove that line $A K$ is perpendicular to line $B C$. + +We present several distinct approaches. +\I Inversion solution submitted by Ankan Bhattacharya and Nikolai Beluhov. Let $H$ be the orthocenter of $\triangle A B C$. We use inversion in the circle with diameter $\overline{B C}$. We identify a few images: + +- The circumcircles of $\triangle B M E$ and $\triangle C M F$ are mapped to lines $B E$ and $C F$. +- The common external tangents are mapped to the two circles through $M$ which are tangent to lines $B E$ and $C F$. +- The image of $K$, denoted $K^{*}$, is the second intersection of these circles. +- The assertion that $K$ lies on $(A B C)$ is equivalent to $K^{*}$ lying on $(B H C)$. + +However, now $K^{*}$ is simple to identify directly: it's just the reflection of $M$ in the bisector of $\angle B H C$. +![](https://cdn.mathpix.com/cropped/2024_11_19_5c8ecd58885c530bab97g-06.jpg?height=1148&width=1218&top_left_y=237&top_left_x=419) + +In particular, $\overline{H K^{*}}$ is a symmedian of $\triangle B H C$. However, since $K^{*}$ lies on $(B H C)$, this means $\left(H K^{*} ; B C\right)=-1$. + +Then, we obtain that $\overline{B C}$ bisects $\angle H M K^{*} \equiv \angle H M K$. However, $K$ also lies on $(A B C)$, which forces $K$ to be the reflection of $H$ in $\overline{B C}$. Thus $\overline{A K} \perp \overline{B C}$, as wanted. + +ๆ Solution with coaxial circles (Pitchayut Saengrungkongka). Let $H$ be the orthocenter of $\triangle A B C$. Let $Q$ be the second intersection of $\odot(B M E)$ and $\odot(C M F)$. We first prove the following well-known properties of $Q$. + +Claim - $Q$ is the Miquel point of $B C E F$. In particular, $Q$ lies on both $\odot(A E F)$ and $\odot(A B C)$. + +Proof. Follows since $B C E F$ is cyclic with $M$ being the circumcenter. + +Claim - $A(Q, H ; B, C)=-1$. + +Proof. By the radical center theorem on $\odot(A E F), \odot(A B C)$, and $\odot(B C E F)$, we get that $A Q, E F$, and $B C$ are concurrent. Now, the result follows from a well-known harmonic property. + +Now, we get to the meat of the solution. Let the circumcircle of $\odot(Q M K)$ meet $B C$ again at $T \neq M$. The key claim is the following. + +Claim - $Q T$ is tangent to $\odot(B Q C)$. + +Proof. We use the "forgotten coaxiality lemma". + +$$ +\begin{aligned} +\frac{B T}{T C} & =\frac{T B \cdot T M}{T C \cdot T M} \\ +& =\frac{\operatorname{pow}(T, \odot(B M E))}{\operatorname{pow}(T, \odot(C M F))} \\ +& =\frac{\operatorname{pow}(K, \odot(B M E))}{\operatorname{pow}(K, \odot(C M F))} \\ +& =\left(\frac{r_{\odot(B M E)}}{r_{\odot(C M F)}}\right)^{2} \\ +& =\left(\frac{B Q / \sin \angle Q M B}{C Q / \sin \angle Q M C}\right)^{2} \\ +& =\frac{B Q^{2}}{C Q^{2}} +\end{aligned} +$$ + +implying the result. +To finish, let $O$ be the center of $\odot(A B C)$. Then, from the claim, $\angle O Q T=90^{\circ}=$ $\angle O M T$, so $O$ also lies on $\odot(Q M T K)$. Thus, $\angle O K T=90^{\circ}$, so $K T$ is also tangent to $\odot(A B C)$ as well. This implies that $Q B K C$ is harmonic quadrilateral, and the result follows from the second claim. + +【 Solution by Luke Robitaille. Let $Q$ be the second intersection of $\odot(B M E)$ and $\odot(C M F)$. We use the first two claims of the previous solution. In particular, $Q \in$ $\odot(A B C)$. We have the following claim. + +Claim (Also appeared in ISL 2017 G7) - We have $\measuredangle Q K M=\measuredangle Q B M+\measuredangle Q C M$. + +Proof. Let $K Q$ and $K M$ meet $\odot(B M E)$ again at $Q^{\prime}$ and $M^{\prime}$. Then, by homothety, $\measuredangle Q^{\prime} Q M^{\prime}=\measuredangle Q C M$, so + +$$ +\begin{aligned} +\measuredangle Q K M & =\measuredangle Q^{\prime} Q M^{\prime}+\measuredangle Q M^{\prime} M \\ +& =\measuredangle Q C M+\measuredangle Q B M, +\end{aligned} +$$ + +as desired. +Now, we extend $K M$ to meet $\odot(A B C)$ again at $Q_{1}$. We have + +$$ +\begin{aligned} +\measuredangle Q_{1} Q B=\measuredangle Q_{1} K B & =\measuredangle Q_{1} K Q+\measuredangle Q C B \\ +& =\measuredangle M K Q+\measuredangle Q K B \\ +& =(\measuredangle M B Q+\measuredangle M C Q)+\measuredangle Q C B \\ +& =\measuredangle C B Q, +\end{aligned} +$$ + +implying that $Q Q_{1} \| B C$. This implies that $Q B K C$ is harmonic quadrilateral, so we are done. + +I Synthetic solution due to Andrew Gu (Harvard 2026). Define $O_{1}$ and $O_{2}$ as the circumcenters of $(B M E)$ and $(C M F)$. Let $T$ be the point on $(A B C)$ such that $\overline{A T} \perp \overline{B C}$. Denote by $L$ the midpoint of minor arc $\overparen{B C}$. + +We are going to ignore the condition that $K$ lies on the circumcircle of $A B C$, and prove the following unconditional result: + +## Proposition + +The points $T, L, K$ are collinear. + +This will solve the problem because if $K$ is on the circumcircle of $A B C$, it follows $K=T$ or $K=L$; but $K=L$ can never occur since $O_{1}$ and $O_{2}$ are obviously on different sides of line $L M$ so line $L M$ must meet $O_{1} O_{2}$ inside segment $O_{1} O_{2}$, and $K$ lies outside this segment. +![](https://cdn.mathpix.com/cropped/2024_11_19_5c8ecd58885c530bab97g-08.jpg?height=1243&width=1400&top_left_y=926&top_left_x=334) + +We now turn to the proof of the main lemma. Let $P_{1}$ and $P_{2}$ be the antipodes of $M$ on these circles. + +Claim - Lines $A C$ and $L M$ meet at the antipode $Q_{1}$ of $B$ on $(B M E)$, so that $B P_{1} Q_{1} M$ is a rectangle. Similarly, lines $A B$ and $L M$ meet at the antipode $Q_{2}$ of $C$ on $(C M F)$, so that $C P_{2} Q_{2} M$ is a rectangle. + +Proof. Let $Q_{1}^{\prime}=\omega_{1} \cap A C \neq E$. Then $\measuredangle B M Q_{1}^{\prime}=\measuredangle B E Q_{1}^{\prime}=90^{\circ}$ hence $Q_{1}^{\prime} \in L M$. The other half of the lemma follows similarly. + +From this, it follows that $P_{1} Q_{1}=B M=\frac{1}{2} B C=M C=P_{2} Q_{2}$. Letting $r_{1}$ denote the radius of $\omega_{1}$ (and similarly for $\omega_{2}$ ), we deduce that $C Q_{1}=B Q_{1}=2 r_{1}$. + +Claim - $K M=K L$. + +Proof. I first claim that $\overline{C L}$ is the external bisector of $\angle Q_{1} C Q_{2}$; this follows from + +$$ +\measuredangle Q_{1} C L=\measuredangle A C L=\measuredangle A B L=\measuredangle Q_{2} B L=\measuredangle Q_{2} C L +$$ + +The external angle bisector theorem then gives an equality of directed ratios + +$$ +\frac{L Q_{1}}{L Q_{2}}=\frac{\left|C Q_{1}\right|}{\left|C Q_{2}\right|}=\frac{\left|B Q_{1}\right|}{\left|C Q_{2}\right|}=\frac{2 r_{1}}{2 r_{2}}=\frac{r_{1}}{r_{2}} +$$ + +Let the reflection of $M$ over $K$ be $P$; then $P$ lies on $\overline{P_{1} P_{2}}$ and + +$$ +\frac{P P_{1}}{P P_{2}}=\frac{2 K O_{1}}{2 K O_{2}}=\frac{K O_{1}}{K O_{2}}=\frac{r_{1}}{r_{2}}=\frac{L Q_{1}}{L Q_{2}} +$$ + +where again the ratios are directed. Projecting everything onto line $L M$, so that $P_{1}$ lands at $Q_{1}$ and $P_{2}$ lands at $Q_{2}$, we find that the projection of $P$ must land exactly at $L$. + +Claim - Line $K M$ is an external angle bisector of $\angle O_{1} M O_{2}$. + +Proof. Because $\frac{K O_{1}}{K O_{2}}=\frac{r_{1}}{r_{2}}=\frac{M O_{1}}{M O_{2}}$. +To finish, note that we know that $\overline{M P_{1}} \| \overline{C Q_{1}} \equiv \overline{A C}$ and $\overline{M P_{2}} \| \overline{B Q_{2}} \equiv \overline{A B}$, meaning the angles $\angle O_{1} M O_{2}$ and $\angle C A B$ have parallel legs. Hence, if $N$ is the antipode of $L$, it follows that $\overline{M K} \| \overline{A N}$. Now from $M K=K L$ and the fact that $A N L T$ is an isosceles trapezoid, we deduce that $\overline{L T}$ and $\overline{L K}$ are lines in the same direction (namely, the reflection of $M K \| A N$ across $\overline{B C}$ ), as needed. + +【 Complex numbers approach with Apollonian circles, by Carl Schildkraut. We use complex numbers. As in the first approach, we will ignore the hypothesis that $K$ lies on ( $A B C$ ). + +Let $Q:=(A H) \cap(A B C) \cap(A E F) \neq A$ be the Miquel point of $B F E C$ again. Construct the point $T$ on $(A B C)$ for which $A T \perp B C$; note that $T=-\frac{b c}{a}$. This time the unconditional result is: + +## Proposition + +We have $Q, M, T, K$ are concyclic (or collinear) on an Apollonian circle of $\overline{O_{1} O_{2}}$. + +This will solve the original problem since once $K$ lies on $(A B C)$ it must be either $Q$ or $T$. But since $K$ is not on $(B M E), K \neq Q$, it will have to be $T$. + +We now prove the proposition. Suppose $(A B C)$ is the unit circle and let $A=a, B=b$, $C=c$. Let $H=a+b+c$ be the orthocenter of $\triangle A B C$. By the usual formulas, + +$$ +E:=\frac{1}{2}\left(a+b+c-\frac{b c}{a}\right) . +$$ + +Let $O_{1}$ be the center of $(B M E)$ and $O_{2}$ be the center of $(C M F)$. + +Claim (Calculation of the Miquel point) - We have $Q=\frac{2 a+b+c}{a\left(\frac{1}{a}+\frac{1}{b}+\frac{1}{c}\right)+1}$. +Proof. We now compute that $Q=q$ satisfies $\bar{q}=1 / q$ (since $Q$ is on the unit circle) and $\frac{q-h}{q-a} \in i \mathbb{R}$ (since $A Q \perp Q H$ ), which expands to + +$$ +0=\frac{q-h}{q-a}+\frac{1 / q-\bar{h}}{1 / q-1 / a}=\frac{q-h}{q-a}-\frac{a(1-q \bar{h})}{q-a} +$$ + +This solves to $q=\frac{h+a}{a \bar{h}+1}=\frac{2 a+b+c}{a \bar{h}+1}$. + +Claim (Calculation of $O_{1}$ and $\left.O_{2}\right)-$ We have $O_{1}=\frac{b(2 a+b+c)}{2(a+b)}$ and $O_{2}=\frac{c(2 a+b+c)}{2(a+c)}$. + +Proof. We now compute $O_{1}$ and $O_{2}$. For $x, y, z \in \mathbb{C}$, let $\operatorname{Circum}(x, y, z)$ denote the circumcenter of the triangle defined by vertices $x, y$, and $z$ in $\mathbb{C}$. We have + +$$ +\begin{aligned} +O_{1} & =\operatorname{Circum}(B, M, E) \\ +& =b+\frac{1}{2} \operatorname{Circum}\left(0, c-b, \frac{(a-b)(b-c)}{b}\right) \\ +& =b-\frac{b-c}{2 b} \operatorname{Circum}(0, b, b-a) \\ +& =b-\frac{b-c}{2 b}(b-\operatorname{Circum}(0, b, a)) \\ +& =b-\frac{b-c}{2 b}\left(b-\frac{a b}{a+b}\right)=b-\frac{b(b-c)}{2(a+b)}=\frac{b(2 a+b+c)}{2(a+b)} +\end{aligned} +$$ + +Similarly, $O_{2}=\frac{c(2 a+b+c)}{2(a+c)}$. +We are now going to prove the following: +Claim - We have + +$$ +\frac{T O_{1}}{T O_{2}}=\frac{M O_{1}}{M O_{2}}=\frac{Q O_{1}}{Q O_{2}} +$$ + +Proof. We now compute + +$$ +M O_{1}=B O_{1}=\left|b-\frac{b(2 a+b+c)}{2(a+b)}\right|=\left|\frac{b(b-c)}{2(a+b)}\right|=\frac{1}{2}\left|\frac{b-c}{a+b}\right| +$$ + +and +$Q O_{1}=\left|r-\frac{b(2 a+b+c)}{2(a+b)}\right|=\left|1-\frac{b(a+h)}{2(a+b) r}\right|=\left|1-\frac{b(a \bar{h}+1)}{2(a+b)}\right|=\left|\frac{a-\frac{a b}{c}}{2(a+b)}\right|=\frac{1}{2}\left|\frac{b-c}{a+b}\right|$. +This implies both (by symmetry) that $\frac{M O_{1}}{M O_{2}}=\frac{Q O_{1}}{Q O_{2}}=\left|\frac{a+c}{a+b}\right|$ and that $Q$ is on (BME) and $(C M F)$. Also, + +$$ +\frac{T O_{1}}{T O_{2}}=\frac{\left|\frac{b(2 a+b+c)}{2(a+b)}+\frac{b c}{a}\right|}{\left|\frac{c(2 a+b+c)}{2(a+c)}+\frac{b c}{a}\right|}=\left|\frac{\frac{b\left(2 a^{2}+a b+a c+2 a c+2 b c\right)}{2 a(a+b)}}{\frac{c\left(2 a^{2}+a b+a c+2 a b+2 b c\right)}{2 a(a+c)}}\right|=\left|\frac{a+c}{a+b}\right| \cdot\left|\frac{2 a^{2}+2 b c+a b+3 a c}{2 a^{2}+2 b c+3 a b+a c}\right| +$$ + +if $z=2 a^{2}+2 b c+a b+3 a c$, then $a^{2} b c \bar{z}=2 a^{2}+2 b c+3 a b+a c$, so the second term has magnitude 1. This means $\frac{T O_{1}}{T O_{2}}=\frac{M O_{1}}{M O_{2}}=\frac{Q O_{1}}{Q O_{2}}$, as desired. + +To finish, note that this common ratio is the ratio between the radii of these two circles, so it is also $\frac{K O_{1}}{K O_{2}}$. By Apollonian circles the points $\{Q, M, T, K\}$ lie on a circle or a line. + +## §1.3 USA TST 2023/3, proposed by Sean Li + +Available online at https://aops.com/community/p26685437. + +## Problem statement + +Consider pairs $(f, g)$ of functions from the set of nonnegative integers to itself such that + +- $f(0) \geq f(1) \geq f(2) \geq \cdots \geq f(300) \geq 0 ;$ +- $f(0)+f(1)+f(2)+\cdots+f(300) \leq 300 ;$ +- for any 20 nonnegative integers $n_{1}, n_{2}, \ldots, n_{20}$, not necessarily distinct, we have + +$$ +g\left(n_{1}+n_{2}+\cdots+n_{20}\right) \leq f\left(n_{1}\right)+f\left(n_{2}\right)+\cdots+f\left(n_{20}\right) +$$ + +Determine the maximum possible value of $g(0)+g(1)+\cdots+g(6000)$ over all such pairs of functions. + +Replace $300=\frac{24 \cdot 25}{2}$ with $\frac{s(s+1)}{2}$ where $s=24$, and 20 with $k$. The answer is $115440=$ $\frac{k s(k s+1)}{2}$. Equality is achieved at $f(n)=\max (s-n, 0)$ and $g(n)=\max (k s-n, 0)$. To prove + +$$ +g\left(n_{1}+\cdots+n_{k}\right) \leq f\left(n_{1}\right)+\cdots+f\left(n_{k}\right) +$$ + +write it as + +$$ +\max \left(x_{1}+\cdots+x_{k}, 0\right) \leq \max \left(x_{1}, 0\right)+\cdots+\max \left(x_{k}, 0\right) +$$ + +with $x_{i}=s-n_{i}$. This can be proven from the $k=2$ case and induction. +It remains to show the upper bound. For this problem, define a partition to be a nonincreasing function $p: \mathbb{Z}_{\geq 0} \rightarrow \mathbb{Z}_{\geq 0}$ such that $p(n)=0$ for some $n$. The sum of $p$ is defined to be $\sum_{n=0}^{\infty} p(n)$, which is finite under the previous assumption. Let $L=\mathbb{Z}_{\geq 0}^{2}$. The Young diagram of the partition is the set of points + +$$ +\mathcal{P}:=\{(x, y) \in L: yn \text {. } +$$ + +This is a partition with the same sum as $p$. Geometrically, the Young diagrams of $p$ and $p_{*}$ are reflections about $x=y$. + +Since each $g(n)$ is independent, we may maximize each one separately for all $n$ and assume that + +$$ +g(n)=\min _{n_{1}+\cdots+n_{k}=n}\left(f\left(n_{1}\right)+\cdots+f\left(n_{k}\right)\right) . +$$ + +The conditions of the problem statement imply that $f\left(\frac{s(s+1)}{2}\right)=0$. Then, for any $n \leq k \frac{s(s+1)}{2}$, there exists an optimal combination $\left(n_{1}, \ldots, n_{k}\right)$ in $\left(^{*}\right)$ where all $n_{i}$ are at most $\frac{s(s+1)}{2}$, by replacing any term in an optimum greater than $\frac{s(s+1)}{2}$ by $\frac{s(s+1)}{2}$ and shifting the excess to smaller terms (because $f$ is nonincreasing). Therefore we may extend $f$ to a partition by letting $f(n)=0$ for $n>\frac{s(s+1)}{2}$ without affecting the relevant values of $g$. Then $\left(^{*}\right)$ implies that $g$ is a partition as well. + +The problem can be restated as follows: $f$ is a partition with sum $\frac{s(s+1)}{2}$, and $g$ is a partition defined by $\left(^{*}\right)$. Find the maximum possible sum of $g$. The key claim is that the problem is the same under conjugation. + +Claim - Under these conditions, we have + +$$ +g_{*}(n)=\min _{n_{1}+\cdots+n_{k}=n}\left(f_{*}\left(n_{1}\right)+\cdots+f_{*}\left(n_{k}\right)\right) . +$$ + +Proof. Let $\mathcal{F}$ and $\mathcal{G}$ be the Young diagrams of $f$ and $g$ respectively, and $\overline{\mathcal{F}}=L \backslash \mathcal{F}$ and $\overline{\mathcal{G}}=L \backslash \mathcal{G}$ be their complements. The lower boundary of $\overline{\mathcal{F}}$ is formed by the points $(n, f(n))$ for $i \in \mathbb{Z}_{\geq 0}$. By the definition of $g$, the lower boundary of $\overline{\mathcal{G}}$ consists of points $(n, g(n))$ which are formed by adding $k$ points of $\overline{\mathcal{F}}$. This means + +$$ +\overline{\mathcal{G}}=\underbrace{\overline{\mathcal{F}}+\cdots+\overline{\mathcal{F}}}_{k \overline{\mathcal{F}}^{\prime} \mathrm{s}} +$$ + +where + denotes set addition. This definition remains invariant under reflection about $x=y$, which swaps $f$ and $g$ with their conjugates. + +Let $A$ be the sum of $g$. We now derive different bounds on $A$. First, by Hermite's identity + +$$ +n=\sum_{i=0}^{k-1}\left\lfloor\frac{n+i}{k}\right\rfloor +$$ + +we have + +$$ +\begin{aligned} +A & =\sum_{n=0}^{\infty} g(n) \\ +& \leq \sum_{n=0}^{\infty} \sum_{i=0}^{k-1} f\left(\left\lfloor\frac{n+i}{k}\right\rfloor\right) \\ +& =k^{2} \sum_{n=0}^{\infty} f(n)-\frac{k(k-1)}{2} f(0) \\ +& =k^{2} \frac{s(s+1)}{2}-\frac{k(k-1)}{2} f(0) . +\end{aligned} +$$ + +By the claim, we also get the second bound $A \leq k^{2} \frac{s(s+1)}{2}-\frac{k(k-1)}{2} f_{*}(0)$. +For the third bound, note that $f\left(f_{*}(0)\right)=0$ and thus $g\left(k f_{*}(0)\right)=0$. Moreover, + +$$ +g\left(q f_{*}(0)+r\right) \leq q \cdot f\left(f_{*}(0)\right)+(k-q-1) f(0)+f(r)=(k-q-1) f(0)+f(r), +$$ + +so we have + +$$ +\begin{aligned} +A & =\sum_{\substack{0 \leq qy \geq 0$, we have + +$$ +x \oplus a x \neq y \oplus a y . +$$ + +Answer: the function $x \mapsto x \oplus a x$ is injective if and only if $a$ is an even integer. +đ Even case. First, assume $\nu_{2}(a)=k>0$. We wish to recover $x$ from $c:=x \oplus a x$. Notice that: + +- The last $k$ bits of $c$ coincide with the last $k$ bits of $x$. +- Now the last $k$ bits of $x$ give us also the last $2 k$ bits of $a x$, so we may recover the last $2 k$ bits of $x$ as well. +- Then the last $2 k$ bits of $x$ give us also the last $3 k$ bits of $a x$, so we may recover the last $3 k$ bits of $x$ as well. +- ...and so on. +đ Odd case. Conversely, suppose $a$ is odd. To produce the desired collision: +Claim - Let $n$ be any integer such that $2^{n}>a$, and define + +$$ +x=\underbrace{1 \ldots 1}_{n}=2^{n}-1, \quad y=1 \underbrace{0 \ldots 0}_{n} 1=2^{n}+1 . +$$ + +Then $x \oplus a x=y \oplus a y$. + +Proof. Let $P$ be the binary string for $a$, zero-padded to length $n$, and let $Q$ be the binary string for $a-1$, zero-padded to length $n$, Then let $R$ be the bitwise complement of $Q$. (Hence all three of are binary strings of length $n$.) Then + +$$ +\begin{aligned} +& a x=\overline{Q R} \Longrightarrow x \oplus a x=\overline{Q Q} \\ +& a y=\overline{P P} \Longrightarrow y \oplus a y=\overline{Q Q} . +\end{aligned} +$$ + +We're done. + +## §2.2 USA TST 2023/5, proposed by Nikolai Beluhov + +Available online at https://aops.com/community/p26896130. + +## Problem statement + +Let $m$ and $n$ be fixed positive integers. Tsvety and Freyja play a game on an infinite grid of unit square cells. Tsvety has secretly written a real number inside of each cell so that the sum of the numbers within every rectangle of size either $m \times n$ or $n \times m$ is zero. Freyja wants to learn all of these numbers. + +One by one, Freyja asks Tsvety about some cell in the grid, and Tsvety truthfully reveals what number is written in it. Freyja wins if, at any point, Freyja can simultaneously deduce the number written in every cell of the entire infinite grid. (If this never occurs, Freyja has lost the game and Tsvety wins.) + +In terms of $m$ and $n$, find the smallest number of questions that Freyja must ask to win, or show that no finite number of questions can suffice. + +The answer is the following: + +- If $\operatorname{gcd}(m, n)>1$, then Freyja cannot win. +- If $\operatorname{gcd}(m, n)=1$, then Freyja can win in a minimum of $(m-1)^{2}+(n-1)^{2}$ questions. + +First, we dispose of the case where $\operatorname{gcd}(m, n)>1$. Write $d=\operatorname{gcd}(m, n)$. The idea is that any labeling where each $1 \times d$ rectangle has sum zero is valid. Thus, to learn the labeling, Freyja must ask at least one question in every row, which is clearly not possible in a finite number of questions. + +Now suppose $\operatorname{gcd}(m, n)=1$. We split the proof into two halves. +【 Lower bound. Clearly, any labeling where each $m \times 1$ and $1 \times m$ rectangle has sum zero is valid. These labelings form a vector space with dimension $(m-1)^{2}$, by inspection. (Set the values in an $(m-1) \times(m-1)$ square arbitrarily and every other value is uniquely determined.) + +Similarly, labelings where each $n \times 1$ and $1 \times n$ rectangle have sum zero are also valid, and have dimension $(n-1)^{2}$. + +It is also easy to see that no labeling other than the all-zero labeling belongs to both categories; labelings in the first space are periodic in both directions with period $m$, while labelings in the second space are periodic in both directions with period $n$; and hence any labeling in both categories must be constant, ergo all-zero. + +Taking sums of these labelings gives a space of valid labelings of dimension $(m-1)^{2}+$ $(n-1)^{2}$. Thus, Freyja needs at least $(m-1)^{2}+(n-1)^{2}$ questions to win. + +【 Proof of upper bound using generating functions, by Ankan Bhattacharya. We prove: + +Claim (Periodicity) - Any valid labeling is doubly periodic with period $m n$. +Proof. By Chicken McNugget, there exists $N$ such that $N$ and $N+1$ are both nonnegative integer linear combinations of $m$ and $n$. + +Then both $m n \times N$ and $m n \times(N+1)$ rectangles have zero sum, so $m n \times 1$ rectangles have zero sum. This implies that any two cells with a vertical displacement of $m n$ are equal; similarly for horizontal displacements. + +With that in mind, consider a valid labeling. It naturally corresponds to a generating function + +$$ +f(x, y)=\sum_{a=0}^{m n-1} \sum_{b=0}^{m n-1} c_{a, b} x^{a} y^{b} +$$ + +where $c_{a, b}$ is the number in $(a, b)$. +The generating function corresponding to sums over $n \times m$ rectangles is + +$$ +f(x, y)\left(1+x+\cdots+x^{m-1}\right)\left(1+y+\cdots+y^{n-1}\right)=f(x, y) \cdot \frac{x^{m}-1}{x-1} \cdot \frac{y^{n}-1}{y-1} . +$$ + +Similarly, the one for $m \times n$ rectangles is + +$$ +f(x, y) \cdot \frac{x^{n}-1}{x-1} \cdot \frac{y^{m}-1}{y-1} . +$$ + +Thus, the constraints for $f$ to be valid are equivalent to + +$$ +f(x, y) \cdot \frac{x^{m}-1}{x-1} \cdot \frac{y^{n}-1}{y-1} \quad \text { and } \quad f(x, y) \cdot \frac{x^{n}-1}{x-1} \cdot \frac{y^{m}-1}{y-1} +$$ + +being zero when reduced modulo $x^{m n}-1$ and $y^{m n}-1$, or, letting $\omega=\exp (2 \pi i / m n)$, both terms being zero when powers of $\omega$ are plugged in. + +To restate the constraints one final time, we need + +$$ +f\left(\omega^{a}, \omega^{b}\right) \cdot \frac{\omega^{a m}-1}{\omega^{a}-1} \cdot \frac{\omega^{b n}-1}{\omega^{b}-1}=f\left(\omega^{a}, \omega^{b}\right) \cdot \frac{\omega^{a n}-1}{\omega^{a}-1} \cdot \frac{\omega^{b m}-1}{\omega^{b}-1}=0 +$$ + +for all $a, b \in\{0, \ldots, m n-1\}$. +Claim - This implies that $f\left(\omega^{a}, \omega^{b}\right)=0$ for all but at most $(m-1)^{2}+(n-1)^{2}$ values of $(a, b) \in\{0, \ldots, m n-1\}^{2}$. + +Proof. Consider a pair $(a, b)$ such that $f\left(\omega^{a}, \omega^{b}\right) \neq 0$. Then we need + +$$ +\frac{\omega^{a m}-1}{\omega^{a}-1} \cdot \frac{\omega^{b n}-1}{\omega^{b}-1}=\frac{\omega^{a n}-1}{\omega^{a}-1} \cdot \frac{\omega^{b m}-1}{\omega^{b}-1}=0 . +$$ + +This happens when (at least) one fraction in either product is zero. + +- If the first fraction is zero, then either $n \mid a$ and $a>0$, or $m \mid b$ and $b>0$. +- If the second fraction is zero, then either $m \mid a$ and $a>0$, or $n \mid b$ and $b>0$. + +If the first condition holds in both cases, then $m n \mid a$, but $0C$. + +Suppose for the sake of contradiction that $|\Delta(m, n)| \leq N$ for all $m, n$. Note that $f$ is injective, as + +$$ +f(m)=f(n) \Longrightarrow \Delta(m, n)=0 \Longrightarrow m=n +$$ + +as desired. +Let $G$ be the "arrow graph" of $f$, which is the directed graph with vertex set $\mathbb{N}$ and edges $n \rightarrow f(n)$. The first step in the solution is to classify the structure of $G$. Injectivity implies that $G$ is a disjoint collection of chains (infinite and half-infinite) and cycles. We have the following sequence of claims that further refine the structure. + +Claim - The graph $G$ has no cycles. +Proof. Suppose for the sake of contradiction that $f^{k}(n)=n$ for some $k \geq 2$ and $n \in \mathbb{N}$. As $m$ varies over $\mathbb{N}$, we have $|\Delta(m, n)| \leq N$, so $f^{f(n)}(m)$ can only take on some finite set of values. In particular, this means that + +$$ +f^{f(n)}\left(m_{1}\right)=f^{f(n)}\left(m_{2}\right) +$$ + +for some $m_{1} \neq m_{2}$, which contradicts injectivity. +Claim - The graph $G$ has at most $2 N+1$ chains. +Proof. Suppose we have numbers $m_{1}, \ldots, m_{k}$ in distinct chains. Select a positive integer $B>\max \left\{f\left(m_{1}\right), \ldots, f\left(m_{k}\right)\right\}$. Now, + +$$ +\left|\Delta\left(m_{i}, f^{B-f\left(m_{i}\right)}(1)\right)\right| \leq N \Longrightarrow\left|f^{B}(1)-f^{f B-f\left(m_{i}\right)+1}(1)\left(m_{i}\right)\right| \leq N . +$$ + +Since the $m_{i} \mathrm{~S}$ are in different chains, we have that $f^{f\left(-f\left(m_{i}\right)+1\right.}(1)\left(m_{i}\right)$ are distinct for each $i$, which implies that $k \leq 2 N+1$, as desired. + +Claim - The graph $G$ consists of exactly one half-infinite chain. +Proof. Fix some $c \in \mathbb{N}$. Call an element of $\mathbb{N}$ bad if it is not of the form $f^{k}(c)$ for some $k \geq 0$. It suffices to show that there are only finitely many bad numbers. +Since there are only finitely many chains, $f^{f(c)}(n)$ achieves all sufficiently large positive integers, say all positive integers at least $M$. Fix $A$ and $B$ such that $B>A \geq M$. + +If $f^{f(c)}(n) \in[A, B]$, then $f^{f(n)}(c) \in[A-N, B+N]$, and distinct $n$ generate distinct $f^{f(n)}(c)$ due to the structure of $G$. Therefore, we have at least $B-A+1$ good numbers in $[A-N, B+N]$, so there are at most $2 N$ bad numbers in $[A-N, B+N]$. + +Varying $B$, this shows there are at most $2 N$ bad numbers at least $A-N$. +Let $c$ be the starting point of the chain, so every integer is of the form $f^{k}(c)$, where $k \geq 0$. Define a function $g: \mathbb{Z}_{\geq 0} \rightarrow \mathbb{N}$ by + +$$ +g(k):=f^{k}(c) +$$ + +Due to the structure of $G, g$ is a bijection. Define + +$$ +\delta(a, b):=\Delta\left(f^{a}(c), f^{b}(c)\right)=g(g(b+1)+a)-g(g(a+1)+b) +$$ + +so the conditions are equivalent to $|\delta(a, b)| \leq N$ for all $a, b \in \mathbb{Z}_{\geq 0}$ and $\delta(a, b) \neq 0$ for $a \neq b$, which is equivalent to $g(a+1)-a \neq g(b+1)-b$ for $a \neq b$. This tells us that $g(x)-x$ is injective for $x \geq 1$. + +## Lemma + +For all $M$, there exists a nonnegative integer $x$ with $g(x) \leq x-M$. + +Proof. Assume for the sake of contradiction that $g(x)-x$ is bounded below. Fix some large positive $K$. Since $g(x)-x$ is injective, there exists $B$ such that $g(x)-x \geq K$ for all $x \geq B$. Then $\min \{g(B+1), g(B+2), \ldots\} \geq B+K$, while $\{g(0), \ldots, g(B)\}$ only achieve $B+1$ values. Thus, at least $K-1$ values are not achieved by $g$, which is a contradiction. + +Now pick $B$ such that $g(B)+N \leq B$ and $g(B)>N$. Note that infinitely many such $B$ exist, since we can take $M$ to be arbitrarily small in the above lemma. Let + +$$ +t=\max \left\{g^{-1}(g(B)-N), g^{-1}(g(B)-N+1), \ldots, g^{-1}(g(B)+N)\right\} +$$ + +Note that $g(t) \leq g(B)+N \leq B$, so we have + +$$ +|\delta(t-1, B-g(t))|=|g(B)-g(t-1+g(B+1-g(t)))| \leq N +$$ + +so + +$$ +t-1+g(B+1-g(t)) \in\left\{g^{-1}(g(B)-N), g^{-1}(g(B)-N+1), \ldots, g^{-1}(g(B)+N)\right\} +$$ + +so by the maximality of $t$, we must have $g(B+1-g(t))=1$, so $B+1-g(t)=g^{-1}(1)$. We have $|g(t)-g(B)| \leq N$, so + +$$ +\left|(B-g(B))+1-g^{-1}(1)\right| \leq N . +$$ + +This is true for infinitely many values of $B$, so infinitely many values of $B-g(B)$ (by injectivity of $g(x)-x)$, which is a contradiction. This completes the proof. + diff --git a/USA_TST/raw/en-sols-TST-IMO-2014.pdf b/USA_TST/raw/en-sols-TST-IMO-2014.pdf new file mode 100644 index 0000000000000000000000000000000000000000..9ec679152ca507cc282de30745131d50fd3e5122 --- /dev/null +++ b/USA_TST/raw/en-sols-TST-IMO-2014.pdf @@ -0,0 +1,3 @@ +version https://git-lfs.github.com/spec/v1 +oid sha256:aa29a9eacba45c6681018e030a2db801f37f1007cb4b1827a5c19735f79150b1 +size 197293 diff --git a/USA_TST/raw/en-sols-TST-IMO-2015.pdf b/USA_TST/raw/en-sols-TST-IMO-2015.pdf new file mode 100644 index 0000000000000000000000000000000000000000..d22e995d51f54c4ea5ab40b35e7cea3bd2af32d0 --- /dev/null +++ b/USA_TST/raw/en-sols-TST-IMO-2015.pdf @@ -0,0 +1,3 @@ +version https://git-lfs.github.com/spec/v1 +oid sha256:53031c289b4e3a66eeddc306b280c6e6c43dbeed6c109c29e94ab409d06a3f5f +size 285610 diff --git a/USA_TST/raw/en-sols-TST-IMO-2016.pdf b/USA_TST/raw/en-sols-TST-IMO-2016.pdf new file mode 100644 index 0000000000000000000000000000000000000000..c29bc72a9abc8ba1a8f3fc25692ce3719c74a285 --- /dev/null +++ b/USA_TST/raw/en-sols-TST-IMO-2016.pdf @@ -0,0 +1,3 @@ +version https://git-lfs.github.com/spec/v1 +oid sha256:06d739a4f5ad2aa457b64ca9858f62a1331c854cf7dda59835624e6ea11fbe0a +size 240070 diff --git a/USA_TST/raw/en-sols-TST-IMO-2017.pdf b/USA_TST/raw/en-sols-TST-IMO-2017.pdf new file mode 100644 index 0000000000000000000000000000000000000000..6077e7af9c37c5730bfb8253df7f39d382d6968c --- /dev/null +++ b/USA_TST/raw/en-sols-TST-IMO-2017.pdf @@ -0,0 +1,3 @@ +version https://git-lfs.github.com/spec/v1 +oid sha256:4790ed887755cd72dcf71d921898605fcddae75ec078875cd1d5779a21abbf01 +size 251469 diff --git a/USA_TST/raw/en-sols-TST-IMO-2018.pdf b/USA_TST/raw/en-sols-TST-IMO-2018.pdf new file mode 100644 index 0000000000000000000000000000000000000000..5af2886e85c0f26996f487acde7744c6c764710e --- /dev/null +++ b/USA_TST/raw/en-sols-TST-IMO-2018.pdf @@ -0,0 +1,3 @@ +version https://git-lfs.github.com/spec/v1 +oid sha256:8d149ff271f83ca93f9dce391eda2eee927e52c4dbc026bd37a8f50dd406cfba +size 320622 diff --git a/USA_TST/raw/en-sols-TST-IMO-2019.pdf b/USA_TST/raw/en-sols-TST-IMO-2019.pdf new file mode 100644 index 0000000000000000000000000000000000000000..919486eca474b3283918dea60ff21e454df0a806 --- /dev/null +++ b/USA_TST/raw/en-sols-TST-IMO-2019.pdf @@ -0,0 +1,3 @@ +version https://git-lfs.github.com/spec/v1 +oid sha256:9ab239bba3f9d9cdd5a71fd64af952e6f453a19dd23a8980734c10b4ccffb300 +size 467274 diff --git a/USA_TST/raw/en-sols-TST-IMO-2020.pdf b/USA_TST/raw/en-sols-TST-IMO-2020.pdf new file mode 100644 index 0000000000000000000000000000000000000000..deed9433cbb5ee3be66be83a7086d401f497521e --- /dev/null +++ b/USA_TST/raw/en-sols-TST-IMO-2020.pdf @@ -0,0 +1,3 @@ +version https://git-lfs.github.com/spec/v1 +oid sha256:cd2426c00a683fad3f24eae343b539fc392ddb12f05813e77c2134a0672913a6 +size 428117 diff --git a/USA_TST/raw/en-sols-TST-IMO-2021.pdf b/USA_TST/raw/en-sols-TST-IMO-2021.pdf new file mode 100644 index 0000000000000000000000000000000000000000..ea4066029885c72cc25575dd17b52f6a415a2294 --- /dev/null +++ b/USA_TST/raw/en-sols-TST-IMO-2021.pdf @@ -0,0 +1,3 @@ +version https://git-lfs.github.com/spec/v1 +oid sha256:d2715ed90e219e5563e3cb5d25e46098d925572e8b64d7e6c355c55e6915f19b +size 325371 diff --git a/USA_TST/raw/en-sols-TST-IMO-2023.pdf b/USA_TST/raw/en-sols-TST-IMO-2023.pdf new file mode 100644 index 0000000000000000000000000000000000000000..72a629c5dd0fd031d035add6671c40e7ee274969 --- /dev/null +++ b/USA_TST/raw/en-sols-TST-IMO-2023.pdf @@ -0,0 +1,3 @@ +version https://git-lfs.github.com/spec/v1 +oid sha256:23111b7c227675a7dd6342add6662e37991cb02d068a388e1d77004d06b4021c +size 380772 diff --git a/USA_TST/segment_script/segment_usamo.py b/USA_TST/segment_script/segment_usamo.py new file mode 100644 index 0000000000000000000000000000000000000000..0fb17da47cecbb1c7443d21332018f569c13d2ff --- /dev/null +++ b/USA_TST/segment_script/segment_usamo.py @@ -0,0 +1,306 @@ +# ----------------------------------------------------------------------------- +# Author: Marina +# Date: 2024-11-15 +# ----------------------------------------------------------------------------- +''' Script to segment md files in en-usamo, en-tstst, en-tst, en-jmo folder using regex. +To run: +`python segment_script/segment_usamo.py` +''' + +import warnings +warnings.filterwarnings("ignore", category=DeprecationWarning) + +import os +import re +import pandas as pd +from rapidfuzz import fuzz + + +# "## §0 Problems" -> match +section_re = re.compile(r"^#{1,2}\s(?:Contents|Problem|§[\d.]+.*)") +# section_re = re.compile(r"^#{1,2}\s(?:§[\d.]+.*)") + +# "## §1.3 USAMO 2024/3, proposed by Krit Boonsiriseth" ->3 +# "## §2.3 TSTST 2011/6" ->6 +# "## §1.1 TSTST 2021/1, proposed by Holden Mui" -> 1 +# "## §1.2 USA TST 2020/2, proposed by Merlijn Staps" -> 2 +# "# §1.3 USA TST 2019/3, proposed by Nikolai Beluhov" -> 3 +solution_label_re = re.compile( + r"^#{1,2}\s§[\d.]+\s[A-Za-z0-9 ]+\s\d{4}/(\d+)(?:,\s.*)?$" +) + +# "1. Prove that the average of the numbers $n" -> match +# "2 . For any nonempty set $S$ of real numbers,"" -> match +problem_re = re.compile(r"^(\d+)\s?\.\s(.*(?:\n\s+.*)*)") + +# "## Problem statement extra text" -> match +# "Problem statement" -> match +# "##Problem statement (missing space)" -> match +solution_re = re.compile(r"^#{0,2}\s?Problem statement\b.*$") + +# not actually used, only for debugging +pattern_debug = re.compile( + r"^[【『\\]*.*?\b(First|Second|Third|Fourth|Fifth|Sixth|Seventh|Eighth|Ninth|Tenth|Complex|Inversion|Synthetic|One|Another|Solution)\b.*\b(solution|approach|proof)\b.*", + re.IGNORECASE +) + +# "Solution 1" -> match +# "【 Solution 1" -> match +solution_split_re1 = re.compile(r"\bSolution\s[1-9]\b") + +# "First solution" -> match +# "【 Third approach" -> match +# "【 Second approach" -> match +solution_split_re2 = re.compile(r"\b(First|Second|Third|Fourth|Fifth|Sixth|Seventh|Eighth|Ninth|Synthetic)\b\s+(solution|approach|proof)\b") + +# catch special cases (ugly but it works). To generate them run script with `DEBUG = True`` +# and identify in console which special_cases should be catched as first line of a solution +DEBUG = False +special_cases = [ +"【 First short solution, by Jeffrey Kwan. Let $p_{0", +"II Second longer solution using an invariant. Visu", +"【 Complex solution (Evan Chen). Toss on the comple", +"Second (longer) solution. If one does not notice t", +"『 Second calculation approach (along the lines of ", +"T Outline of second approach (by convexity, due t", +"I Inversion solution submitted by Ankan Bhattacha", +"【 Complex numbers approach with Apollonian circles", +" A second solution. Both lemmas above admit varia", +"【 A third remixed solution. We use Lemma I and Lem", +"【I A fourth remixed solution. We also can combine ", +"I First grid-based solution. The following solutio", +"Another short solution. Let $Z$ be on line $B D E$", +"【 Most common synthetic approach. The solution hin", +"\\ First \"local\" solution by swapping two points. L", +"Second general solution by angle chasing. By Rei", +"Third general solution by Pascal. Extend rays $A", +"【 Second length solution by tangent lengths. By $t", +"【 Angle chasing solution. Note that $(B D A)$ and", +"【 Harmonic solution (mine). Let $T$ be the point o", +"【 Pascal solution (Zuming Feng). Extend ray $F D$", +"『 A spiral similarity approach (Hans $\mathbf{Y u}", +"ब The author's original solution. Complete isoscel", +"l Evan's permutation-based solution. Retain the n", +"I Original proposer's solution. To this end, let's", +"【 Cartesian coordinates approach with power of a p", +"【 Cartesian coordinates approach without power of", +"I III-advised barycentric approach (outline). Use", +"【 Approach using difference of squares (from autho", +"【 Divisibility approach (Aharshi Roy). Since $p q-", +"Solution with Danielle Wang: the answer is that $|", +"【 Homothety solution (Alex Whatley). Let $G, N, O$", +"【 Power of a point solution (Zuming Feng, official", +"【 Solution by Luke Robitaille. Let $Q$ be the seco", +"ๆ Solution with coaxial circles (Pitchayut Saengru", +"【 Solution to generalization (Nikolai Beluhov). We", +"【 Approach by deleting teams (Gopal Goel). Initial", +"【 Approach by adding colors. For a constructive al", +"【 Solution using spiral similarity. We will ignore", +"『 Barycentric solution (by Carl, Krit, Milan). We", +"I A Menelaus-based approach (Kevin Ren). Let $P$ b", +"【 Barycentric solution. First, we find the coordin", +"【 Angle chasing solution (Mason Fang). Obviously $", +"【 Inversive solution (Kelin Zhu). Invert about $A$", +"l The one-liner. ", +" The external power solution. We distinguish betw", +"Cauchy-Schwarz approach. Apply Titu lemma to get", +"đ Cauchy-Schwarz approach. The main magical claim ", +"『 Alternate solution (by proposer). Let $L$ be dia" +] + + +def add_content(current_dict): + if not current_dict["lines"] or not current_dict["label"] : + return + text_str = " ".join(current_dict["lines"]).strip() + entry = {"label": current_dict["label"]} + if current_dict["class"] == "problem": + entry["problem"] = text_str + current_dict["problems"].append(entry) + elif current_dict["class"] == "solution": + entry["solution"] = text_str + entry["solution_lines"] = current_dict["lines"] + current_dict["solutions"].append(entry) + + +def parse(file): + with open(file, 'r') as file: + content = file.read() + current = { + "label": None, + "class": None, + "lines": [], + "problems": [], + "solutions": [] + } + for line in content.splitlines(): + if match := section_re.match(line): # match a section + add_content(current) + if "problems" in line.lower(): # match problem section + current["class"] = "problem" + elif sub_match:= solution_label_re.match(line): # match solutions section, extract label for join + current["class"] = "other" + current["label"] = sub_match.group(1) + elif match := solution_re.match(line): # match solutions subsection + current["class"] = "solution" + else: + current["class"] = "other" + current["lines"] = [] + elif match := problem_re.match(line): # match a problem + if current["class"] == "solution": # handle wrong problem match + current["lines"].append(line) + else: + add_content(current) + label, text = match.groups() + current["label"] = label + current["lines"] = [text] + else: + if current["class"]=="solution" or current["class"]=="problem": + current["lines"].append(line) + add_content(current) + problems_df = pd.DataFrame(current["problems"]) + solutions_df = pd.DataFrame(current["solutions"]) + return problems_df, solutions_df + + +def parse_solution(lines): + """parses lines of a solution, finds multiple solutions and splits them""" + solutions = [] + current = [] + for line in lines: + if match := solution_split_re1.search(line): # match a solution (case1) + solutions.append(" ".join(current).strip()) + current = [line] + elif match := solution_split_re2.search(line): # match a solution (case1) + solutions.append(" ".join(current).strip()) + current = [line] + elif any(case.lower() in line.lower() for case in special_cases): # match a solution (handle special_case) + solutions.append(" ".join(current).strip()) + current = [line] + elif any(case.lower() in line[:50].lower() for case in ["solution", "approach", "proof"]): + if DEBUG: + if not any(case.lower() in line[:50].lower() for case in ["remark", "proof.", "proof", "approaches", "solutions"]): + print(line[:50]) + else: + current.append(line) + solutions.append(" ".join(current).strip()) + return solutions + +def find_mult_solutions(solutions_df): + """apply parse_solution to all df""" + solutions_df["solution"] = solutions_df["solution_lines"].apply(lambda v: parse_solution(v)) + solutions_df = solutions_df.drop(columns=["solution_lines"]) + solutions_df = solutions_df.explode('solution', ignore_index=True) + return solutions_df + + +def join(problems_df, solutions_df): + pairs_df = problems_df.merge(solutions_df, on=["label"], how="outer") + return pairs_df + + +def clean(pairs_df): + '''removes the problem statement from the solution in an approximate way''' + def find_closest_char(s, i, char): + left = s.rfind(char, 0, i) # Find the last '.' before index i + right = s.find(char, i) # Find the first '.' after or at index i + if left == -1 and right == -1: + return None # No '.' found + elif left == -1: # No '.' on the left + return right + elif right == -1: # No '.' on the right + return left + else: # Closest '.' on either side + return left if abs(i - left) <= abs(i - right) else right + def remove_approx_match(row, threshold=90): + problem = row["problem"] + solution = row["solution"] + similarity = fuzz.partial_ratio(problem, solution) + if similarity >= threshold: + i = find_closest_char(solution, len(problem), problem[-1]) + if i is not None: + solution = solution[i+1:] + return solution + pairs_df["solution"] = pairs_df.apply(remove_approx_match, axis=1) + return pairs_df + + +def process_mult_solutions(pairs_df): + '''in case of multiple solutions, prepend common text to all solutions''' + def prepend_to_solution(group): + if len(group) == 1: + return group + first_row = group.iloc[0] + comment = f"{first_row['solution']}" + group = group.iloc[1:].copy() + group["solution"] = group["solution"].apply(lambda x: f"{comment} {x}") + return group + pairs_df = pairs_df.groupby("label", group_keys=False).apply(prepend_to_solution).reset_index(drop=True) + return pairs_df + + +def add_metadata(pairs_df, yar, tier): + pairs_df['year'] = year + pairs_df['tier'] = tier # according to omnimath + return pairs_df[['year', 'label', 'problem', 'solution']] + + +def write_pairs(filename, pairs_df): + pairs_df.to_json(filename, orient="records", lines=True) + + +configs = [ + ('en-usamo', lambda year: f'en-USAMO-{year}-notes', range(1996, 2025), 1), + ('en-tstst', lambda year: f'en-sols-TSTST-{year}', range(2011, 2024), 0), + ('en-tst', lambda year: f'en-sols-TST-IMO-{year}', range(2014, 2024), 0), + # ('en-elmo', lambda year: f'en-ELMO-{year}-sols', range(2010, 2017), None), #TODO: needs another parser + # ('en-elmo', lambda year: f'en-ELMO-{year}-SL', range(2017, 2019), None), #TODO: needs another parser + ('en-jmo', lambda year: f'en-JMO-{year}-notes', range(2010, 2025), 3), + # ('en-usemo', lambda year: f'en-report-usemo-{year}', range(2019, 2023), None), #TODO: needs another parser +] + + +total_problem_count = 0 +total_solution_count = 0 +for base, basename_, years, tier in configs: + print(base) + problem_count = 0 + solution_count = 0 + seg_base = f"{base}-seg" + os.makedirs(seg_base, exist_ok=True) + for year in years: + basename = basename_(year) + file_path = f"{base}/{basename}.md" + if os.path.exists(file_path): + # print(basename) + problems, solutions = parse(file_path) + solutions = find_mult_solutions(solutions) + pairs_df = join(problems, solutions) + pairs_df = clean(pairs_df) + pairs_df = process_mult_solutions(pairs_df) + pairs_df = add_metadata(pairs_df, year, tier) + problem_count += len(problems) + solution_count += len(pairs_df) + # print(pairs_df) + write_pairs(f"{seg_base}/{basename}.jsonl", pairs_df) + print(f"problem count: {problem_count}") + print(f"solution count: {solution_count}") + total_problem_count += problem_count + total_solution_count += solution_count +print(f"total problem count: {total_problem_count}") +print(f"total solution count: {total_solution_count}") + +# en-usamo +# problem count: 174 +# solution count: 203 +# en-tstst +# problem count: 105 +# solution count: 168 +# en-tst +# problem count: 51 +# solution count: 82 +# en-jmo +# problem count: 90 +# solution count: 127 +# total problem count: 420 +# total solution count: 580 \ No newline at end of file diff --git a/USA_TST/segmented/en-sols-TST-IMO-2014.jsonl b/USA_TST/segmented/en-sols-TST-IMO-2014.jsonl new file mode 100644 index 0000000000000000000000000000000000000000..6854f6efa66bbc881e87e450ccf1ad303be69e58 --- /dev/null +++ b/USA_TST/segmented/en-sols-TST-IMO-2014.jsonl @@ -0,0 +1,6 @@ +{"year":2014,"label":"1","problem":"Let $A B C$ be an acute triangle, and let $X$ be a variable interior point on the minor $\\operatorname{arc} B C$ of its circumcircle. Let $P$ and $Q$ be the feet of the perpendiculars from $X$ to lines $C A$ and $C B$, respectively. Let $R$ be the intersection of line $P Q$ and the perpendicular from $B$ to $A C$. Let $\\ell$ be the line through $P$ parallel to $X R$. Prove that as $X$ varies along minor arc $B C$, the line $\\ell$ always passes through a fixed point.","solution":" The fixed point is the orthocenter, since $\\ell$ is a Simson line. See Lemma 4.4 of Euclidean Geometry in Math Olympiads."} +{"year":2014,"label":"2","problem":"Let $a_{1}, a_{2}, a_{3}, \\ldots$ be a sequence of integers, with the property that every consecutive group of $a_{i}$ 's averages to a perfect square. More precisely, for all positive integers $n$ and $k$, the quantity $$ \\frac{a_{n}+a_{n+1}+\\cdots+a_{n+k-1}}{k} $$ is always the square of an integer. Prove that the sequence must be constant (all $a_{i}$ are equal to the same perfect square).","solution":" Let $\\nu_{p}(n)$ denote the largest exponent of $p$ dividing $n$. The problem follows from the following proposition. ## Proposition Let $\\left(a_{n}\\right)$ be a sequence of integers and let $p$ be a prime. Suppose that every consecutive group of $a_{i}$ 's with length at most $p$ averages to a perfect square. Then $\\nu_{p}\\left(a_{i}\\right)$ is independent of $i$. We proceed by induction on the smallest value of $\\nu_{p}\\left(a_{i}\\right)$ as $i$ ranges (which must be even, as each of the $a_{i}$ are themselves a square). First we prove two claims. Claim - If $j \\equiv k(\\bmod p)$ then $a_{j} \\equiv a_{k}(\\bmod p)$. Claim - If some $a_{i}$ is divisible by $p$ then all of them are. $$ S_{n}=a_{1}+a_{2}+\\cdots+a_{n} \\equiv a_{2}+\\cdots+a_{n} \\quad(\\bmod p) $$ Call an integer $k$ with $2 \\leq k0$, the result is vacuous for $x \\leq 0$, so we restrict attention to $x>0$. Then letting $c_{i}=1-d_{i}$ for each $i$, the inequality we want to prove becomes $$ x^{n}+1+\\frac{x^{n+1}+1}{x+1}>\\sum_{1}^{n-1} d_{i} x^{i} \\quad \\text { given } \\sum\\left|d_{i}\\right|<1 $$ But obviously $x^{n}+1>x^{i}$ for any $1 \\leq i \\leq n-1$ and $x>0$. So in fact $x^{n}+1>\\sum_{1}^{n-1}\\left|d_{i}\\right| x^{i}$ holds for $x>0$, as needed."} +{"year":2014,"label":"5","problem":"Let $A B C D$ be a cyclic quadrilateral, and let $E, F, G$, and $H$ be the midpoints of $A B, B C, C D$, and $D A$ respectively. Let $W, X, Y$ and $Z$ be the orthocenters of triangles $A H E, B E F, C F G$ and $D G H$, respectively. Prove that the quadrilaterals $A B C D$ and $W X Y Z$ have the same area.","solution":" We begin with: Claim - Point $W$ has coordinates $\\frac{1}{2}(2 a+b+d)$. By symmetry, we have $$ \\begin{aligned} w & =\\frac{1}{2}(2 a+b+d) \\\\ x & =\\frac{1}{2}(2 b+c+a) \\\\ y & =\\frac{1}{2}(2 c+d+b) \\\\ z & =\\frac{1}{2}(2 d+a+c) . \\end{aligned} $$ We see that $w-y=a-c, x-z=b-d$. So the diagonals of $W X Y Z$ have the same length as those of $A B C D$ as well as the same directed angle between them. This implies the areas are equal, too."} +{"year":2014,"label":"6","problem":"For a prime $p$, a subset $S$ of residues modulo $p$ is called a sum-free multiplicative subgroup of $\\mathbb{F}_{p}$ if - there is a nonzero residue $\\alpha$ modulo $p$ such that $S=\\left\\{1, \\alpha^{1}, \\alpha^{2}, \\ldots\\right\\}$ (all considered $\\bmod p$ ), and - there are no $a, b, c \\in S$ (not necessarily distinct) such that $a+b \\equiv c(\\bmod p)$. Prove that for every integer $N$, there is a prime $p$ and a sum-free multiplicative subgroup $S$ of $\\mathbb{F}_{p}$ such that $|S| \\geq N$.","solution":" We first prove the following general lemma. ## Lemma If $f, g \\in \\mathbb{Z}[X]$ are relatively prime nonconstant polynomials, then for sufficiently large primes $p$, they have no common root modulo $p$. $$ a(X) f(X)+b(X) g(X) \\equiv c $$ So, plugging in $X=r$ we get $p \\mid c$, so the set of permissible primes $p$ is finite. With this we can give the construction. ## Claim - Suppose that - $n$ is a positive integer with $n \\not \\equiv 0(\\bmod 3)$; - $p$ is a prime which is $1 \\bmod n$; and - $\\alpha$ is a primitive $n^{\\prime}$ th root of unity modulo $p$. Then $|S|=n$ and, if $p$ is sufficiently large in $n$, is also sum-free. $$ 1+\\alpha^{k} \\equiv \\alpha^{m} \\quad(\\bmod p) $$ for some integers $k, m \\in \\mathbb{Z}$. This means $(X+1)^{n}-1$ and $X^{n}-1$ have common root $X=\\alpha^{k}$. But $$ \\underset{\\mathbb{Z}[x]}{\\operatorname{gcd}}\\left((X+1)^{n}-1, X^{n}-1\\right)=1 \\quad \\forall n \\not \\equiv 0 \\quad(\\bmod 3) $$ because when $3 \\nmid n$ the two polynomials have no common complex roots. (Indeed, if $|\\omega|=|1+\\omega|=1$ then $\\omega=-\\frac{1}{2} \\pm \\frac{\\sqrt{3}}{2} i$.) Thus $p$ is bounded by the lemma, as desired."} diff --git a/USA_TST/segmented/en-sols-TST-IMO-2015.jsonl b/USA_TST/segmented/en-sols-TST-IMO-2015.jsonl new file mode 100644 index 0000000000000000000000000000000000000000..8172309bb9f5bf00461250880213c1eb22b510b3 --- /dev/null +++ b/USA_TST/segmented/en-sols-TST-IMO-2015.jsonl @@ -0,0 +1,9 @@ +{"year":2015,"label":"1","problem":"Let $A B C$ be a scalene triangle with incenter $I$ whose incircle is tangent to $\\overline{B C}$, $\\overline{C A}, \\overline{A B}$ at $D, E, F$, respectively. Denote by $M$ the midpoint of $\\overline{B C}$ and let $P$ be a point in the interior of $\\triangle A B C$ so that $M D=M P$ and $\\angle P A B=\\angle P A C$. Let $Q$ be a point on the incircle such that $\\angle A Q D=90^{\\circ}$. Prove that either $\\angle P Q E=90^{\\circ}$ or $\\angle P Q F=90^{\\circ}$.","solution":" ![](https:\/\/cdn.mathpix.com\/cropped\/2024_11_19_2c252c8bc1550e62c4c8g-03.jpg?height=801&width=892&top_left_y=1150&top_left_x=588) First, we claim that $D, P, E$ are collinear. Let $N$ be the midpoint of $\\overline{A B}$. It is well-known that the three lines $M N, D E, A I$ are concurrent at a point (see for example problem 6 of USAJMO 2014). Let $P^{\\prime}$ be this intersection point, noting that $P^{\\prime}$ actually lies on segment $D E$. Then $P^{\\prime}$ lies inside $\\triangle A B C$ and moreover $$ \\triangle D P^{\\prime} M \\sim \\triangle D E C $$ so $M P^{\\prime}=M D$. Hence $P^{\\prime}=P$, proving the claim. Let $S$ be the point diametrically opposite $D$ on the incircle, which is also the second intersection of $\\overline{A Q}$ with the incircle. Let $T=\\overline{A Q} \\cap \\overline{B C}$. Then $T$ is the contact point of the $A$-excircle; consequently, $$ M D=M P=M T $$ and we obtain a circle with diameter $\\overline{D T}$. Since $\\angle D Q T=\\angle D Q S=90^{\\circ}$ we have $Q$ on this circle as well. As $\\overline{S D}$ is tangent to the circle with diameter $\\overline{D T}$, we obtain $$ \\angle P Q D=\\angle S D P=\\angle S D E=\\angle S Q E . $$ Since $\\angle D Q S=90^{\\circ}, \\angle P Q E=90^{\\circ}$ too. \u3010 Solution using spiral similarity. We will ignore for now the point $P$. As before define $S, T$ and note $\\overline{A Q S T}$ collinear, as well as $D P Q T$ cyclic on circle $\\omega$ with diameter $\\overline{D T}$. Let $\\tau$ be the spiral similarity at $Q$ sending $\\omega$ to the incircle. We have $\\tau(T)=D$, $\\tau(D)=S, \\tau(Q)=Q$. Now $$ I=\\overline{D D} \\cap \\overline{Q Q} \\Longrightarrow \\tau(I)=\\overline{S S} \\cap \\overline{Q Q} $$ and hence we conclude $\\tau(I)$ is the pole of $\\overline{A S Q T}$ with respect to the incircle, which lies on line $E F$. Then since $\\overline{A I} \\perp \\overline{E F}$ too, we deduce $\\tau$ sends line $A I$ to line $E F$, hence $\\tau(P)$ must be either $E$ or $F$ as desired. \u3010 Authorship comments. Written April 2014. I found this problem while playing with GeoGebra. Specifically, I started by drawing in the points $A, B, C, I, D, M, T$, common points. I decided to add in the circle with diameter $D T$, because of the synergy it had with the rest of the picture. After a while of playing around, I intersected ray $A I$ with the circle to get $P$, and was surprised to find that $D, P, E$ were collinear, which I thought was impossible since the setup should have been symmetric. On further reflection, I realized it was because $A I$ intersected the circle twice, and set about trying to prove this. I noticed the relation $\\angle P Q E=90^{\\circ}$ in my attempts to prove the result, even though this ended up being a corollary rather than a useful lemma."} +{"year":2015,"label":"2","problem":"Prove that for every positive integer $n$, there exists a set $S$ of $n$ positive integers such that for any two distinct $a, b \\in S, a-b$ divides $a$ and $b$ but none of the other elements of $S$.","solution":" The idea is to look for a sequence $d_{1}, \\ldots, d_{n-1}$ of \"differences\" such that the following two conditions hold. Let $s_{i}=d_{1}+\\cdots+d_{i-1}$, and $t_{i, j}=d_{i}+\\cdots+d_{j-1}$ for $i \\leq j$. (i) No two of the $t_{i, j}$ divide each other. (ii) There exists an integer $a$ satisfying the CRT equivalences $$ a \\equiv-s_{i} \\quad\\left(\\bmod t_{i, j}\\right) \\quad \\forall i \\leq j $$ Then the sequence $a+s_{1}, a+s_{2}, \\ldots, a+s_{n}$ will work. For example, when $n=3$ we can take $\\left(d_{1}, d_{2}\\right)=(2,3)$ giving ![](https:\/\/cdn.mathpix.com\/cropped\/2024_11_19_2c252c8bc1550e62c4c8g-05.jpg?height=152&width=310&top_left_y=1215&top_left_x=876) because the only conditions we need satisfy are $$ \\begin{aligned} a & \\equiv 0 \\quad(\\bmod 2) \\\\ a & \\equiv 0 \\quad(\\bmod 5) \\\\ a & \\equiv-2 \\quad(\\bmod 3) . \\end{aligned} $$ But with this setup we can just construct the $d_{i}$ inductively. To go from $n$ to $n+1$, take a $d_{1}, \\ldots, d_{n-1}$ and let $p$ be a prime not dividing any of the $d_{i}$. Moreover, let $M$ be a multiple of $\\prod_{i \\leq j} t_{i, j}$ coprime to $p$. Then we claim that $d_{1} M, d_{2} M, \\ldots, d_{n-1} M, p$ is such a difference sequence. For example, the previous example extends as follows with $M=300$ and $p=7$. ![](https:\/\/cdn.mathpix.com\/cropped\/2024_11_19_2c252c8bc1550e62c4c8g-05.jpg?height=192&width=366&top_left_y=1886&top_left_x=848) The new numbers $p, p+M t_{n-1, n}, p+M t_{n-2, n}, \\ldots$ are all relatively prime to everything else. Hence (i) still holds. To see that (ii) still holds, just note that we can still get a family of solutions for the first $n$ terms, and then the last $(n+1)$ st term can be made to work by Chinese Remainder Theorem since all the new $p+M t_{n-2, n}$ are coprime to everything."} +{"year":2015,"label":"3","problem":"A physicist encounters 2015 atoms called usamons. Each usamon either has one electron or zero electrons, and the physicist can't tell the difference. The physicist's only tool is a diode. The physicist may connect the diode from any usamon $A$ to any other usamon $B$. (This connection is directed.) When she does so, if usamon $A$ has an electron and usamon $B$ does not, then the electron jumps from $A$ to $B$. In any other case, nothing happens. In addition, the physicist cannot tell whether an electron jumps during any given step. The physicist's goal is to isolate two usamons that she is $100 \\%$ sure are currently in the same state. Is there any series of diode usage that makes this possible?","solution":" The answer is no. Call the usamons $U_{1}, \\ldots, U_{m}$ (here $m=2015$ ). Consider models $M_{k}$ of the following form: $U_{1}, \\ldots, U_{k}$ are all charged for some $0 \\leq k \\leq m$ and the other usamons are not charged. Note that for any pair there's a model where they are different states, by construction. We can consider the physicist as acting on these $m+1$ models simultaneously, and trying to reach a state where there's a pair in all models which are all the same charge. (This is a necessary condition for a winning strategy to exist.) But we claim that any diode operation $U_{i} \\rightarrow U_{j}$ results in the $m+1$ models being an isomorphic copy of the previous set. If $ij$ the operation never does anything. The conclusion follows from this. Remark. This problem is not a \"standard\" olympiad problem, so I can't say it's trivial. But the idea is pretty natural I think. You can motivate it as follows: there's a sequence of diode operations you can do which forces the situation to be one of the $M_{k}$ above: first, use the diode into $U_{1}$ for all other $U_{i}$ 's, so that either no electrons exist at all or $U_{1}$ has an electron. Repeat with the other $U_{i}$. This leaves us at the situation described at the start of the problem. Then you could guess the answer was \"no\" just based on the fact that it's impossible for $n=2,3$ and that there doesn't seem to be a reasonable strategy. In this way it's possible to give a pretty good description of what it's possible to do. One possible phrasing: \"the physicist can arrange the usamons in a line such that all the charged usamons are to the left of the un-charged usamons, but can't determine the number of charged usamons\"."} +{"year":2015,"label":"4","problem":"Let $f: \\mathbb{Q} \\rightarrow \\mathbb{Q}$ be a function such that for any $x, y \\in \\mathbb{Q}$, the number $f(x+y)-$ $f(x)-f(y)$ is an integer. Decide whether there must exist a constant $c$ such that $f(x)-c x$ is an integer for every rational number $x$.","solution":" No, such a constant need not exist. $$ \\begin{aligned} & 2 x_{1}=x_{0} \\\\ & 2 x_{2}=x_{1}+1 \\\\ & 2 x_{3}=x_{2} \\\\ & 2 x_{4}=x_{3}+1 \\\\ & 2 x_{5}=x_{4} \\\\ & 2 x_{6}=x_{5}+1 \\end{aligned} $$ Set $f\\left(2^{-k}\\right)=x_{k}$ and $f\\left(2^{k}\\right)=2^{k}$ for $k=0,1, \\ldots$ Then, let $$ f\\left(a \\cdot 2^{k}+\\frac{b}{c}\\right)=a f\\left(2^{k}\\right)+\\frac{b}{c} $$ for odd integers $a, b, c$. One can verify this works. $$ f\\left(\\frac{p}{q}\\right)=\\frac{p}{q}(1!+2!+\\cdots+q!) . $$ Remark. Silly note: despite appearances, $f(x)=\\lfloor x\\rfloor$ is not a counterexample since one can take $c=0$."} +{"year":2015,"label":"5","problem":"Fix a positive integer $n$. A tournament on $n$ vertices has all its edges colored by $\\chi$ colors, so that any two directed edges $u \\rightarrow v$ and $v \\rightarrow w$ have different colors. Over all possible tournaments on $n$ vertices, determine the minimum possible value of $\\chi$.","solution":" The answer is $$ \\chi=\\left\\lceil\\log _{2} n\\right\\rceil $$ First, we prove by induction on $n$ that $\\chi \\geq \\log _{2} n$ for any coloring and any tournament. The base case $n=1$ is obvious. Now given any tournament, consider any used color $c$. Then it should be possible to divide the tournament into two subsets $A$ and $B$ such that all $c$-colored edges point from $A$ to $B$ (for example by letting $A$ be all vertices which are the starting point of a $c$-edge). ![](https:\/\/cdn.mathpix.com\/cropped\/2024_11_19_2c252c8bc1550e62c4c8g-08.jpg?height=486&width=512&top_left_y=1116&top_left_x=772) One of $A$ and $B$ has size at least $n \/ 2$, say $A$. Since $A$ has no $c$ edges, and uses at least $\\log _{2}|A|$ colors other than $c$, we get $$ \\chi \\geq 1+\\log _{2}(n \/ 2)=\\log _{2} n $$ completing the induction. One can read the construction off from the argument above, but here is a concrete description. For each integer $n$, consider the tournament whose vertices are the binary representations of $S=\\{0, \\ldots, n-1\\}$. Instantiate colors $c_{1}, c_{2}, \\ldots$. Then for $v, w \\in S$, we look at the smallest order bit for which they differ; say the $k$ th one. If $v$ has a zero in the $k$ th bit, and $w$ has a one in the $k$ th bit, we draw $v \\rightarrow w$. Moreover we color the edge with color $c_{k}$. This works and uses at most $\\left\\lceil\\log _{2} n\\right\\rceil$ colors. Remark (Motivation). The philosophy \"combinatorial optimization\" applies here. The idea is given any color $c$, we can find sets $A$ and $B$ such that all $c$-edges point $A$ to $B$. Once you realize this, the next insight is to realize that you may as well color all the edges from $A$ to $B$ by $c$; after all, this doesn't hurt the condition and makes your life easier. Hence, if $f$ is the answer, we have already a proof that $f(n)=1+\\max (f(|A|), f(|B|))$ and we choose $|A| \\approx|B|$. This optimization also gives the inductive construction."} +{"year":2015,"label":"6","problem":"Let $A B C$ be a non-equilateral triangle and let $M_{a}, M_{b}, M_{c}$ be the midpoints of the sides $B C, C A, A B$, respectively. Let $S$ be a point lying on the Euler line. Denote by $X, Y, Z$ the second intersections of $M_{a} S, M_{b} S, M_{c} S$ with the nine-point circle. Prove that $A X, B Y, C Z$ are concurrent.","solution":" We assume now and forever that $A B C$ is scalene since the problem follows by symmetry in the isosceles case. We present four solutions. \u3010 First solution by barycentric coordinates (Evan Chen). Let $A X$ meet $M_{b} M_{c}$ at $D$, and let $X$ reflected over $M_{b} M_{c}^{\\prime}$ 's midpoint be $X^{\\prime}$. Let $Y^{\\prime}, Z^{\\prime}, E, F$ be similarly defined. ![](https:\/\/cdn.mathpix.com\/cropped\/2024_11_19_2c252c8bc1550e62c4c8g-09.jpg?height=687&width=807&top_left_y=1070&top_left_x=633) By Cevian Nest Theorem it suffices to prove that $M_{a} D, M_{b} E, M_{c} F$ are concurrent. Taking the isotomic conjugate and recalling that $M_{a} M_{b} A M_{c}$ is a parallelogram, we see that it suffices to prove $M_{a} X^{\\prime}, M_{b} Y^{\\prime}, M_{c} Z^{\\prime}$ are concurrent. We now use barycentric coordinates on $\\triangle M_{a} M_{b} M_{c}$. Let $$ S=\\left(a^{2} S_{A}+t: b^{2} S_{B}+t: c^{2} S_{C}+t\\right) $$ (possibly $t=\\infty$ if $S$ is the centroid). Let $v=b^{2} S_{B}+t, w=c^{2} S_{C}+t$. Hence $$ X=\\left(-a^{2} v w:\\left(b^{2} w+c^{2} v\\right) v:\\left(b^{2} w+c^{2} v\\right) w\\right) $$ Consequently, $$ X^{\\prime}=\\left(a^{2} v w:-a^{2} v w+\\left(b^{2} w+c^{2} v\\right) w:-a^{2} v w+\\left(b^{2} w+c^{2} v\\right) v\\right) $$ We can compute $$ b^{2} w+c^{2} v=(b c)^{2}\\left(S_{B}+S_{C}\\right)+\\left(b^{2}+c^{2}\\right) t=(a b c)^{2}+\\left(b^{2}+c^{2}\\right) t $$ Thus $$ -a^{2} v+b^{2} w+c^{2} v=\\left(b^{2}+c^{2}\\right) t+(a b c)^{2}-(a b)^{2} S_{B}-a^{2} t=S_{A}\\left((a b)^{2}+t\\right) $$ Finally $$ X^{\\prime}=\\left(a^{2} v w: S_{A}\\left(c^{2} S_{C}+t\\right)\\left((a b)^{2}+2 t\\right): S_{A}\\left(b^{2} S_{B}+t\\right)\\left((a c)^{2}+2 t\\right)\\right) $$ and from this it's evident that $A X^{\\prime}, B Y^{\\prime}, C Z^{\\prime}$ are concurrent."} +{"year":2015,"label":"6","problem":"Let $A B C$ be a non-equilateral triangle and let $M_{a}, M_{b}, M_{c}$ be the midpoints of the sides $B C, C A, A B$, respectively. Let $S$ be a point lying on the Euler line. Denote by $X, Y, Z$ the second intersections of $M_{a} S, M_{b} S, M_{c} S$ with the nine-point circle. Prove that $A X, B Y, C Z$ are concurrent.","solution":" We assume now and forever that $A B C$ is scalene since the problem follows by symmetry in the isosceles case. We present four solutions. \\ Second solution by moving points (Anant Mudgal). Let $H_{a}, H_{b}, H_{c}$ be feet of altitudes, and let $\\gamma$ denote the nine-point circle. The main claim is that: Claim - Lines $X H_{a}, Y H_{b}, Z H_{c}$ are concurrent, $$ \\begin{aligned} & \\ell \\rightarrow \\gamma \\rightarrow \\ell \\\\ & S \\mapsto X \\mapsto S_{a}:=\\ell \\cap \\overline{H_{a} X} \\end{aligned} $$ is projective, because it consists of two perspectivities. So we want the analogous maps $S \\mapsto S_{b}, S \\mapsto S_{c}$ to coincide. For this it suffices to check three positions of $S$; since you're such a good customer here are four. - If $S$ is the orthocenter of $\\triangle M_{a} M_{b} M_{c}$ (equivalently the circumcenter of $\\triangle A B C$ ) then $S_{a}$ coincides with the circumcenter of $M_{a} M_{b} M_{c}$ (equivalently the nine-point center of $\\triangle A B C)$. By symmetry $S_{b}$ and $S_{c}$ are too. - If $S$ is the circumcenter of $\\triangle M_{a} M_{b} M_{c}$ (equivalently the nine-point center of $\\triangle A B C$ ) then $S_{a}$ coincides with the de Longchamps point of $\\triangle M_{a} M_{b} M_{c}$ (equivalently orthocenter of $\\triangle A B C)$. By symmetry $S_{b}$ and $S_{c}$ are too. - If $S$ is either of the intersections of the Euler line with $\\gamma$, then $S=S_{a}=S_{b}=S_{c}$ (as $S=X=Y=Z$ ). ![](https:\/\/cdn.mathpix.com\/cropped\/2024_11_19_2c252c8bc1550e62c4c8g-10.jpg?height=681&width=797&top_left_y=1947&top_left_x=638) We now use Trig Ceva to carry over the concurrence. By sine law, $$ \\frac{\\sin \\angle M_{c} A X}{\\sin \\angle A M_{c} X}=\\frac{M_{c} X}{A X} $$ and a similar relation for $M_{b}$ gives that $$ \\frac{\\sin \\angle M_{c} A X}{\\sin \\angle M_{b} A X}=\\frac{\\sin \\angle A M_{c} X}{\\sin \\angle A M_{b} X} \\cdot \\frac{M_{c} X}{M_{b} X}=\\frac{\\sin \\angle A M_{c} X}{\\sin \\angle A M_{b} X} \\cdot \\frac{\\sin \\angle X M_{a} M_{c}}{\\sin \\angle X M_{a} M_{b}} . $$ Thus multiplying cyclically gives $$ \\prod_{\\text {cyc }} \\frac{\\sin \\angle M_{c} A X}{\\sin \\angle M_{b} A X}=\\prod_{\\text {cyc }} \\frac{\\sin \\angle A M_{c} X}{\\sin \\angle A M_{b} X} \\prod_{\\text {cyc }} \\frac{\\sin \\angle X M_{a} M_{c}}{\\sin \\angle X M_{a} M_{b}} . $$ The latter product on the right-hand side equals 1 by Trig Ceva on $\\triangle M_{a} M_{b} M_{c}$ with cevians $\\overline{M_{a} X}, \\overline{M_{b} Y}, \\overline{M_{c} Z}$. The former product also equals 1 by Trig Ceva for the concurrence in the previous claim (and the fact that $\\angle A M_{c} X=\\angle H_{c} H_{a} X$ ). Hence the left-hand side equals 1 , implying the result."} +{"year":2015,"label":"6","problem":"Let $A B C$ be a non-equilateral triangle and let $M_{a}, M_{b}, M_{c}$ be the midpoints of the sides $B C, C A, A B$, respectively. Let $S$ be a point lying on the Euler line. Denote by $X, Y, Z$ the second intersections of $M_{a} S, M_{b} S, M_{c} S$ with the nine-point circle. Prove that $A X, B Y, C Z$ are concurrent.","solution":" We assume now and forever that $A B C$ is scalene since the problem follows by symmetry in the isosceles case. We present four solutions. \u300e Third solution by moving points (Gopal Goel). In this solution, we will instead use barycentric coordinates with resect to $\\triangle A B C$ to bound the degrees suitably, and then verify for seven distinct choices of $S$. We let $R$ denote the radius of $\\triangle A B C$, and $N$ the nine-point center. First, imagine solving for $X$ in the following way. Suppose $\\vec{X}=\\left(1-t_{a}\\right) \\vec{M}_{a}+t_{a} \\vec{S}$. Then, using the dot product (with $|\\vec{v}|^{2}=\\vec{v} \\cdot \\vec{v}$ in general) $$ \\begin{aligned} \\frac{1}{4} R^{2} & =|\\vec{X}-\\vec{N}|^{2} \\\\ & =\\left|t_{a}\\left(\\vec{S}-\\vec{M}_{a}\\right)+\\vec{M}_{a}-\\vec{N}\\right|^{2} \\\\ & =\\left|t_{a}\\left(\\vec{S}-\\vec{M}_{a}\\right)\\right|^{2}+2 t_{a}\\left(\\vec{S}-\\vec{M}_{a}\\right) \\cdot\\left(\\vec{M}_{a}-\\vec{N}\\right)+\\left|\\vec{M}_{a}-\\vec{N}\\right|^{2} \\\\ & =t_{a}^{2}\\left|\\left(\\vec{S}-\\vec{M}_{a}\\right)\\right|^{2}+2 t_{a}\\left(\\vec{S}-\\vec{M}_{a}\\right) \\cdot\\left(\\vec{M}_{a}-\\vec{N}\\right)+\\frac{1}{4} R^{2} \\end{aligned} $$ Since $t_{a} \\neq 0$ we may solve to obtain $$ t_{a}=-\\frac{2\\left(\\vec{M}_{a}-\\vec{N}\\right) \\cdot\\left(\\vec{S}-\\vec{M}_{a}\\right)}{\\left|\\vec{S}-\\vec{M}_{a}\\right|^{2}} $$ Now imagine $S$ varies along the Euler line, meaning there should exist linear functions $\\alpha, \\beta, \\gamma: \\mathbb{R} \\rightarrow \\mathbb{R}$ such that $$ S=(\\alpha(s), \\beta(s), \\gamma(s)) \\quad s \\in \\mathbb{R} $$ with $\\alpha(s)+\\beta(s)+\\gamma(s)=1$. Thus $t_{a}=\\frac{f_{a}}{g_{a}}=\\frac{f_{a}(s)}{g_{a}(s)}$ is the quotient of a linear function $f_{a}(s)$ and a quadratic function $g_{a}(s)$. So we may write: $$ \\begin{aligned} X & =\\left(1-t_{a}\\right)\\left(0, \\frac{1}{2}, \\frac{1}{2}\\right)+t_{a}(\\alpha, \\beta, \\gamma) \\\\ & =\\left(t_{a} \\alpha, \\frac{1}{2}\\left(1-t_{a}\\right)+t_{a} \\beta, \\frac{1}{2}\\left(1-t_{a}\\right)+t_{a} \\gamma\\right) \\end{aligned} $$ $$ =\\left(2 f_{a} \\alpha: g_{a}-f_{a}+2 f_{a} \\beta: g_{a}-f_{a}+2 f_{a} \\gamma\\right) . $$ Thus the coordinates of $X$ are quadratic polynomials in $s$ when written in this way. In a similar way, the coordinates of $Y$ and $Z$ should be quadratic polynomials in $s$. The Ceva concurrence condition $$ \\prod_{\\text {cyc }} \\frac{g_{a}-f_{a}+2 f_{a} \\beta}{g_{a}-f_{a}+2 f_{a} \\gamma}=1 $$ is thus a polynomial in $s$ of degree at most six. Our goal is to verify it is identically zero, thus it suffices to check seven positions of $S$. - If $S$ is the circumcenter of $\\triangle M_{a} M_{b} M_{c}$ (equivalently the nine-point center of $\\triangle A B C$ ) then $\\overline{A X}, \\overline{B Y}, \\overline{C Z}$ are altitudes of $\\triangle A B C$. - If $S$ is the centroid of $\\triangle M_{a} M_{b} M_{c}$ (equivalently the centroid of $\\triangle A B C$ ), then $\\overline{A X}$, $\\overline{B Y}, \\overline{C Z}$ are medians of $\\triangle A B C$. - If $S$ is either of the intersections of the Euler line with $\\gamma$, then $S=X=Y=Z$ and all cevians concur at $S$. - If $S$ lies on the $\\overline{M_{a} M_{b}}$, then $Y=M_{a}, X=M_{c}$, and thus $\\overline{A X} \\cap \\overline{B Y}=C$, which is of course concurrent with $\\overline{C Z}$ (regardless of $Z$ ). Similarly if $S$ lies on the other sides of $\\triangle M_{a} M_{b} M_{c}$. Thus we are also done."} +{"year":2015,"label":"6","problem":"Let $A B C$ be a non-equilateral triangle and let $M_{a}, M_{b}, M_{c}$ be the midpoints of the sides $B C, C A, A B$, respectively. Let $S$ be a point lying on the Euler line. Denote by $X, Y, Z$ the second intersections of $M_{a} S, M_{b} S, M_{c} S$ with the nine-point circle. Prove that $A X, B Y, C Z$ are concurrent.","solution":" We assume now and forever that $A B C$ is scalene since the problem follows by symmetry in the isosceles case. We present four solutions. \u3010 Fourth solution using Pascal (official one). We give a different proof of the claim that $\\overline{X H_{a}}, \\overline{Y H_{b}}, \\overline{Z H_{c}}$ are concurrent (and then proceed as in the end of the second solution). Let $H$ denote the orthocenter, $N$ the nine-point center, and moreover let $N_{a}, N_{b}, N_{c}$ denote the midpoints of $\\overline{A H}, \\overline{B H}, \\overline{C H}$, which also lie on the nine-point circle (and are the antipodes of $M_{a}, M_{b}, M_{c}$ ). - By Pascal's theorem on $M_{b} N_{b} H_{b} M_{c} N_{c} H_{c}$, the point $P=\\overline{M_{c} H_{b}} \\cap \\overline{M_{b} H_{c}}$ is collinear with $N=\\overline{M_{b} N_{b}} \\cap \\overline{M_{c} N_{c}}$, and $H=\\overline{N_{b} H_{b}} \\cap \\overline{N_{c} H_{c}}$. So $P$ lies on the Euler line. - By Pascal's theorem on $M_{b} Y H_{b} M_{c} Z H_{c}$, the point $\\overline{Y H_{b}} \\cap \\overline{Z H_{c}}$ is collinear with $S=\\overline{M_{b} Y} \\cap \\overline{M_{c} Z}$ and $P=\\overline{M_{b} H_{c}} \\cap \\overline{M_{c} H_{b}}$. Hence $Y H_{b}$ and $Z H_{c}$ meet on the Euler line, as needed."} diff --git a/USA_TST/segmented/en-sols-TST-IMO-2016.jsonl b/USA_TST/segmented/en-sols-TST-IMO-2016.jsonl new file mode 100644 index 0000000000000000000000000000000000000000..6429d06fd925b17acba77ec9826ba3158ad525d8 --- /dev/null +++ b/USA_TST/segmented/en-sols-TST-IMO-2016.jsonl @@ -0,0 +1,8 @@ +{"year":2016,"label":"1","problem":"Let $S=\\{1, \\ldots, n\\}$. Given a bijection $f: S \\rightarrow S$ an orbit of $f$ is a set of the form $\\{x, f(x), f(f(x)), \\ldots\\}$ for some $x \\in S$. We denote by $c(f)$ the number of distinct orbits of $f$. For example, if $n=3$ and $f(1)=2, f(2)=1, f(3)=3$, the two orbits are $\\{1,2\\}$ and $\\{3\\}$, hence $c(f)=2$. Given $k$ bijections $f_{1}, \\ldots, f_{k}$ from $S$ to itself, prove that $$ c\\left(f_{1}\\right)+\\cdots+c\\left(f_{k}\\right) \\leq n(k-1)+c(f) $$ where $f: S \\rightarrow S$ is the composed function $f_{1} \\circ \\cdots \\circ f_{k}$.","solution":""} +{"year":2016,"label":"2","problem":"Let $A B C$ be a scalene triangle with circumcircle $\\Omega$, and suppose the incircle of $A B C$ touches $B C$ at $D$. The angle bisector of $\\angle A$ meets $B C$ and $\\Omega$ at $K$ and $M$. The circumcircle of $\\triangle D K M$ intersects the $A$-excircle at $S_{1}, S_{2}$, and $\\Omega$ at $T \\neq M$. Prove that line $A T$ passes through either $S_{1}$ or $S_{2}$.","solution":" \u3010 First solution (angle chasing). Assume for simplicity $A B
\\operatorname{deg}\\left(Q^{\\prime} P-Q P^{\\prime}\\right) \\end{aligned} $$ This can only occur if $Q^{\\prime} P-Q P^{\\prime}=0$ or $(P \/ Q)^{\\prime}=0$ by the quotient rule! But $P \/ Q$ can't be constant, the end. Remark. The result is previously known; see e.g. Lemma 1.6 of http:\/\/math.mit.edu\/ ebelmont\/ec-notes.pdf or Exercise 6.5.L(a) of Vakil's notes."} +{"year":2017,"label":"4","problem":"You are cheating at a trivia contest. For each question, you can peek at each of the $n>1$ other contestant's guesses before writing your own. For each question, after all guesses are submitted, the emcee announces the correct answer. A correct guess is worth 0 points. An incorrect guess is worth -2 points for other contestants, but only -1 point for you, because you hacked the scoring system. After announcing the correct answer, the emcee proceeds to read out the next question. Show that if you are leading by $2^{n-1}$ points at any time, then you can surely win first place.","solution":" We will prove the result with $2^{n-1}$ replaced even by $2^{n-2}+1$. We first make the following reductions. First, change the weights to be $+1,-1,0$ respectively (rather than $0,-2,-1$ ); this clearly has no effect. Also, WLOG that all contestants except you initially have score zero (and that your score exceeds $2^{n-2}$ ). WLOG ignore rounds in which all answers are the same. Finally, ignore rounds in which you get the correct answer, since that leaves you at least as well off as before - in other words, we'll assume your score is always fixed, but you can pick any group of people with the same answers and ensure they lose 1 point, while some other group gains 1 point. The key observation is the following. Consider two rounds $R_{1}$ and $R_{2}$ such that: - In round $R_{1}$, some set $S$ of contestants gains a point. - In round $R_{2}$, the set $S$ of contestants all have the same answer. Then, if we copy the answers of contestants in $S$ during $R_{2}$, then the sum of the scorings in $R_{1}$ and $R_{2}$ cancel each other out. In other words we can then ignore $R_{1}$ and $R_{2}$ forever. We thus consider the following strategy. We keep a list $\\mathcal{L}$ of subsets of $\\{1, \\ldots, n\\}$, initially empty. Now do the following strategy: - On a round, suppose there exists a set $S$ of people with the same answer such that $S \\in \\mathcal{L}$. (If multiple exist, choose one arbitrarily.) Then, copy the answer of $S$, causing them to lose a point. Delete $S$ from $\\mathcal{L}$. (Importantly, we do not add any new sets to $\\mathcal{L}$.) - Otherwise, copy any set $T$ of contestants, selecting $|T| \\geq n \/ 2$ if possible. Let $S$ be the set of contestants who answer correctly (if any), and add $S$ to the list $\\mathcal{L}$. Note that $|S| \\leq n \/ 2$, since $S$ is disjoint from $T$. By construction, $\\mathcal{L}$ has no duplicate sets. So the score of any contestant $c$ is bounded above by the number of times that $c$ appears among sets in $\\mathcal{L}$. The number of such sets is clearly at most $\\frac{1}{2} \\cdot 2^{n-1}$. So, if you lead by $2^{n-2}+1$ then you ensure victory. This completes the proof! Remark. Several remarks are in order. First, we comment on the bound $2^{n-2}+1$ itself. The most natural solution using only the list idea gives an upper bound of $\\left(2^{n}-2\\right)+1$, which is the number of nonempty proper subsets of $\\{1, \\ldots, n\\}$. Then, there are two optimizations one can observe: - In fact we can improve to the number of times any particular contestant $c$ appears in some set, rather than the total number of sets. - When adding new sets $S$ to $\\mathcal{L}$, one can ensure $|S| \\leq n \/ 2$. Either observation alone improves the bound from $2^{n}-1$ to $2^{n-1}$, but both together give the bound $2^{n-2}+1$. Additionally, when $n$ is odd the calculation of subsets actually gives $2^{n-2}-\\frac{1}{2}\\binom{n-1}{\\frac{n-1}{2}}+1$. This gives the best possible value at both $n=2$ and $n=3$. It seems likely some further improvements are possible, and the true bound is suspected to be polynomial in $n$. 1. The exponential bound $2^{n}$ suggests looking at subsets. 2. The $n=2$ case suggests the idea of \"repeated rounds\". (I think this $n=2$ case is actually really good.) 3. The \"two distinct answers\" case suggests looking at rounds as partitions (even though the WLOG does not work, at least not without further thought). 4. There's something weird about this problem: it's a finite bound over unbounded time. This is a hint to not worry excessively about the actual scores, which turn out to be almost irrelevant."} +{"year":2017,"label":"5","problem":"Let $A B C$ be a triangle with altitude $\\overline{A E}$. The $A$-excircle touches $\\overline{B C}$ at $D$, and intersects the circumcircle at two points $F$ and $G$. Prove that one can select points $V$ and $N$ on lines $D G$ and $D F$ such that quadrilateral $E V A N$ is a rhombus.","solution":" Let $I$ denote the incenter, $J$ the $A$-excenter, and $L$ the midpoint of $\\overline{A E}$. Denote by $\\overline{I Y}$, $\\overline{I Z}$ the tangents from $I$ to the $A$-excircle. Note that lines $\\overline{B C}, \\overline{G F}, \\overline{Y Z}$ then concur at $H$ (unless $A B=A C$, but this case is obvious), as it's the radical center of cyclic hexagon $B I C Y J Z$, the circumcircle and the $A$-excircle. ![](https:\/\/cdn.mathpix.com\/cropped\/2024_11_19_5d386b123511deaa59b4g-12.jpg?height=766&width=1200&top_left_y=968&top_left_x=431) Now let $\\overline{H D}$ and $\\overline{H T}$ be the tangents from $H$ to the $A$-excircle. It follows that $\\overline{D T}$ is the symmedian of $\\triangle D Z Y$, hence passes through $I=\\overline{Y Y} \\cap \\overline{Z Z}$. Moreover, it's well known that $\\overline{D I}$ passes through $L$, the midpoint of the $A$-altitude (for example by homothety). Finally, $(D T ; F G)=-1$, hence project through $D$ onto the line through $L$ parallel to $\\overline{B C}$ to obtain $(\\infty L ; V N)=-1$ as desired. \u3010 Authorship comments. This is a joint proposal with Danielle Wang (mostly by her). The formulation given was that the tangents to the $A$-excircle at $F$ and $G$ was on line $\\overline{D I}$; I solved this formulation using the radical axis argument above. I then got the idea to involve the point $L$, already knowing it was on $\\overline{D I}$. Observing the harmonic quadrilateral, I took perspectivity through $M$ onto the line through $L$ parallel to $\\overline{B C}$ (before this I had tried to use the $A$-altitude with little luck). This yields the rhombus in the problem."} +{"year":2017,"label":"6","problem":"Prove that there are infinitely many triples $(a, b, p)$ of integers, with $p$ prime and $02$, then one can again show by induction $p_{3} \\ldots p_{k} \\geq 2^{k}-1$ (since $p_{3} \\geq 7$ ), which also implies the result."} +{"year":2018,"label":"2","problem":"Find all functions $f: \\mathbb{Z}^{2} \\rightarrow[0,1]$ such that for any integers $x$ and $y$, $$ f(x, y)=\\frac{f(x-1, y)+f(x, y-1)}{2} $$","solution":" We claim that the only functions $f$ are constant functions. (It is easy to see that they work.) TI First solution (hands-on). First, iterating the functional equation relation to the $n$th level shows that $$ f(x, y)=\\frac{1}{2^{n}} \\sum_{i=0}^{n}\\binom{n}{i} f(x-i, y-(n-i)) $$ In particular, $$ \\begin{aligned} |f(x, y)-f(x-1, y+1)| & =\\frac{1}{2^{n}}\\left|\\sum_{i=0}^{n+1} f(x-i, y-(n-i)) \\cdot\\left(\\binom{n}{i}-\\binom{n}{i-1}\\right)\\right| \\\\ & \\leq \\frac{1}{2^{n}} \\sum_{i=0}^{n+1}\\left|\\binom{n}{i}-\\binom{n}{i-1}\\right| \\\\ & =\\frac{1}{2^{n}} \\cdot 2\\binom{n}{\\lfloor n \/ 2\\rfloor} \\end{aligned} $$ where we define $\\binom{n}{n+1}=\\binom{n}{-1}=0$ for convenience. Since $$ \\binom{n}{\\lfloor n \/ 2\\rfloor}=o\\left(2^{n}\\right) $$ it follows that $f$ must be constant."} +{"year":2018,"label":"2","problem":"Find all functions $f: \\mathbb{Z}^{2} \\rightarrow[0,1]$ such that for any integers $x$ and $y$, $$ f(x, y)=\\frac{f(x-1, y)+f(x, y-1)}{2} $$","solution":" We claim that the only functions $f$ are constant functions. (It is easy to see that they work.) I Second solution (random walks, Mark Sellke). We show that if $x+y=x^{\\prime}+y^{\\prime}$ then $f(x, y)=f\\left(x^{\\prime}, y^{\\prime}\\right)$. Let $Z_{n}, Z_{n}^{\\prime}$ be random walks starting at $(x, y)$ and $\\left(x^{\\prime}, y^{\\prime}\\right)$ and moving down\/left. Then $f\\left(Z_{n}\\right)$ is a martingale so we have $$ \\mathbb{E}\\left[f\\left(Z_{n}\\right)\\right]=f(x, y), \\quad \\mathbb{E}\\left[f\\left(Z_{n}^{\\prime}\\right)\\right]=f\\left(x^{\\prime}, y^{\\prime}\\right) $$ We'll take $Z_{n}, Z_{n}^{\\prime}$ to be independent until they hit each other, after which they will stay together. Then $$ \\left|\\mathbb{E}\\left[f\\left(Z_{n}\\right)-f\\left(Z_{n}^{\\prime}\\right)\\right]\\right| \\leq \\mathbb{E}\\left[\\left|f\\left(Z_{n}\\right)-f\\left(Z_{n}^{\\prime}\\right)\\right|\\right] \\leq p_{n} $$ where $p_{n}$ is the probability that $Z_{n}, Z_{n}^{\\prime}$ never collide. But the distance between $Z_{n}, Z_{n}^{\\prime}$ is essentially a 1 -dimensional random walk, so they will collide with probability 1 , meaning $\\lim _{n \\rightarrow \\infty} p_{n}=0$. Hence $$ \\left|f(x, y)-f\\left(x^{\\prime}, y^{\\prime}\\right)\\right|=\\left|\\mathbb{E}\\left[f\\left(Z_{n}\\right)-f\\left(Z_{n}^{\\prime}\\right)\\right]\\right|=o(1) $$ as desired. Remark. If the problem were in $\\mathbb{Z}^{d}$ for large $d$, this solution wouldn't work as written because the independent random walks wouldn't hit each other. However, this isn't a serious problem because $Z_{n}, Z_{n}^{\\prime}$ don't have to be independent before hitting each other. Indeed, if every time $Z_{n}, Z_{n}^{\\prime}$ agree on a new coordinate we force them to agree on that coordinate forever, we can make the two walks individually have the distribution of a coordinate-decreasing random walk but make them intersect eventually with probability 1. The difference in each coordinate will be a 1-dimensional random walk which gets stuck at 0 ."} +{"year":2018,"label":"2","problem":"Find all functions $f: \\mathbb{Z}^{2} \\rightarrow[0,1]$ such that for any integers $x$ and $y$, $$ f(x, y)=\\frac{f(x-1, y)+f(x, y-1)}{2} $$","solution":" We claim that the only functions $f$ are constant functions. (It is easy to see that they work.) I Third solution (martingales). Imagine starting at $(x, y)$ and taking a random walk down and to the left. This is a martingale. As $f$ is bounded, this martingale converges with probability 1 . Let $X_{1}, X_{2}, \\ldots$ each be random variables that represent either down moves or left moves with equal probability. Note that by the Hewitt-Savage 0-1 law, we have that for any real numbers $ax_{i+1} \\geq x_{j-1}>x_{j}$. Then replacing $\\left(x_{i}, x_{j}\\right)$ by $\\left(x_{i}-1, x_{j}+1\\right)$ strictly increases $A$ preserving all conditions. Thus we may assume all numbers in $\\left\\{x_{1}, \\ldots, x_{64}\\right\\}$ differ by at most 1 . - Suppose $x_{65} \\geq 4$. Then we can replace $\\left(x_{1}, x_{2}, x_{3}, x_{4}, x_{65}\\right)$ by $\\left(x_{1}+1, x_{2}+1, x_{3}+\\right.$ $\\left.1, x_{4}+1, x_{65}-4\\right)$ and strictly increase $A$. Hence we may assume $x_{65} \\leq 3$. We will also tacitly assume $\\sum x_{i}=4034$, since otherwise we can increase $x_{1}$. These two properties leave only four sequences to examine: - $x_{1}=x_{2}=x_{3}=\\cdots=x_{63}=63, x_{64}=62$, and $x_{65}=3$, which gives $A=126948$. - $x_{1}=x_{2}=x_{3}=\\cdots=x_{63}=x_{64}=63$ and $x_{65}=2$, which gives $A=127010$. - $x_{1}=64, x_{2}=x_{3}=\\cdots=x_{63}=x_{64}=63$ and $x_{65}=1$, which gives $A=127009$. - $x_{1}=x_{2}=64, x_{3}=\\cdots=x_{63}=x_{64}=63$ and $x_{65}=0$, which gives $A=127009$. This proves that $A \\leq 127010$. To see that equality occurs only in the second case above, note that all the smoothing operations other than incrementing $x_{1}$ were strict, and that $x_{1}$ could not have been incremented in this way as $x_{1}=x_{2}=63$. This shows that $S(G) \\leq 127010$ for all graphs $G$, so it remains to show equality never occurs. Retain the notation $d_{i}$ and $a_{i}$ of the combinatorial bound now; we would need to have $d_{1}=\\cdots=d_{64}=63$ and $d_{65}=2$ (in particular, deleting isolated vertices from $G$, we may assume $n=65$ ). In that case, we have $a_{i} \\leq i-1$ but also $a_{65}=2$ by definition (the last vertex gets all edges associated to it). Finally, $$ \\begin{aligned} S(G) & =\\sum_{i=1}^{n} a_{i} d_{i}=63\\left(a_{1}+\\cdots+a_{64}\\right)+a_{65} \\\\ & =63\\left(2017-a_{65}\\right)+a_{65} \\leq 63 \\cdot 2015+2=126947 \\end{aligned} $$ Remark. Another way to finish once $S(G) \\leq 127010$ is note there is a unique graph (up to isomorphism and deletion of universal vertices) with degree sequence $\\left(d_{1}, \\ldots, d_{65}\\right)=$ $(63, \\ldots, 63,2)$. Indeed, the complement of the graph has degree sequence $(1, \\ldots, 1,63)$, and so it must be a 63 -star plus a single edge. One can then compute $S(G)$ explicitly for this graph. ## \u3010 Some further remarks. Remark. Interestingly, the graph $C_{4}$ has $\\binom{3}{2}+1=4$ edges and $S\\left(C_{4}\\right)=8$, while $S\\left(L_{3}\\right)=7$. This boundary case is visible in the combinatorial solution in the base case of the first claim. It also explains why we end up with the bound $S(G) \\leq 127010$ in the second algebraic solution, and why it is necessary to analyze the equality cases so carefully; observe in $k=3$ the situation $d_{1}=d_{2}=d_{3}=d_{4}=2$. Remark. Some comments about further context for this problem: - The obvious generalization of 2017 to any constant was resolved in September 2018 by Mehtaab Sawhney and Ashwin Sah. The relevant paper is On the discrepancy between two Zagreb indices, published in Discrete Mathematics, Volume 341, Issue 9, pages 2575-2589. The arXiv link is https:\/\/arxiv.org\/pdf\/1801.02532.pdf. - The quantity $$ S(G)=\\sum_{e=v w} \\min (\\operatorname{deg} v, \\operatorname{deg} w) $$ in the problem has an interpretation: it can be used to provide a bound on the number of triangles in a graph $G$. To be precise, $\\# E(G) \\leq \\frac{1}{3} S(G)$, since an edge $e=v w$ is part of at most $\\min (\\operatorname{deg} v, \\operatorname{deg} w)$ triangles. - For planar graphs it is known $S(G) \\leq 18 n-36$ and it is conjectured that for $n$ large enough, $S(G) \\leq 18 n-72$. See https:\/\/mathoverflow.net\/a\/273694\/70654. I Authorship comments. I came up with the quantity $S(G)$ in a failed attempt to provide a bound on the number of triangles in a graph, since this is natural to consider when you do a standard double-counting via the edges of the triangle. I think the problem was actually APMO 1989, and I ended up not solving the problem (the solution is much simpler), but the quantity $S(G)$ stuck in my head for a while after that. Later on that month I was keeping Danielle company while she was working on art project (flower necklace), and with not much to do except doodle on tables I began thinking about $S(G)$ again. I did have the sense that $S(G)$ should be maximized at a graph close to a complete graph. But to my frustration I could not prove it for a long time. Finally after many hours of trying various approaches I was able to at least show that $S(G)$ was maximized for complete graphs if the number of edges was a triangular number. I had come up with this in March 2016, which would have been perfect since 2016 is a triangular number, but it was too late to submit it to any contest (the USAMO and IMO deadlines were long past). So on December 31, 2016 I finally sat down and solved it for the case 2017, which took another few hours of thought, then submitted it to that year's IMO. To my dismay it was rejected, but I passed it along to the USA TST after that, thus making it just in time for the close of the calendar year."} +{"year":2018,"label":"4","problem":"Let $n$ be a positive integer and let $S \\subseteq\\{0,1\\}^{n}$ be a set of binary strings of length $n$. Given an odd number $x_{1}, \\ldots, x_{2 k+1} \\in S$ of binary strings (not necessarily distinct), their majority is defined as the binary string $y \\in\\{0,1\\}^{n}$ for which the $i^{\\text {th }}$ bit of $y$ is the most common bit among the $i^{\\text {th }}$ bits of $x_{1}, \\ldots, x_{2 k+1}$. (For example, if $n=4$ the majority of $0000,0000,1101,1100,0101$ is 0100 .) Suppose that for some positive integer $k, S$ has the property $P_{k}$ that the majority of any $2 k+1$ binary strings in $S$ (possibly with repetition) is also in $S$. Prove that $S$ has the same property $P_{k}$ for all positive integers $k$.","solution":" Let $M$ denote the majority function (of any length). \u3010 First solution (induction). We prove all $P_{k}$ are equivalent by induction on $n \\geq 2$, with the base case $n=2$ being easy to check by hand. (The case $n=1$ is also vacuous; however, the inductive step is not able to go from $n=1$ to $n=2$.) For the inductive step, we proceed by contradiction; assume $S$ satisfies $P_{\\ell}$, but not $P_{k}$, so there exist $x_{1}, \\ldots, x_{2 k+1} \\in S$ whose majority $y=M\\left(x_{1}, \\ldots, x_{k}\\right)$ is not in $S$. We contend that: Claim - Let $y_{i}$ be the string which differs from $y$ only in the $i^{\\text {th }}$ bit. Then $y_{i} \\in S$. $$ T=\\{\\hat{s} \\mid s \\in S\\} $$ Since $S$ satisfies $P_{\\ell}$, so does $T$; thus by the induction hypothesis on $n, T$ satisfies $P_{k}$. Consequently, $T \\ni M\\left(\\hat{x}_{1}, \\ldots, \\hat{x}_{2 k+1}\\right)=\\hat{y}$. Thus there exists $s \\in S$ such that $\\hat{s}=\\hat{y}$. This implies $s=y$ or $s=y_{i}$. But since we assumed $y \\notin S$ it follows $y_{i} \\in S$ instead. Now take any $2 \\ell+1$ copies of the $y_{i}$, about equally often (i.e. the number of times any two $y_{i}$ are taken differs by at most 1 ). We see the majority of these is $y$ itself, contradiction."} +{"year":2018,"label":"4","problem":"Let $n$ be a positive integer and let $S \\subseteq\\{0,1\\}^{n}$ be a set of binary strings of length $n$. Given an odd number $x_{1}, \\ldots, x_{2 k+1} \\in S$ of binary strings (not necessarily distinct), their majority is defined as the binary string $y \\in\\{0,1\\}^{n}$ for which the $i^{\\text {th }}$ bit of $y$ is the most common bit among the $i^{\\text {th }}$ bits of $x_{1}, \\ldots, x_{2 k+1}$. (For example, if $n=4$ the majority of $0000,0000,1101,1100,0101$ is 0100 .) Suppose that for some positive integer $k, S$ has the property $P_{k}$ that the majority of any $2 k+1$ binary strings in $S$ (possibly with repetition) is also in $S$. Prove that $S$ has the same property $P_{k}$ for all positive integers $k$.","solution":" Let $M$ denote the majority function (of any length). \u3010 Second solution (circuit construction). Note that $P_{k} \\Longrightarrow P_{1}$ for any $k$, since $$ M(\\underbrace{a, \\ldots, a}_{k}, \\underbrace{b, \\ldots, b}_{k}, c)=M(a, b, c) $$ for any $a, b, c$. We will now prove $P_{1}+P_{k} \\Longrightarrow P_{k+1}$ for any $k$, which will prove the result. Actually, we will show that the majority of any $2 k+3$ strings $x_{1}, \\ldots, x_{2 k+3}$ can be expressed by 3 and $(2 k+1)$-majorities. WLOG assume that $M\\left(x_{1}, \\ldots, x_{2 k+3}\\right)=0 \\ldots 0$, and let $\\odot$ denote binary AND. Claim - We have $M\\left(x_{1}, x_{2}, M\\left(x_{3}, \\ldots, x_{2 k+3}\\right)\\right)=x_{1} \\odot x_{2}$. By analogy we can construct any $x_{i} \\odot x_{j}$. Finally, note that $$ M\\left(x_{1} \\odot x_{2}, x_{2} \\odot x_{3}, \\ldots, x_{2 k+1} \\odot x_{2 k+2}\\right)=0 \\ldots 0 $$ as desired. (Indeed, if we look at any index, there were at most $k+1$ 's in the $x_{i}$ strings, and hence there will be at most $k$ 's among $x_{i} \\odot x_{i+1}$ for $i=1, \\ldots, 2 k+1$.)"} +{"year":2018,"label":"5","problem":"Let $A B C D$ be a convex cyclic quadrilateral which is not a kite, but whose diagonals are perpendicular and meet at $H$. Denote by $M$ and $N$ the midpoints of $\\overline{B C}$ and $\\overline{C D}$. Rays $M H$ and $N H$ meet $\\overline{A D}$ and $\\overline{A B}$ at $S$ and $T$, respectively. Prove there exists a point $E$, lying outside quadrilateral $A B C D$, such that - ray $E H$ bisects both angles $\\angle B E S, \\angle T E D$, and - $\\angle B E N=\\angle M E D$.","solution":" The main claim is that $E$ is the intersection of $(A B C D)$ with the circle with diameter $\\overline{A H}$. ![](https:\/\/cdn.mathpix.com\/cropped\/2024_11_19_420dc3a54c986f92a77dg-14.jpg?height=1012&width=786&top_left_y=1050&top_left_x=635) The following observation can be quickly made without reference to $E$. ## Lemma We have $\\angle H S A=\\angle H T A=90^{\\circ}$. Consequently, quadrilateral $B T S D$ is cyclic. From here it follows that $E$ is the Miquel point of cyclic quadrilateral $B T S D$. Define $F$ to be the point diametrically opposite $A$, so that $E, H, F$ are collinear, $\\overline{C F} \\| \\overline{B D}$. By now we already have $$ \\measuredangle B E H=\\measuredangle B E F=\\measuredangle B A F=\\measuredangle C A D=\\measuredangle H A S=\\measuredangle H E S $$ so $\\overline{E H}$ bisects $\\angle B E S$, and $\\angle T E D$. Hence it only remains to show $\\angle B E M=\\angle N E D$; we present several proofs below. I First proof (original solution). Let $P$ be the circumcenter of $B T S D$. The properties of the Miquel point imply $P$ lies on the common bisector $\\overline{E H}$ already, and it also lies on the perpendicular bisector of $\\overline{B D}$, hence it must be the midpoint of $\\overline{H F}$. We now contend quadrilaterals $B M P S$ and $D N P T$ are cyclic. Obviously $\\overline{M P}$ is the external angle bisector of $\\angle B M S$, and $P B=P S$, so $P$ is the arc midpoint of $(B M S)$. The proof for $D N P T$ is analogous. It remains to show $\\angle B E N=\\angle M E D$, or equivalently $\\angle B E M=\\angle N E D$. By properties of Miquel point we have $E \\in(B M P S) \\cap(T P N D)$, so $$ \\measuredangle B E M=\\measuredangle B P M=\\measuredangle P B D=\\measuredangle B D P=\\measuredangle N P D=\\measuredangle N E D $$ as desired."} +{"year":2018,"label":"5","problem":"Let $A B C D$ be a convex cyclic quadrilateral which is not a kite, but whose diagonals are perpendicular and meet at $H$. Denote by $M$ and $N$ the midpoints of $\\overline{B C}$ and $\\overline{C D}$. Rays $M H$ and $N H$ meet $\\overline{A D}$ and $\\overline{A B}$ at $S$ and $T$, respectively. Prove there exists a point $E$, lying outside quadrilateral $A B C D$, such that - ray $E H$ bisects both angles $\\angle B E S, \\angle T E D$, and - $\\angle B E N=\\angle M E D$.","solution":" The main claim is that $E$ is the intersection of $(A B C D)$ with the circle with diameter $\\overline{A H}$. ![](https:\/\/cdn.mathpix.com\/cropped\/2024_11_19_420dc3a54c986f92a77dg-14.jpg?height=1012&width=786&top_left_y=1050&top_left_x=635) The following observation can be quickly made without reference to $E$. ## Lemma We have $\\angle H S A=\\angle H T A=90^{\\circ}$. Consequently, quadrilateral $B T S D$ is cyclic. From here it follows that $E$ is the Miquel point of cyclic quadrilateral $B T S D$. Define $F$ to be the point diametrically opposite $A$, so that $E, H, F$ are collinear, $\\overline{C F} \\| \\overline{B D}$. By now we already have $$ \\measuredangle B E H=\\measuredangle B E F=\\measuredangle B A F=\\measuredangle C A D=\\measuredangle H A S=\\measuredangle H E S $$ so $\\overline{E H}$ bisects $\\angle B E S$, and $\\angle T E D$. Hence it only remains to show $\\angle B E M=\\angle N E D$; we present several proofs below. \u3010 Second proof (2011 G4). By 2011 G 4 , the circumcircle of $\\triangle E M N$ is tangent to the circumcircle of $A B C D$. Hence if we extend $\\overline{E M}$ and $\\overline{E N}$ to meet $(A B C D)$ again at $X$ and $Y$, we get $\\overline{X Y}\\|\\overline{M N}\\| \\overline{B D}$. Thus $\\measuredangle B E M=\\measuredangle B E X=\\measuredangle Y E D=\\measuredangle N E D$."} +{"year":2018,"label":"5","problem":"Let $A B C D$ be a convex cyclic quadrilateral which is not a kite, but whose diagonals are perpendicular and meet at $H$. Denote by $M$ and $N$ the midpoints of $\\overline{B C}$ and $\\overline{C D}$. Rays $M H$ and $N H$ meet $\\overline{A D}$ and $\\overline{A B}$ at $S$ and $T$, respectively. Prove there exists a point $E$, lying outside quadrilateral $A B C D$, such that - ray $E H$ bisects both angles $\\angle B E S, \\angle T E D$, and - $\\angle B E N=\\angle M E D$.","solution":" The main claim is that $E$ is the intersection of $(A B C D)$ with the circle with diameter $\\overline{A H}$. ![](https:\/\/cdn.mathpix.com\/cropped\/2024_11_19_420dc3a54c986f92a77dg-14.jpg?height=1012&width=786&top_left_y=1050&top_left_x=635) The following observation can be quickly made without reference to $E$. ## Lemma We have $\\angle H S A=\\angle H T A=90^{\\circ}$. Consequently, quadrilateral $B T S D$ is cyclic. From here it follows that $E$ is the Miquel point of cyclic quadrilateral $B T S D$. Define $F$ to be the point diametrically opposite $A$, so that $E, H, F$ are collinear, $\\overline{C F} \\| \\overline{B D}$. By now we already have $$ \\measuredangle B E H=\\measuredangle B E F=\\measuredangle B A F=\\measuredangle C A D=\\measuredangle H A S=\\measuredangle H E S $$ so $\\overline{E H}$ bisects $\\angle B E S$, and $\\angle T E D$. Hence it only remains to show $\\angle B E M=\\angle N E D$; we present several proofs below. I Third proof (involutions, submitted by Daniel Liu). Let $G=\\overline{B N} \\cap \\overline{M D}$ denote the centroid of $\\triangle B C D$, and note that it lies on $\\overline{E H F}$. Now consider the dual of Desargues involution theorem on complete quadrilateral $B M D N C G$ at point $E$. We get $$ (E B, E D), \\quad(E M, E N), \\quad(E C, E G) $$ form an involutive pairing. However, the bisector of $\\angle B E D$, say $\\ell$, is also the angle bisector of $\\angle C E F$ (since $\\overline{C F} \\| \\overline{B D})$. So the involution we found must coincide with reflection across $\\ell$. This means $\\angle M E N$ is bisected by $\\ell$ as well, as desired. \u3010 Authorship comments. This diagram actually comes from the inverted picture in IMO 2014\/3 (which I attended). I had heard for many years that one could solve this problem quickly by inversion at $H$ afterwards. But when I actually tried to do it during an OTIS class years later, I ended up with the picture in the TST problem, and couldn't see why it was true! In the process of trying to reconstruct this rumored solution, I ended up finding most of the properties that ended up in the January TST problem (but were overkill for the original IMO problem). Let us make the equivalence explicit by deducing the IMO problem from our work. Let rays $E M$ and $E N$ meet the circumcircles of $\\triangle B H C$ and $\\triangle B N C$ again at $X$ and $Y$, with $E Me$, as needed. Assume to the contrary that $p \\leq 101$ is a prime dividing $n$, and a valiant $g: \\mathbb{Z} \/ n \\mathbb{Z} \\rightarrow$ $\\mathbb{Z} \/ n \\mathbb{Z}$ exists. Take $k=p-1$ in Lemma I' to contradict Lemma II'"} +{"year":2019,"label":"2","problem":"Let $\\mathbb{Z} \/ n \\mathbb{Z}$ denote the set of integers considered modulo $n$ (hence $\\mathbb{Z} \/ n \\mathbb{Z}$ has $n$ elements). Find all positive integers $n$ for which there exists a bijective function $g: \\mathbb{Z} \/ n \\mathbb{Z} \\rightarrow \\mathbb{Z} \/ n \\mathbb{Z}$, such that the 101 functions $$ g(x), \\quad g(x)+x, \\quad g(x)+2 x, \\quad \\ldots, \\quad g(x)+100 x $$ are all bijections on $\\mathbb{Z} \/ n \\mathbb{Z}$.","solution":" Call a function $g$ valiant if it obeys this condition. We claim the answer is all numbers relatively prime to 101 !. The construction is to just let $g$ be the identity function. Remark (Motivation for both parts). The following solution is dense, and it is easier to think about some small cases first, to motivate the ideas. We consider the result where 101 is replaced by 2 or 3 . - If we replaced 101 with 2 , you can show $2 \\nmid n$ easily: write $$ \\sum_{x} x \\equiv \\sum_{x} g(x) \\equiv \\sum_{x}(g(x)+x) \\quad(\\bmod n) $$ which implies $$ 0 \\equiv \\sum_{x} x=\\frac{1}{2} n(n+1) \\quad(\\bmod n) $$ which means $\\frac{1}{2} n(n+1) \\equiv 0(\\bmod n)$, hence $n$ odd. - If we replaced 101 with 3 , then you can try a similar approach using squares, since $$ \\begin{aligned} 0 & \\equiv \\sum_{x}\\left[(g(x)+2 x)^{2}-2(g(x)+x)^{2}+g(x)^{2}\\right] \\quad(\\bmod n) \\\\ & =\\sum_{x} 2 x^{2}=2 \\cdot \\frac{n(n+1)(2 n+1)}{6} \\end{aligned} $$ which is enough to force $3 \\nmid n$. \u3010 A third remixed solution. We use Lemma I and Lemma II' from before. As before, assume $g: \\mathbb{Z} \/ n \\mathbb{Z} \\rightarrow \\mathbb{Z} \/ n \\mathbb{Z}$ is valiant, and $n$ has a prime divisor $p \\leq 101$. Also, let $e=\\nu_{p}(n)$. Then $(p-1)!\\sum_{x} x^{p-1} \\equiv 0(\\bmod n)$ by Lemma I, and now $$ \\begin{aligned} 0 & \\equiv \\sum_{x} x^{p-1} \\quad\\left(\\bmod p^{e}\\right) \\\\ & \\equiv \\frac{n}{p^{e}} \\sum_{x=1}^{p^{e}-1} x^{p-1} \\not \\equiv 0 \\quad\\left(\\bmod p^{e}\\right) \\end{aligned} $$ by Lemma II', contradiction."} +{"year":2019,"label":"2","problem":"Let $\\mathbb{Z} \/ n \\mathbb{Z}$ denote the set of integers considered modulo $n$ (hence $\\mathbb{Z} \/ n \\mathbb{Z}$ has $n$ elements). Find all positive integers $n$ for which there exists a bijective function $g: \\mathbb{Z} \/ n \\mathbb{Z} \\rightarrow \\mathbb{Z} \/ n \\mathbb{Z}$, such that the 101 functions $$ g(x), \\quad g(x)+x, \\quad g(x)+2 x, \\quad \\ldots, \\quad g(x)+100 x $$ are all bijections on $\\mathbb{Z} \/ n \\mathbb{Z}$.","solution":" Call a function $g$ valiant if it obeys this condition. We claim the answer is all numbers relatively prime to 101 !. The construction is to just let $g$ be the identity function. Remark (Motivation for both parts). The following solution is dense, and it is easier to think about some small cases first, to motivate the ideas. We consider the result where 101 is replaced by 2 or 3 . - If we replaced 101 with 2 , you can show $2 \\nmid n$ easily: write $$ \\sum_{x} x \\equiv \\sum_{x} g(x) \\equiv \\sum_{x}(g(x)+x) \\quad(\\bmod n) $$ which implies $$ 0 \\equiv \\sum_{x} x=\\frac{1}{2} n(n+1) \\quad(\\bmod n) $$ which means $\\frac{1}{2} n(n+1) \\equiv 0(\\bmod n)$, hence $n$ odd. - If we replaced 101 with 3 , then you can try a similar approach using squares, since $$ \\begin{aligned} 0 & \\equiv \\sum_{x}\\left[(g(x)+2 x)^{2}-2(g(x)+x)^{2}+g(x)^{2}\\right] \\quad(\\bmod n) \\\\ & =\\sum_{x} 2 x^{2}=2 \\cdot \\frac{n(n+1)(2 n+1)}{6} \\end{aligned} $$ which is enough to force $3 \\nmid n$. \u3010I A fourth remixed solution. We also can combine Lemma I' and Lemma II. As before, assume $g: \\mathbb{Z} \/ n \\mathbb{Z} \\rightarrow \\mathbb{Z} \/ n \\mathbb{Z}$ is valiant, and let $p$ be the smallest prime divisor of $n$. Assume for contradiction $p \\leq 101$. By Lemma I' we have $$ \\sum_{x} x^{k} \\equiv 0 \\quad\\left(\\bmod p^{e}\\right) $$ for $k=0, \\ldots, p-1$. This directly contradicts Lemma II with $M=p^{e}$."} +{"year":2019,"label":"3","problem":"A snake of length $k$ is an animal which occupies an ordered $k$-tuple $\\left(s_{1}, \\ldots, s_{k}\\right)$ of cells in an $n \\times n$ grid of square unit cells. These cells must be pairwise distinct, and $s_{i}$ and $s_{i+1}$ must share a side for $i=1, \\ldots, k-1$. If the snake is currently occupying $\\left(s_{1}, \\ldots, s_{k}\\right)$ and $s$ is an unoccupied cell sharing a side with $s_{1}$, the snake can move to occupy ( $s, s_{1}, \\ldots, s_{k-1}$ ) instead. The snake has turned around if it occupied $\\left(s_{1}, s_{2}, \\ldots, s_{k}\\right)$ at the beginning, but after a finite number of moves occupies $\\left(s_{k}, s_{k-1}, \\ldots, s_{1}\\right)$ instead. Determine whether there exists an integer $n>1$ such that one can place some snake of length at least $0.9 n^{2}$ in an $n \\times n$ grid which can turn around.","solution":null} +{"year":2019,"label":"4","problem":"We say a function $f: \\mathbb{Z}_{\\geq 0} \\times \\mathbb{Z}_{\\geq 0} \\rightarrow \\mathbb{Z}$ is great if for any nonnegative integers $m$ and $n$, $$ f(m+1, n+1) f(m, n)-f(m+1, n) f(m, n+1)=1 $$ If $A=\\left(a_{0}, a_{1}, \\ldots\\right)$ and $B=\\left(b_{0}, b_{1}, \\ldots\\right)$ are two sequences of integers, we write $A \\sim B$ if there exists a great function $f$ satisfying $f(n, 0)=a_{n}$ and $f(0, n)=b_{n}$ for every nonnegative integer $n$ (in particular, $a_{0}=b_{0}$ ). Prove that if $A, B, C$, and $D$ are four sequences of integers satisfying $A \\sim B$, $B \\sim C$, and $C \\sim D$, then $D \\sim A$.","solution":" \u3010 First solution (Nikolai Beluhov). Let $k=a_{0}=b_{0}=c_{0}=d_{0}$. We let $f, g, h$ be great functions for $(A, B),(B, C),(C, D)$ and write the following infinite array: $$ \\left[\\begin{array}{ccccccc} & \\vdots & \\vdots & b_{3} & \\vdots & \\vdots & \\\\ \\cdots & g(2,2) & g(2,1) & b_{2} & f(1,2) & f(2,2) & \\cdots \\\\ \\cdots & g(1,2) & g(1,1) & b_{1} & f(1,1) & f(2,1) & \\cdots \\\\ c_{3} & c_{2} & c_{1} & k & a_{1} & a_{2} & a_{3} \\\\ \\cdots & h(2,1) & h(1,1) & d_{1} & & & \\\\ \\cdots & h(2,2) & h(1,2) & d_{2} & & & \\\\ & \\vdots & \\vdots & d_{3} & & & \\ddots \\end{array}\\right] $$ The greatness condition is then equivalent to saying that any $2 \\times 2$ sub-grid has determinant $\\pm 1$ (the sign is +1 in two quadrants and -1 in the other two), and we wish to fill in the lower-right quadrant. To this end, it suffices to prove the following. ## Lemma Suppose we have a $3 \\times 3$ sub-grid $$ \\left[\\begin{array}{lll} a & b & c \\\\ x & y & z \\\\ p & q & \\end{array}\\right] $$ satisfying the determinant conditions. Then we can fill in the ninth entry in the lower right with an integer while retaining greatness. If $y=0$ we have $-1=b z=b x=x q$, hence $q z=-1$, and we can fill in the entry arbitrarily. Otherwise, we have $b x \\equiv x q \\equiv b z \\equiv-1(\\bmod y)$. This is enough to imply $q z \\equiv-1$ $(\\bmod y)$, and so we can fill in the integer $\\frac{q z+1}{y}$. Remark. In this case (of all +1 determinants), I think it turns out the bottom entry is exactly equal to $q z a-c y p-c-p$, which is obviously an integer."} +{"year":2019,"label":"4","problem":"We say a function $f: \\mathbb{Z}_{\\geq 0} \\times \\mathbb{Z}_{\\geq 0} \\rightarrow \\mathbb{Z}$ is great if for any nonnegative integers $m$ and $n$, $$ f(m+1, n+1) f(m, n)-f(m+1, n) f(m, n+1)=1 $$ If $A=\\left(a_{0}, a_{1}, \\ldots\\right)$ and $B=\\left(b_{0}, b_{1}, \\ldots\\right)$ are two sequences of integers, we write $A \\sim B$ if there exists a great function $f$ satisfying $f(n, 0)=a_{n}$ and $f(0, n)=b_{n}$ for every nonnegative integer $n$ (in particular, $a_{0}=b_{0}$ ). Prove that if $A, B, C$, and $D$ are four sequences of integers satisfying $A \\sim B$, $B \\sim C$, and $C \\sim D$, then $D \\sim A$.","solution":" Second solution (Ankan Bhattacharya). We will give an explicit classification of great sequences: ## Lemma The pair $(A, B)$ is great if and only if $a_{0}=b_{0}, a_{0} \\mid a_{1} b_{1}+1$, and $a_{n} \\mid a_{n-1}+a_{n+1}$ and $b_{n} \\mid b_{n-1}+b_{n+1}$ for all $n$. Now, focus on six entries $f(x, y)$ with $x \\in\\{n-1, n, n+1\\}$ and $y \\in\\{0,1\\}$. Let $f(n-1,1)=u, f(n, 1)=v$, and $f(n+1,1)=w$, so $$ \\begin{aligned} v a_{n-1}-u a_{n} & =1 \\\\ w a_{n}-v a_{n+1} & =1 \\end{aligned} $$ Then $$ u+w=\\frac{v\\left(a_{n-1}+a_{n+1}\\right)}{a_{n}} $$ and from above $\\operatorname{gcd}\\left(v, a_{n}\\right)=1$, so $a_{n} \\mid a_{n-1}+a_{n+1}$; similarly for $b_{n}$. (If $a_{n}=0$, we have $v a_{n-1}=1$ and $v a_{n+1}=-1$, so this is OK.) First we set $f(1,1)$ so that $a_{0} f(1,1)=a_{1} b_{1}+1$, which is possible. Consider an uninitialized $f(s, t)$; without loss of generality suppose $s \\geq 2$. Then we know five values of $f$ and wish to set a sixth one $z$, as in the matrix below: $$ \\begin{array}{cc} u & x \\\\ v & y \\\\ w & z \\end{array} $$ (We imagine $a$-indices to increase southwards and $b$-indices to increase eastwards.) If $v \\neq 0$, then the choice $y \\cdot \\frac{u+w}{v}-x$ works as $u y-v x=1$. If $v=0$, it easily follows that $\\{u, w\\}=\\{1,-1\\}$ and $y=w$ as $y w=1$. Then we set the uninitialized entry to anything. Now we verify that this is compatible with the inductive hypothesis. From the determinant 1 condition, it easily follows that $\\operatorname{gcd}(w, z)=\\operatorname{gcd}(v, z)=1$. The proof that $y \\mid x+z$ is almost identical to a step performed in the \"necessary\" part of the lemma and we do not repeat it here. By induction, a desired great function $f$ exists. Remark. To simplify the problem, we may restrict the codomain of great functions and elements in great pairs of sequences to $\\mathbb{Z}_{>0}$. This allows the parts of the solution dealing with zero entries to be ignored. $$ f(x+1, y+1) f(x, y)-f(x+1, y) f(x, y+1)=-1 $$ A quick counterexample to transitivity of $\\sim$ as is without the condition $f(0,0)=1$, for concreteness: let $a_{n}=c_{n}=3+n$ and $b_{n}=3+2 n$ for $n \\geq 0$."} +{"year":2019,"label":"5","problem":"Let $n$ be a positive integer. Tasty and Stacy are given a circular necklace with $3 n$ sapphire beads and $3 n$ turquoise beads, such that no three consecutive beads have the same color. They play a cooperative game where they alternate turns removing three consecutive beads, subject to the following conditions: - Tasty must remove three consecutive beads which are turquoise, sapphire, and turquoise, in that order, on each of his turns. - Stacy must remove three consecutive beads which are sapphire, turquoise, and sapphire, in that order, on each of her turns. They win if all the beads are removed in $2 n$ turns. Prove that if they can win with Tasty going first, they can also win with Stacy going first.","solution":" In the necklace, we draw a divider between any two beads of the same color. Unless there are no dividers, this divides the necklace into several zigzags in which the beads in each zigzag alternate. Each zigzag has two endpoints (adjacent to dividers). Observe that the condition about not having three consecutive matching beads is equivalent to saying there are no zigzags of lengths 1. ![](https:\/\/cdn.mathpix.com\/cropped\/2024_11_19_a0a7a76bd95576fab5b9g-17.jpg?height=712&width=715&top_left_y=1426&top_left_x=676) The main claim is that the game is winnable (for either player going first) if and only if there are at most $2 n$ dividers. We prove this in two parts, the first part not using the hypothesis about three consecutive letters. Claim - The game cannot be won with Tasty going first if there are more than $2 n$ dividers. Consider removing a TST in some zigzag (necessarily of length at least 3). We illustrate the three possibilities in the following table, with Tasty's move shown in red. | Before | After | Change | | :---: | :---: | :---: | | . ST \\| TST | TS . | ST \\| TS . | One less divider; two zigzags merge | | . . ST \\| TSTST . | STST | One less divider; two zigzags merge | | .STSTS . . | ..S\\|S... | One more divider; a zigzag splits in t | The analysis for Stacy's move is identical. Claim - If there are at most $2 n$ dividers and there are no zigzags of length 1 then the game can be won (with either player going first). At any point if there are no dividers at all, then the necklace alternates $T S T S T \\ldots$ and the game can be won. So we will prove that on each of Tasty's turns, if there exists at least one divider, then Tasty and Stacy can each make a move at an endpoint of some zigzag (i.e. the first two cases above). As we saw in the previous proof, such moves will (a) decrease the number of dividers by exactly one, (b) not introduce any singleton zigzags (because the old zigzags merge, rather than split). Since there are fewer than $2 n$ dividers, our duo can eliminate all dividers and then win. Note that as the number of $S$ and $T$ 's are equal, there must be an equal number of - zigzags of odd length ( $\\geq 3$ ) with $T$ at the endpoints (i.e. one more $T$ than $S$ ), and - zigzags of odd length $(\\geq 3)$ with $S$ at the endpoints (i.e. one more $S$ than $T$ ). Now iff there is at least one of each, then Tasty removes a TST from the end of such a zigzag while Stacy removes an $S T S$ from the end of such a zigzag. Otherwise suppose all zigzags have even size. Then Tasty finds any zigzag of length $\\geq 4$ (which must exist since the average zigzag length is 3 ) and removes TST from the end containing $T$. The resulting merged zigzag is odd and hence $S$ endpoints, hence Stacy can move as well. Remark. There are many equivalent ways to phrase the solution. For example, the number of dividers is equal to the number of pairs of two consecutive letters (rather than singleton letters). So the win condition can also be phrased in terms of the number of adjacent pairs of letters being at least $2 n$, or equivalently the number of differing pairs being at least $4 n$. If one thinks about the game as a process, this is a natural \"monovariant\" to consider anyways, so the solution is not so unmotivated. Remark. The constraint of no three consecutive identical beads is actually needed: a counterexample without this constraint is TTSTSTSTTSSS. (They win if Tasty goes first and lose if Stacy goes first.) Remark (Why induction is unlikely to work). Many contestants attempted induction. However, in doing so they often implicitly proved a different problem: \"prove that if they can win with Tasty going first without ever creating a triplet, they can also win in such a way with Stacy going first\". This essentially means nearly all induction attempts fail. Amusingly, even the modified problem (which is much more amenable to induction) sill seems difficult without some sort of global argument. Consider a position in which Tasty wins going first, with the sequence of winning moves being Tasty's first move in red below and Stacy's second move in blue below: ![](https:\/\/cdn.mathpix.com\/cropped\/2024_11_19_a0a7a76bd95576fab5b9g-19.jpg?height=144&width=449&top_left_y=319&top_left_x=826) There is no \"nearby\" STS that Stacy can remove instead on her first turn, without introducing a triple- $T$ and also preventing Tasty from taking a TST. So it does not seem possible to easily change a Tasty-first winning sequence to a Stacy-first one, even in the modified version."} +{"year":2019,"label":"6","problem":"Let $A B C$ be a triangle with incenter $I$, and let $D$ be a point on line $B C$ satisfying $\\angle A I D=90^{\\circ}$. Let the excircle of triangle $A B C$ opposite the vertex $A$ be tangent to $\\overline{B C}$ at point $A_{1}$. Define points $B_{1}$ on $\\overline{C A}$ and $C_{1}$ on $\\overline{A B}$ analogously, using the excircles opposite $B$ and $C$, respectively. Prove that if quadrilateral $A B_{1} A_{1} C_{1}$ is cyclic, then $\\overline{A D}$ is tangent to the circumcircle of $\\triangle D B_{1} C_{1}$.","solution":" \u3010 First solution using spiral similarity (Ankan Bhattacharya). First, we prove the part of the problem which does not depend on the condition $A B_{1} A_{1} C_{1}$ is cyclic. ## Lemma Let $A B C$ be a triangle and define $I, D, B_{1}, C_{1}$ as in the problem. Moreover, let $M$ denote the midpoint of $\\overline{A D}$. Then $\\overline{A D}$ is tangent to $\\left(A B_{1} C_{1}\\right)$, and moreover $\\overline{B_{1} C_{1}} \\| \\overline{I M}$. Note that $A, Z, D$ are collinear, by radical axis on $(A B C),(A F I E),(B I C)$. ![](https:\/\/cdn.mathpix.com\/cropped\/2024_11_19_a0a7a76bd95576fab5b9g-20.jpg?height=849&width=1109&top_left_y=1583&top_left_x=476) Then the spiral similarity gives us $$ \\frac{Z F}{Z E}=\\frac{B F}{C E}=\\frac{A C_{1}}{A B_{1}} $$ which together with $\\measuredangle F Z E=\\measuredangle F A E=\\measuredangle B A C$ implies that $\\triangle Z F E$ and $\\triangle A C_{1} B_{1}$ are (directly) similar. (See IMO Shortlist 2006 G9 for a similar application of spiral similarity.) $$ \\measuredangle D A C_{1}=\\measuredangle Z A F=\\measuredangle Z E F=\\measuredangle A B_{1} C_{1} $$ we have $\\overline{A D}$ is tangent to $\\left(A B_{1} C_{1}\\right)$. Moreover, to see that $\\overline{I M} \\| \\overline{B_{1} C_{1}}$, write $$ \\begin{aligned} \\measuredangle\\left(\\overline{A I}, \\overline{B_{1} C_{1}}\\right) & =\\measuredangle I A C+\\measuredangle A B_{1} C_{1}=\\measuredangle B A I+\\measuredangle Z E F=\\measuredangle F A I+\\measuredangle Z A F \\\\ & =\\measuredangle Z A I=\\measuredangle M A I=\\measuredangle A I M \\end{aligned} $$ the last step since $\\triangle A I D$ is right with hypotenuse $\\overline{A D}$, and median $\\overline{I M}$. Now we return to the present problem with the additional condition. ![](https:\/\/cdn.mathpix.com\/cropped\/2024_11_19_a0a7a76bd95576fab5b9g-21.jpg?height=523&width=1366&top_left_y=955&top_left_x=331) Claim - Given the condition, we actually have $\\angle A B_{1} A_{1}=\\angle A C_{1} A_{1}=90^{\\circ}$. Now $\\triangle A B_{1} C_{1}$ has circumdiameter $\\overline{A V}$. We are given $A_{1}$ lies on this circle, so if $V \\neq A_{1}$ then $\\overline{A A_{1}} \\perp \\overline{A_{1} V}$. But $\\overline{A_{1} V} \\perp \\overline{B C}$ by definition, which would imply $\\overline{A A_{1}} \\| \\overline{B C}$, which is absurd. Claim - Given the condition the points $B_{1}, I, C_{1}$ are collinear (hence with $M$ ). To finish, since $\\overline{D M A}$ was tangent to the circumcircle of $\\triangle A B_{1} C_{1}$, we have $M D^{2}=$ $M A^{2}=M C_{1} \\cdot M B_{1}$, implying the required tangency. Remark. The triangles satisfying the problem hypothesis are exactly the ones satisfying $r_{A}=2 R$, where $R$ and $r_{A}$ denote the circumradius and $A$-exradius. Remark. If $P$ is the foot of the $A$-altitude then this should also imply $A B_{1} P C_{1}$ is harmonic."} +{"year":2019,"label":"6","problem":"Let $A B C$ be a triangle with incenter $I$, and let $D$ be a point on line $B C$ satisfying $\\angle A I D=90^{\\circ}$. Let the excircle of triangle $A B C$ opposite the vertex $A$ be tangent to $\\overline{B C}$ at point $A_{1}$. Define points $B_{1}$ on $\\overline{C A}$ and $C_{1}$ on $\\overline{A B}$ analogously, using the excircles opposite $B$ and $C$, respectively. Prove that if quadrilateral $A B_{1} A_{1} C_{1}$ is cyclic, then $\\overline{A D}$ is tangent to the circumcircle of $\\triangle D B_{1} C_{1}$.","solution":" \u3010 Second solution by inversion and mixtilinears (Anant Mudgal). As in the end of the preceding solution, we have $\\angle A B_{1} A_{1}=\\angle A C_{1} A_{1}=90^{\\circ}$ and $I \\in \\overline{B_{1} C_{1}}$. Let $M$ be the midpoint of minor arc $B C$ and $N$ be the midpoint of $\\operatorname{arc} \\widehat{B A C}$. Let $L$ be the intouch point on $\\overline{B C}$. Let $O$ be the circumcenter of $\\triangle A B C$. Let $K=\\overline{A I} \\cap \\overline{B C}$. ![](https:\/\/cdn.mathpix.com\/cropped\/2024_11_19_a0a7a76bd95576fab5b9g-22.jpg?height=803&width=1232&top_left_y=475&top_left_x=409) Claim \u2014 We have $\\angle\\left(\\overline{A I}, \\overline{B_{1} C_{1}}\\right)=\\angle I A D$. Inversion in $(B I C)$ maps $\\overline{A I}$ to $\\overline{K I}$ and $(A B C)$ to $\\overline{B C}$. Thus, $Z$ maps to $L$, so $Z, L, M$ are collinear. Since $B L=C V$ and $O I=O V$, we see that MLIN is a trapezoid with $\\overline{I L} \\| \\overline{M N}$. Thus, $\\overline{Z T} \\| \\overline{M N}$. It is known that $\\overline{A T}$ and $\\overline{A A_{1}}$ are isogonal in angle $B A C$. Since $\\overline{A V}$ is a circumdiameter in $\\left(A B_{1} C_{1}\\right)$, so $\\overline{A T} \\perp \\overline{B_{1} C_{1}}$. So $\\measuredangle Z A I=\\measuredangle N M T=90^{\\circ}-\\measuredangle T A I=\\measuredangle\\left(\\overline{A I}, \\overline{B_{1} C_{1}}\\right)$. Let $X$ be the midpoint of $\\overline{A D}$ and $G$ be the reflection of $I$ in $X$. Since $A I D G$ is a rectangle, we have $\\measuredangle A I G=\\measuredangle Z A I=\\measuredangle\\left(\\overline{A I}, \\overline{B_{1} C_{1}}\\right)$, by the previous claim. So $\\overline{I G}$ coincides with $\\overline{B_{1} C_{1}}$. Now $\\overline{A I}$ bisects $\\angle B_{1} A C_{1}$ and $\\angle I A G=90^{\\circ}$, so $\\left(\\overline{I G} ; \\overline{B_{1} C_{1}}\\right)=-1$. Since $\\angle I D G=90^{\\circ}$, we see that $\\overline{D I}$ and $\\overline{D G}$ are bisectors of angle $B_{1} D C_{1}$. Now $\\angle X D I=\\angle X I D \\Longrightarrow \\angle X D C_{1}=\\angle X I D-\\angle I D B_{1}=\\angle D B_{1} C_{1}$, so $\\overline{X D}$ is tangent to $\\left(D B_{1} C_{1}\\right)$."} diff --git a/USA_TST/segmented/en-sols-TST-IMO-2020.jsonl b/USA_TST/segmented/en-sols-TST-IMO-2020.jsonl new file mode 100644 index 0000000000000000000000000000000000000000..56e194fc960251819c28a8f587f14d3fad0677d3 --- /dev/null +++ b/USA_TST/segmented/en-sols-TST-IMO-2020.jsonl @@ -0,0 +1,9 @@ +{"year":2020,"label":"1","problem":"Choose positive integers $b_{1}, b_{2}, \\ldots$ satisfying $$ 1=\\frac{b_{1}}{1^{2}}>\\frac{b_{2}}{2^{2}}>\\frac{b_{3}}{3^{2}}>\\frac{b_{4}}{4^{2}}>\\cdots $$ and let $r$ denote the largest real number satisfying $\\frac{b_{n}}{n^{2}} \\geq r$ for all positive integers $n$. What are the possible values of $r$ across all possible choices of the sequence $\\left(b_{n}\\right)$ ?","solution":" The answer is $0 \\leq r \\leq 1 \/ 2$. Obviously $r \\geq 0$. In one direction, we show that Claim (Greedy bound) - For all integers $n$, we have $$ \\frac{b_{n}}{n^{2}} \\leq \\frac{1}{2}+\\frac{1}{2 n} $$ $$ \\begin{aligned} b_{n} & \\frac{\\left[r(n+1)^{2}+(n+1)\\right]+1}{(n+1)^{2}}>\\frac{b_{n+1}}{(n+1)^{2}} $$ where the middle inequality is true since it rearranges to $\\frac{1}{n}>\\frac{n+2}{(n+1)^{2}}$."} +{"year":2020,"label":"2","problem":"Two circles $\\Gamma_{1}$ and $\\Gamma_{2}$ have common external tangents $\\ell_{1}$ and $\\ell_{2}$ meeting at $T$. Suppose $\\ell_{1}$ touches $\\Gamma_{1}$ at $A$ and $\\ell_{2}$ touches $\\Gamma_{2}$ at $B$. A circle $\\Omega$ through $A$ and $B$ intersects $\\Gamma_{1}$ again at $C$ and $\\Gamma_{2}$ again at $D$, such that quadrilateral $A B C D$ is convex. Suppose lines $A C$ and $B D$ meet at point $X$, while lines $A D$ and $B C$ meet at point $Y$. Show that $T, X, Y$ are collinear.","solution":" \u092c First solution, elementary (original). We have $\\triangle Y A C \\sim \\triangle Y B D$, from which it follows $$ \\frac{d(Y, A C)}{d(Y, B D)}=\\frac{A C}{B D} $$ Moreover, if we denote by $r_{1}$ and $r_{2}$ the radii of $\\Gamma_{1}$ and $\\Gamma_{2}$, then $$ \\frac{d(T, A C)}{d(T, B D)}=\\frac{T A \\sin \\angle\\left(A C, \\ell_{1}\\right)}{T B \\sin \\angle\\left(B D, \\ell_{2}\\right)}=\\frac{2 r_{1} \\sin \\angle\\left(A C, \\ell_{1}\\right)}{2 r_{2} \\sin \\angle\\left(B D, \\ell_{2}\\right)}=\\frac{A C}{B D} $$ the last step by the law of sines. ![](https:\/\/cdn.mathpix.com\/cropped\/2024_11_19_1d87ca6c42af65f32daag-05.jpg?height=721&width=1269&top_left_y=1447&top_left_x=396) This solves the problem up to configuration issues: we claim that $Y$ and $T$ both lie inside $\\angle A X B \\equiv \\angle C X D$. WLOG $T A2$ then Hephaestus wins, but when $\\alpha=2$ (and hence $\\alpha \\leq 2$ ) Hephaestus cannot contain even a single-cell flood initially. Strategy for $\\alpha>2$ : Impose $\\mathbb{Z}^{2}$ coordinates on the cells. Adding more flooded cells does not make our task easier, so let us assume that initially the cells $(x, y)$ with $|x|+|y| \\leq d$ are flooded for some $d \\geq 2$; thus on Hephaestus's $k$ th turn, the water is contained in $|x|+|y| \\leq d+k-1$. Our goal is to contain the flood with a large rectangle. We pick large integers $N_{1}$ and $N_{2}$ such that $$ \\begin{gathered} \\alpha N_{1}>2 N_{1}+(2 d+3) \\\\ \\alpha\\left(N_{1}+N_{2}\\right)>2 N_{2}+\\left(6 N_{1}+8 d+4\\right) \\end{gathered} $$ Mark the points $X_{i}, Y_{i}$ as shown in the figure for $1 \\leq i \\leq 6$. The red figures indicate the distance between the marked points on the rectangle. ![](https:\/\/cdn.mathpix.com\/cropped\/2024_11_19_1d87ca6c42af65f32daag-09.jpg?height=1004&width=983&top_left_y=246&top_left_x=542) We follow the following plan. - Turn 1: place wall $X_{1} Y_{1}$. This cuts off the flood to the north. - Turns 2 through $N_{1}+1$ : extend the levee to segment $X_{2} Y_{2}$. This prevents further flooding to the north. - Turn $N_{1}+2$ : add in broken lines $X_{4} X_{3} X_{2}$ and $Y_{4} Y_{3} Y_{2}$ all at once. This cuts off the flood west and east. - Turns $N_{1}+2$ to $N_{1}+N_{2}+1$ : extend the levee along segments $X_{4} X_{5}$ and $Y_{4} Y_{5}$. This prevents further flooding west and east. - Turn $N_{1}+N_{2}+2$ : add in the broken line $X_{5} X_{6} Y_{6} Y_{5}$ all at once and win. Let $c_{0}, c_{1}, \\ldots, c_{n}$ be a path of cells such that $c_{0}$ is the initial cell flooded, and in general $c_{i}$ is flooded on Poseidon's $i$ th turn from $c_{i-1}$. The levee now forms a closed loop enclosing all $c_{i}$. Claim - If $c_{i}$ and $c_{j}$ are adjacent then $|i-j|=1$. Thus the $c_{i}$ actually form a path. We color green any edge of the unit grid (wall or not) which is an edge of exactly one $c_{i}$ (i.e. the boundary of the polyomino). It is easy to see there are exactly $2 n+4$ green edges. Now, from the center of each cell $c_{i}$, shine a laser towards each green edge of $c_{i}$ (hence a total of $2 n+4$ lasers are emitted). An example below is shown for $n=6$, with the levee marked in brown. ![](https:\/\/cdn.mathpix.com\/cropped\/2024_11_19_1d87ca6c42af65f32daag-10.jpg?height=803&width=806&top_left_y=415&top_left_x=631) Claim - No wall is hit by more than one laser. We consider two cases on the position of $w$. - If $w$ is between $c_{i}$ and $c_{j}$, then we have found a segment intersecting the levee exactly once. But the endpoints of the segment lie inside the levee. This contradicts the assumption that the levee is a closed loop. - Suppose $w$ lies above both $c_{i}$ and $c_{j}$ and assume WLOG $i2$. Remark (Author comments). The author provides the following remarks. - Even though the flood can be stopped when $\\alpha=2+\\varepsilon$, it takes a very long time to do that. Starting from a single flooded cell, the strategy I have outlined requires $\\Theta\\left(1 \/ \\varepsilon^{2}\\right)$ days. Starting from several flooded cells contained within an area of diameter $D$, it takes $\\Theta\\left(D \/ \\varepsilon^{2}\\right)$ days. I do not know any strategies that require fewer days than that. - There is a gaping chasm between $\\alpha \\leq 2$ and $\\alpha>2$. Since $\\alpha \\leq 2$ does not suffice even when only one cell is flooded in the beginning, there are in fact no initial configurations at all for which it is sufficient. On the other hand, $\\alpha>2$ works for all initial configurations. It appears that this has not been done before, or at least I was unable to find any reference for it. I did find tons of references where the perimeter of a polyomino is estimated in terms of its area, but nothing concerning the diameter. My argument is a formalisation of the intuition that if $P$ is any shortest path within some weirdly-shaped polyomino, then the boundary of that polyomino must hug $P$ rather closely so that $P$ cannot be shortened."} +{"year":2020,"label":"4","problem":"For a finite simple graph $G$, we define $G^{\\prime}$ to be the graph on the same vertex set as $G$, where for any two vertices $u \\neq v$, the pair $\\{u, v\\}$ is an edge of $G^{\\prime}$ if and only if $u$ and $v$ have a common neighbor in $G$. Prove that if $G$ is a finite simple graph which is isomorphic to $\\left(G^{\\prime}\\right)^{\\prime}$, then $G$ is also isomorphic to $G^{\\prime}$.","solution":" We say a vertex of a graph is fatal if it has degree at least 3, and some two of its neighbors are not adjacent. Claim - The graph $G^{\\prime}$ has at least as many triangles as $G$, and has strictly more if $G$ has any fatal vertices. Thus we only need to consider graphs $G$ with no fatal vertices. Looking at the connected components, the only possibilities are cliques (including single vertices), cycles, and paths. So in what follows we restrict our attention to graphs $G$ only consisting of such components. Remark (Warning). Beware: assuming $G$ is connected loses generality. For example, it could be that $G=G_{1} \\sqcup G_{2}$, where $G_{1}^{\\prime} \\cong G_{2}$ and $G_{2}^{\\prime} \\cong G_{1}$. First, note that the following are stable under the operation: - an isolated vertex, - a cycle of odd length, or - a clique with at least three vertices. In particular, $G \\cong G^{\\prime \\prime}$ holds for such graphs. On the other hand, cycles of even length or paths of nonzero length will break into more connected components. For this reason, a graph $G$ with any of these components will not satisfy $G \\cong G^{\\prime \\prime}$ because $G^{\\prime}$ will have strictly more connected components than $G$, and $G^{\\prime \\prime}$ will have at least as many as $G^{\\prime}$. Therefore $G \\cong G^{\\prime \\prime}$ if and only if $G$ is a disjoint union of the three types of connected components named earlier. Since $G \\cong G^{\\prime}$ holds for such graphs as well, the problem statement follows right away. Remark. The proposers included a variant of the problem where given any graph $G$, the operation stabilized after at most $O(\\log n)$ operations, where $n$ was the number of vertices of $G$."} +{"year":2020,"label":"5","problem":"Find all integers $n \\geq 2$ for which there exists an integer $m$ and a polynomial $P(x)$ with integer coefficients satisfying the following three conditions: - $m>1$ and $\\operatorname{gcd}(m, n)=1$; - the numbers $P(0), P^{2}(0), \\ldots, P^{m-1}(0)$ are not divisible by $n$; and - $P^{m}(0)$ is divisible by $n$. Here $P^{k}$ means $P$ applied $k$ times, so $P^{1}(0)=P(0), P^{2}(0)=P(P(0))$, etc.","solution":" The answer is that this is possible if and only if there exists primes $p^{\\prime}0 \\mid P^{e}(0) \\equiv 0 \\bmod N\\right\\} $$ where we set $\\min \\varnothing=0$ by convention. Note that in general we have $$ \\operatorname{zord}(P \\bmod N)=\\operatorname{lcm}_{q \\mid N}(\\operatorname{zord}(P \\bmod q)) $$ where the index runs over all prime powers $q \\mid N$ (by Chinese remainder theorem). This will be used in both directions. Construction: First, we begin by giving a construction. The idea is to first use the following prime power case. Claim - Let $p^{e}$ be a prime power, and $1 \\leq ks$, contradiction."} +{"year":2021,"label":"2","problem":"Points $A, V_{1}, V_{2}, B, U_{2}, U_{1}$ lie fixed on a circle $\\Gamma$, in that order, and such that $B U_{2}>A U_{1}>B V_{2}>A V_{1}$. Let $X$ be a variable point on the $\\operatorname{arc} V_{1} V_{2}$ of $\\Gamma$ not containing $A$ or $B$. Line $X A$ meets line $U_{1} V_{1}$ at $C$, while line $X B$ meets line $U_{2} V_{2}$ at $D$. Prove there exists a fixed point $K$, independent of $X$, such that the power of $K$ to the circumcircle of $\\triangle X C D$ is constant.","solution":" For brevity, we let $\\ell_{i}$ denote line $U_{i} V_{i}$ for $i=1,2$. We first give an explicit description of the fixed point $K$. Let $E$ and $F$ be points on $\\Gamma$ such that $\\overline{A E} \\| \\ell_{1}$ and $\\overline{B F} \\| \\ell_{2}$. The problem conditions imply that $E$ lies between $U_{1}$ and $A$ while $F$ lies between $U_{2}$ and $B$. Then we let $$ K=\\overline{A F} \\cap \\overline{B E} $$ This point exists because $A E F B$ are the vertices of a convex quadrilateral. Remark (How to identify the fixed point). If we drop the condition that $X$ lies on the arc, then the choice above is motivated by choosing $X \\in\\{E, F\\}$. Essentially, when one chooses $X \\rightarrow E$, the point $C$ approaches an infinity point. So in this degenerate case, the only points whose power is finite to $(X C D)$ are bounded are those on line $B E$. The same logic shows that $K$ must lie on line $A F$. Therefore, if the problem is going to work, the fixed point must be exactly $\\overline{A F} \\cap \\overline{B E}$. \u3010 First approach by Vincent Huang. We need the following claim: Claim - Suppose distinct lines $A C$ and $B D$ meet at $X$. Then for any point $K$ $$ \\operatorname{pow}(K, X A B)+\\operatorname{pow}(K, X C D)=\\operatorname{pow}(K, X A D)+\\operatorname{pow}(K, X B C) $$ Construct the points $P=\\ell_{1} \\cap \\overline{B E}$ and $Q=\\ell_{2} \\cap \\overline{A F}$, which do not depend on $X$. Claim - Quadrilaterals $B P C X$ and $A Q D X$ are cyclic. ![](https:\/\/cdn.mathpix.com\/cropped\/2024_11_19_9733e686be42a6dd888ag-5.jpg?height=803&width=792&top_left_y=244&top_left_x=638) Now, for the particular $K$ we choose, we have $$ \\begin{aligned} \\operatorname{pow}(K, X C D) & =\\operatorname{pow}(K, X A D)+\\operatorname{pow}(K, X B C)-\\operatorname{pow}(K, X A B) \\\\ & =K A \\cdot K Q+K B \\cdot K P-\\operatorname{pow}(K, \\Gamma) . \\end{aligned} $$"} +{"year":2021,"label":"2","problem":"Points $A, V_{1}, V_{2}, B, U_{2}, U_{1}$ lie fixed on a circle $\\Gamma$, in that order, and such that $B U_{2}>A U_{1}>B V_{2}>A V_{1}$. Let $X$ be a variable point on the $\\operatorname{arc} V_{1} V_{2}$ of $\\Gamma$ not containing $A$ or $B$. Line $X A$ meets line $U_{1} V_{1}$ at $C$, while line $X B$ meets line $U_{2} V_{2}$ at $D$. Prove there exists a fixed point $K$, independent of $X$, such that the power of $K$ to the circumcircle of $\\triangle X C D$ is constant.","solution":" For brevity, we let $\\ell_{i}$ denote line $U_{i} V_{i}$ for $i=1,2$. We first give an explicit description of the fixed point $K$. Let $E$ and $F$ be points on $\\Gamma$ such that $\\overline{A E} \\| \\ell_{1}$ and $\\overline{B F} \\| \\ell_{2}$. The problem conditions imply that $E$ lies between $U_{1}$ and $A$ while $F$ lies between $U_{2}$ and $B$. Then we let $$ K=\\overline{A F} \\cap \\overline{B E} $$ This point exists because $A E F B$ are the vertices of a convex quadrilateral. Remark (How to identify the fixed point). If we drop the condition that $X$ lies on the arc, then the choice above is motivated by choosing $X \\in\\{E, F\\}$. Essentially, when one chooses $X \\rightarrow E$, the point $C$ approaches an infinity point. So in this degenerate case, the only points whose power is finite to $(X C D)$ are bounded are those on line $B E$. The same logic shows that $K$ must lie on line $A F$. Therefore, if the problem is going to work, the fixed point must be exactly $\\overline{A F} \\cap \\overline{B E}$. \u3010 Second approach by authors. Let $Y$ be the second intersection of $(X C D)$ with $\\Gamma$. Let $S=\\overline{E Y} \\cap \\ell_{1}$ and $T=\\overline{F Y} \\cap \\ell_{2}$. Claim - Points $S$ and $T$ lies on ( $X C D)$ as well. Now let $X^{\\prime}$ be any other choice of $X$, and define $C^{\\prime}$ and $D^{\\prime}$ in the obvious way. We are going to show that $K$ lies on the radical axis of $(X C D)$ and $\\left(X^{\\prime} C^{\\prime} D^{\\prime}\\right)$. ![](https:\/\/cdn.mathpix.com\/cropped\/2024_11_19_9733e686be42a6dd888ag-5.jpg?height=798&width=798&top_left_y=1828&top_left_x=629) The main idea is as follows: Claim - The point $L=\\overline{E Y} \\cap \\overline{A X^{\\prime}}$ lies on the radical axis. By symmetry, so does the point $M=\\overline{F Y} \\cap \\overline{B X^{\\prime}}$ (not pictured). $$ \\operatorname{pow}\\left(L, X^{\\prime} C^{\\prime} D^{\\prime}\\right)=L C^{\\prime} \\cdot L X^{\\prime}=L S \\cdot L Y=\\operatorname{pow}(L, X C D) $$ To conclude, note that by Pascal theorem on $$ E Y F A X^{\\prime} B $$ it follows $K, L, M$ are collinear, as needed. Remark. All the conditions about $U_{1}, V_{1}, U_{2}, V_{2}$ at the beginning are there to eliminate configuration issues, making the problem less obnoxious to the contestant. In particular, without the various assumptions, there exist configurations in which the point $K$ is at infinity. In these cases, the center of $X C D$ moves along a fixed line."} +{"year":2021,"label":"3","problem":"Find all functions $f: \\mathbb{R} \\rightarrow \\mathbb{R}$ that satisfy the inequality $$ f(y)-\\left(\\frac{z-y}{z-x} f(x)+\\frac{y-x}{z-x} f(z)\\right) \\leq f\\left(\\frac{x+z}{2}\\right)-\\frac{f(x)+f(z)}{2} $$ for all real numbers $x0$. Indeed by the problem condition, $$ \\begin{aligned} & f(y) \\leq f(y-\\varepsilon)+\\frac{f(y+\\varepsilon)-f(y-3 \\varepsilon)}{4} \\\\ & f(y) \\leq f(y+\\varepsilon)-\\frac{f(y+3 \\varepsilon)-f(y-\\varepsilon)}{4} . \\end{aligned} $$ ![](https:\/\/cdn.mathpix.com\/cropped\/2024_11_19_9733e686be42a6dd888ag-8.jpg?height=223&width=609&top_left_y=1699&top_left_x=1029) Summing gives the desired conclusion. Now suppose that $f$ has two supergradients $\\alpha<\\alpha^{\\prime}$ at point $y$. For small enough $\\varepsilon$, we should have we have $f(y-\\varepsilon) \\leq f(y)-\\alpha^{\\prime} \\varepsilon$ and $f(y+\\varepsilon) \\leq f(y)+\\alpha \\varepsilon$, hence $$ g(\\varepsilon)=\\frac{f(y)-f(y-\\varepsilon)}{\\varepsilon}-\\frac{f(y+\\varepsilon)-f(y)}{\\varepsilon} \\geq \\alpha^{\\prime}-\\alpha . $$ This is impossible since $g(\\varepsilon)$ may be arbitrarily small. Claim - The function $f$ is quadratic on the rational numbers. $$ f(x+3 d)-3 f(x+2 d)+3 f(x+d)-f(x)=0 . $$ If we fix $d=1 \/ n$, it follows inductively that $f$ agrees with a quadratic function $\\widetilde{f}_{n}$ on the set $\\frac{1}{n} \\mathbb{Z}$. On the other hand, for any $m \\neq n$, we apparently have $\\widetilde{f}_{n}=\\widetilde{f}_{m n}=\\widetilde{f}_{m}$, so the quadratic functions on each \"layer\" are all equal. Since $f$ is continuous, it follows $f$ is quadratic, as needed. Remark (Alternate finish using differentiability due to Michael Ren). In the proof of the main claim (about uniqueness of supergradients), we can actually notice the two terms $\\frac{f(y)-f(y-t)}{t}$ and $\\frac{f(y+t)-f(y)}{t}$ in the definition of $g(t)$ are both monotonic (by concavity). Since we supplied a proof that $\\lim _{t \\rightarrow 0} g(t)=0$, we find $f$ is differentiable. Now, if the derivative at some point exists, it must coincide with all the supergradients; (informally, this is why \"tangent line trick\" always has the slope as the derivative, and formally, we use the mean value theorem). In other words, we must have $$ f(x+y)-f(x-y)=2 f^{\\prime}(x) \\cdot y $$ holds for all real numbers $x$ and $y$. By choosing $y=1$ we obtain that $f^{\\prime}(x)=f(x+1)-f(x-1)$ which means $f^{\\prime}$ is also continuous. Finally differentiating both sides with respect to $y$ gives $$ f^{\\prime}(x+y)-f^{\\prime}(x-y)=2 f^{\\prime}(x) $$ which means $f^{\\prime}$ obeys Jensen's functional equation. Since $f^{\\prime}$ is continuous, this means $f^{\\prime}$ is linear. Thus $f$ is quadratic, as needed."} diff --git a/USA_TST/segmented/en-sols-TST-IMO-2023.jsonl b/USA_TST/segmented/en-sols-TST-IMO-2023.jsonl new file mode 100644 index 0000000000000000000000000000000000000000..bd4e9e97de64ca25c08f58ae5431f5501917853a --- /dev/null +++ b/USA_TST/segmented/en-sols-TST-IMO-2023.jsonl @@ -0,0 +1,11 @@ +{"year":2023,"label":"1","problem":"There are 2022 equally spaced points on a circular track $\\gamma$ of circumference 2022. The points are labeled $A_{1}, A_{2}, \\ldots, A_{2022}$ in some order, each label used once. Initially, Bunbun the Bunny begins at $A_{1}$. She hops along $\\gamma$ from $A_{1}$ to $A_{2}$, then from $A_{2}$ to $A_{3}$, until she reaches $A_{2022}$, after which she hops back to $A_{1}$. When hopping from $P$ to $Q$, she always hops along the shorter of the two $\\operatorname{arcs} \\overparen{P Q}$ of $\\gamma$; if $\\overline{P Q}$ is a diameter of $\\gamma$, she moves along either semicircle. Determine the maximal possible sum of the lengths of the 2022 arcs which Bunbun traveled, over all possible labellings of the 2022 points.","solution":" Replacing 2022 with $2 n$, the answer is $2 n^{2}-2 n+2$. (When $n=1011$, the number is 2042222.) ![](https:\/\/cdn.mathpix.com\/cropped\/2024_11_19_5c8ecd58885c530bab97g-03.jpg?height=402&width=899&top_left_y=1202&top_left_x=586) \u3010 Construction. The construction for $n=5$ shown on the left half of the figure easily generalizes for all $n$. Remark. The validity of this construction can also be seen from the below proof. \u092c First proof of bound. Let $d_{i}$ be the shorter distance from $A_{2 i-1}$ to $A_{2 i+1}$. Claim - The distance of the leg of the journey $A_{2 i-1} \\rightarrow A_{2 i} \\rightarrow A_{2 i+1}$ is at most $2 n-d_{i}$. That means the total distance is at most $$ \\sum_{i=1}^{n}\\left(2 n-d_{i}\\right)=2 n^{2}-\\left(d_{1}+d_{2}+\\cdots+d_{n}\\right) $$ Claim - We have $$ d_{1}+d_{2}+\\cdots+d_{n} \\geq 2 n-2 $$"} +{"year":2023,"label":"1","problem":"There are 2022 equally spaced points on a circular track $\\gamma$ of circumference 2022. The points are labeled $A_{1}, A_{2}, \\ldots, A_{2022}$ in some order, each label used once. Initially, Bunbun the Bunny begins at $A_{1}$. She hops along $\\gamma$ from $A_{1}$ to $A_{2}$, then from $A_{2}$ to $A_{3}$, until she reaches $A_{2022}$, after which she hops back to $A_{1}$. When hopping from $P$ to $Q$, she always hops along the shorter of the two $\\operatorname{arcs} \\overparen{P Q}$ of $\\gamma$; if $\\overline{P Q}$ is a diameter of $\\gamma$, she moves along either semicircle. Determine the maximal possible sum of the lengths of the 2022 arcs which Bunbun traveled, over all possible labellings of the 2022 points.","solution":" Replacing 2022 with $2 n$, the answer is $2 n^{2}-2 n+2$. (When $n=1011$, the number is 2042222.) ![](https:\/\/cdn.mathpix.com\/cropped\/2024_11_19_5c8ecd58885c530bab97g-03.jpg?height=402&width=899&top_left_y=1202&top_left_x=586) \u3010 Construction. The construction for $n=5$ shown on the left half of the figure easily generalizes for all $n$. Remark. The validity of this construction can also be seen from the below proof. Second proof of bound. Draw the $n$ diameters through the $2 n$ arc midpoints, as shown on the right half of the figure for $n=5$ in red. Claim (Interpretation of distances) - The distance between any two points equals the number of diameters crossed to travel between the points. With this in mind, call a diameter critical if it is crossed by all $2 n$ arcs. Claim - At most one diameter is critical. Claim - Every diameter is crossed an even number of times. This immediately implies that the maximum possible total distance is achieved when one diameter is crossed all $2 n$ times, and every other diameter is crossed $2 n-2$ times, for a total distance of at most $$ n \\cdot(2 n-2)+2=2 n^{2}-2 n+2 $$"} +{"year":2023,"label":"2","problem":"Let $A B C$ be an acute triangle. Let $M$ be the midpoint of side $B C$, and let $E$ and $F$ be the feet of the altitudes from $B$ and $C$, respectively. Suppose that the common external tangents to the circumcircles of triangles $B M E$ and $C M F$ intersect at a point $K$, and that $K$ lies on the circumcircle of $A B C$. Prove that line $A K$ is perpendicular to line $B C$.","solution":" \\I Inversion solution submitted by Ankan Bhattacharya and Nikolai Beluhov. Let $H$ be the orthocenter of $\\triangle A B C$. We use inversion in the circle with diameter $\\overline{B C}$. We identify a few images: - The circumcircles of $\\triangle B M E$ and $\\triangle C M F$ are mapped to lines $B E$ and $C F$. - The common external tangents are mapped to the two circles through $M$ which are tangent to lines $B E$ and $C F$. - The image of $K$, denoted $K^{*}$, is the second intersection of these circles. - The assertion that $K$ lies on $(A B C)$ is equivalent to $K^{*}$ lying on $(B H C)$. However, now $K^{*}$ is simple to identify directly: it's just the reflection of $M$ in the bisector of $\\angle B H C$. ![](https:\/\/cdn.mathpix.com\/cropped\/2024_11_19_5c8ecd58885c530bab97g-06.jpg?height=1148&width=1218&top_left_y=237&top_left_x=419) In particular, $\\overline{H K^{*}}$ is a symmedian of $\\triangle B H C$. However, since $K^{*}$ lies on $(B H C)$, this means $\\left(H K^{*} ; B C\\right)=-1$. Then, we obtain that $\\overline{B C}$ bisects $\\angle H M K^{*} \\equiv \\angle H M K$. However, $K$ also lies on $(A B C)$, which forces $K$ to be the reflection of $H$ in $\\overline{B C}$. Thus $\\overline{A K} \\perp \\overline{B C}$, as wanted."} +{"year":2023,"label":"2","problem":"Let $A B C$ be an acute triangle. Let $M$ be the midpoint of side $B C$, and let $E$ and $F$ be the feet of the altitudes from $B$ and $C$, respectively. Suppose that the common external tangents to the circumcircles of triangles $B M E$ and $C M F$ intersect at a point $K$, and that $K$ lies on the circumcircle of $A B C$. Prove that line $A K$ is perpendicular to line $B C$.","solution":" \u0e46 Solution with coaxial circles (Pitchayut Saengrungkongka). Let $H$ be the orthocenter of $\\triangle A B C$. Let $Q$ be the second intersection of $\\odot(B M E)$ and $\\odot(C M F)$. We first prove the following well-known properties of $Q$. Claim - $Q$ is the Miquel point of $B C E F$. In particular, $Q$ lies on both $\\odot(A E F)$ and $\\odot(A B C)$. Claim - $A(Q, H ; B, C)=-1$. Claim - $Q T$ is tangent to $\\odot(B Q C)$. $$ \\begin{aligned} \\frac{B T}{T C} & =\\frac{T B \\cdot T M}{T C \\cdot T M} \\\\ & =\\frac{\\operatorname{pow}(T, \\odot(B M E))}{\\operatorname{pow}(T, \\odot(C M F))} \\\\ & =\\frac{\\operatorname{pow}(K, \\odot(B M E))}{\\operatorname{pow}(K, \\odot(C M F))} \\\\ & =\\left(\\frac{r_{\\odot(B M E)}}{r_{\\odot(C M F)}}\\right)^{2} \\\\ & =\\left(\\frac{B Q \/ \\sin \\angle Q M B}{C Q \/ \\sin \\angle Q M C}\\right)^{2} \\\\ & =\\frac{B Q^{2}}{C Q^{2}} \\end{aligned} $$ implying the result. To finish, let $O$ be the center of $\\odot(A B C)$. Then, from the claim, $\\angle O Q T=90^{\\circ}=$ $\\angle O M T$, so $O$ also lies on $\\odot(Q M T K)$. Thus, $\\angle O K T=90^{\\circ}$, so $K T$ is also tangent to $\\odot(A B C)$ as well. This implies that $Q B K C$ is harmonic quadrilateral, and the result follows from the second claim."} +{"year":2023,"label":"2","problem":"Let $A B C$ be an acute triangle. Let $M$ be the midpoint of side $B C$, and let $E$ and $F$ be the feet of the altitudes from $B$ and $C$, respectively. Suppose that the common external tangents to the circumcircles of triangles $B M E$ and $C M F$ intersect at a point $K$, and that $K$ lies on the circumcircle of $A B C$. Prove that line $A K$ is perpendicular to line $B C$.","solution":" \u3010 Solution by Luke Robitaille. Let $Q$ be the second intersection of $\\odot(B M E)$ and $\\odot(C M F)$. We use the first two claims of the previous solution. In particular, $Q \\in$ $\\odot(A B C)$. We have the following claim. Claim (Also appeared in ISL 2017 G7) - We have $\\measuredangle Q K M=\\measuredangle Q B M+\\measuredangle Q C M$. $$ \\begin{aligned} \\measuredangle Q K M & =\\measuredangle Q^{\\prime} Q M^{\\prime}+\\measuredangle Q M^{\\prime} M \\\\ & =\\measuredangle Q C M+\\measuredangle Q B M, \\end{aligned} $$ as desired. Now, we extend $K M$ to meet $\\odot(A B C)$ again at $Q_{1}$. We have $$ \\begin{aligned} \\measuredangle Q_{1} Q B=\\measuredangle Q_{1} K B & =\\measuredangle Q_{1} K Q+\\measuredangle Q C B \\\\ & =\\measuredangle M K Q+\\measuredangle Q K B \\\\ & =(\\measuredangle M B Q+\\measuredangle M C Q)+\\measuredangle Q C B \\\\ & =\\measuredangle C B Q, \\end{aligned} $$ implying that $Q Q_{1} \\| B C$. This implies that $Q B K C$ is harmonic quadrilateral, so we are done."} +{"year":2023,"label":"2","problem":"Let $A B C$ be an acute triangle. Let $M$ be the midpoint of side $B C$, and let $E$ and $F$ be the feet of the altitudes from $B$ and $C$, respectively. Suppose that the common external tangents to the circumcircles of triangles $B M E$ and $C M F$ intersect at a point $K$, and that $K$ lies on the circumcircle of $A B C$. Prove that line $A K$ is perpendicular to line $B C$.","solution":" I Synthetic solution due to Andrew Gu (Harvard 2026). Define $O_{1}$ and $O_{2}$ as the circumcenters of $(B M E)$ and $(C M F)$. Let $T$ be the point on $(A B C)$ such that $\\overline{A T} \\perp \\overline{B C}$. Denote by $L$ the midpoint of minor arc $\\overparen{B C}$. We are going to ignore the condition that $K$ lies on the circumcircle of $A B C$, and prove the following unconditional result: ## Proposition The points $T, L, K$ are collinear. This will solve the problem because if $K$ is on the circumcircle of $A B C$, it follows $K=T$ or $K=L$; but $K=L$ can never occur since $O_{1}$ and $O_{2}$ are obviously on different sides of line $L M$ so line $L M$ must meet $O_{1} O_{2}$ inside segment $O_{1} O_{2}$, and $K$ lies outside this segment. ![](https:\/\/cdn.mathpix.com\/cropped\/2024_11_19_5c8ecd58885c530bab97g-08.jpg?height=1243&width=1400&top_left_y=926&top_left_x=334) Claim - Lines $A C$ and $L M$ meet at the antipode $Q_{1}$ of $B$ on $(B M E)$, so that $B P_{1} Q_{1} M$ is a rectangle. Similarly, lines $A B$ and $L M$ meet at the antipode $Q_{2}$ of $C$ on $(C M F)$, so that $C P_{2} Q_{2} M$ is a rectangle. From this, it follows that $P_{1} Q_{1}=B M=\\frac{1}{2} B C=M C=P_{2} Q_{2}$. Letting $r_{1}$ denote the radius of $\\omega_{1}$ (and similarly for $\\omega_{2}$ ), we deduce that $C Q_{1}=B Q_{1}=2 r_{1}$. Claim - $K M=K L$. $$ \\measuredangle Q_{1} C L=\\measuredangle A C L=\\measuredangle A B L=\\measuredangle Q_{2} B L=\\measuredangle Q_{2} C L $$ The external angle bisector theorem then gives an equality of directed ratios $$ \\frac{L Q_{1}}{L Q_{2}}=\\frac{\\left|C Q_{1}\\right|}{\\left|C Q_{2}\\right|}=\\frac{\\left|B Q_{1}\\right|}{\\left|C Q_{2}\\right|}=\\frac{2 r_{1}}{2 r_{2}}=\\frac{r_{1}}{r_{2}} $$ Let the reflection of $M$ over $K$ be $P$; then $P$ lies on $\\overline{P_{1} P_{2}}$ and $$ \\frac{P P_{1}}{P P_{2}}=\\frac{2 K O_{1}}{2 K O_{2}}=\\frac{K O_{1}}{K O_{2}}=\\frac{r_{1}}{r_{2}}=\\frac{L Q_{1}}{L Q_{2}} $$ where again the ratios are directed. Projecting everything onto line $L M$, so that $P_{1}$ lands at $Q_{1}$ and $P_{2}$ lands at $Q_{2}$, we find that the projection of $P$ must land exactly at $L$. Claim - Line $K M$ is an external angle bisector of $\\angle O_{1} M O_{2}$. To finish, note that we know that $\\overline{M P_{1}} \\| \\overline{C Q_{1}} \\equiv \\overline{A C}$ and $\\overline{M P_{2}} \\| \\overline{B Q_{2}} \\equiv \\overline{A B}$, meaning the angles $\\angle O_{1} M O_{2}$ and $\\angle C A B$ have parallel legs. Hence, if $N$ is the antipode of $L$, it follows that $\\overline{M K} \\| \\overline{A N}$. Now from $M K=K L$ and the fact that $A N L T$ is an isosceles trapezoid, we deduce that $\\overline{L T}$ and $\\overline{L K}$ are lines in the same direction (namely, the reflection of $M K \\| A N$ across $\\overline{B C}$ ), as needed."} +{"year":2023,"label":"2","problem":"Let $A B C$ be an acute triangle. Let $M$ be the midpoint of side $B C$, and let $E$ and $F$ be the feet of the altitudes from $B$ and $C$, respectively. Suppose that the common external tangents to the circumcircles of triangles $B M E$ and $C M F$ intersect at a point $K$, and that $K$ lies on the circumcircle of $A B C$. Prove that line $A K$ is perpendicular to line $B C$.","solution":" \u3010 Complex numbers approach with Apollonian circles, by Carl Schildkraut. We use complex numbers. As in the first approach, we will ignore the hypothesis that $K$ lies on ( $A B C$ ). Let $Q:=(A H) \\cap(A B C) \\cap(A E F) \\neq A$ be the Miquel point of $B F E C$ again. Construct the point $T$ on $(A B C)$ for which $A T \\perp B C$; note that $T=-\\frac{b c}{a}$. This time the unconditional result is: ## Proposition We have $Q, M, T, K$ are concyclic (or collinear) on an Apollonian circle of $\\overline{O_{1} O_{2}}$. This will solve the original problem since once $K$ lies on $(A B C)$ it must be either $Q$ or $T$. But since $K$ is not on $(B M E), K \\neq Q$, it will have to be $T$. We now prove the proposition. Suppose $(A B C)$ is the unit circle and let $A=a, B=b$, $C=c$. Let $H=a+b+c$ be the orthocenter of $\\triangle A B C$. By the usual formulas, $$ E:=\\frac{1}{2}\\left(a+b+c-\\frac{b c}{a}\\right) . $$ Let $O_{1}$ be the center of $(B M E)$ and $O_{2}$ be the center of $(C M F)$. Claim (Calculation of the Miquel point) - We have $Q=\\frac{2 a+b+c}{a\\left(\\frac{1}{a}+\\frac{1}{b}+\\frac{1}{c}\\right)+1}$. $$ 0=\\frac{q-h}{q-a}+\\frac{1 \/ q-\\bar{h}}{1 \/ q-1 \/ a}=\\frac{q-h}{q-a}-\\frac{a(1-q \\bar{h})}{q-a} $$ This solves to $q=\\frac{h+a}{a \\bar{h}+1}=\\frac{2 a+b+c}{a \\bar{h}+1}$. Claim (Calculation of $O_{1}$ and $\\left.O_{2}\\right)-$ We have $O_{1}=\\frac{b(2 a+b+c)}{2(a+b)}$ and $O_{2}=\\frac{c(2 a+b+c)}{2(a+c)}$. $$ \\begin{aligned} O_{1} & =\\operatorname{Circum}(B, M, E) \\\\ & =b+\\frac{1}{2} \\operatorname{Circum}\\left(0, c-b, \\frac{(a-b)(b-c)}{b}\\right) \\\\ & =b-\\frac{b-c}{2 b} \\operatorname{Circum}(0, b, b-a) \\\\ & =b-\\frac{b-c}{2 b}(b-\\operatorname{Circum}(0, b, a)) \\\\ & =b-\\frac{b-c}{2 b}\\left(b-\\frac{a b}{a+b}\\right)=b-\\frac{b(b-c)}{2(a+b)}=\\frac{b(2 a+b+c)}{2(a+b)} \\end{aligned} $$ Similarly, $O_{2}=\\frac{c(2 a+b+c)}{2(a+c)}$. We are now going to prove the following: Claim - We have $$ \\frac{T O_{1}}{T O_{2}}=\\frac{M O_{1}}{M O_{2}}=\\frac{Q O_{1}}{Q O_{2}} $$ $$ M O_{1}=B O_{1}=\\left|b-\\frac{b(2 a+b+c)}{2(a+b)}\\right|=\\left|\\frac{b(b-c)}{2(a+b)}\\right|=\\frac{1}{2}\\left|\\frac{b-c}{a+b}\\right| $$ and $Q O_{1}=\\left|r-\\frac{b(2 a+b+c)}{2(a+b)}\\right|=\\left|1-\\frac{b(a+h)}{2(a+b) r}\\right|=\\left|1-\\frac{b(a \\bar{h}+1)}{2(a+b)}\\right|=\\left|\\frac{a-\\frac{a b}{c}}{2(a+b)}\\right|=\\frac{1}{2}\\left|\\frac{b-c}{a+b}\\right|$. This implies both (by symmetry) that $\\frac{M O_{1}}{M O_{2}}=\\frac{Q O_{1}}{Q O_{2}}=\\left|\\frac{a+c}{a+b}\\right|$ and that $Q$ is on (BME) and $(C M F)$. Also, $$ \\frac{T O_{1}}{T O_{2}}=\\frac{\\left|\\frac{b(2 a+b+c)}{2(a+b)}+\\frac{b c}{a}\\right|}{\\left|\\frac{c(2 a+b+c)}{2(a+c)}+\\frac{b c}{a}\\right|}=\\left|\\frac{\\frac{b\\left(2 a^{2}+a b+a c+2 a c+2 b c\\right)}{2 a(a+b)}}{\\frac{c\\left(2 a^{2}+a b+a c+2 a b+2 b c\\right)}{2 a(a+c)}}\\right|=\\left|\\frac{a+c}{a+b}\\right| \\cdot\\left|\\frac{2 a^{2}+2 b c+a b+3 a c}{2 a^{2}+2 b c+3 a b+a c}\\right| $$ if $z=2 a^{2}+2 b c+a b+3 a c$, then $a^{2} b c \\bar{z}=2 a^{2}+2 b c+3 a b+a c$, so the second term has magnitude 1. This means $\\frac{T O_{1}}{T O_{2}}=\\frac{M O_{1}}{M O_{2}}=\\frac{Q O_{1}}{Q O_{2}}$, as desired. To finish, note that this common ratio is the ratio between the radii of these two circles, so it is also $\\frac{K O_{1}}{K O_{2}}$. By Apollonian circles the points $\\{Q, M, T, K\\}$ lie on a circle or a line."} +{"year":2023,"label":"3","problem":"Consider pairs $(f, g)$ of functions from the set of nonnegative integers to itself such that - $f(0) \\geq f(1) \\geq f(2) \\geq \\cdots \\geq f(300) \\geq 0 ;$ - $f(0)+f(1)+f(2)+\\cdots+f(300) \\leq 300$; - for any 20 nonnegative integers $n_{1}, n_{2}, \\ldots, n_{20}$, not necessarily distinct, we have $$ g\\left(n_{1}+n_{2}+\\cdots+n_{20}\\right) \\leq f\\left(n_{1}\\right)+f\\left(n_{2}\\right)+\\cdots+f\\left(n_{20}\\right) . $$ Determine the maximum possible value of $g(0)+g(1)+\\cdots+g(6000)$ over all such pairs of functions.","solution":" Replace $300=\\frac{24 \\cdot 25}{2}$ with $\\frac{s(s+1)}{2}$ where $s=24$, and 20 with $k$. The answer is $115440=$ $\\frac{k s(k s+1)}{2}$. Equality is achieved at $f(n)=\\max (s-n, 0)$ and $g(n)=\\max (k s-n, 0)$. To prove $$ g\\left(n_{1}+\\cdots+n_{k}\\right) \\leq f\\left(n_{1}\\right)+\\cdots+f\\left(n_{k}\\right) $$ write it as $$ \\max \\left(x_{1}+\\cdots+x_{k}, 0\\right) \\leq \\max \\left(x_{1}, 0\\right)+\\cdots+\\max \\left(x_{k}, 0\\right) $$ with $x_{i}=s-n_{i}$. This can be proven from the $k=2$ case and induction. It remains to show the upper bound. For this problem, define a partition to be a nonincreasing function $p: \\mathbb{Z}_{\\geq 0} \\rightarrow \\mathbb{Z}_{\\geq 0}$ such that $p(n)=0$ for some $n$. The sum of $p$ is defined to be $\\sum_{n=0}^{\\infty} p(n)$, which is finite under the previous assumption. Let $L=\\mathbb{Z}_{\\geq 0}^{2}$. The Young diagram of the partition is the set of points $$ \\mathcal{P}:=\\{(x, y) \\in L: yn \\text {. } $$ This is a partition with the same sum as $p$. Geometrically, the Young diagrams of $p$ and $p_{*}$ are reflections about $x=y$. Since each $g(n)$ is independent, we may maximize each one separately for all $n$ and assume that $$ g(n)=\\min _{n_{1}+\\cdots+n_{k}=n}\\left(f\\left(n_{1}\\right)+\\cdots+f\\left(n_{k}\\right)\\right) . $$ The conditions of the problem statement imply that $f\\left(\\frac{s(s+1)}{2}\\right)=0$. Then, for any $n \\leq k \\frac{s(s+1)}{2}$, there exists an optimal combination $\\left(n_{1}, \\ldots, n_{k}\\right)$ in $\\left(^{*}\\right)$ where all $n_{i}$ are at most $\\frac{s(s+1)}{2}$, by replacing any term in an optimum greater than $\\frac{s(s+1)}{2}$ by $\\frac{s(s+1)}{2}$ and shifting the excess to smaller terms (because $f$ is nonincreasing). Therefore we may extend $f$ to a partition by letting $f(n)=0$ for $n>\\frac{s(s+1)}{2}$ without affecting the relevant values of $g$. Then $\\left(^{*}\\right)$ implies that $g$ is a partition as well. The problem can be restated as follows: $f$ is a partition with sum $\\frac{s(s+1)}{2}$, and $g$ is a partition defined by $\\left(^{*}\\right)$. Find the maximum possible sum of $g$. The key claim is that the problem is the same under conjugation. Claim - Under these conditions, we have $$ g_{*}(n)=\\min _{n_{1}+\\cdots+n_{k}=n}\\left(f_{*}\\left(n_{1}\\right)+\\cdots+f_{*}\\left(n_{k}\\right)\\right) . $$ $$ \\overline{\\mathcal{G}}=\\underbrace{\\overline{\\mathcal{F}}+\\cdots+\\overline{\\mathcal{F}}}_{k \\overline{\\mathcal{F}}^{\\prime} \\mathrm{s}} $$ where + denotes set addition. This definition remains invariant under reflection about $x=y$, which swaps $f$ and $g$ with their conjugates. Let $A$ be the sum of $g$. We now derive different bounds on $A$. First, by Hermite's identity $$ n=\\sum_{i=0}^{k-1}\\left\\lfloor\\frac{n+i}{k}\\right\\rfloor $$ we have $$ \\begin{aligned} A & =\\sum_{n=0}^{\\infty} g(n) \\\\ & \\leq \\sum_{n=0}^{\\infty} \\sum_{i=0}^{k-1} f\\left(\\left\\lfloor\\frac{n+i}{k}\\right\\rfloor\\right) \\\\ & =k^{2} \\sum_{n=0}^{\\infty} f(n)-\\frac{k(k-1)}{2} f(0) \\\\ & =k^{2} \\frac{s(s+1)}{2}-\\frac{k(k-1)}{2} f(0) . \\end{aligned} $$ By the claim, we also get the second bound $A \\leq k^{2} \\frac{s(s+1)}{2}-\\frac{k(k-1)}{2} f_{*}(0)$. For the third bound, note that $f\\left(f_{*}(0)\\right)=0$ and thus $g\\left(k f_{*}(0)\\right)=0$. Moreover, $$ g\\left(q f_{*}(0)+r\\right) \\leq q \\cdot f\\left(f_{*}(0)\\right)+(k-q-1) f(0)+f(r)=(k-q-1) f(0)+f(r), $$ so we have $$ \\begin{aligned} A & =\\sum_{\\substack{0 \\leq qy \\geq 0$, we have $$ x \\oplus a x \\neq y \\oplus a y . $$","solution":" Answer: the function $x \\mapsto x \\oplus a x$ is injective if and only if $a$ is an even integer. \u0111 Even case. First, assume $\\nu_{2}(a)=k>0$. We wish to recover $x$ from $c:=x \\oplus a x$. Notice that: - The last $k$ bits of $c$ coincide with the last $k$ bits of $x$. - Now the last $k$ bits of $x$ give us also the last $2 k$ bits of $a x$, so we may recover the last $2 k$ bits of $x$ as well. - Then the last $2 k$ bits of $x$ give us also the last $3 k$ bits of $a x$, so we may recover the last $3 k$ bits of $x$ as well. - ...and so on. \u0111 Odd case. Conversely, suppose $a$ is odd. To produce the desired collision: Claim - Let $n$ be any integer such that $2^{n}>a$, and define $$ x=\\underbrace{1 \\ldots 1}_{n}=2^{n}-1, \\quad y=1 \\underbrace{0 \\ldots 0}_{n} 1=2^{n}+1 . $$ Then $x \\oplus a x=y \\oplus a y$. $$ \\begin{aligned} & a x=\\overline{Q R} \\Longrightarrow x \\oplus a x=\\overline{Q Q} \\\\ & a y=\\overline{P P} \\Longrightarrow y \\oplus a y=\\overline{Q Q} . \\end{aligned} $$ We're done."} +{"year":2023,"label":"5","problem":"Let $m$ and $n$ be fixed positive integers. Tsvety and Freyja play a game on an infinite grid of unit square cells. Tsvety has secretly written a real number inside of each cell so that the sum of the numbers within every rectangle of size either $m \\times n$ or $n \\times m$ is zero. Freyja wants to learn all of these numbers. One by one, Freyja asks Tsvety about some cell in the grid, and Tsvety truthfully reveals what number is written in it. Freyja wins if, at any point, Freyja can simultaneously deduce the number written in every cell of the entire infinite grid. (If this never occurs, Freyja has lost the game and Tsvety wins.) In terms of $m$ and $n$, find the smallest number of questions that Freyja must ask to win, or show that no finite number of questions can suffice.","solution":" The answer is the following: - If $\\operatorname{gcd}(m, n)>1$, then Freyja cannot win. - If $\\operatorname{gcd}(m, n)=1$, then Freyja can win in a minimum of $(m-1)^{2}+(n-1)^{2}$ questions. First, we dispose of the case where $\\operatorname{gcd}(m, n)>1$. Write $d=\\operatorname{gcd}(m, n)$. The idea is that any labeling where each $1 \\times d$ rectangle has sum zero is valid. Thus, to learn the labeling, Freyja must ask at least one question in every row, which is clearly not possible in a finite number of questions. Now suppose $\\operatorname{gcd}(m, n)=1$. We split the proof into two halves. \u3010 Lower bound. Clearly, any labeling where each $m \\times 1$ and $1 \\times m$ rectangle has sum zero is valid. These labelings form a vector space with dimension $(m-1)^{2}$, by inspection. (Set the values in an $(m-1) \\times(m-1)$ square arbitrarily and every other value is uniquely determined.) Similarly, labelings where each $n \\times 1$ and $1 \\times n$ rectangle have sum zero are also valid, and have dimension $(n-1)^{2}$. It is also easy to see that no labeling other than the all-zero labeling belongs to both categories; labelings in the first space are periodic in both directions with period $m$, while labelings in the second space are periodic in both directions with period $n$; and hence any labeling in both categories must be constant, ergo all-zero. Taking sums of these labelings gives a space of valid labelings of dimension $(m-1)^{2}+$ $(n-1)^{2}$. Thus, Freyja needs at least $(m-1)^{2}+(n-1)^{2}$ questions to win. Claim (Periodicity) - Any valid labeling is doubly periodic with period $m n$. Then both $m n \\times N$ and $m n \\times(N+1)$ rectangles have zero sum, so $m n \\times 1$ rectangles have zero sum. This implies that any two cells with a vertical displacement of $m n$ are equal; similarly for horizontal displacements. With that in mind, consider a valid labeling. It naturally corresponds to a generating function $$ f(x, y)=\\sum_{a=0}^{m n-1} \\sum_{b=0}^{m n-1} c_{a, b} x^{a} y^{b} $$ where $c_{a, b}$ is the number in $(a, b)$. The generating function corresponding to sums over $n \\times m$ rectangles is $$ f(x, y)\\left(1+x+\\cdots+x^{m-1}\\right)\\left(1+y+\\cdots+y^{n-1}\\right)=f(x, y) \\cdot \\frac{x^{m}-1}{x-1} \\cdot \\frac{y^{n}-1}{y-1} . $$ Similarly, the one for $m \\times n$ rectangles is $$ f(x, y) \\cdot \\frac{x^{n}-1}{x-1} \\cdot \\frac{y^{m}-1}{y-1} . $$ Thus, the constraints for $f$ to be valid are equivalent to $$ f(x, y) \\cdot \\frac{x^{m}-1}{x-1} \\cdot \\frac{y^{n}-1}{y-1} \\quad \\text { and } \\quad f(x, y) \\cdot \\frac{x^{n}-1}{x-1} \\cdot \\frac{y^{m}-1}{y-1} $$ being zero when reduced modulo $x^{m n}-1$ and $y^{m n}-1$, or, letting $\\omega=\\exp (2 \\pi i \/ m n)$, both terms being zero when powers of $\\omega$ are plugged in. To restate the constraints one final time, we need $$ f\\left(\\omega^{a}, \\omega^{b}\\right) \\cdot \\frac{\\omega^{a m}-1}{\\omega^{a}-1} \\cdot \\frac{\\omega^{b n}-1}{\\omega^{b}-1}=f\\left(\\omega^{a}, \\omega^{b}\\right) \\cdot \\frac{\\omega^{a n}-1}{\\omega^{a}-1} \\cdot \\frac{\\omega^{b m}-1}{\\omega^{b}-1}=0 $$ for all $a, b \\in\\{0, \\ldots, m n-1\\}$. Claim - This implies that $f\\left(\\omega^{a}, \\omega^{b}\\right)=0$ for all but at most $(m-1)^{2}+(n-1)^{2}$ values of $(a, b) \\in\\{0, \\ldots, m n-1\\}^{2}$. $$ \\frac{\\omega^{a m}-1}{\\omega^{a}-1} \\cdot \\frac{\\omega^{b n}-1}{\\omega^{b}-1}=\\frac{\\omega^{a n}-1}{\\omega^{a}-1} \\cdot \\frac{\\omega^{b m}-1}{\\omega^{b}-1}=0 . $$ This happens when (at least) one fraction in either product is zero. - If the first fraction is zero, then either $n \\mid a$ and $a>0$, or $m \\mid b$ and $b>0$. - If the second fraction is zero, then either $m \\mid a$ and $a>0$, or $n \\mid b$ and $b>0$. If the first condition holds in both cases, then $m n \\mid a$, but $0C$.","solution":" Suppose for the sake of contradiction that $|\\Delta(m, n)| \\leq N$ for all $m, n$. Note that $f$ is injective, as $$ f(m)=f(n) \\Longrightarrow \\Delta(m, n)=0 \\Longrightarrow m=n $$ as desired. Let $G$ be the \"arrow graph\" of $f$, which is the directed graph with vertex set $\\mathbb{N}$ and edges $n \\rightarrow f(n)$. The first step in the solution is to classify the structure of $G$. Injectivity implies that $G$ is a disjoint collection of chains (infinite and half-infinite) and cycles. We have the following sequence of claims that further refine the structure. Claim - The graph $G$ has no cycles. $$ f^{f(n)}\\left(m_{1}\\right)=f^{f(n)}\\left(m_{2}\\right) $$ for some $m_{1} \\neq m_{2}$, which contradicts injectivity. Claim - The graph $G$ has at most $2 N+1$ chains. $$ \\left|\\Delta\\left(m_{i}, f^{B-f\\left(m_{i}\\right)}(1)\\right)\\right| \\leq N \\Longrightarrow\\left|f^{B}(1)-f^{f B-f\\left(m_{i}\\right)+1}(1)\\left(m_{i}\\right)\\right| \\leq N . $$ Since the $m_{i} \\mathrm{~S}$ are in different chains, we have that $f^{f\\left(-f\\left(m_{i}\\right)+1\\right.}(1)\\left(m_{i}\\right)$ are distinct for each $i$, which implies that $k \\leq 2 N+1$, as desired. Claim - The graph $G$ consists of exactly one half-infinite chain. Since there are only finitely many chains, $f^{f(c)}(n)$ achieves all sufficiently large positive integers, say all positive integers at least $M$. Fix $A$ and $B$ such that $B>A \\geq M$. If $f^{f(c)}(n) \\in[A, B]$, then $f^{f(n)}(c) \\in[A-N, B+N]$, and distinct $n$ generate distinct $f^{f(n)}(c)$ due to the structure of $G$. Therefore, we have at least $B-A+1$ good numbers in $[A-N, B+N]$, so there are at most $2 N$ bad numbers in $[A-N, B+N]$. Varying $B$, this shows there are at most $2 N$ bad numbers at least $A-N$. Let $c$ be the starting point of the chain, so every integer is of the form $f^{k}(c)$, where $k \\geq 0$. Define a function $g: \\mathbb{Z}_{\\geq 0} \\rightarrow \\mathbb{N}$ by $$ g(k):=f^{k}(c) $$ Due to the structure of $G, g$ is a bijection. Define $$ \\delta(a, b):=\\Delta\\left(f^{a}(c), f^{b}(c)\\right)=g(g(b+1)+a)-g(g(a+1)+b) $$ so the conditions are equivalent to $|\\delta(a, b)| \\leq N$ for all $a, b \\in \\mathbb{Z}_{\\geq 0}$ and $\\delta(a, b) \\neq 0$ for $a \\neq b$, which is equivalent to $g(a+1)-a \\neq g(b+1)-b$ for $a \\neq b$. This tells us that $g(x)-x$ is injective for $x \\geq 1$. ## Lemma For all $M$, there exists a nonnegative integer $x$ with $g(x) \\leq x-M$. Now pick $B$ such that $g(B)+N \\leq B$ and $g(B)>N$. Note that infinitely many such $B$ exist, since we can take $M$ to be arbitrarily small in the above lemma. Let $$ t=\\max \\left\\{g^{-1}(g(B)-N), g^{-1}(g(B)-N+1), \\ldots, g^{-1}(g(B)+N)\\right\\} $$ Note that $g(t) \\leq g(B)+N \\leq B$, so we have $$ |\\delta(t-1, B-g(t))|=|g(B)-g(t-1+g(B+1-g(t)))| \\leq N $$ so $$ t-1+g(B+1-g(t)) \\in\\left\\{g^{-1}(g(B)-N), g^{-1}(g(B)-N+1), \\ldots, g^{-1}(g(B)+N)\\right\\} $$ so by the maximality of $t$, we must have $g(B+1-g(t))=1$, so $B+1-g(t)=g^{-1}(1)$. We have $|g(t)-g(B)| \\leq N$, so $$ \\left|(B-g(B))+1-g^{-1}(1)\\right| \\leq N . $$ This is true for infinitely many values of $B$, so infinitely many values of $B-g(B)$ (by injectivity of $g(x)-x)$, which is a contradiction. This completes the proof."} diff --git a/USA_TSTST/md/en-sols-TSTST-2011.md b/USA_TSTST/md/en-sols-TSTST-2011.md new file mode 100644 index 0000000000000000000000000000000000000000..caab7201c5e37b150290c50db820d1cef04ccffd --- /dev/null +++ b/USA_TSTST/md/en-sols-TSTST-2011.md @@ -0,0 +1,321 @@ +# TSTST 2011 Solution Notes
Lincoln, Nebraska
\author{ Evan Chen《陳誼廷》 + +} + +15 April 2024 + +This is a compilation of solutions for the 2011 TSTST. The ideas of the solution are a mix of my own work, the solutions provided by the competition organizers, and solutions found by the community. However, all the writing is maintained by me. + +These notes will tend to be a bit more advanced and terse than the "official" solutions from the organizers. In particular, if a theorem or technique is not known to beginners but is still considered "standard", then I often prefer to use this theory anyways, rather than try to work around or conceal it. For example, in geometry problems I typically use directed angles without further comment, rather than awkwardly work around configuration issues. Similarly, sentences like "let $\mathbb{R}$ denote the set of real numbers" are typically omitted entirely. + +Corrections and comments are welcome! + +## Contents + +0 Problems ..... 2 +1 Solutions to Day 1 ..... 3 +1.1 TSTST 2011/1 ..... 3 +1.2 TSTST 2011/2 ..... 4 +1.3 TSTST 2011/3 ..... 6 +2 Solutions to Day 2 ..... 7 +2.1 TSTST 2011/4 ..... 7 +2.2 TSTST 2011/5 ..... 8 +2.3 TSTST 2011/6 ..... 9 +3 Solutions to Day 3 ..... 10 +3.1 TSTST 2011/7 ..... 10 +3.2 TSTST 2011/8 ..... 11 +3.3 TSTST 2011/9 ..... 12 + +## §0 Problems + +1. Find all real-valued functions $f$ defined on pairs of real numbers, having the following property: for all real numbers $a, b, c$, the median of $f(a, b), f(b, c), f(c, a)$ equals the median of $a, b, c$. +(The median of three real numbers, not necessarily distinct, is the number that is in the middle when the three numbers are arranged in nondecreasing order.) +2. Two circles $\omega_{1}$ and $\omega_{2}$ intersect at points $A$ and $B$. Line $\ell$ is tangent to $\omega_{1}$ at $P$ and to $\omega_{2}$ at $Q$ so that $A$ is closer to $\ell$ than $B$. Let $X$ and $Y$ be points on major $\operatorname{arcs} \widehat{P A}\left(\right.$ on $\left.\omega_{1}\right)$ and $\overparen{A Q}$ (on $\left.\omega_{2}\right)$, respectively, such that $A X / P X=A Y / Q Y=c$. Extend segments $P A$ and $Q A$ through $A$ to $R$ and $S$, respectively, such that $A R=A S=c \cdot P Q$. Given that the circumcenter of triangle $A R S$ lies on line $X Y$, prove that $\angle X P A=\angle A Q Y$. +3. Prove that there exists a real constant $c$ such that for any pair $(x, y)$ of real numbers, there exist relatively prime integers $m$ and $n$ satisfying the relation + +$$ +\sqrt{(x-m)^{2}+(y-n)^{2}}d_{2}>\cdots>d_{k}$ and $\operatorname{gcd}\left(d_{1}, d_{2}, \ldots, d_{k}\right)=1$. For every integer $n \geq n_{0}$, define + +$$ +x_{n}=\left\lfloor\frac{x_{n-d_{1}}+x_{n-d_{2}}+\cdots+x_{n-d_{k}}}{k}\right\rfloor . +$$ + +Show that the sequence $\left(x_{n}\right)$ is eventually constant. +9. Let $n$ be a positive integer. Suppose we are given $2^{n}+1$ distinct sets, each containing finitely many objects. Place each set into one of two categories, the red sets and the blue sets, so that there is at least one set in each category. We define the symmetric difference of two sets as the set of objects belonging to exactly one of the two sets. Prove that there are at least $2^{n}$ different sets which can be obtained as the symmetric difference of a red set and a blue set. + +## §1 Solutions to Day 1 + +## §1.1 TSTST 2011/1 + +Available online at https://aops.com/community/p2374841. + +## Problem statement + +Find all real-valued functions $f$ defined on pairs of real numbers, having the following property: for all real numbers $a, b, c$, the median of $f(a, b), f(b, c), f(c, a)$ equals the median of $a, b, c$. +(The median of three real numbers, not necessarily distinct, is the number that is in the middle when the three numbers are arranged in nondecreasing order.) + +The following solution is joint with Andrew He. +We prove the following main claim, from which repeated applications can deduce the problem. + +Claim - Let $ad_{2}>\cdots>d_{k}$ and $\operatorname{gcd}\left(d_{1}, d_{2}, \ldots, d_{k}\right)=1$. For every integer $n \geq n_{0}$, define + +$$ +x_{n}=\left\lfloor\frac{x_{n-d_{1}}+x_{n-d_{2}}+\cdots+x_{n-d_{k}}}{k}\right\rfloor +$$ + +Show that the sequence $\left(x_{n}\right)$ is eventually constant. + +Note that if the initial terms are contained in some interval $[A, B]$ then they will remain in that interval. Thus the sequence is eventually periodic. Discard initial terms and let the period be $T$; we will consider all indices modulo $T$ from now on. + +Let $M$ be the maximal term in the sequence (which makes sense since the sequence is periodic). Note that if $x_{n}=M$, we must have $x_{n-d_{i}}=M$ for all $i$ as well. By taking a linear combination $\sum c_{i} d_{i} \equiv 1(\bmod T)($ possibly be Bezout's theorem, since $\operatorname{gcd}_{i}\left(d_{i}\right)=1$ ), we conclude $x_{n-1}=M$, as desired. + +## §3.3 TSTST 2011/9 + +Available online at https://aops.com/community/p2374857. + +## Problem statement + +Let $n$ be a positive integer. Suppose we are given $2^{n}+1$ distinct sets, each containing finitely many objects. Place each set into one of two categories, the red sets and the blue sets, so that there is at least one set in each category. We define the symmetric difference of two sets as the set of objects belonging to exactly one of the two sets. Prove that there are at least $2^{n}$ different sets which can be obtained as the symmetric difference of a red set and a blue set. + +We can interpret the problem as working with binary strings of length $\ell \geq n+1$, with $\ell$ the number of elements across all sets. + +Let $F$ be a field of cardinality $2^{\ell}$, hence $F \cong \mathbb{F}_{2}^{\oplus \ell}$. +Then, we can think of red/blue as elements of $F$, so we have some $B \subseteq F$, and an $R \subseteq F$. We wish to prove that $|B+R| \geq 2^{n}$. Want $|B+R| \geq 2^{n}$. + +Equivalently, any element of a set $X$ with $|X|=2^{n}-1$ should omit some element of $|B+R|$. To prove this: we know $|B|+|R|=2^{n}+1$, and define + +$$ +P(b, r)=\prod_{x \in X}(b+r-x) +$$ + +Consider $b^{|B|-1} r^{|R|-1}$. The coefficient of is $\binom{2^{n}-1}{|B|-1}$, which is odd (say by Lucas theorem), so the nullstellensatz applies. + diff --git a/USA_TSTST/md/en-sols-TSTST-2012.md b/USA_TSTST/md/en-sols-TSTST-2012.md new file mode 100644 index 0000000000000000000000000000000000000000..1b37f2aca94da2084816d9e2cad22f79c15ae0b2 --- /dev/null +++ b/USA_TSTST/md/en-sols-TSTST-2012.md @@ -0,0 +1,454 @@ +# TSTST 2012 Solution Notes
Lincoln, Nebraska
\author{ Evan Chen《陳誼廷》 + +} + +15 April 2024 + +This is a compilation of solutions for the 2012 TSTST. The ideas of the solution are a mix of my own work, the solutions provided by the competition organizers, and solutions found by the community. However, all the writing is maintained by me. + +These notes will tend to be a bit more advanced and terse than the "official" solutions from the organizers. In particular, if a theorem or technique is not known to beginners but is still considered "standard", then I often prefer to use this theory anyways, rather than try to work around or conceal it. For example, in geometry problems I typically use directed angles without further comment, rather than awkwardly work around configuration issues. Similarly, sentences like "let $\mathbb{R}$ denote the set of real numbers" are typically omitted entirely. + +Corrections and comments are welcome! + +## Contents + +0 Problems ..... 2 +1 Solutions to Day 1 ..... 4 +1.1 TSTST 2012/1, proposed by Palmer Mebane ..... 4 +1.2 TSTST 2012/2 ..... 5 +1.3 TSTST 2012/3 ..... 6 +2 Solutions to Day 2 ..... 8 +2.1 TSTST 2012/4 ..... 8 +2.2 TSTST 2012/5 ..... 9 +2.3 TSTST 2012/6, proposed by Sung-Yoon Kim ..... 10 +3 Solutions to Day 3 ..... 11 +3.1 TSTST 2012/7 ..... 11 +3.2 TSTST 2012/8, proposed by Palmer Mebane ..... 13 +3.3 TSTST 2012/9, proposed by John Berman ..... 15 + +## §0 Problems + +1. Determine all infinite strings of letters with the following properties: +(a) Each letter is either $T$ or $S$, +(b) If position $i$ and $j$ both have the letter $T$, then position $i+j$ has the letter $S$, +(c) There are infinitely many integers $k$ such that position $2 k-1$ has the $k$ th $T$. +2. Let $A B C D$ be a quadrilateral with $A C=B D$. Diagonals $A C$ and $B D$ meet at $P$. Let $\omega_{1}$ and $O_{1}$ denote the circumcircle and circumcenter of triangle $A B P$. Let $\omega_{2}$ and $O_{2}$ denote the circumcircle and circumcenter of triangle $C D P$. Segment $B C$ meets $\omega_{1}$ and $\omega_{2}$ again at $S$ and $T$ (other than $B$ and $C$ ), respectively. Let $M$ and $N$ be the midpoints of minor arcs $\widehat{S P}$ (not including $B$ ) and $\overparen{T P}$ (not including $C$ ). Prove that $\overline{M N} \| \overline{O_{1} O_{2}}$. +3. Let $\mathbb{N}$ be the set of positive integers. Let $f: \mathbb{N} \rightarrow \mathbb{N}$ be a function satisfying the following two conditions: +(a) $f(m)$ and $f(n)$ are relatively prime whenever $m$ and $n$ are relatively prime. +(b) $n \leq f(n) \leq n+2012$ for all $n$. + +Prove that for any natural number $n$ and any prime $p$, if $p$ divides $f(n)$ then $p$ divides $n$. +4. In scalene triangle $A B C$, let the feet of the perpendiculars from $A$ to $\overline{B C}, B$ to $\overline{C A}$, $C$ to $\overline{A B}$ be $A_{1}, B_{1}, C_{1}$, respectively. Denote by $A_{2}$ the intersection of lines $B C$ and $B_{1} C_{1}$. Define $B_{2}$ and $C_{2}$ analogously. Let $D, E, F$ be the respective midpoints of sides $\overline{B C}, \overline{C A}, \overline{A B}$. Show that the perpendiculars from $D$ to $\overline{A A_{2}}, E$ to $\overline{B B_{2}}$ and $F$ to $\overline{C C_{2}}$ are concurrent. +5. A rational number $x$ is given. Prove that there exists a sequence $x_{0}, x_{1}, x_{2}, \ldots$ of rational numbers with the following properties: +(a) $x_{0}=x$; +(b) for every $n \geq 1$, either $x_{n}=2 x_{n-1}$ or $x_{n}=2 x_{n-1}+\frac{1}{n}$; +(c) $x_{n}$ is an integer for some $n$. +6. Positive real numbers $x, y, z$ satisfy $x y z+x y+y z+z x=x+y+z+1$. Prove that + +$$ +\frac{1}{3}\left(\sqrt{\frac{1+x^{2}}{1+x}}+\sqrt{\frac{1+y^{2}}{1+y}}+\sqrt{\frac{1+z^{2}}{1+z}}\right) \leq\left(\frac{x+y+z}{3}\right)^{5 / 8} . +$$ + +7. Triangle $A B C$ is inscribed in circle $\Omega$. The interior angle bisector of angle $A$ intersects side $B C$ and $\Omega$ at $D$ and $L$ (other than $A$ ), respectively. Let $M$ be the midpoint of side $B C$. The circumcircle of triangle $A D M$ intersects sides $A B$ and $A C$ again at $Q$ and $P$ (other than $A$ ), respectively. Let $N$ be the midpoint of segment $P Q$, and let $H$ be the foot of the perpendicular from $L$ to line $N D$. Prove that line $M L$ is tangent to the circumcircle of triangle $H M N$. +8. Let $n$ be a positive integer. Consider a triangular array of nonnegative integers as follows: +![](https://cdn.mathpix.com/cropped/2024_11_19_b4303c9fa48a8c74b45cg-03.jpg?height=244&width=809&top_left_y=238&top_left_x=675) + +Call such a triangular array stable if for every $0 \leq i1$, let $A_{k}=\left\{a_{1}, a_{2}, \ldots, a_{k}\right\}$. By (b) and symmetry, we have + +$$ +2 k-1 \geq \frac{\left|A_{k}-A_{k}\right|-1}{2}+\left|A_{k}\right| \geq \frac{2\left|A_{k}\right|-2}{2}+\left|A_{k}\right|=2 k-1 . +$$ + +But in order for $\left|A_{k}-A_{k}\right|=2\left|A_{k}\right|-1$, we must have $A_{k}$ an arithmetic progression, whence $a_{n}=2 n-1$ for all $n$ by taking $k$ arbitrarily large. + +## §1.2 TSTST 2012/2 + +Available online at https://aops.com/community/p2745851. + +## Problem statement + +Let $A B C D$ be a quadrilateral with $A C=B D$. Diagonals $A C$ and $B D$ meet at $P$. Let $\omega_{1}$ and $O_{1}$ denote the circumcircle and circumcenter of triangle $A B P$. Let $\omega_{2}$ and $O_{2}$ denote the circumcircle and circumcenter of triangle $C D P$. Segment $B C$ meets $\omega_{1}$ and $\omega_{2}$ again at $S$ and $T$ (other than $B$ and $C$ ), respectively. Let $M$ and $N$ be the midpoints of minor arcs $\widehat{S P}$ (not including $B$ ) and $\widehat{T P}$ (not including $C$ ). Prove that $\overline{M N} \| \overline{O_{1} O_{2}}$. + +Let $Q$ be the second intersection point of $\omega_{1}, \omega_{2}$. Suffice to show $\overline{Q P} \perp \overline{M N}$. Now $Q$ is the center of a spiral congruence which sends $\overline{A C} \mapsto \overline{B D}$. So $\triangle Q A B$ and $\triangle Q C D$ are similar isosceles. Now, + +$$ +\measuredangle Q P A=\measuredangle Q B A=\measuredangle D C Q=\measuredangle D P Q +$$ + +and so $\overline{Q P}$ is bisects $\angle B P C$. +![](https://cdn.mathpix.com/cropped/2024_11_19_b4303c9fa48a8c74b45cg-05.jpg?height=673&width=1200&top_left_y=1234&top_left_x=428) + +Now, let $I=\overline{B M} \cap \overline{C N} \cap \overline{P Q}$ be the incenter of $\triangle P B C$. Then $I M \cdot I B=I P \cdot I Q=$ $I N \cdot I C$, so $B M N C$ is cyclic, meaning $\overline{M N}$ is antiparallel to $\overline{B C}$ through $\angle B I C$. Since $\overline{Q P I}$ passes through the circumcenter of $\triangle B I C$, it follows now $\overline{Q P I} \perp \overline{M N}$ as desired. + +## §1.3 TSTST 2012/3 + +Available online at https://aops.com/community/p2745877. + +## Problem statement + +Let $\mathbb{N}$ be the set of positive integers. Let $f: \mathbb{N} \rightarrow \mathbb{N}$ be a function satisfying the following two conditions: +(a) $f(m)$ and $f(n)$ are relatively prime whenever $m$ and $n$ are relatively prime. +(b) $n \leq f(n) \leq n+2012$ for all $n$. + +Prove that for any natural number $n$ and any prime $p$, if $p$ divides $f(n)$ then $p$ divides $n$. + +【 First short solution, by Jeffrey Kwan. Let $p_{0}, p_{1}, p_{2}, \ldots$ denote the sequence of all prime numbers, in any order. Pick any primes $q_{i}$ such that + +$$ +q_{0}\left|f\left(p_{0}\right), \quad q_{1}\right| f\left(p_{1}\right), \quad q_{2} \mid f\left(p_{2}\right), \text { etc. } +$$ + +This is possible since each $f$ value above exceeds 1 . Also, since by hypothesis the $f\left(p_{i}\right)$ are pairwise coprime, the primes $q_{i}$ are all pairwise distinct. + +Claim - We must have $q_{i}=p_{i}$ for each $i$. (Therefore, $f\left(p_{i}\right)$ is a power of $p_{i}$ for each $i$.) + +Proof. Assume to the contrary that $q_{0} \neq p_{0}$. By changing labels if necessary, assume $\min \left(p_{1}, p_{2}, \ldots, p_{2012}\right)>2012$. Then by Chinese remainder theorem we can choose an integer $m$ such that + +$$ +\begin{array}{rr} +m+i & \equiv 0 \\ +m & \left(\bmod q_{i}\right) \\ +m & \equiv{ }^{\prime} \\ +\left(\bmod p_{i}\right) +\end{array} +$$ + +for $0 \leq i \leq 2012$. But now $f(m)$ should be coprime to all $f\left(p_{i}\right)$, ergo coprime to $q_{0} q_{1} \ldots q_{2012}$, violating $m \leq f(m) \leq m+2012$. + +All that is left to do is note that whenever $p \nmid n$, we have $\operatorname{gcd}(f(p), f(n))=1$, hence $p \nmid f(n)$. This is the contrapositive of the problem statement. + +【 Second solution with a grid. Fix $n$ and $p$, and assume for contradiction $p \nmid n$. +Claim - There exists a large integer $N$ with $f(N)=N$, that also satisfies $N \equiv 1$ $(\bmod n)$ and $N \equiv 0(\bmod p)$. + +Proof. We'll need to pick both $N$ and an ancillary integer $M$. Here is how: pick $2012 \cdot 2013$ distinct primes $q_{i, j}>n+p+2013$ for every $i=1, \ldots, 2012$ and $j=0, \ldots, 2012$, and use +it to fill in the following table: + +| | $N+1$ | $N+2$ | $\ldots$ | $N+2012$ | +| :---: | :---: | :---: | :---: | :---: | +| $M$ | $q_{0,1}$ | $q_{0,2}$ | $\ldots$ | $q_{0,2012}$ | +| $M+1$ | $q_{1,1}$ | $q_{1,2}$ | $\ldots$ | $q_{1,2012}$ | +| $\vdots$ | $\vdots$ | $\vdots$ | $\ddots$ | $\vdots$ | +| $M+2012$ | $q_{2012,1}$ | $q_{2012,2}$ | $\ldots$ | $q_{2012,2012}$ |. + +By the Chinese Remainder Theorem, we can construct $N$ such that $N+1 \equiv 0\left(\bmod q_{i, 1}\right)$ for every $i$, and similarly for $N+2$, and so on. Moreover, we can also tack on the extra conditions $N \equiv 0(\bmod p)$ and $N \equiv 1(\bmod n)$ we wanted. + +Notice that $N$ cannot be divisible by any of the $q_{i, j}$ 's, since the $q_{i, j}$ 's are greater than 2012. + +After we've chosen $N$, we can pick $M$ such that $M \equiv 0\left(\bmod q_{0, j}\right)$ for every $j$, and similarly $M+1 \equiv 0\left(\bmod q_{1, j}\right)$, et cetera. Moreover, we can tack on the condition $M \equiv 1$ $(\bmod N)$, which ensures $\operatorname{gcd}(M, N)=1$. + +What does this do? We claim that $f(N)=N$ now. Indeed $f(M)$ and $f(N)$ are relatively prime; but look at the table! The table tells us that $f(M)$ must have a common factor with each of $N+1, \ldots, N+2012$. So the only possibility is that $f(N)=N$. + +Now we're basically done. Since $N \equiv 1(\bmod n)$, we have $\operatorname{gcd}(N, n)=1$ and hence $1=\operatorname{gcd}(f(N), f(n))=\operatorname{gcd}(N, f(n))$. But $p \mid N$ and $p \mid f(n)$, contradiction. + +## §2 Solutions to Day 2 + +## §2.1 TSTST 2012/4 + +Available online at https://aops.com/community/p2745854. + +## Problem statement + +In scalene triangle $A B C$, let the feet of the perpendiculars from $A$ to $\overline{B C}, B$ to $\overline{C A}$, $C$ to $\overline{A B}$ be $A_{1}, B_{1}, C_{1}$, respectively. Denote by $A_{2}$ the intersection of lines $B C$ and $B_{1} C_{1}$. Define $B_{2}$ and $C_{2}$ analogously. Let $D, E, F$ be the respective midpoints of sides $\overline{B C}, \overline{C A}, \overline{A B}$. Show that the perpendiculars from $D$ to $\overline{A A_{2}}, E$ to $\overline{B B_{2}}$ and $F$ to $\overline{C C_{2}}$ are concurrent. + +We claim that they pass through the orthocenter $H$. Indeed, consider the circle with diameter $\overline{B C}$, which circumscribes quadrilateral $B C B_{1} C_{1}$ and has center $D$. Then by Brokard theorem, $\overline{A A_{2}}$ is the polar of line $H$. Thus $\overline{D H} \perp \overline{A A_{2}}$. + +## §2.2 TSTST 2012/5 + +Available online at https://aops.com/community/p2745867. + +## Problem statement + +A rational number $x$ is given. Prove that there exists a sequence $x_{0}, x_{1}, x_{2}, \ldots$ of rational numbers with the following properties: +(a) $x_{0}=x$; +(b) for every $n \geq 1$, either $x_{n}=2 x_{n-1}$ or $x_{n}=2 x_{n-1}+\frac{1}{n}$; +(c) $x_{n}$ is an integer for some $n$. + +Think of the sequence as a process over time. We'll show that: +Claim - At any given time $t$, if the denominator of $x_{t}$ has some odd prime power $q=p^{e}$, then we can delete a factor of $p$ from the denominator, while only adding powers of two to the denominator. +(Thus we can just delete off all the odd primes one by one and then double appropriately many times.) + +Proof. The idea is to add only fractions of the form $\left(2^{k} q\right)^{-1}$. +Indeed, let $n$ be large, and suppose $t<2^{r+1} q<2^{r+2} q<\cdots<2^{r+m} qb$ if $a b=b a=a$. The following are proved by finite casework, using the fact that $\{a b, b c, c a\}$ always has exactly two distinct elements for any different $a, b, c$. + +- If $a>b$ and $b>c$ then $a>c$. +- If $a \sim b$ and $b \sim c$ then $a b=a$ if and only if $b c=b$. +- If $a \sim b$ and $b \sim c$ then $a \sim c$. +- If $a \sim b$ and $a>c$ then $b>c$. +- If $a \sim b$ and $c>a$ then $c>b$. + +This gives us the total ordering on the elements and the equivalence classes by $\sim$. In this we way can check the claimed operations are the only ones. + +We can then (as in the first solution) verify that every full string is equivalent to a unique double rainbow - but this time we prove it by simply considering all possible $\times$, because we have classified them all. + diff --git a/USA_TSTST/md/en-sols-TSTST-2013.md b/USA_TSTST/md/en-sols-TSTST-2013.md new file mode 100644 index 0000000000000000000000000000000000000000..cbdc3c1a36246c57597081294e61fe93ca951e60 --- /dev/null +++ b/USA_TSTST/md/en-sols-TSTST-2013.md @@ -0,0 +1,437 @@ +# TSTST 2013 Solution Notes
Lincoln, Nebraska
\author{ Evan Chen《陳誼廷》 + +} + +23 April 2024 + +This is a compilation of solutions for the 2013 TSTST. The ideas of the solution are a mix of my own work, the solutions provided by the competition organizers, and solutions found by the community. However, all the writing is maintained by me. + +These notes will tend to be a bit more advanced and terse than the "official" solutions from the organizers. In particular, if a theorem or technique is not known to beginners but is still considered "standard", then I often prefer to use this theory anyways, rather than try to work around or conceal it. For example, in geometry problems I typically use directed angles without further comment, rather than awkwardly work around configuration issues. Similarly, sentences like "let $\mathbb{R}$ denote the set of real numbers" are typically omitted entirely. + +Corrections and comments are welcome! + +## Contents + +0 Problems ..... 2 +1 Solutions to Day 1 ..... 4 +1.1 TSTST 2013/1 ..... 4 +1.2 TSTST 2013/2 ..... 5 +1.3 TSTST 2013/3 ..... 6 +2 Solutions to Day 2 ..... 8 +2.1 TSTST 2013/4 ..... 8 +2.2 TSTST 2013/5 ..... 10 +2.3 TSTST 2013/6 ..... 11 +3 Solutions to Day 3 ..... 13 +3.1 TSTST 2013/7 ..... 13 +3.2 TSTST 2013/8 ..... 14 +3.3 TSTST 2013/9 ..... 15 + +## §0 Problems + +1. Let $A B C$ be a triangle and $D, E, F$ be the midpoints of $\operatorname{arcs} B C, C A, A B$ on the circumcircle. Line $\ell_{a}$ passes through the feet of the perpendiculars from $A$ to $\overline{D B}$ and $\overline{D C}$. Line $m_{a}$ passes through the feet of the perpendiculars from $D$ to $\overline{A B}$ and $\overline{A C}$. Let $A_{1}$ denote the intersection of lines $\ell_{a}$ and $m_{a}$. Define points $B_{1}$ and $C_{1}$ similarly. Prove that triangles $D E F$ and $A_{1} B_{1} C_{1}$ are similar to each other. +2. A finite sequence of integers $a_{1}, a_{2}, \ldots, a_{n}$ is called regular if there exists a real number $x$ satisfying + +$$ +\lfloor k x\rfloor=a_{k} \quad \text { for } 1 \leq k \leq n +$$ + +Given a regular sequence $a_{1}, a_{2}, \ldots, a_{n}$, for $1 \leq k \leq n$ we say that the term $a_{k}$ is forced if the following condition is satisfied: the sequence + +$$ +a_{1}, a_{2}, \ldots, a_{k-1}, b +$$ + +is regular if and only if $b=a_{k}$. +Find the maximum possible number of forced terms in a regular sequence with 1000 terms. +3. Divide the plane into an infinite square grid by drawing all the lines $x=m$ and $y=n$ for $m, n \in \mathbb{Z}$. Next, if a square's upper-right corner has both coordinates even, color it black; otherwise, color it white (in this way, exactly $1 / 4$ of the squares are black and no two black squares are adjacent). Let $r$ and $s$ be odd integers, and let $(x, y)$ be a point in the interior of any white square such that $r x-s y$ is irrational. Shoot a laser out of this point with slope $r / s$; lasers pass through white squares and reflect off black squares. Prove that the path of this laser will from a closed loop. +4. Circle $\omega$, centered at $X$, is internally tangent to circle $\Omega$, centered at $Y$, at $T$. Let $P$ and $S$ be variable points on $\Omega$ and $\omega$, respectively, such that line $P S$ is tangent to $\omega($ at $S)$. Determine the locus of $O$ - the circumcenter of triangle $P S T$. +5. Let $p$ be a prime. Prove that in a complete graph with $1000 p$ vertices whose edges are labelled with integers, one can find a cycle whose sum of labels is divisible by $p$. +6. Let $\mathbb{N}$ be the set of positive integers. Find all functions $f: \mathbb{N} \rightarrow \mathbb{N}$ that satisfy the equation + +$$ +f^{a b c-a}(a b c)+f^{a b c-b}(a b c)+f^{a b c-c}(a b c)=a+b+c +$$ + +for all $a, b, c \geq 2$. (Here $f^{k}$ means $f$ applied $k$ times.) +7. A country has $n$ cities, labelled $1,2,3, \ldots, n$. It wants to build exactly $n-1$ roads between certain pairs of cities so that every city is reachable from every other city via some sequence of roads. However, it is not permitted to put roads between pairs of cities that have labels differing by exactly 1 , and it is also not permitted to put a road between cities 1 and $n$. Let $T_{n}$ be the total number of possible ways to build these roads. +(a) For all odd $n$, prove that $T_{n}$ is divisible by $n$. +(b) For all even $n$, prove that $T_{n}$ is divisible by $n / 2$. +8. Define a function $f: \mathbb{N} \rightarrow \mathbb{N}$ by $f(1)=1, f(n+1)=f(n)+2^{f(n)}$ for every positive integer $n$. Prove that $f(1), f(2), \ldots, f\left(3^{2013}\right)$ leave distinct remainders when divided by $3^{2013}$. +9. Let $r$ be a rational number in the interval $[-1,1]$ and let $\theta=\cos ^{-1} r$. Call a subset $S$ of the plane good if $S$ is unchanged upon rotation by $\theta$ around any point of $S$ (in both clockwise and counterclockwise directions). Determine all values of $r$ satisfying the following property: The midpoint of any two points in a good set also lies in the set. + +## §1 Solutions to Day 1 + +## §1.1 TSTST 2013/1 + +Available online at https://aops.com/community/p3181479. + +## Problem statement + +Let $A B C$ be a triangle and $D, E, F$ be the midpoints of $\operatorname{arcs} B C, C A, A B$ on the circumcircle. Line $\ell_{a}$ passes through the feet of the perpendiculars from $A$ to $\overline{D B}$ and $\overline{D C}$. Line $m_{a}$ passes through the feet of the perpendiculars from $D$ to $\overline{A B}$ and $\overline{A C}$. Let $A_{1}$ denote the intersection of lines $\ell_{a}$ and $m_{a}$. Define points $B_{1}$ and $C_{1}$ similarly. Prove that triangles $D E F$ and $A_{1} B_{1} C_{1}$ are similar to each other. + +In fact, it is true for any points $D, E, F$ on the circumcircle. More strongly we contend: + +Claim - Point $A_{1}$ is the midpoint of $\overline{H D}$. + +Proof. Lines $m_{a}$ and $\ell_{a}$ are Simson lines, so they both pass through the point $(a+b+$ $c+d) / 2$ in complex coordinates. +![](https://cdn.mathpix.com/cropped/2024_11_19_3879e3a8fa25bc819b5bg-04.jpg?height=718&width=801&top_left_y=1297&top_left_x=633) + +Hence $A_{1} B_{1} C_{1}$ is similar to $D E F$ through a homothety at $H$ with ratio $\frac{1}{2}$. + +## §1.2 TSTST 2013/2 + +Available online at https://aops.com/community/p3181480. + +## Problem statement + +A finite sequence of integers $a_{1}, a_{2}, \ldots, a_{n}$ is called regular if there exists a real number $x$ satisfying + +$$ +\lfloor k x\rfloor=a_{k} \quad \text { for } 1 \leq k \leq n . +$$ + +Given a regular sequence $a_{1}, a_{2}, \ldots, a_{n}$, for $1 \leq k \leq n$ we say that the term $a_{k}$ is forced if the following condition is satisfied: the sequence + +$$ +a_{1}, a_{2}, \ldots, a_{k-1}, b +$$ + +is regular if and only if $b=a_{k}$. +Find the maximum possible number of forced terms in a regular sequence with 1000 terms. + +The answer is 985 . WLOG, by shifting $a_{1}=0$ (clearly $a_{1}$ isn't forced). Now, we construct regular sequences inductively using the following procedure. Start with the inequality + +$$ +\frac{0}{1} \leq x<\frac{1}{1} +$$ + +Then for each $k=2,3, \ldots, 1000$ we perform the following procedure. If there is no fraction of the form $F=\frac{m}{k}$ in the interval $A \leq xq>\max \{a, b\}$ be primes. Suppose $s=a^{p} b^{q}$ and $t=s^{2}$; then + +$$ +p g_{t}(a)+q g_{t}(b)=g_{t}\left(a^{p} b^{q}\right)=g_{t}(s)=f^{s^{2}-s}(s)-s=0 +$$ + +Now + +$$ +q \mid g_{t}(a)>-a \quad \text { and } \quad p \mid g_{t}(b)>-b \Longrightarrow g_{t}(a)=g_{t}(b)=0 +$$ + +and so we conclude $f^{t-a}(t)=a$ and $f^{t-b}(t)=b$ for $a, b \geq 2$. +In particular, if $a=n$ and $b=n+1$ then we deduce $f(n+1)=n$ for all $n \geq 2$, as desired. + +Remark. If you let $c=(a b)^{2}$ after the first lemma, you recover the 2-variable version! + +## §3 Solutions to Day 3 + +## §3.1 TSTST 2013/7 + +Available online at https://aops.com/community/p3181485. + +## Problem statement + +A country has $n$ cities, labelled $1,2,3, \ldots, n$. It wants to build exactly $n-1$ roads between certain pairs of cities so that every city is reachable from every other city via some sequence of roads. However, it is not permitted to put roads between pairs of cities that have labels differing by exactly 1 , and it is also not permitted to put a road between cities 1 and $n$. Let $T_{n}$ be the total number of possible ways to build these roads. +(a) For all odd $n$, prove that $T_{n}$ is divisible by $n$. +(b) For all even $n$, prove that $T_{n}$ is divisible by $n / 2$. + +You can just spin the tree! +Fixing $n$, the group $G=\mathbb{Z} / n \mathbb{Z}$ acts on the set of trees by rotation (where we imagine placing $1,2, \ldots, n$ along a circle). + +Claim - For odd $n$, all trees have trivial stabilizer. +Proof. One way to see this is to look at the degree sequence. Suppose $g^{e}$ fixes a tree $T$. Then so does $g^{k}$, for $k=\operatorname{gcd}(e, n)$. Then it follows that $n / k$ divides $\sum_{v} \operatorname{deg} v=2 n-2$. Since $\operatorname{gcd}(2 n-2, n)=1$ we must then have $k=n$. + +The proof for even $n$ is identical except that $\operatorname{gcd}(2 n-2, n)=2$ and hence each tree either has stabilizer with size $\leq 2$. + +There is also a proof using linear algebra, using Kirchoff's tree formula. (Overkill.) + +## §3.2 TSTST 2013/8 + +Available online at https://aops.com/community/p3181486. + +## Problem statement + +Define a function $f: \mathbb{N} \rightarrow \mathbb{N}$ by $f(1)=1, f(n+1)=f(n)+2^{f(n)}$ for every positive integer $n$. Prove that $f(1), f(2), \ldots, f\left(3^{2013}\right)$ leave distinct remainders when divided by $3^{2013}$. + +I'll prove by induction on $k \geq 1$ that any $3^{k}$ consecutive values of $f$ produce distinct residues modulo $3^{k}$. The base case $k=1$ is easily checked ( $f$ is always odd, hence $f$ cycles $1,0,2 \bmod 3)$. + +For the inductive step, assume it's true up to $k$. Since $2^{\bullet}\left(\bmod 3^{k+1}\right)$ cycles every $2 \cdot 3^{k}$, and $f$ is always odd, it follows that + +$$ +\begin{aligned} +f\left(n+3^{k}\right)-f(n) & =2^{f(n)}+2^{f(n+1)}+\cdots+2^{f\left(n+3^{k}-1\right)} \quad\left(\bmod 3^{k+1}\right) \\ +& \equiv 2^{1}+2^{3}+\cdots+2^{2 \cdot 3^{k}-1} \quad\left(\bmod 3^{k+1}\right) \\ +& =2 \cdot \frac{4^{3^{k}}-1}{4-1} +\end{aligned} +$$ + +Hence + +$$ +f\left(n+3^{k}\right)-f(n) \equiv C \quad\left(\bmod 3^{k+1}\right) \quad \text { where } \quad C=2 \cdot \frac{4^{3^{k}}-1}{4-1} +$$ + +noting that $C$ does not depend on $n$. Exponent lifting gives $\nu_{3}(C)=k$ hence $f(n)$, $f\left(n+3^{k}\right), f\left(n+2 \cdot 3^{k}\right)$ differ mod $3^{k+1}$ for all $n$, and the inductive hypothesis now solves the problem. + +## §3.3 TSTST 2013/9 + +Available online at https://aops.com/community/p3181487. + +## Problem statement + +Let $r$ be a rational number in the interval $[-1,1]$ and let $\theta=\cos ^{-1} r$. Call a subset $S$ of the plane good if $S$ is unchanged upon rotation by $\theta$ around any point of $S$ (in both clockwise and counterclockwise directions). Determine all values of $r$ satisfying the following property: The midpoint of any two points in a good set also lies in the set. + +The answer is that $r$ has this property if and only if $r=\frac{4 n-1}{4 n}$ for some integer $n$. +Throughout the solution, we will let $r=\frac{a}{b}$ with $b>0$ and $\operatorname{gcd}(a, b)=1$. We also let + +$$ +\omega=e^{i \theta}=\frac{a}{b} \pm \frac{\sqrt{b^{2}-a^{2}}}{b} i +$$ + +This means we may work with complex multiplication in the usual way; the rotation of $z$ through center $c$ is given by $z \mapsto \omega(z-c)+c$. + +For most of our proof, we start by constructing a good set as follows. + +- Start by letting $S_{0}=\{0,1\}$. +- Let $S_{i}$ consist of $S_{i-1}$ plus all points that can be obtained by rotating a point of $S_{i-1}$ through a different point of $S_{i-1}$ (with scale factor $\omega$ ). +- Let $S_{\infty}=\bigcup_{i \geq 0} S_{i}$. + +The set $S_{\infty}$ is the (minimal, by inclusion) good set containing 0 and 1 . We are going to show that for most values of $r$, we have $\frac{1}{2} \notin S_{\infty}$. + +Claim - If $b$ is odd, then $\frac{1}{2} \notin S_{\infty}$. +Proof. Idea: denominators that appear are always odd. +Consider the ring + +$$ +A=\mathbb{Z}_{\{b\}}=\left\{\frac{s}{t}|s, t \in \mathbb{Z}, t| b^{\infty}\right\} +$$ + +which consists of all rational numbers whose denominators divide $b^{\infty}$. Then, $0,1, \omega \in$ $A\left[\sqrt{b^{2}-a^{2}}\right]$ and hence $S_{\infty} \subseteq A\left[\sqrt{b^{2}-a^{2}}\right]$ too. (This works even if $\sqrt{b^{2}-a^{2}} \in \mathbb{Z}$, in which case $S_{\infty} \subseteq A=A\left[\sqrt{b^{2}-a^{2}}\right]$.) +But $\frac{1}{2} \notin A\left[\sqrt{b^{2}-a^{2}}\right]$. + +Claim - If $b$ is even and $|b-a| \neq 1$, then $\frac{1}{2} \notin S_{\infty}$. +Proof. Idea: take modulo a prime dividing $b-a$. +Let $D=b^{2}-a^{2} \equiv 3(\bmod 4)$. Let $p$ be a prime divisor of $b-a$. Because $\operatorname{gcd}(a, b)=1$, we have $p \neq 2$ and $p \nmid b$. + +Consider the ring + +$$ +A=\mathbb{Z}_{(p)}=\left\{\left.\frac{s}{t} \right\rvert\, s, t \in \mathbb{Z}, p \perp t\right\} +$$ + +which consists of all rational numbers whose denominators are coprime to $p$. Then, $0,1, \omega \in A[\sqrt{-D}]$ and hence $S_{\infty} \subseteq A[\sqrt{-D}]$ too. + +Now, there is a well-defined "mod- $p$ " ring homomorphism + +$$ +\Psi: A[\sqrt{-D}] \rightarrow \mathbb{F}_{p} \quad \text { by } \quad x+y \sqrt{-D} \mapsto x \bmod p +$$ + +which commutes with addition and multiplication (as $p \mid D$ ). Under this map, + +$$ +\omega \mapsto \frac{a}{b} \bmod p=1 +$$ + +Consequently, the rotation $z \mapsto \omega(z-c)+c$ is just the identity map modulo $p$. In other words, the pre-image of any point in $S_{\infty}$ under $\Psi$ must be either $\Psi(0)=0$ or $\Psi(1)=1$. + +However, $\Psi(1 / 2)=1 / 2$ is neither of these. So this point cannot be achieved. + +Claim - Suppose $a=2 n-1$ and $b=2 n$ for $n$ an odd integer. Then $\frac{1}{2} \notin S_{\infty}$ + +Proof. Idea: $\omega$ is "algebraic integer" sans odd denominators. +This time, we define the ring + +$$ +B=\mathbb{Z}_{(2)}=\left\{\left.\frac{s}{t} \right\rvert\, s, t \in \mathbb{Z}, t \text { odd }\right\} +$$ + +of rational numbers with odd denominator. We carefully consider the ring $B[\omega]$ where + +$$ +\omega=\frac{2 n-1 \pm \sqrt{1-4 n}}{2 n} +$$ + +So $S_{\infty} \subseteq B[\omega]$ as $0,1, \omega \in B[\omega]$. +I claim that $B[\omega]$ is an integral extension of $B$; equivalently that $\omega$ is integral over $B$. Indeed, $\omega$ is the root of the monic polynomial + +$$ +(T-1)^{2}+\frac{1}{n}(T-1)-\frac{1}{n}=0 +$$ + +where $\frac{1}{n} \in B$ makes sense as $n$ is odd. +On the other hand, $\frac{1}{2}$ is not integral over $B$ so it is not an element of $B[\omega]$. +It remains to show that if $r=\frac{4 n-1}{4 n}$, then goods sets satisfy the midpoint property. Again starting from the points $z_{0}=0, z_{1}=1$ construct the sequence + +$$ +\begin{aligned} +z_{2} & =\omega\left(z_{1}-z_{0}\right)+z_{0} \\ +z_{3} & =\omega^{-1}\left(z_{0}-z_{2}\right)+z_{2} \\ +z_{4} & =\omega^{-1}\left(z_{2}-z_{3}\right)+z_{3} \\ +z_{5} & =\omega\left(z_{3}-z_{4}\right)+z_{4} +\end{aligned} +$$ + +as shown in the diagram below. +![](https://cdn.mathpix.com/cropped/2024_11_19_3879e3a8fa25bc819b5bg-16.jpg?height=321&width=795&top_left_y=2307&top_left_x=633) + +This construction shows that if we have the length-one segment $\{0,1\}$ then we can construct the length-one segment $\{2 r-2,2 r-1\}$. In other words, we can shift the segment to the left by + +$$ +1-(2 r-1)=2(1-r)=\frac{1}{2 n} +$$ + +Repeating this construction $n$ times gives the desired midpoint $\frac{1}{2}$. + diff --git a/USA_TSTST/md/en-sols-TSTST-2014.md b/USA_TSTST/md/en-sols-TSTST-2014.md new file mode 100644 index 0000000000000000000000000000000000000000..a92c6ffe27354eb4368632d05024e9d8d858dafd --- /dev/null +++ b/USA_TSTST/md/en-sols-TSTST-2014.md @@ -0,0 +1,254 @@ +# TSTST 2014 Solution Notes + +## Lincoln, Nebraska + +## Evan Chen《陳誼廷》 + +15 April 2024 + +This is a compilation of solutions for the 2014 TSTST. The ideas of the solution are a mix of my own work, the solutions provided by the competition organizers, and solutions found by the community. However, all the writing is maintained by me. + +These notes will tend to be a bit more advanced and terse than the "official" solutions from the organizers. In particular, if a theorem or technique is not known to beginners but is still considered "standard", then I often prefer to use this theory anyways, rather than try to work around or conceal it. For example, in geometry problems I typically use directed angles without further comment, rather than awkwardly work around configuration issues. Similarly, sentences like "let $\mathbb{R}$ denote the set of real numbers" are typically omitted entirely. + +Corrections and comments are welcome! + +## Contents + +0 Problems ..... 2 +1 Solutions to Day 1 ..... 3 +1.1 TSTST 2014/1 ..... 3 +1.2 TSTST 2014/2 ..... 4 +1.3 TSTST 2014/3 ..... 5 +2 Solutions to Day 2 ..... 6 +2.1 TSTST 2014/4 ..... 6 +2.2 TSTST 2014/5 ..... 7 +2.3 TSTST 2014/6 ..... 8 + +## §0 Problems + +1. Let $\leftarrow$ denote the left arrow key on a standard keyboard. If one opens a text editor and types the keys "ab $\leftarrow \mathrm{cd} \leftarrow \leftarrow \mathrm{e} \leftarrow \leftarrow \mathrm{f}$ ", the result is "faecdb". We say that a string $B$ is reachable from a string $A$ if it is possible to insert some amount of $\leftarrow$ 's in $A$, such that typing the resulting characters produces $B$. So, our example shows that "faecdb" is reachable from "abcdef". + +Prove that for any two strings $A$ and $B, A$ is reachable from $B$ if and only if $B$ is reachable from $A$. +2. Consider a convex pentagon circumscribed about a circle. We name the lines that connect vertices of the pentagon with the opposite points of tangency with the circle gergonnians. +(a) Prove that if four gergonnians are concurrent, then all five of them are concurrent. +(b) Prove that if there is a triple of gergonnians that are concurrent, then there is another triple of gergonnians that are concurrent. +3. Find all polynomials $P(x)$ with real coefficients that satisfy + +$$ +P(x \sqrt{2})=P\left(x+\sqrt{1-x^{2}}\right) +$$ + +for all real numbers $x$ with $|x| \leq 1$. +4. Let $P(x)$ and $Q(x)$ be arbitrary polynomials with real coefficients, with $P \neq 0$, and let $d=\operatorname{deg} P$. Prove that there exist polynomials $A(x)$ and $B(x)$, not both zero, such that $\max \{\operatorname{deg} A, \operatorname{deg} B\} \leq d / 2$ and $P(x) \mid A(x)+Q(x) \cdot B(x)$. +5. Find the maximum number $E$ such that the following holds: there is an edge-colored graph with 60 vertices and $E$ edges, with each edge colored either red or blue, such that in that coloring, there is no monochromatic cycles of length 3 and no monochromatic cycles of length 5 . +6. Suppose we have distinct positive integers $a, b, c, d$ and an odd prime $p$ not dividing any of them, and an integer $M$ such that if one considers the infinite sequence + +$$ +\begin{gathered} +c a-d b \\ +c a^{2}-d b^{2} \\ +c a^{3}-d b^{3} \\ +c a^{4}-d b^{4} +\end{gathered} +$$ + +and looks at the highest power of $p$ that divides each of them, these powers are not all zero, and are all at most $M$. Prove that there exists some $T$ (which may depend on $a, b, c, d, p, M)$ such that whenever $p$ divides an element of this sequence, the maximum power of $p$ that divides that element is exactly $p^{T}$. + +## §1 Solutions to Day 1 + +## §1.1 TSTST 2014/1 + +Available online at https://aops.com/community/p3549404. + +## Problem statement + +Let $\leftarrow$ denote the left arrow key on a standard keyboard. If one opens a text editor and types the keys "ab $\leftarrow \mathrm{cd} \leftarrow \leftarrow \mathrm{e} \leftarrow \leftarrow \mathrm{f}$ ", the result is "faecdb". We say that a string $B$ is reachable from a string $A$ if it is possible to insert some amount of $\leftarrow$ 's in $A$, such that typing the resulting characters produces $B$. So, our example shows that "faecdb" is reachable from "abcdef". + +Prove that for any two strings $A$ and $B, A$ is reachable from $B$ if and only if $B$ is reachable from $A$. + +Obviously $A$ and $B$ should have the same multiset of characters, and we focus only on that situation. + +Claim - If $A=123 \ldots n$ and $B=\sigma(1) \sigma(2) \ldots \sigma(n)$ is a permutation of $A$, then $B$ is reachable if and only if it is $\mathbf{2 1 3}$-avoiding, i.e. there are no indices $i0}$ such that $x \equiv y \equiv 1(\bmod p)$. If the sequence $\nu_{p}\left(x^{n}-y\right)$ of positive integers is nonconstant, then it is unbounded. + +For this it would be sufficient to prove the following claim. +Claim - Let $p$ be an odd prime. Let $x, y \in \mathbb{Q}>0$ such that $x \equiv y \equiv 1(\bmod p)$. Suppose $m$ and $n$ are positive integers such that + +$$ +d=\nu_{p}\left(x^{n}-y\right)<\nu_{p}\left(x^{m}-y\right)=e . +$$ + +Then there exists $\ell$ such that $\nu_{p}\left(x^{\ell}-y\right) \geq e+1$. +Proof. First, note that $\nu_{p}\left(x^{m}-x^{n}\right)=\nu_{p}\left(\left(x^{m}-y\right)-\left(x^{n}-y\right)\right)=d$ and so by exponent lifting we can find some $k$ such that + +$$ +\nu_{p}\left(x^{k}-1\right)=e +$$ + +namely $k=p^{e-d}|m-n|$. (In fact, one could also choose more carefully $k=p^{e-d} \cdot \operatorname{gcd}(m-$ $\left.n, p^{\infty}\right)$, so that $k$ is a power of $p$.) + +Suppose we set $x^{k}=p^{e} u+1$ and $x^{m}=p^{e} v+y$ where $u, v \in \mathbb{Q}$ aren't divisible by $p$. Now for any integer $1 \leq r \leq p-1$ we consider + +$$ +\begin{aligned} +x^{k r+m}-y & =\left(p^{e} u+1\right)^{r} \cdot\left(p^{e} v+y\right)-y \\ +& =p^{e}(v+y u \cdot r)+p^{2 e}(\ldots) +\end{aligned} +$$ + +By selecting $r$ with $r \equiv-v / u(\bmod p)$, we ensure $p^{e+1} \mid x^{k r+m}-y$, hence $\ell=k r+m$ is as desired. + +Remark. One way to motivate the proof of the claim is as follows. Suppose we are given $\nu_{p}\left(x^{m}-y\right)=e$, and we wish to find $\ell$ such that $\nu_{p}\left(x^{\ell}-y\right)>e$. Then, it is necessary (albeit insufficient) that + +$$ +x^{\ell-m} \equiv 1 \quad\left(\bmod p^{e}\right) \text { but } x^{\ell-m} \not \equiv 1 \quad\left(\bmod p^{e+1}\right) +$$ + +In particular, we need $\nu_{p}\left(x^{\ell-m}-1\right)=e$ exactly. So the $k$ in the claim must exist if we are going to succeed. + +On the other hand, if $k$ is some integer for which $\nu_{p}\left(x^{k}-1\right)=e$, then by choosing $\ell-m$ to be some multiple of $k$ with no extra factors of $p$, we hope that we can get $\nu_{p}\left(x^{\ell}-y\right)=e+1$. That's why we write $\ell=k r+m$ and see what happens when we expand. + diff --git a/USA_TSTST/md/en-sols-TSTST-2015.md b/USA_TSTST/md/en-sols-TSTST-2015.md new file mode 100644 index 0000000000000000000000000000000000000000..02871c0434039a1345406029ad37bb630783b57e --- /dev/null +++ b/USA_TSTST/md/en-sols-TSTST-2015.md @@ -0,0 +1,304 @@ +# TSTST 2015 Solution Notes + +## Pittsburgh, PA + +Evan Chen《陳誼廷》 + +15 April 2024 + +This is a compilation of solutions for the 2015 TSTST. The ideas of the solution are a mix of my own work, the solutions provided by the competition organizers, and solutions found by the community. However, all the writing is maintained by me. + +These notes will tend to be a bit more advanced and terse than the "official" solutions from the organizers. In particular, if a theorem or technique is not known to beginners but is still considered "standard", then I often prefer to use this theory anyways, rather than try to work around or conceal it. For example, in geometry problems I typically use directed angles without further comment, rather than awkwardly work around configuration issues. Similarly, sentences like "let $\mathbb{R}$ denote the set of real numbers" are typically omitted entirely. + +Corrections and comments are welcome! + +## Contents + +0 Problems ..... 2 +1 Solutions to Day 1 ..... 3 +1.1 TSTST 2015/1, proposed by Mark Sellke ..... 3 +1.2 TSTST 2015/2, proposed by Ivan Borsenco +1.3 TSTST 2015/3, proposed by Alex Zhai +2 Solutions to Day 2 ..... 6 +2.1 TSTST 2015/4, proposed by Alyazeed Basyoni ..... 6 +2.2 TSTST 2015/5 +2.3 TSTST 2015/6, proposed by Linus Hamilton + +## §0 Problems + +1. Let $a_{1}, a_{2}, \ldots, a_{n}$ be a sequence of real numbers, and let $m$ be a fixed positive integer less than $n$. We say an index $k$ with $1 \leq k \leq n$ is good if there exists some $\ell$ with $1 \leq \ell \leq m$ such that + +$$ +a_{k}+a_{k+1}+\cdots+a_{k+\ell-1} \geq 0 +$$ + +where the indices are taken modulo $n$. Let $T$ be the set of all good indices. Prove that + +$$ +\sum_{k \in T} a_{k} \geq 0 +$$ + +2. Let $A B C$ be a scalene triangle. Let $K_{a}, L_{a}$, and $M_{a}$ be the respective intersections with $B C$ of the internal angle bisector, external angle bisector, and the median from $A$. The circumcircle of $A K_{a} L_{a}$ intersects $A M_{a}$ a second time at a point $X_{a}$ different from $A$. Define $X_{b}$ and $X_{c}$ analogously. Prove that the circumcenter of $X_{a} X_{b} X_{c}$ lies on the Euler line of $A B C$. +3. Let $P$ be the set of all primes, and let $M$ be a non-empty subset of $P$. Suppose that for any non-empty subset $\left\{p_{1}, p_{2}, \ldots, p_{k}\right\}$ of $M$, all prime factors of $p_{1} p_{2} \ldots p_{k}+1$ are also in $M$. Prove that $M=P$. +4. Let $x, y, z$ be real numbers (not necessarily positive) such that $x^{4}+y^{4}+z^{4}+x y z=4$. Prove that $x \leq 2$ and + +$$ +\sqrt{2-x} \geq \frac{y+z}{2} +$$ + +5. Let $\varphi(n)$ denote the number of positive integers less than $n$ that are relatively prime to $n$. Prove that there exists a positive integer $m$ for which the equation $\varphi(n)=m$ has at least 2015 solutions in $n$. +6. A Nim-style game is defined as follows. Two positive integers $k$ and $n$ are specified, along with a finite set $S$ of $k$-tuples of integers (not necessarily positive). At the start of the game, the $k$-tuple $(~ n, 0,0, \ldots, 0)$ is written on the blackboard. +A legal move consists of erasing the tuple $\left(a_{1}, a_{2}, \ldots, a_{k}\right)$ which is written on the blackboard and replacing it with $\left(a_{1}+b_{1}, a_{2}+b_{2}, \ldots, a_{k}+b_{k}\right)$, where $\left(b_{1}, b_{2}, \ldots, b_{k}\right)$ is an element of the set $S$. Two players take turns making legal moves, and the first to write a negative integer loses. In the event that neither player is ever forced to write a negative integer, the game is a draw. +Prove that there is a choice of $k$ and $S$ with the following property: the first player has a winning strategy if $n$ is a power of 2 , and otherwise the second player has a winning strategy. + +## §1 Solutions to Day 1 + +## §1.1 TSTST 2015/1, proposed by Mark Sellke + +Available online at https://aops.com/community/p5017901. + +## Problem statement + +Let $a_{1}, a_{2}, \ldots, a_{n}$ be a sequence of real numbers, and let $m$ be a fixed positive integer less than $n$. We say an index $k$ with $1 \leq k \leq n$ is good if there exists some $\ell$ with $1 \leq \ell \leq m$ such that + +$$ +a_{k}+a_{k+1}+\cdots+a_{k+\ell-1} \geq 0 +$$ + +where the indices are taken modulo $n$. Let $T$ be the set of all good indices. Prove that + +$$ +\sum_{k \in T} a_{k} \geq 0 +$$ + +First we prove the result if the indices are not taken modulo $n$. Call a number $\ell$-good if $\ell$ is the smallest number such that $a_{k}+a_{k+1}+\cdots+a_{k+\ell-1} \geq 0$, and $\ell \leq m$. Then if $a_{k}$ is $\ell$-good, the numbers $a_{k+1}, \ldots, a_{k+\ell-1}$ are good as well. + +Then by greedy from left to right, we can group all the good numbers into blocks with nonnegative sums. Repeatedly take the first good number, if $\ell$-good, group it with the next $\ell$ numbers. An example for $m=3$ : + +$$ +\langle 4\rangle \quad \begin{array}{ccccccccc} +-1 & -2 & 6\rangle & -9 & -7 & \langle 3\rangle & \langle-2 & 4\rangle & \langle-1 . +\end{array} +$$ + +We can now return to the original problem. Let $N$ be a large integer; applying the algorithm to $N$ copies of the sequence, we deduce that + +$$ +N \sum_{k \in T} a_{k}+c_{N} \geq 0 +$$ + +where $c_{N}$ represents some "error" from left-over terms. As $\left|c_{N}\right| \leq \sum\left|a_{i}\right|$, by taking $N$ large enough we deduce the problem. + +Remark. This solution was motivated by looking at the case $m=1$, realizing how dumb it was, then looking at $m=2$, and realizing it was equally dumb. + +## §1.2 TSTST 2015/2, proposed by Ivan Borsenco + +Available online at https://aops.com/community/p5017915. + +## Problem statement + +Let $A B C$ be a scalene triangle. Let $K_{a}, L_{a}$, and $M_{a}$ be the respective intersections with $B C$ of the internal angle bisector, external angle bisector, and the median from $A$. The circumcircle of $A K_{a} L_{a}$ intersects $A M_{a}$ a second time at a point $X_{a}$ different from $A$. Define $X_{b}$ and $X_{c}$ analogously. Prove that the circumcenter of $X_{a} X_{b} X_{c}$ lies on the Euler line of $A B C$. + +The main content of the problem: +Claim - $\angle H X_{a} G=90^{\circ}$. +This implies the result, since then the desired circumcenter is the midpoint of $\overline{G H}$. (This is the main difficulty; the Euler line is a red herring.) +In what follows, we abbreviate $K_{a} L_{a}, M_{a}, X_{a}$ to $K, L, M, X$. +First proof by Brokard. To do this, it suffices to show that $M$ has the same power with respect to the circle with diameter $\overline{A H}$ and the circle with diameter $\overline{K L}$. In fact I claim both circles are orthogonal to the circle with diameter $\overline{B C}$ ! The former follows from Brokard's theorem, noting that $A$ is on the polar of $H$, and the latter follows from the harmonic bundle. +![](https://cdn.mathpix.com/cropped/2024_11_19_8fb7b517def5801eef36g-04.jpg?height=635&width=1109&top_left_y=1410&top_left_x=482) + +Then $\overline{A M}$ is the radical axis, so $X$ lies on both circles. +Second proof by orthocenter reflection, Bendit Chan. As before, we know $M X \cdot M A=$ $M K \cdot M L=M B \cdot M C$, but $X$ lies inside segment $A M$. Construct parallelogram $A B A^{\prime} C$. Then $M X \cdot M A^{\prime}=M B \cdot M C$, so $X B A^{\prime} C$ is concyclic. + +However, it is well-known the circumcircle of $\triangle B A^{\prime} C$ (which is the reflection of $(A B C)$ across $\overline{B C}$ ) passes through $H$ and in fact has diameter $\overline{A^{\prime} H}$. So this gives $\angle H X A^{\prime}=90^{\circ}$ as needed. +Third proof by barycentric coordinates. Alternatively we may just compute $X=\left(a^{2}\right.$ : $\left.2 S_{A}: 2 S_{A}\right)$. Let $F=\left(0: S_{C}: S_{B}\right)$ be the foot from $H$. Then we check that $X H F M$ is cyclic, which is power of a point from $A$. + +## §1.3 TSTST 2015/3, proposed by Alex Zhai + +Available online at https://aops.com/community/p5017928. + +## Problem statement + +Let $P$ be the set of all primes, and let $M$ be a non-empty subset of $P$. Suppose that for any non-empty subset $\left\{p_{1}, p_{2}, \ldots, p_{k}\right\}$ of $M$, all prime factors of $p_{1} p_{2} \ldots p_{k}+1$ are also in $M$. Prove that $M=P$. + +The following solution was found by user Aiscrim on AOPS. +Obviously $|M|=\infty$. Assume for contradiction $p \notin M$. We say a prime $q \in M$ is sparse if there are only finitely many elements of $M$ which are $q(\bmod p)$ (in particular there are finitely many sparse primes). + +Now let $C$ be the product of all sparse primes (note $p \nmid C$ ). First, set $a_{0}=1$. For $k \geq 0$, consider then the prime factorization of the number + +$$ +C a_{k}+1 +$$ + +No prime in its factorization is sparse, so consider the number $a_{k+1}$ obtained by replacing each prime in its factorization with some arbitrary representative of that prime's residue class. In this way we select a number $a_{k+1}$ such that + +- $a_{k+1} \equiv C a_{k}+1(\bmod p)$, and +- $a_{k+1}$ is a product of distinct primes in $M$. + +In particular, $a_{k} \equiv C^{k}+C^{k-1}+\cdots+1(\bmod p)$ +But since $C \not \equiv 0(\bmod p)$, we can find a $k$ such that $a_{k} \equiv 0(\bmod p)($ namely, $k=p-1$ if $C \equiv 1$ and $k=p-2$ else) which is clearly impossible since $a_{k}$ is a product of primes in $M$ ! + +## §2 Solutions to Day 2 + +## §2.1 TSTST 2015/4, proposed by Alyazeed Basyoni + +Available online at https://aops.com/community/p5017801. + +## Problem statement + +Let $x, y, z$ be real numbers (not necessarily positive) such that $x^{4}+y^{4}+z^{4}+x y z=4$. Prove that $x \leq 2$ and + +$$ +\sqrt{2-x} \geq \frac{y+z}{2} +$$ + +We prove that the condition $x^{4}+y^{4}+z^{4}+x y z=4$ implies + +$$ +\sqrt{2-x} \geq \frac{y+z}{2} +$$ + +We first prove the easy part. +Claim - We have $x \leq 2$. +Proof. Indeed, AM-GM gives that + +$$ +\begin{aligned} +5=x^{4}+y^{4}+\left(z^{4}+1\right)+x y z & =\frac{3 x^{4}}{4}+\left(\frac{x^{4}}{4}+y^{4}\right)+\left(z^{4}+1\right)+x y z \\ +& \geq \frac{3 x^{4}}{4}+x^{2} y^{2}+2 z^{2}+x y z . +\end{aligned} +$$ + +We evidently have that $x^{2} y^{2}+2 z^{2}+x y z \geq 0$ because the quadratic form $a^{2}+a b+2 b^{2}$ is positive definite, so $x^{4} \leq \frac{20}{3} \Longrightarrow x \leq 2$. + +Now, the desired statement is implied by its square, so it suffices to show that + +$$ +2-x \geq\left(\frac{y+z}{2}\right)^{2} +$$ + +We are going to proceed by contradiction (it seems that many solutions do this) and assume that + +$$ +2-x<\left(\frac{y+z}{2}\right)^{2} \Longleftrightarrow 4 x+y^{2}+2 y z+z^{2}>8 +$$ + +By AM-GM, + +$$ +\begin{aligned} +x^{4}+3 & \geq 4 x \\ +\frac{y^{4}+1}{2} & \geq y^{2} \\ +\frac{z^{4}+1}{2} & \geq z^{2} +\end{aligned} +$$ + +which yields that + +$$ +x^{4}+\frac{y^{4}+z^{4}}{2}+2 y z+4>8 +$$ + +If we replace $x^{4}=4-\left(y^{4}+z^{4}+x y z\right)$ now, this gives + +$$ +-\frac{y^{4}+z^{4}}{2}+(2-x) y z>0 \Longrightarrow(2-x) y z>\frac{y^{4}+z^{4}}{2} +$$ + +Since $2-x$ and the right-hand side are positive, we have $y z \geq 0$. Now + +$$ +\frac{y^{4}+z^{4}}{2 y z}<2-x<\left(\frac{y+z}{2}\right)^{2} \Longrightarrow 2 y^{4}+2 z^{4}2015$ we're done. +Remark. This solution is motivated by the deep fact that $\varphi(11 \cdot 1000)=\varphi(10 \cdot 1000)$, for example. + +【 Second solution with smallest primes, by Yang Liu. Let $2=p_{1}1$. Then Alice can complete the phase without losing if and only if $x$ is even; if so she begins a $Y$-phase with $(X, Y)=(0, x / 2)$. + +Proof. As $x>1$, Alice cannot play ClaimX since Bob will respond with FakeX and win. Now by alternating between WorkX and WorkX', Alice can repeatedly deduct 2 from $X$ and add 1 to $Y$, leading to $(x-2, y+1),(x-4, y+2)$, and so on. (During this time, Bob can only play Sleep.) Eventually, she must stop this process by playing DoneX, which begins a $Y$-phase. + +Now note that unless $X=0$, Bob now has a winning move WrongX. Conversely he may only play Sleep if $X=0$. + +We have an analogous claim for $Y$-phases. Thus if $n$ is not a power of 2 , we see that Alice eventually loses. + +Now suppose $n=2^{k}$; then Alice reaches $(X, Y)=\left(0,2^{k-1}\right),\left(2^{k-2}, 0\right), \ldots$ until either reaching $(1,0)$ or $(0,1)$. At this point she can play ClaimX or ClaimY, respectively; the game is now in state Cl. Bob cannot play either FakeX or FakeY, so he must play Sleep, and then Alice wins by playing Win. Thus Alice has a winning strategy when $n=2^{k}$. + diff --git a/USA_TSTST/md/en-sols-TSTST-2016.md b/USA_TSTST/md/en-sols-TSTST-2016.md new file mode 100644 index 0000000000000000000000000000000000000000..fe89cf8b0ea70b2071fd4affbeb8c0ad493e9eef --- /dev/null +++ b/USA_TSTST/md/en-sols-TSTST-2016.md @@ -0,0 +1,278 @@ +# USA TSTST 2016 Solutions
United States of America - TST Selection Test
Evan Chen《陳誼廷》
$58^{\text {th }}$ IMO 2017 Brazil and $6^{\text {th }}$ EGMO 2017 Switzerland + +## Contents + +0 Problems ..... 2 +1 Solutions to Day 1 ..... 3 +1.1 TSTST 2016/1, proposed by Victor Wang ..... 3 +1.2 TSTST 2016/2, proposed by Evan Chen ..... 4 +1.3 TSTST 2016/3, proposed by Yang Liu ..... 6 +2 Solutions to Day 2 ..... 8 +2.1 TSTST 2016/4, proposed by Linus Hamilton ..... 8 +2.2 TSTST 2016/5, proposed by Linus Hamilton, Cynthia Stoner ..... 9 +2.3 TSTST 2016/6, proposed by Danielle Wang ..... 10 + +## §0 Problems + +1. Let $A=A(x, y)$ and $B=B(x, y)$ be two-variable polynomials with real coefficients. Suppose that $A(x, y) / B(x, y)$ is a polynomial in $x$ for infinitely many values of $y$, and a polynomial in $y$ for infinitely many values of $x$. Prove that $B$ divides $A$, meaning there exists a third polynomial $C$ with real coefficients such that $A=B \cdot C$. +2. Let $A B C$ be a scalene triangle with orthocenter $H$ and circumcenter $O$ and denote by $M, N$ the midpoints of $\overline{A H}, \overline{B C}$. Suppose the circle $\gamma$ with diameter $\overline{A H}$ meets the circumcircle of $A B C$ at $G \neq A$, and meets line $\overline{A N}$ at $Q \neq A$. The tangent to $\gamma$ at $G$ meets line $O M$ at $P$. Show that the circumcircles of $\triangle G N Q$ and $\triangle M B C$ intersect on $\overline{P N}$. +3. Decide whether or not there exists a nonconstant polynomial $Q(x)$ with integer coefficients with the following property: for every positive integer $n>2$, the numbers + +$$ +Q(0), Q(1), Q(2), \ldots, Q(n-1) +$$ + +produce at most $0.499 n$ distinct residues when taken modulo $n$. +4. Prove that if $n$ and $k$ are positive integers satisfying $\varphi^{k}(n)=1$, then $n \leq 3^{k}$. (Here $\varphi^{k}$ denotes $k$ applications of the Euler phi function.) +5. In the coordinate plane are finitely many walls, which are disjoint line segments, none of which are parallel to either axis. A bulldozer starts at an arbitrary point and moves in the $+x$ direction. Every time it hits a wall, it turns at a right angle to its path, away from the wall, and continues moving. (Thus the bulldozer always moves parallel to the axes.) +Prove that it is impossible for the bulldozer to hit both sides of every wall. +6. Let $A B C$ be a triangle with incenter $I$, and whose incircle is tangent to $\overline{B C}, \overline{C A}$, $\overline{A B}$ at $D, E, F$, respectively. Let $K$ be the foot of the altitude from $D$ to $\overline{E F}$. Suppose that the circumcircle of $\triangle A I B$ meets the incircle at two distinct points $C_{1}$ and $C_{2}$, while the circumcircle of $\triangle A I C$ meets the incircle at two distinct points $B_{1}$ and $B_{2}$. Prove that the radical axis of the circumcircles of $\triangle B B_{1} B_{2}$ and $\triangle C C_{1} C_{2}$ passes through the midpoint $M$ of $\overline{D K}$. + +## §1 Solutions to Day 1 + +## §1.1 TSTST 2016/1, proposed by Victor Wang + +Available online at https://aops.com/community/p6575197. + +## Problem statement + +Let $A=A(x, y)$ and $B=B(x, y)$ be two-variable polynomials with real coefficients. Suppose that $A(x, y) / B(x, y)$ is a polynomial in $x$ for infinitely many values of $y$, and a polynomial in $y$ for infinitely many values of $x$. Prove that $B$ divides $A$, meaning there exists a third polynomial $C$ with real coefficients such that $A=B \cdot C$. + +This is essentially an application of the division algorithm, but the details require significant care. + +First, we claim that $A / B$ can be written as a polynomial in $x$ whose coefficients are rational functions in $y$. To see this, use the division algorithm to get + +$$ +A=Q \cdot B+R \quad Q, R \in(\mathbb{R}(y))[x] +$$ + +where $Q$ and $R$ are polynomials in $x$ whose coefficients are rational functions in $y$, and moreover $\operatorname{deg}_{x} B>\operatorname{deg}_{x} R$. + +Now, we claim that $R \equiv 0$. Indeed, we have by hypothesis that for infinitely many values of $y_{0}$ that $B\left(x, y_{0}\right)$ divides $A\left(x, y_{0}\right)$, which means $B\left(x, y_{0}\right) \mid R\left(x, y_{0}\right)$ as polynomials in $\mathbb{R}[x]$. Now, we have $\operatorname{deg}_{x} B\left(x, y_{0}\right)>\operatorname{deg}_{x} R\left(x, y_{0}\right)$ outside of finitely many values of $y_{0}$ (but not all of them!); this means for infinitely many $y_{0}$ we have $R\left(x, y_{0}\right) \equiv 0$. So each coefficient of $x^{i}$ (in $\left.\mathbb{R}(y)\right)$ has infinitely many roots, hence is a zero polynomial. + +Consequently, we are able to write $A / B=F(x, y) / M(y)$ where $F \in \mathbb{R}[x, y]$ and $M \in \mathbb{R}[y]$ are each polynomials. Repeating the same argument now gives + +$$ +\frac{A}{B}=\frac{F(x, y)}{M(y)}=\frac{G(x, y)}{N(x)} +$$ + +Now, by unique factorization of polynomials in $\mathbb{R}[x, y]$, we can discuss GCD's. So, we tacitly assume $\operatorname{gcd}(F, M)=\operatorname{gcd}(G, N)=(1)$. Also, we obviously have $\operatorname{gcd}(M, N)=(1)$. But $F \cdot N=G \cdot M$, so $M \mid F \cdot N$, thus we conclude $M$ is the constant polynomial. This implies the result. + +Remark. This fact does not generalize to arbitrary functions that are separately polynomial: see e.g. http://aops.com/community/c6h523650p2978180. + +## §1.2 TSTST 2016/2, proposed by Evan Chen + +Available online at https://aops.com/community/p6575204. + +## Problem statement + +Let $A B C$ be a scalene triangle with orthocenter $H$ and circumcenter $O$ and denote by $M, N$ the midpoints of $\overline{A H}, \overline{B C}$. Suppose the circle $\gamma$ with diameter $\overline{A H}$ meets the circumcircle of $A B C$ at $G \neq A$, and meets line $\overline{A N}$ at $Q \neq A$. The tangent to $\gamma$ at $G$ meets line $O M$ at $P$. Show that the circumcircles of $\triangle G N Q$ and $\triangle M B C$ intersect on $\overline{P N}$. + +We present two solutions, one using essentially only power of a point, and the other more involved. + +【 First solution (found by contestants). Denote by $\triangle D E F$ the orthic triangle. Observe $\overline{P A}$ and $\overline{P G}$ are tangents to $\gamma$, since $\overline{O M}$ is the perpendicular bisector of $\overline{A G}$. Also note that $\overline{A G}, \overline{E F}, \overline{B C}$ are concurrent at some point $R$ by radical axis on $(A B C), \gamma$, (BFEC). + +Now, consider circles (PAGM), (MFDNE), and (MBC). We already saw the point $R$ satisfies + +$$ +R A \cdot R G=R E \cdot R F=R B \cdot R C +$$ + +and hence has equal powers to all three circles; but since the circles at $M$ already, they must actually be coaxial. Assume they meet again at $T \in \overline{R M}$, say. Then $\angle P T M$ and $\angle M T N$ are both right angles, hence $T$ lies on $\overline{P N}$. + +Finally $H$ is the orthocenter of $\triangle A R N$, and thus the circle with diameter $\overline{R N}$ passes through $G, Q, N$. +![](https://cdn.mathpix.com/cropped/2024_11_19_f48de8bcaa95c5ff4531g-04.jpg?height=969&width=1117&top_left_y=1591&top_left_x=475) + +『 Alternate solution (by proposer). Let $L$ be diametrically opposite $A$ on the circumcircle. Denote by $\triangle D E F$ the orthic triangle. Let $X=\overline{A H} \cap \overline{E F}$. Finally, let $T$ be the second intersection of (MFDNE) and (MBC). +![](https://cdn.mathpix.com/cropped/2024_11_19_f48de8bcaa95c5ff4531g-05.jpg?height=838&width=1109&top_left_y=429&top_left_x=482) + +We begin with a few easy observations. First, points $H, G, N, L$ are collinear and $\angle A G L=90^{\circ}$. Also, $Q$ is the foot from $H$ to $\overline{A N}$. Consequently, lines $A G, E F, H Q$, $B C, T M$ concur at a point $R$ (radical axis). Moreover, we already know $\angle M T N=90^{\circ}$. This implies $T$ lies on the circle with diameter $\overline{R N}$, which is exactly the circumcircle of $\triangle G Q N$. + +Note by Brokard's Theorem on $A F H E$, the point $X$ is the orthocenter of $\triangle M B C$. But $\angle M T N=90^{\circ}$ already, and $N$ is the midpoint of $\overline{B C}$. Consequently, points $T, X$, $N$ are collinear. + +Finally, we claim $P, X, N$ are collinear, which solves the problem. Note $P=\overline{G G} \cap \overline{A A}$. Set $K=\overline{H N L} \cap \overline{A P}$. Then by noting + +$$ +-1=(D, X ; A, H) \stackrel{N}{=}(\infty, \overline{N X} \cap \overline{A K} ; A, K) +$$ + +we see that $\overline{N X}$ bisects segment $\overline{A K}$, as desired. (A more projective finish is to show that $\overline{P X N}$ is the polar of $R$ to $\gamma$ ). + +Remark. The original problem proposal reads as follows: +Let $A B C$ be a triangle with orthocenter $H$ and circumcenter $O$ and denote by $M, N$ the midpoints of $\overline{A H}, \overline{B C}$. Suppose ray $O M$ meets the line parallel to $\overline{B C}$ through $A$ at $P$. Prove that the line through the circumcenter of $\triangle M B C$ and the midpoint of $\overline{O H}$ is parallel to $\overline{N P}$. +The points $G$ and $Q$ were added to the picture later to prevent the problem from being immediate by coordinates. + +## §1.3 TSTST 2016/3, proposed by Yang Liu + +Available online at https://aops.com/community/p6575217. + +## Problem statement + +Decide whether or not there exists a nonconstant polynomial $Q(x)$ with integer coefficients with the following property: for every positive integer $n>2$, the numbers + +$$ +Q(0), Q(1), Q(2), \ldots, Q(n-1) +$$ + +produce at most $0.499 n$ distinct residues when taken modulo $n$. + +We claim that + +$$ +Q(x)=420\left(x^{2}-1\right)^{2} +$$ + +works. Clearly, it suffices to prove the result when $n=4$ and when $n$ is an odd prime $p$. The case $n=4$ is trivial, so assume now $n=p$ is an odd prime. + +First, we prove the following easy claim. +Claim - For any odd prime $p$, there are at least $\frac{1}{2}(p-3)$ values of $a$ for which $\left(\frac{1-a^{2}}{p}\right)=+1$. + +Proof. Note that if $k \neq 0$ and $k^{2} \neq-1$, then $a=\frac{1-k^{2}}{k^{2}+1}$ works. +Remark. The above identity comes from starting with the equation $1-a^{2}=b^{2}$, and writing it as $\left(\frac{1}{b}\right)^{2}-\left(\frac{a}{b}\right)^{2}=1$. Then solve $\frac{1}{b}-\frac{a}{b}=k$ and $\frac{1}{b}+\frac{a}{b}=1 / k$ for $a$. + +Let $F(x)=\left(x^{2}-1\right)^{2}$. The range of $F$ modulo $p$ is contained within the $\frac{1}{2}(p+1)$ quadratic residues modulo $p$. On the other hand, if for some $t$ neither of $1 \pm t$ is a quadratic residue, then $t^{2}$ is omitted from the range of $F$ as well. Call such a value of $t$ useful, and let $N$ be the number of useful residues. We aim to show $N \geq \frac{1}{4} p-2$. + +We compute a lower bound on the number $N$ of useful $t$ by writing + +$$ +\begin{aligned} +N & =\frac{1}{4}\left(\sum_{t}\left[\left(1-\left(\frac{1-t}{p}\right)\right)\left(1-\left(\frac{1+t}{p}\right)\right)\right]-\left(1-\left(\frac{2}{p}\right)\right)-\left(1-\left(\frac{-2}{p}\right)\right)\right) \\ +& \geq \frac{1}{4} \sum_{t}\left[\left(1-\left(\frac{1-t}{p}\right)\right)\left(1-\left(\frac{1+t}{p}\right)\right)\right]-1 \\ +& =\frac{1}{4}\left(p+\sum_{t}\left(\frac{1-t^{2}}{p}\right)\right)-1 \\ +& \geq \frac{1}{4}\left(p+(+1) \cdot \frac{1}{2}(p-3)+0 \cdot 2+(-1) \cdot\left((p-2)-\frac{1}{2}(p-3)\right)\right)-1 \\ +& \geq \frac{1}{4}(p-5) . +\end{aligned} +$$ + +Thus, the range of $F$ has size at most + +$$ +\frac{1}{2}(p+1)-\frac{1}{2} N \leq \frac{3}{8}(p+3) . +$$ + +This is less than $0.499 p$ for any $p \geq 11$. + +Remark. In fact, the computation above is essentially an equality. There are only two points where terms are dropped: one, when $p \equiv 3(\bmod 4)$ there are no $k^{2}=-1$ in the lemma, and secondly, the terms $1-(2 / p)$ and $1-(-2 / p)$ are dropped in the initial estimate for $N$. With suitable modifications, one can show that in fact, the range of $F$ is exactly equal to + +$$ +\frac{1}{2}(p+1)-\frac{1}{2} N=\left\{\begin{array}{lll} +\frac{1}{8}(3 p+5) & p \equiv 1 & (\bmod 8) \\ +\frac{1}{8}(3 p+7) & p \equiv 3 & (\bmod 8) \\ +\frac{1}{8}(3 p+9) & p \equiv 5 & (\bmod 8) \\ +\frac{1}{8}(3 p+3) & p \equiv 7 & (\bmod 8) +\end{array}\right. +$$ + +## §2 Solutions to Day 2 + +## §2.1 TSTST 2016/4, proposed by Linus Hamilton + +Available online at https://aops.com/community/p6580534. + +## Problem statement + +Prove that if $n$ and $k$ are positive integers satisfying $\varphi^{k}(n)=1$, then $n \leq 3^{k}$. (Here $\varphi^{k}$ denotes $k$ applications of the Euler phi function.) + +The main observation is that the exponent of 2 decreases by at most 1 with each application of $\varphi$. This will give us the desired estimate. + +Define the weight function $w$ on positive integers as follows: it satisfies + +$$ +\begin{aligned} +w(a b) & =w(a)+w(b) \\ +w(2) & =1 ; \quad \text { and } \\ +w(p) & =w(p-1) \quad \text { for any prime } p>2 +\end{aligned} +$$ + +By induction, we see that $w(n)$ counts the powers of 2 that are produced as $\varphi$ is repeatedly applied to $n$. In particular, $k \geq w(n)$. + +From $w(2)=1$, it suffices to prove that $w(p) \geq \log _{3} p$ for every $p>2$. We use strong induction and note that + +$$ +w(p)=w(2)+w\left(\frac{p-1}{2}\right) \geq 1+\log _{3}(p-1)-\log _{3} 2 \geq \log _{3} p +$$ + +for any $p>2$. This solves the problem. +Remark. One can motivate this solution through small cases $2^{x} 3^{y}$ like $2^{x} 17^{w}, 2^{x} 3^{y} 7^{z}$, $2^{x} 11^{t}$ 。 + +Moreover, the stronger bound + +$$ +n \leq 2 \cdot 3^{k-1} +$$ + +is true and best possible. + +## §2.2 TSTST 2016/5, proposed by Linus Hamilton, Cynthia Stoner + +Available online at https://aops.com/community/p6580545. + +## Problem statement + +In the coordinate plane are finitely many walls, which are disjoint line segments, none of which are parallel to either axis. A bulldozer starts at an arbitrary point and moves in the $+x$ direction. Every time it hits a wall, it turns at a right angle to its path, away from the wall, and continues moving. (Thus the bulldozer always moves parallel to the axes.) +Prove that it is impossible for the bulldozer to hit both sides of every wall. + +We say a wall $v$ is above another wall $w$ if some point on $v$ is directly above a point on $w$. (This relation is anti-symmetric, as walls do not intersect). + +The critical claim is as follows: +Claim - There exists a lowest wall, i.e. a wall not above any other walls. +Proof. Assume not. Then we get a directed cycle of some length $n \geq 3$ : it's possible to construct a series of points $P_{i}, Q_{i}$, for $i=1, \ldots, n$ (indices modulo $n$ ), such that the point $Q_{i}$ is directly above $P_{i+1}$ for each $i$, the segment $\overline{Q_{i} P_{i+1}}$ does not intersect any wall in its interior, and finally each segment $\overline{P_{i} Q_{i}}$ is contained inside a wall. This gives us a broken line on $2 n$ vertices which is not self-intersecting. +Now consider the leftmost vertical segment $\overline{Q_{i} P_{i+1}}$ and the rightmost vertical segment $\overline{Q_{j} P_{j+1}}$. The broken line gives a path from $P_{i+1}$ to $Q_{j}$, as well as a path from $P_{j+1}$ to $Q_{i}$. These clearly must intersect, contradiction. + +Remark. This claim is Iran TST 2010. +Thus if the bulldozer eventually moves upwards indefinitely, it may never hit the bottom side of the lowest wall. Similarly, if the bulldozer eventually moves downwards indefinitely, it may never hit the upper side of the highest wall. + +## §2.3 TSTST 2016/6, proposed by Danielle Wang + +Available online at https://aops.com/community/p6580553. + +## Problem statement + +Let $A B C$ be a triangle with incenter $I$, and whose incircle is tangent to $\overline{B C}, \overline{C A}$, $\overline{A B}$ at $D, E, F$, respectively. Let $K$ be the foot of the altitude from $D$ to $\overline{E F}$. Suppose that the circumcircle of $\triangle A I B$ meets the incircle at two distinct points $C_{1}$ and $C_{2}$, while the circumcircle of $\triangle A I C$ meets the incircle at two distinct points $B_{1}$ and $B_{2}$. Prove that the radical axis of the circumcircles of $\triangle B B_{1} B_{2}$ and $\triangle C C_{1} C_{2}$ passes through the midpoint $M$ of $\overline{D K}$. +\I First solution (Allen Liu). Let $X, Y, Z$ be midpoints of $E F, F D, D E$, and let $G$ be the Gergonne point. By radical axis on $(A E I F),(D E F),(A I C)$ we see that $B_{1}$, $X, B_{2}$ are collinear. Likewise, $B_{1}, Z, B_{2}$ are collinear, so lines $B_{1} B_{2}$ and $X Z$ coincide. Similarly, lines $C_{1} C_{2}$ and $X Y$ coincide. In particular lines $B_{1} B_{2}$ and $C_{1} C_{2}$ meet at $X$. +![](https://cdn.mathpix.com/cropped/2024_11_19_f48de8bcaa95c5ff4531g-10.jpg?height=1212&width=1043&top_left_y=1150&top_left_x=512) + +Note $G$ is the symmedian point of $D E F$, so it is well-known that $X G$ passes through the midpoint of $D K$. So we just have to prove $G$ lies on the radical axis. + +First, note that $\triangle D E F$ is the cevian triangle of the Gergonne point $G$. Set $V=$ $\overline{X Y} \cap \overline{A B}, W=\overline{X Z} \cap \overline{A C}$, and $T=\overline{B W} \cap \overline{C V}$. + +We begin with the following completely projective claim. + +Claim - The points $X, G, T$ are collinear. +Proof. It suffices to view $\triangle X Y Z$ as any cevian triangle of $\triangle D E F$ (which is likewise any cevian triangle of $\triangle A B C)$. Then + +- By Cevian Nest on $\triangle A B C$, it follows that $\overline{A X}, \overline{B Y}, \overline{C Z}$ are concurrent. +- Hence $\triangle B Y V$ and $\triangle C Z W$ are perspective. +- Hence $\triangle B Z W$ and $\triangle C Y V$ are perspective too. +- Hence we deduce by Desargues theorem that $T, X$, and $\overline{B Z} \cap \overline{C Y}$ are collinear. +- Finally, the Cevian Nest theorem applied on $\triangle G B C$ (which has cevian triangles $\triangle D F E, \triangle X Z Y$ ) we deduce $G, X$, and $\overline{B Z} \cap \overline{C Y}$, proving the claim. + +One could also proceed by using barycentric coordinates on $\triangle D E F$. + +Remark (Eric Shen). The first four bullets can be replaced by non-projective means: one can check that $\overline{B Z} \cap \overline{C Y}$ is the radical center of $(B I C),\left(B B_{1} B_{2}\right),\left(C C_{1} C_{2}\right)$ and therefore it lies on line $\overline{X T}$. + +Now, we contend point $V$ is the radical center $\left(C C_{1} C_{2}\right),(A B C)$ and $(D E F)$. To see this, let $V^{\prime}=\overline{E D} \cap \overline{A B}$; then $\left(F V^{\prime} ; A B\right)$ is harmonic, and $V$ is the midpoint of $\overline{F V^{\prime}}$, and thus $V A \cdot V B=V F^{2}=V C_{1} \cdot V C_{2}$. + +So in fact $\overline{C V}$ is the radical axis of $(A B C)$ and $\left(C C_{1} C_{2}\right)$. +Similarly, $\overline{B W}$ is the radical axis of $(A B C)$ and $\left(B B_{1} B_{2}\right)$. Thus $T$ is the radical center of $(A B C),\left(B B_{1} B_{2}\right),\left(C C_{1} C_{2}\right)$. + +This completes the proof, as now $\overline{X T}$ is the desired radical axis. +『 Second solution (Evan Chen). As before, we just have to prove $G$ lies on the radical axis. +![](https://cdn.mathpix.com/cropped/2024_11_19_f48de8bcaa95c5ff4531g-12.jpg?height=1392&width=1260&top_left_y=249&top_left_x=404) + +Construct parallelograms $G P F Q, G R D S, G T U E$ such that $P, R \in D F, S, T \in D E$, $Q, U \in E F$. As $F G$ bisects $P Q$ and is isogonal to $F Z$, we find $P Q E D$, hence $P Q R U$, is cyclic. Repeating the same logic and noticing $P R, S T, Q U$ not concurrent, all six points $P Q R S T U$ are cyclic. Moreover, since $P Q$ bisects $G F$, we see that a dilation with factor 2 at $G$ sends $P Q$ to $P^{\prime}, Q^{\prime} \in A B$, say, with $F$ the midpoint of $P^{\prime} Q^{\prime}$. Define $R^{\prime}, S^{\prime} \in B C$ similarly now and $T^{\prime}, U^{\prime} \in C A$. + +Note that $E Q P D S^{\prime}$ is in cyclic too, as $\measuredangle D S^{\prime} Q=\measuredangle D R S=\measuredangle D E F$. By homothety through $B$, points $B, P, X$ are collinear; assume they meet ( $\left.E Q P D S^{\prime}\right)$ again at $V$. Thus $E V Q P D S^{\prime}$ is cyclic, and now + +$$ +\measuredangle B V S^{\prime}=\measuredangle P V S^{\prime}=\measuredangle P Q S=\measuredangle P T S=\measuredangle F E D=\measuredangle X E Z=\measuredangle X V Z +$$ + +hence $V$ lies on $\left(B Q^{\prime} S^{\prime}\right)$. +Since $F B \| Q P$, we get $E V F B$ is cyclic too, so $X V \cdot X B=X E \cdot X F$ now; thus $X$ lies on the radical axis of $\left(B S^{\prime} Q^{\prime}\right)$ and $(D E F)$. By the same argument with $W \in B Z$, we get $Z$ lies on the radical axis too. Thus the radical axis of $\left(B S^{\prime} Q^{\prime}\right)$ and ( $D E F$ ) must be line $X Z$, which coincides with $B_{1} B_{2}$; so $\left(B B_{1} B_{2}\right)=\left(B S^{\prime} Q^{\prime}\right)$. + +Analogously, $\left(C C_{1} C_{2}\right)=\left(C R^{\prime} U^{\prime}\right)$. Since $G=Q^{\prime} S^{\prime} \cap R^{\prime} U^{\prime}$, we need only prove that $Q^{\prime} R^{\prime} S^{\prime} U^{\prime}$ is cyclic. But $Q R S U$ is cyclic, so we are done. + +The circle ( $P Q R S T U$ ) is called the Lemoine circle of $A B C$. + diff --git a/USA_TSTST/md/en-sols-TSTST-2017.md b/USA_TSTST/md/en-sols-TSTST-2017.md new file mode 100644 index 0000000000000000000000000000000000000000..10b5ff3b2c4772f64ff410c3214ecf4ddaccdcce --- /dev/null +++ b/USA_TSTST/md/en-sols-TSTST-2017.md @@ -0,0 +1,571 @@ +# USA TSTST 2017 Solutions
United States of America - TST Selection Test
Evan Chen《陳誼廷》
$59^{\text {th }}$ IMO 2018 Romania and $7^{\text {th }}$ EGMO 2018 Italy + +## Contents + +0 Problems ..... 2 +1 Solutions to Day 1 ..... 3 +1.1 TSTST 2017/1, proposed by Ray Li ..... 3 +1.2 TSTST 2017/2, proposed by Kevin Sun ..... 5 +1.3 TSTST 2017/3, proposed by Calvin Deng, Linus Hamilton ..... 7 +2 Solutions to Day 2 ..... 9 +2.1 TSTST 2017/4, proposed by Mark Sellke ..... 9 +2.2 TSTST 2017/5, proposed by Ray Li ..... 10 +2.3 TSTST 2017/6, proposed by Ivan Borsenco ..... 12 + +## §0 Problems + +1. Let $A B C$ be a triangle with circumcircle $\Gamma$, circumcenter $O$, and orthocenter $H$. Assume that $A B \neq A C$ and $\angle A \neq 90^{\circ}$. Let $M$ and $N$ be the midpoints of $\overline{A B}$ and $\overline{A C}$, respectively, and let $E$ and $F$ be the feet of the altitudes from $B$ and $C$ in $\triangle A B C$, respectively. Let $P$ be the intersection point of line $M N$ with the tangent line to $\Gamma$ at $A$. Let $Q$ be the intersection point, other than $A$, of $\Gamma$ with the circumcircle of $\triangle A E F$. Let $R$ be the intersection point of lines $A Q$ and $E F$. Prove that $\overline{P R} \perp \overline{O H}$. +2. Ana and Banana are playing a game. First Ana picks a word, which is defined to be a nonempty sequence of capital English letters. Then Banana picks a nonnegative integer $k$ and challenges Ana to supply a word with exactly $k$ subsequences which are equal to Ana's word. Ana wins if she is able to supply such a word, otherwise she loses. For example, if Ana picks the word "TST", and Banana chooses $k=4$, then Ana can supply the word "TSTST" which has 4 subsequences which are equal to Ana's word. Which words can Ana pick so that she can win no matter what value of $k$ Banana chooses? +3. Consider solutions to the equation + +$$ +x^{2}-c x+1=\frac{f(x)}{g(x)} +$$ + +where $f$ and $g$ are nonzero polynomials with nonnegative real coefficients. For each $c>0$, determine the minimum possible degree of $f$, or show that no such $f, g$ exist. +4. Find all nonnegative integer solutions to + +$$ +2^{a}+3^{b}+5^{c}=n! +$$ + +5. Let $A B C$ be a triangle with incenter $I$. Let $D$ be a point on side $B C$ and let $\omega_{B}$ and $\omega_{C}$ be the incircles of $\triangle A B D$ and $\triangle A C D$, respectively. Suppose that $\omega_{B}$ and $\omega_{C}$ are tangent to segment $B C$ at points $E$ and $F$, respectively. Let $P$ be the intersection of segment $A D$ with the line joining the centers of $\omega_{B}$ and $\omega_{C}$. Let $X$ be the intersection point of lines $B I$ and $C P$ and let $Y$ be the intersection point of lines $C I$ and $B P$. Prove that lines $E X$ and $F Y$ meet on the incircle of $\triangle A B C$. +6. A sequence of positive integers $\left(a_{n}\right)_{n \geq 1}$ is of Fibonacci type if it satisfies the recursive relation $a_{n+2}=a_{n+1}+a_{n}$ for all $n \geq 1$. Is it possible to partition the set of positive integers into an infinite number of Fibonacci type sequences? + +## §1 Solutions to Day 1 + +## §1.1 TSTST 2017/1, proposed by Ray Li + +Available online at https://aops.com/community/p8526098. + +## Problem statement + +Let $A B C$ be a triangle with circumcircle $\Gamma$, circumcenter $O$, and orthocenter $H$. Assume that $A B \neq A C$ and $\angle A \neq 90^{\circ}$. Let $M$ and $N$ be the midpoints of $\overline{A B}$ and $\overline{A C}$, respectively, and let $E$ and $F$ be the feet of the altitudes from $B$ and $C$ in $\triangle A B C$, respectively. Let $P$ be the intersection point of line $M N$ with the tangent line to $\Gamma$ at $A$. Let $Q$ be the intersection point, other than $A$, of $\Gamma$ with the circumcircle of $\triangle A E F$. Let $R$ be the intersection point of lines $A Q$ and $E F$. Prove that $\overline{P R} \perp \overline{O H}$. + +【 First solution (power of a point). Let $\gamma$ denote the nine-point circle of $A B C$. +![](https://cdn.mathpix.com/cropped/2024_11_19_b5f71e11be2d66f7b920g-03.jpg?height=609&width=795&top_left_y=1140&top_left_x=639) + +Note that + +- $P A^{2}=P M \cdot P N$, so $P$ lies on the radical axis of $\Gamma$ and $\gamma$. +- $R A \cdot R Q=R E \cdot R F$, so $R$ lies on the radical axis of $\Gamma$ and $\gamma$. + +Thus $\overline{P R}$ is the radical axis of $\Gamma$ and $\gamma$, which is evidently perpendicular to $\overline{O H}$. +Remark. In fact, by power of a point one may also observe that $R$ lies on $\overline{B C}$, since it is on the radical axis of $(A Q F H E),(B F E C),(A B C)$. Ironically, this fact is not used in the solution. + +II Second solution (barycentric coordinates). Again note first $R \in \overline{B C}$ (although this can be avoided too). We compute the points in much the same way as before. Since $\overline{A P} \cap \overline{B C}=\left(0: b^{2}:-c^{2}\right)$ we have + +$$ +P=\left(b^{2}-c^{2}: b^{2}:-c^{2}\right) +$$ + +(since $x=y+z$ is the equation of line $\overline{M N}$ ). Now in Conway notation we have + +$$ +R=\overline{E F} \cap \overline{B C}=\left(0: S_{C}:-S_{B}\right)=\left(0: a^{2}+b^{2}-c^{2}:-a^{2}+b^{2}-c^{2}\right) . +$$ + +Hence + +$$ +\overrightarrow{P R}=\frac{1}{2\left(b^{2}-c^{2}\right)}\left(b^{2}-c^{2}, c^{2}-a^{2}, a^{2}-b^{2}\right) +$$ + +On the other hand, we have $\overrightarrow{O H}=\vec{A}+\vec{B}+\vec{C}$. So it suffices to check that + +$$ +\sum_{\mathrm{cyc}} a^{2}\left(\left(a^{2}-b^{2}\right)+\left(c^{2}-a^{2}\right)\right)=0 +$$ + +which is immediate. + +『 Third solution (complex numbers). Let $A B C$ be the unit circle. We first compute $P$ as the midpoint of $A$ and $\overline{A A} \cap \overline{B C}$ : + +$$ +\begin{aligned} +p & =\frac{1}{2}\left(a+\frac{a^{2}(b+c)-b c \cdot 2 a}{a^{2}-b c}\right) \\ +& =\frac{a\left(a^{2}-b c\right)+a^{2}(b+c)-2 a b c}{2\left(a^{2}-b c\right)} +\end{aligned} +$$ + +Using the remark above, $R$ is the inverse of $D$ with respect to the circle with diameter $\overline{B C}$, which has radius $\left|\frac{1}{2}(b-c)\right|$. Thus + +$$ +\begin{aligned} +r-\frac{b+c}{2} & =\frac{\frac{1}{4}(b-c)\left(\frac{1}{b}-\frac{1}{c}\right)}{\frac{1}{2}\left(a-\frac{b c}{a}\right)} \\ +r & =\frac{b+c}{2}+\frac{-\frac{1}{2} \frac{(b-c)^{2}}{b c}}{\frac{1}{a}-\frac{a}{b c}} \\ +& =\frac{b+c}{2}+\frac{a(b-c)^{2}}{2\left(a^{2}-b c\right)} \\ +& =\frac{a(b-c)^{2}+(b+c)\left(a^{2}-b c\right)}{2\left(a^{2}-b c\right)} +\end{aligned} +$$ + +Expanding and subtracting gives + +$$ +p-r=\frac{a^{3}-a b c-a b^{2}-a c^{2}+b^{2} c+b c^{2}}{2\left(a^{2}-b c\right)}=\frac{(a+b+c)(a-b)(a-c)}{2\left(a^{2}-b c\right)} +$$ + +which is visibly equal to the negation of its conjugate once the factor of $a+b+c$ is deleted. +(Actually, one can guess this factorization ahead of time by noting that if $A=B$, then $P=B=R$, so $a-b$ must be a factor; analogously $a-c$ must be as well.) + +## §1.2 TSTST 2017/2, proposed by Kevin Sun + +Available online at https://aops.com/community/p8526115. + +## Problem statement + +Ana and Banana are playing a game. First Ana picks a word, which is defined to be a nonempty sequence of capital English letters. Then Banana picks a nonnegative integer $k$ and challenges Ana to supply a word with exactly $k$ subsequences which are equal to Ana's word. Ana wins if she is able to supply such a word, otherwise she loses. For example, if Ana picks the word "TST", and Banana chooses $k=4$, then Ana can supply the word "TSTST" which has 4 subsequences which are equal to Ana's word. Which words can Ana pick so that she can win no matter what value of $k$ Banana chooses? + +First we introduce some notation. Define a block of letters to be a maximal contiguous subsequence of consecutive letters. For example, the word $A A B B B C A A A$ has four blocks, namely $A A, B B B, C, A A A$. Throughout the solution, we fix the word $A$ that Ana picks, and introduce the following notation for its $m$ blocks: + +$$ +A=A_{1} A_{2} \ldots A_{m}=\underbrace{a_{1} \ldots a_{1}}_{x_{1}} \underbrace{a_{2} \ldots a_{2}}_{x_{2}} \cdots \underbrace{a_{m} \ldots a_{m}}_{x_{m}} +$$ + +A rainbow will be a subsequence equal to Ana's initial word $A$ (meaning Ana seeks words with exactly $k$ rainbows). Finally, for brevity, let $A_{i}=\underbrace{a_{i} \ldots a_{i}}_{x_{i}}$, so $A=A_{1} \ldots A_{m}$. + +We prove two claims that resolve the problem. +Claim - If $x_{i}=1$ for some $i$, then for any $k \geq 1$, the word + +$$ +W=A_{1} \ldots A_{i-1} \underbrace{a_{i} \ldots a_{i}}_{k} A_{i+1} \ldots A_{m} +$$ + +obtained by repeating the $i$ th letter $k$ times has exactly $k$ rainbows. +Proof. Obviously there are at least $\binom{k}{k-1}=k$ rainbows, obtained by deleting $k-1$ choices of the letter $a_{i}$ in the repeated block. We show they are the only ones. + +Given a rainbow, consider the location of this singleton block in $W$. It cannot occur within the first $\left|A_{1}\right|+\cdots+\left|A_{i-1}\right|$ letters, nor can it occur within the final $\left|A_{i+1}\right|+\cdots+\left|A_{m}\right|$ letters. So it must appear in the $i$ th block of $W$. That implies that all the other $a_{i}$ 's in the $i$ th block of $W$ must be deleted, as desired. (This last argument is actually nontrivial, and has some substance; many students failed to realize that the upper bound requires care.) + +Claim - If $x_{i} \geq 2$ for all $i$, then no word $W$ has exactly two rainbows. +Proof. We prove if there are two rainbows of $W$, then we can construct at least three rainbows. + +Let $W=w_{1} \ldots w_{n}$ and consider the two rainbows of $W$. Since they are not the same, there must be a block $A_{p}$ of the rainbow, of length $\ell \geq 2$, which do not occupy the same locations in $W$. + +Assume the first rainbow uses $w_{i_{1}}, \ldots, w_{i_{\ell}}$ for this block and the second rainbow uses $w_{j_{1}}, \ldots, w_{j_{\ell}}$ for this block. Then among the letters $w_{q}$ for $\min \left(i_{1}, j_{1}\right) \leq q \leq \max \left(i_{\ell}, j_{\ell}\right)$, there must be at least $\ell+1$ copies of the letter $a_{p}$. Moreover, given a choice of $\ell$ copies of the letter $a_{p}$ in this range, one can complete the subsequence to a rainbow. So the number of rainbows is at least $\binom{\ell+1}{\ell} \geq \ell+1$. + +Since $\ell \geq 2$, this proves $W$ has at least three rainbows. +In summary, Ana wins if and only if $x_{i}=1$ for some $i$, since she can duplicate the isolated letter $k$ times; but if $x_{i} \geq 2$ for all $i$ then Banana only needs to supply $k=2$. + +## §1.3 TSTST 2017/3, proposed by Calvin Deng, Linus Hamilton + +Available online at https://aops.com/community/p8526130. + +## Problem statement + +Consider solutions to the equation + +$$ +x^{2}-c x+1=\frac{f(x)}{g(x)} +$$ + +where $f$ and $g$ are nonzero polynomials with nonnegative real coefficients. For each $c>0$, determine the minimum possible degree of $f$, or show that no such $f, g$ exist. + +First, if $c \geq 2$ then we claim no such $f$ and $g$ exist. Indeed, one simply takes $x=1$ to get $f(1) / g(1) \leq 0$, impossible. + +For $c<2$, let $c=2 \cos \theta$, where $0<\theta<\pi$. We claim that $f$ exists and has minimum degree equal to $n$, where $n$ is defined as the smallest integer satisfying $\sin n \theta \leq 0$. In other words + +$$ +n=\left\lceil\frac{\pi}{\arccos (c / 2)}\right\rceil +$$ + +First we show that this is necessary. To see it, write explicitly + +$$ +g(x)=a_{0}+a_{1} x+a_{2} x^{2}+\cdots+a_{n-2} x^{n-2} +$$ + +with each $a_{i} \geq 0$, and $a_{n-2} \neq 0$. Assume that $n$ is such that $\sin (k \theta) \geq 0$ for $k=1, \ldots, n-1$. Then, we have the following system of inequalities: + +$$ +\begin{aligned} +& a_{1} \geq 2 \cos \theta \cdot a_{0} \\ +& a_{0}+a_{2} \geq 2 \cos \theta \cdot a_{1} \\ +& a_{1}+a_{3} \geq 2 \cos \theta \cdot a_{2} \\ +& \vdots \\ +& a_{n-5}+a_{n-3} \geq 2 \cos \theta \cdot a_{n-4} \\ +& a_{n-4}+a_{n-2} \geq 2 \cos \theta \cdot a_{n-3} \\ +& a_{n-3} \geq 2 \cos \theta \cdot a_{n-2} . +\end{aligned} +$$ + +Now, multiply the first equation by $\sin \theta$, the second equation by $\sin 2 \theta$, et cetera, up to $\sin ((n-1) \theta)$. This choice of weights is selected since we have + +$$ +\sin (k \theta)+\sin ((k+2) \theta)=2 \sin ((k+1) \theta) \cos \theta +$$ + +so that summing the entire expression cancels nearly all terms and leaves only + +$$ +\sin ((n-2) \theta) a_{n-2} \geq \sin ((n-1) \theta) \cdot 2 \cos \theta \cdot a_{n-2} +$$ + +and so by dividing by $a_{n-2}$ and using the same identity gives us $\sin (n \theta) \leq 0$, as claimed. +This bound is best possible, because the example + +$$ +a_{k}=\sin ((k+1) \theta) \geq 0 +$$ + +makes all inequalities above sharp, hence giving a working pair $(f, g)$. + +Remark. Calvin Deng points out that a cleaner proof of the lower bound is to take $\alpha=\cos \theta+i \sin \theta$. Then $f(\alpha)=0$, but by condition the imaginary part of $f(\alpha)$ is apparently strictly positive, contradiction. + +Remark. Guessing that $c<2$ works at all (and realizing $c \geq 2$ fails) is the first part of the problem. + +The introduction of trigonometry into the solution may seem magical, but is motivated in one of two ways: + +- Calvin Deng points out that it's possible to guess the answer from small cases: For $c \leq 1$ we have $n=3$, tight at $\frac{x^{3}+1}{x+1}=x^{2}-x+1$, and essentially the "sharpest $n=3$ example". A similar example exists at $n=4$ with $\frac{x^{4}+1}{x^{2}+\sqrt{2} x+1}=x^{2}-\sqrt{2} x+1$ by the Sophie-Germain identity. In general, one can do long division to extract an optimal value of $c$ for any given $n$, although $c$ will be the root of some polynomial. +The thresholds $c \leq 1$ for $n=3, c \leq \sqrt{2}$ for $n=4, c \leq \frac{1+\sqrt{5}}{2}$ for $n=5$, and $c \leq 2$ for $n<\infty$ suggest the unusual form of the answer via trigonometry. +- One may imagine trying to construct a polynomial recursively / greedily by making all inequalities above hold (again the "sharpest situation" in which $f$ has few coefficients). If one sets $c=2 t$, then we have + +$$ +a_{0}=1, \quad a_{1}=2 t, \quad a_{2}=4 t^{2}-1, \quad a_{3}=8 t^{3}-4 t, \quad \ldots +$$ + +which are the Chebyshev polynomials of the second type. This means that trigonometry is essentially mandatory. (One may also run into this when by using standard linear recursion techniques, and noting that the characteristic polynomial has two conjugate complex roots.) + +Remark. Mitchell Lee notes that an IMO longlist problem from 1997 shows that if $P(x)$ is any polynomial satisfying $P(x)>0$ for $x>0$, then $(x+1)^{n} P(x)$ has nonnegative coefficients for large enough $n$. This show that $f$ and $g$ at least exist for $c \leq 2$, but provides no way of finding the best possible $\operatorname{deg} f$. + +Meghal Gupta also points out that showing $f$ and $g$ exist is possible in the following way: + +$$ +\left(x^{2}-1.99 x+1\right)\left(x^{2}+1.99 x+1\right)=\left(x^{4}-1.9601 x^{2}+1\right) +$$ + +and so on, repeatedly multiplying by the "conjugate" until all coefficients become positive. To my best knowledge, this also does not give any way of actually minimizing $\operatorname{deg} f$, although Ankan Bhattacharya points out that this construction is actually optimal in the case where $n$ is a power of 2 . + +Remark. It's pointed out that Matematicheskoe Prosveshchenie, issue 1, 1997, page 194 contains a nearly analogous result, available at https://mccme.ru/free-books/matpros/ pdf/mp-01.pdf with solutions presented in https://mccme.ru/free-books/matpros/pdf/ mp-05.pdf, pages 221-223; and https://mccme.ru/free-books/matpros/pdf/mp-10.pdf, page 274. + +## §2 Solutions to Day 2 + +## §2.1 TSTST 2017/4, proposed by Mark Sellke + +Available online at https://aops.com/community/p8526131. + +## Problem statement + +Find all nonnegative integer solutions to + +$$ +2^{a}+3^{b}+5^{c}=n! +$$ + +For $n \leq 4$, one can check the only solutions are: + +$$ +\begin{aligned} +& 2^{2}+3^{0}+5^{0}=3! \\ +& 2^{1}+3^{1}+5^{0}=3! \\ +& 2^{4}+3^{1}+5^{1}=4! +\end{aligned} +$$ + +Now we prove there are no solutions for $n \geq 5$. +A tricky way to do this is to take modulo 120 , since + +$$ +\begin{aligned} +2^{a} \quad(\bmod 120) & \in\{1,2,4,8,16,32,64\} \\ +3^{b} \quad(\bmod 120) & \in\{1,3,9,27,81\} \\ +5^{c} \quad(\bmod 120) & \in\{1,5,25\} +\end{aligned} +$$ + +and by inspection one notes that no three elements have vanishing sum modulo 120 . +I expect most solutions to instead use casework. Here is one possible approach with cases (with $n \geq 5$ ). First, we analyze the cases where $a<3$ : + +- $a=0$ : No solutions for parity reasons. +- $a=1$ : since $3^{b}+5^{c} \equiv 6(\bmod 8)$, we find $b$ even and $c$ odd (hence $\left.c \neq 0\right)$. Now looking modulo 5 gives that $3^{b}+5^{c} \equiv 3(\bmod 5)$, +- $a=2$ : From $3^{b}+5^{c} \equiv 4(\bmod 8)$, we find $b$ is odd and $c$ is even. Now looking modulo 5 gives a contradiction, even if $c=0$, since $3^{b} \in\{2,3(\bmod 5)\}$ but $3^{b}+5^{c} \equiv 1(\bmod 5)$. + +Henceforth assume $a \geq 3$. Next, by taking modulo 8 we have $3^{b}+5^{c} \equiv 0(\bmod 8)$, which forces both $b$ and $c$ to be odd (in particular, $b, c>0$ ). We now have + +$$ +\begin{aligned} +& 2^{a}+5^{c} \equiv 0 \quad(\bmod 3) \\ +& 2^{a}+3^{b} \equiv 0 \quad(\bmod 5) . +\end{aligned} +$$ + +The first equation implies $a$ is even, but the second equation requires $a$ to be odd, contradiction. Hence no solutions with $n \geq 5$. + +## §2.2 TSTST 2017/5, proposed by Ray Li + +Available online at https://aops.com/community/p8526136. + +## Problem statement + +Let $A B C$ be a triangle with incenter $I$. Let $D$ be a point on side $B C$ and let $\omega_{B}$ and $\omega_{C}$ be the incircles of $\triangle A B D$ and $\triangle A C D$, respectively. Suppose that $\omega_{B}$ and $\omega_{C}$ are tangent to segment $B C$ at points $E$ and $F$, respectively. Let $P$ be the intersection of segment $A D$ with the line joining the centers of $\omega_{B}$ and $\omega_{C}$. Let $X$ be the intersection point of lines $B I$ and $C P$ and let $Y$ be the intersection point of lines $C I$ and $B P$. Prove that lines $E X$ and $F Y$ meet on the incircle of $\triangle A B C$. + +『 First solution (homothety). Let $Z$ be the diametrically opposite point on the incircle. We claim this is the desired intersection. +![](https://cdn.mathpix.com/cropped/2024_11_19_b5f71e11be2d66f7b920g-10.jpg?height=586&width=809&top_left_y=1055&top_left_x=629) + +Note that: + +- $P$ is the insimilicenter of $\omega_{B}$ and $\omega_{C}$ +- $C$ is the exsimilicenter of $\omega$ and $\omega_{C}$. + +Thus by Monge theorem, the insimilicenter of $\omega_{B}$ and $\omega$ lies on line $C P$. +This insimilicenter should also lie on the line joining the centers of $\omega$ and $\omega_{B}$, which is $\overline{B I}$, hence it coincides with the point $X$. So $X \in \overline{E Z}$ as desired. + +【 Second solution (harmonic). Let $T=\overline{I_{B} I_{C}} \cap \overline{B C}$, and $W$ the foot from $I$ to $\overline{B C}$. Define $Z=\overline{F Y} \cap \overline{I W}$. Because $\angle I_{B} D I_{C}=90^{\circ}$, we have + +$$ +-1=\left(I_{B} I_{C} ; P T\right) \stackrel{B}{\stackrel{B}{2}}\left(I I_{C} ; Y C\right) \stackrel{F}{=}(I \infty ; Z W) +$$ + +So $I$ is the midpoint of $\overline{Z W}$ as desired. +【 Third solution (outline, barycentric, Andrew Gu). Let $A D=t, B D=x, C D=y$, so $a=x+y$ and by Stewart's theorem we have + +$$ +(x+y)\left(x y+t^{2}\right)=b^{2} x+c^{2} y +$$ + +We then have $D=(0: y: x)$ and so + +$$ +\overline{A I_{B}} \cap \overline{B C}=\left(0: y+\frac{t x}{c+t}: \frac{c x}{c+t}\right) +$$ + +hence intersection with $B I$ gives + +$$ +I_{B}=(a x: c y+a t: c x) +$$ + +Similarly, + +$$ +I_{C}=(a y: b y: b x+a t) +$$ + +Then, we can compute + +$$ +P=(2 a x y: y(a t+b x+c y): x(a t+b x+c y)) +$$ + +since $P \in \overline{I_{B} I_{C}}$, and clearly $P \in \overline{A D}$. Intersection now gives + +$$ +\begin{aligned} +& X=(2 a x: a t+b x+c y: 2 c x) \\ +& Y=(2 a y: 2 b y: a t+b x+c y) +\end{aligned} +$$ + +Finally, we have $B E=\frac{1}{2}(c+x-t)$, and similarly for $C F$. Now if we reflect $D=$ $\left(0, \frac{s-c}{a}, \frac{s-b}{a}\right)$ over $I=\left(\frac{a}{2 s}, \frac{b}{2 s}, \frac{c}{2 s}\right)$, we get the antipode + +$$ +Q:=\left(4 a^{2}:-a^{2}+2 a b-b^{2}+c^{2}:-a^{2}+2 a c-c^{2}+b^{2}\right) . +$$ + +We may then check $Q$ lies on each of lines $E X$ and $F Y$ (by checking $\operatorname{det}(Q, E, X)=0$ using the equation (1)). + +## §2.3 TSTST 2017/6, proposed by Ivan Borsenco + +Available online at https://aops.com/community/p8526142. + +## Problem statement + +A sequence of positive integers $\left(a_{n}\right)_{n \geq 1}$ is of Fibonacci type if it satisfies the recursive relation $a_{n+2}=a_{n+1}+a_{n}$ for all $n \geq 1$. Is it possible to partition the set of positive integers into an infinite number of Fibonacci type sequences? + +Yes, it is possible. The following solutions were written for me by Kevin Sun and Mark Sellke. We let $F_{1}=F_{2}=1, F_{3}=2, F_{4}=3, F_{5}=5, \ldots$ denote the Fibonacci numbers. + +【 First solution (Kevin Sun). We are going to appeal to the so-called Zeckendorf theorem: + +Theorem (Zeckendorf) +Every positive integer can be uniquely expressed as the sum of nonconsecutive Fibonacci numbers. + +This means every positive integer has a Zeckendorf ("Fibonacci-binary") representation where we put 1 in the $i$ th digit from the right if $F_{i+1}$ is used. The idea is then to take the following so-called Wythoff array: + +- Row 1: 1, 2, 3, 5, ... +- Row 101: $1+3,2+5,3+8, \ldots$ +- Row 1001: $1+5,2+8,3+13, \ldots$ +- Row 10001: $1+8,2+13,3+21, \ldots$ +- Row 10101: $1+3+8,2+5+13,3+8+21, \ldots$ +- . . .et cetera. + +More concretely, the array has the following rows to start: + +| 1 | 2 | 3 | 5 | 8 | 13 | 21 | $\ldots$ | +| :---: | :---: | :---: | :---: | :---: | :---: | :---: | :---: | +| 4 | 7 | 11 | 18 | 29 | 47 | 76 | $\ldots$ | +| 6 | 10 | 16 | 26 | 42 | 68 | 110 | $\ldots$ | +| 9 | 15 | 24 | 39 | 63 | 102 | 165 | $\ldots$ | +| 12 | 20 | 32 | 52 | 84 | 136 | 220 | $\ldots$ | +| 14 | 23 | 37 | 60 | 97 | 157 | 254 | $\ldots$ | +| 17 | 28 | 45 | 73 | 118 | 191 | 309 | $\ldots$ | +| $\vdots$ | $\vdots$ | $\vdots$ | $\vdots$ | $\vdots$ | $\vdots$ | $\vdots$ | $\ddots$ | + +Here are the full details. +We begin by outlining a proof of Zeckendorf's theorem, which implies the representation above is unique. Note that if $F_{k}$ is the greatest Fibonacci number at most $n$, then + +$$ +n-F_{k}F_{k-1}+F_{k-3}+F_{k-5}+\cdots +$$ + +This shows, by a simple inductive argument, that such a representation exists and unique. +We write $n={\bar{a} k \ldots a_{1}}_{\text {Fib }}$ for the Zeckendorf representation as we described (where $a_{i}=1$ if $F_{i+1}$ is used). Now for each ${\bar{a} k \ldots a_{1}}^{\text {Fib }}$ with $a_{1}=1$, consider the sequence + +$$ +{\overline{a_{k} \ldots a_{1}}}_{\mathrm{Fib}},{\overline{a_{k} \ldots a_{1} 0}}_{\mathrm{Fib}},{\overline{a_{k} \ldots a_{1} 00}}_{\mathrm{Fib}}, \ldots +$$ + +These sequences are Fibonacci-type by definition, and partition the positive integers since each positive integer has exactly one Fibonacci base representation. +\I Second solution. Call an infinite set of integers $S$ sandwiched if there exist increasing sequences $\left\{a_{i}\right\}_{i=0}^{\infty},\left\{b_{i}\right\}_{i=0}^{\infty}$ such that the following are true: + +- $a_{i}+a_{i+1}=a_{i+2}$ and $b_{i}+b_{i+1}=b_{i+2}$. +- The intervals $\left[a_{i}+1, b_{i}-1\right]$ are disjoint and are nondecreasing in length. +- $S=\bigcup_{i=0}^{\infty}\left[a_{i}+1, b_{i}-1\right]$. + +We claim that if $S$ is any nonempty sandwiched set, then $S$ can be partitioned into a Fibonacci-type sequence (involving the smallest element of $S$ ) and two smaller sandwiched sets. If this claim is proven, then we can start with $\mathbb{N} \backslash\{1,2,3,5, \ldots\}$, which is a sandwiched set, and repeatedly perform this partition, which will eventually sort each natural number into a Fibonacci-type sequence. + +Let $S$ be a sandwiched set given by $\left\{a_{i}\right\}_{i=0}^{\infty},\left\{b_{i}\right\}_{i=0}^{\infty}$, so the smallest element in $S$ is $x=a_{0}+1$. Note that $y=a_{1}+1$ is also in $S$ and $x1$. Then + +$$ +a \neq b \Longrightarrow\left\lfloor\varphi a+\frac{1}{2}\right\rfloor \neq\left\lfloor\varphi b+\frac{1}{2}\right\rfloor, +$$ + +and since the first term of each sequence is chosen to not overlap with any previous sequences, these sequences are disjoint. + +Remark. Ankan Bhattacharya points out that the same sequence essentially appears in IMO 1993, Problem 5 - in other words, a strictly increasing function $f: \mathbb{Z}_{>0} \rightarrow \mathbb{Z}_{>0}$ with $f(1)=2$, and $f(f(n))=f(n)+n$. + +Nikolai Beluhov sent us an older reference from March 1977, where Martin Gardner wrote in his column about Wythoff's Nim. The relevant excerpt goes: +"Imagine that we go through the infinite sequence of safe pairs (in the manner of Eratosthenes' sieve for sifting out primes) and cross out the infinite set of all safe pairs that are pairs in the Fibonacci sequence. The smallest pair that is not crossed out is $4 / 7$. We can now cross out a second infinite set of safe pairs, starting with $4 / 7$, that are pairs in the Lucas sequence. An infinite number of safe pairs, of which the lowest is now $6 / 10$, remain. This pair too begins another infinite Fibonacci sequence, all of whose pairs are safe. The process continues forever. Robert Silber, a mathematician at North Carolina State University, calls a safe pair "primitive" if it is the first safe pair that generates a Fibonacci sequence." + +The relevant article by Robert Silber is A Fibonacci Property of Wythoff Pairs, from The Fibonacci Quarterly 11/1976. + +I Fourth solution (Mark Sellke). For later reference let + +$$ +f_{1}=0, f_{2}=1, f_{3}=1, \ldots +$$ + +denote the ordinary Fibonacci numbers. We will denote the Fibonacci-like sequences by $F^{i}$ and the elements with subscripts; hence $F_{1}^{2}$ is the first element of the second sequence. Our construction amounts to just iteratively add new sequences; hence the following claim is the whole problem. + +## Lemma + +For any disjoint collection of Fibonacci-like sequences $F^{1}, \ldots, F^{k}$ and any integer $m$ contained in none of them, there is a new Fibonacci-like sequence $F^{k+1}$ beginning with $F_{1}^{k+1}=m$ which is disjoint from the previous sequences. + +Observe first that for each sequence $F^{j}$ there is $c^{j} \in \mathbb{R}^{n}$ such that + +$$ +F_{n}^{j}=c^{j} \phi^{n}+o(1) +$$ + +where + +$$ +\phi=\frac{1+\sqrt{5}}{2} +$$ + +Collapse the group $\left(\mathbb{R}^{+}, \times\right)$into the half-open interval $J=\{x \mid 1 \leq x<\phi\}$ by defining $T(x)=y$ for the unique $y \in J$ with $x=y \phi^{n}$ for some integer $n$. + +Fix an interval $I=[a, b] \subseteq[1.2,1.3]$ (the last condition is to avoid wrap-around issues) which contains none of the $c^{j}$, and take $\varepsilon<0.001$ to be small enough that in fact each $c^{j}$ has distance at least $10 \varepsilon$ from $I$; this means any $c_{j}$ and element of $I$ differ by at least a $(1+10 \varepsilon)$ factor. The idea will be to take $F_{1}^{k+1}=m$ and $F_{2}^{k+1}$ to be a large such that the induced values of $F_{j}^{k+1}$ grow like $k \phi^{j}$ for $j \in T^{-1}(I)$, so that $F_{n}^{k+1}$ is separated from the $c^{j}$ after applying $T$. What's left to check is the convergence. + +Now let + +$$ +c=\lim _{n \rightarrow \infty} \frac{f_{n}}{\phi^{n}} +$$ + +and take $M$ large enough that for $n>M$ we have + +$$ +\left|\frac{f_{n}}{c \phi^{n}}-1\right|<\varepsilon +$$ + +Now $\frac{T^{-1}(I)}{c}$ contains arbitrarily large integers, so there are infinitely many $N$ with $c N \in T^{-1}(I)$ with $N>\frac{10 m}{\varepsilon}$. We claim that for any such $N$, the sequence $F^{(N)}$ defined by + +$$ +F_{1}^{(N)}=m, F_{2}^{(N)}=N +$$ + +will be very multiplicatively similar to the normal Fibonacci numbers up to rescaling; indeed for $j=2, j=3$ we have $\frac{F_{2}^{(N)}}{f_{2}}=N, \frac{F_{3}^{(N)}}{f_{3}}=N+m$ and so by induction we will have + +$$ +\frac{F_{j}^{(N)}}{f_{j}} \in[N, N+m] \subseteq[N, N(1+\varepsilon)] +$$ + +for $j \geq 2$. Therefore, up to small multiplicative errors, we have + +$$ +F_{j}^{(N)} \approx N f_{j} \approx c N \phi^{j} +$$ + +From this we see that for $j>M$ we have + +$$ +T\left(F_{j}^{(N)}\right) \in T(c N) \cdot[1-2 \varepsilon, 1+2 \varepsilon] +$$ + +In particular, since $T(c N) \in I$ and $I$ is separated from each $c_{j}$ by a factor of $(1+10 \varepsilon)$, we get that $F_{j}^{(N)}$ is not in any of $F^{1}, F^{2}, \ldots, F^{k}$. + +Finishing is easy, since we now have a uniform estimate on how many terms we need to check for a new element before the exponential growth takes over. We will just use pigeonhole to argue that there are few possible collisions among those early terms, so we can easily pick a value of $N$ which avoids them all. We write it out below. + +For large $L$, the set + +$$ +S_{L}=\left(I \cdot \phi^{L}\right) \cap \mathbb{Z} +$$ + +contains at least $k_{I} \phi^{L}$ elements. As $N$ ranges over $S_{L}$, for each fixed $j$, the value of $F_{j}^{(N)}$ varies by at most a factor of 1.1 because we imposed $I \subseteq[1.2,1.3]$ and so this is true for the first two terms, hence for all subsequent terms by induction. Now suppose $L$ is very large, and consider a fixed pair $(i, j)$ with $i \leq k$ and $j \leq M$. We claim there is at most 1 possible value $k$ such that the term $F_{k}^{i}$ could equal $F_{j}^{(N)}$ for some $N \in S_{L}$; indeed, the terms of $F^{i}$ are growing at exponential rate with factor $\phi>1.1$, so at most one will be in a given interval of multiplicative width at most 1.1. + +Hence, of these $k_{I} \phi^{L}$ values of $N$, at most $k M$ could cause problems, one for each pair $(i, j)$. However by monotonicity of $F_{j}^{(N)}$ in $N$, at most 1 value of $N$ causes a collision for each pair $(i, j)$. Hence for large $L$ so that $k_{I} \phi^{L}>10 k M$ we can find a suitable $N \in S_{L}$ by pigeonhole and the sequence $F^{(N)}$ defined by $(m, N, N+m, \ldots)$ works. + diff --git a/USA_TSTST/md/en-sols-TSTST-2018.md b/USA_TSTST/md/en-sols-TSTST-2018.md new file mode 100644 index 0000000000000000000000000000000000000000..1c68fb9902ff3b9d3b529b98496b2ec6541c4d43 --- /dev/null +++ b/USA_TSTST/md/en-sols-TSTST-2018.md @@ -0,0 +1,678 @@ +# USA TSTST 2018 Solutions
United States of America - TST Selection Test
Evan Chen《陳誼廷》
$60^{\text {th }}$ IMO 2019 United Kingdom and $8^{\text {th }}$ EGMO 2019 Ukraine + +## Contents + +0 Problems ..... 2 +1 Solutions to Day 1 ..... 4 +1.1 TSTST 2018/1, proposed by Evan Chen, Yang Liu ..... 4 +1.2 TSTST 2018/2, proposed by Victor Wang ..... 6 +1.3 TSTST 2018/3, proposed by Yannick Yao, Evan Chen ..... 8 +2 Solutions to Day 2 ..... 11 +2.1 TSTST 2018/4, proposed by Ivan Borsenco ..... 11 +2.2 TSTST 2018/5, proposed by Ankan Bhattacharya, Evan Chen ..... 13 +2.3 TSTST 2018/6, proposed by Ray Li ..... 17 +3 Solutions to Day 3 ..... 20 +3.1 TSTST 2018/7, proposed by Ashwin Sah ..... 20 +3.2 TSTST 2018/8, proposed by Ankan Bhattacharya, Evan Chen ..... 22 +3.3 TSTST 2018/9, proposed by Linus Hamilton ..... 24 + +## §0 Problems + +1. As usual, let $\mathbb{Z}[x]$ denote the set of single-variable polynomials in $x$ with integer coefficients. Find all functions $\theta: \mathbb{Z}[x] \rightarrow \mathbb{Z}$ such that for any polynomials $p, q \in$ $\mathbb{Z}[x]$, + +- $\theta(p+1)=\theta(p)+1$, and +- if $\theta(p) \neq 0$ then $\theta(p)$ divides $\theta(p \cdot q)$. + +2. In the nation of Onewaynia, certain pairs of cities are connected by one-way roads. Every road connects exactly two cities (roads are allowed to cross each other, e.g., via bridges), and each pair of cities has at most one road between them. Moreover, every city has exactly two roads leaving it and exactly two roads entering it. +We wish to close half the roads of Onewaynia in such a way that every city has exactly one road leaving it and exactly one road entering it. Show that the number of ways to do so is a power of 2 greater than 1 (i.e. of the form $2^{n}$ for some integer $n \geq 1$ ). +3. Let $A B C$ be an acute triangle with incenter $I$, circumcenter $O$, and circumcircle $\Gamma$. Let $M$ be the midpoint of $\overline{A B}$. Ray $A I$ meets $\overline{B C}$ at $D$. Denote by $\omega$ and $\gamma$ the circumcircles of $\triangle B I C$ and $\triangle B A D$, respectively. Line $M O$ meets $\omega$ at $X$ and $Y$, while line $C O$ meets $\omega$ at $C$ and $Q$. Assume that $Q$ lies inside $\triangle A B C$ and $\angle A Q M=\angle A C B$. + +Consider the tangents to $\omega$ at $X$ and $Y$ and the tangents to $\gamma$ at $A$ and $D$. Given that $\angle B A C \neq 60^{\circ}$, prove that these four lines are concurrent on $\Gamma$. +4. For an integer $n>0$, denote by $\mathcal{F}(n)$ the set of integers $m>0$ for which the polynomial $p(x)=x^{2}+m x+n$ has an integer root. +(a) Let $S$ denote the set of integers $n>0$ for which $\mathcal{F}(n)$ contains two consecutive integers. Show that $S$ is infinite but + +$$ +\sum_{n \in S} \frac{1}{n} \leq 1 +$$ + +(b) Prove that there are infinitely many positive integers $n$ such that $\mathcal{F}(n)$ contains three consecutive integers. +5. Let $A B C$ be an acute triangle with circumcircle $\omega$, and let $H$ be the foot of the altitude from $A$ to $\overline{B C}$. Let $P$ and $Q$ be the points on $\omega$ with $P A=P H$ and $Q A=Q H$. The tangent to $\omega$ at $P$ intersects lines $A C$ and $A B$ at $E_{1}$ and $F_{1}$ respectively; the tangent to $\omega$ at $Q$ intersects lines $A C$ and $A B$ at $E_{2}$ and $F_{2}$ respectively. Show that the circumcircles of $\triangle A E_{1} F_{1}$ and $\triangle A E_{2} F_{2}$ are congruent, and the line through their centers is parallel to the tangent to $\omega$ at $A$. +6. Let $S=\{1, \ldots, 100\}$, and for every positive integer $n$ define + +$$ +T_{n}=\left\{\left(a_{1}, \ldots, a_{n}\right) \in S^{n} \mid a_{1}+\cdots+a_{n} \equiv 0 \quad(\bmod 100)\right\} +$$ + +Determine which $n$ have the following property: if we color any 75 elements of $S$ red, then at least half of the $n$-tuples in $T_{n}$ have an even number of coordinates with red elements. +7. Let $n$ be a positive integer. A frog starts on the number line at 0 . Suppose it makes a finite sequence of hops, subject to two conditions: + +- The frog visits only points in $\left\{1,2, \ldots, 2^{n}-1\right\}$, each at most once. +- The length of each hop is in $\left\{2^{0}, 2^{1}, 2^{2}, \ldots\right\}$. (The hops may be either direction, left or right.) + +Let $S$ be the sum of the (positive) lengths of all hops in the sequence. What is the maximum possible value of $S$ ? +8. For which positive integers $b>2$ do there exist infinitely many positive integers $n$ such that $n^{2}$ divides $b^{n}+1$ ? +9. Show that there is an absolute constant $c<1$ with the following property: whenever $\mathcal{P}$ is a polygon with area 1 in the plane, one can translate it by a distance of $\frac{1}{100}$ in some direction to obtain a polygon $\mathcal{Q}$, for which the intersection of the interiors of $\mathcal{P}$ and $\mathcal{Q}$ has total area at most $c$. + +## §1 Solutions to Day 1 + +## §1.1 TSTST 2018/1, proposed by Evan Chen, Yang Liu + +Available online at https://aops.com/community/p10570981. + +## Problem statement + +As usual, let $\mathbb{Z}[x]$ denote the set of single-variable polynomials in $x$ with integer coefficients. Find all functions $\theta: \mathbb{Z}[x] \rightarrow \mathbb{Z}$ such that for any polynomials $p, q \in \mathbb{Z}[x]$, + +- $\theta(p+1)=\theta(p)+1$, and +- if $\theta(p) \neq 0$ then $\theta(p)$ divides $\theta(p \cdot q)$. + +The answer is $\theta: p \mapsto p(c)$, for each choice of $c \in \mathbb{Z}$. Obviously these work, so we prove these are the only ones. In what follows, $x \in \mathbb{Z}[x]$ is the identity polynomial, and $c=\theta(x)$. + +【 First solution (Merlijn Staps). Consider an integer $n \neq c$. Because $x-n \mid p(x)-p(n)$, we have + +$$ +\theta(x-n)|\theta(p(x)-p(n)) \Longrightarrow c-n| \theta(p(x))-p(n) . +$$ + +On the other hand, $c-n \mid p(c)-p(n)$. Combining the previous two gives $c-n \mid$ $\theta(p(x))-p(c)$, and by letting $n$ large we conclude $\theta(p(x))-p(c)=0$, so $\theta(p(x))=p(c)$. + +II Second solution. First, we settle the case $\operatorname{deg} p=0$. In that case, from the second property, $\theta(m)=m+\theta(0)$ for every integer $m \in \mathbb{Z}$ (viewed as a constant polynomial). Thus $m+\theta(0) \mid 2 m+\theta(0)$, hence $m+\theta(0) \mid-\theta(0)$, so $\theta(0)=0$ by taking $m$ large. Thus $\theta(m)=m$ for $m \in \mathbb{Z}$. + +Next, we address the case of $\operatorname{deg} p=1$. We know $\theta(x+b)=c+b$ for $b \in \mathbb{Z}$. Now for each particular $a \in \mathbb{Z}$, we have + +$$ +c+k|\theta(x+k)| \theta(a x+a k)=\theta(a x)+a k \Longrightarrow c+k \mid \theta(a x)-a c . +$$ + +for any $k \neq-c$. Since this is true for large enough $k$, we conclude $\theta(a x)=a c$. Thus $\theta(a x+b)=a c+b$. + +We now proceed by induction on $\operatorname{deg} p$. Fix a polynomial $p$ and assume it's true for all $p$ of smaller degree. Choose a large integer $n$ (to be determined later) for which $p(n) \neq p(c)$. We then have + +$$ +\left.\frac{p(c)-p(n)}{c-n}=\theta\left(\frac{p-p(n)}{x-n}\right) \right\rvert\, \theta(p-p(n))=\theta(p)-p(n) +$$ + +Subtracting off $c-n$ times the left-hand side gives + +$$ +\left.\frac{p(c)-p(n)}{c-n} \right\rvert\, \theta(p)-p(c) +$$ + +The left-hand side can be made arbitrarily large by letting $n \rightarrow \infty$, since $\operatorname{deg} p \geq 2$. Thus $\theta(p)=p(c)$, concluding the proof. +\ Authorship comments. I will tell you a story about the creation of this problem. Yang Liu and I were looking over the drafts of December and January TST in October 2017, and both of us had the impression that the test was too difficult. This sparked a non-serious suggestion that we should try to come up with a problem now that would be easy enough to use. While we ended up just joking about changing the TST, we did get this problem out of it. + +Our idea was to come up with a functional equation that was different from the usual fare: at first we tried $\mathbb{Z}[x] \rightarrow \mathbb{Z}[x]$, but then I suggested the idea of using $\mathbb{Z}[x] \rightarrow \mathbb{Z}$, with the answer being the "evaluation" map. Well, what properties does that satisfy? One answer was $a-b \mid p(a)-p(b)$; this didn't immediately lead to anything, but eventually we hit on the form of the problem above off this idea. At first we didn't require $\theta(p) \neq 0$ in the bullet, but without the condition the problem was too easy, since 0 divides only itself; and so the condition was added and we got the functional equation. + +I proposed the problem to USAMO 2018, but it was rejected (unsurprisingly; I think the problem may be too abstract for novice contestants). Instead it was used for TSTST, which I thought fit better. + +## §1.2 TSTST 2018/2, proposed by Victor Wang + +Available online at https://aops.com/community/p10570985. + +## Problem statement + +In the nation of Onewaynia, certain pairs of cities are connected by one-way roads. Every road connects exactly two cities (roads are allowed to cross each other, e.g., via bridges), and each pair of cities has at most one road between them. Moreover, every city has exactly two roads leaving it and exactly two roads entering it. + +We wish to close half the roads of Onewaynia in such a way that every city has exactly one road leaving it and exactly one road entering it. Show that the number of ways to do so is a power of 2 greater than 1 (i.e. of the form $2^{n}$ for some integer $n \geq 1$ ). + +In the language of graph theory, we have a simple digraph $G$ which is 2 -regular and we seek the number of sub-digraphs which are 1-regular. We now present two solution paths. + +『 First solution, combinatorial. We construct a simple undirected bipartite graph $\Gamma$ as follows: + +- the vertex set consists of two copies of $V(G)$, say $V_{\text {out }}$ and $V_{\text {in }}$; and +- for $v \in V_{\text {out }}$ and $w \in V_{\text {in }}$ we have an undirected edge $v w \in E(\Gamma)$ if and only if the directed edge $v \rightarrow w$ is in $G$. + +Moreover, the desired sub-digraphs of $H$ correspond exactly to perfect matchings of $\Gamma$. +However the graph $\Gamma$ is 2 -regular and hence consists of several disjoint (simple) cycles of even length. If there are $n$ such cycles, the number of perfect matchings is $2^{n}$, as desired. + +Remark. The construction of $\Gamma$ is not as magical as it may first seem. +Suppose we pick a road $v_{1} \rightarrow v_{2}$ to use. Then, the other road $v_{3} \rightarrow v_{2}$ is certainly not used; hence some other road $v_{3} \rightarrow v_{4}$ must be used, etc. We thus get a cycle of forced decisions until we eventually return to the vertex $v_{1}$. + +These cycles in the original graph $G$ (where the arrows alternate directions) correspond to the cycles we found in $\Gamma$. It's merely that phrasing the solution in terms of $\Gamma$ makes it cleaner in a linguistic sense, but not really in a mathematical sense. + +【 Second solution by linear algebra over $\mathbb{F}_{2}$ (Brian Lawrence). This is actually not that different from the first solution. For each edge $e$, we create an indicator variable $x_{e}$. We then require for each vertex $v$ that: + +- If $e_{1}$ and $e_{2}$ are the two edges leaving $v$, then we require $x_{e_{1}}+x_{e_{2}} \equiv 1(\bmod 2)$. +- If $e_{3}$ and $e_{4}$ are the two edges entering $v$, then we require $x_{e_{3}}+x_{e_{4}} \equiv 1(\bmod 2)$. + +We thus get a large system of equations. Moreover, the solutions come in natural pairs $\vec{x}$ and $\vec{x}+\overrightarrow{1}$ and therefore the number of solutions is either zero, or a power of two. So we just have to prove there is at least one solution. + +For linear algebra reasons, there can only be zero solutions if some nontrivial linear combination of the equations gives the sum $0 \equiv 1$. So suppose we added up some subset $S$ +of the equations for which every variable appeared on the left-hand side an even number of times. Then every variable that did appear appeared exactly twice; and accordingly we see that the edges corresponding to these variables form one or more even cycles as in the previous solution. Of course, this means $|S|$ is even, so we really have $0 \equiv 0(\bmod 2)$ as needed. + +Remark. The author's original proposal contained a second part asking to show that it was not always possible for the resulting $H$ to be connected, even if $G$ was strongly connected. This problem is related to IMO Shortlist 2002 C6, which gives an example of a strongly connected graph which does have a full directed Hamiltonian cycle. + +## §1.3 TSTST 2018/3, proposed by Yannick Yao, Evan Chen + +Available online at https://aops.com/community/p10570988. + +## Problem statement + +Let $A B C$ be an acute triangle with incenter $I$, circumcenter $O$, and circumcircle $\Gamma$. Let $M$ be the midpoint of $\overline{A B}$. Ray $A I$ meets $\overline{B C}$ at $D$. Denote by $\omega$ and $\gamma$ the circumcircles of $\triangle B I C$ and $\triangle B A D$, respectively. Line $M O$ meets $\omega$ at $X$ and $Y$, while line $C O$ meets $\omega$ at $C$ and $Q$. Assume that $Q$ lies inside $\triangle A B C$ and $\angle A Q M=\angle A C B$. + +Consider the tangents to $\omega$ at $X$ and $Y$ and the tangents to $\gamma$ at $A$ and $D$. Given that $\angle B A C \neq 60^{\circ}$, prove that these four lines are concurrent on $\Gamma$. + +Henceforth assume $\angle A \neq 60^{\circ}$; we prove the concurrence. Let $L$ denote the center of $\omega$, which is the midpoint of minor arc $B C$. + +Claim - Let $K$ be the point on $\omega$ such that $\overline{K L} \| \overline{A B}$ and $\overline{K C} \| \overline{A L}$. Then $\overline{K A}$ is tangent to $\gamma$, and we may put + +$$ +x=K A=L B=L C=L X=L Y=K X=K Y . +$$ + +Proof. By construction, $K A=L B=L C$. Also, $\overline{M O}$ is the perpendicular bisector of $\overline{K L}$ (since the chords $\overline{K L}, \overline{A B}$ of $\omega$ are parallel) and so $K X L Y$ is a rhombus as well. + +Moreover, $\overline{K A}$ is tangent to $\gamma$ as well since + +$$ +\measuredangle K A D=\measuredangle K A L=\measuredangle K A C+\measuredangle C A L=\measuredangle K B C+\measuredangle A B K=\measuredangle A B C . +$$ + +![](https://cdn.mathpix.com/cropped/2024_11_19_a0f01f2a44c8136e2158g-08.jpg?height=918&width=914&top_left_y=1551&top_left_x=571) + +Up to now we have not used the existence of $Q$; we henceforth do so. + +Note that $Q \neq O$, since $\angle A \neq 60^{\circ} \Longrightarrow O \notin \omega$. Moreover, we have $\angle A O M=\angle A C B$ too. Since $O$ and $Q$ both lie inside $\triangle A B C$, this implies that $A, M, O, Q$ are concyclic. As $Q \neq O$ we conclude $\angle C Q A=90^{\circ}$. + +The main claim is now: +Claim - Assuming $Q$ exists, the rhombus $L X K Y$ is a square. In particular, $\overline{K X}$ and $\overline{K Y}$ are tangent to $\omega$. + +First proof of Claim, communicated by Milan Haiman. Observe that $\triangle Q L C \sim \triangle L O C$ since both triangles are isosceles and share a base angle. Hence, $C L^{2}=C O \cdot C Q$. + +Let $N$ be the midpoint of $\overline{A C}$, which lies on $(A M O Q)$. Then, + +$$ +x^{2}=C L^{2}=C O \cdot C Q=C N \cdot C A=\frac{1}{2} C A^{2}=\frac{1}{2} L K^{2} +$$ + +where we have also used the fact $A Q O N$ is cyclic. Thus $L K=\sqrt{2} x$ and so the rhombus $L X K Y$ is actually a square. + +Second proof of Claim, Evan Chen. Observe that $Q$ lies on the circle with diameter $\overline{A C}$, centered at $N$, say. This means that $O$ lies on the radical axis of $\omega$ and $(N)$, hence $\overline{N L} \perp \overline{C O}$ implying + +$$ +\begin{aligned} +N O^{2}+C L^{2} & =N C^{2}+L O^{2}=N C^{2}+O C^{2}=N C^{2}+N O^{2}+N C^{2} \\ +\Longrightarrow x^{2} & =2 N C^{2} \\ +\Longrightarrow x & =\sqrt{2} N C=\frac{1}{\sqrt{2}} A C=\frac{1}{\sqrt{2}} L K +\end{aligned} +$$ + +So $L X K Y$ is a rhombus with $L K=\sqrt{2} x$. Hence it is a square. +Third proof of Claim. A solution by trig is also possible. As in the previous claims, it suffices to show that $A C=\sqrt{2} x$. + +First, we compute the length $C Q$ in two ways; by angle chasing one can show $\angle C B Q=$ $180^{\circ}-(\angle B Q C+\angle Q C B)=\frac{1}{2} \angle A$, and so + +$$ +\begin{aligned} +A C \sin B=C Q & =\frac{B C}{\sin \left(90^{\circ}+\frac{1}{2} \angle A\right)} \cdot \sin \frac{1}{2} \angle A \\ +\Longleftrightarrow \sin ^{2} B & =\frac{\sin A \cdot \sin \frac{1}{2} \angle A}{\cos \frac{1}{2} \angle A} \\ +\Longleftrightarrow \sin ^{2} B & =2 \sin ^{2} \frac{1}{2} \angle A \\ +\Longleftrightarrow \sin B & =\sqrt{2} \sin \frac{1}{2} \angle A \\ +\Longleftrightarrow 2 R \sin B & =\sqrt{2}\left(2 R \sin \frac{1}{2} \angle A\right) \\ +\Longleftrightarrow A C & =\sqrt{2} x +\end{aligned} +$$ + +as desired (we have here used the fact $\triangle A B C$ is acute to take square roots). +It is interesting to note that $\sin ^{2} B=2 \sin ^{2} \frac{1}{2} \angle A$ can be rewritten as + +$$ +\cos A=\cos ^{2} B +$$ + +since $\cos ^{2} B=1-\sin ^{2} B=1-2 \sin ^{2} \frac{1}{2} \angle A=\cos A$; this is the condition for the existence of the point $Q$. + +We finish by proving that + +$$ +K D=K A +$$ + +and hence line $\overline{K D}$ is tangent to $\gamma$. Let $E=\overline{B C} \cap \overline{K L}$. Then + +$$ +L E \cdot L K=L C^{2}=L X^{2}=\frac{1}{2} L K^{2} +$$ + +and so $E$ is the midpoint of $\overline{L K}$. Thus $\overline{M X O Y}, \overline{B C}, \overline{K L}$ are concurrent at $E$. As $\overline{D L} \| \overline{K C}$, we find that $D L C K$ is a parallelogram, so $K D=C L=K A$ as well. Thus $\overline{K D}$ and $\overline{K A}$ are tangent to $\gamma$. + +Remark. The condition $\angle A \neq 60^{\circ}$ cannot be dropped, since if $Q=O$ the problem is not true. + +On the other hand, nearly all solutions begin by observing $Q \neq O$ and then obtaining $\angle A Q O=90^{\circ}$. This gives a way to construct the diagram by hand with ruler and compass. One draws an arbitrary chord $\overline{B C}$ of a circle $\omega$ centered at $L$, and constructs $O$ as the circumcenter of $\triangle B L C$ (hence obtaining $\Gamma$ ). Then $Q$ is defined as the intersection of ray $C O$ with $\omega$, and $A$ is defined by taking the perpendicular line through $Q$ on the circle $\Gamma$. In this way we can draw a triangle $A B C$ satisfying the problem conditions. +\ Authorship comments. In the notation of the present points, the question originally sent to me by Yannick Yao read: + +Circles $(L)$ and $(O)$ are drawn, meeting at $B$ and $C$, with $L$ on $(O)$. Ray $C O$ meets $(L)$ at $Q$, and $A$ is on $(O)$ such that $\angle C Q A=90^{\circ}$. The angle bisector of $\angle A O B$ meets $(L)$ at $X$ and $Y$. Show that $\angle X L Y=90^{\circ}$. + +Notice the points $M$ and $K$ are absent from the problem. I am told this was found as part of the computer game "Euclidea". Using this as the starting point, I constructed the TSTST problem by recognizing the significance of that special point $K$, which became the center of attention. + +## §2 Solutions to Day 2 + +## §2.1 TSTST 2018/4, proposed by Ivan Borsenco + +Available online at https://aops.com/community/p10570991. + +## Problem statement + +For an integer $n>0$, denote by $\mathcal{F}(n)$ the set of integers $m>0$ for which the polynomial $p(x)=x^{2}+m x+n$ has an integer root. +(a) Let $S$ denote the set of integers $n>0$ for which $\mathcal{F}(n)$ contains two consecutive integers. Show that $S$ is infinite but + +$$ +\sum_{n \in S} \frac{1}{n} \leq 1 +$$ + +(b) Prove that there are infinitely many positive integers $n$ such that $\mathcal{F}(n)$ contains three consecutive integers. + +We prove the following. +Claim - The set $S$ is given explicitly by $S=\{x(x+1) y(y+1) \mid x, y>0\}$. + +Proof. Note that $m, m+1 \in \mathcal{F}(n)$ if and only if there exist integers $q>p \geq 0$ such that + +$$ +\begin{aligned} +m^{2}-4 n & =p^{2} \\ +(m+1)^{2}-4 n & =q^{2} +\end{aligned} +$$ + +Subtraction gives $2 m+1=q^{2}-p^{2}$, so $p$ and $q$ are different parities. We can thus let $q-p=2 x+1, q+p=2 y+1$, where $y \geq x \geq 0$ are integers. It follows that + +$$ +\begin{aligned} +4 n & =m^{2}-p^{2} \\ +& =\left(\frac{q^{2}-p^{2}-1}{2}\right)^{2}-p^{2}=\left(\frac{q^{2}-p^{2}-1}{2}-p\right)\left(\frac{q^{2}-p^{2}-1}{2}+p\right) \\ +& =\frac{q^{2}-\left(p^{2}+2 p+1\right)}{2} \cdot \frac{q^{2}-\left(p^{2}-2 p+1\right)}{2} \\ +& =\frac{1}{4}(q-p-1)(q-p+1)(q+p-1)(q+p+1)=\frac{1}{4}(2 x)(2 x+2)(2 y)(2 y+2) \\ +\Longrightarrow n & =x(x+1) y(y+1) . +\end{aligned} +$$ + +Since $n>0$ we require $x, y>0$. Conversely, if $n=x(x+1) y(y+1)$ for positive $x$ and $y$ then $m=\sqrt{p^{2}+4 n}=\sqrt{(y-x)^{2}+4 n}=2 x y+x+y=x(y+1)+(x+1) y$ and $m+1=2 x y+x+y+1=x y+(x+1)(y+1)$. Thus we conclude the main claim. + +From this, part (a) follows as + +$$ +\sum_{n \in S} n^{-1} \leq\left(\sum_{x \geq 1} \frac{1}{x(x+1)}\right)\left(\sum_{y \geq 1} \frac{1}{y(y+1)}\right)=1 \cdot 1=1 +$$ + +As for (b), retain the notation in the proof of the claim. Now $m+2 \in S$ if and only if $(m+2)^{2}-4 n$ is a square, say $r^{2}$. Writing in terms of $p$ and $q$ as parameters we find + +$$ +\begin{aligned} +r^{2} & =(m+2)^{2}-4 n=m^{2}-4 n+4 m+4=p^{2}+2+2(2 m+1) \\ +& =p^{2}+2\left(q^{2}-p^{2}\right)+2=2 q^{2}-p^{2}+2 \\ +\Longleftrightarrow 2 q^{2}+2 & =p^{2}+r^{2} \quad(\dagger) +\end{aligned} +$$ + +with $q>p$ of different parity and $n=\frac{1}{16}(q-p-1)(q-p+1)(q+p-1)(q+p+1)$. +Note that (by taking modulo 8 ) we have $q \not \equiv p \equiv r(\bmod 2)$, and so there are no parity issues and we will always assume $p2$ do there exist infinitely many positive integers $n$ such that $n^{2}$ divides $b^{n}+1$ ? + +This problem is sort of the union of IMO 1990/3 and IMO 2000/5. +The answer is any $b$ such that $b+1$ is not a power of 2 . In the forwards direction, we first prove more carefully the following claim. + +Claim - If $b+1$ is a power of 2 , then the only $n$ which is valid is $n=1$. +Proof. Assume $n>1$ and let $p$ be the smallest prime dividing $n$. We cannot have $p=2$, since then $4 \mid b^{n}+1 \equiv 2(\bmod 4)$. Thus, + +$$ +b^{2 n} \equiv 1 \quad(\bmod p) +$$ + +so the order of $b(\bmod p)$ divides $\operatorname{gcd}(2 n, p-1)=2$. Hence $p \mid b^{2}-1=(b-1)(b+1)$. +But since $b+1$ was a power of 2 , this forces $p \mid b-1$. Then $0 \equiv b^{n}+1 \equiv 2(\bmod p)$, contradiction. + +On the other hand, suppose that $b+1$ is not a power of 2 (and that $b>2$ ). We will inductively construct an infinite sequence of distinct primes $p_{0}, p_{1}, \ldots$, such that the following two properties hold for each $k \geq 0$ : + +- $p_{0}^{2} \ldots p_{k-1}^{2} p_{k} \mid b^{p_{0} \ldots p_{k-1}}+1$, +- and hence $p_{0}^{2} \ldots p_{k-1}^{2} p_{k}^{2} \mid b^{p_{0} \ldots p_{k-1} p_{k}}+1$ by exponent lifting lemma. + +This will solve the problem. +Initially, let $p_{0}$ be any odd prime dividing $b+1$. For the inductive step, we contend there exists an odd prime $q \notin\left\{p_{0}, \ldots, p_{k}\right\}$ such that $q \mid b^{p_{0} \ldots p_{k}}+1$. Indeed, this follows immediately by Zsigmondy theorem since $p_{0} \ldots p_{k}$ divides $b^{p_{0} \ldots p_{k-1}}+1$. Since $\left(b^{p_{0} \ldots p_{k}}\right)^{q} \equiv b^{p_{0} \ldots p_{k}}(\bmod q)$, it follows we can then take $p_{k+1}=q$. This finishes the induction. + +To avoid the use of Zsigmondy, one can instead argue as follows: let $p=p_{k}$ for brevity, and let $c=b^{p_{0} \ldots p_{k-1}}$. Then $\frac{\frac{c}{}^{p}+1}{c+1}=c^{p-1}-c^{p-2}+\cdots+1$ has GCD exactly $p$ with $c+1$. Moreover, this quotient is always odd. Thus as long as $c^{p}+1>p \cdot(c+1)$, there will be some new prime dividing $c^{p}+1$ but not $c+1$. This is true unless $p=3$ and $c=2$, but we assumed $b>2$ so this case does not appear. + +Remark (On new primes). In going from $n^{2} \mid b^{n}+1$ to $(n q)^{2} \mid b^{n q}+1$, one does not necessarily need to pick a $q$ such that $q \nmid n$, as long as $\nu_{q}\left(n^{2}\right)<\nu_{q}\left(b^{n}+1\right)$. In other words it suffices to just check that $\frac{b^{n}+1}{n^{2}}$ is not a power of 2 in this process. + +However, this calculation is a little more involved with this approach. One proceeds by noting that $n$ is odd, hence $\nu_{2}\left(b^{n}+1\right)=\nu_{2}(b+1)$, and thus $\frac{b^{n}+1}{n^{2}}=2^{\nu_{2}(b+1)} \leq b+1$, which is a little harder to bound than the analogous $c^{p}+1>p \cdot(c+1)$ from the previous solution. +\ Authorship comments. I came up with this problem by simply mixing together the main ideas of IMO 1990/3 and IMO 2000/5, late one night after a class. On the other hand, I do not consider it very original; it is an extremely "routine" number theory problem for experienced contestants, using highly standard methods. Thus it may not be that interesting, but is a good discriminator of understanding of fundamentals. + +IMO 1990/3 shows that if $b=2$, then the only $n$ which work are $n=1$ and $n=3$. Thus $b=2$ is a special case and for this reason the problem explicitly requires $b>2$. + +An alternate formulation of the problem is worth mentioning. Originally, the problem statement asked whether there existed $n$ with at least 3 (or 2018, etc.) prime divisors, thus preventing the approach in which one takes a prime $q$ dividing $\frac{b^{n}+1}{n^{2}}$. Ankan Bhattacharya suggested changing it to "infinitely many $n$ ", which is more natural. + +These formulations are actually not so different though. Explicitly, suppose $k^{2} \mid b^{k}+1$ and $p \mid b^{k}+1$. Consider any $k \mid n$ with $n^{2} \mid b^{n}+1$, and let $p$ be an odd prime dividing $b^{k}+1$. Then $2 \nu_{p}(n) \leq \nu_{p}\left(b^{n}+1\right)=\nu_{p}(n / k)+\nu_{p}\left(b^{k}+1\right)$ and thus + +$$ +\nu_{p}(n / k) \leq \nu_{p}\left(\frac{b^{k}+1}{k^{2}}\right) . +$$ + +Effectively, this means we can only add each prime a certain number of times. + +## §3.3 TSTST 2018/9, proposed by Linus Hamilton + +Available online at https://aops.com/community/p10571003. + +## Problem statement + +Show that there is an absolute constant $c<1$ with the following property: whenever $\mathcal{P}$ is a polygon with area 1 in the plane, one can translate it by a distance of $\frac{1}{100}$ in some direction to obtain a polygon $\mathcal{Q}$, for which the intersection of the interiors of $\mathcal{P}$ and $\mathcal{Q}$ has total area at most $c$. + +The following solution is due to Brian Lawrence. We will prove the result with the generality of any measurable set $\mathcal{P}$ (rather than a polygon). For a vector $v$ in the plane, write $\mathcal{P}+v$ for the translate of $\mathcal{P}$ by $v$. + +Suppose $\mathcal{P}$ is a polygon of area 1 , and $\varepsilon>0$ is a constant, such that for any translate $\mathcal{Q}=\mathcal{P}+v$, where $v$ has length exactly $\frac{1}{100}$, the intersection of $\mathcal{P}$ and $\mathcal{Q}$ has area at least $1-\varepsilon$. The problem asks us to prove a lower bound on $\varepsilon$. + +## Lemma + +Fix a sequence of $n$ vectors $v_{1}, v_{2}, \ldots, v_{n}$, each of length $\frac{1}{100}$. A grasshopper starts at a random point $x$ of $\mathcal{P}$, and makes $n$ jumps to $x+v_{1}+\cdots+v_{n}$. Then it remains in $\mathcal{P}$ with probability at least $1-n \varepsilon$. + +Proof. In order for the grasshopper to leave $\mathcal{P}$ at step $i$, the grasshopper's position before step $i$ must be inside the difference set $\mathcal{P} \backslash\left(\mathcal{P}-v_{i}\right)$. Since this difference set has area at most $\varepsilon$, the probability the grasshopper leaves $\mathcal{P}$ at step $i$ is at most $\varepsilon$. Summing over the $n$ steps, the probability that the grasshopper ever manages to leave $\mathcal{P}$ is at most $n \varepsilon$. + +## Corollary + +Fix a vector $w$ of length at most 8. A grasshopper starts at a random point $x$ of $\mathcal{P}$, and jumps to $x+w$. Then it remains in $\mathcal{P}$ with probability at least $1-800 \varepsilon$. + +Proof. Apply the previous lemma with 800 jumps. Any vector $w$ of length at most 8 can be written as $w=v_{1}+v_{2}+\cdots+v_{800}$, where each $v_{i}$ has length exactly $\frac{1}{100}$. + +Now consider the process where we select a random starting point $x \in \mathcal{P}$ for our grasshopper, and a random vector $w$ of length at most 8 (sampled uniformly from the closed disk of radius 8 ). Let $q$ denote the probability of staying inside $\mathcal{P}$ we will bound $q$ from above and below. + +- On the one hand, suppose we pick $w$ first. By the previous corollary, $q \geq 1-800 \varepsilon$ (irrespective of the chosen $w$ ). +- On the other hand, suppose we pick $x$ first. Then the possible landing points $x+w$ are uniformly distributed over a closed disk of radius 8 , which has area $64 \pi$. The probability of landing in $\mathcal{P}$ is certainly at most $\frac{[\mathcal{P}]}{64 \pi}$. + +Consequently, we deduce + +$$ +1-800 \varepsilon \leq q \leq \frac{[\mathcal{P}]}{64 \pi} \Longrightarrow \varepsilon>\frac{1-\frac{[\mathcal{P}]}{64 \pi}}{800}>0.001 +$$ + +as desired. +Remark. The choice of 800 jumps is only for concreteness; any constant $n$ for which $\pi(n / 100)^{2}>1$ works. I think $n=98$ gives the best bound following this approach. + diff --git a/USA_TSTST/md/en-sols-TSTST-2019.md b/USA_TSTST/md/en-sols-TSTST-2019.md new file mode 100644 index 0000000000000000000000000000000000000000..d253d024fe18424779ea39b847c78f5759f5f14f --- /dev/null +++ b/USA_TSTST/md/en-sols-TSTST-2019.md @@ -0,0 +1,764 @@ +# USA TSTST 2019 Solutions
United States of America - TST Selection Test
Ankan Bhattacharya and Evan Chen
$61^{\text {st }}$ IMO 2020 Russia and $9^{\text {th }}$ EGMO 2020 Netherlands + +## Contents + +0 Problems ..... 2 +1 Solutions to Day 1 ..... 3 +1.1 TSTST 2019/1, proposed by Evan Chen ..... 3 +1.2 TSTST 2019/2, proposed by Merlijn Staps ..... 6 +1.3 TSTST 2019/3, proposed by Nikolai Beluhov ..... 9 +2 Solutions to Day 2 ..... 11 +2.1 TSTST 2019/4, proposed by Merlijn Staps ..... 11 +2.2 TSTST 2019/5, proposed by Gunmay Handa ..... 13 +2.3 TSTST 2019/6, proposed by Nikolai Beluhov ..... 19 +3 Solutions to Day 3 ..... 21 +3.1 TSTST 2019/7, proposed by Ankan Bhattacharya ..... 21 +3.2 TSTST 2019/8, proposed by Ankan Bhattacharya ..... 23 +3.3 TSTST 2019/9, proposed by Ankan Bhattacharya ..... 25 + +## §0 Problems + +1. Find all binary operations $\diamond: \mathbb{R}_{>0} \times \mathbb{R}_{>0} \rightarrow \mathbb{R}_{>0}$ (meaning $\diamond$ takes pairs of positive real numbers to positive real numbers) such that for any real numbers $a, b, c>0$, + +- the equation $a \diamond(b \diamond c)=(a \diamond b) \cdot c$ holds; and +- if $a \geq 1$ then $a \diamond a \geq 1$. + +2. Let $A B C$ be an acute triangle with circumcircle $\Omega$ and orthocenter $H$. Points $D$ and $E$ lie on segments $A B$ and $A C$ respectively, such that $A D=A E$. The lines through $B$ and $C$ parallel to $\overline{D E}$ intersect $\Omega$ again at $P$ and $Q$, respectively. Denote by $\omega$ the circumcircle of $\triangle A D E$. +(a) Show that lines $P E$ and $Q D$ meet on $\omega$. +(b) Prove that if $\omega$ passes through $H$, then lines $P D$ and $Q E$ meet on $\omega$ as well. +3. On an infinite square grid we place finitely many cars, which each occupy a single cell and face in one of the four cardinal directions. Cars may never occupy the same cell. It is given that the cell immediately in front of each car is empty, and moreover no two cars face towards each other (no right-facing car is to the left of a left-facing car within a row, etc.). In a move, one chooses a car and shifts it one cell forward to a vacant cell. Prove that there exists an infinite sequence of valid moves using each car infinitely many times. +4. Consider coins with positive real denominations not exceeding 1. Find the smallest $C>0$ such that the following holds: if we are given any 100 such coins with total value 50 , then we can always split them into two stacks of 50 coins each such that the absolute difference between the total values of the two stacks is at most $C$. +5. Let $A B C$ be an acute triangle with orthocenter $H$ and circumcircle $\Gamma$. A line through $H$ intersects segments $A B$ and $A C$ at $E$ and $F$, respectively. Let $K$ be the circumcenter of $\triangle A E F$, and suppose line $A K$ intersects $\Gamma$ again at a point $D$. Prove that line $H K$ and the line through $D$ perpendicular to $\overline{B C}$ meet on $\Gamma$. +6. Suppose $P$ is a polynomial with integer coefficients such that for every positive integer $n$, the sum of the decimal digits of $|P(n)|$ is not a Fibonacci number. Must $P$ be constant? +7. Let $f: \mathbb{Z} \rightarrow\left\{1,2, \ldots, 10^{100}\right\}$ be a function satisfying + +$$ +\operatorname{gcd}(f(x), f(y))=\operatorname{gcd}(f(x), x-y) +$$ + +for all integers $x$ and $y$. Show that there exist positive integers $m$ and $n$ such that $f(x)=\operatorname{gcd}(m+x, n)$ for all integers $x$. +8. Let $\mathcal{S}$ be a set of 16 points in the plane, no three collinear. Let $\chi(\mathcal{S})$ denote the number of ways to draw 8 line segments with endpoints in $\mathcal{S}$, such that no two drawn segments intersect, even at endpoints. Find the smallest possible value of $\chi(\mathcal{S})$ across all such $\mathcal{S}$. +9. Let $A B C$ be a triangle with incenter $I$. Points $K$ and $L$ are chosen on segment $B C$ such that the incircles of $\triangle A B K$ and $\triangle A B L$ are tangent at $P$, and the incircles of $\triangle A C K$ and $\triangle A C L$ are tangent at $Q$. Prove that $I P=I Q$. + +## §1 Solutions to Day 1 + +## §1.1 TSTST 2019/1, proposed by Evan Chen + +Available online at https://aops.com/community/p12608849. + +## Problem statement + +Find all binary operations $\diamond: \mathbb{R}_{>0} \times \mathbb{R}_{>0} \rightarrow \mathbb{R}_{>0}$ (meaning $\diamond$ takes pairs of positive real numbers to positive real numbers) such that for any real numbers $a, b, c>0$, + +- the equation $a \diamond(b \diamond c)=(a \diamond b) \cdot c$ holds; and +- if $a \geq 1$ then $a \diamond a \geq 1$. + +The answer is only multiplication and division, which both obviously work. +We present two approaches, one appealing to theorems on Cauchy's functional equation, and one which avoids it. + +【 First solution using Cauchy FE. We prove: +Claim - We have $a \diamond b=a f(b)$ where $f$ is some involutive and totally multiplicative function. (In fact, this classifies all functions satisfying the first condition completely.) + +Proof. Let $P(a, b, c)$ denote the assertion $a \diamond(b \diamond c)=(a \diamond b) \cdot c$. + +- Note that for any $x$, the function $y \mapsto x \diamond y$ is injective, because if $x \diamond y_{1}=x \diamond y_{2}$ then take $P\left(1, x, y_{i}\right)$ to get $y_{1}=y_{2}$. +- Take $P(1, x, 1)$ and injectivity to get $x \diamond 1=x$. +- Take $P(1,1, y)$ to get $1 \diamond(1 \diamond y)=y$. +- Take $P(x, 1,1 \diamond y)$ to get + +$$ +x \diamond y=x \cdot(1 \diamond y) +$$ + +Henceforth let us define $f(y)=1 \diamond y$, so $f(1)=1, f$ is involutive and + +$$ +x \diamond y=x f(y) +$$ + +Plugging this into the original condition now gives $f(b f(c))=f(b) c$, which (since $f$ is an involution) gives $f$ completely multiplicative. + +In particular, $f(1)=1$. We are now interested only in the second condition, which reads $f(x) \geq 1 / x$ for $x \geq 1$. + +Define the function + +$$ +g(t)=\log f\left(e^{t}\right) +$$ + +so that $g$ is additive, and also $g(t) \geq-t$ for all $t \geq 0$. We appeal to the following theorem: + +## Lemma + +If $h: \mathbb{R} \rightarrow \mathbb{R}$ is an additive function which is not linear, then it is dense in the plane: for any point $\left(x_{0}, y_{0}\right)$ and $\varepsilon>0$ there exists $(x, y)$ such that $h(x)=y$ and $\sqrt{\left(x-x_{0}\right)^{2}+\left(y-y_{0}\right)^{2}}<\varepsilon$. + +Applying this lemma with the fact that $g(t) \geq-t$ implies readily that $g$ is linear. In other words, $f$ is of the form $f(x)=x^{r}$ for some fixed real number $r$. It is easy to check $r= \pm 1$ which finishes. + +【 Second solution manually. As before we arrive at $a \diamond b=a f(b)$, with $f$ an involutive and totally multiplicative function. + +We prove that: +Claim - For any $a>0$, we have $f(a) \in\{1 / a, a\}$. + +Proof. WLOG $b>1$, and suppose $f(b)=a \geq 1 / b$ hence $f(a)=b$. +Assume that $a b>1$; we show $a=b$. Note that for integers $m$ and $n$ with $a^{n} b^{m} \geq 1$, we must have + +$$ +a^{m} b^{n}=f(b)^{m} f(a)^{n}=f\left(a^{n} b^{m}\right) \geq \frac{1}{a^{n} b^{m}} \Longrightarrow(a b)^{m+n} \geq 1 +$$ + +and thus we have arrived at the proposition + +$$ +m+n<0 \Longrightarrow n \log _{b} a+m<0 +$$ + +for all integers $m$ and $n$. Due to the density of $\mathbb{Q}$ in the real numbers, this can only happen if $\log _{b} a=1$ or $a=b$. + +Claim - The function $f$ is continuous. + +Proof. Indeed, it's equivalent to show $g(t)=\log f\left(e^{t}\right)$ is continuous, and we have that + +$$ +|g(t)-g(s)|=\left|\log f\left(e^{t-s}\right)\right|=|t-s| +$$ + +since $f\left(e^{t-s}\right)=e^{ \pm|t-s|}$. Therefore $g$ is Lipschitz. Hence $g$ continuous, and $f$ is too. +Finally, we have from $f$ multiplicative that + +$$ +f\left(2^{q}\right)=f(2)^{q} +$$ + +for every rational number $q$, say. As $f$ is continuous this implies $f(x) \equiv x$ or $f(x) \equiv 1 / x$ identically (depending on whether $f(2)=2$ or $f(2)=1 / 2$, respectively). + +Therefore, $a \diamond b=a b$ or $a \diamond b=a \div b$, as needed. +Remark. The Lipschitz condition is one of several other ways to proceed. The point is that if $f(2)=2$ (say), and $x / 2^{q}$ is close to 1 , then $f(x) / 2^{q}=f\left(x / 2^{q}\right)$ is close to 1 , which is enough to force $f(x)=x$ rather than $f(x)=1 / x$. + +Remark. Compare to AMC 10A 2016 \#23, where the second condition is $a \diamond a=1$. + +## §1.2 TSTST 2019/2, proposed by Merlijn Staps + +Available online at https://aops.com/community/p12608478. + +## Problem statement + +Let $A B C$ be an acute triangle with circumcircle $\Omega$ and orthocenter $H$. Points $D$ and $E$ lie on segments $A B$ and $A C$ respectively, such that $A D=A E$. The lines through $B$ and $C$ parallel to $\overline{D E}$ intersect $\Omega$ again at $P$ and $Q$, respectively. Denote by $\omega$ the circumcircle of $\triangle A D E$. +(a) Show that lines $P E$ and $Q D$ meet on $\omega$. +(b) Prove that if $\omega$ passes through $H$, then lines $P D$ and $Q E$ meet on $\omega$ as well. + +We will give one solution to (a), then several solutions to (b). +【 Solution to (a). Note that $\measuredangle A Q P=\measuredangle A B P=\measuredangle A D E$ and $\measuredangle A P Q=\measuredangle A C Q=$ $\measuredangle A E D$, so we have a spiral similarity $\triangle A D E \sim \triangle A Q P$. Therefore, lines $P E$ and $Q D$ meet at the second intersection of $\omega$ and $\Omega$ other than $A$. Call this point $X$. + +【 Solution to (b) using angle chasing. Let $L$ be the reflection of $H$ across $\overline{A B}$, which lies on $\Omega$. + +Claim - Points $L, D, P$ are collinear. +Proof. This is just angle chasing: + +$$ +\begin{aligned} +\measuredangle C L D & =\measuredangle D H L=\measuredangle D H A+\measuredangle A H L=\measuredangle D E A+\measuredangle A H C \\ +& =\measuredangle A D E+\measuredangle C B A=\measuredangle A B P+\measuredangle C B A=\measuredangle C B P=\measuredangle C L P . +\end{aligned} +$$ + +![](https://cdn.mathpix.com/cropped/2024_11_19_f4f672a97b66743213a8g-06.jpg?height=641&width=709&top_left_y=1707&top_left_x=676) + +Now let $K \in \omega$ such that $D H K E$ is an isosceles trapezoid, i.e. $\measuredangle B A H=\measuredangle K A E$. + +Claim - Points $D, K, P$ are collinear. + +Proof. Using the previous claim, + +$$ +\measuredangle K D E=\measuredangle K A E=\measuredangle B A H=\measuredangle L A B=\measuredangle L P B=\measuredangle D P B=\measuredangle P D E . +$$ + +By symmetry, $\overline{Q E}$ will then pass through the same $K$, as needed. +Remark. These two claims imply each other, so guessing one of them allows one to realize the other. It is likely the latter is easiest to guess from the diagram, since it does not need any additional points. + +【 Solution to (b) by orthogonal circles (found by contestants). We define $K$ as in the previous solution, but do not claim that $K$ is the desired intersection. Instead, we note that: + +Claim - Point $K$ is the orthocenter of isosceles triangle $A P Q$. +Proof. Notice that $A H=A K$ and $B C=P Q$. Moreover from $\overline{A H} \perp \overline{B C}$ we deduce $\overline{A K} \perp \overline{P Q}$ by reflection across the angle bisector. + +In light of the formula " $A H^{2}=4 R^{2}-a^{2}$ ", this implies the conclusion. +Let $M$ be the midpoint of $\overline{P Q}$. Since $\triangle A P Q$ is isosceles, + +$$ +\overline{A K M} \perp \overline{P Q} \Longrightarrow M K \cdot M A=M P^{2} +$$ + +by orthocenter properties. +So to summarize + +- The circle with diameter $\overline{P Q}$ is orthogonal to $\omega$. In other words, point $P$ lies on the polar of $Q$ with respect to $\omega$. +- The point $X=\overline{Q D} \cap \overline{P E}$ is on $\omega$. + +On the other hand, if we let $K^{\prime}=\overline{Q E} \cap \omega$, then by Brokard theorem on $X D K^{\prime} E$, the polar of $Q=\overline{X D} \cap \overline{K^{\prime} E}$ pass through $\overline{D K^{\prime}} \cap \overline{X E}$; this point must therefore be $P$ and $K^{\prime}=K$ as desired. + +『 S Solution to (b) by complex numbers (Yang Liu and Michael Ma). Let $M$ be the arc midpoint of $\widehat{B C}$. We use the standard arc midpoint configuration. We have that + +$$ +A=a^{2}, B=b^{2}, C=c^{2}, M=-b c, H=a^{2}+b^{2}+c^{2}, P=\frac{a^{2} c}{b}, Q=\frac{a^{2} b}{c} +$$ + +where $M$ is the arc midpoint of $\widehat{B C}$. By direct angle chasing we can verify that $\overline{M B} \| \overline{D H}$. Also, $D \in \overline{A B}$. Therefore, we can compute $D$ as follows. + +$$ +d+a^{2} b^{2} \bar{d}=a^{2}+b^{2} \text { and } \frac{d-h}{\bar{d}-\bar{h}}=-m b^{2}=b^{3} c \Longrightarrow d=\frac{a^{2}\left(a^{2} c+b^{2} c+c^{3}-b^{3}\right)}{c\left(b c+a^{2}\right)} . +$$ + +By symmetry, we have that + +$$ +e=\frac{a^{2}\left(a^{2} b+b c^{2}+b^{3}-c^{3}\right)}{b\left(b c+a^{2}\right)} . +$$ + +To finish, we want to show that the angle between $\overline{D P}$ and $\overline{E Q}$ is angle $A$. To show this, we compute $\frac{d-p}{e-q} / \overline{\frac{d-p}{e-q}}$. First, we compute + +$$ +\begin{aligned} +d-p & =\frac{a^{2}\left(a^{2} c+b^{2} c+c^{3}-b^{3}\right)}{c\left(b c+a^{2}\right)}-\frac{a^{2} c}{b} \\ +& =a^{2}\left(\frac{a^{2} c+b^{2} c+c^{3}-b^{3}}{c\left(b c+a^{2}\right)}-\frac{c}{b}\right)=\frac{a^{2}\left(a^{2} c-b^{3}\right)(b-c)}{b c\left(b c+a^{2}\right)} +\end{aligned} +$$ + +By symmetry, + +$$ +\frac{d-p}{e-q}=-\frac{a^{2} c-b^{3}}{a^{2} b-c^{3}} \Longrightarrow \frac{d-p}{e-q} / \overline{\frac{d-p}{e-q}}=\frac{a^{2} b^{3} c}{a^{2} b c^{3}}=\frac{b^{2}}{c^{2}} +$$ + +as desired. +\I Solution to (b) using untethered moving points (Zack Chroman). We work in the real projective plane $\mathbb{R P}^{2}$, and animate $C$ linearly on a fixed line through $A$. + +Recall: + +## Lemma (Zack's lemma) + +Suppose points $A, B$ have degree $d_{1}, d_{2}$, and there are $k$ values of $t$ for which $A=B$. Then line $A B$ has degree at most $d_{1}+d_{2}-k$. Similarly, if lines $\ell_{1}, \ell_{2}$ have degrees $d_{1}, d_{2}$, and there are $k$ values of $t$ for which $\ell_{1}=\ell_{2}$, then the intersection $\ell_{1} \cap \ell_{2}$ has degree at most $d_{1}+d_{2}-k$. + +Now, note that $H$ moves linearly in $C$ on line $B H$. Furthermore, angles $\angle A H E$, $\angle A H F$ are fixed, we get that $D$ and $E$ have degree 2 . One way to see this is using the lemma; $D$ lies on line $A B$, which is fixed, and line $H D$ passes through a point at infinity which is a constant rotation of the point at infinity on line $A H$, and therefore has degree 1. Then $D, E$ have degree at most $1+1-0=2$. + +Now, note that $P, Q$ move linearly in $C$. Both of these are because the circumcenter $O$ moves linearly in $C$, and $P, Q$ are reflections of $B, C$ in a line through $O$ with fixed direction, which also moves linearly. + +So by the lemma, the lines $P D, Q E$ have degree at most 3 . I claim they actually have degree 2; to show this it suffices to give an example of a choice of $C$ for which $P=D$ and one for which $Q=E$. But an easy angle chase shows that in the unique case when $P=B$, we get $D=B$ as well and thus $P=D$. Similarly when $Q=C, E=C$. It follows from the lemma that lines $P D, Q E$ have degree at most 2 . + +Let $\ell_{\infty}$ denote the line at infinity. I claim that the points $P_{1}=P D \cap \ell_{\infty}, P_{2}=Q E \cap \ell_{\infty}$ are projective in $C$. Since $\ell_{\infty}$ is fixed, it suffices to show by the lemma that there exists some value of $C$ for which $Q E=\ell_{\infty}$ and $P D=\ell_{\infty}$. But note that as $C \rightarrow \infty$, all four points $P, D, Q, E$ go to infinity. It follows that $P_{1}, P_{2}$ are projective in $C$. + +Then to finish, recall that we want to show that $\angle(P D, Q E)$ is constant. It suffices then to show that there's a constant rotation sending $P_{1}$ to $P_{2}$. Since $P_{1}, P_{2}$ are projective, it suffices to verify this for 3 values of $C$. + +We can take $C$ such that $\angle A B C=90, \angle A C B=90$, or $A B=A C$, and all three cases are easy to check. + +## §1.3 TSTST 2019/3, proposed by Nikolai Beluhov + +Available online at https://aops.com/community/p12608769. + +## Problem statement + +On an infinite square grid we place finitely many cars, which each occupy a single cell and face in one of the four cardinal directions. Cars may never occupy the same cell. It is given that the cell immediately in front of each car is empty, and moreover no two cars face towards each other (no right-facing car is to the left of a left-facing car within a row, etc.). In a move, one chooses a car and shifts it one cell forward to a vacant cell. Prove that there exists an infinite sequence of valid moves using each car infinitely many times. + +Let $S$ be any rectangle containing all the cars. Partition $S$ into horizontal strips of height 1 , and color them red and green in an alternating fashion. It is enough to prove all the cars may exit $S$. +![](https://cdn.mathpix.com/cropped/2024_11_19_f4f672a97b66743213a8g-09.jpg?height=880&width=1038&top_left_y=1096&top_left_x=512) + +To do so, we outline a five-stage plan for the cars. + +1. All vertical cars in a green cell may advance one cell into a red cell (or exit $S$ altogether), by the given condition. (This is the only place where the hypothesis about empty space is used!) +2. All horizontal cars on green cells may exit $S$, as no vertical cars occupy green cells. +3. All vertical cars in a red cell may advance one cell into a green cell (or exit $S$ altogether), as all green cells are empty. +4. All horizontal cars within red cells may exit $S$, as no vertical car occupy red cells. +5. The remaining cars exit $S$, as they are all vertical. The solution is complete. + +Remark (Author's comments). The solution I've given for this problem is so short and simple that it might appear at first to be about IMO 1 difficulty. I don't believe that's true! There are very many approaches that look perfectly plausible at first, and then fall apart in this or that twisted special case. + +Remark (Higher-dimensional generalization by author). The natural higher-dimensional generalization is true, and can be proved in largely the same fashion. For example, in three dimensions, one may let $S$ be a rectangular prism and partition $S$ into horizontal slabs and color them red and green in an alternating fashion. Stages 1, 3, and 5 generalize immediately, and stages 2 and 4 reduce to an application of the two-dimensional problem. In the same way, the general problem is handled by induction on the dimension. + +Remark (Historical comments). For $k>1$, we could consider a variant of the problem where cars are $1 \times k$ rectangles (moving parallel to the longer edge) instead of occupying single cells. In that case, if there are $2 k-1$ empty spaces in front of each car, the above proof works (with the red and green strips having height $k$ instead). On the other hand, at least $k$ empty spaces are necessary. We don't know the best constant in this case. + +## §2 Solutions to Day 2 + +## §2.1 TSTST 2019/4, proposed by Merlijn Staps + +Available online at https://aops.com/community/p12608513. + +## Problem statement + +Consider coins with positive real denominations not exceeding 1. Find the smallest $C>0$ such that the following holds: if we are given any 100 such coins with total value 50 , then we can always split them into two stacks of 50 coins each such that the absolute difference between the total values of the two stacks is at most $C$. + +The answer is $C=\frac{50}{51}$. The lower bound is obtained if we have 51 coins of value $\frac{1}{51}$ and 49 coins of value 1. (Alternatively, 51 coins of value $1-\frac{\varepsilon}{51}$ and 49 coins of value $\frac{\varepsilon}{49}$ works fine for $\varepsilon>0$.) We now present two (similar) proofs that this $C=\frac{50}{51}$ suffices. + +【 First proof (original). Let $a_{1} \leq \cdots \leq a_{100}$ denote the values of the coins in ascending order. Since the 51 coins $a_{50}, \ldots, a_{100}$ are worth at least $51 a_{50}$, it follows that $a_{50} \leq \frac{50}{51}$; likewise $a_{51} \geq \frac{1}{51}$. + +We claim that choosing the stacks with coin values + +$$ +a_{1}, a_{3}, \ldots, a_{49}, \quad a_{52}, a_{54}, \ldots, a_{100} +$$ + +and + +$$ +a_{2}, a_{4}, \ldots, a_{50}, \quad a_{51}, a_{53}, \ldots, a_{99} +$$ + +works. Let $D$ denote the (possibly negative) difference between the two total values. Then + +$$ +\begin{aligned} +D & =\left(a_{1}-a_{2}\right)+\cdots+\left(a_{49}-a_{50}\right)-a_{51}+\left(a_{52}-a_{53}\right)+\cdots+\left(a_{98}-a_{99}\right)+a_{100} \\ +& \leq 25 \cdot 0-\frac{1}{51}+24 \cdot 0+1=\frac{50}{51} +\end{aligned} +$$ + +Similarly, we have + +$$ +\begin{aligned} +D & =a_{1}+\left(a_{3}-a_{2}\right)+\cdots+\left(a_{49}-a_{48}\right)-a_{50}+\left(a_{52}-a_{51}\right)+\cdots+\left(a_{100}-a_{99}\right) \\ +& \geq 0+24 \cdot 0-\frac{50}{51}+25 \cdot 0=-\frac{50}{51} +\end{aligned} +$$ + +It follows that $|D| \leq \frac{50}{51}$, as required. + +【 Second proof (Evan Chen). Again we sort the coins in increasing order $0a_{i-1}+\frac{50}{51}$; obviously there is at most one such large gap. + +Claim - If there is a large gap, it must be $a_{51}>a_{50}+\frac{50}{51}$. + +Proof. If $i<50$ then we get $a_{50}, \ldots, a_{100}>\frac{50}{51}$ and the sum $\sum_{1}^{100} a_{i}>50$ is too large. Conversely if $i>50$ then we get $a_{1}, \ldots, a_{i-1}<\frac{1}{51}$ and the sum $\sum_{1}^{100} a_{i}<1 / 51 \cdot 51+49$ is too small. + +Now imagine starting with the coins $a_{1}, a_{3}, \ldots, a_{99}$, which have total value $S \leq 25$. We replace $a_{1}$ by $a_{2}$, then $a_{3}$ by $a_{4}$, and so on, until we replace $a_{99}$ by $a_{100}$. At the end of the process we have $S \geq 25$. Moreover, since we did not cross a large gap at any point, the quantity $S$ changed by at most $C=\frac{50}{51}$ at each step. So at some point in the process we need to have $25-C / 2 \leq S \leq 25+C / 2$, which proves $C$ works. + +## §2.2 TSTST 2019/5, proposed by Gunmay Handa + +Available online at https://aops.com/community/p12608496. + +## Problem statement + +Let $A B C$ be an acute triangle with orthocenter $H$ and circumcircle $\Gamma$. A line through $H$ intersects segments $A B$ and $A C$ at $E$ and $F$, respectively. Let $K$ be the circumcenter of $\triangle A E F$, and suppose line $A K$ intersects $\Gamma$ again at a point $D$. Prove that line $H K$ and the line through $D$ perpendicular to $\overline{B C}$ meet on $\Gamma$. + +We present several solutions. (There are more in the official packet; some are omitted here, which explains the numbering.) + +『 First solution (Andrew Gu). We begin with the following two observations. +Claim - Point $K$ lies on the radical axis of $(B E H)$ and $(C F H)$. +Proof. Actually we claim $\overline{K E}$ and $\overline{K F}$ are tangents. Indeed, + +$$ +\measuredangle H E K=90^{\circ}-\measuredangle E A F=90^{\circ}-\measuredangle B A C=\measuredangle H B E +$$ + +implying the result. Since $K E=K F$, this implies the result. + +Claim - The second intersection $M$ of $(B E H)$ and $(C F H)$ lies on $\Gamma$. +Proof. By Miquel's theorem on $\triangle A E F$ with $H \in \overline{E F}, B \in \overline{A E}, C \in \overline{A F}$. +![](https://cdn.mathpix.com/cropped/2024_11_19_f4f672a97b66743213a8g-13.jpg?height=806&width=632&top_left_y=1613&top_left_x=712) + +In particular, $M, H, K$ are collinear. Let $X$ be on $\Gamma$ with $\overline{D X} \perp \overline{B C}$; we then wish to show $X$ lies on the line $M H K$ we found. This is angle chasing: compute + +$$ +\measuredangle X M B=\measuredangle X D B=90^{\circ}-\measuredangle D B C=90^{\circ}-\measuredangle D A C +$$ + +$$ +=90^{\circ}-\measuredangle K A F=\measuredangle F E A=\measuredangle H E B=\measuredangle H M B +$$ + +as needed. + +II Second solution (Ankan Bhattacharya). We let $D^{\prime}$ be the second intersection of $\overline{E F}$ with $(B H C)$ and redefine $D$ as the reflection of $D^{\prime}$ across $\overline{B C}$. We will first prove that this point $D$ coincides with the point $D$ given in the problem statement. The idea is that: + +Claim - $A$ is the $D$-excenter of $\triangle D E F$. + +Proof. We contend $B E D^{\prime} D$ is cyclic. This follows by angle chasing: + +$$ +\begin{aligned} +\measuredangle D^{\prime} D B & =\measuredangle B D^{\prime} D=\measuredangle D^{\prime} B C+90^{\circ}=\measuredangle D^{\prime} H C+90^{\circ} \\ +& =\measuredangle D^{\prime} H C+\measuredangle(H C, A B)=\measuredangle\left(D^{\prime} H, A B\right)=\measuredangle D^{\prime} E B +\end{aligned} +$$ + +Now as $B D=B D^{\prime}$, we obtain $\overline{B E A}$ externally bisects $\angle D E D^{\prime} \cong \angle D E F$. Likewise $\overline{F A}$ externally bisects $\angle D F E$, so $A$ is the $D$-excenter of $\triangle D E F$. + +Hence, by the so-called "Fact 5 ", point $K$ lies on $\overline{D A}$, so this point $D$ is the one given in the problem statement. +![](https://cdn.mathpix.com/cropped/2024_11_19_f4f672a97b66743213a8g-14.jpg?height=778&width=712&top_left_y=1256&top_left_x=678) + +Now choose point $X$ on $(A B C)$ satisfying $\overline{D X} \perp \overline{B C}$. +Claim - Point $K$ lies on line $H X$. + +Proof. Clearly $A H D^{\prime} X$ is a parallelogram. By Ptolemy on $D E K F$, + +$$ +\frac{K D}{K A}=\frac{K D}{K E}=\frac{D E+D F}{E F} +$$ + +On the other hand, if we let $r_{D}$ denote the $D$-exradius of $\triangle D E F$ then + +$$ +\frac{X D}{X D^{\prime}}=\frac{[D E X]+[D F X]}{[X E F]}=\frac{[D E X]+[D F X]}{[A E F]}=\frac{D E \cdot r_{D}+D F \cdot r_{D}}{E F \cdot r_{D}}=\frac{D E+D F}{E F} +$$ + +Thus + +$$ +[A K X]=\frac{K A}{K D} \cdot[D K X]=\frac{K A}{K D} \cdot \frac{X D}{X D^{\prime}} \cdot\left[K D^{\prime} X\right]=\left[D^{\prime} K X\right] +$$ + +This is sufficient to prove $K$ lies on $\overline{H X}$. +The solution is complete: $X$ is the desired concurrency point. + +I Fourth solution, complex numbers with spiral similarity (Evan Chen). First if $\overline{A D} \perp \overline{B C}$ there is nothing to prove, so we assume this is not the case. Let $W$ be the antipode of $D$. Let $S$ denote the second intersection of $(A E F)$ and $(A B C)$. Consider the spiral similarity sending $\triangle S E F$ to $\triangle S B C$ : + +- It maps $H$ to a point $G$ on line $B C$, +- It maps $K$ to $O$. +- It maps the $A$-antipode of $\triangle A E F$ to $D$. +- Hence (by previous two observations) it maps $A$ to $W$. +- Also, the image of line $A D$ is line $W O$, which does not coincide with line $B C$ (as $O$ does not lie on line $B C$ ). + +Therefore, $K$ is the unique point on line $\overline{A D}$ for one can get a direct similarity + +$$ +\triangle A K H \sim \triangle W O G +$$ + +for some point $G$ lying on line $\overline{B C}$. +![](https://cdn.mathpix.com/cropped/2024_11_19_f4f672a97b66743213a8g-15.jpg?height=804&width=775&top_left_y=1477&top_left_x=646) + +On the other hand, let us re-define $K$ as $\overline{X H} \cap \overline{A D}$. We will show that the corresponding $G$ making $(\triangle)$ true lies on line $B C$. + +We apply complex numbers with $\Gamma$ the unit circle, with $a, b, c, d$ taking their usual meanings, $H=a+b+c, X=-b c / d$, and $W=-d$. Then point $K$ is supposed to satisfy + +$$ +k+a d \bar{k}=a+d +$$ + +$$ +\begin{aligned} +\frac{k+\frac{b c}{d}}{a+b+c+\frac{b c}{d}} & =\frac{\bar{k}+\frac{d}{b c}}{\frac{1}{a}+\frac{1}{b}+\frac{1}{c}+\frac{d}{b c}} \\ +\Longleftrightarrow \frac{\frac{1}{a}+\frac{1}{b}+\frac{1}{c}+\frac{d}{b c}}{a+b+c+\frac{b c}{d}}\left(k+\frac{b c}{d}\right) & =\bar{k}+\frac{d}{b c} +\end{aligned} +$$ + +Adding $a d$ times the last line to the first line and cancelling $a d \bar{k}$ now gives + +$$ +\left(a d \cdot \frac{\frac{1}{a}+\frac{1}{b}+\frac{1}{c}+\frac{d}{b c}}{a+b+c+\frac{b c}{d}}+1\right) k=a+d+\frac{a d^{2}}{b c}-a b c \cdot \frac{\frac{1}{a}+\frac{1}{b}+\frac{1}{c}+\frac{d}{b c}}{a+b+c+\frac{b c}{d}} +$$ + +or + +$$ +\begin{aligned} +\left(a d\left(\frac{1}{a}+\frac{1}{b}+\frac{1}{c}+\frac{d}{b c}\right)+a+b+c+\frac{b c}{d}\right) k & =\left(a+b+c+\frac{b c}{d}\right)\left(a+d+\frac{a d^{2}}{b c}\right) \\ +& -a b c \cdot\left(\frac{1}{a}+\frac{1}{b}+\frac{1}{c}+\frac{d}{b c}\right) . +\end{aligned} +$$ + +We begin by simplifying the coefficient of $k$ : + +$$ +\begin{aligned} +a d\left(\frac{1}{a}+\frac{1}{b}+\frac{1}{c}+\frac{d}{b c}\right)+a+b+c+\frac{b c}{d} & =a+b+c+d+\frac{b c}{d}+\frac{a d}{b}+\frac{a d}{c}+\frac{a d^{2}}{b c} \\ +& =a+\frac{b c}{d}+\left(1+\frac{a d}{b c}\right)(b+c+d) \\ +& =\frac{a d+b c}{b c d}[b c+d(b+c+d)] \\ +& =\frac{(a d+b c)(d+b)(d+c)}{b c d} +\end{aligned} +$$ + +Meanwhile, the right-hand side expands to + +$$ +\begin{aligned} +\mathrm{RHS}= & \left(a+b+c+\frac{b c}{d}\right)\left(a+d+\frac{a d^{2}}{b c}\right)-a b c \cdot\left(\frac{1}{a}+\frac{1}{b}+\frac{1}{c}+\frac{d}{b c}\right) \\ += & \left(a^{2}+a b+a c+\frac{a b c}{d}\right)+(d a+d b+d c+b c) \\ +& +\left(\frac{a^{2} d^{2}}{b c}+\frac{a d^{2}}{c}+\frac{a d^{2}}{b}+a d\right)-(a b+b c+c a+a d) \\ += & a^{2}+d(a+b+c)+\frac{a b c}{d}+\frac{a^{2} d^{2}}{b c}+\frac{a d^{2}}{b}+\frac{a d^{2}}{c} \\ += & a^{2}+\frac{a b c}{d}+d(a+b+c) \cdot \frac{a d+b c}{b c} \\ += & \frac{a d+b c}{b c d}\left[a b c+d^{2}(a+b+c)\right] . +\end{aligned} +$$ + +Therefore, we get + +$$ +k=\frac{a b c+d^{2}(a+b+c)}{(d+b)(d+c)} +$$ + +In particular, + +$$ +\begin{aligned} +k-a & =\frac{a b c+d^{2}(a+b+c)-a(d+b)(d+c)}{(d+b)(d+c)} \\ +& =\frac{d^{2}(b+c)-d a(b+c)}{(d+b)(d+c)}=\frac{d(b+c)(d-a)}{(d+b)(d+c)} +\end{aligned} +$$ + +Now the corresponding point $G$ obeying $(\Omega)$ satisfies + +$$ +\begin{aligned} +\frac{g-(-d)}{0-(-d)} & =\frac{(a+b+c)-a}{k-a} \\ +\Longrightarrow g & =-d+\frac{d(b+c)}{k-a} \\ +& =-d+\frac{(d+b)(d+c)}{d-a}=\frac{d b+d c+b c+a d}{d-a} . \\ +\Longrightarrow b c \bar{g} & =\frac{b c \cdot \frac{a c+a b+a d+b c}{a b c d}}{\frac{a-d}{a d}}=-\frac{a b+a c+a d+b c}{d-a} . \\ +\Longrightarrow g+b c \bar{g} & =\frac{(d-a)(b+c)}{d-a}=b+c . +\end{aligned} +$$ + +Hence $G$ lies on $B C$ and this completes the proof. + +【 Seventh solution using moving points (Zack Chroman). We state the converse of the problem as follows: + +Take a point $D$ on $\Gamma$, and let $G \in \Gamma$ such that $\overline{D G} \perp \overline{B C}$. Then define $K$ to lie on $\overline{G H}, \overline{A D}$, and take $L \in \overline{A D}$ such that $K$ is the midpoint of $\overline{A L}$. Then if we define $E$ and $F$ as the projections of $L$ onto $\overline{A B}$ and $\overline{A C}$ we want to show that $E, H, F$ are collinear. + +It's clear that solving this problem will solve the original. In fact we will show later that each line $E F$ through $H$ corresponds bijectively to the point $D$. + +We animate $D$ projectively on $\Gamma$ (hence $\operatorname{deg} D=2$ ). Since $D \mapsto G$ is a projective map $\Gamma \rightarrow \Gamma$, it follows $\operatorname{deg} G=2$. By Zack's lemma, $\operatorname{deg}(\overline{A D}) \leq 0+2-1=1$ (since $D$ can coincide with $A$ ), and $\operatorname{deg}(\overline{H G}) \leq 0+2-0=2$. So again by Zack's lemma, $\operatorname{deg} K \leq 1+2-1=2$, since lines $A D$ and $G H$ can coincide once if $D$ is the reflection of $H$ over $\overline{B C}$. It follows $\operatorname{deg} L=2$, since it is obtained by dilating $K$ by a factor of 2 across the fixed point $A$. + +Let $\infty_{C}$ be the point at infinity on the line perpendicular to $A C$, and similarly $\infty_{B}$. Then + +$$ +F=\overline{A C} \cap \overline{\infty_{C} L}, \quad E=\overline{A B} \cap \overline{\infty_{B} L} . +$$ + +We want to use Zack's lemma again on line $\overline{\infty_{B} L}$. Consider the case $G=B$; we get $\overline{H G} \| \overline{A D}$, so $A D G H$ is a parallelogram, and then $K=L=\infty_{B}$. Thus there is at least one $t$ where $L=\infty_{B}$ and by Zack's lemma we get $\operatorname{deg}\left(\overline{\infty_{B} L}\right) \leq 0+2-1=1$. Again by Zack's lemma, we conclude $\operatorname{deg} E \leq 0+1-0=1$. Similarly, $\operatorname{deg} F \leq 1$. + +We were aiming to show $E, F, H$ collinear which is a condition of degree at most $1+1+0=2$. So it suffices to verify the problem for three distinct choices of $D$. + +- If $D=A$, then line $G H$ is line $A H$, and $L=\overline{A D} \cap \overline{A H}=A$. So $E=F=A$ and the statement is true. +- If $D=B, G$ is the antipode of $C$ on $\Gamma$. Then $K=\overline{H G} \cap \overline{A D}$ is the midpoint of $\overline{A B}$, so $L=B$. Then $E=B$ and $F$ is the projection of $B$ onto $A C$, so $E, H, F$ collinear. +- We finish similarly when $D=C$. + +This completes the proof. + +Remark. Less careful approaches are possible which give a worse bound on the degrees, requiring to check (say) five choices of $D$ instead. We present the most careful one showing $\operatorname{deg} D=2$ for instructional reasons, but the others may be easier to find. + +## §2.3 TSTST 2019/6, proposed by Nikolai Beluhov + +Available online at https://aops.com/community/p12608536. + +## Problem statement + +Suppose $P$ is a polynomial with integer coefficients such that for every positive integer $n$, the sum of the decimal digits of $|P(n)|$ is not a Fibonacci number. Must $P$ be constant? + +The answer is yes, $P$ must be constant. By $S(n)$ we mean the sum of the decimal digits of $|n|$. + +We need two claims. +Claim - If $P(x) \in \mathbb{Z}[x]$ is nonconstant with positive leading coefficient, then there exists an integer polynomial $F(x)$ such that all coefficients of $P \circ F$ are positive except for the second one, which is negative. + +Proof. We will actually construct a cubic $F$. We call a polynomial good if it has the property. + +First, consider $T_{0}(x)=x^{3}+x+1$. Observe that in $T_{0}^{\operatorname{deg} P}$, every coefficient is strictly positive, except for the second one, which is zero. + +Then, let $T_{1}(x)=x^{3}-\frac{1}{D} x^{2}+x+1$. Using continuity as $D \rightarrow \infty$, it follows that if $D$ is large enough (in terms of $\operatorname{deg} P$ ), then $T_{1}^{\operatorname{deg} P}$ is good, with $-\frac{3}{D} x^{3 \operatorname{deg} P-1}$ being the only negative coefficient. + +Finally, we can let $F(x)=C T_{1}(x)$ where $C$ is a sufficiently large multiple of $D$ (in terms of the coefficients of $P$ ); thus the coefficients of $\left(C T_{1}(x)\right)^{\operatorname{deg} P}$ dominate (and are integers), as needed. + +Claim - There are infinitely many Fibonacci numbers in each residue class modulo 9. + +Proof. Note the Fibonacci sequence is periodic modulo 9 (indeed it is periodic modulo any integer). Moreover (allowing negative indices), + +$$ +\begin{aligned} +F_{0}=0 & \equiv 0 \quad(\bmod 9) \\ +F_{1}=1 & \equiv 1 \quad(\bmod 9) \\ +F_{3}=2 & \equiv 2 \quad(\bmod 9) \\ +F_{4}=3 & \equiv 3 \quad(\bmod 9) \\ +F_{7}=13 & \equiv 4 \quad(\bmod 9) \\ +F_{5}=5 & \equiv 5 \quad(\bmod 9) \\ +F_{-4}=-3 & \equiv 6 \quad(\bmod 9) \\ +F_{9}=34 & \equiv 7 \quad(\bmod 9) \\ +F_{6}=8 & \equiv 8 \quad(\bmod 9) . +\end{aligned} +$$ + +We now show how to solve the problem with the two claims. WLOG $P$ satisfies the conditions of the first claim, and choose $F$ as above. Let + +$$ +P(F(x))=c_{N} x^{N}-c_{N-1} x^{N-1}+c_{N-2} x^{N-2}+\cdots+c_{0} +$$ + +where $c_{i}>0$ (and $N=3 \operatorname{deg} P$ ). Then if we select $x=10^{e}$ for $e$ large enough (say $\left.x>10 \max _{i} c_{i}\right)$, the decimal representation $P\left(F\left(10^{e}\right)\right)$ consists of the concatenation of + +- the decimal representation of $c_{N}-1$, +- the decimal representation of $10^{e}-c_{N-1}$ +- the decimal representation of $c_{N-2}$, with several leading zeros, +- the decimal representation of $c_{N-3}$, with several leading zeros, +- ... +- the decimal representation of $c_{0}$, with several leading zeros. +(For example, if $P(F(x))=15 x^{3}-7 x^{2}+4 x+19$, then $P(F(1000))=14,993,004,019$.) Thus, the sum of the digits of this expression is equal to + +$$ +S\left(P\left(F\left(10^{e}\right)\right)\right)=9 e+k +$$ + +for some constant $k$ depending only on $P$ and $F$, independent of $e$. But this will eventually hit a Fibonacci number by the second claim, contradiction. + +Remark. It is important to control the number of negative coefficients in the created polynomial. If one tries to use this approach on a polynomial $P$ with $m>0$ negative coefficients, then one would require that the Fibonacci sequence is surjective modulo $9 m$ for any $m>1$, which is not true: for example the Fibonacci sequence avoids all numbers congruent to $4 \bmod 11($ and thus $4 \bmod 99)$. + +In bases $b$ for which surjectivity modulo $b-1$ fails, the problem is false. For example, $P(x)=11 x+4$ will avoid all Fibonacci numbers if we take sum of digits in base 12, since that base-12 sum is necessarily $4(\bmod 11)$, hence not a Fibonacci number. + +## §3 Solutions to Day 3 + +## §3.1 TSTST 2019/7, proposed by Ankan Bhattacharya + +Available online at https://aops.com/community/p12608512. + +## Problem statement + +Let $f: \mathbb{Z} \rightarrow\left\{1,2, \ldots, 10^{100}\right\}$ be a function satisfying + +$$ +\operatorname{gcd}(f(x), f(y))=\operatorname{gcd}(f(x), x-y) +$$ + +for all integers $x$ and $y$. Show that there exist positive integers $m$ and $n$ such that $f(x)=\operatorname{gcd}(m+x, n)$ for all integers $x$. + +Let $\mathcal{P}$ be the set of primes not exceeding $10^{100}$. For each $p \in \mathcal{P}$, let $e_{p}=\max _{x} \nu_{p}(f(x))$ and let $c_{p} \in \operatorname{argmax}_{x} \nu_{p}(f(x))$. + +We show that this is good enough to compute all values of $x$, by looking at the exponent at each individual prime. + +Claim - For any $p \in \mathcal{P}$, we have + +$$ +\nu_{p}(f(x))=\min \left(\nu_{p}\left(x-c_{p}\right), e_{p}\right) +$$ + +Proof. Note that for any $x$, we have + +$$ +\operatorname{gcd}\left(f\left(c_{p}\right), f(x)\right)=\operatorname{gcd}\left(f\left(c_{p}\right), x-c_{p}\right) +$$ + +We then take $\nu_{p}$ of both sides and recall $\nu_{p}(f(x)) \leq \nu_{p}\left(f\left(c_{p}\right)\right)=e_{p}$; this implies the result. + +This essentially determines $f$, and so now we just follow through. Choose $n$ and $m$ such that + +$$ +\begin{aligned} +n & =\prod_{p \in \mathcal{P}} p^{e_{p}} \\ +m & \equiv-c_{p} \quad\left(\bmod p^{e_{p}}\right) \quad \forall p \in \mathcal{P} +\end{aligned} +$$ + +the latter being possible by Chinese remainder theorem. Then, from the claim we have + +$$ +\begin{aligned} +f(x) & =\prod_{p \in \mathcal{P}} p^{\nu_{p}(f(x))}=\prod_{p \mid n} p^{\min \left(\nu_{p}\left(x-c_{p}\right), e_{p}\right)} \\ +& =\prod_{p \mid n} p^{\min \left(\nu_{p}(x+m), \nu_{p}(n)\right)}=\operatorname{gcd}(x+m, n) +\end{aligned} +$$ + +for every $x \in \mathbb{Z}$, as desired. +Remark. The functions $f(x)=x$ and $f(x)=|2 x-1|$ are examples satisfying the gcd equation (the latter always being strictly positive). Hence the hypothesis $f$ bounded cannot be dropped. + +Remark. The pair $(m, n)$ is essentially unique: every other pair is obtained by shifting $m$ by a multiple of $n$. Hence there is not really any choice in choosing $m$ and $n$. + +## §3.2 TSTST 2019/8, proposed by Ankan Bhattacharya + +Available online at https://aops.com/community/p12608780. + +## Problem statement + +Let $\mathcal{S}$ be a set of 16 points in the plane, no three collinear. Let $\chi(\mathcal{S})$ denote the number of ways to draw 8 line segments with endpoints in $\mathcal{S}$, such that no two drawn segments intersect, even at endpoints. Find the smallest possible value of $\chi(\mathcal{S})$ across all such $\mathcal{S}$. + +The answer is 1430 . In general, we prove that with $2 n$ points the answer is the $n^{\text {th }}$ Catalan number $C_{n}=\frac{1}{n+1}\binom{2 n}{n}$. + +First of all, it is well-known that if $\mathcal{S}$ is a convex $2 n$-gon, then $\chi(\mathcal{S})=C_{n}$. +It remains to prove the lower bound. We proceed by (strong) induction on $n$, with the base case $n=0$ and $n=1$ clear. Suppose the statement is proven for $0,1, \ldots, n$ and consider a set $\mathcal{S}$ with $2(n+1)$ points. + +Let $P$ be a point on the convex hull of $\mathcal{S}$, and label the other $2 n+1$ points $A_{1}, \ldots, A_{2 n+1}$ in order of angle from $P$. + +Consider drawing a segment $\overline{P A_{2 k+1}}$. This splits the $2 n$ remaining points into two halves $\mathcal{U}$ and $\mathcal{V}$, with $2 k$ and $2(n-k)$ points respectively. +![](https://cdn.mathpix.com/cropped/2024_11_19_f4f672a97b66743213a8g-23.jpg?height=581&width=815&top_left_y=1297&top_left_x=626) + +Note that by choice of $P$, no segment in $\mathcal{U}$ can intersect a segment in $\mathcal{V}$. By the inductive hypothesis, + +$$ +\chi(\mathcal{U}) \geq C_{k} \quad \text { and } \quad \chi(\mathcal{V}) \geq C_{n-k} +$$ + +Thus, drawing $\overline{P A_{2 k+1}}$, we have at least $C_{k} C_{n-k}$ ways to complete the drawing. Over all choices of $k$, we obtain + +$$ +\chi(\mathcal{S}) \geq C_{0} C_{n}+\cdots+C_{n} C_{0}=C_{n+1} +$$ + +as desired. +Remark. It is possible to show directly from the lower bound proof that convex $2 n$-gons achieve the minimum: indeed, every inequality is sharp, and no segment $\overline{P A_{2 k}}$ can be drawn (since this splits the rest of the points into two halves with an odd number of points, and no crossing segment can be drawn). + +Bobby Shen points out that in the case of 6 points, a regular pentagon with its center also achieves equality, so this is not the only equality case. + +Remark. The result that $\chi(S) \geq 1$ for all $S$ is known (consider the choice of 8 segments with smallest sum), and appeared on Putnam 1979. However, it does not seem that knowing this gives an advantage for this problem, since the answer is much larger than 1. + +## §3.3 TSTST 2019/9, proposed by Ankan Bhattacharya + +Available online at https://aops.com/community/p12608472. + +## Problem statement + +Let $A B C$ be a triangle with incenter $I$. Points $K$ and $L$ are chosen on segment $B C$ such that the incircles of $\triangle A B K$ and $\triangle A B L$ are tangent at $P$, and the incircles of $\triangle A C K$ and $\triangle A C L$ are tangent at $Q$. Prove that $I P=I Q$. + +We present two solutions. +【 First solution, mostly elementary (original). Let $I_{B}, J_{B}, I_{C}, J_{C}$ be the incenters of $\triangle A B K, \triangle A B L, \triangle A C K, \triangle A C L$ respectively. +![](https://cdn.mathpix.com/cropped/2024_11_19_f4f672a97b66743213a8g-25.jpg?height=515&width=1332&top_left_y=973&top_left_x=359) + +We begin with the following claim which does not depend on the existence of tangency points $P$ and $Q$. + +Claim - Lines $B C, I_{B} J_{C}, J_{B} I_{C}$ meet at a point $R$ (possibly at infinity). + +Proof. By rotating by $\frac{1}{2} \angle A$ we have the equality + +$$ +A\left(B I ; I_{B} J_{B}\right)=A\left(I C ; I_{C} J_{C}\right) . +$$ + +It follows $\left(B I ; I_{B} J_{B}\right)=\left(I C ; I_{C} J_{C}\right)=\left(C I ; J_{C} I_{C}\right)$. (One could also check directly that both cross ratios equal $\frac{\sin \angle B A K / 2}{\sin \angle C A K / 2} \div \frac{\sin \angle B A L / 2}{\sin \angle C A L / 2}$, rather than using rotation.) + +Therefore, the concurrence follows from the so-called prism lemma on $\overline{I B I_{B} J_{B}}$ and $\overline{I C J_{C} I_{C}}$. + +Remark (Nikolai Beluhov). This result is known; it appears as 4.5.32 in Akopyan's Geometry in Figures. The cross ratio is not necessary to prove this claim: it can be proven by length chasing with circumscribed quadrilaterals. (The generalization mentioned later also admits a trig-free proof for the analogous step.) + +We now bring $P$ and $Q$ into the problem. +Claim - Line $P Q$ also passes through $R$. + +Proof. Note $\left(B P ; I_{B} J_{B}\right)=-1=\left(C Q ; J_{C} I_{C}\right)$, so the conclusion again follows by prism lemma. + +We are now ready to complete the proof. Point $R$ is the exsimilicenter of the incircles of $\triangle A B K$ and $\triangle A C L$, so $\frac{P I_{B}}{R I_{B}}=\frac{Q J_{C}}{R J_{C}}$. Now by Menelaus, + +$$ +\frac{I_{B} P}{P I} \cdot \frac{I Q}{Q J_{C}} \cdot \frac{J_{C} R}{R I_{B}}=-1 \Longrightarrow I P=I Q +$$ + +Remark (Author's comments on drawing the diagram). Drawing the diagram directly is quite difficult. If one draws $\triangle A B C$ first, they must locate both $K$ and $L$, which likely involves some trial and error due to the complex interplay between the two points. + +There are alternative simpler ways. For example, one may draw $\triangle A K L$ first; then the remaining points $B$ and $C$ are not related and the task is much simpler (though some trial and error is still required). + +In fact, by breaking symmetry, we may only require one application of guesswork. Start by drawing $\triangle A B K$ and its incircle; then the incircle of $\triangle A B L$ may be constructed, and so point $L$ may be drawn. Thus only the location of point $C$ needs to be guessed. I would be interested in a method to create a general diagram without any trial and error. + +【 Second solution, inversion (Nikolai Beluhov). As above, the lines $B C, I_{B} J_{C}, J_{B} I_{C}$ meet at some point $R$ (possibly at infinity). Let $\omega_{1}, \omega_{2}, \omega_{3}, \omega_{4}$ be the incircles of $\triangle A B K$, $\triangle A C L, \triangle A B L$, and $\triangle A C K$. + +Claim - There exists an inversion $\iota$ at $R$ swapping $\left\{\omega_{1}, \omega_{2}\right\}$ and $\left\{\omega_{3}, \omega_{4}\right\}$. + +Proof. Consider the inversion at $R$ swapping $\omega_{1}$ and $\omega_{2}$. Since $\omega_{1}$ and $\omega_{3}$ are tangent, the image of $\omega_{3}$ is tangent to $\omega_{2}$ and is also tangent to $B C$. The circle $\omega_{4}$ is on the correct side of $\omega_{3}$ to be this image. + +Claim - Circles $\omega_{1}, \omega_{2}, \omega_{3}, \omega_{4}$ share a common radical center. + +Proof. Let $\Omega$ be the circle with center $R$ fixed under $\iota$, and let $k$ be the circle through $P$ centered at the radical center of $\Omega, \omega_{1}, \omega_{3}$. + +Then $k$ is actually orthogonal to $\Omega, \omega_{1}, \omega_{3}$, so $k$ is fixed under $\iota$ and $k$ is also orthogonal to $\omega_{2}$ and $\omega_{4}$. Thus the center of $k$ is the desired radical center. + +The desired statement immediately follows. Indeed, letting $S$ be the radical center, it follows that $\overline{S P}$ and $\overline{S Q}$ are the common internal tangents to $\left\{\omega_{1}, \omega_{3}\right\}$ and $\left\{\omega_{2}, \omega_{4}\right\}$. + +Since $S$ is the radical center, $S P=S Q$. In light of $\angle S P I=\angle S Q I=90^{\circ}$, it follows that $I P=I Q$, as desired. + +Remark (Nikolai Beluhov). There exists a circle tangent to all four incircles, because circle $k$ is orthogonal to all four, and line $B C$ is tangent to all four; thus the inverse of line $B C$ in $k$ is a circle tangent to all four incircles. + +The amusing thing here is that Casey's theorem is completely unhelpful for proving this fact: all it can tell us is that there is a line or circle tangent to these incircles, and line $B C$ already satisfies this property. + +Remark (Generalization by Nikolai Beluhov). The following generalization holds: +Let $A B C D$ be a quadrilateral circumscribed about a circle with center $I$. A line through $A$ meets $\overrightarrow{B C}$ and $\overrightarrow{D C}$ at $K$ and $L$; another line through $A$ meets $\overrightarrow{B C}$ and $\overrightarrow{D C}$ at $M$ and $N$. Suppose that the incircles of $\triangle A B K$ and $\triangle A B M$ are tangent at $P$, and the incircles of $\triangle A C L$ and $\triangle A C N$ are tangent at $Q$. Prove that $I P=I Q$. + +The first approach can be modified to the generalization. There is an extra initial step required: by Monge, the exsimilicenter of the incircles of $\triangle A B K$ and $\triangle A D N$ lies on line $B D$; likewise for the incircles of $\triangle A B L$ and $\triangle A D M$. Now one may prove using the same trig approach that these pairs of incircles have a common exsimilicenter, and the rest of the solution plays out similarly. The second approach can also be modified in the same way, once we obtain that a common exsimilicenter exists. (Thus in the generalization, it seems we also get there exists a circle tangent to all four incircles.) + diff --git a/USA_TSTST/md/en-sols-TSTST-2020.md b/USA_TSTST/md/en-sols-TSTST-2020.md new file mode 100644 index 0000000000000000000000000000000000000000..05ae2a6e9587529e7bcaaf590ddaf2b3a0108eb4 --- /dev/null +++ b/USA_TSTST/md/en-sols-TSTST-2020.md @@ -0,0 +1,535 @@ +# USA TSTST 2020 Solutions
United States of America - TST Selection Test
Ankan Bhattacharya and Evan Chen
62 $2^{\text {nd }}$ IMO 2021 Russia and $10^{\text {th }}$ EGMO 2021 Georgia + +## Contents + +0 Problems ..... 2 +1 Solutions to Day 1 ..... 4 +1.1 TSTST 2020/1, proposed by Ankan Bhattacharya ..... 4 +1.2 TSTST 2020/2, proposed by Zack Chroman, Daniel Liu ..... 6 +1.3 TSTST 2020/3, proposed by Evan Chen, Danielle Wang ..... 8 +2 Solutions to Day 2 ..... 10 +2.1 TSTST 2020/4, proposed by Yang Liu ..... 10 +2.2 TSTST 2020/5, proposed by Ashwin Sah, Mehtaab Sawhney ..... 12 +2.3 TSTST 2020/6, proposed by Andrew Gu . ..... 14 +3 Solutions to Day 3 ..... 16 +3.1 TSTST 2020/7, proposed by Ankan Bhattacharya ..... 16 +3.2 TSTST 2020/8, proposed by Ankan Bhattacharya ..... 18 +3.3 TSTST 2020/9, proposed by Nikolai Beluhov ..... 20 + +## §0 Problems + +1. Let $a, b, c$ be fixed positive integers. There are $a+b+c$ ducks sitting in a circle, one behind the other. Each duck picks either rock, paper, or scissors, with $a$ ducks picking rock, $b$ ducks picking paper, and $c$ ducks picking scissors. + +A move consists of an operation of one of the following three forms: + +- If a duck picking rock sits behind a duck picking scissors, they switch places. +- If a duck picking paper sits behind a duck picking rock, they switch places. +- If a duck picking scissors sits behind a duck picking paper, they switch places. + +Determine, in terms of $a, b$, and $c$, the maximum number of moves which could take place, over all possible initial configurations. +2. Let $A B C$ be a scalene triangle with incenter $I$. The incircle of $A B C$ touches $\overline{B C}$, $\overline{C A}, \overline{A B}$ at points $D, E, F$, respectively. Let $P$ be the foot of the altitude from $D$ to $\overline{E F}$, and let $M$ be the midpoint of $\overline{B C}$. The rays $A P$ and $I P$ intersect the circumcircle of triangle $A B C$ again at points $G$ and $Q$, respectively. Show that the incenter of triangle $G Q M$ coincides with $D$. +3. We say a nondegenerate triangle whose angles have measures $\theta_{1}, \theta_{2}, \theta_{3}$ is quirky if there exists integers $r_{1}, r_{2}, r_{3}$, not all zero, such that + +$$ +r_{1} \theta_{1}+r_{2} \theta_{2}+r_{3} \theta_{3}=0 +$$ + +Find all integers $n \geq 3$ for which a triangle with side lengths $n-1, n, n+1$ is quirky. +4. Find all pairs of positive integers $(a, b)$ satisfying the following conditions: +(i) $a$ divides $b^{4}+1$, +(ii) $b$ divides $a^{4}+1$, +(iii) $\lfloor\sqrt{a}\rfloor=\lfloor\sqrt{b}\rfloor$. +5. Let $\mathbb{N}^{2}$ denote the set of ordered pairs of positive integers. A finite subset $S$ of $\mathbb{N}^{2}$ is stable if whenever $(x, y)$ is in $S$, then so are all points $\left(x^{\prime}, y^{\prime}\right)$ of $\mathbb{N}^{2}$ with both $x^{\prime} \leq x$ and $y^{\prime} \leq y$. +Prove that if $S$ is a stable set, then among all stable subsets of $S$ (including the empty set and $S$ itself), at least half of them have an even number of elements. +6. Let $A, B, C, D$ be four points such that no three are collinear and $D$ is not the orthocenter of triangle $A B C$. Let $P, Q, R$ be the orthocenters of $\triangle B C D, \triangle C A D$, $\triangle A B D$, respectively. Suppose that lines $A P, B Q, C R$ are pairwise distinct and are concurrent. Show that the four points $A, B, C, D$ lie on a circle. +7. Find all nonconstant polynomials $P(z)$ with complex coefficients for which all complex roots of the polynomials $P(z)$ and $P(z)-1$ have absolute value 1. +8. For every positive integer $N$, let $\sigma(N)$ denote the sum of the positive integer divisors of $N$. Find all integers $m \geq n \geq 2$ satisfying + +$$ +\frac{\sigma(m)-1}{m-1}=\frac{\sigma(n)-1}{n-1}=\frac{\sigma(m n)-1}{m n-1} . +$$ + +9. Ten million fireflies are glowing in $\mathbb{R}^{3}$ at midnight. Some of the fireflies are friends, and friendship is always mutual. Every second, one firefly moves to a new position so that its distance from each one of its friends is the same as it was before moving. This is the only way that the fireflies ever change their positions. No two fireflies may ever occupy the same point. +Initially, no two fireflies, friends or not, are more than a meter away. Following some finite number of seconds, all fireflies find themselves at least ten million meters away from their original positions. Given this information, find the greatest possible number of friendships between the fireflies. + +## §1 Solutions to Day 1 + +## §1.1 TSTST 2020/1, proposed by Ankan Bhattacharya + +Available online at https://aops.com/community/p18933796. + +## Problem statement + +Let $a, b, c$ be fixed positive integers. There are $a+b+c$ ducks sitting in a circle, one behind the other. Each duck picks either rock, paper, or scissors, with a ducks picking rock, $b$ ducks picking paper, and $c$ ducks picking scissors. + +A move consists of an operation of one of the following three forms: + +- If a duck picking rock sits behind a duck picking scissors, they switch places. +- If a duck picking paper sits behind a duck picking rock, they switch places. +- If a duck picking scissors sits behind a duck picking paper, they switch places. + +Determine, in terms of $a, b$, and $c$, the maximum number of moves which could take place, over all possible initial configurations. + +The maximum possible number of moves is $\max (a b, a c, b c)$. +First, we prove this is best possible. We define a feisty triplet to be an unordered triple of ducks, one of each of rock, paper, scissors, such that the paper duck is between the rock and scissors duck and facing the rock duck, as shown. (There may be other ducks not pictured, but the orders are irrelevant.) +![](https://cdn.mathpix.com/cropped/2024_11_19_a8f70a0d08e0809deba0g-04.jpg?height=1100&width=1098&top_left_y=1500&top_left_x=479) + +Claim - The number of feisty triplets decreases by $c$ if a paper duck swaps places with a rock duck, and so on. + +## Proof. Clear. + +Obviously the number of feisty triples is at most $a b c$ to start. Thus at most $\max (a b, b c, c a)$ moves may occur, since the number of feisty triplets should always be nonnegative, at which point no moves are possible at all. + +To see that this many moves is possible, assume WLOG $a=\min (a, b, c)$ and suppose we have $a$ rocks, $b$ papers, and $c$ scissors in that clockwise order. +![](https://cdn.mathpix.com/cropped/2024_11_19_a8f70a0d08e0809deba0g-05.jpg?height=1109&width=1052&top_left_y=750&top_left_x=505) + +Then, allow the scissors to filter through the papers while the rocks stay put. Each of the $b$ papers swaps with $c$ scissors, for a total of $b c=\max (a b, a c, b c)$ swaps. + +Remark (Common errors). One small possible mistake: it is not quite kösher to say that "WLOG $a \leq b \leq c$ " because the condition is not symmetric, only cyclic. Therefore in this solution we only assume $a=\min (a, b, c)$. + +It is true here that every pair of ducks swaps at most once, and some solutions make use of this fact. However, this fact implicitly uses the fact that $a, b, c>0$ and is false without this hypothesis. + +## §1.2 TSTST 2020/2, proposed by Zack Chroman, Daniel Liu + +Available online at https://aops.com/community/p18933557. + +## Problem statement + +Let $A B C$ be a scalene triangle with incenter $I$. The incircle of $A B C$ touches $\overline{B C}$, $\overline{C A}, \overline{A B}$ at points $D, E, F$, respectively. Let $P$ be the foot of the altitude from $D$ to $\overline{E F}$, and let $M$ be the midpoint of $\overline{B C}$. The rays $A P$ and $I P$ intersect the circumcircle of triangle $A B C$ again at points $G$ and $Q$, respectively. Show that the incenter of triangle $G Q M$ coincides with $D$. + +Refer to the figure below. +![](https://cdn.mathpix.com/cropped/2024_11_19_a8f70a0d08e0809deba0g-06.jpg?height=798&width=803&top_left_y=906&top_left_x=632) + +Claim - The point $Q$ is the Miquel point of $B F E C$. Also, $\overline{Q D}$ bisects $\angle B Q C$. +Proof. Inversion around the incircle maps line $E F$ to $(A I E F)$ and the nine-point circle of $\triangle D E F$ to the circumcircle of $\triangle A B C$ (as the midpoint of $\overline{E F}$ maps to $A$, etc.). This implies $P$ maps to $Q$; that is, $Q$ coincides with the second intersection of (AFIE) with $(A B C)$. This is the claimed Miquel point. + +The spiral similarity mentioned then gives $\frac{Q B}{B F}=\frac{Q C}{C E}$, so $\overline{Q D}$ bisects $\angle B Q C$. +Remark. The point $Q$ and its properties mentioned in the first claim have appeared in other references. See for example Canada 2007/5, ELMO 2010/6, HMMT 2016 T-10, USA TST 2017/2, USA TST 2019/6 for a few examples. + +Claim - We have $(Q G ; B C)=-1$, so in particular $\overline{G D}$ bisects $\angle B G C$. +Proof. Note that + +$$ +-1=(A I ; E F) \stackrel{Q}{=}(\overline{A Q} \cap \overline{E F}, P ; E, F) \stackrel{A}{=}(Q G ; B C) +$$ + +The last statement follows from Apollonian circle, or more bluntly $\frac{G B}{G C}=\frac{Q B}{Q C}=\frac{B D}{D C}$. +Hence $\overline{Q D}$ and $\overline{G D}$ are angle bisectors of $\angle B Q C$ and $\angle B G C$. However, $\overline{Q M}$ and $\overline{Q G}$ are isogonal in $\angle B Q C$ (as median and symmedian), and similarly for $\angle B G C$, as desired. + +## §1.3 TSTST 2020/3, proposed by Evan Chen, Danielle Wang + +Available online at https://aops.com/community/p18933954. + +## Problem statement + +We say a nondegenerate triangle whose angles have measures $\theta_{1}, \theta_{2}, \theta_{3}$ is quirky if there exists integers $r_{1}, r_{2}, r_{3}$, not all zero, such that + +$$ +r_{1} \theta_{1}+r_{2} \theta_{2}+r_{3} \theta_{3}=0 +$$ + +Find all integers $n \geq 3$ for which a triangle with side lengths $n-1, n, n+1$ is quirky. + +The answer is $n=3,4,5,7$. +We first introduce a variant of the $k$ th Chebyshev polynomials in the following lemma (which is standard, and easily shown by induction). + +## Lemma + +For each $k \geq 0$ there exists $P_{k}(X) \in \mathbb{Z}[X]$, monic for $k \geq 1$ and with degree $k$, such that + +$$ +P_{k}\left(X+X^{-1}\right) \equiv X^{k}+X^{-k} . +$$ + +The first few are $P_{0}(X) \equiv 2, P_{1}(X) \equiv X, P_{2}(X) \equiv X^{2}-2, P_{3}(X) \equiv X^{3}-3 X$. +Suppose the angles of the triangle are $\alpha<\beta<\gamma$, so the law of cosines implies that + +$$ +2 \cos \alpha=\frac{n+4}{n+1} \quad \text { and } \quad 2 \cos \gamma=\frac{n-4}{n-1} +$$ + +Claim - The triangle is quirky iff there exists $r, s \in \mathbb{Z}_{\geq 0}$ not both zero such that + +$$ +\cos (r \alpha)= \pm \cos (s \gamma) \quad \text { or equivalently } \quad P_{r}\left(\frac{n+4}{n+1}\right)= \pm P_{s}\left(\frac{n-4}{n-1}\right) +$$ + +Proof. If there are integers $x, y, z$ for which $x \alpha+y \beta+z \gamma=0$, then we have that $(x-y) \alpha=(y-z) \gamma-\pi y$, whence it follows that we may take $r=|x-y|$ and $s=|y-z|$ (noting $r=s=0$ implies the absurd $x=y=z$ ). Conversely, given such $r$ and $s$ with $\cos (r \alpha)= \pm \cos (s \gamma)$, then it follows that $r \alpha \pm s \gamma=k \pi=k(\alpha+\beta+\gamma)$ for some $k$, so the triangle is quirky. + +If $r=0$, then by rational root theorem on $P_{s}(X) \pm 2$ it follows $\frac{n-4}{n-1}$ must be an integer which occurs only when $n=4$ (recall $n \geq 3$ ). Similarly we may discard the case $s=0$. + +Thus in what follows assume $n \neq 4$ and $r, s>0$. Then, from the fact that $P_{r}$ and $P_{s}$ are nonconstant monic polynomials, we find + +## Corollary + +If $n \neq 4$ works, then when $\frac{n+4}{n+1}$ and $\frac{n-4}{n-1}$ are written as fractions in lowest terms, the denominators have the same set of prime factors. + +But $\operatorname{gcd}(n+1, n-1)$ divides 2 , and $\operatorname{gcd}(n+4, n+1), \operatorname{gcd}(n-4, n-1)$ divide 3 . So we only have three possibilities: + +- $n+1=2^{u}$ and $n-1=2^{v}$ for some $u, v \geq 0$. This is only possible if $n=3$. Here $2 \cos \alpha=\frac{7}{4}$ and $2 \cos \gamma=-\frac{1}{2}$, and indeed $P_{2}(-1 / 2)=-7 / 4$. +- $n+1=3 \cdot 2^{u}$ and $n-1=2^{v}$ for some $u, v \geq 0$, which implies $n=5$. Here $2 \cos \alpha=\frac{3}{2}$ and $2 \cos \gamma=\frac{1}{4}$, and indeed $P_{2}(3 / 2)=1 / 4$. +- $n+1=2^{u}$ and $n-1=3 \cdot 2^{v}$ for some $u, v \geq 0$, which implies $n=7$. Here $2 \cos \alpha=\frac{11}{8}$ and $2 \cos \gamma=\frac{1}{2}$, and indeed $P_{3}(1 / 2)=-11 / 8$. + +Finally, $n=4$ works because the triangle is right, completing the solution. +Remark (Major generalization due to Luke Robitaille). In fact one may find all quirky triangles whose sides are integers in arithmetic progression. + +Indeed, if the side lengths of the triangle are $x-y, x, x+y$ with $\operatorname{gcd}(x, y)=1$ then the problem becomes + +$$ +P_{r}\left(\frac{x+4 y}{x+y}\right)= \pm P_{s}\left(\frac{x-4 y}{x-y}\right) +$$ + +and so in the same way as before, we ought to have $x+y$ and $x-y$ are both of the form $3 \cdot 2^{*}$ unless $r s=0$. This time, when $r s=0$, we get the extra solutions $(1,0)$ and $(5,2)$. + +For $r s \neq 0$, by triangle inequality, we have $x-y \leq x+y<3(x-y)$, and $\min \left(\nu_{2}(x-\right.$ $\left.y), \nu_{2}(x+y)\right) \leq 1$, so it follows one of $x-y$ or $x+y$ must be in $\{1,2,3,6\}$. An exhaustive check then leads to + +$$ +(x, y) \in\{(3,1),(5,1),(7,1),(11,5)\} \cup\{(1,0),(5,2),(4,1)\} +$$ + +as the solution set. And in fact they all work. +In conclusion the equilateral triangle, $3-5-7$ triangle (which has a $120^{\circ}$ angle) and $6-11-16$ triangle (which satisfies $B=3 A+4 C$ ) are exactly the new quirky triangles (up to similarity) whose sides are integers in arithmetic progression. + +## §2 Solutions to Day 2 + +## §2.1 TSTST 2020/4, proposed by Yang Liu + +Available online at https://aops.com/community/p19444614. + +## Problem statement + +Find all pairs of positive integers $(a, b)$ satisfying the following conditions: +(i) $a$ divides $b^{4}+1$, +(ii) $b$ divides $a^{4}+1$, +(iii) $\lfloor\sqrt{a}\rfloor=\lfloor\sqrt{b}\rfloor$. + +The only solutions are $(1,1),(1,2)$, and $(2,1)$, which clearly work. Now we show there are no others. + +Obviously, $\operatorname{gcd}(a, b)=1$, so the problem conditions imply + +$$ +a b \mid(a-b)^{4}+1 +$$ + +since each of $a$ and $b$ divide the right-hand side. We define + +$$ +k \stackrel{\text { def }}{=} \frac{(b-a)^{4}+1}{a b} . +$$ + +Claim (Size estimate) — We must have $k \leq 16$. + +Proof. Let $n=\lfloor\sqrt{a}\rfloor=\lfloor\sqrt{b}\rfloor$, so that $a, b \in\left[n^{2}, n^{2}+2 n\right]$. We have that + +$$ +\begin{aligned} +a b & \geq n^{2}\left(n^{2}+1\right) \geq n^{4}+1 \\ +(b-a)^{4}+1 & \leq(2 n)^{4}+1=16 n^{4}+1 +\end{aligned} +$$ + +which shows $k \leq 16$. + +Claim (Orders argument) - In fact, $k=1$. + +Proof. First of all, note that $k$ cannot be even: if it was, then $a, b$ have opposite parity, but then $4 \mid(b-a)^{4}+1$, contradiction. + +Thus $k$ is odd. However, every odd prime divisor of $(b-a)^{4}+1$ is congruent to 1 $(\bmod 8)$ and is thus at least 17 , so $k=1$ or $k \geq 17$. It follows that $k=1$. + +At this point, we have reduced to solving + +$$ +a b=(b-a)^{4}+1 +$$ + +and we need to prove the claimed solutions are the only ones. Write $b=a+d$, and assume WLOG that $d \geq 0$ : then we have $a(a+d)=d^{4}+1$, or + +$$ +a^{2}-d a-\left(d^{4}+1\right)=0 +$$ + +The discriminant $d^{2}+4\left(d^{4}+1\right)=4 d^{4}+d^{2}+4$ must be a perfect square. + +- The cases $d=0$ and $d=1$ lead to pairs $(1,1)$ and $(1,2)$. +- If $d \geq 2$, then we can sandwich + +$$ +\left(2 d^{2}\right)^{2}<4 d^{4}+d^{2}+4<4 d^{4}+4 d^{2}+1=\left(2 d^{2}+1\right)^{2} +$$ + +so the discriminant is not a square. +The solution is complete. +Remark (Author remarks on origin). This comes from the problem of the existence of a pair of elliptic curves over $\mathbb{F}_{a}, \mathbb{F}_{b}$ respectively, such that the number of points on one is the field size of the other. The bound $n^{2} \leq a, b<(n+1)^{2}$ is the Hasse bound. The divisibility conditions correspond to asserting that the embedding degree of each curve is 8 , so that they are pairing friendly. In this way, the problem is essentially the key result of https://arxiv.org/pdf/1803.02067.pdf, shown in Proposition 3. + +## §2.2 TSTST 2020/5, proposed by Ashwin Sah, Mehtaab Sawhney + +Available online at https://aops.com/community/p19444403. + +## Problem statement + +Let $\mathbb{N}^{2}$ denote the set of ordered pairs of positive integers. A finite subset $S$ of $\mathbb{N}^{2}$ is stable if whenever $(x, y)$ is in $S$, then so are all points $\left(x^{\prime}, y^{\prime}\right)$ of $\mathbb{N}^{2}$ with both $x^{\prime} \leq x$ and $y^{\prime} \leq y$. +Prove that if $S$ is a stable set, then among all stable subsets of $S$ (including the empty set and $S$ itself), at least half of them have an even number of elements. + +The following inductive solution was given by Nikolai Beluhov. We proceed by induction on $|S|$, with $|S| \leq 1$ clear. + +Suppose $|S| \geq 2$. For any $p \in S$, let $R(p)$ denote the stable rectangle with upper-right corner $p$. We say such $p$ is pivotal if $p+(1,1) \notin S$ and $|R(p)|$ is even. +![](https://cdn.mathpix.com/cropped/2024_11_19_a8f70a0d08e0809deba0g-12.jpg?height=455&width=818&top_left_y=1069&top_left_x=622) + +Claim - If $|S| \geq 2$, then a pivotal $p$ always exists. +Proof. Consider the top row of $S$. + +- If it has length at least 2 , one of the two rightmost points in it is pivotal. +- Otherwise, the top row has length 1. Now either the top point or the point below it (which exists as $|S| \geq 2$ ) is pivotal. + +We describe how to complete the induction, given some pivotal $p \in S$. There is a partition + +$$ +S=R(p) \sqcup S_{1} \sqcup S_{2} +$$ + +where $S_{1}$ and $S_{2}$ are the sets of points in $S$ above and to the right of $p$ (possibly empty). +Claim - The desired inequality holds for stable subsets containing $p$. +Proof. Let $E_{1}$ denote the number of even stable subsets of $S_{1}$; denote $E_{2}, O_{1}, O_{2}$ analogously. The stable subsets containing $p$ are exactly $R(p) \sqcup T_{1} \sqcup T_{2}$, where $T_{1} \subseteq S_{1}$ and $T_{2} \subseteq S_{2}$ are stable. + +Since $|R(p)|$ is even, exactly $E_{1} E_{2}+O_{1} O_{2}$ stable subsets containing $p$ are even, and exactly $E_{1} O_{2}+E_{2} O_{1}$ are odd. As $E_{1} \geq O_{1}$ and $E_{2} \geq O_{2}$ by inductive hypothesis, we obtain $E_{1} E_{2}+O_{1} O_{2} \geq E_{1} O_{2}+E_{2} O_{1}$ as desired. + +By the inductive hypothesis, the desired inequality also holds for stable subsets not containing $p$, so we are done. + +## §2.3 TSTST 2020/6, proposed by Andrew Gu + +Available online at https://aops.com/community/p19444197. + +## Problem statement + +Let $A, B, C, D$ be four points such that no three are collinear and $D$ is not the orthocenter of triangle $A B C$. Let $P, Q, R$ be the orthocenters of $\triangle B C D, \triangle C A D$, $\triangle A B D$, respectively. Suppose that lines $A P, B Q, C R$ are pairwise distinct and are concurrent. Show that the four points $A, B, C, D$ lie on a circle. + +Let $T$ be the concurrency point, and let $H$ be the orthocenter of $\triangle A B C$. +![](https://cdn.mathpix.com/cropped/2024_11_19_a8f70a0d08e0809deba0g-14.jpg?height=593&width=806&top_left_y=863&top_left_x=631) + +Claim (Key claim) - $T$ is the midpoint of $\overline{A P}, \overline{B Q}, \overline{C R}, \overline{D H}$, and $D$ is the orthocenter of $\triangle P Q R$. + +Proof. Note that $\overline{A Q} \| \overline{B P}$, as both are perpendicular to $\overline{C D}$. Since lines $A P$ and $B Q$ are distinct, lines $A Q$ and $B P$ are distinct. +By symmetric reasoning, we get that $A Q C P B R$ is a hexagon with opposite sides parallel and concurrent diagonals as $\overline{A P}, \overline{B Q}, \overline{C R}$ meet at $T$. This implies that the hexagon is centrally symmetric about $T$; indeed + +$$ +\frac{A T}{T P}=\frac{T Q}{B T}=\frac{C T}{T R}=\frac{T P}{A T} +$$ + +so all the ratios are equal to +1 . +Next, $\overline{P D} \perp \overline{B C} \| \overline{Q R}$, so by symmetry we get $D$ is the orthocenter of $\triangle P Q R$. This means that $T$ is the midpoint of $\overline{D H}$ as well. + +## Corollary + +The configuration is now symmetric: we have four points $A, B, C, D$, and their reflections in $T$ are four orthocenters $P, Q, R, H$. + +Let $S$ be the centroid of $\{A, B, C, D\}$, and let $O$ be the reflection of $T$ in $S$. We are ready to conclude: + +Claim - $A, B, C, D$ are equidistant from $O$. +Proof. Let $A^{\prime}, O^{\prime}, S^{\prime}, T^{\prime}, D^{\prime}$ be the projections of $A, O, S, T, D$ onto line $B C$. +Then $T^{\prime}$ is the midpoint of $\overline{A^{\prime} D^{\prime}}$, so $S^{\prime}=\frac{1}{4}\left(A^{\prime}+D^{\prime}+B+C\right)$ gives that $O^{\prime}$ is the midpoint of $\overline{B C}$. + +Thus $O B=O C$ and we're done. + +## §3 Solutions to Day 3 + +## §3.1 TSTST 2020/7, proposed by Ankan Bhattacharya + +Available online at https://aops.com/community/p20020202. + +## Problem statement + +Find all nonconstant polynomials $P(z)$ with complex coefficients for which all complex roots of the polynomials $P(z)$ and $P(z)-1$ have absolute value 1 . + +The answer is $P(x)$ should be a polynomial of the form $P(x)=\lambda x^{n}-\mu$ where $|\lambda|=|\mu|$ and $\operatorname{Re} \mu=-\frac{1}{2}$. One may check these all work; let's prove they are the only solutions. +\l First approach (Evan Chen). We introduce the following notations: + +$$ +\begin{aligned} +P(x) & =c_{n} x^{n}+c_{n-1} x^{n-1}+\cdots+c_{1} x+c_{0} \\ +& =c_{n}\left(x+\alpha_{1}\right) \ldots\left(x+\alpha_{n}\right) \\ +P(x)-1 & =c_{n}\left(x+\beta_{1}\right) \ldots\left(x+\beta_{n}\right) +\end{aligned} +$$ + +By taking conjugates, + +$$ +\begin{aligned} +\left(x+\alpha_{1}\right) \cdots\left(x+\alpha_{n}\right) & =\left(x+\beta_{1}\right) \cdots\left(x+\beta_{n}\right)+c_{n}^{-1} \\ +\Longrightarrow\left(x+\frac{1}{\alpha_{1}}\right) \cdots\left(x+\frac{1}{\alpha_{n}}\right) & =\left(x+\frac{1}{\beta_{1}}\right) \cdots\left(x+\frac{1}{\beta_{n}}\right)+\left(\overline{c_{n}}\right)^{-1} +\end{aligned} +$$ + +The equation $(\boldsymbol{\oplus})$ is the main player: +Claim - We have $c_{k}=0$ for all $k=1, \ldots, n-1$. + +Proof. By comparing coefficients of $x^{k}$ in ( $\left.\boldsymbol{\phi}\right)$ we obtain + +$$ +\frac{c_{n-k}}{\prod_{i} \alpha_{i}}=\frac{c_{n-k}}{\prod_{i} \beta_{i}} +$$ + +but $\prod_{i} \alpha_{i}-\prod_{i} \beta_{i}=\frac{1}{c_{n}} \neq 0$. Hence $c_{k}=0$. +It follows that $P(x)$ must be of the form $P(x)=\lambda x^{n}-\mu$, so that $P(x)=\lambda x^{n}-(\mu+1)$. This requires $|\mu|=|\mu+1|=|\lambda|$ which is equivalent to the stated part. + +II Second approach (from the author). We let $A=P$ and $B=P-1$ to make the notation more symmetric. We will as before show that $A$ and $B$ have all coefficients equal to zero other than the leading and constant coefficient; the finish is the same. + +First, we rule out double roots. +Claim - Neither $A$ nor $B$ have double roots. + +Proof. Suppose that $b$ is a double root of $B$. By differentiating, we obtain $A^{\prime}=B^{\prime}$, so $A^{\prime}(b)=0$. However, by Gauss-Lucas, this forces $A(b)=0$, contradiction. + +Let $\omega=e^{2 \pi i / n}$, let $a_{1}, \ldots, a_{n}$ be the roots of $A$, and let $b_{1}, \ldots, b_{n}$ be the roots of $B$. For each $k$, let $A_{k}$ and $B_{k}$ be the points in the complex plane corresponding to $a_{k}$ and $b_{k}$ 。 + +Claim (Main claim) - For any $i$ and $j, \frac{a_{i}}{a_{j}}$ is a power of $\omega$. +Proof. Note that + +$$ +\frac{a_{i}-b_{1}}{a_{j}-b_{1}} \cdots \frac{a_{i}-b_{n}}{a_{j}-b_{n}}=\frac{B\left(a_{i}\right)}{B\left(a_{j}\right)}=\frac{A\left(a_{i}\right)-1}{A\left(a_{j}\right)-1}=\frac{0-1}{0-1}=1 +$$ + +Since the points $A_{i}, A_{j}, B_{k}$ all lie on the unit circle, interpreting the left-hand side geometrically gives + +$$ +\measuredangle A_{i} B_{1} A_{j}+\cdots+\measuredangle A_{i} B_{n} A_{j}=0 \Longrightarrow n \widehat{A_{i} A_{j}}=0 +$$ + +where angles are directed modulo $180^{\circ}$ and arcs are directed modulo $360^{\circ}$. This implies that $\frac{a_{i}}{a_{j}}$ is a power of $\omega$. + +Now the finish is easy: since $a_{1}, \ldots, a_{n}$ are all different, they must be $a_{1} \omega^{0}, \ldots, a_{1} \omega^{n-1}$ in some order; this shows that $A$ is a multiple of $x^{n}-a_{1}^{n}$, as needed. + +## §3.2 TSTST 2020/8, proposed by Ankan Bhattacharya + +Available online at https://aops.com/community/p20020195. + +## Problem statement + +For every positive integer $N$, let $\sigma(N)$ denote the sum of the positive integer divisors of $N$. Find all integers $m \geq n \geq 2$ satisfying + +$$ +\frac{\sigma(m)-1}{m-1}=\frac{\sigma(n)-1}{n-1}=\frac{\sigma(m n)-1}{m n-1} +$$ + +The answer is that $m$ and $n$ should be powers of the same prime number. These all work because for a prime power we have + +$$ +\frac{\sigma\left(p^{e}\right)-1}{p^{e}-1}=\frac{\left(1+p+\cdots+p^{e}\right)-1}{p^{e}-1}=\frac{p\left(1+\cdots+p^{e-1}\right)}{p^{e}-1}=\frac{p}{p-1} +$$ + +So we now prove these are the only ones. Let $\lambda$ be the common value of the three fractions. + +Claim - Any solution $(m, n)$ should satisfy $d(m n)=d(m)+d(n)-1$. +Proof. The divisors of $m n$ include the divisors of $m$, plus $m$ times the divisors of $n$ (counting $m$ only once). Let $\lambda$ be the common value; then this gives + +$$ +\begin{aligned} +\sigma(m n) & \geq \sigma(m)+m \sigma(n)-m \\ +& =(\lambda m-\lambda+1)+m(\lambda n-\lambda+1)-m \\ +& =\lambda m n-\lambda+1 +\end{aligned} +$$ + +and so equality holds. Thus these are all the divisors of $m n$, for a count of $d(m)+d(n)-$ 1. + +Claim - If $d(m n)=d(m)+d(n)-1$ and $\min (m, n) \geq 2$, then $m$ and $n$ are powers of the same prime. + +Proof. Let $A$ denote the set of divisors of $m$ and $B$ denote the set of divisors of $n$. Then $|A \cdot B|=|A|+|B|-1$ and $\min (|A|,|B|)>1$, so $|A|$ and $|B|$ are geometric progressions with the same ratio. It follows that $m$ and $n$ are powers of the same prime. + +Remark (Nikolai Beluhov). Here is a completion not relying on $|A \cdot B|=|A|+|B|-1$. By the above arguments, we see that every divisor of $m n$ is either a divisor of $n$, or $n$ times a divisor of $m$. + +Now suppose that some prime $p \mid m$ but $p \nmid n$. Then $p \mid m n$ but $p$ does not appear in the above classification, a contradiction. By symmetry, it follows that $m$ and $n$ have the same prime divisors. + +Now suppose we have different primes $p \mid m$ and $q \mid n$. Write $\nu_{p}(m)=\alpha$ and $\nu_{p}(n)=\beta$. Then $p^{\alpha+\beta} \mid m n$, but it does not appear in the above characterization, a contradiction. Thus, $m$ and $n$ are powers of the same prime. + +Remark (Comments on the function in the problem). Let $f(n)=\frac{\sigma(n)-1}{n-1}$. Then $f$ is not really injective even outside the above solution; for example, we have $f\left(6 \cdot 11^{k}\right)=\frac{11}{5}$ for all $k$, plus sporadic equivalences like $f(14)=f(404)$, as pointed out by one reviewer during test-solving. This means that both relations should be used at once, not independently. + +Remark (Authorship remarks). Ankan gave the following story for how he came up with the problem while thinking about so-called almost perfect numbers. + +I was in some boring talk when I recalled a conjecture that if $\sigma(n)=2 n-1$, then $n$ is a power of 2 . For some reason (divine intervention, maybe) I had the double idea of (1) seeing whether $m, n, m n$ all almost perfect implies $m, n$ powers of 2 , and (2) trying the naive divisor bound to resolve this. Through sheer dumb luck this happened to work out perfectly. I thought this was kinda cool but I felt that I hadn't really unlocked a lot of the potential this idea had: then I basically tried to find the "general situation" which allows for this manipulation, and was amazed that it led to such a striking statement. + +## §3.3 TSTST 2020/9, proposed by Nikolai Beluhov + +Available online at https://aops.com/community/p20020206. + +## Problem statement + +Ten million fireflies are glowing in $\mathbb{R}^{3}$ at midnight. Some of the fireflies are friends, and friendship is always mutual. Every second, one firefly moves to a new position so that its distance from each one of its friends is the same as it was before moving. This is the only way that the fireflies ever change their positions. No two fireflies may ever occupy the same point. +Initially, no two fireflies, friends or not, are more than a meter away. Following some finite number of seconds, all fireflies find themselves at least ten million meters away from their original positions. Given this information, find the greatest possible number of friendships between the fireflies. + +In general, we show that when $n \geq 70$, the answer is $f(n)=\left\lfloor\frac{n^{2}}{3}\right\rfloor$. +Construction: Choose three pairwise parallel lines $\ell_{A}, \ell_{B}, \ell_{C}$ forming an infinite equilateral triangle prism (with side larger than 1). Split the $n$ fireflies among the lines as equally as possible, and say that two fireflies are friends iff they lie on different lines. + +To see this works: + +1. Reflect $\ell_{A}$ and all fireflies on $\ell_{A}$ in the plane containing $\ell_{B}$ and $\ell_{C}$. +2. Reflect $\ell_{B}$ and all fireflies on $\ell_{B}$ in the plane containing $\ell_{C}$ and $\ell_{A}$. +3. Reflect $\ell_{C}$ and all fireflies on $\ell_{C}$ in the plane containing $\ell_{A}$ and $\ell_{B}$. + +Proof: Consider a valid configuration of fireflies. If there is no 4 -clique of friends, then by Turán's theorem, there are at most $f(n)$ pairs of friends. +Let $g(n)$ be the answer, given that there exist four pairwise friends (say $a, b, c, d$ ). Note that for a firefly to move, all its friends must be coplanar. + +Claim (No coplanar $K_{4}$ ) — We can't have four coplanar fireflies which are pairwise friends. + +Proof. If we did, none of them could move (unless three are collinear, in which case they can't move). + +Claim (Key claim — tetrahedrons don't share faces often) — There are at most 12 fireflies $e$ which are friends with at least three of $a, b, c, d$. + +Proof. First denote by $A, B, C, D$ the locations of fireflies $a, b, c, d$. These four positions change over time as fireflies move, but the tetrahedron $A B C D$ always has a fixed shape, and we will take this tetrahedron as our reference frame for the remainder of the proof. + +WLOG, will assume that $e$ is friends with $a, b, c$. Then $e$ will always be located at one of two points $E_{1}$ and $E_{2}$ relative to $A B C$, such that $E_{1} A B C$ and $E_{2} A B C$ are two +congruent tetrahedrons with fixed shape. We note that points $D, E_{1}$, and $E_{2}$ are all different: clearly $D \neq E_{1}$ and $E_{1} \neq E_{2}$. (If $D=E_{2}$, then some fireflies won't be able to move.) + +Consider the moment where firefly $a$ moves. Its friends must be coplanar at that time, so one of $E_{1}, E_{2}$ lies in plane $B C D$. Similar reasoning holds for planes $A C D$ and $A B D$. + +So, WLOG $E_{1}$ lies on both planes $B C D$ and $A C D$. Then $E_{1}$ lies on line $C D$, and $E_{2}$ lies in plane $A B D$. This uniquely determines $\left(E_{1}, E_{2}\right)$ relative to $A B C D$ : + +- $E_{1}$ is the intersection of line $C D$ with the reflection of plane $A B D$ in plane $A B C$. +- $E_{2}$ is the intersection of plane $A B D$ with the reflection of line $C D$ in plane $A B C$. + +Accounting for WLOGs, there are at most 12 possibilities for the set $\left\{E_{1}, E_{2}\right\}$, and thus at most 12 possibilities for $E$. (It's not possible for both elements of one pair $\left\{E_{1}, E_{2}\right\}$ to be occupied, because then they couldn't move.) + +Thus, the number of friendships involving exactly one of $a, b, c, d$ is at most $(n-16)$. $2+12 \cdot 3=2 n+4$, so removing these four fireflies gives + +$$ +g(n) \leq 6+(2 n+4)+\max \{f(n-4), g(n-4)\} +$$ + +The rest of the solution is bounding. When $n \geq 24$, we have $(2 n+10)+f(n-4) \leq f(n)$, so + +$$ +g(n) \leq \max \{f(n),(2 n+10)+g(n-4)\} \quad \forall n \geq 24 +$$ + +By iterating the above inequality, we get + +$$ +\begin{aligned} +g(n) \leq \max \{f(n),(2 n+10) & +(2(n-4)+10) \\ +& +\cdots+(2(n-4 r)+10)+g(n-4 r-4)\} +\end{aligned} +$$ + +where $r$ satisfies $n-4 r-4<24 \leq n-4 r$. +Now + +$$ +\begin{aligned} +& (2 n+10)+(2(n-4)+10)+\cdots+(2(n-4 r)+10)+g(n-4 r-4) \\ += & (r+1)(2 n-4 r+10)+g(n-4 r-4) \\ +\leq & \left(\frac{n}{4}-5\right)(n+37)+\binom{24}{2} . +\end{aligned} +$$ + +This is less than $f(n)$ for $n \geq 70$, which concludes the solution. +Remark. There are positive integers $n$ such that it is possible to do better than $f(n)$ friendships. For instance, $f(5)=8$, whereas five fireflies $a, b, c, d$, and $e$ as in the proof of the Lemma ( $E_{1}$ being the intersection point of line $C D$ with the reflection of plane $(A B D)$ in plane $(A B C), E_{2}$ being the intersection point of plane $(A B D)$ with the reflection of line $C D$ in plane $(A B C)$, and tetrahedron $A B C D$ being sufficiently arbitrary that points $E_{1}$ and $E_{2}$ exist and points $D, E_{1}$, and $E_{2}$ are pairwise distinct) give a total of nine friendships. + +Remark (Author comments). It is natural to approach the problem by looking at the two-dimensional version first. In two dimensions, the following arrangement suggests itself almost immediately: We distribute all fireflies as equally as possible among two parallel lines, and two fireflies are friends if and only if they are on different lines. + +Similarly to the three-dimensional version, this attains the greatest possible number of friendships for all sufficiently large $n$, though not for all $n$. For instance, at least one friendlier arrangements exists for $n=4$, similarly to the above friendlier arrangement for $n=5$ in three dimensions. + +This observation strongly suggests that in three dimensions we should distribute the fireflies as equally as possible among two parallel planes, and that two fireflies should be friends if and only if they are on different planes. It was a great surprise for me to discover that this arrangement does not in fact give the correct answer! + +Remark. On the other hand, Ankan Bhattacharya gives the following reasoning as to why the answer should not be that surprising: + +I think the answer $\left(10^{14}-1\right) / 3$ is quite natural if you realize that $(n / 2)^{2}$ is probably optimal in 2D and $\binom{n}{2}$ is optimal in super high dimensions (i.e. around $n$ ). So going from dimension 2 to 3 should increase the answer (and indeed it does). + diff --git a/USA_TSTST/md/en-sols-TSTST-2021.md b/USA_TSTST/md/en-sols-TSTST-2021.md new file mode 100644 index 0000000000000000000000000000000000000000..b5442d99aa7c2ad62f5705b4e086be4f5164b58e --- /dev/null +++ b/USA_TSTST/md/en-sols-TSTST-2021.md @@ -0,0 +1,1228 @@ +# USA TSTST 2021 Solutions
United States of America - TST Selection Test
Andrew Gu and Evan Chen
$63^{\text {rd }}$ IMO 2022 Norway and $11^{\text {th }}$ EGMO 2022 Hungary + +## Contents + +0 Problems ..... 2 +1 Solutions to Day 1 ..... 3 +1.1 TSTST 2021/1, proposed by Holden Mui ..... 3 +1.2 TSTST 2021/2, proposed by Merlijn Staps ..... 7 +1.3 TSTST 2021/3, proposed by Merlijn Staps ..... 11 +2 Solutions to Day 2 ..... 13 +2.1 TSTST 2021/4, proposed by Holden Mui ..... 13 +2.2 TSTST 2021/5, proposed by Vincent Huang ..... 15 +2.3 TSTST 2021/6, proposed by Nikolai Beluhov ..... 17 +3 Solutions to Day 3 ..... 23 +3.1 TSTST 2021/7, proposed by Ankit Bisain, Holden Mui ..... 23 +3.2 TSTST 2021/8, proposed by Fedir Yudin ..... 25 +3.3 TSTST 2021/9, proposed by Victor Wang ..... 28 + +## §0 Problems + +1. Let $A B C D$ be a quadrilateral inscribed in a circle with center $O$. Points $X$ and $Y$ lie on sides $A B$ and $C D$, respectively. Suppose the circumcircles of $A D X$ and $B C Y$ meet line $X Y$ again at $P$ and $Q$, respectively. Show that $O P=O Q$. +2. Let $a_{1}1$ for which there exists a positive integer $n$ such that $\binom{n}{k}$ is divisible by $n$, and $\binom{n}{m}$ is not divisible by $n$ for $2 \leq mi$. + +First, each $i$ has finite degree - otherwise + +$$ +\frac{a_{x_{1}}}{x_{1}}=\frac{a_{x_{2}}}{x_{2}}=\cdots +$$ + +for an infinite increasing sequence of positive integers $x_{i}$, but then the $a_{x_{i}}$ are unbounded. +Now we use the following process to build a sequence of indices whose $a_{i}$ we can lower-bound: + +- Start at $x_{1}=1$, which is good. +- If we're currently at good index $x_{i}$, then let $s_{i}$ be the largest positive integer such that $x_{i}$ has an edge to $x_{i}+s_{i}$. (This exists because the degrees are finite.) +- Let $t_{i}$ be the smallest positive integer for which $x_{i}+s_{i}+t_{i}$ is good, and let this be $x_{i+1}$. This exists because if all numbers $k \leq x \leq 2 k$ are bad, they must each connect to some number less than $k$ (if two connect to each other, the smaller one is good), but then two connect to the same number, and therefore to each other this is the idea we will use later to bound the $t_{i}$ as well. + +Then $x_{i}=1+s_{1}+t_{1}+\cdots+s_{i-1}+t_{i-1}$, and we have + +$$ +a_{x_{i+1}}>a_{x_{i}+s_{i}}=\frac{x_{i}+s_{i}}{x_{i}} a_{x_{i}}=\frac{1+\left(s_{1}+\cdots+s_{i-1}+s_{i}\right)+\left(t_{1}+\cdots+t_{i-1}\right)}{1+\left(s_{1}+\cdots+s_{i-1}\right)+\left(t_{1}+\cdots+t_{i-1}\right)} a_{x_{i}} +$$ + +This means + +$$ +c_{n}:=\frac{a_{x_{n}}}{a_{1}}>\prod_{i=1}^{n-1} \frac{1+\left(s_{1}+\cdots+s_{i-1}+s_{i}\right)+\left(t_{1}+\cdots+t_{i-1}\right)}{1+\left(s_{1}+\cdots+s_{i-1}\right)+\left(t_{1}+\cdots+t_{i-1}\right)} +$$ + +## Lemma + +$t_{1}+\cdots+t_{n} \leq s_{1}+\cdots+s_{n}$ for each $n$. + +Proof. Consider $1 \leq i \leq n$. Note that for every $i$, the $t_{i}-1$ integers strictly between $x_{i}+s_{i}$ and $x_{i}+s_{i}+t_{i}$ are all bad, so each such index $x$ must have an edge to some $yx_{i}+s_{i} \geq x_{j}+s_{j}$, contradicting the fact that $x_{j}+s_{j}$ is the largest neighbor of $x_{j}$. + +This also means $x$ doesn't have an edge to $x_{j}+s_{j}$ for any $j \leq i$, since if it did, it would have an edge to $x_{j}$. + +Second, no two bad values of $x$ can have an edge, since then the smaller one is good. This also means no two bad $x$ can have an edge to the same $y$. + +Then each of the $\sum\left(t_{i}-1\right)$ values in the intervals $\left(x_{i}+s_{i}, x_{i}+s_{i}+t_{i}\right)$ for $1 \leq i \leq n$ must have an edge to an unique $y$ in one of the intervals $\left(x_{i}, x_{i}+s_{i}\right)$ (not necessarily with the same $i$ ). Therefore + +$$ +\sum\left(t_{i}-1\right) \leq \sum\left(s_{i}-1\right) \Longrightarrow \sum t_{i} \leq \sum s_{i} +$$ + +Now note that if $a>b$, then $\frac{a+x}{b+x}=1+\frac{a-b}{b+x}$ is decreasing in $x$. This means + +$$ +c_{n}>\prod_{i=1}^{n-1} \frac{1+2 s_{1}+\cdots+2 s_{i-1}+s_{i}}{1+2 s_{1}+\cdots+2 s_{i-1}}>\prod_{i=1}^{n-1} \frac{1+2 s_{1}+\cdots+2 s_{i-1}+2 s_{i}}{1+2 s_{1}+\cdots+2 s_{i-1}+s_{i}} +$$ + +By multiplying both products, we have a telescoping product, which results in + +$$ +c_{n}^{2} \geq 1+2 s_{1}+\cdots+2 s_{n}+2 s_{n+1} +$$ + +The right hand side is unbounded since the $s_{i}$ are positive integers, while $c_{n}=a_{x_{n}} / a_{1}<$ $1 / a_{1}$ is bounded, contradiction. + +【 Solution 3 (Gopal Goel). Suppose for sake of contradiction that the problem is false. Call an index $i$ a pin if + +$$ +\frac{a_{j}}{j}=\frac{a_{i}}{i} \Longrightarrow j \geq i +$$ + +## Lemma + +There exists $k$ such that if we have $\frac{a_{i}}{i}=\frac{a_{j}}{j}$ with $j>i \geq k$, then $j \leq 1.1 i$. + +Proof. Note that for any $i$, there are only finitely many $j$ with $\frac{a_{j}}{j}=\frac{a_{i}}{i}$, otherwise $a_{j}=\frac{j a_{i}}{i}$ is unbounded. Thus it suffices to find $k$ for which $j \leq 1.1 i$ when $j>i \geq k$. + +Suppose no such $k$ exists. Then, take a pair $j_{1}>i_{1}$ such that $\frac{a_{j_{1}}}{j_{1}}=\frac{a_{i_{1}}}{i_{1}}$ and $j_{1}>1.1 i_{1}$, or $a_{j_{1}}>1.1 a_{i_{1}}$. Now, since $k=j_{1}$ can't work, there exists a pair $j_{2}>i_{2} \geq i_{1}$ such that $\frac{a_{j_{2}}}{j_{2}}=\frac{a_{i_{2}}}{i_{2}}$ and $j_{2}>1.1 i_{2}$, or $a_{j_{2}}>1.1 a_{i_{2}}$. Continuing in this fashion, we see that + +$$ +a_{j_{\ell}}>1.1 a_{i_{\ell}}>1.1 a_{j_{\ell-1}} +$$ + +so we have that $a_{j_{\ell}}>1.1^{\ell} a_{i_{1}}$. Taking $\ell>\log _{1.1}\left(1 / a_{1}\right)$ gives the desired contradiction. + +## Lemma + +For $N>k^{2}$, there are at most $0.8 N$ pins in $[\sqrt{N}, N)$. + +Proof. By the first lemma, we see that the number of pins in $\left[\sqrt{N}, \frac{N}{1.1}\right)$ is at most the number of non-pins in $[\sqrt{N}, N)$. Therefore, if the number of pins in $[\sqrt{N}, N)$ is $p$, then we have + +$$ +p-N\left(1-\frac{1}{1.1}\right) \leq N-p +$$ + +so $p \leq 0.8 N$, as desired. +We say that $i$ is the pin of $j$ if it is the smallest index such that $\frac{a_{i}}{i}=\frac{a_{j}}{j}$. The pin of $j$ is always a pin. + +Given an index $i$, let $f(i)$ denote the largest index less than $i$ that is not a pin (we leave the function undefined when no such index exists, as we are only interested in the behavior for large $i$ ). Then $f$ is weakly increasing and unbounded by the first lemma. Let $N_{0}$ be a positive integer such that $f\left(\sqrt{N_{0}}\right)>k$. + +Take any $N>N_{0}$ such that $N$ is not a pin. Let $b_{0}=N$, and $b_{1}$ be the pin of $b_{0}$. Recursively define $b_{2 i}=f\left(b_{2 i-1}\right)$, and $b_{2 i+1}$ to be the pin of $b_{2 i}$. + +Let $\ell$ be the largest odd index such that $b_{\ell} \geq \sqrt{N}$. We first show that $b_{\ell} \leq 100 \sqrt{N}$. Since $N>N_{0}$, we have $b_{\ell+1}>k$. By the choice of $\ell$ we have $b_{\ell+2}<\sqrt{N}$, so + +$$ +b_{\ell+1}<1.1 b_{\ell+2}<1.1 \sqrt{N} +$$ + +by the first lemma. We see that all the indices from $b_{\ell+1}+1$ to $b_{\ell}$ must be pins, so we have at least $b_{\ell}-1.1 \sqrt{N}$ pins in $\left[\sqrt{N}, b_{\ell}\right)$. Combined with the second lemma, this shows that $b_{\ell} \leq 100 \sqrt{N}$. +Now, we have that $a_{b_{2 i}}=\frac{b_{2 i}}{b_{2 i+1}} a_{b_{2 i+1}}$ and $a_{b_{2 i+1}}>a_{b_{2 i+2}}$, so combining gives us + +$$ +\frac{a_{b_{0}}}{a_{b_{\ell}}}>\frac{b_{0}}{b_{1}} \frac{b_{2}}{b_{3}} \cdots \frac{b_{\ell-1}}{b_{\ell}} . +$$ + +Note that there are at least + +$$ +\left(b_{1}-b_{2}\right)+\left(b_{3}-b_{4}\right)+\cdots+\left(b_{\ell-2}-b_{\ell-1}\right) +$$ + +pins in $[\sqrt{N}, N)$, so by the second lemma, that sum is at most $0.8 N$. Thus, + +$$ +\begin{aligned} +\left(b_{0}-b_{1}\right)+\left(b_{2}-b_{3}\right)+\cdots+\left(b_{\ell-1}-b_{\ell}\right) & =b_{0}-\left[\left(b_{1}-b_{2}\right)+\cdots+\left(b_{\ell-2}-b_{\ell-1}\right)\right]-b_{\ell} \\ +& \geq 0.2 N-100 \sqrt{N} . +\end{aligned} +$$ + +Then + +$$ +\begin{aligned} +\frac{b_{0}}{b_{1}} \frac{b_{2}}{b_{3}} \cdots \frac{b_{\ell-1}}{b_{\ell}} & \geq 1+\frac{b_{0}-b_{1}}{b_{1}}+\cdots+\frac{b_{\ell-1}-b_{\ell}}{b_{\ell}} \\ +& >1+\frac{b_{0}-b_{1}}{b_{0}}+\cdots+\frac{b_{\ell-1}-b_{\ell}}{b_{0}} \\ +& \geq 1+\frac{0.2 N-100 \sqrt{N}}{N} +\end{aligned} +$$ + +which is at least 1.01 if $N_{0}$ is large enough. Thus, we see that + +$$ +a_{N}>1.01 a_{b_{\ell}} \geq 1.01 a_{\lfloor\sqrt{N}\rfloor} +$$ + +if $N>N_{0}$ is not a pin. Since there are arbitrarily large non-pins, this implies that the sequence $\left(a_{n}\right)$ is unbounded, which is the desired contradiction. + +## §1.3 TSTST 2021/3, proposed by Merlijn Staps + +Available online at https://aops.com/community/p23586679. + +## Problem statement + +Find all positive integers $k>1$ for which there exists a positive integer $n$ such that $\binom{n}{k}$ is divisible by $n$, and $\binom{n}{m}$ is not divisible by $n$ for $2 \leq mt_{p}$, so $\nu_{p}(n-i)=\nu_{p}(i)$; +- If $p \mid k$ and $i \neq k_{p}$, then we have $\nu_{p}(i), \nu_{p}(n-i) \leq t_{p}$ and $\nu_{p}(n) \geq t_{p}$, so again $\nu_{p}(n-i)=\nu_{p}(i) ;$ +- If $p \mid k$ and $i=k_{p}$, then we have $\nu_{p}(n-i)=\nu_{p}(i)+\nu_{p}(k)$ by (2). + +We conclude that $\nu_{p}(n-i)=\nu_{p}(i)$ always holds, except when $i=k_{p}$, when we have $\nu_{p}(n-i)=\nu_{p}(i)+\nu_{p}(k)$ (this formula holds irrespective of whether $p \mid k$ or $\left.p \nmid k\right)$. + +We can now show that $\binom{n}{k}$ is divisible by $n$, which amounts to showing that $k$ ! divides $(n-1)(n-2) \cdots(n-k+1)$. Indeed, for each prime $p \leq k$ we have + +$$ +\begin{aligned} +\nu_{p}((n-1)(n-2) \ldots(n-k+1)) & =\nu_{p}\left(n-k_{p}\right)+\sum_{i\nu_{p}(k)$, then it follows that + +$$ +\begin{aligned} +\nu_{p}((n-1)(n-2) \ldots(n-m+1)) & =\nu_{p}(k)+\sum_{i=1}^{m-1} \nu_{p}(i) \\ +& <\nu_{p}(m)+\sum_{i=1}^{m-1} \nu_{p}(i) \\ +& =\nu_{p}(m!) +\end{aligned} +$$ + +so $m$ ! cannot divide $(n-1)(n-2) \ldots(n-m+1)$. On the other hand, suppose that $\nu_{p}(m) \leq \nu_{p}(k)$ for all $p \mid k$, which would mean that $m \mid k$ and hence $m \leq \frac{k}{2}$. Consider a prime $p$ dividing $m$. We have $k_{p} \geq \frac{k}{2}$, because otherwise $2 k_{p}$ could have been used instead of $k_{p}$. It follows that $m \leq \frac{k}{2} \leq k_{p}$. Therefore, we obtain + +$$ +\begin{aligned} +\nu_{p}((n-1)(n-2) \ldots(n-m+1)) & =\sum_{i=1}^{m-1} \nu_{p}(n-i) \\ +& =\sum_{i=1}^{m-1} \nu_{p}(i) \\ +& =\nu_{p}((m-1)!)<\nu_{p}(m!) +\end{aligned} +$$ + +showing that $(n-1)(n-2) \cdots(n-m+1)$ is not divisible by $m$ !. This shows that $\binom{n}{m}$ is not divisible by $n$ for $ma / 2$, then $(\dagger)$ forces $r^{2}+s^{2} \leq 2 b$, giving the last case. +Hence, there exists some particular pair $(r, s)$ for which there are infinitely many solutions $(m, n)$. Simplifying the system gives + +$$ +\begin{aligned} +& a n=2 r m+r^{2}-b \\ +& 2 s n=a m+b-s^{2} +\end{aligned} +$$ + +Since the system is linear, for there to be infinitely many solutions $(m, n)$ the system must be dependent. This gives + +$$ +\frac{a}{2 s}=\frac{2 r}{a}=\frac{r^{2}-b}{b-s^{2}} +$$ + +so $a=2 \sqrt{r s}$ and $b=\frac{s^{2} \sqrt{r}+r^{2} \sqrt{s}}{\sqrt{r}+\sqrt{s}}$. Since $r s$ must be square, we can reparametrize as $r=k x^{2}, s=k y^{2}$, and $\operatorname{gcd}(x, y)=1$. This gives + +$$ +\begin{aligned} +a & =2 k x y \\ +b & =k^{2} x y\left(x^{2}-x y+y^{2}\right) +\end{aligned} +$$ + +Thus, $a \mid 2 b$, as desired. + +## §2.2 TSTST 2021/5, proposed by Vincent Huang + +Available online at https://aops.com/community/p23864182. + +## Problem statement + +Let $T$ be a tree on $n$ vertices with exactly $k$ leaves. Suppose that there exists a subset of at least $\frac{n+k-1}{2}$ vertices of $T$, no two of which are adjacent. Show that the longest path in $T$ contains an even number of edges. + +The longest path in $T$ must go between two leaves. The solutions presented here will solve the problem by showing that in the unique 2-coloring of $T$, all leaves are the same color. + +## 【 Solution 1 (Ankan Bhattacharya, Jeffery Li). + +## Lemma + +If $S$ is an independent set of $T$, then + +$$ +\sum_{v \in S} \operatorname{deg}(v) \leq n-1 +$$ + +Equality holds if and only if $S$ is one of the two components of the unique 2-coloring of $T$. + +Proof. Each edge of $T$ is incident to at most one vertex of $S$, so we obtain the inequality by counting how many vertices of $S$ each edge is incident to. For equality to hold, each edge is incident to exactly one vertex of $S$, which implies the 2 -coloring. + +We are given that there exists an independent set of at least $\frac{n+k-1}{2}$ vertices. By greedily choosing vertices of smallest degree, the sum of the degrees of these vertices is at least + +$$ +k+2 \cdot \frac{n-k-1}{2}=n-1 +$$ + +Thus equality holds everywhere, which implies that the independent set contains every leaf and is one of the components of the 2 -coloring. + +## 【 Solution 2 (Andrew Gu). + +## Lemma + +The vertices of $T$ can be partitioned into $k-1$ paths (i.e. the induced subgraph on each set of vertices is a path) such that all edges of $T$ which are not part of a path are incident to an endpoint of a path. + +Proof. Repeatedly trim the tree by taking a leaf and removing the longest path containing that leaf such that the remaining graph is still a tree. + +Now given a path of $a$ vertices, at most $\frac{a+1}{2}$ of those vertices can be in an independent set of $T$. By the lemma, $T$ can be partitioned into $k-1$ paths of $a_{1}, \ldots, a_{k-1}$ vertices, so the maximum size of an independent set of $T$ is + +$$ +\sum \frac{a_{i}+1}{2}=\frac{n+k-1}{2} . +$$ + +For equality to hold, each path in the partition must have an odd number of vertices, and has a unique 2 -coloring in red and blue where the endpoints are red. The unique independent set of $T$ of size $\frac{n+k-1}{2}$ is then the set of red vertices. By the lemma, the edges of $T$ which are not part of a path connect an endpoint of a path (which is colored red) to another vertex (which must be blue, because the red vertices are independent). Thus the coloring of the paths extends to the unique 2 -coloring of $T$. The leaves of $T$ are endpoints of paths, so they are all red. + +## §2.3 TSTST 2021/6, proposed by Nikolai Beluhov + +Available online at https://aops.com/community/p23864189. + +## Problem statement + +Triangles $A B C$ and $D E F$ share circumcircle $\Omega$ and incircle $\omega$ so that points $A, F$, $B, D, C$, and $E$ occur in this order along $\Omega$. Let $\Delta_{A}$ be the triangle formed by lines $A B, A C$, and $E F$, and define triangles $\Delta_{B}, \Delta_{C}, \ldots, \Delta_{F}$ similarly. Furthermore, let $\Omega_{A}$ and $\omega_{A}$ be the circumcircle and incircle of triangle $\Delta_{A}$, respectively, and define circles $\Omega_{B}, \omega_{B}, \ldots, \Omega_{F}, \omega_{F}$ similarly. +(a) Prove that the two common external tangents to circles $\Omega_{A}$ and $\Omega_{D}$ and the two common external tangents to circles $\omega_{A}$ and $\omega_{D}$ are either concurrent or pairwise parallel. +(b) Suppose that these four lines meet at point $T_{A}$, and define points $T_{B}$ and $T_{C}$ similarly. Prove that points $T_{A}, T_{B}$, and $T_{C}$ are collinear. +![](https://cdn.mathpix.com/cropped/2024_11_19_148cb373ffbe5370bbc6g-17.jpg?height=818&width=754&top_left_y=1150&top_left_x=651) + +Let $I$ and $r$ be the center and radius of $\omega$, and let $O$ and $R$ be the center and radius of $\Omega$. Let $O_{A}$ and $I_{A}$ be the circumcenter and incenter of triangle $\Delta_{A}$, and define $O_{B}$, $I_{B}, \ldots, I_{F}$ similarly. Let $\omega$ touch $E F$ at $A_{1}$, and define $B_{1}, C_{1}, \ldots, F_{1}$ similarly. + +I Part (a). All solutions to part (a) will prove the stronger claim that + +$$ +\left(\Omega_{A} \cup \omega_{A}\right) \sim\left(\Omega_{D} \cup \omega_{D}\right) +$$ + +The four lines will concur at the homothetic center of these figures (possibly at infinity). +Solution 1 (author) Let the second tangent to $\omega$ parallel to $E F$ meet lines $A B$ and $A C$ at $P$ and $Q$, respectively, and let the second tangent to $\omega$ parallel to $B C$ meet lines $D E$ and $D F$ at $R$ and $S$, respectively. Furthermore, let $\omega$ touch $P Q$ and $R S$ at $U$ and $V$, respectively. + +Let $h$ be inversion with respect to $\omega$. Then $h$ maps $A, B$, and $C$ onto the midpoints of the sides of triangle $D_{1} E_{1} F_{1}$. So $h$ maps $k$ onto the Euler circle of triangle $D_{1} E_{1} F_{1}$. +Similarly, $h$ maps $k$ onto the Euler circle of triangle $A_{1} B_{1} C_{1}$. Therefore, triangles $A_{1} B_{1} C_{1}$ and $D_{1} E_{1} F_{1}$ share a common nine-point circle $\gamma$. Let $K$ be its center; its radius equals $\frac{1}{2} r$. + +Let $H$ be the reflection of $I$ in $K$. Then $H$ is the common orthocenter of triangles $A_{1} B_{1} C_{1}$ and $D_{1} E_{1} F_{1}$. + +Let $\gamma_{U}$ of center $K_{U}$ and radius $\frac{1}{2} r$ be the Euler circle of triangle $U E_{1} F_{1}$, and let $\gamma_{V}$ of center $K_{V}$ and radius $\frac{1}{2} r$ be the Euler circle of triangle $V B_{1} C_{1}$. + +Let $H_{U}$ be the orthocenter of triangle $U E_{1} F_{1}$. Since quadrilateral $D_{1} E_{1} F_{1} U$ is cyclic, vectors $\overrightarrow{H H_{U}}$ and $\overrightarrow{D_{1} U}$ are equal. Consequently, $\overrightarrow{K K_{U}}=\frac{1}{2} \overrightarrow{D_{1} U}$. Similarly, $\overrightarrow{K K_{V}}=$ $\frac{1}{2} \overrightarrow{A_{1} V}$. +Since both of $A_{1} U$ and $D_{1} V$ are diameters in $\omega$, vectors $\overrightarrow{D_{1} U}$ and $\overrightarrow{A_{1} V}$ are equal. Therefore, $K_{U}$ and $K_{V}$ coincide, and so do $\gamma_{U}$ and $\gamma_{V}$. + +As above, $h$ maps $\gamma_{U}$ onto the circumcircle of triangle $A P Q$ and $\gamma_{V}$ onto the circumcircle of triangle $D R S$. Therefore, triangles $A P Q$ and $D R S$ are inscribed inside the same circle $\Omega_{A D}$. + +Since $E F$ and $P Q$ are parallel, triangles $\Delta_{A}$ and $A P Q$ are homothetic, and so are figures $\Omega_{A} \cup \omega_{A}$ and $\Omega_{A D} \cup \omega$. Consequently, we have + +$$ +\left(\Omega_{A} \cup \omega_{A}\right) \sim\left(\Omega_{A D} \cup \omega\right) \sim\left(\Omega_{D} \cup \omega_{D}\right), +$$ + +which solves part (a). +Solution 2 (Michael Ren) As in the previous solution, let the second tangent to $\omega$ parallel to $E F$ meet $A B$ and $A C$ at $P$ and $Q$, respectively. Let $(A P Q)$ meet $\Omega$ again at $D^{\prime}$, so that $D^{\prime}$ is the Miquel point of $\{A B, A C, B C, P Q\}$. Since the quadrilateral formed by these lines has incircle $\omega$, it is classical that $D^{\prime} I$ bisects $\angle P D^{\prime} C$ and $B D^{\prime} Q$ (e.g. by DDIT). + +Let $\ell$ be the tangent to $\Omega$ at $D^{\prime}$ and $D^{\prime} I$ meet $\Omega$ again at $M$. We have + +$$ +\measuredangle\left(\ell, D^{\prime} B\right)=\measuredangle D^{\prime} C B=\measuredangle D^{\prime} Q P=\measuredangle\left(D^{\prime} Q, E F\right) . +$$ + +Therefore $D^{\prime} I$ also bisects the angle between $\ell$ and the line parallel to $E F$ through $D^{\prime}$. This means that $M$ is one of the arc midpoints of $E F$. Additionally, $D^{\prime}$ lies on $\operatorname{arc} B C$ not containing $A$, so $D^{\prime}=D$. + +Similarly, letting the second tangent to $\omega$ parallel to $B C$ meet $D E$ and $D F$ again at $R$ and $S$, we have $A D R S$ cyclic. + +## Lemma + +There exists a circle $\Omega_{A D}$ tangent to $\Omega_{A}$ and $\Omega_{D}$ at $A$ and $D$, respectively. + +Proof. (This step is due to Ankan Bhattacharya.) It is equivalent to have $\measuredangle O A O_{A}=$ $\measuredangle O_{D} D O$. Taking isogonals with respect to the shared angle of $\triangle A B C$ and $\Delta_{A}$, we see that + +$$ +\measuredangle O A O_{A}=\measuredangle(\perp E F, \perp B C)=\measuredangle(E F, B C) . +$$ + +(Here, $\perp E F$ means the direction perpendicular to $E F$.) By symmetry, this is equal to $\measuredangle O_{D} D O$. + +The circle $(A D P Q)$ passes through $A$ and $D$, and is tangent to $\Omega_{A}$ by homothety. Therefore it coincides with $\Omega_{A D}$, as does ( $A D R S$ ). Like the previous solution, we finish with + +$$ +\left(\Omega_{A} \cup \omega_{A}\right) \sim\left(\Omega_{A D} \cup \omega\right) \sim\left(\Omega_{D} \cup \omega_{D}\right) . +$$ + +Solution 3 (Andrew Gu) Construct triangles homothetic to $\Delta_{A}$ and $\Delta_{D}$ (with positive ratio) which have the same circumcircle; it suffices to show that these copies have the same incircle as well. Let $O$ be the center of this common circumcircle, which we take to be the origin, and $M_{X Y}$ denote the point on the circle such that the tangent at that point is parallel to line $X Y$ (from the two possible choices, we make the choice that corresponds to the arc midpoint on $\Omega$, e.g. $M_{A B}$ should correspond to the arc midpoint on the internal angle bisector of $A C B$ ). The left diagram below shows the original triangles $A B C$ and $D E F$, while the right diagram shows the triangles homothetic to $\Delta_{A}$ and $\Delta_{D}$. +![](https://cdn.mathpix.com/cropped/2024_11_19_148cb373ffbe5370bbc6g-19.jpg?height=566&width=1203&top_left_y=902&top_left_x=432) + +Using the fact that the incenter is the orthocenter of the arc midpoints, the incenter of $\Delta_{A}$ in this reference frame is $M_{A B}+M_{A C}-M_{E F}$ and the incenter of $\Delta_{D}$ in this reference frame is $M_{D E}+M_{D F}-M_{B C}$. Since $A B C$ and $D E F$ share a common incenter, we have + +$$ +M_{A B}+M_{B C}+M_{C A}=M_{D E}+M_{E F}+M_{F D} +$$ + +Thus the copies of $\Delta_{A}$ and $\Delta_{D}$ have the same incenter, and therefore the same incircle as well (Euler's theorem determines the inradius). + +【 Part (b). We present several solutions for this part of the problem. Solutions 3 and 4 require solving part (a) first, while the others do not. Solutions 1,4 , and 5 define $T_{A}$ solely as the exsimilicenter of $\omega_{A}$ and $\omega_{D}$, whereas solutions 2 and 3 define $T_{A}$ solely as the exsimilicenter of $\Omega_{A}$ and $\Omega_{D}$. + +Solution 1 (author) By Monge's theorem applied to $\omega, \omega_{A}$, and $\omega_{D}$, points $A, D$, and $T_{A}$ are collinear. Therefore, $T_{A}=A D \cap I_{A} I_{D}$. + +Let $p$ be pole-and-polar correspondence with respect to $\omega$. Then $p$ maps $A$ onto line $E_{1} F_{1}$ and $D$ onto line $B_{1} C_{1}$. Consequently, $p$ maps line $A D$ onto $X_{A}=B_{1} C_{1} \cap E_{1} F_{1}$. + +Furthermore, $p$ maps the line that bisects the angle formed by lines $A B$ and $E F$ and does not contain $I$ onto the midpoint of segment $A_{1} F_{1}$. Similarly, $p$ maps the line that bisects the angle formed by lines $A C$ and $E F$ and does not contain $I$ onto the midpoint of segment $A_{1} E_{1}$. So $p$ maps $I_{A}$ onto the midline of triangle $A_{1} E_{1} F_{1}$ opposite $A_{1}$. Similarly, $p$ maps $I_{D}$ onto the midline of triangle $D_{1} B_{1} C_{1}$ opposite $D_{1}$. Consequently, $p$ maps line +$I_{A} I_{D}$ onto the intersection point $Y_{A}$ of this pair of midlines, and $p$ maps $T_{A}$ onto line $X_{A} Y_{A}$. + +As in the solution to part (a), let $H$ be the common orthocenter of triangles $A_{1} B_{1} C_{1}$ and $D_{1} E_{1} F_{1}$. Let $H_{A}$ be the foot of the altitude from $A_{1}$ in triangle $A_{1} B_{1} C_{1}$ and let $H_{D}$ be the foot of the altitude from $D_{1}$ in triangle $D_{1} E_{1} F_{1}$. Furthermore, let $L_{A}=H A_{1} \cap E_{1} F_{1}$ and $L_{D}=H D_{1} \cap B_{1} C_{1}$. + +Since the reflection of $H$ in line $B_{1} C_{1}$ lies on $\omega, A_{1} H \cdot H H_{A}$ equals half the power of $H$ with respect to $\omega$. Similarly, $D_{1} H \cdot H H_{D}$ equals half the power of $H$ with respect to $\omega$. + +Then $A_{1} H \cdot H H_{A}=D_{1} H \cdot H H_{D}$ and $A_{1} H H_{D} \sim D_{1} H H_{A}$. Since $\angle H H_{D} L_{A}=90^{\circ}=$ $\angle H H_{A} L_{D}$, figures $A_{1} H H_{D} L_{A}$ and $D_{1} H H_{A} L_{D}$ are similar as well. Consequently, + +$$ +\frac{H L_{A}}{L_{A} A_{1}}=\frac{H L_{D}}{L_{D} D_{1}}=s +$$ + +as a signed ratio. +Let the line through $A_{1}$ parallel to $E_{1} F_{1}$ and the line through $D_{1}$ parallel to $B_{1} C_{1}$ meet at $Z_{A}$. Then $H X_{A} / X_{A} Z_{A}=s$ and $Y_{A}$ is the midpoint of segment $X_{A} Z_{A}$. Therefore, $H$ lies on line $X_{A} Y_{A}$. This means that $T_{A}$ lies on the polar of $H$ with respect to $\omega$, and by symmetry so do $T_{B}$ and $T_{C}$. + +Solution 2 (author) As in the first solution to part (a), let $h$ be inversion with respect to $\omega$, let $\gamma$ of center $K$ be the common Euler circle of triangles $A_{1} B_{1} C_{1}$ and $D_{1} E_{1} F_{1}$, and let $H$ be their common orthocenter. + +Again as in the solution to part (a), $h$ maps $\Omega_{A}$ onto the nine-point circle $\gamma_{A}$ of triangle $A_{1} E_{1} F_{1}$ and $\Omega_{D}$ onto the nine-point circle $\gamma_{D}$ of triangle $D_{1} B_{1} C_{1}$. + +Let $K_{A}$ and $K_{D}$ be the centers of $\gamma_{A}$ and $\gamma_{D}$, respectively, and let $\ell_{A}$ be the perpendicular bisector of segment $K_{A} K_{D}$. Since $\gamma_{A}$ and $\gamma_{D}$ are congruent (both of them are of radius $\left.\frac{1}{2} r\right)$, they are reflections of each other across $\ell_{A}$. + +Inversion $h$ maps the two common external tangents of $\Omega_{A}$ and $\Omega_{D}$ onto the two circles $\alpha$ and $\beta$ through $I$ that are tangent to both of $\gamma_{A}$ and $\gamma_{D}$ in the same way. (That is, either internally to both or externally to both.) Consequently, $\alpha$ and $\beta$ are reflections of each other in $\ell_{A}$ and so their second point of intersection $S_{A}$, which $h$ maps $T_{A}$ onto, is the reflection of $I$ in $\ell_{A}$. + +Define $\ell_{B}, \ell_{C}, S_{B}$, and $S_{C}$ similarly. Then $S_{B}$ is the reflection of $I$ in $\ell_{B}$ and $S_{C}$ is the reflection of $I$ in $\ell_{C}$. + +As in the solution to part (a), $\overrightarrow{K K_{A}}=\frac{1}{2} \overrightarrow{D_{1} A_{1}}$ and $\overrightarrow{K K_{D}}=\frac{1}{2} \overrightarrow{A_{1} D_{1}}$. Consequently, $K$ is the midpoint of segment $K_{A} K_{D}$ and so $K$ lies on $\ell_{A}$. Similarly, $K$ lies on $\ell_{B}$ and $\ell_{C}$. + +Therefore, all four points $I, S_{A}, S_{B}$, and $S_{C}$ lie on the circle of center $K$ that contains $I$. (This is also the circle on diameter $I H$.) Since points $S_{A}, S_{B}$, and $S_{C}$ are concyclic with $I$, their images $T_{A}, T_{B}$, and $T_{C}$ under $h$ are collinear, and the solution is complete. + +Solution 3 (Ankan Bhattacharya) From either of the first two solutions to part (a), we know that there is a circle $\Omega_{A D}$ passing through $A$ and $D$ which is (internally) tangent to $\Omega_{A}$ and $\Omega_{D}$. By Monge's theorem applied to $\Omega_{A}, \Omega_{D}$, and $\Omega_{A D}$, it follows that $A, D$, and $T_{A}$ are collinear. + +The inversion at $T_{A}$ swapping $\Omega_{A}$ with $\Omega_{D}$ also swaps $A$ with $D$ because $T_{A}$ lies on $A D$ and $A$ is not homologous to $D$. Let $\Omega_{A}$ and $\Omega_{D}$ meet $\Omega$ again at $L_{A}$ and $L_{D}$. Since $A D L_{A} L_{D}$ is cyclic, the same inversion at $T_{A}$ also swaps $L_{A}$ and $L_{D}$, so $T_{A}=A D \cap L_{A} L_{D}$. + +By Taiwan TST 2014, $L_{A}$ and $L_{D}$ are the tangency points of the $A$-mixtilinear and $D$-mixtilinear incircles, respectively, with $\Omega$. Therefore $A L_{A} \cap D L_{D}$ is the exsimilicenter $X$ of $\Omega$ and $\omega$. Then $T_{A}, T_{B}$, and $T_{C}$ all lie on the polar of $X$ with respect to $\Omega$. + +Solution 4 (Andrew Gu) We show that $T_{A}$ lies on the radical axis of the point circle at $I$ and $\Omega$, which will solve the problem. Let $I_{A}$ and $I_{D}$ be the centers of $\omega_{A}$ and $\omega_{D}$ respectively. By the Monge's theorem applied to $\omega, \omega_{A}$, and $\omega_{D}$, points $A, D$, and $T_{A}$ are collinear. Additionally, these other triples are collinear: $\left(A, I_{A}, I\right),\left(D, I_{D}, I\right),\left(I_{A}, I_{D}, T\right)$. By Menelaus's theorem, we have + +$$ +\frac{T_{A} D}{T_{A} A}=\frac{I_{A} I}{I_{A} A} \cdot \frac{I_{D} D}{I_{D} I} . +$$ + +If $s$ is the length of the side opposite $A$ in $\Delta_{A}$, then we compute + +$$ +\begin{aligned} +\frac{I_{A} I}{I_{A} A} & =\frac{s / \cos (A / 2)}{r_{A} / \sin (A / 2)} \\ +& =\frac{2 R_{A} \sin (A) \sin (A / 2)}{\cos (A / 2)} \\ +& =\frac{4 R_{A} \sin ^{2}(A / 2)}{r_{A}} \\ +& =\frac{4 R_{A} r^{2}}{r_{A} A I^{2}} . +\end{aligned} +$$ + +From part (a), we know that $\frac{R_{A}}{r_{A}}=\frac{R_{D}}{r_{D}}$. Therefore, doing a similar calculation for $\frac{I_{D} D}{I_{D} I}$, we get + +$$ +\begin{aligned} +\frac{T_{A} D}{T_{A} A} & =\frac{I_{A} I}{I_{A} A} \cdot \frac{I_{D} D}{I_{D} I} \\ +& =\frac{4 R_{A} r^{2}}{r_{A} A I^{2}} \cdot \frac{r_{D} D I^{2}}{4 R_{D} r^{2}} \\ +& =\frac{D I^{2}}{A I^{2}} +\end{aligned} +$$ + +Thus $T_{A}$ is the point where the tangent to $(A I D)$ at $I$ meets $A D$ and $T_{A} I^{2}=T_{A} A \cdot T_{A} D$. This shows what we claimed at the start. + +Solution 5 (Ankit Bisain) As in the previous solution, it suffices to show that $\frac{I_{A} I}{A I_{A}} \cdot \frac{D I_{D}}{I_{D} I}=\frac{D I^{2}}{A I^{2}}$. Let $A I$ and $D I$ meet $\Omega$ again at $M$ and $N$, respectively. Let $\ell$ be the line parallel to $B C$ and tangent to $\omega$ but different from $B C$. Then + +$$ +\frac{D I_{D}}{I_{D} I}=\frac{d(D, B C)}{d(B C, \ell)}=\frac{D B \cdot D C / 2 R}{2 r}=\frac{M I^{2}-M D^{2}}{4 R r} +$$ + +Since $I D M \sim I A N$, we have + +$$ +\frac{D I_{D}}{I_{D} I} \cdot \frac{I_{A} I}{A I_{A}}=\frac{M I^{2}-M D^{2}}{N I^{2}-N A^{2}}=\frac{D I^{2}}{A I^{2}} +$$ + +as desired. +Remark (Author comments on generalization of part (b) with a circumscribed hexagram). Let triangles $A B C$ and $D E F$ be circumscribed about the same circle $\omega$ so that they form a hexagram. However, we do not require anymore that they are inscribed in the same circle. + +Define circles $\Omega_{A}, \omega_{A}, \ldots, \omega_{F}$ as in the problem. Let $T_{A}^{\text {Circ }}$ be the intersection point of the two common external tangents to circles $\Omega_{A}$ and $\Omega_{D}$, and define points $T_{B}^{\text {Circ }}$ and $T_{C}^{\text {Circ }}$ similarly. Also let $T_{A}^{\text {In }}$ be the intersection point of the two common external tangents to circles $\omega_{A}$ and $\omega_{D}$, and define points $T_{B}^{\mathrm{In}}$ and $T_{C}^{\mathrm{In}}$ similarly. + +Then points $T_{A}^{\text {Circ }}, T_{B}^{\text {Circ }}$, and $T_{C}^{\mathrm{Circ}}$ are collinear and points $T_{A}^{\mathrm{In}}, T_{B}^{\mathrm{In}}$, and $T_{C}^{\mathrm{In}}$ are also collinear. + +The second solution to part (b) of the problem works also for the circumcircles part of the generalisation. To see that segments $K_{A} K_{D}, K_{B} K_{E}$, and $K_{C} K_{F}$ still have a common midpoint, let $M$ be the centroid of points $A, B, C, D, E$, and $F$. Then the midpoint of segment $K_{A} K_{D}$ divides segment $O M$ externally in ratio $3: 1$, and so do the other two midpoints as well. + +For the incircles part of the generalisation, we start out as in the first solution to part (b) of the problem, and eventually we reduce everything to the following: + +Let points $A_{1}, B_{1}, C_{1}, D_{1}, E_{1}$, and $F_{1}$ lie on circle $\omega$. Let lines $B_{1} C_{1}$ and $E_{1} F_{1}$ meet at point $X_{A}$, let the line through $A_{1}$ parallel to $B_{1} C_{1}$ and the line through $D_{1}$ parallel to $E_{1} F_{1}$ meet at point $Z_{A}$, and define points $X_{B}, Z_{B}, X_{C}$, and $Z_{C}$ similarly. Then lines $X_{A} Z_{A}$, $X_{B} Z_{B}$, and $X_{C} Z_{C}$ are concurrent. + +Take $\omega$ as the unit circle and assign complex numbers $u, v, w, x, y$, and $z$ to points $A_{1}$, $F_{1}, B_{1}, D_{1}, C_{1}$, and $E_{1}$, respectively, so that when we permute $u, v, w, x, y$, and $z$ cyclically the configuration remains unchanged. Then by standard complex bash formulas we obtain that each two out of our three lines meet at $\varphi / \psi$, where + +$$ +\varphi=\sum_{\mathrm{Cyc}} u^{2} v w(w x-w y+x y)(y-z) +$$ + +and + +$$ +\psi=-u^{2} w^{2} y^{2}-v^{2} x^{2} z^{2}-4 u v w x y z+\sum_{\mathrm{Cyc}} u^{2}(v w x y-v w x z+v w y z-v x y z+w x y z) +$$ + +(But the calculations were too difficult for me to do by hand, so I used SymPy.) + +Remark (Author comments on generalization of part (b) with an inscribed hexagram). Let triangles $A B C$ and $D E F$ be inscribed inside the same circle $\Omega$ so that they form a hexagram. However, we do not require anymore that they are circumscribed about the same circle. + +Define points $T_{A}^{\text {Circ }}, T_{B}^{\text {Circ }}, \ldots, T_{C}^{\mathrm{In}}$ as in the previous remark. It looks like once again points $T_{A}^{\text {Circ }}, T_{B}^{\text {Circ }}$, and $T_{C}^{\text {Circ }}$ are collinear and points $T_{A}^{\mathrm{In}}, T_{B}^{\mathrm{In}}$, and $T_{C}^{\mathrm{In}}$ are also collinear. However, I do not have proofs of these claims. + +Remark (Further generalization from Andrew Gu). Let $A B C$ and $D E F$ be triangles which share an inconic, or equivalently share a circumconic. Define points $T_{A}^{\text {Circ }}, T_{B}^{\text {Circ }}, \ldots, T_{C}^{\mathrm{In}}$ as in the previous remarks. Then it is conjectured that points $T_{A}^{\text {Circ }}, T_{B}^{\text {Circ }}$, and $T_{C}^{\text {Circ }}$ are collinear and points $T_{A}^{\mathrm{In}}, T_{B}^{\mathrm{In}}$, and $T_{C}^{\mathrm{In}}$ are also collinear. (Note that extraversion may be required depending on the configuration of points, e.g. excircles instead of incircles.) Additionally, it appears that the insimilicenters of the circumcircles lie on a line perpendicular to the line through $T_{A}^{\text {Circ }}, T_{B}^{\text {Circ }}$, and $T_{C}^{\text {Circ }}$. + +## §3 Solutions to Day 3 + +## §3.1 TSTST 2021/7, proposed by Ankit Bisain, Holden Mui + +Available online at https://aops.com/community/p24130213. + +## Problem statement + +Let $M$ be a finite set of lattice points and $n$ be a positive integer. A mine-avoiding path is a path of lattice points with length $n$, beginning at $(0,0)$ and ending at a point on the line $x+y=n$, that does not contain any point in $M$. Prove that if there exists a mine-avoiding path, then there exist at least $2^{n-|M|}$ mine-avoiding paths. + +We present two approaches. + +【 Solution 1. We prove the statement by induction on $n$. We use $n=0$ as a base case, where the statement follows from $1 \geq 2^{-|M|}$. For the inductive step, let $n>0$. There exists at least one mine-avoiding path, which must pass through either $(0,1)$ or $(1,0)$. We consider two cases: + +Case 1: there exist mine-avoiding paths starting at both $(1,0)$ and $(0,1)$. +By the inductive hypothesis, there are at least $2^{n-1-|M|}$ mine-avoiding paths starting from each of $(1,0)$ and $(0,1)$. Then the total number of mine-avoiding paths is at least $2^{n-1-|M|}+2^{n-1-|M|}=2^{n-|M|}$. + +Case 2: only one of $(1,0)$ and $(0,1)$ is on a mine-avoiding path. +Without loss of generality, suppose no mine-avoiding path starts at $(0,1)$. Then some element of $M$ must be of the form $(0, k)$ for $1 \leq k \leq n$ in order to block the path along the $y$-axis. This element can be ignored for any mine-avoiding path starting at (1,0). By the inductive hypothesis, there are at least $2^{n-1-(|M|-1)}=2^{n-|M|}$ mine-avoiding paths. + +This completes the induction step, which solves the problem. + +## 【 Solution 2. + +## Lemma + +If $|M|d$, the pattern changes to + +$$ +S_{n}=\sum_{j=1}^{d}(-1)^{j+1} c_{j} S_{n-j} +$$ + +## Lemma + +All of the $c_{i}$ are integers except for $c_{d}$. Furthermore, $c_{d}$ is $1 / p$ times an integer. + +Proof. The $q$ th cyclotomic polynomial is + +$$ +\Phi_{q}(x)=1+x^{p^{r-1}}+x^{2 p^{r-1}}+\cdots+x^{(p-1) p^{r-1}} +$$ + +The polynomial + +$$ +Q(x)=1+(1+x)^{p^{r-1}}+(1+x)^{2 p^{r-1}}+\cdots+(1+x)^{(p-1) p^{r-1}} +$$ + +has roots $\omega-1$ for $\omega \in S_{q}$, so it is equal to $p(-x)^{d} P(-1 / x)$ by comparing constant coefficients. Comparing the remaining coefficients, we find that $c_{n}$ is $1 / p$ times the $x^{n}$ coefficient of $Q$. + +Since $(x+y)^{p} \equiv x^{p}+y^{p}(\bmod p)$, we conclude that, modulo $p$, + +$$ +\begin{aligned} +Q(x) & \equiv 1+\left(1+x^{p^{r-1}}\right)+\left(1+x^{p^{r-1}}\right)^{2}+\cdots+\left(1+x^{p^{r-1}}\right)^{p-1} \\ +& \equiv\left[\left(1+x^{p^{r-1}}\right)^{p}-1\right] / x^{p^{r-1}} +\end{aligned} +$$ + +Since $\binom{p}{j}$ is a multiple of $p$ when $0d$ : + +- If $\nu_{p}\left(S_{n-j}\right) \geq \ell$ for all $1 \leq j \leq d$ and $\nu_{p}\left(S_{n-d}\right) \geq \ell+1$, then $\nu_{p}\left(S_{n}\right) \geq \ell$. +- If $\nu_{p}\left(S_{n-j}\right) \geq \ell$ for all $1 \leq j \leq d$ and $\nu_{p}\left(S_{n-d}\right)=\ell$, then $\nu_{p}\left(S_{n}\right)=\ell-1$. + +Together, these prove the claim by induction. +By the claim, the smallest $n$ for which $\nu_{p}\left(S_{n}\right)<0$ (equivalent to $S_{n}$ not being an integer, by the recurrences) is + +$$ +n=(r-1) d+m+1=((p-1) r-1) p^{r-1}+1 +$$ + +Remark. The original proposal was the following more general version: +Let $n$ be an integer with prime power factorization $q_{1} \cdots q_{m}$. Let $S_{n}$ denote the set of primitive $n$th roots of unity. Find all tuples of nonnegative integers $\left(z_{1}, \ldots, z_{m}\right)$ such that + +$$ +\sum_{\omega \in S_{n}} \frac{f(\omega)}{\left(1-\omega^{n / q_{1}}\right)^{z_{1}} \cdots\left(1-\omega^{n / q_{m}}\right)^{z_{m}}} \in \mathbb{Z} +$$ + +for all polynomials $f \in \mathbb{Z}[x]$. +The maximal $z_{i}$ are exponents in the prime ideal factorization of the different ideal of the cyclotomic extension $\mathbb{Q}\left(\zeta_{n}\right) / \mathbb{Q}$. + +Remark. Let $F=\left(x^{p}-1\right) /(x-1)$ be the minimal polynomial of $\zeta_{p}=e^{2 \pi i / p}$ over $\mathbb{Q}$. A calculation of Euler shows that + +$$ +\left(\mathbb{Z}\left[\zeta_{p}\right]\right)^{*}:=\left\{\alpha=g\left(\zeta_{p}\right) \in \mathbb{Q}\left[\zeta_{p}\right]: \sum_{\omega \in S_{p}} f(\omega) g(\omega) \in \mathbb{Z} \forall f \in \mathbb{Z}[x]\right\}=\frac{1}{F^{\prime}\left(\zeta_{p}\right)} \cdot \mathbb{Z}\left[\zeta_{p}\right], +$$ + +where + +$$ +F^{\prime}\left(\zeta_{p}\right)=\frac{p \zeta_{p}^{p-1}-\left[1+\zeta_{p}+\cdots+\zeta_{p}^{p-1}\right]}{1-\zeta_{p}}=p\left(1-\zeta_{p}\right)^{-1} \zeta_{p}^{p-1} +$$ + +is $\left(1-\zeta_{p}\right)^{[p-1]-1}=\left(1-\zeta_{p}\right)^{p-2}$ times a unit of $\mathbb{Z}\left[\zeta_{p}\right]$. Here, $\left(\mathbb{Z}\left[\zeta_{p}\right]\right)^{*}$ is the dual lattice of $\mathbb{Z}\left[\zeta_{p}\right]$. + +Remark. Let $K=\mathbb{Q}(\omega)$, so $(p)$ factors as $(1-\omega)^{p-1}$ in the ring of integers $\mathcal{O}_{K}$ (which, for cyclotomic fields, can be shown to be $\mathbb{Z}[\omega])$. In particular, the ramification index $e$ of $(1-\omega)$ over $p$ is the exponent, $p-1$. Since $e=p-1$ is not divisible by $p$, we have so-called tame ramification. Now by the ramification theory of Dedekind's different ideal, the exponent $z_{1}$ that works when $n=p$ is $e-1=p-2$. + +Higher prime powers are more interesting because of wild ramification: $p$ divides $\phi\left(p^{r}\right)=$ $p^{r-1}(p-1)$ if and only if $r>1$. (This is a similar phenomena to how Hensel's lemma for $x^{2}-c$ is more interesting mod powers of 2 than mod odd prime powers.) + +Remark. Let $F=\left(x^{q}-1\right) /\left(x^{q / p}-1\right)$ be the minimal polynomial of $\zeta_{q}=e^{2 \pi i / q}$ over $\mathbb{Q}$. The aforementioned calculation of Euler shows that + +$$ +\left(\mathbb{Z}\left[\zeta_{q}\right]\right)^{*}:=\left\{\alpha=g\left(\zeta_{q}\right) \in \mathbb{Q}\left[\zeta_{q}\right]: \sum_{\omega \in S_{q}} f(\omega) g(\omega) \in \mathbb{Z} \forall f \in \mathbb{Z}[x]\right\}=\frac{1}{F^{\prime}\left(\zeta_{q}\right)} \cdot \mathbb{Z}\left[\zeta_{q}\right] +$$ + +where the chain rule implies (using the computation from the prime case) + +$$ +F^{\prime}\left(\zeta_{q}\right)=\left[p\left(1-\zeta_{p}\right)^{-1} \zeta_{p}^{p-1}\right] \cdot \frac{q}{p} \zeta_{q}^{(q / p)-1}=q\left(1-\zeta_{p}\right)^{-1} \zeta_{q}^{-1} +$$ + +is $\left(1-\zeta_{q}\right)^{r \phi(q)-q / p}=\left(1-\zeta_{q}\right)^{z_{q}}$ times a unit of $\mathbb{Z}\left[\zeta_{q}\right]$. + diff --git a/USA_TSTST/md/en-sols-TSTST-2022.md b/USA_TSTST/md/en-sols-TSTST-2022.md new file mode 100644 index 0000000000000000000000000000000000000000..06494c30d0629d2b49cad96fff265e16d4d4d597 --- /dev/null +++ b/USA_TSTST/md/en-sols-TSTST-2022.md @@ -0,0 +1,618 @@ +# USA TSTST 2022 Solutions
United States of America - TST Selection Test
Andrew Gu and Evan Chen
64 ${ }^{\text {th }}$ IMO 2023 Japan and $12^{\text {th }}$ EGMO 2023 Slovenia + +## Contents + +0 Problems ..... 2 +1 Solutions to Day 1 ..... 4 +1.1 TSTST 2022/1, proposed by Holden Mui ..... 4 +1.2 TSTST 2022/2, proposed by Hongzhou Lin ..... 7 +1.3 TSTST 2022/3 ..... 9 +2 Solutions to Day 2 ..... 11 +2.1 TSTST 2022/4, proposed by Merlijn Staps ..... 11 +2.2 TSTST 2022/5, proposed by Ray Li ..... 12 +2.3 TSTST 2022/6, proposed by Hongzhou Lin ..... 14 +3 Solutions to Day 3 ..... 17 +3.1 TSTST 2022/7, proposed by Merlijn Staps ..... 17 +3.2 TSTST 2022/8, proposed by Merlijn Staps ..... 18 +3.3 TSTST 2022/9, proposed by Vincent Huang ..... 19 + +## §0 Problems + +1. Let $n$ be a positive integer. Find the smallest positive integer $k$ such that for any set $S$ of $n$ points in the interior of the unit square, there exists a set of $k$ rectangles such that the following hold: + +- The sides of each rectangle are parallel to the sides of the unit square. +- Each point in $S$ is not in the interior of any rectangle. +- Each point in the interior of the unit square but not in $S$ is in the interior of at least one of the $k$ rectangles. +(The interior of a polygon does not contain its boundary.) + +2. Let $A B C$ be a triangle. Let $\theta$ be a fixed angle for which + +$$ +\theta<\frac{1}{2} \min (\angle A, \angle B, \angle C) +$$ + +Points $S_{A}$ and $T_{A}$ lie on segment $B C$ such that $\angle B A S_{A}=\angle T_{A} A C=\theta$. Let $P_{A}$ and $Q_{A}$ be the feet from $B$ and $C$ to $\overline{A S_{A}}$ and $\overline{A T_{A}}$ respectively. Then $\ell_{A}$ is defined as the perpendicular bisector of $\overline{P_{A} Q_{A}}$. +Define $\ell_{B}$ and $\ell_{C}$ analogously by repeating this construction two more times (using the same value of $\theta$ ). Prove that $\ell_{A}, \ell_{B}$, and $\ell_{C}$ are concurrent or all parallel. +3. Determine all positive integers $N$ for which there exists a strictly increasing sequence of positive integers $s_{0}1$ be a fixed positive integer. Prove that if $n$ is a sufficiently large positive integer, there exists a sequence of integers with the following properties: + +- Each element of the sequence is between 1 and $n$, inclusive. +- For any two different contiguous subsequences of the sequence with length between 2 and $k$ inclusive, the multisets of values in those two subsequences is not the same. +- The sequence has length at least $0.499 n^{2}$. + + +## §1 Solutions to Day 1 + +## §1.1 TSTST 2022/1, proposed by Holden Mui + +Available online at https://aops.com/community/p25516960. + +## Problem statement + +Let $n$ be a positive integer. Find the smallest positive integer $k$ such that for any set $S$ of $n$ points in the interior of the unit square, there exists a set of $k$ rectangles such that the following hold: + +- The sides of each rectangle are parallel to the sides of the unit square. +- Each point in $S$ is not in the interior of any rectangle. +- Each point in the interior of the unit square but not in $S$ is in the interior of at least one of the $k$ rectangles. +(The interior of a polygon does not contain its boundary.) + +We give the author's solution. In terms of $n$, we wish find the smallest integer $k$ for which $(0,1)^{2} \backslash S$ is always a union of $k$ open rectangles for every set $S \subset(0,1)^{2}$ of size $n$. + +We claim the answer is $k=2 n+2$. +The lower bound is given by picking + +$$ +S=\left\{\left(s_{1}, s_{1}\right),\left(s_{2}, s_{2}\right), \ldots,\left(s_{n}, s_{n}\right)\right\} +$$ + +for some real numbers $00$. The four rectangles covering each of + +$$ +\left(s_{1}-\varepsilon, s_{1}\right),\left(s_{1}, s_{1}-\varepsilon\right),\left(s_{n}+\varepsilon, s_{n}\right),\left(s_{n}, s_{n}+\varepsilon\right) +$$ + +cannot cover any other points in $S^{\prime}$; all other rectangles can only cover at most 2 points in $S^{\prime}$, giving a bound of + +$$ +k \geq 4+\frac{\left|S^{\prime}\right|-4}{2}=2 n+2 +$$ + +![](https://cdn.mathpix.com/cropped/2024_11_19_1799270e91f74486a977g-05.jpg?height=812&width=806&top_left_y=242&top_left_x=628) + +To prove that $2 n+2$ rectangles are sufficient, assume that the number of distinct $y$-coordinates is at least the number of distinct $x$-coordinates. Let + +$$ +0=x_{0}0$ gives a total of + +$$ +(m+n+2)+(n-m)=2 n+2 +$$ + +rectangles. + +## §1.2 TSTST 2022/2, proposed by Hongzhou Lin + +Available online at https://aops.com/community/p25516988. + +## Problem statement + +Let $A B C$ be a triangle. Let $\theta$ be a fixed angle for which + +$$ +\theta<\frac{1}{2} \min (\angle A, \angle B, \angle C) +$$ + +Points $S_{A}$ and $T_{A}$ lie on segment $B C$ such that $\angle B A S_{A}=\angle T_{A} A C=\theta$. Let $P_{A}$ and $Q_{A}$ be the feet from $B$ and $C$ to $\overline{A S_{A}}$ and $\overline{A T_{A}}$ respectively. Then $\ell_{A}$ is defined as the perpendicular bisector of $\overline{P_{A} Q_{A}}$. + +Define $\ell_{B}$ and $\ell_{C}$ analogously by repeating this construction two more times (using the same value of $\theta$ ). Prove that $\ell_{A}, \ell_{B}$, and $\ell_{C}$ are concurrent or all parallel. + +We discard the points $S_{A}$ and $T_{A}$ since they are only there to direct the angles correctly in the problem statement. + +【 First solution, by author. Let $X$ be the projection from $C$ to $A P_{A}, Y$ be the projection from $B$ to $A Q_{A}$. +![](https://cdn.mathpix.com/cropped/2024_11_19_1799270e91f74486a977g-07.jpg?height=809&width=792&top_left_y=1292&top_left_x=632) + +Claim - Line $\ell_{A}$ passes through $M_{A}$, the midpoint of $B C$. Also, quadrilateral $P_{A} Q_{A} Y X$ is cyclic with circumcenter $M_{A}$. + +Proof. Since + +$$ +A P_{A} \cdot A X=A B \cdot A C \cdot \cos \theta \cos (\angle A-\theta)=A Q_{A} \cdot A Y +$$ + +it follows that $P_{A}, Q_{A}, Y, X$ are concyclic by power of a point. Moreover, by projection, the perpendicular bisector of $P_{A} X$ passes through $M_{A}$, similar for $Q_{A} Y$, implying that $M_{A}$ is the center of $P_{A} Q_{A} Y X$. Hence $\ell_{A}$ passes through $M_{A}$. + +$$ +\text { Claim }-\measuredangle\left(M_{A} M_{C}, \ell_{A}\right)=\measuredangle Y P_{A} Q_{A} +$$ + +Proof. Indeed, $\ell_{A} \perp P_{A} Q_{A}$, and $M_{A} M_{C} \perp P_{A} Y$ (since $M_{A} P_{A}=M_{A} Y$ from $\left(P_{A} Q_{A} Y_{A} X\right)$ and $M_{C} P_{A}=M_{C} M_{A}=M_{C} Y$ from the circle with diameter $\left.A B\right)$. Hence $\measuredangle\left(M_{A} M_{C}, \ell_{A}\right)=$ $\measuredangle\left(P_{A} Y, P_{A} Q_{A}\right)=\measuredangle Y P_{A} Q_{A}$. + +Therefore, + +$$ +\frac{\sin \angle\left(M_{A} M_{C}, \ell_{A}\right)}{\sin \angle\left(\ell_{A}, M_{A} M_{B}\right)}=\frac{\sin \angle Y P_{A} Q_{A}}{\sin \angle P_{A} Q_{A} X}=\frac{Y Q_{A}}{X P_{A}}=\frac{B C \sin (\angle C+\theta)}{B C \sin (\angle B+\theta)}=\frac{\sin (\angle C+\theta)}{\sin (\angle B+\theta)}, +$$ + +and we conclude by trig Ceva theorem. +【 Second solution via Jacobi, by Maxim Li. Let $D$ be the foot of the $A$-altitude. Note that line $B C$ is the external angle bisector of $\angle P_{A} D Q_{A}$. + +Claim - $\left(D P_{A} Q_{A}\right)$ passes through the midpoint $M_{A}$ of $B C$. +Proof. Perform $\sqrt{b c}$ inversion. Then the intersection of $B C$ and $\left(D P_{A} Q_{A}\right)$ maps to the second intersection of $(A B C)$ and $\left(A^{\prime} P_{A} Q_{A}\right)$, where $A^{\prime}$ is the antipode to $A$ on $(A B C)$, i.e. the center of spiral similarity from $B C$ to $P_{A} Q_{A}$. Since $B P_{A}: C Q_{A}=A B: A C$, we see the center of spiral similarity is the intersection of the $A$-symmedian with $(A B C)$, which is the image of $M_{A}$ in the inversion. + +It follows that $M_{A}$ lies on $\ell_{A}$; we need to identify a second point. We'll use the circumcenter $O_{A}$ of $\left(D P_{A} Q_{A}\right)$. The perpendicular bisector of $D P_{A}$ passes through $M_{C}$; indeed, we can easily show the angle it makes with $M_{C} M_{A}$ is $90^{\circ}-\theta-C$, so $\angle O_{A} M_{C} M_{A}=90-\theta-C$, and then by analogous angle-chasing we can finish with Jacobi's theorem on $\triangle M_{A} M_{B} M_{C}$. + +## §1.3 TSTST 2022/3 + +Available online at https://aops.com/community/p25517008. + +## Problem statement + +Determine all positive integers $N$ for which there exists a strictly increasing sequence of positive integers $s_{0}2^{21}$ possible colorings. If Bob makes less than 22 queries, then he can only output $2^{21}$ possible colorings, which means he is wrong on some coloring. + +Now we show Bob can always win in 22 queries. A key observation is that the set of red points is convex, as is the set of blue points, so if a set of points is all the same color, then their convex hull is all the same color. + +## Lemma + +Let $B_{0}, \ldots, B_{k+1}$ be equally spaced points on a circular arc such that colors of $B_{0}$ and $B_{k+1}$ differ and are known. Then it is possible to determine the colors of $B_{1}$, $\ldots, B_{k}$ in $\left\lceil\log _{2} k\right\rceil$ queries. + +Proof. There exists some $0 \leq i \leq k$ such that $B_{0}, \ldots, B_{i}$ are the same color and $B_{i+1}$, $\ldots, B_{k+1}$ are the same color. (If $i\lceil k / 2\rceil$, then the segment $P B_{i}$ is blue and intersect the segment $B_{0} B_{\lceil k / 2\rceil}$, which is red, contradiction. + +Now the strategy is: Bob picks $A_{1}$. WLOG it is red. Now suppose Bob does not know the colors of $\leq 2^{k}-1$ points $A_{i}, \ldots, A_{j}$ with $j-i+1 \leq 2^{k}-1$ and knows the rest are red. I claim Bob can win in $2 k-1$ queries. First, if $k=1$, there is one point and he wins by querying the point, so the base case holds, so assume $k>1$. Bob queries $A_{i+\lceil(j-i+1) / 2\rceil}$. If it is blue, he finishes in $2 \log _{2}\lceil(j-i+1) / 2\rceil \leq 2(k-1)$ queries by the first lemma, for a total of $2 k-1$ queries. If it is red, he can query one more point and learn some half of $A_{i}, \ldots, A_{j}$ that are red by the second lemma, and then he has reduced it to the case with $\leq 2^{k-1}-1$ points in two queries, at which point we induct. + +## §2.3 TSTST 2022/6, proposed by Hongzhou Lin + +Available online at https://aops.com/community/p25516957. + +## Problem statement + +Let $O$ and $H$ be the circumcenter and orthocenter, respectively, of an acute scalene triangle $A B C$. The perpendicular bisector of $\overline{A H}$ intersects $\overline{A B}$ and $\overline{A C}$ at $X_{A}$ and $Y_{A}$ respectively. Let $K_{A}$ denote the intersection of the circumcircles of triangles $O X_{A} Y_{A}$ and $B O C$ other than $O$. +Define $K_{B}$ and $K_{C}$ analogously by repeating this construction two more times. Prove that $K_{A}, K_{B}, K_{C}$, and $O$ are concyclic. + +We present several approaches. +\ First solution, by author. Let $\odot O X_{A} Y_{A}$ intersects $A B, A C$ again at $U, V$. Then by Reim's theorem $U V C B$ are concyclic. Hence the radical axis of $\odot O X_{A} Y_{A}, \odot O B C$ and $\odot(U V C B)$ are concurrent, i.e. $O K_{A}, B C, U V$ are concurrent, Denote the intersection as $K_{A}^{*}$, which is indeed the inversion of $K_{A}$ with respect to $\odot O$. (The inversion sends $\odot O B C$ to the line $B C$ ). +Let $P_{A}, P_{B}, P_{C}$ be the circumcenters of $\triangle O B C, \triangle O C A, \triangle O A B$ respectively. +Claim $-K_{A}^{*}$ coincides with the intersection of $P_{B} P_{C}$ and $B C$. +Proof. Note that $d(O, B C)=1 / 2 A H=d\left(A, X_{A} Y_{A}\right)$. This means the midpoint $M_{C}$ of $A B$ is equal distance to $X_{A} Y_{A}$ and the line through $O$ parallel to $B C$. Together with $O M_{C} \perp A B$ implies that $\angle M_{C} X_{A} O=\angle B$. Hence $\angle U V O=\angle B=\angle A V U$. Similarly $\angle V U O=\angle A U V$, hence $\triangle A U V \simeq \triangle O U V$. In other words, $U V$ is the perpendicular bisector of $A O$, which pass through $P_{B}, P_{C}$. Hence $K_{A}^{*}$ is indeed $P_{B} P_{C} \cap B C$. + +Finally by Desargue's theorem, it suffices to show that $A P_{A}, B P_{B}, C P_{C}$ are concurrent. Note that + +$$ +\begin{aligned} +& d\left(P_{A}, A B\right)=P_{A} B \sin \left(90^{\circ}+\angle C-\angle A\right) \\ +& d\left(P_{A}, A C\right)=P_{A} C \sin \left(90^{\circ}+\angle B-\angle A\right) +\end{aligned} +$$ + +Hence the symmetric product and trig Ceva finishes the proof. +![](https://cdn.mathpix.com/cropped/2024_11_19_1799270e91f74486a977g-15.jpg?height=1132&width=1194&top_left_y=245&top_left_x=431) + +I Second solution, from Jeffrey Kwan. Let $O_{A}$ be the circumcenter of $\triangle A X_{A} Y_{A}$. The key claim is that: + +Claim $-O_{A} X_{A} Y_{A} O$ is cyclic. + +Proof. Let $D E F$ be the orthic triangle; we will show that $\triangle O X_{A} Y_{A} \sim \triangle D E F$. Indeed, since $A O$ and $A D$ are isogonal, it suffices to note that + +$$ +\frac{A X_{A}}{A B}=\frac{A H / 2}{A D}=\frac{R \cos A}{A D} +$$ + +and so + +$$ +\frac{A O}{A D}=R \cdot \frac{A X_{A}}{A B \cdot R \cos A}=\frac{A X_{A}}{A E}=\frac{A Y_{A}}{A F} +$$ + +Hence $\angle X_{A} O Y_{A}=180^{\circ}-2 \angle A=180^{\circ}-\angle X_{A} O_{A} Y_{A}$, which proves the claim. +Let $P_{A}$ be the circumcenter of $\triangle O B C$, and define $P_{B}, P_{C}$ similarly. By the claim, $A$ is the exsimilicenter of $\left(O X_{A} Y_{A}\right)$ and $(O B C)$, so $A P_{A}$ is the line between their two centers. In particular, $A P_{A}$ is the perpendicular bisector of $O K_{A}$. +![](https://cdn.mathpix.com/cropped/2024_11_19_1799270e91f74486a977g-16.jpg?height=1303&width=1020&top_left_y=245&top_left_x=521) + +Claim $-A P_{A}, B P_{B}, C P_{C}$ concur at $T$. +Proof. The key observation is that $O$ is the incenter of $\triangle P_{A} P_{B} P_{C}$, and that $A, B, C$ are the reflections of $O$ across the sides of $\triangle P_{A} P_{B} P_{C}$. Hence $P_{A} A, P_{B} B, P_{C} C$ concur by Jacobi. + +Now $T$ lies on the perpendicular bisectors of $O K_{A}, O K_{B}$, and $O K_{C}$. Hence $O K_{A} K_{B} K_{C}$ is cyclic with center $T$, as desired. + +## §3 Solutions to Day 3 + +## §3.1 TSTST 2022/7, proposed by Merlijn Staps + +Available online at https://aops.com/community/p25516961. + +## Problem statement + +Let $A B C D$ be a parallelogram. Point $E$ lies on segment $C D$ such that + +$$ +2 \angle A E B=\angle A D B+\angle A C B, +$$ + +and point $F$ lies on segment $B C$ such that + +$$ +2 \angle D F A=\angle D C A+\angle D B A . +$$ + +Let $K$ be the circumcenter of triangle $A B D$. Prove that $K E=K F$. + +Let the circle through $A, B$, and $E$ intersect $C D$ again at $E^{\prime}$, and let the circle through $D$, $A$, and $F$ intersect $B C$ again at $F^{\prime}$. Now $A B E E^{\prime}$ and $D A F^{\prime} F$ are cyclic quadrilaterals with two parallel sides, so they are isosceles trapezoids. From $K A=K B$, it now follows that $K E=K E^{\prime}$, whereas from $K A=K D$ it follows that $K F=K F^{\prime}$. + +Next, let the circle through $A, B$, and $E$ intersect $A C$ again at $S$. Then + +$$ +\angle A S B=\angle A E B=\frac{1}{2}(\angle A D B+\angle A C B)=\frac{1}{2}(\angle A D B+\angle D A C)=\frac{1}{2} \angle A M B, +$$ + +where $M$ is the intersection of $A C$ and $B D$. From $\angle A S B=\frac{1}{2} \angle A M B$, it follows that $M S=M B$, so $S$ is the point on $M C$ such that $M S=M B=M D$. By symmetry, the circle through $A, D$, and $F$ also passes through $S$, and it follows that the line $A S$ is the radical axis of the circles $(A B E)$ and $(A D F)$. + +By power of a point, we now obtain + +$$ +C E \cdot C E^{\prime}=C S \cdot C A=C F \cdot C F^{\prime}, +$$ + +from which it follows that $E, F, E^{\prime}$, and $F^{\prime}$ are concyclic. The segments $E E^{\prime}$ and $F F^{\prime}$ are not parallel, so their perpendicular bisectors only meet at one point, which is $K$. Hence $K E=K F$. + +## §3.2 TSTST 2022/8, proposed by Merlijn Staps + +Available online at https://aops.com/community/p25516968. + +## Problem statement + +Find all functions $f: \mathbb{N} \rightarrow \mathbb{Z}$ such that + +$$ +\left\lfloor\frac{f(m n)}{n}\right\rfloor=f(m) +$$ + +for all positive integers $m, n$. + +There are two families of functions that work: for each $\alpha \in \mathbb{R}$ the function $f(n)=\lfloor\alpha n\rfloor$, and for each $\alpha \in \mathbb{R}$ the function $f(n)=\lceil\alpha n\rceil-1$. (For irrational $\alpha$ these two functions coincide.) It is straightforward to check that these functions indeed work; essentially, this follows from the identity + +$$ +\left\lfloor\frac{\lfloor x n\rfloor}{n}\right\rfloor=\lfloor x\rfloor +$$ + +which holds for all positive integers $n$ and real numbers $x$. +We now show that every function that works must be of one of the above forms. Let $f$ be a function that works, and define the sequence $a_{1}, a_{2}, \ldots$ by $a_{n}=f(n!) / n!$. Applying the give condition with $(n!, n+1)$ yields $a_{n+1} \in\left[a_{n}, a_{n}+\frac{1}{n!}\right)$. It follows that the sequence $a_{1}, a_{2}, \ldots$ is non-decreasing and bounded from above by $a_{1}+e$, so this sequence must converge to some limit $\alpha$. + +If there exists a $k$ such that $a_{k}=\alpha$, then we have $a_{\ell}=\alpha$ for all $\ell>k$. For each positive integer $m$, there exists $\ell>k$ such that $m \mid \ell$ !. Plugging in $m n=\ell$ !, it then follows that + +$$ +f(m)=\left\lfloor\frac{f(\ell!)}{\ell!/ m}\right\rfloor=\lfloor\alpha m\rfloor +$$ + +for all $m$, so $f$ is of the desired form. +If there does not exist a $k$ such that $a_{k}=\alpha$, we must have $a_{k}<\alpha$ for all $k$. For each positive integer $m$, we can now pick an $\ell$ such that $m \mid \ell$ ! and $a_{\ell}=\alpha-x$ with $x$ arbitrarily small. It then follows from plugging in $m n=\ell$ ! that + +$$ +f(m)=\left\lfloor\frac{f(\ell!)}{\ell!/ m}\right\rfloor=\left\lfloor\frac{\ell!(\alpha-x)}{\ell!/ m}\right\rfloor=\lfloor\alpha m-m x\rfloor . +$$ + +If $\alpha m$ is an integer we can choose $\ell$ such that $m x<1$, and it follows that $f(m)=\lceil\alpha m\rceil-1$. If $\alpha m$ is not an integer we can choose $\ell$ such that $m x<\{\alpha m\}$, and it also follows that $f(m)=\lceil\alpha m\rceil-1$. We conclude that in this case $f$ is again of the desired form. + +## §3.3 TSTST 2022/9, proposed by Vincent Huang + +Available online at https://aops.com/community/p25517112. + +## Problem statement + +Let $k>1$ be a fixed positive integer. Prove that if $n$ is a sufficiently large positive integer, there exists a sequence of integers with the following properties: + +- Each element of the sequence is between 1 and $n$, inclusive. +- For any two different contiguous subsequences of the sequence with length between 2 and $k$ inclusive, the multisets of values in those two subsequences is not the same. +- The sequence has length at least $0.499 n^{2}$. + +For any positive integer $n$, define an $(n, k)$-good sequence to be a finite sequence of integers each between 1 and $n$ inclusive satisfying the second property in the problem statement. The problems asks to show that, for all sufficiently large integers $n$, there is an $(n, k)$-good sequence of length at least $0.499 n^{2}$. +Fix $k \geq 2$ and consider some prime power $n=p^{m}$ with $p>k+1$. Consider some $0k+1$ be a prime. Then for $n=p^{2}$ we can find a $(n, k)$-good sequence of length $\frac{p(p-1)\left(p^{2}+2\right)}{2}$. + +Proof of last claim. Let $g$ be the smallest primitive root modulo $n=p^{2}$, so that $a=$ $\frac{p(p-1)}{2}$. As long as we can show that $g<\frac{n}{k}-1$, we can apply the previous claim to get the desired bound. + +We will prove a stronger statement that $g United States of America - TST Selection Test + +Andrew Gu, Evan Chen, Gopal Goel
$65^{\text {th }}$ IMO 2024 United Kingdom and $13^{\text {th }}$ EGMO 2024 Georgia + +## Contents + +0 Problems ..... 2 +1 Solutions to Day 1 ..... 4 +1.1 TSTST 2023/1, proposed by Merlijn Staps ..... 4 +1.2 TSTST 2023/2, proposed by Raymond Feng, Luke Robitaille ..... 9 +1.3 TSTST 2023/3, proposed by Merlijn Staps ..... 13 +2 Solutions to Day 2 ..... 16 +2.1 TSTST 2023/4, proposed by Ankan Bhattacharya ..... 16 +2.2 TSTST 2023/5, proposed by David Altizio ..... 17 +2.3 TSTST 2023/6, proposed by Holden Mui ..... 20 +3 Solutions to Day 3 ..... 24 +3.1 TSTST 2023/7, proposed by Luke Robitaille ..... 24 +3.2 TSTST 2023/8, proposed by Ankan Bhattacharya ..... 25 +3.3 TSTST 2023/9, proposed by Holden Mui ..... 28 + +## §0 Problems + +1. Let $A B C$ be a triangle with centroid $G$. Points $R$ and $S$ are chosen on rays $G B$ and $G C$, respectively, such that + +$$ +\angle A B S=\angle A C R=180^{\circ}-\angle B G C . +$$ + +Prove that $\angle R A S+\angle B A C=\angle B G C$. +2. Let $n \geq m \geq 1$ be integers. Prove that + +$$ +\sum_{k=m}^{n}\left(\frac{1}{k^{2}}+\frac{1}{k^{3}}\right) \geq m \cdot\left(\sum_{k=m}^{n} \frac{1}{k^{2}}\right)^{2} +$$ + +3. Find all positive integers $n$ for which it is possible to color some cells of an infinite grid of unit squares red, such that each rectangle consisting of exactly $n$ cells (and whose edges lie along the lines of the grid) contains an odd number of red cells. +4. Let $n \geq 3$ be an integer and let $K_{n}$ be the complete graph on $n$ vertices. Each edge of $K_{n}$ is colored either red, green, or blue. Let $A$ denote the number of triangles in $K_{n}$ with all edges of the same color, and let $B$ denote the number of triangles in $K_{n}$ with all edges of different colors. Prove that + +$$ +B \leq 2 A+\frac{n(n-1)}{3} . +$$ + +5. Suppose $a, b$, and $c$ are three complex numbers with product 1 . Assume that none of $a, b$, and $c$ are real or have absolute value 1 . Define + +$$ +p=(a+b+c)+\left(\frac{1}{a}+\frac{1}{b}+\frac{1}{c}\right) \quad \text { and } \quad q=\frac{a}{b}+\frac{b}{c}+\frac{c}{a} . +$$ + +Given that both $p$ and $q$ are real numbers, find all possible values of the ordered pair $(p, q)$. +6. Let $A B C$ be a scalene triangle and let $P$ and $Q$ be two distinct points in its interior. Suppose that the angle bisectors of $\angle P A Q, \angle P B Q$, and $\angle P C Q$ are the altitudes of triangle $A B C$. Prove that the midpoint of $\overline{P Q}$ lies on the Euler line of $A B C$. +7. The Bank of Pittsburgh issues coins that have a heads side and a tails side. Vera has a row of 2023 such coins alternately tails-up and heads-up, with the leftmost coin tails-up. +In a move, Vera may flip over one of the coins in the row, subject to the following rules: + +- On the first move, Vera may flip over any of the 2023 coins. +- On all subsequent moves, Vera may only flip over a coin adjacent to the coin she flipped on the previous move. (We do not consider a coin to be adjacent to itself.) + +Determine the smallest possible number of moves Vera can make to reach a state in which every coin is heads-up. +8. Let $A B C$ be an equilateral triangle with side length 1. Points $A_{1}$ and $A_{2}$ are chosen on side $B C$, points $B_{1}$ and $B_{2}$ are chosen on side $C A$, and points $C_{1}$ and $C_{2}$ are chosen on side $A B$ such that $B A_{1}\frac{\left(\sum_{k=m}^{n} \frac{1}{k^{2}}\right)^{2}}{\frac{1}{m}} +\end{aligned} +$$ + +as desired. +Remark (Bound on error). Let $A=\sum_{k=m}^{n} k^{-2}$ and $B=\sum_{k=m}^{n} k^{-3}$. The inequality above becomes tighter for large $m$ and $n \gg m$. If we use Lagrange's identity in place of Cauchy-Schwarz, we get + +$$ +A+B-m A^{2}=m \cdot \sum_{m \leq aA+B +$$ + +Remark (Construction commentary, from author). My motivation was to write an inequality where Titu could be applied creatively to yield a telescoping sum. This can be difficult because most of the time, such a reverse-engineered inequality will be so loose it's trivial +anyways. My first attempt was the not-so-amazing inequality + +$$ +\frac{n^{2}+3 n}{2}=\sum_{1}^{n} i+1=\sum_{1}^{n} \frac{\frac{1}{i}}{\frac{1}{i(i+1)}}>\left(\sum_{1}^{n} \frac{1}{\sqrt{i}}\right)^{2} +$$ + +which is really not surprising given that $\sum \frac{1}{\sqrt{i}} \ll \frac{n}{\sqrt{2}}$. The key here is that we need "near-equality" as dictated by the Cauchy-Schwarz equality case, i.e. the square root of the numerators should be approximately proportional to the denominators. + +This motivates using $\frac{1}{i^{4}}$ as the numerator, which works like a charm. After working out the resulting statement, the LHS and RHS even share a sum, which adds to the simplicity of the problem. + +The final touch was to unrestrict the starting value of the sum, since this allows the strength of the estimate $\frac{1}{i^{2}} \approx \frac{1}{i(i+1)}$ to be fully exploited. + +【 Second approach by inducting down, Luke Robitaille and Carl Schildkraut. Fix $n$; we'll induct downwards on $m$. For the base case of $n=m$ the result is easy, since the left side is $\frac{m+1}{m^{3}}$ and the right side is $\frac{m}{m^{4}}=\frac{1}{m^{3}}$. + +For the inductive step, suppose we have shown the result for $m+1$. Let + +$$ +A=\sum_{k=m+1}^{n} \frac{1}{k^{2}} \quad \text { and } \quad B=\sum_{k=m+1}^{n} \frac{1}{k^{3}} +$$ + +We know $A+B \geq(m+1) A^{2}$, and we want to show + +$$ +\left(A+\frac{1}{m^{2}}\right)+\left(B+\frac{1}{m^{3}}\right) \geq m\left(A+\frac{1}{m^{2}}\right)^{2} +$$ + +Indeed, + +$$ +\begin{aligned} +\left(A+\frac{1}{m^{2}}\right)+\left(B+\frac{1}{m^{3}}\right)-m\left(A+\frac{1}{m^{2}}\right)^{2} & =A+B+\frac{m+1}{m^{3}}-m A^{2}-\frac{2 A}{m}-\frac{1}{m^{3}} \\ +& =\left(A+B-(m+1) A^{2}\right)+\left(A-\frac{1}{m}\right)^{2} \geq 0 +\end{aligned} +$$ + +and we are done. + +I Third approach by reducing $n \rightarrow \infty$, Michael Ren and Carl Schildkraut. First, we give: + +Claim (Reduction to $n \rightarrow \infty$ ) - If the problem is true when $n \rightarrow \infty$, it is true for all $n$. + +Proof. Let $A=\sum_{k=m}^{n} k^{-2}$ and $B=\sum_{k=m}^{n} k^{-3}$. Consider the region of the $x y$-plane defined by $y>m x^{2}-x$. We are interested in whether $(A, B)$ lies in this region. + +However, the region is bounded by a convex curve, and the sequence of points $(0,0)$, $\left(\frac{1}{m^{2}}, \frac{1}{m^{3}}\right),\left(\frac{1}{m^{2}}+\frac{1}{(m+1)^{2}}, \frac{1}{m^{3}}+\frac{1}{(m+1)^{3}}\right), \ldots$ has successively decreasing slopes between consecutive points. Thus it suffices to check that the inequality is true when $n \rightarrow \infty$. + +Set $n=\infty$ henceforth. Let + +$$ +A=\sum_{k=m}^{\infty} \frac{1}{k^{2}} \text { and } B=\sum_{k=m}^{\infty} \frac{1}{k^{3}} +$$ + +we want to show $B \geq m A^{2}-A$, which rearranges to + +$$ +1+4 m B \geq(2 m A-1)^{2} +$$ + +Write + +$$ +C=\sum_{k=m}^{\infty} \frac{1}{k^{2}(2 k-1)(2 k+1)} \text { and } D=\sum_{k=m}^{\infty} \frac{8 k^{2}-1}{k^{3}(2 k-1)^{2}(2 k+1)^{2}} +$$ + +Then + +$$ +\frac{2}{2 k-1}-\frac{2}{2 k+1}=\frac{1}{k^{2}}+\frac{1}{k^{2}(2 k-1)(2 k+1)} +$$ + +and + +$$ +\frac{2}{(2 k-1)^{2}}-\frac{2}{(2 k+1)^{2}}=\frac{1}{k^{3}}+\frac{8 k^{2}-1}{k^{3}(2 k-1)^{2}(2 k+1)^{2}}, +$$ + +so that + +$$ +A=\frac{2}{2 m-1}-C \text { and } B=\frac{2}{(2 m-1)^{2}}-D +$$ + +Our inequality we wish to show becomes + +$$ +\frac{2 m+1}{2 m-1} C \geq D+m C^{2} +$$ + +We in fact show two claims: +Claim - We have + +$$ +\frac{2 m+1 / 2}{2 m-1} C \geq D +$$ + +Proof. We compare termwise; we need + +$$ +\frac{2 m+1 / 2}{2 m-1} \cdot \frac{1}{k^{2}(2 k-1)(2 k+1)} \geq \frac{8 k^{2}-1}{k^{3}(2 k-1)^{2}(2 k+1)^{2}} +$$ + +for $k \geq m$. It suffices to show + +$$ +\frac{2 k+1 / 2}{2 k-1} \cdot \frac{1}{k^{2}(2 k-1)(2 k+1)} \geq \frac{8 k^{2}-1}{k^{3}(2 k-1)^{2}(2 k+1)^{2}} +$$ + +which is equivalent to $k(2 k+1 / 2)(2 k+1) \geq 8 k^{2}-1$. This holds for all $k \geq 1$. + +## Claim - We have + +$$ +\frac{1 / 2}{2 m-1} C \geq m C^{2} +$$ + +Proof. We need $C \leq 1 /(2 m(2 m-1))$; indeed, + +$$ +\frac{1}{2 m(2 m-1)}=\sum_{k=m}^{\infty}\left(\frac{1}{2 k(2 k-1)}-\frac{1}{2(k+1)(2 k+1)}\right)=\sum_{k=m}^{\infty} \frac{4 k+1}{2 k(2 k-1)(k+1)(2 k+1)} +$$ + +comparing term-wise with the definition of $C$ and using the inequality $k(4 k+1) \geq 2(k+1)$ for $k \geq 1$ gives the desired result. + +Combining the two claims finishes the solution. + +『 Fourth approach by bashing, Carl Schildkraut. With a bit more work, the third approach can be adapted to avoid the $n \rightarrow \infty$ reduction. Similarly to before, define + +$$ +A=\sum_{k=m}^{n} \frac{1}{k^{2}} \text { and } B=\sum_{k=m}^{n} \frac{1}{k^{3}} +$$ + +we want to show $1+4 m B \geq(2 m A-1)^{2}$. Writing + +$$ +C=\sum_{k=m}^{n} \frac{1}{k^{2}(2 k-1)(2 k+1)} \text { and } D=\sum_{k=m}^{n} \frac{8 k^{2}-1}{k^{3}(2 k-1)^{2}(2 k+1)^{2}} +$$ + +We compute + +$$ +A=\frac{2}{2 m-1}-\frac{2}{2 n+1}-C \text { and } B=\frac{2}{(2 m-1)^{2}}-\frac{2}{(2 n+1)^{2}}-D . +$$ + +Then, the inequality we wish to show reduces (as in the previous solution) to + +$$ +\frac{2 m+1}{2 m-1} C+\frac{2(2 m+1)}{(2 m-1)(2 n+1)} \geq D+m C^{2}+\frac{2(2 m+1)}{(2 n+1)^{2}}+\frac{4 m}{2 n+1} C +$$ + +We deal first with the terms not containing the variable $n$, i.e. we show that + +$$ +\frac{2 m+1}{2 m-1} C \geq D+m C^{2} +$$ + +For this part, the two claims from the previous solution go through exactly as written above, and we have $C \leq 1 /(2 m(2 m-1))$. We now need to show + +$$ +\frac{2(2 m+1)}{(2 m-1)(2 n+1)} \geq \frac{2(2 m+1)}{(2 n+1)^{2}}+\frac{4 m}{2 n+1} C +$$ + +(this is just the inequality between the remaining terms); our bound on $C$ reduces this to proving + +$$ +\frac{4(2 m+1)(n-m+1)}{(2 m-1)(2 n+1)^{2}} \geq \frac{2}{(2 m-1)(2 n+1)} +$$ + +Expanding and writing in terms of $n$, this is equivalent to + +$$ +n \geq \frac{1+2(m-1)(2 m+1)}{4 m}=m-\frac{2 m+1}{4 m} +$$ + +which holds for all $n \geq m$. + +## §1.3 TSTST 2023/3, proposed by Merlijn Staps + +Available online at https://aops.com/community/p28015682. + +## Problem statement + +Find all positive integers $n$ for which it is possible to color some cells of an infinite grid of unit squares red, such that each rectangle consisting of exactly $n$ cells (and whose edges lie along the lines of the grid) contains an odd number of red cells. + +We claim that this is possible for all positive integers $n$. Call a positive integer for which such a coloring is possible good. To show that all positive integers $n$ are good we prove the following: +(i) If $n$ is good and $p$ is an odd prime, then $p n$ is good; +(ii) For every $k \geq 0$, the number $n=2^{k}$ is good. + +Together, (i) and (ii) imply that all positive integers are good. +【 Proof of (i). We simply observe that if every rectangle consisting of $n$ cells contains an odd number of red cells, then so must every rectangle consisting of $p n$ cells. Indeed, because $p$ is prime, a rectangle consisting of $p n$ cells must have a dimension (length or width) divisible by $p$ and can thus be subdivided into $p$ rectangles consisting of $n$ cells. + +Thus every coloring that works for $n$ automatically also works for $p n$. +【 Proof of (ii). Observe that rectangles with $n=2^{k}$ cells have $k+1$ possible shapes: $2^{m} \times 2^{k-m}$ for $0 \leq m \leq k$. + +Claim - For each of these $k+1$ shapes, there exists a coloring with two properties: + +- Every rectangle with $n$ cells and shape $2^{m} \times 2^{k-m}$ contains an odd number of red cells. +- Every rectangle with $n$ cells and a different shape contains an even number of red cells. + +Proof. This can be achieved as follows: assuming the cells are labeled with $(x, y) \in \mathbb{Z}^{2}$, color a cell red if $x \equiv 0\left(\bmod 2^{m}\right)$ and $y \equiv 0\left(\bmod 2^{k-m}\right)$. For example, a $4 \times 2$ rectangle gets the following coloring: +![](https://cdn.mathpix.com/cropped/2024_11_19_1cf0843c143ab4f13b3fg-13.jpg?height=395&width=401&top_left_y=2084&top_left_x=833) + +A $2^{m} \times 2^{k-m}$ rectangle contains every possible pair $\left(x \bmod 2^{m}, y \bmod 2^{k-m}\right)$ exactly once, so such a rectangle will contain one red cell (an odd number). + +On the other hand, consider a $2^{\ell} \times 2^{k-\ell}$ rectangle with $\ell>m$. The set of cells this covers is $(x, y)$ where $x$ covers a range of size $2^{\ell}$ and $y$ covers a range of size $2^{k-\ell}$. The number of red cells is the count of $x$ with $x \equiv 0 \bmod 2^{m}$ multiplied by the count of $y$ with $y \equiv 0 \bmod 2^{k-m}$. The former number is exactly $2^{\ell-k}$ because $2^{k}$ divides $2^{\ell}$ (while the latter is 0 or 1) so the number of red cells is even. The $\ell2 \\ +2\left(g(x)+g\left(x+p^{d}\right)+\cdots+g\left(x+(b-1) p^{d}\right)\right) & p=2\end{cases} \\ +\equiv & 0 \quad(\bmod p) +\end{aligned} +$$ + +as desired. + +## Corollary + +Let $g: \mathbb{Z} / a \mathbb{Z} \rightarrow \mathbb{Z} / p^{e} \mathbb{Z}$ be any function, and let $h=\Delta^{e p^{d}} g$. Then + +$$ +h(x)+h\left(x+p^{d}\right)+\cdots+h\left(x+(b-1) p^{d}\right)=0 +$$ + +for all $x$. + +Proof. Starting with the lemma, define + +$$ +h_{1}(x)=\frac{h(x)+h\left(x+p^{d}\right)+\cdots+h\left(x+(b-1) p^{d}\right)}{p} . +$$ + +Applying the lemma to $h_{1}$ shows the corollary for $e=2$, since $h_{1}(x)$ is divisible by $p$, hence the numerator is divisible by $p^{2}$. Continue in this manner to get the result for general $e>2$. + +This immediately settles this direction, since $f$ is in the image of $\Delta^{e p^{d}}$. + +Proof the equation implies essential Let $\mathcal{S}$ be the set of all functions satisfying 2 ; then it's easy to see that $\Delta$ is a function on $\mathcal{S}$. To show that all functions in $\mathcal{S}$ are essential, it's equivalent to show that $\Delta$ is a permutation on $\mathcal{S}$. + +We will show that $\Delta$ is injective on $\mathcal{S}$. Suppose otherwise, and consider two functions $f$, $g$ in $\mathcal{S}$ with $\Delta f=\Delta g$. Then, we obtain that $f$ and $g$ differ by a constant; say $g=f+\lambda$. However, then + +$$ +\begin{aligned} +& g(0)+g\left(p^{e}\right)+\cdots+g\left((b-1) p^{e}\right) \\ += & (f(0)+\lambda)+\left(f\left(p^{e}\right)+\lambda\right)+\cdots+\left(f\left((b-1) p^{e}\right)+\lambda\right) \\ += & b \lambda +\end{aligned} +$$ + +This should also be zero. Since $p \nmid b$, we obtain $\lambda=0$, as desired. + +Counting Finally, we can count the essential functions: all but the last $p^{d}$ entries can be chosen arbitrarily, and then each remaining entry has exactly one possible choice. This leads to a count of + +$$ +\left(p^{e}\right)^{a-p^{d}}=p^{e\left(a-p^{\nu_{p}(a)}\right)}, +$$ + +as promised. + +II Second solution by Daniel Zhu. There are two parts to the proof: solving the $e=1$ case, and using the $e=1$ result to solve the general problem by induction on $e$. These parts are independent of each other. + +The case $e=1 \quad$ Represent functions $f$ as elements + +$$ +\alpha_{f}:=\sum_{k \in \mathbb{Z} / a \mathbb{Z}} f(-k) x^{k} \in \mathbb{F}_{p}[x] /\left(x^{a}-1\right) +$$ + +Then, since $\alpha_{\Delta f}=(x-1) \alpha_{f}$, we wish to find the number of $\alpha \in \mathbb{F}_{p}[x] /\left(x^{a}-1\right)$ such that $(x-1)^{m} \alpha=\alpha$ for some $m$. + +Now, make the substitution $y=x-1$ and let $P(y)=(y+1)^{a}-1$; we want to find $\alpha \in \mathbb{F}_{p}[y] /(P(y))$ such that $y^{m} \alpha=\alpha$ for some $m$. + +If we write $P(y)=y^{d} Q(y)$ with $Q(0) \neq 0$, then by the Chinese Remainder Theorem we have the ring isomorphism + +$$ +\mathbb{F}_{p}[y] /(P(y)) \cong \mathbb{F}_{p}[y] /\left(y^{d}\right) \times \mathbb{F}_{p}[y] /(Q(y)) +$$ + +Note that $y$ is nilpotent in the first factor, while it is a unit in the second factor. So the $\alpha$ that work are exactly those that are zero in the first factor; thus there are $p^{a-d}$ such $\alpha$. We can calculate $d=p^{v_{p}(a)}$ (via, say, Lucas's Theorem), so we are done. + +The general problem The general idea is as follows: call a $f: \mathbb{Z} / a \mathbb{Z} \rightarrow \mathbb{Z} / p^{e} \mathbb{Z} e$-good if $\Delta^{m} f=f$ for some $m$. Our result above allows us to count the 1-good functions. Then, if $e \geq 1$, every $(e+1)$-good function, when reduced $\bmod p^{e}$, yields an $e$-good function, so we count $(e+1)$-good functions by counting how many reduce to any given $e$-good function. + +Formally, we use induction on $e$, with the $e=1$ case being treated above. Suppose now we have solved the problem for a given $e \geq 1$, and we now wish to solve it for $e+1$. For any function $g: \mathbb{Z} / a \mathbb{Z} \rightarrow \mathbb{Z} / p^{e+1} \mathbb{Z}$, let $\bar{g}: \mathbb{Z} / a \mathbb{Z} \rightarrow \mathbb{Z} / p^{e} \mathbb{Z}$ be its reduction $\bmod p^{e}$. For a given $e$-good $f$, let $n(f)$ be the number of $(e+1)$-good $g$ with $\bar{g}=f$. The following two claims now finish the problem: + +Claim - If $f$ is $e$-good, then $n(f)>0$. + +Proof. Suppose $m$ is such that $\Delta^{m} f=f$. Pick any $g$ with $\bar{g}=f$, and consider the sequence of functions + +$$ +g, \Delta^{m} g, \Delta^{2 m} g, \ldots +$$ + +Since there are finitely many functions $\mathbb{Z} / a \mathbb{Z} \rightarrow \mathbb{Z} / p^{e+1} \mathbb{Z}$, there must exist $a0$, then $n(f)$ is exactly the number of 1-good functions, i.e. $p^{a-p^{v_{p}(a)}}$. + +Proof. Let $g$ be any $(e+1)$-good function with $\bar{g}=f$. We claim that the $(e+1)$-good $g_{1}$ with $\bar{g}_{1}=f$ are exactly the functions of the form $g+p^{e} h$ for any 1-good $h$. Since these functions are clearly distinct, this characterization will prove the claim. + +To show that this condition is sufficient, note that $\overline{g+p^{e} h}=\bar{g}=f$. Moreover, if $\Delta^{m} g=g$ and $\Delta^{m^{\prime}} h=h$, then + +$$ +\Delta^{m m^{\prime}}\left(g+p^{e} h\right)=\Delta^{m m^{\prime}} g+p^{e} \Delta^{m m^{\prime}} h=g+p^{e} h . +$$ + +To show that this condition is necessary, let $g_{1}$ be any $(e+1)$-good function such that $\bar{g}_{1}=f$. Then $g_{1}-g$ is also $(e+1)$-good, since if $\Delta^{m} g=g, \Delta^{m^{\prime}} g_{1}=g_{1}$, we have + +$$ +\Delta^{m m^{\prime}}\left(g_{1}-g\right)=\Delta^{m m^{\prime}} g_{1}-\Delta^{m m^{\prime}} g=g_{1}-g . +$$ + +On the other hand, we also know that $g_{1}-g$ is divisible by $p^{e}$. This means that it must be $p^{e} h$ for some function $f: \mathbb{Z} / a \mathbb{Z} \rightarrow \mathbb{Z} / p \mathbb{Z}$, and it is not hard to show that $g_{1}-g$ being $(e+1)$-good means that $h$ is 1 -good. + diff --git a/USA_TSTST/raw/en-sols-TSTST-2011.pdf b/USA_TSTST/raw/en-sols-TSTST-2011.pdf new file mode 100644 index 0000000000000000000000000000000000000000..05d5fd3fc39d30ac61570851109b8a5793c0475f --- /dev/null +++ b/USA_TSTST/raw/en-sols-TSTST-2011.pdf @@ -0,0 +1,3 @@ +version https://git-lfs.github.com/spec/v1 +oid sha256:74068c73f2c89d2115f7f78fe9bab75832a884e68fb562b9218c0dbd91f113cf +size 234980 diff --git a/USA_TSTST/raw/en-sols-TSTST-2012.pdf b/USA_TSTST/raw/en-sols-TSTST-2012.pdf new file mode 100644 index 0000000000000000000000000000000000000000..4d6a0c86a905b7f7a5f043065e98284e55597f93 --- /dev/null +++ b/USA_TSTST/raw/en-sols-TSTST-2012.pdf @@ -0,0 +1,3 @@ +version https://git-lfs.github.com/spec/v1 +oid sha256:2c8f8fa61c649825cbc2dc49a8bcca25508d1790fc3f560c53423e77059cea8d +size 381753 diff --git a/USA_TSTST/raw/en-sols-TSTST-2013.pdf b/USA_TSTST/raw/en-sols-TSTST-2013.pdf new file mode 100644 index 0000000000000000000000000000000000000000..074c80b26beee71331fa08015624f41d2fc7980c --- /dev/null +++ b/USA_TSTST/raw/en-sols-TSTST-2013.pdf @@ -0,0 +1,3 @@ +version https://git-lfs.github.com/spec/v1 +oid sha256:4533aa1a087a031b9ef15982e3046300f1472f4440f2ac8f264e742a375099bd +size 284180 diff --git a/USA_TSTST/raw/en-sols-TSTST-2014.pdf b/USA_TSTST/raw/en-sols-TSTST-2014.pdf new file mode 100644 index 0000000000000000000000000000000000000000..64458f31d58a69c07d847ef66b69a02f0573bf11 --- /dev/null +++ b/USA_TSTST/raw/en-sols-TSTST-2014.pdf @@ -0,0 +1,3 @@ +version https://git-lfs.github.com/spec/v1 +oid sha256:fb2085395dee1c991a510e42d51dba95d5623fa955f2cfc8f1cf6eb82a50b9ad +size 187303 diff --git a/USA_TSTST/raw/en-sols-TSTST-2015.pdf b/USA_TSTST/raw/en-sols-TSTST-2015.pdf new file mode 100644 index 0000000000000000000000000000000000000000..389be1de0a9970aefe8cc291d96f6815d0b06d02 --- /dev/null +++ b/USA_TSTST/raw/en-sols-TSTST-2015.pdf @@ -0,0 +1,3 @@ +version https://git-lfs.github.com/spec/v1 +oid sha256:566c599c4a935e41e1744ec4409725f2d2457dcc6bc642f0fb74fb33bc35be1f +size 202252 diff --git a/USA_TSTST/raw/en-sols-TSTST-2016.pdf b/USA_TSTST/raw/en-sols-TSTST-2016.pdf new file mode 100644 index 0000000000000000000000000000000000000000..a2e3e3e1ca9a5c900c3870c8dcdecc7c5cefa644 --- /dev/null +++ b/USA_TSTST/raw/en-sols-TSTST-2016.pdf @@ -0,0 +1,3 @@ +version https://git-lfs.github.com/spec/v1 +oid sha256:d240763a70a050897cf0f29ae97c2dafb0369c5ef8366cd25d5a6455d5463889 +size 311220 diff --git a/USA_TSTST/raw/en-sols-TSTST-2017.pdf b/USA_TSTST/raw/en-sols-TSTST-2017.pdf new file mode 100644 index 0000000000000000000000000000000000000000..57c425537106380890387e4b97fd23c98204d87b --- /dev/null +++ b/USA_TSTST/raw/en-sols-TSTST-2017.pdf @@ -0,0 +1,3 @@ +version https://git-lfs.github.com/spec/v1 +oid sha256:aca15284395b987bc9542c45a7e655e557ba08099effb6f784050b69cd4f310a +size 305543 diff --git a/USA_TSTST/raw/en-sols-TSTST-2018.pdf b/USA_TSTST/raw/en-sols-TSTST-2018.pdf new file mode 100644 index 0000000000000000000000000000000000000000..fcf83ff4d150114fa6685a6190955721c68eb9c2 --- /dev/null +++ b/USA_TSTST/raw/en-sols-TSTST-2018.pdf @@ -0,0 +1,3 @@ +version https://git-lfs.github.com/spec/v1 +oid sha256:ef5334d493612ebe62f118b2d0c249ff84c662e9d02f6ecad109c42a495cf583 +size 340192 diff --git a/USA_TSTST/raw/en-sols-TSTST-2019.pdf b/USA_TSTST/raw/en-sols-TSTST-2019.pdf new file mode 100644 index 0000000000000000000000000000000000000000..105ed76ea9ab3ab78b01d0b4cd214572f8de9a80 --- /dev/null +++ b/USA_TSTST/raw/en-sols-TSTST-2019.pdf @@ -0,0 +1,3 @@ +version https://git-lfs.github.com/spec/v1 +oid sha256:61150c826fc19204d7b4b1372a60bb1324a92de887a454d9d1d16b4c90767e3d +size 419600 diff --git a/USA_TSTST/raw/en-sols-TSTST-2020.pdf b/USA_TSTST/raw/en-sols-TSTST-2020.pdf new file mode 100644 index 0000000000000000000000000000000000000000..cfda5ce94d08c255fb49c918013cb3cd4625a885 --- /dev/null +++ b/USA_TSTST/raw/en-sols-TSTST-2020.pdf @@ -0,0 +1,3 @@ +version https://git-lfs.github.com/spec/v1 +oid sha256:3bb5569f690fd1cfe31777bd239bcc2e6a96181abe82a1726d1d2fe8cf51d207 +size 348489 diff --git a/USA_TSTST/raw/en-sols-TSTST-2021.pdf b/USA_TSTST/raw/en-sols-TSTST-2021.pdf new file mode 100644 index 0000000000000000000000000000000000000000..9c8e4888d93282e9ec670297e3e0aff2bd8569d0 --- /dev/null +++ b/USA_TSTST/raw/en-sols-TSTST-2021.pdf @@ -0,0 +1,3 @@ +version https://git-lfs.github.com/spec/v1 +oid sha256:f45991ce74fdf19d8bfe81befadf589b41e61092d9e29b5deeff9a903ffe6e33 +size 664142 diff --git a/USA_TSTST/raw/en-sols-TSTST-2022.pdf b/USA_TSTST/raw/en-sols-TSTST-2022.pdf new file mode 100644 index 0000000000000000000000000000000000000000..41f2dfd043458dd8804184b185e98f096819444d --- /dev/null +++ b/USA_TSTST/raw/en-sols-TSTST-2022.pdf @@ -0,0 +1,3 @@ +version https://git-lfs.github.com/spec/v1 +oid sha256:f61d38aa86a76184c5f82c78c5c5b4a49790ed708dc5911b63b6e8d459d44585 +size 361550 diff --git a/USA_TSTST/raw/en-sols-TSTST-2023.pdf b/USA_TSTST/raw/en-sols-TSTST-2023.pdf new file mode 100644 index 0000000000000000000000000000000000000000..7e64709171212674fed014c969b23e2516c186db --- /dev/null +++ b/USA_TSTST/raw/en-sols-TSTST-2023.pdf @@ -0,0 +1,3 @@ +version https://git-lfs.github.com/spec/v1 +oid sha256:3ae9f91558d2c626d4fa1323a5af7b76cac73b6ad990472cbb4b569a1bc09462 +size 471452 diff --git a/USA_TSTST/segment_script/segment_usamo.py b/USA_TSTST/segment_script/segment_usamo.py new file mode 100644 index 0000000000000000000000000000000000000000..0fb17da47cecbb1c7443d21332018f569c13d2ff --- /dev/null +++ b/USA_TSTST/segment_script/segment_usamo.py @@ -0,0 +1,306 @@ +# ----------------------------------------------------------------------------- +# Author: Marina +# Date: 2024-11-15 +# ----------------------------------------------------------------------------- +''' Script to segment md files in en-usamo, en-tstst, en-tst, en-jmo folder using regex. +To run: +`python segment_script/segment_usamo.py` +''' + +import warnings +warnings.filterwarnings("ignore", category=DeprecationWarning) + +import os +import re +import pandas as pd +from rapidfuzz import fuzz + + +# "## §0 Problems" -> match +section_re = re.compile(r"^#{1,2}\s(?:Contents|Problem|§[\d.]+.*)") +# section_re = re.compile(r"^#{1,2}\s(?:§[\d.]+.*)") + +# "## §1.3 USAMO 2024/3, proposed by Krit Boonsiriseth" ->3 +# "## §2.3 TSTST 2011/6" ->6 +# "## §1.1 TSTST 2021/1, proposed by Holden Mui" -> 1 +# "## §1.2 USA TST 2020/2, proposed by Merlijn Staps" -> 2 +# "# §1.3 USA TST 2019/3, proposed by Nikolai Beluhov" -> 3 +solution_label_re = re.compile( + r"^#{1,2}\s§[\d.]+\s[A-Za-z0-9 ]+\s\d{4}/(\d+)(?:,\s.*)?$" +) + +# "1. Prove that the average of the numbers $n" -> match +# "2 . For any nonempty set $S$ of real numbers,"" -> match +problem_re = re.compile(r"^(\d+)\s?\.\s(.*(?:\n\s+.*)*)") + +# "## Problem statement extra text" -> match +# "Problem statement" -> match +# "##Problem statement (missing space)" -> match +solution_re = re.compile(r"^#{0,2}\s?Problem statement\b.*$") + +# not actually used, only for debugging +pattern_debug = re.compile( + r"^[【『\\]*.*?\b(First|Second|Third|Fourth|Fifth|Sixth|Seventh|Eighth|Ninth|Tenth|Complex|Inversion|Synthetic|One|Another|Solution)\b.*\b(solution|approach|proof)\b.*", + re.IGNORECASE +) + +# "Solution 1" -> match +# "【 Solution 1" -> match +solution_split_re1 = re.compile(r"\bSolution\s[1-9]\b") + +# "First solution" -> match +# "【 Third approach" -> match +# "【 Second approach" -> match +solution_split_re2 = re.compile(r"\b(First|Second|Third|Fourth|Fifth|Sixth|Seventh|Eighth|Ninth|Synthetic)\b\s+(solution|approach|proof)\b") + +# catch special cases (ugly but it works). To generate them run script with `DEBUG = True`` +# and identify in console which special_cases should be catched as first line of a solution +DEBUG = False +special_cases = [ +"【 First short solution, by Jeffrey Kwan. Let $p_{0", +"II Second longer solution using an invariant. Visu", +"【 Complex solution (Evan Chen). Toss on the comple", +"Second (longer) solution. If one does not notice t", +"『 Second calculation approach (along the lines of ", +"T Outline of second approach (by convexity, due t", +"I Inversion solution submitted by Ankan Bhattacha", +"【 Complex numbers approach with Apollonian circles", +" A second solution. Both lemmas above admit varia", +"【 A third remixed solution. We use Lemma I and Lem", +"【I A fourth remixed solution. We also can combine ", +"I First grid-based solution. The following solutio", +"Another short solution. Let $Z$ be on line $B D E$", +"【 Most common synthetic approach. The solution hin", +"\\ First \"local\" solution by swapping two points. L", +"Second general solution by angle chasing. By Rei", +"Third general solution by Pascal. Extend rays $A", +"【 Second length solution by tangent lengths. By $t", +"【 Angle chasing solution. Note that $(B D A)$ and", +"【 Harmonic solution (mine). Let $T$ be the point o", +"【 Pascal solution (Zuming Feng). Extend ray $F D$", +"『 A spiral similarity approach (Hans $\mathbf{Y u}", +"ब The author's original solution. Complete isoscel", +"l Evan's permutation-based solution. Retain the n", +"I Original proposer's solution. To this end, let's", +"【 Cartesian coordinates approach with power of a p", +"【 Cartesian coordinates approach without power of", +"I III-advised barycentric approach (outline). Use", +"【 Approach using difference of squares (from autho", +"【 Divisibility approach (Aharshi Roy). Since $p q-", +"Solution with Danielle Wang: the answer is that $|", +"【 Homothety solution (Alex Whatley). Let $G, N, O$", +"【 Power of a point solution (Zuming Feng, official", +"【 Solution by Luke Robitaille. Let $Q$ be the seco", +"ๆ Solution with coaxial circles (Pitchayut Saengru", +"【 Solution to generalization (Nikolai Beluhov). We", +"【 Approach by deleting teams (Gopal Goel). Initial", +"【 Approach by adding colors. For a constructive al", +"【 Solution using spiral similarity. We will ignore", +"『 Barycentric solution (by Carl, Krit, Milan). We", +"I A Menelaus-based approach (Kevin Ren). Let $P$ b", +"【 Barycentric solution. First, we find the coordin", +"【 Angle chasing solution (Mason Fang). Obviously $", +"【 Inversive solution (Kelin Zhu). Invert about $A$", +"l The one-liner. ", +" The external power solution. We distinguish betw", +"Cauchy-Schwarz approach. Apply Titu lemma to get", +"đ Cauchy-Schwarz approach. The main magical claim ", +"『 Alternate solution (by proposer). Let $L$ be dia" +] + + +def add_content(current_dict): + if not current_dict["lines"] or not current_dict["label"] : + return + text_str = " ".join(current_dict["lines"]).strip() + entry = {"label": current_dict["label"]} + if current_dict["class"] == "problem": + entry["problem"] = text_str + current_dict["problems"].append(entry) + elif current_dict["class"] == "solution": + entry["solution"] = text_str + entry["solution_lines"] = current_dict["lines"] + current_dict["solutions"].append(entry) + + +def parse(file): + with open(file, 'r') as file: + content = file.read() + current = { + "label": None, + "class": None, + "lines": [], + "problems": [], + "solutions": [] + } + for line in content.splitlines(): + if match := section_re.match(line): # match a section + add_content(current) + if "problems" in line.lower(): # match problem section + current["class"] = "problem" + elif sub_match:= solution_label_re.match(line): # match solutions section, extract label for join + current["class"] = "other" + current["label"] = sub_match.group(1) + elif match := solution_re.match(line): # match solutions subsection + current["class"] = "solution" + else: + current["class"] = "other" + current["lines"] = [] + elif match := problem_re.match(line): # match a problem + if current["class"] == "solution": # handle wrong problem match + current["lines"].append(line) + else: + add_content(current) + label, text = match.groups() + current["label"] = label + current["lines"] = [text] + else: + if current["class"]=="solution" or current["class"]=="problem": + current["lines"].append(line) + add_content(current) + problems_df = pd.DataFrame(current["problems"]) + solutions_df = pd.DataFrame(current["solutions"]) + return problems_df, solutions_df + + +def parse_solution(lines): + """parses lines of a solution, finds multiple solutions and splits them""" + solutions = [] + current = [] + for line in lines: + if match := solution_split_re1.search(line): # match a solution (case1) + solutions.append(" ".join(current).strip()) + current = [line] + elif match := solution_split_re2.search(line): # match a solution (case1) + solutions.append(" ".join(current).strip()) + current = [line] + elif any(case.lower() in line.lower() for case in special_cases): # match a solution (handle special_case) + solutions.append(" ".join(current).strip()) + current = [line] + elif any(case.lower() in line[:50].lower() for case in ["solution", "approach", "proof"]): + if DEBUG: + if not any(case.lower() in line[:50].lower() for case in ["remark", "proof.", "proof", "approaches", "solutions"]): + print(line[:50]) + else: + current.append(line) + solutions.append(" ".join(current).strip()) + return solutions + +def find_mult_solutions(solutions_df): + """apply parse_solution to all df""" + solutions_df["solution"] = solutions_df["solution_lines"].apply(lambda v: parse_solution(v)) + solutions_df = solutions_df.drop(columns=["solution_lines"]) + solutions_df = solutions_df.explode('solution', ignore_index=True) + return solutions_df + + +def join(problems_df, solutions_df): + pairs_df = problems_df.merge(solutions_df, on=["label"], how="outer") + return pairs_df + + +def clean(pairs_df): + '''removes the problem statement from the solution in an approximate way''' + def find_closest_char(s, i, char): + left = s.rfind(char, 0, i) # Find the last '.' before index i + right = s.find(char, i) # Find the first '.' after or at index i + if left == -1 and right == -1: + return None # No '.' found + elif left == -1: # No '.' on the left + return right + elif right == -1: # No '.' on the right + return left + else: # Closest '.' on either side + return left if abs(i - left) <= abs(i - right) else right + def remove_approx_match(row, threshold=90): + problem = row["problem"] + solution = row["solution"] + similarity = fuzz.partial_ratio(problem, solution) + if similarity >= threshold: + i = find_closest_char(solution, len(problem), problem[-1]) + if i is not None: + solution = solution[i+1:] + return solution + pairs_df["solution"] = pairs_df.apply(remove_approx_match, axis=1) + return pairs_df + + +def process_mult_solutions(pairs_df): + '''in case of multiple solutions, prepend common text to all solutions''' + def prepend_to_solution(group): + if len(group) == 1: + return group + first_row = group.iloc[0] + comment = f"{first_row['solution']}" + group = group.iloc[1:].copy() + group["solution"] = group["solution"].apply(lambda x: f"{comment} {x}") + return group + pairs_df = pairs_df.groupby("label", group_keys=False).apply(prepend_to_solution).reset_index(drop=True) + return pairs_df + + +def add_metadata(pairs_df, yar, tier): + pairs_df['year'] = year + pairs_df['tier'] = tier # according to omnimath + return pairs_df[['year', 'label', 'problem', 'solution']] + + +def write_pairs(filename, pairs_df): + pairs_df.to_json(filename, orient="records", lines=True) + + +configs = [ + ('en-usamo', lambda year: f'en-USAMO-{year}-notes', range(1996, 2025), 1), + ('en-tstst', lambda year: f'en-sols-TSTST-{year}', range(2011, 2024), 0), + ('en-tst', lambda year: f'en-sols-TST-IMO-{year}', range(2014, 2024), 0), + # ('en-elmo', lambda year: f'en-ELMO-{year}-sols', range(2010, 2017), None), #TODO: needs another parser + # ('en-elmo', lambda year: f'en-ELMO-{year}-SL', range(2017, 2019), None), #TODO: needs another parser + ('en-jmo', lambda year: f'en-JMO-{year}-notes', range(2010, 2025), 3), + # ('en-usemo', lambda year: f'en-report-usemo-{year}', range(2019, 2023), None), #TODO: needs another parser +] + + +total_problem_count = 0 +total_solution_count = 0 +for base, basename_, years, tier in configs: + print(base) + problem_count = 0 + solution_count = 0 + seg_base = f"{base}-seg" + os.makedirs(seg_base, exist_ok=True) + for year in years: + basename = basename_(year) + file_path = f"{base}/{basename}.md" + if os.path.exists(file_path): + # print(basename) + problems, solutions = parse(file_path) + solutions = find_mult_solutions(solutions) + pairs_df = join(problems, solutions) + pairs_df = clean(pairs_df) + pairs_df = process_mult_solutions(pairs_df) + pairs_df = add_metadata(pairs_df, year, tier) + problem_count += len(problems) + solution_count += len(pairs_df) + # print(pairs_df) + write_pairs(f"{seg_base}/{basename}.jsonl", pairs_df) + print(f"problem count: {problem_count}") + print(f"solution count: {solution_count}") + total_problem_count += problem_count + total_solution_count += solution_count +print(f"total problem count: {total_problem_count}") +print(f"total solution count: {total_solution_count}") + +# en-usamo +# problem count: 174 +# solution count: 203 +# en-tstst +# problem count: 105 +# solution count: 168 +# en-tst +# problem count: 51 +# solution count: 82 +# en-jmo +# problem count: 90 +# solution count: 127 +# total problem count: 420 +# total solution count: 580 \ No newline at end of file diff --git a/USA_TSTST/segmented/en-sols-TSTST-2011.jsonl b/USA_TSTST/segmented/en-sols-TSTST-2011.jsonl new file mode 100644 index 0000000000000000000000000000000000000000..6100bab146b4c044d16e2a6d3bea183a5bf1cf9a --- /dev/null +++ b/USA_TSTST/segmented/en-sols-TSTST-2011.jsonl @@ -0,0 +1,9 @@ +{"year":2011,"label":"1","problem":"Find all real-valued functions $f$ defined on pairs of real numbers, having the following property: for all real numbers $a, b, c$, the median of $f(a, b), f(b, c), f(c, a)$ equals the median of $a, b, c$. (The median of three real numbers, not necessarily distinct, is the number that is in the middle when the three numbers are arranged in nondecreasing order.)","solution":" We prove the following main claim, from which repeated applications can deduce the problem. Claim - Let $ad_{2}>\\cdots>d_{k}$ and $\\operatorname{gcd}\\left(d_{1}, d_{2}, \\ldots, d_{k}\\right)=1$. For every integer $n \\geq n_{0}$, define $$ x_{n}=\\left\\lfloor\\frac{x_{n-d_{1}}+x_{n-d_{2}}+\\cdots+x_{n-d_{k}}}{k}\\right\\rfloor . $$ Show that the sequence $\\left(x_{n}\\right)$ is eventually constant.","solution":" Note that if the initial terms are contained in some interval $[A, B]$ then they will remain in that interval. Thus the sequence is eventually periodic. Discard initial terms and let the period be $T$; we will consider all indices modulo $T$ from now on. Let $M$ be the maximal term in the sequence (which makes sense since the sequence is periodic). Note that if $x_{n}=M$, we must have $x_{n-d_{i}}=M$ for all $i$ as well. By taking a linear combination $\\sum c_{i} d_{i} \\equiv 1(\\bmod T)($ possibly be Bezout's theorem, since $\\operatorname{gcd}_{i}\\left(d_{i}\\right)=1$ ), we conclude $x_{n-1}=M$, as desired."} +{"year":2011,"label":"9","problem":"Let $n$ be a positive integer. Suppose we are given $2^{n}+1$ distinct sets, each containing finitely many objects. Place each set into one of two categories, the red sets and the blue sets, so that there is at least one set in each category. We define the symmetric difference of two sets as the set of objects belonging to exactly one of the two sets. Prove that there are at least $2^{n}$ different sets which can be obtained as the symmetric difference of a red set and a blue set.","solution":" We can interpret the problem as working with binary strings of length $\\ell \\geq n+1$, with $\\ell$ the number of elements across all sets. Let $F$ be a field of cardinality $2^{\\ell}$, hence $F \\cong \\mathbb{F}_{2}^{\\oplus \\ell}$. Then, we can think of red\/blue as elements of $F$, so we have some $B \\subseteq F$, and an $R \\subseteq F$. We wish to prove that $|B+R| \\geq 2^{n}$. Want $|B+R| \\geq 2^{n}$. Equivalently, any element of a set $X$ with $|X|=2^{n}-1$ should omit some element of $|B+R|$. To prove this: we know $|B|+|R|=2^{n}+1$, and define $$ P(b, r)=\\prod_{x \\in X}(b+r-x) $$ Consider $b^{|B|-1} r^{|R|-1}$. The coefficient of is $\\binom{2^{n}-1}{|B|-1}$, which is odd (say by Lucas theorem), so the nullstellensatz applies."} diff --git a/USA_TSTST/segmented/en-sols-TSTST-2012.jsonl b/USA_TSTST/segmented/en-sols-TSTST-2012.jsonl new file mode 100644 index 0000000000000000000000000000000000000000..3b0cf6765d84e413ae83903560c78be9386923e8 --- /dev/null +++ b/USA_TSTST/segmented/en-sols-TSTST-2012.jsonl @@ -0,0 +1,13 @@ +{"year":2012,"label":"1","problem":"Determine all infinite strings of letters with the following properties: (a) Each letter is either $T$ or $S$, (b) If position $i$ and $j$ both have the letter $T$, then position $i+j$ has the letter $S$, (c) There are infinitely many integers $k$ such that position $2 k-1$ has the $k$ th $T$.","solution":" We wish to find all infinite sequences $a_{1}, a_{2}, \\ldots$ of positive integers satisfying the following properties: (a) $a_{1}1$, let $A_{k}=\\left\\{a_{1}, a_{2}, \\ldots, a_{k}\\right\\}$. By (b) and symmetry, we have $$ 2 k-1 \\geq \\frac{\\left|A_{k}-A_{k}\\right|-1}{2}+\\left|A_{k}\\right| \\geq \\frac{2\\left|A_{k}\\right|-2}{2}+\\left|A_{k}\\right|=2 k-1 . $$ But in order for $\\left|A_{k}-A_{k}\\right|=2\\left|A_{k}\\right|-1$, we must have $A_{k}$ an arithmetic progression, whence $a_{n}=2 n-1$ for all $n$ by taking $k$ arbitrarily large."} +{"year":2012,"label":"2","problem":"Let $A B C D$ be a quadrilateral with $A C=B D$. Diagonals $A C$ and $B D$ meet at $P$. Let $\\omega_{1}$ and $O_{1}$ denote the circumcircle and circumcenter of triangle $A B P$. Let $\\omega_{2}$ and $O_{2}$ denote the circumcircle and circumcenter of triangle $C D P$. Segment $B C$ meets $\\omega_{1}$ and $\\omega_{2}$ again at $S$ and $T$ (other than $B$ and $C$ ), respectively. Let $M$ and $N$ be the midpoints of minor arcs $\\widehat{S P}$ (not including $B$ ) and $\\overparen{T P}$ (not including $C$ ). Prove that $\\overline{M N} \\| \\overline{O_{1} O_{2}}$.","solution":" Let $Q$ be the second intersection point of $\\omega_{1}, \\omega_{2}$. Suffice to show $\\overline{Q P} \\perp \\overline{M N}$. Now $Q$ is the center of a spiral congruence which sends $\\overline{A C} \\mapsto \\overline{B D}$. So $\\triangle Q A B$ and $\\triangle Q C D$ are similar isosceles. Now, $$ \\measuredangle Q P A=\\measuredangle Q B A=\\measuredangle D C Q=\\measuredangle D P Q $$ and so $\\overline{Q P}$ is bisects $\\angle B P C$. ![](https:\/\/cdn.mathpix.com\/cropped\/2024_11_19_b4303c9fa48a8c74b45cg-05.jpg?height=673&width=1200&top_left_y=1234&top_left_x=428) Now, let $I=\\overline{B M} \\cap \\overline{C N} \\cap \\overline{P Q}$ be the incenter of $\\triangle P B C$. Then $I M \\cdot I B=I P \\cdot I Q=$ $I N \\cdot I C$, so $B M N C$ is cyclic, meaning $\\overline{M N}$ is antiparallel to $\\overline{B C}$ through $\\angle B I C$. Since $\\overline{Q P I}$ passes through the circumcenter of $\\triangle B I C$, it follows now $\\overline{Q P I} \\perp \\overline{M N}$ as desired."} +{"year":2012,"label":"3","problem":"Let $\\mathbb{N}$ be the set of positive integers. Let $f: \\mathbb{N} \\rightarrow \\mathbb{N}$ be a function satisfying the following two conditions: (a) $f(m)$ and $f(n)$ are relatively prime whenever $m$ and $n$ are relatively prime. (b) $n \\leq f(n) \\leq n+2012$ for all $n$. Prove that for any natural number $n$ and any prime $p$, if $p$ divides $f(n)$ then $p$ divides $n$.","solution":" \u3010 First short solution, by Jeffrey Kwan. Let $p_{0}, p_{1}, p_{2}, \\ldots$ denote the sequence of all prime numbers, in any order. Pick any primes $q_{i}$ such that $$ q_{0}\\left|f\\left(p_{0}\\right), \\quad q_{1}\\right| f\\left(p_{1}\\right), \\quad q_{2} \\mid f\\left(p_{2}\\right), \\text { etc. } $$ This is possible since each $f$ value above exceeds 1 . Also, since by hypothesis the $f\\left(p_{i}\\right)$ are pairwise coprime, the primes $q_{i}$ are all pairwise distinct. Claim - We must have $q_{i}=p_{i}$ for each $i$. (Therefore, $f\\left(p_{i}\\right)$ is a power of $p_{i}$ for each $i$.) $$ \\begin{array}{rr} m+i & \\equiv 0 \\\\ m & \\left(\\bmod q_{i}\\right) \\\\ m & \\equiv{ }^{\\prime} \\\\ \\left(\\bmod p_{i}\\right) \\end{array} $$ for $0 \\leq i \\leq 2012$. But now $f(m)$ should be coprime to all $f\\left(p_{i}\\right)$, ergo coprime to $q_{0} q_{1} \\ldots q_{2012}$, violating $m \\leq f(m) \\leq m+2012$. All that is left to do is note that whenever $p \\nmid n$, we have $\\operatorname{gcd}(f(p), f(n))=1$, hence $p \\nmid f(n)$. This is the contrapositive of the problem statement."} +{"year":2012,"label":"3","problem":"Let $\\mathbb{N}$ be the set of positive integers. Let $f: \\mathbb{N} \\rightarrow \\mathbb{N}$ be a function satisfying the following two conditions: (a) $f(m)$ and $f(n)$ are relatively prime whenever $m$ and $n$ are relatively prime. (b) $n \\leq f(n) \\leq n+2012$ for all $n$. Prove that for any natural number $n$ and any prime $p$, if $p$ divides $f(n)$ then $p$ divides $n$.","solution":" \u3010 Second solution with a grid. Fix $n$ and $p$, and assume for contradiction $p \\nmid n$. Claim - There exists a large integer $N$ with $f(N)=N$, that also satisfies $N \\equiv 1$ $(\\bmod n)$ and $N \\equiv 0(\\bmod p)$. it to fill in the following table: | | $N+1$ | $N+2$ | $\\ldots$ | $N+2012$ | | :---: | :---: | :---: | :---: | :---: | | $M$ | $q_{0,1}$ | $q_{0,2}$ | $\\ldots$ | $q_{0,2012}$ | | $M+1$ | $q_{1,1}$ | $q_{1,2}$ | $\\ldots$ | $q_{1,2012}$ | | $\\vdots$ | $\\vdots$ | $\\vdots$ | $\\ddots$ | $\\vdots$ | | $M+2012$ | $q_{2012,1}$ | $q_{2012,2}$ | $\\ldots$ | $q_{2012,2012}$ |. By the Chinese Remainder Theorem, we can construct $N$ such that $N+1 \\equiv 0\\left(\\bmod q_{i, 1}\\right)$ for every $i$, and similarly for $N+2$, and so on. Moreover, we can also tack on the extra conditions $N \\equiv 0(\\bmod p)$ and $N \\equiv 1(\\bmod n)$ we wanted. Notice that $N$ cannot be divisible by any of the $q_{i, j}$ 's, since the $q_{i, j}$ 's are greater than 2012. After we've chosen $N$, we can pick $M$ such that $M \\equiv 0\\left(\\bmod q_{0, j}\\right)$ for every $j$, and similarly $M+1 \\equiv 0\\left(\\bmod q_{1, j}\\right)$, et cetera. Moreover, we can tack on the condition $M \\equiv 1$ $(\\bmod N)$, which ensures $\\operatorname{gcd}(M, N)=1$. What does this do? We claim that $f(N)=N$ now. Indeed $f(M)$ and $f(N)$ are relatively prime; but look at the table! The table tells us that $f(M)$ must have a common factor with each of $N+1, \\ldots, N+2012$. So the only possibility is that $f(N)=N$. Now we're basically done. Since $N \\equiv 1(\\bmod n)$, we have $\\operatorname{gcd}(N, n)=1$ and hence $1=\\operatorname{gcd}(f(N), f(n))=\\operatorname{gcd}(N, f(n))$. But $p \\mid N$ and $p \\mid f(n)$, contradiction."} +{"year":2012,"label":"4","problem":"In scalene triangle $A B C$, let the feet of the perpendiculars from $A$ to $\\overline{B C}, B$ to $\\overline{C A}$, $C$ to $\\overline{A B}$ be $A_{1}, B_{1}, C_{1}$, respectively. Denote by $A_{2}$ the intersection of lines $B C$ and $B_{1} C_{1}$. Define $B_{2}$ and $C_{2}$ analogously. Let $D, E, F$ be the respective midpoints of sides $\\overline{B C}, \\overline{C A}, \\overline{A B}$. Show that the perpendiculars from $D$ to $\\overline{A A_{2}}, E$ to $\\overline{B B_{2}}$ and $F$ to $\\overline{C C_{2}}$ are concurrent.","solution":" We claim that they pass through the orthocenter $H$. Indeed, consider the circle with diameter $\\overline{B C}$, which circumscribes quadrilateral $B C B_{1} C_{1}$ and has center $D$. Then by Brokard theorem, $\\overline{A A_{2}}$ is the polar of line $H$. Thus $\\overline{D H} \\perp \\overline{A A_{2}}$."} +{"year":2012,"label":"5","problem":"A rational number $x$ is given. Prove that there exists a sequence $x_{0}, x_{1}, x_{2}, \\ldots$ of rational numbers with the following properties: (a) $x_{0}=x$; (b) for every $n \\geq 1$, either $x_{n}=2 x_{n-1}$ or $x_{n}=2 x_{n-1}+\\frac{1}{n}$; (c) $x_{n}$ is an integer for some $n$.","solution":" Think of the sequence as a process over time. We'll show that: Claim - At any given time $t$, if the denominator of $x_{t}$ has some odd prime power $q=p^{e}$, then we can delete a factor of $p$ from the denominator, while only adding powers of two to the denominator. (Thus we can just delete off all the odd primes one by one and then double appropriately many times.) Indeed, let $n$ be large, and suppose $t<2^{r+1} q<2^{r+2} q<\\cdots<2^{r+m} qb$ if $a b=b a=a$. The following are proved by finite casework, using the fact that $\\{a b, b c, c a\\}$ always has exactly two distinct elements for any different $a, b, c$. - If $a>b$ and $b>c$ then $a>c$. - If $a \\sim b$ and $b \\sim c$ then $a b=a$ if and only if $b c=b$. - If $a \\sim b$ and $b \\sim c$ then $a \\sim c$. - If $a \\sim b$ and $a>c$ then $b>c$. - If $a \\sim b$ and $c>a$ then $c>b$. This gives us the total ordering on the elements and the equivalence classes by $\\sim$. In this we way can check the claimed operations are the only ones."} diff --git a/USA_TSTST/segmented/en-sols-TSTST-2013.jsonl b/USA_TSTST/segmented/en-sols-TSTST-2013.jsonl new file mode 100644 index 0000000000000000000000000000000000000000..8f8fde55e80b86e54230a84d535b23365e0f5003 --- /dev/null +++ b/USA_TSTST/segmented/en-sols-TSTST-2013.jsonl @@ -0,0 +1,9 @@ +{"year":2013,"label":"1","problem":"Let $A B C$ be a triangle and $D, E, F$ be the midpoints of $\\operatorname{arcs} B C, C A, A B$ on the circumcircle. Line $\\ell_{a}$ passes through the feet of the perpendiculars from $A$ to $\\overline{D B}$ and $\\overline{D C}$. Line $m_{a}$ passes through the feet of the perpendiculars from $D$ to $\\overline{A B}$ and $\\overline{A C}$. Let $A_{1}$ denote the intersection of lines $\\ell_{a}$ and $m_{a}$. Define points $B_{1}$ and $C_{1}$ similarly. Prove that triangles $D E F$ and $A_{1} B_{1} C_{1}$ are similar to each other.","solution":" In fact, it is true for any points $D, E, F$ on the circumcircle. More strongly we contend: Claim - Point $A_{1}$ is the midpoint of $\\overline{H D}$. ![](https:\/\/cdn.mathpix.com\/cropped\/2024_11_19_3879e3a8fa25bc819b5bg-04.jpg?height=718&width=801&top_left_y=1297&top_left_x=633) Hence $A_{1} B_{1} C_{1}$ is similar to $D E F$ through a homothety at $H$ with ratio $\\frac{1}{2}$."} +{"year":2013,"label":"2","problem":"A finite sequence of integers $a_{1}, a_{2}, \\ldots, a_{n}$ is called regular if there exists a real number $x$ satisfying $$ \\lfloor k x\\rfloor=a_{k} \\quad \\text { for } 1 \\leq k \\leq n $$ Given a regular sequence $a_{1}, a_{2}, \\ldots, a_{n}$, for $1 \\leq k \\leq n$ we say that the term $a_{k}$ is forced if the following condition is satisfied: the sequence $$ a_{1}, a_{2}, \\ldots, a_{k-1}, b $$ is regular if and only if $b=a_{k}$. Find the maximum possible number of forced terms in a regular sequence with 1000 terms.","solution":" The answer is 985 . WLOG, by shifting $a_{1}=0$ (clearly $a_{1}$ isn't forced). Now, we construct regular sequences inductively using the following procedure. Start with the inequality $$ \\frac{0}{1} \\leq x<\\frac{1}{1} $$ Then for each $k=2,3, \\ldots, 1000$ we perform the following procedure. If there is no fraction of the form $F=\\frac{m}{k}$ in the interval $A \\leq xq>\\max \\{a, b\\}$ be primes. Suppose $s=a^{p} b^{q}$ and $t=s^{2}$; then $$ p g_{t}(a)+q g_{t}(b)=g_{t}\\left(a^{p} b^{q}\\right)=g_{t}(s)=f^{s^{2}-s}(s)-s=0 $$ Now $$ q \\mid g_{t}(a)>-a \\quad \\text { and } \\quad p \\mid g_{t}(b)>-b \\Longrightarrow g_{t}(a)=g_{t}(b)=0 $$ and so we conclude $f^{t-a}(t)=a$ and $f^{t-b}(t)=b$ for $a, b \\geq 2$. In particular, if $a=n$ and $b=n+1$ then we deduce $f(n+1)=n$ for all $n \\geq 2$, as desired. Remark. If you let $c=(a b)^{2}$ after the first lemma, you recover the 2-variable version!"} +{"year":2013,"label":"7","problem":"A country has $n$ cities, labelled $1,2,3, \\ldots, n$. It wants to build exactly $n-1$ roads between certain pairs of cities so that every city is reachable from every other city via some sequence of roads. However, it is not permitted to put roads between pairs of cities that have labels differing by exactly 1 , and it is also not permitted to put a road between cities 1 and $n$. Let $T_{n}$ be the total number of possible ways to build these roads. (a) For all odd $n$, prove that $T_{n}$ is divisible by $n$. (b) For all even $n$, prove that $T_{n}$ is divisible by $n \/ 2$.","solution":" You can just spin the tree! Fixing $n$, the group $G=\\mathbb{Z} \/ n \\mathbb{Z}$ acts on the set of trees by rotation (where we imagine placing $1,2, \\ldots, n$ along a circle). Claim - For odd $n$, all trees have trivial stabilizer."} +{"year":2013,"label":"8","problem":"Define a function $f: \\mathbb{N} \\rightarrow \\mathbb{N}$ by $f(1)=1, f(n+1)=f(n)+2^{f(n)}$ for every positive integer $n$. Prove that $f(1), f(2), \\ldots, f\\left(3^{2013}\\right)$ leave distinct remainders when divided by $3^{2013}$.","solution":" I'll prove by induction on $k \\geq 1$ that any $3^{k}$ consecutive values of $f$ produce distinct residues modulo $3^{k}$. The base case $k=1$ is easily checked ( $f$ is always odd, hence $f$ cycles $1,0,2 \\bmod 3)$. For the inductive step, assume it's true up to $k$. Since $2^{\\bullet}\\left(\\bmod 3^{k+1}\\right)$ cycles every $2 \\cdot 3^{k}$, and $f$ is always odd, it follows that $$ \\begin{aligned} f\\left(n+3^{k}\\right)-f(n) & =2^{f(n)}+2^{f(n+1)}+\\cdots+2^{f\\left(n+3^{k}-1\\right)} \\quad\\left(\\bmod 3^{k+1}\\right) \\\\ & \\equiv 2^{1}+2^{3}+\\cdots+2^{2 \\cdot 3^{k}-1} \\quad\\left(\\bmod 3^{k+1}\\right) \\\\ & =2 \\cdot \\frac{4^{3^{k}}-1}{4-1} \\end{aligned} $$ Hence $$ f\\left(n+3^{k}\\right)-f(n) \\equiv C \\quad\\left(\\bmod 3^{k+1}\\right) \\quad \\text { where } \\quad C=2 \\cdot \\frac{4^{3^{k}}-1}{4-1} $$ noting that $C$ does not depend on $n$. Exponent lifting gives $\\nu_{3}(C)=k$ hence $f(n)$, $f\\left(n+3^{k}\\right), f\\left(n+2 \\cdot 3^{k}\\right)$ differ mod $3^{k+1}$ for all $n$, and the inductive hypothesis now solves the problem."} +{"year":2013,"label":"9","problem":"Let $r$ be a rational number in the interval $[-1,1]$ and let $\\theta=\\cos ^{-1} r$. Call a subset $S$ of the plane good if $S$ is unchanged upon rotation by $\\theta$ around any point of $S$ (in both clockwise and counterclockwise directions). Determine all values of $r$ satisfying the following property: The midpoint of any two points in a good set also lies in the set.","solution":" The answer is that $r$ has this property if and only if $r=\\frac{4 n-1}{4 n}$ for some integer $n$. $$ \\omega=e^{i \\theta}=\\frac{a}{b} \\pm \\frac{\\sqrt{b^{2}-a^{2}}}{b} i $$ This means we may work with complex multiplication in the usual way; the rotation of $z$ through center $c$ is given by $z \\mapsto \\omega(z-c)+c$. - Start by letting $S_{0}=\\{0,1\\}$. - Let $S_{i}$ consist of $S_{i-1}$ plus all points that can be obtained by rotating a point of $S_{i-1}$ through a different point of $S_{i-1}$ (with scale factor $\\omega$ ). - Let $S_{\\infty}=\\bigcup_{i \\geq 0} S_{i}$. The set $S_{\\infty}$ is the (minimal, by inclusion) good set containing 0 and 1 . We are going to show that for most values of $r$, we have $\\frac{1}{2} \\notin S_{\\infty}$. Claim - If $b$ is odd, then $\\frac{1}{2} \\notin S_{\\infty}$. Consider the ring $$ A=\\mathbb{Z}_{\\{b\\}}=\\left\\{\\frac{s}{t}|s, t \\in \\mathbb{Z}, t| b^{\\infty}\\right\\} $$ which consists of all rational numbers whose denominators divide $b^{\\infty}$. Then, $0,1, \\omega \\in$ $A\\left[\\sqrt{b^{2}-a^{2}}\\right]$ and hence $S_{\\infty} \\subseteq A\\left[\\sqrt{b^{2}-a^{2}}\\right]$ too. (This works even if $\\sqrt{b^{2}-a^{2}} \\in \\mathbb{Z}$, in which case $S_{\\infty} \\subseteq A=A\\left[\\sqrt{b^{2}-a^{2}}\\right]$.) But $\\frac{1}{2} \\notin A\\left[\\sqrt{b^{2}-a^{2}}\\right]$. Claim - If $b$ is even and $|b-a| \\neq 1$, then $\\frac{1}{2} \\notin S_{\\infty}$. Let $D=b^{2}-a^{2} \\equiv 3(\\bmod 4)$. Let $p$ be a prime divisor of $b-a$. Because $\\operatorname{gcd}(a, b)=1$, we have $p \\neq 2$ and $p \\nmid b$. Consider the ring $$ A=\\mathbb{Z}_{(p)}=\\left\\{\\left.\\frac{s}{t} \\right\\rvert\\, s, t \\in \\mathbb{Z}, p \\perp t\\right\\} $$ which consists of all rational numbers whose denominators are coprime to $p$. Then, $0,1, \\omega \\in A[\\sqrt{-D}]$ and hence $S_{\\infty} \\subseteq A[\\sqrt{-D}]$ too. Now, there is a well-defined \"mod- $p$ \" ring homomorphism $$ \\Psi: A[\\sqrt{-D}] \\rightarrow \\mathbb{F}_{p} \\quad \\text { by } \\quad x+y \\sqrt{-D} \\mapsto x \\bmod p $$ which commutes with addition and multiplication (as $p \\mid D$ ). Under this map, $$ \\omega \\mapsto \\frac{a}{b} \\bmod p=1 $$ Consequently, the rotation $z \\mapsto \\omega(z-c)+c$ is just the identity map modulo $p$. In other words, the pre-image of any point in $S_{\\infty}$ under $\\Psi$ must be either $\\Psi(0)=0$ or $\\Psi(1)=1$. However, $\\Psi(1 \/ 2)=1 \/ 2$ is neither of these. So this point cannot be achieved. Claim - Suppose $a=2 n-1$ and $b=2 n$ for $n$ an odd integer. Then $\\frac{1}{2} \\notin S_{\\infty}$ This time, we define the ring $$ B=\\mathbb{Z}_{(2)}=\\left\\{\\left.\\frac{s}{t} \\right\\rvert\\, s, t \\in \\mathbb{Z}, t \\text { odd }\\right\\} $$ of rational numbers with odd denominator. We carefully consider the ring $B[\\omega]$ where $$ \\omega=\\frac{2 n-1 \\pm \\sqrt{1-4 n}}{2 n} $$ So $S_{\\infty} \\subseteq B[\\omega]$ as $0,1, \\omega \\in B[\\omega]$. I claim that $B[\\omega]$ is an integral extension of $B$; equivalently that $\\omega$ is integral over $B$. Indeed, $\\omega$ is the root of the monic polynomial $$ (T-1)^{2}+\\frac{1}{n}(T-1)-\\frac{1}{n}=0 $$ where $\\frac{1}{n} \\in B$ makes sense as $n$ is odd. On the other hand, $\\frac{1}{2}$ is not integral over $B$ so it is not an element of $B[\\omega]$. It remains to show that if $r=\\frac{4 n-1}{4 n}$, then goods sets satisfy the midpoint property. Again starting from the points $z_{0}=0, z_{1}=1$ construct the sequence $$ \\begin{aligned} z_{2} & =\\omega\\left(z_{1}-z_{0}\\right)+z_{0} \\\\ z_{3} & =\\omega^{-1}\\left(z_{0}-z_{2}\\right)+z_{2} \\\\ z_{4} & =\\omega^{-1}\\left(z_{2}-z_{3}\\right)+z_{3} \\\\ z_{5} & =\\omega\\left(z_{3}-z_{4}\\right)+z_{4} \\end{aligned} $$ as shown in the diagram below. ![](https:\/\/cdn.mathpix.com\/cropped\/2024_11_19_3879e3a8fa25bc819b5bg-16.jpg?height=321&width=795&top_left_y=2307&top_left_x=633) This construction shows that if we have the length-one segment $\\{0,1\\}$ then we can construct the length-one segment $\\{2 r-2,2 r-1\\}$. In other words, we can shift the segment to the left by $$ 1-(2 r-1)=2(1-r)=\\frac{1}{2 n} $$ Repeating this construction $n$ times gives the desired midpoint $\\frac{1}{2}$."} diff --git a/USA_TSTST/segmented/en-sols-TSTST-2014.jsonl b/USA_TSTST/segmented/en-sols-TSTST-2014.jsonl new file mode 100644 index 0000000000000000000000000000000000000000..2535577c182d9f605e2090a7ca58c859b21ef3ce --- /dev/null +++ b/USA_TSTST/segmented/en-sols-TSTST-2014.jsonl @@ -0,0 +1,6 @@ +{"year":2014,"label":"1","problem":"Let $\\leftarrow$ denote the left arrow key on a standard keyboard. If one opens a text editor and types the keys \"ab $\\leftarrow \\mathrm{cd} \\leftarrow \\leftarrow \\mathrm{e} \\leftarrow \\leftarrow \\mathrm{f}$ \", the result is \"faecdb\". We say that a string $B$ is reachable from a string $A$ if it is possible to insert some amount of $\\leftarrow$ 's in $A$, such that typing the resulting characters produces $B$. So, our example shows that \"faecdb\" is reachable from \"abcdef\". Prove that for any two strings $A$ and $B, A$ is reachable from $B$ if and only if $B$ is reachable from $A$.","solution":" Obviously $A$ and $B$ should have the same multiset of characters, and we focus only on that situation. Claim - If $A=123 \\ldots n$ and $B=\\sigma(1) \\sigma(2) \\ldots \\sigma(n)$ is a permutation of $A$, then $B$ is reachable if and only if it is $\\mathbf{2 1 3}$-avoiding, i.e. there are no indices $i0}$ such that $x \\equiv y \\equiv 1(\\bmod p)$. If the sequence $\\nu_{p}\\left(x^{n}-y\\right)$ of positive integers is nonconstant, then it is unbounded. For this it would be sufficient to prove the following claim. Claim - Let $p$ be an odd prime. Let $x, y \\in \\mathbb{Q}>0$ such that $x \\equiv y \\equiv 1(\\bmod p)$. Suppose $m$ and $n$ are positive integers such that $$ d=\\nu_{p}\\left(x^{n}-y\\right)<\\nu_{p}\\left(x^{m}-y\\right)=e . $$ Then there exists $\\ell$ such that $\\nu_{p}\\left(x^{\\ell}-y\\right) \\geq e+1$. $$ \\nu_{p}\\left(x^{k}-1\\right)=e $$ namely $k=p^{e-d}|m-n|$. (In fact, one could also choose more carefully $k=p^{e-d} \\cdot \\operatorname{gcd}(m-$ $\\left.n, p^{\\infty}\\right)$, so that $k$ is a power of $p$.) Suppose we set $x^{k}=p^{e} u+1$ and $x^{m}=p^{e} v+y$ where $u, v \\in \\mathbb{Q}$ aren't divisible by $p$. Now for any integer $1 \\leq r \\leq p-1$ we consider $$ \\begin{aligned} x^{k r+m}-y & =\\left(p^{e} u+1\\right)^{r} \\cdot\\left(p^{e} v+y\\right)-y \\\\ & =p^{e}(v+y u \\cdot r)+p^{2 e}(\\ldots) \\end{aligned} $$ By selecting $r$ with $r \\equiv-v \/ u(\\bmod p)$, we ensure $p^{e+1} \\mid x^{k r+m}-y$, hence $\\ell=k r+m$ is as desired. $$ x^{\\ell-m} \\equiv 1 \\quad\\left(\\bmod p^{e}\\right) \\text { but } x^{\\ell-m} \\not \\equiv 1 \\quad\\left(\\bmod p^{e+1}\\right) $$ In particular, we need $\\nu_{p}\\left(x^{\\ell-m}-1\\right)=e$ exactly. So the $k$ in the claim must exist if we are going to succeed. On the other hand, if $k$ is some integer for which $\\nu_{p}\\left(x^{k}-1\\right)=e$, then by choosing $\\ell-m$ to be some multiple of $k$ with no extra factors of $p$, we hope that we can get $\\nu_{p}\\left(x^{\\ell}-y\\right)=e+1$. That's why we write $\\ell=k r+m$ and see what happens when we expand."} diff --git a/USA_TSTST/segmented/en-sols-TSTST-2015.jsonl b/USA_TSTST/segmented/en-sols-TSTST-2015.jsonl new file mode 100644 index 0000000000000000000000000000000000000000..54f867718d97a982e1bb0c45992c6fd143b9aaaa --- /dev/null +++ b/USA_TSTST/segmented/en-sols-TSTST-2015.jsonl @@ -0,0 +1,9 @@ +{"year":2015,"label":"1","problem":"Let $a_{1}, a_{2}, \\ldots, a_{n}$ be a sequence of real numbers, and let $m$ be a fixed positive integer less than $n$. We say an index $k$ with $1 \\leq k \\leq n$ is good if there exists some $\\ell$ with $1 \\leq \\ell \\leq m$ such that $$ a_{k}+a_{k+1}+\\cdots+a_{k+\\ell-1} \\geq 0 $$ where the indices are taken modulo $n$. Let $T$ be the set of all good indices. Prove that $$ \\sum_{k \\in T} a_{k} \\geq 0 $$","solution":" First we prove the result if the indices are not taken modulo $n$. Call a number $\\ell$-good if $\\ell$ is the smallest number such that $a_{k}+a_{k+1}+\\cdots+a_{k+\\ell-1} \\geq 0$, and $\\ell \\leq m$. Then if $a_{k}$ is $\\ell$-good, the numbers $a_{k+1}, \\ldots, a_{k+\\ell-1}$ are good as well. Then by greedy from left to right, we can group all the good numbers into blocks with nonnegative sums. Repeatedly take the first good number, if $\\ell$-good, group it with the next $\\ell$ numbers. An example for $m=3$ : $$ \\langle 4\\rangle \\quad \\begin{array}{ccccccccc} -1 & -2 & 6\\rangle & -9 & -7 & \\langle 3\\rangle & \\langle-2 & 4\\rangle & \\langle-1 . \\end{array} $$ We can now return to the original problem. Let $N$ be a large integer; applying the algorithm to $N$ copies of the sequence, we deduce that $$ N \\sum_{k \\in T} a_{k}+c_{N} \\geq 0 $$ where $c_{N}$ represents some \"error\" from left-over terms. As $\\left|c_{N}\\right| \\leq \\sum\\left|a_{i}\\right|$, by taking $N$ large enough we deduce the problem."} +{"year":2015,"label":"2","problem":"Let $A B C$ be a scalene triangle. Let $K_{a}, L_{a}$, and $M_{a}$ be the respective intersections with $B C$ of the internal angle bisector, external angle bisector, and the median from $A$. The circumcircle of $A K_{a} L_{a}$ intersects $A M_{a}$ a second time at a point $X_{a}$ different from $A$. Define $X_{b}$ and $X_{c}$ analogously. Prove that the circumcenter of $X_{a} X_{b} X_{c}$ lies on the Euler line of $A B C$.","solution":" The main content of the problem: Claim - $\\angle H X_{a} G=90^{\\circ}$. This implies the result, since then the desired circumcenter is the midpoint of $\\overline{G H}$. (This is the main difficulty; the Euler line is a red herring.) In what follows, we abbreviate $K_{a} L_{a}, M_{a}, X_{a}$ to $K, L, M, X$. First proof by Brokard. To do this, it suffices to show that $M$ has the same power with respect to the circle with diameter $\\overline{A H}$ and the circle with diameter $\\overline{K L}$. In fact I claim both circles are orthogonal to the circle with diameter $\\overline{B C}$ ! The former follows from Brokard's theorem, noting that $A$ is on the polar of $H$, and the latter follows from the harmonic bundle. ![](https:\/\/cdn.mathpix.com\/cropped\/2024_11_19_8fb7b517def5801eef36g-04.jpg?height=635&width=1109&top_left_y=1410&top_left_x=482) Then $\\overline{A M}$ is the radical axis, so $X$ lies on both circles."} +{"year":2015,"label":"2","problem":"Let $A B C$ be a scalene triangle. Let $K_{a}, L_{a}$, and $M_{a}$ be the respective intersections with $B C$ of the internal angle bisector, external angle bisector, and the median from $A$. The circumcircle of $A K_{a} L_{a}$ intersects $A M_{a}$ a second time at a point $X_{a}$ different from $A$. Define $X_{b}$ and $X_{c}$ analogously. Prove that the circumcenter of $X_{a} X_{b} X_{c}$ lies on the Euler line of $A B C$.","solution":" The main content of the problem: Claim - $\\angle H X_{a} G=90^{\\circ}$. This implies the result, since then the desired circumcenter is the midpoint of $\\overline{G H}$. (This is the main difficulty; the Euler line is a red herring.) In what follows, we abbreviate $K_{a} L_{a}, M_{a}, X_{a}$ to $K, L, M, X$. Second proof by orthocenter reflection, Bendit Chan. As before, we know $M X \\cdot M A=$ $M K \\cdot M L=M B \\cdot M C$, but $X$ lies inside segment $A M$. Construct parallelogram $A B A^{\\prime} C$. Then $M X \\cdot M A^{\\prime}=M B \\cdot M C$, so $X B A^{\\prime} C$ is concyclic. However, it is well-known the circumcircle of $\\triangle B A^{\\prime} C$ (which is the reflection of $(A B C)$ across $\\overline{B C}$ ) passes through $H$ and in fact has diameter $\\overline{A^{\\prime} H}$. So this gives $\\angle H X A^{\\prime}=90^{\\circ}$ as needed."} +{"year":2015,"label":"2","problem":"Let $A B C$ be a scalene triangle. Let $K_{a}, L_{a}$, and $M_{a}$ be the respective intersections with $B C$ of the internal angle bisector, external angle bisector, and the median from $A$. The circumcircle of $A K_{a} L_{a}$ intersects $A M_{a}$ a second time at a point $X_{a}$ different from $A$. Define $X_{b}$ and $X_{c}$ analogously. Prove that the circumcenter of $X_{a} X_{b} X_{c}$ lies on the Euler line of $A B C$.","solution":" The main content of the problem: Claim - $\\angle H X_{a} G=90^{\\circ}$. This implies the result, since then the desired circumcenter is the midpoint of $\\overline{G H}$. (This is the main difficulty; the Euler line is a red herring.) In what follows, we abbreviate $K_{a} L_{a}, M_{a}, X_{a}$ to $K, L, M, X$. Third proof by barycentric coordinates. Alternatively we may just compute $X=\\left(a^{2}\\right.$ : $\\left.2 S_{A}: 2 S_{A}\\right)$. Let $F=\\left(0: S_{C}: S_{B}\\right)$ be the foot from $H$. Then we check that $X H F M$ is cyclic, which is power of a point from $A$."} +{"year":2015,"label":"3","problem":"Let $P$ be the set of all primes, and let $M$ be a non-empty subset of $P$. Suppose that for any non-empty subset $\\left\\{p_{1}, p_{2}, \\ldots, p_{k}\\right\\}$ of $M$, all prime factors of $p_{1} p_{2} \\ldots p_{k}+1$ are also in $M$. Prove that $M=P$.","solution":" Obviously $|M|=\\infty$. Assume for contradiction $p \\notin M$. We say a prime $q \\in M$ is sparse if there are only finitely many elements of $M$ which are $q(\\bmod p)$ (in particular there are finitely many sparse primes). Now let $C$ be the product of all sparse primes (note $p \\nmid C$ ). First, set $a_{0}=1$. For $k \\geq 0$, consider then the prime factorization of the number $$ C a_{k}+1 $$ No prime in its factorization is sparse, so consider the number $a_{k+1}$ obtained by replacing each prime in its factorization with some arbitrary representative of that prime's residue class. In this way we select a number $a_{k+1}$ such that - $a_{k+1} \\equiv C a_{k}+1(\\bmod p)$, and - $a_{k+1}$ is a product of distinct primes in $M$. In particular, $a_{k} \\equiv C^{k}+C^{k-1}+\\cdots+1(\\bmod p)$ But since $C \\not \\equiv 0(\\bmod p)$, we can find a $k$ such that $a_{k} \\equiv 0(\\bmod p)($ namely, $k=p-1$ if $C \\equiv 1$ and $k=p-2$ else) which is clearly impossible since $a_{k}$ is a product of primes in $M$ !"} +{"year":2015,"label":"4","problem":"Let $x, y, z$ be real numbers (not necessarily positive) such that $x^{4}+y^{4}+z^{4}+x y z=4$. Prove that $x \\leq 2$ and $$ \\sqrt{2-x} \\geq \\frac{y+z}{2} $$","solution":" We prove that the condition $x^{4}+y^{4}+z^{4}+x y z=4$ implies $$ \\sqrt{2-x} \\geq \\frac{y+z}{2} $$ We first prove the easy part. Claim - We have $x \\leq 2$. $$ \\begin{aligned} 5=x^{4}+y^{4}+\\left(z^{4}+1\\right)+x y z & =\\frac{3 x^{4}}{4}+\\left(\\frac{x^{4}}{4}+y^{4}\\right)+\\left(z^{4}+1\\right)+x y z \\\\ & \\geq \\frac{3 x^{4}}{4}+x^{2} y^{2}+2 z^{2}+x y z . \\end{aligned} $$ We evidently have that $x^{2} y^{2}+2 z^{2}+x y z \\geq 0$ because the quadratic form $a^{2}+a b+2 b^{2}$ is positive definite, so $x^{4} \\leq \\frac{20}{3} \\Longrightarrow x \\leq 2$. Now, the desired statement is implied by its square, so it suffices to show that $$ 2-x \\geq\\left(\\frac{y+z}{2}\\right)^{2} $$ We are going to proceed by contradiction (it seems that many solutions do this) and assume that $$ 2-x<\\left(\\frac{y+z}{2}\\right)^{2} \\Longleftrightarrow 4 x+y^{2}+2 y z+z^{2}>8 $$ By AM-GM, $$ \\begin{aligned} x^{4}+3 & \\geq 4 x \\\\ \\frac{y^{4}+1}{2} & \\geq y^{2} \\\\ \\frac{z^{4}+1}{2} & \\geq z^{2} \\end{aligned} $$ which yields that $$ x^{4}+\\frac{y^{4}+z^{4}}{2}+2 y z+4>8 $$ If we replace $x^{4}=4-\\left(y^{4}+z^{4}+x y z\\right)$ now, this gives $$ -\\frac{y^{4}+z^{4}}{2}+(2-x) y z>0 \\Longrightarrow(2-x) y z>\\frac{y^{4}+z^{4}}{2} $$ Since $2-x$ and the right-hand side are positive, we have $y z \\geq 0$. Now $$ \\frac{y^{4}+z^{4}}{2 y z}<2-x<\\left(\\frac{y+z}{2}\\right)^{2} \\Longrightarrow 2 y^{4}+2 z^{4}2015$ we're done."} +{"year":2015,"label":"5","problem":"Let $\\varphi(n)$ denote the number of positive integers less than $n$ that are relatively prime to $n$. Prove that there exists a positive integer $m$ for which the equation $\\varphi(n)=m$ has at least 2015 solutions in $n$.","solution":" \u3010 Second solution with smallest primes, by Yang Liu. Let $2=p_{1}1$. Then Alice can complete the phase without losing if and only if $x$ is even; if so she begins a $Y$-phase with $(X, Y)=(0, x \/ 2)$. Now note that unless $X=0$, Bob now has a winning move WrongX. Conversely he may only play Sleep if $X=0$. We have an analogous claim for $Y$-phases. Thus if $n$ is not a power of 2 , we see that Alice eventually loses. Now suppose $n=2^{k}$; then Alice reaches $(X, Y)=\\left(0,2^{k-1}\\right),\\left(2^{k-2}, 0\\right), \\ldots$ until either reaching $(1,0)$ or $(0,1)$. At this point she can play ClaimX or ClaimY, respectively; the game is now in state Cl. Bob cannot play either FakeX or FakeY, so he must play Sleep, and then Alice wins by playing Win. Thus Alice has a winning strategy when $n=2^{k}$."} diff --git a/USA_TSTST/segmented/en-sols-TSTST-2016.jsonl b/USA_TSTST/segmented/en-sols-TSTST-2016.jsonl new file mode 100644 index 0000000000000000000000000000000000000000..e59409cd04ec0f6097fdd5e9646ca673e76fa545 --- /dev/null +++ b/USA_TSTST/segmented/en-sols-TSTST-2016.jsonl @@ -0,0 +1,8 @@ +{"year":2016,"label":"1","problem":"Let $A=A(x, y)$ and $B=B(x, y)$ be two-variable polynomials with real coefficients. Suppose that $A(x, y) \/ B(x, y)$ is a polynomial in $x$ for infinitely many values of $y$, and a polynomial in $y$ for infinitely many values of $x$. Prove that $B$ divides $A$, meaning there exists a third polynomial $C$ with real coefficients such that $A=B \\cdot C$.","solution":" This is essentially an application of the division algorithm, but the details require significant care. First, we claim that $A \/ B$ can be written as a polynomial in $x$ whose coefficients are rational functions in $y$. To see this, use the division algorithm to get $$ A=Q \\cdot B+R \\quad Q, R \\in(\\mathbb{R}(y))[x] $$ where $Q$ and $R$ are polynomials in $x$ whose coefficients are rational functions in $y$, and moreover $\\operatorname{deg}_{x} B>\\operatorname{deg}_{x} R$. Now, we claim that $R \\equiv 0$. Indeed, we have by hypothesis that for infinitely many values of $y_{0}$ that $B\\left(x, y_{0}\\right)$ divides $A\\left(x, y_{0}\\right)$, which means $B\\left(x, y_{0}\\right) \\mid R\\left(x, y_{0}\\right)$ as polynomials in $\\mathbb{R}[x]$. Now, we have $\\operatorname{deg}_{x} B\\left(x, y_{0}\\right)>\\operatorname{deg}_{x} R\\left(x, y_{0}\\right)$ outside of finitely many values of $y_{0}$ (but not all of them!); this means for infinitely many $y_{0}$ we have $R\\left(x, y_{0}\\right) \\equiv 0$. So each coefficient of $x^{i}$ (in $\\left.\\mathbb{R}(y)\\right)$ has infinitely many roots, hence is a zero polynomial. Consequently, we are able to write $A \/ B=F(x, y) \/ M(y)$ where $F \\in \\mathbb{R}[x, y]$ and $M \\in \\mathbb{R}[y]$ are each polynomials. Repeating the same argument now gives $$ \\frac{A}{B}=\\frac{F(x, y)}{M(y)}=\\frac{G(x, y)}{N(x)} $$ Now, by unique factorization of polynomials in $\\mathbb{R}[x, y]$, we can discuss GCD's. So, we tacitly assume $\\operatorname{gcd}(F, M)=\\operatorname{gcd}(G, N)=(1)$. Also, we obviously have $\\operatorname{gcd}(M, N)=(1)$. But $F \\cdot N=G \\cdot M$, so $M \\mid F \\cdot N$, thus we conclude $M$ is the constant polynomial. This implies the result. Remark. This fact does not generalize to arbitrary functions that are separately polynomial: see e.g. http:\/\/aops.com\/community\/c6h523650p2978180."} +{"year":2016,"label":"2","problem":"Let $A B C$ be a scalene triangle with orthocenter $H$ and circumcenter $O$ and denote by $M, N$ the midpoints of $\\overline{A H}, \\overline{B C}$. Suppose the circle $\\gamma$ with diameter $\\overline{A H}$ meets the circumcircle of $A B C$ at $G \\neq A$, and meets line $\\overline{A N}$ at $Q \\neq A$. The tangent to $\\gamma$ at $G$ meets line $O M$ at $P$. Show that the circumcircles of $\\triangle G N Q$ and $\\triangle M B C$ intersect on $\\overline{P N}$.","solution":" \u3010 First solution (found by contestants). Denote by $\\triangle D E F$ the orthic triangle. Observe $\\overline{P A}$ and $\\overline{P G}$ are tangents to $\\gamma$, since $\\overline{O M}$ is the perpendicular bisector of $\\overline{A G}$. Also note that $\\overline{A G}, \\overline{E F}, \\overline{B C}$ are concurrent at some point $R$ by radical axis on $(A B C), \\gamma$, (BFEC). Now, consider circles (PAGM), (MFDNE), and (MBC). We already saw the point $R$ satisfies $$ R A \\cdot R G=R E \\cdot R F=R B \\cdot R C $$ and hence has equal powers to all three circles; but since the circles at $M$ already, they must actually be coaxial. Assume they meet again at $T \\in \\overline{R M}$, say. Then $\\angle P T M$ and $\\angle M T N$ are both right angles, hence $T$ lies on $\\overline{P N}$. Finally $H$ is the orthocenter of $\\triangle A R N$, and thus the circle with diameter $\\overline{R N}$ passes through $G, Q, N$. ![](https:\/\/cdn.mathpix.com\/cropped\/2024_11_19_f48de8bcaa95c5ff4531g-04.jpg?height=969&width=1117&top_left_y=1591&top_left_x=475)"} +{"year":2016,"label":"2","problem":"Let $A B C$ be a scalene triangle with orthocenter $H$ and circumcenter $O$ and denote by $M, N$ the midpoints of $\\overline{A H}, \\overline{B C}$. Suppose the circle $\\gamma$ with diameter $\\overline{A H}$ meets the circumcircle of $A B C$ at $G \\neq A$, and meets line $\\overline{A N}$ at $Q \\neq A$. The tangent to $\\gamma$ at $G$ meets line $O M$ at $P$. Show that the circumcircles of $\\triangle G N Q$ and $\\triangle M B C$ intersect on $\\overline{P N}$.","solution":" \u300e Alternate solution (by proposer). Let $L$ be diametrically opposite $A$ on the circumcircle. Denote by $\\triangle D E F$ the orthic triangle. Let $X=\\overline{A H} \\cap \\overline{E F}$. Finally, let $T$ be the second intersection of (MFDNE) and (MBC). ![](https:\/\/cdn.mathpix.com\/cropped\/2024_11_19_f48de8bcaa95c5ff4531g-05.jpg?height=838&width=1109&top_left_y=429&top_left_x=482) We begin with a few easy observations. First, points $H, G, N, L$ are collinear and $\\angle A G L=90^{\\circ}$. Also, $Q$ is the foot from $H$ to $\\overline{A N}$. Consequently, lines $A G, E F, H Q$, $B C, T M$ concur at a point $R$ (radical axis). Moreover, we already know $\\angle M T N=90^{\\circ}$. This implies $T$ lies on the circle with diameter $\\overline{R N}$, which is exactly the circumcircle of $\\triangle G Q N$. Note by Brokard's Theorem on $A F H E$, the point $X$ is the orthocenter of $\\triangle M B C$. But $\\angle M T N=90^{\\circ}$ already, and $N$ is the midpoint of $\\overline{B C}$. Consequently, points $T, X$, $N$ are collinear. Finally, we claim $P, X, N$ are collinear, which solves the problem. Note $P=\\overline{G G} \\cap \\overline{A A}$. Set $K=\\overline{H N L} \\cap \\overline{A P}$. Then by noting $$ -1=(D, X ; A, H) \\stackrel{N}{=}(\\infty, \\overline{N X} \\cap \\overline{A K} ; A, K) $$ we see that $\\overline{N X}$ bisects segment $\\overline{A K}$, as desired. (A more projective finish is to show that $\\overline{P X N}$ is the polar of $R$ to $\\gamma$ ). Remark. The original problem proposal reads as follows: Let $A B C$ be a triangle with orthocenter $H$ and circumcenter $O$ and denote by $M, N$ the midpoints of $\\overline{A H}, \\overline{B C}$. Suppose ray $O M$ meets the line parallel to $\\overline{B C}$ through $A$ at $P$. Prove that the line through the circumcenter of $\\triangle M B C$ and the midpoint of $\\overline{O H}$ is parallel to $\\overline{N P}$. The points $G$ and $Q$ were added to the picture later to prevent the problem from being immediate by coordinates."} +{"year":2016,"label":"3","problem":"Decide whether or not there exists a nonconstant polynomial $Q(x)$ with integer coefficients with the following property: for every positive integer $n>2$, the numbers $$ Q(0), Q(1), Q(2), \\ldots, Q(n-1) $$ produce at most $0.499 n$ distinct residues when taken modulo $n$.","solution":" We claim that $$ Q(x)=420\\left(x^{2}-1\\right)^{2} $$ works. Clearly, it suffices to prove the result when $n=4$ and when $n$ is an odd prime $p$. The case $n=4$ is trivial, so assume now $n=p$ is an odd prime. First, we prove the following easy claim. Claim - For any odd prime $p$, there are at least $\\frac{1}{2}(p-3)$ values of $a$ for which $\\left(\\frac{1-a^{2}}{p}\\right)=+1$. Remark. The above identity comes from starting with the equation $1-a^{2}=b^{2}$, and writing it as $\\left(\\frac{1}{b}\\right)^{2}-\\left(\\frac{a}{b}\\right)^{2}=1$. Then solve $\\frac{1}{b}-\\frac{a}{b}=k$ and $\\frac{1}{b}+\\frac{a}{b}=1 \/ k$ for $a$. Let $F(x)=\\left(x^{2}-1\\right)^{2}$. The range of $F$ modulo $p$ is contained within the $\\frac{1}{2}(p+1)$ quadratic residues modulo $p$. On the other hand, if for some $t$ neither of $1 \\pm t$ is a quadratic residue, then $t^{2}$ is omitted from the range of $F$ as well. Call such a value of $t$ useful, and let $N$ be the number of useful residues. We aim to show $N \\geq \\frac{1}{4} p-2$. We compute a lower bound on the number $N$ of useful $t$ by writing $$ \\begin{aligned} N & =\\frac{1}{4}\\left(\\sum_{t}\\left[\\left(1-\\left(\\frac{1-t}{p}\\right)\\right)\\left(1-\\left(\\frac{1+t}{p}\\right)\\right)\\right]-\\left(1-\\left(\\frac{2}{p}\\right)\\right)-\\left(1-\\left(\\frac{-2}{p}\\right)\\right)\\right) \\\\ & \\geq \\frac{1}{4} \\sum_{t}\\left[\\left(1-\\left(\\frac{1-t}{p}\\right)\\right)\\left(1-\\left(\\frac{1+t}{p}\\right)\\right)\\right]-1 \\\\ & =\\frac{1}{4}\\left(p+\\sum_{t}\\left(\\frac{1-t^{2}}{p}\\right)\\right)-1 \\\\ & \\geq \\frac{1}{4}\\left(p+(+1) \\cdot \\frac{1}{2}(p-3)+0 \\cdot 2+(-1) \\cdot\\left((p-2)-\\frac{1}{2}(p-3)\\right)\\right)-1 \\\\ & \\geq \\frac{1}{4}(p-5) . \\end{aligned} $$ Thus, the range of $F$ has size at most $$ \\frac{1}{2}(p+1)-\\frac{1}{2} N \\leq \\frac{3}{8}(p+3) . $$ This is less than $0.499 p$ for any $p \\geq 11$. Remark. In fact, the computation above is essentially an equality. There are only two points where terms are dropped: one, when $p \\equiv 3(\\bmod 4)$ there are no $k^{2}=-1$ in the lemma, and secondly, the terms $1-(2 \/ p)$ and $1-(-2 \/ p)$ are dropped in the initial estimate for $N$. With suitable modifications, one can show that in fact, the range of $F$ is exactly equal to $$ \\frac{1}{2}(p+1)-\\frac{1}{2} N=\\left\\{\\begin{array}{lll} \\frac{1}{8}(3 p+5) & p \\equiv 1 & (\\bmod 8) \\\\ \\frac{1}{8}(3 p+7) & p \\equiv 3 & (\\bmod 8) \\\\ \\frac{1}{8}(3 p+9) & p \\equiv 5 & (\\bmod 8) \\\\ \\frac{1}{8}(3 p+3) & p \\equiv 7 & (\\bmod 8) \\end{array}\\right. $$"} +{"year":2016,"label":"4","problem":"Prove that if $n$ and $k$ are positive integers satisfying $\\varphi^{k}(n)=1$, then $n \\leq 3^{k}$. (Here $\\varphi^{k}$ denotes $k$ applications of the Euler phi function.)","solution":" The main observation is that the exponent of 2 decreases by at most 1 with each application of $\\varphi$. This will give us the desired estimate. Define the weight function $w$ on positive integers as follows: it satisfies $$ \\begin{aligned} w(a b) & =w(a)+w(b) \\\\ w(2) & =1 ; \\quad \\text { and } \\\\ w(p) & =w(p-1) \\quad \\text { for any prime } p>2 \\end{aligned} $$ By induction, we see that $w(n)$ counts the powers of 2 that are produced as $\\varphi$ is repeatedly applied to $n$. In particular, $k \\geq w(n)$. From $w(2)=1$, it suffices to prove that $w(p) \\geq \\log _{3} p$ for every $p>2$. We use strong induction and note that $$ w(p)=w(2)+w\\left(\\frac{p-1}{2}\\right) \\geq 1+\\log _{3}(p-1)-\\log _{3} 2 \\geq \\log _{3} p $$ for any $p>2$. This solves the problem. Moreover, the stronger bound $$ n \\leq 2 \\cdot 3^{k-1} $$ is true and best possible."} +{"year":2016,"label":"5","problem":"In the coordinate plane are finitely many walls, which are disjoint line segments, none of which are parallel to either axis. A bulldozer starts at an arbitrary point and moves in the $+x$ direction. Every time it hits a wall, it turns at a right angle to its path, away from the wall, and continues moving. (Thus the bulldozer always moves parallel to the axes.) Prove that it is impossible for the bulldozer to hit both sides of every wall.","solution":" We say a wall $v$ is above another wall $w$ if some point on $v$ is directly above a point on $w$. (This relation is anti-symmetric, as walls do not intersect). The critical claim is as follows: Claim - There exists a lowest wall, i.e. a wall not above any other walls. Now consider the leftmost vertical segment $\\overline{Q_{i} P_{i+1}}$ and the rightmost vertical segment $\\overline{Q_{j} P_{j+1}}$. The broken line gives a path from $P_{i+1}$ to $Q_{j}$, as well as a path from $P_{j+1}$ to $Q_{i}$. These clearly must intersect, contradiction. Remark. This claim is Iran TST 2010. Thus if the bulldozer eventually moves upwards indefinitely, it may never hit the bottom side of the lowest wall. Similarly, if the bulldozer eventually moves downwards indefinitely, it may never hit the upper side of the highest wall."} +{"year":2016,"label":"6","problem":"Let $A B C$ be a triangle with incenter $I$, and whose incircle is tangent to $\\overline{B C}, \\overline{C A}$, $\\overline{A B}$ at $D, E, F$, respectively. Let $K$ be the foot of the altitude from $D$ to $\\overline{E F}$. Suppose that the circumcircle of $\\triangle A I B$ meets the incircle at two distinct points $C_{1}$ and $C_{2}$, while the circumcircle of $\\triangle A I C$ meets the incircle at two distinct points $B_{1}$ and $B_{2}$. Prove that the radical axis of the circumcircles of $\\triangle B B_{1} B_{2}$ and $\\triangle C C_{1} C_{2}$ passes through the midpoint $M$ of $\\overline{D K}$.","solution":" \\I First solution (Allen Liu). Let $X, Y, Z$ be midpoints of $E F, F D, D E$, and let $G$ be the Gergonne point. By radical axis on $(A E I F),(D E F),(A I C)$ we see that $B_{1}$, $X, B_{2}$ are collinear. Likewise, $B_{1}, Z, B_{2}$ are collinear, so lines $B_{1} B_{2}$ and $X Z$ coincide. Similarly, lines $C_{1} C_{2}$ and $X Y$ coincide. In particular lines $B_{1} B_{2}$ and $C_{1} C_{2}$ meet at $X$. ![](https:\/\/cdn.mathpix.com\/cropped\/2024_11_19_f48de8bcaa95c5ff4531g-10.jpg?height=1212&width=1043&top_left_y=1150&top_left_x=512) Note $G$ is the symmedian point of $D E F$, so it is well-known that $X G$ passes through the midpoint of $D K$. So we just have to prove $G$ lies on the radical axis. First, note that $\\triangle D E F$ is the cevian triangle of the Gergonne point $G$. Set $V=$ $\\overline{X Y} \\cap \\overline{A B}, W=\\overline{X Z} \\cap \\overline{A C}$, and $T=\\overline{B W} \\cap \\overline{C V}$. We begin with the following completely projective claim. Claim - The points $X, G, T$ are collinear. - By Cevian Nest on $\\triangle A B C$, it follows that $\\overline{A X}, \\overline{B Y}, \\overline{C Z}$ are concurrent. - Hence $\\triangle B Y V$ and $\\triangle C Z W$ are perspective. - Hence $\\triangle B Z W$ and $\\triangle C Y V$ are perspective too. - Hence we deduce by Desargues theorem that $T, X$, and $\\overline{B Z} \\cap \\overline{C Y}$ are collinear. - Finally, the Cevian Nest theorem applied on $\\triangle G B C$ (which has cevian triangles $\\triangle D F E, \\triangle X Z Y$ ) we deduce $G, X$, and $\\overline{B Z} \\cap \\overline{C Y}$, proving the claim. One could also proceed by using barycentric coordinates on $\\triangle D E F$. Remark (Eric Shen). The first four bullets can be replaced by non-projective means: one can check that $\\overline{B Z} \\cap \\overline{C Y}$ is the radical center of $(B I C),\\left(B B_{1} B_{2}\\right),\\left(C C_{1} C_{2}\\right)$ and therefore it lies on line $\\overline{X T}$. Now, we contend point $V$ is the radical center $\\left(C C_{1} C_{2}\\right),(A B C)$ and $(D E F)$. To see this, let $V^{\\prime}=\\overline{E D} \\cap \\overline{A B}$; then $\\left(F V^{\\prime} ; A B\\right)$ is harmonic, and $V$ is the midpoint of $\\overline{F V^{\\prime}}$, and thus $V A \\cdot V B=V F^{2}=V C_{1} \\cdot V C_{2}$. So in fact $\\overline{C V}$ is the radical axis of $(A B C)$ and $\\left(C C_{1} C_{2}\\right)$. Similarly, $\\overline{B W}$ is the radical axis of $(A B C)$ and $\\left(B B_{1} B_{2}\\right)$. Thus $T$ is the radical center of $(A B C),\\left(B B_{1} B_{2}\\right),\\left(C C_{1} C_{2}\\right)$."} +{"year":2016,"label":"6","problem":"Let $A B C$ be a triangle with incenter $I$, and whose incircle is tangent to $\\overline{B C}, \\overline{C A}$, $\\overline{A B}$ at $D, E, F$, respectively. Let $K$ be the foot of the altitude from $D$ to $\\overline{E F}$. Suppose that the circumcircle of $\\triangle A I B$ meets the incircle at two distinct points $C_{1}$ and $C_{2}$, while the circumcircle of $\\triangle A I C$ meets the incircle at two distinct points $B_{1}$ and $B_{2}$. Prove that the radical axis of the circumcircles of $\\triangle B B_{1} B_{2}$ and $\\triangle C C_{1} C_{2}$ passes through the midpoint $M$ of $\\overline{D K}$.","solution":" \u300e Second solution (Evan Chen). As before, we just have to prove $G$ lies on the radical axis. ![](https:\/\/cdn.mathpix.com\/cropped\/2024_11_19_f48de8bcaa95c5ff4531g-12.jpg?height=1392&width=1260&top_left_y=249&top_left_x=404) Construct parallelograms $G P F Q, G R D S, G T U E$ such that $P, R \\in D F, S, T \\in D E$, $Q, U \\in E F$. As $F G$ bisects $P Q$ and is isogonal to $F Z$, we find $P Q E D$, hence $P Q R U$, is cyclic. Repeating the same logic and noticing $P R, S T, Q U$ not concurrent, all six points $P Q R S T U$ are cyclic. Moreover, since $P Q$ bisects $G F$, we see that a dilation with factor 2 at $G$ sends $P Q$ to $P^{\\prime}, Q^{\\prime} \\in A B$, say, with $F$ the midpoint of $P^{\\prime} Q^{\\prime}$. Define $R^{\\prime}, S^{\\prime} \\in B C$ similarly now and $T^{\\prime}, U^{\\prime} \\in C A$. Note that $E Q P D S^{\\prime}$ is in cyclic too, as $\\measuredangle D S^{\\prime} Q=\\measuredangle D R S=\\measuredangle D E F$. By homothety through $B$, points $B, P, X$ are collinear; assume they meet ( $\\left.E Q P D S^{\\prime}\\right)$ again at $V$. Thus $E V Q P D S^{\\prime}$ is cyclic, and now $$ \\measuredangle B V S^{\\prime}=\\measuredangle P V S^{\\prime}=\\measuredangle P Q S=\\measuredangle P T S=\\measuredangle F E D=\\measuredangle X E Z=\\measuredangle X V Z $$ hence $V$ lies on $\\left(B Q^{\\prime} S^{\\prime}\\right)$. Since $F B \\| Q P$, we get $E V F B$ is cyclic too, so $X V \\cdot X B=X E \\cdot X F$ now; thus $X$ lies on the radical axis of $\\left(B S^{\\prime} Q^{\\prime}\\right)$ and $(D E F)$. By the same argument with $W \\in B Z$, we get $Z$ lies on the radical axis too. Thus the radical axis of $\\left(B S^{\\prime} Q^{\\prime}\\right)$ and ( $D E F$ ) must be line $X Z$, which coincides with $B_{1} B_{2}$; so $\\left(B B_{1} B_{2}\\right)=\\left(B S^{\\prime} Q^{\\prime}\\right)$. Analogously, $\\left(C C_{1} C_{2}\\right)=\\left(C R^{\\prime} U^{\\prime}\\right)$. Since $G=Q^{\\prime} S^{\\prime} \\cap R^{\\prime} U^{\\prime}$, we need only prove that $Q^{\\prime} R^{\\prime} S^{\\prime} U^{\\prime}$ is cyclic. But $Q R S U$ is cyclic, so we are done. The circle ( $P Q R S T U$ ) is called the Lemoine circle of $A B C$."} diff --git a/USA_TSTST/segmented/en-sols-TSTST-2017.jsonl b/USA_TSTST/segmented/en-sols-TSTST-2017.jsonl new file mode 100644 index 0000000000000000000000000000000000000000..cb7a32b8a6ae423be65c1ec097ef9f37a1c5ef6b --- /dev/null +++ b/USA_TSTST/segmented/en-sols-TSTST-2017.jsonl @@ -0,0 +1,13 @@ +{"year":2017,"label":"1","problem":"Let $A B C$ be a triangle with circumcircle $\\Gamma$, circumcenter $O$, and orthocenter $H$. Assume that $A B \\neq A C$ and $\\angle A \\neq 90^{\\circ}$. Let $M$ and $N$ be the midpoints of $\\overline{A B}$ and $\\overline{A C}$, respectively, and let $E$ and $F$ be the feet of the altitudes from $B$ and $C$ in $\\triangle A B C$, respectively. Let $P$ be the intersection point of line $M N$ with the tangent line to $\\Gamma$ at $A$. Let $Q$ be the intersection point, other than $A$, of $\\Gamma$ with the circumcircle of $\\triangle A E F$. Let $R$ be the intersection point of lines $A Q$ and $E F$. Prove that $\\overline{P R} \\perp \\overline{O H}$.","solution":" \u3010 First solution (power of a point). Let $\\gamma$ denote the nine-point circle of $A B C$. ![](https:\/\/cdn.mathpix.com\/cropped\/2024_11_19_b5f71e11be2d66f7b920g-03.jpg?height=609&width=795&top_left_y=1140&top_left_x=639) Note that - $P A^{2}=P M \\cdot P N$, so $P$ lies on the radical axis of $\\Gamma$ and $\\gamma$. - $R A \\cdot R Q=R E \\cdot R F$, so $R$ lies on the radical axis of $\\Gamma$ and $\\gamma$. Thus $\\overline{P R}$ is the radical axis of $\\Gamma$ and $\\gamma$, which is evidently perpendicular to $\\overline{O H}$. Remark. In fact, by power of a point one may also observe that $R$ lies on $\\overline{B C}$, since it is on the radical axis of $(A Q F H E),(B F E C),(A B C)$. Ironically, this fact is not used in the solution."} +{"year":2017,"label":"1","problem":"Let $A B C$ be a triangle with circumcircle $\\Gamma$, circumcenter $O$, and orthocenter $H$. Assume that $A B \\neq A C$ and $\\angle A \\neq 90^{\\circ}$. Let $M$ and $N$ be the midpoints of $\\overline{A B}$ and $\\overline{A C}$, respectively, and let $E$ and $F$ be the feet of the altitudes from $B$ and $C$ in $\\triangle A B C$, respectively. Let $P$ be the intersection point of line $M N$ with the tangent line to $\\Gamma$ at $A$. Let $Q$ be the intersection point, other than $A$, of $\\Gamma$ with the circumcircle of $\\triangle A E F$. Let $R$ be the intersection point of lines $A Q$ and $E F$. Prove that $\\overline{P R} \\perp \\overline{O H}$.","solution":" II Second solution (barycentric coordinates). Again note first $R \\in \\overline{B C}$ (although this can be avoided too). We compute the points in much the same way as before. Since $\\overline{A P} \\cap \\overline{B C}=\\left(0: b^{2}:-c^{2}\\right)$ we have $$ P=\\left(b^{2}-c^{2}: b^{2}:-c^{2}\\right) $$ (since $x=y+z$ is the equation of line $\\overline{M N}$ ). Now in Conway notation we have $$ R=\\overline{E F} \\cap \\overline{B C}=\\left(0: S_{C}:-S_{B}\\right)=\\left(0: a^{2}+b^{2}-c^{2}:-a^{2}+b^{2}-c^{2}\\right) . $$ Hence $$ \\overrightarrow{P R}=\\frac{1}{2\\left(b^{2}-c^{2}\\right)}\\left(b^{2}-c^{2}, c^{2}-a^{2}, a^{2}-b^{2}\\right) $$ On the other hand, we have $\\overrightarrow{O H}=\\vec{A}+\\vec{B}+\\vec{C}$. So it suffices to check that $$ \\sum_{\\mathrm{cyc}} a^{2}\\left(\\left(a^{2}-b^{2}\\right)+\\left(c^{2}-a^{2}\\right)\\right)=0 $$ which is immediate."} +{"year":2017,"label":"1","problem":"Let $A B C$ be a triangle with circumcircle $\\Gamma$, circumcenter $O$, and orthocenter $H$. Assume that $A B \\neq A C$ and $\\angle A \\neq 90^{\\circ}$. Let $M$ and $N$ be the midpoints of $\\overline{A B}$ and $\\overline{A C}$, respectively, and let $E$ and $F$ be the feet of the altitudes from $B$ and $C$ in $\\triangle A B C$, respectively. Let $P$ be the intersection point of line $M N$ with the tangent line to $\\Gamma$ at $A$. Let $Q$ be the intersection point, other than $A$, of $\\Gamma$ with the circumcircle of $\\triangle A E F$. Let $R$ be the intersection point of lines $A Q$ and $E F$. Prove that $\\overline{P R} \\perp \\overline{O H}$.","solution":" \u300e Third solution (complex numbers). Let $A B C$ be the unit circle. We first compute $P$ as the midpoint of $A$ and $\\overline{A A} \\cap \\overline{B C}$ : $$ \\begin{aligned} p & =\\frac{1}{2}\\left(a+\\frac{a^{2}(b+c)-b c \\cdot 2 a}{a^{2}-b c}\\right) \\\\ & =\\frac{a\\left(a^{2}-b c\\right)+a^{2}(b+c)-2 a b c}{2\\left(a^{2}-b c\\right)} \\end{aligned} $$ Using the remark above, $R$ is the inverse of $D$ with respect to the circle with diameter $\\overline{B C}$, which has radius $\\left|\\frac{1}{2}(b-c)\\right|$. Thus $$ \\begin{aligned} r-\\frac{b+c}{2} & =\\frac{\\frac{1}{4}(b-c)\\left(\\frac{1}{b}-\\frac{1}{c}\\right)}{\\frac{1}{2}\\left(a-\\frac{b c}{a}\\right)} \\\\ r & =\\frac{b+c}{2}+\\frac{-\\frac{1}{2} \\frac{(b-c)^{2}}{b c}}{\\frac{1}{a}-\\frac{a}{b c}} \\\\ & =\\frac{b+c}{2}+\\frac{a(b-c)^{2}}{2\\left(a^{2}-b c\\right)} \\\\ & =\\frac{a(b-c)^{2}+(b+c)\\left(a^{2}-b c\\right)}{2\\left(a^{2}-b c\\right)} \\end{aligned} $$ Expanding and subtracting gives $$ p-r=\\frac{a^{3}-a b c-a b^{2}-a c^{2}+b^{2} c+b c^{2}}{2\\left(a^{2}-b c\\right)}=\\frac{(a+b+c)(a-b)(a-c)}{2\\left(a^{2}-b c\\right)} $$ which is visibly equal to the negation of its conjugate once the factor of $a+b+c$ is deleted. (Actually, one can guess this factorization ahead of time by noting that if $A=B$, then $P=B=R$, so $a-b$ must be a factor; analogously $a-c$ must be as well.)"} +{"year":2017,"label":"2","problem":"Ana and Banana are playing a game. First Ana picks a word, which is defined to be a nonempty sequence of capital English letters. Then Banana picks a nonnegative integer $k$ and challenges Ana to supply a word with exactly $k$ subsequences which are equal to Ana's word. Ana wins if she is able to supply such a word, otherwise she loses. For example, if Ana picks the word \"TST\", and Banana chooses $k=4$, then Ana can supply the word \"TSTST\" which has 4 subsequences which are equal to Ana's word. Which words can Ana pick so that she can win no matter what value of $k$ Banana chooses?","solution":" First we introduce some notation. Define a block of letters to be a maximal contiguous subsequence of consecutive letters. For example, the word $A A B B B C A A A$ has four blocks, namely $A A, B B B, C, A A A$. Throughout the solution, we fix the word $A$ that Ana picks, and introduce the following notation for its $m$ blocks: $$ A=A_{1} A_{2} \\ldots A_{m}=\\underbrace{a_{1} \\ldots a_{1}}_{x_{1}} \\underbrace{a_{2} \\ldots a_{2}}_{x_{2}} \\cdots \\underbrace{a_{m} \\ldots a_{m}}_{x_{m}} $$ A rainbow will be a subsequence equal to Ana's initial word $A$ (meaning Ana seeks words with exactly $k$ rainbows). Finally, for brevity, let $A_{i}=\\underbrace{a_{i} \\ldots a_{i}}_{x_{i}}$, so $A=A_{1} \\ldots A_{m}$. We prove two claims that resolve the problem. Claim - If $x_{i}=1$ for some $i$, then for any $k \\geq 1$, the word $$ W=A_{1} \\ldots A_{i-1} \\underbrace{a_{i} \\ldots a_{i}}_{k} A_{i+1} \\ldots A_{m} $$ obtained by repeating the $i$ th letter $k$ times has exactly $k$ rainbows. Given a rainbow, consider the location of this singleton block in $W$. It cannot occur within the first $\\left|A_{1}\\right|+\\cdots+\\left|A_{i-1}\\right|$ letters, nor can it occur within the final $\\left|A_{i+1}\\right|+\\cdots+\\left|A_{m}\\right|$ letters. So it must appear in the $i$ th block of $W$. That implies that all the other $a_{i}$ 's in the $i$ th block of $W$ must be deleted, as desired. (This last argument is actually nontrivial, and has some substance; many students failed to realize that the upper bound requires care.) Claim - If $x_{i} \\geq 2$ for all $i$, then no word $W$ has exactly two rainbows. Let $W=w_{1} \\ldots w_{n}$ and consider the two rainbows of $W$. Since they are not the same, there must be a block $A_{p}$ of the rainbow, of length $\\ell \\geq 2$, which do not occupy the same locations in $W$. Assume the first rainbow uses $w_{i_{1}}, \\ldots, w_{i_{\\ell}}$ for this block and the second rainbow uses $w_{j_{1}}, \\ldots, w_{j_{\\ell}}$ for this block. Then among the letters $w_{q}$ for $\\min \\left(i_{1}, j_{1}\\right) \\leq q \\leq \\max \\left(i_{\\ell}, j_{\\ell}\\right)$, there must be at least $\\ell+1$ copies of the letter $a_{p}$. Moreover, given a choice of $\\ell$ copies of the letter $a_{p}$ in this range, one can complete the subsequence to a rainbow. So the number of rainbows is at least $\\binom{\\ell+1}{\\ell} \\geq \\ell+1$. Since $\\ell \\geq 2$, this proves $W$ has at least three rainbows. In summary, Ana wins if and only if $x_{i}=1$ for some $i$, since she can duplicate the isolated letter $k$ times; but if $x_{i} \\geq 2$ for all $i$ then Banana only needs to supply $k=2$."} +{"year":2017,"label":"3","problem":"Consider solutions to the equation $$ x^{2}-c x+1=\\frac{f(x)}{g(x)} $$ where $f$ and $g$ are nonzero polynomials with nonnegative real coefficients. For each $c>0$, determine the minimum possible degree of $f$, or show that no such $f, g$ exist.","solution":" Indeed, one simply takes $x=1$ to get $f(1) \/ g(1) \\leq 0$, impossible. For $c<2$, let $c=2 \\cos \\theta$, where $0<\\theta<\\pi$. We claim that $f$ exists and has minimum degree equal to $n$, where $n$ is defined as the smallest integer satisfying $\\sin n \\theta \\leq 0$. In other words $$ n=\\left\\lceil\\frac{\\pi}{\\arccos (c \/ 2)}\\right\\rceil $$ First we show that this is necessary. To see it, write explicitly $$ g(x)=a_{0}+a_{1} x+a_{2} x^{2}+\\cdots+a_{n-2} x^{n-2} $$ with each $a_{i} \\geq 0$, and $a_{n-2} \\neq 0$. Assume that $n$ is such that $\\sin (k \\theta) \\geq 0$ for $k=1, \\ldots, n-1$. Then, we have the following system of inequalities: $$ \\begin{aligned} & a_{1} \\geq 2 \\cos \\theta \\cdot a_{0} \\\\ & a_{0}+a_{2} \\geq 2 \\cos \\theta \\cdot a_{1} \\\\ & a_{1}+a_{3} \\geq 2 \\cos \\theta \\cdot a_{2} \\\\ & \\vdots \\\\ & a_{n-5}+a_{n-3} \\geq 2 \\cos \\theta \\cdot a_{n-4} \\\\ & a_{n-4}+a_{n-2} \\geq 2 \\cos \\theta \\cdot a_{n-3} \\\\ & a_{n-3} \\geq 2 \\cos \\theta \\cdot a_{n-2} . \\end{aligned} $$ Now, multiply the first equation by $\\sin \\theta$, the second equation by $\\sin 2 \\theta$, et cetera, up to $\\sin ((n-1) \\theta)$. This choice of weights is selected since we have $$ \\sin (k \\theta)+\\sin ((k+2) \\theta)=2 \\sin ((k+1) \\theta) \\cos \\theta $$ so that summing the entire expression cancels nearly all terms and leaves only $$ \\sin ((n-2) \\theta) a_{n-2} \\geq \\sin ((n-1) \\theta) \\cdot 2 \\cos \\theta \\cdot a_{n-2} $$ and so by dividing by $a_{n-2}$ and using the same identity gives us $\\sin (n \\theta) \\leq 0$, as claimed. This bound is best possible, because the example $$ a_{k}=\\sin ((k+1) \\theta) \\geq 0 $$ makes all inequalities above sharp, hence giving a working pair $(f, g)$. Remark. Calvin Deng points out that a cleaner proof of the lower bound is to take $\\alpha=\\cos \\theta+i \\sin \\theta$. Then $f(\\alpha)=0$, but by condition the imaginary part of $f(\\alpha)$ is apparently strictly positive, contradiction. Remark. Guessing that $c<2$ works at all (and realizing $c \\geq 2$ fails) is the first part of the problem. - Calvin Deng points out that it's possible to guess the answer from small cases: For $c \\leq 1$ we have $n=3$, tight at $\\frac{x^{3}+1}{x+1}=x^{2}-x+1$, and essentially the \"sharpest $n=3$ example\". A similar example exists at $n=4$ with $\\frac{x^{4}+1}{x^{2}+\\sqrt{2} x+1}=x^{2}-\\sqrt{2} x+1$ by the Sophie-Germain identity. In general, one can do long division to extract an optimal value of $c$ for any given $n$, although $c$ will be the root of some polynomial. The thresholds $c \\leq 1$ for $n=3, c \\leq \\sqrt{2}$ for $n=4, c \\leq \\frac{1+\\sqrt{5}}{2}$ for $n=5$, and $c \\leq 2$ for $n<\\infty$ suggest the unusual form of the answer via trigonometry. - One may imagine trying to construct a polynomial recursively \/ greedily by making all inequalities above hold (again the \"sharpest situation\" in which $f$ has few coefficients). If one sets $c=2 t$, then we have $$ a_{0}=1, \\quad a_{1}=2 t, \\quad a_{2}=4 t^{2}-1, \\quad a_{3}=8 t^{3}-4 t, \\quad \\ldots $$ which are the Chebyshev polynomials of the second type. This means that trigonometry is essentially mandatory. (One may also run into this when by using standard linear recursion techniques, and noting that the characteristic polynomial has two conjugate complex roots.) Remark. Mitchell Lee notes that an IMO longlist problem from 1997 shows that if $P(x)$ is any polynomial satisfying $P(x)>0$ for $x>0$, then $(x+1)^{n} P(x)$ has nonnegative coefficients for large enough $n$. This show that $f$ and $g$ at least exist for $c \\leq 2$, but provides no way of finding the best possible $\\operatorname{deg} f$. Meghal Gupta also points out that showing $f$ and $g$ exist is possible in the following way: $$ \\left(x^{2}-1.99 x+1\\right)\\left(x^{2}+1.99 x+1\\right)=\\left(x^{4}-1.9601 x^{2}+1\\right) $$ and so on, repeatedly multiplying by the \"conjugate\" until all coefficients become positive. To my best knowledge, this also does not give any way of actually minimizing $\\operatorname{deg} f$, although Ankan Bhattacharya points out that this construction is actually optimal in the case where $n$ is a power of 2 . Remark. It's pointed out that Matematicheskoe Prosveshchenie, issue 1, 1997, page 194 contains a nearly analogous result, available at https:\/\/mccme.ru\/free-books\/matpros\/ pdf\/mp-01.pdf with solutions presented in https:\/\/mccme.ru\/free-books\/matpros\/pdf\/ mp-05.pdf, pages 221-223; and https:\/\/mccme.ru\/free-books\/matpros\/pdf\/mp-10.pdf, page 274."} +{"year":2017,"label":"4","problem":"Find all nonnegative integer solutions to $$ 2^{a}+3^{b}+5^{c}=n! $$","solution":"$$ 2^{a}+3^{b}+5^{c}=n! $$ $$ \\begin{aligned} & 2^{2}+3^{0}+5^{0}=3! \\\\ & 2^{1}+3^{1}+5^{0}=3! \\\\ & 2^{4}+3^{1}+5^{1}=4! \\end{aligned} $$ A tricky way to do this is to take modulo 120 , since $$ \\begin{aligned} 2^{a} \\quad(\\bmod 120) & \\in\\{1,2,4,8,16,32,64\\} \\\\ 3^{b} \\quad(\\bmod 120) & \\in\\{1,3,9,27,81\\} \\\\ 5^{c} \\quad(\\bmod 120) & \\in\\{1,5,25\\} \\end{aligned} $$ and by inspection one notes that no three elements have vanishing sum modulo 120 . - $a=1$ : since $3^{b}+5^{c} \\equiv 6(\\bmod 8)$, we find $b$ even and $c$ odd (hence $\\left.c \\neq 0\\right)$. Now looking modulo 5 gives that $3^{b}+5^{c} \\equiv 3(\\bmod 5)$, - $a=2$ : From $3^{b}+5^{c} \\equiv 4(\\bmod 8)$, we find $b$ is odd and $c$ is even. Now looking modulo 5 gives a contradiction, even if $c=0$, since $3^{b} \\in\\{2,3(\\bmod 5)\\}$ but $3^{b}+5^{c} \\equiv 1(\\bmod 5)$. Henceforth assume $a \\geq 3$. Next, by taking modulo 8 we have $3^{b}+5^{c} \\equiv 0(\\bmod 8)$, which forces both $b$ and $c$ to be odd (in particular, $b, c>0$ ). We now have $$ \\begin{aligned} & 2^{a}+5^{c} \\equiv 0 \\quad(\\bmod 3) \\\\ & 2^{a}+3^{b} \\equiv 0 \\quad(\\bmod 5) . \\end{aligned} $$ The first equation implies $a$ is even, but the second equation requires $a$ to be odd, contradiction. Hence no solutions with $n \\geq 5$."} +{"year":2017,"label":"5","problem":"Let $A B C$ be a triangle with incenter $I$. Let $D$ be a point on side $B C$ and let $\\omega_{B}$ and $\\omega_{C}$ be the incircles of $\\triangle A B D$ and $\\triangle A C D$, respectively. Suppose that $\\omega_{B}$ and $\\omega_{C}$ are tangent to segment $B C$ at points $E$ and $F$, respectively. Let $P$ be the intersection of segment $A D$ with the line joining the centers of $\\omega_{B}$ and $\\omega_{C}$. Let $X$ be the intersection point of lines $B I$ and $C P$ and let $Y$ be the intersection point of lines $C I$ and $B P$. Prove that lines $E X$ and $F Y$ meet on the incircle of $\\triangle A B C$.","solution":" \u300e First solution (homothety). Let $Z$ be the diametrically opposite point on the incircle. We claim this is the desired intersection. ![](https:\/\/cdn.mathpix.com\/cropped\/2024_11_19_b5f71e11be2d66f7b920g-10.jpg?height=586&width=809&top_left_y=1055&top_left_x=629) Note that: - $P$ is the insimilicenter of $\\omega_{B}$ and $\\omega_{C}$ - $C$ is the exsimilicenter of $\\omega$ and $\\omega_{C}$. Thus by Monge theorem, the insimilicenter of $\\omega_{B}$ and $\\omega$ lies on line $C P$. This insimilicenter should also lie on the line joining the centers of $\\omega$ and $\\omega_{B}$, which is $\\overline{B I}$, hence it coincides with the point $X$. So $X \\in \\overline{E Z}$ as desired."} +{"year":2017,"label":"5","problem":"Let $A B C$ be a triangle with incenter $I$. Let $D$ be a point on side $B C$ and let $\\omega_{B}$ and $\\omega_{C}$ be the incircles of $\\triangle A B D$ and $\\triangle A C D$, respectively. Suppose that $\\omega_{B}$ and $\\omega_{C}$ are tangent to segment $B C$ at points $E$ and $F$, respectively. Let $P$ be the intersection of segment $A D$ with the line joining the centers of $\\omega_{B}$ and $\\omega_{C}$. Let $X$ be the intersection point of lines $B I$ and $C P$ and let $Y$ be the intersection point of lines $C I$ and $B P$. Prove that lines $E X$ and $F Y$ meet on the incircle of $\\triangle A B C$.","solution":" \u3010 Second solution (harmonic). Let $T=\\overline{I_{B} I_{C}} \\cap \\overline{B C}$, and $W$ the foot from $I$ to $\\overline{B C}$. Define $Z=\\overline{F Y} \\cap \\overline{I W}$. Because $\\angle I_{B} D I_{C}=90^{\\circ}$, we have $$ -1=\\left(I_{B} I_{C} ; P T\\right) \\stackrel{B}{\\stackrel{B}{2}}\\left(I I_{C} ; Y C\\right) \\stackrel{F}{=}(I \\infty ; Z W) $$ So $I$ is the midpoint of $\\overline{Z W}$ as desired."} +{"year":2017,"label":"5","problem":"Let $A B C$ be a triangle with incenter $I$. Let $D$ be a point on side $B C$ and let $\\omega_{B}$ and $\\omega_{C}$ be the incircles of $\\triangle A B D$ and $\\triangle A C D$, respectively. Suppose that $\\omega_{B}$ and $\\omega_{C}$ are tangent to segment $B C$ at points $E$ and $F$, respectively. Let $P$ be the intersection of segment $A D$ with the line joining the centers of $\\omega_{B}$ and $\\omega_{C}$. Let $X$ be the intersection point of lines $B I$ and $C P$ and let $Y$ be the intersection point of lines $C I$ and $B P$. Prove that lines $E X$ and $F Y$ meet on the incircle of $\\triangle A B C$.","solution":" \u3010 Third solution (outline, barycentric, Andrew Gu). Let $A D=t, B D=x, C D=y$, so $a=x+y$ and by Stewart's theorem we have $$ (x+y)\\left(x y+t^{2}\\right)=b^{2} x+c^{2} y $$ We then have $D=(0: y: x)$ and so $$ \\overline{A I_{B}} \\cap \\overline{B C}=\\left(0: y+\\frac{t x}{c+t}: \\frac{c x}{c+t}\\right) $$ hence intersection with $B I$ gives $$ I_{B}=(a x: c y+a t: c x) $$ Similarly, $$ I_{C}=(a y: b y: b x+a t) $$ Then, we can compute $$ P=(2 a x y: y(a t+b x+c y): x(a t+b x+c y)) $$ since $P \\in \\overline{I_{B} I_{C}}$, and clearly $P \\in \\overline{A D}$. Intersection now gives $$ \\begin{aligned} & X=(2 a x: a t+b x+c y: 2 c x) \\\\ & Y=(2 a y: 2 b y: a t+b x+c y) \\end{aligned} $$ Finally, we have $B E=\\frac{1}{2}(c+x-t)$, and similarly for $C F$. Now if we reflect $D=$ $\\left(0, \\frac{s-c}{a}, \\frac{s-b}{a}\\right)$ over $I=\\left(\\frac{a}{2 s}, \\frac{b}{2 s}, \\frac{c}{2 s}\\right)$, we get the antipode $$ Q:=\\left(4 a^{2}:-a^{2}+2 a b-b^{2}+c^{2}:-a^{2}+2 a c-c^{2}+b^{2}\\right) . $$ We may then check $Q$ lies on each of lines $E X$ and $F Y$ (by checking $\\operatorname{det}(Q, E, X)=0$ using the equation (1))."} +{"year":2017,"label":"6","problem":"A sequence of positive integers $\\left(a_{n}\\right)_{n \\geq 1}$ is of Fibonacci type if it satisfies the recursive relation $a_{n+2}=a_{n+1}+a_{n}$ for all $n \\geq 1$. Is it possible to partition the set of positive integers into an infinite number of Fibonacci type sequences?","solution":" \u3010 First solution (Kevin Sun). We are going to appeal to the so-called Zeckendorf theorem: Theorem (Zeckendorf) Every positive integer can be uniquely expressed as the sum of nonconsecutive Fibonacci numbers. This means every positive integer has a Zeckendorf (\"Fibonacci-binary\") representation where we put 1 in the $i$ th digit from the right if $F_{i+1}$ is used. The idea is then to take the following so-called Wythoff array: - Row 1: 1, 2, 3, 5, ... - Row 101: $1+3,2+5,3+8, \\ldots$ - Row 1001: $1+5,2+8,3+13, \\ldots$ - Row 10001: $1+8,2+13,3+21, \\ldots$ - Row 10101: $1+3+8,2+5+13,3+8+21, \\ldots$ - . . .et cetera. More concretely, the array has the following rows to start: | 1 | 2 | 3 | 5 | 8 | 13 | 21 | $\\ldots$ | | :---: | :---: | :---: | :---: | :---: | :---: | :---: | :---: | | 4 | 7 | 11 | 18 | 29 | 47 | 76 | $\\ldots$ | | 6 | 10 | 16 | 26 | 42 | 68 | 110 | $\\ldots$ | | 9 | 15 | 24 | 39 | 63 | 102 | 165 | $\\ldots$ | | 12 | 20 | 32 | 52 | 84 | 136 | 220 | $\\ldots$ | | 14 | 23 | 37 | 60 | 97 | 157 | 254 | $\\ldots$ | | 17 | 28 | 45 | 73 | 118 | 191 | 309 | $\\ldots$ | | $\\vdots$ | $\\vdots$ | $\\vdots$ | $\\vdots$ | $\\vdots$ | $\\vdots$ | $\\vdots$ | $\\ddots$ | Here are the full details. $$ n-F_{k}F_{k-1}+F_{k-3}+F_{k-5}+\\cdots $$ This shows, by a simple inductive argument, that such a representation exists and unique. We write $n={\\bar{a} k \\ldots a_{1}}_{\\text {Fib }}$ for the Zeckendorf representation as we described (where $a_{i}=1$ if $F_{i+1}$ is used). Now for each ${\\bar{a} k \\ldots a_{1}}^{\\text {Fib }}$ with $a_{1}=1$, consider the sequence $$ {\\overline{a_{k} \\ldots a_{1}}}_{\\mathrm{Fib}},{\\overline{a_{k} \\ldots a_{1} 0}}_{\\mathrm{Fib}},{\\overline{a_{k} \\ldots a_{1} 00}}_{\\mathrm{Fib}}, \\ldots $$ These sequences are Fibonacci-type by definition, and partition the positive integers since each positive integer has exactly one Fibonacci base representation."} +{"year":2017,"label":"6","problem":"A sequence of positive integers $\\left(a_{n}\\right)_{n \\geq 1}$ is of Fibonacci type if it satisfies the recursive relation $a_{n+2}=a_{n+1}+a_{n}$ for all $n \\geq 1$. Is it possible to partition the set of positive integers into an infinite number of Fibonacci type sequences?","solution":" \\I Second solution. Call an infinite set of integers $S$ sandwiched if there exist increasing sequences $\\left\\{a_{i}\\right\\}_{i=0}^{\\infty},\\left\\{b_{i}\\right\\}_{i=0}^{\\infty}$ such that the following are true: - $a_{i}+a_{i+1}=a_{i+2}$ and $b_{i}+b_{i+1}=b_{i+2}$. - The intervals $\\left[a_{i}+1, b_{i}-1\\right]$ are disjoint and are nondecreasing in length. - $S=\\bigcup_{i=0}^{\\infty}\\left[a_{i}+1, b_{i}-1\\right]$. We claim that if $S$ is any nonempty sandwiched set, then $S$ can be partitioned into a Fibonacci-type sequence (involving the smallest element of $S$ ) and two smaller sandwiched sets. If this claim is proven, then we can start with $\\mathbb{N} \\backslash\\{1,2,3,5, \\ldots\\}$, which is a sandwiched set, and repeatedly perform this partition, which will eventually sort each natural number into a Fibonacci-type sequence. Let $S$ be a sandwiched set given by $\\left\\{a_{i}\\right\\}_{i=0}^{\\infty},\\left\\{b_{i}\\right\\}_{i=0}^{\\infty}$, so the smallest element in $S$ is $x=a_{0}+1$. Note that $y=a_{1}+1$ is also in $S$ and $x1$. Then $$ a \\neq b \\Longrightarrow\\left\\lfloor\\varphi a+\\frac{1}{2}\\right\\rfloor \\neq\\left\\lfloor\\varphi b+\\frac{1}{2}\\right\\rfloor, $$ and since the first term of each sequence is chosen to not overlap with any previous sequences, these sequences are disjoint. Remark. Ankan Bhattacharya points out that the same sequence essentially appears in IMO 1993, Problem 5 - in other words, a strictly increasing function $f: \\mathbb{Z}_{>0} \\rightarrow \\mathbb{Z}_{>0}$ with $f(1)=2$, and $f(f(n))=f(n)+n$. Nikolai Beluhov sent us an older reference from March 1977, where Martin Gardner wrote in his column about Wythoff's Nim. The relevant excerpt goes: \"Imagine that we go through the infinite sequence of safe pairs (in the manner of Eratosthenes' sieve for sifting out primes) and cross out the infinite set of all safe pairs that are pairs in the Fibonacci sequence. The smallest pair that is not crossed out is $4 \/ 7$. We can now cross out a second infinite set of safe pairs, starting with $4 \/ 7$, that are pairs in the Lucas sequence. An infinite number of safe pairs, of which the lowest is now $6 \/ 10$, remain. This pair too begins another infinite Fibonacci sequence, all of whose pairs are safe. The process continues forever. Robert Silber, a mathematician at North Carolina State University, calls a safe pair \"primitive\" if it is the first safe pair that generates a Fibonacci sequence.\" The relevant article by Robert Silber is A Fibonacci Property of Wythoff Pairs, from The Fibonacci Quarterly 11\/1976."} +{"year":2017,"label":"6","problem":"A sequence of positive integers $\\left(a_{n}\\right)_{n \\geq 1}$ is of Fibonacci type if it satisfies the recursive relation $a_{n+2}=a_{n+1}+a_{n}$ for all $n \\geq 1$. Is it possible to partition the set of positive integers into an infinite number of Fibonacci type sequences?","solution":" I Fourth solution (Mark Sellke). For later reference let $$ f_{1}=0, f_{2}=1, f_{3}=1, \\ldots $$ denote the ordinary Fibonacci numbers. We will denote the Fibonacci-like sequences by $F^{i}$ and the elements with subscripts; hence $F_{1}^{2}$ is the first element of the second sequence. Our construction amounts to just iteratively add new sequences; hence the following claim is the whole problem. ## Lemma For any disjoint collection of Fibonacci-like sequences $F^{1}, \\ldots, F^{k}$ and any integer $m$ contained in none of them, there is a new Fibonacci-like sequence $F^{k+1}$ beginning with $F_{1}^{k+1}=m$ which is disjoint from the previous sequences. Observe first that for each sequence $F^{j}$ there is $c^{j} \\in \\mathbb{R}^{n}$ such that $$ F_{n}^{j}=c^{j} \\phi^{n}+o(1) $$ where $$ \\phi=\\frac{1+\\sqrt{5}}{2} $$ Collapse the group $\\left(\\mathbb{R}^{+}, \\times\\right)$into the half-open interval $J=\\{x \\mid 1 \\leq x<\\phi\\}$ by defining $T(x)=y$ for the unique $y \\in J$ with $x=y \\phi^{n}$ for some integer $n$. Fix an interval $I=[a, b] \\subseteq[1.2,1.3]$ (the last condition is to avoid wrap-around issues) which contains none of the $c^{j}$, and take $\\varepsilon<0.001$ to be small enough that in fact each $c^{j}$ has distance at least $10 \\varepsilon$ from $I$; this means any $c_{j}$ and element of $I$ differ by at least a $(1+10 \\varepsilon)$ factor. The idea will be to take $F_{1}^{k+1}=m$ and $F_{2}^{k+1}$ to be a large such that the induced values of $F_{j}^{k+1}$ grow like $k \\phi^{j}$ for $j \\in T^{-1}(I)$, so that $F_{n}^{k+1}$ is separated from the $c^{j}$ after applying $T$. What's left to check is the convergence. Now let $$ c=\\lim _{n \\rightarrow \\infty} \\frac{f_{n}}{\\phi^{n}} $$ and take $M$ large enough that for $n>M$ we have $$ \\left|\\frac{f_{n}}{c \\phi^{n}}-1\\right|<\\varepsilon $$ Now $\\frac{T^{-1}(I)}{c}$ contains arbitrarily large integers, so there are infinitely many $N$ with $c N \\in T^{-1}(I)$ with $N>\\frac{10 m}{\\varepsilon}$. We claim that for any such $N$, the sequence $F^{(N)}$ defined by $$ F_{1}^{(N)}=m, F_{2}^{(N)}=N $$ will be very multiplicatively similar to the normal Fibonacci numbers up to rescaling; indeed for $j=2, j=3$ we have $\\frac{F_{2}^{(N)}}{f_{2}}=N, \\frac{F_{3}^{(N)}}{f_{3}}=N+m$ and so by induction we will have $$ \\frac{F_{j}^{(N)}}{f_{j}} \\in[N, N+m] \\subseteq[N, N(1+\\varepsilon)] $$ for $j \\geq 2$. Therefore, up to small multiplicative errors, we have $$ F_{j}^{(N)} \\approx N f_{j} \\approx c N \\phi^{j} $$ From this we see that for $j>M$ we have $$ T\\left(F_{j}^{(N)}\\right) \\in T(c N) \\cdot[1-2 \\varepsilon, 1+2 \\varepsilon] $$ In particular, since $T(c N) \\in I$ and $I$ is separated from each $c_{j}$ by a factor of $(1+10 \\varepsilon)$, we get that $F_{j}^{(N)}$ is not in any of $F^{1}, F^{2}, \\ldots, F^{k}$. Finishing is easy, since we now have a uniform estimate on how many terms we need to check for a new element before the exponential growth takes over. We will just use pigeonhole to argue that there are few possible collisions among those early terms, so we can easily pick a value of $N$ which avoids them all. We write it out below. For large $L$, the set $$ S_{L}=\\left(I \\cdot \\phi^{L}\\right) \\cap \\mathbb{Z} $$ contains at least $k_{I} \\phi^{L}$ elements. As $N$ ranges over $S_{L}$, for each fixed $j$, the value of $F_{j}^{(N)}$ varies by at most a factor of 1.1 because we imposed $I \\subseteq[1.2,1.3]$ and so this is true for the first two terms, hence for all subsequent terms by induction. Now suppose $L$ is very large, and consider a fixed pair $(i, j)$ with $i \\leq k$ and $j \\leq M$. We claim there is at most 1 possible value $k$ such that the term $F_{k}^{i}$ could equal $F_{j}^{(N)}$ for some $N \\in S_{L}$; indeed, the terms of $F^{i}$ are growing at exponential rate with factor $\\phi>1.1$, so at most one will be in a given interval of multiplicative width at most 1.1. Hence, of these $k_{I} \\phi^{L}$ values of $N$, at most $k M$ could cause problems, one for each pair $(i, j)$. However by monotonicity of $F_{j}^{(N)}$ in $N$, at most 1 value of $N$ causes a collision for each pair $(i, j)$. Hence for large $L$ so that $k_{I} \\phi^{L}>10 k M$ we can find a suitable $N \\in S_{L}$ by pigeonhole and the sequence $F^{(N)}$ defined by $(m, N, N+m, \\ldots)$ works."} diff --git a/USA_TSTST/segmented/en-sols-TSTST-2018.jsonl b/USA_TSTST/segmented/en-sols-TSTST-2018.jsonl new file mode 100644 index 0000000000000000000000000000000000000000..1f221ca585cfa144164a3fd39b40618f9e6e8ee1 --- /dev/null +++ b/USA_TSTST/segmented/en-sols-TSTST-2018.jsonl @@ -0,0 +1,14 @@ +{"year":2018,"label":"1","problem":"As usual, let $\\mathbb{Z}[x]$ denote the set of single-variable polynomials in $x$ with integer coefficients. Find all functions $\\theta: \\mathbb{Z}[x] \\rightarrow \\mathbb{Z}$ such that for any polynomials $p, q \\in$ $\\mathbb{Z}[x]$, - $\\theta(p+1)=\\theta(p)+1$, and - if $\\theta(p) \\neq 0$ then $\\theta(p)$ divides $\\theta(p \\cdot q)$.","solution":" The answer is $\\theta: p \\mapsto p(c)$, for each choice of $c \\in \\mathbb{Z}$. Obviously these work, so we prove these are the only ones. In what follows, $x \\in \\mathbb{Z}[x]$ is the identity polynomial, and $c=\\theta(x)$. \u3010 First solution (Merlijn Staps). Consider an integer $n \\neq c$. Because $x-n \\mid p(x)-p(n)$, we have $$ \\theta(x-n)|\\theta(p(x)-p(n)) \\Longrightarrow c-n| \\theta(p(x))-p(n) . $$ On the other hand, $c-n \\mid p(c)-p(n)$. Combining the previous two gives $c-n \\mid$ $\\theta(p(x))-p(c)$, and by letting $n$ large we conclude $\\theta(p(x))-p(c)=0$, so $\\theta(p(x))=p(c)$."} +{"year":2018,"label":"1","problem":"As usual, let $\\mathbb{Z}[x]$ denote the set of single-variable polynomials in $x$ with integer coefficients. Find all functions $\\theta: \\mathbb{Z}[x] \\rightarrow \\mathbb{Z}$ such that for any polynomials $p, q \\in$ $\\mathbb{Z}[x]$, - $\\theta(p+1)=\\theta(p)+1$, and - if $\\theta(p) \\neq 0$ then $\\theta(p)$ divides $\\theta(p \\cdot q)$.","solution":" The answer is $\\theta: p \\mapsto p(c)$, for each choice of $c \\in \\mathbb{Z}$. Obviously these work, so we prove these are the only ones. In what follows, $x \\in \\mathbb{Z}[x]$ is the identity polynomial, and $c=\\theta(x)$. II Second solution. First, we settle the case $\\operatorname{deg} p=0$. In that case, from the second property, $\\theta(m)=m+\\theta(0)$ for every integer $m \\in \\mathbb{Z}$ (viewed as a constant polynomial). Thus $m+\\theta(0) \\mid 2 m+\\theta(0)$, hence $m+\\theta(0) \\mid-\\theta(0)$, so $\\theta(0)=0$ by taking $m$ large. Thus $\\theta(m)=m$ for $m \\in \\mathbb{Z}$. Next, we address the case of $\\operatorname{deg} p=1$. We know $\\theta(x+b)=c+b$ for $b \\in \\mathbb{Z}$. Now for each particular $a \\in \\mathbb{Z}$, we have $$ c+k|\\theta(x+k)| \\theta(a x+a k)=\\theta(a x)+a k \\Longrightarrow c+k \\mid \\theta(a x)-a c . $$ for any $k \\neq-c$. Since this is true for large enough $k$, we conclude $\\theta(a x)=a c$. Thus $\\theta(a x+b)=a c+b$. We now proceed by induction on $\\operatorname{deg} p$. Fix a polynomial $p$ and assume it's true for all $p$ of smaller degree. Choose a large integer $n$ (to be determined later) for which $p(n) \\neq p(c)$. We then have $$ \\left.\\frac{p(c)-p(n)}{c-n}=\\theta\\left(\\frac{p-p(n)}{x-n}\\right) \\right\\rvert\\, \\theta(p-p(n))=\\theta(p)-p(n) $$ Subtracting off $c-n$ times the left-hand side gives $$ \\left.\\frac{p(c)-p(n)}{c-n} \\right\\rvert\\, \\theta(p)-p(c) $$ The left-hand side can be made arbitrarily large by letting $n \\rightarrow \\infty$, since $\\operatorname{deg} p \\geq 2$. Thus $\\theta(p)=p(c)$, concluding the proof. \\ Authorship comments. I will tell you a story about the creation of this problem. Yang Liu and I were looking over the drafts of December and January TST in October 2017, and both of us had the impression that the test was too difficult. This sparked a non-serious suggestion that we should try to come up with a problem now that would be easy enough to use. While we ended up just joking about changing the TST, we did get this problem out of it. Our idea was to come up with a functional equation that was different from the usual fare: at first we tried $\\mathbb{Z}[x] \\rightarrow \\mathbb{Z}[x]$, but then I suggested the idea of using $\\mathbb{Z}[x] \\rightarrow \\mathbb{Z}$, with the answer being the \"evaluation\" map. Well, what properties does that satisfy? One answer was $a-b \\mid p(a)-p(b)$; this didn't immediately lead to anything, but eventually we hit on the form of the problem above off this idea. At first we didn't require $\\theta(p) \\neq 0$ in the bullet, but without the condition the problem was too easy, since 0 divides only itself; and so the condition was added and we got the functional equation. I proposed the problem to USAMO 2018, but it was rejected (unsurprisingly; I think the problem may be too abstract for novice contestants). Instead it was used for TSTST, which I thought fit better."} +{"year":2018,"label":"2","problem":"In the nation of Onewaynia, certain pairs of cities are connected by one-way roads. Every road connects exactly two cities (roads are allowed to cross each other, e.g., via bridges), and each pair of cities has at most one road between them. Moreover, every city has exactly two roads leaving it and exactly two roads entering it. We wish to close half the roads of Onewaynia in such a way that every city has exactly one road leaving it and exactly one road entering it. Show that the number of ways to do so is a power of 2 greater than 1 (i.e. of the form $2^{n}$ for some integer $n \\geq 1$ ).","solution":" In the language of graph theory, we have a simple digraph $G$ which is 2 -regular and we seek the number of sub-digraphs which are 1-regular. We now present two solution paths. \u300e First solution, combinatorial. We construct a simple undirected bipartite graph $\\Gamma$ as follows: - the vertex set consists of two copies of $V(G)$, say $V_{\\text {out }}$ and $V_{\\text {in }}$; and - for $v \\in V_{\\text {out }}$ and $w \\in V_{\\text {in }}$ we have an undirected edge $v w \\in E(\\Gamma)$ if and only if the directed edge $v \\rightarrow w$ is in $G$. Moreover, the desired sub-digraphs of $H$ correspond exactly to perfect matchings of $\\Gamma$. However the graph $\\Gamma$ is 2 -regular and hence consists of several disjoint (simple) cycles of even length. If there are $n$ such cycles, the number of perfect matchings is $2^{n}$, as desired. Remark. The construction of $\\Gamma$ is not as magical as it may first seem. Suppose we pick a road $v_{1} \\rightarrow v_{2}$ to use. Then, the other road $v_{3} \\rightarrow v_{2}$ is certainly not used; hence some other road $v_{3} \\rightarrow v_{4}$ must be used, etc. We thus get a cycle of forced decisions until we eventually return to the vertex $v_{1}$. These cycles in the original graph $G$ (where the arrows alternate directions) correspond to the cycles we found in $\\Gamma$. It's merely that phrasing the solution in terms of $\\Gamma$ makes it cleaner in a linguistic sense, but not really in a mathematical sense."} +{"year":2018,"label":"2","problem":"In the nation of Onewaynia, certain pairs of cities are connected by one-way roads. Every road connects exactly two cities (roads are allowed to cross each other, e.g., via bridges), and each pair of cities has at most one road between them. Moreover, every city has exactly two roads leaving it and exactly two roads entering it. We wish to close half the roads of Onewaynia in such a way that every city has exactly one road leaving it and exactly one road entering it. Show that the number of ways to do so is a power of 2 greater than 1 (i.e. of the form $2^{n}$ for some integer $n \\geq 1$ ).","solution":" In the language of graph theory, we have a simple digraph $G$ which is 2 -regular and we seek the number of sub-digraphs which are 1-regular. We now present two solution paths. \u3010 Second solution by linear algebra over $\\mathbb{F}_{2}$ (Brian Lawrence). This is actually not that different from the first solution. For each edge $e$, we create an indicator variable $x_{e}$. We then require for each vertex $v$ that: - If $e_{1}$ and $e_{2}$ are the two edges leaving $v$, then we require $x_{e_{1}}+x_{e_{2}} \\equiv 1(\\bmod 2)$. - If $e_{3}$ and $e_{4}$ are the two edges entering $v$, then we require $x_{e_{3}}+x_{e_{4}} \\equiv 1(\\bmod 2)$. We thus get a large system of equations. Moreover, the solutions come in natural pairs $\\vec{x}$ and $\\vec{x}+\\overrightarrow{1}$ and therefore the number of solutions is either zero, or a power of two. So we just have to prove there is at least one solution. For linear algebra reasons, there can only be zero solutions if some nontrivial linear combination of the equations gives the sum $0 \\equiv 1$. So suppose we added up some subset $S$ of the equations for which every variable appeared on the left-hand side an even number of times. Then every variable that did appear appeared exactly twice; and accordingly we see that the edges corresponding to these variables form one or more even cycles as in the previous solution. Of course, this means $|S|$ is even, so we really have $0 \\equiv 0(\\bmod 2)$ as needed. Remark. The author's original proposal contained a second part asking to show that it was not always possible for the resulting $H$ to be connected, even if $G$ was strongly connected. This problem is related to IMO Shortlist 2002 C6, which gives an example of a strongly connected graph which does have a full directed Hamiltonian cycle."} +{"year":2018,"label":"3","problem":"Let $A B C$ be an acute triangle with incenter $I$, circumcenter $O$, and circumcircle $\\Gamma$. Let $M$ be the midpoint of $\\overline{A B}$. Ray $A I$ meets $\\overline{B C}$ at $D$. Denote by $\\omega$ and $\\gamma$ the circumcircles of $\\triangle B I C$ and $\\triangle B A D$, respectively. Line $M O$ meets $\\omega$ at $X$ and $Y$, while line $C O$ meets $\\omega$ at $C$ and $Q$. Assume that $Q$ lies inside $\\triangle A B C$ and $\\angle A Q M=\\angle A C B$. Consider the tangents to $\\omega$ at $X$ and $Y$ and the tangents to $\\gamma$ at $A$ and $D$. Given that $\\angle B A C \\neq 60^{\\circ}$, prove that these four lines are concurrent on $\\Gamma$.","solution":" Henceforth assume $\\angle A \\neq 60^{\\circ}$; we prove the concurrence. Let $L$ denote the center of $\\omega$, which is the midpoint of minor arc $B C$. Claim - Let $K$ be the point on $\\omega$ such that $\\overline{K L} \\| \\overline{A B}$ and $\\overline{K C} \\| \\overline{A L}$. Then $\\overline{K A}$ is tangent to $\\gamma$, and we may put $$ x=K A=L B=L C=L X=L Y=K X=K Y . $$ Moreover, $\\overline{K A}$ is tangent to $\\gamma$ as well since $$ \\measuredangle K A D=\\measuredangle K A L=\\measuredangle K A C+\\measuredangle C A L=\\measuredangle K B C+\\measuredangle A B K=\\measuredangle A B C . $$ ![](https:\/\/cdn.mathpix.com\/cropped\/2024_11_19_a0f01f2a44c8136e2158g-08.jpg?height=918&width=914&top_left_y=1551&top_left_x=571) Up to now we have not used the existence of $Q$; we henceforth do so. Note that $Q \\neq O$, since $\\angle A \\neq 60^{\\circ} \\Longrightarrow O \\notin \\omega$. Moreover, we have $\\angle A O M=\\angle A C B$ too. Since $O$ and $Q$ both lie inside $\\triangle A B C$, this implies that $A, M, O, Q$ are concyclic. As $Q \\neq O$ we conclude $\\angle C Q A=90^{\\circ}$. The main claim is now: Claim - Assuming $Q$ exists, the rhombus $L X K Y$ is a square. In particular, $\\overline{K X}$ and $\\overline{K Y}$ are tangent to $\\omega$. First proof of Claim, communicated by Milan Haiman. Observe that $\\triangle Q L C \\sim \\triangle L O C$ since both triangles are isosceles and share a base angle. Hence, $C L^{2}=C O \\cdot C Q$. Let $N$ be the midpoint of $\\overline{A C}$, which lies on $(A M O Q)$. Then, $$ x^{2}=C L^{2}=C O \\cdot C Q=C N \\cdot C A=\\frac{1}{2} C A^{2}=\\frac{1}{2} L K^{2} $$ where we have also used the fact $A Q O N$ is cyclic. Thus $L K=\\sqrt{2} x$ and so the rhombus $L X K Y$ is actually a square."} +{"year":2018,"label":"3","problem":"Let $A B C$ be an acute triangle with incenter $I$, circumcenter $O$, and circumcircle $\\Gamma$. Let $M$ be the midpoint of $\\overline{A B}$. Ray $A I$ meets $\\overline{B C}$ at $D$. Denote by $\\omega$ and $\\gamma$ the circumcircles of $\\triangle B I C$ and $\\triangle B A D$, respectively. Line $M O$ meets $\\omega$ at $X$ and $Y$, while line $C O$ meets $\\omega$ at $C$ and $Q$. Assume that $Q$ lies inside $\\triangle A B C$ and $\\angle A Q M=\\angle A C B$. Consider the tangents to $\\omega$ at $X$ and $Y$ and the tangents to $\\gamma$ at $A$ and $D$. Given that $\\angle B A C \\neq 60^{\\circ}$, prove that these four lines are concurrent on $\\Gamma$.","solution":" Henceforth assume $\\angle A \\neq 60^{\\circ}$; we prove the concurrence. Let $L$ denote the center of $\\omega$, which is the midpoint of minor arc $B C$. Claim - Let $K$ be the point on $\\omega$ such that $\\overline{K L} \\| \\overline{A B}$ and $\\overline{K C} \\| \\overline{A L}$. Then $\\overline{K A}$ is tangent to $\\gamma$, and we may put $$ x=K A=L B=L C=L X=L Y=K X=K Y . $$ Moreover, $\\overline{K A}$ is tangent to $\\gamma$ as well since $$ \\measuredangle K A D=\\measuredangle K A L=\\measuredangle K A C+\\measuredangle C A L=\\measuredangle K B C+\\measuredangle A B K=\\measuredangle A B C . $$ ![](https:\/\/cdn.mathpix.com\/cropped\/2024_11_19_a0f01f2a44c8136e2158g-08.jpg?height=918&width=914&top_left_y=1551&top_left_x=571) Up to now we have not used the existence of $Q$; we henceforth do so. Note that $Q \\neq O$, since $\\angle A \\neq 60^{\\circ} \\Longrightarrow O \\notin \\omega$. Moreover, we have $\\angle A O M=\\angle A C B$ too. Since $O$ and $Q$ both lie inside $\\triangle A B C$, this implies that $A, M, O, Q$ are concyclic. As $Q \\neq O$ we conclude $\\angle C Q A=90^{\\circ}$. The main claim is now: Claim - Assuming $Q$ exists, the rhombus $L X K Y$ is a square. In particular, $\\overline{K X}$ and $\\overline{K Y}$ are tangent to $\\omega$. Second proof of Claim, Evan Chen. Observe that $Q$ lies on the circle with diameter $\\overline{A C}$, centered at $N$, say. This means that $O$ lies on the radical axis of $\\omega$ and $(N)$, hence $\\overline{N L} \\perp \\overline{C O}$ implying $$ \\begin{aligned} N O^{2}+C L^{2} & =N C^{2}+L O^{2}=N C^{2}+O C^{2}=N C^{2}+N O^{2}+N C^{2} \\\\ \\Longrightarrow x^{2} & =2 N C^{2} \\\\ \\Longrightarrow x & =\\sqrt{2} N C=\\frac{1}{\\sqrt{2}} A C=\\frac{1}{\\sqrt{2}} L K \\end{aligned} $$ So $L X K Y$ is a rhombus with $L K=\\sqrt{2} x$. Hence it is a square."} +{"year":2018,"label":"3","problem":"Let $A B C$ be an acute triangle with incenter $I$, circumcenter $O$, and circumcircle $\\Gamma$. Let $M$ be the midpoint of $\\overline{A B}$. Ray $A I$ meets $\\overline{B C}$ at $D$. Denote by $\\omega$ and $\\gamma$ the circumcircles of $\\triangle B I C$ and $\\triangle B A D$, respectively. Line $M O$ meets $\\omega$ at $X$ and $Y$, while line $C O$ meets $\\omega$ at $C$ and $Q$. Assume that $Q$ lies inside $\\triangle A B C$ and $\\angle A Q M=\\angle A C B$. Consider the tangents to $\\omega$ at $X$ and $Y$ and the tangents to $\\gamma$ at $A$ and $D$. Given that $\\angle B A C \\neq 60^{\\circ}$, prove that these four lines are concurrent on $\\Gamma$.","solution":" Henceforth assume $\\angle A \\neq 60^{\\circ}$; we prove the concurrence. Let $L$ denote the center of $\\omega$, which is the midpoint of minor arc $B C$. Claim - Let $K$ be the point on $\\omega$ such that $\\overline{K L} \\| \\overline{A B}$ and $\\overline{K C} \\| \\overline{A L}$. Then $\\overline{K A}$ is tangent to $\\gamma$, and we may put $$ x=K A=L B=L C=L X=L Y=K X=K Y . $$ Moreover, $\\overline{K A}$ is tangent to $\\gamma$ as well since $$ \\measuredangle K A D=\\measuredangle K A L=\\measuredangle K A C+\\measuredangle C A L=\\measuredangle K B C+\\measuredangle A B K=\\measuredangle A B C . $$ ![](https:\/\/cdn.mathpix.com\/cropped\/2024_11_19_a0f01f2a44c8136e2158g-08.jpg?height=918&width=914&top_left_y=1551&top_left_x=571) Up to now we have not used the existence of $Q$; we henceforth do so. Note that $Q \\neq O$, since $\\angle A \\neq 60^{\\circ} \\Longrightarrow O \\notin \\omega$. Moreover, we have $\\angle A O M=\\angle A C B$ too. Since $O$ and $Q$ both lie inside $\\triangle A B C$, this implies that $A, M, O, Q$ are concyclic. As $Q \\neq O$ we conclude $\\angle C Q A=90^{\\circ}$. The main claim is now: Claim - Assuming $Q$ exists, the rhombus $L X K Y$ is a square. In particular, $\\overline{K X}$ and $\\overline{K Y}$ are tangent to $\\omega$. Third proof of Claim. A solution by trig is also possible. As in the previous claims, it suffices to show that $A C=\\sqrt{2} x$. First, we compute the length $C Q$ in two ways; by angle chasing one can show $\\angle C B Q=$ $180^{\\circ}-(\\angle B Q C+\\angle Q C B)=\\frac{1}{2} \\angle A$, and so $$ \\begin{aligned} A C \\sin B=C Q & =\\frac{B C}{\\sin \\left(90^{\\circ}+\\frac{1}{2} \\angle A\\right)} \\cdot \\sin \\frac{1}{2} \\angle A \\\\ \\Longleftrightarrow \\sin ^{2} B & =\\frac{\\sin A \\cdot \\sin \\frac{1}{2} \\angle A}{\\cos \\frac{1}{2} \\angle A} \\\\ \\Longleftrightarrow \\sin ^{2} B & =2 \\sin ^{2} \\frac{1}{2} \\angle A \\\\ \\Longleftrightarrow \\sin B & =\\sqrt{2} \\sin \\frac{1}{2} \\angle A \\\\ \\Longleftrightarrow 2 R \\sin B & =\\sqrt{2}\\left(2 R \\sin \\frac{1}{2} \\angle A\\right) \\\\ \\Longleftrightarrow A C & =\\sqrt{2} x \\end{aligned} $$ as desired (we have here used the fact $\\triangle A B C$ is acute to take square roots). It is interesting to note that $\\sin ^{2} B=2 \\sin ^{2} \\frac{1}{2} \\angle A$ can be rewritten as $$ \\cos A=\\cos ^{2} B $$ since $\\cos ^{2} B=1-\\sin ^{2} B=1-2 \\sin ^{2} \\frac{1}{2} \\angle A=\\cos A$; this is the condition for the existence of the point $Q$. We finish by proving that $$ K D=K A $$ and hence line $\\overline{K D}$ is tangent to $\\gamma$. Let $E=\\overline{B C} \\cap \\overline{K L}$. Then $$ L E \\cdot L K=L C^{2}=L X^{2}=\\frac{1}{2} L K^{2} $$ and so $E$ is the midpoint of $\\overline{L K}$. Thus $\\overline{M X O Y}, \\overline{B C}, \\overline{K L}$ are concurrent at $E$. As $\\overline{D L} \\| \\overline{K C}$, we find that $D L C K$ is a parallelogram, so $K D=C L=K A$ as well. Thus $\\overline{K D}$ and $\\overline{K A}$ are tangent to $\\gamma$. Remark. The condition $\\angle A \\neq 60^{\\circ}$ cannot be dropped, since if $Q=O$ the problem is not true. \\ Authorship comments. In the notation of the present points, the question originally sent to me by Yannick Yao read: Circles $(L)$ and $(O)$ are drawn, meeting at $B$ and $C$, with $L$ on $(O)$. Ray $C O$ meets $(L)$ at $Q$, and $A$ is on $(O)$ such that $\\angle C Q A=90^{\\circ}$. The angle bisector of $\\angle A O B$ meets $(L)$ at $X$ and $Y$. Show that $\\angle X L Y=90^{\\circ}$. Notice the points $M$ and $K$ are absent from the problem. I am told this was found as part of the computer game \"Euclidea\". Using this as the starting point, I constructed the TSTST problem by recognizing the significance of that special point $K$, which became the center of attention."} +{"year":2018,"label":"4","problem":"For an integer $n>0$, denote by $\\mathcal{F}(n)$ the set of integers $m>0$ for which the polynomial $p(x)=x^{2}+m x+n$ has an integer root. (a) Let $S$ denote the set of integers $n>0$ for which $\\mathcal{F}(n)$ contains two consecutive integers. Show that $S$ is infinite but $$ \\sum_{n \\in S} \\frac{1}{n} \\leq 1 $$ (b) Prove that there are infinitely many positive integers $n$ such that $\\mathcal{F}(n)$ contains three consecutive integers.","solution":" We prove the following. Claim - The set $S$ is given explicitly by $S=\\{x(x+1) y(y+1) \\mid x, y>0\\}$. $$ \\begin{aligned} m^{2}-4 n & =p^{2} \\\\ (m+1)^{2}-4 n & =q^{2} \\end{aligned} $$ Subtraction gives $2 m+1=q^{2}-p^{2}$, so $p$ and $q$ are different parities. We can thus let $q-p=2 x+1, q+p=2 y+1$, where $y \\geq x \\geq 0$ are integers. It follows that $$ \\begin{aligned} 4 n & =m^{2}-p^{2} \\\\ & =\\left(\\frac{q^{2}-p^{2}-1}{2}\\right)^{2}-p^{2}=\\left(\\frac{q^{2}-p^{2}-1}{2}-p\\right)\\left(\\frac{q^{2}-p^{2}-1}{2}+p\\right) \\\\ & =\\frac{q^{2}-\\left(p^{2}+2 p+1\\right)}{2} \\cdot \\frac{q^{2}-\\left(p^{2}-2 p+1\\right)}{2} \\\\ & =\\frac{1}{4}(q-p-1)(q-p+1)(q+p-1)(q+p+1)=\\frac{1}{4}(2 x)(2 x+2)(2 y)(2 y+2) \\\\ \\Longrightarrow n & =x(x+1) y(y+1) . \\end{aligned} $$ Since $n>0$ we require $x, y>0$. Conversely, if $n=x(x+1) y(y+1)$ for positive $x$ and $y$ then $m=\\sqrt{p^{2}+4 n}=\\sqrt{(y-x)^{2}+4 n}=2 x y+x+y=x(y+1)+(x+1) y$ and $m+1=2 x y+x+y+1=x y+(x+1)(y+1)$. Thus we conclude the main claim. From this, part (a) follows as $$ \\sum_{n \\in S} n^{-1} \\leq\\left(\\sum_{x \\geq 1} \\frac{1}{x(x+1)}\\right)\\left(\\sum_{y \\geq 1} \\frac{1}{y(y+1)}\\right)=1 \\cdot 1=1 $$ $$ \\begin{aligned} r^{2} & =(m+2)^{2}-4 n=m^{2}-4 n+4 m+4=p^{2}+2+2(2 m+1) \\\\ & =p^{2}+2\\left(q^{2}-p^{2}\\right)+2=2 q^{2}-p^{2}+2 \\\\ \\Longleftrightarrow 2 q^{2}+2 & =p^{2}+r^{2} \\quad(\\dagger) \\end{aligned} $$ with $q>p$ of different parity and $n=\\frac{1}{16}(q-p-1)(q-p+1)(q+p-1)(q+p+1)$. Note that (by taking modulo 8 ) we have $q \\not \\equiv p \\equiv r(\\bmod 2)$, and so there are no parity issues and we will always assume $p2$ do there exist infinitely many positive integers $n$ such that $n^{2}$ divides $b^{n}+1$ ?","solution":" This problem is sort of the union of IMO 1990\/3 and IMO 2000\/5. The answer is any $b$ such that $b+1$ is not a power of 2 . In the forwards direction, we first prove more carefully the following claim. Claim - If $b+1$ is a power of 2 , then the only $n$ which is valid is $n=1$. $$ b^{2 n} \\equiv 1 \\quad(\\bmod p) $$ so the order of $b(\\bmod p)$ divides $\\operatorname{gcd}(2 n, p-1)=2$. Hence $p \\mid b^{2}-1=(b-1)(b+1)$. But since $b+1$ was a power of 2 , this forces $p \\mid b-1$. Then $0 \\equiv b^{n}+1 \\equiv 2(\\bmod p)$, contradiction. On the other hand, suppose that $b+1$ is not a power of 2 (and that $b>2$ ). We will inductively construct an infinite sequence of distinct primes $p_{0}, p_{1}, \\ldots$, such that the following two properties hold for each $k \\geq 0$ : - $p_{0}^{2} \\ldots p_{k-1}^{2} p_{k} \\mid b^{p_{0} \\ldots p_{k-1}}+1$, - and hence $p_{0}^{2} \\ldots p_{k-1}^{2} p_{k}^{2} \\mid b^{p_{0} \\ldots p_{k-1} p_{k}}+1$ by exponent lifting lemma. This will solve the problem. Initially, let $p_{0}$ be any odd prime dividing $b+1$. For the inductive step, we contend there exists an odd prime $q \\notin\\left\\{p_{0}, \\ldots, p_{k}\\right\\}$ such that $q \\mid b^{p_{0} \\ldots p_{k}}+1$. Indeed, this follows immediately by Zsigmondy theorem since $p_{0} \\ldots p_{k}$ divides $b^{p_{0} \\ldots p_{k-1}}+1$. Since $\\left(b^{p_{0} \\ldots p_{k}}\\right)^{q} \\equiv b^{p_{0} \\ldots p_{k}}(\\bmod q)$, it follows we can then take $p_{k+1}=q$. This finishes the induction. To avoid the use of Zsigmondy, one can instead argue as follows: let $p=p_{k}$ for brevity, and let $c=b^{p_{0} \\ldots p_{k-1}}$. Then $\\frac{\\frac{c}{}^{p}+1}{c+1}=c^{p-1}-c^{p-2}+\\cdots+1$ has GCD exactly $p$ with $c+1$. Moreover, this quotient is always odd. Thus as long as $c^{p}+1>p \\cdot(c+1)$, there will be some new prime dividing $c^{p}+1$ but not $c+1$. This is true unless $p=3$ and $c=2$, but we assumed $b>2$ so this case does not appear. Remark (On new primes). In going from $n^{2} \\mid b^{n}+1$ to $(n q)^{2} \\mid b^{n q}+1$, one does not necessarily need to pick a $q$ such that $q \\nmid n$, as long as $\\nu_{q}\\left(n^{2}\\right)<\\nu_{q}\\left(b^{n}+1\\right)$. In other words it suffices to just check that $\\frac{b^{n}+1}{n^{2}}$ is not a power of 2 in this process. However, this calculation is a little more involved with this approach. One proceeds by noting that $n$ is odd, hence $\\nu_{2}\\left(b^{n}+1\\right)=\\nu_{2}(b+1)$, and thus $\\frac{b^{n}+1}{n^{2}}=2^{\\nu_{2}(b+1)} \\leq b+1$, which is a little harder to bound than the analogous $c^{p}+1>p \\cdot(c+1)$ from the previous solution. \\ Authorship comments. I came up with this problem by simply mixing together the main ideas of IMO 1990\/3 and IMO 2000\/5, late one night after a class. On the other hand, I do not consider it very original; it is an extremely \"routine\" number theory problem for experienced contestants, using highly standard methods. Thus it may not be that interesting, but is a good discriminator of understanding of fundamentals. IMO 1990\/3 shows that if $b=2$, then the only $n$ which work are $n=1$ and $n=3$. Thus $b=2$ is a special case and for this reason the problem explicitly requires $b>2$. An alternate formulation of the problem is worth mentioning. Originally, the problem statement asked whether there existed $n$ with at least 3 (or 2018, etc.) prime divisors, thus preventing the approach in which one takes a prime $q$ dividing $\\frac{b^{n}+1}{n^{2}}$. Ankan Bhattacharya suggested changing it to \"infinitely many $n$ \", which is more natural. These formulations are actually not so different though. Explicitly, suppose $k^{2} \\mid b^{k}+1$ and $p \\mid b^{k}+1$. Consider any $k \\mid n$ with $n^{2} \\mid b^{n}+1$, and let $p$ be an odd prime dividing $b^{k}+1$. Then $2 \\nu_{p}(n) \\leq \\nu_{p}\\left(b^{n}+1\\right)=\\nu_{p}(n \/ k)+\\nu_{p}\\left(b^{k}+1\\right)$ and thus $$ \\nu_{p}(n \/ k) \\leq \\nu_{p}\\left(\\frac{b^{k}+1}{k^{2}}\\right) . $$ Effectively, this means we can only add each prime a certain number of times."} +{"year":2018,"label":"9","problem":"Show that there is an absolute constant $c<1$ with the following property: whenever $\\mathcal{P}$ is a polygon with area 1 in the plane, one can translate it by a distance of $\\frac{1}{100}$ in some direction to obtain a polygon $\\mathcal{Q}$, for which the intersection of the interiors of $\\mathcal{P}$ and $\\mathcal{Q}$ has total area at most $c$.","solution":" Suppose $\\mathcal{P}$ is a polygon of area 1 , and $\\varepsilon>0$ is a constant, such that for any translate $\\mathcal{Q}=\\mathcal{P}+v$, where $v$ has length exactly $\\frac{1}{100}$, the intersection of $\\mathcal{P}$ and $\\mathcal{Q}$ has area at least $1-\\varepsilon$. The problem asks us to prove a lower bound on $\\varepsilon$. ## Lemma Fix a sequence of $n$ vectors $v_{1}, v_{2}, \\ldots, v_{n}$, each of length $\\frac{1}{100}$. A grasshopper starts at a random point $x$ of $\\mathcal{P}$, and makes $n$ jumps to $x+v_{1}+\\cdots+v_{n}$. Then it remains in $\\mathcal{P}$ with probability at least $1-n \\varepsilon$. ## Corollary Fix a vector $w$ of length at most 8. A grasshopper starts at a random point $x$ of $\\mathcal{P}$, and jumps to $x+w$. Then it remains in $\\mathcal{P}$ with probability at least $1-800 \\varepsilon$. Now consider the process where we select a random starting point $x \\in \\mathcal{P}$ for our grasshopper, and a random vector $w$ of length at most 8 (sampled uniformly from the closed disk of radius 8 ). Let $q$ denote the probability of staying inside $\\mathcal{P}$ we will bound $q$ from above and below. - On the one hand, suppose we pick $w$ first. By the previous corollary, $q \\geq 1-800 \\varepsilon$ (irrespective of the chosen $w$ ). - On the other hand, suppose we pick $x$ first. Then the possible landing points $x+w$ are uniformly distributed over a closed disk of radius 8 , which has area $64 \\pi$. The probability of landing in $\\mathcal{P}$ is certainly at most $\\frac{[\\mathcal{P}]}{64 \\pi}$. Consequently, we deduce $$ 1-800 \\varepsilon \\leq q \\leq \\frac{[\\mathcal{P}]}{64 \\pi} \\Longrightarrow \\varepsilon>\\frac{1-\\frac{[\\mathcal{P}]}{64 \\pi}}{800}>0.001 $$ as desired. Remark. The choice of 800 jumps is only for concreteness; any constant $n$ for which $\\pi(n \/ 100)^{2}>1$ works. I think $n=98$ gives the best bound following this approach."} diff --git a/USA_TSTST/segmented/en-sols-TSTST-2019.jsonl b/USA_TSTST/segmented/en-sols-TSTST-2019.jsonl new file mode 100644 index 0000000000000000000000000000000000000000..eef29cbe0d69cd3fecbdfb88dc8c974da00b6a7f --- /dev/null +++ b/USA_TSTST/segmented/en-sols-TSTST-2019.jsonl @@ -0,0 +1,15 @@ +{"year":2019,"label":"1","problem":"Find all binary operations $\\diamond: \\mathbb{R}_{>0} \\times \\mathbb{R}_{>0} \\rightarrow \\mathbb{R}_{>0}$ (meaning $\\diamond$ takes pairs of positive real numbers to positive real numbers) such that for any real numbers $a, b, c>0$, - the equation $a \\diamond(b \\diamond c)=(a \\diamond b) \\cdot c$ holds; and - if $a \\geq 1$ then $a \\diamond a \\geq 1$.","solution":" The answer is only multiplication and division, which both obviously work. \u3010 First solution using Cauchy FE. We prove: Claim - We have $a \\diamond b=a f(b)$ where $f$ is some involutive and totally multiplicative function. (In fact, this classifies all functions satisfying the first condition completely.) - Note that for any $x$, the function $y \\mapsto x \\diamond y$ is injective, because if $x \\diamond y_{1}=x \\diamond y_{2}$ then take $P\\left(1, x, y_{i}\\right)$ to get $y_{1}=y_{2}$. - Take $P(1, x, 1)$ and injectivity to get $x \\diamond 1=x$. - Take $P(1,1, y)$ to get $1 \\diamond(1 \\diamond y)=y$. - Take $P(x, 1,1 \\diamond y)$ to get $$ x \\diamond y=x \\cdot(1 \\diamond y) $$ Henceforth let us define $f(y)=1 \\diamond y$, so $f(1)=1, f$ is involutive and $$ x \\diamond y=x f(y) $$ Plugging this into the original condition now gives $f(b f(c))=f(b) c$, which (since $f$ is an involution) gives $f$ completely multiplicative. In particular, $f(1)=1$. We are now interested only in the second condition, which reads $f(x) \\geq 1 \/ x$ for $x \\geq 1$. Define the function $$ g(t)=\\log f\\left(e^{t}\\right) $$ so that $g$ is additive, and also $g(t) \\geq-t$ for all $t \\geq 0$. We appeal to the following theorem: ## Lemma If $h: \\mathbb{R} \\rightarrow \\mathbb{R}$ is an additive function which is not linear, then it is dense in the plane: for any point $\\left(x_{0}, y_{0}\\right)$ and $\\varepsilon>0$ there exists $(x, y)$ such that $h(x)=y$ and $\\sqrt{\\left(x-x_{0}\\right)^{2}+\\left(y-y_{0}\\right)^{2}}<\\varepsilon$. Applying this lemma with the fact that $g(t) \\geq-t$ implies readily that $g$ is linear. In other words, $f$ is of the form $f(x)=x^{r}$ for some fixed real number $r$. It is easy to check $r= \\pm 1$ which finishes."} +{"year":2019,"label":"1","problem":"Find all binary operations $\\diamond: \\mathbb{R}_{>0} \\times \\mathbb{R}_{>0} \\rightarrow \\mathbb{R}_{>0}$ (meaning $\\diamond$ takes pairs of positive real numbers to positive real numbers) such that for any real numbers $a, b, c>0$, - the equation $a \\diamond(b \\diamond c)=(a \\diamond b) \\cdot c$ holds; and - if $a \\geq 1$ then $a \\diamond a \\geq 1$.","solution":" The answer is only multiplication and division, which both obviously work. \u3010 Second solution manually. As before we arrive at $a \\diamond b=a f(b)$, with $f$ an involutive and totally multiplicative function. We prove that: Claim - For any $a>0$, we have $f(a) \\in\\{1 \/ a, a\\}$. Assume that $a b>1$; we show $a=b$. Note that for integers $m$ and $n$ with $a^{n} b^{m} \\geq 1$, we must have $$ a^{m} b^{n}=f(b)^{m} f(a)^{n}=f\\left(a^{n} b^{m}\\right) \\geq \\frac{1}{a^{n} b^{m}} \\Longrightarrow(a b)^{m+n} \\geq 1 $$ and thus we have arrived at the proposition $$ m+n<0 \\Longrightarrow n \\log _{b} a+m<0 $$ for all integers $m$ and $n$. Due to the density of $\\mathbb{Q}$ in the real numbers, this can only happen if $\\log _{b} a=1$ or $a=b$. Claim - The function $f$ is continuous. $$ |g(t)-g(s)|=\\left|\\log f\\left(e^{t-s}\\right)\\right|=|t-s| $$ since $f\\left(e^{t-s}\\right)=e^{ \\pm|t-s|}$. Therefore $g$ is Lipschitz. Hence $g$ continuous, and $f$ is too. Finally, we have from $f$ multiplicative that $$ f\\left(2^{q}\\right)=f(2)^{q} $$ for every rational number $q$, say. As $f$ is continuous this implies $f(x) \\equiv x$ or $f(x) \\equiv 1 \/ x$ identically (depending on whether $f(2)=2$ or $f(2)=1 \/ 2$, respectively). Therefore, $a \\diamond b=a b$ or $a \\diamond b=a \\div b$, as needed. Remark. The Lipschitz condition is one of several other ways to proceed. The point is that if $f(2)=2$ (say), and $x \/ 2^{q}$ is close to 1 , then $f(x) \/ 2^{q}=f\\left(x \/ 2^{q}\\right)$ is close to 1 , which is enough to force $f(x)=x$ rather than $f(x)=1 \/ x$. Remark. Compare to AMC 10A 2016 \\#23, where the second condition is $a \\diamond a=1$."} +{"year":2019,"label":"2","problem":"Let $A B C$ be an acute triangle with circumcircle $\\Omega$ and orthocenter $H$. Points $D$ and $E$ lie on segments $A B$ and $A C$ respectively, such that $A D=A E$. The lines through $B$ and $C$ parallel to $\\overline{D E}$ intersect $\\Omega$ again at $P$ and $Q$, respectively. Denote by $\\omega$ the circumcircle of $\\triangle A D E$. (a) Show that lines $P E$ and $Q D$ meet on $\\omega$. (b) Prove that if $\\omega$ passes through $H$, then lines $P D$ and $Q E$ meet on $\\omega$ as well.","solution":" Claim - Points $L, D, P$ are collinear. $$ \\begin{aligned} \\measuredangle C L D & =\\measuredangle D H L=\\measuredangle D H A+\\measuredangle A H L=\\measuredangle D E A+\\measuredangle A H C \\\\ & =\\measuredangle A D E+\\measuredangle C B A=\\measuredangle A B P+\\measuredangle C B A=\\measuredangle C B P=\\measuredangle C L P . \\end{aligned} $$ ![](https:\/\/cdn.mathpix.com\/cropped\/2024_11_19_f4f672a97b66743213a8g-06.jpg?height=641&width=709&top_left_y=1707&top_left_x=676) Now let $K \\in \\omega$ such that $D H K E$ is an isosceles trapezoid, i.e. $\\measuredangle B A H=\\measuredangle K A E$. Claim - Points $D, K, P$ are collinear. $$ \\measuredangle K D E=\\measuredangle K A E=\\measuredangle B A H=\\measuredangle L A B=\\measuredangle L P B=\\measuredangle D P B=\\measuredangle P D E . $$ By symmetry, $\\overline{Q E}$ will then pass through the same $K$, as needed. Remark. These two claims imply each other, so guessing one of them allows one to realize the other. It is likely the latter is easiest to guess from the diagram, since it does not need any additional points. Claim - Point $K$ is the orthocenter of isosceles triangle $A P Q$. In light of the formula \" $A H^{2}=4 R^{2}-a^{2}$ \", this implies the conclusion. Let $M$ be the midpoint of $\\overline{P Q}$. Since $\\triangle A P Q$ is isosceles, $$ \\overline{A K M} \\perp \\overline{P Q} \\Longrightarrow M K \\cdot M A=M P^{2} $$ by orthocenter properties. So to summarize - The circle with diameter $\\overline{P Q}$ is orthogonal to $\\omega$. In other words, point $P$ lies on the polar of $Q$ with respect to $\\omega$. - The point $X=\\overline{Q D} \\cap \\overline{P E}$ is on $\\omega$. On the other hand, if we let $K^{\\prime}=\\overline{Q E} \\cap \\omega$, then by Brokard theorem on $X D K^{\\prime} E$, the polar of $Q=\\overline{X D} \\cap \\overline{K^{\\prime} E}$ pass through $\\overline{D K^{\\prime}} \\cap \\overline{X E}$; this point must therefore be $P$ and $K^{\\prime}=K$ as desired. $$ A=a^{2}, B=b^{2}, C=c^{2}, M=-b c, H=a^{2}+b^{2}+c^{2}, P=\\frac{a^{2} c}{b}, Q=\\frac{a^{2} b}{c} $$ where $M$ is the arc midpoint of $\\widehat{B C}$. By direct angle chasing we can verify that $\\overline{M B} \\| \\overline{D H}$. Also, $D \\in \\overline{A B}$. Therefore, we can compute $D$ as follows. $$ d+a^{2} b^{2} \\bar{d}=a^{2}+b^{2} \\text { and } \\frac{d-h}{\\bar{d}-\\bar{h}}=-m b^{2}=b^{3} c \\Longrightarrow d=\\frac{a^{2}\\left(a^{2} c+b^{2} c+c^{3}-b^{3}\\right)}{c\\left(b c+a^{2}\\right)} . $$ By symmetry, we have that $$ e=\\frac{a^{2}\\left(a^{2} b+b c^{2}+b^{3}-c^{3}\\right)}{b\\left(b c+a^{2}\\right)} . $$ To finish, we want to show that the angle between $\\overline{D P}$ and $\\overline{E Q}$ is angle $A$. To show this, we compute $\\frac{d-p}{e-q} \/ \\overline{\\frac{d-p}{e-q}}$. First, we compute $$ \\begin{aligned} d-p & =\\frac{a^{2}\\left(a^{2} c+b^{2} c+c^{3}-b^{3}\\right)}{c\\left(b c+a^{2}\\right)}-\\frac{a^{2} c}{b} \\\\ & =a^{2}\\left(\\frac{a^{2} c+b^{2} c+c^{3}-b^{3}}{c\\left(b c+a^{2}\\right)}-\\frac{c}{b}\\right)=\\frac{a^{2}\\left(a^{2} c-b^{3}\\right)(b-c)}{b c\\left(b c+a^{2}\\right)} \\end{aligned} $$ By symmetry, $$ \\frac{d-p}{e-q}=-\\frac{a^{2} c-b^{3}}{a^{2} b-c^{3}} \\Longrightarrow \\frac{d-p}{e-q} \/ \\overline{\\frac{d-p}{e-q}}=\\frac{a^{2} b^{3} c}{a^{2} b c^{3}}=\\frac{b^{2}}{c^{2}} $$ as desired. Recall: ## Lemma (Zack's lemma) Suppose points $A, B$ have degree $d_{1}, d_{2}$, and there are $k$ values of $t$ for which $A=B$. Then line $A B$ has degree at most $d_{1}+d_{2}-k$. Similarly, if lines $\\ell_{1}, \\ell_{2}$ have degrees $d_{1}, d_{2}$, and there are $k$ values of $t$ for which $\\ell_{1}=\\ell_{2}$, then the intersection $\\ell_{1} \\cap \\ell_{2}$ has degree at most $d_{1}+d_{2}-k$. Now, note that $H$ moves linearly in $C$ on line $B H$. Furthermore, angles $\\angle A H E$, $\\angle A H F$ are fixed, we get that $D$ and $E$ have degree 2 . One way to see this is using the lemma; $D$ lies on line $A B$, which is fixed, and line $H D$ passes through a point at infinity which is a constant rotation of the point at infinity on line $A H$, and therefore has degree 1. Then $D, E$ have degree at most $1+1-0=2$. Now, note that $P, Q$ move linearly in $C$. Both of these are because the circumcenter $O$ moves linearly in $C$, and $P, Q$ are reflections of $B, C$ in a line through $O$ with fixed direction, which also moves linearly. So by the lemma, the lines $P D, Q E$ have degree at most 3 . I claim they actually have degree 2; to show this it suffices to give an example of a choice of $C$ for which $P=D$ and one for which $Q=E$. But an easy angle chase shows that in the unique case when $P=B$, we get $D=B$ as well and thus $P=D$. Similarly when $Q=C, E=C$. It follows from the lemma that lines $P D, Q E$ have degree at most 2 . Let $\\ell_{\\infty}$ denote the line at infinity. I claim that the points $P_{1}=P D \\cap \\ell_{\\infty}, P_{2}=Q E \\cap \\ell_{\\infty}$ are projective in $C$. Since $\\ell_{\\infty}$ is fixed, it suffices to show by the lemma that there exists some value of $C$ for which $Q E=\\ell_{\\infty}$ and $P D=\\ell_{\\infty}$. But note that as $C \\rightarrow \\infty$, all four points $P, D, Q, E$ go to infinity. It follows that $P_{1}, P_{2}$ are projective in $C$. Then to finish, recall that we want to show that $\\angle(P D, Q E)$ is constant. It suffices then to show that there's a constant rotation sending $P_{1}$ to $P_{2}$. Since $P_{1}, P_{2}$ are projective, it suffices to verify this for 3 values of $C$. We can take $C$ such that $\\angle A B C=90, \\angle A C B=90$, or $A B=A C$, and all three cases are easy to check."} +{"year":2019,"label":"3","problem":"On an infinite square grid we place finitely many cars, which each occupy a single cell and face in one of the four cardinal directions. Cars may never occupy the same cell. It is given that the cell immediately in front of each car is empty, and moreover no two cars face towards each other (no right-facing car is to the left of a left-facing car within a row, etc.). In a move, one chooses a car and shifts it one cell forward to a vacant cell. Prove that there exists an infinite sequence of valid moves using each car infinitely many times.","solution":" Let $S$ be any rectangle containing all the cars. Partition $S$ into horizontal strips of height 1 , and color them red and green in an alternating fashion. It is enough to prove all the cars may exit $S$. ![](https:\/\/cdn.mathpix.com\/cropped\/2024_11_19_f4f672a97b66743213a8g-09.jpg?height=880&width=1038&top_left_y=1096&top_left_x=512) To do so, we outline a five-stage plan for the cars. 1. All vertical cars in a green cell may advance one cell into a red cell (or exit $S$ altogether), by the given condition. (This is the only place where the hypothesis about empty space is used!) 2. All horizontal cars on green cells may exit $S$, as no vertical cars occupy green cells. 3. All vertical cars in a red cell may advance one cell into a green cell (or exit $S$ altogether), as all green cells are empty. 4. All horizontal cars within red cells may exit $S$, as no vertical car occupy red cells. 5. The remaining cars exit $S$, as they are all vertical. The solution is complete. Remark (Higher-dimensional generalization by author). The natural higher-dimensional generalization is true, and can be proved in largely the same fashion. For example, in three dimensions, one may let $S$ be a rectangular prism and partition $S$ into horizontal slabs and color them red and green in an alternating fashion. Stages 1, 3, and 5 generalize immediately, and stages 2 and 4 reduce to an application of the two-dimensional problem. In the same way, the general problem is handled by induction on the dimension. Remark (Historical comments). For $k>1$, we could consider a variant of the problem where cars are $1 \\times k$ rectangles (moving parallel to the longer edge) instead of occupying single cells. In that case, if there are $2 k-1$ empty spaces in front of each car, the above proof works (with the red and green strips having height $k$ instead). On the other hand, at least $k$ empty spaces are necessary. We don't know the best constant in this case."} +{"year":2019,"label":"4","problem":"Consider coins with positive real denominations not exceeding 1. Find the smallest $C>0$ such that the following holds: if we are given any 100 such coins with total value 50 , then we can always split them into two stacks of 50 coins each such that the absolute difference between the total values of the two stacks is at most $C$.","solution":" The answer is $C=\\frac{50}{51}$. The lower bound is obtained if we have 51 coins of value $\\frac{1}{51}$ and 49 coins of value 1. (Alternatively, 51 coins of value $1-\\frac{\\varepsilon}{51}$ and 49 coins of value $\\frac{\\varepsilon}{49}$ works fine for $\\varepsilon>0$.) We now present two (similar) proofs that this $C=\\frac{50}{51}$ suffices. \u3010 First proof (original). Let $a_{1} \\leq \\cdots \\leq a_{100}$ denote the values of the coins in ascending order. Since the 51 coins $a_{50}, \\ldots, a_{100}$ are worth at least $51 a_{50}$, it follows that $a_{50} \\leq \\frac{50}{51}$; likewise $a_{51} \\geq \\frac{1}{51}$. We claim that choosing the stacks with coin values $$ a_{1}, a_{3}, \\ldots, a_{49}, \\quad a_{52}, a_{54}, \\ldots, a_{100} $$ and $$ a_{2}, a_{4}, \\ldots, a_{50}, \\quad a_{51}, a_{53}, \\ldots, a_{99} $$ works. Let $D$ denote the (possibly negative) difference between the two total values. Then $$ \\begin{aligned} D & =\\left(a_{1}-a_{2}\\right)+\\cdots+\\left(a_{49}-a_{50}\\right)-a_{51}+\\left(a_{52}-a_{53}\\right)+\\cdots+\\left(a_{98}-a_{99}\\right)+a_{100} \\\\ & \\leq 25 \\cdot 0-\\frac{1}{51}+24 \\cdot 0+1=\\frac{50}{51} \\end{aligned} $$ Similarly, we have $$ \\begin{aligned} D & =a_{1}+\\left(a_{3}-a_{2}\\right)+\\cdots+\\left(a_{49}-a_{48}\\right)-a_{50}+\\left(a_{52}-a_{51}\\right)+\\cdots+\\left(a_{100}-a_{99}\\right) \\\\ & \\geq 0+24 \\cdot 0-\\frac{50}{51}+25 \\cdot 0=-\\frac{50}{51} \\end{aligned} $$ It follows that $|D| \\leq \\frac{50}{51}$, as required."} +{"year":2019,"label":"4","problem":"Consider coins with positive real denominations not exceeding 1. Find the smallest $C>0$ such that the following holds: if we are given any 100 such coins with total value 50 , then we can always split them into two stacks of 50 coins each such that the absolute difference between the total values of the two stacks is at most $C$.","solution":" The answer is $C=\\frac{50}{51}$. The lower bound is obtained if we have 51 coins of value $\\frac{1}{51}$ and 49 coins of value 1. (Alternatively, 51 coins of value $1-\\frac{\\varepsilon}{51}$ and 49 coins of value $\\frac{\\varepsilon}{49}$ works fine for $\\varepsilon>0$.) We now present two (similar) proofs that this $C=\\frac{50}{51}$ suffices. \u3010 Second proof (Evan Chen). Again we sort the coins in increasing order $0a_{i-1}+\\frac{50}{51}$; obviously there is at most one such large gap. Claim - If there is a large gap, it must be $a_{51}>a_{50}+\\frac{50}{51}$. Now imagine starting with the coins $a_{1}, a_{3}, \\ldots, a_{99}$, which have total value $S \\leq 25$. We replace $a_{1}$ by $a_{2}$, then $a_{3}$ by $a_{4}$, and so on, until we replace $a_{99}$ by $a_{100}$. At the end of the process we have $S \\geq 25$. Moreover, since we did not cross a large gap at any point, the quantity $S$ changed by at most $C=\\frac{50}{51}$ at each step. So at some point in the process we need to have $25-C \/ 2 \\leq S \\leq 25+C \/ 2$, which proves $C$ works."} +{"year":2019,"label":"5","problem":"Let $A B C$ be an acute triangle with orthocenter $H$ and circumcircle $\\Gamma$. A line through $H$ intersects segments $A B$ and $A C$ at $E$ and $F$, respectively. Let $K$ be the circumcenter of $\\triangle A E F$, and suppose line $A K$ intersects $\\Gamma$ again at a point $D$. Prove that line $H K$ and the line through $D$ perpendicular to $\\overline{B C}$ meet on $\\Gamma$.","solution":" \u300e First solution (Andrew Gu). We begin with the following two observations. Claim - Point $K$ lies on the radical axis of $(B E H)$ and $(C F H)$. $$ \\measuredangle H E K=90^{\\circ}-\\measuredangle E A F=90^{\\circ}-\\measuredangle B A C=\\measuredangle H B E $$ implying the result. Since $K E=K F$, this implies the result. Claim - The second intersection $M$ of $(B E H)$ and $(C F H)$ lies on $\\Gamma$. ![](https:\/\/cdn.mathpix.com\/cropped\/2024_11_19_f4f672a97b66743213a8g-13.jpg?height=806&width=632&top_left_y=1613&top_left_x=712) In particular, $M, H, K$ are collinear. Let $X$ be on $\\Gamma$ with $\\overline{D X} \\perp \\overline{B C}$; we then wish to show $X$ lies on the line $M H K$ we found. This is angle chasing: compute $$ \\measuredangle X M B=\\measuredangle X D B=90^{\\circ}-\\measuredangle D B C=90^{\\circ}-\\measuredangle D A C $$ $$ =90^{\\circ}-\\measuredangle K A F=\\measuredangle F E A=\\measuredangle H E B=\\measuredangle H M B $$ as needed."} +{"year":2019,"label":"5","problem":"Let $A B C$ be an acute triangle with orthocenter $H$ and circumcircle $\\Gamma$. A line through $H$ intersects segments $A B$ and $A C$ at $E$ and $F$, respectively. Let $K$ be the circumcenter of $\\triangle A E F$, and suppose line $A K$ intersects $\\Gamma$ again at a point $D$. Prove that line $H K$ and the line through $D$ perpendicular to $\\overline{B C}$ meet on $\\Gamma$.","solution":" II Second solution (Ankan Bhattacharya). We let $D^{\\prime}$ be the second intersection of $\\overline{E F}$ with $(B H C)$ and redefine $D$ as the reflection of $D^{\\prime}$ across $\\overline{B C}$. We will first prove that this point $D$ coincides with the point $D$ given in the problem statement. The idea is that: Claim - $A$ is the $D$-excenter of $\\triangle D E F$. $$ \\begin{aligned} \\measuredangle D^{\\prime} D B & =\\measuredangle B D^{\\prime} D=\\measuredangle D^{\\prime} B C+90^{\\circ}=\\measuredangle D^{\\prime} H C+90^{\\circ} \\\\ & =\\measuredangle D^{\\prime} H C+\\measuredangle(H C, A B)=\\measuredangle\\left(D^{\\prime} H, A B\\right)=\\measuredangle D^{\\prime} E B \\end{aligned} $$ Now as $B D=B D^{\\prime}$, we obtain $\\overline{B E A}$ externally bisects $\\angle D E D^{\\prime} \\cong \\angle D E F$. Likewise $\\overline{F A}$ externally bisects $\\angle D F E$, so $A$ is the $D$-excenter of $\\triangle D E F$. Hence, by the so-called \"Fact 5 \", point $K$ lies on $\\overline{D A}$, so this point $D$ is the one given in the problem statement. ![](https:\/\/cdn.mathpix.com\/cropped\/2024_11_19_f4f672a97b66743213a8g-14.jpg?height=778&width=712&top_left_y=1256&top_left_x=678) Now choose point $X$ on $(A B C)$ satisfying $\\overline{D X} \\perp \\overline{B C}$. Claim - Point $K$ lies on line $H X$. $$ \\frac{K D}{K A}=\\frac{K D}{K E}=\\frac{D E+D F}{E F} $$ On the other hand, if we let $r_{D}$ denote the $D$-exradius of $\\triangle D E F$ then $$ \\frac{X D}{X D^{\\prime}}=\\frac{[D E X]+[D F X]}{[X E F]}=\\frac{[D E X]+[D F X]}{[A E F]}=\\frac{D E \\cdot r_{D}+D F \\cdot r_{D}}{E F \\cdot r_{D}}=\\frac{D E+D F}{E F} $$ Thus $$ [A K X]=\\frac{K A}{K D} \\cdot[D K X]=\\frac{K A}{K D} \\cdot \\frac{X D}{X D^{\\prime}} \\cdot\\left[K D^{\\prime} X\\right]=\\left[D^{\\prime} K X\\right] $$ This is sufficient to prove $K$ lies on $\\overline{H X}$."} +{"year":2019,"label":"5","problem":"Let $A B C$ be an acute triangle with orthocenter $H$ and circumcircle $\\Gamma$. A line through $H$ intersects segments $A B$ and $A C$ at $E$ and $F$, respectively. Let $K$ be the circumcenter of $\\triangle A E F$, and suppose line $A K$ intersects $\\Gamma$ again at a point $D$. Prove that line $H K$ and the line through $D$ perpendicular to $\\overline{B C}$ meet on $\\Gamma$.","solution":" I Fourth solution, complex numbers with spiral similarity (Evan Chen). First if $\\overline{A D} \\perp \\overline{B C}$ there is nothing to prove, so we assume this is not the case. Let $W$ be the antipode of $D$. Let $S$ denote the second intersection of $(A E F)$ and $(A B C)$. Consider the spiral similarity sending $\\triangle S E F$ to $\\triangle S B C$ : - It maps $H$ to a point $G$ on line $B C$, - It maps $K$ to $O$. - It maps the $A$-antipode of $\\triangle A E F$ to $D$. - Hence (by previous two observations) it maps $A$ to $W$. - Also, the image of line $A D$ is line $W O$, which does not coincide with line $B C$ (as $O$ does not lie on line $B C$ ). Therefore, $K$ is the unique point on line $\\overline{A D}$ for one can get a direct similarity $$ \\triangle A K H \\sim \\triangle W O G $$ for some point $G$ lying on line $\\overline{B C}$. ![](https:\/\/cdn.mathpix.com\/cropped\/2024_11_19_f4f672a97b66743213a8g-15.jpg?height=804&width=775&top_left_y=1477&top_left_x=646) On the other hand, let us re-define $K$ as $\\overline{X H} \\cap \\overline{A D}$. We will show that the corresponding $G$ making $(\\triangle)$ true lies on line $B C$. We apply complex numbers with $\\Gamma$ the unit circle, with $a, b, c, d$ taking their usual meanings, $H=a+b+c, X=-b c \/ d$, and $W=-d$. Then point $K$ is supposed to satisfy $$ k+a d \\bar{k}=a+d $$ $$ \\begin{aligned} \\frac{k+\\frac{b c}{d}}{a+b+c+\\frac{b c}{d}} & =\\frac{\\bar{k}+\\frac{d}{b c}}{\\frac{1}{a}+\\frac{1}{b}+\\frac{1}{c}+\\frac{d}{b c}} \\\\ \\Longleftrightarrow \\frac{\\frac{1}{a}+\\frac{1}{b}+\\frac{1}{c}+\\frac{d}{b c}}{a+b+c+\\frac{b c}{d}}\\left(k+\\frac{b c}{d}\\right) & =\\bar{k}+\\frac{d}{b c} \\end{aligned} $$ Adding $a d$ times the last line to the first line and cancelling $a d \\bar{k}$ now gives $$ \\left(a d \\cdot \\frac{\\frac{1}{a}+\\frac{1}{b}+\\frac{1}{c}+\\frac{d}{b c}}{a+b+c+\\frac{b c}{d}}+1\\right) k=a+d+\\frac{a d^{2}}{b c}-a b c \\cdot \\frac{\\frac{1}{a}+\\frac{1}{b}+\\frac{1}{c}+\\frac{d}{b c}}{a+b+c+\\frac{b c}{d}} $$ or $$ \\begin{aligned} \\left(a d\\left(\\frac{1}{a}+\\frac{1}{b}+\\frac{1}{c}+\\frac{d}{b c}\\right)+a+b+c+\\frac{b c}{d}\\right) k & =\\left(a+b+c+\\frac{b c}{d}\\right)\\left(a+d+\\frac{a d^{2}}{b c}\\right) \\\\ & -a b c \\cdot\\left(\\frac{1}{a}+\\frac{1}{b}+\\frac{1}{c}+\\frac{d}{b c}\\right) . \\end{aligned} $$ We begin by simplifying the coefficient of $k$ : $$ \\begin{aligned} a d\\left(\\frac{1}{a}+\\frac{1}{b}+\\frac{1}{c}+\\frac{d}{b c}\\right)+a+b+c+\\frac{b c}{d} & =a+b+c+d+\\frac{b c}{d}+\\frac{a d}{b}+\\frac{a d}{c}+\\frac{a d^{2}}{b c} \\\\ & =a+\\frac{b c}{d}+\\left(1+\\frac{a d}{b c}\\right)(b+c+d) \\\\ & =\\frac{a d+b c}{b c d}[b c+d(b+c+d)] \\\\ & =\\frac{(a d+b c)(d+b)(d+c)}{b c d} \\end{aligned} $$ Meanwhile, the right-hand side expands to $$ \\begin{aligned} \\mathrm{RHS}= & \\left(a+b+c+\\frac{b c}{d}\\right)\\left(a+d+\\frac{a d^{2}}{b c}\\right)-a b c \\cdot\\left(\\frac{1}{a}+\\frac{1}{b}+\\frac{1}{c}+\\frac{d}{b c}\\right) \\\\ = & \\left(a^{2}+a b+a c+\\frac{a b c}{d}\\right)+(d a+d b+d c+b c) \\\\ & +\\left(\\frac{a^{2} d^{2}}{b c}+\\frac{a d^{2}}{c}+\\frac{a d^{2}}{b}+a d\\right)-(a b+b c+c a+a d) \\\\ = & a^{2}+d(a+b+c)+\\frac{a b c}{d}+\\frac{a^{2} d^{2}}{b c}+\\frac{a d^{2}}{b}+\\frac{a d^{2}}{c} \\\\ = & a^{2}+\\frac{a b c}{d}+d(a+b+c) \\cdot \\frac{a d+b c}{b c} \\\\ = & \\frac{a d+b c}{b c d}\\left[a b c+d^{2}(a+b+c)\\right] . \\end{aligned} $$ Therefore, we get $$ k=\\frac{a b c+d^{2}(a+b+c)}{(d+b)(d+c)} $$ In particular, $$ \\begin{aligned} k-a & =\\frac{a b c+d^{2}(a+b+c)-a(d+b)(d+c)}{(d+b)(d+c)} \\\\ & =\\frac{d^{2}(b+c)-d a(b+c)}{(d+b)(d+c)}=\\frac{d(b+c)(d-a)}{(d+b)(d+c)} \\end{aligned} $$ Now the corresponding point $G$ obeying $(\\Omega)$ satisfies $$ \\begin{aligned} \\frac{g-(-d)}{0-(-d)} & =\\frac{(a+b+c)-a}{k-a} \\\\ \\Longrightarrow g & =-d+\\frac{d(b+c)}{k-a} \\\\ & =-d+\\frac{(d+b)(d+c)}{d-a}=\\frac{d b+d c+b c+a d}{d-a} . \\\\ \\Longrightarrow b c \\bar{g} & =\\frac{b c \\cdot \\frac{a c+a b+a d+b c}{a b c d}}{\\frac{a-d}{a d}}=-\\frac{a b+a c+a d+b c}{d-a} . \\\\ \\Longrightarrow g+b c \\bar{g} & =\\frac{(d-a)(b+c)}{d-a}=b+c . \\end{aligned} $$ Hence $G$ lies on $B C$ and this completes the proof."} +{"year":2019,"label":"5","problem":"Let $A B C$ be an acute triangle with orthocenter $H$ and circumcircle $\\Gamma$. A line through $H$ intersects segments $A B$ and $A C$ at $E$ and $F$, respectively. Let $K$ be the circumcenter of $\\triangle A E F$, and suppose line $A K$ intersects $\\Gamma$ again at a point $D$. Prove that line $H K$ and the line through $D$ perpendicular to $\\overline{B C}$ meet on $\\Gamma$.","solution":" \u3010 Seventh solution using moving points (Zack Chroman). We state the converse of the problem as follows: Take a point $D$ on $\\Gamma$, and let $G \\in \\Gamma$ such that $\\overline{D G} \\perp \\overline{B C}$. Then define $K$ to lie on $\\overline{G H}, \\overline{A D}$, and take $L \\in \\overline{A D}$ such that $K$ is the midpoint of $\\overline{A L}$. Then if we define $E$ and $F$ as the projections of $L$ onto $\\overline{A B}$ and $\\overline{A C}$ we want to show that $E, H, F$ are collinear. It's clear that solving this problem will solve the original. In fact we will show later that each line $E F$ through $H$ corresponds bijectively to the point $D$. We animate $D$ projectively on $\\Gamma$ (hence $\\operatorname{deg} D=2$ ). Since $D \\mapsto G$ is a projective map $\\Gamma \\rightarrow \\Gamma$, it follows $\\operatorname{deg} G=2$. By Zack's lemma, $\\operatorname{deg}(\\overline{A D}) \\leq 0+2-1=1$ (since $D$ can coincide with $A$ ), and $\\operatorname{deg}(\\overline{H G}) \\leq 0+2-0=2$. So again by Zack's lemma, $\\operatorname{deg} K \\leq 1+2-1=2$, since lines $A D$ and $G H$ can coincide once if $D$ is the reflection of $H$ over $\\overline{B C}$. It follows $\\operatorname{deg} L=2$, since it is obtained by dilating $K$ by a factor of 2 across the fixed point $A$. Let $\\infty_{C}$ be the point at infinity on the line perpendicular to $A C$, and similarly $\\infty_{B}$. Then $$ F=\\overline{A C} \\cap \\overline{\\infty_{C} L}, \\quad E=\\overline{A B} \\cap \\overline{\\infty_{B} L} . $$ We want to use Zack's lemma again on line $\\overline{\\infty_{B} L}$. Consider the case $G=B$; we get $\\overline{H G} \\| \\overline{A D}$, so $A D G H$ is a parallelogram, and then $K=L=\\infty_{B}$. Thus there is at least one $t$ where $L=\\infty_{B}$ and by Zack's lemma we get $\\operatorname{deg}\\left(\\overline{\\infty_{B} L}\\right) \\leq 0+2-1=1$. Again by Zack's lemma, we conclude $\\operatorname{deg} E \\leq 0+1-0=1$. Similarly, $\\operatorname{deg} F \\leq 1$. We were aiming to show $E, F, H$ collinear which is a condition of degree at most $1+1+0=2$. So it suffices to verify the problem for three distinct choices of $D$. - If $D=A$, then line $G H$ is line $A H$, and $L=\\overline{A D} \\cap \\overline{A H}=A$. So $E=F=A$ and the statement is true. - If $D=B, G$ is the antipode of $C$ on $\\Gamma$. Then $K=\\overline{H G} \\cap \\overline{A D}$ is the midpoint of $\\overline{A B}$, so $L=B$. Then $E=B$ and $F$ is the projection of $B$ onto $A C$, so $E, H, F$ collinear. - We finish similarly when $D=C$."} +{"year":2019,"label":"6","problem":"Suppose $P$ is a polynomial with integer coefficients such that for every positive integer $n$, the sum of the decimal digits of $|P(n)|$ is not a Fibonacci number. Must $P$ be constant?","solution":" The answer is yes, $P$ must be constant. By $S(n)$ we mean the sum of the decimal digits of $|n|$. We need two claims. Claim - If $P(x) \\in \\mathbb{Z}[x]$ is nonconstant with positive leading coefficient, then there exists an integer polynomial $F(x)$ such that all coefficients of $P \\circ F$ are positive except for the second one, which is negative. First, consider $T_{0}(x)=x^{3}+x+1$. Observe that in $T_{0}^{\\operatorname{deg} P}$, every coefficient is strictly positive, except for the second one, which is zero. Then, let $T_{1}(x)=x^{3}-\\frac{1}{D} x^{2}+x+1$. Using continuity as $D \\rightarrow \\infty$, it follows that if $D$ is large enough (in terms of $\\operatorname{deg} P$ ), then $T_{1}^{\\operatorname{deg} P}$ is good, with $-\\frac{3}{D} x^{3 \\operatorname{deg} P-1}$ being the only negative coefficient. Finally, we can let $F(x)=C T_{1}(x)$ where $C$ is a sufficiently large multiple of $D$ (in terms of the coefficients of $P$ ); thus the coefficients of $\\left(C T_{1}(x)\\right)^{\\operatorname{deg} P}$ dominate (and are integers), as needed. Claim - There are infinitely many Fibonacci numbers in each residue class modulo 9. $$ \\begin{aligned} F_{0}=0 & \\equiv 0 \\quad(\\bmod 9) \\\\ F_{1}=1 & \\equiv 1 \\quad(\\bmod 9) \\\\ F_{3}=2 & \\equiv 2 \\quad(\\bmod 9) \\\\ F_{4}=3 & \\equiv 3 \\quad(\\bmod 9) \\\\ F_{7}=13 & \\equiv 4 \\quad(\\bmod 9) \\\\ F_{5}=5 & \\equiv 5 \\quad(\\bmod 9) \\\\ F_{-4}=-3 & \\equiv 6 \\quad(\\bmod 9) \\\\ F_{9}=34 & \\equiv 7 \\quad(\\bmod 9) \\\\ F_{6}=8 & \\equiv 8 \\quad(\\bmod 9) . \\end{aligned} $$ We now show how to solve the problem with the two claims. WLOG $P$ satisfies the conditions of the first claim, and choose $F$ as above. Let $$ P(F(x))=c_{N} x^{N}-c_{N-1} x^{N-1}+c_{N-2} x^{N-2}+\\cdots+c_{0} $$ where $c_{i}>0$ (and $N=3 \\operatorname{deg} P$ ). Then if we select $x=10^{e}$ for $e$ large enough (say $\\left.x>10 \\max _{i} c_{i}\\right)$, the decimal representation $P\\left(F\\left(10^{e}\\right)\\right)$ consists of the concatenation of - the decimal representation of $c_{N}-1$, - the decimal representation of $10^{e}-c_{N-1}$ - the decimal representation of $c_{N-2}$, with several leading zeros, - the decimal representation of $c_{N-3}$, with several leading zeros, - ... - the decimal representation of $c_{0}$, with several leading zeros. (For example, if $P(F(x))=15 x^{3}-7 x^{2}+4 x+19$, then $P(F(1000))=14,993,004,019$.) Thus, the sum of the digits of this expression is equal to $$ S\\left(P\\left(F\\left(10^{e}\\right)\\right)\\right)=9 e+k $$ for some constant $k$ depending only on $P$ and $F$, independent of $e$. But this will eventually hit a Fibonacci number by the second claim, contradiction. Remark. It is important to control the number of negative coefficients in the created polynomial. If one tries to use this approach on a polynomial $P$ with $m>0$ negative coefficients, then one would require that the Fibonacci sequence is surjective modulo $9 m$ for any $m>1$, which is not true: for example the Fibonacci sequence avoids all numbers congruent to $4 \\bmod 11($ and thus $4 \\bmod 99)$. In bases $b$ for which surjectivity modulo $b-1$ fails, the problem is false. For example, $P(x)=11 x+4$ will avoid all Fibonacci numbers if we take sum of digits in base 12, since that base-12 sum is necessarily $4(\\bmod 11)$, hence not a Fibonacci number."} +{"year":2019,"label":"7","problem":"Let $f: \\mathbb{Z} \\rightarrow\\left\\{1,2, \\ldots, 10^{100}\\right\\}$ be a function satisfying $$ \\operatorname{gcd}(f(x), f(y))=\\operatorname{gcd}(f(x), x-y) $$ for all integers $x$ and $y$. Show that there exist positive integers $m$ and $n$ such that $f(x)=\\operatorname{gcd}(m+x, n)$ for all integers $x$.","solution":" Let $\\mathcal{P}$ be the set of primes not exceeding $10^{100}$. For each $p \\in \\mathcal{P}$, let $e_{p}=\\max _{x} \\nu_{p}(f(x))$ and let $c_{p} \\in \\operatorname{argmax}_{x} \\nu_{p}(f(x))$. We show that this is good enough to compute all values of $x$, by looking at the exponent at each individual prime. Claim - For any $p \\in \\mathcal{P}$, we have $$ \\nu_{p}(f(x))=\\min \\left(\\nu_{p}\\left(x-c_{p}\\right), e_{p}\\right) $$ $$ \\operatorname{gcd}\\left(f\\left(c_{p}\\right), f(x)\\right)=\\operatorname{gcd}\\left(f\\left(c_{p}\\right), x-c_{p}\\right) $$ We then take $\\nu_{p}$ of both sides and recall $\\nu_{p}(f(x)) \\leq \\nu_{p}\\left(f\\left(c_{p}\\right)\\right)=e_{p}$; this implies the result. This essentially determines $f$, and so now we just follow through. Choose $n$ and $m$ such that $$ \\begin{aligned} n & =\\prod_{p \\in \\mathcal{P}} p^{e_{p}} \\\\ m & \\equiv-c_{p} \\quad\\left(\\bmod p^{e_{p}}\\right) \\quad \\forall p \\in \\mathcal{P} \\end{aligned} $$ the latter being possible by Chinese remainder theorem. Then, from the claim we have $$ \\begin{aligned} f(x) & =\\prod_{p \\in \\mathcal{P}} p^{\\nu_{p}(f(x))}=\\prod_{p \\mid n} p^{\\min \\left(\\nu_{p}\\left(x-c_{p}\\right), e_{p}\\right)} \\\\ & =\\prod_{p \\mid n} p^{\\min \\left(\\nu_{p}(x+m), \\nu_{p}(n)\\right)}=\\operatorname{gcd}(x+m, n) \\end{aligned} $$ for every $x \\in \\mathbb{Z}$, as desired. Remark. The functions $f(x)=x$ and $f(x)=|2 x-1|$ are examples satisfying the gcd equation (the latter always being strictly positive). Hence the hypothesis $f$ bounded cannot be dropped. Remark. The pair $(m, n)$ is essentially unique: every other pair is obtained by shifting $m$ by a multiple of $n$. Hence there is not really any choice in choosing $m$ and $n$."} +{"year":2019,"label":"8","problem":"Let $\\mathcal{S}$ be a set of 16 points in the plane, no three collinear. Let $\\chi(\\mathcal{S})$ denote the number of ways to draw 8 line segments with endpoints in $\\mathcal{S}$, such that no two drawn segments intersect, even at endpoints. Find the smallest possible value of $\\chi(\\mathcal{S})$ across all such $\\mathcal{S}$.","solution":" The answer is 1430 . In general, we prove that with $2 n$ points the answer is the $n^{\\text {th }}$ Catalan number $C_{n}=\\frac{1}{n+1}\\binom{2 n}{n}$. First of all, it is well-known that if $\\mathcal{S}$ is a convex $2 n$-gon, then $\\chi(\\mathcal{S})=C_{n}$. It remains to prove the lower bound. We proceed by (strong) induction on $n$, with the base case $n=0$ and $n=1$ clear. Suppose the statement is proven for $0,1, \\ldots, n$ and consider a set $\\mathcal{S}$ with $2(n+1)$ points. Let $P$ be a point on the convex hull of $\\mathcal{S}$, and label the other $2 n+1$ points $A_{1}, \\ldots, A_{2 n+1}$ in order of angle from $P$. Consider drawing a segment $\\overline{P A_{2 k+1}}$. This splits the $2 n$ remaining points into two halves $\\mathcal{U}$ and $\\mathcal{V}$, with $2 k$ and $2(n-k)$ points respectively. ![](https:\/\/cdn.mathpix.com\/cropped\/2024_11_19_f4f672a97b66743213a8g-23.jpg?height=581&width=815&top_left_y=1297&top_left_x=626) Note that by choice of $P$, no segment in $\\mathcal{U}$ can intersect a segment in $\\mathcal{V}$. By the inductive hypothesis, $$ \\chi(\\mathcal{U}) \\geq C_{k} \\quad \\text { and } \\quad \\chi(\\mathcal{V}) \\geq C_{n-k} $$ Thus, drawing $\\overline{P A_{2 k+1}}$, we have at least $C_{k} C_{n-k}$ ways to complete the drawing. Over all choices of $k$, we obtain $$ \\chi(\\mathcal{S}) \\geq C_{0} C_{n}+\\cdots+C_{n} C_{0}=C_{n+1} $$ as desired. Remark. It is possible to show directly from the lower bound proof that convex $2 n$-gons achieve the minimum: indeed, every inequality is sharp, and no segment $\\overline{P A_{2 k}}$ can be drawn (since this splits the rest of the points into two halves with an odd number of points, and no crossing segment can be drawn). Bobby Shen points out that in the case of 6 points, a regular pentagon with its center also achieves equality, so this is not the only equality case. Remark. The result that $\\chi(S) \\geq 1$ for all $S$ is known (consider the choice of 8 segments with smallest sum), and appeared on Putnam 1979. However, it does not seem that knowing this gives an advantage for this problem, since the answer is much larger than 1."} +{"year":2019,"label":"9","problem":"Let $A B C$ be a triangle with incenter $I$. Points $K$ and $L$ are chosen on segment $B C$ such that the incircles of $\\triangle A B K$ and $\\triangle A B L$ are tangent at $P$, and the incircles of $\\triangle A C K$ and $\\triangle A C L$ are tangent at $Q$. Prove that $I P=I Q$.","solution":" \u3010 First solution, mostly elementary (original). Let $I_{B}, J_{B}, I_{C}, J_{C}$ be the incenters of $\\triangle A B K, \\triangle A B L, \\triangle A C K, \\triangle A C L$ respectively. ![](https:\/\/cdn.mathpix.com\/cropped\/2024_11_19_f4f672a97b66743213a8g-25.jpg?height=515&width=1332&top_left_y=973&top_left_x=359) We begin with the following claim which does not depend on the existence of tangency points $P$ and $Q$. Claim - Lines $B C, I_{B} J_{C}, J_{B} I_{C}$ meet at a point $R$ (possibly at infinity). $$ A\\left(B I ; I_{B} J_{B}\\right)=A\\left(I C ; I_{C} J_{C}\\right) . $$ It follows $\\left(B I ; I_{B} J_{B}\\right)=\\left(I C ; I_{C} J_{C}\\right)=\\left(C I ; J_{C} I_{C}\\right)$. (One could also check directly that both cross ratios equal $\\frac{\\sin \\angle B A K \/ 2}{\\sin \\angle C A K \/ 2} \\div \\frac{\\sin \\angle B A L \/ 2}{\\sin \\angle C A L \/ 2}$, rather than using rotation.) Therefore, the concurrence follows from the so-called prism lemma on $\\overline{I B I_{B} J_{B}}$ and $\\overline{I C J_{C} I_{C}}$. Remark (Nikolai Beluhov). This result is known; it appears as 4.5.32 in Akopyan's Geometry in Figures. The cross ratio is not necessary to prove this claim: it can be proven by length chasing with circumscribed quadrilaterals. (The generalization mentioned later also admits a trig-free proof for the analogous step.) We now bring $P$ and $Q$ into the problem. Claim - Line $P Q$ also passes through $R$. $$ \\frac{I_{B} P}{P I} \\cdot \\frac{I Q}{Q J_{C}} \\cdot \\frac{J_{C} R}{R I_{B}}=-1 \\Longrightarrow I P=I Q $$ Remark (Author's comments on drawing the diagram). Drawing the diagram directly is quite difficult. If one draws $\\triangle A B C$ first, they must locate both $K$ and $L$, which likely involves some trial and error due to the complex interplay between the two points. There are alternative simpler ways. For example, one may draw $\\triangle A K L$ first; then the remaining points $B$ and $C$ are not related and the task is much simpler (though some trial and error is still required). In fact, by breaking symmetry, we may only require one application of guesswork. Start by drawing $\\triangle A B K$ and its incircle; then the incircle of $\\triangle A B L$ may be constructed, and so point $L$ may be drawn. Thus only the location of point $C$ needs to be guessed. I would be interested in a method to create a general diagram without any trial and error."} +{"year":2019,"label":"9","problem":"Let $A B C$ be a triangle with incenter $I$. Points $K$ and $L$ are chosen on segment $B C$ such that the incircles of $\\triangle A B K$ and $\\triangle A B L$ are tangent at $P$, and the incircles of $\\triangle A C K$ and $\\triangle A C L$ are tangent at $Q$. Prove that $I P=I Q$.","solution":" \u3010 Second solution, inversion (Nikolai Beluhov). As above, the lines $B C, I_{B} J_{C}, J_{B} I_{C}$ meet at some point $R$ (possibly at infinity). Let $\\omega_{1}, \\omega_{2}, \\omega_{3}, \\omega_{4}$ be the incircles of $\\triangle A B K$, $\\triangle A C L, \\triangle A B L$, and $\\triangle A C K$. Claim - There exists an inversion $\\iota$ at $R$ swapping $\\left\\{\\omega_{1}, \\omega_{2}\\right\\}$ and $\\left\\{\\omega_{3}, \\omega_{4}\\right\\}$. Claim - Circles $\\omega_{1}, \\omega_{2}, \\omega_{3}, \\omega_{4}$ share a common radical center. Then $k$ is actually orthogonal to $\\Omega, \\omega_{1}, \\omega_{3}$, so $k$ is fixed under $\\iota$ and $k$ is also orthogonal to $\\omega_{2}$ and $\\omega_{4}$. Thus the center of $k$ is the desired radical center. The desired statement immediately follows. Indeed, letting $S$ be the radical center, it follows that $\\overline{S P}$ and $\\overline{S Q}$ are the common internal tangents to $\\left\\{\\omega_{1}, \\omega_{3}\\right\\}$ and $\\left\\{\\omega_{2}, \\omega_{4}\\right\\}$. Since $S$ is the radical center, $S P=S Q$. In light of $\\angle S P I=\\angle S Q I=90^{\\circ}$, it follows that $I P=I Q$, as desired. Remark (Nikolai Beluhov). There exists a circle tangent to all four incircles, because circle $k$ is orthogonal to all four, and line $B C$ is tangent to all four; thus the inverse of line $B C$ in $k$ is a circle tangent to all four incircles. The amusing thing here is that Casey's theorem is completely unhelpful for proving this fact: all it can tell us is that there is a line or circle tangent to these incircles, and line $B C$ already satisfies this property. Remark (Generalization by Nikolai Beluhov). The following generalization holds: Let $A B C D$ be a quadrilateral circumscribed about a circle with center $I$. A line through $A$ meets $\\overrightarrow{B C}$ and $\\overrightarrow{D C}$ at $K$ and $L$; another line through $A$ meets $\\overrightarrow{B C}$ and $\\overrightarrow{D C}$ at $M$ and $N$. Suppose that the incircles of $\\triangle A B K$ and $\\triangle A B M$ are tangent at $P$, and the incircles of $\\triangle A C L$ and $\\triangle A C N$ are tangent at $Q$. Prove that $I P=I Q$."} diff --git a/USA_TSTST/segmented/en-sols-TSTST-2020.jsonl b/USA_TSTST/segmented/en-sols-TSTST-2020.jsonl new file mode 100644 index 0000000000000000000000000000000000000000..d24cdafe67eca9df22e9c91c81fbdb06fc338759 --- /dev/null +++ b/USA_TSTST/segmented/en-sols-TSTST-2020.jsonl @@ -0,0 +1,10 @@ +{"year":2020,"label":"1","problem":"Let $a, b, c$ be fixed positive integers. There are $a+b+c$ ducks sitting in a circle, one behind the other. Each duck picks either rock, paper, or scissors, with $a$ ducks picking rock, $b$ ducks picking paper, and $c$ ducks picking scissors. A move consists of an operation of one of the following three forms: - If a duck picking rock sits behind a duck picking scissors, they switch places. - If a duck picking paper sits behind a duck picking rock, they switch places. - If a duck picking scissors sits behind a duck picking paper, they switch places. Determine, in terms of $a, b$, and $c$, the maximum number of moves which could take place, over all possible initial configurations.","solution":" The maximum possible number of moves is $\\max (a b, a c, b c)$. First, we prove this is best possible. We define a feisty triplet to be an unordered triple of ducks, one of each of rock, paper, scissors, such that the paper duck is between the rock and scissors duck and facing the rock duck, as shown. (There may be other ducks not pictured, but the orders are irrelevant.) ![](https:\/\/cdn.mathpix.com\/cropped\/2024_11_19_a8f70a0d08e0809deba0g-04.jpg?height=1100&width=1098&top_left_y=1500&top_left_x=479) Claim - The number of feisty triplets decreases by $c$ if a paper duck swaps places with a rock duck, and so on. Obviously the number of feisty triples is at most $a b c$ to start. Thus at most $\\max (a b, b c, c a)$ moves may occur, since the number of feisty triplets should always be nonnegative, at which point no moves are possible at all. To see that this many moves is possible, assume WLOG $a=\\min (a, b, c)$ and suppose we have $a$ rocks, $b$ papers, and $c$ scissors in that clockwise order. ![](https:\/\/cdn.mathpix.com\/cropped\/2024_11_19_a8f70a0d08e0809deba0g-05.jpg?height=1109&width=1052&top_left_y=750&top_left_x=505) Then, allow the scissors to filter through the papers while the rocks stay put. Each of the $b$ papers swaps with $c$ scissors, for a total of $b c=\\max (a b, a c, b c)$ swaps. Remark (Common errors). One small possible mistake: it is not quite k\u00f6sher to say that \"WLOG $a \\leq b \\leq c$ \" because the condition is not symmetric, only cyclic. Therefore in this solution we only assume $a=\\min (a, b, c)$. It is true here that every pair of ducks swaps at most once, and some solutions make use of this fact. However, this fact implicitly uses the fact that $a, b, c>0$ and is false without this hypothesis."} +{"year":2020,"label":"2","problem":"Let $A B C$ be a scalene triangle with incenter $I$. The incircle of $A B C$ touches $\\overline{B C}$, $\\overline{C A}, \\overline{A B}$ at points $D, E, F$, respectively. Let $P$ be the foot of the altitude from $D$ to $\\overline{E F}$, and let $M$ be the midpoint of $\\overline{B C}$. The rays $A P$ and $I P$ intersect the circumcircle of triangle $A B C$ again at points $G$ and $Q$, respectively. Show that the incenter of triangle $G Q M$ coincides with $D$.","solution":" Refer to the figure below. ![](https:\/\/cdn.mathpix.com\/cropped\/2024_11_19_a8f70a0d08e0809deba0g-06.jpg?height=798&width=803&top_left_y=906&top_left_x=632) Claim - The point $Q$ is the Miquel point of $B F E C$. Also, $\\overline{Q D}$ bisects $\\angle B Q C$. The spiral similarity mentioned then gives $\\frac{Q B}{B F}=\\frac{Q C}{C E}$, so $\\overline{Q D}$ bisects $\\angle B Q C$. Remark. The point $Q$ and its properties mentioned in the first claim have appeared in other references. See for example Canada 2007\/5, ELMO 2010\/6, HMMT 2016 T-10, USA TST 2017\/2, USA TST 2019\/6 for a few examples. Claim - We have $(Q G ; B C)=-1$, so in particular $\\overline{G D}$ bisects $\\angle B G C$. $$ -1=(A I ; E F) \\stackrel{Q}{=}(\\overline{A Q} \\cap \\overline{E F}, P ; E, F) \\stackrel{A}{=}(Q G ; B C) $$ The last statement follows from Apollonian circle, or more bluntly $\\frac{G B}{G C}=\\frac{Q B}{Q C}=\\frac{B D}{D C}$. Hence $\\overline{Q D}$ and $\\overline{G D}$ are angle bisectors of $\\angle B Q C$ and $\\angle B G C$. However, $\\overline{Q M}$ and $\\overline{Q G}$ are isogonal in $\\angle B Q C$ (as median and symmedian), and similarly for $\\angle B G C$, as desired."} +{"year":2020,"label":"3","problem":"We say a nondegenerate triangle whose angles have measures $\\theta_{1}, \\theta_{2}, \\theta_{3}$ is quirky if there exists integers $r_{1}, r_{2}, r_{3}$, not all zero, such that $$ r_{1} \\theta_{1}+r_{2} \\theta_{2}+r_{3} \\theta_{3}=0 $$ Find all integers $n \\geq 3$ for which a triangle with side lengths $n-1, n, n+1$ is quirky.","solution":" The answer is $n=3,4,5,7$. We first introduce a variant of the $k$ th Chebyshev polynomials in the following lemma (which is standard, and easily shown by induction). ## Lemma For each $k \\geq 0$ there exists $P_{k}(X) \\in \\mathbb{Z}[X]$, monic for $k \\geq 1$ and with degree $k$, such that $$ P_{k}\\left(X+X^{-1}\\right) \\equiv X^{k}+X^{-k} . $$ The first few are $P_{0}(X) \\equiv 2, P_{1}(X) \\equiv X, P_{2}(X) \\equiv X^{2}-2, P_{3}(X) \\equiv X^{3}-3 X$. Suppose the angles of the triangle are $\\alpha<\\beta<\\gamma$, so the law of cosines implies that $$ 2 \\cos \\alpha=\\frac{n+4}{n+1} \\quad \\text { and } \\quad 2 \\cos \\gamma=\\frac{n-4}{n-1} $$ Claim - The triangle is quirky iff there exists $r, s \\in \\mathbb{Z}_{\\geq 0}$ not both zero such that $$ \\cos (r \\alpha)= \\pm \\cos (s \\gamma) \\quad \\text { or equivalently } \\quad P_{r}\\left(\\frac{n+4}{n+1}\\right)= \\pm P_{s}\\left(\\frac{n-4}{n-1}\\right) $$ If $r=0$, then by rational root theorem on $P_{s}(X) \\pm 2$ it follows $\\frac{n-4}{n-1}$ must be an integer which occurs only when $n=4$ (recall $n \\geq 3$ ). Similarly we may discard the case $s=0$. Thus in what follows assume $n \\neq 4$ and $r, s>0$. Then, from the fact that $P_{r}$ and $P_{s}$ are nonconstant monic polynomials, we find ## Corollary If $n \\neq 4$ works, then when $\\frac{n+4}{n+1}$ and $\\frac{n-4}{n-1}$ are written as fractions in lowest terms, the denominators have the same set of prime factors. But $\\operatorname{gcd}(n+1, n-1)$ divides 2 , and $\\operatorname{gcd}(n+4, n+1), \\operatorname{gcd}(n-4, n-1)$ divide 3 . So we only have three possibilities: - $n+1=2^{u}$ and $n-1=2^{v}$ for some $u, v \\geq 0$. This is only possible if $n=3$. Here $2 \\cos \\alpha=\\frac{7}{4}$ and $2 \\cos \\gamma=-\\frac{1}{2}$, and indeed $P_{2}(-1 \/ 2)=-7 \/ 4$. - $n+1=3 \\cdot 2^{u}$ and $n-1=2^{v}$ for some $u, v \\geq 0$, which implies $n=5$. Here $2 \\cos \\alpha=\\frac{3}{2}$ and $2 \\cos \\gamma=\\frac{1}{4}$, and indeed $P_{2}(3 \/ 2)=1 \/ 4$. - $n+1=2^{u}$ and $n-1=3 \\cdot 2^{v}$ for some $u, v \\geq 0$, which implies $n=7$. Here $2 \\cos \\alpha=\\frac{11}{8}$ and $2 \\cos \\gamma=\\frac{1}{2}$, and indeed $P_{3}(1 \/ 2)=-11 \/ 8$. Finally, $n=4$ works because the triangle is right, completing the solution. Remark (Major generalization due to Luke Robitaille). In fact one may find all quirky triangles whose sides are integers in arithmetic progression. Indeed, if the side lengths of the triangle are $x-y, x, x+y$ with $\\operatorname{gcd}(x, y)=1$ then the problem becomes $$ P_{r}\\left(\\frac{x+4 y}{x+y}\\right)= \\pm P_{s}\\left(\\frac{x-4 y}{x-y}\\right) $$ and so in the same way as before, we ought to have $x+y$ and $x-y$ are both of the form $3 \\cdot 2^{*}$ unless $r s=0$. This time, when $r s=0$, we get the extra solutions $(1,0)$ and $(5,2)$. For $r s \\neq 0$, by triangle inequality, we have $x-y \\leq x+y<3(x-y)$, and $\\min \\left(\\nu_{2}(x-\\right.$ $\\left.y), \\nu_{2}(x+y)\\right) \\leq 1$, so it follows one of $x-y$ or $x+y$ must be in $\\{1,2,3,6\\}$. An exhaustive check then leads to $$ (x, y) \\in\\{(3,1),(5,1),(7,1),(11,5)\\} \\cup\\{(1,0),(5,2),(4,1)\\} $$ In conclusion the equilateral triangle, $3-5-7$ triangle (which has a $120^{\\circ}$ angle) and $6-11-16$ triangle (which satisfies $B=3 A+4 C$ ) are exactly the new quirky triangles (up to similarity) whose sides are integers in arithmetic progression."} +{"year":2020,"label":"4","problem":"Find all pairs of positive integers $(a, b)$ satisfying the following conditions: (i) $a$ divides $b^{4}+1$, (ii) $b$ divides $a^{4}+1$, (iii) $\\lfloor\\sqrt{a}\\rfloor=\\lfloor\\sqrt{b}\\rfloor$.","solution":" Obviously, $\\operatorname{gcd}(a, b)=1$, so the problem conditions imply $$ a b \\mid(a-b)^{4}+1 $$ since each of $a$ and $b$ divide the right-hand side. We define $$ k \\stackrel{\\text { def }}{=} \\frac{(b-a)^{4}+1}{a b} . $$ Claim (Size estimate) \u2014 We must have $k \\leq 16$. $$ \\begin{aligned} a b & \\geq n^{2}\\left(n^{2}+1\\right) \\geq n^{4}+1 \\\\ (b-a)^{4}+1 & \\leq(2 n)^{4}+1=16 n^{4}+1 \\end{aligned} $$ which shows $k \\leq 16$. Claim (Orders argument) - In fact, $k=1$. Thus $k$ is odd. However, every odd prime divisor of $(b-a)^{4}+1$ is congruent to 1 $(\\bmod 8)$ and is thus at least 17 , so $k=1$ or $k \\geq 17$. It follows that $k=1$. At this point, we have reduced to solving $$ a b=(b-a)^{4}+1 $$ $$ a^{2}-d a-\\left(d^{4}+1\\right)=0 $$ The discriminant $d^{2}+4\\left(d^{4}+1\\right)=4 d^{4}+d^{2}+4$ must be a perfect square. - The cases $d=0$ and $d=1$ lead to pairs $(1,1)$ and $(1,2)$. - If $d \\geq 2$, then we can sandwich $$ \\left(2 d^{2}\\right)^{2}<4 d^{4}+d^{2}+4<4 d^{4}+4 d^{2}+1=\\left(2 d^{2}+1\\right)^{2} $$ so the discriminant is not a square. Remark (Author remarks on origin). This comes from the problem of the existence of a pair of elliptic curves over $\\mathbb{F}_{a}, \\mathbb{F}_{b}$ respectively, such that the number of points on one is the field size of the other. The bound $n^{2} \\leq a, b<(n+1)^{2}$ is the Hasse bound. The divisibility conditions correspond to asserting that the embedding degree of each curve is 8 , so that they are pairing friendly. In this way, the problem is essentially the key result of https:\/\/arxiv.org\/pdf\/1803.02067.pdf, shown in Proposition 3."} +{"year":2020,"label":"5","problem":"Let $\\mathbb{N}^{2}$ denote the set of ordered pairs of positive integers. A finite subset $S$ of $\\mathbb{N}^{2}$ is stable if whenever $(x, y)$ is in $S$, then so are all points $\\left(x^{\\prime}, y^{\\prime}\\right)$ of $\\mathbb{N}^{2}$ with both $x^{\\prime} \\leq x$ and $y^{\\prime} \\leq y$. Prove that if $S$ is a stable set, then among all stable subsets of $S$ (including the empty set and $S$ itself), at least half of them have an even number of elements.","solution":" Suppose $|S| \\geq 2$. For any $p \\in S$, let $R(p)$ denote the stable rectangle with upper-right corner $p$. We say such $p$ is pivotal if $p+(1,1) \\notin S$ and $|R(p)|$ is even. ![](https:\/\/cdn.mathpix.com\/cropped\/2024_11_19_a8f70a0d08e0809deba0g-12.jpg?height=455&width=818&top_left_y=1069&top_left_x=622) Claim - If $|S| \\geq 2$, then a pivotal $p$ always exists. - If it has length at least 2 , one of the two rightmost points in it is pivotal. - Otherwise, the top row has length 1. Now either the top point or the point below it (which exists as $|S| \\geq 2$ ) is pivotal. We describe how to complete the induction, given some pivotal $p \\in S$. There is a partition $$ S=R(p) \\sqcup S_{1} \\sqcup S_{2} $$ where $S_{1}$ and $S_{2}$ are the sets of points in $S$ above and to the right of $p$ (possibly empty). Claim - The desired inequality holds for stable subsets containing $p$. Since $|R(p)|$ is even, exactly $E_{1} E_{2}+O_{1} O_{2}$ stable subsets containing $p$ are even, and exactly $E_{1} O_{2}+E_{2} O_{1}$ are odd. As $E_{1} \\geq O_{1}$ and $E_{2} \\geq O_{2}$ by inductive hypothesis, we obtain $E_{1} E_{2}+O_{1} O_{2} \\geq E_{1} O_{2}+E_{2} O_{1}$ as desired. By the inductive hypothesis, the desired inequality also holds for stable subsets not containing $p$, so we are done."} +{"year":2020,"label":"6","problem":"Let $A, B, C, D$ be four points such that no three are collinear and $D$ is not the orthocenter of triangle $A B C$. Let $P, Q, R$ be the orthocenters of $\\triangle B C D, \\triangle C A D$, $\\triangle A B D$, respectively. Suppose that lines $A P, B Q, C R$ are pairwise distinct and are concurrent. Show that the four points $A, B, C, D$ lie on a circle.","solution":" Let $T$ be the concurrency point, and let $H$ be the orthocenter of $\\triangle A B C$. ![](https:\/\/cdn.mathpix.com\/cropped\/2024_11_19_a8f70a0d08e0809deba0g-14.jpg?height=593&width=806&top_left_y=863&top_left_x=631) Claim (Key claim) - $T$ is the midpoint of $\\overline{A P}, \\overline{B Q}, \\overline{C R}, \\overline{D H}$, and $D$ is the orthocenter of $\\triangle P Q R$. By symmetric reasoning, we get that $A Q C P B R$ is a hexagon with opposite sides parallel and concurrent diagonals as $\\overline{A P}, \\overline{B Q}, \\overline{C R}$ meet at $T$. This implies that the hexagon is centrally symmetric about $T$; indeed $$ \\frac{A T}{T P}=\\frac{T Q}{B T}=\\frac{C T}{T R}=\\frac{T P}{A T} $$ so all the ratios are equal to +1 . Next, $\\overline{P D} \\perp \\overline{B C} \\| \\overline{Q R}$, so by symmetry we get $D$ is the orthocenter of $\\triangle P Q R$. This means that $T$ is the midpoint of $\\overline{D H}$ as well. ## Corollary The configuration is now symmetric: we have four points $A, B, C, D$, and their reflections in $T$ are four orthocenters $P, Q, R, H$. Let $S$ be the centroid of $\\{A, B, C, D\\}$, and let $O$ be the reflection of $T$ in $S$. We are ready to conclude: Claim - $A, B, C, D$ are equidistant from $O$. Then $T^{\\prime}$ is the midpoint of $\\overline{A^{\\prime} D^{\\prime}}$, so $S^{\\prime}=\\frac{1}{4}\\left(A^{\\prime}+D^{\\prime}+B+C\\right)$ gives that $O^{\\prime}$ is the midpoint of $\\overline{B C}$. Thus $O B=O C$ and we're done."} +{"year":2020,"label":"7","problem":"Find all nonconstant polynomials $P(z)$ with complex coefficients for which all complex roots of the polynomials $P(z)$ and $P(z)-1$ have absolute value 1.","solution":" The answer is $P(x)$ should be a polynomial of the form $P(x)=\\lambda x^{n}-\\mu$ where $|\\lambda|=|\\mu|$ and $\\operatorname{Re} \\mu=-\\frac{1}{2}$. One may check these all work; let's prove they are the only solutions. \\l First approach (Evan Chen). We introduce the following notations: $$ \\begin{aligned} P(x) & =c_{n} x^{n}+c_{n-1} x^{n-1}+\\cdots+c_{1} x+c_{0} \\\\ & =c_{n}\\left(x+\\alpha_{1}\\right) \\ldots\\left(x+\\alpha_{n}\\right) \\\\ P(x)-1 & =c_{n}\\left(x+\\beta_{1}\\right) \\ldots\\left(x+\\beta_{n}\\right) \\end{aligned} $$ By taking conjugates, $$ \\begin{aligned} \\left(x+\\alpha_{1}\\right) \\cdots\\left(x+\\alpha_{n}\\right) & =\\left(x+\\beta_{1}\\right) \\cdots\\left(x+\\beta_{n}\\right)+c_{n}^{-1} \\\\ \\Longrightarrow\\left(x+\\frac{1}{\\alpha_{1}}\\right) \\cdots\\left(x+\\frac{1}{\\alpha_{n}}\\right) & =\\left(x+\\frac{1}{\\beta_{1}}\\right) \\cdots\\left(x+\\frac{1}{\\beta_{n}}\\right)+\\left(\\overline{c_{n}}\\right)^{-1} \\end{aligned} $$ The equation $(\\boldsymbol{\\oplus})$ is the main player: Claim - We have $c_{k}=0$ for all $k=1, \\ldots, n-1$. $$ \\frac{c_{n-k}}{\\prod_{i} \\alpha_{i}}=\\frac{c_{n-k}}{\\prod_{i} \\beta_{i}} $$ but $\\prod_{i} \\alpha_{i}-\\prod_{i} \\beta_{i}=\\frac{1}{c_{n}} \\neq 0$. Hence $c_{k}=0$. It follows that $P(x)$ must be of the form $P(x)=\\lambda x^{n}-\\mu$, so that $P(x)=\\lambda x^{n}-(\\mu+1)$. This requires $|\\mu|=|\\mu+1|=|\\lambda|$ which is equivalent to the stated part."} +{"year":2020,"label":"7","problem":"Find all nonconstant polynomials $P(z)$ with complex coefficients for which all complex roots of the polynomials $P(z)$ and $P(z)-1$ have absolute value 1.","solution":" The answer is $P(x)$ should be a polynomial of the form $P(x)=\\lambda x^{n}-\\mu$ where $|\\lambda|=|\\mu|$ and $\\operatorname{Re} \\mu=-\\frac{1}{2}$. One may check these all work; let's prove they are the only solutions. II Second approach (from the author). We let $A=P$ and $B=P-1$ to make the notation more symmetric. We will as before show that $A$ and $B$ have all coefficients equal to zero other than the leading and constant coefficient; the finish is the same. First, we rule out double roots. Claim - Neither $A$ nor $B$ have double roots. Let $\\omega=e^{2 \\pi i \/ n}$, let $a_{1}, \\ldots, a_{n}$ be the roots of $A$, and let $b_{1}, \\ldots, b_{n}$ be the roots of $B$. For each $k$, let $A_{k}$ and $B_{k}$ be the points in the complex plane corresponding to $a_{k}$ and $b_{k}$ \u3002 Claim (Main claim) - For any $i$ and $j, \\frac{a_{i}}{a_{j}}$ is a power of $\\omega$. $$ \\frac{a_{i}-b_{1}}{a_{j}-b_{1}} \\cdots \\frac{a_{i}-b_{n}}{a_{j}-b_{n}}=\\frac{B\\left(a_{i}\\right)}{B\\left(a_{j}\\right)}=\\frac{A\\left(a_{i}\\right)-1}{A\\left(a_{j}\\right)-1}=\\frac{0-1}{0-1}=1 $$ Since the points $A_{i}, A_{j}, B_{k}$ all lie on the unit circle, interpreting the left-hand side geometrically gives $$ \\measuredangle A_{i} B_{1} A_{j}+\\cdots+\\measuredangle A_{i} B_{n} A_{j}=0 \\Longrightarrow n \\widehat{A_{i} A_{j}}=0 $$ where angles are directed modulo $180^{\\circ}$ and arcs are directed modulo $360^{\\circ}$. This implies that $\\frac{a_{i}}{a_{j}}$ is a power of $\\omega$. Now the finish is easy: since $a_{1}, \\ldots, a_{n}$ are all different, they must be $a_{1} \\omega^{0}, \\ldots, a_{1} \\omega^{n-1}$ in some order; this shows that $A$ is a multiple of $x^{n}-a_{1}^{n}$, as needed."} +{"year":2020,"label":"8","problem":"For every positive integer $N$, let $\\sigma(N)$ denote the sum of the positive integer divisors of $N$. Find all integers $m \\geq n \\geq 2$ satisfying $$ \\frac{\\sigma(m)-1}{m-1}=\\frac{\\sigma(n)-1}{n-1}=\\frac{\\sigma(m n)-1}{m n-1} . $$","solution":" The answer is that $m$ and $n$ should be powers of the same prime number. These all work because for a prime power we have $$ \\frac{\\sigma\\left(p^{e}\\right)-1}{p^{e}-1}=\\frac{\\left(1+p+\\cdots+p^{e}\\right)-1}{p^{e}-1}=\\frac{p\\left(1+\\cdots+p^{e-1}\\right)}{p^{e}-1}=\\frac{p}{p-1} $$ So we now prove these are the only ones. Let $\\lambda$ be the common value of the three fractions. $$ \\begin{aligned} \\sigma(m n) & \\geq \\sigma(m)+m \\sigma(n)-m \\\\ & =(\\lambda m-\\lambda+1)+m(\\lambda n-\\lambda+1)-m \\\\ & =\\lambda m n-\\lambda+1 \\end{aligned} $$ and so equality holds. Thus these are all the divisors of $m n$, for a count of $d(m)+d(n)-$ 1. Claim - If $d(m n)=d(m)+d(n)-1$ and $\\min (m, n) \\geq 2$, then $m$ and $n$ are powers of the same prime. Remark (Nikolai Beluhov). Here is a completion not relying on $|A \\cdot B|=|A|+|B|-1$. By the above arguments, we see that every divisor of $m n$ is either a divisor of $n$, or $n$ times a divisor of $m$. Now suppose that some prime $p \\mid m$ but $p \\nmid n$. Then $p \\mid m n$ but $p$ does not appear in the above classification, a contradiction. By symmetry, it follows that $m$ and $n$ have the same prime divisors. Now suppose we have different primes $p \\mid m$ and $q \\mid n$. Write $\\nu_{p}(m)=\\alpha$ and $\\nu_{p}(n)=\\beta$. Then $p^{\\alpha+\\beta} \\mid m n$, but it does not appear in the above characterization, a contradiction. Thus, $m$ and $n$ are powers of the same prime. Remark (Comments on the function in the problem). Let $f(n)=\\frac{\\sigma(n)-1}{n-1}$. Then $f$ is not really injective even outside the above solution; for example, we have $f\\left(6 \\cdot 11^{k}\\right)=\\frac{11}{5}$ for all $k$, plus sporadic equivalences like $f(14)=f(404)$, as pointed out by one reviewer during test-solving. This means that both relations should be used at once, not independently. Remark (Authorship remarks). Ankan gave the following story for how he came up with the problem while thinking about so-called almost perfect numbers. I was in some boring talk when I recalled a conjecture that if $\\sigma(n)=2 n-1$, then $n$ is a power of 2 . For some reason (divine intervention, maybe) I had the double idea of (1) seeing whether $m, n, m n$ all almost perfect implies $m, n$ powers of 2 , and (2) trying the naive divisor bound to resolve this. Through sheer dumb luck this happened to work out perfectly. I thought this was kinda cool but I felt that I hadn't really unlocked a lot of the potential this idea had: then I basically tried to find the \"general situation\" which allows for this manipulation, and was amazed that it led to such a striking statement."} +{"year":2020,"label":"9","problem":"Ten million fireflies are glowing in $\\mathbb{R}^{3}$ at midnight. Some of the fireflies are friends, and friendship is always mutual. Every second, one firefly moves to a new position so that its distance from each one of its friends is the same as it was before moving. This is the only way that the fireflies ever change their positions. No two fireflies may ever occupy the same point. Initially, no two fireflies, friends or not, are more than a meter away. Following some finite number of seconds, all fireflies find themselves at least ten million meters away from their original positions. Given this information, find the greatest possible number of friendships between the fireflies.","solution":" In general, we show that when $n \\geq 70$, the answer is $f(n)=\\left\\lfloor\\frac{n^{2}}{3}\\right\\rfloor$. Construction: Choose three pairwise parallel lines $\\ell_{A}, \\ell_{B}, \\ell_{C}$ forming an infinite equilateral triangle prism (with side larger than 1). Split the $n$ fireflies among the lines as equally as possible, and say that two fireflies are friends iff they lie on different lines. To see this works: 1. Reflect $\\ell_{A}$ and all fireflies on $\\ell_{A}$ in the plane containing $\\ell_{B}$ and $\\ell_{C}$. 2. Reflect $\\ell_{B}$ and all fireflies on $\\ell_{B}$ in the plane containing $\\ell_{C}$ and $\\ell_{A}$. 3. Reflect $\\ell_{C}$ and all fireflies on $\\ell_{C}$ in the plane containing $\\ell_{A}$ and $\\ell_{B}$. Let $g(n)$ be the answer, given that there exist four pairwise friends (say $a, b, c, d$ ). Note that for a firefly to move, all its friends must be coplanar. Claim (No coplanar $K_{4}$ ) \u2014 We can't have four coplanar fireflies which are pairwise friends. Claim (Key claim \u2014 tetrahedrons don't share faces often) \u2014 There are at most 12 fireflies $e$ which are friends with at least three of $a, b, c, d$. WLOG, will assume that $e$ is friends with $a, b, c$. Then $e$ will always be located at one of two points $E_{1}$ and $E_{2}$ relative to $A B C$, such that $E_{1} A B C$ and $E_{2} A B C$ are two congruent tetrahedrons with fixed shape. We note that points $D, E_{1}$, and $E_{2}$ are all different: clearly $D \\neq E_{1}$ and $E_{1} \\neq E_{2}$. (If $D=E_{2}$, then some fireflies won't be able to move.) Consider the moment where firefly $a$ moves. Its friends must be coplanar at that time, so one of $E_{1}, E_{2}$ lies in plane $B C D$. Similar reasoning holds for planes $A C D$ and $A B D$. So, WLOG $E_{1}$ lies on both planes $B C D$ and $A C D$. Then $E_{1}$ lies on line $C D$, and $E_{2}$ lies in plane $A B D$. This uniquely determines $\\left(E_{1}, E_{2}\\right)$ relative to $A B C D$ : - $E_{1}$ is the intersection of line $C D$ with the reflection of plane $A B D$ in plane $A B C$. - $E_{2}$ is the intersection of plane $A B D$ with the reflection of line $C D$ in plane $A B C$. Accounting for WLOGs, there are at most 12 possibilities for the set $\\left\\{E_{1}, E_{2}\\right\\}$, and thus at most 12 possibilities for $E$. (It's not possible for both elements of one pair $\\left\\{E_{1}, E_{2}\\right\\}$ to be occupied, because then they couldn't move.) Thus, the number of friendships involving exactly one of $a, b, c, d$ is at most $(n-16)$. $2+12 \\cdot 3=2 n+4$, so removing these four fireflies gives $$ g(n) \\leq 6+(2 n+4)+\\max \\{f(n-4), g(n-4)\\} $$ $$ g(n) \\leq \\max \\{f(n),(2 n+10)+g(n-4)\\} \\quad \\forall n \\geq 24 $$ By iterating the above inequality, we get $$ \\begin{aligned} g(n) \\leq \\max \\{f(n),(2 n+10) & +(2(n-4)+10) \\\\ & +\\cdots+(2(n-4 r)+10)+g(n-4 r-4)\\} \\end{aligned} $$ where $r$ satisfies $n-4 r-4<24 \\leq n-4 r$. Now $$ \\begin{aligned} & (2 n+10)+(2(n-4)+10)+\\cdots+(2(n-4 r)+10)+g(n-4 r-4) \\\\ = & (r+1)(2 n-4 r+10)+g(n-4 r-4) \\\\ \\leq & \\left(\\frac{n}{4}-5\\right)(n+37)+\\binom{24}{2} . \\end{aligned} $$ This is less than $f(n)$ for $n \\geq 70$, which concludes the solution. Remark. There are positive integers $n$ such that it is possible to do better than $f(n)$ friendships. For instance, $f(5)=8$, whereas five fireflies $a, b, c, d$, and $e$ as in the proof of the Lemma ( $E_{1}$ being the intersection point of line $C D$ with the reflection of plane $(A B D)$ in plane $(A B C), E_{2}$ being the intersection point of plane $(A B D)$ with the reflection of line $C D$ in plane $(A B C)$, and tetrahedron $A B C D$ being sufficiently arbitrary that points $E_{1}$ and $E_{2}$ exist and points $D, E_{1}$, and $E_{2}$ are pairwise distinct) give a total of nine friendships. Remark (Author comments). It is natural to approach the problem by looking at the two-dimensional version first. In two dimensions, the following arrangement suggests itself almost immediately: We distribute all fireflies as equally as possible among two parallel lines, and two fireflies are friends if and only if they are on different lines. Similarly to the three-dimensional version, this attains the greatest possible number of friendships for all sufficiently large $n$, though not for all $n$. For instance, at least one friendlier arrangements exists for $n=4$, similarly to the above friendlier arrangement for $n=5$ in three dimensions. This observation strongly suggests that in three dimensions we should distribute the fireflies as equally as possible among two parallel planes, and that two fireflies should be friends if and only if they are on different planes. It was a great surprise for me to discover that this arrangement does not in fact give the correct answer! Remark. On the other hand, Ankan Bhattacharya gives the following reasoning as to why the answer should not be that surprising: I think the answer $\\left(10^{14}-1\\right) \/ 3$ is quite natural if you realize that $(n \/ 2)^{2}$ is probably optimal in 2D and $\\binom{n}{2}$ is optimal in super high dimensions (i.e. around $n$ ). So going from dimension 2 to 3 should increase the answer (and indeed it does)."} diff --git a/USA_TSTST/segmented/en-sols-TSTST-2021.jsonl b/USA_TSTST/segmented/en-sols-TSTST-2021.jsonl new file mode 100644 index 0000000000000000000000000000000000000000..01e35a00a5cad161af881e1a9ce9c212f0892033 --- /dev/null +++ b/USA_TSTST/segmented/en-sols-TSTST-2021.jsonl @@ -0,0 +1,28 @@ +{"year":2021,"label":"1","problem":"Let $A B C D$ be a quadrilateral inscribed in a circle with center $O$. Points $X$ and $Y$ lie on sides $A B$ and $C D$, respectively. Suppose the circumcircles of $A D X$ and $B C Y$ meet line $X Y$ again at $P$ and $Q$, respectively. Show that $O P=O Q$.","solution":" \u300e First solution, angle chasing only (Ankit Bisain). Let lines $B Q$ and $D P$ meet $(A B C D)$ again at $D^{\\prime}$ and $B^{\\prime}$, respectively. ![](https:\/\/cdn.mathpix.com\/cropped\/2024_11_19_148cb373ffbe5370bbc6g-03.jpg?height=618&width=737&top_left_y=1047&top_left_x=668) Then $B B^{\\prime} \\| P X$ and $D D^{\\prime} \\| Q Y$ by Reim's theorem. Segments $B B^{\\prime}, D D^{\\prime}$, and $P Q$ share a perpendicular bisector which passes through $O$, so $O P=O Q$."} +{"year":2021,"label":"1","problem":"Let $A B C D$ be a quadrilateral inscribed in a circle with center $O$. Points $X$ and $Y$ lie on sides $A B$ and $C D$, respectively. Suppose the circumcircles of $A D X$ and $B C Y$ meet line $X Y$ again at $P$ and $Q$, respectively. Show that $O P=O Q$.","solution":" \u3010 Second solution via isosceles triangles (from contestants). Let $T=\\overline{B Q} \\cap \\overline{D P}$. ![](https:\/\/cdn.mathpix.com\/cropped\/2024_11_19_148cb373ffbe5370bbc6g-03.jpg?height=455&width=732&top_left_y=1897&top_left_x=665) Note that $P Q T$ is isosceles because $$ \\measuredangle P Q T=\\measuredangle Y Q B=\\measuredangle B C D=\\measuredangle B A D=\\measuredangle X P D=\\measuredangle T P Q $$ Then $(B O D T)$ is cyclic because $$ \\measuredangle B O D=2 \\measuredangle B C D=\\measuredangle P Q T+\\measuredangle T P Q=\\measuredangle B T D . $$ Since $B O=O D, \\overline{T O}$ is an angle bisector of $\\measuredangle B T D$. Since $\\triangle P Q T$ is isosceles, $\\overline{T O} \\perp \\overline{P Q}$, so $O P=O Q$."} +{"year":2021,"label":"1","problem":"Let $A B C D$ be a quadrilateral inscribed in a circle with center $O$. Points $X$ and $Y$ lie on sides $A B$ and $C D$, respectively. Suppose the circumcircles of $A D X$ and $B C Y$ meet line $X Y$ again at $P$ and $Q$, respectively. Show that $O P=O Q$.","solution":" \u092c Third solution using a parallelogram (from contestants). Let $(B C Y)$ meet $\\overline{A B}$ again at $W$ and let $(A D X)$ meet $\\overline{C D}$ again at $Z$. Additionally, let $O_{1}$ be the center of $(A D X)$ and $O_{2}$ be the center of $(B C Y)$. ![](https:\/\/cdn.mathpix.com\/cropped\/2024_11_19_148cb373ffbe5370bbc6g-04.jpg?height=598&width=707&top_left_y=569&top_left_x=686) Note that $(W X Y Z)$ is cyclic since $$ \\measuredangle X W Y+\\measuredangle Y Z X=\\measuredangle Y W B+\\measuredangle X Z D=\\measuredangle Y C B+\\measuredangle X A D=0^{\\circ} $$ so let $O^{\\prime}$ be the center of ( $W X Y Z$ ). Since $\\overline{A D} \\| \\overline{W Y}$ and $\\overline{B C} \\| \\overline{X Z}$ by Reim's theorem, $O O_{1} O^{\\prime} O_{2}$ is a parallelogram. To finish the problem, note that projecting $O_{1}, O_{2}$, and $O^{\\prime}$ onto $\\overline{X Y}$ gives the midpoints of $\\overline{P X}, \\overline{Q Y}$, and $\\overline{X Y}$. Since $O O_{1} O^{\\prime} O_{2}$ is a parallelogram, projecting $O$ onto $\\overline{X Y}$ must give the midpoint of $\\overline{P Q}$, so $O P=O Q$."} +{"year":2021,"label":"1","problem":"Let $A B C D$ be a quadrilateral inscribed in a circle with center $O$. Points $X$ and $Y$ lie on sides $A B$ and $C D$, respectively. Suppose the circumcircles of $A D X$ and $B C Y$ meet line $X Y$ again at $P$ and $Q$, respectively. Show that $O P=O Q$.","solution":" \u092c Fourth solution using congruent circles (from contestants). Let the angle bisector of $\\measuredangle B O D$ meet $\\overline{X Y}$ at $K$. ![](https:\/\/cdn.mathpix.com\/cropped\/2024_11_19_148cb373ffbe5370bbc6g-04.jpg?height=569&width=652&top_left_y=1783&top_left_x=702) Then $(B Q O K)$ is cyclic because $\\measuredangle K O D=\\measuredangle B A D=\\measuredangle K P D$, and $(D O P K)$ is cyclic similarly. By symmetry over $K O$, these circles have the same radius $r$, so $$ O P=2 r \\sin \\angle O K P=2 r \\sin \\angle O K Q=O Q $$ by the Law of Sines."} +{"year":2021,"label":"1","problem":"Let $A B C D$ be a quadrilateral inscribed in a circle with center $O$. Points $X$ and $Y$ lie on sides $A B$ and $C D$, respectively. Suppose the circumcircles of $A D X$ and $B C Y$ meet line $X Y$ again at $P$ and $Q$, respectively. Show that $O P=O Q$.","solution":" \u3010 Fifth solution by ratio calculation (from contestants). Let $\\overline{X Y}$ meet $(A B C D)$ at $X^{\\prime}$ and $Y^{\\prime}$. ![](https:\/\/cdn.mathpix.com\/cropped\/2024_11_19_148cb373ffbe5370bbc6g-05.jpg?height=618&width=715&top_left_y=362&top_left_x=682) Since $\\measuredangle Y^{\\prime} B D=\\measuredangle P X^{\\prime} D$ and $\\measuredangle B Y^{\\prime} D=\\measuredangle B A D=\\measuredangle X^{\\prime} P D$, $$ \\triangle B Y^{\\prime} D \\sim \\triangle X P^{\\prime} D \\Longrightarrow P X^{\\prime}=B Y^{\\prime} \\cdot \\frac{D X^{\\prime}}{B D} $$ Similarly, $$ \\triangle B X^{\\prime} D \\sim \\triangle B Q Y^{\\prime} \\Longrightarrow Q Y^{\\prime}=D X^{\\prime} \\cdot \\frac{B Y^{\\prime}}{B D} $$ Thus $P X^{\\prime}=Q Y^{\\prime}$, which gives $O P=O Q$."} +{"year":2021,"label":"1","problem":"Let $A B C D$ be a quadrilateral inscribed in a circle with center $O$. Points $X$ and $Y$ lie on sides $A B$ and $C D$, respectively. Suppose the circumcircles of $A D X$ and $B C Y$ meet line $X Y$ again at $P$ and $Q$, respectively. Show that $O P=O Q$.","solution":" I Sixth solution using radical axis (from author). Without loss of generality, assume $\\overline{A D} \\nVdash \\overline{B C}$, as this case holds by continuity. Let $(B C Y)$ meet $\\overline{A B}$ again at $W$, let $(A D X)$ meet $\\overline{C D}$ again at $Z$, and let $\\overline{W Z}$ meet $(A D X)$ and $(B C Y)$ again at $R$ and $S$. ![](https:\/\/cdn.mathpix.com\/cropped\/2024_11_19_148cb373ffbe5370bbc6g-05.jpg?height=610&width=712&top_left_y=1574&top_left_x=678) Note that $(W X Y Z)$ is cyclic since $$ \\measuredangle X W Y+\\measuredangle Y Z X=\\measuredangle Y W B+\\measuredangle X Z D=\\measuredangle Y C B+\\measuredangle X A D=0^{\\circ} $$ and $(P Q R S)$ is cyclic since $$ \\measuredangle P Q S=\\measuredangle Y Q S=\\measuredangle Y W S=\\measuredangle P X Z=\\measuredangle P R Z=\\measuredangle S R P $$ Additionally, $\\overline{A D} \\| \\overline{P R}$ since $$ \\measuredangle D A X+\\measuredangle A X P+\\measuredangle X P R=\\measuredangle Y W X+\\measuredangle W X Y+\\measuredangle X Y W=0^{\\circ}, $$ and $\\overline{B C} \\| \\overline{S Q}$ similarly. Lastly, $(A B C D)$ and $(P Q R S)$ are concentric; if not, using the radical axis theorem twice shows that their radical axis must be parallel to both $\\overline{A D}$ and $\\overline{B C}$, contradiction."} +{"year":2021,"label":"1","problem":"Let $A B C D$ be a quadrilateral inscribed in a circle with center $O$. Points $X$ and $Y$ lie on sides $A B$ and $C D$, respectively. Suppose the circumcircles of $A D X$ and $B C Y$ meet line $X Y$ again at $P$ and $Q$, respectively. Show that $O P=O Q$.","solution":" \u3010 Seventh solution using Cayley-Bacharach (author). Define points $W, Z, R, S$ as in the previous solution. ![](https:\/\/cdn.mathpix.com\/cropped\/2024_11_19_148cb373ffbe5370bbc6g-06.jpg?height=621&width=743&top_left_y=569&top_left_x=668) The quartics $(A D X Z) \\cup(B C W Y)$ and $\\overline{X Y} \\cup \\overline{W Z} \\cup(A B C D)$ meet at the 16 points $$ A, B, C, D, W, X, Y, Z, P, Q, R, S, I, I, J, J $$ where $I$ and $J$ are the circular points at infinity. Since $\\overline{A B} \\cup \\overline{C D} \\cup(P Q R)$ contains the 13 points $$ A, B, C, D, P, Q, R, W, X, Y, Z, I, J $$ it must contain $S, I$, and $J$ as well, by quartic Cayley-Bacharach. Thus, $(P Q R S)$ is cyclic and intersects $(A B C D)$ at $I, I, J$, and $J$, implying that the two circles are concentric, as desired. Remark (Author comments). Holden says he came up with this problem via the CayleyBacharach solution, by trying to get two quartics to intersect."} +{"year":2021,"label":"2","problem":"Let $a_{1}i$. First, each $i$ has finite degree - otherwise $$ \\frac{a_{x_{1}}}{x_{1}}=\\frac{a_{x_{2}}}{x_{2}}=\\cdots $$ for an infinite increasing sequence of positive integers $x_{i}$, but then the $a_{x_{i}}$ are unbounded. Now we use the following process to build a sequence of indices whose $a_{i}$ we can lower-bound: - Start at $x_{1}=1$, which is good. - If we're currently at good index $x_{i}$, then let $s_{i}$ be the largest positive integer such that $x_{i}$ has an edge to $x_{i}+s_{i}$. (This exists because the degrees are finite.) - Let $t_{i}$ be the smallest positive integer for which $x_{i}+s_{i}+t_{i}$ is good, and let this be $x_{i+1}$. This exists because if all numbers $k \\leq x \\leq 2 k$ are bad, they must each connect to some number less than $k$ (if two connect to each other, the smaller one is good), but then two connect to the same number, and therefore to each other this is the idea we will use later to bound the $t_{i}$ as well. Then $x_{i}=1+s_{1}+t_{1}+\\cdots+s_{i-1}+t_{i-1}$, and we have $$ a_{x_{i+1}}>a_{x_{i}+s_{i}}=\\frac{x_{i}+s_{i}}{x_{i}} a_{x_{i}}=\\frac{1+\\left(s_{1}+\\cdots+s_{i-1}+s_{i}\\right)+\\left(t_{1}+\\cdots+t_{i-1}\\right)}{1+\\left(s_{1}+\\cdots+s_{i-1}\\right)+\\left(t_{1}+\\cdots+t_{i-1}\\right)} a_{x_{i}} $$ This means $$ c_{n}:=\\frac{a_{x_{n}}}{a_{1}}>\\prod_{i=1}^{n-1} \\frac{1+\\left(s_{1}+\\cdots+s_{i-1}+s_{i}\\right)+\\left(t_{1}+\\cdots+t_{i-1}\\right)}{1+\\left(s_{1}+\\cdots+s_{i-1}\\right)+\\left(t_{1}+\\cdots+t_{i-1}\\right)} $$ ## Lemma $t_{1}+\\cdots+t_{n} \\leq s_{1}+\\cdots+s_{n}$ for each $n$. First we claim that if $x \\in\\left(x_{i}+s_{i}, x_{i}+s_{i}+t_{i}\\right)$, then $x$ cannot have an edge to $x_{j}$ for any $j \\leq i$. This is because $x>x_{i}+s_{i} \\geq x_{j}+s_{j}$, contradicting the fact that $x_{j}+s_{j}$ is the largest neighbor of $x_{j}$. This also means $x$ doesn't have an edge to $x_{j}+s_{j}$ for any $j \\leq i$, since if it did, it would have an edge to $x_{j}$. Second, no two bad values of $x$ can have an edge, since then the smaller one is good. This also means no two bad $x$ can have an edge to the same $y$. Then each of the $\\sum\\left(t_{i}-1\\right)$ values in the intervals $\\left(x_{i}+s_{i}, x_{i}+s_{i}+t_{i}\\right)$ for $1 \\leq i \\leq n$ must have an edge to an unique $y$ in one of the intervals $\\left(x_{i}, x_{i}+s_{i}\\right)$ (not necessarily with the same $i$ ). Therefore $$ \\sum\\left(t_{i}-1\\right) \\leq \\sum\\left(s_{i}-1\\right) \\Longrightarrow \\sum t_{i} \\leq \\sum s_{i} $$ Now note that if $a>b$, then $\\frac{a+x}{b+x}=1+\\frac{a-b}{b+x}$ is decreasing in $x$. This means $$ c_{n}>\\prod_{i=1}^{n-1} \\frac{1+2 s_{1}+\\cdots+2 s_{i-1}+s_{i}}{1+2 s_{1}+\\cdots+2 s_{i-1}}>\\prod_{i=1}^{n-1} \\frac{1+2 s_{1}+\\cdots+2 s_{i-1}+2 s_{i}}{1+2 s_{1}+\\cdots+2 s_{i-1}+s_{i}} $$ By multiplying both products, we have a telescoping product, which results in $$ c_{n}^{2} \\geq 1+2 s_{1}+\\cdots+2 s_{n}+2 s_{n+1} $$ The right hand side is unbounded since the $s_{i}$ are positive integers, while $c_{n}=a_{x_{n}} \/ a_{1}<$ $1 \/ a_{1}$ is bounded, contradiction."} +{"year":2021,"label":"2","problem":"Let $a_{1}i \\geq k$, then $j \\leq 1.1 i$. Suppose no such $k$ exists. Then, take a pair $j_{1}>i_{1}$ such that $\\frac{a_{j_{1}}}{j_{1}}=\\frac{a_{i_{1}}}{i_{1}}$ and $j_{1}>1.1 i_{1}$, or $a_{j_{1}}>1.1 a_{i_{1}}$. Now, since $k=j_{1}$ can't work, there exists a pair $j_{2}>i_{2} \\geq i_{1}$ such that $\\frac{a_{j_{2}}}{j_{2}}=\\frac{a_{i_{2}}}{i_{2}}$ and $j_{2}>1.1 i_{2}$, or $a_{j_{2}}>1.1 a_{i_{2}}$. Continuing in this fashion, we see that $$ a_{j_{\\ell}}>1.1 a_{i_{\\ell}}>1.1 a_{j_{\\ell-1}} $$ so we have that $a_{j_{\\ell}}>1.1^{\\ell} a_{i_{1}}$. Taking $\\ell>\\log _{1.1}\\left(1 \/ a_{1}\\right)$ gives the desired contradiction. ## Lemma For $N>k^{2}$, there are at most $0.8 N$ pins in $[\\sqrt{N}, N)$. $$ p-N\\left(1-\\frac{1}{1.1}\\right) \\leq N-p $$ so $p \\leq 0.8 N$, as desired. We say that $i$ is the pin of $j$ if it is the smallest index such that $\\frac{a_{i}}{i}=\\frac{a_{j}}{j}$. The pin of $j$ is always a pin. Given an index $i$, let $f(i)$ denote the largest index less than $i$ that is not a pin (we leave the function undefined when no such index exists, as we are only interested in the behavior for large $i$ ). Then $f$ is weakly increasing and unbounded by the first lemma. Let $N_{0}$ be a positive integer such that $f\\left(\\sqrt{N_{0}}\\right)>k$. Take any $N>N_{0}$ such that $N$ is not a pin. Let $b_{0}=N$, and $b_{1}$ be the pin of $b_{0}$. Recursively define $b_{2 i}=f\\left(b_{2 i-1}\\right)$, and $b_{2 i+1}$ to be the pin of $b_{2 i}$. Let $\\ell$ be the largest odd index such that $b_{\\ell} \\geq \\sqrt{N}$. We first show that $b_{\\ell} \\leq 100 \\sqrt{N}$. Since $N>N_{0}$, we have $b_{\\ell+1}>k$. By the choice of $\\ell$ we have $b_{\\ell+2}<\\sqrt{N}$, so $$ b_{\\ell+1}<1.1 b_{\\ell+2}<1.1 \\sqrt{N} $$ by the first lemma. We see that all the indices from $b_{\\ell+1}+1$ to $b_{\\ell}$ must be pins, so we have at least $b_{\\ell}-1.1 \\sqrt{N}$ pins in $\\left[\\sqrt{N}, b_{\\ell}\\right)$. Combined with the second lemma, this shows that $b_{\\ell} \\leq 100 \\sqrt{N}$. Now, we have that $a_{b_{2 i}}=\\frac{b_{2 i}}{b_{2 i+1}} a_{b_{2 i+1}}$ and $a_{b_{2 i+1}}>a_{b_{2 i+2}}$, so combining gives us $$ \\frac{a_{b_{0}}}{a_{b_{\\ell}}}>\\frac{b_{0}}{b_{1}} \\frac{b_{2}}{b_{3}} \\cdots \\frac{b_{\\ell-1}}{b_{\\ell}} . $$ Note that there are at least $$ \\left(b_{1}-b_{2}\\right)+\\left(b_{3}-b_{4}\\right)+\\cdots+\\left(b_{\\ell-2}-b_{\\ell-1}\\right) $$ pins in $[\\sqrt{N}, N)$, so by the second lemma, that sum is at most $0.8 N$. Thus, $$ \\begin{aligned} \\left(b_{0}-b_{1}\\right)+\\left(b_{2}-b_{3}\\right)+\\cdots+\\left(b_{\\ell-1}-b_{\\ell}\\right) & =b_{0}-\\left[\\left(b_{1}-b_{2}\\right)+\\cdots+\\left(b_{\\ell-2}-b_{\\ell-1}\\right)\\right]-b_{\\ell} \\\\ & \\geq 0.2 N-100 \\sqrt{N} . \\end{aligned} $$ Then $$ \\begin{aligned} \\frac{b_{0}}{b_{1}} \\frac{b_{2}}{b_{3}} \\cdots \\frac{b_{\\ell-1}}{b_{\\ell}} & \\geq 1+\\frac{b_{0}-b_{1}}{b_{1}}+\\cdots+\\frac{b_{\\ell-1}-b_{\\ell}}{b_{\\ell}} \\\\ & >1+\\frac{b_{0}-b_{1}}{b_{0}}+\\cdots+\\frac{b_{\\ell-1}-b_{\\ell}}{b_{0}} \\\\ & \\geq 1+\\frac{0.2 N-100 \\sqrt{N}}{N} \\end{aligned} $$ which is at least 1.01 if $N_{0}$ is large enough. Thus, we see that $$ a_{N}>1.01 a_{b_{\\ell}} \\geq 1.01 a_{\\lfloor\\sqrt{N}\\rfloor} $$ if $N>N_{0}$ is not a pin. Since there are arbitrarily large non-pins, this implies that the sequence $\\left(a_{n}\\right)$ is unbounded, which is the desired contradiction."} +{"year":2021,"label":"3","problem":"Find all positive integers $k>1$ for which there exists a positive integer $n$ such that $\\binom{n}{k}$ is divisible by $n$, and $\\binom{n}{m}$ is not divisible by $n$ for $2 \\leq mt_{p}$, so $\\nu_{p}(n-i)=\\nu_{p}(i)$; - If $p \\mid k$ and $i \\neq k_{p}$, then we have $\\nu_{p}(i), \\nu_{p}(n-i) \\leq t_{p}$ and $\\nu_{p}(n) \\geq t_{p}$, so again $\\nu_{p}(n-i)=\\nu_{p}(i) ;$ - If $p \\mid k$ and $i=k_{p}$, then we have $\\nu_{p}(n-i)=\\nu_{p}(i)+\\nu_{p}(k)$ by (2). We conclude that $\\nu_{p}(n-i)=\\nu_{p}(i)$ always holds, except when $i=k_{p}$, when we have $\\nu_{p}(n-i)=\\nu_{p}(i)+\\nu_{p}(k)$ (this formula holds irrespective of whether $p \\mid k$ or $\\left.p \\nmid k\\right)$. We can now show that $\\binom{n}{k}$ is divisible by $n$, which amounts to showing that $k$ ! divides $(n-1)(n-2) \\cdots(n-k+1)$. Indeed, for each prime $p \\leq k$ we have $$ \\begin{aligned} \\nu_{p}((n-1)(n-2) \\ldots(n-k+1)) & =\\nu_{p}\\left(n-k_{p}\\right)+\\sum_{i\\nu_{p}(k)$, then it follows that $$ \\begin{aligned} \\nu_{p}((n-1)(n-2) \\ldots(n-m+1)) & =\\nu_{p}(k)+\\sum_{i=1}^{m-1} \\nu_{p}(i) \\\\ & <\\nu_{p}(m)+\\sum_{i=1}^{m-1} \\nu_{p}(i) \\\\ & =\\nu_{p}(m!) \\end{aligned} $$ so $m$ ! cannot divide $(n-1)(n-2) \\ldots(n-m+1)$. On the other hand, suppose that $\\nu_{p}(m) \\leq \\nu_{p}(k)$ for all $p \\mid k$, which would mean that $m \\mid k$ and hence $m \\leq \\frac{k}{2}$. Consider a prime $p$ dividing $m$. We have $k_{p} \\geq \\frac{k}{2}$, because otherwise $2 k_{p}$ could have been used instead of $k_{p}$. It follows that $m \\leq \\frac{k}{2} \\leq k_{p}$. Therefore, we obtain $$ \\begin{aligned} \\nu_{p}((n-1)(n-2) \\ldots(n-m+1)) & =\\sum_{i=1}^{m-1} \\nu_{p}(n-i) \\\\ & =\\sum_{i=1}^{m-1} \\nu_{p}(i) \\\\ & =\\nu_{p}((m-1)!)<\\nu_{p}(m!) \\end{aligned} $$ showing that $(n-1)(n-2) \\cdots(n-m+1)$ is not divisible by $m$ !. This shows that $\\binom{n}{m}$ is not divisible by $n$ for $ma \/ 2$, then $(\\dagger)$ forces $r^{2}+s^{2} \\leq 2 b$, giving the last case. Hence, there exists some particular pair $(r, s)$ for which there are infinitely many solutions $(m, n)$. Simplifying the system gives $$ \\begin{aligned} & a n=2 r m+r^{2}-b \\\\ & 2 s n=a m+b-s^{2} \\end{aligned} $$ Since the system is linear, for there to be infinitely many solutions $(m, n)$ the system must be dependent. This gives $$ \\frac{a}{2 s}=\\frac{2 r}{a}=\\frac{r^{2}-b}{b-s^{2}} $$ so $a=2 \\sqrt{r s}$ and $b=\\frac{s^{2} \\sqrt{r}+r^{2} \\sqrt{s}}{\\sqrt{r}+\\sqrt{s}}$. Since $r s$ must be square, we can reparametrize as $r=k x^{2}, s=k y^{2}$, and $\\operatorname{gcd}(x, y)=1$. This gives $$ \\begin{aligned} a & =2 k x y \\\\ b & =k^{2} x y\\left(x^{2}-x y+y^{2}\\right) \\end{aligned} $$ Thus, $a \\mid 2 b$, as desired."} +{"year":2021,"label":"5","problem":"Let $T$ be a tree on $n$ vertices with exactly $k$ leaves. Suppose that there exists a subset of at least $\\frac{n+k-1}{2}$ vertices of $T$, no two of which are adjacent. Show that the longest path in $T$ contains an even number of edges.","solution":" The longest path in $T$ must go between two leaves. The solutions presented here will solve the problem by showing that in the unique 2-coloring of $T$, all leaves are the same color. ## \u3010 Solution 1 (Ankan Bhattacharya, Jeffery Li). ## Lemma If $S$ is an independent set of $T$, then $$ \\sum_{v \\in S} \\operatorname{deg}(v) \\leq n-1 $$ Equality holds if and only if $S$ is one of the two components of the unique 2-coloring of $T$. We are given that there exists an independent set of at least $\\frac{n+k-1}{2}$ vertices. By greedily choosing vertices of smallest degree, the sum of the degrees of these vertices is at least $$ k+2 \\cdot \\frac{n-k-1}{2}=n-1 $$ Thus equality holds everywhere, which implies that the independent set contains every leaf and is one of the components of the 2 -coloring."} +{"year":2021,"label":"5","problem":"Let $T$ be a tree on $n$ vertices with exactly $k$ leaves. Suppose that there exists a subset of at least $\\frac{n+k-1}{2}$ vertices of $T$, no two of which are adjacent. Show that the longest path in $T$ contains an even number of edges.","solution":" The longest path in $T$ must go between two leaves. The solutions presented here will solve the problem by showing that in the unique 2-coloring of $T$, all leaves are the same color. ## \u3010 Solution 2 (Andrew Gu). ## Lemma The vertices of $T$ can be partitioned into $k-1$ paths (i.e. the induced subgraph on each set of vertices is a path) such that all edges of $T$ which are not part of a path are incident to an endpoint of a path. Now given a path of $a$ vertices, at most $\\frac{a+1}{2}$ of those vertices can be in an independent set of $T$. By the lemma, $T$ can be partitioned into $k-1$ paths of $a_{1}, \\ldots, a_{k-1}$ vertices, so the maximum size of an independent set of $T$ is $$ \\sum \\frac{a_{i}+1}{2}=\\frac{n+k-1}{2} . $$ For equality to hold, each path in the partition must have an odd number of vertices, and has a unique 2 -coloring in red and blue where the endpoints are red. The unique independent set of $T$ of size $\\frac{n+k-1}{2}$ is then the set of red vertices. By the lemma, the edges of $T$ which are not part of a path connect an endpoint of a path (which is colored red) to another vertex (which must be blue, because the red vertices are independent). Thus the coloring of the paths extends to the unique 2 -coloring of $T$. The leaves of $T$ are endpoints of paths, so they are all red."} +{"year":2021,"label":"6","problem":"Triangles $A B C$ and $D E F$ share circumcircle $\\Omega$ and incircle $\\omega$ so that points $A, F$, $B, D, C$, and $E$ occur in this order along $\\Omega$. Let $\\Delta_{A}$ be the triangle formed by lines $A B, A C$, and $E F$, and define triangles $\\Delta_{B}, \\Delta_{C}, \\ldots, \\Delta_{F}$ similarly. Furthermore, let $\\Omega_{A}$ and $\\omega_{A}$ be the circumcircle and incircle of triangle $\\Delta_{A}$, respectively, and define circles $\\Omega_{B}, \\omega_{B}, \\ldots, \\Omega_{F}, \\omega_{F}$ similarly. (a) Prove that the two common external tangents to circles $\\Omega_{A}$ and $\\Omega_{D}$ and the two common external tangents to circles $\\omega_{A}$ and $\\omega_{D}$ are either concurrent or pairwise parallel. (b) Suppose that these four lines meet at point $T_{A}$, and define points $T_{B}$ and $T_{C}$ similarly. Prove that points $T_{A}, T_{B}$, and $T_{C}$ are collinear.","solution":" ![](https:\/\/cdn.mathpix.com\/cropped\/2024_11_19_148cb373ffbe5370bbc6g-17.jpg?height=818&width=754&top_left_y=1150&top_left_x=651) Let $I$ and $r$ be the center and radius of $\\omega$, and let $O$ and $R$ be the center and radius of $\\Omega$. Let $O_{A}$ and $I_{A}$ be the circumcenter and incenter of triangle $\\Delta_{A}$, and define $O_{B}$, $I_{B}, \\ldots, I_{F}$ similarly. Let $\\omega$ touch $E F$ at $A_{1}$, and define $B_{1}, C_{1}, \\ldots, F_{1}$ similarly. $$ \\left(\\Omega_{A} \\cup \\omega_{A}\\right) \\sim\\left(\\Omega_{D} \\cup \\omega_{D}\\right) $$ The four lines will concur at the homothetic center of these figures (possibly at infinity). Solution 1 (author) Let the second tangent to $\\omega$ parallel to $E F$ meet lines $A B$ and $A C$ at $P$ and $Q$, respectively, and let the second tangent to $\\omega$ parallel to $B C$ meet lines $D E$ and $D F$ at $R$ and $S$, respectively. Furthermore, let $\\omega$ touch $P Q$ and $R S$ at $U$ and $V$, respectively. Let $h$ be inversion with respect to $\\omega$. Then $h$ maps $A, B$, and $C$ onto the midpoints of the sides of triangle $D_{1} E_{1} F_{1}$. So $h$ maps $k$ onto the Euler circle of triangle $D_{1} E_{1} F_{1}$. Similarly, $h$ maps $k$ onto the Euler circle of triangle $A_{1} B_{1} C_{1}$. Therefore, triangles $A_{1} B_{1} C_{1}$ and $D_{1} E_{1} F_{1}$ share a common nine-point circle $\\gamma$. Let $K$ be its center; its radius equals $\\frac{1}{2} r$. Let $H$ be the reflection of $I$ in $K$. Then $H$ is the common orthocenter of triangles $A_{1} B_{1} C_{1}$ and $D_{1} E_{1} F_{1}$. Let $\\gamma_{U}$ of center $K_{U}$ and radius $\\frac{1}{2} r$ be the Euler circle of triangle $U E_{1} F_{1}$, and let $\\gamma_{V}$ of center $K_{V}$ and radius $\\frac{1}{2} r$ be the Euler circle of triangle $V B_{1} C_{1}$. Let $H_{U}$ be the orthocenter of triangle $U E_{1} F_{1}$. Since quadrilateral $D_{1} E_{1} F_{1} U$ is cyclic, vectors $\\overrightarrow{H H_{U}}$ and $\\overrightarrow{D_{1} U}$ are equal. Consequently, $\\overrightarrow{K K_{U}}=\\frac{1}{2} \\overrightarrow{D_{1} U}$. Similarly, $\\overrightarrow{K K_{V}}=$ $\\frac{1}{2} \\overrightarrow{A_{1} V}$. Since both of $A_{1} U$ and $D_{1} V$ are diameters in $\\omega$, vectors $\\overrightarrow{D_{1} U}$ and $\\overrightarrow{A_{1} V}$ are equal. Therefore, $K_{U}$ and $K_{V}$ coincide, and so do $\\gamma_{U}$ and $\\gamma_{V}$. As above, $h$ maps $\\gamma_{U}$ onto the circumcircle of triangle $A P Q$ and $\\gamma_{V}$ onto the circumcircle of triangle $D R S$. Therefore, triangles $A P Q$ and $D R S$ are inscribed inside the same circle $\\Omega_{A D}$. Since $E F$ and $P Q$ are parallel, triangles $\\Delta_{A}$ and $A P Q$ are homothetic, and so are figures $\\Omega_{A} \\cup \\omega_{A}$ and $\\Omega_{A D} \\cup \\omega$. Consequently, we have $$ \\left(\\Omega_{A} \\cup \\omega_{A}\\right) \\sim\\left(\\Omega_{A D} \\cup \\omega\\right) \\sim\\left(\\Omega_{D} \\cup \\omega_{D}\\right), $$ which solves part (a)."} +{"year":2021,"label":"6","problem":"Triangles $A B C$ and $D E F$ share circumcircle $\\Omega$ and incircle $\\omega$ so that points $A, F$, $B, D, C$, and $E$ occur in this order along $\\Omega$. Let $\\Delta_{A}$ be the triangle formed by lines $A B, A C$, and $E F$, and define triangles $\\Delta_{B}, \\Delta_{C}, \\ldots, \\Delta_{F}$ similarly. Furthermore, let $\\Omega_{A}$ and $\\omega_{A}$ be the circumcircle and incircle of triangle $\\Delta_{A}$, respectively, and define circles $\\Omega_{B}, \\omega_{B}, \\ldots, \\Omega_{F}, \\omega_{F}$ similarly. (a) Prove that the two common external tangents to circles $\\Omega_{A}$ and $\\Omega_{D}$ and the two common external tangents to circles $\\omega_{A}$ and $\\omega_{D}$ are either concurrent or pairwise parallel. (b) Suppose that these four lines meet at point $T_{A}$, and define points $T_{B}$ and $T_{C}$ similarly. Prove that points $T_{A}, T_{B}$, and $T_{C}$ are collinear.","solution":" ![](https:\/\/cdn.mathpix.com\/cropped\/2024_11_19_148cb373ffbe5370bbc6g-17.jpg?height=818&width=754&top_left_y=1150&top_left_x=651) Let $I$ and $r$ be the center and radius of $\\omega$, and let $O$ and $R$ be the center and radius of $\\Omega$. Let $O_{A}$ and $I_{A}$ be the circumcenter and incenter of triangle $\\Delta_{A}$, and define $O_{B}$, $I_{B}, \\ldots, I_{F}$ similarly. Let $\\omega$ touch $E F$ at $A_{1}$, and define $B_{1}, C_{1}, \\ldots, F_{1}$ similarly. $$ \\left(\\Omega_{A} \\cup \\omega_{A}\\right) \\sim\\left(\\Omega_{D} \\cup \\omega_{D}\\right) $$ The four lines will concur at the homothetic center of these figures (possibly at infinity). Solution 2 (Michael Ren) As in the previous solution, let the second tangent to $\\omega$ parallel to $E F$ meet $A B$ and $A C$ at $P$ and $Q$, respectively. Let $(A P Q)$ meet $\\Omega$ again at $D^{\\prime}$, so that $D^{\\prime}$ is the Miquel point of $\\{A B, A C, B C, P Q\\}$. Since the quadrilateral formed by these lines has incircle $\\omega$, it is classical that $D^{\\prime} I$ bisects $\\angle P D^{\\prime} C$ and $B D^{\\prime} Q$ (e.g. by DDIT). Let $\\ell$ be the tangent to $\\Omega$ at $D^{\\prime}$ and $D^{\\prime} I$ meet $\\Omega$ again at $M$. We have $$ \\measuredangle\\left(\\ell, D^{\\prime} B\\right)=\\measuredangle D^{\\prime} C B=\\measuredangle D^{\\prime} Q P=\\measuredangle\\left(D^{\\prime} Q, E F\\right) . $$ Therefore $D^{\\prime} I$ also bisects the angle between $\\ell$ and the line parallel to $E F$ through $D^{\\prime}$. This means that $M$ is one of the arc midpoints of $E F$. Additionally, $D^{\\prime}$ lies on $\\operatorname{arc} B C$ not containing $A$, so $D^{\\prime}=D$. Similarly, letting the second tangent to $\\omega$ parallel to $B C$ meet $D E$ and $D F$ again at $R$ and $S$, we have $A D R S$ cyclic. ## Lemma There exists a circle $\\Omega_{A D}$ tangent to $\\Omega_{A}$ and $\\Omega_{D}$ at $A$ and $D$, respectively. $$ \\measuredangle O A O_{A}=\\measuredangle(\\perp E F, \\perp B C)=\\measuredangle(E F, B C) . $$ (Here, $\\perp E F$ means the direction perpendicular to $E F$.) By symmetry, this is equal to $\\measuredangle O_{D} D O$. The circle $(A D P Q)$ passes through $A$ and $D$, and is tangent to $\\Omega_{A}$ by homothety. Therefore it coincides with $\\Omega_{A D}$, as does ( $A D R S$ ). Like the previous solution, we finish with $$ \\left(\\Omega_{A} \\cup \\omega_{A}\\right) \\sim\\left(\\Omega_{A D} \\cup \\omega\\right) \\sim\\left(\\Omega_{D} \\cup \\omega_{D}\\right) . $$"} +{"year":2021,"label":"6","problem":"Triangles $A B C$ and $D E F$ share circumcircle $\\Omega$ and incircle $\\omega$ so that points $A, F$, $B, D, C$, and $E$ occur in this order along $\\Omega$. Let $\\Delta_{A}$ be the triangle formed by lines $A B, A C$, and $E F$, and define triangles $\\Delta_{B}, \\Delta_{C}, \\ldots, \\Delta_{F}$ similarly. Furthermore, let $\\Omega_{A}$ and $\\omega_{A}$ be the circumcircle and incircle of triangle $\\Delta_{A}$, respectively, and define circles $\\Omega_{B}, \\omega_{B}, \\ldots, \\Omega_{F}, \\omega_{F}$ similarly. (a) Prove that the two common external tangents to circles $\\Omega_{A}$ and $\\Omega_{D}$ and the two common external tangents to circles $\\omega_{A}$ and $\\omega_{D}$ are either concurrent or pairwise parallel. (b) Suppose that these four lines meet at point $T_{A}$, and define points $T_{B}$ and $T_{C}$ similarly. Prove that points $T_{A}, T_{B}$, and $T_{C}$ are collinear.","solution":" ![](https:\/\/cdn.mathpix.com\/cropped\/2024_11_19_148cb373ffbe5370bbc6g-17.jpg?height=818&width=754&top_left_y=1150&top_left_x=651) Let $I$ and $r$ be the center and radius of $\\omega$, and let $O$ and $R$ be the center and radius of $\\Omega$. Let $O_{A}$ and $I_{A}$ be the circumcenter and incenter of triangle $\\Delta_{A}$, and define $O_{B}$, $I_{B}, \\ldots, I_{F}$ similarly. Let $\\omega$ touch $E F$ at $A_{1}$, and define $B_{1}, C_{1}, \\ldots, F_{1}$ similarly. $$ \\left(\\Omega_{A} \\cup \\omega_{A}\\right) \\sim\\left(\\Omega_{D} \\cup \\omega_{D}\\right) $$ The four lines will concur at the homothetic center of these figures (possibly at infinity). Solution 3 (Andrew Gu) Construct triangles homothetic to $\\Delta_{A}$ and $\\Delta_{D}$ (with positive ratio) which have the same circumcircle; it suffices to show that these copies have the same incircle as well. Let $O$ be the center of this common circumcircle, which we take to be the origin, and $M_{X Y}$ denote the point on the circle such that the tangent at that point is parallel to line $X Y$ (from the two possible choices, we make the choice that corresponds to the arc midpoint on $\\Omega$, e.g. $M_{A B}$ should correspond to the arc midpoint on the internal angle bisector of $A C B$ ). The left diagram below shows the original triangles $A B C$ and $D E F$, while the right diagram shows the triangles homothetic to $\\Delta_{A}$ and $\\Delta_{D}$. ![](https:\/\/cdn.mathpix.com\/cropped\/2024_11_19_148cb373ffbe5370bbc6g-19.jpg?height=566&width=1203&top_left_y=902&top_left_x=432) Using the fact that the incenter is the orthocenter of the arc midpoints, the incenter of $\\Delta_{A}$ in this reference frame is $M_{A B}+M_{A C}-M_{E F}$ and the incenter of $\\Delta_{D}$ in this reference frame is $M_{D E}+M_{D F}-M_{B C}$. Since $A B C$ and $D E F$ share a common incenter, we have $$ M_{A B}+M_{B C}+M_{C A}=M_{D E}+M_{E F}+M_{F D} $$ Thus the copies of $\\Delta_{A}$ and $\\Delta_{D}$ have the same incenter, and therefore the same incircle as well (Euler's theorem determines the inradius)."} +{"year":2021,"label":"6","problem":"Triangles $A B C$ and $D E F$ share circumcircle $\\Omega$ and incircle $\\omega$ so that points $A, F$, $B, D, C$, and $E$ occur in this order along $\\Omega$. Let $\\Delta_{A}$ be the triangle formed by lines $A B, A C$, and $E F$, and define triangles $\\Delta_{B}, \\Delta_{C}, \\ldots, \\Delta_{F}$ similarly. Furthermore, let $\\Omega_{A}$ and $\\omega_{A}$ be the circumcircle and incircle of triangle $\\Delta_{A}$, respectively, and define circles $\\Omega_{B}, \\omega_{B}, \\ldots, \\Omega_{F}, \\omega_{F}$ similarly. (a) Prove that the two common external tangents to circles $\\Omega_{A}$ and $\\Omega_{D}$ and the two common external tangents to circles $\\omega_{A}$ and $\\omega_{D}$ are either concurrent or pairwise parallel. (b) Suppose that these four lines meet at point $T_{A}$, and define points $T_{B}$ and $T_{C}$ similarly. Prove that points $T_{A}, T_{B}$, and $T_{C}$ are collinear.","solution":" ![](https:\/\/cdn.mathpix.com\/cropped\/2024_11_19_148cb373ffbe5370bbc6g-17.jpg?height=818&width=754&top_left_y=1150&top_left_x=651) Let $I$ and $r$ be the center and radius of $\\omega$, and let $O$ and $R$ be the center and radius of $\\Omega$. Let $O_{A}$ and $I_{A}$ be the circumcenter and incenter of triangle $\\Delta_{A}$, and define $O_{B}$, $I_{B}, \\ldots, I_{F}$ similarly. Let $\\omega$ touch $E F$ at $A_{1}$, and define $B_{1}, C_{1}, \\ldots, F_{1}$ similarly. $$ \\left(\\Omega_{A} \\cup \\omega_{A}\\right) \\sim\\left(\\Omega_{D} \\cup \\omega_{D}\\right) $$ The four lines will concur at the homothetic center of these figures (possibly at infinity). Solution 1 (author) By Monge's theorem applied to $\\omega, \\omega_{A}$, and $\\omega_{D}$, points $A, D$, and $T_{A}$ are collinear. Therefore, $T_{A}=A D \\cap I_{A} I_{D}$. Let $p$ be pole-and-polar correspondence with respect to $\\omega$. Then $p$ maps $A$ onto line $E_{1} F_{1}$ and $D$ onto line $B_{1} C_{1}$. Consequently, $p$ maps line $A D$ onto $X_{A}=B_{1} C_{1} \\cap E_{1} F_{1}$. Furthermore, $p$ maps the line that bisects the angle formed by lines $A B$ and $E F$ and does not contain $I$ onto the midpoint of segment $A_{1} F_{1}$. Similarly, $p$ maps the line that bisects the angle formed by lines $A C$ and $E F$ and does not contain $I$ onto the midpoint of segment $A_{1} E_{1}$. So $p$ maps $I_{A}$ onto the midline of triangle $A_{1} E_{1} F_{1}$ opposite $A_{1}$. Similarly, $p$ maps $I_{D}$ onto the midline of triangle $D_{1} B_{1} C_{1}$ opposite $D_{1}$. Consequently, $p$ maps line $I_{A} I_{D}$ onto the intersection point $Y_{A}$ of this pair of midlines, and $p$ maps $T_{A}$ onto line $X_{A} Y_{A}$. Since the reflection of $H$ in line $B_{1} C_{1}$ lies on $\\omega, A_{1} H \\cdot H H_{A}$ equals half the power of $H$ with respect to $\\omega$. Similarly, $D_{1} H \\cdot H H_{D}$ equals half the power of $H$ with respect to $\\omega$. Then $A_{1} H \\cdot H H_{A}=D_{1} H \\cdot H H_{D}$ and $A_{1} H H_{D} \\sim D_{1} H H_{A}$. Since $\\angle H H_{D} L_{A}=90^{\\circ}=$ $\\angle H H_{A} L_{D}$, figures $A_{1} H H_{D} L_{A}$ and $D_{1} H H_{A} L_{D}$ are similar as well. Consequently, $$ \\frac{H L_{A}}{L_{A} A_{1}}=\\frac{H L_{D}}{L_{D} D_{1}}=s $$ as a signed ratio. Let the line through $A_{1}$ parallel to $E_{1} F_{1}$ and the line through $D_{1}$ parallel to $B_{1} C_{1}$ meet at $Z_{A}$. Then $H X_{A} \/ X_{A} Z_{A}=s$ and $Y_{A}$ is the midpoint of segment $X_{A} Z_{A}$. Therefore, $H$ lies on line $X_{A} Y_{A}$. This means that $T_{A}$ lies on the polar of $H$ with respect to $\\omega$, and by symmetry so do $T_{B}$ and $T_{C}$."} +{"year":2021,"label":"6","problem":"Triangles $A B C$ and $D E F$ share circumcircle $\\Omega$ and incircle $\\omega$ so that points $A, F$, $B, D, C$, and $E$ occur in this order along $\\Omega$. Let $\\Delta_{A}$ be the triangle formed by lines $A B, A C$, and $E F$, and define triangles $\\Delta_{B}, \\Delta_{C}, \\ldots, \\Delta_{F}$ similarly. Furthermore, let $\\Omega_{A}$ and $\\omega_{A}$ be the circumcircle and incircle of triangle $\\Delta_{A}$, respectively, and define circles $\\Omega_{B}, \\omega_{B}, \\ldots, \\Omega_{F}, \\omega_{F}$ similarly. (a) Prove that the two common external tangents to circles $\\Omega_{A}$ and $\\Omega_{D}$ and the two common external tangents to circles $\\omega_{A}$ and $\\omega_{D}$ are either concurrent or pairwise parallel. (b) Suppose that these four lines meet at point $T_{A}$, and define points $T_{B}$ and $T_{C}$ similarly. Prove that points $T_{A}, T_{B}$, and $T_{C}$ are collinear.","solution":" ![](https:\/\/cdn.mathpix.com\/cropped\/2024_11_19_148cb373ffbe5370bbc6g-17.jpg?height=818&width=754&top_left_y=1150&top_left_x=651) Let $I$ and $r$ be the center and radius of $\\omega$, and let $O$ and $R$ be the center and radius of $\\Omega$. Let $O_{A}$ and $I_{A}$ be the circumcenter and incenter of triangle $\\Delta_{A}$, and define $O_{B}$, $I_{B}, \\ldots, I_{F}$ similarly. Let $\\omega$ touch $E F$ at $A_{1}$, and define $B_{1}, C_{1}, \\ldots, F_{1}$ similarly. $$ \\left(\\Omega_{A} \\cup \\omega_{A}\\right) \\sim\\left(\\Omega_{D} \\cup \\omega_{D}\\right) $$ The four lines will concur at the homothetic center of these figures (possibly at infinity). Solution 2 (author) As in the first solution to part (a), let $h$ be inversion with respect to $\\omega$, let $\\gamma$ of center $K$ be the common Euler circle of triangles $A_{1} B_{1} C_{1}$ and $D_{1} E_{1} F_{1}$, and let $H$ be their common orthocenter. Let $K_{A}$ and $K_{D}$ be the centers of $\\gamma_{A}$ and $\\gamma_{D}$, respectively, and let $\\ell_{A}$ be the perpendicular bisector of segment $K_{A} K_{D}$. Since $\\gamma_{A}$ and $\\gamma_{D}$ are congruent (both of them are of radius $\\left.\\frac{1}{2} r\\right)$, they are reflections of each other across $\\ell_{A}$. Inversion $h$ maps the two common external tangents of $\\Omega_{A}$ and $\\Omega_{D}$ onto the two circles $\\alpha$ and $\\beta$ through $I$ that are tangent to both of $\\gamma_{A}$ and $\\gamma_{D}$ in the same way. (That is, either internally to both or externally to both.) Consequently, $\\alpha$ and $\\beta$ are reflections of each other in $\\ell_{A}$ and so their second point of intersection $S_{A}$, which $h$ maps $T_{A}$ onto, is the reflection of $I$ in $\\ell_{A}$. Define $\\ell_{B}, \\ell_{C}, S_{B}$, and $S_{C}$ similarly. Then $S_{B}$ is the reflection of $I$ in $\\ell_{B}$ and $S_{C}$ is the reflection of $I$ in $\\ell_{C}$. Therefore, all four points $I, S_{A}, S_{B}$, and $S_{C}$ lie on the circle of center $K$ that contains $I$. (This is also the circle on diameter $I H$.) Since points $S_{A}, S_{B}$, and $S_{C}$ are concyclic with $I$, their images $T_{A}, T_{B}$, and $T_{C}$ under $h$ are collinear, and the solution is complete."} +{"year":2021,"label":"6","problem":"Triangles $A B C$ and $D E F$ share circumcircle $\\Omega$ and incircle $\\omega$ so that points $A, F$, $B, D, C$, and $E$ occur in this order along $\\Omega$. Let $\\Delta_{A}$ be the triangle formed by lines $A B, A C$, and $E F$, and define triangles $\\Delta_{B}, \\Delta_{C}, \\ldots, \\Delta_{F}$ similarly. Furthermore, let $\\Omega_{A}$ and $\\omega_{A}$ be the circumcircle and incircle of triangle $\\Delta_{A}$, respectively, and define circles $\\Omega_{B}, \\omega_{B}, \\ldots, \\Omega_{F}, \\omega_{F}$ similarly. (a) Prove that the two common external tangents to circles $\\Omega_{A}$ and $\\Omega_{D}$ and the two common external tangents to circles $\\omega_{A}$ and $\\omega_{D}$ are either concurrent or pairwise parallel. (b) Suppose that these four lines meet at point $T_{A}$, and define points $T_{B}$ and $T_{C}$ similarly. Prove that points $T_{A}, T_{B}$, and $T_{C}$ are collinear.","solution":" ![](https:\/\/cdn.mathpix.com\/cropped\/2024_11_19_148cb373ffbe5370bbc6g-17.jpg?height=818&width=754&top_left_y=1150&top_left_x=651) Let $I$ and $r$ be the center and radius of $\\omega$, and let $O$ and $R$ be the center and radius of $\\Omega$. Let $O_{A}$ and $I_{A}$ be the circumcenter and incenter of triangle $\\Delta_{A}$, and define $O_{B}$, $I_{B}, \\ldots, I_{F}$ similarly. Let $\\omega$ touch $E F$ at $A_{1}$, and define $B_{1}, C_{1}, \\ldots, F_{1}$ similarly. $$ \\left(\\Omega_{A} \\cup \\omega_{A}\\right) \\sim\\left(\\Omega_{D} \\cup \\omega_{D}\\right) $$ The four lines will concur at the homothetic center of these figures (possibly at infinity). Solution 3 (Ankan Bhattacharya) From either of the first two solutions to part (a), we know that there is a circle $\\Omega_{A D}$ passing through $A$ and $D$ which is (internally) tangent to $\\Omega_{A}$ and $\\Omega_{D}$. By Monge's theorem applied to $\\Omega_{A}, \\Omega_{D}$, and $\\Omega_{A D}$, it follows that $A, D$, and $T_{A}$ are collinear. The inversion at $T_{A}$ swapping $\\Omega_{A}$ with $\\Omega_{D}$ also swaps $A$ with $D$ because $T_{A}$ lies on $A D$ and $A$ is not homologous to $D$. Let $\\Omega_{A}$ and $\\Omega_{D}$ meet $\\Omega$ again at $L_{A}$ and $L_{D}$. Since $A D L_{A} L_{D}$ is cyclic, the same inversion at $T_{A}$ also swaps $L_{A}$ and $L_{D}$, so $T_{A}=A D \\cap L_{A} L_{D}$. By Taiwan TST 2014, $L_{A}$ and $L_{D}$ are the tangency points of the $A$-mixtilinear and $D$-mixtilinear incircles, respectively, with $\\Omega$. Therefore $A L_{A} \\cap D L_{D}$ is the exsimilicenter $X$ of $\\Omega$ and $\\omega$. Then $T_{A}, T_{B}$, and $T_{C}$ all lie on the polar of $X$ with respect to $\\Omega$."} +{"year":2021,"label":"6","problem":"Triangles $A B C$ and $D E F$ share circumcircle $\\Omega$ and incircle $\\omega$ so that points $A, F$, $B, D, C$, and $E$ occur in this order along $\\Omega$. Let $\\Delta_{A}$ be the triangle formed by lines $A B, A C$, and $E F$, and define triangles $\\Delta_{B}, \\Delta_{C}, \\ldots, \\Delta_{F}$ similarly. Furthermore, let $\\Omega_{A}$ and $\\omega_{A}$ be the circumcircle and incircle of triangle $\\Delta_{A}$, respectively, and define circles $\\Omega_{B}, \\omega_{B}, \\ldots, \\Omega_{F}, \\omega_{F}$ similarly. (a) Prove that the two common external tangents to circles $\\Omega_{A}$ and $\\Omega_{D}$ and the two common external tangents to circles $\\omega_{A}$ and $\\omega_{D}$ are either concurrent or pairwise parallel. (b) Suppose that these four lines meet at point $T_{A}$, and define points $T_{B}$ and $T_{C}$ similarly. Prove that points $T_{A}, T_{B}$, and $T_{C}$ are collinear.","solution":" ![](https:\/\/cdn.mathpix.com\/cropped\/2024_11_19_148cb373ffbe5370bbc6g-17.jpg?height=818&width=754&top_left_y=1150&top_left_x=651) Let $I$ and $r$ be the center and radius of $\\omega$, and let $O$ and $R$ be the center and radius of $\\Omega$. Let $O_{A}$ and $I_{A}$ be the circumcenter and incenter of triangle $\\Delta_{A}$, and define $O_{B}$, $I_{B}, \\ldots, I_{F}$ similarly. Let $\\omega$ touch $E F$ at $A_{1}$, and define $B_{1}, C_{1}, \\ldots, F_{1}$ similarly. $$ \\left(\\Omega_{A} \\cup \\omega_{A}\\right) \\sim\\left(\\Omega_{D} \\cup \\omega_{D}\\right) $$ The four lines will concur at the homothetic center of these figures (possibly at infinity). Solution 4 (Andrew Gu) We show that $T_{A}$ lies on the radical axis of the point circle at $I$ and $\\Omega$, which will solve the problem. Let $I_{A}$ and $I_{D}$ be the centers of $\\omega_{A}$ and $\\omega_{D}$ respectively. By the Monge's theorem applied to $\\omega, \\omega_{A}$, and $\\omega_{D}$, points $A, D$, and $T_{A}$ are collinear. Additionally, these other triples are collinear: $\\left(A, I_{A}, I\\right),\\left(D, I_{D}, I\\right),\\left(I_{A}, I_{D}, T\\right)$. By Menelaus's theorem, we have $$ \\frac{T_{A} D}{T_{A} A}=\\frac{I_{A} I}{I_{A} A} \\cdot \\frac{I_{D} D}{I_{D} I} . $$ If $s$ is the length of the side opposite $A$ in $\\Delta_{A}$, then we compute $$ \\begin{aligned} \\frac{I_{A} I}{I_{A} A} & =\\frac{s \/ \\cos (A \/ 2)}{r_{A} \/ \\sin (A \/ 2)} \\\\ & =\\frac{2 R_{A} \\sin (A) \\sin (A \/ 2)}{\\cos (A \/ 2)} \\\\ & =\\frac{4 R_{A} \\sin ^{2}(A \/ 2)}{r_{A}} \\\\ & =\\frac{4 R_{A} r^{2}}{r_{A} A I^{2}} . \\end{aligned} $$ From part (a), we know that $\\frac{R_{A}}{r_{A}}=\\frac{R_{D}}{r_{D}}$. Therefore, doing a similar calculation for $\\frac{I_{D} D}{I_{D} I}$, we get $$ \\begin{aligned} \\frac{T_{A} D}{T_{A} A} & =\\frac{I_{A} I}{I_{A} A} \\cdot \\frac{I_{D} D}{I_{D} I} \\\\ & =\\frac{4 R_{A} r^{2}}{r_{A} A I^{2}} \\cdot \\frac{r_{D} D I^{2}}{4 R_{D} r^{2}} \\\\ & =\\frac{D I^{2}}{A I^{2}} \\end{aligned} $$ Thus $T_{A}$ is the point where the tangent to $(A I D)$ at $I$ meets $A D$ and $T_{A} I^{2}=T_{A} A \\cdot T_{A} D$. This shows what we claimed at the start."} +{"year":2021,"label":"6","problem":"Triangles $A B C$ and $D E F$ share circumcircle $\\Omega$ and incircle $\\omega$ so that points $A, F$, $B, D, C$, and $E$ occur in this order along $\\Omega$. Let $\\Delta_{A}$ be the triangle formed by lines $A B, A C$, and $E F$, and define triangles $\\Delta_{B}, \\Delta_{C}, \\ldots, \\Delta_{F}$ similarly. Furthermore, let $\\Omega_{A}$ and $\\omega_{A}$ be the circumcircle and incircle of triangle $\\Delta_{A}$, respectively, and define circles $\\Omega_{B}, \\omega_{B}, \\ldots, \\Omega_{F}, \\omega_{F}$ similarly. (a) Prove that the two common external tangents to circles $\\Omega_{A}$ and $\\Omega_{D}$ and the two common external tangents to circles $\\omega_{A}$ and $\\omega_{D}$ are either concurrent or pairwise parallel. (b) Suppose that these four lines meet at point $T_{A}$, and define points $T_{B}$ and $T_{C}$ similarly. Prove that points $T_{A}, T_{B}$, and $T_{C}$ are collinear.","solution":" ![](https:\/\/cdn.mathpix.com\/cropped\/2024_11_19_148cb373ffbe5370bbc6g-17.jpg?height=818&width=754&top_left_y=1150&top_left_x=651) Let $I$ and $r$ be the center and radius of $\\omega$, and let $O$ and $R$ be the center and radius of $\\Omega$. Let $O_{A}$ and $I_{A}$ be the circumcenter and incenter of triangle $\\Delta_{A}$, and define $O_{B}$, $I_{B}, \\ldots, I_{F}$ similarly. Let $\\omega$ touch $E F$ at $A_{1}$, and define $B_{1}, C_{1}, \\ldots, F_{1}$ similarly. $$ \\left(\\Omega_{A} \\cup \\omega_{A}\\right) \\sim\\left(\\Omega_{D} \\cup \\omega_{D}\\right) $$ The four lines will concur at the homothetic center of these figures (possibly at infinity). Solution 5 (Ankit Bisain) As in the previous solution, it suffices to show that $\\frac{I_{A} I}{A I_{A}} \\cdot \\frac{D I_{D}}{I_{D} I}=\\frac{D I^{2}}{A I^{2}}$. Let $A I$ and $D I$ meet $\\Omega$ again at $M$ and $N$, respectively. Let $\\ell$ be the line parallel to $B C$ and tangent to $\\omega$ but different from $B C$. Then $$ \\frac{D I_{D}}{I_{D} I}=\\frac{d(D, B C)}{d(B C, \\ell)}=\\frac{D B \\cdot D C \/ 2 R}{2 r}=\\frac{M I^{2}-M D^{2}}{4 R r} $$ Since $I D M \\sim I A N$, we have $$ \\frac{D I_{D}}{I_{D} I} \\cdot \\frac{I_{A} I}{A I_{A}}=\\frac{M I^{2}-M D^{2}}{N I^{2}-N A^{2}}=\\frac{D I^{2}}{A I^{2}} $$ as desired. Remark (Author comments on generalization of part (b) with a circumscribed hexagram). Let triangles $A B C$ and $D E F$ be circumscribed about the same circle $\\omega$ so that they form a hexagram. However, we do not require anymore that they are inscribed in the same circle. Define circles $\\Omega_{A}, \\omega_{A}, \\ldots, \\omega_{F}$ as in the problem. Let $T_{A}^{\\text {Circ }}$ be the intersection point of the two common external tangents to circles $\\Omega_{A}$ and $\\Omega_{D}$, and define points $T_{B}^{\\text {Circ }}$ and $T_{C}^{\\text {Circ }}$ similarly. Also let $T_{A}^{\\text {In }}$ be the intersection point of the two common external tangents to circles $\\omega_{A}$ and $\\omega_{D}$, and define points $T_{B}^{\\mathrm{In}}$ and $T_{C}^{\\mathrm{In}}$ similarly. Then points $T_{A}^{\\text {Circ }}, T_{B}^{\\text {Circ }}$, and $T_{C}^{\\mathrm{Circ}}$ are collinear and points $T_{A}^{\\mathrm{In}}, T_{B}^{\\mathrm{In}}$, and $T_{C}^{\\mathrm{In}}$ are also collinear. For the incircles part of the generalisation, we start out as in the first solution to part (b) of the problem, and eventually we reduce everything to the following: Let points $A_{1}, B_{1}, C_{1}, D_{1}, E_{1}$, and $F_{1}$ lie on circle $\\omega$. Let lines $B_{1} C_{1}$ and $E_{1} F_{1}$ meet at point $X_{A}$, let the line through $A_{1}$ parallel to $B_{1} C_{1}$ and the line through $D_{1}$ parallel to $E_{1} F_{1}$ meet at point $Z_{A}$, and define points $X_{B}, Z_{B}, X_{C}$, and $Z_{C}$ similarly. Then lines $X_{A} Z_{A}$, $X_{B} Z_{B}$, and $X_{C} Z_{C}$ are concurrent. Take $\\omega$ as the unit circle and assign complex numbers $u, v, w, x, y$, and $z$ to points $A_{1}$, $F_{1}, B_{1}, D_{1}, C_{1}$, and $E_{1}$, respectively, so that when we permute $u, v, w, x, y$, and $z$ cyclically the configuration remains unchanged. Then by standard complex bash formulas we obtain that each two out of our three lines meet at $\\varphi \/ \\psi$, where $$ \\varphi=\\sum_{\\mathrm{Cyc}} u^{2} v w(w x-w y+x y)(y-z) $$ and $$ \\psi=-u^{2} w^{2} y^{2}-v^{2} x^{2} z^{2}-4 u v w x y z+\\sum_{\\mathrm{Cyc}} u^{2}(v w x y-v w x z+v w y z-v x y z+w x y z) $$ (But the calculations were too difficult for me to do by hand, so I used SymPy.) Remark (Author comments on generalization of part (b) with an inscribed hexagram). Let triangles $A B C$ and $D E F$ be inscribed inside the same circle $\\Omega$ so that they form a hexagram. However, we do not require anymore that they are circumscribed about the same circle. Define points $T_{A}^{\\text {Circ }}, T_{B}^{\\text {Circ }}, \\ldots, T_{C}^{\\mathrm{In}}$ as in the previous remark. It looks like once again points $T_{A}^{\\text {Circ }}, T_{B}^{\\text {Circ }}$, and $T_{C}^{\\text {Circ }}$ are collinear and points $T_{A}^{\\mathrm{In}}, T_{B}^{\\mathrm{In}}$, and $T_{C}^{\\mathrm{In}}$ are also collinear. However, I do not have proofs of these claims. Remark (Further generalization from Andrew Gu). Let $A B C$ and $D E F$ be triangles which share an inconic, or equivalently share a circumconic. Define points $T_{A}^{\\text {Circ }}, T_{B}^{\\text {Circ }}, \\ldots, T_{C}^{\\mathrm{In}}$ as in the previous remarks. Then it is conjectured that points $T_{A}^{\\text {Circ }}, T_{B}^{\\text {Circ }}$, and $T_{C}^{\\text {Circ }}$ are collinear and points $T_{A}^{\\mathrm{In}}, T_{B}^{\\mathrm{In}}$, and $T_{C}^{\\mathrm{In}}$ are also collinear. (Note that extraversion may be required depending on the configuration of points, e.g. excircles instead of incircles.) Additionally, it appears that the insimilicenters of the circumcircles lie on a line perpendicular to the line through $T_{A}^{\\text {Circ }}, T_{B}^{\\text {Circ }}$, and $T_{C}^{\\text {Circ }}$."} +{"year":2021,"label":"7","problem":"Let $M$ be a finite set of lattice points and $n$ be a positive integer. A mine-avoiding path is a path of lattice points with length $n$, beginning at $(0,0)$ and ending at a point on the line $x+y=n$, that does not contain any point in $M$. Prove that if there exists a mine-avoiding path, then there exist at least $2^{n-|M|}$ mine-avoiding paths.","solution":" \u3010 Solution 1. We prove the statement by induction on $n$. We use $n=0$ as a base case, where the statement follows from $1 \\geq 2^{-|M|}$. For the inductive step, let $n>0$. There exists at least one mine-avoiding path, which must pass through either $(0,1)$ or $(1,0)$. We consider two cases: Case 1: there exist mine-avoiding paths starting at both $(1,0)$ and $(0,1)$. By the inductive hypothesis, there are at least $2^{n-1-|M|}$ mine-avoiding paths starting from each of $(1,0)$ and $(0,1)$. Then the total number of mine-avoiding paths is at least $2^{n-1-|M|}+2^{n-1-|M|}=2^{n-|M|}$. Case 2: only one of $(1,0)$ and $(0,1)$ is on a mine-avoiding path. Without loss of generality, suppose no mine-avoiding path starts at $(0,1)$. Then some element of $M$ must be of the form $(0, k)$ for $1 \\leq k \\leq n$ in order to block the path along the $y$-axis. This element can be ignored for any mine-avoiding path starting at (1,0). By the inductive hypothesis, there are at least $2^{n-1-(|M|-1)}=2^{n-|M|}$ mine-avoiding paths. This completes the induction step, which solves the problem."} +{"year":2021,"label":"7","problem":"Let $M$ be a finite set of lattice points and $n$ be a positive integer. A mine-avoiding path is a path of lattice points with length $n$, beginning at $(0,0)$ and ending at a point on the line $x+y=n$, that does not contain any point in $M$. Prove that if there exists a mine-avoiding path, then there exist at least $2^{n-|M|}$ mine-avoiding paths.","solution":" ## \u3010 Solution 2. ## Lemma If $|M|d$, the pattern changes to $$ S_{n}=\\sum_{j=1}^{d}(-1)^{j+1} c_{j} S_{n-j} $$ ## Lemma All of the $c_{i}$ are integers except for $c_{d}$. Furthermore, $c_{d}$ is $1 \/ p$ times an integer. $$ \\Phi_{q}(x)=1+x^{p^{r-1}}+x^{2 p^{r-1}}+\\cdots+x^{(p-1) p^{r-1}} $$ The polynomial $$ Q(x)=1+(1+x)^{p^{r-1}}+(1+x)^{2 p^{r-1}}+\\cdots+(1+x)^{(p-1) p^{r-1}} $$ has roots $\\omega-1$ for $\\omega \\in S_{q}$, so it is equal to $p(-x)^{d} P(-1 \/ x)$ by comparing constant coefficients. Comparing the remaining coefficients, we find that $c_{n}$ is $1 \/ p$ times the $x^{n}$ coefficient of $Q$. Since $(x+y)^{p} \\equiv x^{p}+y^{p}(\\bmod p)$, we conclude that, modulo $p$, $$ \\begin{aligned} Q(x) & \\equiv 1+\\left(1+x^{p^{r-1}}\\right)+\\left(1+x^{p^{r-1}}\\right)^{2}+\\cdots+\\left(1+x^{p^{r-1}}\\right)^{p-1} \\\\ & \\equiv\\left[\\left(1+x^{p^{r-1}}\\right)^{p}-1\\right] \/ x^{p^{r-1}} \\end{aligned} $$ Since $\\binom{p}{j}$ is a multiple of $p$ when $0d$ : - If $\\nu_{p}\\left(S_{n-j}\\right) \\geq \\ell$ for all $1 \\leq j \\leq d$ and $\\nu_{p}\\left(S_{n-d}\\right) \\geq \\ell+1$, then $\\nu_{p}\\left(S_{n}\\right) \\geq \\ell$. - If $\\nu_{p}\\left(S_{n-j}\\right) \\geq \\ell$ for all $1 \\leq j \\leq d$ and $\\nu_{p}\\left(S_{n-d}\\right)=\\ell$, then $\\nu_{p}\\left(S_{n}\\right)=\\ell-1$. Together, these prove the claim by induction. By the claim, the smallest $n$ for which $\\nu_{p}\\left(S_{n}\\right)<0$ (equivalent to $S_{n}$ not being an integer, by the recurrences) is $$ n=(r-1) d+m+1=((p-1) r-1) p^{r-1}+1 $$ Remark. The original proposal was the following more general version: Let $n$ be an integer with prime power factorization $q_{1} \\cdots q_{m}$. Let $S_{n}$ denote the set of primitive $n$th roots of unity. Find all tuples of nonnegative integers $\\left(z_{1}, \\ldots, z_{m}\\right)$ such that $$ \\sum_{\\omega \\in S_{n}} \\frac{f(\\omega)}{\\left(1-\\omega^{n \/ q_{1}}\\right)^{z_{1}} \\cdots\\left(1-\\omega^{n \/ q_{m}}\\right)^{z_{m}}} \\in \\mathbb{Z} $$ for all polynomials $f \\in \\mathbb{Z}[x]$. The maximal $z_{i}$ are exponents in the prime ideal factorization of the different ideal of the cyclotomic extension $\\mathbb{Q}\\left(\\zeta_{n}\\right) \/ \\mathbb{Q}$. Remark. Let $F=\\left(x^{p}-1\\right) \/(x-1)$ be the minimal polynomial of $\\zeta_{p}=e^{2 \\pi i \/ p}$ over $\\mathbb{Q}$. A calculation of Euler shows that $$ \\left(\\mathbb{Z}\\left[\\zeta_{p}\\right]\\right)^{*}:=\\left\\{\\alpha=g\\left(\\zeta_{p}\\right) \\in \\mathbb{Q}\\left[\\zeta_{p}\\right]: \\sum_{\\omega \\in S_{p}} f(\\omega) g(\\omega) \\in \\mathbb{Z} \\forall f \\in \\mathbb{Z}[x]\\right\\}=\\frac{1}{F^{\\prime}\\left(\\zeta_{p}\\right)} \\cdot \\mathbb{Z}\\left[\\zeta_{p}\\right], $$ where $$ F^{\\prime}\\left(\\zeta_{p}\\right)=\\frac{p \\zeta_{p}^{p-1}-\\left[1+\\zeta_{p}+\\cdots+\\zeta_{p}^{p-1}\\right]}{1-\\zeta_{p}}=p\\left(1-\\zeta_{p}\\right)^{-1} \\zeta_{p}^{p-1} $$ is $\\left(1-\\zeta_{p}\\right)^{[p-1]-1}=\\left(1-\\zeta_{p}\\right)^{p-2}$ times a unit of $\\mathbb{Z}\\left[\\zeta_{p}\\right]$. Here, $\\left(\\mathbb{Z}\\left[\\zeta_{p}\\right]\\right)^{*}$ is the dual lattice of $\\mathbb{Z}\\left[\\zeta_{p}\\right]$. Remark. Let $K=\\mathbb{Q}(\\omega)$, so $(p)$ factors as $(1-\\omega)^{p-1}$ in the ring of integers $\\mathcal{O}_{K}$ (which, for cyclotomic fields, can be shown to be $\\mathbb{Z}[\\omega])$. In particular, the ramification index $e$ of $(1-\\omega)$ over $p$ is the exponent, $p-1$. Since $e=p-1$ is not divisible by $p$, we have so-called tame ramification. Now by the ramification theory of Dedekind's different ideal, the exponent $z_{1}$ that works when $n=p$ is $e-1=p-2$. Higher prime powers are more interesting because of wild ramification: $p$ divides $\\phi\\left(p^{r}\\right)=$ $p^{r-1}(p-1)$ if and only if $r>1$. (This is a similar phenomena to how Hensel's lemma for $x^{2}-c$ is more interesting mod powers of 2 than mod odd prime powers.) Remark. Let $F=\\left(x^{q}-1\\right) \/\\left(x^{q \/ p}-1\\right)$ be the minimal polynomial of $\\zeta_{q}=e^{2 \\pi i \/ q}$ over $\\mathbb{Q}$. The aforementioned calculation of Euler shows that $$ \\left(\\mathbb{Z}\\left[\\zeta_{q}\\right]\\right)^{*}:=\\left\\{\\alpha=g\\left(\\zeta_{q}\\right) \\in \\mathbb{Q}\\left[\\zeta_{q}\\right]: \\sum_{\\omega \\in S_{q}} f(\\omega) g(\\omega) \\in \\mathbb{Z} \\forall f \\in \\mathbb{Z}[x]\\right\\}=\\frac{1}{F^{\\prime}\\left(\\zeta_{q}\\right)} \\cdot \\mathbb{Z}\\left[\\zeta_{q}\\right] $$ where the chain rule implies (using the computation from the prime case) $$ F^{\\prime}\\left(\\zeta_{q}\\right)=\\left[p\\left(1-\\zeta_{p}\\right)^{-1} \\zeta_{p}^{p-1}\\right] \\cdot \\frac{q}{p} \\zeta_{q}^{(q \/ p)-1}=q\\left(1-\\zeta_{p}\\right)^{-1} \\zeta_{q}^{-1} $$ is $\\left(1-\\zeta_{q}\\right)^{r \\phi(q)-q \/ p}=\\left(1-\\zeta_{q}\\right)^{z_{q}}$ times a unit of $\\mathbb{Z}\\left[\\zeta_{q}\\right]$."} diff --git a/USA_TSTST/segmented/en-sols-TSTST-2022.jsonl b/USA_TSTST/segmented/en-sols-TSTST-2022.jsonl new file mode 100644 index 0000000000000000000000000000000000000000..0058c5f52ad270ddf8d9572537cb1fddca1cc80a --- /dev/null +++ b/USA_TSTST/segmented/en-sols-TSTST-2022.jsonl @@ -0,0 +1,12 @@ +{"year":2022,"label":"1","problem":"Let $n$ be a positive integer. Find the smallest positive integer $k$ such that for any set $S$ of $n$ points in the interior of the unit square, there exists a set of $k$ rectangles such that the following hold: - The sides of each rectangle are parallel to the sides of the unit square. - Each point in $S$ is not in the interior of any rectangle. - Each point in the interior of the unit square but not in $S$ is in the interior of at least one of the $k$ rectangles. (The interior of a polygon does not contain its boundary.)","solution":" We claim the answer is $k=2 n+2$. The lower bound is given by picking $$ S=\\left\\{\\left(s_{1}, s_{1}\\right),\\left(s_{2}, s_{2}\\right), \\ldots,\\left(s_{n}, s_{n}\\right)\\right\\} $$ for some real numbers $00$. The four rectangles covering each of $$ \\left(s_{1}-\\varepsilon, s_{1}\\right),\\left(s_{1}, s_{1}-\\varepsilon\\right),\\left(s_{n}+\\varepsilon, s_{n}\\right),\\left(s_{n}, s_{n}+\\varepsilon\\right) $$ cannot cover any other points in $S^{\\prime}$; all other rectangles can only cover at most 2 points in $S^{\\prime}$, giving a bound of $$ k \\geq 4+\\frac{\\left|S^{\\prime}\\right|-4}{2}=2 n+2 $$ ![](https:\/\/cdn.mathpix.com\/cropped\/2024_11_19_1799270e91f74486a977g-05.jpg?height=812&width=806&top_left_y=242&top_left_x=628) To prove that $2 n+2$ rectangles are sufficient, assume that the number of distinct $y$-coordinates is at least the number of distinct $x$-coordinates. Let $$ 0=x_{0}0$ gives a total of $$ (m+n+2)+(n-m)=2 n+2 $$ rectangles."} +{"year":2022,"label":"2","problem":"Let $A B C$ be a triangle. Let $\\theta$ be a fixed angle for which $$ \\theta<\\frac{1}{2} \\min (\\angle A, \\angle B, \\angle C) $$ Points $S_{A}$ and $T_{A}$ lie on segment $B C$ such that $\\angle B A S_{A}=\\angle T_{A} A C=\\theta$. Let $P_{A}$ and $Q_{A}$ be the feet from $B$ and $C$ to $\\overline{A S_{A}}$ and $\\overline{A T_{A}}$ respectively. Then $\\ell_{A}$ is defined as the perpendicular bisector of $\\overline{P_{A} Q_{A}}$. Define $\\ell_{B}$ and $\\ell_{C}$ analogously by repeating this construction two more times (using the same value of $\\theta$ ). Prove that $\\ell_{A}, \\ell_{B}$, and $\\ell_{C}$ are concurrent or all parallel.","solution":" We discard the points $S_{A}$ and $T_{A}$ since they are only there to direct the angles correctly in the problem statement. \u3010 First solution, by author. Let $X$ be the projection from $C$ to $A P_{A}, Y$ be the projection from $B$ to $A Q_{A}$. ![](https:\/\/cdn.mathpix.com\/cropped\/2024_11_19_1799270e91f74486a977g-07.jpg?height=809&width=792&top_left_y=1292&top_left_x=632) Claim - Line $\\ell_{A}$ passes through $M_{A}$, the midpoint of $B C$. Also, quadrilateral $P_{A} Q_{A} Y X$ is cyclic with circumcenter $M_{A}$. $$ A P_{A} \\cdot A X=A B \\cdot A C \\cdot \\cos \\theta \\cos (\\angle A-\\theta)=A Q_{A} \\cdot A Y $$ it follows that $P_{A}, Q_{A}, Y, X$ are concyclic by power of a point. Moreover, by projection, the perpendicular bisector of $P_{A} X$ passes through $M_{A}$, similar for $Q_{A} Y$, implying that $M_{A}$ is the center of $P_{A} Q_{A} Y X$. Hence $\\ell_{A}$ passes through $M_{A}$. $$ \\text { Claim }-\\measuredangle\\left(M_{A} M_{C}, \\ell_{A}\\right)=\\measuredangle Y P_{A} Q_{A} $$ Therefore, $$ \\frac{\\sin \\angle\\left(M_{A} M_{C}, \\ell_{A}\\right)}{\\sin \\angle\\left(\\ell_{A}, M_{A} M_{B}\\right)}=\\frac{\\sin \\angle Y P_{A} Q_{A}}{\\sin \\angle P_{A} Q_{A} X}=\\frac{Y Q_{A}}{X P_{A}}=\\frac{B C \\sin (\\angle C+\\theta)}{B C \\sin (\\angle B+\\theta)}=\\frac{\\sin (\\angle C+\\theta)}{\\sin (\\angle B+\\theta)}, $$ and we conclude by trig Ceva theorem."} +{"year":2022,"label":"2","problem":"Let $A B C$ be a triangle. Let $\\theta$ be a fixed angle for which $$ \\theta<\\frac{1}{2} \\min (\\angle A, \\angle B, \\angle C) $$ Points $S_{A}$ and $T_{A}$ lie on segment $B C$ such that $\\angle B A S_{A}=\\angle T_{A} A C=\\theta$. Let $P_{A}$ and $Q_{A}$ be the feet from $B$ and $C$ to $\\overline{A S_{A}}$ and $\\overline{A T_{A}}$ respectively. Then $\\ell_{A}$ is defined as the perpendicular bisector of $\\overline{P_{A} Q_{A}}$. Define $\\ell_{B}$ and $\\ell_{C}$ analogously by repeating this construction two more times (using the same value of $\\theta$ ). Prove that $\\ell_{A}, \\ell_{B}$, and $\\ell_{C}$ are concurrent or all parallel.","solution":" We discard the points $S_{A}$ and $T_{A}$ since they are only there to direct the angles correctly in the problem statement. \u3010 Second solution via Jacobi, by Maxim Li. Let $D$ be the foot of the $A$-altitude. Note that line $B C$ is the external angle bisector of $\\angle P_{A} D Q_{A}$. Claim - $\\left(D P_{A} Q_{A}\\right)$ passes through the midpoint $M_{A}$ of $B C$. It follows that $M_{A}$ lies on $\\ell_{A}$; we need to identify a second point. We'll use the circumcenter $O_{A}$ of $\\left(D P_{A} Q_{A}\\right)$. The perpendicular bisector of $D P_{A}$ passes through $M_{C}$; indeed, we can easily show the angle it makes with $M_{C} M_{A}$ is $90^{\\circ}-\\theta-C$, so $\\angle O_{A} M_{C} M_{A}=90-\\theta-C$, and then by analogous angle-chasing we can finish with Jacobi's theorem on $\\triangle M_{A} M_{B} M_{C}$."} +{"year":2022,"label":"3","problem":"Determine all positive integers $N$ for which there exists a strictly increasing sequence of positive integers $s_{0}2^{21}$ possible colorings. If Bob makes less than 22 queries, then he can only output $2^{21}$ possible colorings, which means he is wrong on some coloring. Now we show Bob can always win in 22 queries. A key observation is that the set of red points is convex, as is the set of blue points, so if a set of points is all the same color, then their convex hull is all the same color. ## Lemma Let $B_{0}, \\ldots, B_{k+1}$ be equally spaced points on a circular arc such that colors of $B_{0}$ and $B_{k+1}$ differ and are known. Then it is possible to determine the colors of $B_{1}$, $\\ldots, B_{k}$ in $\\left\\lceil\\log _{2} k\\right\\rceil$ queries. ## Lemma Let $B_{0}, \\ldots, B_{k+1}$ be equally spaced points on a circular arc such that colors of $B_{0}, B_{\\lceil k \/ 2\\rceil}, B_{k+1}$ are both red and are known. Then at least one of the following holds: all of $B_{1}, \\ldots, B_{\\lceil k \/ 2\\rceil}$ are red or all of $B_{\\lceil k \/ 2\\rceil}, \\ldots, B_{k}$ are red. Furthermore, in one query we can determine which one of the cases holds. Now the strategy is: Bob picks $A_{1}$. WLOG it is red. Now suppose Bob does not know the colors of $\\leq 2^{k}-1$ points $A_{i}, \\ldots, A_{j}$ with $j-i+1 \\leq 2^{k}-1$ and knows the rest are red. I claim Bob can win in $2 k-1$ queries. First, if $k=1$, there is one point and he wins by querying the point, so the base case holds, so assume $k>1$. Bob queries $A_{i+\\lceil(j-i+1) \/ 2\\rceil}$. If it is blue, he finishes in $2 \\log _{2}\\lceil(j-i+1) \/ 2\\rceil \\leq 2(k-1)$ queries by the first lemma, for a total of $2 k-1$ queries. If it is red, he can query one more point and learn some half of $A_{i}, \\ldots, A_{j}$ that are red by the second lemma, and then he has reduced it to the case with $\\leq 2^{k-1}-1$ points in two queries, at which point we induct."} +{"year":2022,"label":"6","problem":"Let $O$ and $H$ be the circumcenter and orthocenter, respectively, of an acute scalene triangle $A B C$. The perpendicular bisector of $\\overline{A H}$ intersects $\\overline{A B}$ and $\\overline{A C}$ at $X_{A}$ and $Y_{A}$ respectively. Let $K_{A}$ denote the intersection of the circumcircles of triangles $O X_{A} Y_{A}$ and $B O C$ other than $O$. Define $K_{B}$ and $K_{C}$ analogously by repeating this construction two more times. Prove that $K_{A}, K_{B}, K_{C}$, and $O$ are concyclic.","solution":" \\ First solution, by author. Let $\\odot O X_{A} Y_{A}$ intersects $A B, A C$ again at $U, V$. Then by Reim's theorem $U V C B$ are concyclic. Hence the radical axis of $\\odot O X_{A} Y_{A}, \\odot O B C$ and $\\odot(U V C B)$ are concurrent, i.e. $O K_{A}, B C, U V$ are concurrent, Denote the intersection as $K_{A}^{*}$, which is indeed the inversion of $K_{A}$ with respect to $\\odot O$. (The inversion sends $\\odot O B C$ to the line $B C$ ). Let $P_{A}, P_{B}, P_{C}$ be the circumcenters of $\\triangle O B C, \\triangle O C A, \\triangle O A B$ respectively. Claim $-K_{A}^{*}$ coincides with the intersection of $P_{B} P_{C}$ and $B C$. Finally by Desargue's theorem, it suffices to show that $A P_{A}, B P_{B}, C P_{C}$ are concurrent. Note that $$ \\begin{aligned} & d\\left(P_{A}, A B\\right)=P_{A} B \\sin \\left(90^{\\circ}+\\angle C-\\angle A\\right) \\\\ & d\\left(P_{A}, A C\\right)=P_{A} C \\sin \\left(90^{\\circ}+\\angle B-\\angle A\\right) \\end{aligned} $$ Hence the symmetric product and trig Ceva finishes the proof. ![](https:\/\/cdn.mathpix.com\/cropped\/2024_11_19_1799270e91f74486a977g-15.jpg?height=1132&width=1194&top_left_y=245&top_left_x=431)"} +{"year":2022,"label":"6","problem":"Let $O$ and $H$ be the circumcenter and orthocenter, respectively, of an acute scalene triangle $A B C$. The perpendicular bisector of $\\overline{A H}$ intersects $\\overline{A B}$ and $\\overline{A C}$ at $X_{A}$ and $Y_{A}$ respectively. Let $K_{A}$ denote the intersection of the circumcircles of triangles $O X_{A} Y_{A}$ and $B O C$ other than $O$. Define $K_{B}$ and $K_{C}$ analogously by repeating this construction two more times. Prove that $K_{A}, K_{B}, K_{C}$, and $O$ are concyclic.","solution":" I Second solution, from Jeffrey Kwan. Let $O_{A}$ be the circumcenter of $\\triangle A X_{A} Y_{A}$. The key claim is that: Claim $-O_{A} X_{A} Y_{A} O$ is cyclic. $$ \\frac{A X_{A}}{A B}=\\frac{A H \/ 2}{A D}=\\frac{R \\cos A}{A D} $$ and so $$ \\frac{A O}{A D}=R \\cdot \\frac{A X_{A}}{A B \\cdot R \\cos A}=\\frac{A X_{A}}{A E}=\\frac{A Y_{A}}{A F} $$ Hence $\\angle X_{A} O Y_{A}=180^{\\circ}-2 \\angle A=180^{\\circ}-\\angle X_{A} O_{A} Y_{A}$, which proves the claim. Let $P_{A}$ be the circumcenter of $\\triangle O B C$, and define $P_{B}, P_{C}$ similarly. By the claim, $A$ is the exsimilicenter of $\\left(O X_{A} Y_{A}\\right)$ and $(O B C)$, so $A P_{A}$ is the line between their two centers. In particular, $A P_{A}$ is the perpendicular bisector of $O K_{A}$. ![](https:\/\/cdn.mathpix.com\/cropped\/2024_11_19_1799270e91f74486a977g-16.jpg?height=1303&width=1020&top_left_y=245&top_left_x=521) Claim $-A P_{A}, B P_{B}, C P_{C}$ concur at $T$. Now $T$ lies on the perpendicular bisectors of $O K_{A}, O K_{B}$, and $O K_{C}$. Hence $O K_{A} K_{B} K_{C}$ is cyclic with center $T$, as desired."} +{"year":2022,"label":"7","problem":"Let $A B C D$ be a parallelogram. Point $E$ lies on segment $C D$ such that $$ 2 \\angle A E B=\\angle A D B+\\angle A C B, $$ and point $F$ lies on segment $B C$ such that $$ 2 \\angle D F A=\\angle D C A+\\angle D B A . $$ Let $K$ be the circumcenter of triangle $A B D$. Prove that $K E=K F$.","solution":" Let the circle through $A, B$, and $E$ intersect $C D$ again at $E^{\\prime}$, and let the circle through $D$, $A$, and $F$ intersect $B C$ again at $F^{\\prime}$. Now $A B E E^{\\prime}$ and $D A F^{\\prime} F$ are cyclic quadrilaterals with two parallel sides, so they are isosceles trapezoids. From $K A=K B$, it now follows that $K E=K E^{\\prime}$, whereas from $K A=K D$ it follows that $K F=K F^{\\prime}$. Next, let the circle through $A, B$, and $E$ intersect $A C$ again at $S$. Then $$ \\angle A S B=\\angle A E B=\\frac{1}{2}(\\angle A D B+\\angle A C B)=\\frac{1}{2}(\\angle A D B+\\angle D A C)=\\frac{1}{2} \\angle A M B, $$ where $M$ is the intersection of $A C$ and $B D$. From $\\angle A S B=\\frac{1}{2} \\angle A M B$, it follows that $M S=M B$, so $S$ is the point on $M C$ such that $M S=M B=M D$. By symmetry, the circle through $A, D$, and $F$ also passes through $S$, and it follows that the line $A S$ is the radical axis of the circles $(A B E)$ and $(A D F)$. By power of a point, we now obtain $$ C E \\cdot C E^{\\prime}=C S \\cdot C A=C F \\cdot C F^{\\prime}, $$ from which it follows that $E, F, E^{\\prime}$, and $F^{\\prime}$ are concyclic. The segments $E E^{\\prime}$ and $F F^{\\prime}$ are not parallel, so their perpendicular bisectors only meet at one point, which is $K$. Hence $K E=K F$."} +{"year":2022,"label":"8","problem":"Find all functions $f: \\mathbb{N} \\rightarrow \\mathbb{Z}$ such that $$ \\left\\lfloor\\frac{f(m n)}{n}\\right\\rfloor=f(m) $$ for all positive integers $m, n$.","solution":" There are two families of functions that work: for each $\\alpha \\in \\mathbb{R}$ the function $f(n)=\\lfloor\\alpha n\\rfloor$, and for each $\\alpha \\in \\mathbb{R}$ the function $f(n)=\\lceil\\alpha n\\rceil-1$. (For irrational $\\alpha$ these two functions coincide.) It is straightforward to check that these functions indeed work; essentially, this follows from the identity $$ \\left\\lfloor\\frac{\\lfloor x n\\rfloor}{n}\\right\\rfloor=\\lfloor x\\rfloor $$ which holds for all positive integers $n$ and real numbers $x$. We now show that every function that works must be of one of the above forms. Let $f$ be a function that works, and define the sequence $a_{1}, a_{2}, \\ldots$ by $a_{n}=f(n!) \/ n!$. Applying the give condition with $(n!, n+1)$ yields $a_{n+1} \\in\\left[a_{n}, a_{n}+\\frac{1}{n!}\\right)$. It follows that the sequence $a_{1}, a_{2}, \\ldots$ is non-decreasing and bounded from above by $a_{1}+e$, so this sequence must converge to some limit $\\alpha$. If there exists a $k$ such that $a_{k}=\\alpha$, then we have $a_{\\ell}=\\alpha$ for all $\\ell>k$. For each positive integer $m$, there exists $\\ell>k$ such that $m \\mid \\ell$ !. Plugging in $m n=\\ell$ !, it then follows that $$ f(m)=\\left\\lfloor\\frac{f(\\ell!)}{\\ell!\/ m}\\right\\rfloor=\\lfloor\\alpha m\\rfloor $$ for all $m$, so $f$ is of the desired form. If there does not exist a $k$ such that $a_{k}=\\alpha$, we must have $a_{k}<\\alpha$ for all $k$. For each positive integer $m$, we can now pick an $\\ell$ such that $m \\mid \\ell$ ! and $a_{\\ell}=\\alpha-x$ with $x$ arbitrarily small. It then follows from plugging in $m n=\\ell$ ! that $$ f(m)=\\left\\lfloor\\frac{f(\\ell!)}{\\ell!\/ m}\\right\\rfloor=\\left\\lfloor\\frac{\\ell!(\\alpha-x)}{\\ell!\/ m}\\right\\rfloor=\\lfloor\\alpha m-m x\\rfloor . $$ If $\\alpha m$ is an integer we can choose $\\ell$ such that $m x<1$, and it follows that $f(m)=\\lceil\\alpha m\\rceil-1$. If $\\alpha m$ is not an integer we can choose $\\ell$ such that $m x<\\{\\alpha m\\}$, and it also follows that $f(m)=\\lceil\\alpha m\\rceil-1$. We conclude that in this case $f$ is again of the desired form."} +{"year":2022,"label":"9","problem":"Let $k>1$ be a fixed positive integer. Prove that if $n$ is a sufficiently large positive integer, there exists a sequence of integers with the following properties: - Each element of the sequence is between 1 and $n$, inclusive. - For any two different contiguous subsequences of the sequence with length between 2 and $k$ inclusive, the multisets of values in those two subsequences is not the same. - The sequence has length at least $0.499 n^{2}$.","solution":" For any positive integer $n$, define an $(n, k)$-good sequence to be a finite sequence of integers each between 1 and $n$ inclusive satisfying the second property in the problem statement. The problems asks to show that, for all sufficiently large integers $n$, there is an $(n, k)$-good sequence of length at least $0.499 n^{2}$. Fix $k \\geq 2$ and consider some prime power $n=p^{m}$ with $p>k+1$. Consider some $0k+1$ be a prime. Then for $n=p^{2}$ we can find a $(n, k)$-good sequence of length $\\frac{p(p-1)\\left(p^{2}+2\\right)}{2}$. We will prove a stronger statement that $g\\frac{\\left(\\sum_{k=m}^{n} \\frac{1}{k^{2}}\\right)^{2}}{\\frac{1}{m}} \\end{aligned} $$ as desired. Remark (Bound on error). Let $A=\\sum_{k=m}^{n} k^{-2}$ and $B=\\sum_{k=m}^{n} k^{-3}$. The inequality above becomes tighter for large $m$ and $n \\gg m$. If we use Lagrange's identity in place of Cauchy-Schwarz, we get $$ A+B-m A^{2}=m \\cdot \\sum_{m \\leq aA+B $$ Remark (Construction commentary, from author). My motivation was to write an inequality where Titu could be applied creatively to yield a telescoping sum. This can be difficult because most of the time, such a reverse-engineered inequality will be so loose it's trivial anyways. My first attempt was the not-so-amazing inequality $$ \\frac{n^{2}+3 n}{2}=\\sum_{1}^{n} i+1=\\sum_{1}^{n} \\frac{\\frac{1}{i}}{\\frac{1}{i(i+1)}}>\\left(\\sum_{1}^{n} \\frac{1}{\\sqrt{i}}\\right)^{2} $$ which is really not surprising given that $\\sum \\frac{1}{\\sqrt{i}} \\ll \\frac{n}{\\sqrt{2}}$. The key here is that we need \"near-equality\" as dictated by the Cauchy-Schwarz equality case, i.e. the square root of the numerators should be approximately proportional to the denominators. This motivates using $\\frac{1}{i^{4}}$ as the numerator, which works like a charm. After working out the resulting statement, the LHS and RHS even share a sum, which adds to the simplicity of the problem. The final touch was to unrestrict the starting value of the sum, since this allows the strength of the estimate $\\frac{1}{i^{2}} \\approx \\frac{1}{i(i+1)}$ to be fully exploited."} +{"year":2023,"label":"2","problem":"Let $n \\geq m \\geq 1$ be integers. Prove that $$ \\sum_{k=m}^{n}\\left(\\frac{1}{k^{2}}+\\frac{1}{k^{3}}\\right) \\geq m \\cdot\\left(\\sum_{k=m}^{n} \\frac{1}{k^{2}}\\right)^{2} $$","solution":" \u3010 Second approach by inducting down, Luke Robitaille and Carl Schildkraut. Fix $n$; we'll induct downwards on $m$. For the base case of $n=m$ the result is easy, since the left side is $\\frac{m+1}{m^{3}}$ and the right side is $\\frac{m}{m^{4}}=\\frac{1}{m^{3}}$. For the inductive step, suppose we have shown the result for $m+1$. Let $$ A=\\sum_{k=m+1}^{n} \\frac{1}{k^{2}} \\quad \\text { and } \\quad B=\\sum_{k=m+1}^{n} \\frac{1}{k^{3}} $$ We know $A+B \\geq(m+1) A^{2}$, and we want to show $$ \\left(A+\\frac{1}{m^{2}}\\right)+\\left(B+\\frac{1}{m^{3}}\\right) \\geq m\\left(A+\\frac{1}{m^{2}}\\right)^{2} $$ Indeed, $$ \\begin{aligned} \\left(A+\\frac{1}{m^{2}}\\right)+\\left(B+\\frac{1}{m^{3}}\\right)-m\\left(A+\\frac{1}{m^{2}}\\right)^{2} & =A+B+\\frac{m+1}{m^{3}}-m A^{2}-\\frac{2 A}{m}-\\frac{1}{m^{3}} \\\\ & =\\left(A+B-(m+1) A^{2}\\right)+\\left(A-\\frac{1}{m}\\right)^{2} \\geq 0 \\end{aligned} $$ and we are done."} +{"year":2023,"label":"2","problem":"Let $n \\geq m \\geq 1$ be integers. Prove that $$ \\sum_{k=m}^{n}\\left(\\frac{1}{k^{2}}+\\frac{1}{k^{3}}\\right) \\geq m \\cdot\\left(\\sum_{k=m}^{n} \\frac{1}{k^{2}}\\right)^{2} $$","solution":" I Third approach by reducing $n \\rightarrow \\infty$, Michael Ren and Carl Schildkraut. First, we give: Claim (Reduction to $n \\rightarrow \\infty$ ) - If the problem is true when $n \\rightarrow \\infty$, it is true for all $n$. However, the region is bounded by a convex curve, and the sequence of points $(0,0)$, $\\left(\\frac{1}{m^{2}}, \\frac{1}{m^{3}}\\right),\\left(\\frac{1}{m^{2}}+\\frac{1}{(m+1)^{2}}, \\frac{1}{m^{3}}+\\frac{1}{(m+1)^{3}}\\right), \\ldots$ has successively decreasing slopes between consecutive points. Thus it suffices to check that the inequality is true when $n \\rightarrow \\infty$. Set $n=\\infty$ henceforth. Let $$ A=\\sum_{k=m}^{\\infty} \\frac{1}{k^{2}} \\text { and } B=\\sum_{k=m}^{\\infty} \\frac{1}{k^{3}} $$ we want to show $B \\geq m A^{2}-A$, which rearranges to $$ 1+4 m B \\geq(2 m A-1)^{2} $$ Write $$ C=\\sum_{k=m}^{\\infty} \\frac{1}{k^{2}(2 k-1)(2 k+1)} \\text { and } D=\\sum_{k=m}^{\\infty} \\frac{8 k^{2}-1}{k^{3}(2 k-1)^{2}(2 k+1)^{2}} $$ Then $$ \\frac{2}{2 k-1}-\\frac{2}{2 k+1}=\\frac{1}{k^{2}}+\\frac{1}{k^{2}(2 k-1)(2 k+1)} $$ and $$ \\frac{2}{(2 k-1)^{2}}-\\frac{2}{(2 k+1)^{2}}=\\frac{1}{k^{3}}+\\frac{8 k^{2}-1}{k^{3}(2 k-1)^{2}(2 k+1)^{2}}, $$ so that $$ A=\\frac{2}{2 m-1}-C \\text { and } B=\\frac{2}{(2 m-1)^{2}}-D $$ Our inequality we wish to show becomes $$ \\frac{2 m+1}{2 m-1} C \\geq D+m C^{2} $$ We in fact show two claims: Claim - We have $$ \\frac{2 m+1 \/ 2}{2 m-1} C \\geq D $$ $$ \\frac{2 m+1 \/ 2}{2 m-1} \\cdot \\frac{1}{k^{2}(2 k-1)(2 k+1)} \\geq \\frac{8 k^{2}-1}{k^{3}(2 k-1)^{2}(2 k+1)^{2}} $$ for $k \\geq m$. It suffices to show $$ \\frac{2 k+1 \/ 2}{2 k-1} \\cdot \\frac{1}{k^{2}(2 k-1)(2 k+1)} \\geq \\frac{8 k^{2}-1}{k^{3}(2 k-1)^{2}(2 k+1)^{2}} $$ which is equivalent to $k(2 k+1 \/ 2)(2 k+1) \\geq 8 k^{2}-1$. This holds for all $k \\geq 1$. ## Claim - We have $$ \\frac{1 \/ 2}{2 m-1} C \\geq m C^{2} $$ $$ \\frac{1}{2 m(2 m-1)}=\\sum_{k=m}^{\\infty}\\left(\\frac{1}{2 k(2 k-1)}-\\frac{1}{2(k+1)(2 k+1)}\\right)=\\sum_{k=m}^{\\infty} \\frac{4 k+1}{2 k(2 k-1)(k+1)(2 k+1)} $$ comparing term-wise with the definition of $C$ and using the inequality $k(4 k+1) \\geq 2(k+1)$ for $k \\geq 1$ gives the desired result."} +{"year":2023,"label":"2","problem":"Let $n \\geq m \\geq 1$ be integers. Prove that $$ \\sum_{k=m}^{n}\\left(\\frac{1}{k^{2}}+\\frac{1}{k^{3}}\\right) \\geq m \\cdot\\left(\\sum_{k=m}^{n} \\frac{1}{k^{2}}\\right)^{2} $$","solution":" \u300e Fourth approach by bashing, Carl Schildkraut. With a bit more work, the third approach can be adapted to avoid the $n \\rightarrow \\infty$ reduction. Similarly to before, define $$ A=\\sum_{k=m}^{n} \\frac{1}{k^{2}} \\text { and } B=\\sum_{k=m}^{n} \\frac{1}{k^{3}} $$ we want to show $1+4 m B \\geq(2 m A-1)^{2}$. Writing $$ C=\\sum_{k=m}^{n} \\frac{1}{k^{2}(2 k-1)(2 k+1)} \\text { and } D=\\sum_{k=m}^{n} \\frac{8 k^{2}-1}{k^{3}(2 k-1)^{2}(2 k+1)^{2}} $$ We compute $$ A=\\frac{2}{2 m-1}-\\frac{2}{2 n+1}-C \\text { and } B=\\frac{2}{(2 m-1)^{2}}-\\frac{2}{(2 n+1)^{2}}-D . $$ Then, the inequality we wish to show reduces (as in the previous solution) to $$ \\frac{2 m+1}{2 m-1} C+\\frac{2(2 m+1)}{(2 m-1)(2 n+1)} \\geq D+m C^{2}+\\frac{2(2 m+1)}{(2 n+1)^{2}}+\\frac{4 m}{2 n+1} C $$ We deal first with the terms not containing the variable $n$, i.e. we show that $$ \\frac{2 m+1}{2 m-1} C \\geq D+m C^{2} $$ For this part, the two claims from the previous solution go through exactly as written above, and we have $C \\leq 1 \/(2 m(2 m-1))$. We now need to show $$ \\frac{2(2 m+1)}{(2 m-1)(2 n+1)} \\geq \\frac{2(2 m+1)}{(2 n+1)^{2}}+\\frac{4 m}{2 n+1} C $$ (this is just the inequality between the remaining terms); our bound on $C$ reduces this to proving $$ \\frac{4(2 m+1)(n-m+1)}{(2 m-1)(2 n+1)^{2}} \\geq \\frac{2}{(2 m-1)(2 n+1)} $$ Expanding and writing in terms of $n$, this is equivalent to $$ n \\geq \\frac{1+2(m-1)(2 m+1)}{4 m}=m-\\frac{2 m+1}{4 m} $$ which holds for all $n \\geq m$."} +{"year":2023,"label":"3","problem":"Find all positive integers $n$ for which it is possible to color some cells of an infinite grid of unit squares red, such that each rectangle consisting of exactly $n$ cells (and whose edges lie along the lines of the grid) contains an odd number of red cells.","solution":" We claim that this is possible for all positive integers $n$. Call a positive integer for which such a coloring is possible good. To show that all positive integers $n$ are good we prove the following: (i) If $n$ is good and $p$ is an odd prime, then $p n$ is good; (ii) For every $k \\geq 0$, the number $n=2^{k}$ is good. Together, (i) and (ii) imply that all positive integers are good. Thus every coloring that works for $n$ automatically also works for $p n$. Claim - For each of these $k+1$ shapes, there exists a coloring with two properties: - Every rectangle with $n$ cells and shape $2^{m} \\times 2^{k-m}$ contains an odd number of red cells. - Every rectangle with $n$ cells and a different shape contains an even number of red cells. ![](https:\/\/cdn.mathpix.com\/cropped\/2024_11_19_1cf0843c143ab4f13b3fg-13.jpg?height=395&width=401&top_left_y=2084&top_left_x=833) A $2^{m} \\times 2^{k-m}$ rectangle contains every possible pair $\\left(x \\bmod 2^{m}, y \\bmod 2^{k-m}\\right)$ exactly once, so such a rectangle will contain one red cell (an odd number). On the other hand, consider a $2^{\\ell} \\times 2^{k-\\ell}$ rectangle with $\\ell>m$. The set of cells this covers is $(x, y)$ where $x$ covers a range of size $2^{\\ell}$ and $y$ covers a range of size $2^{k-\\ell}$. The number of red cells is the count of $x$ with $x \\equiv 0 \\bmod 2^{m}$ multiplied by the count of $y$ with $y \\equiv 0 \\bmod 2^{k-m}$. The former number is exactly $2^{\\ell-k}$ because $2^{k}$ divides $2^{\\ell}$ (while the latter is 0 or 1) so the number of red cells is even. The $\\ell2 \\\\ 2\\left(g(x)+g\\left(x+p^{d}\\right)+\\cdots+g\\left(x+(b-1) p^{d}\\right)\\right) & p=2\\end{cases} \\\\ \\equiv & 0 \\quad(\\bmod p) \\end{aligned} $$ as desired. ## Corollary Let $g: \\mathbb{Z} \/ a \\mathbb{Z} \\rightarrow \\mathbb{Z} \/ p^{e} \\mathbb{Z}$ be any function, and let $h=\\Delta^{e p^{d}} g$. Then $$ h(x)+h\\left(x+p^{d}\\right)+\\cdots+h\\left(x+(b-1) p^{d}\\right)=0 $$ for all $x$. $$ h_{1}(x)=\\frac{h(x)+h\\left(x+p^{d}\\right)+\\cdots+h\\left(x+(b-1) p^{d}\\right)}{p} . $$ Applying the lemma to $h_{1}$ shows the corollary for $e=2$, since $h_{1}(x)$ is divisible by $p$, hence the numerator is divisible by $p^{2}$. Continue in this manner to get the result for general $e>2$. This immediately settles this direction, since $f$ is in the image of $\\Delta^{e p^{d}}$. We will show that $\\Delta$ is injective on $\\mathcal{S}$. Suppose otherwise, and consider two functions $f$, $g$ in $\\mathcal{S}$ with $\\Delta f=\\Delta g$. Then, we obtain that $f$ and $g$ differ by a constant; say $g=f+\\lambda$. However, then $$ \\begin{aligned} & g(0)+g\\left(p^{e}\\right)+\\cdots+g\\left((b-1) p^{e}\\right) \\\\ = & (f(0)+\\lambda)+\\left(f\\left(p^{e}\\right)+\\lambda\\right)+\\cdots+\\left(f\\left((b-1) p^{e}\\right)+\\lambda\\right) \\\\ = & b \\lambda \\end{aligned} $$ This should also be zero. Since $p \\nmid b$, we obtain $\\lambda=0$, as desired. Counting Finally, we can count the essential functions: all but the last $p^{d}$ entries can be chosen arbitrarily, and then each remaining entry has exactly one possible choice. This leads to a count of $$ \\left(p^{e}\\right)^{a-p^{d}}=p^{e\\left(a-p^{\\nu_{p}(a)}\\right)}, $$ as promised."} +{"year":2023,"label":"9","problem":"Let $p$ be a fixed prime and let $a \\geq 2$ and $e \\geq 1$ be fixed integers. Given a function $f: \\mathbb{Z} \/ a \\mathbb{Z} \\rightarrow \\mathbb{Z} \/ p^{e} \\mathbb{Z}$ and an integer $k \\geq 0$, the $k$ th finite difference, denoted $\\Delta^{k} f$, is the function from $\\mathbb{Z} \/ a \\mathbb{Z}$ to $\\mathbb{Z} \/ p^{e} \\mathbb{Z}$ defined recursively by $$ \\begin{aligned} & \\Delta^{0} f(n)=f(n) \\\\ & \\Delta^{k} f(n)=\\Delta^{k-1} f(n+1)-\\Delta^{k-1} f(n) \\quad \\text { for } k=1,2, \\ldots \\end{aligned} $$ Determine the number of functions $f$ such that there exists some $k \\geq 1$ for which $\\Delta^{k} f=f$ \u3002","solution":"Let $p$ be a fixed prime and let $a \\geq 2$ and $e \\geq 1$ be fixed integers. Given a function $f: \\mathbb{Z} \/ a \\mathbb{Z} \\rightarrow \\mathbb{Z} \/ p^{e} \\mathbb{Z}$ and an integer $k \\geq 0$, the $k$ th finite difference, denoted $\\Delta^{k} f$, is the function from $\\mathbb{Z} \/ a \\mathbb{Z}$ to $\\mathbb{Z} \/ p^{e} \\mathbb{Z}$ defined recursively by $$ \\begin{aligned} & \\Delta^{0} f(n)=f(n) \\\\ & \\Delta^{k} f(n)=\\Delta^{k-1} f(n+1)-\\Delta^{k-1} f(n) \\quad \\text { for } k=1,2, \\ldots \\end{aligned} $$ Determine the number of functions $f$ such that there exists some $k \\geq 1$ for which $\\Delta^{k} f=f$. The answer is $$ \\left(p^{e}\\right)^{a} \\cdot p^{-e p^{\\nu} p(a)}=p^{e\\left(a-p^{\\nu_{p}(a)}\\right)} $$ II Second solution by Daniel Zhu. There are two parts to the proof: solving the $e=1$ case, and using the $e=1$ result to solve the general problem by induction on $e$. These parts are independent of each other. The case $e=1 \\quad$ Represent functions $f$ as elements $$ \\alpha_{f}:=\\sum_{k \\in \\mathbb{Z} \/ a \\mathbb{Z}} f(-k) x^{k} \\in \\mathbb{F}_{p}[x] \/\\left(x^{a}-1\\right) $$ Then, since $\\alpha_{\\Delta f}=(x-1) \\alpha_{f}$, we wish to find the number of $\\alpha \\in \\mathbb{F}_{p}[x] \/\\left(x^{a}-1\\right)$ such that $(x-1)^{m} \\alpha=\\alpha$ for some $m$. Now, make the substitution $y=x-1$ and let $P(y)=(y+1)^{a}-1$; we want to find $\\alpha \\in \\mathbb{F}_{p}[y] \/(P(y))$ such that $y^{m} \\alpha=\\alpha$ for some $m$. If we write $P(y)=y^{d} Q(y)$ with $Q(0) \\neq 0$, then by the Chinese Remainder Theorem we have the ring isomorphism $$ \\mathbb{F}_{p}[y] \/(P(y)) \\cong \\mathbb{F}_{p}[y] \/\\left(y^{d}\\right) \\times \\mathbb{F}_{p}[y] \/(Q(y)) $$ Note that $y$ is nilpotent in the first factor, while it is a unit in the second factor. So the $\\alpha$ that work are exactly those that are zero in the first factor; thus there are $p^{a-d}$ such $\\alpha$. We can calculate $d=p^{v_{p}(a)}$ (via, say, Lucas's Theorem), so we are done. The general problem The general idea is as follows: call a $f: \\mathbb{Z} \/ a \\mathbb{Z} \\rightarrow \\mathbb{Z} \/ p^{e} \\mathbb{Z} e$-good if $\\Delta^{m} f=f$ for some $m$. Our result above allows us to count the 1-good functions. Then, if $e \\geq 1$, every $(e+1)$-good function, when reduced $\\bmod p^{e}$, yields an $e$-good function, so we count $(e+1)$-good functions by counting how many reduce to any given $e$-good function. Formally, we use induction on $e$, with the $e=1$ case being treated above. Suppose now we have solved the problem for a given $e \\geq 1$, and we now wish to solve it for $e+1$. For any function $g: \\mathbb{Z} \/ a \\mathbb{Z} \\rightarrow \\mathbb{Z} \/ p^{e+1} \\mathbb{Z}$, let $\\bar{g}: \\mathbb{Z} \/ a \\mathbb{Z} \\rightarrow \\mathbb{Z} \/ p^{e} \\mathbb{Z}$ be its reduction $\\bmod p^{e}$. For a given $e$-good $f$, let $n(f)$ be the number of $(e+1)$-good $g$ with $\\bar{g}=f$. The following two claims now finish the problem: Claim - If $f$ is $e$-good, then $n(f)>0$. $$ g, \\Delta^{m} g, \\Delta^{2 m} g, \\ldots $$ Since there are finitely many functions $\\mathbb{Z} \/ a \\mathbb{Z} \\rightarrow \\mathbb{Z} \/ p^{e+1} \\mathbb{Z}$, there must exist $a0$, then $n(f)$ is exactly the number of 1-good functions, i.e. $p^{a-p^{v_{p}(a)}}$. To show that this condition is sufficient, note that $\\overline{g+p^{e} h}=\\bar{g}=f$. Moreover, if $\\Delta^{m} g=g$ and $\\Delta^{m^{\\prime}} h=h$, then $$ \\Delta^{m m^{\\prime}}\\left(g+p^{e} h\\right)=\\Delta^{m m^{\\prime}} g+p^{e} \\Delta^{m m^{\\prime}} h=g+p^{e} h . $$ To show that this condition is necessary, let $g_{1}$ be any $(e+1)$-good function such that $\\bar{g}_{1}=f$. Then $g_{1}-g$ is also $(e+1)$-good, since if $\\Delta^{m} g=g, \\Delta^{m^{\\prime}} g_{1}=g_{1}$, we have $$ \\Delta^{m m^{\\prime}}\\left(g_{1}-g\\right)=\\Delta^{m m^{\\prime}} g_{1}-\\Delta^{m m^{\\prime}} g=g_{1}-g . $$ On the other hand, we also know that $g_{1}-g$ is divisible by $p^{e}$. This means that it must be $p^{e} h$ for some function $f: \\mathbb{Z} \/ a \\mathbb{Z} \\rightarrow \\mathbb{Z} \/ p \\mathbb{Z}$, and it is not hard to show that $g_{1}-g$ being $(e+1)$-good means that $h$ is 1 -good."} diff --git a/USA_TST_TST/md/en-sols-TSTST-2011.md b/USA_TST_TST/md/en-sols-TSTST-2011.md new file mode 100644 index 0000000000000000000000000000000000000000..caab7201c5e37b150290c50db820d1cef04ccffd --- /dev/null +++ b/USA_TST_TST/md/en-sols-TSTST-2011.md @@ -0,0 +1,321 @@ +# TSTST 2011 Solution Notes
Lincoln, Nebraska
\author{ Evan Chen《陳誼廷》 + +} + +15 April 2024 + +This is a compilation of solutions for the 2011 TSTST. The ideas of the solution are a mix of my own work, the solutions provided by the competition organizers, and solutions found by the community. However, all the writing is maintained by me. + +These notes will tend to be a bit more advanced and terse than the "official" solutions from the organizers. In particular, if a theorem or technique is not known to beginners but is still considered "standard", then I often prefer to use this theory anyways, rather than try to work around or conceal it. For example, in geometry problems I typically use directed angles without further comment, rather than awkwardly work around configuration issues. Similarly, sentences like "let $\mathbb{R}$ denote the set of real numbers" are typically omitted entirely. + +Corrections and comments are welcome! + +## Contents + +0 Problems ..... 2 +1 Solutions to Day 1 ..... 3 +1.1 TSTST 2011/1 ..... 3 +1.2 TSTST 2011/2 ..... 4 +1.3 TSTST 2011/3 ..... 6 +2 Solutions to Day 2 ..... 7 +2.1 TSTST 2011/4 ..... 7 +2.2 TSTST 2011/5 ..... 8 +2.3 TSTST 2011/6 ..... 9 +3 Solutions to Day 3 ..... 10 +3.1 TSTST 2011/7 ..... 10 +3.2 TSTST 2011/8 ..... 11 +3.3 TSTST 2011/9 ..... 12 + +## §0 Problems + +1. Find all real-valued functions $f$ defined on pairs of real numbers, having the following property: for all real numbers $a, b, c$, the median of $f(a, b), f(b, c), f(c, a)$ equals the median of $a, b, c$. +(The median of three real numbers, not necessarily distinct, is the number that is in the middle when the three numbers are arranged in nondecreasing order.) +2. Two circles $\omega_{1}$ and $\omega_{2}$ intersect at points $A$ and $B$. Line $\ell$ is tangent to $\omega_{1}$ at $P$ and to $\omega_{2}$ at $Q$ so that $A$ is closer to $\ell$ than $B$. Let $X$ and $Y$ be points on major $\operatorname{arcs} \widehat{P A}\left(\right.$ on $\left.\omega_{1}\right)$ and $\overparen{A Q}$ (on $\left.\omega_{2}\right)$, respectively, such that $A X / P X=A Y / Q Y=c$. Extend segments $P A$ and $Q A$ through $A$ to $R$ and $S$, respectively, such that $A R=A S=c \cdot P Q$. Given that the circumcenter of triangle $A R S$ lies on line $X Y$, prove that $\angle X P A=\angle A Q Y$. +3. Prove that there exists a real constant $c$ such that for any pair $(x, y)$ of real numbers, there exist relatively prime integers $m$ and $n$ satisfying the relation + +$$ +\sqrt{(x-m)^{2}+(y-n)^{2}}d_{2}>\cdots>d_{k}$ and $\operatorname{gcd}\left(d_{1}, d_{2}, \ldots, d_{k}\right)=1$. For every integer $n \geq n_{0}$, define + +$$ +x_{n}=\left\lfloor\frac{x_{n-d_{1}}+x_{n-d_{2}}+\cdots+x_{n-d_{k}}}{k}\right\rfloor . +$$ + +Show that the sequence $\left(x_{n}\right)$ is eventually constant. +9. Let $n$ be a positive integer. Suppose we are given $2^{n}+1$ distinct sets, each containing finitely many objects. Place each set into one of two categories, the red sets and the blue sets, so that there is at least one set in each category. We define the symmetric difference of two sets as the set of objects belonging to exactly one of the two sets. Prove that there are at least $2^{n}$ different sets which can be obtained as the symmetric difference of a red set and a blue set. + +## §1 Solutions to Day 1 + +## §1.1 TSTST 2011/1 + +Available online at https://aops.com/community/p2374841. + +## Problem statement + +Find all real-valued functions $f$ defined on pairs of real numbers, having the following property: for all real numbers $a, b, c$, the median of $f(a, b), f(b, c), f(c, a)$ equals the median of $a, b, c$. +(The median of three real numbers, not necessarily distinct, is the number that is in the middle when the three numbers are arranged in nondecreasing order.) + +The following solution is joint with Andrew He. +We prove the following main claim, from which repeated applications can deduce the problem. + +Claim - Let $ad_{2}>\cdots>d_{k}$ and $\operatorname{gcd}\left(d_{1}, d_{2}, \ldots, d_{k}\right)=1$. For every integer $n \geq n_{0}$, define + +$$ +x_{n}=\left\lfloor\frac{x_{n-d_{1}}+x_{n-d_{2}}+\cdots+x_{n-d_{k}}}{k}\right\rfloor +$$ + +Show that the sequence $\left(x_{n}\right)$ is eventually constant. + +Note that if the initial terms are contained in some interval $[A, B]$ then they will remain in that interval. Thus the sequence is eventually periodic. Discard initial terms and let the period be $T$; we will consider all indices modulo $T$ from now on. + +Let $M$ be the maximal term in the sequence (which makes sense since the sequence is periodic). Note that if $x_{n}=M$, we must have $x_{n-d_{i}}=M$ for all $i$ as well. By taking a linear combination $\sum c_{i} d_{i} \equiv 1(\bmod T)($ possibly be Bezout's theorem, since $\operatorname{gcd}_{i}\left(d_{i}\right)=1$ ), we conclude $x_{n-1}=M$, as desired. + +## §3.3 TSTST 2011/9 + +Available online at https://aops.com/community/p2374857. + +## Problem statement + +Let $n$ be a positive integer. Suppose we are given $2^{n}+1$ distinct sets, each containing finitely many objects. Place each set into one of two categories, the red sets and the blue sets, so that there is at least one set in each category. We define the symmetric difference of two sets as the set of objects belonging to exactly one of the two sets. Prove that there are at least $2^{n}$ different sets which can be obtained as the symmetric difference of a red set and a blue set. + +We can interpret the problem as working with binary strings of length $\ell \geq n+1$, with $\ell$ the number of elements across all sets. + +Let $F$ be a field of cardinality $2^{\ell}$, hence $F \cong \mathbb{F}_{2}^{\oplus \ell}$. +Then, we can think of red/blue as elements of $F$, so we have some $B \subseteq F$, and an $R \subseteq F$. We wish to prove that $|B+R| \geq 2^{n}$. Want $|B+R| \geq 2^{n}$. + +Equivalently, any element of a set $X$ with $|X|=2^{n}-1$ should omit some element of $|B+R|$. To prove this: we know $|B|+|R|=2^{n}+1$, and define + +$$ +P(b, r)=\prod_{x \in X}(b+r-x) +$$ + +Consider $b^{|B|-1} r^{|R|-1}$. The coefficient of is $\binom{2^{n}-1}{|B|-1}$, which is odd (say by Lucas theorem), so the nullstellensatz applies. + diff --git a/USA_TST_TST/md/en-sols-TSTST-2012.md b/USA_TST_TST/md/en-sols-TSTST-2012.md new file mode 100644 index 0000000000000000000000000000000000000000..1b37f2aca94da2084816d9e2cad22f79c15ae0b2 --- /dev/null +++ b/USA_TST_TST/md/en-sols-TSTST-2012.md @@ -0,0 +1,454 @@ +# TSTST 2012 Solution Notes
Lincoln, Nebraska
\author{ Evan Chen《陳誼廷》 + +} + +15 April 2024 + +This is a compilation of solutions for the 2012 TSTST. The ideas of the solution are a mix of my own work, the solutions provided by the competition organizers, and solutions found by the community. However, all the writing is maintained by me. + +These notes will tend to be a bit more advanced and terse than the "official" solutions from the organizers. In particular, if a theorem or technique is not known to beginners but is still considered "standard", then I often prefer to use this theory anyways, rather than try to work around or conceal it. For example, in geometry problems I typically use directed angles without further comment, rather than awkwardly work around configuration issues. Similarly, sentences like "let $\mathbb{R}$ denote the set of real numbers" are typically omitted entirely. + +Corrections and comments are welcome! + +## Contents + +0 Problems ..... 2 +1 Solutions to Day 1 ..... 4 +1.1 TSTST 2012/1, proposed by Palmer Mebane ..... 4 +1.2 TSTST 2012/2 ..... 5 +1.3 TSTST 2012/3 ..... 6 +2 Solutions to Day 2 ..... 8 +2.1 TSTST 2012/4 ..... 8 +2.2 TSTST 2012/5 ..... 9 +2.3 TSTST 2012/6, proposed by Sung-Yoon Kim ..... 10 +3 Solutions to Day 3 ..... 11 +3.1 TSTST 2012/7 ..... 11 +3.2 TSTST 2012/8, proposed by Palmer Mebane ..... 13 +3.3 TSTST 2012/9, proposed by John Berman ..... 15 + +## §0 Problems + +1. Determine all infinite strings of letters with the following properties: +(a) Each letter is either $T$ or $S$, +(b) If position $i$ and $j$ both have the letter $T$, then position $i+j$ has the letter $S$, +(c) There are infinitely many integers $k$ such that position $2 k-1$ has the $k$ th $T$. +2. Let $A B C D$ be a quadrilateral with $A C=B D$. Diagonals $A C$ and $B D$ meet at $P$. Let $\omega_{1}$ and $O_{1}$ denote the circumcircle and circumcenter of triangle $A B P$. Let $\omega_{2}$ and $O_{2}$ denote the circumcircle and circumcenter of triangle $C D P$. Segment $B C$ meets $\omega_{1}$ and $\omega_{2}$ again at $S$ and $T$ (other than $B$ and $C$ ), respectively. Let $M$ and $N$ be the midpoints of minor arcs $\widehat{S P}$ (not including $B$ ) and $\overparen{T P}$ (not including $C$ ). Prove that $\overline{M N} \| \overline{O_{1} O_{2}}$. +3. Let $\mathbb{N}$ be the set of positive integers. Let $f: \mathbb{N} \rightarrow \mathbb{N}$ be a function satisfying the following two conditions: +(a) $f(m)$ and $f(n)$ are relatively prime whenever $m$ and $n$ are relatively prime. +(b) $n \leq f(n) \leq n+2012$ for all $n$. + +Prove that for any natural number $n$ and any prime $p$, if $p$ divides $f(n)$ then $p$ divides $n$. +4. In scalene triangle $A B C$, let the feet of the perpendiculars from $A$ to $\overline{B C}, B$ to $\overline{C A}$, $C$ to $\overline{A B}$ be $A_{1}, B_{1}, C_{1}$, respectively. Denote by $A_{2}$ the intersection of lines $B C$ and $B_{1} C_{1}$. Define $B_{2}$ and $C_{2}$ analogously. Let $D, E, F$ be the respective midpoints of sides $\overline{B C}, \overline{C A}, \overline{A B}$. Show that the perpendiculars from $D$ to $\overline{A A_{2}}, E$ to $\overline{B B_{2}}$ and $F$ to $\overline{C C_{2}}$ are concurrent. +5. A rational number $x$ is given. Prove that there exists a sequence $x_{0}, x_{1}, x_{2}, \ldots$ of rational numbers with the following properties: +(a) $x_{0}=x$; +(b) for every $n \geq 1$, either $x_{n}=2 x_{n-1}$ or $x_{n}=2 x_{n-1}+\frac{1}{n}$; +(c) $x_{n}$ is an integer for some $n$. +6. Positive real numbers $x, y, z$ satisfy $x y z+x y+y z+z x=x+y+z+1$. Prove that + +$$ +\frac{1}{3}\left(\sqrt{\frac{1+x^{2}}{1+x}}+\sqrt{\frac{1+y^{2}}{1+y}}+\sqrt{\frac{1+z^{2}}{1+z}}\right) \leq\left(\frac{x+y+z}{3}\right)^{5 / 8} . +$$ + +7. Triangle $A B C$ is inscribed in circle $\Omega$. The interior angle bisector of angle $A$ intersects side $B C$ and $\Omega$ at $D$ and $L$ (other than $A$ ), respectively. Let $M$ be the midpoint of side $B C$. The circumcircle of triangle $A D M$ intersects sides $A B$ and $A C$ again at $Q$ and $P$ (other than $A$ ), respectively. Let $N$ be the midpoint of segment $P Q$, and let $H$ be the foot of the perpendicular from $L$ to line $N D$. Prove that line $M L$ is tangent to the circumcircle of triangle $H M N$. +8. Let $n$ be a positive integer. Consider a triangular array of nonnegative integers as follows: +![](https://cdn.mathpix.com/cropped/2024_11_19_b4303c9fa48a8c74b45cg-03.jpg?height=244&width=809&top_left_y=238&top_left_x=675) + +Call such a triangular array stable if for every $0 \leq i1$, let $A_{k}=\left\{a_{1}, a_{2}, \ldots, a_{k}\right\}$. By (b) and symmetry, we have + +$$ +2 k-1 \geq \frac{\left|A_{k}-A_{k}\right|-1}{2}+\left|A_{k}\right| \geq \frac{2\left|A_{k}\right|-2}{2}+\left|A_{k}\right|=2 k-1 . +$$ + +But in order for $\left|A_{k}-A_{k}\right|=2\left|A_{k}\right|-1$, we must have $A_{k}$ an arithmetic progression, whence $a_{n}=2 n-1$ for all $n$ by taking $k$ arbitrarily large. + +## §1.2 TSTST 2012/2 + +Available online at https://aops.com/community/p2745851. + +## Problem statement + +Let $A B C D$ be a quadrilateral with $A C=B D$. Diagonals $A C$ and $B D$ meet at $P$. Let $\omega_{1}$ and $O_{1}$ denote the circumcircle and circumcenter of triangle $A B P$. Let $\omega_{2}$ and $O_{2}$ denote the circumcircle and circumcenter of triangle $C D P$. Segment $B C$ meets $\omega_{1}$ and $\omega_{2}$ again at $S$ and $T$ (other than $B$ and $C$ ), respectively. Let $M$ and $N$ be the midpoints of minor arcs $\widehat{S P}$ (not including $B$ ) and $\widehat{T P}$ (not including $C$ ). Prove that $\overline{M N} \| \overline{O_{1} O_{2}}$. + +Let $Q$ be the second intersection point of $\omega_{1}, \omega_{2}$. Suffice to show $\overline{Q P} \perp \overline{M N}$. Now $Q$ is the center of a spiral congruence which sends $\overline{A C} \mapsto \overline{B D}$. So $\triangle Q A B$ and $\triangle Q C D$ are similar isosceles. Now, + +$$ +\measuredangle Q P A=\measuredangle Q B A=\measuredangle D C Q=\measuredangle D P Q +$$ + +and so $\overline{Q P}$ is bisects $\angle B P C$. +![](https://cdn.mathpix.com/cropped/2024_11_19_b4303c9fa48a8c74b45cg-05.jpg?height=673&width=1200&top_left_y=1234&top_left_x=428) + +Now, let $I=\overline{B M} \cap \overline{C N} \cap \overline{P Q}$ be the incenter of $\triangle P B C$. Then $I M \cdot I B=I P \cdot I Q=$ $I N \cdot I C$, so $B M N C$ is cyclic, meaning $\overline{M N}$ is antiparallel to $\overline{B C}$ through $\angle B I C$. Since $\overline{Q P I}$ passes through the circumcenter of $\triangle B I C$, it follows now $\overline{Q P I} \perp \overline{M N}$ as desired. + +## §1.3 TSTST 2012/3 + +Available online at https://aops.com/community/p2745877. + +## Problem statement + +Let $\mathbb{N}$ be the set of positive integers. Let $f: \mathbb{N} \rightarrow \mathbb{N}$ be a function satisfying the following two conditions: +(a) $f(m)$ and $f(n)$ are relatively prime whenever $m$ and $n$ are relatively prime. +(b) $n \leq f(n) \leq n+2012$ for all $n$. + +Prove that for any natural number $n$ and any prime $p$, if $p$ divides $f(n)$ then $p$ divides $n$. + +【 First short solution, by Jeffrey Kwan. Let $p_{0}, p_{1}, p_{2}, \ldots$ denote the sequence of all prime numbers, in any order. Pick any primes $q_{i}$ such that + +$$ +q_{0}\left|f\left(p_{0}\right), \quad q_{1}\right| f\left(p_{1}\right), \quad q_{2} \mid f\left(p_{2}\right), \text { etc. } +$$ + +This is possible since each $f$ value above exceeds 1 . Also, since by hypothesis the $f\left(p_{i}\right)$ are pairwise coprime, the primes $q_{i}$ are all pairwise distinct. + +Claim - We must have $q_{i}=p_{i}$ for each $i$. (Therefore, $f\left(p_{i}\right)$ is a power of $p_{i}$ for each $i$.) + +Proof. Assume to the contrary that $q_{0} \neq p_{0}$. By changing labels if necessary, assume $\min \left(p_{1}, p_{2}, \ldots, p_{2012}\right)>2012$. Then by Chinese remainder theorem we can choose an integer $m$ such that + +$$ +\begin{array}{rr} +m+i & \equiv 0 \\ +m & \left(\bmod q_{i}\right) \\ +m & \equiv{ }^{\prime} \\ +\left(\bmod p_{i}\right) +\end{array} +$$ + +for $0 \leq i \leq 2012$. But now $f(m)$ should be coprime to all $f\left(p_{i}\right)$, ergo coprime to $q_{0} q_{1} \ldots q_{2012}$, violating $m \leq f(m) \leq m+2012$. + +All that is left to do is note that whenever $p \nmid n$, we have $\operatorname{gcd}(f(p), f(n))=1$, hence $p \nmid f(n)$. This is the contrapositive of the problem statement. + +【 Second solution with a grid. Fix $n$ and $p$, and assume for contradiction $p \nmid n$. +Claim - There exists a large integer $N$ with $f(N)=N$, that also satisfies $N \equiv 1$ $(\bmod n)$ and $N \equiv 0(\bmod p)$. + +Proof. We'll need to pick both $N$ and an ancillary integer $M$. Here is how: pick $2012 \cdot 2013$ distinct primes $q_{i, j}>n+p+2013$ for every $i=1, \ldots, 2012$ and $j=0, \ldots, 2012$, and use +it to fill in the following table: + +| | $N+1$ | $N+2$ | $\ldots$ | $N+2012$ | +| :---: | :---: | :---: | :---: | :---: | +| $M$ | $q_{0,1}$ | $q_{0,2}$ | $\ldots$ | $q_{0,2012}$ | +| $M+1$ | $q_{1,1}$ | $q_{1,2}$ | $\ldots$ | $q_{1,2012}$ | +| $\vdots$ | $\vdots$ | $\vdots$ | $\ddots$ | $\vdots$ | +| $M+2012$ | $q_{2012,1}$ | $q_{2012,2}$ | $\ldots$ | $q_{2012,2012}$ |. + +By the Chinese Remainder Theorem, we can construct $N$ such that $N+1 \equiv 0\left(\bmod q_{i, 1}\right)$ for every $i$, and similarly for $N+2$, and so on. Moreover, we can also tack on the extra conditions $N \equiv 0(\bmod p)$ and $N \equiv 1(\bmod n)$ we wanted. + +Notice that $N$ cannot be divisible by any of the $q_{i, j}$ 's, since the $q_{i, j}$ 's are greater than 2012. + +After we've chosen $N$, we can pick $M$ such that $M \equiv 0\left(\bmod q_{0, j}\right)$ for every $j$, and similarly $M+1 \equiv 0\left(\bmod q_{1, j}\right)$, et cetera. Moreover, we can tack on the condition $M \equiv 1$ $(\bmod N)$, which ensures $\operatorname{gcd}(M, N)=1$. + +What does this do? We claim that $f(N)=N$ now. Indeed $f(M)$ and $f(N)$ are relatively prime; but look at the table! The table tells us that $f(M)$ must have a common factor with each of $N+1, \ldots, N+2012$. So the only possibility is that $f(N)=N$. + +Now we're basically done. Since $N \equiv 1(\bmod n)$, we have $\operatorname{gcd}(N, n)=1$ and hence $1=\operatorname{gcd}(f(N), f(n))=\operatorname{gcd}(N, f(n))$. But $p \mid N$ and $p \mid f(n)$, contradiction. + +## §2 Solutions to Day 2 + +## §2.1 TSTST 2012/4 + +Available online at https://aops.com/community/p2745854. + +## Problem statement + +In scalene triangle $A B C$, let the feet of the perpendiculars from $A$ to $\overline{B C}, B$ to $\overline{C A}$, $C$ to $\overline{A B}$ be $A_{1}, B_{1}, C_{1}$, respectively. Denote by $A_{2}$ the intersection of lines $B C$ and $B_{1} C_{1}$. Define $B_{2}$ and $C_{2}$ analogously. Let $D, E, F$ be the respective midpoints of sides $\overline{B C}, \overline{C A}, \overline{A B}$. Show that the perpendiculars from $D$ to $\overline{A A_{2}}, E$ to $\overline{B B_{2}}$ and $F$ to $\overline{C C_{2}}$ are concurrent. + +We claim that they pass through the orthocenter $H$. Indeed, consider the circle with diameter $\overline{B C}$, which circumscribes quadrilateral $B C B_{1} C_{1}$ and has center $D$. Then by Brokard theorem, $\overline{A A_{2}}$ is the polar of line $H$. Thus $\overline{D H} \perp \overline{A A_{2}}$. + +## §2.2 TSTST 2012/5 + +Available online at https://aops.com/community/p2745867. + +## Problem statement + +A rational number $x$ is given. Prove that there exists a sequence $x_{0}, x_{1}, x_{2}, \ldots$ of rational numbers with the following properties: +(a) $x_{0}=x$; +(b) for every $n \geq 1$, either $x_{n}=2 x_{n-1}$ or $x_{n}=2 x_{n-1}+\frac{1}{n}$; +(c) $x_{n}$ is an integer for some $n$. + +Think of the sequence as a process over time. We'll show that: +Claim - At any given time $t$, if the denominator of $x_{t}$ has some odd prime power $q=p^{e}$, then we can delete a factor of $p$ from the denominator, while only adding powers of two to the denominator. +(Thus we can just delete off all the odd primes one by one and then double appropriately many times.) + +Proof. The idea is to add only fractions of the form $\left(2^{k} q\right)^{-1}$. +Indeed, let $n$ be large, and suppose $t<2^{r+1} q<2^{r+2} q<\cdots<2^{r+m} qb$ if $a b=b a=a$. The following are proved by finite casework, using the fact that $\{a b, b c, c a\}$ always has exactly two distinct elements for any different $a, b, c$. + +- If $a>b$ and $b>c$ then $a>c$. +- If $a \sim b$ and $b \sim c$ then $a b=a$ if and only if $b c=b$. +- If $a \sim b$ and $b \sim c$ then $a \sim c$. +- If $a \sim b$ and $a>c$ then $b>c$. +- If $a \sim b$ and $c>a$ then $c>b$. + +This gives us the total ordering on the elements and the equivalence classes by $\sim$. In this we way can check the claimed operations are the only ones. + +We can then (as in the first solution) verify that every full string is equivalent to a unique double rainbow - but this time we prove it by simply considering all possible $\times$, because we have classified them all. + diff --git a/USA_TST_TST/md/en-sols-TSTST-2013.md b/USA_TST_TST/md/en-sols-TSTST-2013.md new file mode 100644 index 0000000000000000000000000000000000000000..cbdc3c1a36246c57597081294e61fe93ca951e60 --- /dev/null +++ b/USA_TST_TST/md/en-sols-TSTST-2013.md @@ -0,0 +1,437 @@ +# TSTST 2013 Solution Notes
Lincoln, Nebraska
\author{ Evan Chen《陳誼廷》 + +} + +23 April 2024 + +This is a compilation of solutions for the 2013 TSTST. The ideas of the solution are a mix of my own work, the solutions provided by the competition organizers, and solutions found by the community. However, all the writing is maintained by me. + +These notes will tend to be a bit more advanced and terse than the "official" solutions from the organizers. In particular, if a theorem or technique is not known to beginners but is still considered "standard", then I often prefer to use this theory anyways, rather than try to work around or conceal it. For example, in geometry problems I typically use directed angles without further comment, rather than awkwardly work around configuration issues. Similarly, sentences like "let $\mathbb{R}$ denote the set of real numbers" are typically omitted entirely. + +Corrections and comments are welcome! + +## Contents + +0 Problems ..... 2 +1 Solutions to Day 1 ..... 4 +1.1 TSTST 2013/1 ..... 4 +1.2 TSTST 2013/2 ..... 5 +1.3 TSTST 2013/3 ..... 6 +2 Solutions to Day 2 ..... 8 +2.1 TSTST 2013/4 ..... 8 +2.2 TSTST 2013/5 ..... 10 +2.3 TSTST 2013/6 ..... 11 +3 Solutions to Day 3 ..... 13 +3.1 TSTST 2013/7 ..... 13 +3.2 TSTST 2013/8 ..... 14 +3.3 TSTST 2013/9 ..... 15 + +## §0 Problems + +1. Let $A B C$ be a triangle and $D, E, F$ be the midpoints of $\operatorname{arcs} B C, C A, A B$ on the circumcircle. Line $\ell_{a}$ passes through the feet of the perpendiculars from $A$ to $\overline{D B}$ and $\overline{D C}$. Line $m_{a}$ passes through the feet of the perpendiculars from $D$ to $\overline{A B}$ and $\overline{A C}$. Let $A_{1}$ denote the intersection of lines $\ell_{a}$ and $m_{a}$. Define points $B_{1}$ and $C_{1}$ similarly. Prove that triangles $D E F$ and $A_{1} B_{1} C_{1}$ are similar to each other. +2. A finite sequence of integers $a_{1}, a_{2}, \ldots, a_{n}$ is called regular if there exists a real number $x$ satisfying + +$$ +\lfloor k x\rfloor=a_{k} \quad \text { for } 1 \leq k \leq n +$$ + +Given a regular sequence $a_{1}, a_{2}, \ldots, a_{n}$, for $1 \leq k \leq n$ we say that the term $a_{k}$ is forced if the following condition is satisfied: the sequence + +$$ +a_{1}, a_{2}, \ldots, a_{k-1}, b +$$ + +is regular if and only if $b=a_{k}$. +Find the maximum possible number of forced terms in a regular sequence with 1000 terms. +3. Divide the plane into an infinite square grid by drawing all the lines $x=m$ and $y=n$ for $m, n \in \mathbb{Z}$. Next, if a square's upper-right corner has both coordinates even, color it black; otherwise, color it white (in this way, exactly $1 / 4$ of the squares are black and no two black squares are adjacent). Let $r$ and $s$ be odd integers, and let $(x, y)$ be a point in the interior of any white square such that $r x-s y$ is irrational. Shoot a laser out of this point with slope $r / s$; lasers pass through white squares and reflect off black squares. Prove that the path of this laser will from a closed loop. +4. Circle $\omega$, centered at $X$, is internally tangent to circle $\Omega$, centered at $Y$, at $T$. Let $P$ and $S$ be variable points on $\Omega$ and $\omega$, respectively, such that line $P S$ is tangent to $\omega($ at $S)$. Determine the locus of $O$ - the circumcenter of triangle $P S T$. +5. Let $p$ be a prime. Prove that in a complete graph with $1000 p$ vertices whose edges are labelled with integers, one can find a cycle whose sum of labels is divisible by $p$. +6. Let $\mathbb{N}$ be the set of positive integers. Find all functions $f: \mathbb{N} \rightarrow \mathbb{N}$ that satisfy the equation + +$$ +f^{a b c-a}(a b c)+f^{a b c-b}(a b c)+f^{a b c-c}(a b c)=a+b+c +$$ + +for all $a, b, c \geq 2$. (Here $f^{k}$ means $f$ applied $k$ times.) +7. A country has $n$ cities, labelled $1,2,3, \ldots, n$. It wants to build exactly $n-1$ roads between certain pairs of cities so that every city is reachable from every other city via some sequence of roads. However, it is not permitted to put roads between pairs of cities that have labels differing by exactly 1 , and it is also not permitted to put a road between cities 1 and $n$. Let $T_{n}$ be the total number of possible ways to build these roads. +(a) For all odd $n$, prove that $T_{n}$ is divisible by $n$. +(b) For all even $n$, prove that $T_{n}$ is divisible by $n / 2$. +8. Define a function $f: \mathbb{N} \rightarrow \mathbb{N}$ by $f(1)=1, f(n+1)=f(n)+2^{f(n)}$ for every positive integer $n$. Prove that $f(1), f(2), \ldots, f\left(3^{2013}\right)$ leave distinct remainders when divided by $3^{2013}$. +9. Let $r$ be a rational number in the interval $[-1,1]$ and let $\theta=\cos ^{-1} r$. Call a subset $S$ of the plane good if $S$ is unchanged upon rotation by $\theta$ around any point of $S$ (in both clockwise and counterclockwise directions). Determine all values of $r$ satisfying the following property: The midpoint of any two points in a good set also lies in the set. + +## §1 Solutions to Day 1 + +## §1.1 TSTST 2013/1 + +Available online at https://aops.com/community/p3181479. + +## Problem statement + +Let $A B C$ be a triangle and $D, E, F$ be the midpoints of $\operatorname{arcs} B C, C A, A B$ on the circumcircle. Line $\ell_{a}$ passes through the feet of the perpendiculars from $A$ to $\overline{D B}$ and $\overline{D C}$. Line $m_{a}$ passes through the feet of the perpendiculars from $D$ to $\overline{A B}$ and $\overline{A C}$. Let $A_{1}$ denote the intersection of lines $\ell_{a}$ and $m_{a}$. Define points $B_{1}$ and $C_{1}$ similarly. Prove that triangles $D E F$ and $A_{1} B_{1} C_{1}$ are similar to each other. + +In fact, it is true for any points $D, E, F$ on the circumcircle. More strongly we contend: + +Claim - Point $A_{1}$ is the midpoint of $\overline{H D}$. + +Proof. Lines $m_{a}$ and $\ell_{a}$ are Simson lines, so they both pass through the point $(a+b+$ $c+d) / 2$ in complex coordinates. +![](https://cdn.mathpix.com/cropped/2024_11_19_3879e3a8fa25bc819b5bg-04.jpg?height=718&width=801&top_left_y=1297&top_left_x=633) + +Hence $A_{1} B_{1} C_{1}$ is similar to $D E F$ through a homothety at $H$ with ratio $\frac{1}{2}$. + +## §1.2 TSTST 2013/2 + +Available online at https://aops.com/community/p3181480. + +## Problem statement + +A finite sequence of integers $a_{1}, a_{2}, \ldots, a_{n}$ is called regular if there exists a real number $x$ satisfying + +$$ +\lfloor k x\rfloor=a_{k} \quad \text { for } 1 \leq k \leq n . +$$ + +Given a regular sequence $a_{1}, a_{2}, \ldots, a_{n}$, for $1 \leq k \leq n$ we say that the term $a_{k}$ is forced if the following condition is satisfied: the sequence + +$$ +a_{1}, a_{2}, \ldots, a_{k-1}, b +$$ + +is regular if and only if $b=a_{k}$. +Find the maximum possible number of forced terms in a regular sequence with 1000 terms. + +The answer is 985 . WLOG, by shifting $a_{1}=0$ (clearly $a_{1}$ isn't forced). Now, we construct regular sequences inductively using the following procedure. Start with the inequality + +$$ +\frac{0}{1} \leq x<\frac{1}{1} +$$ + +Then for each $k=2,3, \ldots, 1000$ we perform the following procedure. If there is no fraction of the form $F=\frac{m}{k}$ in the interval $A \leq xq>\max \{a, b\}$ be primes. Suppose $s=a^{p} b^{q}$ and $t=s^{2}$; then + +$$ +p g_{t}(a)+q g_{t}(b)=g_{t}\left(a^{p} b^{q}\right)=g_{t}(s)=f^{s^{2}-s}(s)-s=0 +$$ + +Now + +$$ +q \mid g_{t}(a)>-a \quad \text { and } \quad p \mid g_{t}(b)>-b \Longrightarrow g_{t}(a)=g_{t}(b)=0 +$$ + +and so we conclude $f^{t-a}(t)=a$ and $f^{t-b}(t)=b$ for $a, b \geq 2$. +In particular, if $a=n$ and $b=n+1$ then we deduce $f(n+1)=n$ for all $n \geq 2$, as desired. + +Remark. If you let $c=(a b)^{2}$ after the first lemma, you recover the 2-variable version! + +## §3 Solutions to Day 3 + +## §3.1 TSTST 2013/7 + +Available online at https://aops.com/community/p3181485. + +## Problem statement + +A country has $n$ cities, labelled $1,2,3, \ldots, n$. It wants to build exactly $n-1$ roads between certain pairs of cities so that every city is reachable from every other city via some sequence of roads. However, it is not permitted to put roads between pairs of cities that have labels differing by exactly 1 , and it is also not permitted to put a road between cities 1 and $n$. Let $T_{n}$ be the total number of possible ways to build these roads. +(a) For all odd $n$, prove that $T_{n}$ is divisible by $n$. +(b) For all even $n$, prove that $T_{n}$ is divisible by $n / 2$. + +You can just spin the tree! +Fixing $n$, the group $G=\mathbb{Z} / n \mathbb{Z}$ acts on the set of trees by rotation (where we imagine placing $1,2, \ldots, n$ along a circle). + +Claim - For odd $n$, all trees have trivial stabilizer. +Proof. One way to see this is to look at the degree sequence. Suppose $g^{e}$ fixes a tree $T$. Then so does $g^{k}$, for $k=\operatorname{gcd}(e, n)$. Then it follows that $n / k$ divides $\sum_{v} \operatorname{deg} v=2 n-2$. Since $\operatorname{gcd}(2 n-2, n)=1$ we must then have $k=n$. + +The proof for even $n$ is identical except that $\operatorname{gcd}(2 n-2, n)=2$ and hence each tree either has stabilizer with size $\leq 2$. + +There is also a proof using linear algebra, using Kirchoff's tree formula. (Overkill.) + +## §3.2 TSTST 2013/8 + +Available online at https://aops.com/community/p3181486. + +## Problem statement + +Define a function $f: \mathbb{N} \rightarrow \mathbb{N}$ by $f(1)=1, f(n+1)=f(n)+2^{f(n)}$ for every positive integer $n$. Prove that $f(1), f(2), \ldots, f\left(3^{2013}\right)$ leave distinct remainders when divided by $3^{2013}$. + +I'll prove by induction on $k \geq 1$ that any $3^{k}$ consecutive values of $f$ produce distinct residues modulo $3^{k}$. The base case $k=1$ is easily checked ( $f$ is always odd, hence $f$ cycles $1,0,2 \bmod 3)$. + +For the inductive step, assume it's true up to $k$. Since $2^{\bullet}\left(\bmod 3^{k+1}\right)$ cycles every $2 \cdot 3^{k}$, and $f$ is always odd, it follows that + +$$ +\begin{aligned} +f\left(n+3^{k}\right)-f(n) & =2^{f(n)}+2^{f(n+1)}+\cdots+2^{f\left(n+3^{k}-1\right)} \quad\left(\bmod 3^{k+1}\right) \\ +& \equiv 2^{1}+2^{3}+\cdots+2^{2 \cdot 3^{k}-1} \quad\left(\bmod 3^{k+1}\right) \\ +& =2 \cdot \frac{4^{3^{k}}-1}{4-1} +\end{aligned} +$$ + +Hence + +$$ +f\left(n+3^{k}\right)-f(n) \equiv C \quad\left(\bmod 3^{k+1}\right) \quad \text { where } \quad C=2 \cdot \frac{4^{3^{k}}-1}{4-1} +$$ + +noting that $C$ does not depend on $n$. Exponent lifting gives $\nu_{3}(C)=k$ hence $f(n)$, $f\left(n+3^{k}\right), f\left(n+2 \cdot 3^{k}\right)$ differ mod $3^{k+1}$ for all $n$, and the inductive hypothesis now solves the problem. + +## §3.3 TSTST 2013/9 + +Available online at https://aops.com/community/p3181487. + +## Problem statement + +Let $r$ be a rational number in the interval $[-1,1]$ and let $\theta=\cos ^{-1} r$. Call a subset $S$ of the plane good if $S$ is unchanged upon rotation by $\theta$ around any point of $S$ (in both clockwise and counterclockwise directions). Determine all values of $r$ satisfying the following property: The midpoint of any two points in a good set also lies in the set. + +The answer is that $r$ has this property if and only if $r=\frac{4 n-1}{4 n}$ for some integer $n$. +Throughout the solution, we will let $r=\frac{a}{b}$ with $b>0$ and $\operatorname{gcd}(a, b)=1$. We also let + +$$ +\omega=e^{i \theta}=\frac{a}{b} \pm \frac{\sqrt{b^{2}-a^{2}}}{b} i +$$ + +This means we may work with complex multiplication in the usual way; the rotation of $z$ through center $c$ is given by $z \mapsto \omega(z-c)+c$. + +For most of our proof, we start by constructing a good set as follows. + +- Start by letting $S_{0}=\{0,1\}$. +- Let $S_{i}$ consist of $S_{i-1}$ plus all points that can be obtained by rotating a point of $S_{i-1}$ through a different point of $S_{i-1}$ (with scale factor $\omega$ ). +- Let $S_{\infty}=\bigcup_{i \geq 0} S_{i}$. + +The set $S_{\infty}$ is the (minimal, by inclusion) good set containing 0 and 1 . We are going to show that for most values of $r$, we have $\frac{1}{2} \notin S_{\infty}$. + +Claim - If $b$ is odd, then $\frac{1}{2} \notin S_{\infty}$. +Proof. Idea: denominators that appear are always odd. +Consider the ring + +$$ +A=\mathbb{Z}_{\{b\}}=\left\{\frac{s}{t}|s, t \in \mathbb{Z}, t| b^{\infty}\right\} +$$ + +which consists of all rational numbers whose denominators divide $b^{\infty}$. Then, $0,1, \omega \in$ $A\left[\sqrt{b^{2}-a^{2}}\right]$ and hence $S_{\infty} \subseteq A\left[\sqrt{b^{2}-a^{2}}\right]$ too. (This works even if $\sqrt{b^{2}-a^{2}} \in \mathbb{Z}$, in which case $S_{\infty} \subseteq A=A\left[\sqrt{b^{2}-a^{2}}\right]$.) +But $\frac{1}{2} \notin A\left[\sqrt{b^{2}-a^{2}}\right]$. + +Claim - If $b$ is even and $|b-a| \neq 1$, then $\frac{1}{2} \notin S_{\infty}$. +Proof. Idea: take modulo a prime dividing $b-a$. +Let $D=b^{2}-a^{2} \equiv 3(\bmod 4)$. Let $p$ be a prime divisor of $b-a$. Because $\operatorname{gcd}(a, b)=1$, we have $p \neq 2$ and $p \nmid b$. + +Consider the ring + +$$ +A=\mathbb{Z}_{(p)}=\left\{\left.\frac{s}{t} \right\rvert\, s, t \in \mathbb{Z}, p \perp t\right\} +$$ + +which consists of all rational numbers whose denominators are coprime to $p$. Then, $0,1, \omega \in A[\sqrt{-D}]$ and hence $S_{\infty} \subseteq A[\sqrt{-D}]$ too. + +Now, there is a well-defined "mod- $p$ " ring homomorphism + +$$ +\Psi: A[\sqrt{-D}] \rightarrow \mathbb{F}_{p} \quad \text { by } \quad x+y \sqrt{-D} \mapsto x \bmod p +$$ + +which commutes with addition and multiplication (as $p \mid D$ ). Under this map, + +$$ +\omega \mapsto \frac{a}{b} \bmod p=1 +$$ + +Consequently, the rotation $z \mapsto \omega(z-c)+c$ is just the identity map modulo $p$. In other words, the pre-image of any point in $S_{\infty}$ under $\Psi$ must be either $\Psi(0)=0$ or $\Psi(1)=1$. + +However, $\Psi(1 / 2)=1 / 2$ is neither of these. So this point cannot be achieved. + +Claim - Suppose $a=2 n-1$ and $b=2 n$ for $n$ an odd integer. Then $\frac{1}{2} \notin S_{\infty}$ + +Proof. Idea: $\omega$ is "algebraic integer" sans odd denominators. +This time, we define the ring + +$$ +B=\mathbb{Z}_{(2)}=\left\{\left.\frac{s}{t} \right\rvert\, s, t \in \mathbb{Z}, t \text { odd }\right\} +$$ + +of rational numbers with odd denominator. We carefully consider the ring $B[\omega]$ where + +$$ +\omega=\frac{2 n-1 \pm \sqrt{1-4 n}}{2 n} +$$ + +So $S_{\infty} \subseteq B[\omega]$ as $0,1, \omega \in B[\omega]$. +I claim that $B[\omega]$ is an integral extension of $B$; equivalently that $\omega$ is integral over $B$. Indeed, $\omega$ is the root of the monic polynomial + +$$ +(T-1)^{2}+\frac{1}{n}(T-1)-\frac{1}{n}=0 +$$ + +where $\frac{1}{n} \in B$ makes sense as $n$ is odd. +On the other hand, $\frac{1}{2}$ is not integral over $B$ so it is not an element of $B[\omega]$. +It remains to show that if $r=\frac{4 n-1}{4 n}$, then goods sets satisfy the midpoint property. Again starting from the points $z_{0}=0, z_{1}=1$ construct the sequence + +$$ +\begin{aligned} +z_{2} & =\omega\left(z_{1}-z_{0}\right)+z_{0} \\ +z_{3} & =\omega^{-1}\left(z_{0}-z_{2}\right)+z_{2} \\ +z_{4} & =\omega^{-1}\left(z_{2}-z_{3}\right)+z_{3} \\ +z_{5} & =\omega\left(z_{3}-z_{4}\right)+z_{4} +\end{aligned} +$$ + +as shown in the diagram below. +![](https://cdn.mathpix.com/cropped/2024_11_19_3879e3a8fa25bc819b5bg-16.jpg?height=321&width=795&top_left_y=2307&top_left_x=633) + +This construction shows that if we have the length-one segment $\{0,1\}$ then we can construct the length-one segment $\{2 r-2,2 r-1\}$. In other words, we can shift the segment to the left by + +$$ +1-(2 r-1)=2(1-r)=\frac{1}{2 n} +$$ + +Repeating this construction $n$ times gives the desired midpoint $\frac{1}{2}$. + diff --git a/USA_TST_TST/md/en-sols-TSTST-2014.md b/USA_TST_TST/md/en-sols-TSTST-2014.md new file mode 100644 index 0000000000000000000000000000000000000000..a92c6ffe27354eb4368632d05024e9d8d858dafd --- /dev/null +++ b/USA_TST_TST/md/en-sols-TSTST-2014.md @@ -0,0 +1,254 @@ +# TSTST 2014 Solution Notes + +## Lincoln, Nebraska + +## Evan Chen《陳誼廷》 + +15 April 2024 + +This is a compilation of solutions for the 2014 TSTST. The ideas of the solution are a mix of my own work, the solutions provided by the competition organizers, and solutions found by the community. However, all the writing is maintained by me. + +These notes will tend to be a bit more advanced and terse than the "official" solutions from the organizers. In particular, if a theorem or technique is not known to beginners but is still considered "standard", then I often prefer to use this theory anyways, rather than try to work around or conceal it. For example, in geometry problems I typically use directed angles without further comment, rather than awkwardly work around configuration issues. Similarly, sentences like "let $\mathbb{R}$ denote the set of real numbers" are typically omitted entirely. + +Corrections and comments are welcome! + +## Contents + +0 Problems ..... 2 +1 Solutions to Day 1 ..... 3 +1.1 TSTST 2014/1 ..... 3 +1.2 TSTST 2014/2 ..... 4 +1.3 TSTST 2014/3 ..... 5 +2 Solutions to Day 2 ..... 6 +2.1 TSTST 2014/4 ..... 6 +2.2 TSTST 2014/5 ..... 7 +2.3 TSTST 2014/6 ..... 8 + +## §0 Problems + +1. Let $\leftarrow$ denote the left arrow key on a standard keyboard. If one opens a text editor and types the keys "ab $\leftarrow \mathrm{cd} \leftarrow \leftarrow \mathrm{e} \leftarrow \leftarrow \mathrm{f}$ ", the result is "faecdb". We say that a string $B$ is reachable from a string $A$ if it is possible to insert some amount of $\leftarrow$ 's in $A$, such that typing the resulting characters produces $B$. So, our example shows that "faecdb" is reachable from "abcdef". + +Prove that for any two strings $A$ and $B, A$ is reachable from $B$ if and only if $B$ is reachable from $A$. +2. Consider a convex pentagon circumscribed about a circle. We name the lines that connect vertices of the pentagon with the opposite points of tangency with the circle gergonnians. +(a) Prove that if four gergonnians are concurrent, then all five of them are concurrent. +(b) Prove that if there is a triple of gergonnians that are concurrent, then there is another triple of gergonnians that are concurrent. +3. Find all polynomials $P(x)$ with real coefficients that satisfy + +$$ +P(x \sqrt{2})=P\left(x+\sqrt{1-x^{2}}\right) +$$ + +for all real numbers $x$ with $|x| \leq 1$. +4. Let $P(x)$ and $Q(x)$ be arbitrary polynomials with real coefficients, with $P \neq 0$, and let $d=\operatorname{deg} P$. Prove that there exist polynomials $A(x)$ and $B(x)$, not both zero, such that $\max \{\operatorname{deg} A, \operatorname{deg} B\} \leq d / 2$ and $P(x) \mid A(x)+Q(x) \cdot B(x)$. +5. Find the maximum number $E$ such that the following holds: there is an edge-colored graph with 60 vertices and $E$ edges, with each edge colored either red or blue, such that in that coloring, there is no monochromatic cycles of length 3 and no monochromatic cycles of length 5 . +6. Suppose we have distinct positive integers $a, b, c, d$ and an odd prime $p$ not dividing any of them, and an integer $M$ such that if one considers the infinite sequence + +$$ +\begin{gathered} +c a-d b \\ +c a^{2}-d b^{2} \\ +c a^{3}-d b^{3} \\ +c a^{4}-d b^{4} +\end{gathered} +$$ + +and looks at the highest power of $p$ that divides each of them, these powers are not all zero, and are all at most $M$. Prove that there exists some $T$ (which may depend on $a, b, c, d, p, M)$ such that whenever $p$ divides an element of this sequence, the maximum power of $p$ that divides that element is exactly $p^{T}$. + +## §1 Solutions to Day 1 + +## §1.1 TSTST 2014/1 + +Available online at https://aops.com/community/p3549404. + +## Problem statement + +Let $\leftarrow$ denote the left arrow key on a standard keyboard. If one opens a text editor and types the keys "ab $\leftarrow \mathrm{cd} \leftarrow \leftarrow \mathrm{e} \leftarrow \leftarrow \mathrm{f}$ ", the result is "faecdb". We say that a string $B$ is reachable from a string $A$ if it is possible to insert some amount of $\leftarrow$ 's in $A$, such that typing the resulting characters produces $B$. So, our example shows that "faecdb" is reachable from "abcdef". + +Prove that for any two strings $A$ and $B, A$ is reachable from $B$ if and only if $B$ is reachable from $A$. + +Obviously $A$ and $B$ should have the same multiset of characters, and we focus only on that situation. + +Claim - If $A=123 \ldots n$ and $B=\sigma(1) \sigma(2) \ldots \sigma(n)$ is a permutation of $A$, then $B$ is reachable if and only if it is $\mathbf{2 1 3}$-avoiding, i.e. there are no indices $i0}$ such that $x \equiv y \equiv 1(\bmod p)$. If the sequence $\nu_{p}\left(x^{n}-y\right)$ of positive integers is nonconstant, then it is unbounded. + +For this it would be sufficient to prove the following claim. +Claim - Let $p$ be an odd prime. Let $x, y \in \mathbb{Q}>0$ such that $x \equiv y \equiv 1(\bmod p)$. Suppose $m$ and $n$ are positive integers such that + +$$ +d=\nu_{p}\left(x^{n}-y\right)<\nu_{p}\left(x^{m}-y\right)=e . +$$ + +Then there exists $\ell$ such that $\nu_{p}\left(x^{\ell}-y\right) \geq e+1$. +Proof. First, note that $\nu_{p}\left(x^{m}-x^{n}\right)=\nu_{p}\left(\left(x^{m}-y\right)-\left(x^{n}-y\right)\right)=d$ and so by exponent lifting we can find some $k$ such that + +$$ +\nu_{p}\left(x^{k}-1\right)=e +$$ + +namely $k=p^{e-d}|m-n|$. (In fact, one could also choose more carefully $k=p^{e-d} \cdot \operatorname{gcd}(m-$ $\left.n, p^{\infty}\right)$, so that $k$ is a power of $p$.) + +Suppose we set $x^{k}=p^{e} u+1$ and $x^{m}=p^{e} v+y$ where $u, v \in \mathbb{Q}$ aren't divisible by $p$. Now for any integer $1 \leq r \leq p-1$ we consider + +$$ +\begin{aligned} +x^{k r+m}-y & =\left(p^{e} u+1\right)^{r} \cdot\left(p^{e} v+y\right)-y \\ +& =p^{e}(v+y u \cdot r)+p^{2 e}(\ldots) +\end{aligned} +$$ + +By selecting $r$ with $r \equiv-v / u(\bmod p)$, we ensure $p^{e+1} \mid x^{k r+m}-y$, hence $\ell=k r+m$ is as desired. + +Remark. One way to motivate the proof of the claim is as follows. Suppose we are given $\nu_{p}\left(x^{m}-y\right)=e$, and we wish to find $\ell$ such that $\nu_{p}\left(x^{\ell}-y\right)>e$. Then, it is necessary (albeit insufficient) that + +$$ +x^{\ell-m} \equiv 1 \quad\left(\bmod p^{e}\right) \text { but } x^{\ell-m} \not \equiv 1 \quad\left(\bmod p^{e+1}\right) +$$ + +In particular, we need $\nu_{p}\left(x^{\ell-m}-1\right)=e$ exactly. So the $k$ in the claim must exist if we are going to succeed. + +On the other hand, if $k$ is some integer for which $\nu_{p}\left(x^{k}-1\right)=e$, then by choosing $\ell-m$ to be some multiple of $k$ with no extra factors of $p$, we hope that we can get $\nu_{p}\left(x^{\ell}-y\right)=e+1$. That's why we write $\ell=k r+m$ and see what happens when we expand. + diff --git a/USA_TST_TST/md/en-sols-TSTST-2015.md b/USA_TST_TST/md/en-sols-TSTST-2015.md new file mode 100644 index 0000000000000000000000000000000000000000..02871c0434039a1345406029ad37bb630783b57e --- /dev/null +++ b/USA_TST_TST/md/en-sols-TSTST-2015.md @@ -0,0 +1,304 @@ +# TSTST 2015 Solution Notes + +## Pittsburgh, PA + +Evan Chen《陳誼廷》 + +15 April 2024 + +This is a compilation of solutions for the 2015 TSTST. The ideas of the solution are a mix of my own work, the solutions provided by the competition organizers, and solutions found by the community. However, all the writing is maintained by me. + +These notes will tend to be a bit more advanced and terse than the "official" solutions from the organizers. In particular, if a theorem or technique is not known to beginners but is still considered "standard", then I often prefer to use this theory anyways, rather than try to work around or conceal it. For example, in geometry problems I typically use directed angles without further comment, rather than awkwardly work around configuration issues. Similarly, sentences like "let $\mathbb{R}$ denote the set of real numbers" are typically omitted entirely. + +Corrections and comments are welcome! + +## Contents + +0 Problems ..... 2 +1 Solutions to Day 1 ..... 3 +1.1 TSTST 2015/1, proposed by Mark Sellke ..... 3 +1.2 TSTST 2015/2, proposed by Ivan Borsenco +1.3 TSTST 2015/3, proposed by Alex Zhai +2 Solutions to Day 2 ..... 6 +2.1 TSTST 2015/4, proposed by Alyazeed Basyoni ..... 6 +2.2 TSTST 2015/5 +2.3 TSTST 2015/6, proposed by Linus Hamilton + +## §0 Problems + +1. Let $a_{1}, a_{2}, \ldots, a_{n}$ be a sequence of real numbers, and let $m$ be a fixed positive integer less than $n$. We say an index $k$ with $1 \leq k \leq n$ is good if there exists some $\ell$ with $1 \leq \ell \leq m$ such that + +$$ +a_{k}+a_{k+1}+\cdots+a_{k+\ell-1} \geq 0 +$$ + +where the indices are taken modulo $n$. Let $T$ be the set of all good indices. Prove that + +$$ +\sum_{k \in T} a_{k} \geq 0 +$$ + +2. Let $A B C$ be a scalene triangle. Let $K_{a}, L_{a}$, and $M_{a}$ be the respective intersections with $B C$ of the internal angle bisector, external angle bisector, and the median from $A$. The circumcircle of $A K_{a} L_{a}$ intersects $A M_{a}$ a second time at a point $X_{a}$ different from $A$. Define $X_{b}$ and $X_{c}$ analogously. Prove that the circumcenter of $X_{a} X_{b} X_{c}$ lies on the Euler line of $A B C$. +3. Let $P$ be the set of all primes, and let $M$ be a non-empty subset of $P$. Suppose that for any non-empty subset $\left\{p_{1}, p_{2}, \ldots, p_{k}\right\}$ of $M$, all prime factors of $p_{1} p_{2} \ldots p_{k}+1$ are also in $M$. Prove that $M=P$. +4. Let $x, y, z$ be real numbers (not necessarily positive) such that $x^{4}+y^{4}+z^{4}+x y z=4$. Prove that $x \leq 2$ and + +$$ +\sqrt{2-x} \geq \frac{y+z}{2} +$$ + +5. Let $\varphi(n)$ denote the number of positive integers less than $n$ that are relatively prime to $n$. Prove that there exists a positive integer $m$ for which the equation $\varphi(n)=m$ has at least 2015 solutions in $n$. +6. A Nim-style game is defined as follows. Two positive integers $k$ and $n$ are specified, along with a finite set $S$ of $k$-tuples of integers (not necessarily positive). At the start of the game, the $k$-tuple $(~ n, 0,0, \ldots, 0)$ is written on the blackboard. +A legal move consists of erasing the tuple $\left(a_{1}, a_{2}, \ldots, a_{k}\right)$ which is written on the blackboard and replacing it with $\left(a_{1}+b_{1}, a_{2}+b_{2}, \ldots, a_{k}+b_{k}\right)$, where $\left(b_{1}, b_{2}, \ldots, b_{k}\right)$ is an element of the set $S$. Two players take turns making legal moves, and the first to write a negative integer loses. In the event that neither player is ever forced to write a negative integer, the game is a draw. +Prove that there is a choice of $k$ and $S$ with the following property: the first player has a winning strategy if $n$ is a power of 2 , and otherwise the second player has a winning strategy. + +## §1 Solutions to Day 1 + +## §1.1 TSTST 2015/1, proposed by Mark Sellke + +Available online at https://aops.com/community/p5017901. + +## Problem statement + +Let $a_{1}, a_{2}, \ldots, a_{n}$ be a sequence of real numbers, and let $m$ be a fixed positive integer less than $n$. We say an index $k$ with $1 \leq k \leq n$ is good if there exists some $\ell$ with $1 \leq \ell \leq m$ such that + +$$ +a_{k}+a_{k+1}+\cdots+a_{k+\ell-1} \geq 0 +$$ + +where the indices are taken modulo $n$. Let $T$ be the set of all good indices. Prove that + +$$ +\sum_{k \in T} a_{k} \geq 0 +$$ + +First we prove the result if the indices are not taken modulo $n$. Call a number $\ell$-good if $\ell$ is the smallest number such that $a_{k}+a_{k+1}+\cdots+a_{k+\ell-1} \geq 0$, and $\ell \leq m$. Then if $a_{k}$ is $\ell$-good, the numbers $a_{k+1}, \ldots, a_{k+\ell-1}$ are good as well. + +Then by greedy from left to right, we can group all the good numbers into blocks with nonnegative sums. Repeatedly take the first good number, if $\ell$-good, group it with the next $\ell$ numbers. An example for $m=3$ : + +$$ +\langle 4\rangle \quad \begin{array}{ccccccccc} +-1 & -2 & 6\rangle & -9 & -7 & \langle 3\rangle & \langle-2 & 4\rangle & \langle-1 . +\end{array} +$$ + +We can now return to the original problem. Let $N$ be a large integer; applying the algorithm to $N$ copies of the sequence, we deduce that + +$$ +N \sum_{k \in T} a_{k}+c_{N} \geq 0 +$$ + +where $c_{N}$ represents some "error" from left-over terms. As $\left|c_{N}\right| \leq \sum\left|a_{i}\right|$, by taking $N$ large enough we deduce the problem. + +Remark. This solution was motivated by looking at the case $m=1$, realizing how dumb it was, then looking at $m=2$, and realizing it was equally dumb. + +## §1.2 TSTST 2015/2, proposed by Ivan Borsenco + +Available online at https://aops.com/community/p5017915. + +## Problem statement + +Let $A B C$ be a scalene triangle. Let $K_{a}, L_{a}$, and $M_{a}$ be the respective intersections with $B C$ of the internal angle bisector, external angle bisector, and the median from $A$. The circumcircle of $A K_{a} L_{a}$ intersects $A M_{a}$ a second time at a point $X_{a}$ different from $A$. Define $X_{b}$ and $X_{c}$ analogously. Prove that the circumcenter of $X_{a} X_{b} X_{c}$ lies on the Euler line of $A B C$. + +The main content of the problem: +Claim - $\angle H X_{a} G=90^{\circ}$. +This implies the result, since then the desired circumcenter is the midpoint of $\overline{G H}$. (This is the main difficulty; the Euler line is a red herring.) +In what follows, we abbreviate $K_{a} L_{a}, M_{a}, X_{a}$ to $K, L, M, X$. +First proof by Brokard. To do this, it suffices to show that $M$ has the same power with respect to the circle with diameter $\overline{A H}$ and the circle with diameter $\overline{K L}$. In fact I claim both circles are orthogonal to the circle with diameter $\overline{B C}$ ! The former follows from Brokard's theorem, noting that $A$ is on the polar of $H$, and the latter follows from the harmonic bundle. +![](https://cdn.mathpix.com/cropped/2024_11_19_8fb7b517def5801eef36g-04.jpg?height=635&width=1109&top_left_y=1410&top_left_x=482) + +Then $\overline{A M}$ is the radical axis, so $X$ lies on both circles. +Second proof by orthocenter reflection, Bendit Chan. As before, we know $M X \cdot M A=$ $M K \cdot M L=M B \cdot M C$, but $X$ lies inside segment $A M$. Construct parallelogram $A B A^{\prime} C$. Then $M X \cdot M A^{\prime}=M B \cdot M C$, so $X B A^{\prime} C$ is concyclic. + +However, it is well-known the circumcircle of $\triangle B A^{\prime} C$ (which is the reflection of $(A B C)$ across $\overline{B C}$ ) passes through $H$ and in fact has diameter $\overline{A^{\prime} H}$. So this gives $\angle H X A^{\prime}=90^{\circ}$ as needed. +Third proof by barycentric coordinates. Alternatively we may just compute $X=\left(a^{2}\right.$ : $\left.2 S_{A}: 2 S_{A}\right)$. Let $F=\left(0: S_{C}: S_{B}\right)$ be the foot from $H$. Then we check that $X H F M$ is cyclic, which is power of a point from $A$. + +## §1.3 TSTST 2015/3, proposed by Alex Zhai + +Available online at https://aops.com/community/p5017928. + +## Problem statement + +Let $P$ be the set of all primes, and let $M$ be a non-empty subset of $P$. Suppose that for any non-empty subset $\left\{p_{1}, p_{2}, \ldots, p_{k}\right\}$ of $M$, all prime factors of $p_{1} p_{2} \ldots p_{k}+1$ are also in $M$. Prove that $M=P$. + +The following solution was found by user Aiscrim on AOPS. +Obviously $|M|=\infty$. Assume for contradiction $p \notin M$. We say a prime $q \in M$ is sparse if there are only finitely many elements of $M$ which are $q(\bmod p)$ (in particular there are finitely many sparse primes). + +Now let $C$ be the product of all sparse primes (note $p \nmid C$ ). First, set $a_{0}=1$. For $k \geq 0$, consider then the prime factorization of the number + +$$ +C a_{k}+1 +$$ + +No prime in its factorization is sparse, so consider the number $a_{k+1}$ obtained by replacing each prime in its factorization with some arbitrary representative of that prime's residue class. In this way we select a number $a_{k+1}$ such that + +- $a_{k+1} \equiv C a_{k}+1(\bmod p)$, and +- $a_{k+1}$ is a product of distinct primes in $M$. + +In particular, $a_{k} \equiv C^{k}+C^{k-1}+\cdots+1(\bmod p)$ +But since $C \not \equiv 0(\bmod p)$, we can find a $k$ such that $a_{k} \equiv 0(\bmod p)($ namely, $k=p-1$ if $C \equiv 1$ and $k=p-2$ else) which is clearly impossible since $a_{k}$ is a product of primes in $M$ ! + +## §2 Solutions to Day 2 + +## §2.1 TSTST 2015/4, proposed by Alyazeed Basyoni + +Available online at https://aops.com/community/p5017801. + +## Problem statement + +Let $x, y, z$ be real numbers (not necessarily positive) such that $x^{4}+y^{4}+z^{4}+x y z=4$. Prove that $x \leq 2$ and + +$$ +\sqrt{2-x} \geq \frac{y+z}{2} +$$ + +We prove that the condition $x^{4}+y^{4}+z^{4}+x y z=4$ implies + +$$ +\sqrt{2-x} \geq \frac{y+z}{2} +$$ + +We first prove the easy part. +Claim - We have $x \leq 2$. +Proof. Indeed, AM-GM gives that + +$$ +\begin{aligned} +5=x^{4}+y^{4}+\left(z^{4}+1\right)+x y z & =\frac{3 x^{4}}{4}+\left(\frac{x^{4}}{4}+y^{4}\right)+\left(z^{4}+1\right)+x y z \\ +& \geq \frac{3 x^{4}}{4}+x^{2} y^{2}+2 z^{2}+x y z . +\end{aligned} +$$ + +We evidently have that $x^{2} y^{2}+2 z^{2}+x y z \geq 0$ because the quadratic form $a^{2}+a b+2 b^{2}$ is positive definite, so $x^{4} \leq \frac{20}{3} \Longrightarrow x \leq 2$. + +Now, the desired statement is implied by its square, so it suffices to show that + +$$ +2-x \geq\left(\frac{y+z}{2}\right)^{2} +$$ + +We are going to proceed by contradiction (it seems that many solutions do this) and assume that + +$$ +2-x<\left(\frac{y+z}{2}\right)^{2} \Longleftrightarrow 4 x+y^{2}+2 y z+z^{2}>8 +$$ + +By AM-GM, + +$$ +\begin{aligned} +x^{4}+3 & \geq 4 x \\ +\frac{y^{4}+1}{2} & \geq y^{2} \\ +\frac{z^{4}+1}{2} & \geq z^{2} +\end{aligned} +$$ + +which yields that + +$$ +x^{4}+\frac{y^{4}+z^{4}}{2}+2 y z+4>8 +$$ + +If we replace $x^{4}=4-\left(y^{4}+z^{4}+x y z\right)$ now, this gives + +$$ +-\frac{y^{4}+z^{4}}{2}+(2-x) y z>0 \Longrightarrow(2-x) y z>\frac{y^{4}+z^{4}}{2} +$$ + +Since $2-x$ and the right-hand side are positive, we have $y z \geq 0$. Now + +$$ +\frac{y^{4}+z^{4}}{2 y z}<2-x<\left(\frac{y+z}{2}\right)^{2} \Longrightarrow 2 y^{4}+2 z^{4}2015$ we're done. +Remark. This solution is motivated by the deep fact that $\varphi(11 \cdot 1000)=\varphi(10 \cdot 1000)$, for example. + +【 Second solution with smallest primes, by Yang Liu. Let $2=p_{1}1$. Then Alice can complete the phase without losing if and only if $x$ is even; if so she begins a $Y$-phase with $(X, Y)=(0, x / 2)$. + +Proof. As $x>1$, Alice cannot play ClaimX since Bob will respond with FakeX and win. Now by alternating between WorkX and WorkX', Alice can repeatedly deduct 2 from $X$ and add 1 to $Y$, leading to $(x-2, y+1),(x-4, y+2)$, and so on. (During this time, Bob can only play Sleep.) Eventually, she must stop this process by playing DoneX, which begins a $Y$-phase. + +Now note that unless $X=0$, Bob now has a winning move WrongX. Conversely he may only play Sleep if $X=0$. + +We have an analogous claim for $Y$-phases. Thus if $n$ is not a power of 2 , we see that Alice eventually loses. + +Now suppose $n=2^{k}$; then Alice reaches $(X, Y)=\left(0,2^{k-1}\right),\left(2^{k-2}, 0\right), \ldots$ until either reaching $(1,0)$ or $(0,1)$. At this point she can play ClaimX or ClaimY, respectively; the game is now in state Cl. Bob cannot play either FakeX or FakeY, so he must play Sleep, and then Alice wins by playing Win. Thus Alice has a winning strategy when $n=2^{k}$. + diff --git a/USA_TST_TST/md/en-sols-TSTST-2016.md b/USA_TST_TST/md/en-sols-TSTST-2016.md new file mode 100644 index 0000000000000000000000000000000000000000..fe89cf8b0ea70b2071fd4affbeb8c0ad493e9eef --- /dev/null +++ b/USA_TST_TST/md/en-sols-TSTST-2016.md @@ -0,0 +1,278 @@ +# USA TSTST 2016 Solutions
United States of America - TST Selection Test
Evan Chen《陳誼廷》
$58^{\text {th }}$ IMO 2017 Brazil and $6^{\text {th }}$ EGMO 2017 Switzerland + +## Contents + +0 Problems ..... 2 +1 Solutions to Day 1 ..... 3 +1.1 TSTST 2016/1, proposed by Victor Wang ..... 3 +1.2 TSTST 2016/2, proposed by Evan Chen ..... 4 +1.3 TSTST 2016/3, proposed by Yang Liu ..... 6 +2 Solutions to Day 2 ..... 8 +2.1 TSTST 2016/4, proposed by Linus Hamilton ..... 8 +2.2 TSTST 2016/5, proposed by Linus Hamilton, Cynthia Stoner ..... 9 +2.3 TSTST 2016/6, proposed by Danielle Wang ..... 10 + +## §0 Problems + +1. Let $A=A(x, y)$ and $B=B(x, y)$ be two-variable polynomials with real coefficients. Suppose that $A(x, y) / B(x, y)$ is a polynomial in $x$ for infinitely many values of $y$, and a polynomial in $y$ for infinitely many values of $x$. Prove that $B$ divides $A$, meaning there exists a third polynomial $C$ with real coefficients such that $A=B \cdot C$. +2. Let $A B C$ be a scalene triangle with orthocenter $H$ and circumcenter $O$ and denote by $M, N$ the midpoints of $\overline{A H}, \overline{B C}$. Suppose the circle $\gamma$ with diameter $\overline{A H}$ meets the circumcircle of $A B C$ at $G \neq A$, and meets line $\overline{A N}$ at $Q \neq A$. The tangent to $\gamma$ at $G$ meets line $O M$ at $P$. Show that the circumcircles of $\triangle G N Q$ and $\triangle M B C$ intersect on $\overline{P N}$. +3. Decide whether or not there exists a nonconstant polynomial $Q(x)$ with integer coefficients with the following property: for every positive integer $n>2$, the numbers + +$$ +Q(0), Q(1), Q(2), \ldots, Q(n-1) +$$ + +produce at most $0.499 n$ distinct residues when taken modulo $n$. +4. Prove that if $n$ and $k$ are positive integers satisfying $\varphi^{k}(n)=1$, then $n \leq 3^{k}$. (Here $\varphi^{k}$ denotes $k$ applications of the Euler phi function.) +5. In the coordinate plane are finitely many walls, which are disjoint line segments, none of which are parallel to either axis. A bulldozer starts at an arbitrary point and moves in the $+x$ direction. Every time it hits a wall, it turns at a right angle to its path, away from the wall, and continues moving. (Thus the bulldozer always moves parallel to the axes.) +Prove that it is impossible for the bulldozer to hit both sides of every wall. +6. Let $A B C$ be a triangle with incenter $I$, and whose incircle is tangent to $\overline{B C}, \overline{C A}$, $\overline{A B}$ at $D, E, F$, respectively. Let $K$ be the foot of the altitude from $D$ to $\overline{E F}$. Suppose that the circumcircle of $\triangle A I B$ meets the incircle at two distinct points $C_{1}$ and $C_{2}$, while the circumcircle of $\triangle A I C$ meets the incircle at two distinct points $B_{1}$ and $B_{2}$. Prove that the radical axis of the circumcircles of $\triangle B B_{1} B_{2}$ and $\triangle C C_{1} C_{2}$ passes through the midpoint $M$ of $\overline{D K}$. + +## §1 Solutions to Day 1 + +## §1.1 TSTST 2016/1, proposed by Victor Wang + +Available online at https://aops.com/community/p6575197. + +## Problem statement + +Let $A=A(x, y)$ and $B=B(x, y)$ be two-variable polynomials with real coefficients. Suppose that $A(x, y) / B(x, y)$ is a polynomial in $x$ for infinitely many values of $y$, and a polynomial in $y$ for infinitely many values of $x$. Prove that $B$ divides $A$, meaning there exists a third polynomial $C$ with real coefficients such that $A=B \cdot C$. + +This is essentially an application of the division algorithm, but the details require significant care. + +First, we claim that $A / B$ can be written as a polynomial in $x$ whose coefficients are rational functions in $y$. To see this, use the division algorithm to get + +$$ +A=Q \cdot B+R \quad Q, R \in(\mathbb{R}(y))[x] +$$ + +where $Q$ and $R$ are polynomials in $x$ whose coefficients are rational functions in $y$, and moreover $\operatorname{deg}_{x} B>\operatorname{deg}_{x} R$. + +Now, we claim that $R \equiv 0$. Indeed, we have by hypothesis that for infinitely many values of $y_{0}$ that $B\left(x, y_{0}\right)$ divides $A\left(x, y_{0}\right)$, which means $B\left(x, y_{0}\right) \mid R\left(x, y_{0}\right)$ as polynomials in $\mathbb{R}[x]$. Now, we have $\operatorname{deg}_{x} B\left(x, y_{0}\right)>\operatorname{deg}_{x} R\left(x, y_{0}\right)$ outside of finitely many values of $y_{0}$ (but not all of them!); this means for infinitely many $y_{0}$ we have $R\left(x, y_{0}\right) \equiv 0$. So each coefficient of $x^{i}$ (in $\left.\mathbb{R}(y)\right)$ has infinitely many roots, hence is a zero polynomial. + +Consequently, we are able to write $A / B=F(x, y) / M(y)$ where $F \in \mathbb{R}[x, y]$ and $M \in \mathbb{R}[y]$ are each polynomials. Repeating the same argument now gives + +$$ +\frac{A}{B}=\frac{F(x, y)}{M(y)}=\frac{G(x, y)}{N(x)} +$$ + +Now, by unique factorization of polynomials in $\mathbb{R}[x, y]$, we can discuss GCD's. So, we tacitly assume $\operatorname{gcd}(F, M)=\operatorname{gcd}(G, N)=(1)$. Also, we obviously have $\operatorname{gcd}(M, N)=(1)$. But $F \cdot N=G \cdot M$, so $M \mid F \cdot N$, thus we conclude $M$ is the constant polynomial. This implies the result. + +Remark. This fact does not generalize to arbitrary functions that are separately polynomial: see e.g. http://aops.com/community/c6h523650p2978180. + +## §1.2 TSTST 2016/2, proposed by Evan Chen + +Available online at https://aops.com/community/p6575204. + +## Problem statement + +Let $A B C$ be a scalene triangle with orthocenter $H$ and circumcenter $O$ and denote by $M, N$ the midpoints of $\overline{A H}, \overline{B C}$. Suppose the circle $\gamma$ with diameter $\overline{A H}$ meets the circumcircle of $A B C$ at $G \neq A$, and meets line $\overline{A N}$ at $Q \neq A$. The tangent to $\gamma$ at $G$ meets line $O M$ at $P$. Show that the circumcircles of $\triangle G N Q$ and $\triangle M B C$ intersect on $\overline{P N}$. + +We present two solutions, one using essentially only power of a point, and the other more involved. + +【 First solution (found by contestants). Denote by $\triangle D E F$ the orthic triangle. Observe $\overline{P A}$ and $\overline{P G}$ are tangents to $\gamma$, since $\overline{O M}$ is the perpendicular bisector of $\overline{A G}$. Also note that $\overline{A G}, \overline{E F}, \overline{B C}$ are concurrent at some point $R$ by radical axis on $(A B C), \gamma$, (BFEC). + +Now, consider circles (PAGM), (MFDNE), and (MBC). We already saw the point $R$ satisfies + +$$ +R A \cdot R G=R E \cdot R F=R B \cdot R C +$$ + +and hence has equal powers to all three circles; but since the circles at $M$ already, they must actually be coaxial. Assume they meet again at $T \in \overline{R M}$, say. Then $\angle P T M$ and $\angle M T N$ are both right angles, hence $T$ lies on $\overline{P N}$. + +Finally $H$ is the orthocenter of $\triangle A R N$, and thus the circle with diameter $\overline{R N}$ passes through $G, Q, N$. +![](https://cdn.mathpix.com/cropped/2024_11_19_f48de8bcaa95c5ff4531g-04.jpg?height=969&width=1117&top_left_y=1591&top_left_x=475) + +『 Alternate solution (by proposer). Let $L$ be diametrically opposite $A$ on the circumcircle. Denote by $\triangle D E F$ the orthic triangle. Let $X=\overline{A H} \cap \overline{E F}$. Finally, let $T$ be the second intersection of (MFDNE) and (MBC). +![](https://cdn.mathpix.com/cropped/2024_11_19_f48de8bcaa95c5ff4531g-05.jpg?height=838&width=1109&top_left_y=429&top_left_x=482) + +We begin with a few easy observations. First, points $H, G, N, L$ are collinear and $\angle A G L=90^{\circ}$. Also, $Q$ is the foot from $H$ to $\overline{A N}$. Consequently, lines $A G, E F, H Q$, $B C, T M$ concur at a point $R$ (radical axis). Moreover, we already know $\angle M T N=90^{\circ}$. This implies $T$ lies on the circle with diameter $\overline{R N}$, which is exactly the circumcircle of $\triangle G Q N$. + +Note by Brokard's Theorem on $A F H E$, the point $X$ is the orthocenter of $\triangle M B C$. But $\angle M T N=90^{\circ}$ already, and $N$ is the midpoint of $\overline{B C}$. Consequently, points $T, X$, $N$ are collinear. + +Finally, we claim $P, X, N$ are collinear, which solves the problem. Note $P=\overline{G G} \cap \overline{A A}$. Set $K=\overline{H N L} \cap \overline{A P}$. Then by noting + +$$ +-1=(D, X ; A, H) \stackrel{N}{=}(\infty, \overline{N X} \cap \overline{A K} ; A, K) +$$ + +we see that $\overline{N X}$ bisects segment $\overline{A K}$, as desired. (A more projective finish is to show that $\overline{P X N}$ is the polar of $R$ to $\gamma$ ). + +Remark. The original problem proposal reads as follows: +Let $A B C$ be a triangle with orthocenter $H$ and circumcenter $O$ and denote by $M, N$ the midpoints of $\overline{A H}, \overline{B C}$. Suppose ray $O M$ meets the line parallel to $\overline{B C}$ through $A$ at $P$. Prove that the line through the circumcenter of $\triangle M B C$ and the midpoint of $\overline{O H}$ is parallel to $\overline{N P}$. +The points $G$ and $Q$ were added to the picture later to prevent the problem from being immediate by coordinates. + +## §1.3 TSTST 2016/3, proposed by Yang Liu + +Available online at https://aops.com/community/p6575217. + +## Problem statement + +Decide whether or not there exists a nonconstant polynomial $Q(x)$ with integer coefficients with the following property: for every positive integer $n>2$, the numbers + +$$ +Q(0), Q(1), Q(2), \ldots, Q(n-1) +$$ + +produce at most $0.499 n$ distinct residues when taken modulo $n$. + +We claim that + +$$ +Q(x)=420\left(x^{2}-1\right)^{2} +$$ + +works. Clearly, it suffices to prove the result when $n=4$ and when $n$ is an odd prime $p$. The case $n=4$ is trivial, so assume now $n=p$ is an odd prime. + +First, we prove the following easy claim. +Claim - For any odd prime $p$, there are at least $\frac{1}{2}(p-3)$ values of $a$ for which $\left(\frac{1-a^{2}}{p}\right)=+1$. + +Proof. Note that if $k \neq 0$ and $k^{2} \neq-1$, then $a=\frac{1-k^{2}}{k^{2}+1}$ works. +Remark. The above identity comes from starting with the equation $1-a^{2}=b^{2}$, and writing it as $\left(\frac{1}{b}\right)^{2}-\left(\frac{a}{b}\right)^{2}=1$. Then solve $\frac{1}{b}-\frac{a}{b}=k$ and $\frac{1}{b}+\frac{a}{b}=1 / k$ for $a$. + +Let $F(x)=\left(x^{2}-1\right)^{2}$. The range of $F$ modulo $p$ is contained within the $\frac{1}{2}(p+1)$ quadratic residues modulo $p$. On the other hand, if for some $t$ neither of $1 \pm t$ is a quadratic residue, then $t^{2}$ is omitted from the range of $F$ as well. Call such a value of $t$ useful, and let $N$ be the number of useful residues. We aim to show $N \geq \frac{1}{4} p-2$. + +We compute a lower bound on the number $N$ of useful $t$ by writing + +$$ +\begin{aligned} +N & =\frac{1}{4}\left(\sum_{t}\left[\left(1-\left(\frac{1-t}{p}\right)\right)\left(1-\left(\frac{1+t}{p}\right)\right)\right]-\left(1-\left(\frac{2}{p}\right)\right)-\left(1-\left(\frac{-2}{p}\right)\right)\right) \\ +& \geq \frac{1}{4} \sum_{t}\left[\left(1-\left(\frac{1-t}{p}\right)\right)\left(1-\left(\frac{1+t}{p}\right)\right)\right]-1 \\ +& =\frac{1}{4}\left(p+\sum_{t}\left(\frac{1-t^{2}}{p}\right)\right)-1 \\ +& \geq \frac{1}{4}\left(p+(+1) \cdot \frac{1}{2}(p-3)+0 \cdot 2+(-1) \cdot\left((p-2)-\frac{1}{2}(p-3)\right)\right)-1 \\ +& \geq \frac{1}{4}(p-5) . +\end{aligned} +$$ + +Thus, the range of $F$ has size at most + +$$ +\frac{1}{2}(p+1)-\frac{1}{2} N \leq \frac{3}{8}(p+3) . +$$ + +This is less than $0.499 p$ for any $p \geq 11$. + +Remark. In fact, the computation above is essentially an equality. There are only two points where terms are dropped: one, when $p \equiv 3(\bmod 4)$ there are no $k^{2}=-1$ in the lemma, and secondly, the terms $1-(2 / p)$ and $1-(-2 / p)$ are dropped in the initial estimate for $N$. With suitable modifications, one can show that in fact, the range of $F$ is exactly equal to + +$$ +\frac{1}{2}(p+1)-\frac{1}{2} N=\left\{\begin{array}{lll} +\frac{1}{8}(3 p+5) & p \equiv 1 & (\bmod 8) \\ +\frac{1}{8}(3 p+7) & p \equiv 3 & (\bmod 8) \\ +\frac{1}{8}(3 p+9) & p \equiv 5 & (\bmod 8) \\ +\frac{1}{8}(3 p+3) & p \equiv 7 & (\bmod 8) +\end{array}\right. +$$ + +## §2 Solutions to Day 2 + +## §2.1 TSTST 2016/4, proposed by Linus Hamilton + +Available online at https://aops.com/community/p6580534. + +## Problem statement + +Prove that if $n$ and $k$ are positive integers satisfying $\varphi^{k}(n)=1$, then $n \leq 3^{k}$. (Here $\varphi^{k}$ denotes $k$ applications of the Euler phi function.) + +The main observation is that the exponent of 2 decreases by at most 1 with each application of $\varphi$. This will give us the desired estimate. + +Define the weight function $w$ on positive integers as follows: it satisfies + +$$ +\begin{aligned} +w(a b) & =w(a)+w(b) \\ +w(2) & =1 ; \quad \text { and } \\ +w(p) & =w(p-1) \quad \text { for any prime } p>2 +\end{aligned} +$$ + +By induction, we see that $w(n)$ counts the powers of 2 that are produced as $\varphi$ is repeatedly applied to $n$. In particular, $k \geq w(n)$. + +From $w(2)=1$, it suffices to prove that $w(p) \geq \log _{3} p$ for every $p>2$. We use strong induction and note that + +$$ +w(p)=w(2)+w\left(\frac{p-1}{2}\right) \geq 1+\log _{3}(p-1)-\log _{3} 2 \geq \log _{3} p +$$ + +for any $p>2$. This solves the problem. +Remark. One can motivate this solution through small cases $2^{x} 3^{y}$ like $2^{x} 17^{w}, 2^{x} 3^{y} 7^{z}$, $2^{x} 11^{t}$ 。 + +Moreover, the stronger bound + +$$ +n \leq 2 \cdot 3^{k-1} +$$ + +is true and best possible. + +## §2.2 TSTST 2016/5, proposed by Linus Hamilton, Cynthia Stoner + +Available online at https://aops.com/community/p6580545. + +## Problem statement + +In the coordinate plane are finitely many walls, which are disjoint line segments, none of which are parallel to either axis. A bulldozer starts at an arbitrary point and moves in the $+x$ direction. Every time it hits a wall, it turns at a right angle to its path, away from the wall, and continues moving. (Thus the bulldozer always moves parallel to the axes.) +Prove that it is impossible for the bulldozer to hit both sides of every wall. + +We say a wall $v$ is above another wall $w$ if some point on $v$ is directly above a point on $w$. (This relation is anti-symmetric, as walls do not intersect). + +The critical claim is as follows: +Claim - There exists a lowest wall, i.e. a wall not above any other walls. +Proof. Assume not. Then we get a directed cycle of some length $n \geq 3$ : it's possible to construct a series of points $P_{i}, Q_{i}$, for $i=1, \ldots, n$ (indices modulo $n$ ), such that the point $Q_{i}$ is directly above $P_{i+1}$ for each $i$, the segment $\overline{Q_{i} P_{i+1}}$ does not intersect any wall in its interior, and finally each segment $\overline{P_{i} Q_{i}}$ is contained inside a wall. This gives us a broken line on $2 n$ vertices which is not self-intersecting. +Now consider the leftmost vertical segment $\overline{Q_{i} P_{i+1}}$ and the rightmost vertical segment $\overline{Q_{j} P_{j+1}}$. The broken line gives a path from $P_{i+1}$ to $Q_{j}$, as well as a path from $P_{j+1}$ to $Q_{i}$. These clearly must intersect, contradiction. + +Remark. This claim is Iran TST 2010. +Thus if the bulldozer eventually moves upwards indefinitely, it may never hit the bottom side of the lowest wall. Similarly, if the bulldozer eventually moves downwards indefinitely, it may never hit the upper side of the highest wall. + +## §2.3 TSTST 2016/6, proposed by Danielle Wang + +Available online at https://aops.com/community/p6580553. + +## Problem statement + +Let $A B C$ be a triangle with incenter $I$, and whose incircle is tangent to $\overline{B C}, \overline{C A}$, $\overline{A B}$ at $D, E, F$, respectively. Let $K$ be the foot of the altitude from $D$ to $\overline{E F}$. Suppose that the circumcircle of $\triangle A I B$ meets the incircle at two distinct points $C_{1}$ and $C_{2}$, while the circumcircle of $\triangle A I C$ meets the incircle at two distinct points $B_{1}$ and $B_{2}$. Prove that the radical axis of the circumcircles of $\triangle B B_{1} B_{2}$ and $\triangle C C_{1} C_{2}$ passes through the midpoint $M$ of $\overline{D K}$. +\I First solution (Allen Liu). Let $X, Y, Z$ be midpoints of $E F, F D, D E$, and let $G$ be the Gergonne point. By radical axis on $(A E I F),(D E F),(A I C)$ we see that $B_{1}$, $X, B_{2}$ are collinear. Likewise, $B_{1}, Z, B_{2}$ are collinear, so lines $B_{1} B_{2}$ and $X Z$ coincide. Similarly, lines $C_{1} C_{2}$ and $X Y$ coincide. In particular lines $B_{1} B_{2}$ and $C_{1} C_{2}$ meet at $X$. +![](https://cdn.mathpix.com/cropped/2024_11_19_f48de8bcaa95c5ff4531g-10.jpg?height=1212&width=1043&top_left_y=1150&top_left_x=512) + +Note $G$ is the symmedian point of $D E F$, so it is well-known that $X G$ passes through the midpoint of $D K$. So we just have to prove $G$ lies on the radical axis. + +First, note that $\triangle D E F$ is the cevian triangle of the Gergonne point $G$. Set $V=$ $\overline{X Y} \cap \overline{A B}, W=\overline{X Z} \cap \overline{A C}$, and $T=\overline{B W} \cap \overline{C V}$. + +We begin with the following completely projective claim. + +Claim - The points $X, G, T$ are collinear. +Proof. It suffices to view $\triangle X Y Z$ as any cevian triangle of $\triangle D E F$ (which is likewise any cevian triangle of $\triangle A B C)$. Then + +- By Cevian Nest on $\triangle A B C$, it follows that $\overline{A X}, \overline{B Y}, \overline{C Z}$ are concurrent. +- Hence $\triangle B Y V$ and $\triangle C Z W$ are perspective. +- Hence $\triangle B Z W$ and $\triangle C Y V$ are perspective too. +- Hence we deduce by Desargues theorem that $T, X$, and $\overline{B Z} \cap \overline{C Y}$ are collinear. +- Finally, the Cevian Nest theorem applied on $\triangle G B C$ (which has cevian triangles $\triangle D F E, \triangle X Z Y$ ) we deduce $G, X$, and $\overline{B Z} \cap \overline{C Y}$, proving the claim. + +One could also proceed by using barycentric coordinates on $\triangle D E F$. + +Remark (Eric Shen). The first four bullets can be replaced by non-projective means: one can check that $\overline{B Z} \cap \overline{C Y}$ is the radical center of $(B I C),\left(B B_{1} B_{2}\right),\left(C C_{1} C_{2}\right)$ and therefore it lies on line $\overline{X T}$. + +Now, we contend point $V$ is the radical center $\left(C C_{1} C_{2}\right),(A B C)$ and $(D E F)$. To see this, let $V^{\prime}=\overline{E D} \cap \overline{A B}$; then $\left(F V^{\prime} ; A B\right)$ is harmonic, and $V$ is the midpoint of $\overline{F V^{\prime}}$, and thus $V A \cdot V B=V F^{2}=V C_{1} \cdot V C_{2}$. + +So in fact $\overline{C V}$ is the radical axis of $(A B C)$ and $\left(C C_{1} C_{2}\right)$. +Similarly, $\overline{B W}$ is the radical axis of $(A B C)$ and $\left(B B_{1} B_{2}\right)$. Thus $T$ is the radical center of $(A B C),\left(B B_{1} B_{2}\right),\left(C C_{1} C_{2}\right)$. + +This completes the proof, as now $\overline{X T}$ is the desired radical axis. +『 Second solution (Evan Chen). As before, we just have to prove $G$ lies on the radical axis. +![](https://cdn.mathpix.com/cropped/2024_11_19_f48de8bcaa95c5ff4531g-12.jpg?height=1392&width=1260&top_left_y=249&top_left_x=404) + +Construct parallelograms $G P F Q, G R D S, G T U E$ such that $P, R \in D F, S, T \in D E$, $Q, U \in E F$. As $F G$ bisects $P Q$ and is isogonal to $F Z$, we find $P Q E D$, hence $P Q R U$, is cyclic. Repeating the same logic and noticing $P R, S T, Q U$ not concurrent, all six points $P Q R S T U$ are cyclic. Moreover, since $P Q$ bisects $G F$, we see that a dilation with factor 2 at $G$ sends $P Q$ to $P^{\prime}, Q^{\prime} \in A B$, say, with $F$ the midpoint of $P^{\prime} Q^{\prime}$. Define $R^{\prime}, S^{\prime} \in B C$ similarly now and $T^{\prime}, U^{\prime} \in C A$. + +Note that $E Q P D S^{\prime}$ is in cyclic too, as $\measuredangle D S^{\prime} Q=\measuredangle D R S=\measuredangle D E F$. By homothety through $B$, points $B, P, X$ are collinear; assume they meet ( $\left.E Q P D S^{\prime}\right)$ again at $V$. Thus $E V Q P D S^{\prime}$ is cyclic, and now + +$$ +\measuredangle B V S^{\prime}=\measuredangle P V S^{\prime}=\measuredangle P Q S=\measuredangle P T S=\measuredangle F E D=\measuredangle X E Z=\measuredangle X V Z +$$ + +hence $V$ lies on $\left(B Q^{\prime} S^{\prime}\right)$. +Since $F B \| Q P$, we get $E V F B$ is cyclic too, so $X V \cdot X B=X E \cdot X F$ now; thus $X$ lies on the radical axis of $\left(B S^{\prime} Q^{\prime}\right)$ and $(D E F)$. By the same argument with $W \in B Z$, we get $Z$ lies on the radical axis too. Thus the radical axis of $\left(B S^{\prime} Q^{\prime}\right)$ and ( $D E F$ ) must be line $X Z$, which coincides with $B_{1} B_{2}$; so $\left(B B_{1} B_{2}\right)=\left(B S^{\prime} Q^{\prime}\right)$. + +Analogously, $\left(C C_{1} C_{2}\right)=\left(C R^{\prime} U^{\prime}\right)$. Since $G=Q^{\prime} S^{\prime} \cap R^{\prime} U^{\prime}$, we need only prove that $Q^{\prime} R^{\prime} S^{\prime} U^{\prime}$ is cyclic. But $Q R S U$ is cyclic, so we are done. + +The circle ( $P Q R S T U$ ) is called the Lemoine circle of $A B C$. + diff --git a/USA_TST_TST/md/en-sols-TSTST-2017.md b/USA_TST_TST/md/en-sols-TSTST-2017.md new file mode 100644 index 0000000000000000000000000000000000000000..10b5ff3b2c4772f64ff410c3214ecf4ddaccdcce --- /dev/null +++ b/USA_TST_TST/md/en-sols-TSTST-2017.md @@ -0,0 +1,571 @@ +# USA TSTST 2017 Solutions
United States of America - TST Selection Test
Evan Chen《陳誼廷》
$59^{\text {th }}$ IMO 2018 Romania and $7^{\text {th }}$ EGMO 2018 Italy + +## Contents + +0 Problems ..... 2 +1 Solutions to Day 1 ..... 3 +1.1 TSTST 2017/1, proposed by Ray Li ..... 3 +1.2 TSTST 2017/2, proposed by Kevin Sun ..... 5 +1.3 TSTST 2017/3, proposed by Calvin Deng, Linus Hamilton ..... 7 +2 Solutions to Day 2 ..... 9 +2.1 TSTST 2017/4, proposed by Mark Sellke ..... 9 +2.2 TSTST 2017/5, proposed by Ray Li ..... 10 +2.3 TSTST 2017/6, proposed by Ivan Borsenco ..... 12 + +## §0 Problems + +1. Let $A B C$ be a triangle with circumcircle $\Gamma$, circumcenter $O$, and orthocenter $H$. Assume that $A B \neq A C$ and $\angle A \neq 90^{\circ}$. Let $M$ and $N$ be the midpoints of $\overline{A B}$ and $\overline{A C}$, respectively, and let $E$ and $F$ be the feet of the altitudes from $B$ and $C$ in $\triangle A B C$, respectively. Let $P$ be the intersection point of line $M N$ with the tangent line to $\Gamma$ at $A$. Let $Q$ be the intersection point, other than $A$, of $\Gamma$ with the circumcircle of $\triangle A E F$. Let $R$ be the intersection point of lines $A Q$ and $E F$. Prove that $\overline{P R} \perp \overline{O H}$. +2. Ana and Banana are playing a game. First Ana picks a word, which is defined to be a nonempty sequence of capital English letters. Then Banana picks a nonnegative integer $k$ and challenges Ana to supply a word with exactly $k$ subsequences which are equal to Ana's word. Ana wins if she is able to supply such a word, otherwise she loses. For example, if Ana picks the word "TST", and Banana chooses $k=4$, then Ana can supply the word "TSTST" which has 4 subsequences which are equal to Ana's word. Which words can Ana pick so that she can win no matter what value of $k$ Banana chooses? +3. Consider solutions to the equation + +$$ +x^{2}-c x+1=\frac{f(x)}{g(x)} +$$ + +where $f$ and $g$ are nonzero polynomials with nonnegative real coefficients. For each $c>0$, determine the minimum possible degree of $f$, or show that no such $f, g$ exist. +4. Find all nonnegative integer solutions to + +$$ +2^{a}+3^{b}+5^{c}=n! +$$ + +5. Let $A B C$ be a triangle with incenter $I$. Let $D$ be a point on side $B C$ and let $\omega_{B}$ and $\omega_{C}$ be the incircles of $\triangle A B D$ and $\triangle A C D$, respectively. Suppose that $\omega_{B}$ and $\omega_{C}$ are tangent to segment $B C$ at points $E$ and $F$, respectively. Let $P$ be the intersection of segment $A D$ with the line joining the centers of $\omega_{B}$ and $\omega_{C}$. Let $X$ be the intersection point of lines $B I$ and $C P$ and let $Y$ be the intersection point of lines $C I$ and $B P$. Prove that lines $E X$ and $F Y$ meet on the incircle of $\triangle A B C$. +6. A sequence of positive integers $\left(a_{n}\right)_{n \geq 1}$ is of Fibonacci type if it satisfies the recursive relation $a_{n+2}=a_{n+1}+a_{n}$ for all $n \geq 1$. Is it possible to partition the set of positive integers into an infinite number of Fibonacci type sequences? + +## §1 Solutions to Day 1 + +## §1.1 TSTST 2017/1, proposed by Ray Li + +Available online at https://aops.com/community/p8526098. + +## Problem statement + +Let $A B C$ be a triangle with circumcircle $\Gamma$, circumcenter $O$, and orthocenter $H$. Assume that $A B \neq A C$ and $\angle A \neq 90^{\circ}$. Let $M$ and $N$ be the midpoints of $\overline{A B}$ and $\overline{A C}$, respectively, and let $E$ and $F$ be the feet of the altitudes from $B$ and $C$ in $\triangle A B C$, respectively. Let $P$ be the intersection point of line $M N$ with the tangent line to $\Gamma$ at $A$. Let $Q$ be the intersection point, other than $A$, of $\Gamma$ with the circumcircle of $\triangle A E F$. Let $R$ be the intersection point of lines $A Q$ and $E F$. Prove that $\overline{P R} \perp \overline{O H}$. + +【 First solution (power of a point). Let $\gamma$ denote the nine-point circle of $A B C$. +![](https://cdn.mathpix.com/cropped/2024_11_19_b5f71e11be2d66f7b920g-03.jpg?height=609&width=795&top_left_y=1140&top_left_x=639) + +Note that + +- $P A^{2}=P M \cdot P N$, so $P$ lies on the radical axis of $\Gamma$ and $\gamma$. +- $R A \cdot R Q=R E \cdot R F$, so $R$ lies on the radical axis of $\Gamma$ and $\gamma$. + +Thus $\overline{P R}$ is the radical axis of $\Gamma$ and $\gamma$, which is evidently perpendicular to $\overline{O H}$. +Remark. In fact, by power of a point one may also observe that $R$ lies on $\overline{B C}$, since it is on the radical axis of $(A Q F H E),(B F E C),(A B C)$. Ironically, this fact is not used in the solution. + +II Second solution (barycentric coordinates). Again note first $R \in \overline{B C}$ (although this can be avoided too). We compute the points in much the same way as before. Since $\overline{A P} \cap \overline{B C}=\left(0: b^{2}:-c^{2}\right)$ we have + +$$ +P=\left(b^{2}-c^{2}: b^{2}:-c^{2}\right) +$$ + +(since $x=y+z$ is the equation of line $\overline{M N}$ ). Now in Conway notation we have + +$$ +R=\overline{E F} \cap \overline{B C}=\left(0: S_{C}:-S_{B}\right)=\left(0: a^{2}+b^{2}-c^{2}:-a^{2}+b^{2}-c^{2}\right) . +$$ + +Hence + +$$ +\overrightarrow{P R}=\frac{1}{2\left(b^{2}-c^{2}\right)}\left(b^{2}-c^{2}, c^{2}-a^{2}, a^{2}-b^{2}\right) +$$ + +On the other hand, we have $\overrightarrow{O H}=\vec{A}+\vec{B}+\vec{C}$. So it suffices to check that + +$$ +\sum_{\mathrm{cyc}} a^{2}\left(\left(a^{2}-b^{2}\right)+\left(c^{2}-a^{2}\right)\right)=0 +$$ + +which is immediate. + +『 Third solution (complex numbers). Let $A B C$ be the unit circle. We first compute $P$ as the midpoint of $A$ and $\overline{A A} \cap \overline{B C}$ : + +$$ +\begin{aligned} +p & =\frac{1}{2}\left(a+\frac{a^{2}(b+c)-b c \cdot 2 a}{a^{2}-b c}\right) \\ +& =\frac{a\left(a^{2}-b c\right)+a^{2}(b+c)-2 a b c}{2\left(a^{2}-b c\right)} +\end{aligned} +$$ + +Using the remark above, $R$ is the inverse of $D$ with respect to the circle with diameter $\overline{B C}$, which has radius $\left|\frac{1}{2}(b-c)\right|$. Thus + +$$ +\begin{aligned} +r-\frac{b+c}{2} & =\frac{\frac{1}{4}(b-c)\left(\frac{1}{b}-\frac{1}{c}\right)}{\frac{1}{2}\left(a-\frac{b c}{a}\right)} \\ +r & =\frac{b+c}{2}+\frac{-\frac{1}{2} \frac{(b-c)^{2}}{b c}}{\frac{1}{a}-\frac{a}{b c}} \\ +& =\frac{b+c}{2}+\frac{a(b-c)^{2}}{2\left(a^{2}-b c\right)} \\ +& =\frac{a(b-c)^{2}+(b+c)\left(a^{2}-b c\right)}{2\left(a^{2}-b c\right)} +\end{aligned} +$$ + +Expanding and subtracting gives + +$$ +p-r=\frac{a^{3}-a b c-a b^{2}-a c^{2}+b^{2} c+b c^{2}}{2\left(a^{2}-b c\right)}=\frac{(a+b+c)(a-b)(a-c)}{2\left(a^{2}-b c\right)} +$$ + +which is visibly equal to the negation of its conjugate once the factor of $a+b+c$ is deleted. +(Actually, one can guess this factorization ahead of time by noting that if $A=B$, then $P=B=R$, so $a-b$ must be a factor; analogously $a-c$ must be as well.) + +## §1.2 TSTST 2017/2, proposed by Kevin Sun + +Available online at https://aops.com/community/p8526115. + +## Problem statement + +Ana and Banana are playing a game. First Ana picks a word, which is defined to be a nonempty sequence of capital English letters. Then Banana picks a nonnegative integer $k$ and challenges Ana to supply a word with exactly $k$ subsequences which are equal to Ana's word. Ana wins if she is able to supply such a word, otherwise she loses. For example, if Ana picks the word "TST", and Banana chooses $k=4$, then Ana can supply the word "TSTST" which has 4 subsequences which are equal to Ana's word. Which words can Ana pick so that she can win no matter what value of $k$ Banana chooses? + +First we introduce some notation. Define a block of letters to be a maximal contiguous subsequence of consecutive letters. For example, the word $A A B B B C A A A$ has four blocks, namely $A A, B B B, C, A A A$. Throughout the solution, we fix the word $A$ that Ana picks, and introduce the following notation for its $m$ blocks: + +$$ +A=A_{1} A_{2} \ldots A_{m}=\underbrace{a_{1} \ldots a_{1}}_{x_{1}} \underbrace{a_{2} \ldots a_{2}}_{x_{2}} \cdots \underbrace{a_{m} \ldots a_{m}}_{x_{m}} +$$ + +A rainbow will be a subsequence equal to Ana's initial word $A$ (meaning Ana seeks words with exactly $k$ rainbows). Finally, for brevity, let $A_{i}=\underbrace{a_{i} \ldots a_{i}}_{x_{i}}$, so $A=A_{1} \ldots A_{m}$. + +We prove two claims that resolve the problem. +Claim - If $x_{i}=1$ for some $i$, then for any $k \geq 1$, the word + +$$ +W=A_{1} \ldots A_{i-1} \underbrace{a_{i} \ldots a_{i}}_{k} A_{i+1} \ldots A_{m} +$$ + +obtained by repeating the $i$ th letter $k$ times has exactly $k$ rainbows. +Proof. Obviously there are at least $\binom{k}{k-1}=k$ rainbows, obtained by deleting $k-1$ choices of the letter $a_{i}$ in the repeated block. We show they are the only ones. + +Given a rainbow, consider the location of this singleton block in $W$. It cannot occur within the first $\left|A_{1}\right|+\cdots+\left|A_{i-1}\right|$ letters, nor can it occur within the final $\left|A_{i+1}\right|+\cdots+\left|A_{m}\right|$ letters. So it must appear in the $i$ th block of $W$. That implies that all the other $a_{i}$ 's in the $i$ th block of $W$ must be deleted, as desired. (This last argument is actually nontrivial, and has some substance; many students failed to realize that the upper bound requires care.) + +Claim - If $x_{i} \geq 2$ for all $i$, then no word $W$ has exactly two rainbows. +Proof. We prove if there are two rainbows of $W$, then we can construct at least three rainbows. + +Let $W=w_{1} \ldots w_{n}$ and consider the two rainbows of $W$. Since they are not the same, there must be a block $A_{p}$ of the rainbow, of length $\ell \geq 2$, which do not occupy the same locations in $W$. + +Assume the first rainbow uses $w_{i_{1}}, \ldots, w_{i_{\ell}}$ for this block and the second rainbow uses $w_{j_{1}}, \ldots, w_{j_{\ell}}$ for this block. Then among the letters $w_{q}$ for $\min \left(i_{1}, j_{1}\right) \leq q \leq \max \left(i_{\ell}, j_{\ell}\right)$, there must be at least $\ell+1$ copies of the letter $a_{p}$. Moreover, given a choice of $\ell$ copies of the letter $a_{p}$ in this range, one can complete the subsequence to a rainbow. So the number of rainbows is at least $\binom{\ell+1}{\ell} \geq \ell+1$. + +Since $\ell \geq 2$, this proves $W$ has at least three rainbows. +In summary, Ana wins if and only if $x_{i}=1$ for some $i$, since she can duplicate the isolated letter $k$ times; but if $x_{i} \geq 2$ for all $i$ then Banana only needs to supply $k=2$. + +## §1.3 TSTST 2017/3, proposed by Calvin Deng, Linus Hamilton + +Available online at https://aops.com/community/p8526130. + +## Problem statement + +Consider solutions to the equation + +$$ +x^{2}-c x+1=\frac{f(x)}{g(x)} +$$ + +where $f$ and $g$ are nonzero polynomials with nonnegative real coefficients. For each $c>0$, determine the minimum possible degree of $f$, or show that no such $f, g$ exist. + +First, if $c \geq 2$ then we claim no such $f$ and $g$ exist. Indeed, one simply takes $x=1$ to get $f(1) / g(1) \leq 0$, impossible. + +For $c<2$, let $c=2 \cos \theta$, where $0<\theta<\pi$. We claim that $f$ exists and has minimum degree equal to $n$, where $n$ is defined as the smallest integer satisfying $\sin n \theta \leq 0$. In other words + +$$ +n=\left\lceil\frac{\pi}{\arccos (c / 2)}\right\rceil +$$ + +First we show that this is necessary. To see it, write explicitly + +$$ +g(x)=a_{0}+a_{1} x+a_{2} x^{2}+\cdots+a_{n-2} x^{n-2} +$$ + +with each $a_{i} \geq 0$, and $a_{n-2} \neq 0$. Assume that $n$ is such that $\sin (k \theta) \geq 0$ for $k=1, \ldots, n-1$. Then, we have the following system of inequalities: + +$$ +\begin{aligned} +& a_{1} \geq 2 \cos \theta \cdot a_{0} \\ +& a_{0}+a_{2} \geq 2 \cos \theta \cdot a_{1} \\ +& a_{1}+a_{3} \geq 2 \cos \theta \cdot a_{2} \\ +& \vdots \\ +& a_{n-5}+a_{n-3} \geq 2 \cos \theta \cdot a_{n-4} \\ +& a_{n-4}+a_{n-2} \geq 2 \cos \theta \cdot a_{n-3} \\ +& a_{n-3} \geq 2 \cos \theta \cdot a_{n-2} . +\end{aligned} +$$ + +Now, multiply the first equation by $\sin \theta$, the second equation by $\sin 2 \theta$, et cetera, up to $\sin ((n-1) \theta)$. This choice of weights is selected since we have + +$$ +\sin (k \theta)+\sin ((k+2) \theta)=2 \sin ((k+1) \theta) \cos \theta +$$ + +so that summing the entire expression cancels nearly all terms and leaves only + +$$ +\sin ((n-2) \theta) a_{n-2} \geq \sin ((n-1) \theta) \cdot 2 \cos \theta \cdot a_{n-2} +$$ + +and so by dividing by $a_{n-2}$ and using the same identity gives us $\sin (n \theta) \leq 0$, as claimed. +This bound is best possible, because the example + +$$ +a_{k}=\sin ((k+1) \theta) \geq 0 +$$ + +makes all inequalities above sharp, hence giving a working pair $(f, g)$. + +Remark. Calvin Deng points out that a cleaner proof of the lower bound is to take $\alpha=\cos \theta+i \sin \theta$. Then $f(\alpha)=0$, but by condition the imaginary part of $f(\alpha)$ is apparently strictly positive, contradiction. + +Remark. Guessing that $c<2$ works at all (and realizing $c \geq 2$ fails) is the first part of the problem. + +The introduction of trigonometry into the solution may seem magical, but is motivated in one of two ways: + +- Calvin Deng points out that it's possible to guess the answer from small cases: For $c \leq 1$ we have $n=3$, tight at $\frac{x^{3}+1}{x+1}=x^{2}-x+1$, and essentially the "sharpest $n=3$ example". A similar example exists at $n=4$ with $\frac{x^{4}+1}{x^{2}+\sqrt{2} x+1}=x^{2}-\sqrt{2} x+1$ by the Sophie-Germain identity. In general, one can do long division to extract an optimal value of $c$ for any given $n$, although $c$ will be the root of some polynomial. +The thresholds $c \leq 1$ for $n=3, c \leq \sqrt{2}$ for $n=4, c \leq \frac{1+\sqrt{5}}{2}$ for $n=5$, and $c \leq 2$ for $n<\infty$ suggest the unusual form of the answer via trigonometry. +- One may imagine trying to construct a polynomial recursively / greedily by making all inequalities above hold (again the "sharpest situation" in which $f$ has few coefficients). If one sets $c=2 t$, then we have + +$$ +a_{0}=1, \quad a_{1}=2 t, \quad a_{2}=4 t^{2}-1, \quad a_{3}=8 t^{3}-4 t, \quad \ldots +$$ + +which are the Chebyshev polynomials of the second type. This means that trigonometry is essentially mandatory. (One may also run into this when by using standard linear recursion techniques, and noting that the characteristic polynomial has two conjugate complex roots.) + +Remark. Mitchell Lee notes that an IMO longlist problem from 1997 shows that if $P(x)$ is any polynomial satisfying $P(x)>0$ for $x>0$, then $(x+1)^{n} P(x)$ has nonnegative coefficients for large enough $n$. This show that $f$ and $g$ at least exist for $c \leq 2$, but provides no way of finding the best possible $\operatorname{deg} f$. + +Meghal Gupta also points out that showing $f$ and $g$ exist is possible in the following way: + +$$ +\left(x^{2}-1.99 x+1\right)\left(x^{2}+1.99 x+1\right)=\left(x^{4}-1.9601 x^{2}+1\right) +$$ + +and so on, repeatedly multiplying by the "conjugate" until all coefficients become positive. To my best knowledge, this also does not give any way of actually minimizing $\operatorname{deg} f$, although Ankan Bhattacharya points out that this construction is actually optimal in the case where $n$ is a power of 2 . + +Remark. It's pointed out that Matematicheskoe Prosveshchenie, issue 1, 1997, page 194 contains a nearly analogous result, available at https://mccme.ru/free-books/matpros/ pdf/mp-01.pdf with solutions presented in https://mccme.ru/free-books/matpros/pdf/ mp-05.pdf, pages 221-223; and https://mccme.ru/free-books/matpros/pdf/mp-10.pdf, page 274. + +## §2 Solutions to Day 2 + +## §2.1 TSTST 2017/4, proposed by Mark Sellke + +Available online at https://aops.com/community/p8526131. + +## Problem statement + +Find all nonnegative integer solutions to + +$$ +2^{a}+3^{b}+5^{c}=n! +$$ + +For $n \leq 4$, one can check the only solutions are: + +$$ +\begin{aligned} +& 2^{2}+3^{0}+5^{0}=3! \\ +& 2^{1}+3^{1}+5^{0}=3! \\ +& 2^{4}+3^{1}+5^{1}=4! +\end{aligned} +$$ + +Now we prove there are no solutions for $n \geq 5$. +A tricky way to do this is to take modulo 120 , since + +$$ +\begin{aligned} +2^{a} \quad(\bmod 120) & \in\{1,2,4,8,16,32,64\} \\ +3^{b} \quad(\bmod 120) & \in\{1,3,9,27,81\} \\ +5^{c} \quad(\bmod 120) & \in\{1,5,25\} +\end{aligned} +$$ + +and by inspection one notes that no three elements have vanishing sum modulo 120 . +I expect most solutions to instead use casework. Here is one possible approach with cases (with $n \geq 5$ ). First, we analyze the cases where $a<3$ : + +- $a=0$ : No solutions for parity reasons. +- $a=1$ : since $3^{b}+5^{c} \equiv 6(\bmod 8)$, we find $b$ even and $c$ odd (hence $\left.c \neq 0\right)$. Now looking modulo 5 gives that $3^{b}+5^{c} \equiv 3(\bmod 5)$, +- $a=2$ : From $3^{b}+5^{c} \equiv 4(\bmod 8)$, we find $b$ is odd and $c$ is even. Now looking modulo 5 gives a contradiction, even if $c=0$, since $3^{b} \in\{2,3(\bmod 5)\}$ but $3^{b}+5^{c} \equiv 1(\bmod 5)$. + +Henceforth assume $a \geq 3$. Next, by taking modulo 8 we have $3^{b}+5^{c} \equiv 0(\bmod 8)$, which forces both $b$ and $c$ to be odd (in particular, $b, c>0$ ). We now have + +$$ +\begin{aligned} +& 2^{a}+5^{c} \equiv 0 \quad(\bmod 3) \\ +& 2^{a}+3^{b} \equiv 0 \quad(\bmod 5) . +\end{aligned} +$$ + +The first equation implies $a$ is even, but the second equation requires $a$ to be odd, contradiction. Hence no solutions with $n \geq 5$. + +## §2.2 TSTST 2017/5, proposed by Ray Li + +Available online at https://aops.com/community/p8526136. + +## Problem statement + +Let $A B C$ be a triangle with incenter $I$. Let $D$ be a point on side $B C$ and let $\omega_{B}$ and $\omega_{C}$ be the incircles of $\triangle A B D$ and $\triangle A C D$, respectively. Suppose that $\omega_{B}$ and $\omega_{C}$ are tangent to segment $B C$ at points $E$ and $F$, respectively. Let $P$ be the intersection of segment $A D$ with the line joining the centers of $\omega_{B}$ and $\omega_{C}$. Let $X$ be the intersection point of lines $B I$ and $C P$ and let $Y$ be the intersection point of lines $C I$ and $B P$. Prove that lines $E X$ and $F Y$ meet on the incircle of $\triangle A B C$. + +『 First solution (homothety). Let $Z$ be the diametrically opposite point on the incircle. We claim this is the desired intersection. +![](https://cdn.mathpix.com/cropped/2024_11_19_b5f71e11be2d66f7b920g-10.jpg?height=586&width=809&top_left_y=1055&top_left_x=629) + +Note that: + +- $P$ is the insimilicenter of $\omega_{B}$ and $\omega_{C}$ +- $C$ is the exsimilicenter of $\omega$ and $\omega_{C}$. + +Thus by Monge theorem, the insimilicenter of $\omega_{B}$ and $\omega$ lies on line $C P$. +This insimilicenter should also lie on the line joining the centers of $\omega$ and $\omega_{B}$, which is $\overline{B I}$, hence it coincides with the point $X$. So $X \in \overline{E Z}$ as desired. + +【 Second solution (harmonic). Let $T=\overline{I_{B} I_{C}} \cap \overline{B C}$, and $W$ the foot from $I$ to $\overline{B C}$. Define $Z=\overline{F Y} \cap \overline{I W}$. Because $\angle I_{B} D I_{C}=90^{\circ}$, we have + +$$ +-1=\left(I_{B} I_{C} ; P T\right) \stackrel{B}{\stackrel{B}{2}}\left(I I_{C} ; Y C\right) \stackrel{F}{=}(I \infty ; Z W) +$$ + +So $I$ is the midpoint of $\overline{Z W}$ as desired. +【 Third solution (outline, barycentric, Andrew Gu). Let $A D=t, B D=x, C D=y$, so $a=x+y$ and by Stewart's theorem we have + +$$ +(x+y)\left(x y+t^{2}\right)=b^{2} x+c^{2} y +$$ + +We then have $D=(0: y: x)$ and so + +$$ +\overline{A I_{B}} \cap \overline{B C}=\left(0: y+\frac{t x}{c+t}: \frac{c x}{c+t}\right) +$$ + +hence intersection with $B I$ gives + +$$ +I_{B}=(a x: c y+a t: c x) +$$ + +Similarly, + +$$ +I_{C}=(a y: b y: b x+a t) +$$ + +Then, we can compute + +$$ +P=(2 a x y: y(a t+b x+c y): x(a t+b x+c y)) +$$ + +since $P \in \overline{I_{B} I_{C}}$, and clearly $P \in \overline{A D}$. Intersection now gives + +$$ +\begin{aligned} +& X=(2 a x: a t+b x+c y: 2 c x) \\ +& Y=(2 a y: 2 b y: a t+b x+c y) +\end{aligned} +$$ + +Finally, we have $B E=\frac{1}{2}(c+x-t)$, and similarly for $C F$. Now if we reflect $D=$ $\left(0, \frac{s-c}{a}, \frac{s-b}{a}\right)$ over $I=\left(\frac{a}{2 s}, \frac{b}{2 s}, \frac{c}{2 s}\right)$, we get the antipode + +$$ +Q:=\left(4 a^{2}:-a^{2}+2 a b-b^{2}+c^{2}:-a^{2}+2 a c-c^{2}+b^{2}\right) . +$$ + +We may then check $Q$ lies on each of lines $E X$ and $F Y$ (by checking $\operatorname{det}(Q, E, X)=0$ using the equation (1)). + +## §2.3 TSTST 2017/6, proposed by Ivan Borsenco + +Available online at https://aops.com/community/p8526142. + +## Problem statement + +A sequence of positive integers $\left(a_{n}\right)_{n \geq 1}$ is of Fibonacci type if it satisfies the recursive relation $a_{n+2}=a_{n+1}+a_{n}$ for all $n \geq 1$. Is it possible to partition the set of positive integers into an infinite number of Fibonacci type sequences? + +Yes, it is possible. The following solutions were written for me by Kevin Sun and Mark Sellke. We let $F_{1}=F_{2}=1, F_{3}=2, F_{4}=3, F_{5}=5, \ldots$ denote the Fibonacci numbers. + +【 First solution (Kevin Sun). We are going to appeal to the so-called Zeckendorf theorem: + +Theorem (Zeckendorf) +Every positive integer can be uniquely expressed as the sum of nonconsecutive Fibonacci numbers. + +This means every positive integer has a Zeckendorf ("Fibonacci-binary") representation where we put 1 in the $i$ th digit from the right if $F_{i+1}$ is used. The idea is then to take the following so-called Wythoff array: + +- Row 1: 1, 2, 3, 5, ... +- Row 101: $1+3,2+5,3+8, \ldots$ +- Row 1001: $1+5,2+8,3+13, \ldots$ +- Row 10001: $1+8,2+13,3+21, \ldots$ +- Row 10101: $1+3+8,2+5+13,3+8+21, \ldots$ +- . . .et cetera. + +More concretely, the array has the following rows to start: + +| 1 | 2 | 3 | 5 | 8 | 13 | 21 | $\ldots$ | +| :---: | :---: | :---: | :---: | :---: | :---: | :---: | :---: | +| 4 | 7 | 11 | 18 | 29 | 47 | 76 | $\ldots$ | +| 6 | 10 | 16 | 26 | 42 | 68 | 110 | $\ldots$ | +| 9 | 15 | 24 | 39 | 63 | 102 | 165 | $\ldots$ | +| 12 | 20 | 32 | 52 | 84 | 136 | 220 | $\ldots$ | +| 14 | 23 | 37 | 60 | 97 | 157 | 254 | $\ldots$ | +| 17 | 28 | 45 | 73 | 118 | 191 | 309 | $\ldots$ | +| $\vdots$ | $\vdots$ | $\vdots$ | $\vdots$ | $\vdots$ | $\vdots$ | $\vdots$ | $\ddots$ | + +Here are the full details. +We begin by outlining a proof of Zeckendorf's theorem, which implies the representation above is unique. Note that if $F_{k}$ is the greatest Fibonacci number at most $n$, then + +$$ +n-F_{k}F_{k-1}+F_{k-3}+F_{k-5}+\cdots +$$ + +This shows, by a simple inductive argument, that such a representation exists and unique. +We write $n={\bar{a} k \ldots a_{1}}_{\text {Fib }}$ for the Zeckendorf representation as we described (where $a_{i}=1$ if $F_{i+1}$ is used). Now for each ${\bar{a} k \ldots a_{1}}^{\text {Fib }}$ with $a_{1}=1$, consider the sequence + +$$ +{\overline{a_{k} \ldots a_{1}}}_{\mathrm{Fib}},{\overline{a_{k} \ldots a_{1} 0}}_{\mathrm{Fib}},{\overline{a_{k} \ldots a_{1} 00}}_{\mathrm{Fib}}, \ldots +$$ + +These sequences are Fibonacci-type by definition, and partition the positive integers since each positive integer has exactly one Fibonacci base representation. +\I Second solution. Call an infinite set of integers $S$ sandwiched if there exist increasing sequences $\left\{a_{i}\right\}_{i=0}^{\infty},\left\{b_{i}\right\}_{i=0}^{\infty}$ such that the following are true: + +- $a_{i}+a_{i+1}=a_{i+2}$ and $b_{i}+b_{i+1}=b_{i+2}$. +- The intervals $\left[a_{i}+1, b_{i}-1\right]$ are disjoint and are nondecreasing in length. +- $S=\bigcup_{i=0}^{\infty}\left[a_{i}+1, b_{i}-1\right]$. + +We claim that if $S$ is any nonempty sandwiched set, then $S$ can be partitioned into a Fibonacci-type sequence (involving the smallest element of $S$ ) and two smaller sandwiched sets. If this claim is proven, then we can start with $\mathbb{N} \backslash\{1,2,3,5, \ldots\}$, which is a sandwiched set, and repeatedly perform this partition, which will eventually sort each natural number into a Fibonacci-type sequence. + +Let $S$ be a sandwiched set given by $\left\{a_{i}\right\}_{i=0}^{\infty},\left\{b_{i}\right\}_{i=0}^{\infty}$, so the smallest element in $S$ is $x=a_{0}+1$. Note that $y=a_{1}+1$ is also in $S$ and $x1$. Then + +$$ +a \neq b \Longrightarrow\left\lfloor\varphi a+\frac{1}{2}\right\rfloor \neq\left\lfloor\varphi b+\frac{1}{2}\right\rfloor, +$$ + +and since the first term of each sequence is chosen to not overlap with any previous sequences, these sequences are disjoint. + +Remark. Ankan Bhattacharya points out that the same sequence essentially appears in IMO 1993, Problem 5 - in other words, a strictly increasing function $f: \mathbb{Z}_{>0} \rightarrow \mathbb{Z}_{>0}$ with $f(1)=2$, and $f(f(n))=f(n)+n$. + +Nikolai Beluhov sent us an older reference from March 1977, where Martin Gardner wrote in his column about Wythoff's Nim. The relevant excerpt goes: +"Imagine that we go through the infinite sequence of safe pairs (in the manner of Eratosthenes' sieve for sifting out primes) and cross out the infinite set of all safe pairs that are pairs in the Fibonacci sequence. The smallest pair that is not crossed out is $4 / 7$. We can now cross out a second infinite set of safe pairs, starting with $4 / 7$, that are pairs in the Lucas sequence. An infinite number of safe pairs, of which the lowest is now $6 / 10$, remain. This pair too begins another infinite Fibonacci sequence, all of whose pairs are safe. The process continues forever. Robert Silber, a mathematician at North Carolina State University, calls a safe pair "primitive" if it is the first safe pair that generates a Fibonacci sequence." + +The relevant article by Robert Silber is A Fibonacci Property of Wythoff Pairs, from The Fibonacci Quarterly 11/1976. + +I Fourth solution (Mark Sellke). For later reference let + +$$ +f_{1}=0, f_{2}=1, f_{3}=1, \ldots +$$ + +denote the ordinary Fibonacci numbers. We will denote the Fibonacci-like sequences by $F^{i}$ and the elements with subscripts; hence $F_{1}^{2}$ is the first element of the second sequence. Our construction amounts to just iteratively add new sequences; hence the following claim is the whole problem. + +## Lemma + +For any disjoint collection of Fibonacci-like sequences $F^{1}, \ldots, F^{k}$ and any integer $m$ contained in none of them, there is a new Fibonacci-like sequence $F^{k+1}$ beginning with $F_{1}^{k+1}=m$ which is disjoint from the previous sequences. + +Observe first that for each sequence $F^{j}$ there is $c^{j} \in \mathbb{R}^{n}$ such that + +$$ +F_{n}^{j}=c^{j} \phi^{n}+o(1) +$$ + +where + +$$ +\phi=\frac{1+\sqrt{5}}{2} +$$ + +Collapse the group $\left(\mathbb{R}^{+}, \times\right)$into the half-open interval $J=\{x \mid 1 \leq x<\phi\}$ by defining $T(x)=y$ for the unique $y \in J$ with $x=y \phi^{n}$ for some integer $n$. + +Fix an interval $I=[a, b] \subseteq[1.2,1.3]$ (the last condition is to avoid wrap-around issues) which contains none of the $c^{j}$, and take $\varepsilon<0.001$ to be small enough that in fact each $c^{j}$ has distance at least $10 \varepsilon$ from $I$; this means any $c_{j}$ and element of $I$ differ by at least a $(1+10 \varepsilon)$ factor. The idea will be to take $F_{1}^{k+1}=m$ and $F_{2}^{k+1}$ to be a large such that the induced values of $F_{j}^{k+1}$ grow like $k \phi^{j}$ for $j \in T^{-1}(I)$, so that $F_{n}^{k+1}$ is separated from the $c^{j}$ after applying $T$. What's left to check is the convergence. + +Now let + +$$ +c=\lim _{n \rightarrow \infty} \frac{f_{n}}{\phi^{n}} +$$ + +and take $M$ large enough that for $n>M$ we have + +$$ +\left|\frac{f_{n}}{c \phi^{n}}-1\right|<\varepsilon +$$ + +Now $\frac{T^{-1}(I)}{c}$ contains arbitrarily large integers, so there are infinitely many $N$ with $c N \in T^{-1}(I)$ with $N>\frac{10 m}{\varepsilon}$. We claim that for any such $N$, the sequence $F^{(N)}$ defined by + +$$ +F_{1}^{(N)}=m, F_{2}^{(N)}=N +$$ + +will be very multiplicatively similar to the normal Fibonacci numbers up to rescaling; indeed for $j=2, j=3$ we have $\frac{F_{2}^{(N)}}{f_{2}}=N, \frac{F_{3}^{(N)}}{f_{3}}=N+m$ and so by induction we will have + +$$ +\frac{F_{j}^{(N)}}{f_{j}} \in[N, N+m] \subseteq[N, N(1+\varepsilon)] +$$ + +for $j \geq 2$. Therefore, up to small multiplicative errors, we have + +$$ +F_{j}^{(N)} \approx N f_{j} \approx c N \phi^{j} +$$ + +From this we see that for $j>M$ we have + +$$ +T\left(F_{j}^{(N)}\right) \in T(c N) \cdot[1-2 \varepsilon, 1+2 \varepsilon] +$$ + +In particular, since $T(c N) \in I$ and $I$ is separated from each $c_{j}$ by a factor of $(1+10 \varepsilon)$, we get that $F_{j}^{(N)}$ is not in any of $F^{1}, F^{2}, \ldots, F^{k}$. + +Finishing is easy, since we now have a uniform estimate on how many terms we need to check for a new element before the exponential growth takes over. We will just use pigeonhole to argue that there are few possible collisions among those early terms, so we can easily pick a value of $N$ which avoids them all. We write it out below. + +For large $L$, the set + +$$ +S_{L}=\left(I \cdot \phi^{L}\right) \cap \mathbb{Z} +$$ + +contains at least $k_{I} \phi^{L}$ elements. As $N$ ranges over $S_{L}$, for each fixed $j$, the value of $F_{j}^{(N)}$ varies by at most a factor of 1.1 because we imposed $I \subseteq[1.2,1.3]$ and so this is true for the first two terms, hence for all subsequent terms by induction. Now suppose $L$ is very large, and consider a fixed pair $(i, j)$ with $i \leq k$ and $j \leq M$. We claim there is at most 1 possible value $k$ such that the term $F_{k}^{i}$ could equal $F_{j}^{(N)}$ for some $N \in S_{L}$; indeed, the terms of $F^{i}$ are growing at exponential rate with factor $\phi>1.1$, so at most one will be in a given interval of multiplicative width at most 1.1. + +Hence, of these $k_{I} \phi^{L}$ values of $N$, at most $k M$ could cause problems, one for each pair $(i, j)$. However by monotonicity of $F_{j}^{(N)}$ in $N$, at most 1 value of $N$ causes a collision for each pair $(i, j)$. Hence for large $L$ so that $k_{I} \phi^{L}>10 k M$ we can find a suitable $N \in S_{L}$ by pigeonhole and the sequence $F^{(N)}$ defined by $(m, N, N+m, \ldots)$ works. + diff --git a/USA_TST_TST/md/en-sols-TSTST-2018.md b/USA_TST_TST/md/en-sols-TSTST-2018.md new file mode 100644 index 0000000000000000000000000000000000000000..1c68fb9902ff3b9d3b529b98496b2ec6541c4d43 --- /dev/null +++ b/USA_TST_TST/md/en-sols-TSTST-2018.md @@ -0,0 +1,678 @@ +# USA TSTST 2018 Solutions
United States of America - TST Selection Test
Evan Chen《陳誼廷》
$60^{\text {th }}$ IMO 2019 United Kingdom and $8^{\text {th }}$ EGMO 2019 Ukraine + +## Contents + +0 Problems ..... 2 +1 Solutions to Day 1 ..... 4 +1.1 TSTST 2018/1, proposed by Evan Chen, Yang Liu ..... 4 +1.2 TSTST 2018/2, proposed by Victor Wang ..... 6 +1.3 TSTST 2018/3, proposed by Yannick Yao, Evan Chen ..... 8 +2 Solutions to Day 2 ..... 11 +2.1 TSTST 2018/4, proposed by Ivan Borsenco ..... 11 +2.2 TSTST 2018/5, proposed by Ankan Bhattacharya, Evan Chen ..... 13 +2.3 TSTST 2018/6, proposed by Ray Li ..... 17 +3 Solutions to Day 3 ..... 20 +3.1 TSTST 2018/7, proposed by Ashwin Sah ..... 20 +3.2 TSTST 2018/8, proposed by Ankan Bhattacharya, Evan Chen ..... 22 +3.3 TSTST 2018/9, proposed by Linus Hamilton ..... 24 + +## §0 Problems + +1. As usual, let $\mathbb{Z}[x]$ denote the set of single-variable polynomials in $x$ with integer coefficients. Find all functions $\theta: \mathbb{Z}[x] \rightarrow \mathbb{Z}$ such that for any polynomials $p, q \in$ $\mathbb{Z}[x]$, + +- $\theta(p+1)=\theta(p)+1$, and +- if $\theta(p) \neq 0$ then $\theta(p)$ divides $\theta(p \cdot q)$. + +2. In the nation of Onewaynia, certain pairs of cities are connected by one-way roads. Every road connects exactly two cities (roads are allowed to cross each other, e.g., via bridges), and each pair of cities has at most one road between them. Moreover, every city has exactly two roads leaving it and exactly two roads entering it. +We wish to close half the roads of Onewaynia in such a way that every city has exactly one road leaving it and exactly one road entering it. Show that the number of ways to do so is a power of 2 greater than 1 (i.e. of the form $2^{n}$ for some integer $n \geq 1$ ). +3. Let $A B C$ be an acute triangle with incenter $I$, circumcenter $O$, and circumcircle $\Gamma$. Let $M$ be the midpoint of $\overline{A B}$. Ray $A I$ meets $\overline{B C}$ at $D$. Denote by $\omega$ and $\gamma$ the circumcircles of $\triangle B I C$ and $\triangle B A D$, respectively. Line $M O$ meets $\omega$ at $X$ and $Y$, while line $C O$ meets $\omega$ at $C$ and $Q$. Assume that $Q$ lies inside $\triangle A B C$ and $\angle A Q M=\angle A C B$. + +Consider the tangents to $\omega$ at $X$ and $Y$ and the tangents to $\gamma$ at $A$ and $D$. Given that $\angle B A C \neq 60^{\circ}$, prove that these four lines are concurrent on $\Gamma$. +4. For an integer $n>0$, denote by $\mathcal{F}(n)$ the set of integers $m>0$ for which the polynomial $p(x)=x^{2}+m x+n$ has an integer root. +(a) Let $S$ denote the set of integers $n>0$ for which $\mathcal{F}(n)$ contains two consecutive integers. Show that $S$ is infinite but + +$$ +\sum_{n \in S} \frac{1}{n} \leq 1 +$$ + +(b) Prove that there are infinitely many positive integers $n$ such that $\mathcal{F}(n)$ contains three consecutive integers. +5. Let $A B C$ be an acute triangle with circumcircle $\omega$, and let $H$ be the foot of the altitude from $A$ to $\overline{B C}$. Let $P$ and $Q$ be the points on $\omega$ with $P A=P H$ and $Q A=Q H$. The tangent to $\omega$ at $P$ intersects lines $A C$ and $A B$ at $E_{1}$ and $F_{1}$ respectively; the tangent to $\omega$ at $Q$ intersects lines $A C$ and $A B$ at $E_{2}$ and $F_{2}$ respectively. Show that the circumcircles of $\triangle A E_{1} F_{1}$ and $\triangle A E_{2} F_{2}$ are congruent, and the line through their centers is parallel to the tangent to $\omega$ at $A$. +6. Let $S=\{1, \ldots, 100\}$, and for every positive integer $n$ define + +$$ +T_{n}=\left\{\left(a_{1}, \ldots, a_{n}\right) \in S^{n} \mid a_{1}+\cdots+a_{n} \equiv 0 \quad(\bmod 100)\right\} +$$ + +Determine which $n$ have the following property: if we color any 75 elements of $S$ red, then at least half of the $n$-tuples in $T_{n}$ have an even number of coordinates with red elements. +7. Let $n$ be a positive integer. A frog starts on the number line at 0 . Suppose it makes a finite sequence of hops, subject to two conditions: + +- The frog visits only points in $\left\{1,2, \ldots, 2^{n}-1\right\}$, each at most once. +- The length of each hop is in $\left\{2^{0}, 2^{1}, 2^{2}, \ldots\right\}$. (The hops may be either direction, left or right.) + +Let $S$ be the sum of the (positive) lengths of all hops in the sequence. What is the maximum possible value of $S$ ? +8. For which positive integers $b>2$ do there exist infinitely many positive integers $n$ such that $n^{2}$ divides $b^{n}+1$ ? +9. Show that there is an absolute constant $c<1$ with the following property: whenever $\mathcal{P}$ is a polygon with area 1 in the plane, one can translate it by a distance of $\frac{1}{100}$ in some direction to obtain a polygon $\mathcal{Q}$, for which the intersection of the interiors of $\mathcal{P}$ and $\mathcal{Q}$ has total area at most $c$. + +## §1 Solutions to Day 1 + +## §1.1 TSTST 2018/1, proposed by Evan Chen, Yang Liu + +Available online at https://aops.com/community/p10570981. + +## Problem statement + +As usual, let $\mathbb{Z}[x]$ denote the set of single-variable polynomials in $x$ with integer coefficients. Find all functions $\theta: \mathbb{Z}[x] \rightarrow \mathbb{Z}$ such that for any polynomials $p, q \in \mathbb{Z}[x]$, + +- $\theta(p+1)=\theta(p)+1$, and +- if $\theta(p) \neq 0$ then $\theta(p)$ divides $\theta(p \cdot q)$. + +The answer is $\theta: p \mapsto p(c)$, for each choice of $c \in \mathbb{Z}$. Obviously these work, so we prove these are the only ones. In what follows, $x \in \mathbb{Z}[x]$ is the identity polynomial, and $c=\theta(x)$. + +【 First solution (Merlijn Staps). Consider an integer $n \neq c$. Because $x-n \mid p(x)-p(n)$, we have + +$$ +\theta(x-n)|\theta(p(x)-p(n)) \Longrightarrow c-n| \theta(p(x))-p(n) . +$$ + +On the other hand, $c-n \mid p(c)-p(n)$. Combining the previous two gives $c-n \mid$ $\theta(p(x))-p(c)$, and by letting $n$ large we conclude $\theta(p(x))-p(c)=0$, so $\theta(p(x))=p(c)$. + +II Second solution. First, we settle the case $\operatorname{deg} p=0$. In that case, from the second property, $\theta(m)=m+\theta(0)$ for every integer $m \in \mathbb{Z}$ (viewed as a constant polynomial). Thus $m+\theta(0) \mid 2 m+\theta(0)$, hence $m+\theta(0) \mid-\theta(0)$, so $\theta(0)=0$ by taking $m$ large. Thus $\theta(m)=m$ for $m \in \mathbb{Z}$. + +Next, we address the case of $\operatorname{deg} p=1$. We know $\theta(x+b)=c+b$ for $b \in \mathbb{Z}$. Now for each particular $a \in \mathbb{Z}$, we have + +$$ +c+k|\theta(x+k)| \theta(a x+a k)=\theta(a x)+a k \Longrightarrow c+k \mid \theta(a x)-a c . +$$ + +for any $k \neq-c$. Since this is true for large enough $k$, we conclude $\theta(a x)=a c$. Thus $\theta(a x+b)=a c+b$. + +We now proceed by induction on $\operatorname{deg} p$. Fix a polynomial $p$ and assume it's true for all $p$ of smaller degree. Choose a large integer $n$ (to be determined later) for which $p(n) \neq p(c)$. We then have + +$$ +\left.\frac{p(c)-p(n)}{c-n}=\theta\left(\frac{p-p(n)}{x-n}\right) \right\rvert\, \theta(p-p(n))=\theta(p)-p(n) +$$ + +Subtracting off $c-n$ times the left-hand side gives + +$$ +\left.\frac{p(c)-p(n)}{c-n} \right\rvert\, \theta(p)-p(c) +$$ + +The left-hand side can be made arbitrarily large by letting $n \rightarrow \infty$, since $\operatorname{deg} p \geq 2$. Thus $\theta(p)=p(c)$, concluding the proof. +\ Authorship comments. I will tell you a story about the creation of this problem. Yang Liu and I were looking over the drafts of December and January TST in October 2017, and both of us had the impression that the test was too difficult. This sparked a non-serious suggestion that we should try to come up with a problem now that would be easy enough to use. While we ended up just joking about changing the TST, we did get this problem out of it. + +Our idea was to come up with a functional equation that was different from the usual fare: at first we tried $\mathbb{Z}[x] \rightarrow \mathbb{Z}[x]$, but then I suggested the idea of using $\mathbb{Z}[x] \rightarrow \mathbb{Z}$, with the answer being the "evaluation" map. Well, what properties does that satisfy? One answer was $a-b \mid p(a)-p(b)$; this didn't immediately lead to anything, but eventually we hit on the form of the problem above off this idea. At first we didn't require $\theta(p) \neq 0$ in the bullet, but without the condition the problem was too easy, since 0 divides only itself; and so the condition was added and we got the functional equation. + +I proposed the problem to USAMO 2018, but it was rejected (unsurprisingly; I think the problem may be too abstract for novice contestants). Instead it was used for TSTST, which I thought fit better. + +## §1.2 TSTST 2018/2, proposed by Victor Wang + +Available online at https://aops.com/community/p10570985. + +## Problem statement + +In the nation of Onewaynia, certain pairs of cities are connected by one-way roads. Every road connects exactly two cities (roads are allowed to cross each other, e.g., via bridges), and each pair of cities has at most one road between them. Moreover, every city has exactly two roads leaving it and exactly two roads entering it. + +We wish to close half the roads of Onewaynia in such a way that every city has exactly one road leaving it and exactly one road entering it. Show that the number of ways to do so is a power of 2 greater than 1 (i.e. of the form $2^{n}$ for some integer $n \geq 1$ ). + +In the language of graph theory, we have a simple digraph $G$ which is 2 -regular and we seek the number of sub-digraphs which are 1-regular. We now present two solution paths. + +『 First solution, combinatorial. We construct a simple undirected bipartite graph $\Gamma$ as follows: + +- the vertex set consists of two copies of $V(G)$, say $V_{\text {out }}$ and $V_{\text {in }}$; and +- for $v \in V_{\text {out }}$ and $w \in V_{\text {in }}$ we have an undirected edge $v w \in E(\Gamma)$ if and only if the directed edge $v \rightarrow w$ is in $G$. + +Moreover, the desired sub-digraphs of $H$ correspond exactly to perfect matchings of $\Gamma$. +However the graph $\Gamma$ is 2 -regular and hence consists of several disjoint (simple) cycles of even length. If there are $n$ such cycles, the number of perfect matchings is $2^{n}$, as desired. + +Remark. The construction of $\Gamma$ is not as magical as it may first seem. +Suppose we pick a road $v_{1} \rightarrow v_{2}$ to use. Then, the other road $v_{3} \rightarrow v_{2}$ is certainly not used; hence some other road $v_{3} \rightarrow v_{4}$ must be used, etc. We thus get a cycle of forced decisions until we eventually return to the vertex $v_{1}$. + +These cycles in the original graph $G$ (where the arrows alternate directions) correspond to the cycles we found in $\Gamma$. It's merely that phrasing the solution in terms of $\Gamma$ makes it cleaner in a linguistic sense, but not really in a mathematical sense. + +【 Second solution by linear algebra over $\mathbb{F}_{2}$ (Brian Lawrence). This is actually not that different from the first solution. For each edge $e$, we create an indicator variable $x_{e}$. We then require for each vertex $v$ that: + +- If $e_{1}$ and $e_{2}$ are the two edges leaving $v$, then we require $x_{e_{1}}+x_{e_{2}} \equiv 1(\bmod 2)$. +- If $e_{3}$ and $e_{4}$ are the two edges entering $v$, then we require $x_{e_{3}}+x_{e_{4}} \equiv 1(\bmod 2)$. + +We thus get a large system of equations. Moreover, the solutions come in natural pairs $\vec{x}$ and $\vec{x}+\overrightarrow{1}$ and therefore the number of solutions is either zero, or a power of two. So we just have to prove there is at least one solution. + +For linear algebra reasons, there can only be zero solutions if some nontrivial linear combination of the equations gives the sum $0 \equiv 1$. So suppose we added up some subset $S$ +of the equations for which every variable appeared on the left-hand side an even number of times. Then every variable that did appear appeared exactly twice; and accordingly we see that the edges corresponding to these variables form one or more even cycles as in the previous solution. Of course, this means $|S|$ is even, so we really have $0 \equiv 0(\bmod 2)$ as needed. + +Remark. The author's original proposal contained a second part asking to show that it was not always possible for the resulting $H$ to be connected, even if $G$ was strongly connected. This problem is related to IMO Shortlist 2002 C6, which gives an example of a strongly connected graph which does have a full directed Hamiltonian cycle. + +## §1.3 TSTST 2018/3, proposed by Yannick Yao, Evan Chen + +Available online at https://aops.com/community/p10570988. + +## Problem statement + +Let $A B C$ be an acute triangle with incenter $I$, circumcenter $O$, and circumcircle $\Gamma$. Let $M$ be the midpoint of $\overline{A B}$. Ray $A I$ meets $\overline{B C}$ at $D$. Denote by $\omega$ and $\gamma$ the circumcircles of $\triangle B I C$ and $\triangle B A D$, respectively. Line $M O$ meets $\omega$ at $X$ and $Y$, while line $C O$ meets $\omega$ at $C$ and $Q$. Assume that $Q$ lies inside $\triangle A B C$ and $\angle A Q M=\angle A C B$. + +Consider the tangents to $\omega$ at $X$ and $Y$ and the tangents to $\gamma$ at $A$ and $D$. Given that $\angle B A C \neq 60^{\circ}$, prove that these four lines are concurrent on $\Gamma$. + +Henceforth assume $\angle A \neq 60^{\circ}$; we prove the concurrence. Let $L$ denote the center of $\omega$, which is the midpoint of minor arc $B C$. + +Claim - Let $K$ be the point on $\omega$ such that $\overline{K L} \| \overline{A B}$ and $\overline{K C} \| \overline{A L}$. Then $\overline{K A}$ is tangent to $\gamma$, and we may put + +$$ +x=K A=L B=L C=L X=L Y=K X=K Y . +$$ + +Proof. By construction, $K A=L B=L C$. Also, $\overline{M O}$ is the perpendicular bisector of $\overline{K L}$ (since the chords $\overline{K L}, \overline{A B}$ of $\omega$ are parallel) and so $K X L Y$ is a rhombus as well. + +Moreover, $\overline{K A}$ is tangent to $\gamma$ as well since + +$$ +\measuredangle K A D=\measuredangle K A L=\measuredangle K A C+\measuredangle C A L=\measuredangle K B C+\measuredangle A B K=\measuredangle A B C . +$$ + +![](https://cdn.mathpix.com/cropped/2024_11_19_a0f01f2a44c8136e2158g-08.jpg?height=918&width=914&top_left_y=1551&top_left_x=571) + +Up to now we have not used the existence of $Q$; we henceforth do so. + +Note that $Q \neq O$, since $\angle A \neq 60^{\circ} \Longrightarrow O \notin \omega$. Moreover, we have $\angle A O M=\angle A C B$ too. Since $O$ and $Q$ both lie inside $\triangle A B C$, this implies that $A, M, O, Q$ are concyclic. As $Q \neq O$ we conclude $\angle C Q A=90^{\circ}$. + +The main claim is now: +Claim - Assuming $Q$ exists, the rhombus $L X K Y$ is a square. In particular, $\overline{K X}$ and $\overline{K Y}$ are tangent to $\omega$. + +First proof of Claim, communicated by Milan Haiman. Observe that $\triangle Q L C \sim \triangle L O C$ since both triangles are isosceles and share a base angle. Hence, $C L^{2}=C O \cdot C Q$. + +Let $N$ be the midpoint of $\overline{A C}$, which lies on $(A M O Q)$. Then, + +$$ +x^{2}=C L^{2}=C O \cdot C Q=C N \cdot C A=\frac{1}{2} C A^{2}=\frac{1}{2} L K^{2} +$$ + +where we have also used the fact $A Q O N$ is cyclic. Thus $L K=\sqrt{2} x$ and so the rhombus $L X K Y$ is actually a square. + +Second proof of Claim, Evan Chen. Observe that $Q$ lies on the circle with diameter $\overline{A C}$, centered at $N$, say. This means that $O$ lies on the radical axis of $\omega$ and $(N)$, hence $\overline{N L} \perp \overline{C O}$ implying + +$$ +\begin{aligned} +N O^{2}+C L^{2} & =N C^{2}+L O^{2}=N C^{2}+O C^{2}=N C^{2}+N O^{2}+N C^{2} \\ +\Longrightarrow x^{2} & =2 N C^{2} \\ +\Longrightarrow x & =\sqrt{2} N C=\frac{1}{\sqrt{2}} A C=\frac{1}{\sqrt{2}} L K +\end{aligned} +$$ + +So $L X K Y$ is a rhombus with $L K=\sqrt{2} x$. Hence it is a square. +Third proof of Claim. A solution by trig is also possible. As in the previous claims, it suffices to show that $A C=\sqrt{2} x$. + +First, we compute the length $C Q$ in two ways; by angle chasing one can show $\angle C B Q=$ $180^{\circ}-(\angle B Q C+\angle Q C B)=\frac{1}{2} \angle A$, and so + +$$ +\begin{aligned} +A C \sin B=C Q & =\frac{B C}{\sin \left(90^{\circ}+\frac{1}{2} \angle A\right)} \cdot \sin \frac{1}{2} \angle A \\ +\Longleftrightarrow \sin ^{2} B & =\frac{\sin A \cdot \sin \frac{1}{2} \angle A}{\cos \frac{1}{2} \angle A} \\ +\Longleftrightarrow \sin ^{2} B & =2 \sin ^{2} \frac{1}{2} \angle A \\ +\Longleftrightarrow \sin B & =\sqrt{2} \sin \frac{1}{2} \angle A \\ +\Longleftrightarrow 2 R \sin B & =\sqrt{2}\left(2 R \sin \frac{1}{2} \angle A\right) \\ +\Longleftrightarrow A C & =\sqrt{2} x +\end{aligned} +$$ + +as desired (we have here used the fact $\triangle A B C$ is acute to take square roots). +It is interesting to note that $\sin ^{2} B=2 \sin ^{2} \frac{1}{2} \angle A$ can be rewritten as + +$$ +\cos A=\cos ^{2} B +$$ + +since $\cos ^{2} B=1-\sin ^{2} B=1-2 \sin ^{2} \frac{1}{2} \angle A=\cos A$; this is the condition for the existence of the point $Q$. + +We finish by proving that + +$$ +K D=K A +$$ + +and hence line $\overline{K D}$ is tangent to $\gamma$. Let $E=\overline{B C} \cap \overline{K L}$. Then + +$$ +L E \cdot L K=L C^{2}=L X^{2}=\frac{1}{2} L K^{2} +$$ + +and so $E$ is the midpoint of $\overline{L K}$. Thus $\overline{M X O Y}, \overline{B C}, \overline{K L}$ are concurrent at $E$. As $\overline{D L} \| \overline{K C}$, we find that $D L C K$ is a parallelogram, so $K D=C L=K A$ as well. Thus $\overline{K D}$ and $\overline{K A}$ are tangent to $\gamma$. + +Remark. The condition $\angle A \neq 60^{\circ}$ cannot be dropped, since if $Q=O$ the problem is not true. + +On the other hand, nearly all solutions begin by observing $Q \neq O$ and then obtaining $\angle A Q O=90^{\circ}$. This gives a way to construct the diagram by hand with ruler and compass. One draws an arbitrary chord $\overline{B C}$ of a circle $\omega$ centered at $L$, and constructs $O$ as the circumcenter of $\triangle B L C$ (hence obtaining $\Gamma$ ). Then $Q$ is defined as the intersection of ray $C O$ with $\omega$, and $A$ is defined by taking the perpendicular line through $Q$ on the circle $\Gamma$. In this way we can draw a triangle $A B C$ satisfying the problem conditions. +\ Authorship comments. In the notation of the present points, the question originally sent to me by Yannick Yao read: + +Circles $(L)$ and $(O)$ are drawn, meeting at $B$ and $C$, with $L$ on $(O)$. Ray $C O$ meets $(L)$ at $Q$, and $A$ is on $(O)$ such that $\angle C Q A=90^{\circ}$. The angle bisector of $\angle A O B$ meets $(L)$ at $X$ and $Y$. Show that $\angle X L Y=90^{\circ}$. + +Notice the points $M$ and $K$ are absent from the problem. I am told this was found as part of the computer game "Euclidea". Using this as the starting point, I constructed the TSTST problem by recognizing the significance of that special point $K$, which became the center of attention. + +## §2 Solutions to Day 2 + +## §2.1 TSTST 2018/4, proposed by Ivan Borsenco + +Available online at https://aops.com/community/p10570991. + +## Problem statement + +For an integer $n>0$, denote by $\mathcal{F}(n)$ the set of integers $m>0$ for which the polynomial $p(x)=x^{2}+m x+n$ has an integer root. +(a) Let $S$ denote the set of integers $n>0$ for which $\mathcal{F}(n)$ contains two consecutive integers. Show that $S$ is infinite but + +$$ +\sum_{n \in S} \frac{1}{n} \leq 1 +$$ + +(b) Prove that there are infinitely many positive integers $n$ such that $\mathcal{F}(n)$ contains three consecutive integers. + +We prove the following. +Claim - The set $S$ is given explicitly by $S=\{x(x+1) y(y+1) \mid x, y>0\}$. + +Proof. Note that $m, m+1 \in \mathcal{F}(n)$ if and only if there exist integers $q>p \geq 0$ such that + +$$ +\begin{aligned} +m^{2}-4 n & =p^{2} \\ +(m+1)^{2}-4 n & =q^{2} +\end{aligned} +$$ + +Subtraction gives $2 m+1=q^{2}-p^{2}$, so $p$ and $q$ are different parities. We can thus let $q-p=2 x+1, q+p=2 y+1$, where $y \geq x \geq 0$ are integers. It follows that + +$$ +\begin{aligned} +4 n & =m^{2}-p^{2} \\ +& =\left(\frac{q^{2}-p^{2}-1}{2}\right)^{2}-p^{2}=\left(\frac{q^{2}-p^{2}-1}{2}-p\right)\left(\frac{q^{2}-p^{2}-1}{2}+p\right) \\ +& =\frac{q^{2}-\left(p^{2}+2 p+1\right)}{2} \cdot \frac{q^{2}-\left(p^{2}-2 p+1\right)}{2} \\ +& =\frac{1}{4}(q-p-1)(q-p+1)(q+p-1)(q+p+1)=\frac{1}{4}(2 x)(2 x+2)(2 y)(2 y+2) \\ +\Longrightarrow n & =x(x+1) y(y+1) . +\end{aligned} +$$ + +Since $n>0$ we require $x, y>0$. Conversely, if $n=x(x+1) y(y+1)$ for positive $x$ and $y$ then $m=\sqrt{p^{2}+4 n}=\sqrt{(y-x)^{2}+4 n}=2 x y+x+y=x(y+1)+(x+1) y$ and $m+1=2 x y+x+y+1=x y+(x+1)(y+1)$. Thus we conclude the main claim. + +From this, part (a) follows as + +$$ +\sum_{n \in S} n^{-1} \leq\left(\sum_{x \geq 1} \frac{1}{x(x+1)}\right)\left(\sum_{y \geq 1} \frac{1}{y(y+1)}\right)=1 \cdot 1=1 +$$ + +As for (b), retain the notation in the proof of the claim. Now $m+2 \in S$ if and only if $(m+2)^{2}-4 n$ is a square, say $r^{2}$. Writing in terms of $p$ and $q$ as parameters we find + +$$ +\begin{aligned} +r^{2} & =(m+2)^{2}-4 n=m^{2}-4 n+4 m+4=p^{2}+2+2(2 m+1) \\ +& =p^{2}+2\left(q^{2}-p^{2}\right)+2=2 q^{2}-p^{2}+2 \\ +\Longleftrightarrow 2 q^{2}+2 & =p^{2}+r^{2} \quad(\dagger) +\end{aligned} +$$ + +with $q>p$ of different parity and $n=\frac{1}{16}(q-p-1)(q-p+1)(q+p-1)(q+p+1)$. +Note that (by taking modulo 8 ) we have $q \not \equiv p \equiv r(\bmod 2)$, and so there are no parity issues and we will always assume $p2$ do there exist infinitely many positive integers $n$ such that $n^{2}$ divides $b^{n}+1$ ? + +This problem is sort of the union of IMO 1990/3 and IMO 2000/5. +The answer is any $b$ such that $b+1$ is not a power of 2 . In the forwards direction, we first prove more carefully the following claim. + +Claim - If $b+1$ is a power of 2 , then the only $n$ which is valid is $n=1$. +Proof. Assume $n>1$ and let $p$ be the smallest prime dividing $n$. We cannot have $p=2$, since then $4 \mid b^{n}+1 \equiv 2(\bmod 4)$. Thus, + +$$ +b^{2 n} \equiv 1 \quad(\bmod p) +$$ + +so the order of $b(\bmod p)$ divides $\operatorname{gcd}(2 n, p-1)=2$. Hence $p \mid b^{2}-1=(b-1)(b+1)$. +But since $b+1$ was a power of 2 , this forces $p \mid b-1$. Then $0 \equiv b^{n}+1 \equiv 2(\bmod p)$, contradiction. + +On the other hand, suppose that $b+1$ is not a power of 2 (and that $b>2$ ). We will inductively construct an infinite sequence of distinct primes $p_{0}, p_{1}, \ldots$, such that the following two properties hold for each $k \geq 0$ : + +- $p_{0}^{2} \ldots p_{k-1}^{2} p_{k} \mid b^{p_{0} \ldots p_{k-1}}+1$, +- and hence $p_{0}^{2} \ldots p_{k-1}^{2} p_{k}^{2} \mid b^{p_{0} \ldots p_{k-1} p_{k}}+1$ by exponent lifting lemma. + +This will solve the problem. +Initially, let $p_{0}$ be any odd prime dividing $b+1$. For the inductive step, we contend there exists an odd prime $q \notin\left\{p_{0}, \ldots, p_{k}\right\}$ such that $q \mid b^{p_{0} \ldots p_{k}}+1$. Indeed, this follows immediately by Zsigmondy theorem since $p_{0} \ldots p_{k}$ divides $b^{p_{0} \ldots p_{k-1}}+1$. Since $\left(b^{p_{0} \ldots p_{k}}\right)^{q} \equiv b^{p_{0} \ldots p_{k}}(\bmod q)$, it follows we can then take $p_{k+1}=q$. This finishes the induction. + +To avoid the use of Zsigmondy, one can instead argue as follows: let $p=p_{k}$ for brevity, and let $c=b^{p_{0} \ldots p_{k-1}}$. Then $\frac{\frac{c}{}^{p}+1}{c+1}=c^{p-1}-c^{p-2}+\cdots+1$ has GCD exactly $p$ with $c+1$. Moreover, this quotient is always odd. Thus as long as $c^{p}+1>p \cdot(c+1)$, there will be some new prime dividing $c^{p}+1$ but not $c+1$. This is true unless $p=3$ and $c=2$, but we assumed $b>2$ so this case does not appear. + +Remark (On new primes). In going from $n^{2} \mid b^{n}+1$ to $(n q)^{2} \mid b^{n q}+1$, one does not necessarily need to pick a $q$ such that $q \nmid n$, as long as $\nu_{q}\left(n^{2}\right)<\nu_{q}\left(b^{n}+1\right)$. In other words it suffices to just check that $\frac{b^{n}+1}{n^{2}}$ is not a power of 2 in this process. + +However, this calculation is a little more involved with this approach. One proceeds by noting that $n$ is odd, hence $\nu_{2}\left(b^{n}+1\right)=\nu_{2}(b+1)$, and thus $\frac{b^{n}+1}{n^{2}}=2^{\nu_{2}(b+1)} \leq b+1$, which is a little harder to bound than the analogous $c^{p}+1>p \cdot(c+1)$ from the previous solution. +\ Authorship comments. I came up with this problem by simply mixing together the main ideas of IMO 1990/3 and IMO 2000/5, late one night after a class. On the other hand, I do not consider it very original; it is an extremely "routine" number theory problem for experienced contestants, using highly standard methods. Thus it may not be that interesting, but is a good discriminator of understanding of fundamentals. + +IMO 1990/3 shows that if $b=2$, then the only $n$ which work are $n=1$ and $n=3$. Thus $b=2$ is a special case and for this reason the problem explicitly requires $b>2$. + +An alternate formulation of the problem is worth mentioning. Originally, the problem statement asked whether there existed $n$ with at least 3 (or 2018, etc.) prime divisors, thus preventing the approach in which one takes a prime $q$ dividing $\frac{b^{n}+1}{n^{2}}$. Ankan Bhattacharya suggested changing it to "infinitely many $n$ ", which is more natural. + +These formulations are actually not so different though. Explicitly, suppose $k^{2} \mid b^{k}+1$ and $p \mid b^{k}+1$. Consider any $k \mid n$ with $n^{2} \mid b^{n}+1$, and let $p$ be an odd prime dividing $b^{k}+1$. Then $2 \nu_{p}(n) \leq \nu_{p}\left(b^{n}+1\right)=\nu_{p}(n / k)+\nu_{p}\left(b^{k}+1\right)$ and thus + +$$ +\nu_{p}(n / k) \leq \nu_{p}\left(\frac{b^{k}+1}{k^{2}}\right) . +$$ + +Effectively, this means we can only add each prime a certain number of times. + +## §3.3 TSTST 2018/9, proposed by Linus Hamilton + +Available online at https://aops.com/community/p10571003. + +## Problem statement + +Show that there is an absolute constant $c<1$ with the following property: whenever $\mathcal{P}$ is a polygon with area 1 in the plane, one can translate it by a distance of $\frac{1}{100}$ in some direction to obtain a polygon $\mathcal{Q}$, for which the intersection of the interiors of $\mathcal{P}$ and $\mathcal{Q}$ has total area at most $c$. + +The following solution is due to Brian Lawrence. We will prove the result with the generality of any measurable set $\mathcal{P}$ (rather than a polygon). For a vector $v$ in the plane, write $\mathcal{P}+v$ for the translate of $\mathcal{P}$ by $v$. + +Suppose $\mathcal{P}$ is a polygon of area 1 , and $\varepsilon>0$ is a constant, such that for any translate $\mathcal{Q}=\mathcal{P}+v$, where $v$ has length exactly $\frac{1}{100}$, the intersection of $\mathcal{P}$ and $\mathcal{Q}$ has area at least $1-\varepsilon$. The problem asks us to prove a lower bound on $\varepsilon$. + +## Lemma + +Fix a sequence of $n$ vectors $v_{1}, v_{2}, \ldots, v_{n}$, each of length $\frac{1}{100}$. A grasshopper starts at a random point $x$ of $\mathcal{P}$, and makes $n$ jumps to $x+v_{1}+\cdots+v_{n}$. Then it remains in $\mathcal{P}$ with probability at least $1-n \varepsilon$. + +Proof. In order for the grasshopper to leave $\mathcal{P}$ at step $i$, the grasshopper's position before step $i$ must be inside the difference set $\mathcal{P} \backslash\left(\mathcal{P}-v_{i}\right)$. Since this difference set has area at most $\varepsilon$, the probability the grasshopper leaves $\mathcal{P}$ at step $i$ is at most $\varepsilon$. Summing over the $n$ steps, the probability that the grasshopper ever manages to leave $\mathcal{P}$ is at most $n \varepsilon$. + +## Corollary + +Fix a vector $w$ of length at most 8. A grasshopper starts at a random point $x$ of $\mathcal{P}$, and jumps to $x+w$. Then it remains in $\mathcal{P}$ with probability at least $1-800 \varepsilon$. + +Proof. Apply the previous lemma with 800 jumps. Any vector $w$ of length at most 8 can be written as $w=v_{1}+v_{2}+\cdots+v_{800}$, where each $v_{i}$ has length exactly $\frac{1}{100}$. + +Now consider the process where we select a random starting point $x \in \mathcal{P}$ for our grasshopper, and a random vector $w$ of length at most 8 (sampled uniformly from the closed disk of radius 8 ). Let $q$ denote the probability of staying inside $\mathcal{P}$ we will bound $q$ from above and below. + +- On the one hand, suppose we pick $w$ first. By the previous corollary, $q \geq 1-800 \varepsilon$ (irrespective of the chosen $w$ ). +- On the other hand, suppose we pick $x$ first. Then the possible landing points $x+w$ are uniformly distributed over a closed disk of radius 8 , which has area $64 \pi$. The probability of landing in $\mathcal{P}$ is certainly at most $\frac{[\mathcal{P}]}{64 \pi}$. + +Consequently, we deduce + +$$ +1-800 \varepsilon \leq q \leq \frac{[\mathcal{P}]}{64 \pi} \Longrightarrow \varepsilon>\frac{1-\frac{[\mathcal{P}]}{64 \pi}}{800}>0.001 +$$ + +as desired. +Remark. The choice of 800 jumps is only for concreteness; any constant $n$ for which $\pi(n / 100)^{2}>1$ works. I think $n=98$ gives the best bound following this approach. + diff --git a/USA_TST_TST/md/en-sols-TSTST-2019.md b/USA_TST_TST/md/en-sols-TSTST-2019.md new file mode 100644 index 0000000000000000000000000000000000000000..d253d024fe18424779ea39b847c78f5759f5f14f --- /dev/null +++ b/USA_TST_TST/md/en-sols-TSTST-2019.md @@ -0,0 +1,764 @@ +# USA TSTST 2019 Solutions
United States of America - TST Selection Test
Ankan Bhattacharya and Evan Chen
$61^{\text {st }}$ IMO 2020 Russia and $9^{\text {th }}$ EGMO 2020 Netherlands + +## Contents + +0 Problems ..... 2 +1 Solutions to Day 1 ..... 3 +1.1 TSTST 2019/1, proposed by Evan Chen ..... 3 +1.2 TSTST 2019/2, proposed by Merlijn Staps ..... 6 +1.3 TSTST 2019/3, proposed by Nikolai Beluhov ..... 9 +2 Solutions to Day 2 ..... 11 +2.1 TSTST 2019/4, proposed by Merlijn Staps ..... 11 +2.2 TSTST 2019/5, proposed by Gunmay Handa ..... 13 +2.3 TSTST 2019/6, proposed by Nikolai Beluhov ..... 19 +3 Solutions to Day 3 ..... 21 +3.1 TSTST 2019/7, proposed by Ankan Bhattacharya ..... 21 +3.2 TSTST 2019/8, proposed by Ankan Bhattacharya ..... 23 +3.3 TSTST 2019/9, proposed by Ankan Bhattacharya ..... 25 + +## §0 Problems + +1. Find all binary operations $\diamond: \mathbb{R}_{>0} \times \mathbb{R}_{>0} \rightarrow \mathbb{R}_{>0}$ (meaning $\diamond$ takes pairs of positive real numbers to positive real numbers) such that for any real numbers $a, b, c>0$, + +- the equation $a \diamond(b \diamond c)=(a \diamond b) \cdot c$ holds; and +- if $a \geq 1$ then $a \diamond a \geq 1$. + +2. Let $A B C$ be an acute triangle with circumcircle $\Omega$ and orthocenter $H$. Points $D$ and $E$ lie on segments $A B$ and $A C$ respectively, such that $A D=A E$. The lines through $B$ and $C$ parallel to $\overline{D E}$ intersect $\Omega$ again at $P$ and $Q$, respectively. Denote by $\omega$ the circumcircle of $\triangle A D E$. +(a) Show that lines $P E$ and $Q D$ meet on $\omega$. +(b) Prove that if $\omega$ passes through $H$, then lines $P D$ and $Q E$ meet on $\omega$ as well. +3. On an infinite square grid we place finitely many cars, which each occupy a single cell and face in one of the four cardinal directions. Cars may never occupy the same cell. It is given that the cell immediately in front of each car is empty, and moreover no two cars face towards each other (no right-facing car is to the left of a left-facing car within a row, etc.). In a move, one chooses a car and shifts it one cell forward to a vacant cell. Prove that there exists an infinite sequence of valid moves using each car infinitely many times. +4. Consider coins with positive real denominations not exceeding 1. Find the smallest $C>0$ such that the following holds: if we are given any 100 such coins with total value 50 , then we can always split them into two stacks of 50 coins each such that the absolute difference between the total values of the two stacks is at most $C$. +5. Let $A B C$ be an acute triangle with orthocenter $H$ and circumcircle $\Gamma$. A line through $H$ intersects segments $A B$ and $A C$ at $E$ and $F$, respectively. Let $K$ be the circumcenter of $\triangle A E F$, and suppose line $A K$ intersects $\Gamma$ again at a point $D$. Prove that line $H K$ and the line through $D$ perpendicular to $\overline{B C}$ meet on $\Gamma$. +6. Suppose $P$ is a polynomial with integer coefficients such that for every positive integer $n$, the sum of the decimal digits of $|P(n)|$ is not a Fibonacci number. Must $P$ be constant? +7. Let $f: \mathbb{Z} \rightarrow\left\{1,2, \ldots, 10^{100}\right\}$ be a function satisfying + +$$ +\operatorname{gcd}(f(x), f(y))=\operatorname{gcd}(f(x), x-y) +$$ + +for all integers $x$ and $y$. Show that there exist positive integers $m$ and $n$ such that $f(x)=\operatorname{gcd}(m+x, n)$ for all integers $x$. +8. Let $\mathcal{S}$ be a set of 16 points in the plane, no three collinear. Let $\chi(\mathcal{S})$ denote the number of ways to draw 8 line segments with endpoints in $\mathcal{S}$, such that no two drawn segments intersect, even at endpoints. Find the smallest possible value of $\chi(\mathcal{S})$ across all such $\mathcal{S}$. +9. Let $A B C$ be a triangle with incenter $I$. Points $K$ and $L$ are chosen on segment $B C$ such that the incircles of $\triangle A B K$ and $\triangle A B L$ are tangent at $P$, and the incircles of $\triangle A C K$ and $\triangle A C L$ are tangent at $Q$. Prove that $I P=I Q$. + +## §1 Solutions to Day 1 + +## §1.1 TSTST 2019/1, proposed by Evan Chen + +Available online at https://aops.com/community/p12608849. + +## Problem statement + +Find all binary operations $\diamond: \mathbb{R}_{>0} \times \mathbb{R}_{>0} \rightarrow \mathbb{R}_{>0}$ (meaning $\diamond$ takes pairs of positive real numbers to positive real numbers) such that for any real numbers $a, b, c>0$, + +- the equation $a \diamond(b \diamond c)=(a \diamond b) \cdot c$ holds; and +- if $a \geq 1$ then $a \diamond a \geq 1$. + +The answer is only multiplication and division, which both obviously work. +We present two approaches, one appealing to theorems on Cauchy's functional equation, and one which avoids it. + +【 First solution using Cauchy FE. We prove: +Claim - We have $a \diamond b=a f(b)$ where $f$ is some involutive and totally multiplicative function. (In fact, this classifies all functions satisfying the first condition completely.) + +Proof. Let $P(a, b, c)$ denote the assertion $a \diamond(b \diamond c)=(a \diamond b) \cdot c$. + +- Note that for any $x$, the function $y \mapsto x \diamond y$ is injective, because if $x \diamond y_{1}=x \diamond y_{2}$ then take $P\left(1, x, y_{i}\right)$ to get $y_{1}=y_{2}$. +- Take $P(1, x, 1)$ and injectivity to get $x \diamond 1=x$. +- Take $P(1,1, y)$ to get $1 \diamond(1 \diamond y)=y$. +- Take $P(x, 1,1 \diamond y)$ to get + +$$ +x \diamond y=x \cdot(1 \diamond y) +$$ + +Henceforth let us define $f(y)=1 \diamond y$, so $f(1)=1, f$ is involutive and + +$$ +x \diamond y=x f(y) +$$ + +Plugging this into the original condition now gives $f(b f(c))=f(b) c$, which (since $f$ is an involution) gives $f$ completely multiplicative. + +In particular, $f(1)=1$. We are now interested only in the second condition, which reads $f(x) \geq 1 / x$ for $x \geq 1$. + +Define the function + +$$ +g(t)=\log f\left(e^{t}\right) +$$ + +so that $g$ is additive, and also $g(t) \geq-t$ for all $t \geq 0$. We appeal to the following theorem: + +## Lemma + +If $h: \mathbb{R} \rightarrow \mathbb{R}$ is an additive function which is not linear, then it is dense in the plane: for any point $\left(x_{0}, y_{0}\right)$ and $\varepsilon>0$ there exists $(x, y)$ such that $h(x)=y$ and $\sqrt{\left(x-x_{0}\right)^{2}+\left(y-y_{0}\right)^{2}}<\varepsilon$. + +Applying this lemma with the fact that $g(t) \geq-t$ implies readily that $g$ is linear. In other words, $f$ is of the form $f(x)=x^{r}$ for some fixed real number $r$. It is easy to check $r= \pm 1$ which finishes. + +【 Second solution manually. As before we arrive at $a \diamond b=a f(b)$, with $f$ an involutive and totally multiplicative function. + +We prove that: +Claim - For any $a>0$, we have $f(a) \in\{1 / a, a\}$. + +Proof. WLOG $b>1$, and suppose $f(b)=a \geq 1 / b$ hence $f(a)=b$. +Assume that $a b>1$; we show $a=b$. Note that for integers $m$ and $n$ with $a^{n} b^{m} \geq 1$, we must have + +$$ +a^{m} b^{n}=f(b)^{m} f(a)^{n}=f\left(a^{n} b^{m}\right) \geq \frac{1}{a^{n} b^{m}} \Longrightarrow(a b)^{m+n} \geq 1 +$$ + +and thus we have arrived at the proposition + +$$ +m+n<0 \Longrightarrow n \log _{b} a+m<0 +$$ + +for all integers $m$ and $n$. Due to the density of $\mathbb{Q}$ in the real numbers, this can only happen if $\log _{b} a=1$ or $a=b$. + +Claim - The function $f$ is continuous. + +Proof. Indeed, it's equivalent to show $g(t)=\log f\left(e^{t}\right)$ is continuous, and we have that + +$$ +|g(t)-g(s)|=\left|\log f\left(e^{t-s}\right)\right|=|t-s| +$$ + +since $f\left(e^{t-s}\right)=e^{ \pm|t-s|}$. Therefore $g$ is Lipschitz. Hence $g$ continuous, and $f$ is too. +Finally, we have from $f$ multiplicative that + +$$ +f\left(2^{q}\right)=f(2)^{q} +$$ + +for every rational number $q$, say. As $f$ is continuous this implies $f(x) \equiv x$ or $f(x) \equiv 1 / x$ identically (depending on whether $f(2)=2$ or $f(2)=1 / 2$, respectively). + +Therefore, $a \diamond b=a b$ or $a \diamond b=a \div b$, as needed. +Remark. The Lipschitz condition is one of several other ways to proceed. The point is that if $f(2)=2$ (say), and $x / 2^{q}$ is close to 1 , then $f(x) / 2^{q}=f\left(x / 2^{q}\right)$ is close to 1 , which is enough to force $f(x)=x$ rather than $f(x)=1 / x$. + +Remark. Compare to AMC 10A 2016 \#23, where the second condition is $a \diamond a=1$. + +## §1.2 TSTST 2019/2, proposed by Merlijn Staps + +Available online at https://aops.com/community/p12608478. + +## Problem statement + +Let $A B C$ be an acute triangle with circumcircle $\Omega$ and orthocenter $H$. Points $D$ and $E$ lie on segments $A B$ and $A C$ respectively, such that $A D=A E$. The lines through $B$ and $C$ parallel to $\overline{D E}$ intersect $\Omega$ again at $P$ and $Q$, respectively. Denote by $\omega$ the circumcircle of $\triangle A D E$. +(a) Show that lines $P E$ and $Q D$ meet on $\omega$. +(b) Prove that if $\omega$ passes through $H$, then lines $P D$ and $Q E$ meet on $\omega$ as well. + +We will give one solution to (a), then several solutions to (b). +【 Solution to (a). Note that $\measuredangle A Q P=\measuredangle A B P=\measuredangle A D E$ and $\measuredangle A P Q=\measuredangle A C Q=$ $\measuredangle A E D$, so we have a spiral similarity $\triangle A D E \sim \triangle A Q P$. Therefore, lines $P E$ and $Q D$ meet at the second intersection of $\omega$ and $\Omega$ other than $A$. Call this point $X$. + +【 Solution to (b) using angle chasing. Let $L$ be the reflection of $H$ across $\overline{A B}$, which lies on $\Omega$. + +Claim - Points $L, D, P$ are collinear. +Proof. This is just angle chasing: + +$$ +\begin{aligned} +\measuredangle C L D & =\measuredangle D H L=\measuredangle D H A+\measuredangle A H L=\measuredangle D E A+\measuredangle A H C \\ +& =\measuredangle A D E+\measuredangle C B A=\measuredangle A B P+\measuredangle C B A=\measuredangle C B P=\measuredangle C L P . +\end{aligned} +$$ + +![](https://cdn.mathpix.com/cropped/2024_11_19_f4f672a97b66743213a8g-06.jpg?height=641&width=709&top_left_y=1707&top_left_x=676) + +Now let $K \in \omega$ such that $D H K E$ is an isosceles trapezoid, i.e. $\measuredangle B A H=\measuredangle K A E$. + +Claim - Points $D, K, P$ are collinear. + +Proof. Using the previous claim, + +$$ +\measuredangle K D E=\measuredangle K A E=\measuredangle B A H=\measuredangle L A B=\measuredangle L P B=\measuredangle D P B=\measuredangle P D E . +$$ + +By symmetry, $\overline{Q E}$ will then pass through the same $K$, as needed. +Remark. These two claims imply each other, so guessing one of them allows one to realize the other. It is likely the latter is easiest to guess from the diagram, since it does not need any additional points. + +【 Solution to (b) by orthogonal circles (found by contestants). We define $K$ as in the previous solution, but do not claim that $K$ is the desired intersection. Instead, we note that: + +Claim - Point $K$ is the orthocenter of isosceles triangle $A P Q$. +Proof. Notice that $A H=A K$ and $B C=P Q$. Moreover from $\overline{A H} \perp \overline{B C}$ we deduce $\overline{A K} \perp \overline{P Q}$ by reflection across the angle bisector. + +In light of the formula " $A H^{2}=4 R^{2}-a^{2}$ ", this implies the conclusion. +Let $M$ be the midpoint of $\overline{P Q}$. Since $\triangle A P Q$ is isosceles, + +$$ +\overline{A K M} \perp \overline{P Q} \Longrightarrow M K \cdot M A=M P^{2} +$$ + +by orthocenter properties. +So to summarize + +- The circle with diameter $\overline{P Q}$ is orthogonal to $\omega$. In other words, point $P$ lies on the polar of $Q$ with respect to $\omega$. +- The point $X=\overline{Q D} \cap \overline{P E}$ is on $\omega$. + +On the other hand, if we let $K^{\prime}=\overline{Q E} \cap \omega$, then by Brokard theorem on $X D K^{\prime} E$, the polar of $Q=\overline{X D} \cap \overline{K^{\prime} E}$ pass through $\overline{D K^{\prime}} \cap \overline{X E}$; this point must therefore be $P$ and $K^{\prime}=K$ as desired. + +『 S Solution to (b) by complex numbers (Yang Liu and Michael Ma). Let $M$ be the arc midpoint of $\widehat{B C}$. We use the standard arc midpoint configuration. We have that + +$$ +A=a^{2}, B=b^{2}, C=c^{2}, M=-b c, H=a^{2}+b^{2}+c^{2}, P=\frac{a^{2} c}{b}, Q=\frac{a^{2} b}{c} +$$ + +where $M$ is the arc midpoint of $\widehat{B C}$. By direct angle chasing we can verify that $\overline{M B} \| \overline{D H}$. Also, $D \in \overline{A B}$. Therefore, we can compute $D$ as follows. + +$$ +d+a^{2} b^{2} \bar{d}=a^{2}+b^{2} \text { and } \frac{d-h}{\bar{d}-\bar{h}}=-m b^{2}=b^{3} c \Longrightarrow d=\frac{a^{2}\left(a^{2} c+b^{2} c+c^{3}-b^{3}\right)}{c\left(b c+a^{2}\right)} . +$$ + +By symmetry, we have that + +$$ +e=\frac{a^{2}\left(a^{2} b+b c^{2}+b^{3}-c^{3}\right)}{b\left(b c+a^{2}\right)} . +$$ + +To finish, we want to show that the angle between $\overline{D P}$ and $\overline{E Q}$ is angle $A$. To show this, we compute $\frac{d-p}{e-q} / \overline{\frac{d-p}{e-q}}$. First, we compute + +$$ +\begin{aligned} +d-p & =\frac{a^{2}\left(a^{2} c+b^{2} c+c^{3}-b^{3}\right)}{c\left(b c+a^{2}\right)}-\frac{a^{2} c}{b} \\ +& =a^{2}\left(\frac{a^{2} c+b^{2} c+c^{3}-b^{3}}{c\left(b c+a^{2}\right)}-\frac{c}{b}\right)=\frac{a^{2}\left(a^{2} c-b^{3}\right)(b-c)}{b c\left(b c+a^{2}\right)} +\end{aligned} +$$ + +By symmetry, + +$$ +\frac{d-p}{e-q}=-\frac{a^{2} c-b^{3}}{a^{2} b-c^{3}} \Longrightarrow \frac{d-p}{e-q} / \overline{\frac{d-p}{e-q}}=\frac{a^{2} b^{3} c}{a^{2} b c^{3}}=\frac{b^{2}}{c^{2}} +$$ + +as desired. +\I Solution to (b) using untethered moving points (Zack Chroman). We work in the real projective plane $\mathbb{R P}^{2}$, and animate $C$ linearly on a fixed line through $A$. + +Recall: + +## Lemma (Zack's lemma) + +Suppose points $A, B$ have degree $d_{1}, d_{2}$, and there are $k$ values of $t$ for which $A=B$. Then line $A B$ has degree at most $d_{1}+d_{2}-k$. Similarly, if lines $\ell_{1}, \ell_{2}$ have degrees $d_{1}, d_{2}$, and there are $k$ values of $t$ for which $\ell_{1}=\ell_{2}$, then the intersection $\ell_{1} \cap \ell_{2}$ has degree at most $d_{1}+d_{2}-k$. + +Now, note that $H$ moves linearly in $C$ on line $B H$. Furthermore, angles $\angle A H E$, $\angle A H F$ are fixed, we get that $D$ and $E$ have degree 2 . One way to see this is using the lemma; $D$ lies on line $A B$, which is fixed, and line $H D$ passes through a point at infinity which is a constant rotation of the point at infinity on line $A H$, and therefore has degree 1. Then $D, E$ have degree at most $1+1-0=2$. + +Now, note that $P, Q$ move linearly in $C$. Both of these are because the circumcenter $O$ moves linearly in $C$, and $P, Q$ are reflections of $B, C$ in a line through $O$ with fixed direction, which also moves linearly. + +So by the lemma, the lines $P D, Q E$ have degree at most 3 . I claim they actually have degree 2; to show this it suffices to give an example of a choice of $C$ for which $P=D$ and one for which $Q=E$. But an easy angle chase shows that in the unique case when $P=B$, we get $D=B$ as well and thus $P=D$. Similarly when $Q=C, E=C$. It follows from the lemma that lines $P D, Q E$ have degree at most 2 . + +Let $\ell_{\infty}$ denote the line at infinity. I claim that the points $P_{1}=P D \cap \ell_{\infty}, P_{2}=Q E \cap \ell_{\infty}$ are projective in $C$. Since $\ell_{\infty}$ is fixed, it suffices to show by the lemma that there exists some value of $C$ for which $Q E=\ell_{\infty}$ and $P D=\ell_{\infty}$. But note that as $C \rightarrow \infty$, all four points $P, D, Q, E$ go to infinity. It follows that $P_{1}, P_{2}$ are projective in $C$. + +Then to finish, recall that we want to show that $\angle(P D, Q E)$ is constant. It suffices then to show that there's a constant rotation sending $P_{1}$ to $P_{2}$. Since $P_{1}, P_{2}$ are projective, it suffices to verify this for 3 values of $C$. + +We can take $C$ such that $\angle A B C=90, \angle A C B=90$, or $A B=A C$, and all three cases are easy to check. + +## §1.3 TSTST 2019/3, proposed by Nikolai Beluhov + +Available online at https://aops.com/community/p12608769. + +## Problem statement + +On an infinite square grid we place finitely many cars, which each occupy a single cell and face in one of the four cardinal directions. Cars may never occupy the same cell. It is given that the cell immediately in front of each car is empty, and moreover no two cars face towards each other (no right-facing car is to the left of a left-facing car within a row, etc.). In a move, one chooses a car and shifts it one cell forward to a vacant cell. Prove that there exists an infinite sequence of valid moves using each car infinitely many times. + +Let $S$ be any rectangle containing all the cars. Partition $S$ into horizontal strips of height 1 , and color them red and green in an alternating fashion. It is enough to prove all the cars may exit $S$. +![](https://cdn.mathpix.com/cropped/2024_11_19_f4f672a97b66743213a8g-09.jpg?height=880&width=1038&top_left_y=1096&top_left_x=512) + +To do so, we outline a five-stage plan for the cars. + +1. All vertical cars in a green cell may advance one cell into a red cell (or exit $S$ altogether), by the given condition. (This is the only place where the hypothesis about empty space is used!) +2. All horizontal cars on green cells may exit $S$, as no vertical cars occupy green cells. +3. All vertical cars in a red cell may advance one cell into a green cell (or exit $S$ altogether), as all green cells are empty. +4. All horizontal cars within red cells may exit $S$, as no vertical car occupy red cells. +5. The remaining cars exit $S$, as they are all vertical. The solution is complete. + +Remark (Author's comments). The solution I've given for this problem is so short and simple that it might appear at first to be about IMO 1 difficulty. I don't believe that's true! There are very many approaches that look perfectly plausible at first, and then fall apart in this or that twisted special case. + +Remark (Higher-dimensional generalization by author). The natural higher-dimensional generalization is true, and can be proved in largely the same fashion. For example, in three dimensions, one may let $S$ be a rectangular prism and partition $S$ into horizontal slabs and color them red and green in an alternating fashion. Stages 1, 3, and 5 generalize immediately, and stages 2 and 4 reduce to an application of the two-dimensional problem. In the same way, the general problem is handled by induction on the dimension. + +Remark (Historical comments). For $k>1$, we could consider a variant of the problem where cars are $1 \times k$ rectangles (moving parallel to the longer edge) instead of occupying single cells. In that case, if there are $2 k-1$ empty spaces in front of each car, the above proof works (with the red and green strips having height $k$ instead). On the other hand, at least $k$ empty spaces are necessary. We don't know the best constant in this case. + +## §2 Solutions to Day 2 + +## §2.1 TSTST 2019/4, proposed by Merlijn Staps + +Available online at https://aops.com/community/p12608513. + +## Problem statement + +Consider coins with positive real denominations not exceeding 1. Find the smallest $C>0$ such that the following holds: if we are given any 100 such coins with total value 50 , then we can always split them into two stacks of 50 coins each such that the absolute difference between the total values of the two stacks is at most $C$. + +The answer is $C=\frac{50}{51}$. The lower bound is obtained if we have 51 coins of value $\frac{1}{51}$ and 49 coins of value 1. (Alternatively, 51 coins of value $1-\frac{\varepsilon}{51}$ and 49 coins of value $\frac{\varepsilon}{49}$ works fine for $\varepsilon>0$.) We now present two (similar) proofs that this $C=\frac{50}{51}$ suffices. + +【 First proof (original). Let $a_{1} \leq \cdots \leq a_{100}$ denote the values of the coins in ascending order. Since the 51 coins $a_{50}, \ldots, a_{100}$ are worth at least $51 a_{50}$, it follows that $a_{50} \leq \frac{50}{51}$; likewise $a_{51} \geq \frac{1}{51}$. + +We claim that choosing the stacks with coin values + +$$ +a_{1}, a_{3}, \ldots, a_{49}, \quad a_{52}, a_{54}, \ldots, a_{100} +$$ + +and + +$$ +a_{2}, a_{4}, \ldots, a_{50}, \quad a_{51}, a_{53}, \ldots, a_{99} +$$ + +works. Let $D$ denote the (possibly negative) difference between the two total values. Then + +$$ +\begin{aligned} +D & =\left(a_{1}-a_{2}\right)+\cdots+\left(a_{49}-a_{50}\right)-a_{51}+\left(a_{52}-a_{53}\right)+\cdots+\left(a_{98}-a_{99}\right)+a_{100} \\ +& \leq 25 \cdot 0-\frac{1}{51}+24 \cdot 0+1=\frac{50}{51} +\end{aligned} +$$ + +Similarly, we have + +$$ +\begin{aligned} +D & =a_{1}+\left(a_{3}-a_{2}\right)+\cdots+\left(a_{49}-a_{48}\right)-a_{50}+\left(a_{52}-a_{51}\right)+\cdots+\left(a_{100}-a_{99}\right) \\ +& \geq 0+24 \cdot 0-\frac{50}{51}+25 \cdot 0=-\frac{50}{51} +\end{aligned} +$$ + +It follows that $|D| \leq \frac{50}{51}$, as required. + +【 Second proof (Evan Chen). Again we sort the coins in increasing order $0a_{i-1}+\frac{50}{51}$; obviously there is at most one such large gap. + +Claim - If there is a large gap, it must be $a_{51}>a_{50}+\frac{50}{51}$. + +Proof. If $i<50$ then we get $a_{50}, \ldots, a_{100}>\frac{50}{51}$ and the sum $\sum_{1}^{100} a_{i}>50$ is too large. Conversely if $i>50$ then we get $a_{1}, \ldots, a_{i-1}<\frac{1}{51}$ and the sum $\sum_{1}^{100} a_{i}<1 / 51 \cdot 51+49$ is too small. + +Now imagine starting with the coins $a_{1}, a_{3}, \ldots, a_{99}$, which have total value $S \leq 25$. We replace $a_{1}$ by $a_{2}$, then $a_{3}$ by $a_{4}$, and so on, until we replace $a_{99}$ by $a_{100}$. At the end of the process we have $S \geq 25$. Moreover, since we did not cross a large gap at any point, the quantity $S$ changed by at most $C=\frac{50}{51}$ at each step. So at some point in the process we need to have $25-C / 2 \leq S \leq 25+C / 2$, which proves $C$ works. + +## §2.2 TSTST 2019/5, proposed by Gunmay Handa + +Available online at https://aops.com/community/p12608496. + +## Problem statement + +Let $A B C$ be an acute triangle with orthocenter $H$ and circumcircle $\Gamma$. A line through $H$ intersects segments $A B$ and $A C$ at $E$ and $F$, respectively. Let $K$ be the circumcenter of $\triangle A E F$, and suppose line $A K$ intersects $\Gamma$ again at a point $D$. Prove that line $H K$ and the line through $D$ perpendicular to $\overline{B C}$ meet on $\Gamma$. + +We present several solutions. (There are more in the official packet; some are omitted here, which explains the numbering.) + +『 First solution (Andrew Gu). We begin with the following two observations. +Claim - Point $K$ lies on the radical axis of $(B E H)$ and $(C F H)$. +Proof. Actually we claim $\overline{K E}$ and $\overline{K F}$ are tangents. Indeed, + +$$ +\measuredangle H E K=90^{\circ}-\measuredangle E A F=90^{\circ}-\measuredangle B A C=\measuredangle H B E +$$ + +implying the result. Since $K E=K F$, this implies the result. + +Claim - The second intersection $M$ of $(B E H)$ and $(C F H)$ lies on $\Gamma$. +Proof. By Miquel's theorem on $\triangle A E F$ with $H \in \overline{E F}, B \in \overline{A E}, C \in \overline{A F}$. +![](https://cdn.mathpix.com/cropped/2024_11_19_f4f672a97b66743213a8g-13.jpg?height=806&width=632&top_left_y=1613&top_left_x=712) + +In particular, $M, H, K$ are collinear. Let $X$ be on $\Gamma$ with $\overline{D X} \perp \overline{B C}$; we then wish to show $X$ lies on the line $M H K$ we found. This is angle chasing: compute + +$$ +\measuredangle X M B=\measuredangle X D B=90^{\circ}-\measuredangle D B C=90^{\circ}-\measuredangle D A C +$$ + +$$ +=90^{\circ}-\measuredangle K A F=\measuredangle F E A=\measuredangle H E B=\measuredangle H M B +$$ + +as needed. + +II Second solution (Ankan Bhattacharya). We let $D^{\prime}$ be the second intersection of $\overline{E F}$ with $(B H C)$ and redefine $D$ as the reflection of $D^{\prime}$ across $\overline{B C}$. We will first prove that this point $D$ coincides with the point $D$ given in the problem statement. The idea is that: + +Claim - $A$ is the $D$-excenter of $\triangle D E F$. + +Proof. We contend $B E D^{\prime} D$ is cyclic. This follows by angle chasing: + +$$ +\begin{aligned} +\measuredangle D^{\prime} D B & =\measuredangle B D^{\prime} D=\measuredangle D^{\prime} B C+90^{\circ}=\measuredangle D^{\prime} H C+90^{\circ} \\ +& =\measuredangle D^{\prime} H C+\measuredangle(H C, A B)=\measuredangle\left(D^{\prime} H, A B\right)=\measuredangle D^{\prime} E B +\end{aligned} +$$ + +Now as $B D=B D^{\prime}$, we obtain $\overline{B E A}$ externally bisects $\angle D E D^{\prime} \cong \angle D E F$. Likewise $\overline{F A}$ externally bisects $\angle D F E$, so $A$ is the $D$-excenter of $\triangle D E F$. + +Hence, by the so-called "Fact 5 ", point $K$ lies on $\overline{D A}$, so this point $D$ is the one given in the problem statement. +![](https://cdn.mathpix.com/cropped/2024_11_19_f4f672a97b66743213a8g-14.jpg?height=778&width=712&top_left_y=1256&top_left_x=678) + +Now choose point $X$ on $(A B C)$ satisfying $\overline{D X} \perp \overline{B C}$. +Claim - Point $K$ lies on line $H X$. + +Proof. Clearly $A H D^{\prime} X$ is a parallelogram. By Ptolemy on $D E K F$, + +$$ +\frac{K D}{K A}=\frac{K D}{K E}=\frac{D E+D F}{E F} +$$ + +On the other hand, if we let $r_{D}$ denote the $D$-exradius of $\triangle D E F$ then + +$$ +\frac{X D}{X D^{\prime}}=\frac{[D E X]+[D F X]}{[X E F]}=\frac{[D E X]+[D F X]}{[A E F]}=\frac{D E \cdot r_{D}+D F \cdot r_{D}}{E F \cdot r_{D}}=\frac{D E+D F}{E F} +$$ + +Thus + +$$ +[A K X]=\frac{K A}{K D} \cdot[D K X]=\frac{K A}{K D} \cdot \frac{X D}{X D^{\prime}} \cdot\left[K D^{\prime} X\right]=\left[D^{\prime} K X\right] +$$ + +This is sufficient to prove $K$ lies on $\overline{H X}$. +The solution is complete: $X$ is the desired concurrency point. + +I Fourth solution, complex numbers with spiral similarity (Evan Chen). First if $\overline{A D} \perp \overline{B C}$ there is nothing to prove, so we assume this is not the case. Let $W$ be the antipode of $D$. Let $S$ denote the second intersection of $(A E F)$ and $(A B C)$. Consider the spiral similarity sending $\triangle S E F$ to $\triangle S B C$ : + +- It maps $H$ to a point $G$ on line $B C$, +- It maps $K$ to $O$. +- It maps the $A$-antipode of $\triangle A E F$ to $D$. +- Hence (by previous two observations) it maps $A$ to $W$. +- Also, the image of line $A D$ is line $W O$, which does not coincide with line $B C$ (as $O$ does not lie on line $B C$ ). + +Therefore, $K$ is the unique point on line $\overline{A D}$ for one can get a direct similarity + +$$ +\triangle A K H \sim \triangle W O G +$$ + +for some point $G$ lying on line $\overline{B C}$. +![](https://cdn.mathpix.com/cropped/2024_11_19_f4f672a97b66743213a8g-15.jpg?height=804&width=775&top_left_y=1477&top_left_x=646) + +On the other hand, let us re-define $K$ as $\overline{X H} \cap \overline{A D}$. We will show that the corresponding $G$ making $(\triangle)$ true lies on line $B C$. + +We apply complex numbers with $\Gamma$ the unit circle, with $a, b, c, d$ taking their usual meanings, $H=a+b+c, X=-b c / d$, and $W=-d$. Then point $K$ is supposed to satisfy + +$$ +k+a d \bar{k}=a+d +$$ + +$$ +\begin{aligned} +\frac{k+\frac{b c}{d}}{a+b+c+\frac{b c}{d}} & =\frac{\bar{k}+\frac{d}{b c}}{\frac{1}{a}+\frac{1}{b}+\frac{1}{c}+\frac{d}{b c}} \\ +\Longleftrightarrow \frac{\frac{1}{a}+\frac{1}{b}+\frac{1}{c}+\frac{d}{b c}}{a+b+c+\frac{b c}{d}}\left(k+\frac{b c}{d}\right) & =\bar{k}+\frac{d}{b c} +\end{aligned} +$$ + +Adding $a d$ times the last line to the first line and cancelling $a d \bar{k}$ now gives + +$$ +\left(a d \cdot \frac{\frac{1}{a}+\frac{1}{b}+\frac{1}{c}+\frac{d}{b c}}{a+b+c+\frac{b c}{d}}+1\right) k=a+d+\frac{a d^{2}}{b c}-a b c \cdot \frac{\frac{1}{a}+\frac{1}{b}+\frac{1}{c}+\frac{d}{b c}}{a+b+c+\frac{b c}{d}} +$$ + +or + +$$ +\begin{aligned} +\left(a d\left(\frac{1}{a}+\frac{1}{b}+\frac{1}{c}+\frac{d}{b c}\right)+a+b+c+\frac{b c}{d}\right) k & =\left(a+b+c+\frac{b c}{d}\right)\left(a+d+\frac{a d^{2}}{b c}\right) \\ +& -a b c \cdot\left(\frac{1}{a}+\frac{1}{b}+\frac{1}{c}+\frac{d}{b c}\right) . +\end{aligned} +$$ + +We begin by simplifying the coefficient of $k$ : + +$$ +\begin{aligned} +a d\left(\frac{1}{a}+\frac{1}{b}+\frac{1}{c}+\frac{d}{b c}\right)+a+b+c+\frac{b c}{d} & =a+b+c+d+\frac{b c}{d}+\frac{a d}{b}+\frac{a d}{c}+\frac{a d^{2}}{b c} \\ +& =a+\frac{b c}{d}+\left(1+\frac{a d}{b c}\right)(b+c+d) \\ +& =\frac{a d+b c}{b c d}[b c+d(b+c+d)] \\ +& =\frac{(a d+b c)(d+b)(d+c)}{b c d} +\end{aligned} +$$ + +Meanwhile, the right-hand side expands to + +$$ +\begin{aligned} +\mathrm{RHS}= & \left(a+b+c+\frac{b c}{d}\right)\left(a+d+\frac{a d^{2}}{b c}\right)-a b c \cdot\left(\frac{1}{a}+\frac{1}{b}+\frac{1}{c}+\frac{d}{b c}\right) \\ += & \left(a^{2}+a b+a c+\frac{a b c}{d}\right)+(d a+d b+d c+b c) \\ +& +\left(\frac{a^{2} d^{2}}{b c}+\frac{a d^{2}}{c}+\frac{a d^{2}}{b}+a d\right)-(a b+b c+c a+a d) \\ += & a^{2}+d(a+b+c)+\frac{a b c}{d}+\frac{a^{2} d^{2}}{b c}+\frac{a d^{2}}{b}+\frac{a d^{2}}{c} \\ += & a^{2}+\frac{a b c}{d}+d(a+b+c) \cdot \frac{a d+b c}{b c} \\ += & \frac{a d+b c}{b c d}\left[a b c+d^{2}(a+b+c)\right] . +\end{aligned} +$$ + +Therefore, we get + +$$ +k=\frac{a b c+d^{2}(a+b+c)}{(d+b)(d+c)} +$$ + +In particular, + +$$ +\begin{aligned} +k-a & =\frac{a b c+d^{2}(a+b+c)-a(d+b)(d+c)}{(d+b)(d+c)} \\ +& =\frac{d^{2}(b+c)-d a(b+c)}{(d+b)(d+c)}=\frac{d(b+c)(d-a)}{(d+b)(d+c)} +\end{aligned} +$$ + +Now the corresponding point $G$ obeying $(\Omega)$ satisfies + +$$ +\begin{aligned} +\frac{g-(-d)}{0-(-d)} & =\frac{(a+b+c)-a}{k-a} \\ +\Longrightarrow g & =-d+\frac{d(b+c)}{k-a} \\ +& =-d+\frac{(d+b)(d+c)}{d-a}=\frac{d b+d c+b c+a d}{d-a} . \\ +\Longrightarrow b c \bar{g} & =\frac{b c \cdot \frac{a c+a b+a d+b c}{a b c d}}{\frac{a-d}{a d}}=-\frac{a b+a c+a d+b c}{d-a} . \\ +\Longrightarrow g+b c \bar{g} & =\frac{(d-a)(b+c)}{d-a}=b+c . +\end{aligned} +$$ + +Hence $G$ lies on $B C$ and this completes the proof. + +【 Seventh solution using moving points (Zack Chroman). We state the converse of the problem as follows: + +Take a point $D$ on $\Gamma$, and let $G \in \Gamma$ such that $\overline{D G} \perp \overline{B C}$. Then define $K$ to lie on $\overline{G H}, \overline{A D}$, and take $L \in \overline{A D}$ such that $K$ is the midpoint of $\overline{A L}$. Then if we define $E$ and $F$ as the projections of $L$ onto $\overline{A B}$ and $\overline{A C}$ we want to show that $E, H, F$ are collinear. + +It's clear that solving this problem will solve the original. In fact we will show later that each line $E F$ through $H$ corresponds bijectively to the point $D$. + +We animate $D$ projectively on $\Gamma$ (hence $\operatorname{deg} D=2$ ). Since $D \mapsto G$ is a projective map $\Gamma \rightarrow \Gamma$, it follows $\operatorname{deg} G=2$. By Zack's lemma, $\operatorname{deg}(\overline{A D}) \leq 0+2-1=1$ (since $D$ can coincide with $A$ ), and $\operatorname{deg}(\overline{H G}) \leq 0+2-0=2$. So again by Zack's lemma, $\operatorname{deg} K \leq 1+2-1=2$, since lines $A D$ and $G H$ can coincide once if $D$ is the reflection of $H$ over $\overline{B C}$. It follows $\operatorname{deg} L=2$, since it is obtained by dilating $K$ by a factor of 2 across the fixed point $A$. + +Let $\infty_{C}$ be the point at infinity on the line perpendicular to $A C$, and similarly $\infty_{B}$. Then + +$$ +F=\overline{A C} \cap \overline{\infty_{C} L}, \quad E=\overline{A B} \cap \overline{\infty_{B} L} . +$$ + +We want to use Zack's lemma again on line $\overline{\infty_{B} L}$. Consider the case $G=B$; we get $\overline{H G} \| \overline{A D}$, so $A D G H$ is a parallelogram, and then $K=L=\infty_{B}$. Thus there is at least one $t$ where $L=\infty_{B}$ and by Zack's lemma we get $\operatorname{deg}\left(\overline{\infty_{B} L}\right) \leq 0+2-1=1$. Again by Zack's lemma, we conclude $\operatorname{deg} E \leq 0+1-0=1$. Similarly, $\operatorname{deg} F \leq 1$. + +We were aiming to show $E, F, H$ collinear which is a condition of degree at most $1+1+0=2$. So it suffices to verify the problem for three distinct choices of $D$. + +- If $D=A$, then line $G H$ is line $A H$, and $L=\overline{A D} \cap \overline{A H}=A$. So $E=F=A$ and the statement is true. +- If $D=B, G$ is the antipode of $C$ on $\Gamma$. Then $K=\overline{H G} \cap \overline{A D}$ is the midpoint of $\overline{A B}$, so $L=B$. Then $E=B$ and $F$ is the projection of $B$ onto $A C$, so $E, H, F$ collinear. +- We finish similarly when $D=C$. + +This completes the proof. + +Remark. Less careful approaches are possible which give a worse bound on the degrees, requiring to check (say) five choices of $D$ instead. We present the most careful one showing $\operatorname{deg} D=2$ for instructional reasons, but the others may be easier to find. + +## §2.3 TSTST 2019/6, proposed by Nikolai Beluhov + +Available online at https://aops.com/community/p12608536. + +## Problem statement + +Suppose $P$ is a polynomial with integer coefficients such that for every positive integer $n$, the sum of the decimal digits of $|P(n)|$ is not a Fibonacci number. Must $P$ be constant? + +The answer is yes, $P$ must be constant. By $S(n)$ we mean the sum of the decimal digits of $|n|$. + +We need two claims. +Claim - If $P(x) \in \mathbb{Z}[x]$ is nonconstant with positive leading coefficient, then there exists an integer polynomial $F(x)$ such that all coefficients of $P \circ F$ are positive except for the second one, which is negative. + +Proof. We will actually construct a cubic $F$. We call a polynomial good if it has the property. + +First, consider $T_{0}(x)=x^{3}+x+1$. Observe that in $T_{0}^{\operatorname{deg} P}$, every coefficient is strictly positive, except for the second one, which is zero. + +Then, let $T_{1}(x)=x^{3}-\frac{1}{D} x^{2}+x+1$. Using continuity as $D \rightarrow \infty$, it follows that if $D$ is large enough (in terms of $\operatorname{deg} P$ ), then $T_{1}^{\operatorname{deg} P}$ is good, with $-\frac{3}{D} x^{3 \operatorname{deg} P-1}$ being the only negative coefficient. + +Finally, we can let $F(x)=C T_{1}(x)$ where $C$ is a sufficiently large multiple of $D$ (in terms of the coefficients of $P$ ); thus the coefficients of $\left(C T_{1}(x)\right)^{\operatorname{deg} P}$ dominate (and are integers), as needed. + +Claim - There are infinitely many Fibonacci numbers in each residue class modulo 9. + +Proof. Note the Fibonacci sequence is periodic modulo 9 (indeed it is periodic modulo any integer). Moreover (allowing negative indices), + +$$ +\begin{aligned} +F_{0}=0 & \equiv 0 \quad(\bmod 9) \\ +F_{1}=1 & \equiv 1 \quad(\bmod 9) \\ +F_{3}=2 & \equiv 2 \quad(\bmod 9) \\ +F_{4}=3 & \equiv 3 \quad(\bmod 9) \\ +F_{7}=13 & \equiv 4 \quad(\bmod 9) \\ +F_{5}=5 & \equiv 5 \quad(\bmod 9) \\ +F_{-4}=-3 & \equiv 6 \quad(\bmod 9) \\ +F_{9}=34 & \equiv 7 \quad(\bmod 9) \\ +F_{6}=8 & \equiv 8 \quad(\bmod 9) . +\end{aligned} +$$ + +We now show how to solve the problem with the two claims. WLOG $P$ satisfies the conditions of the first claim, and choose $F$ as above. Let + +$$ +P(F(x))=c_{N} x^{N}-c_{N-1} x^{N-1}+c_{N-2} x^{N-2}+\cdots+c_{0} +$$ + +where $c_{i}>0$ (and $N=3 \operatorname{deg} P$ ). Then if we select $x=10^{e}$ for $e$ large enough (say $\left.x>10 \max _{i} c_{i}\right)$, the decimal representation $P\left(F\left(10^{e}\right)\right)$ consists of the concatenation of + +- the decimal representation of $c_{N}-1$, +- the decimal representation of $10^{e}-c_{N-1}$ +- the decimal representation of $c_{N-2}$, with several leading zeros, +- the decimal representation of $c_{N-3}$, with several leading zeros, +- ... +- the decimal representation of $c_{0}$, with several leading zeros. +(For example, if $P(F(x))=15 x^{3}-7 x^{2}+4 x+19$, then $P(F(1000))=14,993,004,019$.) Thus, the sum of the digits of this expression is equal to + +$$ +S\left(P\left(F\left(10^{e}\right)\right)\right)=9 e+k +$$ + +for some constant $k$ depending only on $P$ and $F$, independent of $e$. But this will eventually hit a Fibonacci number by the second claim, contradiction. + +Remark. It is important to control the number of negative coefficients in the created polynomial. If one tries to use this approach on a polynomial $P$ with $m>0$ negative coefficients, then one would require that the Fibonacci sequence is surjective modulo $9 m$ for any $m>1$, which is not true: for example the Fibonacci sequence avoids all numbers congruent to $4 \bmod 11($ and thus $4 \bmod 99)$. + +In bases $b$ for which surjectivity modulo $b-1$ fails, the problem is false. For example, $P(x)=11 x+4$ will avoid all Fibonacci numbers if we take sum of digits in base 12, since that base-12 sum is necessarily $4(\bmod 11)$, hence not a Fibonacci number. + +## §3 Solutions to Day 3 + +## §3.1 TSTST 2019/7, proposed by Ankan Bhattacharya + +Available online at https://aops.com/community/p12608512. + +## Problem statement + +Let $f: \mathbb{Z} \rightarrow\left\{1,2, \ldots, 10^{100}\right\}$ be a function satisfying + +$$ +\operatorname{gcd}(f(x), f(y))=\operatorname{gcd}(f(x), x-y) +$$ + +for all integers $x$ and $y$. Show that there exist positive integers $m$ and $n$ such that $f(x)=\operatorname{gcd}(m+x, n)$ for all integers $x$. + +Let $\mathcal{P}$ be the set of primes not exceeding $10^{100}$. For each $p \in \mathcal{P}$, let $e_{p}=\max _{x} \nu_{p}(f(x))$ and let $c_{p} \in \operatorname{argmax}_{x} \nu_{p}(f(x))$. + +We show that this is good enough to compute all values of $x$, by looking at the exponent at each individual prime. + +Claim - For any $p \in \mathcal{P}$, we have + +$$ +\nu_{p}(f(x))=\min \left(\nu_{p}\left(x-c_{p}\right), e_{p}\right) +$$ + +Proof. Note that for any $x$, we have + +$$ +\operatorname{gcd}\left(f\left(c_{p}\right), f(x)\right)=\operatorname{gcd}\left(f\left(c_{p}\right), x-c_{p}\right) +$$ + +We then take $\nu_{p}$ of both sides and recall $\nu_{p}(f(x)) \leq \nu_{p}\left(f\left(c_{p}\right)\right)=e_{p}$; this implies the result. + +This essentially determines $f$, and so now we just follow through. Choose $n$ and $m$ such that + +$$ +\begin{aligned} +n & =\prod_{p \in \mathcal{P}} p^{e_{p}} \\ +m & \equiv-c_{p} \quad\left(\bmod p^{e_{p}}\right) \quad \forall p \in \mathcal{P} +\end{aligned} +$$ + +the latter being possible by Chinese remainder theorem. Then, from the claim we have + +$$ +\begin{aligned} +f(x) & =\prod_{p \in \mathcal{P}} p^{\nu_{p}(f(x))}=\prod_{p \mid n} p^{\min \left(\nu_{p}\left(x-c_{p}\right), e_{p}\right)} \\ +& =\prod_{p \mid n} p^{\min \left(\nu_{p}(x+m), \nu_{p}(n)\right)}=\operatorname{gcd}(x+m, n) +\end{aligned} +$$ + +for every $x \in \mathbb{Z}$, as desired. +Remark. The functions $f(x)=x$ and $f(x)=|2 x-1|$ are examples satisfying the gcd equation (the latter always being strictly positive). Hence the hypothesis $f$ bounded cannot be dropped. + +Remark. The pair $(m, n)$ is essentially unique: every other pair is obtained by shifting $m$ by a multiple of $n$. Hence there is not really any choice in choosing $m$ and $n$. + +## §3.2 TSTST 2019/8, proposed by Ankan Bhattacharya + +Available online at https://aops.com/community/p12608780. + +## Problem statement + +Let $\mathcal{S}$ be a set of 16 points in the plane, no three collinear. Let $\chi(\mathcal{S})$ denote the number of ways to draw 8 line segments with endpoints in $\mathcal{S}$, such that no two drawn segments intersect, even at endpoints. Find the smallest possible value of $\chi(\mathcal{S})$ across all such $\mathcal{S}$. + +The answer is 1430 . In general, we prove that with $2 n$ points the answer is the $n^{\text {th }}$ Catalan number $C_{n}=\frac{1}{n+1}\binom{2 n}{n}$. + +First of all, it is well-known that if $\mathcal{S}$ is a convex $2 n$-gon, then $\chi(\mathcal{S})=C_{n}$. +It remains to prove the lower bound. We proceed by (strong) induction on $n$, with the base case $n=0$ and $n=1$ clear. Suppose the statement is proven for $0,1, \ldots, n$ and consider a set $\mathcal{S}$ with $2(n+1)$ points. + +Let $P$ be a point on the convex hull of $\mathcal{S}$, and label the other $2 n+1$ points $A_{1}, \ldots, A_{2 n+1}$ in order of angle from $P$. + +Consider drawing a segment $\overline{P A_{2 k+1}}$. This splits the $2 n$ remaining points into two halves $\mathcal{U}$ and $\mathcal{V}$, with $2 k$ and $2(n-k)$ points respectively. +![](https://cdn.mathpix.com/cropped/2024_11_19_f4f672a97b66743213a8g-23.jpg?height=581&width=815&top_left_y=1297&top_left_x=626) + +Note that by choice of $P$, no segment in $\mathcal{U}$ can intersect a segment in $\mathcal{V}$. By the inductive hypothesis, + +$$ +\chi(\mathcal{U}) \geq C_{k} \quad \text { and } \quad \chi(\mathcal{V}) \geq C_{n-k} +$$ + +Thus, drawing $\overline{P A_{2 k+1}}$, we have at least $C_{k} C_{n-k}$ ways to complete the drawing. Over all choices of $k$, we obtain + +$$ +\chi(\mathcal{S}) \geq C_{0} C_{n}+\cdots+C_{n} C_{0}=C_{n+1} +$$ + +as desired. +Remark. It is possible to show directly from the lower bound proof that convex $2 n$-gons achieve the minimum: indeed, every inequality is sharp, and no segment $\overline{P A_{2 k}}$ can be drawn (since this splits the rest of the points into two halves with an odd number of points, and no crossing segment can be drawn). + +Bobby Shen points out that in the case of 6 points, a regular pentagon with its center also achieves equality, so this is not the only equality case. + +Remark. The result that $\chi(S) \geq 1$ for all $S$ is known (consider the choice of 8 segments with smallest sum), and appeared on Putnam 1979. However, it does not seem that knowing this gives an advantage for this problem, since the answer is much larger than 1. + +## §3.3 TSTST 2019/9, proposed by Ankan Bhattacharya + +Available online at https://aops.com/community/p12608472. + +## Problem statement + +Let $A B C$ be a triangle with incenter $I$. Points $K$ and $L$ are chosen on segment $B C$ such that the incircles of $\triangle A B K$ and $\triangle A B L$ are tangent at $P$, and the incircles of $\triangle A C K$ and $\triangle A C L$ are tangent at $Q$. Prove that $I P=I Q$. + +We present two solutions. +【 First solution, mostly elementary (original). Let $I_{B}, J_{B}, I_{C}, J_{C}$ be the incenters of $\triangle A B K, \triangle A B L, \triangle A C K, \triangle A C L$ respectively. +![](https://cdn.mathpix.com/cropped/2024_11_19_f4f672a97b66743213a8g-25.jpg?height=515&width=1332&top_left_y=973&top_left_x=359) + +We begin with the following claim which does not depend on the existence of tangency points $P$ and $Q$. + +Claim - Lines $B C, I_{B} J_{C}, J_{B} I_{C}$ meet at a point $R$ (possibly at infinity). + +Proof. By rotating by $\frac{1}{2} \angle A$ we have the equality + +$$ +A\left(B I ; I_{B} J_{B}\right)=A\left(I C ; I_{C} J_{C}\right) . +$$ + +It follows $\left(B I ; I_{B} J_{B}\right)=\left(I C ; I_{C} J_{C}\right)=\left(C I ; J_{C} I_{C}\right)$. (One could also check directly that both cross ratios equal $\frac{\sin \angle B A K / 2}{\sin \angle C A K / 2} \div \frac{\sin \angle B A L / 2}{\sin \angle C A L / 2}$, rather than using rotation.) + +Therefore, the concurrence follows from the so-called prism lemma on $\overline{I B I_{B} J_{B}}$ and $\overline{I C J_{C} I_{C}}$. + +Remark (Nikolai Beluhov). This result is known; it appears as 4.5.32 in Akopyan's Geometry in Figures. The cross ratio is not necessary to prove this claim: it can be proven by length chasing with circumscribed quadrilaterals. (The generalization mentioned later also admits a trig-free proof for the analogous step.) + +We now bring $P$ and $Q$ into the problem. +Claim - Line $P Q$ also passes through $R$. + +Proof. Note $\left(B P ; I_{B} J_{B}\right)=-1=\left(C Q ; J_{C} I_{C}\right)$, so the conclusion again follows by prism lemma. + +We are now ready to complete the proof. Point $R$ is the exsimilicenter of the incircles of $\triangle A B K$ and $\triangle A C L$, so $\frac{P I_{B}}{R I_{B}}=\frac{Q J_{C}}{R J_{C}}$. Now by Menelaus, + +$$ +\frac{I_{B} P}{P I} \cdot \frac{I Q}{Q J_{C}} \cdot \frac{J_{C} R}{R I_{B}}=-1 \Longrightarrow I P=I Q +$$ + +Remark (Author's comments on drawing the diagram). Drawing the diagram directly is quite difficult. If one draws $\triangle A B C$ first, they must locate both $K$ and $L$, which likely involves some trial and error due to the complex interplay between the two points. + +There are alternative simpler ways. For example, one may draw $\triangle A K L$ first; then the remaining points $B$ and $C$ are not related and the task is much simpler (though some trial and error is still required). + +In fact, by breaking symmetry, we may only require one application of guesswork. Start by drawing $\triangle A B K$ and its incircle; then the incircle of $\triangle A B L$ may be constructed, and so point $L$ may be drawn. Thus only the location of point $C$ needs to be guessed. I would be interested in a method to create a general diagram without any trial and error. + +【 Second solution, inversion (Nikolai Beluhov). As above, the lines $B C, I_{B} J_{C}, J_{B} I_{C}$ meet at some point $R$ (possibly at infinity). Let $\omega_{1}, \omega_{2}, \omega_{3}, \omega_{4}$ be the incircles of $\triangle A B K$, $\triangle A C L, \triangle A B L$, and $\triangle A C K$. + +Claim - There exists an inversion $\iota$ at $R$ swapping $\left\{\omega_{1}, \omega_{2}\right\}$ and $\left\{\omega_{3}, \omega_{4}\right\}$. + +Proof. Consider the inversion at $R$ swapping $\omega_{1}$ and $\omega_{2}$. Since $\omega_{1}$ and $\omega_{3}$ are tangent, the image of $\omega_{3}$ is tangent to $\omega_{2}$ and is also tangent to $B C$. The circle $\omega_{4}$ is on the correct side of $\omega_{3}$ to be this image. + +Claim - Circles $\omega_{1}, \omega_{2}, \omega_{3}, \omega_{4}$ share a common radical center. + +Proof. Let $\Omega$ be the circle with center $R$ fixed under $\iota$, and let $k$ be the circle through $P$ centered at the radical center of $\Omega, \omega_{1}, \omega_{3}$. + +Then $k$ is actually orthogonal to $\Omega, \omega_{1}, \omega_{3}$, so $k$ is fixed under $\iota$ and $k$ is also orthogonal to $\omega_{2}$ and $\omega_{4}$. Thus the center of $k$ is the desired radical center. + +The desired statement immediately follows. Indeed, letting $S$ be the radical center, it follows that $\overline{S P}$ and $\overline{S Q}$ are the common internal tangents to $\left\{\omega_{1}, \omega_{3}\right\}$ and $\left\{\omega_{2}, \omega_{4}\right\}$. + +Since $S$ is the radical center, $S P=S Q$. In light of $\angle S P I=\angle S Q I=90^{\circ}$, it follows that $I P=I Q$, as desired. + +Remark (Nikolai Beluhov). There exists a circle tangent to all four incircles, because circle $k$ is orthogonal to all four, and line $B C$ is tangent to all four; thus the inverse of line $B C$ in $k$ is a circle tangent to all four incircles. + +The amusing thing here is that Casey's theorem is completely unhelpful for proving this fact: all it can tell us is that there is a line or circle tangent to these incircles, and line $B C$ already satisfies this property. + +Remark (Generalization by Nikolai Beluhov). The following generalization holds: +Let $A B C D$ be a quadrilateral circumscribed about a circle with center $I$. A line through $A$ meets $\overrightarrow{B C}$ and $\overrightarrow{D C}$ at $K$ and $L$; another line through $A$ meets $\overrightarrow{B C}$ and $\overrightarrow{D C}$ at $M$ and $N$. Suppose that the incircles of $\triangle A B K$ and $\triangle A B M$ are tangent at $P$, and the incircles of $\triangle A C L$ and $\triangle A C N$ are tangent at $Q$. Prove that $I P=I Q$. + +The first approach can be modified to the generalization. There is an extra initial step required: by Monge, the exsimilicenter of the incircles of $\triangle A B K$ and $\triangle A D N$ lies on line $B D$; likewise for the incircles of $\triangle A B L$ and $\triangle A D M$. Now one may prove using the same trig approach that these pairs of incircles have a common exsimilicenter, and the rest of the solution plays out similarly. The second approach can also be modified in the same way, once we obtain that a common exsimilicenter exists. (Thus in the generalization, it seems we also get there exists a circle tangent to all four incircles.) + diff --git a/USA_TST_TST/md/en-sols-TSTST-2020.md b/USA_TST_TST/md/en-sols-TSTST-2020.md new file mode 100644 index 0000000000000000000000000000000000000000..05ae2a6e9587529e7bcaaf590ddaf2b3a0108eb4 --- /dev/null +++ b/USA_TST_TST/md/en-sols-TSTST-2020.md @@ -0,0 +1,535 @@ +# USA TSTST 2020 Solutions
United States of America - TST Selection Test
Ankan Bhattacharya and Evan Chen
62 $2^{\text {nd }}$ IMO 2021 Russia and $10^{\text {th }}$ EGMO 2021 Georgia + +## Contents + +0 Problems ..... 2 +1 Solutions to Day 1 ..... 4 +1.1 TSTST 2020/1, proposed by Ankan Bhattacharya ..... 4 +1.2 TSTST 2020/2, proposed by Zack Chroman, Daniel Liu ..... 6 +1.3 TSTST 2020/3, proposed by Evan Chen, Danielle Wang ..... 8 +2 Solutions to Day 2 ..... 10 +2.1 TSTST 2020/4, proposed by Yang Liu ..... 10 +2.2 TSTST 2020/5, proposed by Ashwin Sah, Mehtaab Sawhney ..... 12 +2.3 TSTST 2020/6, proposed by Andrew Gu . ..... 14 +3 Solutions to Day 3 ..... 16 +3.1 TSTST 2020/7, proposed by Ankan Bhattacharya ..... 16 +3.2 TSTST 2020/8, proposed by Ankan Bhattacharya ..... 18 +3.3 TSTST 2020/9, proposed by Nikolai Beluhov ..... 20 + +## §0 Problems + +1. Let $a, b, c$ be fixed positive integers. There are $a+b+c$ ducks sitting in a circle, one behind the other. Each duck picks either rock, paper, or scissors, with $a$ ducks picking rock, $b$ ducks picking paper, and $c$ ducks picking scissors. + +A move consists of an operation of one of the following three forms: + +- If a duck picking rock sits behind a duck picking scissors, they switch places. +- If a duck picking paper sits behind a duck picking rock, they switch places. +- If a duck picking scissors sits behind a duck picking paper, they switch places. + +Determine, in terms of $a, b$, and $c$, the maximum number of moves which could take place, over all possible initial configurations. +2. Let $A B C$ be a scalene triangle with incenter $I$. The incircle of $A B C$ touches $\overline{B C}$, $\overline{C A}, \overline{A B}$ at points $D, E, F$, respectively. Let $P$ be the foot of the altitude from $D$ to $\overline{E F}$, and let $M$ be the midpoint of $\overline{B C}$. The rays $A P$ and $I P$ intersect the circumcircle of triangle $A B C$ again at points $G$ and $Q$, respectively. Show that the incenter of triangle $G Q M$ coincides with $D$. +3. We say a nondegenerate triangle whose angles have measures $\theta_{1}, \theta_{2}, \theta_{3}$ is quirky if there exists integers $r_{1}, r_{2}, r_{3}$, not all zero, such that + +$$ +r_{1} \theta_{1}+r_{2} \theta_{2}+r_{3} \theta_{3}=0 +$$ + +Find all integers $n \geq 3$ for which a triangle with side lengths $n-1, n, n+1$ is quirky. +4. Find all pairs of positive integers $(a, b)$ satisfying the following conditions: +(i) $a$ divides $b^{4}+1$, +(ii) $b$ divides $a^{4}+1$, +(iii) $\lfloor\sqrt{a}\rfloor=\lfloor\sqrt{b}\rfloor$. +5. Let $\mathbb{N}^{2}$ denote the set of ordered pairs of positive integers. A finite subset $S$ of $\mathbb{N}^{2}$ is stable if whenever $(x, y)$ is in $S$, then so are all points $\left(x^{\prime}, y^{\prime}\right)$ of $\mathbb{N}^{2}$ with both $x^{\prime} \leq x$ and $y^{\prime} \leq y$. +Prove that if $S$ is a stable set, then among all stable subsets of $S$ (including the empty set and $S$ itself), at least half of them have an even number of elements. +6. Let $A, B, C, D$ be four points such that no three are collinear and $D$ is not the orthocenter of triangle $A B C$. Let $P, Q, R$ be the orthocenters of $\triangle B C D, \triangle C A D$, $\triangle A B D$, respectively. Suppose that lines $A P, B Q, C R$ are pairwise distinct and are concurrent. Show that the four points $A, B, C, D$ lie on a circle. +7. Find all nonconstant polynomials $P(z)$ with complex coefficients for which all complex roots of the polynomials $P(z)$ and $P(z)-1$ have absolute value 1. +8. For every positive integer $N$, let $\sigma(N)$ denote the sum of the positive integer divisors of $N$. Find all integers $m \geq n \geq 2$ satisfying + +$$ +\frac{\sigma(m)-1}{m-1}=\frac{\sigma(n)-1}{n-1}=\frac{\sigma(m n)-1}{m n-1} . +$$ + +9. Ten million fireflies are glowing in $\mathbb{R}^{3}$ at midnight. Some of the fireflies are friends, and friendship is always mutual. Every second, one firefly moves to a new position so that its distance from each one of its friends is the same as it was before moving. This is the only way that the fireflies ever change their positions. No two fireflies may ever occupy the same point. +Initially, no two fireflies, friends or not, are more than a meter away. Following some finite number of seconds, all fireflies find themselves at least ten million meters away from their original positions. Given this information, find the greatest possible number of friendships between the fireflies. + +## §1 Solutions to Day 1 + +## §1.1 TSTST 2020/1, proposed by Ankan Bhattacharya + +Available online at https://aops.com/community/p18933796. + +## Problem statement + +Let $a, b, c$ be fixed positive integers. There are $a+b+c$ ducks sitting in a circle, one behind the other. Each duck picks either rock, paper, or scissors, with a ducks picking rock, $b$ ducks picking paper, and $c$ ducks picking scissors. + +A move consists of an operation of one of the following three forms: + +- If a duck picking rock sits behind a duck picking scissors, they switch places. +- If a duck picking paper sits behind a duck picking rock, they switch places. +- If a duck picking scissors sits behind a duck picking paper, they switch places. + +Determine, in terms of $a, b$, and $c$, the maximum number of moves which could take place, over all possible initial configurations. + +The maximum possible number of moves is $\max (a b, a c, b c)$. +First, we prove this is best possible. We define a feisty triplet to be an unordered triple of ducks, one of each of rock, paper, scissors, such that the paper duck is between the rock and scissors duck and facing the rock duck, as shown. (There may be other ducks not pictured, but the orders are irrelevant.) +![](https://cdn.mathpix.com/cropped/2024_11_19_a8f70a0d08e0809deba0g-04.jpg?height=1100&width=1098&top_left_y=1500&top_left_x=479) + +Claim - The number of feisty triplets decreases by $c$ if a paper duck swaps places with a rock duck, and so on. + +## Proof. Clear. + +Obviously the number of feisty triples is at most $a b c$ to start. Thus at most $\max (a b, b c, c a)$ moves may occur, since the number of feisty triplets should always be nonnegative, at which point no moves are possible at all. + +To see that this many moves is possible, assume WLOG $a=\min (a, b, c)$ and suppose we have $a$ rocks, $b$ papers, and $c$ scissors in that clockwise order. +![](https://cdn.mathpix.com/cropped/2024_11_19_a8f70a0d08e0809deba0g-05.jpg?height=1109&width=1052&top_left_y=750&top_left_x=505) + +Then, allow the scissors to filter through the papers while the rocks stay put. Each of the $b$ papers swaps with $c$ scissors, for a total of $b c=\max (a b, a c, b c)$ swaps. + +Remark (Common errors). One small possible mistake: it is not quite kösher to say that "WLOG $a \leq b \leq c$ " because the condition is not symmetric, only cyclic. Therefore in this solution we only assume $a=\min (a, b, c)$. + +It is true here that every pair of ducks swaps at most once, and some solutions make use of this fact. However, this fact implicitly uses the fact that $a, b, c>0$ and is false without this hypothesis. + +## §1.2 TSTST 2020/2, proposed by Zack Chroman, Daniel Liu + +Available online at https://aops.com/community/p18933557. + +## Problem statement + +Let $A B C$ be a scalene triangle with incenter $I$. The incircle of $A B C$ touches $\overline{B C}$, $\overline{C A}, \overline{A B}$ at points $D, E, F$, respectively. Let $P$ be the foot of the altitude from $D$ to $\overline{E F}$, and let $M$ be the midpoint of $\overline{B C}$. The rays $A P$ and $I P$ intersect the circumcircle of triangle $A B C$ again at points $G$ and $Q$, respectively. Show that the incenter of triangle $G Q M$ coincides with $D$. + +Refer to the figure below. +![](https://cdn.mathpix.com/cropped/2024_11_19_a8f70a0d08e0809deba0g-06.jpg?height=798&width=803&top_left_y=906&top_left_x=632) + +Claim - The point $Q$ is the Miquel point of $B F E C$. Also, $\overline{Q D}$ bisects $\angle B Q C$. +Proof. Inversion around the incircle maps line $E F$ to $(A I E F)$ and the nine-point circle of $\triangle D E F$ to the circumcircle of $\triangle A B C$ (as the midpoint of $\overline{E F}$ maps to $A$, etc.). This implies $P$ maps to $Q$; that is, $Q$ coincides with the second intersection of (AFIE) with $(A B C)$. This is the claimed Miquel point. + +The spiral similarity mentioned then gives $\frac{Q B}{B F}=\frac{Q C}{C E}$, so $\overline{Q D}$ bisects $\angle B Q C$. +Remark. The point $Q$ and its properties mentioned in the first claim have appeared in other references. See for example Canada 2007/5, ELMO 2010/6, HMMT 2016 T-10, USA TST 2017/2, USA TST 2019/6 for a few examples. + +Claim - We have $(Q G ; B C)=-1$, so in particular $\overline{G D}$ bisects $\angle B G C$. +Proof. Note that + +$$ +-1=(A I ; E F) \stackrel{Q}{=}(\overline{A Q} \cap \overline{E F}, P ; E, F) \stackrel{A}{=}(Q G ; B C) +$$ + +The last statement follows from Apollonian circle, or more bluntly $\frac{G B}{G C}=\frac{Q B}{Q C}=\frac{B D}{D C}$. +Hence $\overline{Q D}$ and $\overline{G D}$ are angle bisectors of $\angle B Q C$ and $\angle B G C$. However, $\overline{Q M}$ and $\overline{Q G}$ are isogonal in $\angle B Q C$ (as median and symmedian), and similarly for $\angle B G C$, as desired. + +## §1.3 TSTST 2020/3, proposed by Evan Chen, Danielle Wang + +Available online at https://aops.com/community/p18933954. + +## Problem statement + +We say a nondegenerate triangle whose angles have measures $\theta_{1}, \theta_{2}, \theta_{3}$ is quirky if there exists integers $r_{1}, r_{2}, r_{3}$, not all zero, such that + +$$ +r_{1} \theta_{1}+r_{2} \theta_{2}+r_{3} \theta_{3}=0 +$$ + +Find all integers $n \geq 3$ for which a triangle with side lengths $n-1, n, n+1$ is quirky. + +The answer is $n=3,4,5,7$. +We first introduce a variant of the $k$ th Chebyshev polynomials in the following lemma (which is standard, and easily shown by induction). + +## Lemma + +For each $k \geq 0$ there exists $P_{k}(X) \in \mathbb{Z}[X]$, monic for $k \geq 1$ and with degree $k$, such that + +$$ +P_{k}\left(X+X^{-1}\right) \equiv X^{k}+X^{-k} . +$$ + +The first few are $P_{0}(X) \equiv 2, P_{1}(X) \equiv X, P_{2}(X) \equiv X^{2}-2, P_{3}(X) \equiv X^{3}-3 X$. +Suppose the angles of the triangle are $\alpha<\beta<\gamma$, so the law of cosines implies that + +$$ +2 \cos \alpha=\frac{n+4}{n+1} \quad \text { and } \quad 2 \cos \gamma=\frac{n-4}{n-1} +$$ + +Claim - The triangle is quirky iff there exists $r, s \in \mathbb{Z}_{\geq 0}$ not both zero such that + +$$ +\cos (r \alpha)= \pm \cos (s \gamma) \quad \text { or equivalently } \quad P_{r}\left(\frac{n+4}{n+1}\right)= \pm P_{s}\left(\frac{n-4}{n-1}\right) +$$ + +Proof. If there are integers $x, y, z$ for which $x \alpha+y \beta+z \gamma=0$, then we have that $(x-y) \alpha=(y-z) \gamma-\pi y$, whence it follows that we may take $r=|x-y|$ and $s=|y-z|$ (noting $r=s=0$ implies the absurd $x=y=z$ ). Conversely, given such $r$ and $s$ with $\cos (r \alpha)= \pm \cos (s \gamma)$, then it follows that $r \alpha \pm s \gamma=k \pi=k(\alpha+\beta+\gamma)$ for some $k$, so the triangle is quirky. + +If $r=0$, then by rational root theorem on $P_{s}(X) \pm 2$ it follows $\frac{n-4}{n-1}$ must be an integer which occurs only when $n=4$ (recall $n \geq 3$ ). Similarly we may discard the case $s=0$. + +Thus in what follows assume $n \neq 4$ and $r, s>0$. Then, from the fact that $P_{r}$ and $P_{s}$ are nonconstant monic polynomials, we find + +## Corollary + +If $n \neq 4$ works, then when $\frac{n+4}{n+1}$ and $\frac{n-4}{n-1}$ are written as fractions in lowest terms, the denominators have the same set of prime factors. + +But $\operatorname{gcd}(n+1, n-1)$ divides 2 , and $\operatorname{gcd}(n+4, n+1), \operatorname{gcd}(n-4, n-1)$ divide 3 . So we only have three possibilities: + +- $n+1=2^{u}$ and $n-1=2^{v}$ for some $u, v \geq 0$. This is only possible if $n=3$. Here $2 \cos \alpha=\frac{7}{4}$ and $2 \cos \gamma=-\frac{1}{2}$, and indeed $P_{2}(-1 / 2)=-7 / 4$. +- $n+1=3 \cdot 2^{u}$ and $n-1=2^{v}$ for some $u, v \geq 0$, which implies $n=5$. Here $2 \cos \alpha=\frac{3}{2}$ and $2 \cos \gamma=\frac{1}{4}$, and indeed $P_{2}(3 / 2)=1 / 4$. +- $n+1=2^{u}$ and $n-1=3 \cdot 2^{v}$ for some $u, v \geq 0$, which implies $n=7$. Here $2 \cos \alpha=\frac{11}{8}$ and $2 \cos \gamma=\frac{1}{2}$, and indeed $P_{3}(1 / 2)=-11 / 8$. + +Finally, $n=4$ works because the triangle is right, completing the solution. +Remark (Major generalization due to Luke Robitaille). In fact one may find all quirky triangles whose sides are integers in arithmetic progression. + +Indeed, if the side lengths of the triangle are $x-y, x, x+y$ with $\operatorname{gcd}(x, y)=1$ then the problem becomes + +$$ +P_{r}\left(\frac{x+4 y}{x+y}\right)= \pm P_{s}\left(\frac{x-4 y}{x-y}\right) +$$ + +and so in the same way as before, we ought to have $x+y$ and $x-y$ are both of the form $3 \cdot 2^{*}$ unless $r s=0$. This time, when $r s=0$, we get the extra solutions $(1,0)$ and $(5,2)$. + +For $r s \neq 0$, by triangle inequality, we have $x-y \leq x+y<3(x-y)$, and $\min \left(\nu_{2}(x-\right.$ $\left.y), \nu_{2}(x+y)\right) \leq 1$, so it follows one of $x-y$ or $x+y$ must be in $\{1,2,3,6\}$. An exhaustive check then leads to + +$$ +(x, y) \in\{(3,1),(5,1),(7,1),(11,5)\} \cup\{(1,0),(5,2),(4,1)\} +$$ + +as the solution set. And in fact they all work. +In conclusion the equilateral triangle, $3-5-7$ triangle (which has a $120^{\circ}$ angle) and $6-11-16$ triangle (which satisfies $B=3 A+4 C$ ) are exactly the new quirky triangles (up to similarity) whose sides are integers in arithmetic progression. + +## §2 Solutions to Day 2 + +## §2.1 TSTST 2020/4, proposed by Yang Liu + +Available online at https://aops.com/community/p19444614. + +## Problem statement + +Find all pairs of positive integers $(a, b)$ satisfying the following conditions: +(i) $a$ divides $b^{4}+1$, +(ii) $b$ divides $a^{4}+1$, +(iii) $\lfloor\sqrt{a}\rfloor=\lfloor\sqrt{b}\rfloor$. + +The only solutions are $(1,1),(1,2)$, and $(2,1)$, which clearly work. Now we show there are no others. + +Obviously, $\operatorname{gcd}(a, b)=1$, so the problem conditions imply + +$$ +a b \mid(a-b)^{4}+1 +$$ + +since each of $a$ and $b$ divide the right-hand side. We define + +$$ +k \stackrel{\text { def }}{=} \frac{(b-a)^{4}+1}{a b} . +$$ + +Claim (Size estimate) — We must have $k \leq 16$. + +Proof. Let $n=\lfloor\sqrt{a}\rfloor=\lfloor\sqrt{b}\rfloor$, so that $a, b \in\left[n^{2}, n^{2}+2 n\right]$. We have that + +$$ +\begin{aligned} +a b & \geq n^{2}\left(n^{2}+1\right) \geq n^{4}+1 \\ +(b-a)^{4}+1 & \leq(2 n)^{4}+1=16 n^{4}+1 +\end{aligned} +$$ + +which shows $k \leq 16$. + +Claim (Orders argument) - In fact, $k=1$. + +Proof. First of all, note that $k$ cannot be even: if it was, then $a, b$ have opposite parity, but then $4 \mid(b-a)^{4}+1$, contradiction. + +Thus $k$ is odd. However, every odd prime divisor of $(b-a)^{4}+1$ is congruent to 1 $(\bmod 8)$ and is thus at least 17 , so $k=1$ or $k \geq 17$. It follows that $k=1$. + +At this point, we have reduced to solving + +$$ +a b=(b-a)^{4}+1 +$$ + +and we need to prove the claimed solutions are the only ones. Write $b=a+d$, and assume WLOG that $d \geq 0$ : then we have $a(a+d)=d^{4}+1$, or + +$$ +a^{2}-d a-\left(d^{4}+1\right)=0 +$$ + +The discriminant $d^{2}+4\left(d^{4}+1\right)=4 d^{4}+d^{2}+4$ must be a perfect square. + +- The cases $d=0$ and $d=1$ lead to pairs $(1,1)$ and $(1,2)$. +- If $d \geq 2$, then we can sandwich + +$$ +\left(2 d^{2}\right)^{2}<4 d^{4}+d^{2}+4<4 d^{4}+4 d^{2}+1=\left(2 d^{2}+1\right)^{2} +$$ + +so the discriminant is not a square. +The solution is complete. +Remark (Author remarks on origin). This comes from the problem of the existence of a pair of elliptic curves over $\mathbb{F}_{a}, \mathbb{F}_{b}$ respectively, such that the number of points on one is the field size of the other. The bound $n^{2} \leq a, b<(n+1)^{2}$ is the Hasse bound. The divisibility conditions correspond to asserting that the embedding degree of each curve is 8 , so that they are pairing friendly. In this way, the problem is essentially the key result of https://arxiv.org/pdf/1803.02067.pdf, shown in Proposition 3. + +## §2.2 TSTST 2020/5, proposed by Ashwin Sah, Mehtaab Sawhney + +Available online at https://aops.com/community/p19444403. + +## Problem statement + +Let $\mathbb{N}^{2}$ denote the set of ordered pairs of positive integers. A finite subset $S$ of $\mathbb{N}^{2}$ is stable if whenever $(x, y)$ is in $S$, then so are all points $\left(x^{\prime}, y^{\prime}\right)$ of $\mathbb{N}^{2}$ with both $x^{\prime} \leq x$ and $y^{\prime} \leq y$. +Prove that if $S$ is a stable set, then among all stable subsets of $S$ (including the empty set and $S$ itself), at least half of them have an even number of elements. + +The following inductive solution was given by Nikolai Beluhov. We proceed by induction on $|S|$, with $|S| \leq 1$ clear. + +Suppose $|S| \geq 2$. For any $p \in S$, let $R(p)$ denote the stable rectangle with upper-right corner $p$. We say such $p$ is pivotal if $p+(1,1) \notin S$ and $|R(p)|$ is even. +![](https://cdn.mathpix.com/cropped/2024_11_19_a8f70a0d08e0809deba0g-12.jpg?height=455&width=818&top_left_y=1069&top_left_x=622) + +Claim - If $|S| \geq 2$, then a pivotal $p$ always exists. +Proof. Consider the top row of $S$. + +- If it has length at least 2 , one of the two rightmost points in it is pivotal. +- Otherwise, the top row has length 1. Now either the top point or the point below it (which exists as $|S| \geq 2$ ) is pivotal. + +We describe how to complete the induction, given some pivotal $p \in S$. There is a partition + +$$ +S=R(p) \sqcup S_{1} \sqcup S_{2} +$$ + +where $S_{1}$ and $S_{2}$ are the sets of points in $S$ above and to the right of $p$ (possibly empty). +Claim - The desired inequality holds for stable subsets containing $p$. +Proof. Let $E_{1}$ denote the number of even stable subsets of $S_{1}$; denote $E_{2}, O_{1}, O_{2}$ analogously. The stable subsets containing $p$ are exactly $R(p) \sqcup T_{1} \sqcup T_{2}$, where $T_{1} \subseteq S_{1}$ and $T_{2} \subseteq S_{2}$ are stable. + +Since $|R(p)|$ is even, exactly $E_{1} E_{2}+O_{1} O_{2}$ stable subsets containing $p$ are even, and exactly $E_{1} O_{2}+E_{2} O_{1}$ are odd. As $E_{1} \geq O_{1}$ and $E_{2} \geq O_{2}$ by inductive hypothesis, we obtain $E_{1} E_{2}+O_{1} O_{2} \geq E_{1} O_{2}+E_{2} O_{1}$ as desired. + +By the inductive hypothesis, the desired inequality also holds for stable subsets not containing $p$, so we are done. + +## §2.3 TSTST 2020/6, proposed by Andrew Gu + +Available online at https://aops.com/community/p19444197. + +## Problem statement + +Let $A, B, C, D$ be four points such that no three are collinear and $D$ is not the orthocenter of triangle $A B C$. Let $P, Q, R$ be the orthocenters of $\triangle B C D, \triangle C A D$, $\triangle A B D$, respectively. Suppose that lines $A P, B Q, C R$ are pairwise distinct and are concurrent. Show that the four points $A, B, C, D$ lie on a circle. + +Let $T$ be the concurrency point, and let $H$ be the orthocenter of $\triangle A B C$. +![](https://cdn.mathpix.com/cropped/2024_11_19_a8f70a0d08e0809deba0g-14.jpg?height=593&width=806&top_left_y=863&top_left_x=631) + +Claim (Key claim) - $T$ is the midpoint of $\overline{A P}, \overline{B Q}, \overline{C R}, \overline{D H}$, and $D$ is the orthocenter of $\triangle P Q R$. + +Proof. Note that $\overline{A Q} \| \overline{B P}$, as both are perpendicular to $\overline{C D}$. Since lines $A P$ and $B Q$ are distinct, lines $A Q$ and $B P$ are distinct. +By symmetric reasoning, we get that $A Q C P B R$ is a hexagon with opposite sides parallel and concurrent diagonals as $\overline{A P}, \overline{B Q}, \overline{C R}$ meet at $T$. This implies that the hexagon is centrally symmetric about $T$; indeed + +$$ +\frac{A T}{T P}=\frac{T Q}{B T}=\frac{C T}{T R}=\frac{T P}{A T} +$$ + +so all the ratios are equal to +1 . +Next, $\overline{P D} \perp \overline{B C} \| \overline{Q R}$, so by symmetry we get $D$ is the orthocenter of $\triangle P Q R$. This means that $T$ is the midpoint of $\overline{D H}$ as well. + +## Corollary + +The configuration is now symmetric: we have four points $A, B, C, D$, and their reflections in $T$ are four orthocenters $P, Q, R, H$. + +Let $S$ be the centroid of $\{A, B, C, D\}$, and let $O$ be the reflection of $T$ in $S$. We are ready to conclude: + +Claim - $A, B, C, D$ are equidistant from $O$. +Proof. Let $A^{\prime}, O^{\prime}, S^{\prime}, T^{\prime}, D^{\prime}$ be the projections of $A, O, S, T, D$ onto line $B C$. +Then $T^{\prime}$ is the midpoint of $\overline{A^{\prime} D^{\prime}}$, so $S^{\prime}=\frac{1}{4}\left(A^{\prime}+D^{\prime}+B+C\right)$ gives that $O^{\prime}$ is the midpoint of $\overline{B C}$. + +Thus $O B=O C$ and we're done. + +## §3 Solutions to Day 3 + +## §3.1 TSTST 2020/7, proposed by Ankan Bhattacharya + +Available online at https://aops.com/community/p20020202. + +## Problem statement + +Find all nonconstant polynomials $P(z)$ with complex coefficients for which all complex roots of the polynomials $P(z)$ and $P(z)-1$ have absolute value 1 . + +The answer is $P(x)$ should be a polynomial of the form $P(x)=\lambda x^{n}-\mu$ where $|\lambda|=|\mu|$ and $\operatorname{Re} \mu=-\frac{1}{2}$. One may check these all work; let's prove they are the only solutions. +\l First approach (Evan Chen). We introduce the following notations: + +$$ +\begin{aligned} +P(x) & =c_{n} x^{n}+c_{n-1} x^{n-1}+\cdots+c_{1} x+c_{0} \\ +& =c_{n}\left(x+\alpha_{1}\right) \ldots\left(x+\alpha_{n}\right) \\ +P(x)-1 & =c_{n}\left(x+\beta_{1}\right) \ldots\left(x+\beta_{n}\right) +\end{aligned} +$$ + +By taking conjugates, + +$$ +\begin{aligned} +\left(x+\alpha_{1}\right) \cdots\left(x+\alpha_{n}\right) & =\left(x+\beta_{1}\right) \cdots\left(x+\beta_{n}\right)+c_{n}^{-1} \\ +\Longrightarrow\left(x+\frac{1}{\alpha_{1}}\right) \cdots\left(x+\frac{1}{\alpha_{n}}\right) & =\left(x+\frac{1}{\beta_{1}}\right) \cdots\left(x+\frac{1}{\beta_{n}}\right)+\left(\overline{c_{n}}\right)^{-1} +\end{aligned} +$$ + +The equation $(\boldsymbol{\oplus})$ is the main player: +Claim - We have $c_{k}=0$ for all $k=1, \ldots, n-1$. + +Proof. By comparing coefficients of $x^{k}$ in ( $\left.\boldsymbol{\phi}\right)$ we obtain + +$$ +\frac{c_{n-k}}{\prod_{i} \alpha_{i}}=\frac{c_{n-k}}{\prod_{i} \beta_{i}} +$$ + +but $\prod_{i} \alpha_{i}-\prod_{i} \beta_{i}=\frac{1}{c_{n}} \neq 0$. Hence $c_{k}=0$. +It follows that $P(x)$ must be of the form $P(x)=\lambda x^{n}-\mu$, so that $P(x)=\lambda x^{n}-(\mu+1)$. This requires $|\mu|=|\mu+1|=|\lambda|$ which is equivalent to the stated part. + +II Second approach (from the author). We let $A=P$ and $B=P-1$ to make the notation more symmetric. We will as before show that $A$ and $B$ have all coefficients equal to zero other than the leading and constant coefficient; the finish is the same. + +First, we rule out double roots. +Claim - Neither $A$ nor $B$ have double roots. + +Proof. Suppose that $b$ is a double root of $B$. By differentiating, we obtain $A^{\prime}=B^{\prime}$, so $A^{\prime}(b)=0$. However, by Gauss-Lucas, this forces $A(b)=0$, contradiction. + +Let $\omega=e^{2 \pi i / n}$, let $a_{1}, \ldots, a_{n}$ be the roots of $A$, and let $b_{1}, \ldots, b_{n}$ be the roots of $B$. For each $k$, let $A_{k}$ and $B_{k}$ be the points in the complex plane corresponding to $a_{k}$ and $b_{k}$ 。 + +Claim (Main claim) - For any $i$ and $j, \frac{a_{i}}{a_{j}}$ is a power of $\omega$. +Proof. Note that + +$$ +\frac{a_{i}-b_{1}}{a_{j}-b_{1}} \cdots \frac{a_{i}-b_{n}}{a_{j}-b_{n}}=\frac{B\left(a_{i}\right)}{B\left(a_{j}\right)}=\frac{A\left(a_{i}\right)-1}{A\left(a_{j}\right)-1}=\frac{0-1}{0-1}=1 +$$ + +Since the points $A_{i}, A_{j}, B_{k}$ all lie on the unit circle, interpreting the left-hand side geometrically gives + +$$ +\measuredangle A_{i} B_{1} A_{j}+\cdots+\measuredangle A_{i} B_{n} A_{j}=0 \Longrightarrow n \widehat{A_{i} A_{j}}=0 +$$ + +where angles are directed modulo $180^{\circ}$ and arcs are directed modulo $360^{\circ}$. This implies that $\frac{a_{i}}{a_{j}}$ is a power of $\omega$. + +Now the finish is easy: since $a_{1}, \ldots, a_{n}$ are all different, they must be $a_{1} \omega^{0}, \ldots, a_{1} \omega^{n-1}$ in some order; this shows that $A$ is a multiple of $x^{n}-a_{1}^{n}$, as needed. + +## §3.2 TSTST 2020/8, proposed by Ankan Bhattacharya + +Available online at https://aops.com/community/p20020195. + +## Problem statement + +For every positive integer $N$, let $\sigma(N)$ denote the sum of the positive integer divisors of $N$. Find all integers $m \geq n \geq 2$ satisfying + +$$ +\frac{\sigma(m)-1}{m-1}=\frac{\sigma(n)-1}{n-1}=\frac{\sigma(m n)-1}{m n-1} +$$ + +The answer is that $m$ and $n$ should be powers of the same prime number. These all work because for a prime power we have + +$$ +\frac{\sigma\left(p^{e}\right)-1}{p^{e}-1}=\frac{\left(1+p+\cdots+p^{e}\right)-1}{p^{e}-1}=\frac{p\left(1+\cdots+p^{e-1}\right)}{p^{e}-1}=\frac{p}{p-1} +$$ + +So we now prove these are the only ones. Let $\lambda$ be the common value of the three fractions. + +Claim - Any solution $(m, n)$ should satisfy $d(m n)=d(m)+d(n)-1$. +Proof. The divisors of $m n$ include the divisors of $m$, plus $m$ times the divisors of $n$ (counting $m$ only once). Let $\lambda$ be the common value; then this gives + +$$ +\begin{aligned} +\sigma(m n) & \geq \sigma(m)+m \sigma(n)-m \\ +& =(\lambda m-\lambda+1)+m(\lambda n-\lambda+1)-m \\ +& =\lambda m n-\lambda+1 +\end{aligned} +$$ + +and so equality holds. Thus these are all the divisors of $m n$, for a count of $d(m)+d(n)-$ 1. + +Claim - If $d(m n)=d(m)+d(n)-1$ and $\min (m, n) \geq 2$, then $m$ and $n$ are powers of the same prime. + +Proof. Let $A$ denote the set of divisors of $m$ and $B$ denote the set of divisors of $n$. Then $|A \cdot B|=|A|+|B|-1$ and $\min (|A|,|B|)>1$, so $|A|$ and $|B|$ are geometric progressions with the same ratio. It follows that $m$ and $n$ are powers of the same prime. + +Remark (Nikolai Beluhov). Here is a completion not relying on $|A \cdot B|=|A|+|B|-1$. By the above arguments, we see that every divisor of $m n$ is either a divisor of $n$, or $n$ times a divisor of $m$. + +Now suppose that some prime $p \mid m$ but $p \nmid n$. Then $p \mid m n$ but $p$ does not appear in the above classification, a contradiction. By symmetry, it follows that $m$ and $n$ have the same prime divisors. + +Now suppose we have different primes $p \mid m$ and $q \mid n$. Write $\nu_{p}(m)=\alpha$ and $\nu_{p}(n)=\beta$. Then $p^{\alpha+\beta} \mid m n$, but it does not appear in the above characterization, a contradiction. Thus, $m$ and $n$ are powers of the same prime. + +Remark (Comments on the function in the problem). Let $f(n)=\frac{\sigma(n)-1}{n-1}$. Then $f$ is not really injective even outside the above solution; for example, we have $f\left(6 \cdot 11^{k}\right)=\frac{11}{5}$ for all $k$, plus sporadic equivalences like $f(14)=f(404)$, as pointed out by one reviewer during test-solving. This means that both relations should be used at once, not independently. + +Remark (Authorship remarks). Ankan gave the following story for how he came up with the problem while thinking about so-called almost perfect numbers. + +I was in some boring talk when I recalled a conjecture that if $\sigma(n)=2 n-1$, then $n$ is a power of 2 . For some reason (divine intervention, maybe) I had the double idea of (1) seeing whether $m, n, m n$ all almost perfect implies $m, n$ powers of 2 , and (2) trying the naive divisor bound to resolve this. Through sheer dumb luck this happened to work out perfectly. I thought this was kinda cool but I felt that I hadn't really unlocked a lot of the potential this idea had: then I basically tried to find the "general situation" which allows for this manipulation, and was amazed that it led to such a striking statement. + +## §3.3 TSTST 2020/9, proposed by Nikolai Beluhov + +Available online at https://aops.com/community/p20020206. + +## Problem statement + +Ten million fireflies are glowing in $\mathbb{R}^{3}$ at midnight. Some of the fireflies are friends, and friendship is always mutual. Every second, one firefly moves to a new position so that its distance from each one of its friends is the same as it was before moving. This is the only way that the fireflies ever change their positions. No two fireflies may ever occupy the same point. +Initially, no two fireflies, friends or not, are more than a meter away. Following some finite number of seconds, all fireflies find themselves at least ten million meters away from their original positions. Given this information, find the greatest possible number of friendships between the fireflies. + +In general, we show that when $n \geq 70$, the answer is $f(n)=\left\lfloor\frac{n^{2}}{3}\right\rfloor$. +Construction: Choose three pairwise parallel lines $\ell_{A}, \ell_{B}, \ell_{C}$ forming an infinite equilateral triangle prism (with side larger than 1). Split the $n$ fireflies among the lines as equally as possible, and say that two fireflies are friends iff they lie on different lines. + +To see this works: + +1. Reflect $\ell_{A}$ and all fireflies on $\ell_{A}$ in the plane containing $\ell_{B}$ and $\ell_{C}$. +2. Reflect $\ell_{B}$ and all fireflies on $\ell_{B}$ in the plane containing $\ell_{C}$ and $\ell_{A}$. +3. Reflect $\ell_{C}$ and all fireflies on $\ell_{C}$ in the plane containing $\ell_{A}$ and $\ell_{B}$. + +Proof: Consider a valid configuration of fireflies. If there is no 4 -clique of friends, then by Turán's theorem, there are at most $f(n)$ pairs of friends. +Let $g(n)$ be the answer, given that there exist four pairwise friends (say $a, b, c, d$ ). Note that for a firefly to move, all its friends must be coplanar. + +Claim (No coplanar $K_{4}$ ) — We can't have four coplanar fireflies which are pairwise friends. + +Proof. If we did, none of them could move (unless three are collinear, in which case they can't move). + +Claim (Key claim — tetrahedrons don't share faces often) — There are at most 12 fireflies $e$ which are friends with at least three of $a, b, c, d$. + +Proof. First denote by $A, B, C, D$ the locations of fireflies $a, b, c, d$. These four positions change over time as fireflies move, but the tetrahedron $A B C D$ always has a fixed shape, and we will take this tetrahedron as our reference frame for the remainder of the proof. + +WLOG, will assume that $e$ is friends with $a, b, c$. Then $e$ will always be located at one of two points $E_{1}$ and $E_{2}$ relative to $A B C$, such that $E_{1} A B C$ and $E_{2} A B C$ are two +congruent tetrahedrons with fixed shape. We note that points $D, E_{1}$, and $E_{2}$ are all different: clearly $D \neq E_{1}$ and $E_{1} \neq E_{2}$. (If $D=E_{2}$, then some fireflies won't be able to move.) + +Consider the moment where firefly $a$ moves. Its friends must be coplanar at that time, so one of $E_{1}, E_{2}$ lies in plane $B C D$. Similar reasoning holds for planes $A C D$ and $A B D$. + +So, WLOG $E_{1}$ lies on both planes $B C D$ and $A C D$. Then $E_{1}$ lies on line $C D$, and $E_{2}$ lies in plane $A B D$. This uniquely determines $\left(E_{1}, E_{2}\right)$ relative to $A B C D$ : + +- $E_{1}$ is the intersection of line $C D$ with the reflection of plane $A B D$ in plane $A B C$. +- $E_{2}$ is the intersection of plane $A B D$ with the reflection of line $C D$ in plane $A B C$. + +Accounting for WLOGs, there are at most 12 possibilities for the set $\left\{E_{1}, E_{2}\right\}$, and thus at most 12 possibilities for $E$. (It's not possible for both elements of one pair $\left\{E_{1}, E_{2}\right\}$ to be occupied, because then they couldn't move.) + +Thus, the number of friendships involving exactly one of $a, b, c, d$ is at most $(n-16)$. $2+12 \cdot 3=2 n+4$, so removing these four fireflies gives + +$$ +g(n) \leq 6+(2 n+4)+\max \{f(n-4), g(n-4)\} +$$ + +The rest of the solution is bounding. When $n \geq 24$, we have $(2 n+10)+f(n-4) \leq f(n)$, so + +$$ +g(n) \leq \max \{f(n),(2 n+10)+g(n-4)\} \quad \forall n \geq 24 +$$ + +By iterating the above inequality, we get + +$$ +\begin{aligned} +g(n) \leq \max \{f(n),(2 n+10) & +(2(n-4)+10) \\ +& +\cdots+(2(n-4 r)+10)+g(n-4 r-4)\} +\end{aligned} +$$ + +where $r$ satisfies $n-4 r-4<24 \leq n-4 r$. +Now + +$$ +\begin{aligned} +& (2 n+10)+(2(n-4)+10)+\cdots+(2(n-4 r)+10)+g(n-4 r-4) \\ += & (r+1)(2 n-4 r+10)+g(n-4 r-4) \\ +\leq & \left(\frac{n}{4}-5\right)(n+37)+\binom{24}{2} . +\end{aligned} +$$ + +This is less than $f(n)$ for $n \geq 70$, which concludes the solution. +Remark. There are positive integers $n$ such that it is possible to do better than $f(n)$ friendships. For instance, $f(5)=8$, whereas five fireflies $a, b, c, d$, and $e$ as in the proof of the Lemma ( $E_{1}$ being the intersection point of line $C D$ with the reflection of plane $(A B D)$ in plane $(A B C), E_{2}$ being the intersection point of plane $(A B D)$ with the reflection of line $C D$ in plane $(A B C)$, and tetrahedron $A B C D$ being sufficiently arbitrary that points $E_{1}$ and $E_{2}$ exist and points $D, E_{1}$, and $E_{2}$ are pairwise distinct) give a total of nine friendships. + +Remark (Author comments). It is natural to approach the problem by looking at the two-dimensional version first. In two dimensions, the following arrangement suggests itself almost immediately: We distribute all fireflies as equally as possible among two parallel lines, and two fireflies are friends if and only if they are on different lines. + +Similarly to the three-dimensional version, this attains the greatest possible number of friendships for all sufficiently large $n$, though not for all $n$. For instance, at least one friendlier arrangements exists for $n=4$, similarly to the above friendlier arrangement for $n=5$ in three dimensions. + +This observation strongly suggests that in three dimensions we should distribute the fireflies as equally as possible among two parallel planes, and that two fireflies should be friends if and only if they are on different planes. It was a great surprise for me to discover that this arrangement does not in fact give the correct answer! + +Remark. On the other hand, Ankan Bhattacharya gives the following reasoning as to why the answer should not be that surprising: + +I think the answer $\left(10^{14}-1\right) / 3$ is quite natural if you realize that $(n / 2)^{2}$ is probably optimal in 2D and $\binom{n}{2}$ is optimal in super high dimensions (i.e. around $n$ ). So going from dimension 2 to 3 should increase the answer (and indeed it does). + diff --git a/USA_TST_TST/md/en-sols-TSTST-2021.md b/USA_TST_TST/md/en-sols-TSTST-2021.md new file mode 100644 index 0000000000000000000000000000000000000000..b5442d99aa7c2ad62f5705b4e086be4f5164b58e --- /dev/null +++ b/USA_TST_TST/md/en-sols-TSTST-2021.md @@ -0,0 +1,1228 @@ +# USA TSTST 2021 Solutions
United States of America - TST Selection Test
Andrew Gu and Evan Chen
$63^{\text {rd }}$ IMO 2022 Norway and $11^{\text {th }}$ EGMO 2022 Hungary + +## Contents + +0 Problems ..... 2 +1 Solutions to Day 1 ..... 3 +1.1 TSTST 2021/1, proposed by Holden Mui ..... 3 +1.2 TSTST 2021/2, proposed by Merlijn Staps ..... 7 +1.3 TSTST 2021/3, proposed by Merlijn Staps ..... 11 +2 Solutions to Day 2 ..... 13 +2.1 TSTST 2021/4, proposed by Holden Mui ..... 13 +2.2 TSTST 2021/5, proposed by Vincent Huang ..... 15 +2.3 TSTST 2021/6, proposed by Nikolai Beluhov ..... 17 +3 Solutions to Day 3 ..... 23 +3.1 TSTST 2021/7, proposed by Ankit Bisain, Holden Mui ..... 23 +3.2 TSTST 2021/8, proposed by Fedir Yudin ..... 25 +3.3 TSTST 2021/9, proposed by Victor Wang ..... 28 + +## §0 Problems + +1. Let $A B C D$ be a quadrilateral inscribed in a circle with center $O$. Points $X$ and $Y$ lie on sides $A B$ and $C D$, respectively. Suppose the circumcircles of $A D X$ and $B C Y$ meet line $X Y$ again at $P$ and $Q$, respectively. Show that $O P=O Q$. +2. Let $a_{1}1$ for which there exists a positive integer $n$ such that $\binom{n}{k}$ is divisible by $n$, and $\binom{n}{m}$ is not divisible by $n$ for $2 \leq mi$. + +First, each $i$ has finite degree - otherwise + +$$ +\frac{a_{x_{1}}}{x_{1}}=\frac{a_{x_{2}}}{x_{2}}=\cdots +$$ + +for an infinite increasing sequence of positive integers $x_{i}$, but then the $a_{x_{i}}$ are unbounded. +Now we use the following process to build a sequence of indices whose $a_{i}$ we can lower-bound: + +- Start at $x_{1}=1$, which is good. +- If we're currently at good index $x_{i}$, then let $s_{i}$ be the largest positive integer such that $x_{i}$ has an edge to $x_{i}+s_{i}$. (This exists because the degrees are finite.) +- Let $t_{i}$ be the smallest positive integer for which $x_{i}+s_{i}+t_{i}$ is good, and let this be $x_{i+1}$. This exists because if all numbers $k \leq x \leq 2 k$ are bad, they must each connect to some number less than $k$ (if two connect to each other, the smaller one is good), but then two connect to the same number, and therefore to each other this is the idea we will use later to bound the $t_{i}$ as well. + +Then $x_{i}=1+s_{1}+t_{1}+\cdots+s_{i-1}+t_{i-1}$, and we have + +$$ +a_{x_{i+1}}>a_{x_{i}+s_{i}}=\frac{x_{i}+s_{i}}{x_{i}} a_{x_{i}}=\frac{1+\left(s_{1}+\cdots+s_{i-1}+s_{i}\right)+\left(t_{1}+\cdots+t_{i-1}\right)}{1+\left(s_{1}+\cdots+s_{i-1}\right)+\left(t_{1}+\cdots+t_{i-1}\right)} a_{x_{i}} +$$ + +This means + +$$ +c_{n}:=\frac{a_{x_{n}}}{a_{1}}>\prod_{i=1}^{n-1} \frac{1+\left(s_{1}+\cdots+s_{i-1}+s_{i}\right)+\left(t_{1}+\cdots+t_{i-1}\right)}{1+\left(s_{1}+\cdots+s_{i-1}\right)+\left(t_{1}+\cdots+t_{i-1}\right)} +$$ + +## Lemma + +$t_{1}+\cdots+t_{n} \leq s_{1}+\cdots+s_{n}$ for each $n$. + +Proof. Consider $1 \leq i \leq n$. Note that for every $i$, the $t_{i}-1$ integers strictly between $x_{i}+s_{i}$ and $x_{i}+s_{i}+t_{i}$ are all bad, so each such index $x$ must have an edge to some $yx_{i}+s_{i} \geq x_{j}+s_{j}$, contradicting the fact that $x_{j}+s_{j}$ is the largest neighbor of $x_{j}$. + +This also means $x$ doesn't have an edge to $x_{j}+s_{j}$ for any $j \leq i$, since if it did, it would have an edge to $x_{j}$. + +Second, no two bad values of $x$ can have an edge, since then the smaller one is good. This also means no two bad $x$ can have an edge to the same $y$. + +Then each of the $\sum\left(t_{i}-1\right)$ values in the intervals $\left(x_{i}+s_{i}, x_{i}+s_{i}+t_{i}\right)$ for $1 \leq i \leq n$ must have an edge to an unique $y$ in one of the intervals $\left(x_{i}, x_{i}+s_{i}\right)$ (not necessarily with the same $i$ ). Therefore + +$$ +\sum\left(t_{i}-1\right) \leq \sum\left(s_{i}-1\right) \Longrightarrow \sum t_{i} \leq \sum s_{i} +$$ + +Now note that if $a>b$, then $\frac{a+x}{b+x}=1+\frac{a-b}{b+x}$ is decreasing in $x$. This means + +$$ +c_{n}>\prod_{i=1}^{n-1} \frac{1+2 s_{1}+\cdots+2 s_{i-1}+s_{i}}{1+2 s_{1}+\cdots+2 s_{i-1}}>\prod_{i=1}^{n-1} \frac{1+2 s_{1}+\cdots+2 s_{i-1}+2 s_{i}}{1+2 s_{1}+\cdots+2 s_{i-1}+s_{i}} +$$ + +By multiplying both products, we have a telescoping product, which results in + +$$ +c_{n}^{2} \geq 1+2 s_{1}+\cdots+2 s_{n}+2 s_{n+1} +$$ + +The right hand side is unbounded since the $s_{i}$ are positive integers, while $c_{n}=a_{x_{n}} / a_{1}<$ $1 / a_{1}$ is bounded, contradiction. + +【 Solution 3 (Gopal Goel). Suppose for sake of contradiction that the problem is false. Call an index $i$ a pin if + +$$ +\frac{a_{j}}{j}=\frac{a_{i}}{i} \Longrightarrow j \geq i +$$ + +## Lemma + +There exists $k$ such that if we have $\frac{a_{i}}{i}=\frac{a_{j}}{j}$ with $j>i \geq k$, then $j \leq 1.1 i$. + +Proof. Note that for any $i$, there are only finitely many $j$ with $\frac{a_{j}}{j}=\frac{a_{i}}{i}$, otherwise $a_{j}=\frac{j a_{i}}{i}$ is unbounded. Thus it suffices to find $k$ for which $j \leq 1.1 i$ when $j>i \geq k$. + +Suppose no such $k$ exists. Then, take a pair $j_{1}>i_{1}$ such that $\frac{a_{j_{1}}}{j_{1}}=\frac{a_{i_{1}}}{i_{1}}$ and $j_{1}>1.1 i_{1}$, or $a_{j_{1}}>1.1 a_{i_{1}}$. Now, since $k=j_{1}$ can't work, there exists a pair $j_{2}>i_{2} \geq i_{1}$ such that $\frac{a_{j_{2}}}{j_{2}}=\frac{a_{i_{2}}}{i_{2}}$ and $j_{2}>1.1 i_{2}$, or $a_{j_{2}}>1.1 a_{i_{2}}$. Continuing in this fashion, we see that + +$$ +a_{j_{\ell}}>1.1 a_{i_{\ell}}>1.1 a_{j_{\ell-1}} +$$ + +so we have that $a_{j_{\ell}}>1.1^{\ell} a_{i_{1}}$. Taking $\ell>\log _{1.1}\left(1 / a_{1}\right)$ gives the desired contradiction. + +## Lemma + +For $N>k^{2}$, there are at most $0.8 N$ pins in $[\sqrt{N}, N)$. + +Proof. By the first lemma, we see that the number of pins in $\left[\sqrt{N}, \frac{N}{1.1}\right)$ is at most the number of non-pins in $[\sqrt{N}, N)$. Therefore, if the number of pins in $[\sqrt{N}, N)$ is $p$, then we have + +$$ +p-N\left(1-\frac{1}{1.1}\right) \leq N-p +$$ + +so $p \leq 0.8 N$, as desired. +We say that $i$ is the pin of $j$ if it is the smallest index such that $\frac{a_{i}}{i}=\frac{a_{j}}{j}$. The pin of $j$ is always a pin. + +Given an index $i$, let $f(i)$ denote the largest index less than $i$ that is not a pin (we leave the function undefined when no such index exists, as we are only interested in the behavior for large $i$ ). Then $f$ is weakly increasing and unbounded by the first lemma. Let $N_{0}$ be a positive integer such that $f\left(\sqrt{N_{0}}\right)>k$. + +Take any $N>N_{0}$ such that $N$ is not a pin. Let $b_{0}=N$, and $b_{1}$ be the pin of $b_{0}$. Recursively define $b_{2 i}=f\left(b_{2 i-1}\right)$, and $b_{2 i+1}$ to be the pin of $b_{2 i}$. + +Let $\ell$ be the largest odd index such that $b_{\ell} \geq \sqrt{N}$. We first show that $b_{\ell} \leq 100 \sqrt{N}$. Since $N>N_{0}$, we have $b_{\ell+1}>k$. By the choice of $\ell$ we have $b_{\ell+2}<\sqrt{N}$, so + +$$ +b_{\ell+1}<1.1 b_{\ell+2}<1.1 \sqrt{N} +$$ + +by the first lemma. We see that all the indices from $b_{\ell+1}+1$ to $b_{\ell}$ must be pins, so we have at least $b_{\ell}-1.1 \sqrt{N}$ pins in $\left[\sqrt{N}, b_{\ell}\right)$. Combined with the second lemma, this shows that $b_{\ell} \leq 100 \sqrt{N}$. +Now, we have that $a_{b_{2 i}}=\frac{b_{2 i}}{b_{2 i+1}} a_{b_{2 i+1}}$ and $a_{b_{2 i+1}}>a_{b_{2 i+2}}$, so combining gives us + +$$ +\frac{a_{b_{0}}}{a_{b_{\ell}}}>\frac{b_{0}}{b_{1}} \frac{b_{2}}{b_{3}} \cdots \frac{b_{\ell-1}}{b_{\ell}} . +$$ + +Note that there are at least + +$$ +\left(b_{1}-b_{2}\right)+\left(b_{3}-b_{4}\right)+\cdots+\left(b_{\ell-2}-b_{\ell-1}\right) +$$ + +pins in $[\sqrt{N}, N)$, so by the second lemma, that sum is at most $0.8 N$. Thus, + +$$ +\begin{aligned} +\left(b_{0}-b_{1}\right)+\left(b_{2}-b_{3}\right)+\cdots+\left(b_{\ell-1}-b_{\ell}\right) & =b_{0}-\left[\left(b_{1}-b_{2}\right)+\cdots+\left(b_{\ell-2}-b_{\ell-1}\right)\right]-b_{\ell} \\ +& \geq 0.2 N-100 \sqrt{N} . +\end{aligned} +$$ + +Then + +$$ +\begin{aligned} +\frac{b_{0}}{b_{1}} \frac{b_{2}}{b_{3}} \cdots \frac{b_{\ell-1}}{b_{\ell}} & \geq 1+\frac{b_{0}-b_{1}}{b_{1}}+\cdots+\frac{b_{\ell-1}-b_{\ell}}{b_{\ell}} \\ +& >1+\frac{b_{0}-b_{1}}{b_{0}}+\cdots+\frac{b_{\ell-1}-b_{\ell}}{b_{0}} \\ +& \geq 1+\frac{0.2 N-100 \sqrt{N}}{N} +\end{aligned} +$$ + +which is at least 1.01 if $N_{0}$ is large enough. Thus, we see that + +$$ +a_{N}>1.01 a_{b_{\ell}} \geq 1.01 a_{\lfloor\sqrt{N}\rfloor} +$$ + +if $N>N_{0}$ is not a pin. Since there are arbitrarily large non-pins, this implies that the sequence $\left(a_{n}\right)$ is unbounded, which is the desired contradiction. + +## §1.3 TSTST 2021/3, proposed by Merlijn Staps + +Available online at https://aops.com/community/p23586679. + +## Problem statement + +Find all positive integers $k>1$ for which there exists a positive integer $n$ such that $\binom{n}{k}$ is divisible by $n$, and $\binom{n}{m}$ is not divisible by $n$ for $2 \leq mt_{p}$, so $\nu_{p}(n-i)=\nu_{p}(i)$; +- If $p \mid k$ and $i \neq k_{p}$, then we have $\nu_{p}(i), \nu_{p}(n-i) \leq t_{p}$ and $\nu_{p}(n) \geq t_{p}$, so again $\nu_{p}(n-i)=\nu_{p}(i) ;$ +- If $p \mid k$ and $i=k_{p}$, then we have $\nu_{p}(n-i)=\nu_{p}(i)+\nu_{p}(k)$ by (2). + +We conclude that $\nu_{p}(n-i)=\nu_{p}(i)$ always holds, except when $i=k_{p}$, when we have $\nu_{p}(n-i)=\nu_{p}(i)+\nu_{p}(k)$ (this formula holds irrespective of whether $p \mid k$ or $\left.p \nmid k\right)$. + +We can now show that $\binom{n}{k}$ is divisible by $n$, which amounts to showing that $k$ ! divides $(n-1)(n-2) \cdots(n-k+1)$. Indeed, for each prime $p \leq k$ we have + +$$ +\begin{aligned} +\nu_{p}((n-1)(n-2) \ldots(n-k+1)) & =\nu_{p}\left(n-k_{p}\right)+\sum_{i\nu_{p}(k)$, then it follows that + +$$ +\begin{aligned} +\nu_{p}((n-1)(n-2) \ldots(n-m+1)) & =\nu_{p}(k)+\sum_{i=1}^{m-1} \nu_{p}(i) \\ +& <\nu_{p}(m)+\sum_{i=1}^{m-1} \nu_{p}(i) \\ +& =\nu_{p}(m!) +\end{aligned} +$$ + +so $m$ ! cannot divide $(n-1)(n-2) \ldots(n-m+1)$. On the other hand, suppose that $\nu_{p}(m) \leq \nu_{p}(k)$ for all $p \mid k$, which would mean that $m \mid k$ and hence $m \leq \frac{k}{2}$. Consider a prime $p$ dividing $m$. We have $k_{p} \geq \frac{k}{2}$, because otherwise $2 k_{p}$ could have been used instead of $k_{p}$. It follows that $m \leq \frac{k}{2} \leq k_{p}$. Therefore, we obtain + +$$ +\begin{aligned} +\nu_{p}((n-1)(n-2) \ldots(n-m+1)) & =\sum_{i=1}^{m-1} \nu_{p}(n-i) \\ +& =\sum_{i=1}^{m-1} \nu_{p}(i) \\ +& =\nu_{p}((m-1)!)<\nu_{p}(m!) +\end{aligned} +$$ + +showing that $(n-1)(n-2) \cdots(n-m+1)$ is not divisible by $m$ !. This shows that $\binom{n}{m}$ is not divisible by $n$ for $ma / 2$, then $(\dagger)$ forces $r^{2}+s^{2} \leq 2 b$, giving the last case. +Hence, there exists some particular pair $(r, s)$ for which there are infinitely many solutions $(m, n)$. Simplifying the system gives + +$$ +\begin{aligned} +& a n=2 r m+r^{2}-b \\ +& 2 s n=a m+b-s^{2} +\end{aligned} +$$ + +Since the system is linear, for there to be infinitely many solutions $(m, n)$ the system must be dependent. This gives + +$$ +\frac{a}{2 s}=\frac{2 r}{a}=\frac{r^{2}-b}{b-s^{2}} +$$ + +so $a=2 \sqrt{r s}$ and $b=\frac{s^{2} \sqrt{r}+r^{2} \sqrt{s}}{\sqrt{r}+\sqrt{s}}$. Since $r s$ must be square, we can reparametrize as $r=k x^{2}, s=k y^{2}$, and $\operatorname{gcd}(x, y)=1$. This gives + +$$ +\begin{aligned} +a & =2 k x y \\ +b & =k^{2} x y\left(x^{2}-x y+y^{2}\right) +\end{aligned} +$$ + +Thus, $a \mid 2 b$, as desired. + +## §2.2 TSTST 2021/5, proposed by Vincent Huang + +Available online at https://aops.com/community/p23864182. + +## Problem statement + +Let $T$ be a tree on $n$ vertices with exactly $k$ leaves. Suppose that there exists a subset of at least $\frac{n+k-1}{2}$ vertices of $T$, no two of which are adjacent. Show that the longest path in $T$ contains an even number of edges. + +The longest path in $T$ must go between two leaves. The solutions presented here will solve the problem by showing that in the unique 2-coloring of $T$, all leaves are the same color. + +## 【 Solution 1 (Ankan Bhattacharya, Jeffery Li). + +## Lemma + +If $S$ is an independent set of $T$, then + +$$ +\sum_{v \in S} \operatorname{deg}(v) \leq n-1 +$$ + +Equality holds if and only if $S$ is one of the two components of the unique 2-coloring of $T$. + +Proof. Each edge of $T$ is incident to at most one vertex of $S$, so we obtain the inequality by counting how many vertices of $S$ each edge is incident to. For equality to hold, each edge is incident to exactly one vertex of $S$, which implies the 2 -coloring. + +We are given that there exists an independent set of at least $\frac{n+k-1}{2}$ vertices. By greedily choosing vertices of smallest degree, the sum of the degrees of these vertices is at least + +$$ +k+2 \cdot \frac{n-k-1}{2}=n-1 +$$ + +Thus equality holds everywhere, which implies that the independent set contains every leaf and is one of the components of the 2 -coloring. + +## 【 Solution 2 (Andrew Gu). + +## Lemma + +The vertices of $T$ can be partitioned into $k-1$ paths (i.e. the induced subgraph on each set of vertices is a path) such that all edges of $T$ which are not part of a path are incident to an endpoint of a path. + +Proof. Repeatedly trim the tree by taking a leaf and removing the longest path containing that leaf such that the remaining graph is still a tree. + +Now given a path of $a$ vertices, at most $\frac{a+1}{2}$ of those vertices can be in an independent set of $T$. By the lemma, $T$ can be partitioned into $k-1$ paths of $a_{1}, \ldots, a_{k-1}$ vertices, so the maximum size of an independent set of $T$ is + +$$ +\sum \frac{a_{i}+1}{2}=\frac{n+k-1}{2} . +$$ + +For equality to hold, each path in the partition must have an odd number of vertices, and has a unique 2 -coloring in red and blue where the endpoints are red. The unique independent set of $T$ of size $\frac{n+k-1}{2}$ is then the set of red vertices. By the lemma, the edges of $T$ which are not part of a path connect an endpoint of a path (which is colored red) to another vertex (which must be blue, because the red vertices are independent). Thus the coloring of the paths extends to the unique 2 -coloring of $T$. The leaves of $T$ are endpoints of paths, so they are all red. + +## §2.3 TSTST 2021/6, proposed by Nikolai Beluhov + +Available online at https://aops.com/community/p23864189. + +## Problem statement + +Triangles $A B C$ and $D E F$ share circumcircle $\Omega$ and incircle $\omega$ so that points $A, F$, $B, D, C$, and $E$ occur in this order along $\Omega$. Let $\Delta_{A}$ be the triangle formed by lines $A B, A C$, and $E F$, and define triangles $\Delta_{B}, \Delta_{C}, \ldots, \Delta_{F}$ similarly. Furthermore, let $\Omega_{A}$ and $\omega_{A}$ be the circumcircle and incircle of triangle $\Delta_{A}$, respectively, and define circles $\Omega_{B}, \omega_{B}, \ldots, \Omega_{F}, \omega_{F}$ similarly. +(a) Prove that the two common external tangents to circles $\Omega_{A}$ and $\Omega_{D}$ and the two common external tangents to circles $\omega_{A}$ and $\omega_{D}$ are either concurrent or pairwise parallel. +(b) Suppose that these four lines meet at point $T_{A}$, and define points $T_{B}$ and $T_{C}$ similarly. Prove that points $T_{A}, T_{B}$, and $T_{C}$ are collinear. +![](https://cdn.mathpix.com/cropped/2024_11_19_148cb373ffbe5370bbc6g-17.jpg?height=818&width=754&top_left_y=1150&top_left_x=651) + +Let $I$ and $r$ be the center and radius of $\omega$, and let $O$ and $R$ be the center and radius of $\Omega$. Let $O_{A}$ and $I_{A}$ be the circumcenter and incenter of triangle $\Delta_{A}$, and define $O_{B}$, $I_{B}, \ldots, I_{F}$ similarly. Let $\omega$ touch $E F$ at $A_{1}$, and define $B_{1}, C_{1}, \ldots, F_{1}$ similarly. + +I Part (a). All solutions to part (a) will prove the stronger claim that + +$$ +\left(\Omega_{A} \cup \omega_{A}\right) \sim\left(\Omega_{D} \cup \omega_{D}\right) +$$ + +The four lines will concur at the homothetic center of these figures (possibly at infinity). +Solution 1 (author) Let the second tangent to $\omega$ parallel to $E F$ meet lines $A B$ and $A C$ at $P$ and $Q$, respectively, and let the second tangent to $\omega$ parallel to $B C$ meet lines $D E$ and $D F$ at $R$ and $S$, respectively. Furthermore, let $\omega$ touch $P Q$ and $R S$ at $U$ and $V$, respectively. + +Let $h$ be inversion with respect to $\omega$. Then $h$ maps $A, B$, and $C$ onto the midpoints of the sides of triangle $D_{1} E_{1} F_{1}$. So $h$ maps $k$ onto the Euler circle of triangle $D_{1} E_{1} F_{1}$. +Similarly, $h$ maps $k$ onto the Euler circle of triangle $A_{1} B_{1} C_{1}$. Therefore, triangles $A_{1} B_{1} C_{1}$ and $D_{1} E_{1} F_{1}$ share a common nine-point circle $\gamma$. Let $K$ be its center; its radius equals $\frac{1}{2} r$. + +Let $H$ be the reflection of $I$ in $K$. Then $H$ is the common orthocenter of triangles $A_{1} B_{1} C_{1}$ and $D_{1} E_{1} F_{1}$. + +Let $\gamma_{U}$ of center $K_{U}$ and radius $\frac{1}{2} r$ be the Euler circle of triangle $U E_{1} F_{1}$, and let $\gamma_{V}$ of center $K_{V}$ and radius $\frac{1}{2} r$ be the Euler circle of triangle $V B_{1} C_{1}$. + +Let $H_{U}$ be the orthocenter of triangle $U E_{1} F_{1}$. Since quadrilateral $D_{1} E_{1} F_{1} U$ is cyclic, vectors $\overrightarrow{H H_{U}}$ and $\overrightarrow{D_{1} U}$ are equal. Consequently, $\overrightarrow{K K_{U}}=\frac{1}{2} \overrightarrow{D_{1} U}$. Similarly, $\overrightarrow{K K_{V}}=$ $\frac{1}{2} \overrightarrow{A_{1} V}$. +Since both of $A_{1} U$ and $D_{1} V$ are diameters in $\omega$, vectors $\overrightarrow{D_{1} U}$ and $\overrightarrow{A_{1} V}$ are equal. Therefore, $K_{U}$ and $K_{V}$ coincide, and so do $\gamma_{U}$ and $\gamma_{V}$. + +As above, $h$ maps $\gamma_{U}$ onto the circumcircle of triangle $A P Q$ and $\gamma_{V}$ onto the circumcircle of triangle $D R S$. Therefore, triangles $A P Q$ and $D R S$ are inscribed inside the same circle $\Omega_{A D}$. + +Since $E F$ and $P Q$ are parallel, triangles $\Delta_{A}$ and $A P Q$ are homothetic, and so are figures $\Omega_{A} \cup \omega_{A}$ and $\Omega_{A D} \cup \omega$. Consequently, we have + +$$ +\left(\Omega_{A} \cup \omega_{A}\right) \sim\left(\Omega_{A D} \cup \omega\right) \sim\left(\Omega_{D} \cup \omega_{D}\right), +$$ + +which solves part (a). +Solution 2 (Michael Ren) As in the previous solution, let the second tangent to $\omega$ parallel to $E F$ meet $A B$ and $A C$ at $P$ and $Q$, respectively. Let $(A P Q)$ meet $\Omega$ again at $D^{\prime}$, so that $D^{\prime}$ is the Miquel point of $\{A B, A C, B C, P Q\}$. Since the quadrilateral formed by these lines has incircle $\omega$, it is classical that $D^{\prime} I$ bisects $\angle P D^{\prime} C$ and $B D^{\prime} Q$ (e.g. by DDIT). + +Let $\ell$ be the tangent to $\Omega$ at $D^{\prime}$ and $D^{\prime} I$ meet $\Omega$ again at $M$. We have + +$$ +\measuredangle\left(\ell, D^{\prime} B\right)=\measuredangle D^{\prime} C B=\measuredangle D^{\prime} Q P=\measuredangle\left(D^{\prime} Q, E F\right) . +$$ + +Therefore $D^{\prime} I$ also bisects the angle between $\ell$ and the line parallel to $E F$ through $D^{\prime}$. This means that $M$ is one of the arc midpoints of $E F$. Additionally, $D^{\prime}$ lies on $\operatorname{arc} B C$ not containing $A$, so $D^{\prime}=D$. + +Similarly, letting the second tangent to $\omega$ parallel to $B C$ meet $D E$ and $D F$ again at $R$ and $S$, we have $A D R S$ cyclic. + +## Lemma + +There exists a circle $\Omega_{A D}$ tangent to $\Omega_{A}$ and $\Omega_{D}$ at $A$ and $D$, respectively. + +Proof. (This step is due to Ankan Bhattacharya.) It is equivalent to have $\measuredangle O A O_{A}=$ $\measuredangle O_{D} D O$. Taking isogonals with respect to the shared angle of $\triangle A B C$ and $\Delta_{A}$, we see that + +$$ +\measuredangle O A O_{A}=\measuredangle(\perp E F, \perp B C)=\measuredangle(E F, B C) . +$$ + +(Here, $\perp E F$ means the direction perpendicular to $E F$.) By symmetry, this is equal to $\measuredangle O_{D} D O$. + +The circle $(A D P Q)$ passes through $A$ and $D$, and is tangent to $\Omega_{A}$ by homothety. Therefore it coincides with $\Omega_{A D}$, as does ( $A D R S$ ). Like the previous solution, we finish with + +$$ +\left(\Omega_{A} \cup \omega_{A}\right) \sim\left(\Omega_{A D} \cup \omega\right) \sim\left(\Omega_{D} \cup \omega_{D}\right) . +$$ + +Solution 3 (Andrew Gu) Construct triangles homothetic to $\Delta_{A}$ and $\Delta_{D}$ (with positive ratio) which have the same circumcircle; it suffices to show that these copies have the same incircle as well. Let $O$ be the center of this common circumcircle, which we take to be the origin, and $M_{X Y}$ denote the point on the circle such that the tangent at that point is parallel to line $X Y$ (from the two possible choices, we make the choice that corresponds to the arc midpoint on $\Omega$, e.g. $M_{A B}$ should correspond to the arc midpoint on the internal angle bisector of $A C B$ ). The left diagram below shows the original triangles $A B C$ and $D E F$, while the right diagram shows the triangles homothetic to $\Delta_{A}$ and $\Delta_{D}$. +![](https://cdn.mathpix.com/cropped/2024_11_19_148cb373ffbe5370bbc6g-19.jpg?height=566&width=1203&top_left_y=902&top_left_x=432) + +Using the fact that the incenter is the orthocenter of the arc midpoints, the incenter of $\Delta_{A}$ in this reference frame is $M_{A B}+M_{A C}-M_{E F}$ and the incenter of $\Delta_{D}$ in this reference frame is $M_{D E}+M_{D F}-M_{B C}$. Since $A B C$ and $D E F$ share a common incenter, we have + +$$ +M_{A B}+M_{B C}+M_{C A}=M_{D E}+M_{E F}+M_{F D} +$$ + +Thus the copies of $\Delta_{A}$ and $\Delta_{D}$ have the same incenter, and therefore the same incircle as well (Euler's theorem determines the inradius). + +【 Part (b). We present several solutions for this part of the problem. Solutions 3 and 4 require solving part (a) first, while the others do not. Solutions 1,4 , and 5 define $T_{A}$ solely as the exsimilicenter of $\omega_{A}$ and $\omega_{D}$, whereas solutions 2 and 3 define $T_{A}$ solely as the exsimilicenter of $\Omega_{A}$ and $\Omega_{D}$. + +Solution 1 (author) By Monge's theorem applied to $\omega, \omega_{A}$, and $\omega_{D}$, points $A, D$, and $T_{A}$ are collinear. Therefore, $T_{A}=A D \cap I_{A} I_{D}$. + +Let $p$ be pole-and-polar correspondence with respect to $\omega$. Then $p$ maps $A$ onto line $E_{1} F_{1}$ and $D$ onto line $B_{1} C_{1}$. Consequently, $p$ maps line $A D$ onto $X_{A}=B_{1} C_{1} \cap E_{1} F_{1}$. + +Furthermore, $p$ maps the line that bisects the angle formed by lines $A B$ and $E F$ and does not contain $I$ onto the midpoint of segment $A_{1} F_{1}$. Similarly, $p$ maps the line that bisects the angle formed by lines $A C$ and $E F$ and does not contain $I$ onto the midpoint of segment $A_{1} E_{1}$. So $p$ maps $I_{A}$ onto the midline of triangle $A_{1} E_{1} F_{1}$ opposite $A_{1}$. Similarly, $p$ maps $I_{D}$ onto the midline of triangle $D_{1} B_{1} C_{1}$ opposite $D_{1}$. Consequently, $p$ maps line +$I_{A} I_{D}$ onto the intersection point $Y_{A}$ of this pair of midlines, and $p$ maps $T_{A}$ onto line $X_{A} Y_{A}$. + +As in the solution to part (a), let $H$ be the common orthocenter of triangles $A_{1} B_{1} C_{1}$ and $D_{1} E_{1} F_{1}$. Let $H_{A}$ be the foot of the altitude from $A_{1}$ in triangle $A_{1} B_{1} C_{1}$ and let $H_{D}$ be the foot of the altitude from $D_{1}$ in triangle $D_{1} E_{1} F_{1}$. Furthermore, let $L_{A}=H A_{1} \cap E_{1} F_{1}$ and $L_{D}=H D_{1} \cap B_{1} C_{1}$. + +Since the reflection of $H$ in line $B_{1} C_{1}$ lies on $\omega, A_{1} H \cdot H H_{A}$ equals half the power of $H$ with respect to $\omega$. Similarly, $D_{1} H \cdot H H_{D}$ equals half the power of $H$ with respect to $\omega$. + +Then $A_{1} H \cdot H H_{A}=D_{1} H \cdot H H_{D}$ and $A_{1} H H_{D} \sim D_{1} H H_{A}$. Since $\angle H H_{D} L_{A}=90^{\circ}=$ $\angle H H_{A} L_{D}$, figures $A_{1} H H_{D} L_{A}$ and $D_{1} H H_{A} L_{D}$ are similar as well. Consequently, + +$$ +\frac{H L_{A}}{L_{A} A_{1}}=\frac{H L_{D}}{L_{D} D_{1}}=s +$$ + +as a signed ratio. +Let the line through $A_{1}$ parallel to $E_{1} F_{1}$ and the line through $D_{1}$ parallel to $B_{1} C_{1}$ meet at $Z_{A}$. Then $H X_{A} / X_{A} Z_{A}=s$ and $Y_{A}$ is the midpoint of segment $X_{A} Z_{A}$. Therefore, $H$ lies on line $X_{A} Y_{A}$. This means that $T_{A}$ lies on the polar of $H$ with respect to $\omega$, and by symmetry so do $T_{B}$ and $T_{C}$. + +Solution 2 (author) As in the first solution to part (a), let $h$ be inversion with respect to $\omega$, let $\gamma$ of center $K$ be the common Euler circle of triangles $A_{1} B_{1} C_{1}$ and $D_{1} E_{1} F_{1}$, and let $H$ be their common orthocenter. + +Again as in the solution to part (a), $h$ maps $\Omega_{A}$ onto the nine-point circle $\gamma_{A}$ of triangle $A_{1} E_{1} F_{1}$ and $\Omega_{D}$ onto the nine-point circle $\gamma_{D}$ of triangle $D_{1} B_{1} C_{1}$. + +Let $K_{A}$ and $K_{D}$ be the centers of $\gamma_{A}$ and $\gamma_{D}$, respectively, and let $\ell_{A}$ be the perpendicular bisector of segment $K_{A} K_{D}$. Since $\gamma_{A}$ and $\gamma_{D}$ are congruent (both of them are of radius $\left.\frac{1}{2} r\right)$, they are reflections of each other across $\ell_{A}$. + +Inversion $h$ maps the two common external tangents of $\Omega_{A}$ and $\Omega_{D}$ onto the two circles $\alpha$ and $\beta$ through $I$ that are tangent to both of $\gamma_{A}$ and $\gamma_{D}$ in the same way. (That is, either internally to both or externally to both.) Consequently, $\alpha$ and $\beta$ are reflections of each other in $\ell_{A}$ and so their second point of intersection $S_{A}$, which $h$ maps $T_{A}$ onto, is the reflection of $I$ in $\ell_{A}$. + +Define $\ell_{B}, \ell_{C}, S_{B}$, and $S_{C}$ similarly. Then $S_{B}$ is the reflection of $I$ in $\ell_{B}$ and $S_{C}$ is the reflection of $I$ in $\ell_{C}$. + +As in the solution to part (a), $\overrightarrow{K K_{A}}=\frac{1}{2} \overrightarrow{D_{1} A_{1}}$ and $\overrightarrow{K K_{D}}=\frac{1}{2} \overrightarrow{A_{1} D_{1}}$. Consequently, $K$ is the midpoint of segment $K_{A} K_{D}$ and so $K$ lies on $\ell_{A}$. Similarly, $K$ lies on $\ell_{B}$ and $\ell_{C}$. + +Therefore, all four points $I, S_{A}, S_{B}$, and $S_{C}$ lie on the circle of center $K$ that contains $I$. (This is also the circle on diameter $I H$.) Since points $S_{A}, S_{B}$, and $S_{C}$ are concyclic with $I$, their images $T_{A}, T_{B}$, and $T_{C}$ under $h$ are collinear, and the solution is complete. + +Solution 3 (Ankan Bhattacharya) From either of the first two solutions to part (a), we know that there is a circle $\Omega_{A D}$ passing through $A$ and $D$ which is (internally) tangent to $\Omega_{A}$ and $\Omega_{D}$. By Monge's theorem applied to $\Omega_{A}, \Omega_{D}$, and $\Omega_{A D}$, it follows that $A, D$, and $T_{A}$ are collinear. + +The inversion at $T_{A}$ swapping $\Omega_{A}$ with $\Omega_{D}$ also swaps $A$ with $D$ because $T_{A}$ lies on $A D$ and $A$ is not homologous to $D$. Let $\Omega_{A}$ and $\Omega_{D}$ meet $\Omega$ again at $L_{A}$ and $L_{D}$. Since $A D L_{A} L_{D}$ is cyclic, the same inversion at $T_{A}$ also swaps $L_{A}$ and $L_{D}$, so $T_{A}=A D \cap L_{A} L_{D}$. + +By Taiwan TST 2014, $L_{A}$ and $L_{D}$ are the tangency points of the $A$-mixtilinear and $D$-mixtilinear incircles, respectively, with $\Omega$. Therefore $A L_{A} \cap D L_{D}$ is the exsimilicenter $X$ of $\Omega$ and $\omega$. Then $T_{A}, T_{B}$, and $T_{C}$ all lie on the polar of $X$ with respect to $\Omega$. + +Solution 4 (Andrew Gu) We show that $T_{A}$ lies on the radical axis of the point circle at $I$ and $\Omega$, which will solve the problem. Let $I_{A}$ and $I_{D}$ be the centers of $\omega_{A}$ and $\omega_{D}$ respectively. By the Monge's theorem applied to $\omega, \omega_{A}$, and $\omega_{D}$, points $A, D$, and $T_{A}$ are collinear. Additionally, these other triples are collinear: $\left(A, I_{A}, I\right),\left(D, I_{D}, I\right),\left(I_{A}, I_{D}, T\right)$. By Menelaus's theorem, we have + +$$ +\frac{T_{A} D}{T_{A} A}=\frac{I_{A} I}{I_{A} A} \cdot \frac{I_{D} D}{I_{D} I} . +$$ + +If $s$ is the length of the side opposite $A$ in $\Delta_{A}$, then we compute + +$$ +\begin{aligned} +\frac{I_{A} I}{I_{A} A} & =\frac{s / \cos (A / 2)}{r_{A} / \sin (A / 2)} \\ +& =\frac{2 R_{A} \sin (A) \sin (A / 2)}{\cos (A / 2)} \\ +& =\frac{4 R_{A} \sin ^{2}(A / 2)}{r_{A}} \\ +& =\frac{4 R_{A} r^{2}}{r_{A} A I^{2}} . +\end{aligned} +$$ + +From part (a), we know that $\frac{R_{A}}{r_{A}}=\frac{R_{D}}{r_{D}}$. Therefore, doing a similar calculation for $\frac{I_{D} D}{I_{D} I}$, we get + +$$ +\begin{aligned} +\frac{T_{A} D}{T_{A} A} & =\frac{I_{A} I}{I_{A} A} \cdot \frac{I_{D} D}{I_{D} I} \\ +& =\frac{4 R_{A} r^{2}}{r_{A} A I^{2}} \cdot \frac{r_{D} D I^{2}}{4 R_{D} r^{2}} \\ +& =\frac{D I^{2}}{A I^{2}} +\end{aligned} +$$ + +Thus $T_{A}$ is the point where the tangent to $(A I D)$ at $I$ meets $A D$ and $T_{A} I^{2}=T_{A} A \cdot T_{A} D$. This shows what we claimed at the start. + +Solution 5 (Ankit Bisain) As in the previous solution, it suffices to show that $\frac{I_{A} I}{A I_{A}} \cdot \frac{D I_{D}}{I_{D} I}=\frac{D I^{2}}{A I^{2}}$. Let $A I$ and $D I$ meet $\Omega$ again at $M$ and $N$, respectively. Let $\ell$ be the line parallel to $B C$ and tangent to $\omega$ but different from $B C$. Then + +$$ +\frac{D I_{D}}{I_{D} I}=\frac{d(D, B C)}{d(B C, \ell)}=\frac{D B \cdot D C / 2 R}{2 r}=\frac{M I^{2}-M D^{2}}{4 R r} +$$ + +Since $I D M \sim I A N$, we have + +$$ +\frac{D I_{D}}{I_{D} I} \cdot \frac{I_{A} I}{A I_{A}}=\frac{M I^{2}-M D^{2}}{N I^{2}-N A^{2}}=\frac{D I^{2}}{A I^{2}} +$$ + +as desired. +Remark (Author comments on generalization of part (b) with a circumscribed hexagram). Let triangles $A B C$ and $D E F$ be circumscribed about the same circle $\omega$ so that they form a hexagram. However, we do not require anymore that they are inscribed in the same circle. + +Define circles $\Omega_{A}, \omega_{A}, \ldots, \omega_{F}$ as in the problem. Let $T_{A}^{\text {Circ }}$ be the intersection point of the two common external tangents to circles $\Omega_{A}$ and $\Omega_{D}$, and define points $T_{B}^{\text {Circ }}$ and $T_{C}^{\text {Circ }}$ similarly. Also let $T_{A}^{\text {In }}$ be the intersection point of the two common external tangents to circles $\omega_{A}$ and $\omega_{D}$, and define points $T_{B}^{\mathrm{In}}$ and $T_{C}^{\mathrm{In}}$ similarly. + +Then points $T_{A}^{\text {Circ }}, T_{B}^{\text {Circ }}$, and $T_{C}^{\mathrm{Circ}}$ are collinear and points $T_{A}^{\mathrm{In}}, T_{B}^{\mathrm{In}}$, and $T_{C}^{\mathrm{In}}$ are also collinear. + +The second solution to part (b) of the problem works also for the circumcircles part of the generalisation. To see that segments $K_{A} K_{D}, K_{B} K_{E}$, and $K_{C} K_{F}$ still have a common midpoint, let $M$ be the centroid of points $A, B, C, D, E$, and $F$. Then the midpoint of segment $K_{A} K_{D}$ divides segment $O M$ externally in ratio $3: 1$, and so do the other two midpoints as well. + +For the incircles part of the generalisation, we start out as in the first solution to part (b) of the problem, and eventually we reduce everything to the following: + +Let points $A_{1}, B_{1}, C_{1}, D_{1}, E_{1}$, and $F_{1}$ lie on circle $\omega$. Let lines $B_{1} C_{1}$ and $E_{1} F_{1}$ meet at point $X_{A}$, let the line through $A_{1}$ parallel to $B_{1} C_{1}$ and the line through $D_{1}$ parallel to $E_{1} F_{1}$ meet at point $Z_{A}$, and define points $X_{B}, Z_{B}, X_{C}$, and $Z_{C}$ similarly. Then lines $X_{A} Z_{A}$, $X_{B} Z_{B}$, and $X_{C} Z_{C}$ are concurrent. + +Take $\omega$ as the unit circle and assign complex numbers $u, v, w, x, y$, and $z$ to points $A_{1}$, $F_{1}, B_{1}, D_{1}, C_{1}$, and $E_{1}$, respectively, so that when we permute $u, v, w, x, y$, and $z$ cyclically the configuration remains unchanged. Then by standard complex bash formulas we obtain that each two out of our three lines meet at $\varphi / \psi$, where + +$$ +\varphi=\sum_{\mathrm{Cyc}} u^{2} v w(w x-w y+x y)(y-z) +$$ + +and + +$$ +\psi=-u^{2} w^{2} y^{2}-v^{2} x^{2} z^{2}-4 u v w x y z+\sum_{\mathrm{Cyc}} u^{2}(v w x y-v w x z+v w y z-v x y z+w x y z) +$$ + +(But the calculations were too difficult for me to do by hand, so I used SymPy.) + +Remark (Author comments on generalization of part (b) with an inscribed hexagram). Let triangles $A B C$ and $D E F$ be inscribed inside the same circle $\Omega$ so that they form a hexagram. However, we do not require anymore that they are circumscribed about the same circle. + +Define points $T_{A}^{\text {Circ }}, T_{B}^{\text {Circ }}, \ldots, T_{C}^{\mathrm{In}}$ as in the previous remark. It looks like once again points $T_{A}^{\text {Circ }}, T_{B}^{\text {Circ }}$, and $T_{C}^{\text {Circ }}$ are collinear and points $T_{A}^{\mathrm{In}}, T_{B}^{\mathrm{In}}$, and $T_{C}^{\mathrm{In}}$ are also collinear. However, I do not have proofs of these claims. + +Remark (Further generalization from Andrew Gu). Let $A B C$ and $D E F$ be triangles which share an inconic, or equivalently share a circumconic. Define points $T_{A}^{\text {Circ }}, T_{B}^{\text {Circ }}, \ldots, T_{C}^{\mathrm{In}}$ as in the previous remarks. Then it is conjectured that points $T_{A}^{\text {Circ }}, T_{B}^{\text {Circ }}$, and $T_{C}^{\text {Circ }}$ are collinear and points $T_{A}^{\mathrm{In}}, T_{B}^{\mathrm{In}}$, and $T_{C}^{\mathrm{In}}$ are also collinear. (Note that extraversion may be required depending on the configuration of points, e.g. excircles instead of incircles.) Additionally, it appears that the insimilicenters of the circumcircles lie on a line perpendicular to the line through $T_{A}^{\text {Circ }}, T_{B}^{\text {Circ }}$, and $T_{C}^{\text {Circ }}$. + +## §3 Solutions to Day 3 + +## §3.1 TSTST 2021/7, proposed by Ankit Bisain, Holden Mui + +Available online at https://aops.com/community/p24130213. + +## Problem statement + +Let $M$ be a finite set of lattice points and $n$ be a positive integer. A mine-avoiding path is a path of lattice points with length $n$, beginning at $(0,0)$ and ending at a point on the line $x+y=n$, that does not contain any point in $M$. Prove that if there exists a mine-avoiding path, then there exist at least $2^{n-|M|}$ mine-avoiding paths. + +We present two approaches. + +【 Solution 1. We prove the statement by induction on $n$. We use $n=0$ as a base case, where the statement follows from $1 \geq 2^{-|M|}$. For the inductive step, let $n>0$. There exists at least one mine-avoiding path, which must pass through either $(0,1)$ or $(1,0)$. We consider two cases: + +Case 1: there exist mine-avoiding paths starting at both $(1,0)$ and $(0,1)$. +By the inductive hypothesis, there are at least $2^{n-1-|M|}$ mine-avoiding paths starting from each of $(1,0)$ and $(0,1)$. Then the total number of mine-avoiding paths is at least $2^{n-1-|M|}+2^{n-1-|M|}=2^{n-|M|}$. + +Case 2: only one of $(1,0)$ and $(0,1)$ is on a mine-avoiding path. +Without loss of generality, suppose no mine-avoiding path starts at $(0,1)$. Then some element of $M$ must be of the form $(0, k)$ for $1 \leq k \leq n$ in order to block the path along the $y$-axis. This element can be ignored for any mine-avoiding path starting at (1,0). By the inductive hypothesis, there are at least $2^{n-1-(|M|-1)}=2^{n-|M|}$ mine-avoiding paths. + +This completes the induction step, which solves the problem. + +## 【 Solution 2. + +## Lemma + +If $|M|d$, the pattern changes to + +$$ +S_{n}=\sum_{j=1}^{d}(-1)^{j+1} c_{j} S_{n-j} +$$ + +## Lemma + +All of the $c_{i}$ are integers except for $c_{d}$. Furthermore, $c_{d}$ is $1 / p$ times an integer. + +Proof. The $q$ th cyclotomic polynomial is + +$$ +\Phi_{q}(x)=1+x^{p^{r-1}}+x^{2 p^{r-1}}+\cdots+x^{(p-1) p^{r-1}} +$$ + +The polynomial + +$$ +Q(x)=1+(1+x)^{p^{r-1}}+(1+x)^{2 p^{r-1}}+\cdots+(1+x)^{(p-1) p^{r-1}} +$$ + +has roots $\omega-1$ for $\omega \in S_{q}$, so it is equal to $p(-x)^{d} P(-1 / x)$ by comparing constant coefficients. Comparing the remaining coefficients, we find that $c_{n}$ is $1 / p$ times the $x^{n}$ coefficient of $Q$. + +Since $(x+y)^{p} \equiv x^{p}+y^{p}(\bmod p)$, we conclude that, modulo $p$, + +$$ +\begin{aligned} +Q(x) & \equiv 1+\left(1+x^{p^{r-1}}\right)+\left(1+x^{p^{r-1}}\right)^{2}+\cdots+\left(1+x^{p^{r-1}}\right)^{p-1} \\ +& \equiv\left[\left(1+x^{p^{r-1}}\right)^{p}-1\right] / x^{p^{r-1}} +\end{aligned} +$$ + +Since $\binom{p}{j}$ is a multiple of $p$ when $0d$ : + +- If $\nu_{p}\left(S_{n-j}\right) \geq \ell$ for all $1 \leq j \leq d$ and $\nu_{p}\left(S_{n-d}\right) \geq \ell+1$, then $\nu_{p}\left(S_{n}\right) \geq \ell$. +- If $\nu_{p}\left(S_{n-j}\right) \geq \ell$ for all $1 \leq j \leq d$ and $\nu_{p}\left(S_{n-d}\right)=\ell$, then $\nu_{p}\left(S_{n}\right)=\ell-1$. + +Together, these prove the claim by induction. +By the claim, the smallest $n$ for which $\nu_{p}\left(S_{n}\right)<0$ (equivalent to $S_{n}$ not being an integer, by the recurrences) is + +$$ +n=(r-1) d+m+1=((p-1) r-1) p^{r-1}+1 +$$ + +Remark. The original proposal was the following more general version: +Let $n$ be an integer with prime power factorization $q_{1} \cdots q_{m}$. Let $S_{n}$ denote the set of primitive $n$th roots of unity. Find all tuples of nonnegative integers $\left(z_{1}, \ldots, z_{m}\right)$ such that + +$$ +\sum_{\omega \in S_{n}} \frac{f(\omega)}{\left(1-\omega^{n / q_{1}}\right)^{z_{1}} \cdots\left(1-\omega^{n / q_{m}}\right)^{z_{m}}} \in \mathbb{Z} +$$ + +for all polynomials $f \in \mathbb{Z}[x]$. +The maximal $z_{i}$ are exponents in the prime ideal factorization of the different ideal of the cyclotomic extension $\mathbb{Q}\left(\zeta_{n}\right) / \mathbb{Q}$. + +Remark. Let $F=\left(x^{p}-1\right) /(x-1)$ be the minimal polynomial of $\zeta_{p}=e^{2 \pi i / p}$ over $\mathbb{Q}$. A calculation of Euler shows that + +$$ +\left(\mathbb{Z}\left[\zeta_{p}\right]\right)^{*}:=\left\{\alpha=g\left(\zeta_{p}\right) \in \mathbb{Q}\left[\zeta_{p}\right]: \sum_{\omega \in S_{p}} f(\omega) g(\omega) \in \mathbb{Z} \forall f \in \mathbb{Z}[x]\right\}=\frac{1}{F^{\prime}\left(\zeta_{p}\right)} \cdot \mathbb{Z}\left[\zeta_{p}\right], +$$ + +where + +$$ +F^{\prime}\left(\zeta_{p}\right)=\frac{p \zeta_{p}^{p-1}-\left[1+\zeta_{p}+\cdots+\zeta_{p}^{p-1}\right]}{1-\zeta_{p}}=p\left(1-\zeta_{p}\right)^{-1} \zeta_{p}^{p-1} +$$ + +is $\left(1-\zeta_{p}\right)^{[p-1]-1}=\left(1-\zeta_{p}\right)^{p-2}$ times a unit of $\mathbb{Z}\left[\zeta_{p}\right]$. Here, $\left(\mathbb{Z}\left[\zeta_{p}\right]\right)^{*}$ is the dual lattice of $\mathbb{Z}\left[\zeta_{p}\right]$. + +Remark. Let $K=\mathbb{Q}(\omega)$, so $(p)$ factors as $(1-\omega)^{p-1}$ in the ring of integers $\mathcal{O}_{K}$ (which, for cyclotomic fields, can be shown to be $\mathbb{Z}[\omega])$. In particular, the ramification index $e$ of $(1-\omega)$ over $p$ is the exponent, $p-1$. Since $e=p-1$ is not divisible by $p$, we have so-called tame ramification. Now by the ramification theory of Dedekind's different ideal, the exponent $z_{1}$ that works when $n=p$ is $e-1=p-2$. + +Higher prime powers are more interesting because of wild ramification: $p$ divides $\phi\left(p^{r}\right)=$ $p^{r-1}(p-1)$ if and only if $r>1$. (This is a similar phenomena to how Hensel's lemma for $x^{2}-c$ is more interesting mod powers of 2 than mod odd prime powers.) + +Remark. Let $F=\left(x^{q}-1\right) /\left(x^{q / p}-1\right)$ be the minimal polynomial of $\zeta_{q}=e^{2 \pi i / q}$ over $\mathbb{Q}$. The aforementioned calculation of Euler shows that + +$$ +\left(\mathbb{Z}\left[\zeta_{q}\right]\right)^{*}:=\left\{\alpha=g\left(\zeta_{q}\right) \in \mathbb{Q}\left[\zeta_{q}\right]: \sum_{\omega \in S_{q}} f(\omega) g(\omega) \in \mathbb{Z} \forall f \in \mathbb{Z}[x]\right\}=\frac{1}{F^{\prime}\left(\zeta_{q}\right)} \cdot \mathbb{Z}\left[\zeta_{q}\right] +$$ + +where the chain rule implies (using the computation from the prime case) + +$$ +F^{\prime}\left(\zeta_{q}\right)=\left[p\left(1-\zeta_{p}\right)^{-1} \zeta_{p}^{p-1}\right] \cdot \frac{q}{p} \zeta_{q}^{(q / p)-1}=q\left(1-\zeta_{p}\right)^{-1} \zeta_{q}^{-1} +$$ + +is $\left(1-\zeta_{q}\right)^{r \phi(q)-q / p}=\left(1-\zeta_{q}\right)^{z_{q}}$ times a unit of $\mathbb{Z}\left[\zeta_{q}\right]$. + diff --git a/USA_TST_TST/md/en-sols-TSTST-2022.md b/USA_TST_TST/md/en-sols-TSTST-2022.md new file mode 100644 index 0000000000000000000000000000000000000000..06494c30d0629d2b49cad96fff265e16d4d4d597 --- /dev/null +++ b/USA_TST_TST/md/en-sols-TSTST-2022.md @@ -0,0 +1,618 @@ +# USA TSTST 2022 Solutions
United States of America - TST Selection Test
Andrew Gu and Evan Chen
64 ${ }^{\text {th }}$ IMO 2023 Japan and $12^{\text {th }}$ EGMO 2023 Slovenia + +## Contents + +0 Problems ..... 2 +1 Solutions to Day 1 ..... 4 +1.1 TSTST 2022/1, proposed by Holden Mui ..... 4 +1.2 TSTST 2022/2, proposed by Hongzhou Lin ..... 7 +1.3 TSTST 2022/3 ..... 9 +2 Solutions to Day 2 ..... 11 +2.1 TSTST 2022/4, proposed by Merlijn Staps ..... 11 +2.2 TSTST 2022/5, proposed by Ray Li ..... 12 +2.3 TSTST 2022/6, proposed by Hongzhou Lin ..... 14 +3 Solutions to Day 3 ..... 17 +3.1 TSTST 2022/7, proposed by Merlijn Staps ..... 17 +3.2 TSTST 2022/8, proposed by Merlijn Staps ..... 18 +3.3 TSTST 2022/9, proposed by Vincent Huang ..... 19 + +## §0 Problems + +1. Let $n$ be a positive integer. Find the smallest positive integer $k$ such that for any set $S$ of $n$ points in the interior of the unit square, there exists a set of $k$ rectangles such that the following hold: + +- The sides of each rectangle are parallel to the sides of the unit square. +- Each point in $S$ is not in the interior of any rectangle. +- Each point in the interior of the unit square but not in $S$ is in the interior of at least one of the $k$ rectangles. +(The interior of a polygon does not contain its boundary.) + +2. Let $A B C$ be a triangle. Let $\theta$ be a fixed angle for which + +$$ +\theta<\frac{1}{2} \min (\angle A, \angle B, \angle C) +$$ + +Points $S_{A}$ and $T_{A}$ lie on segment $B C$ such that $\angle B A S_{A}=\angle T_{A} A C=\theta$. Let $P_{A}$ and $Q_{A}$ be the feet from $B$ and $C$ to $\overline{A S_{A}}$ and $\overline{A T_{A}}$ respectively. Then $\ell_{A}$ is defined as the perpendicular bisector of $\overline{P_{A} Q_{A}}$. +Define $\ell_{B}$ and $\ell_{C}$ analogously by repeating this construction two more times (using the same value of $\theta$ ). Prove that $\ell_{A}, \ell_{B}$, and $\ell_{C}$ are concurrent or all parallel. +3. Determine all positive integers $N$ for which there exists a strictly increasing sequence of positive integers $s_{0}1$ be a fixed positive integer. Prove that if $n$ is a sufficiently large positive integer, there exists a sequence of integers with the following properties: + +- Each element of the sequence is between 1 and $n$, inclusive. +- For any two different contiguous subsequences of the sequence with length between 2 and $k$ inclusive, the multisets of values in those two subsequences is not the same. +- The sequence has length at least $0.499 n^{2}$. + + +## §1 Solutions to Day 1 + +## §1.1 TSTST 2022/1, proposed by Holden Mui + +Available online at https://aops.com/community/p25516960. + +## Problem statement + +Let $n$ be a positive integer. Find the smallest positive integer $k$ such that for any set $S$ of $n$ points in the interior of the unit square, there exists a set of $k$ rectangles such that the following hold: + +- The sides of each rectangle are parallel to the sides of the unit square. +- Each point in $S$ is not in the interior of any rectangle. +- Each point in the interior of the unit square but not in $S$ is in the interior of at least one of the $k$ rectangles. +(The interior of a polygon does not contain its boundary.) + +We give the author's solution. In terms of $n$, we wish find the smallest integer $k$ for which $(0,1)^{2} \backslash S$ is always a union of $k$ open rectangles for every set $S \subset(0,1)^{2}$ of size $n$. + +We claim the answer is $k=2 n+2$. +The lower bound is given by picking + +$$ +S=\left\{\left(s_{1}, s_{1}\right),\left(s_{2}, s_{2}\right), \ldots,\left(s_{n}, s_{n}\right)\right\} +$$ + +for some real numbers $00$. The four rectangles covering each of + +$$ +\left(s_{1}-\varepsilon, s_{1}\right),\left(s_{1}, s_{1}-\varepsilon\right),\left(s_{n}+\varepsilon, s_{n}\right),\left(s_{n}, s_{n}+\varepsilon\right) +$$ + +cannot cover any other points in $S^{\prime}$; all other rectangles can only cover at most 2 points in $S^{\prime}$, giving a bound of + +$$ +k \geq 4+\frac{\left|S^{\prime}\right|-4}{2}=2 n+2 +$$ + +![](https://cdn.mathpix.com/cropped/2024_11_19_1799270e91f74486a977g-05.jpg?height=812&width=806&top_left_y=242&top_left_x=628) + +To prove that $2 n+2$ rectangles are sufficient, assume that the number of distinct $y$-coordinates is at least the number of distinct $x$-coordinates. Let + +$$ +0=x_{0}0$ gives a total of + +$$ +(m+n+2)+(n-m)=2 n+2 +$$ + +rectangles. + +## §1.2 TSTST 2022/2, proposed by Hongzhou Lin + +Available online at https://aops.com/community/p25516988. + +## Problem statement + +Let $A B C$ be a triangle. Let $\theta$ be a fixed angle for which + +$$ +\theta<\frac{1}{2} \min (\angle A, \angle B, \angle C) +$$ + +Points $S_{A}$ and $T_{A}$ lie on segment $B C$ such that $\angle B A S_{A}=\angle T_{A} A C=\theta$. Let $P_{A}$ and $Q_{A}$ be the feet from $B$ and $C$ to $\overline{A S_{A}}$ and $\overline{A T_{A}}$ respectively. Then $\ell_{A}$ is defined as the perpendicular bisector of $\overline{P_{A} Q_{A}}$. + +Define $\ell_{B}$ and $\ell_{C}$ analogously by repeating this construction two more times (using the same value of $\theta$ ). Prove that $\ell_{A}, \ell_{B}$, and $\ell_{C}$ are concurrent or all parallel. + +We discard the points $S_{A}$ and $T_{A}$ since they are only there to direct the angles correctly in the problem statement. + +【 First solution, by author. Let $X$ be the projection from $C$ to $A P_{A}, Y$ be the projection from $B$ to $A Q_{A}$. +![](https://cdn.mathpix.com/cropped/2024_11_19_1799270e91f74486a977g-07.jpg?height=809&width=792&top_left_y=1292&top_left_x=632) + +Claim - Line $\ell_{A}$ passes through $M_{A}$, the midpoint of $B C$. Also, quadrilateral $P_{A} Q_{A} Y X$ is cyclic with circumcenter $M_{A}$. + +Proof. Since + +$$ +A P_{A} \cdot A X=A B \cdot A C \cdot \cos \theta \cos (\angle A-\theta)=A Q_{A} \cdot A Y +$$ + +it follows that $P_{A}, Q_{A}, Y, X$ are concyclic by power of a point. Moreover, by projection, the perpendicular bisector of $P_{A} X$ passes through $M_{A}$, similar for $Q_{A} Y$, implying that $M_{A}$ is the center of $P_{A} Q_{A} Y X$. Hence $\ell_{A}$ passes through $M_{A}$. + +$$ +\text { Claim }-\measuredangle\left(M_{A} M_{C}, \ell_{A}\right)=\measuredangle Y P_{A} Q_{A} +$$ + +Proof. Indeed, $\ell_{A} \perp P_{A} Q_{A}$, and $M_{A} M_{C} \perp P_{A} Y$ (since $M_{A} P_{A}=M_{A} Y$ from $\left(P_{A} Q_{A} Y_{A} X\right)$ and $M_{C} P_{A}=M_{C} M_{A}=M_{C} Y$ from the circle with diameter $\left.A B\right)$. Hence $\measuredangle\left(M_{A} M_{C}, \ell_{A}\right)=$ $\measuredangle\left(P_{A} Y, P_{A} Q_{A}\right)=\measuredangle Y P_{A} Q_{A}$. + +Therefore, + +$$ +\frac{\sin \angle\left(M_{A} M_{C}, \ell_{A}\right)}{\sin \angle\left(\ell_{A}, M_{A} M_{B}\right)}=\frac{\sin \angle Y P_{A} Q_{A}}{\sin \angle P_{A} Q_{A} X}=\frac{Y Q_{A}}{X P_{A}}=\frac{B C \sin (\angle C+\theta)}{B C \sin (\angle B+\theta)}=\frac{\sin (\angle C+\theta)}{\sin (\angle B+\theta)}, +$$ + +and we conclude by trig Ceva theorem. +【 Second solution via Jacobi, by Maxim Li. Let $D$ be the foot of the $A$-altitude. Note that line $B C$ is the external angle bisector of $\angle P_{A} D Q_{A}$. + +Claim - $\left(D P_{A} Q_{A}\right)$ passes through the midpoint $M_{A}$ of $B C$. +Proof. Perform $\sqrt{b c}$ inversion. Then the intersection of $B C$ and $\left(D P_{A} Q_{A}\right)$ maps to the second intersection of $(A B C)$ and $\left(A^{\prime} P_{A} Q_{A}\right)$, where $A^{\prime}$ is the antipode to $A$ on $(A B C)$, i.e. the center of spiral similarity from $B C$ to $P_{A} Q_{A}$. Since $B P_{A}: C Q_{A}=A B: A C$, we see the center of spiral similarity is the intersection of the $A$-symmedian with $(A B C)$, which is the image of $M_{A}$ in the inversion. + +It follows that $M_{A}$ lies on $\ell_{A}$; we need to identify a second point. We'll use the circumcenter $O_{A}$ of $\left(D P_{A} Q_{A}\right)$. The perpendicular bisector of $D P_{A}$ passes through $M_{C}$; indeed, we can easily show the angle it makes with $M_{C} M_{A}$ is $90^{\circ}-\theta-C$, so $\angle O_{A} M_{C} M_{A}=90-\theta-C$, and then by analogous angle-chasing we can finish with Jacobi's theorem on $\triangle M_{A} M_{B} M_{C}$. + +## §1.3 TSTST 2022/3 + +Available online at https://aops.com/community/p25517008. + +## Problem statement + +Determine all positive integers $N$ for which there exists a strictly increasing sequence of positive integers $s_{0}2^{21}$ possible colorings. If Bob makes less than 22 queries, then he can only output $2^{21}$ possible colorings, which means he is wrong on some coloring. + +Now we show Bob can always win in 22 queries. A key observation is that the set of red points is convex, as is the set of blue points, so if a set of points is all the same color, then their convex hull is all the same color. + +## Lemma + +Let $B_{0}, \ldots, B_{k+1}$ be equally spaced points on a circular arc such that colors of $B_{0}$ and $B_{k+1}$ differ and are known. Then it is possible to determine the colors of $B_{1}$, $\ldots, B_{k}$ in $\left\lceil\log _{2} k\right\rceil$ queries. + +Proof. There exists some $0 \leq i \leq k$ such that $B_{0}, \ldots, B_{i}$ are the same color and $B_{i+1}$, $\ldots, B_{k+1}$ are the same color. (If $i\lceil k / 2\rceil$, then the segment $P B_{i}$ is blue and intersect the segment $B_{0} B_{\lceil k / 2\rceil}$, which is red, contradiction. + +Now the strategy is: Bob picks $A_{1}$. WLOG it is red. Now suppose Bob does not know the colors of $\leq 2^{k}-1$ points $A_{i}, \ldots, A_{j}$ with $j-i+1 \leq 2^{k}-1$ and knows the rest are red. I claim Bob can win in $2 k-1$ queries. First, if $k=1$, there is one point and he wins by querying the point, so the base case holds, so assume $k>1$. Bob queries $A_{i+\lceil(j-i+1) / 2\rceil}$. If it is blue, he finishes in $2 \log _{2}\lceil(j-i+1) / 2\rceil \leq 2(k-1)$ queries by the first lemma, for a total of $2 k-1$ queries. If it is red, he can query one more point and learn some half of $A_{i}, \ldots, A_{j}$ that are red by the second lemma, and then he has reduced it to the case with $\leq 2^{k-1}-1$ points in two queries, at which point we induct. + +## §2.3 TSTST 2022/6, proposed by Hongzhou Lin + +Available online at https://aops.com/community/p25516957. + +## Problem statement + +Let $O$ and $H$ be the circumcenter and orthocenter, respectively, of an acute scalene triangle $A B C$. The perpendicular bisector of $\overline{A H}$ intersects $\overline{A B}$ and $\overline{A C}$ at $X_{A}$ and $Y_{A}$ respectively. Let $K_{A}$ denote the intersection of the circumcircles of triangles $O X_{A} Y_{A}$ and $B O C$ other than $O$. +Define $K_{B}$ and $K_{C}$ analogously by repeating this construction two more times. Prove that $K_{A}, K_{B}, K_{C}$, and $O$ are concyclic. + +We present several approaches. +\ First solution, by author. Let $\odot O X_{A} Y_{A}$ intersects $A B, A C$ again at $U, V$. Then by Reim's theorem $U V C B$ are concyclic. Hence the radical axis of $\odot O X_{A} Y_{A}, \odot O B C$ and $\odot(U V C B)$ are concurrent, i.e. $O K_{A}, B C, U V$ are concurrent, Denote the intersection as $K_{A}^{*}$, which is indeed the inversion of $K_{A}$ with respect to $\odot O$. (The inversion sends $\odot O B C$ to the line $B C$ ). +Let $P_{A}, P_{B}, P_{C}$ be the circumcenters of $\triangle O B C, \triangle O C A, \triangle O A B$ respectively. +Claim $-K_{A}^{*}$ coincides with the intersection of $P_{B} P_{C}$ and $B C$. +Proof. Note that $d(O, B C)=1 / 2 A H=d\left(A, X_{A} Y_{A}\right)$. This means the midpoint $M_{C}$ of $A B$ is equal distance to $X_{A} Y_{A}$ and the line through $O$ parallel to $B C$. Together with $O M_{C} \perp A B$ implies that $\angle M_{C} X_{A} O=\angle B$. Hence $\angle U V O=\angle B=\angle A V U$. Similarly $\angle V U O=\angle A U V$, hence $\triangle A U V \simeq \triangle O U V$. In other words, $U V$ is the perpendicular bisector of $A O$, which pass through $P_{B}, P_{C}$. Hence $K_{A}^{*}$ is indeed $P_{B} P_{C} \cap B C$. + +Finally by Desargue's theorem, it suffices to show that $A P_{A}, B P_{B}, C P_{C}$ are concurrent. Note that + +$$ +\begin{aligned} +& d\left(P_{A}, A B\right)=P_{A} B \sin \left(90^{\circ}+\angle C-\angle A\right) \\ +& d\left(P_{A}, A C\right)=P_{A} C \sin \left(90^{\circ}+\angle B-\angle A\right) +\end{aligned} +$$ + +Hence the symmetric product and trig Ceva finishes the proof. +![](https://cdn.mathpix.com/cropped/2024_11_19_1799270e91f74486a977g-15.jpg?height=1132&width=1194&top_left_y=245&top_left_x=431) + +I Second solution, from Jeffrey Kwan. Let $O_{A}$ be the circumcenter of $\triangle A X_{A} Y_{A}$. The key claim is that: + +Claim $-O_{A} X_{A} Y_{A} O$ is cyclic. + +Proof. Let $D E F$ be the orthic triangle; we will show that $\triangle O X_{A} Y_{A} \sim \triangle D E F$. Indeed, since $A O$ and $A D$ are isogonal, it suffices to note that + +$$ +\frac{A X_{A}}{A B}=\frac{A H / 2}{A D}=\frac{R \cos A}{A D} +$$ + +and so + +$$ +\frac{A O}{A D}=R \cdot \frac{A X_{A}}{A B \cdot R \cos A}=\frac{A X_{A}}{A E}=\frac{A Y_{A}}{A F} +$$ + +Hence $\angle X_{A} O Y_{A}=180^{\circ}-2 \angle A=180^{\circ}-\angle X_{A} O_{A} Y_{A}$, which proves the claim. +Let $P_{A}$ be the circumcenter of $\triangle O B C$, and define $P_{B}, P_{C}$ similarly. By the claim, $A$ is the exsimilicenter of $\left(O X_{A} Y_{A}\right)$ and $(O B C)$, so $A P_{A}$ is the line between their two centers. In particular, $A P_{A}$ is the perpendicular bisector of $O K_{A}$. +![](https://cdn.mathpix.com/cropped/2024_11_19_1799270e91f74486a977g-16.jpg?height=1303&width=1020&top_left_y=245&top_left_x=521) + +Claim $-A P_{A}, B P_{B}, C P_{C}$ concur at $T$. +Proof. The key observation is that $O$ is the incenter of $\triangle P_{A} P_{B} P_{C}$, and that $A, B, C$ are the reflections of $O$ across the sides of $\triangle P_{A} P_{B} P_{C}$. Hence $P_{A} A, P_{B} B, P_{C} C$ concur by Jacobi. + +Now $T$ lies on the perpendicular bisectors of $O K_{A}, O K_{B}$, and $O K_{C}$. Hence $O K_{A} K_{B} K_{C}$ is cyclic with center $T$, as desired. + +## §3 Solutions to Day 3 + +## §3.1 TSTST 2022/7, proposed by Merlijn Staps + +Available online at https://aops.com/community/p25516961. + +## Problem statement + +Let $A B C D$ be a parallelogram. Point $E$ lies on segment $C D$ such that + +$$ +2 \angle A E B=\angle A D B+\angle A C B, +$$ + +and point $F$ lies on segment $B C$ such that + +$$ +2 \angle D F A=\angle D C A+\angle D B A . +$$ + +Let $K$ be the circumcenter of triangle $A B D$. Prove that $K E=K F$. + +Let the circle through $A, B$, and $E$ intersect $C D$ again at $E^{\prime}$, and let the circle through $D$, $A$, and $F$ intersect $B C$ again at $F^{\prime}$. Now $A B E E^{\prime}$ and $D A F^{\prime} F$ are cyclic quadrilaterals with two parallel sides, so they are isosceles trapezoids. From $K A=K B$, it now follows that $K E=K E^{\prime}$, whereas from $K A=K D$ it follows that $K F=K F^{\prime}$. + +Next, let the circle through $A, B$, and $E$ intersect $A C$ again at $S$. Then + +$$ +\angle A S B=\angle A E B=\frac{1}{2}(\angle A D B+\angle A C B)=\frac{1}{2}(\angle A D B+\angle D A C)=\frac{1}{2} \angle A M B, +$$ + +where $M$ is the intersection of $A C$ and $B D$. From $\angle A S B=\frac{1}{2} \angle A M B$, it follows that $M S=M B$, so $S$ is the point on $M C$ such that $M S=M B=M D$. By symmetry, the circle through $A, D$, and $F$ also passes through $S$, and it follows that the line $A S$ is the radical axis of the circles $(A B E)$ and $(A D F)$. + +By power of a point, we now obtain + +$$ +C E \cdot C E^{\prime}=C S \cdot C A=C F \cdot C F^{\prime}, +$$ + +from which it follows that $E, F, E^{\prime}$, and $F^{\prime}$ are concyclic. The segments $E E^{\prime}$ and $F F^{\prime}$ are not parallel, so their perpendicular bisectors only meet at one point, which is $K$. Hence $K E=K F$. + +## §3.2 TSTST 2022/8, proposed by Merlijn Staps + +Available online at https://aops.com/community/p25516968. + +## Problem statement + +Find all functions $f: \mathbb{N} \rightarrow \mathbb{Z}$ such that + +$$ +\left\lfloor\frac{f(m n)}{n}\right\rfloor=f(m) +$$ + +for all positive integers $m, n$. + +There are two families of functions that work: for each $\alpha \in \mathbb{R}$ the function $f(n)=\lfloor\alpha n\rfloor$, and for each $\alpha \in \mathbb{R}$ the function $f(n)=\lceil\alpha n\rceil-1$. (For irrational $\alpha$ these two functions coincide.) It is straightforward to check that these functions indeed work; essentially, this follows from the identity + +$$ +\left\lfloor\frac{\lfloor x n\rfloor}{n}\right\rfloor=\lfloor x\rfloor +$$ + +which holds for all positive integers $n$ and real numbers $x$. +We now show that every function that works must be of one of the above forms. Let $f$ be a function that works, and define the sequence $a_{1}, a_{2}, \ldots$ by $a_{n}=f(n!) / n!$. Applying the give condition with $(n!, n+1)$ yields $a_{n+1} \in\left[a_{n}, a_{n}+\frac{1}{n!}\right)$. It follows that the sequence $a_{1}, a_{2}, \ldots$ is non-decreasing and bounded from above by $a_{1}+e$, so this sequence must converge to some limit $\alpha$. + +If there exists a $k$ such that $a_{k}=\alpha$, then we have $a_{\ell}=\alpha$ for all $\ell>k$. For each positive integer $m$, there exists $\ell>k$ such that $m \mid \ell$ !. Plugging in $m n=\ell$ !, it then follows that + +$$ +f(m)=\left\lfloor\frac{f(\ell!)}{\ell!/ m}\right\rfloor=\lfloor\alpha m\rfloor +$$ + +for all $m$, so $f$ is of the desired form. +If there does not exist a $k$ such that $a_{k}=\alpha$, we must have $a_{k}<\alpha$ for all $k$. For each positive integer $m$, we can now pick an $\ell$ such that $m \mid \ell$ ! and $a_{\ell}=\alpha-x$ with $x$ arbitrarily small. It then follows from plugging in $m n=\ell$ ! that + +$$ +f(m)=\left\lfloor\frac{f(\ell!)}{\ell!/ m}\right\rfloor=\left\lfloor\frac{\ell!(\alpha-x)}{\ell!/ m}\right\rfloor=\lfloor\alpha m-m x\rfloor . +$$ + +If $\alpha m$ is an integer we can choose $\ell$ such that $m x<1$, and it follows that $f(m)=\lceil\alpha m\rceil-1$. If $\alpha m$ is not an integer we can choose $\ell$ such that $m x<\{\alpha m\}$, and it also follows that $f(m)=\lceil\alpha m\rceil-1$. We conclude that in this case $f$ is again of the desired form. + +## §3.3 TSTST 2022/9, proposed by Vincent Huang + +Available online at https://aops.com/community/p25517112. + +## Problem statement + +Let $k>1$ be a fixed positive integer. Prove that if $n$ is a sufficiently large positive integer, there exists a sequence of integers with the following properties: + +- Each element of the sequence is between 1 and $n$, inclusive. +- For any two different contiguous subsequences of the sequence with length between 2 and $k$ inclusive, the multisets of values in those two subsequences is not the same. +- The sequence has length at least $0.499 n^{2}$. + +For any positive integer $n$, define an $(n, k)$-good sequence to be a finite sequence of integers each between 1 and $n$ inclusive satisfying the second property in the problem statement. The problems asks to show that, for all sufficiently large integers $n$, there is an $(n, k)$-good sequence of length at least $0.499 n^{2}$. +Fix $k \geq 2$ and consider some prime power $n=p^{m}$ with $p>k+1$. Consider some $0k+1$ be a prime. Then for $n=p^{2}$ we can find a $(n, k)$-good sequence of length $\frac{p(p-1)\left(p^{2}+2\right)}{2}$. + +Proof of last claim. Let $g$ be the smallest primitive root modulo $n=p^{2}$, so that $a=$ $\frac{p(p-1)}{2}$. As long as we can show that $g<\frac{n}{k}-1$, we can apply the previous claim to get the desired bound. + +We will prove a stronger statement that $g United States of America - TST Selection Test + +Andrew Gu, Evan Chen, Gopal Goel
$65^{\text {th }}$ IMO 2024 United Kingdom and $13^{\text {th }}$ EGMO 2024 Georgia + +## Contents + +0 Problems ..... 2 +1 Solutions to Day 1 ..... 4 +1.1 TSTST 2023/1, proposed by Merlijn Staps ..... 4 +1.2 TSTST 2023/2, proposed by Raymond Feng, Luke Robitaille ..... 9 +1.3 TSTST 2023/3, proposed by Merlijn Staps ..... 13 +2 Solutions to Day 2 ..... 16 +2.1 TSTST 2023/4, proposed by Ankan Bhattacharya ..... 16 +2.2 TSTST 2023/5, proposed by David Altizio ..... 17 +2.3 TSTST 2023/6, proposed by Holden Mui ..... 20 +3 Solutions to Day 3 ..... 24 +3.1 TSTST 2023/7, proposed by Luke Robitaille ..... 24 +3.2 TSTST 2023/8, proposed by Ankan Bhattacharya ..... 25 +3.3 TSTST 2023/9, proposed by Holden Mui ..... 28 + +## §0 Problems + +1. Let $A B C$ be a triangle with centroid $G$. Points $R$ and $S$ are chosen on rays $G B$ and $G C$, respectively, such that + +$$ +\angle A B S=\angle A C R=180^{\circ}-\angle B G C . +$$ + +Prove that $\angle R A S+\angle B A C=\angle B G C$. +2. Let $n \geq m \geq 1$ be integers. Prove that + +$$ +\sum_{k=m}^{n}\left(\frac{1}{k^{2}}+\frac{1}{k^{3}}\right) \geq m \cdot\left(\sum_{k=m}^{n} \frac{1}{k^{2}}\right)^{2} +$$ + +3. Find all positive integers $n$ for which it is possible to color some cells of an infinite grid of unit squares red, such that each rectangle consisting of exactly $n$ cells (and whose edges lie along the lines of the grid) contains an odd number of red cells. +4. Let $n \geq 3$ be an integer and let $K_{n}$ be the complete graph on $n$ vertices. Each edge of $K_{n}$ is colored either red, green, or blue. Let $A$ denote the number of triangles in $K_{n}$ with all edges of the same color, and let $B$ denote the number of triangles in $K_{n}$ with all edges of different colors. Prove that + +$$ +B \leq 2 A+\frac{n(n-1)}{3} . +$$ + +5. Suppose $a, b$, and $c$ are three complex numbers with product 1 . Assume that none of $a, b$, and $c$ are real or have absolute value 1 . Define + +$$ +p=(a+b+c)+\left(\frac{1}{a}+\frac{1}{b}+\frac{1}{c}\right) \quad \text { and } \quad q=\frac{a}{b}+\frac{b}{c}+\frac{c}{a} . +$$ + +Given that both $p$ and $q$ are real numbers, find all possible values of the ordered pair $(p, q)$. +6. Let $A B C$ be a scalene triangle and let $P$ and $Q$ be two distinct points in its interior. Suppose that the angle bisectors of $\angle P A Q, \angle P B Q$, and $\angle P C Q$ are the altitudes of triangle $A B C$. Prove that the midpoint of $\overline{P Q}$ lies on the Euler line of $A B C$. +7. The Bank of Pittsburgh issues coins that have a heads side and a tails side. Vera has a row of 2023 such coins alternately tails-up and heads-up, with the leftmost coin tails-up. +In a move, Vera may flip over one of the coins in the row, subject to the following rules: + +- On the first move, Vera may flip over any of the 2023 coins. +- On all subsequent moves, Vera may only flip over a coin adjacent to the coin she flipped on the previous move. (We do not consider a coin to be adjacent to itself.) + +Determine the smallest possible number of moves Vera can make to reach a state in which every coin is heads-up. +8. Let $A B C$ be an equilateral triangle with side length 1. Points $A_{1}$ and $A_{2}$ are chosen on side $B C$, points $B_{1}$ and $B_{2}$ are chosen on side $C A$, and points $C_{1}$ and $C_{2}$ are chosen on side $A B$ such that $B A_{1}\frac{\left(\sum_{k=m}^{n} \frac{1}{k^{2}}\right)^{2}}{\frac{1}{m}} +\end{aligned} +$$ + +as desired. +Remark (Bound on error). Let $A=\sum_{k=m}^{n} k^{-2}$ and $B=\sum_{k=m}^{n} k^{-3}$. The inequality above becomes tighter for large $m$ and $n \gg m$. If we use Lagrange's identity in place of Cauchy-Schwarz, we get + +$$ +A+B-m A^{2}=m \cdot \sum_{m \leq aA+B +$$ + +Remark (Construction commentary, from author). My motivation was to write an inequality where Titu could be applied creatively to yield a telescoping sum. This can be difficult because most of the time, such a reverse-engineered inequality will be so loose it's trivial +anyways. My first attempt was the not-so-amazing inequality + +$$ +\frac{n^{2}+3 n}{2}=\sum_{1}^{n} i+1=\sum_{1}^{n} \frac{\frac{1}{i}}{\frac{1}{i(i+1)}}>\left(\sum_{1}^{n} \frac{1}{\sqrt{i}}\right)^{2} +$$ + +which is really not surprising given that $\sum \frac{1}{\sqrt{i}} \ll \frac{n}{\sqrt{2}}$. The key here is that we need "near-equality" as dictated by the Cauchy-Schwarz equality case, i.e. the square root of the numerators should be approximately proportional to the denominators. + +This motivates using $\frac{1}{i^{4}}$ as the numerator, which works like a charm. After working out the resulting statement, the LHS and RHS even share a sum, which adds to the simplicity of the problem. + +The final touch was to unrestrict the starting value of the sum, since this allows the strength of the estimate $\frac{1}{i^{2}} \approx \frac{1}{i(i+1)}$ to be fully exploited. + +【 Second approach by inducting down, Luke Robitaille and Carl Schildkraut. Fix $n$; we'll induct downwards on $m$. For the base case of $n=m$ the result is easy, since the left side is $\frac{m+1}{m^{3}}$ and the right side is $\frac{m}{m^{4}}=\frac{1}{m^{3}}$. + +For the inductive step, suppose we have shown the result for $m+1$. Let + +$$ +A=\sum_{k=m+1}^{n} \frac{1}{k^{2}} \quad \text { and } \quad B=\sum_{k=m+1}^{n} \frac{1}{k^{3}} +$$ + +We know $A+B \geq(m+1) A^{2}$, and we want to show + +$$ +\left(A+\frac{1}{m^{2}}\right)+\left(B+\frac{1}{m^{3}}\right) \geq m\left(A+\frac{1}{m^{2}}\right)^{2} +$$ + +Indeed, + +$$ +\begin{aligned} +\left(A+\frac{1}{m^{2}}\right)+\left(B+\frac{1}{m^{3}}\right)-m\left(A+\frac{1}{m^{2}}\right)^{2} & =A+B+\frac{m+1}{m^{3}}-m A^{2}-\frac{2 A}{m}-\frac{1}{m^{3}} \\ +& =\left(A+B-(m+1) A^{2}\right)+\left(A-\frac{1}{m}\right)^{2} \geq 0 +\end{aligned} +$$ + +and we are done. + +I Third approach by reducing $n \rightarrow \infty$, Michael Ren and Carl Schildkraut. First, we give: + +Claim (Reduction to $n \rightarrow \infty$ ) - If the problem is true when $n \rightarrow \infty$, it is true for all $n$. + +Proof. Let $A=\sum_{k=m}^{n} k^{-2}$ and $B=\sum_{k=m}^{n} k^{-3}$. Consider the region of the $x y$-plane defined by $y>m x^{2}-x$. We are interested in whether $(A, B)$ lies in this region. + +However, the region is bounded by a convex curve, and the sequence of points $(0,0)$, $\left(\frac{1}{m^{2}}, \frac{1}{m^{3}}\right),\left(\frac{1}{m^{2}}+\frac{1}{(m+1)^{2}}, \frac{1}{m^{3}}+\frac{1}{(m+1)^{3}}\right), \ldots$ has successively decreasing slopes between consecutive points. Thus it suffices to check that the inequality is true when $n \rightarrow \infty$. + +Set $n=\infty$ henceforth. Let + +$$ +A=\sum_{k=m}^{\infty} \frac{1}{k^{2}} \text { and } B=\sum_{k=m}^{\infty} \frac{1}{k^{3}} +$$ + +we want to show $B \geq m A^{2}-A$, which rearranges to + +$$ +1+4 m B \geq(2 m A-1)^{2} +$$ + +Write + +$$ +C=\sum_{k=m}^{\infty} \frac{1}{k^{2}(2 k-1)(2 k+1)} \text { and } D=\sum_{k=m}^{\infty} \frac{8 k^{2}-1}{k^{3}(2 k-1)^{2}(2 k+1)^{2}} +$$ + +Then + +$$ +\frac{2}{2 k-1}-\frac{2}{2 k+1}=\frac{1}{k^{2}}+\frac{1}{k^{2}(2 k-1)(2 k+1)} +$$ + +and + +$$ +\frac{2}{(2 k-1)^{2}}-\frac{2}{(2 k+1)^{2}}=\frac{1}{k^{3}}+\frac{8 k^{2}-1}{k^{3}(2 k-1)^{2}(2 k+1)^{2}}, +$$ + +so that + +$$ +A=\frac{2}{2 m-1}-C \text { and } B=\frac{2}{(2 m-1)^{2}}-D +$$ + +Our inequality we wish to show becomes + +$$ +\frac{2 m+1}{2 m-1} C \geq D+m C^{2} +$$ + +We in fact show two claims: +Claim - We have + +$$ +\frac{2 m+1 / 2}{2 m-1} C \geq D +$$ + +Proof. We compare termwise; we need + +$$ +\frac{2 m+1 / 2}{2 m-1} \cdot \frac{1}{k^{2}(2 k-1)(2 k+1)} \geq \frac{8 k^{2}-1}{k^{3}(2 k-1)^{2}(2 k+1)^{2}} +$$ + +for $k \geq m$. It suffices to show + +$$ +\frac{2 k+1 / 2}{2 k-1} \cdot \frac{1}{k^{2}(2 k-1)(2 k+1)} \geq \frac{8 k^{2}-1}{k^{3}(2 k-1)^{2}(2 k+1)^{2}} +$$ + +which is equivalent to $k(2 k+1 / 2)(2 k+1) \geq 8 k^{2}-1$. This holds for all $k \geq 1$. + +## Claim - We have + +$$ +\frac{1 / 2}{2 m-1} C \geq m C^{2} +$$ + +Proof. We need $C \leq 1 /(2 m(2 m-1))$; indeed, + +$$ +\frac{1}{2 m(2 m-1)}=\sum_{k=m}^{\infty}\left(\frac{1}{2 k(2 k-1)}-\frac{1}{2(k+1)(2 k+1)}\right)=\sum_{k=m}^{\infty} \frac{4 k+1}{2 k(2 k-1)(k+1)(2 k+1)} +$$ + +comparing term-wise with the definition of $C$ and using the inequality $k(4 k+1) \geq 2(k+1)$ for $k \geq 1$ gives the desired result. + +Combining the two claims finishes the solution. + +『 Fourth approach by bashing, Carl Schildkraut. With a bit more work, the third approach can be adapted to avoid the $n \rightarrow \infty$ reduction. Similarly to before, define + +$$ +A=\sum_{k=m}^{n} \frac{1}{k^{2}} \text { and } B=\sum_{k=m}^{n} \frac{1}{k^{3}} +$$ + +we want to show $1+4 m B \geq(2 m A-1)^{2}$. Writing + +$$ +C=\sum_{k=m}^{n} \frac{1}{k^{2}(2 k-1)(2 k+1)} \text { and } D=\sum_{k=m}^{n} \frac{8 k^{2}-1}{k^{3}(2 k-1)^{2}(2 k+1)^{2}} +$$ + +We compute + +$$ +A=\frac{2}{2 m-1}-\frac{2}{2 n+1}-C \text { and } B=\frac{2}{(2 m-1)^{2}}-\frac{2}{(2 n+1)^{2}}-D . +$$ + +Then, the inequality we wish to show reduces (as in the previous solution) to + +$$ +\frac{2 m+1}{2 m-1} C+\frac{2(2 m+1)}{(2 m-1)(2 n+1)} \geq D+m C^{2}+\frac{2(2 m+1)}{(2 n+1)^{2}}+\frac{4 m}{2 n+1} C +$$ + +We deal first with the terms not containing the variable $n$, i.e. we show that + +$$ +\frac{2 m+1}{2 m-1} C \geq D+m C^{2} +$$ + +For this part, the two claims from the previous solution go through exactly as written above, and we have $C \leq 1 /(2 m(2 m-1))$. We now need to show + +$$ +\frac{2(2 m+1)}{(2 m-1)(2 n+1)} \geq \frac{2(2 m+1)}{(2 n+1)^{2}}+\frac{4 m}{2 n+1} C +$$ + +(this is just the inequality between the remaining terms); our bound on $C$ reduces this to proving + +$$ +\frac{4(2 m+1)(n-m+1)}{(2 m-1)(2 n+1)^{2}} \geq \frac{2}{(2 m-1)(2 n+1)} +$$ + +Expanding and writing in terms of $n$, this is equivalent to + +$$ +n \geq \frac{1+2(m-1)(2 m+1)}{4 m}=m-\frac{2 m+1}{4 m} +$$ + +which holds for all $n \geq m$. + +## §1.3 TSTST 2023/3, proposed by Merlijn Staps + +Available online at https://aops.com/community/p28015682. + +## Problem statement + +Find all positive integers $n$ for which it is possible to color some cells of an infinite grid of unit squares red, such that each rectangle consisting of exactly $n$ cells (and whose edges lie along the lines of the grid) contains an odd number of red cells. + +We claim that this is possible for all positive integers $n$. Call a positive integer for which such a coloring is possible good. To show that all positive integers $n$ are good we prove the following: +(i) If $n$ is good and $p$ is an odd prime, then $p n$ is good; +(ii) For every $k \geq 0$, the number $n=2^{k}$ is good. + +Together, (i) and (ii) imply that all positive integers are good. +【 Proof of (i). We simply observe that if every rectangle consisting of $n$ cells contains an odd number of red cells, then so must every rectangle consisting of $p n$ cells. Indeed, because $p$ is prime, a rectangle consisting of $p n$ cells must have a dimension (length or width) divisible by $p$ and can thus be subdivided into $p$ rectangles consisting of $n$ cells. + +Thus every coloring that works for $n$ automatically also works for $p n$. +【 Proof of (ii). Observe that rectangles with $n=2^{k}$ cells have $k+1$ possible shapes: $2^{m} \times 2^{k-m}$ for $0 \leq m \leq k$. + +Claim - For each of these $k+1$ shapes, there exists a coloring with two properties: + +- Every rectangle with $n$ cells and shape $2^{m} \times 2^{k-m}$ contains an odd number of red cells. +- Every rectangle with $n$ cells and a different shape contains an even number of red cells. + +Proof. This can be achieved as follows: assuming the cells are labeled with $(x, y) \in \mathbb{Z}^{2}$, color a cell red if $x \equiv 0\left(\bmod 2^{m}\right)$ and $y \equiv 0\left(\bmod 2^{k-m}\right)$. For example, a $4 \times 2$ rectangle gets the following coloring: +![](https://cdn.mathpix.com/cropped/2024_11_19_1cf0843c143ab4f13b3fg-13.jpg?height=395&width=401&top_left_y=2084&top_left_x=833) + +A $2^{m} \times 2^{k-m}$ rectangle contains every possible pair $\left(x \bmod 2^{m}, y \bmod 2^{k-m}\right)$ exactly once, so such a rectangle will contain one red cell (an odd number). + +On the other hand, consider a $2^{\ell} \times 2^{k-\ell}$ rectangle with $\ell>m$. The set of cells this covers is $(x, y)$ where $x$ covers a range of size $2^{\ell}$ and $y$ covers a range of size $2^{k-\ell}$. The number of red cells is the count of $x$ with $x \equiv 0 \bmod 2^{m}$ multiplied by the count of $y$ with $y \equiv 0 \bmod 2^{k-m}$. The former number is exactly $2^{\ell-k}$ because $2^{k}$ divides $2^{\ell}$ (while the latter is 0 or 1) so the number of red cells is even. The $\ell2 \\ +2\left(g(x)+g\left(x+p^{d}\right)+\cdots+g\left(x+(b-1) p^{d}\right)\right) & p=2\end{cases} \\ +\equiv & 0 \quad(\bmod p) +\end{aligned} +$$ + +as desired. + +## Corollary + +Let $g: \mathbb{Z} / a \mathbb{Z} \rightarrow \mathbb{Z} / p^{e} \mathbb{Z}$ be any function, and let $h=\Delta^{e p^{d}} g$. Then + +$$ +h(x)+h\left(x+p^{d}\right)+\cdots+h\left(x+(b-1) p^{d}\right)=0 +$$ + +for all $x$. + +Proof. Starting with the lemma, define + +$$ +h_{1}(x)=\frac{h(x)+h\left(x+p^{d}\right)+\cdots+h\left(x+(b-1) p^{d}\right)}{p} . +$$ + +Applying the lemma to $h_{1}$ shows the corollary for $e=2$, since $h_{1}(x)$ is divisible by $p$, hence the numerator is divisible by $p^{2}$. Continue in this manner to get the result for general $e>2$. + +This immediately settles this direction, since $f$ is in the image of $\Delta^{e p^{d}}$. + +Proof the equation implies essential Let $\mathcal{S}$ be the set of all functions satisfying 2 ; then it's easy to see that $\Delta$ is a function on $\mathcal{S}$. To show that all functions in $\mathcal{S}$ are essential, it's equivalent to show that $\Delta$ is a permutation on $\mathcal{S}$. + +We will show that $\Delta$ is injective on $\mathcal{S}$. Suppose otherwise, and consider two functions $f$, $g$ in $\mathcal{S}$ with $\Delta f=\Delta g$. Then, we obtain that $f$ and $g$ differ by a constant; say $g=f+\lambda$. However, then + +$$ +\begin{aligned} +& g(0)+g\left(p^{e}\right)+\cdots+g\left((b-1) p^{e}\right) \\ += & (f(0)+\lambda)+\left(f\left(p^{e}\right)+\lambda\right)+\cdots+\left(f\left((b-1) p^{e}\right)+\lambda\right) \\ += & b \lambda +\end{aligned} +$$ + +This should also be zero. Since $p \nmid b$, we obtain $\lambda=0$, as desired. + +Counting Finally, we can count the essential functions: all but the last $p^{d}$ entries can be chosen arbitrarily, and then each remaining entry has exactly one possible choice. This leads to a count of + +$$ +\left(p^{e}\right)^{a-p^{d}}=p^{e\left(a-p^{\nu_{p}(a)}\right)}, +$$ + +as promised. + +II Second solution by Daniel Zhu. There are two parts to the proof: solving the $e=1$ case, and using the $e=1$ result to solve the general problem by induction on $e$. These parts are independent of each other. + +The case $e=1 \quad$ Represent functions $f$ as elements + +$$ +\alpha_{f}:=\sum_{k \in \mathbb{Z} / a \mathbb{Z}} f(-k) x^{k} \in \mathbb{F}_{p}[x] /\left(x^{a}-1\right) +$$ + +Then, since $\alpha_{\Delta f}=(x-1) \alpha_{f}$, we wish to find the number of $\alpha \in \mathbb{F}_{p}[x] /\left(x^{a}-1\right)$ such that $(x-1)^{m} \alpha=\alpha$ for some $m$. + +Now, make the substitution $y=x-1$ and let $P(y)=(y+1)^{a}-1$; we want to find $\alpha \in \mathbb{F}_{p}[y] /(P(y))$ such that $y^{m} \alpha=\alpha$ for some $m$. + +If we write $P(y)=y^{d} Q(y)$ with $Q(0) \neq 0$, then by the Chinese Remainder Theorem we have the ring isomorphism + +$$ +\mathbb{F}_{p}[y] /(P(y)) \cong \mathbb{F}_{p}[y] /\left(y^{d}\right) \times \mathbb{F}_{p}[y] /(Q(y)) +$$ + +Note that $y$ is nilpotent in the first factor, while it is a unit in the second factor. So the $\alpha$ that work are exactly those that are zero in the first factor; thus there are $p^{a-d}$ such $\alpha$. We can calculate $d=p^{v_{p}(a)}$ (via, say, Lucas's Theorem), so we are done. + +The general problem The general idea is as follows: call a $f: \mathbb{Z} / a \mathbb{Z} \rightarrow \mathbb{Z} / p^{e} \mathbb{Z} e$-good if $\Delta^{m} f=f$ for some $m$. Our result above allows us to count the 1-good functions. Then, if $e \geq 1$, every $(e+1)$-good function, when reduced $\bmod p^{e}$, yields an $e$-good function, so we count $(e+1)$-good functions by counting how many reduce to any given $e$-good function. + +Formally, we use induction on $e$, with the $e=1$ case being treated above. Suppose now we have solved the problem for a given $e \geq 1$, and we now wish to solve it for $e+1$. For any function $g: \mathbb{Z} / a \mathbb{Z} \rightarrow \mathbb{Z} / p^{e+1} \mathbb{Z}$, let $\bar{g}: \mathbb{Z} / a \mathbb{Z} \rightarrow \mathbb{Z} / p^{e} \mathbb{Z}$ be its reduction $\bmod p^{e}$. For a given $e$-good $f$, let $n(f)$ be the number of $(e+1)$-good $g$ with $\bar{g}=f$. The following two claims now finish the problem: + +Claim - If $f$ is $e$-good, then $n(f)>0$. + +Proof. Suppose $m$ is such that $\Delta^{m} f=f$. Pick any $g$ with $\bar{g}=f$, and consider the sequence of functions + +$$ +g, \Delta^{m} g, \Delta^{2 m} g, \ldots +$$ + +Since there are finitely many functions $\mathbb{Z} / a \mathbb{Z} \rightarrow \mathbb{Z} / p^{e+1} \mathbb{Z}$, there must exist $a0$, then $n(f)$ is exactly the number of 1-good functions, i.e. $p^{a-p^{v_{p}(a)}}$. + +Proof. Let $g$ be any $(e+1)$-good function with $\bar{g}=f$. We claim that the $(e+1)$-good $g_{1}$ with $\bar{g}_{1}=f$ are exactly the functions of the form $g+p^{e} h$ for any 1-good $h$. Since these functions are clearly distinct, this characterization will prove the claim. + +To show that this condition is sufficient, note that $\overline{g+p^{e} h}=\bar{g}=f$. Moreover, if $\Delta^{m} g=g$ and $\Delta^{m^{\prime}} h=h$, then + +$$ +\Delta^{m m^{\prime}}\left(g+p^{e} h\right)=\Delta^{m m^{\prime}} g+p^{e} \Delta^{m m^{\prime}} h=g+p^{e} h . +$$ + +To show that this condition is necessary, let $g_{1}$ be any $(e+1)$-good function such that $\bar{g}_{1}=f$. Then $g_{1}-g$ is also $(e+1)$-good, since if $\Delta^{m} g=g, \Delta^{m^{\prime}} g_{1}=g_{1}$, we have + +$$ +\Delta^{m m^{\prime}}\left(g_{1}-g\right)=\Delta^{m m^{\prime}} g_{1}-\Delta^{m m^{\prime}} g=g_{1}-g . +$$ + +On the other hand, we also know that $g_{1}-g$ is divisible by $p^{e}$. This means that it must be $p^{e} h$ for some function $f: \mathbb{Z} / a \mathbb{Z} \rightarrow \mathbb{Z} / p \mathbb{Z}$, and it is not hard to show that $g_{1}-g$ being $(e+1)$-good means that $h$ is 1 -good. + diff --git a/USA_TST_TST/segment_script/segment_usamo.py b/USA_TST_TST/segment_script/segment_usamo.py new file mode 100644 index 0000000000000000000000000000000000000000..0fb17da47cecbb1c7443d21332018f569c13d2ff --- /dev/null +++ b/USA_TST_TST/segment_script/segment_usamo.py @@ -0,0 +1,306 @@ +# ----------------------------------------------------------------------------- +# Author: Marina +# Date: 2024-11-15 +# ----------------------------------------------------------------------------- +''' Script to segment md files in en-usamo, en-tstst, en-tst, en-jmo folder using regex. +To run: +`python segment_script/segment_usamo.py` +''' + +import warnings +warnings.filterwarnings("ignore", category=DeprecationWarning) + +import os +import re +import pandas as pd +from rapidfuzz import fuzz + + +# "## §0 Problems" -> match +section_re = re.compile(r"^#{1,2}\s(?:Contents|Problem|§[\d.]+.*)") +# section_re = re.compile(r"^#{1,2}\s(?:§[\d.]+.*)") + +# "## §1.3 USAMO 2024/3, proposed by Krit Boonsiriseth" ->3 +# "## §2.3 TSTST 2011/6" ->6 +# "## §1.1 TSTST 2021/1, proposed by Holden Mui" -> 1 +# "## §1.2 USA TST 2020/2, proposed by Merlijn Staps" -> 2 +# "# §1.3 USA TST 2019/3, proposed by Nikolai Beluhov" -> 3 +solution_label_re = re.compile( + r"^#{1,2}\s§[\d.]+\s[A-Za-z0-9 ]+\s\d{4}/(\d+)(?:,\s.*)?$" +) + +# "1. Prove that the average of the numbers $n" -> match +# "2 . For any nonempty set $S$ of real numbers,"" -> match +problem_re = re.compile(r"^(\d+)\s?\.\s(.*(?:\n\s+.*)*)") + +# "## Problem statement extra text" -> match +# "Problem statement" -> match +# "##Problem statement (missing space)" -> match +solution_re = re.compile(r"^#{0,2}\s?Problem statement\b.*$") + +# not actually used, only for debugging +pattern_debug = re.compile( + r"^[【『\\]*.*?\b(First|Second|Third|Fourth|Fifth|Sixth|Seventh|Eighth|Ninth|Tenth|Complex|Inversion|Synthetic|One|Another|Solution)\b.*\b(solution|approach|proof)\b.*", + re.IGNORECASE +) + +# "Solution 1" -> match +# "【 Solution 1" -> match +solution_split_re1 = re.compile(r"\bSolution\s[1-9]\b") + +# "First solution" -> match +# "【 Third approach" -> match +# "【 Second approach" -> match +solution_split_re2 = re.compile(r"\b(First|Second|Third|Fourth|Fifth|Sixth|Seventh|Eighth|Ninth|Synthetic)\b\s+(solution|approach|proof)\b") + +# catch special cases (ugly but it works). To generate them run script with `DEBUG = True`` +# and identify in console which special_cases should be catched as first line of a solution +DEBUG = False +special_cases = [ +"【 First short solution, by Jeffrey Kwan. Let $p_{0", +"II Second longer solution using an invariant. Visu", +"【 Complex solution (Evan Chen). Toss on the comple", +"Second (longer) solution. If one does not notice t", +"『 Second calculation approach (along the lines of ", +"T Outline of second approach (by convexity, due t", +"I Inversion solution submitted by Ankan Bhattacha", +"【 Complex numbers approach with Apollonian circles", +" A second solution. Both lemmas above admit varia", +"【 A third remixed solution. We use Lemma I and Lem", +"【I A fourth remixed solution. We also can combine ", +"I First grid-based solution. The following solutio", +"Another short solution. Let $Z$ be on line $B D E$", +"【 Most common synthetic approach. The solution hin", +"\\ First \"local\" solution by swapping two points. L", +"Second general solution by angle chasing. By Rei", +"Third general solution by Pascal. Extend rays $A", +"【 Second length solution by tangent lengths. By $t", +"【 Angle chasing solution. Note that $(B D A)$ and", +"【 Harmonic solution (mine). Let $T$ be the point o", +"【 Pascal solution (Zuming Feng). Extend ray $F D$", +"『 A spiral similarity approach (Hans $\mathbf{Y u}", +"ब The author's original solution. Complete isoscel", +"l Evan's permutation-based solution. Retain the n", +"I Original proposer's solution. To this end, let's", +"【 Cartesian coordinates approach with power of a p", +"【 Cartesian coordinates approach without power of", +"I III-advised barycentric approach (outline). Use", +"【 Approach using difference of squares (from autho", +"【 Divisibility approach (Aharshi Roy). Since $p q-", +"Solution with Danielle Wang: the answer is that $|", +"【 Homothety solution (Alex Whatley). Let $G, N, O$", +"【 Power of a point solution (Zuming Feng, official", +"【 Solution by Luke Robitaille. Let $Q$ be the seco", +"ๆ Solution with coaxial circles (Pitchayut Saengru", +"【 Solution to generalization (Nikolai Beluhov). We", +"【 Approach by deleting teams (Gopal Goel). Initial", +"【 Approach by adding colors. For a constructive al", +"【 Solution using spiral similarity. We will ignore", +"『 Barycentric solution (by Carl, Krit, Milan). We", +"I A Menelaus-based approach (Kevin Ren). Let $P$ b", +"【 Barycentric solution. First, we find the coordin", +"【 Angle chasing solution (Mason Fang). Obviously $", +"【 Inversive solution (Kelin Zhu). Invert about $A$", +"l The one-liner. ", +" The external power solution. We distinguish betw", +"Cauchy-Schwarz approach. Apply Titu lemma to get", +"đ Cauchy-Schwarz approach. The main magical claim ", +"『 Alternate solution (by proposer). Let $L$ be dia" +] + + +def add_content(current_dict): + if not current_dict["lines"] or not current_dict["label"] : + return + text_str = " ".join(current_dict["lines"]).strip() + entry = {"label": current_dict["label"]} + if current_dict["class"] == "problem": + entry["problem"] = text_str + current_dict["problems"].append(entry) + elif current_dict["class"] == "solution": + entry["solution"] = text_str + entry["solution_lines"] = current_dict["lines"] + current_dict["solutions"].append(entry) + + +def parse(file): + with open(file, 'r') as file: + content = file.read() + current = { + "label": None, + "class": None, + "lines": [], + "problems": [], + "solutions": [] + } + for line in content.splitlines(): + if match := section_re.match(line): # match a section + add_content(current) + if "problems" in line.lower(): # match problem section + current["class"] = "problem" + elif sub_match:= solution_label_re.match(line): # match solutions section, extract label for join + current["class"] = "other" + current["label"] = sub_match.group(1) + elif match := solution_re.match(line): # match solutions subsection + current["class"] = "solution" + else: + current["class"] = "other" + current["lines"] = [] + elif match := problem_re.match(line): # match a problem + if current["class"] == "solution": # handle wrong problem match + current["lines"].append(line) + else: + add_content(current) + label, text = match.groups() + current["label"] = label + current["lines"] = [text] + else: + if current["class"]=="solution" or current["class"]=="problem": + current["lines"].append(line) + add_content(current) + problems_df = pd.DataFrame(current["problems"]) + solutions_df = pd.DataFrame(current["solutions"]) + return problems_df, solutions_df + + +def parse_solution(lines): + """parses lines of a solution, finds multiple solutions and splits them""" + solutions = [] + current = [] + for line in lines: + if match := solution_split_re1.search(line): # match a solution (case1) + solutions.append(" ".join(current).strip()) + current = [line] + elif match := solution_split_re2.search(line): # match a solution (case1) + solutions.append(" ".join(current).strip()) + current = [line] + elif any(case.lower() in line.lower() for case in special_cases): # match a solution (handle special_case) + solutions.append(" ".join(current).strip()) + current = [line] + elif any(case.lower() in line[:50].lower() for case in ["solution", "approach", "proof"]): + if DEBUG: + if not any(case.lower() in line[:50].lower() for case in ["remark", "proof.", "proof", "approaches", "solutions"]): + print(line[:50]) + else: + current.append(line) + solutions.append(" ".join(current).strip()) + return solutions + +def find_mult_solutions(solutions_df): + """apply parse_solution to all df""" + solutions_df["solution"] = solutions_df["solution_lines"].apply(lambda v: parse_solution(v)) + solutions_df = solutions_df.drop(columns=["solution_lines"]) + solutions_df = solutions_df.explode('solution', ignore_index=True) + return solutions_df + + +def join(problems_df, solutions_df): + pairs_df = problems_df.merge(solutions_df, on=["label"], how="outer") + return pairs_df + + +def clean(pairs_df): + '''removes the problem statement from the solution in an approximate way''' + def find_closest_char(s, i, char): + left = s.rfind(char, 0, i) # Find the last '.' before index i + right = s.find(char, i) # Find the first '.' after or at index i + if left == -1 and right == -1: + return None # No '.' found + elif left == -1: # No '.' on the left + return right + elif right == -1: # No '.' on the right + return left + else: # Closest '.' on either side + return left if abs(i - left) <= abs(i - right) else right + def remove_approx_match(row, threshold=90): + problem = row["problem"] + solution = row["solution"] + similarity = fuzz.partial_ratio(problem, solution) + if similarity >= threshold: + i = find_closest_char(solution, len(problem), problem[-1]) + if i is not None: + solution = solution[i+1:] + return solution + pairs_df["solution"] = pairs_df.apply(remove_approx_match, axis=1) + return pairs_df + + +def process_mult_solutions(pairs_df): + '''in case of multiple solutions, prepend common text to all solutions''' + def prepend_to_solution(group): + if len(group) == 1: + return group + first_row = group.iloc[0] + comment = f"{first_row['solution']}" + group = group.iloc[1:].copy() + group["solution"] = group["solution"].apply(lambda x: f"{comment} {x}") + return group + pairs_df = pairs_df.groupby("label", group_keys=False).apply(prepend_to_solution).reset_index(drop=True) + return pairs_df + + +def add_metadata(pairs_df, yar, tier): + pairs_df['year'] = year + pairs_df['tier'] = tier # according to omnimath + return pairs_df[['year', 'label', 'problem', 'solution']] + + +def write_pairs(filename, pairs_df): + pairs_df.to_json(filename, orient="records", lines=True) + + +configs = [ + ('en-usamo', lambda year: f'en-USAMO-{year}-notes', range(1996, 2025), 1), + ('en-tstst', lambda year: f'en-sols-TSTST-{year}', range(2011, 2024), 0), + ('en-tst', lambda year: f'en-sols-TST-IMO-{year}', range(2014, 2024), 0), + # ('en-elmo', lambda year: f'en-ELMO-{year}-sols', range(2010, 2017), None), #TODO: needs another parser + # ('en-elmo', lambda year: f'en-ELMO-{year}-SL', range(2017, 2019), None), #TODO: needs another parser + ('en-jmo', lambda year: f'en-JMO-{year}-notes', range(2010, 2025), 3), + # ('en-usemo', lambda year: f'en-report-usemo-{year}', range(2019, 2023), None), #TODO: needs another parser +] + + +total_problem_count = 0 +total_solution_count = 0 +for base, basename_, years, tier in configs: + print(base) + problem_count = 0 + solution_count = 0 + seg_base = f"{base}-seg" + os.makedirs(seg_base, exist_ok=True) + for year in years: + basename = basename_(year) + file_path = f"{base}/{basename}.md" + if os.path.exists(file_path): + # print(basename) + problems, solutions = parse(file_path) + solutions = find_mult_solutions(solutions) + pairs_df = join(problems, solutions) + pairs_df = clean(pairs_df) + pairs_df = process_mult_solutions(pairs_df) + pairs_df = add_metadata(pairs_df, year, tier) + problem_count += len(problems) + solution_count += len(pairs_df) + # print(pairs_df) + write_pairs(f"{seg_base}/{basename}.jsonl", pairs_df) + print(f"problem count: {problem_count}") + print(f"solution count: {solution_count}") + total_problem_count += problem_count + total_solution_count += solution_count +print(f"total problem count: {total_problem_count}") +print(f"total solution count: {total_solution_count}") + +# en-usamo +# problem count: 174 +# solution count: 203 +# en-tstst +# problem count: 105 +# solution count: 168 +# en-tst +# problem count: 51 +# solution count: 82 +# en-jmo +# problem count: 90 +# solution count: 127 +# total problem count: 420 +# total solution count: 580 \ No newline at end of file diff --git a/USA_TST_TST/segmented/en-sols-TSTST-2011.jsonl b/USA_TST_TST/segmented/en-sols-TSTST-2011.jsonl new file mode 100644 index 0000000000000000000000000000000000000000..6100bab146b4c044d16e2a6d3bea183a5bf1cf9a --- /dev/null +++ b/USA_TST_TST/segmented/en-sols-TSTST-2011.jsonl @@ -0,0 +1,9 @@ +{"year":2011,"label":"1","problem":"Find all real-valued functions $f$ defined on pairs of real numbers, having the following property: for all real numbers $a, b, c$, the median of $f(a, b), f(b, c), f(c, a)$ equals the median of $a, b, c$. (The median of three real numbers, not necessarily distinct, is the number that is in the middle when the three numbers are arranged in nondecreasing order.)","solution":" We prove the following main claim, from which repeated applications can deduce the problem. Claim - Let $ad_{2}>\\cdots>d_{k}$ and $\\operatorname{gcd}\\left(d_{1}, d_{2}, \\ldots, d_{k}\\right)=1$. For every integer $n \\geq n_{0}$, define $$ x_{n}=\\left\\lfloor\\frac{x_{n-d_{1}}+x_{n-d_{2}}+\\cdots+x_{n-d_{k}}}{k}\\right\\rfloor . $$ Show that the sequence $\\left(x_{n}\\right)$ is eventually constant.","solution":" Note that if the initial terms are contained in some interval $[A, B]$ then they will remain in that interval. Thus the sequence is eventually periodic. Discard initial terms and let the period be $T$; we will consider all indices modulo $T$ from now on. Let $M$ be the maximal term in the sequence (which makes sense since the sequence is periodic). Note that if $x_{n}=M$, we must have $x_{n-d_{i}}=M$ for all $i$ as well. By taking a linear combination $\\sum c_{i} d_{i} \\equiv 1(\\bmod T)($ possibly be Bezout's theorem, since $\\operatorname{gcd}_{i}\\left(d_{i}\\right)=1$ ), we conclude $x_{n-1}=M$, as desired."} +{"year":2011,"label":"9","problem":"Let $n$ be a positive integer. Suppose we are given $2^{n}+1$ distinct sets, each containing finitely many objects. Place each set into one of two categories, the red sets and the blue sets, so that there is at least one set in each category. We define the symmetric difference of two sets as the set of objects belonging to exactly one of the two sets. Prove that there are at least $2^{n}$ different sets which can be obtained as the symmetric difference of a red set and a blue set.","solution":" We can interpret the problem as working with binary strings of length $\\ell \\geq n+1$, with $\\ell$ the number of elements across all sets. Let $F$ be a field of cardinality $2^{\\ell}$, hence $F \\cong \\mathbb{F}_{2}^{\\oplus \\ell}$. Then, we can think of red\/blue as elements of $F$, so we have some $B \\subseteq F$, and an $R \\subseteq F$. We wish to prove that $|B+R| \\geq 2^{n}$. Want $|B+R| \\geq 2^{n}$. Equivalently, any element of a set $X$ with $|X|=2^{n}-1$ should omit some element of $|B+R|$. To prove this: we know $|B|+|R|=2^{n}+1$, and define $$ P(b, r)=\\prod_{x \\in X}(b+r-x) $$ Consider $b^{|B|-1} r^{|R|-1}$. The coefficient of is $\\binom{2^{n}-1}{|B|-1}$, which is odd (say by Lucas theorem), so the nullstellensatz applies."} diff --git a/USA_TST_TST/segmented/en-sols-TSTST-2012.jsonl b/USA_TST_TST/segmented/en-sols-TSTST-2012.jsonl new file mode 100644 index 0000000000000000000000000000000000000000..3b0cf6765d84e413ae83903560c78be9386923e8 --- /dev/null +++ b/USA_TST_TST/segmented/en-sols-TSTST-2012.jsonl @@ -0,0 +1,13 @@ +{"year":2012,"label":"1","problem":"Determine all infinite strings of letters with the following properties: (a) Each letter is either $T$ or $S$, (b) If position $i$ and $j$ both have the letter $T$, then position $i+j$ has the letter $S$, (c) There are infinitely many integers $k$ such that position $2 k-1$ has the $k$ th $T$.","solution":" We wish to find all infinite sequences $a_{1}, a_{2}, \\ldots$ of positive integers satisfying the following properties: (a) $a_{1}1$, let $A_{k}=\\left\\{a_{1}, a_{2}, \\ldots, a_{k}\\right\\}$. By (b) and symmetry, we have $$ 2 k-1 \\geq \\frac{\\left|A_{k}-A_{k}\\right|-1}{2}+\\left|A_{k}\\right| \\geq \\frac{2\\left|A_{k}\\right|-2}{2}+\\left|A_{k}\\right|=2 k-1 . $$ But in order for $\\left|A_{k}-A_{k}\\right|=2\\left|A_{k}\\right|-1$, we must have $A_{k}$ an arithmetic progression, whence $a_{n}=2 n-1$ for all $n$ by taking $k$ arbitrarily large."} +{"year":2012,"label":"2","problem":"Let $A B C D$ be a quadrilateral with $A C=B D$. Diagonals $A C$ and $B D$ meet at $P$. Let $\\omega_{1}$ and $O_{1}$ denote the circumcircle and circumcenter of triangle $A B P$. Let $\\omega_{2}$ and $O_{2}$ denote the circumcircle and circumcenter of triangle $C D P$. Segment $B C$ meets $\\omega_{1}$ and $\\omega_{2}$ again at $S$ and $T$ (other than $B$ and $C$ ), respectively. Let $M$ and $N$ be the midpoints of minor arcs $\\widehat{S P}$ (not including $B$ ) and $\\overparen{T P}$ (not including $C$ ). Prove that $\\overline{M N} \\| \\overline{O_{1} O_{2}}$.","solution":" Let $Q$ be the second intersection point of $\\omega_{1}, \\omega_{2}$. Suffice to show $\\overline{Q P} \\perp \\overline{M N}$. Now $Q$ is the center of a spiral congruence which sends $\\overline{A C} \\mapsto \\overline{B D}$. So $\\triangle Q A B$ and $\\triangle Q C D$ are similar isosceles. Now, $$ \\measuredangle Q P A=\\measuredangle Q B A=\\measuredangle D C Q=\\measuredangle D P Q $$ and so $\\overline{Q P}$ is bisects $\\angle B P C$. ![](https:\/\/cdn.mathpix.com\/cropped\/2024_11_19_b4303c9fa48a8c74b45cg-05.jpg?height=673&width=1200&top_left_y=1234&top_left_x=428) Now, let $I=\\overline{B M} \\cap \\overline{C N} \\cap \\overline{P Q}$ be the incenter of $\\triangle P B C$. Then $I M \\cdot I B=I P \\cdot I Q=$ $I N \\cdot I C$, so $B M N C$ is cyclic, meaning $\\overline{M N}$ is antiparallel to $\\overline{B C}$ through $\\angle B I C$. Since $\\overline{Q P I}$ passes through the circumcenter of $\\triangle B I C$, it follows now $\\overline{Q P I} \\perp \\overline{M N}$ as desired."} +{"year":2012,"label":"3","problem":"Let $\\mathbb{N}$ be the set of positive integers. Let $f: \\mathbb{N} \\rightarrow \\mathbb{N}$ be a function satisfying the following two conditions: (a) $f(m)$ and $f(n)$ are relatively prime whenever $m$ and $n$ are relatively prime. (b) $n \\leq f(n) \\leq n+2012$ for all $n$. Prove that for any natural number $n$ and any prime $p$, if $p$ divides $f(n)$ then $p$ divides $n$.","solution":" \u3010 First short solution, by Jeffrey Kwan. Let $p_{0}, p_{1}, p_{2}, \\ldots$ denote the sequence of all prime numbers, in any order. Pick any primes $q_{i}$ such that $$ q_{0}\\left|f\\left(p_{0}\\right), \\quad q_{1}\\right| f\\left(p_{1}\\right), \\quad q_{2} \\mid f\\left(p_{2}\\right), \\text { etc. } $$ This is possible since each $f$ value above exceeds 1 . Also, since by hypothesis the $f\\left(p_{i}\\right)$ are pairwise coprime, the primes $q_{i}$ are all pairwise distinct. Claim - We must have $q_{i}=p_{i}$ for each $i$. (Therefore, $f\\left(p_{i}\\right)$ is a power of $p_{i}$ for each $i$.) $$ \\begin{array}{rr} m+i & \\equiv 0 \\\\ m & \\left(\\bmod q_{i}\\right) \\\\ m & \\equiv{ }^{\\prime} \\\\ \\left(\\bmod p_{i}\\right) \\end{array} $$ for $0 \\leq i \\leq 2012$. But now $f(m)$ should be coprime to all $f\\left(p_{i}\\right)$, ergo coprime to $q_{0} q_{1} \\ldots q_{2012}$, violating $m \\leq f(m) \\leq m+2012$. All that is left to do is note that whenever $p \\nmid n$, we have $\\operatorname{gcd}(f(p), f(n))=1$, hence $p \\nmid f(n)$. This is the contrapositive of the problem statement."} +{"year":2012,"label":"3","problem":"Let $\\mathbb{N}$ be the set of positive integers. Let $f: \\mathbb{N} \\rightarrow \\mathbb{N}$ be a function satisfying the following two conditions: (a) $f(m)$ and $f(n)$ are relatively prime whenever $m$ and $n$ are relatively prime. (b) $n \\leq f(n) \\leq n+2012$ for all $n$. Prove that for any natural number $n$ and any prime $p$, if $p$ divides $f(n)$ then $p$ divides $n$.","solution":" \u3010 Second solution with a grid. Fix $n$ and $p$, and assume for contradiction $p \\nmid n$. Claim - There exists a large integer $N$ with $f(N)=N$, that also satisfies $N \\equiv 1$ $(\\bmod n)$ and $N \\equiv 0(\\bmod p)$. it to fill in the following table: | | $N+1$ | $N+2$ | $\\ldots$ | $N+2012$ | | :---: | :---: | :---: | :---: | :---: | | $M$ | $q_{0,1}$ | $q_{0,2}$ | $\\ldots$ | $q_{0,2012}$ | | $M+1$ | $q_{1,1}$ | $q_{1,2}$ | $\\ldots$ | $q_{1,2012}$ | | $\\vdots$ | $\\vdots$ | $\\vdots$ | $\\ddots$ | $\\vdots$ | | $M+2012$ | $q_{2012,1}$ | $q_{2012,2}$ | $\\ldots$ | $q_{2012,2012}$ |. By the Chinese Remainder Theorem, we can construct $N$ such that $N+1 \\equiv 0\\left(\\bmod q_{i, 1}\\right)$ for every $i$, and similarly for $N+2$, and so on. Moreover, we can also tack on the extra conditions $N \\equiv 0(\\bmod p)$ and $N \\equiv 1(\\bmod n)$ we wanted. Notice that $N$ cannot be divisible by any of the $q_{i, j}$ 's, since the $q_{i, j}$ 's are greater than 2012. After we've chosen $N$, we can pick $M$ such that $M \\equiv 0\\left(\\bmod q_{0, j}\\right)$ for every $j$, and similarly $M+1 \\equiv 0\\left(\\bmod q_{1, j}\\right)$, et cetera. Moreover, we can tack on the condition $M \\equiv 1$ $(\\bmod N)$, which ensures $\\operatorname{gcd}(M, N)=1$. What does this do? We claim that $f(N)=N$ now. Indeed $f(M)$ and $f(N)$ are relatively prime; but look at the table! The table tells us that $f(M)$ must have a common factor with each of $N+1, \\ldots, N+2012$. So the only possibility is that $f(N)=N$. Now we're basically done. Since $N \\equiv 1(\\bmod n)$, we have $\\operatorname{gcd}(N, n)=1$ and hence $1=\\operatorname{gcd}(f(N), f(n))=\\operatorname{gcd}(N, f(n))$. But $p \\mid N$ and $p \\mid f(n)$, contradiction."} +{"year":2012,"label":"4","problem":"In scalene triangle $A B C$, let the feet of the perpendiculars from $A$ to $\\overline{B C}, B$ to $\\overline{C A}$, $C$ to $\\overline{A B}$ be $A_{1}, B_{1}, C_{1}$, respectively. Denote by $A_{2}$ the intersection of lines $B C$ and $B_{1} C_{1}$. Define $B_{2}$ and $C_{2}$ analogously. Let $D, E, F$ be the respective midpoints of sides $\\overline{B C}, \\overline{C A}, \\overline{A B}$. Show that the perpendiculars from $D$ to $\\overline{A A_{2}}, E$ to $\\overline{B B_{2}}$ and $F$ to $\\overline{C C_{2}}$ are concurrent.","solution":" We claim that they pass through the orthocenter $H$. Indeed, consider the circle with diameter $\\overline{B C}$, which circumscribes quadrilateral $B C B_{1} C_{1}$ and has center $D$. Then by Brokard theorem, $\\overline{A A_{2}}$ is the polar of line $H$. Thus $\\overline{D H} \\perp \\overline{A A_{2}}$."} +{"year":2012,"label":"5","problem":"A rational number $x$ is given. Prove that there exists a sequence $x_{0}, x_{1}, x_{2}, \\ldots$ of rational numbers with the following properties: (a) $x_{0}=x$; (b) for every $n \\geq 1$, either $x_{n}=2 x_{n-1}$ or $x_{n}=2 x_{n-1}+\\frac{1}{n}$; (c) $x_{n}$ is an integer for some $n$.","solution":" Think of the sequence as a process over time. We'll show that: Claim - At any given time $t$, if the denominator of $x_{t}$ has some odd prime power $q=p^{e}$, then we can delete a factor of $p$ from the denominator, while only adding powers of two to the denominator. (Thus we can just delete off all the odd primes one by one and then double appropriately many times.) Indeed, let $n$ be large, and suppose $t<2^{r+1} q<2^{r+2} q<\\cdots<2^{r+m} qb$ if $a b=b a=a$. The following are proved by finite casework, using the fact that $\\{a b, b c, c a\\}$ always has exactly two distinct elements for any different $a, b, c$. - If $a>b$ and $b>c$ then $a>c$. - If $a \\sim b$ and $b \\sim c$ then $a b=a$ if and only if $b c=b$. - If $a \\sim b$ and $b \\sim c$ then $a \\sim c$. - If $a \\sim b$ and $a>c$ then $b>c$. - If $a \\sim b$ and $c>a$ then $c>b$. This gives us the total ordering on the elements and the equivalence classes by $\\sim$. In this we way can check the claimed operations are the only ones."} diff --git a/USA_TST_TST/segmented/en-sols-TSTST-2013.jsonl b/USA_TST_TST/segmented/en-sols-TSTST-2013.jsonl new file mode 100644 index 0000000000000000000000000000000000000000..8f8fde55e80b86e54230a84d535b23365e0f5003 --- /dev/null +++ b/USA_TST_TST/segmented/en-sols-TSTST-2013.jsonl @@ -0,0 +1,9 @@ +{"year":2013,"label":"1","problem":"Let $A B C$ be a triangle and $D, E, F$ be the midpoints of $\\operatorname{arcs} B C, C A, A B$ on the circumcircle. Line $\\ell_{a}$ passes through the feet of the perpendiculars from $A$ to $\\overline{D B}$ and $\\overline{D C}$. Line $m_{a}$ passes through the feet of the perpendiculars from $D$ to $\\overline{A B}$ and $\\overline{A C}$. Let $A_{1}$ denote the intersection of lines $\\ell_{a}$ and $m_{a}$. Define points $B_{1}$ and $C_{1}$ similarly. Prove that triangles $D E F$ and $A_{1} B_{1} C_{1}$ are similar to each other.","solution":" In fact, it is true for any points $D, E, F$ on the circumcircle. More strongly we contend: Claim - Point $A_{1}$ is the midpoint of $\\overline{H D}$. ![](https:\/\/cdn.mathpix.com\/cropped\/2024_11_19_3879e3a8fa25bc819b5bg-04.jpg?height=718&width=801&top_left_y=1297&top_left_x=633) Hence $A_{1} B_{1} C_{1}$ is similar to $D E F$ through a homothety at $H$ with ratio $\\frac{1}{2}$."} +{"year":2013,"label":"2","problem":"A finite sequence of integers $a_{1}, a_{2}, \\ldots, a_{n}$ is called regular if there exists a real number $x$ satisfying $$ \\lfloor k x\\rfloor=a_{k} \\quad \\text { for } 1 \\leq k \\leq n $$ Given a regular sequence $a_{1}, a_{2}, \\ldots, a_{n}$, for $1 \\leq k \\leq n$ we say that the term $a_{k}$ is forced if the following condition is satisfied: the sequence $$ a_{1}, a_{2}, \\ldots, a_{k-1}, b $$ is regular if and only if $b=a_{k}$. Find the maximum possible number of forced terms in a regular sequence with 1000 terms.","solution":" The answer is 985 . WLOG, by shifting $a_{1}=0$ (clearly $a_{1}$ isn't forced). Now, we construct regular sequences inductively using the following procedure. Start with the inequality $$ \\frac{0}{1} \\leq x<\\frac{1}{1} $$ Then for each $k=2,3, \\ldots, 1000$ we perform the following procedure. If there is no fraction of the form $F=\\frac{m}{k}$ in the interval $A \\leq xq>\\max \\{a, b\\}$ be primes. Suppose $s=a^{p} b^{q}$ and $t=s^{2}$; then $$ p g_{t}(a)+q g_{t}(b)=g_{t}\\left(a^{p} b^{q}\\right)=g_{t}(s)=f^{s^{2}-s}(s)-s=0 $$ Now $$ q \\mid g_{t}(a)>-a \\quad \\text { and } \\quad p \\mid g_{t}(b)>-b \\Longrightarrow g_{t}(a)=g_{t}(b)=0 $$ and so we conclude $f^{t-a}(t)=a$ and $f^{t-b}(t)=b$ for $a, b \\geq 2$. In particular, if $a=n$ and $b=n+1$ then we deduce $f(n+1)=n$ for all $n \\geq 2$, as desired. Remark. If you let $c=(a b)^{2}$ after the first lemma, you recover the 2-variable version!"} +{"year":2013,"label":"7","problem":"A country has $n$ cities, labelled $1,2,3, \\ldots, n$. It wants to build exactly $n-1$ roads between certain pairs of cities so that every city is reachable from every other city via some sequence of roads. However, it is not permitted to put roads between pairs of cities that have labels differing by exactly 1 , and it is also not permitted to put a road between cities 1 and $n$. Let $T_{n}$ be the total number of possible ways to build these roads. (a) For all odd $n$, prove that $T_{n}$ is divisible by $n$. (b) For all even $n$, prove that $T_{n}$ is divisible by $n \/ 2$.","solution":" You can just spin the tree! Fixing $n$, the group $G=\\mathbb{Z} \/ n \\mathbb{Z}$ acts on the set of trees by rotation (where we imagine placing $1,2, \\ldots, n$ along a circle). Claim - For odd $n$, all trees have trivial stabilizer."} +{"year":2013,"label":"8","problem":"Define a function $f: \\mathbb{N} \\rightarrow \\mathbb{N}$ by $f(1)=1, f(n+1)=f(n)+2^{f(n)}$ for every positive integer $n$. Prove that $f(1), f(2), \\ldots, f\\left(3^{2013}\\right)$ leave distinct remainders when divided by $3^{2013}$.","solution":" I'll prove by induction on $k \\geq 1$ that any $3^{k}$ consecutive values of $f$ produce distinct residues modulo $3^{k}$. The base case $k=1$ is easily checked ( $f$ is always odd, hence $f$ cycles $1,0,2 \\bmod 3)$. For the inductive step, assume it's true up to $k$. Since $2^{\\bullet}\\left(\\bmod 3^{k+1}\\right)$ cycles every $2 \\cdot 3^{k}$, and $f$ is always odd, it follows that $$ \\begin{aligned} f\\left(n+3^{k}\\right)-f(n) & =2^{f(n)}+2^{f(n+1)}+\\cdots+2^{f\\left(n+3^{k}-1\\right)} \\quad\\left(\\bmod 3^{k+1}\\right) \\\\ & \\equiv 2^{1}+2^{3}+\\cdots+2^{2 \\cdot 3^{k}-1} \\quad\\left(\\bmod 3^{k+1}\\right) \\\\ & =2 \\cdot \\frac{4^{3^{k}}-1}{4-1} \\end{aligned} $$ Hence $$ f\\left(n+3^{k}\\right)-f(n) \\equiv C \\quad\\left(\\bmod 3^{k+1}\\right) \\quad \\text { where } \\quad C=2 \\cdot \\frac{4^{3^{k}}-1}{4-1} $$ noting that $C$ does not depend on $n$. Exponent lifting gives $\\nu_{3}(C)=k$ hence $f(n)$, $f\\left(n+3^{k}\\right), f\\left(n+2 \\cdot 3^{k}\\right)$ differ mod $3^{k+1}$ for all $n$, and the inductive hypothesis now solves the problem."} +{"year":2013,"label":"9","problem":"Let $r$ be a rational number in the interval $[-1,1]$ and let $\\theta=\\cos ^{-1} r$. Call a subset $S$ of the plane good if $S$ is unchanged upon rotation by $\\theta$ around any point of $S$ (in both clockwise and counterclockwise directions). Determine all values of $r$ satisfying the following property: The midpoint of any two points in a good set also lies in the set.","solution":" The answer is that $r$ has this property if and only if $r=\\frac{4 n-1}{4 n}$ for some integer $n$. $$ \\omega=e^{i \\theta}=\\frac{a}{b} \\pm \\frac{\\sqrt{b^{2}-a^{2}}}{b} i $$ This means we may work with complex multiplication in the usual way; the rotation of $z$ through center $c$ is given by $z \\mapsto \\omega(z-c)+c$. - Start by letting $S_{0}=\\{0,1\\}$. - Let $S_{i}$ consist of $S_{i-1}$ plus all points that can be obtained by rotating a point of $S_{i-1}$ through a different point of $S_{i-1}$ (with scale factor $\\omega$ ). - Let $S_{\\infty}=\\bigcup_{i \\geq 0} S_{i}$. The set $S_{\\infty}$ is the (minimal, by inclusion) good set containing 0 and 1 . We are going to show that for most values of $r$, we have $\\frac{1}{2} \\notin S_{\\infty}$. Claim - If $b$ is odd, then $\\frac{1}{2} \\notin S_{\\infty}$. Consider the ring $$ A=\\mathbb{Z}_{\\{b\\}}=\\left\\{\\frac{s}{t}|s, t \\in \\mathbb{Z}, t| b^{\\infty}\\right\\} $$ which consists of all rational numbers whose denominators divide $b^{\\infty}$. Then, $0,1, \\omega \\in$ $A\\left[\\sqrt{b^{2}-a^{2}}\\right]$ and hence $S_{\\infty} \\subseteq A\\left[\\sqrt{b^{2}-a^{2}}\\right]$ too. (This works even if $\\sqrt{b^{2}-a^{2}} \\in \\mathbb{Z}$, in which case $S_{\\infty} \\subseteq A=A\\left[\\sqrt{b^{2}-a^{2}}\\right]$.) But $\\frac{1}{2} \\notin A\\left[\\sqrt{b^{2}-a^{2}}\\right]$. Claim - If $b$ is even and $|b-a| \\neq 1$, then $\\frac{1}{2} \\notin S_{\\infty}$. Let $D=b^{2}-a^{2} \\equiv 3(\\bmod 4)$. Let $p$ be a prime divisor of $b-a$. Because $\\operatorname{gcd}(a, b)=1$, we have $p \\neq 2$ and $p \\nmid b$. Consider the ring $$ A=\\mathbb{Z}_{(p)}=\\left\\{\\left.\\frac{s}{t} \\right\\rvert\\, s, t \\in \\mathbb{Z}, p \\perp t\\right\\} $$ which consists of all rational numbers whose denominators are coprime to $p$. Then, $0,1, \\omega \\in A[\\sqrt{-D}]$ and hence $S_{\\infty} \\subseteq A[\\sqrt{-D}]$ too. Now, there is a well-defined \"mod- $p$ \" ring homomorphism $$ \\Psi: A[\\sqrt{-D}] \\rightarrow \\mathbb{F}_{p} \\quad \\text { by } \\quad x+y \\sqrt{-D} \\mapsto x \\bmod p $$ which commutes with addition and multiplication (as $p \\mid D$ ). Under this map, $$ \\omega \\mapsto \\frac{a}{b} \\bmod p=1 $$ Consequently, the rotation $z \\mapsto \\omega(z-c)+c$ is just the identity map modulo $p$. In other words, the pre-image of any point in $S_{\\infty}$ under $\\Psi$ must be either $\\Psi(0)=0$ or $\\Psi(1)=1$. However, $\\Psi(1 \/ 2)=1 \/ 2$ is neither of these. So this point cannot be achieved. Claim - Suppose $a=2 n-1$ and $b=2 n$ for $n$ an odd integer. Then $\\frac{1}{2} \\notin S_{\\infty}$ This time, we define the ring $$ B=\\mathbb{Z}_{(2)}=\\left\\{\\left.\\frac{s}{t} \\right\\rvert\\, s, t \\in \\mathbb{Z}, t \\text { odd }\\right\\} $$ of rational numbers with odd denominator. We carefully consider the ring $B[\\omega]$ where $$ \\omega=\\frac{2 n-1 \\pm \\sqrt{1-4 n}}{2 n} $$ So $S_{\\infty} \\subseteq B[\\omega]$ as $0,1, \\omega \\in B[\\omega]$. I claim that $B[\\omega]$ is an integral extension of $B$; equivalently that $\\omega$ is integral over $B$. Indeed, $\\omega$ is the root of the monic polynomial $$ (T-1)^{2}+\\frac{1}{n}(T-1)-\\frac{1}{n}=0 $$ where $\\frac{1}{n} \\in B$ makes sense as $n$ is odd. On the other hand, $\\frac{1}{2}$ is not integral over $B$ so it is not an element of $B[\\omega]$. It remains to show that if $r=\\frac{4 n-1}{4 n}$, then goods sets satisfy the midpoint property. Again starting from the points $z_{0}=0, z_{1}=1$ construct the sequence $$ \\begin{aligned} z_{2} & =\\omega\\left(z_{1}-z_{0}\\right)+z_{0} \\\\ z_{3} & =\\omega^{-1}\\left(z_{0}-z_{2}\\right)+z_{2} \\\\ z_{4} & =\\omega^{-1}\\left(z_{2}-z_{3}\\right)+z_{3} \\\\ z_{5} & =\\omega\\left(z_{3}-z_{4}\\right)+z_{4} \\end{aligned} $$ as shown in the diagram below. ![](https:\/\/cdn.mathpix.com\/cropped\/2024_11_19_3879e3a8fa25bc819b5bg-16.jpg?height=321&width=795&top_left_y=2307&top_left_x=633) This construction shows that if we have the length-one segment $\\{0,1\\}$ then we can construct the length-one segment $\\{2 r-2,2 r-1\\}$. In other words, we can shift the segment to the left by $$ 1-(2 r-1)=2(1-r)=\\frac{1}{2 n} $$ Repeating this construction $n$ times gives the desired midpoint $\\frac{1}{2}$."} diff --git a/USA_TST_TST/segmented/en-sols-TSTST-2014.jsonl b/USA_TST_TST/segmented/en-sols-TSTST-2014.jsonl new file mode 100644 index 0000000000000000000000000000000000000000..2535577c182d9f605e2090a7ca58c859b21ef3ce --- /dev/null +++ b/USA_TST_TST/segmented/en-sols-TSTST-2014.jsonl @@ -0,0 +1,6 @@ +{"year":2014,"label":"1","problem":"Let $\\leftarrow$ denote the left arrow key on a standard keyboard. If one opens a text editor and types the keys \"ab $\\leftarrow \\mathrm{cd} \\leftarrow \\leftarrow \\mathrm{e} \\leftarrow \\leftarrow \\mathrm{f}$ \", the result is \"faecdb\". We say that a string $B$ is reachable from a string $A$ if it is possible to insert some amount of $\\leftarrow$ 's in $A$, such that typing the resulting characters produces $B$. So, our example shows that \"faecdb\" is reachable from \"abcdef\". Prove that for any two strings $A$ and $B, A$ is reachable from $B$ if and only if $B$ is reachable from $A$.","solution":" Obviously $A$ and $B$ should have the same multiset of characters, and we focus only on that situation. Claim - If $A=123 \\ldots n$ and $B=\\sigma(1) \\sigma(2) \\ldots \\sigma(n)$ is a permutation of $A$, then $B$ is reachable if and only if it is $\\mathbf{2 1 3}$-avoiding, i.e. there are no indices $i0}$ such that $x \\equiv y \\equiv 1(\\bmod p)$. If the sequence $\\nu_{p}\\left(x^{n}-y\\right)$ of positive integers is nonconstant, then it is unbounded. For this it would be sufficient to prove the following claim. Claim - Let $p$ be an odd prime. Let $x, y \\in \\mathbb{Q}>0$ such that $x \\equiv y \\equiv 1(\\bmod p)$. Suppose $m$ and $n$ are positive integers such that $$ d=\\nu_{p}\\left(x^{n}-y\\right)<\\nu_{p}\\left(x^{m}-y\\right)=e . $$ Then there exists $\\ell$ such that $\\nu_{p}\\left(x^{\\ell}-y\\right) \\geq e+1$. $$ \\nu_{p}\\left(x^{k}-1\\right)=e $$ namely $k=p^{e-d}|m-n|$. (In fact, one could also choose more carefully $k=p^{e-d} \\cdot \\operatorname{gcd}(m-$ $\\left.n, p^{\\infty}\\right)$, so that $k$ is a power of $p$.) Suppose we set $x^{k}=p^{e} u+1$ and $x^{m}=p^{e} v+y$ where $u, v \\in \\mathbb{Q}$ aren't divisible by $p$. Now for any integer $1 \\leq r \\leq p-1$ we consider $$ \\begin{aligned} x^{k r+m}-y & =\\left(p^{e} u+1\\right)^{r} \\cdot\\left(p^{e} v+y\\right)-y \\\\ & =p^{e}(v+y u \\cdot r)+p^{2 e}(\\ldots) \\end{aligned} $$ By selecting $r$ with $r \\equiv-v \/ u(\\bmod p)$, we ensure $p^{e+1} \\mid x^{k r+m}-y$, hence $\\ell=k r+m$ is as desired. $$ x^{\\ell-m} \\equiv 1 \\quad\\left(\\bmod p^{e}\\right) \\text { but } x^{\\ell-m} \\not \\equiv 1 \\quad\\left(\\bmod p^{e+1}\\right) $$ In particular, we need $\\nu_{p}\\left(x^{\\ell-m}-1\\right)=e$ exactly. So the $k$ in the claim must exist if we are going to succeed. On the other hand, if $k$ is some integer for which $\\nu_{p}\\left(x^{k}-1\\right)=e$, then by choosing $\\ell-m$ to be some multiple of $k$ with no extra factors of $p$, we hope that we can get $\\nu_{p}\\left(x^{\\ell}-y\\right)=e+1$. That's why we write $\\ell=k r+m$ and see what happens when we expand."} diff --git a/USA_TST_TST/segmented/en-sols-TSTST-2015.jsonl b/USA_TST_TST/segmented/en-sols-TSTST-2015.jsonl new file mode 100644 index 0000000000000000000000000000000000000000..54f867718d97a982e1bb0c45992c6fd143b9aaaa --- /dev/null +++ b/USA_TST_TST/segmented/en-sols-TSTST-2015.jsonl @@ -0,0 +1,9 @@ +{"year":2015,"label":"1","problem":"Let $a_{1}, a_{2}, \\ldots, a_{n}$ be a sequence of real numbers, and let $m$ be a fixed positive integer less than $n$. We say an index $k$ with $1 \\leq k \\leq n$ is good if there exists some $\\ell$ with $1 \\leq \\ell \\leq m$ such that $$ a_{k}+a_{k+1}+\\cdots+a_{k+\\ell-1} \\geq 0 $$ where the indices are taken modulo $n$. Let $T$ be the set of all good indices. Prove that $$ \\sum_{k \\in T} a_{k} \\geq 0 $$","solution":" First we prove the result if the indices are not taken modulo $n$. Call a number $\\ell$-good if $\\ell$ is the smallest number such that $a_{k}+a_{k+1}+\\cdots+a_{k+\\ell-1} \\geq 0$, and $\\ell \\leq m$. Then if $a_{k}$ is $\\ell$-good, the numbers $a_{k+1}, \\ldots, a_{k+\\ell-1}$ are good as well. Then by greedy from left to right, we can group all the good numbers into blocks with nonnegative sums. Repeatedly take the first good number, if $\\ell$-good, group it with the next $\\ell$ numbers. An example for $m=3$ : $$ \\langle 4\\rangle \\quad \\begin{array}{ccccccccc} -1 & -2 & 6\\rangle & -9 & -7 & \\langle 3\\rangle & \\langle-2 & 4\\rangle & \\langle-1 . \\end{array} $$ We can now return to the original problem. Let $N$ be a large integer; applying the algorithm to $N$ copies of the sequence, we deduce that $$ N \\sum_{k \\in T} a_{k}+c_{N} \\geq 0 $$ where $c_{N}$ represents some \"error\" from left-over terms. As $\\left|c_{N}\\right| \\leq \\sum\\left|a_{i}\\right|$, by taking $N$ large enough we deduce the problem."} +{"year":2015,"label":"2","problem":"Let $A B C$ be a scalene triangle. Let $K_{a}, L_{a}$, and $M_{a}$ be the respective intersections with $B C$ of the internal angle bisector, external angle bisector, and the median from $A$. The circumcircle of $A K_{a} L_{a}$ intersects $A M_{a}$ a second time at a point $X_{a}$ different from $A$. Define $X_{b}$ and $X_{c}$ analogously. Prove that the circumcenter of $X_{a} X_{b} X_{c}$ lies on the Euler line of $A B C$.","solution":" The main content of the problem: Claim - $\\angle H X_{a} G=90^{\\circ}$. This implies the result, since then the desired circumcenter is the midpoint of $\\overline{G H}$. (This is the main difficulty; the Euler line is a red herring.) In what follows, we abbreviate $K_{a} L_{a}, M_{a}, X_{a}$ to $K, L, M, X$. First proof by Brokard. To do this, it suffices to show that $M$ has the same power with respect to the circle with diameter $\\overline{A H}$ and the circle with diameter $\\overline{K L}$. In fact I claim both circles are orthogonal to the circle with diameter $\\overline{B C}$ ! The former follows from Brokard's theorem, noting that $A$ is on the polar of $H$, and the latter follows from the harmonic bundle. ![](https:\/\/cdn.mathpix.com\/cropped\/2024_11_19_8fb7b517def5801eef36g-04.jpg?height=635&width=1109&top_left_y=1410&top_left_x=482) Then $\\overline{A M}$ is the radical axis, so $X$ lies on both circles."} +{"year":2015,"label":"2","problem":"Let $A B C$ be a scalene triangle. Let $K_{a}, L_{a}$, and $M_{a}$ be the respective intersections with $B C$ of the internal angle bisector, external angle bisector, and the median from $A$. The circumcircle of $A K_{a} L_{a}$ intersects $A M_{a}$ a second time at a point $X_{a}$ different from $A$. Define $X_{b}$ and $X_{c}$ analogously. Prove that the circumcenter of $X_{a} X_{b} X_{c}$ lies on the Euler line of $A B C$.","solution":" The main content of the problem: Claim - $\\angle H X_{a} G=90^{\\circ}$. This implies the result, since then the desired circumcenter is the midpoint of $\\overline{G H}$. (This is the main difficulty; the Euler line is a red herring.) In what follows, we abbreviate $K_{a} L_{a}, M_{a}, X_{a}$ to $K, L, M, X$. Second proof by orthocenter reflection, Bendit Chan. As before, we know $M X \\cdot M A=$ $M K \\cdot M L=M B \\cdot M C$, but $X$ lies inside segment $A M$. Construct parallelogram $A B A^{\\prime} C$. Then $M X \\cdot M A^{\\prime}=M B \\cdot M C$, so $X B A^{\\prime} C$ is concyclic. However, it is well-known the circumcircle of $\\triangle B A^{\\prime} C$ (which is the reflection of $(A B C)$ across $\\overline{B C}$ ) passes through $H$ and in fact has diameter $\\overline{A^{\\prime} H}$. So this gives $\\angle H X A^{\\prime}=90^{\\circ}$ as needed."} +{"year":2015,"label":"2","problem":"Let $A B C$ be a scalene triangle. Let $K_{a}, L_{a}$, and $M_{a}$ be the respective intersections with $B C$ of the internal angle bisector, external angle bisector, and the median from $A$. The circumcircle of $A K_{a} L_{a}$ intersects $A M_{a}$ a second time at a point $X_{a}$ different from $A$. Define $X_{b}$ and $X_{c}$ analogously. Prove that the circumcenter of $X_{a} X_{b} X_{c}$ lies on the Euler line of $A B C$.","solution":" The main content of the problem: Claim - $\\angle H X_{a} G=90^{\\circ}$. This implies the result, since then the desired circumcenter is the midpoint of $\\overline{G H}$. (This is the main difficulty; the Euler line is a red herring.) In what follows, we abbreviate $K_{a} L_{a}, M_{a}, X_{a}$ to $K, L, M, X$. Third proof by barycentric coordinates. Alternatively we may just compute $X=\\left(a^{2}\\right.$ : $\\left.2 S_{A}: 2 S_{A}\\right)$. Let $F=\\left(0: S_{C}: S_{B}\\right)$ be the foot from $H$. Then we check that $X H F M$ is cyclic, which is power of a point from $A$."} +{"year":2015,"label":"3","problem":"Let $P$ be the set of all primes, and let $M$ be a non-empty subset of $P$. Suppose that for any non-empty subset $\\left\\{p_{1}, p_{2}, \\ldots, p_{k}\\right\\}$ of $M$, all prime factors of $p_{1} p_{2} \\ldots p_{k}+1$ are also in $M$. Prove that $M=P$.","solution":" Obviously $|M|=\\infty$. Assume for contradiction $p \\notin M$. We say a prime $q \\in M$ is sparse if there are only finitely many elements of $M$ which are $q(\\bmod p)$ (in particular there are finitely many sparse primes). Now let $C$ be the product of all sparse primes (note $p \\nmid C$ ). First, set $a_{0}=1$. For $k \\geq 0$, consider then the prime factorization of the number $$ C a_{k}+1 $$ No prime in its factorization is sparse, so consider the number $a_{k+1}$ obtained by replacing each prime in its factorization with some arbitrary representative of that prime's residue class. In this way we select a number $a_{k+1}$ such that - $a_{k+1} \\equiv C a_{k}+1(\\bmod p)$, and - $a_{k+1}$ is a product of distinct primes in $M$. In particular, $a_{k} \\equiv C^{k}+C^{k-1}+\\cdots+1(\\bmod p)$ But since $C \\not \\equiv 0(\\bmod p)$, we can find a $k$ such that $a_{k} \\equiv 0(\\bmod p)($ namely, $k=p-1$ if $C \\equiv 1$ and $k=p-2$ else) which is clearly impossible since $a_{k}$ is a product of primes in $M$ !"} +{"year":2015,"label":"4","problem":"Let $x, y, z$ be real numbers (not necessarily positive) such that $x^{4}+y^{4}+z^{4}+x y z=4$. Prove that $x \\leq 2$ and $$ \\sqrt{2-x} \\geq \\frac{y+z}{2} $$","solution":" We prove that the condition $x^{4}+y^{4}+z^{4}+x y z=4$ implies $$ \\sqrt{2-x} \\geq \\frac{y+z}{2} $$ We first prove the easy part. Claim - We have $x \\leq 2$. $$ \\begin{aligned} 5=x^{4}+y^{4}+\\left(z^{4}+1\\right)+x y z & =\\frac{3 x^{4}}{4}+\\left(\\frac{x^{4}}{4}+y^{4}\\right)+\\left(z^{4}+1\\right)+x y z \\\\ & \\geq \\frac{3 x^{4}}{4}+x^{2} y^{2}+2 z^{2}+x y z . \\end{aligned} $$ We evidently have that $x^{2} y^{2}+2 z^{2}+x y z \\geq 0$ because the quadratic form $a^{2}+a b+2 b^{2}$ is positive definite, so $x^{4} \\leq \\frac{20}{3} \\Longrightarrow x \\leq 2$. Now, the desired statement is implied by its square, so it suffices to show that $$ 2-x \\geq\\left(\\frac{y+z}{2}\\right)^{2} $$ We are going to proceed by contradiction (it seems that many solutions do this) and assume that $$ 2-x<\\left(\\frac{y+z}{2}\\right)^{2} \\Longleftrightarrow 4 x+y^{2}+2 y z+z^{2}>8 $$ By AM-GM, $$ \\begin{aligned} x^{4}+3 & \\geq 4 x \\\\ \\frac{y^{4}+1}{2} & \\geq y^{2} \\\\ \\frac{z^{4}+1}{2} & \\geq z^{2} \\end{aligned} $$ which yields that $$ x^{4}+\\frac{y^{4}+z^{4}}{2}+2 y z+4>8 $$ If we replace $x^{4}=4-\\left(y^{4}+z^{4}+x y z\\right)$ now, this gives $$ -\\frac{y^{4}+z^{4}}{2}+(2-x) y z>0 \\Longrightarrow(2-x) y z>\\frac{y^{4}+z^{4}}{2} $$ Since $2-x$ and the right-hand side are positive, we have $y z \\geq 0$. Now $$ \\frac{y^{4}+z^{4}}{2 y z}<2-x<\\left(\\frac{y+z}{2}\\right)^{2} \\Longrightarrow 2 y^{4}+2 z^{4}2015$ we're done."} +{"year":2015,"label":"5","problem":"Let $\\varphi(n)$ denote the number of positive integers less than $n$ that are relatively prime to $n$. Prove that there exists a positive integer $m$ for which the equation $\\varphi(n)=m$ has at least 2015 solutions in $n$.","solution":" \u3010 Second solution with smallest primes, by Yang Liu. Let $2=p_{1}1$. Then Alice can complete the phase without losing if and only if $x$ is even; if so she begins a $Y$-phase with $(X, Y)=(0, x \/ 2)$. Now note that unless $X=0$, Bob now has a winning move WrongX. Conversely he may only play Sleep if $X=0$. We have an analogous claim for $Y$-phases. Thus if $n$ is not a power of 2 , we see that Alice eventually loses. Now suppose $n=2^{k}$; then Alice reaches $(X, Y)=\\left(0,2^{k-1}\\right),\\left(2^{k-2}, 0\\right), \\ldots$ until either reaching $(1,0)$ or $(0,1)$. At this point she can play ClaimX or ClaimY, respectively; the game is now in state Cl. Bob cannot play either FakeX or FakeY, so he must play Sleep, and then Alice wins by playing Win. Thus Alice has a winning strategy when $n=2^{k}$."} diff --git a/USA_TST_TST/segmented/en-sols-TSTST-2016.jsonl b/USA_TST_TST/segmented/en-sols-TSTST-2016.jsonl new file mode 100644 index 0000000000000000000000000000000000000000..e59409cd04ec0f6097fdd5e9646ca673e76fa545 --- /dev/null +++ b/USA_TST_TST/segmented/en-sols-TSTST-2016.jsonl @@ -0,0 +1,8 @@ +{"year":2016,"label":"1","problem":"Let $A=A(x, y)$ and $B=B(x, y)$ be two-variable polynomials with real coefficients. Suppose that $A(x, y) \/ B(x, y)$ is a polynomial in $x$ for infinitely many values of $y$, and a polynomial in $y$ for infinitely many values of $x$. Prove that $B$ divides $A$, meaning there exists a third polynomial $C$ with real coefficients such that $A=B \\cdot C$.","solution":" This is essentially an application of the division algorithm, but the details require significant care. First, we claim that $A \/ B$ can be written as a polynomial in $x$ whose coefficients are rational functions in $y$. To see this, use the division algorithm to get $$ A=Q \\cdot B+R \\quad Q, R \\in(\\mathbb{R}(y))[x] $$ where $Q$ and $R$ are polynomials in $x$ whose coefficients are rational functions in $y$, and moreover $\\operatorname{deg}_{x} B>\\operatorname{deg}_{x} R$. Now, we claim that $R \\equiv 0$. Indeed, we have by hypothesis that for infinitely many values of $y_{0}$ that $B\\left(x, y_{0}\\right)$ divides $A\\left(x, y_{0}\\right)$, which means $B\\left(x, y_{0}\\right) \\mid R\\left(x, y_{0}\\right)$ as polynomials in $\\mathbb{R}[x]$. Now, we have $\\operatorname{deg}_{x} B\\left(x, y_{0}\\right)>\\operatorname{deg}_{x} R\\left(x, y_{0}\\right)$ outside of finitely many values of $y_{0}$ (but not all of them!); this means for infinitely many $y_{0}$ we have $R\\left(x, y_{0}\\right) \\equiv 0$. So each coefficient of $x^{i}$ (in $\\left.\\mathbb{R}(y)\\right)$ has infinitely many roots, hence is a zero polynomial. Consequently, we are able to write $A \/ B=F(x, y) \/ M(y)$ where $F \\in \\mathbb{R}[x, y]$ and $M \\in \\mathbb{R}[y]$ are each polynomials. Repeating the same argument now gives $$ \\frac{A}{B}=\\frac{F(x, y)}{M(y)}=\\frac{G(x, y)}{N(x)} $$ Now, by unique factorization of polynomials in $\\mathbb{R}[x, y]$, we can discuss GCD's. So, we tacitly assume $\\operatorname{gcd}(F, M)=\\operatorname{gcd}(G, N)=(1)$. Also, we obviously have $\\operatorname{gcd}(M, N)=(1)$. But $F \\cdot N=G \\cdot M$, so $M \\mid F \\cdot N$, thus we conclude $M$ is the constant polynomial. This implies the result. Remark. This fact does not generalize to arbitrary functions that are separately polynomial: see e.g. http:\/\/aops.com\/community\/c6h523650p2978180."} +{"year":2016,"label":"2","problem":"Let $A B C$ be a scalene triangle with orthocenter $H$ and circumcenter $O$ and denote by $M, N$ the midpoints of $\\overline{A H}, \\overline{B C}$. Suppose the circle $\\gamma$ with diameter $\\overline{A H}$ meets the circumcircle of $A B C$ at $G \\neq A$, and meets line $\\overline{A N}$ at $Q \\neq A$. The tangent to $\\gamma$ at $G$ meets line $O M$ at $P$. Show that the circumcircles of $\\triangle G N Q$ and $\\triangle M B C$ intersect on $\\overline{P N}$.","solution":" \u3010 First solution (found by contestants). Denote by $\\triangle D E F$ the orthic triangle. Observe $\\overline{P A}$ and $\\overline{P G}$ are tangents to $\\gamma$, since $\\overline{O M}$ is the perpendicular bisector of $\\overline{A G}$. Also note that $\\overline{A G}, \\overline{E F}, \\overline{B C}$ are concurrent at some point $R$ by radical axis on $(A B C), \\gamma$, (BFEC). Now, consider circles (PAGM), (MFDNE), and (MBC). We already saw the point $R$ satisfies $$ R A \\cdot R G=R E \\cdot R F=R B \\cdot R C $$ and hence has equal powers to all three circles; but since the circles at $M$ already, they must actually be coaxial. Assume they meet again at $T \\in \\overline{R M}$, say. Then $\\angle P T M$ and $\\angle M T N$ are both right angles, hence $T$ lies on $\\overline{P N}$. Finally $H$ is the orthocenter of $\\triangle A R N$, and thus the circle with diameter $\\overline{R N}$ passes through $G, Q, N$. ![](https:\/\/cdn.mathpix.com\/cropped\/2024_11_19_f48de8bcaa95c5ff4531g-04.jpg?height=969&width=1117&top_left_y=1591&top_left_x=475)"} +{"year":2016,"label":"2","problem":"Let $A B C$ be a scalene triangle with orthocenter $H$ and circumcenter $O$ and denote by $M, N$ the midpoints of $\\overline{A H}, \\overline{B C}$. Suppose the circle $\\gamma$ with diameter $\\overline{A H}$ meets the circumcircle of $A B C$ at $G \\neq A$, and meets line $\\overline{A N}$ at $Q \\neq A$. The tangent to $\\gamma$ at $G$ meets line $O M$ at $P$. Show that the circumcircles of $\\triangle G N Q$ and $\\triangle M B C$ intersect on $\\overline{P N}$.","solution":" \u300e Alternate solution (by proposer). Let $L$ be diametrically opposite $A$ on the circumcircle. Denote by $\\triangle D E F$ the orthic triangle. Let $X=\\overline{A H} \\cap \\overline{E F}$. Finally, let $T$ be the second intersection of (MFDNE) and (MBC). ![](https:\/\/cdn.mathpix.com\/cropped\/2024_11_19_f48de8bcaa95c5ff4531g-05.jpg?height=838&width=1109&top_left_y=429&top_left_x=482) We begin with a few easy observations. First, points $H, G, N, L$ are collinear and $\\angle A G L=90^{\\circ}$. Also, $Q$ is the foot from $H$ to $\\overline{A N}$. Consequently, lines $A G, E F, H Q$, $B C, T M$ concur at a point $R$ (radical axis). Moreover, we already know $\\angle M T N=90^{\\circ}$. This implies $T$ lies on the circle with diameter $\\overline{R N}$, which is exactly the circumcircle of $\\triangle G Q N$. Note by Brokard's Theorem on $A F H E$, the point $X$ is the orthocenter of $\\triangle M B C$. But $\\angle M T N=90^{\\circ}$ already, and $N$ is the midpoint of $\\overline{B C}$. Consequently, points $T, X$, $N$ are collinear. Finally, we claim $P, X, N$ are collinear, which solves the problem. Note $P=\\overline{G G} \\cap \\overline{A A}$. Set $K=\\overline{H N L} \\cap \\overline{A P}$. Then by noting $$ -1=(D, X ; A, H) \\stackrel{N}{=}(\\infty, \\overline{N X} \\cap \\overline{A K} ; A, K) $$ we see that $\\overline{N X}$ bisects segment $\\overline{A K}$, as desired. (A more projective finish is to show that $\\overline{P X N}$ is the polar of $R$ to $\\gamma$ ). Remark. The original problem proposal reads as follows: Let $A B C$ be a triangle with orthocenter $H$ and circumcenter $O$ and denote by $M, N$ the midpoints of $\\overline{A H}, \\overline{B C}$. Suppose ray $O M$ meets the line parallel to $\\overline{B C}$ through $A$ at $P$. Prove that the line through the circumcenter of $\\triangle M B C$ and the midpoint of $\\overline{O H}$ is parallel to $\\overline{N P}$. The points $G$ and $Q$ were added to the picture later to prevent the problem from being immediate by coordinates."} +{"year":2016,"label":"3","problem":"Decide whether or not there exists a nonconstant polynomial $Q(x)$ with integer coefficients with the following property: for every positive integer $n>2$, the numbers $$ Q(0), Q(1), Q(2), \\ldots, Q(n-1) $$ produce at most $0.499 n$ distinct residues when taken modulo $n$.","solution":" We claim that $$ Q(x)=420\\left(x^{2}-1\\right)^{2} $$ works. Clearly, it suffices to prove the result when $n=4$ and when $n$ is an odd prime $p$. The case $n=4$ is trivial, so assume now $n=p$ is an odd prime. First, we prove the following easy claim. Claim - For any odd prime $p$, there are at least $\\frac{1}{2}(p-3)$ values of $a$ for which $\\left(\\frac{1-a^{2}}{p}\\right)=+1$. Remark. The above identity comes from starting with the equation $1-a^{2}=b^{2}$, and writing it as $\\left(\\frac{1}{b}\\right)^{2}-\\left(\\frac{a}{b}\\right)^{2}=1$. Then solve $\\frac{1}{b}-\\frac{a}{b}=k$ and $\\frac{1}{b}+\\frac{a}{b}=1 \/ k$ for $a$. Let $F(x)=\\left(x^{2}-1\\right)^{2}$. The range of $F$ modulo $p$ is contained within the $\\frac{1}{2}(p+1)$ quadratic residues modulo $p$. On the other hand, if for some $t$ neither of $1 \\pm t$ is a quadratic residue, then $t^{2}$ is omitted from the range of $F$ as well. Call such a value of $t$ useful, and let $N$ be the number of useful residues. We aim to show $N \\geq \\frac{1}{4} p-2$. We compute a lower bound on the number $N$ of useful $t$ by writing $$ \\begin{aligned} N & =\\frac{1}{4}\\left(\\sum_{t}\\left[\\left(1-\\left(\\frac{1-t}{p}\\right)\\right)\\left(1-\\left(\\frac{1+t}{p}\\right)\\right)\\right]-\\left(1-\\left(\\frac{2}{p}\\right)\\right)-\\left(1-\\left(\\frac{-2}{p}\\right)\\right)\\right) \\\\ & \\geq \\frac{1}{4} \\sum_{t}\\left[\\left(1-\\left(\\frac{1-t}{p}\\right)\\right)\\left(1-\\left(\\frac{1+t}{p}\\right)\\right)\\right]-1 \\\\ & =\\frac{1}{4}\\left(p+\\sum_{t}\\left(\\frac{1-t^{2}}{p}\\right)\\right)-1 \\\\ & \\geq \\frac{1}{4}\\left(p+(+1) \\cdot \\frac{1}{2}(p-3)+0 \\cdot 2+(-1) \\cdot\\left((p-2)-\\frac{1}{2}(p-3)\\right)\\right)-1 \\\\ & \\geq \\frac{1}{4}(p-5) . \\end{aligned} $$ Thus, the range of $F$ has size at most $$ \\frac{1}{2}(p+1)-\\frac{1}{2} N \\leq \\frac{3}{8}(p+3) . $$ This is less than $0.499 p$ for any $p \\geq 11$. Remark. In fact, the computation above is essentially an equality. There are only two points where terms are dropped: one, when $p \\equiv 3(\\bmod 4)$ there are no $k^{2}=-1$ in the lemma, and secondly, the terms $1-(2 \/ p)$ and $1-(-2 \/ p)$ are dropped in the initial estimate for $N$. With suitable modifications, one can show that in fact, the range of $F$ is exactly equal to $$ \\frac{1}{2}(p+1)-\\frac{1}{2} N=\\left\\{\\begin{array}{lll} \\frac{1}{8}(3 p+5) & p \\equiv 1 & (\\bmod 8) \\\\ \\frac{1}{8}(3 p+7) & p \\equiv 3 & (\\bmod 8) \\\\ \\frac{1}{8}(3 p+9) & p \\equiv 5 & (\\bmod 8) \\\\ \\frac{1}{8}(3 p+3) & p \\equiv 7 & (\\bmod 8) \\end{array}\\right. $$"} +{"year":2016,"label":"4","problem":"Prove that if $n$ and $k$ are positive integers satisfying $\\varphi^{k}(n)=1$, then $n \\leq 3^{k}$. (Here $\\varphi^{k}$ denotes $k$ applications of the Euler phi function.)","solution":" The main observation is that the exponent of 2 decreases by at most 1 with each application of $\\varphi$. This will give us the desired estimate. Define the weight function $w$ on positive integers as follows: it satisfies $$ \\begin{aligned} w(a b) & =w(a)+w(b) \\\\ w(2) & =1 ; \\quad \\text { and } \\\\ w(p) & =w(p-1) \\quad \\text { for any prime } p>2 \\end{aligned} $$ By induction, we see that $w(n)$ counts the powers of 2 that are produced as $\\varphi$ is repeatedly applied to $n$. In particular, $k \\geq w(n)$. From $w(2)=1$, it suffices to prove that $w(p) \\geq \\log _{3} p$ for every $p>2$. We use strong induction and note that $$ w(p)=w(2)+w\\left(\\frac{p-1}{2}\\right) \\geq 1+\\log _{3}(p-1)-\\log _{3} 2 \\geq \\log _{3} p $$ for any $p>2$. This solves the problem. Moreover, the stronger bound $$ n \\leq 2 \\cdot 3^{k-1} $$ is true and best possible."} +{"year":2016,"label":"5","problem":"In the coordinate plane are finitely many walls, which are disjoint line segments, none of which are parallel to either axis. A bulldozer starts at an arbitrary point and moves in the $+x$ direction. Every time it hits a wall, it turns at a right angle to its path, away from the wall, and continues moving. (Thus the bulldozer always moves parallel to the axes.) Prove that it is impossible for the bulldozer to hit both sides of every wall.","solution":" We say a wall $v$ is above another wall $w$ if some point on $v$ is directly above a point on $w$. (This relation is anti-symmetric, as walls do not intersect). The critical claim is as follows: Claim - There exists a lowest wall, i.e. a wall not above any other walls. Now consider the leftmost vertical segment $\\overline{Q_{i} P_{i+1}}$ and the rightmost vertical segment $\\overline{Q_{j} P_{j+1}}$. The broken line gives a path from $P_{i+1}$ to $Q_{j}$, as well as a path from $P_{j+1}$ to $Q_{i}$. These clearly must intersect, contradiction. Remark. This claim is Iran TST 2010. Thus if the bulldozer eventually moves upwards indefinitely, it may never hit the bottom side of the lowest wall. Similarly, if the bulldozer eventually moves downwards indefinitely, it may never hit the upper side of the highest wall."} +{"year":2016,"label":"6","problem":"Let $A B C$ be a triangle with incenter $I$, and whose incircle is tangent to $\\overline{B C}, \\overline{C A}$, $\\overline{A B}$ at $D, E, F$, respectively. Let $K$ be the foot of the altitude from $D$ to $\\overline{E F}$. Suppose that the circumcircle of $\\triangle A I B$ meets the incircle at two distinct points $C_{1}$ and $C_{2}$, while the circumcircle of $\\triangle A I C$ meets the incircle at two distinct points $B_{1}$ and $B_{2}$. Prove that the radical axis of the circumcircles of $\\triangle B B_{1} B_{2}$ and $\\triangle C C_{1} C_{2}$ passes through the midpoint $M$ of $\\overline{D K}$.","solution":" \\I First solution (Allen Liu). Let $X, Y, Z$ be midpoints of $E F, F D, D E$, and let $G$ be the Gergonne point. By radical axis on $(A E I F),(D E F),(A I C)$ we see that $B_{1}$, $X, B_{2}$ are collinear. Likewise, $B_{1}, Z, B_{2}$ are collinear, so lines $B_{1} B_{2}$ and $X Z$ coincide. Similarly, lines $C_{1} C_{2}$ and $X Y$ coincide. In particular lines $B_{1} B_{2}$ and $C_{1} C_{2}$ meet at $X$. ![](https:\/\/cdn.mathpix.com\/cropped\/2024_11_19_f48de8bcaa95c5ff4531g-10.jpg?height=1212&width=1043&top_left_y=1150&top_left_x=512) Note $G$ is the symmedian point of $D E F$, so it is well-known that $X G$ passes through the midpoint of $D K$. So we just have to prove $G$ lies on the radical axis. First, note that $\\triangle D E F$ is the cevian triangle of the Gergonne point $G$. Set $V=$ $\\overline{X Y} \\cap \\overline{A B}, W=\\overline{X Z} \\cap \\overline{A C}$, and $T=\\overline{B W} \\cap \\overline{C V}$. We begin with the following completely projective claim. Claim - The points $X, G, T$ are collinear. - By Cevian Nest on $\\triangle A B C$, it follows that $\\overline{A X}, \\overline{B Y}, \\overline{C Z}$ are concurrent. - Hence $\\triangle B Y V$ and $\\triangle C Z W$ are perspective. - Hence $\\triangle B Z W$ and $\\triangle C Y V$ are perspective too. - Hence we deduce by Desargues theorem that $T, X$, and $\\overline{B Z} \\cap \\overline{C Y}$ are collinear. - Finally, the Cevian Nest theorem applied on $\\triangle G B C$ (which has cevian triangles $\\triangle D F E, \\triangle X Z Y$ ) we deduce $G, X$, and $\\overline{B Z} \\cap \\overline{C Y}$, proving the claim. One could also proceed by using barycentric coordinates on $\\triangle D E F$. Remark (Eric Shen). The first four bullets can be replaced by non-projective means: one can check that $\\overline{B Z} \\cap \\overline{C Y}$ is the radical center of $(B I C),\\left(B B_{1} B_{2}\\right),\\left(C C_{1} C_{2}\\right)$ and therefore it lies on line $\\overline{X T}$. Now, we contend point $V$ is the radical center $\\left(C C_{1} C_{2}\\right),(A B C)$ and $(D E F)$. To see this, let $V^{\\prime}=\\overline{E D} \\cap \\overline{A B}$; then $\\left(F V^{\\prime} ; A B\\right)$ is harmonic, and $V$ is the midpoint of $\\overline{F V^{\\prime}}$, and thus $V A \\cdot V B=V F^{2}=V C_{1} \\cdot V C_{2}$. So in fact $\\overline{C V}$ is the radical axis of $(A B C)$ and $\\left(C C_{1} C_{2}\\right)$. Similarly, $\\overline{B W}$ is the radical axis of $(A B C)$ and $\\left(B B_{1} B_{2}\\right)$. Thus $T$ is the radical center of $(A B C),\\left(B B_{1} B_{2}\\right),\\left(C C_{1} C_{2}\\right)$."} +{"year":2016,"label":"6","problem":"Let $A B C$ be a triangle with incenter $I$, and whose incircle is tangent to $\\overline{B C}, \\overline{C A}$, $\\overline{A B}$ at $D, E, F$, respectively. Let $K$ be the foot of the altitude from $D$ to $\\overline{E F}$. Suppose that the circumcircle of $\\triangle A I B$ meets the incircle at two distinct points $C_{1}$ and $C_{2}$, while the circumcircle of $\\triangle A I C$ meets the incircle at two distinct points $B_{1}$ and $B_{2}$. Prove that the radical axis of the circumcircles of $\\triangle B B_{1} B_{2}$ and $\\triangle C C_{1} C_{2}$ passes through the midpoint $M$ of $\\overline{D K}$.","solution":" \u300e Second solution (Evan Chen). As before, we just have to prove $G$ lies on the radical axis. ![](https:\/\/cdn.mathpix.com\/cropped\/2024_11_19_f48de8bcaa95c5ff4531g-12.jpg?height=1392&width=1260&top_left_y=249&top_left_x=404) Construct parallelograms $G P F Q, G R D S, G T U E$ such that $P, R \\in D F, S, T \\in D E$, $Q, U \\in E F$. As $F G$ bisects $P Q$ and is isogonal to $F Z$, we find $P Q E D$, hence $P Q R U$, is cyclic. Repeating the same logic and noticing $P R, S T, Q U$ not concurrent, all six points $P Q R S T U$ are cyclic. Moreover, since $P Q$ bisects $G F$, we see that a dilation with factor 2 at $G$ sends $P Q$ to $P^{\\prime}, Q^{\\prime} \\in A B$, say, with $F$ the midpoint of $P^{\\prime} Q^{\\prime}$. Define $R^{\\prime}, S^{\\prime} \\in B C$ similarly now and $T^{\\prime}, U^{\\prime} \\in C A$. Note that $E Q P D S^{\\prime}$ is in cyclic too, as $\\measuredangle D S^{\\prime} Q=\\measuredangle D R S=\\measuredangle D E F$. By homothety through $B$, points $B, P, X$ are collinear; assume they meet ( $\\left.E Q P D S^{\\prime}\\right)$ again at $V$. Thus $E V Q P D S^{\\prime}$ is cyclic, and now $$ \\measuredangle B V S^{\\prime}=\\measuredangle P V S^{\\prime}=\\measuredangle P Q S=\\measuredangle P T S=\\measuredangle F E D=\\measuredangle X E Z=\\measuredangle X V Z $$ hence $V$ lies on $\\left(B Q^{\\prime} S^{\\prime}\\right)$. Since $F B \\| Q P$, we get $E V F B$ is cyclic too, so $X V \\cdot X B=X E \\cdot X F$ now; thus $X$ lies on the radical axis of $\\left(B S^{\\prime} Q^{\\prime}\\right)$ and $(D E F)$. By the same argument with $W \\in B Z$, we get $Z$ lies on the radical axis too. Thus the radical axis of $\\left(B S^{\\prime} Q^{\\prime}\\right)$ and ( $D E F$ ) must be line $X Z$, which coincides with $B_{1} B_{2}$; so $\\left(B B_{1} B_{2}\\right)=\\left(B S^{\\prime} Q^{\\prime}\\right)$. Analogously, $\\left(C C_{1} C_{2}\\right)=\\left(C R^{\\prime} U^{\\prime}\\right)$. Since $G=Q^{\\prime} S^{\\prime} \\cap R^{\\prime} U^{\\prime}$, we need only prove that $Q^{\\prime} R^{\\prime} S^{\\prime} U^{\\prime}$ is cyclic. But $Q R S U$ is cyclic, so we are done. The circle ( $P Q R S T U$ ) is called the Lemoine circle of $A B C$."} diff --git a/USA_TST_TST/segmented/en-sols-TSTST-2017.jsonl b/USA_TST_TST/segmented/en-sols-TSTST-2017.jsonl new file mode 100644 index 0000000000000000000000000000000000000000..cb7a32b8a6ae423be65c1ec097ef9f37a1c5ef6b --- /dev/null +++ b/USA_TST_TST/segmented/en-sols-TSTST-2017.jsonl @@ -0,0 +1,13 @@ +{"year":2017,"label":"1","problem":"Let $A B C$ be a triangle with circumcircle $\\Gamma$, circumcenter $O$, and orthocenter $H$. Assume that $A B \\neq A C$ and $\\angle A \\neq 90^{\\circ}$. Let $M$ and $N$ be the midpoints of $\\overline{A B}$ and $\\overline{A C}$, respectively, and let $E$ and $F$ be the feet of the altitudes from $B$ and $C$ in $\\triangle A B C$, respectively. Let $P$ be the intersection point of line $M N$ with the tangent line to $\\Gamma$ at $A$. Let $Q$ be the intersection point, other than $A$, of $\\Gamma$ with the circumcircle of $\\triangle A E F$. Let $R$ be the intersection point of lines $A Q$ and $E F$. Prove that $\\overline{P R} \\perp \\overline{O H}$.","solution":" \u3010 First solution (power of a point). Let $\\gamma$ denote the nine-point circle of $A B C$. ![](https:\/\/cdn.mathpix.com\/cropped\/2024_11_19_b5f71e11be2d66f7b920g-03.jpg?height=609&width=795&top_left_y=1140&top_left_x=639) Note that - $P A^{2}=P M \\cdot P N$, so $P$ lies on the radical axis of $\\Gamma$ and $\\gamma$. - $R A \\cdot R Q=R E \\cdot R F$, so $R$ lies on the radical axis of $\\Gamma$ and $\\gamma$. Thus $\\overline{P R}$ is the radical axis of $\\Gamma$ and $\\gamma$, which is evidently perpendicular to $\\overline{O H}$. Remark. In fact, by power of a point one may also observe that $R$ lies on $\\overline{B C}$, since it is on the radical axis of $(A Q F H E),(B F E C),(A B C)$. Ironically, this fact is not used in the solution."} +{"year":2017,"label":"1","problem":"Let $A B C$ be a triangle with circumcircle $\\Gamma$, circumcenter $O$, and orthocenter $H$. Assume that $A B \\neq A C$ and $\\angle A \\neq 90^{\\circ}$. Let $M$ and $N$ be the midpoints of $\\overline{A B}$ and $\\overline{A C}$, respectively, and let $E$ and $F$ be the feet of the altitudes from $B$ and $C$ in $\\triangle A B C$, respectively. Let $P$ be the intersection point of line $M N$ with the tangent line to $\\Gamma$ at $A$. Let $Q$ be the intersection point, other than $A$, of $\\Gamma$ with the circumcircle of $\\triangle A E F$. Let $R$ be the intersection point of lines $A Q$ and $E F$. Prove that $\\overline{P R} \\perp \\overline{O H}$.","solution":" II Second solution (barycentric coordinates). Again note first $R \\in \\overline{B C}$ (although this can be avoided too). We compute the points in much the same way as before. Since $\\overline{A P} \\cap \\overline{B C}=\\left(0: b^{2}:-c^{2}\\right)$ we have $$ P=\\left(b^{2}-c^{2}: b^{2}:-c^{2}\\right) $$ (since $x=y+z$ is the equation of line $\\overline{M N}$ ). Now in Conway notation we have $$ R=\\overline{E F} \\cap \\overline{B C}=\\left(0: S_{C}:-S_{B}\\right)=\\left(0: a^{2}+b^{2}-c^{2}:-a^{2}+b^{2}-c^{2}\\right) . $$ Hence $$ \\overrightarrow{P R}=\\frac{1}{2\\left(b^{2}-c^{2}\\right)}\\left(b^{2}-c^{2}, c^{2}-a^{2}, a^{2}-b^{2}\\right) $$ On the other hand, we have $\\overrightarrow{O H}=\\vec{A}+\\vec{B}+\\vec{C}$. So it suffices to check that $$ \\sum_{\\mathrm{cyc}} a^{2}\\left(\\left(a^{2}-b^{2}\\right)+\\left(c^{2}-a^{2}\\right)\\right)=0 $$ which is immediate."} +{"year":2017,"label":"1","problem":"Let $A B C$ be a triangle with circumcircle $\\Gamma$, circumcenter $O$, and orthocenter $H$. Assume that $A B \\neq A C$ and $\\angle A \\neq 90^{\\circ}$. Let $M$ and $N$ be the midpoints of $\\overline{A B}$ and $\\overline{A C}$, respectively, and let $E$ and $F$ be the feet of the altitudes from $B$ and $C$ in $\\triangle A B C$, respectively. Let $P$ be the intersection point of line $M N$ with the tangent line to $\\Gamma$ at $A$. Let $Q$ be the intersection point, other than $A$, of $\\Gamma$ with the circumcircle of $\\triangle A E F$. Let $R$ be the intersection point of lines $A Q$ and $E F$. Prove that $\\overline{P R} \\perp \\overline{O H}$.","solution":" \u300e Third solution (complex numbers). Let $A B C$ be the unit circle. We first compute $P$ as the midpoint of $A$ and $\\overline{A A} \\cap \\overline{B C}$ : $$ \\begin{aligned} p & =\\frac{1}{2}\\left(a+\\frac{a^{2}(b+c)-b c \\cdot 2 a}{a^{2}-b c}\\right) \\\\ & =\\frac{a\\left(a^{2}-b c\\right)+a^{2}(b+c)-2 a b c}{2\\left(a^{2}-b c\\right)} \\end{aligned} $$ Using the remark above, $R$ is the inverse of $D$ with respect to the circle with diameter $\\overline{B C}$, which has radius $\\left|\\frac{1}{2}(b-c)\\right|$. Thus $$ \\begin{aligned} r-\\frac{b+c}{2} & =\\frac{\\frac{1}{4}(b-c)\\left(\\frac{1}{b}-\\frac{1}{c}\\right)}{\\frac{1}{2}\\left(a-\\frac{b c}{a}\\right)} \\\\ r & =\\frac{b+c}{2}+\\frac{-\\frac{1}{2} \\frac{(b-c)^{2}}{b c}}{\\frac{1}{a}-\\frac{a}{b c}} \\\\ & =\\frac{b+c}{2}+\\frac{a(b-c)^{2}}{2\\left(a^{2}-b c\\right)} \\\\ & =\\frac{a(b-c)^{2}+(b+c)\\left(a^{2}-b c\\right)}{2\\left(a^{2}-b c\\right)} \\end{aligned} $$ Expanding and subtracting gives $$ p-r=\\frac{a^{3}-a b c-a b^{2}-a c^{2}+b^{2} c+b c^{2}}{2\\left(a^{2}-b c\\right)}=\\frac{(a+b+c)(a-b)(a-c)}{2\\left(a^{2}-b c\\right)} $$ which is visibly equal to the negation of its conjugate once the factor of $a+b+c$ is deleted. (Actually, one can guess this factorization ahead of time by noting that if $A=B$, then $P=B=R$, so $a-b$ must be a factor; analogously $a-c$ must be as well.)"} +{"year":2017,"label":"2","problem":"Ana and Banana are playing a game. First Ana picks a word, which is defined to be a nonempty sequence of capital English letters. Then Banana picks a nonnegative integer $k$ and challenges Ana to supply a word with exactly $k$ subsequences which are equal to Ana's word. Ana wins if she is able to supply such a word, otherwise she loses. For example, if Ana picks the word \"TST\", and Banana chooses $k=4$, then Ana can supply the word \"TSTST\" which has 4 subsequences which are equal to Ana's word. Which words can Ana pick so that she can win no matter what value of $k$ Banana chooses?","solution":" First we introduce some notation. Define a block of letters to be a maximal contiguous subsequence of consecutive letters. For example, the word $A A B B B C A A A$ has four blocks, namely $A A, B B B, C, A A A$. Throughout the solution, we fix the word $A$ that Ana picks, and introduce the following notation for its $m$ blocks: $$ A=A_{1} A_{2} \\ldots A_{m}=\\underbrace{a_{1} \\ldots a_{1}}_{x_{1}} \\underbrace{a_{2} \\ldots a_{2}}_{x_{2}} \\cdots \\underbrace{a_{m} \\ldots a_{m}}_{x_{m}} $$ A rainbow will be a subsequence equal to Ana's initial word $A$ (meaning Ana seeks words with exactly $k$ rainbows). Finally, for brevity, let $A_{i}=\\underbrace{a_{i} \\ldots a_{i}}_{x_{i}}$, so $A=A_{1} \\ldots A_{m}$. We prove two claims that resolve the problem. Claim - If $x_{i}=1$ for some $i$, then for any $k \\geq 1$, the word $$ W=A_{1} \\ldots A_{i-1} \\underbrace{a_{i} \\ldots a_{i}}_{k} A_{i+1} \\ldots A_{m} $$ obtained by repeating the $i$ th letter $k$ times has exactly $k$ rainbows. Given a rainbow, consider the location of this singleton block in $W$. It cannot occur within the first $\\left|A_{1}\\right|+\\cdots+\\left|A_{i-1}\\right|$ letters, nor can it occur within the final $\\left|A_{i+1}\\right|+\\cdots+\\left|A_{m}\\right|$ letters. So it must appear in the $i$ th block of $W$. That implies that all the other $a_{i}$ 's in the $i$ th block of $W$ must be deleted, as desired. (This last argument is actually nontrivial, and has some substance; many students failed to realize that the upper bound requires care.) Claim - If $x_{i} \\geq 2$ for all $i$, then no word $W$ has exactly two rainbows. Let $W=w_{1} \\ldots w_{n}$ and consider the two rainbows of $W$. Since they are not the same, there must be a block $A_{p}$ of the rainbow, of length $\\ell \\geq 2$, which do not occupy the same locations in $W$. Assume the first rainbow uses $w_{i_{1}}, \\ldots, w_{i_{\\ell}}$ for this block and the second rainbow uses $w_{j_{1}}, \\ldots, w_{j_{\\ell}}$ for this block. Then among the letters $w_{q}$ for $\\min \\left(i_{1}, j_{1}\\right) \\leq q \\leq \\max \\left(i_{\\ell}, j_{\\ell}\\right)$, there must be at least $\\ell+1$ copies of the letter $a_{p}$. Moreover, given a choice of $\\ell$ copies of the letter $a_{p}$ in this range, one can complete the subsequence to a rainbow. So the number of rainbows is at least $\\binom{\\ell+1}{\\ell} \\geq \\ell+1$. Since $\\ell \\geq 2$, this proves $W$ has at least three rainbows. In summary, Ana wins if and only if $x_{i}=1$ for some $i$, since she can duplicate the isolated letter $k$ times; but if $x_{i} \\geq 2$ for all $i$ then Banana only needs to supply $k=2$."} +{"year":2017,"label":"3","problem":"Consider solutions to the equation $$ x^{2}-c x+1=\\frac{f(x)}{g(x)} $$ where $f$ and $g$ are nonzero polynomials with nonnegative real coefficients. For each $c>0$, determine the minimum possible degree of $f$, or show that no such $f, g$ exist.","solution":" Indeed, one simply takes $x=1$ to get $f(1) \/ g(1) \\leq 0$, impossible. For $c<2$, let $c=2 \\cos \\theta$, where $0<\\theta<\\pi$. We claim that $f$ exists and has minimum degree equal to $n$, where $n$ is defined as the smallest integer satisfying $\\sin n \\theta \\leq 0$. In other words $$ n=\\left\\lceil\\frac{\\pi}{\\arccos (c \/ 2)}\\right\\rceil $$ First we show that this is necessary. To see it, write explicitly $$ g(x)=a_{0}+a_{1} x+a_{2} x^{2}+\\cdots+a_{n-2} x^{n-2} $$ with each $a_{i} \\geq 0$, and $a_{n-2} \\neq 0$. Assume that $n$ is such that $\\sin (k \\theta) \\geq 0$ for $k=1, \\ldots, n-1$. Then, we have the following system of inequalities: $$ \\begin{aligned} & a_{1} \\geq 2 \\cos \\theta \\cdot a_{0} \\\\ & a_{0}+a_{2} \\geq 2 \\cos \\theta \\cdot a_{1} \\\\ & a_{1}+a_{3} \\geq 2 \\cos \\theta \\cdot a_{2} \\\\ & \\vdots \\\\ & a_{n-5}+a_{n-3} \\geq 2 \\cos \\theta \\cdot a_{n-4} \\\\ & a_{n-4}+a_{n-2} \\geq 2 \\cos \\theta \\cdot a_{n-3} \\\\ & a_{n-3} \\geq 2 \\cos \\theta \\cdot a_{n-2} . \\end{aligned} $$ Now, multiply the first equation by $\\sin \\theta$, the second equation by $\\sin 2 \\theta$, et cetera, up to $\\sin ((n-1) \\theta)$. This choice of weights is selected since we have $$ \\sin (k \\theta)+\\sin ((k+2) \\theta)=2 \\sin ((k+1) \\theta) \\cos \\theta $$ so that summing the entire expression cancels nearly all terms and leaves only $$ \\sin ((n-2) \\theta) a_{n-2} \\geq \\sin ((n-1) \\theta) \\cdot 2 \\cos \\theta \\cdot a_{n-2} $$ and so by dividing by $a_{n-2}$ and using the same identity gives us $\\sin (n \\theta) \\leq 0$, as claimed. This bound is best possible, because the example $$ a_{k}=\\sin ((k+1) \\theta) \\geq 0 $$ makes all inequalities above sharp, hence giving a working pair $(f, g)$. Remark. Calvin Deng points out that a cleaner proof of the lower bound is to take $\\alpha=\\cos \\theta+i \\sin \\theta$. Then $f(\\alpha)=0$, but by condition the imaginary part of $f(\\alpha)$ is apparently strictly positive, contradiction. Remark. Guessing that $c<2$ works at all (and realizing $c \\geq 2$ fails) is the first part of the problem. - Calvin Deng points out that it's possible to guess the answer from small cases: For $c \\leq 1$ we have $n=3$, tight at $\\frac{x^{3}+1}{x+1}=x^{2}-x+1$, and essentially the \"sharpest $n=3$ example\". A similar example exists at $n=4$ with $\\frac{x^{4}+1}{x^{2}+\\sqrt{2} x+1}=x^{2}-\\sqrt{2} x+1$ by the Sophie-Germain identity. In general, one can do long division to extract an optimal value of $c$ for any given $n$, although $c$ will be the root of some polynomial. The thresholds $c \\leq 1$ for $n=3, c \\leq \\sqrt{2}$ for $n=4, c \\leq \\frac{1+\\sqrt{5}}{2}$ for $n=5$, and $c \\leq 2$ for $n<\\infty$ suggest the unusual form of the answer via trigonometry. - One may imagine trying to construct a polynomial recursively \/ greedily by making all inequalities above hold (again the \"sharpest situation\" in which $f$ has few coefficients). If one sets $c=2 t$, then we have $$ a_{0}=1, \\quad a_{1}=2 t, \\quad a_{2}=4 t^{2}-1, \\quad a_{3}=8 t^{3}-4 t, \\quad \\ldots $$ which are the Chebyshev polynomials of the second type. This means that trigonometry is essentially mandatory. (One may also run into this when by using standard linear recursion techniques, and noting that the characteristic polynomial has two conjugate complex roots.) Remark. Mitchell Lee notes that an IMO longlist problem from 1997 shows that if $P(x)$ is any polynomial satisfying $P(x)>0$ for $x>0$, then $(x+1)^{n} P(x)$ has nonnegative coefficients for large enough $n$. This show that $f$ and $g$ at least exist for $c \\leq 2$, but provides no way of finding the best possible $\\operatorname{deg} f$. Meghal Gupta also points out that showing $f$ and $g$ exist is possible in the following way: $$ \\left(x^{2}-1.99 x+1\\right)\\left(x^{2}+1.99 x+1\\right)=\\left(x^{4}-1.9601 x^{2}+1\\right) $$ and so on, repeatedly multiplying by the \"conjugate\" until all coefficients become positive. To my best knowledge, this also does not give any way of actually minimizing $\\operatorname{deg} f$, although Ankan Bhattacharya points out that this construction is actually optimal in the case where $n$ is a power of 2 . Remark. It's pointed out that Matematicheskoe Prosveshchenie, issue 1, 1997, page 194 contains a nearly analogous result, available at https:\/\/mccme.ru\/free-books\/matpros\/ pdf\/mp-01.pdf with solutions presented in https:\/\/mccme.ru\/free-books\/matpros\/pdf\/ mp-05.pdf, pages 221-223; and https:\/\/mccme.ru\/free-books\/matpros\/pdf\/mp-10.pdf, page 274."} +{"year":2017,"label":"4","problem":"Find all nonnegative integer solutions to $$ 2^{a}+3^{b}+5^{c}=n! $$","solution":"$$ 2^{a}+3^{b}+5^{c}=n! $$ $$ \\begin{aligned} & 2^{2}+3^{0}+5^{0}=3! \\\\ & 2^{1}+3^{1}+5^{0}=3! \\\\ & 2^{4}+3^{1}+5^{1}=4! \\end{aligned} $$ A tricky way to do this is to take modulo 120 , since $$ \\begin{aligned} 2^{a} \\quad(\\bmod 120) & \\in\\{1,2,4,8,16,32,64\\} \\\\ 3^{b} \\quad(\\bmod 120) & \\in\\{1,3,9,27,81\\} \\\\ 5^{c} \\quad(\\bmod 120) & \\in\\{1,5,25\\} \\end{aligned} $$ and by inspection one notes that no three elements have vanishing sum modulo 120 . - $a=1$ : since $3^{b}+5^{c} \\equiv 6(\\bmod 8)$, we find $b$ even and $c$ odd (hence $\\left.c \\neq 0\\right)$. Now looking modulo 5 gives that $3^{b}+5^{c} \\equiv 3(\\bmod 5)$, - $a=2$ : From $3^{b}+5^{c} \\equiv 4(\\bmod 8)$, we find $b$ is odd and $c$ is even. Now looking modulo 5 gives a contradiction, even if $c=0$, since $3^{b} \\in\\{2,3(\\bmod 5)\\}$ but $3^{b}+5^{c} \\equiv 1(\\bmod 5)$. Henceforth assume $a \\geq 3$. Next, by taking modulo 8 we have $3^{b}+5^{c} \\equiv 0(\\bmod 8)$, which forces both $b$ and $c$ to be odd (in particular, $b, c>0$ ). We now have $$ \\begin{aligned} & 2^{a}+5^{c} \\equiv 0 \\quad(\\bmod 3) \\\\ & 2^{a}+3^{b} \\equiv 0 \\quad(\\bmod 5) . \\end{aligned} $$ The first equation implies $a$ is even, but the second equation requires $a$ to be odd, contradiction. Hence no solutions with $n \\geq 5$."} +{"year":2017,"label":"5","problem":"Let $A B C$ be a triangle with incenter $I$. Let $D$ be a point on side $B C$ and let $\\omega_{B}$ and $\\omega_{C}$ be the incircles of $\\triangle A B D$ and $\\triangle A C D$, respectively. Suppose that $\\omega_{B}$ and $\\omega_{C}$ are tangent to segment $B C$ at points $E$ and $F$, respectively. Let $P$ be the intersection of segment $A D$ with the line joining the centers of $\\omega_{B}$ and $\\omega_{C}$. Let $X$ be the intersection point of lines $B I$ and $C P$ and let $Y$ be the intersection point of lines $C I$ and $B P$. Prove that lines $E X$ and $F Y$ meet on the incircle of $\\triangle A B C$.","solution":" \u300e First solution (homothety). Let $Z$ be the diametrically opposite point on the incircle. We claim this is the desired intersection. ![](https:\/\/cdn.mathpix.com\/cropped\/2024_11_19_b5f71e11be2d66f7b920g-10.jpg?height=586&width=809&top_left_y=1055&top_left_x=629) Note that: - $P$ is the insimilicenter of $\\omega_{B}$ and $\\omega_{C}$ - $C$ is the exsimilicenter of $\\omega$ and $\\omega_{C}$. Thus by Monge theorem, the insimilicenter of $\\omega_{B}$ and $\\omega$ lies on line $C P$. This insimilicenter should also lie on the line joining the centers of $\\omega$ and $\\omega_{B}$, which is $\\overline{B I}$, hence it coincides with the point $X$. So $X \\in \\overline{E Z}$ as desired."} +{"year":2017,"label":"5","problem":"Let $A B C$ be a triangle with incenter $I$. Let $D$ be a point on side $B C$ and let $\\omega_{B}$ and $\\omega_{C}$ be the incircles of $\\triangle A B D$ and $\\triangle A C D$, respectively. Suppose that $\\omega_{B}$ and $\\omega_{C}$ are tangent to segment $B C$ at points $E$ and $F$, respectively. Let $P$ be the intersection of segment $A D$ with the line joining the centers of $\\omega_{B}$ and $\\omega_{C}$. Let $X$ be the intersection point of lines $B I$ and $C P$ and let $Y$ be the intersection point of lines $C I$ and $B P$. Prove that lines $E X$ and $F Y$ meet on the incircle of $\\triangle A B C$.","solution":" \u3010 Second solution (harmonic). Let $T=\\overline{I_{B} I_{C}} \\cap \\overline{B C}$, and $W$ the foot from $I$ to $\\overline{B C}$. Define $Z=\\overline{F Y} \\cap \\overline{I W}$. Because $\\angle I_{B} D I_{C}=90^{\\circ}$, we have $$ -1=\\left(I_{B} I_{C} ; P T\\right) \\stackrel{B}{\\stackrel{B}{2}}\\left(I I_{C} ; Y C\\right) \\stackrel{F}{=}(I \\infty ; Z W) $$ So $I$ is the midpoint of $\\overline{Z W}$ as desired."} +{"year":2017,"label":"5","problem":"Let $A B C$ be a triangle with incenter $I$. Let $D$ be a point on side $B C$ and let $\\omega_{B}$ and $\\omega_{C}$ be the incircles of $\\triangle A B D$ and $\\triangle A C D$, respectively. Suppose that $\\omega_{B}$ and $\\omega_{C}$ are tangent to segment $B C$ at points $E$ and $F$, respectively. Let $P$ be the intersection of segment $A D$ with the line joining the centers of $\\omega_{B}$ and $\\omega_{C}$. Let $X$ be the intersection point of lines $B I$ and $C P$ and let $Y$ be the intersection point of lines $C I$ and $B P$. Prove that lines $E X$ and $F Y$ meet on the incircle of $\\triangle A B C$.","solution":" \u3010 Third solution (outline, barycentric, Andrew Gu). Let $A D=t, B D=x, C D=y$, so $a=x+y$ and by Stewart's theorem we have $$ (x+y)\\left(x y+t^{2}\\right)=b^{2} x+c^{2} y $$ We then have $D=(0: y: x)$ and so $$ \\overline{A I_{B}} \\cap \\overline{B C}=\\left(0: y+\\frac{t x}{c+t}: \\frac{c x}{c+t}\\right) $$ hence intersection with $B I$ gives $$ I_{B}=(a x: c y+a t: c x) $$ Similarly, $$ I_{C}=(a y: b y: b x+a t) $$ Then, we can compute $$ P=(2 a x y: y(a t+b x+c y): x(a t+b x+c y)) $$ since $P \\in \\overline{I_{B} I_{C}}$, and clearly $P \\in \\overline{A D}$. Intersection now gives $$ \\begin{aligned} & X=(2 a x: a t+b x+c y: 2 c x) \\\\ & Y=(2 a y: 2 b y: a t+b x+c y) \\end{aligned} $$ Finally, we have $B E=\\frac{1}{2}(c+x-t)$, and similarly for $C F$. Now if we reflect $D=$ $\\left(0, \\frac{s-c}{a}, \\frac{s-b}{a}\\right)$ over $I=\\left(\\frac{a}{2 s}, \\frac{b}{2 s}, \\frac{c}{2 s}\\right)$, we get the antipode $$ Q:=\\left(4 a^{2}:-a^{2}+2 a b-b^{2}+c^{2}:-a^{2}+2 a c-c^{2}+b^{2}\\right) . $$ We may then check $Q$ lies on each of lines $E X$ and $F Y$ (by checking $\\operatorname{det}(Q, E, X)=0$ using the equation (1))."} +{"year":2017,"label":"6","problem":"A sequence of positive integers $\\left(a_{n}\\right)_{n \\geq 1}$ is of Fibonacci type if it satisfies the recursive relation $a_{n+2}=a_{n+1}+a_{n}$ for all $n \\geq 1$. Is it possible to partition the set of positive integers into an infinite number of Fibonacci type sequences?","solution":" \u3010 First solution (Kevin Sun). We are going to appeal to the so-called Zeckendorf theorem: Theorem (Zeckendorf) Every positive integer can be uniquely expressed as the sum of nonconsecutive Fibonacci numbers. This means every positive integer has a Zeckendorf (\"Fibonacci-binary\") representation where we put 1 in the $i$ th digit from the right if $F_{i+1}$ is used. The idea is then to take the following so-called Wythoff array: - Row 1: 1, 2, 3, 5, ... - Row 101: $1+3,2+5,3+8, \\ldots$ - Row 1001: $1+5,2+8,3+13, \\ldots$ - Row 10001: $1+8,2+13,3+21, \\ldots$ - Row 10101: $1+3+8,2+5+13,3+8+21, \\ldots$ - . . .et cetera. More concretely, the array has the following rows to start: | 1 | 2 | 3 | 5 | 8 | 13 | 21 | $\\ldots$ | | :---: | :---: | :---: | :---: | :---: | :---: | :---: | :---: | | 4 | 7 | 11 | 18 | 29 | 47 | 76 | $\\ldots$ | | 6 | 10 | 16 | 26 | 42 | 68 | 110 | $\\ldots$ | | 9 | 15 | 24 | 39 | 63 | 102 | 165 | $\\ldots$ | | 12 | 20 | 32 | 52 | 84 | 136 | 220 | $\\ldots$ | | 14 | 23 | 37 | 60 | 97 | 157 | 254 | $\\ldots$ | | 17 | 28 | 45 | 73 | 118 | 191 | 309 | $\\ldots$ | | $\\vdots$ | $\\vdots$ | $\\vdots$ | $\\vdots$ | $\\vdots$ | $\\vdots$ | $\\vdots$ | $\\ddots$ | Here are the full details. $$ n-F_{k}F_{k-1}+F_{k-3}+F_{k-5}+\\cdots $$ This shows, by a simple inductive argument, that such a representation exists and unique. We write $n={\\bar{a} k \\ldots a_{1}}_{\\text {Fib }}$ for the Zeckendorf representation as we described (where $a_{i}=1$ if $F_{i+1}$ is used). Now for each ${\\bar{a} k \\ldots a_{1}}^{\\text {Fib }}$ with $a_{1}=1$, consider the sequence $$ {\\overline{a_{k} \\ldots a_{1}}}_{\\mathrm{Fib}},{\\overline{a_{k} \\ldots a_{1} 0}}_{\\mathrm{Fib}},{\\overline{a_{k} \\ldots a_{1} 00}}_{\\mathrm{Fib}}, \\ldots $$ These sequences are Fibonacci-type by definition, and partition the positive integers since each positive integer has exactly one Fibonacci base representation."} +{"year":2017,"label":"6","problem":"A sequence of positive integers $\\left(a_{n}\\right)_{n \\geq 1}$ is of Fibonacci type if it satisfies the recursive relation $a_{n+2}=a_{n+1}+a_{n}$ for all $n \\geq 1$. Is it possible to partition the set of positive integers into an infinite number of Fibonacci type sequences?","solution":" \\I Second solution. Call an infinite set of integers $S$ sandwiched if there exist increasing sequences $\\left\\{a_{i}\\right\\}_{i=0}^{\\infty},\\left\\{b_{i}\\right\\}_{i=0}^{\\infty}$ such that the following are true: - $a_{i}+a_{i+1}=a_{i+2}$ and $b_{i}+b_{i+1}=b_{i+2}$. - The intervals $\\left[a_{i}+1, b_{i}-1\\right]$ are disjoint and are nondecreasing in length. - $S=\\bigcup_{i=0}^{\\infty}\\left[a_{i}+1, b_{i}-1\\right]$. We claim that if $S$ is any nonempty sandwiched set, then $S$ can be partitioned into a Fibonacci-type sequence (involving the smallest element of $S$ ) and two smaller sandwiched sets. If this claim is proven, then we can start with $\\mathbb{N} \\backslash\\{1,2,3,5, \\ldots\\}$, which is a sandwiched set, and repeatedly perform this partition, which will eventually sort each natural number into a Fibonacci-type sequence. Let $S$ be a sandwiched set given by $\\left\\{a_{i}\\right\\}_{i=0}^{\\infty},\\left\\{b_{i}\\right\\}_{i=0}^{\\infty}$, so the smallest element in $S$ is $x=a_{0}+1$. Note that $y=a_{1}+1$ is also in $S$ and $x1$. Then $$ a \\neq b \\Longrightarrow\\left\\lfloor\\varphi a+\\frac{1}{2}\\right\\rfloor \\neq\\left\\lfloor\\varphi b+\\frac{1}{2}\\right\\rfloor, $$ and since the first term of each sequence is chosen to not overlap with any previous sequences, these sequences are disjoint. Remark. Ankan Bhattacharya points out that the same sequence essentially appears in IMO 1993, Problem 5 - in other words, a strictly increasing function $f: \\mathbb{Z}_{>0} \\rightarrow \\mathbb{Z}_{>0}$ with $f(1)=2$, and $f(f(n))=f(n)+n$. Nikolai Beluhov sent us an older reference from March 1977, where Martin Gardner wrote in his column about Wythoff's Nim. The relevant excerpt goes: \"Imagine that we go through the infinite sequence of safe pairs (in the manner of Eratosthenes' sieve for sifting out primes) and cross out the infinite set of all safe pairs that are pairs in the Fibonacci sequence. The smallest pair that is not crossed out is $4 \/ 7$. We can now cross out a second infinite set of safe pairs, starting with $4 \/ 7$, that are pairs in the Lucas sequence. An infinite number of safe pairs, of which the lowest is now $6 \/ 10$, remain. This pair too begins another infinite Fibonacci sequence, all of whose pairs are safe. The process continues forever. Robert Silber, a mathematician at North Carolina State University, calls a safe pair \"primitive\" if it is the first safe pair that generates a Fibonacci sequence.\" The relevant article by Robert Silber is A Fibonacci Property of Wythoff Pairs, from The Fibonacci Quarterly 11\/1976."} +{"year":2017,"label":"6","problem":"A sequence of positive integers $\\left(a_{n}\\right)_{n \\geq 1}$ is of Fibonacci type if it satisfies the recursive relation $a_{n+2}=a_{n+1}+a_{n}$ for all $n \\geq 1$. Is it possible to partition the set of positive integers into an infinite number of Fibonacci type sequences?","solution":" I Fourth solution (Mark Sellke). For later reference let $$ f_{1}=0, f_{2}=1, f_{3}=1, \\ldots $$ denote the ordinary Fibonacci numbers. We will denote the Fibonacci-like sequences by $F^{i}$ and the elements with subscripts; hence $F_{1}^{2}$ is the first element of the second sequence. Our construction amounts to just iteratively add new sequences; hence the following claim is the whole problem. ## Lemma For any disjoint collection of Fibonacci-like sequences $F^{1}, \\ldots, F^{k}$ and any integer $m$ contained in none of them, there is a new Fibonacci-like sequence $F^{k+1}$ beginning with $F_{1}^{k+1}=m$ which is disjoint from the previous sequences. Observe first that for each sequence $F^{j}$ there is $c^{j} \\in \\mathbb{R}^{n}$ such that $$ F_{n}^{j}=c^{j} \\phi^{n}+o(1) $$ where $$ \\phi=\\frac{1+\\sqrt{5}}{2} $$ Collapse the group $\\left(\\mathbb{R}^{+}, \\times\\right)$into the half-open interval $J=\\{x \\mid 1 \\leq x<\\phi\\}$ by defining $T(x)=y$ for the unique $y \\in J$ with $x=y \\phi^{n}$ for some integer $n$. Fix an interval $I=[a, b] \\subseteq[1.2,1.3]$ (the last condition is to avoid wrap-around issues) which contains none of the $c^{j}$, and take $\\varepsilon<0.001$ to be small enough that in fact each $c^{j}$ has distance at least $10 \\varepsilon$ from $I$; this means any $c_{j}$ and element of $I$ differ by at least a $(1+10 \\varepsilon)$ factor. The idea will be to take $F_{1}^{k+1}=m$ and $F_{2}^{k+1}$ to be a large such that the induced values of $F_{j}^{k+1}$ grow like $k \\phi^{j}$ for $j \\in T^{-1}(I)$, so that $F_{n}^{k+1}$ is separated from the $c^{j}$ after applying $T$. What's left to check is the convergence. Now let $$ c=\\lim _{n \\rightarrow \\infty} \\frac{f_{n}}{\\phi^{n}} $$ and take $M$ large enough that for $n>M$ we have $$ \\left|\\frac{f_{n}}{c \\phi^{n}}-1\\right|<\\varepsilon $$ Now $\\frac{T^{-1}(I)}{c}$ contains arbitrarily large integers, so there are infinitely many $N$ with $c N \\in T^{-1}(I)$ with $N>\\frac{10 m}{\\varepsilon}$. We claim that for any such $N$, the sequence $F^{(N)}$ defined by $$ F_{1}^{(N)}=m, F_{2}^{(N)}=N $$ will be very multiplicatively similar to the normal Fibonacci numbers up to rescaling; indeed for $j=2, j=3$ we have $\\frac{F_{2}^{(N)}}{f_{2}}=N, \\frac{F_{3}^{(N)}}{f_{3}}=N+m$ and so by induction we will have $$ \\frac{F_{j}^{(N)}}{f_{j}} \\in[N, N+m] \\subseteq[N, N(1+\\varepsilon)] $$ for $j \\geq 2$. Therefore, up to small multiplicative errors, we have $$ F_{j}^{(N)} \\approx N f_{j} \\approx c N \\phi^{j} $$ From this we see that for $j>M$ we have $$ T\\left(F_{j}^{(N)}\\right) \\in T(c N) \\cdot[1-2 \\varepsilon, 1+2 \\varepsilon] $$ In particular, since $T(c N) \\in I$ and $I$ is separated from each $c_{j}$ by a factor of $(1+10 \\varepsilon)$, we get that $F_{j}^{(N)}$ is not in any of $F^{1}, F^{2}, \\ldots, F^{k}$. Finishing is easy, since we now have a uniform estimate on how many terms we need to check for a new element before the exponential growth takes over. We will just use pigeonhole to argue that there are few possible collisions among those early terms, so we can easily pick a value of $N$ which avoids them all. We write it out below. For large $L$, the set $$ S_{L}=\\left(I \\cdot \\phi^{L}\\right) \\cap \\mathbb{Z} $$ contains at least $k_{I} \\phi^{L}$ elements. As $N$ ranges over $S_{L}$, for each fixed $j$, the value of $F_{j}^{(N)}$ varies by at most a factor of 1.1 because we imposed $I \\subseteq[1.2,1.3]$ and so this is true for the first two terms, hence for all subsequent terms by induction. Now suppose $L$ is very large, and consider a fixed pair $(i, j)$ with $i \\leq k$ and $j \\leq M$. We claim there is at most 1 possible value $k$ such that the term $F_{k}^{i}$ could equal $F_{j}^{(N)}$ for some $N \\in S_{L}$; indeed, the terms of $F^{i}$ are growing at exponential rate with factor $\\phi>1.1$, so at most one will be in a given interval of multiplicative width at most 1.1. Hence, of these $k_{I} \\phi^{L}$ values of $N$, at most $k M$ could cause problems, one for each pair $(i, j)$. However by monotonicity of $F_{j}^{(N)}$ in $N$, at most 1 value of $N$ causes a collision for each pair $(i, j)$. Hence for large $L$ so that $k_{I} \\phi^{L}>10 k M$ we can find a suitable $N \\in S_{L}$ by pigeonhole and the sequence $F^{(N)}$ defined by $(m, N, N+m, \\ldots)$ works."} diff --git a/USA_TST_TST/segmented/en-sols-TSTST-2018.jsonl b/USA_TST_TST/segmented/en-sols-TSTST-2018.jsonl new file mode 100644 index 0000000000000000000000000000000000000000..1f221ca585cfa144164a3fd39b40618f9e6e8ee1 --- /dev/null +++ b/USA_TST_TST/segmented/en-sols-TSTST-2018.jsonl @@ -0,0 +1,14 @@ +{"year":2018,"label":"1","problem":"As usual, let $\\mathbb{Z}[x]$ denote the set of single-variable polynomials in $x$ with integer coefficients. Find all functions $\\theta: \\mathbb{Z}[x] \\rightarrow \\mathbb{Z}$ such that for any polynomials $p, q \\in$ $\\mathbb{Z}[x]$, - $\\theta(p+1)=\\theta(p)+1$, and - if $\\theta(p) \\neq 0$ then $\\theta(p)$ divides $\\theta(p \\cdot q)$.","solution":" The answer is $\\theta: p \\mapsto p(c)$, for each choice of $c \\in \\mathbb{Z}$. Obviously these work, so we prove these are the only ones. In what follows, $x \\in \\mathbb{Z}[x]$ is the identity polynomial, and $c=\\theta(x)$. \u3010 First solution (Merlijn Staps). Consider an integer $n \\neq c$. Because $x-n \\mid p(x)-p(n)$, we have $$ \\theta(x-n)|\\theta(p(x)-p(n)) \\Longrightarrow c-n| \\theta(p(x))-p(n) . $$ On the other hand, $c-n \\mid p(c)-p(n)$. Combining the previous two gives $c-n \\mid$ $\\theta(p(x))-p(c)$, and by letting $n$ large we conclude $\\theta(p(x))-p(c)=0$, so $\\theta(p(x))=p(c)$."} +{"year":2018,"label":"1","problem":"As usual, let $\\mathbb{Z}[x]$ denote the set of single-variable polynomials in $x$ with integer coefficients. Find all functions $\\theta: \\mathbb{Z}[x] \\rightarrow \\mathbb{Z}$ such that for any polynomials $p, q \\in$ $\\mathbb{Z}[x]$, - $\\theta(p+1)=\\theta(p)+1$, and - if $\\theta(p) \\neq 0$ then $\\theta(p)$ divides $\\theta(p \\cdot q)$.","solution":" The answer is $\\theta: p \\mapsto p(c)$, for each choice of $c \\in \\mathbb{Z}$. Obviously these work, so we prove these are the only ones. In what follows, $x \\in \\mathbb{Z}[x]$ is the identity polynomial, and $c=\\theta(x)$. II Second solution. First, we settle the case $\\operatorname{deg} p=0$. In that case, from the second property, $\\theta(m)=m+\\theta(0)$ for every integer $m \\in \\mathbb{Z}$ (viewed as a constant polynomial). Thus $m+\\theta(0) \\mid 2 m+\\theta(0)$, hence $m+\\theta(0) \\mid-\\theta(0)$, so $\\theta(0)=0$ by taking $m$ large. Thus $\\theta(m)=m$ for $m \\in \\mathbb{Z}$. Next, we address the case of $\\operatorname{deg} p=1$. We know $\\theta(x+b)=c+b$ for $b \\in \\mathbb{Z}$. Now for each particular $a \\in \\mathbb{Z}$, we have $$ c+k|\\theta(x+k)| \\theta(a x+a k)=\\theta(a x)+a k \\Longrightarrow c+k \\mid \\theta(a x)-a c . $$ for any $k \\neq-c$. Since this is true for large enough $k$, we conclude $\\theta(a x)=a c$. Thus $\\theta(a x+b)=a c+b$. We now proceed by induction on $\\operatorname{deg} p$. Fix a polynomial $p$ and assume it's true for all $p$ of smaller degree. Choose a large integer $n$ (to be determined later) for which $p(n) \\neq p(c)$. We then have $$ \\left.\\frac{p(c)-p(n)}{c-n}=\\theta\\left(\\frac{p-p(n)}{x-n}\\right) \\right\\rvert\\, \\theta(p-p(n))=\\theta(p)-p(n) $$ Subtracting off $c-n$ times the left-hand side gives $$ \\left.\\frac{p(c)-p(n)}{c-n} \\right\\rvert\\, \\theta(p)-p(c) $$ The left-hand side can be made arbitrarily large by letting $n \\rightarrow \\infty$, since $\\operatorname{deg} p \\geq 2$. Thus $\\theta(p)=p(c)$, concluding the proof. \\ Authorship comments. I will tell you a story about the creation of this problem. Yang Liu and I were looking over the drafts of December and January TST in October 2017, and both of us had the impression that the test was too difficult. This sparked a non-serious suggestion that we should try to come up with a problem now that would be easy enough to use. While we ended up just joking about changing the TST, we did get this problem out of it. Our idea was to come up with a functional equation that was different from the usual fare: at first we tried $\\mathbb{Z}[x] \\rightarrow \\mathbb{Z}[x]$, but then I suggested the idea of using $\\mathbb{Z}[x] \\rightarrow \\mathbb{Z}$, with the answer being the \"evaluation\" map. Well, what properties does that satisfy? One answer was $a-b \\mid p(a)-p(b)$; this didn't immediately lead to anything, but eventually we hit on the form of the problem above off this idea. At first we didn't require $\\theta(p) \\neq 0$ in the bullet, but without the condition the problem was too easy, since 0 divides only itself; and so the condition was added and we got the functional equation. I proposed the problem to USAMO 2018, but it was rejected (unsurprisingly; I think the problem may be too abstract for novice contestants). Instead it was used for TSTST, which I thought fit better."} +{"year":2018,"label":"2","problem":"In the nation of Onewaynia, certain pairs of cities are connected by one-way roads. Every road connects exactly two cities (roads are allowed to cross each other, e.g., via bridges), and each pair of cities has at most one road between them. Moreover, every city has exactly two roads leaving it and exactly two roads entering it. We wish to close half the roads of Onewaynia in such a way that every city has exactly one road leaving it and exactly one road entering it. Show that the number of ways to do so is a power of 2 greater than 1 (i.e. of the form $2^{n}$ for some integer $n \\geq 1$ ).","solution":" In the language of graph theory, we have a simple digraph $G$ which is 2 -regular and we seek the number of sub-digraphs which are 1-regular. We now present two solution paths. \u300e First solution, combinatorial. We construct a simple undirected bipartite graph $\\Gamma$ as follows: - the vertex set consists of two copies of $V(G)$, say $V_{\\text {out }}$ and $V_{\\text {in }}$; and - for $v \\in V_{\\text {out }}$ and $w \\in V_{\\text {in }}$ we have an undirected edge $v w \\in E(\\Gamma)$ if and only if the directed edge $v \\rightarrow w$ is in $G$. Moreover, the desired sub-digraphs of $H$ correspond exactly to perfect matchings of $\\Gamma$. However the graph $\\Gamma$ is 2 -regular and hence consists of several disjoint (simple) cycles of even length. If there are $n$ such cycles, the number of perfect matchings is $2^{n}$, as desired. Remark. The construction of $\\Gamma$ is not as magical as it may first seem. Suppose we pick a road $v_{1} \\rightarrow v_{2}$ to use. Then, the other road $v_{3} \\rightarrow v_{2}$ is certainly not used; hence some other road $v_{3} \\rightarrow v_{4}$ must be used, etc. We thus get a cycle of forced decisions until we eventually return to the vertex $v_{1}$. These cycles in the original graph $G$ (where the arrows alternate directions) correspond to the cycles we found in $\\Gamma$. It's merely that phrasing the solution in terms of $\\Gamma$ makes it cleaner in a linguistic sense, but not really in a mathematical sense."} +{"year":2018,"label":"2","problem":"In the nation of Onewaynia, certain pairs of cities are connected by one-way roads. Every road connects exactly two cities (roads are allowed to cross each other, e.g., via bridges), and each pair of cities has at most one road between them. Moreover, every city has exactly two roads leaving it and exactly two roads entering it. We wish to close half the roads of Onewaynia in such a way that every city has exactly one road leaving it and exactly one road entering it. Show that the number of ways to do so is a power of 2 greater than 1 (i.e. of the form $2^{n}$ for some integer $n \\geq 1$ ).","solution":" In the language of graph theory, we have a simple digraph $G$ which is 2 -regular and we seek the number of sub-digraphs which are 1-regular. We now present two solution paths. \u3010 Second solution by linear algebra over $\\mathbb{F}_{2}$ (Brian Lawrence). This is actually not that different from the first solution. For each edge $e$, we create an indicator variable $x_{e}$. We then require for each vertex $v$ that: - If $e_{1}$ and $e_{2}$ are the two edges leaving $v$, then we require $x_{e_{1}}+x_{e_{2}} \\equiv 1(\\bmod 2)$. - If $e_{3}$ and $e_{4}$ are the two edges entering $v$, then we require $x_{e_{3}}+x_{e_{4}} \\equiv 1(\\bmod 2)$. We thus get a large system of equations. Moreover, the solutions come in natural pairs $\\vec{x}$ and $\\vec{x}+\\overrightarrow{1}$ and therefore the number of solutions is either zero, or a power of two. So we just have to prove there is at least one solution. For linear algebra reasons, there can only be zero solutions if some nontrivial linear combination of the equations gives the sum $0 \\equiv 1$. So suppose we added up some subset $S$ of the equations for which every variable appeared on the left-hand side an even number of times. Then every variable that did appear appeared exactly twice; and accordingly we see that the edges corresponding to these variables form one or more even cycles as in the previous solution. Of course, this means $|S|$ is even, so we really have $0 \\equiv 0(\\bmod 2)$ as needed. Remark. The author's original proposal contained a second part asking to show that it was not always possible for the resulting $H$ to be connected, even if $G$ was strongly connected. This problem is related to IMO Shortlist 2002 C6, which gives an example of a strongly connected graph which does have a full directed Hamiltonian cycle."} +{"year":2018,"label":"3","problem":"Let $A B C$ be an acute triangle with incenter $I$, circumcenter $O$, and circumcircle $\\Gamma$. Let $M$ be the midpoint of $\\overline{A B}$. Ray $A I$ meets $\\overline{B C}$ at $D$. Denote by $\\omega$ and $\\gamma$ the circumcircles of $\\triangle B I C$ and $\\triangle B A D$, respectively. Line $M O$ meets $\\omega$ at $X$ and $Y$, while line $C O$ meets $\\omega$ at $C$ and $Q$. Assume that $Q$ lies inside $\\triangle A B C$ and $\\angle A Q M=\\angle A C B$. Consider the tangents to $\\omega$ at $X$ and $Y$ and the tangents to $\\gamma$ at $A$ and $D$. Given that $\\angle B A C \\neq 60^{\\circ}$, prove that these four lines are concurrent on $\\Gamma$.","solution":" Henceforth assume $\\angle A \\neq 60^{\\circ}$; we prove the concurrence. Let $L$ denote the center of $\\omega$, which is the midpoint of minor arc $B C$. Claim - Let $K$ be the point on $\\omega$ such that $\\overline{K L} \\| \\overline{A B}$ and $\\overline{K C} \\| \\overline{A L}$. Then $\\overline{K A}$ is tangent to $\\gamma$, and we may put $$ x=K A=L B=L C=L X=L Y=K X=K Y . $$ Moreover, $\\overline{K A}$ is tangent to $\\gamma$ as well since $$ \\measuredangle K A D=\\measuredangle K A L=\\measuredangle K A C+\\measuredangle C A L=\\measuredangle K B C+\\measuredangle A B K=\\measuredangle A B C . $$ ![](https:\/\/cdn.mathpix.com\/cropped\/2024_11_19_a0f01f2a44c8136e2158g-08.jpg?height=918&width=914&top_left_y=1551&top_left_x=571) Up to now we have not used the existence of $Q$; we henceforth do so. Note that $Q \\neq O$, since $\\angle A \\neq 60^{\\circ} \\Longrightarrow O \\notin \\omega$. Moreover, we have $\\angle A O M=\\angle A C B$ too. Since $O$ and $Q$ both lie inside $\\triangle A B C$, this implies that $A, M, O, Q$ are concyclic. As $Q \\neq O$ we conclude $\\angle C Q A=90^{\\circ}$. The main claim is now: Claim - Assuming $Q$ exists, the rhombus $L X K Y$ is a square. In particular, $\\overline{K X}$ and $\\overline{K Y}$ are tangent to $\\omega$. First proof of Claim, communicated by Milan Haiman. Observe that $\\triangle Q L C \\sim \\triangle L O C$ since both triangles are isosceles and share a base angle. Hence, $C L^{2}=C O \\cdot C Q$. Let $N$ be the midpoint of $\\overline{A C}$, which lies on $(A M O Q)$. Then, $$ x^{2}=C L^{2}=C O \\cdot C Q=C N \\cdot C A=\\frac{1}{2} C A^{2}=\\frac{1}{2} L K^{2} $$ where we have also used the fact $A Q O N$ is cyclic. Thus $L K=\\sqrt{2} x$ and so the rhombus $L X K Y$ is actually a square."} +{"year":2018,"label":"3","problem":"Let $A B C$ be an acute triangle with incenter $I$, circumcenter $O$, and circumcircle $\\Gamma$. Let $M$ be the midpoint of $\\overline{A B}$. Ray $A I$ meets $\\overline{B C}$ at $D$. Denote by $\\omega$ and $\\gamma$ the circumcircles of $\\triangle B I C$ and $\\triangle B A D$, respectively. Line $M O$ meets $\\omega$ at $X$ and $Y$, while line $C O$ meets $\\omega$ at $C$ and $Q$. Assume that $Q$ lies inside $\\triangle A B C$ and $\\angle A Q M=\\angle A C B$. Consider the tangents to $\\omega$ at $X$ and $Y$ and the tangents to $\\gamma$ at $A$ and $D$. Given that $\\angle B A C \\neq 60^{\\circ}$, prove that these four lines are concurrent on $\\Gamma$.","solution":" Henceforth assume $\\angle A \\neq 60^{\\circ}$; we prove the concurrence. Let $L$ denote the center of $\\omega$, which is the midpoint of minor arc $B C$. Claim - Let $K$ be the point on $\\omega$ such that $\\overline{K L} \\| \\overline{A B}$ and $\\overline{K C} \\| \\overline{A L}$. Then $\\overline{K A}$ is tangent to $\\gamma$, and we may put $$ x=K A=L B=L C=L X=L Y=K X=K Y . $$ Moreover, $\\overline{K A}$ is tangent to $\\gamma$ as well since $$ \\measuredangle K A D=\\measuredangle K A L=\\measuredangle K A C+\\measuredangle C A L=\\measuredangle K B C+\\measuredangle A B K=\\measuredangle A B C . $$ ![](https:\/\/cdn.mathpix.com\/cropped\/2024_11_19_a0f01f2a44c8136e2158g-08.jpg?height=918&width=914&top_left_y=1551&top_left_x=571) Up to now we have not used the existence of $Q$; we henceforth do so. Note that $Q \\neq O$, since $\\angle A \\neq 60^{\\circ} \\Longrightarrow O \\notin \\omega$. Moreover, we have $\\angle A O M=\\angle A C B$ too. Since $O$ and $Q$ both lie inside $\\triangle A B C$, this implies that $A, M, O, Q$ are concyclic. As $Q \\neq O$ we conclude $\\angle C Q A=90^{\\circ}$. The main claim is now: Claim - Assuming $Q$ exists, the rhombus $L X K Y$ is a square. In particular, $\\overline{K X}$ and $\\overline{K Y}$ are tangent to $\\omega$. Second proof of Claim, Evan Chen. Observe that $Q$ lies on the circle with diameter $\\overline{A C}$, centered at $N$, say. This means that $O$ lies on the radical axis of $\\omega$ and $(N)$, hence $\\overline{N L} \\perp \\overline{C O}$ implying $$ \\begin{aligned} N O^{2}+C L^{2} & =N C^{2}+L O^{2}=N C^{2}+O C^{2}=N C^{2}+N O^{2}+N C^{2} \\\\ \\Longrightarrow x^{2} & =2 N C^{2} \\\\ \\Longrightarrow x & =\\sqrt{2} N C=\\frac{1}{\\sqrt{2}} A C=\\frac{1}{\\sqrt{2}} L K \\end{aligned} $$ So $L X K Y$ is a rhombus with $L K=\\sqrt{2} x$. Hence it is a square."} +{"year":2018,"label":"3","problem":"Let $A B C$ be an acute triangle with incenter $I$, circumcenter $O$, and circumcircle $\\Gamma$. Let $M$ be the midpoint of $\\overline{A B}$. Ray $A I$ meets $\\overline{B C}$ at $D$. Denote by $\\omega$ and $\\gamma$ the circumcircles of $\\triangle B I C$ and $\\triangle B A D$, respectively. Line $M O$ meets $\\omega$ at $X$ and $Y$, while line $C O$ meets $\\omega$ at $C$ and $Q$. Assume that $Q$ lies inside $\\triangle A B C$ and $\\angle A Q M=\\angle A C B$. Consider the tangents to $\\omega$ at $X$ and $Y$ and the tangents to $\\gamma$ at $A$ and $D$. Given that $\\angle B A C \\neq 60^{\\circ}$, prove that these four lines are concurrent on $\\Gamma$.","solution":" Henceforth assume $\\angle A \\neq 60^{\\circ}$; we prove the concurrence. Let $L$ denote the center of $\\omega$, which is the midpoint of minor arc $B C$. Claim - Let $K$ be the point on $\\omega$ such that $\\overline{K L} \\| \\overline{A B}$ and $\\overline{K C} \\| \\overline{A L}$. Then $\\overline{K A}$ is tangent to $\\gamma$, and we may put $$ x=K A=L B=L C=L X=L Y=K X=K Y . $$ Moreover, $\\overline{K A}$ is tangent to $\\gamma$ as well since $$ \\measuredangle K A D=\\measuredangle K A L=\\measuredangle K A C+\\measuredangle C A L=\\measuredangle K B C+\\measuredangle A B K=\\measuredangle A B C . $$ ![](https:\/\/cdn.mathpix.com\/cropped\/2024_11_19_a0f01f2a44c8136e2158g-08.jpg?height=918&width=914&top_left_y=1551&top_left_x=571) Up to now we have not used the existence of $Q$; we henceforth do so. Note that $Q \\neq O$, since $\\angle A \\neq 60^{\\circ} \\Longrightarrow O \\notin \\omega$. Moreover, we have $\\angle A O M=\\angle A C B$ too. Since $O$ and $Q$ both lie inside $\\triangle A B C$, this implies that $A, M, O, Q$ are concyclic. As $Q \\neq O$ we conclude $\\angle C Q A=90^{\\circ}$. The main claim is now: Claim - Assuming $Q$ exists, the rhombus $L X K Y$ is a square. In particular, $\\overline{K X}$ and $\\overline{K Y}$ are tangent to $\\omega$. Third proof of Claim. A solution by trig is also possible. As in the previous claims, it suffices to show that $A C=\\sqrt{2} x$. First, we compute the length $C Q$ in two ways; by angle chasing one can show $\\angle C B Q=$ $180^{\\circ}-(\\angle B Q C+\\angle Q C B)=\\frac{1}{2} \\angle A$, and so $$ \\begin{aligned} A C \\sin B=C Q & =\\frac{B C}{\\sin \\left(90^{\\circ}+\\frac{1}{2} \\angle A\\right)} \\cdot \\sin \\frac{1}{2} \\angle A \\\\ \\Longleftrightarrow \\sin ^{2} B & =\\frac{\\sin A \\cdot \\sin \\frac{1}{2} \\angle A}{\\cos \\frac{1}{2} \\angle A} \\\\ \\Longleftrightarrow \\sin ^{2} B & =2 \\sin ^{2} \\frac{1}{2} \\angle A \\\\ \\Longleftrightarrow \\sin B & =\\sqrt{2} \\sin \\frac{1}{2} \\angle A \\\\ \\Longleftrightarrow 2 R \\sin B & =\\sqrt{2}\\left(2 R \\sin \\frac{1}{2} \\angle A\\right) \\\\ \\Longleftrightarrow A C & =\\sqrt{2} x \\end{aligned} $$ as desired (we have here used the fact $\\triangle A B C$ is acute to take square roots). It is interesting to note that $\\sin ^{2} B=2 \\sin ^{2} \\frac{1}{2} \\angle A$ can be rewritten as $$ \\cos A=\\cos ^{2} B $$ since $\\cos ^{2} B=1-\\sin ^{2} B=1-2 \\sin ^{2} \\frac{1}{2} \\angle A=\\cos A$; this is the condition for the existence of the point $Q$. We finish by proving that $$ K D=K A $$ and hence line $\\overline{K D}$ is tangent to $\\gamma$. Let $E=\\overline{B C} \\cap \\overline{K L}$. Then $$ L E \\cdot L K=L C^{2}=L X^{2}=\\frac{1}{2} L K^{2} $$ and so $E$ is the midpoint of $\\overline{L K}$. Thus $\\overline{M X O Y}, \\overline{B C}, \\overline{K L}$ are concurrent at $E$. As $\\overline{D L} \\| \\overline{K C}$, we find that $D L C K$ is a parallelogram, so $K D=C L=K A$ as well. Thus $\\overline{K D}$ and $\\overline{K A}$ are tangent to $\\gamma$. Remark. The condition $\\angle A \\neq 60^{\\circ}$ cannot be dropped, since if $Q=O$ the problem is not true. \\ Authorship comments. In the notation of the present points, the question originally sent to me by Yannick Yao read: Circles $(L)$ and $(O)$ are drawn, meeting at $B$ and $C$, with $L$ on $(O)$. Ray $C O$ meets $(L)$ at $Q$, and $A$ is on $(O)$ such that $\\angle C Q A=90^{\\circ}$. The angle bisector of $\\angle A O B$ meets $(L)$ at $X$ and $Y$. Show that $\\angle X L Y=90^{\\circ}$. Notice the points $M$ and $K$ are absent from the problem. I am told this was found as part of the computer game \"Euclidea\". Using this as the starting point, I constructed the TSTST problem by recognizing the significance of that special point $K$, which became the center of attention."} +{"year":2018,"label":"4","problem":"For an integer $n>0$, denote by $\\mathcal{F}(n)$ the set of integers $m>0$ for which the polynomial $p(x)=x^{2}+m x+n$ has an integer root. (a) Let $S$ denote the set of integers $n>0$ for which $\\mathcal{F}(n)$ contains two consecutive integers. Show that $S$ is infinite but $$ \\sum_{n \\in S} \\frac{1}{n} \\leq 1 $$ (b) Prove that there are infinitely many positive integers $n$ such that $\\mathcal{F}(n)$ contains three consecutive integers.","solution":" We prove the following. Claim - The set $S$ is given explicitly by $S=\\{x(x+1) y(y+1) \\mid x, y>0\\}$. $$ \\begin{aligned} m^{2}-4 n & =p^{2} \\\\ (m+1)^{2}-4 n & =q^{2} \\end{aligned} $$ Subtraction gives $2 m+1=q^{2}-p^{2}$, so $p$ and $q$ are different parities. We can thus let $q-p=2 x+1, q+p=2 y+1$, where $y \\geq x \\geq 0$ are integers. It follows that $$ \\begin{aligned} 4 n & =m^{2}-p^{2} \\\\ & =\\left(\\frac{q^{2}-p^{2}-1}{2}\\right)^{2}-p^{2}=\\left(\\frac{q^{2}-p^{2}-1}{2}-p\\right)\\left(\\frac{q^{2}-p^{2}-1}{2}+p\\right) \\\\ & =\\frac{q^{2}-\\left(p^{2}+2 p+1\\right)}{2} \\cdot \\frac{q^{2}-\\left(p^{2}-2 p+1\\right)}{2} \\\\ & =\\frac{1}{4}(q-p-1)(q-p+1)(q+p-1)(q+p+1)=\\frac{1}{4}(2 x)(2 x+2)(2 y)(2 y+2) \\\\ \\Longrightarrow n & =x(x+1) y(y+1) . \\end{aligned} $$ Since $n>0$ we require $x, y>0$. Conversely, if $n=x(x+1) y(y+1)$ for positive $x$ and $y$ then $m=\\sqrt{p^{2}+4 n}=\\sqrt{(y-x)^{2}+4 n}=2 x y+x+y=x(y+1)+(x+1) y$ and $m+1=2 x y+x+y+1=x y+(x+1)(y+1)$. Thus we conclude the main claim. From this, part (a) follows as $$ \\sum_{n \\in S} n^{-1} \\leq\\left(\\sum_{x \\geq 1} \\frac{1}{x(x+1)}\\right)\\left(\\sum_{y \\geq 1} \\frac{1}{y(y+1)}\\right)=1 \\cdot 1=1 $$ $$ \\begin{aligned} r^{2} & =(m+2)^{2}-4 n=m^{2}-4 n+4 m+4=p^{2}+2+2(2 m+1) \\\\ & =p^{2}+2\\left(q^{2}-p^{2}\\right)+2=2 q^{2}-p^{2}+2 \\\\ \\Longleftrightarrow 2 q^{2}+2 & =p^{2}+r^{2} \\quad(\\dagger) \\end{aligned} $$ with $q>p$ of different parity and $n=\\frac{1}{16}(q-p-1)(q-p+1)(q+p-1)(q+p+1)$. Note that (by taking modulo 8 ) we have $q \\not \\equiv p \\equiv r(\\bmod 2)$, and so there are no parity issues and we will always assume $p2$ do there exist infinitely many positive integers $n$ such that $n^{2}$ divides $b^{n}+1$ ?","solution":" This problem is sort of the union of IMO 1990\/3 and IMO 2000\/5. The answer is any $b$ such that $b+1$ is not a power of 2 . In the forwards direction, we first prove more carefully the following claim. Claim - If $b+1$ is a power of 2 , then the only $n$ which is valid is $n=1$. $$ b^{2 n} \\equiv 1 \\quad(\\bmod p) $$ so the order of $b(\\bmod p)$ divides $\\operatorname{gcd}(2 n, p-1)=2$. Hence $p \\mid b^{2}-1=(b-1)(b+1)$. But since $b+1$ was a power of 2 , this forces $p \\mid b-1$. Then $0 \\equiv b^{n}+1 \\equiv 2(\\bmod p)$, contradiction. On the other hand, suppose that $b+1$ is not a power of 2 (and that $b>2$ ). We will inductively construct an infinite sequence of distinct primes $p_{0}, p_{1}, \\ldots$, such that the following two properties hold for each $k \\geq 0$ : - $p_{0}^{2} \\ldots p_{k-1}^{2} p_{k} \\mid b^{p_{0} \\ldots p_{k-1}}+1$, - and hence $p_{0}^{2} \\ldots p_{k-1}^{2} p_{k}^{2} \\mid b^{p_{0} \\ldots p_{k-1} p_{k}}+1$ by exponent lifting lemma. This will solve the problem. Initially, let $p_{0}$ be any odd prime dividing $b+1$. For the inductive step, we contend there exists an odd prime $q \\notin\\left\\{p_{0}, \\ldots, p_{k}\\right\\}$ such that $q \\mid b^{p_{0} \\ldots p_{k}}+1$. Indeed, this follows immediately by Zsigmondy theorem since $p_{0} \\ldots p_{k}$ divides $b^{p_{0} \\ldots p_{k-1}}+1$. Since $\\left(b^{p_{0} \\ldots p_{k}}\\right)^{q} \\equiv b^{p_{0} \\ldots p_{k}}(\\bmod q)$, it follows we can then take $p_{k+1}=q$. This finishes the induction. To avoid the use of Zsigmondy, one can instead argue as follows: let $p=p_{k}$ for brevity, and let $c=b^{p_{0} \\ldots p_{k-1}}$. Then $\\frac{\\frac{c}{}^{p}+1}{c+1}=c^{p-1}-c^{p-2}+\\cdots+1$ has GCD exactly $p$ with $c+1$. Moreover, this quotient is always odd. Thus as long as $c^{p}+1>p \\cdot(c+1)$, there will be some new prime dividing $c^{p}+1$ but not $c+1$. This is true unless $p=3$ and $c=2$, but we assumed $b>2$ so this case does not appear. Remark (On new primes). In going from $n^{2} \\mid b^{n}+1$ to $(n q)^{2} \\mid b^{n q}+1$, one does not necessarily need to pick a $q$ such that $q \\nmid n$, as long as $\\nu_{q}\\left(n^{2}\\right)<\\nu_{q}\\left(b^{n}+1\\right)$. In other words it suffices to just check that $\\frac{b^{n}+1}{n^{2}}$ is not a power of 2 in this process. However, this calculation is a little more involved with this approach. One proceeds by noting that $n$ is odd, hence $\\nu_{2}\\left(b^{n}+1\\right)=\\nu_{2}(b+1)$, and thus $\\frac{b^{n}+1}{n^{2}}=2^{\\nu_{2}(b+1)} \\leq b+1$, which is a little harder to bound than the analogous $c^{p}+1>p \\cdot(c+1)$ from the previous solution. \\ Authorship comments. I came up with this problem by simply mixing together the main ideas of IMO 1990\/3 and IMO 2000\/5, late one night after a class. On the other hand, I do not consider it very original; it is an extremely \"routine\" number theory problem for experienced contestants, using highly standard methods. Thus it may not be that interesting, but is a good discriminator of understanding of fundamentals. IMO 1990\/3 shows that if $b=2$, then the only $n$ which work are $n=1$ and $n=3$. Thus $b=2$ is a special case and for this reason the problem explicitly requires $b>2$. An alternate formulation of the problem is worth mentioning. Originally, the problem statement asked whether there existed $n$ with at least 3 (or 2018, etc.) prime divisors, thus preventing the approach in which one takes a prime $q$ dividing $\\frac{b^{n}+1}{n^{2}}$. Ankan Bhattacharya suggested changing it to \"infinitely many $n$ \", which is more natural. These formulations are actually not so different though. Explicitly, suppose $k^{2} \\mid b^{k}+1$ and $p \\mid b^{k}+1$. Consider any $k \\mid n$ with $n^{2} \\mid b^{n}+1$, and let $p$ be an odd prime dividing $b^{k}+1$. Then $2 \\nu_{p}(n) \\leq \\nu_{p}\\left(b^{n}+1\\right)=\\nu_{p}(n \/ k)+\\nu_{p}\\left(b^{k}+1\\right)$ and thus $$ \\nu_{p}(n \/ k) \\leq \\nu_{p}\\left(\\frac{b^{k}+1}{k^{2}}\\right) . $$ Effectively, this means we can only add each prime a certain number of times."} +{"year":2018,"label":"9","problem":"Show that there is an absolute constant $c<1$ with the following property: whenever $\\mathcal{P}$ is a polygon with area 1 in the plane, one can translate it by a distance of $\\frac{1}{100}$ in some direction to obtain a polygon $\\mathcal{Q}$, for which the intersection of the interiors of $\\mathcal{P}$ and $\\mathcal{Q}$ has total area at most $c$.","solution":" Suppose $\\mathcal{P}$ is a polygon of area 1 , and $\\varepsilon>0$ is a constant, such that for any translate $\\mathcal{Q}=\\mathcal{P}+v$, where $v$ has length exactly $\\frac{1}{100}$, the intersection of $\\mathcal{P}$ and $\\mathcal{Q}$ has area at least $1-\\varepsilon$. The problem asks us to prove a lower bound on $\\varepsilon$. ## Lemma Fix a sequence of $n$ vectors $v_{1}, v_{2}, \\ldots, v_{n}$, each of length $\\frac{1}{100}$. A grasshopper starts at a random point $x$ of $\\mathcal{P}$, and makes $n$ jumps to $x+v_{1}+\\cdots+v_{n}$. Then it remains in $\\mathcal{P}$ with probability at least $1-n \\varepsilon$. ## Corollary Fix a vector $w$ of length at most 8. A grasshopper starts at a random point $x$ of $\\mathcal{P}$, and jumps to $x+w$. Then it remains in $\\mathcal{P}$ with probability at least $1-800 \\varepsilon$. Now consider the process where we select a random starting point $x \\in \\mathcal{P}$ for our grasshopper, and a random vector $w$ of length at most 8 (sampled uniformly from the closed disk of radius 8 ). Let $q$ denote the probability of staying inside $\\mathcal{P}$ we will bound $q$ from above and below. - On the one hand, suppose we pick $w$ first. By the previous corollary, $q \\geq 1-800 \\varepsilon$ (irrespective of the chosen $w$ ). - On the other hand, suppose we pick $x$ first. Then the possible landing points $x+w$ are uniformly distributed over a closed disk of radius 8 , which has area $64 \\pi$. The probability of landing in $\\mathcal{P}$ is certainly at most $\\frac{[\\mathcal{P}]}{64 \\pi}$. Consequently, we deduce $$ 1-800 \\varepsilon \\leq q \\leq \\frac{[\\mathcal{P}]}{64 \\pi} \\Longrightarrow \\varepsilon>\\frac{1-\\frac{[\\mathcal{P}]}{64 \\pi}}{800}>0.001 $$ as desired. Remark. The choice of 800 jumps is only for concreteness; any constant $n$ for which $\\pi(n \/ 100)^{2}>1$ works. I think $n=98$ gives the best bound following this approach."} diff --git a/USA_TST_TST/segmented/en-sols-TSTST-2019.jsonl b/USA_TST_TST/segmented/en-sols-TSTST-2019.jsonl new file mode 100644 index 0000000000000000000000000000000000000000..eef29cbe0d69cd3fecbdfb88dc8c974da00b6a7f --- /dev/null +++ b/USA_TST_TST/segmented/en-sols-TSTST-2019.jsonl @@ -0,0 +1,15 @@ +{"year":2019,"label":"1","problem":"Find all binary operations $\\diamond: \\mathbb{R}_{>0} \\times \\mathbb{R}_{>0} \\rightarrow \\mathbb{R}_{>0}$ (meaning $\\diamond$ takes pairs of positive real numbers to positive real numbers) such that for any real numbers $a, b, c>0$, - the equation $a \\diamond(b \\diamond c)=(a \\diamond b) \\cdot c$ holds; and - if $a \\geq 1$ then $a \\diamond a \\geq 1$.","solution":" The answer is only multiplication and division, which both obviously work. \u3010 First solution using Cauchy FE. We prove: Claim - We have $a \\diamond b=a f(b)$ where $f$ is some involutive and totally multiplicative function. (In fact, this classifies all functions satisfying the first condition completely.) - Note that for any $x$, the function $y \\mapsto x \\diamond y$ is injective, because if $x \\diamond y_{1}=x \\diamond y_{2}$ then take $P\\left(1, x, y_{i}\\right)$ to get $y_{1}=y_{2}$. - Take $P(1, x, 1)$ and injectivity to get $x \\diamond 1=x$. - Take $P(1,1, y)$ to get $1 \\diamond(1 \\diamond y)=y$. - Take $P(x, 1,1 \\diamond y)$ to get $$ x \\diamond y=x \\cdot(1 \\diamond y) $$ Henceforth let us define $f(y)=1 \\diamond y$, so $f(1)=1, f$ is involutive and $$ x \\diamond y=x f(y) $$ Plugging this into the original condition now gives $f(b f(c))=f(b) c$, which (since $f$ is an involution) gives $f$ completely multiplicative. In particular, $f(1)=1$. We are now interested only in the second condition, which reads $f(x) \\geq 1 \/ x$ for $x \\geq 1$. Define the function $$ g(t)=\\log f\\left(e^{t}\\right) $$ so that $g$ is additive, and also $g(t) \\geq-t$ for all $t \\geq 0$. We appeal to the following theorem: ## Lemma If $h: \\mathbb{R} \\rightarrow \\mathbb{R}$ is an additive function which is not linear, then it is dense in the plane: for any point $\\left(x_{0}, y_{0}\\right)$ and $\\varepsilon>0$ there exists $(x, y)$ such that $h(x)=y$ and $\\sqrt{\\left(x-x_{0}\\right)^{2}+\\left(y-y_{0}\\right)^{2}}<\\varepsilon$. Applying this lemma with the fact that $g(t) \\geq-t$ implies readily that $g$ is linear. In other words, $f$ is of the form $f(x)=x^{r}$ for some fixed real number $r$. It is easy to check $r= \\pm 1$ which finishes."} +{"year":2019,"label":"1","problem":"Find all binary operations $\\diamond: \\mathbb{R}_{>0} \\times \\mathbb{R}_{>0} \\rightarrow \\mathbb{R}_{>0}$ (meaning $\\diamond$ takes pairs of positive real numbers to positive real numbers) such that for any real numbers $a, b, c>0$, - the equation $a \\diamond(b \\diamond c)=(a \\diamond b) \\cdot c$ holds; and - if $a \\geq 1$ then $a \\diamond a \\geq 1$.","solution":" The answer is only multiplication and division, which both obviously work. \u3010 Second solution manually. As before we arrive at $a \\diamond b=a f(b)$, with $f$ an involutive and totally multiplicative function. We prove that: Claim - For any $a>0$, we have $f(a) \\in\\{1 \/ a, a\\}$. Assume that $a b>1$; we show $a=b$. Note that for integers $m$ and $n$ with $a^{n} b^{m} \\geq 1$, we must have $$ a^{m} b^{n}=f(b)^{m} f(a)^{n}=f\\left(a^{n} b^{m}\\right) \\geq \\frac{1}{a^{n} b^{m}} \\Longrightarrow(a b)^{m+n} \\geq 1 $$ and thus we have arrived at the proposition $$ m+n<0 \\Longrightarrow n \\log _{b} a+m<0 $$ for all integers $m$ and $n$. Due to the density of $\\mathbb{Q}$ in the real numbers, this can only happen if $\\log _{b} a=1$ or $a=b$. Claim - The function $f$ is continuous. $$ |g(t)-g(s)|=\\left|\\log f\\left(e^{t-s}\\right)\\right|=|t-s| $$ since $f\\left(e^{t-s}\\right)=e^{ \\pm|t-s|}$. Therefore $g$ is Lipschitz. Hence $g$ continuous, and $f$ is too. Finally, we have from $f$ multiplicative that $$ f\\left(2^{q}\\right)=f(2)^{q} $$ for every rational number $q$, say. As $f$ is continuous this implies $f(x) \\equiv x$ or $f(x) \\equiv 1 \/ x$ identically (depending on whether $f(2)=2$ or $f(2)=1 \/ 2$, respectively). Therefore, $a \\diamond b=a b$ or $a \\diamond b=a \\div b$, as needed. Remark. The Lipschitz condition is one of several other ways to proceed. The point is that if $f(2)=2$ (say), and $x \/ 2^{q}$ is close to 1 , then $f(x) \/ 2^{q}=f\\left(x \/ 2^{q}\\right)$ is close to 1 , which is enough to force $f(x)=x$ rather than $f(x)=1 \/ x$. Remark. Compare to AMC 10A 2016 \\#23, where the second condition is $a \\diamond a=1$."} +{"year":2019,"label":"2","problem":"Let $A B C$ be an acute triangle with circumcircle $\\Omega$ and orthocenter $H$. Points $D$ and $E$ lie on segments $A B$ and $A C$ respectively, such that $A D=A E$. The lines through $B$ and $C$ parallel to $\\overline{D E}$ intersect $\\Omega$ again at $P$ and $Q$, respectively. Denote by $\\omega$ the circumcircle of $\\triangle A D E$. (a) Show that lines $P E$ and $Q D$ meet on $\\omega$. (b) Prove that if $\\omega$ passes through $H$, then lines $P D$ and $Q E$ meet on $\\omega$ as well.","solution":" Claim - Points $L, D, P$ are collinear. $$ \\begin{aligned} \\measuredangle C L D & =\\measuredangle D H L=\\measuredangle D H A+\\measuredangle A H L=\\measuredangle D E A+\\measuredangle A H C \\\\ & =\\measuredangle A D E+\\measuredangle C B A=\\measuredangle A B P+\\measuredangle C B A=\\measuredangle C B P=\\measuredangle C L P . \\end{aligned} $$ ![](https:\/\/cdn.mathpix.com\/cropped\/2024_11_19_f4f672a97b66743213a8g-06.jpg?height=641&width=709&top_left_y=1707&top_left_x=676) Now let $K \\in \\omega$ such that $D H K E$ is an isosceles trapezoid, i.e. $\\measuredangle B A H=\\measuredangle K A E$. Claim - Points $D, K, P$ are collinear. $$ \\measuredangle K D E=\\measuredangle K A E=\\measuredangle B A H=\\measuredangle L A B=\\measuredangle L P B=\\measuredangle D P B=\\measuredangle P D E . $$ By symmetry, $\\overline{Q E}$ will then pass through the same $K$, as needed. Remark. These two claims imply each other, so guessing one of them allows one to realize the other. It is likely the latter is easiest to guess from the diagram, since it does not need any additional points. Claim - Point $K$ is the orthocenter of isosceles triangle $A P Q$. In light of the formula \" $A H^{2}=4 R^{2}-a^{2}$ \", this implies the conclusion. Let $M$ be the midpoint of $\\overline{P Q}$. Since $\\triangle A P Q$ is isosceles, $$ \\overline{A K M} \\perp \\overline{P Q} \\Longrightarrow M K \\cdot M A=M P^{2} $$ by orthocenter properties. So to summarize - The circle with diameter $\\overline{P Q}$ is orthogonal to $\\omega$. In other words, point $P$ lies on the polar of $Q$ with respect to $\\omega$. - The point $X=\\overline{Q D} \\cap \\overline{P E}$ is on $\\omega$. On the other hand, if we let $K^{\\prime}=\\overline{Q E} \\cap \\omega$, then by Brokard theorem on $X D K^{\\prime} E$, the polar of $Q=\\overline{X D} \\cap \\overline{K^{\\prime} E}$ pass through $\\overline{D K^{\\prime}} \\cap \\overline{X E}$; this point must therefore be $P$ and $K^{\\prime}=K$ as desired. $$ A=a^{2}, B=b^{2}, C=c^{2}, M=-b c, H=a^{2}+b^{2}+c^{2}, P=\\frac{a^{2} c}{b}, Q=\\frac{a^{2} b}{c} $$ where $M$ is the arc midpoint of $\\widehat{B C}$. By direct angle chasing we can verify that $\\overline{M B} \\| \\overline{D H}$. Also, $D \\in \\overline{A B}$. Therefore, we can compute $D$ as follows. $$ d+a^{2} b^{2} \\bar{d}=a^{2}+b^{2} \\text { and } \\frac{d-h}{\\bar{d}-\\bar{h}}=-m b^{2}=b^{3} c \\Longrightarrow d=\\frac{a^{2}\\left(a^{2} c+b^{2} c+c^{3}-b^{3}\\right)}{c\\left(b c+a^{2}\\right)} . $$ By symmetry, we have that $$ e=\\frac{a^{2}\\left(a^{2} b+b c^{2}+b^{3}-c^{3}\\right)}{b\\left(b c+a^{2}\\right)} . $$ To finish, we want to show that the angle between $\\overline{D P}$ and $\\overline{E Q}$ is angle $A$. To show this, we compute $\\frac{d-p}{e-q} \/ \\overline{\\frac{d-p}{e-q}}$. First, we compute $$ \\begin{aligned} d-p & =\\frac{a^{2}\\left(a^{2} c+b^{2} c+c^{3}-b^{3}\\right)}{c\\left(b c+a^{2}\\right)}-\\frac{a^{2} c}{b} \\\\ & =a^{2}\\left(\\frac{a^{2} c+b^{2} c+c^{3}-b^{3}}{c\\left(b c+a^{2}\\right)}-\\frac{c}{b}\\right)=\\frac{a^{2}\\left(a^{2} c-b^{3}\\right)(b-c)}{b c\\left(b c+a^{2}\\right)} \\end{aligned} $$ By symmetry, $$ \\frac{d-p}{e-q}=-\\frac{a^{2} c-b^{3}}{a^{2} b-c^{3}} \\Longrightarrow \\frac{d-p}{e-q} \/ \\overline{\\frac{d-p}{e-q}}=\\frac{a^{2} b^{3} c}{a^{2} b c^{3}}=\\frac{b^{2}}{c^{2}} $$ as desired. Recall: ## Lemma (Zack's lemma) Suppose points $A, B$ have degree $d_{1}, d_{2}$, and there are $k$ values of $t$ for which $A=B$. Then line $A B$ has degree at most $d_{1}+d_{2}-k$. Similarly, if lines $\\ell_{1}, \\ell_{2}$ have degrees $d_{1}, d_{2}$, and there are $k$ values of $t$ for which $\\ell_{1}=\\ell_{2}$, then the intersection $\\ell_{1} \\cap \\ell_{2}$ has degree at most $d_{1}+d_{2}-k$. Now, note that $H$ moves linearly in $C$ on line $B H$. Furthermore, angles $\\angle A H E$, $\\angle A H F$ are fixed, we get that $D$ and $E$ have degree 2 . One way to see this is using the lemma; $D$ lies on line $A B$, which is fixed, and line $H D$ passes through a point at infinity which is a constant rotation of the point at infinity on line $A H$, and therefore has degree 1. Then $D, E$ have degree at most $1+1-0=2$. Now, note that $P, Q$ move linearly in $C$. Both of these are because the circumcenter $O$ moves linearly in $C$, and $P, Q$ are reflections of $B, C$ in a line through $O$ with fixed direction, which also moves linearly. So by the lemma, the lines $P D, Q E$ have degree at most 3 . I claim they actually have degree 2; to show this it suffices to give an example of a choice of $C$ for which $P=D$ and one for which $Q=E$. But an easy angle chase shows that in the unique case when $P=B$, we get $D=B$ as well and thus $P=D$. Similarly when $Q=C, E=C$. It follows from the lemma that lines $P D, Q E$ have degree at most 2 . Let $\\ell_{\\infty}$ denote the line at infinity. I claim that the points $P_{1}=P D \\cap \\ell_{\\infty}, P_{2}=Q E \\cap \\ell_{\\infty}$ are projective in $C$. Since $\\ell_{\\infty}$ is fixed, it suffices to show by the lemma that there exists some value of $C$ for which $Q E=\\ell_{\\infty}$ and $P D=\\ell_{\\infty}$. But note that as $C \\rightarrow \\infty$, all four points $P, D, Q, E$ go to infinity. It follows that $P_{1}, P_{2}$ are projective in $C$. Then to finish, recall that we want to show that $\\angle(P D, Q E)$ is constant. It suffices then to show that there's a constant rotation sending $P_{1}$ to $P_{2}$. Since $P_{1}, P_{2}$ are projective, it suffices to verify this for 3 values of $C$. We can take $C$ such that $\\angle A B C=90, \\angle A C B=90$, or $A B=A C$, and all three cases are easy to check."} +{"year":2019,"label":"3","problem":"On an infinite square grid we place finitely many cars, which each occupy a single cell and face in one of the four cardinal directions. Cars may never occupy the same cell. It is given that the cell immediately in front of each car is empty, and moreover no two cars face towards each other (no right-facing car is to the left of a left-facing car within a row, etc.). In a move, one chooses a car and shifts it one cell forward to a vacant cell. Prove that there exists an infinite sequence of valid moves using each car infinitely many times.","solution":" Let $S$ be any rectangle containing all the cars. Partition $S$ into horizontal strips of height 1 , and color them red and green in an alternating fashion. It is enough to prove all the cars may exit $S$. ![](https:\/\/cdn.mathpix.com\/cropped\/2024_11_19_f4f672a97b66743213a8g-09.jpg?height=880&width=1038&top_left_y=1096&top_left_x=512) To do so, we outline a five-stage plan for the cars. 1. All vertical cars in a green cell may advance one cell into a red cell (or exit $S$ altogether), by the given condition. (This is the only place where the hypothesis about empty space is used!) 2. All horizontal cars on green cells may exit $S$, as no vertical cars occupy green cells. 3. All vertical cars in a red cell may advance one cell into a green cell (or exit $S$ altogether), as all green cells are empty. 4. All horizontal cars within red cells may exit $S$, as no vertical car occupy red cells. 5. The remaining cars exit $S$, as they are all vertical. The solution is complete. Remark (Higher-dimensional generalization by author). The natural higher-dimensional generalization is true, and can be proved in largely the same fashion. For example, in three dimensions, one may let $S$ be a rectangular prism and partition $S$ into horizontal slabs and color them red and green in an alternating fashion. Stages 1, 3, and 5 generalize immediately, and stages 2 and 4 reduce to an application of the two-dimensional problem. In the same way, the general problem is handled by induction on the dimension. Remark (Historical comments). For $k>1$, we could consider a variant of the problem where cars are $1 \\times k$ rectangles (moving parallel to the longer edge) instead of occupying single cells. In that case, if there are $2 k-1$ empty spaces in front of each car, the above proof works (with the red and green strips having height $k$ instead). On the other hand, at least $k$ empty spaces are necessary. We don't know the best constant in this case."} +{"year":2019,"label":"4","problem":"Consider coins with positive real denominations not exceeding 1. Find the smallest $C>0$ such that the following holds: if we are given any 100 such coins with total value 50 , then we can always split them into two stacks of 50 coins each such that the absolute difference between the total values of the two stacks is at most $C$.","solution":" The answer is $C=\\frac{50}{51}$. The lower bound is obtained if we have 51 coins of value $\\frac{1}{51}$ and 49 coins of value 1. (Alternatively, 51 coins of value $1-\\frac{\\varepsilon}{51}$ and 49 coins of value $\\frac{\\varepsilon}{49}$ works fine for $\\varepsilon>0$.) We now present two (similar) proofs that this $C=\\frac{50}{51}$ suffices. \u3010 First proof (original). Let $a_{1} \\leq \\cdots \\leq a_{100}$ denote the values of the coins in ascending order. Since the 51 coins $a_{50}, \\ldots, a_{100}$ are worth at least $51 a_{50}$, it follows that $a_{50} \\leq \\frac{50}{51}$; likewise $a_{51} \\geq \\frac{1}{51}$. We claim that choosing the stacks with coin values $$ a_{1}, a_{3}, \\ldots, a_{49}, \\quad a_{52}, a_{54}, \\ldots, a_{100} $$ and $$ a_{2}, a_{4}, \\ldots, a_{50}, \\quad a_{51}, a_{53}, \\ldots, a_{99} $$ works. Let $D$ denote the (possibly negative) difference between the two total values. Then $$ \\begin{aligned} D & =\\left(a_{1}-a_{2}\\right)+\\cdots+\\left(a_{49}-a_{50}\\right)-a_{51}+\\left(a_{52}-a_{53}\\right)+\\cdots+\\left(a_{98}-a_{99}\\right)+a_{100} \\\\ & \\leq 25 \\cdot 0-\\frac{1}{51}+24 \\cdot 0+1=\\frac{50}{51} \\end{aligned} $$ Similarly, we have $$ \\begin{aligned} D & =a_{1}+\\left(a_{3}-a_{2}\\right)+\\cdots+\\left(a_{49}-a_{48}\\right)-a_{50}+\\left(a_{52}-a_{51}\\right)+\\cdots+\\left(a_{100}-a_{99}\\right) \\\\ & \\geq 0+24 \\cdot 0-\\frac{50}{51}+25 \\cdot 0=-\\frac{50}{51} \\end{aligned} $$ It follows that $|D| \\leq \\frac{50}{51}$, as required."} +{"year":2019,"label":"4","problem":"Consider coins with positive real denominations not exceeding 1. Find the smallest $C>0$ such that the following holds: if we are given any 100 such coins with total value 50 , then we can always split them into two stacks of 50 coins each such that the absolute difference between the total values of the two stacks is at most $C$.","solution":" The answer is $C=\\frac{50}{51}$. The lower bound is obtained if we have 51 coins of value $\\frac{1}{51}$ and 49 coins of value 1. (Alternatively, 51 coins of value $1-\\frac{\\varepsilon}{51}$ and 49 coins of value $\\frac{\\varepsilon}{49}$ works fine for $\\varepsilon>0$.) We now present two (similar) proofs that this $C=\\frac{50}{51}$ suffices. \u3010 Second proof (Evan Chen). Again we sort the coins in increasing order $0a_{i-1}+\\frac{50}{51}$; obviously there is at most one such large gap. Claim - If there is a large gap, it must be $a_{51}>a_{50}+\\frac{50}{51}$. Now imagine starting with the coins $a_{1}, a_{3}, \\ldots, a_{99}$, which have total value $S \\leq 25$. We replace $a_{1}$ by $a_{2}$, then $a_{3}$ by $a_{4}$, and so on, until we replace $a_{99}$ by $a_{100}$. At the end of the process we have $S \\geq 25$. Moreover, since we did not cross a large gap at any point, the quantity $S$ changed by at most $C=\\frac{50}{51}$ at each step. So at some point in the process we need to have $25-C \/ 2 \\leq S \\leq 25+C \/ 2$, which proves $C$ works."} +{"year":2019,"label":"5","problem":"Let $A B C$ be an acute triangle with orthocenter $H$ and circumcircle $\\Gamma$. A line through $H$ intersects segments $A B$ and $A C$ at $E$ and $F$, respectively. Let $K$ be the circumcenter of $\\triangle A E F$, and suppose line $A K$ intersects $\\Gamma$ again at a point $D$. Prove that line $H K$ and the line through $D$ perpendicular to $\\overline{B C}$ meet on $\\Gamma$.","solution":" \u300e First solution (Andrew Gu). We begin with the following two observations. Claim - Point $K$ lies on the radical axis of $(B E H)$ and $(C F H)$. $$ \\measuredangle H E K=90^{\\circ}-\\measuredangle E A F=90^{\\circ}-\\measuredangle B A C=\\measuredangle H B E $$ implying the result. Since $K E=K F$, this implies the result. Claim - The second intersection $M$ of $(B E H)$ and $(C F H)$ lies on $\\Gamma$. ![](https:\/\/cdn.mathpix.com\/cropped\/2024_11_19_f4f672a97b66743213a8g-13.jpg?height=806&width=632&top_left_y=1613&top_left_x=712) In particular, $M, H, K$ are collinear. Let $X$ be on $\\Gamma$ with $\\overline{D X} \\perp \\overline{B C}$; we then wish to show $X$ lies on the line $M H K$ we found. This is angle chasing: compute $$ \\measuredangle X M B=\\measuredangle X D B=90^{\\circ}-\\measuredangle D B C=90^{\\circ}-\\measuredangle D A C $$ $$ =90^{\\circ}-\\measuredangle K A F=\\measuredangle F E A=\\measuredangle H E B=\\measuredangle H M B $$ as needed."} +{"year":2019,"label":"5","problem":"Let $A B C$ be an acute triangle with orthocenter $H$ and circumcircle $\\Gamma$. A line through $H$ intersects segments $A B$ and $A C$ at $E$ and $F$, respectively. Let $K$ be the circumcenter of $\\triangle A E F$, and suppose line $A K$ intersects $\\Gamma$ again at a point $D$. Prove that line $H K$ and the line through $D$ perpendicular to $\\overline{B C}$ meet on $\\Gamma$.","solution":" II Second solution (Ankan Bhattacharya). We let $D^{\\prime}$ be the second intersection of $\\overline{E F}$ with $(B H C)$ and redefine $D$ as the reflection of $D^{\\prime}$ across $\\overline{B C}$. We will first prove that this point $D$ coincides with the point $D$ given in the problem statement. The idea is that: Claim - $A$ is the $D$-excenter of $\\triangle D E F$. $$ \\begin{aligned} \\measuredangle D^{\\prime} D B & =\\measuredangle B D^{\\prime} D=\\measuredangle D^{\\prime} B C+90^{\\circ}=\\measuredangle D^{\\prime} H C+90^{\\circ} \\\\ & =\\measuredangle D^{\\prime} H C+\\measuredangle(H C, A B)=\\measuredangle\\left(D^{\\prime} H, A B\\right)=\\measuredangle D^{\\prime} E B \\end{aligned} $$ Now as $B D=B D^{\\prime}$, we obtain $\\overline{B E A}$ externally bisects $\\angle D E D^{\\prime} \\cong \\angle D E F$. Likewise $\\overline{F A}$ externally bisects $\\angle D F E$, so $A$ is the $D$-excenter of $\\triangle D E F$. Hence, by the so-called \"Fact 5 \", point $K$ lies on $\\overline{D A}$, so this point $D$ is the one given in the problem statement. ![](https:\/\/cdn.mathpix.com\/cropped\/2024_11_19_f4f672a97b66743213a8g-14.jpg?height=778&width=712&top_left_y=1256&top_left_x=678) Now choose point $X$ on $(A B C)$ satisfying $\\overline{D X} \\perp \\overline{B C}$. Claim - Point $K$ lies on line $H X$. $$ \\frac{K D}{K A}=\\frac{K D}{K E}=\\frac{D E+D F}{E F} $$ On the other hand, if we let $r_{D}$ denote the $D$-exradius of $\\triangle D E F$ then $$ \\frac{X D}{X D^{\\prime}}=\\frac{[D E X]+[D F X]}{[X E F]}=\\frac{[D E X]+[D F X]}{[A E F]}=\\frac{D E \\cdot r_{D}+D F \\cdot r_{D}}{E F \\cdot r_{D}}=\\frac{D E+D F}{E F} $$ Thus $$ [A K X]=\\frac{K A}{K D} \\cdot[D K X]=\\frac{K A}{K D} \\cdot \\frac{X D}{X D^{\\prime}} \\cdot\\left[K D^{\\prime} X\\right]=\\left[D^{\\prime} K X\\right] $$ This is sufficient to prove $K$ lies on $\\overline{H X}$."} +{"year":2019,"label":"5","problem":"Let $A B C$ be an acute triangle with orthocenter $H$ and circumcircle $\\Gamma$. A line through $H$ intersects segments $A B$ and $A C$ at $E$ and $F$, respectively. Let $K$ be the circumcenter of $\\triangle A E F$, and suppose line $A K$ intersects $\\Gamma$ again at a point $D$. Prove that line $H K$ and the line through $D$ perpendicular to $\\overline{B C}$ meet on $\\Gamma$.","solution":" I Fourth solution, complex numbers with spiral similarity (Evan Chen). First if $\\overline{A D} \\perp \\overline{B C}$ there is nothing to prove, so we assume this is not the case. Let $W$ be the antipode of $D$. Let $S$ denote the second intersection of $(A E F)$ and $(A B C)$. Consider the spiral similarity sending $\\triangle S E F$ to $\\triangle S B C$ : - It maps $H$ to a point $G$ on line $B C$, - It maps $K$ to $O$. - It maps the $A$-antipode of $\\triangle A E F$ to $D$. - Hence (by previous two observations) it maps $A$ to $W$. - Also, the image of line $A D$ is line $W O$, which does not coincide with line $B C$ (as $O$ does not lie on line $B C$ ). Therefore, $K$ is the unique point on line $\\overline{A D}$ for one can get a direct similarity $$ \\triangle A K H \\sim \\triangle W O G $$ for some point $G$ lying on line $\\overline{B C}$. ![](https:\/\/cdn.mathpix.com\/cropped\/2024_11_19_f4f672a97b66743213a8g-15.jpg?height=804&width=775&top_left_y=1477&top_left_x=646) On the other hand, let us re-define $K$ as $\\overline{X H} \\cap \\overline{A D}$. We will show that the corresponding $G$ making $(\\triangle)$ true lies on line $B C$. We apply complex numbers with $\\Gamma$ the unit circle, with $a, b, c, d$ taking their usual meanings, $H=a+b+c, X=-b c \/ d$, and $W=-d$. Then point $K$ is supposed to satisfy $$ k+a d \\bar{k}=a+d $$ $$ \\begin{aligned} \\frac{k+\\frac{b c}{d}}{a+b+c+\\frac{b c}{d}} & =\\frac{\\bar{k}+\\frac{d}{b c}}{\\frac{1}{a}+\\frac{1}{b}+\\frac{1}{c}+\\frac{d}{b c}} \\\\ \\Longleftrightarrow \\frac{\\frac{1}{a}+\\frac{1}{b}+\\frac{1}{c}+\\frac{d}{b c}}{a+b+c+\\frac{b c}{d}}\\left(k+\\frac{b c}{d}\\right) & =\\bar{k}+\\frac{d}{b c} \\end{aligned} $$ Adding $a d$ times the last line to the first line and cancelling $a d \\bar{k}$ now gives $$ \\left(a d \\cdot \\frac{\\frac{1}{a}+\\frac{1}{b}+\\frac{1}{c}+\\frac{d}{b c}}{a+b+c+\\frac{b c}{d}}+1\\right) k=a+d+\\frac{a d^{2}}{b c}-a b c \\cdot \\frac{\\frac{1}{a}+\\frac{1}{b}+\\frac{1}{c}+\\frac{d}{b c}}{a+b+c+\\frac{b c}{d}} $$ or $$ \\begin{aligned} \\left(a d\\left(\\frac{1}{a}+\\frac{1}{b}+\\frac{1}{c}+\\frac{d}{b c}\\right)+a+b+c+\\frac{b c}{d}\\right) k & =\\left(a+b+c+\\frac{b c}{d}\\right)\\left(a+d+\\frac{a d^{2}}{b c}\\right) \\\\ & -a b c \\cdot\\left(\\frac{1}{a}+\\frac{1}{b}+\\frac{1}{c}+\\frac{d}{b c}\\right) . \\end{aligned} $$ We begin by simplifying the coefficient of $k$ : $$ \\begin{aligned} a d\\left(\\frac{1}{a}+\\frac{1}{b}+\\frac{1}{c}+\\frac{d}{b c}\\right)+a+b+c+\\frac{b c}{d} & =a+b+c+d+\\frac{b c}{d}+\\frac{a d}{b}+\\frac{a d}{c}+\\frac{a d^{2}}{b c} \\\\ & =a+\\frac{b c}{d}+\\left(1+\\frac{a d}{b c}\\right)(b+c+d) \\\\ & =\\frac{a d+b c}{b c d}[b c+d(b+c+d)] \\\\ & =\\frac{(a d+b c)(d+b)(d+c)}{b c d} \\end{aligned} $$ Meanwhile, the right-hand side expands to $$ \\begin{aligned} \\mathrm{RHS}= & \\left(a+b+c+\\frac{b c}{d}\\right)\\left(a+d+\\frac{a d^{2}}{b c}\\right)-a b c \\cdot\\left(\\frac{1}{a}+\\frac{1}{b}+\\frac{1}{c}+\\frac{d}{b c}\\right) \\\\ = & \\left(a^{2}+a b+a c+\\frac{a b c}{d}\\right)+(d a+d b+d c+b c) \\\\ & +\\left(\\frac{a^{2} d^{2}}{b c}+\\frac{a d^{2}}{c}+\\frac{a d^{2}}{b}+a d\\right)-(a b+b c+c a+a d) \\\\ = & a^{2}+d(a+b+c)+\\frac{a b c}{d}+\\frac{a^{2} d^{2}}{b c}+\\frac{a d^{2}}{b}+\\frac{a d^{2}}{c} \\\\ = & a^{2}+\\frac{a b c}{d}+d(a+b+c) \\cdot \\frac{a d+b c}{b c} \\\\ = & \\frac{a d+b c}{b c d}\\left[a b c+d^{2}(a+b+c)\\right] . \\end{aligned} $$ Therefore, we get $$ k=\\frac{a b c+d^{2}(a+b+c)}{(d+b)(d+c)} $$ In particular, $$ \\begin{aligned} k-a & =\\frac{a b c+d^{2}(a+b+c)-a(d+b)(d+c)}{(d+b)(d+c)} \\\\ & =\\frac{d^{2}(b+c)-d a(b+c)}{(d+b)(d+c)}=\\frac{d(b+c)(d-a)}{(d+b)(d+c)} \\end{aligned} $$ Now the corresponding point $G$ obeying $(\\Omega)$ satisfies $$ \\begin{aligned} \\frac{g-(-d)}{0-(-d)} & =\\frac{(a+b+c)-a}{k-a} \\\\ \\Longrightarrow g & =-d+\\frac{d(b+c)}{k-a} \\\\ & =-d+\\frac{(d+b)(d+c)}{d-a}=\\frac{d b+d c+b c+a d}{d-a} . \\\\ \\Longrightarrow b c \\bar{g} & =\\frac{b c \\cdot \\frac{a c+a b+a d+b c}{a b c d}}{\\frac{a-d}{a d}}=-\\frac{a b+a c+a d+b c}{d-a} . \\\\ \\Longrightarrow g+b c \\bar{g} & =\\frac{(d-a)(b+c)}{d-a}=b+c . \\end{aligned} $$ Hence $G$ lies on $B C$ and this completes the proof."} +{"year":2019,"label":"5","problem":"Let $A B C$ be an acute triangle with orthocenter $H$ and circumcircle $\\Gamma$. A line through $H$ intersects segments $A B$ and $A C$ at $E$ and $F$, respectively. Let $K$ be the circumcenter of $\\triangle A E F$, and suppose line $A K$ intersects $\\Gamma$ again at a point $D$. Prove that line $H K$ and the line through $D$ perpendicular to $\\overline{B C}$ meet on $\\Gamma$.","solution":" \u3010 Seventh solution using moving points (Zack Chroman). We state the converse of the problem as follows: Take a point $D$ on $\\Gamma$, and let $G \\in \\Gamma$ such that $\\overline{D G} \\perp \\overline{B C}$. Then define $K$ to lie on $\\overline{G H}, \\overline{A D}$, and take $L \\in \\overline{A D}$ such that $K$ is the midpoint of $\\overline{A L}$. Then if we define $E$ and $F$ as the projections of $L$ onto $\\overline{A B}$ and $\\overline{A C}$ we want to show that $E, H, F$ are collinear. It's clear that solving this problem will solve the original. In fact we will show later that each line $E F$ through $H$ corresponds bijectively to the point $D$. We animate $D$ projectively on $\\Gamma$ (hence $\\operatorname{deg} D=2$ ). Since $D \\mapsto G$ is a projective map $\\Gamma \\rightarrow \\Gamma$, it follows $\\operatorname{deg} G=2$. By Zack's lemma, $\\operatorname{deg}(\\overline{A D}) \\leq 0+2-1=1$ (since $D$ can coincide with $A$ ), and $\\operatorname{deg}(\\overline{H G}) \\leq 0+2-0=2$. So again by Zack's lemma, $\\operatorname{deg} K \\leq 1+2-1=2$, since lines $A D$ and $G H$ can coincide once if $D$ is the reflection of $H$ over $\\overline{B C}$. It follows $\\operatorname{deg} L=2$, since it is obtained by dilating $K$ by a factor of 2 across the fixed point $A$. Let $\\infty_{C}$ be the point at infinity on the line perpendicular to $A C$, and similarly $\\infty_{B}$. Then $$ F=\\overline{A C} \\cap \\overline{\\infty_{C} L}, \\quad E=\\overline{A B} \\cap \\overline{\\infty_{B} L} . $$ We want to use Zack's lemma again on line $\\overline{\\infty_{B} L}$. Consider the case $G=B$; we get $\\overline{H G} \\| \\overline{A D}$, so $A D G H$ is a parallelogram, and then $K=L=\\infty_{B}$. Thus there is at least one $t$ where $L=\\infty_{B}$ and by Zack's lemma we get $\\operatorname{deg}\\left(\\overline{\\infty_{B} L}\\right) \\leq 0+2-1=1$. Again by Zack's lemma, we conclude $\\operatorname{deg} E \\leq 0+1-0=1$. Similarly, $\\operatorname{deg} F \\leq 1$. We were aiming to show $E, F, H$ collinear which is a condition of degree at most $1+1+0=2$. So it suffices to verify the problem for three distinct choices of $D$. - If $D=A$, then line $G H$ is line $A H$, and $L=\\overline{A D} \\cap \\overline{A H}=A$. So $E=F=A$ and the statement is true. - If $D=B, G$ is the antipode of $C$ on $\\Gamma$. Then $K=\\overline{H G} \\cap \\overline{A D}$ is the midpoint of $\\overline{A B}$, so $L=B$. Then $E=B$ and $F$ is the projection of $B$ onto $A C$, so $E, H, F$ collinear. - We finish similarly when $D=C$."} +{"year":2019,"label":"6","problem":"Suppose $P$ is a polynomial with integer coefficients such that for every positive integer $n$, the sum of the decimal digits of $|P(n)|$ is not a Fibonacci number. Must $P$ be constant?","solution":" The answer is yes, $P$ must be constant. By $S(n)$ we mean the sum of the decimal digits of $|n|$. We need two claims. Claim - If $P(x) \\in \\mathbb{Z}[x]$ is nonconstant with positive leading coefficient, then there exists an integer polynomial $F(x)$ such that all coefficients of $P \\circ F$ are positive except for the second one, which is negative. First, consider $T_{0}(x)=x^{3}+x+1$. Observe that in $T_{0}^{\\operatorname{deg} P}$, every coefficient is strictly positive, except for the second one, which is zero. Then, let $T_{1}(x)=x^{3}-\\frac{1}{D} x^{2}+x+1$. Using continuity as $D \\rightarrow \\infty$, it follows that if $D$ is large enough (in terms of $\\operatorname{deg} P$ ), then $T_{1}^{\\operatorname{deg} P}$ is good, with $-\\frac{3}{D} x^{3 \\operatorname{deg} P-1}$ being the only negative coefficient. Finally, we can let $F(x)=C T_{1}(x)$ where $C$ is a sufficiently large multiple of $D$ (in terms of the coefficients of $P$ ); thus the coefficients of $\\left(C T_{1}(x)\\right)^{\\operatorname{deg} P}$ dominate (and are integers), as needed. Claim - There are infinitely many Fibonacci numbers in each residue class modulo 9. $$ \\begin{aligned} F_{0}=0 & \\equiv 0 \\quad(\\bmod 9) \\\\ F_{1}=1 & \\equiv 1 \\quad(\\bmod 9) \\\\ F_{3}=2 & \\equiv 2 \\quad(\\bmod 9) \\\\ F_{4}=3 & \\equiv 3 \\quad(\\bmod 9) \\\\ F_{7}=13 & \\equiv 4 \\quad(\\bmod 9) \\\\ F_{5}=5 & \\equiv 5 \\quad(\\bmod 9) \\\\ F_{-4}=-3 & \\equiv 6 \\quad(\\bmod 9) \\\\ F_{9}=34 & \\equiv 7 \\quad(\\bmod 9) \\\\ F_{6}=8 & \\equiv 8 \\quad(\\bmod 9) . \\end{aligned} $$ We now show how to solve the problem with the two claims. WLOG $P$ satisfies the conditions of the first claim, and choose $F$ as above. Let $$ P(F(x))=c_{N} x^{N}-c_{N-1} x^{N-1}+c_{N-2} x^{N-2}+\\cdots+c_{0} $$ where $c_{i}>0$ (and $N=3 \\operatorname{deg} P$ ). Then if we select $x=10^{e}$ for $e$ large enough (say $\\left.x>10 \\max _{i} c_{i}\\right)$, the decimal representation $P\\left(F\\left(10^{e}\\right)\\right)$ consists of the concatenation of - the decimal representation of $c_{N}-1$, - the decimal representation of $10^{e}-c_{N-1}$ - the decimal representation of $c_{N-2}$, with several leading zeros, - the decimal representation of $c_{N-3}$, with several leading zeros, - ... - the decimal representation of $c_{0}$, with several leading zeros. (For example, if $P(F(x))=15 x^{3}-7 x^{2}+4 x+19$, then $P(F(1000))=14,993,004,019$.) Thus, the sum of the digits of this expression is equal to $$ S\\left(P\\left(F\\left(10^{e}\\right)\\right)\\right)=9 e+k $$ for some constant $k$ depending only on $P$ and $F$, independent of $e$. But this will eventually hit a Fibonacci number by the second claim, contradiction. Remark. It is important to control the number of negative coefficients in the created polynomial. If one tries to use this approach on a polynomial $P$ with $m>0$ negative coefficients, then one would require that the Fibonacci sequence is surjective modulo $9 m$ for any $m>1$, which is not true: for example the Fibonacci sequence avoids all numbers congruent to $4 \\bmod 11($ and thus $4 \\bmod 99)$. In bases $b$ for which surjectivity modulo $b-1$ fails, the problem is false. For example, $P(x)=11 x+4$ will avoid all Fibonacci numbers if we take sum of digits in base 12, since that base-12 sum is necessarily $4(\\bmod 11)$, hence not a Fibonacci number."} +{"year":2019,"label":"7","problem":"Let $f: \\mathbb{Z} \\rightarrow\\left\\{1,2, \\ldots, 10^{100}\\right\\}$ be a function satisfying $$ \\operatorname{gcd}(f(x), f(y))=\\operatorname{gcd}(f(x), x-y) $$ for all integers $x$ and $y$. Show that there exist positive integers $m$ and $n$ such that $f(x)=\\operatorname{gcd}(m+x, n)$ for all integers $x$.","solution":" Let $\\mathcal{P}$ be the set of primes not exceeding $10^{100}$. For each $p \\in \\mathcal{P}$, let $e_{p}=\\max _{x} \\nu_{p}(f(x))$ and let $c_{p} \\in \\operatorname{argmax}_{x} \\nu_{p}(f(x))$. We show that this is good enough to compute all values of $x$, by looking at the exponent at each individual prime. Claim - For any $p \\in \\mathcal{P}$, we have $$ \\nu_{p}(f(x))=\\min \\left(\\nu_{p}\\left(x-c_{p}\\right), e_{p}\\right) $$ $$ \\operatorname{gcd}\\left(f\\left(c_{p}\\right), f(x)\\right)=\\operatorname{gcd}\\left(f\\left(c_{p}\\right), x-c_{p}\\right) $$ We then take $\\nu_{p}$ of both sides and recall $\\nu_{p}(f(x)) \\leq \\nu_{p}\\left(f\\left(c_{p}\\right)\\right)=e_{p}$; this implies the result. This essentially determines $f$, and so now we just follow through. Choose $n$ and $m$ such that $$ \\begin{aligned} n & =\\prod_{p \\in \\mathcal{P}} p^{e_{p}} \\\\ m & \\equiv-c_{p} \\quad\\left(\\bmod p^{e_{p}}\\right) \\quad \\forall p \\in \\mathcal{P} \\end{aligned} $$ the latter being possible by Chinese remainder theorem. Then, from the claim we have $$ \\begin{aligned} f(x) & =\\prod_{p \\in \\mathcal{P}} p^{\\nu_{p}(f(x))}=\\prod_{p \\mid n} p^{\\min \\left(\\nu_{p}\\left(x-c_{p}\\right), e_{p}\\right)} \\\\ & =\\prod_{p \\mid n} p^{\\min \\left(\\nu_{p}(x+m), \\nu_{p}(n)\\right)}=\\operatorname{gcd}(x+m, n) \\end{aligned} $$ for every $x \\in \\mathbb{Z}$, as desired. Remark. The functions $f(x)=x$ and $f(x)=|2 x-1|$ are examples satisfying the gcd equation (the latter always being strictly positive). Hence the hypothesis $f$ bounded cannot be dropped. Remark. The pair $(m, n)$ is essentially unique: every other pair is obtained by shifting $m$ by a multiple of $n$. Hence there is not really any choice in choosing $m$ and $n$."} +{"year":2019,"label":"8","problem":"Let $\\mathcal{S}$ be a set of 16 points in the plane, no three collinear. Let $\\chi(\\mathcal{S})$ denote the number of ways to draw 8 line segments with endpoints in $\\mathcal{S}$, such that no two drawn segments intersect, even at endpoints. Find the smallest possible value of $\\chi(\\mathcal{S})$ across all such $\\mathcal{S}$.","solution":" The answer is 1430 . In general, we prove that with $2 n$ points the answer is the $n^{\\text {th }}$ Catalan number $C_{n}=\\frac{1}{n+1}\\binom{2 n}{n}$. First of all, it is well-known that if $\\mathcal{S}$ is a convex $2 n$-gon, then $\\chi(\\mathcal{S})=C_{n}$. It remains to prove the lower bound. We proceed by (strong) induction on $n$, with the base case $n=0$ and $n=1$ clear. Suppose the statement is proven for $0,1, \\ldots, n$ and consider a set $\\mathcal{S}$ with $2(n+1)$ points. Let $P$ be a point on the convex hull of $\\mathcal{S}$, and label the other $2 n+1$ points $A_{1}, \\ldots, A_{2 n+1}$ in order of angle from $P$. Consider drawing a segment $\\overline{P A_{2 k+1}}$. This splits the $2 n$ remaining points into two halves $\\mathcal{U}$ and $\\mathcal{V}$, with $2 k$ and $2(n-k)$ points respectively. ![](https:\/\/cdn.mathpix.com\/cropped\/2024_11_19_f4f672a97b66743213a8g-23.jpg?height=581&width=815&top_left_y=1297&top_left_x=626) Note that by choice of $P$, no segment in $\\mathcal{U}$ can intersect a segment in $\\mathcal{V}$. By the inductive hypothesis, $$ \\chi(\\mathcal{U}) \\geq C_{k} \\quad \\text { and } \\quad \\chi(\\mathcal{V}) \\geq C_{n-k} $$ Thus, drawing $\\overline{P A_{2 k+1}}$, we have at least $C_{k} C_{n-k}$ ways to complete the drawing. Over all choices of $k$, we obtain $$ \\chi(\\mathcal{S}) \\geq C_{0} C_{n}+\\cdots+C_{n} C_{0}=C_{n+1} $$ as desired. Remark. It is possible to show directly from the lower bound proof that convex $2 n$-gons achieve the minimum: indeed, every inequality is sharp, and no segment $\\overline{P A_{2 k}}$ can be drawn (since this splits the rest of the points into two halves with an odd number of points, and no crossing segment can be drawn). Bobby Shen points out that in the case of 6 points, a regular pentagon with its center also achieves equality, so this is not the only equality case. Remark. The result that $\\chi(S) \\geq 1$ for all $S$ is known (consider the choice of 8 segments with smallest sum), and appeared on Putnam 1979. However, it does not seem that knowing this gives an advantage for this problem, since the answer is much larger than 1."} +{"year":2019,"label":"9","problem":"Let $A B C$ be a triangle with incenter $I$. Points $K$ and $L$ are chosen on segment $B C$ such that the incircles of $\\triangle A B K$ and $\\triangle A B L$ are tangent at $P$, and the incircles of $\\triangle A C K$ and $\\triangle A C L$ are tangent at $Q$. Prove that $I P=I Q$.","solution":" \u3010 First solution, mostly elementary (original). Let $I_{B}, J_{B}, I_{C}, J_{C}$ be the incenters of $\\triangle A B K, \\triangle A B L, \\triangle A C K, \\triangle A C L$ respectively. ![](https:\/\/cdn.mathpix.com\/cropped\/2024_11_19_f4f672a97b66743213a8g-25.jpg?height=515&width=1332&top_left_y=973&top_left_x=359) We begin with the following claim which does not depend on the existence of tangency points $P$ and $Q$. Claim - Lines $B C, I_{B} J_{C}, J_{B} I_{C}$ meet at a point $R$ (possibly at infinity). $$ A\\left(B I ; I_{B} J_{B}\\right)=A\\left(I C ; I_{C} J_{C}\\right) . $$ It follows $\\left(B I ; I_{B} J_{B}\\right)=\\left(I C ; I_{C} J_{C}\\right)=\\left(C I ; J_{C} I_{C}\\right)$. (One could also check directly that both cross ratios equal $\\frac{\\sin \\angle B A K \/ 2}{\\sin \\angle C A K \/ 2} \\div \\frac{\\sin \\angle B A L \/ 2}{\\sin \\angle C A L \/ 2}$, rather than using rotation.) Therefore, the concurrence follows from the so-called prism lemma on $\\overline{I B I_{B} J_{B}}$ and $\\overline{I C J_{C} I_{C}}$. Remark (Nikolai Beluhov). This result is known; it appears as 4.5.32 in Akopyan's Geometry in Figures. The cross ratio is not necessary to prove this claim: it can be proven by length chasing with circumscribed quadrilaterals. (The generalization mentioned later also admits a trig-free proof for the analogous step.) We now bring $P$ and $Q$ into the problem. Claim - Line $P Q$ also passes through $R$. $$ \\frac{I_{B} P}{P I} \\cdot \\frac{I Q}{Q J_{C}} \\cdot \\frac{J_{C} R}{R I_{B}}=-1 \\Longrightarrow I P=I Q $$ Remark (Author's comments on drawing the diagram). Drawing the diagram directly is quite difficult. If one draws $\\triangle A B C$ first, they must locate both $K$ and $L$, which likely involves some trial and error due to the complex interplay between the two points. There are alternative simpler ways. For example, one may draw $\\triangle A K L$ first; then the remaining points $B$ and $C$ are not related and the task is much simpler (though some trial and error is still required). In fact, by breaking symmetry, we may only require one application of guesswork. Start by drawing $\\triangle A B K$ and its incircle; then the incircle of $\\triangle A B L$ may be constructed, and so point $L$ may be drawn. Thus only the location of point $C$ needs to be guessed. I would be interested in a method to create a general diagram without any trial and error."} +{"year":2019,"label":"9","problem":"Let $A B C$ be a triangle with incenter $I$. Points $K$ and $L$ are chosen on segment $B C$ such that the incircles of $\\triangle A B K$ and $\\triangle A B L$ are tangent at $P$, and the incircles of $\\triangle A C K$ and $\\triangle A C L$ are tangent at $Q$. Prove that $I P=I Q$.","solution":" \u3010 Second solution, inversion (Nikolai Beluhov). As above, the lines $B C, I_{B} J_{C}, J_{B} I_{C}$ meet at some point $R$ (possibly at infinity). Let $\\omega_{1}, \\omega_{2}, \\omega_{3}, \\omega_{4}$ be the incircles of $\\triangle A B K$, $\\triangle A C L, \\triangle A B L$, and $\\triangle A C K$. Claim - There exists an inversion $\\iota$ at $R$ swapping $\\left\\{\\omega_{1}, \\omega_{2}\\right\\}$ and $\\left\\{\\omega_{3}, \\omega_{4}\\right\\}$. Claim - Circles $\\omega_{1}, \\omega_{2}, \\omega_{3}, \\omega_{4}$ share a common radical center. Then $k$ is actually orthogonal to $\\Omega, \\omega_{1}, \\omega_{3}$, so $k$ is fixed under $\\iota$ and $k$ is also orthogonal to $\\omega_{2}$ and $\\omega_{4}$. Thus the center of $k$ is the desired radical center. The desired statement immediately follows. Indeed, letting $S$ be the radical center, it follows that $\\overline{S P}$ and $\\overline{S Q}$ are the common internal tangents to $\\left\\{\\omega_{1}, \\omega_{3}\\right\\}$ and $\\left\\{\\omega_{2}, \\omega_{4}\\right\\}$. Since $S$ is the radical center, $S P=S Q$. In light of $\\angle S P I=\\angle S Q I=90^{\\circ}$, it follows that $I P=I Q$, as desired. Remark (Nikolai Beluhov). There exists a circle tangent to all four incircles, because circle $k$ is orthogonal to all four, and line $B C$ is tangent to all four; thus the inverse of line $B C$ in $k$ is a circle tangent to all four incircles. The amusing thing here is that Casey's theorem is completely unhelpful for proving this fact: all it can tell us is that there is a line or circle tangent to these incircles, and line $B C$ already satisfies this property. Remark (Generalization by Nikolai Beluhov). The following generalization holds: Let $A B C D$ be a quadrilateral circumscribed about a circle with center $I$. A line through $A$ meets $\\overrightarrow{B C}$ and $\\overrightarrow{D C}$ at $K$ and $L$; another line through $A$ meets $\\overrightarrow{B C}$ and $\\overrightarrow{D C}$ at $M$ and $N$. Suppose that the incircles of $\\triangle A B K$ and $\\triangle A B M$ are tangent at $P$, and the incircles of $\\triangle A C L$ and $\\triangle A C N$ are tangent at $Q$. Prove that $I P=I Q$."} diff --git a/USA_TST_TST/segmented/en-sols-TSTST-2020.jsonl b/USA_TST_TST/segmented/en-sols-TSTST-2020.jsonl new file mode 100644 index 0000000000000000000000000000000000000000..d24cdafe67eca9df22e9c91c81fbdb06fc338759 --- /dev/null +++ b/USA_TST_TST/segmented/en-sols-TSTST-2020.jsonl @@ -0,0 +1,10 @@ +{"year":2020,"label":"1","problem":"Let $a, b, c$ be fixed positive integers. There are $a+b+c$ ducks sitting in a circle, one behind the other. Each duck picks either rock, paper, or scissors, with $a$ ducks picking rock, $b$ ducks picking paper, and $c$ ducks picking scissors. A move consists of an operation of one of the following three forms: - If a duck picking rock sits behind a duck picking scissors, they switch places. - If a duck picking paper sits behind a duck picking rock, they switch places. - If a duck picking scissors sits behind a duck picking paper, they switch places. Determine, in terms of $a, b$, and $c$, the maximum number of moves which could take place, over all possible initial configurations.","solution":" The maximum possible number of moves is $\\max (a b, a c, b c)$. First, we prove this is best possible. We define a feisty triplet to be an unordered triple of ducks, one of each of rock, paper, scissors, such that the paper duck is between the rock and scissors duck and facing the rock duck, as shown. (There may be other ducks not pictured, but the orders are irrelevant.) ![](https:\/\/cdn.mathpix.com\/cropped\/2024_11_19_a8f70a0d08e0809deba0g-04.jpg?height=1100&width=1098&top_left_y=1500&top_left_x=479) Claim - The number of feisty triplets decreases by $c$ if a paper duck swaps places with a rock duck, and so on. Obviously the number of feisty triples is at most $a b c$ to start. Thus at most $\\max (a b, b c, c a)$ moves may occur, since the number of feisty triplets should always be nonnegative, at which point no moves are possible at all. To see that this many moves is possible, assume WLOG $a=\\min (a, b, c)$ and suppose we have $a$ rocks, $b$ papers, and $c$ scissors in that clockwise order. ![](https:\/\/cdn.mathpix.com\/cropped\/2024_11_19_a8f70a0d08e0809deba0g-05.jpg?height=1109&width=1052&top_left_y=750&top_left_x=505) Then, allow the scissors to filter through the papers while the rocks stay put. Each of the $b$ papers swaps with $c$ scissors, for a total of $b c=\\max (a b, a c, b c)$ swaps. Remark (Common errors). One small possible mistake: it is not quite k\u00f6sher to say that \"WLOG $a \\leq b \\leq c$ \" because the condition is not symmetric, only cyclic. Therefore in this solution we only assume $a=\\min (a, b, c)$. It is true here that every pair of ducks swaps at most once, and some solutions make use of this fact. However, this fact implicitly uses the fact that $a, b, c>0$ and is false without this hypothesis."} +{"year":2020,"label":"2","problem":"Let $A B C$ be a scalene triangle with incenter $I$. The incircle of $A B C$ touches $\\overline{B C}$, $\\overline{C A}, \\overline{A B}$ at points $D, E, F$, respectively. Let $P$ be the foot of the altitude from $D$ to $\\overline{E F}$, and let $M$ be the midpoint of $\\overline{B C}$. The rays $A P$ and $I P$ intersect the circumcircle of triangle $A B C$ again at points $G$ and $Q$, respectively. Show that the incenter of triangle $G Q M$ coincides with $D$.","solution":" Refer to the figure below. ![](https:\/\/cdn.mathpix.com\/cropped\/2024_11_19_a8f70a0d08e0809deba0g-06.jpg?height=798&width=803&top_left_y=906&top_left_x=632) Claim - The point $Q$ is the Miquel point of $B F E C$. Also, $\\overline{Q D}$ bisects $\\angle B Q C$. The spiral similarity mentioned then gives $\\frac{Q B}{B F}=\\frac{Q C}{C E}$, so $\\overline{Q D}$ bisects $\\angle B Q C$. Remark. The point $Q$ and its properties mentioned in the first claim have appeared in other references. See for example Canada 2007\/5, ELMO 2010\/6, HMMT 2016 T-10, USA TST 2017\/2, USA TST 2019\/6 for a few examples. Claim - We have $(Q G ; B C)=-1$, so in particular $\\overline{G D}$ bisects $\\angle B G C$. $$ -1=(A I ; E F) \\stackrel{Q}{=}(\\overline{A Q} \\cap \\overline{E F}, P ; E, F) \\stackrel{A}{=}(Q G ; B C) $$ The last statement follows from Apollonian circle, or more bluntly $\\frac{G B}{G C}=\\frac{Q B}{Q C}=\\frac{B D}{D C}$. Hence $\\overline{Q D}$ and $\\overline{G D}$ are angle bisectors of $\\angle B Q C$ and $\\angle B G C$. However, $\\overline{Q M}$ and $\\overline{Q G}$ are isogonal in $\\angle B Q C$ (as median and symmedian), and similarly for $\\angle B G C$, as desired."} +{"year":2020,"label":"3","problem":"We say a nondegenerate triangle whose angles have measures $\\theta_{1}, \\theta_{2}, \\theta_{3}$ is quirky if there exists integers $r_{1}, r_{2}, r_{3}$, not all zero, such that $$ r_{1} \\theta_{1}+r_{2} \\theta_{2}+r_{3} \\theta_{3}=0 $$ Find all integers $n \\geq 3$ for which a triangle with side lengths $n-1, n, n+1$ is quirky.","solution":" The answer is $n=3,4,5,7$. We first introduce a variant of the $k$ th Chebyshev polynomials in the following lemma (which is standard, and easily shown by induction). ## Lemma For each $k \\geq 0$ there exists $P_{k}(X) \\in \\mathbb{Z}[X]$, monic for $k \\geq 1$ and with degree $k$, such that $$ P_{k}\\left(X+X^{-1}\\right) \\equiv X^{k}+X^{-k} . $$ The first few are $P_{0}(X) \\equiv 2, P_{1}(X) \\equiv X, P_{2}(X) \\equiv X^{2}-2, P_{3}(X) \\equiv X^{3}-3 X$. Suppose the angles of the triangle are $\\alpha<\\beta<\\gamma$, so the law of cosines implies that $$ 2 \\cos \\alpha=\\frac{n+4}{n+1} \\quad \\text { and } \\quad 2 \\cos \\gamma=\\frac{n-4}{n-1} $$ Claim - The triangle is quirky iff there exists $r, s \\in \\mathbb{Z}_{\\geq 0}$ not both zero such that $$ \\cos (r \\alpha)= \\pm \\cos (s \\gamma) \\quad \\text { or equivalently } \\quad P_{r}\\left(\\frac{n+4}{n+1}\\right)= \\pm P_{s}\\left(\\frac{n-4}{n-1}\\right) $$ If $r=0$, then by rational root theorem on $P_{s}(X) \\pm 2$ it follows $\\frac{n-4}{n-1}$ must be an integer which occurs only when $n=4$ (recall $n \\geq 3$ ). Similarly we may discard the case $s=0$. Thus in what follows assume $n \\neq 4$ and $r, s>0$. Then, from the fact that $P_{r}$ and $P_{s}$ are nonconstant monic polynomials, we find ## Corollary If $n \\neq 4$ works, then when $\\frac{n+4}{n+1}$ and $\\frac{n-4}{n-1}$ are written as fractions in lowest terms, the denominators have the same set of prime factors. But $\\operatorname{gcd}(n+1, n-1)$ divides 2 , and $\\operatorname{gcd}(n+4, n+1), \\operatorname{gcd}(n-4, n-1)$ divide 3 . So we only have three possibilities: - $n+1=2^{u}$ and $n-1=2^{v}$ for some $u, v \\geq 0$. This is only possible if $n=3$. Here $2 \\cos \\alpha=\\frac{7}{4}$ and $2 \\cos \\gamma=-\\frac{1}{2}$, and indeed $P_{2}(-1 \/ 2)=-7 \/ 4$. - $n+1=3 \\cdot 2^{u}$ and $n-1=2^{v}$ for some $u, v \\geq 0$, which implies $n=5$. Here $2 \\cos \\alpha=\\frac{3}{2}$ and $2 \\cos \\gamma=\\frac{1}{4}$, and indeed $P_{2}(3 \/ 2)=1 \/ 4$. - $n+1=2^{u}$ and $n-1=3 \\cdot 2^{v}$ for some $u, v \\geq 0$, which implies $n=7$. Here $2 \\cos \\alpha=\\frac{11}{8}$ and $2 \\cos \\gamma=\\frac{1}{2}$, and indeed $P_{3}(1 \/ 2)=-11 \/ 8$. Finally, $n=4$ works because the triangle is right, completing the solution. Remark (Major generalization due to Luke Robitaille). In fact one may find all quirky triangles whose sides are integers in arithmetic progression. Indeed, if the side lengths of the triangle are $x-y, x, x+y$ with $\\operatorname{gcd}(x, y)=1$ then the problem becomes $$ P_{r}\\left(\\frac{x+4 y}{x+y}\\right)= \\pm P_{s}\\left(\\frac{x-4 y}{x-y}\\right) $$ and so in the same way as before, we ought to have $x+y$ and $x-y$ are both of the form $3 \\cdot 2^{*}$ unless $r s=0$. This time, when $r s=0$, we get the extra solutions $(1,0)$ and $(5,2)$. For $r s \\neq 0$, by triangle inequality, we have $x-y \\leq x+y<3(x-y)$, and $\\min \\left(\\nu_{2}(x-\\right.$ $\\left.y), \\nu_{2}(x+y)\\right) \\leq 1$, so it follows one of $x-y$ or $x+y$ must be in $\\{1,2,3,6\\}$. An exhaustive check then leads to $$ (x, y) \\in\\{(3,1),(5,1),(7,1),(11,5)\\} \\cup\\{(1,0),(5,2),(4,1)\\} $$ In conclusion the equilateral triangle, $3-5-7$ triangle (which has a $120^{\\circ}$ angle) and $6-11-16$ triangle (which satisfies $B=3 A+4 C$ ) are exactly the new quirky triangles (up to similarity) whose sides are integers in arithmetic progression."} +{"year":2020,"label":"4","problem":"Find all pairs of positive integers $(a, b)$ satisfying the following conditions: (i) $a$ divides $b^{4}+1$, (ii) $b$ divides $a^{4}+1$, (iii) $\\lfloor\\sqrt{a}\\rfloor=\\lfloor\\sqrt{b}\\rfloor$.","solution":" Obviously, $\\operatorname{gcd}(a, b)=1$, so the problem conditions imply $$ a b \\mid(a-b)^{4}+1 $$ since each of $a$ and $b$ divide the right-hand side. We define $$ k \\stackrel{\\text { def }}{=} \\frac{(b-a)^{4}+1}{a b} . $$ Claim (Size estimate) \u2014 We must have $k \\leq 16$. $$ \\begin{aligned} a b & \\geq n^{2}\\left(n^{2}+1\\right) \\geq n^{4}+1 \\\\ (b-a)^{4}+1 & \\leq(2 n)^{4}+1=16 n^{4}+1 \\end{aligned} $$ which shows $k \\leq 16$. Claim (Orders argument) - In fact, $k=1$. Thus $k$ is odd. However, every odd prime divisor of $(b-a)^{4}+1$ is congruent to 1 $(\\bmod 8)$ and is thus at least 17 , so $k=1$ or $k \\geq 17$. It follows that $k=1$. At this point, we have reduced to solving $$ a b=(b-a)^{4}+1 $$ $$ a^{2}-d a-\\left(d^{4}+1\\right)=0 $$ The discriminant $d^{2}+4\\left(d^{4}+1\\right)=4 d^{4}+d^{2}+4$ must be a perfect square. - The cases $d=0$ and $d=1$ lead to pairs $(1,1)$ and $(1,2)$. - If $d \\geq 2$, then we can sandwich $$ \\left(2 d^{2}\\right)^{2}<4 d^{4}+d^{2}+4<4 d^{4}+4 d^{2}+1=\\left(2 d^{2}+1\\right)^{2} $$ so the discriminant is not a square. Remark (Author remarks on origin). This comes from the problem of the existence of a pair of elliptic curves over $\\mathbb{F}_{a}, \\mathbb{F}_{b}$ respectively, such that the number of points on one is the field size of the other. The bound $n^{2} \\leq a, b<(n+1)^{2}$ is the Hasse bound. The divisibility conditions correspond to asserting that the embedding degree of each curve is 8 , so that they are pairing friendly. In this way, the problem is essentially the key result of https:\/\/arxiv.org\/pdf\/1803.02067.pdf, shown in Proposition 3."} +{"year":2020,"label":"5","problem":"Let $\\mathbb{N}^{2}$ denote the set of ordered pairs of positive integers. A finite subset $S$ of $\\mathbb{N}^{2}$ is stable if whenever $(x, y)$ is in $S$, then so are all points $\\left(x^{\\prime}, y^{\\prime}\\right)$ of $\\mathbb{N}^{2}$ with both $x^{\\prime} \\leq x$ and $y^{\\prime} \\leq y$. Prove that if $S$ is a stable set, then among all stable subsets of $S$ (including the empty set and $S$ itself), at least half of them have an even number of elements.","solution":" Suppose $|S| \\geq 2$. For any $p \\in S$, let $R(p)$ denote the stable rectangle with upper-right corner $p$. We say such $p$ is pivotal if $p+(1,1) \\notin S$ and $|R(p)|$ is even. ![](https:\/\/cdn.mathpix.com\/cropped\/2024_11_19_a8f70a0d08e0809deba0g-12.jpg?height=455&width=818&top_left_y=1069&top_left_x=622) Claim - If $|S| \\geq 2$, then a pivotal $p$ always exists. - If it has length at least 2 , one of the two rightmost points in it is pivotal. - Otherwise, the top row has length 1. Now either the top point or the point below it (which exists as $|S| \\geq 2$ ) is pivotal. We describe how to complete the induction, given some pivotal $p \\in S$. There is a partition $$ S=R(p) \\sqcup S_{1} \\sqcup S_{2} $$ where $S_{1}$ and $S_{2}$ are the sets of points in $S$ above and to the right of $p$ (possibly empty). Claim - The desired inequality holds for stable subsets containing $p$. Since $|R(p)|$ is even, exactly $E_{1} E_{2}+O_{1} O_{2}$ stable subsets containing $p$ are even, and exactly $E_{1} O_{2}+E_{2} O_{1}$ are odd. As $E_{1} \\geq O_{1}$ and $E_{2} \\geq O_{2}$ by inductive hypothesis, we obtain $E_{1} E_{2}+O_{1} O_{2} \\geq E_{1} O_{2}+E_{2} O_{1}$ as desired. By the inductive hypothesis, the desired inequality also holds for stable subsets not containing $p$, so we are done."} +{"year":2020,"label":"6","problem":"Let $A, B, C, D$ be four points such that no three are collinear and $D$ is not the orthocenter of triangle $A B C$. Let $P, Q, R$ be the orthocenters of $\\triangle B C D, \\triangle C A D$, $\\triangle A B D$, respectively. Suppose that lines $A P, B Q, C R$ are pairwise distinct and are concurrent. Show that the four points $A, B, C, D$ lie on a circle.","solution":" Let $T$ be the concurrency point, and let $H$ be the orthocenter of $\\triangle A B C$. ![](https:\/\/cdn.mathpix.com\/cropped\/2024_11_19_a8f70a0d08e0809deba0g-14.jpg?height=593&width=806&top_left_y=863&top_left_x=631) Claim (Key claim) - $T$ is the midpoint of $\\overline{A P}, \\overline{B Q}, \\overline{C R}, \\overline{D H}$, and $D$ is the orthocenter of $\\triangle P Q R$. By symmetric reasoning, we get that $A Q C P B R$ is a hexagon with opposite sides parallel and concurrent diagonals as $\\overline{A P}, \\overline{B Q}, \\overline{C R}$ meet at $T$. This implies that the hexagon is centrally symmetric about $T$; indeed $$ \\frac{A T}{T P}=\\frac{T Q}{B T}=\\frac{C T}{T R}=\\frac{T P}{A T} $$ so all the ratios are equal to +1 . Next, $\\overline{P D} \\perp \\overline{B C} \\| \\overline{Q R}$, so by symmetry we get $D$ is the orthocenter of $\\triangle P Q R$. This means that $T$ is the midpoint of $\\overline{D H}$ as well. ## Corollary The configuration is now symmetric: we have four points $A, B, C, D$, and their reflections in $T$ are four orthocenters $P, Q, R, H$. Let $S$ be the centroid of $\\{A, B, C, D\\}$, and let $O$ be the reflection of $T$ in $S$. We are ready to conclude: Claim - $A, B, C, D$ are equidistant from $O$. Then $T^{\\prime}$ is the midpoint of $\\overline{A^{\\prime} D^{\\prime}}$, so $S^{\\prime}=\\frac{1}{4}\\left(A^{\\prime}+D^{\\prime}+B+C\\right)$ gives that $O^{\\prime}$ is the midpoint of $\\overline{B C}$. Thus $O B=O C$ and we're done."} +{"year":2020,"label":"7","problem":"Find all nonconstant polynomials $P(z)$ with complex coefficients for which all complex roots of the polynomials $P(z)$ and $P(z)-1$ have absolute value 1.","solution":" The answer is $P(x)$ should be a polynomial of the form $P(x)=\\lambda x^{n}-\\mu$ where $|\\lambda|=|\\mu|$ and $\\operatorname{Re} \\mu=-\\frac{1}{2}$. One may check these all work; let's prove they are the only solutions. \\l First approach (Evan Chen). We introduce the following notations: $$ \\begin{aligned} P(x) & =c_{n} x^{n}+c_{n-1} x^{n-1}+\\cdots+c_{1} x+c_{0} \\\\ & =c_{n}\\left(x+\\alpha_{1}\\right) \\ldots\\left(x+\\alpha_{n}\\right) \\\\ P(x)-1 & =c_{n}\\left(x+\\beta_{1}\\right) \\ldots\\left(x+\\beta_{n}\\right) \\end{aligned} $$ By taking conjugates, $$ \\begin{aligned} \\left(x+\\alpha_{1}\\right) \\cdots\\left(x+\\alpha_{n}\\right) & =\\left(x+\\beta_{1}\\right) \\cdots\\left(x+\\beta_{n}\\right)+c_{n}^{-1} \\\\ \\Longrightarrow\\left(x+\\frac{1}{\\alpha_{1}}\\right) \\cdots\\left(x+\\frac{1}{\\alpha_{n}}\\right) & =\\left(x+\\frac{1}{\\beta_{1}}\\right) \\cdots\\left(x+\\frac{1}{\\beta_{n}}\\right)+\\left(\\overline{c_{n}}\\right)^{-1} \\end{aligned} $$ The equation $(\\boldsymbol{\\oplus})$ is the main player: Claim - We have $c_{k}=0$ for all $k=1, \\ldots, n-1$. $$ \\frac{c_{n-k}}{\\prod_{i} \\alpha_{i}}=\\frac{c_{n-k}}{\\prod_{i} \\beta_{i}} $$ but $\\prod_{i} \\alpha_{i}-\\prod_{i} \\beta_{i}=\\frac{1}{c_{n}} \\neq 0$. Hence $c_{k}=0$. It follows that $P(x)$ must be of the form $P(x)=\\lambda x^{n}-\\mu$, so that $P(x)=\\lambda x^{n}-(\\mu+1)$. This requires $|\\mu|=|\\mu+1|=|\\lambda|$ which is equivalent to the stated part."} +{"year":2020,"label":"7","problem":"Find all nonconstant polynomials $P(z)$ with complex coefficients for which all complex roots of the polynomials $P(z)$ and $P(z)-1$ have absolute value 1.","solution":" The answer is $P(x)$ should be a polynomial of the form $P(x)=\\lambda x^{n}-\\mu$ where $|\\lambda|=|\\mu|$ and $\\operatorname{Re} \\mu=-\\frac{1}{2}$. One may check these all work; let's prove they are the only solutions. II Second approach (from the author). We let $A=P$ and $B=P-1$ to make the notation more symmetric. We will as before show that $A$ and $B$ have all coefficients equal to zero other than the leading and constant coefficient; the finish is the same. First, we rule out double roots. Claim - Neither $A$ nor $B$ have double roots. Let $\\omega=e^{2 \\pi i \/ n}$, let $a_{1}, \\ldots, a_{n}$ be the roots of $A$, and let $b_{1}, \\ldots, b_{n}$ be the roots of $B$. For each $k$, let $A_{k}$ and $B_{k}$ be the points in the complex plane corresponding to $a_{k}$ and $b_{k}$ \u3002 Claim (Main claim) - For any $i$ and $j, \\frac{a_{i}}{a_{j}}$ is a power of $\\omega$. $$ \\frac{a_{i}-b_{1}}{a_{j}-b_{1}} \\cdots \\frac{a_{i}-b_{n}}{a_{j}-b_{n}}=\\frac{B\\left(a_{i}\\right)}{B\\left(a_{j}\\right)}=\\frac{A\\left(a_{i}\\right)-1}{A\\left(a_{j}\\right)-1}=\\frac{0-1}{0-1}=1 $$ Since the points $A_{i}, A_{j}, B_{k}$ all lie on the unit circle, interpreting the left-hand side geometrically gives $$ \\measuredangle A_{i} B_{1} A_{j}+\\cdots+\\measuredangle A_{i} B_{n} A_{j}=0 \\Longrightarrow n \\widehat{A_{i} A_{j}}=0 $$ where angles are directed modulo $180^{\\circ}$ and arcs are directed modulo $360^{\\circ}$. This implies that $\\frac{a_{i}}{a_{j}}$ is a power of $\\omega$. Now the finish is easy: since $a_{1}, \\ldots, a_{n}$ are all different, they must be $a_{1} \\omega^{0}, \\ldots, a_{1} \\omega^{n-1}$ in some order; this shows that $A$ is a multiple of $x^{n}-a_{1}^{n}$, as needed."} +{"year":2020,"label":"8","problem":"For every positive integer $N$, let $\\sigma(N)$ denote the sum of the positive integer divisors of $N$. Find all integers $m \\geq n \\geq 2$ satisfying $$ \\frac{\\sigma(m)-1}{m-1}=\\frac{\\sigma(n)-1}{n-1}=\\frac{\\sigma(m n)-1}{m n-1} . $$","solution":" The answer is that $m$ and $n$ should be powers of the same prime number. These all work because for a prime power we have $$ \\frac{\\sigma\\left(p^{e}\\right)-1}{p^{e}-1}=\\frac{\\left(1+p+\\cdots+p^{e}\\right)-1}{p^{e}-1}=\\frac{p\\left(1+\\cdots+p^{e-1}\\right)}{p^{e}-1}=\\frac{p}{p-1} $$ So we now prove these are the only ones. Let $\\lambda$ be the common value of the three fractions. $$ \\begin{aligned} \\sigma(m n) & \\geq \\sigma(m)+m \\sigma(n)-m \\\\ & =(\\lambda m-\\lambda+1)+m(\\lambda n-\\lambda+1)-m \\\\ & =\\lambda m n-\\lambda+1 \\end{aligned} $$ and so equality holds. Thus these are all the divisors of $m n$, for a count of $d(m)+d(n)-$ 1. Claim - If $d(m n)=d(m)+d(n)-1$ and $\\min (m, n) \\geq 2$, then $m$ and $n$ are powers of the same prime. Remark (Nikolai Beluhov). Here is a completion not relying on $|A \\cdot B|=|A|+|B|-1$. By the above arguments, we see that every divisor of $m n$ is either a divisor of $n$, or $n$ times a divisor of $m$. Now suppose that some prime $p \\mid m$ but $p \\nmid n$. Then $p \\mid m n$ but $p$ does not appear in the above classification, a contradiction. By symmetry, it follows that $m$ and $n$ have the same prime divisors. Now suppose we have different primes $p \\mid m$ and $q \\mid n$. Write $\\nu_{p}(m)=\\alpha$ and $\\nu_{p}(n)=\\beta$. Then $p^{\\alpha+\\beta} \\mid m n$, but it does not appear in the above characterization, a contradiction. Thus, $m$ and $n$ are powers of the same prime. Remark (Comments on the function in the problem). Let $f(n)=\\frac{\\sigma(n)-1}{n-1}$. Then $f$ is not really injective even outside the above solution; for example, we have $f\\left(6 \\cdot 11^{k}\\right)=\\frac{11}{5}$ for all $k$, plus sporadic equivalences like $f(14)=f(404)$, as pointed out by one reviewer during test-solving. This means that both relations should be used at once, not independently. Remark (Authorship remarks). Ankan gave the following story for how he came up with the problem while thinking about so-called almost perfect numbers. I was in some boring talk when I recalled a conjecture that if $\\sigma(n)=2 n-1$, then $n$ is a power of 2 . For some reason (divine intervention, maybe) I had the double idea of (1) seeing whether $m, n, m n$ all almost perfect implies $m, n$ powers of 2 , and (2) trying the naive divisor bound to resolve this. Through sheer dumb luck this happened to work out perfectly. I thought this was kinda cool but I felt that I hadn't really unlocked a lot of the potential this idea had: then I basically tried to find the \"general situation\" which allows for this manipulation, and was amazed that it led to such a striking statement."} +{"year":2020,"label":"9","problem":"Ten million fireflies are glowing in $\\mathbb{R}^{3}$ at midnight. Some of the fireflies are friends, and friendship is always mutual. Every second, one firefly moves to a new position so that its distance from each one of its friends is the same as it was before moving. This is the only way that the fireflies ever change their positions. No two fireflies may ever occupy the same point. Initially, no two fireflies, friends or not, are more than a meter away. Following some finite number of seconds, all fireflies find themselves at least ten million meters away from their original positions. Given this information, find the greatest possible number of friendships between the fireflies.","solution":" In general, we show that when $n \\geq 70$, the answer is $f(n)=\\left\\lfloor\\frac{n^{2}}{3}\\right\\rfloor$. Construction: Choose three pairwise parallel lines $\\ell_{A}, \\ell_{B}, \\ell_{C}$ forming an infinite equilateral triangle prism (with side larger than 1). Split the $n$ fireflies among the lines as equally as possible, and say that two fireflies are friends iff they lie on different lines. To see this works: 1. Reflect $\\ell_{A}$ and all fireflies on $\\ell_{A}$ in the plane containing $\\ell_{B}$ and $\\ell_{C}$. 2. Reflect $\\ell_{B}$ and all fireflies on $\\ell_{B}$ in the plane containing $\\ell_{C}$ and $\\ell_{A}$. 3. Reflect $\\ell_{C}$ and all fireflies on $\\ell_{C}$ in the plane containing $\\ell_{A}$ and $\\ell_{B}$. Let $g(n)$ be the answer, given that there exist four pairwise friends (say $a, b, c, d$ ). Note that for a firefly to move, all its friends must be coplanar. Claim (No coplanar $K_{4}$ ) \u2014 We can't have four coplanar fireflies which are pairwise friends. Claim (Key claim \u2014 tetrahedrons don't share faces often) \u2014 There are at most 12 fireflies $e$ which are friends with at least three of $a, b, c, d$. WLOG, will assume that $e$ is friends with $a, b, c$. Then $e$ will always be located at one of two points $E_{1}$ and $E_{2}$ relative to $A B C$, such that $E_{1} A B C$ and $E_{2} A B C$ are two congruent tetrahedrons with fixed shape. We note that points $D, E_{1}$, and $E_{2}$ are all different: clearly $D \\neq E_{1}$ and $E_{1} \\neq E_{2}$. (If $D=E_{2}$, then some fireflies won't be able to move.) Consider the moment where firefly $a$ moves. Its friends must be coplanar at that time, so one of $E_{1}, E_{2}$ lies in plane $B C D$. Similar reasoning holds for planes $A C D$ and $A B D$. So, WLOG $E_{1}$ lies on both planes $B C D$ and $A C D$. Then $E_{1}$ lies on line $C D$, and $E_{2}$ lies in plane $A B D$. This uniquely determines $\\left(E_{1}, E_{2}\\right)$ relative to $A B C D$ : - $E_{1}$ is the intersection of line $C D$ with the reflection of plane $A B D$ in plane $A B C$. - $E_{2}$ is the intersection of plane $A B D$ with the reflection of line $C D$ in plane $A B C$. Accounting for WLOGs, there are at most 12 possibilities for the set $\\left\\{E_{1}, E_{2}\\right\\}$, and thus at most 12 possibilities for $E$. (It's not possible for both elements of one pair $\\left\\{E_{1}, E_{2}\\right\\}$ to be occupied, because then they couldn't move.) Thus, the number of friendships involving exactly one of $a, b, c, d$ is at most $(n-16)$. $2+12 \\cdot 3=2 n+4$, so removing these four fireflies gives $$ g(n) \\leq 6+(2 n+4)+\\max \\{f(n-4), g(n-4)\\} $$ $$ g(n) \\leq \\max \\{f(n),(2 n+10)+g(n-4)\\} \\quad \\forall n \\geq 24 $$ By iterating the above inequality, we get $$ \\begin{aligned} g(n) \\leq \\max \\{f(n),(2 n+10) & +(2(n-4)+10) \\\\ & +\\cdots+(2(n-4 r)+10)+g(n-4 r-4)\\} \\end{aligned} $$ where $r$ satisfies $n-4 r-4<24 \\leq n-4 r$. Now $$ \\begin{aligned} & (2 n+10)+(2(n-4)+10)+\\cdots+(2(n-4 r)+10)+g(n-4 r-4) \\\\ = & (r+1)(2 n-4 r+10)+g(n-4 r-4) \\\\ \\leq & \\left(\\frac{n}{4}-5\\right)(n+37)+\\binom{24}{2} . \\end{aligned} $$ This is less than $f(n)$ for $n \\geq 70$, which concludes the solution. Remark. There are positive integers $n$ such that it is possible to do better than $f(n)$ friendships. For instance, $f(5)=8$, whereas five fireflies $a, b, c, d$, and $e$ as in the proof of the Lemma ( $E_{1}$ being the intersection point of line $C D$ with the reflection of plane $(A B D)$ in plane $(A B C), E_{2}$ being the intersection point of plane $(A B D)$ with the reflection of line $C D$ in plane $(A B C)$, and tetrahedron $A B C D$ being sufficiently arbitrary that points $E_{1}$ and $E_{2}$ exist and points $D, E_{1}$, and $E_{2}$ are pairwise distinct) give a total of nine friendships. Remark (Author comments). It is natural to approach the problem by looking at the two-dimensional version first. In two dimensions, the following arrangement suggests itself almost immediately: We distribute all fireflies as equally as possible among two parallel lines, and two fireflies are friends if and only if they are on different lines. Similarly to the three-dimensional version, this attains the greatest possible number of friendships for all sufficiently large $n$, though not for all $n$. For instance, at least one friendlier arrangements exists for $n=4$, similarly to the above friendlier arrangement for $n=5$ in three dimensions. This observation strongly suggests that in three dimensions we should distribute the fireflies as equally as possible among two parallel planes, and that two fireflies should be friends if and only if they are on different planes. It was a great surprise for me to discover that this arrangement does not in fact give the correct answer! Remark. On the other hand, Ankan Bhattacharya gives the following reasoning as to why the answer should not be that surprising: I think the answer $\\left(10^{14}-1\\right) \/ 3$ is quite natural if you realize that $(n \/ 2)^{2}$ is probably optimal in 2D and $\\binom{n}{2}$ is optimal in super high dimensions (i.e. around $n$ ). So going from dimension 2 to 3 should increase the answer (and indeed it does)."} diff --git a/USA_TST_TST/segmented/en-sols-TSTST-2021.jsonl b/USA_TST_TST/segmented/en-sols-TSTST-2021.jsonl new file mode 100644 index 0000000000000000000000000000000000000000..01e35a00a5cad161af881e1a9ce9c212f0892033 --- /dev/null +++ b/USA_TST_TST/segmented/en-sols-TSTST-2021.jsonl @@ -0,0 +1,28 @@ +{"year":2021,"label":"1","problem":"Let $A B C D$ be a quadrilateral inscribed in a circle with center $O$. Points $X$ and $Y$ lie on sides $A B$ and $C D$, respectively. Suppose the circumcircles of $A D X$ and $B C Y$ meet line $X Y$ again at $P$ and $Q$, respectively. Show that $O P=O Q$.","solution":" \u300e First solution, angle chasing only (Ankit Bisain). Let lines $B Q$ and $D P$ meet $(A B C D)$ again at $D^{\\prime}$ and $B^{\\prime}$, respectively. ![](https:\/\/cdn.mathpix.com\/cropped\/2024_11_19_148cb373ffbe5370bbc6g-03.jpg?height=618&width=737&top_left_y=1047&top_left_x=668) Then $B B^{\\prime} \\| P X$ and $D D^{\\prime} \\| Q Y$ by Reim's theorem. Segments $B B^{\\prime}, D D^{\\prime}$, and $P Q$ share a perpendicular bisector which passes through $O$, so $O P=O Q$."} +{"year":2021,"label":"1","problem":"Let $A B C D$ be a quadrilateral inscribed in a circle with center $O$. Points $X$ and $Y$ lie on sides $A B$ and $C D$, respectively. Suppose the circumcircles of $A D X$ and $B C Y$ meet line $X Y$ again at $P$ and $Q$, respectively. Show that $O P=O Q$.","solution":" \u3010 Second solution via isosceles triangles (from contestants). Let $T=\\overline{B Q} \\cap \\overline{D P}$. ![](https:\/\/cdn.mathpix.com\/cropped\/2024_11_19_148cb373ffbe5370bbc6g-03.jpg?height=455&width=732&top_left_y=1897&top_left_x=665) Note that $P Q T$ is isosceles because $$ \\measuredangle P Q T=\\measuredangle Y Q B=\\measuredangle B C D=\\measuredangle B A D=\\measuredangle X P D=\\measuredangle T P Q $$ Then $(B O D T)$ is cyclic because $$ \\measuredangle B O D=2 \\measuredangle B C D=\\measuredangle P Q T+\\measuredangle T P Q=\\measuredangle B T D . $$ Since $B O=O D, \\overline{T O}$ is an angle bisector of $\\measuredangle B T D$. Since $\\triangle P Q T$ is isosceles, $\\overline{T O} \\perp \\overline{P Q}$, so $O P=O Q$."} +{"year":2021,"label":"1","problem":"Let $A B C D$ be a quadrilateral inscribed in a circle with center $O$. Points $X$ and $Y$ lie on sides $A B$ and $C D$, respectively. Suppose the circumcircles of $A D X$ and $B C Y$ meet line $X Y$ again at $P$ and $Q$, respectively. Show that $O P=O Q$.","solution":" \u092c Third solution using a parallelogram (from contestants). Let $(B C Y)$ meet $\\overline{A B}$ again at $W$ and let $(A D X)$ meet $\\overline{C D}$ again at $Z$. Additionally, let $O_{1}$ be the center of $(A D X)$ and $O_{2}$ be the center of $(B C Y)$. ![](https:\/\/cdn.mathpix.com\/cropped\/2024_11_19_148cb373ffbe5370bbc6g-04.jpg?height=598&width=707&top_left_y=569&top_left_x=686) Note that $(W X Y Z)$ is cyclic since $$ \\measuredangle X W Y+\\measuredangle Y Z X=\\measuredangle Y W B+\\measuredangle X Z D=\\measuredangle Y C B+\\measuredangle X A D=0^{\\circ} $$ so let $O^{\\prime}$ be the center of ( $W X Y Z$ ). Since $\\overline{A D} \\| \\overline{W Y}$ and $\\overline{B C} \\| \\overline{X Z}$ by Reim's theorem, $O O_{1} O^{\\prime} O_{2}$ is a parallelogram. To finish the problem, note that projecting $O_{1}, O_{2}$, and $O^{\\prime}$ onto $\\overline{X Y}$ gives the midpoints of $\\overline{P X}, \\overline{Q Y}$, and $\\overline{X Y}$. Since $O O_{1} O^{\\prime} O_{2}$ is a parallelogram, projecting $O$ onto $\\overline{X Y}$ must give the midpoint of $\\overline{P Q}$, so $O P=O Q$."} +{"year":2021,"label":"1","problem":"Let $A B C D$ be a quadrilateral inscribed in a circle with center $O$. Points $X$ and $Y$ lie on sides $A B$ and $C D$, respectively. Suppose the circumcircles of $A D X$ and $B C Y$ meet line $X Y$ again at $P$ and $Q$, respectively. Show that $O P=O Q$.","solution":" \u092c Fourth solution using congruent circles (from contestants). Let the angle bisector of $\\measuredangle B O D$ meet $\\overline{X Y}$ at $K$. ![](https:\/\/cdn.mathpix.com\/cropped\/2024_11_19_148cb373ffbe5370bbc6g-04.jpg?height=569&width=652&top_left_y=1783&top_left_x=702) Then $(B Q O K)$ is cyclic because $\\measuredangle K O D=\\measuredangle B A D=\\measuredangle K P D$, and $(D O P K)$ is cyclic similarly. By symmetry over $K O$, these circles have the same radius $r$, so $$ O P=2 r \\sin \\angle O K P=2 r \\sin \\angle O K Q=O Q $$ by the Law of Sines."} +{"year":2021,"label":"1","problem":"Let $A B C D$ be a quadrilateral inscribed in a circle with center $O$. Points $X$ and $Y$ lie on sides $A B$ and $C D$, respectively. Suppose the circumcircles of $A D X$ and $B C Y$ meet line $X Y$ again at $P$ and $Q$, respectively. Show that $O P=O Q$.","solution":" \u3010 Fifth solution by ratio calculation (from contestants). Let $\\overline{X Y}$ meet $(A B C D)$ at $X^{\\prime}$ and $Y^{\\prime}$. ![](https:\/\/cdn.mathpix.com\/cropped\/2024_11_19_148cb373ffbe5370bbc6g-05.jpg?height=618&width=715&top_left_y=362&top_left_x=682) Since $\\measuredangle Y^{\\prime} B D=\\measuredangle P X^{\\prime} D$ and $\\measuredangle B Y^{\\prime} D=\\measuredangle B A D=\\measuredangle X^{\\prime} P D$, $$ \\triangle B Y^{\\prime} D \\sim \\triangle X P^{\\prime} D \\Longrightarrow P X^{\\prime}=B Y^{\\prime} \\cdot \\frac{D X^{\\prime}}{B D} $$ Similarly, $$ \\triangle B X^{\\prime} D \\sim \\triangle B Q Y^{\\prime} \\Longrightarrow Q Y^{\\prime}=D X^{\\prime} \\cdot \\frac{B Y^{\\prime}}{B D} $$ Thus $P X^{\\prime}=Q Y^{\\prime}$, which gives $O P=O Q$."} +{"year":2021,"label":"1","problem":"Let $A B C D$ be a quadrilateral inscribed in a circle with center $O$. Points $X$ and $Y$ lie on sides $A B$ and $C D$, respectively. Suppose the circumcircles of $A D X$ and $B C Y$ meet line $X Y$ again at $P$ and $Q$, respectively. Show that $O P=O Q$.","solution":" I Sixth solution using radical axis (from author). Without loss of generality, assume $\\overline{A D} \\nVdash \\overline{B C}$, as this case holds by continuity. Let $(B C Y)$ meet $\\overline{A B}$ again at $W$, let $(A D X)$ meet $\\overline{C D}$ again at $Z$, and let $\\overline{W Z}$ meet $(A D X)$ and $(B C Y)$ again at $R$ and $S$. ![](https:\/\/cdn.mathpix.com\/cropped\/2024_11_19_148cb373ffbe5370bbc6g-05.jpg?height=610&width=712&top_left_y=1574&top_left_x=678) Note that $(W X Y Z)$ is cyclic since $$ \\measuredangle X W Y+\\measuredangle Y Z X=\\measuredangle Y W B+\\measuredangle X Z D=\\measuredangle Y C B+\\measuredangle X A D=0^{\\circ} $$ and $(P Q R S)$ is cyclic since $$ \\measuredangle P Q S=\\measuredangle Y Q S=\\measuredangle Y W S=\\measuredangle P X Z=\\measuredangle P R Z=\\measuredangle S R P $$ Additionally, $\\overline{A D} \\| \\overline{P R}$ since $$ \\measuredangle D A X+\\measuredangle A X P+\\measuredangle X P R=\\measuredangle Y W X+\\measuredangle W X Y+\\measuredangle X Y W=0^{\\circ}, $$ and $\\overline{B C} \\| \\overline{S Q}$ similarly. Lastly, $(A B C D)$ and $(P Q R S)$ are concentric; if not, using the radical axis theorem twice shows that their radical axis must be parallel to both $\\overline{A D}$ and $\\overline{B C}$, contradiction."} +{"year":2021,"label":"1","problem":"Let $A B C D$ be a quadrilateral inscribed in a circle with center $O$. Points $X$ and $Y$ lie on sides $A B$ and $C D$, respectively. Suppose the circumcircles of $A D X$ and $B C Y$ meet line $X Y$ again at $P$ and $Q$, respectively. Show that $O P=O Q$.","solution":" \u3010 Seventh solution using Cayley-Bacharach (author). Define points $W, Z, R, S$ as in the previous solution. ![](https:\/\/cdn.mathpix.com\/cropped\/2024_11_19_148cb373ffbe5370bbc6g-06.jpg?height=621&width=743&top_left_y=569&top_left_x=668) The quartics $(A D X Z) \\cup(B C W Y)$ and $\\overline{X Y} \\cup \\overline{W Z} \\cup(A B C D)$ meet at the 16 points $$ A, B, C, D, W, X, Y, Z, P, Q, R, S, I, I, J, J $$ where $I$ and $J$ are the circular points at infinity. Since $\\overline{A B} \\cup \\overline{C D} \\cup(P Q R)$ contains the 13 points $$ A, B, C, D, P, Q, R, W, X, Y, Z, I, J $$ it must contain $S, I$, and $J$ as well, by quartic Cayley-Bacharach. Thus, $(P Q R S)$ is cyclic and intersects $(A B C D)$ at $I, I, J$, and $J$, implying that the two circles are concentric, as desired. Remark (Author comments). Holden says he came up with this problem via the CayleyBacharach solution, by trying to get two quartics to intersect."} +{"year":2021,"label":"2","problem":"Let $a_{1}i$. First, each $i$ has finite degree - otherwise $$ \\frac{a_{x_{1}}}{x_{1}}=\\frac{a_{x_{2}}}{x_{2}}=\\cdots $$ for an infinite increasing sequence of positive integers $x_{i}$, but then the $a_{x_{i}}$ are unbounded. Now we use the following process to build a sequence of indices whose $a_{i}$ we can lower-bound: - Start at $x_{1}=1$, which is good. - If we're currently at good index $x_{i}$, then let $s_{i}$ be the largest positive integer such that $x_{i}$ has an edge to $x_{i}+s_{i}$. (This exists because the degrees are finite.) - Let $t_{i}$ be the smallest positive integer for which $x_{i}+s_{i}+t_{i}$ is good, and let this be $x_{i+1}$. This exists because if all numbers $k \\leq x \\leq 2 k$ are bad, they must each connect to some number less than $k$ (if two connect to each other, the smaller one is good), but then two connect to the same number, and therefore to each other this is the idea we will use later to bound the $t_{i}$ as well. Then $x_{i}=1+s_{1}+t_{1}+\\cdots+s_{i-1}+t_{i-1}$, and we have $$ a_{x_{i+1}}>a_{x_{i}+s_{i}}=\\frac{x_{i}+s_{i}}{x_{i}} a_{x_{i}}=\\frac{1+\\left(s_{1}+\\cdots+s_{i-1}+s_{i}\\right)+\\left(t_{1}+\\cdots+t_{i-1}\\right)}{1+\\left(s_{1}+\\cdots+s_{i-1}\\right)+\\left(t_{1}+\\cdots+t_{i-1}\\right)} a_{x_{i}} $$ This means $$ c_{n}:=\\frac{a_{x_{n}}}{a_{1}}>\\prod_{i=1}^{n-1} \\frac{1+\\left(s_{1}+\\cdots+s_{i-1}+s_{i}\\right)+\\left(t_{1}+\\cdots+t_{i-1}\\right)}{1+\\left(s_{1}+\\cdots+s_{i-1}\\right)+\\left(t_{1}+\\cdots+t_{i-1}\\right)} $$ ## Lemma $t_{1}+\\cdots+t_{n} \\leq s_{1}+\\cdots+s_{n}$ for each $n$. First we claim that if $x \\in\\left(x_{i}+s_{i}, x_{i}+s_{i}+t_{i}\\right)$, then $x$ cannot have an edge to $x_{j}$ for any $j \\leq i$. This is because $x>x_{i}+s_{i} \\geq x_{j}+s_{j}$, contradicting the fact that $x_{j}+s_{j}$ is the largest neighbor of $x_{j}$. This also means $x$ doesn't have an edge to $x_{j}+s_{j}$ for any $j \\leq i$, since if it did, it would have an edge to $x_{j}$. Second, no two bad values of $x$ can have an edge, since then the smaller one is good. This also means no two bad $x$ can have an edge to the same $y$. Then each of the $\\sum\\left(t_{i}-1\\right)$ values in the intervals $\\left(x_{i}+s_{i}, x_{i}+s_{i}+t_{i}\\right)$ for $1 \\leq i \\leq n$ must have an edge to an unique $y$ in one of the intervals $\\left(x_{i}, x_{i}+s_{i}\\right)$ (not necessarily with the same $i$ ). Therefore $$ \\sum\\left(t_{i}-1\\right) \\leq \\sum\\left(s_{i}-1\\right) \\Longrightarrow \\sum t_{i} \\leq \\sum s_{i} $$ Now note that if $a>b$, then $\\frac{a+x}{b+x}=1+\\frac{a-b}{b+x}$ is decreasing in $x$. This means $$ c_{n}>\\prod_{i=1}^{n-1} \\frac{1+2 s_{1}+\\cdots+2 s_{i-1}+s_{i}}{1+2 s_{1}+\\cdots+2 s_{i-1}}>\\prod_{i=1}^{n-1} \\frac{1+2 s_{1}+\\cdots+2 s_{i-1}+2 s_{i}}{1+2 s_{1}+\\cdots+2 s_{i-1}+s_{i}} $$ By multiplying both products, we have a telescoping product, which results in $$ c_{n}^{2} \\geq 1+2 s_{1}+\\cdots+2 s_{n}+2 s_{n+1} $$ The right hand side is unbounded since the $s_{i}$ are positive integers, while $c_{n}=a_{x_{n}} \/ a_{1}<$ $1 \/ a_{1}$ is bounded, contradiction."} +{"year":2021,"label":"2","problem":"Let $a_{1}i \\geq k$, then $j \\leq 1.1 i$. Suppose no such $k$ exists. Then, take a pair $j_{1}>i_{1}$ such that $\\frac{a_{j_{1}}}{j_{1}}=\\frac{a_{i_{1}}}{i_{1}}$ and $j_{1}>1.1 i_{1}$, or $a_{j_{1}}>1.1 a_{i_{1}}$. Now, since $k=j_{1}$ can't work, there exists a pair $j_{2}>i_{2} \\geq i_{1}$ such that $\\frac{a_{j_{2}}}{j_{2}}=\\frac{a_{i_{2}}}{i_{2}}$ and $j_{2}>1.1 i_{2}$, or $a_{j_{2}}>1.1 a_{i_{2}}$. Continuing in this fashion, we see that $$ a_{j_{\\ell}}>1.1 a_{i_{\\ell}}>1.1 a_{j_{\\ell-1}} $$ so we have that $a_{j_{\\ell}}>1.1^{\\ell} a_{i_{1}}$. Taking $\\ell>\\log _{1.1}\\left(1 \/ a_{1}\\right)$ gives the desired contradiction. ## Lemma For $N>k^{2}$, there are at most $0.8 N$ pins in $[\\sqrt{N}, N)$. $$ p-N\\left(1-\\frac{1}{1.1}\\right) \\leq N-p $$ so $p \\leq 0.8 N$, as desired. We say that $i$ is the pin of $j$ if it is the smallest index such that $\\frac{a_{i}}{i}=\\frac{a_{j}}{j}$. The pin of $j$ is always a pin. Given an index $i$, let $f(i)$ denote the largest index less than $i$ that is not a pin (we leave the function undefined when no such index exists, as we are only interested in the behavior for large $i$ ). Then $f$ is weakly increasing and unbounded by the first lemma. Let $N_{0}$ be a positive integer such that $f\\left(\\sqrt{N_{0}}\\right)>k$. Take any $N>N_{0}$ such that $N$ is not a pin. Let $b_{0}=N$, and $b_{1}$ be the pin of $b_{0}$. Recursively define $b_{2 i}=f\\left(b_{2 i-1}\\right)$, and $b_{2 i+1}$ to be the pin of $b_{2 i}$. Let $\\ell$ be the largest odd index such that $b_{\\ell} \\geq \\sqrt{N}$. We first show that $b_{\\ell} \\leq 100 \\sqrt{N}$. Since $N>N_{0}$, we have $b_{\\ell+1}>k$. By the choice of $\\ell$ we have $b_{\\ell+2}<\\sqrt{N}$, so $$ b_{\\ell+1}<1.1 b_{\\ell+2}<1.1 \\sqrt{N} $$ by the first lemma. We see that all the indices from $b_{\\ell+1}+1$ to $b_{\\ell}$ must be pins, so we have at least $b_{\\ell}-1.1 \\sqrt{N}$ pins in $\\left[\\sqrt{N}, b_{\\ell}\\right)$. Combined with the second lemma, this shows that $b_{\\ell} \\leq 100 \\sqrt{N}$. Now, we have that $a_{b_{2 i}}=\\frac{b_{2 i}}{b_{2 i+1}} a_{b_{2 i+1}}$ and $a_{b_{2 i+1}}>a_{b_{2 i+2}}$, so combining gives us $$ \\frac{a_{b_{0}}}{a_{b_{\\ell}}}>\\frac{b_{0}}{b_{1}} \\frac{b_{2}}{b_{3}} \\cdots \\frac{b_{\\ell-1}}{b_{\\ell}} . $$ Note that there are at least $$ \\left(b_{1}-b_{2}\\right)+\\left(b_{3}-b_{4}\\right)+\\cdots+\\left(b_{\\ell-2}-b_{\\ell-1}\\right) $$ pins in $[\\sqrt{N}, N)$, so by the second lemma, that sum is at most $0.8 N$. Thus, $$ \\begin{aligned} \\left(b_{0}-b_{1}\\right)+\\left(b_{2}-b_{3}\\right)+\\cdots+\\left(b_{\\ell-1}-b_{\\ell}\\right) & =b_{0}-\\left[\\left(b_{1}-b_{2}\\right)+\\cdots+\\left(b_{\\ell-2}-b_{\\ell-1}\\right)\\right]-b_{\\ell} \\\\ & \\geq 0.2 N-100 \\sqrt{N} . \\end{aligned} $$ Then $$ \\begin{aligned} \\frac{b_{0}}{b_{1}} \\frac{b_{2}}{b_{3}} \\cdots \\frac{b_{\\ell-1}}{b_{\\ell}} & \\geq 1+\\frac{b_{0}-b_{1}}{b_{1}}+\\cdots+\\frac{b_{\\ell-1}-b_{\\ell}}{b_{\\ell}} \\\\ & >1+\\frac{b_{0}-b_{1}}{b_{0}}+\\cdots+\\frac{b_{\\ell-1}-b_{\\ell}}{b_{0}} \\\\ & \\geq 1+\\frac{0.2 N-100 \\sqrt{N}}{N} \\end{aligned} $$ which is at least 1.01 if $N_{0}$ is large enough. Thus, we see that $$ a_{N}>1.01 a_{b_{\\ell}} \\geq 1.01 a_{\\lfloor\\sqrt{N}\\rfloor} $$ if $N>N_{0}$ is not a pin. Since there are arbitrarily large non-pins, this implies that the sequence $\\left(a_{n}\\right)$ is unbounded, which is the desired contradiction."} +{"year":2021,"label":"3","problem":"Find all positive integers $k>1$ for which there exists a positive integer $n$ such that $\\binom{n}{k}$ is divisible by $n$, and $\\binom{n}{m}$ is not divisible by $n$ for $2 \\leq mt_{p}$, so $\\nu_{p}(n-i)=\\nu_{p}(i)$; - If $p \\mid k$ and $i \\neq k_{p}$, then we have $\\nu_{p}(i), \\nu_{p}(n-i) \\leq t_{p}$ and $\\nu_{p}(n) \\geq t_{p}$, so again $\\nu_{p}(n-i)=\\nu_{p}(i) ;$ - If $p \\mid k$ and $i=k_{p}$, then we have $\\nu_{p}(n-i)=\\nu_{p}(i)+\\nu_{p}(k)$ by (2). We conclude that $\\nu_{p}(n-i)=\\nu_{p}(i)$ always holds, except when $i=k_{p}$, when we have $\\nu_{p}(n-i)=\\nu_{p}(i)+\\nu_{p}(k)$ (this formula holds irrespective of whether $p \\mid k$ or $\\left.p \\nmid k\\right)$. We can now show that $\\binom{n}{k}$ is divisible by $n$, which amounts to showing that $k$ ! divides $(n-1)(n-2) \\cdots(n-k+1)$. Indeed, for each prime $p \\leq k$ we have $$ \\begin{aligned} \\nu_{p}((n-1)(n-2) \\ldots(n-k+1)) & =\\nu_{p}\\left(n-k_{p}\\right)+\\sum_{i\\nu_{p}(k)$, then it follows that $$ \\begin{aligned} \\nu_{p}((n-1)(n-2) \\ldots(n-m+1)) & =\\nu_{p}(k)+\\sum_{i=1}^{m-1} \\nu_{p}(i) \\\\ & <\\nu_{p}(m)+\\sum_{i=1}^{m-1} \\nu_{p}(i) \\\\ & =\\nu_{p}(m!) \\end{aligned} $$ so $m$ ! cannot divide $(n-1)(n-2) \\ldots(n-m+1)$. On the other hand, suppose that $\\nu_{p}(m) \\leq \\nu_{p}(k)$ for all $p \\mid k$, which would mean that $m \\mid k$ and hence $m \\leq \\frac{k}{2}$. Consider a prime $p$ dividing $m$. We have $k_{p} \\geq \\frac{k}{2}$, because otherwise $2 k_{p}$ could have been used instead of $k_{p}$. It follows that $m \\leq \\frac{k}{2} \\leq k_{p}$. Therefore, we obtain $$ \\begin{aligned} \\nu_{p}((n-1)(n-2) \\ldots(n-m+1)) & =\\sum_{i=1}^{m-1} \\nu_{p}(n-i) \\\\ & =\\sum_{i=1}^{m-1} \\nu_{p}(i) \\\\ & =\\nu_{p}((m-1)!)<\\nu_{p}(m!) \\end{aligned} $$ showing that $(n-1)(n-2) \\cdots(n-m+1)$ is not divisible by $m$ !. This shows that $\\binom{n}{m}$ is not divisible by $n$ for $ma \/ 2$, then $(\\dagger)$ forces $r^{2}+s^{2} \\leq 2 b$, giving the last case. Hence, there exists some particular pair $(r, s)$ for which there are infinitely many solutions $(m, n)$. Simplifying the system gives $$ \\begin{aligned} & a n=2 r m+r^{2}-b \\\\ & 2 s n=a m+b-s^{2} \\end{aligned} $$ Since the system is linear, for there to be infinitely many solutions $(m, n)$ the system must be dependent. This gives $$ \\frac{a}{2 s}=\\frac{2 r}{a}=\\frac{r^{2}-b}{b-s^{2}} $$ so $a=2 \\sqrt{r s}$ and $b=\\frac{s^{2} \\sqrt{r}+r^{2} \\sqrt{s}}{\\sqrt{r}+\\sqrt{s}}$. Since $r s$ must be square, we can reparametrize as $r=k x^{2}, s=k y^{2}$, and $\\operatorname{gcd}(x, y)=1$. This gives $$ \\begin{aligned} a & =2 k x y \\\\ b & =k^{2} x y\\left(x^{2}-x y+y^{2}\\right) \\end{aligned} $$ Thus, $a \\mid 2 b$, as desired."} +{"year":2021,"label":"5","problem":"Let $T$ be a tree on $n$ vertices with exactly $k$ leaves. Suppose that there exists a subset of at least $\\frac{n+k-1}{2}$ vertices of $T$, no two of which are adjacent. Show that the longest path in $T$ contains an even number of edges.","solution":" The longest path in $T$ must go between two leaves. The solutions presented here will solve the problem by showing that in the unique 2-coloring of $T$, all leaves are the same color. ## \u3010 Solution 1 (Ankan Bhattacharya, Jeffery Li). ## Lemma If $S$ is an independent set of $T$, then $$ \\sum_{v \\in S} \\operatorname{deg}(v) \\leq n-1 $$ Equality holds if and only if $S$ is one of the two components of the unique 2-coloring of $T$. We are given that there exists an independent set of at least $\\frac{n+k-1}{2}$ vertices. By greedily choosing vertices of smallest degree, the sum of the degrees of these vertices is at least $$ k+2 \\cdot \\frac{n-k-1}{2}=n-1 $$ Thus equality holds everywhere, which implies that the independent set contains every leaf and is one of the components of the 2 -coloring."} +{"year":2021,"label":"5","problem":"Let $T$ be a tree on $n$ vertices with exactly $k$ leaves. Suppose that there exists a subset of at least $\\frac{n+k-1}{2}$ vertices of $T$, no two of which are adjacent. Show that the longest path in $T$ contains an even number of edges.","solution":" The longest path in $T$ must go between two leaves. The solutions presented here will solve the problem by showing that in the unique 2-coloring of $T$, all leaves are the same color. ## \u3010 Solution 2 (Andrew Gu). ## Lemma The vertices of $T$ can be partitioned into $k-1$ paths (i.e. the induced subgraph on each set of vertices is a path) such that all edges of $T$ which are not part of a path are incident to an endpoint of a path. Now given a path of $a$ vertices, at most $\\frac{a+1}{2}$ of those vertices can be in an independent set of $T$. By the lemma, $T$ can be partitioned into $k-1$ paths of $a_{1}, \\ldots, a_{k-1}$ vertices, so the maximum size of an independent set of $T$ is $$ \\sum \\frac{a_{i}+1}{2}=\\frac{n+k-1}{2} . $$ For equality to hold, each path in the partition must have an odd number of vertices, and has a unique 2 -coloring in red and blue where the endpoints are red. The unique independent set of $T$ of size $\\frac{n+k-1}{2}$ is then the set of red vertices. By the lemma, the edges of $T$ which are not part of a path connect an endpoint of a path (which is colored red) to another vertex (which must be blue, because the red vertices are independent). Thus the coloring of the paths extends to the unique 2 -coloring of $T$. The leaves of $T$ are endpoints of paths, so they are all red."} +{"year":2021,"label":"6","problem":"Triangles $A B C$ and $D E F$ share circumcircle $\\Omega$ and incircle $\\omega$ so that points $A, F$, $B, D, C$, and $E$ occur in this order along $\\Omega$. Let $\\Delta_{A}$ be the triangle formed by lines $A B, A C$, and $E F$, and define triangles $\\Delta_{B}, \\Delta_{C}, \\ldots, \\Delta_{F}$ similarly. Furthermore, let $\\Omega_{A}$ and $\\omega_{A}$ be the circumcircle and incircle of triangle $\\Delta_{A}$, respectively, and define circles $\\Omega_{B}, \\omega_{B}, \\ldots, \\Omega_{F}, \\omega_{F}$ similarly. (a) Prove that the two common external tangents to circles $\\Omega_{A}$ and $\\Omega_{D}$ and the two common external tangents to circles $\\omega_{A}$ and $\\omega_{D}$ are either concurrent or pairwise parallel. (b) Suppose that these four lines meet at point $T_{A}$, and define points $T_{B}$ and $T_{C}$ similarly. Prove that points $T_{A}, T_{B}$, and $T_{C}$ are collinear.","solution":" ![](https:\/\/cdn.mathpix.com\/cropped\/2024_11_19_148cb373ffbe5370bbc6g-17.jpg?height=818&width=754&top_left_y=1150&top_left_x=651) Let $I$ and $r$ be the center and radius of $\\omega$, and let $O$ and $R$ be the center and radius of $\\Omega$. Let $O_{A}$ and $I_{A}$ be the circumcenter and incenter of triangle $\\Delta_{A}$, and define $O_{B}$, $I_{B}, \\ldots, I_{F}$ similarly. Let $\\omega$ touch $E F$ at $A_{1}$, and define $B_{1}, C_{1}, \\ldots, F_{1}$ similarly. $$ \\left(\\Omega_{A} \\cup \\omega_{A}\\right) \\sim\\left(\\Omega_{D} \\cup \\omega_{D}\\right) $$ The four lines will concur at the homothetic center of these figures (possibly at infinity). Solution 1 (author) Let the second tangent to $\\omega$ parallel to $E F$ meet lines $A B$ and $A C$ at $P$ and $Q$, respectively, and let the second tangent to $\\omega$ parallel to $B C$ meet lines $D E$ and $D F$ at $R$ and $S$, respectively. Furthermore, let $\\omega$ touch $P Q$ and $R S$ at $U$ and $V$, respectively. Let $h$ be inversion with respect to $\\omega$. Then $h$ maps $A, B$, and $C$ onto the midpoints of the sides of triangle $D_{1} E_{1} F_{1}$. So $h$ maps $k$ onto the Euler circle of triangle $D_{1} E_{1} F_{1}$. Similarly, $h$ maps $k$ onto the Euler circle of triangle $A_{1} B_{1} C_{1}$. Therefore, triangles $A_{1} B_{1} C_{1}$ and $D_{1} E_{1} F_{1}$ share a common nine-point circle $\\gamma$. Let $K$ be its center; its radius equals $\\frac{1}{2} r$. Let $H$ be the reflection of $I$ in $K$. Then $H$ is the common orthocenter of triangles $A_{1} B_{1} C_{1}$ and $D_{1} E_{1} F_{1}$. Let $\\gamma_{U}$ of center $K_{U}$ and radius $\\frac{1}{2} r$ be the Euler circle of triangle $U E_{1} F_{1}$, and let $\\gamma_{V}$ of center $K_{V}$ and radius $\\frac{1}{2} r$ be the Euler circle of triangle $V B_{1} C_{1}$. Let $H_{U}$ be the orthocenter of triangle $U E_{1} F_{1}$. Since quadrilateral $D_{1} E_{1} F_{1} U$ is cyclic, vectors $\\overrightarrow{H H_{U}}$ and $\\overrightarrow{D_{1} U}$ are equal. Consequently, $\\overrightarrow{K K_{U}}=\\frac{1}{2} \\overrightarrow{D_{1} U}$. Similarly, $\\overrightarrow{K K_{V}}=$ $\\frac{1}{2} \\overrightarrow{A_{1} V}$. Since both of $A_{1} U$ and $D_{1} V$ are diameters in $\\omega$, vectors $\\overrightarrow{D_{1} U}$ and $\\overrightarrow{A_{1} V}$ are equal. Therefore, $K_{U}$ and $K_{V}$ coincide, and so do $\\gamma_{U}$ and $\\gamma_{V}$. As above, $h$ maps $\\gamma_{U}$ onto the circumcircle of triangle $A P Q$ and $\\gamma_{V}$ onto the circumcircle of triangle $D R S$. Therefore, triangles $A P Q$ and $D R S$ are inscribed inside the same circle $\\Omega_{A D}$. Since $E F$ and $P Q$ are parallel, triangles $\\Delta_{A}$ and $A P Q$ are homothetic, and so are figures $\\Omega_{A} \\cup \\omega_{A}$ and $\\Omega_{A D} \\cup \\omega$. Consequently, we have $$ \\left(\\Omega_{A} \\cup \\omega_{A}\\right) \\sim\\left(\\Omega_{A D} \\cup \\omega\\right) \\sim\\left(\\Omega_{D} \\cup \\omega_{D}\\right), $$ which solves part (a)."} +{"year":2021,"label":"6","problem":"Triangles $A B C$ and $D E F$ share circumcircle $\\Omega$ and incircle $\\omega$ so that points $A, F$, $B, D, C$, and $E$ occur in this order along $\\Omega$. Let $\\Delta_{A}$ be the triangle formed by lines $A B, A C$, and $E F$, and define triangles $\\Delta_{B}, \\Delta_{C}, \\ldots, \\Delta_{F}$ similarly. Furthermore, let $\\Omega_{A}$ and $\\omega_{A}$ be the circumcircle and incircle of triangle $\\Delta_{A}$, respectively, and define circles $\\Omega_{B}, \\omega_{B}, \\ldots, \\Omega_{F}, \\omega_{F}$ similarly. (a) Prove that the two common external tangents to circles $\\Omega_{A}$ and $\\Omega_{D}$ and the two common external tangents to circles $\\omega_{A}$ and $\\omega_{D}$ are either concurrent or pairwise parallel. (b) Suppose that these four lines meet at point $T_{A}$, and define points $T_{B}$ and $T_{C}$ similarly. Prove that points $T_{A}, T_{B}$, and $T_{C}$ are collinear.","solution":" ![](https:\/\/cdn.mathpix.com\/cropped\/2024_11_19_148cb373ffbe5370bbc6g-17.jpg?height=818&width=754&top_left_y=1150&top_left_x=651) Let $I$ and $r$ be the center and radius of $\\omega$, and let $O$ and $R$ be the center and radius of $\\Omega$. Let $O_{A}$ and $I_{A}$ be the circumcenter and incenter of triangle $\\Delta_{A}$, and define $O_{B}$, $I_{B}, \\ldots, I_{F}$ similarly. Let $\\omega$ touch $E F$ at $A_{1}$, and define $B_{1}, C_{1}, \\ldots, F_{1}$ similarly. $$ \\left(\\Omega_{A} \\cup \\omega_{A}\\right) \\sim\\left(\\Omega_{D} \\cup \\omega_{D}\\right) $$ The four lines will concur at the homothetic center of these figures (possibly at infinity). Solution 2 (Michael Ren) As in the previous solution, let the second tangent to $\\omega$ parallel to $E F$ meet $A B$ and $A C$ at $P$ and $Q$, respectively. Let $(A P Q)$ meet $\\Omega$ again at $D^{\\prime}$, so that $D^{\\prime}$ is the Miquel point of $\\{A B, A C, B C, P Q\\}$. Since the quadrilateral formed by these lines has incircle $\\omega$, it is classical that $D^{\\prime} I$ bisects $\\angle P D^{\\prime} C$ and $B D^{\\prime} Q$ (e.g. by DDIT). Let $\\ell$ be the tangent to $\\Omega$ at $D^{\\prime}$ and $D^{\\prime} I$ meet $\\Omega$ again at $M$. We have $$ \\measuredangle\\left(\\ell, D^{\\prime} B\\right)=\\measuredangle D^{\\prime} C B=\\measuredangle D^{\\prime} Q P=\\measuredangle\\left(D^{\\prime} Q, E F\\right) . $$ Therefore $D^{\\prime} I$ also bisects the angle between $\\ell$ and the line parallel to $E F$ through $D^{\\prime}$. This means that $M$ is one of the arc midpoints of $E F$. Additionally, $D^{\\prime}$ lies on $\\operatorname{arc} B C$ not containing $A$, so $D^{\\prime}=D$. Similarly, letting the second tangent to $\\omega$ parallel to $B C$ meet $D E$ and $D F$ again at $R$ and $S$, we have $A D R S$ cyclic. ## Lemma There exists a circle $\\Omega_{A D}$ tangent to $\\Omega_{A}$ and $\\Omega_{D}$ at $A$ and $D$, respectively. $$ \\measuredangle O A O_{A}=\\measuredangle(\\perp E F, \\perp B C)=\\measuredangle(E F, B C) . $$ (Here, $\\perp E F$ means the direction perpendicular to $E F$.) By symmetry, this is equal to $\\measuredangle O_{D} D O$. The circle $(A D P Q)$ passes through $A$ and $D$, and is tangent to $\\Omega_{A}$ by homothety. Therefore it coincides with $\\Omega_{A D}$, as does ( $A D R S$ ). Like the previous solution, we finish with $$ \\left(\\Omega_{A} \\cup \\omega_{A}\\right) \\sim\\left(\\Omega_{A D} \\cup \\omega\\right) \\sim\\left(\\Omega_{D} \\cup \\omega_{D}\\right) . $$"} +{"year":2021,"label":"6","problem":"Triangles $A B C$ and $D E F$ share circumcircle $\\Omega$ and incircle $\\omega$ so that points $A, F$, $B, D, C$, and $E$ occur in this order along $\\Omega$. Let $\\Delta_{A}$ be the triangle formed by lines $A B, A C$, and $E F$, and define triangles $\\Delta_{B}, \\Delta_{C}, \\ldots, \\Delta_{F}$ similarly. Furthermore, let $\\Omega_{A}$ and $\\omega_{A}$ be the circumcircle and incircle of triangle $\\Delta_{A}$, respectively, and define circles $\\Omega_{B}, \\omega_{B}, \\ldots, \\Omega_{F}, \\omega_{F}$ similarly. (a) Prove that the two common external tangents to circles $\\Omega_{A}$ and $\\Omega_{D}$ and the two common external tangents to circles $\\omega_{A}$ and $\\omega_{D}$ are either concurrent or pairwise parallel. (b) Suppose that these four lines meet at point $T_{A}$, and define points $T_{B}$ and $T_{C}$ similarly. Prove that points $T_{A}, T_{B}$, and $T_{C}$ are collinear.","solution":" ![](https:\/\/cdn.mathpix.com\/cropped\/2024_11_19_148cb373ffbe5370bbc6g-17.jpg?height=818&width=754&top_left_y=1150&top_left_x=651) Let $I$ and $r$ be the center and radius of $\\omega$, and let $O$ and $R$ be the center and radius of $\\Omega$. Let $O_{A}$ and $I_{A}$ be the circumcenter and incenter of triangle $\\Delta_{A}$, and define $O_{B}$, $I_{B}, \\ldots, I_{F}$ similarly. Let $\\omega$ touch $E F$ at $A_{1}$, and define $B_{1}, C_{1}, \\ldots, F_{1}$ similarly. $$ \\left(\\Omega_{A} \\cup \\omega_{A}\\right) \\sim\\left(\\Omega_{D} \\cup \\omega_{D}\\right) $$ The four lines will concur at the homothetic center of these figures (possibly at infinity). Solution 3 (Andrew Gu) Construct triangles homothetic to $\\Delta_{A}$ and $\\Delta_{D}$ (with positive ratio) which have the same circumcircle; it suffices to show that these copies have the same incircle as well. Let $O$ be the center of this common circumcircle, which we take to be the origin, and $M_{X Y}$ denote the point on the circle such that the tangent at that point is parallel to line $X Y$ (from the two possible choices, we make the choice that corresponds to the arc midpoint on $\\Omega$, e.g. $M_{A B}$ should correspond to the arc midpoint on the internal angle bisector of $A C B$ ). The left diagram below shows the original triangles $A B C$ and $D E F$, while the right diagram shows the triangles homothetic to $\\Delta_{A}$ and $\\Delta_{D}$. ![](https:\/\/cdn.mathpix.com\/cropped\/2024_11_19_148cb373ffbe5370bbc6g-19.jpg?height=566&width=1203&top_left_y=902&top_left_x=432) Using the fact that the incenter is the orthocenter of the arc midpoints, the incenter of $\\Delta_{A}$ in this reference frame is $M_{A B}+M_{A C}-M_{E F}$ and the incenter of $\\Delta_{D}$ in this reference frame is $M_{D E}+M_{D F}-M_{B C}$. Since $A B C$ and $D E F$ share a common incenter, we have $$ M_{A B}+M_{B C}+M_{C A}=M_{D E}+M_{E F}+M_{F D} $$ Thus the copies of $\\Delta_{A}$ and $\\Delta_{D}$ have the same incenter, and therefore the same incircle as well (Euler's theorem determines the inradius)."} +{"year":2021,"label":"6","problem":"Triangles $A B C$ and $D E F$ share circumcircle $\\Omega$ and incircle $\\omega$ so that points $A, F$, $B, D, C$, and $E$ occur in this order along $\\Omega$. Let $\\Delta_{A}$ be the triangle formed by lines $A B, A C$, and $E F$, and define triangles $\\Delta_{B}, \\Delta_{C}, \\ldots, \\Delta_{F}$ similarly. Furthermore, let $\\Omega_{A}$ and $\\omega_{A}$ be the circumcircle and incircle of triangle $\\Delta_{A}$, respectively, and define circles $\\Omega_{B}, \\omega_{B}, \\ldots, \\Omega_{F}, \\omega_{F}$ similarly. (a) Prove that the two common external tangents to circles $\\Omega_{A}$ and $\\Omega_{D}$ and the two common external tangents to circles $\\omega_{A}$ and $\\omega_{D}$ are either concurrent or pairwise parallel. (b) Suppose that these four lines meet at point $T_{A}$, and define points $T_{B}$ and $T_{C}$ similarly. Prove that points $T_{A}, T_{B}$, and $T_{C}$ are collinear.","solution":" ![](https:\/\/cdn.mathpix.com\/cropped\/2024_11_19_148cb373ffbe5370bbc6g-17.jpg?height=818&width=754&top_left_y=1150&top_left_x=651) Let $I$ and $r$ be the center and radius of $\\omega$, and let $O$ and $R$ be the center and radius of $\\Omega$. Let $O_{A}$ and $I_{A}$ be the circumcenter and incenter of triangle $\\Delta_{A}$, and define $O_{B}$, $I_{B}, \\ldots, I_{F}$ similarly. Let $\\omega$ touch $E F$ at $A_{1}$, and define $B_{1}, C_{1}, \\ldots, F_{1}$ similarly. $$ \\left(\\Omega_{A} \\cup \\omega_{A}\\right) \\sim\\left(\\Omega_{D} \\cup \\omega_{D}\\right) $$ The four lines will concur at the homothetic center of these figures (possibly at infinity). Solution 1 (author) By Monge's theorem applied to $\\omega, \\omega_{A}$, and $\\omega_{D}$, points $A, D$, and $T_{A}$ are collinear. Therefore, $T_{A}=A D \\cap I_{A} I_{D}$. Let $p$ be pole-and-polar correspondence with respect to $\\omega$. Then $p$ maps $A$ onto line $E_{1} F_{1}$ and $D$ onto line $B_{1} C_{1}$. Consequently, $p$ maps line $A D$ onto $X_{A}=B_{1} C_{1} \\cap E_{1} F_{1}$. Furthermore, $p$ maps the line that bisects the angle formed by lines $A B$ and $E F$ and does not contain $I$ onto the midpoint of segment $A_{1} F_{1}$. Similarly, $p$ maps the line that bisects the angle formed by lines $A C$ and $E F$ and does not contain $I$ onto the midpoint of segment $A_{1} E_{1}$. So $p$ maps $I_{A}$ onto the midline of triangle $A_{1} E_{1} F_{1}$ opposite $A_{1}$. Similarly, $p$ maps $I_{D}$ onto the midline of triangle $D_{1} B_{1} C_{1}$ opposite $D_{1}$. Consequently, $p$ maps line $I_{A} I_{D}$ onto the intersection point $Y_{A}$ of this pair of midlines, and $p$ maps $T_{A}$ onto line $X_{A} Y_{A}$. Since the reflection of $H$ in line $B_{1} C_{1}$ lies on $\\omega, A_{1} H \\cdot H H_{A}$ equals half the power of $H$ with respect to $\\omega$. Similarly, $D_{1} H \\cdot H H_{D}$ equals half the power of $H$ with respect to $\\omega$. Then $A_{1} H \\cdot H H_{A}=D_{1} H \\cdot H H_{D}$ and $A_{1} H H_{D} \\sim D_{1} H H_{A}$. Since $\\angle H H_{D} L_{A}=90^{\\circ}=$ $\\angle H H_{A} L_{D}$, figures $A_{1} H H_{D} L_{A}$ and $D_{1} H H_{A} L_{D}$ are similar as well. Consequently, $$ \\frac{H L_{A}}{L_{A} A_{1}}=\\frac{H L_{D}}{L_{D} D_{1}}=s $$ as a signed ratio. Let the line through $A_{1}$ parallel to $E_{1} F_{1}$ and the line through $D_{1}$ parallel to $B_{1} C_{1}$ meet at $Z_{A}$. Then $H X_{A} \/ X_{A} Z_{A}=s$ and $Y_{A}$ is the midpoint of segment $X_{A} Z_{A}$. Therefore, $H$ lies on line $X_{A} Y_{A}$. This means that $T_{A}$ lies on the polar of $H$ with respect to $\\omega$, and by symmetry so do $T_{B}$ and $T_{C}$."} +{"year":2021,"label":"6","problem":"Triangles $A B C$ and $D E F$ share circumcircle $\\Omega$ and incircle $\\omega$ so that points $A, F$, $B, D, C$, and $E$ occur in this order along $\\Omega$. Let $\\Delta_{A}$ be the triangle formed by lines $A B, A C$, and $E F$, and define triangles $\\Delta_{B}, \\Delta_{C}, \\ldots, \\Delta_{F}$ similarly. Furthermore, let $\\Omega_{A}$ and $\\omega_{A}$ be the circumcircle and incircle of triangle $\\Delta_{A}$, respectively, and define circles $\\Omega_{B}, \\omega_{B}, \\ldots, \\Omega_{F}, \\omega_{F}$ similarly. (a) Prove that the two common external tangents to circles $\\Omega_{A}$ and $\\Omega_{D}$ and the two common external tangents to circles $\\omega_{A}$ and $\\omega_{D}$ are either concurrent or pairwise parallel. (b) Suppose that these four lines meet at point $T_{A}$, and define points $T_{B}$ and $T_{C}$ similarly. Prove that points $T_{A}, T_{B}$, and $T_{C}$ are collinear.","solution":" ![](https:\/\/cdn.mathpix.com\/cropped\/2024_11_19_148cb373ffbe5370bbc6g-17.jpg?height=818&width=754&top_left_y=1150&top_left_x=651) Let $I$ and $r$ be the center and radius of $\\omega$, and let $O$ and $R$ be the center and radius of $\\Omega$. Let $O_{A}$ and $I_{A}$ be the circumcenter and incenter of triangle $\\Delta_{A}$, and define $O_{B}$, $I_{B}, \\ldots, I_{F}$ similarly. Let $\\omega$ touch $E F$ at $A_{1}$, and define $B_{1}, C_{1}, \\ldots, F_{1}$ similarly. $$ \\left(\\Omega_{A} \\cup \\omega_{A}\\right) \\sim\\left(\\Omega_{D} \\cup \\omega_{D}\\right) $$ The four lines will concur at the homothetic center of these figures (possibly at infinity). Solution 2 (author) As in the first solution to part (a), let $h$ be inversion with respect to $\\omega$, let $\\gamma$ of center $K$ be the common Euler circle of triangles $A_{1} B_{1} C_{1}$ and $D_{1} E_{1} F_{1}$, and let $H$ be their common orthocenter. Let $K_{A}$ and $K_{D}$ be the centers of $\\gamma_{A}$ and $\\gamma_{D}$, respectively, and let $\\ell_{A}$ be the perpendicular bisector of segment $K_{A} K_{D}$. Since $\\gamma_{A}$ and $\\gamma_{D}$ are congruent (both of them are of radius $\\left.\\frac{1}{2} r\\right)$, they are reflections of each other across $\\ell_{A}$. Inversion $h$ maps the two common external tangents of $\\Omega_{A}$ and $\\Omega_{D}$ onto the two circles $\\alpha$ and $\\beta$ through $I$ that are tangent to both of $\\gamma_{A}$ and $\\gamma_{D}$ in the same way. (That is, either internally to both or externally to both.) Consequently, $\\alpha$ and $\\beta$ are reflections of each other in $\\ell_{A}$ and so their second point of intersection $S_{A}$, which $h$ maps $T_{A}$ onto, is the reflection of $I$ in $\\ell_{A}$. Define $\\ell_{B}, \\ell_{C}, S_{B}$, and $S_{C}$ similarly. Then $S_{B}$ is the reflection of $I$ in $\\ell_{B}$ and $S_{C}$ is the reflection of $I$ in $\\ell_{C}$. Therefore, all four points $I, S_{A}, S_{B}$, and $S_{C}$ lie on the circle of center $K$ that contains $I$. (This is also the circle on diameter $I H$.) Since points $S_{A}, S_{B}$, and $S_{C}$ are concyclic with $I$, their images $T_{A}, T_{B}$, and $T_{C}$ under $h$ are collinear, and the solution is complete."} +{"year":2021,"label":"6","problem":"Triangles $A B C$ and $D E F$ share circumcircle $\\Omega$ and incircle $\\omega$ so that points $A, F$, $B, D, C$, and $E$ occur in this order along $\\Omega$. Let $\\Delta_{A}$ be the triangle formed by lines $A B, A C$, and $E F$, and define triangles $\\Delta_{B}, \\Delta_{C}, \\ldots, \\Delta_{F}$ similarly. Furthermore, let $\\Omega_{A}$ and $\\omega_{A}$ be the circumcircle and incircle of triangle $\\Delta_{A}$, respectively, and define circles $\\Omega_{B}, \\omega_{B}, \\ldots, \\Omega_{F}, \\omega_{F}$ similarly. (a) Prove that the two common external tangents to circles $\\Omega_{A}$ and $\\Omega_{D}$ and the two common external tangents to circles $\\omega_{A}$ and $\\omega_{D}$ are either concurrent or pairwise parallel. (b) Suppose that these four lines meet at point $T_{A}$, and define points $T_{B}$ and $T_{C}$ similarly. Prove that points $T_{A}, T_{B}$, and $T_{C}$ are collinear.","solution":" ![](https:\/\/cdn.mathpix.com\/cropped\/2024_11_19_148cb373ffbe5370bbc6g-17.jpg?height=818&width=754&top_left_y=1150&top_left_x=651) Let $I$ and $r$ be the center and radius of $\\omega$, and let $O$ and $R$ be the center and radius of $\\Omega$. Let $O_{A}$ and $I_{A}$ be the circumcenter and incenter of triangle $\\Delta_{A}$, and define $O_{B}$, $I_{B}, \\ldots, I_{F}$ similarly. Let $\\omega$ touch $E F$ at $A_{1}$, and define $B_{1}, C_{1}, \\ldots, F_{1}$ similarly. $$ \\left(\\Omega_{A} \\cup \\omega_{A}\\right) \\sim\\left(\\Omega_{D} \\cup \\omega_{D}\\right) $$ The four lines will concur at the homothetic center of these figures (possibly at infinity). Solution 3 (Ankan Bhattacharya) From either of the first two solutions to part (a), we know that there is a circle $\\Omega_{A D}$ passing through $A$ and $D$ which is (internally) tangent to $\\Omega_{A}$ and $\\Omega_{D}$. By Monge's theorem applied to $\\Omega_{A}, \\Omega_{D}$, and $\\Omega_{A D}$, it follows that $A, D$, and $T_{A}$ are collinear. The inversion at $T_{A}$ swapping $\\Omega_{A}$ with $\\Omega_{D}$ also swaps $A$ with $D$ because $T_{A}$ lies on $A D$ and $A$ is not homologous to $D$. Let $\\Omega_{A}$ and $\\Omega_{D}$ meet $\\Omega$ again at $L_{A}$ and $L_{D}$. Since $A D L_{A} L_{D}$ is cyclic, the same inversion at $T_{A}$ also swaps $L_{A}$ and $L_{D}$, so $T_{A}=A D \\cap L_{A} L_{D}$. By Taiwan TST 2014, $L_{A}$ and $L_{D}$ are the tangency points of the $A$-mixtilinear and $D$-mixtilinear incircles, respectively, with $\\Omega$. Therefore $A L_{A} \\cap D L_{D}$ is the exsimilicenter $X$ of $\\Omega$ and $\\omega$. Then $T_{A}, T_{B}$, and $T_{C}$ all lie on the polar of $X$ with respect to $\\Omega$."} +{"year":2021,"label":"6","problem":"Triangles $A B C$ and $D E F$ share circumcircle $\\Omega$ and incircle $\\omega$ so that points $A, F$, $B, D, C$, and $E$ occur in this order along $\\Omega$. Let $\\Delta_{A}$ be the triangle formed by lines $A B, A C$, and $E F$, and define triangles $\\Delta_{B}, \\Delta_{C}, \\ldots, \\Delta_{F}$ similarly. Furthermore, let $\\Omega_{A}$ and $\\omega_{A}$ be the circumcircle and incircle of triangle $\\Delta_{A}$, respectively, and define circles $\\Omega_{B}, \\omega_{B}, \\ldots, \\Omega_{F}, \\omega_{F}$ similarly. (a) Prove that the two common external tangents to circles $\\Omega_{A}$ and $\\Omega_{D}$ and the two common external tangents to circles $\\omega_{A}$ and $\\omega_{D}$ are either concurrent or pairwise parallel. (b) Suppose that these four lines meet at point $T_{A}$, and define points $T_{B}$ and $T_{C}$ similarly. Prove that points $T_{A}, T_{B}$, and $T_{C}$ are collinear.","solution":" ![](https:\/\/cdn.mathpix.com\/cropped\/2024_11_19_148cb373ffbe5370bbc6g-17.jpg?height=818&width=754&top_left_y=1150&top_left_x=651) Let $I$ and $r$ be the center and radius of $\\omega$, and let $O$ and $R$ be the center and radius of $\\Omega$. Let $O_{A}$ and $I_{A}$ be the circumcenter and incenter of triangle $\\Delta_{A}$, and define $O_{B}$, $I_{B}, \\ldots, I_{F}$ similarly. Let $\\omega$ touch $E F$ at $A_{1}$, and define $B_{1}, C_{1}, \\ldots, F_{1}$ similarly. $$ \\left(\\Omega_{A} \\cup \\omega_{A}\\right) \\sim\\left(\\Omega_{D} \\cup \\omega_{D}\\right) $$ The four lines will concur at the homothetic center of these figures (possibly at infinity). Solution 4 (Andrew Gu) We show that $T_{A}$ lies on the radical axis of the point circle at $I$ and $\\Omega$, which will solve the problem. Let $I_{A}$ and $I_{D}$ be the centers of $\\omega_{A}$ and $\\omega_{D}$ respectively. By the Monge's theorem applied to $\\omega, \\omega_{A}$, and $\\omega_{D}$, points $A, D$, and $T_{A}$ are collinear. Additionally, these other triples are collinear: $\\left(A, I_{A}, I\\right),\\left(D, I_{D}, I\\right),\\left(I_{A}, I_{D}, T\\right)$. By Menelaus's theorem, we have $$ \\frac{T_{A} D}{T_{A} A}=\\frac{I_{A} I}{I_{A} A} \\cdot \\frac{I_{D} D}{I_{D} I} . $$ If $s$ is the length of the side opposite $A$ in $\\Delta_{A}$, then we compute $$ \\begin{aligned} \\frac{I_{A} I}{I_{A} A} & =\\frac{s \/ \\cos (A \/ 2)}{r_{A} \/ \\sin (A \/ 2)} \\\\ & =\\frac{2 R_{A} \\sin (A) \\sin (A \/ 2)}{\\cos (A \/ 2)} \\\\ & =\\frac{4 R_{A} \\sin ^{2}(A \/ 2)}{r_{A}} \\\\ & =\\frac{4 R_{A} r^{2}}{r_{A} A I^{2}} . \\end{aligned} $$ From part (a), we know that $\\frac{R_{A}}{r_{A}}=\\frac{R_{D}}{r_{D}}$. Therefore, doing a similar calculation for $\\frac{I_{D} D}{I_{D} I}$, we get $$ \\begin{aligned} \\frac{T_{A} D}{T_{A} A} & =\\frac{I_{A} I}{I_{A} A} \\cdot \\frac{I_{D} D}{I_{D} I} \\\\ & =\\frac{4 R_{A} r^{2}}{r_{A} A I^{2}} \\cdot \\frac{r_{D} D I^{2}}{4 R_{D} r^{2}} \\\\ & =\\frac{D I^{2}}{A I^{2}} \\end{aligned} $$ Thus $T_{A}$ is the point where the tangent to $(A I D)$ at $I$ meets $A D$ and $T_{A} I^{2}=T_{A} A \\cdot T_{A} D$. This shows what we claimed at the start."} +{"year":2021,"label":"6","problem":"Triangles $A B C$ and $D E F$ share circumcircle $\\Omega$ and incircle $\\omega$ so that points $A, F$, $B, D, C$, and $E$ occur in this order along $\\Omega$. Let $\\Delta_{A}$ be the triangle formed by lines $A B, A C$, and $E F$, and define triangles $\\Delta_{B}, \\Delta_{C}, \\ldots, \\Delta_{F}$ similarly. Furthermore, let $\\Omega_{A}$ and $\\omega_{A}$ be the circumcircle and incircle of triangle $\\Delta_{A}$, respectively, and define circles $\\Omega_{B}, \\omega_{B}, \\ldots, \\Omega_{F}, \\omega_{F}$ similarly. (a) Prove that the two common external tangents to circles $\\Omega_{A}$ and $\\Omega_{D}$ and the two common external tangents to circles $\\omega_{A}$ and $\\omega_{D}$ are either concurrent or pairwise parallel. (b) Suppose that these four lines meet at point $T_{A}$, and define points $T_{B}$ and $T_{C}$ similarly. Prove that points $T_{A}, T_{B}$, and $T_{C}$ are collinear.","solution":" ![](https:\/\/cdn.mathpix.com\/cropped\/2024_11_19_148cb373ffbe5370bbc6g-17.jpg?height=818&width=754&top_left_y=1150&top_left_x=651) Let $I$ and $r$ be the center and radius of $\\omega$, and let $O$ and $R$ be the center and radius of $\\Omega$. Let $O_{A}$ and $I_{A}$ be the circumcenter and incenter of triangle $\\Delta_{A}$, and define $O_{B}$, $I_{B}, \\ldots, I_{F}$ similarly. Let $\\omega$ touch $E F$ at $A_{1}$, and define $B_{1}, C_{1}, \\ldots, F_{1}$ similarly. $$ \\left(\\Omega_{A} \\cup \\omega_{A}\\right) \\sim\\left(\\Omega_{D} \\cup \\omega_{D}\\right) $$ The four lines will concur at the homothetic center of these figures (possibly at infinity). Solution 5 (Ankit Bisain) As in the previous solution, it suffices to show that $\\frac{I_{A} I}{A I_{A}} \\cdot \\frac{D I_{D}}{I_{D} I}=\\frac{D I^{2}}{A I^{2}}$. Let $A I$ and $D I$ meet $\\Omega$ again at $M$ and $N$, respectively. Let $\\ell$ be the line parallel to $B C$ and tangent to $\\omega$ but different from $B C$. Then $$ \\frac{D I_{D}}{I_{D} I}=\\frac{d(D, B C)}{d(B C, \\ell)}=\\frac{D B \\cdot D C \/ 2 R}{2 r}=\\frac{M I^{2}-M D^{2}}{4 R r} $$ Since $I D M \\sim I A N$, we have $$ \\frac{D I_{D}}{I_{D} I} \\cdot \\frac{I_{A} I}{A I_{A}}=\\frac{M I^{2}-M D^{2}}{N I^{2}-N A^{2}}=\\frac{D I^{2}}{A I^{2}} $$ as desired. Remark (Author comments on generalization of part (b) with a circumscribed hexagram). Let triangles $A B C$ and $D E F$ be circumscribed about the same circle $\\omega$ so that they form a hexagram. However, we do not require anymore that they are inscribed in the same circle. Define circles $\\Omega_{A}, \\omega_{A}, \\ldots, \\omega_{F}$ as in the problem. Let $T_{A}^{\\text {Circ }}$ be the intersection point of the two common external tangents to circles $\\Omega_{A}$ and $\\Omega_{D}$, and define points $T_{B}^{\\text {Circ }}$ and $T_{C}^{\\text {Circ }}$ similarly. Also let $T_{A}^{\\text {In }}$ be the intersection point of the two common external tangents to circles $\\omega_{A}$ and $\\omega_{D}$, and define points $T_{B}^{\\mathrm{In}}$ and $T_{C}^{\\mathrm{In}}$ similarly. Then points $T_{A}^{\\text {Circ }}, T_{B}^{\\text {Circ }}$, and $T_{C}^{\\mathrm{Circ}}$ are collinear and points $T_{A}^{\\mathrm{In}}, T_{B}^{\\mathrm{In}}$, and $T_{C}^{\\mathrm{In}}$ are also collinear. For the incircles part of the generalisation, we start out as in the first solution to part (b) of the problem, and eventually we reduce everything to the following: Let points $A_{1}, B_{1}, C_{1}, D_{1}, E_{1}$, and $F_{1}$ lie on circle $\\omega$. Let lines $B_{1} C_{1}$ and $E_{1} F_{1}$ meet at point $X_{A}$, let the line through $A_{1}$ parallel to $B_{1} C_{1}$ and the line through $D_{1}$ parallel to $E_{1} F_{1}$ meet at point $Z_{A}$, and define points $X_{B}, Z_{B}, X_{C}$, and $Z_{C}$ similarly. Then lines $X_{A} Z_{A}$, $X_{B} Z_{B}$, and $X_{C} Z_{C}$ are concurrent. Take $\\omega$ as the unit circle and assign complex numbers $u, v, w, x, y$, and $z$ to points $A_{1}$, $F_{1}, B_{1}, D_{1}, C_{1}$, and $E_{1}$, respectively, so that when we permute $u, v, w, x, y$, and $z$ cyclically the configuration remains unchanged. Then by standard complex bash formulas we obtain that each two out of our three lines meet at $\\varphi \/ \\psi$, where $$ \\varphi=\\sum_{\\mathrm{Cyc}} u^{2} v w(w x-w y+x y)(y-z) $$ and $$ \\psi=-u^{2} w^{2} y^{2}-v^{2} x^{2} z^{2}-4 u v w x y z+\\sum_{\\mathrm{Cyc}} u^{2}(v w x y-v w x z+v w y z-v x y z+w x y z) $$ (But the calculations were too difficult for me to do by hand, so I used SymPy.) Remark (Author comments on generalization of part (b) with an inscribed hexagram). Let triangles $A B C$ and $D E F$ be inscribed inside the same circle $\\Omega$ so that they form a hexagram. However, we do not require anymore that they are circumscribed about the same circle. Define points $T_{A}^{\\text {Circ }}, T_{B}^{\\text {Circ }}, \\ldots, T_{C}^{\\mathrm{In}}$ as in the previous remark. It looks like once again points $T_{A}^{\\text {Circ }}, T_{B}^{\\text {Circ }}$, and $T_{C}^{\\text {Circ }}$ are collinear and points $T_{A}^{\\mathrm{In}}, T_{B}^{\\mathrm{In}}$, and $T_{C}^{\\mathrm{In}}$ are also collinear. However, I do not have proofs of these claims. Remark (Further generalization from Andrew Gu). Let $A B C$ and $D E F$ be triangles which share an inconic, or equivalently share a circumconic. Define points $T_{A}^{\\text {Circ }}, T_{B}^{\\text {Circ }}, \\ldots, T_{C}^{\\mathrm{In}}$ as in the previous remarks. Then it is conjectured that points $T_{A}^{\\text {Circ }}, T_{B}^{\\text {Circ }}$, and $T_{C}^{\\text {Circ }}$ are collinear and points $T_{A}^{\\mathrm{In}}, T_{B}^{\\mathrm{In}}$, and $T_{C}^{\\mathrm{In}}$ are also collinear. (Note that extraversion may be required depending on the configuration of points, e.g. excircles instead of incircles.) Additionally, it appears that the insimilicenters of the circumcircles lie on a line perpendicular to the line through $T_{A}^{\\text {Circ }}, T_{B}^{\\text {Circ }}$, and $T_{C}^{\\text {Circ }}$."} +{"year":2021,"label":"7","problem":"Let $M$ be a finite set of lattice points and $n$ be a positive integer. A mine-avoiding path is a path of lattice points with length $n$, beginning at $(0,0)$ and ending at a point on the line $x+y=n$, that does not contain any point in $M$. Prove that if there exists a mine-avoiding path, then there exist at least $2^{n-|M|}$ mine-avoiding paths.","solution":" \u3010 Solution 1. We prove the statement by induction on $n$. We use $n=0$ as a base case, where the statement follows from $1 \\geq 2^{-|M|}$. For the inductive step, let $n>0$. There exists at least one mine-avoiding path, which must pass through either $(0,1)$ or $(1,0)$. We consider two cases: Case 1: there exist mine-avoiding paths starting at both $(1,0)$ and $(0,1)$. By the inductive hypothesis, there are at least $2^{n-1-|M|}$ mine-avoiding paths starting from each of $(1,0)$ and $(0,1)$. Then the total number of mine-avoiding paths is at least $2^{n-1-|M|}+2^{n-1-|M|}=2^{n-|M|}$. Case 2: only one of $(1,0)$ and $(0,1)$ is on a mine-avoiding path. Without loss of generality, suppose no mine-avoiding path starts at $(0,1)$. Then some element of $M$ must be of the form $(0, k)$ for $1 \\leq k \\leq n$ in order to block the path along the $y$-axis. This element can be ignored for any mine-avoiding path starting at (1,0). By the inductive hypothesis, there are at least $2^{n-1-(|M|-1)}=2^{n-|M|}$ mine-avoiding paths. This completes the induction step, which solves the problem."} +{"year":2021,"label":"7","problem":"Let $M$ be a finite set of lattice points and $n$ be a positive integer. A mine-avoiding path is a path of lattice points with length $n$, beginning at $(0,0)$ and ending at a point on the line $x+y=n$, that does not contain any point in $M$. Prove that if there exists a mine-avoiding path, then there exist at least $2^{n-|M|}$ mine-avoiding paths.","solution":" ## \u3010 Solution 2. ## Lemma If $|M|d$, the pattern changes to $$ S_{n}=\\sum_{j=1}^{d}(-1)^{j+1} c_{j} S_{n-j} $$ ## Lemma All of the $c_{i}$ are integers except for $c_{d}$. Furthermore, $c_{d}$ is $1 \/ p$ times an integer. $$ \\Phi_{q}(x)=1+x^{p^{r-1}}+x^{2 p^{r-1}}+\\cdots+x^{(p-1) p^{r-1}} $$ The polynomial $$ Q(x)=1+(1+x)^{p^{r-1}}+(1+x)^{2 p^{r-1}}+\\cdots+(1+x)^{(p-1) p^{r-1}} $$ has roots $\\omega-1$ for $\\omega \\in S_{q}$, so it is equal to $p(-x)^{d} P(-1 \/ x)$ by comparing constant coefficients. Comparing the remaining coefficients, we find that $c_{n}$ is $1 \/ p$ times the $x^{n}$ coefficient of $Q$. Since $(x+y)^{p} \\equiv x^{p}+y^{p}(\\bmod p)$, we conclude that, modulo $p$, $$ \\begin{aligned} Q(x) & \\equiv 1+\\left(1+x^{p^{r-1}}\\right)+\\left(1+x^{p^{r-1}}\\right)^{2}+\\cdots+\\left(1+x^{p^{r-1}}\\right)^{p-1} \\\\ & \\equiv\\left[\\left(1+x^{p^{r-1}}\\right)^{p}-1\\right] \/ x^{p^{r-1}} \\end{aligned} $$ Since $\\binom{p}{j}$ is a multiple of $p$ when $0d$ : - If $\\nu_{p}\\left(S_{n-j}\\right) \\geq \\ell$ for all $1 \\leq j \\leq d$ and $\\nu_{p}\\left(S_{n-d}\\right) \\geq \\ell+1$, then $\\nu_{p}\\left(S_{n}\\right) \\geq \\ell$. - If $\\nu_{p}\\left(S_{n-j}\\right) \\geq \\ell$ for all $1 \\leq j \\leq d$ and $\\nu_{p}\\left(S_{n-d}\\right)=\\ell$, then $\\nu_{p}\\left(S_{n}\\right)=\\ell-1$. Together, these prove the claim by induction. By the claim, the smallest $n$ for which $\\nu_{p}\\left(S_{n}\\right)<0$ (equivalent to $S_{n}$ not being an integer, by the recurrences) is $$ n=(r-1) d+m+1=((p-1) r-1) p^{r-1}+1 $$ Remark. The original proposal was the following more general version: Let $n$ be an integer with prime power factorization $q_{1} \\cdots q_{m}$. Let $S_{n}$ denote the set of primitive $n$th roots of unity. Find all tuples of nonnegative integers $\\left(z_{1}, \\ldots, z_{m}\\right)$ such that $$ \\sum_{\\omega \\in S_{n}} \\frac{f(\\omega)}{\\left(1-\\omega^{n \/ q_{1}}\\right)^{z_{1}} \\cdots\\left(1-\\omega^{n \/ q_{m}}\\right)^{z_{m}}} \\in \\mathbb{Z} $$ for all polynomials $f \\in \\mathbb{Z}[x]$. The maximal $z_{i}$ are exponents in the prime ideal factorization of the different ideal of the cyclotomic extension $\\mathbb{Q}\\left(\\zeta_{n}\\right) \/ \\mathbb{Q}$. Remark. Let $F=\\left(x^{p}-1\\right) \/(x-1)$ be the minimal polynomial of $\\zeta_{p}=e^{2 \\pi i \/ p}$ over $\\mathbb{Q}$. A calculation of Euler shows that $$ \\left(\\mathbb{Z}\\left[\\zeta_{p}\\right]\\right)^{*}:=\\left\\{\\alpha=g\\left(\\zeta_{p}\\right) \\in \\mathbb{Q}\\left[\\zeta_{p}\\right]: \\sum_{\\omega \\in S_{p}} f(\\omega) g(\\omega) \\in \\mathbb{Z} \\forall f \\in \\mathbb{Z}[x]\\right\\}=\\frac{1}{F^{\\prime}\\left(\\zeta_{p}\\right)} \\cdot \\mathbb{Z}\\left[\\zeta_{p}\\right], $$ where $$ F^{\\prime}\\left(\\zeta_{p}\\right)=\\frac{p \\zeta_{p}^{p-1}-\\left[1+\\zeta_{p}+\\cdots+\\zeta_{p}^{p-1}\\right]}{1-\\zeta_{p}}=p\\left(1-\\zeta_{p}\\right)^{-1} \\zeta_{p}^{p-1} $$ is $\\left(1-\\zeta_{p}\\right)^{[p-1]-1}=\\left(1-\\zeta_{p}\\right)^{p-2}$ times a unit of $\\mathbb{Z}\\left[\\zeta_{p}\\right]$. Here, $\\left(\\mathbb{Z}\\left[\\zeta_{p}\\right]\\right)^{*}$ is the dual lattice of $\\mathbb{Z}\\left[\\zeta_{p}\\right]$. Remark. Let $K=\\mathbb{Q}(\\omega)$, so $(p)$ factors as $(1-\\omega)^{p-1}$ in the ring of integers $\\mathcal{O}_{K}$ (which, for cyclotomic fields, can be shown to be $\\mathbb{Z}[\\omega])$. In particular, the ramification index $e$ of $(1-\\omega)$ over $p$ is the exponent, $p-1$. Since $e=p-1$ is not divisible by $p$, we have so-called tame ramification. Now by the ramification theory of Dedekind's different ideal, the exponent $z_{1}$ that works when $n=p$ is $e-1=p-2$. Higher prime powers are more interesting because of wild ramification: $p$ divides $\\phi\\left(p^{r}\\right)=$ $p^{r-1}(p-1)$ if and only if $r>1$. (This is a similar phenomena to how Hensel's lemma for $x^{2}-c$ is more interesting mod powers of 2 than mod odd prime powers.) Remark. Let $F=\\left(x^{q}-1\\right) \/\\left(x^{q \/ p}-1\\right)$ be the minimal polynomial of $\\zeta_{q}=e^{2 \\pi i \/ q}$ over $\\mathbb{Q}$. The aforementioned calculation of Euler shows that $$ \\left(\\mathbb{Z}\\left[\\zeta_{q}\\right]\\right)^{*}:=\\left\\{\\alpha=g\\left(\\zeta_{q}\\right) \\in \\mathbb{Q}\\left[\\zeta_{q}\\right]: \\sum_{\\omega \\in S_{q}} f(\\omega) g(\\omega) \\in \\mathbb{Z} \\forall f \\in \\mathbb{Z}[x]\\right\\}=\\frac{1}{F^{\\prime}\\left(\\zeta_{q}\\right)} \\cdot \\mathbb{Z}\\left[\\zeta_{q}\\right] $$ where the chain rule implies (using the computation from the prime case) $$ F^{\\prime}\\left(\\zeta_{q}\\right)=\\left[p\\left(1-\\zeta_{p}\\right)^{-1} \\zeta_{p}^{p-1}\\right] \\cdot \\frac{q}{p} \\zeta_{q}^{(q \/ p)-1}=q\\left(1-\\zeta_{p}\\right)^{-1} \\zeta_{q}^{-1} $$ is $\\left(1-\\zeta_{q}\\right)^{r \\phi(q)-q \/ p}=\\left(1-\\zeta_{q}\\right)^{z_{q}}$ times a unit of $\\mathbb{Z}\\left[\\zeta_{q}\\right]$."} diff --git a/USA_TST_TST/segmented/en-sols-TSTST-2022.jsonl b/USA_TST_TST/segmented/en-sols-TSTST-2022.jsonl new file mode 100644 index 0000000000000000000000000000000000000000..0058c5f52ad270ddf8d9572537cb1fddca1cc80a --- /dev/null +++ b/USA_TST_TST/segmented/en-sols-TSTST-2022.jsonl @@ -0,0 +1,12 @@ +{"year":2022,"label":"1","problem":"Let $n$ be a positive integer. Find the smallest positive integer $k$ such that for any set $S$ of $n$ points in the interior of the unit square, there exists a set of $k$ rectangles such that the following hold: - The sides of each rectangle are parallel to the sides of the unit square. - Each point in $S$ is not in the interior of any rectangle. - Each point in the interior of the unit square but not in $S$ is in the interior of at least one of the $k$ rectangles. (The interior of a polygon does not contain its boundary.)","solution":" We claim the answer is $k=2 n+2$. The lower bound is given by picking $$ S=\\left\\{\\left(s_{1}, s_{1}\\right),\\left(s_{2}, s_{2}\\right), \\ldots,\\left(s_{n}, s_{n}\\right)\\right\\} $$ for some real numbers $00$. The four rectangles covering each of $$ \\left(s_{1}-\\varepsilon, s_{1}\\right),\\left(s_{1}, s_{1}-\\varepsilon\\right),\\left(s_{n}+\\varepsilon, s_{n}\\right),\\left(s_{n}, s_{n}+\\varepsilon\\right) $$ cannot cover any other points in $S^{\\prime}$; all other rectangles can only cover at most 2 points in $S^{\\prime}$, giving a bound of $$ k \\geq 4+\\frac{\\left|S^{\\prime}\\right|-4}{2}=2 n+2 $$ ![](https:\/\/cdn.mathpix.com\/cropped\/2024_11_19_1799270e91f74486a977g-05.jpg?height=812&width=806&top_left_y=242&top_left_x=628) To prove that $2 n+2$ rectangles are sufficient, assume that the number of distinct $y$-coordinates is at least the number of distinct $x$-coordinates. Let $$ 0=x_{0}0$ gives a total of $$ (m+n+2)+(n-m)=2 n+2 $$ rectangles."} +{"year":2022,"label":"2","problem":"Let $A B C$ be a triangle. Let $\\theta$ be a fixed angle for which $$ \\theta<\\frac{1}{2} \\min (\\angle A, \\angle B, \\angle C) $$ Points $S_{A}$ and $T_{A}$ lie on segment $B C$ such that $\\angle B A S_{A}=\\angle T_{A} A C=\\theta$. Let $P_{A}$ and $Q_{A}$ be the feet from $B$ and $C$ to $\\overline{A S_{A}}$ and $\\overline{A T_{A}}$ respectively. Then $\\ell_{A}$ is defined as the perpendicular bisector of $\\overline{P_{A} Q_{A}}$. Define $\\ell_{B}$ and $\\ell_{C}$ analogously by repeating this construction two more times (using the same value of $\\theta$ ). Prove that $\\ell_{A}, \\ell_{B}$, and $\\ell_{C}$ are concurrent or all parallel.","solution":" We discard the points $S_{A}$ and $T_{A}$ since they are only there to direct the angles correctly in the problem statement. \u3010 First solution, by author. Let $X$ be the projection from $C$ to $A P_{A}, Y$ be the projection from $B$ to $A Q_{A}$. ![](https:\/\/cdn.mathpix.com\/cropped\/2024_11_19_1799270e91f74486a977g-07.jpg?height=809&width=792&top_left_y=1292&top_left_x=632) Claim - Line $\\ell_{A}$ passes through $M_{A}$, the midpoint of $B C$. Also, quadrilateral $P_{A} Q_{A} Y X$ is cyclic with circumcenter $M_{A}$. $$ A P_{A} \\cdot A X=A B \\cdot A C \\cdot \\cos \\theta \\cos (\\angle A-\\theta)=A Q_{A} \\cdot A Y $$ it follows that $P_{A}, Q_{A}, Y, X$ are concyclic by power of a point. Moreover, by projection, the perpendicular bisector of $P_{A} X$ passes through $M_{A}$, similar for $Q_{A} Y$, implying that $M_{A}$ is the center of $P_{A} Q_{A} Y X$. Hence $\\ell_{A}$ passes through $M_{A}$. $$ \\text { Claim }-\\measuredangle\\left(M_{A} M_{C}, \\ell_{A}\\right)=\\measuredangle Y P_{A} Q_{A} $$ Therefore, $$ \\frac{\\sin \\angle\\left(M_{A} M_{C}, \\ell_{A}\\right)}{\\sin \\angle\\left(\\ell_{A}, M_{A} M_{B}\\right)}=\\frac{\\sin \\angle Y P_{A} Q_{A}}{\\sin \\angle P_{A} Q_{A} X}=\\frac{Y Q_{A}}{X P_{A}}=\\frac{B C \\sin (\\angle C+\\theta)}{B C \\sin (\\angle B+\\theta)}=\\frac{\\sin (\\angle C+\\theta)}{\\sin (\\angle B+\\theta)}, $$ and we conclude by trig Ceva theorem."} +{"year":2022,"label":"2","problem":"Let $A B C$ be a triangle. Let $\\theta$ be a fixed angle for which $$ \\theta<\\frac{1}{2} \\min (\\angle A, \\angle B, \\angle C) $$ Points $S_{A}$ and $T_{A}$ lie on segment $B C$ such that $\\angle B A S_{A}=\\angle T_{A} A C=\\theta$. Let $P_{A}$ and $Q_{A}$ be the feet from $B$ and $C$ to $\\overline{A S_{A}}$ and $\\overline{A T_{A}}$ respectively. Then $\\ell_{A}$ is defined as the perpendicular bisector of $\\overline{P_{A} Q_{A}}$. Define $\\ell_{B}$ and $\\ell_{C}$ analogously by repeating this construction two more times (using the same value of $\\theta$ ). Prove that $\\ell_{A}, \\ell_{B}$, and $\\ell_{C}$ are concurrent or all parallel.","solution":" We discard the points $S_{A}$ and $T_{A}$ since they are only there to direct the angles correctly in the problem statement. \u3010 Second solution via Jacobi, by Maxim Li. Let $D$ be the foot of the $A$-altitude. Note that line $B C$ is the external angle bisector of $\\angle P_{A} D Q_{A}$. Claim - $\\left(D P_{A} Q_{A}\\right)$ passes through the midpoint $M_{A}$ of $B C$. It follows that $M_{A}$ lies on $\\ell_{A}$; we need to identify a second point. We'll use the circumcenter $O_{A}$ of $\\left(D P_{A} Q_{A}\\right)$. The perpendicular bisector of $D P_{A}$ passes through $M_{C}$; indeed, we can easily show the angle it makes with $M_{C} M_{A}$ is $90^{\\circ}-\\theta-C$, so $\\angle O_{A} M_{C} M_{A}=90-\\theta-C$, and then by analogous angle-chasing we can finish with Jacobi's theorem on $\\triangle M_{A} M_{B} M_{C}$."} +{"year":2022,"label":"3","problem":"Determine all positive integers $N$ for which there exists a strictly increasing sequence of positive integers $s_{0}2^{21}$ possible colorings. If Bob makes less than 22 queries, then he can only output $2^{21}$ possible colorings, which means he is wrong on some coloring. Now we show Bob can always win in 22 queries. A key observation is that the set of red points is convex, as is the set of blue points, so if a set of points is all the same color, then their convex hull is all the same color. ## Lemma Let $B_{0}, \\ldots, B_{k+1}$ be equally spaced points on a circular arc such that colors of $B_{0}$ and $B_{k+1}$ differ and are known. Then it is possible to determine the colors of $B_{1}$, $\\ldots, B_{k}$ in $\\left\\lceil\\log _{2} k\\right\\rceil$ queries. ## Lemma Let $B_{0}, \\ldots, B_{k+1}$ be equally spaced points on a circular arc such that colors of $B_{0}, B_{\\lceil k \/ 2\\rceil}, B_{k+1}$ are both red and are known. Then at least one of the following holds: all of $B_{1}, \\ldots, B_{\\lceil k \/ 2\\rceil}$ are red or all of $B_{\\lceil k \/ 2\\rceil}, \\ldots, B_{k}$ are red. Furthermore, in one query we can determine which one of the cases holds. Now the strategy is: Bob picks $A_{1}$. WLOG it is red. Now suppose Bob does not know the colors of $\\leq 2^{k}-1$ points $A_{i}, \\ldots, A_{j}$ with $j-i+1 \\leq 2^{k}-1$ and knows the rest are red. I claim Bob can win in $2 k-1$ queries. First, if $k=1$, there is one point and he wins by querying the point, so the base case holds, so assume $k>1$. Bob queries $A_{i+\\lceil(j-i+1) \/ 2\\rceil}$. If it is blue, he finishes in $2 \\log _{2}\\lceil(j-i+1) \/ 2\\rceil \\leq 2(k-1)$ queries by the first lemma, for a total of $2 k-1$ queries. If it is red, he can query one more point and learn some half of $A_{i}, \\ldots, A_{j}$ that are red by the second lemma, and then he has reduced it to the case with $\\leq 2^{k-1}-1$ points in two queries, at which point we induct."} +{"year":2022,"label":"6","problem":"Let $O$ and $H$ be the circumcenter and orthocenter, respectively, of an acute scalene triangle $A B C$. The perpendicular bisector of $\\overline{A H}$ intersects $\\overline{A B}$ and $\\overline{A C}$ at $X_{A}$ and $Y_{A}$ respectively. Let $K_{A}$ denote the intersection of the circumcircles of triangles $O X_{A} Y_{A}$ and $B O C$ other than $O$. Define $K_{B}$ and $K_{C}$ analogously by repeating this construction two more times. Prove that $K_{A}, K_{B}, K_{C}$, and $O$ are concyclic.","solution":" \\ First solution, by author. Let $\\odot O X_{A} Y_{A}$ intersects $A B, A C$ again at $U, V$. Then by Reim's theorem $U V C B$ are concyclic. Hence the radical axis of $\\odot O X_{A} Y_{A}, \\odot O B C$ and $\\odot(U V C B)$ are concurrent, i.e. $O K_{A}, B C, U V$ are concurrent, Denote the intersection as $K_{A}^{*}$, which is indeed the inversion of $K_{A}$ with respect to $\\odot O$. (The inversion sends $\\odot O B C$ to the line $B C$ ). Let $P_{A}, P_{B}, P_{C}$ be the circumcenters of $\\triangle O B C, \\triangle O C A, \\triangle O A B$ respectively. Claim $-K_{A}^{*}$ coincides with the intersection of $P_{B} P_{C}$ and $B C$. Finally by Desargue's theorem, it suffices to show that $A P_{A}, B P_{B}, C P_{C}$ are concurrent. Note that $$ \\begin{aligned} & d\\left(P_{A}, A B\\right)=P_{A} B \\sin \\left(90^{\\circ}+\\angle C-\\angle A\\right) \\\\ & d\\left(P_{A}, A C\\right)=P_{A} C \\sin \\left(90^{\\circ}+\\angle B-\\angle A\\right) \\end{aligned} $$ Hence the symmetric product and trig Ceva finishes the proof. ![](https:\/\/cdn.mathpix.com\/cropped\/2024_11_19_1799270e91f74486a977g-15.jpg?height=1132&width=1194&top_left_y=245&top_left_x=431)"} +{"year":2022,"label":"6","problem":"Let $O$ and $H$ be the circumcenter and orthocenter, respectively, of an acute scalene triangle $A B C$. The perpendicular bisector of $\\overline{A H}$ intersects $\\overline{A B}$ and $\\overline{A C}$ at $X_{A}$ and $Y_{A}$ respectively. Let $K_{A}$ denote the intersection of the circumcircles of triangles $O X_{A} Y_{A}$ and $B O C$ other than $O$. Define $K_{B}$ and $K_{C}$ analogously by repeating this construction two more times. Prove that $K_{A}, K_{B}, K_{C}$, and $O$ are concyclic.","solution":" I Second solution, from Jeffrey Kwan. Let $O_{A}$ be the circumcenter of $\\triangle A X_{A} Y_{A}$. The key claim is that: Claim $-O_{A} X_{A} Y_{A} O$ is cyclic. $$ \\frac{A X_{A}}{A B}=\\frac{A H \/ 2}{A D}=\\frac{R \\cos A}{A D} $$ and so $$ \\frac{A O}{A D}=R \\cdot \\frac{A X_{A}}{A B \\cdot R \\cos A}=\\frac{A X_{A}}{A E}=\\frac{A Y_{A}}{A F} $$ Hence $\\angle X_{A} O Y_{A}=180^{\\circ}-2 \\angle A=180^{\\circ}-\\angle X_{A} O_{A} Y_{A}$, which proves the claim. Let $P_{A}$ be the circumcenter of $\\triangle O B C$, and define $P_{B}, P_{C}$ similarly. By the claim, $A$ is the exsimilicenter of $\\left(O X_{A} Y_{A}\\right)$ and $(O B C)$, so $A P_{A}$ is the line between their two centers. In particular, $A P_{A}$ is the perpendicular bisector of $O K_{A}$. ![](https:\/\/cdn.mathpix.com\/cropped\/2024_11_19_1799270e91f74486a977g-16.jpg?height=1303&width=1020&top_left_y=245&top_left_x=521) Claim $-A P_{A}, B P_{B}, C P_{C}$ concur at $T$. Now $T$ lies on the perpendicular bisectors of $O K_{A}, O K_{B}$, and $O K_{C}$. Hence $O K_{A} K_{B} K_{C}$ is cyclic with center $T$, as desired."} +{"year":2022,"label":"7","problem":"Let $A B C D$ be a parallelogram. Point $E$ lies on segment $C D$ such that $$ 2 \\angle A E B=\\angle A D B+\\angle A C B, $$ and point $F$ lies on segment $B C$ such that $$ 2 \\angle D F A=\\angle D C A+\\angle D B A . $$ Let $K$ be the circumcenter of triangle $A B D$. Prove that $K E=K F$.","solution":" Let the circle through $A, B$, and $E$ intersect $C D$ again at $E^{\\prime}$, and let the circle through $D$, $A$, and $F$ intersect $B C$ again at $F^{\\prime}$. Now $A B E E^{\\prime}$ and $D A F^{\\prime} F$ are cyclic quadrilaterals with two parallel sides, so they are isosceles trapezoids. From $K A=K B$, it now follows that $K E=K E^{\\prime}$, whereas from $K A=K D$ it follows that $K F=K F^{\\prime}$. Next, let the circle through $A, B$, and $E$ intersect $A C$ again at $S$. Then $$ \\angle A S B=\\angle A E B=\\frac{1}{2}(\\angle A D B+\\angle A C B)=\\frac{1}{2}(\\angle A D B+\\angle D A C)=\\frac{1}{2} \\angle A M B, $$ where $M$ is the intersection of $A C$ and $B D$. From $\\angle A S B=\\frac{1}{2} \\angle A M B$, it follows that $M S=M B$, so $S$ is the point on $M C$ such that $M S=M B=M D$. By symmetry, the circle through $A, D$, and $F$ also passes through $S$, and it follows that the line $A S$ is the radical axis of the circles $(A B E)$ and $(A D F)$. By power of a point, we now obtain $$ C E \\cdot C E^{\\prime}=C S \\cdot C A=C F \\cdot C F^{\\prime}, $$ from which it follows that $E, F, E^{\\prime}$, and $F^{\\prime}$ are concyclic. The segments $E E^{\\prime}$ and $F F^{\\prime}$ are not parallel, so their perpendicular bisectors only meet at one point, which is $K$. Hence $K E=K F$."} +{"year":2022,"label":"8","problem":"Find all functions $f: \\mathbb{N} \\rightarrow \\mathbb{Z}$ such that $$ \\left\\lfloor\\frac{f(m n)}{n}\\right\\rfloor=f(m) $$ for all positive integers $m, n$.","solution":" There are two families of functions that work: for each $\\alpha \\in \\mathbb{R}$ the function $f(n)=\\lfloor\\alpha n\\rfloor$, and for each $\\alpha \\in \\mathbb{R}$ the function $f(n)=\\lceil\\alpha n\\rceil-1$. (For irrational $\\alpha$ these two functions coincide.) It is straightforward to check that these functions indeed work; essentially, this follows from the identity $$ \\left\\lfloor\\frac{\\lfloor x n\\rfloor}{n}\\right\\rfloor=\\lfloor x\\rfloor $$ which holds for all positive integers $n$ and real numbers $x$. We now show that every function that works must be of one of the above forms. Let $f$ be a function that works, and define the sequence $a_{1}, a_{2}, \\ldots$ by $a_{n}=f(n!) \/ n!$. Applying the give condition with $(n!, n+1)$ yields $a_{n+1} \\in\\left[a_{n}, a_{n}+\\frac{1}{n!}\\right)$. It follows that the sequence $a_{1}, a_{2}, \\ldots$ is non-decreasing and bounded from above by $a_{1}+e$, so this sequence must converge to some limit $\\alpha$. If there exists a $k$ such that $a_{k}=\\alpha$, then we have $a_{\\ell}=\\alpha$ for all $\\ell>k$. For each positive integer $m$, there exists $\\ell>k$ such that $m \\mid \\ell$ !. Plugging in $m n=\\ell$ !, it then follows that $$ f(m)=\\left\\lfloor\\frac{f(\\ell!)}{\\ell!\/ m}\\right\\rfloor=\\lfloor\\alpha m\\rfloor $$ for all $m$, so $f$ is of the desired form. If there does not exist a $k$ such that $a_{k}=\\alpha$, we must have $a_{k}<\\alpha$ for all $k$. For each positive integer $m$, we can now pick an $\\ell$ such that $m \\mid \\ell$ ! and $a_{\\ell}=\\alpha-x$ with $x$ arbitrarily small. It then follows from plugging in $m n=\\ell$ ! that $$ f(m)=\\left\\lfloor\\frac{f(\\ell!)}{\\ell!\/ m}\\right\\rfloor=\\left\\lfloor\\frac{\\ell!(\\alpha-x)}{\\ell!\/ m}\\right\\rfloor=\\lfloor\\alpha m-m x\\rfloor . $$ If $\\alpha m$ is an integer we can choose $\\ell$ such that $m x<1$, and it follows that $f(m)=\\lceil\\alpha m\\rceil-1$. If $\\alpha m$ is not an integer we can choose $\\ell$ such that $m x<\\{\\alpha m\\}$, and it also follows that $f(m)=\\lceil\\alpha m\\rceil-1$. We conclude that in this case $f$ is again of the desired form."} +{"year":2022,"label":"9","problem":"Let $k>1$ be a fixed positive integer. Prove that if $n$ is a sufficiently large positive integer, there exists a sequence of integers with the following properties: - Each element of the sequence is between 1 and $n$, inclusive. - For any two different contiguous subsequences of the sequence with length between 2 and $k$ inclusive, the multisets of values in those two subsequences is not the same. - The sequence has length at least $0.499 n^{2}$.","solution":" For any positive integer $n$, define an $(n, k)$-good sequence to be a finite sequence of integers each between 1 and $n$ inclusive satisfying the second property in the problem statement. The problems asks to show that, for all sufficiently large integers $n$, there is an $(n, k)$-good sequence of length at least $0.499 n^{2}$. Fix $k \\geq 2$ and consider some prime power $n=p^{m}$ with $p>k+1$. Consider some $0k+1$ be a prime. Then for $n=p^{2}$ we can find a $(n, k)$-good sequence of length $\\frac{p(p-1)\\left(p^{2}+2\\right)}{2}$. We will prove a stronger statement that $g\\frac{\\left(\\sum_{k=m}^{n} \\frac{1}{k^{2}}\\right)^{2}}{\\frac{1}{m}} \\end{aligned} $$ as desired. Remark (Bound on error). Let $A=\\sum_{k=m}^{n} k^{-2}$ and $B=\\sum_{k=m}^{n} k^{-3}$. The inequality above becomes tighter for large $m$ and $n \\gg m$. If we use Lagrange's identity in place of Cauchy-Schwarz, we get $$ A+B-m A^{2}=m \\cdot \\sum_{m \\leq aA+B $$ Remark (Construction commentary, from author). My motivation was to write an inequality where Titu could be applied creatively to yield a telescoping sum. This can be difficult because most of the time, such a reverse-engineered inequality will be so loose it's trivial anyways. My first attempt was the not-so-amazing inequality $$ \\frac{n^{2}+3 n}{2}=\\sum_{1}^{n} i+1=\\sum_{1}^{n} \\frac{\\frac{1}{i}}{\\frac{1}{i(i+1)}}>\\left(\\sum_{1}^{n} \\frac{1}{\\sqrt{i}}\\right)^{2} $$ which is really not surprising given that $\\sum \\frac{1}{\\sqrt{i}} \\ll \\frac{n}{\\sqrt{2}}$. The key here is that we need \"near-equality\" as dictated by the Cauchy-Schwarz equality case, i.e. the square root of the numerators should be approximately proportional to the denominators. This motivates using $\\frac{1}{i^{4}}$ as the numerator, which works like a charm. After working out the resulting statement, the LHS and RHS even share a sum, which adds to the simplicity of the problem. The final touch was to unrestrict the starting value of the sum, since this allows the strength of the estimate $\\frac{1}{i^{2}} \\approx \\frac{1}{i(i+1)}$ to be fully exploited."} +{"year":2023,"label":"2","problem":"Let $n \\geq m \\geq 1$ be integers. Prove that $$ \\sum_{k=m}^{n}\\left(\\frac{1}{k^{2}}+\\frac{1}{k^{3}}\\right) \\geq m \\cdot\\left(\\sum_{k=m}^{n} \\frac{1}{k^{2}}\\right)^{2} $$","solution":" \u3010 Second approach by inducting down, Luke Robitaille and Carl Schildkraut. Fix $n$; we'll induct downwards on $m$. For the base case of $n=m$ the result is easy, since the left side is $\\frac{m+1}{m^{3}}$ and the right side is $\\frac{m}{m^{4}}=\\frac{1}{m^{3}}$. For the inductive step, suppose we have shown the result for $m+1$. Let $$ A=\\sum_{k=m+1}^{n} \\frac{1}{k^{2}} \\quad \\text { and } \\quad B=\\sum_{k=m+1}^{n} \\frac{1}{k^{3}} $$ We know $A+B \\geq(m+1) A^{2}$, and we want to show $$ \\left(A+\\frac{1}{m^{2}}\\right)+\\left(B+\\frac{1}{m^{3}}\\right) \\geq m\\left(A+\\frac{1}{m^{2}}\\right)^{2} $$ Indeed, $$ \\begin{aligned} \\left(A+\\frac{1}{m^{2}}\\right)+\\left(B+\\frac{1}{m^{3}}\\right)-m\\left(A+\\frac{1}{m^{2}}\\right)^{2} & =A+B+\\frac{m+1}{m^{3}}-m A^{2}-\\frac{2 A}{m}-\\frac{1}{m^{3}} \\\\ & =\\left(A+B-(m+1) A^{2}\\right)+\\left(A-\\frac{1}{m}\\right)^{2} \\geq 0 \\end{aligned} $$ and we are done."} +{"year":2023,"label":"2","problem":"Let $n \\geq m \\geq 1$ be integers. Prove that $$ \\sum_{k=m}^{n}\\left(\\frac{1}{k^{2}}+\\frac{1}{k^{3}}\\right) \\geq m \\cdot\\left(\\sum_{k=m}^{n} \\frac{1}{k^{2}}\\right)^{2} $$","solution":" I Third approach by reducing $n \\rightarrow \\infty$, Michael Ren and Carl Schildkraut. First, we give: Claim (Reduction to $n \\rightarrow \\infty$ ) - If the problem is true when $n \\rightarrow \\infty$, it is true for all $n$. However, the region is bounded by a convex curve, and the sequence of points $(0,0)$, $\\left(\\frac{1}{m^{2}}, \\frac{1}{m^{3}}\\right),\\left(\\frac{1}{m^{2}}+\\frac{1}{(m+1)^{2}}, \\frac{1}{m^{3}}+\\frac{1}{(m+1)^{3}}\\right), \\ldots$ has successively decreasing slopes between consecutive points. Thus it suffices to check that the inequality is true when $n \\rightarrow \\infty$. Set $n=\\infty$ henceforth. Let $$ A=\\sum_{k=m}^{\\infty} \\frac{1}{k^{2}} \\text { and } B=\\sum_{k=m}^{\\infty} \\frac{1}{k^{3}} $$ we want to show $B \\geq m A^{2}-A$, which rearranges to $$ 1+4 m B \\geq(2 m A-1)^{2} $$ Write $$ C=\\sum_{k=m}^{\\infty} \\frac{1}{k^{2}(2 k-1)(2 k+1)} \\text { and } D=\\sum_{k=m}^{\\infty} \\frac{8 k^{2}-1}{k^{3}(2 k-1)^{2}(2 k+1)^{2}} $$ Then $$ \\frac{2}{2 k-1}-\\frac{2}{2 k+1}=\\frac{1}{k^{2}}+\\frac{1}{k^{2}(2 k-1)(2 k+1)} $$ and $$ \\frac{2}{(2 k-1)^{2}}-\\frac{2}{(2 k+1)^{2}}=\\frac{1}{k^{3}}+\\frac{8 k^{2}-1}{k^{3}(2 k-1)^{2}(2 k+1)^{2}}, $$ so that $$ A=\\frac{2}{2 m-1}-C \\text { and } B=\\frac{2}{(2 m-1)^{2}}-D $$ Our inequality we wish to show becomes $$ \\frac{2 m+1}{2 m-1} C \\geq D+m C^{2} $$ We in fact show two claims: Claim - We have $$ \\frac{2 m+1 \/ 2}{2 m-1} C \\geq D $$ $$ \\frac{2 m+1 \/ 2}{2 m-1} \\cdot \\frac{1}{k^{2}(2 k-1)(2 k+1)} \\geq \\frac{8 k^{2}-1}{k^{3}(2 k-1)^{2}(2 k+1)^{2}} $$ for $k \\geq m$. It suffices to show $$ \\frac{2 k+1 \/ 2}{2 k-1} \\cdot \\frac{1}{k^{2}(2 k-1)(2 k+1)} \\geq \\frac{8 k^{2}-1}{k^{3}(2 k-1)^{2}(2 k+1)^{2}} $$ which is equivalent to $k(2 k+1 \/ 2)(2 k+1) \\geq 8 k^{2}-1$. This holds for all $k \\geq 1$. ## Claim - We have $$ \\frac{1 \/ 2}{2 m-1} C \\geq m C^{2} $$ $$ \\frac{1}{2 m(2 m-1)}=\\sum_{k=m}^{\\infty}\\left(\\frac{1}{2 k(2 k-1)}-\\frac{1}{2(k+1)(2 k+1)}\\right)=\\sum_{k=m}^{\\infty} \\frac{4 k+1}{2 k(2 k-1)(k+1)(2 k+1)} $$ comparing term-wise with the definition of $C$ and using the inequality $k(4 k+1) \\geq 2(k+1)$ for $k \\geq 1$ gives the desired result."} +{"year":2023,"label":"2","problem":"Let $n \\geq m \\geq 1$ be integers. Prove that $$ \\sum_{k=m}^{n}\\left(\\frac{1}{k^{2}}+\\frac{1}{k^{3}}\\right) \\geq m \\cdot\\left(\\sum_{k=m}^{n} \\frac{1}{k^{2}}\\right)^{2} $$","solution":" \u300e Fourth approach by bashing, Carl Schildkraut. With a bit more work, the third approach can be adapted to avoid the $n \\rightarrow \\infty$ reduction. Similarly to before, define $$ A=\\sum_{k=m}^{n} \\frac{1}{k^{2}} \\text { and } B=\\sum_{k=m}^{n} \\frac{1}{k^{3}} $$ we want to show $1+4 m B \\geq(2 m A-1)^{2}$. Writing $$ C=\\sum_{k=m}^{n} \\frac{1}{k^{2}(2 k-1)(2 k+1)} \\text { and } D=\\sum_{k=m}^{n} \\frac{8 k^{2}-1}{k^{3}(2 k-1)^{2}(2 k+1)^{2}} $$ We compute $$ A=\\frac{2}{2 m-1}-\\frac{2}{2 n+1}-C \\text { and } B=\\frac{2}{(2 m-1)^{2}}-\\frac{2}{(2 n+1)^{2}}-D . $$ Then, the inequality we wish to show reduces (as in the previous solution) to $$ \\frac{2 m+1}{2 m-1} C+\\frac{2(2 m+1)}{(2 m-1)(2 n+1)} \\geq D+m C^{2}+\\frac{2(2 m+1)}{(2 n+1)^{2}}+\\frac{4 m}{2 n+1} C $$ We deal first with the terms not containing the variable $n$, i.e. we show that $$ \\frac{2 m+1}{2 m-1} C \\geq D+m C^{2} $$ For this part, the two claims from the previous solution go through exactly as written above, and we have $C \\leq 1 \/(2 m(2 m-1))$. We now need to show $$ \\frac{2(2 m+1)}{(2 m-1)(2 n+1)} \\geq \\frac{2(2 m+1)}{(2 n+1)^{2}}+\\frac{4 m}{2 n+1} C $$ (this is just the inequality between the remaining terms); our bound on $C$ reduces this to proving $$ \\frac{4(2 m+1)(n-m+1)}{(2 m-1)(2 n+1)^{2}} \\geq \\frac{2}{(2 m-1)(2 n+1)} $$ Expanding and writing in terms of $n$, this is equivalent to $$ n \\geq \\frac{1+2(m-1)(2 m+1)}{4 m}=m-\\frac{2 m+1}{4 m} $$ which holds for all $n \\geq m$."} +{"year":2023,"label":"3","problem":"Find all positive integers $n$ for which it is possible to color some cells of an infinite grid of unit squares red, such that each rectangle consisting of exactly $n$ cells (and whose edges lie along the lines of the grid) contains an odd number of red cells.","solution":" We claim that this is possible for all positive integers $n$. Call a positive integer for which such a coloring is possible good. To show that all positive integers $n$ are good we prove the following: (i) If $n$ is good and $p$ is an odd prime, then $p n$ is good; (ii) For every $k \\geq 0$, the number $n=2^{k}$ is good. Together, (i) and (ii) imply that all positive integers are good. Thus every coloring that works for $n$ automatically also works for $p n$. Claim - For each of these $k+1$ shapes, there exists a coloring with two properties: - Every rectangle with $n$ cells and shape $2^{m} \\times 2^{k-m}$ contains an odd number of red cells. - Every rectangle with $n$ cells and a different shape contains an even number of red cells. ![](https:\/\/cdn.mathpix.com\/cropped\/2024_11_19_1cf0843c143ab4f13b3fg-13.jpg?height=395&width=401&top_left_y=2084&top_left_x=833) A $2^{m} \\times 2^{k-m}$ rectangle contains every possible pair $\\left(x \\bmod 2^{m}, y \\bmod 2^{k-m}\\right)$ exactly once, so such a rectangle will contain one red cell (an odd number). On the other hand, consider a $2^{\\ell} \\times 2^{k-\\ell}$ rectangle with $\\ell>m$. The set of cells this covers is $(x, y)$ where $x$ covers a range of size $2^{\\ell}$ and $y$ covers a range of size $2^{k-\\ell}$. The number of red cells is the count of $x$ with $x \\equiv 0 \\bmod 2^{m}$ multiplied by the count of $y$ with $y \\equiv 0 \\bmod 2^{k-m}$. The former number is exactly $2^{\\ell-k}$ because $2^{k}$ divides $2^{\\ell}$ (while the latter is 0 or 1) so the number of red cells is even. The $\\ell2 \\\\ 2\\left(g(x)+g\\left(x+p^{d}\\right)+\\cdots+g\\left(x+(b-1) p^{d}\\right)\\right) & p=2\\end{cases} \\\\ \\equiv & 0 \\quad(\\bmod p) \\end{aligned} $$ as desired. ## Corollary Let $g: \\mathbb{Z} \/ a \\mathbb{Z} \\rightarrow \\mathbb{Z} \/ p^{e} \\mathbb{Z}$ be any function, and let $h=\\Delta^{e p^{d}} g$. Then $$ h(x)+h\\left(x+p^{d}\\right)+\\cdots+h\\left(x+(b-1) p^{d}\\right)=0 $$ for all $x$. $$ h_{1}(x)=\\frac{h(x)+h\\left(x+p^{d}\\right)+\\cdots+h\\left(x+(b-1) p^{d}\\right)}{p} . $$ Applying the lemma to $h_{1}$ shows the corollary for $e=2$, since $h_{1}(x)$ is divisible by $p$, hence the numerator is divisible by $p^{2}$. Continue in this manner to get the result for general $e>2$. This immediately settles this direction, since $f$ is in the image of $\\Delta^{e p^{d}}$. We will show that $\\Delta$ is injective on $\\mathcal{S}$. Suppose otherwise, and consider two functions $f$, $g$ in $\\mathcal{S}$ with $\\Delta f=\\Delta g$. Then, we obtain that $f$ and $g$ differ by a constant; say $g=f+\\lambda$. However, then $$ \\begin{aligned} & g(0)+g\\left(p^{e}\\right)+\\cdots+g\\left((b-1) p^{e}\\right) \\\\ = & (f(0)+\\lambda)+\\left(f\\left(p^{e}\\right)+\\lambda\\right)+\\cdots+\\left(f\\left((b-1) p^{e}\\right)+\\lambda\\right) \\\\ = & b \\lambda \\end{aligned} $$ This should also be zero. Since $p \\nmid b$, we obtain $\\lambda=0$, as desired. Counting Finally, we can count the essential functions: all but the last $p^{d}$ entries can be chosen arbitrarily, and then each remaining entry has exactly one possible choice. This leads to a count of $$ \\left(p^{e}\\right)^{a-p^{d}}=p^{e\\left(a-p^{\\nu_{p}(a)}\\right)}, $$ as promised."} +{"year":2023,"label":"9","problem":"Let $p$ be a fixed prime and let $a \\geq 2$ and $e \\geq 1$ be fixed integers. Given a function $f: \\mathbb{Z} \/ a \\mathbb{Z} \\rightarrow \\mathbb{Z} \/ p^{e} \\mathbb{Z}$ and an integer $k \\geq 0$, the $k$ th finite difference, denoted $\\Delta^{k} f$, is the function from $\\mathbb{Z} \/ a \\mathbb{Z}$ to $\\mathbb{Z} \/ p^{e} \\mathbb{Z}$ defined recursively by $$ \\begin{aligned} & \\Delta^{0} f(n)=f(n) \\\\ & \\Delta^{k} f(n)=\\Delta^{k-1} f(n+1)-\\Delta^{k-1} f(n) \\quad \\text { for } k=1,2, \\ldots \\end{aligned} $$ Determine the number of functions $f$ such that there exists some $k \\geq 1$ for which $\\Delta^{k} f=f$ \u3002","solution":"Let $p$ be a fixed prime and let $a \\geq 2$ and $e \\geq 1$ be fixed integers. Given a function $f: \\mathbb{Z} \/ a \\mathbb{Z} \\rightarrow \\mathbb{Z} \/ p^{e} \\mathbb{Z}$ and an integer $k \\geq 0$, the $k$ th finite difference, denoted $\\Delta^{k} f$, is the function from $\\mathbb{Z} \/ a \\mathbb{Z}$ to $\\mathbb{Z} \/ p^{e} \\mathbb{Z}$ defined recursively by $$ \\begin{aligned} & \\Delta^{0} f(n)=f(n) \\\\ & \\Delta^{k} f(n)=\\Delta^{k-1} f(n+1)-\\Delta^{k-1} f(n) \\quad \\text { for } k=1,2, \\ldots \\end{aligned} $$ Determine the number of functions $f$ such that there exists some $k \\geq 1$ for which $\\Delta^{k} f=f$. The answer is $$ \\left(p^{e}\\right)^{a} \\cdot p^{-e p^{\\nu} p(a)}=p^{e\\left(a-p^{\\nu_{p}(a)}\\right)} $$ II Second solution by Daniel Zhu. There are two parts to the proof: solving the $e=1$ case, and using the $e=1$ result to solve the general problem by induction on $e$. These parts are independent of each other. The case $e=1 \\quad$ Represent functions $f$ as elements $$ \\alpha_{f}:=\\sum_{k \\in \\mathbb{Z} \/ a \\mathbb{Z}} f(-k) x^{k} \\in \\mathbb{F}_{p}[x] \/\\left(x^{a}-1\\right) $$ Then, since $\\alpha_{\\Delta f}=(x-1) \\alpha_{f}$, we wish to find the number of $\\alpha \\in \\mathbb{F}_{p}[x] \/\\left(x^{a}-1\\right)$ such that $(x-1)^{m} \\alpha=\\alpha$ for some $m$. Now, make the substitution $y=x-1$ and let $P(y)=(y+1)^{a}-1$; we want to find $\\alpha \\in \\mathbb{F}_{p}[y] \/(P(y))$ such that $y^{m} \\alpha=\\alpha$ for some $m$. If we write $P(y)=y^{d} Q(y)$ with $Q(0) \\neq 0$, then by the Chinese Remainder Theorem we have the ring isomorphism $$ \\mathbb{F}_{p}[y] \/(P(y)) \\cong \\mathbb{F}_{p}[y] \/\\left(y^{d}\\right) \\times \\mathbb{F}_{p}[y] \/(Q(y)) $$ Note that $y$ is nilpotent in the first factor, while it is a unit in the second factor. So the $\\alpha$ that work are exactly those that are zero in the first factor; thus there are $p^{a-d}$ such $\\alpha$. We can calculate $d=p^{v_{p}(a)}$ (via, say, Lucas's Theorem), so we are done. The general problem The general idea is as follows: call a $f: \\mathbb{Z} \/ a \\mathbb{Z} \\rightarrow \\mathbb{Z} \/ p^{e} \\mathbb{Z} e$-good if $\\Delta^{m} f=f$ for some $m$. Our result above allows us to count the 1-good functions. Then, if $e \\geq 1$, every $(e+1)$-good function, when reduced $\\bmod p^{e}$, yields an $e$-good function, so we count $(e+1)$-good functions by counting how many reduce to any given $e$-good function. Formally, we use induction on $e$, with the $e=1$ case being treated above. Suppose now we have solved the problem for a given $e \\geq 1$, and we now wish to solve it for $e+1$. For any function $g: \\mathbb{Z} \/ a \\mathbb{Z} \\rightarrow \\mathbb{Z} \/ p^{e+1} \\mathbb{Z}$, let $\\bar{g}: \\mathbb{Z} \/ a \\mathbb{Z} \\rightarrow \\mathbb{Z} \/ p^{e} \\mathbb{Z}$ be its reduction $\\bmod p^{e}$. For a given $e$-good $f$, let $n(f)$ be the number of $(e+1)$-good $g$ with $\\bar{g}=f$. The following two claims now finish the problem: Claim - If $f$ is $e$-good, then $n(f)>0$. $$ g, \\Delta^{m} g, \\Delta^{2 m} g, \\ldots $$ Since there are finitely many functions $\\mathbb{Z} \/ a \\mathbb{Z} \\rightarrow \\mathbb{Z} \/ p^{e+1} \\mathbb{Z}$, there must exist $a0$, then $n(f)$ is exactly the number of 1-good functions, i.e. $p^{a-p^{v_{p}(a)}}$. To show that this condition is sufficient, note that $\\overline{g+p^{e} h}=\\bar{g}=f$. Moreover, if $\\Delta^{m} g=g$ and $\\Delta^{m^{\\prime}} h=h$, then $$ \\Delta^{m m^{\\prime}}\\left(g+p^{e} h\\right)=\\Delta^{m m^{\\prime}} g+p^{e} \\Delta^{m m^{\\prime}} h=g+p^{e} h . $$ To show that this condition is necessary, let $g_{1}$ be any $(e+1)$-good function such that $\\bar{g}_{1}=f$. Then $g_{1}-g$ is also $(e+1)$-good, since if $\\Delta^{m} g=g, \\Delta^{m^{\\prime}} g_{1}=g_{1}$, we have $$ \\Delta^{m m^{\\prime}}\\left(g_{1}-g\\right)=\\Delta^{m m^{\\prime}} g_{1}-\\Delta^{m m^{\\prime}} g=g_{1}-g . $$ On the other hand, we also know that $g_{1}-g$ is divisible by $p^{e}$. This means that it must be $p^{e} h$ for some function $f: \\mathbb{Z} \/ a \\mathbb{Z} \\rightarrow \\mathbb{Z} \/ p \\mathbb{Z}$, and it is not hard to show that $g_{1}-g$ being $(e+1)$-good means that $h$ is 1 -good."}